id
string
paper_link
string
title
string
full_text
string
2412.02046v1
http://arxiv.org/abs/2412.02046v1
Optimal Runge approximation for damped nonlocal wave equations and simultaneous determination results
\documentclass[a4paper, 10pt, twoside, notitlepage]{amsart} \usepackage{amsmath,amscd} \usepackage{amssymb} \usepackage{amsthm} \usepackage{comment} \usepackage{graphicx, xcolor} \usepackage{mathrsfs} \usepackage[linktocpage,ocgcolorlinks, linkcolor=blue]{hyperref} \usepackage{bm} \usepackage{bbm} \usepackage{url} \usepackage[utf8]{inputenc} \usepackage{mathtools,amssymb} \usepackage{esint} \usepackage{tikz} \usepackage{dsfont} \usepackage{relsize} \usepackage{url} \urlstyle{same} \usepackage{xcolor} \usepackage{graphicx} \usepackage{mathrsfs} \usepackage[shortlabels]{enumitem} \usepackage{lineno} \usepackage{amsmath} \usepackage{enumitem} \usepackage{amsthm} \usepackage{verbatim} \usepackage{dsfont} \numberwithin{equation}{section} \renewcommand{\thefigure}{\thesection.\arabic{figure}} \DeclarePairedDelimiter\ceil{\lceil}{\rceil} \DeclarePairedDelimiter\floor{\lfloor}{\rfloor} \allowdisplaybreaks \newcommand{\para}[1]{\vspace{3mm} \noindent\textbf{#1.}} \mathtoolsset{showonlyrefs} \graphicspath{{images/}} \newtheorem{theorem}{Theorem}[section] \newtheorem{lemma}[theorem]{Lemma} \newtheorem{definition}[theorem]{Definition} \newtheorem{corollary}[theorem]{Corollary} \newtheorem{conjecture}[theorem]{Conjecture} \newtheorem{claim}[theorem]{Claim} \newtheorem{proposition}[theorem]{Proposition} \newtheorem{example}[theorem]{Example} \newtheorem{problem}[theorem]{Problem} \newtheorem{question}{Question} \newtheorem{remark}[theorem]{Remark} \newtheorem{assumption}{Assumption} \newtheorem{observation}{Observation} \newtheorem*{centertext}{} \title[Inverse problems for damped nonlocal wave equations]{Optimal Runge approximation for damped nonlocal wave equations and simultaneous determination results} \author[P. Zimmermann]{Philipp Zimmermann} \address{Departament de Matem\`atiques i Inform\`atica, Universitat de Barcelona, Barcelona, Spain} \email{[email protected]} \newcommand{\todo}[1]{\footnote{TODO: #1}} \newcommand{\C}{{\mathbb C}} \newcommand{\R}{{\mathbb R}} \newcommand{\Z}{{\mathbb Z}} \newcommand{\N}{{\mathbb N}} \newcommand{\Q}{{\mathbb Q}} \newcommand{\A}{{\mathcal A}} \newcommand{\Order}{{\mathcal O}} \newcommand{\order}{o} \newcommand{\eps}{\varepsilon} \newcommand{\der}{{\mathrm d}} \newcommand{\id}{\mathrm{Id}} \newcommand {\p} {\partial} \newcommand{\LC}{\left(} \newcommand{\RC}{\right)} \newcommand{\wt}{\widetilde} \newcommand{\Kelvin}{K}\newcommand{\riesz}{I_{\alpha}}\newcommand{\xrt}{X}\newcommand{\dplane}{R_d} \newcommand{\no}{N}\newcommand{\nod}{N_d} \newcommand{\schwartz}{\mathscr{S}} \newcommand{\cschwartz}{\mathscr{S}_0} \newcommand{\tempered}{\mathscr{S}^{\prime}} \newcommand{\rapidly}{\mathscr{O}_C^{\prime}} \newcommand{\slowly}{\mathscr{O}_M} \newcommand{\fraclaplace}{(-\Delta)^s} \newcommand{\fourier}{\mathcal{F}} \newcommand{\ifourier}{\mathcal{F}^{-1}} \newcommand{\vev}[1]{\left\langle#1\right\rangle} \newcommand{\pol}{\mathcal{O}_M} \newcommand{\borel}{\mathcal{M}} \newcommand{\Hcirc}{\overset{\hspace{-0.08cm}\circ}{H^s}} \newcommand{\test}{\mathscr{D}}\newcommand{\smooth}{\mathscr{E}}\newcommand{\cdistr}{\mathscr{E}'}\newcommand{\distr}{\mathscr{D}^{\prime}}\newcommand{\dimens}{n}\newcommand{\kernel}{h_{\alpha}} \newcommand{\norm}[1]{\lVert #1 \rVert} \newcommand{\abs}[1]{\left\lvert #1 \right\rvert}\newcommand{\aabs}[1]{\left\lVert #1 \right\rVert}\newcommand{\ip}[2]{\left\langle #1,#2 \right\rangle}\newcommand{\im}{\mathsf{i}} \DeclareMathOperator{\spt}{spt}\DeclareMathOperator{\ch}{ch}\DeclareMathOperator{\Div}{div} \DeclareMathOperator{\supp}{supp} \DeclareMathOperator{\dist}{dist} \DeclareMathOperator{\loc}{loc} \newcommand{\radon}{\mathscr{M}}\newcommand{\weak}{\rightharpoonup}\newcommand{\weakstar}{\overset{\ast}{\rightharpoonup}} \newcommand{\Vareps}{\boldsymbol{\varepsilon}} \begin{document} \maketitle \begin{abstract} The main purpose of this article is to establish new uniqueness results for Calder\'on type inverse problems related to damped nonlocal wave equations. To achieve this goal we extend the theory of very weak solutions to our setting, which allows to deduce an optimal Runge approximation theorem. With this result at our disposal, we can prove simultaneous determination results in the linear and semilinear regime. \medskip \noindent{\bf Keywords.} Fractional Laplacian, wave equations, nonlinear PDEs, inverse problems, Runge approximation, very weak solutions. \noindent{\bf Mathematics Subject Classification (2020)}: Primary 35R30; secondary 26A33, 42B37 \end{abstract} \tableofcontents \section{Introduction} \label{sec: introduction} In recent years, inverse problems for nonlocal partial differential equations (PDEs) of elliptic, parabolic and hyperbolic type have been studied. This line of research was initiated by Ghosh, Salo and Uhlman \cite{GSU20}, in which they have considered the (partial data) Calder\'on problem related to the \emph{fractional Schr\"odinger equation} \begin{equation} \label{eq: fractional Schroedinger equation} \begin{cases} ((-\Delta)^s+q)u=0&\text{ in }\Omega,\\ u =\varphi & \text{ in } \Omega_e, \end{cases} \end{equation} where $\Omega\subset\R^n$ is a bounded domain, $\Omega_e=\R^n\setminus \overline{\Omega}$, $0<s<1$, $q$ is a suitable potential and $(-\Delta)^s$ is the \emph{fractional Laplacian} which is the operator with Fourier symbol $|\xi|^{2s}$. In this problem one asks whether the knowledge of the \emph{(partial) Dirichlet to Neumann (DN) map} \begin{equation} \label{eq: partial DN map Schroeding} \Lambda_q \varphi= (-\Delta)^s u_\varphi|_{W_2}, \quad \varphi\in C_c^{\infty}(W_1), \end{equation} where $W_1,W_2\subset\Omega_e$ are given measurement sets (i.e.~nonempty open sets) and $u_\varphi$ denotes the unique solution to \eqref{eq: fractional Schroedinger equation}, uniquely determines the potential $q$. The overall strategy to establish unique determination results for the above Calder\'on problem is as follows (see \cite{GSU20,RS17,RZ-unbounded}): \begin{enumerate}[(i)] \item\label{item: integral identity} \emph{Integral identity:} Assume that the potentials $q_j$ are suitably regular, then one can write \begin{equation} \label{eq: integral identity schroeding} \langle (\Lambda_{q_1}-\Lambda_{q_2})\varphi_1,\varphi_2\rangle=\int_{\Omega} (q_1-q_2)(u_{\varphi_1}-\varphi_1),(u_{\varphi_2}-\varphi_2)\,dx, \end{equation} when the right hand side is interpreted accordingly. \item\label{item: Runge approx} Establish one of the following \emph{Runge approximation theorems}: \begin{enumerate}[(I)] \item\label{L2 Runge} $\mathcal{R}_W =\{u_f|_{\Omega}\,;\,f\in C_c^{\infty}(W)\}$ is dense in $L^2(\Omega)$ (see \cite{GSU20} for $q\in L^{\infty}(\Omega)$). \item\label{Hs Runge} $\mathscr{R}_W =\{u_f-f\,;\,f\in C_c^{\infty}(W)\}$ is dense in $\widetilde{H}^s(\Omega)$ (see \cite{RS17} for Sobolev multipliers $q$ or \cite{RZ-unbounded} for local, bounded bilinear forms). \end{enumerate} \item\label{item: Conclusion} If the potentials $q_j$ for $j=1,2$ have suitable continuity properties, then $\Lambda_{q_1}=\Lambda_{q_2}$ together with \ref{item: Runge approx} ensure that there holds $q_1=q_2$ in $\Omega$. \end{enumerate} In \ref{Hs Runge}, the space $\widetilde{H}^s(\Omega)$ is the closure of $C_c^{\infty}(\Omega)$ in the energy space \[ H^s(\R^n)=\{u\in\tempered(\R^n)\,;\,\|u\|_{H^s(\R^n)}\vcentcolon = \|\langle D\rangle^s u\|_{L^2(\R^n)}<\infty\}, \] where $\langle D\rangle^s$ is the Bessel potential operator. Observe the similarity of the above strategy to the one of \cite{SU87} for showing unique determination for the classical Calder\'on problem, where instead of the Runge approximation theorem suitable geometric optics solutions are used. Moreover, the Runge approximation \ref{item: Runge approx} relies on a Hahn--Banach argument and the \emph{unique continuation property (UCP)} of the fractional Laplacian $(-\Delta)^s$. For more results on Calder\'on problems for elliptic nonlocal PDEs, we refer the interested reader to \cite{GLX,cekic2020calderon,CLL2017simultaneously,LL2020inverse,LL2022inverse, LZ2023unique,KLZ-2022,KLW2022,LRZ2022calder,Semilinear-nonlocal-wave-paper,LLU2023calder,CGRU2023reduction,LLU2023calder,RZ-unbounded,RZ2022LowReg,CRTZ-2022,LZ2024uniqueness,feizmohammadi2021fractional,feizmohammadi2021fractional_closed,FKU24,Trans-anisotropic-LNZ} and the references therein. \subsection{Mathematical model and main results} \label{subsec: mathematical model and main results} Recently, the above approach for solving elliptic nonlocal inverse problems has also been adapted to deduce uniqueness results for the Calder\'on problem of nonlocal hyperbolic equations. Let us next describe some of these results in more detail and for this purpose consider the problem \begin{equation} \label{eq: discussion existing results} \begin{cases} \partial_t^2u+\lambda (-\Delta)^s \partial_t u+(-\Delta)^s u +f(u)= 0 & \text{ in } \Omega_T,\\ u =\varphi & \text{ on } (\Omega_e)_T,\\ u(0) = 0,\, \partial_{t}u(0) = 0 & \text{ on } \Omega, \end{cases} \end{equation} where $\lambda\in\R$ and $f\colon \Omega\times\R\to\R$ is a possibly nonlinear function. If the problem \eqref{eq: discussion existing results} is well-posed in the energy class $H^s(\R^n)$, then for any two given measurement sets $W_1,W_2\subset\Omega_e$ we may introduce the DN map $\Lambda^{\lambda}_{f}$ via \[ \Lambda^{\lambda}_{f}\varphi=\left.(\lambda (-\Delta)^s\partial_t u_\varphi+(-\Delta)^s u_\varphi)\right|_{(W_2)_T}, \] whenever $\varphi$ is supported in $(W_1)_T$ and $u_\varphi$ is the solution of \eqref{eq: discussion existing results}. The \emph{Calder\'on problem} for \eqref{eq: discussion existing results} reads as follows: \begin{question} \label{question: calderon discussion} Does the DN map $\Lambda^{\lambda}_{f}$ uniquely determine the function $f$? \end{question} A suitable class of nonlinearities are the so-called weak nonlinearities, which are defined next. \begin{definition}\label{main assumptions on nonlinearities} We call a Carath\'eodory function $f\colon \Omega\times \R\to\R$ \emph{weak nonlinearity}, if it satisfies the following conditions: \begin{enumerate}[(i)] \item\label{prop f} $f$ has partial derivative $\partial_{\tau}f$, which is a Carath\'eodory function, and there exists $a\in L^p(\Omega)$ such that \begin{equation} \label{eq: bound on derivative} \left|\partial_\tau f(x,\tau)\right|\lesssim a(x)+|\tau|^r \end{equation} for all $\tau\in\R$ and a.e. $x\in\Omega$. Here the exponents $p$ and $r$ satisfy the restrictions \begin{equation} \label{eq: restrictions on p} \begin{cases} n/s\leq p\leq \infty, &\, \text{if }\, 2s< n,\\ 2<p\leq \infty, &\, \text{if }\, 2s= n,\\ 2\leq p\leq \infty, &\, \text{if }\, 2s\geq n. \end{cases} \end{equation} and \begin{equation} \label{eq: cond on r} \begin{cases} 0\leq r<\infty, &\, \text{if }\, 2s\geq n,\\ 0\leq r\leq \frac{2s}{n-2s}, &\, \text{if }\, 2s< n, \end{cases} \end{equation} respectively. Moreover, $f$ fulfills the integrability condition $f(\cdot,0)\in L^2(\Omega)$. \item\label{prop F} The function $F\colon \Omega\times\R\to\R$ defined via \[ F(x,\tau)=\int_0^\tau f(x,\rho)\,d\rho \] satisfies $F(x,\tau)\geq 0$ for all $\tau\in\R$ and $x\in\Omega$. \end{enumerate} \end{definition} Let us note that $f(x,\tau)=q(x)|\tau|^r\tau$ with $r$ satisfying \eqref{eq: cond on r} and $0\leq q\in L^{\infty}(\Omega)$ is a weak nonlinearity. Using the above notions, we can now discuss some of the existing results. \begin{enumerate}[(a)] \item\label{item: basic nonlocal wave} The article \cite{KLW2022} gives a positive answer for $\lambda=0, f(x,\tau)=q(x)\tau$ with $q\in L^{\infty}(\Omega)$. Their proof relied on the observation that the related nonlocal wave equation \eqref{eq: discussion existing results} satisfies an $L^2(\Omega_T)$ Runge approximation theorem. \item\label{item: viscous nonlocal wave} The work \cite{zimmermann2024calderon} deals on the one hand with the linear case $\lambda=1$, $f(x,\tau)=q(x)\tau$ with $q\in L^{\infty}(0,T;L^p(\Omega))$, where $p$ satisfies the restrictions \eqref{eq: restrictions on p}, and $q$ is weakly continuous in $t$ and on the other hand with the nonlinear case $\lambda=1$ and $f$ is a $r+1$ homogeneous, weak nonlinearity. The uniqueness proofs use substantially that due to presence of the viscosity term $(-\Delta)^s\partial_t$ solutions $u$ to \eqref{eq: discussion existing results} satisfy $\partial_t u\in L^2(0,T;H^s(\R^n))$ and as a consequence the linearized equations have the Runge approximation property in $L^2(0,T;\widetilde{H}^s(\Omega))$. \item\label{item: semilinear nonlocal wave} In \cite{Semilinear-nonlocal-wave-eq}, uniqueness is proved in the case $\lambda=0$ and $f$ satisfies the same properties as in \ref{item: viscous nonlocal wave}, but with the additional restriction $r\leq 1$. This article only uses an $L^2(\Omega_T)$ Runge approximation result for the linearized nonlocal wave equation. \item\label{item: optimal runge} By establishing a theory for very weak solutions of linear nonlocal wave equations with $\lambda=0$, the authors of \cite{Optimal-Runge-nonlocal-wave} could deduce an optimal $L^2(0,T;\widetilde{H}^s(\Omega))$ Runge approximation theorem for these equations. This allowed to extend the results in \ref{item: semilinear nonlocal wave} to the cases $r>1$ and additionally showed that one can recover any linear perturbation $q\in L^p(\Omega)$ with $p$ satisfying the restrictions \eqref{eq: restrictions on p}. Furthermore, by this improved Runge approximation theorem the authors could also treat the case of serially or asymptotically polyhomogeneous nonlinearities (see \cite[Theorem 1.5]{Optimal-Runge-nonlocal-wave}). \end{enumerate} In this context, let us also mention the recent article \cite{fu2024wellposednessinverseproblemsnonlocal} which deals with the Calder\'on problem for a third order semilinear, nonlocal, viscous wave equation. The goal of this paper us to present an extension of the models described in \ref{item: viscous nonlocal wave} and \ref{item: semilinear nonlocal wave}, which we discuss next. Let $0<s<1$ and suppose that we have given coefficients $(\gamma,q)\in C^{0,\alpha}(\R^n)\times L^p(\Omega)$, where $0<s<\alpha\leq 1$ and $1\leq p\leq \infty$ satisfies the restrictions in \eqref{eq: restrictions on p}. Then we define the following \emph{damped, nonlocal wave operator} \begin{equation} \label{eq: damped, nonlocal wave operator} L_{\gamma}\vcentcolon = \partial_t^2+\gamma\partial_t +(-\Delta)^s \end{equation} and consider the problem \begin{equation} \label{eq: wave problem} \begin{cases} L_{\gamma}u +f(u)= F & \text{ in } \Omega_T,\\ u =\varphi & \text{ on } (\Omega_e)_T,\\ u(0) = u_0,\, \partial_{t}u(0) = u_1 & \text{ on } \Omega, \end{cases} \end{equation} where $f$ is a weak nonlinearity or $f(u)=q(x)u$. In fact, this is a possibly nonlinear generalization of the model (G2) with $s_1=0$ in \cite[Section 1.1]{zimmermann2024calderon}. By \cite[Proposition 3.7]{Semilinear-nonlocal-wave-eq} (see Section \ref{subsec: weak solutions} for the linear case), we know that the problem \eqref{eq: wave problem} is well-posed, whenever the source $F$, exterior condition $\varphi$ and initial conditions $u_0,u_1$ are sufficiently regular. Thus, we can introduce the related (partial) DN map via \begin{equation} \label{eq: formal DN map} \Lambda_{\gamma,f}\varphi\vcentcolon = \left.(-\Delta)^s u_{\varphi}\right|_{(W_2)_T}, \end{equation} where $W_1,W_2\subset \Omega_e$ are some measurement sets, $\varphi$ is supported in $(W_1)_T$ and $u_\varphi$ is the unique solution of \eqref{eq: wave problem} with $u_0=u_1=0$. Then we ask the following question: \begin{question} \label{question: Caldeorn problem of this work} Does the partial DN map $\Lambda_{\gamma,f}$ uniquely determine the damping coefficient $\gamma$ and the function $f$? \end{question} In this work we establish the following affirmative answers to this question, whereas the first result discusses the linear case and the second one the semilinear perturbations. \begin{theorem}[Uniqueness for linear perturbations] \label{thm: uniqueness linear} Let $\Omega \subset\R^n$ be a bounded Lipschitz domain, $T>0$, $0<s<\alpha\leq 1$ and suppose that $1\leq p\leq \infty$ satisfies \eqref{eq: restrictions on p}. Assume that for $j=1,2$ we have given coefficients $(\gamma_j,q_j)\in C^{0,\alpha}(\R^n)\times L^{p}(\Omega)$ and let $\Lambda_{\gamma_j,q_j}$ be the DN map associated to the problem \begin{equation} \label{eq: PDEs uniqueness theorem} \begin{cases} (L_{\gamma_j}+q_j)u = 0 & \text{ in } \Omega_T,\\ u =\varphi & \text{ on } (\Omega_e)_T,\\ u(0) = 0,\, \partial_{t}u(0) = 0 & \text{ on } \Omega \end{cases} \end{equation} for $j=1,2$. If $W_1,W_2\subset\Omega_e$ are two measurement sets such that \begin{equation} \label{eq: equality of DN maps} \left.\Lambda_{\gamma_1,q_1}\varphi\right|_{(W_2)_T}=\left.\Lambda_{\gamma_2,q_2}\varphi\right|_{(W_2)_T} \end{equation} for all $\varphi\in C_c^{\infty}((W_1)_T)$, then there holds \begin{equation} \label{eq: equal coefficients} \gamma_1=\gamma_2\text{ and }q_1=q_2 \text{ in }\Omega. \end{equation} \end{theorem} \begin{theorem}[Uniqueness for semilinear perturbations] \label{thm: uniqueness semilinear} Let $\Omega \subset\R^n$ be a bounded Lipschitz domain, $T>0$ and $0<s<\alpha\leq 1$. Assume that for $j=1,2$ we have given coefficients $\gamma_j\in C^{0,\alpha}(\R^n)$ and $r+1$ homogeneous, weak nonlinearities $f_j$, where $r>0$ satisfies \eqref{eq: cond on r}. Let $\Lambda_{\gamma_j,f_j}$ be the DN map associated to the problem \begin{equation} \label{eq: PDEs uniqueness theorem} \begin{cases} L_{\gamma_j}u +f_j(u) = 0 & \text{ in } \Omega_T,\\ u =\varphi & \text{ on } (\Omega_e)_T,\\ u(0) = 0,\, \partial_{t}u(0) = 0 & \text{ on } \Omega \end{cases} \end{equation} for $j=1,2$. If $W_1,W_2\subset\Omega_e$ are two measurement sets such that \begin{equation} \label{eq: equality of DN maps semilinear} \left.\Lambda_{\gamma_1,f_1}\varphi\right|_{(W_2)_T}=\left.\Lambda_{\gamma_2,f_2}\varphi\right|_{(W_2)_T} \end{equation} for all $\varphi\in C_c^{\infty}((W_1)_T)$, then there holds \begin{equation} \label{eq: equal coefficients} \gamma_1=\gamma_2\text{ in }\Omega\text{ and }f_1=f_2 \text{ in }\Omega\times \R. \end{equation} \end{theorem} \begin{remark} For simplicity we restrict our attention to homogeneous nonlinearities $f$, but the unique determination remains valid in some polyhomogeneous cases as described in \cite{Optimal-Runge-nonlocal-wave} for $\gamma=0$. \end{remark} \subsection{Organization of the article} The rest of this article is structured as follows. In Section \ref{sec: Very weak solutions to damped, nonlocal wave equations}, we establish the existence of unique weak and very weak solutions to damped nonlocal wave equations. In Section \ref{sec: inverse problem} we then move on to the inverse problem part of this work. First, in Section \ref{sec: Runge approx} we establish the optimal Runge approximation theorem. Afterwards, in Section \ref{sec: integral identity} we prove a suitable integral identity that allows us to recover simultaneously the damping coefficient $\gamma$ and potential $q$ in Section \ref{sec: linear uniqueness}. Finally, Section \ref{sec: semilinear uniqueness} contains the proof of the simultaneous determination of the damping coefficient $\gamma$ and the homogeneous nonlinearity $f$. \section{Weak and very weak solutions to damped, nonlocal wave equations} \label{sec: Very weak solutions to damped, nonlocal wave equations} The main purpose of this section is to show existence of unique weak and very weak solutions to \emph{damped, nonlocal wave equations (DNWEQ)} \begin{equation} \label{eq: damped nonlocal wave equations well-posedness} \begin{cases} (L_{\gamma}+q)u = F & \text{ in } \Omega_T,\\ u =\varphi & \text{ on } (\Omega_e)_T,\\ u(0) = u_0,\, \partial_{t}u(0) = u_1 & \text{ on } \Omega, \end{cases} \end{equation} where $L_\gamma$ is given by \eqref{eq: damped, nonlocal wave operator} and only the case $\varphi=0$ is considered for very weak solutions. \subsection{Weak solutions} \label{subsec: weak solutions} This section deals with the well-posedness of \eqref{eq: damped nonlocal wave equations well-posedness} for regular sources, exterior conditions and initial data. We also prove well-posedness for the case when instead of initial values the values at $t=T$ are specified, which will be needed for the development of the theory of very weak solutions. \begin{theorem}[Weak solutions to homogeneous DNWEQ] \label{thm: weak sol to hom DNWEQ} Let $\Omega \subset\R^n$ be a bounded Lipschitz domain, $T>0$, $0<s< 1$ and suppose that $1\leq p\leq \infty$ satisfies \eqref{eq: restrictions on p}. Assume that we have given coefficients $(\gamma,q)\in L^{\infty}(\Omega)\times L^{p}(\Omega)$. Then for any $F\in L^2(0,T;\widetilde{L}^2(\Omega))$\footnote{Here and below we set $\widetilde{L}^2(\Omega)\vcentcolon =\widetilde{H}^0(\Omega)$.} and initial conditions $(u_0,u_1)\in \widetilde{H}^s(\Omega)\times\widetilde{L}^2(\Omega)$, there exists a unique weak solution $u\in C([0,T];\widetilde{H}^s(\Omega))\cap C^1([0,T];\widetilde{L}^2(\Omega))$ of \begin{equation} \label{eq: damped nonlocal wave equations well-posedness weak sol} \begin{cases} (L_{\gamma}+q)u = F & \text{ in } \Omega_T,\\ u =0 & \text{ on } (\Omega_e)_T,\\ u(0) = u_0,\, \partial_{t}u(0) = u_1 & \text{ on } \Omega, \end{cases} \end{equation} which means that $(u(0),\partial_tu(0))=(u_0,u_1)$ in $\widetilde{H}^s(\Omega)\times \widetilde{L}^2(\Omega)$ and there holds \begin{equation} \label{eq: weak formulation damped nonlocal wave eq} \begin{split} &\frac{d}{dt} \langle \partial_t u,v\rangle_{L^2(\Omega)}+\langle \gamma\partial_t u,v\rangle_{L^2(\Omega)}+\langle (-\Delta)^{s/2}u,(-\Delta)^{s/2}v\rangle_{L^2(\R^n)}+\langle qu,v\rangle_{L^2(\Omega)}\\ &=\langle F,v\rangle_{L^2(\Omega)} \end{split} \end{equation} for all $v\in \widetilde{H}^s(\Omega)$ in the sense of distributions on $(0,T)$. Moreover, the unique solution $u$ obeys the energy identity \begin{equation} \label{eq: energy identity} \begin{split} &\|\partial_t u(t)\|_{L^2(\Omega)}^2+\|(-\Delta)^{s/2}u(t)\|_{L^2(\R^n)}^2+2 \int_0^t \langle \gamma\partial_t u(\tau)+qu(\tau),\partial_t u(\tau)\rangle_{L^2(\Omega)}\,d\tau\\ &=\|(-\Delta)^{s/2}u_0\|_{L^2(\R^n)}^2+\|u_1\|_{L^2(\Omega)}^2+2 \int_0^t\langle F(\tau),\partial_tu (\tau)\rangle_{L^2(\Omega)}\,d\tau \end{split} \end{equation} which implies \begin{equation} \label{eq: energy estimate} \begin{split} &\|\partial_t u(t)\|_{L^2(\Omega)}+\|(-\Delta)^{s/2}u(t)\|_{L^2(\R^n)}\\ &\leq C(\|u_1\|_{L^2(\Omega)}+\|(-\Delta)^{s/2}u_0\|_{L^2(\R^n)}+\|F\|_{L^2(0,t;L^2(\Omega))}) \end{split} \end{equation} for all $0\leq t\leq T$ and some $C>0$ only depending on $\|q\|_{L^p(\Omega)}$, $\|\gamma\|_{L^{{\infty}(\Omega)}}$ and $T>0$. \end{theorem} \begin{proof} Throughout the proof, we endow $\widetilde{H}^s(\Omega)$ with the equivalent norm $\|u\|_{\widetilde{H}^s(\Omega)}=\|(-\Delta)^{s/2}u\|_{L^2(\R^n)}$ (see \cite[Lemma 2.3]{Semilinear-nonlocal-wave-eq}) and we introduce the following continuous sesquilinear forms \begin{equation} \label{eq: sesquilinear forms} a_0(u,v)=\langle (-\Delta)^{s/2}u,(-\Delta)^{s/2}v\rangle_{L^2(\R^n)},\quad a_1(u,v)= \langle qu,v\rangle_{L^2(\Omega)} \end{equation} for $u,v\in \widetilde{H}^s(\Omega)$ and \begin{equation} b(u,v)=\langle \gamma u,v\rangle_{L^2(\Omega)} \end{equation} for $u,v\in \widetilde{L}^2(\Omega)$. Next, recall that by \cite[eq.~(3.7)]{Semilinear-nonlocal-wave-eq} one has \begin{equation} \label{eq: L2 estimate potential} \|qu\|_{L^2(\Omega)}\leq C\|q\|_{L^p(\Omega)}\|u\|_{\widetilde{H}^s(\Omega)} \end{equation} for all $u\in \widetilde{H}^s(\Omega)$. It is not hard to see that we can invoke the existence and uniqueness results \cite[Chapter XVIII, \S 5, Theorem~3 \& 4]{DautrayLionsVol5} (see \cite[p.~571]{DautrayLionsVol5}), which ensure the existence of a unique, real-valued solution $u\in C([0,T];\widetilde{H}^s(\Omega))\cap C^1([0,T];\widetilde{L}^2(\Omega))$ to \eqref{eq: damped nonlocal wave equations well-posedness weak sol}. Furthermore, by \cite[Chapter XVIII, \S 5, Lemma~7]{DautrayLionsVol5} the solution $ u$ satisfies the following energy identity \begin{equation} \label{eq: energy identity proof} \begin{split} &\|\partial_t u(t)\|_{L^2(\Omega)}^2+\|(-\Delta)^{s/2}u(t)\|_{L^2(\R^n)}^2+2 \int_0^t \langle \gamma\partial_t u(\tau)+qu(\tau),\partial_t u(\tau)\rangle_{L^2(\Omega)}\,d\tau\\ &=\|(-\Delta)^{s/2}u_0\|_{L^2(\R^n)}^2+\|u_1\|_{L^2(\Omega)}^2+2 \int_0^t\langle F(\tau),\partial_tu (\tau)\rangle_{L^2(\Omega)}\,d\tau \end{split} \end{equation} for $0\leq t\leq T$. Hence, we have shown the identity \eqref{eq: energy identity}. Let us define $\Psi \in C([0,T])$ by \[ \Psi(t)\vcentcolon = \|\partial_t u(t)\|_{L^2(\Omega)}^2+\|(-\Delta)^{s/2}u(t)\|_{L^2(\R^n)}^2 \] for $0\leq t\leq T$. Using \eqref{eq: L2 estimate potential}, \eqref{eq: energy identity proof} and $\gamma\in L^{\infty}(\Omega)$, we get \[ \begin{split} &\Psi(t)\leq \Psi(0)+\int_0^t \|F(\tau)\|_{L^2(\Omega)}^2\,d\tau+ C\int_0^t (1+\|\gamma\|_{L^{\infty}(\Omega)}+\|q\|^2_{L^p(\Omega)})\Psi(\tau)\,d\tau \end{split} \] and via Gronwall's inequality we deduce the energy estimate \begin{equation} \begin{split} &\Psi(t)\leq C(\Psi(0)+\|F\|_{L^2(0,t;L^2(\Omega))}^2) \end{split} \end{equation} for all $0\leq t\leq T$ and some $C>0$ only depending on $\|q\|_{L^p(\Omega)}$, $\|\gamma\|_{L^{\infty}(\Omega)}$ and $T>0$. This establishes the estimate \eqref{eq: energy estimate}. \end{proof} As a consequence we have the following result: \begin{proposition}[Weak solutions to inhomogeneous DNWEQ] \label{prop: Weak solutions to inhomogeneous DNWEQ} Let $\Omega \subset\R^n$ be a bounded Lipschitz domain, $T>0$, $0<s< 1$ and suppose that $1\leq p\leq \infty$ satisfies \eqref{eq: restrictions on p}. Assume that we have given coefficients $(\gamma,q)\in L^{\infty}(\Omega)\times L^{p}(\Omega)$. Then for any $F\in L^2(0,T;\widetilde{L}^2(\Omega))$, exterior condition $\varphi\in C^2([0,T];H^{2s}(\R^n))$ and initial conditions $(u_0,u_1)\in H^s(\R^n)\times L^2(\R^n)$ satisfying the compatibility conditions $u_0-\varphi(0)\in \widetilde{H}^s(\Omega)$ and $u_1-\partial_t\varphi(0)\in \widetilde{L}^2(\Omega)$, there exists a unique weak solution $u\in C([0,T];H^s(\R^n))\cap C^1([0,T];L^2(\R^n))$ of \begin{equation} \label{eq: damped nonlocal wave equations well-posedness weak sol inhom} \begin{cases} (L_{\gamma}+q)u = F & \text{ in } \Omega_T,\\ u =\varphi & \text{ on } (\Omega_e)_T,\\ u(0) = u_0,\, \partial_{t}u(0) = u_1 & \text{ on } \Omega, \end{cases} \end{equation} which means that $u$ satisfies \eqref{eq: weak formulation damped nonlocal wave eq}, the $(u(0),\partial_t u(0))=(u_0,u_1)$ in $H^s(\R^n)\times L^2(\R^n)$ and $u=\varphi$ in $(\Omega_e)_T$ means that $u(t)=\varphi(t)$ a.e. in $\Omega_e$ for any $0<t<T$. Furthermore, the following energy estimate holds \begin{equation} \label{eq: energy estimate inhomogeneous} \begin{split} &\|\partial_t u(t)\|_{L^2(\R^n)}+\|(-\Delta)^{s/2}u(t)\|_{L^2(\R^n)}\\ &\leq C(\|u_1\|_{L^2(\R^n)}+\|(-\Delta)^{s/2}u_0\|_{L^2(\R^n)}+\|\varphi\|_{C^2([0,t];H^{2s}(\R^n))}+\|F\|_{L^2(0,t;L^2(\Omega))}) \end{split} \end{equation} for any $0\leq t\leq T$. \end{proposition} \begin{proof} Observe, under the current regularity assumptions and compatibility conditions, that $u$ solves \eqref{eq: damped nonlocal wave equations well-posedness weak sol inhom} if and only if $w\vcentcolon = u-\varphi$ solves \begin{equation} \label{eq: Usual cauchy homogeneous} \begin{cases} (L_{\gamma}+q)w = F-(L_\gamma+q) \varphi & \text{ in } \Omega_T,\\ w =0 & \text{ on } (\Omega_e)_T,\\ w(0) = u_0-\varphi(0),\, \partial_{t}w(0) = u_1-\partial_t\varphi(0) & \text{ on } \Omega. \end{cases} \end{equation} The only fact to keep in mind is that if $u\in C([0,T];H^s(\R^n))$, then the condition $u(t)=\varphi(t)$ a.e. in $\Omega_e$ is equivalent to $u(t)-\varphi(t)\in \widetilde{H}^s(\Omega)$ as $\Omega\subset\R^n$ is a bounded Lipschitz domain. So, the assertions of Propsition \ref{prop: Weak solutions to inhomogeneous DNWEQ} follow immediately from Theorem \ref{thm: weak sol to hom DNWEQ}. \end{proof} Next, let us define for any $g\in L^1_{loc}(V_T)$, $V\subset\R^n$ open, its \emph{time-reversal} \begin{equation} \label{eq: time reversal} g^\star(x,t)=g(x,T-t). \end{equation} Then, we have the following lemma. \begin{lemma} \label{lemma: time reversal of solution} Let $\Omega \subset\R^n$ be a bounded Lipschitz domain, $T>0$, $0<s< 1$ and suppose that $1\leq p\leq \infty$ satisfies \eqref{eq: restrictions on p}. Assume that we have given coefficients $(\gamma,q)\in L^{\infty}(\Omega)\times L^{p}(\Omega)$. Let $F\in L^2(0,T;\widetilde{L}^2(\Omega))$, $\varphi\in C^2([0,T];H^{2s}(\R^n))$ and $(u_0,u_1)\in H^s(\R^n)\times L^2(\R^n)$ satisfying the compatibility conditions $u_0-\varphi(0)\in \widetilde{H}^s(\Omega)$ and $u_1-\partial_t\varphi(0)\in \widetilde{L}^2(\Omega)$. Then $u$ solves \begin{equation} \label{eq: Usual cauchy} \begin{cases} (L_{\gamma}+q)u = F & \text{ in } \Omega_T,\\ u =\varphi & \text{ on } (\Omega_e)_T,\\ u(0) = u_0,\, \partial_{t}u(0) = u_1 & \text{ on } \Omega, \end{cases} \end{equation} if and only if $u^\star$ solves \begin{equation} \label{eq: time reversed cauchy} \begin{cases} (L_{-\gamma}+q)v = F^\star & \text{ in } \Omega_T,\\ v =\varphi^\star & \text{ on } (\Omega_e)_T,\\ v(T) = u_0,\, \partial_{t}v(T) = u_1 & \text{ on } \Omega. \end{cases} \end{equation} In particular, for any $F\in L^2(\Omega_T)$, $\varphi\in C^2([0,T];H^{2s}(\R^n))$ and $(u_0,u_1)\in H^s(\R^n)\times L^2(\R^n)$ satisfying the compatibility conditions $u_0-\varphi(T)\in \widetilde{H}^s(\Omega)$ and $u_1-\partial_t\varphi(T)\in \widetilde{L}^2(\Omega)$, there exists a unique solution $u^\star$ of \begin{equation} \label{eq: backwards damped, nonlocal wave equation} \begin{cases} (L_{-\gamma}+q)v = F & \text{ in } \Omega_T,\\ v =\varphi & \text{ on } (\Omega_e)_T,\\ v(T) = u_0,\, \partial_{t}v(T) = u_1 & \text{ on } \Omega. \end{cases} \end{equation} \end{lemma} \begin{proof} First, note that by the proof of Proposition \ref{prop: Weak solutions to inhomogeneous DNWEQ}, we can assume without loss of generality that $\varphi=0$. Secondly, one easily sees that $\partial_t u^\star=-(\partial_t u)^\star$ and thus a change of variables in \eqref{eq: weak formulation damped nonlocal wave eq} gives the asserted equivalence. The unique solvability of \eqref{eq: backwards damped, nonlocal wave equation} follows from the equivalence and Theorem \ref{thm: weak sol to hom DNWEQ}. \end{proof} \subsection{Very weak solutions} Let us start by making some simple observations. Suppose that $u$ and $v$ are smooth solutions of the problems \begin{equation} \label{eq: motivation very weak solutions 1} \begin{cases} (L_\gamma+q) u = F & \text{ in } \Omega_T,\\ u =0 & \text{ on } (\Omega_e)_T,\\ u(0) = u_0,\, \partial_{t}u(0) = u_1 & \text{ on } \Omega, \end{cases} \end{equation} and \begin{equation} \label{eq: motivation very weak solutions 2} \begin{cases} (L_{-\gamma}+q) v = G & \text{ in } \Omega_T,\\ v =0 & \text{ on } (\Omega_e)_T,\\ v(T) = 0,\, \partial_{t}v(T) = 0 & \text{ on } \Omega, \end{cases} \end{equation} respectively. If we multiply the PDE \eqref{eq: motivation very weak solutions 1} by $v$ and integrate over $\Omega_T$, then we get \begin{equation} \label{eq: formal computation} \begin{split} &\int_{\Omega_T}Fv\,dxdt=\int_{\Omega_T}[(L_{\gamma}+q)u]v\,dxdt\\ &= \int_{\Omega}u_0\partial_t v(0)\,dx-\int_{\Omega}u_1v(0)\,dx-\int_\Omega \gamma u_0v(0)\,dx+\int_{\Omega_T}u(L_{-\gamma}+q)v\,dxdt\\ &= \int_{\Omega}u_0\partial_t v(0)\,dx-\int_{\Omega}u_1v(0)\,dx-\int_\Omega \gamma u_0v(0)\,dx+\int_{\Omega_T}Gu\,dxdt. \end{split} \end{equation} Notice that if $G\in L^2(0,T;\widetilde{L}^2(\Omega))$, $v\in L^2(0,T;\widetilde{H}^s(\Omega))$ and $(v(0),\partial_t v(0))\in \widetilde{H}^s(\Omega)\times\widetilde{L}^2(\Omega)$, then one can make sense of the first integral and the last line in \eqref{eq: formal computation}, even in the case $F\in L^2(0,T;H^{-s}(\Omega))$, $(u_0,u_1)\in \widetilde{L}^2(\Omega)\times H^{-s}(\Omega)$ and $u\in L^2(0,T;\widetilde{L}^2(\Omega))$. Here, $H^{-s}(\Omega)\subset\distr(\Omega)$ is defined by \[ H^{-s}(\Omega)=\{u|_\Omega\,;\,u\in H^{-s}(\R^n)\} \] and it can be identified with the dual space of $\widetilde{H}^s(\Omega)$, when $\Omega$ is Lipschitz. The previous computation motivates the following definition. \begin{definition}[Very weak solutions] \label{def: very weak solutions} Let $\Omega \subset\R^n$ be a bounded Lipschitz domain, $T>0$, $0<s< 1$ and suppose that $1\leq p\leq \infty$ satisfies \eqref{eq: restrictions on p}. Assume that we have given coefficients $(\gamma,q)\in L^{\infty}(\Omega)\times L^{p}(\Omega)$, source $F\in L^2(0,T;H^{-s}(\Omega))$ and initial conditions $(u_0,u_1)\in \widetilde{L}^2(\Omega)\times H^{-s}(\Omega)$. Then we say that $u\in C([0,T];\widetilde{L}^2(\Omega))\cap C^1([0,T];H^{-s}(\Omega))$ is a \emph{very weak solution} of \begin{equation} \label{eq: def very weak} \begin{cases} (L_\gamma+q) u = F & \text{ in } \Omega_T,\\ u =0 & \text{ on } (\Omega_e)_T,\\ u(0) = u_0,\, \partial_{t}u(0) = u_1 & \text{ on } \Omega, \end{cases} \end{equation} whenever there holds\footnote{Here and below we sometimes write $\langle \cdot,\cdot\rangle$ to denote the duality pairing between $H^{-s}(\Omega)\times \widetilde{H}^s(\Omega)$.} \begin{equation} \label{eq: weak formulation of very weak sols} \int_0^T \langle G,u\rangle_{L^2(\Omega)}\,dt=\int_0^T\langle F,v\rangle\,dt+\langle u_1,v(0)\rangle-\langle u_0,\partial_t v(0)\rangle_{L^2(\Omega)}+\langle \gamma u_0,v(0)\rangle \end{equation} for all $G\in L^2(0,T;\widetilde{L}^2(\Omega))$, where $v\in C([0,T];\widetilde{H}^s(\Omega))\cap C^1([0,T];\widetilde{L}^2(\Omega))$ is the unique weak solution of the adjoint equation \begin{equation} \label{eq: adjoint eq of def very weak} \begin{cases} (L_{-\gamma}+q) v = G & \text{ in } \Omega_T,\\ v =0 & \text{ on } (\Omega_e)_T,\\ v(T) = 0,\, \partial_{t}v(T) = 0 & \text{ on } \Omega, \end{cases} \end{equation} (see Theorem \ref{thm: weak sol to hom DNWEQ}). \end{definition} Next, let us recall the following well-posedness result of very weak solutions. \begin{theorem}[{Very weak solutions for $\gamma=q=0$,\cite[Theorem 3.6]{Optimal-Runge-nonlocal-wave}}] \label{thm: well-posedness very weak without damping} Let $\Omega \subset\R^n$ be a bounded Lipschitz domain, $T>0$ and $0<s< 1$. Then for any given source $F\in L^2(0,T;H^{-s}(\Omega))$ and initial conditions $(u_0,u_1)\in \widetilde{L}^2(\Omega)\times H^{-s}(\Omega)$, there exists a unique solution to \begin{equation} \label{eq: very weak without damping and potential} \begin{cases} (\partial_t^2+(-\Delta)^s) u = F & \text{ in } \Omega_T,\\ u =0 & \text{ on } (\Omega_e)_T,\\ u(0) = u_0,\, \partial_{t}u(0) = u_1 & \text{ on } \Omega \end{cases} \end{equation} and it satisfies the following energy estimate \begin{equation} \label{eq: energy estimate very weak without damping and potential} \|u(t)\|_{L^2(\Omega)}+\|\partial_t u(t)\|_{H^{-s}(\Omega)}\leq C(\|u_0\|_{L^2(\Omega)}+\|u_1\|_{H^{-s}(\Omega)}+\|F\|_{L^2(0,t;H^{-s}(\Omega))}) \end{equation} for all $0\leq t\leq T$. \end{theorem} Hence, we have a well-defined solution map. \begin{proposition}[Solution map] \label{prop: solution map} Let $\Omega \subset\R^n$ be a bounded Lipschitz domain, $T>0$, $0<s< 1$ and let $X_s\vcentcolon =\widetilde{L}^2(\Omega)\times H^{-s}(\Omega)$ be endowed with the usual product norm \[ \|(u,w)\|_{X_s}\vcentcolon = (\|u\|_{L^2(\Omega)}^2+\|w\|_{H^{-s}(\Omega)}^2)^{1/2}. \] Then the \emph{solution map} $S\colon L^2(0,T;H^{-s}(\Omega))\to C([0,T];X_s)$ defined by \begin{equation} \label{eq: solution map} S(F)\vcentcolon = (u,\partial_t u), \end{equation} where $u\in C([0,T];\widetilde{L}^2(\Omega))\cap C^1([0,T];H^{-s}(\Omega))$ is the unique solution of \eqref{eq: very weak without damping and potential} with $(u_0,u_1)=0$. Moreover, the solution map is continuous and satisfies the estimate \begin{equation} \label{eq: continuity estimate solution map} \|S(F)(t)\|_{X_s}\leq C\|F\|_{L^2(0,t;H^{-s}(\Omega))} \end{equation} for any $0\leq t\leq T$. \end{proposition} \begin{proof} First of all note that the solution map $S$ is well-defined by Theorem \ref{thm: well-posedness very weak without damping}. The estimate \eqref{eq: continuity estimate solution map} follows from \eqref{eq: energy estimate very weak without damping and potential}, which together with the linearity of $S$ gives the continuity of $S$. Observe that the linearity of $S$ is a direct consequence of the unique solvability of \eqref{eq: very weak without damping and potential} and the fact that the PDE is linear. \end{proof} \begin{theorem} \label{thm: general well-posedness result of very weak solutions} Let $\Omega \subset\R^n$ be a bounded Lipschitz domain, $T>0$, $0<s< 1$ and suppose that $\mathcal{F}\colon C([0,T];X_s)\to L^2(0,T;H^{-s}(\Omega))$ satisfies the Lipschitz estimate \begin{equation} \label{eq: Lipschitz estimate nonlinearity} \|\mathcal{F}(U)(t)-\mathcal{F}(V)(t)\|_{H^{-s}(\Omega)}\leq C\|U(t)-V(t)\|_{X_s} \end{equation} for a.e.~$0\leq t\leq T$ and $U,V\in C([0,T];X_s)$. Then for all $(u_0,u_1)\in X_s$, there exists a unique solution $u$ of \begin{equation} \label{eq: well-posedness nonlinear very weak} \begin{cases} (\partial_t^2+(-\Delta)^s) u = \mathcal{F}(u,\partial_t u) & \text{ in } \Omega_T,\\ u =0 & \text{ on } (\Omega_e)_T,\\ u(0) = u_0,\, \partial_{t}u(0) = u_1 & \text{ on } \Omega, \end{cases} \end{equation} that is the formula \eqref{eq: weak formulation of very weak sols} holds with $F$ replaced by $\mathcal{F}(u,\partial_t u)$ in which we test against every weak solution $v$ of the adjoint equation \begin{equation} \label{eq: adjoint eq nonlinear} \begin{cases} (\partial_t^2+(-\Delta)^s) v = G & \text{ in } \Omega_T,\\ v =0 & \text{ on } (\Omega_e)_T,\\ v(T) = 0,\, \partial_{t}v(T) = 0 & \text{ on } \Omega \end{cases} \end{equation} with $G\in L^2(0,T;\widetilde{L}^2(\Omega))$. \end{theorem} \begin{proof}[Proof of Theorem \ref{thm: general well-posedness result of very weak solutions}] Let $u_h\in C([0,T];\widetilde{L}^2(\Omega))\cap C^1([0,T];H^{-s}(\Omega))$ be the unique solution to \begin{equation} \label{eq: homogeneous part} \begin{cases} (\partial_t^2+(-\Delta)^s) u = 0 & \text{ in } \Omega_T,\\ u =0 & \text{ on } (\Omega_e)_T,\\ u(0) = u_0,\, \partial_{t}u(0) = u_1 & \text{ on } \Omega \end{cases} \end{equation} and let us set $U_h\vcentcolon =(u_h,\partial_t u_h)\in C([0,T];X_s)$. Furthermore, we define the operator $\mathcal{T}\colon C([0,T];X_s)\to C([0,T];X_s)$ as \begin{equation} \mathcal{T}(U)\vcentcolon = U_h+S(\mathcal{F}(U)), \end{equation} which is well-defined by \eqref{prop: solution map} and the properties of $\mathcal{F}$. Next, we show that $\mathcal{T}$ has a unique fixed point $U=(U_1,U_2)$. \medskip \noindent\textit{Step 1.~Existence.} Let $U,V\in C([0,T];X_s)$, then by linearity of $S$, \eqref{eq: continuity estimate solution map} and \eqref{eq: Lipschitz estimate nonlinearity} we get \[ \begin{split} \|\mathcal{T}(U)(t)-\mathcal{T}(V)(t)\|_{X_s}&=\|S(\mathcal{F}(U))(t)-S(\mathcal{F}(V))(t)\|_{X_s}\\ &=\|S(\mathcal{F}(U)-\mathcal{F}(V))(t)\|_{X_s}\\ &\leq C\|\mathcal{F}(U)-\mathcal{F}(V)\|_{L^2(0,t;H^{-s}(\Omega))}\\ &\leq C\|U-V\|_{L^2(0,t;X_s)}. \end{split} \] Next, let us define the following norm on $X_s$ \begin{equation} \label{eq: new norm on Xs} \|U\|_{\theta}\vcentcolon = \sup_{0\leq t\leq T}\left(e^{-\theta t}\|U(t)\|_{X_s}\right) \end{equation} for $\theta>0$, which will be fixed in a moment. Then we have the estimate \[ \|\mathcal{T}(U)(t)-\mathcal{T}(V)(t)\|_{X_s}\leq C\left(\int_0^te^{2\theta \tau}\,d\tau\right)^{1/2}\|U-V\|_{\theta}\leq \frac{C}{(2\theta)^{1/2}}e^{\theta t}\|U-V\|_{\theta} \] and hence there holds \[ \|\mathcal{T}(U)(t)-\mathcal{T}(V)(t)\|_{\theta}\leq \frac{C}{(2\theta)^{1/2}}\|U-V\|_{\theta}. \] Therefore, we deduce that $\mathcal{T}$ is a strict contraction from the complete metric space $(C([0,T];X_s),\|\cdot\|_{\theta})$ to itself, when $\theta>0$ is chosen such that $C/(2\theta)^{1/2}<1$. Now, we may invoke Banach's fixed point theorem to obtain a unique fixed point $U=(u,w)$ of $\mathcal{T}$. Next, observe that the definition of the solution map $S$ and $U=\mathcal{T}(U)=U_h+S(\mathcal{F}(U))$ imply \[ u=u_h+u_n \text{ and } w=\partial_t u, \] where $u_n$ solves \begin{equation} \label{eq: nonlinear part} \begin{cases} (\partial_t^2+(-\Delta)^s) v = \mathcal{F}(U) & \text{ in } \Omega_T,\\ v =0 & \text{ on } (\Omega_e)_T,\\ v(0) = 0,\, \partial_{t}v(0) = 0 & \text{ on } \Omega. \end{cases} \end{equation} Going back to the definition of very weak solutions, we see this implies that $u$ solves \eqref{eq: well-posedness nonlinear very weak}. \medskip \noindent\textit{Step 2.~Uniqueness.} Suppose $\Tilde{u}\in C([0,T];\widetilde{L}^2(\Omega))\cap C^1([0,T];H^{-s}(\Omega))$ is any other solution to \eqref{eq: well-posedness nonlinear very weak}, then $\bar{u}\vcentcolon = u-\widetilde{u}$ solves \begin{equation} \label{eq: uniquenes very weak nonlinear} \begin{cases} (\partial_t^2+(-\Delta)^s) v = \mathcal{F}(u,\partial_t u)-\mathcal{F}(\widetilde{u},\partial_t \widetilde{u}) & \text{ in } \Omega_T,\\ v =0 & \text{ on } (\Omega_e)_T,\\ v(0) = 0,\, \partial_{t}v(0) = 0 & \text{ on } \Omega. \end{cases} \end{equation} Thus, applying the energy estimate \eqref{eq: energy estimate very weak without damping and potential} together with the Lipschitz assumption on $\mathcal{F}$, we see that \[ \begin{split} \|U(t)-\widetilde{U}(t)\|^2_{X_s}&\leq C\int_0^t\|\mathcal{F}(U)(\tau)-\mathcal{F}(\widetilde{U})(\tau)\|_{H^{-s}(\Omega)}^2\,d\tau\\ &\leq C\int_0^t\|U(t)-\widetilde{U}(t)\|^2_{X_s}\,d\tau, \end{split} \] where $U=(u,\partial_tu)$ and $\widetilde{U}=(\widetilde{u},\partial_t\widetilde{u})$. So, Gronwall's inequality shows that $u=\widetilde{u}$. This establishes the uniqueness assertion and we can conclude the proof. \end{proof} As an application of Theorem \ref{thm: general well-posedness result of very weak solutions}, we can show the unique solvability of \eqref{eq: damped nonlocal wave equations well-posedness} for rough source and initial data. \begin{theorem}[Very weak solutions to DNWEQ] \label{thm: very weak sol DNWEQ} Let $\Omega \subset\R^n$ be a bounded Lipschitz domain, $T>0$, $0<s<\alpha\leq 1$ and suppose that $1\leq p\leq \infty$ satisfies \eqref{eq: restrictions on p}. Assume that we have given coefficients $(\gamma,q)\in C^{0,\alpha}(\R^n)\times L^{p}(\Omega)$. Then for any $F\in L^2(0,T;H^{-s}(\Omega))$ and $(u_0,u_1)\in \widetilde{L}^2(\Omega)\times H^{-s}(\Omega)$, there exists a unique solution of \begin{equation} \label{eq: well-posedness very weak DNWEQ} \begin{cases} (L_\gamma+q) u = F & \text{ in } \Omega_T,\\ u =0 & \text{ on } (\Omega_e)_T,\\ u(0) = u_0,\, \partial_{t}u(0) = u_1 & \text{ on } \Omega. \end{cases} \end{equation} \end{theorem} \begin{proof} Let us define the mapping $\mathcal{F}\colon C([0,T];X_s)\to L^2(0,T;H^{-s}(\Omega))$ by \[ \mathcal{F}(U)(t)\vcentcolon =F-\gamma w(t)-qu(t), \] where $U=(u,w)\in C([0,T];X_s)$. On the one hand, using the estimate \eqref{eq: L2 estimate potential} we see that for any $u\in \widetilde{L}^2(\Omega)$ one has $qu\in H^{-s}(\Omega)$ and there holds \begin{equation} \label{eq: continuity estimate dual potential} \begin{split} \|qu\|_{H^{-s}(\Omega)} &=\sup_{\|v\|_{\widetilde{H}^s(\Omega)}\leq 1}|\langle u,qv\rangle_{L^2(\Omega)}|\\ &\leq C\|q\|_{L^p(\Omega)}\|u\|_{L^2(\Omega)}. \end{split} \end{equation} On the other hand, by applying \cite[Lemma 3.1]{Stability-fractional-conductivity} and $\partial\Omega\in C^0$ we deduce that for any $v\in \widetilde{H}^s(\Omega)$ one has $\gamma v\in \widetilde{H}^s(\Omega)$ and it obeys the estimate \begin{equation} \label{eq: multiplication by gamma} \|\gamma v\|_{\widetilde{H}^s(\Omega)}\leq C\|\gamma\|_{C^{0,\alpha}(\R^n)}\|v\|_{\widetilde{H}^s(\Omega)}. \end{equation} Thus, we can again infer from a duality argument that $H^{-s}(\Omega)\ni w\mapsto \gamma w\in H^{-s}(\Omega)$ is a continuous map satisfying \begin{equation} \label{eq: continuity estimate dual damping} \begin{split} \|\gamma w\|_{H^{-s}(\Omega)}&=\sup_{\|v\|_{\widetilde{H}^s(\Omega)}\leq 1}|\langle \gamma w,v\rangle|\\ &=\sup_{\|v\|_{\widetilde{H}^s(\Omega)}\leq 1}|\langle w,\gamma v\rangle|\\ &\leq C\|\gamma\|_{C^{0,\alpha}(\R^n)}\|w\|_{H^{-s}(\Omega)}. \end{split} \end{equation} From the estimates \eqref{eq: continuity estimate dual potential} and \eqref{eq: continuity estimate dual damping}, we easily deduce that $\mathcal{F}$ is well-defined and satisfies the Lipschitz estimate \begin{equation} \label{eq: Lipschitz estimate application} \|\mathcal{F}(U)(t)-\mathcal{F}(V)(t)\|_{H^{-s}(\Omega)}\leq C(\|\gamma\|_{C^{0,\alpha}(\R^n)}+\|q\|_{L^p(\Omega)})\|U(t)-V(t)\|_{X_s} \end{equation} for all $U,V\in C([0,T];X_s)$. Thus, we can apply Theorem \ref{thm: general well-posedness result of very weak solutions} to get the existence of a unique solution to \eqref{eq: well-posedness very weak DNWEQ} in the sense that for any $G\in L^2(0,T;\widetilde{L}^2(\Omega))$ and corresponding solution $v$ of \eqref{eq: adjoint eq nonlinear}, there holds \begin{equation} \label{eq: prelim def of very weak sol} \int_0^T \langle G,u\rangle_{L^2(\Omega)}\,dt=\int_0^T\langle (F-\gamma \partial_t u-qu),v\rangle\,dt+\langle u_1,v(0)\rangle-\langle u_0,\partial_t v(0)\rangle_{L^2(\Omega)}. \end{equation} It remains to verify that $u$ is indeed a solution of \eqref{eq: well-posedness very weak DNWEQ} in the sense of Definition \ref{def: very weak solutions}. For this purpose let $G\in L^2(0,T;\widetilde{L}^2(\Omega))$ and suppose that $v$ is the unique solution to \eqref{eq: adjoint eq of def very weak}. Hence, $v$ solves \[ \begin{cases} (\partial_t^2+(-\Delta)^s) v = \widetilde{G} & \text{ in } \Omega_T,\\ v =0 & \text{ on } (\Omega_e)_T,\\ v(T) = 0,\, \partial_{t}v(T) = 0 & \text{ on } \Omega \end{cases} \] with $\widetilde{G}=G+\gamma \partial_t v-qv\in L^2(0,T;\widetilde{L}^2(\Omega))$ (see \eqref{eq: L2 estimate potential}). Next, we claim that there holds \begin{equation} \label{eq: integration by parts formula} \int_0^T \langle \gamma \partial_t u,v\rangle\,dt=-\int_0^T \langle \gamma\partial_t v,u\rangle_{L^2(\Omega)}\,dt-\langle \gamma u_0,v(0)\rangle. \end{equation} For this purpose, let us consider for $\eps>0$ the unique solution $v_\eps\in H^1(0,T;\widetilde{H}^s(\Omega))$ with $\partial_t^2 v_\eps\in L^2(0,T;H^{-s}(\Omega))$ to the following parabolically regularized problem \begin{equation} \label{eq: regularization for equation of w} \begin{cases} (\partial_t^2 -\eps (-\Delta)^s \partial_t +(-\Delta)^s)v = \widetilde{G}&\text{ in }\Omega_T,\\ v=0 &\text{ in }(\Omega_e)_T,\\ v(T)= \partial_t v(T)=0 &\text{ in }\Omega \end{cases} \end{equation} (see \cite[Chapter XVIII, Section 5.3.1]{DautrayLionsVol5}). By \cite[Chapter XVIII, Section 5.3.4]{DautrayLionsVol5} we know that there holds \begin{equation} \label{eq: convergence} \begin{split} v_{\eps}&\weakstar v \text{ in }L^{\infty}(0,T;\widetilde{H}^s(\Omega)),\\ \partial_t v_{\eps}&\weakstar \partial_t v \text{ in }L^{\infty}(0,T;\widetilde{L}^2(\Omega)),\\ v_\eps(t)&\to v(t) \text{ in } \widetilde{H}^s(\Omega)\text{ for all }0\leq t\leq T. \end{split} \end{equation} First, note that the conditions $u\in C^1([0,T];H^{-s}(\Omega))$ and $v_\eps\in C^1([0,T];\widetilde{L}^2(\Omega))$, where the latter follows from the Sobolev embedding, guarantee that $\langle \gamma u,v_\eps\rangle\in C^1([0,T])$ with \begin{equation} \label{eq: product rule} \partial_t \langle \gamma u,v_\eps\rangle=\langle \partial_t u,\gamma v_\eps\rangle+\langle u,\gamma \partial_t v_\eps\rangle_{L^2(\Omega)}. \end{equation} Thus, by the fundamental theorem of calculus we deduce that there holds \[ \langle \gamma u(T),v_\eps(T)\rangle-\langle \gamma u_0,v_\eps(0)\rangle=\int_0^T\langle \partial_t u,\gamma v_\eps\rangle+\langle u,\gamma \partial_t v_\eps\rangle_{L^2(\Omega)}\,dt. \] By the convergence assertions \eqref{eq: convergence} and $v_\eps (T)=0$, we get \[ -\langle \gamma u_0,v(0)\rangle=\int_0^T\langle \partial_t u,\gamma v\rangle+\langle u,\gamma \partial_t v\rangle_{L^2(\Omega)}\,dt. \] This proves \eqref{eq: integration by parts formula}. Hence, inserting this into \eqref{eq: prelim def of very weak sol}, we obtain \[ \begin{split} \int_0^T\langle\widetilde{G},u\rangle_{L^2(\Omega)}\,dt&=\int_0^T\langle (F-\gamma \partial_t u-qu),v\rangle\,dt+\langle u_1,v(0)\rangle-\langle u_0,\partial_t v(0)\rangle_{L^2(\Omega)}\\ &=\int_0^T\langle F,v\rangle\,dt+\int_0^T \langle u,\gamma \partial_t v\rangle_{L^2(\Omega)}\,dt-\int_0^T\langle u,qv\rangle\,dt\\ &\quad+\langle u_1,v(0)\rangle-\langle u_0,\partial_t v(0)\rangle_{L^2(\Omega)}+\langle \gamma u_0,v( 0)\rangle. \end{split} \] As $\widetilde{G}=G+\gamma \partial_t v-qv$ this gives \[ \begin{split} \int_0^T \langle G,u\rangle_{L^2(\Omega)}\,dt&=\int_0^T \langle F,v\rangle\, dt+\langle u_1,v(0)\rangle-\langle u_0,\partial_t v(0)\rangle_{L^2(\Omega)}+\langle \gamma u_0,v(0)\rangle. \end{split} \] Hence, we observe that $u$ is indeed a solution of \eqref{eq: well-posedness very weak DNWEQ} in the sense of Definition \ref{def: very weak solutions}. By reversing the above arguments one can also observe that if $u$ is a solution in the sense of Definition \ref{def: very weak solutions}, then by \eqref{eq: integration by parts formula} it is a solution in the sense of \eqref{eq: prelim def of very weak sol} and thus the solution in the sense of Definition \ref{def: very weak solutions} is unique. \end{proof} \section{The inverse problem} \label{sec: inverse problem} After establishing the theory of very weak solutions to damped, nonlocal wave equations, we now turn our attention to the inverse problem part. First, in Section \ref{sec: Runge approx} we prove the optimal Runge approximation theorem (Theorem \ref{thm: Runge approx}) and in Section \ref{sec: integral identity} a suitable integral identity. Using these results, we then show in Section \ref{sec: linear uniqueness} our first main result dealing with linear perturbations (Theorem \ref{thm: uniqueness linear}). Finally, in Section \ref{sec: semilinear uniqueness} we prove Theorem \ref{thm: uniqueness semilinear} showing that the damping coefficient and the nonlinearity can be determined simultaneously. \subsection{Runge approximation} \label{sec: Runge approx} With the material from Section \ref{sec: Very weak solutions to damped, nonlocal wave equations} at our disposal, we can now show the following Runge approximation theorem, whose proof is very similar to the one of \cite[Theorem 1.2]{Optimal-Runge-nonlocal-wave}. \begin{theorem}[Runge approximation] \label{thm: Runge approx} Let $\Omega \subset\R^n$ be a bounded Lipschitz domain, $T>0$, $0<s<\alpha\leq 1$ and suppose that $1\leq p\leq \infty$ satisfies \eqref{eq: restrictions on p}. Assume that we have given coefficients $(\gamma,q)\in C^{0,\alpha}(\R^n)\times L^{p}(\Omega)$. Then for any measurement set $W\subset\Omega_e$ and initial conditions $(u_0,u_1)\in \widetilde{H}^s(\Omega)\times\widetilde{L}^2(\Omega)$, the \emph{Runge set} \begin{equation} \label{eq: Runge set} \mathcal{R}^{u_0,u_1}_W\vcentcolon = \{u_\varphi-\varphi\,;\,\varphi\in C_c^{\infty}(W_T)\} \end{equation} is dense in $L^2(0,T;\widetilde{H}^s(\Omega))$, where $u_\varphi$ is the unique solution to \begin{equation} \label{eq: PDE Runge} \begin{cases} (L_{\gamma}+q)u = 0 & \text{ in } \Omega_T,\\ u =\varphi & \text{ on } (\Omega_e)_T,\\ u(0) = u_0,\, \partial_{t}u(0) = u_1 & \text{ on } \Omega, \end{cases} \end{equation} (see Proposition \ref{prop: Weak solutions to inhomogeneous DNWEQ}). \end{theorem} \begin{proof} First of all note that it is enough to consider the case $(u_0,u_1)=0$. To see this assume that the density holds for $\mathcal{R}_W\vcentcolon =\mathcal{R}_W^{0,0}$ and let $f\in L^2(0,T;\widetilde{H}^s(\Omega))$. Let $v_0$ be the unique solution to \begin{equation} \begin{cases} (L_{\gamma}+q)v = 0 & \text{ in } \Omega_T,\\ v =0 & \text{ on } (\Omega_e)_T,\\ v(0) = u_0,\, \partial_{t}v(0) = u_1 & \text{ on } \Omega \end{cases} \end{equation} and define $\widetilde{f}\vcentcolon = f-v_0\in L^2(0,T;\widetilde{H}^s(\Omega))$. By assumption there exists $(\varphi_k)_{k\in\N}\subset C_c^{\infty}(W_T)$ such that $u_k-\varphi_k\to \widetilde{f}$ in $L^2(0,T;\widetilde{H}^s(\Omega))$ as $k\to \infty$, where $u_k$ is the unique solution to \begin{equation} \label{eq: PDE Runge vanishing initial} \begin{cases} (L_{\gamma}+q)u = 0 & \text{ in } \Omega_T,\\ u =\varphi & \text{ on } (\Omega_e)_T,\\ u(0) = 0,\, \partial_{t}u(0) = 0 & \text{ on } \Omega \end{cases} \end{equation} with $\varphi=\varphi_k$. Then $v_k\vcentcolon = u_k+v_0$ is the unique solution to \eqref{eq: PDE Runge} with $\varphi=\varphi_k$. The above convergence now implies $v_k-\varphi_k\to f$ in $L^2(0,T;\widetilde{H}^s(\Omega))$ as $k\to\infty$ and we get that $\mathcal{R}_W^{u_0,u_1}$ is dense in $L^2(0,T;\widetilde{H}^s(\Omega))$. Therefore, it remains to show that $\mathcal{R}_W$ is dense in $L^2(0,T;\widetilde{H}^s(\Omega))$. As usual, we show this by a Hahn--Banach argument. Thus, suppose that $F\in L^2(0,T;H^{-s}(\Omega))$ vanishes on $\mathcal{R}_W$. Let us recall that if $\varphi\in C_c^{\infty}(W_T)$ and $u$ solves \eqref{eq: PDE Runge vanishing initial}, then by \eqref{eq: Usual cauchy homogeneous} and Lemma \ref{lemma: time reversal of solution} the function $v=(u-\varphi)^\star$ satisfies \begin{equation} \begin{cases} (L_{-\gamma}+q)v = -(-\Delta)^s\varphi^\star &\text{ in }\Omega_T,\\ v=0 &\text{ in }(\Omega_e)_T,\\ v(T)=\partial_t v(T)=0 &\text{ in }\Omega. \end{cases} \end{equation} Next, let $w$ be the unique solution to \begin{equation} \label{eq: solution to weak RHS} \begin{cases} (L_{\gamma}+q)w= F^\star &\text{ in }\Omega_T,\\ w=0 &\text{ in }(\Omega_e)_T,\\ w(0)= \partial_t w(0)=0 &\text{ in }\Omega \end{cases} \end{equation} (see Theorem \ref{thm: very weak sol DNWEQ}). By testing the equation for $w$ by $v$, we get \[ -\int_0^T \langle (-\Delta)^s\varphi^\star, w\rangle_{L^2(\Omega)}\,dt=\int_0^T\langle F^\star,v\rangle\,dt=\int_0^T\langle F,u_\varphi-\varphi\rangle\,dt=0 \] for any $\varphi\in C_c^{\infty}(W_T)$. This ensures that there holds \[ (-\Delta)^s w=0\quad \text{ in }W_T. \] Furthermore, by construction $w$ vanishes in $(\Omega_e)_T$ and hence the unique continuation principle for the fractional Laplacian guarantees $w=0$ in $\R^n_T$ (see \cite{GSU20}). As very weak solutions are distributional solutions, we get \[ \int_0^T\langle F^\star,\Phi\rangle\,dt=\int_0^T \langle (L_{-\gamma}+q)\Phi,w\rangle_{L^2(\Omega)}\,dt=0 \] for all $\Phi\in C_c^{\infty}(\Omega_T)$. To see that very weak solutions are distributional solutions, one can simply take $G=\chi_\Omega(L_\gamma+q)\Phi$ with $\Phi\in C_c^{\infty}(\Omega\times [0,T))$ in Definition \ref{def: very weak solutions}, where $\chi_\Omega$ denotes the characteristic function of $\Omega$ (see also \cite[Proposition 3.8]{Optimal-Runge-nonlocal-wave}). By density of $C_c^{\infty}(\Omega_T)$ in $L^2(0,T;\widetilde{H}^s(\Omega))$ we deduce that $F=0$. This concludes the proof. \end{proof} As a consequence we have the following lemma. \begin{lemma}[Convergence of time derivative] \label{lemma: convergence of time derivatives} Let $\Omega \subset\R^n$ be a bounded Lipschitz domain, $T>0$, $0<s<\alpha\leq 1$ and suppose that $1\leq p\leq \infty$ satisfies \eqref{eq: restrictions on p}. Assume that we have given coefficients $(\gamma,q)\in C^{0,\alpha}(\R^n)\times L^{p}(\Omega)$. Let $\Phi,\Psi\in L^2(0,T;\widetilde{H}^s(\Omega))\cap H^1(0,T;H^{-s}(\Omega))$ and suppose $(\varphi_k)_{k\in\N}\subset C_c^{\infty}((\Omega_e)_T)$ is such that \begin{equation} \label{eq: convergence assertion for time derivative} u_k-\varphi_k\to \Phi\text{ in }L^2(0,T;\widetilde{H}^s(\Omega))\text{ as }k\to\infty, \end{equation} where $u_k$ solves \begin{equation} \label{eq: approx time derivative} \begin{cases} (L_{\gamma}+q)u = 0 & \text{ in } \Omega_T,\\ u =\varphi_k & \text{ on } (\Omega_e)_T,\\ u(0) = 0,\, \partial_{t}u(0) = 0 & \text{ on } \Omega \end{cases} \end{equation} for $k\in\N$. If $\Phi,\Psi$ satisfy one of the conditions \begin{enumerate}[(a)] \item \label{item cond 1 first} $\Psi(T)=\Phi(0)=0$ \item \label{item cond 2 first} or $\Psi(T)=\Psi(0)=0$, \end{enumerate} then we have \begin{equation} \label{eq: first order limit} \lim_{k\to\infty}\int_0^T\langle \partial_t (u_k-\varphi_k),\Psi\rangle \,dt =\int_0^T\langle \partial_t\Phi,\Psi\rangle\,dt. \end{equation} \end{lemma} \begin{remark} Let us note that the same formula \eqref{eq: first order limit} holds for second order time derivatives under appropriate conditions. \end{remark} \begin{proof} Using the integration by parts formula, we may compute \[\small \begin{split} &\lim_{k\to\infty}\int_0^T\langle \partial_t (u_k-\varphi_k),\Psi\rangle \,dt \\ &= \lim_{k\to\infty}\left(\langle (u_k-\varphi_k)(T),\Psi(T)\rangle_{L^2(\Omega)}-\langle (u_k-\varphi_k)(0),\Psi(0)\rangle_{L^2(\Omega)}-\int_0^T\langle \partial_t\Psi,u_k-\varphi_k\rangle \,dt\right)\\ &=-\lim_{k\to\infty}\int_0^T\langle \partial_t\Psi,u_k-\varphi_k\rangle \,dt\\ &=-\int_0^T \langle \partial_t \Psi,\Phi\rangle \, dt\\ &=\langle \Phi(0),\Psi(0)\rangle_{L^2(\Omega)}-\langle \Phi(T),\Psi(T)\rangle_{L^2(\Omega)}+\int_0^T \langle \partial_t\Phi, \Psi\rangle \, dt\\ &=\int_0^T \langle \partial_t\Phi, \Psi\rangle \, dt. \end{split} \] In the first equality sign we used an integration by parts, in the second equality we used \eqref{eq: approx time derivative}, $\Psi(T)=0$ and \eqref{eq: approx time derivative}, in the third equality the convergence \eqref{eq: convergence assertion for time derivative}, in the fourth equality again an integration by parts and finally in the last equality the conditions \ref{item cond 1 first} or \ref{item cond 2 first}. \end{proof} \subsection{DN map and integral identities} \label{sec: integral identity} Next, we define the \emph{Dirichlet to Neumann (DN) map} $\Lambda_{\gamma,q}$ related to \begin{equation} \label{eq: PDE for integral identity} \begin{cases} (L_{\gamma}+q)u = 0 & \text{ in } \Omega_T,\\ u =\varphi & \text{ on } (\Omega_e)_T,\\ u(0) = 0,\, \partial_{t}u(0) = 0 & \text{ on } \Omega, \end{cases} \end{equation} via \begin{equation} \label{eq: DN map} \langle \Lambda_{\gamma,q}\varphi,\psi\rangle=\int_{\R^n_T}(-\Delta)^{s/2} u_\varphi (-\Delta)^{s/2}\psi\,dx \end{equation} for all $\varphi,\psi\in C_c^{\infty}((\Omega_e)_T)$, where $u_\varphi$ is the unique solution to \eqref{eq: PDE for integral identity} with exterior condition $\varphi$. Using the above preparation, we now establish the following integral identity. \begin{proposition}[Integral identity for linear perturbations] \label{prop: integral identity} Let $\Omega \subset\R^n$ be a bounded Lipschitz domain, $T>0$, $0<s<\alpha\leq 1$ and suppose that $1\leq p\leq \infty$ satisfies \eqref{eq: restrictions on p}. Assume that we have given coefficients $(\gamma_j,q_j)\in C^{0,\alpha}(\R^n)\times L^{p}(\Omega)$ for $j=1,2$. Let $\varphi_j\in C_c^{\infty}((\Omega_e)_T)$ and denote by $u_j$ the corresponding solution of \eqref{eq: PDE for integral identity} with $(\gamma,q)=(\gamma_j,q_j)$. Then there holds \begin{equation} \label{eq: integral identity} \begin{split} &\langle (\Lambda_{\gamma_1,q_1}-\Lambda_{\gamma_2,q_2})\varphi_1,\varphi_2^\star\rangle\\ &\quad =\int_{\Omega_T}\{[(\gamma_1-\gamma_2)\partial_t+q_1-q_2](u_1-\varphi_1)\}(u_2-\varphi_2)^\star\,dxdt. \end{split} \end{equation} \end{proposition} \begin{proof} Let $(\Gamma_j,Q_j)\in C^{0,\alpha}(\R^n)\times L^p(\Omega)$, $j=1,2$, and suppose $U_j$ is the unique solutions of \eqref{eq: PDE for integral identity} with $(\gamma,q)=(\Gamma_j,Q_j)$ and exterior condition $\varphi=\psi_j$. Then we may compute \begin{equation} \label{eq: calculation for integral identity} \begin{split} &\int_{\Omega_T}\{[(\Gamma_1-\Gamma_2)\partial_t+Q_1-Q_2](U_1-\psi_1)\}(U_2-\psi_2)^\star\,dxdt\\ & =\int_{\Omega_T}\{[\Gamma_1\partial_t+Q_1](U_1-\psi_1)\}(U_2-\psi_2)^\star\,dxdt\\ &\quad -\int_{\Omega_T}(U_1-\psi_1)[- \Gamma_2\partial_t+Q_2](U_2-\psi_2)^\star\,dxdt\\ &=\int_{0}^T\langle L_{\Gamma_1,Q_1}(U_1-\psi_1),(U_2-\psi_2)^\star\rangle\,dt\\ &\quad -\int_{0}^T\langle (\partial_t^2+(-\Delta)^s)(U_1-\psi_1),(U_2-\psi_2)^\star\rangle\,dt\\ &\quad -\int_{0}^T\langle L_{-\Gamma_2,Q_2}(U_2-\psi_2)^\star,(U_1-\psi_1)\rangle\,dt\\ &\quad +\int_{0}^T\langle (\partial_t^2+(-\Delta)^s)(U_2-\psi_2)^\star,(U_1-\psi_1)\rangle\,dt\\ &=-\int_{0}^T\langle (-\Delta)^s\psi_1,(U_2-\psi_2)^\star\rangle_{L^2(\Omega)}\,dt +\int_{0}^T\langle (-\Delta)^s \psi_2^\star,(U_1-\psi_1)\rangle_{L^2(\Omega)}\,dt\\ &=\int_{\R^n_T} ((-\Delta)^s\psi_2^\star) U_1\,dxdt-\int_{\R^n_T} ((-\Delta)^s\psi_1) U_2^\star\,dxdt\\ &=\langle \Lambda_{\Gamma_1,Q_1}\psi_1,\psi_2^\star\rangle-\langle \Lambda_{\Gamma_2,Q_2}\psi_2,\psi_1^\star\rangle. \end{split} \end{equation} In the first equality we used that $U_1-\psi_1$ has vanishing initial conditions, $(U_2-\psi_2)^\star$ has vanishing terminal conditions and an integration by parts. In the third equality we used that the PDEs for $U_1-\psi_1$ and $(U_2-\psi_2)^\star$ hold in the sense of $L^2(0,T;H^{-s}(\Omega))=(L^2(0,T;\widetilde{H}^s(\Omega))'$ (see Lemma \ref{lemma: time reversal of solution}). In the fourth equality, we used the PDEs for $U_1$ and $U_2$, Lemma \ref{lemma: time reversal of solution} and that there holds \[ \begin{split} &\int_{0}^T\langle (\partial_t^2+(-\Delta)^s)(U_2-\psi_2)^\star,(U_1-\psi_1)\rangle\,dt\\ &=\int_{0}^T\langle (\partial_t^2+(-\Delta)^s)(U_1-\psi_1),(U_2-\psi_2)^\star\rangle\,dt, \end{split} \] which can be established similarly as \cite[Claim 4.2]{Semilinear-nonlocal-wave-eq} (see also the proof of Theorem \ref{thm: very weak sol DNWEQ}). In the last equality, we have made the change of variables $\tau=T-t$ for the second integral. On the one hand, using \eqref{eq: calculation for integral identity} with \[ \Gamma_1=\Gamma_2=\gamma_j\text{ and }Q_1=Q_2=q_j, \] we observe that \begin{equation} \label{eq: self-adjointness DN map} \langle \Lambda_{\gamma_j,q_j}\psi_1,\psi_2^\star\rangle=\langle \Lambda_{\gamma_j,q_j}\psi_2,\psi_1^\star\rangle \end{equation} for all $\psi_j\in C_c^{\infty}((\Omega_e)_T)$, $j=1,2$. On the other hand, choosing \[ \Gamma_j=\gamma_j,\,Q_j=q_j\text{ and }\psi_j=\varphi_j \] in \eqref{eq: calculation for integral identity} and taking into account the self-adjointness \eqref{eq: self-adjointness DN map}, we get \eqref{eq: integral identity}. \end{proof} \subsection{Simultaneous determination of damping coefficient and linear perturbations} \label{sec: linear uniqueness} \begin{proof}[Proof of Theorem \ref{thm: uniqueness linear}] First note that by the integral identity in Proposition \ref{prop: integral identity}, we may deduce from the condition \eqref{eq: equality of DN maps} that there holds \begin{equation} \label{eq: uniqueness proof help identity} \int_{\Omega_T}\{[(\gamma_1-\gamma_2)\partial_t+q_1-q_2](u_1-\varphi_1)\}(u_2-\varphi_2)^\star\,dxdt=0 \end{equation} for all $\varphi_j\in C_c^{\infty}((W_j)_T)$, where $u_j$ denotes the unique solution to \begin{equation} \label{eq: PDE uniqueness proof} \begin{cases} (L_{\gamma_j}+q_j)u = 0 & \text{ in } \Omega_T,\\ u =\varphi_j & \text{ on } (\Omega_e)_T,\\ u(0) = 0,\, \partial_{t}u(0) = 0 & \text{ on } \Omega. \end{cases} \end{equation} Let $\omega\Subset \Omega$ and choose a cutoff function $\Phi_1\in C_c^{\infty}(\Omega)$ satisfying $\Phi_1=1$ on $\omega$. Moreover, let $\Phi_2\in C_c^{\infty}(\omega_T)$. By the Runge approximation (Theorem \ref{thm: Runge approx}), there exist sequences $(\varphi_j^k)_{k\in\N}\subset C_c^{\infty}((W_j)_T)$ with corresponding solutions $u_j^k$ of \eqref{eq: PDE uniqueness proof} with $\varphi_j=\varphi_j^k$ such that $u_j^k-\varphi_j^k\to \Phi_j$ in $L^2(0,T;\widetilde{H}^s(\Omega))$. Taking $\varphi_1=\varphi_1^k$ and $\varphi_2=\varphi_2^{\ell}$ in \eqref{eq: uniqueness proof help identity} gives \[ \int_{\Omega_T}\{[(\gamma_1-\gamma_2)\partial_t+q_1-q_2](u^k_1-\varphi^k_1)\}(u^{\ell}_2-\varphi^{\ell}_2)^\star\,dxdt=0 \] for all $k,\ell\in\N$. First, we let $\ell\to\infty$ to deduce \begin{equation} \label{eq: uniqueness proof help identity 2} \int_{\Omega_T}\{[(\gamma_1-\gamma_2)\partial_t+q_1-q_2](u^k_1-\varphi^k_1)\}\Phi_2^\star\,dxdt=0 \end{equation} for all $k\in\N$. As $\gamma_1-\gamma_2\in C^{0,\alpha}(\R^n)$ the estimate \eqref{eq: multiplication by gamma} ensures that we can apply Lemma \ref{lemma: convergence of time derivatives} under the condition \ref{item cond 2 first} and so $\partial_t\Phi_1=0$ shows that the first term in \eqref{eq: uniqueness proof help identity 2} goes to zero. So in the limit $k\to\infty$ what remains is \[ \int_{\Omega_T}(q_1-q_2)\Phi_2^\star \,dxdt=0, \] where we used $\Phi_1=1$ on $\omega$. This ensures that $q_1=q_2$ on $\omega$. As the set $\omega$ is arbitrary, we get $q_1=q_2$ in $\Omega$. Now, the identity \eqref{eq: uniqueness proof help identity} reduces to \[ \int_{\Omega_T}\{[(\gamma_1-\gamma_2)\partial_t](u_1-\varphi_1)\}(u_2-\varphi_2)^\star\,dxdt=0 \] for all $\varphi_j\in C_c^{\infty}((W_j)_T)$. We choose $\eta\in C_c^{\infty}(\Omega_T)$, define \[ \Phi_1(x,t)=\int_0^t\eta(x,\tau)\,d\tau\in C_c^{\infty}(\Omega\times (0,T]) \] and take $\Phi_2\in C_c^{\infty}(\Omega_T)$. Then using $\partial_t \Phi_1=\eta$ and arguing as above via a Runge approximation and Lemma \ref{lemma: convergence of time derivatives}, we get from \eqref{eq: uniqueness proof help identity} the identity \[ \int_{\Omega_T}(\gamma_1-\gamma_2)\eta\Phi_2^\star\,dxdt=0. \] This again implies $\gamma_1=\gamma_2$ in $\Omega$. \end{proof} \subsection{Simultaneous determination of damping coefficient and nonlinearity} \label{sec: semilinear uniqueness} Before turning to the proof of our second main result, let us recall that the \emph{DN map} related to the problem \begin{equation} \label{eq: PDE for semilinear problem} \begin{cases} L_{\gamma}u+f(u) = 0 & \text{ in } \Omega_T,\\ u =\varphi & \text{ on } (\Omega_e)_T,\\ u(0) = 0,\, \partial_{t}u(0) = 0 & \text{ on } \Omega \end{cases} \end{equation} is defined by \begin{equation} \langle \Lambda_{\gamma,f}\varphi,\psi\rangle\vcentcolon = \int_{\R^n_T}(-\Delta)^{s/2}u_\varphi(-\Delta)^{s/2}\psi\,dxdt, \end{equation} where $\varphi,\psi\in C_c^{\infty}((\Omega_e)_T)$ and $u_\varphi$ is the unique solution to \eqref{eq: PDE for semilinear problem} (see \cite[Proposition 3.7]{Semilinear-nonlocal-wave-eq}). \begin{proof}[Proof of Theorem \ref{thm: uniqueness semilinear}] Let $\eps>0$ and denote by $u^{(j)}_\eps$ the unique solutions to \eqref{eq: PDE for semilinear problem} with $f=f_j$, $\gamma=\gamma_j$ and $\varphi=\eps \eta$ for some fixed $\eta\in C_c^{\infty}((W_1)_T)$. Let us observe that the UCP for the fractional Laplacian and the condition \eqref{eq: equality of DN maps semilinear} imply that $u_\eps\vcentcolon = u^{(1)}_\eps=u^{(2)}_\eps$. Next, let us note that we can write \begin{equation} \label{eq: decomposition of u eps} u_\eps=\eps v_j+R^{(j)}_\eps \end{equation} for $j=1,2$, where $v_j$ and $R^{(j)}_\eps$ are the unique solutions of \begin{equation} \label{eq: linear part of u eps} \begin{cases} L_{\gamma_j}v = 0 & \text{ in } \Omega_T,\\ v =\eta & \text{ on } (\Omega_e)_T,\\ v(0) = 0,\, \partial_{t}v(0) = 0 & \text{ on } \Omega \end{cases} \end{equation} and \begin{equation} \label{eq: nonlinear part of u eps} \begin{cases} L_{\gamma_j}R = -f_j(u_\eps) & \text{ in } \Omega_T,\\ R =0 & \text{ on } (\Omega_e)_T,\\ R(0) = 0,\, \partial_{t}R(0) = 0 & \text{ on } \Omega, \end{cases} \end{equation} respectively. This simply follows from the unique solvability of \eqref{eq: PDE for semilinear problem} and both functions $u_\eps$ and $\eps v_j+R^{(j)}_\eps$ are solutions. Furthermore, we notice that the energy estimate of \cite[Theorem 3.1]{Semilinear-nonlocal-wave-eq}, \cite[eq.~(3.18)]{Semilinear-nonlocal-wave-eq} and the $r+1$ homogeneity of $f_j$ ensure that $R^{(j)}_\eps$ satisfies \begin{equation} \label{eq: energy estimate remainder} \begin{split} \|\partial_t R^{(j)}_\eps\|_{L^{\infty}(0,T;L^2(\Omega))}+\|R^{(j)}_\eps (t)\|_{L^{\infty}(0,T;H^s(\R^n))}&\lesssim \|f_j(u_\eps)\|_{L^2(\Omega_T)}\\ &\lesssim \|u_\eps\|^{r+1}_{L^{\infty}(0,T;H^s(\R^n))}. \end{split} \end{equation} Moreover, we may estimate \begin{equation} \label{eq: estimate u eps} \begin{split} &\|\partial_t u_\eps\|_{L^{\infty}(0,T;L^2(\R^n))}+\|u_\eps\|_{L^{\infty}(0,T;H^s(\R^n))}\\ &\lesssim \|\partial_t (u_\eps-\eps\eta)\|_{L^{\infty}(0,T;L^2(\Omega))}+\|u_\eps-\eps\eta\|_{L^{\infty}(0,T;H^s(\R^n))}+\eps\|\eta\|_{W^{1,\infty}(0,T;H^{2s}(\R^n))}\\ &\lesssim \eps\|\eta\|_{W^{1,\infty}(0,T;H^{2s}(\R^n))}. \end{split} \end{equation} This follows from the following observations. If $u$ solves \eqref{eq: PDE for semilinear problem} for a damping coefficient $\gamma\in C^{0,\alpha}(\R^n)$, a weak nonlinearity $f$ and $\varphi\in C_c^{\infty}((\Omega_e)_T)$, then $v=u-\varphi$ solves \begin{equation} \begin{cases} L_{\gamma}v+f(v) = -(-\Delta)^s\varphi & \text{ in } \Omega_T,\\ v =0& \text{ on } (\Omega_e)_T,\\ v(0) = 0,\, \partial_{t}v(0) = 0 & \text{ on } \Omega. \end{cases} \end{equation} Now, we may invoke \cite[eq.~(3.15)]{Semilinear-nonlocal-wave-eq} to find that there holds \[ \begin{split} &\|\partial_t v(t)\|_{L^2(\Omega)}^2+\|v(t)\|_{H^s(\R^n)}^2\\ &\lesssim \int_0^t|\langle \gamma\partial_t v,\partial_t v\rangle_{L^2(\Omega)}|\,d\tau +\int_0^t|\langle (-\Delta)^{s}\varphi,\partial_t v\rangle_{L^2(\Omega)}|\,d\tau\\ &\lesssim \|(-\Delta)^{s}\varphi\|_{L^2(0,t;L^2(\Omega))}^2+\int_0^t\|\partial_t v\|_{L^2(\Omega)}^2\,d\tau. \end{split} \] Thus, Gronwall's inequality gives \[ \|\partial_t v(t)\|_{L^2(\Omega)}+\|v(t)\|_{H^s(\R^n)}\lesssim \|(-\Delta)^{s}\varphi\|_{L^2(0,t;L^2(\Omega))}. \] This ensures the validity of the second estimate in \eqref{eq: estimate u eps}. Next, observe that by subtracting the PDEs for $u^{(1)}_\eps$ and $u^{(2)}_\eps$, we deduce that \begin{equation} \label{eq: condition for uniqueness semilinear} (\gamma_1-\gamma_2)\partial_t u_\eps=f_2(u_\eps)-f_1(u_\eps)\text{ in }\Omega_T. \end{equation} By \eqref{eq: decomposition of u eps}, we may write \begin{equation} \label{eq: condition for uniqueness semilinear 2} (\gamma_1-\gamma_2)(\eps\partial_tv_1+\partial_t R^{(1)}_\eps) =f_2(u_\eps)-f_1(u_\eps)\text{ in }\Omega_T. \end{equation} Combining \eqref{eq: energy estimate remainder} and \eqref{eq: estimate u eps}, we see that \begin{equation} \label{eq: decay estimate R eps} \|\partial_t R^{(j)}_\eps\|_{L^{\infty}(0,T;L^2(\Omega))}+\|R^{(j)}_\eps (t)\|_{L^{\infty}(0,T;H^s(\R^n))}\lesssim \eps^{r+1}. \end{equation} Multiplying \label{eq: condition for uniqueness semilinear 2} by $\eps^{-1}$ gives \begin{equation} \label{eq: condition for uniqueness semilinear 3} \begin{split} &(\gamma_1-\gamma_2)(\partial_tv_1+\eps^{-1}\partial_t R^{(1)}_\eps) =f_2(\eps^{-1/(r+1)}u_\eps)-f_1(\eps^{-1/(r+1)}u_\eps)\text{ in }\Omega_T. \end{split} \end{equation} Next, let us focus one the case $2s<n$ as the other one can be treated similarly. As $r>0$ we deduce from \eqref{eq: estimate u eps} that $\eps^{-1/(1+r)}u_\eps\to 0$ in $L^{\infty}(0,T;H^s(\R^n))$ and so by Sobolev's embedding in $L^q(0,T;L^{2_s^*}(\Omega))$ for all $1\leq q\leq \infty$ and $2_s^*=\frac{2n}{n-2s}$. Hence, by our assumptions on $f_j$ and \cite[Lemma 3.6]{zimmermann2024calderon}, we get \begin{equation} \label{eq: zero convergence nonlinearity} f_j(\eps^{-1/(r+1)}u_\eps)\to 0\text{ in }L^{q/(r+1)}(0,T;L^{2_s^*/(r+1)}(\Omega)) \end{equation} for all $q\geq r+1$ as $\eps\to 0$. Additionally, using \eqref{eq: decay estimate R eps} we know that \begin{equation} \label{eq: zero convergence time derivative remainder} \eps^{-1}\partial_t R^{(j)}_\eps\to 0\text{ in }L^{\infty}(0,T;L^2(\Omega)). \end{equation} Therefore, from \eqref{eq: condition for uniqueness semilinear 3}, \eqref{eq: zero convergence nonlinearity} and \eqref{eq: zero convergence time derivative remainder}, we infer \[ (\gamma_1-\gamma_2)\partial_t v_1=0\text{ in }\Omega_T. \] In particular, this ensures that there holds \[ \int_{\Omega_T}(\gamma_1-\gamma_2)\partial_t (v_1-\eta)(w_2-\psi)^\star\,dxdt=0 \] for any $\psi \in C_c^{\infty}((W_2)_T)$, where $w_2$ is the unique solution of \begin{equation} \begin{cases} L_{\gamma_2}w = 0 & \text{ in } \Omega_T,\\ w =\psi & \text{ on } (\Omega_e)_T,\\ w(0) = 0,\, \partial_{t}w(0) = 0 & \text{ on } \Omega. \end{cases} \end{equation} Now, arguing as in the previous section, we get $\gamma_1=\gamma_2$ in $\Omega$. Hence, \eqref{eq: condition for uniqueness semilinear} reduces to \begin{equation} f_1(u_\eps)=f_2(u_\eps)\text{ in }\Omega_T. \end{equation} Multiplying this identity by $\eps^{-(r+1)}$ and arguing as before, we deduce that \[ f_1(v)=f_2(v)\text{ in }\Omega_T, \] where $v\vcentcolon =v_1=v_2$ as $\gamma_1=\gamma_2$. One can now show $f_1(x,\tau)=f_2(x,\tau)$ for all $x\in\Omega$ and $\tau\in\R$ exactly as described in \cite[p.~29]{Optimal-Runge-nonlocal-wave}. Hence, we can conclude the proof. \end{proof} \medskip \noindent\textbf{Acknowledgments.} P.~Zimmermann was supported by the Swiss National Science Foundation (SNSF), under grant number 214500. \section*{Statements and Declarations} \subsection*{Data availability statement} No datasets were generated or analyzed during the current study. \subsection*{Conflict of Interests} Hereby we declare there are no conflict of interests. \bibliography{refs} \bibliographystyle{alpha} \end{document}
2412.02042v1
http://arxiv.org/abs/2412.02042v1
Delta invariants of plumbed 3-manifolds
\documentclass[11pt,a4paper, reqno,dvipsnames]{amsart} \usepackage[utf8]{inputenc} \usepackage{amsfonts, amsmath,amsthm, amssymb} \usepackage{enumerate} \usepackage{rotating} \usepackage{tikz} \usetikzlibrary{decorations.pathreplacing} \usepackage{graphicx} \usepackage{mathtools} \usepackage{comment} \usepackage[giveninits=true]{biblatex} \renewbibmacro{in:}{} \addbibresource{bibliography.bib} \usepackage[mode=buildnew]{standalone} \usepackage{geometry} \geometry{left=35mm,right=35mm,top=42mm,bottom=44mm} \usepackage{hyperref} \hypersetup{ colorlinks=true, linkcolor=Violet, filecolor=magenta, urlcolor=cyan, citecolor=Violet, pdftitle={Delta invariants of plumbed manifolds}, } \usepackage[capitalise]{cleveref} \usepackage{mymacros} \title{$\Delta$ invariants of plumbed manifolds} \author{Shimal Harichurn, Andr\'as N\'emethi, Josef Svoboda} \date{\today} \begin{document} \maketitle \begin{abstract} We study the minimal $q$-exponent $\Delta$ in the BPS $q$-series $\widehat{Z}$ of negative definite plumbed 3-manifolds equipped with a spin$^c$-structure. We express $\Delta$ of Seifert manifolds in terms of an invariant commonly used in singularity theory. We provide several examples illustrating the interesting behaviour of $\Delta$ for non-Seifert manifolds. Finally, we compare $\Delta$ invariants with correction terms in Heegaard--Floer homology. \end{abstract} \tableofcontents \section{Introduction} \(\Zhat_b(Y; q)\) is a $q$-series invariant of a negative definite plumbed 3-manifold \(Y\) equipped with a \(\spc\) structure \(b\) \cite{GPPV20}. It recovers Witten--Reshetikhin--Turaev $U_q(\mathfrak{sl}_2)$-invariants in radial limits to roots of unity as conjectured by \cite{GPPV20}, and recently proved in \cite{Mur23}. In this paper, we focus on the behavior of \(\Zhat_b(Y; q)\) near \(q=0\). In other words, we study the smallest \(q\)-exponents \(\Delta_b\) in \(\Zhat_b(Y,q)\): \[ \Zhat_b(Y,q) = q^{\Delta_b}(c_0+c_1 q + c_2 q^2 + \dots), \quad c_0 \neq 0. \] The rational numbers $\Delta_b$ were studied in \cite{GPP21} where their fractional part was related to various invariants of 3-manifolds. In this work, we focus on the actual value of $\Delta_b$. For plumbed manifolds it is natural to consider the `canonical' $\spc$ structure $can$. Related to it, there is a numerical topological invariant \(\gamma(Y) := k^2+s\) (see \ref{ss:spinc}). It appears, e.g., in the study of Seiberg--Witten invariants \cite{NeNi02} of the associated plumbed 3-manifold, and its use in topology goes back to Gompf \cite{Gompf98}. It also plays an important role in singularity theory, e.g. in Laufer's formula \cite{Laufer1977} for Milnor number of a Gorenstein normal surface singularity. As the main result of this paper, we prove that for Seifert manifolds, \(\Delta_{can}(Y)\) can be expressed using $\gamma(Y)$: \begin{theorem}\label{thm:seifert_hom} Let $Y = M(b_0;(a_1, \omega_1), \dots, (a_n, \omega_n))$ be a Seifert manifold associated with negative definite plumbing graph. Let $\emph{can}$ be the canonical $\spc$ structure of $Y$. Then $\Delta_{can}$ satisfies \begin{equation} \Delta_{can} = -\frac{\gamma(Y)}{4} + \frac{1}{2}. \end{equation} If $Y$ is not a lens space, then $\Delta_{can}$ is minimal among all ${\Delta_b, b \in \spc(Y)}$. \end{theorem} In the special case of Seifert homology spheres $Y=\Sigma(a_1,a_2,\dots,a_n)$, we deduce that $\Delta_{can}(Y)$ grows polynomially with the leading term given by \[ \Delta_{can}(Y) \approx (n-2)a_1 a_2 \cdots a_n. \] The idea of the proof is to express $\Delta_{can}$ as a minimum of a quadratic form, given by the linking form of the plumbing graph, over certain integral vectors. The key is identifying this set over which we minimize (\cref{lm:delta_minimizer}). We show that in this set, there exists a unique vector (up to sign) that minimizes the quadratic form. Once we leave Seifert manifolds, the computation of \(\Delta_b\) invariants becomes much more complicated. We illustrate this on plumbing graphs with exactly two vertices of degree 3 and no vertices of degree $\geq 4$ in \cref{s:H_shaped}. In this case, \(\Delta_{can}\) is often smaller than $-\gamma(Y)/4 + 1/2$, because we have a larger freedom in finding minimizing vectors. To analyze $\Delta$ of these graphs, we use splice diagrams \cite{Sie80,NeumannWallBook86,NW05a} of plumbed manifolds, building on \cite{GKS23}. Surprisingly, $\Delta_{can}$ can also be larger than $-\gamma(Y)/4 + 1/2$, as a result of interesting cancellations in the formula for $\Zhat_b(Y)$, see \cref{ex:cancel_H}. Namely, $\Zhat_b(Y)$ can be expressed as a weighted sum over certain lattice points and those can sometimes be organized in pairs with weights of opposite signs. This can be avoided by refining the weights, as provided by the two-variable series $\Zhathat_b(q,t)$ defined in \cite{AJK23}. However, \cref{ex:t_cancel_0} shows that cancellations may occur even in $\Zhathat_b(q,t)$. An analogy between $\Delta_b(Y)$ invariants and the correction terms $d_b(Y)$ in Heegaard--Floer homology was proposed in \cite{GPP21}. In \cite{harichurn2023deltaa}, the first author demonstrated on Brieskorn spheres that unlike $d_b(Y)$, $\Delta_b(Y)$ are not cobordism invariants. Using the explicit formula for $\Delta_{can}$ of Seifert manifolds given by \cref{thm:seifert_hom}, we compare $\Delta_{can}$ and $d_{can}$ for some classes of Brieskorn spheres, where $d_{can}$ is explicitly known \cite{BorNem2011}. We find that $\Delta_{can}$ is generically much larger than $d_{can}$. The reason for this discrepancy is that although $\Delta_{can}$ and $d_{can}$ are both minima of certain closely related quadratic forms, for $\Delta_{can}$, the quadratic form is being minimized over a much smaller set of vectors than the one for $d_{can}$. \subsection*{Acknowledgement} We are grateful to Sergei Gukov, Mrunmay Jagadale and Sunghyuk Park for useful discussions. A. N\'emethi was partially supported by ``\'Elvonal (Frontier)'' Grant KKP 144148. J. Svoboda was supported by the Simons Foundation Grant {\it New structures in low-dimensional topology}. S. Harichurn was supported by the 2020 FirstRand FNB Fund Education Scholarship Award. \section{Preliminaries} In this section, we collect the necessary material on plumbed manifolds and \(\spc\) structures on them. \subsection{Plumbed 3-manifolds} Let $\Gamma = (V,E,m)$ be a finite tree with the set of vertices $V$ and edges $E$, and a vector \(m \in \ZZ^{\lvert V \rvert}\) consisting of integer labels \(m_v\) for each vertex \(v \in V\). Let $s = \lvert V\rvert$ and let $\delta = (\delta_v)_{v \in V} \in \ZZ^s$ be the vector of the degrees (valencies) of the vertices. We often implicitly order vertices of \(V\), so that \(V = {v_1,v_2,\dots,v_s}\) and write the quantities associated to \(v_i\) with subscript \(i\). We can record \(\Gamma\) using $s\times s$ \emph{plumbing matrix} \(M=M(\Gamma)\), defined by \[ M_{vw} = \begin{cases} m_v & \text{if} \quad v = w \\ 1 & \text{if} \quad (v, w) \in E \\ 0 & \text{otherwise.} \end{cases} \] We always assume that \(M\) is a negative definite matrix. For a vector $l \in \Z^s$ we write \[ l^2 = l^T M^{-1}l. \] Note that the quadratic form \(l \mapsto -l^2\) is a positive definite. From \(\Gamma\), we can construct a closed oriented 3-manifold $Y := Y(\Gamma)$ by \emph{plumbing} \cite{Neu81}: For each vertex \(v\) we consider a circle bundle over \(S^2\) with Euler number \(m_v\). Then we glue the bundles together along tori corresponding to the edges in \(E\). Manifolds given by this construction, with \(M\) negative definite, are called \emph{negative definite plumbed manifolds}. From the construction above, it follows that \(Y\) is a \emph{rational homology sphere}, that is, \(H_1(Y) = H_1(Y,\ZZ)\) is finite. We have \(H_1(Y) \cong \ZZ^s/M \ZZ^s\) and $\lvert H_1(Y)\rvert=\det M$. The quadratic form $l \mapsto -l^2$ takes values in $\lvert H\rvert^{-1}\ZZ$ as the adjugate matrix $\adj M = (\det M) M^{-1}$ of $M$ has integer entries. \subsection{\texorpdfstring{\(\Spc\)}{Spinc} structures}\label{ss:spinc} The set $\spc(Y)$ of \(\spc\) structures on $Y$ admits a natural free and transitive action of \(H_1(Y)\), hence it is finite. It can be identified with the set $(2\ZZ^s + m)/2M\ZZ^s$ of \emph{characteristic vectors}. For us, it is convenient to use another identification, with \(\spc(Y) \cong (2\ZZ^s + \delta)/2M\ZZ^s\), obtained from the usual characteristic vectors via the map \(l \to l - Mu\), where \(u = (1, 1, \dots, 1)\). This is justified by the identity \( \delta + m = Mu\). For a vector $b \in 2\ZZ^s + \delta,$ we denote the corresponding $\spc$ structure as $[b] \in (2\ZZ^s + \delta)/2M\ZZ^s$. We often omit the brackets when it is clear from the context, e.g. we write $\Zhat_b$ for $\Zhat_{[b]}$. We consider the vector $2u - \delta$ and the corresponding `canonical' $\spc$ structure $can = [2u-\delta]$. Its characteristic vector is \(k = 2u-\delta + Mu = m + 2u\). The rational number \(\gamma(Y) := k^2+s = (2u-\delta +Mu)^2+s\) does not depend on the plumbing representation of $Y$, so it is a topological invariant of $Y$. Denote by $\Tr(M) = \sum_{v \in V}m_v$ the trace of the plumbing matrix $M$. Then $\gamma(Y)$ can be expressed as follows: \begin{proposition}[\cite{NeNi02}]\label{base-k2-s} Let $Y:=Y(\Gamma)$ be a negative definite plumbed manifold, which is a rational homology sphere. Then \begin{equation} \begin{split} \gamma(Y) &= 3s + \Tr(M) + 2 + \sum_{v,w \in V} (2-\delta_v)(2-\delta_w)M_{vw}^{-1}\\ &=3s + \Tr(M) + 2 + (2u-\delta)^2. \end{split} \end{equation} \end{proposition} \subsection{The \texorpdfstring{\(\Zhat_b\)}{Zhat} invariants} For the rest of the paper, let \(Y = Y(\Gamma)\) be a negative definite plumbed rational homology sphere with a chosen negative definite plumbing matrix \(M\) of $\Gamma$. \begin{definition} Let \(b \in 2\ZZ^s + \delta\) be a vector representing a $\spc$ structure $[b]$. For $\lvert q \rvert<1$, the GPPV $q$-series $\Zhat_b(Y;q)$ is defined by \begin{equation}\label{def:zhat} \Zhat_b(Y;q) = q^{\frac{-3s - \Tr(M)}{4}} \vp\oint\limits_{\lvert z_i \rvert = 1}\prod_{v_i\in V} \frac{dz_i}{2\pi i z_i} \left(z_i - \frac{1}{z_i}\right)^{2-\delta_i} \Theta_{b}(z), \end{equation} where \begin{equation}\label{eq:theta} \Theta_{b}(z) = \sum_{l \in 2M\ZZ^s + b} z^l q^{-\frac{l^2}{4}}, \end{equation} and $\vp$ denotes the principal part of the integral. \end{definition} We often omit the manifold $Y$ when it is clear from the context, e.g. we write $\Zhat_b(q)$ for $\Zhat_b(Y,q)$. Clearly, $\Zhat_b(q)$ does not depend on the choice of $b \in [b]$. The convergence of $\Zhat_b(Y)$ for $\lvert q \rvert<1$ follows from the negative definiteness of \(M\). \section{\texorpdfstring{$\Delta_b$}{Delta} invariants} \begin{definition} The \emph{Delta invariant} \(\Delta_b\) is the smallest $q$-exponent in \(\Zhat_b(q)\). If $\Zhat_b(q) = 0$, we set \(\Delta_b = \infty\). \end{definition} Using $\Delta_b$, we can write the $\Zhat_b(q)$ in the following form: \[ \Zhat_b(q) = 2^{-s} q^{\Delta_b}\sum_{i=0}^\infty r_iq^i = 2^{-s} q^{\Delta_b}(r_0 + r_1q^1 + r_2q^2 + \cdots) \] for some integers $r_i$ with $r_0 \neq 0$. This is justified by part (1) of the following Lemma (recall that $\lvert H\rvert$ is the order of $H_1(Y,\ZZ)$): \begin{lemma}\label{delta-h-mod-1} \begin{enumerate} \item The differences between the exponents of \(\Zhat_b(q)\) are integers. \item The fractional part of the exponents (and in particular of \(\Delta_b\)) is given by \[ \frac{-3s - \Tr(M) -b^2}{4} \Mod{1}. \] Consequently $4 \lvert H \rvert \Delta_b \in \ZZ \cup \{\infty\}$. \item $\lvert H\rvert\Delta_a \equiv \lvert H\rvert\Delta_{b} \Mod{1}$ for any two $\spc$ structures $[a],[b]$ on $Y$. \end{enumerate} \end{lemma} \begin{proof} \begin{enumerate} \item Consider two vectors \(l, l+2Mn \in [b]\), where $l,n \in \ZZ^s$. The difference of the corresponding exponents is \[ \frac{-l^2}{4}+\frac{(l+2Mn)^2}{4} = l^Tn + n^T Mn \in \ZZ. \] \item The exponent of a representative $b \in [b]$ reads \[ \frac{-3s - \Tr(M) - b^2}{4} \in \frac{1}{4\lvert H\rvert}\ZZ \] By (1), all the other exponents have the same fractional part. \item We have \[ \lvert H\rvert\Delta_b(Y) - \lvert H\rvert\Delta_{a}(Y) \equiv \frac{\lvert H\rvert(b^2 -a^2)}{4} \Mod{1}. \] By writing $b = 2l + \delta$ and $a = 2n+\delta$ with $l,n \in \ZZ$, we see that \[ \frac{1}{4}\lvert H\rvert (b^2 - a^2) = \lvert H\rvert (l^2 + l^TM^{-1}\delta - n^2 + n^TM^{-1}\delta) \in \ZZ. \] \end{enumerate} \end{proof} \begin{remark} The set of $\spc$ structures admits a natural involution, called conjugation, denoted by $b \mapsto \bar{b}$. It is known that \(\Zhat_b(q) = \Zhat_{\bar{b}}(q)\), hence we have \(\Delta_b =\Delta_{\bar{b}}\). \end{remark} \subsection{The exponents of \texorpdfstring{$\Zhat_b$}{Zhat}}\label{computation-via-set-c} The \(\Delta_b\) invariant is the minimal $q$-exponent in the series \(\Zhat_b(q)\). The exponents are, up to an overall shift, given by the quadratic form \(l \mapsto -l^2\). Here $l$ are lattice vectors that run over a certain subset \(\LC_b \subset \ZZ^s\). We need to identify this subset. We order the set of vertices \(V\) of the plumbing tree $\Gamma$ by their degree: \[ V = \{v_1, \dots, v_{s_1}, v_{s_{1}+1}, \dots, v_{s_{1}+s_{2}}, v_{s_{1}+s_{2}+1}, \dots, v_{s_{1}+s_{2} + s_{3} = s}\}. \] We have \(s_1\) leaves, \(s_2\) vertices of degree 2 and \(s_3\) vertices of degree \(\geq 3\). The integrand of \(\Zhat\) contains the rational function \begin{equation}\label{eq:rat_function} \prod_{i=1}^s(z_i- z_i^{-1})^{2-\delta_i} = \prod_{i=1}^{s_1} (z_i-z_i^{-1}) \prod_{i=s_1+s_2+1}^{s} \frac{1}{(z_i-z_i^{-1})^{\delta_i-2}}. \end{equation} The integration in \eqref{def:zhat} is equivalent to the following procedure: First, we expand each term of the product above using the \emph{symmetric expansion}---the average of Laurent expansions as $z_i \rightarrow 0$ and $z_i \rightarrow \infty$, and multiply these together, giving an element of $\Z[z_1^{\pm 1},\dots, z_s^{\pm 1}]$: \begin{equation}\label{eq:rat_function_expn} \prod_{i=1}^s \se (z_i- z_i^{-1})^{2-\delta_i} = \sum_{l \in \ZZ^s} \ct_l z^l = \sum_{l \in \ZZ^s} \ct_l z_1^{l_1}z_2^{l_2}\cdots z_s^{l_s}. \end{equation} Then we multiply the result with the theta function $\Theta_b(z)$ in \eqref{eq:theta}, and we extract the constant coefficient in variables $z_i$, giving the $q$-series $\Zhat_b(q)$. Let \(\LCt\) denote the set of all the vectors \(l \in \ZZ^s\) with nonvanishing coefficient $\ct_l \neq 0$ in \eqref{eq:rat_function_expn}. Similarly, define $\LCt_b = \LCt \cap -(2\ZZ^s+b)$. The reason for the sign is that we are pairing $l \in \LCt$ with $-l \in 2\ZZ^s+b$ when extracting the constant coefficient. Note that while \(\LCt\) is symmetric about the origin, $2\ZZ^s+b$ in general is not. \begin{lemma}\label{description-c} The set $\LCt$ consists of vectors whose components satisfy the following conditions: \[\LCt = \left\{(l_1,\dots,l_{s_1}, 0, \dots 0, m_1, \dots, m_{s_3}) \in \ZZ^s \;\middle\vert\; \begin{aligned} &l_i = \pm 1 \\ &m_i \equiv \delta_i \Mod{2} \\ &\lvert m_i \rvert \geq \delta_i-2 \end{aligned} \right\}. \] \end{lemma} \begin{proof} The expansion for 1-vertices is simply \(z_i-z_i^{-1}\), giving the entries \(l_i = \pm 1\). The variables corresponding to 2-vertices are absent in \ref{eq:rat_function_expn}. For a vertex \(v_i\) of degree \(\delta_{i} \geq 3\), put \(d := \delta_{i}-2 \geq 1\). We have the following symmetric expansion: \begin{align*} 2 \cdot \se (z-z^{-1})^{-d} &= \underset{z \to \infty}{\expn} \frac{z^{-d}}{(1-z^{-2})^{d}} + \underset{z \to 0}{\expn} \frac{z^{d}}{(z^2-1)^{d}}\\ &= z^{-d} \sum_{k=0}^\infty \binom{k-1+d}{k}z^{-2k} + (-1)^d z^{d} \sum_{k=0}^\infty \binom{k-1+d}{k} z^{2k}\\ &= (z^{-d} + d z^{-d-2} + \dots) + (-1)^{d} (z^{d} + d z^{d+2} +\dots) \end{align*} It follows that the corresponding entry \(m_i\) must have the same parity as \(\delta_i\) and \(\lvert m_i \rvert \geq d = \delta_i-2\). \end{proof} \subsection{Cancellations}\label{ss:cancel} By the previous section, the $q$-series \(\Zhat_b(Y,q)\) can then be expressed as a sum over $\LCt_b$: \begin{equation}\label{eq:before_cancel} \Zhat_b(Y,q) = q^{\frac{-3s - \Tr(M)}{4}} \sum_{l \in \LCt_b} \ct_l q^{\frac{-l^2}{4}}. \end{equation} The $q$-exponents of $\Zhat_b(q)$ are therefore given by $( -3s - \Tr(M) -l^2)/4$ for those $l \in \LCt_b$ whose contribution does not cancel out in the sum above, in other words: \begin{equation}\label{eq:cancel} c_l:= \sum_{\substack{l' \in \LCt_b\\l'^2 = l^2}} \ct_{l'} \neq 0. \end{equation} However, $c_l$ can vanish even if there are more nonzero terms $\tilde{c}_l$ is \eqref{eq:cancel}. We refer to this phenomenon as `cancellations' -- see Examples \ref{ex:cancel_H} and \ref{ex:cancel_seifert}. Motivated by this, we define \[ \LC_b := \{l \mid l \in \LCt_b; l \text{ satisfies \eqref{eq:cancel}} \} \subseteq \LCt_b. \] It follows that $\Delta_b$ is determined by minimizing $-l^2$ over the set $\LC_b$. We refer to the elements $l$ in $C_b$ for which $-l^2$ is minimal, as \emph{minimizing vectors}. \begin{lemma}\label{lm:delta_minimizer} \begin{equation} \Delta_b(Y) = \frac14 \left(-3s - \Tr(M) + \min_{l \in \LC_b} \{ -l^2 \}\right). \end{equation} \end{lemma} \section{$\Delta$ for Seifert Manifolds} In this section, we will prove \cref{thm:seifert} which gives an explicit formula for $\Delta_{can}(Y)$ of a Seifert manifold $Y$ equipped with the canonical $\spc$ structure $can$. \subsection{Seifert manifolds} Seifert manifold $Y = M(b_0;(a_1, \omega_1), \dots, (a_n, \omega_n))$, fibered over $S^2$, is given by an integer $b_0$ and tuples of integers $0<\omega_i<a_i$ for $1 \leq i \leq n$. It can be represented by a star-shaped plumbing as shown in \cref{fig:seifert}. The graph consists of a central vertex with label $-b_0$ and $n$ `strings'. The labels $-b_{j_1},-b_{j_2},\dots, -b_{j_{s_j}}$ on the $j$-th string are determined by Hirzebruch--Jung continued fraction \[\frac{a_j}{\omega_j} = [b_{j_1}, \dots, b_{j_{s_j}}] =b_{j_1} - \cfrac{1}{b_{j_2} - \cfrac{1}{\cdots - \cfrac{1}{b_{j_{s_j}}}}}. \] \begin{figure} \centering \includestandalone{graphics/seifert/new_seifert_plumbing} \caption{Plumbing graph of a Seifert manifold.} \label{fig:seifert} \end{figure} The intersection form $M$ is negative definite if and only if the orbifold Euler number $e = -b_0 + \sum_i \omega_i/a_i$ is negative. We have the following formula for $\gamma(Y)$ of a Seifert manifold $Y$ \cite[p. 296]{NeNi02}, a consequence of \cref{base-k2-s}: \begin{equation}\label{eq:formula_gamma} \gamma(Y) = \frac{1}{e}\left(2-n + \sum_{i=1}^n \frac{1}{a_i}\right)^2 + e + 5 + 12 \sum_{i=1}^n \ds(\omega_i, a_i). \end{equation} Here $\ds$ denotes the Dedekind sum. \subsection{\texorpdfstring{$\Delta_{can}$}{Delta} of Seifert manifolds} \begin{theorem}\label{thm:seifert} Let $Y = M(b_0;(a_1, \omega_1), \dots, (a_n, \omega_n))$ be a negative definite Seifert manifold. Then $\Delta_{can}$ of the canonical $\spc$ structure satisfies \begin{equation}\label{eq:seifert} \Delta_{can} = -\frac{\gamma(Y)}{4} + \frac{1}{2}. \end{equation} If $Y$ is not a lens space, then $\Delta_{can}$ is minimal among all ${\Delta_b, b \in \spc(Y)}$. \end{theorem} \begin{proof} We will treat the lens spaces separately in \cref{ssec:lens}, so we may assume that $n \geq 3$ and \(a_i \geq 2\) for each \(i\). Denote $A = \prod_{i=1}^n a_i$. We will first show that over the set $\LCt$ from \cref{description-c}, \(-l^2\) is minimized by exactly be vectors $\pm(2u-\delta)$. From that, it will follow that $2u-\delta \in \LC_{can}$ is a true minimizing vector in the sense of \cref{ss:cancel}. The formula \eqref{eq:seifert} for $\Delta_{can}$ then follows from \cref{base-k2-s}. The set $\LCt$ consists of the vectors \[ l = (l_1, l_2, \dots, l_n, 0, \dots, 0, m) \] where $l_i = \pm 1$ and $m \equiv n \Mod{2}$ and $\lvert m \rvert \geq n-2$ by \cref{description-c}. The quadratic form \(l^2\) can be expressed using Seifert data as follows: \begin{equation}\label{eq:quadratic} -l^2 = m^2 A + \sum_{i=1}^n l_i m \frac{A}{a_i} + \sum_{i \neq j}^n l_i l_j \frac{A}{a_ia_j} - \sum_{i=1}^n M^{-1}_{ii}. \end{equation} By the symmetry \(l^2 = (-l)^2\), we may assume that $m \geq n-2$. Taking the derivative with respect to \(m\), we obtain \[ \frac{1}{A} \frac{\partial}{\partial m}\left(-l^2\right) = 2m + \sum_{i=1}^n \frac{l_i}{a_i} \geq 2(n-2) - \frac{n}{2} > 0, \] so the minimum is attained when \(\lvert m \rvert = n-2.\) Similarly, pick \(j \in 1,\dots, s_1.\) Let \(l^+, l^-\) be two vectors with \(m = n-2\) that only differ by having \(l_j^+=1\), \(l^-_j=-1\), respectively. We have \begin{align*} l_+^2 - l_-^2 & = \frac{2A}{a_j} (n-2 + \sum_{i \neq j} \frac{l_i}{a_i}) \\ & \geq \frac{2A}{a_j} (n-2 - \frac{n-1}{2}) \geq 0. \end{align*} It follows that \[ l = (-1,\dots, -1, 0, \dots, 0, n-2) = \delta - 2u. \] minimizes \eqref{eq:quadratic}. The argument also implies that the only other vector giving the same value of $l^2$ is $-l=2u-\delta$ which represents the canonical $\spc$ structure $can$. These two vectors have the same coefficients in the expansion of \eqref{eq:rat_function}: \[ c_l = c_{-l} = \frac12\operatorname{sgn}(n-2)^n \binom{\frac{n+\lvert n-2 \rvert}{2}-2}{n-3}. \] Therefore even in the case that $\pm l$ belong to the same $\spc$ structure, i.e. $can$ is $\spin$, their contributions do not cancel in $\Zhat_{can}(q)$ and consequently $-l \in \LC_{can}$. Thus the minimum of \(-l^2\) over \(\LC_{can}\) is given by $-(2u-\delta)^2$ and we have \begin{equation}\label{delta-cleaning-up} \Delta = \frac{-3s - \Tr(M)}{4} - \frac{(2u-\delta)^2}{4} = - \frac{\gamma(Y)}{4} + \frac{1}{2} \end{equation} The last equality follows the formula for \(\gamma(Y)\) in \cref{base-k2-s}. \end{proof} \subsection{Lens spaces}\label{ssec:lens} In this section, we compute $\Delta_b$ of lens spaces. Let \(Y= L(p,r) \) be a lens space with \(p>r>0\). Denote by \(g\) a generator of \(H_1(Y,\ZZ) \cong \ZZ/p\ZZ\) and $can$ the canonical \(\spc\) structure. We have the following formula for \(\Zhat_b(q)\) of lens spaces: \[ \sum_{i=0}^{p-1} \Zhat_{g^i can}(q)g^i = q^{3\ds(r,p)} \left((g^{-r-1}+1) q^{1/2p} - (g^{-r}+g^{-1}) q^{-1/2p}\right) \] Here \(\ds(r,p)\) denotes the Dedekind sum. From the formula, we read off the four finite \(\Delta_b\) invariants. \begin{equation}\label{eq:delta_lens} \Delta_{g^{-r-1}can} = \Delta_{can} = 3\ds(r,p) + \frac{1}{2p}, \quad \Delta_{g^{-r}can} = \Delta_{g^{-1}can} = 3\ds(r,p) - \frac{1}{2p}. \end{equation} \noindent \(\gamma(Y)\) can be described using Dedekind sums in the following manner \cite[p. 304]{NeNi02}: \[ \gamma(Y) = 2 - \frac{2}{p} - 12\ds(p,r). \] Comparing with \eqref{eq:delta_lens}, we obtain that \(\Delta_{can}\) satisfies the same formula as for the other Seifert manifolds: \[ \Delta_{can} = -\frac{\gamma(Y)}{4} + \frac12. \] Note that \eqref{eq:delta_lens} shows that for lens spaces, $\Delta_{can}$ is not minimal among all $\Delta_b$. See also \cref{ex:cancel_H}. \begin{remark} We originally proved \cref{thm:seifert} using the reduction theorem \cite[Thm. 4.2]{GKS23} which may be used to compute all $q$-exponents of $\Zhat_b(q)$ of Seifert manifolds. Later, we found a simpler argument presented here, which focuses on the smallest exponent. It also emphasizes the role of the vector $2u-\delta$, making the presence of the invariant $\gamma(Y)$ more transparent. \end{remark} \begin{remark} As we have seen above, \(\Delta_b\) invariants are often infinite. In \cite{GPPV20}, the authors conjectured, based on Physics considerations, the existence of a categorification of \(\Zhat_b(q)\), i.e. a doubly-graded cohomology theory \(\mathcal{H}_b^{i,j}(Y)\) whose graded Euler characteristic is \(\Zhat_b(q)\): \[ \Zhat_b(q) = 2^{-s} q^{\Delta_b} \sum_{i, j \in \ZZ} q^i (-1)^j \dim \mathcal{H}_b^{i,j}(Y). \] Smaller \(q\)-exponents than \(\Delta_b\) could appear in the corresponding two-variable generating series \(q^{\Delta_b}\sum_{i,j} q^i t^j \dim \mathcal{H}_b^{i,j}(Y)\). In \cite{GPV16}, a Poincar\'e series of this sort was defined for lens spaces \(L(p,1)\). Its minimal \(q\)-power is finite for all \(\spc\) structures, in contrast with \(\Delta_b\). \end{remark} \section{Beyond Seifert manifolds}\label{s:H_shaped} In the proof of \cref{thm:seifert} giving the formula for $\Delta_{can}$ of Seifert manifolds, we used a special form \eqref{eq:quadratic} of the quadratic from $l \mapsto -l^2$ following from the fact that Seifert manifolds admit a star-shaped graph. For more general graphs, we need a generalization of \eqref{eq:quadratic}. This is realized by some properties of \emph{splice diagrams}, which are certain weighted graphs built from plumbing graphs. We will illustrate this method on plumbing graphs with exactly two vertices of degree 3 and no vertices of degree 4 or more. The corresponding splice diagrams are ``H-shaped'' graphs with 6 vertices, as in \cref{fig:H_splice}. In general, there is no uniform choice of minimizing vector as was the vector $2u-\delta$ for $\Delta_{can}$ of Seifert manifolds. Therefore we cannot hope for a simple universal formula for $\Delta_{can}$ as in \cref{thm:seifert}. Nevertheless, the minimizing vectors keep a specific form in certain regions given by the relative size of the weights in splice diagram. On the boundaries of these regions, we may see multiple minimizing vectors. In principle, one can divide the study into those particular cases. Some of these can be effectively reduced to the case of Seifert manifolds, as we will illustrate in \cref{ss:computation_h_shaped}. The techniques described here can be used for more general plumbings, but the number of cases grows significantly with the complexity of the splice diagram. \subsection{Splice diagrams} Following \cite{NW05a} (see also \cite{SavelievBook}), given a plumbed manifold $Y$ with plumbing graph $\Gamma$, we construct a splice diagram $\Omega$ of $Y$ as follows: $\Omega$ is a tree obtained by replacing each maximal string in $\Gamma$ (a simple path in $\Gamma$ whose interior is open in $\Gamma$) by a single edge. Thus $\Omega$ is homeomorphic to $\Gamma$ but has no vertices of degree two. We identify the vertices of $\Omega$ with the corresponding vertices of $\Gamma$. At each vertex \(v\) of $\Omega$ of degree \(\geq 3\), we assign a weight $w_{v\varepsilon}$ on an incident edge $\varepsilon$ as follows. The edge $\varepsilon$ in $\Omega$ corresponds to a string in $\Gamma$ starting in $v$ with some edge $e$. Let $\Gamma_{ve}$ be the subgraph of $\Gamma$ cut off by the edge of $\Gamma$ at $v$ in the direction of $e$, as in the following picture. \begin{center} \includestandalone{graphics/H_shaped/explanation_subgraph} \end{center} The corresponding weight \(w_{v\varepsilon}\) is given by the determinant of \(-M(\Gamma_{ve})\), where \(M(\Gamma_{ve})\) denotes the intersection matrix of $\Gamma_{ve}$. We draw the weight \(w_{v \varepsilon}\) on the edge \(\varepsilon\) near \(v\). For example, the following is a plumbing graph and its splice diagram: \begin{center} \includestandalone{graphics/H_shaped/explanation_splice} \end{center} If $Y(\Gamma)$ is a homology sphere, the splice diagram uniquely determines the plumbing graph, see \cite{NW02}. Although $\Gamma$ and $\Omega$ are essentially equivalent in this case, $\Omega$ is much smaller. More importantly, it is the relative size of the weights of $\Omega$, which directly influences the minimizing vectors. For example, if one weights is significantly larger than the others, the minimizing vectors must be of some specific form, see \cref{ss:computation_h_shaped}. The values of the quadratic form $l \mapsto -l^2=-l^T M^{-1}l$ can be computed from the splice diagram as follows: For two vertices \(v\), \(v'\) of the splice diagram, consider the shortest path $P$ connecting them. Let \(N_{vv'}\) be the product of all weights adjacent to vertices of $P$, but not lying on $P$. \begin{center} \includestandalone{graphics/H_shaped/explanation_path} \end{center} \begin{theorem}[{\cite[Thm. 12.2]{NW05a}}]\label{theorem:neumann} With the notation above, we have \[ M^{-1}_{vv'} = -\frac{N_{vv'}}{\det M}. \] \end{theorem} Recall that the vectors $l \in \Z^s$ that contribute to $\Zhat_b(q)$ lie in the set $\LCt$ from \cref{description-c}. If $l \in \LCt$, we have $l_v=0$ if $v$ is of degree 2 and $l_v^2 = (\pm 1)^2=1$ if $v$ is of degree 1. We obtain the following expression for $-l^2$ generalizing \eqref{eq:quadratic}: \begin{equation}\label{eq:quadr_form_restricted} -l^2 = \sum_{\substack{v \neq v'\\ \delta_v,\delta_{v'} \neq 2}} M^{-1}_{vv'} l_v l_{v'} + \sum_{\delta_v \geq 3} M^{-1}_{vv} l_v^2 + \sum_{\delta_v =1} M^{-1}_{vv} \end{equation} Note that the coefficients in the first and second sum can be expressed using the weights of the splice diagram $\Omega$, up to the multiplication by $\det M$. On the other hand, the third sum is not expressed in terms of weights of $\Omega$ in a simple way, but it is independent of $l$ (whenever $l \in \LCt$), so it does not influence which vectors $l \in \LCt$ have the minimal value of $-l^2$. \subsection{\texorpdfstring{$\Delta$}{Delta} for homology spheres with $H$-shaped splice diagrams} \label{ss:computation_h_shaped} We now consider a plumbing graph $\Gamma$ with exactly two vertices of degree 3 and no vertices of higher degree, e.g. the graph in \cref{fig:plumbing_from_splice}. For simplicity, we assume that the associated plumbed manifold $Y$ is a homology sphere. The associated splice diagram $\Omega$ is an `H-shaped' graph with six vertices, see \cref{fig:H_splice}. Its six weights are denoted \(a_1,a_2,a_3\) and \(a'_1,a'_2,a'_3\). They are pairwise coprime integers, which we further assume to be $\geq 2$. In this case, $Y$ can be realized as splicing of Brieskorn spheres $\Sigma(a_1,a_2,a_3)$ and $\Sigma(a'_1,a'_2,a'_3)$ along their third singular fibers. We consider the projection $\ZZ^s \to \ZZ^6$, denoted by $l \mapsto \bar{l}$, which removes the components corresponding to the vertices of degree 2. We order the components of $\bar{l} = (x_1,x_2,x_3,x'_1,x'_2,x'_3)$ as in \cref{fig:H_splice} and keep the ordering throughout this section. \begin{figure}[ht] \centering \includestandalone{graphics/H_shaped/splice_H_shaped} \caption{H-shaped splice diagram $\Omega$} \label{fig:H_splice} \end{figure} The quadratic form $l \mapsto -l^2$, restricted to the set $\LCt$ from \cref{description-c}, can be expressed in terms of the weights of $\Omega$ as: \begin{equation}\label{eq:form_H_shaped} \begin{split} -l^2 &= x_3^2 a_1 a_2 a_3 + x_1 x_3 a_2 a_3 + x_2 x_3 a_1 a_3 + x_1 x_2 a_3 + (\dots)' + \\ & + x_1 x'_2 a_2 a'_1 + x_1' x_3 a_1 a_2 a'_2 + x'_2 x_3 a_1 a_2 a'_1 + (\dots)' + \\ & + x_1 x'_1 a_2 a'_2 + x_2 x'_2 a_1 a'_1 + x_3x'_3a_1a_2a'_1a'_2 + C. \end{split} \end{equation} Here $(\dots)'$ means that we repeat the terms on each line with the usual and dashed variables reversed. \(C\) is constant on the set $\LCt$ so it does not influence the minimizing vectors for $\Delta_{can}$. The general strategy to identify $\Delta_{can}$ can be described as follows: We know that $x_1,x_2,x'_1,x'_2 \in \{\pm1\}$ because $l \in \LCt$. For each of the $2^4$ possibilities of the signs, the form above reduces to a quadratic form in variables $x_3$, and $x_3'$. We can then minimize these forms over odd integers, using standard optimization methods. Finally, we must check that the resulting vectors do not cancel out, as in \cref{ss:cancel}, so they are true minimizing vectors. Clearly, the form of minimizing vectors depends on the relative size of the coefficients \(a_i, a'_i\). We describe in greater detail the case when \(a_3\) is very large compared to other \(a_i\) and \(a'_i\). This allows us to effectively reduce the minimizing problem to the Seifert case. If $a_3 \gg a_1,a_2,a'_1,a'_2,a'_3$, the substantial terms are those containing \(a_3\): \begin{equation}\label{eq:half_graph} x^2_3 a_1 a_2 a_3 + x_1 x_3 a_2 a_3 + x_2 x_3 a_1 a_3 + x_1 x_2 a_3 \end{equation} Any vector $l \in \LCt$ with the minimal value of $-l^2$ also minimizes the expression \eqref{eq:half_graph}. As this is (almost) a quadratic form of a star-shaped graph, we can repeat the first part of the argument in the proof of \cref{thm:seifert}. Namely, assuming that $a_i \geq 2$ and taking the $x_3$-derivative, we obtain that \(x_3=\pm 1\) in any minimizing vector, say $x_3=1$. The signs of $x_1$ and $x_2$ are easily determined by the relative size of $a_1$ and $a_2$. This consideration `freezes' the left-hand side of the graph and for each choice of $x'_1 = \pm 1$ and $x'_2 = \pm 1$, we are left with a quadratic function of a single variable $x'_3$. Explicitly, in the case $x_1=x_2=-1$, the minimum in the variable $x'_3$ (over $\R$) is given by \begin{equation} \begin{split} x_3' &= \frac{x'_1 a'_2 a'_3 + x'_2 a'_1 a'_3 - a_1 a_2 a'_2 + a_1 a_2 a'_1 a'_2 - a_1 a_2 a'_1}{2a'_1 a'_2 a'_3}\\ &= \frac12 \left(\frac{x_1'}{a'_1}+\frac{x_2'}{a'_2}-\frac{a_1 a_2}{a'_1 a'_3}+\frac{a_1 a_2}{a'_2 a'_3}-\frac{a_1 a_2}{a'_3}\right). \end{split} \end{equation} This computation illustrates that the value of $x'_3$ can be arbitrarily large. This shows that the minimizing vector is very different from the vector $2u-\delta$ which was minimizing in the case of Seifert manifolds. In a similar way, we could analyze other cases of the relative size of the weights, giving (at least approximate) `explicit formulas' for $\Delta_{can}$. However, in practice, it is easier to use a computer to search for the minimizing vectors. We illustrate the variability of possible minimizing vectors and possible values of $\Delta_{can}$ on several examples in this and the following section. \begin{example}\label{ex:original} Consider the integral homology sphere $Y$ associated with the splice diagram \begin{center} \includestandalone{graphics/H_shaped/example_splice_big} \end{center} Then $\Delta(Y) = \frac{3045}{1000}$, with minimizing vectors $\pm(-1,-1,3,1,1,-1).$ $\Delta(Y)$ is strictly smaller than $-\frac{\gamma(Y)}{4} + \frac{1}{2} = \frac{3885}{1000}$ given by the vector $2u-\delta=(-1,-1,1,-1,-1,1)$. The corresponding plumbing graph on $28$ vertices is shown in Figure \ref{fig:plumbing_from_splice}. \begin{figure}[ht] \centering \resizebox{\columnwidth}{!}{ \includestandalone{graphics/H_shaped/example_plumbing_from_splice} } \caption{The plumbing graph associated to the splice diagram in Example \ref{ex:original}} \label{fig:plumbing_from_splice} \end{figure} \end{example} \begin{example} Consider the manifold $Y$ given by the following plumbing: \begin{center} \includestandalone{graphics/H_shaped/example_H_shaped/plumbing_H} \end{center} Then $Y$ is an integral homology sphere with $\Delta(Y) = \frac{1}{2}$ and the minimizing vectors are $\{\pm v, \pm w \}$, where $v=(1, 1, -1, -1, 1, 1)$, $w = (1, 1, -1, 1, -1, 1)$. Again, $\Delta(Y) < - \frac{\gamma(Y)}{4} + \frac{1}{2} = \frac{5}{2}$. Note that this plumbing corresponds to the following splice diagram: \begin{center} \includestandalone{graphics/H_shaped/example_H_shaped/splice_H} \end{center} \end{example} \section{Upper Bounds and Cancellations} In this section, we focus on upper bounds for $\Delta_b$. In principle, bounding $\Delta_b$ from above should be easy---the (shifted) norm $(-3s - \Tr(M) -l^2)/4$ of any element $l \in \mathcal{C}_b$ gives an upper bound. However, when finding elements of $\LC_b$, we are facing two issues. First of all, $\LCt_b$ can be empty, e.g. for some $\spc$ structures on lens spaces, giving $\Zhat_b(q)=0$ and $\Delta_b = \infty$. Secondly, even if $\LCt_b$ is non-empty, there may be drastic cancellations of the coefficients, as we will illustrate on the Seifert manifold in \cref{ex:cancel_seifert}. This prevents us from establishing general results in this direction. In particular, the vector $2u-\delta$ does not always give an upper bound for $\Delta_{can}$, unlike for Seifert manifolds (where it was optimal), see \cref{ex:cancel_H}. We believe that those cancellations are rather special, being related to some additional symmetry of the plumbing graph. In particular, it would be rather surprising if they occurred for all $\spc$ structures at once. Therefore, we expect the following: \begin{conjecture}\label{conj:ineq} For a negative-definite plumbed manifold $Y$ we have \[\min_{b \in \spc{Y}} \Delta_b \leq -\frac{\gamma(Y)}{4}+\frac12.\] \end{conjecture} We now proceed with several examples of manifolds for which the cancellations occur. We also compare $\Zhat(q)$ with the two-variable extension $\Zhathat$ defined in \cite{AJK23}. The new variable \(t\) can distinguish vectors $l$ with the same value of $l^2$, removing some cancellations, but not all of them, see \cref{ex:t_cancel_0}. \begin{example}\label{ex:cancel_H} Consider the following plumbing: \begin{center} \includestandalone{graphics/H_shaped/example_cancel/plumbing_cancel} \end{center} The resulting plumbed manifold $Y$ admits $20$ $\spc$ structures. For the canonical $\spc$ structure $can$ we have $\Delta_{can} = \frac{33}{20}$ with the (sole) minimizing vector $(1, -1, 1, -1, -1, 1)$. This is strictly larger than $-\frac{\gamma(Y)}{4} + \frac12 = \frac{13}{20}$. The only vectors within $\LCt_{can}$ that satisfy $\frac{1}{4}(-3s - \operatorname{Tr}(M) - l^2) = \frac{13}{20}$ are $l_1 = (1, 1, -1, 1, 1, -1)$ and $l_2 = (-1, -1, 1, 1, 1, -3)$. However their coefficients are $c_{l_1} = \frac{1}{4} = -c_{l_2}$ and so they cancel out in $\Zhat_{can}(q)$. The above cancellation does not happen for $\Zhathat_{can}(q,t)$: \[ \Zhathat_{can}(q,t) = -\frac{1}{4}\left((t^{-1} -t)q^\frac{13}{20} +q^\frac{33}{20} -t^{-1}q^\frac{53}{20} + (t^{-2} + t^2)q^\frac{73}{20} - t^{-1}q^\frac{93}{20} +\cdots\right). \] \end{example} \begin{example}\label{ex:cancel_seifert} Consider Seifert manifold \(M(2;(3,1),(3,2),(3,2))\). It is described by the following negative definite plumbing: \begin{center} \includestandalone{graphics/seifert/plumbingM2_13_23_23} \end{center} Let \(b\) be the \(\spc\) structure with the associated vector \(\delta = (1,2,3,2,1,1) \in 2\ZZ^s+\delta\). The $q$-series $\Zhat_b(q)$ is a single monomial: \[ \Zhat_b(q) = \frac12 q^{-5/6} \] This is due to the following cancellation: All vectors but one in the set $\mathcal{C}$ can be split into pairs \(l_1,l_2\) satisfying \(l_1^2=l_2^2\) and \(c_{l_1} = -c_{l_2}\). Similarly to the previous example, $\Zhathat_b(q,t)$ removes the cancellations and it gives an infinite series: \[ \Zhathat_b(q,t) = \frac12 ( q^{-5/6} t^{-1} + q^{13/6} (1-t^{-2}) + q^{67/6} (t-t^{-3}) + q^{157/6} (t^2 - t^{-4}) \dots). \] \end{example} \begin{example}\label{ex:t_cancel_0} Consider the following plumbing: \begin{center} \includestandalone{graphics/H_shaped/example_cancel/plumbing_t_cancel} \end{center} The resulting plumbed manifold $Y$ admits $21$ $\spc$ structures. For the canonical $\spc$ structure $can$ represented by $2u - \delta = ( 1, 1, -1, 1, 1, -1)$ we have that $\Delta_{can}' = \frac{5}{2}$ and the minimizing vectors for the quadratic form that produces $\Delta'_{can}$ are given by $l_1 = (-1, -1, -3, 1, 1, 1)$, $l_2 = (-1, -1, 7, -1, -1, -1)$, $l_3 = (1, 1, -7, 1, 1, 1)$ and $l_4 = (1, 1, 3, -1, -1, -1)$ with coefficients $c_{l_1} = c_{l_3}= -\frac{1}{4}t^{-1}$ and $c_{l_2} = c_{l_4 }= -\frac{1}{4}t$. This is strictly larger than $-\frac{\gamma(Y)}{4} + \frac12 = \frac{1}{2}$. For this manifold, the only vectors within $\tilde{\mathcal{C}}_{can}$ which satisfy $\frac{1}{4}(-3s - \operatorname{Tr}(M) - l^2) = \frac{1}{2}$ are $l_1 = (-1, -1, 1, -1, -1, 1)$, $l_2 = (-1, -1, 3, 1, 1, -1)$, $l_3 = (1, 1, -1, 1, 1, -1)$ and $l_4 = (1, 1, -3, -1, -1, 1)$. However their coefficients are $c_{l_1} = \frac{1}{4}t^{-1} = -c_{l_4}$ and $c_{l_2} = -\frac{1}{4}t = -c_{l_3}$ and so they cancel out in $\Zhathat_{can}(q,t)$. The entire $(t, q)$ series is given by: \[ \Zhathat_{can}(q,t) = -\frac{1}{4}\left((2t^{-1} + 2t)q^\frac{5}{2} + (2t^{-3} +2t^3)q^\frac{9}{2} + (-t^{-4} + t^{-2} +t^2 -t^4)q^\frac{15}{2} + \cdots\right) \] This example shows that cancellations do occur even for $\Zhathat_{can}(q,t)$ and as a result $\Delta_{can}' > -\frac{\gamma(Y)}{4} + \frac{1}{2}$ where $\Delta_{can}'$ denotes the smallest $q$-exponent of $\Zhathat_{can}(q,t)$. \end{example} \section{Comparison with correction terms} In the last section, we compare $\Delta_b(Y)$ with correction terms $d_b(Y)=d(Y,[b])$ in Heegaard--Floer homology\footnote{Again, we omit the brackets for $\spc$ structures.}. We include this discussion because there were some expectations that $\Delta_b(Y)$ and $d_b(Y)$ might be related \cite{GPP21, AJK23}. In the Seifert case, we have an explicit formula for $\Delta_{can}(Y)$ in terms of the $\gamma(Y)$ invariant by \cref{thm:seifert}. We can then use elementary bounds for Dedekind sums to obtain estimates on $\Delta_{can}(Y)$. Finally, we compare $\Delta_{can}(Y)$ to $d_{can}(Y)$ for some classes of Brieskorn spheres, where $d_{can}(Y)$ is known, finding that they are very different. \subsection{Quadratic forms} Correction terms can be expressed as minimizers of a quadratic form over the characteristic vectors: \begin{theorem}[{\cite{Nem05}}]\label{thm:cor_terms} For an almost rational graph $\Gamma$, the correction terms are given by \[ d_k(Y) = \max_{k' \in [k]} \frac{(k')^2+s}{4} \] \end{theorem} Note that this formula holds in many other cases and was conjectured by the second author to be true for any negative definite rational homology sphere \cite{Nemethi_lattice08}. To compare with $\Delta$, we need to shift to our conventions on $\spc$ structures, so we set $l = k' - Mu$. Then we have \begin{align*} \frac{(k')^2+s}{4} &= \frac{(l + Mu)^T M^{-1}(l + Mu) + s}{4}\\ &= \left(\frac{\Tr(M) +3s}{4} + \frac{l^2}{4} \right) + \frac{l^T u}{2} - \frac12 \end{align*} We see that the minimized quadratic forms for $d$ and $\Delta$ differ in the linear term $l^T u/2= (\sum_{i=1}^s l_i)/2$. \begin{remark} In \cite{GPP21}, the authors observed that in the setting of \cref{thm:cor_terms}, the difference between $d_b(Y)$ and $\Delta_b(Y)-\frac12$ is an integer. For Seifert manifold $Y$ with the canonical $\spc$ structure, this can be explained as follows: By \cite[Thm. 8.3.]{Nem08}, $d_{can}(Y)$ and $\gamma(Y)$ (and therefore $\Delta_{can}(Y)$) satisfy the following relation: \begin{equation} d_{can}(Y)=\frac{\gamma(Y)}{4} - 2 \chi_{can} \left( = -\Delta_{can} + \frac12 - 2 \chi_{can} \right). \end{equation} Here $\chi_{can}$ is the holomorphic Euler characteristic of a certain holomorphic line bundle on the corresponding quasihomogeneous singularity. In particular, it is an integer. \end{remark} \subsubsection{Lower Bound for Seifert Manifolds} We end this section with some elementary estimates considering the $\gamma(Y)$ invariant of Seifert manifolds, and hence of $\Delta_{can}(Y)$. The formula \eqref{eq:formula_gamma} for $\gamma(Y)$ contains Dedekind sums. We have the following well-known inequality for $p>0$, $a \in \ZZ$: \begin{equation} -\mathbf{s}(1, p) \leq \mathbf{s}(a, p) \leq \mathbf{s}(1, p) = \frac{p}{12} + \frac{1}{6p} - \frac{1}{4}. \end{equation} Combining this with \eqref{eq:formula_gamma}, we obtain the following result. \begin{proposition}\label{lower-bound-delta-general-seifert} For the Seifert manifold $Y = M(b_0;(a_1, \omega_1), \dots, (a_n, \omega_n))$ and the canonical $\spc$ structure, we have \[ \Delta_{can} \geq - \frac{1}{4e}\left(2-n + \sum_{i=1}^n \frac{1}{a_i}\right)^2 - \frac{e+3(n+1)}{4} + \sum_{i=1}^n \frac{a_i}{4} + \frac{1}{2a_i} \] and \[ \Delta_{can} \leq -\frac{1}{4e}\left(2-n + \sum_{i=1}^n \frac{1}{a_i}\right)^2 -\frac{e+3(1-n)}{4} - \sum_{i=1}^n \frac{a_i}{4} + \frac{1}{2a_i} \] where $e$ is the orbifold Euler number of $Y$. \end{proposition} If $Y$ is a Brieskorn sphere $\Sigma(a_1, \dots, a_n)$, then $e = -\prod_{i=1}^n a_i^{-1}=-A^{-1}$. In particular, if $a_i \gg 1$, the leading term of $\Delta$ is given by $(n-2)^2A$. We obtain that the $\Delta$ invariant grows polynomially with $a_i$ for large values of $a_i$. \subsubsection{Brieskorn spheres} We illustrate the difference between $\Delta = \Delta_{can}$ and $d = d_{can}$ for some families of Brieskorn spheres, for which correction terms are explicitly known \cite{BorNem2011}. For $p, q > 0$ set $\rho = (p-1)(q-1)/2$. Then for $\Sigma(p, q, p q+1)= S^3_{-1}(T_{p, q})$ we have $\gamma(Y) = -4\rho(\rho-1)$. From \cref{thm:seifert} we immediately obtain the next result: \begin{corollary} For $Y = \Sigma(p, q, pq+1)$, \begin{equation}\label{eq:Delta-p-p+1} \Delta(Y) = \rho(\rho-1) + \frac{1}{2}. \end{equation} \end{corollary} In contrast to this, the correction term vanishes, so that $\Delta(Y) \gg d(Y) = 0$. For $Y = \Sigma(p, p+1, p(p+1)-1)$, we have \begin{equation}\label{eq:d} d(Y) = \bigg\lfloor \frac{p}{2} \bigg\rfloor\left(\bigg\lfloor \frac{p}{2} \bigg\rfloor+1\right).\end{equation} From \cref{lower-bound-delta-general-seifert} we obtain, after some manipulations, that for $p>2$ \begin{equation}\label{inequality-delta-d-comparison} \Delta(Y) \geq \frac{1}{4}\left(p^4 + 2p^3-5p \right)-3 > d(Y). \end{equation} The case of $p = 2$ correspond to Poincar\'e sphere $Y = \Sigma(2, 3, 5)$, where we have $\Delta(Y) = -\frac{3}{2} < d(Y) = 2$. This seems to be a boundary case and in general, we expect the following conjecture: \begin{conjecture} For all but finitely many Seifert manifolds $Y$ we have \[ d_{can}(Y) < \Delta_{can}(Y). \] \end{conjecture} \printbibliography \end{document} \documentclass[tikz]{standalone} \begin{document} \begin{tikzpicture} \begin{scope}[every node/.style={circle,fill,draw,inner sep=1.5pt}] \node (1) at (-2,0.5) {}; \node (2) at (-2,-0.5) {}; \node (3) at (-1,0) {}; \node (4) at (1,0) {}; \node (5) at (2,0.5) {}; \node (6) at (2,-0.5) {}; \end{scope} \node[xshift=-.3cm] at (2) {$v$}; \node[yshift=.3cm] at (4) {$v'$}; \node[xshift=.3cm, yshift=-.3cm] at (3) {$P$}; \begin{scope}[every edge/.style ={draw}, every node/.style ={fill=white,inner sep=0pt,scale=.9}] \draw (1) -- node[near end] {$2$} (3); \draw[thick] (2) -- (3); \draw[thick] (3) -- (4); \draw (4) -- node[near start] {$2$} (5); \draw (4) -- node[near start] {$2$} (6); \end{scope} \node[xshift=2cm] at (4) {$N_{vv'}=8$}; \end{tikzpicture} \end{document} \documentclass[tikz]{standalone} \begin{document} \begin{tikzpicture} \begin{scope}[every node/.style={circle,fill,draw,inner sep=1.5pt}] \node (1) at (-2,0.5) {}; \node (2) at (-2,-0.5) {}; \node (3) at (-1,0) {}; \node (4) at (1,0) {}; \node (5) at (2,0.5) {}; \node (6) at (2,-0.5) {}; \end{scope} \node[xshift=-.3cm] at (1) {\(x_1\)}; \node[xshift=-.3cm] at (2) {\(x_2\)}; \node[yshift=.3cm] at (3) {\(x_3\)}; \node[yshift=.3cm] at (4) {\(x'_3\)}; \node[xshift=.3cm] at (5) {\(x'_1\)}; \node[xshift=.3cm] at (6) {\(x'_2\)}; \begin{scope}[every edge/.style ={draw}, every node/.style ={fill=white,inner sep=0pt,scale=.9}] \draw (1) -- node[near end] {$a_1$} (3); \draw (2) -- node[near end] {$a_2$} (3); \draw (3) -- node[near start] {$a_3$} node[near end] {$a'_3$} (4); \draw (4) -- node[near start] {$a'_1$} (5); \draw (4) -- node[near start] {$a'_2$} (6); \end{scope} \end{tikzpicture} \end{document} \documentclass[tikz]{standalone} \begin{document} \begin{tikzpicture} \begin{scope}[every node/.style={circle,fill,draw,inner sep=1.5pt}] \node (1) at (-2,0.5) {}; \node (2) at (-2,-0.5) {}; \node (3) at (-1,0) {}; \node (4) at (0,0) {}; \node (5) at (1,0) {}; \node (6) at (2,0.5) {}; \node (7) at (2,-0.5) {}; \end{scope} \node[xshift=-.3cm] at (1) {-2}; \node[xshift=-.3cm] at (2) {-2}; \node[yshift=.3cm] at (3) {-2}; \node[yshift=.3cm] at (4) {-3}; \node[yshift=.3cm] at (5) {-2}; \node[xshift=.3cm] at (6) {-2}; \node[xshift=.3cm] at (7) {-2}; \node (0) at (-2.5,0) {$\Gamma = $}; \begin{scope}[every edge/.style ={draw}] \draw (1) -- (3); \draw (2) -- (3); \draw (3) -- (4); \draw (4) -- (5); \draw (5) -- (6); \draw (5) -- (7); \end{scope} \begin{scope}[xshift=6.5cm] \begin{scope}[every node/.style={circle,fill,draw,inner sep=1.5pt}] \node (1) at (-2,0.5) {}; \node (2) at (-2,-0.5) {}; \node (3) at (-1,0) {}; \node (4) at (1,0) {}; \node (5) at (2,0.5) {}; \node (6) at (2,-0.5) {}; \end{scope} \begin{scope}[every edge/.style ={draw}, every node/.style ={fill=white,inner sep=0pt,scale=.9}] \draw (1) -- node[near end] {$2$} (3); \draw (2) -- node[near end] {$2$} (3); \draw (3) -- node[near start] {$8$} node[near end] {$8$} (4); \draw (4) -- node[near start] {$2$} (5); \draw (4) -- node[near start] {$2$} (6); \end{scope} \node (0) at (-2.5,0) {$\Omega = $}; \end{scope} \end{tikzpicture} \end{document} \documentclass[tikz]{standalone} \usetikzlibrary{decorations.pathreplacing} \begin{document} \begin{tikzpicture} \begin{scope}[every node/.style={circle,fill,draw,inner sep=1.5pt}] \node (3) at (-1,0) {}; \node (4) at (1,0) {}; \end{scope} \node[yshift=.3cm] at (3) {$m_v$}; \node[yshift=-.3cm] at (3) {$v$}; \node[yshift=.3cm] at (4) {$m_{v'}$}; \node[yshift=-.2cm] at (0,0) {$e$}; \begin{scope}[every edge/.style ={draw}] \draw (3) -- (4); \end{scope} \begin{scope}[every edge/.style ={draw}, dashed] \draw (-3,0.5) -- (3) -- (-3,-0.5); \draw (3,0.5) -- (4) -- (3,-0.5); \end{scope} \draw[decorate,decoration={brace,amplitude=5pt,mirror,raise=4ex}] (1,0) -- (3,0) node[midway, yshift=-3em] {$\Gamma_{ve}$}; \end{tikzpicture} \end{document} \documentclass[tikz]{standalone} \begin{document} \begin{tikzpicture} \begin{scope}[every node/.style={circle,fill,draw,inner sep=1.5pt}] \node (1) at (-1,0.5) {}; \node (2A) at (-1,-0.5) {}; \node (2) at (-1.5,-1.0) {}; \node (3) at (-0.5,0) {}; \node (3A) at (0,0) {}; \node (3B) at (0.5,0) {}; \node (3C) at (1,0) {}; \node (3D) at (1.5,0) {}; \node (3E) at (2,0) {}; \node (3F) at (2.5,0) {}; \node (3G) at (3,0) {}; \node (3H) at (3.5,0) {}; \node (3I) at (4,0) {}; \node (3J) at (4.5,0) {}; \node (3K) at (5,0) {}; \node (3L) at (5.5,0) {}; \node (3M) at (6,0) {}; \node (3N) at (6.5,0) {}; \node (3O) at (7,0) {}; \node (4) at (7.5,0) {}; \node (5) at (8,0.5) {}; \node (5A) at (8.5,1) {}; \node (5B) at (9,1.5) {}; \node (5C) at (9.5,2) {}; \node (5D) at (10,2.5) {}; \node (5E) at (10.5,3) {}; \node (5F) at (11,3.5) {}; \node (6) at (8,-0.5) {}; \node (6A) at (8.5,-1) {}; \end{scope} \node[xshift=-.3cm] at (1) {-2}; \node[xshift=-.4cm] at (2A) {-2}; \node[xshift=-.4cm] at (2) {-2}; \node[yshift=.3cm] at (3) {-2}; \node[yshift=.3cm] at (4) {-2}; \node[xshift=.3cm] at (5) {-2}; \node[xshift=.3cm] at (5A) {-2}; \node[xshift=.3cm] at (5B) {-2}; \node[xshift=.3cm] at (5C) {-2}; \node[xshift=.3cm] at (5D) {-2}; \node[xshift=.3cm] at (5E) {-2}; \node[xshift=.3cm] at (5F) {-2}; \node[xshift=.3cm] at (6) {-3}; \node[xshift=.3cm] at (6A) {-2}; \node[yshift=.3cm, xshift=-0.05cm] at (3A) {-2}; \node[yshift=.3cm, xshift=-0.05cm] at (3B) {-2}; \node[yshift=.3cm, xshift=-0.05cm] at (3C) {-2}; \node[yshift=.3cm, xshift=-0.05cm] at (3D) {-2}; \node[yshift=.3cm, xshift=-0.05cm] at (3E) {-4}; \node[yshift=.3cm, xshift=-0.05cm] at (3F) {-3}; \node[yshift=.3cm, xshift=-0.05cm] at (3G) {-2}; \node[yshift=.3cm, xshift=-0.05cm] at (3H) {-2}; \node[yshift=.3cm, xshift=-0.05cm] at (3I) {-2}; \node[yshift=.3cm, xshift=-0.05cm] at (3J) {-2}; \node[yshift=.3cm, xshift=-0.05cm] at (3K) {-2}; \node[yshift=.3cm, xshift=-0.05cm] at (3L) {-2}; \node[yshift=.3cm, xshift=-0.05cm] at (3M) {-3}; \node[yshift=.3cm, xshift=-0.05cm] at (3N) {-2}; \node[yshift=.3cm, xshift=-0.05cm] at (3O) {-2}; \begin{scope}[every edge/.style ={draw}] \draw (1) -- (3); \draw (2A) -- (2); \draw (2) -- (3); \draw (3) -- (4); \draw (4) -- (5); \draw (4) -- (6); \draw (6) -- (6A); \draw (5) -- (5A); \draw (5A) -- (5B); \draw (5B) -- (5C); \draw (5C) -- (5D); \draw (5D) -- (5E); \draw (5E) -- (5F); \end{scope} \end{tikzpicture} \end{document} \documentclass[tikz]{standalone} \begin{document} \begin{tikzpicture} \begin{scope}[every node/.style={circle,fill,draw,inner sep=1.5pt}] \node (1) at (-2,0.5) {}; \node (2) at (-2,-0.5) {}; \node (3) at (-1,0) {}; \node (4) at (1,0) {}; \node (5) at (2,0.5) {}; \node (6) at (2,-0.5) {}; \end{scope} \begin{scope}[every edge/.style ={draw}, every node/.style ={fill=white,inner sep=0pt,scale=.9}] \draw (1) -- node[near end] {$2$} (3); \draw (2) -- node[near end] {$3$} (3); \draw (3) -- node[near start] {$11$} node[near end] {$101$} (4); \draw (4) -- node[near start] {$5$} (5); \draw (4) -- node[near start] {$7$} (6); \end{scope} \end{tikzpicture} \end{document} \documentclass[tikz]{standalone} \begin{document} \begin{tikzpicture} \begin{scope}[every node/.style={circle,fill,draw,inner sep=1.5pt}] \node (1) at (-2,0.5) {}; \node (2) at (-2,-0.5) {}; \node (3) at (-1,0) {}; \node (4) at (0,0) {}; \node (5) at (1,0.5) {}; \node (6) at (1,-0.5) {}; \end{scope} \node[xshift=-.3cm] at (1) {-2}; \node[xshift=-.3cm] at (2) {-3}; \node[yshift=.3cm] at (3) {-5}; \node[yshift=.3cm] at (4) {-1}; \node[xshift=.3cm] at (5) {-3}; \node[xshift=.3cm] at (6) {-3}; \begin{scope}[every edge/.style ={draw}] \draw (1) -- (3); \draw (2) -- (3); \draw (3) -- (4); \draw (4) -- (5); \draw (4) -- (6); \end{scope} \end{tikzpicture} \end{document} \documentclass[tikz]{standalone} \begin{document} \begin{tikzpicture} \begin{scope}[every node/.style={circle,fill,draw,inner sep=1.5pt}] \node (1) at (-2,0.5) {}; \node (2) at (-2,-0.5) {}; \node (3) at (-1,0) {}; \node (4) at (0,0) {}; \node (5) at (1,0.5) {}; \node (6) at (1,-0.5) {}; \end{scope} \node[xshift=-.3cm] at (1) {-5}; \node[xshift=-.3cm] at (2) {-2}; \node[yshift=.3cm] at (3) {-1}; \node[yshift=.3cm] at (4) {-6}; \node[xshift=.3cm] at (5) {-2}; \node[xshift=.3cm] at (6) {-2}; \begin{scope}[every edge/.style ={draw}] \draw (1) -- (3); \draw (2) -- (3); \draw (3) -- (4); \draw (4) -- (5); \draw (4) -- (6); \end{scope} \end{tikzpicture} \end{document} \documentclass[tikz]{standalone} \begin{document} \begin{tikzpicture} \begin{scope}[every node/.style={circle,fill,draw,inner sep=1.5pt}] \node (1) at (-2,0.5) {}; \node (2) at (-2,-0.5) {}; \node (3) at (-1,0) {}; \node (4) at (1,0) {}; \node (5) at (2,0.5) {}; \node (6) at (2,-0.5) {}; \end{scope} \begin{scope}[every edge/.style ={draw}, every node/.style ={fill=white,inner sep=0pt,scale=.9}] \draw (1) -- node[near end] {$2$} (3); \draw (2) -- node[near end] {$3$} (3); \draw (3) -- node[near start] {$37$} node[near end] {$1$} (4); \draw (4) -- node[near start] {$2$} (5); \draw (4) -- node[near start] {$3$} (6); \end{scope} \end{tikzpicture} \end{document} \documentclass[tikz]{standalone} \begin{document} \begin{tikzpicture} \begin{scope}[every node/.style={circle,fill,draw,inner sep=1.5pt}] \node (1) at (-2,0.5) {}; \node (2) at (-2,-0.5) {}; \node (3) at (-1,0) {}; \node (4) at (0,0) {}; \node (5) at (1,0.5) {}; \node (6) at (1,-0.5) {}; \end{scope} \node[xshift=-.3cm] at (1) {-2}; \node[xshift=-.3cm] at (2) {-3}; \node[yshift=.3cm] at (3) {-1}; \node[yshift=.3cm] at (4) {-7}; \node[xshift=.3cm] at (5) {-2}; \node[xshift=.3cm] at (6) {-3}; \begin{scope}[every edge/.style ={draw}] \draw (1) -- (3); \draw (2) -- (3); \draw (3) -- (4); \draw (4) -- (5); \draw (4) -- (6); \end{scope} \end{tikzpicture} \end{document} \documentclass[tikz]{standalone} \begin{document} \begin{tikzpicture} \begin{scope}[every node/.style={circle,fill,draw,inner sep=1.5pt}] \node (1) at (-2,0) {}; \node (2) at (-1,0) {}; \node (3) at (-0,0) {}; \node (4) at (1,0) {}; \node (5) at (2,0) {}; \node (6) at (0,-1) {}; \end{scope} \node[yshift=.3cm] at (1) {-2}; \node[yshift=.3cm] at (2) {-2}; \node[yshift=.3cm] at (3) {-2}; \node[yshift=.3cm] at (4) {-2}; \node[yshift=.3cm] at (5) {-2}; \node[xshift=-.3cm] at (6) {-3}; \begin{scope}[every edge/.style ={draw}] \draw (1) -- (2); \draw (2) -- (3); \draw (3) -- (6); \draw (3) -- (4); \draw (4) -- (5); \end{scope} \end{tikzpicture} \end{document} \documentclass[tikz]{standalone} \begin{document} \begin{tikzpicture} \begin{scope}[every node/.style={circle,fill,draw,inner sep=1pt}] \node (1) at (0,0) {}; \node (2) at (1,1) {}; \node (3) at (2,1) {}; \node (4) at (4,1) {}; \node (5) at (1,-1) {}; \node (6) at (2,-1) {}; \node (7) at (4,-1) {}; \end{scope} \node (14) at (2,0) {$\dots$}; \node (15) at (3,1) {$\dots$}; \node (16) at (3,-1) {$\dots$}; \node[xshift=-.4cm] at (1) {$-b_0$}; \node[yshift=.4cm] at (2) {$-b_{1_1}$}; \node[yshift=.4cm] at (3) {$-b_{1_2}$}; \node[yshift=.4cm] at (4) {$-b_{1_{s_1}}$}; \node[yshift=-.4cm] at (5) {$-b_{n_1}$}; \node[yshift=-.4cm] at (6) {$-b_{n_2}$}; \node[yshift=-.4cm] at (7) {$-b_{n_{s_n}}$}; \begin{scope}[every edge/.style ={draw}] \draw (1) -- (2); \draw (2) -- (3); \draw (3) -- (2.5,1); \draw (3.5,1) -- (4); \draw (1) -- (5); \draw (5) -- (6); \draw (6) -- (2.5,-1); \draw (3.5,-1) -- (7); \draw (1) -- (1,0.25); \draw (1) -- (1,-0.25); \end{scope} \end{tikzpicture} \end{document}
2412.02123v3
http://arxiv.org/abs/2412.02123v3
Self-embedding similitudes of Bedford-McMullen carpets with dependent ratios
\documentclass[11pt]{amsart} \usepackage{amsfonts,amsthm,amsmath,enumerate,amssymb,latexsym,color,tcolorbox,tikz,mathrsfs,bm,subfig,tcolorbox} \usetikzlibrary{patterns,patterns.meta,bending,angles,quotes,shapes.geometric} \usepackage[text={6in,9in},centering]{geometry} \usepackage[backref=page, colorlinks, linkcolor=blue, anchorcolor=red, citecolor=red]{hyperref} \renewcommand{\baselinestretch}{1.2} \theoremstyle{plain} \newtheorem{theorem}{Theorem}[section] \newtheorem{lemma}[theorem]{Lemma} \newtheorem{corollary}[theorem]{Corollary} \newtheorem{proposition}[theorem]{Proposition} \newtheorem{conjecture}[theorem]{Conjecture} \newtheorem*{fact}{Fact} \newtheorem{problem}[theorem]{Problem} \newtheorem{question}[theorem]{Question} \theoremstyle{definition} \newtheorem{definition}[theorem]{Definition} \newtheorem{example}[theorem]{Example} \theoremstyle{remark} \newtheorem{remark}[theorem]{Remark} \numberwithin{equation}{section} \DeclareMathOperator{\dimh}{dim_H} \DeclareMathOperator{\dist}{dist} \DeclareMathOperator{\id}{id} \DeclareMathOperator{\conv}{conv} \DeclareMathOperator{\inte}{int} \DeclareMathOperator{\por}{por} \DeclareMathOperator{\dimb}{dim_B} \DeclareMathOperator{\diam}{diam} \newcommand{\blue}{\color{blue}} \newcommand{\red}{\color{red}} \newcommand{\purple}{\color{purple}} \newcommand{\rd}{\,\mathrm{d}} \def\R{\mathbb R} \def\Z{\mathbb Z} \def\D{\mathcal D} \def\A{\mathcal A} \def\C{\mathcal C} \def\L{\mathcal L} \def\i{\bm i} \def\j{\bm j} \def\k{\bm k} \def\bi{\mathbf i} \def\bj{\mathbf j} \def\N{\mathbb N} \def\Q{\mathbb Q} \def\h{\mathcal H} \def\S{\mathcal S} \def\U{\mathcal U} \def\dh{d_{\textup{H}}} \date{\today} \begin{document} \title[Self-embeddings of Bedford-McMullen carpets with dependent ratios]{Self-embedding similitudes of Bedford-McMullen carpets with dependent ratios} \author{Jian-Ci Xiao} \address{School of Mathematics, Nanjing University of Aeronautics and Astronautics, Nanjing, China} \email{[email protected]} \subjclass[2010]{Primary 28A80; Secondary 28A78} \keywords{Bedford-McMullen carpets, self-embedding, obliqueness, generalized Sierpi\'nski carpets, logarithmic commensurability, deleted-digit sets.} \begin{abstract} We prove that any non-degenerate Bedford-McMullen carpet does not allow oblique self-embedding similitudes; that is, if $f$ is a similitude sending the carpet into itself, then the image of the $x$-axis under $f$ must be parallel to one of the principal axes. We also establish a logarithmic commensurability result on the contraction ratios of such embeddings. This completes a previous study by Algom and Hochman [Ergod. Th. \& Dynam. Sys. {\bf 39} (2019), 577--603] on Bedford-McMullen carpets generated by multiplicatively independent exponents, together with a new proof of their non-obliqueness statement. For the self-similar case, however, we construct a generalized Sierpi\'nski carpet that is symmetric with respect to an appropriate oblique line and hence allows a reflectional oblique self-embedding. As a complement, we prove that if a generalized Sierpi\'nski carpet satisfies the strong separation condition and permits an oblique rotational self-embedding similitude, then the tangent of the rotation angle takes values $\pm 1$. \end{abstract} \maketitle \section{Introduction} One way to gain a better comprehension of the geometric structure of fractal sets is to delve into the study of their self-embedding mappings, especially affine mappings or even similitudes. Note that a similitude $f$ on $\R^d$ can be written as $f(x)=\lambda Ox+a$, where $\lambda\geq 0$ is the \emph{similarity ratio} (also \emph{contraction ratio} if $\lambda<1$), $O$ is an orthogonal transformation on $\R^d$ (the \emph{orthogonal part}) and $a\in\R^d$. Here a rough principle is that the imposition of additional restrictions on these sets usually corresponds to an increase in the constraints placed on their self-embedding mappings. A pioneering work in this direction is a logarithmic commensurability theorem established by Feng and Wang~\cite{FW09}. This theorem asserts that if $K\subset\R$ is a self-similar set generated by a homogeneous iterated function system (IFS) with the open set condition and $f$ is a similitude with $f(K)\subset K$, then the contraction ratio of $f$ must be a rational power of the common ratio of mappings in the IFS. This result was later extended to higher dimensional cases by Elekes, Keleti and M\'ath\'e~\cite{EKM10}, albeit with the open set condition being substituted by the strong separation condition. If in addition the mappings in the corresponding IFS share a common contraction ratio and orthogonal part, the author~\cite{Xiao24} proved the relative openness of the embedded image. In~\cite{Alg201}, Algom characerized all affine self-embeddings of self-similar sets with the strong separation condition. For self-affine sets, however, there are much fewer existing results. One reason for this disparity is that a natural rescaling approach, which works well in the self-similar case, usually fails in the self-affine case due to the inherent distortion. Nevertheless, the classic family of Bedford-McMullen carpets, with their simple lattice structure and delicate intrinsic properties, presents an optimal subject for investigation. A formal definition is as follows. \begin{definition} Let $n\geq m\geq 2$ be integers and let $\Lambda\subset \{0,1,\ldots,n-1\}\times\{0,1,\ldots,m-1\}$ be non-empty. Set $\varphi_{i,j}$ to be the affine map given by \begin{equation}\label{eq:varphiij} \varphi_{i,j}(x,y) = \Big( \frac{x+i}{n}, \frac{y+j}{m} \Big), \quad (i,j)\in\Lambda. \end{equation} The attractor $K(n,m,\Lambda)$ associated with the self-affine IFS $\{\varphi_{i,j}:(i,j)\in\Lambda\}$ is usually called a \emph{Bedford-McMullen carpet} when $n>m$, and a \emph{generalized Sierpi\'nski carpet} when $n=m$. More precisely, \[ K(n,m,\Lambda) = \Big\{ \Big( \sum_{k=1}^\infty \frac{i_k}{n^k}, \sum_{k=1}^\infty \frac{j_k}{m^k} \Big): (i_k,j_k)\in\Lambda \Big\}. \] To avoid trivial cases, in this paper we always assume that $1<\#\Lambda<mn$, where $\#$ denotes the cardinality. \end{definition} In~\cite{EKM10}, Elekes, Keleti and M\'ath\'e studied the intersections of Bedford-McMullen carpets with their translated copies. They observed a measure drop phenomenon occurring in these intersections, except in trivial cases. A systematic study on self-embedding similitudes of Bedford-McMullen carpets was conducted by Algom and Hochman~\cite{AH19}, who showed that if the horizontal and vertical ratios $n,m$ are multiplicatively independent, then the carpet permits only nearly trivial self-embedding similitudes. To state and extend their findings, let us introduce a convenient terminology. \begin{definition}[Obliqueness] \, \begin{enumerate} \item A line $L\subset\R^2$ is called \emph{oblique} if it is not parallel to one of the two principle coordinate axes. \item A similitude $f$ on $\R^2$ with positive similarity ratio is called \emph{oblique} if $f$ sends the $x$-axis to an oblique line. \end{enumerate} \end{definition} Algom and Hochman's main result is the following theorem. \begin{theorem}[\cite{AH19}]\label{thm:AH19} Let $K=K(n,m,\Lambda)$ be a Bedford-McMullen carpet with $\frac{\log n}{\log m}\notin\Q$. Suppose $K$ is not supported on any line and is not a Cartesian product of the unit interval $[0,1]$ and some Cantor set. If $f$ is a similitude sending $K$ into itself, then $f$ is not oblique and has similarity ratio $1$. \end{theorem} The non-degenerate and non-product assumptions on $K$ are natural. For example, if $K=[0,1]\times\{0\}$ or if $K=[0,1]\times C_{1/3}$ (where $C_{1/3}$ denotes the middle third Cantor set), then $f$ clearly need not to be an isometry. Meanwhile, the assumption of the independence between $n$ and $m$ is a key factor preventing oblique and contracting self-embeddings of the carpet. In fact, it is well known that a Cantor $m$-set cannot be embedded into another Cantor $n$-set if $\frac{\log m}{\log n}\notin\Q$, see~\cite{FHR14}. Heuristically, this phenomenon rules out oblique embeddings of non-degenerate Bedford-McMullen carpets with independent ratios, and forces the contraction ratio of any embedding similitude to be dependent on $n$ and $m$ simultaneously (so isometry). Our main purpose is to study to what extent Theorem~\ref{thm:AH19} remains true for the dependent case (i.e., $\frac{\log n}{\log m}\in\Q$) or even the self-similar case (i.e., $n=m$). The presence of dependence inherently complicates the self-embedding problem. Fortunately, first we are able to extend the non-obliqueness statement to the dependent case, together with a new proof for the independent case without analyzing the tangent sets of $K$ as in~\cite{AH19}. \begin{theorem}\label{thm:main1} Let $K=K(n,m,\Lambda)$ be a Bedford-McMullen carpet that is not supported on any line. If $f$ is a similitude sending $K$ into itself, then $f$ is not oblique. \end{theorem} We remark that the non-obliqueness statement is closely related to the slicing problem of Bedford-McMullen carpets. For example, writing $N$ to be the maximal number of rectangles that are selected in a row in the initial pattern (see Section 2.1 for the definition), it is easy to find some $0\leq y\leq 1$ such that $\dimh (K\cap (\R\times\{y\}))=\frac{\log N}{\log n}$, where $\dimh$ is the Hausdorff dimension. But an oblique self-embedding similitude sends this horizontal slice into an oblique one, and if one can show that any oblique slice of $K$ has Hausdorff dimension strictly less than $\frac{\log N}{\log n}$, then the existence of oblique self-embeddings is denied. A natural question is: when do we have such an upper bound for oblique slices? By Marstrand's slicing theorem~\cite{Mar54}, for any oblique line $L$ in $\R^2$, \begin{equation}\label{eq:marstrand} \dimh (K\cap (L+a)) \leq \max\{\dimh K-1,0\} \quad \text{for a.e. } a\in L^\perp. \end{equation} So if $K$ has non-uniform horizontal fibres (that is, there are two rows containing different numbers of selected rectangles), it is easy to check (see~\eqref{eq:dimhk} for the formula of $\dimh K$) that \[ \dimh K-1 < \frac{1}{\log m}\cdot\log \Big( m\cdot N^\frac{\log m}{\log n} \Big)-1 = \frac{\log N}{\log n}. \] So almost all oblique slices have Hausdorff dimension strictly less than $\frac{\log N}{\log n}$. When $\frac{\log n}{\log m}\notin\Q$, it is highly probable that the upper bound specified in~\eqref{eq:marstrand} holds for all oblique slices rather than for only typical ones, which leads to a quick proof for certain special cases of Theorem~\ref{thm:main1}. This slicing problem is formally stated in the nice survey~\cite{Fra21} and seems quite challenging. See~\cite{Alg20,AW23} for recent attempts. However, in the dependent case this upper bound may not hold, as indicated in~\cite{BR14}. On the other hand, one cannot hope to extend the isometry statement in Theorem~\ref{thm:AH19} to the dependent case. For example, when $n=m^2$, it is easy to construct a Bedford-McMullen carpet which is actually a self-similar carpet. In such instances, there exist numerous non-isometric self-embedding similitudes. Nevertheless, we are able to prove a logarithmic commensurability conclusion as follows. \begin{theorem}\label{thm:main2} Let $K=K(n,m,\Lambda)$ be a Bedford-McMullen carpet with $\frac{\log n}{\log m}\in\Q$ and let $f$ be a similitude sending $K$ into itself. If $K$ is not contained in any line, then $\frac{\log\lambda}{\log n}\in\Q$, where $\lambda$ denotes the similarity ratio of $f$. \end{theorem} We outline here some rough ideas on how to prove the above two results. For the non-obliqueness statement, we rescale $f^k(K)$ in an appropriate manner so that, in the absence of vacant rows, a geometric observation can lead us to the desired conclusion for both independent and dependent cases. On the other hand, if there is a vacant row, the independent case can be directly addressed using results on the dimensions of projections of Bedford-McMullen carpets as formulated in~\cite{FJS10} . For the dependent case, we first rescale $f^k(K)$ in a new way so that there is another oblique self-embedding $g$ for which $g(K)\subset C\times\R$, where $C$ is a ``generalized'' Cantor-$n$ set. Utilizing a nontrivial extension of the logarithmic commensurability theorem in~\cite{FW09}, we establish the non-existence of such a $g$, thereby deriving Theorem~\ref{thm:main2}. Finally, we consider the self-similar case when $n=m$, which can be regarded as the ``most dependent'' case. While one might anticipate a non-oblique statement in this context, it is not hard to to construct a generalized Sierpi\'nski carpet which is symmetric with respect to an appropriate oblique line, thus permitting an oblique reflectional self-embedding. See Example~\ref{exa:nonobsiercat}. So the non-oblique statement fails in the self-similar setting. However, considering rotational self-embeddings in lieu of reflectional ones and under the assumption of the strong separation condition, we indeed eliminate almost all possibilities. Here a generalized Sierpi\'nski carpet $K=K(n,\Lambda)$ is said to satisfy the \emph{strong separation condition} if elements in $\{\varphi_{i,j}(K)\}_{(i,j)\in\Lambda}$ are mutually disjoint. \begin{theorem}\label{thm:main3} Let $K=K(n,\Lambda)$ be a generalized Sierpi\'nski carpet satisfying the strong separation condition. Let $\theta\in\R$ and let $R_\theta$ be the counterclockwise rotation at the origin by angle $\theta$. If $f(x)=\lambda R_\theta x+a$ is an oblique similitude sending $K$ into itself, then $|\tan\theta|=1$. \end{theorem} Theorem~\ref{thm:main3} is achieved as follows. Note that the convex hull of any non-degenerate Sierpi\'nski carpet is a polygon. On the one hand, we show that if there exists an oblique rotational $f$, then the rotational angle is a rational multiple of $\pi$, and simultaneously can be expressed as the sum of finitely many inner angles (modulus $2\pi$) of the above polygon. On the other hand, we prove that all of these inner angles have rational tangents. So the rotation angle has a rational tangent, and the desired conclusion follows directly from a theorem of Niven (see Lemma~\ref{lem:niven}). Here the strong separation condition plays a crucial role in deriving the logarithmic commensurability of the contraction ratio of $f$ (by a result in~\cite{EKM10}). In fact, by employing some mass comparison arguments, the separation condition in Theorem~\ref{thm:main3} could be relaxed to the requirement of total disconnectedness; however, this does not present a substantial improvement and we will not explore it further within the scope of this paper. \vspace{4pt} \paragraph{{\bf Organization}} In the subsequent section, we provide some useful observations on Bedford-McMullen carpets and deleted-digit sets. In Section 3, we present our new proof of the non-obliqueness statement in the independent case. Section 4 is devoted to the proofs of Theorems~\ref{thm:main1} and~\ref{thm:main2}. Finally, we establish Theorem~\ref{thm:main3} in Section 5. \vspace{4pt} \paragraph{{\bf Notation}} We write $\mathcal{S}(\R^d)$ to be the collection of all similitudes on $\R^d$ with positive similarity ratio. For any collection $\mathcal{A}$ of sets in $\R^d$, $\bigcup\mathcal{A}$ is the union of sets in $\mathcal{A}$. For $A\subset\R^d$ and $x\in\R^d$, we denote by $A+x$ (or $x+A$) the translation of $A$ by $x$, that is, $A+x:=\{a+x: a\in A\}$. For any oblique line $L\subset\R^2$, $\pi_L$ denotes the orthogonal projection onto $L$. On the other hand, we write $\pi_1$ and $\pi_2$ to be the orthogonal projections onto the $x$-axis and $y$-axis, respectively. Finally, the concatenation $fg$ of any two mappings $f,g$ simply means their composition $f\circ g$. \section{Preliminaries} \subsection{Bedford-McMullen carpets} A Bedford-McMullen carpet $K=K(n,m,\Lambda)$ can be obtained by a standard iteration process as follows. One first divides the unit square $[0,1]^2$ into an $n\times m$ grid, selecting a subset of rectangles formed by the grid (called the \emph{initial pattern}) and then repeatedly substituting the initial pattern on each of the selected rectangles. The limit set is just $K$. For $k\geq 1$, we call every element in \[ \{\varphi_{i_1,j_1}\circ\cdots\circ\varphi_{i_k,j_k}([0,1]^2): (i_1,j_1),\ldots,(i_k,j_k)\in\Lambda\} \] a level-$k$ rectangle. For any level-$k$ rectangle $R$, we write $\varphi_R$ to be the natural affine map sending $[0,1]^2$ to $R$. We first record two geometric observations of Bedford-McMullen carpets, which rely heavily on the self-affinity (more precisely, the fact that $n>m$) and might be of independent interest. \begin{proposition}\label{prop:projofkisinf} Let $K=K(n,m,\Lambda)$ be a Bedford-McMullen carpet. If there is at most one selected rectangle in each row, then for any oblique line $L$, $\pi_L(K)$ is an infinite set. \end{proposition} The condition that there is at most one selected rectangle in each row is not necessary, but the above version will suffice for our purpose. \begin{proof} It suffices to consider when $L$ passes through the origin. In this case, there is an angle $\theta$ such that $\pi_L(x)$ is essentially given by $\pi_1(x)\cos\theta+\pi_2(x)\sin\theta$. Since $L$ is oblique, $\cos\theta$ and $\sin\theta$ are both nonzero. Let $p\geq 1$ and pick a large $k$ so that $\frac{m^{k+p}}{n^k}<|\frac{\sin\theta}{\cos\theta}|$. Let $R$ be any level-$k$ rectangle. Since there is at most one selected rectangle in each row, there are at least $(\#\Lambda)^p$ many level-$(k+p)$ rectangles in $R$ that lies in different rows in the $n^{-k-p}\times m^{-k-p}$ grid. Since every level-$(k+p)$ rectangle intersects $K$, we can select $(\#\Lambda)^p/2$ many points in $\varphi_R(K)$ with $m^{-k-p}$-separated $y$-coordinates. Then their projections under $\pi_L$ are mutually distinct: otherwise, there are two of these points, say $a,b$, such that $\pi_L(a)=\pi_L(b)$, which implies that \begin{align*} \frac{|\sin\theta|}{|\cos\theta|} &= \frac{|\pi_1(a)-\pi_1(b)|}{|\pi_2(a)-\pi_2(b)|} && \text{(since $\pi_L(a)=\pi_L(b)$)} \\ &\leq \frac{n^{-k}}{|\pi_2(a)-\pi_2(b)|} && \text{(since $a,b\in R$)} \\ &\leq \frac{n^{-k}}{m^{-k-p}} && \text{(by the choice of that subset)} \\ &= \frac{m^{k+p}}{n^k} < \frac{|\sin\theta|}{|\cos\theta|}, && \text{(by our choice of $k$)} \end{align*} a contradiction. In particular, $\pi_L(K)\supset\pi_L(\varphi_R(K))$ contains at least $(\#\Lambda)^p/2$ many points. Since $\#\Lambda\geq 2$ and $p$ is arbitrary, $\pi_L(K)$ must be an infinite set. \end{proof} \begin{proposition}\label{prop:noobliquelines} Let $K=K(n,m,\Lambda)$ be a Bedford-McMullen carpet. Then $K$ does not contain any oblique segment of positive length. \end{proposition} When $n=m$, this proposition sometimes fails. For example, when $\Lambda=\{(i,i):0\leq i\leq n-1\}$, the carpet is nothing but a diagonal of the unit square. \begin{proof} Since $\#\Lambda<mn$, there are $0\leq i_0\leq n-1,0\leq j_0\leq m-1$ such that $(i_0,j_0)\notin\Lambda$. Denote by $Q$ the open rectangle $=(\frac{i_0}{n},\frac{i_0+1}{n})\times(\frac{j_0}{m},\frac{j_0+1}{m})$. Since $(i_0,j_0)\notin\Lambda$, $Q\cap K=\varnothing$. By the self-affinity of $K$, for any $k\geq 1$ and any level-$k$ rectangle $R$, $\varphi_R(Q)\cap K=\varnothing$. Note that if $R$ is of the form $[\frac{p}{n^k},\frac{p+1}{n^k}]\times[\frac{q}{m^k},\frac{q+1}{m^k}]$, then \begin{equation}\label{eq:thevacantrect} \varphi_R(Q) = \Big( \frac{p}{n^k}+\frac{i_0}{n^{k+1}},\frac{p}{n^k}+\frac{i_0+1}{n^{k+1}} \Big) \times \Big(\frac{q}{m^k}+\frac{j_0}{m^{k+1}},\frac{q}{m^k}+\frac{j_0+1}{m^{k+1}} \Big). \end{equation} Suppose $K$ contains an oblique line segment $\ell$ of length $|\ell|>0$. Let $u$ be the slope of $\ell$. Since $\ell$ is oblique, we may assume without loss of generality that $u>0$. Fix $k$ so large that $\frac{m^ku}{n^k}<\frac{1}{2}$ and that there exists a level-$k$ rectangle $R$ such that $\ell$ meets both the left and right edges of $R$. The existence of such a rectangle can be found at the end of the proof. For convenience, we also assume that $\ell\subset R$ (considering the truncation $\ell\cap R$ instead). In particular, $\ell\subset\varphi_R(K)$ and hence $\varphi_R^{-1}(\ell)\subset K$. Note that $\varphi_{R}^{-1}(\ell)$ has slope $\frac{m^ku}{n^k}$. Pick a large integer $p$ so that $\frac{1}{m^p}<\frac{m^ku}{n^k}$ and $\frac{1}{n^{p+2}}<\frac{1}{m^{p+3}}$. Let $(0,a)$ be the intersection of $\varphi_{R}^{-1}(\ell)$ and the left edge of $\varphi_{R}^{-1}(R)=[0,1]^2$. So $\varphi_R^{-1}(\ell)$ meets the right edge of $[0,1]^2$ at $(1,a+\frac{m^ku}{n^k})$. Since $\frac{m^ku}{n^k}>\frac{1}{m^p}\geq \frac{2}{m^{p+1}}$, there is some integer $j$ such that $[\frac{j}{m^{p+1}},\frac{j+1}{m^{p+1}}]\subset [a,a+\frac{m^ku}{n^k}]$. For convenience, write $g(x):=\frac{m^ku}{n^k}x+a$ to be the linear function whose graph contains $\varphi_{R}^{-1}(\ell)$. For $0\leq t\leq n^{p+2}$, set $x_t:=\frac{t}{n^{p+2}}$ and $y_t:=g(x_t)$. By our choices of $p$ and $k$, \begin{equation}\label{eq:noobliqueline2} y_{t+1}-y_t = g(x_{t+1})-g(x_t) = \frac{m^ku}{n^k}\cdot\frac{1}{n^{p+2}}<\frac{1}{2}\cdot\frac{1}{m^{p+3}}. \end{equation} Therefore, there must exist some $0\leq t_0\leq n^{p+2}-1$ such that \begin{equation}\label{eq:noobliqueline4} [y_{t_0},y_{t_0+1}] \subset \Big[ \frac{j}{m^{p+1}}+\frac{j_0}{m^{p+3}}, \frac{j}{m^{p+1}}+\frac{j_0+1}{m^{p+3}} \Big] \end{equation} because the interval on the right hand side has length $m^{-(p+3)}$. Write \[ \widetilde{R} := [x_{t_0},x_{t_0+1}] \times \Big[ \frac{j}{m^{p+1}},\frac{j}{m^{p+1}}+\frac{1}{m^{p+2}} \Big]. \] By~\eqref{eq:noobliqueline4}, $\varphi_{R}^{-1}(\ell)$ passes the interior of $\widetilde{R}$. So $\widetilde{R}$ is a level-$(p+2)$ rectangle. By~\eqref{eq:thevacantrect}, \[ \varphi_{\widetilde{R}}(Q) = \Big( x_{t_0}+\frac{i_0}{n^{p+3}}, x_{t_0}+\frac{i_0+1}{n^{p+3}} \Big) \times \Big( \frac{j}{m^{p+1}}+\frac{j_0}{m^{p+3}}, \frac{j}{m^{p+1}}+\frac{j_0+1}{m^{p+3}} \Big). \] Letting $x_*:=x_{t_0}+\frac{i_0}{n^{p+3}}+\frac{1}{2n^{p+3}}$, we have by~\eqref{eq:noobliqueline4} that \[ g(x_*) \in [g(x_{t_0}), g(x_{t_0+1})] = [y_{t_0},y_{t_0+1}] \subset \Big[ \frac{j}{m^{p+1}}+\frac{j_0}{m^{p+3}}, \frac{j}{m^{p+1}}+\frac{j_0+1}{m^{p+3}} \Big] \] and hence the point $(x_*, g(x_*))\in\varphi_{\widetilde{R}}(Q)$. But from the definition of $g$, this point is also in $\varphi_{R}^{-1}(\ell)\subset K$, which contradicts that $\varphi_{\widetilde{R}}(Q)\cap K=\varnothing$. The existence of $R$ can be deduced in a similar way we choose $\widetilde{R}$. Let $(a_1,b_1)$ be the left endpoint of the original $\ell$ (before the truncation). Pick $k$ large enough such that $n^{-k}$ is much smaller than $|\pi_1(\ell)|$. Let $x'_0:=\min\{\frac{i}{n^k}: \frac{i}{n_k}\geq a_1\}$ and $y'_0$ be such that $(x'_0,y'_0)\in\ell$. Then define recursively that \[ x'_t = x'_{t-1}+\frac{1}{n^k} \text{ and } y'_t := y'_{t-1}+\frac{u}{n^k}, \quad 1\leq t\leq \lfloor|\pi_1(\ell)|n^k\rfloor-1, \] where $\lfloor\cdot\rfloor$ denotes the integer part. So $(x'_t,y'_t)\in\ell$ because $u$ is the slope of $\ell$. Since $\frac{m^ku}{n^k}<\frac{1}{2}$, $|y'_t-y'_{t-1}|=\frac{u}{n^k}<\frac{1}{2m^k}$. So there must exist some $t_0$ such that $[y'_{t_0-1},y'_{t_0}]$ is entirely contained in some $m$-adic subinterval of $[0,1]$ of the form $[\frac{j'}{m^k},\frac{j'+1}{m^k}]$. In particular, $y'_{t_0-1}\geq\frac{j'}{m^k}$ and $y'_{t_0}\leq\frac{j'+1}{m^k}$. Then it suffices to take \[ R = [x'_{t_0-1},x'_{t_0}]\times \Big[ \frac{j'}{m^k},\frac{j'+1}{m^k} \Big]. \] \end{proof} In the rest of this paper, we adopt the following notations. For $K=K(n,m,\Lambda)$, \begin{itemize} \item $J:=\{0\leq j\leq m-1: \exists i \text{ s.t. } (i,j)\in\Lambda\}$, which collects the digits of non-empty rows in the initial pattern. \item $I:=\{0\leq i\leq n-1: \exists j \text{ s.t. } (i,j)\in\Lambda\}$, which collects the digits of non-empty columns in the initial pattern. \item For $0\leq j\leq m-1$, $I_j:=\{0\leq i\leq n-1: (i,j)\in\Lambda\}$, which collects the digits of selected rectangles in the $j$-th row. \item For $0\leq i\leq n-1$, $J_i:=\{0\leq j\leq m-1: (i,j)\in\Lambda\}$, which collects the digits of selected rectangles in the $i$-th column. \item $N:=\max_{0\leq j\leq m-1}\#I_j$. \item $K_x:=\{y: (x,y)\in K\}$ (vertical slice) and $K^y:=\{x: (x,y)\in K\}$ (horizontal slice). \end{itemize} Note for every $y\in\pi_2(K)$ with a unique expansion $y=\sum_{k=1}^\infty y_km^{-k}$, where $y_k\in J$, \begin{align} K^y &= \Big\{ \sum_{k=1}^\infty \frac{x_k}{n^k}: \Big( \sum_{k=1}^\infty \frac{x_k}{n^k},\sum_{k=1}^\infty\frac{y_k}{m^k} \Big) \in K \Big\} \notag \\ &= \Big\{ \sum_{k=1}^\infty \frac{x_k}{n^k}: (x_k,y_k)\in\Lambda \text{ for all } k \Big\} \notag\\ &= \Big\{ \sum_{k=1}^\infty \frac{x_k}{n^k}: x_k\in I_{y_k} \text{ for all } k \Big\}. \label{eq:expressionofky} \end{align} For future use, we write \begin{equation}\label{eq:kyp} K^y_p:= \bigcup\Big\{ [x,x+n^{-p}]: x\in \Big\{ \sum_{k=1}^p \frac{x_k}{n^k}: x_k\in I_{y_k} \text{ for } 1\leq k\leq p \Big\} \Big\} \end{equation} and call the interior of every interval in the above union a \emph{basic open interval} of $K^y_p$. It is easy to see that $\{K^y_p\}_{p=1}^\infty$ is decreasing, $\bigcap_{p=1}^\infty K^y_p=K^y$ and \begin{equation}\label{eq:dimlbofky} \dimh K^y \leq \underline{\dim}_{\textup{B}}\, K^y \leq \liminf_{M\to\infty} \frac{\log (\prod_{k=1}^M \#I_{y_k})}{-\log n^M}\leq\frac{\log N}{\log n}, \end{equation} where $\underline{\dim}_{\textup{B}}$ denotes the lower box dimension. Furthermore, it is standard to show that $\h^{\log N/\log n}(K^y)\leq 1<\infty$ for all $y$ and we omit the proof. Here and afterwards, $\h^s$ denotes the $s$-dimensional Hausdorff measure. The following lemma records some basic facts about these horizontal slices. \begin{lemma}\label{lem:bunchoffacts} Let $K=K(n,m,\Lambda)$ be a Bedford-McMullen carpet. Then for any $y\in\pi_2(K)$ with a unique expansion $y=\sum_{k=1}^\infty y_km^{-k}$, $\{y_k\}_{k=1}^\infty\subset J$, \begin{enumerate} \item For every $p\geq 1$ and every pair of basic open intervals $U,V$ of $K^y_p$, $K^y\cap U$ is simply a translated copy of $K^y\cap V$. \item For every $p,t\geq 1$ and every basic open interval $U$ of $K^y_p$, $U\cap K^y_p$ contains $\prod_{k=p+1}^{p+t}\# I_{y_k}$ many basic open intervals of $K^y_{p+t}$. \end{enumerate} In addition, if $\#I_{y_k}\geq 2$ for all large $k$, then the following properties hold. \begin{enumerate} \setcounter{enumi}{2} \item $\inf_{p\geq 1}\min\{\diam(K^y\cap U)/|U|: U\text{ is a basic open interval of $K^y_p$}\}>0$, where $\diam(\cdot)$ is the diameter. \item Calling an open interval $G$ a \emph{gap} of $K^y_p$ if $G$ is a connected component of $U\setminus K^y$, where $U$ is some basic open interval of $K^y_p$, we have \[ \sup_{p\geq 1}\max\{|G|n^{p}: G \text{ is a gap of } K^y_p\}<1. \] \end{enumerate} \end{lemma} \begin{proof} Let $p\geq 1$ and let $U$ be any basic open interval of $K^y_p$. Writing $a$ to be the left endpoint of $U$, we immediately see from~\eqref{eq:expressionofky} and~\eqref{eq:kyp} that \begin{align*} K^y \cap U &= K^y\cap (a,a+n^{-p}) \\ &= \Big\{ a+\sum_{k=p+1}^\infty \frac{z_k}{n^k}: z_k\in I_{y_k} \text{ for all } k \Big\}\setminus\{a,a+n^{-p}\} \\ &= a + \Big( \Big\{ \sum_{k=p+1}^\infty \frac{z_k}{n^k}: z_k\in I_{y_k} \text{ for all } k \Big\}\setminus\{0,n^{-p}\} \Big). \end{align*} This proves (1). Also, \[ K^y_{p+t}\cap U = \bigcup\Big\{\Big[ a+\sum_{k=p+1}^{p+t} \frac{x_k}{n^k}, \Big( a+\sum_{k=p+1}^{p+t} \frac{x_k}{n^k} \Big)+n^{-p-t}\Big] : x_k\in I_{y_k} \Big\}\setminus\{a,a+n^{-p}\}, \] which implies (2). Next, let $p$ be so large that $\#I_{y_k}\geq 2$ whenever $k\geq p$. Let $G\subset U$ be any gap of $K^y_p$. By (2), $U$ clearly contains at least $4$ basic open intervals of $K^{y}_{p+2}$. Since these intervals are disjoint and each of them contains some point in $K^y$, $\diam(K^y\cap U)\geq n^{-p-2}$ (which proves (3)) and $|G|\leq |U|-2n^{-p-2}= (1-2n^{-2})n^{-p}$, which gives (4). \end{proof} It is also worthy of mentioning that the Hausdorff dimension of Bedford-McMullen carpets was obtained by Bedford~\cite{Bed84} and McMullen~\cite{Mcm84} independently as follows: \begin{equation}\label{eq:dimhk} \dimh K = \frac{1}{\log m}\cdot\log\Big( \sum_{j=0}^{m-1} (\# I_j)^{\log m/\log n} \Big). \end{equation} \subsection{Deleted-digit sets} Deleted-digit sets naturally appear in the study of horizontal and vertical slices and projections of Bedford-McMullen carpets. \begin{definition}[Deleted-digit set] Let $n\geq 2$ be an integer. For any non-empty set $\D\subset\{0,1,\ldots,n-1\}$, we write $E(n,\D)$ to be the self-similar set associated with the IFS $\{\frac{x+i}{n}:i\in\D\}$ on $\R$. In other words, \begin{equation*} E(n,\D) = \Big\{ \sum_{k=1}^\infty \varepsilon_kn^{-k}: \varepsilon_k\in\D \Big\}. \end{equation*} \end{definition} A common example is the middle-third Cantor set, where $n=3$ and $\D=\{0,2\}$. Note that for any deleted-digit set $E(n,\D)$, the associated IFS $\{\frac{x+i}{n}:i\in\D\}$ always satisfies the open set condition, see~\cite{Fal14}. Since $\frac{[0,1]+i}{n}\subset[0,1]$ for all $n\geq 1$ and all $0\leq i\leq n-1$, we can use the unit interval $[0,1]$ for the standard iteration process to get $E(n,\D)$. More precisely, letting $E_0(n,\D):=[0,1]$ and recursively define \begin{equation}\label{eq:eknd} E_k(n,\D) := \bigcup_{i\in\D} \frac{E_{k-1}(n,\D)+i}{n}, \quad k\geq 1, \end{equation} we get a decreasing sequence $\{E_{k}(n,\D)\}_{k=0}^\infty$ such that $\bigcap_{k=0}^\infty E_k(n,\D)=E(n,\D)$. It is clear that $E(n,\D)$ has non-empty interior if and only if $\#\D=n$. In particular, if $\dimh E(n,\D)<1$ then it must be totally disconnected. It is easy to check by induction that $E_k(n,\D)$ is a union of intervals of the form $[\frac{i}{n^k},\frac{i+1}{n^k}]$, and each pair of such intervals are either adjacent (that is, they share a common endpoint) or disjoint. Many horizontal and vertical slices of a Bedford-McMullen carpet $K=K(n,m,\Lambda)$ are deleted-digit sets or a finite union of them. For example, if $y=\sum_{k} jm^{-k}$ for some $j\in J$, then we have by~\eqref{eq:expressionofky} that $K^y = E(n,I_j)$. Similarly, if $x=\sum_{k=1}^\infty in^{-k}$ for some $i\in I$, then $K_{x}=E(m,J_i)$. On the other hand, the projections of $K$ onto the principal axes are also deleted-digit sets: \begin{align*} \pi_1(K) &= \Big\{ \sum_{k=1}^\infty \frac{x_k}{n^k}: \Big( \sum_{k=1}^\infty \frac{x_k}{n^k},\sum_{k=1}^\infty \frac{y_k}{m^k} \Big) \in K \Big\} \\ &= \Big\{ \sum_{k=1}^\infty \frac{x_k}{n^k}: \exists \{y_k\} \text{ such that } (x_k,y_k)\in\Lambda \Big\}\\ &= \Big\{ \sum_{k=1}^\infty \frac{x_k}{n^k}: x_k\in I \text{ for all $k$} \Big\} = E(n,I) \end{align*} and similarly, $\pi_2(K)=E(m,J)$. \begin{lemma}\label{lem:fw09} Let $E=E(n,\D)$ be a deleted-digit set with $1<\#\D<n$. If $g$ is a similitude sending $E$ into itself, then the similarity ratio of $g$ is a rational power of $n$. \end{lemma} \begin{proof} This is a special case of~\cite[Theorem 1.1]{FW09}. \end{proof} A generalization of this logarithmic commensurability theorem as follows is one of the main ingredients in our proof of the dependent and self-similar cases. Recall that $N:=\max_{0\leq j\leq m-1}\#I_j$. \begin{proposition}\label{prop:allnthenrational} Let $K=K(n,m,\Lambda)$ be a Bedford-McMullen carpet with $2\leq N\leq n-1$ and let $y\in\pi_2(K)$. Write $\alpha:=\frac{\log N}{\log n}$. If $0<\h^\alpha(K^y)<\infty$ and there is a contraction $h\in\S(\R)$ sending $K^y$ into itself, then the contraction ratio of $h$ is a rational power of $n$. \end{proposition} Some idea of the proof stems from an ongoing work of the author and Rao~\cite{RXtodedone}. \begin{proof} Considering $h^2$ if necessary, we may assume that $h$ is orientation-preserving. If $y\in\pi_2(K)$ has more than one expansion with coefficients in $J$, $y\in\bigcup_{k\geq 1}\frac{\Z}{m^k}\cap[0,1]$. In this case, it is easy to see that $K^y$ is a finite union of scaled copies of $K^0$ and $K^1$, say $K^y=\bigcup_{s\in\Omega} K^{0,s}\cup \bigcup_{t\in\Gamma}K^{1,t}$, where $\Omega,\Gamma\subset\Z$ and \[ K^{0,s}:=\frac{K^0+s}{n^k} \quad\text{and}\quad K^{1,s}:=\frac{K^1+t}{n^k} \] for some large integer $k$. Since $h(K^y)\subset K^y$, by Baire's theorem, some $K^{0,s}$ or $K^{1,s}$, say $K^{0,s}$, must contain an (relative) interior part of $h(K^y)$ and hence of $h(K^{0,s})$. So we can find a scaled copy $E$ of $K^0$ contracting by a factor $n^{-q}$ for some large integer $q$ such that $h(E)\subset K^{0,s}$. Since $K^0=E(n,I_0)$ is a deleted-digit set with positive Hausdorff dimension, by Lemma~\ref{lem:fw09}, the contraction ratio of $h$ is a rational power of $n$. Now suppose $y$ has a unique expansion $y=\sum_{k=1}^\infty y_km^{-k}$, $\{y_k\}\subset J$. We first prove that $\#I_{y_k}=N$ for all large $k$. Suppose on the contrary that $\#I_{y_k}<N$ for infinitely many $k$. Then for any $p\geq 1$, when $\widetilde{M}$ is large enough, we have \begin{equation}\label{eq:newproofofalln} \prod_{k=1}^{\widetilde{M}} \#I_{y_k} \leq (N-1)^pN^{\widetilde{M}-p} = \Big( \frac{N-1}{N} \Big)^p N^{\widetilde{M}}. \end{equation} Since $0<\h^\alpha(K^y)<\infty$, there is a constant $\delta_0$ such that $\h^\alpha_\delta(K^y)>\h^\alpha(K^y)/2>0$ when $0<\delta\leq\delta_0$ (for the definition of $\h^\alpha_\delta$, see~\cite{Fal14}). However, we have by~\eqref{eq:newproofofalln} that \[ \h^\alpha_{n^{-\widetilde{M}}} (K^y) \leq \Big( \frac{N-1}{N} \Big)^p N^{\widetilde{M}} \cdot (n^{-\widetilde{M}})^\alpha = \Big( \frac{N-1}{N} \Big)^p < \frac{1}{2}\h^\alpha(K^y) \] as long as $p$ and $\widetilde{M}$ are large enough. This is a contradiction. Denote by $\rho$ the contraction ratio of $h$. Suppose on the contrary that $\frac{\log\rho}{\log n}\notin\Q$. Let $\varepsilon>0$ be a small fixed constant (will be specified later) and pick positive integers $s,t$ such that $-\varepsilon\leq s\cdot\frac{\log\rho}{-\log n}-t<0$. Equivalently, $n^{-t}<\rho^s\leq n^{\varepsilon-t}$. Since $\#I_{y_k} \leq N\leq n-1$ for all $k$, it is not hard to check that \[ M:= \sup_{p\geq 1}\max\Big\{|B|n^p: B \text{ is a connected component of } K^y_p \Big\} \leq 2N<\infty, \] that is, for all $p$, every connected component of $K^y_p$ contains at most $2N$ basic open intervals of $K^y_p$. Pick a large $p_0\geq 1$ such that there is a connected component $B$ of $K^y_{p_0}$ consisting of exactly $M$ basic open intervals of $K^y_{p_0}$ and $\#I_{y_k}=N$ for all $k\geq p_0$. Note that $B$ is a closed interval and $h^s(K^y)\subset K^y$. We distinguish two cases. {\bf Case 1}: $h^s(B)$ intersects exactly one connected component of $K^y_{p_0+t}$, say $B'$. In this case, $h^s(B\cap K^y)\subset B'\cap K^y$. Since $B$ consists of $M$ basic open intervals of $K^y_{p_0}$ and their intersections with $K^y$ are distinct from one another by only a translation (recall Lemma~\ref{lem:bunchoffacts}(1)), one can find a $\rho^sn^{-p_0}$-separated subset of $h^s(B\cap K^y)$ that has cardinality $M$. Since $\rho^sn^{-p_0}>n^{-p_0-t}$, every basic open interval of $K^y_{p_0+t}$ in $B'$, which has length $n^{-p_0-t}$, contains at most one point in that subset. Thus by the maximality of $M$, $B'$ contains exactly $M$ basic open intervals of $K^y_{p_0+t}$. Recall that $0<\h^\alpha(K^y)<\infty$. Writing $c$ to be the $\alpha$-Hausdorff measure of the intersection of $K^y$ and any basic open interval in $K^y_{p_0+t}$, we see from Lemma~\ref{lem:bunchoffacts}(1) that $0<c<\infty$ and $\h^\alpha(B'\cap K^y)=Mc$. On the other hand, by Lemma~\ref{lem:bunchoffacts}(2), each basic open interval in $B$ contains exactly $\prod_{k=p_0+1}^{p_0+t} \#I_{y_k} = N^{t}$ ones in $K^y_{p_0+t}$. So $\h^\alpha(B\cap K^y)=MN^tc$ and \begin{align} \h^\alpha(h^s(B\cap K^y)) = \rho^{\alpha s}\h^\alpha(B\cap K^y) &= \rho^{\alpha s}\cdot MN^tc \notag \\ &> n^{-\alpha t}MN^tc \notag \\ &= N^{-t}\cdot M\cdot N^tc = Mc = \h^\alpha(B'\cap K^y), \label{eq:rationpropcase1} \end{align} which contradicts that $h^s(B\cap K^y)\subset B'\cap K^y$. {\bf Case 2}: $h^s(B)$ intersects at least two connected components of $K^y_{p_0+t}$. In this case, $h^s(B)$ contains a hole between those components. More precisely, there exists some integer $i$ such that $(\frac{i}{n^{p_0+t}},\frac{i+1}{n^{p_0+t}})\cap K^y=\varnothing$ and $(\frac{i}{n^{p_0+t}},\frac{i+1}{n^{p_0+t}})\subset h^s(B)$. Write $V:=(\frac{i}{n^{p_0+t}},\frac{i+1}{n^{p_0+t}})$. Comparing the length, we can find either exactly one or two adjacent basic open intervals of $K^y_{p_0}$ in $B$ of which the images under $h^s$ meet $V$. {\bf Case 2.1}: $V\subset h^s(\widetilde{I})$ for only one basic open interval $\widetilde{I}$ in $B$ (see Figure~\ref{fig:vi1i2}(A)). Then, since $h^s(\widetilde{I} \cap K^y)\subset K^y$, $h^s(\widetilde{I}\cap K^y)\cap V=\varnothing$. This in turn tells us that $\widetilde{I}$ contains a gap of length $\geq|h^{-s}(V)|$. Therefore, \begin{equation}\label{eq:rationpropcase2} \max\{|G|n^{p_0}: G \text{ is a gap of } K^y_{p_0}\} \geq |h^{-s}(V)|n^{p_0} = \rho^{-s}n^{-p_0-t}\cdot n^{-p_0} > n^{-\varepsilon}. \end{equation} Choosing $\varepsilon$ small enough at the beginning, this contradicts Lemma~\ref{lem:bunchoffacts}(4). {\bf Case 2.2}: There are two basic open intervals $\widetilde{I}_1,\widetilde{I}_2$ of $K^y_{p_0}$ in $B$ such that $V\cap h^s(\widetilde{I}_i)\neq\varnothing$, $1\leq i\leq 2$. Without loss of generality, assume that $\widetilde{I}_1$ is to the left of $\widetilde{I}_2$. Denote by $a_i$ the left endpoint of $h^s(\widetilde{I}_i)$. See Figure~\ref{fig:vi1i2}(B) for an illustration. \begin{figure}[htbp] \centering \subfloat[One-interval case] { \begin{minipage}[t]{155pt} \centering \begin{tikzpicture}[scale=1] \draw[thick] (0.1,0) to (1.9,0); \node at(1,0.4) {$h^s(\widetilde{I})$}; \node[draw,shape=circle,inner sep=1.5pt,thick] at(0,0) {}; \node[draw,shape=circle,inner sep=1.5pt,thick] at(2,0) {}; \node[draw,shape=circle,inner sep=1.5pt,thick] at(0.2,-1) {}; \node[draw,shape=circle,inner sep=1.5pt,thick] at(1.8,-1) {}; \draw[thick] (0.3,-1) to (1.7,-1); \node at(1,-1.3) {$V$}; \draw[thick,dashed] (0.2,-1) to (0.2,0); \draw[thick,dashed] (1.8,-1) to (1.8,0); \end{tikzpicture} \end{minipage} } \subfloat[Two-intervals case] { \begin{minipage}[t]{155pt} \centering \begin{tikzpicture}[scale=1] \draw[thick] (0.1,0) to (1.9,0); \draw[thick] (2.1,0) to (3.9,0); \node at(1,0.4) {$h^s(\widetilde{I}_1)$}; \node at(3,0.4) {$h^s(\widetilde{I}_2)$}; \node[draw,shape=circle,inner sep=1.5pt,thick] at(0,0) {}; \node[draw,shape=circle,inner sep=1.5pt,thick] at(2,0) {}; \node[draw,shape=circle,inner sep=1.5pt,thick] at(4,0) {}; \node at(0,-0.3) {$a_1$}; \node at(2,-0.3) {$a_2$}; \node[draw,shape=circle,inner sep=1.5pt,thick] at(1,-1) {}; \node[draw,shape=circle,inner sep=1.5pt,thick] at(2.6,-1) {}; \draw[thick] (1.1,-1) to (2.5,-1); \node at(1.8,-1.3) {$V$}; \draw[thick,dashed] (1,-1) to (1,0); \draw[thick,dashed] (2,-1) to (2,0); \draw[thick,dashed] (2.6,-1) to (2.6,0); \end{tikzpicture} \end{minipage} } \caption{An illustration of the local behavior of Case 2} \label{fig:vi1i2} \end{figure} By Lemma~\ref{lem:bunchoffacts}(1), \begin{align*} \big( a_1,a_1+|h^s(\widetilde{I}_2)\cap V| \big)\cap K^y &= \big( a_2,a_2+|h^s(\widetilde{I}_2)\cap V| \big)\cap K^y \\ &= h^s(\widetilde{I}_2)\cap V \cap K^y \\ &= \varnothing. \end{align*} Thus \[ h^s(\widetilde{I}_1)\cap K^y \subset (h^s(\widetilde{I}_1)\setminus V) \setminus \big( a_1,a_1+|h^s(\widetilde{I}_2)\cap V| \big). \] But the right hand side is an interval of length \begin{equation}\label{eq:rationpropcase3} |h^s(\widetilde{I}_1)| -|h^s(\widetilde{I_1})\cap V|- |h^s(\widetilde{I}_2)\cap V| = \rho^s n^{-p_0}-|V| < n^{\varepsilon-p_0-t}-n^{-p_0-t}. \end{equation} Choosing $\varepsilon$ small enough at the beginning, this contradicts Lemma~\ref{lem:bunchoffacts}(3). \end{proof} For convenience, for any compact set $A\subset\R$ and $a\in A$, we call $a$ a \emph{left} (resp. \emph{right}) \emph{isolated point} of $A$ if there is some $r>0$ such that $(a-r,a)\cap A=\varnothing$ (resp. $(a,a+r)\cap A=\varnothing$). \begin{corollary}\label{cor:leftrightendpt} Let $K,N,y,h$ be as in Proposition~\ref{prop:allnthenrational}. Then there are left and right isolated points $a,b$ of $K^y$ such that $h(a),h(b)$ (or $h(b),h(a)$ if $h$ is orientation-reversing) are also left and right isolated points of $K^y$. Moreover, we can guarantee that $\pi_2(R_k)=\pi_2(R'_k)$ for all large $k$, where $R_k,R'_k$ are two level-$k$ rectangles with $(a,y)\in\varphi_{R_k}(K)$ and $(b,y)\in\varphi_{R'_k}(K)$, respectively. \end{corollary} Roughly speaking, the ``moreover'' part just requires that the two points $(a,y),(b,y)$ lie in the same row at all stages during the iteration process. This is needed for some technical reason. \begin{proof} Suppose first that $y$ has a unique expansion with coefficients in $J$. For the first statement, it suffices to prove it when $h$ is orientation-preserving. In fact, if $h$ is orientation-reversing, then $h^2$ is orientation-preserving. If $a,h^2(a)$ are both left (resp. right) isolated points of $K^y$, then $h(a)$ must be a right (resp. left) isolated point and we are done. Similarly as before, let $\rho$ be the contraction ratio of $h$. By Proposition~\ref{prop:allnthenrational}, $\rho=n^{-p/q}$ for some positive integers $p,q$. Applying the argument in the proof of Proposition~\ref{prop:allnthenrational} to $s=q$, $t=p$ (so $\varepsilon=s\cdot\frac{\log\rho}{\log n}-t=0$), we see that Case 2 in that proof is impossible because: (1) in Case 2.1, the right hand side of~\eqref{eq:rationpropcase2} now equals $1$, i.e., $\widetilde{I}$ contains a gap of the same length as itself, which contradicts that $\widetilde{I}\cap K^y\neq\varnothing$; (2) in Case 2.2 the right hand side of~\eqref{eq:rationpropcase3} now equals $0$, i.e., $|h^q(\widetilde{I_1}\cap K^y)|=0$, which contradicts that $\h^\alpha(h^q(\widetilde{I_1}\cap K^y))>0$. On the other hand, for Case 1 in that proof, we now have $\h^\alpha(h^q(B\cap K^y))=\h^\alpha(B'\cap K^y)$. We claim that \begin{equation}\label{eq:hqkyhasnonemptyinte} h^q(B\cap K^y)=B'\cap K^y. \end{equation} Otherwise, since $h^q(B\cap K^y)\subset B'\cap K^y$, there exists some $x\in B'\cap K^y\setminus h^q(B\cap K^y)$. So we can find a large $t$ and a basic open interval $W$ of $K^y_t$ such that $x\in W\cap B'$ but $W\cap h^q(B\cap K^y)=\varnothing$. However, since $0<\h^\alpha(K^y)<\infty$, we have again by Lemma~\ref{lem:bunchoffacts}(1) that $\h^\alpha(W\cap K^y)>0$. Thus \[ \h^\alpha(h^q(B\cap K^y)) \leq \h^\alpha(B'\cap K^y)-\h^\alpha(W\cap K^y) < \h^\alpha(B'\cap K^y), \] which is a contradiction. By the claim, $h^q$ maps all the left and right isolated points of $B\cap K^y$ to the corresponding isolated points of $B'\cap K^y$, respectively. Pick any pair of them, say $a'$ and $b'$. It is easy to check by contradiction that $(h^{q})^{-1}(a'),(h^{q})^{-1}(b')$ are left and right isolated points of $K^y$, respectively. This establishes the first requirement. For the second one, since $y$ has a unique expansion, $R_k$ and $R'_k$ are unique for all $k$. Moreover, $y$ belongs to the interior of both of $\pi_2(R_k)$ and $\pi_2(R'_k)$, which implies that $\pi_2(R_k)=\pi_2(R'_k)$. Now, suppose $y$ has more than one expansion. Similarly as in the proof of Proposition~\ref{prop:allnthenrational}, we can write $K^y=\bigcup_{i\in\Omega} K^{0,i} \cup \bigcup_{j\in\Gamma} K^{1,j}$, where $\Omega,\Gamma\subset\Z$ and \[ K^{0,i}:=\frac{K^0+i}{n^k} \quad\text{and}\quad K^{1,j}:=\frac{K^1+j}{n^k} \] for some large integer $k$. By Baire's theorem again, we may assume without loss of generality that some $K^{0,i}$ contains an interior part of $h(K^y)$. In particular, there is a scaled copy of $K^{0,i}$, say $E=\frac{K^0+z}{n^c}\subset K^{0,i}$, such that $h(E)\subset K^{0,i}$. Note that \[ h(E)\subset K^{0,i} \Longleftrightarrow h\Big( \frac{K^0+z}{n^c} \Big) \subset \frac{K^0+i}{n^k}. \] We can easily reformulate this as $g(K^0)\subset K^0$ for some similitude $g$ with contraction ratio $\rho n^{k-c}$. Since $0$ has a unique expansion, as in the first part of this proof, there are nondegenerate open intervals $U,V$ such that $g(U\cap K^0)=V\cap K^0\neq\varnothing$ (recall~\eqref{eq:hqkyhasnonemptyinte}). In particular, $g$ maps all the one-sided isolated points of $K^0$ in $U$ to those of $K^0$ in $V$. Since the inclusion $g(K^0)\subset K^0$ is just a reformulation of $h(E)\subset K^{0,i}$, $h$ maps all one-sided isolated points of $K^{0,i}$ in $U'$ (for some $U'$) to one-sided isolated points of $K^{0,i}$ in $V'$ (for some $V'$). It remains to prove that these points are also one-sided isolated points of $K^y$ (for all large $k$, one can pick $R_k,R'_k$ to be those that lie above adjacent to $\R\times\{y\}$ and thus $\pi_2(R_k)=\pi_2(R'_k)$). To get the desired statement, note first that we may pick $V'$ properly so that \begin{equation}\label{eq:koslocallygoodv} K^{0,i}\cap V'=K^y\cap V'. \end{equation} In fact, if $j\neq i$ for all $j\in\Gamma$, then we already have~\eqref{eq:koslocallygoodv}. If some $j$ equals $i$, then $K^{0,i}$ and $K^{1,j}=K^{1,i}$ may overlap. However, note that $\dimh K^{0,i}\geq\dimh K^y\geq\dimh K^{1,i}$ (recall that $h(K^y)\subset K^{0,i}$), we have either $K^{1,i}=K^{0,i}$ or these two deleted-digit sets have different patterns (in particular, $K^{0,i}\neq K^{1,i}$). So there is a small open interval $V''\subset V'$ such that $(K^{0,i}\cap V'') \cap K^{1,i}=\varnothing$ but $K^{0,i}\cap V''\neq\varnothing$. Replacing $V'$ with $V''$ gives us~\eqref{eq:koslocallygoodv}. In particular, all left (resp. right ) isolated points of $K^{0,i}$ in $V'$ are left (resp. right) isolated points of $K^y$. Next, if a one-sided isolated point $a$ of $K^{0,i}$ in $U'$ is not a one-sided isolated point of $K^y$, there is a sequence $\{x_n\}\subset K^y$ such that $x_n\to a$ from the left or from the right. Then $h(x_n)\to h(a)$ from one direction. Since $h(x_n)\in K^y$, this contradicts the fact that $h(a)$ is a one-sided isolated point of $K^y$ in $V'$, which completes the proof. \end{proof} \section{The independent case} One significant benefit of the independence assumption regarding $n$ and $m$ is that it elucidates the dimensions of the orthogonal projections of $K$ onto all lines. \begin{lemma}[\cite{FJS10}]\label{lem:projofcarpet} Let $K=K(n,m,\Lambda)$ be a Bedford-McMullen carpet. If $\frac{\log n}{\log m}\notin\Q$, then $\dimh \pi_L(K)=\min\{\dimh K,1\}$ for every oblique line $L$. \end{lemma} \begin{theorem}\label{thm:notoblique} Let $K,f$ be as in Theorem~\ref{thm:main1}. If $\frac{\log n}{\log m}\notin\Q$, then $f$ is not oblique. \end{theorem} We distinguish three cases. Recall that $N=\max_j\#I_j$. \paragraph{\noindent{\bf Case 1}} $\#J<m$. In other words, there is an empty row in the initial pattern of $K$. \begin{proof}[Proof of Theorem~\ref{thm:notoblique} under Case 1] If $f$ is oblique, then by Lemma~\ref{lem:projofcarpet}, \begin{equation}\label{eq:pitklowerbd} \dimh \pi_t(K) \geq \dimh \pi_t(f(K)) = \min\{\dimh K,1\}, \quad t=1,2. \end{equation} If there is some $j$ such that $\#I_j\geq 2$, then \begin{align*} \min\{1,\dimh K\} &> \frac{1}{\log m}\cdot \log(\# J) &&\text{(since $\#J<m$ and~\eqref{eq:dimhk})}\\ &= \dimh E(m,J) \\ & =\dimh \pi_2(K), &&\text{(since $E(m,J)=\pi_2(K)$)} \end{align*} which contradicts~\eqref{eq:pitklowerbd}. However, if $\#I_j=1$ for all $j\in J$, then $\#\Lambda=\#J$. Therefore, \[ \dimh\pi_1(K) = \dimh E(n,I) \leq \frac{\log\#\Lambda}{\log n} < \frac{\log\#J}{\log m} = \dimh\pi_2(K) \leq \min\{\dimh K,1\}, \] which again contradicts~\eqref{eq:pitklowerbd}. \end{proof} \paragraph{\noindent{\bf Case 2}} $\#J=m$ and $N=n$. In other words, there is no empty row but a full row in the initial pattern of $K$. \begin{proof}[Proof of Theorem~\ref{thm:notoblique} under Case 2] In this case, pick $j$ such that $\#I_j=n$. Writing $y:=\sum_{k=1}^\infty jm^{-k}$, we have seen that $K^{y}=E(n,I_j)$, which is a horizontal line segment. So if $f$ is oblique, then $K\supset f(K)$ contains an oblique segment, which contradicts Proposition~\ref{prop:noobliquelines}. \end{proof} \paragraph{\noindent{\bf Case 3}} $\#J=m$ but $N<n$. In other words, one can find a selected rectangle and an unselected one in each row in the initial pattern of $K$. \begin{proof}[Proof of Theorem~\ref{thm:notoblique} under Case 3] In this case, $K$ does not contain any horizontal line segment. On the other hand, $\pi_2(K)=[0,1]$ and hence every horizontal slice of $K$ is non-empty. Let $0<\lambda\leq 1$ be the similarity ratio of $f$. If $f$ is oblique, without loss of generality, assume that $f(x\text{-axis})$ is supported on a line of slope $u>0$. Fix a large integer $p>0$ so that $m^{-p}\lambda\leq\frac{1}{9m^2n}$. Let us pick a sequence $\{R_k\}_{k=1}^\infty$, where each $R_k$ is a level-$k$ rectangle and $R_{k+1}\subset R_k$ for all $k\geq 1$. Write $a_k$ to be the left bottom vertex of $R_k$ and $Q_k$ to be any $n^{-k+p}\times m^{-k+p}$ grid rectangle containing $f(a_k)$. For convenience, we also let $\ell^k,\ell_k$ be the bottom and left edges of $R_k$, respectively. So $\ell^k\cap\ell_k=\{a_k\}$. See Figure~\ref{fig:rkfrkinvfrk} for an illustration. Below we record two simple but key facts. Here we abuse notation slightly by writing $\varphi_{Q_k}$ to be the natural affine map sending $[0,1]^2$ onto $Q_k$. \begin{enumerate} \item Since $\ell^k$ is parallel to the $x$-axis and $f(x\text{-axis})$ is supported on a line of slope $u>0$, the line supporting $\varphi_{Q_k}^{-1}f(\ell^k)$ has slope $\frac{m^{k-p}u}{n^{k-p}}$, which tends to $0$ as $k\to\infty$. Moreover, $\varphi_{Q_k}^{-1}f(\ell^k)$ has length \begin{align} |\varphi_{Q_k}^{-1}f(\ell^k)| &= \sqrt{n^{2(k-p)}|\pi_1(f(\ell^k))|^2+m^{2(k-p)}|\pi_2(f(\ell^k))|^2} \notag \\ &= \sqrt{n^{2(k-p)}\cdot\lambda^2|\ell^k|^2\cdot\frac{1}{1+u^2} + m^{2(k-p)}\cdot\lambda^2|\ell^k|^2\cdot\frac{u^2}{1+u^2}} \notag \\ &= \sqrt{n^{2(k-p)}\cdot\frac{\lambda^2}{n^{2k}}\cdot\frac{1}{1+u^2} + m^{2(k-p)}\cdot\frac{\lambda^2}{n^{2k}}\cdot\frac{u^2}{1+u^2}} \notag \\ &\to \frac{n^{-p}\lambda}{\sqrt{1+u^2}}, \quad k\to\infty \label{eq:lengthofvflk} \end{align} \item Similarly, the line supporting $\varphi_{Q_k}^{-1}f(\ell_k)$ has slope $-\frac{m^{k-p}}{n^{k-p}u}$, which also tends to $0$ as $k\to\infty$. Moreover, $\varphi_{Q_k}^{-1}f(\ell_k)$ has length \begin{align*} |\varphi_{Q_k}^{-1}f(\ell_k)| &= \sqrt{n^{2(k-p)}|\pi_1(f(\ell_k))|^2+m^{2(k-p)}|\pi_2(f(\ell_k))|^2} \\ &= \sqrt{n^{2(k-p)}\cdot\frac{\lambda^2}{m^{2k}}\cdot\frac{1}{1+u^{-2}}+m^{2(k-p)}\cdot\frac{\lambda^2}{m^{2k}}\cdot\frac{u^{-2}}{1+u^{-2}}} \\ &\to\infty, \quad k\to\infty. \end{align*} \end{enumerate} Roughly speaking, for all large $k$, $\varphi_{Q_k}^{-1}f(R_k)$ is a very flat parallelogram with one edge short and the other one pretty long. \begin{figure}[htbp] \centering \begin{tikzpicture}[scale=0.6] \draw[thick] (0,0) to (1,0) to (1,5) to (0,5) to (0,0); \node at (0,0)[circle,fill,inner sep=1.5pt, red]{}; \node at (-0.4,-0.4)[circle,inner sep=2pt,red]{$a_k$}; \node at (0.5,2.5)[circle,inner sep=2pt]{$R_k$}; \node at (2,2.5)[circle, inner sep=2pt]{$\xrightarrow{f}$}; \draw[thick] (3,20/11+3/2) to (3+10/11,1.5) to (3+14/11,1.5+2/11) to (3+4/11,3.5) to (3,20/11+3/2); \node at (3+10/11,1.5)[circle,fill, inner sep=1.5pt, red]{}; \node at (3+10/11+0.9,1.2)[circle, inner sep=2pt, red]{$f(a_k)$}; \draw[thick,dashed] (3,0) to (4,0) to (4,5) to (3,5) to (3,0); \node at (3.5,0.5)[circle,inner sep=2pt]{$Q_k$}; \node at (5.5,2.5)[circle, inner sep=2pt]{$\xrightarrow{\varphi_{Q_k}^{-1}}$}; \draw[thick] (7,1.5+5/4) to (7+25/4,1.5) to (7+15/2,1.5+1/4) to (7+5/4,3) to (7,1.5+5/4); \node at (7+13/4,1.3)[circle,inner sep=2pt]{$\varphi_{Q_k}^{-1}f(R_k)$}; \node at (7+25/4,1.5)[circle,fill, inner sep=1.5pt,red]{}; \node at (7+25/4,0.9)[circle, inner sep=2pt,red]{$\widetilde{a}_k$}; \node at (-0.4,2) {$\ell_k$}; \node at (0.5,-0.4) {$\ell^k$}; \end{tikzpicture} \caption{The evolution from $R_k$ to $f(R_k)$ to $\varphi_{Q_k}^{-1}f(R_k)$ (color online)} \label{fig:rkfrkinvfrk} \end{figure} Write $\widetilde{a}_k :=\varphi_{Q_k}^{-1}f(a_k)$. Since $f(a_k)\in Q_k$, $\widetilde{a}_k\in [0,1]^2$. Fix a large $k$ (will be specified later) so that $\frac{m^{k-p}}{n^{k-p}}\cdot\max\{u^{-1},u\}<\lambda$ and $|\varphi_{Q_k}^{-1}f(\ell^k)|\leq \frac{2n^{-p}\lambda}{\sqrt{1+u^2}}$ (recall~\eqref{eq:lengthofvflk}). Let us truncate the parallelogram $\varphi_{Q_k}^{-1}f(R_k)$ as $\bigcup_{t\in\Z} \widetilde{E}_t$, where \[ \widetilde{E}_t := \varphi_{Q_k}^{-1}f(R_k) \cap \{x\in\R^2: \pi_1(x)\in[-t,-t+1]\}. \] Note that for every $t$, \begin{align} |\pi_2(\widetilde{E}_t)| &\leq |\pi_2(\varphi_{Q_k}^{-1}f(R_k))| \notag \\ &\leq |\pi_2(\varphi_{Q_k}^{-1}f(\ell^k))|+|\pi_2(\varphi_{Q_k}^{-1}f(\ell_k))| \notag \\ &= m^{k-p}\cdot\frac{\lambda}{n^k}\cdot\frac{u}{\sqrt{1+u^2}}+m^{k-p}\cdot\frac{\lambda}{m^k}\cdot\frac{u^{-1}}{\sqrt{1+u^{-2}}} < 2m^{-p}\lambda.\label{eq:lessthan13m} \end{align} Let us prove Case 3 under the following Condition A first and then show its validity. \paragraph{{\bf Condition A}} There are integers $t_0\geq -1$ and $0\leq j_0\leq m$ such that $|\pi_1(\widetilde{E}_{t_0})|=1$ while $\pi_2(\widetilde{E}_{t_0})\subset [\tfrac{j_0}{m},\tfrac{j_0+1}{m}]$. Assume that Condition A holds. Let $j'_0:=j_0\pmod {m}$. Recall that $\varphi_{Q_k}^{-1}f(\ell_k)$ is supported on a line of slope $-\frac{m^{k-p}}{n^{k-p}u}$, say $g(x)=-\frac{m^{k-p}}{n^{k-p}u}x+c$ (the equation of that line). Since $\#I_{j'_0}<n$, there is some $0\leq i_0\leq n-1$ such that $K$ does not intersect the open rectangle $(\frac{i_0}{n},\frac{i_0+1}{n})\times(\frac{j'_0}{m},\frac{j'_0+1}{m})=:U$. Write $e$ to be the point $(-t_0,\lfloor\frac{j_0}{m}\rfloor)$ and $x_0:=-t_0+\frac{i_0+1/2}{n}$. Then the point $\xi_0:=(x_0,g(x_0))$ lies on the segment $\varphi_{Q_k}^{-1}f(\ell_k)$, and \begin{align*} U+e &= \Big( \frac{i_0}{n}-t_0,\frac{i_0+1}{n}-t_0 \Big)\times \Big( \frac{j'_0}{m}+\lfloor\frac{j_0}{m}\rfloor, \frac{j'_0+1}{m}+\lfloor\frac{j_0}{m}\rfloor \Big) \\ &= \Big( \frac{i_0}{n}-t_0,\frac{i_0+1}{n}-t_0 \Big)\times\Big( \frac{j_0}{m}, \frac{j_0+1}{m} \Big). \end{align*} Since $\pi_2(\widetilde{E}_{t_0})\subset [\tfrac{j_0}{m},\tfrac{j_0+1}{m}]$, it is not hard to check that \begin{equation}\label{eq:sliceinupluse} \xi_0+\varphi_{Q_k}^{-1}f(\ell^k) \subset U+e. \end{equation} See Figure~\ref{fig:localofthehole} for an illustration. \begin{figure}[htbp] \centering \begin{tikzpicture}[scale=1] \draw[thick,dashed] (0,0) to (0,4); \draw[thick,dashed] (4,0) to (4,4); \draw[thick,dashed] (-1.5,1) to (5.5,1); \draw[thick,dashed] (-1.5,2.5) to (5.5,2.5); \node at (2,3.5) {$\widetilde{E}_{t_0}$}; \draw[thick] (-1, 1.75) to (5,1.45); \draw[thick] (-1,2.05) to (5.5,1.725); \draw[thick,red] (5,1.45) to (5.5,1.725); \draw[thick,red] (2,1.6) to (2.5,1.875); \node at (2,1.6)[circle,fill, inner sep=1.5pt]{}; \node at (2,1.3) {$\xi_0$}; \node at (5.8,1.3) [red] {$\varphi_{Q_k}^{-1}f(\ell^k)$}; \node at (-1.9,1) {$\tfrac{j_0}{m}$}; \node at (-1.9,2.5) {$\tfrac{j_0+1}{m}$}; \draw[thick,blue] (1.4,1) to (2.6,1) to (2.6,2.5) to (1.4,2.5) to (1.4,1); \node at (2,0.7) [blue] {$U+e$}; \end{tikzpicture} \caption{$\varphi_{Q_k}^{-1}f(\ell^k)+\xi_0$ is contained in the hole $U+e$ (color online)} \label{fig:localofthehole} \end{figure} Since $\xi_0+\varphi_{Q_k}^{-1}f(\ell^k)$ is a translated copy of $\varphi_{Q_k}^{-1}f(\ell^k)$ and is contained in $\varphi_{Q_k}^{-1}f(R_k)$, $f^{-1}\varphi_{Q_k}(\xi_0)+\ell^k$ is a translated copy of $\ell^k$ and is contained in $R_k$. Since $\ell^k$ is the bottom edge of the level-$k$ rectangle $R_k$, $\ell^k\cap \varphi_{R_k}(K)=\varphi_{R_k}(K^0)$. So there is $y_0\in[0,1]$ such that \[ (f^{-1}\varphi_{Q_k}(\xi_0)+\ell^k)\cap\varphi_{R_k}(K) = \varphi_{R_k}(K^{y_0}\times\{y_0\})\neq\varnothing, \] where the non-empty conclusion is because $K^{y}\neq\varnothing$ for all $y$ (as pointed out at the beginning of Case 3). This in turn implies that \begin{equation}\label{eq:tranisanotherhorislice} (\xi_0+\varphi_{Q_k}^{-1}f(\ell^k)) \cap \varphi_{Q_k}^{-1}f\varphi_{R_k}(K) = \varphi_{Q_k}^{-1}f\varphi_{R_k}(K^{y_0}\times\{y_0\}) \neq\varnothing. \end{equation} Thus \begin{align*} \varnothing &\neq (\xi_0+\varphi_{Q_k}^{-1}f(\ell^k)) \cap \varphi_{Q_k}^{-1}f\varphi_{R_k}(K) && \text{(just~\eqref{eq:tranisanotherhorislice})} \\ &\subset (U+e) \cap \varphi_{Q_k}^{-1}f\varphi_{R_k}(K) && \text{(by~\eqref{eq:sliceinupluse})} \\ &\subset (U+e) \cap \varphi_{Q_k}^{-1}f(K) \\ &\subset (U+e) \cap \Big( \bigcup_{\substack{\text{$R$ is a level-$(k-p)$ rectangle} \\ \text{$R\cap f(K)\neq\varnothing$}}} \varphi_{Q_k}^{-1}\varphi_R(K) \Big) && \text{(since $f(K)\subset K$)} \\ &\subset (U+e) \cap \bigcup_{z\in\Z^2}(K+z) \\ &= (U+e) \cap (K+e) = \varnothing, && \text{(since $e$ is a lattice point)} \end{align*} which leads to a contradiction. It remains to prove Condition A. The key is to note that for every $y$, $\varphi_{Q_k}^{-1}f(R_k)\cap(\R\times\{y\})$ (which is a horizontal slice of the parallelogram) has length at most $Cn^{-p}\lambda$. To see this, just look at Figure~\ref{fig:thetriangle} and recall that the two acute edges of that triangle has slope $\frac{m^{k-p}u}{n^{k-p}}$ and $-\frac{m^{k-p}}{n^{k-p}u}$, respectively. \begin{figure}[htbp] \centering \begin{tikzpicture}[scale=1] \draw[thick] (0,0) to (2,0.5); \draw[thick,dashed] (2,-0.25) to (-4,0.5); \draw[thick,dashed] (2,0.5) to (6,0); \draw[thick] (0,0) to (6,0); \draw[thick,dashed] (2,0.5) to (2,0); \node at (0,-0.3) {$\widetilde{a}_k$}; \node at (0.45,0.56) {$\varphi_{Q_k}^{-1}f(\ell^k)$}; \end{tikzpicture} \caption{Length of horizontal slices of $\varphi_{Q_k}^{-1}f(R_k)$} \label{fig:thetriangle} \end{figure} If Condition A holds for $t_0=1$ and some suitable $j_0$ then we are done. Otherwise, there exists some $j_*$ such that $\widetilde{E}_{1}\cap(\R\times\{\frac{j_*}{m}\})\neq\varnothing$. Since $|\varphi_{Q_k}^{-1}f(R_k)\cap(\R\times\{\frac{j_*}{m}\})|\leq Cn^{-p}\lambda$, picking $k$ large at the beginning, it is not hard to check that Condition A holds for $t_*$ and $j_*$, where $t_*:= \min\{t\geq 2: \widetilde{E}_t\cap(\R\times\{\tfrac{j_*}{m}\})=\varnothing\}$. ll[teal,opacity=0.2] (0,0.9) to (4,0.5) to (4,0.9) to (0,1.3) to (0,0.9); ll[purple,opacity=0.2] (0,1.8) to (4,1.4) to (4,1.8) to (0,2.2) to (0,1.8); ll[purple,opacity=0.2](0,0.4) to (4,0) to (4,-0.4) to (0,0) to (0,0.4); \end{proof} \begin{remark}\label{rem:deindwork} Note that in the proof of Cases 2 and 3, we do not need the independence assumption $\frac{\log n}{\log m}\notin\Q$ but only the fact that $n>m$. This will save us much effort in the dependent self-affine case. \end{remark} \section{The dependent self-affine case} In this section, we fix a Bedford-McMullen carpet $K=K(n,m,\Lambda)$ that is not supported in any line and assume that $\frac{\log n}{\log m}\in\Q$. Without loss of generality, write $n=m^{p/q}$, where $p,q\in\Z^+$ are coprime. It follows from $n>m$ that $p>q$. Recall that $N=\max_{j} \#I_j$. Unlike the independent case, now we only know the dimension of the projections of $K$ in almost all directions (by Marstrand's projection theorem). This limitation renders the argument in Case 1 of Theorem~\ref{thm:notoblique} invalid. Instead, we adopt a rescaling approach as follows. \begin{lemma}\label{lem:dep1} Let $f\in\S(\R^2)$ be a contracting map. If $f(K)\subset K$, then we can find $g\in\mathcal{S}(\R^2)$ and a compact set $C\subset\R$ with $\dimh C \leq \frac{\log N}{\log n}$ such that $g(K)\subset C\times\R$. Moreover, if $f$ is oblique, then $g$ can be picked oblique as well. \end{lemma} \begin{proof} Let $\lambda$ be the contraction ratio of $f$. For all positive integers $t$ with $\lambda^t\leq m^{-1}$, write \begin{equation}\label{eq:ktinlemma4.1} k_t := \max\{k\geq 0: m^{-pk-1}\geq \lambda^t\}. \end{equation} Since the contraction ratio of $f^t$ is $\lambda^t$, $K\subset[0,1]^2$ and $f^t(K)\subset K$, $f^t(K)$ is contained in at most $4$ squares of side length $m^{-pk_t}=n^{-qk_t}$. More precisely, there are $x\in n^{-qk_t}\mathbb{N}, y\in m^{-pk_t}\mathbb{N}$ such that \[ f^t(K) \subset (x,x+2n^{-qk_t}) \times (y,y+2m^{-pk_t}) =: Q_t. \] Denote by $h_t$ the homothety sending $(0,1)^2$ to the square $(x,x+n^{-qk_t}) \times (y,y+m^{-pk_t})=:Q'_t$, the ``left bottom square'' of $Q_t$. Note that for all $t$, \[ h_t^{-1}f^t(K) \subset h_t^{-1}(Q_t) = (0,2)^2. \] The contraction ratio of $h_t^{-1}f^t$ equals $n^{qk_t}\lambda^t$, which is at most $n^{qk_t}\cdot m^{-pk_t-1}=m^{-1}$ and at least $n^{qk_t}\cdot m^{-p(k_t+1)-1}=m^{-p-1}$. So the sequence $\{h_t^{-1}f^t\}_{t}$ has accumulation points in $\mathcal{S}(\R^2)$. For simplicity, assume that $h_t^{-1}f^t$ converges to some $g$. Suppose $g(K)\cap(0,1)^2\neq\varnothing$. Then $h_t^{-1}f^t(K)\cap(0,1)^2\neq\varnothing$ for all large $t$. For each of these $t$, write $y=\sum_{k=1}^{pk_t}y_km^{-k}$, where $y_k\in\{0,\ldots,m-1\}$. Since $p>q$, the collection of all level-$pk_t$ rectangles in $\overline{Q'_t}$, denoted by $\mathcal{R}_t$, is \begin{equation*} \Big\{ \Big[ x+\sum_{k=qk_t+1}^{pk_t} \frac{x_k}{n^k}, \Big( x+\sum_{k=qk_t+1}^{pk_t} \frac{x_k}{n^k} \Big)+n^{-pk_t} \Big] \times [y,y+m^{-pk_t}]: (x_k,y_k)\in\Lambda \Big\}. \end{equation*} Then we have \begin{align} h_t^{-1}f^t(K) \cap (0,1)^2 &= h_t^{-1}f^t(K) \cap h_t^{-1}(Q'_t) \notag \\ &= h_t^{-1}(f^t(K)\cap Q'_t) \notag \\ &\subset h_t^{-1}\Big( \bigcup\mathcal{R}_t \Big) \label{eq:rtisnotempty} \\ &\subset \bigcup_{\substack{(x_k,y_k)\in\Lambda, \notag \\ qk_t+1\leq k\leq pk_t}} \Big[ \sum_{k=qk_t+1}^{pk_t} \frac{x_k}{n^{k-qk_t}}, n^{-(p-q)k_t}+ \sum_{k=qk_t+1}^{pk_t} \frac{x_k}{n^{k-qk_t}} \Big] \times\R \notag \\ &=: E_t \times\R. \label{eq:htinversesupset} \end{align} Moreover, write $\j_t:=y_{qk_t+1}\cdots y_{pk_t}$, which is a word of length $(p-q)k_t\to\infty$ as $t\to\infty$. Using Cantor's diagonal argument, it is not hard to find an infinite word $\j=j_1j_2\cdots\in J^\infty$ such that for all $M\geq 1$, there is a large $t$ such that $|\j_t\wedge\j|\geq M$, where $\j_t\wedge\j$ denotes their longest common prefix and $|\cdot|$ stands for the length. Let $z:=\sum_{k=1}^\infty j_km^{-k}$. We claim that \begin{equation}\label{eq:productstructure} g(K)\cap(0,1)^2 \subset K^{z} \times \R. \end{equation} Fix any $\delta>0$. Let $M$ be a large integer with $n^{-M}<\frac{\delta}{2}$. Pick $t$ so large that the following conditions hold: \begin{enumerate} \item $g(K)\cap (0,1)^2 \subset \mathcal{N}_{\delta/2}(h_t^{-1}f^t(K)\cap(0,1)^2)$, where $\mathcal{N}_{\delta/2}(\cdot)$ denotes the $\delta/2$-neighborhood. \item $(p-q)k_t\geq M$; \item there is some $\j_t$ such that $|\j_t\wedge\j|\geq M$. \end{enumerate} Since $|\j_t\wedge\j|\geq M$, it is not hard to see from the definition of $E_t$ and $\j_t$ that $E_t\subset \mathcal{N}_{n^{-M}}(K^z)$. Hence \begin{align*} g(K)\cap(0,1)^2 &\subset \mathcal{N}_{\delta/2}(h_t^{-1}f^t(K)\cap(0,1)^2) &&\text{(condition (1))} \\ &\subset \mathcal{N}_{\delta/2}(E_t\times\R) && \text{(by~\eqref{eq:htinversesupset})} \\ &\subset \mathcal{N}_{n^{-M}+\delta/2}(K^z\times\R) \subset \mathcal{N}_{\delta}(K^z\times\R). \end{align*} Since $\delta$ is arbitrary, we establish~\eqref{eq:productstructure}. Applying the same argument to the other three squares in $Q_t$, we find at most four points $z_{0,0}:=z,z_{0,1},z_{1,0},z_{1,1}\in\pi_2(K)$ such that \begin{equation}\label{eq:formofc} g(K) \subset \bigcup_{0\leq i,j\leq 1} (K^{z_{i,j}}+i)\times\R. \end{equation} Write $C:=\bigcup_{0\leq i,j\leq 1} (K^{z_{i,j}}+i)$. By~\eqref{eq:dimlbofky}, $\dimh C\leq\frac{\log N}{\log n}$. It remains to prove that if $f$ is oblique then $g$ can be picked oblique as well. Write $O$ to be the orthogonal part of $f$. If $\{O^t\}_{t=1}^\infty$ is a finite collection, then every element in it appears infinitely many times. Picking an oblique transformation $\widetilde{O}\in\{O^t\}_t$ and a subsequence $t_k$ so that $O^{t_k}=\widetilde{O}$ for all $k$, we can consider $f^{t_k}$ instead of $f^t$ at the beginning of the above arguments and get an oblique $g$ because $g$, as the limit of $h_t^{-1}f^t$, has $\widetilde{O}$ as its orthogonal part. If $\{O^t\}_{t=1}^\infty$ is an infinite collection, then since a $2\times 2$ orthogonal matrix is either a rotation or a reflection, $O$ must be an irrational rotation (i.e., the rotation angle is an irrational multiple of $\pi$) and hence every rotation matrix is an accumulation point of $\{O^t\}_{t=1}^\infty$. Then one just picks a subsequence $\{O^{t_k}\}_k$ so that $O^{t_k}$ converges to an oblique transformation, which, similarly as before, should be the orthogonal part of $g$. This completes the proof. \end{proof} \begin{remark}\label{rem:bairetofindint} In the above proof, if we apply Baire's theorem to~\eqref{eq:formofc}, some $(K^{z_{i,j}}+i)\times\R$ should contain an interior part of $g(K)$. In particular, there is an integer $k$ and a level-$k$ rectangle $R$ such that $g\varphi_R(K)\subset (K^{z_{i,j}}+i)\times\R$. As a consequence, \begin{equation}\label{eq:fvarphirkyinkz} g\varphi_R(K^y\times\{y\}) \subset (K^{z_{i,j}}+i)\times\R, \quad \forall y\in\pi_2(K). \end{equation} We also remark that the integer $k_t$ defined in~\eqref{eq:ktinlemma4.1} equals $\lfloor (-\frac{t\log\lambda}{\log m}-1)/p\rfloor$. So if $\frac{\log\lambda}{\log n}\notin\Q$, then $\frac{\log\lambda}{\log m}\notin\Q$ and hence $\{n^{qk_t}\lambda^t\}_t$ is dense in an open subinterval of $(0,1)$. Since the proof of Lemma~\ref{lem:dep1} works for all convergent subsequence of $\{h_t^{-1}f^t\}_{t=1}^\infty$, one can find such an accumulation map $\widetilde{g}$ so that $\widetilde{g}(K)\subset C_{\widetilde{g}}\times\R$, where $C_{\widetilde{g}}$ is a closed set with $\dimh C_{\widetilde{g}}\leq\frac{\log N}{\log n}$ (just as $C$ in Lemma~\ref{lem:dep1}), and the contraction ratio of $\widetilde{g}$ is an irrational power of $n$. \end{remark} Now we are able to prove the non-obliqueness theorem for the dependent case. The proof is separated into two parts, depending on whether $f$ is a contracting map or an isometry. \begin{theorem}\label{thm:dependentaffine} If $f$ is a similitude sending $K$ into itself, then $f$ is not oblique. \end{theorem} \begin{proof}[Proof of Theorem~\ref{thm:dependentaffine}: the contracting case] By Proposition~\ref{prop:noobliquelines}, it suffices to consider when $N\leq n-1$. Suppose on the contrary that $f$ is oblique. Pick $g, C$ as in Lemma~\ref{lem:dep1} with $g$ oblique and denote by $\rho$ the contraction ratio of $g$. If $N=1$, then it is not hard to see that every horizontal slice of $K$ is a finite set. From the form of $C$ (recall~\eqref{eq:formofc}), $C$ is a finite set. But by Proposition~\ref{prop:projofkisinf} and the obliqueness of $g$, $\pi_1(g(K))$ is an infinite set. This contradicts $\pi_1(g(K))\subset \pi_1(C\times\R)= C$. Now suppose $N\geq 2$. By Remark~\ref{rem:bairetofindint} (see~\eqref{eq:fvarphirkyinkz}), we can find $z\in\pi_2(K)$, $i\in\Z$, $k\geq 1$ and a level-$k$ rectangle $R$ such that \begin{equation}\label{eq:origvarphir} g\varphi_R(K^y\times\{y\}) \subset (K^z+i)\times\R, \quad \forall y\in\pi_2(K). \end{equation} Since $g$ is a similitude, there is a unit directional vector $v:=(v_1,v_2)$ such that for every $y\in\pi_2(K)$, $g\varphi_R$ maps the horizontal line $\R\times\{y\}$ to $L_y:=\{rv+c_y:r\in\R\}$ for some $c_y\in\R^2$. Together with~\eqref{eq:origvarphir}, we have \begin{equation}\label{eq:gky0slice} g\varphi_R(K^{y}\times\{y\}) \subset L_y \cap ((K^z+i)\times \R),\quad \forall y\in\pi_2(K). \end{equation} Since $g$ is oblique, $v_1,v_2$ are both nonzero. Without loss of generality, assume that $v_1,v_2>0$ (other cases can be similarly discussed). Note that for each $y\in\pi_2(K)$, $g\varphi_R(K^y\times\{y\})$ can be regarded as a similar copy of $K^y$ contracted by $n^{-k}\rho$, while $L_y\cap((K^z+i)\times\R)$ can be regarded as a translated copy of $\frac{|v|}{v_1}K^z$. In particular, one can obtain from~\eqref{eq:gky0slice} a similitude $h_y\in\S(\R)$ with contraction ratio $n^{-k}\rho\cdot\frac{v_1}{|v|}$ (which is independent of $y$) such that \begin{equation}\label{eq:hykysubsetc} h_y(K^y)\subset K^{z}, \quad \forall y\in\pi_2(K). \end{equation} Pick $j_*\in J$ so that $\#I_{j_*}=N$ and let $y_*:=\sum_{k=1}^\infty j_*m^{-k}$. Then $K^{y_*}=E(n,I_{j_*})$ (recall Section 2.2). By~\eqref{eq:hykysubsetc}, $h_{y_*}(K^{y_*})\subset K^{z}$. So $0<\h^\alpha(K^{z})<\infty$, where $\alpha:=\frac{\log N}{\log n}$ (recall the observation before Lemma~\ref{lem:bunchoffacts}). It might be suitable for us to point out an observation here (which will be of help later but not in this proof): since~\eqref{eq:hykysubsetc} also implies $h_z(K^z)\subset K^z$, it follows from Proposition~\ref{prop:allnthenrational} that \begin{equation}\label{eq:thenewlogformula} \frac{\log(n^{-k}\rho\cdot v_1/|v|)}{\log n}\in\Q. \end{equation} Next, since $h_z(K^z)\subset K^z$, by Corollary~\ref{cor:leftrightendpt}, $h_z$ maps a left isolated point $a$ and a right one $b$ both to one-sided isolated points of $K^z$. Since $h_z(K^z)\subset K^z$ is simply a one-dimensional reformulation of the inclusion $g\varphi_R(K^z\times\{z\})\subset L_z\cap ((K^z+i)\times\R)$, this in turn gives us two points $\widetilde{a},\widetilde{b}\in g\varphi_R(K^{z}\times\{z\})$ and some $\delta>0$ such that both of the tubes $(\pi_1(\widetilde{a})- \delta,\pi_1(\widetilde{a})) \times\R$ and $(\pi_1(\widetilde{b}),\pi_1(\widetilde{b})+\delta)\times\R$ do not intersect $(K^{z}+i)\times\R$. Moreover, there are two level-$t$ rectangles $R_t,R'_t\subset R$ such that $\widetilde{a}\in g\varphi_{R_t}(K)$ and $\widetilde{b}\in g\varphi_{R'_t}(K)$, respectively, and (again by Corollary~\ref{cor:leftrightendpt}) $\pi_2(R_t)=\pi_2(R'_t)$ for all $t\geq k_0$ for some $k_0$. See Figure~\ref{fig:leftrightendpt} for an illustration. \begin{figure}[htbp] \centering \begin{tikzpicture}[scale=1] \draw[thick] (0,0) to (0,4); \draw[thick] (4,0) to (4,4); ll[pattern=dots] (-1,0) rectangle (0,4); ll[pattern=dots] (4,0) rectangle (5,4); \draw[thick] (1,0) to (1,4); \draw[thick] (1.4,0) to (1.4,4); ll[pattern=dots] (1,0) rectangle (1.4,4); \draw[thick] (1.7,0) to (1.7,4); \draw[thick] (2,0) to (2,4); ll[pattern=dots] (1.7,0) rectangle (2,4); \draw[thick] (2.3,0) to (2.3,4); \draw[thick] (2.5,0) to (2.5,4); ll[pattern=dots] (2.3,0) rectangle (2.5,4); \draw[thick] (2.65,0) to (2.65,4); \draw[thick] (3,0) to (3,4); ll[pattern=dots] (2.65,0) rectangle (3,4); \node at (1,2)[circle,fill, inner sep=1.5pt]{}; \node at (3,2.4)[circle,fill, inner sep=1.5pt]{}; \draw[thick,dashed] (-1.5,1.5) to (5.5,2.9); \node at (0.8,1.7) {$\widetilde{a}$}; \node at (3.2,2.1) {$\widetilde{b}$}; \node at (5.25,2.55) {$L_z$}; \draw[thick,red] (0.95,1.75) to (1.13,1.79) to (1.01,2.4) to (0.82,2.36) to (0.95,1.75); \draw[thick,red] (2.95,2.15) to (3.13,2.19) to (3.01,2.8) to (2.82,2.76) to (2.95,2.15); \node at (0.55,2.6) [red] {$g(R_t)$}; \node at (3.5,2.9) [red] {$g(R'_t)$}; \end{tikzpicture} \caption{An illustration of $\widetilde{a},\widetilde{b},g(R_t)$ and $g(R'_t)$, where $(K^z+c)\times\R$ is supported in the shaded region. (color online)} \label{fig:leftrightendpt} \end{figure} Pick $x,x'\in K$ with $\pi_2(x)=\max\pi_2(K)$ and $\pi_2(x')=\min\pi_2(K)$. Since $\widetilde{a}\in g\varphi_{R_t}(K)$, $\widetilde{b}\in g\varphi_{R'_t}(K)$, $x$ can be expressed as \begin{equation}\label{eq:expressionofx} x = (g\varphi_{R_t})^{-1}(\widetilde{a})+ \Big(\begin{matrix} c_t \\ d_t \end{matrix}\Big) = (g\varphi_{R'_t})^{-1}(\widetilde{b})+\Big(\begin{matrix} \gamma_t \\ \eta_t \end{matrix}\Big) \end{equation} for some $-1\leq c_t,d_t,\gamma_t,\eta_t\leq 1$. Since $\pi_2(x)$ is the maximum, $d_t,\eta_t\geq 0$. Since $\widetilde{a}$, $\widetilde{b}$ are on the same slice and $\pi_2(R_t)=\pi_2(R'_t)$ for $t\geq k_0$, we have $d_t=\eta_t$ for $t\geq k_0$. Similarly, there are $-1\leq c'_t,\gamma'_t\leq 1$ and $-1\leq d'_t\leq 0$ such that for all $t\geq k_0$, \begin{equation}\label{eq:expressionofxprime} x' = (g\varphi_{R_t})^{-1}(\widetilde{a})+\Big(\begin{matrix} c'_t \\ d'_t \end{matrix}\Big) = (g\varphi_{R'_t})^{-1}(\widetilde{b})+\Big(\begin{matrix} \gamma'_t \\ \eta'_t \end{matrix}\Big). \end{equation} Fix a large $t\geq k_0$ (will be specified later). Note that \[ d_t-d'_t=\pi_2(x-x')=\max\pi_2(K)-\min\pi_2(K)=:\beta>0. \] So at least one of $d_t,-d'_t$ is no less than $\beta/2$. Writing $O=(\begin{smallmatrix} o_{1,1} & o_{1,2} \\ o_{2,1} & o_{2,2} \end{smallmatrix})$ to be the orthogonal part of $g$, we have by the obliqueness of $g$ that $o_{1,1},o_{1,2}\neq 0$. We distinguish four simple cases. \paragraph*{{\bf Case 1}: $d_t\geq\beta/2$ and $o_{1,2}>0$} In this case, we have by~\eqref{eq:expressionofx} that \[ g\varphi_{R'_t}(x) - \widetilde{b} = \Big( \begin{matrix} o_{1,1} & o_{1,2} \\ o_{2,1} & o_{2,2} \end{matrix}\Big)\cdot\Big(\begin{matrix} \rho n^{-t}\gamma_t \\ \rho m^{-t}d_t \end{matrix}\Big). \] Since $d_t\geq\beta/2$, $n>m$ and $|\gamma_t|\leq 1$, $0<\rho n^{-t}\gamma_to_{1,1}+ \rho m^{-t}d_to_{1,2}<\delta$ when $t$ is large and hence \[ \pi_1(g\varphi_{R'_t}(x)) = \pi_1(\widetilde{b})+\rho n^{-t}\gamma_to_{1,1}+\rho m^{-t}d_to_{1,2} \in (\pi_1(\widetilde{b}), \pi_1(\widetilde{b})+\delta). \] In other words, $g\varphi_{R'_t}(x)$ lies in the tube $(\pi_1(\widetilde{b}),\pi_1(\widetilde{b})+\delta)\times\R$. But we have seen that this tube does not intersect $(K^{z}+c)\times\R\supset g\varphi_R(K)\ni g\varphi_{R'_t}(x)$, which leads to a contradiction. \paragraph*{{\bf Case 2}: $d_t\geq\beta/2$ and $o_{1,2}<0$} In this case, we have by~\eqref{eq:expressionofx} that \[ g\varphi_{R_t}(x) - \widetilde{a} = \Big( \begin{matrix} o_{1,1} & o_{1,2} \\ o_{2,1} & o_{2,2} \end{matrix}\Big)\cdot\Big(\begin{matrix} \rho n^{-t}c_t \\ \rho m^{-t}d_t \end{matrix}\Big). \] Since $d_t\geq\beta/2$, $n>m$ and $|c_t|\leq 1$, we have $-\delta<\rho n^{-t}c_to_{1,1}+ \rho m^{-t}d_to_{1,2}<0$ when $t$ is large and hence \[ \pi_1(g\varphi_{R_t}(x)) = \pi_1(\widetilde{a})+ \rho n^{-t}c_to_{1,1}+ \rho m^{-t}d_to_{1,2} \in (\pi_1(\widetilde{a})-\delta, \pi_1(\widetilde{a})). \] In other words, $g\varphi_{R_t}(x)$ lies in the tube $(\pi_1(\widetilde{a})-\delta,\pi_1(\widetilde{a}))\times\R$. Again, we have seen that this tube does not intersect $(K^{z}+c)\times\R\supset g\varphi_R(K)\ni g\varphi_{R_t}(x)$, which leads to a contradiction. \paragraph*{{\bf Case 3}: $-d'_t\geq\beta/2$ and $o_{1,2}>0$} Applying the argument in Case 2 to~\eqref{eq:expressionofxprime}, we have \[ \pi_1(g\varphi_{R_t}(x')) = \pi_1(\widetilde{a})+ \rho n^{-t}c'_to_{1,1}+\rho m^{-t}d'_to_{1,2} \in (\pi_1(\widetilde{a})-\delta, \pi_1(\widetilde{a})) \] when $t$ is large, which leads to a contradiction. \paragraph*{{\bf Case 4}: $-d'_t\geq\beta/2$ and $o_{1,2}<0$} Applying the argument in Case 1 to~\eqref{eq:expressionofxprime}, we have \[ \pi_1(g\varphi_{R'_t}(x')) = \pi_1(\widetilde{b})+\rho n^{-t}\gamma'_to_{1,1}+ \rho m^{-t}d'_to_{1,2} \in (\pi_1(\widetilde{b}), \pi_1(\widetilde{b})+\delta) \] when $t$ is large, which leads to a contradiction. \end{proof} For the isometry case, we need a rescaling lemma of which the argument is analogous to the one of Lemma~\ref{lem:dep1}. \begin{lemma} Let $f\in\S(\R^2)$ be an oblique isometry. If $f(K)\subset K$, then we can find some $z\in\pi_2(K)$ and a nondegenerate scaled copy $E$ of $E(m,J)$ such that $E\subset K^z$. \end{lemma} \begin{proof} Pick any sequence $\{R_k\}_{k=1}^\infty$, where each $R_k$ is a level-$k$ rectangle and $R_{k+1}\subset R_k$ for all $k\geq 1$. Since $f$ is an isometry, we have \[ \max\{|\pi_1(f(R_k))|,|\pi_2(f(R_k))|\} < 2m^{-k}, \quad \forall k\geq 1. \] As a result, $f(R_{pk})$ is contained in at most $4$ squares of side length $m^{-pk}=n^{-qk}$. More precisely, there are $x\in n^{-qk}\mathbb{Z}, y\in m^{-pk}\mathbb{Z}$ such that \[ f\varphi_{R_{pk}}(K) \subset (x,x+2n^{-qk}) \times (y,y+2m^{-pk}) =: Q_k. \] Denote by $h_k$ the homothety sending $(0,1)^2$ to the square $(x,x+n^{-qk}) \times (y,y+m^{-pk})=:Q'_k$, the ``left bottom square'' of $Q_k$. Since $E(m,J)=\pi_2(K)$, it is easy to see that $d_H(\varphi_{R_{pk}}(K),\varphi_{R_{p_k}}(\{0\}\times E(m,J)))\leq n^{-pk}$, where $d_H$ denotes the Hausdorff distance (see~\cite{Fal14}). So \begin{equation}\label{eq:tendstotheleftproj} d_H(h_k^{-1}f\varphi_{R_{pk}}(K), h_k^{-1}f\varphi_{R_{pk}}(\{0\}\times E(m,J))) \leq n^{(q-p)k} \to 0, \quad k\to\infty. \end{equation} Moreover, writing $O$ to be the orthogonal part of $f$, each $h_k^{-1}f\varphi_{R_{pk}}(\{0\}\times E(m,J))$ is simply a translated copy of $O(\{0\}\times E(m,J))$, say $O(\{0\}\times E(m,J))+a_k$. So~\eqref{eq:tendstotheleftproj} becomes \[ d_H(h_k^{-1}f\varphi_{R_{pk}}(K), O(\{0\}\times E(m,J))+a_k) \leq n^{(q-p)k} \to 0, \quad k\to\infty. \] By the definition of $h_k$, the sequence $\{a_k\}_k$ lies in a bounded region in the plane. For notational simplicity, let us assume without loss of generality that $\{a_k\}_k$ is convergent, say $a_k\to a$. Then under the Hausdorff distance, \[ h_k^{-1}f\varphi_{R_{pk}}(K) \to O(\{0\}\times E(m,J))+a, \quad k\to\infty. \] Applying the rescaling argument in the proof of Lemma~\ref{lem:dep1} (now to $\{h_k^{-1}f\varphi_{R_{pk}}\}$ instead of $\{h_t^{-1}f^t\}_t$), we can find at most four points $z_{i,j}$, $0\leq i,j\leq 1$, such that \[ O(\{0\}\times E(m,J))+a \subset \bigcup_{0\leq i,j\leq 1} (K^{z_{i,j}}+i)\times\R. \] Since $O$ is oblique, projecting the above two sets to the $x$-axis, we see that $\bigcup_{0\leq i,j\leq 1} (K^{z_{i,j}}+i)$ contains a nondegenerate scaled copy of $E(m,J)$. Then by Baire's theorem, some $K^{z_{i,j}}+i$ must contain an interior part of that copy of $E(m,J)$. Since $E(m,J)$ is self-similar, this completes the proof. \end{proof} \begin{proof}[Proof of Theorem~\ref{thm:dependentaffine}: the isometry case] By Remark~\ref{rem:deindwork} and Proposition~\ref{prop:noobliquelines}, it suffices to consider when $N\leq n-1$ and $\#J\leq m-1$. Since $K$ is not contained in any line, $\#J>1$. Thus $E(m,J)$ is a deleted-digit set with Hausdorff dimension $\frac{\log\#J}{\log m}\in(0,1)$. Assume on the contrary that $f$ is oblique. Note that \[ f(K^y\times\{y\}) \subset f(K)\subset K \subset \R\times\pi_2(K) = \R\times E(m,J), \quad \forall y\in\pi_2(K). \] Therefore, $\pi_2(f(K^y\times\{y\})) \subset E(m,J)$ for all $y$. Since $f$ is oblique, the left hand side is a nondegenerate scaled copy of $K^y$. Picking $y_*$ as in the proof of Theorem~\ref{thm:dependentaffine} (before~\eqref{eq:thenewlogformula}), we have $\dimh E(m,J)\geq\dimh K^{y_*}=\frac{\log N}{\log n}=:\alpha$. Pick $z\in\pi_2(K)$ according to the above lemma. So there are also $\beta_2,c_2\in\R$ such that $E(m,J)\subset\beta_2K^z+c_2$. On the other hand, we have seen that $E(m,J)$ contains a scaled copy of $K^z$, say $\beta_1K^z+c_1$. So $\dimh E(m,J)\leq\dimh K^z\leq\alpha$ and hence $\dimh E(m,J)=\alpha$. Since $\alpha>0$, $N\geq 2$. Since $\h^\alpha(E(m,J))>0$, $\h^\alpha(K^z)\in(0,\infty)$ (again, the $<\infty$ part is an observation before Lemma~\ref{lem:bunchoffacts}). From $f(K^z\times\{z\})\subset\R\times E(m,J)\subset \R\times (\beta_2 K^z+c_2)$ and the above observation, we can apply Corollary~\ref{cor:leftrightendpt} and argue as in the second part of the proof of Theorem~\ref{thm:dependentaffine} (the one after~\eqref{eq:thenewlogformula}) and obtain a contradiction (the only difference is that in the previous proof, we embedded some oblique slice into a union of vertical lines, whereas now we embed some oblique slice into a union of horizontal lines).\end{proof} Combining with these results and Proposition~\ref{prop:allnthenrational}, we can quickly prove Theorem~\ref{thm:main2}. \begin{proof}[Proof of Theorem~\ref{thm:main2}] Let $K,n,m,f$ be as in Theorem~\ref{thm:main2}. If $\lambda=1$ then there is nothing to prove. If $\lambda<1$, then Theorem~\ref{thm:dependentaffine} tells us that $f$ is not oblique. So it is easy to find some integer $k\geq 1$ such that $f^k(x)=\lambda^kx+\eta$ for some $\eta\in\R^2$, i.e., the orthogonal part of $f^k$ is simply the identity matrix. Note that $f^k$ sends the $x$-axis to a horizontal line. If $\frac{\log\lambda}{\log n}\notin\Q$, we can apply Lemma~\ref{lem:dep1} to $f^k$ and get (recall the second paragraph in Remark~\ref{rem:bairetofindint}) some $g\in\S(\R^2)$ of which the contraction ratio $\rho$ is an irrational power of $n$. Also, since $f^k$ involves no rotations nor reflections, so does $g$. However, if $2\leq N\leq n-1$, applying the argument in the proof of the above theorem (where now $v=(1,0)$), we can still arrive at~\eqref{eq:thenewlogformula} and hence $\frac{\log\rho}{\log n}\in\Q$. This is a contradiction. If $N=1$, then every row contains at most one selected rectangle in the initial pattern of $K$. In particular, $\#\Lambda= \#J\leq m<n$. So there must be a vacant column. On the other hand, since $\#\Lambda\geq 2$ and $K$ is not contained in any vertical line, $\#I\geq 2$. Therefore, $\pi_1(K)=E(n,I)$ is a deleted-digit set with Hausdorff dimension $0<\frac{\log\#I}{\log n}\leq\frac{\log\#\Lambda}{\log n}<1$. Since $f^k(K)\subset K$, $\pi_1(f^k(K)) \subset \pi_1(K)=E(n,I)$. So \[ \lambda^kE(n,I)+\pi_1(\eta) = \lambda^k\pi_1(K)+\pi_1(\eta) = \pi_1(f^k(K)) \subset E(n,I). \] Then it follows from Lemma~\ref{lem:fw09} that $\frac{\log\lambda}{\log n}\in\Q$. Finally, assume that $N=n$. Let $J':=\{j\in J: \#I_{j}=n\}$. Note that \begin{equation}\label{eq:ycontainsintervals} \{y\in\pi_2(K): K^y \text{ contains an interval}\} \subset \bigcup_{t\geq 0}\bigcup_{z\in\Z} \frac{E(m,J')+z}{n^t}. \end{equation} The case when $\#J'=1$ has been settled in~\cite[Section 6.6]{AH19}. If $\#J'\geq 2$ (clearly, $\#J'\leq m-1$), then $0<\dimh E(m,J')<1$. Note that $K^y$ is an interval of length $1$ whenever $y\in E(m,J')$. Since $f^k(K)\subset K$, it follows from~\eqref{eq:ycontainsintervals} that \[ \lambda^k E(m,J')+\pi_2(\eta) \subset \bigcup_{t\geq 0}\bigcup_{z\in\Z} \frac{E(m,J')+z}{n^t}. \] By Baire's theorem, there are some $t,z$ such that $\frac{E(m,J')+z}{n^t}$ contains an interior part of $\lambda^k E(m,J')+\pi_2(\eta)$. By Lemma~\ref{lem:fw09}, $\frac{\log\lambda}{\log n}\in\Q$, which completes the proof. \end{proof} \section{The dependent self-similar case} Finally, we consider the case when $m=n$, that is, $K=K(n,\Lambda)$ is a generalized Sierpi\'nski carpet. However, there does exist such a self-similar carpet allowing oblique embeddings. \begin{example}\label{exa:nonobsiercat} Consider the carpet \[ K = \Big\{ \sum_{k=1}^\infty \frac{(x_k,y_k)}{4^k}: (x_k,y_k)\in\{(0,1),(1,3),(2,0),(3,2)\} \Big\}. \] See Figure~\ref{fig:specialcarpet}. Let $O=\big(\begin{smallmatrix} \frac{3}{5} & -\frac{4}{5}\\ -\frac{4}{5} & -\frac{3}{5} \end{smallmatrix}\big)$, which is the matrix for the reflection with respect to some line of slope $-1/2$. It is easy to check that \begin{align*} OK &+ ( \tfrac{9}{5}, \tfrac{18}{5} ) \\ &= \Big\{ \sum_{k=1}^\infty \frac{(\frac{3}{5}x_k-\frac{4}{5}y_k+\frac{9}{5},-\frac{4}{5}x_k-\frac{3}{5}y_k+\frac{18}{5})}{4^k}: (x_k,y_k)\in\{(0,1),(1,3),(2,0),(3,2)\} \Big\} \\ &= \Big\{ \sum_{k=1}^\infty \frac{(x_k,y_k)}{4^k}: (x_k,y_k)\in\{(0,1),(1,3),(2,0),(3,2)\} \Big\} = K. \end{align*} Therefore, $K$ allows an oblique self-embedding $f(x)=Ox+(\frac{9}{5},\frac{18}{5})$. \begin{figure}[htbp] \centering \includegraphics[width=3.8cm]{specialcarpet.eps} \caption{A carpet allowing oblique self-embeddings} \label{fig:specialcarpet} \end{figure} \end{example} In the rest of this section, we only consider rotational self-embeddings of $K$ and hope to obtain some information on the rotation angle. Recall the notation~\eqref{eq:varphiij}. The following result is well known (see e.g.~\cite{SW99}). \begin{lemma}\label{lem:convandfixpt} If $K$ is not contained in any line, then the convex hull of $K$ is a polygon. Moreover, every vertex of this polygon is the fixed point of $\varphi_{i,j}$ for some $(i,j)\in\Lambda$. \end{lemma} From now on, let us fix a generalized Sierpi\'nski carpet $K$ satisfying the strong separation condition and not supported in any line. Denote by $P$ the convex hull of $K$. By the above lemma, $P$ is a polygon with finitely many vertices, say $v_1,\ldots,v_p$. For $1\leq t\leq p$, we write $\alpha_t$ to be the interior angle of $P$ at the vertex $v_t$, and write $\ell_t$ to be the edge of $P$ joining $v_t$ and $v_{t+1}$. We claim that every $v_t$ is not isolated in $K$ even along its adjacent edges. \begin{corollary}\label{cor:limitptonell} For $1\leq t\leq p$, there exists $\{x_k\}_{k=1}^\infty\subset\ell_t\cap K$ such that $x_k\to v_t$ as $k\to\infty$. \end{corollary} \begin{proof} Fix any $1\leq t\leq p$. By Lemma~\ref{lem:convandfixpt}, there is some $(i,j)\in\Lambda$ such that $v_t$ is the fixed point of $\varphi_{i,j}$. Then \[ v_t -\varphi_{i,j}^k(v_{t+1}) = \varphi_{i,j}^k(v_{t})-\varphi_{i,j}^k(v_{t+1}) = n^{-k}(v_t-v_{t+1}), \quad \forall k\geq 1. \] In particular, $\varphi_{i,j}^k(v_{t+1})$ must lie on $\ell_t$ and tends to $v_t$ as $k\to\infty$. \end{proof} \begin{corollary}\label{cor:rationaltangent} For each $1\leq t\leq p$, $\tan\alpha_t\in\Q$. \end{corollary} \begin{proof} For $1\leq t\leq p$, we again find by Lemma~\ref{lem:convandfixpt} some $(i_t,j_t)\in\Lambda$ such that $v_t$ is the fixed point of $\varphi_{i_t,j_t}$. Therefore, $v_t=(\frac{i_t}{n-1},\frac{j_t}{n-1})$. This implies that the angle made by every edge $\ell_t$ and the $x$-axis has a rational tangent. It follows directly that $\tan\alpha_t\in\Q$ for all $t$. \end{proof} A ``local openness'' result as follows will be of great help. \begin{lemma}[{\cite[Theorems~4.5 and 4.9]{EKM10}}]\label{lem:ekmtworesults} Let $f\in\S(\R^2)$. If $f(K)\subset K$, then there is an open set $U\subset\R^2$ such that $U\cap f(K)=U\cap K\neq\varnothing$. Moreover, $\{O^t\}_{t=1}^\infty$ is a finite collection, where $O$ is the orthogonal part of $f$. \end{lemma} \begin{proposition}\label{prop:rationalangles} If $f\in\S(\R^2)$ sends $K$ into itself, then for every $1\leq t\leq p$, we can find $t'$ and two squares $R,R'$ (of level $k$ and $k'$, respectively) such that the following properties hold: \begin{enumerate} \item $f\varphi_{R'}(v_{t'})=\varphi_R(v_t)$, \item either $f\varphi_{R'}(\ell_{t'})\subset\varphi_R(\ell_t)$ or $f\varphi_{R'}(\ell_{t'+1})\subset\varphi_{R}(\ell_{t})$. \end{enumerate} \end{proposition} \begin{proof} Fix $1\leq t\leq p$. Without loss of generality, assume that $v_t$ lies in the ``left bottom area'' of $[0,1]^2$, that is, $v_t\in[0,1/2]^2$ (other cases can be similarly discussed). Let us pick an integer $k$ and a level-$k$ square $R=[\frac{i}{n^k},\frac{i+1}{n^k}]\times[\frac{j}{n^k},\frac{j+1}{n^k}]$ such that $\varphi_R(K)\subset f(K)$ and \[ K \cap \big( (\tfrac{i-1}{n^k},\tfrac{i}{n^k})\times(\tfrac{j-1}{n^k},\tfrac{j+1}{n^k}) \big) = \varnothing \quad\text{and}\quad K \cap \big( (\tfrac{i}{n^k},\tfrac{i+1}{n^k})\times(\tfrac{j-1}{n^k},\tfrac{j}{n^k}) \big) = \varnothing. \] The existence of $R$ will be proved at the end of this proof. As a consequence, there is a small $r>0$ such that \begin{equation}\label{eq:localareaofphirvt} K \cap B(\varphi_R(v_t),r) = \varphi_R(K) \cap B(\varphi_R(v_t),r) = K \cap \varphi_R(P) \cap B(\varphi_R(v_t),r), \end{equation} where $B(x,r)$ denotes the ball centered at $x$ and of radius $r$. Since $\varphi_R(v_t)\in\varphi_R(K)\subset f(K)$, there is a level-$k'$ square $R'$ (for some large $k'$) such that $\varphi_R(v_t)\in f\varphi_{R'}(K)$. We claim that $\varphi_R(v_t)$ is a vertex of the polygon $f\varphi_{R'}(P)$ if $k'$ is picked large. In particular, there exists $t'$ such that $\varphi_R(v_t)=f\varphi_{R'}(v_{t'})$, which establishes (1). To see the claim, note that since $P$ is the convex hull of $K$, $\varphi_R(v_t)\in f\varphi_{R'}(K)\subset f\varphi_{R'}(P)$. So if $k'$ is large enough, then \begin{equation}\label{eq:fvarphirinball} f\varphi_{R'}(P) \subset B(\varphi_{R}(v_t),r). \end{equation} Therefore, \begin{align*} f\varphi_{R'}(K) &\subset f(K) \cap f\varphi_{R'}(P) && \text{(since $K\subset P$)} \\ &\subset K\cap B(\varphi_R(v_t),r) && \text{(by $f(K)\subset K$ and~\eqref{eq:fvarphirinball})} \\ &\subset \varphi_R(P) &&\text{(by~\eqref{eq:localareaofphirvt})}. \end{align*} So $f\varphi_{R'}(P)\subset\varphi_R(P)$. Since $f\varphi_{R'}(P)$, $\varphi_R(P)$ are both polygons and $\varphi_R(v_t)\in f\varphi_{R'}(P)$, $\varphi_R(v_t)$ must be a vertex of $f\varphi_{R'}(P)$. If (2) is false, we see by (1) that $\varphi_{R}(\ell_t)$ meets $f\varphi_{R'}(P)$ only at the vertex $\varphi_R(v_t)$. By Corollary~\ref{cor:limitptonell}, there is a sequence $\{x_p\}_{p=1}^\infty\subset\ell_t\cap K$ such that $x_p\to v_t$ as $p\to\infty$. So $\varphi_R(x_p)\to\varphi_R(v_t) \in f\varphi_{R'}(K)$ and $\varphi_R(x_p)\in\varphi_R(\ell_t)$. Since $\varphi_{R}(\ell_t)$ meets $f\varphi_{R'}(P)$ only at the vertex $\varphi_R(v_t)$, $\varphi_R(x_p)\notin f\varphi_{R'}(K)$. But recalling that $\varphi_R(K)\subset f(K)$, we have $\varphi_R(x_p)\in f(K)$ for all $p$. This implies that \[ \dist\big( f\varphi_{R'}(K), f(K)\setminus f\varphi_{R'}(K) \big) \leq \inf_p\dist(f\varphi_{R'}(K),\{\varphi_R(x_p)\})= 0, \] which contradicts the strong separation condition. It remains to prove the existence of $R$. By Lemma~\ref{lem:ekmtworesults}, there is some $k_0$ and a level-$k_0$ square $Q$ such that $\varphi_Q(K)\subset f(K)$. Since $K$ is not contained in any line, we have $\#I\geq 2$ and $\#J\geq 2$ (here we abuse slightly these notations from Section 2.1). Iterating the initial pattern if necessary, we may assume that there are $(i_1,j_1),(i_2,j_2),(i_3,j_3)\in\Lambda$ such that \[ (i_1,j_1-1), (i_2-1,j_2),(i_3-1,j_3-1) \in \{0,1,\ldots,n-1\}^2\setminus\Lambda. \] Also, write $\underline{i}:=\min I$ and $\underline{j}:= \min J$. If there is some $1\leq i\leq n-1$ such that $(i,\underline{j})\in\Lambda$ but $(i-1,\underline{j})\notin\Lambda$, then it suffices to take $R=\varphi_Q\varphi_{i_1,j_1}\varphi_{i,\underline{j}}([0,1]^2)$. If such a digit does not exist, then $(0,\underline{j})\in\Lambda$. If $I_{\underline{j}}\supsetneq\{0\}$, then $2\leq\#I_{\underline{j}}\leq n-1$, where the upper bound holds because $K$ does not contain horizontal segments. So there are at least $4$ level-$2$ squares in the bottom row and it is not hard to see that at least one of them, say $Q'$, satisfies that the grid square of side $n^{-2}$ left adjacent to $Q'$ is contained in $[0,1]^2$ but unselected. Then it suffices to take $R=\varphi_Q\varphi_{i_1,j_1}(Q')$. So we may assume that $I_{\underline{j}}=\{0\}$. Similarly (looking at $(i_2,j_2)$ instead), it suffices to consider when $J_{\underline{i}}=\{0\}$. Since $(0,\underline{j})\in\Lambda$, $\underline{i}=0$. So $(0,0)\in\Lambda$ but $\{(0,j),(j,0):j\geq 1\}\cap\Lambda=\varnothing$. Then it is easy to see that taking $R=\varphi_Q\varphi_{i_3,j_3}\varphi_{0,0}([0,1]^2)$ would work. \end{proof} \begin{lemma}[Niven's theorem]\label{lem:niven} If $\theta\in\pi\Q$ and $\tan\theta\in\Q$, then $\tan\theta\in\{0,\pm 1\}$. \end{lemma} \begin{proof} For a proof, see~\cite{PV21}. \end{proof} \begin{corollary} Let $f(x)=\lambda R_\theta x+a$ be as in Theorem~\ref{thm:main3}. If $\frac{\theta}{\pi}\in\Q$ and $f$ is an oblique self-embedding similitude of $K$, then $|\tan\theta|=1$. \end{corollary} \begin{proof} By Lemma~\ref{lem:niven} and the obliqueness of $f$, it suffices to show that $\tan\theta\in\Q$. Fix any $1\leq t\leq p$ and pick $R,R',k,k',t'$ accordingly as in Proposition~\ref{prop:rationalangles}. Without loss of generality, assume that $f\varphi_{R'}(\ell_{t'})\subset\varphi_{R}(\ell_t)$, which is the first case in Proposition~\ref{prop:rationalangles}(2). \begin{figure}[htbp] \centering \begin{tikzpicture}[rotate=30, scale=0.5,>=stealth] \draw[thick] (0,0) to (-2,0); \draw[thick,red] (-2,0) to (-3,-2); \draw[thick] (-3,-2) to (-2.5,-2.8); \draw[thick] (-2.5,-2.8) to (-1.5,-4.8); \draw[thick,red] (-1.5,-4.8) to (0,-4.8); \node at (-3,-1) [red] {$\ell_{t'}$}; \node at (-0.5,-5.3) [red] {$\ell_t$}; \node at (-2,0)[circle,fill, inner sep=1.5pt]{}; \node at (-2.5,0.3) {$v_{t'}$}; \node at (-1.5,-4.8)[circle,fill, inner sep=1.5pt]{}; \node at (-1.8,-5.3) {$v_t$}; \draw[thick,dashed] (-3,-2) to (-3.8,-3.6); \draw[thick,dashed] (-1.5,-4.8) to (-1,-5.8); \coordinate (A) at (-2,0); \coordinate (B) at (-3,-2); \coordinate (C) at (-2.5,-2.8); \coordinate (D) at (-1.5,-4.8); \coordinate (E) at (0,0); \coordinate (F) at (0,-4.8); \draw pic["$\alpha_{t'}$", draw=black, -, angle eccentricity=2.3,angle radius=0.2cm]{angle = B--A--E}; \draw pic["$\alpha_{t'+1}$", draw=black, -, angle eccentricity=2.6,angle radius=0.2cm]{angle = C--B--A}; \draw pic["$\alpha_{t}$", draw=black, -, angle eccentricity=1.8,angle radius=0.2cm]{angle = F--D--C}; \end{tikzpicture} \caption{The rotation effect of $f$ (color online)} \label{fig:rotationoff} \end{figure} Since $\varphi_R(\ell_{t})$ is parallel to $\varphi_{R'}(\ell_{t})$ and $\varphi_{R'},\varphi_R$ involve no rotations nor reflections, $f$ sends a line parallel to $\ell_{t'}$ to another line parallel to $\ell_t$. Without loss of generality, assume that $t'<t$ and $v_1,\ldots,v_p$ are in counterclockwise order. Then a simple geometric observation (see Figure~\ref{fig:rotationoff}) tells us that $R_\theta$ can be realized as follows: first rotate the line containing $\ell_{t'}$ to the one containing $\ell_{t'+1}$, and then to the one containing $\ell_{t'+2}$ and so on until to $\ell_t$. Thus \[ \theta = \sum_{p=1}^{t-t'} (\pi-\alpha_{t'+p}) \pmod{2\pi}. \] By Corollary~\ref{cor:rationaltangent}, $\tan\theta\in\Q$ as desired. \end{proof} With all these observations in hand, Theorem~\ref{thm:main3} can be easily proved. \begin{proof}[Proof of Theorem~\ref{thm:main3}] Again, we may assume that $K$ is not supported in any line. By Lemma~\ref{lem:ekmtworesults}, $\frac{\theta}{\pi}\in\Q$. Then the statement follows directly from the above corollary. \end{proof} \bigskip \noindent{\bf Acknowledgements.} I thank Professor Huo-Jun Ruan for reading the manuscript carefully and pointing out several mistakes and a number of typos in an early version of it. I also thank Dingkun Hu for a nice observation regarding the setting of Condition A in the proof of Theorem~\ref{thm:notoblique} (under Case 3). \small \bibliographystyle{amsplain} \begin{thebibliography}{100} \bibitem{Alg201} A. Algom, Affine embeddings of Cantor sets in the plane, J. Anal. Math. {\bf 140} (2020), 695--757. \bibitem{Alg20} A. Algom, Slicing theorems and rigidity phenomena for self-affine carpets, Proc. Lond. Math. Soc. {\bf 121} (2020), 312--353. \bibitem{AH19} A. Algom and M. Hochman, Self embeddings of Bedford-McMullen carpets, Ergod. Th. \& Dynam. Sys. {\bf 39} (2019), 577--603. \bibitem{AW23} A. Algom and M. Wu, Improved versions of some Furstenberg type slicing theorems for self-affine carpets, Int. Math. Res. Not. {\bf 3} (2023), 2304--2343. \bibitem{BR14} B. B\'ar\'any and M. Rams, Dimension of slices of Sierpi\'nski-like carpets, J. Fractal Geom. {\bf 1} (2014), 273--294. \bibitem{Bed84} T. Bedford, Crinkly curves, Markov partitions and box dimensions in self-similar sets, Ph.D. thesis, University of Warwick, 1984. \bibitem{EKM10} M. Elekes, T. Keleti, and A. M\'ath\'e, Self-similar and self-affine sets: measure of the intersection of two copies, Ergod. Th. \& Dynam. Sys. {\bf 30} (2010), 399--440. \bibitem{Fal14} K. J. Falconer, \textit{Fractal Geometry: Mathematical Foundations and Applications}, 3rd edn. John Wiley \& Sons, Chichester, 2014. \bibitem{FHR14} D.-J. Feng, W. Huang, and H. Rao, Affine embeddings and intersections of Cantor sets, J. Math. Pures Appl. {\bf 102} (2014), 1062--1079. \bibitem{FW09} D.-J. Feng, Y. Wang, On the structures of generating iterated function systems of Cantor sets, Adv. Math. {\bf 222} (2009), 1964--1981. \bibitem{FJS10} A. Ferguson, T. Jordan, P. Shmerkin, The Hausdorff dimension of the projections of self-affine carpets, Fund. Math. {\bf 209} (2010), 193--213. \bibitem{Fra21} J. Fraser, Fractal geometry of Bedford-McMullen carpets, In M. Pollicot and S. Vaienti, editors, Proceedings of the Fall 2019 Jean-Morlet Chair programme, Springer Lecture Notes Series, 2021. \bibitem{Mar54} J. M. Marstrand, Some fundamental geometrical properties of plane sets of fractional dimensions, Proc. Lond. Math. Soc. {\bf 4} (1954), 257--302. \bibitem{Mcm84} C. McMullen, The Hausdorff dimension of general Sierpi\'nski carpets, Nagoya Math. J. {\bf 96} (1984), 1--9. \bibitem{PV21} B. Paolillo and G. Vincenzi, On the rational values of trigonometric functions of angles that are rational in degrees, Mathematics Magazine {\bf 94} (2021), 132--134. \bibitem{RXtodedone} H. Rao and J.-C. Xiao, manuscript in preparation. \bibitem{SW99} R. Strichartz and Y. Wang, Geometry of self-affine tiles I, Indiana Univ. Math. J. {\bf 48} (1999), 1--23. \bibitem{Xiao24} J.-C. Xiao, On a self-embedding problem for self-similar sets, Ergod. Th. \& Dynam. Sys. {\bf 44} (2024), 3002--3011. \end{thebibliography} \end{document}
2412.02118v2
http://arxiv.org/abs/2412.02118v2
Algebraic properties of Indigenous semirings
\documentclass[psamsfonts]{amsart} \usepackage{amssymb,amsfonts} \usepackage[all,arc]{xy} \usepackage{enumerate} \usepackage{mathrsfs} \usepackage{graphicx} \usepackage{orcidlink} \newtheorem{theorem}{Theorem}[section] \newtheorem{corollary}[theorem]{Corollary} \newtheorem{proposition}[theorem]{Proposition} \newtheorem{lemma}[theorem]{Lemma} \theoremstyle{definition} \newtheorem{definition}[theorem]{Definition} \newtheorem{definitions}[theorem]{Definitions} \newtheorem{construction}[theorem]{Construction} \newtheorem{example}[theorem]{Example} \newtheorem{examples}[theorem]{Examples} \newtheorem{notation}[theorem]{Notation} \newtheorem{notations}[theorem]{Notations} \newtheorem{addendum}[theorem]{Addendum} \newtheorem{exercise}[theorem]{Exercise} \newtheorem{conjecture}[theorem]{Conjecture} \newtheorem{question}[theorem]{Question} \newtheorem{remark}[theorem]{Remark} \newtheorem{remarks}[theorem]{Remarks} \newtheorem{warning}[theorem]{Warning} \newtheorem{scholium}[theorem]{Scholium} \DeclareMathOperator{\Jac}{Jac} \DeclareMathOperator{\Nil}{Nil} \DeclareMathOperator{\rad}{rad} \DeclareMathOperator{\FId}{FId} \DeclareMathOperator{\End}{End} \DeclareMathOperator{\Id}{Id} \DeclareMathOperator{\Max}{Max} \DeclareMathOperator{\Spec}{Spec} \DeclareMathOperator{\Sub}{Sub} \DeclareMathOperator{\Ann}{Ann} \DeclareMathOperator{\zd}{zd} \DeclareMathOperator{\diam}{diam} \DeclareMathOperator{\supp}{supp} \DeclareMathOperator{\IG}{IG} \setcounter{section}{-1} \makeatletter \makeatother \numberwithin{equation}{section} \begin{document} \title{Algebraic properties of Indigenous semirings} \author[H. Behzadipour]{Hussein Behzadipour\orcidlink{0000-0001-7037-4110}} \address{Hussein Behzadipour\\ Department of Mathematics \\ Sharif University of Technology\\ Tehran\\ Iran} \email{[email protected]} \author[H. Koppelaar]{Henk Koppelaar\orcidlink{0000-0001-7487-6564}} \address{Henk Koppelaar\\ Faculty of Electrical Engineering, Mathematics and Computer Science\\ Delft University of Technology\\ Delft\\ The Netherlands} \email{[email protected]} \author[P. Nasehpour]{Peyman Nasehpour\orcidlink{0000-0001-6625-364X}} \address{Peyman Nasehpour\\ Education Department\\ The New York Academy of Sciences \\ New York, NY, USA} \email{[email protected]} \subjclass[2010]{16Y60, 13A15, 01A07.} \keywords{Indigenous semirings, Information algebras, Graph invariants} \begin{abstract} In this paper, we introduce Indigenous semirings and show that they are examples of information algebras. We also attribute a graph to them and discuss their diameters, girths, and clique numbers. On the other hand, we prove that the Zariski topology of any Indigenous semiring is the Sierpi\'{n}ski space. Next, we investigate their algebraic properties (including ideal theory). In the last section, we characterize units and idempotent elements of formal power series over Indigenous semirings. \end{abstract} \maketitle \section{Introduction} Inspired by the Indigenous number systems (cf. \cite{BenderBeller2021}, \cite{Everett2005}, \cite{Gordon2004}, \S1 in \cite{Ifrah2000}, and \cite{Vandendriessche2022}) and the Indigenous presemiring $\mathcal{M} = \{1,2,3,m\}$ proposed by Sibley in Example 5 on p. 419 of his educational book on abstract algebra \cite{Sibley2021}, we give a general definition for Indigenous semirings and investigate their algebraic properties. It turns out that the Indigenous semirings belong to a particular family of semirings called information algebras which have applications in various fields of science and engineering and have attracted the interests of some authors since 1972 (see Remark \ref{Informationalgebrasrem}). From an algebraic perspective, one could argue that a number is not its physical appearance or digit representation. A number is an abstract mathematical object, whereas its appearance is a sequence of symbols on a paper (or a sequence of bits in computer memory, or a sequence of sounds if read aloud). We never see a number itself, but always its representation. So, we become accustomed to identifying a number by its representation. In Ethnomathematics (one of the subjects of this paper) it is similar. This confusing identification exposes one of the difficulties of the field. Other difficulties are exemplified in Examples \ref{historicalexamplesIndigenous} of this paper. A plea to alleviate the study problems of indigenous number systems is to develop cultural tools for numerical cognition \cite{BenderBeller2018}. For instance, the Yapese indigenous money counting is based on stone disks called ``rai stones''. A typical ``rai stone'' is carved out of crystalline limestone and shaped like a disk with a hole in the center. The smallest may be 3.5 centimeters in diameter while the largest extant stone is 3.6 meters in diameter and 50 centimeters thick, and weighs 4,000 kilograms \cite{Gillilland1975}. This paper develops the underlying algebra (presemiring) of indigenous number systems to an unprecedented degree. Since the language of semiring-like algebraic structures is not standardized yet, we first introduce some terminology. Recall that a bimagma $(R,+,\cdot)$ is a ringoid \cite[p. 206]{Rosenfeld1968} if the binary operation ``$\cdot$'' (multiplication) distributes on the binary operation ``$+$'' (addition) from both sides. A ringoid $(R,+,\cdot)$ is a presemiring if $(R,+)$ is a commutative semigroup and $(R,\cdot)$ a semigroup \cite[Definition 4.2.1]{GondranMinoux2008}. A presemiring is commutative if $(R,\cdot)$ is a commutative semigroup. In this paper, all presemirings are supposed to be commutative. A presemiring $S$ is a semiring if it has a neutral element $0$ for its addition which is also an absorbing element for its multiplication and it has also a neutral element $1 \neq 0$ for its multiplication \cite[p. 1]{Golan1999(b)}. A semiring $S$ is entire if it is zero-divisor free, i.e., $ab = 0$ implies $a = 0$ or $b = 0$, for all $a$ and $b$ in $S$. A semiring $S$ is zerosumfree if $a+b = 0$ implies $a = b = 0$, for all $a$ and $b$ in $S$. A semiring $S$ is an information algebra if it is both zero-divisor free and zerosumfree \cite[p. 4]{Golan1999(b)}. A nonempty set $I$ of a semiring $S$ is an ideal of $S$ if $(I,+)$ is a submonoid of $(S,+)$ and $SI \subseteq I$ \cite{Bourne1951}. We collect all ideals of a semiring $S$ in $\Id(S)$. An ideal $M$ of a semiring $S$ is maximal if there is no ideal properly between $M$ and $S$. A semiring $S$ is local if it has a unique maximal ideal. An ideal $P$ of a semiring $S$ is prime if $P \neq S$ and $IJ \subseteq S$ implies $I \subseteq S$ or $J \subseteq S$, for all ideals $I$ and $J$ of $S$. All prime ideals of a semiring $S$ are collected in $\Spec(S)$. In a (commutative) semiring $S$, an ideal $P \neq S$ is prime if and only if $ab \in P$ implies either $a \in P$ or $b \in P$, for all elements $a$ and $b$ in $S$ \cite[Corollary 7.6]{Golan1999(b)}. The first section of the paper is devoted to some results for entire semirings. Recall that for each ideal $I$ of a semiring $S$ \[V(I) = \{P \in \Spec(S) : P \supseteq I\}.\] It is, then, easy to verify that $\mathcal{C} = \{V(I) : I \in \Id(S)\}$ is the family of closed sets for a topology on $X = \Spec(S)$, called the Zariski topology \cite[p. 89]{Golan1999(b)}. A topological space with exactly two points and three closed subsets is called the Sierpi\'{n}ski space (see \cite[p. 17]{ArensDugundji1951} and \cite[Exercise 1.7]{Rotman1988}). In Theorem \ref{Zariskitopologyentiresemiringwithtwoprimes}, we show that the Zariski topology of an entire semiring with exactly one nonzero prime ideal is the Sierpi\'{n}ski space. Then, we proceed to show that the localization of an information algebra is an information algebra (see Proposition \ref{Localizationofinformationalgebras}). Let us recall that if $S$ is a semiring, then $\Id(S)$ equipped with addition and multiplication of ideals is a semiring \cite[Proposition 6.29]{Golan1999(b)}. In Theorem \ref{Semiringidealsinformationalgebra}, we prove that $\Id(S)$ is an information algebra if and only if $S$ is an entire semiring. Inspired by Example 5 on p. 419 in \cite{Sibley2021}, in \S\ref{sec:indigenousgraphs} of our paper, we define Indigenous addition and multiplication on $I_k = \mathbb{N}_k \cup \{m\}$, where $\mathbb{N}_k$ is the set of positive integer numbers less than $k+1$ and $m$ is just a symbol standing for ``many'' (see Definition \ref{Indigenouspresemiringdef}), and next in Proposition \ref{Indigenoussemiringpro}, we show that the bimagma $(I_k, \oplus, \odot)$ is a unital presemiring. For some historical examples of Indigenous presemirings, check Examples \ref{historicalexamplesIndigenous}. In Definition \ref{Indigenousgraphs}, we attribute a graph $\IG_k$ to any Indigenous presemiring $I_k$ where its vertices' set is $I_k$ and $\{a,b\}$ is an edge of $\IG_k$ if $a \neq b$ are elements of $I_k$ with $ab = m$. In the rest of \S\ref{sec:indigenousgraphs}, we discuss some graph invariants of Indigenous graphs. For example in Theorem \ref{diamIG}, we prove that the Indigenous graph $\IG_k$ is a connected graph with $\diam(\IG_1) = 1$ and $\diam(\IG_k) = 2$ if $k > 1$. In Theorem \ref{girthIG}, we show that the girth of the Indigenous graph $\IG_k$ is $3$ if $k \geq 3$, and is infinity, otherwise. Finally in Theorem \ref{cliqueIG}, we prove that the clique number of the Indigenous graph $\IG_k$ is at least $\left\lfloor \frac{k}{2} \right\rfloor + 1$, for any positive integer $k$. By annexing $0$ to the Indigenous presemiring $I_k$, we define the Indigenous semiring $S_k$ and in Theorem \ref{GeneralpropertiesofIndigenoussemirings}, we discuss algebraic properties of the Indigenous semirings. In this theorem with 12 items, we show that, for instance, any Indigenous semiring $S_k$ is a local information algebra and $\{0\}$ and $S_k \setminus \{1\}$ are the only prime ideals. A corollary to this statement is that the Zariski topology of any Indigenous semiring $S_k$ is the Sierpi\'{n}ski space. Next, we verify that $S_k$ is not a semidomain (recall that a semiring $S$ is a semidomain \cite{Nasehpour2019} if $S\setminus\{0\}$ is a multiplicatively cancellative monoid). In the same result, we show that any Indigenous semiring is austere and discuss the algebra of the ideals of the Indigenous semirings. In fact, we show that $(\Id(S_k)\setminus\{\mathbf{0}\}, \cdot)$ is a nilpotent monoid with the absorbing element $\mathfrak{s}_k = \{0,m\}$, where by $\mathbf{0}$, we mean the zero ideal $\{0\}$ of $S_k$. In \S\ref{sec:distinguishedelements}, we characterize units and idempotent elements of polynomials and formal power series over the Indigenous semirings (see Proposition \ref{UnitsIndigenoussemirings}, Proposition \ref{IdempotentsIndigenoussemirings1}, and Theorem \ref{IdempotentsIndigenoussemirings2}). Golan's book \cite{Golan1999(b)} is a general reference for semiring theory, and our terminology closely follows it. \section{Some results in entire semirings}\label{sec:entiresemirings} \begin{theorem}\label{Zariskitopologyentiresemiringwithtwoprimes} The Zariski topology of an entire semiring with exactly one nonzero prime ideal is the Sierpi\'{n}ski space. \end{theorem} \begin{proof} Let $S$ be an entire semiring with exactly one nonzero prime ideal. This means that $\Spec(S) = \{\{0\},\mathfrak{m}\}$, with $\mathfrak{m} \neq \{0\}$. It follows that the only closed subsets of the Zariski topology of $S$ are \begin{itemize} \item $V(S) = \emptyset$, \item $V(I) = V(\mathfrak{m}) = \{\mathfrak{m}\}$, for any nonzero proper ideal $I$ of $S$. \item $V(\{0\}) = \{\{0\}, \mathfrak{m}\}$ \end{itemize} Therefore, the Zariski topology of $S$ has two points with three closed subsets which is the Sierpi\'{n}ski space. This completes the proof. \end{proof} \begin{remark}\label{Informationalgebrasrem} Due to the applications of information algebras, they configure an important family of semirings. Perhaps the oldest example for information algebras is the semiring of non-negative integer numbers $\mathbb{N}_0$ equipped with usual addition and multiplication of numbers. The term ``information algebra" was introduced by Jean Kuntzmann \cite{Kuntzmann1972}. Traditionally, information algebras had some applications in graph theory \cite{Kuntzmann1972} and the theory of discrete-event dynamical systems \cite[p. 7]{Golan2003}. The other example of information algebras is the min-plus semiring $(\mathbb{R} \cup \{+\infty\}, \oplus, \otimes)$ in which its operations are defined as \[a \oplus b = \min \{a,b\} \text{~and~} a \otimes b = a+b.\] The min-plus semiring has essential applications in the shortest path problem in optimization \cite[Example 1.22]{Golan1999(b)} and is used extensively in tropical geometry \cite[\S1.1]{MaclaganSturmfels2015}. Information algebras have attracted the interests of some authors working recently on factorization problems \cite{BaethGotti2020,BaethSampson2020,ChenZhaoLiu2015}. \end{remark} A nonempty subset $U$ of a semiring $S$ is multiplicatively closed if $(U,\cdot)$ is a submonoid of $(S,\cdot)$. The localization of a semiring $S$ at a multiplicatively closed set $U$ of $S$, denoted by $U^{-1}S$, is defined similar to its counterpart in commutative ring theory (for the details see \S5 in \cite{Nasehpour2018S}). \begin{proposition}\label{Localizationofinformationalgebras} Let $U \subseteq S \setminus \{0\}$ be a multiplicatively closed subset of a semiring $S$. Then, the following statements hold: \begin{enumerate} \item If $S$ is entire, then so is $U^{-1}S$. \item If $S$ is an information algebra, then so is $U^{-1}S$. \end{enumerate} \end{proposition} \begin{proof} (1): This is straightforward because $a/u = 0/1$ if and only if $a = 0$, for all $a \in S$ and $u \in U$. (2): Let $(a/u) + (b/v) = 0/1$. It follows that $va + ub = 0$. Since $S$ is zerosumfree, $va = 0$ and $ub = 0$. Now, since $u$ and $v$ are nonzero and $S$ is entire, $a$ and $b$ are both zero, and so, $a/u = 0/1$ and $b/v = 0/1$. Thus $U^{-1}S$ is an information algebra and the proof is complete. \end{proof} \begin{theorem}\label{Semiringidealsinformationalgebra} Let $S$ be a semiring. Then, the following statements hold: \begin{enumerate} \item\label{Semiringidealszerosumfree} $\Id(S)$ is a partially ordered zerosumfree and additively idempotent semiring. \item $\Id(S)$ is an information algebra if and only if $S$ is entire. \end{enumerate} \end{theorem} \begin{proof} (\ref{Semiringidealszerosumfree}): By Proposition 6.29 in \cite{Golan1999(b)}, $\Id(S)$ is zerosumfree and additively idempotent. By Proposition 2.3 in \cite{Nasehpour2018P}, $(\Id(S), \subseteq)$ is a partially ordered semiring. (2): Let $S$ be entire. If $I$ and $J$ are nonzero ideals, then there are nonzero elements $a \in I$ and $b \in J$. So, $ab \in IJ$ is nonzero showing that $\Id(S)$ is entire. Now, by the statement (\ref{Semiringidealszerosumfree}), $\Id(S)$ is an information algebra. Conversely, if $\Id(S)$ is an information algebra and $a$ and $b$ are nonzero elements of $S$, then the principal ideals $(a)$ and $(b)$ are nonzero. It follows that $(a)(b)$ is also a nonzero ideal of $S$. However, $(a)(b) = (ab)$ because $S$ is commutative. Therefore, $ab$ is nonzero, i.e., $S$ is entire and the proof is complete. \end{proof} \section{Indigenous presemirings and their graphs}\label{sec:indigenousgraphs} \begin{definition} A bimagma $(R,+,\cdot)$ is a (commutative) presemiring if $(R,+)$ is a commutative semigroup, $(R,\cdot)$ is a (commutative) semigroup, and $\cdot$ distributes on $+$ from both sides (see Definition 4.2.1 in \cite{GondranMinoux2008}). The presemiring $R$ is unital if there is an element $1$ in $R$ such that $r \cdot 1 = 1 \cdot r = r$, for all $r \in R$. \end{definition} \begin{example} The bimagma $(\mathbb{N},+,\cdot)$ is a unital presemiring. \end{example} Inspired by Example 5 on p. 419 in \cite{Sibley2021}, we give the following definition: \begin{definition}\label{Indigenouspresemiringdef} Let $k$ be a positive integer. Set $I_k = \mathbb{N}_k \cup \{m\}$, where $m$ is just a symbol standing for ``many'' and is not in $\mathbb{N}_k$. Define Indigenous addition and multiplication on $I_k$ as follows: If $a, b \in \mathbb{N}_k$, then \[ a \oplus b = \begin{cases} a+b & \text{if $ a+b \leq k$} \\ m & \text{if $a+b > k$} \end{cases} \text{, and~} a \odot b = \begin{cases} ab & \text{if $ab\leq k$} \\ m & \text{if $ab > k$} \end{cases} \] and if either $a= m$ or $b = m$, then \[a \oplus b = a \odot b = m.\] \end{definition} \begin{proposition}\label{Indigenoussemiringpro} The bimagma $(I_k, \oplus, \odot)$ defined in Definition \ref{Indigenouspresemiringdef} is a unital presemiring and an epimorphic image of the presemiring $\mathbb{N}$. \end{proposition} \begin{proof} It is easy to see that $(I_k, \oplus)$ is a commutative semigroup and $(I_k,\odot,1)$ is a commutative monoid. Now, let $a$, $b$, and $c$ be elements of $I_k$. If $ab + ac$ is less than $k+1$, then $ab$ and $ac$ are also less than $k+1$, and so, the distributive laws hold because the computation is done in natural numbers. If at least one of the elements $ab$ and $ac$ are greater than $k$, then their addition is $m$ and we have \[a \odot (b \oplus c) = m = (a \odot b) \oplus (a \odot c).\] This means that $(I_k, \oplus, \odot)$ is a unital presemiring. It is easy to check that the function $f: \mathbb{N} \rightarrow I_k$ defined by \[f(x) = \begin{cases} x & \text{if $x \leq k$} \\ m & \text{if $x>k$} \end{cases}.\] is a presemiring epimorphism and the proof is complete. \end{proof} \begin{examples}\label{historicalexamplesIndigenous} Let $k$ be a positive integer number. We call the presemiring $I_k$ an Indigenous presemiring of order $k$. In the following, we give some examples mainly discussed in the literature. \begin{enumerate} \item Gordon illustrated that the Pirah\~{a} applied a numerical vocabulary corresponding to the terms ``h\'{o}i'' (for ``one''), ``ho\'{i}'' (for ``two''), and ``baagiso'' (for ``many'') \cite{Gordon2004}. One may correspond this to the Indigenous presemiring of order $2$. \item (Sibley's Indigenous presemiring) As we have already explained, we were inspired by an example given on p. 419 in Sibley's book \cite{Sibley2021}. Sibley's Indigenous presemiring is the Indigenous presemiring of order $3$. \item On p. 5 in his book \cite{Ifrah2000} on the universal history of numbers, Ifrah explains that the Botocudos had only two real terms for numbers: one for ``one'', and the other for ``a pair''. With these lexical items they could manage to express three and four by saying something like ``one and two'' and ``two and two''. However, these people had as much difficulty conceptualizing a number above four. In fact, for larger numbers, some of the Botocudos just pointed to their hair as if they were trying to say there are as ``many'' as there are hairs on their head. This may correspond to the Indigenous presemiring of order $4$. \item In his academic book \cite{Sommerfelt1938}, Sommerfelt reports that the Aranda had only two number terms, ``ninta'' for one, and ``tara'' for two. Three and four were expressed as ``tara-mi-ninta'' (i.e., two and one) and ``tara-ma-tara'' (i.e., two and two), respectively, and the number series of Aranda stopped there. For larger quantities, imprecise terms resembling ``a lot'', ``several'', and so on were used. This may also correspond to the Indigenous presemiring of order $4$. \end{enumerate} \end{examples} \begin{definition}\label{Indigenousgraphs} We define the Indigenous graph $\IG_k$ as follows: \begin{enumerate} \item The set of the vertices of $\IG_k$ is $I_k = \mathbb{N}_k \cup \{m\}$. \item The doubleton $\{a,b\}$ is an edge of $\IG_k$ if $a \neq b$ and their multiplication $a \odot b$ equals $m$. \end{enumerate} \end{definition} Let us recall that the diameter of a graph $G$, denoted by $\diam(G)$, is the greatest distance between two vertices of $G$ \cite[\S3.1.7]{BondyMurty2008}. \begin{theorem}\label{diamIG} For any positive integer $k$, the Indigenous graph $\IG_k$ is a connected graph with $\diam(\IG_1) = 1$ and $\diam(\IG_k) = 2$ if $k > 1$. \end{theorem} \begin{proof} The Indigenous graph $\IG_1$ has only two vertices $1$ and $m$ and they are connected because $1 \odot m = m$. Therefore, $\IG_1$ is the complete graph $K_2$ which is a connected graph with $\diam(\IG_1) = 1$. Now, let $k \geq 2$ and $a \neq b$ be arbitrary elements in $\mathbb{N}_k$. Since $a \odot m = b \odot m = m$, $a$ is connected to $m$ and $m$ is connected to $b$. Therefore, $\IG_k$ is a connected graph with $\diam(\IG_k) \leq 2$. However, $1$ and $k$ are not connected. So, $\diam(\IG_k) > 1$. This completes the proof. \end{proof} Let us recall that if a graph $G$ has at least one cycle, the length of a shortest cycle is its girth \cite[p. 42]{BondyMurty2008}. If a graph has no cycle, its girth is defined to be infinity. The girth of a graph is usually denoted by $g(G)$. \begin{theorem}\label{girthIG} The girth of the Indigenous graph $\IG_k$ is $3$ if $k \geq 3$, and is infinity, otherwise. \end{theorem} \begin{proof} Let $k \geq 3$. Then, the vertices $k$ and $k-1$ are connected because $k(k-1) > k$ in $\mathbb{N}$, and so, $k \odot (k-1) = m$ in $I_k$. Therefore, the triangle with the vertices $k-1$, $k$, and $m$ is a subgraph of $\IG_k$, and so, $g(\IG_k) = 3$. It is evident that the graph $\IG_1$ has no cycle. Also, in the graph $\IG_2$, the vertices $1$ and $2$ are not connected and again $\IG_2$ has no cycle. This completes the proof. \end{proof} Recall that a clique of a graph is a set of mutually adjacent vertices, and that the maximum size of a clique of a graph $G$, the clique number of $G$, is denoted by $\omega(G)$ \cite[p. 296]{BondyMurty2008}. \begin{proposition}\label{cliqueIGleq4} The clique number of $\IG_1$, $\IG_2$, $\IG_3$, and $\IG_4$ is $2$, $2$, $3$, and $4$, respectively. \end{proposition} \begin{proof} We compute the clique number of $\IG_k$ for $k \leq 4$ as follows: \begin{enumerate} \item In $\IG_1$, the vertices $1$ and $m$ are adjacent and its clique number of $\IG_1$ is $2$. \item In $\IG_2$, the vertex $m$ is adjacent to the vertices $1$ and $2$ but the vertices $1$ and $2$ are not adjacent. So, the clique number of $\IG_2$ is again $2$. \item In $\IG_3$, the vertices $m$, $2$, and $3$ are mutually adjacent while $1$ is not connected to $2$ and $3$. It follows that the clique number of $\IG_3$ is $3$. \item In $\IG_4$, the vertices $m$, $2$, $3$, and $4$ are mutually adjacent while $1$ is not connected to $2$, $3$, and $4$. This means that the clique number of $\IG_4$ is $4$. \end{enumerate} This completes the proof. \end{proof} \begin{theorem}\label{cliqueIG} The clique number of the Indigenous graph $\IG_k$ is at least $\left\lfloor \frac{k}{2} \right\rfloor + 1$, for any positive integer $k$. \end{theorem} \begin{proof} In view of Proposition \ref{cliqueIGleq4}, the inequality $\omega(\IG_k) \geq \left\lfloor \frac{k}{2} \right\rfloor + 1$ holds for all $k \leq 4$. Now, let $k \geq 5$. We distinguish two cases. Case I. If $k = 2 \alpha$ is an even number with $\alpha \geq 3$, then \[\left(k - \left\lfloor \frac{k}{2} \right\rfloor \right)^2 - k = \alpha^2 - 2\alpha > 0.\] Case II. If $k = 2\alpha + 1$ is an odd number with $\alpha \geq 2$, then \[\left(k - \left\lfloor \frac{k}{2} \right\rfloor \right)^2 - k = (\alpha+1)^2 - (2\alpha + 1) = \alpha^2 > 0.\] Therefore, all vertices \[\left(k - \left\lfloor \frac{k}{2} \right\rfloor\right), \left(k - \left\lfloor \frac{k}{2} \right\rfloor\right) + 1, \dots, k, m\] are mutually adjacent to each other in $\IG_k$. Thus $\omega(\IG_k) \geq \left\lfloor \frac{k}{2} \right\rfloor + 1$ for any positive integer number $k$ and the proof is complete. \end{proof} Recall that the smallest number of colors needed to color the vertices of a graph $G$ such that no two adjacent vertices share the same color is called the chromatic number of $G$, denoted by $\chi(G)$ \cite[p. 358]{BondyMurty2008}. It is clear that $\chi(G) \geq \omega(G)$ \cite[p. 359]{BondyMurty2008}. So, we have the following corollary: \begin{corollary}\label{chromaticIG} The chromatic number of the Indigenous graph $\IG_k$ is at least $\left\lfloor \frac{k}{2} \right\rfloor + 1$, for any positive integer $k$. \end{corollary} \section{Indigenous semirings and their ideals}\label{sec:indigenousideals} \begin{proposition}\label{presemiringembedhemiring} Any presemiring can be embedded into a hemiring. \end{proposition} \begin{proof} Let $E$ be a presemiring and suppose that $0 \notin E$. Set $E^{\prime} = E \cup \{0\}$ and define $+^{\prime}$ and ${\cdot}^{\prime}$ on $E^{\prime}$ as follows: \begin{itemize} \item $a +^{\prime} b = a + b$ for all $a,b\in E$ and $a +^{\prime} 0 = 0 +^{\prime} a = a$ for all $a\in E^{\prime}$. \item $a {\cdot}^{\prime} b = a \cdot b$ for all $a,b\in E$ and $a {\cdot}^{\prime} 0 = 0 {\cdot}^{\prime} a = 0$ for all $a\in E^{\prime}$. \end{itemize} One can easily check that $(E^{\prime}, +^{\prime}, {\cdot}^{\prime})$ is also a presemiring and the element $0$ is an identity element for addition and an absorbing element for multiplication. Now, define $\varphi : E \rightarrow E^{\prime}$ by $\varphi(x) = x$. It is clear that $\varphi$ is a presemiring monomorphism. This completes the proof. \end{proof} \begin{corollary}\label{Indigenoussemiringcor} The Indigenous presemiring $I_k$ can be embedded into the semiring $I_k \cup \{0\}$, for each $k \in \mathbb{N}$. \end{corollary} \begin{definition}\label{Indigenoussemiringdef} For any positive integer $k$, we call the semiring $I_k \cup \{0\}$ given in Corollary \ref{Indigenoussemiringcor}, the Indigenous semiring and denote it by $S_k$. \end{definition} \begin{theorem}\label{GeneralpropertiesofIndigenoussemirings} Let $k$ be a positive integer and $S_k$ the Indigenous semiring. Then, the following statements hold: \begin{enumerate} \item\label{IndigenoustotallyorderedIA} The semiring $S_k$ is a totally ordered information algebra with the smallest element $0$ and the largest element $m$. However, $S_k$ is not a semidomain. \item\label{LocalizationIndigenous} Let $k > 1$ and $U$ be a multiplicatively closed set in $S_k$ having a positive integer $a > 1$ and $0 \notin U$. Then, the localization $U^{-1} S_k$ of $S_k$ at $U$ is isomorphic to the Boolean semiring $\mathbb{B} = \{0,1\}$. \item\label{Indigenouslocal} $(S_k, S_k \setminus\{1\})$ is a local semiring. \item\label{misinanynonzeroideal} The set $\mathfrak{s}_k = \{0,m\}$ is the smallest nonzero ideal of the Indigenous semiring $S_k$. In particular, any nonzero ideal of $S_k$ possesses $m$. \item\label{Subtractiveidealsindigenoussemirings} The semiring $S_k$ is austere, i.e., the only subtractive ideals of $S_k$ are $\{0\}$ and $S_k$ (see p. 71 in \cite{Golan1999(b)}). \item\label{PrimeidealsIndigenoussemiring} The only prime ideals of the Indigenous semiring $S_k$ are $\{0\}$ and $S_k \setminus \{1\}$. \item\label{Nonzeroprincipalprime} $S_k$ has a nonzero principal prime ideal if and only if $k\leq 2$. \item\label{ZariskitopologyIndigenoussemiring} The Zariski topology of the Indigenous semiring $S_k$ is the Sierpi\'{n}ski space. \item\label{RadicalidealsIndigenoussemirings} If $I$ is a nonzero proper ideal of $S_k$, then $\sqrt{I} = S_k \setminus \{1\}$. In particular, the only radical ideals of $S_k$ are $\{0\}$, $S_k \setminus \{1\}$, and $S_k$. \item\label{SemiringidealsIndigenousinformationalgebra} $\Id(S_k)$ is a partially ordered information algebra and if $I$ is a proper nonzero ideal of $S_k$, then \[\{0\} \subseteq \mathfrak{s}_k \subseteq I \subseteq \mathfrak{m}_k \subseteq S_k.\] \item\label{Absorbingelementnonzeroideals} $(\Id(S_k)\setminus\{\mathbf{0}\}, \cdot)$ is a monoid with the absorbing element $\mathfrak{s}_k$, where by $\mathbf{0}$, we mean the zero ideal $\{0\}$ of $S_k$. \item\label{Monoidnonzeroidealsnilpotent} If $n$ is a positive integer number such that $2^n > k$, then for any nonzero proper ideals $\{I_i\}_{i=1}^{n}$ of $S_k$, we have $\prod_{i=1}^{n} I_i= \mathfrak{s}_k$; in other words, the multiplicative monoid $M = \Id(S_k)\setminus\{\mathbf{0}\}$ is nilpotent (i.e., there is a positive integer number $n$ with $M^n = \{\mathfrak{s}_k\}$). \end{enumerate} \end{theorem} \begin{proof} (\ref{IndigenoustotallyorderedIA}): If $a$ and $b$ are nonzero elements of $S_k$, then their multiplication (addition) is either a positive integer number less than $k+1$ or $m$. So, $S_k$ is entire and zerosumfree. If we set \[0 < 1 < \cdots < k < m,\] then it is easy to see that $a \leq b$ implies $a+c \leq b+c$ and $ac \leq bc$, for all $a$, $b$, and $c$ in $S_k$. This means that $S_k$ is a totally ordered information algebra with the smallest element $0$ and the largest element $m$. Note that while $m \neq 1$, we always have \[m \cdot 1 = m = m \cdot m\] which means that $S_k$ is not a semidomain. (\ref{LocalizationIndigenous}): Since $a \in U$ and $U$ is multiplicatively closed, $a^n \in U$ for each natural number $n$. It is clear that for sufficiently large enough $n$, $a^n > k$ in $\mathbb{N}$, and so, $a^n$ which is $m$ in $S_k$ is an element of $U$. Now, let $a / u$ be a nonzero element in $U^{-1} S_k$. Note that since $m \in U$ and $m / m = 1/1$ is the multiplicative identity of the semiring $U^{-1} S_k$, we have \[a/u = (a/u)(m/m) = (am)/(mm) = m/m\] showing that $U^{-1} S_k$ has only two elements $0/1$ and $1/1$. On the other hand, \[(1/1) + (1/1) = (m/m) + (m/m) = (m+m)/m = m/m = 1/1.\] This shows that $U^{-1} S_k$ is isomorphic to the Boolean semiring $\mathbb{B}$. (\ref{Indigenouslocal}): Let $a \neq 1$ and $b \neq 1$. It is easy to see that $a+b \neq 1$. Also, since $xy = 1$ if and only if $x = 1$ and $y = 1$ in $S_k$, we see that if $s \in S_k$ is arbitrary and $a \neq 1$, then $sa \neq 1$. This means that $\mathfrak{m}_k = S_k \setminus \{1\}$ is an ideal of $S_k$. However, there is no other ideals strictly between $\mathfrak{m}_k$ and $S_k$. So, $\mathfrak{m}_k$ is a maximal ideal of $S_k$. On the other hand, an ideal $I$ of a semiring is proper if and only if $1 \notin I$. Therefore, any proper ideal of $S_k$ is a subset of $\mathfrak{m}_k$. Thus $\mathfrak{m}_k$ is the only maximal ideal of $S_k$. (\ref{misinanynonzeroideal}): It is easy to see that $\mathfrak{s}_k = \{0,m\}$ is an ideal of $S_k$. Now, let $I$ be a nonzero ideal of $S_k$. If $s$ is a nonzero element of $I$, then $m = ms$ must be in $I$. (\ref{Subtractiveidealsindigenoussemirings}): Let $I$ be a nonzero proper ideal of $S_k$. Then by the statement (\ref{misinanynonzeroideal}), $m \in I$ while $1 \notin I$. However, $m+1 = m \in I$. This means that $I$ is not subtractive. It is evident that $\{0\}$ and $S_k$ are subtractive. (\ref{PrimeidealsIndigenoussemiring}): By the statement (\ref{IndigenoustotallyorderedIA}), $\{0\}$ in prime. In view of the statement (\ref{Indigenouslocal}) and Corollary 7.13 in \cite{Golan1999(b)}, $S_k \setminus \{1\}$ is also prime. Now, let $P$ be a nonzero prime ideal of $S_k$. If $a \in S_k \setminus \{0,1\}$, then either $2 \leq a \leq k$ or $a = m$. In any case, there is a positive integer $n$ such that $a^n = m$. On the other hand, by the statement (\ref{misinanynonzeroideal}), $m$ is an element of each nonzero ideal of $S_k$. Consequently, $a^n \in P$. Since $P$ is prime, we have $a \in P$. Therefore, $P = S_k \setminus \{1\}$. (\ref{Nonzeroprincipalprime}): In view of the statement (\ref{PrimeidealsIndigenoussemiring}), in $S_1$, the principal ideal $(m) = S_1 \setminus \{0\}$ is prime, and in $S_2$, $(2) = S_2 \setminus \{1\}$ is also prime. Now, let $k \geq 3$. The principal ideal $(2)$ is not prime because a suitable power of $3$ is $m \in (2)$ but $3$ is not an element of $(2)$. Now, let $p > 2$ be a prime number in $\mathbb{N}_k$. The principal ideal $(p)$ is not prime because a suitable power of $2$ is $m \in (p)$ while $2$ is not in $(p)$. If $c$ is a composite number in $\mathbb{N}_k$, then the principal ideal $(c)$ is clearly not prime. Also, $(m) = \{0,m\}$ is not prime because a suitable power of $2$ is $m \in (m)$ while $2$ is not in $(m)$. (\ref{ZariskitopologyIndigenoussemiring}): By the statement (\ref{PrimeidealsIndigenoussemiring}), the only prime ideals of $S_k$ are $\{0\}$ and $\mathfrak{m}_k = S_k \setminus \{1\}$. Therefore, by Theorem \ref{Zariskitopologyentiresemiringwithtwoprimes}, the Zariski topology of $S_k$ is the Sierpi\'{n}ski space. (\ref{RadicalidealsIndigenoussemirings}): For any ideal $I$ of $S_k$, the radical $\sqrt{I}$ of $I$ is the intersection of prime ideals of $S_k$ containing $I$ (cf. Theorem 3.2 in \cite{Nasehpour2018P}). Now, if $I$ is nonzero, then $\sqrt{I} = S_k \setminus \{1\}$ because by the statement (\ref{PrimeidealsIndigenoussemiring}), $S_k \setminus \{1\}$ is the only prime ideal of $S_k$ containing $I$. Therefore, a nonzero proper ideal of $S_k$ is radical if and only if $I = S_k \setminus \{1\}$. Now, since by the statement (\ref{IndigenoustotallyorderedIA}), $S_k$ is entire, $\{0\}$ is a radical ideal. It is evident that $S_k$ is a radical ideal. (\ref{SemiringidealsIndigenousinformationalgebra}): By Theorem \ref{Semiringidealsinformationalgebra}, $\Id(S_k)$ is a partially ordered information algebra. By the statement (\ref{misinanynonzeroideal}), $\mathfrak{s}_k$ is the smallest nonzero ideal. By the statement (\ref{Indigenouslocal}), $\mathfrak{m}_k = S_k \setminus \{1\}$ is the largest proper ideal. (\ref{Absorbingelementnonzeroideals}): Since $\Id(S_k)$ is an entire semiring, $(\Id(S_k)\setminus\{\mathbf{0}\}, \cdot)$ is a monoid. Now, let $I$ be a nonzero ideal. We need to prove that $I \cdot \mathfrak{s}_k = \mathfrak{s}_k$. By the statement (\ref{misinanynonzeroideal}), $\mathfrak{s}_k$ is the smallest nonzero ideal of $S_k$. On the other hand, \[I \cdot \mathfrak{s}_k \subseteq I \cap \mathfrak{s}_k = \mathfrak{s}_k.\] (\ref{Monoidnonzeroidealsnilpotent}): Let $n$ be a positive integer number with $2^n > k$. Let $\{I_i\}_{i=1}^{n}$ be arbitrary nonzero proper ideals of $S_k$. By the statement (\ref{misinanynonzeroideal}), $m \in I_i$ while $1 \notin I_i$. Consider $a_i \in I_i$. Observe that if at least one of the $a_i \in I_i$ is zero, then $\prod_{i=1}^{n} a_i = 0$. Also, if all of the $a_i$s are nonzero and at least one of them is $m$, then $\prod_{i=1}^{n} a_i = m$. Now, let $a_i \notin \{0,m\}$. This means that $2 \leq a_i$, for each $i$, and so, in $\mathbb{N}$, we have \[\prod_{i=1}^{n} a_i \geq 2^n > k.\] This means that $\prod_{i=1}^{n} a_i = m$ in $S_k$ and the proof is complete. \end{proof} \begin{theorem} Let $S_k$ be the Indigenous semiring and $M$ a commutative monoid. Then, the following statements hold: \begin{enumerate} \item The monoid semiring $S_k[M]$ is an information algebra. \item If $M$ is a totally ordered commutative monoid, then the function \[\deg: (S_k[M],+,\cdot,0,1) \rightarrow (M_{\infty},\max,+,-\infty,0)\] is a semiring morphism. \end{enumerate} \end{theorem} \begin{proof} (1): By Theorem 4.10 in \cite{Nasehpour2025}, if $M$ is a commutative monoid and $S$ an information algebra, then the monoid semiring $S[M]$ is also an information algebra. Thus in view of Theorem \ref{GeneralpropertiesofIndigenoussemirings}, $S_k[M]$ is an information algebra. (2): In view of Theorem \ref{GeneralpropertiesofIndigenoussemirings}, this is a special case of Corollary 4.18 in \cite{Nasehpour2025}. This completes the proof. \end{proof} \section{Distinguished elements of polynomials and formal power series over the Indigenous semirings}\label{sec:distinguishedelements} In this section, $S_k$ denotes the Indigenous semiring defined in Definition \ref{Indigenoussemiringdef}. Since $S_k$ is an entire semiring, $S_k[X]$ \cite[Corollary 2.4]{Nasehpour2021} and $S_k[[X]]$ \cite[Lemma 43]{Nasehpour2016} are also entire semirings. Therefore, $S_k$, $S_k[X]$, and $S_k[[X]]$ have no nontrivial zero-divisors (and nilpotent elements). Now, we proceed to discuss their units and idempotent elements. \begin{proposition}\label{UnitsIndigenoussemirings} The only unit element of $S_k$, $S_k[X]$, and $S_k[[X]]$ is $1$. \end{proposition} \begin{proof} Obviously in $S_k$, $ab = 1$, implies that $a = b = 1$. So, the only unit element of $S_k$ is $1$. Let $f,g \in S_k[X]$ with $fg = 1$. Since $S_k$ is an entire semiring, we obtain that $\deg(f) = \deg(g) = 0$. Therefore, $f = g = 1$. Now, let $f, g \in S_k[[X]]$ with $fg = 1$. Suppose that $f = \sum_{i=0}^{+\infty} a_i X^i$ and $g = \sum_{j=0}^{+\infty} b_j X^j$. Clearly, $a_0 = b_0 = 1$. Since $S_k$ is zerosumfree, from $fg = 1$, we obtain that $a_i = b_i = 0$, for all $i \geq 1$. This completes the proof. \end{proof} \begin{proposition}\label{IdempotentsIndigenoussemirings1} The only idempotent elements of $S_k$ and $S_k[X]$ are $0$, $1$, and $m$. \end{proposition} \begin{proof} In $S_k$, if $a$ is different from $0$, $1$, and $m$, then $2 \leq a \leq k$. If $a^2 \leq k$, then $a^2 \neq a$. If $a^2 > k$, then $a^2 = m$ which means that again $a^2 \neq a$. Thus the only idempotent elements of $S_k$ are $0$, $1$, and $m$. Now, let $f \in S_k[X]$. Since $S_k$ is an entire semiring, then \[\deg(f^2) > \deg(f)\] except the case that $f$ is a constant polynomial. Therefore, $f^2 = f$ if and only if $f = a$, where $a \in S_k$. This means that $f$ is idempotent in $S_k[X]$ if and only if $f$ is either $0$ or $1$ or $m$. This completes the proof. \end{proof} \begin{theorem}\label{IdempotentsIndigenoussemirings2} An element $f$ in $S_k[[X]]$ is idempotent if and only if either $f = 0$ or $f = 1$ or $f = m$ or \[f = a_0 + \sum_{i=1}^{+\infty} m X^{s_i},\] where $a_0 = 1,m$ and the set $\{s_i\}_{i=1}^{+\infty}$ is a subsemigroup of $(\mathbb{N},+)$. \end{theorem} \begin{proof} Let $f = \sum_{i=0}^{+\infty} a_i X^i$ be idempotent. It follows that $a^2_0 = a_0$, and so, by using Proposition \ref{IdempotentsIndigenoussemirings1}, we can distinguish three cases: \begin{enumerate} \item The case $a_0 = 0$. If $a_0 = 0$, then $f = \sum_{i=1}^{+\infty} a_i X^i$. Now, if $a_i \neq 0$, for some $i > 0$, then obviously $f^2 \neq f$. Therefore, $f = 0$. \item The case $a_0 = 1$. Our claim is that $a_i \in \{0,m\}$, for any $i > 0$. This is because if $a_i \neq 0,m$, for some $i > 0$, then the coefficient of $X^i$ in $f^2$ is at least $2a_i$ in $\mathbb{N}$ which is never $a_i$ in $S_k$. Now, consider \[f = 1 + mX^{s_1} + mX^{s_2} + \cdots.\] Our claim is that $f$ is idempotent if and only if the set $E = \{s_i\}_{i=1}^{+\infty}$ is a subsemigroup of $(\mathbb{N},+)$. For the direct implication, we need to show that $s_i + s_j \in E$, for all $s_i \in E$ and $s_j \in E$. Consider the monomials $mX^{s_i}$ and $mX^{s_j}$ in $f$. Therefore, $mX^{s_i + s_j}$ is a monomial in $f^2$. Since $f^2 = f$, $s_i + s_j$ needs to be one of the exponents of the monomials of $f$ which means that $E$ is a subsemigroup of $\mathbb{N}$. For the converse implication, observe that an easy calculation shows that if $\{s_i\}_{i=1}^{+\infty}$ is a subsemigroup of $\mathbb{N}$, then $f^2 = f$. \item The case $a_0 = m$. Our claim is that in this case also $a_i \in \{0,m\}$, for any $i > 0$. This is because if $a_i \neq 0,m$, for some $i > 0$, then the coefficient of $X^i$ in $f^2$ is \[a_0 a_i + \dots + a_ia_0 = m a_i + \dots + a_i m = m\] which is never the same as $a_i$, i.e., the coefficient $X^i$ in $f$. Now, consider \[f = m + mX^{s_1} + mX^{s_2} + \cdots.\] Similar to the second case, one can easily check that $f$ is idempotent if and only if the set $\{s_i\}_{i=1}^{+\infty}$ is a subsemigroup of $(\mathbb{N},+)$. \end{enumerate} This completes the proof. \end{proof} \begin{theorem} Let $\alpha \neq 0$ and $\beta$ be elements of the Indigenous semiring $S_k$ and $X$ an indeterminate over $S_k$. Then, $f=\alpha X^2+\beta$ is irreducible if and only if one of the following cases happens: \begin{enumerate} \item $\alpha$ and $\beta$ are in $\mathbb{N}_k$ and $\gcd(\alpha, \beta) = 1$. \item $\alpha = m$ and $\beta = 1$. \item $\alpha = 1$ and $\beta = m$. \end{enumerate} \end{theorem} \begin{proof} In view of Proposition 6.2 in \cite{Nasehpour2025} and Theorem \ref{GeneralpropertiesofIndigenoussemirings} in the current paper, $f$ cannot be factored into $g = aX+b$ and $h = cX+d$ in $S_k[X]$, where $a$ and $c$ are nonzero in $S_k$. Therefore, $f$ is reducible if and only if there is a nonzero nonunit $\gamma$ in $S_k$, i.e., $\gamma \neq 0,1$ such that $f = \gamma g$, for some $g = \alpha'X^2 + \beta' \in S_k[X]$ which means that \[\alpha = \gamma \alpha' \wedge \beta = \gamma \beta'.\] Note that if $\alpha = \beta = m$, then $f$ is reducible. Therefore, if $f$ is irreducible, then at least one of the coefficients of $f$ must be a natural number. Now, observe the following: \begin{enumerate} \item If $\alpha$ and $\beta$ are natural numbers, then $\gamma \neq m$, and $f$ is reducible if and only if $ \gamma \mid \gcd(\alpha,\beta)$, i.e., $\gcd(\alpha,\beta) > 1$. \item If $\alpha = m$, then $f$ is reducible if and only if $\beta > 1$ because $mX^2 + 1$ is irreducible and \[f = mX^2 + \beta = \beta (mX^2 + 1).\] \item If $\beta = m$, then $f$ is reducible if and only if $\alpha > 1$ because $X^2 + m$ is irreducible and \[f = \alpha X^2 + m = \alpha (X^2 + m).\] \end{enumerate} This gives the characterization of all irreducible polynomials of the form $\alpha X^2+\beta$ and the proof is complete. \end{proof} \subsection*{Acknowledgments} The authors wish to thank the anonymous referees for their valuable comments and suggestions, which improved the presentation of this paper. \bibliographystyle{plain} \begin{thebibliography}{15.} \bibitem{ArensDugundji1951} Arens, R., Dugundji, J.: Topologies for function spaces. Pac. J. Math. 1, 5--31 (1951). \bibitem{BaethGotti2020} Baeth, N.R., Gotti, F.: Factorizations in upper triangular matrices over information semialgebras. J. Algebra 562, 466--496 (2020). \bibitem{BaethSampson2020} Baeth, N.R., Sampson, R.: Upper triangular matrices over information algebras. Linear Algebra Appl. 587, 334--357 (2020). \bibitem{BenderBeller2018} Bender, A., Beller, S.: Numeration systems as cultural tools for numerical cognition. In Language and culture in mathematical cognition, 297--320. Academic Press, (2018). \bibitem{BenderBeller2021} Bender, A., Beller, S.: Ways of counting in Micronesia. Hist. Math. 56, 40--72 (2021). \bibitem{BondyMurty2008} Bondy, J.A., Murty, U.S.R.: Graph theory. Graduate Texts in Mathematics 244. Berlin: Springer. xii, 651 p. (2008). \bibitem{Bourne1951} Bourne, S.: The Jacobson radical of a semiring. Proc. Natl. Acad. Sci. USA 37, 163--170 (1951). \bibitem{ChenZhaoLiu2015} Chen, Y., Zhao, X.; Liu, Z.: On upper triangular nonnegative matrices. Czech. Math. J. 65, No. 1, 1--20 (2015). \bibitem{Everett2005} Everett, D.L.: Cultural constraints on grammar and cognition in Pirah\~{a}. Current Anthropology, 46, 621--646 (2005). \bibitem{Gillilland1975} Gillilland, C.L.C.: Stone money of Yap: a numismatic survey. Washington: Smithsonian Institution Press. Smithsonian Studies in History and Technology. 23, 75 p. (1975). \bibitem{Golan2003} Golan, J.S.: Semirings and affine equations over them: theory and applications. Mathematics and its Applications (Dordrecht) 556. Dordrecht: Kluwer Academic Publishers. xiv, 241 p. (2003). \bibitem{Golan1999(b)} Golan, J.S.: Semirings and their applications. Dordrecht: Kluwer Academic Publishers. xi, 381 p. (1999). \bibitem{GondranMinoux2008} Gondran, M., Minoux, M.: Graphs, dioids and semirings. New models and algorithms. Operations Research/Computer Science Interfaces Series 41. Dordrecht: Springer. xix, 383 p. (2008). \bibitem{Gordon2004} Gordon, P.: Numerical cognition without words: Evidence from Amazonia. Science, 306, 496--499 (2004). \bibitem{Ifrah2000} Ifrah, G.: The universal history of numbers. From prehistory to the invention of the computer. Translated from the 1994 French original by David Bellos, E.F. Harding, Sophie Wood and Ian Monk. New York, NY: Wiley. xxii, 633 p. (2000). \bibitem{Kuntzmann1972} Kuntzmann J.: Th\'{e}orie des r\'{e}seaux: graphes. Paris: Dunod. 228 p. (1972). \bibitem{MaclaganSturmfels2015} Maclagan, D., Sturmfels, B.: Introduction to tropical geometry. Graduate Studies in Mathematics 161. Providence, RI: American Mathematical Society (AMS). xii, 363 p. (2015). \bibitem{Nasehpour2025} Nasehpour, P.: Distinguished elements in semiring extensions. J. Algebra Appl. 24, No. 3, Article ID 2550071, 26 p. (2025). \bibitem{Nasehpour2016} Nasehpour, P.: On the content of polynomials over semirings and its applications. J. Algebra Appl. 15, No. 5, Article ID 1650088, 32 p. (2016). \bibitem{Nasehpour2021} Nasehpour, P.: On zero-divisors of semimodules and semialgebras. Georgian Math. J. 28, No. 3, 413--428 (2021). \bibitem{Nasehpour2018P} Nasehpour, P.: Pseudocomplementation and minimal prime ideals in semirings. Algebra Univers. 79, No. 1, Paper No. 11, 15 p. (2018). \bibitem{Nasehpour2018S} Nasehpour, P.: Some remarks on ideals of commutative semirings. Quasigroups Relat. Syst. 26, No. 2, 281--298 (2018). \bibitem{Nasehpour2019} Nasehpour, P.: Some remarks on semirings and their ideals. Asian-Eur. J. Math. 12, No. 7, Article ID 2050002, 14 p. (2019). \bibitem{Rosenfeld1968} Rosenfeld, A.: An introduction to algebraic structures. San Francisco-Cambridge-London-Amsterdam: Holden-Day. XIV, 285 p. (1968). \bibitem{Rotman1988} Rotman, J.J.: An introduction to algebraic topology. Graduate Texts in Mathematics, 119. New York (USA) etc.: Springer-Verlag. xiii, 433 p. (1988). \bibitem{Sibley2021} Sibley, T.Q.: Thinking algebraically. An introduction to abstract algebra. AMS/MAA Textbooks 65. Providence, RI: American Mathematical Society (AMS). xiii, 478 p. (2021). \bibitem{Sommerfelt1938} Sommerfelt, A.: La langue et la soci\'{e}t\'{e}. Oslo: H. Aschehoug \& Co., 233 p. (1938). \bibitem{Vandendriessche2022} Vandendriessche, E.: The concrete numbers of ``primitive'' societies: a historiographical approach. Hist. Math. 59, 12--34 (2022). \end{thebibliography} \end{document}
2412.02093v1
http://arxiv.org/abs/2412.02093v1
Twist Coefficients of Periodic Orbits of Minkowski Billiards
\documentclass[reqno]{amsart} \usepackage[left=1in, top=1in, right=1in, bottom=1in]{geometry} \usepackage{amsmath, amsthm, amssymb} \usepackage{mathtools} \usepackage{mathrsfs} \usepackage{graphicx} \usepackage{subcaption} \usepackage[all]{xy} \usepackage{enumerate} \usepackage{tikz} \usetikzlibrary{math,angles,quotes,intersections,calc,through} \newtheorem{theorem}{Theorem}[section] \newtheorem{corollary}[theorem]{Corollary} \newtheorem{proposition}[theorem]{Proposition} \theoremstyle{definition} \newtheorem{definition}[theorem]{Definition} \newtheorem{remark}[theorem]{Remark} \numberwithin{equation}{section} \DeclareMathOperator{\Tr}{Tr} \def\bC{\mathbb{C}} \def\bQ{\mathbb{Q}} \def\bR{\mathbb{R}} \def\bT{\mathbb{T}} \def\bZ{\mathbb{Z}} \def\cM{\mathcal{M}} \def\cT{\mathcal{T}} \def\dsds{\frac{\partial s_1}{\partial s}} \def\dsdu{\frac{\partial s_1}{\partial u}} \def\duds{\frac{\partial u_1}{\partial s}} \def\dudu{\frac{\partial u_1}{\partial u}} \def\gt{\gamma(t)} \def\dgs{\dot{\gamma}(s)} \def\dgso{\dot{\gamma}(s_1)} \def\dgis{\gamma'_j(s)} \def\dgjs{\gamma'_k(s)} \def\dgiss{\gamma'_j(s_1)} \def\dgjss{\gamma'_k(s_1)} \def\ddgis{\gamma''_j(s)} \def\ddgiss{\gamma''_j(s_1)} \def\dgos{\gamma'_1(s)} \def\dgts{\gamma'_2(s)} \def\dgoss{\gamma'_1(s_1)} \def\dgtss{\gamma'_2(s_1)} \def\ddgos{\gamma''_1(s)} \def\ddgs{\gamma''(s)} \def\ddgss{\gamma''(s_1)} \def\ddgts{\gamma''_2(s)} \def\ddgoss{\gamma''_1(s_1)} \def\ddgtss{\gamma''_2(s_1)} \def\Fi{F_{v_j}(v)} \def\Fij{F_{v_jv_k}(v)} \def\ta{\tilde{a}} \def\tb{\tilde{b}} \def\pa{\partial} \def\ds{\displaystyle} \begin{document} \title{Twist Coefficients of Periodic Orbits of Minkowski Billiards} \author[C. Villanueva]{Carlos Villanueva$^*$} \address{Department of Mathematics, University of Oklahoma, Norman, OK 73019, USA} \email{\tt [email protected]} \thanks{C.V. is supported in part by the Department of Defense SMART Scholarship OMB NO. 0704-0466.} \thanks{$^*$ Author to whom any correspondence should be addressed.} \author[P. Zhang]{Pengfei Zhang} \address{Department of Mathematics, University of Oklahoma, Norman, OK 73019, USA} \email{\tt [email protected]} \subjclass[2020]{37G05 37C83 37E40 53B40} \keywords{Minkowski norm, Finsler norm, billiards, normal form, twist coefficient, nonlinear stability} \dedicatory{To Leonid Bunimovich on his 75th birthday} \begin{abstract} We investigate the fundamental properties of Minkowski billiards and introduce a new coordinate system $(s,u)$ on the phase space $\mathcal{M}$. In this coordinate system, the Minkowski billiard map $\mathcal{T}$ preserves the standard area form $\omega = ds \wedge du$. We then classify the periodic orbits of Minkowski billiards with period $2$ and derive formulas for the twist coefficient $\tau_1$ for elliptic periodic orbits, expressed in terms of the geometric characteristics of the billiard table. Additionally, we analyze the stability properties of these elliptic periodic orbits. \end{abstract} \maketitle \tableofcontents \section{Introduction}\label{sec.introduction} Dynamical billiards describe a system in which a particle moves within a container and reflects off its walls according to specific reflection rules. In \cite{Bi27, Bi27b} Birkhoff studied the dynamical billiards on convex domains with smooth boundaries and proved the existence of periodic orbits of period $n$ for any $n\ge 2$. In \cite{La73} Lazutkin proved the existence of caustics of such convex billiards near the boundaries of the phase space. Sinai pioneered the study of chaotic billiards. In his seminal paper \cite{Sin70}, he investigated dynamical billiards with dispersing boundaries and proved both the hyperbolicity and ergodicity of such systems. In \cite{Bu74a, Bu74}, Bunimovich observed that convex circular arcs could be used to construct chaotic billiards and introduced the stadium billiard, which became a favorite model among physicists. The construction of chaotic billiards was significantly extended by Wojtkowski \cite{Woj86}, Markarian \cite{Mar88}, Donnay \cite{Don91}, and Bunimovich \cite{Bu92}. Since then, several classes of chaotic billiards have been discovered. Notable examples include the elliptic stadium \cite{MaKaPdC}, track billiards \cite{BuDM}, asymmetric lemon billiards \cite{BZZ, JZ21}. An overview of chaotic billiards can be found in \cite{ChMa}. Birkhoff Normal Form plays an important role in the study of stability properties of elliptic periodic orbits, see \cite{Bi27, SiMo}. Under certain nonresonance assumptions, the dynamical system around an elliptic fixed point can be viewed as a perturbation of an integrable model, which is called the normal form of the map. If the integrable model has nonzero twist, then the elliptic fixed point must be nonlinearly stable \cite{Mos56, Mos73}. That is, the fixed point is contained in a nesting sequence of invariant disks whose boundaries are smooth invariant curves. Generally speaking, the twist coefficients can be represented as rational functions of the coefficients of the Taylor polynomials at the elliptic fixed point, see \cite{Moe90, CGM}. These rational functions, albeit explicit, are generally very involved. For dynamical systems with geometric background, it is expected that these twist coefficients can be represented as much simpler rational functions of the geometric quantities such as distances and curvatures. In \cite{KP05}, Kamphorst and Pinto-de-Carvalho showed that the first twist coefficient of a elliptic periodic orbit of period 2 of a planar billiard can be expressed a simple rational function in terms of the geometric quantities such as the orbit length and the radii of curvature of the boundary of the billiard table. In \cite{JZ22}, Jin and Zhang obtained an explicit formula of the second twist coefficient of periodic orbits of period 2 in terms of the same geometric quantities of the billiards. The authors also provided applications of their formula in the study of stability properties to various billiard systems. Dynamical billiards can also be defined on compact domains of Riemannian manifolds, on which a particle moves along geodesics in the interior of the domain and makes elastic reflections upon impact on the boundaries, see \cite{Vet, Zha17}. Therefore, billiard flows can be viewed as extensions of geodesic flows to Riemannian manifolds with boundaries. Interestingly, dynamical billiards on curved surfaces are related to the study of quantum magnetic confinement of non-planar 2D electron gases (2DEG) in semiconductors \cite{FLBP}, where the effect of varying the curvature of the surface corresponds to a change in the potential energy of the system. In \cite{GT02}, Gutkin and Tabachnikov expanded the theory by introducing dynamical billiards on Minkowski spaces, and more generally, on Finsler manifolds. Unlike Euclidean and Riemannian settings, these spaces are not equipped with an inner product but rather a norm. As a result, the concept of angles, which plays a crucial role in defining reflections in Euclidean and Riemannian billiards, no longer exists. Instead, in \cite{GT02}, they revisited the fundamental concept of reflections through the critical points of the trajectory length function. Since then, several notable results have emerged regarding Minkowski and Finsler billiards (see \cite{AO14, AFOR}), along with some unexpected applications in symplectic and contact geometry (see \cite{AKO14}). In this paper, we investigate the properties of periodic orbits in dynamical billiards on Minkowski planes. Let $F:\mathbb{R}^2 \to \mathbb{R}$ be a Minkowski norm and $Q \subset \mathbb{R}^2$ be a connected, bounded domain with piecewise smooth boundaries. We consider a point mass moving freely inside $Q$, with reflections occurring at the boundary according to Finsler reflection laws (see Proposition~\ref{pro.Finsler.reflection.law}), thereby defining a Minkowski dynamical billiard system (or {\it Minkowski billiards} for short). The phase space $\mathcal{M}$ comprises all unit vectors at the boundary $\partial Q$, pointing inward toward $Q$. Notably, this definition aligns with that of Euclidean billiards. However, the conventional arclength-angle coordinate system $(s, \theta)$ is no longer applicable, as the angle is undefined in a normed space. We introduce a new coordinate system $(s,u)$ for the phase space $\mathcal{M}$ and demonstrate that the Minkowski billiard map $\mathcal{T}$ is symplectic using the generating function method. Subsequently, we express the tangent map $D\mathcal{T}$ in terms of geometric quantities of the billiard table, see Proposition~\ref{pro.DT.Minkowski}. This enables classification of period-$2$ orbits as elliptic, parabolic, or hyperbolic. Under mild nonresonance conditions for elliptic orbits, we derive a concise formula for the first twist coefficient $\tau_1$ in terms of the geometric quantities of the billiard table, see Theorem~\ref{thm.t1.sym} and \ref{thm.t1.asym}. Combining with Moser's twist mapping theorem, we investigate specific Minkowski billiard families and determine conditions for nonlinear stability of periodic orbits, see Section~\ref{sec.applications}. \section{Preliminaries} \subsection{Minkowski spaces and Finsler manifolds} Finsler geometry has its origin in the study of the calculus of variations. It allows to study more expansive cases such as objects traveling in anisotropic mediums where speed depends on the direction of travel. For a thorough introduction to Minkowski Spaces and Finsler Geometry, see \cite{BCS, ChSh, Ru59}. \begin{definition}\label{def.Minkowski} A \textit{Minkowski norm} on the space $\bR^n$ is a function $F: \bR^n \rightarrow [0, \infty)$ satisfying the following conditions: \begin{enumerate}[(i)] \item Regularity: $F$ is $C^\infty$ on $\bR^n \backslash \{0\}$. \item Positive homogeneity: $F(c \, v) = c\, F(v)$ for any number $c >0$ and any vector $v \in \bR^n$. \item Strong convexity: The $n \times n$ Hessian matrix $ (g_{jk}(v)) := \bigg( \frac{\pa^2}{\pa v^j \pa v^k} \big( \frac{1}{2} F^2\big)(v) \bigg)$ is positive definite at every point in $\bR^n \backslash \{0\}$. \end{enumerate} Given a Minkowski norm $F$ on $\bR^n$, the pair $(\bR^n, F)$ is called a \textit{Minkowski space}. The subset $I=\{v\in \bR^n: F(v) =1\}$ of all $F$-unit vectors is called the \textit{indicatrix} of the Minkowski space. \end{definition} It is clear that the norm $F$ can be recovered from its indicatrix $I$. \begin{definition}\label{def.Finsler} Let $M$ be an $n$-dimensional closed manifold. A \textit{Finsler structure} on the manifold $M$ is a function $F: TM \rightarrow [0, \infty)$ satisfying the following conditions: \begin{enumerate}[(i)] \item Regularity: $F$ is $C^\infty$ on $TM \backslash \{0\}$. \item Positive homogeneity: $F(x, c \, v) = c\, F(x, v)$ for any $c >0$ and any $v \in T_xM$. \item Strong convexity: The $n \times n$ Hessian matrix $ (g_{jk}(x, v)) := \bigg(\frac{\pa^2}{\pa v^j \pa v^k} \big( \frac{1}{2} F^2\big)(x, v) \bigg)$ is positive definite at every point in $TM \backslash \{0\}$. \end{enumerate} Given a manifold $M$ and a Finsler structure $F$ on $TM$, the pair $(M,F)$ is called a \textit{Finsler manifold}. \end{definition} It is clear that restricting a Finsler structure $F$ to the tangent space $T_xM$ gives rise to a Minkowski space $(T_xM, F(x,\cdot))$ for each $x\in M$, and a Minkowski space can be viewed as a homogeneous Finsler manifold, where the Finsler structure is independent of the base point. \begin{definition} The \textit{Legendre transform} on the Minkowski space $(\bR^n, F)$ is defined by \begin{align*} \mathcal{L}: I &\to (\bR^n)^{\ast} \\ u &\mapsto p=\mathcal{L}(u) \end{align*} such that $\ker (p) = T_uI$ and $p(u) = 1$. \end{definition} \subsection{Mixed norm spaces} Given two numbers $a>0$ and $b\ge 0$, and a positive integer $k\ge 1$, we consider the Minkowski space $(\bR^2, F_{a,b, 2k})$, where the norm $F_{a,b, 2k}$ is given by \begin{equation} F_{a,b,2k}(v) = a\lVert v \rVert_{2} + b\lVert v \lVert_{2k}, \end{equation} where $\lVert v \rVert_2 = \left( v_1^2 + v_2^2 \right)^{\frac{1}{2}}$ and $\lVert v \rVert_{2k} = \left(v_1^{2k} + v_2^{2k}\right)^{\frac{1}{2k}}$. Clearly, $F_{1,0,k}(v) = \lVert v \rVert_2$ is just the usual Euclidean norm on $\bR^2$. \begin{proposition} $(\bR^2, F_{a,b,2k})$ is a Minkowski space. \label{prop: Mixed norm is Minkowski} \end{proposition} \begin{proof} We only need to check that condition $(iii)$ in Definition \ref{def.Finsler} is satisfied as the first two are easy to see. Let $F = F_{a,b,2k}$ and $(g_{jk}) = \left(\left[\frac{1}{2}F^2\right]_{v_jv_k}\right)$ be the Hessian matrix of $\frac{1}{2}F^2$ whose entries are given by \begin{align*} g_{11} &= \left(F_{v_1}\right)^2 + FF_{v_1v_1}, \\ g_{12} &= g_{21} = F_{v_1}F_{v_2} + FF_{v_1v_2}, \\ g_{22} &= \left(F_{v_2}\right)^2 + FF_{v_2v_2}. \end{align*} It suffices to show that $g_{11} > 0$ and $\det(g_{jk})>0$. The first and second order partials of $F$ are given by \begin{align*} F_{v_1} &= av_1(v_1^2 + v_2^2)^{-\frac{1}{2}} + bv_1^{2k-1}(v_1^{2k} + v_2^{2k})^{\frac{1}{2k}-1}, \\ F_{v_2} &= av_2(v_1^2 + v_2^2)^{-\frac{1}{2}} + bv_2^{2k-1}(v_1^{2k} + v_2^{2k})^{\frac{1}{2k}-1}, \\ F_{v_1v_1} &= \alpha v_2^2 + (2k-1)\beta v_1^{2k-2}v_2^{2k}, \\ F_{v_2v_2} &= \alpha v_1^2 + (2k-1)\beta v_1^{2k}v_2^{2k-2}, \\ F_{v_1v_2} &= -v_1v_2\left[ \alpha + \beta(2k-1)(v_1v_2)^{2k-2} \right], \end{align*} where $\alpha = a(v_1^2 + v_2^2)^{-\frac{3}{2}}$ and $\beta = b(v_1^{2k} + v_2^{2k})^{\frac{1}{2k}-2}$. Note $F_{v_1v_1},F_{v_2v_2} > 0$ implies $g_{11} > 0$. Further, $\left( F_{v_1v_2} \right)^2 = F_{v_1v_1}F_{v_2v_2}$, which we can use to show \begin{align*} \det(g_{jk}) &= F\left[(F_{v_1})^2F_{v_2v_2} - 2F_{v_1}F_{v_2}F_{v_1v_2} + F_{v_1v_1}(F_{v_2})^2 \right] \\[5pt] &= F\left(F_{v_1}\sqrt{F_{v_2v_2}}-F_{v_2}\sqrt{F_{v_1v_1}}\right)^2 > 0. \end{align*} This completes the proof. \end{proof} Starting from Section~\ref{sec.billiards.symmetry}, we will restrict to the case $k=2$, and will denote $F_{a,b} = F_{a,b, 2}$ for short. \subsection{Curvatures in Minkowski planes} As mentioned in the introduction, the twist coefficients of an elliptic periodic billiard orbit depend on the geometry of the billiard table, namely the curvatures at the points of reflection and the distance between them. To analyze the stability properties of billiard orbits on Minkowski planes, it is helpful to extend the concept of curvature from Euclidean spaces to Minkowski spaces. In the following we will explain how curvatures extend to Minkowski planes with norms following the exposition \cite{BMS19}. On the Euclidean plane, given a curve $\gamma$ with Euclidean length $\ell$, we can intuitively think of its curvature as how fast the tangent field varies with respect to the distance traveled on $\gamma$. Formally we define curvature as follows. Let $\gamma:[0, \ell] \rightarrow \bR^2$ be a smooth curve with arc-length parameter $s$, $A_s$ be the area of the sector in the unit circle between $\gamma'(0)$ and $\gamma'(s)$, and $\nu(s) = 2A_s$. The (signed) curvature of the curve $\gamma$ at $\gamma(s)$ is defined as $\kappa(s) = \nu'(s)$. Equivalently, since $\gamma''$ is normal to $\gamma'$, we can define $\kappa(s)$ in terms of the positively oriented normal vector field $n(s)$ of unit length: $\gamma''(s) = \kappa(s)n(s)$. Geometrically, this means that the area of the unit circle determined by the normal field $n(s)$ is the same as the area determined by the tangent field $\gamma'$. Lastly one can show that $\kappa(s) = \frac{1}{R(s)}$, where $R(s)$ is the radius of the circle attached to $\gamma(s)$ such that their tangent lines agree. While these three definitions of curvature yield the same quantity in the Euclidean plane, they diverge and produce distinct quantities in the more general Minkowski planes, as demonstrated in \cite{BMS19}. The difference arises from using a new definition of orthogonality that is not necessarily symmetric due to the lack of an inner product structure. For the rest of this section, let $(\bR^2, F)$ be a normed plane with $\lVert \cdot \rVert = F(\cdot)$ and $I$ denote the indicatrix at an arbitrary point. \begin{definition} Given two vectors $v,w \in V$, $v$ is said to be \textit{Birkhoff orthogonal} to $w$, denoted $v \dashv_B w$, if $\lVert v \rVert \leq \lVert v + tw \rVert$ for all $t \in \bR$. \end{definition} Geometrically, $v \dashv_B w$ indicates that $T_{v/\lVert v \rVert}I$ is parallel to $w$. Birkhoff orthogonality is not symmetric, and reversing it involves the use of a different norm. In fact, closed curves that do reverse Birkhoff orthogonality are called \textit{Radon Curves}. \begin{definition} Fix a determinant form $[\cdot, \cdot]$ on a normed space $V$ i.e. a non-degenerate bi-linear form such that $[v,v] = 0$ for all $v$. The \textit{anti-norm} on $V$ is defined as \begin{equation} \lVert v \rVert_a := \sup\{ |[w,v]| : \lVert w \rVert = 1 \} \label{eq: anti norm} \end{equation} \end{definition} In $\bR^2$, a determinant form is unique up to a constant multiple since it amounts to fixing an orientation with unit area. Further, although $\dim V = \dim V^*$, there is no canonical isomorphism between $V$ and $V^*$. However, any isomorphism is given by an identification $v \mapsto \left[\cdot, v \right]$ for any $v \in V$, where $[\cdot, \cdot]$ is a non-degenerate bilinear form. It follows from Eq.~\eqref{eq: anti norm} that $|[w,v]| \leq \lVert w \rVert \lVert v \rVert_a$, and it is shown in \cite{MS06} that $w \dashv_B v$ if and only if $v \dashv_B^a w$, where $\dashv_B^a$ denotes Birkhoff orthogonality in the antinorm. Note that whenever $w \dashv_B v$ and $\Vert w \rVert = 1 $, Eq.~\eqref{eq: anti norm} is equal to the dual norm of the functional $[\cdot,v] \in V^*$. So for example, whenever $\frac{1}{p} + \frac{1}{q} = 1$, the norms on $\ell_p$ and $\ell_q$ reverse Birkhoff orthogonality. Having redefined orthogonality we can now generalize the normal field and the first curvature concept, where curvature was given in terms of the area swept within the unit circle by the tangent field $\gamma'$. Let $n_\gamma : [0,\ell_\gamma] \rightarrow V$ be the vector field such that each $s \in [0,\ell_\gamma]$ gets mapped to the unique vector such that $\gamma'(s) \dashv_B n_\gamma(s)$ and $[\gamma'(s),n(s)] = 1$. We call $n_\gamma$ the \textit{right normal field to $\gamma$}, and in \cite{BMS19} it is shown how $n_\gamma(s)$ is unit in the anti-norm for all $s$. \begin{definition} Let $\gamma : [0,\ell_\gamma] \rightarrow \bR^2$ be a smooth curve of normed length $\ell_\gamma$ with arc-length parameter $s$ (in $\lVert \cdot \rVert$), and $\varphi: [0,2A(I)] \rightarrow \bR^2$ be a parameterization of the indicatrix $I$ by twice the area of its sectors. Identify the tangent field $\gamma'$ within the indicatrix so that \begin{equation} \gamma'(s) = \varphi(\nu(s)), \end{equation} where $\nu: [0, \ell_\gamma] \rightarrow \bR$ maps the arc-length of $T$ to twice the area of the sectors spanned by $\gamma'(0)$ and $\gamma'(s)$. The \textit{Minkowski curvature} of $\gamma$ at $\gamma(s)$ is defined as \begin{equation} \kappa_m := \nu'(s). \label{eq: Minkowski curvature} \end{equation} \end{definition} Differentiating equation (\ref{eq: Minkowski curvature}) yields \begin{equation} \gamma''(s) = \nu'(s) \frac{d \varphi}{d \nu}(\nu(s)) = \kappa_m(s)n_\gamma(s). \label{eq: d2 gamma} \end{equation} Geometrically, this means that as $\gamma'$ rotates about the indicatrix $I$, $n_\gamma$ rotates about the indicatrix of the anti-norm $I_a$. The analogous definition to equation (\ref{eq: Minkowski curvature}), but where the area is spanned by $n_\gamma$ within $I_a$ is called the \textit{normal curvature} and is denoted $\kappa_n$. However, since the indicatrix in the norm and anti-norm are generally not the same, $\gamma'$ and $n_\gamma$ sweep different areas and thus yield different values for $\kappa_m(s)$ and $\kappa_n(s)$. For later convenience, it will be useful to express equation \eqref{eq: d2 gamma} in terms of a Euclidean structure to facilitate the calculation of the higher order derivatives of $\gamma''(s)$ needed in the Taylor expansion of the billiard map $\mathcal{T}$. Given a parameterization $r(\theta(s))$ of the indicatrix $I$ such that $\gamma'(s) = r(\theta(s))\cdot (\cos\theta(s),\sin\theta(s))$ for $\theta(s) \in [0,2\pi]$, \cite{BMS19} shows how $\kappa_m$ and $n_\gamma$ may be expressed as \begin{align} \kappa_m(s) &= k_e(s)r(\theta(s))^3 \label{eq: km in Euclid}\\[10pt] n_\gamma(s) &= \frac{1}{r(\theta(s))^2}\frac{dr}{d\theta}(\theta(s)) \cdot (\cos\theta(s),\sin\theta(s)) \nonumber \\ &\qquad + \frac{1}{r(\theta(s))} \cdot (-\sin\theta(s),\cos\theta(s)). \label{eq: n in Euclid} \end{align} \subsection{Periodic orbits}\label{sec.dysy} Let $X$ be a topological space and $f: X \rightarrow X$ a continuous map. Then the pair $(X, f)$ is called a dynamical system. The iterates of the map $f$ can be defined recursively via $f^{n+1} = f\circ f^{n}$ for any $n\ge 0$. If $f$ is a homeomorphism, then we can define the backward iterates $f^{-n}$, $n\ge 1$. A point $x \in X$ is {periodic} if $f^n(x) = x$ for some $n \ge 1$, and the {period} is the smallest $n$ satisfying this condition. Given a periodic point $x$, its orbit $\mathcal{O}(x)$ is a {periodic orbit}. If $f(x) = x$, then $x$ is a {fixed point} of $f$. In the following we will consider a smooth surface $S$ (possibly with boundaries) with smooth area form $\omega$. A diffeomorphism $f: S\to S$ is said to be symplectic if it preserves the area form $\omega$. Let $p$ be a fixed point of the symplectic map $f$, $(U, \phi)$ be a local coordinate chart around $p$. Then the tangent map $D_pf : T_p S \to T_p S$ can be viewed as a $2\times 2$ matrix with determinant $1$. Let $\lambda(p,f)$ be an eigenvalue of the matrix $D_pf$ (the other one will be $\frac{1}{\lambda(p,f)}$, if exists). Even though the entries of $D_p f$ depend on the choice of the local coordinate system $(U, \phi)$, its eigenvalues and hence the trace $\Tr(D_p f)$, do not depend on such choices. The fixed point $p$ is said to be \begin{enumerate} \item parabolic if $\lambda(p,f) =\pm 1$ (or equally, $|\Tr(D_p f)| =2$); \item hyperbolic if $|\lambda(p,f)| \neq 1$ (or equally, $|\Tr(D_p f)| > 2$); \item elliptic if $|\lambda(p,f)|= 1$ and $\lambda(p,f)\neq \pm 1$ (or equally, $|\Tr(D_p f)| < 2$). \end{enumerate} \subsection{Euclidean billiards} Let $Q \subset \bR^2$ be a bounded and connected domain with (piecewise) smooth boundary $\gamma=\pa Q$. Consider a point mass that moves within $Q$ freely and makes elastic reflects off the boundary. The induced system is called a Euclidean dynamical billiard, and $Q$ is called the billiard table. For convenience, we will assume that $Q$ is strictly convex and $\gamma$ is smooth. The term Euclidean billiards is not standard. However we will use it repeatedly to distinguish it from a Minkowski billiard system that will be introduced later. All billiard systems are Euclidean for the remainder of this subsection. The phase space $\cM$ of the dynamical billiard on the table $Q$ consists of tangent vectors $(q,v)$, where $q\in \gamma$, and $v\in T_q Q$ is a unit vector pointing to the interior of $Q$. Let $s$ be the arc-length parameter of $\gamma$ oriented counterclockwise, and $\theta \in [0,\pi]$ be the angle between $v$ and $\dgs$. Then each point $(p,v)$ can be expressed in terms of a pair of new coordinates $(s,\theta)$. Let $\ell_\gamma$ be the length of $\gamma$. Under the assumption that $Q$ is strictly convex, this gives rise to a new coordinate system on the phase space: $\mathcal{M} = \{(s,\theta): 0 \leq s \leq \ell_\gamma,\ 0 \leq \theta \leq \pi \}/\sim$ with the identification of the endpoints $s=0$ and $s=\ell_{\gamma}$. That is, the phase space $\mathcal{M}$ can be viewed as a cylinder $\bR/\ell_{\gamma} \times [0,\pi]$. Let $(q_0,v_0) \in \mathcal{M}$ be the initial position and direction of the billiard ball. Let $q_1 \in \gamma$ be the first point of reflection, so that at $q_1$, the billiard changes direction from $v_0$ to $v_1$. Using their corresponding coordinates $(s_i, \theta_i)$, $i=0, 1$, we can thus define the billiard map $T:\mathcal{M}\rightarrow \mathcal{M}$, $(s_0, \theta_0) \mapsto (s_1, \theta_1)$. For strictly convex billiard tables (even in $\bR^n$) whose boundary is $C^k$ for $k \geq 2$, then the billiard map $T$ is of class $C^{k-1}$. In particular, if the boundary is smooth, then $T$ is as well. By defining the billiard map, we can convert our billiard model to a discrete dynamical system on the cylinder $\bR/\ell_{\gamma} \times [0,\pi]$. Let $L = L(s,s_1)$ denote the distance between two points $\gamma(s)$ and $\gamma(s_1)$, and let $\kappa(s)$ and $\kappa(s_1)$ be the signed curvatures of $\gamma$ at $s$ and $s_1$ respectively. \begin{proposition} \label{pro:DT.euclid} The billiard map $T$ preserves the area form $\omega = \sin\theta ds \wedge d\theta$ on the phase space $\cM$. Moreover, the differential of the billiard map $T: (s, \theta) \mapsto (s_1, \theta_1)$ is given by \begin{equation}\label{eq: DpT} D_{(s,\theta)}T = \frac{1}{\sin\theta_1} \begin{bmatrix} L\kappa(s)\kappa(s_1) & L \\ L\kappa(s)\kappa(s_1) - \kappa(s_1)\sin\theta - \kappa(s)\sin\theta_1 & \qquad L\kappa(s_1)-\sin\theta_1 \end{bmatrix}. \end{equation} \end{proposition} See \cite[Section 2.11]{ChMa} for more details. Our proof below is slightly different, which provides the setup that will be used again to derive the tangent map of Minkowski billiards, see Section~\ref{sec.su} and Proposition~\ref{pro.DT.Minkowski} for more details. \begin{proof} First note that $\sin\theta > 0$ so that $\omega$ is in fact an area form on $\cM$. Let $\gamma(s)$ and $\gamma(s_1)$ be distinct points on the boundary of the billiard table, and $L=L(s,s_1)$ be as above. Then $\frac{\partial L}{\partial s} = -\cos\theta$ and $\frac{\partial L}{\partial s_1} = \cos\theta_1$. It follows that \begin{align} dL &= \frac{\partial L}{\partial s}ds + \frac{\partial L}{\partial s_1}ds_1 = -\cos\theta ds + \cos\theta_1 ds_1, \label{eq.dLa}\\ d^2L &= \sin\theta \ d\theta \wedge ds - \sin\theta_1 \ d\theta_1 \wedge ds_1 = 0. \nonumber \end{align} This proves the first part of the theorem. To compute the differential $DT$, first note that since $\gamma$ is parameterized by arc-length, $\dgs \cdot \left(\gamma(s_1)-\gamma(s) \right) = L(s,s_1)\cos\theta$ and $\dgso \cdot \left(\gamma(s_1)-\gamma(s) \right) = L(s,s_1)\cos\theta_1$. Hence \begin{align} \cos\theta &= \frac{1}{L(s,s_1)}\left(\gamma(s_1)-\gamma(s)\right)\cdot\dgs, \label{eq: costheta} \\ \cos\theta_1 &= \frac{1}{L(s,s_1)}\left(\gamma(s_1)-\gamma(s)\right)\cdot\dgso. \label{eq: costheta1} \end{align} Taking the differential of equation (\ref{eq: costheta}) yields \begin{align*} -\sin\theta d\theta &= \bigg[-\frac{1}{L^2}\frac{\partial L}{\partial s} \left(\gamma(s_1)-\gamma(s)\right) \cdot \dgs + \frac{1}{L} \bigg( \left( \gamma(s_1)-\gamma(s) \right)\cdot \ddgs -\dgs \cdot \dgs \bigg) \bigg]ds \\[5pt] &\qquad + \bigg[ -\frac{1}{L^2}\frac{\partial L}{\partial s_1} \left(\gamma(s_1)-\gamma(s)\right) \cdot \dgs + \bigg]ds_1 \\[5pt] &= \left[\frac{\cos^2\theta-1}{L} + \kappa(s)\sin\theta \right]ds + \left[\frac{-\cos\theta\cos\theta_1}{L} + \frac{\cos(\theta + \theta_1)}{L}\right]ds_1 \\[5pt] &= \left[\frac{-\sin^2\theta}{L} + \kappa(s)\sin\theta \right]ds - \frac{\sin\theta\theta_1}{L}ds_1. \end{align*} Solving for $\sin\theta_1ds_1$ gives \begin{equation} \sin\theta_1ds_1 = \left[L \kappa(s)-\sin\theta\right]ds + L d\theta. \label{eq: sintheta1} \end{equation} It follows that \begin{align*} \frac{\partial s_1}{\partial s} &= \frac{L\kappa(s)-\sin\theta}{\sin\theta_1},\\[5pt] \frac{\partial s_1}{\partial \theta} &= \frac{L}{\sin\theta_1}. \end{align*} Similarly, taking the differential of equation (\ref{eq: costheta1}) yields \begin{align} -\sin\theta_1 d\theta_1 &= \left[\frac{\cos\theta\cos\theta_1}{L} - \frac{\cos(\theta+\theta_1)}{L}\right]ds + \left[\frac{1-\cos^2\theta_1}{L}-\kappa(s_1)\sin\theta_1 \right]ds_1. \end{align} Simplifying the equation, we have \begin{align*} \sin\theta_1d\theta_1 = -\frac{\sin\theta\sin\theta_1}{L}ds + \left[\kappa(s_1)-\frac{\sin\theta_1}{L}\right]\sin\theta_1ds_1. \end{align*} Plugging in equation (\ref{eq: sintheta1}) into the above equation, we get \begin{equation*} \sin\theta_1d\theta_1 = \bigg[L \kappa(s)\kappa(s_1)-\kappa(s_1)\sin\theta - \sin\theta_1\kappa(s)\bigg]ds + \bigg[\kappa(s_1)L-\sin\theta_1\bigg]d\theta. \end{equation*} It follows that \begin{align*} \frac{\partial \theta_1}{\partial s} &= \frac{1}{\sin\theta_1}\bigg[L\kappa(s)\kappa(s_1) - \kappa(s_1)\sin\theta - \kappa(s)\sin\theta_1 \bigg], \\[5pt] \frac{\partial \theta_1}{\partial \theta} &= \frac{1}{\sin\theta_1}\bigg[L\kappa(s_1)-\sin\theta_1\bigg]. \end{align*} Collecting terms, we complete the proof of the proposition. \end{proof} Here we consider another variable $u= - \cos\theta$, which gives rise to an equivalent coordinate system of the phase space $\cM$ via $\bR/\ell_{\gamma} \times [-1, 1]$. In particular, the corresponding area form $\omega$ on $\cM$ becomes the standard area: \begin{align} \omega = \sin\theta ds \wedge d\theta = ds \wedge du, \label{eq.Tsymp} \end{align} and Eq.~\eqref{eq.dLa} becomes \begin{align} dL &=\ -\cos\theta ds + \cos\theta_1 ds_1 = u ds - u_1 ds_1. \label{eq.dLu} \end{align} In this way, the function $L=L(s,s_1)$ is a generating function of the billiard map $T: (s, u)\mapsto (s_1, u_1)$ in the sense that \begin{align} u = \frac{\pa L}{\pa s}(s, s_1), \quad u_1 = -\frac{\pa L}{\pa s_1}(s, s_1). \label{eq.def.u} \end{align} This change is necessary when studying Minkowski billiards since the angles are no long defined. \subsection{Stability of periodic billiard orbits} Now that we have introduced some basic properties of dynamical billiards, it is only natural to ask whether periodic orbits exist, and if so study their properties. Despite its simplicity, this is a surprisingly deep question. For example, in 1775 Fagnano proved that when the billiard table is an acute triangle, there always exists a periodic orbit - namely the \textit{orthic triangle}. However, the case of an obtuse triangle has proven to be much more elusive. It wasn't until the last fifteen years that \cite{Sch09} proved the existence of periodic orbits when the billiard table is a triangle whose angles are all at most one hundred degrees. In contrast, Birkhoff \cite{Bi27} showed that every convex billiard table with a sufficiently smooth boundary has many periodic orbits. Given a periodic orbit a natural question to ask is: What if we vary the initial conditions (maybe ever so slightly)? Do we obtain a trajectory that closely resembles the periodic orbit, or do we get something vastly different? And if the result is vastly different, is it at least in some sense 'predictable', or does it appear to be totally random? So the first step is to classify the periodic orbit according to the eigenvalues of the tangent map iterated up to period, see Section \ref{sec.dysy}. It follows from the Hartman-Grobman theorem that hyperbolic fixed points have the stable and unstable manifolds on the phase space and are locally unstable. On the other hand, the geometric characterizations for general elliptic fixed points can be very different. Given an elliptic fixed point $x^*$, the linearization $D_{x^*}f$ acts as a rigid rotation around the fixed point. However, the local dynamics around an elliptic fixed point can be very rich for the underlying map $f$. This is in contrast to the hyperbolic case where $f$ and $D_pf$ always posses the similar dynamic behaviors. In this section we give sufficient conditions for when the elliptic islands are inherited by $f$. For more details, see the classic text \cite{SiMo}. \begin{definition} A fixed point $x^*$ of $f: U \subseteq \bR^2 \rightarrow \bR^2$ is said to be \textit{Moser stable} or \textit{nonlinearly stable} if there exists a nested sequence of $f$-invariant neighborhoods $U_n$, $n\ge 1$, containing $x^*$, whose boundaries $\partial U_n$ are invariant circles and $\bigcap\limits_{n\geq1} U_n = \{x^*\}$. \end{definition} The definition of nonlinearly stable elliptic periodic points of period $n$ can be done using the iterate $f^n$. Note that elliptic periodic points are not always nonlinearly stable and there are cases for which elliptic periodic points are not surrounded by any invariant circle. As an example we consider lemon billiards first introduced in \cite{He93}. Let $Q(L)$ denote the class lemon billiard tables formed by intersecting two unit circles where $\ell$ represents the distance between the two centers. Denote by $\mathcal{O}_2(L) = \{P, \cT(P)\}$ the periodic 2-orbit reflecting between $\gamma_0(0)$ and $\gamma_1(0)$ (See Figure \ref{fig.lemon}). It is shown in \cite{JZ22} that $\mathcal{O}_2(L)$ is elliptic for $L \in (0,2)$, and nonlinearly stable for $L \in (0,2) \backslash \{1\}$. However, numerical calculations in \cite{Ch13} appear to demonstrate that the billiard map is ergodic on $Q(1)$ so that in particular, $\mathcal{O}_2(1)$ is not nonlinearly stable. \begin{figure}[htbp] \tikzmath{ \r=2; \a=sqrt(3); } \begin{tikzpicture} \draw[domain=-60:60, thick , samples=100] plot ({\r*cos(\x)}, {\r*sin(\x)}); \draw[domain=120:240, thick, samples=100] plot ({\r+\r*cos(\x)}, {\r*sin(\x)}); ll (0,0) node[left]{$\gamma_{0}$} circle[radius=0.05]; ll (2,0) node[right]{$\gamma_{1}$} circle[radius=0.05]; \draw[thick] (0,0) -- (2, 0); \end{tikzpicture} \caption{The lemon table $Q(1)$.}\label{fig.lemon} \end{figure} \subsection{Birkhoff normal form and Moser's twist mapping theorem}\label{sec.BNF} Let $f: U\to \bR^2$ be a symplectic embedding and $P=(0,0)$ be an elliptic fixed point of $f$. That is, the eigenvalues of the tangent matrix $D_P f$ satisfies $|\lambda| =1$ and $\lambda \neq \pm 1$. Then the point $P$ is said to be non-resonant if $\lambda^n \neq 1$ for any $n\ge 3$. For any $N\ge 1$, there exists an area preserving change of coordinates of the form \begin{align} h_N: U \rightarrow \bR^2, \quad \begin{bmatrix} x \\ y \end{bmatrix} \mapsto \begin{bmatrix} x + p_2(x,y) + \cdots + p_{2N+1}(x,y) \\ y + q_2(x,y) + \cdots + q_{2N+1}(x,y) \end{bmatrix} + \text{h.o.t} \label{eq: Birkhoff transformation} \end{align} where $p_j$ and $q_j$ are $j^\text{th}$ degree polynomials for every $j=2,3,...,2N+1$, under which one can express \begin{equation} h_N^{-1}\circ f \circ h_N \left( \begin{bmatrix} x \\ y \end{bmatrix} \right) = \begin{bmatrix} \cos\Theta(r^2) & -\sin\Theta(r^2) \\ \sin\Theta(r^2) & \cos\Theta(r^2) \end{bmatrix} \begin{bmatrix} x \\ y \end{bmatrix} + \text{h.o.t.} \label{eq: BNF} \end{equation} where $r^2 = x^2 + y^2$, \begin{equation} \Theta(r^2) = \theta + \tau_1 r^2 + \tau_2 r^4 + \cdots + \tau_N r^N, \label{eq: Theta} \end{equation} and h.o.t stands for "higher order terms". \label{prop: BNF} The symplectic transformation (\ref{eq: Birkhoff transformation}) is called the $N$-th order \textit{Birkhoff transformation}, (\ref{eq: BNF}) is called the $N$-th order \textit{Birkhoff normal form} of the map $f$ around the elliptic fixed point $P$, and the coefficient $\tau_j$ is called the \textit{$j$-{th} twist coefficient} (or Birkhoff coefficient) of $f$ at $P$. The cases such that $\lambda^n = 1$ for some $n\ge 3$ are called \textit{resonances}. The name "twist coefficient" is given because geometrically, $\Theta(r^2)$ defined in (\ref{eq: Theta}) measures the amount of rotation about the fixed point. The same results hold for elliptic periodic orbits. See \cite{SiMo, Mos73} for more details. Moreover, Moeckel \cite{Moe90} studied the behavior of the first twist coefficient near an elliptic fixed point for a one-parameter family of area-preserving diffeomorphisms and gave a method for calculating $\tau_1$. These twist coefficients play an important role when determining the nonlinear stability of the elliptic fixed points. \begin{theorem}[Moser's twist mapping theorem]\label{thm: Moser twist} If for an area-preserving map of the form (\ref{eq: BNF}), the polynomial $\Theta(r^2)$ defined in (\ref{eq: Theta}) is not identically zero, then the elliptic fixed point $P$ is nonlinearly stable. \end{theorem} There have been many applications of Moser's Twist Mapping Theorem in the study of dynamical billiards, see \cite{KP01, DKP03, BuGr, XiZh14} and the references therein. In \cite{KP05} the authors obtained an explicit formula of the first twist coefficients of elliptic $2$-periodic orbits in terms of the geometric characterizations of the billiard table. In \cite{JZ22} the authors studied the Birkhoff normal form around elliptic periodic points for a large class of billiards and used it to give explicit formulas for the first two twist coefficients in terms of the geometric parameters of the billiard table. The authors were then able to apply the formulas of the twist coefficients to obtain characterizations of the nonlinear stability and local analytic integrability of billiards around elliptic periodic points. \section{Minkowski Billiards} Minkowski billiards and Finsler billiards were introduced by Gutkin and Tabachnikov in \cite{GT02} as a natural generalization of Euclidean billiards that use a Finsler structure to represent a billiard traveling on an anisotropic and inhomogeneous medium. On Euclidean billiard systems, a billiard travels with uniform motion at every point on the interior of the billiard table -- a consequence of the indicatrix of the Euclidean metric being a unit circle at each point. However, the indicatrix of a Finsler structure is not necessarily round or symmetric, but only a strictly convex curve that may vary from point to point. Hence the motion of the billiard depends on its location and direction. In the remainder of this section we will restate the reflection law for Minkowski billiard systems given the lack of angles in Minkowski geometry, and prove that the Minkowski billiard map is symplectic under an appropriate set of coordinates. \subsection{Reflection laws in Minkowski spaces}\label{ss.reflectionM} In Euclidean billiard systems, the reflection law is that the angle of incidence equals the angle of reflection. However, because Minkowski structures are not induced by an inner product, Minkowski geometry lacks the notion of angles. Hence to work with billiards in the Minkowski setting, a reflection law must be specified. Let $(\bR^2, F)$ be a Minkowski space, $Q\subset \bR^2$ be a bounded and strictly convex domain with (piecewise) smooth boundary $\partial Q$. As usual, $Q$ will be called the billiard table and $\pa Q$ will be parameterized by a piece-wise smooth curve $\gamma(s)$ (counterclockwise) with $F(\dot\gamma(s)) =1$ when it exists. Further, let $y_0$ and $y_1$ be two points in the interior of $Q$ and $x$ be a point on the boundary of $Q$. Then the Minkowski distances from $y_0$ to $x$ and from $x$ to $y_1$ are given as $F(x-y_0)$ and $F(y_1-x)$ respectively. Then $xy_1$ is the reflected ray of $y_0x$ if and only if $x$ is a critical point of the distance function $F(x-y_0) + F(y_1-x)$. In \cite{GT02} Gutkin and Tabachnikov established the reflection law for Finsler billiards using the geometry of the indicatrix as follows: \begin{proposition}[Finsler reflection law]\label{pro.Finsler.reflection.law} Let $y_0,y_1 \in Q$, and $x \in \pa Q$ be as above so that $xy_1$ is the billiard reflection of $y_0x$, and let $u,v \in I_x$ be the $F$-unit vectors traveling along $y_0x$ and $xy_1$ respectively. Then $\mathcal{L}(v)-\mathcal{L}(u)$ is conormal to $T_x \partial Q$ i.e. $\left(\mathcal{L}(v)-\mathcal{L}(u)\right)(w) = 0 $ for $w \in T_x \partial Q$. \end{proposition} Given an incident billiard trajectory, we can use Proposition \ref{pro.Finsler.reflection.law} to find the reflected trajectory in the following manner: \begin{enumerate} \item $T_uI_x$ is not parallel to $T_x\partial Q$: the line $T_uI_x$ intersects $T_x\partial Q$ at a unique point, say $w \in T_x\partial Q$. The Finsler reflection law states that the reflected vector $v \in T_xI$ satisfies $\mathcal{L}(u)(w) = \mathcal{L}(v)(w) = 1$. Therefore, so $w \in T_vI_x$. This determines $v$, see Fig.~\ref{fig.F.reflection}. \item $T_uI_x$ is parallel to $T_x\partial Q$: $\mathcal{L}(u)$ is proportional to a covector that is conormal to $T_x\partial Q$. The Finsler reflection law states that $\mathcal{L}(v)$, where $v$ is the reflected ray, is also proportional to $\mathcal{L}(u)$. Hence $T_vI_x$ is parallel to $T_uI_x$ thus determining the reflected vector $v$. \end{enumerate} It is important to note that Finsler billiards are generally irreversible, i.e. given an incident and reflected billiard trajectory, we cannot reverse their roles unless the norm $F$ reversible: $F(x,-\tilde{v}) = F(x,\tilde{v})$ for $\tilde{v} \in T_xM$. It is clear that the reflection is that of equal angles when the indicatrix is a circle centered at the origin. \begin{figure}[htbp] \tikzmath{ \a=18; \b=155; } \begin{tikzpicture}[scale=1.5] \draw [domain=0:360, samples=100] plot ({2*cos(\x) + 1}, {0.5*sin(\x)}) node[yshift=-0.2in]{$I_x$}; \draw [thick, domain=-0.6:0.7, samples=100] plot ({\x}, {\x-\x*\x}) node[xshift=-0.6in,yshift=-0.6in]{$\pa Q$}; \draw (-1,-1) -- (2,2); \draw ({2*cos(\a) + 1}, {0.5*sin(\a)}) -- (1.5,1.5) node[pos=0.5, above right]{$T_v I_x$} node(w){} -- ({2*cos(\b) + 1}, {0.5*sin(\b)}) node[pos=0.5, above left]{$T_u I_x$}; \draw ({2*cos(\a) + 1}, {0.5*sin(\a)}) node(v){} -- (0,0) node[pos=0.03, sloped]{$>$} node[pos=0.5](A){}; \draw[dashed] ({2*cos(\b) + 1}, {0.5*sin(\b)}) node(u){} -- (0,0) node[pos=0.1, sloped]{$<$} node[pos=3](B){}; \draw[thick] (0,0) -- (A) node[pos=0.5, sloped]{$>$}; \draw[thick] (0,0) -- (B) node[pos=0.5, sloped]{$<$}; ll (0,0) circle(0.06) node[below]{$x$}; ll (u) circle(0.05) node[left]{$u$}; ll (v) circle(0.05) node[right]{$v$}; ll (w) circle(0.05) node[left]{$w$}; \end{tikzpicture} \caption{An illustration of the Finsler reflection law.}\label{fig.F.reflection} \end{figure} \subsection{New coordinate system on $\cM$ for Minkowski billiards}\label{sec.su} Let $(\bR^2, F)$ be a Minkowski plane, $Q\subset \bR^2$ be a strictly convex domain with (piecewise) smooth boundary. The phase space $\mathcal{M}$ of the Minkowski billiard on $Q$ is given by all points $(x, v)$ where $x \in \partial Q$, and $v \in I_x$ points to the interior of $Q$, as we have had for Euclidean billiards. The Minkowski billiard map $\mathcal{T}$ is defined using the new reflection law in Proposition \ref{pro.Finsler.reflection.law}. That is, let $x_0 = \gamma(s_0)$ and $x_1 = \gamma(s_1)$ be points on $\partial Q$, $v_0 \in I_{x_0}$ be the $F$-unit vector at $x_0$ pointing to $x_1$, and $v_1 \in I_{x_1}$ the $F$-unit vector pointing in the direction of the reflected billiard trajectory. Then the Minkowski billiard map is $\mathcal{T}: \mathcal{M} \rightarrow \mathcal{M}$, $(s_0,v_0) \mapsto (s_1,v_1)$. Next we introduce a new variable $u=u(s, s_1)$ and a new coordinate system $(s,u)$ on $\cM$ for the billiard map $\cT$ using the generating function method mimicking what has been done in Eq.~\eqref{eq.def.u}. \begin{definition}\label{def.new.variable} Let $Q \subset (\bR^2, F)$ be a compact and convex domain with piecewise smooth boundary $\pa Q$, $\gamma(s)$ be a parametrization of $\pa Q$ with the $F$-arc-length parameter $s$ in the sense that $F(\dot \gamma(s)) =1$, $L(s,s_1) = F(\gamma(s_1)-\gamma(s))$ the Minkowski length from $\gamma(s)$ to $\gamma(s_1)$. Then the following defines a new variable \begin{align} u = u(s, s_1) := \frac{\partial L}{\partial s}(s,s_1). \label{def.u} \end{align} \end{definition} It is easy to see that this introduces a new coordinate system $(s,u)$ on $\cM$. Note that the lower and upper bounds of $u$ depend on the first coordinate $s$, and hence the phase space $\cM$ becomes a cylinder with varying lower and upper boundaries. Applying the above definition to the next iterate of the billiard orbit, we get that $u_1 =u(s_1,s_2)$. Then the Minkowski billiard map can be rewritten using the new coordinate as $\mathcal{T}: (s,u) \mapsto (s_1,u_1)$. \begin{remark} It is worth pointing out that the second identity in Eq.~\eqref{eq.def.u} should not be interpreted as the definition of the coordinate $u_1$. Rather, it represents a property of the Euclidean billiard map. We will verify that the same identity does hold for Minkowski billiard maps, see \eqref{eq.verify.u1} in the proof of Proposition \ref{pro.DT.Minkowski}. \end{remark} \subsection{The tangent map $D\cT$} With these new coordinates we can derive the tangent maps for Minkowski billiards as we have done in Proposition \ref{pro:DT.euclid} for Euclidean billiards. \begin{proposition} \label{pro.DT.Minkowski} The Minkowski billiard map $\mathcal{T}$ preserves the area form $\omega = ds \wedge du$. Moreover, the tangent map $D_{(s,u)}\mathcal{T} = \begin{bmatrix} \frac{\partial s_1}{\partial s} & \frac{\partial s_1}{\partial u} \\ \frac{\partial u_1}{\partial s} & \frac{\partial u_1}{\partial u}, \end{bmatrix}$ with entires given by \begin{align} \dsds &= \frac{\sum\limits_{i,j=1}^2 \Fij\dgis\dgjs - \sum\limits_{i=1}^2\Fi\ddgis}{\sum\limits_{i,j=1}^2\Fij\dgis\dgjss}, \label{eq: dsds} \\[10pt] \dsdu &= \frac{-1}{\sum\limits_{i,j=1}^2\Fij\dgis\dgjss}, \label{eq: dsdu} \\[10pt] \duds &= \sum\limits_{i,j}^2\Fij \dgiss \dgjs - \bigg(\sum\limits_{i,j=1}^2\Fij \dgiss \dgjss \nonumber \\[10pt] & \qquad + \sum\limits_{i=1}^2 \Fi \ddgiss \bigg) \left( \frac{\sum\limits_{i,j=1}^2 \Fij\dgis\dgjs - \sum\limits_{i=1}^2\Fi\ddgis}{\sum\limits_{i,j=1}^2\Fij\dgis\dgjss} \right), \label{eq: duds} \\[10pt] \dudu &= \frac{\sum\limits_{i,j=1}^2\Fij \dgiss \dgjss + \sum\limits_{i=1}^2\Fi \ddgiss}{\sum\limits_{i,j=1}^{2}\Fij \dgis \dgjss}. \label{eq: dudu} \end{align} \end{proposition} \begin{proof} We proceed in much the same as in Proposition \ref{pro:DT.euclid}. Let $\gamma(s),\gamma(s_1)$, and $\gamma(s_2)$ be points on the boundary $\pa Q$ such the the segment $\gamma(s_1)\gamma(s_2)$ is the reflected billiard trajectory of the billiard trajectory $\gamma(s)\gamma(s_1)$. Recall the distance function $L(s,s_1) = F(\gamma(s_1)-\gamma(s))$ and the variable $u =u(s,s_1) = \frac{\partial L}{\partial s}(s,s_1)$ as above. Moreover, $u_1 = u(s_1,s_2)$. By the billiard reflection law, $s_1$ is a critical point of the combined distance $L(s,s_1) + L(s_1,s_2)$. Differentiating $s_1$, we get \begin{align*} \frac{\partial}{\partial s_1}\left[ L(s,s_1) + L(s_1,s_2) \right] =\frac{\partial L}{\partial s_1}(s,s_1)+ u(s_1,s_2) =\frac{\partial L}{\partial s_1}(s,s_1)+ u_1= 0. \end{align*} Therefore, \begin{align} u_1 = -\frac{\partial L}{\partial s_1}(s,s_1). \label{eq.verify.u1} \end{align} This establishes the second identity in \eqref{eq.def.u} for Minkowski billiards. Applying the newly obtained identity \eqref{eq.verify.u1}, we can rewrite the total differential of $L(s, s_1)$ as \begin{equation} dL(s,s_1) = \frac{\partial L}{\partial s}(s,s_1) ds + \frac{\partial L}{\partial s_1}(s,s_1) ds_1 = u \, ds - u_1 \, ds_1. \label{eq.dL.Minkowski} \end{equation} Taking exterior differential of \eqref{eq.dL.Minkowski} again, we get $ddL = du \wedge ds - du_1 \wedge ds_1 = 0$, thus proving the first part of the proposition. Next we will derive the differential of the Minkowski billiard map $\cT: (s,u) \mapsto (s_1, u_1)$. To shorten our expressions, we will use $v = \gamma(s_1)-\gamma(s)$. Then $L(s,s_1) =F(v)$ and \begin{align} u &= \frac{\partial L}{\partial s}(s,s_1) = - \sum\limits_{j=1}^2 F_{v_j}(v)\dgis, \\ u_1 &= -\frac{\partial L}{\partial s_1}(s,s_1) = -\sum\limits_{j=1}^2 F_{v_j}(v)\dgiss, \end{align} where we have applied \eqref{eq.verify.u1} in the second equation. Taking the differentials of $u$ and $u_1$ yield \begin{align} du &= \left[ \sum\limits_{j,k =1}^2 F_{v_jv_k}(v)\dgis\dgjs - \sum\limits_{j=1}^2 F_{v_j}(v)\ddgis \right]ds \nonumber \\[10pt] &\qquad - \sum\limits_{j,k=1}^2 F_{v_jv_k}(v)\dgis\dgjss \ ds_1 \label{eq: du} \\[10pt] du_1 &= \sum\limits_{j=1}^2 F_{v_jv_k}(v) \dgiss \dgjs \ ds - \bigg[ \sum\limits_{j,k=1}^2 F_{v_jv_k}(v) \dgiss \dgjss \nonumber \\[10pt] &\qquad + \sum\limits_{j=1}^2 F_{v_j}(v) \ddgiss \bigg]ds_1. \label{eq: du1} \end{align} Solving for $ds_1$ in Eq.~(\ref{eq: du}) gives \begin{align} ds_1 &= \frac{\sum\limits_{i,j=1}^2 \Fij\dgis\dgjs - \sum\limits_{i=1}^2\Fi\ddgis}{\sum\limits_{i,j=1}^2\Fij\dgis\dgjss} \ ds - \frac{du}{\sum\limits_{i,j=1}^2\Fij\dgis\dgjss} \label{eq: ds1}. \end{align} Substituting Eq.~\eqref{eq: ds1} into Eq.~\eqref{eq: du1} and collecting $ds$ and $du$ terms yield \begin{align} du_1 &= \left[ \sum\limits_{i,j}^2\Fij \dgiss \dgjs - \bigg(\sum\limits_{i,j=1}^2\Fij \dgiss \dgjss \right. \nonumber \\[10pt] & \qquad \left. + \sum\limits_{i=1}^2 \Fi \ddgiss \bigg) \left( \frac{\sum\limits_{i,j=1}^2 \Fij\dgis\dgjs - \sum\limits_{i=1}^2\Fi\ddgis}{\sum\limits_{i,j=1}^2\Fij\dgis\dgjss} \right) \right] \ ds \nonumber \\[10pt] & \qquad + \frac{\sum\limits_{i,j=1}^2\Fij \dgiss \dgjss + \sum\limits_{i=1}^2\Fi \ddgiss}{\sum\limits_{i,j=1}^{2}\Fij \dgis \dgjss}du \label{eq: du1 second}. \end{align} Then the entries of the tangent map $D\cT$ follows from Eq.~\eqref{eq: ds1} and Eq.~\eqref{eq: du1 second}. \end{proof} \section{Billiards in Mixed Norm Spaces}\label{sec.billiards.symmetry} In this section we will study the properties of periodic 2-orbits of Minkowski billiards on the Minkowski plane $(\bR^2,F)$, where the Minkowski norm $F(v) = F_{a,b}(v) = a\lVert v \rVert_2 + b\lVert v \rVert_4$ with $a > 0$, $b \geq 0$, and $a+b=1$. We have showed that $(\bR^2,F)$ is a Minkowski space in Proposition \ref{prop: Mixed norm is Minkowski}, and in fact is a normed space so that the indicatrix $I$ given by the level curve $F(v) = 1$ is symmetric about the origin. The assumption $a+b=1$ not only makes it so that the calculations for the twist coefficient $\tau_1$ are easier to manage, but make it so that the billiard has identical motion to a Euclidean billiard system along the horizontal and vertical axes (see Figure \ref{fig: Mixed norm indicatrix}). \begin{figure}[htbp] \centering \subcaptionbox{$a = 0.7$ and $b = 0.3$}{\includegraphics[width=0.32\textwidth]{Images/mixed_norm_0.7}} \hfill \subcaptionbox{$a = 0.4$ and $b = 0.6$}{\includegraphics[width=0.32\textwidth]{Images/mixed_norm_0.4}} \hfill \subcaptionbox{$a = 0.1$ and $b = 0.9$}{\includegraphics[width=0.32\textwidth]{Images/mixed_norm_0.1}} \caption{Indicatricies of Minkowski spaces with the mixed norm $F_{a,b}$.} \label{fig: Mixed norm indicatrix} \end{figure} Let $\alpha(t) = \sum\limits_{n \geq 1} a_{2n}t^{2n}$ and $\beta(t) = \sum\limits_{n\geq1}b_{2n}t^{2n}$, $t \in (-\epsilon, \epsilon)$, be two even functions for some $\epsilon > 0$, and $\gamma_0: t\mapsto (\alpha(t),t)$ and $\gamma_1: t\mapsto (L-\beta(t),-t)$, $t \in (-\epsilon, \epsilon)$, be two curves on $\bR^2$. The class of billiard tables $Q(L,\alpha,\beta)$ we will consider are those whose boundary $\partial Q$ is given by a piece-wise smooth curve $\gamma$ consisting of two horizontal line segments connecting the curves $\gamma_0$ and $\gamma_1$ at their endpoints (see Figure \ref{fig.btable}). Rather than parameterizing $\gamma$ with a global Minkowski arc-length parameter, we will use a local arc-length parameter $s$ on each of the curves $\gamma_j$ such that $F(\gamma_j'(s)) \equiv 1$, $s(\gamma_j(0)) = 0$ for $j = 0,1$. It is clear from the Finsler reflection law that there exists a periodic 2-orbit $\mathcal{O}_2$, bouncing back and forth between $(0,0)$ and $(L,0)$ where $L$ is the Euclidean length between the two reflection points. \begin{figure}[htbp] \begin{tikzpicture} \draw[->] (-1,0) -- (5.5,0); \draw[->] (0,-1.5) -- (0,1.5); \draw [domain=-1:1, samples=100, thick] plot ({4-\x*\x +0.3*\x*\x*\x*\x }, {\x}); \node at (3.9, 0.7) {$\gamma_1$}; \draw [domain=-1:1, samples=100, thick] plot ({\x*\x +0.5*\x*\x*\x*\x }, {\x}); \node at (0.3,0.8) {$\gamma_0$}; \draw[thick, ->] (0,0) node[below left]{$0$} -- (1,0) node[below]{$p$}; \draw[thick, ->] (4,0) node[below right]{$L$} -- (3,0) node[below]{$\cT(p)$}; \draw[thick] (3.3, 1) -- (1.5, 1) (3.3, -1) -- (1.5, -1); ll (0,0) circle[radius=0.06]; ll (4,0) circle[radius=0.06]; \end{tikzpicture} \caption{An illustration of the billiard table.} \label{fig.btable} \end{figure} The phase space $\mathcal{M}$ is the set of unit vectors with foot on $\partial Q$ that point inside $Q$. Recall that the new coordinates $(s,u)$ on $\mathcal{M}$ introduced in Section~\ref{sec.su}, where $u = u(s, s_1) = \frac{\partial L}{\partial s}(s,s_1)$, see Definition~\ref{def.new.variable}. Therefore the billiard map $\mathcal{T}: \mathcal{M} \rightarrow \mathcal{M}$ is given by $\mathcal{T}: (s,u) \mapsto (s_1,u_1)$, where $u_1 = u(s_1,s_2)$. For example, the periodic 2-orbit in Figure \ref{fig.btable} is given by $\mathcal{O}_2 = \{p,\mathcal{T}(p)\}$ where $p = (0,0)$ and $\mathcal{T}(p) = (0,0)$ with respect to the corresponding local coordinate systems of the phase space on the two curved parts. \subsection{Symmetric Minkowski billiards}\label{ss.sym.billiards} In Section~\ref{sec.BNF} we have stated how Moser's Twist Mapping Theorem (Theorem \ref{thm: Moser twist}) utilizes the twist coefficients $\tau_j$ in the Birkhoff normal form to obtain the nonlinear stability of elliptic periodic billiard orbits. We now construct Birkhoff transformations and give an explicit formula for the first twist coefficient in terms of the geometric properties of the Minkowski billiards. We will then use the first twist coefficient to investigate the nonlinear stability property of periodic orbits of Minkowski billiards. We will start with the case $\alpha(t) = \beta(t)$, so that the billiard table $Q(L, \alpha, \alpha)$ is symmetric about the line $x = L/2$. Combining with the symmetry of the Minkowski norm $F_{a,b}$, we get that the radius of curvature functions $R_j(s)$ on the two parts $\gamma_j$ are identical, so we set $R(s) = R_0(s) = R_1(s)$. Moreover, let $\text{Rot}_\pi$ be the rotation of the table $Q(L,\alpha,\alpha)$ about the point $(\frac{L}{2},0)$ by $\pi$. Since $F(v) = F(-v)$ then $\text{Rot}_\pi$ identifies the phase space $\mathcal{M}_1$ of $\gamma_1$ with the phase space $\mathcal{M}_0$ of $\gamma_0$.\footnote{This is false for general Minkowski norms that don't satisfy $F(-v) = F(v)$.} \subsection{Reducing the two-step map using the symmetry}\label{ss.reducing.one.step} Pick a small neighborhood $U_0 \subset \mathcal{M}_0$ of $P\in \cM_0$. Then $U_1 = \text{Rot}_\pi(U_0) \subset \mathcal{M}_1$ is a small neighborhood of $\mathcal{T}(P) \in \cM_1$. Let $\cT_i = \cT|_{U_i}: U_i \to \cM_{1-i}$ be the restriction of the billiard map $\cT$ to $U_i$, $i=0,1$. Observe that $ \text{Rot}_\pi\circ\cT_0 = \cT_1\circ\text{Rot}_\pi$. It follows that, restricting on $U_0$, \begin{align} \cT^2 = \cT_1\circ \cT_0 = \text{Rot}_\pi\circ\cT_0 \circ \text{Rot}_\pi\circ\cT_0 =(\text{Rot}_\pi\circ\cT_0)^2. \end{align} If follows that the twist coefficient $\tau_1(\cT^2, P)= 2\tau_1(\text{Rot}_\pi\circ\cT_0, P)$. Identifying $\cM_1$ with $\cM_0$ using $\text{Rot}_\pi$, we can identify the one-step billiard map $\cT_0$ with the abstract self-map $\text{Rot}_\pi\circ\cT_0: U_0\to \cM_0$. In the following we will abuse our notation and use $\cT$ for the abstract map $\text{Rot}_\pi\circ\cT_0$. In particular, $\tau_1(\cT^2, P)= 2\tau_1(\cT, P)$. \subsection{Tangent matrix of the billiard map}\label{ss.tangent.map} We start with computing the tangent matrix of the billiard map $\mathcal{T}$. The first and second order partial derivatives of the Minkowski norm $F(v)=a \|v\|_2 + b\|v\|_4$ are given by \begin{equation*} \begin{aligned}[c] F_{v^1} &= \frac{av_1}{\sqrt{v_1^2 + v_2^2}} + \frac{bv_1}{\left( v_1^4 + v_2^4 \right)^{\frac{3}{4}}}\\[10pt] F_{v^2} &= \frac{av_2}{\sqrt{v_1^2 + v_2^2}} + \frac{bv_2}{\left( v_1^4 + v_2^4 \right)^{\frac{3}{4}}} \end{aligned} \qquad\qquad \begin{aligned}[c] F_{v^1v^1} &= \frac{av_2^2}{\left(v_1^2 + v_2^2 \right)^{\frac{3}{2}}} + \frac{3bv_1^2v_2^4}{\left( v_1^4 + v_2^4 \right)^{\frac{7}{4}}} \\[10pt] F_{v^1v^2} &= -\frac{av_1v_2}{\left(v_1^2 + v_2^2\right)^{\frac{3}{2}}} - \frac{3bv_1^3v_2^3}{\left(v_1^4 + v_2^4 \right)} \\[10pt] F_{v^2v^2} &= \frac{av_1^2}{\left(v_1^2 + v_2^2 \right)^{\frac{3}{2}}} + \frac{3bv_1^4v_2^2}{\left( v_1^4 + v_2^4 \right)^{\frac{7}{4}}}. \end{aligned} \end{equation*} Note that the tangent vector at $(0,0)$ and $(L,0)$ are given by $\gamma'_0(0) = \langle 0, -1 \rangle$ and $\gamma'_1(0) = \langle 0, 1 \rangle$ respectively. Recall $\gamma''(s) = \kappa_m(s)n_\gamma(s)$ where $\kappa_m$ and $n_\gamma$ can be written in terms of a parameterization $r(\theta(s))$ of the indicatrix as shown in equations (\ref{eq: km in Euclid}) and (\ref{eq: n in Euclid}). Given the norm $F(v) = a\lVert v \rVert_2 + b\lVert v \rVert_4$ for $a>0$ and $b \geq 0$, the parameterization of $\gamma_0$ is given by \begin{equation} r(\theta(s)) = \frac{1}{a + b \left(\cos^4\theta(s) + \sin^4\theta(s) \right)^{\frac{1}{4}}}. \end{equation} Moreover, from equation (\ref{eq: n in Euclid}) we see that $\kappa_m$ may be written in terms of the Euclidean radius of curvature since $\kappa_e(s) = \frac{1}{R(s)}$. Letting $R(0) = R$ and $a+b = 1$, Eq.~\eqref{eq: dsds}--\eqref{eq: dudu} at $P = (0,0)$ simplify to: \begin{align} \dsds(0,0) &= \frac{L}{aR}-1, \label{eq: dsds mixed norm} \\[10pt] \dsdu(0,0) &= \frac{L}{a}, \label{eq: dsdu mixed norm} \\[10pt] \duds(0,0) &= \frac{L-2aR}{aR^2}, \label{eq: duds mixed norm} \\[10pt] \dudu(0,0) &= \frac{L}{aR}-1. \label{eq: dudu mixed norm} \end{align} The tangent matrix of the one-step billiard map $\mathcal{T}$ at $P$ is thus given by \begin{equation} D_p\mathcal{T} = \begin{bmatrix} \ta_{10} & \ta_{01} \\ \tb_{10} & \ta_{01} \end{bmatrix} := \begin{bmatrix} \frac{L}{aR}-1 & \frac{L}{a} \\[10pt] \frac{L-2aR}{aR^2} & \frac{L}{aR}-1 \end{bmatrix}. \label{eq: DpT mixed norm} \end{equation} Note that in the Euclidean case where the indicatrix is a unit circle ($a = 1$, $b = 0$), Eq.~\eqref{eq: DpT mixed norm} reduces to the more familiar tangent matrix $D_P\mathcal{T} = \begin{bmatrix} \frac{L}{R} -1 & L \\ \frac{L}{R^2} - \frac{2}{R} & \frac{L}{R} -1 \end{bmatrix}$, see \cite[Eq.~(1.3)]{JZ22}. Because $\mathcal{T}$ is symplectic, the eigenvalues $\lambda$ of $D_p\mathcal{T}$ satisfy $\lambda^2 - \Tr(D_P\mathcal{T})\lambda + 1 = 0$, where $\Tr(D_P\mathcal{T}) = \frac{2L}{aR} - 2$. It follows that $D_P\cT$ is parabolic if $L = 2aR$, hyperbolic if $L > 2aR$, and elliptic if $ 0 < L < 2aR$. Going forward, we will restrict our discussion to the elliptic case. Then the eigenvalues of the tangent mattrix $D_P\cT$ are given by $\lambda = \ta_{10} - i\sqrt{-\ta_{01}\tb_{10}}$ and $\overline{\lambda} = \frac{1}{\lambda}$, see Eq.~\eqref{eq: DpT mixed norm}. The following non-resonance condition is needed to find the Birkhoff normal form and the first twist coefficient: \begin{itemize} \item [(A)] $\lambda^4 \neq 1 \text{ or equivalently, } L \in \left(0, 2aR \right) \backslash \bigl \{aR \bigl \}$. \end{itemize} \begin{theorem}\label{thm.t1.sym} Let $0< a \le 1$, $a + b=1$, $(\bR^2, F)$ be the Minkowski plane with the mixed norm $F=F_{a,b}$, $Q(L,\alpha, \beta)$ be the symmetric Minkowski billiard table, $R_0 = R_1 = R$ be the curvature at $t=0$ (the vertices). Suppose the nonresonance condition (A) is satisfied. Then the first twist coefficient of the one-step billiard map $\mathcal{T}:(s,u) \mapsto (s_1, u_1)$ at the elliptic periodic point $P$ is given by \begin{equation} \tau_1(\cT, P) = \frac{a^2 L R R''+(4-3 a) a L+2 (2 a-9) a R+12 R}{8 a^2 R (L-2 a R)} \label{eq: symmetric twist} \end{equation} \end{theorem} \begin{remark} As done in \cite{JZ22}, the functions $\alpha(t)$ and $\beta(t)$ are assumed to be even polynomials in order to simplify the computation of $\tau_1$. Without these assumptions, the expression for $\tau_1$ would be significantly more complex as it would also depend on $R'$. The proof of Theorem \ref{thm.t1.sym} is given over the subsequent sections and follows closely the approaches given in \cite{JZ22, Moe90}. Plugging in $a=1$ and $b=0$, we recover the first twist coefficient for the periodic orbit of period $2$ for Euclidean billiards: $\ds \tau_1 = \frac{1}{8}\left(\frac{1}{R}-\frac{LR''}{2R-L}\right)$, see also \cite{KP05, JZ22}. \end{remark} \subsection{Taylor polynomials of the billiard map}\label{ss.Taylor.coefficients} To begin, we first compute the Taylor expansion of the billiard map $\mathcal{T}(s,u) = (s_1,u_1)$ at $P = (0,0)$. We have computed the first order partial derivatives of $s_1$ and $u_1$ at $P$ which are given in equations \eqref{eq: dsds mixed norm}--\eqref{eq: dudu mixed norm}. To compute the higher order derivatives, we can apply the chain rule to equations \eqref{eq: dsds}--\eqref{eq: dudu}. More details for this calculation can be found in \cite{Vil}. Note that $R(s)$ has critical points at $\gamma_0(0)$ and $\gamma_1(0)$. Thus $R'(0) = 0$, and because of this, further calculations show that $\frac{\partial^{j+k} s_1}{\partial s^j \partial u^k}(0,0) = \frac{\partial^{j+k} u_1}{\partial s^j \partial u^k}(0,0) = 0$ for $j+k = 2$. Denote $R^{(j)}(0) = R^{(j)}$, we list the third order partial derivatives of $s_1$ and $u_1$ needed for the calculation of $\tau_1$: \begin{flalign} \frac{\partial^3 s_1}{\partial s^3}(0,0) &= \frac{1}{a^4R^5} \bigg[2a^4LR^2-3a^3R \left(L^2-LR+2R^2\right)-a^3LR^3R'' \nonumber \\ &\qquad + a^2 \left(L^3+3 L R^2\right)-3 a L R (L-3 R)-6 L R^2 \bigg], \end{flalign} \begin{flalign} \frac{\partial^3 s_1}{\partial s^2 \partial u} (0,0) &= \frac{1}{a^4R^4} \bigg[ 2a^4LR^2-2a^3R \left(L^2+R^2\right)+a^2 \left(L^3+2LR^2\right) \nonumber \\ &\qquad -3aLR(L-3R)-6LR^2 \bigg], \end{flalign} \begin{flalign} \frac{\partial^3 s_1}{\partial s \partial u^2}(0,0) &= \frac{L \left(-a^3 L R+a^2 \left(L^2+R^2\right)-3 a R (L-3 R)-6 R^2\right)}{a^4 R^3}, \end{flalign} \begin{flalign} \frac{\partial^3 s_1}{\partial u^3}(0,0) &= \frac{L \left(a^2 L^2-3 a R (L-3 R)-6 R^2\right)}{a^4 R^2}, \end{flalign} \begin{flalign} \frac{\partial^3 u_1}{\partial s^3}(0,0) &= \frac{1}{a^4R^6} \bigg[ 6LR^2-8a^5R^3-12a^3LR(L+R)+3a^2L^2(L + 3R) + 2a^4R^2(8L + 3R)\nonumber \\ &\qquad - 3aL(L^2 - LR + 3R^2)+ aR(L^3-3aL^2R+4a^2LR^2-2a^3R^3) R^{''} \bigg], \end{flalign} \begin{flalign} \frac{\partial^3 u_1}{\partial s^2 \partial u}(0,0) &= \frac{-1}{a^4R^5} \bigg[ 6a^4 LR^2-a^3R \left(9L^2+3 LR + 2R^2\right) +a^2 L \left(3 L^2+6 L R+R^2\right) \nonumber \\ &\qquad- 3aL\left(L^2- LR + 3R^2\right) + aLRR''(L-a R)^2 + 6LR^2 \bigg], \end{flalign} \begin{flalign} \frac{\partial^3 u_1}{\partial s \partial u^2}(0,0) &= \frac{-1}{a^4R^4} \bigg[ 6LR^2 + 2a^4 LR^2 + a^2L(3L^2 + 3LR + 2R^2) \nonumber \\ &\qquad - 3aL(L^2 - LR + 3R^2) - 2a^3(3L^2R + R^3) + aL^2R(L - aR)R^{''} \bigg], \end{flalign} \begin{flalign} \frac{\partial^3 u_1}{\partial u^3}(0,0) &= \frac{3 (a-1) L \left(a^2 L R-a \left(L^2-L R+R^2\right)+2 R^2\right)-a L^3 R R''}{a^4 R^3}. \end{flalign} The Taylor polynomial of $\mathcal{T}(s,u) = (s_1, u_1)$ at $P = (0,0)$ is thus given by \begin{align} s_1(s,u) &= \ta_{10}s + \ta_{01}u + \sum_{j+k = 3} \ta_{jk}s^ku^k + \text{h.o.t} \label{eq: s1 mixed norm}\\ u_1(s,u) &= \tb_{10}s + \tb_{01}u + \sum_{j+k = 3} \tb_{jk}s^ku^k + \text{h.o.t}, \label{eq: u1 mixed norm} \end{align} where $\ta_{jk} = \frac{1}{j!k!} \frac{\partial^{j+k} s_1}{\partial s^j \partial u^k}(0,0)$, $\tb_{jk} = \frac{1}{j!k!} \frac{\partial^{j+k} u_1}{\partial s^j \partial u^k}(0,0)$, and h.o.t stands for higher order terms. \subsection{The first transformation: a rigid rotation}\label{ss.rigid} Having computed the third order Taylor polynomial of the billiard map $\mathcal{T}$, we may now initiate the construction of a symplectic coordinate transformation for $\mathcal{T}$ to be in Birkhoff normal form. The first step consists of finding a coordinate transformation for which $D_p\mathcal{T}$ acts as a rigid rotation. Given $D_P\mathcal{T}$ as in equation (\ref{eq: DpT mixed norm}), $\lambda = \ta_{10} - i\sqrt{-\ta_{01}\tb_{10}}$ is an eigenvalue of $D_P\mathcal{T}$ with corresponding eigenvector $v_\lambda = \left[ \begin{smallmatrix} i\eta \\[7pt] \eta^{-1} \end{smallmatrix} \right]$, where $\eta = \left(-\ta_{01}\tb_{10}\right)^{1/4} > 0$. It follows that $D_P\mathcal{T}$ is conjugate to a rotation matrix $R_\theta$ where $\theta \in (0, 2\pi)$ is the argument of $\lambda$. That is, we can write $D_PT = BR_\theta B^{-1}$ where \begin{equation} R_\theta = \begin{bmatrix} \cos \theta & -\sin \theta \\ \sin \theta & \cos \theta \end{bmatrix}, \qquad B = \begin{bmatrix} \eta & \ 0 \\ 0 & \ \eta^{-1} \end{bmatrix}. \end{equation} Define the coordinate transformation $(s,u) \mapsto (x,y)$ given by \begin{equation} \begin{bmatrix} x \\ y \end{bmatrix} = B^{-1}\begin{bmatrix} s \\ u \end{bmatrix} = \begin{bmatrix} s/\eta \\ \eta u \end{bmatrix}. \end{equation} \noindent We abuse notation by adopting the convention that $\mathcal{T}$ will always denote the billiard map regardless of the coordinate system. This will hold true for later transformations as well. Therefore we denote $\mathcal{T}(x,y) = (x_1, y_1) = (s_1/\eta, \eta u_1)$, and it follows form equations (\ref{eq: s1 mixed norm}) and (\ref{eq: u1 mixed norm}) that \begin{align} \begin{bmatrix} x_1 \\ y_1 \end{bmatrix} &= B^{-1} \begin{bmatrix} \ta_{10}s + \ta_{01}u + \sum\limits_{j+k =3} \ta_{jk}s^ju^k \\ \tb_{10}s + \tb_{01}u + \sum\limits_{j+k =3} \tb_{jk}s^ju^k \end{bmatrix} + \ \text{h.o.t} \label{eq: x1y1 B-1 mixed norm} \\[5pt] &= R_\theta \begin{bmatrix} x \\ y \end{bmatrix} + \begin{bmatrix} \sum\limits_{j+k = 3} a_{jk}x^jy^k \\ \sum\limits_{j+k = 3} b_{jk}x^jy^k \end{bmatrix} + \ \text{h.o.t} \label{eq: x1y1 R mixed norm} \end{align} \noindent where we obtained (\ref{eq: x1y1 B-1 mixed norm}) from (\ref{eq: x1y1 R mixed norm}) by using the relation $\begin{bmatrix} s \\ u \end{bmatrix} = B \begin{bmatrix} x \\ y \end{bmatrix} = \begin{bmatrix} \eta x \\ y/\eta \end{bmatrix}$. \noindent The coefficients $a_{jk}$ and $b_{jk}$ of the billiard map $\mathcal{T}$ in the new coordinates $(x,y)$ are given by \begin{equation} a_{jk} = \eta^{j-k-1}\ta_{jk} \qquad \text{and} \qquad b_{jk} = \eta^{j-k+1}\tb_{jk} \end{equation} \noindent for $j+k = 1,3$. Finally, note that $\mathcal{T}:(x,y) \mapsto (x_1,y_1)$ is symplectic since both $B$ and $T:(s,u) \mapsto (s_1,u_1)$ are symplectic. \begin{figure}[htbp] \begin{align*} \xymatrixcolsep{3pc} \xymatrixrowsep{3pc} \xymatrix{(s,u) \ar[d]_{\mathcal{T}} \ar[r]^{B^{-1}} & (x,y) \ar[d]_{\mathcal{T}}\\ (s_1,u_1) \ar[r]_{B^{-1}} & (x_1,y_1)} \end{align*} \caption{The commutative diagram of the first coordinate transformation} \label{fig: first tower of transformations} \end{figure} \subsection{The second transformation: diagonalizing the linear terms}\label{ss.diagonal} Next, we now need a second coordinate transformation in which $R_\theta$ becomes the diagonal matrix $D = \begin{bmatrix} \lambda & \hspace{10pt} 0 \\ 0 & \hspace{10pt} \overline{\lambda} \end{bmatrix}$. To do so, we embed $\bR^2 \subset \bC^2$ and consider $\mathcal{T}: (x,y) \rightarrow (x_1, y_1)$ to be a complex symplectic function on a small neighborhood $(0,0)\in \bC^2$. Define new complex coordinates $z$ and $w$ as \begin{equation} z = x + iy \qquad w = x - iy \label{z,w def} \end{equation} \noindent and let $\mathcal{T}(z,w) = (z_1, w_1) = (x_1+iy_1, x_1-iy_1)$. Note that under this coordinate transformation, the subspace $\bR^2$ of $\bC^2$ is mapped to $\{(z,w) \in \bC^2 : w = \overline{z} \}$ which may also be regarded as a real subspace. Observe further that $\det D_P\mathcal{T} \neq 0$. Thus by Inverse Function Theorem, there exists a neighborhood about $P$ on which $\mathcal{T}:(z,w) \mapsto (z_1,w_1)$ is a diffeomorphism so that $w_1 = \overline{z}_1$ if and only if $w = \overline{z}$. To compute the coefficients of the billiard map $\mathcal{T}:(z,w) \mapsto (z_1,w_1)$, we plug the Taylor expansions of $x_1$ and $y_1$ given in equation \eqref{eq: x1y1 R mixed norm} into the definitions for $z_1$ and $w_1$ to get \begin{align} z_1 &= x_1 + iy_1 \nonumber \\ &= x\cos\theta - y\sin\theta + \sum_{j+k =3}a_{jk}x^jy^k + i\left( x\sin\theta + y\cos\theta + \sum_{j+k = 3} b_{jk}x^jy^k \right) + \text{h.o.t.} \nonumber \\[10pt] &= \lambda \left( z + \sum_{j+k=3} c_{jk}z^jw^k \right) + \text{h.o.t.}, \label{eq: z1} \\ \intertext{and similarly,} w_1 &= \overline{\lambda}\left( w + \sum_{j+k = 3} \overline{c}_{jk}w^jz^k \right) + \text{h.o.t.}. \label{eq: w1} \end{align} \noindent The $c_{jk}$ in equations (\ref{eq: z1}) and (\ref{eq: w1}) are given by \begin{align} c_{30} &= 2^{-3}\overline{\lambda}\left( a_{30} + ib_{30} - ia_{21} + b_{21} - a_{12} - ib_{12} + ia_{03} - b_{03} \right), \\ c_{21} &= 2^{-3}\overline{\lambda}\left( 3a_{30} + 3ib_{30} - ia_{21} + b_{21} + a_{12} + ib_{12} - 3ia_{03} + 3b_{03} \right), \\ c_{12} &= 2^{-3}\overline{\lambda}\left( 3a_{30} + 3ib_{30} + ia_{21} - b_{21} + a_{12} + ib_{12} + 3ia_{03} - 3b_{03} \right), \\ c_{03} &= 2^{-3}\overline{\lambda}\left( a_{30} + ib_{30} + ia_{21} - b_{21} - a_{12} - ib_{12} -ia_{03} + b_{03}\right). \end{align} \begin{figure}[htbp] \begin{align*} \xymatrixcolsep{3pc} \xymatrixrowsep{3pc} \xymatrix{(s,u) \ar[d]_{\mathcal{T}} \ar[r]^{B^{-1}} & (x,y) \ar[d]_{\mathcal{T}} \ar[r] & (z,w) \ar[d]_{\mathcal{T}} \\ (s_1,u_1) \ar[r]_{B^{-1}} & (x_1,y_1) \ar[r] & (z_1, w_1) } \end{align*} \caption{The diagram after the second coordinate transformation} \label{fig: second tower of transformations} \end{figure} Since $\mathcal{T}:(x,y) \mapsto (x_1,y_1)$ is symplectic, then $D_P\mathcal{T} = 1$ in the $(x,y)$ coordinates. Further, the transformation $(x,y) \mapsto (z,w)$ has constant Jacobian. It follows that $ \frac{\partial(z_1, w_1)}{(z,w)} = 1 + (3c_{30} + \overline{c}_{12})z^2 + (2c_{21}+2\overline{c}_{21})zw + (c_{12}+3\overline{c}_{30})w^2 + \text{h.o.t.}$ is identically equal to one so that all the non-constant terms vanish. We thus get the following relations: \begin{equation} c_{12} = -3\overline{c}_{30} \qquad c_{21} = \overline{c}_{21} \end{equation} \noindent In particular, $c_{21}$ is purely imaginary, and we can thus write $c_{21} = i \tau_1$ where $\tau_1 \in \bR$ is given by: \begin{equation} \tau_1 = \frac{a\left(a + 4b\right)L-2\left(a^2 -3ab-6b^2\right)R + a^2\left(a+b\right)^2LRR''}{8a^2R\left[\left(a+b\right)L-2aR\right]} \label{eq: twist mixed norm symmetric}. \end{equation} Shortly we will show that $\tau_1$ is in fact, the first twist coefficient of $\mathcal{T}$ at $P$. \subsection{The third transformation: killing most 3rd-order terms}\label{ss.killing} Now we need a transformation $(z,w) \mapsto (z',w')$ such that most third order terms cancel. Let this transformation be given by \begin{align} z' &:= z + p_3(z,w) = z + \sum_{j+k = 3}d_{jk}z^jw'^k, \label{eq: z' mixed norm}\\ w' &:= w + \overline{p}_3(w,z) = w + \sum_{j+k = 3}\overline{d}_{jk}w^jz^k. \end{align} Then $z'_1$ satisfies \begin{align} z'_1 &= z_1 + \sum_{j+k = 3}d_{jk}z^j_1w^k_1= \lambda \left(z + \sum_{j+k = 3}c_{jk}z^jw^k \right) + \sum_{j+k = 3}d_{jk}\lambda^{j-k}z^jw^k + \ \text{h.o.t} \\[5pt] &= \lambda \left[ z' + \sum_{j+k = 3} \left( -d_{jk} + c_{jk} + d_{jk}\lambda^{j-k-1} \right)(z')^j(w')^k \right] + \ \text{h.o.t.} \label{eq: z1' mixed norm}. \end{align} \noindent Observe how in equation (\ref{eq: z1' mixed norm}) the $d_{21}$ terms always cancel. Therefore without loss of generality, we can set $d_{21} = 0$. By setting $-d_{jk} + c_{jk} + d_{jk}\lambda^{j-k-1} = 0$ for $(j,k) \in \{ (3,0), (1,2), (2,1) \}$ we obtain \begin{equation} d_{30} = \frac{c_{30}}{1-\lambda^2} , \qquad d_{12} = \frac{c_{12}}{1-\overline{\lambda}^2} = -3\overline{d}_{30}, \qquad d_{03} = \frac{c_{03}}{1-\overline{\lambda}^4}, \end{equation} \noindent so that equation (\ref{eq: z1' mixed norm}) can be rewritten as $z'_1 = \lambda\big(z' + c_{21}(z')^2(w')\big) + \text{h.o.t.}$. The ellipticity of $D_P\mathcal{T}$ implies $\lambda \neq \pm 1$, ensuring that the expressions for $d_{30}$ and $d_{12}$ do not involve division by zero. Similarly, assumption (A1) does the same for the expression of $d_{03}$. Having $c_{21} = i\tau$ and $\lambda = e^{i\theta}$, we can further rewrite $z'_1$ if we restrict to the subspace $\{w = \overline{z}\}$: \begin{equation} z'_1 = e^{i\theta}\left(z' + i\tau |z'|^2z'\right) + \text{h.o.t.} = e^{i\left(\theta + \tau_1 |z'|^2 \right)}z' + h.o.t.. \end{equation} This shows how the linear portion of billiard map $\mathcal{T}:(z',w') \mapsto (z'_1,w'_1)$ acts as a rotation about the elliptic fixed point. However, in the $(z',w')$ coordinates, $\mathcal{T}$ is not necessarily symplectic. \begin{figure}[htbp] \begin{align*} \xymatrixcolsep{3pc} \xymatrixrowsep{3pc} \xymatrix{(s,u) \ar[d]_{\mathcal{T}} \ar[r]^{B^{-1}} & (x,y) \ar[d]_{\mathcal{T}} \ar[r] & (z,w) \ar[d]_{\mathcal{T}} \ar[r]^{} & (z',w') \ar[d]_{\mathcal{T}} \ar[r] & \cdots \\ (s_1,u_1) \ar[r]_{B^{-1}} & (x_1,y_1) \ar[r] & (z_1, w_1) \ar[r] & (z'_1,w'_1) \ar[r] & \cdots} \end{align*} \caption{The diagram after the third coordinate transformation} \label{fig: tower of transformations} \end{figure} \subsection{The fourth transformation: a symplectic transformation}\label{ss.symplectic} As already mentioned, the billiard map $\mathcal{T}:(z',w') \mapsto (z'_1,w'_1)$ given in the previous section is not necessarily symplectic. This is an obstacle to using Moser's twist mapping theorem to prove the nonlinear stability of periodic billiard orbits. In this section, we will use the previous transformation $(z,w) \mapsto (z',w')$ to compute a real symplectic transformation $h_N:(x,y) \mapsto (X,Y)$ such that \begin{equation} h_N^{-1} \circ T \circ h_N \left( \begin{bmatrix} x \\ y \end{bmatrix} \right) = \begin{bmatrix} \cos\Theta(r) & -\sin\Theta(r) \\ \sin\Theta(r) & \cos\Theta(r) \end{bmatrix} \begin{bmatrix} x \\ y \end{bmatrix} + \text{h.o.t.}, \end{equation} where $r^2 = x^2 + y^2$ and $\Theta(r) = \theta + \tau_1r^2 + \tau_2r^4 + \cdots + \tau_{N-1}r^{2(N-1)}$. First, notice that $p_3(z,w)$ satisfies $\frac{\partial p_3}{\partial z} = - \frac{\partial \overline{p}_3}{\partial w}$. Hence there exists a polynomial $s_4(z,w)$ satisfying $\frac{\partial s_4}{\partial w}(z,w) = p_3(z,w)$ and $\frac{\partial s_4}{\partial z}(z,w) = -\overline{p}_3(w,z)$, given by \begin{equation} s_4(s,w) = \frac{1}{4}\frac{\overline{c}_{03}}{\lambda^4-1}z^4 - \frac{1}{4}\frac{c_{03}}{\overline{\lambda}^4-1}w^4 + \frac{\overline{c}_{30}}{\overline{\lambda}^2-1}zw^3 - \frac{c_{30}}{\lambda^2-1}z^3w, \end{equation} that is purely imaginary whenever $w = \overline{z}$. Hence we can thus define a real valued function $g_4(x+iy,x-iy) = \frac{i}{2}s_4(x+iy,x-iy)$. Next, consider the generating function $G(x,Y) := xY + g_4(x+iY, x-iY)$ of the symplectic transformation $(x,y) \mapsto (X,Y)$ where $X = X(x,y)$ and $Y = Y(x,y)$. Then \begin{alignat}{3} X &:= \frac{\partial G}{\partial Y} & &= x + \frac{i}{2} \big[ -\overline{p}_3(x-iY,x+iY)(i) + p_3(x+iY,x-iY)(-i) \big] \nonumber \\ & & &= x + \Re\left(p_3(x+iY, x-iY)\right) \nonumber \\ & & &= x + \Re\left( \sum\limits_{j+k=3}d_{jk}(x+iY)^j(x-iY)^k \right) \nonumber \\ & & &=: x + \sum_{j+k =3}p_{jk}x^jY^k ,\label{eq: X}\\ y &:= \frac{\partial G}{\partial x} & &= Y + \frac{i}{2} \bigg[ -\overline{p}_3(x-iY,x+iY) + p_3(x+iY,x-iY) \bigg] \nonumber \\ & & &= Y - \Im\left(p_3(x+iY,x-iY)\right) \nonumber \\ & & &= Y - \Im\left( \sum\limits_{j+k=3}d_{jk}(x+iY)^j(x-iY)^k \right) \nonumber\\ & & &=: Y - \sum_{j+k = 3}q_{jk}x^jY^k. \label{eq: y} \end{alignat} It follows from equations (\ref{eq: X}) and (\ref{eq: y}), that on a small neighborhood about $(x,y) = (0,0)$, we have \begin{align} X &= x + \sum_{j+k = 3}p_{jk}x^jy^k + \text{h.o.t.},\\ Y &= y + \sum_{j+k = 3}q_{jk}x^jy^k + \text{h.o.t.}. \end{align} Letting, $Z = X + iY$ and $W = X - iY$ gives \begin{align} Z &= x + iy + \Re\left(p_3(x+iy, x-iy)\right) + i\Im\left(p_3(x+iy, x-iy)\right) + \text{h.o.t.} \nonumber \\ &= z + p_3(z,w) + \text{h.o.t.} \intertext{Similarly,} W &= w + \overline{p}_3(w,z) + \text{h.o.t.}, \end{align} so that the billiard map $\mathcal{T}: (Z,W) \mapsto (Z_1,W_1)$ is given by \begin{align} Z_1 &= e^{i\left(\theta + \tau_1|Z| \right)}Z + \text{h.o.t.}, \label{eq: Z1}\\ W_1 &= e^{-i\left(\theta + \tau_1|Z|\right)}W + \text{h.o.t.}. \label{eq: W1} \end{align} Letting $\Theta = \theta + \tau_1|Z|^2$ and converting back to the variables $X$ and $Y$ yields the desired Birkhoff normal form, showing $\tau_1$ is the first twist coefficient of $\mathcal{T}$: \begin{equation} \begin{bmatrix} X_1 \\ Y_1 \end{bmatrix} = \begin{bmatrix} \cos\Theta & -\sin\Theta \\ \sin\Theta & \cos\Theta \end{bmatrix} \begin{bmatrix} X \\ Y \end{bmatrix} + \text{h.o.t.} \end{equation} \begin{figure}[htbp] \begin{align*} \xymatrixcolsep{3pc} \xymatrixrowsep{3pc} \xymatrix{(s,u) \ar[d]_{\mathcal{T}} \ar[r]^{B^{-1}} & (x,y) \ar[d]_{\mathcal{T}} \ar[r] & (X,Y) \ar[d]_{\mathcal{T}} \ar[r]^{} & (Z,W) \ar[d]_{\mathcal{T}}\\ (s_1,u_1) \ar[r]_{B^{-1}} & (x_1,y_1) \ar[r] & (X_1, Y_1) \ar[r] & (Z_1,W_1)} \end{align*} \caption{The diagram after the fourth coordinate transformation.} \label{fig: fourth transformation} \end{figure} \subsection{The first twist coefficient for asymmetric Minkowski billiards}\label{ss.asymmetry} In this subsection we study the general case where the (Euclidean) radius of curvature functions $R_j(s)$, have different values around $s=0$. In particular, $R_0(0) = R_0$ and $R_1(0) = R_1$ may be different. For the class of billiard tables $Q(L,\alpha, \beta)$, this will occur whenever the even polynomials $\alpha(t) = \sum_{n\geq1}\alpha_{2n}t^{2n}$ and $\beta(t) = \sum_{n\geq1}\beta_{2n}t^{2n}$ have different values for $\alpha_2$ and $\beta_2$. Going forward we will assume that $\alpha_2 > \beta_2$ so that $R_0 \leq R_1$. As before, we will consider the periodic 2-orbit $\mathcal{O}_2 = \{P, T(P)\}$. The billiard map ${\mathcal{T}}^2$ is given by the composition $(s,u) \xrightarrow{{\mathcal{T}}} (s_1,u_1) \xrightarrow{{\mathcal{T}}} (s_2, u_2)$ where as before $s_1 = s_1(s,u)$, $u_1 = u_1(s,u)$, $ s_2=s_2(s_1,u_1)$ and $u_2 =u_2(s_1,u_1)$. The partial derivatives $\frac{\partial^{j+k}s_2}{\partial s^j \partial u^k}$ and $\frac{\partial^{j+k}u_2}{\partial s^j \partial u^k}$ are straightforward albeit tedious applications of the chain rule. To illustrate it, we give expressions for $\frac{\partial s_2}{\partial s}$ and $\frac{\partial^2 s_2}{\partial s^2}$: \begin{align} \frac{\partial s_2}{\partial s} &= \frac{\partial s_2}{\partial s_1}\dsds + \frac{\partial s_2}{\partial u_1}\duds, \\[10pt] \frac{\partial^2 s_2}{\partial s^2} &= \frac{\partial^2 s_2}{\partial s_1^2} \left(\dsds \right)^2 + 2 \frac{\partial^2 s_2}{\partial s_1 \partial u_1}\duds\dsds + \frac{\partial s_2}{\partial s_1}\frac{\partial^2 s_1}{\partial s^2} + \frac{\partial^2 s_2}{\partial u_1^2} \left(\duds \right)^2 + \frac{\partial s_2}{\partial u_1}\frac{\partial^2 u_1}{\partial s^2}. \end{align} Denote $\tilde{a}_{jk} =\frac{1}{j!k!}\frac{\partial^{j+k}s_2}{\partial s^j \pa u^k}(0,0)$ and $\tilde{b}_{jk} =\frac{1}{j!k!}\frac{\partial ^{j+k}u_2}{\partial s^j \pa u^k}(0,0)$. Calculating the remaining first order partial derivatives and evaluating at $\mathcal{O}_2$, show that $D_P{\mathcal{T}}^2$ is given by \begin{equation} D_P{\mathcal{T}}^2 = \begin{bmatrix} \ta_{10} & \ta_{01} \\ \tb_{10} & \tb_{01} \end{bmatrix} = \begin{bmatrix} \frac{L}{aR_1}-1 & \frac{L}{a} \\ \frac{L-a(R_0 + R_1)}{aR_0R_1} & \frac{L}{aR_0}-1 \end{bmatrix} \begin{bmatrix} \frac{L}{aR_0}-1 & \frac{L}{a} \\ \frac{L-a(R_0+R_1)}{aR_0R_1} & \frac{L}{aR_1}-1 \end{bmatrix}, \end{equation} where \begin{align} \ta_{10} &= \tb_{01} = \frac{a R_0 \left(a R_1-2 L\right)+2 L \left(L-a R_1\right)}{a^2 R_0 R_1}, \\[10pt] \ta_{01} &= \frac{2L}{a}\left( \frac{L}{aR_1}-1 \right), \\[10pt] \tb_{10 }&=\frac{2 \left(L-a R_0\right) \left(L-a \left(R_0+R_1\right)\right)}{a^2 R_0^2 R_1}. \end{align} Then the orbit $\mathcal{O}_2$ is parabolic for $L \in \left\{ aR_0, aR_1, a(R_0 + R_1) \right\}$, hyperbolic for $L \in \left(aR_0, aR_1\right) \cup \left(a(R_0 + R_1), \infty \right)$, and elliptic for $L \in \left(0, a(R_0 +R_1)\right) \cup \left(aR_1, a(R_0 +R_1)\right)$. We assume the following non-resonance condition: \begin{itemize} \item[(B)] $\lambda^4 \neq 1$, or equivalently, $\left(L-aR_0\right)\left(L-aR_1 \right) \neq 0$. \end{itemize} \begin{theorem}\label{thm.t1.asym} Let $F(v) = a(v_1^2 + v_2^2)^{1/2} + b(v_1^4 + b_2^4)^{1/4}$ with $a+b=1$, and $Q(\alpha(t), \beta(t),L)$ be given. Additionally, assume that the resonance condition (B) is satisfied. Then the first twist coefficient of the two-step billiard map $T^2$ is given by \begin{equation} \tau_1 = \frac{\Delta}{8aR_0R_1(L-aR_0)(L-aR_1)\left(L-a(R_0+R_1)\right)}, \end{equation} \noindent where \begin{align} \Delta &= 2 a R_0^2 \left(-L R_1 \left(a^2 L R_1''+4 a^2-18 a+12\right)+a \left(2 a^2-9 a+6\right) R_1^2+a (3 a-4) L^2\right) \nonumber \\ &+ 2 a R_0^2 \left(-L R_1 \left(a^2 L R_1''+4 a^2-18 a+12\right)+a \left(2 a^2-9 a+6\right) R_1^2+a (3 a-4) L^2\right) \nonumber \\ &+ R_0 \bigg(L^2 R_1 \left(a^2 L R_0''+a^2 L R_1''+8 a^2-36 a+24\right)-2 a L R_1^2 \left(a^2 L R_0''+4 a^2-18 a+12\right) \\ &+a^2 R_1^3 \left(a^2 L R_0''+2 a^2-9 a+6\right)+a (4-3 a) L^3 \bigg) +a^2 R_0^3 \big(R_1 \left(a^2 L R_1''+2 a^2-9 a+6\right) \nonumber \\ &+a (4-3 a) L\big)-a (3 a-4) L R_1 \left(L-a R_1\right){}^2. \nonumber \end{align} \end{theorem} \begin{proof} We only give a sketch of the computation. The detailed computation be found in \cite{Vil}. The Taylor coefficients $\ta_{jk}$ and $\tb_{jk}$ can be done by taking derivatives of the two functions $s_2$ and $u_2$ repeatedly. Combining with the assumption on two functions $\alpha$ and $\beta$ being even, we have that $\ta_{jk}$'s and $\tb_{jk}$'s for $j+k = 2$ are identically zero. The remaining of the computation for $\tau_1$ is exactly the same as in Section~\ref{ss.rigid}, \ref{ss.killing} and \ref{ss.symplectic}. \end{proof} \begin{remark} The first twist coefficient of elliptic periodic orbits of period $2$ in Euclidean billiards \cite{JZ22, KP05} is recovered by setting $a=1$ and $b=0$: \begin{equation} \tau_1 = \frac{1}{8}\bigg(\frac{R_0+R_1}{R_0R_1}-\frac{L}{R_0+R_1-L}\left(\frac{R_1-L}{R_0-L}R_0'' + \frac{R_0-L}{R_1-L}R_1''\right)\bigg). \end{equation} \end{remark} \section{Applications}\label{sec.applications} In this section we use the first twist coefficient given in Theorem \ref{eq: symmetric twist} to prove the nonlinear stability of the periodic orbits of period $2$ on various symmetric billiard tables. Using the assumption $a+b =1$, we will shorten the notation for Minkowski norm as $F_{a}= F_{a, 1-a}$ with $ 0< a \le 1$, and will denote the Minkowski billiard map as $\cT_a$ to emphasize its dependence on the parameter $a$. We have showed in Section~\ref{ss.tangent.map} that the periodic point $P$ of period $2$ is elliptic for $0<L<2aR$, parabolic for $L=2aR$, and hyperbolic for $L>2aR$. \subsection{Euclidean circular tables} Consider the Euclidean circular billiard table $Q_r$ on the Minkowski plane $(\bR^2, F_{a})$ bounded by the circle $x^2 + y^2 = r^2$. Then the orbit bouncing back and forth along the $y$-axis is a periodic orbit of period $2$, say $\mathcal{O}_2(a, r) =\{P_r, \cT_a(P_r)\}$. In this case, we have $R = r$ and $L=2r$. From the stability conditions above, it follows that the periodic orbit $\mathcal{O}_2(a, r)$ is hyperbolic for $0< a < 1$ (see Figures \ref{fig:circleorbits}(a) and \ref{fig:circlephase}) and parabolic only for $a=1$, which corresponds to the Euclidean norm $F_{1}(v) = \|v\|_2$. In particular, such an orbit cannot be elliptic. Additionally, numerical simulations demonstrate the nonlinear stability of the periodic 2-orbits $\{(\frac{\sqrt{2}}{2},\frac{\sqrt{2}}{2}),(-\frac{\sqrt{2}}{2},-\frac{\sqrt{2}}{2})\}$ and $\{(-\frac{\sqrt{2}}{2},\frac{\sqrt{2}}{2}),(\frac{\sqrt{2}}{2},-\frac{\sqrt{2}}{2})\}$ (see Figures \ref{fig:circleorbits}(b) and \ref{fig:circlephase}). Although the twist coefficient in equation (\ref{eq: symmetric twist}) does not pertain to these periodic 2-orbits, we can obtain their twist coefficient in a similar way as presented above. However, equations (\ref{eq: dsds})-(\ref{eq: dudu}) would not simplify as nicely as they do in equations (\ref{eq: dsds mixed norm})-(\ref{eq: dudu mixed norm}). \begin{figure}[htbp] \centering \subcaptionbox{100 reflections of the billiard orbit with initial point $(0,-1)$ and direction $\pi/2+0.01$}{\includegraphics[width=0.48\textwidth]{Images/B1a08_hyperbolic}} \hfill \subcaptionbox{100 reflections of the billiard orbit wit initial point $(\frac{\sqrt{2}}{2},\frac{\sqrt{2}}{2})$ and initial direction $\frac{5\pi}{4}+0.1$}{\includegraphics[width=0.48\textwidth]{Images/B1a08_stable2up}} \caption{Dynamics billiards on the table bounded by $x^2 + y^2 = 1$ with Minkowski norm $F_{0.8}$.} \label{fig:circleorbits} \end{figure} \begin{figure}[htbp] \centering \includegraphics[width=0.7\textwidth]{Images/phase_a08B1} \caption{Phase space of Minkowski billiards with norm $F_{0.8}$ on the table bounded by $x^2 + y^2 =1$} \label{fig:circlephase} \end{figure} \subsection{Symmetric lemon billiards} Next, consider the symmetric billiard table $Q(L)$ discussed in Section~\ref{ss.sym.billiards}, where $\gamma_0$ and $\gamma_1$ are Euclidean circular arcs of radius $1$. In this case, $R=1, R'' = 0$, and $L\in (0,2)$. From the stability conditions, the orbit $\mathcal{O}_2(a,L) =\{P_L, \cT_a(P_L)\}$ is elliptic whenever $L<2a$ with nonresonance condition $L\neq a$. The first twist coefficient is given by \begin{equation} \tau_1(\mathcal{T}_a, P_L) = \frac{a^2 (3 L-4)+a (18-4 L)-12}{8 a^2 (2 a-L)}. \end{equation} Note that $\tau_1(\mathcal{T}_a, P_L)=0$ when $L = \frac{2\left(2a^2-9a+6\right)}{a\left(3a-4\right)}$. \begin{figure}[htbp] \centering \includegraphics[width=0.8\linewidth]{Images/lemon_symmetric_graph} \caption{Stability regions for the orbit $\mathcal{O}_2(a, L)$ on a symmetric lemon table $Q(L)$ on the Minkowski plane with norm $F_{a}$.} \label{fig:lemonsymmetricgraph} \end{figure} \begin{corollary} Let $0<a <1$ and $\mathcal{O}_2(a, L)$ be the periodic orbit on the symmetric lemon table $Q(L)$ running along the $x$-axis. Then the orbit $\mathcal{O}_2(a, L)$ is nonlinearly stable for $L \in (0,2a) \backslash \left\{ a, \frac{4a^2-18a+12}{a(3a-4)} \right\}$. \end{corollary} \begin{remark} It is not clear if the periodic orbit $\mathcal{O}_2(a, L)$ is nonlinearly stable when $L= \frac{4a^2-18a+12}{a(3a-4)}$. In this case, $\tau_1(\mathcal{T}_a, P_L) =0$, and finding the next coefficient $\tau_2(\mathcal{T}_a, P_L)$ may be helpful. \end{remark} \subsection{Euclidean elliptical tables} Let $\delta \in (0,1)$, and $E(\delta)$ be the billiard table $E(\delta)$ bounded by the Euclidean ellipse given $x^2 + \frac{y^2}{\delta^2}= 1$. It is easy to see that there is a periodic orbit $\mathcal{O}_2(a, \delta)=\{P_{\delta}, \cT_a(P_{\delta}\}$ of period $1$ running along the minor axis of the ellipse $E(\delta)$. Then $L = 2\delta$ and $R = 1/\delta$, so that $\frac{L}{R}=2\delta^2$. It follows that $\mathcal{O}_2(a, \delta)$ is elliptic for $\delta < \sqrt{a}$, parabolic for $\delta = \sqrt{a}$, and hyperbolic for $\delta > \sqrt{a}$. Consequently, $\mathcal{O}_2(a, \delta)$ is always elliptic in the Euclidean case, and the non-resonance condition is satisfied for $\delta \neq \frac{\sqrt{a}}{2}$. A short calculation shows $R''=\frac{3\left(\delta^2-1\right)}{\delta}$ so that the first twist coefficient is given by \begin{equation} \tau_1(P_{\delta}, \cT_a) = \frac{\delta\left(-6+a(9+a-4\delta^2)\right)}{8a^2(a-\delta^2)}. \end{equation} \begin{figure}[htbp] \centering \includegraphics[width=0.8\linewidth]{Images/elliptic_billiards_graph} \caption{Stability regions for the orbit $\mathcal{O}_2(a, \delta)$ on an elliptic table $E(\delta)$ on the Minkowski plane with norm $F_{a}$.} \label{fig: elliptic billiard graph} \end{figure} Note that $\tau_1(P_{\delta}, \cT_a)=0$ when $\delta = \frac{\sqrt{a^2+9a-6}}{2\sqrt{a}}$. This value of $\delta$ only makes sense when it is real and less than $\sqrt{a}$ which occurs whenever $a \in \left(\frac{\sqrt{105}-9}{2},1\right)$ \begin{corollary} On the elliptical billiard table $E(\delta)$, the periodic orbit $\mathcal{O}_2(a, \delta)$ running along the minor axis is nonlinearly stable whenever $\delta < \sqrt{a}$ and $\delta \neq \frac{\sqrt{a^2 + 9a -6}}{2\sqrt{a}}$ and $\delta \neq \frac{\sqrt{a}}{2}$. \end{corollary} \begin{figure}[htbp] \centering \includegraphics[width=0.7\textwidth]{Images/phase_a08B05} \caption{Phase space of Minkowski billiards with norm $F_{0.8}$ on the table $E(0.5)$} \end{figure} \subsection{General symmetric billiards} For the class of billiard tables $Q(\alpha, \beta, L)$ where $\alpha(t) = \beta(t)$, $R(s)$ satisfies $R = \frac{1}{2 \alpha_2}$ and $R'' = 6\alpha_2 - \frac{6\alpha_4}{\alpha_2^2}$ at both $\gamma_0(0)$ and $\gamma_1(0)$. Hence the orbit $\mathcal{O}_2(a, \alpha)=\{P_{\alpha}, \cT_a(P_{\alpha}\}$ is elliptic whenever $L \in \left(0, \frac{a}{\alpha_2}\right)$. Further, the non-resonance condition is satisfied whenever $L \neq \frac{a}{2\alpha_2}$. From equation (\ref{eq: symmetric twist}), we have that $\tau_1(P_{\alpha}, \cT_a) =0$ when \begin{align} R'' = -\frac{-3 a^2 L+4 a^2 R+4 a L-18 a R+12 R}{a^2 L R}, \end{align} or equivalently, \begin{align} \alpha_4 = -\frac{-2 a^2 \alpha _2^2+9 a \alpha _2^2-4 a \alpha _2^3 L-6 \alpha _2^2}{3 a^2 L}. \end{align} \begin{thebibliography}{99} \bibitem{AFOR} S. Artstein-Avidan, D. I. Florentin, Y. Ostrover, D. Rosen. \emph{Duality of caustics in Minkowski billiards}. Nonlinearity 31 (2018), 1197--1226. \bibitem{AKO14} S. Artstein-Avidan, R. Karasev, Y. Ostrover. \emph{From symplectic measurements to the Mahler conjecture}. Duke Math. J. 163 (11), 2003--2022 (2014). \bibitem{AO14} S. Artstein-Avidan, Y. Ostrover. \emph{Bounds for Minkowski billiard trajectories in convex bodies}. International Mathematics Research Notices 2014 (1), 2014, 165--193. \bibitem{BMS19} V. Balestro, H. Martini, E. Shonoda. \emph{Concepts of curvatures in normed planes}. Expositiones Mathematicae. 37(4), 347--381, 2019. \bibitem{BCS} D. Bao, S.-S. Chern, Z. Shen. \emph{An introduction to Riemann-Finsler geometry}. Grad. Texts in Math. 200, Springer-Verlag, New York, 2000. \bibitem{Bi27} G. D. Birkhoff. \emph{Dynamical Systems}. {Providence R.I.: AMS Coll. Pub.} Vol. 9, 1927. \bibitem{Bi27b} G. D. Birkhoff. \emph{On the periodic motions of dynamical systems}. Acta Math. 50 (1): 359--379, 1927. \bibitem{Bu74a} L. A. Bunimovich. \emph{On billiards close to dispersing.} Math. USSR Sb. 23 (1), 45--67 (1974). \bibitem{Bu74} L. A. Bunimovich. \emph{On ergodic properties of certain billiards}. Funct. Anal. Its Appl. 8, 254--255 (1974). \bibitem{Bu92} L. A. Bunimovich. \emph{On absolutely focusing mirrors}. Ergodic Theory and Related Topics, III (Güstrow, 1990) Lecture Notes Math. 1514, Berlin: Springer, 62--82, 1992. \bibitem{BuDM} L. A. Bunimovich, G. Del Magno. \emph{Track Billiards}. Commun. Math. Phys. 288, 699--713 (2009). \bibitem{BuGr} L. A. Bunimovich, A. Grigo. \emph{Focusing components in typical chaotic billiards should be absolutely focusing}. Commun. Math. Phys. 293 (2010), 127--143. \bibitem{BZZ} L. A. Bunimovich, H.-K. Zhang, P. Zhang. \emph{On another edge of defocusing: hyperbolicity of asymmetric lemon billiards}. Commun. Math. Phys. 341 (2016), 781--803. \bibitem{Ch13} J. Chen, L. Mohr, H.-K. Zhang, P. Zhang. \emph{Ergodicity of the generalized lemon billiards} Chaos 23, 043137 (2013). \bibitem{ChSh} X. Cheng, Z. Shen. \emph{Finsler Geometry: An Approach via Randers Spaces}. Springer Berlin, Heidelberg, 2012. \bibitem{ChMa} N. Chernov, R. Markarian. \emph{Chaotic Billiards}. Math. Surveys Monographs 127, Providence, RI, AMS, 2006. \bibitem{CGM} A. Cima, A. Gasull, V. Ma\~{s}osa. \emph{Global periodicity conditions for maps and recurrences via normal forms}. Internat. J. Bifur. Chaos Appl. Sci. Engrg. 23 (2013), no. 11, 1350182, 18 pages. \bibitem{DKP03} M. J. Dias Carneiro, S. O. Kamphorst, S. Pinto-de-Carvalho. \emph{Elliptic islands in strictly convex billiards}. Ergod. Th. Dynam. Syst. 23 (3), 799--812 (2003). \bibitem{Don91} V. J. Donnay. \emph{Using integrability to produce chaos: billiards with positive entropy}. Commun. Math. Phys. 141 (1991), 225--257. \bibitem{FLBP} C. L. Foden, M. L. Leadbeater, J. H. Burroughes, M. Peper. \emph{Quantum magnetic confinement in a curved two-dimensional electron gas}. {J. Phys. Condens. Matter} {\bf 6} (1994), L127. \bibitem{GT02} E. Gutkin, S. Tabachnikov. \emph{Billiards in Finsler and Minkowski geometries}. J. Geom. Phys. 40 (2002), no. 3-4, 277--301. \bibitem{He93} E. Heller, S. Tomsovic. \emph{Postmodern quantum mechanics}. Physics Today 1 (1993), 46(7): 38--46. \bibitem{JZ21} X. Jin, P. Zhang. \emph{Hyperbolicity of asymmetric lemon billiards}. Nonlinearity 34 (1) (2021), 92--117. \bibitem{JZ22} X. Jin, P. Zhang. \emph{Birkhoff Normal Form and Twist coefficients of periodic orbits of billiards}. Nonlinearity 35 (8) (2022), 3907--3943. \bibitem{KP01} S. O. Kamphorst, S. Pinto-de-Carvalho. \emph{Elliptic islands on the elliptical stadium}. Discrete and Continuous Dynamical Systems, 2001, 7 (4): 663--674. \bibitem{KP05} S. O. Kamphorst, S. Pinto-de-Carvalho. \emph{The first Birkhoff coefficient and the stability of 2-periodic orbits on billiards}. {Experiment. Math.} 14 (2005), No. 3, 299--306. \bibitem{La73} V. F. Lazutkin. \emph{Existence of a continuum of closed invariant curves for a convex billiard}. Math. USSR Izv. 7 (1973), 185--214. \bibitem{Mar88} R. Markarian. \emph{Billiards with Pesin region of measure one}. Commun. Math. Phys. 118 (1988), 87--97. \bibitem{MaKaPdC} R. Markarian, S. O. Kamphorst, S. Pinto-de-Carvalho. \emph{Chaotic Properties of the Elliptical Stadium}. Commim. Math. Phys. 174 (1996), 661--679. \bibitem{MS06} H. Martini, K. J. Swanepoel. \emph{Antinorms and Radon curves}. Aequ. math. 72 (2006), 110--138. \bibitem{Moe90} R. Moeckel. \emph{Generic bifurcations of the twist coefficient}. {Ergod. Th. Dynam. Sys.} {10} (1990), 185--195. \bibitem{Mos56} J. Moser. \emph{The analytic invariants of an area-preserving mapping near a hyperbolic fixed point} Commun. Pure Appl. Math. 9 (1956) 673--692. \bibitem{Mos73} J. Moser. \emph{Stable and Random Motions in Dynamical Systems. With Special Emphasis on Celestial Mechanics}. Princeton, NJ: Princeton University Press, 1973. \bibitem{Ru59} H. Rund. \emph{The differential geometry of Finsler spaces}. Grundlehren der mathematischen Wissenschaften (GL, v. 101), 1959. \bibitem{Sch09} R. E. Schwartz. \emph{Obtuse Triangular Billiards II: One Hundred Degrees Worth of Periodic Trajectories.} {Experiment. Math.}{18} (2) 131--171, 2009. \bibitem{SiMo} C. L. Siegel, J. K. Moser. \emph{Lectures on Celestial Mechanics}. Berlin: Springer-Verlag, 1971. \bibitem{Sin70} Y. G. Sinai. \emph{Dynamical systems with elastic reflections}. Russ. Math. Surv. 25 (1970), 137--189. \bibitem{Vet} A. Vetier. \emph{Sinai billiard in potential field (construction of stable and unstable fibers)}. in ``Limit theorems in probability and statistics'', 1079--1146, Colloq. Math. Soc. Janos Bolyai, 36, North-Holland, Amsterdam, 1984. \bibitem{Vil} C. Villanueva. \emph{On the Stability of 2-Periodic Orbits in Minkowski Billiard Systems}. PhD Thesis, University of Oklahoma, 2024. \bibitem{Woj86} M. Wojtkowski \emph{Principles for the design of billiards with nonvanishing Lyapunov exponents.} {Commun. Math. Phys.} 105 (1986), 391--414. \bibitem{XiZh14} Z. Xia, P. Zhang. \emph{Homoclinic points for convex billiards}. Nonlinearity 27 (6) (2014), 1181--1192. \bibitem{Zha17} P. Zhang. \emph{Convex billiards on convex spheres}. Ann. Inst. H. Poincar\'e Anal. Non Lin\'eaire 34 (2017), 793--816. \end{thebibliography} \end{document}
2412.03601v3
http://arxiv.org/abs/2412.03601v3
Relations between average shortest path length and another centralities in graphs
\documentclass[12pt, a4paper]{extarticle} \usepackage[margin=2cm]{geometry} \usepackage[affil-it]{authblk} \usepackage[english]{babel} \usepackage{graphicx} \usepackage{wrapfig} \usepackage{amsmath,amsthm,amssymb,scrextend} \usepackage{booktabs} \usepackage{multirow} \usepackage{amsfonts} \usepackage{multicol} \usepackage[shortlabels]{enumitem} \usepackage{cutwin} \setlength{\columnsep}{1cm} \usepackage{float} \usepackage{csquotes} \usepackage{enumitem} \usepackage{tikz} \usepackage{hyperref} \def \rR {\mathbb{R}} \def \bf#1 {\textbf{#1 }} \def \sumt {\sum\limits} \providecommand{\keywords}[1] { \small \textbf{\textit{Keywords:}} #1 } \let\newproof\proof \renewenvironment{proof}{\begin{addmargin}[1em]{0em}\begin{newproof}}{\end{newproof}\end{addmargin}\qed} \newtheorem{thm}{Theorem} \newtheorem{lm}{Lemma} \newtheorem{cor}{Corollary} \newtheorem*{lm*}{Lemma} \newtheorem{defin}{Definition} \newtheorem*{defin*}{Definition} \def \sumt {\sum\limits} \DeclareMathOperator{\diam}{diam} \DeclareMathOperator{\dist}{dist} \DeclareMathOperator{\BC}{BC} \DeclareMathOperator{\Clo}{Clo} \DeclareMathOperator{\Rad}{Rad} \DeclareMathOperator{\Str}{Str} \begin{document} \title{Relations between average shortest path length and another centralities in graphs} \author{Mikhail Tuzhilin \thanks{Affiliation: Moscow State University, Electronic address: \texttt{[email protected]};} } \date{} \maketitle \begin{abstract} Relations between average shortest path length and radiality, closeness, stress centralities and average clustering coefficient were obtained for simple connected graphs. \end{abstract} \keywords{Networks, centralities, local and global properties of graphs, average shortest path length, Watts-Strogatz clustering coefficient.} \section{Introduction.} Centrality measures are important characteristics in network science which contain intrinsic hidden information about networks. Centrality is a local (with relation to a vertex) or global (with relation to a whole graph) real functions on graphs. There exist many centralities~\cite{Borgatti}-~\cite{Lee}: local efficiency, radiality, closeness, betweenness, stress centralities, etc. One of the most important centralities for ``real'' networks (or small-world networks) are average clustering coefficient and average shortest path length~\cite{Watts}. The network is called small-world if it has small average shortest path length with relation to the size and big average clustering coefficient. This measures are separately calculated to prove that current network is small-world. In this article we proved that for geodetic graphs (or graphs with odd length cycles) with additional condition there is relation between average shortest path length and average clustering coefficient. \section{Main definitions.} All subsequent definitions are given for a simple, connected, undirected graph $G$. Let's denote by \begin{itemize} \item $V(G)$ the set of vertices,\;\; $E(G)$ the set of edges and\;\; $A = \{a_{ij}\}$ adjacency matrix of graph $G$. \item Neighborhood $N(v)$ --- the induced subgraph in $G$ on vertices which adjacent to the vertex $v$, \item $N'(v)$ induced subgraph in $G$ on vertices $ = V\big(N(v)\big)\bigcup \{v\}$, \item $\bar{f}(x_1, x_2, ... , x_k)$, where $f$ is any function $V\times V \times ... \times V\rightarrow\rR$, the restriction of this function on $N'(v)$ (for example $\bar L(x,y)$ will be the average shortest path between $x$ and $y$ with restriction to subgraph $N'(v)$), \item $d_i = deg(v_i)$, \item $n = |V(G)|,\;\; m = |E(G)|$, \item $X(i) = X(v_i)$ for any $X$ --- set or function corresponding to vertex $v_i$, \end{itemize} Let's give definitions of centralities: \begin{enumerate} \item \bf{Diameter} $diam(G) = \max_{s,t\in V(G)} dist(s,t)$. \item \bf{Average shortest path length} $L(G) = \frac 1 {n(n-1)} \sumt_{s\neq t} dist(s,t)$. \item \bf{Local cluster coefficient} $c_i = c(i) = \frac {\text{number of edges in }N(i)} {\text{maximum possible number of edges in }N(i)}= \frac{2 |E(N(i))|} {d_i(d_i-1)}$. \item \bf{Average clustering coefficient} $C_{WS}(G) = \frac 1 {n} \sumt_{i\in V(G)} c_i = \frac 1 {n} \sumt_{i\in V(G)} \frac{2 |E(N(i)))|} {d_i(d_i-1)} = \frac 1 n \sumt_{i\in V(G)} \frac {\sumt_{j, k\in V(G)} a_{ij} a_{jk} a_{ki}} {d_i (d_i-1)}$. \item \bf{Global clustering coefficient} $C(G) = \frac {\text{number of closed triplets in $G$}} {\text{number of all triplets in $G$}} = \frac {\sumt_{i, j, k\in V(G)} a_{ij} a_{jk} a_{ki}} {\sumt_{i\in V(G)} d_i (d_i-1)}$. \item \bf{Closeness centrality} $Clo(v) = \frac {n-1} {\sumt_{t\in V(G)} dist(v, t)}$. \item \bf{Radiality} $Rad(v) = \frac {\sumt_{t\in V(G), t\neq v} (diam(G)+1-dist(v, t))} {n-1}$. \item \bf{Stress} $Str(i) = \sumt_{s,t\in V(G),\; s\neq t\neq i} \sigma_{st}(i)$, where $\sigma_{st}(i)$ is the total number of shortest paths from $s$ to $t$ which contains vertex $i$. \end{enumerate} Note that all centralities are non-negative and $c_i, C_{WS}, \Clo(v)$ are less or equal 1. Also let's give the definition of geodetic graph: \begin{defin} If there exists the unique shortest path between any two vertices in $G$ then the graph $G$ is called \bf{geodetic}. \end{defin} This definition is equivalent to the condition then there is no even cycles in a graph. \section{Main results.} Let's consider a induced subgraph $G'\subset G$. In general $G'$ can be not connected graph. In this case let's define the average shortest path length for vertices of $G'$ with relation to the distance $\dist$ in the ambient graph $G$. Let's call $L\big(N(i)\big)$ the local average shortest path length for the vertex $i$. Let's start with simple relations: let's proof the relation between local shortest path length and local clustering coefficient: \begin{lm}\label{lm1}\label{lm1} $$L(N(i)) = 2-c_i.$$ \end{lm} \begin{proof} $$ L(N(i)) = \frac 1 {d_i(d_i-1)} \sumt_{s,t\in N(i), s\neq t} dist(s,t) = \frac 1 {d_i(d_i-1)} \sumt_{(s,t)\in E(N(i))} dist(s,t)+ \sumt_{s,t\in N(i), (s,t)\notin E(N(i))} dist(s,t) = $$ $$ = \frac 1 {d_i(d_i-1)} (2 |E(N(i))| + \sumt_{(s,i), (i, t)\in E(G), (s,t) \notin E(G)} dist(s,t)) = $$ $$ = \frac 1 {d_i(d_i-1)} (2 |E(N(i))|+2 (d_i(d_i-1) - 2 | E(N(i))|)) = 2 - c_i. $$ Note that shortest paths for vertices in $N(i)$ are defined corresponding to whole graph $G$. \end{proof} Averaging by $i$ we obtain simple corollary about the relation between local shortest path length and average clustering coefficient. \begin{cor} $$C_{WS}(G) = 2- \frac 1 n \sumt_{i\in V(G)}L\big(N(i)\big).$$ \end{cor} Let's proof the relation between shortest path length in subgraph $N'(i)$ and shortest path length in $N(i)$. \begin{lm} $$L\big(N'(i)\big) = \frac {(d_i-1)L\big(N(i)\big)+2} {d_i+1}.$$ \end{lm} \begin{proof} By definition $$ L\big(N'(i)\big) = \frac 1 {(d_i+1)d_i} \sumt_{s,t\in V(N'(i)),\;s\neq t} \dist(s,t) = \frac {d_i-1} {d_i+1} L\big(N(i)\big) + \frac 2 {d_i+1}. $$ \end{proof} Let's proof the relation between shortest path length in a induced subgraph and shortest path length in ambient graph if induced subgraph is obtained from ambient graph by deleting one vertex. \begin{thm} Let a graph $G$ is obtained from a connected simple graph $G'$ by deleting one vertex and $|V(G)| = n$. Then $$L(G')\geq \frac n {n+1} L(G),$$ where the average shortest path length $L(G)$ is defined in the ambient graph $G'$, if $G$ is not connected. \end{thm} \begin{proof} Let's define the deleted vertex by $v$. By the triangle inequality $\forall s, t\in V(G): \dist(s,v)+\dist(v,t)\geq \dist(s,t)$, where the equality holds if, there are no paths from $s$ to $t$ in $G$. Therefore, $$\sumt_{s,t\in V(G), s\neq t}\big(\dist(s,v)+\dist(v,t)\big)\geq \sumt_{s,t\in V(G), s\neq t}\dist(s,t)$$ $$\frac {2(n-1)} {n(n-1)}\sumt_{t\in V(G)}\dist(v,t)\geq\frac 1 {n(n-1)} \sumt_{s,t\in V(G), s\neq t}\dist(s,t)$$ $$\frac {2} {n}\sumt_{t\in V(G)}\dist(v,t)\geq L(G)$$ Then, $$L(G') = \frac 1 {(n+1)n} \sumt_{s,t\in V(G'),s\neq t}\dist(s,t) = \frac 1 {(n+1)n} \Big(2 \sumt_{t\in V(G)}\dist(v,t)+\sumt_{s,t\in V(G),s\neq t}\dist(s,t)\Big) =$$ $$=\frac 1 {n+1} \frac 2 {n} \sumt_{t\in V(G)}\dist(v,t) + \frac {n-1} {n+1} L(G)\geq \frac n {n+1} L(G)$$ Note that if $G$ consists of $n$ isolated vertices then the equality holds. \end{proof} We obtain corollaries \begin{cor} Let a graph $G$ is obtained from a connected simple graph $G' \subset H$ by deleting one vertex, $|V(G)| = n$ and $H$ is connected and simple. Then $$L(G')\geq \frac n {n+1} L(G),$$ where the average shortest path lengths $L(G)$ and $L(G')$ are defined in the ambient graph $H$, if corresponded graphs are not connected. \end{cor} \begin{proof} The proof is the same as for the previous theorem and also if $G$ consists of $n$ isolated vertices then the equality holds. \end{proof} \begin{cor}\label{cor1} $$L\big(N'(i)\big) \geq \frac {d_i} {d_i+1}L\big(N(i)\big).$$ \end{cor} Let's proof the relation between shortest path length in a induced subgraph and shortest path length in ambient graph. \begin{thm} Let's $G'\subset G$ be induced subgraph and $|V(G)| = n, |V(G')| = n+k$. Then $$L(G')\geq \frac n {n+k} L(G),$$ where the average shortest path length $L(G)$ is defined in the ambient graph $G'$, if $G$ is not connected. \end{thm} \begin{proof} Let's construct the graph $G'$ from $G$ by adding sequentially $k$ vertices and corresponded edges. Let's first sequentially add vertices adjacent to vertices of the graph $G$. Adding these vertices one by one we obtain a sequence of graphs $G_1, G_2, ... , G_p, $ where $|V(G_i)| = n+i$. Further, let's add is the same way vertices adjacent to vertices of the graph $G_p$ and so on. In the end we obtain the graph $G'$. By previous corollary $$\hspace{-15pt}L(G')\geq \frac {n+k-1} {n+k} L(G_{k-1})\geq \frac {n+k-1} {n+k} \frac {n-k-2} {n-k-1} L(G_{k-2}) = \frac {n-k-2} {n+k} L(G_{k-2})\geq \cdots \geq \frac n {n+k} L(G).$$ \end{proof} Let's proof the relation between average closeness centrality and average shortest path length: \begin{lm} $$ L(G)\geq \frac n {\sumt_{v\in V(G)} \Clo(v)}. $$ \end{lm} \begin{proof} By the inequality of harmonic mean and arithmetic mean $$ \frac 1 {n} \sumt_{v\in V(G)} Clo(v) = \frac 1 n \sumt_{v\in V(G)} \frac {n-1} {\sumt_{t\in V(G)} dist(v, t)}\geq \frac {n(n-1)} {\sumt_{v, t\in V(G)} dist(v, t)} = \frac 1 {L(G)}. $$ Note that an equality holds when all average shortest path lengths from any vertex to all remaining vertices are equal. \end{proof} Let's proof the relation between average shortest path length and average radiality: \begin{lm}\label{lm3} $$ L(G) = \diam(G)+1 -\frac 1 {n} \sumt_{v\in V(G)} {\Rad}(v). $$ \end{lm} \begin{proof} The proof holds from definition $$ \frac 1 {n} \sumt_{v\in V(G)} {Rad}(v) = \frac 1 {n} \sumt_{v\in V(G)} \frac {(n-1) (diam(G)+1)- \sumt_{t\in V(G),\; t\neq v} dist(v,t))} {n-1} = diam(G)+1 -L(G). $$ \end{proof} Now let's prove theorem about a relation between average clustering coefficient and radiality using previous theorem. \begin{thm} $$ \frac {C_{WS}(G)} 2 \geq \frac 1 n \sumt_{i\in V(G)} \frac {\sumt_{v\in N(i)} \overline{\Rad}(v)} {d_i} -2 + \frac {\#\{N(i) \text{ which are complete graphs}\}} {n}. $$ \end{thm} \begin{proof} By lemmas~\ref{cor1}: $L(N'(i))\geq \frac {d_i} {d_i+1}L(N(i))\geq\frac 1 2 L(N(i)) = 1-\frac 1 2 c_i$. Therefore, $$ \frac 1 {d_i} \sumt_{v\in N(i)} \overline{\Rad}(v) = \diam(N'(i))+1-L(N'(i)) \leq \diam (N'(i))+\frac 1 2 c_i = \frac 1 2 c_i+2-\chi_{K_{d_i}}(N(i)), $$ where $ \chi_{K_{d_i}}(N(i)) = \begin{cases} 1 & \text{if $N(i) = K_{d_i}$} \\ 0 & \text{otherwise} \end{cases} $. Averaging by $i$ ends the proof. \end{proof} Let's prove a theorem about a connection between the average stress centrality and average shortest path length for geodetic graphs. \begin{thm}\label{thm1} If $G$ is geodetic, then $$ L(G) = 1+\frac {1} {n(n-1)} \sumt_{i\in V(G)}\Str(i). $$ \end{thm} \begin{proof} Let's define $$\chi_{st}(i) = \begin{cases} 1 & \text{if $i\neq s\neq t$ is the vertex of the shortest path between $s$ and $t$,}\\ 0 & \text{otherwise.} \end{cases}$$ Thus, $\Str(i) = \sumt_{s,t\in V(G)} \chi_{st}(i)$. If there exists the unique shortest path between any two vertices in $G$, then $\dist(s,t) = \sumt_{i\in V(G)} \chi_{st}(i)+1$ (otherwise, $\dist(s,t) \leq \sumt_{i\in V(G)} \chi_{st}(i)+1$). Therefore, for any $i$ $$ \sumt_{s,t\in V(G)} \dist(s,t) = 2|E|+\sumt_{s,t\in V(G),\,\dist(s,t)\geq 2} \dist(s,t) = 2|E|+\sumt_{s,t\in V(G),\,\dist(s,t)\geq 2} \Big(\sumt_{i\in V(G)} \chi_{st}(i)+1\Big) = $$ $$ = 2|E|+\sumt_{i\in V(G)}\Str(i)+n(n-1)-2|E|=\sumt_{i\in V(G)}\Str(i)+n(n-1). $$ \end{proof} \begin{cor} For any simple connected $G$ $$ L(G) \leq 1+\frac {1} {n(n-1)} \sumt_{i\in V(G)}\Str(i). $$ \end{cor} First let's proof theorem about a connection between average clustering coefficient and stress centrality and we will use it for a connection between the average shortest path length and the average local clustering coefficient further. \begin{thm}\label{thm2} If there is no pendant vertices in graph, then $$ C_{WS}(G)\geq 1- \frac 1 n \sumt_{i\in V(G)} \frac {Str(i)} {d_i(d_i-1)}. $$ \end{thm} \begin{proof} Note that $\forall j,k\in N(i): (j,k)\notin E(N(i))$ the shortest path between $j$ and $k$ is $j\rightarrow i\rightarrow k$. Therefore, $$ Str(i)\geq 2( \frac {d_i(d_i-1)} 2-|E(N(i))|), $$ $$ \frac 1 {d_i(d_i-1)} Str(i)\geq 1-c_i, $$ Averaging by $i$ $$ C_{WS}(G) \geq \frac 1 n \sumt_{i\in V(G)} (1- \frac {Str(i)} {d_i(d_i-1)}). $$ Note that for $diam(G) = 2$ holds an equality. \end{proof} \begin{cor} In the general case $$ C_{WS}(G)\geq 1- \frac 1 n \sumt_{i\in V(G)} \frac {Str(i)} {d_i(d_i-1)}-\frac {\text{number of pendant vertices}} n. $$ \end{cor} \begin{proof} Let's for pendant vertex define $\frac {Str(i)} {d_i(d_i-1)}$ as 0, then in the inequality $\frac 1 {d_i(d_i-1)} Str(i)\geq 1-c_i$ in the right side will be 1 and in the left side 0, thus if we add in the left side 1 for every pendant vertex, will be right equality. \end{proof} Now let's prove a theorem about a connection between the average shortest path length and the average local clustering coefficient for geodetic graphs. \begin{thm}\label{thm6} If $G$ is geodetic and $\forall i,j\in V(G)$ hold, if $d_i\leq d_j $ then $ \Str(i)\leq \Str(j)$, then $$ 1-C_{WS}(G) \leq\frac 1 n \sumt_{i\in V(G)} \frac {\big(L(G)-1\big)(n-1)} {d_i(d_i-1)}+\frac {\text{number of pendant vertices}} n. $$ \end{thm} \begin{proof} Let's re-numerate vertices such that $\forall i\leq j:d_i\leq d_j$. Then for $i\leq j$ hold $\Str(i)\leq \Str(j)$ and $d_i(d_i-1)\leq d_j(d_j-1)$. By theorems~\ref{thm1} and~\ref{thm2} for the case if there is no pendant vertices in graph $$\hspace{-150pt} 1-C_{WS}(G)\leq \frac 1 n \sumt_{i\in V(G)} \frac {Str(i)} {d_i(d_i-1)}\stackrel{\text{Chebyshev's sum inequality}}{\leq} $$ $$\hspace{70pt} \leq\Big(\frac 1 n \sumt_{i\in V(G)} {Str(i)}\Big) \Big(\frac 1 n \sumt_{i\in V(G)} \frac {1} {d_i(d_i-1)} \Big)= \frac 1 n\sumt_{i\in V(G)} \frac {\big(L(G)-1\big)(n-1)} {d_i(d_i-1)}. $$ For pendant vertices we should add $\frac {\text{number of pendant vertices}} n$ in the right side. Note that if there exist two vertices $i,j\in V(G)$ such that $d_i < d_j $ and $ \Str(i)< \Str(j)$ then the inequality in this theorem will be strict. \end{proof} Let's consider a star $G$ with $V(G) = n+1$ vertices. The star is geodetic graph. The central vertex has degree $n$, local clustering coefficient $c_i = 0$ and stress centrality $\Str(i) = n(n-1)$. Other vertices are pendant ($d_i = 1,\ c_i = 0,\ \Str(i) = 0$). Thus for this graph holds theorem~\ref{thm6}. $$ L(G) = \frac {n(2n-1)+n} {(n+1)n} = \frac {2n} {n+1},\quad C_{WS} = 0. $$ $$ 1-C_{WS}(G) = 1 = \frac {\frac {n-1} {n+1}\, n} {n(n-1)}+\frac n {n+1} = \frac 1 {n+1} \sumt_{i\in V(G)} \frac {\big(L(G)-1\big)n} {d_i(d_i-1)}+\frac {\text{number of pendant vertices}} {n+1}. $$ Thus, for this example holds equality. \begin{thebibliography}{99} \bibitem{Borgatti} Borgatti S. P., Everett M. G. A graph-theoretic perspective on centrality //Social networks. 2006. \bf{28}. №~4. 466--484. \bibitem{Kiss} Kiss C., Bichler M. Identification of influencers—measuring influence in customer networks //Decision Support Systems. 2008. \bf{46}. №~1. 233--253. \bibitem{Lee} Lee S. H. M., Cotte J., Noseworthy T. J. The role of network centrality in the flow of consumer influence //Journal of Consumer Psychology. 2010. \bf{20}. №~1. 66--77. \bibitem{Watts} Watts D. J., Strogatz S. H. Collective dynamics of ‘small-world’networks //nature. 1998. \bf{393}. №~6684. 440--442. \end{thebibliography} \end{document}
2412.02420v1
http://arxiv.org/abs/2412.02420v1
Fitting parameters of a Fokker-Planck-like equation with constraint
\documentclass[12pt]{article} \usepackage{amsmath,amsfonts,amssymb} \usepackage{mathrsfs} \usepackage{dsfont} \usepackage[utf8x]{inputenc} \usepackage[T1]{fontenc} \usepackage{lmodern} \usepackage{ucs} \usepackage{fullpage} \usepackage{float} \usepackage{graphicx} \usepackage{caption} \usepackage{array} \usepackage{multicol} \usepackage{multirow} \usepackage[format=hang, justification=raggedright]{caption} \usepackage{subfig} \usepackage{color} \usepackage{authblk} \usepackage{esint} \usepackage{enumitem,hyperref} \usepackage[normalem]{ulem} \usepackage{tikz} \usetikzlibrary{decorations} \usetikzlibrary{decorations.pathreplacing} \usetikzlibrary{decorations.pathmorphing} \usepackage{tkz-tab} \usetikzlibrary{shapes} \tikzset{t style/.style={style=solid}} \newtheorem{lemma}{Lemma}[section] \newtheorem{theo}{Theorem} \newtheorem{rmk}[lemma]{Remark} \newtheorem{prop}[lemma]{Proposition} \newtheorem{defin}[lemma]{Definition} \newtheorem{coro}[lemma]{Corollary} \newtheorem{example}[lemma]{Example} \makeatletter \def\namedlabel#1#2{\begingroup #2 \def\@currentlabel{#2} \phantomsection\label{#1}\endgroup } \makeatother \makeatletter \renewcommand*{\eqref}[1]{ \hyperref[{#1}]{\textup{\tagform@{\ref*{#1}}}}} \makeatother \newenvironment{Proof}{\noindent \abovedisplayskip = 0.5\abovedisplayskip \belowdisplayskip=\abovedisplayskip{\bfseries Proof. }}{\QED\medskip} \newenvironment{ProofOf}[1]{\noindent \abovedisplayskip = 0.5\abovedisplayskip \belowdisplayskip=\abovedisplayskip{\bfseries Proof of #1. }}{\QED\medskip} \newcommand{\rouge}[2]{{\textcolor{blue}{#1} \textcolor{red}{#2}}} \def\dbar{{\mathchar'26\mkern-12mu \mathrm d}} \newcommand{\hslashslash}{ \raisebox{.9ex}{ \scalebox{.7}{ \rotatebox[origin=c]{18}{$-$} } }} \newcommand{\QED}{\mbox{}\hfill \raisebox{-0.2pt}{\rule{5.6pt}{6pt}\rule{0pt}{0pt}} \medskip\par} \newcommand{\N}{\mathbb{N}} \newcommand{\Z}{\mathbb{Z}} \newcommand{\R}{\mathbb{R}} \newcommand{\dd}{\mathrm{d}} \newcommand{\J}{\mathbf{J}} \newcommand{\JJ}{\mathbb{J}} \newcommand{\JJJ}{\mathcal{J}} \renewcommand{\L}{\mathbf{L}} \newcommand{\D}{\mathrm{D}} \newcommand{\ds}{\displaystyle} \newcommand{\ud}{\, {\mathrm{d}}} \newcommand{\avex}[1]{\big\langle{#1} \big\rangle_{\mathbb X^d}} \newcommand{\avev}[1]{\big\langle{#1} \big\rangle_{\mathbb R^d}} \newcommand{\Td}{\mathbb T^d} \newcommand{\TumCellunit}{[cell_{\TumVar}]} \newcommand{\ImCellunit}{[cell_{\ImVar}]} \newcommand{\TumCellunitNoBracket}{cell_{\TumVar}} \newcommand{\ImCellunitNoBracket}{cell_{\ImVar}} \newcommand{\Lunit}{[L]} \newcommand{\lunit}{[l]} \newcommand{\tunit}{[t]} \newcommand{\micrometer}{\mu m} \newcommand{\Vunit}{\micrometer^3 \cdot day^{-1}} \newcommand{\Aunit}{day^{-1}} \newcommand{\Deltaunit}{\mu m^{-3} \cdot day^{-1}} \newcommand{\Chiunit}{mm^2 \cdot mmol^{-1} \cdot day^{-1}} \newcommand{\Dunit}{mm^2 \cdot day^{-1}} \newcommand{\Punit}{day^{-1}} \newcommand{\Sunit}{\ImCellunitNoBracket \cdot mm^{-3}} \newcommand{\Gammaunit}{day^{-1}} \newcommand{\Kunit}{mm^2 \cdot day^{day^{-1}}} \newcommand{\Sigmaunit}{mmol \cdot \TumCellunitNoBracket^{-1} \cdot \micrometer^{-3} \cdot day^{-1}} \newcommand{\Alphaunit}{\TumCellunitNoBracket \cdot \micrometer^{-3}} \newcommand{\Iunit}{mol} \newcommand{\TumVar}{n} \newcommand{\DivRateVar}{a} \newcommand{\DivProdVar}{k} \newcommand{\muzero}{\mu_0} \newcommand{\muone}{\mu} \newcommand{\ImVar}{c} \newcommand{\snormal}{\nu} \newcommand{\Snormal}{\nu} \newcommand{\ChemPoVar}{\phi} \newcommand{\mukc}{\mu_k^{\ImVar}} \newcommand{\mukonec}{\mu_{k-1}^{\ImVar}} \newcommand{\muktwoc}{\mu_{k-2}^{\ImVar}} \newcommand{\muc}{\mu_0^{\ImVar}} \newcommand{\TregVar}{r} \newcommand{\com}[1]{\textcolor{red}{#1}} \newcommand{\comb}[1]{\textcolor{blue}{#1}} \title {Fitting parameters of a Fokker-Planck-like equation with constraint} \markboth{Constrainted Fokker-Planck equation }{K. Atsou, T. Goudon, P.-E. Jabin} \author[1]{Kevin Atsou\thanks{{\tt [email protected]}} } \author[2]{Thierry~Goudon\thanks{{\tt [email protected]} }} \author[3]{Pierre-Emmanuel Jabin\thanks{{\tt [email protected]}} } \affil[1]{\small Pfizer } \affil[2]{\small Universit\'e C\^ote d'Azur, CNRS, LJAD } \affil[3]{\small Penn State University, Math. Dept.} \date{} \begin{document} \maketitle \abstract{We analyse a Fokker-Planck like equation, driven by a scalar parameter in order to reach an integral constraint. We exhibit criteria guaranteeing existence-uniqueness of a solution. We also provide counter-examples. This problem is motivated by an application to the immune control of tumor growth. } \vspace*{.5cm} {\small \noindent{\bf Keywords.} Fokker-Planck equation. Constrained elliptic problems. Tumor growth. Equilibrium phase. \vspace*{.5cm} \noindent{\bf Math.~Subject Classification.} 35B99. 92C50, 92C17, } \section{Introduction} We are interested in the following problem: given a certain ``confining'' (the meaning of which will be made precise later on) potential $\Phi:\mathbb R^N\to \mathbb R$, $\gamma, \lambda>0$ and two non negative functions $\delta, S:\mathbb R^N\to \mathbb R$, we consider the PDE \begin{equation}\label{def_FP} \gamma u -\mu\nabla\cdot(\nabla \Phi u)-\Delta u=\mu S\end{equation} where the parameter $\mu>0$ is selected so that the associated solution $u_\mu$ satisfies the constraint \begin{equation}\label{constraint} \ds\int_{\mathbb R^N} \delta u\ud x=\lambda>0.\end{equation} This question has been introduced in \cite{aabg_pub}, motivated by the modeling of the immune response to tumor growth, in order to explain equilibrium phases where the tumor is kept under control by the action of the immune cells. Numerical simulations show that the formation of equilibria, and thus the existence and stability of solutions of \eqref{def_FP}-\eqref{constraint}, is a quite robust phenomenom, see also \cite{aabg_fr}. However, the analysis provided in \cite{aabg_pub}, by means of the implicit function theorem, is restricted to small values of the constraint parameter $\lambda$. We wish to extend the existence-uniqueness of the pair $(\mu, u_\mu)$ satisfying \eqref{def_FP}-\eqref{constraint}, associated to any $\lambda\geq 0$. In fact, our analysis shows that this requires further ``compatibility'' conditions between the potential $\Phi$, the source $S$ and the contraint function $\delta$. We provide counter-examples explaining the role of these conditions. Our arguments, which are likely of interest beyond the original application to tumor-immune system interactions, rely on properties of the underlying Fokker-Planck operator, moment propagation and duality reasonings. The paper is organized as follows. First, Section~\ref{Sec:Motiv} motivates the problem \eqref{def_FP}-\eqref{constraint} by rapidly coming back to the modeling introduced in \cite{aabg_pub}. Next, Section~\ref{Sec:FP} discusses some properties of the Fokker-Planck operator $\nabla\cdot(\nabla\Phi u)+\Delta u$ which arises in \eqref{def_FP} and are crucial for the analysis. In Section~\ref{Sec:Asymp}, we study in details the behavior of the constraint functional $$\mathscr F:\mu\mapsto \int_{\mathbb R^N} \delta u_\mu\ud x$$ for small and large $\mu$'s. Finally, in Section~\ref{Sec:Monot} we analyze the monotonicity of the mapping $\mathscr F$, depending on assumptions on the data $ \Phi, S, \delta$. The analytical results are further illustrated by a few numerical examples. \section{Motivation} \label{Sec:Motiv} We remind the reader the modeling principles that lead to the problem \eqref{def_FP}-\eqref{constraint}. The earliest stages of tumor growth can be described through the evolution of the density of tumor cells $(t,z) \mapsto \TumVar(t,z)$: the integral $\int_a^b z \TumVar(t,z)\ud z$ gives the volume of the tumor occupied at time $t$ by tumor cells having their size $z$ in the interval $(a,b)$. It is governed by two phenomena: a natural growth, embodied in the rate $z\mapsto V(z)\geq 0$ and cell division mechanisms, where a cell with size $z'$ divides into cells with respective sizes $ z$ and $z'-z$. The latter depend on the frequency of division $z\mapsto a(z)$ and the size-distribution $k(z\vert z')$ from the division of a tumor cell with size $z'$. Therefore, without any further interaction, the evolution of the tumor cells obey the initial-boundary value problem: \begin{equation}\label{evol_n} \partial_t n+\partial_z (Vn)=Q(n), \qquad n(0,z)=n_0(z),\qquad n(t,0)=0,\end{equation} with \[ Q(n)(t,z) = -a(z)n(t,z) + \int_z^{\infty}a(z')k(z\vert z')n(t,z')\ud z '. \] A basic example of such cell- division operator is given by the binary division operator \[ Q(n)(t,z) = 4a(2z)n(t,2z) -a(z)n(t,z). \] In any cases, the assumption on the kernel $k$ are such that \begin{itemize} \item The total number of tumor cells is non decreasing \[ \dfrac{\ud}{\ud t}\ds\int_0^{\infty} n(t,z)\ud z \geq 0, \] \item The total mass of the tumor is non decreasing \[ \dfrac{\ud}{\ud t} \ds\int_0^{\infty} z n(t,z)\ud z = \ds\int_0^\infty V(z) n(t,z)\ud z \geq 0. \] \end{itemize} Note that the former is due to cell division, the later to the natural growth. A remarkable fact about this growth-division equation is the existence of an eigenpair $(\lambda, N)$, with $\lambda >0$ and $z\geq 0 \mapsto N(z)$ taking non negative values, that satisfy \begin{equation} \label{tumorgrowthEigenProblem} \left\{ \begin{array}{l} \partial_z (V N) -Q( N) + \lambda N = 0 \text{ for } z \geq 0 \\ \nonumber N(0) = 0, \qquad N(z)>0 \ \text{ for $z>0$},\qquad \ds\int_{0}^{+\infty} N(z)\ud z = 1. \end{array}\right.\end{equation} We refer the reader to \cite{DoGa, Mich1,PerBk} for precise assumptions and statements with proofs relying on suitable applications of the Krein-Rutman theorem. Note that, in the specific case where $a, V$ are constant and $Q$ is the binary division operator, we have $\lambda=a$ and the profile $N$ is explicitly known, \cite{Bac, PerBk,PeRy}. Dedicated numerical methods to compute the eigenpair are presented in \cite{aabg_fr}. Furthermore, it can be shown that this eigenstate drives the large time behavior of the Cauchy problem for \eqref{evol_n}: we have $ n(t,z) \sim_{t\to \infty} \nu_0 e^{\lambda t} \overline N(z) $ where $\nu_0$ is a constant determined by the initial condition, see \cite{DGL,Mich1,MMP}. The modeling of immune response adopted in \cite{aabg_pub} assumes that the displacement of the immune cells holds at a larger scale, described by a space variable $x\in \mathbb R^N$, while the tumor is attached at a given location $x_0$. The immune cells are activated from a reservoir of resting cells, are their motion is driven by diffusion and chemotaxis directed towards the tumor. The strength of both the activation and the directed drift depends on the total tumor mass \[\mu(t)=\ds\int_0^\infty z n(t,z)\ud z.\] Let $\Phi:\mathbb R^N\rightarrow \mathbb R$ be a potential, intended to create an attractive force towards the tumor location $x_0$ (and from now on, wlog, we suppose $x_0=0$). The time evolution of the concentration of immune cells $C:(0,\infty)\times \mathbb R^N\rightarrow [0,\infty)$ is governed by \[\partial_ t C-\Delta C-\mu\nabla\cdot(\nabla\Phi C) =\mu S-\gamma C, \] where $S: \mathbb R^N\rightarrow[0,\infty)$ describes the source of resting immune cells, and $\gamma>0$ is the natural death rate of the immune cells. The action of the immune cells on the tumor cells is taken into account through a death term \[-\ds\int_{\mathbb R^N} \delta(x)C(t,x)\ud x,\] in the right hand side of \eqref{evol_n} where $\delta: \mathbb R^N\rightarrow[0,\infty)$ is intended to describe the killing effects on the tumor cells; it acts as a mollified delta-Dirac at $x_0=0$. Performing simulations of the coupled problem, one observes the formation of an equilibrium phase, with a residual tumor, having a positive mass, controlled by the action of the immune cells. Such an equilibrium can be explained by coming back to the eigenproblem \eqref{tumorgrowthEigenProblem}: the death term induced by the immune cells is expected to counterbalance the natural growth rate of the cell-division equation. The other way around, we expect that $C(t,x)$ tends to $u(x) $, solution of \eqref{def_FP}-\eqref{constraint} as $t$ goes to $\infty$, with $\ell=\lambda$, the eigenvalue determined by \eqref{tumorgrowthEigenProblem}. We refer the reader to \cite{aabg_pub, aabg_fr} for numerical illustration of such a behavior, which seems very robust. Moreover, this interpretation of the equilibrium phase by means of an eigenvalue problem permits to compute a priori the final mass of the tumor, given the biological parameters \cite{aabg_fr}. Unfortunately, a direct reasoning justifies this interpretation only for small values of $\lambda$. \begin{theo}\label{theo} If $\ell>0$ is small enough, there exists a unique $\mu(\ell)>0$ such that $u_{\mu(\ell)} $, solution of the stationary equation \eqref{def_FP}, satisfies \eqref{constraint}. \end{theo} \noindent {\bf Proof.} The framework slightly differs from \cite{aabg_pub} which deals with the problem set in a bounded domain, endowed with appropriate boundary conditions. The argument uses the results in Proposition~\ref{prop:LxM}, detailed below. We are searching for the zeroes of the mapping \[\mathscr X:(\ell,\mu)\in [0,\infty)\times [0,\infty)\longmapsto \ds\int _\Omega \delta u_{\mu} \ud x-\ell\] where $u_{\mu} $ is the solution of \eqref{def_FP} associated to $\mu$ and knowing that $\mathscr X(0,0)=0$, since $u_{0}=0$. We have $\partial_{\mu} \mathscr X(\ell,\mu)=\int _\Omega \delta u'_{\mu} \ud x$, with $u'_{\mu}$, solution of \[ \gamma u'-\Delta u'-\mu\nabla_x\cdot (u'\nabla_x\Phi)= S +\nabla_x\cdot (u_{\mu}\nabla_x\Phi). \] Since $u_0=0$ and $S\geq 0$, we get $u'_{0}>0$ (see Proposition~\ref{prop:LxM} below). It follows that $\partial_{\mu} \mathscr X(0,0)=\int _{\mathbb R^N} \delta u'_{0} \ud x>0$. The implicit function theorem tells us that there exists $\ell _\star>0$ and a mapping $\mu:\ell\in [0,\ell_\star)\mapsto\mu(\ell)$ such that $\mathscr X(\ell,\mu(\ell))=0$ holds for any $\ell\in [0,\ell_\star)$, which means that $u_\ell$ satisfies \eqref{def_FP}-\eqref{constraint}. Observe that $$ \begin{array}{l} \partial_{\ell} \mathscr X(\ell,\mu(\ell))+ \mu'(\ell)\partial_{\mu} \mathscr X(\ell,\mu(\ell)) =-1 +\mu'(\ell)\partial_{\mu} \mathscr X(\ell,\mu(\ell))=0\end{array}$$ holds with $\partial_{\mu} \mathscr X(0,0)>0$. Hence, $\ell\mapsto \mu(\ell)$ is increasing on the neighborood of $\ell=0$, and it thus takes positive values. Note that the argument cannot be extended for any $\ell$, since we do not have a direct knowledge on the sign of $\nabla_x\cdot (u_{\mu}\nabla_x\Phi)$ for $\mu\neq 0$. However, the proof do not use the confining feature of the potential $\Phi$. Hence, we are going to develop a viewpoint that further exploits these properties. \QED \section{Fundamental properties of the operator $L_\mu$} \label{Sec:FP} Let us make the following assumption on the potential $\Phi$: \begin{equation} \label{hPhi1} \begin{array}{l} \textrm{for any $x\in \mathbb R^N$, we have $\Phi(x)\geq 0$}, \\ \textrm{ and, for any $\mu>0$, $x\mapsto M_\mu(x)=e^{-\mu \Phi(x)}\in L^1(\mathbb R^N)$}. \end{array} \end{equation} As a matter of fact, the latter integrability property is guaranteed by the following strengthened convexity condition: assuming $\Phi\in C^2$, \begin{equation} \label{hPhi2} \begin{array}{l} \textrm{$\nabla \Phi(0)=0$ and there exists a constant $\Lambda>0$ such that } \\ \textrm{the hessian matrix of $\Phi$ satisfies, for any $x\in \mathbb R^N$, $D^2_{ij}\Phi(x)\geq \Lambda\mathbb I$.} \end{array} \end{equation} These conditions describe the confining feature of the potential, having an attractive effect towards $x=0$, which is a strict global minimizor of the potential. Then, we introduce the Fokker-Planck operator \begin{equation} \label{def_fp} L_\mu u =\mu\nabla\cdot(\nabla\Phi u)+\Delta u \end{equation} and its adjoint operator (defined with the standard $L^2$ inner product) \begin{equation} \label{def_fpst} L_\mu^\star \psi =-\mu\nabla\Phi \cdot\nabla \psi+\Delta \psi. \end{equation} It is convenient to recast these operators by making the function $M_\mu$ appear \[ L_\mu u=\nabla\cdot\left(M_\mu \nabla\left(\ds\frac u{M_\mu}\right)\right), \qquad L_\mu^\star \psi=\ds\frac{1}{M_\mu}\nabla\cdot\left( M_\mu \nabla\psi \right). \] Accordingly, we observe that \begin{equation}\label{vari_L} -\ds\int_{\mathrm R^N} \ds\frac u{M_\mu}L_\mu u\ud x =\ds\int_{\mathrm R^N} M_\mu \left|\nabla\left(\ds\frac u{M_\mu}\right) \right|^2\ud x \geq 0. \end{equation} Note that, owing to \eqref{hPhi2}, the following Sobolev inequality \begin{equation}\label{coer_L}\begin{array}{lll} \ds\int_{\mathrm R^N} M_\mu \left|\nabla\left(\ds\frac u{M_\mu}\right) \right|^2 \langle M_\mu\rangle\ud x&\geq& 2\Lambda\mu \ds\int_{\mathrm R^N} \left| u-\langle u\rangle \ds\frac{M_\mu}{\langle M_\mu\rangle} \right|^2\ds\frac{\langle M_\mu\rangle\ud x}{M_\mu} \\&\geq& 2\Lambda\mu\left( \ds\int_{\mathrm R^N} \left| u-\langle u\rangle \ds\frac{M_\mu}{\langle M_\mu\rangle} \right|\ud x\right)^2 , \end{array}\end{equation} holds, where $\langle u\rangle=\int_{\mathbb R^N} u\ud x$, see \cite[condition (A2), Corollary 2.18]{AMTU}. Similarly, we have \[ -\ds\int_{\mathrm R^N} M_\mu \psi L^\star_\mu \psi\ud x =\ds\int_{\mathrm R^N} M_\mu \left| \nabla\psi \right|^2\ud x \geq 0. \] \begin{prop} \label{prop:LxM} The following assertions hold: \begin{itemize} \item[i)] $\mathrm{Ker}(L_\mu)=\mathrm{Span}(M_\mu)$ and $\mathrm{Ker}(L^\star_\mu)=\mathrm{Span}(\mathbf 1)$, \item[ii)] Let $\gamma>0$. For any $S\in L^2(\mathbb R^N,\frac{\ud x}{M_\mu})$, there exists a unique solution $u\in L^2(\mathbb R^N,\frac{\ud x}{M_\mu})$, with $\nabla\frac u{M_\mu}\in L^2(\mathbb R^N,M_\mu \ud x)$ of $(\gamma\mathbb I-L_\mu)u=S$. Moreover, if $S\geq 0$, then $u\geq 0$ too. \item[iii)] Let $\gamma>0$. For any $\delta \in L^2(\mathbb R^N,M_\mu \ud x)$, there exists a unique solution $\psi\in L^2(\mathbb R^N,M_\mu \ud x)$, with $\nabla\psi\in L^2(\mathbb R^N,M_\mu \ud x)$ of $(\gamma\mathbb I-L^\star_\mu)\psi=\delta$. Moreover, if $\delta \geq 0$, then $\psi\geq 0$ too. \end{itemize} \end{prop} \noindent {\bf Proof.} The first item is a direct consequence of \eqref{vari_L} and \eqref{coer_L}. Next, we simply apply the Lax-Milgram theorem (or, in the present context the Riesz theorem) in the Hilbert space \[ H=\Big\{u\in L^2(\mathbb R^N,\frac{\ud x}{M_\mu}),\ \nabla \ds\frac u{M_\mu}\in L^2(\mathbb R^N,M_\mu \ud x)\Big\}\] to solve the variational problem: to find $u\in H$, such that, for any $v\in H$, we have \[ \gamma \ds\int_{\mathbb R^N}uv \ds\frac{\ud x}{M_\mu}+\ds\int_{\mathrm R^N} M_\mu \left|\nabla\left(\ds\frac u{M_\mu}\right) \right|^2\ud x=\ds\int_{\mathbb R^N} Sv\ds\frac{\ud x}{M_\mu}.\] We obtain the sign property by using $v=u_-$ as trial function in the variational formulation: it yields \[ \gamma \ds\int_{\mathbb R^N} u^2_- \ds\frac{\ud x}{M_\mu}+\ds\int_{\mathrm R^N} M_\mu \left|\nabla\left(\ds\frac {u_-}{M_\mu}\right) \right|^2\ud x=\ds\int_{\mathbb R^N} Su_-\ds\frac{\ud x}{M_\mu}\leq 0\] when $S$ takes non negative values. It implies $u_-=0$ a. e. A similar argument applies readily to the adjoint problem. \QED \section{Asymptotic behavior of $ \mu\mapsto \mathscr F(\mu)$} \label{Sec:Asymp} \begin{lemma} Suppose \eqref{hPhi2}. Let $(1+x^2)S\in L^1(\mathbb R^N)$ with $S\geq 0$ and $S\in L^2(\mathbb R^N, \frac{\ud x}{M_\mu})$ for any $\mu>0$. Let $\delta:\mathbb R^N\to [0,\infty)$ be a non negative function such that $\delta\in L^2(\mathbb R^N, M_\mu)$ for any $\mu>0$. We suppose that there exists $\eta, r>0$ such that $\delta (x)\geq \eta$ on $B(0,r)$. Let $u_\mu$ given by Proposition~\ref{prop:LxM}-ii) for right hand side $\mu S$. Then, $\lim_{\mu\to \infty}\mathscr F(\mu)=+\infty$. \end{lemma} \noindent {\bf Proof.} By integrating the equation $(\gamma -L_\mu)u_\mu=\mu S$, we get \[ \gamma\ds\int_{\mathbb R^N} u_\mu\ud x=\mu \ds\int_{\mathbb R^N} S\ud x.\] Similarly, considering the second moment and using integration by parts, we are led to \[ \begin{array}{lll} \gamma \ds\int_{\mathbb R^N} x^2u_\mu\ud x&=&\mu \ds\int_{\mathbb R^N} x^2S\ud x +\ds\int_{\mathbb R^N} x^2\nabla\cdot(\nabla u_\mu+\mu u_\mu\nabla \Phi )\ud x \\ &=& \mu \ds\int_{\mathbb R^N} x^2S\ud x +2N \ds\int_{\mathbb R^N}u_\mu\ud x-2\mu \ds\int_{\mathbb R^N} x\cdot \nabla \Phi u_\mu \ud x \\ &\leq & \mu \ds\int_{\mathbb R^N} (2N+x^2) S\ud x-2\Lambda \mu \ds\int_{\mathbb R^N} x^2u_\mu\ud x, \end{array}\] since \eqref{hPhi2} implies $x\cdot \nabla\Phi(x)=x\cdot (\nabla\Phi(x)-\nabla\Phi(0))\geq \Lambda x^2$. It follows that the second moment is bounded uniformly wrt $\mu>0$ since \[ \ds\int_{\mathbb R^N} x^2u_\mu\ud x\leq \ds\frac{\mu}{\gamma + 2\Lambda\mu} \ds\int_{\mathbb R^N} (2N+x^2) S\ud x \leq \ds\frac{1}{\Lambda} \ds\int_{\mathbb R^N} (2N+x^2) S\ud x.\] We now split, for $r>0$, \[\begin{array}{lll} \ds\int_{\mathbb R^N} \delta u_\mu\ud x&=&\ds\int_{|x|\leq r} \delta u_\mu\ud x+\ds\int_{|x|>r} \delta u_\mu\ud x \\ & \geq & \eta \ds\int_{|x|\leq r} u_\mu\ud x=\eta \left(\ds\int_{\mathbb R^N} u_\mu\ud x- \ds\int_{|x|> r} u_\mu\ud x\right) \\ & \geq & \eta \mu \ds\int_{\mathbb R^N} S\ud x- \ds\frac{\eta}{r^2}\ds\int_{\mathbb R^N} x^2 u_\mu\ud x \\&\geq& \eta \mu \ds\int_{\mathbb R^N} S\ud x- \ds\frac{\eta}{\Lambda r^2}\ds\int_{\mathbb R^N} (2N+x^2) S\ud x \end{array}\] where the RHS tends to $+\infty$ as $\mu\to \infty$. \QED In fact, we can make the behavior for large $\mu$'s more precise, by appealing to the Laplace method, which can be summarized in the following claim \cite[Theorem~15.2.2]{BaSi}. \begin{lemma} Let $f:\mathbb R^N\to \mathbb R$ be a continuous function such that $f(0)\neq 0$. Then, as $\mu$ goes to $+\infty$, $\int_{\mathbb R^N} f M_{\mu}\ud x$ is equivalent to \[\ds\frac{f(0)}{\mu^{N/2}}\sqrt{ \ds\frac{(2\pi)^N}{\mathrm{det}(\mathrm D^2\Phi(0))}} .\] \end{lemma} Let us set $$ m(\mu)=\ds\int_{\mathbb R^N} M_\mu\ud x, \qquad \varsigma(\mu)=\ds\int_{\mathbb R^N} \ds\frac{S^2}{M_\mu}\ud x$$ and introduce the following rescaling $$\tilde u_\mu(x)=\ds\frac{u_\mu(x)}{\mu\sqrt{\varsigma(\mu)}}.$$ The latter satisfies \[\gamma \tilde u_\mu-\nabla\cdot\left(M_\mu\nabla \ds\frac{\tilde u_\mu}{M_\mu}\right)=\ds\frac{S}{\sqrt{\varsigma(\mu)}}.\] In turn, we obtain the following estimate \[\gamma \ds\int_{\mathbb R^N} \ds\frac{|\tilde u_\mu|^2}{M_\mu}\ud x +2\ds\int_{\mathbb R^N} M_\mu \left|\nabla \ds\frac{\tilde u_\mu}{M_\mu}\right|^2 \ud x \leq 1,\] together with \[\langle \tilde u_\mu \rangle = \ds\frac{\langle S\rangle}{\sqrt{\varsigma(\mu)}}.\] Owing to \eqref{coer_L}, it leads to \[ \ds\int_{\mathbb R^N} \left |\tilde u_\mu-\ds\frac{\langle S\rangle}{\sqrt{\varsigma(\mu)}} \ds\frac{M_\mu}{m(\mu)}\right|\ud x \leq \ds\frac1{2\sqrt{\Lambda\mu}}\xrightarrow [\mu\to \infty]{}0. \] Therefore, assuming $\delta\in L^\infty(\mathbb R^N)$, $\delta $ continuous with $\delta (0)\neq 0$, we get \[\begin{array}{lll} \mathscr F(\mu)&=&\mu \sqrt{\varsigma(\mu)} \ds\int_{\mathbb R^N} \tilde u_\mu \delta \ud x \underset{\mu \to \infty}{ \sim} \mu \sqrt{\varsigma(\mu)} \ds\int_{\mathbb R^N} \ds\frac{\langle S\rangle}{\sqrt{\varsigma(\mu)}} \ds\frac{M_\mu}{m(\mu)} \delta \ud x \\&\underset{\mu \to \infty}{ \sim}& \langle S\rangle \ds\frac{\mu}{m(\mu)} \ds\frac{\delta (0)}{\mu^{N/2}}\sqrt{\ds\frac{(2\pi)^N}{\mathrm{det}(\mathrm D^2\Phi(0))}} \underset{\mu \to \infty}{ \sim} \mu \delta (0)\langle S\rangle.\end{array} \] Since the function $\mu\mapsto \mathscr F(\mu)$ is continuous, with $\mathscr F(0)=0$, we deduce the following existence result. \begin{coro} Suppose $\delta\in L^\infty(\mathbb R^N)$, $\delta $ continuous with $\delta (0)\neq 0$. For any $\ell\geq 0$, there exists at least a $\mu\in [0,\infty)$ such that \eqref{def_FP}-\eqref{constraint} holds. \end{coro} \section{Monotonicity} \label{Sec:Monot} We are going to show that, under certain compatibility conditions, the function $\mathscr F$ is increasing; to this end we use a duality argument. We introduce the solution $\psi_\mu$ of \[ (\gamma-L^\star_\mu) \psi_\mu= \delta\] so that $\mathscr F(\mu) $ recasts as \[ \ds\int_{\mathbb R^N} u_\mu\delta\ud x = \ds\int_{\mathbb R^N} u_\mu(\gamma-L^\star_\mu) \psi_\mu\ud x =\ds\int_{\mathbb R^N} (\gamma-L^\star_\mu) u_\mu\psi_\mu\ud x =\mu\ds\int_{\mathbb R^N} S\psi_\mu\ud x. \] Accordingly, showing the monotonicity of $\mathscr F$ reduces to investigating the sign of \begin{equation}\label{deriv} \ds\frac{\ud}{\ud \mu} \mathscr F(\mu) =\ds\int_{\mathbb R^N} S\psi_\mu\ud x+\mu\ds\int_{\mathbb R^N} S\psi'_\mu\ud x\end{equation} where $\psi'_\mu$ satisfies \[(\gamma-L^\star_\mu) \psi'_\mu=-\nabla\Phi\cdot \nabla\psi_\mu.\] The data $S$ and $\delta$ being non negative, the first integral in the right hand side of \eqref{deriv} is non negative. We are going to show that the second term is equally non negative, under appropriate assumptions on the data. \subsection{Problem in radial symmetry} From now on, we assume that all data $S,\delta, \Phi$ are \emph{radially symmetric}. In turn, $M_\mu$ and the solutions of the associated PDE are also radially symmetric. We write the equation in radial coordinates: we get \[ \gamma u_\mu -\mu \partial_r (u_\mu\partial_ r \Phi)-\mu\ds\frac{N-1}{r}\partial_r \Phi u_\mu-\ds\frac{1}{r^{N-1}}\partial_r(r^{N-1}\partial_r u_\mu)=\mu S.\] It casts as \[ \gamma u_\mu -\ds\frac{1}{r^{N-1}}\partial_r \left(M_\mu r^{N-1} \partial_r\Big(\ds\frac{u_\mu}{M_\mu}\Big)\right) =\mu S.\] For the adjoint equation, we obtain \begin{equation}\label{adjrad}\begin{array}{l} \gamma \psi_\mu +\mu \partial_r \Phi\partial_r \psi_\mu -\ds\frac{1}{r^{N-1}}\partial_r(r^{N-1}\partial_ r \psi_\mu)=\delta \\ \qquad\qquad = \gamma \psi_\mu - \ds\frac{1}{r^{N-1} M_\mu}\partial_r(r^{N-1}M_\mu\partial_r \psi_\mu). \end{array}\end{equation} Let us set $\chi_\mu=\partial_r\psi_\mu$. It satisfies \[\begin{array}{l} \left(\gamma + \ds\frac{N-1}{r^2}+\mu \partial^2_r \Phi\right)\chi_\mu +\mu \partial_r \Phi\partial_r \chi_\mu -\ds\frac{1}{r^{N-1}}\partial_r(r^{N-1}\partial_ r \chi_\mu)=\partial_r\delta \\\qquad\qquad =\left(\gamma + \ds\frac{N-1}{r^2}+\mu \partial^2_r \Phi\right)\chi_\mu -\ds\frac{1}{r^{N-1}M_\mu }\partial_r(r^{N-1}M_\mu \partial_ r \chi_\mu).\end{array}\] We assume the convexity/monotonicity properties \begin{equation}\label{cx}\partial_r \Phi\geq 0,\qquad \partial^2_r \Phi\geq 0,\qquad \partial_r\delta\leq 0.\end{equation} Under these assumptions, we obtain $$\chi_\mu=\partial_r\psi_\mu \leq 0.$$ Now, we go back to \eqref{adjrad}, which yields \[ \begin{array}{l} \gamma\psi'_\mu +\mu \partial_r \Phi\partial_r \psi'_\mu -\ds\frac{1}{r^{N-1}}\partial_r(r^{N-1}\partial_ r \psi'_\mu)=- \underbrace{\partial_r \Phi}_{\geq 0} \underbrace{\partial_r \psi_\mu }_{\leq 0} \\ \qquad\qquad = \gamma \psi'_\mu - \ds\frac{1}{r^{N-1} M_\mu}\partial_r(r^{N-1}M_\mu\partial_r \psi'_\mu).\end{array}\] The right hand side thus satisfies $-\partial_r \Phi\partial_r \psi_\mu\geq 0$ so that $ \psi'_\mu\geq 0$. Coming back to \eqref{deriv}, we conclude that $\mathscr F$ is non decreasing. We need to slightly improve the result, requiring further regularity on $\delta, \Phi$, say $\delta\in C^{1}$, $\Phi\in C^2$, with $\delta, \Phi$ not identically 0. We can thus apply the strong maximum principle \cite[Section~3.2]{GT} which tells us that $\psi'_\mu>0$ on $(0,\infty)$. Our findings recap as follows. \begin{theo}\label{mainth} In the radially symmetric framework, we suppose that $S, \delta\in C^1$ and $\Phi\in C^2$ take non negative values, but are not identically 0, and that \eqref{cx} is fulfilled. Then, $\mathscr F$ is increasing and, for any $\ell\geq 0$, the problem \eqref{def_FP}-\eqref{constraint} admits a unique solution $0<\mu <\infty$. \end{theo} The assumptions of radial symmetry together with \eqref{cx} are quite natural and relevant for the presented modeling: the tumor being located at $x=0$, the action of the immune cells, embodied into $\delta$, is centred on this position and the chemotactic potential $\Phi$ drives the immune cells towards the tumoral centre. Let us detail a simple example showing that these assumptions are also technically important. We consider the simplest confining potential $\Phi(x)=x^2$ and we compute the first even moments of the solutions of \eqref{def_FP}: we have already seen that $\int_{\mathbb R^N} u_\mu\ud x=\mu \int_{\mathbb R^N} S\ud x$; next we have \[\begin{array}{lll} \gamma\ds\int_{\mathbb R^N} |x|^2 u_\mu\ud x&=& \mu \ds\int_{\mathbb R^N} |x|^2 S\ud x +2N \ds\int_{\mathbb R^N} u_\mu \ud x- 2\mu \ds\int_{\mathbb R^N} u_\mu x\cdot\nabla\Phi\ud x\\ &=& \mu \ds\int_{\mathbb R^N} |x|^2S\ud x +2N\mu \ds\int_{\mathbb R^N} S \ud x-4 \mu \ds\int_{\mathbb R^N} |x|^2 u_\mu\ud x \end{array}\] so that \[ \ds\int_{\mathbb R^N} |x|^2 u_\mu\ud x=\ds\frac{\mu}{\gamma + 4\mu} \ds\int_{\mathbb R^N} (|x|^2+2N) S\ud x. \] It follows that $\mu\mapsto \int_{\mathbb R^N} x^2 u_\mu\ud x$ is an increasing function. We turn to \[ \gamma\ds\int_{\mathbb R^N} |x|^4 u_\mu\ud x= \mu \ds\int_{\mathbb R^N} |x|^4 S\ud x +(4N+8) \ds\int_{\mathbb R^N} |x|^2 u_\mu \ud x- 8\mu \ds\int_{\mathbb R^N} |x|^4 u_\mu \ud x. \] It leads to the expression \[ \ds\int_{\mathbb R^N} |x|^4 u_\mu\ud x=\underbrace{A\frac{\mu}{\gamma + 8\mu}+ B\frac{1}{\gamma + 8\mu}\frac{\mu}{\gamma + 4\mu}}_{:=f(\mu)}\] with \[A=\ds\int_{\mathbb R^N} |x|^4 S\ud x,\qquad B=(4N+8)\ds\int_{\mathbb R^N} (|x|^2+2N) S\ud x .\] The forth momentum is not necessarily a monotone function of $\mu$ since \[ f'(\mu)=\ds\frac{1}{(\gamma + 8\mu)^2}\left(\gamma A+B\ds\frac{\gamma}{\gamma+4\mu} -4B\ds\frac{\mu(\gamma+8\mu)}{(\gamma+4\mu)^2}\right)\] might change sign. But, in this example, $\delta(x)=|x|^4$ vanishes at $x=0$, contradicting the modelling assumptions. \subsection{Numerical illustrations} Dealing with the radially symmetric problem, the finite elements framework is a reliable way to get rid of the singularity at $r=0$. For realizing simulations, we consider the problem set on the slab $[0,1]$: since the phenomena are naturally quite concentrated next to the origin, we expect that this does not influence too much the final results (by the way, we indeed do not observe significant differences when imposing Dirichlet or Neumann conditions at $r=1$ or extending the domain for larger $r$'s). We introduce a discretization of $ [0,1]$ with $N+1$ points \[r_0=0<r_1=h<...<r_N=Nh=1,\qquad h=1/N.\] We introduce the associated $\mathbb P_1$ basis functions, $\chi_1,...,\chi_{N-1}$: \[\chi_j(r)=\ds\frac{r-(j-1)h}h\mathbf 1_{(j-1)h<r< jh}+ \ds\frac{(j+1)h-r}h\mathbf 1_{jh\leq r<(j+1)h},\quad \chi_0(r)=- \ds\frac{r-h}h\mathbf 1_{0\leq r<h}.\] Then, we define the matrices with coefficients \[ M_{ij}=\ds\int_0^1\chi'_i(r)\chi'_j(r) r^{n-1}\ud r,\quad A_{ij}=\ds\int_0^1\chi_i(r)\chi_j(r) r^{n-1}\ud r.\] Given the potential $\Phi$, we also define the centered difference matrix $C$, which is skew-symmetric with \[ C_{j,j+1}=\ds\frac12((j+1)h)^{n-1}\Phi'((j+1)h). \] Then, for a given source term $S$, we define the vector with components \[S(j)=\ds\int_{ih}^{(i+1)h} S(r)r^{n-1}\ud r.\] Eventually, we solve the linear system \[(\gamma A -\mu C+M)U=\mu S,\] and we compute the associated discrete version of $F(\mu)=\int_0^1 \delta u_\mu(r)r^{n-1}\ud r$. We perform the simulation with a source term given by \[S(r)=\mathbf 1_{.3\leq r\leq .5}\] which, for the application to tumor-immune system interactions, corresponds to a located reservoir of resting immune cells, for instance a blood vessel or a lymph node. We set $n=3$ and $\gamma=1$. We start with simulations of the expected situation, with a confining potential pointing towards the origin $\Phi(r)=2r^2$, and a constraint kernel peaked at the origin $$\delta(r)=\frac{e^{-r^2/\epsilon}}{(4\pi\epsilon)^{n/2}},\qquad \epsilon=10^{-3}.$$ Fig.~\ref{F_OK1} represents the profile of the solutions for relatively small values of $\mu$: as $\mu$ increases, the value at $r=0$ increases and the solution concentrates near $r=0$. We numerically check that $\mu\mapsto F(\mu)$ is non decreasing, see Fig.~\ref{F_OK2}; the solution $u_\mu$ becomes highly concentrated to $r=0$, which thus requires a very fine mesh to resolve the solution when $\mu$ becomes large. \begin{figure}[!hbtp] \begin{center} \includegraphics[height=6cm]{./Fig/CasFavo_SolMuPt.eps} \caption{Solutions $r\mapsto u_\mu(r)$ for 10 equidistant values of $\mu$ in $[0,10]$. As $\mu$ increases the solution takes larger value at the origin and presents a stiffer profile for transient radius \label{F_OK1}} \end{center} \end{figure} \begin{figure}[!hbtp] \begin{center} \includegraphics[height=6cm]{./Fig/FMu_OK.eps} \caption{ $\mu\mapsto F(\mu)$ for $\mu$ up to $10^7$. \label{F_OK2}} \end{center} \end{figure} Simulations of the counter-example detailed in the previous section are displayed in Fig.~\ref{F_delta4}: we just modify $\delta$ into \[ \delta(r)=\epsilon r^4,\qquad \epsilon=10^{-3}.\] Then the function $\mu\mapsto F(\mu)$ looses its monotony, but it is still increasing at $\mu=0$ and for large values of $\mu$. \begin{figure}[!hbtp] \begin{center} \includegraphics[height=4cm]{./Fig/Delta_r4.eps}~\includegraphics[height=4cm]{./Fig/Delta_r4Bis.eps}~\includegraphics[height=4cm]{./Fig/UMu_Delta_r4.eps} \caption{Profile of $\mu\mapsto F(\mu)$ for $\mu$ up to $100$ (left), up to $500$ (middle), and snapshots of the corresponding solution profiles $r\mapsto u_\mu(r)$ (right) \label{F_delta4}} \end{center} \end{figure} Next, we consider a situation which can find some physical motivation: the potential is given by \[ \Phi(r)=2r^2\times (r-r_0)^2,\qquad r_0=.2.\] It has a quite flat profile between the two minima $r=0$ and $r=.2$. Note that \eqref{hPhi2} is not satisfied. It describes a defect of the attractivity of the immune cells towards the tumor, due either to the geometry of the tissues around the tumor, or to pro-tumoral effects that reduce the efficacy of the immune response. Another pro-tumoral effect can result in a reduction of the capacity of the immune cells to eliminate tumor cells, that we traduce by shifting the kernel $\delta$ \[\delta(r) = \frac{e^{-(r-r_1)^2/\epsilon}}{(4\pi\epsilon)^{n/2}},\qquad \epsilon=10^{-3},\qquad r_1=.05.\] Note that it still keeps a significantly positive value at $r=0$, see Fig.~\ref{F_Bad}-Top Left. We indeed observe that $\mu\mapsto F(\mu)$ does not tend to infinity as $\mu \to \infty$, and the monotonicity is compromised, see Fig.~\ref{F_Bad}-Bottom. The solution $u_\mu$ tends to form a high peak in the interior of the domain, thus far from the tumor, Fig.~\ref{F_Bad}-Top Right. \begin{figure}[!hbtp] \begin{center} \includegraphics[height=4cm]{./Fig/Delta_Bad.eps}~\includegraphics[height=4cm]{./Fig/Bad_UMu.eps} \\ \includegraphics[height=4cm]{./Fig/F_Bad.eps}~\includegraphics[height=4cm]{./Fig/FMu_Bad2.eps} \caption{Profile of $r\mapsto \delta(r)$ (top-left), profile of $\mu\mapsto F(\mu)$ for $\mu$ up to $10^4$ (bottom-left) and $15\cdot 10^4$ (bottom-right), the solution profiles $r\mapsto u_\mu(r)$ for several $\mu$ up to $10^4$ (top-right) \label{F_Bad}} \end{center} \end{figure} Eventually, we challenge the condition that $\delta$ takes positive value near the origine: we come back to the quadratic potential, but now we work with \[\delta(r) = \frac{e^{-(r-r_1)^2/\epsilon}}{(4\pi\epsilon)^{n/2}},\qquad \epsilon=10^{-3},\qquad r_1=.1\] which (almost) vanishes at $r=0$. Results are reported in Fig.~\ref{F_Bad2}. Again, we observe that $\mu\mapsto F(\mu)$ is not monotone and does not tend to $\infty$ (it seems to be decaying for large $\mu$'s), at least as far as it can be numerically checked. \begin{figure}[!hbtp] \begin{center} \includegraphics[height=4cm]{./Fig/Delta_Bad2.eps}~\includegraphics[height=4cm]{./Fig/Bad_UMu2.eps}~ \includegraphics[height=4cm]{./Fig/F_Bad2.eps} \caption{Profile of $r\mapsto \delta(r)$ (left), profile of $\mu\mapsto F(\mu)$ for $\mu$ up to $2500$ (bottom-left) (bottom-right), snapshot on the corresponding solution profiles $r\mapsto u_\mu(r)$ (top-right) \label{F_Bad2}} \end{center} \end{figure} These numerical experiments highlight the role of the assumptions on both the potential, and the constraint kernel. Coming back to the motivation from the modeling of tumor-immune system interactions, these findings shed light on the role of the pro-tumor mechanisms which not only may promote tumor proliferation, but can also reduce the efficacy of the immune response, and eventually allow the tumor to escape to the control of the immune system \cite{aabg_pone}. \section*{Ackowledgements} This research is supported by the CNRS program ``International Emerging Actions''. We acknowledge the support of the Math.~Dept. at Penn State University. \bibliography{AGJ} \bibliographystyle{plain} \end{document}
2412.02523v1
http://arxiv.org/abs/2412.02523v1
Density formulas for $p$-adically bounded primes for hypergeometric series with rational and quadratic irrational parameters
\documentclass[12pt]{amsart} \usepackage{amssymb,latexsym,amsmath,amsthm,amscd} \usepackage{setspace} \usepackage[active]{srcltx} \usepackage[all]{xy} \xyoption{arc} \usepackage[left=3cm,top=2cm,right=3cm,bottom = 2cm]{geometry} \usepackage{graphicx} \usepackage[usenames,dvipsnames]{color} \usepackage{charter} \theoremstyle{plain} \newtheorem{thm}{Theorem}[section] \newtheorem{lem}[thm]{Lemma} \newtheorem{prop}[thm]{Proposition} \newtheorem{cor}[thm]{Corollary} \newtheorem{conj}[thm]{Conjecture} \newtheorem{exer}[thm]{Exercise} \theoremstyle{definition} \newtheorem{dfn}[thm]{Definition} \newtheorem{ex}[thm]{Example} \newtheorem{claim}[thm]{Claim} \newtheorem{ques}[thm]{Question} \theoremstyle{remark} \newtheorem{rmk}[thm]{Remark} \newtheorem{recall}[thm]{Recall} \newcommand{\lcm}{\operatorname{lcm}} \input{preamble} \DeclareMathOperator{\denom}{denom} \pagenumbering{arabic} \pagestyle{headings} \setcounter{secnumdepth}{4} \setcounter{tocdepth}{2} \setlength{\parindent}{1cm} \begin{document} \title[Density formulas for bounded primes in hypergeometric series]{Density formulas for $p$-adically bounded primes for hypergeometric series with rational and quadratic irrational parameters} \author{Cameron Franc} \address{Department of Mathematics, McMaster University} \curraddr{} \email{[email protected]} \thanks{} \author{Nathan Heisz} \address{Department of Mathematics, McMaster University} \curraddr{} \email{[email protected]} \author{Hannah Nardone} \address{Department of Mathematics, McMaster University} \curraddr{} \email{[email protected]} \thanks{We thank NSERC for support via a Discovery grant.} \date{} \begin{abstract} We study densities of $p$-adically bounded primes for hypergeometric series in two cases: the case of generalized hypergeometric series with rational parameters, and the case of $_2F_1$ with parameters in a quadratic extension of the rational numbers. In the rational case we extend work from $_2F_1$ to $_nF_{n-1}$ for an exact formula giving the density of bounded primes for the series. The density is shown to be one exactly in accordance with the case of finite monodromy as classified by Beukers-Heckmann. In the quadratic irrational case, we obtain an unconditional lower bound on the density of bounded primes. Assuming the normality of the $p$-adic digits of quadratic irrationalities, this lower bound is shown to be an exact formula for the density of bounded primes. In the quadratic irrational case, there is a trivial upper bound of $1/2$ on the density of bounded primes. In the final section of the paper we discuss some results and computations on series that attain this bound. In particular, all such examples we have found are associated to imaginary quadratic fields, though we do not prove this is always the case. \end{abstract} \maketitle \tableofcontents \setcounter{tocdepth}{1} \section{Introduction} A classical problem in the study of hypergeometric series is to determine which series are integral. This problem is closely related to integrality of ratios of factorials, algebraicity of generalized hypergeometric series, and even the Riemann hypothesis --- see \cite{Bober} for a review of some of this work, and the recent paper \cite{AdolphsonSperber} for an extension of some of these questions to the case of GKZ-hypergeometric series. The related problem of determining when a generalized hypergeometric series has finite monodromy was solved by Beukers-Heckmann \cite{beukers}, preceded in the case of $_2F_1$ by classical work of Schwarz. In these cases the algebraic series have rational hypergeometric parameters, and it is known by a classical result of Eisenstein that while the series need not have integral coefficients, there are at worst a finite number of primes that arise in the denominators of the series. Thus, while algebraic generalized hypergeometric series need not be integral, the density of primes where such a series is $p$-adically bounded equals one. In this paper we study densities of $p$-adically bounded primes for various hypergeometric series, typically non-algebraic. The study of \(p\)-adic boundedness of a hypergeometric series, spearheaded by Dwork \cite{dwork} and Christol \cite{christol}, provided necessary and sufficient conditions for a generalized hypergeometric series \(_nF_{n-1}\) to be \(p\)-adically bounded. Franc-Gannon-Mason \cite{FGM} showed that for all series $_2F_1$ with rational hypergeometric parameters, there is a corresponding density of $p$-adically bounded primes, and this density is one exactly when the series is algebraic, that is, it comes from Schwarz's classical list. Franc et.al. in \cite{fetal} built on this work to establish an exact formula for the Dirichlet densities of bounded primes in the denominators of series \(_2F_1\) with rational parameters. Our first main result Theorem \ref{t:rational} extends this density formula to generalized hypergeometric series \(_mF_n\) and furthermore shows that the only case of interest in terms of boundedness of primes is the case where \(m = n+1\). These results can be summarized as follows: \begin{thm} \label{t:main1} Let $F(z) = {} _mF_n(\alpha_1,\ldots, \alpha_m; \beta_1,\ldots, \beta_n;z)$ denote a generalized hypergeometric series with rational hypergeometric parameters, $\alpha_i,\beta_j \in \QQ$. Then the set $B$ of primes where $F(z)$ is $p$-adically bounded has a Dirichlet density. This density is one if $m > n+1$, and it is zero of $m < n+1$. When $m=n+1$, the density can be computed exactly using a simple formula given in Corollary \ref{c:rationaldensityformula}. In this case, the density is one if and only if $F(z)$ is an algebraic function. \end{thm} The next case we discuss, starting in Section \ref{s:quadratic}, is that of hypergeometric series of the form \[ f(z) = {}_2F_1(a+b\sqrt{D},a-b\sqrt{D};c;z) \] where $a,b,c,D \in \QQ$. Such series have rational expansions yet, if $D$ is not square, then they are transcendental functions by \cite{beukers}. The \(N\)-integrality of hypergeometric series \(_mF_n\) with parameters from quadratic fields has been studied by Hong-Wang \cite{hong}, and specific cases of \(_2F_1\) with quadratic irrational parameters have appeared as solutions to a differential equation concerning the arithmetic of modular forms on \(\Gamma_0(2)\) in work by Gottesman \cite{gottesman}. Some of our results in Section \ref{s:quadratic} can be summarized as follows: \begin{thm} \label{t:main2} Let $f(z) \in \pseries{\QQ}{z}$ be a hypergeometric series as above, where $D$ is not a perfect square, and let $B$ be the set of primes where $f(z)$ is $p$-adically bounded. Then $B$ has a Dirichlet density that is at most $1/2$, and which is bounded below by explicit formulas described in Propositions \ref{prop1} and \ref{prop2}. If one assumes that the $p$-adic digits of quadratic irrationalities are normal, then the density bounds in Propositions \ref{prop1} and \ref{prop2} are exact formulas. \end{thm} In the rational case of Theorem \ref{t:main1}, our arguments rely on the periodic $p$-adic expansions of rational numbers. In the quadratic irrational case of Theorem \ref{t:main2}, the expansions are no longer periodic, and so normality serves as a convenient replacement for this property. Actually, to establish our conjectural density formulas in Section \ref{s:quadratic}, we only need to assume a result about $p$-adic expansions of quadratic irrationalities, Conjecture \ref{conj} below, that is implied by $p$-adic normality of digits. We do not know if Conjecture \ref{conj} is in fact equivalent to $p$-adic normality. The density in Theorem \ref{t:main2} is bounded above by $1/2$ for the simple reason that a series such as $f(z)$ can only be $p$-adically bounded for primes $p$ that split in $\QQ(\sqrt{D}$), cf. Proposition \ref{p:split}. The reason why there are two density formulas, cf. Propositions \ref{prop1} and \ref{prop2}, is because the formulas are different in the cases where $2a$ is integral or not. Since the trace of $a+b\sqrt{D}$ plays an important role in the proofs, whether the trace $2a$ has a truncating or periodic $p$-adic expansion affects the argument and the formulas. Section \ref{s:schwarz} is inspired by the Schwarz list for ${}_2F_1$, which essentially classifies series with density of bounded primes equal to one. For the case of series such as $f(z)$ above, a natural quadratic irrational analogue is to classify series where the density of bounded primes is $1/2$. We do not resolve this question, but we discuss some computations towards it, and also establish some upper-bounds that indicate that it should be reasonably rare for a series such as $f(z)$ to have a density of bounded primes equal to $1/2$. For example, in all of our examples, we only ever observed a density of $1/2$ when the field $K=\QQ(\sqrt{D})$ is imaginary quadratic. See Table \ref{tab:dvaryk} for some examples with $a$ and $c$ of small height. This rarity of large densities is in-line with the classical Schwarz list being rather sparse among all rational hypergeometric series. An interesting open question is how the irrational case extends to higher values of \(n\) for a general series \(_nF_{n-1}\) with irrational but algebraic hypergeometric parameters. For example, one could consider series \(_3F_2\) with a pair of quadratic conjugates in the numerator, along with a third rational numerator parameter, and a separate pair of conjugate quadratic irrationalities in the denominator. This will still have rational coefficients, and presumably also a density of bounded primes. Or, one could consider $_3F_2$ with three cyclic Galois conjugates of degree three as the numerator parameters --- again, is there a density of bounded primes for such series? While we find these questions interesting, the array of conceivable patterns only grows more vast as \(n\) increases, so it is difficult to imagine what sort of general result might exist. Nevertheless, the density results in the quadratic irrational case of ${}_2F_1$ are surprisingly clean, and they require a nice application of $p$-adic normality of algebraic numbers --- or its a priori weaker form of Conjecture \ref{conj} --- that could provide an impetus for work in the area of $p$-adic metric number theory. \section{Background} The rising factorials are defined for integers $k\geq 0$ as $(x)_0 = 1$ and otherwise \[ (x)_k \df x(x+1)\cdots (x+k-1). \] Let $\alpha = (\alpha_1,\ldots ,\alpha_{m})$ and $\beta = (\beta_1,\ldots, \beta_m)$ denote complex numbers, and assume that $\beta_j \not \in \ZZ_{\leq 0}$ for all $j=1,\ldots, n$. The corresponding \emph{generalized hypergeometric series}, or just hypergeometric series, ${}_mF_n(\alpha;\beta;z)$ is defined by the infinite series \[ {}_mF_n(\alpha;\beta;z) = \sum_{k\geq 0} \frac{(\alpha_1)_k\cdots (\alpha_m)_k}{(\beta_1)_k\cdots (\beta_n)_k} \frac{z^k}{k!}. \] These series satisfy hypergeometric differential equations, and the equations are regular singular exactly when $m=n+1$. The case of ${}_2F_1$ has a long history dating back at least to work of Gauss and Riemann. The finite monodromy groups of these equations were classified by Schwarz, and the case of ${}_mF_n$ was treated more recently by Beukers-Heckmann\cite{beukers}. Let $p$ be a rational prime, and let $\nu_p$ denote the corresponding $p$-adic valuation normalized so that $\nu_p(p)=1$. \begin{dfn} A hypergeometric series for parameters $(\alpha,\beta)$ is said to be \emph{rational} if ${}_mF_n(\alpha;\beta;z) \in \pseries{\QQ}{z}$. \end{dfn} \begin{rmk} Obviously, hypergeometric parameters such that $\alpha_i\in \QQ$ and $\beta_j \in \QQ$ for every $i$ and $j$ yield rational series, but there exist other examples such as $\alpha = (a+b\sqrt{2},a-b\sqrt{2})$ and $\beta = (c)$ for any rational numbers $a,b,c$ with $c \not \in \ZZ_{\leq 0}$. \end{rmk} \begin{dfn} A prime $p$ is said to be \emph{bounded} for a rational hypergeometric series if the $p$-adic valuations of the coefficients of ${}_mF_n(\alpha;\beta;z)$ are bounded below. Equivalently, there exists $N \in \ZZ$ such that \[ p^N{}_mF_n(\alpha;\beta;z) \in \pseries{\ZZ_p}{z}. \] Otherwise $p$ is said to be \emph{unbounded} for the given series. \end{dfn} In \cite{FGM} it was shown that if $\alpha,\beta,\gamma \in \QQ$ with $\gamma \not \in \ZZ_{\leq 0}$, then the set of primes such that the classical hypergeometric series ${}_2F_1(\alpha,\beta;\gamma;z)$ has bounded denominators has a natural density. In Theorem 4 of \cite{fetal}, precision was added to this result in the form of an explicitly computable formula for this natural density: \begin{thm}\label{t:fetal} Let $a,b$ and $c$ denote rational numbers with $0 < a,b,c, < 1$, where $c \neq a,b$. Let $N$ denote the least common multiple of the denominators of $a-1$, $b-1$ and $c-1$, and define \[ B(a,b;c) \df \{u \in (\ZZ/N\ZZ)^\times \mid \textrm{ for all } j \in \ZZ,~ \{-u^jc\} \leq \max(\{-u^ja\},\{-u^jb\})\}. \] Then for all primes $p > N$, the series ${}_2F_1(a,b;c;z)$ is $p$-adically bounded if and only if $p$ is congruent to an element of $B(a,b;c)$ mod $N$. Therefore, the set of bounded primes for ${}_2F_1(a,b;c;z)$ has a density equal to $\tfrac{\abs{B(a,b;c)}}{\phi(N)}$. \end{thm} \begin{proof} This is Theorem 4 of \cite{fetal}. \end{proof} Our goals below are twofold: \begin{enumerate} \item generalize Theorem \ref{t:fetal} to the case of general $m$ and $n$; \item generalize Theorem \ref{t:fetal} when $m=2$, $n=1$, to the case of certain rational parameters living in quadratic number fields. \end{enumerate} In the case of quadratic fields, our density result is conditional on a seemingly plausible conjecture about the normality of the $p$-adic expansions of irrational algebraic numbers at split primes. Such a conjecture is a substitute for the lack of periodicity of the $p$-adic expansions of the hypergeometric parameters in these cases. To study the $p$-adic boundedness of hypergeometric series, following \cite{FGM} we introduce: \begin{dfn} \label{d:carries} Let $c_p$ denote the function \[c_p \colon \ZZ_p\times \ZZ_{\geq 0} \to \ZZ_{\geq 0} \cup \{\infty\}\] where $c_p(x,k)$ computes the number of carries needed to evaluate the $p$-adic sum $x+k$ using the usual add-and-carry method. \end{dfn} In \cite{FGM} it was observed that this function is locally constant in $x$, and using a classical result of Kummer, one in fact has the following: \begin{thm}[Kummer] \label{t:kummer} One has $\nu_p\binom{x+k}{k} = c_p(x,k)$. \end{thm} \begin{proof} See Theorem 2.5 of \cite{FGM}. \end{proof} The following result is a slight generalization of Theorem 3.4 of \cite{FGM}, to the case of general $m$ and $n$. \begin{cor} \label{c:FGM} Let $\alpha$ and $\beta$ denote hypergeometric parameters contained in $\ZZ_p$, such that no $\beta$ parameter is a negative integer. Then if we write ${}_m F_n(\alpha;\beta;z) = \sum_{k\geq 0}A_kz^k$ for $A_k \in \QQ_p$, we have \[ \nu_p(A_k) = \sum_{i=1}^mc_p(\alpha_i-1,k)-\sum_{j=1}^n c_p(\beta_j-1,k) + (m-n-1)\nu_p(k!). \] \end{cor} \begin{proof} By a direct computation one sees that \[ A_k=\frac{\prod_{i=1}^m\binom{\alpha_i-1+k}{k}}{\prod_{j=1}^n\binom{\beta_j-1+k}{k}}\cdot (k!)^{m-n-1} \] Therefore, this Corollary follows immediately from Theorem \ref{t:kummer}. \end{proof} \begin{rmk} \label{r:nontrivial} Since $\nu_p(k!) = O(k)$, while $c_p(x,k) = O(\log k)$ away from the poles $x$ of $c_p(x,k)$, one can use Corollary \ref{c:FGM} to prove that for rational hypergeometric parameters, all primes are bounded if $m > n+1$, whereas no primes are bounded if $m < n+1$. Thus, as far as densities go, for rational parameters the only interesting case is when $m=n+1$ and the corresponding hypergeometric differential equation is regular singular. \end{rmk} \section{Rational parameters} \label{s:rational} This section considers the case of rational parameters $\alpha,\beta$. In light of Remark \ref{r:nontrivial}, we restrict also to the case of ${}_nF_{n-1}$. We begin by recalling some facts on the $p$-adic expansions of rational numbers, which are always eventually periodic. In fact, if $a$ is a rational number satisfying $-1 < a < 0$ and $a \in \ZZ_p^\times$, then the $p$-adic expansion of $a$ is purely periodic, of period equal to the multiplicative order of $p$ modulo the denominator of $a$. Since the density of bounded primes for a given set of hypergeometric parameters only depends on the rational parameters mod $\ZZ$, we will normalize our parameters to lie in the interval $(0,1)$. We state the following lemma on the digits of our hypergeometric parameters in a way which will make the application of Corollary \ref{c:FGM} more straightforward. \begin{lem} \label{l:digitformula} Let $a = \tfrac uv$ denote a rational number with $\gcd(u,v) =1$ satisfying $0 < a < 1$, and let $p$ denote a prime such that $a-1$ is a $p$-adic unit. Let $M$ denote the multiplicative order of $p \pmod{v}$, and let the periodic expansion of $a-1$ be denoted as: \[ a-1 = \overline{\alpha_0\alpha_1\cdots \alpha_{M-1}}. \] Then for each $j$ with $0\leq j \leq M-1$ we have \[ \alpha_j = \floor{\{-p^{M-1-j}a\}p}. \] In particular, if $p > v$, then each $\alpha_j$ is nonzero. \end{lem} \begin{proof} Most of this is Lemma 2.4 of \cite{FGM}. All that remains to observe is the final claim that each $\alpha_j$ is nonzero. For this, begin with the observation that \[ \{-p^{M-1-j}a\} -\tfrac 1p < \tfrac{\alpha_j}{p} \leq \{-p^{M-1-j}a\}. \] Since $p$ is coprime to $a$, then $\{-p^{M-1-j}a\}$ is a rational number of the form $k/v$ for some integer $1\leq k \leq v-1$. Hence, \[ 0<\frac{1}{v}-\frac 1p < \frac{\alpha_j}{p} \] as claimed. \end{proof} \begin{dfn} A prime $p$ is \emph{good} for hypergeometric parameters $\alpha$, $\beta$ provided that all of $\alpha_i-1$ and $\beta_i-1$ are $p$-adic units, and such that $p$ does not divide any of the differences $\alpha_i-\beta_j$. \end{dfn} Note that as long as no $\alpha_i$ is equal to a $\beta_j$, a harmless hypothesis that we enforce below, then the set of good primes for a set of hypergeometric parameters contains all primes that are large enough. Hence, as far as densities of bounded primes go, it is harmless to restrict to considering good primes only. Let $p$ denote a good prime for some rational hypergeometric parameters, and let $M$ denote the order of $p$ modulo the least common multiple of the denominators of the hypergeometric parameters. Then by Lemma \ref{l:digitformula}, the $p$-adic periods of each parameter all divide $M$. \begin{dfn} \label{d:numeratormajorized} Let $\alpha = (\alpha_i)_{i=1}^n$ and $\beta = (\beta_i)_{i=1}^{n-1}$ denote rational hypergeometric parameters, and let $N$ denote the least common multiple of the denominators of the parameters. Fix an invertible congruence class $u\pmod{N}$. Then a set of hypergeometric parameters is said to be \emph{numerator majorized} for this congruence class provided that for all integers $j\geq 0$, there exists a permutation $\sigma_j \in S_n$ such that \[ \{-u^j\beta_i\} \leq \{-u^j\alpha_{\sigma_j(i)}\} \] for all $i=1,\ldots, n-1$. \end{dfn} Definition \ref{d:numeratormajorized} is reminiscient of the interlacing of roots of unity condition that appears in the study of finite monodromy in \cite{Landau} and \cite{beukers}. For a given set of parameters, it is a finite computation to determine which congruence classes are numerator majorized or not. \begin{ex} \label{ex1} Consider the hypergeometric series \[ {}_3F_2(\tfrac 15, \tfrac 25, \tfrac 35; \tfrac 45, \tfrac 12;z). \] In this case $N=10$, and so we only need to test the numerator majorization condition for $j=0,1,2,3$, since $\phi(10) = 4$, and thus $p^4 \equiv 1 \pmod{10}$. \begin{table} \renewcommand{\arraystretch}{1.5} \begin{tabular}{c||c|c|c||c|c} $p\pmod{10}$ & $\tfrac 15$ & $\tfrac 25$ & $\tfrac 35$ & $\tfrac 45$ & $\tfrac 12$\\ \hline $1$ &$\left[\frac{4}{5}, \frac{4}{5}, \frac{4}{5}, \frac{4}{5}\right]$&$\left[\frac{3}{5}, \frac{3}{5}, \frac{3}{5}, \frac{3}{5}\right]$&$\left[\frac{2}{5}, \frac{2}{5}, \frac{2}{5}, \frac{2}{5}\right]$&$\left[\frac{1}{5}, \frac{1}{5}, \frac{1}{5}, \frac{1}{5}\right]$&$\left[\frac{1}{2}, \frac{1}{2}, \frac{1}{2}, \frac{1}{2}\right]$\\ $3$ &$\left[\frac{4}{5}, \frac{2}{5}, \frac{1}{5}, \frac{3}{5}\right]$&$\left[\frac{3}{5}, \frac{4}{5}, \frac{2}{5}, \frac{1}{5}\right]$&$\left[\frac{2}{5}, \frac{1}{5}, \frac{3}{5}, \frac{4}{5}\right]$&$\left[\frac{1}{5}, \frac{3}{5}, \frac{4}{5}, \frac{2}{5}\right]$&$\left[\frac{1}{2}, \frac{1}{2}, \frac{1}{2}, \frac{1}{2}\right]$\\ $7$ &$\left[\frac{4}{5}, \frac{3}{5}, \frac{1}{5}, \frac{2}{5}\right]$&$\left[\frac{3}{5}, \frac{1}{5}, \frac{2}{5}, \frac{4}{5}\right]$&$\left[\frac{2}{5}, \frac{4}{5}, \frac{3}{5}, \frac{1}{5}\right]$&$\left[\frac{1}{5}, \frac{2}{5}, \frac{4}{5}, \frac{3}{5}\right]$&$\left[\frac{1}{2}, \frac{1}{2}, \frac{1}{2}, \frac{1}{2}\right]$\\ $9$ &$\left[\frac{4}{5}, \frac{1}{5}, \frac{4}{5}, \frac{1}{5}\right]$&$\left[\frac{3}{5}, \frac{2}{5}, \frac{3}{5}, \frac{2}{5}\right]$&$\left[\frac{2}{5}, \frac{3}{5}, \frac{2}{5}, \frac{3}{5}\right]$&$\left[\frac{1}{5}, \frac{4}{5}, \frac{1}{5}, \frac{4}{5}\right]$&$\left[\frac{1}{2}, \frac{1}{2}, \frac{1}{2}, \frac{1}{2}\right]$ \end{tabular} \caption{The values $\{-p^j\alpha_i\}$ and $\{-p^j\beta_i\}$ for the hypergeometric parameters $\alpha = (\tfrac 15,\tfrac 25, \tfrac 35)$ and $\beta = (\tfrac 45, \tfrac 12)$.} \label{t1} \end{table} Table \ref{t1} shows that the class $1 \pmod{10}$ is numerator majorized for these parameters. The class $3\pmod{10}$ fails the numerator majorization condition for $j=2$, as the third entry of the list for the denominator parameter $\tfrac 45$ in that case is $\tfrac 45$, which is maximal and thus can't be numerator majorized. Similarly, $7 \pmod{10}$ and $9\pmod{10}$ fail the numerator majorization condition in this example. \end{ex} \begin{lem} \label{l:equivalent} Let $\alpha = (\alpha_i)_{i=1}^n$ and $\beta = (\beta_i)_{i=1}^{n-1}$ denote rational hypergeometric parameters taken from the interval $(0,1)$, and let $N$ denote the least common multiple of the denominators of the parameters. Then these parameters are numerator majorized for a a congruence class $u\pmod{N}$ if and only if for all large enough primes $p\equiv u \pmod{N}$, if we write the $p$-adic digits of $\alpha_i-1$ as $\alpha_{i,j}(p)$ and similarly for $\beta_i-1$ and $\beta_{i,j}(p)$, then for all integers $j\geq 0$ there exists a permutation $\sigma_j \in S_n$ such that \[ \beta_{i,j}(p) \leq \alpha_{\sigma_j(i),j}(p) \] for all $i=1,\ldots, n-1$. \end{lem} \begin{proof} For all primes $p\equiv u \pmod{N}$, being numerator majorized is equivalent to the condition \[ \{-p^j\beta_i\}p \leq \{-p^j\alpha_{\sigma_j(i)}\}p. \] If $p > 1/(\alpha_u-\beta_v)$ for all $u,v$, then this inequality is \emph{equivalent} to the inequality obtained by taking floors above: \[ \floor{\{-p^j\beta_i\}p} \leq \floor{\{-p^j\alpha_{\sigma_j(i)}\}p}. \] Therefore, the equivalence of these two formulations of numerator majorization follows from Lemma \ref{l:digitformula}. \end{proof} \begin{thm} \label{t:rational} Let $\alpha = (\alpha_i)_{i=1}^n$ and $\beta = (\beta_i)_{i=1}^{n-1}$ denote hypergeometric parameters, and let $N$ denote the least common multiple of the denominators of the parameters, and fix an invertible congruence class $u\pmod{N}$. Then the corresponding hypergeometric series ${}_nF_{n-1}(\alpha,\beta,z)$ is $p$-adically bounded for all sufficiently large primes $p\equiv u \pmod{N}$ if and only if the hypergeometric parameters are numerator majorized with respect to the congruence class $p \equiv u\pmod{N}$. \end{thm} \begin{proof} Assume that $p > \max_{i,j}(\alpha_i-\beta_j)^{-1}$ so that Lemma \ref{l:equivalent} holds for such primes. Assume that the series is numerator majorized with respect to $u \pmod{N}$, so that the $p$-adic digits are numerator majorized by Lemma \ref{l:equivalent}. Now Corollary \ref{c:FGM} shows that the negative contributions for carries from terms $c_p(\beta_j-1,k)$ are compensated for by carries $c_p(\alpha_i-1,k)$. Hence $\nu_p(a_k) \geq 0$ for such primes $p$, so that in fact ${}_nF_{n-1}(\alpha,\beta,z) \in \pseries{\ZZ_p}{z}$. This proves one direction of the Theorem. Assume conversely that the parameters are not numerator majorized with respect to a congruence class $u\pmod{N}$. This means there exists a digit index $j$ such that for every $\sigma \in S_n$, there exists a hypergeometric parameter index $i$ such that \[ \beta_{i,j}(p) > \alpha_{\sigma(i),j}(p). \] First suppose that there is an index $i$ such that $\beta_{i,j}(p) > \alpha_{k,j}(p)$ for all $k$. For each integer $A \geq 0$, consider the term of the hypergeometric series indexed by \[k_A \df (p-\beta_{i,j}(p))\sum_{u=0}^Ap^{j+u\phi(N)}.\] Since $\phi(N)$ is a common period for the $p$-adic expansions of all of the hypergeometric parameters, we see that $c_p(\beta_i-1,k_A) = A+1$, but otherwise by construction $c_p(\beta_u-1,k_A) = 0$ for $u\neq i$ and $c_p(\alpha_v-1,k_A) = 0$ for all $v$. Therefore Corollary \ref{c:FGM} shows that in this case $v_p(a_{k_A}) \geq A+1$, where the $a_{k}$ denote the coefficients of ${}_nF_{n-1}(\alpha,\beta,z)$. This treats the case where one of the $\beta_{i,j}(p)$ digits is maximal among all of the $j$-th coefficients of the given set of hypergeometric parameters. Otherwise, we can without loss of generality assume that $\alpha_{1,j}(p) \geq \cdots \geq \alpha_{n,j}(p)$ and similarly $\beta_{1,j}(p) \geq \cdots \geq \beta_{n-1,j}(p)$. We have treated in the previous paragraph the case when $\alpha_{1,j}(p) < \beta_{1,j}(p)$, and since numerator majorization fails, there must exist a smallest index $i$ such that $\alpha_{u,j}(p) \geq \beta_{u,j}(p)$ for $1\leq u \leq i-1$ but $\beta_{i,j}(p) > \alpha_{i,j}(p)$. Now defining $k_A$ as above, this time we see that carries coming from the $\beta_{u,j}(p)$ are compensated for by carries coming from $\alpha_{u,j}(p)$ for $1 \leq u \leq i-1$, but we have $c_p(\alpha_{i,j}(p),k_A) \geq A+1$ and $c_p(\beta_{i,j}(p),k_A) =0$. Therefore we again find that $\nu_p(a_{k_A}) \leq -A-1$, and so in all cases, if the parameters are not numerator majorized for $u \pmod{N}$, then the corresponding series is not $p$-adically bounded for all primes $p > \max_{i,j}(\alpha_i-\beta_j)^{-1}$ satisfying $p\equiv u\pmod{N}$. This concludes the proof of the theorem. \end{proof} \begin{rmk} \label{r:explicit} We emphasize that this proof shows that if the parameters are numerator majorized for $u\pmod{N}$, then for all primes $p > \max_{i,j}(\alpha_i-\beta_j)^{-1}$ satisfying $p\equiv u \pmod{N}$, we have ${}_nF_{n-1}(\alpha,\beta,z) \in \pseries{\ZZ_p}{z}$. \end{rmk} \begin{cor} \label{c:rationaldensityformula} Let $\alpha = (\alpha_i)_{i=1}^n$ and $\beta = (\beta_i)_{i=1}^{n-1}$ denote hypergeometric parameters, and let $N$ denote the least common multiple of the denominators of the parameters. Then for every prime $p > \max_{i,j}(\alpha_i-\beta_j)^{-1}$, the series ${}_nF_{n-1}(\alpha,\beta,z)$ is $p$-adically bounded if and only if $p$ is congruent to an element of the following set: \[ B(\alpha;\beta) =\{u \in (\ZZ/N\ZZ)^\times \mid \alpha \textrm{ and } \beta \textrm{ are numerator majorized for } u\}. \] In fact, for such primes $p$, the series ${}_nF_{n-1}(\alpha,\beta,z)$ is $p$-adically integral. In particular, the set of bounded primes for $\alpha,\beta$ has a Dirichlet density equal to $\tfrac{\abs{B(\alpha;\beta)}}{\phi(N)}$. \end{cor} \begin{proof} This follows directly from Theorem \ref{t:rational} and Remark \ref{r:explicit}. \end{proof} \begin{rmk} \label{r:cyclic} Notice that the set $B(\alpha;\beta)$, if nonempty, is a union of cyclic subgroups of $(\ZZ/N\ZZ)^\times$. \end{rmk} \begin{ex} \label{ex2} In Example \ref{ex1} we saw that for \[ {}_3F_2(\tfrac 15, \tfrac 25, \tfrac 35; \tfrac 45, \tfrac 12;z) \] we have $N = 10$ and $B(\tfrac 15, \tfrac 25, \tfrac 35; \tfrac 45, \tfrac 12) = \{1\} \subseteq (\ZZ/10\ZZ)^\times$. Therefore, the bounded primes $p > 10$ for this series are precisely those satisfying $p\equiv 1 \pmod{10}$, so that the density of bounded primes in this case is $\tfrac 14$. In fact, this series is $p$-integral for such primes, so that the only primes appearing in the denominators of this series satisfy $p \leq 7$ or $p\equiv 3,7,9 \pmod{10}$. \end{ex} The following result generalizes Theorem 4.14 from \cite{FGM}. \begin{cor} Let $\alpha = (\alpha_i)_{i=1}^n$ and $\beta = (\beta_i)_{i=1}^{n-1}$ denote hypergeometric parameters, and assume that they are ordered in increasing order of their fractional parts. Then the density of bounded primes for ${}_nF_{n-1}(\alpha,\beta,z)$ is zero if and only if there exists an index $i$ such that \[ \{\beta_i\} < \{\alpha_i\}. \] \end{cor} \begin{proof} By Corollary \ref{c:rationaldensityformula} and Remark \ref{r:cyclic}, the density of bounded primes is zero if and only if $1 \not \in B(\alpha,\beta)$. The numerator majorization failure for $u=1$ is equivalent to the existence of an index $i$ with $\{-\beta_i\} > \{-\alpha_i\}$, or equivalently, $\{\beta_i\} < \{\alpha_i\}$ as claimed. \end{proof} \section{Quadratic irrational parameters} \label{s:quadratic} Let now $D \in \QQ$ be a nonsquare rational, and let $K = \QQ(\sqrt{D})$. Notice that for $a,b,c\in\QQ$ with $c \not \in \ZZ_{\leq 0}$, the series \[ F_D(a,b,c,z) \df {}_2F_1(a+b\sqrt{D},a-b\sqrt{D};c;z) \] has rational coefficients. We will assume $b \neq 0$, so that $F_D$ does not have rational parameters. \begin{prop} \label{p:split} For all but finitely many primes $p$, a necessary condition for $F_D(a,b,c,z)$ to be $p$-adically bounded is that $p$ splits in $K$. \end{prop} \begin{proof} Let $A_n$ denote the $n$th coefficient of $F_D(a,b,c,z)$, and let $P(x)$ denote the minimal polynomial of $a+b\sqrt{D}$: \[P(x) = x^2 - 2ax + a^2-b^2D.\] Then we find that \[ A_n = \frac{(a+b\sqrt{D})_n(a-b\sqrt{D})_n}{(c)_nn!} =\frac{\prod_{j=0}^{n-1}P(-j)}{(c)_nn!}. \] Since $a+b\sqrt{D}$ generates $K/\QQ$, and $P(x)$ is its minimal polynomial, with at most finitely many exceptions only rational primes that split in $K$ divide the values $P(j)$ in the numerator of $A_n$. On the other hand, the denominator is $(c)_nn!$, and all but finitely many exceptions depending only on $c$ divide this denominator to larger and larger powers as $n$ grows. Therefore, a necessary condition to have cancellation in $A_n$ for large enough primes is that $p$ be split in $K$. \end{proof} \begin{rmk} We say \(u\) splits in \(K\) if primes equivalent to \(u\mod M\) split in \(K\). This is well defined as the splitting condition of a prime in a number field \(K\) relies on the congruence class of the prime \(\mod \Delta_K\), where \(\Delta_K\) denotes the discriminant of the number field \(K\). Since \(M = \lcm(\denom(a),\denom(c),\Delta_K)\), all primes equivalent to \(u\mod M\) split in \(K\) if any prime equivalent to \(u\mod M\) splits in \(K\). \end{rmk} Let $S$ denote the set of rational primes that split in $K$. Notice that Corollary \ref{c:FGM} still applies to $F_D(a,b,c,z)$ for all but finitely many primes $p \in S$. In the rational case we are able to leverage the periodicity of $p$-adic expansions in conjunction with Corollary \ref{c:FGM} to produce unbounded denominators. But the $p$-adic expansions of the irrational hypergeometric parameters $\alpha = (a+b\sqrt{D},a-b\sqrt{D})$ are no longer periodic. However, it turns out that we can still study the density of bounded primes in a similar way if these $p$-adic irrational numbers have sufficiently randomly distributed digits. To begin, we recall the following definition. \begin{dfn} \label{d:normal} A $p$-adic integer $\alpha \in \ZZ_p$ is said to be \emph{normal} provided that every sequence of $p$-adic digits occurs equally often in the following asymptotic sense: let $\alpha = \sum_{n\geq 0}\alpha_np^n$ be the $p$-adic expansion of $\alpha$ for $\alpha_n \in \{0,1,\ldots, p-1\}$ for all $n$, and let $B = B_1B_2\cdots B_{m}$ be a bit string, where $B_j \in \{0,1,\ldots, p-1\}$ for all $j$. Then \[ \lim_{N\to \infty}\frac{\abs{\{j\leq N \mid \alpha_j\alpha_{j+1}\cdots \alpha_{j+m-1} = B\}}}{N} = \frac{1}{p^m}. \] This should hold for any choice of $p$-adic bit string $B$. \end{dfn} Normality is an established property over the real numbers. For more information see a text on metric number theory, such as \cite{MNT}. There exist $p$-adic transcendental numbers that are not normal. Computations suggest that $p$-adic numbers that are algebraic over $\QQ$ of degree $d\geq 2$ may always be normal, though this is currently an open question. The following conjecture may be regarded as a weakened form of normality. \begin{conj} \label{conj} Let $\alpha \in \overline \QQ\setminus \QQ$, and let $S$ be the set of rational primes that are totally split in $\QQ(\alpha)$. Let $r$ and $s$ be integers, and let $u,v$ be real numbers with $0\leq u < v \leq 1$. Let $\alpha_n(p,\sigma)$ denote the $n$th $p$-adic digit of $\alpha$ for $p \in S$ with respect to the field embedding $\sigma \colon \QQ(\alpha) \to \QQ_p$. Then for all but finitely many primes $p \in S$ (where the finite exceptional set depends on $\alpha$, $r$, $s$, $u$ and $v$), the digit $\alpha_n(p,\sigma)$ is contained in the interval $(u(p-1),v(p-1))$ for infinitely many integers $n\geq 0$ in the arithmetic progression $n = rm+s$. \end{conj} \begin{prop} \label{p:normalimpliesconj} Suppose that $\alpha \in \overline\QQ \setminus \QQ$ has the property that $\sigma(\alpha)$ is $p$-adically normal for any embedding $\sigma \colon \QQ(\alpha) \to \QQ_p$. Then Conjecture \ref{conj} holds for $\alpha$. \end{prop} \begin{proof} We will show that given an arithmetic progression $n = rm+s$, then \emph{any} digit occurs infinitely often in the sequence $(\alpha_{rm+s}(p,\sigma))_{m\geq 0}$ of $p$-adic digits, which clearly implies Conjecture \ref{conj}. Suppose that a digit $b$ only occurs finitely many times along this progression, and let $B$ be the string which is $m$ copies of $b$. Then the quantity \[ \abs{\{j\leq N \mid \alpha_j(p,\sigma)\alpha_{j+1}(p,\sigma)\cdots \alpha_{j+m-1}(p,\sigma) = B\}} \] is bounded absolutely, independently of $N$, since at least one of the terms $\alpha_{j+i}(p,\sigma)$ for $0\leq i \leq m-1$ must meet the sequence $(\alpha_{rm+s}(p,\sigma))_{m\geq 0}$. As this contradicts the normality of $\sigma(\alpha)$, it thus verifies our claim that every digit appears infinitely often in $(\alpha_{rm+s}(p,\sigma))_{m\geq 0}$. This concludes the proof. \end{proof} Before establishing our density results, we reformulate the condition of $p$-adic unboundedness for $F_D$ slightly: \begin{thm} Let $a,b,c$ denote rational numbers as above, and write the $p$-adic digits of $a+b\sqrt{D}-1$, $a-b\sqrt{D}-1$, and $c-1$ as $\alpha_j(p)$, $\alpha_j'(p)$ and $\gamma_j(p)$, respectively. Then the following are equivalent for all but finitely many primes that split in $K$: \begin{enumerate} \item the inequality $\gamma_j(p) > \max(\alpha_j(p),\alpha'_j(p))$ holds for infinitely many $j$; \item $F_D(a,b;c;z)$ has $p$-adically unbounded coefficients. \end{enumerate} \end{thm} \begin{proof} First suppose that (1) holds and let $j_1,j_2,\ldots$ denote the sequence of indices where $\gamma_j(p) > \max(\alpha_j(p),\alpha'_j(p))$. If we set \[ m_r=\sum_{k=1}^r (p-\gamma_{j_k}(p))p^{j_k} \] and let $A_n$ denote the $n$th coefficient of $F_D(a,b;c;z)$, then we find by Corollary \ref{c:FGM} that $\nu_p(A_{m_r}) \leq -r$. Thus (1) implies (2). Conversely, if (2) holds, then by Corollary \ref{c:FGM} there are infinitely many terms where the $j$th digit of $c-1$ is larger than the $j$th digits of both $a\pm b\sqrt{D}-1$. \end{proof} Before establishing our density results we prove a simple lemma: \begin{lem} \label{l:digitsum} Let $x,y\in\ZZ_p$ and set $z = x+y$. Then \[ x[j] + y[j] \geq z[j]-1. \] \end{lem} \begin{proof} Since $p$-adic addition involves carries, we deduce that \[ x[j]+y[j]= \begin{cases} z[j]& \textrm{no carries at digits $j-1$ and $j$,}\\ z[j]-1 & \textrm{carry at $j-1$, not at $j$,}\\ z[j] + \alpha p& \textrm{carry at $j$, not at $j-1$,}\\ z[j]+\alpha p-1& \textrm{carries at $j-1$ and $j$,} \end{cases} \] where $\alpha \in \{1,2\}$. Therefore the lemma holds for elementary reasons. \end{proof} The density results differ depending on whether $2a$ is an integer or not. We first treat the case where $2a$ is an integer: \begin{prop} \label{prop1} Let $a$ be rational number with $2a \in \ZZ$, let $b$ be rational and nonzero, and let \( c\) be rational and different from a negative integer. Let $K = \QQ(\sqrt{D})$ with $D \in \ZZ$ not a perfect square. Let $M$ be the least common multiple of the denominator of \(c\), and twice the discriminant of \(K\). Consider the hypergeometric series defined by: \[ F_D= {}_2F_1\left(a+b\sqrt D, a-b\sqrt D; c ; z\right) \] Then there exists a density $\delta$ of $p$-adically bounded primes for $F_D$. If we set \[ B_K(a;c) \df \{u\in \left(\ZZ/M\ZZ\right)^\times {\vert} \{-u^jc\} \leq \{-u^j\tfrac 12\} \text{ for all j and \(u\) splits in K}\}, \] then $\delta$ satisfies \[ \frac{\abs{B_K(a;c)}}{\phi(M)} \leq \delta \leq \frac 12. \] Furthermore, if Conjecture \ref{conj} holds, then $\delta = \frac{\abs{B_K(a;c)}}{\phi(M)}$. In particular, the density is conjecturally independent of $b$. \end{prop} \begin{proof} In order to apply Lemma \ref{l:digitformula} below, we need to adjust our rational numbers $a$ and $c$ so that they lie in the range $0 < a,c < 1$. This can be done using classical transformation formulas for ${}_2F_1$ relating series whose parameters differ by integers. These transformation formulas will change denominators of the series at finitely many primes, so that it it harmless to assume that $0 < a,c <1$. Henceforth we assume this throughout the proof. In particular, we now take $a = 1/2$. Since $M$ is even, any integer \(u\) coprime to \(M\) must be odd, the inequality in the definition of $B_K$ simplifies to \[ \{-u^j c\} \le \frac 12. \] By Lemma \ref{l:digitformula}, this implies that if $p$ is congruent to an element of $B_K(a,c)$ mod $M$, then we have \begin{equation} \label{eq:cdigitsbound} (c-1)[j] \le \tfrac{p-1}2 \end{equation} for all $j$, where now $x[j]$, for a $p$-adic integer $x$, denotes the $j$th $p$-adic digit of $x$. Apply Lemma \ref{l:digitsum} with $x = \tfrac 12 + b\sqrt{D}-1$ and $y=\tfrac 12 - b\sqrt{D}-1$. We deduce that \[ (\tfrac 12 + b\sqrt{D}-1)[j]+(\tfrac 12 - b\sqrt{D}-1)[j] \geq (-1)[j]-1 = p-2. \] In particular, at least one of $(\tfrac 12 \pm b\sqrt{D}-1)[j]$ is $\geq (p-1)/2$. But then it follows by Corollary \ref{c:FGM} and equation \eqref{eq:cdigitsbound}, as in the arguments of \cite{FGM}, that carries arising from the denominator parameter $c$ are balanced by the carries arising from the numerator parameters $a\pm b\sqrt{D}$. Therefore, $F_D$ is $p$-adically bounded for all primes congruent to elements of $B_D(a;c)$. Now assume Conjecture \ref{conj}. For every prime $p$ large enough and such that $p$ is not congruent to an element of $B_K(a;c)$ we will show that $F_D$ is $p$-adically unbounded. If $p$ is such a prime, where the size bound will be identified below, then by definition of $B_K(a;c)$, there exists an integer $j$ such that \[ \{-p^jc\} > \frac 12. \] If $A$ is the multiplicative order of $p \pmod{M}$, we then obtain \[ \{-p^{j+nA}c\} > \frac 12 \] for all integers $n$, since the left side is independent of $n$. Moreover, the left hand side of this inequality is an element of $(1/M)\ZZ$, and so we deduce that: \[ \{-p^{j+nA}c\} \geq \frac 12 + \frac{1}{M} \] for all integers $n$. Now, notice by Lemma \ref{l:digitformula} that $c-1$ has a periodic $p$-adic expansion of period dividing $A$, since the denominator of $c$ divides $M$. In particular, the terms \[(c-1)[j+nA]\] are independent of $n$. Also, if $p$ is large enough, Lemma \ref{l:digitformula} and the inequality above imply that \[ (c-1)[j+nA] = \floor{\{-p^{j+nA}c\}p} \geq \floor{\frac p2+\frac pM} \geq \frac{p-1}{2}+\floor{\frac pM} \] That is, for all $n\geq 0$ we have shown that \begin{equation} \label{eq:cboundprop1} \floor{\frac pM} \leq (c-1)[j+nA] - \frac{p-1}{2}. \end{equation} Now, Conjecture \ref{conj} yields an infinite subsequence $(x_n)$ of the arithmetic progression $j+nA$ such that \begin{equation} \label{eq:cboundprop1,2} \frac{p-1}{2}-\frac 12 \floor{\frac pM} < (\tfrac 12 +b\sqrt{D}-1)[x_n] < \frac{p-1}{2}+\frac 12 \floor{\frac pM} \end{equation} for all $n\geq 0$ as long as $p$ is large enough. Notice that \[ (\tfrac 12 -b\sqrt{D}-1)[m] = p-1-(\tfrac 12 +b\sqrt{D}-1)[m] \] for all $m\geq 0$, since \[ (\tfrac 12 -b\sqrt{D}-1) + (\tfrac 12 +b\sqrt{D}-1) = -1 = \sum_{n\geq 0} (p-1)p^n. \] If we combine this observation with equation \eqref{eq:cboundprop1,2} we deduce likewise: \begin{equation} \label{eq:negativebound} \frac{p-1}{2}-\frac 12 \floor{\frac pM} < (\tfrac 12 -b\sqrt{D}-1)[x_n] < \frac{p-1}{2}+\frac 12 \floor{\frac pM} \end{equation} Now equations \eqref{eq:cboundprop1}, \eqref{eq:cboundprop1,2} and \eqref{eq:negativebound} yield \[ \max\left((\tfrac12+b\sqrt D-1)[x_n],(\tfrac12-b\sqrt D-1)[x_n] \right) < (c-1)[x_n] \] for all $n\geq 0$. Using this, it is now straightforward to construct a sequence of indices along which the coefficients of $F_D$ go to infinity in the $p$-adic absolute value, similarly to the argument in the proof of Theorem \ref{t:rational}. This concludes the proof. \end{proof} \begin{cor} Assume Conjecture \ref{conj}. Then for a set of parameters with \(2a\in\ZZ \), the series $F_D$ has a Dirichlet density of bounded primes equal to $0$ if and only if \(c<\tfrac12\). \end{cor} \begin{proof} As the set of bounded primes is a union of cyclic subgroups of $(\ZZ/M\ZZ)^\times$, the set of bounded primes is empty if and only if \(1\) is not in the bounded prime set. Thus, the Corollary follows from Proposition \ref{prop1}. \end{proof} Now we treat the case where $2a \not \in \ZZ$. \begin{prop} \label{prop2} Given two rational parameters $a$ and $c$ where $ a \neq \frac{1}{2}$, and a quadratic number field \(K\) defined by $K = \QQ(\sqrt{D})$, taking M to be the least common multiple of the denominators of \(a\), \(c\) and the discriminant of \(K\), consider the hypergeometric series defined by: \[ _2F_1(a+\sqrt D, a-\sqrt D,c ; z) := \sum_{n=0}^\infty \frac{(a+\sqrt D)_n(a-\sqrt D)_n}{(c)_n n!}z^n. \] The following set provides a lower bound on the density of bounded primes in the denominators of the series: \[ B_K(a;c) \df \{u\in \left(\ZZ/M\ZZ\right)^\times {\vert} \{-u^jc\}\leq\tfrac 12\{-2u^ja\} \text{ for all j, and \(u\) splits in K}\}. \] This set is \emph{equal} to the set of all equivalence classes such that $F_D$ is $p$-adically bounded for primes in the corresponding congruence if Conjecture \ref{conj} holds. \end{prop} \begin{proof} Let $p$ be a prime that is large enough, so that $p$ is congruent mod $M$ to an element of $B_K(a;c)$. We will show that our hypergeometric series is $p$-adically bounded for all such primes. Thus, we are assuming that $p$ satisfies \[ \{-p^jc\}\leq \tfrac 12\{-2p^ja\} \] for all $j\in \ZZ$. Multiply by $p$, take floors, and use both $\lfloor \tfrac 12 x\rfloor \leq \tfrac 12 \lfloor x\rfloor$ and Lemma \ref{l:digitformula}: \[ (c-1)[j] \leq \lfloor \tfrac 12\{-2p^ja\}p\rfloor \leq \tfrac 12 \lfloor\{-2p^ja\}p\rfloor \leq\tfrac 12((2a-1)[j]). \] That is, we are assuming that for all $j$ we have \begin{equation} \label{eq:firstdirection} (c-1)[j] \leq \tfrac 12 ((2a-1)[j]). \end{equation} Now, by Lemma \ref{l:digitsum} with $x=a+b\sqrt{D}-1$ and $y=a-b\sqrt{D}-1$, so that $z = x+y = 2a-2$, we deduce that \[ (a+b\sqrt{D}-1)[j]+(a-b\sqrt{D}-1)[j] \geq (2a-2)[j]-1 \] for all $j$. It follows that for all $j$, \[ \max ((a+b\sqrt{D}-1)[j],(a-b\sqrt{D}-1)[j]) \geq \tfrac 12((2a-2)[j]-1) \] For all but finitely many $j$ we have $(2a-2)[j] = (2a-1)[j]$, and so for all but finitely many $j$ we have \[ \max ((a+b\sqrt{D}-1)[j],(a-b\sqrt{D}-1)[j]) \geq \tfrac 12((2a-1)[j]-1). \] Combining this with inequality \eqref{eq:firstdirection}, we obtain that for all but finitely many $j$ we have \[ \max ((a+b\sqrt{D}-1)[j],(a-b\sqrt{D}-1)[j]) \geq (c-1)[j]-\tfrac 12. \] But since the left hand side is an integer, as is $(c-1)[j]$, it follows that there exists an $N$ such that for all $j > N$ we have \begin{equation} \label{eq:maxwithc} \max ((a+b\sqrt{D}-1)[j],(a-b\sqrt{D}-1)[j]) \geq (c-1)[j]. \end{equation} Now again, the proof can be completed in this case using Corollary \ref{c:FGM} and the argument of \cite{FGM}: equation \eqref{eq:maxwithc} assures that carries arising in the formula of Corollary \ref{c:FGM} from $c-1$ are balanced by carries arising from at least one of the numerator parameters $a\pm b\sqrt{D}-1$. Hence, $F_D$ is $p$-adically integral for all such primes congruence to elements of $B_K(a;c)$. Now, assume Conjecture \ref{conj}, and suppose that $p$ is a prime that is not congruent to an element of $B_K(a;c)$. This means that for some $j$ we have \[ \{-p^jc\} > \tfrac12\{-2p^ja\}. \] In the inequality above, both sides are rational numbers such that multiplying by $M$ yields an integer. Thus, the inequality above implies that \(\{-p^jc\} \ge \tfrac12\{-2p^ja\} + \frac1M\). Then we deduce \[ \floor{\{-p^jc\}p} \ge \floor{\tfrac12\{-2p^ja\}p + \frac{p}{M}} \geq \floor{\tfrac12\{-2p^ja\}p} + \floor{\frac{p}{M}}. \] Therefore, by Lemma \ref{l:digitformula} and the above mentioned inequality \(\floor{\tfrac12x}\geq\tfrac12\floor{x}\), we arrive at \begin{equation} \label{e:otherway} (c-1)[j] \geq \tfrac12((2a-1)[j])+\floor{\tfrac pM}. \end{equation} And as above, if $A$ is the order of $p \pmod{M}$, then the expansion of $c-1$ is periodic of period $A$, and the expansion of $2a-1$ is eventually periodic of period $A$. So, after possibly replacing $j$ by some large term $j+nA$ to avoid the part of the expansion of $2a-1$ that is not periodic, we deduce that \begin{equation} \label{eq:cboundprop2} \floor{\tfrac pM}+\tfrac12((2a-1)[j+nA]) \leq (c-1)[j+nA] \end{equation} for all integers $n\geq 0$, since both sequences of digits in inequality \eqref{eq:cboundprop2} are independent of $n$. Up until this point, the proof has been essentially the same as the proof of Proposition \ref{prop1}. At this point the proofs diverge, as now the relationship between the digits of $a+b\sqrt{D}-1$ and $a-b\sqrt{D}-1$ is not as simple as in the case when $a=\tfrac 12$. Let us first observe that if $p$ is large enough, where this restriction on the size only depends on $a$, then all of the $p$-adic digits of $2a-1$ in its periodic part are nonzero by the last claim in Lemma \ref{l:digitformula}. Therefore, we may suppose that $(2a-1)[j] \neq 0$. Choose a constant $K > 1$ such that \[ \frac{1}{KM} < \frac{1}{2p}((2a-1)[x_n]). \] It may look as though this constant depends on $p$, but by Lemma \ref{l:digitformula}, when $p$ is large, the digits of $(2a-1)$ are all approximately equal to rational numbers of the form $\alpha p/M$ for $\alpha =1,2,\ldots, M-1$. Hence when $p$ is large enough, the constant $K$ can be chosen independently of $p$. Conjecture \ref{conj} yields an infinite subsequence $(x_n)$ of the arithmetic progression \((j+nA)\) such that \begin{equation} \label{eq:bsqrtDprop2} \tfrac12((2a-1)[x_n])-\tfrac 1K\floor{\tfrac pM} < (a+b\sqrt D-1)[x_n] < \tfrac12((2a-1)[x_n])+\tfrac 1K\floor{\tfrac pM} \end{equation} for all $n\geq 0$, assuming $p$ is large enough. As in the proof of Lemma 4.7, we have one of the following four possibilities depending on how certain carries take place when adding $a\pm b\sqrt{D}-1$: \[ (a+b\sqrt{D}-1)[x_n] + (a-b\sqrt{D}-1)[x_n] =\begin{cases} (2a-1)[x_n] + \alpha p,\\ (2a-1)[x_n]-1 + \alpha p,\\ \end{cases} \] where $\alpha \in \{0,1,2\}$, for $j$ large enough so that $(2a-1)[x_n]=(2a-2)[x_n]$. However, from equation \eqref{eq:bsqrtDprop2} and the trivial bound $(a-b\sqrt{D}-1)[x_n] \leq p-1$ we obtain \[ (a+b\sqrt{D}-1)[x_n] + (a-b\sqrt{D}-1)[x_n]\leq \tfrac12((2a-1)[x_n])+\tfrac 1K\floor{\tfrac pM} + p-1. \] It follows that \[ \alpha \leq \tfrac {1}{pK}\floor{\tfrac pM} + 1-\tfrac{1}{2p}((2a-1)[x_n]) \leq 1+\tfrac {1}{KM}-\tfrac{1}{2p}((2a-1)[x_n]) < 1, \] where the last inequality follows by our choice of $K$. Therefore, $\alpha=0$ and we have \[ (a+b\sqrt{D}-1)[x_n] + (a-b\sqrt{D}-1)[x_n]\leq (2a-1)[x_n], \] so that if we apply the left side of inequality \eqref{eq:bsqrtDprop2} then \[ (a-b\sqrt{D}-1)[x_n] \leq \tfrac 12((2a-1)[x_n] + \tfrac{1}{K}\floor{\tfrac pM} \] Now, the line above and inequalities \eqref{eq:bsqrtDprop2} and \eqref{eq:cboundprop2} give \[ \max((a+b\sqrt{D}-1)[x_n], (a-b\sqrt{D}-1)[x_n]) < (c-1)[x_n] \] for all $n$ large enough, and thus $F_D$ is unbounded at $p$ using the same argument as in Proposition \ref{prop1} \end{proof} \begin{cor} If Conjecture \ref{conj} holds, then a set of parameters $(a,c)$ as above with \(a\not\in \tfrac12 \ZZ\) has series $F_D$ with a density of bounded primes equal to zero if and only if \(2\{-c\}>\{-2a\}\). \end{cor} \begin{proof} Notice that the set of bounded primes is a union of cyclic subgroups, if nonempty. Therefore, the set of bounded primes is empty if and only if $1$ is not in the bounded prime set. This is equivalent to the statement of the Corollary by the preceding proposition. \end{proof} \section{Towards a quadratic irrational Schwarz list} \label{s:schwarz} Obviously, due to the splitting condition on $p$ in $K = \QQ(\sqrt{D})$, the density \[ D_K(a;c) = \frac{\abs{B_K(a;c)}}{\phi(M)} \] is at most equal to $1/2$. In analogy with the Schwarz list, it is natural to ask how often this maximal density is reached. Computations suggest that frequently we have $D_K(a;c) \leq 1/4$, but this is not always the case. To understand why densities frequently do not exceed $1/4$, let us introduce some more notation: let $M_1$ denote the lcm of the denominators of $a$ and $c$, and let $M$ denote the lcm of $M_1$ and $\Delta_K$, where $\Delta_K$ is the discriminant of our field $K=\QQ(\sqrt{D})$. Define $B(a;c)$ in the same way as $B_K(a;c)$ except we use $M_1$ in place of $M$ and we ignore the splitting condition. Then define \begin{equation} \label{e:dac} D(a;c) = \frac{\abs{B(a;c)}}{\phi(M_1)}. \end{equation} Suppose that $M_1$ and $\Delta_K$ are coprime. Then by the Chinese remainder theorem we have \[ (\ZZ/M\ZZ)^\times \cong (\ZZ/M_1\ZZ)^\times \times (\ZZ/\Delta_K\ZZ)^\times \] and this isomorphism induces a bijection of sets: \begin{equation} \label{eq:coprimeDbound} B_K(a;c) \cong B(a;c) \times \{u\in (\ZZ/\Delta_K\ZZ)^\times \mid u \textrm{ splits in }K\}. \end{equation} \begin{prop} \label{p:Dbound} Suppose that $a,c$ are rationals such that \(2c\) and \(2a\) are not both integers. Then \[ D(a;c) \leq 1/2. \] \end{prop} \begin{proof} We first consider the case where \(2a\not\in\ZZ\). Notice that \[ B(a;c) \subseteq A(a;c) = \{u \in (\ZZ/M_1\ZZ)^\times \mid \{-uc\} \leq \tfrac 12 \{-2ua\}\}. \] It suffices to show that at most one of \(u\) and \(-u\) are in the set \(A(a;c)\). Assume towards a contradiction that both $u$ and $-u$ are in A. Considering $u \in A(a;c)$, we have: \begin{align*} \{-uc\} &\leq \tfrac{1}{2}\{-2ua\}\\ \intertext{Noting $ \{-x\} = 1 - \{x\}$ holds when $x \in \QQ\setminus\ZZ$,} 1 - \{uc\} & \leq \tfrac{1}{2}(1 - \{2ua\})\\ 1 - \{uc\} & \leq \tfrac{1}{2} - \tfrac{1}{2}\{2ua\}\\ - \{uc\} & \leq -\tfrac{1}{2} - \tfrac{1}{2}\{2ua\}\\ \tfrac{1}{2} + \tfrac{1}{2}\{2ua\} & \leq \{uc\}. \end{align*} As $-u \in A(a;c)$, we also have \[\{uc\} \leq \tfrac{1}{2}\{2ua\}.\] Therefore, \[\tfrac{1}{2} + \tfrac{1}{2}\{2ua\} \leq \{uc\} \leq \tfrac{1}{2}\{2ua\},\] a contradiction! Thus only one of \(u\) and \(-u\) are in \(A(a;c)\) in this case. In the case where \(2a\in\ZZ\), we must instead consider \[ B(a;c)\subseteq A(a;c) = \{u\in(\ZZ/M\ZZ)^\times \mid \{-uc\}\le\tfrac12\}. \] Once again, it suffices to show that at most half of all \(u\) can appear in this set. Assume towards a contradiction that \(u\) and \(-u\) are both in this set. Taking \(u\in A(a;c)\), we have: \begin{align*} \{-uc\} &\le \tfrac12\\ 1-\{uc\} &\le \tfrac12\\ -\{uc\} &\le -\tfrac12\\ \tfrac12 &\le \{uc\} \end{align*} By our assumptions, \(-u\in A(a;c)\), so \[ \{uc\} \le \tfrac12. \] Thus, since we have assumed that in this case \(2c\not\in\ZZ\), this is a contradiction. So \(B(a;c)\) contains at most half of the elements of \((\ZZ/M_1\ZZ)^\times\), and so \(D(a;c)\) is at most \(\tfrac12\). \end{proof} Now we can prove that the field $K$ must be ramified at some of the primes dividing our hypergeometric parameters in order to find examples $(a,c,K)$ where $D_K(a;c)=\tfrac 12$: \begin{cor} \label{cor:Dbound} Suppose that $\gcd(\Delta_K,M_1) = 1$ and that $2a$ and $2c$ are not both in $\ZZ$. Then \[ D_K(a;c) \leq 1/4. \] \end{cor} \begin{proof} Combining equation \eqref{eq:coprimeDbound} and Proposition \ref{p:Dbound} we find that \[ D_K(a;c) \leq D(a;c)\times \tfrac 12 \leq \tfrac 14. \] \end{proof} Table \ref{tab:largedensitiesnoK} lists pairs of rational parameters $a,c \in (0,1)$ with $a$ and $c$ of height at most $48$, and where $2a \not \in \ZZ$, such that $D(a,c) > 1/4$. We do not know if there exists an infinite number of such examples as the height of $a$ and $c$ grow. Since we have $D_K(a,c) \leq D(a,c)$ for all fields $K = \QQ(\sqrt{D})$, in each case from Table \ref{tab:largedensitiesnoK}, it is natural to ask whether we can find a field where equality holds. By Corollary \ref{cor:Dbound}, we know that in such examples, $D$ can't be coprime to the least common multiple of the denominators of $a$ and $c$. We thus examined all choices of $K=\QQ(\sqrt{D})$ for $D$ dividing this least common multiple. In all cases of pairs $(a,c)$ from Table \ref{tab:largedensitiesnoK} with $D(a,c)=1/2$, we found exactly one field $K$ such that $D_K(a,c)=1/2$. Further, it was always imaginary quadratic. \nocite{MNT} \nocite{kummer} \begin{table} \resizebox{\textwidth}{!}{ \renewcommand{\arraystretch}{1} \centering \begin{tabular}{p{2.8cm}p{2.8cm}p{2.8cm}p{2.8cm}p{2.8cm}p{2.8cm}} $D(a;c)=\tfrac12$ &$D(a;c)=\tfrac12$ & $D(a;c) = \tfrac38$& $D(a;c) = \tfrac38$&$D(a;c)=\tfrac5{16}$&$D(a;c)=\tfrac13$\\ \begin{tabular}{cc} $a$&$c$ \\ \hline 1/3&5/6\\ 2/3&2/3\\ 2/3&5/6\\ 1/4&3/4\\ 1/4&5/6\\ 3/4&3/4\\ 3/4&5/6\\ 1/6&2/3\\ 1/6&5/6\\ 5/6&5/6\\ 1/8&3/4\\ 1/8&5/8\\ 3/8&7/8\\ 5/8&3/4\\ 5/8&5/8\\ 7/8&7/8\\ 1/12&2/3\\ 1/12&5/6\\ 1/12&7/12\\ 5/12&11/12\\ 7/12&2/3\\ 7/12&5/6\\ 7/12&7/12\\ 11/12&11/12\\ 1/16&5/8\\ 3/16&7/8\\ 9/16&5/8\\ 11/16&7/8\\ 1/20&11/20\\ 3/20&13/20\\ 7/20&17/20\\ 9/20&19/20\\ 11/20&11/20\\ 13/20&13/20\\ 17/20&17/20\\ 19/20&19/20 \end{tabular} & \begin{tabular}{cc} $a$&$c$\\ \hline 1/24&3/4\\ 1/24& 5/6\\ 1/24&7/12\\ 1/24&13/24\\ 5/24&3/4\\ 5/24&11/12\\ 5/24&17/24\\ 7/24&5/6\\ 7/24&19/24\\ 11/24&23/24\\ 13/24&3/4\\ 13/24&5/6\\ 13/24&7/12\\ 13/24&13/24\\ 17/24&3/4\\ 17/24&11/12\\ 17/24&17/24\\ 19/24&5/6\\ 19/24&19/24\\ 23/24&23/24\\ 1/40&11/20\\ 3/40&13/20\\ 7/40&17/20\\ 9/40&19/20\\ 21/40&11/20\\ 23/40&13/20\\ 27/40&17/20\\ 29/40&19/20\\ 1/48&13/24\\ 5/48&17/24\\ 7/48&19/24\\ 11/48&23/24\\ 25/48&13/24\\ 29/48&17/24\\ 31/48&19/24\\ 35/48&23/24 \end{tabular} & \vspace{-9.215cm} \begin{tabular}{cc} $a$&$c$ \\ \hline 1/16&5/6\\ 1/16&7/12\\ 1/16&11/12\\ 1/16&17/24\\ 1/16&19/24\\ 3/16&5/6\\ 3/16&17/24\\ 3/16&19/24\\ 5/16&5/6\\ 5/16&11/12\\ 5/16&23/24\\ 7/16&23/24\\ 9/16&5/6\\ 9/16&7/12\\ 9/16&11/12\\ 9/16&17/24\\ 9/16&19/24\\ 11/16&5/6\\ 11/16&17/24\\ 11/16&19/24\\ 13/16&5/6\\ 13/16&11/12\\ 13/16&23/24\\ 15/16&23/24 \end{tabular} & \vspace{-9.215cm} \begin{tabular}{cc} $a$&$c$ \\ \hline 1/48&5/6\\ 1/48&7/12\\ 1/48&17/24\\ 1/48&23/24\\ 5/48&11/12\\ 5/48&19/24\\ 7/48&5/6\\ 7/48&17/24\\ 7/48&23/24\\ 11/48&19/24\\ 13/48&5/6\\ 17/48&11/12\\ 25/48&7/12\\ 25/48&17/24\\ 25/48&5/6\\ 25/48&23/24\\ 29/48&11/12\\ 29/48&19/24\\ 31/48&5/6\\ 31/48&17/24\\ 31/48&23/24\\ 35/48&19/24\\ 37/48&5/6\\ 41/48&11/12 \end{tabular}& \vspace{-9.215cm} \begin{tabular}{cc} $a$&$c$\\ \hline 1/32&11/12\\ 5/32&23/24\\ 9/32&11/12\\ 13/32&23/24\\ 17/32&11/12\\ 21/32&23/24\\ 25/32&11/12\\ 29/32&23/24\\ 1/40&5/6\\ 3/40&5/6\\ 7/40&5/6\\ 9/40&5/6\\ 21/40&5/6\\ 23/40&5/6\\ 27/40&5/6\\ 29/40&5/6 \end{tabular} & \vspace{-9.215cm} \begin{tabular}{cc} $a$&$c$ \\ \hline 2/7&5/6\\ 11/14&5/6\\ 1/21&5/6\\ 2/39&5/6\\ 11/39&5/6\\ 23/42&5/6 \end{tabular} \end{tabular} } \caption{All examples where $D(a;c) >1/4$ for $0<a,c<1$ with $2a\not \in \ZZ$, and the height of $a$ and $c$ are at most $48$.} \label{tab:largedensitiesnoK} \end{table} \begin{table} \renewcommand{\arraystretch}{1.3} \centering \begin{tabular}{ccc} $a$&$c$&$D$\\ \hline 1/3&5/6&-3\\ 2/3&2/3&-3\\ 2/3&5/6&-3\\ 1/4&3/4&-1\\ 1/4&5/6&-3\\ 3/4&3/4&-1\\ 3/4&5/6&-3\\ 1/6&2/3&-3\\ 1/6&5/6&-3\\ 5/6&5/6&-3\\ 1/8&3/4&-1\\ 1/8&5/8&-2\\ 3/8&7/8&-2\\ 5/8&3/4&-1\\ 5/8&5/8&-2\\ 7/8&7/8&-2\\ 1/12&2/3&-3\\ 1/12&5/6&-3\\ 1/12&7/12&-1\\ 5/12&11/12&-1\\ 7/12&2/3&-3\\ 7/12&5/6&-3\\ 7/12&7/12&-1\\ 11/12&11/12&-1\\ 1/16&5/8&-2\\ 3/16&7/8&-2\\ 9/16&5/8&-2\\ 11/16&7/8&-2\\ 1/20&11/20&-5\\ 3/20&13/20&-5\\ 7/20&17/20&-5\\ 9/20&19/20&-5\\ 11/20&11/20&-5\\ 13/20&13/20&-5\\ 17/20&17/20&-5\\ 19/20&19/20&-5 \end{tabular} \hspace{1cm} \begin{tabular}{ccc} $a$&$c$&$D$\\ \hline 1/24&3/4&-1\\ 1/24& 5/6&-3\\ 1/24&7/12&-1\\ 1/24&13/24&-6\\ 5/24&3/4&-1\\ 5/24&11/12&-1\\ 5/24&17/24&-6\\ 7/24&5/6&-3\\ 7/24&19/24&-6\\ 11/24&23/24&-6\\ 13/24&3/4&-1\\ 13/24&5/6&-3\\ 13/24&7/12&-1\\ 13/24&13/24&-6\\ 17/24&3/4&-1\\ 17/24&11/12&-1\\ 17/24&17/24&-6\\ 19/24&5/6&-3\\ 19/24&19/24&-6\\ 23/24&23/24&-6\\ 1/40&11/20&-5\\ 3/40&13/20&-5\\ 7/40&17/20&-5\\ 9/40&19/20&-5\\ 21/40&11/20&-5\\ 23/40&13/20&-5\\ 27/40&17/20&-5\\ 29/40&19/20&-5\\ 1/48&13/24&-6\\ 5/48&17/24&-6\\ 7/48&19/24&-6\\ 11/48&23/24&-6\\ 25/48&13/24&-6\\ 29/48&17/24&-6\\ 31/48&19/24&-6\\ 35/48&23/24&-6 \end{tabular} \caption{Examples of \(a,c,D\) such that $D_K(a;c) = 1/2$ for \(K = \QQ(D)\).} \label{tab:dvaryk} \end{table} \bibliographystyle{plain} \bibliography{refs} \end{document} \newenvironment{pf}{\begin{proof}\small}{\end{proof}} \newenvironment{sol}{\begin{proof}[Solution.] \small} {\end{proof}} \newenvironment{lm}{\begin{proof}[Lemma]\small}{\end{proof}} \newcommand{\cA}{\mathcal{A}} \newcommand{\cB}{\mathcal{B}} \newcommand{\cC}{\mathcal{C}} \newcommand{\cD}{\mathcal{D}} \newcommand{\cE}{\mathcal{E}} \newcommand{\cF}{\mathcal{F}} \newcommand{\cG}{\mathcal{G}} \newcommand{\cH}{\mathcal{H}} \newcommand{\cI}{\mathcal{I}} \newcommand{\cJ}{\mathcal{J}} \newcommand{\cK}{\mathcal{K}} \newcommand{\cL}{\mathcal{L}} \newcommand{\cM}{\mathcal{M}} \newcommand{\cN}{\mathcal{N}} \newcommand{\cO}{\mathcal{O}} \newcommand{\cP}{\mathcal{P}} \newcommand{\cQ}{\mathcal{Q}} \newcommand{\cR}{\mathcal{R}} \newcommand{\cS}{\mathcal{S}} \newcommand{\cT}{\mathcal{T}} \newcommand{\cU}{\mathcal{U}} \newcommand{\cV}{\mathcal{V}} \newcommand{\cW}{\mathcal{W}} \newcommand{\cX}{\mathcal{X}} \newcommand{\cY}{\mathcal{Y}} \newcommand{\cZ}{\mathcal{Z}} \newcommand{\bA}{\mathbf{A}} \newcommand{\bB}{\mathbf{B}} \newcommand{\bC}{\mathbf{C}} \newcommand{\bD}{\mathbf{D}} \newcommand{\bE}{\mathbf{E}} \newcommand{\bF}{\mathbf{F}} \newcommand{\bG}{\mathbf{G}} \newcommand{\bH}{\mathbf{H}} \newcommand{\bI}{\mathbf{I}} \newcommand{\bJ}{\mathbf{J}} \newcommand{\bK}{\mathbf{K}} \newcommand{\bL}{\mathbf{L}} \newcommand{\bM}{\mathbf{M}} \newcommand{\bN}{\mathbf{N}} \newcommand{\bO}{\mathbf{O}} \newcommand{\bP}{\mathbf{P}} \newcommand{\bQ}{\mathbf{Q}} \newcommand{\bR}{\mathbf{R}} \newcommand{\bS}{\mathbf{S}} \newcommand{\bT}{\mathbf{T}} \newcommand{\bU}{\mathbf{U}} \newcommand{\bV}{\mathbf{V}} \newcommand{\bW}{\mathbf{W}} \newcommand{\bX}{\mathbf{X}} \newcommand{\bY}{\mathbf{Y}} \newcommand{\bZ}{\mathbf{Z}} \newcommand{\frakA}{\mathfrak{A}} \newcommand{\frakB}{\mathfrak{B}} \newcommand{\frakC}{\mathfrak{C}} \newcommand{\frakD}{\mathfrak{D}} \newcommand{\frakE}{\mathfrak{E}} \newcommand{\frakF}{\mathfrak{F}} \newcommand{\frakG}{\mathfrak{G}} \newcommand{\frakH}{\mathfrak{H}} \newcommand{\frakI}{\mathfrak{I}} \newcommand{\frakJ}{\mathfrak{J}} \newcommand{\frakK}{\mathfrak{K}} \newcommand{\frakL}{\mathfrak{L}} \newcommand{\frakM}{\mathfrak{M}} \newcommand{\frakN}{\mathfrak{N}} \newcommand{\frakO}{\mathfrak{O}} \newcommand{\frakP}{\mathfrak{P}} \newcommand{\frakQ}{\mathfrak{Q}} \newcommand{\frakR}{\mathfrak{R}} \newcommand{\frakS}{\mathfrak{S}} \newcommand{\frakT}{\mathfrak{T}} \newcommand{\frakU}{\mathfrak{U}} \newcommand{\frakV}{\mathfrak{V}} \newcommand{\frakW}{\mathfrak{W}} \newcommand{\frakX}{\mathfrak{X}} \newcommand{\frakY}{\mathfrak{Y}} \newcommand{\frakZ}{\mathfrak{Z}} \newcommand{\fraka}{\mathfrak{a}} \newcommand{\frakb}{\mathfrak{b}} \newcommand{\frakc}{\mathfrak{c}} \newcommand{\frakd}{\mathfrak{d}} \newcommand{\frake}{\mathfrak{e}} \newcommand{\frakf}{\mathfrak{f}} \newcommand{\frakg}{\mathfrak{g}} \newcommand{\frakh}{\mathfrak{h}} \newcommand{\fraki}{\mathfrak{i}} \newcommand{\frakj}{\mathfrak{j}} \newcommand{\frakk}{\mathfrak{k}} \newcommand{\frakl}{\mathfrak{l}} \newcommand{\frakm}{\mathfrak{m}} \newcommand{\frakn}{\mathfrak{n}} \newcommand{\frako}{\mathfrak{o}} \newcommand{\frakp}{\mathfrak{p}} \newcommand{\frakq}{\mathfrak{q}} \newcommand{\frakr}{\mathfrak{r}} \newcommand{\fraks}{\mathfrak{s}} \newcommand{\frakt}{\mathfrak{t}} \newcommand{\fraku}{\mathfrak{u}} \newcommand{\frakv}{\mathfrak{v}} \newcommand{\frakw}{\mathfrak{w}} \newcommand{\frakx}{\mathfrak{x}} \newcommand{\fraky}{\mathfrak{y}} \newcommand{\frakz}{\mathfrak{z}} \newcommand{\veps}{\varepsilon} \DeclareMathOperator{\Mod}{\mathbf{Mod}} \DeclareMathOperator{\Set}{\mathbf{Set}} \DeclareMathOperator{\Ring}{\mathbf{Ring}} \DeclareMathOperator{\Group}{\mathbf{Group}} \DeclareMathOperator{\Bigebra}{\mathbf{Bigebra}} \DeclareMathOperator{\kFun}{{k \text{-}\mathbf{Fun}}} \DeclareMathOperator{\Field}{\mathbf{Field}} \DeclareMathOperator{\Scheme}{\mathbf{Scheme}} \DeclareMathOperator{\Vect}{\mathbf{Vect}} \DeclareMathOperator{\Alg}{\mathbf{Alg}} \DeclareMathOperator{\uhp}{\mathcal{H}} \DeclareMathOperator{\rk}{rk} \DeclareMathOperator{\tr}{tr} \DeclareMathOperator{\Tr}{Tr} \DeclareMathOperator{\Gal}{Gal} \DeclareMathOperator{\Hom}{Hom} \DeclareMathOperator{\Spec}{Spec} \DeclareMathOperator{\Sp}{Sp} \DeclareMathOperator{\Spf}{Spf} \DeclareMathOperator{\Aut}{Aut} \DeclareMathOperator{\Lie}{Lie} \DeclareMathOperator{\Vol}{Vol} \DeclareMathOperator{\rank}{rank} \DeclareMathOperator{\jacobson}{\mathfrak J} \DeclareMathOperator{\nilradical}{\mathfrak N} \DeclareMathOperator{\Endo}{End} \DeclareMathOperator{\coker}{coker} \DeclareMathOperator{\im}{im} \DeclareMathOperator{\divv}{div} \DeclareMathOperator{\Divv}{Div} \DeclareMathOperator{\Disc}{Disc} \DeclareMathOperator{\ord}{ord} \DeclareMathOperator{\Pic}{Pic} \DeclareMathOperator{\Frob}{Frob} \DeclareMathOperator{\GL}{GL} \DeclareMathOperator{\PGL}{PGL} \DeclareMathOperator{\SL}{SL} \DeclareMathOperator{\PSL}{PSL} \DeclareMathOperator{\SO}{SO} \DeclareMathOperator{\GSpin}{GSpin} \DeclareMathOperator{\supp}{\text{supp}} \DeclareMathOperator{\chara}{\text{char}} \DeclareMathOperator{\sign}{\text{sign}} \DeclareMathOperator{\disc}{\text{disc}} \DeclareMathOperator{\Sh}{Sh} \DeclareMathOperator{\Sym}{Sym} \DeclareMathOperator{\Frac}{Frac} \DeclareMathOperator{\Hdr}{H_{\text{dR}}} \DeclareMathOperator{\Res}{Res} \DeclareMathOperator{\lngth}{length} \DeclareMathOperator{\Pet}{Pet} \DeclareMathOperator{\reg}{reg} \DeclareMathOperator{\Ind}{Ind} \DeclareMathOperator{\id}{id} \DeclareMathOperator{\Meas}{Meas} \DeclareMathOperator{\cotimes}{\widehat{\otimes}} \DeclareMathOperator{\et}{et} \DeclareMathOperator{\diag}{diag} \newcommand{\cont}{\text{cont}} \newcommand{\doubleto}{ \mathrel{\vcenter{\mathsurround0pt \ialign{##\crcr \noalign{\nointerlineskip}$\rightarrow$\crcr \noalign{\nointerlineskip}$\rightarrow$\crcr } }}} \newcommand*{\df}{\mathrel{\vcenter{\baselineskip0.5ex \lineskiplimit0pt \hbox{\scriptsize.}\hbox{\scriptsize.}}} =} \providecommand{\trans}[1]{\vphantom{#1}^{\intercal}\hspace{-0.5mm}#1} \providecommand{\norm}[1]{\left\lVert#1\right\rVert} \providecommand{\abs}[1]{\left\lvert#1\right\rvert} \providecommand{\pabs}[1]{\left\lvert#1\right\rvert_p} \providecommand{\twomat}[4]{\left(\begin{matrix}#1&#2\\#3&#4\end{matrix}\right)} \providecommand{\stwomat}[4]{\left(\begin{smallmatrix}#1&#2\\#3&#4\end{smallmatrix}\right)} \providecommand{\twovec}[2]{\left(\begin{matrix}#1\\#2\end{matrix}\right)} \providecommand{\stwovec}[2]{\left(\begin{smallmatrix}#1\\#2\end{smallmatrix}\right)} \providecommand{\pseries}[2]{#1[\![ #2 ]\!]} \providecommand{\lseries}[2]{#1(\!( #2 )\!)} \providecommand{\demph}[1]{{\textsc{#1}}} \providecommand{\floor}[1]{{\left\lfloor #1 \right \rfloor}} \newcommand{\Qp}{\mathbf{Q}_p} \newcommand{\Zp}{\mathbf{Z}_p} \newcommand{\Cp}{\mathbf{C}_p} \newcommand{\Fp}{\mathbf{F}_p} \newcommand{\QQ}{\mathbf{Q}} \newcommand{\FF}{\mathbf{F}} \newcommand{\CC}{\mathbf{C}} \newcommand{\Qbar}{\overline{\mathbf{Q}}} \newcommand{\ZZ}{\mathbf{Z}} \newcommand{\PP}{\mathbf{P}} \newcommand{\RR}{\mathbf{R}} \newcommand{\GG}{\mathbf{G}} \newcommand{\NN}{\mathbf{N}} \newcommand{\DD}{\mathbf{D}} \newcommand{\KK}{\mathbf{K}} \newcommand{\adeles}{\mathbf{A}} \newcommand{\ideles}{\adeles^\times} xme}[1]{\footnote{#1}} \newcommand{\cVbar}{\overline{\cV}} \newcommand{\cUbar}{\overline{\cU}} \newcommand{\cObar}{\cO} \newcommand{\cWbar}{\overline{\cW}} \newcommand{\cSbar}{\overline{\cS}} \newcommand{\cEbar}{\overline{\cE}} \newcommand{\dblbs}{\backslash \hspace{-1.5mm}\backslash\hspace{-.5mm}} \newcommand{\dbls}{/ \hspace{-1.5mm}/\hspace{-.5mm}} \renewcommand{\Im}{\operatorname{Im}} \renewcommand{\Re}{\operatorname{Re}} \endinput
2412.02494v2
http://arxiv.org/abs/2412.02494v2
On the hit problem for the polynomial algebra and the algebraic transfer
\PassOptionsToPackage{top=1cm, bottom=1cm, left=2cm, right=2cm}{geometry} \documentclass[final,3p,times]{elsarticle} \makeatletter \def\ps@pprintTitle{ \let\@oddhead\@empty \let\@evenhead\@empty \def\@oddfoot{\reset@font\hfil\thepage\hfil} \let\@evenfoot\@oddfoot } \makeatother \usepackage{amsthm, amssymb, amsfonts, mathrsfs, amsmath} \everymath{\displaystyle} \usepackage{palatino} \usepackage[T5]{fontenc} \usepackage{amscd} \usepackage{graphicx} \usepackage{pb-diagram} \usepackage[x11names]{xcolor} \usepackage[ colorlinks, ]{hyperref} \newcommand\rurl[1]{ \href{http://#1}{\nolinkurl{#1}}} \AtBeginDocument{\hypersetup{citecolor=blue, urlcolor=blue, linkcolor=blue}} \theoremstyle{plain} \newtheorem{thm}{Theorem}[subsection] \newtheorem{dly}[thm]{Theorem} \newtheorem{prop}[thm]{Proposition} \newtheorem{lem}[thm]{Lemma} \newtheorem{corl}[thm]{Corollary} \theoremstyle{definition} \newtheorem{rema}[thm]{Remark} \newtheorem{nota}[thm]{Notation} \renewcommand{\theequation}{\arabic{section}.\arabic{equation}} \newtheorem*{acknow}{Acknowledgments} \theoremstyle{plain} \newtheorem{therm}{Theorem}[section] \newtheorem{conj}[therm]{Conjecture} \newtheorem{propo}[therm]{Proposition} \newtheorem{lema}[therm]{Lemma} \newtheorem{corls}[therm]{Corollary} \theoremstyle{definition} \newtheorem{rems}[therm]{Remark} \newtheorem{cy}[therm]{Note} \newtheorem{notas}[therm]{Notation} \theoremstyle{definition} \newtheorem{dn}{Definition}[subsection] \newtheorem{kh}[dn]{Notation} \newtheorem{note}[thm]{Note} \newtheorem{nx}[thm]{Remark} \everymath{\displaystyle} \def\leq{\leqslant} \def\geq{\geqslant} \def\DD{D\kern-.7em\raise0.4ex\hbox{\char '55}\kern.33em} \renewcommand{\baselinestretch}{1.2} \usepackage{sectsty} \subsectionfont{\fontsize{11.5}{11.5}\selectfont} \makeatletter \def\blfootnote{\xdef\@thefnmark{}\@footnotetext} \makeatother \begin{document} \fontsize{11.5pt}{11.5}\selectfont \begin{frontmatter} \title{{\bf On the hit problem for the polynomial algebra and the algebraic transfer}} \author{\DD\d{\u A}NG V\~O PH\'UC} \address{{\fontsize{10pt}{10}\selectfont Department of Information Technology, FPT University, Quy Nhon A.I Campus,\\ An Phu Thinh New Urban Area, Quy Nhon City, Binh Dinh, Vietnam\\[2.5mm] \textbf{\textit{(Dedicated to my beloved wife and cherished son)}}}\\[2.5mm] \textit{\fontsize{11pt}{11}\selectfont \textbf{ORCID: \url{https://orcid.org/0000-0002-6885-3996}}} } \cortext[]{\href{mailto:[email protected], [email protected]}{\texttt{Email address: [email protected], [email protected]}}} \begin{abstract} Let $\mathcal A$ be the classical, singly-graded Steenrod algebra over the prime order field $\mathbb F_2$ and let $P^{\otimes h}: = \mathbb F_2[t_1, \ldots, t_h]$ denote the polynomial algebra on $h$ generators, each of degree $1.$ Write $GL_h$ for the usual general linear group of rank $h$ over $\mathbb F_2.$ Then, $P^{\otimes h}$ is an $\mathcal A[GL_h]$-module. As is well known, for all homological degrees $h \geq 6$, the cohomology groups ${\rm Ext}_{\mathcal A}^{h, h+\bullet}(\mathbb F_2, \mathbb F_2)$ of the algebra $\mathcal A$ are still shrouded in mystery. The algebraic transfer $Tr_h^{\mathcal A}: (\mathbb F_2\otimes_{GL_h}{\rm Ann}_{\overline{\mathcal A}}[P^{\otimes h}]^{*})_{\bullet}\longrightarrow {\rm Ext}_{\mathcal A}^{h, h+\bullet}(\mathbb F_2, \mathbb F_2)$ of rank $h,$ constructed by W. Singer [Math. Z. \textbf{202} (1989), 493-523], is a beneficial technique for describing the Ext groups. Singer's conjecture about this transfer states that \textit{it is always a one-to-one map}. Despite significant effort, neither a complete proof nor a counterexample has been found to date. The unresolved nature of the conjecture makes it an interesting topic of research in Algebraic topology in general and in homotopy theory in particular. \medskip The objective of this paper is to investigate Singer's conjecture, with a focus on all $h\geq 1$ in degrees $n\leq 10 = 6(2^{0}-1) + 10\cdot 2^{0}$ and for $h=6$ in the general degree $n:=n_s=6(2^{s}-1) + 10\cdot 2^{s},\, s\geq 0.$ Our methodology relies on the hit problem techniques for the polynomial algebra $P^{\otimes h}$, which allows us to investigate the Singer conjecture in the specified degrees. Our work is a continuation of the work presented by Mothebe et al. [J. Math. Res. \textbf{8} (2016), 92-100] with regard to the hit problem for $P^{\otimes 6}$ in degree $n_s$, expanding upon their results and providing novel contributions to this subject. More generally, for $h\geq 6,$ we show that the dimension of the cohit module $\mathbb F_2\otimes_{\mathcal A}P^{\otimes h}$ in degrees $2^{s+4}-h$ is equal to the order of the factor group of $GL_{h-1}$ by the Borel subgroup $B_{h-1}$ for every $s\geq h-5.$ Especially, for the Galois field $\mathbb F_{q}$ ($q$ denoting the power of a prime number), based on Hai's recent work [C. R. Math. Acad. Sci. Paris \textbf{360} (2022), 1009-1026], we claim that the dimension of the space of the indecomposable elements of $\mathbb F_q[t_1, \ldots t_h]$ in general degree $q^{h-1}-h$ is equal to the order of the factor group of $GL_{h-1}(\mathbb F_q)$ by a subgroup of the Borel group $B_{h-1}(\mathbb F_q).$ As applications, we establish the dimension result for the cohit module $\mathbb F_2\otimes_{\mathcal A}P^{\otimes 7}$ in degrees $n_{s+5},\, s > 0.$ Simultaneously, we demonstrate that the non-zero elements $h_2^{2}g_1 = h_4Ph_2\in {\rm Ext}_{\mathcal A}^{6, 6+n_1}(\mathbb F_2, \mathbb F_2)$ and $D_2\in {\rm Ext}_{\mathcal A}^{6, 6+n_2}(\mathbb F_2, \mathbb F_2)$ do not belong to the image of the sixth Singer algebraic transfer, $Tr_6^{\mathcal A}.$ This discovery holds significant implications for Singer's conjecture concerning algebraic transfers. We further deliberate on the correlation between these conjectures and antecedent studies, thus furnishing a comprehensive analysis of their implications. \end{abstract} \begin{keyword} Hit problem; Steenrod algebra; Primary cohomology operations; Algebraic transfer \MSC[2010] 55R12, 55S10, 55S05, 55T15 \end{keyword} \end{frontmatter} \tableofcontents \section{Introduction}\label{s1} \setcounter{equation}{0} Throughout this text, we adopt the convention of working over the prime field $\mathbb F_2$ and denote the Steenrod algebra over this field by $\mathcal A$, unless otherwise stated. Let $V_h$ be the $h$-dimensional $\mathbb F_2$-vector space. It is well-known that the mod 2 cohomology of the classifying space $BV_h$ is given by $P^{\otimes h} := H^{*}(BV_h) = H^{*}((\mathbb RP^{\infty})^{\times h}) = \mathbb F_2[t_1, \ldots, t_h],$ where $(t_1, \ldots, t_h)$ is a basis of $H^{1}(BV_h) = {\rm Hom}(V_h, \mathbb F_2).$ The polynomial algebra $P^{\otimes h}$ is a connected $\mathbb Z$-graded algebra, that is, $P^{\otimes h} = \{P_n^{\otimes h}\}_{n\geq 0},$ in which $P_n^{\otimes h}:= (P^{\otimes h})_n$ denotes the vector subspace of homogeneous polynomials of degree $n$ with $P_0^{\otimes h}\equiv \mathbb F_2$ and $P_n^{\otimes h} = \{0\}$ for all $n < 0.$ We know that the algebra $\mathcal A$ is generated by the Steenrod squares $Sq^{k}$ ($k\geq 0$) and subject to the Adem relations (see e.g., \cite{Walker-Wood}). The Steenrod squares are the cohomology operations satisfying the naturality property. Moreover, they commute with the suspension maps, and therefore, they are stable. In particular, these squares $Sq^k$ were applied to the vector fields on spheres and the Hopf invariant one problem, which asks for which $k$ there exist maps of Hopf invariant $\pm 1$. The action of $\mathcal A$ over $P^{\otimes h} $ is determined by instability axioms. By Cartan's formula, it suffices to determine $Sq^{i}(t_j)$ and the instability axioms give $Sq^{1}(t_j)= t_j^{2}$ while $Sq^{k}(t_j) = 0$ if $k > 1.$ The investigation of the Steenrod operations and related problems has been undertaken by numerous authors (for instance \cite{Brunetti1, Singer1, Silverman, Monks, Wood}). An illustration of the importance of the Steenrod operators is their stability property, which, when used in conjunction with the Freudenthal suspension theorem \cite{Freudenthal}, enables us to make the claim that the homotopy groups $\pi_{k+1}(\mathbb S^{k})$ are non-trivial for $k\geq 2$ (see also \cite{Phuc13} for further details). The Steenrod algebra naturally acts on the cohomology ring $H^{*}(X)$ of a CW-complex $X.$ In several cases, the resulting $\mathcal A$-module structure on $H^{*}(X)$ provides additional information about $X$ (for instance the CW-complexes $\mathbb{C}P^4/\mathbb{C}P^2$ and $\mathbb{S}^6\vee \mathbb{S}^8$ have cohomology rings that agree as graded commutative $\mathbb F_2$-algebras, but are different as modules over $\mathcal A.$ We also refer the readers to \cite{Phuc10-0} for an explicit proof.) Hence the Steenrod algebra is one of the important tools in Algebraic topology. Especially, its cohomology ${\rm Ext}_{\mathcal A}^{*, *}(\mathbb F_2, \mathbb F_2)$ is an algebraic object that serves as the input to the Adams (bigraded) spectral sequence \cite{J.A} and therefore, computing this cohomology is of fundamental importance to the study of the stable homotopy groups of spheres. \medskip The identification of a minimal generating set for the $\mathcal A$-module $P^{\otimes h}$ has been a significant and challenging open problem in Algebraic topology in general and in homotopy theory in particular for several decades. This problem, famously known as the "hit" problem, was first proposed by Frank Peterson \cite{Peterson, Peterson2} through computations for cases where $h<2$ and has since captured the attention of numerous researchers in the field, as evidenced by works such as Kameko \cite{Kameko}, Repka and Selick \cite{Repka-Selick}, Singer \cite{Singer1}, Wood \cite{Wood}, Mothebe et al. \cite{Mothebe0, Mothebe, MKR}, Walker and Wood \cite{Walker-Wood, Walker-Wood2}, Sum \cite{Sum00, Sum1, Sum2, Sum2-0, Sum3, Sum4, Sum5}, Ph\'uc and Sum \cite{P.S1, P.S2}, Ph\'uc \cite{Phuc4, Phuc6, Phuc10, Phuc10-0, Phuc11, Phuc13, Phuc15, Phuc16}, Hai \cite{Hai}), and others. Peterson himself, as well as several works such as \cite{Priddy, Singer, Wood}, have shown that the hit problem is closely connected to some classical problems in homotopy theory. To gain a deeper understanding of this problem and its numerous applications, readers are cordially invited to refer to the excellent volumes written by Walker and Wood \cite{Walker-Wood, Walker-Wood2}. An interesting fact is that, if $\mathbb F_2$ is a trivial $\mathcal A$-module, then the hit problem is essentially the problem of finding a monomial basis for the graded vector space $$QP^{\otimes h}=\big\{QP_n^{\otimes h} := (QP^{\otimes h})_n =P_n^{\otimes h}/\overline{\mathcal A}P_n^{\otimes h} = (\mathbb F_2\otimes_{\mathcal A}P^{\otimes h})_n = {\rm Tor}_{0, n}^{\mathcal A}(\mathbb F_2, P^{\otimes h})\big\}_{n\geq 0}.$$ Here $\overline{\mathcal A}P_n^{\otimes h}:=P_n^{\otimes h}\cap \overline{\mathcal A}P^{\otimes h}$ and $\overline{\mathcal A}$ denotes the set of positive degree elements in $\mathcal A.$ The investigation into the structure of $QP^{\otimes h}$ has seen significant progress in recent years. Notably, it has been able to explicitly describe $QP^{\otimes h}$ for $h\leq 4$ and all $n > 0$ through the works of Peterson \cite{Peterson} for $h = 1, 2$, Kameko \cite{Kameko} for $h = 3$, and Sum \cite{Sum1, Sum2} for $h = 4$. Even so, current techniques have yet to full address the problem. While the information that follows may not be integral to the chief content of this paper, it will be beneficial for readers who desire a more in-depth comprehension of the hit problem. When considering the field $\mathbb F_p$, where $p$ is an odd prime, one must address the challenge of the "hit problem," which arises in the polynomial algebra $\mathbb F_p[t_1, \ldots, t_h] = H^{*}((\mathbb CP^{\infty})^{\times h}; \mathbb F_p)$ on generators of degree $2$. This algebra is viewed as a module over the mod $p$ Steenrod algebra $\mathcal A_p$. Here $\mathbb CP^{\infty}$ denotes the infinite complex projective space. The action of $\mathcal A_p$ on $\mathbb F_p[t_1, \ldots, t_h]$ can be succinctly expressed by $\mathscr P^{p^{j}}(t_i^{r}) = \binom{r}{p^{j}}t_i^{r+p^{j+1}-p^{j}},\, \beta(t_i) = 0$ ($\beta\in \mathcal A_p$ being the Bockstein operator) and the usual Cartan formula. In particular, if we write $r = \sum_{j\geq 0}\alpha_j(r)p^{j}$ for the $p$-adic expansion of $r,$ then $\mathscr P^{p^{j}}(t_i^{r}) \neq 0$ if and only if $\binom{r}{p^{j}}\equiv \alpha_j(r)\, ({\rm mod}\, p)\, \neq 0.$ Since each Steenrod reduced power $\mathscr P^{j}$ is decomposable unless $j$ is a power of $p,$ a homogeneous polynomial $f$ is hit if and only if it can be represented as $\sum_{j \geq 0}\mathscr P^{p^{j}}(f_j)$ for some homogeneous polynomials $f_j\in \mathbb F_p[t_1, \ldots, t_h].$ In other words, $f$ belongs to $\overline{\mathcal A_p}\mathbb F_p[t_1, \ldots, t_h].$ (This is analogous to the widely recognized case when $p = 2.$) To illustrate, let us consider the monomial $t^{p(p+1)-1}\in \mathbb F_p[t].$ Since $\binom{2p-1}{p}\equiv \binom{p-1}{0}\binom{1}{1} \equiv 1\, ({\rm mod}\, p),$ $t^{p(p+1)-1} = \mathscr P^{p}(t^{2p-1}),$ i.e., $t^{p(p+1)-1}$ is hit. Actually, the hit problem for the algebra $ \mathbb F_p[t_1, \ldots, t_h]$ is an intermediate problem of identifying a minimal set of generators for the ring $H^{*}(V; \mathbb F_p) = H^{*}(BV; \mathbb F_p) = \Lambda(V^{\sharp})\otimes_{\mathbb F_p}S(V^{\sharp})$ as a module over $\mathcal A_p.$ Here $\Lambda(V^{\sharp})$ is an exterior algebra on generators of degree $1$ while $S(V^{\sharp})$ is a symmetric algebra on generators of degree $2.$ In both situations, the generators may be identified as a basis for $V^{\sharp}$, the linear dual of an elementary abelian $p$-group $V$ of rank $h$, which can be regarded as an $h$-dimensional vector space over the field $\mathbb F_p.$ Viewed as an algebra over the Steenrod algebra, $S(V^{\sharp})$ can be identified with $H^{*}((\mathbb CP^{\infty})^{\times h}; \mathbb F_p)$. Consequently, the cohomology of $V$ over the field $\mathbb F_p$ can be expressed as $\Lambda(V^{\sharp})\otimes_{\mathbb F_p} \mathbb F_p[t_1, \ldots, t_h].$ Thus, the information about the hit problem for $H^{*}(V; \mathbb F_p)$ as an $\mathcal A_p$-module can usuallly be obtained from the similar information about the hit problem for the $\mathcal A_p$-module $\mathbb F_p[t_1, \ldots, t_h]$ without much difficulty. With a monomial $f = t_1^{a_1}t_2^{a_2}\ldots t_h^{a_h}\in \mathbb F_p[t_1, \ldots, t_h],$ we denote its degree by $\deg(f) = \sum_{1\leq i\leq h}a_i.$ This coincides with the usual grading of $P^{\otimes h}$ for $p = 2.$ Notwithstanding, it is one half of the usual grading of $\mathbb F_p[t_1, \ldots, t_h]$ for $p$ odd. With respect to this grading, Peterson's conjecture \cite{Peterson2} is no longer true for $p$ odd in general. As a case in point, our work \cite{Phuc14} provides a detailed proof that $\alpha((i+1)p^{r}-1+1) = \alpha_r((i+1)p^{r}-1+1) = i+1 > 1,$ but the monomials $t^{(i+1)p^{r}-1}\in \mathbb F_p[t],$ for $1\leq i< p-1,\, r\geq 0,$ are not hit. Returning to the topic of the indecomposables $QP^{\otimes h}_n,$ let $\mu: \mathbb{N} \longrightarrow \mathbb{N}$ be defined by $\mu(n) = \min\bigg\{k \in \mathbb{N}: \alpha(n+k) \leq k\bigg\}$, where $\alpha(n)$ denotes the number of ones in the binary expansion of $n$. In the work of Sum \cite{Sum4}, it has been demonstrated that $\mu(n) = h$ if and only if there exists uniquely a sequence of integers $d_1 > d_2 > \cdots > d_{h-1}\geq d_h > 0$ such that $n = \sum_{1\leq j\leq h}(2^{d_j} - 1).$ On the other side, according to Wood \cite{Wood}, if $\mu(n) > h,$ then $\dim QP^{\otimes h}_{n} = 0.$ This validates also Peterson's conjecture \cite{Peterson2} in general. Singer \cite{Singer1} later proved a generalization of Wood's result, identifying a larger class of hit monomials. In \cite{Silverman}, Silverman makes progress toward proving a conjecture of Singer which would identify yet another class of hit monomials. In \cite{Monks}, Monks extended Wood's result to determine a new family of hit polynomials in $P^{\otimes h}.$ Notably, Kameko \cite{Kameko} showed that if $\mu(2n+h) = h,$ then $QP^{\otimes h}_{2n+h}\cong QP^{\otimes h}_{n}.$ This occurrence elucidates that the surjective map $(\widetilde {Sq^0_*})_{2n+h}: QP^{\otimes h}_{2n+h} \longrightarrow QP^{\otimes h}_{n},\ \mbox{[}u\mbox{]}\longmapsto \left\{\begin{array}{ll} \mbox{[}y\mbox{]}& \text{if $u = \prod_{1\leq j\leq h}t_jy^{2}$},\\ 0& \text{otherwise}, \end{array}\right.$ defined by Kameko himself, transforms into a bijective map when $\mu(2n+h) = h.$ Thus it is only necessary to calculate $\dim QP^{\otimes h}_n$ for degrees $n$ in the "generic" form: \begin{equation}\label{pt} n= k(2^{s} - 1) + r\cdot 2^{s} \end{equation} whenever $k,\, s,\, r$ are non-negative integers satisfying $\mu(r) < k \leq h.$ (For more comprehensive information regarding this matter, kindly refer to Remark \ref{nxpt} in Sect.\ref{s2}.) The dual problem to the hit problem for the algebra $P^{\otimes h}$ is to ascertain a subring consisting of elements of the Pontrjagin ring $H_*(BV_h) = [P^{\otimes h}]^{*}$ that are mapped to zero by all Steenrod squares of positive degrees. This subring is commonly denoted by ${\rm Ann}_{\overline{\mathcal A}}[P^{\otimes h}]^{*}.$ Let $GL_h = GL(V_h)$ be the general linear group. This $GL_h$ acts on $V_h$ and then on $QP_n^{\otimes h}.$ For each positive integer $n,$ denote by $[QP_n^{\otimes h}]^{GL_h}$ the subspace of elements that are invariant under the action of $GL_h.$ It is known that there exists an isomorphism between $(\mathbb F_2\otimes_{GL_h}{\rm Ann}_{\overline{\mathcal A}}[P^{\otimes h}]^{*})_n$ and $[QP_n^{\otimes h}]^{GL_h}$, which establishes a close relationship between the hit problem and the $h$-th algebraic transfer \cite{Singer}, $$Tr_h^{\mathcal A}: (\mathbb F_2\otimes_{GL_h}{\rm Ann}_{\overline{\mathcal A}}[P^{\otimes h}]^{*})_n\longrightarrow {\rm Ext}_{\mathcal A}^{h, h+n}(\mathbb F_2, \mathbb F_2).$$ The homomorphism $Tr_h^{\mathcal A}$ was constructed by William Singer while studying the Ext groups, employing the modular invariant theory. One notable aspect is that the Singer transfer can be regarded as an algebraic formulation of the stable transfer $B(V_h)_+^{S}\longrightarrow \mathbb S^{0}.$ It is a well-established fact, as demonstrated by Liulevicius \cite{Liulevicius}, that there exist squaring operations $Sq^{i}$ for $i\geq 0$ that act on the $\mathbb F_2$-cohomology of the Steenrod algebra $\mathcal A$. These operations share many of the same properties as the Steenrod operations $Sq^{i}$ that act on the $\mathbb F_2$-cohomology of spaces. Nonetheless, $Sq^{0}$ is not the identity. On the other side, there exists an analogous squaring operation $Sq^{0}$, called the Kameko operation, which acts on the domain of the algebraic transfer and commutes with the classical $Sq^{0}$ on ${\rm Ext}_{\mathcal A}^{*,*}(\mathbb F_2, \mathbb F_2)$ thourgh Singer's transfer (see Sect.\ref{s2} for its precise meaning). Hence, the highly non-trivial character of the algebraic transfer establishes it as a tool of potential in the study of the inscrutable Ext groups. Moreover, the hit problem and the Singer transfer have been shown in the papers \cite{Minami0, Minami} to be significant tools for investigating the Kervaire invariant one problem. It is noteworthy that Singer made the following prediction. \begin{conj}[see \cite{Singer}]\label{gtSinger} The transfer $Tr_h^{\mathcal A}$ is a one-to-one map for any $h.$ \end{conj} Despite not necessarily resulting in a one-to-one correspondence, Singer's transfer is a valuable tool for analyzing the structure of the Ext groups. It is established, based on the works of Singer \cite{Singer} and Boardman \cite{Boardman}, that the Singer conjecture is true for homological degrees up to $3$. In these degrees, the transfer is known to be an isomorphism. We are thrilled to announce that our recent works \cite{Phuc12, Phuc10-3, Phuc10-2} has finally brought closure to the complex and long-standing problem of verifying Singer's conjecture in the case of rank four. Our study, detailed in \cite{Phuc12, Phuc10-3, Phuc10-2}, specifically establishes the truth of Conjecture \ref{gtSinger} in the case where $h=4.$ For some information on the interesting case of rank five, we recommend consulting works such as \cite{Phuc4, Phuc6, Phuc10, Phuc10-0, Phuc17, Sum4}. It is essential to underscore that the isomorphism between the domain of the homomorphism $Tr_h^{\mathcal A}$ and $(QP_n^{\otimes h})^{GL_h}$ (the subspace of $GL_h$-invariants of $QP_n^{\otimes h}$) implies that it is sufficient to explore Singer's transfer in internal degrees $n$ of the form \eqref{pt}. \medskip Despite extensive research, no all-encompassing methodology exists for the investigation of the hit problem and Singer's algebraic transfer in every positive degree. Therefore, each computation holds considerable importance and serves as an independent contribution to these subjects. By this reason, our primary objective in this work is to extend the findings of Mothebe et al. \cite{MKR} regarding the hit problem of six variables, while simultaneously verifying Singer's conjecture for all ranks $h\geq 1$ in certain internal degrees. Our methodology is based on utilizing the techniques developed for the hit problem, which have proven to be quite effective in determining the Singer transfer. More precisely, using the calculations in \cite{MKR}, we embark on an investigation of Singer's conjecture for bidegrees $(h, h+n)$, where $h\geq 1$ and $1\leq n\leq 10 = 6(2^{0}-1) + 10\cdot 2^{0}$. Subsequently, we proceed to solve the hit problem for $P^{\otimes 6} = \mathbb F_2[t_1, \ldots, t_6]$ in degrees of the form \eqref{pt}, with $k = 6$ and $r = 1$ (i.e., degree $n:=n_s= 6(2^{s}-1) + 10\cdot 2^{s}$, $s\geq 0$). Furthermore, for $h\geq 6$ and degree $2^{s+4}-h$, we establish that for each $s\geq h-5$. the dimension of the cohit module $QP^{\otimes h}_{2^{s+4}-h}$ is equal to the order of the factor group of $GL_{h-1}$ by the Borel subgroup $B_{h-1}.$ Additionally, utilizing the algebra $\pmb{A}_q$ of Steenrod reduced powers over the Galois field $\mathbb F_{q}$ of $q = p^{m}$ elements, and based on Hai's recent work \cite{Hai}, we assert that for any $h\geq 2,$ the dimension of the cohit module $\mathbb F_q[t_1, \ldots t_h]/\overline{\pmb{A}}_q\mathbb F_q[t_1, \ldots t_h]$ in degree $q^{h-1}-h$ is equal to the order of the factor group of $GL_{h-1}(\mathbb F_q)$ by a subgroup of the Borel group $B_{h-1}(\mathbb F_q).$ As a result, we establish the dimension result for the space $QP^{\otimes 7}$ in degrees $n_{s+5}$, where $s > 0$, and explicitly determine the dimension of the domain of $Tr_6^{\mathcal A}$ in degrees $n_s$. Our findings reveal that $Tr_h^{\mathcal A}$ is an isomorphism in some degrees $\leq 10$ and that $Tr_6^{\mathcal A}$ does not detect the non-zero elements $h_2^{2}g_1 = Sq^{0}(h_1Ph_1) = h_4Ph_2$ and $D_2$ in the sixth cohomology groups ${\rm Ext}_{\mathcal A}^{6, 6+n_s}(\mathbb F_2, \mathbb F_2)$. This finding carries significant implications for Singer's conjecture on algebraic transfers. Specifically, we affirm the validity of Conjecture \ref{gtSinger} for the bidegrees $(h, h+n)$ where $h\geq 1$, $1\leq n\leq n_0$, as well as for any bidegree $(6, 6+n_s).$ It is worth noting that the results have been rigorously verified using \textbf{MAGMA} \cite{Bosma}. \medskip {\bf Organization of the rest of this work.}\ In Sect.\ref{s2}, we provide a brief overview of the necessary background material. The main findings are then presented in Sect.\ref{s3}, with the proofs being thoroughly explained in Sect.\ref{s4}. As an insightful consolidation, Sect.\ref{s5} will encapsulate the core essence of the paper by distilling its key discoveries and notable contributions. Finally, in the appendix (referred to as Sect.\ref{s6}), we provide an extensive list of admissible monomials of degree $n_1 =6(2^{1}-1) + 10\cdot 2^{1}$ in the $\mathcal A$-module $P^{\otimes 6}.$ \begin{acknow} I would like to extend my heartfelt gratitude and appreciation to the editor and the anonymous referees for their meticulous review of my work and insightful recommendations, which greatly contributed to the enhancement of this manuscript. Furthermore, their constructive feedback has served as a source of inspiration for me to elevate the outcomes of my research beyond the initial expectations. I am greatly appreciative of Bill Singer for his keen interest, constructive criticism, and enlightening e-mail correspondence. My sincere thanks are also due to Bob Bruner, John Rognes and Weinan Lin for helpful discussions on the Ext groups. \textbf{A condensed version of this article was published in the Journal of Algebra, 613 (2023), 1--31.} \end{acknow} \section{Several fundamental facts}\label{s2} In order to provide the reader with necessary background information for later use, we present some related knowledge before presenting the main results. Readers may also refer to \cite{Kameko, Phuc4, Sum2} for further information. We will now provide an overview of some fundamental concepts related to the hit problem. \medskip {\bf Weight and exponent vectors of a monomial.}\ For a natural number $k,$ writing $\alpha_j(k)$ and $\alpha(k)$ for the $j$-th coefficients and the number of $1$'s in dyadic expansion of $k$, respectively. Thence, $\alpha(k) = \sum_{j\geq 0}\alpha_j(k),$ and $k$ can be written in the form $\sum_{j\geq 0}\alpha_j(k)2^j.$ For a monomial $t = t_1^{a_1}t_2^{a_2}\ldots t_h^{a_h}$ in $P^{\otimes h},$ we consider a sequence associated with $t$ by $\omega(t) :=(\omega_1(t), \omega_2(t), \ldots, \omega_i(t), \ldots)$ where $\omega_i(t)=\sum_{1\leq j\leq h}\alpha_{i-1}(a_j)\leq h,$ for all $i\geq 1.$ This sequence is called the {\it weight vector} of $t.$ One defines $\deg(\omega(t)) = \sum_{j\geq 1}2^{j-1}\omega_j(t).$ We use the notation $a(t) = (a_1, \ldots, a_h)$ to denote the exponent vector of $t.$ Both the sets of weight vectors and exponent vectors are assigned the left lexicographical order. \medskip {\bf The linear order on \mbox{\boldmath $P^{\otimes h}$.}}\ Consider the monomials $t = t_1^{a_1}t_2^{a_2}\ldots t_h^{a_h}$ and $t' = t_1^{a'_1}t_2^{a'_2}\ldots t_h^{a'_h}$ in the $\mathcal A$-module $P^{\otimes h}$. We define the relation "$<$" between these monomials as follows: $t < t'$ if and only if either $\omega(t) < \omega(t')$, or $\omega(t) = \omega(t')$ and $a(t) < a'(t').$ \medskip {\bf The equivalence relations on \mbox{\boldmath $P^{\otimes h}$.}}\ Let $\omega$ be a weight vector of degree $n$. We define two subspaces of $P_n^{\otimes h}$ associated with $\omega$ as follows: $P_n^{\otimes h}(\omega) = {\rm span}\{ t\in P_n^{\otimes h}|\, \deg(t) = \deg(\omega) = n,\ \omega(t)\leq \omega\},$ and $P_n^{\otimes h}(< \omega) = {\rm span}\{t\in P_n^{\otimes h}|\, \deg(t) = \deg(\omega) = n,\ \omega(t) < \omega\}.$ Let $u$ and $v$ be two homogeneous polynomials in $P_n^{\otimes h}.$ We define the equivalence relations "$\equiv$" and "$\equiv_{\omega}$" on $P_n^{\otimes h}$ by setting: $u\equiv v$ if and only if $(u+v)\in \overline{\mathcal {A}}P_n^{\otimes h}$, while $u \equiv_{\omega} v$ if and only if $u,\, v\in P_n^{\otimes h}(\omega) $ and $$(u +v)\in \overline{\mathcal {A}}P_n^{\otimes h} \cap P_n^{\otimes h}(\omega) + P_n^{\otimes h}(< \omega).$$ (In particular, if $u\equiv 0,$ then $u$ is a hit monomial. If $u \equiv_{\omega} 0,$ then $u$ is called {\it $\omega$-hit}.) We will denote the factor space of $P_n^{\otimes h}(\omega)$ by the equivalence relation $\equiv_{\omega}$ as $QP_n^{\otimes h}(\omega)$. According to \cite{Sum4, Walker-Wood}, this $QP_n^{\otimes h}(\omega)$ admits a natural $\mathbb F_2[GL_h]$-module structure, and the reader is recommended to \cite{Sum4} for a detailed proof. It is noteworthy that if we define $\widetilde{(QP^{\otimes h}_{n})^{\omega}}:= \langle \{[t]\in QP^{\otimes h}_{n}:\ \omega(t) = \omega,\, \mbox{ and $t$ is admissible} \}\rangle,$ then this $\widetilde{(QP^{\otimes h}_{n})^{\omega}}$ is an $\mathbb F_2$-subspace of $QP^{\otimes h}_n.$ Furthermore, the mapping $QP^{\otimes h}_{n}(\omega)\longrightarrow \widetilde{(QP^{\otimes h}_{n})^{\omega}}$ determined by $[t]_{\omega}\longmapsto [t]$ is an isomorphism. This implies that $ \dim QP^{\otimes h}_{n} = \sum_{\deg(\omega) = n}\dim\widetilde{(QP^{\otimes h}_{n})^{\omega}} = \sum_{\deg(\omega) = n}\dim QP^{\otimes h}_{n}(\omega).$ \medskip \begin{cy}\label{cyP} (i) The conjecture proposed by Kameko in the thesis \cite{Kameko} asserts that $\dim QP^{\otimes h}_{n}\leq \prod_{1\leq j\leq h}(2^{j}-1)$ for all values of $h$ and $n$. While this inequality has been proven for $h \leq 4$ and all $n$, counterexamples provided by Sum in \cite{Sum00, Sum2} demonstrate that it is wrong when $h > 4$. It is worth noting, however, that the local version of Kameko's conjecture, which concerns the inequality $\dim QP^{\otimes h}_{n}(\omega)\leq \prod_{1\leq j\leq h}(2^{j}-1)$, remains an open question. (ii) As it is known, the algebra of divided powers $[P^{\otimes h}]^{*} = H_{*}(BV_h)= \Gamma(a_1, a_2, \ldots, a_h)$ is generated by $a_1, \ldots, a_h,$ each of degree $1.$ Here $a_i = a_i^{(1)}$ is dual to $t_i\in P^{\otimes h}_1,$ with duality taken with respect to the basis of $P^{\otimes h}$ consisting of all monomials in $t_1, \ldots, t_h.$ Kameko defined in \cite{Kameko} a homomorphism of $\mathbb F_2[GL_h]$-modules $\widetilde{Sq}^{0}: [P^{\otimes h}]^{*}=H_{*}(BV_h)\longrightarrow [P^{\otimes h}]^{*} = H_{*}(BV_h),$ which is determined by $\widetilde{Sq}^{0}(a_1^{(i_1)}\ldots a_h^{(i_h)}) = a_1^{(2i_1+1)}\ldots a_h^{(2i_h+1)}.$ The dual of this $\widetilde{Sq}^{0}$ induced the homomorphism $(\widetilde {Sq^0_*})_{2n+h}: QP^{\otimes h}_{2n+h} \longrightarrow QP^{\otimes h}_{n}$ (see Sect.\ref{s1}). Further, as $Sq^{2k+1}_*\widetilde{Sq}^{0} = 0$ and $Sq_{*}^{2k}\widetilde{Sq}^{0} = \widetilde{Sq}^{0}Sq_{*}^{k},$ $\widetilde{Sq}^{0}$ maps ${\rm Ann}_{\overline{\mathcal A}}[P^{\otimes h}]^{*}$ to itself. Here we write $Sq^u_*: H_{*}(BV_h)\longrightarrow H_{*-u}(BV_h)$ for the operation on homology which by duality of vector spaces is induced by the square $Sq^{u}: H_{*}(BV_h)\longrightarrow H_{*+u}(BV_h).$ The Kameko $Sq^{0}$ is defined by $Sq^{0}: \mathbb F_2\otimes_{GL_h}{\rm Ann}_{\overline{\mathcal A}}[P^{\otimes h}]^{*}\longrightarrow \mathbb F_2\otimes_{GL_h}{\rm Ann}_{\overline{\mathcal A}}[P^{\otimes h}]^{*},$ which commutes with the classical $Sq^{0}$ on the $\mathbb F_2$-cohomology of $\mathcal A$ through the Singer algebraic transfer. Thus, for any integer $n\geq 1,$ the following diagram commutes: $$ \begin{diagram} \node{(\mathbb F_2\otimes_{GL_h}{\rm Ann}_{\overline{\mathcal A}}[P^{\otimes h}]^{*})_n} \arrow{e,t}{Tr_h^{\mathcal A}}\arrow{s,r}{Sq^0} \node{{\rm Ext}_{\mathcal A}^{h, h+n}(\mathbb F_2, \mathbb F_2)} \arrow{s,r}{Sq^0}\\ \node{(\mathbb F_2\otimes_{GL_h}{\rm Ann}_{\overline{\mathcal A}}[P^{\otimes h}]^{*})_{2n+h}} \arrow{e,t}{Tr_h^{\mathcal A}} \node{{\rm Ext}_{\mathcal A}^{h, 2h+2n}(\mathbb F_2, \mathbb F_2)} \end{diagram}$$ Thus, Kameko's $Sq^0$ is known to be compatible via the Singer transfer with $Sq^0$ on ${\rm Ext}_{\mathcal A}^{*,*}(\mathbb F_2, \mathbb F_2)$. Moreover, the $GL_h$-coinvariants $(\mathbb F_2\otimes_{GL_h}{\rm Ann}_{\overline{\mathcal A}}[P^{\otimes h}]^{*})_n$ form a bigraded algebra and the Singer algebraic transfers $Tr_*^{\mathcal A}$ yield a morphism of bigraded algebras with values in ${\rm Ext}_{\mathcal A}^{*,*}(\mathbb F_2, \mathbb F_2).$ These compatibilities are suggestive of a far closer relationship between these structures. In addition, the operations $Sq^0$ and the algebra structure on ${\rm Ext}_{\mathcal A}^{*,*}(\mathbb F_2, \mathbb F_2)$ are key ingredients in understanding the image of the algebraic transfer. Unfortunately, detecting the image of the Singer transfer by mapping \textit{out} of ${\rm Ext}_{\mathcal A}^{*,*}(\mathbb F_2, \mathbb F_2)$ is not easy. For example, Lannes and Zarati \cite{LZ} constructed an algebraic approximation to the Hurewicz map: for an unstable $\mathcal A$-module $M$ this is of the form ${\rm Ext}_{\mathcal A}^{h, h+n}(\Sigma^{-h}M, \mathbb F_2)\longrightarrow [(\mathbb F_2\otimes_{\mathcal A}\mathscr R_hM)_n]^*,$ where $\mathscr R_h$ is the $h$-th Singer functor (as defined by Lannes and Zarati). However, it is conjectured by H\uhorn ng \cite{Hung0} that this vanishes for $h>2$ in positive stem, an algebraic version of the long-standing and difficult \textit{generalized spherical class conjecture} in Algebraic topology, due to Curtis \cite{Curtis}. In \cite{Hung2}, H\uhorn ng and Powell proved the weaker result that this holds on the image of the transfer homomorphism. This illustrates the difficulty of studying the Singer transfer. An analogous diagram has also been established for the case of odd primes $p$ \cite{Minami}: $$ \begin{diagram} \node{(\mathbb F_p\otimes_{GL_h(\mathbb F_p)}{\rm Ann}_{\overline{\mathcal A_p}}H_*(V; \mathbb F_p))_n} \arrow{e,t}{Tr_h^{\mathcal A_p}}\arrow{s,r}{Sq^{0}} \node{{\rm Ext}_{\mathcal A_p}^{h, h+n}(\mathbb F_p, \mathbb F_p)} \arrow{s,r}{Sq^{0}}\\ \node{(\mathbb F_p\otimes_{GL_h(\mathbb F_p)}{\rm Ann}_{\overline{\mathcal A_p}}H_*(V; \mathbb F_p))_{p(n+h)-h}} \arrow{e,t}{Tr_h^{\mathcal A_p}} \node{{\rm Ext}_{\mathcal A_p}^{h, p(h+n)}(\mathbb F_p, \mathbb F_p)} \end{diagram}$$ Here, the left vertical arrow represents the Kameko $Sq^0,$ and the right vertical one represents the classical squaring operation. Our recent work \cite{Phuc14} proposes a conjecture that \textit{the transfer $Tr_h^{\mathcal A_p}$ is an injective map for all $1\leq h\leq 4$ and odd primes $p$.} We have also established the validity of this conjecture in certain generic degrees. \end{cy} {\bf (Strictly) inadmissible monomial.}\ We say that a monomial $t\in P_n^{\otimes h}$ is {\it inadmissible} if there exist monomials $z_1, z_2,\ldots, z_k\in P_n^{\otimes h}$ such that $z_j < t$ for $1\leq j\leq k$ and $t = \sum_{1\leq j\leq k}z_j + \sum_{m > 0}Sq^{m}(z_m),$ for some $m\in \mathbb N$ and $z_m\in P_{n-m}^{\otimes h},\, m<n.$ Then, $t$ is said to be {\it admissible} if it is not inadmissible. A monomial $t\in P_n^{\otimes h}$ is said to be {\it strictly inadmissible} if and only if there exist monomials $z_1, z_2,\ldots, z_k$ in $P_n^{\otimes h}$ such that $z_j < t$ for $1\leq j \leq k$ and $t = \sum_{1\leq j\leq k}z_j + \sum_{0\leq m \leq s-1}Sq^{2^{m}}(z_m),$ where $s = {\rm max}\{i\in\mathbb N: \omega_i(t) > 0\}$ and suitable polynomials $z_m\in P^{\otimes h}_{n-2^{m}}.$ \medskip Note that every strictly inadmissible monomial is inadmissible but the converse is not generally true. For example, consider the monomial $t = t_1t_2^{2}t_3^{2}t_4^{2}t_5^{6}t_6\in P^{\otimes 6}_{14},$ we see that this monomial is not strictly inadmissible, despite its inadmissibility. This can be demonstrated through the application of the Cartan formula, which yields $$t = Sq^{1}(t_1^{4}t_2t_3t_4t_5^{5}t_6) + Sq^{3}(t_1^{2}t_2t_3t_4t_5^{5}t_6) + Sq^{6}(t_1t_2t_3t_4t_5^{3}t_6) + \sum_{X<t}X.$$ \begin{therm}[{\bf Criteria on inadmissible monomials}]\label{dlKS} The following claims are each true: \begin{itemize} \item[(i)] Let $t$ and $z$ be monomials in $P^{\otimes h}.$ For an integer $r >0,$ assume that there exists an index $i>r$ such that $\omega_i(t) = 0.$ If $z$ is inadmissible, then $tz^{2^r}$ is, too {\rm (see Kameko \cite{Kameko})}; \medskip \item[(ii)] Let $z, w$ be monomials in $P^{\otimes h}$ and let $r$ be a positive integer. Suppose that there is an index $j > r$ such that $\omega_j(z) = 0$ and $\omega_r(z)\neq 0.$ If $z$ is strictly inadmissible, then so is, $zw^{2^{r}}$ {\rm (see Sum \cite{Sum2})}. \end{itemize} \end{therm} \medskip We shall heavily rely on the arithmetic function $\mu: \mathbb{N}\longrightarrow \mathbb{N},$ as well as Kameko's map $(\widetilde{Sq^0_*})_{2n+h}: QP^{\otimes h}_{2n+h}\longrightarrow QP^{\otimes h}_{n}$, both of which are elucidated in Sect. \ref{s1}. The technical theorem below related to the $\mu$-function holds crucial significance. \begin{therm}\label{dlWS} The following statements are each true: \begin{itemize} \item[(i)] {\rm (cf. Sum \cite{Sum4})}. $\mu(n) = r\leq h$ if and only if there exists uniquely a sequence of positive integers $d_1 > d_2 > \cdots > d_{r-1} \geq d_r$ such that $n = \sum_{1\leq i\leq r}(2^{d_i} - 1).$ \medskip \item[(ii)] {\rm (cf. Wood \cite{Wood})}. For each positive integer $n,$ the space $QP^{\otimes h}_n$ is trivial if and only if $\mu(n) > h.$ \medskip \item[(iii)] {\rm (cf. Kameko \cite{Kameko})}. The homomorphism $(\widetilde {Sq^0_*})_{2n+h}$ is an isomorphism of $\mathbb F_2$-vector spaces if and only if $\mu(2n+h) = h.$ \end{itemize} \end{therm} \begin{rems}\label{nxpt} In Sect.\ref{s1}, it was noted that the hit problem needs to be solved only in degrees of the form \eqref{pt}. Furthermore, in \cite[Introduction]{Sum2}, Sum made the remark that for every positive integer $n$, the condition $3\leq \mu(n)\leq h$ holds if and only if there exist uniquely positive integers $s$ and $r$ satisfying $1\leq \mu(n)-2\leq \mu(r) = \alpha(r+\mu(r))\leq \mu(n)-1$ and $n = \mu(n)(2^{s}-1) + r\cdot2^{s}.$ This can be demonstrated straightforwardly by utilizing Theorem \ref{dlWS}(i). Suppose that $\mu(n) = k\geq 3.$ Then, the "only if" part has been shown in \cite{Phuc4}. The "if" part is established as follows: if $n = k(2^s-1) + r\cdot 2^s$ and $1\leq k-2\leq \mu(r) = \alpha(r+\mu(r)) \leq k-1.$ Then, either $\mu(r) = k-2$ or $\mu(r) = k-1.$ We set $\mu(r) = \ell$ and see that by Theorem \ref{dlWS}(i), there exist uniquely a sequence of integers $c_1 > c_2>\ldots>c_{\ell-1} \geq c_{\ell} > 0$ such that $r = 2^{c_1} + 2^{c_2} + \cdots + 2^{c_{\ell-1}} + 2^{c_{\ell}} - \ell.$ Obviously, $\alpha(r + \ell) = \ell,$ and so, $n =k(2^s-1) + r\cdot2^{s} = 2^{c_1 + s} + 2^{c_2 + s} + \cdots + 2^{c_{\ell} + s} + 2^{s}(s-\ell) - s.$ Now, if $\ell = k-2,$ then $n = 2^{c_1 + s} + 2^{c_2 + s} + \cdots + 2^{c_{\ell} + s} + 2^{s}(s-\ell) - s = 2^{c_1 + s} + 2^{c_2 + s} + \cdots + 2^{c_{k-2} + s} + 2^{s} + 2^{s} - k.$ Let $u_i = c_i + s$ with $1\leq i\leq k-2$ and let $u_{k-1} = u_k = s.$ Since $u_1 > u_2 > \cdots > u_{k-2} > u_{k-1} = u_k,$ by Theorem \ref{dlWS}(i), $\mu(n) = k.$ Finally, if $\ell = k-1$ then $ n = 2^{c_1 + s} + 2^{c_2 + s} + \cdots + 2^{c_{k-1} + s} + 2^{s} - k.$ We put $v_i = c_i + s$ where $1\leq i\leq k-1$ and $v_k = s.$ Since $v_1 > v_2 > \cdots > v_{k-1} > v_k,$ according to Theorem \ref{dlWS}(i), one derives $\mu(n) = k.$ \end{rems} {\bf Spike monomial.}\ A monomial $t_1^{a_1}t_2^{a_2}\ldots t_h^{a_h}$ in $P^{\otimes h}$ is called a {\it spike} if every exponent $a_j$ is of the form $2^{\beta_j} - 1.$ In particular, if the exponents $\beta_j$ can be arranged to satisfy $\beta_1 > \beta_2 > \ldots > \beta_{r-1}\geq \beta_r \geq 1,$ where only the last two smallest exponents can be equal, and $\beta_j = 0$ for $ r < j \leq h,$ then the monomial $t_1^{a_1}t_2^{a_2}\ldots t_h^{a_h}$ is called a {\it minimal spike}. \medskip \begin{therm}[see Ph\'uc and Sum \cite{P.S1}]\label{dlPS} All the spikes in $P^{\otimes h}$ are admissible and their weight vectors are weakly decreasing. Furthermore, if a weight vector $\omega = (\omega_1, \omega_2, \ldots)$ is weakly decreasing and $\omega_1\leq h,$ then there is a spike $z\in P^{\otimes h}$ such that $\omega(z) = \omega.$ \end{therm} The subsequent information demonstrates the correlation between minimal spike and hit monomials. \begin{therm}[{\bf Singer's criterion on hit monomials} {\rm \cite{Singer1}}]\label{dlSin} Suppose that $t\in P^{\otimes h}$ and $\mu(\deg(t))\leq h.$ Consequently, if $z$ is a minimal spike in $P^{\otimes h}$ such that $\omega(t) < \omega(z),$ then $t\equiv 0$ (or equivalently, $t$ is hit). \end{therm} It is of importance to observe that the converse of Theorem \ref{dlSin} is generally not valid. As a case in point, let us consider $z = t_1^{31}t_2^{3}t_3^{3}t_4^{0}t_5^{0}\in P^{\otimes 5}_{37}$ and $t = t_1(t_2t_3t_4t_5)^{9}\in P^{\otimes 5}_{37}.$ One has $\mu(37) = 3 < 5,$ and $t = fg^{2^3},$ where $f = t_1t_2t_3t_4t_5$ and $g =t_2t_3t_4t_5.$ Then $\deg(f) = 5 < (2^{3}-1)\mu(\deg(g)),$ and so, due to Silverman \cite[Theorem 1.2]{Silverman2}, we must have $t\equiv 0.$ It can be observed that despite $z$ being the minimal spike of degree $37$ in the $\mathcal A$-module $P^{\otimes 5},$ the weight $\omega(t) = (5,0,0,4,0)$ exceeds the weight of $z,$ which is $\omega(z) = (3,3,1,1,1).$ The reader may also refer to \cite{Phuc16} for further information regarding the cohit module $QP^{\otimes 5}_{37}.$ \begin{notas} We will adopt the following notations for convenience and consistency: \begin{itemize} \item [$\bullet$] Let us denote by $(P^{\otimes h})^{0}:= {\rm span}\bigg\{\prod_{1\leq j\leq h}t_j^{\alpha_j} \in P^{\otimes h}\bigg|\, \prod_{1\leq j\leq h}\alpha_j = 0\bigg\}$ and $(P^{\otimes h})^{> 0}:= {\rm span}\bigg \{\prod_{1\leq j\leq h}t_j^{\alpha_j} \in P^{\otimes h}\bigg|\, \prod_{1\leq j\leq h}\alpha_j > 0\bigg\}.$ It can be readily observed that these spaces are $\mathcal A$-submodules of $P^{\otimes h}.$ Moreover, for each positive degree $n,$ we have $QP_n^{\otimes h} \cong (QP_n^{\otimes h})^0\,\bigoplus\, (QP_n^{\otimes h})^{>0},$ where $(QP_n^{\otimes h})^0:= (Q(P^{\otimes h})^{0})_n = (\mathbb F_2\otimes_{\mathcal A} (P^{\otimes h})^{0})_n$ and $(QP_n^{\otimes h})^{>0}:= (Q(P^{\otimes h})^{>0})_n = (\mathbb F_2\otimes_{\mathcal A} (P^{\otimes h})^{>0})_n$ are the $\mathbb F_2$-subspaces of $QP_n^{\otimes h}.$ \medskip \item [$\bullet$] Given a monomial $t\in P^{\otimes h}_n,$ we write $[t]$ as the equivalence class of $t$ in $QP^{\otimes h}_n.$ If $\omega$ is a weight vector of degree $n$ and $t\in P_n^{\otimes h}(\omega),$ we denote by $[t]_\omega$ the equivalence class of $t$ in $QP^{\otimes h}_n(\omega).$ Noting that if $\omega$ is a weight vector of a minimal spike, then $[t]_{\omega} = [t].$ For a subset $C\subset P^{\otimes h}_n,$ we will often write $|C|$ to denote the cardinality of $C$ and use notation $[C] = \{[t]\, :\, t\in C\}.$ If $C\subset P_n^{\otimes h}(\omega),$ then we denote $[C]_{\omega} = \{[t]_{\omega}\, :\, t\in C\}.$ \medskip \item [$\bullet$] Write $\mathscr {C}^{\otimes h}_{n},\, (\mathscr {C}^{\otimes h}_{n})^{0}$ and $(\mathscr {C}^{\otimes h}_{n})^{>0}$ as the sets of all the admissible monomials of degree $n$ in the $\mathcal A$-modules $P^{\otimes h},$\ $(P^{\otimes h})^{0}$ and $(P^{\otimes h})^{>0},$ respectively. If $\omega$ is a weight vector of degree $n.$ then we put $\mathscr {C}^{\otimes h}_{n}(\omega) := \mathscr {C}^{\otimes h}_{n}\cap P_n(\omega),$\ $(\mathscr {C}^{\otimes h}_{n})^{0}(\omega) := (\mathscr {C}^{\otimes h}_{n})^{0}\cap P_n^{\otimes h}(\omega),$ and $(\mathscr {C}^{\otimes h}_{n})^{>0}(\omega) := (\mathscr {C}^{\otimes h}_{n})^{>0}\cap P_n^{\otimes h}(\omega).$ \end{itemize} \end{notas} \section{Statement of main results}\label{s3} We are now able to present the principal findings of this paper. The demonstration of these results will be exhaustively expounded in subsequent section. As previously alluded to, our examination commences with a critical analysis of the hit problem for the polynomial algebra $P^{\otimes 6}$ in degree $n_s:= 6(2^{s}-1) + 10\cdot 2^{s}$, wherein $s$ is an arbitrary non-negative integer. \medskip {\bf Case $\pmb{s = 0.}$}\ Mothebe et al. demonstrated in \cite{MKR} the following outcome. \begin{therm}[see \cite{MKR}]\label{dlMKR} For each integer $h\geq 2,$ $\dim QP^{\otimes h}_{n_0} = \sum_{2\leq j\leq n_0}C_j\binom{h}{j},$ where $\binom{h}{j} = 0$ if $h < j$ and $C_2 = 2,$ $C_3 = 8,$ $C_4 = 26,$ $C_5 = 50,$ $C_6 = 65,$ $C_7 = 55,$ $C_8 = 28,$ $C_9 = 8,$ $C_{n_0} = 1.$ This means that there exist exactly $945$ admissible monomials of degree $n_0$ in the $\mathcal A$-module $P^{\otimes 6}.$ \end{therm} The following corollary is readily apparent. \begin{corls}\label{hq10-1} \begin{itemize} \item[(i)] One has an isomorphism of $\mathbb F_2$-vector spaces: $ QP^{\otimes 6}_{n_0}\cong \bigoplus_{1\leq j\leq 5} QP^{\otimes 6}_{n_0}(\overline{\omega}^{(j)}),$ where $\overline{\omega}^{(1)}:= (2,2,1),$\ $\overline{\omega}^{(2)}:=(2,4),$\ $\overline{\omega}^{(3)}:=(4,1,1),$\ $\overline{\omega}^{(4)}:=(4,3),$ and $\overline{\omega}^{(5)}:=(6,2).$ \item[(ii)] $(QP^{\otimes 6}_{n_0})^{0}\cong \bigoplus_{1\leq j\leq 4}(QP^{\otimes 6}_{n_0})^{0}(\overline{\omega}^{(j)})$ and $(QP^{\otimes 6}_{n_0})^{>0}\cong \bigoplus_{2\leq j\leq 5}(QP^{\otimes 6}_{n_0})^{>0}(\overline{\omega}^{(j)}),$ and \centerline{\begin{tabular}{c||cccccc} $j$ &$1$ & $2$ & $3$ & $4$& $5$\cr \hline \hline \ $\dim (QP^{\otimes 6}_{n_0})^{0}(\overline{\omega}^{(j)})$ & $400$ & $30$ & $270$ &$180$ &$0$ \cr \hline \hline \ $\dim (QP^{\otimes 6}_{n_0})^{>0}(\overline{\omega}^{(j)})$ & $0$ & $4$ & $10$ &$36$ & $15$ \cr \end{tabular}} \end{itemize} \end{corls} It is worth noting that the epimorphism of $\mathbb{F}_2$-vector spaces, Kameko's squaring operation $(\widetilde {Sq^0_*})_{n_0}: QP^{\otimes 6}_{n_0} \longrightarrow (QP^{\otimes 6}_{2})^{0}$, implies that $QP^{\otimes 6}_{n_0}$ is isomorphic to ${\rm Ker}((\widetilde {Sq^0_*})_{n_0})\bigoplus \psi((QP_{2}^{\otimes 6})^{0})$. Here, $\psi: (QP^{\otimes 6}_{2})^{0}\longrightarrow QP^{\otimes 6}_{n_0}$ is induced by the up Kameko map $\psi: (P^{\otimes 6}_2)^{0}\longrightarrow P^{\otimes 6}_{n_0},\, t\longmapsto t_1t_2\ldots t_6t^{2}$. Hence, by virtue of Corollary \ref{hq10-1}, one has the isomorphisms: ${\rm Ker}((\widetilde {Sq^0_*})_{n_0})\cong \bigoplus_{1\leq j\leq 4}QP^{\otimes 6}_{n_0}(\overline{\omega}^{(j)}),$ and $\psi((QP_{2}^{\otimes 6})^{0}) \cong QP^{\otimes 6}_{n_0}(\overline{\omega}^{(5)}).$ \begin{rems}\label{nxp10} Let us consider the set $\mathcal L_{h, k} = \{J = (j_1, \ldots, j_k):\, 1\leq j_1 < j_2 < \cdots < j_k\leq h \},\ 1\leq k < h.$ Obviously, $|\mathcal L_{h, k}| = \binom{h}{k}.$ For each $J\in \mathcal L_{h, k},$ we define the the homomorphism $\varphi_J: P^{\otimes k}\longrightarrow P^{\otimes h}$ of algebras by setting $\varphi_J(t_{u}) = t_{j_{u}},\ 1\leq u\leq h.$ It is straightforward to see that this homomorphism is also a homomorphism of $\mathcal A$-modules. For each $1\leq k<h,$ we have the isomorphism of $\mathbb F_2$-vector spaces $Q(\varphi_J((P^{\otimes k})^{>0}))_{n}(\omega) = (\mathbb F_2\otimes_{\mathcal A} \varphi_J((P^{\otimes k})^{>0}))_{n}\cong (QP_{n}^{\otimes k})^{>0}(\omega),$ wherein $\omega$ is a weight vector of degree $n.$ As a consequence of this, and based on the work of \cite{Walker-Wood}, we get $$ (QP_{n}^{\otimes h})^{0}(\omega)\cong \bigoplus_{1\leq k\leq h-1}\bigoplus_{J\in\mathcal L_{h, k}}Q(\varphi_J((P^{\otimes k})^{>0}))_{n}(\omega) \cong\bigoplus_{1\leq k\leq h-1}\bigoplus_{1\leq d\leq \binom{h}{k}}(QP_{n}^{\otimes k})^{>0}(\omega), $$ which implies $ \dim (QP_{n}^{\otimes h})^{0}(\omega) = \sum_{1\leq k\leq h-1}\binom{h}{k}\dim (QP_{n}^{\otimes k})^{> 0}(\omega).$ By utilizing Theorem \ref{dlWS}(ii) in combination, we obtain $$ \dim (QP_{n}^{\otimes h})^{0}(\omega) = \sum_{\mu(n)\leq k\leq h-1}\binom{h}{k}\dim (QP_{n}^{\otimes k})^{> 0}(\omega). $$ \end{rems} Through a straightforward calculation utilizing Theorem \ref{dlMKR}, we can claim the following. \begin{corls}\label{hq10-1-0} Let $\overline{\omega}^{(j)}$ be the weight vectors as in Corollary \ref{hq10-1} with $1\leq j\leq 5.$ Then, for each $h\geq 7,$ the dimension of $(QP^{\otimes h}_{n_0})^{>0}(\overline{\omega}^{(j)})$ is determined by the following table: \centerline{\begin{tabular}{c||cccccccccccccccc} $j$ &&$1$ && $2$ && $3$ && $4$&& $5$\cr \hline \hline \ $\dim (QP^{\otimes 7}_{n_0})^{>0}(\overline{\omega}^{(j)})$ && $0$ && $0$ && $0$ &&$20$ &&$35$ \cr \hline \hline \ $\dim (QP^{\otimes 8}_{n_0})^{>0}(\overline{\omega}^{(j)})$ && $0$ && $0$ && $0$ &&$0$ && $20$ \cr \hline \hline \ $\dim (QP^{\otimes h}_{n_0})^{>0}(\overline{\omega}^{(j)}),\, h \geq 9$ && $0$ && $0$ && $0$ &&$0$ && $0$ \cr \end{tabular}} \end{corls} Through a basic computation, in conjunction with Remark \ref{nxp10}, Corollaries \ref{hq10-1}, \ref{hq10-1-0}, as well as the preceding outcomes established by \cite{Peterson}, \cite{Kameko}, and \cite{Sum2}, we are able to deduce the subsequent corollary. \begin{corls}\label{hq10-1-1} Let $\overline{\omega}^{(j)}$ be the weight vectors as in Corollary \ref{hq10-1} with $1\leq j\leq 5.$ Then, for each $h\geq 7,$ the dimension of $(QP^{\otimes h}_{n_0})^{0}(\overline{\omega}^{(j)})$ is given as follows: $$ \dim (QP^{\otimes h}_{n_0})^{0}(\overline{\omega}^{(j)}) = \left\{\begin{array}{ll} 2\bigg[\binom{h}{2} + 4\binom{h}{3} + 6\binom{h}{4}\bigg] + 5\binom{h}{5}&\mbox{if $j = 1$},\\[1mm] 5\binom{h}{5}+ 4\binom{h}{6} &\mbox{if $j = 2$},\\[1mm] 10\bigg[\binom{h}{4}+2\binom{h}{5} + \binom{h}{6}\bigg]&\mbox{if $j = 3$},\\[1mm] 812&\mbox{if $j = 4,\, h =7$},\\[1mm] 4\bigg[\binom{h}{4}+ 5\binom{h}{5}+9\binom{h}{6} + 5\binom{h}{7}\bigg] &\mbox{if $j = 4,\, h \geq 8$},\\[1mm] 105 &\mbox{if $j = 5,\, h =7$},\\[1mm] 700&\mbox{if $j = 5,\, h = 8$},\\[1mm] 5\bigg[3\binom{h}{6}+ 7\binom{h}{7}+ 4\binom{h}{8}\bigg]&\mbox{if $j = 5,\, h \geq 9$}. \end{array}\right.$$ \end{corls} As a direct implication of the findings presented in \cite{MKR}, we derive \begin{corls}\label{hq10-2} For each integer $h\geq 6,$ consider the following weight vectors of degree $n = h+4$: $$\overline{\omega}^{(1,\, h)}:= (h-4, 4),\ \ \overline{\omega}^{(2,\, h)}:= (h-2, 1,1),\ \ \overline{\omega}^{(3,\, h)}:= (h-2, 3),\ \ \overline{\omega}^{(4,\, h)}:= (h, 2).$$ Then, for each rank $h\geq 6,$ the dimension of $(QP^{\otimes h}_{h+4})^{>0}(\overline{\omega}^{(j,\, h)})$ is determined by the following table: \centerline{\begin{tabular}{c||cccccccccccc} $j$ &&$1$ && $2$ && $3$ && $4$\cr \hline \hline \ $\dim (QP^{\otimes h}_{h+4})^{>0}(\overline{\omega}^{(j,\, h)})$ && $\binom{h-1}{4}-1$ && $\binom{h-1}{2}$ && $h\binom{h-2}{2}$ &&$\binom{h}{2}.$ \cr \end{tabular}} \end{corls} Owing to Corollary \ref{hq10-1}, one has $\overline{\omega}^{(j,\, h)} = \overline{\omega}^{(j+1)}$ for $h = 6$ and $1\leq j\leq 4.$ So we can infer that the dimension of $(QP^{\otimes 6}_{n_0})^{>0}(\overline{\omega}^{(j)}),\, 2\leq j\leq 5$ in Corollary \ref{hq10-1} can be derived from Corollary \ref{hq10-2}. Furthermore, in light of Corollaries \ref{hq10-1-0}, \ref{hq10-1-1}, \ref{hq10-2}, as well as the previous results established by Peterson \cite{Peterson}, Kameko \cite{Kameko}, Sum \cite{Sum2}, and Mothebe et al. \cite{MKR}, we are also able to confirm that a local version of Kameko's conjecture (as articulated in Note \ref{cyP}) holds true for certain weight vectors of degrees $h+4$, where $h\geq 1.$ \medskip As is well-known, Mothebe et al. \cite{MKR} computed the dimension of $QP_n^{\otimes h}$ for $h\geq 1$ and degrees $n$ satisfying $1\leq n\leq 9$. The following theorem provides more details. \begin{therm}[see \cite{MKR}]\label{dlMM} Given any $h\geq 1,$ the dimension of $QP_n^{\otimes h}$ is determined as follows: $$ \begin{array}{ll} \medskip \dim QP_1^{\otimes h} &=h ,\ \ \dim QP_2^{\otimes h} = \binom{h}{2},\ \ \dim QP_3^{\otimes h} = \sum_{1\leq j\leq 3}\binom{h}{j},\\ \medskip \dim QP_4^{\otimes h} &= 2\binom{h}{2} + 2\binom{h}{3}+\binom{h}{4},\ \ \dim QP_5^{\otimes h} = 3\binom{h}{3} + 3\binom{h}{4}+\binom{h}{5},\\ \medskip \dim QP_6^{\otimes h} &=\binom{h}{2} + 3\binom{h}{3}+6\binom{h}{4} + 4\binom{h}{5} + \binom{h}{6},\\ \medskip \dim QP_7^{\otimes h} &= \binom{h}{1} + \binom{h}{2}+4\binom{h}{3} + 9\binom{h}{4} + 10\binom{h}{5}+ 5\binom{h}{6} + \binom{h}{7},\\ \medskip \dim QP_8^{\otimes h} &= 3\binom{h}{2}+6\binom{h}{3} + 13\binom{h}{4} + 19\binom{h}{5}+ 15\binom{h}{6} + 6\binom{h}{7} + \binom{h}{8},\\ \medskip \dim QP_9^{\otimes h} &= 7\binom{h}{3} + 18\binom{h}{4} + 31\binom{h}{5}+ 34\binom{h}{6} + 21\binom{h}{7} + 7\binom{h}{8}+\binom{h}{9}, \end{array}$$ wherein the binomial coefficients $\binom{h}{k}$ are to be interpreted modulo 2 with the usual convention $\binom{h}{k} = 0$ if $k \geq h+1.$ \end{therm} The theorem has also been demonstrated by Peterson \cite{Peterson} for $h\leq 2$, by Kameko's thesis \cite{Kameko} for $h = 3$ and by Sum \cite{Sum2} for $h =4.$ Using Theorems \ref{dlMKR}, \ref{dlMM}, and Corollaries \ref{hq10-1-0}, \ref{hq10-1-1}, we aim to analyze the behavior of the Singer transfer in bidegree $(h, h+n)$ for $1\leq n\leq n_0$ and any $h\geq 1.$ As a result of our investigation, we establish the following first main result. \begin{therm}\label{dlc0} For any integer $n$ satisfying $1\leq n\leq n_0,$ the algebraic transfer $$Tr_h^{\mathcal A}: (\mathbb F_2\otimes_{GL_h}{\rm Ann}_{\overline{\mathcal A}}[P^{\otimes h}]^{*})_{n}\longrightarrow {\rm Ext}_{\mathcal A}^{h, h+n}(\mathbb F_2, \mathbb F_2)$$ is a trivial isomorphism for all $h\geq 1$, except for the cases of rank 5 in degree 9 and rank 6 in degree $n_0$. In these exceptional cases, $Tr_h^{\mathcal A}$ is a monomorphism. Consequently, Singer's Conjecture \ref{gtSinger} holds true in bidegrees $(h, h+n)$ for $h\geq 1$ and $1\leq n\leq n_0.$ \end{therm} The theorem has been proven by Singer \cite{Singer} for $1\leq h\leq 2$, by Boardman \cite{Boardman} for $h = 3$, by Sum \cite{Sum2-1} and the author \cite{Phuc12} for $h = 4,$ by Sum \cite{Sum0, Sum2-0, Sum3} for $h =5$ and $n = 4,\, 5,\, n_0,$ by Sum and T\'in \cite{Sum-Tin0, Sum-Tin} for $h = 5$ and $n =1,\, 2,\, 3,\, 7,\, 9.$ The present writer has established the theorem for the case $h=5$ and degree $n=6, 8,$ as well as for the cases $6\leq h\leq 8$ and any degree $n$, as shown in \cite{Phuc0, Phuc6, Phuc11}. It should be brought to the attention of the readers that, $Tr_h^{\mathcal A}$ is not an epimorphism for rank 5 in degree 9 \cite{Singer}, and also for rank 6 in degree $n_0$ \cite{CHa, CHa0}. These imply that $Ph_1\not\in {\rm Im}(Tr_5^{\mathcal A})$ and $h_1Ph_1\not\in {\rm Im}(Tr_6^{\mathcal A}),$ wherein $\{Ph_1\}\subset {\rm Ext}_{\mathcal A}^{5, 5+9}(\mathbb F_2, \mathbb F_2)$ and $\{h_1Ph_1\}\subset {\rm Ext}_{\mathcal A}^{6, 6+n_0}(\mathbb F_2, \mathbb F_2)$ are sets that generate ${\rm Ext}_{\mathcal A}^{5, 5+9}(\mathbb F_2, \mathbb F_2)$ and ${\rm Ext}_{\mathcal A}^{6, 6+n_0}(\mathbb F_2, \mathbb F_2),$ respectively. \medskip {\bf Case $\pmb{s = 1}.$}\ We notice that $n_1 = 6(2^{1}-1) + 10\cdot 2^{1} = 26$ and make the following observation. \begin{rems}\label{nxp0} Let us consider the Kameko map $(\widetilde {Sq^0_*})_{n_1}: QP^{\otimes 6}_{n_1} \longrightarrow QP^{\otimes 6}_{n_0}$ which is an epimorphism of the $\mathbb F_2$-vector spaces and is determined by $(\widetilde {Sq^0_*})_{n_1}([u]) = [t]$ if $u = t_1t_2\ldots t_6t^{2}$ and $(\widetilde {Sq^0_*})_{n_1}([u]) = 0$ otherwise. Then, since the homomorphism $q: {\rm Ker}((\widetilde {Sq^0_*})_{n_1})\longrightarrow QP^{\otimes 6}_{n_1}$ is an embedding, we have a short exact sequence of $\mathbb F_2$-vector spaces.: $0\longrightarrow {\rm Ker}((\widetilde {Sq^0_*})_{n_1})\longrightarrow QP^{\otimes 6}_{n_1}\longrightarrow QP^{\otimes 6}_{n_0}\longrightarrow 0.$ Let us consider the up Kameko map $\psi: P_{n_0}^{\otimes 6}\longrightarrow P_{n_1}^{\otimes 6},$ which is determined by $\psi(t) = t_1t_2\ldots t_6t^{2}$ for any $t\in P_{n_0}^{\otimes 6}.$ This $\psi$ induces a homomorphism $\psi: QP_{n_0}^{\otimes 6}\longrightarrow QP_{n_1}^{\otimes 6}.$ These data imply that the above exact sequence is split and so, $QP^{\otimes 6}_{n_1} \cong {\rm Ker}((\widetilde {Sq^0_*})_{n_1})\bigoplus QP^{\otimes 6}_{n_0}.$ Furthermore, as well known, $(QP^{\otimes 6}_{n_1})^{0}$ and ${\rm Ker}((\widetilde {Sq^0_*})_{n_1})\cap (QP^{\otimes 6}_{n_1})^{>0}$ are the $\mathbb F_2$-vector subspaces of ${\rm Ker}((\widetilde {Sq^0_*})_{n_1}$ and $QP^{\otimes 6}_{n_1}\cong (QP^{\otimes 6}_{n_1})^{0}\bigoplus (QP^{\otimes 6}_{n_1})^{>0},$ one gets $$ QP^{\otimes 6}_{n_1} \cong (QP^{\otimes 6}_{n_1})^{0}\bigoplus \big({\rm Ker}((\widetilde {Sq^0_*})_{n_1})\cap (QP^{\otimes 6}_{n_1})^{>0}\big)\bigoplus QP^{\otimes 6}_{n_0}. $$ Through the combination of the previously mentioned remark along with the utilization of Corollary \ref{hq10-1}, we arrive at the following conclusion. \begin{corls}\label{hqMKR} We have an isomorphism of $\mathbb F_2$-vector spaces: $$ QP_{n_0}^{\otimes 6}\cong \langle \{[t_1t_2\ldots t_6t^{2}]:\, t\in \mathscr C^{\otimes 6}_{n_0}\} \rangle \cong \bigoplus_{1\leq j\leq 5}(QP^{\otimes 6}_{n_1})^{>0}(\overline{\omega}^{(j)}),$$ where $\overline{\omega}^{(1)}:= (6,2,2,1),$\ $\overline{\omega}^{(2)}:=(6,2,4),$\ $\overline{\omega}^{(3)}:= (6,4,1,1),$\ $\overline{\omega}^{(4)}:=(6,4,3)$ and $\overline{\omega}^{(5)}:=(6,6,2),$ and the dimension of $(QP^{\otimes 6}_{n_1})^{>0}(\overline{\omega}^{(j)}) $ is determined by $$\dim (QP^{\otimes 6}_{n_1})^{>0}(\overline{\omega}^{(j)}) = \dim (QP^{\otimes 6}_{n_0})^{0}(\overline{\omega}^{(j)})+\dim (QP^{\otimes 6}_{n_0})^{>0}(\overline{\omega}^{(j)}),\ \mbox{for $1\leq j\leq 5.$}$$ Here the dimensions of $(QP^{\otimes 6}_{n_0})^{0}(\overline{\omega}^{(j)})$ and $(QP^{\otimes 6}_{n_0})^{>0}(\overline{\omega}^{(j)})$ are given as in Corollary \ref{hq10-1}. \end{corls} \end{rems} We must now determine the dimensions of $(QP^{\otimes 6}_{n_1})^{0}$ and ${\rm Ker}((\widetilde {Sq^0*})_{n_1})\cap (QP^{\otimes 6}_{n_1})^{>0}.$ To accomplish this, we invoke a well-known outcome concerning the dimension of $QP^{\otimes 5}$ at degree $n_1$. \begin{therm}[see Walker and Wood \cite{Walker-Wood2}]\label{dlWW} In any minimal generating set for the $\mathcal A$-module $P^{\otimes h},$ there are $2^{\binom{h}{2}}$ elements in degree $2^{h}-(h+1).$ Consequently, $QP^{\otimes 5}_{n_1}$ is an $\mathbb F_2$-vector of dimension $1024.$ \end{therm} Walker and Wood proved this by considering the special case of the Steinberg representation ${\rm St}_h,$ using the hook formula to count the number of semistandard Young tableaux. More precisely, they claim that by the hook formula, the dimension of the cohit module $QP^{\otimes h}_{2^{h}-h-1}$ is upper bounded by $2^{\binom{h}{2}}.$ The equality then follows from the first occurrence of the Steinberg representation in this degree. Thus, $QP^{\otimes h}_{2^{h}-h-1}\cong {\rm St}_h$ for the first occurrence degree $2^{h}-h-1.$ It would also be interesting to see that the dimension of this cohit module is equal to the order of the Borel subgroup $B_h$ of $GL_h.$ \medskip We also employ the following homomorphisms: Let $h$ be a fixed integer with $5\leq h\leq 6$, and for each $l\in\mathbb Z$ such that $1\leq l\leq h$, we define a homomorphism $\mathsf{q}_{l}: P^{\otimes (h-1)}\longrightarrow P^{\otimes h}$ of algebras by setting $\mathsf{q}_{l}(t_j) = t_j$ for $1\leq j \leq l-1$ and $\mathsf{q}_{l}(t_j) = t_{j+1}$ for $l\leq j \leq h-1.$ Obviously, this $\mathsf{q}_{l}$ is also a homomorphism of $\mathcal A$-modules. The following comment is a crucial factor in computing $(QP^{\otimes 6}_{n_1})^{0}$ and ${\rm Ker}((\widetilde {Sq^0*})_{n_1})\cap (QP^{\otimes 6}_{n_1})^{>0}.$ \begin{rems}\label{nxp1} \begin{itemize} \item[(i)] It is patently obvious that the weight vector of the minimal spike $t_1^{15}t_2^{7}t_3^{3}t_4$ of degree $n_1$ in the $\mathcal A$-module $P^{\otimes 6}$ is $(4,3,2,1),$ In \cite{MKR}, Mothebe et.al proved that the cohit $QP^{\otimes 6}$ has dimension $1205$ in degree $11.$ So, $\omega(\underline{t})\in\big\{(3,2,1),\, (3,4),\, (5,1,1),\, (5,3)\big\}.$ As an immediate consequence of these results and Theorems \ref{dlKS}(i), \ref{dlSin}, we state that if $t$ is an admissible monomial in $P^{\otimes 6}_{n_1}$ such that $[t]$ belongs to the kernel of Kameko's map $(\widetilde {Sq^0_*})_{n_1},$ then $\omega(t)\in \big\{(4, 3,2,1),\ (4, 3,4),\ (4, 5,1,1),\ (4, 5,3)\big\}$ and $t$ can represented as $t_it_jt_kt_{\ell}\underline{t}^2,$ where $\underline{t}$ is an admissible monomial of degree $11$ in $P^{\otimes 6}$ and $1\leq i<j<k<\ell\leq 6.$ \item[(ii)] Since $QP^{\otimes 6}_{n_1}(\omega) \cong (QP^{\otimes 6}_{n_1})^{0}(\omega)\bigoplus (QP^{\otimes 6}_{n_1})^{> 0}(\omega),$ where $\omega$ is a weight vector of degree $n_1,$ one obtains an isomorphism: $QP^{\otimes 6}_{n_1} \cong (QP^{\otimes 6}_{n_1})^{0}\bigoplus \big(\bigoplus_{\deg(\omega)=n_1}(QP^{\otimes 6}_{n_1})^{>0}(\omega)\big).$ On the other hand, by Remark \ref{nxp0} and Corollary \ref{hqMKR}, we infer that $$QP^{\otimes 6}_{n_1} \cong (QP^{\otimes 6}_{n_1})^{0}\bigoplus \big({\rm Ker}((\widetilde {Sq^0_*})_{n_1})\cap (QP^{\otimes 6}_{n_1})^{>0}\big)\bigoplus \big(\bigoplus_{1\leq j\leq 5}(QP^{\otimes 6}_{n_1})^{>0}(\omega^{(j)})\big),$$ where $(QP^{\otimes 6}_{n_1})^{0}\subset {\rm Ker}((\widetilde {Sq^0_*})_{n_1}).$ Hence, by invoking the aforementioned argument (i), an isomorphism will be established as follows: ${\rm Ker}((\widetilde {Sq^0_*})_{n_1})\cap (QP^{\otimes 6}_{n_1})^{>0}\cong U_1\bigoplus U_2,$ wherein $$U_1:=(QP^{\otimes 6}_{n_1})^{>0}(4,5,1,1)\bigoplus (QP^{\otimes 6}_{n_1})^{>0}(4,5,3),\ \mbox{ and }\ U_2:= (QP^{\otimes 6}_{n_1})^{>0}(4,3,2,1)\bigoplus (QP^{\otimes 6}_{n_1})^{>0}(4,3,4).$$ \end{itemize} \end{rems} Drawing on Remark \ref{nxp1}, we establish the second main result of this paper. \begin{therm}\label{dlc1} With the above notation, the following assertions are true: \begin{itemize} \item[(i)] $ \dim (QP^{\otimes 6}_{n_1})^{0}(\omega) = \left\{\begin{array}{ll} 5184&\mbox{if $\omega = (4,3,2,1)$},\\[1mm] 0&\mbox{if $\omega\neq (4,3,2,1)$}. \end{array}\right.$\\[1mm] Consequently, $(QP^{\otimes 6}_{n_1})^{0}$ is isomorphic to $(QP^{\otimes 6}_{n_1})^{0}(4,3,2,1)$ and $(\mathscr C^{\otimes 6}_{n_1})^{0} = (\mathscr C^{\otimes 6}_{n_1})^{0}(4,3,2,1)$ has all $5184$ admissible monomials. \item[(ii)] $\dim U_1 = 546$ and $\dim U_2 = 3090.$ These imply that there exist exactly $9765$ admissible monomials of degree $n_1$ in the $\mathcal A$-module $P^{\otimes 6}.$ Consequently, the cohit $QP^{\otimes 6}_{n_1}$ is $9765$-dimensional. \end{itemize} \end{therm} We will now recall a previously established result on the Kameko squaring operation. \begin{therm}[see Kameko \cite{Kameko}]\label{dlMK} The homomorphism $(\widetilde {Sq^0_*})_{2n+h}: QP^{\otimes h}_{2n+h} \longrightarrow QP^{\otimes h}_{n}$ is an isomorphism of the $\mathbb F_2$-vector spaces if and only if $\mu(2n+h) = h.$ Then, one has an inverse homomorphism $ \psi: QP^{\otimes h}_{n}\longrightarrow QP^{\otimes h}_{2n+h}$ of $(\widetilde {Sq^0_*})_{2n+h},$ which is induced by the mapping $\psi: P^{\otimes h}\longrightarrow P^{\otimes h},$ $t\longmapsto \prod_{1\leq j\leq h}t_jt^{2}.$ \end{therm} Write $\mathbb F_q$ for the Galois field of size $q$ ($q$ being a power of the prime characteristic $p$ of this field), let $B_h(\mathbb F_q)$ be the Borel subgroup of the general linear group $GL_h(\mathbb F_q)$ over $\mathbb F_q.$ When $q = 2,$ we put $GL_h:= GL_h(\mathbb F_2)$ and $B_h:= B_h(\mathbb F_2).$ Note that $\mathbb F_{q = p^{m}}\cong \mathbb F_p^{\oplus m}$ as groups (in fact as $\mathbb F_p$-modules). For the sake of completeness, let us remind the readers that the algebra of Steenrod $q$-th reduced powers $\pmb {A}_q$ can be defined as an algebra over $\mathbb F_q$ by generators $\mathscr P^{j},\, j\geq 0,$ subject to the relation $\mathscr P^{0} = 1$ and the Adem relations, $\mathscr P^{a}\mathscr P^{b} = \sum_{0\leq j\leq [a/q]}(-1)^{i+j}\binom{(q-1)(b-j)-1}{a-qj}\mathscr P^{a+b-j}\mathscr P^{j},\ a<qb.$ For $q = p,$ as a subalgebra of the mod $p$ Steenrod algebra $\mathcal A_p,$ the element $\mathscr P^{k}$ is given the degree $2k(p-1),$ but for simplicity, one regrade $\pmb{A}_q$ by giving $\mathscr P^{j}$ the "reduced" degree $k.$ So, when $q = p =2,$ $\mathscr P^{k}$ will mean Steenrod squares $Sq^{k},$ and not $Sq^{2k}$ (see also \cite{Smith}). Consider an $h$-dimensional vector space $\pmb{V}_h$ over $\mathbb F_q,$ the symmetric power algebra $S(\pmb{V}^{*}_h)$ on the dual $\pmb{V}_h^{*} = {\rm Hom}_{\mathbb F_q}(\pmb{V}_h, \mathbb F_q)$ of $\pmb{V}_h$ is identified with the polynomial algebra $\mathbb F_q[t_1, \ldots, t_h],$ where $\deg(t_i) = 1$ for every $i$ and $\{t_1, \ldots, t_h\}$ is a basis of $\pmb{V}^{*}_h.$ Applying Theorem \ref{dlMK} in conjunction with the work by Hai \cite{Hai}, we derive the following corollaries. \begin{corls} In degree $q^{h-1}-h,$ we have $$ \dim_{\mathbb F_q} (\mathbb F_q[t_1, \ldots t_h]/\overline{\pmb{A}}_q\mathbb F_q[t_1, \ldots t_h])_{q^{h-1}-h} = {\rm ord}(GL_{h-1}(\mathbb F_q)/B^{*}_{h-1}(\mathbb F_q)) = \prod_{1\leq j\leq h-1}(q^{j}-1),$$ on which $B_{h-1}^{*}(\mathbb F_q)\subset B_{h-1}(\mathbb F_q)\cap {\rm Ker}(\det)$ and each element of $B_{h-1}^{*}(\mathbb F_q)$ has 1's in the main diagonal. Here $\det$ denotes the $\mathbb F_q$-linear map $GL_{h-1}(\mathbb F_q)\longrightarrow \mathbb F_q^{*}.$ \end{corls} \begin{corls}\label{hqs22} Let $h \geq 6$ be a given fixed integer. Setting $n_{h, s}:=2^{s+4}-h,$ then, for each $s\geq h-5,$ we have $$ \dim QP^{\otimes h}_{n_{h, s}} = {\rm ord}(GL_{h-1}/B_{h-1}) = \prod_{1\leq j\leq h-1}(2^{j}-1).$$ Moreover, $QP^{\otimes h}_{n_{h, s}}\cong {\rm Ker}((\widetilde {Sq^0_*})_{n_{h, h-5}})\bigoplus QP^{\otimes h}_{2^{h-2}-h}$ for any $s\geq h-5.$ \end{corls} Indeed, we have that the order of the Borel subgroup $B_h(\mathbb F_q)$ is $q^{\binom{h}{2}}\prod_{1\leq j\leq h}(q-1),$ since elements in the main diagonal are taken from $\mathbb F_q^{*}$ and elements above to the main diagonal can be any element of $\mathbb F_q.$ The order of $GL_h(\mathbb F_q)$ is determined as follows: the first row $u_1$ of the matrix can be anything but the $0$-vector, so there are $q^{h}-1$ possibilities for the first row. For any one of these possibilities, the second row $u_2$ can be anything but a multiple of the first row, giving $q^{h}-q$ possibilities. For any choice $u_1,\, u_2$ of the first two rows, the third row can be anything but a linear combination of $u_1$ and $u_2.$ The number of linear combinations $\sum_{1\leq i\leq 2}\gamma_iu_i$ is just the number of choices for the pair $(\gamma_1,\gamma_2),$ and there are $q^{2}$ of these. It follows that for every $u_1$ and $u_2,$ there are $q^{h}-q^{2}$ possibilities for the third row. For any allowed choice $u_1, u_2, u_3,$ the fourth row can be anything except a linear combination $\sum_{1\leq i\leq 3}\gamma_iu_i$ of the first three rows. Thus for every allowed $u_1, u_2, u_3$ there are $q^{3}$ forbidden fourth rows, and therefore $q^{h}-q^{3}$ allowed fourth rows. In the same way, the number of non-singular matrices is $(q^{h}-1)(q^{h}-q)\ldots (q^{h}-q^{h-1}),$ and so, $${\rm ord}(GL_h(\mathbb F_q)) = \prod_{0\leq j\leq h-1}(q^{h}-q^{j}) = q^{\binom{h}{2}}\prod_{1\leq j\leq h}(q^{j}-1).$$ Given the $\mathbb F_q$-linear $\det: GL_h(\mathbb F_q)\longrightarrow \mathbb F_q^{*},$ consider the subsets $B^{*}_h(\mathbb F_q)$ of the groups $B_h(\mathbb F_q)\cap {\rm Ker}(\det),$ where each element of $B^{*}_h(\mathbb F_q)$ has 1's in the main diagonal. Then, $B^{*}_h(\mathbb F_q)$ is also a self-conjugate subgroup of $GL_h(\mathbb F_q).$ It is straightforward to see that the order of $B^{*}_h(\mathbb F_q)$ is $q^{\binom{h}{2}}.$ In particular, when $q = 2,$ we have ${\rm Ker}(\det) = GL_h$ and $B^{*}_h = B_h.$ Thus ${\rm ord}(B_h(\mathbb F_q)) = {\rm ord}(B^{*}_h(\mathbb F_q))\prod_{1\leq j\leq h}(q-1)$ and ${\rm ord}(GL_h(\mathbb F_q)) = {\rm ord}(B^{*}_h(\mathbb F_q))\prod_{1\leq j\leq h}(q^{j}-1).$ In \cite[Theorem 4]{Hai}, by considering a variant of a family of finite quotient rings of $\mathbb F_q[t_1, \ldots, t_h]$, Hai proved that the space of the indecomposable elements of $\mathbb F_q[t_1, \ldots, t_h]$ has dimension $(q-1)(q^{2}-1)\ldots (q^{h-1}-1)$ in degree $q^{h-1}-h.$ From these data, we get $$ \dim_{\mathbb F_q}(\mathbb F_q[t_1, \ldots t_h]/\overline{\pmb{A}}_q\mathbb F_q[t_1, \ldots t_h])_{q^{h-1}-h} = \prod_{1\leq j\leq h-1}(q^{j}-1) = {\rm ord}(GL_{h-1}(\mathbb F_q)/B^{*}_{h-1}(\mathbb F_q)).$$ (The reader should also keep in mind that the product $\prod_{1\leq j\leq h-1}(q^{j}-1)$ is also a well known formula for the degree of a cuspidal character of $GL_h(\mathbb F_q).$ The cuspidal characters are of great importance for characters of $GL_h(\mathbb F_q)$ since each character of this linear group is build up from cuspidal characters.) Now, with the field $\mathbb F_2$ and degree $n_{h, s} = 2^{s+4}-h,$ since $$n_{h, s} = (2^{s+3}-1) + (2^{s+2}-1) + (2^{s+1}-1) + \cdots + (2^{s-(h-5)}-1)+(2^{s-(h-5)}-1),$$ $\mu(n_{h, s}) = h$ for any $s \geq h-4,$ and so, by Theorem \ref{dlMK}, the iterated Kameko squaring operation $(\widetilde {Sq^0_*})^{s-h+5}_{n_{h, s}}: QP^{\otimes h}_{n_{h, s}} \longrightarrow QP^{\otimes h}_{n_{h, h-5}}$ is an isomorphism for every $s \geq h-5.$ Combining this with the facts that $\dim QP^{\otimes h}_{n_{h, h-5}} = \prod_{1\leq j\leq h-1}(2^{j}-1)$ and $B_{h-1}^{*} = B_{h-1},$ we must have $$ \dim QP^{\otimes h}_{n_{h, s}} = \prod_{1\leq j\leq h-1}(2^{j}-1) = {\rm ord}(GL_{h-1}/B^{*}_{h-1})= {\rm ord}(GL_{h-1}/B_{h-1}),\ \mbox{for all $s\geq h-5.$}$$ Moreover, as the Kameko homomorphism $(\widetilde {Sq^0_*})_{n_{h, s}}: QP^{\otimes h}_{n_{h, s}}\longrightarrow QP^{\otimes h}_{n_{h, s-1}}$ is an epimorphism and $QP^{\otimes h}_{n_{h, s}}\cong QP^{\otimes h}_{n_{h, h-5}},$ one gets $QP^{\otimes h}_{n_{h, s}}\cong {\rm Ker}((\widetilde {Sq^0_*})_{n_{h, h-5}})\bigoplus QP^{\otimes h}_{2^{h-2}-h}$ for arbitrary $s\geq h-5.$ \medskip Let us take notice that, in the case of $q=2$ and $h=6$, the dimensionality of $QP^{\otimes 6}_{n_1}$ is equal to $(2^{1}-1)\ldots (2^{6-1}-1) = 9765$, a result that can be gleaned from Theorem \ref{dlc1}. Therefore, our research stands independently of Hai's, and our approach is completely distinct. Furthermore our work offers a precise and unambiguous description of a monomial basis for the cohit module $QP^{\otimes 6}_{2^{6-1}-6= n_1}$, which serves as a representation of $GL_6$. Theoretically, our technique can be extended to any values of $h$ and $n.$ Nonetheless, the process of calculation becomes increasingly complex as the dimensions of $QP^{\otimes h}_n$ grow larger with increasing $h$ and $n$. \begin{rems} Consider general degree $n_h = 2^{h-2}-h,\, h\geq 4,$ we have $\dim QP^{\otimes 4}_{n_4} = \dim \mathbb F_2 = 1$, $\dim QP^{\otimes 5}_{n_5} = 7$ (see Theorem \ref{dlMM}) and $\dim QP^{\otimes 6}_{n_6} = 945$ (see Theorem \ref{dlMKR}). Given any $h\geq 7,$ by Corollary \ref{hqs22}, $\dim QP^{\otimes h}_{n_h} = 3.7\ldots (2^{h-1}-1)-\dim {\rm Ker}((\widetilde {Sq^0_*})_{n_{h, h-5}}).$ Hence, in order to determine the dimension of $QP^{\otimes h}_{n_h}$ for all $h>6,$ it suffices to calculate the dimension of the kernel of Kameko's map $(\widetilde {Sq^0_*})_{n_{h, h-5}}$. However, this aspect will be investigated in a separate study. Utilizing a result from Hai \cite[Corollary 3]{Hai}, we have $QP^{\otimes (h-2)}_{n_h}\cong {\rm St}_{h-2}\otimes_{\mathbb F_2}{\rm det}^{1},$ where ${\rm det}^{1}$ denotes the first power of the determinant representation of $GL_{h-2}$ and ${\rm St}_{h-2}$ is the Steinberg module (a.k.a the Steinberg representation). Remarkably, for $h = 8,\, 9,$ since the cohomology groups ${\rm Ext}_{\mathcal A}^{h-2, n_h + h-2}(\mathbb F_2, \mathbb F_2)$ are trivial \cite{Bruner}, the Singer conjecture is wrong if $\dim[QP^{\otimes (h-2)}_{n_h}]^{GL_{h-2}} > 0.$ For $h =4,$ one has an isomorphism $(\mathbb F_2 \otimes_{GL_2} {\rm Ann}_{\overline{\mathcal A}}[P^{\otimes 2}]^{*})_{0} \cong \mathbb F_2\cong {\rm Ext}_{\mathcal A}^{2, 2}(\mathbb F_2, \mathbb F_2),$ which implies that the Singer conjecture holds for bidegree $(2, 2).$ For $h = 5,\, 6,\, 7,$ Singer's conjecture for bidegree $(h-2, n_h+h-2)$ has been verified by Boardman \cite{Boardman} for $h = 5,$ by the present author \cite{Phuc12} for $h = 6$ and by Sum \cite{Sum3} for $h = 7.$ By these, it would also be of significant interest to determine explicit generators of $QP^{\otimes (h-2)}_{n_h}.$ The dimension of this cohit module was determined by Peterson \cite{Peterson} for $h = 4,$ by Kameko \cite{Kameko} for $h = 5,$ and by Sum \cite{Sum2, Sum3} for $h = 6,\, 7.$ (See also Theorems \ref{dlMKR} and \ref{dlMM} for the cases where $4\leq h\leq 6.$) \end{rems} Building upon Corollary \ref{hqs22} and the calculations in \cite{Tangora, Bruner, Bruner2, Lin2}, we can see that with degree $n_{h, s}$ as in Corollary \ref{hqs22}, $$ \begin{array}{ll} {\rm Ext}_{\mathcal A}^{7, 7+n_{7, s}}(\mathbb F_2, \mathbb F_2)=\left\{\begin{array}{ll} 0 &\mbox{if $s = 1,\, 4$},\\[1mm] \langle Q_2(0)\rangle &\mbox{if $s = 2$},\\[1mm] \langle \{Q_2(1), h_6D_2 \} \rangle &\mbox{if $s = 3$},\\[1mm] \end{array}\right.\\ \\ {\rm Ext}_{\mathcal A}^{8, 8+n_{8, s}}(\mathbb F_2, \mathbb F_2)=\left\{\begin{array}{ll} 0 &\mbox{if $s = 1,\, 2$},\\[1mm] \langle h_{6}Q_2(0)\rangle &\mbox{if $s = 3$},\\[1mm] \langle x_{n_{8, 4}, 8}\rangle &\mbox{if $s = 4$}, \end{array}\right. \end{array}$$ where $x_{n_{8, 4}, 8}$ is an indecomposable element. We believe that the following prediction would be of significant interest to investigate regarding Conjecture \ref{gtSinger} in high homological degrees. \newpage \begin{conj}\label{gtbsm} The family $\big\{Q_2(k):\, k\geq 0\big\}$ is a finite $Sq^{0}$-family. Furthermore, we have that: \begin{itemize} \item[(i)] the transfer $Tr_7^{\mathcal A}$ does not detect the non-zero elements $Q_2(0),\, Q_2(1)$ and $h_6D_2;$ \item[(ii)] the transfer $Tr_8^{\mathcal A}$ does not detect the non-zero elements $h_{6}Q_2(0)$ and $x_{n_{8, 4}, 8}.$ \end{itemize} \end{conj} Note that an $Sq^{0}$ -family is called \textit{finite} if it has only finitely many nonzero elements, \textit{infinite} if all of its elements are nonzero \cite{Hung}. Due to Corollary \ref{hqs22}, it is observed that the conjecture for items (i) and (ii) is valid under the following circumstances: $$ \begin{array}{ll} (\mathbb F_2 \otimes_{GL_7} {\rm Ann}_{\overline{\mathcal A}}[P^{\otimes 7}]^{*})_{n_{7, 1}}\cong (\mathbb F_2 \otimes_{GL_7} {\rm Ann}_{\overline{\mathcal A}}[P^{\otimes 7}]^{*})_{n_{7, 2}} \cong [{\rm Ker}((\widetilde {Sq^0_*})_{n_{7, 2}})]^{GL_7} = 0,\\ (\mathbb F_2 \otimes_{GL_8} {\rm Ann}_{\overline{\mathcal A}}[P^{\otimes 8}]^{*})_{n_{8, 2}}\cong (\mathbb F_2 \otimes_{GL_8} {\rm Ann}_{\overline{\mathcal A}}[P^{\otimes 8}]^{*})_{n_{8, 3}} \cong [{\rm Ker}((\widetilde {Sq^0_*})_{n_{8, 3}})]^{GL_8} = 0. \end{array}$$ Our approach for determining the domain of $Tr_7^{\mathcal A}$ with respect to degrees $n_{7, 1}$ and $n_{7, 2}$, as well as the domain of $Tr_8^{\mathcal A}$ with respect to degree $n_{8, 3}$, will involve the use of Theorems \ref{dlMM} and \ref{dlWW}, alongside Corollary \ref{hqs22}. Nevertheless, the calculation at hand seems to be rather daunting. Adopting an alternative perspective, H\uhorn ng \cite{Hung} proposed an interesting notion concerning a \textit{critical element} that exists within ${\rm Ext}_{\mathcal A}^{h, h+n}(\mathbb F_2, \mathbb F_2)$. Specifically, a non-zero element $\zeta$ in ${\rm Ext}_{\mathcal A}^{h, h+n}(\mathbb F_2, \mathbb F_2)$ is deemed \textit{critical} if it satisfies two conditions: firstly, $\mu(2n+h) = h$, and secondly, the image of $\zeta$ under the classical squaring operation $Sq^{0}$ is zero. It is well-established that $Sq^0$ is a monomorphism in positive stems of ${\rm Ext}_{\mathcal A}^{h, *}(\mathbb F_2, \mathbb F_2)$ for $h < 5,$ thereby implying the absence of any critical element for $h < 5.$ Remarkably, H\uhorn ng's work \cite[Theorem 5.9]{Hung} states that Singer's Conjecture \ref{gtSinger} is not valid, if the algebraic transfer detects the critical elements. Now, given the non-zero elements $Q_2(1)$ and $h_6D_2$, we are able to deduce that $\mu(2{\rm Stem}(Q_2(1))+7) = \mu(2{\rm Stem}(h_6D_2)+7) = 7.$ Furthermore, it is worth noting that $Sq^{0}(Q_2(1)) = 0 = Sq^{0}(h_6D_2)$, an observation which can be attributed to the fact that ${\rm Ext}_{\mathcal A}^{7, 7+n_{7,4}}(\mathbb F_2, \mathbb F_2) = 0$, as previously discussed. Thus $Q_2(1)$ and $h_6D_2$ must be critical elements. By this reason, in the event that Conjecture \ref{gtbsm}(i) is proven to be false, it would entail the refutation of Singer's conjecture in general. \medskip We now turn our attention to the hit problem for $P^{\otimes 6}$ in degree $n_s$ with $s > 1.$ \medskip {\bf Cases $\pmb{s > 1}.$}\ By Theorem \ref{dlc1} and Corollary \ref{hqs22}, $\dim QP^{\otimes 6}_{n_s} = \dim QP^{\otimes 6}_{n_1} = 9765$ for any $s > 0.$ Moreover, since the iterated homomorphism $((\widetilde {Sq^0_*})_{n_{s}})^{s-1}: QP^{\otimes 6}_{n_s} \longrightarrow QP^{\otimes 6}_{n_1}$ is an isomorphism, for every positive integer $s,$ we have immediately the below corollary. \begin{corls}\label{dlc2} For each integer $s\geq 2,$ the set $\big\{[t]:\ t\in \psi^{s-1}(\mathscr C^{\otimes 6}_{n_1})\big\}$ is a monomial basis of the $\mathbb F_2$-vector space $QP^{\otimes 6}_{n_s},$ on which $\psi: P^{\otimes 6}\longrightarrow P^{\otimes 6},\, t\longmapsto t_1t_2\ldots t_6t^{2}$ and $\psi^{s-1}(\mathscr C^{\otimes 6}_{n_1}) = \bigg\{\prod_{1\leq j\leq 6}t^{2^{s-1}-1}_ju^{2^{s-1}}:\ u\in \mathscr C^{\otimes 6}_{n_1}\bigg\}.$ \end{corls} The next contribution of this work is to apply the aforementioned results into the investigation of the cohit module $QP^{\otimes 7}$ in general degree $n_{s+5}$ and the behavior of the sixth algebraic transfer in internal degrees $n_s$ for any $s > 0.$ To achieve this goal, we will begin by recalling an interesting result on an inductive formula for the dimension of $QP^{\otimes h}_n.$ \begin{therm}[see Sum \cite{Sum2}]\label{dlS} Consider the degree $n$ of the form \eqref{pt} with $k = h-1,$ and $s, r$ positive integers such that $1\leq h-3\leq \mu(r)\leq h-2,$ and $\mu(r) = \alpha(r + \mu(r)).$ Then for each $s\geq h-1,$ we have $\dim QP^{\otimes h}_n = (2^{h}-1)\dim QP^{\otimes (h-1)}_r.$ \end{therm} \begin{rems}\label{nxP} With the general degrees $n_s:= (h-1)(2^{s}-1) + r\cdot 2^{s},$ assume there is a non-negative integer $\zeta$ such that $\zeta < s$ and $1\leq h-3\leq \mu(n_{\zeta}) = \alpha(n_{\zeta}+ \mu(n_{\zeta}))\leq h-2.$ Let us consider generic degrees of the form $k(2^{s-\zeta + h-1}-1) + r\cdot 2^{s-\zeta + h-1},$ where $k = h-1$, $r = n_{\zeta}$ and $s\geq \zeta\geq 0.$ Consequently, due to $\mu(r) = \alpha(r+\mu(r)),$ we have the following inductive formula, which is deduced from Theorem \ref{dlS} and the proof of this theorem on pages 445-446 of \cite{Sum2}: $$ \dim QP^{\otimes h}_{(h-1)(2^{s-\zeta + h-1}-1) + n_{\zeta}2^{s-\zeta + h-1}} = (2^{h}-1)\dim QP^{\otimes (h-1)}_{n_s},\ \mbox{ for every $s\geq \zeta.$}$$ Now, with $h = 7,$ $r = 10$, $\zeta = 1,$ and degree $n_s,$ we have $\mu(n_{1}) = 4 = \alpha(n_{1}+ \mu(n_{1})).$ Hence the following is immediate from Corollary \ref{hqs22} and Remark \ref{nxP}. \end{rems} \begin{corls}\label{dlc3} For every positive integer $s,$ the cohit module $QP^{\otimes 7}$ has dimension $1240155$ in degree $n_{s+5} = 6(2^{s+5}-1) + 10\cdot 2^{s+5}.$ \end{corls} As a consequence of Theorem \ref{dlc1} and the computations done in \cite{Tangora, Bruner, Bruner2}, we are able to establish the third main result of this paper. \begin{therm}\label{dlc4} For each integer $s > 0,$ the coinvariant $(\mathbb F_2 \otimes_{GL_6} {\rm Ann}_{\overline{\mathcal A}}[P^{\otimes 6}]^{*})_{n_s}$ is trivial. Consequently, the algebraic transfer $Tr_6^{\mathcal A}: (\mathbb F_2 \otimes_{GL_6} {\rm Ann}_{\overline{\mathcal A}}[P^{\otimes 6}]^{*})_{n_s}\longrightarrow {\rm Ext}_{\mathcal A}^{6, 6+n_s}(\mathbb F_2, \mathbb F_2)$ is a monomorphism, but it is not an epimorphism for $0< s < 3.$ This means that the transfer $Tr_6^{\mathcal A}$ does not detect the non-zero elements $h_4Ph_2 = h_2^{2}g_1\in {\rm Ext}_{\mathcal A}^{6, 6+n_1}(\mathbb F_2, \mathbb F_2)$ and $D_2\in {\rm Ext}_{\mathcal A}^{6, 6+n_2}(\mathbb F_2, \mathbb F_2).$ When $s = 3,$ the transfer $Tr_6^{\mathcal A}$ is a trivial isomorphism. \end{therm} By invoking Theorems \ref{dlc0} and \ref{dlc4}, it is possible to deduce that in the bidegrees $(6, 6+n_s),$ Singer's transfer is a monomorphism, but not an epimorphism for $0\leq s\leq 2.$ In the case where $s\geq 3,$ the transfer is a trivial isomorphism if its codomain is zero, and a monomorphism otherwise. These lead to an immediate consequence \begin{corls}\label{hqbs} Conjecture \ref{gtSinger} is valid in the bidegrees of $(6, 6+n_s)$ for any non-negative integer $s$. \end{corls} {\bf Final remarks.} Drawing upon the findings in \cite{Tangora, Bruner, Bruner2, Lin, Chen, Chen2, Lin2} on the structure of ${\rm Ext}_{\mathcal A}^{s, *}(\mathbb F_2, \mathbb F_2)$ for $s\leq 6,$ we end this section by presenting the following conjecture, which predicts the structure of the sixth cohomology group ${\rm Ext}_{\mathcal A}^{6, 6+n_s}(\mathbb F_2, \mathbb F_2)$ for all $s\geq 0$. \begin{conj}\label{gtP} With degree $n_s = 6(2^{s}-1) + 10\cdot 2^{s},$ we have \begin{equation*} {\rm Ext}_{\mathcal A}^{6, 6+n_s}(\mathbb F_2, \mathbb F_2)=\left\{\begin{array}{ll} \langle h_1Ph_1 \rangle &\mbox{if $s = 0$},\\[1mm] \langle h_2^{2}g_1\rangle &\mbox{if $s = 1$},\\[1mm] \langle D_2 \rangle &\mbox{if $s = 2$},\\[1mm] \langle \{h_s^{2}h_{s+1}^{2}h_{s+2}^{2},\, h_{s+1}^{2}g_s= h_{s+1}h_{s+3}g_{s-1},\, h_sh_{s+2}f_{s-1},\, h_sh_{s+1}^{2}c_s\} \rangle = 0 &\mbox{if $s \geq 3$},\\[1mm] \end{array}\right. \end{equation*} where $P$ denotes the Adams periodicity operator and \begin{equation*} \begin{array}{ll} \medskip &h_1Ph_1=[\lambda_1\lambda_2 \lambda_0^3 \lambda_7 + \lambda_1^2 \lambda_2 \lambda_4 \lambda_1^2 + \lambda_1^3 \lambda_2 \lambda_4 \lambda_1 + \lambda_1^2 \lambda_2 \lambda_1^2 \lambda_4]\neq 0,\\ & h_2^{2}g_1= Sq^{0}(h_1Ph_1) = h_4Ph_2= [\lambda_{15}\lambda_4\lambda_0^3 \lambda_7 + \lambda_{15}\lambda_3\lambda_5 \lambda_1^3 + \lambda_{15}\lambda_3\lambda_2\lambda_4 \lambda_1^2 + \lambda_{15}\lambda_3\lambda_1\lambda_2 \lambda_4 \lambda_1 \\ \medskip &\hspace{5cm} + \lambda_{15}\lambda_3\lambda_2\lambda_1^2\lambda_4 + \lambda_{15}\lambda_2\lambda_2 \lambda_0^2 \lambda_7 + \lambda_{15}\lambda_1\lambda_1 \lambda_2\lambda_0 \lambda_7]\neq 0,\\ &D_2 = [\lambda_0^{4}\lambda_{11}\lambda_{47}]\neq 0. \end{array} \end{equation*} \end{conj} Note that $h_{s+1}g_s = h_{s+3}g_{s-1}$ for any $s\geq 2.$ Given the calculations presented in \cite{Tangora, Bruner, Lin2}, it has been unequivocally established that the conjecture holds for $s\leq 3.$ Additionally, if the conjecture is confirmed to be accurate in general, then Singer's algebraic transfer would be a trivial isomorphism in the bidegrees $(6, 6+n_s)$ for $s\geq 3$. The readers will observe that Singer's Conjecture \ref{gtSinger} in these bidegrees would be disproven if the dimension of the invariant $[QP^{\otimes 6}_{n_1}]^{GL_6}$ is equal to 1. However, as demonstrated in Theorem \ref{dlc4}, this eventuality did not transpire. Inspired by the calculations in \cite{Bruner2}, we are confident that Conjecture \ref{gtP} also holds true for all $s > 3$. From another perspective, by virtue of the calculations set forth in the works \cite{Lin, Chen2, Bruner2}, and by making use of the fundamental property that $Sq^{0}$ is an algebraic homomorphism, it follows that $Sq^{0}(h_2^{2}g_1) = Sq^{0}(h_4Ph_2)= h_5h_3g_1 = 0.$ On the other hand, in \cite{Bruner}, Bruner claimed that ${\rm Ext}_{\mathcal A}^{6, 6+n_3}(\mathbb F_2, \mathbb F_2)$ is trivial. So, $Sq^{0}$ must send the indecomposable element $D_2$ to zero. Thus, since $\mu(2n_1+6) = 6= \mu(2n_2+6),$ both $h_2^{2}g_1$ and $D_2$ are critical elements. (Additionally, considering the fact that $2^{4} = {\rm Stem}(Ph_2) +5 < 4({\rm Stem}(Ph_2))^2$ and that $Ph_2\in {\rm Ext}_{\mathcal A}^{5, 16}(\mathbb F_2, \mathbb F_2)$ is a critical element, as noted in \cite{Hung}, it follows that $h_2^{2}g_1=h_4Ph_2 $ is a critical element. This explanation further strengthens the aforementioned assertion.) An interesting observation from H\uhorn ng's paper \cite[Lemma 5.3]{Hung} is that the condition $2^{m}\geq \max\big\{4d^{2}, d+h\big\}$ is insufficient to identify all critical elements of the form $h_mx$ in ${\rm Ext}_{\mathcal A}^{h, *}(\mathbb F_2, \mathbb F_2)$ when $x$ is also a critical element and $d = {\rm Stem}(x).$ This can be inferred from the foregoing facts that $D_2\in {\rm Ext}_{\mathcal A}^{6, 6+n_2}(\mathbb F_2, \mathbb F_2)$ and $h_6D_2\in {\rm Ext}_{\mathcal A}^{7, 7+n_{7, 3}}(\mathbb F_2, \mathbb F_2)$ are critical elements, but $2^{6} < \max\big\{4({\rm Stem}(D_2))^{2},\, {\rm Stem}(D_2)+6\big\}.$ In summary, even thourgh the elements $h_4Ph_2$ and $D_2$ are critical, they cannot be detected by the algebraic transfer. (It should be brought to the attention of the readers that the research conducted by Qu\`ynh \cite{Quynh} demonstrates that the indecomposable element $Ph_2$ does not belong to the image of the fifth transfer.) This reinforces the conclusion that Conjecture \ref{gtSinger} continues to hold for the bidegrees $(6, 6+n_1)$ and $(6, 6+n_2),$ as established in Theorem \ref{dlc4} and Corollary \ref{hqbs}. \section{Proofs of the main results}\label{s4} This section is devoted to proving Theorems \ref{dlc0}, \ref{dlc1} and \ref{dlc4}. To begin with, we need the following homomorphisms and a helpful remark below. One should note that $V_h\cong \langle \{t_1, \ldots, t_h\}\rangle \subset P^{\otimes h}.$ For $1\leq d\leq h,$ we define the $\mathbb F_2$-linear map $\sigma_d: V_h\longrightarrow V_h$ by setting $$ \left\{\begin{array}{ll} \sigma_d(t_d) = t_{d+1},\\[1mm] \sigma_d(t_{d+1}) = t_d,\\[1mm] \sigma_d(t_i) = t_i,\ \mbox{for $i\neq d, d +1,\; 1\leq d\leq h-1,$}\\[1mm] \sigma_h(t_1) = t_1 + t_2,\ \ \sigma_h(t_i) = t_i,\ \mbox{for $2\leq i \leq h.$} \end{array}\right.$$ \medskip Denote by $\Sigma_h\subset GL_h$ the symmetric group of degree $h.$ Then, $\Sigma_h$ is generated by the ones associated with $\sigma_1,\, \ldots, \sigma_{h-1}.$ For each permutation in $\Sigma_h$, consider corresponding permutation matrix; these form a group of matrices isomorphic to $\Sigma_h.$ Indeed, consider the following map $\Delta: \Sigma_h\longrightarrow \mathcal P_{h\times h},$ where the latter is the set of permutation matrices of order $h.$ This map is defined as follows: given $\sigma\in \Sigma_h,$ the $i$-th column of $\Delta(\sigma)$ is the column vector with a $1$ in the $\rho(i)$-th position, and $0$ elsewhere. It is easy to see that $\Delta(\rho)$ is indeed a permutation matrix, since a $1$ occurs in any position if and only if that position is described by $(\rho(i), i),$ for any $1\leq i\leq h.$ The map $\Delta$ is clearly multiplicative. (It is to be noted that because these are matrices, it is enough to show that each corresponding entry is equal. So let us take the entry $(i, j)$ of each matrix.) Then, $\Delta(\rho\circ\rho')_{ij} =1$ if and only if $i = \rho\circ \rho'(j).$ Note also that by ordinary matrix multiplication, one has $(\Delta(\rho)\Delta(\rho'))_{ij} = \sum_{1\leq k\leq h}\Delta(\rho)_{ik}\Delta(\rho')_{kj}.$ Now, we know that $\Delta(\rho)_{ik} = 1$ only when $i = \rho(k).$ Similarly, $\Delta(\rho')_{kj} = 1$ only when $k = \rho'(j).$ Hence, their product is one precisely when both of these happen: $i = \rho(k),$ and $k = \rho'(j).$ If both these do not happen simultaneously, then whenever one of $\Delta(\rho)_{ik},\, \Delta(\rho')_{kj}$ is one of the other will be zero, so the whole sum will be zero. However, this is the same as saying that the sum is one exactly when $i = \rho\circ\rho'(j).$ This description matches with the description for $\Delta(\rho\circ \rho')_{ij}$ given earlier. Hence, entry by entry these matrices are the same. Therefore the matrices are the same, and hence $\Delta$ is a homomorphism between the two spaces, an isomorphism as it has trivial kernel and the sets are of the same cardinality. Thus, $GL_h\cong GL(V_h),$ and $GL_h$ is generated by the matrices associated with $\sigma_1, \ldots, \sigma_h.$ \medskip Let $T = t_1^{a_1}t_2^{a_2}\ldots t_h^{a_h}$ be a monomial in $P_n^{\otimes h}$. Then, the weight vector $\omega(T)$ is invariant under the permutation of the generators $t_j,\ j = 1, 2, \ldots, h;$ hence $QP_n^{\otimes h}(\omega(T))$ also has a $\Sigma_h$-module structure. We see that the linear map $\sigma_d$ induces a homomorphism of $\mathcal A$-algebras which is also denoted by $\sigma_d: P^{\otimes h}\longrightarrow P^{\otimes h}.$ So, a class $[T]_{\omega(T)}\in QP_n^{\otimes h}(\omega)$ is an $GL_h$-invariant if and only if $\sigma_d(T) \equiv_{\omega(T)} T$ for $1\leq d\leq h.$ If $\sigma_d(T) \equiv_{\omega(T)} T$ for $1\leq d\leq h-1,$ then $[T]_{\omega(T)}$ is an $\Sigma_h$-invariant. (We must stress that the explicit calculation of the $GL_h$-invariants of $QP_n^{\otimes h}(\omega)$ in every positive degree $n$ is a non-trivial undertaking. Nonetheless, this computation becomes significantly more tractable when a monomial basis of $QP_n^{\otimes h}(\omega)$ is precisely determined.) \subsection{Proof of Theorem \ref{dlc0}}\label{s2.0} Undoubtedly if $h > n,$ then $QP^{\otimes h}_{n} \cong (QP^{\otimes h}_{n})^{0}.$ So, the coinvariant $(\mathbb F_2\otimes_{GL_h}{\rm Ann}_{\overline{\mathcal A}}[P^{\otimes h}]^{*})_{n}$ vanishes for any $1\leq n\leq n_0$ and $h\geq n+1.$ Let us consider the following weight vectors: $$ {\omega}^{*}_{(1)}:=(3,1,1), \ \ {\omega}^{*}_{(2)}:=(3,3), \ \ {\omega}^{*}_{(3)}:=(5,2), \ \ {\omega}^{*}_{(4)}:=(7,1),\ \ {\omega}^{*}_{(5)}:=(9,0).$$ Consequently $\deg({\omega}^{*}_{(1)}) = \deg({\omega}^{*}_{(2)}) = \deg({\omega}^{*}_{(3)}) = \deg({\omega}^{*}_{(4)}) = \deg({\omega}^{*}_{(5)}) = 9.$ It can be seen that $(QP^{\otimes 9}_{9})^{>0}\cong (\widetilde {Sq^0_*})_{9}(QP^{\otimes 9}_{9}) \cong \mathbb F_2$, and so, $(QP^{\otimes 9}_{9})^{>0}= \mathbb F_2\big[\prod_{1\leq i\leq 9}t_i\big]_{\omega^{*}_{(5)}}.$ In combination with the earlier studies by Peterson \cite{Peterson}, Kameko \cite{Kameko}, Sum \cite{Sum2}, Sum and T'in \cite{Sum-Tin}, and Mothebe et al. \cite{MKR}, the following isomorphisms are obtained: $$ (QP^{\otimes h}_{9})^{>0}\cong \left\{\begin{array}{ll} (QP^{\otimes h}_{9})^{>0}({\omega}^{*}_{(1)})\bigoplus (QP^{\otimes h}_{9})^{>0}({\omega}^{*}_{(2)})&\mbox{if $3\leq h\leq 4$},\\[2mm] (QP^{\otimes h}_{9})^{>0}({\omega}^{*}_{(1)})\bigoplus (QP^{\otimes h}_{9})^{>0}({\omega}^{*}_{(2)})\bigoplus (QP^{\otimes h}_{9})^{>0}({\omega}^{*}_{(3)})&\mbox{if $h = 5$},\\[2mm] (QP^{\otimes h}_{9})^{>0}({\omega}^{*}_{(2)})\bigoplus (QP^{\otimes h}_{9})^{>0}({\omega}^{*}_{(3)})&\mbox{if $h = 6$},\\[2mm] (QP^{\otimes h}_{9})^{>0}({\omega}^{*}_{(3)})\bigoplus (QP^{\otimes h}_{9})^{>0}({\omega}^{*}_{(4)})&\mbox{if $h = 7$},\\[2mm] (QP^{\otimes h}_{9})^{>0}({\omega}^{*}_{(4)})&\mbox{if $h = 8$},\\[2mm] (QP^{\otimes h}_{9})^{>0}({\omega}^{*}_{(5)})&\mbox{if $h = 9$},\\[2mm] O&\mbox{if $h \geq n_0.$} \end{array}\right.$$ Hence the dimensions of the indecomposables $(QP^{\otimes h}_{9})^{>0}({\omega}^{*}_{(j)})$ are determined as follows: \centerline{\begin{tabular}{c||cccccccccccccccc} $j$ &&$1$ && $2$ && $3$ && $4$ && $5$ \cr \hline \hline \ $\dim (QP^{\otimes 3}_{9})^{>0}(\omega^{*}_{(j)})$ && $6$ && $1$ && $0$ &&$0$ &&$0$ \cr \hline \hline \ $\dim (QP^{\otimes 4}_{9})^{>0}({\omega}^{*}_{(j)})$ && $12$ && $6$ && $0$ &&$0$ &&$0$ \cr \hline \hline \ $\dim (QP^{\otimes 5}_{9})^{>0}({\omega}^{*}_{(j)})$ && $6$ && $15$ && $10$ &&$0$ &&$0$ \cr \hline \hline \ $\dim (QP^{\otimes 6}_{9})^{>0}({\omega}^{*}_{(j)})$ && $0$ && $10$ && $24$ &&$0$ &&$0$ \cr \hline \hline \ $\dim (QP^{\otimes 7}_{9})^{>0}({\omega}^{*}_{(j)})$ && $0$ && $0$ && $14$ &&$7$ &&$0$ \cr \hline \hline \ $\dim (QP^{\otimes 8}_{9})^{>0}({\omega}^{*}_{(j)})$ && $0$ && $0$ && $0$ &&$7$ &&$0$ \cr \hline \hline \ $\dim (QP^{\otimes 9}_{9})^{>0}({\omega}^{*}_{(j)})$ && $0$ && $0$ && $0$ &&$0$ &&$1$ \cr \end{tabular}} Through a straightforward calculation utilizing the aforementioned data and Remark \ref{nxp10}, we obtain $$ \dim (QP^{\otimes h}_{9})^{0}(\omega^{*}_{(j)})= \left\{\begin{array}{ll} 6\binom{h}{3} &\mbox{if $j = 1$ and $h=4$},\\[2mm] 6\binom{h}{3} + 12\binom{h}{4}&\mbox{if $j = 1$ and $h=5$},\\[2mm] 6\binom{h}{3} + 12\binom{h}{4} + 6\binom{h}{5}&\mbox{if $j = 1$ and $h\geq 6$},\\[2mm] \binom{h}{3} &\mbox{if $j = 2$ and $h=4$},\\[2mm] \binom{h}{3} + 6\binom{h}{4}&\mbox{if $j = 2$ and $h=5$},\\[2mm] \binom{h}{3} + 6\binom{h}{4} + 15\binom{h}{5}&\mbox{if $j = 2$ and $h=6$},\\[2mm] \binom{h}{3} + 6\binom{h}{4} + 15\binom{h}{5} + 10\binom{h}{6}&\mbox{if $j = 2$ and $h\geq 7$},\\[2mm] 10\binom{h}{5} &\mbox{if $j = 3$ and $h=6$},\\[2mm] 10\binom{h}{5} + 24\binom{h}{6}&\mbox{if $j = 3$ and $h=7$},\\[2mm] 10\binom{h}{5} + 24\binom{h}{6} + 14\binom{h}{7} &\mbox{if $j = 3$ and $h\geq 8$},\\[2mm] 7\binom{h}{7} &\mbox{if $j = 4$ and $h=8$},\\[2mm] 7\bigg(\binom{h}{7} + \binom{h}{8}\bigg) &\mbox{if $j = 4$ and $h\geq 9.$} \end{array}\right.$$ Then, for each $h\geq n_0$ we have an isomorphism $QP^{\otimes h}_{9} \cong \bigoplus_{1\leq j\leq 5}(QP^{\otimes h}_{9})^{0}(\omega^{*}_{(j)}).$ \medskip $\bullet$ For $h = 9$ and $n = 9,$ since $(\widetilde {Sq^0_*})_{9}: QP^{\otimes 9}_{9} \longrightarrow \mathbb F_2$ is an epimorphism, $$QP^{\otimes 9}_{9} \cong \mathbb F_2\bigoplus {\rm Ker}((\widetilde {Sq^0_*})_{9}) \cong \mathbb F_2\big[\prod_{1\leq i\leq 9}t_i\big]_{\omega^{*}_{(5)}}\bigoplus \big(\bigoplus_{1\leq j\leq 4}(QP^{\otimes 9}_{9})^{0}(\omega^{*}_{(j)})\big).$$ This shows that $(QP^{\otimes 9}_{9})^{0}\cong \bigoplus_{1\leq j\leq 4}(QP^{\otimes 9}_{9})^{0}(\omega^{*}_{(j)}).$ Hence, $\dim (QP^{\otimes 9}_{9})^{0} = \sum_{1\leq j\leq 4}\dim (QP^{\otimes 9}_{9})^{0}(\omega^{*}_{(j)}) = 10437$ and $\dim QP^{\otimes 9}_{9} = 10438.$ Using this result and the homomorphisms $\sigma_d: P^{\otimes 9}\longrightarrow P^{\otimes 9},\, 1\leq d\leq 9,$ we claim that $[QP^{\otimes 9}_{9}]^{GL_9}$ is zero, and so is, $(\mathbb F_2\otimes_{GL_9}{\rm Ann}_{\overline{\mathcal A}}[P^{\otimes 9}]^{*})_{9}.$ \medskip $\bullet$ For $9\leq h\leq n_0$ and $n = n_0,$ by a simple computation using Theorem \ref{dlMKR} and Corollaries \ref{hq10-1-0}, \ref{hq10-1-1}, one has the following isomorphisms: $$ \begin{array}{ll} \medskip QP^{\otimes h}_{n_0}&\cong \bigoplus_{1\leq j\leq 6}(QP^{\otimes h}_{n_0})^{0}(\overline{\omega}^{j})\bigoplus (QP^{\otimes h}_{n_0})^{>0}(\overline{\omega}^{6}),\ \mbox{for $h = 9$},\\ QP^{\otimes h}_{n_0}&\cong \bigoplus_{1\leq j\leq 6}(QP^{\otimes h}_{n_0})^{0}(\overline{\omega}^{j})\bigoplus (QP^{\otimes h}_{n_0})^{>0}(\overline{\omega}^{7}),\ \mbox{for $h = n_0,$} \end{array}$$ where $\overline{\omega}^{6} := (8,1)$ and $\overline{\omega}^{7} := (n_0,0).$ It is to be noted that $\bigoplus_{1\leq j\leq 6}(QP^{\otimes n_0}_{n_0})^{0}(\overline{\omega}^{j}) \cong {\rm Ker}((\widetilde {Sq^0_*})_{n_0}),$ where ${\rm Ker}((\widetilde {Sq^0_*})_{n_0})$ is the kernel of the Kameko homomorphism $(\widetilde {Sq^0_*})_{n_0}: QP^{\otimes n_0}_{n_0}\longrightarrow \mathbb F_2.$ The dimensions of the cohit spaces $(QP^{\otimes h}_{n_0})^{0}(\overline{\omega}^{j}),\, 1\leq j\leq 5$ are explicitly determined as in Corollary \ref{hq10-1-1}. Based on Theorem \ref{dlMKR} and direct calculations, we find that $$ \dim (QP^{\otimes h}_{n_0})^{0}(\overline{\omega}^{6}) = \left\{\begin{array}{ll} 72&\mbox{if $h = 9$},\\[1mm] 8\bigg(\binom{n_0}{8} + \binom{n_0}{9}\bigg) = 125 &\mbox{if $h = n_0$}, \end{array}\right.$$ $$ \dim (QP^{\otimes h}_{n_0})^{>0}(\overline{\omega}^{j}) = \left\{\begin{array}{ll} 8&\mbox{if $h = 9$ and $j = 6$},\\[1mm] \dim \mathbb F_2 = 1 &\mbox{if $h = n_0$ and $j = 7$}. \end{array}\right.$$ Using these data and the homomorphisms $\sigma_d,\, 1\leq d\leq n_0,$ we state that $[QP^{n_0}_{n_0}]^{GL_{n_0}} = 0$ and that for each $1\leq j\leq 6,$ the invariant $[(QP^{\otimes h}_{n_0})^{>0}(\overline{\omega}^{j})]^{GL_9}$ is zero, and so is, $(\mathbb F_2\otimes_{GL_9}{\rm Ann}_{\overline{\mathcal A}}[P^{\otimes 9}]^{*})_{n_0}.$ We will describe explicitly $[(QP^{\otimes 9}_{n_0})^{>0}(\overline{\omega}^{j})]^{GL_9}$ for $j = 6.$ The others can be obtained by similar calculations. As shown above, $(QP^{\otimes 9}_{n_0})^{>0}(\overline{\omega}^{6})$ is an $\mathbb F_2$-vector space of dimension $8$ with a monomial basis represented by the following admissible monomials: \begin{center} \begin{tabular}{llr} ${\rm a}_{73}=t_1t_2t_3t_4t_5t_6t_7t_8t_9^{2}$, & ${\rm a}_{74}=t_1t_2t_3t_4t_5t_6t_7t_8^{2}t_9$, & \multicolumn{1}{l}{${\rm a}_{75}=t_1t_2t_3t_4t_5t_6t_7^{2}t_8t_9$,} \\ ${\rm a}_{76}=t_1t_2t_3t_4t_5t_6^{2}t_7t_8t_9$, & ${\rm a}_{77}=t_1t_2t_3t_4t_5^{2}t_6t_7t_8t_9$, & \multicolumn{1}{l}{${\rm a}_{78}=t_1t_2t_3t_4^{2}t_5t_6t_7t_8t_9$,} \\ ${\rm a}_{79}=t_1t_2t_3^{2}t_4t_5t_6t_7t_8t_9$, & ${\rm a}_{80}=t_1t_2^{2}t_3t_4t_5t_6t_7t_8t_9.$ & \end{tabular}\end{center} Suppose $[f]_{\overline{\omega}^{6}}\in [(QP^{\otimes 9}_{n_0})^{>0}(\overline{\omega}^{6})]^{\Sigma_9}.$ Then, we have $f\equiv_{\overline{\omega}^{6}} \sum_{73\leq i\leq 80} \gamma_{i}{\rm a}_{i}$ where $\gamma_i\in \mathbb F_2$ for every $i.$ Let us consider the homomorphisms $\sigma_d: P^{\otimes 9}\longrightarrow P^{\otimes 9},\, 1\leq d\leq 8.$ An easy calculation shows: $$ \begin{array}{ll} \sigma_1(f) &\equiv_{\overline{\omega}^{6}}\sum_{73\leq j\leq 79} \gamma_{j}{\rm a}_{j} + \gamma_{80}t_1^{2}t_2t_3t_4t_5t_6^{2}t_7t_8t_9\\ &\equiv_{\overline{\omega}^{6}} \sum_{73\leq j\leq 79}(\gamma_{j} + \gamma_{80}){\rm a}_{j} + \gamma_{80}{\rm a}_{80},\\ &(\mbox{since $t_1^{2}t_2t_3t_4t_5t_6^{2}t_7t_8t_9 = Sq^{1}(t_1t_2t_3t_4t_5t_6^{2}t_7t_8t_9) + \sum_{73\leq j\leq 80}{\rm a}_{j}$}),\\ \medskip \sigma_2(f) &\equiv_{\overline{\omega}^{6}}\sum_{73\leq j\leq 78} \gamma_{j}{\rm a}_{j} + \gamma_{79}{\rm a}_{80} + \gamma_{80}{\rm a}_{79},\ \ \sigma_3(f) \equiv_{\overline{\omega}^{6}}\sum_{j\neq 78,\, 79} \gamma_{j}{\rm a}_{j} + \gamma_{78}{\rm a}_{79} + \gamma_{79}{\rm a}_{78} ,\\ \medskip \sigma_4(f) &\equiv_{\overline{\omega}^{6}}\sum_{j\neq 77,\, 78} \gamma_{j}{\rm a}_{j} + \gamma_{77}{\rm a}_{78} + \gamma_{78}{\rm a}_{77},\ \ \sigma_5(f) \equiv_{\overline{\omega}^{6}}\sum_{j\neq 76,\, 77} \gamma_{j}{\rm a}_{j} + \gamma_{76}{\rm a}_{77} + \gamma_{77}{\rm a}_{76},\\ \medskip \sigma_6(f) &\equiv_{\overline{\omega}^{6}}\sum_{j\neq 75,\, 76} \gamma_{j}{\rm a}_{j} + \gamma_{75}{\rm a}_{76} + \gamma_{76}{\rm a}_{75},\ \ \sigma_7(f) \equiv_{\overline{\omega}^{6}}\sum_{j\neq 74,\, 75} \gamma_{j}{\rm a}_{j} + \gamma_{74}{\rm a}_{75} + \gamma_{75}{\rm a}_{74},\\ \sigma_8(f) &\equiv_{\overline{\omega}^{6}}\sum_{j\neq 73,\, 74} \gamma_{j}{\rm a}_{j} + \gamma_{73}{\rm a}_{74} + \gamma_{74}{\rm a}_{73}. \end{array}$$ By these equalities and the relations $\sigma_d(f) + f\equiv_{\overline{\omega}^{6}} 0,\, 1\leq d\leq 8$ we get $\gamma_{i} = 0,\, 73\leq i\leq 80.$ Thus, $[QP^{\otimes 9}_{n_0}(\overline{\omega}^{6})]^{GL_9} = [(QP^{\otimes 9}_{n_0})^{0}(\overline{\omega}^{6})]^{GL_9}.$ Note that $(QP^{\otimes 9}_{n_0})^{0}(\overline{\omega}^{6})$ is an $\mathbb F_2$-vector space of dimension $72$ with a monomial basis represented by the following admissible monomials: \begin{center} \begin{tabular}{llll} ${\rm a}_{1}=t_2t_3t_4t_5t_6t_7t_8t_9^{3}$, & ${\rm a}_{2}=t_2t_3t_4t_5t_6t_7t_8^{3}t_9$, & ${\rm a}_{3}=t_2t_3t_4t_5t_6t_7^{3}t_8t_9$, & ${\rm a}_{4}=t_2t_3t_4t_5t_6^{3}t_7t_8t_9$, \\ ${\rm a}_{5}=t_2t_3t_4t_5^{3}t_6t_7t_8t_9$, & ${\rm a}_{6}=t_2t_3t_4^{3}t_5t_6t_7t_8t_9$, & ${\rm a}_{7}=t_2t_3^{3}t_4t_5t_6t_7t_8t_9$, & ${\rm a}_{8}=t_2^{3}t_3t_4t_5t_6t_7t_8t_9$, \\ ${\rm a}_{9}=t_1t_3t_4t_5t_6t_7t_8t_9^{3}$, & ${\rm a}_{10}=t_1t_3t_4t_5t_6t_7t_8^{3}t_9$, & ${\rm a}_{11}=t_1t_3t_4t_5t_6t_7^{3}t_8t_9$, & ${\rm a}_{12}=t_1t_3t_4t_5t_6^{3}t_7t_8t_9$, \\ ${\rm a}_{13}=t_1t_3t_4t_5^{3}t_6t_7t_8t_9$, & ${\rm a}_{14}=t_1t_3t_4^{3}t_5t_6t_7t_8t_9$, & ${\rm a}_{15}=t_1t_3^{3}t_4t_5t_6t_7t_8t_9$, & ${\rm a}_{16}=t_1^{3}t_3t_4t_5t_6t_7t_8t_9$, \\ ${\rm a}_{17}=t_1t_2t_4t_5t_6t_7t_8t_9^{3}$, & ${\rm a}_{18}=t_1t_2t_4t_5t_6t_7t_8^{3}t_9$, & ${\rm a}_{19}=t_1t_2t_4t_5t_6t_7^{3}t_8t_9$, & ${\rm a}_{20}=t_1t_2t_4t_5t_6^{3}t_7t_8t_9$, \\ ${\rm a}_{21}=t_1t_2t_4t_5^{3}t_6t_7t_8t_9$, & ${\rm a}_{22}=t_1t_2t_4^{3}t_5t_6t_7t_8t_9$, & ${\rm a}_{23}=t_1t_2^{3}t_4t_5t_6t_7t_8t_9$, & ${\rm a}_{24}=t_1^{3}t_2t_4t_5t_6t_7t_8t_9$, \\ ${\rm a}_{25}=t_1t_2t_3t_5t_6t_7t_8t_9^{3}$, & ${\rm a}_{26}=t_1t_2t_3t_5t_6t_7t_8^{3}t_9$, & ${\rm a}_{27}=t_1t_2t_3t_5t_6t_7^{3}t_8t_9$, & ${\rm a}_{28}=t_1t_2t_3t_5t_6^{3}t_7t_8t_9$, \\ ${\rm a}_{29}=t_1t_2t_3t_5^{3}t_6t_7t_8t_9$, & ${\rm a}_{30}=t_1t_2t_3^{3}t_5t_6t_7t_8t_9$, & ${\rm a}_{31}=t_1t_2^{3}t_3t_5t_6t_7t_8t_9$, & ${\rm a}_{32}=t_1^{3}t_2t_3t_5t_6t_7t_8t_9$, \\ ${\rm a}_{33}=t_1t_2t_3t_4t_6t_7t_8t_9^{3}$, & ${\rm a}_{34}=t_1t_2t_3t_4t_6t_7t_8^{3}t_9$, & ${\rm a}_{35}=t_1t_2t_3t_4t_6t_7^{3}t_8t_9$, & ${\rm a}_{36}=t_1t_2t_3t_4t_6^{3}t_7t_8t_9$, \\ ${\rm a}_{37}=t_1t_2t_3t_4^{3}t_6t_7t_8t_9$, & ${\rm a}_{38}=t_1t_2t_3^{3}t_4t_6t_7t_8t_9$, & ${\rm a}_{39}=t_1t_2^{3}t_3t_4t_6t_7t_8t_9$, & ${\rm a}_{40}=t_1^{3}t_2t_3t_4t_6t_7t_8t_9$, \\ ${\rm a}_{41}=t_1t_2t_3t_4t_5t_7t_8t_9^{3}$, & ${\rm a}_{42}=t_1t_2t_3t_4t_5t_7t_8^{3}t_9$, & ${\rm a}_{43}=t_1t_2t_3t_4t_5t_7^{3}t_8t_9$, & ${\rm a}_{44}=t_1t_2t_3t_4t_5^{3}t_7t_8t_9$, \\ ${\rm a}_{45}=t_1t_2t_3t_4^{3}t_5t_7t_8t_9$, & ${\rm a}_{46}=t_1t_2t_3^{3}t_4t_5t_7t_8t_9$, & ${\rm a}_{47}=t_1t_2^{3}t_3t_4t_5t_7t_8t_9$, & ${\rm a}_{48}=t_1^{3}t_2t_3t_4t_5t_7t_8t_9$, \\ ${\rm a}_{49}=t_1t_2t_3t_4t_5t_6t_7t_9^{3}$, & ${\rm a}_{50}=t_1t_2t_3t_4t_5t_6t_7^{3}t_9$, & ${\rm a}_{51}=t_1t_2t_3t_4t_5t_6^{3}t_7t_9$, & ${\rm a}_{52}=t_1t_2t_3t_4t_5^{3}t_6t_7t_9$, \\ ${\rm a}_{53}=t_1t_2t_3t_4^{3}t_5t_6t_7t_9$, & ${\rm a}_{54}=t_1t_2t_3^{3}t_4t_5t_6t_7t_9$, & ${\rm a}_{55}=t_1t_2^{3}t_3t_4t_5t_6t_7t_9$, & ${\rm a}_{56}=t_1^{3}t_2t_3t_4t_5t_6t_7t_9$, \\ ${\rm a}_{57}=t_1t_2t_3t_4t_5t_6t_8t_9^{3}$, & ${\rm a}_{58}=t_1t_2t_3t_4t_5t_6t_8^{3}t_9$, & ${\rm a}_{59}=t_1t_2t_3t_4t_5t_6^{3}t_8t_9$, & ${\rm a}_{60}=t_1t_2t_3t_4t_5^{3}t_6t_8t_9$, \\ ${\rm a}_{61}=t_1t_2t_3t_4^{3}t_5t_6t_8t_9$, & ${\rm a}_{62}=t_1t_2t_3^{3}t_4t_5t_6t_8t_9$, & ${\rm a}_{63}=t_1t_2^{3}t_3t_4t_5t_6t_8t_9$, & ${\rm a}_{64}=t_1^{3}t_2t_3t_4t_5t_6t_8t_9$, \\ ${\rm a}_{65}=t_1t_2t_3t_4t_5t_6t_7t_8^{3}$, & ${\rm a}_{66}=t_1t_2t_3t_4t_5t_6t_7^{3}t_8$, & ${\rm a}_{67}=t_1t_2t_3t_4t_5t_6^{3}t_7t_8$, & ${\rm a}_{68}=t_1t_2t_3t_4t_5^{3}t_6t_7t_8$, \\ ${\rm a}_{69}=t_1t_2t_3t_4^{3}t_5t_6t_7t_8$, & ${\rm a}_{70}=t_1t_2t_3^{3}t_4t_5t_6t_7t_8$, & ${\rm a}_{71}=t_1t_2^{3}t_3t_4t_5t_6t_7t_8$, & ${\rm a}_{72}=t_1^{3}t_2t_3t_4t_5t_6t_7t_8.$ \end{tabular}\end{center} \medskip Suppose $[g]_{\overline{\omega}^{6}}\in [QP^{\otimes 9}_{n_0}(\overline{\omega}^{6}]^{\Sigma_9}.$ Then, one has $g\equiv_{\overline{\omega}^{6}} \sum_{1\leq j\leq 72} \beta_{i}{\rm a}_{i},$ in which $\beta_i\in \mathbb F_2,\, 1\leq i\leq 72.$ Using the homomorphisms $\sigma_d: P^{\otimes 9}\longrightarrow P^{\otimes 9},$ for $1\leq d\leq 8,$ we obtain the following equalities: $$ \begin{array}{ll} \sigma_1(g)&\equiv_{\overline{\omega}^{6}} \sum_{1\leq i\leq 8}\beta_{i}{\rm a}_{i+8} + \sum_{9\leq i\leq 16}\beta_{i}{\rm a}_{i-8} + \sum_{17\leq i\leq 22}\beta_{i}{\rm a}_{i} + \beta_{23}{\rm a}_{24} + \beta_{24}{\rm a}_{23}\\ &\quad + \sum_{25\leq i\leq 30}\beta_{i}{\rm a}_{i} + \beta_{31}{\rm a}_{32} + \beta_{32}{\rm a}_{31} + \sum_{33\leq i\leq 38}\beta_{i}{\rm a}_{i} + \beta_{39}{\rm a}_{40} + \beta_{40}{\rm a}_{39} \\ &\quad+ \sum_{41\leq i\leq 46}\beta_{i}{\rm a}_{i} + \beta_{47}{\rm a}_{48} + \beta_{48}{\rm a}_{47} + \sum_{49\leq i\leq 54}\beta_{i}{\rm a}_{i} + \beta_{55}{\rm a}_{56} + \beta_{56}{\rm a}_{55}\\ \medskip &\quad + \sum_{57\leq i\leq 62}\beta_{i}{\rm a}_{i} + \beta_{63}{\rm a}_{64} + \beta_{64}{\rm a}_{63}+\sum_{65\leq i\leq 70}\beta_{i}{\rm a}_{i} + \beta_{71}{\rm a}_{72} + \beta_{72}{\rm a}_{71},\\ \sigma_2(g)&\equiv_{\overline{\omega}^{6}} \sum_{1\leq i\leq 6}\beta_{i}{\rm a}_{i} +\beta_{7}{\rm a}_{8} + \beta_{8}{\rm a}_{7} + \sum_{9\leq i\leq 16}\beta_{i}{\rm a}_{i+8} + \sum_{17\leq i\leq 24}\beta_{i}{\rm a}_{i-8}\\ &\quad + \sum_{25\leq i\leq 29}\beta_{i}{\rm a}_{i} + \beta_{30}{\rm a}_{31} + \beta_{31}{\rm a}_{30} + \sum_{32\leq i\leq 37}\beta_{i}{\rm a}_{i} + \beta_{38}{\rm a}_{39} + \beta_{39}{\rm a}_{38}\\ &\quad + \sum_{40\leq i\leq 45}\beta_{i}{\rm a}_{i} + \beta_{46}{\rm a}_{47} + \beta_{47}{\rm a}_{46} + \sum_{48\leq i\leq 53}\beta_{i}{\rm a}_{i} + \beta_{54}{\rm a}_{55} + \beta_{55}{\rm a}_{54}\\ \medskip &\quad+ \sum_{56\leq i\leq 61}\beta_{i}{\rm a}_{i} + \beta_{62}{\rm a}_{63} + \beta_{63}{\rm a}_{62}+\sum_{64\leq i\leq 69}\beta_{i}{\rm a}_{i} + \beta_{70}{\rm a}_{71} + \beta_{71}{\rm a}_{70} + \beta_{72}{\rm a}_{72},\\ \sigma_3(g)&\equiv_{\overline{\omega}^{6}} \sum_{1\leq i\leq 5}\beta_{i}{\rm a}_{i} +\beta_{6}{\rm a}_{7} + \beta_{7}{\rm a}_{6} + \sum_{8\leq i\leq 13}\beta_{i}{\rm a}_{i} +\beta_{14}{\rm a}_{15} + \beta_{15}{\rm a}_{14} + \beta_{16}{\rm a}_{16} \\ &\quad+ \sum_{17\leq i\leq 24}\beta_{i}{\rm a}_{i+8} + \sum_{25\leq i\leq 32}\beta_{i}{\rm a}_{i-8} + \sum_{33\leq i\leq 36}\beta_{i}{\rm a}_{i} + \beta_{37}{\rm a}_{38} + \beta_{38}{\rm a}_{37}\\ &\quad + \sum_{39\leq i\leq 44}\beta_{i}{\rm a}_{i} + \beta_{45}{\rm a}_{46} + \beta_{46}{\rm a}_{45} + \sum_{47\leq i\leq 52}\beta_{i}{\rm a}_{i} + \beta_{53}{\rm a}_{54} + \beta_{54}{\rm a}_{53}\\ \medskip &\quad+ \sum_{55\leq i\leq 60}\beta_{i}{\rm a}_{i} + \beta_{61}{\rm a}_{62} + \beta_{62}{\rm a}_{61}+\sum_{63\leq i\leq 68}\beta_{i}{\rm a}_{i} + \beta_{69}{\rm a}_{70} + \beta_{70}{\rm a}_{69} + \sum_{71\leq i\leq 72}\beta_{i}{\rm a}_{i},\\ \sigma_4(g)&\equiv_{\overline{\omega}^{6}} \sum_{1\leq i\leq 4}\beta_{i}{\rm a}_{i} +\beta_{5}{\rm a}_{6} + \beta_{6}{\rm a}_{5} + \sum_{7\leq i\leq 12}\beta_{i}{\rm a}_{i} +\beta_{13}{\rm a}_{14} + \beta_{14}{\rm a}_{13}\\ &\quad + \sum_{15\leq i\leq 20}\beta_{i}{\rm a}_{i} +\beta_{21}{\rm a}_{22} + \beta_{22}{\rm a}_{21} + \sum_{23\leq i\leq 24}\beta_{i}{\rm a}_{i} + \sum_{25\leq i\leq 32}\beta_{i}{\rm a}_{i+8} \\ &\quad+ \sum_{33\leq i\leq 40}\beta_{i}{\rm a}_{i-8} + \sum_{41\leq i\leq 43}\beta_{i}{\rm a}_{i} + \beta_{44}{\rm a}_{45} + \beta_{45}{\rm a}_{44}+ \sum_{46\leq i\leq 51}\beta_{i}{\rm a}_{i} + \beta_{52}{\rm a}_{53} + \beta_{53}{\rm a}_{52}\\ \medskip &\quad+\sum_{54\leq i\leq 59}\beta_{i}{\rm a}_{i} + \beta_{60}{\rm a}_{61} + \beta_{61}{\rm a}_{60} + \sum_{62\leq i\leq 67}\beta_{i}{\rm a}_{i}+ \beta_{68}{\rm a}_{69} + \beta_{69}{\rm a}_{68} + \sum_{70\leq i\leq 72}\beta_{i}{\rm a}_{i},\\ \end{array}$$ \newpage $$ \begin{array}{ll} \sigma_5(g)&\equiv_{\overline{\omega}^{6}} \sum_{1\leq i\leq 3}\beta_{i}{\rm a}_{i} +\beta_{4}{\rm a}_{5} + \beta_{5}{\rm a}_{4} + \sum_{6\leq i\leq 11}\beta_{i}{\rm a}_{i} +\beta_{12}{\rm a}_{13} + \beta_{13}{\rm a}_{12}\\ &\quad + \sum_{14\leq i\leq 19}\beta_{i}{\rm a}_{i} +\beta_{20}{\rm a}_{21} + \beta_{21}{\rm a}_{20} + \sum_{22\leq i\leq 27}\beta_{i}{\rm a}_{i} +\beta_{28}{\rm a}_{29} + \beta_{29}{\rm a}_{28} + \sum_{30\leq i\leq 32}\beta_{i}{\rm a}_{i} \\ &\quad+ \sum_{33\leq i\leq 40}\beta_{i}{\rm a}_{i+8} + \sum_{41\leq i\leq 48}\beta_{i}{\rm a}_{i-8} + \beta_{49}{\rm a}_{50} + \beta_{50}{\rm a}_{49}+ \beta_{51}{\rm a}_{52} + \beta_{52}{\rm a}_{51}\\ \medskip &\quad + \sum_{53\leq i\leq 58}\beta_{i}{\rm a}_{i} + \beta_{59}{\rm a}_{60} + \beta_{60}{\rm a}_{59}+\sum_{61\leq i\leq 66}\beta_{i}{\rm a}_{i} + \beta_{67}{\rm a}_{68} + \beta_{68}{\rm a}_{67} + \sum_{69\leq i\leq 72}\beta_{i}{\rm a}_{i},\\ \sigma_6(g)&\equiv_{\overline{\omega}^{6}} \sum_{1\leq i\leq 2}\beta_{i}{\rm a}_{i} +\beta_{3}{\rm a}_{4} + \beta_{4}{\rm a}_{3} + \sum_{5\leq i\leq 10}\beta_{i}{\rm a}_{i} +\beta_{11}{\rm a}_{12} + \beta_{12}{\rm a}_{11}\\ &\quad + \sum_{13\leq i\leq 18}\beta_{i}{\rm a}_{i} +\beta_{19}{\rm a}_{20} + \beta_{20}{\rm a}_{19} + \sum_{21\leq i\leq 26}\beta_{i}{\rm a}_{i} +\beta_{27}{\rm a}_{28} + \beta_{28}{\rm a}_{27} + \sum_{29\leq i\leq 34}\beta_{i}{\rm a}_{i} \\ &\quad +\beta_{35}{\rm a}_{36} + \beta_{36}{\rm a}_{35} + \sum_{37\leq i\leq 40}\beta_{i}{\rm a}_{i} + \sum_{41\leq i\leq 48}\beta_{i}{\rm a}_{i+16} + \beta_{49}{\rm a}_{49} + \beta_{50}{\rm a}_{51}+ \beta_{51}{\rm a}_{50} \\ \medskip &\quad + \sum_{52\leq i\leq 56}\beta_{i}{\rm a}_{i}+ \sum_{57\leq i\leq 64}\beta_{i}{\rm a}_{i-16} + \beta_{65}{\rm a}_{65} + \beta_{66}{\rm a}_{67} + \beta_{67}{\rm a}_{66} +\sum_{68\leq i\leq 72}\beta_{i}{\rm a}_{i},\\ \sigma_7(g)&\equiv_{\overline{\omega}^{6}} \beta_{1}{\rm a}_{1} + \beta_{2}{\rm a}_{3}+\beta_{3}{\rm a}_{2} + \sum_{4\leq i\leq 9}\beta_{i}{\rm a}_{i} +\beta_{10}{\rm a}_{11} + \beta_{11}{\rm a}_{10} + \sum_{12\leq i\leq 17}\beta_{i}{\rm a}_{i} +\beta_{18}{\rm a}_{19} \\ &\quad + \beta_{19}{\rm a}_{18} + \sum_{20\leq i\leq 25}\beta_{i}{\rm a}_{i} +\beta_{26}{\rm a}_{27} + \beta_{27}{\rm a}_{26} + \sum_{28\leq i\leq 33}\beta_{i}{\rm a}_{i} +\beta_{34}{\rm a}_{35} + \beta_{35}{\rm a}_{34} \\ &\quad + \sum_{36\leq i\leq 41}\beta_{i}{\rm a}_{i} + \beta_{42}{\rm a}_{43}+\beta_{43}{\rm a}_{42} + \sum_{44\leq i\leq 48}\beta_{i}{\rm a}_{i}+ \sum_{49\leq i\leq 56}\beta_{i}{\rm a}_{i+8} + \sum_{57\leq i\leq 64}\beta_{i}{\rm a}_{i-8} \\ \medskip &\quad + \beta_{65}{\rm a}_{66} + \beta_{66}{\rm a}_{65} + \sum_{67\leq i\leq 72}\beta_{i}{\rm a}_{i},\\ \sigma_8(g)&\equiv_{\overline{\omega}^{6}} \beta_{1}{\rm a}_{2} + \beta_{2}{\rm a}_{1}+ \sum_{3\leq i\leq 8}\beta_{i}{\rm a}_{i} +\beta_{9}{\rm a}_{10} + \beta_{10}{\rm a}_{9} + \sum_{11\leq i\leq 16}\beta_{i}{\rm a}_{i} +\beta_{17}{\rm a}_{18} + \beta_{18}{\rm a}_{17} + \sum_{19\leq i\leq 24}\beta_{i}{\rm a}_{i} \\ &\quad +\beta_{25}{\rm a}_{26} + \beta_{26}{\rm a}_{25} + \sum_{27\leq i\leq 32}\beta_{i}{\rm a}_{i} +\beta_{33}{\rm a}_{34} + \beta_{34}{\rm a}_{33} + \sum_{35\leq i\leq 40}\beta_{i}{\rm a}_{i} + \beta_{41}{\rm a}_{42}+\beta_{42}{\rm a}_{41} \\ &\quad + \sum_{43\leq i\leq 48}\beta_{i}{\rm a}_{i}+ \sum_{49\leq i\leq 56}\beta_{i}{\rm a}_{i+16} +\beta_{57}{\rm a}_{58} + \beta_{58}{\rm a}_{57} + \sum_{59\leq i\leq 64}\beta_{i}{\rm a}_{i} + \sum_{65\leq i\leq 72}\beta_{i}{\rm a}_{i-16}. \end{array}$$ Then, from the relations $\sigma_d(g)\equiv_{\overline{\omega}^{6}} g,\, 1\leq d\leq 8,$ one gets $\beta_i = \beta_1$ for all $i,\, 2\leq i\leq 72.$ This means that $[QP^{\otimes 9}_{n_0}(\overline{\omega}^{6}]^{\Sigma_9} = \mathbb F_2\big[\sum_{1\leq i\leq 72}{\rm a}_{i}\big]_{\overline{\omega}^{6}}.$ Then, given any $[h]_{\overline{\omega}^{6}}\in [QP^{\otimes 9}_{n_0}(\overline{\omega}^{6}]^{GL_9},$ we have $h\equiv_{\overline{\omega}^{6}} \zeta\sum_{1\leq j\leq 72}{\rm a}_{i}$ with $\zeta\in \mathbb F_2.$ Since $\sigma_9(h) + h\equiv_{\overline{\omega}^{6}} 0,$ $\zeta = 0$, and therefore, $[QP^{\otimes 9}_{n_0}(\overline{\omega}^{6}]^{GL_9} = 0.$ Now, the theorem can be derived from the above results, in conjunction with the following facts: firstly, the transfer $\{Tr_h^{\mathcal A}\}_{h\geq 0}: \{\mathbb F_2\otimes_{GL_h}{\rm Ann}_{\overline{\mathcal A}}[P^{\otimes h}]^{*}\}_{h\geq 0}\longrightarrow \{{\rm Ext}_{\mathcal A}^{h, *}(\mathbb F_2, \mathbb F_2)\}_{h\geq 0}$ is an algebra homomorphism (see Singer \cite{Singer}), and secondly, according to \cite{Tangora, Bruner, Lin, Chen, Lin2}, the cohomology groups ${\rm Ext}_{\mathcal A}^{h, h+n}(\mathbb F_2, \mathbb F_2),$ where $h\geq 1$ and $1\leq n\leq n_0$, can be identified as follows: $$ {\rm Ext}_{\mathcal A}^{h, h+n}(\mathbb F_2, \mathbb F_2)=\left\{\begin{array}{ll} \mathbb F_2 h_1 &\mbox{if $h = 1$ and $n = 1,$}\\[1mm] 0 &\mbox{if $h \geq 2$ and $n = 1,$}\\[1mm] \mathbb F_2 h^{2}_1 &\mbox{if $h = 2$ and $n = 2,$}\\[1mm] 0 &\mbox{if $h \geq 1,\, h\neq 2$ and $n = 2,$}\\[1mm] \mathbb F_2 h_2 &\mbox{if $h = 1$ and $n = 3,$}\\[1mm] \mathbb F_2 h_0h_2 &\mbox{if $h = 2$ and $n = 3,$}\\[1mm] \mathbb F_2 h_1^{3} &\mbox{if $h = 3$ and $n = 3,$}\\[1mm] 0 &\mbox{if $h \geq 4$ and $n = 3,$}\\[1mm] 0 &\mbox{if $h \geq 1$ and $4\leq n\leq 5,$}\\[1mm] \mathbb F_2 h^{2}_2 &\mbox{if $h = 2$ and $n = 6,$}\\[1mm] \end{array}\right.$$ \newpage $$ {\rm Ext}_{\mathcal A}^{h, h+n}(\mathbb F_2, \mathbb F_2)=\left\{\begin{array}{ll} 0 &\mbox{if $h \geq 1,\, h\neq 2$ and $n = 6,$}\\[1mm] \mathbb F_2 h_3 &\mbox{if $h = 1$ and $n = 7,$}\\[1mm] \mathbb F_2 h_0h_3 &\mbox{if $h = 2$ and $n = 7,$}\\[1mm] \mathbb F_2 h_0^{2}h_3 &\mbox{if $h = 3$ and $n = 7,$}\\[1mm] \mathbb F_2 h_0^{3}h_3 &\mbox{if $h = 4$ and $n = 7,$}\\[1mm] 0 &\mbox{if $h \geq 5$ and $n = 7,$}\\[1mm] \mathbb F_2 h_1h_3 &\mbox{if $h = 2$ and $n = 8,$}\\[1mm] \mathbb F_2 c_0 &\mbox{if $h = 3$ and $n = 8,$}\\[1mm] 0 &\mbox{if $h\geq 1,\, h \neq 2,\, 3$ and $n = 8,$}\\[1mm] \mathbb F_2 h^{3}_2 &\mbox{if $h = 3$ and $n = 9,$}\\[1mm] \mathbb F_2 h_1c_0 &\mbox{if $h = 4$ and $n = 9,$}\\[1mm] \mathbb F_2 Ph_1 &\mbox{if $h = 5$ and $n = 9,$}\\[1mm] 0 &\mbox{if $h\geq 1,\, h \neq 3,\, 4,\, 5$ and $n = 9,$}\\[1mm] \mathbb F_2 h_1Ph_1 &\mbox{if $h = 6$ and $n = n_0,$}\\[1mm] 0&\mbox{if $h\geq 1,\, h \neq 6$ and $n = n_0.$} \end{array}\right.$$ \subsection{Proof of Theorem \ref{dlc1}}\label{s2.1} Recall that by Remark \ref{nxp10}, one gets $\dim (QP_{n_1}^{\otimes h})^{0}(\omega) = \sum_{\mu(n_1) = 4\leq k\leq h-1}\binom{h}{k}\dim (QP_{n_1}^{\otimes k})^{> 0}(\omega),$ where $\omega$ is a weight vector of degree $n_1.$ Sine $\mu(n_1) = 4,$ by Theorem \ref{dlWS}(iii), the Kameko homomorphism $(\widetilde {Sq^0_*})_{n_1}: QP^{\otimes 4}_{n_1}\longrightarrow QP^{\otimes 4}_{11}$ is an isomorphism. So, we have the inverse homomorphism $((\widetilde {Sq^0_*})_{n_1})^{-1}: QP^{\otimes 4}_{11}\longrightarrow QP^{\otimes 4}_{n_1},$ determined by $((\widetilde {Sq^0_*})_{n_1})^{-1}([y]) = [t_1t_2t_3t_4y^{2}]$ for all $[y]\in QP^{\otimes 4}_{11}.$ So, a basis of $QP^{\otimes 4}_{n_1} = (QP^{\otimes 4}_{n_1})^{>0}$ is the set of all the equivalence classes represented by the admissible monomials of the form $t_1t_2t_3t_4y^{2}$ where $y\in \mathscr C^{\otimes 4}_{11}.$ By Sum \cite{Sum2}, $|\mathscr C^{\otimes 4}_{11}| = 64$, which means that $\dim (QP^{\otimes 4}_{n_1})^{>0}(\omega) = \dim (QP^{\otimes 4}_{n_1})^{>0} = 64$ if $\omega = (4,3,2,1)$ and $(QP^{\otimes 4}_{n_1})^{>0}(\omega) = 0$ otherwise. So, by the above formula, $\dim (QP_{n_1}^{\otimes 5})^{0}(\omega) = \binom{5}{4}\dim (QP_{n_1}^{\otimes 4})^{> 0}(\omega) = 320$ if $\omega = (4,3,2,1)$ and $(QP^{\otimes 5}_{n_1})^{>0}(\omega) = 0$ otherwise. On the other side, since $QP_{n_1}^{\otimes 5}\cong (QP_{n_1}^{\otimes 5})^{0}\bigoplus (QP_{n_1}^{\otimes 5})^{>0},$ by Theorem \ref{dlWW}, we derive $(QP^{\otimes 5}_{n_1})^{>0}(\omega) = 0$ if $\omega\neq (4,3,2,1)$ and $\dim (QP_{n_1}^{\otimes 5})^{>0}(4,3,21) = \dim (QP_{n_1}^{\otimes 5})^{>0}(4,3,2,1) = 1024 - 320 = 704.$ Therefore, $$\dim (QP_{n_1}^{\otimes 6})^{0}(4,3,2,1) = \sum_{\mu(n_1) = 4\leq k\leq 5}\binom{6}{k}\dim (QP_{n_1}^{\otimes k})^{> 0}(4,3,2,1)= 5184, $$ and $(QP^{\otimes 6}_{n_1})^{0}(\omega) = 0$ if $\omega\neq (4,3,2,1).$ Since $(QP^{\otimes 6}_{n_1})^{0}\cong \bigoplus_{\deg(\omega) = n_1}(QP^{\otimes 6}_{n_1})^{0}(\omega),$ by the above calculations, we obtain $(QP^{\otimes 6}_{n_1})^{0}\cong (QP^{\otimes 6}_{n_1})^{0}(4,3,2,1).$ This completes the proof of Part (i). We will now proceed to prove Part (ii) of the theorem. Our first step is to compute the space $U_1$ by explicitly determining the monomial bases of $(QP_{n_1}^{\otimes 6})^{>0}(4,5,1,1)$ and $(QP_{n_1}^{\otimes 6})^{>0}(4,5,3).$ It is important to note that according to Remark \ref{nxp1}(i), if $t\in (P_{n_1}^{\otimes 6})^{>0}(4,5,1,1)$ or $t\in (P_{n_1}^{\otimes 6})^{>0}(4,5,3)$ such that $[t]\in {\rm Ker}((\widetilde {Sq^0_*})_{n_1}),$ then $t$ can be represented as $t_it_jt_kt_l\underline{t}^2,$ where $1\leq i<j<k<l\leq 6$ and $\underline{t}\in \mathscr C^{\otimes 6}_{11}.$ Therefore, in order to compute a monomial basis of $U_1,$ we need to determine all admissible monomials of degree $11$ in $P^{\otimes 6}.$ In \cite{MKR}, Mothebe et al. showed that $QP^{\otimes 6}_{11}$ has dimension $1205.$ Utilizing this result, we can explicitly determine all monomials of the form $t_it_jt_kt_l\underline{t}^2\in P^{\otimes 6}_{n_1}.$ In particular, utilizing the dimension result for $QP^{\otimes h}_{11}$ in \cite{MKR} and conducting a straightforward computation with Remark \ref{nxp10}, Corollary \ref{hq10-2}, and our previous work in \cite{Phuc11}, we obtain the following corollary. \begin{corl}\label{hqT} Let us consider the weight vectors of degree $11$: $$ \begin{array}{ll} & \widehat{\omega}_{(1)}:= (3,2,1),\ \widehat{\omega}_{(2)}:= (3,4),\ \widehat{\omega}_{(3)}:= (5,1,1),\ \widehat{\omega}_{(4)}:= (5,3),\\ & \widehat{\omega}_{(5)}:= (7,2),\ \widehat{\omega}_{(6)}:= (9,1),\ \widehat{\omega}_{(7)}:= (11,0). \end{array}$$ Then, for each $h\geq 6,$ we have the isomorphisms: $$ QP^{\otimes h}_{11}\cong \left\{\begin{array}{ll} \bigoplus_{1\leq j\leq 4}QP^{\otimes h}_{11}(\widehat{\omega}_{(j)}) &\mbox{if $h = 6$},\\[1mm] \bigoplus_{1\leq j\leq 5}QP^{\otimes h}_{11}(\widehat{\omega}_{(j)}) &\mbox{if $7\leq h\leq 8$},\\[1mm] \bigoplus_{1\leq j\leq 6}QP^{\otimes h}_{11}(\widehat{\omega}_{(j)}) &\mbox{if $9\leq h\leq 10$},\\[1mm] \bigoplus_{1\leq j\leq 7}QP^{\otimes h}_{11}(\widehat{\omega}_{(j)}) &\mbox{if $h \geq 11,$} \end{array}\right.$$ where $QP^{\otimes h}_{11}(\widehat{\omega}_{(j)})\cong (QP^{\otimes h}_{11})^{0}(\widehat{\omega}_{(j)})\bigoplus (QP^{\otimes h}_{11})^{>0}(\widehat{\omega}_{(j)}).$ The dimensions of $(QP^{\otimes h}_{11})^{0}(\widehat{\omega}_{(j)})$ and $(QP^{\otimes h}_{11})^{>0}(\widehat{\omega}_{(j)})$ are determined as follows: $$ \dim (QP^{\otimes h}_{11})^{0}(\widehat{\omega}_{(j)})=\left\{\begin{array}{ll} 8\binom{h}{3} + 32\binom{h}{4} + 40\binom{h}{5} &\mbox{if $h = 6,\, j=1$ {\rm(see \cite{Phuc11})}},\\[1mm] 10\binom{h}{5} &\mbox{if $h = 6,\, j=2$ {\rm(see \cite{Phuc11})}},\\[1mm] 15\binom{h}{5} &\mbox{if $h = 6,\, j=3$ {\rm(see \cite{Phuc11})}},\\[1mm] 10\binom{h}{5} &\mbox{if $h = 6,\, j=4$ {\rm(see \cite{Phuc11})}},\\[1mm] 0 &\mbox{if $h = 6,\, 5\leq j\leq 7$},\\[1mm] 8\binom{h}{3} + 32\binom{h}{4} + 40\binom{h}{5} + 16\binom{h}{6} &\mbox{if $h\geq 7,\, j=1$ {\rm (see \cite{Phuc11})}},\\[1mm] 10\binom{h}{5} +24\binom{h}{6} &\mbox{if $h = 7,\, j=2$ {\rm(see \cite{Phuc11})}},\\[1mm] 15\binom{h}{5} + 30\binom{h}{6} &\mbox{if $h = 7,\, j=3$ {\rm(see \cite{Phuc11})}},\\[1mm] 10\binom{h}{5}+45\binom{h}{6} &\mbox{if $h = 7,\, j=4$ {\rm(see \cite{Phuc11})}},\\[1mm] 0&\mbox{if $h = 7,\, 5\leq j\leq 7$},\\[1mm] 10\binom{h}{5} +24\binom{h}{6} +14\binom{h}{7} &\mbox{if $h \geq 8,\, j=2$},\\[1mm] 15\binom{h}{5} + 30\binom{h}{6} + 15\binom{h}{7} &\mbox{if $h \geq 8,\, j=3$},\\[1mm] 10\binom{h}{5}+45\binom{h}{6} + 70\binom{h}{7} &\mbox{if $h = 8,\, j=4$},\\[1mm] 21\binom{h}{7} &\mbox{if $h = 8,\, j=5$},\\[1mm] 0 &\mbox{if $h = 8,\, 6\leq j\leq 7$},\\[1mm] 10\binom{h}{5}+45\binom{h}{6} + 70\binom{h}{7} +35\binom{h}{8} &\mbox{if $h \geq 9,\, j=4$},\\[1mm] 21\binom{h}{7}+48\binom{h}{8} &\mbox{if $h \geq 9,\, j=5$},\\[1mm] 9\binom{h}{9} &\mbox{if $h = 10,\, j=6$},\\[1mm] 9\bigg(\binom{h}{9} + \binom{h}{10}\bigg) &\mbox{if $h \geq 11,\, j=6$},\\[1mm] 0 &\mbox{if $h = 11,\, j=7$},\\[1mm] \binom{h}{11} &\mbox{if $h \geq 12,\, j=7$}, \end{array}\right.$$ \newpage $$ \dim (QP^{\otimes h}_{11})^{> 0}(\widehat{\omega}_{(j)})=\left\{\begin{array}{ll} 16 &\mbox{if $h = 6,\, j = 1$ {\rm(see \cite{Phuc11})}},\\[1mm] 24 &\mbox{if $h = 6,\, j = 2$ {\rm(see \cite{Phuc11})}},\\[1mm] 30 &\mbox{if $h = 6,\, j = 3$ {\rm(see \cite{Phuc11})}},\\[1mm] 45 &\mbox{if $h = 6,\, j = 4$ {\rm(see \cite{Phuc11})}},\\[1mm] 0 &\mbox{if $h = 6,\, 5\leq j\leq 7$},\\[1mm] 0 &\mbox{if $h = 7,\, j = 1,\, 6,\, 7$},\\[1mm] \dim (QP^{\otimes h}_{h+4})^{> 0}(\overline{\omega}^{(j-1,\, h)}) &\mbox{if $h = 7,\, 2\leq j\leq 5$ {\rm(see Corollary \ref{hq10-2})}},\\[1mm] 0 &\mbox{if $h = 8,\, j\neq 4,\, 5$},\\[1mm] 35 &\mbox{if $h = 8,\, j = 4$},\\[1mm] 48 &\mbox{if $h = 8,\, j = 5$},\\[1mm] 0 &\mbox{if $h = 9,\, j\neq 5,\, 6$},\\[1mm] 27 &\mbox{if $h = 9,\, j = 5$},\\[1mm] \dim (QP^{\otimes h}_{1})^{0} = 9 &\mbox{if $h = 9,\, j = 6$},\\[1mm] 9 &\mbox{if $h = 10,\, j = 6$},\\[1mm] 0 &\mbox{if $h \geq 10,\, j \neq 6$}. \end{array}\right.$$ \end{corl} It must be noted that for $h = 7,\, 9,\, 11,$ we only need to compute the kernels of the Kameko squaring operations $(\widetilde {Sq^0_*})_{11}: QP^{\otimes h}_{11}\longrightarrow QP^{\otimes h}_{(11-h)/2}.$ Furthermore, we can readily deduce the following: $$ {\rm Ker}((\widetilde {Sq^0_*})_{11})\cong \left\{ \begin{array}{ll} (QP^{\otimes h}_{11})^{0}(\widehat{\omega}_{(1)})\bigoplus \bigg(\bigoplus_{2\leq j\leq 4}QP^{\otimes h}_{11}(\widehat{\omega}_{(j)})\bigg) &\mbox{if $h = 7$},\\[1mm] \bigg(\bigoplus_{1\leq j\leq 4}(QP^{\otimes h}_{11})^{0}(\widehat{\omega}_{(j)})\bigg)\bigoplus QP^{\otimes h}_{11}(\widehat{\omega}_{(5)})\bigoplus (QP^{\otimes h}_{11})^{>0}(\widehat{\omega}_{(6)})&\mbox{if $h = 9$},\\[1mm] \bigoplus_{1\leq j\leq 6}(QP^{\otimes h}_{11})^{0}(\widehat{\omega}_{(j)}) &\mbox{if $h = 11$}. \end{array}\right.$$ \medskip The following relevant observation is indispensable in order to establish the theorem: For each positive integer $n,$ the \textit{up Kameko map} $\psi: P_{n}^{\otimes 6}\longrightarrow P_{2n+6}^{\otimes 6}$ is an injective linear map defined on monomials by $\psi(t) = t_1t_2t_3t_4t_5t_6t^2.$ Then, from the calculations in \cite{Mothebe}, we deduce that for each $1\leq d\leq 4,\, d\in\mathbb Z,$ if $t\in (\mathscr {C}^{\otimes 5}_{n+1-2^{d}})^{>0},$ then $t_l^{2^{d}-1}\mathsf{q}_{l}(t)\in (\mathscr {C}^{\otimes 6}_{n})^{>0}$ for any $1\leq l\leq 6.$ In other words, if $t$ is an admissible monomial of degree $n+1-2^{d}$ in the $\mathcal A$-module $P^{\otimes 5},$ then $t_l^{2^{d}-1}\mathsf{q}_{l}(t)$ is an admissible monomial of degree $n$ in $\mathcal A$-module $P^{\otimes 6}.$ We set $ (\mathscr C(d, n))^{>0}:= \big\{t_l^{2^{d}-1}\mathsf{q}_{l}(t)|\, t\in (\mathscr {C}^{\otimes 5}_{n+1-2^{d}})^{>0},\, 1\leq l\leq 6\big\},\ 1\leq d\leq 4.$ Notice that when $n = n_1$ and $h = 6$, Kameko's maps can be rewritten as $ (\widetilde {Sq^0_*})_{n_1}: QP^{\otimes h}_{n_1} \longrightarrow QP^{\otimes h}_{\frac{n_1-h}{2}},\ \ \psi: P_{\frac{n_1-h}{2}}^{\otimes h}\longrightarrow P_{n_1}^{\otimes h}.$ So, $\psi\big(\mathscr {C}^{\otimes 6}_{\frac{n_1-h}{2}}\big)\subset (\mathscr C(d, n_1))^{>0}$ and $\widetilde {Sq^0_*}([u]) = [0]$ for all $u\in (\mathscr C(d, n_1))^{>0}\setminus \psi\big(\mathscr {C}^{\otimes 6}_{\frac{n_1-h}{2}}\big).$ These lead to $[u]\in {\rm Ker}((\widetilde {Sq^0_*})_{n_1}).$ According to the works in \cite{Sum3, Tin}, we have $$ \big|(\mathscr {C}^{\otimes 5}_{n_1+1-2^{d}})^{>0}\big| =\left \{\begin{array}{ll} 720 &\mbox{if $d = 1$},\\[1mm] 610 &\mbox{if $d = 2$},\\[1mm] 642 &\mbox{if $d = 3$},\\[1mm] 75&\mbox{if $d = 4$}. \end{array}\right.$$ Thanks to the results, a straightforward computation yields $$ \bigcup_{1\leq d\leq 4}(\mathscr C(d, n_1))^{>0} = D_1\bigcup D_2 \bigcup E,$$ wherein $$ \begin{array}{ll} &D_1 = \big\{c_j:\ 1\leq j\leq 336\big\},\ \ D_1\subset (\mathscr {C}^{\otimes 6}_{n_1})^{>0}(4,5,1,1),\\[1mm] &D_2 = \big\{d_j:\ 1\leq j\leq 210 \big\},\ \ D_2\subset (\mathscr {C}^{\otimes 6}_{n_1})^{>0}(4,5,3),\\[1mm] &E\subset \big((\mathscr {C}^{\otimes 6}_{n_1})^{>0}(4,3,2,1)\bigcup (\mathscr {C}^{\otimes 6}_{n_1})^{>0}(4,3,4)\bigcup \psi(\mathscr {C}^{\otimes 6}_{n_0})\big),\\[1mm] \end{array}$$ and the admissible monomials $c_j$ and $d_j$ are respectively described by Appendices \ref{s51} and \ref{s52}. The results obtained are based on filtering and removing the same monomials. For each monomial $u\in D_1,$ we notice that for any $[t]_{(4,5,1,1)}\in (QP^{\otimes 6}_{n_1})^{>0}(4,5,1,1),$ if $t$ is not equal to $u,$ then $t$ must take one of the following forms: \begin{enumerate} \item[$*$] $t_i^{3}t_j^{15}t_k^{2}t_{\ell}t_m^{2}t_p^{3}$,\ $t_i^{3}t_j^{15}t_k^{2}t_{\ell}^{2}t_mt_p^{3}$,\ $t_i^{3}t_j^{2}t_kt_{\ell}^{2}t_m^{7}t_p^{11}$,\ $t_i^{3}t_j^{2}t_k^{2}t_{\ell}t_m^{7}t_p^{11}$ where $j < k < \ell,$ and $(i, j, k, \ell, m, p)$ is a premutation of $(1, 2, 3, 4, 5, 6);$ \medskip \item [$*$] $t_i^{3}t_j^{2}t_kt_{\ell}^{14}t_m^{3}t_p^{3}$,\ $t_i^{3}t_j^{2}t_k^{14}t_{\ell}t_m^{3}t_p^{3}$,\ $t_i^{3}t_j^{14}t_kt_{\ell}^{2}t_m^{3}t_p^{3}$,\ $t_i^{3}t_j^{14}t_k^{2}t_{\ell}t_m^{3}t_p^{3}$,\ $t_1t_2^{2}t_3^{14}t_{4}^{3}t_5^{3}t_6^{3},$ where $(i, j, k, \ell, m, p)$ is a premutation of $(1, 2, 3, 4, 5, 6)$ and $j < k < \ell;$ \medskip \item [$*$] $t_i^{3}t_j^{2}t_kt_{\ell}^{7}t_m^{3}t_p^{10},$\ $t_i^{3}t_j^{2}t_k^{7}t_{\ell}t_m^{3}t_p^{10},$\ $t_i^{3}t_j^{7}t_kt_{\ell}^{2}t_m^{3}t_p^{10},$\ $t_i^{3}t_j^{7}t_k^{2}t_{\ell}t_m^{3}t_p^{10},$\ $t_i^{3}t_j^{2}t_k^{5}t_{\ell}^{10}t_m^{3}t_p^{3},$\ $t_i^{3}t_j^{2}t_k^{6}t_{\ell}^{9}t_m^{3}t_p^{3},$\ $t_i^{3}t_j^{6}t_k^{2}t_{\ell}^{9}t_m^{3}t_p^{3},$\ $t_i^{3}t_j^{6}t_k^{9}t_{\ell}^{2}t_m^{3}t_p^{3},$ where $j < k < \ell,$ and $(i, j, k, \ell, m, p)$ is a premutation of $(1, 2, 3, 4, 5, 6);$ \medskip \item [$*$] $t_i^{3}t_j^{3}t_k^{6}t_{\ell}t_m^{3}t_p^{10},$\ $t_1t_2^{6}t_3^{3}t_{4}^{3}t_5^{3}t_6^{10},$\ $t_1t_2^{6}t_3^{3}t_{4}^{3}t_5^{10}t_6^{3},$\ $t_1t_2^{6}t_3^{3}t_{4}^{10}t_5^{3}t_6^{3},$\ $t_1t_2^{6}t_3^{10}t_{4}^{3}t_5^{3}t_6^{3},$\ $t_i^{3}t_j^{2}t_k^{2}t_{\ell}^{5}t_m^{3}t_p^{11},$\ $t_i^{3}t_j^{2}t_k^{5}t_{\ell}^{2}t_m^{3}t_p^{11},$\\ $t_i^{3}t_j^{2}t_k^{13}t_{\ell}^{2}t_m^{3}t_p^{3},$\ $t_i^{3}t_j^{2}t_k^{2}t_{\ell}^{13}t_m^{3}t_p^{3},$\ $t_i^{3}t_j^{2}t_k^{2}t_{\ell}^{9}t_m^{7}t_p^{3},$\ $t_i^{3}t_j^{2}t_k^{9}t_{\ell}^{2}t_m^{7}t_p^{3},$ where $j < k < \ell,$ and $(i, j, k, \ell, m, p)$ is a premutation of $(1, 2, 3, 4, 5, 6);$ \medskip \item [$*$] $t_1t_j^{2}t_k^{6}t_{\ell}^{3}t_m^{3}t_p^{11},$\ $t_1t_j^{6}t_k^{2}t_{\ell}^{3}t_m^{3}t_p^{11},$ $t_1t_j^{6}t_k^{3}t_{\ell}^{2}t_m^{3}t_p^{11},$ $t_1^{3}t_j^{2}t_kt_{\ell}^{6}t_m^{3}t_p^{11},$\ $t_1^{3}t_j^{2}t_k^{6}t_{\ell}t_m^{3}t_p^{11},$ $t_1^{3}t_j^{6}t_kt_{\ell}^{2}t_m^{3}t_p^{11},$\ $t_1^{3}t_j^{6}t_k^{2}t_{\ell}t_m^{3}t_p^{11}$ with $j < k < \ell,$ and $(j, k, \ell, m, p)$ is a premutation of $(2, 3, 4, 5, 6);$ \medskip \item [$*$] $t_i^{3}t_j^{3}t_k^{3}t_{\ell}^{3}t_m^{2}t_p^{12},$\ $t_i^{3}t_j^{3}t_k^{3}t_{\ell}^{2}t_m^{4}t_p^{11},$\ $t_i^{3}t_j^{3}t_k^{3}t_{\ell}^{7}t_m^{2}t_p^{8},$\ $t_i^{3}t_j^{3}t_k^{3}t_{\ell}^{3}t_m^{4}t_p^{10},$\ $t_i^{3}t_j^{3}t_k^{3}t_{\ell}^{3}t_m^{6}t_p^{8},$ where $(i, j, k, \ell, m, p)$ is a premutation of $(1, 2, 3, 4, 5, 6).$ \end{enumerate} It is straightforward to check that these monomials are strictly inadmissible, and so, they are inadmissible. To exemplify, let us consider the monomials $t_it_j^{6}t_k^{3}t_{\ell}^{3}t_m^{3}t_p^{10}$ and $t_i^{3}t_j^{3}t_k^{3}t_{\ell}^{3}t_m^{2}t_p^{12}.$ It is clear that $\omega(t_it_j^{6}t_k^{3}t_{\ell}^{3}t_m^{3}t_p^{10}) = \omega(t_i^{3}t_j^{3}t_k^{3}t_{\ell}^{3}t_m^{2}t_p^{12}) = (4,5,1,1).$ As well known, the action of the Steenrod algebra $\mathcal A$ on the polynomial algebra $P^{\otimes 6}$ is given by the rule $$ Sq^k(t_j) = \left\{\begin{array}{ll} t_j &\mbox{if $k = 0,$}\\[1mm] t_j^2& \mbox{if $k = 1,$}\ \ \mbox{(\textit{the instability condition})},\\[1mm] 0 &\mbox{otherwise,} \end{array}\right.$$ and the Cartan formula $Sq^k(fg) = \sum_{a+b = k}Sq^{a}(f)Sq^{b}(g),$ for all $f,\, g\in P^{\otimes 6}.$ Note that for each $t\in P^{\otimes 1}$ and each positive integer $n,$ $Sq^{a}(t^{n}) = \binom{n}{a}t^{n+a},$ where the binomial coefficients $\binom{n}{a}$ are to be interpreted modulo 2 with the usual convention $\binom{n}{a} = 0$ if $n < a.$ Therefore, through a direct calculation, we obtain: $$ \begin{array}{ll} \medskip t_it_j^{6}t_k^{3}t_{\ell}^{3}t_m^{3}t_p^{10} &= Sq^{1}(t_i^{2}t_j^{3}t_k^{5}t_{\ell}^{3}t_m^{3}t_p^{9} + t_i^{2}t_j^{5}t_k^{3}t_{\ell}^{3}t_m^{3}t_p^{9} + t_i^{2}t_j^{3}t_k^{3}t_{\ell}^{3}t_m^{5}t_p^{9} + t_i^{2}t_j^{3}t_k^{3}t_{\ell}^{5}t_m^{3}t_p^{9} )\\ \medskip &\quad + Sq^{2}(t_it_j^{3}t_k^{5}t_{\ell}^{3}t_m^{3}t_p^{9} + t_it_j^{5}t_k^{3}t_{\ell}^{3}t_m^{3}t_p^{9} + t_it_j^{3}t_k^{3}t_{\ell}^{3}t_m^{5}t_p^{9} + t_it_j^{3}t_k^{3}t_{\ell}^{5}t_m^{3}t_p^{9})\\ \medskip &\quad + t_it_j^{3}t_k^{6}t_{\ell}^{3}t_m^{3}t_p^{10} + t_it_j^{3}t_k^{3}t_{\ell}^{3}t_m^{6}t_p^{10} + t_it_j^{3}t_k^{3}t_{\ell}^{6}t_m^{3}t_p^{10} \mod P_{26}^{\otimes 6}(< (4,5,1,1)),\\ t_i^{3}t_j^{3}t_k^{3}t_{\ell}^{3}t_m^{2}t_p^{12}&=Sq^{1}(t_i^{3}t_j^{3}t_k^{3}t_{\ell}^{3}t_mt_p^{12}) \mod P_{n_1}^{\otimes 6}(< (4,5,1,1)), \end{array}$$ which imply $t_it_j^{6}t_k^{3}t_{\ell}^{3}t_m^{3}t_p^{10}\equiv_{(4,5,1,1)} (t_it_j^{3}t_k^{6}t_{\ell}^{3}t_m^{3}t_p^{10} + t_it_j^{3}t_k^{3}t_{\ell}^{3}t_m^{6}t_p^{10} + t_it_j^{3}t_k^{3}t_{\ell}^{6}t_m^{3}t_p^{10}),$ and $t_i^{3}t_j^{3}t_k^{3}t_{\ell}^{3}t_m^{2}t_p^{12}\equiv_{(4,5,1,1)} 0.$ Hence, the monomials $ t_it_j^{6}t_k^{3}t_{\ell}^{3}t_m^{3}t_p^{10}$ and $t_i^{3}t_j^{3}t_k^{3}t_{\ell}^{3}t_m^{2}t_p^{12}$ are strictly inadmissible and $(4,5,1,1)$-hit, respectively. Since the monomials in $D_1$ are admissible, $\mathscr (C_{n_1}^{\otimes 6})^{>0}(4,5,1,1) = D_1.$ Thus, it may be claimed that $\dim (QP^{\otimes 6}_{n_1})^{>0}(4,5,1,1) = |D_1| = 336.$ Next, we observe that for each monomial $\widehat{u}\in D_2,$ if $[\widehat{t}]_{(4,5,3)}\in (QP^{\otimes 6}_{n_1})^{>0}(4,5,3),$ and $\widehat{t}\neq \widehat{u},$ then $\widehat{t}$ must have one of the following forms: \begin{enumerate} \item[$-$] $t_i^{3}t_j^{2}t_k^{2}t_{\ell}^{5}t_m^{7}t_p^{7}$,\ $t_i^{3}t_j^{2}t_k^{5}t_{\ell}^{2}t_m^{7}t_p^{7}$,\ $t_i^{7}t_j^{2}t_kt_{\ell}^{2}t_m^{7}t_p^{7}$,\ $t_i^{7}t_j^{2}t_k^{2}t_{\ell}t_m^{7}t_p^{7},$\ $t_i^{3}t_j^{6}t_k^{5}t_{\ell}^{6}t_m^{3}t_p^{3},$\ $t_i^{3}t_j^{6}t_k^{6}t_{\ell}^{5}t_m^{3}t_p^{3},$ where $j < k < \ell,$ and $(i, j, k, \ell, m, p)$ is a premutation of $(1, 2, 3, 4, 5, 6);$ \medskip \item[$-$] $t_it_j^{2}t_k^{6}t_{\ell}^{3}t_m^{7}t_p^{7},$\ $t_it_j^{6}t_k^{2}t_{\ell}^{3}t_m^{7}t_p^{7},$\ $t_it_j^{6}t_k^{3}t_{\ell}^{2}t_m^{7}t_p^{7},$\ $t_i^{3}t_j^{2}t_k^{6}t_{\ell}t_m^{7}t_p^{7},$\ $t_i^{3}t_j^{6}t_k^{2}t_{\ell}t_m^{7}t_p^{7},$ where $j < k < \ell,$ and $(i, j, k, \ell, m, p)$ is a premutation of $(1, 2, 3, 4, 5, 6);$ \medskip \item[$-$] $t_i^{3}t_j^{6}t_kt_{\ell}^{6}t_m^{3}t_p^{7}$,\ $t_i^{3}t_j^{6}t_k^{6}t_{\ell}t_m^{3}t_p^{7},$\ $t_it_j^{6}t_k^{3}t_{\ell}^{6}t_m^{3}t_p^{7},$\ $t_it_j^{6}t_k^{6}t_{\ell}^{3}t_m^{3}t_p^{7},$\ $t_i^{3}t_j^{2}t_k^{5}t_{\ell}^{6}t_m^{3}t_p^{7},$\ $t_i^{3}t_j^{2}t_k^{6}t_{\ell}^{5}t_m^{3}t_p^{7},$\ $t_i^{3}t_j^{6}t_k^{2}t_{\ell}^{5}t_m^{3}t_p^{7},$\\ $t_i^{3}t_j^{6}t_k^{5}t_{\ell}^{2}t_m^{3}t_p^{7},$ where $j < k < \ell,$ and $(i, j, k, \ell, m, p)$ is a premutation of $(1, 2, 3, 4, 5, 6);$ \medskip \item[$-$] $t_i^{3}t_j^{3}t_k^{7}t_{\ell}^{3}t_m^{4}t_p^{6}$,\ $t_i^{3}t_j^{3}t_k^{7}t_{\ell}^{7}t_m^{2}t_p^{4},$ where $(i, j, k, \ell, m, p)$ is a premutation of $(1, 2, 3, 4, 5, 6).$ \end{enumerate} It is indeed facile to simply assert that these monomials are inadmissible. As an illustration, let us consider the monomials $t_it_j^{2}t_k^{6}t_{\ell}^{3}t_m^{7}t_p^{7}$ and $t_i^{3}t_j^{3}t_k^{7}t_{\ell}^{3}t_m^{4}t_p^{6}.$ Then, $\omega(t_it_j^{2}t_k^{6}t_{\ell}^{3}t_m^{7}t_p^{7}) = \omega(t_i^{3}t_j^{3}t_k^{7}t_{\ell}^{3}t_m^{4}t_p^{6}) = (4,5,3)$ and by a simple computation, we get $$ \begin{array}{ll} \medskip t_it_j^{2}t_k^{6}t_{\ell}^{3}t_m^{7}t_p^{7} &= Sq^{2}(t_it_jt_k^{3}t_{\ell}^{5}t_m^{7}t_p^{7} + t_it_jt_k^{5}t_{\ell}^{3}t_m^{7}t_p^{7} + t_it_jt_k^{3}t_{\ell}^{3}t_m^{7}t_p^{9} + t_it_jt_k^{3}t_{\ell}^{3}t_m^{9}t_p^{7})\\ \medskip &\quad + Sq^{1}( t_i^{2}t_jt_k^{3}t_{\ell}^{5}t_m^{7}t_p^{7} + t_i^{2}t_jt_k^{5}t_{\ell}^{3}t_m^{7}t_p^{7}) + t_it_j^{2}t_k^{3}t_{\ell}^{6}t_m^{7}t_p^{7} \mod P_{n_1}^{\otimes 6}(< (4,5,3)),\\ t_i^{3}t_j^{3}t_k^{7}t_{\ell}^{3}t_m^{4}t_p^{6}&= Sq^{1}(t_i^{3}t_j^{3}t_k^{7}t_{\ell}^{3}t_m^{4}t_p^{5}) \mod P_{n_1}^{\otimes 6}(< (4,5,3)). \end{array} $$ These equalities show that $ t_it_j^{2}t_k^{6}t_{\ell}^{3}t_m^{7}t_p^{7}$ is strictly inadmissible (since $t_it_j^{2}t_k^{3}t_{\ell}^{6}t_m^{7}t_p^{7}< t_it_j^{2}t_k^{6}t_{\ell}^{3}t_m^{7}t_p^{7}$) and that $t_i^{3}t_j^{3}t_k^{7}t_{\ell}^{3}t_m^{4}t_p^{6}$ is $(4,5,3)$-hit. \medskip Since the monomials in $D_2$ are admissible, $\mathscr (C_{n_1}^{\otimes 6})^{>0}(4,5,3) = D_2.$ So, $\dim (QP^{\otimes 6}_{n_1})^{>0}(4,5,3) = |D_2| = 210.$ Incorporating the above-mentioned computation, we arrive at the conclusion that the dimension of $U_1$ is equal to $|D_1| + |D_2| = 336 + 210 = 546.$ \medskip The next step is to determine the dimension of $U_2$. Directly computing the dimension of this cohit module is a task of considerable complexity. Nevertheless, the methods employed to compute it are akin to those utilized in our earlier publications \cite{P.S1, Phuc4, Phuc6, Phuc10}. Therefore, to avoid redundancy, we shall present only a rough outline of the calculations for this $U_2.$ We commence by considering the set \begin{equation*} \mathcal{N}_h := \bigg\{(i; I)\;|\; I = (i_1,i_2,\ldots,i_r), 1\leqslant i < i_1<i_2 < \ldots < i_r\leqslant h, 0\leqslant r < h\bigg\},\ 5\leq h\leq 6. \end{equation*} Here we also use the notation $r=\ell(I)$ to denote the length of $I.$ If $r = 0,$ then by convention we take $I$ to be the empty set. For each pair $(i; I)\in\mathcal{N}_h,$ we define an $\mathcal A$-homomorphism $p_{(i; I)}: P^{\otimes h}\longrightarrow P^{\otimes (h-1)}$ by the following substitutions: $p_{(i; I)}(t_j) = t_j$ if $1\leqslant j < i,$ $p_{(i; I)}(t_j) = \sum\limits_{k\in I}t_{k-1}$ if $j = i,$ and $p_{(i; I)}(t_j) = t_{j-1}$ if $i < j \leqslant h.$ In particular, $p_{(i; \emptyset)}(t_i) = 0$ for every $i.$ and $p_{(i; I)}(\mathsf{q}_i(t)) = t$ for all $(i; I)\in \mathcal {N}_h$ and any $t\in P^{\otimes (h-1)}.$ Moreover, as shown in our previous work \cite{P.S1}, we can observe that $p_{(i; I)}$ induces a homomorphism from $QP^{\otimes h}_{n_1}(\omega)$ to $QP^{\otimes (h-1)}_{n_1}(\omega),$ where $\deg(\omega) = n_1.$ \medskip For each the pair $(i; I)\in\mathcal{N}_h,$ and $1\leq u < r,$ let us denote by $t_{(I;\,u)} := t_{i_u}^{2^{r-1} + 2^{r-2} +\, \cdots\, + 2^{r-u}}\prod_{u < d\leq r}t_{i_d}^{2^{r-d}},$ where $t_{(\emptyset; 1)} = 1.$ In \cite{Sum2}, Sum gives an interesting $\mathbb F_2$-linear function $\phi_{(i; I)}: P^{\otimes (h-1)}\longrightarrow P^{\otimes h},$ which is determined by $\phi_{(i; I)}(\prod_{1\leq j\leq h-1}t_j^{a_j}) = \dfrac{t_i^{2^{r} - 1}\mathsf{q}_{i}(\prod_{1\leq k\leq h-1}t_k^{a_k})}{t_{(I;\,u)}}, $ if there exists $u$ such that: \begin{equation*} \left\{ \begin{array}{ll} a_{i_1 - 1} +1= \cdots = a_{i_{(u-1)} - 1} +1 = 2^{r},\\[1mm] a_{i_{u} - 1} + 1 > 2^{r},\\[1mm] \alpha_{r-d}(a_{i_{u} - 1}) -1 = 0,\, 1\leq d\leq u, \\[1mm] \alpha_{r-d}(a_{i_{d}-1}) -1 = 0,\, \ u+1 \leq d \leq r, \end{array}\right. \end{equation*} and $\phi_{(i; I)}(\prod_{1\leq j\leq h-1}t_j^{a_j}) = 0$ otherwise. One has the following observation. If $I = \emptyset,$ then $\phi_{(i; I)} = \mathsf{q}_{i}$ for $1\leq i\leq h.$ It is in fact not hard to show that if $\phi_{(i; I)}( \prod_{1\leq j\leq h-1}t_j^{a_j})\neq 0,$ then $\omega(\phi_{(i; I)}( \prod_{1\leq j\leq h-1}t_j^{a_j})) = \omega(\prod_{1\leq j\leq h-1}t_j^{a_j}).$ By an elementary calculation, it is not so difficult to assert the following. \begin{lem}\label{bdbs} With the notations as above, if $t$ is an admissible monomial in the $\mathcal A$-module $P^{\otimes (h-1)},$ then $\phi_{(i; I)}(t)$ is also admissible monomial in the $\mathcal A$-module $P^{\otimes h}$ for $5\leq h\leq 6.$ \end{lem} We also believe that this result can be extended to any $h.$ For the convenience of readers, let us provide an example to illustrate this point. We put $\Gamma_6 = \{1,2,\ldots, 6\},\ T_{S} = T_{\{j_1,j_2,\ldots, j_d\}} = \prod_{j\in\Gamma_6\setminus S}t_j,$ where $S = \{j_1,j_2,\ldots, j_d\}\subseteq \Gamma_6.$ Now for $Y = \prod_{1\leq j\leq 6}t_j^{a_j}\in P^{\otimes 6}$ and for each non-negative integer $\gamma,$ we denote by $S_{\gamma}(Y) = \{j\in \Gamma_6:\; \alpha_{\gamma}(a_j) = 0\}\subseteq \Gamma_6.$ Then, the monomial $Y$ can be represented as $Y = \prod_{\gamma\geq 0}T_{S_{\gamma}(Y)}^{2^{\gamma}}.$ Given the monomial $Y =T_{\{6\}} = \prod_{1\leq j\leq 5}t_j\in P^{\otimes 6}_{5}$. Let $m$ be a positive integer such that $m > r = \ell(I).$ Then, one has the following equality: $$ \begin{array}{lll} \medskip \phi_{(i; I)}(Y^{2^m-1})&=& \dfrac{t_i^{2^r-1}t_2^{2^m-1}\ldots t_{i_1}^{2^m-1}\ldots t_{i_r}^{2^m-1}\ldots t_6^{2^m-1}}{t_{i_1}^{2^{r-1}}t_{i_2}^{2^{r-2}}\ldots t_{i_r}^{2^{r-r}}}= t_i^{2^r-1}t_{i_1}^{2^m-2^{r-1}-1}\ldots t_{i_2}^{2^m- 2^{r-2}-1}\ldots t_{i_r}^{2^m- 2^{r-r}-1}T^{2^m-1}_{\{i, i_1, \ldots, i_r\}}\\ &=&t_i^{2^r-1}\prod_{1\leq k\leq r}t_{i_k}^{2^m-2^{r-k}-1}T^{2^m-1}_{\{i, i_1, \ldots, i_r\}}\neq 0. \end{array}$$ Now, for each integer $q,\, 2\leq q \leq 6,$ if $r = q - 1,$ then $m\geq q.$ Let us consider the pair $(i; I)$ where $i = 1$ and $I = (i_1, i_2, \ldots, i_r) = (2, 3, \ldots, q).$ A direct computation shows that $$ \phi_{(1; I)}(Y^{2^m-1}) = t_1^{2^{q-1}-1}\prod_{1\leq k \leq q-1}t_{j}^{2^m-2^{q-1-k}-1}T^{2^m-1}_{\{1, 2, 3,\ldots, q\}}= t_1^{2^{q-1}-1}\ldots t_j^{2^m-2^{q-j}-1}\ldots t_q^{2^m-2}t_{q+1}^{2^m-1}\ldots t_6^{2^m-1}. $$ Owing to Theorem \ref{dlPS}, we know that $Y^{2^m-1}\in P^{\otimes 5}$ is admissible. On the other side, following the work of Mothebe \cite{Mothebe0}, the monomials $t_1^{2^{q-1}-1}\ldots t_j^{2^m-2^{q-j}-1}\ldots t_q^{2^m-2}t_{q+1}^{2^m-1}\ldots t_6^{2^m-1}$ are admissible. As a result, the non-zero monomial $\phi_{(1; I)}(Y^{2^m-1})\in P^{\otimes 6}$ is admissible as well. \medskip Applying a result in Sum \cite{Sum2} and combining it with Theorem \ref{dlWW}, we obtain $$\dim (QP^{\otimes 5}_{n_1})^{>0} = \dim (QP^{\otimes 5}_{n_1})^{>0}(4,3,2,1) =2^{\binom{5}{2}}-64\binom{5}{4} = 704.$$ Furthermore, a simple computation shows $(\mathscr C^{\otimes 5}_{n_1})^{>0} = \bigcup_{(i; I)\in \mathcal N_5}\phi_{(i; I)}(\mathscr C^{\otimes 4}_{n_1}) \bigcup S_1\bigcup S_2,$ where $$\begin{array}{ll} \medskip &S_1:= \bigcup_{1\leq d\leq 4}\big\{t_l^{2^{d}-1}\mathsf{q}_{l}(t)|\, t\in (\mathscr {C}^{\otimes 4}_{n_1+1-2^{d}})^{>0},\, 1\leq l\leq 5\big\},\ \mathsf{q}_{l}: P^{\otimes 4}\longrightarrow P^{\otimes 5}, \\ &S_2 := \bigg\{t_1^{3}t_2^{12}t_3^{3}t_4^{3}t_5^{5},\, t_1^{3}t_2^{12}t_3^{3}t_4^{5}t_5^{3},\, t_1^{3}t_2^{4}t_3^{3}t_4^{5}t_5^{11},\, t_1^{7}t_2^{9}t_3^{6}t_4t_5^{3},\, t_1^{7}t_2^{9}t_3^{6}t_4^{3}t_5,\, t_1^{7}t_2^{8}t_3^{3}t_4^{3}t_5^{5},\, t_1^{7}t_2^{8}t_3^{3}t_4^{5}t_5^{3}\bigg\}. \end{array}$$ The readers should also take note that, $QP^{\otimes 4}_{n_1} = (QP^{\otimes 4}_{n_1})^{>0}\cong QP^{\otimes 4}_{11},$ a cohit module of dimension $64$ (see \cite{Sum2}). This, in conjunction with Theorems \ref{dlKS}, \ref{dlSin}, \ref{dlWW}, and Lemma \ref{bdbs}, implies that $U_2 = \langle [\mathcal S] \rangle,$ wherein \begin{equation*} \begin{array}{ll} \mathcal S &= \bigg(E\setminus \psi(\mathscr C^{\otimes 6}_{n_0})\bigg)\bigcup F \bigcup \bigg(\bigcup_{(i; I)\in \mathcal N_6}\phi_{(i; I)}((\mathscr C^{\otimes 5}_{n_1})^{>0})\setminus (\mathscr C^{\otimes 6}_{n_1})^{0}\bigg)\\ & = \bigg\{z_j\in (P^{\otimes 6}_{n_1})^{>0}:\, \mbox{$z_j$ is admissible},\, \forall j,\, 1\leq j\leq 3090\bigg\}. \end{array} \end{equation*} Here $|F| = 3090 - \bigg|E\setminus \psi(\mathscr C^{\otimes 6}_{n_0})\bigg| - \bigg|\bigg(\bigcup_{(i; I)\in \mathcal N_6}\phi_{(i; I)}((\mathscr C^{\otimes 5}_{n_1})^{>0})\setminus (\mathscr C^{\otimes 6}_{n_1})^{0}\bigg)\bigg|,$ where the set $E$ was described earlier above. Hence, it may be asserted that $|\mathcal S| = 250 + 2840 = 3090.$ Suppose there exists a linear relation $\sum_{z\in \mathcal S}\gamma_zz\equiv 0,$ where $\gamma_z\in \mathbb F_2$ are coefficients. Using Theorems \ref{dlSin}, \ref{dlWW} and the homomorphisms $p_{(i; I)},$ for each pair $(i; I)\in \mathcal N_6,\, \ell(I) > 0,$ we determine explicitly $p_{(i; I)}(\sum_{z\in \mathcal S}\gamma_zz)$ in admissible terms in $(\mathscr C^{\otimes 5}_{n_1})^{>0}$ modulo $\overline{\mathcal A}P^{\otimes 5}_{n_1}.$ Then, since $p_{(i; I)}(\sum_{z\in\mathcal S}\gamma_zz)\equiv 0,$ for all $(i; I)\in \mathcal N_6,$ we obtain $\gamma_z = 0$ for all $z.$ These data lead to $U_2$ being an $\mathbb F_2$-vector space of dimension $|\mathcal S|.$ Thus, from the above calculations, ${\rm Ker}((\widetilde {Sq^0_*})_{n_1})\cap (QP^{\otimes 6}_{n_1})^{>0}$ is an $\mathbb F_2$-vector of dimension $3636.$ Finally, as $QP^{\otimes 6}_{n_1} \cong (QP^{\otimes 6}_{n_1})^{0}\bigoplus \big({\rm Ker}((\widetilde {Sq^0_*})_{n_1})\cap (QP^{\otimes 6}_{n_1})^{>0}\big)\bigoplus QP^{\otimes 6}_{n_0},$ we conclude that $$ \begin{array}{ll} \medskip \dim QP_{n_1}^{\otimes 6} &= \dim (QP_{n_1}^{\otimes 6})^{0} + \dim {\rm Ker}((\widetilde {Sq^0_*})_{n_1})\cap (QP^{\otimes 6}_{n_1})^{>0} + \dim QP_{n_0}^{\otimes 6}\\ & = 5184 + 3636+ 945 = 9765. \end{array}$$ The proof of the theorem is complete. \newpage To close this subsection, we would like to provide some insightful observations and remarks regarding the indecomposables $Q^{\otimes h}_{11}.$ \begin{nx} \begin{itemize} \item[(i)] With Corollary \ref{hqT} and a result from \cite{Tin} concerning $\dim Q^{\otimes 5}_{11}$ as our basis, we assert that the localized version of Kameko's conjecture in Note \ref{cyP}(i) holds true for all $h$ and degree $11$. \item[(ii)] Clearly, Corollary \ref{hqT} implies that the coinvariant $(\mathbb F_2\otimes_{GL_h}{\rm Ann}_{\overline{\mathcal A}}[P^{\otimes h}]^{*})_{11}$ is trivial for all $h\geq 12.$ Hence, Singer's Conjecture \ref{gtSinger} is true for bidegrees $(h, h+11)$ with $h>11.$ Moreover, in \cite{Phuc11}, we have demonstrated that the conjecture is also true for $6\leq h\leq 8.$ This is achieved by explicitly computing the dimensions of the invariants $[{\rm Ker}((\widetilde {Sq^0_*})_{11})]^{GL_7},\, [QP^{\otimes 7}_2]^{GL_7},$ and $[QP^{\otimes h}_{11}(\widehat{\omega}_{(j)})]^{GL_h}$ for $h = 6,\, 8,\, 1\leq j\leq 5.$ We then prove that the transfer homomorphism $Tr_h^{\mathcal A}$ is a monomorphism if $6\leq h\leq 7$ and is a trivial isomorphism if $h = 8.$ Here the weight vectors $\widehat{\omega}_{(j)}$ are given as in Corollary \ref{hqT}. It is noteworthy to mention that, the works of Ch\ohorn n and H\`a \cite{CHa, CHa0} demonstrated the non-surjectivity of the transfer in the bidegrees $(6, 6+11)$ and $(7, 7+11).$ According to the research conducted by \cite{Tangora, Bruner, Bruner2, Lin2}, it can be concluded that for every $h\geq 6,$ $$ {\rm Ext}_{\mathcal A}^{h, h+11}(\mathbb F_2, \mathbb F_2) = \left\{\begin{array}{ll} \mathbb F_2h_0Ph_2&\mbox{if $h = 6$},\\[1mm] \mathbb F_2h_0^{2}Ph_2 = \mathbb F_2 h_1^{2}Ph_1&\mbox{if $h = 7$},\\[1mm] 0&\mbox{if $h\geq 8$}. \end{array}\right.$$ For $h = 9,$ according to Corollary \ref{hqT}, we have $QP^{\otimes 9}_{1} = (QP^{\otimes 9}_{1})^{0} = \langle \{[t_i]\}_{1\leq i\leq 9}\rangle,$ and $QP^{\otimes 9}_{11}(\widehat{\omega}_{(j)}) = (QP^{\otimes 9}_{11})^{0}(\widehat{\omega}_{(j)})$ for $1\leq j\leq 4.$ So the invariants $[QP^{\otimes 9}_{1}]^{GL_9}$ and $QP^{\otimes 9}_{11}(\widehat{\omega}_{(j)}),\, 1\leq j\leq 4$ are trivial. By combining these data with Corollary \ref{hqT} and taking into account that the mapping $(\widetilde {Sq^0_*})_{11}: QP^{\otimes 9}_{11}\longrightarrow QP^{\otimes 9}_2$ is a surjective, we can derive an estimate $$\dim (\mathbb F_2\otimes_{GL_{9}}{\rm Ann}_{\overline{\mathcal A}}[P^{\otimes 9}]^{*})_{11}=\dim [QP^{\otimes 9}_{11}]^{GL_9}\leq \dim [{\rm Ker}((\widetilde {Sq^0_*})_{11})]^{GL_9}\leq \dim [QP^{\otimes 9}_{11}(\widehat{\omega}_{(5)})]^{GL_9}.$$ Owing to Corollary \ref{hqT}, one has an isomorphism $QP^{\otimes 9}_{11}(\widehat{\omega}_{(5)})\cong (QP^{\otimes 9}_{11})^{0}(\widehat{\omega}_{(5)})\bigoplus (QP^{\otimes 9}_{11})^{>0}(\widehat{\omega}_{(5)})$ where $\dim (QP^{\otimes 9}_{11})^{0}(\widehat{\omega}_{(5)}) = 21\binom{9}{7} + 48\binom{9}{8} = 1188$ and $\dim (QP^{\otimes 9}_{11})^{>0}(\widehat{\omega}_{(5)}) = 27.$ When $h=10$, it follows from Corollary \ref{hqT} that the invariants $QP^{\otimes 10}_{11}(\widehat{\omega}_{(j)})$ are trivial for $1\leq j\leq 5.$ As a result, we can establish an inequality $$\dim (\mathbb F_2\otimes_{GL_{10}}{\rm Ann}_{\overline{\mathcal A}}[P^{\otimes 10}]^{*})_{11}=\dim [QP^{\otimes 10}_{11}]^{GL_{10}}\leq \dim [QP^{\otimes 10}_{11}(\widehat{\omega}_{(6)})]^{GL_{10}}.$$ According to Corollary \ref{hqT}, $QP^{\otimes 10}_{11}(\widehat{\omega}_{(6)})\cong (QP^{\otimes 10}_{11})^{0}(\widehat{\omega}_{(6)})\bigoplus (QP^{\otimes 10}_{11})^{>0}(\widehat{\omega}_{(6)})$ where $$\dim (QP^{\otimes 10}_{11})^{0}(\widehat{\omega}_{(6)}) = 9\binom{10}{9} =90\ \mbox{and}\ \dim (QP^{\otimes 10}_{11})^{>0}(\widehat{\omega}_{(6)}) = 9.$$ In the case where $h=11$, it is possible to derive from Corollary \ref{hqT} that $$QP^{\otimes 11}_{11}\cong \bigoplus_{1\leq j\leq 6}(QP^{\otimes 11}_{11})^{0}(\widehat{\omega}_{(j)})\bigoplus (QP^{\otimes 11}_{11})^{>0}(\widehat{\omega}_{(7)}),$$ where $(QP^{\otimes 11}_{11})^{>0}(\widehat{\omega}_{(7)}) \cong \mathbb F_2[t_1t_2\ldots t_{11}]_{\widehat{\omega}_{(7)}}.$ Using the $\mathcal A$-homomorphisms $\sigma_d: P^{\otimes 11}\longrightarrow P^{\otimes 11},$ for $1\leq d\leq 11,$ one gets $\dim (\mathbb F_2\otimes_{GL_{11}}{\rm Ann}_{\overline{\mathcal A}}[P^{\otimes 11}]^{*})_{11} =\dim [QP^{\otimes 11}_{11}]^{GL_{11}} = 0 = \dim {\rm Ext}_{\mathcal A}^{11, 22}(\mathbb F_2, \mathbb F_2).$ Hence, Singer's transfer is a trivial isomorphism in bidegree $(11, 22).$ Thus if the invariants $[QP^{\otimes 9}_{11}(\widehat{\omega}_{(5)})]^{GL_9}$ and $[QP^{\otimes 10}_{11}(\widehat{\omega}_{(6)})]^{GL_{10}}$ are trivial, then Singer's Conjecture \ref{gtSinger} also holds for bidegrees $(h, h+11)$ with $9\leq h\leq 11.$ This matter will be thoroughly investigated and discussed in another context. \end{itemize} \end{nx} \subsection{Proof of Theorem \ref{dlc4}} In what follows, suppose that $\omega$ is a weight vector of degree $n_1.$ We denote by $\mathscr C^{\otimes 6}_{n_1}(\omega)$ the set of all admissible monomials in $P^{\otimes 6}_{n_1}(\omega)$ and by $[\mathscr C^{\otimes 6}_{n_1}(\omega)]_{\omega} = \{[t]_{\omega}:\ t\in \mathscr C^{\otimes 6}_{n_1}(\omega)\}.$ For $z_1, z_2,\ldots, z_m\in P^{\otimes 6}_{n_1}(\omega)$ with $m\geq 1,$ we put $$ \begin{array}{ll} \medskip \Sigma_6(z_1, \ldots, z_m) &= \big\{\theta(z_j):\ \theta\in \Sigma_6,\, 1\leq j\leq m\big\},\\ \medskip [\mathscr C(z_1, \ldots, z_m)]_{\omega}&= [\mathscr C^{\otimes 6}_{n_1}(\omega)]_{\omega}\cap \langle [\Sigma_6(z_1, \ldots, z_m)]_{\omega} \rangle,\\ \widehat{p(z)}&= \sum_{y\in \mathscr C^{\otimes 6}_{n_1}(\omega)\cap \Sigma_6(z)}y. \end{array}$$ $\langle [\Sigma_6(z_1, \ldots, z_m)]_{\omega} \rangle$ is manifestly a $\Sigma_6$-submodule of $QP^{\otimes 6}_{n_1}(\omega).$ As we have pointed out before, $$ {\rm Ker}((\widetilde {Sq^0_*})_{n_1})\cong (QP_{n_1}^{\otimes 6})(4,3,2,1)\bigoplus QP_{n_1}^{\otimes 6}(4,3,4)\bigoplus QP_{n_1}^{\otimes 6}(4,5,1,1)\bigoplus QP_{n_1}^{\otimes 6}(4,5,3), $$ where $$ \begin{array}{ll} \medskip &(QP_{n_1}^{\otimes 6})(4,3,4) = (QP_{n_1}^{\otimes 6})^{>0}(4,3,4),\ \ QP_{n_1}^{\otimes 6}(4,5,1,1) = (QP_{n_1}^{\otimes 6})^{>0}(4,5,1,1),\\ &QP_{n_1}^{\otimes 6}(4,5,3) = (QP_{n_1}^{\otimes 6})^{>0}(4,5,3). \end{array}$$ So, one gets an estimate $$\begin{array}{ll} \dim {\rm [}{\rm Ker}((\widetilde {Sq^0_*})_{n_1}){\rm]}^{GL_6}&\leq \dim {\rm [}QP_{n_1}^{\otimes 6}(4,3,2,1){\rm ]}^{GL_6} +\dim {\rm [}QP_{n_1}^{\otimes 6}(4,3,4){\rm ]}^{GL_6} \\ &\quad + \dim {\rm [}QP_{n_1}^{\otimes 6}(4,5,1,1){\rm ]}^{GL_6} + \dim {\rm [}QP_{n_1}^{\otimes 6}(4,5,3){\rm ]}^{GL_6}. \end{array}$$ By using the monomial basis of the space $QP_{n_1}^{\otimes 6}(\omega)$ with $\omega\in \{(4,3,2,1), (4,3,4), (4,5,1,1), (4,5,3)\}$ (see Theorem \ref{dlc1}) and the homomorphisms $\sigma_d$ for $1\leq d\leq 6,$ we find that the invariants $[QP_{n_1}^{\otimes 6}(\omega)]^{GL_6}$ are zero. Indeed, we will prove this claim for the invariants $[QP_{n_1}^{\otimes 6}(4,5,1,1)]^{GL_6}$ and $[QP_{n_1}^{\otimes 6}(4,5,3)]^{GL_6}$ in detail. Similarly, we also obtain the results for the other spaces. \medskip We set $\widetilde{\omega}:= (4,5,1,1)$ and $\omega:= (4,5,3).$ We first describe the space $[QP_{n_1}^{\otimes 6}(\widetilde{\omega})]^{GL_6}.$ Following the proof of Theorem \ref{dlc1}, the space $QP_{n_1}^{\otimes 6}(\widetilde{\omega}) = (QP_{n_1}^{\otimes 6})^{>0}(\widetilde{\omega})$ is $336$-dimensional with the monomial basis $\{[c_j]_{\widetilde{\omega}}:\ 1\leq j\leq 336\}.$ Note that the admissible monomials $c_j$ are explicitly described as in Subsect. \ref{s51}. Let us consider the following admissible monomials: $$ \begin{array}{ll} \medskip c_1 &= t_1^{3}t_2t_3^{15}t_4^{2}t_5^{2}t_6^{3}, \ \ c_{61} = t_1^{3}t_2^{7}t_3^{11}t_4t_5^{2}t_6^{2},\ \ c_{121} = t_1t_2^{3}t_3^{14}t_4^{2}t_5^{2}t_6^{3},\\ \medskip c_{156} &= t_1^{3}t_2^{3}t_3^{13}t_4^{2}t_5^{2}t_6^{3}, \ \ c_{166} = t_1^{3}t_2t_3^{2}t_4^{3}t_5^{6}t_6^{11},\ \ c_{196} = t_1^{3}t_2^{5}t_3^{11}t_4^{2}t_5^{2}t_6^{3},\\ c_{211} &= t_1t_2^{7}t_3^{10}t_4^{2}t_5^{3}t_6^{3}, \ \ c_{286} = t_1^{3}t_2^{7}t_3^{9}t_4^{2}t_5^{2}t_6^{3},\ \ c_{301} = t_1^{3}t_2t_3^{3}t_4^{3}t_5^{6}t_6^{10}. \end{array}$$ It is easy to see that $[c_{156}]_{\widetilde{\omega}} = [\theta_1(c_{121})]_{\widetilde{\omega}},$\ $[c_{196}]_{\widetilde{\omega}} = [\theta_2(c_{166})]_{\widetilde{\omega}},$ and $[c_{286}]_{\widetilde{\omega}} = [\theta_3(c_{211})]_{\widetilde{\omega}},$ where $\theta_j$ are suitable permutations in $\Sigma_6.$ So, the following spaces are $\Sigma_6$-submodules of $QP_{n_1}^{\otimes 6}(\widetilde{\omega})$: $$ \begin{array}{ll} \medskip \langle[\Sigma_6(c_{1})]_{\widetilde{\omega}}\rangle &= \langle \{[c_j]_{\widetilde{\omega}}:\ 1\leq j\leq 60\} \rangle,\ \ \langle[\Sigma_6(c_{61})]_{\widetilde{\omega}}\rangle = \langle \{[c_j]_{\widetilde{\omega}}:\ 61\leq j\leq 120\} \rangle,\\ \medskip \langle[\Sigma_6(c_{121})]_{\widetilde{\omega}}\rangle &= \langle \{[c_j]_{\widetilde{\omega}}:\ 121\leq j\leq 165\} \rangle,\ \ \langle[\Sigma_6(c_{166})]_{\widetilde{\omega}}\rangle = \langle \{[c_j]_{\widetilde{\omega}}:\ 166\leq j\leq 210\} \rangle,\\ \langle[\Sigma_6(c_{211})]_{\widetilde{\omega}}\rangle &= \langle \{[c_j]_{\widetilde{\omega}}:\ 211\leq j\leq 300\} \rangle,\ \ \langle[\Sigma_6(c_{301})]_{\widetilde{\omega}}\rangle = \langle \{[c_j]_{\widetilde{\omega}}:\ 301\leq j\leq 336\} \rangle. \end{array}$$ So, we have an isomorphism $$ \begin{array}{ll} \medskip QP_{n_1}^{\otimes 6}(\widetilde{\omega}) &\cong \langle[\Sigma_6(c_{1})]_{\widetilde{\omega}}\rangle\bigoplus \langle[\Sigma_6(c_{61})]_{\widetilde{\omega}}\rangle \bigoplus \langle[\Sigma_6(c_{121})]_{\omega}\rangle\\ &\quad\bigoplus \langle[\Sigma_6(c_{166})]_{\widetilde{\omega}}\rangle\bigoplus \langle[\Sigma_6(c_{211})]_{\widetilde{\omega}}\rangle\bigoplus \langle[\Sigma_6(c_{301})]_{\widetilde{\omega}}\rangle. \end{array}$$ By direct calculations using the homomorphisms $\sigma_i: P^{\otimes 6}\longrightarrow P^{\otimes 6}$ for $1\leq i\leq 5,$ we obtain the following results: $$ \begin{array}{ll} \medskip \langle[\Sigma_6(c_{j})]_{\widetilde{\omega}}\rangle^{\Sigma_6} &= 0,\ \ \mbox{for $j = 121,\, 166,\, 211,$}\\ \langle[\Sigma_6(c_{j})]_{\widetilde{\omega}}\rangle^{\Sigma_6} &= \langle [\widehat{p(c_j)}]_{\widetilde{\omega}}\rangle,\ \ \mbox{for $j = 1,\, 61,\, 301$}, \end{array}$$ where $\widehat{p(c_j)} = \sum_{c_j\in \mathscr C(c_j)} c_j$ with $\mathscr C(c_1) = \{c_j:\ 1\leq j\leq 60\},$\ $\mathscr C(c_{61}) = \{c_j:\ 61\leq j\leq 120\},$ and $\mathscr C(c_{301}) = \{c_j:\ 301\leq j\leq 336\}.$ Note that the sets $[\mathscr C(c_j)]_{\widetilde{\omega}}$ are the bases of the spaces $\langle[\Sigma_6(c_{j})]_{\widetilde{\omega}}\rangle$ for $j = 1,\, 61,\, 301.$ Thus, one gets $[QP_{n_1}^{\otimes 6}(\widetilde{\omega})]^{\Sigma_6} = \langle [\widehat{p(c_1)}]_{\widetilde{\omega}},\ [\widehat{p(c_{61})}]_{\widetilde{\omega}},\ [\widehat{p(c_{301})}]_{\widetilde{\omega}} \rangle.$ Now, assume that $[g]_{\widetilde{\omega}}\in [QP_{n_1}^{\otimes 6}(\widetilde{\omega})]^{GL_6},$ then because $\Sigma_6\subset GL_6,$ we must have that $g\equiv_{\widetilde{\omega}}\gamma_1\widehat{p(c_1)} + \gamma_2\widehat{p(c_{61})} + \gamma_3\widehat{p(c_{301})},$ in which $\gamma_j\in \mathbb F_2$ for every $j.$ By a simple computation using the homomorphism $\sigma_6$ and the relation $\sigma_6(g) +g\equiv_{\widetilde{\omega}}0,$ we get $\sigma_6(g) +g\equiv_{\widetilde{\omega}} (\gamma_1c_5 + (\gamma_1 + \gamma_2)(c_{23} + c_{24} + c_{25}) + \gamma_{3}c_{308} + \ \mbox{other terms })\equiv_{\widetilde{\omega}}0,$ which implies that $\gamma_1 = \gamma_2 = \gamma_3 = 0.$ Hence, the invariant $[QP_{n_1}^{\otimes 6}(\widetilde{\omega})]^{GL_6}$ is equal to $0.$ \medskip Next, we compute the space $[QP_{n_1}^{\otimes 6}(\omega)]^{GL_6}.$ Following the proof of Theorem \ref{dlc1}, $\dim QP_{n_1}^{\otimes 6}(\omega) = 210,$ and $QP_{n_1}^{\otimes 6}(\omega) = \langle \{[d_j]_{\omega}:\ 1\leq j\leq 210\}\rangle,$ where the admissible monomials $d_j$ are explicitly described as in Subsect. \ref{s52}. By a simple computation, we have a direct summand decomposition of the $\Sigma_6$-submodules: $$ QP_{n_1}^{\otimes 6}(\omega) = \langle[\Sigma_6(d_1)]_{\omega}\rangle\bigoplus \langle[\Sigma_6(d_{21})]_{\omega}\rangle \bigoplus \langle[\Sigma_6(d_{111})]_{\omega}\rangle\bigoplus \langle[\Sigma_6(d_{201})]_{\omega}\rangle,$$ where $$ \begin{array}{ll} \medskip \langle[\Sigma_6(d_1)]_{\omega}\rangle\ &=\langle \{[d_j]_{\omega}:\ 1\leq j\leq 20\} \rangle, \ \ \ \langle[\Sigma_6(d_{21})]_{\omega}\rangle =\langle \{[d_j]_{\omega}:\ 21\leq j\leq 110\} \rangle,\\ \langle[\Sigma_6(d_{111})]_{\omega}\rangle &=\langle \{[d_j]_{\omega}:\ 111\leq j\leq 200\} \rangle, \ \ \ \langle[\Sigma_6(d_{201})]_{\omega}\rangle =\langle \{[d_j]_{\omega}:\ 201\leq j\leq 210\} \rangle. \end{array}$$ We first compute the action of the symmetric group $\Sigma_6$ on $ QP_{n_1}^{\otimes 6}(\omega).$ We find that $$[QP_{n_1}^{\otimes 6}(\omega)]^{\Sigma_6} = \big\langle \{[\sum_{1\leq j\leq 20}d_j]_{\omega},\ [\sum_{21\leq j\leq 110}d_j]_{\omega},\ [\sum_{111\leq j\leq 200}d_j]_{\omega}\}\big \rangle.$$ This is immediate from the following assertions: \begin{enumerate} \item[i)] $ \langle[\Sigma_6(d_1)]_{\omega}\rangle^{\Sigma_6} = \langle [\widehat{p(d_1)}]_{\omega}$ with $\widehat{p(d_1)}:= \sum_{1\leq j\leq 20}d_j;$ \medskip \item[ii)] $\langle[\Sigma_6(d_{21})]_{\omega}\rangle^{\Sigma_6} = \langle [\widehat{p(d_{21})}]_{\omega} \rangle$ with $\widehat{p(d_{21})}:= \sum_{21\leq j\leq 110}d_j;$ \medskip \item[iii)] $\langle[\Sigma_6(d_{111})]_{\omega}\rangle^{\Sigma_6} = \langle [\widehat{p(d_{111})}]_{\omega} \rangle$ with $\widehat{p(d_{111})}:= \sum_{111\leq j\leq 200}d_j;$ \medskip \item[iv)] $\langle[\Sigma_6(d_{201})]_{\omega}\rangle^{\Sigma_6} = 0.$ \end{enumerate} We compute the cases i) and iv) and leave the rest to the reader. It is straightforward to see that the sets $[\mathscr C(d_1)]_{\omega} = \{[d_j]_{\omega}:\ 1\leq j\leq 20\}$ and $[\mathscr C(d_{201})]_{\omega} =\{[d_j]_{\omega}:\ 201\leq j\leq 210\}$ are the bases of the spaces $\langle[\Sigma_6(d_1)]_{\omega}\rangle$ and $\langle[\Sigma_6(d_{201})]_{\omega}\rangle,$ respectively. Suppose that $[f]_{\omega}\in \langle[\Sigma_6(d_1)]_{\omega}\rangle^{\Sigma_6}$ and $[g]_{\omega}\in \langle[\Sigma_6(d_{201})]_{\omega}\rangle^{\Sigma_6}.$ Then, one has that $f\equiv_{\omega}\sum_{1\leq j\leq 20}\gamma_jd_j,$ and $g\equiv_{\omega}\sum_{201\leq j\leq 210}\beta_jd_j,$ in which the coefficients $\gamma_j$ and $\beta_j$ belong to $\mathbb F_2$ for all $j.$ By direct calculations using the homomorphisms $\sigma_i: P^{\otimes 6}\longrightarrow P^{\otimes 6}$ for $1\leq i\leq 5,$ and the relations $\sigma_i(f) + f\equiv_{\omega}0,$ and $\sigma_i(g) + g\equiv_{\omega}0,$ we obtain the following equalities: $$ \begin{array}{ll} \sigma_1(f) +f& \equiv_{\omega} (\gamma_5 + \gamma_{11})(d_5 + d_{11}) + (\gamma_6 + \gamma_{12})(d_6 + d_{12}) + (\gamma_7 + \gamma_{13})(d_7 + d_{13}) \\ \medskip &\quad + (\gamma_8 + \gamma_{14})(d_8 + d_{14})+ (\gamma_9 + \gamma_{15})(d_9 + d_{15}) + (\gamma_{10} + \gamma_{16})(d_{10} + d_{16}) \equiv_{\omega} 0,\\ \sigma_2(f) + f&\equiv_{\omega} (\gamma_2 + \gamma_{5})(d_2 + d_{5}) + (\gamma_3 + \gamma_{6})(d_3 + d_{6}) + (\gamma_4 + \gamma_{7})(d_4 + d_{7})\\ \medskip &\quad + (\gamma_{14} + \gamma_{17})(d_{14} + d_{17}) + (\gamma_{15} + \gamma_{18})(d_{15} + d_{18}) + (\gamma_{16} + \gamma_{19})(d_{16} + d_{19})\equiv_{\omega} 0,\\ \sigma_3(f) + f&\equiv_{\omega} (\gamma_1 + \gamma_{2})(d_1 + d_{2}) + (\gamma_6 + \gamma_{8})(d_6 + d_{8}) + (\gamma_7 + \gamma_{9})(d_7 + d_{9}) \\ \medskip &\quad + (\gamma_{12} + \gamma_{14})(d_{12} + d_{14}) + (\gamma_{13} + \gamma_{15})(d_{13} + d_{15}) + (\gamma_{19} + \gamma_{20})(d_{19} + d_{20})\equiv_{\omega} 0,\\ \sigma_4(f) + f&\equiv_{\omega} (\gamma_2 + \gamma_{3})(d_2 + d_{3}) + (\gamma_5 + \gamma_{6})(d_5 + d_{6}) + (\gamma_9 + \gamma_{10})(d_9 + d_{10})\\ \medskip &\quad + (\gamma_{11} + \gamma_{12})(d_{11} + d_{12}) + (\gamma_{15} + \gamma_{16})(d_{15} + d_{16}) + (\gamma_{18} + \gamma_{19})(d_{18} + d_{19})\equiv_{\omega} 0,\\ \sigma_5(f) + f&\equiv_{\omega} (\gamma_3 + \gamma_{4})(d_3 + d_{4}) + (\gamma_6 + \gamma_{7})(d_6 + d_{7}) + (\gamma_8 + \gamma_{9})(d_8 + d_{9})\\ \medskip &\quad + (\gamma_{12} + \gamma_{13})(d_{12} + d_{13}) + (\gamma_{14} + \gamma_{15})(d_{14} + d_{15}) + (\gamma_{17} + \gamma_{18})(d_{17} + d_{18}) \equiv_{\omega} 0,\\ \end{array}$$ \newpage $$ \begin{array}{ll} \medskip \sigma_1(g) + g &\equiv_{\omega} \beta_{205}d_{205} + (\beta_{205} + \beta_{208} + \beta_{209})(d_{205} + d_{208} + d_{209})\\ &\quad + (\beta_{206} + \beta_{208} + \beta_{210})(d_{206} + d_{208} + d_{210})\\ \medskip &\quad + (\beta_{207} + \beta_{209} + \beta_{210})(d_{207} + d_{209} + d_{210}) \equiv_{\omega} 0,\\ \sigma_2(g) + g &\equiv_{\omega} (\beta_{202} + \beta_{205})(d_{202} + d_{205}) + (\beta_{203} + \beta_{206})(d_{203} + d_{206})\\ \medskip &\quad + (\beta_{204} + \beta_{207})(d_{204} + d_{207}) \equiv_{\omega} 0,\\ \sigma_3(g) + g &\equiv_{\omega} (\beta_{201} + \beta_{202})(d_{201} + d_{202}) + (\beta_{206} + \beta_{208})(d_{206} + d_{208})\\ \medskip &\quad + (\beta_{207} + \beta_{209})(d_{207} + d_{209})\equiv_{\omega} 0,\\ \sigma_4(g) + g &\equiv_{\omega} (\beta_{202} + \beta_{203})(d_{202} + d_{203}) + (\beta_{205} + \beta_{206})(d_{205} + d_{206})\\ \medskip &\quad + (\beta_{209} + \beta_{210})(d_{209} + d_{210})\equiv_{\omega} 0,\\ \sigma_5(g) + g &\equiv_{\omega} (\beta_{203} + \beta_{204})(d_{203} + d_{204}) + (\beta_{206} + \beta_{207})(d_{206} + d_{207})\\ &\quad + (\beta_{208} + \beta_{209})(d_{208} + d_{209}) \equiv_{\omega} 0. \end{array}$$ The above equalities imply that $\gamma_{j} = \gamma_{1}$ for all $j,\, 2\leq j\leq 20,$ and $\beta_{201} = \beta_{202} = \cdots = \beta_{210} = 0.$ Next, we compute the action of the general linear group $GL_6$ on $QP_{n_1}^{\otimes 6}(\omega).$ Since $\Sigma_6\subset GL_6,$ if the equivalence class $[h]_{\omega}$ belongs to the invariant $[QP_{n_1}^{\otimes 6}(\omega)]^{GL_6},$ then $$h\equiv_{\omega} \xi_1\widehat{p(d_1)} + \xi_2\widehat{p(d_{21})} + \xi_3\widehat{p(d_{111})},\ \ \xi_j\in \mathbb F_2,\ j = 1,\, 2,\, 3.$$ Using the homomorphism $\sigma_6: P^{\otimes 6}\longrightarrow P^{\otimes 6}$ and the relation $\sigma_6(h) + h\equiv_{\omega} 0,$ we get $$ \sigma_6(h) + h\equiv_{\omega} \big(\xi_1\big(\sum_{5\leq j\leq 10}d_j\big) + \xi_2d_{27} + \xi_3d_{111} + \mbox{other terms }\big)\equiv_{\omega} 0.$$ The above equality indicates that $\xi_1 = \xi_2 = \xi_3 = 0,$ and therefore $[QP_{n_1}^{\otimes 6}(\omega)]^{GL_6}$ is zero. \medskip Thus, from the above calculations, one gets $[{\rm Ker}((\widetilde {Sq^0_*})_{n_1})]^{GL_6} = 0$. On the other hand, because $$\dim [QP_{n_1}^{\otimes 6}]^{GL_6}\leq \dim [{\rm Ker}((\widetilde {Sq^0_*})_{n_1})]^{GL_6} + \dim [QP_{n_0}^{\otimes 6}]^{GL_6}$$ and $[QP_{n_0}^{\otimes 6}]^{GL_6} \cong (\mathbb F_2 \otimes_{GL_6} {\rm Ann}_{\overline{\mathcal A}}[P^{\otimes 6}]^{*})_{n_0} = 0$ (see \cite{Phuc11}), one gets $[QP_{n_1}^{\otimes 6}]^{GL_6} = 0.$ By this and Corollary \ref{hqs22}), the coinvariant $(\mathbb F_2 \otimes_{GL_6} {\rm Ann}_{\overline{\mathcal A}}[P^{\otimes 6}]^{*})_{n_s}$ vanishes for every positive integer $s.$ Now, it is well-known (see, for instance, Tangora \cite{Tangora}) that the only elements $h_2^{2}g_1 = h_4Ph_2$, and $D_2$ are non-zero in ${\rm Ext}_{\mathcal A}^{6, 6+n_1}(\mathbb F_2, \mathbb F_2)$, and ${\rm Ext}_{\mathcal A}^{6, 6+n_2}(\mathbb F_2, \mathbb F_2),$ respectively. These data and the above calculations indicate that the sixth algebraic transfer, $Tr_6^{\mathcal A}$ is a monomorphism, but not an epimorphism in degrees $n_s,$ for $0 < s\leq 2.$ Therefore, Singer's transfer $Tr_6^{\mathcal A}$ does not detect the non-zero elements $h_4Ph_2$ and $D_2.$ One can observe that the result for $s=3$ is a consequence of the fact (as referenced in \cite{Bruner, Bruner2, Lin2}) that the sixth cohomology group ${\rm Ext}_{\mathcal A}^{6,6+n_3}(\mathbb F_2, \mathbb F_2)$ is trivial. The proof of the theorem is complete. \section{Conclusion}\label{s5} The central emphasis of our work is to further investigate the hit problem for the polynomial algebra $P^{\otimes 6} = \mathbb F_2[t_1, \ldots, t_6]$ in degree $n_s = 16\cdot 2^{s}-6$ for general $s$, building on the results from a previous work in \cite{MKR} which addressed the case $s = 0$. We have shown that the cohit $\mathbb F_2$-module $(P^{\otimes h}/\overline{\mathcal A}P^{\otimes h}){n_{h, s}}$ is equal to the order of the factor group $GL_{h-1}/B_{h-1}$ in general degrees $n_{h, s}=2^{s+4}-h$ with $h\geq 6$ and $s\geq h-5$. Also, based on a previously established result in \cite{Hai}, we have indicated that the cohit $\mathbb F_q$-module $(\mathbb F_q[t_1,\ldots, t_h]/\overline{\pmb A}_q\mathbb F_q[t_1,\ldots, t_h])_{q^{h-1}-h}$ is equal to the order of the factor group $GL_{h-1}(\mathbb F_q)/B^{*}_{h-1}(\mathbb F_q).$ As applications, we have established the dimension result for the cohit module $(\mathbb F_2\otimes_{\mathcal A}P^{\otimes 7})_{n_{s+5}}$ and confirmed Singer's Conjecture \ref{gtSinger} for bidegrees $(h, h+n)$ with $h\geq 1$ and $1\leq n\leq n_0$, as well as for bidegrees $(6, 6+n_s)$ with $s\geq 1$. One of the important corollaries then states that the sixth algebraic transfer does not detect the non-zero elements $h_2^{2}g_1 = h_4Ph_2\in {\rm Ext}_{\mathcal A}^{6, 6+n_1}(\mathbb F_2, \mathbb F_2)$ and $D_2\in {\rm Ext}_{\mathcal A}^{6, 6+n_2}(\mathbb F_2, \mathbb F_2).$ The research conducted in this article advances the understanding of the Peterson hit problem and Singer's algebraic transfer, representing a significant contribution to the literature on this topic. In particular, the application of the hit problem technique in this study has showcased its effectiveness as a powerful tool for investigating the algebraic transfer. We therefore can expect that this approach will lead to more significant breakthroughs in the future. \section{Appendix}\label{s6} In this section, we first enumerate all admissible monomials in the spaces $(P^{\otimes 6}_{n_1})^{> 0}(4,5,1,1) = P^{\otimes 6}_{n_1}(4,5,1,1)$ and $(P^{\otimes 6}_{n_1})^{> 0}(4,5,3) = P^{\otimes 6}_{n_1}(4,5,3).$ \subsection{Admissible monomials in \mbox{\boldmath $(P^{\otimes 6}_{n_1})^{> 0}(4,5,1,1)$}}\label{s51} According to the proof of Theorem \ref{dlc1}, the dimension of $(QP^{\otimes 6}_{n_1})^{>0}(4,5,1,1)$ is equal to the cardinality of $D_1,$ where $D_1 = \big\{c_j:\ 1\leq j\leq 336\big\}$ and the admissible monomials $c_j$ are given as follows: \begin{center} \begin{tabular}{llll} ${\rm c}_{1}=t_1^{3}t_2t_3^{15}t_4^{2}t_5^{2}t_6^{3}$, & ${\rm c}_{2}=t_1^{3}t_2t_3^{15}t_4^{2}t_5^{3}t_6^{2}$, & ${\rm c}_{3}=t_1^{3}t_2t_3^{15}t_4^{3}t_5^{2}t_6^{2}$, & ${\rm c}_{4}=t_1^{3}t_2^{3}t_3^{15}t_4t_5^{2}t_6^{2}$, \\ ${\rm c}_{5}=t_1t_2^{3}t_3^{15}t_4^{2}t_5^{2}t_6^{3}$, & ${\rm c}_{6}=t_1t_2^{3}t_3^{15}t_4^{2}t_5^{3}t_6^{2}$, & ${\rm c}_{7}=t_1t_2^{3}t_3^{15}t_4^{3}t_5^{2}t_6^{2}$, & ${\rm c}_{8}=t_1t_2^{2}t_3^{15}t_4^{2}t_5^{3}t_6^{3}$, \\ ${\rm c}_{9}=t_1t_2^{2}t_3^{15}t_4^{3}t_5^{2}t_6^{3}$, & ${\rm c}_{10}=t_1t_2^{2}t_3^{15}t_4^{3}t_5^{3}t_6^{2}$, & ${\rm c}_{11}=t_1^{3}t_2t_3^{2}t_4^{2}t_5^{3}t_6^{15}$, & ${\rm c}_{12}=t_1^{3}t_2t_3^{2}t_4^{2}t_5^{15}t_6^{3}$, \\ ${\rm c}_{13}=t_1^{3}t_2t_3^{2}t_4^{3}t_5^{2}t_6^{15}$, & ${\rm c}_{14}=t_1^{3}t_2t_3^{2}t_4^{3}t_5^{15}t_6^{2}$, & ${\rm c}_{15}=t_1^{3}t_2t_3^{2}t_4^{15}t_5^{2}t_6^{3}$, & ${\rm c}_{16}=t_1^{3}t_2t_3^{2}t_4^{15}t_5^{3}t_6^{2}$, \\ ${\rm c}_{17}=t_1^{3}t_2t_3^{3}t_4^{2}t_5^{2}t_6^{15}$, & ${\rm c}_{18}=t_1^{3}t_2t_3^{3}t_4^{2}t_5^{15}t_6^{2}$, & ${\rm c}_{19}=t_1^{3}t_2t_3^{3}t_4^{15}t_5^{2}t_6^{2}$, & ${\rm c}_{20}=t_1^{3}t_2^{3}t_3t_4^{2}t_5^{2}t_6^{15}$, \\ ${\rm c}_{21}=t_1^{3}t_2^{3}t_3t_4^{2}t_5^{15}t_6^{2}$, & ${\rm c}_{22}=t_1^{3}t_2^{3}t_3t_4^{15}t_5^{2}t_6^{2}$, & ${\rm c}_{23}=t_1^{3}t_2^{15}t_3t_4^{2}t_5^{2}t_6^{3}$, & ${\rm c}_{24}=t_1^{3}t_2^{15}t_3t_4^{2}t_5^{3}t_6^{2}$, \\ ${\rm c}_{25}=t_1^{3}t_2^{15}t_3t_4^{3}t_5^{2}t_6^{2}$, & ${\rm c}_{26}=t_1^{3}t_2^{15}t_3^{3}t_4t_5^{2}t_6^{2}$, & ${\rm c}_{27}=t_1^{15}t_2t_3^{2}t_4^{2}t_5^{3}t_6^{3}$, & ${\rm c}_{28}=t_1^{15}t_2t_3^{2}t_4^{3}t_5^{2}t_6^{3}$, \\ ${\rm c}_{29}=t_1^{15}t_2t_3^{2}t_4^{3}t_5^{3}t_6^{2}$, & ${\rm c}_{30}=t_1^{15}t_2t_3^{3}t_4^{2}t_5^{2}t_6^{3}$, & ${\rm c}_{31}=t_1^{15}t_2t_3^{3}t_4^{2}t_5^{3}t_6^{2}$, & ${\rm c}_{32}=t_1^{15}t_2t_3^{3}t_4^{3}t_5^{2}t_6^{2}$, \\ ${\rm c}_{33}=t_1^{15}t_2^{3}t_3t_4^{2}t_5^{2}t_6^{3}$, & ${\rm c}_{34}=t_1^{15}t_2^{3}t_3t_4^{2}t_5^{3}t_6^{2}$, & ${\rm c}_{35}=t_1^{15}t_2^{3}t_3t_4^{3}t_5^{2}t_6^{2}$, & ${\rm c}_{36}=t_1^{15}t_2^{3}t_3^{3}t_4t_5^{2}t_6^{2}$, \\ ${\rm c}_{37}=t_1t_2^{3}t_3^{2}t_4^{2}t_5^{3}t_6^{15}$, & ${\rm c}_{38}=t_1t_2^{3}t_3^{2}t_4^{2}t_5^{15}t_6^{3}$, & ${\rm c}_{39}=t_1t_2^{3}t_3^{2}t_4^{3}t_5^{2}t_6^{15}$, & ${\rm c}_{40}=t_1t_2^{3}t_3^{2}t_4^{3}t_5^{15}t_6^{2}$, \\ ${\rm c}_{41}=t_1t_2^{3}t_3^{2}t_4^{15}t_5^{2}t_6^{3}$, & ${\rm c}_{42}=t_1t_2^{3}t_3^{2}t_4^{15}t_5^{3}t_6^{2}$, & ${\rm c}_{43}=t_1t_2^{3}t_3^{3}t_4^{2}t_5^{2}t_6^{15}$, & ${\rm c}_{44}=t_1t_2^{3}t_3^{3}t_4^{2}t_5^{15}t_6^{2}$, \\ ${\rm c}_{45}=t_1t_2^{3}t_3^{3}t_4^{15}t_5^{2}t_6^{2}$, & ${\rm c}_{46}=t_1t_2^{15}t_3^{2}t_4^{2}t_5^{3}t_6^{3}$, & ${\rm c}_{47}=t_1t_2^{15}t_3^{2}t_4^{3}t_5^{2}t_6^{3}$, & ${\rm c}_{48}=t_1t_2^{15}t_3^{2}t_4^{3}t_5^{3}t_6^{2}$, \\ ${\rm c}_{49}=t_1t_2^{15}t_3^{3}t_4^{2}t_5^{2}t_6^{3}$, & ${\rm c}_{50}=t_1t_2^{15}t_3^{3}t_4^{2}t_5^{3}t_6^{2}$, & ${\rm c}_{51}=t_1t_2^{15}t_3^{3}t_4^{3}t_5^{2}t_6^{2}$, & ${\rm c}_{52}=t_1t_2^{2}t_3^{3}t_4^{2}t_5^{3}t_6^{15}$, \\ ${\rm c}_{53}=t_1t_2^{2}t_3^{3}t_4^{2}t_5^{15}t_6^{3}$, & ${\rm c}_{54}=t_1t_2^{2}t_3^{3}t_4^{3}t_5^{2}t_6^{15}$, & ${\rm c}_{55}=t_1t_2^{2}t_3^{3}t_4^{3}t_5^{15}t_6^{2}$, & ${\rm c}_{56}=t_1t_2^{2}t_3^{3}t_4^{15}t_5^{2}t_6^{3}$, \\ ${\rm c}_{57}=t_1t_2^{2}t_3^{3}t_4^{15}t_5^{3}t_6^{2}$, & ${\rm c}_{58}=t_1t_2^{2}t_3^{2}t_4^{3}t_5^{15}t_6^{3}$, & ${\rm c}_{59}=t_1t_2^{2}t_3^{2}t_4^{15}t_5^{3}t_6^{3}$, & ${\rm c}_{60}=t_1t_2^{2}t_3^{2}t_4^{3}t_5^{3}t_6^{15}$, \\ ${\rm c}_{61}=t_1^{3}t_2^{7}t_3^{11}t_4t_5^{2}t_6^{2}$, & ${\rm c}_{62}=t_1^{7}t_2^{3}t_3^{11}t_4t_5^{2}t_6^{2}$, & ${\rm c}_{63}=t_1t_2^{7}t_3^{11}t_4^{2}t_5^{2}t_6^{3}$, & ${\rm c}_{64}=t_1^{7}t_2t_3^{11}t_4^{2}t_5^{2}t_6^{3}$, \\ ${\rm c}_{65}=t_1t_2^{7}t_3^{11}t_4^{2}t_5^{3}t_6^{2}$, & ${\rm c}_{66}=t_1^{7}t_2t_3^{11}t_4^{2}t_5^{3}t_6^{2}$, & ${\rm c}_{67}=t_1t_2^{7}t_3^{11}t_4^{3}t_5^{2}t_6^{2}$, & ${\rm c}_{68}=t_1^{7}t_2t_3^{11}t_4^{3}t_5^{2}t_6^{2}$, \\ ${\rm c}_{69}=t_1^{3}t_2t_3^{2}t_4^{2}t_5^{7}t_6^{11}$, & ${\rm c}_{70}=t_1^{3}t_2t_3^{2}t_4^{7}t_5^{2}t_6^{11}$, & ${\rm c}_{71}=t_1^{3}t_2t_3^{2}t_4^{7}t_5^{11}t_6^{2}$, & ${\rm c}_{72}=t_1^{3}t_2t_3^{7}t_4^{2}t_5^{2}t_6^{11}$, \\ ${\rm c}_{73}=t_1^{3}t_2t_3^{7}t_4^{2}t_5^{11}t_6^{2}$, & ${\rm c}_{74}=t_1^{3}t_2t_3^{7}t_4^{11}t_5^{2}t_6^{2}$, & ${\rm c}_{75}=t_1^{3}t_2^{7}t_3t_4^{2}t_5^{2}t_6^{11}$, & ${\rm c}_{76}=t_1^{3}t_2^{7}t_3t_4^{2}t_5^{11}t_6^{2}$, \\ ${\rm c}_{77}=t_1^{3}t_2^{7}t_3t_4^{11}t_5^{2}t_6^{2}$, & ${\rm c}_{78}=t_1t_2^{3}t_3^{2}t_4^{2}t_5^{7}t_6^{11}$, & ${\rm c}_{79}=t_1t_2^{3}t_3^{2}t_4^{7}t_5^{2}t_6^{11}$, & ${\rm c}_{80}=t_1t_2^{3}t_3^{2}t_4^{7}t_5^{11}t_6^{2}$, \\ ${\rm c}_{81}=t_1t_2^{3}t_3^{7}t_4^{2}t_5^{2}t_6^{11}$, & ${\rm c}_{82}=t_1t_2^{3}t_3^{7}t_4^{2}t_5^{11}t_6^{2}$, & ${\rm c}_{83}=t_1t_2^{3}t_3^{7}t_4^{11}t_5^{2}t_6^{2}$, & ${\rm c}_{84}=t_1^{7}t_2^{3}t_3t_4^{2}t_5^{2}t_6^{11}$, \\ ${\rm c}_{85}=t_1^{7}t_2^{3}t_3t_4^{2}t_5^{11}t_6^{2}$, & ${\rm c}_{86}=t_1^{7}t_2^{3}t_3t_4^{11}t_5^{2}t_6^{2}$, & ${\rm c}_{87}=t_1t_2^{2}t_3^{3}t_4^{2}t_5^{7}t_6^{11}$, & ${\rm c}_{88}=t_1t_2^{2}t_3^{3}t_4^{7}t_5^{2}t_6^{11}$, \\ ${\rm c}_{89}=t_1t_2^{2}t_3^{3}t_4^{7}t_5^{11}t_6^{2}$, & ${\rm c}_{90}=t_1t_2^{7}t_3^{3}t_4^{2}t_5^{2}t_6^{11}$, & ${\rm c}_{91}=t_1t_2^{7}t_3^{3}t_4^{2}t_5^{11}t_6^{2}$, & ${\rm c}_{92}=t_1t_2^{7}t_3^{3}t_4^{11}t_5^{2}t_6^{2}$, \\ ${\rm c}_{93}=t_1^{7}t_2t_3^{3}t_4^{2}t_5^{2}t_6^{11}$, & ${\rm c}_{94}=t_1^{7}t_2t_3^{3}t_4^{2}t_5^{11}t_6^{2}$, & ${\rm c}_{95}=t_1^{7}t_2t_3^{3}t_4^{11}t_5^{2}t_6^{2}$, & ${\rm c}_{96}=t_1^{7}t_2^{11}t_3^{3}t_4t_5^{2}t_6^{2}$, \\ ${\rm c}_{97}=t_1t_2^{2}t_3^{2}t_4^{7}t_5^{11}t_6^{3}$, & ${\rm c}_{98}=t_1t_2^{2}t_3^{7}t_4^{2}t_5^{11}t_6^{3}$, & ${\rm c}_{99}=t_1t_2^{2}t_3^{7}t_4^{11}t_5^{2}t_6^{3}$, & ${\rm c}_{100}=t_1t_2^{7}t_3^{2}t_4^{2}t_5^{11}t_6^{3}$, \\ ${\rm c}_{101}=t_1t_2^{7}t_3^{2}t_4^{11}t_5^{2}t_6^{3}$, & ${\rm c}_{102}=t_1^{7}t_2t_3^{2}t_4^{2}t_5^{11}t_6^{3}$, & ${\rm c}_{103}=t_1^{7}t_2t_3^{2}t_4^{11}t_5^{2}t_6^{3}$, & ${\rm c}_{104}=t_1^{7}t_2^{11}t_3t_4^{2}t_5^{2}t_6^{3}$, \\ ${\rm c}_{105}=t_1t_2^{2}t_3^{2}t_4^{7}t_5^{3}t_6^{11}$, & ${\rm c}_{106}=t_1t_2^{2}t_3^{7}t_4^{2}t_5^{3}t_6^{11}$, & ${\rm c}_{107}=t_1t_2^{2}t_3^{7}t_4^{11}t_5^{3}t_6^{2}$, & ${\rm c}_{108}=t_1t_2^{7}t_3^{2}t_4^{2}t_5^{3}t_6^{11}$, \\ ${\rm c}_{109}=t_1t_2^{7}t_3^{2}t_4^{11}t_5^{3}t_6^{2}$, & ${\rm c}_{110}=t_1^{7}t_2t_3^{2}t_4^{2}t_5^{3}t_6^{11}$, & ${\rm c}_{111}=t_1^{7}t_2t_3^{2}t_4^{11}t_5^{3}t_6^{2}$, & ${\rm c}_{112}=t_1^{7}t_2^{11}t_3t_4^{2}t_5^{3}t_6^{2}$, \\ ${\rm c}_{113}=t_1t_2^{2}t_3^{2}t_4^{3}t_5^{7}t_6^{11}$, & ${\rm c}_{114}=t_1t_2^{2}t_3^{7}t_4^{3}t_5^{2}t_6^{11}$, & ${\rm c}_{115}=t_1t_2^{2}t_3^{7}t_4^{3}t_5^{11}t_6^{2}$, & ${\rm c}_{116}=t_1t_2^{7}t_3^{2}t_4^{3}t_5^{2}t_6^{11}$, \\ ${\rm c}_{117}=t_1t_2^{7}t_3^{2}t_4^{3}t_5^{11}t_6^{2}$, & ${\rm c}_{118}=t_1^{7}t_2t_3^{2}t_4^{3}t_5^{2}t_6^{11}$, & ${\rm c}_{119}=t_1^{7}t_2t_3^{2}t_4^{3}t_5^{11}t_6^{2}$, & ${\rm c}_{120}=t_1^{7}t_2^{11}t_3t_4^{3}t_5^{2}t_6^{2}$, \\ ${\rm c}_{121}=t_1t_2^{3}t_3^{14}t_4^{2}t_5^{3}t_6^{3}$, & ${\rm c}_{122}=t_1t_2^{3}t_3^{14}t_4^{3}t_5^{2}t_6^{3}$, & ${\rm c}_{123}=t_1^{3}t_2t_3^{14}t_4^{2}t_5^{3}t_6^{3}$, & ${\rm c}_{124}=t_1^{3}t_2t_3^{14}t_4^{3}t_5^{2}t_6^{3}$, \\ ${\rm c}_{125}=t_1t_2^{3}t_3^{14}t_4^{3}t_5^{3}t_6^{2}$, & ${\rm c}_{126}=t_1^{3}t_2t_3^{14}t_4^{3}t_5^{3}t_6^{2}$, & ${\rm c}_{127}=t_1^{3}t_2t_3^{2}t_4^{3}t_5^{3}t_6^{14}$, & ${\rm c}_{128}=t_1^{3}t_2t_3^{2}t_4^{3}t_5^{14}t_6^{3}$, \\ ${\rm c}_{129}=t_1^{3}t_2t_3^{3}t_4^{2}t_5^{3}t_6^{14}$, & ${\rm c}_{130}=t_1^{3}t_2t_3^{3}t_4^{2}t_5^{14}t_6^{3}$, & ${\rm c}_{131}=t_1^{3}t_2t_3^{3}t_4^{3}t_5^{2}t_6^{14}$, & ${\rm c}_{132}=t_1^{3}t_2t_3^{3}t_4^{3}t_5^{14}t_6^{2}$, \\ ${\rm c}_{133}=t_1^{3}t_2t_3^{3}t_4^{14}t_5^{2}t_6^{3}$, & ${\rm c}_{134}=t_1^{3}t_2t_3^{3}t_4^{14}t_5^{3}t_6^{2}$, & ${\rm c}_{135}=t_1^{3}t_2^{3}t_3t_4^{2}t_5^{3}t_6^{14}$, & ${\rm c}_{136}=t_1^{3}t_2^{3}t_3t_4^{2}t_5^{14}t_6^{3}$, \\ ${\rm c}_{137}=t_1^{3}t_2^{3}t_3t_4^{3}t_5^{2}t_6^{14}$, & ${\rm c}_{138}=t_1^{3}t_2^{3}t_3t_4^{3}t_5^{14}t_6^{2}$, & ${\rm c}_{139}=t_1^{3}t_2^{3}t_3t_4^{14}t_5^{2}t_6^{3}$, & ${\rm c}_{140}=t_1^{3}t_2^{3}t_3t_4^{14}t_5^{3}t_6^{2}$, \\ ${\rm c}_{141}=t_1^{3}t_2^{3}t_3^{3}t_4t_5^{2}t_6^{14}$, & ${\rm c}_{142}=t_1^{3}t_2^{3}t_3^{3}t_4t_5^{14}t_6^{2}$, & ${\rm c}_{143}=t_1t_2^{3}t_3^{2}t_4^{3}t_5^{3}t_6^{14}$, & ${\rm c}_{144}=t_1t_2^{3}t_3^{2}t_4^{3}t_5^{14}t_6^{3}$, \\ \end{tabular} \end{center} \newpage \begin{center} \begin{tabular}{llll} ${\rm c}_{145}=t_1t_2^{3}t_3^{3}t_4^{2}t_5^{3}t_6^{14}$, & ${\rm c}_{146}=t_1t_2^{3}t_3^{3}t_4^{2}t_5^{14}t_6^{3}$, & ${\rm c}_{147}=t_1t_2^{3}t_3^{3}t_4^{3}t_5^{2}t_6^{14}$, & ${\rm c}_{148}=t_1t_2^{3}t_3^{3}t_4^{3}t_5^{14}t_6^{2}$, \\ ${\rm c}_{149}=t_1t_2^{3}t_3^{3}t_4^{14}t_5^{2}t_6^{3}$, & ${\rm c}_{150}=t_1t_2^{3}t_3^{3}t_4^{14}t_5^{3}t_6^{2}$, & ${\rm c}_{151}=t_1t_2^{2}t_3^{3}t_4^{3}t_5^{3}t_6^{14}$, & ${\rm c}_{152}=t_1t_2^{2}t_3^{3}t_4^{3}t_5^{14}t_6^{3}$, \\ ${\rm c}_{153}=t_1t_2^{2}t_3^{3}t_4^{14}t_5^{3}t_6^{3}$, & ${\rm c}_{154}=t_1t_2^{3}t_3^{2}t_4^{14}t_5^{3}t_6^{3}$, & ${\rm c}_{155}=t_1^{3}t_2t_3^{2}t_4^{14}t_5^{3}t_6^{3}$, & ${\rm c}_{156}=t_1^{3}t_2^{3}t_3^{13}t_4^{2}t_5^{2}t_6^{3}$, \\ ${\rm c}_{157}=t_1^{3}t_2^{3}t_3^{13}t_4^{2}t_5^{3}t_6^{2}$, & ${\rm c}_{158}=t_1^{3}t_2^{3}t_3^{13}t_4^{3}t_5^{2}t_6^{2}$, & ${\rm c}_{159}=t_1^{3}t_2^{3}t_3^{3}t_4^{13}t_5^{2}t_6^{2}$, & ${\rm c}_{160}=t_1^{3}t_2^{13}t_3^{3}t_4^{2}t_5^{2}t_6^{3}$, \\ ${\rm c}_{161}=t_1^{3}t_2^{13}t_3^{3}t_4^{2}t_5^{3}t_6^{2}$, & ${\rm c}_{162}=t_1^{3}t_2^{13}t_3^{3}t_4^{3}t_5^{2}t_6^{2}$, & ${\rm c}_{163}=t_1^{3}t_2^{13}t_3^{2}t_4^{2}t_5^{3}t_6^{3}$, & ${\rm c}_{164}=t_1^{3}t_2^{13}t_3^{2}t_4^{3}t_5^{2}t_6^{3}$, \\ ${\rm c}_{165}=t_1^{3}t_2^{13}t_3^{2}t_4^{3}t_5^{3}t_6^{2}$, & ${\rm c}_{166}=t_1^{3}t_2t_3^{2}t_4^{3}t_5^{6}t_6^{11}$, & ${\rm c}_{167}=t_1^{3}t_2t_3^{3}t_4^{2}t_5^{6}t_6^{11}$, & ${\rm c}_{168}=t_1^{3}t_2t_3^{3}t_4^{6}t_5^{2}t_6^{11}$, \\ ${\rm c}_{169}=t_1^{3}t_2t_3^{3}t_4^{6}t_5^{11}t_6^{2}$, & ${\rm c}_{170}=t_1^{3}t_2^{3}t_3t_4^{2}t_5^{6}t_6^{11}$, & ${\rm c}_{171}=t_1^{3}t_2^{3}t_3t_4^{6}t_5^{2}t_6^{11}$, & ${\rm c}_{172}=t_1^{3}t_2^{3}t_3t_4^{6}t_5^{11}t_6^{2}$, \\ ${\rm c}_{173}=t_1t_2^{3}t_3^{2}t_4^{3}t_5^{6}t_6^{11}$, & ${\rm c}_{174}=t_1t_2^{3}t_3^{3}t_4^{2}t_5^{6}t_6^{11}$, & ${\rm c}_{175}=t_1t_2^{3}t_3^{3}t_4^{6}t_5^{2}t_6^{11}$, & ${\rm c}_{176}=t_1t_2^{3}t_3^{3}t_4^{6}t_5^{11}t_6^{2}$, \\ ${\rm c}_{177}=t_1t_2^{2}t_3^{3}t_4^{3}t_5^{6}t_6^{11}$, & ${\rm c}_{178}=t_1t_2^{2}t_3^{3}t_4^{6}t_5^{11}t_6^{3}$, & ${\rm c}_{179}=t_1t_2^{3}t_3^{2}t_4^{6}t_5^{11}t_6^{3}$, & ${\rm c}_{180}=t_1t_2^{3}t_3^{6}t_4^{2}t_5^{11}t_6^{3}$, \\ ${\rm c}_{181}=t_1t_2^{3}t_3^{6}t_4^{11}t_5^{2}t_6^{3}$, & ${\rm c}_{182}=t_1^{3}t_2t_3^{2}t_4^{6}t_5^{11}t_6^{3}$, & ${\rm c}_{183}=t_1^{3}t_2t_3^{6}t_4^{2}t_5^{11}t_6^{3}$, & ${\rm c}_{184}=t_1^{3}t_2t_3^{6}t_4^{11}t_5^{2}t_6^{3}$, \\ ${\rm c}_{185}=t_1t_2^{2}t_3^{3}t_4^{6}t_5^{3}t_6^{11}$, & ${\rm c}_{186}=t_1t_2^{3}t_3^{2}t_4^{6}t_5^{3}t_6^{11}$, & ${\rm c}_{187}=t_1t_2^{3}t_3^{6}t_4^{2}t_5^{3}t_6^{11}$, & ${\rm c}_{188}=t_1t_2^{3}t_3^{6}t_4^{11}t_5^{3}t_6^{2}$, \\ ${\rm c}_{189}=t_1^{3}t_2t_3^{2}t_4^{6}t_5^{3}t_6^{11}$, & ${\rm c}_{190}=t_1^{3}t_2t_3^{6}t_4^{2}t_5^{3}t_6^{11}$, & ${\rm c}_{191}=t_1^{3}t_2t_3^{6}t_4^{11}t_5^{3}t_6^{2}$, & ${\rm c}_{192}=t_1t_2^{3}t_3^{6}t_4^{3}t_5^{2}t_6^{11}$, \\ ${\rm c}_{193}=t_1t_2^{3}t_3^{6}t_4^{3}t_5^{11}t_6^{2}$, & ${\rm c}_{194}=t_1^{3}t_2t_3^{6}t_4^{3}t_5^{2}t_6^{11}$, & ${\rm c}_{195}=t_1^{3}t_2t_3^{6}t_4^{3}t_5^{11}t_6^{2}$, & ${\rm c}_{196}=t_1^{3}t_2^{5}t_3^{11}t_4^{2}t_5^{2}t_6^{3}$, \\ ${\rm c}_{197}=t_1^{3}t_2^{5}t_3^{11}t_4^{2}t_5^{3}t_6^{2}$, & ${\rm c}_{198}=t_1^{3}t_2^{5}t_3^{11}t_4^{3}t_5^{2}t_6^{2}$, & ${\rm c}_{199}=t_1^{3}t_2^{3}t_3^{5}t_4^{2}t_5^{2}t_6^{11}$, & ${\rm c}_{200}=t_1^{3}t_2^{3}t_3^{5}t_4^{2}t_5^{11}t_6^{2}$, \\ ${\rm c}_{201}=t_1^{3}t_2^{3}t_3^{5}t_4^{11}t_5^{2}t_6^{2}$, & ${\rm c}_{202}=t_1^{3}t_2^{5}t_3^{3}t_4^{2}t_5^{2}t_6^{11}$, & ${\rm c}_{203}=t_1^{3}t_2^{5}t_3^{3}t_4^{2}t_5^{11}t_6^{2}$, & ${\rm c}_{204}=t_1^{3}t_2^{5}t_3^{3}t_4^{11}t_5^{2}t_6^{2}$, \\ ${\rm c}_{205}=t_1^{3}t_2^{5}t_3^{2}t_4^{2}t_5^{11}t_6^{3}$, & ${\rm c}_{206}=t_1^{3}t_2^{5}t_3^{2}t_4^{11}t_5^{2}t_6^{3}$, & ${\rm c}_{207}=t_1^{3}t_2^{5}t_3^{2}t_4^{2}t_5^{3}t_6^{11}$, & ${\rm c}_{208}=t_1^{3}t_2^{5}t_3^{2}t_4^{11}t_5^{3}t_6^{2}$, \\ ${\rm c}_{209}=t_1^{3}t_2^{5}t_3^{2}t_4^{3}t_5^{2}t_6^{11}$, & ${\rm c}_{210}=t_1^{3}t_2^{5}t_3^{2}t_4^{3}t_5^{11}t_6^{2}$, & ${\rm c}_{211}=t_1t_2^{7}t_3^{10}t_4^{2}t_5^{3}t_6^{3}$, & ${\rm c}_{212}=t_1t_2^{7}t_3^{10}t_4^{3}t_5^{2}t_6^{3}$, \\ ${\rm c}_{213}=t_1^{7}t_2t_3^{10}t_4^{2}t_5^{3}t_6^{3}$, & ${\rm c}_{214}=t_1^{7}t_2t_3^{10}t_4^{3}t_5^{2}t_6^{3}$, & ${\rm c}_{215}=t_1t_2^{7}t_3^{10}t_4^{3}t_5^{3}t_6^{2}$, & ${\rm c}_{216}=t_1^{7}t_2t_3^{10}t_4^{3}t_5^{3}t_6^{2}$, \\ ${\rm c}_{217}=t_1^{3}t_2t_3^{2}t_4^{3}t_5^{7}t_6^{10}$, & ${\rm c}_{218}=t_1^{3}t_2t_3^{2}t_4^{7}t_5^{3}t_6^{10}$, & ${\rm c}_{219}=t_1^{3}t_2t_3^{2}t_4^{7}t_5^{10}t_6^{3}$, & ${\rm c}_{220}=t_1^{3}t_2t_3^{3}t_4^{2}t_5^{7}t_6^{10}$, \\ ${\rm c}_{221}=t_1^{3}t_2t_3^{3}t_4^{7}t_5^{2}t_6^{10}$, & ${\rm c}_{222}=t_1^{3}t_2t_3^{3}t_4^{7}t_5^{10}t_6^{2}$, & ${\rm c}_{223}=t_1^{3}t_2t_3^{7}t_4^{2}t_5^{3}t_6^{10}$, & ${\rm c}_{224}=t_1^{3}t_2t_3^{7}t_4^{2}t_5^{10}t_6^{3}$, \\ ${\rm c}_{225}=t_1^{3}t_2t_3^{7}t_4^{3}t_5^{2}t_6^{10}$, & ${\rm c}_{226}=t_1^{3}t_2t_3^{7}t_4^{3}t_5^{10}t_6^{2}$, & ${\rm c}_{227}=t_1^{3}t_2t_3^{7}t_4^{10}t_5^{2}t_6^{3}$, & ${\rm c}_{228}=t_1^{3}t_2t_3^{7}t_4^{10}t_5^{3}t_6^{2}$, \\ ${\rm c}_{229}=t_1^{3}t_2^{3}t_3t_4^{2}t_5^{7}t_6^{10}$, & ${\rm c}_{230}=t_1^{3}t_2^{3}t_3t_4^{7}t_5^{2}t_6^{10}$, & ${\rm c}_{231}=t_1^{3}t_2^{3}t_3t_4^{7}t_5^{10}t_6^{2}$, & ${\rm c}_{232}=t_1^{3}t_2^{3}t_3^{7}t_4t_5^{2}t_6^{10}$, \\ ${\rm c}_{233}=t_1^{3}t_2^{3}t_3^{7}t_4t_5^{10}t_6^{2}$, & ${\rm c}_{234}=t_1^{3}t_2^{7}t_3t_4^{2}t_5^{3}t_6^{10}$, & ${\rm c}_{235}=t_1^{3}t_2^{7}t_3t_4^{2}t_5^{10}t_6^{3}$, & ${\rm c}_{236}=t_1^{3}t_2^{7}t_3t_4^{3}t_5^{2}t_6^{10}$, \\ ${\rm c}_{237}=t_1^{3}t_2^{7}t_3t_4^{3}t_5^{10}t_6^{2}$, & ${\rm c}_{238}=t_1^{3}t_2^{7}t_3t_4^{10}t_5^{2}t_6^{3}$, & ${\rm c}_{239}=t_1^{3}t_2^{7}t_3t_4^{10}t_5^{3}t_6^{2}$, & ${\rm c}_{240}=t_1^{3}t_2^{7}t_3^{3}t_4t_5^{2}t_6^{10}$, \\ ${\rm c}_{241}=t_1^{3}t_2^{7}t_3^{3}t_4t_5^{10}t_6^{2}$, & ${\rm c}_{242}=t_1t_2^{3}t_3^{2}t_4^{3}t_5^{7}t_6^{10}$, & ${\rm c}_{243}=t_1t_2^{3}t_3^{2}t_4^{7}t_5^{3}t_6^{10}$, & ${\rm c}_{244}=t_1t_2^{3}t_3^{2}t_4^{7}t_5^{10}t_6^{3}$, \\ ${\rm c}_{245}=t_1t_2^{3}t_3^{3}t_4^{2}t_5^{7}t_6^{10}$, & ${\rm c}_{246}=t_1t_2^{3}t_3^{3}t_4^{7}t_5^{2}t_6^{10}$, & ${\rm c}_{247}=t_1t_2^{3}t_3^{3}t_4^{7}t_5^{10}t_6^{2}$, & ${\rm c}_{248}=t_1t_2^{3}t_3^{7}t_4^{2}t_5^{3}t_6^{10}$, \\ ${\rm c}_{249}=t_1t_2^{3}t_3^{7}t_4^{2}t_5^{10}t_6^{3}$, & ${\rm c}_{250}=t_1t_2^{3}t_3^{7}t_4^{3}t_5^{2}t_6^{10}$, & ${\rm c}_{251}=t_1t_2^{3}t_3^{7}t_4^{3}t_5^{10}t_6^{2}$, & ${\rm c}_{252}=t_1t_2^{3}t_3^{7}t_4^{10}t_5^{2}t_6^{3}$, \\ ${\rm c}_{253}=t_1t_2^{3}t_3^{7}t_4^{10}t_5^{3}t_6^{2}$, & ${\rm c}_{254}=t_1^{7}t_2^{3}t_3t_4^{2}t_5^{3}t_6^{10}$, & ${\rm c}_{255}=t_1^{7}t_2^{3}t_3t_4^{2}t_5^{10}t_6^{3}$, & ${\rm c}_{256}=t_1^{7}t_2^{3}t_3t_4^{3}t_5^{2}t_6^{10}$, \\ ${\rm c}_{257}=t_1^{7}t_2^{3}t_3t_4^{3}t_5^{10}t_6^{2}$, & ${\rm c}_{258}=t_1^{7}t_2^{3}t_3t_4^{10}t_5^{2}t_6^{3}$, & ${\rm c}_{259}=t_1^{7}t_2^{3}t_3t_4^{10}t_5^{3}t_6^{2}$, & ${\rm c}_{260}=t_1^{7}t_2^{3}t_3^{3}t_4t_5^{2}t_6^{10}$, \\ ${\rm c}_{261}=t_1^{7}t_2^{3}t_3^{3}t_4t_5^{10}t_6^{2}$, & ${\rm c}_{262}=t_1t_2^{2}t_3^{3}t_4^{3}t_5^{7}t_6^{10}$, & ${\rm c}_{263}=t_1t_2^{2}t_3^{3}t_4^{7}t_5^{3}t_6^{10}$, & ${\rm c}_{264}=t_1t_2^{2}t_3^{3}t_4^{7}t_5^{10}t_6^{3}$, \\ ${\rm c}_{265}=t_1t_2^{7}t_3^{3}t_4^{2}t_5^{3}t_6^{10}$, & ${\rm c}_{266}=t_1t_2^{7}t_3^{3}t_4^{2}t_5^{10}t_6^{3}$, & ${\rm c}_{267}=t_1t_2^{7}t_3^{3}t_4^{3}t_5^{2}t_6^{10}$, & ${\rm c}_{268}=t_1t_2^{7}t_3^{3}t_4^{3}t_5^{10}t_6^{2}$, \\ ${\rm c}_{269}=t_1t_2^{7}t_3^{3}t_4^{10}t_5^{2}t_6^{3}$, & ${\rm c}_{270}=t_1t_2^{7}t_3^{3}t_4^{10}t_5^{3}t_6^{2}$, & ${\rm c}_{271}=t_1^{7}t_2t_3^{3}t_4^{2}t_5^{3}t_6^{10}$, & ${\rm c}_{272}=t_1^{7}t_2t_3^{3}t_4^{2}t_5^{10}t_6^{3}$, \\ ${\rm c}_{273}=t_1^{7}t_2t_3^{3}t_4^{3}t_5^{2}t_6^{10}$, & ${\rm c}_{274}=t_1^{7}t_2t_3^{3}t_4^{3}t_5^{10}t_6^{2}$, & ${\rm c}_{275}=t_1^{7}t_2t_3^{3}t_4^{10}t_5^{2}t_6^{3}$, & ${\rm c}_{276}=t_1^{7}t_2t_3^{3}t_4^{10}t_5^{3}t_6^{2}$, \\ ${\rm c}_{277}=t_1t_2^{2}t_3^{7}t_4^{3}t_5^{10}t_6^{3}$, & ${\rm c}_{278}=t_1t_2^{2}t_3^{7}t_4^{10}t_5^{3}t_6^{3}$, & ${\rm c}_{279}=t_1t_2^{7}t_3^{2}t_4^{3}t_5^{10}t_6^{3}$, & ${\rm c}_{280}=t_1t_2^{7}t_3^{2}t_4^{10}t_5^{3}t_6^{3}$, \\ ${\rm c}_{281}=t_1^{7}t_2t_3^{2}t_4^{3}t_5^{10}t_6^{3}$, & ${\rm c}_{282}=t_1^{7}t_2t_3^{2}t_4^{10}t_5^{3}t_6^{3}$, & ${\rm c}_{283}=t_1t_2^{2}t_3^{7}t_4^{3}t_5^{3}t_6^{10}$, & ${\rm c}_{284}=t_1t_2^{7}t_3^{2}t_4^{3}t_5^{3}t_6^{10}$, \\ ${\rm c}_{285}=t_1^{7}t_2t_3^{2}t_4^{3}t_5^{3}t_6^{10}$, & ${\rm c}_{286}=t_1^{3}t_2^{7}t_3^{9}t_4^{2}t_5^{2}t_6^{3}$, & ${\rm c}_{287}=t_1^{3}t_2^{7}t_3^{9}t_4^{2}t_5^{3}t_6^{2}$, & ${\rm c}_{288}=t_1^{3}t_2^{7}t_3^{9}t_4^{3}t_5^{2}t_6^{2}$, \\ ${\rm c}_{289}=t_1^{7}t_2^{3}t_3^{9}t_4^{2}t_5^{2}t_6^{3}$, & ${\rm c}_{290}=t_1^{7}t_2^{3}t_3^{9}t_4^{2}t_5^{3}t_6^{2}$, & ${\rm c}_{291}=t_1^{7}t_2^{3}t_3^{9}t_4^{3}t_5^{2}t_6^{2}$, & ${\rm c}_{292}=t_1^{3}t_2^{3}t_3^{7}t_4^{9}t_5^{2}t_6^{2}$, \\ ${\rm c}_{293}=t_1^{3}t_2^{7}t_3^{3}t_4^{9}t_5^{2}t_6^{2}$, & ${\rm c}_{294}=t_1^{7}t_2^{3}t_3^{3}t_4^{9}t_5^{2}t_6^{2}$, & ${\rm c}_{295}=t_1^{7}t_2^{9}t_3^{3}t_4^{2}t_5^{2}t_6^{3}$, & ${\rm c}_{296}=t_1^{7}t_2^{9}t_3^{3}t_4^{2}t_5^{3}t_6^{2}$, \\ ${\rm c}_{297}=t_1^{7}t_2^{9}t_3^{3}t_4^{3}t_5^{2}t_6^{2}$, & ${\rm c}_{298}=t_1^{7}t_2^{9}t_3^{2}t_4^{2}t_5^{3}t_6^{3}$, & ${\rm c}_{299}=t_1^{7}t_2^{9}t_3^{2}t_4^{3}t_5^{2}t_6^{3}$, & ${\rm c}_{300}=t_1^{7}t_2^{9}t_3^{2}t_4^{3}t_5^{3}t_6^{2}$, \\ ${\rm c}_{301}=t_1^{3}t_2t_3^{3}t_4^{3}t_5^{6}t_6^{10}$, & ${\rm c}_{302}=t_1^{3}t_2t_3^{3}t_4^{6}t_5^{3}t_6^{10}$, & ${\rm c}_{303}=t_1^{3}t_2t_3^{3}t_4^{6}t_5^{10}t_6^{3}$, & ${\rm c}_{304}=t_1^{3}t_2^{3}t_3t_4^{3}t_5^{6}t_6^{10}$, \\ ${\rm c}_{305}=t_1^{3}t_2^{3}t_3t_4^{6}t_5^{3}t_6^{10}$, & ${\rm c}_{306}=t_1^{3}t_2^{3}t_3t_4^{6}t_5^{10}t_6^{3}$, & ${\rm c}_{307}=t_1^{3}t_2^{3}t_3^{3}t_4t_5^{6}t_6^{10}$, & ${\rm c}_{308}=t_1t_2^{3}t_3^{3}t_4^{3}t_5^{6}t_6^{10}$, \\ ${\rm c}_{309}=t_1t_2^{3}t_3^{3}t_4^{6}t_5^{3}t_6^{10}$, & ${\rm c}_{310}=t_1t_2^{3}t_3^{3}t_4^{6}t_5^{10}t_6^{3}$, & ${\rm c}_{311}=t_1t_2^{3}t_3^{6}t_4^{3}t_5^{10}t_6^{3}$, & ${\rm c}_{312}=t_1t_2^{3}t_3^{6}t_4^{10}t_5^{3}t_6^{3}$, \\ ${\rm c}_{313}=t_1^{3}t_2t_3^{6}t_4^{3}t_5^{10}t_6^{3}$, & ${\rm c}_{314}=t_1^{3}t_2t_3^{6}t_4^{10}t_5^{3}t_6^{3}$, & ${\rm c}_{315}=t_1t_2^{3}t_3^{6}t_4^{3}t_5^{3}t_6^{10}$, & ${\rm c}_{316}=t_1^{3}t_2t_3^{6}t_4^{3}t_5^{3}t_6^{10}$, \\ ${\rm c}_{317}=t_1^{3}t_2^{5}t_3^{10}t_4^{2}t_5^{3}t_6^{3}$, & ${\rm c}_{318}=t_1^{3}t_2^{5}t_3^{10}t_4^{3}t_5^{2}t_6^{3}$, & ${\rm c}_{319}=t_1^{3}t_2^{5}t_3^{10}t_4^{3}t_5^{3}t_6^{2}$, & ${\rm c}_{320}=t_1^{3}t_2^{3}t_3^{3}t_4^{5}t_5^{2}t_6^{10}$, \\ ${\rm c}_{321}=t_1^{3}t_2^{3}t_3^{3}t_4^{5}t_5^{10}t_6^{2}$, & ${\rm c}_{322}=t_1^{3}t_2^{3}t_3^{5}t_4^{2}t_5^{3}t_6^{10}$, & ${\rm c}_{323}=t_1^{3}t_2^{3}t_3^{5}t_4^{2}t_5^{10}t_6^{3}$, & ${\rm c}_{324}=t_1^{3}t_2^{3}t_3^{5}t_4^{3}t_5^{2}t_6^{10}$, \\ ${\rm c}_{325}=t_1^{3}t_2^{3}t_3^{5}t_4^{3}t_5^{10}t_6^{2}$, & ${\rm c}_{326}=t_1^{3}t_2^{3}t_3^{5}t_4^{10}t_5^{2}t_6^{3}$, & ${\rm c}_{327}=t_1^{3}t_2^{3}t_3^{5}t_4^{10}t_5^{3}t_6^{2}$, & ${\rm c}_{328}=t_1^{3}t_2^{5}t_3^{3}t_4^{2}t_5^{3}t_6^{10}$, \\ ${\rm c}_{329}=t_1^{3}t_2^{5}t_3^{3}t_4^{2}t_5^{10}t_6^{3}$, & ${\rm c}_{330}=t_1^{3}t_2^{5}t_3^{3}t_4^{3}t_5^{2}t_6^{10}$, & ${\rm c}_{331}=t_1^{3}t_2^{5}t_3^{3}t_4^{3}t_5^{10}t_6^{2}$, & ${\rm c}_{332}=t_1^{3}t_2^{5}t_3^{3}t_4^{10}t_5^{2}t_6^{3}$, \\ ${\rm c}_{333}=t_1^{3}t_2^{5}t_3^{3}t_4^{10}t_5^{3}t_6^{2}$, & ${\rm c}_{334}=t_1^{3}t_2^{5}t_3^{2}t_4^{3}t_5^{10}t_6^{3}$, & ${\rm c}_{335}=t_1^{3}t_2^{5}t_3^{2}t_4^{10}t_5^{3}t_6^{3}$, & ${\rm c}_{336}=t_1^{3}t_2^{5}t_3^{2}t_4^{3}t_5^{3}t_6^{10}$. \end{tabular}\end{center} \newpage \subsection{Admissible monomials in \mbox{\boldmath $(P^{\otimes 6}_{n_1})^{> 0}(4,5,3)$}}\label{s52} By the proof of Theorem \ref{dlc1}, $\dim (QP^{\otimes 6}_{n_1})^{>0}(4,5,3) = |D_2| = 210,$ where $D_2 = \big\{d_j:\ 1\leq j\leq 210\big\}$ and the admissible monomials $d_j$ are determined as follows: \begin{center} \begin{tabular}{llrr} ${\rm d}_{1}=t_1t_2^{2}t_3^{2}t_4^{7}t_5^{7}t_6^{7}$, & ${\rm d}_{2}=t_1t_2^{2}t_3^{7}t_4^{2}t_5^{7}t_6^{7}$, & \multicolumn{1}{l}{${\rm d}_{3}=t_1t_2^{2}t_3^{7}t_4^{7}t_5^{2}t_6^{7}$,} & \multicolumn{1}{l}{${\rm d}_{4}=t_1t_2^{2}t_3^{7}t_4^{7}t_5^{7}t_6^{2}$,} \\ ${\rm d}_{5}=t_1t_2^{7}t_3^{2}t_4^{2}t_5^{7}t_6^{7}$, & ${\rm d}_{6}=t_1t_2^{7}t_3^{2}t_4^{7}t_5^{2}t_6^{7}$, & \multicolumn{1}{l}{${\rm d}_{7}=t_1t_2^{7}t_3^{2}t_4^{7}t_5^{7}t_6^{2}$,} & \multicolumn{1}{l}{${\rm d}_{8}=t_1t_2^{7}t_3^{7}t_4^{2}t_5^{2}t_6^{7}$,} \\ ${\rm d}_{9}=t_1t_2^{7}t_3^{7}t_4^{2}t_5^{7}t_6^{2}$, & ${\rm d}_{10}=t_1t_2^{7}t_3^{7}t_4^{7}t_5^{2}t_6^{2}$, & \multicolumn{1}{l}{${\rm d}_{11}=t_1^{7}t_2t_3^{2}t_4^{2}t_5^{7}t_6^{7}$,} & \multicolumn{1}{l}{${\rm d}_{12}=t_1^{7}t_2t_3^{2}t_4^{7}t_5^{2}t_6^{7}$,} \\ ${\rm d}_{13}=t_1^{7}t_2t_3^{2}t_4^{7}t_5^{7}t_6^{2}$, & ${\rm d}_{14}=t_1^{7}t_2t_3^{7}t_4^{2}t_5^{2}t_6^{7}$, & \multicolumn{1}{l}{${\rm d}_{15}=t_1^{7}t_2t_3^{7}t_4^{2}t_5^{7}t_6^{2}$,} & \multicolumn{1}{l}{${\rm d}_{16}=t_1^{7}t_2t_3^{7}t_4^{7}t_5^{2}t_6^{2}$,} \\ ${\rm d}_{17}=t_1^{7}t_2^{7}t_3t_4^{2}t_5^{2}t_6^{7}$, & ${\rm d}_{18}=t_1^{7}t_2^{7}t_3t_4^{2}t_5^{7}t_6^{2}$, & \multicolumn{1}{l}{${\rm d}_{19}=t_1^{7}t_2^{7}t_3t_4^{7}t_5^{2}t_6^{2}$,} & \multicolumn{1}{l}{${\rm d}_{20}=t_1^{7}t_2^{7}t_3^{7}t_4t_5^{2}t_6^{2}$,} \\ ${\rm d}_{21}=t_1t_2^{2}t_3^{3}t_4^{6}t_5^{7}t_6^{7}$, & ${\rm d}_{22}=t_1t_2^{2}t_3^{3}t_4^{7}t_5^{6}t_6^{7}$, & \multicolumn{1}{l}{${\rm d}_{23}=t_1t_2^{2}t_3^{3}t_4^{7}t_5^{7}t_6^{6}$,} & \multicolumn{1}{l}{${\rm d}_{24}=t_1t_2^{2}t_3^{7}t_4^{3}t_5^{6}t_6^{7}$,} \\ ${\rm d}_{25}=t_1t_2^{2}t_3^{7}t_4^{3}t_5^{7}t_6^{6}$, & ${\rm d}_{26}=t_1t_2^{2}t_3^{7}t_4^{7}t_5^{3}t_6^{6}$, & \multicolumn{1}{l}{${\rm d}_{27}=t_1t_2^{3}t_3^{2}t_4^{6}t_5^{7}t_6^{7}$,} & \multicolumn{1}{l}{${\rm d}_{28}=t_1t_2^{3}t_3^{2}t_4^{7}t_5^{6}t_6^{7}$,} \\ ${\rm d}_{29}=t_1t_2^{3}t_3^{2}t_4^{7}t_5^{7}t_6^{6}$, & ${\rm d}_{30}=t_1t_2^{3}t_3^{6}t_4^{2}t_5^{7}t_6^{7}$, & \multicolumn{1}{l}{${\rm d}_{31}=t_1t_2^{3}t_3^{6}t_4^{7}t_5^{2}t_6^{7}$,} & \multicolumn{1}{l}{${\rm d}_{32}=t_1t_2^{3}t_3^{6}t_4^{7}t_5^{7}t_6^{2}$,} \\ ${\rm d}_{33}=t_1t_2^{3}t_3^{7}t_4^{2}t_5^{6}t_6^{7}$, & ${\rm d}_{34}=t_1t_2^{3}t_3^{7}t_4^{2}t_5^{7}t_6^{6}$, & \multicolumn{1}{l}{${\rm d}_{35}=t_1t_2^{3}t_3^{7}t_4^{6}t_5^{2}t_6^{7}$,} & \multicolumn{1}{l}{${\rm d}_{36}=t_1t_2^{3}t_3^{7}t_4^{6}t_5^{7}t_6^{2}$,} \\ ${\rm d}_{37}=t_1t_2^{3}t_3^{7}t_4^{7}t_5^{2}t_6^{6}$, & ${\rm d}_{38}=t_1t_2^{3}t_3^{7}t_4^{7}t_5^{6}t_6^{2}$, & \multicolumn{1}{l}{${\rm d}_{39}=t_1t_2^{7}t_3^{2}t_4^{3}t_5^{6}t_6^{7}$,} & \multicolumn{1}{l}{${\rm d}_{40}=t_1t_2^{7}t_3^{2}t_4^{3}t_5^{7}t_6^{6}$,} \\ ${\rm d}_{41}=t_1t_2^{7}t_3^{2}t_4^{7}t_5^{3}t_6^{6}$, & ${\rm d}_{42}=t_1t_2^{7}t_3^{3}t_4^{2}t_5^{6}t_6^{7}$, & \multicolumn{1}{l}{${\rm d}_{43}=t_1t_2^{7}t_3^{3}t_4^{2}t_5^{7}t_6^{6}$,} & \multicolumn{1}{l}{${\rm d}_{44}=t_1t_2^{7}t_3^{3}t_4^{6}t_5^{2}t_6^{7}$,} \\ ${\rm d}_{45}=t_1t_2^{7}t_3^{3}t_4^{6}t_5^{7}t_6^{2}$, & ${\rm d}_{46}=t_1t_2^{7}t_3^{3}t_4^{7}t_5^{2}t_6^{6}$, & \multicolumn{1}{l}{${\rm d}_{47}=t_1t_2^{7}t_3^{3}t_4^{7}t_5^{6}t_6^{2}$,} & \multicolumn{1}{l}{${\rm d}_{48}=t_1t_2^{7}t_3^{7}t_4^{2}t_5^{3}t_6^{6}$,} \\ ${\rm d}_{49}=t_1t_2^{7}t_3^{7}t_4^{3}t_5^{2}t_6^{6}$, & ${\rm d}_{50}=t_1t_2^{7}t_3^{7}t_4^{3}t_5^{6}t_6^{2}$, & \multicolumn{1}{l}{${\rm d}_{51}=t_1^{3}t_2t_3^{2}t_4^{6}t_5^{7}t_6^{7}$,} & \multicolumn{1}{l}{${\rm d}_{52}=t_1^{3}t_2t_3^{2}t_4^{7}t_5^{6}t_6^{7}$,} \\ ${\rm d}_{53}=t_1^{3}t_2t_3^{2}t_4^{7}t_5^{7}t_6^{6}$, & ${\rm d}_{54}=t_1^{3}t_2t_3^{6}t_4^{2}t_5^{7}t_6^{7}$, & \multicolumn{1}{l}{${\rm d}_{55}=t_1^{3}t_2t_3^{6}t_4^{7}t_5^{2}t_6^{7}$,} & \multicolumn{1}{l}{${\rm d}_{56}=t_1^{3}t_2t_3^{6}t_4^{7}t_5^{7}t_6^{2}$,} \\ ${\rm d}_{57}=t_1^{3}t_2t_3^{7}t_4^{2}t_5^{6}t_6^{7}$, & ${\rm d}_{58}=t_1^{3}t_2t_3^{7}t_4^{2}t_5^{7}t_6^{6}$, & \multicolumn{1}{l}{${\rm d}_{59}=t_1^{3}t_2t_3^{7}t_4^{6}t_5^{2}t_6^{7}$,} & \multicolumn{1}{l}{${\rm d}_{60}=t_1^{3}t_2t_3^{7}t_4^{6}t_5^{7}t_6^{2}$,} \\ ${\rm d}_{61}=t_1^{3}t_2t_3^{7}t_4^{7}t_5^{2}t_6^{6}$, & ${\rm d}_{62}=t_1^{3}t_2t_3^{7}t_4^{7}t_5^{6}t_6^{2}$, & \multicolumn{1}{l}{${\rm d}_{63}=t_1^{3}t_2^{7}t_3t_4^{2}t_5^{6}t_6^{7}$,} & \multicolumn{1}{l}{${\rm d}_{64}=t_1^{3}t_2^{7}t_3t_4^{2}t_5^{7}t_6^{6}$,} \\ ${\rm d}_{65}=t_1^{3}t_2^{7}t_3t_4^{6}t_5^{2}t_6^{7}$, & ${\rm d}_{66}=t_1^{3}t_2^{7}t_3t_4^{6}t_5^{7}t_6^{2}$, & \multicolumn{1}{l}{${\rm d}_{67}=t_1^{3}t_2^{7}t_3t_4^{7}t_5^{2}t_6^{6}$,} & \multicolumn{1}{l}{${\rm d}_{68}=t_1^{3}t_2^{7}t_3t_4^{7}t_5^{6}t_6^{2}$,} \\ ${\rm d}_{69}=t_1^{3}t_2^{7}t_3^{7}t_4t_5^{2}t_6^{6}$, & ${\rm d}_{70}=t_1^{3}t_2^{7}t_3^{7}t_4t_5^{6}t_6^{2}$, & \multicolumn{1}{l}{${\rm d}_{71}=t_1^{7}t_2t_3^{2}t_4^{3}t_5^{6}t_6^{7}$,} & \multicolumn{1}{l}{${\rm d}_{72}=t_1^{7}t_2t_3^{2}t_4^{3}t_5^{7}t_6^{6}$,} \\ ${\rm d}_{73}=t_1^{7}t_2t_3^{2}t_4^{7}t_5^{3}t_6^{6}$, & ${\rm d}_{74}=t_1^{7}t_2t_3^{3}t_4^{2}t_5^{6}t_6^{7}$, & \multicolumn{1}{l}{${\rm d}_{75}=t_1^{7}t_2t_3^{3}t_4^{2}t_5^{7}t_6^{6}$,} & \multicolumn{1}{l}{${\rm d}_{76}=t_1^{7}t_2t_3^{3}t_4^{6}t_5^{2}t_6^{7}$,} \\ ${\rm d}_{77}=t_1^{7}t_2t_3^{3}t_4^{6}t_5^{7}t_6^{2}$, & ${\rm d}_{78}=t_1^{7}t_2t_3^{3}t_4^{7}t_5^{2}t_6^{6}$, & \multicolumn{1}{l}{${\rm d}_{79}=t_1^{7}t_2t_3^{3}t_4^{7}t_5^{6}t_6^{2}$,} & \multicolumn{1}{l}{${\rm d}_{80}=t_1^{7}t_2t_3^{7}t_4^{2}t_5^{3}t_6^{6}$,} \\ ${\rm d}_{81}=t_1^{7}t_2t_3^{7}t_4^{3}t_5^{2}t_6^{6}$, & ${\rm d}_{82}=t_1^{7}t_2t_3^{7}t_4^{3}t_5^{6}t_6^{2}$, & \multicolumn{1}{l}{${\rm d}_{83}=t_1^{7}t_2^{3}t_3t_4^{2}t_5^{6}t_6^{7}$,} & \multicolumn{1}{l}{${\rm d}_{84}=t_1^{7}t_2^{3}t_3t_4^{2}t_5^{7}t_6^{6}$,} \\ ${\rm d}_{85}=t_1^{7}t_2^{3}t_3t_4^{6}t_5^{2}t_6^{7}$, & ${\rm d}_{86}=t_1^{7}t_2^{3}t_3t_4^{6}t_5^{7}t_6^{2}$, & \multicolumn{1}{l}{${\rm d}_{87}=t_1^{7}t_2^{3}t_3t_4^{7}t_5^{2}t_6^{6}$,} & \multicolumn{1}{l}{${\rm d}_{88}=t_1^{7}t_2^{3}t_3t_4^{7}t_5^{6}t_6^{2}$,} \\ ${\rm d}_{89}=t_1^{7}t_2^{3}t_3^{7}t_4t_5^{2}t_6^{6}$, & ${\rm d}_{90}=t_1^{7}t_2^{3}t_3^{7}t_4t_5^{6}t_6^{2}$, & \multicolumn{1}{l}{${\rm d}_{91}=t_1^{7}t_2^{7}t_3t_4^{2}t_5^{3}t_6^{6}$,} & \multicolumn{1}{l}{${\rm d}_{92}=t_1^{7}t_2^{7}t_3t_4^{3}t_5^{2}t_6^{6}$,} \\ ${\rm d}_{93}=t_1^{7}t_2^{7}t_3t_4^{3}t_5^{6}t_6^{2}$, & ${\rm d}_{94}=t_1^{7}t_2^{7}t_3^{3}t_4t_5^{2}t_6^{6}$, & \multicolumn{1}{l}{${\rm d}_{95}=t_1^{7}t_2^{7}t_3^{3}t_4t_5^{6}t_6^{2}$,} & \multicolumn{1}{l}{${\rm d}_{96}=t_1^{3}t_2^{5}t_3^{2}t_4^{2}t_5^{7}t_6^{7}$,} \\ ${\rm d}_{97}=t_1^{3}t_2^{5}t_3^{2}t_4^{7}t_5^{2}t_6^{7}$, & ${\rm d}_{98}=t_1^{3}t_2^{5}t_3^{2}t_4^{7}t_5^{7}t_6^{2}$, & \multicolumn{1}{l}{${\rm d}_{99}=t_1^{3}t_2^{5}t_3^{7}t_4^{2}t_5^{2}t_6^{7}$,} & \multicolumn{1}{l}{${\rm d}_{100}=t_1^{3}t_2^{5}t_3^{7}t_4^{2}t_5^{7}t_6^{2}$,} \\ ${\rm d}_{101}=t_1^{3}t_2^{5}t_3^{7}t_4^{7}t_5^{2}t_6^{2}$, & ${\rm d}_{102}=t_1^{3}t_2^{7}t_3^{5}t_4^{2}t_5^{2}t_6^{7}$, & \multicolumn{1}{l}{${\rm d}_{103}=t_1^{3}t_2^{7}t_3^{5}t_4^{2}t_5^{7}t_6^{2}$,} & \multicolumn{1}{l}{${\rm d}_{104}=t_1^{3}t_2^{7}t_3^{5}t_4^{7}t_5^{2}t_6^{2}$,} \\ ${\rm d}_{105}=t_1^{3}t_2^{7}t_3^{7}t_4^{5}t_5^{2}t_6^{2}$, & ${\rm d}_{106}=t_1^{7}t_2^{3}t_3^{5}t_4^{2}t_5^{2}t_6^{7}$, & \multicolumn{1}{l}{${\rm d}_{107}=t_1^{7}t_2^{3}t_3^{5}t_4^{2}t_5^{7}t_6^{2}$,} & \multicolumn{1}{l}{${\rm d}_{108}=t_1^{7}t_2^{3}t_3^{5}t_4^{7}t_5^{2}t_6^{2}$,} \\ ${\rm d}_{109}=t_1^{7}t_2^{3}t_3^{7}t_4^{5}t_5^{2}t_6^{2}$, & ${\rm d}_{110}=t_1^{7}t_2^{7}t_3^{3}t_4^{5}t_5^{2}t_6^{2}$, & \multicolumn{1}{l}{${\rm d}_{111}=t_1t_2^{3}t_3^{7}t_4^{3}t_5^{6}t_6^{6}$,} & \multicolumn{1}{l}{${\rm d}_{112}=t_1t_2^{3}t_3^{7}t_4^{6}t_5^{3}t_6^{6}$,} \\ ${\rm d}_{113}=t_1t_2^{3}t_3^{7}t_4^{6}t_5^{6}t_6^{3}$, & ${\rm d}_{114}=t_1^{3}t_2t_3^{7}t_4^{3}t_5^{6}t_6^{6}$, & \multicolumn{1}{l}{${\rm d}_{115}=t_1^{3}t_2t_3^{7}t_4^{6}t_5^{3}t_6^{6}$,} & \multicolumn{1}{l}{${\rm d}_{116}=t_1^{3}t_2t_3^{7}t_4^{6}t_5^{6}t_6^{3}$,} \\ ${\rm d}_{117}=t_1^{3}t_2^{3}t_3^{7}t_4t_5^{6}t_6^{6}$, & ${\rm d}_{118}=t_1t_2^{3}t_3^{3}t_4^{6}t_5^{6}t_6^{7}$, & \multicolumn{1}{l}{${\rm d}_{119}=t_1t_2^{3}t_3^{3}t_4^{6}t_5^{7}t_6^{6}$,} & \multicolumn{1}{l}{${\rm d}_{120}=t_1t_2^{3}t_3^{3}t_4^{7}t_5^{6}t_6^{6}$,} \\ ${\rm d}_{121}=t_1t_2^{3}t_3^{6}t_4^{3}t_5^{6}t_6^{7}$, & ${\rm d}_{122}=t_1t_2^{3}t_3^{6}t_4^{3}t_5^{7}t_6^{6}$, & \multicolumn{1}{l}{${\rm d}_{123}=t_1t_2^{3}t_3^{6}t_4^{6}t_5^{3}t_6^{7}$,} & \multicolumn{1}{l}{${\rm d}_{124}=t_1t_2^{3}t_3^{6}t_4^{6}t_5^{7}t_6^{3}$,} \\ ${\rm d}_{125}=t_1t_2^{3}t_3^{6}t_4^{7}t_5^{3}t_6^{6}$, & ${\rm d}_{126}=t_1t_2^{3}t_3^{6}t_4^{7}t_5^{6}t_6^{3}$, & \multicolumn{1}{l}{${\rm d}_{127}=t_1t_2^{7}t_3^{3}t_4^{3}t_5^{6}t_6^{6}$,} & \multicolumn{1}{l}{${\rm d}_{128}=t_1t_2^{7}t_3^{3}t_4^{6}t_5^{3}t_6^{6}$,} \\ ${\rm d}_{129}=t_1t_2^{7}t_3^{3}t_4^{6}t_5^{6}t_6^{3}$, & ${\rm d}_{130}=t_1^{3}t_2t_3^{3}t_4^{6}t_5^{6}t_6^{7}$, & \multicolumn{1}{l}{${\rm d}_{131}=t_1^{3}t_2t_3^{3}t_4^{6}t_5^{7}t_6^{6}$,} & \multicolumn{1}{l}{${\rm d}_{132}=t_1^{3}t_2t_3^{3}t_4^{7}t_5^{6}t_6^{6}$,} \\ ${\rm d}_{133}=t_1^{3}t_2t_3^{6}t_4^{3}t_5^{6}t_6^{7}$, & ${\rm d}_{134}=t_1^{3}t_2t_3^{6}t_4^{3}t_5^{7}t_6^{6}$, & \multicolumn{1}{l}{${\rm d}_{135}=t_1^{3}t_2t_3^{6}t_4^{6}t_5^{3}t_6^{7}$,} & \multicolumn{1}{l}{${\rm d}_{136}=t_1^{3}t_2t_3^{6}t_4^{6}t_5^{7}t_6^{3}$,} \\ ${\rm d}_{137}=t_1^{3}t_2t_3^{6}t_4^{7}t_5^{3}t_6^{6}$, & ${\rm d}_{138}=t_1^{3}t_2t_3^{6}t_4^{7}t_5^{6}t_6^{3}$, & \multicolumn{1}{l}{${\rm d}_{139}=t_1^{3}t_2^{3}t_3t_4^{6}t_5^{6}t_6^{7}$,} & \multicolumn{1}{l}{${\rm d}_{140}=t_1^{3}t_2^{3}t_3t_4^{6}t_5^{7}t_6^{6}$,} \\ ${\rm d}_{141}=t_1^{3}t_2^{3}t_3t_4^{7}t_5^{6}t_6^{6}$, & ${\rm d}_{142}=t_1^{3}t_2^{7}t_3t_4^{3}t_5^{6}t_6^{6}$, & \multicolumn{1}{l}{${\rm d}_{143}=t_1^{3}t_2^{7}t_3t_4^{6}t_5^{3}t_6^{6}$,} & \multicolumn{1}{l}{${\rm d}_{144}=t_1^{3}t_2^{7}t_3t_4^{6}t_5^{6}t_6^{3}$,} \\ ${\rm d}_{145}=t_1^{3}t_2^{7}t_3^{3}t_4t_5^{6}t_6^{6}$, & ${\rm d}_{146}=t_1^{7}t_2t_3^{3}t_4^{3}t_5^{6}t_6^{6}$, & \multicolumn{1}{l}{${\rm d}_{147}=t_1^{7}t_2t_3^{3}t_4^{6}t_5^{3}t_6^{6}$,} & \multicolumn{1}{l}{${\rm d}_{148}=t_1^{7}t_2t_3^{3}t_4^{6}t_5^{6}t_6^{3}$,} \\ ${\rm d}_{149}=t_1^{7}t_2^{3}t_3t_4^{3}t_5^{6}t_6^{6}$, & ${\rm d}_{150}=t_1^{7}t_2^{3}t_3t_4^{6}t_5^{3}t_6^{6}$, & \multicolumn{1}{l}{${\rm d}_{151}=t_1^{7}t_2^{3}t_3t_4^{6}t_5^{6}t_6^{3}$,} & \multicolumn{1}{l}{${\rm d}_{152}=t_1^{7}t_2^{3}t_3^{3}t_4t_5^{6}t_6^{6}$,} \\ ${\rm d}_{153}=t_1^{3}t_2^{3}t_3^{7}t_4^{5}t_5^{2}t_6^{6}$, & ${\rm d}_{154}=t_1^{3}t_2^{3}t_3^{7}t_4^{5}t_5^{6}t_6^{2}$, & \multicolumn{1}{l}{${\rm d}_{155}=t_1^{3}t_2^{5}t_3^{7}t_4^{2}t_5^{3}t_6^{6}$,} & \multicolumn{1}{l}{${\rm d}_{156}=t_1^{3}t_2^{5}t_3^{7}t_4^{2}t_5^{6}t_6^{3}$,} \\ ${\rm d}_{157}=t_1^{3}t_2^{5}t_3^{7}t_4^{3}t_5^{2}t_6^{6}$, & ${\rm d}_{158}=t_1^{3}t_2^{5}t_3^{7}t_4^{3}t_5^{6}t_6^{2}$, & \multicolumn{1}{l}{${\rm d}_{159}=t_1^{3}t_2^{5}t_3^{7}t_4^{6}t_5^{2}t_6^{3}$,} & \multicolumn{1}{l}{${\rm d}_{160}=t_1^{3}t_2^{5}t_3^{7}t_4^{6}t_5^{3}t_6^{2}$,} \\ ${\rm d}_{161}=t_1^{3}t_2^{3}t_3^{5}t_4^{2}t_5^{6}t_6^{7}$, & ${\rm d}_{162}=t_1^{3}t_2^{3}t_3^{5}t_4^{2}t_5^{7}t_6^{6}$, & \multicolumn{1}{l}{${\rm d}_{163}=t_1^{3}t_2^{3}t_3^{5}t_4^{6}t_5^{2}t_6^{7}$,} & \multicolumn{1}{l}{${\rm d}_{164}=t_1^{3}t_2^{3}t_3^{5}t_4^{6}t_5^{7}t_6^{2}$,} \\ ${\rm d}_{165}=t_1^{3}t_2^{3}t_3^{5}t_4^{7}t_5^{2}t_6^{6}$, & ${\rm d}_{166}=t_1^{3}t_2^{3}t_3^{5}t_4^{7}t_5^{6}t_6^{2}$, & \multicolumn{1}{l}{${\rm d}_{167}=t_1^{3}t_2^{5}t_3^{2}t_4^{3}t_5^{6}t_6^{7}$,} & \multicolumn{1}{l}{${\rm d}_{168}=t_1^{3}t_2^{5}t_3^{2}t_4^{3}t_5^{7}t_6^{6}$,} \\ ${\rm d}_{169}=t_1^{3}t_2^{5}t_3^{2}t_4^{6}t_5^{3}t_6^{7}$, & ${\rm d}_{170}=t_1^{3}t_2^{5}t_3^{2}t_4^{6}t_5^{7}t_6^{3}$, & \multicolumn{1}{l}{${\rm d}_{171}=t_1^{3}t_2^{5}t_3^{2}t_4^{7}t_5^{3}t_6^{6}$,} & \multicolumn{1}{l}{${\rm d}_{172}=t_1^{3}t_2^{5}t_3^{2}t_4^{7}t_5^{6}t_6^{3}$,} \\ ${\rm d}_{173}=t_1^{3}t_2^{5}t_3^{3}t_4^{2}t_5^{6}t_6^{7}$, & ${\rm d}_{174}=t_1^{3}t_2^{5}t_3^{3}t_4^{2}t_5^{7}t_6^{6}$, & \multicolumn{1}{l}{${\rm d}_{175}=t_1^{3}t_2^{5}t_3^{3}t_4^{6}t_5^{2}t_6^{7}$,} & \multicolumn{1}{l}{${\rm d}_{176}=t_1^{3}t_2^{5}t_3^{3}t_4^{6}t_5^{7}t_6^{2}$,} \\ ${\rm d}_{177}=t_1^{3}t_2^{5}t_3^{3}t_4^{7}t_5^{2}t_6^{6}$, & ${\rm d}_{178}=t_1^{3}t_2^{5}t_3^{3}t_4^{7}t_5^{6}t_6^{2}$, & \multicolumn{1}{l}{${\rm d}_{179}=t_1^{3}t_2^{5}t_3^{6}t_4^{2}t_5^{3}t_6^{7}$,} & \multicolumn{1}{l}{${\rm d}_{180}=t_1^{3}t_2^{5}t_3^{6}t_4^{2}t_5^{7}t_6^{3}$,} \\ ${\rm d}_{181}=t_1^{3}t_2^{5}t_3^{6}t_4^{3}t_5^{2}t_6^{7}$, & ${\rm d}_{182}=t_1^{3}t_2^{5}t_3^{6}t_4^{3}t_5^{7}t_6^{2}$, & \multicolumn{1}{l}{${\rm d}_{183}=t_1^{3}t_2^{5}t_3^{6}t_4^{7}t_5^{2}t_6^{3}$,} & \multicolumn{1}{l}{${\rm d}_{184}=t_1^{3}t_2^{5}t_3^{6}t_4^{7}t_5^{3}t_6^{2}$,} \\ ${\rm d}_{185}=t_1^{3}t_2^{7}t_3^{3}t_4^{5}t_5^{2}t_6^{6}$, & ${\rm d}_{186}=t_1^{3}t_2^{7}t_3^{3}t_4^{5}t_5^{6}t_6^{2}$, & \multicolumn{1}{l}{${\rm d}_{187}=t_1^{3}t_2^{7}t_3^{5}t_4^{2}t_5^{3}t_6^{6}$,} & \multicolumn{1}{l}{${\rm d}_{188}=t_1^{3}t_2^{7}t_3^{5}t_4^{2}t_5^{6}t_6^{3}$,} \\ ${\rm d}_{189}=t_1^{3}t_2^{7}t_3^{5}t_4^{3}t_5^{2}t_6^{6}$, & ${\rm d}_{190}=t_1^{3}t_2^{7}t_3^{5}t_4^{3}t_5^{6}t_6^{2}$, & \multicolumn{1}{l}{${\rm d}_{191}=t_1^{3}t_2^{7}t_3^{5}t_4^{6}t_5^{2}t_6^{3}$,} & \multicolumn{1}{l}{${\rm d}_{192}=t_1^{3}t_2^{7}t_3^{5}t_4^{6}t_5^{3}t_6^{2}$,} \\ \end{tabular} \end{center} \newpage \begin{center} \begin{tabular}{llrr} ${\rm d}_{193}=t_1^{7}t_2^{3}t_3^{3}t_4^{5}t_5^{2}t_6^{6}$, & ${\rm d}_{194}=t_1^{7}t_2^{3}t_3^{3}t_4^{5}t_5^{6}t_6^{2}$, & \multicolumn{1}{l}{${\rm d}_{195}=t_1^{7}t_2^{3}t_3^{5}t_4^{2}t_5^{3}t_6^{6}$,} & \multicolumn{1}{l}{${\rm d}_{196}=t_1^{7}t_2^{3}t_3^{5}t_4^{2}t_5^{6}t_6^{3}$,} \\ ${\rm d}_{197}=t_1^{7}t_2^{3}t_3^{5}t_4^{3}t_5^{2}t_6^{6}$, & ${\rm d}_{198}=t_1^{7}t_2^{3}t_3^{5}t_4^{3}t_5^{6}t_6^{2}$, & \multicolumn{1}{l}{${\rm d}_{199}=t_1^{7}t_2^{3}t_3^{5}t_4^{6}t_5^{2}t_6^{3}$,} & \multicolumn{1}{l}{${\rm d}_{200}=t_1^{7}t_2^{3}t_3^{5}t_4^{6}t_5^{3}t_6^{2}$,} \\ ${\rm d}_{201}=t_1^{3}t_2^{3}t_3^{3}t_4^{5}t_5^{6}t_6^{6}$, & ${\rm d}_{202}=t_1^{3}t_2^{3}t_3^{5}t_4^{3}t_5^{6}t_6^{6}$, & \multicolumn{1}{l}{${\rm d}_{203}=t_1^{3}t_2^{3}t_3^{5}t_4^{6}t_5^{3}t_6^{6}$,} & \multicolumn{1}{l}{${\rm d}_{204}=t_1^{3}t_2^{3}t_3^{5}t_4^{6}t_5^{6}t_6^{3}$,} \\ ${\rm d}_{205}=t_1^{3}t_2^{5}t_3^{3}t_4^{3}t_5^{6}t_6^{6}$, & ${\rm d}_{206}=t_1^{3}t_2^{5}t_3^{3}t_4^{6}t_5^{3}t_6^{6}$, & \multicolumn{1}{l}{${\rm d}_{207}=t_1^{3}t_2^{5}t_3^{3}t_4^{6}t_5^{6}t_6^{3}$,} & \multicolumn{1}{l}{${\rm d}_{208}=t_1^{3}t_2^{5}t_3^{6}t_4^{3}t_5^{3}t_6^{6}$,} \\ ${\rm d}_{209}=t_1^{3}t_2^{5}t_3^{6}t_4^{3}t_5^{6}t_6^{3}$, & ${\rm d}_{210}=t_1^{3}t_2^{5}t_3^{6}t_4^{6}t_5^{3}t_6^{3}.$ & & \end{tabular}\end{center} \begin{thebibliography}{99} \bibitem[Ada60]{J.A} J.F. Adams, On the non-existence of elements of Hopf invariant one, Ann. of Math. (2) 72 (1960), 20-104, \url{https://doi.org/10.2307/1970147}. \bibitem[Boa93]{Boardman} J.M. Boardman, Modular representations on the homology of power of real projective space, in Contemporary Mathematics. Algebraic Topology: Oaxtepec 1991, vol. 146, pp. 49-70 (Amer. Math. Soc., Providence, RI, 1993), \url{http://dx.doi.org/10.1090/conm/146}. \bibitem[BCP97]{Bosma} W. Bosma, J. Cannon and C. Playoust, The Magma Algebra System I: The User Language, J. Symbolic Comput. 24 (1997), 235-265, \url{https://doi.org/10.1006/jsco.1996.0125}. \bibitem[Bru97]{Bruner} R.R. Bruner, The cohomology of the mod 2 Steenrod algebra: A computer calculation, WSU Research Report 37 (1997), available online at \url{http://www.rrb.wayne.edu/papers/cohom.pdf}. \bibitem[BR22]{Bruner2} R.R. Bruner and J. Rognes, The cohomology of the mod 2 Steenrod algebra, Preprint (2022), arXiv:2109.13117. \bibitem[BCL12]{Brunetti1} M. Brunetti, A. Ciampella and A.L. Lomonaco, A total Steenrod operation as homomorphism of Steenrod algebra-modules, Ric. Mat. 61 (2012), 1-17, \url{https://doi.org/10.1007/s11587-011-0111-3}. \bibitem[Che11]{Chen} T.W. Chen, Determination of $\mbox{Ext}^{5,*}_{\mathscr{A}}(\mathbb{Z}/2, \mathbb{Z}/2),$ Topology Appl. 158 (2011), 660-689, \url{https://doi.org/10.1016/j.topol.2011.01.002}. \bibitem[Che12]{Chen2} T.W. Chen, The structure of decomposable elements in $\mbox{Ext}^{6,*}_{\mathscr{A}}(\mathbb{Z}/2, \mathbb{Z}/2),$ Preprint (2012), 35 pages. \bibitem[CH11]{CHa} P.H. Ch\ohorn n and L.M. H\`a, Lambda algebra and the Singer transfer, C. R. Math. Acad. Sci. Paris 349 (2011), 21-23, \url{https://doi.org/10.1016/j.crma.2010.11.008}. \bibitem[CH12]{CHa0} P.H. Ch\ohorn n and L.M. H\`a, On May spectral sequence and the algebraic transfer, Manuscripta Math. 138 (2012), 141-160, \url{https://doi.org/10.1007/s00229-011-0487-0}. \bibitem[Cur75]{Curtis} E.B. Curtis, The Dyer-Lashof algebra and the Lambda algebra, Illinois Jour. Math. 18, 231-246 (1975), \url{https://doi.org/10.1215/ijm/1256050812}. \bibitem[Fre38]{Freudenthal} H. Freudenthal, \"Uber die Klassen der Sph\"arenabbildungen I. Gro\ss e Dimensionen, Compos. Math. 5 (1938), 299-314. \bibitem[Hai22]{Hai} N.D.H. Hai, Stanley-Reisner rings and the occurrence of the Steinberg representation in the hit problem, C. R. Math. Acad. Sci. Paris 360 (2022), 1009-1026, \url{https://doi.org/10.5802/crmath.359}. \bibitem[Hun99]{Hung0} N.H.V. H\uhorn ng, The weak conjecture on spherical classes, Math. Z. 231 (1999), 727-743, \url{https://doi.org/10.1007/PL00004750}. \bibitem[Hun05]{Hung} N.H.V. H\uhorn ng, The cohomology of the Steenrod algebra and representations of the general linear groups, Trans. Amer. Math. Soc. 357 (2005), 4065-4089, \url{https://doi.org/10.1090/S0002-9947-05-03889-4}. \bibitem[HP19]{Hung2} N.H.V. H\uhorn ng and G. Powell, The $\mathscr A$-decomposability of the Singer construction, J. Algebra 517 (2019), 186-206, \url{https://doi.org/10.1016/j.jalgebra.2018.09.030}. \bibitem[Kam90]{Kameko} M. Kameko, \textit{Products of projective spaces as Steenrod modules}, PhD. thesis, Johns Hopkins University, USA, 1990. \bibitem[LZ87]{LZ} J. Lannes and S. Zarati, Sur les foncteurs d\'eriv\'es de la d\'estabilisation, Math. Z. 194 (1987), 25-59, \url{https://doi.org/10.1007/BF01168004}. \bibitem[Lin08]{Lin} W.H. Lin, ${\rm Ext}_{\mathcal A}^{4, *}(\mathbb Z/2, \mathbb Z/2)$ and ${\rm Ext}_{\mathcal A}^{5, *}(\mathbb Z/2, \mathbb Z/2),$ Topology Appl. 155 (2008), 459-496, \url{https://doi.org/10.1016/j.topol.2007.11.003}. \bibitem[Lin23]{Lin2} W. Lin, Charts of the cohomology of the mod 2 Steenrod algebra, Preprint (2023), 2276 pages, available online at \url{https://doi.org/10.5281/zenodo.7786290}. \bibitem[Liu60]{Liulevicius} A. Liulevicius, The factorization of cyclic reduced powers by secondary cohomology operations, Proc. Natl. Acad. Sci. USA, 46 (1960), 978-981, \url{https://doi.org/10.1073/pnas.46.7.978}. \bibitem[Min95]{Minami0} N. Minami, The Adams spectral sequence and the triple transfer, Amer. J. Math. 117 (1995), 965-985, \url{https://doi.org/10.2307/2374955}. \bibitem[Min99]{Minami} N. Minami, The iterated transfer analogue of the new doomsday conjecture, Trans. Amer. Math. Soc. 351 (1999), 2325-2351, \url{https://doi.org/10.1090/S0002-9947-99-02037-1}. \bibitem[Mon94]{Monks} K.G. Monks, Polynomial modules over the Steenrod algebra and conjugation in the Milnor basis, Proc. Amer. Math. Soc. 122 (1994), 625-634, \url{https://doi.org/10.2307/2161058}. \bibitem[Mot13]{Mothebe0} M.F. Mothebe, Dimension result for the polynomial algebra $\mathbb F_2[x_1, x_2, \ldots, x_n]$ as a module over the Steenrod algebra, Int. J. Math. Math. Sci. Art. ID 150704, 6 pp (2013), \url{https://doi.org/10.1155/2013/150704}. \bibitem[MU15]{Mothebe} M.F. Mothebe and L. Uys, Some relations between admissible monomials for the polynomial algebra, Int. J. Math. Math. Sci., Article ID 235806 (2015), pp. 7, \url{https://doi.org/10.1155/2015/235806}. \bibitem[MKR16]{MKR} M.F. Mothebe, P. Kaelo and O. Ramatebele, Dimension formulae for the polynomial algebra as a module over the Steenrod algebra in degrees less than or equal to 12, J. Math. Res. 8 (2016), 92-100, \url{http://dx.doi.org/10.5539/jmr.v8n5p92}. \bibitem[Pet87]{Peterson} F.P. Peterson, Generators of $H^*(\mathbb{R}P^{\infty}\times \mathbb{R}P^{\infty})$ as a module over the Steenrod algebra, Abstracts Papers Presented Am. Math. Soc. 833 (1987), 55-89. \bibitem[Pet89]{Peterson2} F.P. Peterson, $\mathcal A$-generators for certain polynomial algebras. Math. Proc. Cambridge Philos. Soc. 105 (1989), 311-312, \url{https://doi.org/10.1017/S0305004100067803}. \bibitem[PS15]{P.S1} \DJ.V. Ph\'uc and N. Sum, On the generators of the polynomial algebra as a module over the Steenrod algebra, C. R. Math. Acad. Sci. Paris 353 (2015), 1035-1040, \url{https://doi.org/10.1016/j.crma.2015.09.002}. \bibitem[PS17]{P.S2} \DJ.V. Ph\'uc and N. Sum, On a Minimal Set of Generators for the Polynomial Algebra of Five Variables as a Module over the Steenrod Algebra, Acta Math. Vietnam. 42 (2017), 149-162, \url{https://doi.org/10.1007/s40306-016-0190-z}. \bibitem[Phu16]{Phuc0} \DJ.V. Ph\'uc, The hit problem for the polynomial algebra of five variables in degree seventeen and its application, East-West J. of Mathematics, 18 (2016), 27-46. \bibitem[Phu20]{Phuc4} \DJ.V. Ph\'uc, The "hit" problem of five variables in the generic degree and its application, Topology Appl. 282 (2020), 107321, \url{https://doi.org/10.1016/j.topol.2020.107321}, available online at \url{https://www.researchgate.net/publication/342827323}. \bibitem[Phu21a]{Phuc6} \DJ.V. Ph\'uc, On Peterson's open problem and representations of the general linear groups, J. Korean Math. Soc. 58 (2021), 643-702, \url{https://doi.org/10.4134/JKMS.j200219}, available online at \url{https://www.researchgate.net/publication/343721577}. \bibitem[Phu21b]{Phuc10} \DJ.V. Ph\'uc, On the dimension of $H^{*}((\mathbb Z_2)^{\times t}, \mathbb Z_2)$ as a module over Steenrod ring, Topology Appl. 303 (2021), 107856, \url{https://doi.org/10.1016/j.topol.2021.107856}, available online at \url{https://www.researchgate.net/publication/350063962}. \bibitem[Phuc21d]{Phuc10-1} \DJ.V. Ph\'uc, A note on the modular representation on the $\mathbb Z/2$-homology groups of the fourth power of real projective space and its application, Preprint (2021), 52 pages, available online at \url{https://www.researchgate.net/publication/353065445}. \bibitem[Phuc21e]{Phuc11} \DJ.V. Ph\'uc, On mod-2 cohomology of the Steenrod ring and Singer's conjecture for the algebraic transfer, Preprint (2021), 40 pages, submitted for publication, available online at \url{https://www.researchgate.net/publication/350988977}. \bibitem[Phu23a]{Phuc12} \DJ.V. Ph\'uc, The affirmative answer to Singer's conjecture on the algebraic transfer of rank four, Proc. Roy. Soc. Edinburgh Sect. A 153 (2023), 1529-1542, \url{https://doi.org/10.1007/10.1017/prm.2022.57}, available online at \url{https://www.researchgate.net/publication/352284459}. \bibitem[Phu23b]{Phuc15} \DJ.V. Ph\'uc, On Peterson's classical hit problem and the structure of ${\rm Ext}_{\mathcal A}^{*, *}(\mathbb Z/2, \mathbb Z/2)$ via the lens of algebraic transfers, Preprint (2023), 32 pages, submitted for publication. \bibitem[Phu24a]{Phuc10-0} \DJ.V. Ph\'uc, On representations of general linear groups and the (symmetric) hit problems for polynomial algebras, Preprint (2024), 40 pages, submitted for publication. \bibitem[Phu24b]{Phuc10-3} \DJ.V. Ph\'uc, \textit{On Singer's conjecture for the fourth algebraic transfer in certain generic degrees}, J. Homotopy Relat. Struct. 19 (2024), 431-473, \url{https://doi.org/10.1007/s40062-024-00351-8}. \bibitem[Phu24c]{Phuc10-2} \DJ.V. Ph\'uc, \textit{On the algebraic transfers of ranks 4 and 6 at generic degrees}, To appear in Rend. Circ. Mat. Palermo, II. Ser (2024), 38 pages. \bibitem[Phu24d]{Phuc13} \DJ.V. Ph\'uc, On admissible $\mathcal A_2$-generators for the cohomology ring $H^*((G_1(\mathbb R^{\infty}))^{\times t}; \mathbb Z_2)$ and the (mod-2) cohomology of the Steenrod algebra $\mathcal A_2$, Preprint (2024), 30 pages, submitted for publication. \bibitem[Phu24e]{Phuc16} \DJ.V. Ph\'uc, \textit{On the dimensions of the graded space $\mathbb F_2\otimes_{\mathscr A}\mathbb F_2[x_1,x_2,\ldots ,x_s]$ at degrees $s + 5$ and its relation to algebraic transfers}, To appear in Internat. J. Algebra Comput. (2024), 57 pages, \url{https://doi.org/10.1142/S0218196724500401}. \bibitem[Phu24f]{Phuc17} \DJ.V. Ph\'uc, \textit{On the image of the Singer transfer in bi-degrees $(5, 5+2^{d})$}, To appear in Vietnam J. Math. (2024). \bibitem[Phu24g]{Phuc14} \DJ.V. Ph\'uc, A note on the hit problem for the polynomial algebra in the case of odd primes and its application, Rev. Real Acad. Cienc. Exactas Fis. Nat. Ser. A-Mat. 118 (2024), \url{https://doi.org/10.1007/s13398-023-01517-4}. \bibitem[Pri90]{Priddy} S. Priddy, On characterizing summands in the classifying space of a group, I. Amer. Jour. Math. 112 (1990), 737-748, \url{https://doi.org/10.2307/2374805}. \bibitem[Quy07]{Quynh} V.T.N. Qu\`ynh, On behavior of the fifth algebraic transfer, Geom. Topol. Monogr. 11 (2007), 309-326, \url{https://doi.org/10.2140/gtm.2007.11.309}. \bibitem[RS98]{Repka-Selick} J. Repka and P. Selick, On the subalgebra of $H_*((\mathbb{R}P^{\infty})^n; \mathbb{F}_2)$ annihilated by Steenrod operations, J. Pure Appl. Algebra 127 (1998), 273-288, \url{https://doi.org/10.1016/S0022-4049(96)00177-6}. \bibitem[Sil93]{Silverman} J.H. Silverman, Conjugation and excess in the Steenrod algebra, Proc. Amer. Math. Soc. 119 (1993), 657-661, \url{https://doi.org/10.2307/2159954}. \bibitem[Sil98]{Silverman2} J.H. Silverman, Hit polynomials and conjugation in the dual Steenrod algebra, Math. Proc. Cambriges Phil. Soc. 123 (1998), 531-547, \url{https://doi.org/10.1017/S0305004197002302}. \bibitem[Sin89]{Singer} W.M. Singer, The transfer in homological algebra, Math, Z. 202 (1989), 493-523, \url{https://doi.org/10.1007/BF01221587}. \bibitem[Sin91]{Singer1} W.M. Singer, On the action of Steenrod squares on Polynomial algebras, Proc. Amer. Math. Soc. 111 (1991), 577-583, \url{https://doi.org/10.2307/2048351}. \bibitem[Smi55]{Smith} L. Smith, \textit{Polynomial Invariants of Finite Groups}, A. K. Peters Ltd., Wellesley MA, 1955. \bibitem[Sum10]{Sum00} N. Sum, The negative answer to Kameko's conjecture on the hit problem, Adv. Math. 225(5) (2010), 2365-2390, \url{https://doi.org/10.1016/j.aim.2010.04.026}. \bibitem[Sum14a]{Sum0} N. Sum, On the Peterson hit problem of five variables and its applications to the fifth Singer transfer, East-West J. of Mathematics, 16 (2014), 47-62. \bibitem[Sum14b]{Sum1} N. Sum, The hit problem for the polynomial algebra of four variables, Preprint (2014), arXiv:1412.1709. \bibitem[Sum15]{Sum2} N. Sum, On the Peterson hit problem, Adv. Math. 274 (2015), 432-489, \url{https://doi.org/10.1016/j.aim.2015.01.010}. \bibitem[ST15]{Sum-Tin0} N. Sum and N.K. T\'in, Some results on the fifth Singer transfer, East-West J. of Mathematics 17 (2015), 70-84. \bibitem[Sum18a]{Sum2-0} N. Sum, The hit problem and the algebraic transfer in some degrees, East-West J. of Mathematics 20 (2018), 158-179. \bibitem[Sum18b]{Sum2-1} N. Sum, On the determination of the Singer transfer, Vietnam J. Sci., Technol. Eng. 60 (2018), 3-16, \url{https://doi.org/10.31276/VJSTE.60(1).03}. \bibitem[Sum19]{Sum3} N. Sum, On a construction for the generators of the polynomial algebra as a module over the Steenrod algebra, in Singh M., Song Y., Wu J. (eds), Algebraic Topology and Related Topics. Trends in Mathematics, pp. 265-286. Birkh\"auser/Springer, Singapore, 2019, \url{https://doi.org/10.1007/978-981-13-5742-8_14}. \bibitem[Sum21]{Sum4} N. Sum, The squaring operation and the Singer algebraic transfer, Vietnam J. Math. 49 (2021), 1079-1096, \url{https://doi.org/10.1007/s10013-020-00423-1}. \bibitem[Sum23]{Sum5} N. Sum, \textit{The squaring operation and the hit problem for the polynomial algebra in a type of generic degree}, J. Algebra 622 (2023), 165-196, \url{https://doi.org/10.1016/j.jalgebra.2023.01.018}. \newpage \bibitem[Tan70]{Tangora} M.C. Tangora, On the cohomology of the Steenrod algebra, Math. Z. 116 (1970), 18-64, \url{https://doi.org/10.1007/BF01110185}. \bibitem[TS16]{Sum-Tin} N.K. T\'in and N. Sum, Kameko's homomorphism and the algebraic transfer, C. R. Math. Acad. Sci. Paris 354 (2016), 940-943, \url{https://doi.org/10.1016/j.crma.2016.06.005}. \bibitem[Tin17]{Tin} N.K. T\'in, \textit{The hit problem for the polynomial algebra in five variables and applications}, PhD. thesis, Quy Nhon University, Vietnam, 2017. \bibitem[WW18a]{Walker-Wood} G. Walker and R.M.W. Wood, \textit{Polynomials and the mod 2 Steenrod Algebra. Vol 1. The Peterson hit problem}, in London Mathematical Society Lecture Note Series. Cambridge University Press, Cambridge, volume 441, 2018. \bibitem[WW18b]{Walker-Wood2} G. Walker and R.M.W. Wood, \textit{Polynomials and the mod 2 Steenrod Algebra. Vol 2. Representations of $GL(n, \mathbb F_2)$}, in London Mathematical Society Lecture Note Series. Cambridge University Press, Cambridge, volume 442, 2018. \bibitem[Woo89]{Wood} R.M.W. Wood, Steenrod squares of polynomials and the Peterson conjecture, Math. Proc. Cambriges Phil. Soc. 105 (1989), 307-309, \url{https://doi.org/10.1017/S0305004100067797}. \end{thebibliography} \end{document} \subsection{The behavior of the fourth algebraic transfer in some generic degrees} This section is intended to present several findings regarding the behavior of the fourth algebraic transfer in various generic degrees. These results have been communicated in the preprints \cite{Phuc10-1, Phuc10-2}. To begin with, it is worth recalling that $H_*(BV_4) = [P^{\otimes 4}]^{*} = \Gamma(a_1,a_2,\ldots, a_4)$ is a divided polynomial algebra generated by $a_1, \ldots, a_4,$ with each generator having a degree of 1. Here $a_i = a_i^{(1)}$ denotes the dual of $t_i\in H^{1}(BV_4) = P^{\otimes 4}_1,$ where the duality is taken with respect to the basis of $H^{*}(BV_4) = P^{\otimes 4}$ consisting of all monomials in $t_1, \ldots, t_4.$ The (right) action of the algebra $\mathcal A$ on $H_*(BV_4)$ is determined by the rule $(a_i^{(\ell)})Sq^k = \binom{\ell-k}{k}a_i^{(\ell-k)}$ and the Cartan formula. In accordance with the paper \cite{Sum2-1}, it is adequate to investigate the behavior of the fourth Singer algebraic transfer in the internal degrees of the following forms: $$\begin{array}{lll} \medskip {\rm (i)} & n &= 2^{k+1} - m, \ \mbox{for $1\leq m\leq 3,$} \\ \medskip {\rm (ii)} & n &= 2^{k+s+1} +2^{k + 1}-3,\\ \medskip {\rm (iii)} & n &= 2^{k+s} +2^{k}-2,\\ \medskip {\rm (iv)} & n &= 2^{k+s+u} + 2^{k+s} + 2^{k} - 3, \end{array}$$ whenever $k,\, s,$ and $u$ are positive integers. Verifying that the aforementioned degrees can be expressed in the form of \eqref{pt} is a straightforward task. The explicit description of the rank four Singer transfer has been provided by Sum \cite{Sum2-1} for item (i), and by the author \cite{Phuc10-3, Phuc12} for item (ii), as well as for item (iii) with $s\geq 1,\, s\neq 2,\, 4$. We will now conduct an investigation into $Tr_4^{\mathcal A}$ for item (iii) when $s$ is equal to 2 and 4, and also for item (iv). \begin{thm} The following assertions are true: \begin{itemize} \item[(I)] Singer's transfer is an isomorphism in bidegrees $(4, 2+5\cdot 2^k)$ and $(4, 2+17\cdot 2^k)$ for any $k > 0.$ \item[(II)] Given the generic degree $n:= n_{k,\,s,\, u} = 2^{k+s+u} + 2^{k+s} + 2^{k} - 3,$ whenever $k,\, s,$ and $u$ are positive integers. Then, the transfer homomorphism of rank four $$Tr_4^{\mathcal A}: (\mathbb F_2\otimes_{GL_4}{\rm Ann}_{\overline{\mathcal A}}[P^{\otimes 4}]^{*})_{n_{k,\,s,\, u}}\longrightarrow {\rm Ext}_{\mathcal A}^{4, 4+n_{k,\,s,\, u}}(\mathbb F_2, \mathbb F_2)$$ is also an isomorphism for all $k,\,s,\, u.$ \end{itemize} \end{thm} \begin{proof} The proof of part $(I)$ can be found in the preprint \cite{Phuc10-1}. We will now proceed to prove part $(II)$. The dimensions of the indecomposables $QP^{\otimes 4}_{n_{k,\,s,\, u}}$ are determined as follows, as a result of the work of Sum \cite{Sum2-1}: $$ \dim QP^{\otimes 4}_{n_{k,\,s,\, u}} =\left\{\begin{array}{ll} 64,&\mbox{if $s = 1$, $u = 1$ and $k=1$},\\[1mm] 120,&\mbox{if $s = 1$, $u = 1$ and $k\geq 2$},\\[1mm] 155, &\mbox{if $s = 2$, $u = 1$ and $k = 1$},\\[1mm] 210, &\mbox{if $s = 2$, $u = 1$ and $k\geq 2$},\\[1mm] 140,&\mbox{if $s\geq 3$, $u=1$ and $k= 1$},\\[1mm] 210,&\mbox{if $s\geq 3$, $u=1$ and $k\geq 2$},\\[1mm] 140,&\mbox{if $s = 1$, $u = 2$ and $k= 1$},\\[1mm] 225,&\mbox{if $s = 1$, $u = 2$ and $k\geq 2$},\\[1mm] 120,&\mbox{if $s = 1$, $u\geq 3$ and $k= 1$},\\[1mm] 210,&\mbox{if $s = 1$, $u\geq 3$ and $k\geq 2$},\\[1mm] 225,&\mbox{if $s = 2$, $u\geq 2$ and $k = 1$},\\[1mm] 210,&\mbox{if $s \geq 3$, $u\geq 2$ and $k = 1$},\\[1mm] 315,&\mbox{if $s \geq 2$, $u\geq 2$ and $k \geq 2$}. \end{array}\right.$$ Thanks to the above results, we are able to compute explicitly the action of the group $GL_4$ on the spaces $[QP^{\otimes 4}_{n_{k,\,s,\, u}}]^{GL_4}.$ By utilizing the homomorphisms $\sigma_d : P^{\otimes 4}\longrightarrow P^{\otimes 4} ,\, 1\leq d\leq 4,$ and performing similar calculations to those in the preprints \cite{Phuc10-1, Phuc10-3}, we obtain that $$ \dim (\mathbb F_2\otimes_{GL_4}{\rm Ann}_{\overline{\mathcal A}}[P^{\otimes 4}]^{*})_{n_{k,\,s,\, u}} = \dim [QP^{\otimes 4}_{n_{k,\,s,\, u}}]^{GL_4} =\left\{\begin{array}{ll} 1&\mbox{if $s = 1$, $u = 2$ and $k\geq 2$},\\[1mm] 1&\mbox{if $s = 2$, $u\geq 1$ and $k = 1$},\\[1mm] 1&\mbox{if $s \geq 2$, $u\geq 2$ and $k\geq 2$},\\[1mm] 0, &\mbox{otherwise}. \end{array}\right. $$ Moreover, the generators of $(\mathbb F_2\otimes_{GL_4}{\rm Ann}_{\overline{\mathcal A}}[P^{\otimes 4}]^{*})_{n_{k,\,s,\, u}}$ can be determined as follows: $$ (\mathbb F_2\otimes_{GL_4}{\rm Ann}_{\overline{\mathcal A}}[P^{\otimes 4}]^{*})_{n_{k,\,s,\, u}} =\left\{\begin{array}{ll} \langle [\zeta_{k,\, 1,\, 2}] \rangle,&\mbox{if $s = 1$, $u = 2$ and $k\geq 2$},\\[1mm] \langle [\zeta_{1,\, 2,\, u}] \rangle,&\mbox{if $s = 2$, $u\geq 1$ and $k = 1$},\\[1mm] \langle [\zeta_{k,\, s,\, u}] \rangle,&\mbox{if $s \geq 2$, $u\geq 2$ and $k\geq 2$},\\[1mm] 0, &\mbox{otherwise}. \end{array}\right.$$ wherein the elements $$\begin{array}{ll} \medskip \zeta_{k,\, 1,\, 2}&= a_2^{(2^{k+2}-1)}a_3^{(2^{k+2}-1)}a_4^{(3.2^{k}-1)} + a_2^{(2^{k+2}-1)}a_3^{(5.2^{k}-1)}a_4^{(2^{k+1}-1)} \\ \medskip &\quad + a_2^{(6.2^{k}-1)}a_3^{(3.2^{k}-1)}a_4^{(2^{k+1}-1)} + a_2^{(7.2^{k}-1)}a_3^{(2^{k+1}-1)}a_4^{(2^{k+1}-1)},\\ \medskip \zeta_{1,\, 2,\, u} &= a_1^{(2^{u+3}-1)}a_2^{(3)}a_3^{(3)}a_4^{(2)} + a_1^{(2^{u+3}-1)}a_2^{(3)}a_3^{(4)}a_4^{(1)}+ a_1^{(2^{u+3}-1)}a_2^{(5)}a_3^{(2)}a_4^{(1)} + a_1^{(2^{u+3}-1)}a_2^{(6)}a_3^{(1)}a_4^{(1)},\\ \medskip \zeta_{k,\, s,\, u} &= a_2^{(2^{k}-1)}a_3^{(2^{k+s}-1)}a_4^{(2^{k+s+u}-1)} \end{array}$$ are $\overline{\mathcal A}$-annihilated. Thus, one has isomorphisms: $$\begin{array}{ll} \medskip (\mathbb F_2\otimes_{GL_4}{\rm Ann}_{\overline{\mathcal A}}[P^{\otimes 4}]^{*})_{n_{k,\,1,\, 2}} \cong \mathbb F_2,\ (k\geq 2),\\ \medskip (\mathbb F_2\otimes_{GL_4}{\rm Ann}_{\overline{\mathcal A}}[P^{\otimes 4}]^{*})_{n_{1,\,2,\, u}} \cong \mathbb F_2,\ (u\geq 1),\\ \medskip (\mathbb F_2\otimes_{GL_4}{\rm Ann}_{\overline{\mathcal A}}[P^{\otimes 4}]^{*})_{n_{k,\,s,\, u}} \cong \mathbb F_2,\, (k\geq 2, s\geq 2, u\geq 2). \end{array}$$ To simplify matters, we shall describe explicitly $(\mathbb F_2\otimes_{GL_4}{\rm Ann}_{\overline{\mathcal A}}[P^{\otimes 4}]^{*})_{n_{1,\,1,\, 1}}$ and $(\mathbb F_2\otimes_{GL_4}{\rm Ann}_{\overline{\mathcal A}}[P^{\otimes 4}]^{*})_{n_{1,\,2,\, 1}}.$ The remaining coinvariants can be determined through analogous techniques. \medskip $\bullet$ We note that the indecomposables $QP^{\otimes 4}_{n_{1,\,1,\, 1}}$ has dimension $64$ and that $$QP^{\otimes 4}_{n_{1,\,1,\, 1}}\cong (QP^{\otimes 4}_{n_{1,\,1,\, 1}})^{0}\bigoplus (QP^{\otimes 4}_{n_{1,\,1,\, 1}})^{>0}.$$ Following \cite{Sum2-1}, $(QP^{\otimes 4}_{n_{1,\,1,\, 1}})^{0}$ has a monomial basis consisting of all the equivalence classes represented by the following admissible monomial: \begin{center} \begin{tabular}{llll} ${\rm adm}_{1}=t_2t_3^{3}t_4^{7}$, & ${\rm adm}_{2}=t_2t_3^{7}t_4^{3}$, & ${\rm adm}_{3}=t_2^{3}t_3t_4^{7}$, & ${\rm adm}_{4}=t_2^{3}t_3^{7}t_4$, \\ ${\rm adm}_{5}=t_2^{7}t_3t_4^{3}$, & ${\rm adm}_{6}=t_2^{7}t_3^{3}t_4$, & ${\rm adm}_{7}=t_1t_3^{3}t_4^{7}$, & ${\rm adm}_{8}=t_1t_3^{7}t_4^{3}$, \\ ${\rm adm}_{9}=t_1t_2^{3}t_4^{7}$, & ${\rm adm}_{10}=t_1t_2^{3}t_3^{7}$, & ${\rm adm}_{11}=t_1t_2^{7}t_4^{3}$, & ${\rm adm}_{12}=t_1t_2^{7}t_3^{3}$, \\ ${\rm adm}_{13}=t_1^{3}t_3t_4^{7}$, & ${\rm adm}_{14}=t_1^{3}t_3^{7}t_4$, & ${\rm adm}_{15}=t_1^{3}t_2t_4^{7}$, & ${\rm adm}_{16}=t_1^{3}t_2t_3^{7}$, \\ ${\rm adm}_{17}=t_1^{3}t_2^{7}t_4$, & ${\rm adm}_{18}=t_1^{3}t_2^{7}t_3$, & ${\rm adm}_{19}=t_1^{7}t_3t_4^{3}$, & ${\rm adm}_{20}=t_1^{7}t_3^{3}t_4$, \\ ${\rm adm}_{21}=t_1^{7}t_2t_4^{3}$, & ${\rm adm}_{22}=t_1^{7}t_2t_3^{3}$, & ${\rm adm}_{23}=t_1^{7}t_2^{3}t_4$, & ${\rm adm}_{24}=t_1^{7}t_2^{3}t_3$, \\ ${\rm adm}_{25}=t_2^{3}t_3^{3}t_4^{5}$, & ${\rm adm}_{26}=t_1^{3}t_3^{3}t_4^{5}$, & ${\rm adm}_{27}=t_1^{3}t_2^{3}t_4^{5}$, & ${\rm adm}_{28}=t_1^{3}t_2^{3}t_3^{5}$, \\ ${\rm adm}_{29}=t_2^{3}t_3^{5}t_4^{3}$, & ${\rm adm}_{30}=t_1^{3}t_3^{5}t_4^{3}$, & ${\rm adm}_{31}=t_1^{3}t_2^{5}t_4^{3}$, & ${\rm adm}_{32}=t_1^{3}t_2^{5}t_3^{3}$. \end{tabular}\end{center} It is easy to see that $[{\rm adm}_j]_{\omega({\rm adm}_j)} = [{\rm adm}_j]$ for every $j.$ By a simple computation, we have a direct summand decomposition of the $\Sigma_4$-submodules: $ (QP^{\otimes 4}_{n_{1,\,1,\, 1}})^{0} = \Sigma_4({\rm adm}_1)\bigoplus \Sigma_4({\rm adm}_{25}),$ where $\Sigma_4({\rm adm}_1) = \langle \{[{\rm adm}_j]:\ 1\leq j\leq 24\}\rangle$ and $\Sigma_4({\rm adm}_{25}) = \langle \{[{\rm adm}_j]:\ 25\leq j\leq 32\}\rangle.$ We compute the action of $\Sigma_4$ on the $(QP^{\otimes 4}_{n_{1,\,1,\, 1}})^{0}$ and obtain $ [(QP^{\otimes 4}_{n_{1,\,1,\, 1}})^{0}]^{\Sigma_4} = \langle [\sum_{1\leq j\leq 24}{\rm adm}_j] \rangle.$ To do this, we shall show that $[\Sigma_4({\rm adm}_1)]^{\Sigma_4} = \langle [\sum_{1\leq j\leq 24}{\rm adm}_j] \rangle$ and $[\Sigma_4({\rm adm}_{25})]^{\Sigma_4} = 0.$ For simplicity, we compute the $\Sigma_4$-invariant subspace $[\Sigma_4({\rm adm}_{25})]^{\Sigma_4}$ in detail. The remaining space can be obtained in the same way. We can see that the set $\{[{\rm adm}_j]:\ 25\leq j\leq 32\}$ is a monomial basis of $\Sigma_4({\rm adm}_{25}).$ Suppose that $[t]\in [\Sigma_4({\rm adm}_{25})]^{\Sigma_4},$ then one has $t\equiv \sum_{25\leq j\leq 32}\gamma_j{\rm adm}_j,$ where $\gamma_j\in \mathbb F_2$ for all $j.$ For each $i,\, 1\leq i\leq 3,$ applying the homomorphism $\sigma_d: P^{\otimes 4}\longrightarrow P^{\otimes 4},\, 1\leq d\leq 3$ to the sum $\sum_{25\leq j\leq 32}\gamma_j{\rm adm}_j,$ one gets the following equalities: $$ \begin{array}{ll} \sigma_1\big(\sum_{25\leq j\leq 32}\gamma_j{\rm adm}_j\big) &\equiv \gamma_{25}{\rm adm}_{26} + \gamma_{26}{\rm adm}_{25} + \gamma_{27}{\rm adm}_{27} + \gamma_{28}{\rm adm}_{28} \\ \medskip &\quad + \gamma_{29}{\rm adm}_{30} + \gamma_{30}{\rm adm}_{29} +\gamma_{31}t_1^{5}t_2^{3}t_4^{3} + \gamma_{32}t_1^{5}t_2^{3}t_3^{3},\\ \sigma_2\big(\sum_{25\leq j\leq 32}\gamma_j{\rm adm}_j\big) &\equiv \gamma_{25}{\rm adm}_{25} + \gamma_{26}{\rm adm}_{27} + \gamma_{27}{\rm adm}_{26} + \gamma_{28}{\rm adm}_{32} \\ \medskip &\quad + \gamma_{29}t_2^{5}t_3^{3}t_4^{3} + \gamma_{30}{\rm adm}_{31} +\gamma_{31}{\rm adm}_{30} + \gamma_{32}{\rm adm}_{28},\\ \sigma_3\big(\sum_{25\leq j\leq 32}\gamma_j{\rm adm}_j\big) &\equiv \gamma_{25}{\rm adm}_{29} + \gamma_{26}{\rm adm}_{30} + \gamma_{27}{\rm adm}_{28} + \gamma_{28}{\rm adm}_{27} \\ \medskip &\quad + \gamma_{29}{\rm adm}_{25}+ \gamma_{30}{\rm adm}_{26} +\gamma_{31}{\rm adm}_{32} + \gamma_{32}{\rm adm}_{31}. \end{array}$$ Applying the Cartan formula, the monomials $t_1^{5}t_2^{3}t_4^{3},$ $t_1^{5}t_2^{3}t_3^{3}$ and $t_2^{5}t_3^{3}t_4^{3} $ can be written as a linear combination of terms ${\rm adm}_j,\, 1\leq j\leq 32$ as follows: $t_1^{5}t_2^{3}t_4^{3} \equiv {\rm adm}_{27} + {\rm adm}_{31},\ t_1^{5}t_2^{3}t_3^{3} \equiv {\rm adm}_{28} + {\rm adm}_{32},\ t_2^{5}t_3^{3}t_4^{3} \equiv {\rm adm}_{25} + {\rm adm}_{29}.$ Combining these calculations and the relations $\sigma_d(t)\equiv t$ for all $d,\, 1\leq d\leq 3,$ we can easily deduce that $$ \begin{array}{ll} \sigma_1(t)+t&\equiv \bigg((\gamma_{25} + \gamma_{26})({\rm adm}_{25} + {\rm adm}_{26}) + \gamma_{31}{\rm adm}_{27} + \gamma_{32}{\rm adm}_{28}\\ \medskip &\quad + (\gamma_{29} + \gamma_{30})({\rm adm}_{29} + {\rm adm}_{30})\bigg)\equiv 0,\\ \sigma_2(t)+t&\equiv \bigg((\gamma_{26} + \gamma_{27})({\rm adm}_{26} + {\rm adm}_{27}) + \gamma_{29}{\rm adm}_{25} + (\gamma_{30} + \gamma_{31})({\rm adm}_{30} + {\rm adm}_{31}) \\ \medskip &\quad+ (\gamma_{28} + \gamma_{32})({\rm adm}_{28} + {\rm adm}_{32})\bigg)\equiv 0,\\ \sigma_3(t)+t&\equiv \bigg((\gamma_{25} + \gamma_{29})({\rm adm}_{25} + {\rm adm}_{29}) + (\gamma_{26} + \gamma_{30})({\rm adm}_{26} + {\rm adm}_{30})\\ &\quad + (\gamma_{31} + \gamma_{32})({\rm adm}_{31} + {\rm adm}_{32})\bigg)\equiv 0. \end{array}$$ These equalities imply that $\gamma_j = 0$ for all $j,\, 25\leq j\leq 32.$ \medskip We now compute the $\Sigma_4$-invariants space $[(QP^{\otimes 4}_{n_{1,\,1,\, 1}})^{>0}]^{\Sigma_4}.$ Under the action of the group $\Sigma_4,$ we have $[(QP^{\otimes 4}_{n_{1,\,1,\, 1}})^{>0}]^{\Sigma_4} = \langle [\sum_{53\leq j\leq 64}{\rm adm}_j] \rangle,$ where the admissible monomials ${\rm adm}_j,\, 53\leq j\leq 64$ are described as below. Indeed, let us recall that from a result in Sum \cite{Sum2-1}, $(QP^{\otimes 4}_{n_{1,\,1,\, 1}})^{>0}$ has a monomial basis consisting of all the equivalence classes represented by the following admissible monomial: \begin{center} \begin{tabular}{llll} ${\rm adm}_{33}=t_1t_2t_3^{2}t_4^{7}$, & ${\rm adm}_{34}=t_1t_2t_3^{7}t_4^{2}$, & ${\rm adm}_{35}=t_1t_2^{2}t_3t_4^{7}$, & ${\rm adm}_{36}=t_1t_2^{2}t_3^{7}t_4$, \\ ${\rm adm}_{37}=t_1t_2^{7}t_3t_4^{2}$, & ${\rm adm}_{38}=t_1t_2^{7}t_3^{2}t_4$, & ${\rm adm}_{39}=t_1^{7}t_2t_3t_4^{2}$, & ${\rm adm}_{40}=t_1^{7}t_2t_3^{2}t_4$, \\ ${\rm adm}_{41}=t_1^{3}t_2t_3t_4^{6}$, & ${\rm adm}_{42}=t_1^{3}t_2t_3^{6}t_4$, & ${\rm adm}_{43}=t_1t_2^{2}t_3^{3}t_4^{5}$, & ${\rm adm}_{44}=t_1t_2^{2}t_3^{5}t_4^{3}$, \\ ${\rm adm}_{45}=t_1t_2^{3}t_3^{2}t_4^{5}$, & ${\rm adm}_{46}=t_1t_2^{3}t_3^{5}t_4^{2}$, & ${\rm adm}_{47}=t_1^{3}t_2t_3^{2}t_4^{5}$, & ${\rm adm}_{48}=t_1^{3}t_2t_3^{5}t_4^{2}$, \\ ${\rm adm}_{49}=t_1^{3}t_2^{5}t_3t_4^{2}$, & ${\rm adm}_{50}=t_1^{3}t_2^{5}t_3^{2}t_4$, & ${\rm adm}_{51}=t_1t_2^{3}t_3^{3}t_4^{4}$, & ${\rm adm}_{52}=t_1t_2^{3}t_3^{4}t_4^{3}$, \\ ${\rm adm}_{53}=t_1t_2t_3^{3}t_4^{6}$, & ${\rm adm}_{54}=t_1t_2t_3^{6}t_4^{3}$, & ${\rm adm}_{55}=t_1t_2^{3}t_3t_4^{6}$, & ${\rm adm}_{56}=t_1t_2^{3}t_3^{6}t_4$, \\ ${\rm adm}_{57}=t_1t_2^{6}t_3t_4^{3}$, & ${\rm adm}_{58}=t_1t_2^{6}t_3^{3}t_4$, & ${\rm adm}_{59}=t_1^{3}t_2t_3^{3}t_4^{4}$, & ${\rm adm}_{60}=t_1^{3}t_2t_3^{4}t_4^{3}$, \\ ${\rm adm}_{61}=t_1^{3}t_2^{3}t_3t_4^{4}$, & ${\rm adm}_{62}=t_1^{3}t_2^{3}t_3^{4}t_4$, & ${\rm adm}_{63}=t_1^{3}t_2^{4}t_3t_4^{3}$, & ${\rm adm}_{64}=t_1^{3}t_2^{4}t_3^{3}t_4$. \end{tabular}\end{center} Through a straightforward calculation, it becomes evident that $$ \begin{array}{ll} \medskip &\Sigma_4({\rm adm}_{33}) = \langle \{[{\rm adm}_j]:\ 33\leq j\leq 40\}\rangle, \\ &\Sigma_4({\rm adm}_{41}, {\rm adm}_{43}, {\rm adm}_{51}) = \langle \{[{\rm adm}_j]:\ 41\leq j\leq 64\}\rangle, \end{array}$$ and so, one has an isomorphism $$(QP^{\otimes 4}_{n_{1,\,1,\, 1}})^{>0} \cong \Sigma_4({\rm adm}_{33})\bigoplus \Sigma_4({\rm adm}_{41}, {\rm adm}_{43}, {\rm adm}_{51}).$$ By similar calculations as above, we obtain $$ [\Sigma_4({\rm adm}_{33})]^{\Sigma_4} = 0, \ \ [\Sigma_4({\rm adm}_{41}, {\rm adm}_{43}, {\rm adm}_{51})]^{\Sigma_4} = \langle [\sum_{53\leq j\leq 64}{\rm adm}_j] \rangle.$$ With the data in hand and under the action of the group $\Sigma_4,$ we get $$ [QP^{\otimes 4}_{n_{1,\,1,\, 1}}]^{\Sigma_4} = \big\langle\big \{[\sum_{1\leq j\leq 24}{\rm adm}_j],\ [\sum_{53\leq j\leq 64}{\rm adm}_j]\big\} \big\rangle.$$ Now, for any $[h]\in [QP^{\otimes 4}_{n_{1,\,1,\, 1}}]^{GL_4},$ since $\Sigma_4\subset GL_4,$ we have $$h\equiv \bigg(\beta_1\sum_{1\leq j\leq 24}{\rm adm}_j + \beta_2\sum_{53\leq j\leq 64}{\rm adm}_j\bigg),$$ wherein $\beta_1$ and $\beta_2$ belong to $\mathbb F_2.$ Applying the homomorphism $\sigma_4: P^{\otimes 4}\longrightarrow P^{\otimes 4}$ to the sum $S:=\beta_1\sum_{1\leq j\leq 24}{\rm adm}_j + \beta_2\sum_{53\leq j\leq 64}{\rm adm}_j$ and we compute explicitly $\sigma_4(S)$ in admissible terms ${\rm adm}_{j}$ modulo ($\overline{\mathcal A}P^{\otimes 4}_{n_{1,\,1,\, 1}}$). Then, by the relation $\sigma_4(h) + h\equiv 0,$ we can easily obtain $$ \begin{array}{ll} \sigma_4(h) + h&\equiv \big(\beta_1(\sum_{1\leq j\leq 6}{\rm adm}_j + \sum_{9\leq j\leq 12}{\rm adm}_j + {\rm adm}_{17} + {\rm adm}_{18}+{\rm adm}_{27} + {\rm adm}_{28}\\ &\quad + {\rm adm}_{33}+{\rm adm}_{34} + {\rm adm}_{47} + {\rm adm}_{48}+ \sum_{55\leq j\leq 60}{\rm adm}_j+ {\rm adm}_{63} + {\rm adm}_{64}\\ &\quad + (\beta_1 +\beta_2)({\rm adm}_{45}+{\rm adm}_{46})\big)\equiv 0. \end{array}$$ The above equality implies that $\beta_1 = \beta_2 =0.$ Thus, the coinvariant $(\mathbb F_2\otimes_{GL_4}{\rm Ann}_{\overline{\mathcal A}}[P^{\otimes 4}]^{*})_{n_{1,\,1,\, 1}}$ vanishes. $\bullet$ In the case of the degree $n_{1,\,2,\,1}$, which is widely acknowledged (as stated in Sum \cite{Sum2-1}), $\dim (QP^{\otimes 4}_{n_{1,\,2,\, 1}})^0 = \dim (QP^{\otimes 4}_{n_{1,\,2,\, 1}})^0(3,2,2,1) = 56$ and $\dim(QP^{\otimes 4}_{n_{1,\,2,\, 1}})^{>0} = \dim (QP^{\otimes 4}_{n_{1,\,2,\, 1}})^{>0}(3,2,2,1) = 99.$ A direct computation then yields: $$ \begin{array}{ll} {\rm [}(QP^{\otimes 4}_{n_{1,\,2,\, 1}})^0{\rm ]}^{\Sigma_4}&= \langle [\widehat{p_1}],\, [\widehat{p_2}],\, [\widehat{p_3}] \rangle,\\ {\rm [}(QP^{\otimes 4}_{n_{1,\,2,\, 1}})^{>0}{\rm ]}^{\Sigma_4}&= \big\langle [\widehat{p_4}],\, [\widehat{p_5}:= \sum_{1\leq j\leq 3}q_j] ,\, [\widehat{p_6}:= \sum_{2\leq j\leq 6}q_j],\, [\widehat{p_7}:= \sum_{6\leq j\leq 9}q_j],\, [\widehat{p_8}:=q_5 + \sum_{9\leq j\leq 11}q_j] \big\rangle,\\ \end{array}$$ where $$ \begin{array}{ll} \widehat{p_1}&= t_2t_3^{7}t_4^{15}+ t_1t_3^{7}t_4^{15}+ t_1t_2^{7}t_4^{15}+ t_1t_2^{7}t_3^{15}+ t_2t_3^{15}t_4^{7}+ t_1t_3^{15}t_4^{7}+ t_1t_2^{15}t_4^{7}+ t_1t_2^{15}t_3^{7}+ \medskip t_2^{7}t_3t_4^{15}\\ &\quad+ t_1^{7}t_3t_4^{15}+ t_1^{7}t_2t_4^{15}+ t_1^{7}t_2t_3^{15}+ t_2^{7}t_3^{15}t_4+ t_1^{7}t_3^{15}t_4+ t_1^{7}t_2^{15}t_4+ t_1^{7}t_2^{15}t_3+ t_2^{15}t_3t_4^{7}+ \medskip t_1^{15}t_3t_4^{7}\\ &\quad+ t_1^{15}t_2t_4^{7}+ t_1^{15}t_2t_3^{7}+ t_2^{15}t_3^{7}t_4+ t_1^{15}t_3^{7}t_4+ t_1^{15}t_2^{7}t_4+ \medskip t_1^{15}t_2^{7}t_3,\\ \widehat{p_2}&= t_2^{3}t_3^{5}t_4^{15}+ t_1^{3}t_3^{5}t_4^{15}+ t_1^{3}t_2^{5}t_4^{15}+ t_1^{3}t_2^{5}t_3^{15}+ t_2^{3}t_3^{15}t_4^{5}+ t_1^{3}t_3^{15}t_4^{5}+ \medskip t_1^{3}t_2^{15}t_4^{5}\\ &\quad+ t_1^{3}t_2^{15}t_3^{5} t_2^{15}t_3^{3}t_4^{5}+ t_1^{15}t_3^{3}t_4^{5}+ t_1^{15}t_2^{3}t_4^{5}+ \medskip t_1^{15}t_2^{3}t_3^{5},\\ \widehat{p_3}&=t_2^{3}t_3^{13}t_4^{7}+ t_1^{3}t_3^{13}t_4^{7}+ t_1^{3}t_2^{13}t_4^{7}+ t_1^{3}t_2^{13}t_3^{7}+ t_2^{7}t_3^{3}t_4^{13}+ t_1^{7}t_3^{3}t_4^{13}+ \medskip t_1^{7}t_2^{3}t_4^{13}\\ &\quad+ t_1^{7}t_2^{3}t_3^{13}+ t_2^{7}t_3^{11}t_4^{5}+ t_1^{7}t_3^{11}t_4^{5}+ t_1^{7}t_2^{11}t_4^{5}+ t_1^{7}t_2^{11}t_3^{5}+ t_2^{7}t_3^{7}t_4^{9}+ \medskip t_1^{7}t_3^{7}t_4^{9}\\ &\quad+ \medskip t_1^{7}t_2^{7}t_4^{9}+ t_1^{7}t_2^{7}t_3^{9},\\ \widehat{p_4}&= t_1t_2t_3^{6}t_4^{15}+ t_1t_2t_3^{15}t_4^{6}+ t_1t_2^{6}t_3t_4^{15}+ t_1t_2^{6}t_3^{15}t_4+ t_1t_2^{15}t_3t_4^{6}+ \medskip t_1t_2^{15}t_3^{6}t_4\\ &\quad+ t_1^{15}t_2t_3t_4^{6}+ t_1^{15}t_2t_3^{6}t_4+ t_1^{3}t_2t_3^{4}t_4^{15}+ t_1^{3}t_2t_3^{15}t_4^{4}+ t_1^{3}t_2^{15}t_3t_4^{4}+ \medskip t_1^{15}t_2^{3}t_3t_4^{4}\\ &\quad+ t_1^{3}t_2^{4}t_3t_4^{15}+ t_1^{3}t_2^{4}t_3^{15}t_4+ t_1^{3}t_2^{15}t_3^{4}t_4+ \medskip t_1^{15}t_2^{3}t_3^{4}t_4,\\ \end{array}$$ \newpage $$ \begin{array}{ll} \medskip q_1&= t_1t_2^{3}t_3^{14}t_4^{5} + t_1^{3}t_2^{3}t_3^{12}t_4^{5},\\ \medskip q_2&= t_1t_2^{3}t_3^{7}t_4^{12} + t_1^{3}t_2^{4}t_3^{11}t_4^{5} + t_1t_2^{6}t_3^{11}t_4^{5} + t_1t_2^{6}t_3^{7}t_4^{9},\\ \medskip q_3&= t_1^3t_2^{5}t_3^{6}t_4^{9} + t_1^{3}t_2^{5}t_3^{7}t_4^{8},\\ q_4&= t_1t_2^{3}t_3^{5}t_4^{14}+ t_1^{3}t_2t_3^{5}t_4^{14}+ t_1^{3}t_2^{5}t_3t_4^{14}+ t_1^{3}t_2^{5}t_3^{14}t_4+ t_1^{3}t_2^{7}t_3t_4^{12}+ t_1^{3}t_2^{3}t_3^{5}t_4^{12}+ \medskip t_1^{3}t_2^{7}t_3^{12}t_4,\\ q_5&=t_1^{3}t_2t_3^{14}t_4^{5}+ t_1t_2^{7}t_3^{3}t_4^{12}+ t_1^{7}t_2t_3^{3}t_4^{12}+ t_1^{3}t_2^{13}t_3^{2}t_4^{5}+ t_1^{3}t_2^{13}t_3^{3}t_4^{4}+ t_1t_2^{7}t_3^{10}t_4^{5}+ \medskip t_1^{7}t_2t_3^{10}t_4^{5}\\ &\quad+ t_1^{3}t_2^{7}t_3^{8}t_4^{5}+ t_1^{7}t_2^{9}t_3^{2}t_4^{5}+ \medskip t_1^{7}t_2^{9}t_3^{3}t_4^{4},\\ q_6&= t_1^{7}t_2^{3}t_3t_4^{12}+ t_1^{3}t_2^{7}t_3^{9}t_4^{4}+ t_1^{7}t_2^{7}t_3t_4^{8}+ t_1^{7}t_2^{3}t_3^{12}t_4+ \medskip t_1^{7}t_2^{7}t_3^{8}t_4,\\ q_7&= t_1t_2t_3^{7}t_4^{14}+ t_1t_2^{7}t_3t_4^{14}+ t_1t_2^{7}t_3^{14}t_4+ t_1^{7}t_2t_3t_4^{14}+ t_1^{7}t_2t_3^{14}t_4+ t_1t_2^{3}t_3^{13}t_4^{6}+ \medskip t_1^{3}t_2t_3^{13}t_4^{6}\\ &\quad+ t_1^{3}t_2^{13}t_3t_4^{6}+ t_1^{3}t_2^{13}t_3^{6}t_4+ t_1^{7}t_2^{11}t_3t_4^{4}+ \medskip t_1^{7}t_2^{11}t_3^{4}t_4,\\ q_8&= t_1t_2^{3}t_3^7t_4^{12}+ t_1t_2^{6}t_3^{7}t_4^9+ \medskip t_1^{3}t_2^{4}t_3^{7}t_4^{9},\\ q_9&= t_1^{7}t_2^{3}t_3^{9}t_4^{4}+ t_1t_2^{7}t_3^{7}t_4^{8}+ t_1^{7}t_2t_3^{7}t_4^{8}+ t_1t_2^{7}t_3^{6}t_4^{9}+ t_1^{7}t_2t_3^{6}t_4^{9}+ t_1^{3}t_2^{7}t_3^{4}t_4^{9}+ \medskip t_1^{7}t_2^{3}t_3^{4}t_4^{9},\\ q_{10}&= t_1t_2^{3}t_3^{7}t_4^{12}+ t_1^{3}t_2^{4}t_3^{11}t_4^{5}+ \medskip t_1t_2^{6}t_3^{11}t_4^{5},\\ q_{11}&= t_1t_2^{3}t_3^{6}t_4^{13}+ t_1t_2^{6}t_3^{3}t_4^{13}+ t_1^{3}t_2t_3^{7}t_4^{12}+ t_1^{3}t_2^{3}t_3^{4}t_4^{13}+ t_1^{3}t_2^{3}t_3^{13}t_4^{4}+ t_1^{3}t_2^{4}t_3^{3}t_4^{13}. \end{array}$$ Let $[w]\in [QP^{\otimes 4}_{n_{1,\,2,\, 1}}]^{GL_4}$ be arbitrary. Then, we can express $w$ as $w\equiv \sum_{1\leq i\leq 8}\gamma_i\widehat{p_i},$ where $\gamma_i\in\mathbb F_2$ for every $i.$ Utilizing the homomorphism $\sigma_4: P^{\otimes 4}\longrightarrow P^{\otimes 4}$ and the relation $\sigma_4(w) +w \equiv 0,$ we obtain $$ \begin{array}{ll} \medskip \sigma_4(w) +w &\equiv \gamma_1t_2t_3^{7}t_4^{15} + (\gamma_1+\gamma_2)t_1t_2^{7}t_4^{15} + (\gamma_1+\gamma_3)t_1t_2^{15}t_4^{7} + (\gamma_3+\gamma_5+\gamma_6)t_2^7t_3^{7}t_4^{9}\\ \medskip &\quad + (\gamma_4+\gamma_7)t_1t_2^{15}t_3t_4^{6} + (\gamma_1+\gamma_6+\gamma_7)t_1t_2^{7}t_3t_4^{14}\\ &\quad + (\gamma_1+\gamma_3+\gamma_6 + \gamma_8)t_1t_2^{3}t_3^5t_4^{14} + \ \mbox{other terms} \equiv 0, \end{array}$$ which leads to $\gamma_1 = \gamma_2 = \gamma_3 = 0$ and $\gamma_4 = \gamma_5 = \gamma_6 = \gamma_7 = \gamma_8.$ Let us consider the following element in $H_*(BV_4)$: $$\zeta_{1,\,2,\, 1} = a_1^{(15)}a_2^{(3)}a_3^{(3)}a_4^{(2)} + a_1^{(15)}a_2^{(3)}a_3^{(4)}a_4^{(1)}+ a_1^{(15)}a_2^{(5)}a_3^{(2)}a_4^{(1)} + a_1^{(15)}a_2^{(6)}a_3^{(1)}a_4^{(1)}.$$ Upon performing a straightforward computation, it can be shown that $ (\zeta_{1,\,2,\, 1})Sq^{2^k}= 0$ for all $k\geq 2$ and $$\begin{array}{ll} \medskip (a_1^{(15)}a_2^{(3)}a_3^{(3)}a_4^{(2)})Sq^{1}&=(a_1^{(15)}a_2^{(3)}a_3^{(4)}a_4^{(1)})Sq^{1}= a_1^{(15)}a_2^{(3)}a_3^{(3)}a_4^{(1)},\\ \medskip (a_1^{(15)}a_2^{(5)}a_3^{(2)}a_4^{(1)} )Sq^{1}&=(a_1^{(15)}a_2^{(6)}a_3^{(1)}a_4^{(1)})Sq^{1}=a_1^{(15)}a_2^{(5)}a_3^{(1)}a_4^{(1)},\\ \medskip (a_1^{(15)}a_2^{(3)}a_3^{(3)}a_4^{(2)})Sq^{2}&=(a_1^{(15)}a_2^{(6)}a_3^{(1)}a_4^{(1)})Sq^2 = 0,\\ (a_1^{(15)}a_2^{(3)}a_3^{(4)}a_4^{(1)})Sq^{2}&=(a_1^{(15)}a_2^{(5)}a_3^{(2)}a_4^{(1)})Sq^2 = a_1^{(15)}a_2^{(3)}a_3^{(2)}a_4^{(1)}. \end{array}$$ So, $\zeta_{1,\,2,\, 1}$ is an $\overline{\mathcal A}$-annihilated element. (In reality, due to the unstable condition, we only need to consider the effects of the Steenrod operations $Sq^{2^{k}}$ for $k = 0,\, 1.$) Furthermore, it is straightforward to check that $<\zeta_{1,\,2,\, 1},\, \sum_{4\leq i\leq 8}\widehat{p_i}> = 1,$ where $<- ,- >$ denotes the canonical non-singular pairing $H_*(BV_4)\times H^*(BV_4)\longrightarrow \mathbb F_2.$ Therefore, it may be concluded that $(\mathbb F_2\otimes_{GL_4}{\rm Ann}_{\overline{\mathcal A}}[P^{\otimes 4}]^{*})_{n_{1,\,2,\, 1}}$ is 1-dimensional and $(\mathbb F_2\otimes_{GL_4}{\rm Ann}_{\overline{\mathcal A}}[P^{\otimes 4}]^{*})_{n_{1,\,2,\, 1}} = \mathbb F_2[\zeta_{1,\,2,\, 1}].$ \medskip Now, according to Lin \cite{Lin}, the $\mathbb F_2$-cohomology ${\rm Ext}_{\mathcal A}^{4, *}(\mathbb F_2, \mathbb F_2)$ is generated by $$h_ih_jh_{\ell}h_m,\, h_uc_v,\, d_t,\, e_t,\, f_t,\, g_{t+1},\, p_t,\, D_3(t),\, p'_t$$ for $m\geq \ell\geq j\geq i\geq 0$, $u,\ v,\ t\geq 0$, and subject to the relations $h_ih_{i+1} = 0,\ h_ih_{i+2}^{2} = 0,\, h_i^3 = h^2_{i-1}h_{i+1},\, h^2_ih^2_{i+3} = 0,\, h_{v-1}c_v = 0,\, h_{v}c_v = 0,\, h_{v+2}c_v = 0$ and $h_{v+3}c_v = 0.$ Then, one gets $${\rm Ext}_{\mathcal A}^{4, 4+n_{k,\, s,\, u}}(\mathbb F_2, \mathbb F_2) = \left\{\begin{array}{ll} \langle h_0c_k \rangle&\mbox{if $s = 1$, $u = 2$, and $k\geq 2$},\\ \langle h_{u+3}c_0 \rangle&\mbox{if $s = 2$, $u\geq 1$, and $k = 1$},\\ \langle h_0h_kh_{k+s}h_{k+s+u} \rangle&\mbox{if $s \geq 2$, $u\geq 2$, and $k\geq 2$},\\ 0&\mbox{otherwise}. \end{array}\right. $$ Therefrom Part $(II)$ can be obtained from the aforementioned calculations, in combination with the following known facts: the indecomposable elements $h_j$ and $c_j$ belong respectively to the image of $Tr_1^{\mathcal A}$ and $Tr_3^{\mathcal A},$ and the total transfer $\{Tr_h^{\mathcal A}\}_{h\geq 0}: \{\mathbb F_2\otimes_{GL_h}{\rm Ann}_{\overline{\mathcal A}}[P^{\otimes h}]^{*}\}_{h\geq 0}\longrightarrow \{{\rm Ext}_{\mathcal A}^{h, *}(\mathbb F_2, \mathbb F_2)\}_{h\geq 0}$ of algebras detects the subalgebra generated by the $Sq^{0}$-family $\{h_j\in {\rm Ext}_{\mathcal A}^{1, 2^{j}}(\mathbb F_2, \mathbb F_2) \}_{j\geq 0}.$ The theorem is proved. \end{proof}
2412.02620v1
http://arxiv.org/abs/2412.02620v1
The Dimension of the Disguised Toric Locus of a Reaction Network
\documentclass[11pt]{article} \usepackage{amsmath,amsfonts,amssymb,amsthm} \usepackage{enumerate} \usepackage{xcolor} \usepackage{url} \usepackage{tcolorbox} \usepackage{hyperref} \usepackage{multicol, latexsym} \usepackage{latexsym} \usepackage{psfrag,import} \usepackage{verbatim} \usepackage{color} \usepackage{epsfig} \usepackage[outdir=./]{epstopdf} \usepackage{hyperref} \hypersetup{ colorlinks=true, linkcolor=blue, filecolor=magenta, urlcolor=cyan } \usepackage[title]{appendix} \usepackage{geometry} \usepackage{mathtools} \usepackage{enumerate} \usepackage{enumitem} \usepackage{multicol} \usepackage{booktabs} \usepackage{enumitem} \usepackage{parcolumns} \usepackage{thmtools} \usepackage{xr} \usepackage{epstopdf} \usepackage{mathrsfs} \usepackage{subcaption} \usepackage{soul} \usepackage{float} \parindent 1ex \parskip1ex \usepackage{comment} \usepackage{authblk} \usepackage{setspace} \usepackage{cleveref} \theoremstyle{plain} \newtheorem{theorem}{Theorem}[section] \newtheorem{corollary}[theorem]{Corollary} \newtheorem{proposition}[theorem]{Proposition} \newtheorem{lemma}[theorem]{Lemma} \newtheorem{conjecture}[theorem]{Conjecture} \newtheorem{question}[theorem]{Question} \theoremstyle{definition} \newtheorem{definition}[theorem]{Definition} \newtheorem{example}[theorem]{Example} \newtheorem{notation}[theorem]{Notation} \newtheorem{remark}[theorem]{Remark} \theoremstyle{remark} \newtheorem{claim}{Claim} \numberwithin{equation}{section} \parskip=0pt plus 1pt \setlength{\parindent}{20pt} \newcommand\RR{\mathbb{R}} \newcommand\GG{\mathcal{G}} \newcommand\bla{\boldsymbol{\lambda}} \newcommand\by{\boldsymbol{y}} \newcommand\bypi{\boldsymbol{y'_i}} \newcommand\byi{\boldsymbol{y_i}} \newcommand\bypj{\boldsymbol{y'_j}} \newcommand\byj{\boldsymbol{y_j}} \newcommand\be{\boldsymbol{e}} \newcommand\bep{\boldsymbol{\varepsilon}} \newcommand\bc{\boldsymbol{c}} \renewcommand\bf{\boldsymbol{f}} \newcommand\bh{\boldsymbol{h}} \newcommand\bk{\boldsymbol{k}} \newcommand\bw{\boldsymbol{w}} \newcommand\bb{\boldsymbol{b}} \newcommand\bW{\boldsymbol{W}} \newcommand\bu{\boldsymbol{u}} \newcommand\bg{\boldsymbol{g}} \newcommand\bx{\boldsymbol{x}} \newcommand\bv{\boldsymbol{v}} \newcommand\bz{\boldsymbol{z}} \newcommand\bY{\boldsymbol{Y}} \newcommand\bA{\boldsymbol{A}} \newcommand\bB{\boldsymbol{B}} \newcommand\bC{\boldsymbol{C}} \newcommand\bF{\boldsymbol{F}} \newcommand\bG{\boldsymbol{G}} \newcommand\bH{\boldsymbol{H}} \newcommand\bI{\boldsymbol{I}} \newcommand\bq{\boldsymbol{q}} \newcommand\bp{\boldsymbol{p}} \newcommand\br{\boldsymbol{r}} \newcommand\bJ{\boldsymbol{J}} \newcommand\bj{\boldsymbol{j}} \newcommand\hbJ{\hat{\boldsymbol{J}}} \newcommand{\mK}{\mathcal{K}} \newcommand{\dK}{\mathcal{K}_{\RR\text{-disg}}} \newcommand{\pK}{\mathcal{K}_{\text{disg}}} \newcommand{\mJ}{\mathcal{J}_{\RR}} \newcommand{\eJ}{\mathcal{J}_{\textbf{0}}} \newcommand{\mD}{\mathcal{D}_{\textbf{0}}} \newcommand{\mS}{\mathcal{S}} \newcommand{\mSG}{\mathcal{S}_G} \newcommand{\hPsi}{\hat{\Psi}} \newcommand{\hbx}{\hat{\bx}} \newcommand{\hbk}{\hat{\bk}} \newcommand{\hbp}{\hat{\bp}} \newcommand{\hbq}{\hat{\bq}} \newcommand{\hmJ}{\hat{\mJ}} \newcommand\bd{\boldsymbol{d}} \newcommand{\defi}{\textbf} \DeclareMathOperator{\spn}{span} \begin{document} \title{ The Dimension of the Disguised Toric Locus of a Reaction Network } \author[1]{ Gheorghe Craciun } \author[2]{ Abhishek Deshpande } \author[3]{ Jiaxin Jin } \affil[1]{\small Department of Mathematics and Department of Biomolecular Chemistry, University of Wisconsin-Madison} \affil[2]{Center for Computational Natural Sciences and Bioinformatics, \protect \\ International Institute of Information Technology Hyderabad} \affil[3]{\small Department of Mathematics, University of Louisiana at Lafayette} \date{} \maketitle \begin{abstract} Under mass-action kinetics, complex-balanced systems emerge from biochemical reaction networks and exhibit stable and predictable dynamics. For a reaction network $G$, the associated dynamical system is called \emph{disguised toric} if it can yield a complex-balanced realization on a possibly different network $G_1$. This concept extends the robust properties of toric systems to those that are not inherently toric. In this work, we study the \emph{disguised toric locus} of a reaction network — i.e., the set of positive rate constants that make the corresponding mass-action system disguised toric. Our primary focus is to compute the exact dimension of this locus. We subsequently apply our results to Thomas-type and circadian clock models. \end{abstract} \begin{NoHyper} \tableofcontents \end{NoHyper} \section{Introduction} Mathematical models of biochemical interaction networks can generally be described by {\em polynomial dynamical systems}. These dynamical systems are ubiquitous in models of biochemical reaction networks, infectious diseases, and population dynamics~\cite{craciun2022homeostasis,deshpande2014autocatalysis}. However, analyzing these systems is a challenging problem in general. Classical nonlinear dynamical properties like multistability, oscillations, or chaotic dynamics are difficult to examine~\cite{Ilyashenko2002, yu2018mathematical}. Studying the dynamical properties of reaction networks is crucial for understanding the behavior of chemical and biological systems. In this paper, we will focus on a class of dynamical systems generated by reaction networks called {\em complex-balanced systems} (also known as {\em toric dynamical systems}~\cite{CraciunDickensteinShiuSturmfels2009} owing to their connection with toric varieties~\cite{dickenstein2020algebraic}). Complex-balanced systems are known to exhibit remarkably robust dynamics, which {\em rules out} multistability, oscillations, and even chaotic dynamics~\cite{horn1972general}. More specifically, there exists a strictly convex Lyapunov function, which implies that all positive steady states are locally asymptotically stable~\cite{horn1972general, yu2018mathematical}. In addition, a unique positive steady state exists within each affine invariant polyhedron. They are also related to the \emph{Global Attractor Conjecture}~\cite{CraciunDickensteinShiuSturmfels2009} which states that complex-balanced dynamical systems have a globally attracting steady state within each stoichiometric compatibility class. Several special cases of this conjecture have been proved~\cite{anderson2011proof,gopalkrishnan2014geometric, pantea2012persistence, craciun2013persistence, boros2020permanence}, and a proof in full generality has been proposed in~\cite{craciun2015toric}. An important limitation of the classical theory of complex-balanced systems is that to be applicable for a large set of parameter values (i.e., choices of reaction rate constants) the reaction network under consideration must satisfy two special properties: {\em weak reversibility} and {\em low deficiency} (see \cite{yu2018mathematical} for definitions). Our focus here will be on understanding how one can take advantage of the notion of {\em dynamical equivalence} in order to greatly relax both of these restrictions. Dynamical equivalence relies on the fact that two different reaction networks can generate the same dynamics for well-chosen parameter values. This phenomenon has also been called \emph{macro-equivalence}~\cite{horn1972general} or {\em confoundability}~\cite{craciun2008identifiability}. Recently, this phenomenon has found applications in the design of efficient algorithms for finding weakly reversible single linkage class and weakly reversible deficiency one realizations~\cite{WR_df_1, WR_DEF_THM}. Moreover, it has also been used to show the existence of infinitely many positive states for weakly reversible and endotactic dynamical systems~\cite{boros2020weakly,kothari2024endotactic}. More recently, it has been used to generate the necessary and sufficient conditions for the existence of realizations using weakly reversible dynamical systems~\cite{kothari2024realizations}. In this paper, we consider the notion of a disguised toric locus for a given reaction network $G$. The \emph{disguised toric locus} is the set of positive reaction rate vectors in $G$ for which the corresponding dynamical system can be realized as a complex-balanced system by a network $G_1$. In other words, this locus consists of positive reaction rate vectors $\bk$ such that the mass-action system $(G, \bk)$ is dynamically equivalent to a complex-balanced system $(G_1, \bk_1)$. Additionally, if the rate constants are allowed to take any real values, we refer to the set of reaction rate vectors in $G$ that satisfy this property as the \emph{$\mathbb{R}$-disguised toric locus} of $G$. The concept of a disguised toric locus was first introduced in \cite{2022disguised}. Since then, several general properties of both the disguised toric locus and the $\mathbb{R}$-disguised toric locus have been established. For example, it was demonstrated in \cite{haque2022disguised} that the disguised toric locus is invariant under invertible affine transformations of the network. Furthermore, \cite{disg_1} showed that both loci are path-connected, and \cite{disg_2} provided a lower bound on the dimension of the $\mathbb{R}$-disguised toric locus. Consider for example the Thomas-type model (E-graph $G$) shown in Figure \ref{fig:thomas_model_intro}. \begin{figure}[!ht] \centering \includegraphics[scale=0.7]{thomas_model.eps} \caption{ (a) The E-graph $G$ represents a Thomas-type model, with all edges labeled by the reaction rate constants $\bk$. (b) The E-graph $G_1$ is weakly reversible, with all edges labeled by the reaction rate constants $\bk_1$. The mass-action system $(G_1, \bk_1)$ is complex-balanced. } \label{fig:thomas_model_intro} \end{figure} Since $G$ is not weakly reversible, the system $(G, \bk)$ is not complex-balanced, so classical complex-balanced theory offers limited insight into the dynamics of $(G, \bk)$. However, by direct computation, $(G, \bk)$ is dynamically equivalent to the complex-balanced system $(G_1, \bk_1)$, which enables us to deduce its dynamical properties. Thus, $\bk$ can be viewed as a “good” reaction rate vector for $G$. The disguised toric locus of $G$ consists of such reaction rate vectors $\bk$. In this paper, we develop a general framework to compute the exact dimensions of both the disguised toric locus and the $\mathbb{R}$-disguised toric locus of a reaction network. Building on \cite{disg_2}, we construct a mapping on the $\mathbb{R}$-disguised toric locus of $G$ and show that this mapping is a homeomorphism, allowing us to determine the dimensions of both the disguised toric locus and the $\mathbb{R}$-disguised toric locus. When applied to Figure \ref{fig:thomas_model_intro}, the disguised toric locus of $G$ is shown to be full-dimensional, significantly larger than its toric locus, which is empty (see details in Example \ref{ex:thomas}). \bigskip \textbf{Structure of the paper.} In Section~\ref{sec:reaction_networks}, we introduce the basic terminology of reaction networks. Section~\ref{sec:flux_systems} presents flux systems and analyzes their properties. In Section~\ref{sec:disguised_locus}, we recall the key concepts of the toric locus, the $\RR$-disguised toric locus, and the disguised toric locus. Section~\ref{sec:map} constructs a continuous bijective map $\hPsi$ connecting the $\RR$-disguised toric locus to a specific flux system. In Section~\ref{sec:continuity}, we first establish key lemmas \ref{lem:key_1} - \ref{lem:key_4} and then use them to prove that $\hPsi$ is a homeomorphism in Theorem \ref{thm:hpsi_homeo}. Section~\ref{sec:dimension} leverages this homeomorphism to establish precise bounds on the dimension of the disguised toric locus and the $\RR$-disguised toric locus, as shown in Theorem~\ref{thm:dim_kisg_main}. In Section~\ref{sec:applications}, we apply our results to Thomas-type models and circadian clock models, showing both disguised toric loci are full-dimensional. Finally, Section~\ref{sec:discussion} summarizes our findings and outlines potential directions for future research. \bigskip \textbf{Notation.} We let $\mathbb{R}_{\geq 0}^n$ and $\mathbb{R}_{>0}^n$ denote the set of vectors in $\mathbb{R}^n$ with non-negative entries and positive entries respectively. For vectors $\bx = (\bx_1, \ldots, \bx_n)^{\intercal}\in \RR^n_{>0}$ and $\by = (\by_1, \ldots, \by_n)^{\intercal} \in \RR^n$, we define: \begin{equation} \notag \bx^{\by} = \bx_1^{y_{1}} \ldots \bx_n^{y_{n}}. \end{equation} For any two vectors $\bx, \by \in \RR^n$, we define $\langle \bx, \by \rangle = \sum\limits^{n}_{i=1} x_i y_i$. For E-graphs (see Definition \ref{def:e-graph}), we always let $G, G'$ denote arbitrary E-graphs, and let $G_1$ denote a weakly reversible E-graph. \section{Reaction networks} \label{sec:reaction_networks} We start with the introduction of the concept of a {\em reaction network} as a directed graph in Euclidean space called {\em E-graph}, and describe some of its properties. \begin{definition}[\cite{craciun2015toric, craciun2019polynomial,craciun2020endotactic}] \label{def:e-graph} \begin{enumerate}[label=(\alph*)] \item A \textbf{reaction network} $G=(V,E)$ is a directed graph, also called a \textbf{Euclidean embedded graph} (or \textbf{E-graph}), such that $V \subset \mathbb{R}^n$ is a finite set of \textbf{vertices} and the set $E\subseteq V\times V$ represents a finite set of \textbf{edges}. We assume that there are neither self-loops nor isolated vertices in $G=(V, E)$. \item A directed edge $(\by,\by')\in E$ connecting two vertices $\by, \by' \in V$ is denoted by $\by \rightarrow \by' \in E$ and represents a reaction in the network. Here $\by$ is called the \textbf{source vertex}, and $\by'$ is called the \textbf{target vertex}. Further, the difference vector $\by' - \by \in\mathbb{R}^n$ is called the \textbf{reaction vector}. \end{enumerate} \end{definition} \begin{definition} Consider an E-graph $G=(V,E)$. Then \begin{enumerate}[label=(\alph*)] \item $G$ is \textbf{weakly reversible}, if every reaction in $G$ is part of an oriented cycle. \item $G$ is a \textbf{(directed) complete} graph, if $\by\rightarrow \by'\in E$ for every two distinct vertices $\by, \by'\in V$. \item An E -graph $G' = (V', E')$ is a \textbf{subgraph} of $G$ (denoted by $G' \subseteq G$), if $V' \subseteq V$ and $E' \subseteq E$. In addition, we let $G' \sqsubseteq G$ denote that $G'$ is a weakly reversible subgraph of $G$. \item We denote the \defi{complete graph on $G$} by $G_c$, which is obtained by connecting every pair of source vertices in $V$. One can check that $G_c$ is weakly reversible and $G \subseteq G_c$. \end{enumerate} \end{definition} \begin{figure}[!ht] \centering \includegraphics[scale=0.4]{euclidean_embedded_graph.eps} \caption{\small (a) An E-graph with two reactions. The stoichiometric subspace corresponding to this graph is $\RR^2$. (b) A weakly reversible E-graph. (c) A directed complete E-graph with three vertices. Note that the E-graph in (b) is a weakly reversible subgraph of the E-graph in (c).} \label{fig:e-graph} \end{figure} \begin{definition}[\cite{adleman2014mathematics,guldberg1864studies,voit2015150,gunawardena2003chemical,yu2018mathematical,feinberg1979lectures}] Consider an E-graph $G=(V,E)$. Let $k_{\by\to \by'}$ denote the \textbf{reaction rate constant} corresponding to the reaction $\by\to \by'\in E$. Further, we let ${\bk} :=(k_{\by\to \by'})_{\by\to \by' \in E} \in \mathbb{R}_{>0}^{E}$ denote the \textbf{vector of reaction rate constants} (\textbf{reaction rate vector}). The \textbf{associated mass-action system} generated by $(G, \bk)$ on $\RR^n_{>0}$ is given by \begin{equation} \label{def:mas_ds} \frac{d\bx}{dt} = \displaystyle\sum_{\by \rightarrow \by' \in E}k_{\by\rightarrow\by'}{\bx}^{\by}(\by'-\by). \end{equation} We denote the \defi{stoichiometric subspace} of $G$ by $\mathcal{S}_G$, which is \begin{equation} \notag \mathcal{S}_G = \spn \{ \by' - \by: \by \rightarrow \by' \in E \}. \end{equation} \cite{sontag2001structure} shows that if $V \subset \mathbb{Z}_{\geq 0}^n$, the positive orthant $\mathbb{R}_{>0}^n$ is forward-invariant under system \eqref{def:mas_ds}. Any solution to \eqref{def:mas_ds} with initial condition $\bx_0 \in \mathbb{R}_{>0}^n$ and $V \subset \mathbb{Z}_{\geq 0}^n$, is confined to $(\bx_0 + \mathcal{S}_G) \cap \mathbb{R}_{>0}^n$. Thus, the set $(\bx_0 + \mathcal{S}_G) \cap \mathbb{R}_{>0}^n$ is called the \textbf{invariant polyhedron} of $\bx_0$. \end{definition} \begin{definition} Let $(G, \bk)$ be a mass-action system. \begin{enumerate}[label=(\alph*)] \item A point $\bx^* \in \mathbb{R}^n_{>0}$ is called a \defi{positive steady state} of the system if \begin{equation} \label{eq:steady_statez} \displaystyle\sum_{\by\rightarrow \by' \in E } k_{\by\rightarrow\by'}{(\bx^*)}^{\by}(\by'-\by)=0. \end{equation} \item A point $\bx^* \in \mathbb{R}^n_{>0}$ is called a \defi{complex-balanced steady state} of the system if for every vertex $\by_0 \in V$, \begin{eqnarray} \notag \sum_{\by_0 \rightarrow \by \in E} k_{\by_0 \rightarrow \by} {(\bx^*)}^{\by_0} = \sum_{\by' \rightarrow \by_0 \in E} k_{\by' \rightarrow \by_0} {(\bx^*)}^{\by'}. \end{eqnarray} Further, if the mass-action system $(G, \bk)$ admits a complex-balanced steady state, then it is called a \defi{complex-balanced (dynamical) system} or \defi{toric dynamical system}. \end{enumerate} \end{definition} \begin{remark} \label{rmk:complex_balance_property} Complex-balanced systems are known to exhibit robust dynamical properties. As mentioned in the introduction, they are connected to the \emph{Global Attractor Conjecture}, which proposes that complex-balanced systems possess a globally attracting steady state within each stoichiometric compatibility class. Several important special cases of this conjecture and related open problems have been proven. In particular, it has been shown that complex-balanced systems consisting of a single linkage class admit a globally attracting steady state \cite{anderson2011proof}. Additionally, two- and three-dimensional endotactic networks are known to be permanent \cite{craciun2013persistence}. Strongly endotactic networks have also been proven to be permanent \cite{gopalkrishnan2014geometric}. Furthermore, complex-balanced systems that are permanent always admit a globally attracting steady state \cite{yu2018mathematical}. \end{remark} \begin{theorem}[\cite{horn1972general}] \label{thm:cb} Consider a complex-balanced system $(G, \bk)$. Then \begin{enumerate} \item[(a)] The E-graph $G = (V, E)$ is weakly reversible. \item[(b)] Every positive steady state is a complex-balanced steady state. Given any $\bx_0 \in \mathbb{R}_{>0}^n$, there is exactly one steady state within the invariant polyhedron $(\bx_0 + \mathcal{S}_G) \cap \mathbb{R}_{>0}^n$. \end{enumerate} \end{theorem} \begin{theorem}[\cite{johnston2012topics}] \label{thm:jacobian} Consider a weakly reversible E-graph $G = (V, E)$ with the stoichiometric subspace $\mS_G$. Suppose $(G, \bk)$ is a complex-balanced system given by \begin{equation} \label{eq:jacobian} \frac{\mathrm{d} \bx}{\mathrm{d} t} = \bf (\bx) = \displaystyle\sum_{\by\rightarrow \by' \in E} k_{\by\rightarrow\by'}{\bx}^{\by}(\by'-\by). \end{equation} For any steady state $\bx^* \in \RR^n_{>0}$ of the system \eqref{eq:jacobian}, then \begin{equation} \label{eq:jacobian_ker} \Big( \ker \big( \mathbf{J}_{\bf} |_{\bx = \bx^*} \big) \Big)^{\perp} = \mS_G, \end{equation} where $\mathbf{J}_{\bf}$ represents the Jacobian matrix of $\bf (\bx)$. \end{theorem} \begin{definition} \label{def:de} Consider two mass-action systems $(G,\bk)$ and $(G',\bk')$. Then $(G,\bk)$ and $(G',\bk')$ are said to be \defi{dynamically equivalent} if for every vertex\footnote{ Note that when $\by_0 \not\in V$ or $\by_0 \not\in V'$, the corresponding side is considered as an empty sum} $\by_0 \in V \cup V'$, \begin{eqnarray} \notag \displaystyle\sum_{\by_0 \rightarrow \by\in E} k_{\by_0 \rightarrow \by} (\by - \by_0) = \displaystyle\sum_{\by_0 \rightarrow \by'\in E'} k'_{\by_0 \rightarrow\by'} (\by' - \by_0). \end{eqnarray} We let $(G,\bk)\sim (G', \bk')$ denote that two mass-action systems $(G,\bk)$ and $(G',\bk')$ are dynamically equivalent. \end{definition} \begin{remark}[\cite{horn1972general,craciun2008identifiability,deshpande2022source}] \label{rmk:de_ss} Following Definition \ref{def:de}, two mass-action systems $(G, \bk)$ and $(G', \bk')$ are dynamically equivalent if and only if for all $\bx \in \RR_{>0}^{n}$, \begin{equation} \label{eq:eqDE} \sum_{\by_i \to \by_j \in E} k_{\by_i \to \by_j} \bx^{\by_i} (\by_j - \by_i) = \sum_{\by'_i \to \by'_j \in E'} k'_{\by'_i \to \by'_j} \bx^{\by'_i} (\by'_j - \by'_i), \end{equation} and thus two dynamically equivalent systems share the same set of steady states. \end{remark} \begin{definition} \label{def:d0} Consider an E-graph $G=(V, E)$. Let $\bla = (\lambda_{\by \to \by'})_{\by \to \by' \in E} \in \RR^{|E|}$. The set $\mD(G)$ is defined as \begin{equation} \notag \mD (G):= \{\bla \in \RR^{|E|} \, \Big| \, \sum_{\by_0 \to \by \in E} \lambda_{\by_0 \to \by} (\by - \by_0) = \mathbf{0} \ \text{for every vertex } \by_0 \in V \}. \end{equation} We can check that $\mD (G)$ is a linear subspace of $\RR^E$. \end{definition} \begin{lemma}[\cite{disg_2}] \label{lem:d0} Consider two mass-action systems $(G, \bk)$ and $(G, \bk')$. Then $\bk' - \bk \in \mD (G)$ if and only if $(G, \bk) \sim (G, \bk')$. \end{lemma} \section{Flux systems} \label{sec:flux_systems} Due to the non-linearity of the dynamical systems, we now introduce linear systems arising from the network structure: the flux systems, and the complex-balanced flux systems, and study their properties. \begin{definition} Consider an E-graph $G=(V, E)$. Then \begin{enumerate}[label=(\alph*)] \item Let $J_{\by \to \by'} > 0$ denote the \textbf{flux} corresponding to the edge $\by \to \by'\in E$. Further, we let $\bJ = (J_{\by \to \by'})_{\by \to \by' \in E} \in \RR_{>0}^E$ denote the \textbf{flux vector} corresponding to the E-graph $G$. The \defi{associated flux system} generated by $(G, \bJ)$ is given by \begin{equation} \label{eq:flux} \frac{\mathrm{d} \bx}{\mathrm{d} t} = \sum_{\byi \to \byj \in E} J_{\byi \to \byj} (\byj - \byi). \end{equation} \item Consider two flux systems $(G,\bJ)$ and $(G', \bJ')$. Then $(G,\bJ)$ and $(G', \bJ')$ are said to be \defi{flux equivalent} if for every vertex\footnote{Note that when $\by_0 \not\in V$ or $\by_0 \not\in V'$, the corresponding side is considered as an empty sum} $\by_0 \in V \cup V'$, \begin{equation} \notag \sum_{\by_0 \to \by \in E} J_{\by_0 \to \by} (\by - \by_0) = \sum_{\by_0 \to \by' \in E'} J'_{\by_0 \to \by'} (\by' - \by_0). \end{equation} We let $(G, \bJ) \sim (G', \bJ')$ denote that two flux systems $(G, \bJ)$ and $(G', \bJ')$ are flux equivalent. \end{enumerate} \end{definition} \begin{definition} Let $(G,\bJ)$ be a flux system. A flux vector $\bJ \in \RR_{>0}^E$ is called a \defi{steady flux vector} to $G$ if \begin{equation} \notag \frac{\mathrm{d} \bx}{\mathrm{d} t} = \sum_{\byi \to \byj \in E} J_{\byi \to \byj} (\byj - \byi) = \mathbf{0}. \end{equation} A steady flux vector $\bJ\in \RR^{E}_{>0}$ is called a \defi{complex-balanced flux vector} to $G$ if for every vertex $\by_0 \in V$, \begin{eqnarray} \notag \sum_{ \by_0 \to \by \in E} J_{\by_0 \to \by} = \sum_{\by' \to \by_0 \in E} J_{\by' \to \by_0}, \end{eqnarray} and then $(G, \bJ)$ is called a \defi{complex-balanced flux system}. Further, let $\mathcal{J}(G)$ denote the set of all complex-balanced flux vectors to $G$ as follows: \begin{equation} \notag \mathcal{J}(G):= \{\bJ \in \RR_{>0}^{E} \mid \bJ \text{ is a complex-balanced flux vector to $G$} \}. \end{equation} \end{definition} \begin{definition} \label{def:j0} Consider an E-graph $G=(V, E)$. Let $\bJ = ({J}_{\byi \to \byj})_{\byi \to \byj \in E} \in \RR^E$. The set $\eJ (G)$ is defined as \begin{equation} \label{eq:J_0} \eJ (G): = \{{\bJ} \in \mD (G) \, \bigg| \, \sum_{\by \to \by_0 \in E} {J}_{\by \to \by_0} = \sum_{\by_0 \to \by' \in E} {J}_{\by_0 \to \by'} \ \text{for every vertex } \by_0 \in V \}. \end{equation} Note that $\eJ(G) \subset \mD (G)$ is a linear subspace of $\RR^E$. \end{definition} \begin{lemma}[\cite{disg_2}] \label{lem:j0} Let $(G, \bJ)$ and $(G, \bJ')$ be two flux systems. Then \begin{enumerate} \item[(a)] $(G, \bJ) \sim (G, \bJ')$ if and only if $\bJ' - \bJ \in \mD (G)$. \item[(b)] If $(G, \bJ)$ and $(G, \bJ')$ are both complex-balanced flux systems, then $(G, \bJ) \sim (G, \bJ')$ if and only if $\bJ' - \bJ \in \eJ(G)$. \end{enumerate} \end{lemma} \begin{proposition}[\cite{craciun2020efficient}] \label{prop:craciun2020efficient} Consider two mass-action systems $(G, \bk)$ and $(G', \bk')$. Let $\bx \in \RR_{>0}^n$. Define the flux vector $\bJ (\bx) = (J_{\by \to \by'})_{\by \to \by' \in E}$ on $G$, such that for every $\by \to \by' \in E$, \begin{equation} \notag J_{\by \to \by'} = k_{\by \to \by'} \bx^{\by}. \end{equation} Further, define the flux vector $\bJ' (\bx) = (J'_{\by \to \by'})_{\by \to \by' \in E'}$ on $G'$, such that for every $\by \to \by' \in E$, \begin{equation} \notag J'_{\by \to \by'} = k'_{\by \to \by'} \bx^{\by}. \end{equation} Then the following are equivalent: \begin{enumerate} \item[(a)] The mass-action systems $(G, \bk)$ and $(G', \bk')$ are dynamically equivalent. \item[(b)] The flux systems $(G, \bJ(\bx))$ and $(G', \bJ')$ are flux equivalent for all $\bx \in \RR_{>0}^n$. \item[(c)] The flux systems $(G, \bJ(\bx))$ and $(G', \bJ'(\bx))$ are flux equivalent for some $\bx \in \RR_{>0}^n$ \end{enumerate} \end{proposition} \section{Toric locus, disguised toric locus and \texorpdfstring{$\RR$}{R}-disguised toric locus} \label{sec:disguised_locus} In this section, we introduce the key concepts in this paper: the Toric locus, the Disguised toric locus, and the $\RR$-disguised toric locus. \begin{definition}[\cite{disg_2}] \label{def:mas_realizable} Let $G=(V, E)$ be an E-graph. Consider a dynamical system \begin{equation} \label{eq:realization_ode} \frac{\mathrm{d} \bx}{\mathrm{d} t} = \bf (\bx). \end{equation} It is said to be \defi{$\RR$-realizable} (or has a \defi{$\RR$-realization}) on $G$, if there exists some $\bk \in \mathbb{R}^{E}$ such that \begin{equation} \label{eq:realization} \bf (\bx) = \sum_{\by_i \rightarrow \by_j \in E}k_{\by_i \rightarrow \by_j} \bx^{\by_i}(\by_j - \by_i). \end{equation} Further, if $\bk \in \mathbb{R}^{E}_{>0}$ in \eqref{eq:realization}, the system \eqref{eq:realization_ode} is said to be \defi{realizable} (or has a \defi{realization}) on $G$. \end{definition} \begin{definition} Consider an E-graph $G=(V, E)$. \begin{enumerate} \item[(a)] Define the \defi{toric locus} of $G$ as \begin{equation} \notag \mK (G) := \{ \bk \in \mathbb{R}_{>0}^{E} \ \big| \ \text{the mass-action system generated by } (G, \bk) \ \text{is toric} \}. \end{equation} \item[(b)] Consider a dynamical system \begin{equation} \label{eq:def_cb_realization} \frac{\mathrm{d} \bx}{\mathrm{d} t} = \bf (\bx). \end{equation} It is said to be \defi{disguised toric} on $G$ if it is realizable on $G$ for some $\bk \in \mK (G)$. Further, we say the system \eqref{eq:def_cb_realization} has a \defi{complex-balanced realization} on $G$. \end{enumerate} \end{definition} \begin{definition} \label{def:de_realizable} Consider two E-graphs $G =(V,E)$ and $G' =(V', E')$. \begin{enumerate} \item[(a)] Define the set $\mK_{\RR}(G', G)$ as \begin{equation} \notag \mK_{\RR}(G', G) := \{ \bk' \in \mK (G') \ \big| \ \text{the mass-action system } (G', \bk' ) \ \text{is $\RR$-realizable on } G \}. \end{equation} \item[(b)] Define the set $\dK(G, G')$ as \begin{equation} \notag \dK(G, G') := \{ \bk \in \mathbb{R}^{E} \ \big| \ \text{the dynamical system} \ (G, \bk) \ \text{is disguised toric on } G' \}. \end{equation} Note that $\bk$ may have negative or zero components. \item[(c)] Define the \defi{$\RR$-disguised toric locus} of $G$ as \begin{equation} \notag \dK(G) := \displaystyle\bigcup_{G' \sqsubseteq G_{c}} \ \dK(G, G'). \end{equation} Note that in the above definition of $\RR$-disguised toric locus of $G$, we take a union over only those E-graphs which are weakly reversible subgraphs of $G_c$. This follows from a result in~\cite{craciun2020efficient} which asserts that if a dynamical system generated by $G$ has a complex-balanced realization using some graph $G_1$, then it also has a complex-balanced realization using $G'\sqsubseteq G_{c}$. \item[(d)] Define the set $\pK (G, G')$ as \begin{equation} \notag \pK (G, G') := \dK(G, G') \cap \mathbb{R}^{E}_{>0}. \end{equation} Further, define the \defi{disguised toric locus} of $G$ as \begin{equation} \notag \pK (G) := \displaystyle\bigcup_{G' \sqsubseteq G_{c}} \ \pK(G, G'). \end{equation} Similar to the $\RR$-disguised toric locus, it is sufficient for us to include those E-graphs which are weakly reversible subgraphs of $G_c$~\cite{craciun2020efficient}. \end{enumerate} \end{definition} \begin{lemma}[\cite{disg_2}] \label{lem:semi_algebaic} Let $G = (V, E)$ be an E-graph. \begin{enumerate} \item[(a)] Suppose that $G_1 = (V_1, E_1)$ is a weakly reversible E-graph, then $\dK(G,G_1)$ and $\pK(G,G_1)$ are semialgebraic sets. \item[(b)] Both $\dK(G)$ and $\pK(G)$ are semialgebraic sets. \end{enumerate} \end{lemma} \begin{proof} For part $(a)$, following Lemma 3.6 in \cite{disg_2}, we obtain that $\dK(G, G_1)$ is a semialgebraic set. The positive orthant is also a semialgebraic set since it can be defined by polynomial inequalities on all components. Since finite intersections of semialgebraic sets are semialgebraic sets, together with Definition \ref{def:de_realizable}, we conclude that $\pK(G, G_1)$ is a semialgebraic set. \smallskip For part $(b)$, since finite unions of semialgebraic sets are semialgebraic sets~\cite{coste2000introduction}, together with Definition \ref{def:de_realizable} and part $(a)$, we conclude that $\dK(G)$ and $\pK(G)$ are semialgebraic sets. \end{proof} \begin{remark}[\cite{lee2010introduction}] \label{rmk:semi_algebaic} From Lemma \ref{lem:semi_algebaic} and \cite{lee2010introduction}, on a dense open subset of any semialgebraic set $\dK(G, G_1)$ or $\pK(G, G_1)$, it is locally a \textbf{submanifold}. The dimension of $\dK(G, G_1)$ or $\pK(G, G_1)$ can be defined to be the largest dimension at points at which it is a submanifold. \end{remark} \begin{remark} \label{rmk:mJ_dK} Let $G_1 = (V_1, E_1)$ be a weakly reversible E-graph and let $G = (V, E)$ be an E-graph. From Definition \ref{def:de_realizable}, it follows that $\dK (G, G_1)$ is empty if and only if $\mK_{\RR} (G_1, G)$ is empty. \end{remark} Analogous to the $\RR$-disguised toric locus, we also introduce the $\RR$-realizable complex-balanced flux system, which plays a crucial role in the rest of the paper. \begin{definition} \label{def:flux_realizable} Consider a flux system $(G', \bJ')$. It is said to be \defi{$\RR$-realizable} on $G$ if there exists some $\bJ \in \mathbb{R}^{E}$, such that for every vertex\footnote{Note that when $\by_0 \not\in V$ or $\by_0 \not\in V'$, the corresponding side is considered as an empty sum} $\by_0 \in V \cup V'$, \begin{equation} \notag \sum_{\by_0 \to \by \in E} J_{\by_0 \to \by} (\by - \by_0) = \sum_{\by_0 \to \by' \in E'} J'_{\by_0 \to \by'} (\by' - \by_0). \end{equation} Further, define the set $\mJ (G', G)$ as \begin{equation} \notag \mJ (G', G) := \{ \bJ' \in \mathcal{J} (G') \ \big| \ \text{the flux system } (G', \bJ') \ \text{is $\RR$-realizable on } G \}. \end{equation} Proposition \ref{prop:craciun2020efficient} implies that $\dK (G, G')$ is empty if and only if $\mJ(G', G)$ is empty. \end{definition} \begin{lemma}[{\cite[Lemma 2.33]{disg_2}}] \label{lem:j_g1_g_cone} Consider a weakly reversible E-graph $G_1 = (V_1, E_1)$ and let $G = (V, E)$ be an E-graph. Then we have the following: \begin{enumerate} \item[(a)] There exists a vectors $\{ \bv_1, \bv_2, \ldots, \bv_k \} \subset \RR^{|E_1|}$, such that \begin{equation} \label{j_g1_g_generator} \mJ (G_1, G) = \{ a_1 \bv_1 + \cdots a_k \bv_k \ | \ a_i \in \RR_{>0}, \bv_i \in \RR^{|E_1|} \}. \end{equation} \item[(b)] $\dim (\mJ (G_1, G)) = \dim ( \spn \{ \bv_1, \bv_2, \ldots, \bv_k \} )$. \item[(c)] If $\mJ (G_1, G) \neq \emptyset$, then \[ \eJ(G_1) \subseteq \spn \{ \bv_1, \bv_2, \ldots, \bv_k \}. \] \end{enumerate} \end{lemma} \section{The map \texorpdfstring{$\hPsi$}{hPsi}} \label{sec:map} The goal of this section is to study the properties of a map $\hat{\Psi}$ (see Definition \ref{def:hpsi}) that relates the sets $\dK(G, G_1)$ and $\hat{\mJ} (G_1, G)$ (see Equation \eqref{def:hat_j_g1_g}). In particular, we show the map $\hat{\Psi}$ is bijective and continuous. \paragraph{Notation.} We introduce the following notation that will be used for the entire section. Let $G = (V, E)$ be an E-graph. Let $b$ denote the dimension of the linear subspace $\mD(G)$, and denote an orthonormal basis of $\mD(G)$ by \[ \{\bB_1, \bB_2, \ldots, \bB_b\}. \] Further, we consider $G_1 = (V_1, E_1)$ to be a weakly reversible E-graph. Let $a$ denote the dimension of the linear subspace $\eJ(G_1)$, and denote an orthonormal basis of $\eJ(G_1)$ by \[ \{\bA_1, \bA_2, \ldots, \bA_a \}. \] \qed \medskip Recall the set $\mJ (G_1,G)$. Now we define the set $\hat{\mJ} (G_1,G) \subset \RR^{|E_1|}$ as \begin{equation} \label{def:hat_j_g1_g} \hat{\mJ} (G_1,G) = \{ \bJ + \sum\limits^a_{i=1} w_i \bA_i \ | \ \bJ \in \mJ (G_1,G), \text{ and } w_i \in \RR \text{ for } 1 \leq i \leq a \}. \end{equation} Further, we define the set $\hat{\mathcal{J}} (G_1) \subset \RR^{|E_1|}$ as \begin{equation} \label{def:hat_j_g1} \hat{\mathcal{J}} (G_1) = \{\bJ \in \RR^{E} \mid \sum_{\by \to \by_0 \in E} J_{\by \to \by_0} = \sum_{\by_0 \to \by' \in E} J_{\by_0 \to \by'} \text{ for every vertex $\by_0 \in V_1$}\}. \end{equation} \begin{remark} \label{rmk:hat_j_g1_g} Following~\eqref{def:hat_j_g1_g}, it is clear that $\mJ (G_1,G) \subset \hat{\mJ} (G_1,G)$. Further, from $\{\bA_i \}^{a}_{i=1} \in \eJ(G)$ and Lemma \ref{lem:j0}, we conclude that \[\hat{\mJ} (G_1,G) \cap \RR^{|E_1|}_{>0} = \mJ (G_1,G). \] Similarly, we have $\hat{\mathcal{J}} (G_1) \cap \RR^{|E_1|}_{>0} = \mathcal{J} (G_1)$. \end{remark} \begin{remark} Note that $\hat{\mathcal{J}} (G_1)$ is a linear subspace of $\RR^{|E_1|}$, while the sets $\hat{\mJ} (G_1,G)$, $\mJ (G_1,G)$ and $\mathcal{J} (G_1)$ are not linear subspaces. \end{remark} \begin{definition} \label{def:hpsi} Given a weakly reversible E-graph $G_1 = (V_1, E_1)$ with its stoichiometric subspace $\mS_{G_1}$. Consider an E-graph $G = (V, E)$ and $\bx_0\in\mathbb{R}^n_{>0}$, define the map \begin{equation} \label{eq:hpsi} \hPsi: \hat{\mJ} (G_1,G) \times [(\bx_0 + \mS_{G_1} )\cap\mathbb{R}^n_{>0}] \times \RR^b \rightarrow \dK(G,G_1) \times \RR^a, \end{equation} such that for $(\hat{\bJ}, \bx, \bp) \in \hat{\mJ} (G_1,G) \times [(\bx_0 + \mS_{G_1} )\cap\mathbb{R}^n_{>0}] \times \mathbb{R}^b$, \begin{equation} \notag \hat{\Psi} (\hat{\bJ},\bx, \bp) : = (\bk, \bq), \end{equation} where \begin{equation} \label{def:hpsi_k} (G, \bk) \sim (G_1, \hat{\bk}_1) \ \text{ with } \ \hat{k}_{1, \by\rightarrow \by'} = \frac{\hat{J}_{\by\rightarrow \by'}}{{\bx}^{\by}}, \end{equation} and \begin{equation} \label{def:hpsi_kq} \bp = ( \langle \bk, \bB_1 \rangle, \langle \bk, \bB_2 \rangle, \ldots, \langle \bk, \bB_b \rangle), \ \ \bq = ( \langle \hat{\bJ}, \bA_1 \rangle, \langle \hat{\bJ}, \bA_2 \rangle, \ldots, \langle \hat{\bJ}, \bA_a \rangle ). \end{equation} \end{definition} Recall Remark \ref{rmk:mJ_dK}, $\dK (G, G_1)$ is empty if and only if $\mJ(G_1, G)$ is empty. If $\mJ(G_1, G) = \dK (G, G_1) = \emptyset$, then the map $\hPsi$ is trivial. However, we are interested in the case when $\dK (G, G_1) \neq \emptyset$, therefore we assume both $\mJ(G_1, G)$ and $\dK (G, G_1)$ are non-empty sets in the rest of the paper. \begin{lemma} \label{lem:hpsi_well_def} The map $\hPsi$ in Definition \ref{def:hpsi} is well-defined. \end{lemma} \begin{proof} Consider any point $(\hbJ^*, \bx^*, \bp^*) \in \hat{\mJ} (G_1,G)\times [(\bx_0 + \mS_{G_1} )\cap\mathbb{R}^n_{>0}] \times \mathbb{R}^b$. From Equation\eqref{def:hat_j_g1_g}, there exist a $\bJ^* = (J^*_{\by\rightarrow \by'})_{\by\rightarrow \by' \in E_1} \in \mJ (G_1,G)$ and $w^*_i \in \RR$ for $1 \leq i \leq a$, such that \[ \hbJ^* = \bJ^* + \sum\limits^a_{i=1} w^*_i \bA_i. \] Since $\{ \bA_i \}^a_{i=1}$ is an orthonormal basis of the subspace $\eJ(G_1)$, we obtain \begin{equation} \label{eq:psi_wd_1} (G_1, \hbJ^*) \sim (G_1, \bJ^*). \end{equation} From $\bJ^* \in \mJ (G_1,G) \subset \bJ (G_1)$, set $\bk_1 = (k_{1, \by\rightarrow \by'})_{\by\rightarrow \by' \in E_1}$ with $k_{1, \by\rightarrow \by'} = \frac{J^*_{\by \rightarrow \by'} }{ (\bx^*)^{\by} }$. Then \begin{equation} \label{eq:psi_wd_2} \bk_1 \in \mK_{\RR} (G_1,G) \subset \mK(G_1). \end{equation} Moreover, $\bx^*$ is the complex-balanced steady state of $(G_1, \bk_1)$. Set $\hbk_1 = (\hat{k}_{1, \by\rightarrow \by'})_{\by\rightarrow \by' \in E_1}$ with $\hat{k}_{1, \by\rightarrow \by'} = \frac{\hat{J}^*_{\by \rightarrow \by'} }{ (\bx^*)^{\by} }$. From Equation\eqref{eq:psi_wd_1} and Proposition \ref{prop:craciun2020efficient}, we have \begin{equation} \label{eq:psi_wd_3} (G_1, \bk_1) \sim (G_1, \hat{\bk}_1). \end{equation} From Equation\eqref{eq:psi_wd_2}, there exists a $\bk \in \dK(G,G_1) \subset \RR^{|E|}$, such that $(G, \bk) \sim (G_1, \bk_1)$. Now suppose $\bp^* = (p^*_1, p^*_2, \ldots, p^*_b) \in \RR^b$, we construct the vector $\bk^* \in \RR^{|E|}$ as \[ \bk^* = \bk + \sum\limits^{b}_{i=1} (p^*_i - \langle \bk, \bB_i \rangle ) \bB_i. \] Since $\{ \bB_i \}^b_{i=1}$ is an orthonormal basis of the subspace $\mD(G)$, then for $1 \leq j \leq b$, \begin{equation} \label{eq:k*p*} \langle \bk^*, \bB_j \rangle = \langle \bk + \sum\limits^{b}_{i=1} (p^*_i - \langle \bk, \bB_i \rangle ) \bB_i, \bB_j \rangle = \langle \bk, \bB_j \rangle + (p^*_j - \langle \bk, \bB_j \rangle ) = p^*_j. \end{equation} Using Lemma \ref{lem:d0}, together with $\sum\limits^{b}_{i=1} (p^*_i - \bk \bB_i ) \bB_i \in \mD(G)$ and \eqref{eq:psi_wd_3}, we obtain \begin{equation} \label{eq:psi_wd_4} (G, \bk^*) \sim (G, \bk) \sim (G_1, \hat{\bk}_1). \end{equation} Therefore, $\bk^*$ satisfies Equations\eqref{def:hpsi_k} and \eqref{def:hpsi_kq}. \smallskip \noindent Let us assume that there exists $\bk^{**} \in \dK(G,G_1)$ satisfying Equations\eqref{def:hpsi_k} and \eqref{def:hpsi_kq}, i.e., \[(G, \bk^{**}) \sim (G_1, \hat{\bk}_1) \ \text{ and } \ \bp^* = ( \langle \bk^{**}, \bB_1 \rangle, \langle \bk^{**}, \bB_2 \rangle, \ldots, \langle \bk^{**}, \bB_b \rangle). \] This implies that $(G, \bk^{**}) \sim (G, \bk^*)$. From Lemma \ref{lem:d0}, we obtain \[ \bk^{**} - \bk^{*} \in \mD(G). \] Using \eqref{eq:k*p*}, we get \[ \langle \bk^*, \bB_j \rangle = \langle \bk^{**}, \bB_j \rangle = p^*_j \ \text{ for any } \ 1 \leq j \leq b. \] Recall that $\{ \bB_i \}^b_{i=1}$ is an orthonormal basis of $\mD(G)$. Therefore, we get \[ \bk^{**} = \bk^{*}. \] This implies that $\bk^* \in \dK(G,G_1)$ is well-defined. Moreover, from \eqref{def:hpsi_kq} we obtain \[ \bq^* = ( \langle \hbJ^*, \bA_1 \rangle, \langle \hbJ^*, \bA_2 \rangle, \ldots, \langle \hbJ^*, \bA_a \rangle ) \ \text{ is well-defined}. \] This implies that we get \[ \hPsi (\hbJ^*, \bx^*, \bp^*) = (\bk^*, \bq^*), \] and thus the map $\hPsi$ is well-defined. \end{proof} The following is a direct consequence of Lemma \ref{lem:hpsi_well_def}. \begin{corollary} \label{cor:hpsi_ss} Consider the map $\hPsi$ in Definition \ref{def:hpsi}. Suppose that $\hat{\Psi} (\hat{\bJ},\bx, \bp) = (\bk, \bq)$, then $\bx$ is a steady state of the system $(G, \bk)$. \end{corollary} \begin{proof} It is clear that $\hat{\bJ} \in \hat{\mJ} (G_1,G)$ and $\bx \in (\bx_0 + \mS_{G_1} )\cap\mathbb{R}^n_{>0}$. From Equation\eqref{def:hat_j_g1_g}, there exist some $\bJ^* = (J^*_{\by\rightarrow \by'})_{\by\rightarrow \by' \in E_1} \in \mJ (G_1,G)$, such that \[ \hbJ - \bJ^* \in \spn \{\bA_i \}^{a}_{i=1}. \] Using \eqref{eq:psi_wd_2} and setting $\bk_1 = (k_{1, \by\rightarrow \by'})_{\by\rightarrow \by' \in E_1}$ with $k_{1, \by\rightarrow \by'} = \frac{J^*_{\by \rightarrow \by'} }{ (\bx^*)^{\by} }$, we derive \[ \bk_1 \in \mK_{\RR} (G_1,G), \] and $\bx^*$ is the complex-balanced steady state of $(G_1, \bk_1)$. Finally, using Equations\eqref{eq:psi_wd_3} and \eqref{eq:psi_wd_4}, together with Remark \ref{rmk:de_ss}, we obtain $(G, \bk) \sim (G_1, \bk_1)$ and conclude that $\bx$ is a steady state of the system $(G, \bk)$. \end{proof} \begin{lemma} \label{lem:hpsi_bijective} The map $\hPsi$ in Definition \ref{def:hpsi} is bijective. \end{lemma} \begin{proof} First, we show the map $\hPsi$ is injective. Suppose two elements $(\hbJ^*, \bx^*, \bp^*)$ and $(\hbJ^{**}, \bx^{**}, \bp^{**})$ of $\hat{\mJ} (G_1,G) \times [(\bx_0 + \mS_{G_1} )\cap\mathbb{R}^n_{>0}] \times \mathbb{R}^b$ satisfy \[ \hPsi (\hbJ^*, \bx^*, \bp^*) = \hPsi (\hbJ^{**}, \bx^{**}, \bp^{**}) = (\bk, \bq) \in \dK(G,G_1)\times \RR^a. \] From \eqref{def:hat_j_g1_g}, there exist $\bJ^* = (J^*_{\by\rightarrow \by'})_{\by\rightarrow \by' \in E_1} \in \mJ (G_1,G)$ and $\bJ^{**} = (J^{**}_{\by\rightarrow \by'})_{\by\rightarrow \by' \in E_1} \in \mJ (G_1,G)$, such that \begin{equation} \label{eq:hpsi_bijective_1} \hbJ^* - \bJ^* \in \spn \{ \bA_i \}^{a}_{i=1} \ \text{ and } \ \hbJ^{**} - \bJ^{**} \in \spn \{ \bA_i \}^{a}_{i=1}. \end{equation} Then we set $\bk^* = (k^*_{\by\rightarrow \by'})_{\by\rightarrow \by' \in E_1}$ and $\bk^{**} = (k^{**}_{\by\rightarrow \by'})_{\by\rightarrow \by' \in E_1}$ with \[ k^*_{\by\rightarrow \by'} = \frac{J^*_{\by\rightarrow \by'}}{{(\bx^*)}^{\by}} \ \text{ and } \ k^{**}_{\by\rightarrow \by'} = \frac{J^{**}_{\by\rightarrow \by'}}{{(\bx^*)}^{\by}}. \] Using Propositions\ref{prop:craciun2020efficient} and \eqref{def:hpsi_k}, we get \[\bk^*, \bk^{**} \in \mK_{\RR} (G_1,G) \ \text{ and } \ (G, \bk) \sim (G_1, \bk^*) \sim (G_1, \bk^{**}). \] Moreover, two complex-balanced system $(G_1, \bk^*)$ and $(G_1, \bk^{**})$ admit steady states \[ \bx^* \in (\bx_0 + \mS_{G_1} )\cap\mathbb{R}^n_{>0} \ \text{ and } \ \bx^{**} \in (\bx_0 + \mS_{G_1} )\cap\mathbb{R}^n_{>0}, \ \text{respectively}. \] Since every complex-balanced system has a unique steady state within each invariant polyhedron and $(G_1, \bk^*) \sim (G_1, \bk^{**})$, then \[ \bx^* = \bx^{**}. \] Now applying Proposition \ref{prop:craciun2020efficient} and Lemma \ref{lem:j0}, we get \begin{equation} \label{eq:hpsi_bijective_2} (G_1, \bJ^*) \sim (G_1, \bJ^{**}) \ \text{ and } \ \bJ^{**} - \bJ^* \in \eJ(G_1). \end{equation} Since $\eJ(G_1) = \spn \{ \bA_i \}^{a}_{i=1}$, using \eqref{eq:hpsi_bijective_1} and \eqref{eq:hpsi_bijective_2}, we have \begin{equation} \label{eq:hpsi_bijective_3} \hbJ^{**} - \hbJ^* \in \spn \{ \bA_i \}^{a}_{i=1}. \end{equation} On the other hand, Equation\eqref{def:hpsi_kq} shows that \[ \bq = ( \langle \hbJ^*, \bA_1 \rangle, \langle \hbJ^*, \bA_2 \rangle, \ldots, \langle \hbJ^*, \bA_a \rangle ) = ( \langle \hbJ^{**}, \bA_1 \rangle, \langle \hbJ^{**}, \bA_2 \rangle, \ldots, \langle \hbJ^{**}, \bA_a \rangle ). \] Since $\{\bA_i \}^{a}_{i=1}$ is an orthonormal basis of the subspace $\eJ(G)$, together with \eqref{eq:hpsi_bijective_3}, then \[ \hbJ^* = \hbJ^{**}. \] Furthermore, from \eqref{def:hpsi_kq} we obtain \[ \bp^* = \bp^{**} = ( \langle \bk, \bB_1 \rangle, \langle \bk, \bB_2 \rangle, \ldots, \langle \bk, \bB_b \rangle). \] Therefore, we show $(\bJ^*, \bx^*, \bp^*) = (\bJ^{**}, \bx^{**}, \bp^{**})$ and conclude the injectivity. \medskip We now show that the map $\hPsi$ is surjective. Assume any point $(\bk, \bq) \in \dK(G,G_1)\times \RR^a$. Since $\bk \in \dK (G, G_1)$, there exists some $\bk_1 \in \mK (G_1, G)$, such that \begin{equation} \label{eq:gk_g1k1} (G, \bk) \sim (G_1, \bk_1) \ \text{ with } \ \bk_1 = (k_{1, \by\rightarrow \by'})_{\by\rightarrow \by' \in E_1}. \end{equation} From Theorem \ref{thm:cb}, the complex-balanced system $(G_1, \bk_1)$ has a unique steady state $\bx \in (\bx_0 + \mS_{G_1} )\cap\mathbb{R}^n_{>0}$. We set the flux vector $\bJ_1$ as \[ \bJ_1 = (J_{1, \by\rightarrow \by'})_{\by\rightarrow \by' \in E_1} \ \text{ with } \ J_{1, \by\rightarrow \by'} = k_{1, \by\rightarrow \by'} {\bx}^{\by}. \] It is clear that $\bJ_1 \in \mJ (G_1,G)$ and the flux system $(G_1, \bJ_1)$ gives rise to the complex-balanced system $(G_1, \bk_1)$ with a steady state $\bx \in (\bx_0 + \mS_{G_1} )\cap\mathbb{R}^n_{>0}$. Now suppose $\bq = (q_1, q_2, \ldots, q_a)$, we construct a new flux vector $\hbJ$ as follows: \[ \hbJ = \bJ_1 + \sum\limits^{a}_{i=1} (q_i - \langle \bJ_1, \bA_i \rangle ) \bA_i. \] Using the fact that $\{ \bA_i \}^a_{i=1}$ is an orthonormal basis of the subspace $\eJ(G_1)$, we can compute \begin{equation} \notag \langle \hbJ, \bA_i \rangle = \hat{q}_i \ \text{ for any } \ 1 \leq i \leq a. \end{equation} From Lemma \ref{lem:j0} and $\sum\limits^{a}_{i=1} (q_i - \langle\bJ_1 \bA_i\rangle ) \bA_i \in \eJ(G_1)$, we obtain \[ (G, \hbJ) \sim (G_1, \bJ_1). \] Let $\hbk_1 = (k_{1, \by\rightarrow \by'})_{\by\rightarrow \by' \in E_1}$ with $\hat{k}_{1, \by\rightarrow \by'} = \frac{\hat{J}_{\by\rightarrow \by'}}{{\bx}^{\by}}$. From Proposition \ref{prop:craciun2020efficient} and \eqref{eq:gk_g1k1}, we have \[ (G, \bk) \sim (G_1, \bk_1) \sim (G, \hbk_1). \] Finally, let $\bp = ( \langle \bk, \bB_1 \rangle, \langle \bk, \bB_2 \rangle, \ldots, \langle \bk, \bB_b \rangle)$ and derive that \[ \hat{\Psi} (\hat{\bJ},\bx, \bp) = (\bk, \bq). \] Therefore, we prove the map $\hat{\Psi}$ is surjective. \end{proof} \begin{lemma} \label{lem:hpsi_cts} The map $\hPsi$ in Definition \ref{def:hpsi} is continuous. \end{lemma} \begin{proof} Consider any fixed point $(\hbJ, \bx, \bp) \in \hmJ (G_1,G)\times [(\bx_0 + \mS_{G_1} )\cap\mathbb{R}^n_{>0}] \times \mathbb{R}^b$, such that \[ \hPsi (\hbJ, \bx, \bp) = (\bk, \bq). \] From \eqref{def:hpsi_kq} in Definition \ref{def:hpsi}, $\bq$ is defined as \[ \bq = ( \langle \hat{\bJ}, \bA_1 \rangle, \langle \hat{\bJ}, \bA_2 \rangle, \ldots, \langle \hat{\bJ}, \bA_a \rangle ). \] It follows that $\bq$ is a continuous function of $\hbJ$. \smallskip Now it remains to show that $\bk$ is also a continuous function of $(\hbJ,\bx,\bq)$. Recall \eqref{def:hpsi_k} in Definition \ref{def:hpsi}, $\bk$ is defined as \[ (G, \bk) \sim (G_1, \hat{\bk}_1) \ \text{ with } \ \hat{k}_{1, \by\rightarrow \by'} = \frac{\hat{J}_{\by\rightarrow \by'}}{{\bx}^{\by}}. \] Together with \eqref{def:hpsi_kq}, we get \begin{equation} \label{eq:k_ct_2} \bp = ( \langle \bk, \bB_1 \rangle, \langle \bk, \bB_2 \rangle, \ldots, \langle \bk, \bB_b \rangle), \end{equation} and for every vertex $\by_0 \in V \cup V_1$, \begin{equation} \label{eq:k_ct_1} \sum_{\by_0 \to \by \in E} k_{\by_0 \to \by} (\by - \by_0) = \sum_{\by_0 \to \by' \in E_1} \frac{\hat{J}_{\by_0 \rightarrow \by'}}{{\bx}^{\by_0}} (\by' - \by_0). \end{equation} Note that $\hbJ$ and $\bx$ are fixed, then \eqref{eq:k_ct_1} can be rewritten as \begin{equation} \label{eq:k_ct_1_1} \sum_{\by_0 \to \by \in E} k_{\by_0 \to \by} (\by - \by_0) = \text{constant}. \end{equation} Assume $\bk'$ is another solution to \eqref{eq:k_ct_1_1}, then \[ (G, \bk) \sim (G, \bk'). \] Using Lemma \ref{lem:d0}, we obtain that \[ \bk' - \bk \in \mD (G). \] Together with the linearity of $\mD (G)$, the solutions to \eqref{eq:k_ct_1_1} form an affine linear subspace. Hence, the tangent space of the solution to \eqref{eq:k_ct_1_1} at $(\bJ, \bx, \bp)$ is $\mD(G)$. Analogously, given fixed $\bp$, the solutions to \eqref{eq:k_ct_2} also form an affine linear subspace, whose tangent space at $(\bJ, \bx, \bp)$ is tangential to \begin{equation} \notag \spn \{\bB_1, \bB_2, \ldots, \bB_b\} = \mD(G). \end{equation} This indicates that two tangent spaces at $(\bJ, \bx, \bp)$ are complementary, and thus intersect transversally~\cite{guillemin2010differential}. From Lemma \ref{lem:hpsi_well_def}, $\bk$ is the unique solution to \eqref{eq:k_ct_2} and \eqref{eq:k_ct_1}. Therefore, we conclude that $\bk$ as the unique intersection point (solution) of two equations \eqref{eq:k_ct_2} and \eqref{eq:k_ct_1} must vary continuously with respect to parameters $(\hbJ, \bx, \bp)$. \end{proof} \section{Continuity of \texorpdfstring{$\hPsi^{-1}$}{hPsi-1}} \label{sec:continuity} In this section, we first introduce the map $\Phi$ (see Definition \ref{def:phi}) and prove $\Phi = \hPsi^{-1}$ is well-defined. Then we show the map $\Phi$ is continuous, i.e. $\hPsi^{-1}$ is also continuous. \begin{definition} \label{def:phi} Given a weakly reversible E-graph $G_1 = (V_1, E_1)$ with its stoichiometric subspace $\mS_{G_1}$. Consider an E-graph $G = (V, E)$ and $\bx_0\in\mathbb{R}^n_{>0}$, define the map \begin{equation} \label{eq:phi} \Phi: \dK(G,G_1)\times \RR^a \rightarrow \hat{\mJ} (G_1,G) \times [(\bx_0 + \mS_{G_1} )\cap\mathbb{R}^n_{>0}] \times \RR^b, \end{equation} such that for $(\bk, \bq) \in \dK(G,G_1)\times \RR^a$, \begin{equation} \notag \Phi (\bk, \bq) := (\hat{\bJ},\bx, \bp), \end{equation} where $\bx \in (\bx_0 + \mS_{G_1} )\cap\mathbb{R}^n_{>0}$ is the steady state of $(G, \bk)$, and \begin{equation} \label{def:phi_k} (G, \bk) \sim (G_1, \hat{\bk}_1) \ \text{ with } \ \hat{k}_{1, \by\rightarrow \by'} = \frac{\hat{J}_{\by\rightarrow \by'}}{{\bx}^{\by}}, \end{equation} and \begin{equation} \label{def:phi_kq} \bp = ( \langle \bk, \bB_1 \rangle, \langle \bk, \bB_2 \rangle, \ldots, \langle \bk, \bB_b \rangle), \ \ \bq = ( \langle \hat{\bJ}, \bA_1 \rangle, \langle \hat{\bJ}, \bA_2 \rangle, \ldots, \langle \hat{\bJ}, \bA_a \rangle ). \end{equation} \end{definition} \medskip \begin{lemma} \label{lem:phi_wd} The map $\Phi$ in Definition \ref{def:phi} is well-defined, and $\Phi = \hPsi^{-1}$ is bijective. \end{lemma} \begin{proof} Assume any point $(\bk^*, \bq^*) \in \dK(G,G_1)\times \RR^a$. There exists $\bk_1 \in \mK_{\RR} (G_1,G)$ satisfying \begin{equation} \label{eq:phi_wd_1} (G, \bk^*) \sim (G_1, \bk_1). \end{equation} From Theorem \ref{thm:cb}, $(G_1, \bk_1)$ has a unique steady state $\bx^* \in (\bx_0 + \mS_{G_1} )\cap\mathbb{R}^n_{>0}$. Further, Remark \ref{rmk:de_ss} shows that $(G, \bk^*)$ and $(G_1, \bk_1)$ share the same steady states, thus $\bx^* \in (\bx_0 + \mS_{G_1} )\cap\mathbb{R}^n_{>0}$ is also the unique steady state of $(G, \bk^*)$, i.e. $\bx^*$ is well-defined. Moreover, from \eqref{def:phi_kq} we obtain \begin{equation} \label{eq:phi_wd_2} \bp^* = ( \langle \bk^*, \bB_1 \rangle, \langle \bk^*, \bB_2 \rangle, \ldots, \langle \bk^*, \bB_b \rangle), \end{equation} which is well-defined. Since $\bk_1 \in \mK_{\RR} (G_1,G)$, then $(G_1, \bk_1)$ and its steady state $\bx^*$ give rise to the complex-balanced flux system $(G_1, \bJ^*)$, such that \[ \bJ^* = (J^*_{\by\rightarrow \by'})_{\by\rightarrow \by' \in E_1} \in \mJ (G_1,G) \ \text{ with } \ J^*_{\by\rightarrow \by'} = k_{1, \by\rightarrow \by'} (\bx^*)^{\by}. \] Suppose $\bq^* = (q^*_1, q^*_2, \ldots, q^*_a) \in \RR^a$, we construct the vector $\hbJ^* \in \RR^{|E|}$ as \[ \hbJ^* = \bJ^* + \sum\limits^a_{i=1} (q^*_i - \langle \bJ^*, \bA_i \rangle ) \bA_i \in \hat{\mJ} (G_1,G). \] Note that $\{ \bA_i \}^a_{i=1}$ is an orthonormal basis of $\eJ(G_1)$, together with Lemma \ref{lem:j0}, we obtain \begin{equation} \notag \bq^* = ( \langle \hbJ^*, \bA_1 \rangle, \langle \hbJ^*, \bA_2 \rangle, \ldots, \langle \hbJ^*, \bA_a \rangle ) \ \text{ and } \ (G_1, \hbJ^*) \sim (G_1, \bJ^*). \end{equation} Using Proposition \ref{prop:craciun2020efficient} and \eqref{eq:phi_wd_1}, we set $\hbk_1 = (\hat{k}_{1, \by\rightarrow \by'})_{\by\rightarrow \by' \in E_1}$ with $\hat{k}_{1, \by\rightarrow \by'} = \frac{\hat{J}^*_{\by\rightarrow \by'}}{{(\bx^*)}^{\by}}$ and derive \begin{equation} \notag (G_1, \hat{\bk}_1) \sim (G_1, \bk_1) \sim (G, \bk^*). \end{equation} Together with \eqref{eq:phi_wd_2}, we conclude that $(\hbJ^*, \bx^*, \bp^*)$ satisfies \eqref{def:phi_k} and \eqref{def:phi_kq}. Now suppose there exists another $(\hbJ^{**}, \bx^{**}, \bp^{**}) \in \hat{\mJ} (G_1,G)\times [(\bx_0 + \mS_{G_1} )\cap\mathbb{R}^n_{>0}] \times \mathbb{R}^b$, which also satisfies \eqref{def:phi_k} and \eqref{def:phi_kq}. From Definition \ref{def:hpsi}, we deduce \begin{equation} \notag \hPsi (\hbJ^*, \bx^*, \bp^*) = \hPsi (\hbJ^{**}, \bx^{**}, \bp^{**}) = (\bk^*, \bq^*). \end{equation} Since $\hPsi$ is proved to be bijective in Lemma \ref{lem:hpsi_bijective}, then \begin{equation} \notag (\hbJ^*, \bx^*, \bp^*) = (\hbJ^{**}, \bx^{**}, \bp^{**}). \end{equation} Thus, we conclude that $\Phi$ is well-defined. \smallskip Next, for any $(\hbJ, \bx, \bp) \in \hat{\mJ} (G_1,G)\times [(\bx_0 + \mS_{G_1} )\cap\mathbb{R}^n_{>0}] \times \mathbb{R}^b$, suppose that \begin{equation} \label{eq:phi_wd_3} \hPsi (\hbJ, \bx, \bp) = (\bk, \bq) \in \dK(G,G_1)\times \RR^a. \end{equation} From Definition \ref{def:hpsi} and Corollary \ref{cor:hpsi_ss}, together with \eqref{def:phi_k} and \eqref{def:phi_kq}, we have \begin{equation} \label{eq:phi_wd_4} \Phi (\bk, \bq) = (\hbJ, \bx, \bp). \end{equation} This implies $\Phi = \hPsi^{-1}$. Recall that $\hPsi$ is bijective, thus its inverse $\hPsi^{-1}$ is well-defined and bijective. Therefore, we prove the lemma. \end{proof} \begin{lemma} \label{lem:inverse_cts_q} Consider the map $\Phi$ in Definition \ref{def:phi}, suppose any fixed $\bk \in \dK(G,G_1)$ and $\bq_1, \bq_2 \in \RR^a$, then \begin{equation} \label{eq:inverse_cts_q_1} \Phi (\bk, \bq_1) - \Phi (\bk, \bq_2) = \left(\sum\limits^{a}_{i=1} \varepsilon_i \bA_i, \mathbf{0}, \mathbf{0}\right), \end{equation} where $\bq_1 - \bq_2 := (\varepsilon_1, \varepsilon_2, \ldots, \varepsilon_a) \in \RR^a$. \end{lemma} \begin{proof} Given fixed $\bk \in \dK(G,G_1)$, consider any $\bq \in \RR^a$, such that \begin{equation} \notag \Phi (\bk, \bq) = (\hat{\bJ},\bx, \bp). \end{equation} From Definition \ref{def:phi}, $\bx \in (\bx_0 + \mS_{G_1} )\cap\mathbb{R}^n_{>0}$ is the steady state of $(G, \bk)$. Further, we have \begin{equation} \label{eq:inverse_cts_q_3} (G, \bk) \sim (G_1, \hat{\bk}_1) \ \text{ with } \ \hat{k}_{1, \by\rightarrow \by'} = \frac{\hat{J}_{\by\rightarrow \by'}}{{\bx}^{\by}}, \end{equation} and \begin{equation} \label{eq:inverse_cts_q_4} \bp = ( \langle \bk, \bB_1 \rangle, \langle \bk, \bB_2 \rangle, \ldots, \langle \bk, \bB_b \rangle), \ \ \bq = ( \langle \hat{\bJ}, \bA_1 \rangle, \langle \hat{\bJ}, \bA_2 \rangle, \ldots, \langle \hat{\bJ}, \bA_a \rangle ). \end{equation} \smallskip Now consider any vector $\bep = (\varepsilon_1, \varepsilon_2, \ldots, \varepsilon_a) \in \RR^a$, it follows that \eqref{eq:inverse_cts_q_1} is equivalent to show the following: \begin{equation} \label{eq:inverse_cts_q_2} \Phi (\bk, \bq + \bep) = (\hat{\bJ} + \sum\limits^{a}_{i=1} \varepsilon_i \bA_i,\bx, \bp). \end{equation} Suppose $\Phi (\bk, \bq + \bep) = (\hbJ^{\bep}, \bx^{\bep}, \bp^{\bep})$. From Definition \ref{def:phi} and Lemma \ref{lem:phi_wd}, $\bx^{\bep}$ is the unique steady state of $(G, \bk)$ in the invariant polyhedron $ (\bx_0 + \mS_{G_1} )\cap\mathbb{R}^n_{>0}$. Recall that $\bx \in (\bx_0 + \mS_{G_1} )\cap\mathbb{R}^n_{>0}$ is also the steady state of $(G, \bk)$, thus we have \begin{equation} \label{eq:inverse_cts_q_6} \bx = \bx^{\bep}. \end{equation} Since $\hat{\bJ} \in \hmJ (G_1,G)$ and $\{ \bA_i \}^a_{i=1}$ is an orthonormal basis of $\eJ(G_1)$, we get \[ (G_1, \hat{\bJ}) \sim (G_1, \hat{\bJ} + \sum\limits^{a}_{i=1} \varepsilon_i \bA_i). \] Using Proposition \ref{prop:craciun2020efficient} and \eqref{eq:inverse_cts_q_3}, by setting $\hat{J}_{\by\rightarrow \by'} + \sum\limits^{a}_{i=1} \varepsilon_i \bA_{i, \by\rightarrow \by'} = \hat{k}^{\bep}_{1, \by\rightarrow \by'} \bx^{\by}$, we obtain \begin{equation} \label{eq:inverse_cts_q_5} (G_1, \hat{\bk}^{\bep}_1) \sim (G_1, \hat{\bk}_1) \sim (G, \bk). \end{equation} Under direct computation, for $1 \leq i \leq a$, \begin{equation} \notag \langle \hat{\bJ} + \sum\limits^{a}_{i=1} \varepsilon_i \bA_i, \bA_i \rangle = \langle \hat{\bJ}, \bA_i \rangle + \langle \sum\limits^{a}_{i=1} \varepsilon_i \bA_i, \bA_i \rangle = \langle \hat{\bJ}, \bA_i \rangle + \varepsilon_i. \end{equation} From Lemma \ref{lem:phi_wd} and \eqref{eq:inverse_cts_q_5}, we get \begin{equation} \label{eq:inverse_cts_q_7} \hbJ^{\bep} = \hat{\bJ} + \sum\limits^{a}_{i=1} \varepsilon_i \bA_i. \end{equation} Finally, from Definition \ref{def:phi} and \eqref{eq:inverse_cts_q_4}, it is clear that \begin{equation} \label{eq:inverse_cts_q_8} \bp^{\bep} = ( \langle \bk, \bB_1 \rangle, \langle \bk, \bB_2 \rangle, \ldots, \langle \bk, \bB_b \rangle ) = \bp. \end{equation} Combining Equations~\eqref{eq:inverse_cts_q_6}, \eqref{eq:inverse_cts_q_7} and \eqref{eq:inverse_cts_q_8}, we prove \eqref{eq:inverse_cts_q_2}. \end{proof} Here we present Proposition \ref{prop:inverse_cts_k}, which is the key for the continuity of $\hPsi^{-1}$. \begin{proposition} \label{prop:inverse_cts_k} Consider the map $\Phi$ in Definition \ref{def:phi} and any fixed $\bq \in \RR^a$, then $\Phi (\cdot, \bq)$ is continuous with respect to $\bk$. \end{proposition} To prove Proposition~\ref{prop:inverse_cts_k}, we need to show Lemmas \ref{lem:key_1} - \ref{lem:key_3} and Proposition \ref{lem:key_4}. The following is the overview of the process. First, Lemma \ref{lem:key_1} shows that if two reaction rate vectors in $\dK (G, G_1)$ are close enough, then there exist two reaction rate vectors (dynamically equivalent respectively) in $\mK (G_1, G_1)$ such that their distance can be controlled. Second, in Lemma \ref{lem:key_2} we show that given a complex-balanced rate vector $\bk_1 \in \mK (G_1)$, there exists a neighborhood around $\bk_1$ of $\RR^{E_1}_{>0}$, in which the steady states of the system associated with the rate constants vary continuously. Combining Lemma \ref{lem:key_1} with \ref{lem:key_2}, we prove in Lemma \ref{lem:key_3} that given a reaction rate vector $\bk \in \dK (G, G_1)$, there exists an open neighborhood $\bk \in U \subset \RR^{E}$, such that the steady states of the system associated with the rate vectors in $U$ vary continuously. Finally, in Proposition \ref{lem:key_4} we prove that given a complex-balanced rate vector $\bk^* \in \mK (G_1, G_1)$, for any sequence $\bk_i \to \bk^*$ in $\mK (G_1, G_1)$, there exists another sequence of reaction rate vectors (dynamically equivalent respectively) $\hbk_i \to \bk^*$ in $\RR^{E_1}$, and all associated fluxes from reaction rate vectors have the same projections on $\eJ (G_1)$. \medskip \begin{lemma} \label{lem:key_1} Let $\bk \in \dK (G,G_1)$. Then we have the following: \begin{enumerate}[label=(\alph*)] \item There exists $\bk_1 \in \mK (G_1)$ satisfying $(G, \bk) \sim (G_1, \bk_1)$. \item There exist constants $\varepsilon = \varepsilon (\bk) > 0$ and $C = C (\bk) > 0$, such that for any $\hbk \in \dK (G,G_1)$ with $\| \hbk - \bk \| \leq \varepsilon$, there exists $\hbk_1 \in \mK (G_1,G_1)$ that satisfies \begin{enumerate}[label=(\roman*)] \item $\|\hbk_1 - \bk_1 \| \leq C \varepsilon $. \item $(G,\hbk) \sim (G_1, \hbk_1)$. \end{enumerate} \end{enumerate} \end{lemma} \begin{proof} For part $(a)$, from Definitions \ref{def:mas_realizable} and \ref{def:de_realizable}, given $\bk \in \dK (G,G_1)$, the system $(G, \bk)$ is disguised toric on $G_1$, that is, there exists $\bk_1 \in \mK_{\RR} (G_1, G) \subset \mK (G_1)$ with $(G, \bk) \sim (G_1, \bk_1)$. \smallskip Now we prove part $(b)$.\\ \textbf{Step 1: } Let $\by \in G \cup G_1$ be a fixed vertex and consider the following vector space: \begin{equation} \notag W_{\by} = \spn \{ \by' - \by: \by \rightarrow \by' \in G_1 \}. \end{equation} Let $d(\by) = \dim (W_{\by})$. Then there exists an orthogonal basis of $W_{\by}$ denoted by: \begin{equation} \label{eq:key_1_1} \{ \bw_1, \bw_2, \ldots, \bw_{d (\by)} \}. \end{equation} For each $\bw_i$ in \eqref{eq:key_1_1}, there exist positive $\{ c_{i, \by \rightarrow \by'} \}_{\by \rightarrow \by' \in G_1}$, that satisfy \begin{equation} \label{eq:key_1_2} \bw_i = \sum\limits_{\by \rightarrow \by' \in G_1} c_{i, \by \rightarrow \by'} (\by' - \by). \end{equation} Let $\hbk \in \dK (G,G_1)$. From Definition \ref{def:de_realizable}, $\sum\limits_{\by \rightarrow \tilde{\by} \in G} \hbk_{\by \rightarrow \tilde{\by}} (\tilde{\by} - \by)$ is realizable on $G_1$ at the vertex $\by \in G \cup G_1$. This implies that \begin{equation} \label{eq:key_1_3} \sum\limits_{\by \rightarrow \tilde{\by} \in G} \hbk_{\by \rightarrow \tilde{\by}} (\tilde{\by} - \by) \in W_{\by}. \end{equation} Since $\bk \in \dK (G,G_1)$, together with Equation~\eqref{eq:key_1_3}, we obtain \begin{equation} \label{eq:key_1_Delta} \Delta_{\by} (\hbk, \bk) := \sum\limits_{\by \rightarrow \tilde{\by} \in G} ( \hbk_{\by \rightarrow \tilde{\by}} - \bk_{\by \rightarrow \tilde{\by}}) (\tilde{\by} - \by) \in W_{\by}. \end{equation} Assume that $\| \hbk - \bk \| \leq \varepsilon$. Consider all reaction vectors in $G$ and let $m = \max\limits_{\by \rightarrow \tilde{\by} \in G} \| \tilde{\by} - \by \|$, then there exists a constant $C_1 = m |E|$, such that \[ \| \Delta_{\by} (\hbk, \bk) \| \leq \sum\limits_{\by \rightarrow \tilde{\by} \in G} m \varepsilon = C_1 \varepsilon. \] On the other side, from \eqref{eq:key_1_1}, $\Delta_{\by} (\hbk, \bk)$ can be expressed as \begin{equation} \label{eq:key_1_4} \Delta_{\by} (\hbk, \bk) = \sum\limits^{d(\by)}_{i=1} \delta_i \bw_i \ \text{ with } \ \delta_i \in \RR. \end{equation} Using \eqref{eq:key_1_4} and the orthogonal basis in \eqref{eq:key_1_1}, for any $1 \leq i \leq d (\by)$, \begin{equation} \label{eq:key_1_5} | \delta_i | \leq \| \Delta_{\by} (\hbk, \bk) \| \leq C_1 \varepsilon. \end{equation} Inputting \eqref{eq:key_1_2} into \eqref{eq:key_1_4}, we get \begin{equation} \label{eq:key_1_6} \Delta_{\by} (\hbk, \bk) = \sum\limits^{d(\by)}_{i=1} \delta_i \big( \sum\limits_{\by \rightarrow \by' \in G_1} c_{i, \by \rightarrow \by'} (\by' - \by) \big) = \sum\limits_{\by \rightarrow \by' \in G_1} \big( \sum\limits^{d(\by)}_{i=1} \delta_i c_{i, \by \rightarrow \by'} \big) (\by' - \by). \end{equation} From \eqref{eq:key_1_5} and \eqref{eq:key_1_6}, there exists a constant $C_2$, such that for any $\by \rightarrow \by' \in G_1$, \begin{equation} \label{eq:key_1_7} \big| \hat{c}_{\by \rightarrow \by'} := \sum\limits^{d(\by)}_{i=1} \delta_i c_{i, \by \rightarrow \by'} \big| \leq C_2 \varepsilon. \end{equation} Then we construct $\hbk_1$ as follows: \begin{equation} \label{eq:key_1_8} \hbk_{1, \by \rightarrow \by'} := \bk_{1, \by \rightarrow \by'} + \hat{c}_{\by \rightarrow \by'} \ \text{ for any } \ \by \rightarrow \by' \in G_1. \end{equation} Consider all reaction vectors in $G_1$, together with \eqref{eq:key_1_7}, we derive \begin{equation} \label{eq:key_1_estimate} \| \hbk_1 - \bk_1 \| \leq \sum\limits_{\by \rightarrow \by' \in G_1} |\hat{c}_{\by \rightarrow \by'}| \leq \sum\limits_{\by \rightarrow \by' \in G_1} C_2 \varepsilon \leq C_2 |E_1| \varepsilon. \end{equation} Similarly, we can go through all vertices in $G \cup G_1$, and take the above steps to update $\hbk_1$. For every vertex, we can derive an estimate similar to \eqref{eq:key_1_estimate}. Collecting the estimates on all vertices, we can find a constant $C$, such that \[ \| \hbk_1 - \bk_1 \| \leq C \varepsilon \ \text{ for any } \ \| \hbk - \bk \| \leq \varepsilon. \] \textbf{Step 2: } We claim that there exists a sufficiently small constant $\varepsilon = \varepsilon (\bk) > 0$, such that for any $\hbk$ with $\| \hbk - \bk \| \leq \varepsilon$, then $\hbk_1$ defined in \eqref{eq:key_1_8} satisfies \begin{equation} \label{eq:key_1_claim} (G, \hbk) \sim (G_1, \hbk_1) \ \text{ and } \ \hbk_1 \in \mK (G_1,G_1). \end{equation} Recall \eqref{eq:key_1_3} and \eqref{eq:key_1_Delta}, at vertex $\by \in G \cup G_1$, \begin{equation} \label{eq:key_1_9} \Delta_{\by} (\hbk, \bk) = \sum\limits_{\by \rightarrow \tilde{\by} \in G} \hbk_{\by \rightarrow \tilde{\by}} (\tilde{\by} - \by) - \sum\limits_{\by \rightarrow \tilde{\by} \in G} \bk_{\by \rightarrow \tilde{\by}} (\tilde{\by} - \by). \end{equation} On the other hand, from \eqref{eq:key_1_6}-\eqref{eq:key_1_8}, at vertex $\by \in G \cup G_1$, \begin{equation} \label{eq:key_1_10} \Delta_{\by} (\hbk, \bk) = \sum\limits_{\by \rightarrow \by' \in G_1} \hbk_{1, \by \rightarrow \by'} (\by' - \by) - \sum\limits_{\by \rightarrow \by' \in G_1} \bk_{1, \by \rightarrow \by'} (\by' - \by). \end{equation} Note that $(G, \bk) \sim (G_1, \bk_1)$ implies that, at vertex $\by \in G \cup G_1$, \[ \sum\limits_{\by \rightarrow \tilde{\by} \in G} \bk_{\by \rightarrow \tilde{\by}} (\tilde{\by} - \by) = \sum\limits_{\by \rightarrow \by' \in G_1} \bk_{1, \by \rightarrow \by'} (\by' - \by). \] Together with \eqref{eq:key_1_9} and \eqref{eq:key_1_10}, we have, at vertex $\by \in G \cup G_1$, \begin{equation} \sum\limits_{\by \rightarrow \tilde{\by} \in G} \hbk_{\by \rightarrow \tilde{\by}} (\tilde{\by} - \by) = \sum\limits_{\by \rightarrow \by' \in G_1} \hbk_{1, \by \rightarrow \by'} (\by' - \by). \end{equation} Hence, we derive $(G, \hbk) \sim (G_1, \hbk_1)$. Moreover, since $\hbk \in \dK (G,G_1)$, there exists $\hbk^* \in \mK (G_1)$ with $(G, \hbk) \sim (G_1, \hbk^*)$, and thus \[ (G_1, \hbk_1) \sim (G_1, \hbk^*). \] Recall that $\bk_1 \in \mK (G_1) \subset \RR^{E_1}_{>0}$, together with \eqref{eq:key_1_estimate}, there must exist a constant $\varepsilon = \varepsilon (\bk) > 0$, such that for any $\hbk$ with $\| \hbk - \bk \| \leq \varepsilon$, we have $\hbk_1 \in \RR^{E_1}_{>0}$. Therefore, we obtain $\hbk_1 \in \mK (G_1,G_1)$ and prove the claim. \end{proof} \begin{lemma} \label{lem:key_2} Suppose $\bx_0 \in \mathbb{R}^n_{>0}$ and $\bk_1 \in \mK (G_1)$, then there exists an open set $U \subset \RR^{E_1}_{>0}$ containing $\bk_1$, such that there exists a unique continuously differentiable function \begin{equation} \label{lem:key_2_1} T : U \rightarrow (\bx_0 + \mS_{G_1} )\cap\mathbb{R}^n_{>0}. \end{equation} such that for any $\hbk \in U$, \begin{equation} \label{lem:key_2_2} T (\hbk) = \hbx, \end{equation} where $\hbx \in (\bx_0 + \mS_{G_1} )\cap\mathbb{R}^n_{>0}$ is the steady state of $(G_1, \hbk)$. \end{lemma} \begin{proof} Given $\bx_0 \in \mathbb{R}^n_{>0}$ and $\bk_1 \in \mK (G_1)$, Theorem \ref{thm:cb} shows the system $(G_1, \bk_1)$ has a unique steady state $\bx^* \in (\bx_0 + \mS_{G_1}) \cap \mathbb{R}^n_{>0}$. Consider the system $(G_1, \bk_1)$ as follows: \begin{equation} \label{eq:key_2_0} \frac{d\bx}{dt} = \bf (\bk_1, \bx) := (\bf_1, \bf_2, \ldots, \bf_n)^{\intercal} = \sum_{\by_i \rightarrow \by_j \in E_1} k_{1, \by_i \rightarrow \by_j} \bx^{\by_i}(\by_j - \by_i). \end{equation} Suppose $\dim (\mS_{G_1}) = s \leq n$. This implies that there exist exactly $s$ linearly independent components among $\bf (\bk_1, \bx)$. Without loss of generality, we assume that $\{\bf_1, \ldots, \bf_s \}$ are linearly independent components, and every $\bf_i$ with $s+1 \leq i \leq n$ can be represented as a linear combination of $\{\bf_i \}^{s}_{i=1}$. Using Theorem~\ref{thm:jacobian}, we obtain that \begin{equation} \notag \ker \Big( \big[ \frac{\partial \bf_i}{ \partial \bx_j} \big]_{1 \leq i, j \leq n} \big|_{\bx = \bx^*} \Big) = \mS^{\perp}_{G_1}. \end{equation} Together with the linear dependence among $\{ \bf_i (\bx) \}^{n}_{i=1}$, we derive \begin{equation} \label{eq:key_2_1} \ker \Big( \big[ \frac{\partial \bf_i}{ \partial \bx_j} \big]_{1 \leq i \leq s, 1 \leq j \leq n} \big|_{\bx = \bx^*} \Big) = \mS^{\perp}_{G_1}. \end{equation} Consider the orthogonal complement $\mS^{\perp}_{G_1}$ to the stoichiometric subspace in $\mathbb{R}^n$, which admits an orthonormal basis given by \[ \{\bv_1, \bv_2, \ldots, \bv_{n-s} \}. \] Now we construct a system of $n$ equations $\bg (\bk, \bx) = (\bg_1, \bg_2, \ldots, \bg_n )^{\intercal}$ as follows: \begin{equation} \label{eq:key_2_2} \bg_i (\bk, \bx) = \begin{cases} \bf_i (\bk, \bx), & \text{ for } 1 \leq i \leq s, \\[5pt] \bx \cdot \bv_{i-s} - \bx_0 \cdot \bv_{i-s}, & \text{ for } s+1 \leq i \leq n. \end{cases} \end{equation} From \eqref{eq:key_2_0}, we can check that $\bg (\bk, \bx) = \mathbf{0}$ if and only if $\bx \in \bx_0 + \mS_{G_1}$ is the steady state of the system $(G_1, \bk)$. Thus, $(\bk_1, \bx^*)$ can be considered as a solution to $\bg (\bk, \bx) = \mathbf{0}$, that is, $\bg (\bk_1, \bx^*) = \mathbf{0}$. Computing the Jacobian matrix of $\bg (\bk, \bx)$ as in Equation~\eqref{eq:key_2_2}, we get \begin{equation} \notag \mathbf{J}_{\bg, \bx} = \begin{pmatrix} \big[ \frac{\partial \bf_i}{ \partial \bx_j} \big]_{1 \leq i \leq s, 1 \leq j \leq n} \\[5pt] \bv_1 \\ \ldots \\ \bv_{n-s} \end{pmatrix}. \end{equation} From~\eqref{eq:key_2_1}, we have \[ \ker \big( \mathbf{J}_{\bg, \bx} |_{\bk = \bk_1, \bx = \bx^*} \big) \subseteq \mS^{\perp}_{G_1}. \] Since the last $n-s$ rows of $\mathbf{J}_{\bg} (\bx)$, $\{\bv_1, \bv_2, \ldots, \bv_{n-s} \}$, is a orthonormal basis of $\mS^{\perp}_{G_1}$, we derive \begin{equation} \label{eq:key_2_3} \det \big( \mathbf{J}_{\bg, \bx} |_{\bk = \bk_1, \bx = \bx^*} \big) \neq 0. \end{equation} Hence, the Jacobian matrix $\mathbf{J}_{\bg, \bx}$ is invertible at $(\bk, \bx) = (\bk_1, \bx^*)$. Further, note that $\bg (\bk, \bx)$ is continuously differentiable. Using the implicit function theorem, for any $\hbk \in U$, we have \begin{equation} \notag T (\hbk) = \hbx, \end{equation} where $\hbx \in (\bx_0 + \mS_{G_1} )\cap\mathbb{R}^n_{>0}$ is the steady state of $(G_1, \hbk)$. \end{proof} \begin{lemma} \label{lem:key_3} Suppose $\bx_0\in\mathbb{R}^n_{>0}$ and $\bk \in \dK (G,G_1)$, then there exists an open set $U \subset \dK (G,G_1)$ containing $\bk$, such that there exists a unique continuous function \begin{equation} \label{eq:key_3_1} h : U \rightarrow (\bx_0 + \mS_{G_1} )\cap\mathbb{R}^n_{>0}. \end{equation} such that for any $\hbk \in U$, \begin{equation} \label{eq:key_3_2} h (\hbk) = \hbx, \end{equation} where $\hbx \in (\bx_0 + \mS_{G_1} )\cap\mathbb{R}^n_{>0}$ is the steady state of $(G, \hbk)$. \end{lemma} \begin{proof} Given $\bk \in \dK (G, G_1)$ and $\bx_0 \in \mathbb{R}^n_{>0}$, there exists $\bk_1 \in \mK (G_1)$ such that \[ (G, \bk) \sim (G_1, \bk_1). \] Theorem \ref{thm:cb} shows the system $(G_1, \bk_1)$ has a unique steady state $\bx^* \in (\bx_0 + \mS_{G_1}) \cap \mathbb{R}^n_{>0}$. Since $(G, \bk) \sim (G_1, \bk_1)$, $\bx^* \in (\bx_0 + \mS_{G_1}) \cap \mathbb{R}^n_{>0}$ is also the unique steady state of the system $(G, \bk)$. Analogously, for any $\hbk \in \dK (G,G_1)$, it has a unique steady state of the system $(G, \hbk)$ in $(\bx_0 + \mS_{G_1}) \cap \mathbb{R}^n_{>0}$. Thus, the function $h$ in \eqref{eq:key_3_1}-\eqref{eq:key_3_2} is well-defined. It remains to prove that there exists an open set $U \subset \dK (G, G_1)$ containing $\bk$ and $h$ is continuous with respect to the domain $U$. From Lemma~\ref{lem:key_2}, there exists an open set $U_1 \subset \RR^{E_1}_{>0}$ containing $\bk_1$, such that there exists a unique continuously differentiable function \begin{equation} \label{eq:key_3_4} T : U_1 \rightarrow (\bx_0 + \mS_{G_1} )\cap\mathbb{R}^n_{>0}. \end{equation} such that for any $\hbk \in U_1$, \begin{equation} \notag T (\hbk) = \hbx, \end{equation} where $\hbx \in (\bx_0 + \mS_{G_1} )\cap\mathbb{R}^n_{>0}$ is the steady state of $(G_1, \hbk)$. Using \eqref{eq:key_3_4}, we can find a constant $\varepsilon_1 = \varepsilon_1 (\bk)$ such that \begin{equation} \label{eq:key_3_B} B = \{ \bk^* \in \RR^{E_1}_{>0}: \|\bk^* - \bk_1 \| \leq \varepsilon_1 \} \subseteq U_1. \end{equation} Hence, it is clear that $T$ is continuous with respect to the domain $B$. On the other hand, from Lemma \ref{lem:key_1}, there exist $\varepsilon = \varepsilon (\bk) > 0$ and $C = C (\bk) > 0$, such that for any $\hbk \in \dK (G,G_1)$ with $\| \hbk - \bk \| \leq \varepsilon$, there exists $\hbk_1 \in \mK (G_1,G_1)$ satisfying \begin{equation} \label{eq:key_3_3} \|\hbk_1 - \bk_1 \| \leq C \varepsilon \ \text{ and } \ (G,\hbk) \sim (G_1, \hbk_1). \end{equation} Now pick $\varepsilon_2 = \min ( \varepsilon, \varepsilon_1 / C)$, and consider the following set: \begin{equation} \notag U := \{ \bk^* \in \RR^{E}_{>0}: \|\bk^* - \bk \| < \varepsilon_2 \} \ \cap \ \dK (G,G_1). \end{equation} Using~\eqref{eq:key_3_3}, we have that for any $\bk^* \in U$, there exists $\bk^*_1 \in \mK (G_1,G_1)$ such that \begin{equation} \label{eq:key_3_5} \| \bk^*_1 - \bk_1 \| \leq C \varepsilon_2 = \varepsilon_1 \ \text{ and } \ (G, \bk^*) \sim (G_1, \bk^*_1). \end{equation} From \eqref{eq:key_3_B}, this shows that $\bk^*_1 \in B$. Further, from \eqref{eq:key_3_4} and \eqref{eq:key_3_3}, we obtain \[ h (\bk^*) = T (\bk^*_1) \] Since $T$ is continuous with respect to the domain $B$, together with \eqref{eq:key_3_5} and $\bk^*_1 \in B$, we conclude that $h$ is continuous on $U$. \end{proof} \begin{proposition} \label{lem:key_4} Suppose $\bx_0 \in \RR^n_{>0}$ and $\bk^* \in \mK (G_1) \subset \mK (G_1,G_1)$. For any $\bk \in \mK (G_1,G_1)$, then we have the following: \begin{enumerate}[label=(\alph*)] \item The system $(G_1, \bk^*)$ has a unique steady state $\bx^* \in (\bx_0 + \mS_{G_1}) \cap \mathbb{R}^n_{>0}$. \item The system $(G_1, \bk)$ has a unique steady state $\bx \in (\bx_0 + \mS_{G_1}) \cap \mathbb{R}^n_{>0}$. \item Consider the steady state $\bx^*$ in part $(a)$ and $\bx$ obtained in part $(b)$. Then there exists a unique $\hbk \in \RR^{E_1}$, such that \begin{enumerate}[label=(\roman*)] \item \label{lem:key_4_a} $(G_1, \bk) \sim (G_1, \hbk)$. \item\label{lem:key_4_b} $\hbJ := (\hat{k}_{\by \to \by'} \bx^{\by})_{\by \to \by' \in E_1} \in \hat{\mathcal{J}} (G_1)$. \item \label{lem:key_4_c} $\langle \hbJ, \bA_i \rangle = \langle \bJ^*, \bA_i \rangle$ for any $1 \leq i \leq a$, where $\bJ^* := (k^*_{\by \to \by'} (\bx^*)^{\by})_{\by \to \by' \in E_1}$. \end{enumerate} \item For any sequence $\{ \bk_i \}^{\infty}_{i = 1}$ in $\mK (G_1,G_1)$ converging to $\bk^*$, there exist a unique corresponding sequence $\{ \hbk_i \}^{\infty}_{i = 1}$ obtained from part $(c)$. Moreover, the sequence $\{ \hbk_i \}^{\infty}_{i = 1}$ satisfies \begin{equation} \notag \hbk_i \to \bk^* \ \text{ as } \ i \to \infty. \end{equation} \end{enumerate} \end{proposition} \begin{proof} For part (a), since $\bk^* \in \mK (G_1)$, Theorem \ref{thm:cb} shows that the system $(G_1, \bk^*)$ has a unique steady state $\bx^* \in (\bx_0 + \mS_{G_1}) \cap \mathbb{R}^n_{>0}$. \smallskip For part (b), given $\bk \in \mK (G_1,G_1)$, there exists some $\bk' \in \mK (G_1)$, such that \begin{equation} \label{eq:key_4_3} (G_1, \bk) \sim (G_1, \bk'). \end{equation} Thus, by Theorem \ref{thm:cb}, the systems $(G_1, \bk)$ and $(G_1, \bk')$ share a unique steady state in $(\bx_0 + \mS_{G_1}) \cap \mathbb{R}^n_{>0}$, denoted by $\bx$. \smallskip For part (c), define $\bJ' := (k'_{\by \to \by'} \bx^{\by})_{\by \to \by' \in E_1}$, then we construct a flux vector on $G_1$ as follows: \begin{equation} \label{eq:key_4_4} \hbJ := \bJ' + \sum\limits^{a}_{i=1} (\langle \bJ^*, \bA_i \rangle - \langle \bJ', \bA_i \rangle) \bA_i. \end{equation} Under direct computation, we have \begin{equation} \label{eq:key_4_5} \langle \hbJ, \bA_i \rangle = \langle \bJ^*, \bA_i \rangle \ \text{ for any } \ 1 \leq i \leq a. \end{equation} Note that $\bk' \in \mK (G_1)$ and $\{\bA_i \}^{a}_{i=1} \in \eJ(G) \subset \hat{\mathcal{J}} (G_1)$, then \eqref{eq:key_4_4} show that \begin{equation} \label{eq:key_4_5.5} \bJ' \in \mathcal{J} (G_1) \ \text{ and } \ \hbJ \in \hat{\mathcal{J}} (G_1). \end{equation} Consider the flux vector $\bJ := (k_{\by \to \by'} \bx^{\by})_{\by \to \by' \in E_1}$. Using Proposition \ref{prop:craciun2020efficient} and \eqref{eq:key_4_3}, we deduce \begin{equation} \notag (G_1, \bJ) \sim (G_1, \bJ'). \end{equation} From Lemma \ref{lem:j0}, this shows $\bJ' - \bJ \in \mD (G_1)$. Together with \eqref{eq:key_4_4}, we get \begin{equation} \notag \hbJ - \bJ \in \mD (G_1). \end{equation} Hence, we rewrite $\hbJ$ as \begin{equation} \label{eq:key_4_6} \hbJ = \bJ + \bv \ \text{ with } \ \bv \in \mD (G_1). \end{equation} Now we set the reaction rate vector as \begin{equation} \label{eq:key_4_6.5} \hbk := ( \frac{\hbJ}{\bx^{\by}} )_{\by \to \by' \in E_1} \in \RR^{E_1}. \end{equation} Using Proposition \ref{prop:craciun2020efficient} and \eqref{eq:key_4_6}, we obtain $(G_1, \bk) \sim (G_1, \hbk)$. Together with \eqref{eq:key_4_5} and \eqref{eq:key_4_5.5}, we derive that the reaction rate vector $\hbk$ satisfies conditions \ref{lem:key_4_a}, \ref{lem:key_4_b} and \ref{lem:key_4_c}. We now show the uniqueness of the vector $\hbk$. Suppose there exists another reaction rate vector $\hbk_1$ satisfying conditions \ref{lem:key_4_a}-\ref{lem:key_4_c}. From the condition \ref{lem:key_4_a}, we have \[ (G_1, \hbk) \sim (G_1, \hbk_1). \] From the condition \ref{lem:key_4_b}, we get \[ \hbJ_1 := (\hat{k}_{1, \by \to \by'} \bx^{\by})_{\by \to \by' \in E_1} \in \hat{\mathcal{J}} (G_1). \] Then Proposition \ref{prop:craciun2020efficient} and Lemma \ref{lem:j0} show \[ (G_1, \hbJ) \sim (G_1, \hbJ_1) \ \text{ and } \ \hbJ_1 - \hbJ \in \eJ (G_1). \] Using the condition \ref{lem:key_4_c}, we obtain \[ \langle \hbJ, \bA_i \rangle = \langle \hbJ_1, \bA_i \rangle \ \text{ for any } \ 1 \leq i \leq a. \] Since $\{\bA_i \}^{a}_{i=1}$ is an orthonormal basis of the subspace $\eJ(G)$, this implies that \[ \hbJ_1 - \hbJ \in \big( \eJ (G_1) \big)^{\perp}. \] Hence, $\hbJ_1 - \hbJ = \mathbf{0}$ and $\hbk_1 = \hbk$. Therefore, we conclude the uniqueness. \smallskip For part (d), we will prove it in a sequence of three steps. \smallskip \textbf{Step 1: } Assume a sequence of reaction rate vectors $\bk_i \in \mK (G_1,G_1)$ with $i \in \mathbb{N}$, such that \[ \bk_i \to \bk^* \ \text{ as } \ i \to \infty. \] Analogously, there exists some $\bk'_i \in \mK (G_1)$, such that $(G_1, \bk_i) \sim (G_1, \bk'_i)$. Moreover, two systems $(G_1, \bk_i)$ and $(G_1, \bk'_i)$ share a unique steady state $\bx^i \in (\bx_0 + \mS_{G_1}) \cap \mathbb{R}^n_{>0}$. Follow the steps in \eqref{eq:key_4_3}-\eqref{eq:key_4_5}, we obtain the corresponding sequences of flux vector as follows: \begin{equation} \begin{split} \label{eq:key_4_7} & \bJ_i := (k_{i, \by \to \by'} (\bx^i)^{\by})_{\by \to \by' \in E_1} \ \text{ with } \ i \in \mathbb{N}, \\& \bJ'_i := (k'_{i, \by \to \by'} (\bx^i)^{\by})_{\by \to \by' \in E_1} \ \text{ with } \ i \in \mathbb{N}. \end{split} \end{equation} and \begin{equation} \label{eq:key_4_8} \hbJ_i := \bJ'_i + \sum\limits^{a}_{j=1} (\langle \bJ^*, \bA_j \rangle - \langle \bJ'_i, \bA_j \rangle) \bA_j \ \text{ with } \ i \in \mathbb{N}. \end{equation} Under direct computation, for any $i \in \mathbb{N}$, \begin{equation} \label{eq:key_4_8.5} \langle \hbJ_i, \bA_j \rangle = \langle \bJ^*, \bA_j \rangle \ \text{ for any } \ 1 \leq j \leq a, \end{equation} and similar from \eqref{eq:key_4_5.5}, we have \begin{equation} \label{eq:key_4_12} \hbJ_i \in \hat{\mathcal{J}} (G_1) \ \text{ for any } \ i \in \mathbb{N}. \end{equation} Using Proposition \ref{prop:craciun2020efficient} and $(G_1, \bk_i) \sim (G_1, \bk'_i)$, we deduce \begin{equation} \notag (G_1, \bJ_i) \sim (G_1, \bJ'_i) \ \text{ for any } \ i \in \mathbb{N}. \end{equation} From Lemma \ref{lem:j0}, together with \eqref{eq:key_4_8}, we get \begin{equation} \notag \hbJ_i - \bJ_i \in \mD (G_1) \ \text{ for any } \ i \in \mathbb{N}. \end{equation} Thus, for any $i \in \mathbb{N}$, $\hbJ_i$ can be expressed as \begin{equation} \label{eq:key_4_9} \hbJ_i = \bJ_i + \bv^i \ \text{ with } \ \bv^i \in \mD (G_1). \end{equation} On the other hand, using Lemma \ref{lem:key_2}, together with $\bk_i \to \bk^*$ as $i \to \infty$, we have \begin{equation} \notag \bx^i \to \bx^* \ \text{ as } \ i \to \infty. \end{equation} Combining with \eqref{eq:key_4_7}, we derive that \begin{equation} \label{eq:key_4_10} \bJ_i \to \bJ^* \ \text{ as } \ i \to \infty. \end{equation} \smallskip \textbf{Step 2: } Now we claim that \begin{equation} \label{eq:key_4_13} \| \bv^i \|_{\infty} \to 0 \ \text{ as } \ i \to \infty. \end{equation} We prove this by contradiction. Suppose not, w.l.o.g. there exists a subsequence $\{\bv^{i_l} \}^{\infty}_{l=1}$, such that for any $l \in \mathbb{N}$, \begin{equation} \notag \| \bv^{i_l} \|_{\infty} \geq 1. \end{equation} Then we consider the sequence $\{ \bw^l \}^{\infty}_{l=1}$ as follows: \begin{equation} \label{eq:key_4_14} \bw^{l} = \frac{\bv^{i_l}}{\| \bv^{i_l} \|_{\infty}} \ \text{ with } \ l \in \mathbb{N}. \end{equation} It is clear that $\| \bw^{l} \|_{\infty} = 1$ for any $l \in \mathbb{N}$. From the Bolzano–Weierstrass theorem, there exists a subsequence $\{ \bw^{l_j} \}^{\infty}_{j=1}$, such that \begin{equation} \notag \bw^{l_j} \to \bw^* \ \text{ as } \ j \to \infty. \end{equation} Recall from \eqref{eq:key_4_9} and \eqref{eq:key_4_14}, we have for any $j \in \mathbb{N}$, \begin{equation} \label{eq:key_4_15} \bw^{l_j} = \frac{\bv^{i_{l_j}}}{\| \bv^{i_{l_j}} \|_{\infty}} = \frac{1}{\| \bv^{i_{l_j}} \|_{\infty}} \big( \hbJ_{i_{l_j}} - \bJ_{i_{l_j}} \big). \end{equation} Since $\bv^i \in \mD (G_1)$, together with $\| \bv^{i_l} \|_{\infty} \geq 1$, we obtain that \[ \bw^{l_j} \in \mD (G_1). \] Note that $\mD (G_1)$ is a linear subspace of finite dimension. Therefore, $\bw^{l_j} \to \bw^*$ implies \begin{equation} \label{eq:key_4_16} \bw^* \in \mD (G_1). \end{equation} Let $\bz \in \big( \hat{\mathcal{J}} (G_1) \big)^{\perp}$. From \eqref{eq:key_4_12}, we have for any $j \in \mathbb{N}$, \begin{equation} \label{eq:key_4_17} \langle \hbJ_{i_{l_j}}, \bz \rangle = 0. \end{equation} From \eqref{eq:key_4_10} and $\bJ \in \mathcal{J} (G_1)$, we obtain \begin{equation} \label{eq:key_4_18} \langle \bJ_{i_{l_j}}, \bz \rangle \to \langle \bJ, \bz \rangle = 0 \ \text{ as } \ j \to \infty. \end{equation} Using \eqref{eq:key_4_15}, \eqref{eq:key_4_17} and \eqref{eq:key_4_18}, together with $\| \bv^{i_l} \|_{\infty} \geq 1$ and $\bw^{l_j} \to \bw^*$, we derive \begin{equation} \notag \langle \bw^{l_j}, \bz \rangle \to \langle \bw^*, \bz \rangle = 0. \end{equation} Since $\bz$ is arbitrary in $\big( \hat{\mathcal{J}} (G_1) \big)^{\perp}$, this shows $\bw^* \in \hat{\mathcal{J}} (G_1)$. Together with \eqref{eq:key_4_16}, we get \begin{equation} \label{eq:key_4_19} \bw^* \in \eJ (G_1). \end{equation} Recall that $\{\bA_i \}^{a}_{i=1}$ is an orthonormal basis of the subspace $\eJ(G)$. Without loss of generality, we pick $\bA_1 \in \eJ(G)$. From \eqref{eq:key_4_8.5} and \eqref{eq:key_4_10}, we get \begin{equation} \notag \langle \hbJ_{i_{l_j}} - \bJ_{i_{l_j}}, \bA_1 \rangle = \langle \bJ^*, \bA_1 \rangle - \langle \bJ_{i_{l_j}}, \bA_1 \rangle \to 0 \ \text{ as } \ j \to \infty. \end{equation} Together with $\| \bv^{i_l} \|_{\infty} \geq 1$ and $\bw^{l_j} \to \bw^*$, we derive \begin{equation} \notag \langle \bw^{l_j}, \bA_1 \rangle \to \langle \bw^*, \bA_1 \rangle = 0. \end{equation} Analogously, we can get $\langle \bw^*, \bA_j \rangle = 0$ for any $1 \leq j \leq a$. This shows that \begin{equation} \label{eq:key_4_20} \bw^* \in \big( \eJ (G_1) \big)^{\perp}. \end{equation} Combining \eqref{eq:key_4_19} with \eqref{eq:key_4_20}, we conclude that $\bw^* = \mathbf{0}$. Since $\| \bw^{l} \|_{\infty} = 1$ for any $l \in \mathbb{N}$, this contradicts with $\bw^{l_j} \to \bw^*$ as $j \to \infty$. Therefore, we prove the claim. \smallskip \textbf{Step 3: } Using \eqref{eq:key_4_9}, \eqref{eq:key_4_10} and \eqref{eq:key_4_13}, we derive that \begin{equation} \label{eq:key_4_21} \hbJ_i = \bJ_i + \bv^i \to \bJ^* \ \text{ as } \ i \to \infty. \end{equation} Since $\bJ \in \mathcal{J} (G_1) \subset \RR^{E_1}_{>0}$, there exists sufficiently large $N$, such that \begin{equation} \notag \hbJ_i \in \RR^{E_1}_{>0} \ \text{ for any } \ i > N. \end{equation} Together with \eqref{eq:key_4_12} and Remark \ref{rmk:hat_j_g1_g}, we obtain that \[ \hbJ_i \in \hat{\mathcal{J}} (G_1) \cap \RR^{|E_1|}_{>0} = \mathcal{J} (G_1) \ \text{ for any } \ i > N. \] Following \eqref{eq:key_4_6.5}, we set $\{ \hbk_i\}^{\infty}_{i=1}$ as follows: \begin{equation} \label{eq:key_4_22} \hbk_i := \big( \frac{\hat{J}_{i, \by \to \by'} }{(\bx^i)^{\by}} \big)_{\by \to \by' \in E_1} \ \text{ with } \ i \in \mathbb{N}. \end{equation} Note that $\bx^i \in (\bx_0 + \mS_{G_1}) \cap \mathbb{R}^n_{>0}$ and $\hbJ_i \in \mathcal{J} (G_1)$ for any $i > N$, we get \begin{equation} \notag \hbk_i \in \mK (G_1) \ \text{ for any } \ i > N. \end{equation} Using \eqref{eq:key_4_9} and Proposition \ref{prop:craciun2020efficient}, we derive \begin{equation} \notag (G_1, \bk_i) \sim (G_1, \hbk_i). \end{equation} Finally, using $\hbJ_i \to \bJ^*$ and $\bx^i \to \bx^*$, together with $\bJ^* = (k^*_{\by \to \by'} (\bx^*)^{\by})_{\by \to \by' \in E_1}$, we have \begin{equation} \hbk_i \to \bk^* \ \text{ as } \ i \to \infty. \end{equation} Therefore, we conclude the proof of this Proposition. \end{proof} Now we are ready to prove Proposition~\ref{prop:inverse_cts_k}. \begin{proof}[Proof of Proposition \ref{prop:inverse_cts_k}] Given fixed $\bq = (q_1, q_2, \ldots, q_a) \in \RR^a$, consider $\bk \in \dK(G,G_1)$ such that \begin{equation} \notag \Phi (\bk, \bq) = (\hat{\bJ},\bx, \bp). \end{equation} Follow definition, there exists $\bk_1 \in \mK (G_1) \subset \mK_{\RR} (G_1,G)$ satisfying \[ (G, \bk) \sim (G_1, \bk_1). \] Remark \ref{rmk:de_ss} shows $\bx \in (\bx_0 + \mS_{G_1} )\cap\mathbb{R}^n_{>0}$ is the steady state of $(G_1, \bk_1)$ and $(G, \bk)$. From Lemma \ref{lem:phi_wd}, by setting \begin{equation} \label{eq:cts_k_1} \bJ = \big( k_{1, \by\rightarrow \by'} \bx^{\by} \big)_{\by\rightarrow \by' \in E_1}, \end{equation} then we obtain \begin{equation} \label{eq:cts_k_2} \hbJ = \bJ + \sum\limits^a_{j=1} (q_j - \langle \bJ, \bA_j \rangle ) \bA_j \in \hat{\mJ} (G_1,G). \end{equation} Moreover, from \eqref{def:phi_kq} we obtain \begin{equation} \notag \bp = ( \langle \bk, \bB_1 \rangle, \langle \bk, \bB_2 \rangle, \ldots, \langle \bk, \bB_b \rangle), \end{equation} which is continuous with respect to $\bk$. \smallskip Now assume any sequence $\{ \bk^i \}^{\infty}_{i = 1}$ in $\dK(G,G_1)$, such that \begin{equation} \label{eq:cts_k_3} \bk^i \to \bk \ \text{ as } \ i \to \infty. \end{equation} Suppose $\Phi (\bk^i, \bq) = (\hbJ^i, \bx^i, \bp^i)$ with $i \in \mathbb{N}$, then $\bx^i \in (\bx_0 + \mS_{G_1} )\cap\mathbb{R}^n_{>0}$ is the steady state of $(G_1, \bk^i)$. Using Lemma \ref{lem:key_3}, together with $\bk^i \to \bk$ in \eqref{eq:cts_k_3}, we derive \begin{equation} \label{eq:cts_k_4} \bx^i \to \bx \ \text{ as } \ i \to \infty. \end{equation} From Lemma \ref{lem:key_1}, there exists a sequence $\{ \bk^i_1 \}^{\infty}_{i = 1}$ in $\mK (G_1,G_1)$, such that \begin{equation} \notag (G, \bk^i) \sim (G_1, \bk^i_1) \ \text{ for any } \ i \in \mathbb{N}, \end{equation} and \begin{equation} \label{eq:cts_k_5} \bk^i_1 \to \bk_1 \ \text{ as } \ i \to \infty. \end{equation} Then apply Proposition \ref{lem:key_4}, there exists a corresponding sequence $\{ \hbk_i \}^{\infty}_{i = 1}$, such that \begin{equation} \notag (G_1, \hbk_i) \sim (G_1, \bk^i_1) \ \text{ for any } \ i \in \mathbb{N}, \end{equation} Set $\hbJ_i = (\hat{k}_{i, \by \to \by'} (\bx^i)^{\by})_{\by \to \by' \in E_1}$, then for any $i \in \mathbb{N}$, \begin{equation} \label{eq:cts_k_6} \hbJ_i \in \hat{\mathcal{J}} (G_1) \ \text{ and } \ \langle \hbJ_i, \bA_j \rangle = \langle \bJ, \bA_j \rangle \ \text{ for any } \ 1 \leq j \leq a. \end{equation} Moreover, from $\bk^i_1 \to \bk_1$ in \eqref{eq:cts_k_5}, we have \begin{equation} \notag \hbk_i \to \bk_1 \ \text{ as } \ i \to \infty. \end{equation} Together with $\bx^i \to \bx$ in \eqref{eq:cts_k_4} and $\bJ$ in \eqref{eq:cts_k_1}, we derive that \begin{equation} \label{eq:cts_k_7} \hbJ_i \to \bJ \ \text{ as } \ i \to \infty. \end{equation} Since $\bJ \in \mathcal{J} (G_1)$ and $\hbJ_i \in \hat{\mathcal{J}} (G_1)$, this shows there exists a sufficiently large $N$, such that \begin{equation} \label{eq:cts_k_8} \hbJ_i \in \mathcal{J} (G_1) \ \text{ for any } \ i > N. \end{equation} Note that $(G_1, \hbk_i) \sim (G_1, \bk^i_1) \sim (G_1, \bk^i)$, thus $\bx^i$ is also the steady state of $(G_1, \hbk_i)$. Since $\hbJ_i = (\hat{k}_{i, \by \to \by'} (\bx^i)^{\by})_{\by \to \by' \in E_1}$, together with \eqref{eq:cts_k_8}, we deduce \begin{equation} \notag \hbk_i \in \mK (G_1) \ \text{ for any } \ i > N. \end{equation} Note that $\Phi (\bk^i, \bq) = (\hbJ^i, \bx^i, \bp^i)$. From \eqref{eq:cts_k_2}, we obtain \begin{equation} \notag \hbJ^i = \hbJ_i + \sum\limits^a_{j=1} (q_j - \langle \hbJ_i, \bA_j \rangle ) \bA_j \ \text{ for any } \ i > N. \end{equation} Using \eqref{eq:cts_k_6} and \eqref{eq:cts_k_7}, we have \begin{equation} \notag \hbJ^i \to \bJ \ \text{ as } \ i \to \infty. \end{equation} Recall that $\Phi (\bk, \bq) = (\bJ, \bx, \bp)$. Suppose any sequence $\bk^i \to \bk$ with $\Phi (\bk^i, \bq) = (\hbJ^i, \bx^i, \bp^i)$, we show the continuity on $\bp$, $\bx^i \to \bx$ and $\hbJ^i \to \bJ$. Therefore, we conclude that $\Phi (\cdot, \bq)$ is continuous with respect to $\bk$. \end{proof} Here we state the first main theorem in this paper. \begin{theorem} \label{thm:inverse_cts} Consider the map $\hPsi$ in Definition \ref{def:hpsi}, then the map $\hPsi^{-1}$ is continuous. \end{theorem} \begin{proof} From Lemma \ref{lem:phi_wd}, consider the map $\Phi$ in Definition \ref{def:phi}, then $\Phi = \hPsi^{-1}$ is well-defined and bijective. Thus, it suffices to show the map $\Phi$ is continuous. Suppose any $(\bk, \bq) \in \dK(G,G_1) \times \RR^a$. Consider any positive real number $\varepsilon > 0$. From Proposition \ref{prop:inverse_cts_k}, $\Phi (\cdot, \bq)$ is continuous with respect to $\bk$. Thus, there exists some positive real number $\delta_1 > 0$, such that for any $\tilde{\bk} \in \dK(G,G_1)$ with $\| \tilde{\bk} - \bk \| < \delta_1$, then \begin{equation} \label{eq:inverse_cts_1} \big\| \Phi (\tilde{\bk}, \bq) - \Phi (\bk, \bq) \big\| < \frac{\varepsilon}{2}. \end{equation} Note that $\{\bA_1, \bA_2, \ldots, \bA_a \}$ is an orthonormal basis of $\eJ(G_1) \subset \RR^a$, there exists some positive real number $\delta_2 > 0$, such that for any $\bv = (v_1, v_2, \ldots, v_a) \in \RR^a$ with $\| \bv \| < \delta_2$, then \begin{equation} \label{eq:inverse_cts_2} \big\| \sum\limits^{a}_{i=1} v_i \bA_i \big\| < \frac{\varepsilon}{2}. \end{equation} Let $\delta = \min \{ \delta_1, \delta_2 \}$, consider any $(\hbk, \hbq) \in \dK(G,G_1) \times \RR^a$ with $| (\hbk, \hbq) - (\bk, \bq) | < \delta$. This implies $\| \hbk - \bk \| < \delta$ and $\| \hbq - \bq \| < \delta$. Then we compute that \begin{equation} \label{eq:inverse_cts_3} \Phi (\hbk, \hbq) - \Phi (\bk, \bq) = \big( \Phi (\hbk, \hbq) - \Phi (\bk, \hbq) \big) + \big( \Phi (\bk, \hbq) - \Phi (\bk, \bq) \big). \end{equation} From \eqref{eq:inverse_cts_1} and $\| \hbk - \bk \| < \delta \leq \delta_1$, we have \begin{equation} \label{eq:inverse_cts_4} \big\| \Phi (\hbk, \hbq) - \Phi (\bk, \hbq) \big\| < \frac{\varepsilon}{2}. \end{equation} Using Lemma \ref{lem:inverse_cts_q} and setting $\hbq - \bq := (v_1, v_2, \ldots, v_a) \in \RR^a$, we have \begin{equation} \notag \Phi (\bk, \hbq) - \Phi (\bk, \bq) = \sum\limits^{a}_{i=1} v_i \bA_i, \end{equation} Together with \eqref{eq:inverse_cts_2} and $\| \hbq - \bq \| < \delta \leq \delta_2$, we obtain \begin{equation} \label{eq:inverse_cts_5} \big\| \Phi (\bk, \hbq) - \Phi (\bk, \bq) \big\| = \big\| \sum\limits^{a}_{i=1} v_i \bA_i \big \| < \frac{\varepsilon}{2}. \end{equation} Inputting \eqref{eq:inverse_cts_4} and \eqref{eq:inverse_cts_5} into \eqref{eq:inverse_cts_3}, we derive \begin{equation} \notag \big\| \Phi (\hbk, \hbq) - \Phi (\bk, \bq) \big\| \leq \frac{\varepsilon}{2} + \frac{\varepsilon}{2} = \varepsilon. \end{equation} Therefore, $\Phi$ is continuous and we conclude this theorem. \end{proof} The following result is a direct consequence of Theorem \ref{thm:inverse_cts}. \begin{theorem} \label{thm:hpsi_homeo} The map $\hPsi$ in Definition \ref{def:hpsi} is a homeomorphism. \end{theorem} \begin{proof} From Lemma \ref{lem:hpsi_bijective} and \ref{lem:hpsi_cts}, we derive that $\hPsi$ is bijective and continuous. On the other hand, Proposition \ref{thm:inverse_cts} shows the inverse map $\hPsi^{-1}$ is also continuous. Therefore, we conclude that the map $\hPsi$ is a homomorphism. \end{proof} \section{Dimension of \texorpdfstring{$\dK(G,G_1)$}{KGG1} and \texorpdfstring{$\pK(G,G_1)$}{pKGG1} } \label{sec:dimension} In this section, we give a precise bound on the dimension of $\dK(G, G_1)$, where $G_1 \sqsubseteq G_c$. Further, we show the dimension of $\pK(G, G_1)$ when $\pK(G, G_1) \neq \emptyset$. Finally, we remark on the dimension of {\em $\RR$-disguised toric locus} $\dK(G)$ and {\em disguised toric locus} $\pK(G)$. \begin{lemma} \label{lem:hat_j_g1_g_cone} Let $G_1 = (V_1, E_1)$ be a weakly reversible E-graph and let $G = (V, E)$ be an E-graph. If $\mJ (G_1, G) \neq \emptyset$, then $\hat{\mJ} (G_1, G)$ is a convex cone, which satisfies \begin{equation} \label{hat_j_g1_g_generator_dim} \dim (\hat{\mJ} (G_1, G)) = \dim (\mJ (G_1, G)). \end{equation} \end{lemma} \begin{proof} From Lemma \ref{lem:j_g1_g_cone}, suppose there exists a set of vectors $\{ \bv_1, \bv_2, \ldots, \bv_k \} \subset \RR^{|E_1|}$, such that \begin{equation} \notag \mJ (G_1, G) = \{ a_1 \bv_1 + \cdots a_k \bv_k \ | \ a_i \in \RR_{>0} \}. \end{equation} Using \eqref{def:hat_j_g1_g}, $\hat{\mJ} (G_1, G)$ can be represented as the positive combination of the following vectors: \begin{equation} \label{hj_g1g_basis} \{ \bv_1, \bv_2, \ldots, \bv_k, \pm \bA_1, \pm \bA_2, \ldots, \pm \bA_a \}. \end{equation} This shows $\hat{\mJ} (G_1, G)$ is a convex cone. Moreover, we have \begin{equation} \notag \dim (\hat{\mJ} (G_1, G)) =\dim ( \spn \{ \bv_1, \bv_2, \ldots, \bv_k, \bA_1, \bA_2, \ldots, \bA_a \} ). \end{equation} Since $\mJ (G_1, G) \neq \emptyset$, Lemma \ref{lem:j_g1_g_cone} shows that \begin{equation} \notag \spn \{ \bA_i \}^a_{i=1} = \eJ(G_1) \subseteq \spn \{ \bv_1, \bv_2, \ldots, \bv_k \}. \end{equation} Therefore, we conclude that \begin{equation} \notag \dim (\hat{\mJ} (G_1, G)) = \dim ( \spn \{ \bv_1, \bv_2, \ldots, \bv_k \} ) = \dim (\mJ (G_1, G)). \end{equation} \end{proof} \begin{theorem} \label{thm:dim_kisg} Let $G_1 = (V_1, E_1)$ be a weakly reversible E-graph with its stoichiometric subspace $\mS_{G_1}$. Suppose an E-graph $G = (V, E)$, recall $\mJ (G_1,G)$, $\mD(G)$ and $\eJ(G_1)$ defined in Definitions~\ref{def:flux_realizable}, \ref{def:d0} and \ref{def:j0} respectively. \begin{enumerate}[label=(\alph*)] \item\label{part_a} Consider $\dK(G,G_1)$ from Definition~\ref{def:de_realizable}, then \begin{equation} \label{eq:dim_kisg} \begin{split} & \dim(\dK(G,G_1)) = \dim (\mJ(G_1,G)) + \dim (\mS_{G_1}) + \dim(\eJ(G_1)) - \dim(\mD(G)). \end{split} \end{equation} \item\label{part_b} Further, consider $\pK (G, G_1)$ from Definition~\ref{def:de_realizable} and assume that $\pK (G, G_1) \neq \emptyset$. Then \begin{equation} \label{eq:dim_kdisg} \dim(\pK (G,G_1)) = \dim(\dK(G,G_1)). \end{equation} \end{enumerate} \end{theorem} \begin{proof} For part $(a)$, recall we prove that $\hat{\Psi}$ is a homeomorphism in Theorem \ref{thm:hpsi_homeo}. Using the invariance of dimension theorem \cite{hatcher2005algebraic,munkres2018elements}, together with Remark \ref{rmk:semi_algebaic} and \eqref{hat_j_g1_g_generator_dim} in Lemma \ref{lem:hat_j_g1_g_cone}, we obtain \begin{equation} \notag \dim (\dK(G, G_1)) + \dim(\mD(G)) = \dim (\mJ (G_1, G)) + \dim (\mS_{G_1}) + \dim(\eJ(G_1)), \end{equation} and conclude \eqref{eq:dim_kisg}. Further, we emphasize that on a dense open subset of $\dK(G, G_1)$, it is locally a submanifold. The homomorphism indicates that all such submanifolds have the same dimension. \smallskip For part $(b)$, since $\pK (G, G_1) \neq \emptyset$, together with Lemma \ref{lem:semi_algebaic} and Remark \ref{rmk:semi_algebaic}, there exists a $\bk \in \pK(G, G_1)$ and a neighborhood of $\bk$ in $\pK(G, G_1)$, denoted by $U$, such that \[ \bk \in U \subset \pK(G, G_1), \] where $U$ is a submanifold with $\dim (U) = \dim (\pK(G, G_1))$. Moreover, $\pK (G, G_1) = \dK(G, G_1) \cap \mathbb{R}^{E}_{>0}$ implies that $U$ is also a neighborhood of $\bk$ in $\dK(G, G_1)$. From part $(a)$, we obtain that on a dense open subset of $\dK(G, G_1)$, all local submanifolds have the same dimension. Therefore, we conclude \eqref{eq:dim_kdisg}. \end{proof} \begin{theorem} \label{thm:dim_kisg_main} Consider an E-graph $G = (V, E)$. \begin{enumerate}[label=(\alph*)] \item Consider $\dK(G)$ from Definition~\ref{def:de_realizable}, then \begin{equation} \notag \dim (\dK(G) ) = \max_{G'\sqsubseteq G_c} \Big\{ \dim (\mJ(G',G)) + \dim (\mS_{G'}) + \dim(\eJ(G')) - \dim(\mD(G)) \Big\}, \end{equation} where $\mJ (G',G)$, $\mD(G)$ and $\eJ(G')$ are defined in Definitions~\ref{def:flux_realizable}, \ref{def:d0} and \ref{def:j0} respectively. \item Further, consider $\pK (G)$ from Definition~\ref{def:de_realizable} and assume that $\pK (G) \neq \emptyset$. Then \begin{equation} \notag \begin{split} & \dim (\pK(G) ) \\& = \max_{ \substack{ G'\sqsubseteq G_c, \\ \pK(G, G') \neq \emptyset } } \Big\{ \dim (\mJ(G',G)) + \dim (\mS_{G'}) + \dim(\eJ(G')) - \dim(\mD(G)) \Big\}. \end{split} \end{equation} \end{enumerate} \end{theorem} \begin{proof} Note that from Lemma~\ref{lem:semi_algebaic}, both $\pK (G, G_1)$ and $\dK(G, G_1)$ are semialgebraic sets. Further, the dimension of the union of finitely many semialgebraic sets is the maximum of the dimensions of these semialgebraic sets \cite{coste2000introduction,basu201738,lairez2021computing}. The result then follows from Definition~\ref{def:de_realizable}, Lemma \ref{lem:semi_algebaic} and Theorem~\ref{thm:dim_kisg}. \end{proof} \section{Examples} \label{sec:applications} \begin{example}[Thomas type models {\cite[Chapter 6]{murray2007mathematical}}] \label{ex:thomas} This model illustrates the substrate inhibition mechanism, capturing the interaction between uric acid and oxygen catalyzed by the enzyme uricase. The E-graph corresponding to this model, shown in Figure~\ref{fig:thomas_model} (a), is denoted by $G$. The system is governed by the following differential equations: \begin{equation} \begin{split} \frac{du}{dt} & = c - u - uv, \\ \frac{dv}{dt} & = \beta(d-v) - uv, \end{split} \end{equation} where $u$ denotes the concentration of uric acid, $v$ represents the concentration of oxygen, and the parameters $c$, $d$, and $\beta$ are positive constants. As demonstrated in \cite{CraciunDickensteinShiuSturmfels2009}, the toric locus associated with $G$ forms a toric variety of codimension one, implying that it has measure zero. \begin{figure}[!ht] \centering \includegraphics[scale=0.7]{thomas_model_2.eps} \caption{ (a) The E-graph $G$ represents a Thomas-type model, where $U$ denotes uric acid and $V$ denotes oxygen. (b) The E-graph $G_1$ is a weakly reversible subgraph of the complete graph formed by the source vertices of $G$. } \label{fig:thomas_model} \end{figure} The graph $G$ contains 6 reactions, hence $\pK(G) \subseteq \mathbb{R}^6_{>0}$. We claim that \[ \rm{dim}(\pK(G)) = 6. \] This implies that the disguised toric locus corresponding to $G$ is of positive measure. To prove this claim, we consider the E-graph $G_1$ and compute $\dim (\pK(G, G_1))$. First, consider a flux vector $\bJ \in \mJ(G_1, G) \subset \mathbb{R}^7$. Since $\bJ$ is a complex-balanced flux vector in $G_1$, this imposes $4 - 1 = 3$ constraints on $\bJ$, as shown in \cite{craciun2020efficient}. Additionally, being $\mathbb{R}$-realizable in $G$ imposes no further constraints, since every flux vector in $G_1$ can be transformed into $G$. Therefore, we have \[ \dim (\mJ(G_1, G)) = 7 - 0 - 3 = 4. \] Second, upon inspecting the graphs $G$ and $G_1$, we obtain the following: \[ \dim (\mathcal{S}_{G_1}) = 2, \ \ \rm{dim}(\mD (G)) = 0 \ \text{ and } \ \dim (\eJ(G_1)) = 0. \] Using Theorem~\ref{thm:dim_kisg} (a), we get that \begin{equation} \begin{split} \notag \rm{dim}(\dK(G, G_1)) & = \dim (\mJ(G_1, G)) + \dim (\mathcal{S}_{G_1}) + \dim(\mD (G)) - \dim(\eJ (G_1)) \\& = 4 + 2 + 0 - 0 = 6. \end{split} \end{equation} Since $G_1$ is weakly reversible, $\mK ({G_1}) \neq \emptyset$. From Figure~\ref{fig:thomas_model}, given any $\bk_1 \in \mK ({G_1})$ there exists $\bk$ such that $(G,\bk)\sim (G_1, \bk_1)$. This implies that $\pK (G, G_1) \neq \emptyset$, and thus Theorem~\ref{thm:dim_kisg} (b) further shows \[ \rm{dim}(\pK(G,G_1)) = \rm{dim}(\dK(G,G_1)) = 6. \] This, together with Theorem \ref{thm:dim_kisg_main}, implies that $\rm{dim}(\pK(G)) = 6$. A lengthy computation (based on the matrix-tree theorem, see \cite{CraciunDickensteinShiuSturmfels2009}) can be used to derive that \[ \pK(G) \ \supseteq \ \{ \bk \in \mathbb{R}^{7}_{>0} \mid \frac{k_{V \to \emptyset} k_{U \to \emptyset}}{k_{U+V \to U}} > k_{\emptyset \to V} \geq k_{\emptyset \to U} \ \text{ or } \ \frac{k_{V \to \emptyset} k_{U \to \emptyset}}{k_{U+V \to U}} > k_{\emptyset \to U} \geq k_{\emptyset \to V} \}, \] which, informally speaking, implies that at least 50\% of all positive parameter choices belong to $\pK(G)$. Finally, we remark that the Thomas-type model $G$ and the corresponding weakly reversible E-graph $G_1$ in Figure~\ref{fig:thomas_model} are both two-dimensional. From Remark \ref{rmk:complex_balance_property}, for any $\bk \in \pK(G, G_1)$, the system $(G, \bk)$ has a globally attracting steady state within each stoichiometric compatibility class. \qed \end{example} \begin{example}[Circadian clock models \cite{leloup1999chaos}] \label{ex:circadian} The circadian clock forms an integral part of the behavioral, physical, and biological changes in the body over a 24-hour cycle. The dynamics of this network are known to exhibit exotic behaviors, such as oscillations and limit cycles. For our analysis, we utilize the model of the circadian clock introduced in \cite{leloup1999chaos}, described by the following reactions: \begin{equation} \notag P + T \rightleftharpoons C \rightarrow \emptyset, \ \ P \rightleftharpoons \emptyset, \ \ T \rightleftharpoons \emptyset, \end{equation} where $P$ denotes period, $T$ denotes time, and $C$ denotes the period-time complex. The E-graph corresponding to this model, shown in Figure~\ref{fig:circadian_clock} (a), is denoted by $G$. Since $G$ is not weakly reversible, \cite{CraciunDickensteinShiuSturmfels2009} demonstrates that the toric locus associated with $G$ is empty. \begin{figure}[!ht] \centering \includegraphics[scale=0.43]{circadian_clock.eps} \caption{ (a) The E-graph $G$ represents a circadian clock model. (b) The E-graph $G_1$ is a weakly reversible subgraph of the complete graph formed by the source vertices of $G$. } \label{fig:circadian_clock} \end{figure} Note that the graph $G$ contains 7 reactions, so $\pK(G) \subseteq \mathbb{R}^7_{>0}$. We claim that \[ \rm{dim}(\pK(G)) = 7. \] This implies that the disguised toric locus corresponding to $G$ is of positive measure. To prove this claim, we consider the E-graph $G_1$ and compute $\dim (\pK(G, G_1))$. First, consider a flux vector $\bJ \in \mJ(G_1, G) \subset \mathbb{R}^8$. Since $\bJ$ is a complex-balanced flux vector in $G_1$, this imposes $5 - 1 = 4$ constraints on $\bJ$, as shown in \cite{craciun2020efficient}. Additionally, being $\mathbb{R}$-realizable in $G$ imposes no further constraints, since every flux vector in $G_1$ can be transformed into $G$. Therefore, we have \[ \dim (\mJ(G_1, G)) = 8 - 0 - 4 = 4. \] Second, upon inspecting the graphs $G$ and $G_1$, we obtain the following: \[ \dim (\mathcal{S}_{G_1}) = 3, \ \ \rm{dim}(\mD (G)) = 0 \ \text{ and } \ \dim (\eJ(G_1)) = 0. \] Using Theorem~\ref{thm:dim_kisg} (a), we get that \begin{equation} \begin{split} \notag \rm{dim}(\dK(G, G_1)) & = \dim (\mJ(G_1, G)) + \dim (\mathcal{S}_{G_1}) + \dim(\mD (G)) - \dim(\eJ (G_1)) \\& = 4 + 3 + 0 - 0 = 7. \end{split} \end{equation} Since $G_1$ is weakly reversible, $\mK ({G_1}) \neq \emptyset$. From Figure~\ref{fig:circadian_clock}, given any $\bk_1 \in \mK ({G_1})$ there exists $\bk$ such that $(G,\bk)\sim (G_1, \bk_1)$. This implies that $\pK (G, G_1) \neq \emptyset$, and thus Theorem~\ref{thm:dim_kisg}~(b) further shows \[ \rm{dim}(\pK(G,G_1) = \rm{dim}(\dK(G,G_1)) = 7. \] This, together with Theorem \ref{thm:dim_kisg_main}, implies that $\rm{dim}(\pK(G)) = 7$. Actually, a computation (based on the matrix-tree theorem~\cite{CraciunDickensteinShiuSturmfels2009}) can be used to derive that $\pK(G) = \mathbb{R}^{7}_{>0}. $ Finally, note that for the circadian clock model $G$, the corresponding weakly reversible E-graph $G_1$ in Figure~\ref{fig:circadian_clock} contains a single linkage class. From Remark \ref{rmk:complex_balance_property} it follows that for any $\bk \in \pK(G, G_1)$, the system $(G, \bk)$ has a globally attracting steady state within each stoichiometric compatibility class. \qed \end{example} \section{Discussion} \label{sec:discussion} Due to their remarkably robust dynamics, complex-balanced dynamical systems form an important class of dynamical systems. In particular, they are notable for their connection to the \emph{Global Attractor Conjecture}~\cite{horn1972general, CraciunDickensteinShiuSturmfels2009}, which asserts that these systems possess a globally attracting steady state within each invariant polyhedron. The set of rate constants of a network that generate complex-balanced systems form a variety called the \emph{toric locus}~\cite{craciun2020structure}, and its codimension is determined by the deficiency of the reaction network~\cite{CraciunDickensteinShiuSturmfels2009}. A generalization of the toric locus concerns the set of rate constants that yield the same dynamics as complex-balanced systems. When the rate constants can take both positive and negative values, this set is referred to as the \emph{$\mathbb{R}$-disguised toric locus}; if only positive values are allowed, it is called the \emph{disguised toric locus} \cite{2022disguised}. In~\cite{disg_1}, it was demonstrated that both the disguised toric locus and the $\mathbb{R}$-disguised toric locus are path-connected. A recent study~\cite{disg_2} established a lower bound for the dimensions of both loci. The key contribution of this paper is to estimate the exact dimensions of both the disguised and $\mathbb{R}$-disguised toric loci through the construction of an explicit homomorphism. Specifically, Theorem~\ref{thm:dim_kisg_main} provides these dimensions as follows: {\small \begin{equation} \notag \begin{split} &\dim (\dK(G) ) = \max_{G'\sqsubseteq G_c} \Big\{ \dim (\mJ(G',G)) + \dim (\mS_{G'}) + \dim(\eJ(G')) - \dim(\mD(G)) \Big\}, \\& \dim (\pK(G) ) = \max_{ \substack{ G'\sqsubseteq G_c, \\ \pK(G, G') \neq \emptyset } } \Big\{ \dim (\mJ(G',G)) + \dim (\mS_{G'}) + \dim(\eJ(G')) - \dim(\mD(G)) \Big\}. \end{split} \end{equation} } This characterization allows us to identify regions of parameter space that produce robust dynamics, thus refining the modeling and prediction of system responses. The results outlined above generate several new directions for future research. One direction involves the efficient computation of the dimensions of both the $\mathbb{R}$-disguised toric locus and the disguised toric locus for a given network. Among the various quantities on the right-hand side of the equations, the most challenging to characterize is $\dim (\mJ(G', G))$, which we specifically aim to address in our upcoming work~\cite{disg_4}. Specifically, computing $\dim (\mJ(G', G))$ is equivalent to solving a linear feasibility problem that incorporates both dynamical equivalence and complex-balancing constraints. Another promising direction is to identify the conditions under which the homeomorphisms constructed in this paper can be shown to be diffeomorphisms. A recent work~\cite{smoothness} has shown that the toric locus of a weakly reversible network is a smooth manifold. Consequently, we aim to establish the conditions on the network that ensure the disguised toric locus is also a smooth manifold, with the corresponding steady states varying continuously along this locus. \section*{Acknowledgements} This work was supported in part by the National Science Foundation grant DMS-2051568. \bibliographystyle{unsrt} \bibliography{Bibliography} \end{document}
2412.02681v1
http://arxiv.org/abs/2412.02681v1
On Rank of Multivectors in Geometric Algebras
\documentclass[AMA,STIX1COL]{WileyNJD-v2} \usepackage{moreverb} \def\cl{{C}\!\ell} \def\R{{\mathbb R}} \def\F{{\mathbb F}} \def\CC{{\mathbb C}} \def\C{\mathcal {G}} \def\P{{\rm P}} \def\A{{\rm A}} \def\B{{\rm B}} \def\Q{{\rm Q}} \def\Z{{\rm Z}} \def\H{{\rm H}} \def\Aut{{\rm Aut}} \def\ker{{\rm ker}} \def\OO{{\rm O}} \def\SO{{\rm SO}} \def\Pin{{\rm Pin}} \def\Spin{{\rm Spin}} \def\ad{{\rm ad}} \def\mod{{\rm \;mod\; }} \newcommand{\BR}{\mathbb{R}} \newcommand{\BC}{\mathbb{C}} \newcommand{\Mat}{{\rm Mat}} \newcommand{\Det}{{\rm Det}} \newcommand{\tr}{{\rm tr}} \newcommand{\rank}{{\rm rank}} \newcommand{\spn}{{\rm span}} \newcommand{\diag}{{\rm diag}} \newcommand{\Adj}{{\rm Adj}} \def\cl{\mathcal {G}} \newcommand{\U}{{\rm U}} \newcommand{\G}{{\rm G}} \newcommand{\T}{{\rm T}} \newtheorem{example}{Example} \newcommand\BibTeX{{\rmfamily B\kern-.05em \textsc{i\kern-.025em b}\kern-.08em T\kern-.1667em\lower.7ex\hbox{E}\kern-.125emX}} \articletype{Research article} \received{<day> <Month>, <year>} \revised{<day> <Month>, <year>} \accepted{<day> <Month>, <year>} \begin{document} \title{On Rank of Multivectors in Geometric Algebras\protect\thanks{The article was prepared within the framework of the project “Mirror Laboratories” HSE University “Quaternions, geometric algebras and applications”.}} \author[1,2]{Dmitry Shirokov*} \authormark{DMITRY SHIROKOV} \address[1]{ \orgname{HSE University}, \orgaddress{\state{Moscow}, \country{Russia}}} \address[2]{ \orgname{Institute for Information Transmission Problems of Russian Academy of Sciences}, \orgaddress{\state{Moscow}, \country{Russia}}} \corres{Dmitry Shirokov. \email{[email protected]}} \presentaddress{HSE University, 101000, Moscow, Russia} \abstract[Abstract]{We introduce the notion of rank of multivector in Clifford geometric algebras of arbitrary dimension without using the corresponding matrix representations and using only geometric algebra operations. We use the concepts of characteristic polynomial in geometric algebras and the method of SVD. The results can be used in various applications of geometric algebras in computer science, engineering, and physics.} \keywords{characteristic polynomial; Clifford algebra; geometric algebra; rank; singular value decomposition; unitary group} \jnlcitation{\cname{\author{D. Shirokov}} (\cyear{2024}), \ctitle{On Rank of Multivectors in Geometric Algebras}} \maketitle \section{Introduction} The notion of rank of matrix is one of the most important concepts of the matrix theory, which is used in different applications -- data analysis, physics, engineering, control theory, computer sciences, etc. The Clifford geometric algebras can be regarded as unified language of mathematics \cite{ABS, Porteous, Helm}, physics \cite{Hestenes, Doran, BT, Snygg}, engineering \cite{Bayro2}, and computer science \cite{Dorst, Bayro1}. The Clifford geometric algebras are isomorphic to the classical matrix algebras. In particular, the complexified Clifford geometric algebras $\cl^\BC_{p,q}:=\BC\otimes \cl_{p,q}$ are isomorphic to the following complex matrix algebras: \begin{eqnarray} \cl^\BC_{p,q}\simeq \begin{cases} \Mat(2^{\frac{n}{2}}, \BC), &\mbox{if $n$ is even,}\\ \Mat(2^{\frac{n-1}{2}}, \BC)\oplus\Mat(2^{\frac{n-1}{2}}, \BC), &\mbox{if $n$ is odd.} \end{cases} \end{eqnarray} An arbitrary element $M\in\cl^\BC_{p,q}$ (a multivector) can be represented as a complex matrix of the corresponding size $$N:=2^{[\frac{n+1}{2}]},$$ where square brackets mean taking the integer part. In the case of odd $n$, we deal with block-diagonal matrices with two nonzero blocks of the same size $2^{\frac{n-1}{2}}$. In this regard, the problem arises of determining the rank of multivectors $M\in\cl^\BC_{p,q}$ without using the matrix representation and using only the operations in Clifford geometric algebras. In this paper, we solve this problem in the case of any dimension. To do this, we use our previous results on SVD and characteristic polynomial in Clifford geometric algebras. Theorems \ref{thrankpr}, \ref{thrankpr2}, \ref{thrank}, \ref{thrankherm} are new. New explicit formulas (\ref{exp1}), (\ref{exp2}) for the cases of dimensions $3$ and $4$ can be used in various applications of geometric algebras in physics, engineering, and computer science. The paper is organized as follows. In Section \ref{secGA}, we discuss real and complexified geometric algebras (GA) and introduce the necessary notation. In Section \ref{secbeta}, we discuss an operation of Hermitian conjugation in GA, introduce a positive scalar product, a norm, unitary space and unitary groups in GA. Also we discuss faithful representations of GA and present an explicit form on one of them. In Section \ref{secSVD}, we discuss singular value decomposition of multivectors in GA. In Section \ref{secDet}, we discuss a realization of the determinant and other characteristic polynomial coefficients in GA. In Section \ref{secRank}, we introduce a notion of rank of multivector in GA and prove a number of properties of this notion. We prove that this notion does not depend on the choosing of matrix representation and present another equivalent definition of this notion using only GA operations. Examples for cases of small dimensions are presented. In Section \ref{secRankherm}, we consider the special case of normal multivectors, for which rank can be determined more simply. The conclusions follow in Section \ref{secConcl}. \section{Real and Complexified Geometric Algebras}\label{secGA} Let us consider the real Clifford geometric algebra $\cl_{p,q}$ \cite{Hestenes,Lounesto,Doran,Bulg} with the identity element $e\equiv 1$ and the generators $e_a$, $a=1, 2, \ldots, n$, where $n=p+q\geq 1$. The generators satisfy the conditions $$ e_a e_b+e_b e_a=2\eta_{ab}e,\qquad \eta=(\eta_{ab})=\diag(\underbrace{1, \ldots , 1}_p, \underbrace{-1, \ldots, -1}_{q}) $$ Consider the subspaces $\cl^k_{p,q}$ of grades $k=0, 1, \ldots, n$, which elements are linear combinations of the basis elements $e_A=e_{a_1 a_2 \ldots a_k}=e_{a_1}e_{a_2}\cdots e_{a_k}$, $1 \leq a_1<a_2<\cdots< a_k \leq n$, with ordered multi-indices of length $k$. An arbitrary element (multivector) $M\in\cl_{p,q}$ has the form $$ M=\sum_A m_A e_A\in\cl_{p,q},\qquad m_A\in\BR, $$ where we have a sum over arbitrary multi-index $A$ of length from $0$ to $n$. The projection of $M$ onto the subspace $\cl^k_{p,q}$ is denoted by $\langle M \rangle_k$. The grade involution and reversion of a multivector $M\in\cl_{p,q}$ are denoted by \begin{eqnarray} \widehat{M}=\sum_{k=0}^n(-1)^{k}\langle M \rangle_k,\qquad \widetilde{M}=\sum_{k=0}^n (-1)^{\frac{k(k-1)}{2}} \langle M \rangle_k. \end{eqnarray} We have \begin{eqnarray} \widehat{M_1 M_2}=\widehat{M_1} \widehat{M_2},\qquad \widetilde{M_1 M_2}=\widetilde{M_2} \widetilde{M_1},\qquad \forall M_1, M_2\in\cl_{p,q}.\label{invol} \end{eqnarray} Let us consider the complexified Clifford geometric algebra $\cl_{p,q}^\BC:=\BC\otimes\cl_{p,q}$ \cite{Bulg}. An arbitrary element of $M\in\cl^\BC_{p,q}$ has the form $$ M=\sum_A m_A e_A\in\cl^\BC_{p,q},\qquad m_A\in\BC. $$ Note that $\cl^\BC_{p,q}$ has the following basis of $2^{n+1}$ elements: \begin{eqnarray} e, ie, e_1, ie_1, e_2, i e_2, \ldots, e_{1\ldots n}, i e_{1\ldots n}.\label{basisC} \end{eqnarray} In addition to the grade involution and reversion, we use the operation of complex conjugation, which takes complex conjugation only from the coordinates $m_A$ and does not change the basis elements $e_A$: $$ \overline{M}=\sum_A \overline{m}_A e_A\in\cl^\BC_{p,q},\qquad m_A\in\BC,\qquad M\in\cl^\BC_{p,q}. $$ We have $$ \overline{M_1 M_2}=\overline{M_1}\,\, \overline{M_2},\qquad \forall M_1, M_2\in\cl^\BC_{p,q}. $$ \section{Hermitian conjugation and unitary groups in Geometric Algebras}\label{secbeta} Let us consider an operation of Hermitian conjugation $\dagger$ in $\cl^\BC_{p,q}$ (see \cite{unitary,Bulg}): \begin{eqnarray} M^\dagger:=M|_{e_A \to (e_A)^{-1},\,\, m_A \to \overline{m}_A}=\sum_A \overline{m}_A (e_A)^{-1}.\label{herm} \end{eqnarray} We have the following two equivalent definitions of this operation: \begin{eqnarray} &&M^\dagger=\begin{cases} e_{1\ldots p} \overline{\widetilde{M}}e_{1\ldots p}^{-1}, & \mbox{if $p$ is odd,}\\ e_{1\ldots p} \overline{\widetilde{\widehat{M}}}e_{1\ldots p}^{-1}, & \mbox{if $p$ is even,}\\ \end{cases}\\ &&M^\dagger= \begin{cases} e_{p+1\ldots n} \overline{\widetilde{M}}e_{p+1\ldots n}^{-1}, & \mbox{if $q$ is even,}\\ e_{p+1\ldots n} \overline{\widetilde{\widehat{M}}}e_{p+1\ldots n}^{-1}, & \mbox{if $q$ is odd.}\\ \end{cases} \end{eqnarray} The operation\footnote{Compare with the well-known operation $M_1 * M_2:=\langle \widetilde{M_1} M_2 \rangle_0$ in the real geometric algebra $\cl_{p,q}$, which is positive definite only in the case of signature $(p,q)=(n,0)$.} $$(M_1, M_2):=\langle M_1^\dagger M_2 \rangle_0$$ is a (positive definite) scalar product with the properties \begin{eqnarray} &&(M_1, M_2)=\overline{(M_2, M_1)},\\ &&(M_1+M_2, M_3)=(M_1, M_3)+(M_2, M_3),\quad (M_1, \lambda M_2)=\lambda (M_1, M_2),\\ &&(M, M)\geq 0,\quad (M, M)=0 \Leftrightarrow M=0.\label{||M||} \end{eqnarray} Using this scalar product we introduce inner product space over the field of complex numbers (unitary space) in $\cl^\BC_{p,q}$. We have a norm \begin{eqnarray} ||M||:=\sqrt{(M,M)}=\sqrt{\langle M^\dagger M \rangle_0}.\label{norm} \end{eqnarray} Let us consider the following faithful representation (isomorphism) of the complexified geometric algebra \begin{eqnarray} \beta:\cl^\BC_{p,q}\quad \to\quad \begin{cases} \Mat(2^{\frac{n}{2}}, \BC), &\mbox{if $n$ is even,}\\ \Mat(2^{\frac{n-1}{2}}, \BC)\oplus\Mat(2^{\frac{n-1}{2}}, \BC), &\mbox{if $n$ is odd.} \end{cases}\label{isom} \end{eqnarray} Let us denote the size of the corresponding matrices by $$N:=2^{[\frac{n+1}{2}]},$$ where square brackets mean taking the integer part. Let us present an explicit form of one of these representations of $\cl^\BC_{p,q}$ (we use it also for $\cl_{p,q}$ in \cite{det} and for $\cl^\BC_{p,q}$ in \cite{LMA}). We denote this fixed representation by $\beta'$. Let us consider the case $p = n$, $q = 0$. To obtain the matrix representation for another signature with $q\neq 0$, we should multiply matrices $\beta'(e_a)$, $a = p + 1, \ldots, n$ by imaginary unit $i$. For the identity element, we always use the identity matrix $\beta'(e)=I_N$ of the corresponding dimension $N$. We always take $\beta'(e_{a_1 a_2 \ldots a_k}) = \beta' (e_{a_1}) \beta' (e_{a_2}) \cdots \beta'(e_{a_k})$. In the case $n=1$, we take $\beta'(e_1)=\diag(1, -1)$. Suppose we know $\beta'_a:=\beta'(e_a)$, $a = 1, \ldots, n$ for some fixed odd $n = 2k + 1$. Then for $n = 2k + 2$, we take the same $\beta'(e_a)$, $a = 1, \ldots , 2k + 1$, and $$\beta'(e_{2k+2})=\left( \begin{array}{cc} 0 & I_{\frac{N}{2}} \\ I_{\frac{N}{2}} & 0 \end{array} \right).$$ For $n = 2k + 3$, we take $$\beta'(e_{a})= \left(\begin{array}{cc} \beta'_a & 0 \\ 0 & -\beta'_a \end{array} \right),\qquad a=1, \ldots, 2k+2,$$ and $$\beta'(e_{2k+3})=\left(\begin{array}{cc} i^{k+1}\beta'_1\cdots \beta'_{2k+2} & 0 \\ 0 & -i^{k+1}\beta'_1\cdots \beta'_{2k+2} \end{array} \right).$$ This recursive method gives us an explicit form of the matrix representation $\beta'$ for all $n$. Note that for this matrix representation we have $$ (\beta'(e_a))^\dagger=\eta_{aa} \beta'(e_a),\qquad a=1, \ldots, n, $$ where $\dagger$ is the Hermitian transpose of a matrix. Using the linearity, we get that Hermitian conjugation of matrix is consistent with Hermitian conjugation of corresponding multivector: \begin{eqnarray} \beta'(M^\dagger)=(\beta'(M))^\dagger,\qquad M\in\cl^\BC_{p,q}.\label{sogl} \end{eqnarray} Note that the same is not true for an arbitrary matrix representations $\beta$ of the form (\ref{isom}). It is true the matrix representations $\gamma=T^{-1}\beta' T$ obtained from $\beta'$ using the matrix $T$ such that $T^\dagger T= I$. Let us consider the group \begin{eqnarray} \U\cl^\BC_{p,q}=\{M\in \cl^\BC_{p,q}: M^\dagger M=e\}, \end{eqnarray} which we call a unitary group in $\cl^\BC_{p,q}$. Note that all the basis elements $e_A$ of $\cl_{p,q}$ belong to this group by the definition. Using (\ref{isom}) and (\ref{sogl}), we get the following isomorphisms to the classical matrix unitary groups: \begin{eqnarray} \U\cl^\BC_{p,q}\simeq\begin{cases} \U(2^{\frac{n}{2}}), &\mbox{if $n$ is even,}\\ \U(2^{\frac{n-1}{2}})\times\U(2^{\frac{n-1}{2}}), &\mbox{if $n$ is odd,} \end{cases}\label{isgr} \end{eqnarray} where \begin{eqnarray} \U(k)=\{A\in\Mat(k, \BC),\quad A^\dagger A=I\}. \end{eqnarray} \section{Singular Value Decomposition in Geometric Algebras}\label{secSVD} The method of singular value decomposition was discovered independently by E. Beltrami in 1873 \cite{Beltrami} and C. Jordan in 1874 \cite{Jordan1,Jordan2}. We have the following well-known theorem on singular value decomposition of an arbitrary complex matrix \cite{For,Van}. For an arbitrary $A\in\BC^{n\times m}$, there exist matrices $U\in \U(n)$ and $V\in\U(m)$ such that \begin{eqnarray} A=U\Sigma V^\dagger,\label{SVD} \end{eqnarray} where $$ \Sigma=\diag(\lambda_1, \lambda_2, \ldots, \lambda_k),\qquad k=\min(n, m),\qquad \BR\ni\lambda_1, \lambda_2, \ldots, \lambda_k\geq 0. $$ Note that choosing matrices $U\in \U(n)$ and $V\in\U(m)$, we can always arrange diagonal elements of the matrix $\Sigma$ in decreasing order $\lambda_1\geq \lambda_2 \geq \cdots \geq \lambda_k\geq 0$. Diagonal elements of the matrix $\Sigma$ are called singular values, they are square roots of eigenvalues of the matrices $A A^\dagger$ or $A^\dagger A$. Columns of the matrices $U$ and $V$ are eigenvectors of the matrices $A A^\dagger$ and $A^\dagger A$ respectively. \begin{theorem}[SVD in GA]\cite{SVDAACA}\label{th1} For an arbitrary multivector $M\in\cl^\BC_{p,q}$, there exist multivectors $U, V\in \U\cl^\BC_{p,q}$, where $$ \U\cl^\BC_{p,q}=\{U\in \cl^\BC_{p,q}: U^\dagger U=e\},\qquad U^\dagger:=\sum_A \overline{u}_A (e_A)^{-1}, $$ such that \begin{eqnarray} M=U\Sigma V^\dagger,\label{SVDMC} \end{eqnarray} where multivector $\Sigma$ belongs to the subspace $K\in\cl^\BC_{p,q}$, which is a real span of a set of $N=2^{[\frac{n+1}{2}]}$ fixed basis elements (\ref{basisC}) of $\cl^\BC_{p,q}$ including the identity element~$e$. \end{theorem} \section{Determinant and other characteristic polynomial coefficients in Geometric Algebras}\label{secDet} Let us consider the concept of determinant \cite{rudn,acus} and characteristic polynomial \cite{det} in geometric algebra. Explicit formulas for characteristic polynomial coefficients are discussed in \cite{Abd,Abd2}, applications to Sylvester equation are discussed in \cite{Sylv,Sylv2}, the relation with noncommutative Vieta theorem is discussed in \cite{Vieta1,Vieta2}, applications to calculation of elementary functions in geometric algebras are discussed in \cite{Acus}. We can introduce the notion of determinant $$\Det(M):=\det(\beta(M))\in\BR,\qquad M\in\cl^\BC_{p,q},$$ where $\beta$ is (\ref{isom}), and the notion of characteristic polynomial \begin{eqnarray} &&\varphi_M(\lambda):=\Det(\lambda e-M)=\lambda^N-C_{(1)}\lambda^{N-1}-\cdots-C_{(N-1)}\lambda-C_{(N)}\in\cl^0_{p,q}\equiv\BR,\nonumber\\ &&M\in\cl^\BC_{p,q},\quad N=2^{[\frac{n+1}{2}]},\quad C_{(k)}=C_{(k)}(M)\in\cl^0_{p,q}\equiv\BR,\quad k=1, \ldots, N.\label{char} \end{eqnarray} The following method based on the Faddeev--LeVerrier algorithm allows us to recursively obtain basis-free formulas for all the characteristic coefficients $C_{(k)}$, $k=1, \ldots, N$ (\ref{char}): \begin{eqnarray} &&M_{(1)}:=M,\qquad M_{(k+1)}=M(M_{(k)}-C_{(k)}),\label{FL0}\\ &&C_{(k)}:=\frac{N}{k}\langle M_{(k)} \rangle_0,\qquad k=1, \ldots, N. \label{FL}\end{eqnarray} In particular, we have \begin{eqnarray} C_{(1)}=N \langle M \rangle_0=\tr(\beta(M)). \end{eqnarray} In this method, we obtain high coefficients from the lowest ones. The determinant is minus the last coefficient \begin{eqnarray} \Det(M)=-C_{(N)}=-M_{(N)}=U(C_{(N-1)}-M_{(N-1)})\label{laststep} \end{eqnarray} and has the properties (see \cite{rudn,det}) \begin{eqnarray} &&\Det(M_1 M_2)=\Det(M_1) \Det (M_2),\qquad M_1, M_2\in\cl^\BC_{p,q},\label{detpr}\\ &&\Det(M)=\Det(\widehat{M})=\Det(\widetilde{M})=\Det(\overline{M})=\Det(M^\dagger),\qquad \forall M\in\cl^\BC_{p,q}.\label{detpr2} \end{eqnarray} The inverse of a multivector $M\in\cl^\BC_{p,q}$ can be computed as \begin{eqnarray} M^{-1}=\frac{\Adj(M)}{\Det(M)}=\frac{C_{(N-1)}-M_{(N-1)}}{\Det(M)},\qquad \Det(M)\neq 0.\label{inv} \end{eqnarray} The presented algorithm and formulas (\ref{FL0}), (\ref{FL}), (\ref{inv}) are actively used to calculate inverse in GA \cite{inv1,inv2,inv3}. \section{Rank in Geometric Algebras}\label{secRank} Let us introduce the notion of rank of a multivector $M\in\cl^\BC_{p,q}$: \begin{eqnarray} \rank(M):=\rank(\beta(M))\in\{0, 1, \ldots, N\},\label{rank} \end{eqnarray} where $\beta$ is (\ref{isom}). Below we present another equivalent definition, which does not depend on the matrix representation $\beta$ (Theorem \ref{thrank}). We use the fact that rank is the number of nonzero singular values in the SVD and Vieta formulas. \begin{lemma}\label{lemmawell} The rank of multivector $\rank(M)$ (\ref{rank}) is well-defined, i.e. it does not depend on the representation $\beta$ (\ref{isom}). \end{lemma} \begin{proof} In the case of even $n$, for an arbitrary representation $\beta$ of type (\ref{isom}), by the Pauli theorem \cite{Pauli}, there exists $T$ such that $\beta(e_a)=T^{-1}\beta'(e_a) T$, where $\beta'$ is fixed matrix representation from Section \ref{secbeta}. We get $\beta(M)=T^{-1}\beta'(M) T$ and $\rank(\beta(M))=\rank(\beta'(M))$. In the case of odd $n$, for an arbitrary representation $\beta$ of type (\ref{isom}), by the Pauli theorem \cite{Pauli}, there exists $T$ such that $\beta(e_a)=T^{-1}\beta'(e_a) T$ or $\beta(e_a)=-T^{-1}\beta'(e_a) T$. In the first case, we get $\rank(\beta(M))=\rank(\beta'(M))$ similarly to the case of even $n$. In the second case, we get $\beta(M)=T^{-1}\beta'(\widehat{M}) T$ and $\rank(\beta(M))=\rank(\beta'(\widehat{M}))$. The equality $\rank(\beta'(\widehat{M}))=\rank(\beta'(M))$ is verified using the explicit form of representation $\beta'$ from Section \ref{secbeta}. Namely, the matrices $\beta'(e_a)=\diag(\beta'_a, -\beta'_a)$, $a=1, \ldots, n$, are block-diagonal matrices with two blocks differing in sign on the main diagonal by construction. Thus the matrix $\beta'(e_{ab})=\beta'(e_a)\beta'(e_b)=\diag(\beta'_a \beta'_b, \beta'_a \beta'_b)$ has two identical blocks. We conclude that the even part of multivector $M$ has the matrix representation $\diag(A, A)$ with two identical blocks, and the odd part of multivector $M$ has the matrix representation $\diag(B, -B)$ with two blocks differing in sign. Finally, we obtain $\rank(\beta'(\widehat{M})=\rank(\diag(A-B, A+B))=\rank(\diag(A+B, A-B))=\rank(\beta'(M))$. \end{proof} \begin{theorem}\label{thrankpr} We have the following properties of the rank of arbitrary multivectors $M_1, M_2, M_3\in\cl^\BC_{p,q}$: \begin{eqnarray} &&\rank(M_1 U)=\rank(U M_1)=\rank (M_1),\qquad \forall \,\,\mbox{invertible}\,\,U\in\cl^\BC_{p,q},\\ &&\rank(M_1 M_2)\leq \min(\rank(M_1), \rank(M_2)),\\ &&\rank(M_1 M_2)+\rank(M_2 M_3)\leq \rank(M_1 M_2 M_3)+\rank(M_2),\\ &&\rank(M_1 )+\rank(M_3)\leq \rank(M_1 M_3)+N. \end{eqnarray} \end{theorem} \begin{proof} These properties are the corollary of the corresponding properties of rank of matrices. \end{proof} \begin{theorem}\label{thrankpr2} We have \begin{eqnarray} &&\rank(M)=\rank(\widehat{M})=\rank(\widetilde{M})=\rank(\overline{M})\\ &&\qquad=\rank(M^\dagger)=\rank(M^\dagger M)=\rank(M M^\dagger),\qquad \forall M\in\cl^\BC_{p,q}. \end{eqnarray} \end{theorem} \begin{proof} Let us prove $\rank(M)=\rank(\widehat{M})$. In the case of even $n$, we have $\rank(\widehat{M})=\rank(e_{1\ldots n}M e_{1\ldots n}^{-1})=\rank (M)$. In the case of odd $n$, we have already proved the statement in the proof of Lemma \ref{lemmawell}. Let us prove $\rank(M)=\rank(\widetilde{M})$. We have the following relation between the reversion (or the superposition of reversion and grade involution) and the transpose (see \cite{nspinors,LMA}): \begin{eqnarray} (\beta'(M))^\T=\begin{cases} \beta'(e_{b_1 \ldots b_k}\widetilde{M}e_{b_1\ldots b_k}^{-1}), & \mbox{if $k$ is odd,}\\ \beta'(e_{b_1 \ldots b_k}\widehat{\widetilde{M}}e_{b_1\ldots b_k}^{-1}), & \mbox{if $k$ is even,} \end{cases} \end{eqnarray} for some fixed basis element $e_{b_1\ldots b_k}$, where $k$ is the number of symmetric matrices among $\beta'(e_a)$, $a=1, \ldots, n$. We get $\rank(M)=\rank(\beta'(M))=\rank((\beta'(M))^\T)=\rank(\widetilde{M})$. Using (\ref{sogl}), we obtain the other formulas for the Hermitian conjugation and complex conjugation, which is a superposition of Hermitian conjugation and transpose. \end{proof} \begin{lemma}\label{lemmaB} Suppose that a square matrix $A\in\BC^{N\times N}$ is diagonalizable. Then \begin{eqnarray} &&\rank(A)=N \quad \Leftrightarrow \quad C_{(N)}\neq 0;\\ && \rank(A)=k\in\{1, \ldots, N-1\} \, \Leftrightarrow \, C_{(k)}\neq 0,\,\, C_{(j)}=0,\, j=k+1, \ldots, N;\\ &&\rank(A)=0 \quad \Leftrightarrow \quad A=0. \end{eqnarray} \end{lemma} \begin{proof} We use Vieta formulas for the eigenvalues $\lambda_1, \lambda_2, \ldots, \lambda_N$: \begin{eqnarray} C_{(1)}&=&\lambda_1+\cdots+\lambda_N,\\ C_{(2)}&=&-(\lambda_1 \lambda_2+\lambda_1 \lambda_3+\cdots+\lambda_{N-1}\lambda_N),\\ && \cdots\\ C_{(N)}&=&-\lambda_1 \cdots \lambda_N. \end{eqnarray} To the right, all statements are obvious. To the left, they are proved by contradiction. \end{proof} \begin{lemma}\label{lemmaC} For an arbitrary multivector $M\in\cl^\BC_{p,q}$, we have \begin{eqnarray} C_{(N)}(M^\dagger M)=0 &\Longleftrightarrow& C_{(N)}(M)=0,\\ C_{(1)}(M^\dagger M)=0 &\Longleftrightarrow& M=0. \end{eqnarray} \end{lemma} \begin{proof} We have \begin{eqnarray*} C_{(N)}(M^\dagger M)&=&-\Det(M^\dagger M)=-\Det(M^\dagger) \Det(M)\\ &=&-(\Det M)^2=(C_{(N)}(M))^2,\\ C_{(1)}(M^\dagger M)&=&N \langle M^\dagger M \rangle_0=N ||M||^2, \end{eqnarray*} where we use (\ref{detpr}), (\ref{detpr2}), (\ref{norm}), and (\ref{||M||}). \end{proof} \begin{theorem}[Rank in GA]\label{thrank} Let us consider an arbitrary multivector $M\in\cl^\BC_{p,q}$ and $T:=M^\dagger M$. We have \begin{eqnarray} \rank(M)=\begin{cases} N,\quad &\mbox{if $C_{(N)}(M)\neq 0$,}\\ N-1,\quad &\mbox{if $C_{(N)}(M)=0$ and $C_{(N-1)}(T)\neq 0$,}\\ N-2\qquad &\mbox{if $C_{(N)}(M)=C_{(N-1)}(T)=0$ and}\\ &\mbox{$C_{(N-2)}(T)\neq 0$,}\\ \cdots &\\ 2,\quad &\mbox{if $C_{(N)}(M)=C_{(N-1)}(T)=\cdots=C_{(3)}(T)=0$ and}\\ &\mbox{$C_{(2)}(T)\neq 0$,}\\ 1,\quad &\mbox{if $C_{(N)}(M)=C_{(N-1)}(T)=\cdots=C_{(2)}(T)=0$ and}\\ &\mbox{$M\neq 0$,}\\ 0,\quad &\mbox{if $M=0$.}\label{rank22} \end{cases} \end{eqnarray} \end{theorem} \begin{proof} We use the fact that the rank of a matrix equals the number of non-zero singular values, which is the same as the number of non-zero diagonal elements of the matrix $\Sigma$ in the singular value decomposition $A=U\Sigma V^\dagger$ (\ref{SVD}): $\rank(A)=\rank(U\Sigma V^\dagger)=\rank(\Sigma)$. The number of non-zero diagonal elements of the matrix $\Sigma$ can be written in terms of zero and non-zero characteristic polynomial coefficients of the matrix $A^\dagger A$ (see Lemma \ref{lemmaB}). Then we use Lemma~\ref{lemmaC}. \end{proof} \begin{example} For an arbitrary $M\in\cl^\BC_{p,q}$, $n=p+q=1$, we have \begin{eqnarray} \rank(M)=\begin{cases} $2,\quad$ &\mbox{if $M\widehat{M}\neq 0$,}\\ $1,\quad$ &\mbox{if $M\widehat{M}=0$ and $M\neq 0$,}\\ $0,\quad$ &\mbox{if $M=0$.} \end{cases}\label{exp-1} \end{eqnarray} \end{example} \begin{example} For an arbitrary $M\in\cl^\BC_{p,q}$, $n=p+q=2$, we have \begin{eqnarray} \rank(M)=\begin{cases} $2,\quad$ &\mbox{if $M\widetilde{\widehat{M}}\neq 0$,}\\ $1,\quad$ &\mbox{if $M\widetilde{\widehat{M}}=0$ and $M\neq 0$,}\\ $0,\quad$ &\mbox{if $M=0$.} \end{cases}\label{exp0} \end{eqnarray} \end{example} \begin{example} For an arbitrary $M\in\cl^\BC_{p,q}$, $n=p+q=3$, we have \begin{eqnarray} \rank(M)=\begin{cases} $4,\quad$ &\mbox{if $M\widetilde{\widehat{M}}\widehat{M}\widetilde{M}\neq 0$,}\\ $3,\quad$ &\mbox{if $M\widetilde{\widehat{M}}\widehat{M}\widetilde{M}=0$ and $T\widetilde{\widehat{T}}\widehat{T}+T\widetilde{\widehat{T}}\widetilde{T}+T\widehat{T}\widetilde{T}+\widetilde{\widehat{T}}\widehat{T}\widetilde{T}\neq 0$,}\\ $2,\quad$ &\mbox{if $M\widetilde{\widehat{M}}\widehat{M}\widetilde{M}=T\widetilde{\widehat{T}}\widehat{T}+T\widetilde{\widehat{T}}\widetilde{T}+T\widehat{T}\widetilde{T}+\widetilde{\widehat{T}}\widehat{T}\widetilde{T}=0$ and} \\ &\mbox{$T\widetilde{\widehat{T}}+T\widehat{T}+T\widetilde{T}+\widetilde{\widehat{T}}\widehat{T}+\widetilde{\widehat{T}}\widetilde{T}+\widehat{T}\widetilde{T}\neq 0$,}\\ $1,\quad$ &\mbox{if $M\widetilde{\widehat{M}}\widehat{M}\widetilde{M}=T\widetilde{\widehat{T}}\widehat{T}+T\widetilde{\widehat{T}}\widetilde{T}+T\widehat{T}\widetilde{T}+\widetilde{\widehat{T}}\widehat{T}\widetilde{T}=$}\\ &\mbox{$=T\widetilde{\widehat{T}}+T\widehat{T}+T\widetilde{T}+\widetilde{\widehat{T}}\widehat{T}+\widetilde{\widehat{T}}\widetilde{T}+\widehat{T}\widetilde{T}=0$ and $M\neq 0$,}\\ $0,\quad$ &\mbox{if $M=0$,} \end{cases}\label{exp1} \end{eqnarray} where $T:=M^\dagger M$. \end{example} \begin{example} Let us consider the $\bigtriangleup$-operation \cite{det} \begin{eqnarray} M^{\bigtriangleup}:=\sum_{k=0}^n (-1)^{\frac{k(k-1)(k-2)(k-3)}{24}}\langle M \rangle_k=\!\!\!\!\!\sum_{k=0, 1, 2, 3\!\!\!\!\mod 8}\!\!\!\!\!\langle M \rangle_k-\!\!\!\!\!\sum_{k=4, 5, 6, 7\!\!\!\!\mod 8}\!\!\!\!\!\langle M \rangle_k.\label{opconj} \end{eqnarray} Note that we have $(M_1 M_2)^\bigtriangleup\neq M_1^\bigtriangleup M_2^\bigtriangleup$ and $(M_1 M_2)^\bigtriangleup\neq M_2^\bigtriangleup M_1^\bigtriangleup $ in the general case. For an arbitrary $M\in\cl^\BC_{p,q}$, $n=p+q=4$, we have \begin{eqnarray} \rank(M)=\begin{cases} $4,$ &\mbox{if $M\widetilde{\widehat{M}}(\widehat{M}\widetilde{M})^\bigtriangleup\neq 0$,}\\ $3,$ &\mbox{if $M\widetilde{\widehat{M}}(\widehat{M}\widetilde{M})^\bigtriangleup=0$ and} \\ &\mbox{$T\widetilde{\widehat{T}}\widehat{T}+T\widetilde{\widehat{T}}\widetilde{T}+T(\widehat{T}\widetilde{T})^\bigtriangleup+\widetilde{\widehat{T}}(\widehat{T}\widetilde{T})^\bigtriangleup\neq 0$,}\\ $2,$ &\mbox{if $M\widetilde{\widehat{M}}(\widehat{M}\widetilde{M})^\bigtriangleup=T\widetilde{\widehat{T}}\widehat{T}+T\widetilde{\widehat{T}}\widetilde{T}+T(\widehat{T}\widetilde{T})^\bigtriangleup+\widetilde{\widehat{T}}(\widehat{T}\widetilde{T})^\bigtriangleup=0$} \\ &\mbox{and $T\widetilde{\widehat{T}}+T\widehat{T}+T\widetilde{T}+\widetilde{\widehat{T}}\widehat{T}+\widetilde{\widehat{T}}\widetilde{T}+(\widehat{T}\widetilde{T})^\bigtriangleup\neq 0$,}\\ $1,$ &\mbox{if $M\widetilde{\widehat{M}}(\widehat{M}\widetilde{M})^\bigtriangleup=T\widetilde{\widehat{T}}\widehat{T}+T\widetilde{\widehat{T}}\widetilde{T}+T(\widehat{T}\widetilde{T})^\bigtriangleup+\widetilde{\widehat{T}}(\widehat{T}\widetilde{T})^\bigtriangleup=$}\\ &\mbox{$=T\widetilde{\widehat{T}}+T\widehat{T}+T\widetilde{T}+\widetilde{\widehat{T}}\widehat{T}+\widetilde{\widehat{T}}\widetilde{T}+(\widehat{T}\widetilde{T})^\bigtriangleup=0$ and $M\neq 0$,}\\ $0,$ &\mbox{if $M=0$,} \end{cases}\label{exp2} \end{eqnarray} where $T:=M^\dagger M$. \end{example} You can get an explicit form of formulas from Theorem \ref{thrank} for the cases $n=5$ and $n=6$ using the explicit formulas for the characteristic polynomial coefficients $C_{(k)}$, $k=1, 2, \dots, N$, from \cite{Abd2}. We do not present them here because they are quite cumbersome. These formulas involve only the operations of summation, multiplication, $\widehat{\quad}$, $\widetilde{\quad}$, and $\,\,^\bigtriangleup$. \section{The Case of Normal Multivectors}\label{secRankherm} We call \textit{a normal multivector} $M\in\cl^\BC_{p,q}$ a multivector with the property $M^\dagger M=M M^\dagger$, where $\dagger$ is (\ref{herm}). \textit{Hermitian multivectors} $M^\dagger=M$, \textit{anti-Hermitian multivectors} $M^\dagger=-M$, \textit{unitary multivectors }$M^\dagger M=e$ are the particular cases of normal multivectors. For example, the basis elements $e_A$ are unitary by the definition. Note that all unitary multivectors have rank equal to $N$. \begin{theorem}\label{thrankherm} Let us consider a normal ($M^\dagger M=M M^\dagger$) multivector $M\in\cl^\BC_{p,q}$. We have \begin{eqnarray} \rank(M)=\begin{cases} N,\quad &\mbox{if $C_{(N)}(M)\neq 0$,}\\ N-1,\quad &\mbox{if $C_{(N)}(M)=0$ and $C_{(N-1)}(M)\neq 0$,}\\ N-2\qquad &\mbox{if $C_{(N)}(M)=C_{(N-1)}(M)=0$ and}\\ &\mbox{$C_{(N-2)}(M)\neq 0$,}\\ \cdots &\\ 2,\quad &\mbox{if $C_{(N)}(M)=C_{(N-1)}(M)=\cdots=C_{(3)}(M)=0$ and}\\ &\mbox{$C_{(2)}(M)\neq 0$,}\\ 1,\quad &\mbox{if $C_{(N)}(M)=C_{(N-1)}(M)=\cdots=C_{(2)}(M)=0$ and}\\ &\mbox{$M\neq 0$,}\\ 0,\quad &\mbox{if $M=0$.}\label{rankherm} \end{cases} \end{eqnarray} \end{theorem} \begin{proof} We use that normal matrix is always diagonalizable. \end{proof} \begin{example} For an arbitrary normal ($M^\dagger M=M M^\dagger$) multivector $M\in\cl^\BC_{p,q}$, $n=p+q=3$, we have \begin{eqnarray} \rank(M)=\begin{cases} $4,\quad$ &\mbox{if $M\widetilde{\widehat{M}}\widehat{M}\widetilde{M}\neq 0$,}\\ $3,\quad$ &\mbox{if $M\widetilde{\widehat{M}}\widehat{M}\widetilde{M}=0$ and $M\widetilde{\widehat{M}}\widehat{M}+M\widetilde{\widehat{M}}\widetilde{M}+M\widehat{M}\widetilde{M}+\widetilde{\widehat{M}}\widehat{M}\widetilde{M}\neq 0$,}\\ $2,\quad$ &\mbox{if $M\widetilde{\widehat{M}}\widehat{M}\widetilde{M}=M\widetilde{\widehat{M}}\widehat{M}+M\widetilde{\widehat{M}}\widetilde{M}+M\widehat{M}\widetilde{M}+\widetilde{\widehat{M}}\widehat{M}\widetilde{M}=0$ and} \\ &\mbox{$M\widetilde{\widehat{M}}+M\widehat{M}+M\widetilde{M}+\widetilde{\widehat{M}}\widehat{M}+\widetilde{\widehat{M}}\widetilde{M}+\widehat{M}\widetilde{M}\neq 0$,}\\ $1,\quad$ &\mbox{if $M\widetilde{\widehat{M}}\widehat{M}\widetilde{M}=M\widetilde{\widehat{M}}\widehat{M}+M\widetilde{\widehat{M}}\widetilde{M}+M\widehat{M}\widetilde{M}+\widetilde{\widehat{M}}\widehat{M}\widetilde{M}=$}\\ &\mbox{$=M\widetilde{\widehat{M}}+M\widehat{M}+M\widetilde{M}+\widetilde{\widehat{M}}\widehat{M}+\widetilde{\widehat{M}}\widetilde{M}+\widehat{M}\widetilde{M}=0$ and $M\neq 0$,}\\ $0,\quad$ &\mbox{if $M=0$.} \end{cases}\label{exp12} \end{eqnarray} \end{example} \begin{example} For an arbitrary normal ($M^\dagger M=M M^\dagger$) multivector $M\in\cl^\BC_{p,q}$, $n=p+q=4$, we have \begin{eqnarray} \rank(M)=\begin{cases} $4,$ &\mbox{if $M\widetilde{\widehat{M}}(\widehat{M}\widetilde{M})^\bigtriangleup\neq 0$,}\\ $3,$ &\mbox{if $M\widetilde{\widehat{M}}(\widehat{M}\widetilde{M})^\bigtriangleup=0$ and} \\ &\mbox{$M\widetilde{\widehat{M}}\widehat{M}+M\widetilde{\widehat{M}}\widetilde{M}+M(\widehat{M}\widetilde{M})^\bigtriangleup+\widetilde{\widehat{M}}(\widehat{M}\widetilde{M})^\bigtriangleup\neq 0$,}\\ $2,$ &\mbox{if $M\widetilde{\widehat{M}}(\widehat{M}\widetilde{M})^\bigtriangleup=M\widetilde{\widehat{M}}\widehat{M}+M\widetilde{\widehat{M}}\widetilde{M}+M(\widehat{M}\widetilde{M})^\bigtriangleup+\widetilde{\widehat{M}}(\widehat{M}\widetilde{M})^\bigtriangleup=0$} \\ &\mbox{and $M\widetilde{\widehat{M}}+M\widehat{M}+M\widetilde{M}+\widetilde{\widehat{M}}\widehat{M}+\widetilde{\widehat{M}}\widetilde{M}+(\widehat{M}\widetilde{M})^\bigtriangleup\neq 0$,}\\ $1,$ &\mbox{if $M\widetilde{\widehat{M}}(\widehat{M}\widetilde{M})^\bigtriangleup=M\widetilde{\widehat{M}}\widehat{M}+M\widetilde{\widehat{M}}\widetilde{M}+M(\widehat{M}\widetilde{M})^\bigtriangleup+\widetilde{\widehat{M}}(\widehat{M}\widetilde{M})^\bigtriangleup=$}\\ &\mbox{$=M\widetilde{\widehat{M}}+M\widehat{M}+M\widetilde{M}+\widetilde{\widehat{M}}\widehat{M}+\widetilde{\widehat{M}}\widetilde{M}+(\widehat{M}\widetilde{M})^\bigtriangleup=0$ and $M\neq 0$,}\\ $0,$ &\mbox{if $M=0$.} \end{cases}\label{exp22} \end{eqnarray} \end{example} Note that formulas (\ref{rankherm}), (\ref{exp12}), (\ref{exp22}) differ from (\ref{rank22}), (\ref{exp1}), (\ref{exp2}), respectively, by replacing the multivector $T=M^\dagger M$ with the multivector $M$. \section{Conclusions}\label{secConcl} In this paper, we implement the notion of rank of multivector in complexified Clifford geometric algebras without using the corresponding matrix representations. Theorem \ref{thrank} involves only operations in geometric algebras. To obtain the results, we use the fact that rank of the matrix $A\in\Mat(N, \BC)$ is the number of nonzero (positive) singular values of $A$, that is, the number of nonzero (positive) eigenvalues of the matrix $A^\dagger A$. Theorems \ref{thrankpr}, \ref{thrankpr2}, \ref{thrank}, \ref{thrankherm} are new. New explicit formulas (\ref{exp1}), (\ref{exp2}) for the cases of dimensions $3$ and $4$ can be used in various applications of geometric algebras in physics, engineering, and computer science. Note that the results of this work are valid not only for complexified Clifford geometric algebras, but also for real Clifford geometric algebras, since we can use the same matrix representations in the real case (but these matrix representations will have non-minimal dimension in this case). Studying the relationship between rank of multivector and other classical matrix concepts related to rank, such as rows and columns, minors, and row echelon form in terms of geometric algebra operations without using the corresponding matrix representation is an interesting task for further research. \begin{ack}[Conflict of interest] This work does not have any conflicts of interest. \end{ack} \bigskip Data sharing not applicable to this article as no datasets were generated or analysed during the current study. \begin{thebibliography}{100} \bibitem{Abd} Abdulkhaev, K., Shirokov, D.: On Explicit Formulas for Characteristic Polynomial Coefficients in Geometric Algebras. In: Magnenat-Thalmann N. et al. (eds) Advances in Computer Graphics. CGI 2021. Lecture Notes in Computer Science, vol 13002, 670--681. Springer, Cham (2021) \bibitem{Abd2} Abdulkhaev, K., Shirokov, D.: Basis-free Formulas for Characteristic Polynomial Coefficients in Geometric Algebras, Adv. Appl. Clifford Algebras \textbf{32}, 57 (2022) \bibitem{Acus} Acus, A., Dargys, A.: The characteristic polynomial in calculation of exponential and elementary functions in Clifford algebras, Mathematical Methods in the Applied Sciences (2023). \bibitem{inv1} Acus, A., Dargys, A.: Geometric Algebra Mathematica package (last visited 2024). https://github.com/ArturasAcus/GeometricAlgebra \bibitem{acus} Acus, A., Dargys, A.: The Inverse of a Multivector: Beyond the Threshold $p+q=5$, Adv. Appl. Clifford Algebras \textbf{28}, 65 (2018) \bibitem{ABS} Atiyah M, Bott R, Shapiro A. \textit{Clifford Modules.} Topology 3; 1964, 3-38. \bibitem{Bayro1} Bayro-Corrochano E. \textit{Geometric Algebra Applications Vol. I: Computer Vision, Graphics and Neurocomputing}. 1st ed; 2019. \bibitem{Bayro2} Bayro-Corrochano E. \textit{Geometric Algebra Applications Vol. II: Robot Modelling and Control} 1st ed; 2020. \bibitem{Beltrami} Beltrami, E.: Sulle funzioni bilineari. Giomale di Matematiche ad Uso degli Studenti Delle Universita. 1873. V. 11. \bibitem{BT} Benn IM, Tucker RW. \textit{An introduction to Spinors and Geometry with Applications in Physics.} Bristol; 1987. \bibitem{Doran} Doran, C., Lasenby, A.: Geometric Algebra for Physicists. Cambridge University Press, Cambridge (2003) \bibitem{Dorst} Dorst L, Fontijne D, Mann S. \textit{Geometric algebra for computer science: an object-oriented approach to geometry}. Morgan Kaufmann; 2007. \bibitem{For} Forsythe, G., Malcolm, M., Moler, C., Computer Methods for Mathematical Computations (Prentice Hall, Upper Saddle River, 1977). \bibitem{Van} Golub G., Van Loan C., Matrix Computations, JHU Press, Baltimore, 1989. \bibitem{Jordan1} Jordan, C.: Memoire sur lesformes bilineaires, J. Math. Pures Appl., 2e serie. 19, 1874. \bibitem{Jordan2} Jordan, C., Sur la reduction desformes bilineaires. Comptes Rendus de l'Academie Sciences, Paris. 78, 1874. \bibitem{inv2} Hadfield, H., Wieser, E., Arsenovic, A., Kern, R., and The Pygae Team: pygae/clifford: v1.4.0 (last visited 2024). https://github.com/pygae/clifford, https://clifford.readthedocs.io/en/latest/api/clifford.MultiVector.html \bibitem{Helm} Helmstetter J, Micali A. \textit{Quadratic Mappings and Clifford Algebras}. Birkhäuser Verlag AG; 2008. \bibitem{Hestenes} Hestenes, D., Sobczyk, G.: Clifford Algebra to Geometric Calculus - A Unified Language for Mathematics and Physics, Reidel Publishing Company, Dordrecht Holland (1984) \bibitem{Lounesto} Lounesto, P.: Clifford Algebras and Spinors. Cambridge Univ. Press, Cambridge (1997) \bibitem{unitary} Marchuk, N., Shirokov, D.: Unitary spaces on Clifford algebras. Adv. Appl. Clifford Algebras \textbf{18}:2 (2008), 237--254. \bibitem{Porteous} Porteous I. \textit{Clifford Algebras and the Classical Groups}. Cambridge University Press, Cambridge; 1995. \bibitem{inv3} Sangwine, S., Hitzer, E.: Clifford multivector toolbox (for MATLAB), (last visited 2024). http://clifford-multivector-toolbox.sourceforge.net/; https://gaupdate.wordpress.com/2024/02/10/cliff-mv-toolbox-matlab-release-2-1/. \bibitem{rudn} Shirokov, D.: Concepts of trace, determinant and inverse of Clifford algebra elements. In: Progress in analysis. Proceedings of the 8th congress of ISAAC, Volume 1, Peoples’ Friendship University of Russia (ISBN 978-5-209-04582-3/hbk), 2012, 187--194; arXiv:1108.5447 (2011) \bibitem{det} Shirokov, D.: On computing the determinant, other characteristic polynomial coefficients, and inverse in Clifford algebras of arbitrary dimension. Computational and Applied Mathematics \textbf{40}, 173 (2021) \bibitem{Sylv} Shirokov, D.: On basis-free solution to Sylvester equation in geometric algebra. In: Magnenat-Thalmann N. et al. (eds) Advances in Computer Graphics. CGI 2020. Lecture Notes in Computer Science, vol 12221, 541--548. Springer, Cham (2020) \bibitem{Sylv2} Shirokov, D.: Basis-free solution to Sylvester equation in Clifford algebra of arbitrary dimension. Adv. Appl. Clifford Algebras \textbf{31}, 70 (2021) \bibitem{Vieta1} Shirokov, D.: On Noncommutative Vieta Theorem in Geometric Algebras, In: Empowering Novel Geometric Algebra for Graphics and Engineering. ENGAGE 2022. Lecture Notes in Computer Science, 13862, eds. Hitzer, E., Papagiannakis, G., Vasik, P., Springer, Cham, 2023, 28--37. \bibitem{Vieta2} Shirokov, D.: Noncommutative Vieta Theorem in Clifford Geometric Algebras, Mathematical Methods in the Applied Sciences, 2023 (First published), 16 pp., https://doi.org/10.1002/mma.9221 \bibitem{SVDENGAGE} Shirokov, D.: On Singular Value Decomposition and Polar Decomposition in Geometric Algebras. In: Advances in Computer Graphics. CGI 2023. Lecture Notes in Computer Science (Shanghai, 28 August – 01 September 2023), \textbf{14498}, eds. Sheng, B., Bi, L., Kim, J., Magnenat-Thalmann, N., Thalmann, D., Springer, Cham, 2024, 391--401. \bibitem{SVDAACA} Shirokov, D.: On SVD and Polar Decomposition in Real and Complexified Clifford Algebras. Adv. Appl. Clifford Algebras \textbf{34}, 23 (2024) \bibitem{Bulg} Shirokov, D.: Clifford algebras and their applications to Lie groups and spinors. In: Proceedings of the Nineteenth International Conference on Geometry, Integrability and Quantization (Varna, Bulgaria, June 2--7, 2017), eds. Ivaïlo Mladenov and Akira Yoshioka, Avangard Prima, Sofia, Bulgaria, 2018, 11--53. \bibitem{LMA} Shirokov, D.: Classification of Lie algebras of specific type in complexified Clifford algebras. Linear and Multilinear Algebra, \textbf{66}:9 (2018), 1870--1887 \bibitem{Pauli} Shirokov, D.: Extension of Pauli’s theorem to Clifford algebras, Dokl. Math., \textbf{84}:2, 699--701 (2011). \bibitem{nspinors} Shirokov, D.: Pauli theorem in the description of n-dimensional spinors in the Clifford algebra formalism. Theoret. and Math. Phys. 2013; \textbf{175}(1): 454--474. \bibitem{Snygg} Snygg J. \textit{Clifford Algebra - A Computational Tool For Physicists}. New York: Oxford University Press; 1997. \end{thebibliography} \end{document}
2412.02679v1
http://arxiv.org/abs/2412.02679v1
On z-Superstable and Critical Configurations of Chip Firing Pairs
\documentclass[11pt,twoside]{amsart} \title{On z-superstable and critical configurations of chip-firing pairs} \author{Zach Benton} \address{Stanford University} \email{[email protected]} \author{Jane Kwak} \address{UCLA} \email{[email protected]} \author{Suho Oh} \address{Texas State University} \email{[email protected]} \author{Mateo Torres} \address{University of Delaware} \email{[email protected]} \author{Mckinley Xie} \address{Texas A\&M University} \email{[email protected]} \usepackage{prefix} \date{} \begin{document} \begin{abstract} It is well known that there is a duality map between the superstable configurations and the critical configurations of a graph. This was extended to all M-matrices in (Guzm\`an-Klivans 2015). We show a natural way to extend this to all $(L,M)$-chip firing pairs introduced in (Guzm\`an-Klivans 2016). In addition, we study various properties of this map. \end{abstract} \maketitle \input{introandprelim} \section{The Duality map for chip-firing pairs} In this section we establish the duality between the superstable configurations and critical configurations of $(L,M)$-pairs, that extends the canonical duality between superstable and critical configurations of $M$-matrices. We are mainly going to be dealing with the pre-images of the configurations in $R^{+}$. The idea is to group up the preimages that have the same floor and then map such groupings into different groupings. \begin{remark} Recall that for chip-firing pairs, the coordinate-wise maximal critical configuration does not necessarily exist, as was the case in \cref{ex:nocmax}. However, for $M$-matrices it does. Throughout the paper, given any $(L,M)$-pair and we write $\cm$, it stands for $\cm$ of $M$. \end{remark} Consider the map $\sst(M) \rightarrow \sst(M)$ that sends $\vec s \in \sst(M)$ to $\sstab(\cmax-\vec s)$, where $\sstab(\vec{v})$ means we are taking the unique superstable configuration in the equivalence class that contains $\vec{v}$ (we use $\crit(\vec{v}$ for the unique critical configuration in the same class). This map only relies on $M$ and is completely independent of $L$. This map sends a superstable configuration to the unique superstable configuration in the same equivalence class (under $M$) as the image of $\vec s$ under the usual duality map. Now we are going to use the information of $L$, to put a mask on this map: \begin{definition} \label{def:ivomap} For any chip-firing pair $(L,M)$, we define the map $\ivo : \sst(M) \rightarrow \sst(M)$: \[ \ivo(\vec s) = \begin{cases} \vec s &\text{if $\{\LM2\vec s\:\} = \{\LM\cm\}$, } \\ \sstab(\cmax-\vec s) &\text{otherwise.} \end{cases} \] \end{definition} \begin{proposition} The map $\ivo$ is an involution. \end{proposition} \begin{proof} If $\{\LM2\vec s\:\} = \{\LM\cm\}$, we have the identity map. If that condition does not hold, then we have \begin{align*} \ivo(\ivo(\vec s)) &= \sstab(\cm-\sstab(\cm-\vec s))\\ &= \sstab(\cm-(\cm-\vec s))\\ &= \sstab(\vec s)\\ &= \vec s \end{align*} Therefore, $\ivo(\ivo(\vec s)) = \vec s$. \end{proof} In the special case where $L=M$ (this recovers the usual chip-firing on $M$-matrices, and in particular when $M$ is the Laplacian of a graph, the classical chip-firing) the above involution is simply the identity map: since $\LM2\vec s$ is an integer vector for any integer vector $\vec s$. \begin{lemma} \label{lem:equivsameblock} Let $\vec{a},\vec{b}$ be integer vectors such that they are equivalent under $M$. Let $\vec{f}$ be a vector such that every entry $f_i$ satisfies $0 \leq f_i < 1$. Then $\vec{a}+\vec{f} \in R^{+}$ if and only if $\vec{b}+\vec{f} \in R^{+}$. \end{lemma} \begin{proof} Recall that we have a nonnegative rational vector $\vec{v} \in R^{+}$ if and only if $LM^{-1}\vec{v}$ is a integer vector. If $\vec{a} \equiv_M \vec{b}$ then we may write $\vec{b} = \vec{a} + M\vec{z}$ where $z$ is an integer vector. This gives us $LM^{-1}\vec{b} = LM^{-1}\vec{a} + L\vec{z}$ and since $L$ and $\vec{z}$ are integral, we get the desired claim. \end{proof} \begin{example} Consider the signed graph from \cref{ex:usualbad}. We start by focusing on the chip-firing of the underlying graph (ignoring $L$ for now). From the configuration $\vec a = (1, 2, 0)$, we can fire vertex 2 to obtain the configuration $\vec b = (2, 0, 1)$, so these configurations are firing-equivalent (equivalent under $M$). Take some non-negative rational vectors where the entries are bounded above by $1$, say $\vec f_1 = (\nicefrac 1 3, \nicefrac 2 3, 0)$ and $\vec f_2 = (\nicefrac 2 3, \nicefrac 1 3, 0)$. Looking at the images under $LM^{-1}$ we get the following table. \begin{center} \begin{tikzpicture}[scale=2] \input{tikz/kyle-signed} \end{tikzpicture} \end{center} \begin{center} \begin{tabular}{c | c} Preimage & Image under $LM^{-1}$ \\ \hline $\vec a + \vec f_1 = (\nicefrac 4 3, \nicefrac 8 3, 0)$ & $(8, 7, 0)$ \\ $\vec b + \vec f_1 = (\nicefrac 7 3, \nicefrac 2 3, 1)$ & $(7, 5, 1)$ \\ \hline $\vec a + \vec f_2 = (\nicefrac 5 3, \nicefrac 7 3, 0)$ & $(8, \nicefrac{27}{4}, 0)$\\ $\vec b + \vec f_2 = (\nicefrac 8 3, \nicefrac 1 3, 1)$ & $(7, \nicefrac{19}{4}, 1)$ \end{tabular} \end{center} Notice from the table that we have $\vec{a} + \vec f_1$ and $\vec{b} + \vec f_1$ are both in $R^{+}$ whereas $\vec{a}+ \vec f_2$ and $\vec{b} + \vec f_2$ are both not in $R^{+}$, which is consistent with \cref{lem:equivsameblock}. \end{example} \begin{theorem} Let $(L,M)$ be any chip-firing pair. The map $\dual : \sst(L,M) \rightarrow \crit(L,M)$ given by $\ss \mapsto \cm-\ivo(\floor{\ss}) + \frp{\ss}$ is a bijection. \end{theorem} \begin{proof} We approach this proof in two cases. When $\ivo(\floor{\vec s})=\sstab(\cm-\floor{\vec s})$, we have \begin{align*} \cm - \ivo(\floor{\vec s}) &= \cm - \sstab(\cm - \floor{\vec s}).\\ \intertext{Since $\cm - \vec s$ is critical for any superstable configuration $\vec s$ of $M$,} &= \crit(\cm - (\cm - \floor{\vec s}))\\ &= \crit(\floor{\vec s}). \end{align*} From \cref{lem:equivsameblock}, since $\vec s = \floor{\vec s} + \{\vec s\} \in R^+$, we have that $\crit(\floor{\vec s}) + \{\vec s\}$ is in $R^+$, and is a critical configuration of $(L,M)$ thanks to \cref{thm:22floor}. In the second case when $\ivo(\floor{\vec s})=\floor{\vec s}$, recall that \begin{align*} \ivo(\floor{\vec s})=\floor{\vec s} &\iff \{\LM(2\floor{\vec s})\} = \{\LM(\cm)\} \\&\iff \{\LM(\floor{\vec s})\} = \{\LM(\cm - \floor{\vec s})\}. \end{align*} Together with the fact that $LM^{-1}(\floor{\vec s} + \{\vec s\})$ is integral, the above tells us that $LM^{-1}(\cm-\floor{\vec s} + \{\vec s\})$ is also integral. Hence $\cm-\floor{\vec s} + \{\vec s\} \in R^{+}$ and is a critical configuration of $(L,M)$ thanks to \cref{thm:22floor}. \end{proof} \begin{example} Again take a look at the signed graph from \cref{ex:usualbad}. \begin{center} \begin{tikzpicture}[scale=2] \input{tikz/kyle-signed} \end{tikzpicture} \end{center} Start from the superstable configuration $(5,4,0)$ in $S^{+}$. As can be seen in the table in \cref{ex:usualbad}, its corresponding preimage is $\vec{s} = (\nicefrac{4}{3}, \nicefrac{7}{6},0)$. We can check that $\{LM^{-1}2\floor{\vec s}\} = (0, \nicefrac{1}{2}, 0) \neq (0,0,0) = \{LM^{-1}\cmax\}$, so $\ivo (\floor{\vec s}) = \sstab(\cmax - \floor{\vec s})$. For this graph, we have $\cmax = (2,1,2)$, so $\ivo (\floor{\vec s}) = (0,0,1)$ as $M^{-1}(\cmax - \floor{\vec s} - (0,0,1)) \in \ZZ$. Then \[\dual(\vec s) = \cmax - \ivo(\floor{\vec s}) + \{\vec s\} = (2,1,2) - (0,0,1) + \paren{\nicefrac{1}{3}, \nicefrac{1}{6}, 0} = \paren{\nicefrac{7}{3}, \nicefrac{7}{6}, 1}\] gives a critical preimage in $R^+$, which corresponds to the critical configuration $(8,6,1)$ of $S^+$. The table of superstables and criticals in \cref{ex:usualbad} is aligned in a way so that the superstable configuration and the critical configuration obtained from this duality map are in the same row. \end{example} When we are looking at chip-firing pairs $(M,M)$ the above duality map is exactly same as the previously known duality map for $M$-matrices: sending $\vec{c}$ to $c_{max} - \vec{c}$. \begin{example} Consider the graph from \cref{ex:unsignedkyle}. \begin{center} \begin{tikzpicture}[scale=2] \input{tikz/kyle-unsigned} \end{tikzpicture} \end{center} We have that $(0,0,1)$ and $(1,1,0)$ are superstable configurations (and also superstable preimages, since $LM^{-1}$ is the identity matrix). Since they are integral, they are fixed points in our involution $\ivo$. Then \[\dual(0,0,1) = \cm-\ivo(\floor{(0,0,1)}) + \frp{(0,0,1)} = (2,1,2) - (0,0,1) + (0,0,0) = (2,1,1),\] \[\dual(1,1,0) = \cm-\ivo(\floor{(1,1,0)}) + \frp{(1,1,0)} = (2,1,2) - (1,1,0) + (0,0,0) = (1,0,2),\] We can see that this aligns with the classical duality map between superstable and critical configurations for graphs. \end{example} The inverse of the above duality map looks like the following. \begin{corollary} Let $(L,M)$ be any chip-firing pair. The map $\dual^{-1} : \crit(L,M) \rightarrow \sst(L,M)$ given by $\vec{c} \mapsto \ivo(\cm - \floor{\vec{c}}) + \frp{\vec{c}}$ is a bijection. \end{corollary} \section{Frackets} In this section, we focus on \newword{frackets}, subgroups of the critical group constructed by looking at the fractional parts of preimages. We provide an elegant formula for calculating the cardinality of such groups, and then use that result to enumerate the number of fixed points of the involution map studied in the previous section. \subsection{The definition of frackets} For any integral, invertible matrix $L$, we let $\critgroup(L)$ to denote the group we get by looking at the equivalence classes given by $\equiv_L$. Let $L$ and $M$ be any integral, invertible $n$-by-$n$ matrices. We call such pair $(L,M)$ as an \newword{ii-pair}. When a vector $\vec{f}$ satisfies $0 \leq f_i < 1$ for every coordinate $i$, we call it a \newword{fractional vector}. Recall that for any vector $\vec{f}$, we use $\{\vec f\}$ to denote its fractional part. \begin{definition} Given any ii-pair $(L,M)$ and any fractional vector $\vec f$, we define the $L$-fracket $F^L_{\vec f}$ as the subset of $\mathcal K(L)$ consisting of every equivalence class that has a vector representation $\vv \in \ZZ^n$ such that $\{ML^{-1}\vv\} = \vec{f}$. \end{definition} In other words, an $L$-fracket consists of all equivalence classes of configurations whose images under $ML^{-1}$ have the same fractional part. It is clear from the definition that for any chip-firing pair $(L,M)$ it is also an ii-pair. Moreover $(M,L)$ is also an ii-pair as well. \begin{example} Let us again consider the chip-firing pair $(L,M)$ studied in \cref{table:signed_toy_ss}. \begin{center} \begin{tikzpicture}[scale=2] \input{tikz/kyle-signed} \end{tikzpicture} \begin{tabular}{c | c} Element of $\critgroup(L)$ & Preimages in $R^{+}$ \\ \hline $(7, 6, 8)$ & $(\nicefrac 1 2, 0, \nicefrac 5 2)$ \\ $(8, 6, 7)$ & $(\nicefrac 5 2, 0, \nicefrac 1 2)$ \end{tabular} \end{center} Their preimages both have fractional part $(\nicefrac 1 2, 0, \nicefrac 1 2)$, so we have $[(7,6,8)]_L, [(8,6,7)]_L \in F^L_{(\nicefrac 1 2, 0, \nicefrac 1 2)}$. \begin{center} \begin{tabular}{c | c} Element of $\critgroup(M)$ & Image under $LM^{-1}$ \\ \hline $(0, 0, 0)$ & $(0, 0, 0)$ \\ $(0, 1, 0)$ & $(3, 3, 3)$ \end{tabular} \end{center} The image under $LM^{-1}$ of these configurations both have fractional part $\vec 0$, so we have $[(0,0,0)]_M,[(0,1,0)]_M \in F^M_0$. \end{example} Let us now establish several basic properties on frackets. In general, $F^L_{\vec{f}}$ is not necessarily a group, but the zero fracket is closed under addition: \begin{lemma} \label{lem:zerofrackgroup} The zero fracket $F^L_0$ is a subgroup of $\mathcal{K} (L)$. \end{lemma} Since all cosets have the same size, we have the following result. \begin{lemma} \label{lem:fracketsizesame} For any fractional vectors $\vec{f}$ and $\vec{g}$ such that $F^L_{\vec{f}}$ and $F^L_{\vec{g}}$ are both non-empty, we have that $|F^L_{\vec{f}}| = |F^L_{\vec{g}}|$. \end{lemma} We can go between the elements of $F^L_0$ and $F^M_0$ by the invertible linear transformations $ML^{-1}$ and $LM^{-1}$. This map preserves vector addition and integrality, so we get the following: \begin{proposition}\label{prop:frack_equals_s_frack} For an arbitrary ii-pair $(L,M)$, we have $F^L_0 \cong F^M_0$. \end{proposition} Recall that the cardinality of the critical group $\critgroup(L)$ is given by the determinant of $L$, denoted by $|L|$. Since $F^L_0$ is a subgroup of $\mathcal{K}(L)$ and $F^M_0$ is a subgroup of $\mathcal{K}(M)$ and from \cref{prop:frack_equals_s_frack}, we immediately see that $|F^L_0|$ divides $\gcd(|L|, |M|)$. Moreover, from \cref{lem:fracketsizesame} and \cref{prop:frack_equals_s_frack} we get the following result. \begin{corollary} \label{cor:fracketallsamesize} For any ii-pair $(L,M)$, all $L$-frackets and all $M$-frackets have the same size. \end{corollary} \begin{lemma} \label{lem:equivalencefrackets} Suppose $\vv, \vec u$ are integral vectors with $\vv \equiv_L \vec u$. Then, $\vv$ and $\vec u$ belong to the same $L$-fracket. \end{lemma} \begin{proof} From $\vv \equiv_L \vec u$, we get $\vv = \vec u + L \vec z$ for some integer vector $\vec z$. Then, $ML^{-1}\vv - ML^{-1}\vec u = M \vec z$. Since $M$ is integral, $M \vec z$ is integral, so $ML^{-1} \vv - ML^{-1} \vec u$ is integral. Therefore, we get $\{ML^{-1} \vv\} = \{ML^{-1} \vec u\}$, so $\vv$ and $\vec u$ belong to the same fracket. \end{proof} Since $F^L_0$ is a subgroup of the group $\mathcal{K}(L)$, we can consider the quotient group $\mathcal{K}(L) / F^L_0$. The elements of this group are the images of the fractional vectors indexing all the frackets of $L$. \begin{example} \label{ex:Lfracketquotient} Consider the $ii$-pair coming from the $(L,M)$ pair studied in \cref{ex:usualbad}. We can look at the 2nd column (or the 5th column) of \cref{table:toy_stats} to extract the frackets. There is an $L$-fracket corresponding to each of the following fractional vectors: \[(0, 0, 0), (\nicefrac 1 3, \nicefrac 1 6, 0), (\nicefrac 2 3, \nicefrac 1 3, 0), (0, \nicefrac 1 2, 0) (\nicefrac 1 3, \nicefrac 2 3, 0), (\nicefrac 2 3, \nicefrac 5 6, 0).\] Beware that the image of these fractional vectors under the map $LM^{-1}$ is not necessarily an integral vector. However, there exists a vector in each of the corresponding frackets. By taking a vector from each fracket, we get the elements of $\mathcal{K}(L) / F^L_0$: $$(0,0,0),(3,2,0),(4,3,2),(1,1,0),(4,3,0),(5,4,2).$$ \end{example} \subsection{Computing the size of the frackets} Recall that from \cref{cor:fracketallsamesize}, for any ii-pair $(L,M)$, all $L$-frackets and all $M$-frackets have the same size. So we only need to be able to compute the size of $F^L_0$ in order to obtain the size of all frackets. In this subsection, we show an elegant formula for computing $|F^L_0|$. We begin with a lemma that describes the largest invariant factor of each of $\mathcal{K}(L) / F^L_0$ and $\mathcal{K}(M) / F^M_0$. Given a matrix $M$ with rational entries, we use $\flcm(M)$ to denote the least common multiple of all the denominators of the entries (written in irreducible fractions). Given a matrix $M$ with integer entries, we use $\gcd(M)$ to denote the greatest common divisor of all entries. Given matrices $L,M$ with integer entries, we use $\gcd(L,M)$ to denote the greatest common divisor of all entries of $L$ and $M$. \begin{lemma}\label{lem:frackets_largest_inv_factors} The largest invariant factors for $\mathcal{K}(M) / F^M_0$ and $\mathcal{K}(L) / F^L_0$ are $\flcm(\LM)$ and $\flcm(\ML)$, respectively. \end{lemma} \begin{proof} Let $k = \flcm(\LM)$. Then for any integral vector $\vec{v}$, since $k\LM$ is an integral matrix, we have $k \vec{v} \in F^M_0$. And $k$ is the smallest integer that have this property, since we can use the unit vectors for our $\vec{v}$ as well. So $k$ is the smallest value such that $k\vec{v} = 0$ for all $\vec{v} \in \critgroup(M) / F^M_0$. Thus, $k$ must be the size of the largest invariant factor of $\critgroup(M) / F^M_0$. The other argument comes from applying the claim we just proved on the ii-pair $(M,L)$. \end{proof} \begin{example} Let us revisit the signed graph from our running example. \begin{center} \begin{tikzpicture}[scale=2] \input{tikz/kyle-signed} \end{tikzpicture} \end{center} Recall that from \cref{ex:Lfracketquotient}, the nonempty frackets are indexed by the following fractional vectors: \[(0, 0, 0), (0, \nicefrac 1 2, 0), (\nicefrac 2 3, \nicefrac 1 3, 0), (\nicefrac 2 3, \nicefrac 5 6, 0), (\nicefrac 1 3, \nicefrac 1 6, 0), (\nicefrac 1 3, \nicefrac 2 3, 0).\] The collection of $L$-frackets has group structure $\mathbb Z_6$. One way to see this is by considering $(\nicefrac 1 3, \nicefrac 1 6, 0)$ as a generator. Therefore, the largest invariant factor of $\critgroup(L) / \fracket^L_0$ has size 6. If we look at \begin{gather*} ML^{-1} = \begin{pmatrix} \nicefrac 4 3 & - \nicefrac 4 3 & - \nicefrac 1 3 \\ - \nicefrac 5 6 & \nicefrac 4 3 & - \nicefrac 1 6 \\ 0 & 0 & 1 \end{pmatrix}, \end{gather*} we can see that $\flcm(ML^{-1}) = 6$, which does indeed match the size of the largest invariant factor of $\critgroup(L) / \fracket^L_0$. \end{example} From \cref{lem:frackets_largest_inv_factors}, we can make a general statement about the size of the zero fracket. \begin{theorem}\label{theorem:zero_fracket_size} Let $(L,M)$ be any $ii$-pair. Let $p_M$ be the product of the invariant factors of $\SK(M)/F_0^M$ excluding the largest invariant factor, and let $p_L$ be the product of the invariant factors of $\SK(M)/F_0^M$ excluding the largest invariant factor. Then, $|\fracket^L_0| = \frac{\gcd(|L|\ML,|M|\LM)}{\gcd(p_M, p_L)}$. \end{theorem} \begin{proof} Let $p_M$ denote the product of the invariant factors of $\critgroup(M) / F^M_0$, excluding the largest invariant factor. Then, we see that from \cref{lem:frackets_largest_inv_factors}, together with $\flcm(LM^{-1}) = |M| / \gcd(|M|\LM)$, we get: \begin{align*} |F^M_0| p_M &= |F^M_0| \paren{ \dfrac {| \critgroup(M) / F^M_0|} { |M| / \gcd(|M|\LM)} } \\ &= |F^M_0| \frac{|M| / |F^M_0|}{|M| / \gcd(|M|\LM)} \\ &= \gcd(|M|\LM). \end{align*} Similarly, we have $|\fracket^L_0|p_L = \gcd(|L|\ML)$. Using \cref{prop:frack_equals_s_frack}, we have $|\fracket^L_0|p_M = |F^M_0| p_M = \gcd(|M|\LM)$. Combining these results together, we get \[|\fracket^L_0| = \frac{\gcd(|L|\ML, |M|\LM)}{ \gcd(p_M, p_L)}.\] \end{proof} \begin{example}\label{example:naive_zero_fracket} Let us revisit the graph from our running example. \begin{center} \begin{tikzpicture}[scale=2] \input{tikz/kyle-signed} \end{tikzpicture} \end{center} \begin{multicols}{2} \begin{center} \begin{tabular}{c | c} $L$-Frackets & $M$-Frackets \\ \hline $(0, 0, 0)$ & $(0, 0, 0)$ \\ $(\nicefrac 1 3, \nicefrac 1 6, 0)$ & $(0, \nicefrac 1 4, 0)$ \\ $(\nicefrac 2 3, \nicefrac 1 3, 0)$ & $(0, \nicefrac 1 2, 0)$ \\ $(0, \nicefrac 1 2, 0)$ & $(0, \nicefrac 3 4, 0)$\\ $(\nicefrac 1 3, \nicefrac 2 3, 0)$ & \\ $(\nicefrac 2 3, \nicefrac 5 6, 0)$ & \end{tabular} \end{center}\columnbreak \[|L| ML^{-1} = \begin{pmatrix} 16 & - 16 & - 4 \\ - 10 & 16 & -2 \\ 0 & 0 & 2 \end{pmatrix}\]\[ |M| LM^{-1} = \begin{pmatrix} 16 & 16 & 8 \\ 10 & 16 & 6 \\ 0 & 0 & 8 \end{pmatrix}\] \end{multicols} From the table, we see that $\mathcal{K}(M) / F^M_0 \cong \ZZ_4$, and $\mathcal{K}(L) / \fracket^L_0 \cong \ZZ_6$. Then $p_M = 1$ and $p_L = 1$ since there is only one invariant factor. We also see that the greatest common divisor of $|M|LM^{-1}$ and $|L|ML^{-1}$ is $2$, so we expect the zero fracket to have size $2$. Indeed, $F^L_0$ consists only of the equivalence classes of $(0, 0, 0)$ and $(3, 3, 3)$. \end{example} In the above example, we needed to compute the structure of $\mathcal{K}(M) / F^M_0$ and $\mathcal{K}(L) / \fracket_0$ in order to obtain the size of the frackets. However, in many cases (conjecturally most cases for chip-firing pairs coming from graphs \cite[Conjecture 2]{clancy2015}), we can bypass this computation. If either $\mathcal{K}(M) / F^M_0$ or $\mathcal{K}(L) / \fracket^L_0$ is cyclic, we get a much simpler expression for $|\fracket^L_0|$. \begin{corollary} \label{cor:cyclic_fracket_size} Choose an arbitrary ii-pair $(L, M)$. Then, $\critgroup(M) / F^M_0$ is cyclic if and only if $|\fracket^L_0| = \gcd(|M| \LM)$. Similarly, $\critgroup(L) / \fracket_0^L$ is cyclic if and only if $|\fracket^L_0| = \gcd(|L|\ML)$. \end{corollary} \begin{proof} If $\SK(M)/F_0^M$ is cyclic, then $|\SK(M)/F_0^M| = \flcm(LM^{-1})$ by \cref{lem:frackets_largest_inv_factors}. Thus, \begin{gather*} \frac{|M|}{|F_0^M|} = |\SK(M)/F_0^M| = \flcm(LM^{-1}) = \frac{|M|}{\gcd(|M|LM^{-1})}, \end{gather*} establishing that $|F_0^M| = \gcd(|M|LM^{-1})$. For the other direction of the proof, suppose $|\fracket^M_0| = \gcd(|M| \LM)$. Then, $|\critgroup(M) / F^M_0| = |M| / \gcd(|M| \LM)$. By \cref{lem:frackets_largest_inv_factors}, the largest invariant factor of $\critgroup(M) / F^M_0$ also has size $\flcm (LM^{-1}) = |M| / \gcd(|M| \LM)$. Therefore, $\critgroup(M) / F^M_0$ has exactly one invariant factor, which means it is cyclic. \end{proof} Therefore, if there somehow is a guarantee that either one of $\mathcal{K}(M) / F^M_0$ or $\mathcal{K}(L) / \fracket^L_0$ is cyclic, \cref{cor:cyclic_fracket_size} gives us a way to enumerate the size of the frackets in a very simple manner. For example, consider the ii-pair $(L,M)$ coming from a signed graph. If we know that the underlying graph has a cyclic critical group $\mathcal{K}(M)$, then $\mathcal{K}(M) / F^M_0$ must also be cyclic. \begin{example}\label{example:soph_zero_fracket} Let us revisit \cref{example:naive_zero_fracket} using \cref{cor:cyclic_fracket_size}, given that the underlying graph's critical group is cyclic. All we need is to compute $|M|LM^{-1}$ (which we have already done in \cref{example:naive_zero_fracket}). We see that $\gcd(|M|LM^{-1}) = 2$, which matches $|\fracket_0^L|$. \end{example} \subsection{Analyzing the number of fixed points in the involution map} In \cref{def:ivomap} we defined an involution $\ivo$ on the set of preimages of superstable configurations of an $M$-matrix, given a chip-firing pair $(L,M)$. In this section, we use the techniques developed in the previous subsections to count the number of fixed points of $\ivo$. Recall that the fixed points of $\ivo$ are $\vec s \in \sstab(M)$ such that $\{\LM(\cm - 2\ss)\} = \vec{0}.$ \begin{theorem}\label{thm:numsols} The number of $\ss \in \sstab(M)$, satisfying $\cm - 2\ss \in F^M_0$ is equal to either $0$ or $|F^M_0|d$, where $d$ is the number of elements of $\mathcal{K}(M) / F^M_0$ with order at most 2. \end{theorem} \begin{proof} Suppose that there exists some $s \in \sst(M)$ such that $\cm - 2s \in F^M_0$. Note that every element $g \in \SK(M)$ is uniquely expressible as $s + h + f$ for some $h \in \SK(M)/F^M_0$, $f \in F^M_0$. Then, $\cm - 2g = (\cm - 2s) - 2f - 2h$. Since $\cm - 2s \in F^M_0$ and $2f \in F^M_0$, we have that $\cm - 2g \in F^M_0$ if and only if $2h \in F^M_0$. There are $|F_0^M|$ possible choices for $f$, and $d$ possible choices for $h$, so there are $|F_0^M|d$ such options for $g$. \end{proof} \begin{example} Let's revisit the running example of a $(L,M)$-pair coming from a signed graph. \begin{center} \begin{tabular}{c | c} $\sstab(M)$ & Its image (under $LM^{-1}$) \\ \hline $(0, 0, 0)$ & $(0, 0, 0)$ \\ $(0, 0, 1)$ & $(1, \nicefrac 3 4, 1)$ \\ $(0, 0, 2)$ & $(2, \nicefrac 3 2, 2)$ \\ $(0, 1, 0)$ & $(2, 2, 0)$ \\ $(0, 1, 1)$ & $(3, \nicefrac{11}{4}, 1)$ \\ $(1, 0, 0)$ & $(2, \nicefrac 5 4, 1)$ \\ $(1, 1, 0)$ & $(4, \nicefrac{13}{4}, 1)$ \\ $(2, 0, 0)$ & $(4, \nicefrac 5 2, 2)$ \\ \hline $\cm = (2, 1, 2)$ & $(8, 6, 4)$ \end{tabular} \end{center} From the above table, we see that $\cm \in F^M_0$. Therefore, the $4$ superstable preimages that $2\ss$ is in the zero fracket of $M$ will be the fixed point of $\mu$. Recall that $\mathcal{K}(M) = \ZZ_8$ and $|F^M_0| = 2$ (from \cref{example:soph_zero_fracket}). Therefore, $\mathcal{K}(M) / F^M_0 \cong \ZZ_4$, which has $2$ elements of order at most $2$. Using $\cref{thm:numsols}$ with $d = 2$ and $|F^M_0| = 2$, we can verify that we get $4$ fixed points. \end{example} The remainder of this subsection will be on other more specific observations and corollaries of \cref{thm:numsols} and \cref{cor:cyclic_fracket_size}. \begin{proposition}\label{oddorder} If $\cm$ has odd order in $\mathcal{K}(M) / F^M_0$, then the number of fixed points of $\ivo$ is nonzero. \end{proposition} \begin{proof} Let $H$ be the subgroup of $\mathcal{K}(M) / F^M_0$ generated by $\cm$. Since $\cm$ has odd order, we know that $|H|$ is odd. There is no element of $H$ with order 2, so it follows that there is some element $g \in H$ such that $2g = \cm$. Then, $g$ is a fixed point. \end{proof} If $\mathcal{K}(M) / F^M_0$ is cyclic and $\cm$ has even order, we have an exact criterion to check whether the number of fixed points is zero or not. \begin{corollary} Suppose $\mathcal{K}(M) / F^M_0$ is cyclic and $\cm$ has even order in $\mathcal{K}(M) / F^M_0$. Then, the number of fixed points of $\ivo$ is nonzero if and only if $\frac{|\mathcal{K}(M) / F^M_0|}{\ord(\cm)}$ is even. \end{corollary} \begin{proof} Suppose $g$ generates $\mathcal{K}(M) / F^M_0$, so $\cm = kg$ for some positive integer $k$. Since $\ord(\cm) k g = 0$ and $\mathcal{K}(M) / F^M_0$ is cyclic, we know that $|\mathcal{K}(M) / F^M_0|$ divides $\ord(\cm)k$. Therefore, $|\mathcal{K}(M) / F^M_0| / \ord(\cm)$ must divide $k$. Since $\frac{|\mathcal{K}(M) / F^M_0|}{\ord(\cm)}$ is even, we see that $k$ must be even as well. Therefore, $(k/2)g$ is a fixed point of $\ivo$. For the other direction of the proof, assume there is some fixed point $g$ of $\ivo$. then we have $2g = \cm$ in $\mathcal{K}(M) / F^M_0$. We know that $2 \ord(\cm)g = 0$, so $\ord(g)$ must divide $2 \ord(\cm)$. Let $H$ be the subgroup of $\mathcal{K}(M) / F^M_0$ generated by $g$. Then, $\cm \in H$ and $|H|$ cyclic, so $\ord(\cm)$ must divide $|H| = \ord(g)$. So we have $\ord(g) | 2\ord(\cm)$ and $\ord(\cm) | \ord(g)$. Thus, one of the two happens: $\ord(g) = \ord(\cm)$ or $\ord(g) = 2 \ord(\cm)$. Suppose for the sake of contradiction that $\ord(g) = \ord(\cm)$. In other words, $\ord(g) = \ord(2g)$. This means that $\ord(g)$ is odd. By our assumption that $\ord(g) = \ord(\cm)$, it follows that $\ord(\cm)$ is also odd, contradicting our assumption that $\cm$ is of even order. Therefore, $\ord(g) = 2\ord(\cm)$. Then $\frac{|\mathcal{K}(M) / F^M_0|}{\ord(\cm)} = 2 \frac{|\mathcal{K}(M) / F^M_0|}{\ord(g)}$, so $\frac{|\mathcal{K}(M) / F^M_0|}{\ord(\cm)}$ must be even. \end{proof} \subsection{Chip-firing pairs coming from complete graphs} In this subsection, we will be focusing on chip-firing pairs coming from signed graphs, where the underlying graph is the complete graph. Same as the chip-firing pairs coming from signed graphs we have seen before, $L$ is going to denote the (reduced) Laplacian of the signed graph and $M$ is going to denote the (reduced) Laplacian of the underlying complete graph $K_n$, where $n$ is even. \begin{lemma}\label{KLMinv.den} For such $(L,M)$ pairs coming from a complete graph $K_n$ where $n$ is even, \[\frac{n}{2}\LM \text{ is an integer matrix.}\] \end{lemma} \begin{proof} We will show that every term in $\LM$ has a denominator dividing $n/2$. The entries of the inverse of $M$ can be described as the following: \[\Mi_{i,j} = \begin{cases} \frac{2}{n} & \text{ if } i = j \\ \frac{1}{n} & \text{ if } i \neq j. \end{cases}\] From this we analyze the entry $(LM^{-1})_{i,j} = \sum_{k = 1}^{n-1} L_{i,k}\Mi_{k,j}$. Let $\alpha_i$ to denote the number of $-1$'s that appear in row $i$. Then there will be exactly $n-2-\alpha_i$ many $+1$'s in that row. We do a case-by-case analysis. In the case $i=j$, we have that $$(\LM)_{i,j} =\sum_{k = 1}^{n-1} L_{i,k}\Mi_{k,j}= \frac{2(n-1)}{n}-\frac{\alpha_i}{n}+\frac{n-2-\alpha_i}{n} = \frac{3n-4-2\alpha_i}{n}.$$ If $L_{i,j} = -1$, then we have $$(\LM)_{i,j} = \sum_{k = 1}^{n-1} L_{i,k}\Mi_{k,j}= \frac{n-1}{n}-\frac{2}{n} - \frac{\alpha_i-1}{n} +\frac{n-2-\alpha_i}{n} = \frac{2n-4-2\alpha_i}{n}.$$ Finally when $L_{i,j} = 1$ we have \[(\LM)_{i,j} = \sum_{k = 1}^{n-1} L_{i,k}\Mi_{k,j}= \frac{n-1}{n} + \frac{2}{n} - \frac{\alpha_i}{n} +\frac{n-3-\alpha_i}{n} = \frac{2n-2-2\alpha_i}{n}.\] Since $n$ is even, the numerators in all cases are also even. So $\frac{n}{2}\LM$ is an integer matrix. \end{proof} Using the above, we now show that any signed graph of a complete graph $K_n$ where $n$ is a fixed even number, contain a nontrivial common subgroup. Actually we show something stronger, that the zero frackets (a subgroup of the critical group) of any signed graph with underlying graph being the complete graph $K_n$ where $n$ is even, contain a nontrivial common subgroup. \begin{theorem} \label{thm:z0common} The zero fracket $F_0^L$ of any $(L,M)$-pair coming from some signed graph where the underlying graph is the complete graph $K_n$ where $n$ is even, has a subgroup isomorphic to $\ZZ_2^{n-2}$. \end{theorem} \begin{proof} Consider the configuration $\vec s_i := \frac{n}{2} \vec{e_i}$. Then by \cref{KLMinv.den}, we have that $\LM\paren{\frac{n}{2}\vec{e_i}}$ is an integer vector, so this is a valid configuration in $R^+$. From \cref{thm:22floor}, we have that $\vec s_i$ is a superstable perimage. Additionally, $n \vec{e_i} = M(\vec{1} + \vec{e_i})$, so $\vec s_i$ has order 2 in $F_0^L$. For each $I \subseteq \{1\dots n\}$ such that $0 \leq |I| \leq \frac{n-2}{2}$, we know that $\sum_{i \in I} s_i$ is a superstable preimage: when we fire some set $S$, for any $v \in S$ we start with either zero or $\frac{n}{2}$ chips, will lose $n-1$ chips, and gain back at most $|I|-1 \leq \frac{n-4}{2}$ chips, resulting in that vertex having at most $-1$ chips after the firing. And since $\sum_{i \in I} s_i$ is a sum of elements of order $2$, it has order $2$ as well in $F_0^L$. There are $\frac{2^{n-1}}{2}$ possible choices for $I$ such that $0 \leq |I| \leq \frac{n-2}{2}$, so the set of $\sum_{i \in I} s_i$'s form a subgroup of $F_0^L$ isomorphic to $\ZZ_2^{n-2}$. \end{proof} \begin{example} Consider the graph $K_6$. Over all possible signed graphs coming from $K_6$, there are seven possible critical groups up to isomorphism: \begin{gather*} \mathbb Z_6 \oplus \ZZ_6 \oplus \ZZ_6 \oplus \ZZ_6, \\ \ZZ_{36} \oplus \ZZ_{12} \oplus \ZZ_2 \oplus \ZZ_2, \\ \ZZ_{78} \oplus \ZZ_6 \oplus \ZZ_2 \oplus \ZZ_2, \\ \ZZ_{50} \oplus \ZZ_{10} \oplus \ZZ_{2} \oplus \ZZ_{2}, \\ \ZZ_{64} \oplus \ZZ_8 \oplus \ZZ_2 \oplus \ZZ_{2}, \\ \ZZ_{132} \oplus \ZZ_4 \oplus \ZZ_2 \oplus \ZZ_{2}, \\ \ZZ_{36} \oplus \ZZ_4 \oplus \ZZ_4 \oplus \ZZ_4. \end{gather*} We can check that $\mathbb Z_2^4$ is a subgroup of all cases above. \end{example} \section{Further questions} In this section, we discuss the remaining questions that naturally follow this study. The duality between $z$-superstable and critical configurations of $(L,M)$-pairs constructed in Section $3$, relied on the involution $\ivo$ on the set of superstable configurations of $M$. When we constructed $\ivo$, we used the equivalence relation given by $M$. It would be interesting if we can skip that process as well. \begin{question} Can one construct a duality between the set of $z$-superstable configurations and the set of critical configuration of chip-firing pairs without ever relying on the equivalence class computation (of $L$ or $M$)? \end{question} A lot of examples we used came from signed graphs and their Laplacian. It would be interesting if we can generalize \cref{thm:z0common} to more general graphs: \begin{question} Let $(L,M)$ be a chip-firing pair coming from a signed graph with an underlying graph $G$. Is there a way to obtain the biggest group that all critical groups of the chip-firing pairs coming from $G$ contain as a subgroup up to isomorphism? \end{question} For signed graphs, there is an operation called \newword{vertex-switching} \cite{Zaslavsky}. It is known that there is an isomorphism between the critical groups of signed graphs that are switching equivalent to one another. So it is natural to ask the following question: \begin{question} Can one construct a natural isomorphism between the critical groups of switching equivalent $(L, M)$ pairs (without any reliance on the obvious bijection coming from the equivalence classes)? \end{question} Recall that in \cref{thm:numsols} we could only describe the number of fixed points of $\ivo$ when we were guaranteed that it was nonzero. It would be interesting if we have an elegant way to describe exactly when the number of fixed points of $\ivo$ is nonzero in terms of $L$ and $M$: \begin{question} Is there a way to describe exactly when the map $\ivo$ has zero fixed points? \end{question} \section{Acknowledgements} The work presented here was conducted as part of an REU at Texas State University in the summer of 2024, sponsored by NSF. We thank NSF and Texas State for the support and the stimulating work environment. \printbibliography \end{document} \section{Introduction} Chip-firing is a game that takes place on a connected graph $G$, where chips are distributed across the vertices of $G$ and moved to adjacent vertices based on a straightforward rule. This dynamical system has a profound theory that links to various fields in mathematics and physics \cite{BakTangWiesenfeld88}, \cite{Dhar90}, \cite{Gabrielov93}. For further details, please see the recent textbooks \cite{Klivans} and \cite{CorryPerk}. Consider a graph $G$ where one specific vertex is designated as the \emph{sink}, and chips are placed on each non-sink vertex. The allocation of chips is described by an integer vector $\vec{c} \in \mathbb{Z}^{n}$, known as a \textit{(chip) configuration}. A non-sink vertex with chips equal to or greater than its degree can fire, distributing one chip to each adjacent vertex. A configuration $\vec{c}$ is termed \emph{stable} if no non-sink vertex is able to fire. For a connected graph $G$, any starting configuration will eventually reach stability via sequence of firings, with some chips moving to the sink vertex. Letting $v_1, ..., v_{n}$ represent the non-sink vertices of $G$, the chip-firing rules can be described using $L_G$, the $n \times n$ \textit{reduced Laplacian} matrix of $G$. The outcome of firing a vertex $v_i$ on a chip configuration $\vec{c}$ is represented by $\vec{c} - L\vec{e_i}$, where $\vec{e_i}$ denotes the $i$-th standard basis vector. The matrix $L_G$ establishes an equivalence relation among the vectors in ${\mathbb Z}^{n}$, with $\vec{c}$ and $\vec{d}$ being \emph{firing equivalent} if $\vec{c} - \vec{d}$ lies within the image of $L_G$. This determines the \emph{critical group} of $G$, as described by $\mathcal{K}(G) := {\mathbb Z}^n/\Img L_G$ \cite{Biggs99}. A configuration $\vec{c}$ is considered \emph{valid} (or \emph{effective}) if for all $i = 1, \dots, n$, the condition $c_i \geq 0$ holds. The goal is to identify notable valid configurations within each equivalence class $[\vec{c}] \in {\mathcal K}(G)$. It turns out that each $[\vec{c}]$ contains a distinct valid configuration that is \emph{critical}, which means it is stable and can be derived from a sufficiently large configuration $\vec{b}$. Stabilizing the sum of two critical configurations results in a critical configuration. One can show that each $[\vec{c}]$ contains a unique valid configuration that is \emph{superstable}, meaning that it is stable under \emph{set-firings}. A superstable configuration represents a solution to an energy minimization problem and aligns with the concept of a \emph{$G$-parking function}. For a connected graph $G$, a straightforward bijection exists linking the set of critical configurations and the set of superstable configurations, which are both correspondingly in bijection with the set of spanning trees of $G$. This simple map (take a configuration, negate it coordinate-wise from a certain maximal configuration) is what is called the \newword{duality map} between superstable configurations and critical configurations, and our focus is to extend this map to more general models. Recently, chip-firing has been extended to more general settings, where the reduced Laplacian of a graph is replaced by other matrices (see for instance \cite{gabrielov} and \cite{GuzKliMmatrices}). We use a matrix $M$ to define a firing rule that mimics the graphical setting: firing $v_i$ now takes a configuration $\vec{c}$ to $\vec{c} - M \vec{e_i}$, where $\vec{e_i}$ stands for the unit vector with $1$ at the $i$-th coordinate. For a well-defined notion of chip-firing we require that $M$ satisfies an \emph{avalanche finite} property, so that repeated firings of any initial configuration eventually stabilize in an appropriate sense. The class of matrices with this property are known as \emph{$M$-matrices}, and can be characterized in a number of ways (see \cref{def:Mmatrix} below). In \cite{GuzKliMmatrices}, Guzm\'an and Klivans have shown that the chip-firing theory defined by an $M$-matrix leads to good notions of critical and superstable configurations. They further generalized this model by introducing an invertible matrix $L$, called chip-firing pairs, and extended the definition of critical and superstable configurations to that model \cite{GuzKlivans}. Our goal is to extend the duality between critical and superstable configurations to $(L,M)$-chip firing pairs. In Section~$2$ we summarize the Guzm\'an-Klivans theory of chip-firing pairs and also review the previously known duality between superstable and critical configurations of $M$-matrices. In Section~$3 $ we provide our main result on extending the duality map to chip-firing pairs. In Section~$4$ we study some properties of the map discussed in the main result. \section{Prerequisites} In this section we review the definition of chip-firing pairs. After that we review the duality map between superstable configurations and critical configurations for $M$-matrices. Then we will go over the tools developed in \cite{cho2024} that we will use to deal with the $z$-superstable and critical configurations of chip-firing pairs. \subsection{Chip firing pairs} In \cite{GuzKliMmatrices}, Guzm\'an and Klivans generalized the chip-firing on graphs to \newword{$M$-matrices}. $M$-matrices are used in various fields such as economics or scientific computing \cite{BurmanErn05}, \cite{CiarletRaviart73}, \cite{Leontief41}, \cite{Plemmons77}. Guzm\'an and Klivans further generalized this by introducing an invertible integer matrix $L$ in \newword{chip-firing pairs} $(L,M)$ introduced in \cite{GuzKlivans}. \begin{definition} \label{def:Mmatrix} Suppose $M$ is an $n \times n$ matrix such that $(M)_{ii} > 0$ for all $i$ and $(M)_{ij} \leq 0$ for all $i \neq j$. Then $M$ is called an (invertible) \emph{$M$-matrix} if any of the following equivalent conditions hold: \begin{enumerate} \item $M$ is avalanche finite; \item The real part of the eigenvalues of $M$ are positive; \item The entries of $M^{-1}$ are non-negative; \item There exists a vector $\vec{x} \in {\mathbb R}^{n}$ with ${\vec x} \geq \vec{0}$ such that $M \vec{x}$ has all positive entries. \end{enumerate} \end{definition} The pair $(L,M)$, an $M$-matrix $M$ together with an invertible integer matrix $L$, is called a \emph{chip-firing pair}. The relevant (chip) \emph{configurations} $\vec{c} \in \mathbb{Z}^{n}$ are simply integer vectors with $n$ entries, and chip-firing is dictated by the matrix $L$. In particular, $(M,M)$ recovers the chip-firing on $M$-matrices and $(L_G,L_G)$ when $L_G$ is the (reduced) Laplacian of a graph recovers the classical chip-firing model on graphs. \begin{definition}\label{defn:valid} Suppose $(L,M)$ is a chip-firing pair. A configuration $\vec{c}$ is \emph{valid} if $\vec{c} \in S^+$, where \[S^+ = \{LM^{-1} \vec{x} : LM^{-1}\vec{x} \in \mathbb{Z}^{n}, \vec{x} \in \mathbb{R}^n_{\geq 0}\}. \] Equivalently, a configuration $\vec{c}$ is valid if $ML^{-1}\vec{c} \in R^+$, where \[R^+ = \{ \vec{x} \in \mathbb{R}^n_{\geq 0} : LM^{-1}\vec{x} \in \mathbb{Z}^{n} \}.\] \end{definition} In particular, for $(M,M)$, being valid is exactly the same as being a nonnegative integer vector. \begin{definition} Suppose $(L,M)$ is a chip-firing pair, and suppose that $\vec{c} \in S^+$ is a valid configuration. A site $i \in \{1,\dots, n\}$ is \emph{ready to fire} if \[\vec{c} - L\vec{e}_i \in S^+,\] so that the vector obtained by subtracting the $i$th row of $L$ from $\vec{c}$ is also valid. Similarly, suppose $\vec{x} \in R^+$. Then a site $i \in \{1,\dots, n\}$ is \emph{ready to fire} if \[\vec{x} - M\vec{e}_i \in R^+.\] A configuration $\vec{c}$ (in $S^+$ or $R^+$) is \emph{stable} if no site is ready to fire. \end{definition} If $i$ is ready to fire, we declare that $\vec{b} = \vec{c} - L \vec{e}_i \in S^+$ is derived from $\vec{c}$ through a \emph{legal firing}. Repeating this process, a vector $\vec{a} \in S^+$ is said to be derived from $\vec{c}$ through a \emph{sequence of legal firings}. For a configuration $\vec{c} \in S^+$ (or conversely, $\vec{d} \in R^+$), we define $\text{stab}_{S^+}(\vec{c})$ (and $\text{stab}_{R^+}(\vec{d})$) as the resulting configuration after executing a series of legal firings until no site remains eligible to fire. Adapting the proof presented in \cite[Theorem 2.2.2]{Klivans}, it can be established that both $\text{stab}_{S^+}(\vec{c})$ and $\text{stab}_{R^+}(\vec{d})$ are uniquely determined. When it is clear whether we are dealing with $S^{+}$ or $R^{+}$, we use $\text{stab}(\vec{x})$ to refer to the stabilization of the configuration $\vec{x}$. The definition of critical and superstable configurations in this model are as follows. \begin{definition} Given an $(L,M)$ pair, a configuration $\vec c \in S^+$ is reachable if there exists some configuration $\vec d \in S^+$ satisfying: \begin{itemize} \item $\vec d - L \vec e_i \in S^+$ for all $1 \leq i \leq n$ \item $\vec c = \vec d - \sum_{j = 1}^k L \vec e_j$ and $\vec d - \sum_{j = 1}^\ell L \vec e_j \in S^+$ for all $\ell < k$. \end{itemize} Given an $(L,M)$ pair, a configuration $\vec c \in S^+$ is critical if $\vec c$ is both stable and reachable. \end{definition} \begin{definition}[{\cite[Definition 4.3]{GuzKliMmatrices}}] A vector $f \in \ZZ^n$ with $f \geq 0$ is \newword{$z$-superstable} if for every $z \in \ZZ^n$ with $z \geq 0$ and $z \neq 0$ there exists $1 \leq i \leq n$ such that $f_i - (Lz)_i < 0$. \end{definition} It turns out that in the equivalence class given by the matrix $L$ in $S^{+}$, we can always find a unique representative that is critical and a unique representative that is $z$-superstable. \begin{theorem}[{\cite[Theorems 3.5, 4.3, 5.5]{GuzKlivans}\label{thm:class}}] Suppose $(L,M)$ is a chip-firing pair. Then there exists exactly one $z$-superstable configuration and one critical configuration in each equivalence class $[\vec{c}]_L$. \end{theorem} \begin{remark} For chip-firing pairs, there is the notion of $\chi$-superstable configurations and $z$-superstable configurations. From \cref{thm:class}, it is the $z$-superstable configurations that have the same size as the critical configurations. We will only focus on the $z$-superstable configurations, and from now on throughout the paper, we will just call them the superstable configurations of the chip-firing pair, omitting the letter $z$. \end{remark} In the next subsection, we go over the duality that is known to exist when $L=M$. \subsection{Duality for \texorpdfstring{$M$}{M}-matrices} If we take a chip-firing pair $(M,M)$, it recovers the chip-firing on $M$-matrices studied in \cite{GuzKliMmatrices}. Chip-firing on $M$-matrices generalizes many properties and results of the classical chip-firing on graphs, and one of them is the duality between superstable and critical configurations. Given an $M$-matrix, it turns out that there is a critical configuration that is a coordinate-wise greater or equal to every other critical configuration. We call this configuration $\cm$, given by taking all diagonal entries of $M$ minus one and forming a vector (in the classical case, this corresponds to having $\deg(v)-1$ chips for each vertex $v$). \begin{theorem}[\cite{GuzKliMmatrices}] \label{thm:usualMduality} Let $M$ be an $M$-matrix. Let $\cm$ denote the vector where each entry is coming from the corresponding diagonal entry $M_{ii}$ minus one. Then we have a bijection between superstable and critical configurations by the map $\vec{c} \rightarrow \cm - \vec{c}$. \end{theorem} \begin{example} \label{ex:unsignedkyle} Consider the following graph that has the reduced Laplacian to be $L_G = \begin{pmatrix} 3 & -1 & -1 \\ -1 & 2 & -1 \\ -1 & -1 & 3 \end{pmatrix}.$ \begin{center} \begin{tikzpicture}[scale=2] \input{tikz/kyle-unsigned} \end{tikzpicture} \end{center} The superstable configurations and critical configurations are given in the following table: \begin{center}\label{table:unsigned_toy_ss} \begin{tabular}{ c | c } Superstables & Criticals \\ \hline $(0, 0, 0)$ & $(2, 1, 2)$\\ $(0, 0, 1)$ & $(2, 1, 1)$\\ $(0, 0, 2)$ & $(2, 1, 0)$\\ $(0, 1, 0)$ & $(2, 0, 2)$\\ \end{tabular} \begin{tabular}{ c | c } Superstables & Criticals \\ \hline $(0, 1, 1)$ & $(2, 0, 1)$\\ $(1, 0, 0)$ & $(1, 1, 2)$\\ $(1, 1, 0)$ & $(1, 0, 2)$\\ $(2, 0, 0)$ & $(0, 1, 2)$ \end{tabular} \end{center} \end{example} Notice that in the above example, we have a bijection between superstable configurations and critical configurations via the map $\vec{c} \rightarrow (2,1,2) - \vec{c}$. Recall that our goal is to extend this to $(L,M)$ chip-firing pairs. Next subsection will show that the map $\vec{c} \rightarrow \cmax - \vec{c}$ does not work for chip-firing pairs. \subsection{The usual duality map does not work for chip-firing pairs} Recall that the usual duality map between the superstable and critical configurations for graphs (and also $M$-matrices) is given by the map $\vec{c} \rightarrow \cm - \vec{c}$ for some fixed $\cm$. As can be seen in the example below, this does not work for $(L,M)$-pairs in general. The examples from this point throughout will be using $(L,M)$-pairs coming from a signed graph. The systematic study of signed graphs and their Laplacian was initiated by Zaslavsky in \cite{Zaslavsky} and also studied in \cite{MR666857}, \cite{MR4641710}, \cite{FundBapat}. \begin{example} \label{table:signed_toy_ss} We take $L$ to be the (reduced) Laplacian of the following signed graph and $M$ to be the (reduced) Laplacian of the underlying unsigned graph. The Laplacian of the signed graph is simply obtained from the Laplacian of the underlying graph, by changing the signs of entries corresponding to negative edges. \begin{figure}[ht] \centering \begin{minipage}{0.3\linewidth} \begin{center} \begin{tikzpicture}[scale=2] \input{tikz/kyle-signed.tex} \end{tikzpicture} \end{center} \end{minipage} \begin{minipage}{0.6\linewidth} $$M = \begin{pmatrix} 3 & -1 & -1 \\ -1 & 2 & -1 \\ -1 & -1 & 3 \end{pmatrix} \hspace{5mm} L = \begin{pmatrix} 3 & 1 & -1 \\ 1 & 2 & -1 \\ -1 & -1 & 3 \end{pmatrix}$$ \end{minipage} \label{fig:Signed-Kyle-S+} \end{figure} \begin{center} Configurations in $S^+$ \renewcommand{\arraystretch}{1.2} \begin{tabular}{ c | c } \label{table:toy_ss} Superstables & Criticals \\ \hline $(0, 0, 0) $ & $(6, 4, 2) $ \\ $(1, 1, 0) $ & $(7, 5, 2) $ \\ $(4, 3, 2) $ & $(8, 6, 0) $ \\ $(5, 4, 2) $ & $(9, 7, 0) $ \\ \end{tabular} \begin{tabular}{c | c} Superstables & Criticals \\\hline $(2, 2, 0) $ & $(8, 6, 2) $ \\ $(3, 3, 0) $ & $(9, 7, 2) $ \\ $(3, 2, 0) $ & $(6, 4, 1) $ \\ $(4, 3, 0) $ & $(7, 5, 1) $ \\ \end{tabular} \begin{tabular}{c | c} Superstables & Criticals \\\hline $(5, 4, 0) $ & $(8, 6, 1) $ \\ $(6, 5, 0) $ & $(9, 7, 1) $ \\ $(6, 4, 0) $ & $(6, 5, 2) $ \\ $(7, 5, 0) $ & $(7, 6, 2)$ \end{tabular} \end{center} \end{example} Notice that in the table of superstable and critical configurations of the chip-firing pair, the coordinate-wise maximal critical configuration is $(9,7,2)$. However if we take the superstable configuration $(1,1,0)$, the vector we get by applying the traditional duality map $(9,7,2) - (1,1,0) = (8,6,2)$ is not a critical configuration. Even worse, there are many cases where $c_{max}$, the critical configuration that has coordinate-wise maximal entries does not even exist. \begin{example} \label{ex:nocmax} Consider the $(L,M)$-pair coming from the signed graph below. The underlying graph is $C_6$, the cycle on six vertices. \begin{figure}[ht] \centering \renewcommand{\arraystretch}{1.2} \begin{minipage}{.45\linewidth} \centering \begin{tikzpicture}[scale=.85] \input{tikz/c6-signed} \end{tikzpicture} \end{minipage} \begin{minipage}{.45\linewidth} \centering \begin{tabular}{ c } Critical configurations \\ \hline (9, 15, 17, 15, 9) \\ (12, 20, 23, 21, 13) \\ (13, 21, 23, 20, 12) \\ (7, 11, 12, 11, 7) \\ (10, 16, 18, 17, 11) \\ (11, 17, 18, 16, 10) \\ \end{tabular} \end{minipage} \label{fig:Cycle-5} \end{figure} As can be checked from the table of critical configurations above, there is no critical configuration that is the maximal in all coordinates. \end{example} \subsection{Finding the superstable/critical configurations of chip firing pairs.} In this subsection, we go over an alternative way to find the ($z$)-superstable and critical configurations of $(L,M)$ chip-firing pairs, developed in \cite{cho2024}. Let $\sstab(M)$ denote the set of superstable configurations of an $M$-matrix $M$ and let $\crit(M)$ denote the set of critical configurations. However, beware we are not going to be using $\sstab(L,M)$ to denote the set of superstable configurations of $(L,M)$ and same for $\crit(L,M)$. It turns out that for configurations in $S^{+}$, it is important to look at their preimages in $R^{+}$. Given any vector $\vec{f}$, we use $\floor{\vec{f}}$ to denote the vector obtained from $f$ by taking the floor at every coordinate. \begin{theorem}[{\cite[Theorem 3.2]{cho2024}}] \label{thm:22floor} Given an $(L, M)$ pair, a configuration $\vec c \in S^+$ is superstable/critical if and only if $\floor{ML^{-1} \vec c}$ is a superstable/critical configuration of $M$. \end{theorem} For example, we look at our running example coming from a signed graph. \begin{example} \label{ex:usualbad} Consider the signed graph studied in \cref{table:signed_toy_ss}. The table lists all superstable and critical configurations in $S^{+}$, their preimage in $R^{+}$, and the floor of the preimage. We can notice that the floor of the preimages are the superstable and critical configurations of the underlying graph we saw in \cref{ex:unsignedkyle} (however, not all superstable/critical configurations of the underlying graph are used). \begin{center} \begin{tikzpicture}[scale=2] \input{tikz/kyle-signed} \end{tikzpicture} \end{center} \begin{center}\label{table:toy_stats} \renewcommand{\arraystretch}{1.2} \begin{tabular}{ c | c | c | c | c | c} $\LM(\sstab)$ & $\sstab$ & $\floor{\sstab}$ & $\LM(\crit)$& $\crit$ & $\floor{\crit}$\\\hline $ (0, 0, 0) $ & $(0, 0, 0) $ & $ (0, 0, 0) $ & $(6,4,2) $& $ (2, 0, 2) $ & $ (2, 0, 2) $ \\ $ (1, 1, 0) $ & $(0, \nicefrac{1}{2}, 0) $ & $ (0, 0, 0) $ & $(7,5,2) $& $ (2, \nicefrac{1}{2}, 2) $ & $ (2, 0, 2) $ \\ $ (4, 3, 2) $ & $(\nicefrac{2}{3}, \nicefrac{1}{3}, 2) $ & $ (0, 0, 2) $ & $(8,6,0) $& $ (\nicefrac{8}{3}, \nicefrac{4}{3}, 0) $ & $ (2, 1, 0) $ \\ $ (5, 4, 2) $ & $(\nicefrac{2}{3}, \nicefrac{5}{6}, 2) $ & $ (0, 0, 2) $ & $(9,7,0) $& $ (\nicefrac{8}{3}, \nicefrac{11}{6}, 0) $ & $ (2, 1, 0) $ \\ $ (2, 2, 0) $ & $(0, 1, 0) $ & $ (0, 1, 0) $ & $(8,6,2) $& $ (2, 1, 2) $ & $ (2, 1, 2) $ \\ $ (3, 3, 0) $ & $(0, \nicefrac{3}{2}, 0) $ & $ (0, 1, 0) $ & $(9,7,2) $& $ (2, \nicefrac{3}{2}, 2) $ & $ (2, 1, 2) $ \\ $ (3, 2, 0) $ & $(\nicefrac{4}{3}, \nicefrac{1}{6}, 0) $ & $ (1, 0, 0) $ & $(6,4,1) $& $ (\nicefrac{7}{3}, \nicefrac{1}{6}, 1) $ & $ (2, 0, 1) $ \\ $ (4, 3, 0) $ & $(\nicefrac{4}{3}, \nicefrac{2}{3}, 0) $ & $ (1, 0, 0) $ & $(7,5,1) $& $ (\nicefrac{7}{3}, \nicefrac{2}{3}, 1) $ & $ (2, 0, 1) $ \\ $ (5, 4, 0) $ & $(\nicefrac{4}{3}, \nicefrac{7}{6}, 0) $ & $ (1, 1, 0) $ & $(8,6,1) $& $ (\nicefrac{7}{3}, \nicefrac{7}{6}, 1) $ & $ (2, 1, 1) $ \\ $ (6, 5, 0) $ & $(\nicefrac{4}{3}, \nicefrac{5}{3}, 0) $ & $ (1, 1, 0) $ & $(9,7,1) $& $ (\nicefrac{7}{3}, \nicefrac{5}{3}, 1) $ & $ (2, 1, 1) $ \\ $ (6, 4, 0) $ & $(\nicefrac{8}{3}, \nicefrac{1}{3}, 0) $ & $ (2, 0, 0) $ & $(6,5,2) $& $ (\nicefrac{2}{3}, \nicefrac{4}{3}, 2) $ & $ (0, 1, 2) $ \\ $ (7, 5, 0) $ & $(\nicefrac{8}{3}, \nicefrac{5}{6}, 0) $ & $ (2, 0, 0) $ & $(7,6,2) $& $ (\nicefrac{2}{3}, \nicefrac{11}{6}, 2) $ & $ (0, 1, 2) $ \end{tabular} \end{center} \end{example} Thanks to \cref{thm:22floor}, it is much more convenient to deal with the preimages of configurations, especially when trying to check if it is superstable or critical. Given a superstable configuration in $S^{+}$, we are going to denote its preimage in $R^{+}$ as \newword{superstable preimage} and for a critical configuration in $S^{+}$, we are going to denote its preimage in $R^{+}$ as \newword{critical preimage}. We are also going to use $\sstab(L,M)$ to denote the set of superstable preimages of a $(L,M)$ chip-firing pair, and use $\crit(L,M)$ to denote the set of critical preimages. \begin{scope}[every node/.style={circle,draw}] \node (1) at (0,1) {$1$}; \node (2) at (1,1) {$2$}; \node (3) at (1,0) {$3$}; \node (q) at (0,0) {$q$}; \end{scope} \begin{scope}[every node/.style={draw=black}, every edge/.style={draw=black}] \path (1) edge (3); \path (1) edge (q); \path (3) edge (2); \path (3) edge (q); \path (1) edge (2); \end{scope} \begin{scope}[every node/.style={circle,draw}] \node (1) at (0,1) {$1$}; \node (2) at (1,1) {$2$}; \node (3) at (1,0) {$3$}; \node (q) at (0,0) {$q$}; \end{scope} \begin{scope}[every node/.style={draw=black}, every edge/.style={draw=black}] \path (1) edge (3); \path (1) edge (q); \path (3) edge (2); \path (3) edge (q); \end{scope} \begin{scope}[every node/.style={text=red,font=\bfseries}, every edge/.style={draw=red}] \path (1) edge node[above] {$-$} (2); \end{scope} \def\r{2cm} \def\a{360/6} \begin{scope}[every node/.style={circle,draw}] \node (1) at (0:\r) {$1$}; \node (2) at (\a:\r) {$2$}; \node (3) at (2*\a:\r) {$3$}; \node (4) at (3*\a:\r) {$4$}; \node (5) at (4*\a:\r) {$5$}; \node (q) at (5*\a:\r) {$q$}; \end{scope} \begin{scope}[every edge/.style={draw=black}] \path (1) edge (2); \path (2) edge (3); \path (3) edge (4); \path (4) edge (5); \path (5) edge (q); \path (q) edge (1); \end{scope} \begin{scope}[every node/.style={text=red,font=\bfseries}, every edge/.style={draw=red}] \path (1) edge node[right] {$-$} (2); \path (2) edge node[above] {$-$} (3); \path (3) edge node[left] {$-$} (4); \path (4) edge node[left] {$-$} (5); \end{scope} \begin{scope}[every node/.style={circle,draw}] \node (1) at (0,1) {$1$}; \node (2) at (1,1) {$2$}; \node (3) at (1,0) {$3$}; \node (q) at (0,0) {$q$}; \end{scope} \begin{scope}[every node/.style={draw=black}, every edge/.style={draw=black}] \path (1) edge (q); \path (3) edge (q); \end{scope} \begin{scope}[every node/.style={text=red,font=\bfseries}, every edge/.style={draw=red}] \path (1) edge node[above] {$-$} (3); \path (3) edge node[right] {$-$} (2); \path (1) edge node[above] {$-$} (2); \end{scope} \begin{scope}[every node/.style={circle,draw}] \node (1) at (0,1) {$1$}; \node (2) at (1,1) {$2$}; \node (3) at (1,0) {$3$}; \node (q) at (0,0) {$q$}; \end{scope} \begin{scope}[every node/.style={draw=black}, every edge/.style={draw=black}] \path (2) edge (q); \path (1) edge (q); \path (3) edge (2); \path (3) edge (q); \path (1) edge (2); \end{scope} \documentclass[11pt,reqno]{amsart} \usepackage{graphicx} \usepackage{xcolor} \usepackage{stackengine} \begin{document} :D This is what abuse of notation actually is \Huge \scalebox{3}[1]{$\mathcal{o}$} \scalebox{1}[3]{$\mathcal{o}$} \scalebox{5}[1]{$\mathcal{o}$} \scalebox{1}[5]{$\mathcal{o}$} \scalebox{5}[1]{$\mathcal{o}$} \scalebox{1}[5]{$\mathcal{o}$} \scalebox{10}[1]{$\mathcal{o}$} \scalebox{1}[10]{$\mathcal{o}$} \scalebox{10}[0.5]{$\mathcal{o}$} \scalebox{1}[5]{$\mathcal{o}$} \scalebox{10}[0.5]{$\mathcal{o}$} \scalebox{1}[5]{$\mathcal{o}$} \scalebox{10}[0.5]{$\mathcal{o}$} \stackinset{c}{}{c}{}{\rotatebox{45}{\scalebox{25}[0.5]{$\mathcal{o}$}}}{\rotatebox{-45}{\scalebox{25}[0.5]{$\mathcal{o}$}}} \color{black!15}$\mathcal{o}$\color{black!30}$\mathcal{o}$\color{black!45}$\mathcal{o}$\color{black!60}$\mathcal{o}$\color{black!75}$\mathcal{o}$\color{black!90}$\mathcal{o}$\color{black!75}$\mathcal{o}$\color{black!60}$\mathcal{o}$\color{black!45}$\mathcal{o}$\color{black!30}$\mathcal{o}$\color{black!15}$\mathcal{o}$\color{black!30}$\mathcal{o}$\color{black!45}$\mathcal{o}$\color{black!60}$\mathcal{o}$\color{black!75}$\mathcal{o}$\color{black!90}$\mathcal{o}$\color{black!75}$\mathcal{o}$\color{black!60}$\mathcal{o}$\color{black!45}$\mathcal{o}$\color{black!30}$\mathcal{o}$\color{black!15}$\mathcal{o}$\color{black!30}$\mathcal{o}$\color{black!45}$\mathcal{o}$\color{black!60}$\mathcal{o}$\color{black!75}$\mathcal{o}$\color{black!90}$\mathcal{o}$\color{black!75}$\mathcal{o}$\color{black!60}$\mathcal{o}$\color{black!45}$\mathcal{o}$\color{black!30}$\mathcal{o}$\color{black!15}$\mathcal{o}$\color{black!30}$\mathcal{o}$\color{black!45}$\mathcal{o}$\color{black!60}$\mathcal{o}$\color{black!75}$\mathcal{o}$\color{black!90}$\mathcal{o}$\color{black!75}$\mathcal{o}$\color{black!60}$\mathcal{o}$\color{black!45}$\mathcal{o}$\color{black!30}$\mathcal{o}$\color{black!15}$\mathcal{o}$\color{black!30}$\mathcal{o}$\color{black!45}$\mathcal{o}$\color{black!60}$\mathcal{o}$\color{black!75}$\mathcal{o}$\color{black!90}$\mathcal{o}$\color{black!75}$\mathcal{o}$\color{black!60}$\mathcal{o}$\color{black!45}$\mathcal{o}$\color{black!30}$\mathcal{o}$\color{black!15}$\mathcal{o}$\color{black!30}$\mathcal{o}$\color{black!45}$\mathcal{o}$\color{black!60}$\mathcal{o}$\color{black!75}$\mathcal{o}$\color{black!90}$\mathcal{o}$\color{black!75}$\mathcal{o}$\color{black!60}$\mathcal{o}$\color{black!45}$\mathcal{o}$\color{black!30}$\mathcal{o}$ \end{document}
2412.02787v1
http://arxiv.org/abs/2412.02787v1
The Ehrhart series of alcoved polytopes
\documentclass[11pt,reqno]{amsart} \usepackage{amsmath, amssymb, amsthm, wasysym,todonotes, graphicx,manfnt} \usepackage{stmaryrd} \usepackage{mathrsfs,mathtools} \usepackage[margin=3cm]{geometry} \usepackage{color} \usepackage{hyperref} \hypersetup{ colorlinks, citecolor=black, filecolor=black, linkcolor=black, urlcolor=black } \usepackage[capitalize,nameinlink,noabbrev,nosort]{cleveref} \usepackage[maxbibnames=99,sorting=nyt,giveninits,maxnames=10,backend=bibtex,style=alphabetic,block=space,url=false]{biblatex} \addbibresource{main.bib} \usepackage{enumerate} \usepackage{caption} \usepackage{subcaption} \usepackage{url} \usepackage{floatrow} \usepackage{chngcntr} \usepackage{manfnt} \usepackage{esvect} \usepackage{verbatim} \usepackage{comment} \usepackage{pgfplots} \pgfplotsset{compat=1.18} \usepackage{tikz} \usetikzlibrary{shapes.geometric} \usetikzlibrary{math} \usetikzlibrary{calc,arrows} \usepackage{tikz-3dplot} \newtheorem{theorem}{Theorem}[section] \newtheorem{lemma}[theorem]{Lemma} \newtheorem{observation}[theorem]{Observations} \newtheorem{proposition}[theorem]{Proposition} \newtheorem{corollary}[theorem]{Corollary} \newtheorem{conjecture}[theorem]{Conjecture} \newtheorem{question}[theorem]{Question} \theoremstyle{definition} \newtheorem{definition}[theorem]{Definition} \newtheorem{example}[theorem]{Example} \newtheorem{remark}[theorem]{Remark} \newtheorem*{theorem*}{Theorem} \newcommand \acknowledgements{\paragraph{\textbf{Acknowledgements}}} \newcommand\inner[2]{\langle #1, #2 \rangle} \newcommand{\N}{\mathbb{N}} \newcommand{\Z}{\mathbb{Z}} \newcommand{\Q}{\mathbb{Q}} \newcommand{\R}{\mathbb{R}} \newcommand{\C}{\mathbb{C}} \newcommand{\F}{\mathcal{F}} \newcommand{\D}{\mathcal{D}} \newcommand{\precdot}{\prec\cdot ~} \newcommand{\cube}{\text{\mancube}} \DeclareMathOperator{\conv}{conv} \DeclareMathOperator{\vertex}{vert} \DeclareMathOperator{\des}{des} \DeclareMathOperator{\GL}{GL} \DeclareMathOperator{\EE}{EE} \DeclareMathOperator{\volu}{vol} \DeclareMathOperator{\cover}{cover} \newcommand{\ts}{\textsuperscript} \DeclareMathOperator{\rank}{rank} \DeclareMathOperator{\trace}{trace} \newcommand{\hv}{\mathcal{H}_{\mathcal{V}}} \newcommand{\shift}{\overset{\mathrm{shift}}{=\joinrel=}} \renewcommand{\P}{\mathcal{P}} \newcommand{\YJ}[1]{{$\spadesuit$ \textcolor{blue}{YJ: #1}}} \newcommand{\id}{\Delta_0} \newcommand{\fan}{\text{Fan}} \newcommand{\dist}{\text{dist}} \DeclareMathOperator{\wt}{wt} \DeclareMathOperator{\Ehr}{Ehr} \newcommand{\aff}{\mathrm{aff}} \DeclarePairedDelimiter{\bbracket}{\llbracket}{\rrbracket} \DeclarePairedDelimiter{\floor}{\lfloor}{\rfloor} \title{The Ehrhart series of alcoved polytopes} \author{Elisabeth Bullock} \address{Massachusetts Institute of Technology} \email{[email protected]} \author{Yuhan Jiang} \address{Harvard University} \email{[email protected]} \date{\today} \begin{document} \begin{abstract} Alcoved polytopes are convex polytopes which are the closure of a union of alcoves in an affine Coxeter arrangement. They are rational polytopes, and therefore have Ehrhart quasipolynomials. We describe a method for computing the generating function of the Ehrhart quasipolynomial, or Ehrhart series, of any alcoved polytope via a particular shelling order of its alcoves. We also conjecture a connection between Early's decorated ordered set partitions and this shelling order for the hypersimplex $\Delta_{2,n}$. \end{abstract} \maketitle \setcounter{tocdepth}{1} \section{Introduction} Let $\Phi \subset V$ be an irreducible \emph{crystallographic root system} and $W$ be the corresponding Weyl group. Associated to $\Phi$ is an infinite hyperplane arrangement known as the affine Coxeter arrangement. The connected components of this hyperplane arrangement are simplices called \emph{alcoves}. Lam and Postnikov defined a \emph{proper alcoved polytope} to be the closure of a union of alcoves. In the special situation of the root system $\Phi = A_n$, examples of alcoved polytopes include \emph{hypersimplices} and \emph{positroid polytopes}. For any root system, Fomin and Zelevinsky's \emph{generalized associahedra} are examples of polytopes which can be realized as alcoved polytopes \cite{fz03,chapoton}. Alcoved polytopes are rational polytopes, i.e., their vertices have rational coordinates. Let $P \subset \R^n$ be a rational convex polytope. Ehrhart \cite{ehrhart} showed that the number of lattice points in the $t$-dilate of $P$ is a \emph{quasipolynomial} in $t$. A quasipolynomial with period $d$ is a function $p: \Z \to \Z$ such that there exist periodic functions $p_i: \Z \to \Z$ with period $d$ so $p(z) = \sum_{i=0}^n p_i(z) z^i$. We call $E(P,t) = \#(t\cdot P) \cap \Z^n$ the \emph{Ehrhart quasipolynomial} of $P$. The generating series $\sum_{t=0}^\infty E(P,t) z^t$ is a rational function in $z$. While the Ehrhart theory of lattice polytopes has been widely studied, the Ehrhart theory of rational polytopes is a younger subject \cite{beck22,BBKV13,linke11}. Ehrhart theory extends naturally to polytopes with some facets removed. The Ehrhart series are \emph{additive}, in the sense that if two half-open polytopes are disjoint, then the Ehrhart series of their union is equal to the sum of their Ehrhart series. Our main result stated imprecisely is the following: \begin{theorem*}\label{thm:main-imprecise} Fix an irreducible crystallographic root system $\Phi\subset V$, where $\dim(V)=n$. Let $P$ be an alcoved polytope and let $\Gamma_P=(V,E)$ be the dual graph to the alcove triangulation of $P$. Pick some $v_0\in V$ and orient the edges of $\Gamma_P$ so that for all $\{u,w\}\in E$, $u\to w$ if and only if $u$ appears before $v$ in the breadth first search algorithm starting at $v_0$. There exists a weighting of the edges $E$ and parameters $\ell_1,\cdots,\ell_n$ depending only on $\Phi$ such that the Ehrhart series of $P$ is equal to $$\Ehr(P,z) = \frac{\sum_{w \in V} z^{\wt(w)}}{\prod_{i=0}^n (1-z^{\ell_i})}$$ where $\wt(w) = \sum_{u \to w} \wt((u,w))$ is the sum of the weights of the ingoing edges to $w$. \end{theorem*} We will prove our main result using the additivity of Ehrhart series. We will decompose each alcoved polytopes into disjoint union of half-open alcoves, and then add up all their Ehrhart series. \begin{figure}[H] \centering \begin{tikzpicture}[scale=1.5] \coordinate (A) at (0,0); \coordinate (B) at (2,0); \coordinate (C) at (1,1); \draw[thick] (A) -- (B) -- (C) -- cycle; \draw node [left, below] at (A) {(0,0)}; \draw node [right, below] at (B) {(1,0)}; \draw node [above] at (C) {($\frac{1}{2}, \frac{1}{2}$)}; \end{tikzpicture} \hspace{3cm} \begin{tikzpicture}[scale=1.5] \coordinate (A) at (0,0); \coordinate (B) at (2,0); \coordinate (C) at (2,2); \coordinate (D) at (3,1); \coordinate (E) at (1,1); \draw[thick] (A) -- (B) -- (D) -- (C) -- cycle; \draw[thick] (E) -- (B); \draw[thick] (B) -- (C); \coordinate (1) at (1,1/2); \coordinate (2) at (3/2,1); \coordinate (3) at (5/2,1); \draw node at (1) {$\bullet$}; \draw node at (2) {$\bullet$}; \draw node at (3) {$\bullet$}; \draw[thick] (1) -- (2) node [midway, above] {1}; \draw[thick] (2) -- (3) node [midway, above] {2}; \end{tikzpicture} \caption{On the left, we have the fundamental alcove of type $B_2$. On the right, we have an alcoved polytope of type $B_2$ and the graph of its alcoved triangulation. The Ehrhart series of the square is $\frac{1+z+z^2}{(1-z)^2 (1-z^2)}$.} \label{fig:B22} \end{figure} \begin{figure}[H] \centering \begin{tikzpicture} \coordinate (A) at (0,0); \coordinate (B) at ($(0,{-2*tan(60)})$); \coordinate (C) at ($(B) - (2,0)$); \draw[thick] (A) -- (B) -- (C) --cycle; \draw node [above] at (A) {$(0,0,0)$}; \draw node [right] at (B) {$(\frac{1}{2},0,-\frac{1}{2})$}; \draw node [left] at (C) {$(\frac{2}{3}, -\frac{1}{3}, -\frac{1}{3})$}; \end{tikzpicture} \hspace{3cm} \begin{tikzpicture} \coordinate (A) at (0,0); \coordinate (B) at (2,0); \coordinate (C) at ($(A) - (0,{2*tan(60)})$); \coordinate (D) at (-2,0); \coordinate (E) at ($(C) - (2,0)$); \draw[thick] (A) -- (B) -- (C) -- (E) -- (D) --cycle; \draw[thick] (E) -- (A) -- (C); \coordinate (F) at ($ (A)!.333!(B)!.333!(C) $); \coordinate (G) at ($ (A)!.333!(E)!.333!(C) $); \coordinate (H) at ($ (A)!.333!(D)!.333!(E) $); \draw[thick] (F) -- (G) node [midway, above] {3}; \draw[thick] (G) -- (H) node [midway, above] {2}; \draw node at (F) {$\bullet$}; \draw node at (G) {$\bullet$}; \draw node at (H) {$\bullet$}; \end{tikzpicture} \caption{On the left, we have the fundamental alcove of type $G_2$. On the right, we have an alcoved polytope of type $G_2$. The Ehrhart series of the trapezoid is $\frac{1+z^2+z^3}{(1-z)(1-z^2)(1-z^3)}$.} \label{fig:G22} \end{figure} After proving this result, we conjecture a relationship between this formula and another formula for the $h^*$-polynomial of the second hypersimplex $\Delta_{2,n}=\{(x_1,\dots,x_n)\in[0,1]^n~|~\sum_{i=1}^nx_i=2\}$ which is given in terms of combinatorial objects called decorated ordered set partitions (DOSPs). The only known proofs of the latter formula rely on algebraic manipulation and enumerative combinatorics; our conjecture is aimed towards providing a more geometric reason for why DOSPs appear in the $h^*$-polynomial of the hypersimplex. \subsection{Organization} In \cref{sec:prelim}, we cover relevant background materials, including root systems, alcoved polytopes, and Coxeter groups. In \cref{sec:ehr}, we review some general Ehrhart theory of rational polytopes and prove a few lemmas that will be used to show our main result. In \cref{sec:shell}, we review Coxeter complexes and the notion of a convex subset of a Coxeter group. In this section, we also describe a shelling order of the Coxeter complex given by \cite{bjorner} and prove a corollary about shelling subcomplexes of the Coxeter complex (\cref{cor:cox}) which will be used to show our main result. We precisely state and prove our main result (\cref{thm:main}) in \cref{sec:proof}. In \cref{sec:2n}, we recall a formula for the $h^*$-polynomial of the hypersimplex $\Delta_{k,n}$ in terms of hypersimplicial decorated ordered set partitions \cite{kim2020combinatorial} and conjecture a connection with the formula yielded by our main theorem in the case $k=2$. \vspace{.5cm} \acknowledgements{We thank Alex Postnikov for suggesting this topic. We thank Lauren Williams for the helpful discussions and comments on the manuscript. We thank Nick Early for helpful discussions.} \section{Preliminaries}\label{sec:prelim} In this section, we recall the relevant background for \emph{root systems} which will be used to define \emph{alcoved polytopes}. We recall notations from \emph{Coxeter groups} which will be used to describe our shellings of the Coxeter complexes related to the alcove triangulation of an alcoved polytope. We follow the conventions in \cite{humphreys} and \cite{alcove2}. \subsection{Root systems}\label{sec:rootsys} Let $V$ be a real Euclidean space of rank $n$ with nondegenerate symmetric inner product $(\cdot,\cdot)$. Let $\Phi \subset V$ be an irreducible \emph{crystallographic root system} with a choise of basis of \emph{simple roots} $\alpha_1, \dots, \alpha_n$. Let $\Phi^+ \subset \Phi$ be the corresponding set of \emph{positive roots}. The \emph{coweight lattice} $\Lambda^\vee$ is the integer lattice defined by $\Lambda^\vee = \Lambda^\vee(\Phi) = \{ \lambda \in V \mid (\lambda, \alpha) \in \Z, \text{ for all } \alpha \in \Phi\}$. Let $\omega_1, \dots, \omega_n \subset V$ be the basis dual to the basis of simple roots, i.e., $(\omega_i, \alpha_j) = \delta_{ij}$. The $\omega_i$ are called the \emph{fundamental coweights}. They generate the coweight lattice $\Lambda^\vee$. Let $\rho = \omega_1 + \cdots + \omega_n$. The \emph{height} of a root $\alpha$ is the number $(\rho, \alpha)$ of simple roots that add up to $\alpha$. Since we assumed that $\Phi$ is irreducible, there exists a unique \emph{highest root} $\theta \in \Phi^+$ of maximal possible height. For convenience we set $\alpha_0 = -\theta$. Let $a_0 = 1$ and $a_1, \dots, a_n$ be the positivie integers given by $a_i = (\omega_i, \theta)$. The \emph{dual Coxeter number} is defined as $h^\vee = (\rho, \theta) + 1 = a_0 + a_1 + \cdots + a_n$. \subsection{Alcoved polytopes}\label{sec:alcove} Let $\Phi\subset V$ be a crystallographic root system. The set of affine hyperplanes of the form $H_{\alpha,k} = \{\lambda \in V \mid (\lambda, \alpha) = k \}$, where $\alpha\in\Phi^+,k\in\mathbb{Z}$, divides $V$ into \emph{open alcoves}: \begin{definition} An (open) \emph{alcove} is the set $$ A = \{ \lambda \in V \mid m_\alpha < (\lambda, \alpha) < m_\alpha+1, \text{ for } \alpha \in \Phi^+\} $$ where $m_\alpha$ is a collection of integers associated with the alcove $A$. A \emph{closed alcove} is the closure of an alcove. \end{definition} \begin{definition}\label{def:fundamental} The \emph{fundamental alcove} is the simplex given by \begin{align*} A_\circ &= \{ \lambda \in V \mid 0 < (\lambda, \alpha) < 1, \text{ for } \alpha \in \Phi^+\} \\ &= \text{Convex Hull of the points } 0, \omega_1/a_1, \dots, \omega_r/a_r. \end{align*} \end{definition} The \emph{affine Weyl group} $W_{\aff}$ associated with the root system $\Phi$ is generated by the reflections $s_{\alpha,k}: V \to V, \alpha \in \Phi, k \in \Z$, with respect to the affine hyperplanes $H_{\alpha,k}$, and acts simply transitively on the collection of all alcoves \cite{humphreys}. In particular, the closure of each alcove has the same Ehrhart series, and we can use alcoves as the building blocks for a family of polytopes with natural triangulations in which all top-dimensional simplices have equal volume: \begin{definition} An \textit{alcoved polytope} is a polytope which is the union of some collection of faces of alcoves. A \textit{proper alcoved polytope} is a polytope which is a union of alcoves, equivalently a top-dimensional alcoved polytope. \end{definition} In other words, every alcoved polytope is of the form $$ P = \{ \lambda \in V \mid k_\alpha \leq (\lambda, \alpha) \leq K_\alpha, \text{ for } \alpha \in \Phi^+ \}, $$ where $k_\alpha, K_\alpha$ are two collections of integers indexd by the positive roots $\alpha \in \Phi^+$. \begin{definition}\label{def:graphlabel} Let $P$ be an alcoved polytope. We associate a graph $\Gamma_P = (V,E)$ with labeled edges to the alcoved triangulation of $P$. We will abuse notation and also use $\Gamma_P$ to denote the simplicial complex of the alcove triangulation of $P$. The vertex set $V$ consists of closed alcoves in $P$, and the edge set $E$ consists of $(A, A')$ if $A$ and $A'$ share a common facet. Let $(A, A')$ be an edge of $G$, and let $F = A \cap A'$ be the facet it represents. Then $F$ can be transformed to a facet $F_\circ$ of the fundamental alcove $A_\circ$ under the action of the affine Weyl group. Let $\omega_i/a_i$ be the vertex of $A_\circ$ that does not belong to $F_\circ$. Then $(A,A')$ has \emph{weight} $\ell_i$, denoted $\wt((A,A')) = \ell_i$, in which $\ell_i$ is the least common multiple of the denominators in $\omega_i/a_i\in\mathbb{Q}^n$ for $i=1,\dots,n$ and $\ell_0 = 1$. \end{definition} \subsection{Coxeter Groups} For a root system $\Phi$, the affine Weyl group $W_\aff$ is an example of a Coxeter group: \begin{definition} Let $S$ be a set. A matrix $m: S \times S \to \{1,2,\dots,\infty\}$ is called a \emph{Coxeter matrix} if it satisfies \begin{align*} m(s,s') &= m(s',s) \\ m(s,s') = 1 & \iff s = s'. \end{align*} A Coxeter matrix $m$ determines a group $W$ with the presentation $$ \langle s \in S \mid (ss')^{m(s,s')} = e \text{ for all } m(s,s') \neq \infty \rangle, $$ where $e$ is the identity element. The pair $(W,S)$ is called a \emph{Coxeter system} and the group $W$ is the \emph{Coxeter group}. \end{definition} \begin{definition} Let $(W, S)$ be a Coxeter system. Each element $w \in W$ can be written as a product of generators $w = s_1 s_2 \cdots s_k$ for $s_i \in S$. If $k$ is minimal among all such expressions for $w$, then $k$ is called the \emph{length} of $w$ (written $\ell(w) = k$) and the word $s_1 s_2 \cdots s_k$ is called a \emph{reduced word} for $w$. \end{definition} \begin{definition} Let $(W, S)$ be a Coxeter system, and let $u, v \in W$. Define the \emph{right weak order} as the partial order $\leq$ on $W$ such that $u \leq w$ if and only if $w = u s_1 s_2 \cdots s_k$ for some $s_i \in S$ such that $\ell(u s_1 s_2 \cdots s_i) = \ell(u) + i$ for all $0 \leq i \leq k$. \end{definition} \begin{definition} Let $W$ be a Coxeter group. For $w \in W$, define the \emph{descent set} $\D(w) = \{s \in S \mid w s < w\}$. For $J \subseteq S$, let $W_J$ be the subgroup of $W$ generated by the set $J$, and let $W^J = \{ w \in W \mid w s > w \text{ for all } s \in J\}$ be a system of distinct coset representatives modulo $W_J$. \end{definition} \section{Ehrhart series of half-open rational simplices}\label{sec:ehr} A key ingredient of the proof of our main theorem is to write an alcoved polytope $P$ as a disjoint union of half-open simplices, or simplices with some facets removed. Since the number of lattice points in $P_1\sqcup P_2$ is the sum of lattice points in $P_1$ and $P_2$, it suffices to look at the Ehrhart series of these \emph{half-open} simplices. If we remove no facets from an alcove, we may use the following theorem to write its Ehrhart series: \begin{theorem}[Theorem 1.3, \cite{stanley80}] \label{rat-simp} Suppose $S$ is a $k$-simplex in $\R^m$ with rational vertices $\beta_0, \dots, \beta_k$. Let $\ell_i$ be the least positive integer $t$ for which $t\beta_i \in \Z^m$. Consider the $(k+1) \times (m+1)$-matrix whose rows are the vectors $(\ell_i \beta_i, \ell_i)$. If the greatest common divisor of all $(k+1)\times(k+1)$ minors of the matrix is equal to 1, then the Ehrhart series of the simplex $S$ is equal to $$ \Ehr(S, z) = \frac{1}{\prod_{i=0}^k (1-z^{\ell_i})}. $$ \end{theorem} The above theorem serves as a base case in the proof of the following more general formula for half-open simplices: \begin{lemma}\label{lem:half-opensimplex} Suppose $S$ is a $k$-simplex in $\R^m$ with rational vertices $\beta_0, \dots, \beta_k$. Let $\ell_i$ be the least positive integer $t$ for which $t\beta_i \in \Z^m$. Let $F_i$ be the facet of $S$ that does not contain the vertex $\beta_i$. Let $\mathcal{F} \subseteq \{0, \dots, k\}$ label a subset of facets of $S$. Then the Ehrhart series of $S$ with facets $\{F_i \mid i \in \mathcal{F}\}$ removed is $$ \Ehr(S \setminus \bigcup_{i \in \mathcal{F}} F_i, z) = \frac{ \prod_{i \in \mathcal{F}} z^{\ell_i}}{\prod_{i=0}^k (1-z^{\ell_i})}. $$ \end{lemma} \begin{proof} Without loss of generality assume $\mathcal{F}=\{0,\ldots,m\}$. We induct on $m$. First, if we remove no facets, we are done by Theorem \ref{rat-simp}. By inclusion-exclusion and our inductive hypothesis, \begin{align*} \Ehr\left(S\setminus\bigcup_{i \in \mathcal{F}} F_i, z\right) &=\Ehr(S,z)+\sum_{\mathcal{G}\subseteq\mathcal{F}}(-1)^{|\mathcal{G}|}\Ehr\left(\bigcap_{F\in\mathcal{G}}F,z\right)\\ &=\frac{1}{\prod_{i=0}^k(1-z^{\ell_i})}+\sum_{\mathcal{G}\subseteq\mathcal{F}}\frac{(-1)^{|\mathcal{G}|}}{\prod_{i\in V(\mathcal{G})}(1-z^{\ell_i})} \end{align*} where $V(\mathcal{G})$ is the set of indices of vertices $\bigcap_{F\in\mathcal{G}}F$. We have $V(\mathcal{G})=\{0,\ldots,k\}\setminus\mathcal{G}$. Clearing denominators, \begin{align*} \Ehr\left(S\setminus\bigcup_{i \in \mathcal{F}} F_i, z\right) &=\frac{1}{\prod_{i=0}^k(1-z^{\ell_i})}\left[\sum_{\mathcal{G}\subseteq\mathcal{F}}(-1)^{|\mathcal{G}|}\prod_{i\in\mathcal{G}}(1-z^{\ell_i})\right]. \end{align*} Using inclusion-exclusion again, the last sum is equal to $\prod_{i\in\mathcal{F}}z^{\ell_i}$. \end{proof} \section{A shelling order}\label{sec:shell} The goal of this section is to build up to a concrete decomposition of any alcoved polytope into half-open alcoves. This is done via a \emph{shelling order}. In particular, we relate the alcove structure to a simplicial complex called the coxeter complex and use existing results about the latter to define a shelling order. We begin by recalling the notion of Coxeter complexes, following \cite{bjornerbrenti}. \begin{definition} An \emph{abstract simplicial complex} $\Delta$ on the vertex set $V$ is a collection $\Delta$ of finite subsets of $V$, called \emph{faces}, such that $x \in V$ implies $\{x\} \in \Delta$ and $F \subseteq F' \in \Delta$ implies $F \in \Delta$. The dimension of a face is $\dim F = |F| - 1$, and $\dim \Delta = \sup_{F \in \Delta} \dim F$. A complex is \emph{pure $d$-dimensional} if every face is contained in some $d$-dimensional face. In this case, we denote the set of $d$-dimensional faces (called facets) by $\mathscr{F}(\Delta)$. Two facets $A, A'$ are \emph{adjacent} if $\dim(A \cap A') = d-1$. If $F \in \Delta$, let $\bar{F}$ be the \emph{simplex} $\{E \mid E \subseteq F\}$. \end{definition} \begin{definition} Let $(W,S)$ be a Coxeter system with $|S|<\infty$. We write $(s) := S \setminus \{s\}$, for $s \in S$, and $V:= \bigcup_{s \in S} W/W_{(s)}$ for the collection of all left cosets of all maximal parabolic subgroups. The \emph{Coxeter complex} $\Delta(W, S)$ is by definition the pure $(|S|-1)$-dimensional simplicial complex on the vertex set $V$ with facets $C_w := \{w W_{(s)} \mid s \in S\}$, for $w \in W$. For a face $F \in \Delta(W, S)$, define its \emph{type} $\tau(F) = \{s \in S \mid F \cap W/W_{(s)} \neq \emptyset\}$. \end{definition} Let $(W, S)$ be a Coxeter system and $u,v \in W$. A path from $u$ to $v$ is a sequence $u = w_0 \to w_2 \to \cdots \to w_s = v$ such that $w_{i+1} = w_i s$ for some simple reflection $s \in S$. A subset $K \subseteq W$ is called \emph{convex} if for every $u, v \in K$ we have that any shortest path from $u$ to $v$ lies in $K$. This mirrors the notion of convexity for unions of closed alcoves: \begin{proposition}[Prop. 3.5, \cite{alcove2}]\label{prop:convex} Let $P$ be a bounded subset which is a union of closed alcoves. Then $P$ is a convex polytope if and only if one of the following conditions hold: \begin{enumerate}[(1)] \item For any two alcoves $A, B \subset P$, any shortest path from $A= A_0 \to A_1 \to A_2 \to \cdots \to A_s = B$ lies in $P$. Here $A_i$ are alcoves and $A' \to A''$ means that the closure of the two alcoves $A'$ and $A''$ share a facet. \item The subset $W_P = \{w \in W_\aff \mid w(A_\circ) \subset P\}$ of the affine Weyl group is a convex subset. \end{enumerate} \end{proposition} Let $(W, S)$ be a Coxeter system and let $\mathscr{F}(\Delta(W, S))$ denote the set of all facets of $\Delta(W, S)$. By \cite[pp. 40-44]{bourbaki} and \cite[Chap. 2]{tits}, there is a bijection between $W$ and $\mathscr{F}(\Delta(W, S))$ given by $w \mapsto C_w$. Two facets $C_w$ and $C_{w'}$ are adjacent if and only if $w' = ws$ for some $s \in S$. By \cite{humphreys}, there is a bijection between $\mathscr{F}(\Delta(W, S))$ and the set of alcoves in the affine Coxeter arrangement. This bijection maps $C_e$ to the fundamental alcove $A_\circ$ (see \cref{def:fundamental}), and maps $C_w$ to the alcove $A$ such that there is a shortest path $A_\circ \overset{s_1}{\to} A_1 \overset{s_2}{\to} \cdots \overset{s_k}{\to} A$ where $s_1 s_2 \cdots s_k$ is a reduced word for $w$. The group $W$ acts on $\Delta(W, S)$ by left translation $w: v W_{(s)} \mapsto wv W_{(s)}$, and this action is \emph{type-preserving}, i.e., $\tau(w(F)) = \tau(F)$ for all faces $F \in \Delta(W, S)$. The key property of Coxeter complexes of interest in this paper is that they have a \emph{shelling order}: \begin{definition}\label{def:shelling} Let $\Delta$ be a pure $d$-dimensional complex of at most countable cardinality. A \emph{shelling} of $\Delta$ is a linear order $A_1, A_2, A_3, \dots$ on the set of facets of $\Delta$ such that $\bar{A_k} \cap \Delta_{k-1}$ is pure $(d-1)$-dimensional for $k = 2,3,\dots$, where $\Delta_{k-1} = \bar{A}_1 \cup \cdots \cup \bar{A}_{k-1}$. \end{definition} Given a shelling, define the \emph{restriction} of a facet $A_k$ by $$\mathscr{R}(A_k) = \{x \in A_k \mid A_k - \{x\} \in \Delta_{k-1}\}.$$ \begin{theorem}[Theorem 2.1, \cite{bjorner}] Let $(W, S)$ be a Coxeter system, $|S|<\infty$. Then any linear extension of the weak ordering of $W$ assigns a shelling order to the facets of $\Delta(W, S)$. \end{theorem} In particular, Bj\"orner showed that the restriction of this shelling is $$ \mathscr{R}(C_w) = \{w W_{(s)} \mid s \in \D(w)\} .$$ \begin{corollary}\label{cor:cox} Let $(W, S)$ be a Coxeter system, $|S|<\infty$. Let $\Gamma_P$ be the subcomplex of $\Delta(W, S)$ induced by a convex subset $P$ of the Coxeter group $W$ that contains the identity element $e$. Any linear extension of the weak order is a shelling of $\Gamma_P$. \end{corollary} \begin{proof} Since $P$ contains the identity and is convex, if $w \in P$, then all the shortest paths from $e$ to $w$ are contained in $P$. That is, all the reduced words of $w$ are contained in $P$. Let $C_1, C_2, \dots$ denote a shelling of $\Delta(W, S)$, and let $C_{a_1}, C_{a_2}, \dots$ denote the subsequence of $C_1, C_2, \dots$ consisting of facets that are in $\Gamma_P$. For $k \geq 2$, let $\Delta_{k-1} = \bar{C}_1 \cup \cdots \cup \bar{C}_{k-1}$ and let $\Delta'_{k-1} = \bar{C}_{a_1} \cup \cdots \cup \bar{C}_{a_{k-1}}$. Then by definition, $\Delta'_{k-1} \subseteq \Delta_{k-1}$, so $C_{a_k} \cap \Delta'_{k-1} \subseteq C_{a_k} \cap \Delta_{{a_k}-1}$. Suppose $C_{a_k} = C_w$ for some $w \in W$. Since all the reduced words of $w$ are contained in $P$, for each $s \in \D(w)$, we have $C_w - \{ w W_{(s)} \} \in \Delta'_{k-1}$, so $C_{a_k} \cap \Delta'_{k-1} = C_{a_k} \cap \Delta_{{a_k}-1}$ is pure $(|S|-1)$-dimensional, concluding the proof. \end{proof} \begin{figure} \centering \tdplotsetmaincoords{60}{70} \begin{tikzpicture}[tdplot_main_coords, scale=5] \coordinate (A) at (1, 0, 0); \coordinate (B) at (1, 1, 0); \coordinate (C) at (0.5, 0.5, 0); \coordinate (D) at (1, 1, 1); \coordinate (E) at (0.5, 0.5, 0.5); \coordinate (F) at (1.5, 0.5, 0); \coordinate (G) at (1.5, 0.5, 0.5); \coordinate (H) at (1, 0.5, 0.5); \node[left,below] at (A) {$(1, 0, 0)$}; \node[right] at (B) {$(1, 1, 0)$}; \node[left] at (C) {$(\frac{1}{2}, \frac{1}{2}, 0)$}; \node[right,above] at (D) {$(1, 1, 1)$}; \node[above] at (E) {$(\frac{1}{2}, \frac{1}{2}, \frac{1}{2})$}; \node[right,below] at (F) {$(\frac{3}{2}, \frac{1}{2}, 0)$}; \node[right] at (G) {$(\frac{3}{2}, \frac{1}{2}, \frac{1}{2})$}; \draw[thick] (A) -- (G) -- (D) -- (E) -- cycle; \draw[thick] (B) -- (D); \draw[thick] (A) -- (F) -- (B); \draw[thick,dotted] (B) -- (C); \draw[thick,dotted] (A) -- (C); \draw[thick,dotted] (E) -- (C); \draw[thick] (G) -- (F); \draw[thick] (A) -- (D); \draw[thick] (E) -- (G); \draw[thick] (G) -- (B); \draw[thick,dotted] (H) -- (B); \draw[thick,dotted] (A) -- (B); \draw[thick,dotted] (E) -- (B); \end{tikzpicture} \begin{tikzpicture} \coordinate (A) at (0,-1); \coordinate (B) at (1,0); \coordinate (C) at (1,1); \coordinate (D) at (3,0); \coordinate (E) at (3,1); \coordinate (F) at (4,-1); \node at (A) {$\bullet$}; \node at (B) {$\bullet$}; \node at (C) {$\bullet$}; \node at (D) {$\bullet$}; \node at (E) {$\bullet$}; \node at (F) {$\bullet$}; \draw[thick] (A) -- (B) node[midway,above] {2}; \draw[thick] (B) -- (C) node[midway,left] {1}; \draw[thick] (D) -- (E) node[midway,right] {1}; \draw[thick] (C) -- (E) node[midway,above] {2}; \draw[thick] (B) -- (D) node[midway,below] {2}; \draw[thick] (D) -- (F) node[midway,above] {2}; \end{tikzpicture} \caption{The generalized hypersimplex for $\Phi = B_3$ and $k = 2$. The Ehrhart series of $\Delta^{B_3}_2$ is $\Ehr(\Delta^{B_3}_2,z) = \frac{1+z+3z^2+z^3}{(1-z)^2 (1-z^2)^2}$} \label{fig:B32} \end{figure} \section{Proof of the main theorem}\label{sec:proof} We can now translate the shelling order of subcomplexes of the Coxeter complex in Corollary \cref{cor:cox} into a shelling order of the alcoves of an alcoved polytope $P$, and use this shelling order to compute the Ehrhart series of $P$. \begin{definition}\label{def:partialorder} Let $\Gamma = (V,E)$ be an undirected graph, and let $v_0 \in V$ be an arbitrary vertex of $\Gamma$. Define the \emph{breadth-first search order of $\Gamma$ with root $v_0$} as the partial order $(\P_{v_0,\Gamma},\prec)$ on $V$ such that for two distinct vertices $u, v \in V$, $u \prec v$ if and only if there is a shortest path from $v_0$ to $v$ passing through $u$. \end{definition} The following is a corollary of \cref{cor:cox}. \begin{corollary}\label{cor:shelling} Let $P$ be an alcoved polytope and let $\Gamma_P = (V, E)$ be the graph of the alcoved triangulation of $P$. For any $v_0 \in V$, any linear extension of the partial order $(\P_{v_0,\Gamma_P},\prec)$ is a shelling order of the alcove triangulation of $P$. \end{corollary} \begin{proof} Let $W = W_\aff$ be the affine Weyl group that acts on the alcoves in $P$. Let $W_P = \{w \in W \mid w(A_\circ) \subset \}$, which is convex by \cref{prop:convex}. Let $v_0$ be the alcove $C_w$ for some $w \in W$. Then $w^{-1}(W_P)$ is a convex subset of $W$ that contains the identity, and $w^{-1}(\mathcal{P}_{v_0, \Gamma_P},\prec)$ is the weak order on the facets of $w^{-1}(\Gamma_P)$. By \cref{cor:shelling}, any linear extension of $w^{-1}(\mathcal{P}_{v_0, \Gamma_P},\prec)$ is a a shelling order of $w^{-1}(\Gamma_P)$. Since $W$ acts transitively on the set of all alcoves, we conclude the proof. \end{proof} \begin{lemma}\label{lem:disjointunion} Let $P$ be an alcoved polytope. Let $\Gamma_P = (V,E)$ be the graph of the alcoved triangulation of $P$. Fix some $v_0 \in V$ and let $(\P_{v_0,\Gamma_P}, \prec)$ be the breadth-first search order on $\Gamma_P$ with root $v_0$. For each alcove $A$ in $P$, let $\mathscr{I}_A=\{\bar{A} \cap \bar{A'} \mid A' \precdot A \}$ be a subset of facets of the closure $A$. Then the set of half-open alcoves $A^\circ = \bar{A} \setminus (\cup \mathscr{I}_A)$, are mutually disjoint, and their union is equal to $P$, i.e., $$ P = \bigsqcup_{A \in V} A^\circ. $$ \end{lemma} \begin{proof} We prove this by induction. Let $A \in V$ be an alcove in $P$. Let $\mathcal{A} = \{A' \in V \mid A' \prec A \}$ be the set of alcoves that comes before $A$ in the topological sort of $\Gamma_P$. We show that \begin{enumerate}[(1)] \item $A^\circ \cap \bigcup_{A' \in \mathcal{A}} A'^{\circ} = \emptyset$; \label{disjoint} \item $A^\circ \cup \bigcup_{A' \in \mathcal{A}} A'^{\circ} = \bar{A} \cup (\bigcup_{A' \in \mathcal{A}} \bar{A'})$.\label{union} \end{enumerate} To show (\ref{disjoint}), assume there exists $A' \in \mathcal{A}$ such that $A^\circ \cap A'^{\circ} \neq \emptyset$. Then $\bar{A} \cap \bar{A}' \neq \emptyset$ and it must be contained in a facet of $\bar{A}$ in $\bar{A} \cap \bigcup_{A' \in \mathcal{A}} \bar{A}'$ because $\prec$ is a shelling. This facet is equal to $\bar{A} \cap \bar{A}'$ for $A' \precdot A$, and therefore will be excluded in $A^\circ$. To show (\ref{union}), we first use the induction hypothesis that $\bigcup_{A' \in \mathcal{A}} A'^{\circ} = \bigcup_{A' \in \mathcal{A}} \bar{A}'$. Then, since $\cup \mathscr{I}_A \subset \bigcup_{A' \in \mathcal{A}} \bar{A}'$, we have $A^\circ \cup (\bigcup_{A' \in \mathcal{A}} \bar{A}' ) = \bar{A} \cup (\bigcup_{A' \in \mathcal{A}} \bar{A}' ) $. \end{proof} We now have all the ingredients we need to compute the Ehrhart polynomial of an arbitrary alcoved polytope: \begin{theorem}\label{thm:main} Let $P$ be an alcoved polytope and let $\Gamma_P = (V, E)$ be the edge-weighted graph of its alcoved triangulation (see \cref{def:graphlabel} for details). Given any $v_0 \in V$, let $\mathcal{P}_{v_0,\Gamma_P}$ be the breadth-first search order of $\Gamma_P$ with root $v_0$ (see \cref{def:partialorder}). The Ehrhart series of $P$ is equal to $$\Ehr(P,z) = \frac{\sum_{v \in V} z^{\wt(v)}}{\prod_{i=0}^n (1-z^{\ell_i})}$$ where $\wt(v) = \sum_{u \precdot v} \wt((u,v))$ is the sum of the weights of the edges between $v$ and the elements it covers. \end{theorem} \begin{proof}[Proof of \cref{thm:main}] By \cref{lem:disjointunion} and \cref{lem:half-opensimplex} and the \emph{additivity} of Ehrhart series, we conclude the proof. \end{proof} \begin{example} Consider the $n$-dimensional hypercube $\cube_n=[0,1]^n$. The graph of alcoved triangulation of a hypercube is the weak Bruhat graph of the symmetric group $S_n$. Therefore, $h^*(\cube_n,z) = \sum_{w \in S_n} z^{\des(w)}$, which is the Eulerian polynomial. This is a well known result from \cite{stanley80}. \end{example} \section{The hypersimplex $\Delta_{2,n}$}\label{sec:2n} We now speculate on a relationship between our shelling order formula and a combinatorial formula for a well-studied polytope. The hypersimplex $\Delta_{k,n}$ is the subset of $[0,1]^n\subset\mathbb{R}^n$ consisting of points $\{(x_1,\dots,x_n) \mid x_1 + \cdots + x_n = k\}$. Under the linear transformation $$ y_i = x_1+ \cdots + x_i, $$ the hypersimplex $\Delta_{k,n}$ can be realized as an alcoved polytope defined by $0 \leq y_i - y_{i-1} \leq 1$ and $y_n = k$ for all $i =1,\dots,n$ with the convention $y_0 = 0$. The coefficients of the $h^*$-polynomial form the \emph{$h^*$-vector}. The $h^*$-vector of the hypersimplex $\Delta_{k,n}$ was proved to be enumerated by \emph{hypersimplicial decorated ordered set partitions}. \begin{definition}[\cite{early2017conjectures}] A \emph{decorated ordered set partition} $((S_1)_{s_1},\dots,(S_p)_{s_p})$ of type $(k,n)$ consists of an ordered set partition $(S_1, \dots, S_p)$ of $[n]$ and an $p$-tuple of integers $(s_1, \dots, s_p) \in \Z^{p}$ such that $\sum_{i=1}^p s_i = k$ and $s_i \geq 1$. We call each $S_i$ a block and we place them on a circle in the clockwise fashion then think of $s_i$ as the clockwise distance between adjacent block $S_i$ and $S_{i+1}$. We regard them up to cyclic rotation of blocks, so $((S_1)_{s_1},(S_2)_{s_2},\dots,(S_p)_{s_p})$ is the same as $((S_2)_{s_2},\dots,(S_p)_{s_p},(S_1)_{s_1})$. A decorated ordered set partition is \emph{hypersimplicial} if $1 \leq s_i \leq |S_i|-1$ for all $i$. The \emph{winding vector} of a decorated ordered set partition is an $n$-tuple of integers $(l_1,\dots,l_n)$ such that $l_i$ is the distance of the path starting from the block containing $i$ to the block containing $(i+1)$ moving clockwise. If $i$ and $(i+1)$ are in the same block then $l_i = 0$. If $l_1 + \cdots + l_n = kd$, then we define the \emph{winding number} to be $d$. \end{definition} \begin{theorem}[\cite{Kimh*}] Let $h^*(\Delta_{k,n},z) = h_0^*(\Delta_{k,n}) + h_1^*(\Delta_{k,n}) z + \cdots + h_{n-1}^*(\Delta_{k,n}) z^{n-1}$ be the $h^*$-polynomial of the hypersimplex $\Delta_{k,n}$. The number of hypersimplicial decorated ordered set partitions of type $(k,n)$ and winding number $d$ is $h_d^*(\Delta_{k,n})$. \end{theorem} In this section, we conjecture the relation between our formula in \cref{thm:main} for the hypersimplex $\Delta_{2,n}$ to the formula of Early and Kim given by the hypersimplicial ordered set partitions. \begin{definition} If a hypersimplicial decorated ordered set partition of type $(2,n)$ with nonzero winding number is of the form $((A)_1, (B)_1)$ for nonempty disjoint subsets $A, B \subseteq [n]$ with $A \cup B = [n]$, then we simply denote it by the set of sets $\{A,B\}$. This notation also emphasizes the fact that we regard each hypersimplicial decorated ordered set partition up to cyclic rotation of blocks. \end{definition} Let $\Gamma_{2,n}$ be the graph of alcoved triangulation of the hypersimplex $\Delta_{2,n}$, see \cref{def:graphlabel}. We associate a hypersimplicial decorated ordered set partition of type $(2,n)$ with winding number 1 to each edge in $\Gamma_{2,n}$. \begin{definition}\label{def:edgelabels} Let $A,A' \subseteq \Delta_{2,n}$ be two alcoves such that $A$ and $A'$ share a common facet. Then $A \cap A'$ is of the form $y_j - y_i = 1$ for some $i \not\equiv j \pm 1 \bmod n$. We associate the hypersimplicial decorated ordered set partition $\{\{i,\dots,j-1\}, \{j,\dots,i-1\}\}$ to the edge $(A,A')$ in the graph $\Gamma_{2,n}$. \end{definition} \begin{definition} Two hypersimplicial decorated ordered set partitions of type $(2,n)$ and winding number 1 are \emph{adjacent} if they can be associated to facets of the same alcove. \end{definition} \begin{example} The hypersimplicial decorated ordered set partitions $\{\{2,3\},\{4,5,1\}\}$ is adjacent to $\{\{1,2\},\{3,4,5\}$ and $\{\{3,4\},\{5,1,2\}\}$, but $\{\{2,3\},\{4,5,1\}\}$ is not adjacent to $\{\{4,5\},\{1,2,3\}\}$. \end{example} To relate the formula in \cref{thm:main} to hypersimplicial decorated ordered set partitions, we define a map $\psi$ from a set of $d$ adjacent hypersimplicial decorated ordered set partitions of winding number 1 to a single hypersimplicial decorated ordered set partition of winding number $d$. \begin{definition}\label{def:newdosp} For two hypersimplicial decorated ordered set partitions $\{A, B\}, \{A', B'\}$ of type $(2,n)$ with nonzero winding numbers, we define $\psi(\{A, B\}, \{A', B'\})$ to be a new hypersimplicial decorated ordered set partition $\{A'', B''\}$ of type $(2,n)$ such that $A'' = A \triangle A'$ is the symmetric difference between $A$ and $A'$ and $B'' = [n] \setminus A''$ is the complement of $A''$. For a collection of $d$ adjacent hypersimplicial decorated ordered set partitions $\{A_1,B_1\},\dots, \{A_d, B_d\}$ of type $(2,n)$ and winding number 1, we define $\psi(\{A_1,B_1\},\dots, \{A_d, B_d\})$ recursively to be $$\psi(\psi(\{A_1,B_1\},\dots, \{A_{d-1}, B_{d-1}\}), \{A_d, B_d\}).$$ \end{definition} \begin{example}\label{ex:1223} Consider $\{\{2,3\},\{4,5,1\}\}$ and $\{\{1,2\},\{3,4,5\}$ of type $(2,5)$ and winding number 1, then $\psi(\{\{2,3\},\{4,5,1\}\}, \{\{1,2\},\{3,4,5\}) = \{\{1,3\},\{2,4,5\}\}$. \end{example} \begin{example} Consider $\{\{1,2,3\},\{4,5,6\}\}$ and $\{\{2,3,4\},\{5,6,1\}\}$ of type $(2,6)$ and winding number 1, then $\psi(\{\{1,2,3\},\{4,5,6\}\}, \{\{2,3,4\},\{5,6,1\}\}) = \{\{1,4\},\{2,3,5,6\}\}$. \end{example} \begin{example} Consider $\{\{1,2,3\},\{4,5,6\}\}$, $\{\{2,3,4\},\{5,6,1\}\}$, and $\{\{3,4,5\},\{6,1,2\}\}$ of type $(2,6)$ and winding number 1, we have \begin{align*} &\psi(\{\{1,2,3\},\{4,5,6\}\}, \{\{2,3,4\},\{5,6,1\}\}, \{\{3,4,5\},\{6,1,2\}\}) \\ &= \psi(\{\{1,4\},\{2,3,5,6\}\}, \{\{3,4,5\},\{6,1,2\}\}) \\ &= \{\{1,3,5\},\{2,4,6\}\}. \end{align*} \end{example} \begin{conjecture}\label{conj:hypersimplicial decorated ordered set partition2n} For any $n$ and for any alcove $A_0$ in $\Delta_{2,n}$, let $\mathcal{P}_{A_0} = (V, E)$ be the breadth-first search order of $\Gamma_{2,n}$ with root $A_0$ (see \cref{def:partialorder}). For $v \in V$, let $\cover(v) = \# \{u \in V \mid u \precdot v \text{ in } \mathcal{P}_{A_0 }\}$ be the number of elements $v$ covers. The map $\psi$ is a bijection from the set $\{v \in V \mid \cover(v) = d\}$ and the set of hypersimplicial decorated ordered set partitions of type $(2,n)$ with winding number $d$. \end{conjecture} \begin{figure} \centering \begin{tikzpicture}[scale=5] \node[inner sep=2pt, scale=.8] (4123) at (0:1) {$\{3,5\}_1, \{1,2,4\}_1$}; \node[inner sep=2pt, scale=.8] (1243) at (72:1) {$\{2,4\}_1,\{1,3,5\}_1$}; \node[inner sep=2pt, scale=.8] (1324) at (144:1) {$\{1,3\}_1,\{2,4,5\}_1$}; \node[inner sep=2pt, scale=.8] (2134) at (216:1) {$\{2,5\}_1,\{1,3,4\}_1$}; \node[inner sep=2pt, scale=.8] (2341) at (288:1) {$\{1,4\}_1,\{2,3,5\}_1$}; \node[inner sep=2pt, scale=.8] (1423) at (36:0.6) {$\{3,4\}_1,\{1,2,5\}_1$}; \node[inner sep=2pt, scale=.8] (3124) at (108:0.6) {$\{2,3\}_1,\{1,4,5\}_1$}; \node[inner sep=2pt, scale=.8] (1342) at (180:0.6) {$\{1,2\}_1,\{3,4,5\}_1$}; \node[inner sep=2pt, scale=.8] (2314) at (252:0.6) {$\{1,5\}_1,\{2,3,4\}_1$}; \node[inner sep=2pt, scale=.8] (3412) at (324:0.6) {$\{4,5\}_1,\{1,2,3\}_1$}; \node[inner sep=2pt, scale=.8] (3142) at (0,0) {$\{1,2,3,4,5\}_2$}; \draw[thick,->] (3142) -- (1423); \draw[thick,->] (3142) -- (3124); \draw[thick,->] (3142) -- (1342); \draw[thick,->] (3142) -- (2314); \draw[thick,->] (3142) -- (3412); \draw[thick,->,orange] (3124) -- (1324); \draw[thick,->] (3124) -- (1243); \draw[thick,->] (1423) -- (4123); \draw[thick,->] (1423) -- (1243); \draw[thick,->] (3412) -- (4123); \draw[thick,->] (3412) -- (2341); \draw[thick,->] (2314) -- (2134); \draw[thick,->] (2314) -- (2341); \draw[thick,->] (1342) -- (2134); \draw[thick,->,orange] (1342) -- (1324); \end{tikzpicture} \caption{This is the graph of alcoved triangulation of the hypersimplex $\Delta_{2,5}$. Our root alcove $A_0$ is at the center. The arrows indicate cover relations in $\mathcal{P}_{A_0}$, pointing in increasing directions. The orange arrows are facets representing the cover relations of the alcove labeled by $\{1,3\}_1,\{2,4,5\}_1$.} \label{fig:hypersimplex25} \end{figure} \begin{example} In \cref{fig:hypersimplex25}, we choose $A_0$ to be the simplex in the center of the graph $\Gamma_{2,5}$ and we label it by the unique hypersimplicial decorated ordered set partition of type $(2,5)$ with winding number 0, which is $\{1,2,3,4,5\}_2$. The arrows are cover relations, and they point in increasing directions in $\mathcal{P}_{A_0}$. The alcove labeled by $\{1,3\}_1,\{2,4,5\}_1$ covers two alcoves in the poset $\mathcal{P}_{A_0}$, through facets colored by orange labeled by $\{1,2\}_1,\{3,4,5\}_1$ and $\{2,3\}_1,\{1,4,5\}_1$. This agrees with previous \cref{ex:1223}. \end{example} \printbibliography \end{document}
2412.02777v1
http://arxiv.org/abs/2412.02777v1
How to quantify the coherence of a set of beliefs
\documentclass[11pt,reqno]{amsart} \usepackage{amsmath,amsthm,verbatim,enumerate} \usepackage{bbm} \pdfoutput=1 \addtolength{\oddsidemargin}{-.2in} \addtolength{\evensidemargin}{-.2in} \addtolength{\textwidth}{.4in} \def\topfraction{.9} \def\floatpagefraction{.8} \newcommand{\arxiv}[1]{{\tt \href{http://arxiv.org/abs/#1}{arXiv:#1}}} \usepackage[utf8]{inputenc}\usepackage{mathtools} \usepackage{amsfonts} \usepackage{amssymb} \usepackage{algpseudocode} \usepackage{xcolor} \usepackage{graphicx} \usepackage{subcaption} \usepackage{float} \newcommand{\set}[1]{\left\{#1\right\}} \newcommand{\rsp}[1]{\langle#1\rangle} \newcommand{\pderiv}[2]{\frac{\partial #1}{\partial #2}} \newcommand{\deriv}[2]{\frac{d #1}{d #2}} \newcommand{\F}{\mathbb{F}} \newcommand{\R}{\mathbb{R}} \newcommand{\prob}{\mathbb{P}} \newcommand{\bec}{\boldsymbol} \usepackage{listings} \usepackage{array} \usepackage[backend=biber,style=numeric,sorting=none]{biblatex} \addbibresource{main.bib} \newtheorem{theorem}{Theorem} \newtheorem{lemma}{Lemma}[section] \newtheorem*{corollary}{Corollary} \theoremstyle{definition} \newtheorem*{definition}{Definition} \DeclareMathOperator*{\argmax}{arg\,max} \DeclareMathOperator*{\argmin}{arg\,min} \newenvironment{proofS}{ \renewcommand{\proofname}{Proof Sketch}\proof}{\endproof} \usepackage{enumitem} \newenvironment{enro}{\begin{enumerate}[label=(\roman*)]}{\end{enumerate}} \usepackage{ bbold } \usepackage{hyperref} \hypersetup{colorlinks} \author[R. Hess]{Rowan Hess} \address{Rowan Hess. Cornell University, Ithaca, NY 14853.} \email{[email protected]} \thanks{LL was partially supported by a grant from Open Philanthropy.} \author[L.\ Levine]{Lionel Levine} \address{Lionel Levine. Department of Mathematics, Cornell University, Ithaca, NY 14853. } \email{[email protected]} \begin{document} \title{How to quantify the coherence of a set of beliefs} \date{\today} \begin{abstract} Given conflicting probability estimates for a set of events, how can we quantify how much they conflict? How can we find a single probability distribution that best encapsulates the given estimates? One approach is to minimize a loss function such as binary KL-divergence that quantifies the dissimilarity between the given estimates and the candidate probability distribution. Given a set of events, we characterize the facets of the polytope of coherent probability estimates about those events. We explore two applications of these ideas: eliciting the beliefs of large language models, and merging expert forecasts into a single coherent forecast. \end{abstract} \maketitle \section{Introduction} \noindent When two or more predictions conflict, what is a principled way to combine them into a single meta-prediction? \\\\ This kind of question comes up in several different domains: \begin{itemize} \item Multiple experts are independently asked to make predictions, and we would like to integrate their predictions into a single meta-prediction that expresses the ``wisdom of the crowd'' \cite{galton_vox_1907, Ungar2012TheGJ, sempere2023alignment, mcandrew_aggregating_2021}. \item A single individual may hold incoherent beliefs without being aware of the incoherence \cite{herzog_harnessing_2014}. However, when someone points out that their beliefs are incoherent they may want to update to the ``closest'' coherent set of beliefs. \item A market maker that has beliefs about underlying probabilities of events may want to give a coherent set of quotes so as to avoid giving a ``Dutch book" that is exploitable via risk-free arbitrage \cite{dutch_book}. \item The expressed opinions of a large language model can be very sensitive to small changes in the prompt. To investigate whether the model holds an internally coherent belief and extract that belief, one approach is to ensemble many weak predictors derived from the model's internal state into a single meta-prediction \cite{burns2022discovering}. \end{itemize} \noindent Given a ground set $\Omega$, a list of events $\mathtt E = E_1, \ldots, E_n \subset \Omega$, and a list of \textit{credences} $q_1,\ldots,q_n$, the vector $\bec q = (q_1,\ldots,q_n)$ is called \textit{coherent} with respect to $\mathtt E$ if there is a probability measure $P$ (on the Boolean algebra generated by $E_1,\ldots,E_n$) such that $q_i = P(E_i)$ for all $i$. \\\\ For example, when predicting the next day's weather, suppose three of our events are ``rainy'', ``warm and rainy'', ``cold and rainy''. Then we have the coherence condition \[ P(\text{warm and rainy}) + P(\text{cold and rainy}) = P(\text{rainy}). \] If ``warm or rainy'' is also one of our given events $E_i$, then we have the additional coherence condition \[ P(\text{warm}) + P(\text{rainy}) - P(\text{warm and rainy}) = P(\text{warm or rainy}). \] These two examples of coherence conditions are both the result of probabilities being overdetermined: fixing the probabilities of some events determines the probabilities of others. There is also a second kind of coherence condition that results from probabilities being between by 0 and 1. For example, even if ``warm or rainy'' is not one of our events, we still have the coherence condition \[ P(\text{warm}) + P(\text{rainy}) - P(\text{warm and rainy}) \leq 1. \] \noindent Given a list of events $\mathtt E$ and credences $\bec q$ we would like a systematic way of measuring incoherence: a loss function $L^*:[0, 1]^n\rightarrow \R\cup \set\infty$ satisfying $L^*(\bec q) \geq 0$, with equality if and only if $\bec q$ is coherent. One natural method of constructing such a loss is via some notion of ``distance'' from the vector $\bec q$ to the set $C(\mathtt E)$ of coherent vectors: \begin{equation} \label{eq:theloss} L^*(\bec q) = \inf \{L(\bec p,\bec q) \,:\, \bec p \in C(\mathtt E) \}. \end{equation} We will discuss several choices of ``distance" function $L(\bec p,\bec q)$. We write distance in quotation marks because $L$ need not be a metric (e.g. it might not be symmetric in $\bec p$ and $\bec q$). \\\\ The forecaster seeking to combine the beliefs of others (or individual seeking to correct their incoherent beliefs) can adopt the minimizer $\bec{p^*}$ of \eqref{eq:theloss}. In Theorem \ref{punique} below we give conditions for existence and uniqueness of $\bec{p^*}$. \subsection{Plan of the paper} We start Section \ref{sec:seperateBeliefs} defining the process of loss function minimization more precisely and giving conditions for when there is a unique minimizer $\bec{p^*}$. We proceed by summarizing a result from Predd et al.\ \cite{predd_probabilistic_2009} that can motivate one's choice of loss function and then by looking at two specific examples of loss functions (binary KL divergence and its transpose). We discuss natural justifications for these loss functions in Theorems \ref{thm:coherentBetter} and \ref{thm:fo}.\\\\ Section \ref{sec:nn} explores a possible application of our method to eliciting beliefs from large language models. We use loss functions to generalize the method suggested in \cite{burns2022discovering} and consider possible modifications that this generalization allows. \\\\ Section \ref{sec:inequalities} explores the geometry of $C(\mathtt{E})$, the set of coherent beliefs, which is a convex polytope. It is easy to describe the vertices of this polytope (Lemma~\ref{lem:convex}), but somewhat more challenging to describe its facets (Theorem~\ref{thm:restatement}). \\\\ In Section \ref{sec:indCohExp}, we consider how predictions should be aggregated in a setting where multiple experts each make internally coherent predictions. We seek a method to aggregate these predictions that leverages the internal coherence of each expert. \\\\ Finally, in Section \ref{sec:ex}, we illustrate these methods in the scenario of masked letter prediction and discuss the results. \subsection{Related work} This problem of the reconciliation has been examined in the past \cite{capotorti_correction_2010, thimm_inconsistency_2013}. In particular, \cite{capotorti_correction_2010} proposes the use of binary KL divergence as a loss function and discusses several of its properties. \subsection{Preliminaries} We will assume that there are $n$ events $E_i$ for $i\in \set{1, \dots, n}$ and that for each event $E_i$, we receive some estimate $q_i$ of its probability. These estimates may come from a single expert or multiple experts. We discuss the implications of one expert giving multiple beliefs in Section \ref{sec:indCohExp}. \begin{definition} A \textit{credence base} is the ordered pair $(\mathtt E, \textbf q)$ where $\mathtt E = (E_i)_{i=1}^{n}$ is a sequence containing the (potentially repeated) events whose probabilities are being estimated and $\bec q\in [0, 1]^n$ is the vector whose $i$th entry is $q_i$. \end{definition} \noindent Recalling that every finite Boolean algebra is the power set of a set, let $2^\Omega$ be the Boolean algebra generated by $\mathtt E$, where $\Omega=\set{\omega_1, \dots, \omega_N}$ is finite as $\mathtt E$ is finite. For example, if $\mathtt E = (\set{\text{rock}}, \set{\text{paper}}, \set{\text{scissors},\text{rock}})$ then $\Omega = \set{\text{rock}, \text{paper}, \text{scissors}}$. \begin{definition} A credence base $(\mathtt E, \bec q)$ is \textit{coherent} if there exists a probability distribution $\pi:2^\Omega\rightarrow [0, 1]$ where $\pi(E_i)=q_i$ for all $i\in\set{1,\dots, n}$. We also call $\bec q$ \textit{coherent} with respect to $\mathtt E$ if $(\mathtt E, \bec q)$ is coherent. \end{definition} \begin{definition} The \textit{set of coherent beliefs} over events $\mathtt E$ is \[ C(\mathtt E)=\set{\boldsymbol p\in [0, 1]^n \,:\, (\mathtt E, \boldsymbol{p}) \text{ is coherent}}. \] \end{definition} \noindent \section{Loss Functions that Quantify Incoherence} \label{sec:seperateBeliefs} \noindent In this section, we examine a class of loss functions $L(\bec p,\bec q)$ that are additive over events. We give conditions that guarantee that for any credence base $(\mathtt E,\bec q)$ there is a unique coherent credence base $(\mathtt E,\bec{p^*})$ minimizing the loss $L(\bec p,\bec q)$. Then we state a theorem of Predd et al.\ \cite{predd_probabilistic_2009} showing that in a certain forecasting setting it is never advantageous to submit an incoherent forecast. We proceed by giving natural justifications for two examples of loss functions: binary KL divergence and its transpose. \\\\ Recall that a function $f$ is \textit{lower semi-continuous} at $x$ if $$\liminf_{x_n\rightarrow x} f(x_n) \ge f(x).$$ Also, we say that a function $f$ is \textit{strictly convex when finite} if for all $t\in(0, 1)$ and $x, x'$ in its domain, then $$tf(x) + (1-t)f(x') \ge f(tx+(1-t)x')$$ and the inequality is strict unless both sides are infinite. \begin{definition} A \textit{dissimilarity function} $\ell(p, q)$ is a function $\ell:[0, 1]\times [0, 1]\rightarrow [0, \infty]$ satisfying \begin{enro} \item $\ell$ is lower semi-continuous with respect to $p$ \item $\ell$ is strictly convex when finite with respect to $p$ \item $\ell(p, q)=0$ if and only if $p=q$. \end{enro} \end{definition} Some examples of dissimilarity functions are: \begin{itemize} \item $f(p, q) = p\ln \frac pq + (1-p) \ln \frac{1-p}{1-q}$ \item $f^o(p, q) = f(q, p)$ \item $\ell(p, q) = (p-q)^2$ \item $\ell(p, q) = \left\{\begin{array}{cc} 0 & p=q \\ \infty & p\ne q \end{array}\right..$ \end{itemize} \noindent Given dissimilarity functions $\ell_1, \dots, \ell_n$, we define an associated \textit{loss function} $L:[0, 1]^n\times [0, 1]^n\rightarrow [0, \infty]$ given by \begin{equation} L(\textbf p, \textbf q)=\sum_{i=1}^n \ell_i(p_i, q_i).\label{eqn:lossfunction} \end{equation} \noindent We typically consider all dissimilarity functions $\ell_i$ to be equal to some common $\ell$. We denote the corresponding loss function $L_\ell$ and call it the loss function derived from dissimilarity function $\ell$. We omit the subscript when it is clear from context. \\\\ Then, given a loss function $L$ with $n$ terms and a credence base $\mathcal{Q}=(\mathtt E, \bec q)$ with $n$ events, we can define the \emph{incoherence} of $\mathcal Q$ to be \begin{equation} L^*(\boldsymbol q):=\min_{\boldsymbol p\in C(\mathtt E)} L(\textbf p, \textbf q).\label{eqn:lstar} \end{equation} We denote the minimizing coherent belief by $\bec p^*(\bec q)$. The next theorem, which extends \cite[Cor.~7]{capotorti_correction_2010} to a more general family of loss functions, gives conditions under which this minimizer is unique. \begin{theorem} \label{punique} Given a credence base $(\mathtt E, \bec q)$ and a loss function $L$ of the form in (\ref{eqn:lossfunction}), if there exists a coherent $\bec p\in C(\mathtt E)$ such that $L(\bec p, \bec q)$ is finite, then there exists a unique $\bec {p^*}$ such that $$L(\bec{p^*}, \bec q) = \min_{\bec p\in C(\mathtt E)} L(\bec p, \bec q).$$ Moreover, $\bec{p^*} = \bec q$ if and only if $\bec q\in C(\mathtt E)$. \end{theorem} \begin{proof} As the sum of lower semi-continuous and strictly convex when finite functions, $L(\cdot, \bec q)$ is also lower semi-continuous and strictly convex when finite. Choose coherent $\bec p\in C(\mathtt E)$ such that $L(\bec p, \bec q)$ is finite. As the set $C(\mathtt E)$ is compact, these is a sequence $\bec p_1, \bec p_2, \dots C(\mathtt E)$ that converges to some $\bec p^*$ so that $$\lim_{k\rightarrow \infty} L(\bec p_k, \bec q) = \inf_{\bec p\in C(\mathtt E)} L(\bec p, \bec q).$$ Then, by lower semi-continuity, $$\inf_{\bec p\in C(\mathtt E)} L(\bec p, \bec q) \le L(\bec p^*, \bec q) \le \lim_{k\rightarrow \infty} L(\bec p_k, \bec q) =\inf_{\bec p\in C(\mathtt E)} L(\bec p, \bec q) $$ so $\bec p^*$ is a minimizer of $L(\cdot, \bec q)$ in $C(\mathtt E)$. \\\\ As for uniqueness, if $L(\bec p, \bec q)=L(\bec p', \bec q)$ for $\bec p \ne \bec p'$, $$L\left(\frac 12 \bec p+\frac 12 \bec p', \bec q\right)<\frac 12 L(\bec p, \bec q)+\frac 12 L(\bec p', \bec q)=L(\bec p, \bec q)$$ because $L(\bec p, \bec q)$ and $L(\bec p', \bec q)$ must be finite. Therefore, $\bec p$ is not a minimizer of $L(\cdot, \bec q)$. \\\\ If $\bec q\in C(\mathtt E)$, then $L(\bec q, \bec q)=0$ by property (iii). Therefore, as $L$ can only take non-negative values and the minimizer is unique, $\bec{p^*} = \bec q$. \end{proof} \noindent We discuss how to compute $\bec p^*$ in Appendix \ref{sec:findpstar} and the continuity of this $\bec p^*$ and $L^*$ in Appendix \ref{sec:continuity}. \subsection{The coherence theorem of Predd et al.}\label{section:predd} The number $L^*(\bec q)$ is a way of quantifying the incoherence of the credence base $(\mathtt E, \bec q)$. But which dissimilarity function $\ell$ should we use? One way to motivate the choice of $\ell$ arises from forecasting competitions: A forecaster submits a list of predicted probabilities $\bec q$ for events $\mathtt E$, and these are scored after the outcomes of all events are known. If event $E_i$ occurred, then the forecaster receives penalty $s(1, q_i)$ for that event; if $E_i$ did not occur, then the forecaster receives penalty $s(0, q_i)$ for that event. The forecaster aims to minimize the expectation of the random variable \[S(\mathtt E, \bec q)=\sum_{i=1}^n s(1_{E_i}, q_i).\] \noindent How should $s$ be chosen in order to elicit the true beliefs of each forecaster? If a forecaster believes that event $E_i$ will happen with probability $q_i$, then the scoring rule should incentivize them to submit prediction $q_i$ in order to minimize their expected score. A function $s$ with this property is called a \textit{proper scoring rule}. More formally, a proper scoring rule is a function $s:\set{0, 1}\times [0, 1] \rightarrow [0, \infty]$ satisfying \cite{predd_probabilistic_2009, winkler_good_1968}: \begin{enro} \item For each $p\in[0, 1]$, the quantity \[ ps(1, q)+(1-p)s(0, q)\] is uniquely minimized at $q=p$. \item $s$ is continuous in $q$. Note that we allow $s$ to take the value $+\infty$, but $(i)$ ensures that all values of $s$ are finite except possibly $s(0, 1)$ and $s(1, 0)$. \end{enro} \noindent Predd et al.\ \cite{predd_probabilistic_2009} prove that for any proper scoring rule $s$, there is a dissimilarity function $\ell$ so that for any incoherent $\bec q$, the prediction $\bec{p^*}(\bec q)$ using $\ell$ scores better than $\bec q$ no matter which events actually happen! Namely, let \begin{equation}\ell(p, q)=p[s(1, q)-s(1, p)]+(1-p)[s(0, q)-s(0, p)]\label{eqn:ell}\end{equation} be the expected excess penalty for predicting $q$ rather than $p$ when the true probability of an event is $p$. Note that for loss functions $L^*$ derived from such dissimilarity functions, we have $L^*(\bec q)\ge 0$ with equality if and only if $(\mathtt E, \bec q)$ is coherent, because such functions $\ell$ are uniquely minimized when $p=q$ as $s$ is a proper scoring rule. \begin{theorem}[\cite{predd_probabilistic_2009}] Let $s$ be any proper scoring rule, and let $\ell$ be given by (\ref{eqn:ell}). For any credence base $(\mathtt E, \bec q)$, let $\bec{p^*}(\bec q)\in C(\mathtt E)$ be the coherent minimizer of Theorem~\ref{punique}. Then, for all $\omega \in \Omega$, $${S(\mathtt E, \bec q)(\omega)-S(\mathtt{E}, \bec{p^*}(\bec q))(\omega)\ge L^*(\bec q)}.$$ \label{thm:coherentBetter} \end{theorem} \noindent Equation (\ref{eqn:ell}) still leaves a lot of possible choices of dissimilarity function $\ell$, since there are many choices of proper scoring rule $s$. Next we examine the case of the widely-used logarithmic scoring rule. The corresponding dissimilarity function $\ell$ turns out to be a variant of the Kulback-Leibler (KL) divergence \cite{kullback_information_1951}. \subsection{Binary KL divergence} Capotorti, Regoli, and Vattari \cite{capotorti_correction_2010} proposed correcting incoherent probability assessments using the binary KL-divergence \begin{equation} f(p, q)=p \ln \frac pq + (1-p)\ln \frac{1-p}{1-q}.\label{eqn:kldivergence} \end{equation} The resulting loss $L(\bec p,\bec q) = \sum_{i=1}^n f(p_i,q_i)$ can be interpreted as the \emph{total expected excess surprise of all experts}, when expert $i$ predicts probability $q_i$ for event $E_i$ and the ground truth probability of $E_i$ is $p_i$. \\\\This loss function can also be motivated by the logarithmic scoring rule $$s(i, q)=i\ln\frac 1 q + (1-i)\ln\frac{1}{1-q}$$ with the goal to minimize total score $S(\mathtt E, \bec q)=\sum_{i=1}^n s(1_{E_i}, q_i)$. By Theorem \ref{thm:coherentBetter}, if $(\mathtt E, \bec{q})$ is incoherent then the prediction $\bec{p^*}(\bec q)$ using $\ell=f$ scores strictly better than the prediction $\bec q$ no matter which events actually occur. \subsection{Transposed binary KL divergence} Next we consider the same loss with the roles of $\bec p$ and $\bec q$ interchanged: $$f^{o}(p, q)=f(q, p).$$ In Table~\ref{fig:Lminimizer} we describe the coherent minimizer $\bec p^*$ determined by $f$ and $f^o$ in each of two basic scenarios: \begin{enumerate} \item Partition: Fix $n$ and let $\Omega = \set{\omega_1, \omega_2, \dots, \omega_n}$. For all $1\le i \le n$, we ;et $E_i = \set{\omega_i}$. In this scenario all experts give estimates for different events, exactly one of which occurs. \item Repetition: Let $\Omega = \set{\omega_1,\omega_2}$ and for all $1\le i \le n$, let $ E_i=\set{\omega_1}$. In this scenario all experts give estimates for the same event. \end{enumerate} \begin{table}[h] \begin{center} \begin{tabular}{|m{4.5em} | m{16.5em}| m{13em} |} \hline \cline{2-3}& Binary KL loss & Transposed binary KL loss \\ \hline Partition & $\ln\frac{p^*_i}{1-p^*_i}-\ln\frac{q_i}{1-q_i}$ does not depend on $i$& $\frac{q_i-p^*_i}{p^*_i(1-p^*_i)}$ does not depend on $i$\\ \hline Repitition & $\frac{p^*}{1-p^*}=\left(\prod_{i} \frac{q_i}{1-q_i}\right)^{1/n}$& $p^*=\frac{1}{n}\sum_{i} q_i$ \\\hline \end{tabular} \caption{Description of the coherent minimizer $\bec{p^*}$ in each of two scenarios, with two different loss functions: Binary KL loss ($\ell=f$ as in (\ref{eqn:kldivergence})) and its transpose $(\ell=f^{o})$.} \label{fig:Lminimizer} \end{center} \end{table} \noindent The motivation for switching the order of the variables in $f$ comes from the following theorem, showing that the minimizer of loss functions determined by $f^{o}$ is now, in a certain scenario, the maximum likelihood estimation of $\boldsymbol{p}$. \\\\ Suppose the $i$th expert independently observes some number $w_i$ of independent trials of event $E_i$, and submits the estimate $q_i = \frac{k_i}{w_i}$ where $k_i$ is the number of trials in which $E_i$ occurred. Note that if $P(E_i)=p_i$, then $k_i$ has the binomial distribution $\text{Bin}(w_i,p_i)$. \\\\ To represent that each expert has a different amount of information, we can multiply each summand in (\ref{eqn:lossfunction}) by some weight $w_i$. \begin{theorem}\label{thm:fo} The maximum likelihood estimator $\bec{\hat p}$ given $\boldsymbol q$ is \[\argmin_{\boldsymbol{p}\in C(\mathtt E)}\sum_{i=1}^nw_if^{o}(p_i, q_i).\] \end{theorem} \begin{proof} The probability of receiving the estimations $\boldsymbol q$ is $$P(\boldsymbol q | \boldsymbol p)=\prod_{i=1}^n\binom{w_i}{q_iw_i}p_i^{q_iw_i}(1-p_i)^{(1-q_i)w_i}.$$ Next, if $\boldsymbol{\hat p}$ is the maximum likelihood estimation, then $$\boldsymbol{\hat p}= \argmax_{\boldsymbol p\in C(\mathtt E)}\prod_{i=1}^n\binom{w_i}{q_iw_i}p_i^{q_iw_i}(1-p_i)^{(1-q_i)w_i}$$ $$=\argmax_{\boldsymbol p\in C(\mathtt E)}\sum_{i=1}^nq_iw_i\ln p_i+(1-q_i)w_i\ln (1-p_i)$$ $$=\argmax_{\boldsymbol p\in C(\mathtt E)}\sum_{i=1}^nw_i(q_i\ln p_i + (1-q_i)\ln(1-p_i))$$ $$=\argmax_{\boldsymbol p\in C(\mathtt E)}\sum_{i=1}^nw_i\left(q_i\ln \frac{p_i}{q_i} + (1-q_i)\ln \frac{1-p_i}{1-q_i}\right)$$ $$=\argmin_{\boldsymbol p\in C(\mathtt E)}\sum_{i=1}^nw_i\left(q_i\ln \frac{q_i}{p_i} + (1-q_i)\ln \frac{1-q_i}{1-p_i}\right)$$ $$=\argmin_{\boldsymbol p\in C(\mathtt E)}\sum_{i=1}^nw_if^o(p_i, q_i).\qedhere$$ \end{proof} \noindent In general, $f$ produces more extreme probabilities than $f^o$ and gives more weight to extreme probability estimates, as demonstrated numerically in Appendices \ref{sec:compare} and \ref{sec:ex}. \section{Application: Eliciting Latent Beliefs from Language Models} \label{sec:nn} \noindent Large language models sometimes express beliefs that they ``know'' to be false, for example because the prompt includes false statements. Burns et al \cite{burns2022discovering} propose a method to elicit the model's actual belief about a natural language claim corresponding to an event $E$ from its hidden state $\phi(\text{des}_E)$ where $\text{des}_E$ is an assertion in natural language that the event $E$ holds. This method learns a linear probe $b(\phi)$ minimizing the loss \begin{equation} (q_{E} + q_{E^c} - 1)^2 + \min(q_{E},q_{E^c})^2\label{eqn:burns} \end{equation} where $q_{E} =\sigma(b(\phi(\text{des}_E)))= \frac{1}{1+e^{-b(\phi(\text{des}_E))}}$ and $q_{E^c} = \sigma(b(\phi(\text{des}_{E^c})))$ are intended to estimate the model's credence in the claims $\text{des}_E$ and $\text{des}_{E^c}$. The idea is that the term $(q_{E} + q_{E^c} - 1)^2$ is a measure of the incoherence of the linear probe, while the $\min(q_{E},q_{E^c})^2$ term encourages the linear probe to be decisive. We propose three possible elaborations of this approach, all of which train probes to minimize an expression of the form $$\mathcal I(\bec q) + \mathcal{J}(\bec q)$$ where $\mathcal I$ is a measure of incoherence and $\mathcal J$ is a measure of indecisiveness. \subsection{The Case of a Single Event and its Complement}\label{subsec:singleEvent} The following proposition shows that the measure of incoherence $(q_E+q_{E^c}-1)^2$ can be derived from the dissimilarity function $\ell(p, q)=2(p-q)^2$. \begin{lemma} When using dissimilarity function $\ell(p, q)=2(p-q)^2$, the incoherence function becomes $$L_\ell^*(q_E, q_{E^c}) = (q_{E} + q_{E^c} - 1)^2.$$ \end{lemma} \begin{proof} See Lemma \ref{lem:burns_loss} in the appendix. \end{proof} \noindent This approach could be generalized to choosing $\ell$ to be either the functions $f$ or $f^o$ discussed previously. The next two lemmas give approximations for $L^*$ when $q_E+q_{E^c}-1$ is small \begin{lemma} When using dissimilarity function $$\ell(p, q)=f(p, q)=p\ln \frac pq + (1-p)\ln \frac{1-p}{1-q}$$ the loss function becomes $$L_f^*(q_E, q_{E^c}) = \frac{(q_E + q_{E^c} - 1)^2}{(1-q_E+q_{E^c})(q_E+1-q_{E^c})}+O((q_E + q_{E^c} - 1)^3).$$\label{lem:burns_f2} \end{lemma} \begin{proof} See Lemma \ref{lem:burns_f} in the appendix. \end{proof} \begin{lemma} When using dissimilarity function $$\ell(p, q)=f^o(p, q)=q\ln \frac qp + (1-q)\ln \frac{1-q}{1-p}$$ the loss function becomes $$L_{f^o}^*(q_E, q_{E^c}) = \frac{(q_E + q_{E^c} - 1)^2}{(1-q_E+q_{E^c})(q_E+1-q_{E^c})}+O((q_E + q_{E^c} - 1)^3).$$\label{lem:burns_fo2} \end{lemma} \begin{proof} See Lemma \ref{lem:burns_fo} in the appendix. \end{proof} \noindent The reason that the $(1-q_E+q_{E^c})(q_E+1-q_{E^c})$ term might be desirable to have in the denominator is that it increases the loss for the same absolute error when the probabilities are more extreme, which mirrors the intuition behind common scoring functions such as log-loss. \subsection{Multiple Probes and Multiple Events} A natural extension of \cite{burns2022discovering} is to train $k$ probes $b_1, \dots, b_k$ where $b_i$ is trained to elicit the model's credence from its $i$th layer hidden state $\phi_i$ using $2m$ rephrasings $\text{des}_{E, j}$ and $\text{des}_{E^c, j}$ of a natural language assertion about an event $E$ and its complement $E^c$. Then, letting $q_i(\text{des}) = \sigma(b_i(\phi_i(\text{des})))$, one could replace the $(q_E + q_{E^c}-1)^2$ term of (\ref{eqn:burns}) with $$\mathcal I(\bec q) = L^*\left(q_i(\text{des}_{E, j}), q_i(\text{des}_{E^c, j}) \right)_{i=1}^{k}\left._{j=1}^m\right..$$ This could be further generalized to a set of $n$ interrelated events $\set{E_1, \dots, E_n}$ each of which has $m$ natural language assertions $\text{des}_{E_h, 1}, \dots, \text{des}_{E_h, m}$ that it holds, using the coherence term $$\mathcal I(\bec q) = L^*\left(q_i(\text{des}_{E_h, j})\right)_{i=1}^{k}\left._{h=1}^n\right.\left._{j=1}^{m}\right..$$ Theorem \ref{thm:holderImpliesDiff} in Appendix \ref{sec:continuity} gives a formula for the gradient of $L^*$ assuming $\bec p^*$ is sufficiently smooth. \\\\ Why would multiple probes ($k> 1$) help elicit beliefs? We hypothesize that the language model's beliefs -- when they exist -- arise from a bundle of internal heuristics that sometimes conflict. Each probe $b_i$ represents one such heuristic. In the case that $L^*$ is close to zero, the model's internal heuristics are mostly self-consistent (the credences are close to an actual probability distribution) and in this case we might say that the model ``has beliefs" about the claims $E_1,...,E_n$. These beliefs could be certain or uncertain: if all the probes think a coin flip is 50\% likely to land heads, then that is a coherent belief. On the other hand, if $L^*$ is large, then it is possible that the model ``does not have beliefs" about the claims $E_i$, or else that the probes simply did not discover the right heuristics. \\\\ Why would multiple events ($n> 2$) help elicit beliefs? By minimizing $L^*$ applied to more events, we are encouraging not just $q_E+q_{E^c}=1$ but also $q_{E\cup F} = q_E + q_F - q_{E\cap F}$ and so on. The philosophy is that to find ``beliefs", one should look for functions of the hidden state that obey the laws of probability. The logical dependencies between events are incorporated in the definition of $L^*$ as a minimum over $C(\mathtt E)$, as defined in equation (\ref{eqn:lstar}). We further study the geometry of $C(\mathtt E)$ in Section \ref{sec:inequalities}. \subsection{Generalizing the Decisiveness Term} Recall that the $\min(q_{E},q_{E^c})^2$ term in Equation (\ref{eqn:burns}) is present to encourage the linear probe to be decisive. However, in the scenarios above with more than 2 estimates, it is unclear what should replace it. We discuss some options below: \begin{itemize} \item We want the probes to be decisive so as to maximize information they give about the language model's internal state. One way to quantify the information content of $\bec p^*$ is by the maximum entropy of a probability distribution that gives $\bec p^*$. Hence, one could use the decisiveness term $$\mathcal J(\bec q) = \max \set{H(\pi) \, : \pi(\bec{ \mathtt E}) = \bec p^*(\bec q)}$$ where $$H(\pi) = \sum_{\omega\in \Omega} -\pi(\omega) \ln \pi(\omega).$$ \item The above notion can be generalized. Recall that ${s(i, q)=i\ln q + (1-i) \ln (1-q)}$ is the log-loss scoring rule. Then, we can write entropy as $$H(\pi) = E_\pi\left[ \sum_{\omega\in \Omega}1_\omega s(1, \pi(\omega))\right].$$ Recall that in Section \ref{section:predd}, we discussed how a proper scoring rule $s$ can be transformed into a loss function $\ell$. If using such a loss function, it might make sense to replace the log-loss function above with the proper scoring rule that $\ell$ is based on, making the decisiveness term $$\mathcal J(\bec q) = \max \set{E_\pi\left[ \sum_{\omega\in \Omega}1_\omega s(1, \pi(\omega))\right]: \pi(\bec{ \mathtt E}) = \bec p^*(\bec q)}$$ for a general proper scoring rule $s$. \item Using the idea that the least decisive distribution $\pi$ is the distribution that maximizes $$E_\pi\left[ \sum_{\omega\in \Omega}1_\omega s(1, \pi(\omega))\right],$$ let $\bec u = \pi(\mathtt E)$ be this least decisive set of coherent beliefs and use either $\mathcal J(\bec q) = -L(\bec u, \bec p^*(\bec q))$ or $\mathcal J(\bec q) = -L(\bec p^*(\bec q), \bec u)$ to reward the probe for being more decisive.\\ \item When using $f$ or $f^o$ as a measure of dissimilarity, the incoherence term by itself might be enough to incentivize decisiveness due to the increased sensitivity of the sigmoid function to noise when its input is close to $0$. To illustrate this, we revisit the scenario where we use one linear probe to find $q_E$ and $q_{E_c}$ from one natural language description of the event $E$ and one of its complement. We suppose that that the value of the linear probe $b(\phi(\text{des}_E))$ has normal distribution $N(\sigma^{-1}(p), S^2)$ and $b(\phi(\text{des}_{E^c}))\sim N(\sigma^{-1}(1-p), S^2)$ independently where $p$ is the probe's goal credence in event $E$. Regardless of which of $f$ or $f^o$ we choose as a dissimilarity function the expected value of the incoherence becomes approximately, using the quadratic approximation given by Lemmas \ref{lem:burns_f2} and \ref{lem:burns_fo2}: $$\mathbb E[L^*((q_{E}, q_{E^c}))]\approx \frac{\mathbb E[(q_{E} + q_{E^c} - 1)^2]}{4p(1-p)}.$$ Noting $q_{E} + q_{E^c} - 1$ has mean $0$ and is the sum of two independent random variables with approximate variances $\sigma'(\sigma^{-1}(p))^2S^2=\sigma'(\sigma^{-1}(1-p))^2S^2$ by the $\delta$-method, this becomes $$\mathbb E[L^*((q_{E}, q_{E^c}))]\approx \frac{2 S^2}{4p(1-p)}\sigma'(\sigma^{-1}(p))^2=\frac{S^2p(1-p)}{2}.$$ As this is minimized when $p$ is close to $0$ or $1$, the incoherence term by itself incentivizes the probe to be decisive. \end{itemize} \section{The Polytope of Coherent Beliefs} \label{sec:inequalities} \noindent Here, we explore the shape of the space of coherent beliefs. We first describe it as the convex hull of a finite set of $0,1$-vectors in Lemma \ref{lem:convex} and end by describing the inequalities that bound this polytope in Theorem \ref{thm:restatement}. \\\\ Given a set of events $\mathtt E$ with beliefs $\bec q$, let $\textbf E_i\in \set{0, 1}^N$ be the vector whose $j$th entry is $1$ if $\omega_j\in E_i$ and $0$ otherwise. Then, we can encode the structure of events $\mathtt E$ in a matrix $V$ whose $i$th row is the vector $\bec E_i$. Because $V$ contains the same information as $\mathtt E$, it will be convenient to use the pair $(V, \bec q)$ to represent the credence base $(\mathtt E, \bec q)$. We also encode a probability distribution $\pi$ on $\Omega$ with the vector $\bec \pi \in [0, 1]^N$ whose $j$th entry $\bec \pi \cdot \bec e_j = \pi(\set{\omega_j})$. \begin{definition} A vector $\bec\pi\in [0, 1]^N$ is a probability vector if $\sum_{i=1}^N \pi_i=1$. \end{definition} \begin{lemma} \cite{capotorti_correction_2010} A credence base $\mathcal{Q}=(V, \bec q)$ is coherent if and only if $\boldsymbol q$ is in the convex hull of the columns of $V$. \label{lem:convex} \end{lemma} \begin{proof} By definition $\mathcal Q$ is coherent if and only if there is a probability vector $\bec\pi \in [0, 1]^{N}$ where $V \bec \pi = \boldsymbol{ q}$. If $V_i$ is the $i$th column of $V$, that is true if and only if $$\boldsymbol q = \sum_{i=1}^{N} \pi_i \boldsymbol V_i.\qedhere$$ \end{proof} \noindent For example, if we receive three beliefs: one about the probability of it being warm, one about the probability of it raining, and one of the probability of it being warm and raining, then the matrix $V$ becomes $$\begin{pmatrix} 1 & 1 & 0 & 0\\ 1 & 0 & 1 & 0\\ 1 & 0 & 0 & 0 \end{pmatrix}.$$ So the polytope of coherent beliefs is the tetrahedron depicted in Figure \ref{fig:poytopeVisual}. \begin{figure}[h] \begin{subfigure}[b]{.4\textwidth} \centering \includegraphics[width=\linewidth]{polytope_view2.png} \end{subfigure} \begin{subfigure}[b]{.5\textwidth} \begin{itemize}[leftmargin=*] \item The facet not containing vertex $(1, 1, 1)$ corresponds to the inequality $P(\text{warm and rainy})\ge 0$. \item The facets not containing either $(1, 0, 0)$ or $(0, 1, 0)$ correspond to the inequalities $P(\text{warm})\ge P(\text{warm and rainy})$ and $P(\text{rainy})\ge P(\text{warm and rainy})$. \item The facet not containing vertex $(0, 0, 0)$ corresponds to the inequality ${1 + P(\text{warm and rainy}) \ge P(\text{warm}) + P(\text{rainy})}$. \end{itemize} \end{subfigure} \caption{The polytope of coherent beliefs about $P(\text{warm}), P(\text{rainy})$, and $P(\text{warm and rainy})$ and the inequalities corresponding to its facets as described in Theorem \ref{thm:restatement}.} \label{fig:poytopeVisual} \end{figure} \noindent \\Let $\bar V$ be the $(n+1)\times N$ matrix obtained by appending a final row of $1$s to $V$. Similarly, take $\bar{\textbf q}\in [0, 1]^{n+1}$ to have the same first $n$ entries as $\textbf q$ and $n+1$th entry $1$ and $E_{n+1}=\bec 1$, the vector whose entries are all $1$s. These represent the fact that $\pi(\Omega)=1$. For two vectors $\bec v, \bec w$, write $\textbf u \geq \bec v$ if $u_i\ge v_i$ for all $i$. Finally, let $O_+$ be $\R_{\ge 0}^N$ and for a matrix $M$, let $\rsp{M}$ denote the row span of $M$. \\\\ The following is a statement of the Dutch Book Theorem \cite{dutch_book}. \begin{lemma} The following are equivalent: \begin{enro} \item $\mathcal{Q}$ is coherent. \item For all $(a_1, ..., a_{n+1})\in \R^{n+1}$, if $\sum_{i=1}^{n+1}a_i\boldsymbol E_i\geq \boldsymbol 0$, then $\sum_{i=1}^{n+1}a_i \bar q_i\ge 0$. \end{enro} \label{lem:qcoherent} \end{lemma} \begin{proof} Suppose $(ii)$. By Farkas' Lemma \cite{Farkas}, as there is no $\bec a \in \R^{n+1}$ with all entries in $\bar V^T \bec a$ non-negative and $\bec a \cdot \bec{ \bar q} < 0$, it must be that there is a solution $\bec \pi \in O_+$ to $\bar V \bec \pi = \bec{\bar q}$, with the row of $1$s in $\bar V$ ensuring that $\bec \pi$ is a probability vector. \\\\ For the converse, consider $\mathcal{Q}$ coherent. Then, there exists $\boldsymbol \pi \in [0, 1]^N$ where $\bec E_i\cdot \boldsymbol \pi = \bar q_i$ for all $1\le i \le n+1$. But, if $\bar V^T \boldsymbol a \ge \bec 0$, then $$\boldsymbol a \cdot \boldsymbol{\bar q}=\boldsymbol a \cdot (\bar V \boldsymbol \pi)=(\bar V^T \boldsymbol a) \cdot \boldsymbol \pi\ge0$$ as the dot product of two non-negative vectors. \end{proof} \begin{lemma} Suppose that $\bar{V}^T\boldsymbol a=\boldsymbol 0$ for some non-zero vector $\boldsymbol a \in \R^{n+1}$ where $\boldsymbol{\bar q} \cdot \boldsymbol a \neq 0$. Then, $\mathcal{Q}$ is incoherent. \label{lem:ConsistencyOnRemoval1} \end{lemma} \begin{proof} If $\boldsymbol{\bar q} \cdot \boldsymbol a<0$, then we have $\bar V^T\boldsymbol a =\bec 0 \ge \bec 0$ but $\bec{\bar q} \cdot \boldsymbol a<0$. By Lemma \ref{lem:qcoherent}, $\mathcal{Q}$ is incoherent. \\\\ If $\boldsymbol {\bar q} \cdot \boldsymbol a>0$, then $\bec{\bar q}\cdot -\bec a<0$, so $\mathcal{Q}$ is incoherent. \end{proof} \begin{lemma} Suppose that $\bar{V}^T\boldsymbol a=\boldsymbol 0$ for some non-zero vector $\boldsymbol a \in \R^{n+1}$ where $\boldsymbol{\bar q} \cdot \boldsymbol a = 0$. For any $1\le i \le n$ where $a_i\ne 0$, we can remove the $i$th entry of $\boldsymbol{ q}$ and the $i$th row from $ V$ to obtain $\boldsymbol{ r}$ and $ W$ so that the credence base $(\boldsymbol r, W)$ is coherent if and only if $\mathcal{Q}$ is coherent. \label{lem:ConsistencyOnRemoval2} \end{lemma} \begin{proof} Without loss of generality, $\boldsymbol E_1 = \sum_{i=2}^{n+1} c_i \boldsymbol E_i$ and $q_1=\sum_{i=2}^{n+1} c_i q_i$. Call this vector $\boldsymbol c\in R^n$. Consider removing the first entry of $\boldsymbol q$ and the first row of $ V$ to yield $\boldsymbol r$ and $W$. \\\\ For any $\boldsymbol a$ where $\bar V^T \boldsymbol a \ge \bec 0$, if $\boldsymbol a_p$ is is the vector $\boldsymbol a$ with the first entry removed, then $\bar V^T \boldsymbol a = \bar W^T(\boldsymbol a_p + \boldsymbol c)$. Then, also $\boldsymbol{\bar q} \cdot \boldsymbol a = \boldsymbol{\bar r} \cdot (\boldsymbol a_p+\boldsymbol c)$. Therefore, if $(\boldsymbol r, W)$ is coherent, then $\mathcal{Q}$ is coherent. \\\\ If $(\boldsymbol r, W)$ were incoherent, then $\sum_{i=2}^{n+1}c_i \boldsymbol E_i\geq 0$ but $\sum_{i=2}^{n+1}c_i q_i< 0$, so $\mathcal{Q}$ is also incoherent. \end{proof} \noindent Based on Lemmas \ref{lem:ConsistencyOnRemoval1} and \ref{lem:ConsistencyOnRemoval2}, from here on, we will only consider $\bar V$ of rank $n+1$. Then, as the rows of $\bar V$ are linearly independent, the function $Q:\rsp{\bar V}\rightarrow \R$ where $Q(\boldsymbol o)=\boldsymbol a \cdot \boldsymbol{\bar q}$ if $\bar V ^T \boldsymbol a = \boldsymbol o$ is well-defined. By Lemma \ref{lem:qcoherent}, $\mathcal{Q}$ is coherent if and only if $Q(\boldsymbol o)\geq 0$ for all $\boldsymbol o \in \rsp{\bar V}\cap O_+$. \\\\ Vectors $\bec a\in \R^{n+1}$ can be thought of as ``bets", where the payout of the bet in world $\omega$ is equal to $$\sum_{i=1}^{n+1} a_i 1_{E_i}(\omega)$$ where $E_{n+1}=\Omega$. We consider the bets to be against a bookmaker with credence base $\mathcal Q$ who expects no edge. \\\\ For such a bet $\bec a$, the vector $\bec b = \bar V^T \bec a\in \R^N$ represents the payout of $\bec a$ atomwise: if atom $\omega_j$ happens, then the payout of bet $\bec a$ will be $b_j=\bec e_j \cdot (\bar V^T \bec a)$. Because we assume that $\bar V$ has full rank, there is a bijection between bets $\bec a\in \R^n$ and atomwise payouts $\bec b\in \rsp{\bar V}$. Note that it does not make sense to talk about atomwise payouts outside of $\rsp{\bar V}$ as it is impossible to make bets with such payouts. \\\\ For finding the coherence of a credence base, it is somewhat more helpful to think in terms of atomwise payouts rather than bets. For instance, if all entries of an atomwise payout $\bec b$ are non-negative, then, if $\mathcal Q$ is coherent, the cost $Q(\bec b)$ of the bet should be non-negative. The Dutch Book Theorem \ref{lem:qcoherent} tells us that having this condition for all atomwise payouts $\bec b\in \rsp{\bar V}$ is equivalent to coherence. Recalling that the set of coherent beliefs is a polytope, each bet whose atomwise payout is non-negative corresponds to an inequality that all elements of the polytope satisfy, and if a belief is outside the polytope, it violates an inequality corresponding to some facet, which is a bet that can be made. The following lemma allows us to just consider a positive spanning set $B$ of atomwise payouts that need to be checked to ensure coherence. \begin{definition} A \textit{positive spanning set} of a set $S\subseteq \R^N$ is a subset $B\subseteq S$ with the property that for any $\boldsymbol s \in S$, there exist coefficients $c_{\boldsymbol b}\in \R_{\ge 0}$ for each $\boldsymbol b \in B$ where only a finite number of $c_{\bec b}$ are non-zero and $$\boldsymbol s = \sum_{\boldsymbol b \in B} c_{\boldsymbol b}\boldsymbol b.$$ \end{definition} \begin{lemma} Let $B$ be a positive spanning set of $\rsp{\bar V}\cap O_+$. Then, $\mathcal{Q}$ is coherent if and only if $Q(\boldsymbol b)\geq 0$ for all $\boldsymbol b \in B$. \end{lemma} \begin{proof} If $Q(\boldsymbol b)<0$, for some $\boldsymbol b \in B$, then $\mathcal{Q}$ is incoherent by Lemma \ref{lem:qcoherent}. \\\\ Suppose that $Q(\boldsymbol b)\ge 0$ for all $\boldsymbol b \in B$. Then, consider any $\boldsymbol o\in \rsp{\bar V}\cap O_+$. As $B$ is a positive spanning set, we can represent $$\boldsymbol o =\sum_{\boldsymbol b \in \mathcal{B}} c_{\boldsymbol b}\boldsymbol b.$$ As $Q$ is linear, $$Q(\boldsymbol o)=\sum_{\boldsymbol b \in \mathcal{B}} c_{\boldsymbol b}Q(\boldsymbol b)\ge 0$$ as the sum of the product of non-negative reals. Therefore, $\mathcal{Q}$ is coherent. \end{proof} \begin{definition} The set of \textit{extremal vectors} of $\rsp{\bar V}\cap O_+$ is $$M:=\set{\boldsymbol b\in \rsp{\bar V}\cap O_+:\forall \boldsymbol o \in \rsp{\bar V}\cap O_+, \bec o\le \bec b \implies \boldsymbol o = \lambda \boldsymbol b \text{ for some $\lambda \in \R$}}.$$ \end{definition} \noindent Let $O_1=\set{\boldsymbol o \in \rsp{\bar V}\cap O_+:\text{the first non-zero entry of }\boldsymbol o \text{ is 1}}$ where we fix an arbitrary ordering of the finite number of entries of $\bec o$. We will show that $M\cap O_1$ is a minimal positive spanning set of $\rsp{\bar V} \cap O_+$. \\\\ \noindent If we think of each positive spanning set as a set of atomwise payouts that the bookmaker has to check is positive to ensure coherence, the following gives some atomwise payouts that the bookmaker definitely has to check. \begin{lemma} Let $B$ be a positive spanning set of $\rsp{\bar V} \cap O_+$ where $B\subseteq O_1$. Then, $$O_1\cap M\subseteq B.$$ \end{lemma} \begin{proof} If $B$ did not include such a $\boldsymbol b\in O_1\cap M$, as $B$ is a positive spanning set, then there is a finite set $S$ where $\boldsymbol b = \sum_{s\in S} c_s\boldsymbol b_s$ for some $c_s\geq 0$. But as not all $c_s = 0$, without loss of generality, suppose $c_1\neq 0$. Then, $\boldsymbol b - c_1\boldsymbol b_1\geq 0$, so $\boldsymbol b = \lambda \boldsymbol b_1$. As the first non-zero entry of each is $1$, $\boldsymbol b = \boldsymbol b_1$. \end{proof} \begin{definition} Let $A$ be a subset of $\R^n$. Consider $Z:A\rightarrow 2^{\set{1, \dots, n}}$ where $Z(\boldsymbol o)=\set{i:o_i=0}$. A vector $\boldsymbol v$ is \textit{maximally-$0$} in $A$ if there there is no $\boldsymbol w\in A\symbol{92}\set{\boldsymbol 0}$ where $Z(\boldsymbol v)\subsetneq Z(\boldsymbol w)$. \end{definition} \begin{lemma} For any vector $\boldsymbol o$ that is maximally-$0$ in $\rsp{\bar V}$, if $Z(\boldsymbol v)=Z(\boldsymbol o)$ for some vector $\boldsymbol v\in \rsp{\bar V}$, then $\boldsymbol v \in \rsp{\boldsymbol o}$. \label{lem:z=maximally0} \end{lemma} \begin{proof} Suppose that $\boldsymbol v \notin \rsp{\boldsymbol o}$. Let the first non-zero entry of $\boldsymbol o$ be $\lambda$ and the first non-zero entry of $\boldsymbol v$ be $\mu$. Then, $\boldsymbol 0\neq\boldsymbol o - \frac \lambda \mu \boldsymbol v$ and $Z(\boldsymbol o) \subsetneq Z(\boldsymbol o-\frac \lambda \mu\boldsymbol v)$. \end{proof} \noindent It turns out that it suffices to just check the maximally-0 elements of $\rsp{\bar V}$, as shown by the following to lemmas. \begin{lemma} A vector $\boldsymbol o \in \rsp{\bar V}\cap O_+$ is maximally-$0$ in $\rsp{\bar V}$ if and only if $\boldsymbol o \in M$\label{lem:MequivMax0}. \end{lemma} \begin{proof} Suppose that $\boldsymbol o \not \in M$. Then, there exists $\boldsymbol v \in \rsp{\bar V}\cap O_+\symbol{92}\rsp{\boldsymbol o}$ where $\boldsymbol o - \boldsymbol v\geq 0$. Then, $Z(\boldsymbol o) \subseteq Z(\boldsymbol v)$, so by Lemma \ref{lem:z=maximally0}, $\boldsymbol o$ is not maximally-0 in $\rsp{\bar V}$. \\\\ Suppose that $\boldsymbol o\in M$. If $\boldsymbol o$ were not maximally-$0$ in $\rsp{\bar V}\cap O_+$, then we would have $\boldsymbol v\in \rsp{\bar V}\cap O_+\setminus\set{\bec 0}$ with with $Z(\bec o) \subsetneq Z(\bec v)$. Let $v_{\uparrow}$ be the maximum entry of $\boldsymbol v$ and $o_{\downarrow}$ be the minimum entry of $\boldsymbol o$. Then, $\boldsymbol o - \frac{o_{\downarrow}}{v_{\uparrow}}\boldsymbol v \geq \bec 0$, which is impossible as $\boldsymbol v$ cannot be in the span of $\boldsymbol o$ by the definition of $M$. Therefore, $\bec o$ is maximally-$0$ in $\rsp{\bar V}\cap O_+$ by contradiction. \\\\ Next, it suffices to show that if $\bec o$ is maximally-$0$ in $\rsp{\bar V}\cap O_+$, then $\bec o$ is maximally-$0$ in $\rsp{\bar V}$. To see this, suppose not and that there exists some $\boldsymbol v\in \rsp{\bar V}\symbol{92}\set {\boldsymbol {0}}$, where $Z(\boldsymbol o)\subsetneq Z(\boldsymbol v)$. Then, $\boldsymbol h =\boldsymbol o - \frac{o_{\downarrow}}{v_{\uparrow}}\boldsymbol v \geq \bec 0$ and $Z(\boldsymbol v)\subseteq Z(\boldsymbol h)$. But $\bec h\in \rsp{\bar V}\cap O_+$ and we have $Z(\boldsymbol o)\subsetneq Z(\boldsymbol v)\subseteq Z(\boldsymbol h)$, which contradicts the fact that $\bec o$ is maximally-$0$ in $\rsp{\bar V}\cap O_+$. \end{proof} \begin{lemma} $O_1\cap M$ is a positive spanning set of $\rsp{\bar V} \cap O_+$. \end{lemma} \begin{proof} For the sake of contradiction, suppose that $O_1\cap M$ is not a positive spanning set and consider $\boldsymbol o\in\rsp{\bar V} \cap O_+$ to be a maximally-$0$ vector not its positive span. As $\boldsymbol o$ is not in the span of any element of $O_1\cap M$, $\boldsymbol o$ is not maximally-$0$ in $\rsp{\bar V}$. Therefore, there exists $0\neq \boldsymbol v \in \rsp{\bar V}$ where $Z(\boldsymbol o)\subsetneq Z(\boldsymbol v)$. For all non-zero elements of $\boldsymbol v$, consider $r_i=\frac{o_i}{v_i}$ and let $r=\min\set{r_i:r_i>0}$. Then, $\boldsymbol w = \boldsymbol o - r \boldsymbol v\in \rsp{\bar V} \cap O_+$, so as $Z(\boldsymbol o) \subsetneq Z(\boldsymbol w)$, $\boldsymbol w$ is in the positive span of $B$. For $w_i\neq 0$, let $s_i=\frac{o_i}{w_i}$ and $s = \min \set{s_i}$. Then, $\boldsymbol o - s \boldsymbol w \in \rsp{\bar V} \cap O_+$ and $Z(\boldsymbol o)\subsetneq Z(\boldsymbol o - s \boldsymbol w)$. Therefore, $\boldsymbol o - s \boldsymbol w$ and $\boldsymbol w$ are both elements of the positive span of $O_1\cap M$, so $\boldsymbol o$ must also be an element of the positive span of $O_1\cap M$. \end{proof} \begin{theorem} The hyperplane $\set{\bec x\in \R^n : \bec a \cdot \bec x = c}$ is a facet of the polytope whose vertices are the columns of $V$ if and only if $$\bar V^T \begin{pmatrix} \bec a\\ -c \end{pmatrix}\in M.$$ \label{thm:restatement} \end{theorem} \begin{proof} By \cite{Ziegler2000}, the hyperplane $\set{\bec x : \bec a \cdot \bec x = c}$ is a facet of a polytope $P$ if and only if for all vertices $\bec v$ of $P$, $\bec a \cdot \bec v \ge c$ with equality for a maximal subset of vertices. This happens if and only if the vector $$V^T \bec a \ge c\bec 1$$ with equality for a maximal subset of the entries, which is true if and only if $$\bec b :=\bar V^T \begin{pmatrix} \bec a\\ -c \end{pmatrix}\ge \bec 0$$ and is maximally-$0$ among vectors in $\rsp{\bar V}$. By Lemma \ref{lem:MequivMax0}, this is equivalent to $\bec b\in M$. \end{proof} \begin{corollary} There is a one to one correspondence between elements of $O_1 \cap M$ and facets of the polytope whose vertices are the columns of $V$. \end{corollary} \noindent By \cite{Ziegler2000}, every non-redundant set of inequalities bounding a polytope $P$ has exactly one inequality for every face. This means that if a bookmaker verifies that they are not giving a Dutch book by checking a fixed set of inequalities, it suffices to check the inequalities $Q(\bec b)\ge 0$ for all $\bec b\in M\cap O_1$, and every non-redundant set of inequalities they must check consists of rescaled versions of these inequalities. \section{Merging Individually Coherent Experts} \label{sec:indCohExp} \noindent In this section we consider a setting in which each of several experts submits a set of internally coherent credences, but the union of all expert credences is possibly incoherent. To aggregate these credences into a single coherent set of beliefs, we could use a loss function of the form (\ref{eqn:lossfunction}), but that ignores the additional information that beliefs from the same expert are coherent. Since logical inferences can be made from an expert's stated credences, the expert can express the same information in multiple ways. As long as the set of possible inferences about an expert's beliefs is the same, the loss function should be invariant under the specific information the expert shares, a property that the method for separate beliefs previously discussed does not have if naively applied here. We call this property \textit{content invariance} and discuss it further below. \\\\ For example, if a meteorologist says that there is a $40\%$ chance of rain tomorrow and a $70\%$ chance of clouds, it is reasonable to infer that there is a $30\%$ chance of sun and a $30\%$ chance of clouds without rain. In such a situation, the meteorologist might also explicitly say the chance of it being sunny, however, because that statement does not provide new information, the loss function should be invariant under whether or not they say it. \\\\ Recall that $\bar V$ obtained by appending a row of $1$s to $V$ and that $\rsp{\bar V}$ is the row span of $\bar V$. \begin{definition} Two coherent credence bases $(V, \bec q)$ and $(V', \bec q')$ are \textit{content equivalent} if $\rsp{\bar V}=\rsp{\bar V'}$ and the credence base $((V, V'), (\bec q, \bec q'))$ is coherent. \end{definition} \begin{lemma} If $(V, \bec q)$ and $(V', \bec q')$ are content equivalent and $\bec \pi$ is a probability vector, then $$V\bec \pi = \bec q \iff V'\bec \pi = \bec q'.$$ \label{lem:piSameContent} \end{lemma} \begin{proof} Since $\ker(\bar V) = \rsp{\bar V}^\perp$, the orthogonal space to the row span of $\bar V$, if $\rsp{\bar V}=\rsp{\bar V'}$, then $\ker(\bar V)=\ker(\bar V')$. By the coherence of $((V, V'), (\bec q, \bec q'))$, there is some $\bec{\tilde\pi}$ so that $V\bec{\tilde\pi}=\bec q$ and $V'\bec{\tilde\pi}=\bec q'$. Then, for any probability vector $\bec \pi$, as $\bec\pi -\bec{\tilde\pi}\in \ker(\bec 1^T)$, $$V\bec \pi = \bec q \iff \bec \pi - \tilde{\bec \pi}\in \ker(\bar V)\iff \bec \pi - \tilde{\bec \pi}\in \ker(\bar V')\iff V'\bec \pi = \bec q'.\qedhere$$ \end{proof} \noindent Lemma \ref{lem:piSameContent} implies that content equivalence is transitive, so is an equivalence relation. \begin{definition} Let $\phi$ be a function of a credence base. $\phi$ is \textit{content invariant} if for content equivalent credence bases $\mathcal{Q}$ and $\mathcal{Q'}$ we have $\phi(\mathcal{Q})=\phi(\mathcal{Q}')$. \end{definition} \subsection{Content Invariance}\label{subsec:contentInvariance} Suppose that $(V, \bec q)$ is a coherent credence base where the $n+1$ rows of $\bar V$ are linearly independent. Let $I=\rsp{\bar V}\cap \set{0, 1}^N$, which the next lemma shows is the set of events whose probabilities are linearly inferable. \begin{lemma} Consider the statements \begin{enro} \item $\bec E\in I$. \item There is a unique belief $q$ so that the credence base $((\bec E, V), (q, \bec q))$ is coherent. \end{enro} (i) implies (ii). Moreover, if there is a probability vector $\bec \pi$ so that $V \bec \pi = \bec q$ where none of the entries of $\bec \pi$ are $0$ or $1$, then (ii) implies (i). \end{lemma} \begin{proof} Suppose (i). The belief $q=\bec E \cdot \bec \pi$ makes $((\bec E, V), (q, \bec q))$ coherent as $(V, \bec q)$ is coherent. For uniqueness, suppose we there are two coherent beliefs $q, q'$ about some event for $\bec E\in I$. Then as there is some $\bec a\in \R^n$ where $V^T \bec a = \bec E$, there are two probability vectors $\bec \pi$ and $\bec \pi'$ where $q = (V^t \bec a) \cdot \bec \pi$ and $q' = (V^t \bec a) \cdot \bec \pi'$ with $\bec q = V \bec \pi = V \bec \pi'$. This means that $$q=(V^t \bec a) \cdot \bec \pi = \bec a^T V \bec\pi = \bec a^T \bec q = \bec a^T V \bec\pi' = q'.$$ Suppose not (i) and let $\bar V'$ be the matrix $\bar V$ with the vector $\bec E$ appended as the last row. Then, there is a vector $\bec v\in \ker(\bar V)\symbol{92}\ker(\bar V')$. By the assumption that no entries of $\bec \pi$ are $0$ or $1$, there is an $\epsilon>0$ so that for all $|a|<\epsilon$, $\bec \pi + a\bec v$ is also a probability vector. Then, for all such $a$, taking $q=\bec E \cdot (\bec \pi + a\bec v)$ makes $((\bec E, V), (q, \bec q))$ coherent. As $\bec v \in \ker(\bar V)\symbol{92}\ker(\bar V')$, we have $\bec E\cdot \bec v\neq 0$, so each distinct value of $a$ yields a distinct $q$. \end{proof} \begin{lemma} Let $R$ be the reduced row-echelon form of $\bar V$. Then, $$I=\set{R^T \boldsymbol v:\boldsymbol v \in \set{0, 1}^{n+1}, R^T \boldsymbol v \in \set{0, 1}^N}.$$ \end{lemma} \begin{proof} Firstly, $$\set{R^T \boldsymbol v:\boldsymbol v \in \set{0, 1}^{n+1}, R^T \boldsymbol v \in \set{0, 1}^N}\subseteq \rsp{\bar V}\cap\set{0, 1}^N=I.$$ Note that all of $R^T$'s columns have a leading $1$ which is the only entry in its row, so each of $\boldsymbol v$'s entries appear somewhere in $R^T\boldsymbol v$. Therefore, if $R^T\boldsymbol v \in \set{0, 1}^N$, then $\boldsymbol v \in \set{0, 1}^{n+1}$. \end{proof} \begin{corollary} The maximum size of $I$ is $2^{n+1}$. \end{corollary} \noindent Because $I$ is defined only in terms of the row span $\rsp{\bar V}$, it is content invariant. However, $I$ tends to be quite large even for well-tamed events, so a smaller, content invariant set might be desirable. \\\\ Let $B\subseteq I$ be a minimal positive spanning set of $I$. \begin{lemma} Any maximally-$0$ element of $I$ must be part of $B$. \end{lemma} \begin{proof} Suppose that $\bec v\in I$ is maximally-$0$ in $I$. As $B$ is a positive spanning set, we can write $$\bec v = \sum_{\bec b\in B} c_{\bec b} \bec b.$$ As $\bec v\ne \bec 0$, there is some $\bec b\in B$ where $c_{\bec b}\ne \bec 0$, so $\bec b \le \bec v$ and $Z(\bec b)\subseteq Z(\bec v)$. As $\bec v$ is maximally-$0$, $Z(\bec b)= Z(\bec v)$ which means that $\bec v = \bec b$ as both are $0,1$-vectors. \end{proof} \begin{theorem} \label{thm:Bmax0}$B$ is the set of maximally-$0$ elements of $I$. \end{theorem} \begin{proof} It suffices to show that the maximally-$0$ elements of $I$ are a positive spanning set. Suppose that this were not the case. Let $\boldsymbol v$ be a maximally-$0$ vector in $I\symbol{92}\rsp{B}_+$, the maximal set positively spanned by $B$. Note that $\boldsymbol v$ is not maximally-$0$ in $I$ or it would be a member of $B$. Then, there exists $\boldsymbol w\in I\symbol{92}\set{0}$ where $Z(\boldsymbol v)\subsetneq Z(\boldsymbol w)$. Note that $Z(\boldsymbol v)\subsetneq Z(\boldsymbol v-\bec w)$, meaning $\boldsymbol w$ and $\boldsymbol v-\boldsymbol w$ are both in the positive span of $B$, so $\boldsymbol v$ is also in the positive span of $B$. \end{proof} \begin{lemma} Let $Q:I \rightarrow [0, 1]$ be the implied belief function, taking an event $\bec E$ to the unique belief $q$ that makes the credence base $((\bec E, V), (q, \bec q))$ coherent. The function $Q$ as well as the sets $I$ and $B$ are content invariant. \label{lem:I_content_invariant} \end{lemma} \begin{proof} Let $(V, \bec q)$ and $(V', \bec q')$ be two content equivalent coherent credence bases with set of inferable events $I=\rsp{\bar V} \cap \set{0, 1}^N$ and $I' = \rsp{\bar V'} \cap \set{0, 1}^N$, positive bases $B$ and $B'$ of $I$ and $I'$, and implied belief functions $Q$ and $Q'$. Then, by definition, $\rsp{\bar V} = \rsp{\bar V'}$, so $I=\rsp{\bar V} \cap \set{0, 1}^N=\rsp{\bar V'} \cap \set{0, 1}^N=I'$ is content invariant. Also, because $B$ and $B'$ are defined solely in terms of $I$ and $I'$ respectively, $B=B'$. \\\\ To show that $Q$ is content invariant, if $Q(\bec E) = q$, then by coherence, there is a probability vector $\bec \pi$ so that $V\bec \pi = \bec q$ and $\bec E \cdot \bec \pi = q$. By Lemma \ref{lem:piSameContent}, we have $V' \bec \pi = \bec q'$, meaning that $\bec \pi$ certifies that $((\bec E, V'), (q, \bec q'))$ is coherent, so by uniqueness, $Q'(\bec E) = q$. \end{proof} \subsection{Loss Functions with Content Invariance} Suppose that there are $k$ experts, the $i$th of which tells us the coherent credence base of their beliefs $\mathcal E_i = (V_i, \bec q_i)$, with event matrix $V_i$ and belief vector $\bec q_i$ defined previously. Let $\mathcal Q$ be the combined credence base of all experts, formed by concatenating $V_1, \dots, V_k$ and $\bec q_1, \dots, \bec q_k$. We will consider loss functions of the form $$L(\bec p, \mathcal{Q})=\sum_{i=1}^{k} \mathcal D_i(\bec p, \mathcal{E}_i)$$ for some content invariant measure of disagreement $\mathcal D_i$. In contrast to Section \ref{sec:seperateBeliefs}, the sum is over experts rather than over individual events. As before, $$L^*(\mathcal{Q})=\min_{\bec p \in C(\mathtt E)}L( \mathcal{Q})=L(\bec{p^*}, \mathcal Q)$$ is a measure of the incoherence of the set of experts as a group. \\\\ For the $i$th expert, let $I_i=\rsp{\bar V_i} \cap \set{0, 1}^N$ be the set of events for which we can infer a probability from expert $i$'s stated beliefs, let $Q_i:I_i\rightarrow [0, 1]$ be this inferred probability, and let $B_i$ be the minimal positive spanning set of $I_i$, as defined in Section \ref{subsec:contentInvariance}. Also, write $P(\bec E)$ be the entry of $\bec p$ corresponding to event $E$. \\\\ As $Q_i$ is content invariant one approach is to let \begin{equation} \mathcal D_i = D_{i, S_i} := \frac{1}{\#S_i} \sum_{\bec b\in S_i} \ell(P(\bec b), Q_i(\bec b))\label{eqn:contentinvariantloss} \end{equation} where $S_i\subseteq I_i$ is a content invariant set of events and $\ell$ is a dissimilarity function as in Section \ref{sec:seperateBeliefs}. As $I_i$ and $B_i$ are both content invariant, they are both choices for $S_i$. \\\\ One possible justification for summing over $B_i$ instead of $I_i$ is that $\#I_i$ can be much larger than $\#B_i$. $B_i$ contains the maximally-$0$ elements of $I_i$, or the elements whose probabilities are smallest. As the dissimilarity functions $f$ and $f^o$ punish more for the same error when $q$ is smaller ($f(0.101, 0.001)>f(0.6, 0.5)$), summing over $B_i$ is summing over the elements of $I_i$ whose relative errors will be greatest while not considering the other terms for ease of computation. \\\\ The normalizing coefficient $\frac{1}{\#S}$ is included in the loss function in order to give each expert equal weight, regardless of how many beliefs they express. If the measures of disagreement were not normalized, then an expert might become twice as influential on the overall loss function by expressing one additional belief because $\#I_i$ and $\#B_i$ can grow exponentially with the number of beliefs expressed. \subsection{Removing the Symmetry between an Event and its Complement} \noindent Looking at the dissimilarity function $f$, there are two terms: $p\ln \frac pq$ representing the expected surprise from the event occurring and $(1-p)\ln \frac{1-p}{1-q}$, representing the expected surprise from the event not occurring. However, if we sum the dissimilarity function over a minimal positive basis, then being surprised by the complement of an event happening is redundant; when one event does not happen, we are surprised instead by other events happening. Alternatively, an agent observing the world might only keep track of which events do happen, so cannot be surprised by an event not happening. \\\\ This motivates the removal the term representing the surprise of an event not happening from the loss function. However, one needs to be careful in doing this: so far, we have required that loss functions $L(\bec p, \bec q)$ be non-negative and equal to $0$ if and only if $\bec p = \bec q$. For example, $p\ln \frac pq$ by itself is not a dissimilarity function, so the corresponding ``loss function" $L$ does not have this property in general. The following lemmas motivates two possible content invariant disagreement functions that have this property: \begin{lemma} Let $(V, \bec p)$ and $(V, \bec q)$ be coherent credence bases and suppose that $V^T \bec 1=k\bec 1$ for some $k\in \mathbb N$. Then, $$\sum_{i=1}^n p_i \ln \frac{p_i}{q_i}\ge 0$$ with equality if and only if $\bec p = \bec q$. \label{lem:log-sum} \end{lemma} \begin{proof} By Lemma \ref{lem:ConsistencyOnRemoval1}, since $V^T \bec 1 - k\bec 1 = \bec 0$, we have $$\sum_{i=1}^n p_i - k = \sum_{i=1}^n q_i - k = 0.$$ Therefore, $\sum_{i=1}^n p_i = k = \sum_{i=1}^n q_i$, so applying the log-sum inequality, Theorem 2.7.1 of \cite{infoTheory}, implies that $$\sum_{i=1}^n p_i \ln \frac{p_i}{q_i}\ge \left(\sum_{i=1}^n p_i\right) \ln \frac{\sum_{i=1}^n p_i}{\sum_{i=1}^n q_i} = 0$$ with equality if and only if $\frac{p_i}{q_i}$ is constant. By coherence, this happens if and only if $\bec p = \bec q$. \end{proof} \noindent Recall from Section \ref{section:predd} that for a proper scoring rule $s(i, q)$, there is an associated dissimilarity function $\ell(p, q) = p(s(1, p) - s(1, q)) + (1-p)(s(0, p) -s(0, q))$. When using the log-loss scoring rule, this becomes $\ell = f$. Can we generalize Lemma \ref{lem:log-sum} to apply to half-dissimilarity functions \begin{equation}\label{eqn:tilde} \tilde{\ell}(p, q):=ps(1, p) - ps(1, q) \end{equation} for a proper scoring rule $s$? No. The following lemma shows that log-loss is the only differentiable proper scoring rule for which Lemma \ref{lem:log-sum} holds. \begin{lemma} Let $s(i, q)$ be a proper scoring rule that is differentiable in $q$ for $q\in (0, 1)$ with $\tilde \ell$ as defined in equation (\ref{eqn:tilde}). Suppose that for all coherent credence bases $(V, \bec q)$ where $V^T \bec 1=k\bec 1$ for some $k\in \mathbb N$, $$\bec q = \argmin_{\bec p \in C(V)} \sum_{i=1}^n \tilde \ell(p_i, q_i)$$ Then for some $\lambda < 0$ and $c\in \R$, $${s(i, q) = \lambda(i\ln q + (1-i)\ln (1-q)) + c}.$$\label{lem:loglossunique} \end{lemma} \begin{proof} Note that by continuity, the value of $s$ when $q\in \set{0, 1}$ are determined by its values on the interval $(0, 1)$. Write $$L(\bec p, \bec q) := \sum_{i=1}^n \tilde \ell(p_i, q_i)$$ Let $V$ be the $3\times 3$ identity matrix and let $\lambda = \frac 12 s'(1, \frac 12)$ where $s'$ is the derivative of $s$ in $q$. For any $0<q < \frac 12$, consider the coherent belief vector $\bec q = (q, \frac 12, \frac 12 - q)^T$. By the assumption that $L(\bec p, \bec q)$ is minimized among coherent $\bec p$ when $\bec p = \bec q$, Lagrange multipliers yield that $$c\bec 1 = \pderiv{L}{\bec p}(\bec q, \bec q) = \begin{pmatrix} \lambda \\ q s'(1, q) \\ (\frac{1}{2}-q) s'(1, \frac{1}{2}-q) \end{pmatrix}$$ so $q s'(1, q)=\lambda$ for all $0<q < \frac 12$. For $\frac 12 < q <1$, take $V$ to be the $2\times 2$ identity matrix and $\bec q = (q, 1 - q)$. The same argument as above shows that $q s'(1, q) = (1-q)s'(1, 1-q) = \lambda$. Therefore, for all $q\in (0, 1)$, we have $q s'(1, q) = \lambda$, so $s'(1, q) = \frac{\lambda}{q}$ and $s(1, q) = \lambda \ln q + c_1$ for some $c_1\in \R$. \\\\ As $s$ is a proper scoring rule, the quantity $p s(1, q) + (1-p)s(0, q)$ is uniquely minimized in $q$ when $p=q$, so $-(1-p) s'(0, p) = ps'(1, p) =\lambda$, meaning that $s'(0, p) = \frac{-1}{1-p}$ and $s(0, p) = \lambda \ln (1-q)+c_2$ for some $c_2\in \R$. Therefore, we have $$s(i, q) = \lambda(i\ln q + (1-i)\ln (1-q)) + c_1+c_2.$$ In order for $s$ to be a proper scoring rule, we must have $\lambda < 0$. \end{proof} \noindent Letting $O_i$ be the set of all subsets of $B_i$ whose sum is $\boldsymbol 1$ and $M_i=\sum_{S\in O_i} \#S$, we can use the dissimilarity function \begin{equation} \mathcal D_i = \tilde D_{i, B_i} = \frac{1}{M_i}\sum_{S \in O_i} \sum_{\boldsymbol b \in S}\tilde \ell(P(\bec b), Q(\bec b)).\label{eqn:assymetricvariant} \end{equation} Lemma \ref{lem:log-sum} implies that when using $\tilde \ell(p, q) = \tilde f(p, q) := p\ln \frac pq$ or $\tilde \ell(p, q) = \tilde f^o(p, q) := q\ln \frac qp$, the measure of disagreement $\tilde D_{i, B_i}$ is minimized when $P(\bec b) = Q_i(\bec b)$ for all $\bec b \in B_i$. \\\\ We compare the methods presented here in Appendix \ref{sec:specialcompare} as well as in Appendix \ref{sec:ex}. \color{black} \section*{Acknowledgements} We thank Kenny Easwaran, Jonathan Gabor, Oliver Hopcroft, David Krueger, Yuval Peres, Suvadip Sana, Luchen Shi, and Ariel Yadin for inspiring discussions. \pagestyle{plain} \printbibliography \pagestyle{headings} \appendix \section{Numerical Comparison between $f$ and $f^o$\label{sec:compare}} \noindent To contrast the two settings in Section \ref{sec:seperateBeliefs}: One should use the dissimilarity function $f$ when submitting multiple forecasts based on their possibility incoherent internal beliefs (and expects these forecasts to be scored with the logarithmic proper scoring rule); its transpose $f^o$ should be used in aggregating independent sources of information, each of which gives information about one event, in order to find the most likely true probability distribution. \\\\ Recall that $\textbf E_i\in \set{0, 1}^N$ is the vector whose $j$th entry is $1$ if $\omega_j\in E_i$ and $0$ otherwise. Additionally, $\textbf e_i$ denotes the $i$th standard basis vector and $\textbf 1$ the vector of all $1$s. For any distribution $\pi$, we associate a vector $\boldsymbol \pi$ with it, whose $j$th entry is $\pi(\omega_j)$, the probability of the $j$th atom in the Boolean algebra. \\\\ Table~\ref{fig:Lnumeric} contains some examples of the numerical differences between using $f$ and $f^o$ as dissimilarity functions. \begin{table}[h] \begin{center} \begin{tabular}{|m{12em} | m{10em}| m{9em} |} \hline Situation & \multicolumn{2}{c|}{$\bec{p^*}$} \\ \cline{2-3}& $\ell=f$ & $\ell=f^o$\\ \hline Exactly one event occurs: & & \\$\boldsymbol E_1=\boldsymbol e_1, q_1 = 0.1$ & $p_1=0.01$&$p_1=0.04$ \\$\boldsymbol E_2=\boldsymbol e_2, q_2 = 0.6$ & $p_2=0.11$&$p_2=0.30$ \\$\boldsymbol E_3=\boldsymbol e_3, q_3 = 0.99$ & $p_3=0.89$&$p_3=0.66$ \\\hline All estimates are for the same event: & & \\${\boldsymbol E_1 = \boldsymbol E_2=\boldsymbol E_3=(1 \, 0)^T}$ & & \\ $q_1 = 0.1, q_2=0.3, q_3=0.5$ &$p_1=p_2=p_3=0.27$ & $p_1=p_2=p_3=0.3$ \\\hline $\boldsymbol E_1 = (1 \,1 \,0 \,0)^T$, $q_1=0.99$ &$p_1=0.87$ & $p_1=0.73$ \\ $\boldsymbol E_2 = (1 \,0 \,1 \,0)^T$, $q_2=0.5$ &$p_2=0.40$ & $p_2=0.47$ \\ $\boldsymbol E_3 = (1 \, 0 \,0 \,0)^T$, $q_3=0.1$ &$p_3=0.27$ & $p_3=0.20$ \\ $\boldsymbol E_4 = (0 \,1 \,0 \,0)^T$, $q_4=0.4$ &$p_4=0.60$ & $p_4=0.53$ \\ $\boldsymbol E_5 = (0 \,0 \,1 \,0)^T$, $q_5=0.4$ &$p_5=0.13$ & $p_5=0.27$ \\\hline \end{tabular} \caption{Three examples of correcting incoherent probabilities with $f$ and $f^o$. By Theorem \ref{punique}, $\bec{p^*}$ is unique.} \label{fig:Lnumeric} \end{center} \end{table} \begin{figure}[h] \centering \begin{subfigure}[b]{0.4\textwidth} \centering \includegraphics[width=\textwidth]{pf} \caption{Using $\ell=f$} \label{fig:p:h=f} \end{subfigure} \hfill \begin{subfigure}[b]{0.4\textwidth} \centering \includegraphics[width=\textwidth]{pfo} \caption{Using $\ell=f^o$} \label{fig:p:h=fo} \end{subfigure} \hfill \caption{Contour plots of $p^*(\bec q)$ when $E_1=E_2$ are the events being estimated. Contour distance is 0.02. \label{fig:p*comparisons} } \end{figure} \\\\ The differences in $f$ and $f^o$ also leads the function $L^*$ to behave differently when using the two functions, as illustrated in Figure \ref{fig:L*comparisons}, where $E_1$ and $E_2=E_1^c$ are the events being estimated. \begin{figure}[h] \centering \begin{subfigure}[b]{0.4\textwidth} \centering \includegraphics[width=\textwidth]{Lf} \caption{Using $\ell=f$} \label{fig:L:h=f} \end{subfigure} \hfill \begin{subfigure}[b]{0.4\textwidth} \centering \includegraphics[width=\textwidth]{Lfo} \caption{Using $\ell=f^o$} \label{fig:L:h=fo} \end{subfigure} \hfill \caption{Contour plots of $L^*(\bec q)$ when $E_1$ and $ E_2=E_1^c$ are the events being estimated. Near the line $q_1+q_2=1$, both functions are approximately quadratic and can be approximated by $\frac{(q_1+q_2-1)^2}{(1-q_1+q_2)(1+q_1-q_2)}$. Also, when using $\ell=f$, $L^*$ is not bounded, while it is when using $\ell=f^o$. Contour distance is 0.1.} \label{fig:L*comparisons} \end{figure} \section{Computing \texorpdfstring{$\bec{p^*}$}{p*} and \texorpdfstring{$L^*$}{L*}\label{sec:findpstar}} \noindent One way to find $L^*$ and $\boldsymbol{p^*}$ numerically is to define a function that, given a vector $\boldsymbol \pi$ and the $n\times N$ matrix $V$ whose $i$th row is $\textbf E_i$, and the credences $\boldsymbol q$, returns $L(V\boldsymbol\pi, \boldsymbol q)$. Then, one can use a gradient descent algorithm to minimize $L(V\boldsymbol\pi, \boldsymbol q)$ under the conditions that the entries of $\boldsymbol \pi$ are nonnegative and sum to $1$. This algorithm is illustrated below in psuedocode.\\ \begin{algorithmic} \State $\bec {\pi_0} \gets \frac{1}{N} \bec{1}$ \Comment{Initial guess is that all atoms have equal probability} \State \Return gradientDescent($\bec \pi \mapsto L(V\boldsymbol\pi, \boldsymbol q)$,\Comment{The loss function in terms of $\bec \pi$}\\ \hspace{1.47in} $\bec{\pi_0}$, \Comment{Initial guess of $\bec \pi$}\\ \hspace{1.47in} $\bec \pi \cdot \bec 1 = 1$, $\bec 0 \le \bec \pi \le \bec 1$) \Comment{Conditions $\bec \pi$ must satisfy}\\ \end{algorithmic} Computing $\bec{p^*}(\bec q)$ to within an accuracy of $\epsilon$ takes $O\left(nN\log\left(\frac{N}{\epsilon}\right)\right)$ time \cite{bubeck_convex_2015}. Python code to compute $\bec{p^*}$ and $L^*$ can be found on \url{https://github.com/scim142/quantifying_coherence}. \section{Holder Continuity of $\bec{p^*}$ and $L^*$\label{sec:continuity}} \noindent Let $||\cdot ||$ denote the Euclidean norm. \begin{lemma} If $\ell$ is (strictly) convex/$n$-times differentiable/Lipschitz in $p$/$q$/both, then $L$ will also be. \label{lem:2.1} \end{lemma} \begin{proof} (convexity in $p$) Let $\ell_i=w_i\ell(\textbf p \cdot \textbf e_i, \textbf q \cdot \textbf e_i)$. Then, for $t\in[0, 1]$, $$\ell_i(t\textbf p + (1-t)\textbf p', \textbf q)=w_i \ell(tp_i+(1-t)p_i', q_i)$$ $$\le w_i t\ell(p_i, q_i)+w_i(1-t)\ell(p_i', q_i)=t\ell_i(\textbf p, \textbf q)+(1-t)\ell_i(\textbf p', \textbf q)$$ so $\ell_i$ is convex. As $L(\textbf p, \textbf q) = \sum_{i=1}^{n} \ell_i(\textbf p, \textbf q)$ is the sum of convex functions, $L$ is convex. \end{proof} \begin{lemma} \label{Lqsim} If $\ell$ is Lipchitz in $q$ in some region $D\subseteq [0, 1]^2$, then $L^*$ will be Lipchitz on the set $\bar{D}:=\set{\textbf q\in [0, 1]^n:(\boldsymbol{p^*}(\textbf q), \textbf q)\in D}$ with Lipschitz constant equal to that of $L$ in $\boldsymbol q$. \end{lemma} \begin{proof} Let $\boldsymbol q, \boldsymbol{q'}\in \bar D$ and consider the case when $L^*(\textbf q)\le L^*(\boldsymbol{q'})$. Let $\boldsymbol \delta = \boldsymbol{q'} - \boldsymbol q$ and $k$ be the Lipschitz constant of $L$. Then, $$L^*(\textbf q)\le L^*(\textbf q + \boldsymbol \delta)\le L(\boldsymbol{p^*}(\boldsymbol q), \boldsymbol q + \boldsymbol \delta)\le L^*(\boldsymbol q) + k ||\boldsymbol \delta||$$ If $L^*(\boldsymbol q)> L^*(\boldsymbol{q'})$, then the above shows that $L^*(\boldsymbol q)-k||\boldsymbol \delta||\le L^*(\boldsymbol{q'})$. Therefore, regardless of the relative sizes of $L^*(\boldsymbol q)$ and $L^*(\boldsymbol{q'})$, we have $$L^*(\boldsymbol q)-k||\boldsymbol \delta||\le L^*(\boldsymbol{q'})\le L^*(\boldsymbol q)+k||\boldsymbol \delta||.\qedhere$$ \end{proof} \begin{lemma} If $\ell$ is strictly convex and twice differentiable in $(0, 1)^n$, then $\boldsymbol{p^*}(\boldsymbol q)$ is continuous. \label{lem:p*continuous} \end{lemma} \begin{proof} Fix any $\bec q \in (0, 1)^n$ and $k_q$, $k_p$ greater than the Lipschitz constants of $L$ with respect to $\bec q$ and $\bec p$ respectively in the neighborhood of $(\bec{p^*}(\bec q), \bec q)$. Let $D$ be a closed region in which $L$ is Lipschitz in $\boldsymbol{p}$ with Lipschitz constant $k_p$ and Lipshcitz in $\boldsymbol{q}$ with Lipschitz constant $k_q$ and $\bar D$ be a closed region contained in $\set{\boldsymbol q':(\boldsymbol{p^*}(\boldsymbol{q'}), \boldsymbol{q'})\in D}$ that contains $(\bec{p^*}(\bec q), \bec q)$.\\\\ Let $\boldsymbol{q_1}, \boldsymbol{q_2}, \dots$ be a sequence in $\bar D$ with limit $\boldsymbol q$. As $(\boldsymbol{p^*}(\boldsymbol a), \boldsymbol{a})\in D$ for all $\boldsymbol a\in \bar D$, and $D$ is sequentially compact, the sequence $\boldsymbol{p^*}(\boldsymbol{q_1}), \boldsymbol{p^*}(\boldsymbol{q_2}), \dots$ has a subsequence with a limit. Let $\boldsymbol{p'}$ be one of those limit and $\boldsymbol{a_1}, \boldsymbol{a_2}, \dots$ be a subsequence of $\boldsymbol{q_1}, \boldsymbol{q_2}, \dots$ where the limit of $\boldsymbol{p^*}(\boldsymbol{a_1}), \boldsymbol{p^*}(\boldsymbol{a_2}), \dots$ is $\boldsymbol{p'}$. Then, for any $\epsilon >0$, there exists $N$ where for all $n>N$, $||\boldsymbol{a_n}-\boldsymbol q||<\epsilon$ and $||\boldsymbol{p^*}(\boldsymbol{a_n})-\boldsymbol{p'}||<\epsilon$. Then, by Lemma~\ref{Lqsim}, $$||L(\boldsymbol{p^*}(\boldsymbol{a_n}), \boldsymbol{a_n})-L(\boldsymbol{p^*}(\boldsymbol{q}), \boldsymbol{a})||\le k_q\epsilon.$$ Using the fact that $L$ is Lipschitz in $\boldsymbol p$ and $\boldsymbol q$, by the triangle inequality $$||L(\boldsymbol{p'}, \boldsymbol{q})-L(\boldsymbol{p^*}(\boldsymbol{q}), \boldsymbol{a})||-k_p\epsilon-k_q\epsilon\le ||L(\boldsymbol{p^*}(\boldsymbol{a_n}), \boldsymbol{a_n})-L(\boldsymbol{p^*}(\boldsymbol{q}), \boldsymbol{a})||$$ $$||L(\boldsymbol{p'}, \boldsymbol{q})-L(\boldsymbol{p^*}(\boldsymbol{q}), \boldsymbol{a})||\le (2k_q+k_p)\epsilon.$$ As this is true for all $\epsilon>0$, we must have $||L(\boldsymbol{p'}, \boldsymbol{q})-L(\boldsymbol{p^*}(\boldsymbol{q}), \boldsymbol{a})||=0$, so by Theorem \ref{punique}, $\boldsymbol{p'}=\boldsymbol{p^*}(\boldsymbol{q})$ and $\boldsymbol{p^*}$ is continuous at $\boldsymbol q$. \end{proof} \noindent Let $\nabla L(\bec p, \bec q)$ be the gradient of $L$ with respect to $\bec p$ at $(\bec p, \bec q)$. \begin{theorem} \label{holder} If $\ell$ is strictly convex and twice differentiable in $(0, 1)^n$, then $\boldsymbol{p^*}(\boldsymbol q)$ is Holder-$\frac 12$. \label{thm:holder} \end{theorem} \begin{proof} Fix any $\bec q \in (0, 1)^n)$ and let $\lambda$ be the least eigenvalue of the Hessian $H$ of $L$ with respect to $\boldsymbol p$ at $(\boldsymbol{p^*}(\boldsymbol q), \boldsymbol q)$, let $D$ be some open region over which $L$ is Lipschitz that contains $(\boldsymbol{p^*}(\boldsymbol q), \boldsymbol q)$, and let $\bar{D}=\set{\textbf q\in [0, 1]^n:(\boldsymbol{p^*}(\textbf q), \textbf q)\in D}$. Additionally, let $k$ be the Lipschitz constant of $L$ with respect to $\boldsymbol q$ on $\bar D$. Then, first of all, by Lemma~\ref{Lqsim}, for all $\boldsymbol \delta$ where $\boldsymbol q + \boldsymbol \delta \in \bar D$, $$|L(\boldsymbol{p^*}(\boldsymbol q+\boldsymbol \delta), \boldsymbol q+\boldsymbol \delta)-L(\boldsymbol{p^*}(\boldsymbol q), \boldsymbol q)|\le k ||\boldsymbol \delta||.$$ Then, by the triangle inequality, as $L$ is Lipschitz in $\boldsymbol q$ $$|L(\boldsymbol{p^*}(\boldsymbol q+\boldsymbol \delta), \boldsymbol q)-L(\boldsymbol{p^*}(\boldsymbol q), \boldsymbol q)|$$ $$\le |L(\boldsymbol{p^*}(\boldsymbol q+\boldsymbol\delta), \boldsymbol q)-L(\boldsymbol{p^*}(\boldsymbol q+\boldsymbol \delta), \boldsymbol q+\boldsymbol \delta)|+|L(\boldsymbol{p^*}(\boldsymbol q+\boldsymbol \delta), \boldsymbol q+\boldsymbol \delta)-L(\boldsymbol{p^*}(\boldsymbol q), \boldsymbol q)|\le2 k ||\boldsymbol \delta||.$$ So, $$L(\boldsymbol{p^*}(\boldsymbol q+\boldsymbol \delta), \boldsymbol q)-L(\boldsymbol{p^*}(\boldsymbol q), \boldsymbol q)\le 2k||\boldsymbol \delta||.$$ Also, as $L$ is twice differentiable in $\boldsymbol p$ and $ \boldsymbol{p^*}$ is continuous at $\boldsymbol q$ by Lemma \ref{lem:p*continuous}, $$0=\lim_{\boldsymbol \delta \rightarrow \boldsymbol 0}\frac{L(\boldsymbol{p^*}(\boldsymbol q+\boldsymbol \delta), \boldsymbol q)-L(\boldsymbol{p^*}(\boldsymbol q), \boldsymbol q)-\nabla L(\bec{p^*}(\bec q), \bec q)\cdot \Delta \bec{p^*}-\Delta \boldsymbol{p^{*}}^T H \Delta \boldsymbol{p^*}}{||\Delta \boldsymbol{p^*}||^2}$$ where $\Delta \boldsymbol{p^*}=\boldsymbol{p^*}(\boldsymbol q+\boldsymbol \delta)-\boldsymbol{p^*}(\boldsymbol q)$ and $\nabla L(\bec{p^*}(\bec q), \bec q)\cdot \Delta \bec{p^*}\ge 0$ as in Theorem \ref{punique}. Then, for any $0<\epsilon <\lambda$, for there exists $\Delta > 0$ where for $0<||\boldsymbol \delta||<\Delta$ $$-\epsilon<\frac{L(\boldsymbol{p^*}(\boldsymbol q+\boldsymbol \delta), \boldsymbol q)-L(\boldsymbol{p^*}(\boldsymbol q), \boldsymbol q)-\nabla L(\bec{p^*}(\bec q), \bec q)\cdot \Delta \bec{p^*}-\Delta \boldsymbol p^{*T} H \Delta \boldsymbol{p^*}}{||\Delta \boldsymbol{p^*}||^2}$$ $$\le\frac{2k||\boldsymbol \delta||-\lambda ||\Delta \boldsymbol{p^*}||^2}{||\Delta \boldsymbol{p^*}||^2}.$$ So $$\frac{||\Delta \boldsymbol p^*||}{\sqrt{||\boldsymbol\delta||}}\le\sqrt{\frac{2k}{\lambda-\epsilon}}.$$ And $\boldsymbol{p^*}$ is Holder-$\frac{1}{2}$ in the open ball of radius $\Delta$ at $\boldsymbol q$. \end{proof} \noindent Let $\nabla_{\boldsymbol q} L(\boldsymbol{p^*}(\boldsymbol q), \boldsymbol q)$ be the gradient of $L(\boldsymbol p, \boldsymbol q)$ as $\boldsymbol q$ is changed at the point $(\boldsymbol{p^*}(\boldsymbol q), \boldsymbol q)$. \begin{theorem} \label{thm:holderImpliesDiff} Suppose there is some open region $\bar D\subseteq(0, 1)^n$ where $\boldsymbol{p^*}$ is Holder-$\frac{1+\alpha}{2}$ for some $\alpha>0$, and $\ell$ is twice differentiable at $(\boldsymbol{p^*}(\boldsymbol q), \boldsymbol q)$ for $\boldsymbol q \in \bar D$. Then, $L^*$ is differentiable in $\bar D$ with derivative $\nabla L^*(\boldsymbol q)=\nabla_{\boldsymbol q} L(\boldsymbol{p^*}(\boldsymbol q), \boldsymbol q)$. \end{theorem} \begin{proof} Let $k$ be the Holder-$\frac{1+\alpha}{2}$ coefficient of $p^*$ and $\lambda$ the least eigenvalue of the Hessian $H$ of $L$ with respect to $\boldsymbol p$ at $(\boldsymbol{p^*}(\boldsymbol q), \boldsymbol q)$. Then, consider $$\lim_{\boldsymbol \delta \rightarrow \boldsymbol 0}\frac{L(\boldsymbol{p^*}(\boldsymbol q + \boldsymbol \delta), \boldsymbol q + \boldsymbol \delta) -L(\boldsymbol{p^*}(\boldsymbol q + \boldsymbol \delta), \boldsymbol q) -L(\boldsymbol{p^*}(\boldsymbol q), \boldsymbol q + \boldsymbol \delta) +L(\boldsymbol{p^*}(\boldsymbol q), \boldsymbol q)}{||\boldsymbol \delta||}$$ $$=\lim_{\boldsymbol \delta \rightarrow \boldsymbol 0}\nabla_{\boldsymbol q} L(\boldsymbol{p^*}(\boldsymbol q + \boldsymbol \delta), \boldsymbol q) - \nabla_{\boldsymbol q} L(\boldsymbol{p^*}(\boldsymbol q), \boldsymbol q)=0$$ as $\boldsymbol{p^*}$ and $\nabla_{\boldsymbol q} L$ are both continuous. Then, $$\lim_{\boldsymbol \delta \rightarrow \boldsymbol 0}\frac{L^*(\boldsymbol q + \boldsymbol\delta)-L^*(\boldsymbol q)-\nabla_{\boldsymbol q} L(\boldsymbol{p^*}(\boldsymbol q), \boldsymbol q)}{||\boldsymbol \delta||}$$ $$=\lim_{\boldsymbol \delta \rightarrow \boldsymbol 0}\frac{L(\boldsymbol{p^*}(\boldsymbol q + \boldsymbol \delta), \boldsymbol q) -L(\boldsymbol{p^*}(\boldsymbol q), \boldsymbol q)+L(\boldsymbol{p^*}(\boldsymbol q), \boldsymbol q + \boldsymbol \delta) -L(\boldsymbol{p^*}(\boldsymbol q), \boldsymbol q)-\nabla_{\boldsymbol q} L(\boldsymbol{p^*}(\boldsymbol q), \boldsymbol q)}{||\boldsymbol \delta||}$$ $$=\lim_{\boldsymbol \delta \rightarrow \boldsymbol 0}\frac{L(\boldsymbol{p^*}(\boldsymbol q + \boldsymbol \delta), \boldsymbol q) -L(\boldsymbol{p^*}(\boldsymbol q), \boldsymbol q)}{||\boldsymbol \delta||}$$ $$=\lim_{\boldsymbol \delta \rightarrow \boldsymbol 0}\frac{L(\boldsymbol{p^*}(\boldsymbol q + \boldsymbol \delta), \boldsymbol q) -L(\boldsymbol{p^*}(\boldsymbol q), \boldsymbol q)}{||\boldsymbol{p^*}(\boldsymbol v + \boldsymbol \delta)-\boldsymbol{p^*}(\boldsymbol v)||^2}\frac{||\boldsymbol{p^*}(\boldsymbol v + \boldsymbol \delta)-\boldsymbol{p^*}(\boldsymbol v)||^2}{||\boldsymbol \delta||}$$ $$\le \lim_{\boldsymbol \delta \rightarrow \boldsymbol 0}\lambda \frac{||\boldsymbol{p^*}(\boldsymbol v + \boldsymbol \delta)-\boldsymbol{p^*}(\boldsymbol v)||^2}{||\boldsymbol \delta||}=\lim_{\boldsymbol \delta \rightarrow \boldsymbol 0}\lambda ||\boldsymbol \delta||^{2\alpha}\frac{||\boldsymbol{p^*}(\boldsymbol v + \boldsymbol \delta)-\boldsymbol{p^*}(\boldsymbol v)||^2}{||\boldsymbol \delta||^{1+2\alpha}}\le\lim_{\boldsymbol \delta \rightarrow \boldsymbol 0}\lambda k ||\boldsymbol\delta||^{2\alpha}=0$$ Note that by the minimum property of $L^*$ $$0\le\lim_{\boldsymbol \delta \rightarrow \boldsymbol 0}\frac{L(\boldsymbol{p^*}(\boldsymbol q + \boldsymbol \delta), \boldsymbol q) -L(\boldsymbol{p^*}(\boldsymbol q), \boldsymbol q)}{||\boldsymbol \delta||}$$ So, $$\lim_{\boldsymbol \delta \rightarrow \boldsymbol 0}\frac{L^*(\boldsymbol q + \boldsymbol\delta)-L^*(\boldsymbol q)-\nabla_q L(\boldsymbol{p^*}(\boldsymbol q), \boldsymbol q)}{||\boldsymbol \delta||}=0$$ and $\nabla L^*(\boldsymbol q)=\nabla_{\boldsymbol q} L(\boldsymbol{p^*}(\boldsymbol q), \boldsymbol q)$. \end{proof} \section{Proofs of Section \ref{subsec:singleEvent}} \begin{lemma} When using dissimilarity function $\ell(p, q)=2(p-q)^2$, the loss function becomes $$L^*(q_E, q_{E^c}) = (q_{E} + q_{E^c} - 1)^2.\label{lem:burns_loss}$$ \end{lemma} \begin{proof} Lagrange multipliers yield $p^*_E=\frac{q_E + 1 - q_{E^c}}{2}=1-p^*_{E^c}$. Therefore \begin{align*} L^*(q_{E}, q_{E^c})&=L((p_{E}, p_{E^c}), (q_{E}, q_{E^c}))\\ &=2(p_E - q_{E})^2+2(p_{E^c} - q_{E^c})^2\\ &=(q_{E} + q_{E^c} - 1)^2.\qedhere \end{align*} \end{proof} \begin{lemma} When using dissimilarity function $$\ell(p, q)=f(p, q)=p\ln \frac pq + (1-p)\ln \frac{1-p}{1-q}$$ the loss function becomes $$L^*(q_E, q_{E^c}) = \frac{(q_E + q_{E^c} - 1)^2}{(1-q_E+q_{E^c})(q_E+1-q_{E^c})}+O((q_E + q_{E^c} - 1)^3).\label{lem:burns_f}$$ \end{lemma} \begin{proof} Lagrange multipliers yield that the closest coherent probability $p^*_{E}$ satisfies $\frac{p^*_{E}}{1-p^*_{E}}=\left(\frac{q_E(1-q_{E^c})}{(1-q_{E})q_{E^c}}\right)^{1/2}$, giving the loss \begin{align*} L^*(q_E, q_{E^c})&=2p^*_{E} \ln \frac{p^*_E}{1-p^*_E} - p^*_E\ln\frac{q_E}{1-q_E} - p^*_E\ln\frac{q_{E_c}}{1-q_{E^c}}+\ln \frac{(1-p^*_E)^2}{(1-q_E)q_{E^c}}\\ &=\ln \frac{(1-p^*_E)^2}{(1-q_E)q_{E^c}}. \end{align*} Letting $o_{E}= \frac{q_E}{1-q_E}$ and $o'_E=\frac{1-q_{E^c}}{q_{E^c}}$ be the odds of $E$ derived from the two probes with $q=q_E$ and $q'=1-q_{E^c}$ the probabilities derived from the probes and Taylor expanding in $o$ about $o=o'$ yields: \begin{align*} L^*(q_E, q_{E^c})&=\ln \frac{(1+o)(1+o')}{(1+\sqrt{oo'})^2}\\ &\approx \frac{1}{2}(o-o')^2 \deriv{^2}{o^2}\ln \frac{(1+o)(1+o')}{(1+\sqrt{oo'})^2}+O((o-o')^3)\\ &=\frac{1}{2}(o-o')^2\left(-\frac{1}{(1+o')^2}+\frac{1+2o'}{2o'(1+o')^2}\right)+O((o-o')^3)\\ &=\frac{1}{4o'}(o-o')^2+O((o-o')^3)\\ &=\frac{(q-q')^2}{4q'(1-q')}+O((q-q')^3)\\ &\approx \frac{(q - q')^2}{(q+q')(2-q-q')}+(q-q')^2O(q-q')+O((q-q')^3)\\ &=\frac{(q - q')^2}{(q+q')(2-q-q')}+O((q - q')^3). \qedhere \end{align*} \end{proof} \begin{lemma} When using dissimilarity function $$\ell(p, q)=f^o(p, q)=q\ln \frac qp + (1-q)\ln \frac{1-q}{1-p}$$ the loss function becomes $$L^*(q_E, q_{E^c}) = \frac{(q_E + q_{E^c} - 1)^2}{(1-q_E+q_{E^c})(q_E+1-q_{E^c})}+O((q_E + q_{E^c} - 1)^3).$$\label{lem:burns_fo} \end{lemma} \begin{proof} Again letting $q=q_E$ and $q'=1-q_{E^c}$, Lagrange multipliers show that $p^*_E = \frac{q+q'}{2}$, which we will denote by $p$ for ease of notation. Taking the derivative of $L^*$ in $q'$ gives $$\pderiv{}{q'}L^*(q, q') = \ln\frac{q'}{1-q'}-\ln \frac{p}{1-p}$$ and $$\pderiv{^2}{q'^2}L^*(q, q') = \frac{1}{q'(1-q')}-\frac{1}{2p(1-p)}.$$ Then, Taylor expanding around $q=q'$ in $q'$ gives \begin{align*} L^*(q_E, q_{E^c}) &\approx \frac{1}{2}(q-q')^2\pderiv{^2}{q'^2}L^*(q, q') + O((q-q')^3)\\ &=\frac{(q-q')^2}{2q'(1-q')}-\frac{q-q')^2}{4p(1-p)}+ O((q-q')^3\\ &\approx \frac{(q-q')^2}{4p(1-p)}+(q-q')^2O((q-q'))+ O((q-q')^3\\ &=\frac{(q-q')^2}{(q+q')(2-q-q')} + O((q-q')^3).\qedhere \end{align*} \end{proof} \section{Comparison of Expert Aggregation Methods\label{sec:specialcompare}} \noindent Here, we will look at the following scenario and consider what various methods from Section \ref{sec:indCohExp} give us as estimates of the probabilities of various events: \\\\ We take $N=4$ and consider two experts. The first expert submits probability estimates for two events $$V_1=\begin{pmatrix} 1 & 1 & 0 & 0\\ 1 & 1 & 1 & 0 \end{pmatrix}, \boldsymbol{q}_1=\begin{pmatrix} 0.5\\0.9 \end{pmatrix}$$ and the second expert submits estimates for three events $$V_2=\begin{pmatrix} 1 & 1 & 0 & 0\\ 0 & 1 & 1 & 0\\ 0 & 0 & 0 & 1 \end{pmatrix}, \boldsymbol{q}_2=\begin{pmatrix} 0.3\\0.2\\0.6 \end{pmatrix}.$$ Table \ref{table:assymetriccomparison} summarizes the aggregated beliefs $\bec p^*$ using four different methods and two different dissimilarity functions.\color{black} \begin{table}[H] \begin{center} \begin{tabular}{|p{15em} |m{2em}|m{2em}| m{6em} | m{6em} |} \hline Aggregation Method & $\bec q_1$& $\bec q_2$ &$\bec p^*$ with $\ell=f$ & $\bec p^*$ with $\ell=f^o$ \\\hline Sum over stated beliefs only (not content invariant): \newline $\mathcal D_i = D_{i, \mathtt E_i}$&$\begin{matrix} 0.5 \\ 0.9 \\ - \\ - \end{matrix}$ & $\begin{matrix} 0.3 \\ - \\ 0.2 \\ 0.6 \end{matrix}$ &$\begin{matrix} 0.43 \\ 0.68 \\ 0.25 \\ 0.32 \end{matrix}$ &$\begin{matrix} 0.41 \\ 0.64 \\ 0.22 \\ 0.36 \end{matrix}$\\ \hline Sum over all inferable beliefs: \newline $\mathcal D_i=D_{i, I_i}$ &$\begin{matrix} 0.5 \\ 0.9 \\ - \\ - \end{matrix}$ & $\begin{matrix} 0.3 \\ - \\ 0.2 \\ 0.6 \end{matrix}$& $\begin{matrix} 0.46 \\ 0.73 \\ 0.46 \\ 0.27 \end{matrix}$ &$\begin{matrix} 0.43 \\ 0.71 \\ 0.47 \\ 0.29 \end{matrix}$\\ \hline Sum over the minimal positive basis: \newline $\mathcal D_i= D_{i, B_i}$ &$\begin{matrix} 0.5 \\ 0.9 \\ - \\ - \end{matrix}$ & $\begin{matrix} 0.3 \\ - \\ 0.2 \\ 0.6 \end{matrix}$& $\begin{matrix} 0.46 \\ 0.72 \\ 0.42 \\ 0.28 \end{matrix}$&$\begin{matrix} 0.42 \\ 0.69 \\ 0.41 \\ 0.31 \end{matrix}$\\ \hline Sum over the minimal positive basis, asymmetric variant: \newline $\mathcal D_i=\tilde D_{i, B_i}$ &$\begin{matrix} 0.5 \\ 0.9 \\ - \\ - \end{matrix}$ & $\begin{matrix} 0.3 \\ - \\ 0.2 \\ 0.6 \end{matrix}$& $\begin{matrix} 0.48 \\ 0.74 \\ 0.42 \\ 0.26 \end{matrix}$&$\begin{matrix} 0.41 \\ 0.69 \\ 0.41 \\ 0.31 \end{matrix}$\\ \hline \end{tabular} \color{black} \caption{Minimizer of $L$ for various loss functions, of four possible forms defined in equations (\ref{eqn:contentinvariantloss}) and (\ref{eqn:assymetricvariant}), three of which are content invariant, each using one of two possible dissimilarity functions. $\bec p^*$ is the nearest coherent vector of beliefs. Observe that all three content invariant aggregation methods give similar estimates $\bec p^*$ provided they use the same loss function.}\label{table:assymetriccomparison} \end{center} \end{table} \noindent It should be noted that the first method assigns a probability of $0$ to atom $\omega_2$, but that it is given non-zero probability by all the content invariant methods. This is because the event $\bec e_2$ is not present in the sum over stated beliefs only, but is present in all the other sums. \section{Extended Example: Masked Letters}\label{sec:ex} \noindent We illustrate the methods of Section \ref{sec:indCohExp} in the following scenario: given a word with one masked letter, we seek to predict the masked letter by combining two heuristics, which will play the role of experts in Section \ref{sec:indCohExp}. One heuristic uses the preceding letters to make predictions and the other heuristic uses the succeeding letters to make its predictions. \\\\For example, given the word EM*IL, with the third letter masked, the first expert would use the 3-gram EM* to make its predictions $Q_1(\text{A})=0.16, Q_1(\text{B})=0.08, \dots$. The second heuristic would use the 3-gram *IL to make the predictions $Q_2(\text{A})=0.32, Q_2(\text{B})=0.27, \dots$. \\\\ If each heuristic is derived from counting 3-gram frequencies in a corpus, then it will be internally coherent, so we use a content invariant loss function as outlined in Section \ref{sec:indCohExp}. As $\#I_1=\#I_2=2^{26}$, the size of the set of events whose probabilities can be inferred, is much too large, one should either sum over the minimal spanning sets $B_i$ or sum over subsets of the minimal spanning sets that add to $\bec 1$. We will outline the calculations using dissimilarity function $\ell=f$ and compare the results with $\ell = f^o$. \subsection{\texorpdfstring{Method 1: Summing over a Positive Basis}{Method 1: Summing over a Positive Basis}} Here we consider the loss function determined by $$\mathcal D_i=D_{i, B_i} = \frac{1}{B_i}\sum_{\boldsymbol b \in B_i}\ell(P(\boldsymbol b), Q_i(\boldsymbol b))$$ Recall that $M_i$ is the normalizing constant, equal to the number of terms in the summand. Additionally, in this situation, the heuristics' predictions are exactly the minimal spanning set. So $$L(\bec p, (\bec q_1, \bec q_2))=L(V\bec \pi, (\bec q_1,\bec q_2))=\frac{1}{26}\sum_{\alpha\in \set{\text{A}, \text{B}, \dots}} \ell(\pi_\alpha, q_{1, \alpha})+\ell(\pi_\alpha, q_{2, \alpha}).$$ Then, if all entries in $\bec \pi$ are strictly positive (which they will be for $f$ and $f^o$ as long as all entries in $\bec q_1$ and $\bec q_2$ are non-zero), by Lagrange multipliers as $\sum_{\alpha\in \set{\text{A}, \text{B}, \dots}}\pi_\alpha=1$, there is some constant $k$ where for all $\alpha$ $$\frac k{26}=\pderiv{L}{\pi_\alpha}=\pderiv{\ell(\pi_\alpha, q_{1, \alpha})}{\pi_\alpha}+\pderiv{\ell(\pi_\alpha, q_{2, \alpha})}{\pi_\alpha}.$$ Taking $\ell=f$ and calculating shows $$k=26\pderiv{L}{\pi_\alpha}=\ln\frac{\pi_\alpha}{1-\pi_\alpha}-\ln\frac{q_{1, \alpha}}{1-q_{1, \alpha}}+\ln\frac{\pi_\alpha}{1-q_\alpha}-\ln\frac{q_{2, \alpha}}{1-q_{2, \alpha}}$$ so $$\ln\frac{\pi_\alpha}{1-\pi_\alpha}-\frac12\left(\ln\frac{q_{1, \alpha}}{1-q_{1, \alpha}}+\ln\frac{q_{2, \alpha}}{1-q_{2, \alpha}}\right)$$ does not depend on $\alpha$. Computing $\bec \pi$ analytically from here is difficult, so some numerical algorithm should be used. \subsection{\texorpdfstring{Method 2: Using an Asymmetric Loss Function}{Method 2: Using an Asymmetric Loss Function}} Here we consider the loss function determined by $$\mathcal D_i=\tilde D_{i, B_i} = \frac{1}{\#M_i}\sum_{S \in O_i} \sum_{\boldsymbol b \in S}\tilde{\ell}(P(\bec b), Q_i(\bec b))$$ Here $O_i\subset 2^{B_i}$ is the set of subsets $S$ of $B_i$ whose sum is $\bec 1$. In this case, $O_i$ is the singleton set $\set{B_i}$ as the heuristic's predictions are the maximal spanning set and their sum is $1$, so we sum over the same events as previously. However, instead of using the function $\ell$, we use $\tilde \ell$ and ignore the terms related to the surprise of a letter not being chosen. As $M_1=M_2=26$ is the number of terms in the summand, the loss function is $$L(V\bec \pi, (\bec q_1,\bec q_2))=\frac{1}{26}\sum_{\alpha\in \set{\text{A}, \text{B}, \dots}} \tilde \ell(\pi_\alpha, q_{1, \alpha})+\tilde \ell(\pi_\alpha, q_{2, \alpha}).$$ As we are using $\ell=f$, so $\tilde \ell(p, q)=p\ln \frac pq$, this becomes $$=\frac{1}{26}\sum_{\alpha\in \set{\text{A}, \text{B}, \dots}} \pi_\alpha \ln \frac{\pi_\alpha}{q_{1, \alpha}}+\pi_\alpha\ln \frac{\pi_\alpha}{q_{2, \alpha}}.$$ And again using Lagrange multipliers to take the derivative, the quantity $$26\pderiv{L}{\pi_\alpha} - 2=\ln \frac{\pi_\alpha}{q_{1, \alpha}} + \ln \frac{\pi_\alpha}{q_{2, \alpha}}$$ does not depend on $\alpha$. Therefore, for some $k$ that does not depend on $\alpha$, $$\pi_\alpha = k(q_{1, \alpha}q_{2, \alpha})^\frac{1}{2}.$$ Using the fact that $\sum_\alpha \pi_\alpha = 1$, $$\pi_\alpha=(q_{1, \alpha}q_{2, \alpha})^\frac{1}{2}\left(\sum_\beta (q_{1, \beta}q_{2, \beta})^\frac12\right)^{-1}.$$ \subsection{Comparison of Methods} Using a list of 10,000 common English words \cite{price_wordlist}, there are 7 letters that appear both after EM and before IL, not necessarily in the same word (A, B, E, M, O, P, S). Applying the methods described above yields Table \ref{fig:extended_ex}: \begin{table}[h] \begin{center} \begin{tabular}{|m{3em}|m{2em}|m{2em}| m{9em} |m{9em}| m{5em} |} \hline Letter &$\bec q_1$ & $\bec q_2$ & \multicolumn{2}{c|}{$\ell = f$} & $\ell = f^o$ \\\cline{4-5} & & & $\bec p^*$ with $\mathcal D_i=D_{i, B_i}$ & $\bec p^*$ with $\mathcal D_i = \tilde{{\scriptstyle D}}_{i, B_i}$ & $\bec p^*$ \\\hline $\begin{matrix} A \\ B \\ E \\ M \\ O \\ P \\ S \end{matrix}$ & $\begin{matrix} 0.16\\0.08\\ 0.39\\ 0.01\\ 0.15\\ 0.17\\ 0.04 \end{matrix}$ & $\begin{matrix} 0.32\\ 0.27\\ 0.02\\ 0.22\\ 0.03\\ 0.07\\ 0.07 \end{matrix}$ &$\begin{matrix} 0.29 \\ 0.20 \\ 0.14 \\ 0.07 \\ 0.09 \\ 0.14 \\ 0.07 \end{matrix}$ &$\begin{matrix} 0.31 \\ 0.20 \\ 0.13 \\ 0.05 \\ 0.09\\0.15\\0.07 \end{matrix}$ & $\begin{matrix} 0.24\\0.18\\0.20\\0.12\\0.09\\0.12\\0.05 \end{matrix}$ \\\hline \end{tabular} \end{center} \caption{The resulting $\bec p^*$s when reconciling the heuristic $\bec q_1$ based on the 3-gram EM* and the heuristic $\bec q_2$ based on the 3-gram *IL with various methods.} \label{fig:extended_ex} \end{table} \noindent \\\\Firstly, in this setup, when using $f^o$, it does not matter whether one includes the $\tilde f^o(1-p, 1-q)$ terms. In either case, the method amounts to setting $p_\alpha = \frac{q_{1, \alpha}+q_{2, \alpha}}{2}$, which is coherent as the set of coherent beliefs is convex. This will not always be the case, and was so here because of the simplicity of the events included in the sum. Looking at the results of using $f$, there is little difference, although low probabilities ($q_{1, m}=0.01$) seem to be given more weight when excluding $\tilde f(1-p, 1-q)$ terms. As usual, $f$ gives more extreme estimates than $f^o$ with either method. \\\\ Here, both methods with either loss function were correct in assigning the letter A the highest probability in EMAIL. However, this is not usually the case. Over all the 5 letter words in \cite{price_wordlist}, both methods using $f$ gave the highest probability to the actual letter about $34\%$ of the time and both methods using $f^o$ were correct about $33\%$ of the time. The code to reproduce this experiment can be found at \url{https://github.com/scim142/quantifying_coherence}. \end{document}
2412.02902v1
http://arxiv.org/abs/2412.02902v1
$\left(p,q\right)$-adic Analysis and the Collatz Conjecture
\documentclass[11pt,oneside,english,refpage,intoc,openany]{book} \usepackage{lmodern} \renewcommand{\sfdefault}{lmss} \renewcommand{\ttdefault}{lmtt} \renewcommand{\familydefault}{\rmdefault} \usepackage[T1]{fontenc} \usepackage[latin9]{inputenc} \usepackage{geometry} \geometry{verbose,tmargin=1in,bmargin=1in,lmargin=1in,rmargin=1in} \setcounter{tocdepth}{3} \usepackage{color} \usepackage{babel} \usepackage{varioref} \usepackage{textcomp} \usepackage{mathrsfs} \usepackage{amsmath} \usepackage{amsthm} \usepackage{amssymb} \usepackage{cancel} \usepackage{makeidx} \makeindex \usepackage{graphicx} \PassOptionsToPackage{normalem}{ulem} \usepackage{ulem} \usepackage{nomencl} \providecommand{\printnomenclature}{\printglossary} \providecommand{\makenomenclature}{\makeglossary} \makenomenclature \usepackage[unicode=true, bookmarks=true,bookmarksnumbered=true,bookmarksopen=false, breaklinks=false,pdfborder={0 0 1},backref=false,colorlinks=true] {hyperref} \hypersetup{pdftitle={$\left(p,q\right)$-Adic Analysis and the Collatz Conjecture}, pdfauthor={LyX Team}, pdfsubject={LyX}, pdfkeywords={LyX}, linkcolor=black,citecolor=black,urlcolor=blue,filecolor=blue,pdfpagelayout=OneColumn,pdfnewwindow=true,pdfstartview=XYZ,plainpages=false} \makeatletter \providecommand{\tabularnewline}{\\} \theoremstyle{definition} \ifx\thechapter\undefined \newtheorem{defn}{\protect\definitionname} \else \newtheorem{defn}{\protect\definitionname}[chapter] \theoremstyle{plain} \ifx\thechapter\undefined \newtheorem{prop}{\protect\propositionname} \else \newtheorem{prop}{\protect\propositionname}[chapter] \theoremstyle{plain} \ifx\thechapter\undefined \newtheorem{lem}{\protect\lemmaname} \else \newtheorem{lem}{\protect\lemmaname}[chapter] \theoremstyle{plain} \ifx\thechapter\undefined \newtheorem{thm}{\protect\theoremname} \else \newtheorem{thm}{\protect\theoremname}[chapter] \theoremstyle{definition} \ifx\thechapter\undefined \newtheorem{example}{\protect\examplename} \else \newtheorem{example}{\protect\examplename}[chapter] \theoremstyle{remark} \ifx\thechapter\undefined \newtheorem{rem}{\protect\remarkname} \else \newtheorem{rem}{\protect\remarkname}[chapter] \theoremstyle{plain} \ifx\thechapter\undefined \newtheorem{fact}{\protect\factname} \else \newtheorem{fact}{\protect\factname}[chapter] \theoremstyle{plain} \ifx\thechapter\undefined \newtheorem{cor}{\protect\corollaryname} \else \newtheorem{cor}{\protect\corollaryname}[chapter] \theoremstyle{plain} \newtheorem*{assumption*}{\protect\assumptionname} \theoremstyle{plain} \ifx\thechapter\undefined \newtheorem{assumption}{\protect\assumptionname} \else \newtheorem{assumption}{\protect\assumptionname}[chapter] \theoremstyle{plain} \ifx\thechapter\undefined \newtheorem{conjecture}{\protect\conjecturename} \else \newtheorem{conjecture}{\protect\conjecturename}[chapter] \theoremstyle{remark} \ifx\thechapter\undefined \newtheorem{claim}{\protect\claimname} \else \newtheorem{claim}{\protect\claimname}[chapter] \theoremstyle{remark} \ifx\thechapter\undefined \newtheorem{notation}{\protect\notationname} \else \newtheorem{notation}{\protect\notationname}[chapter] \theoremstyle{definition} \ifx\thechapter\undefined \newtheorem{problem}{\protect\problemname} \else \newtheorem{problem}{\protect\problemname}[chapter] \theoremstyle{plain} \ifx\thechapter\undefined \newtheorem{question}{\protect\questionname} \else \newtheorem{question}{\protect\questionname}[chapter] \@ifundefined{date}{}{\date{}} \usepackage{babel} \addto\captionsenglish{\def\nomname{List of Symbols}} \usepackage[figure]{hypcap} \newcommand{\IndexDef}[1]{\textit{#1}} \usepackage{fancyhdr} \renewcommand{\headrulewidth}{2pt} \@ifundefined{extrarowheight}{\usepackage{array}}{} \setlength{\extrarowheight}{2pt} \def\eqdeclaration#1{, see equation\nobreakspace(#1)} \def\pagedeclaration#1{, page\nobreakspace#1} \def\nomname{Nomenclature} \providecommand{\assumptionname}{Assumption} \providecommand{\claimname}{Claim} \providecommand{\conjecturename}{Conjecture} \providecommand{\corollaryname}{Corollary} \providecommand{\definitionname}{Definition} \providecommand{\examplename}{Example} \providecommand{\factname}{Fact} \providecommand{\lemmaname}{Lemma} \providecommand{\notationname}{Notation} \providecommand{\problemname}{Problem} \providecommand{\propositionname}{Proposition} \providecommand{\questionname}{Question} \providecommand{\remarkname}{Remark} \providecommand{\theoremname}{Theorem} \makeatother \def\eqdeclaration#1{, see equation\nobreakspace(#1)} \def\pagedeclaration#1{, page\nobreakspace#1} \def\nomname{Nomenclature} \providecommand{\assumptionname}{Assumption} \providecommand{\claimname}{Claim} \providecommand{\conjecturename}{Conjecture} \providecommand{\corollaryname}{Corollary} \providecommand{\definitionname}{Definition} \providecommand{\examplename}{Example} \providecommand{\factname}{Fact} \providecommand{\lemmaname}{Lemma} \providecommand{\notationname}{Notation} \providecommand{\problemname}{Problem} \providecommand{\propositionname}{Proposition} \providecommand{\questionname}{Question} \providecommand{\remarkname}{Remark} \providecommand{\theoremname}{Theorem} \begin{document} \fancypagestyle{plain}{ \fancyhf{} \fancyhead[R]{\thepage} \renewcommand{\headrulewidth}{0pt} \renewcommand{\footrulewidth}{0pt} }\frontmatter \global\long\def\headrulewidth{0pt} \thispagestyle{fancy}\fancyfoot[L]{May 2022}\fancyfoot[R]{Maxwell Charles Siegel}\pagenumbering{gobble} \begin{center} $\left(p,q\right)$-ADIC ANALYSIS AND THE COLLATZ CONJECTURE \par\end{center} \vphantom{} \begin{center} by \par\end{center} \vphantom{} \begin{center} Maxwell Charles Siegel \par\end{center} \vphantom{} \vphantom{} \vphantom{} \begin{center} A Dissertation Presented to the \par\end{center} \begin{center} FACULTY OF THE USC DORNSIFE COLLEGE OF LETTERS AND SCIENCES \par\end{center} \begin{center} UNIVERSITY OF SOUTHERN CALIFORNIA \par\end{center} \begin{center} In Partial Fulfillment of the \par\end{center} \begin{center} Requirements for the Degree \par\end{center} \begin{center} DOCTOR OF PHILOSOPHY \par\end{center} \begin{center} (MATHEMATICS) \par\end{center} \vphantom{} \vphantom{} \vphantom{} \begin{center} May 2022 \par\end{center} \newpage\pagenumbering{roman}\setcounter{page}{2} \addcontentsline{toc}{chapter}{Dedication} \ \ \ \ \ \ \ \ \ \ \ \ \ \ \begin{center} {\large{}{}I dedicate this dissertation to those who listened, and to those who cared. Also, to my characters; I'm sorry it took so long.}{\large\par} \par\end{center} \begin{center} {\large{}{}I'll try to do better next time.}{\large\par} \par\end{center} \tableofcontents\pagebreak \listoftables \newpage{} \section*{Abstract} \pagestyle{fancy}\fancyfoot{}\fancyhead[L]{\sl ABSTRACT}\fancyhead[R]{\thepage}\addcontentsline{toc}{chapter}{Abstract} What use can there be for a function from the $p$-adic numbers to the $q$-adic numbers, where $p$ and $q$ are distinct primes? The traditional answer\textemdash courtesy of the half-century old theory of non-archimedean functional analysis: \emph{not much}. It turns out this judgment was premature. ``$\left(p,q\right)$-adic analysis'' of this sort appears to be naturally suited for studying the infamous Collatz map and similar arithmetical dynamical systems. Given such a map $H:\mathbb{Z}\rightarrow\mathbb{Z}$, one can construct a function $\chi_{H}:\mathbb{Z}_{p}\rightarrow\mathbb{Z}_{q}$ for an appropriate choice of distinct primes $p,q$ with the property that $x\in\mathbb{Z}\backslash\left\{ 0\right\} $ is a periodic point of $H$ if and only if there is a $p$-adic integer $\mathfrak{z}\in\left(\mathbb{Q}\cap\mathbb{Z}_{p}\right)\backslash\left\{ 0,1,2,\ldots\right\} $ so that $\chi_{H}\left(\mathfrak{z}\right)=x$. By generalizing Monna-Springer integration theory and establishing a $\left(p,q\right)$-adic analogue of the Wiener Tauberian Theorem, one can show that the question ``is $x\in\mathbb{Z}\backslash\left\{ 0\right\} $ a periodic point of $H$'' is essentially equivalent to ``is the span of the translates of the Fourier transform of $\chi_{H}\left(\mathfrak{z}\right)-x$ dense in an appropriate non-archimedean function space?'' This presents an exciting new frontier in Collatz research, and these methods can be used to study Collatz-type dynamical systems on the lattice $\mathbb{Z}^{d}$ for any $d\geq1$. \newpage{} \section*{Preface} \fancyhead[L]{\sl PREFACE}\addcontentsline{toc}{chapter}{Preface} Like with so many other of the world's wonders, my first encounter with the Collatz Conjecture was on Wikipedia. I might have previously stumbled across it as an undergraduate, but it wasn't until around the time I started graduate school (Autumn 2015) that I gave it any serious consideration. I remember trying to understand what $p$-adic numbers were solely through what Wikipedia said they were, hoping to apply it to the Conjecture, but then gave up after a day or two. I spent a while fooling around with trying to prove the holomorphy of tetration, and then fiddled with fractional differentiation and the iterated Laplace transform, mostly mindlessly. But then, in March of 2017, the Collatz ``bug'' bit\textemdash and bit \emph{hard}. I realized I could encode the action of the Collatz map as a transformation of holomorphic functions on the disk, and\textemdash with complex analysis being near and dear to my heart\textemdash I was hooked. Obsession was \emph{instantaneous}. As of the writing of this preface some five years later, I suppose the ardor of my obsession has dimmed, somewhat, primarily under the weight of external pressures\textemdash angst, doubt, apprehension and a healthy dose of circumspection, among them. Although all PhD dissertations are uphill battles\textemdash frequently Sisyphean\textemdash the lack of a real guiding figure for my studies made my scholarly journey particularly grueling. The mathematics department of my graduate school\textemdash the University of Southern California\textemdash has many strengths. Unfortunately, these did not include anything in or adjacent to the wide variety of sub-disciplines I drew from in my exploration of Collatz: harmonic analysis, boundary behavior of power series, analytic number theory, $p$-adic analysis, and\textemdash most recently\textemdash non-archimedean functional analysis. I was up the proverbial creek without a paddle: I did not get to benefit from the guidance of a faculty advisor who could give substantive feedback on my mathematical work. The resulting intellectual isolation was vaster than anything I'd previously experienced\textemdash and that was \emph{before }the COVID-19 pandemic hit. Of course, I alone am to blame for this. I \emph{chose }to submit to my stubborn obsession, rather than fight it. The consequences of this decision will most likely haunt me for many years to come. as much trouble as my doltish tenacity has caused me, I still have to thank it. I wouldn't have managed to find a light at the end of the tunnel without it. As I write these words, I remain apprehensive\textemdash even fearful\textemdash of what the future holds for me, mathematical or otherwise. No course of action comes with a guarantee of success, not even when you follow your heart. But, come what may, I can say with confidence that I am proud of what I have accomplished in my dissertation, and prouder to still be able to share it with others. Connectedness is success all its own. But enough about \emph{me}. In the past five years, my research into the Collatz Conjecture can be grouped into two-and-a-half different categories. The first of these was the spark that started it all: my independent discovery that the Collatz Conjecture could be reformulated in terms of the solutions of a \emph{functional equation} defined over the open unit disk $\mathbb{D}$ in $\mathbb{C}$. Specifically, the equation: \begin{equation} f\left(z\right)=f\left(z^{2}\right)+\frac{z^{-1/3}}{3}\sum_{k=0}^{2}e^{-\frac{4k\pi i}{3}}f\left(e^{\frac{2k\pi i}{3}}z^{2/3}\right)\label{eq:Collatz functional equation} \end{equation} where the unknown $f\left(z\right)$ is represented by a power series of the form: \begin{equation} f\left(z\right)=\sum_{n=0}^{\infty}c_{n}z^{n}\label{eq:Power series} \end{equation} In this context, the Collatz Conjecture becomes equivalent to the assertion that the set of holomorphic functions $\mathbb{D}\rightarrow\mathbb{C}$ solving equation (\ref{eq:Collatz functional equation}) is precisely: \begin{equation} \left\{ \alpha+\frac{\beta z}{1-z}:\alpha,\beta\in\mathbb{C}\right\} \end{equation} This discovery was not new. The observation was first made by Meinardus and Berg \cite{Berg =00003D000026 Meinardus,Meinardus and Berg} (see also \cite{Meinardus,Opfer}). What made my take different was its thrust and the broadness of its scope. I arrived at equation (\ref{eq:Collatz functional equation}) in the more general context of considering a linear operator (what I call a \textbf{permutation operator}) induced by a map $H:\mathbb{N}_{0}\rightarrow\mathbb{N}_{0}$; here $\mathbb{N}_{0}$ is the set of integers $\geq0$). The permutation operator $\mathcal{Q}_{H}$ induced by $H$ acts upon the space of holomorphic functions $\mathbb{D}\rightarrow\mathbb{C}$ by way of the formula: \begin{equation} \mathcal{Q}_{H}\left\{ \sum_{n=0}^{\infty}c_{n}z^{n}\right\} \left(z\right)\overset{\textrm{def}}{=}\sum_{n=0}^{\infty}c_{H\left(n\right)}z^{n}\label{eq:Definition of a permutation operator} \end{equation} It is not difficult to see that the space of functions fixed by $\mathcal{Q}_{H}$ is precisely: \begin{equation} \left\{ \sum_{v\in V}z^{v}:V\textrm{ is an orbit class of }H\textrm{ in }\mathbb{N}_{0}\right\} \label{eq:orbit class set-series} \end{equation} A simple computation shows that the functional equation (\ref{eq:Collatz functional equation}) is precisely the equation $\mathcal{Q}_{T_{3}}\left\{ f\right\} \left(z\right)=f\left(z\right)$, where $T_{3}$ is the \textbf{Shortened Collatz map}, with the general \textbf{Shortened}\index{$qx+1$ map} \textbf{$qx+1$ map} being defined by \index{$3x+1$ map} \begin{equation} T_{q}\left(n\right)\overset{\textrm{def}}{=}\begin{cases} \frac{n}{2} & \textrm{if }n=0\mod2\\ \frac{qn+1}{2} & \textrm{if }n=1\mod2 \end{cases}\label{eq:qx plus 1 shortened} \end{equation} where $q$ is any odd integer $\geq3$. Maps like these\textemdash I call them \textbf{Hydra maps}\textemdash have been at the heart of my research in the past five years. Indeed, they are the central topic of this dissertation. My investigations of functional equations of the form $\mathcal{Q}_{H}\left\{ f\right\} \left(z\right)=f\left(z\right)$ eventually led me to a kind of partial version of a famous lemma\footnote{\textbf{Theorem 3.1 }in \cite{On Wiener's Lemma}.} due to Norbert Wiener. I used this to devise a novel kind of Tauberian theory, where the boundary behavior of a holomorphic function on $\mathbb{D}$ was represented by a singular measure on the unit circle\footnote{This viewpoint is intimately connected with the larger topic of representing holomorphic functions on $\mathbb{D}$ in terms of their boundary values, such as by the Poisson Integral Formula \cite{Bounded analytic functions,function classes on the unit disc} or the Cauchy transform and its fractional analogues \cite{Ross et al,Fractional Cauchy transforms}.}. Of particular import was that these measures could also be interpreted as \emph{functions} in the space $L^{2}\left(\mathbb{Q}/\mathbb{Z},\mathbb{C}\right)$\textemdash all complex-valued functions on the additive group $\mathbb{Q}/\mathbb{Z}=\left[0,1\right)\cap\mathbb{Q}$ with finite $L^{2}$-norm with respect to that group's Haar measure (the counting measure): \begin{equation} L^{2}\left(\mathbb{Q}/\mathbb{Z},\mathbb{C}\right)=\left\{ f:\mathbb{Q}/\mathbb{Z}\rightarrow\mathbb{C}\mid\sum_{t\in\mathbb{Q}/\mathbb{Z}}\left|f\left(t\right)\right|^{2}<\infty\right\} \label{eq:L2 of Q/Z} \end{equation} Because the Pontryagin dual of $\mathbb{Q}/\mathbb{Z}$ is the additive group of profinite integers\footnote{The direct product $\prod_{p\in\mathbb{P}}\mathbb{Z}_{p}$ of the rings of $p$-adic integers, over all primes $p$.}, I decided to interpret these measures/functions as the Fourier transforms of complex-valued functions on the profinite integers. Detailed explanations of these can be found in my paper \cite{Dreancatchers for Hydra Maps} on arXiv. The notation in that paper is a mess, and I will likely have to re-write it at some point in the future. During these investigations, my \emph{ide fixe }was to use Fourier analysis and exploit the Hilbert space structure of $L^{2}\left(\mathbb{Q}/\mathbb{Z},\mathbb{C}\right)$ to prove what I called \emph{rationality theorems}. These were generalizations of the notion of a \emph{sufficient set} \index{sufficient set}(see \cite{Andaloro - first paper on sufficient sets,Monks' sufficiency paper for 3x+1}); a set $S\subseteq\mathbb{N}_{0}$ is said to be ``sufficient'' for the Collatz Conjecture whenever the statement ``the Collatz map iterates every element of $S$ to $1$'' is sufficient to prove the Collatz Conjecture in full. \cite{Remmert} showed that, for example, the set $\left\{ 16n+1:n\geq0\right\} $ was sufficient for the Collatz map; Monks et. al. \cite{The many Monks paper} extended this, showing that any infinite arithmetic\index{arithmetic progression} progression (``IAP'') (a set of the form $\left\{ an+b:n\geq0\right\} $ for integers $a,b$, with $a\neq0$) was sufficient for the Collatz map. Unfortunately, all I could manage to prove was a weaker form of this conclusion: namely, that for any Hydra map satisfying certain simple conditions, if an orbit class $V\subseteq\mathbb{N}_{0}$ of the Hydra map could be written as the union of a finite set and finitely many IAPs, then $V$ contained all but at most finitely many elements of $\mathbb{N}_{0}$. The one advantage of my approach was that it \emph{appeared} to generalize to ``multi-dimensional'' Hydra maps\textemdash a Collatz-type map defined on the ring of integers a finite-degree field extension of $\mathbb{Q}$, or, equivalently, on the lattice $\mathbb{Z}^{d}$. I say ``appeared'' because, while the profinite integer Hilbert space part of the generalization worked out without any difficulty, it is not yet clear if (or how) the crux of the original argument could be extended to power series of several complex variables. For the curious, this crux was a beautiful theorem\footnote{See Section 4 in Chapter 11 of \cite{Remmert}.} due to Gbor Szeg\H{o}\index{SzegH{o}, Gbor@Szeg\H{o}, Gbor} regarding the analytic continuability of (\ref{eq:Power series}) in the case where the set: \begin{equation} \left\{ c_{n}:n\in\mathbb{N}_{0}\right\} \end{equation} is finite. Taking the time to investigate this issue would allow for the results of \cite{Dreancatchers for Hydra Maps} to be extended to the multi-dimensional case, the details of which\textemdash modulo the Szeg\H{o} gap\textemdash I have written up but not yet published or uploaded in any form. I will most likely re-write \cite{Dreancatchers for Hydra Maps} at a future date\textemdash assuming I have not already done so by the time you are reading this. The \emph{first-and-a-halfth} category of my work branched off from my quest for rationality theorems. I turned my focus to using $L^{2}\left(\mathbb{Q}/\mathbb{Z},\mathbb{C}\right)$ to establish the possible value(s) of the growth constant $r>0$ \index{growth exponent} for orbit classes of Hydra maps. The essentials are as follows. Let $H$ be a Hydra map. Then, letting $\omega$ be a positive real number and letting $f\left(z\right)=\left(1-z\right)^{-\omega}$, an elementary computation with limits establishes that the limit \begin{equation} \lim_{x\uparrow1}\left(1-x\right)^{\omega}\mathcal{Q}_{H}\left\{ f\right\} \left(x\right)=1\label{eq:Eigenrate Equation} \end{equation} occurs if and only if $\omega$ satisfies a certain transcendental equation. For the case of the Shortened $qx+1$ map, the values of $\omega$ satisfying (\ref{eq:Eigenrate Equation}) are precisely the solutions of the equation: \begin{equation} 2^{\omega}-q^{\omega-1}=1 \end{equation} My hope was to prove that for: \begin{equation} \omega_{0}\overset{\textrm{def}}{=}\min\left\{ \omega\in\left(0,1\right]:2^{\omega}-q^{\omega-1}=1\right\} \end{equation} given any orbit class $V\subseteq\mathbb{N}_{0}$ of $T_{q}$ such that the iterates $T_{q}\left(v\right),T_{q}\left(T_{q}\left(v\right)\right),\ldots$ were bounded for each $v\in V$, we would have: \begin{equation} \left|\left\{ v\in V:v\leq N\right\} \right|=o\left(N^{\omega_{0}+\epsilon}\right)\textrm{ as }N\rightarrow\infty \end{equation} for any $\epsilon>0$. While initially quite promising, obtaining this asymptotic result hinged on justifying an interchange of limits, a justification which appears to be at least as difficult as establishing this asymptotic directly, \emph{without appealing} to $\mathcal{Q}_{H}$ and complex analysis as a middleman. Although still conjecture as of the writing of this dissertation, the $2^{\omega}-q^{\omega-1}=1$ formula would provide an elegant resolution to a conjecture of Kontorovich and Lagarias regarding the growth exponent for the orbit class of all integers that $T_{5}$ iterates to $1$ \cite{Lagarias-Kontorovich Paper}. I have also studied this issue from profinite integer. This turns out to be equivalent to studying the iterations of $H$ on the profinite integer and subgroups thereof, such as $\mathbb{Z}_{2}\times\mathbb{Z}_{3}$, the ring of ``$\left(2,3\right)$-adic integers''. I have obtained minor results in that regard by exploiting the known ergodicity of the $qx+1$ maps on $\mathbb{Z}_{2}$ (see \textbf{Theorem \ref{thm:shift map}}\vpageref{thm:shift map}). I will likely write this work up for publication at a later date. Whereas the above approach was grounded on the unit disk and harmonic analysis on subgroups of the circle, the second-and-a-halfth branch of my research\textemdash the one considered in this dissertation\textemdash concerns a completely different line of reasoning, from a completely different subspecialty of analysis, no less: $p$-adic and non-archimedean analysis. By considering the different branches of a given, suitably well-behaved Hydra map, I discovered a function $\chi_{H}:\mathbb{Z}_{p}\rightarrow\mathbb{Z}_{q}$ (where $p$ and $q$ are distinct primes which depend on $H$) with the remarkable property\footnote{I call this the \textbf{Correspondence Principle}; see \textbf{Theorem \ref{thm:CP v1} }and its other three variants, \textbf{Corollaries \ref{cor:CP v2}}, \textbf{\ref{cor:CP v3}}, and \textbf{\ref{cor:CP v4}}, starting from page \pageref{thm:CP v1}.} that the set of periodic points of $H$ in $\mathbb{Z}$ was \emph{completely determined} by the elements of the image of $\chi_{H}$ which lay in $\mathbb{Z}_{q}\cap\mathbb{Z}$. While I developed this approach in late 2019 and early 2020, and it was only in mid-2020 that I realized that the $\chi_{H}$ corresponding to the Shortened Collatz map played a pivotal role in a paper on the Collatz Conjecture published by Terence Tao at the end of 2019 \cite{Tao Probability paper}. However, due to Tao's \emph{deliberate choice} not\emph{ }to use a fully $p$-adic formalism\textemdash one which would have invoked $\chi_{H}$ in full\textemdash it appears he failed to notice this remarkable property. The relation between Tao's work an my own is discussed in detail starting on page \pageref{subsec:2.2.4 Other-Avenues}. It would be interesting to see my approach investigated from Tao's perspective. As of the writing of this dissertation, I have yet to grok Tao's probabilistic arguments, and have therefore refrained from attempting to employ or emulate them here. To the extent there even is such a thing as ``Collatz studies'', the subject is bazaar filled with odd and ends\textemdash \emph{ad hoc}s\emph{ }of the moment formed to deal with the latest line of thought about the Collatz map \emph{itself}. On the other hand, there have been few, if any, noteworthy attempts to center the study of Collatz-type arithmetical dynamical systems on a sound \emph{theoretical} underpinning\textemdash and by ``sound \emph{theoretical }underpinning'', I mean ``non-probabilistic investigations of multiple Collatz-type maps which \emph{do not }''. While there \emph{are }some works that address larger families of Collatz-type maps (\cite{dgh paper,Matthews' slides,Matthews and Watts,Moller's paper (german)}, for instance), they tend to be probabilistic, and are usually replete with conjectures, heuristic results, or specific findings that do not inspire much confidence in the possibility of a broader underlying non-probabilistic theory. My greatest aspiration for this dissertation\textemdash my labor of love, madness, obsession, and wonder\textemdash is that it will help pull Collatz studies out of the shadows to to emerge as a commendable, \emph{recommendable }field of study, complete with a theoretical foundation worth admiring. If I can do even half of that, I will consider my efforts successful, and all the angst I had to wade through along the way will, at last, be vindicated. \vphantom{}Los Angeles, California, USA March 14, 2022 \vphantom{} \vphantom{} \vphantom{} P.S. If you have questions, or have found typos or grammatical errors, please e-mail me at [email protected] (yes, I went to UCLA as an undergraduate). \newpage{} \pagebreak\pagestyle{headings} \mainmatter \chapter{Introduction \label{chap:Introduction}} \thispagestyle{headings} \includegraphics[scale=0.45]{./PhDDissertationEroica1.png} \vphantom{} Just like the title says, this is the introductory chapter of my PhD dissertation. Section \ref{sec:1.1 A-Guide-for-the-Perplexed} begins with an introduction the general notational conventions and idiosyncrasies used throughout this tome (Subsection \ref{subsec:1.1.1 Notational-Idiosyncrasies}). Subsection \ref{subsec:1.1.2 Some-Much-Needed-Explanations} provides the eponymous much needed explanations regarding my main results, and the agenda informing my (and our) pursuit of them\textemdash not to mention a \emph{mea culpa }regarding my dissertation's prodigious length. Speaking of length, Subsection \ref{subsec:1.1.3 An-Outline-of} contains a chapter-by-chapter outline of the dissertation as a whole, one I hope the reader will find useful. Section \ref{sec:1.2 Dynamical-Systems-Terminology} gives a brief exposition of basic notions from the theory of discrete dynamical systems\textemdash orbit classes and the like. The slightly longer Section \ref{sec:1.3 Crash course in ultrametric analysis} is a crash course in $p$-adic numbers and ultrametric analysis, written under the assumption that the reader has never heard of either topic. That being said, even an expert should take a gander at Subsections \ref{subsec:1.3.3 Field-Extensions-=00003D000026}, \ref{subsec:1.3.4 Pontryagin-Duality-and}, and \ref{subsec:1.3.5 Hensel's-Infamous-Blunder}, because my approach will frequently bring us into dangerous territory where we mix-and-match $p$-adic topologies, either with the real or complex topology, or for different primes $p$. \section{\emph{\label{sec:1.1 A-Guide-for-the-Perplexed}A Guide for the Perplexed}} \subsection{\label{subsec:1.1.1 Notational-Idiosyncrasies}Notational Idiosyncrasies} In order to resolve the endless debate over whether or not $0$ is a natural number, given any real number $x$, I write $\mathbb{N}_{x}$\nomenclature{$\mathbb{N}_{k}$}{the set of integers greater than or equal to $k$ \nopageref} to denote the set of all elements of $\mathbb{Z}$ which are $\geq x$. An expression like $\mathbb{N}_{x}^{r}$, where $r$ is an integer $\geq1$, then denotes the cartesian product of $r$ copies of $\mathbb{N}_{x}$. Likewise, $\mathbb{Z}^{d}$ denotes the cartesian product of $d$ copies of $\mathbb{Z}$. As a matter of personal preference, I use $\mathfrak{fraktur}$ (fraktur, \textbackslash mathfrak) font ($\mathfrak{z},\mathfrak{y}$, etc.) to denote $p$-adic (or $q$-adic) variables, and use the likes of $\mathfrak{a},\mathfrak{b},\mathfrak{c}$ to denote $p$-adic (or $q$-adic) constants. I prefer my notation to be as independent of context as possible, and therefore find it useful and psychologically comforting to distinguish between archimedean and non-archimedean quantities. Also, to be frank, I think the fraktur letters look \emph{cool}. I write $\left[\cdot\right]_{p^{n}}$ to denote the ``output the residue of the enclosed object modulo $p^{n}$'' operator. The output is \emph{always }an element of the set of integers $\left\{ 0,\ldots,p^{n}-1\right\} $. $\left[\cdot\right]_{1}$ denotes ``output the residue of the enclosed object modulo $1$''; hence, any integer (rational or $p$-adic) is always sent by $\left[\cdot\right]_{1}$ to $0$. $\left|\cdot\right|_{p}$ and $\left|\cdot\right|_{q}$ denote the standard $p$-adic and $q$-adic absolute values, respectively, while $v_{p}$ and $v_{q}$ denote the standard $p$-adic and $q$-adic valuations, respectively, with $\left|\cdot\right|_{p}=p^{-v_{p}\left(\cdot\right)}$, and likewise for $\left|\cdot\right|_{q}$ and $v_{q}$. Similarly, I write $\mathbb{Z}/p\mathbb{Z}$ as a short-hand for the set $\left\{ 0,\ldots,p-1\right\} $, in addition to denoting the group thereof formed by addition modulo $1$. I write $\hat{\mathbb{Z}}_{p}$ to denote the Pontryagin dual of the additive group of $p$-adic integers. I identify $\hat{\mathbb{Z}}_{p}$ with the set $\mathbb{Z}\left[1/p\right]/\mathbb{Z}$ of rational numbers in $\left[0,1\right)$ of the form $k/p^{n}$ where $n$ is an integer $\geq0$ and where $k$ is a non-negative integer which is either $0$ or co-prime to $p$. $\hat{\mathbb{Z}}_{p}=\mathbb{Z}\left[1/p\right]/\mathbb{Z}$ is made into an additive group with the operation of addition modulo $1$. All functions defined on $\hat{\mathbb{Z}}_{p}$ are assumed to be $1$-periodic; that is, for such a function $\hat{\phi}$, $\hat{\phi}\left(t+1\right)=\hat{\phi}\left(t\right)$ for all $t\in\hat{\mathbb{Z}}_{p}$. $\left\{ \cdot\right\} _{p}$ denotes the $p$-adic fractional part, viewed here as a homomorphism from the additive group $\mathbb{Q}_{p}$ of $p$-adic rational numbers to the additive group $\mathbb{Q}/\mathbb{Z}$ of rational numbers in $\left[0,1\right)$, equipped with the operation of addition modulo $1$. Because it will be frequently needed, I write $\mathbb{Z}_{p}^{\prime}$ to denote $\mathbb{Z}_{p}\backslash\mathbb{N}_{0}$ (the set of all $p$-adic integers which are not in $\left\{ 0,1,2,3,\ldots\right\} $). I use the standard convention of writing $\mathcal{O}_{\mathbb{F}}$ to denote the ring of integers of a number field $\mathbb{F}$\nomenclature{$\mathcal{O}_{\mathbb{F}}$}{the ring of $\mathbb{F}$-integers \nopageref}. With regard to embeddings (and this discussion will be repeated in Subsections \ref{subsec:1.3.3 Field-Extensions-=00003D000026} and \ref{subsec:1.3.4 Pontryagin-Duality-and}), while an algebraist might be comfortable with the idea that for a primitive third root of unity $\xi$ in $\mathbb{Q}_{7}$, the expression $\left|2-3\xi\right|_{7}$ is not technically defined, for my analytic purposes, this simply \emph{will not do}. As such, throughout this dissertation, given an odd prime $q$, in addition to writing $e^{2\pi i/\left(q-1\right)}$ to denote the complex number $\cos\left(2\pi/\left(q-1\right)\right)+i\sin\left(2\pi/\left(q-1\right)\right)$, I also write $e^{2\pi i/\left(q-1\right)}$ to denote the \emph{unique} primitive $\left(q-1\right)$th root of unity $\xi$ in $\mathbb{Z}_{q}^{\times}$ so that the value of $\xi$ modulo $q$ (that is, the first digit in the $q$-adic representation of $\xi$) is the smallest integer in $\left\{ 2,\ldots,q-2\right\} $ which is a primitive root modulo $q-1$. By far, the most common convention for expressing numerical congruences is of the form: \begin{equation} x=k\mod p \end{equation} However, I have neither any intention nor any inclination of using this cumbersome notation. Instead, I write $\overset{p}{\equiv}$ to denote congruence modulo $p$. This is a compact and extremely malleable notation. For example, given two elements $s,t\in\hat{\mathbb{Z}}_{p}$, I write $s\overset{1}{\equiv}t$ \nomenclature{$\overset{1}{\equiv}$}{congruence modulo $1$ \nopageref}to indicate that $s$ is congruent to $t$ modulo $1$; i.e., $s-t\in\mathbb{Z}$, which\textemdash of course\textemdash makes $s$ and $t$ different representatives of the same element of $\hat{\mathbb{Z}}_{p}$ (example: $s=1/p$ and $t=\left(p+1\right)/p$). This congruence notation is also compatible with $p$-adic numbers (integer or rational): $\mathfrak{z}\overset{p^{n}}{\equiv}k$\nomenclature{$\overset{p^{n}}{\equiv}$}{congruence modulo $p^{n}$ \nopageref} holds true if and only if $\mathfrak{z}$ is an element of $k+p^{n}\mathbb{Z}_{p}$. My congruence notation is particularly useful in conjunction with the indispensable Iverson bracket notation. Given a statement or expression $S$, the Iverson bracket is the convention of writing $\left[S\right]$ to denote a quantity which is equal to $1$ when $S$ is true, and which is $0$ otherwise. For example, letting $k$ be an integer constant and letting $\mathfrak{z}$ be a $p$-adic integer variable, $\left[\mathfrak{z}\overset{p^{n}}{\equiv}k\right]$ is then the indicator function for the set $k+p^{n}\mathbb{Z}_{p}$, being equal to $1$ if $\mathfrak{z}$ is congruent to $k$ mod $p^{n}$ (that is, $\left[\mathfrak{z}\right]_{p^{n}}=k$), and being $0$ otherwise. Given a point $s\in\hat{\mathbb{Z}}_{p}$, I write $\mathbf{1}_{s}\left(t\right)$ to denote the function $\left[t\overset{1}{\equiv}s\right]$ of the variable $t\in\hat{\mathbb{Z}}_{p}$. That is, $\mathbf{1}_{s}\left(t\right)$ is $1$ if and only if $t-s$ is in $\mathbb{Z}$, and is $0$ for all other values of $t$. I write $\mathbf{1}_{\mathfrak{a}}\left(\mathfrak{z}\right)$ to denote the function $\left[\mathfrak{z}=\mathfrak{a}\right]$ of the variable $\mathfrak{z}\in\mathbb{Z}_{p}$, which is $1$ if and only if $\mathfrak{z}=\mathfrak{a}$ and is $0$ otherwise. Because the absolute-value-induced-metric-topology with which we will make sense of limits of sequences or sums of infinite series will often frequently change, I have cultivated a habit of writing the space in which the convergence occurs over the associated equal sign. Thus, $\lim_{n\rightarrow\infty}f\left(x_{n}\right)\overset{\mathbb{Z}_{p}}{=}c$ means that the sequence $f\left(x_{n}\right)$ converges to $c$ in $p$-adic absolute value. Note that this necessarily forces $c$ and the $f\left(x_{n}\right)$s to be quantities whose $p$-adic absolute values are meaningfully defined. Writing $\overset{\overline{\mathbb{Q}}}{=}$ or $\overset{\mathbb{Q}}{=}$ indicates that no limits are actually involved; the sums in question are not infinite, but consist of finitely many terms, all of which are contained in the field indicated above the equals sign. I write $\overset{\textrm{def}}{=}$\nomenclature{$\overset{\textrm{def}}{=}$}{"by definition" \nopageref} to mean ``by definition''. The generalizations of these notations to the multi-dimensional case are explained on pages \pageref{nota:First MD notation batch}, \pageref{nota:Second batch}, and \pageref{nota:third batch}. These also include extensions of other notations introduced elsewhere to the multi-dimensional case, as well as the basic algebraic conventions we will use in the multi-dimensional case (see page \pageref{nota:First MD notation batch}). Also, though it will be used infrequently, I write \nomenclature{$\ll$}{Vinogradov notation (Big O)}$\ll$ and $\gg$ to denote the standard Vinogradov notation (an alternative to Big $O$), indicating a bound on a real quantity which depends on an implicit constant ($f\left(x\right)\ll g\left(x\right)$ iff $\exists$ $K>0$ so that $f\left(x\right)\leq Kg\left(x\right)$). Given a map $T$, I write $T^{\circ n}$ to denote the composition of $n$ copies of $T$. Finally, there is the matter of self-reference. Numbers enclosed in (parenthesis) refer to the numeric designation assigned to one of the many, many equations and expressions that occur in this document. Whenever a Proposition, Lemmata, Theorem, Corollary or the like is invoked, I do so by writing it in bold along with the number assigned to it; ex: \textbf{Theorem 1.1}. Numbers in parenthesis, like (4.265), refer to the equation with that particular number as its labels. References are cited by their assigned number, enclosed in {[}brackets{]}. I also sometimes refer to vector spaces as linear spaces; ergo, a $\overline{\mathbb{Q}}$-linear space is a vector space over $\overline{\mathbb{Q}}$. \subsection{Some Much-Needed Explanations \label{subsec:1.1.2 Some-Much-Needed-Explanations}} Given that this document\textemdash my PhD Dissertation\textemdash is over four-hundred fifty pages long, I certainly owe my audience an explanation. I have explanations in spades, and those given in this incipit are but the first of many. I have spent the past five years studying the Collatz Conjecture from a variety of different analytical approaches. My journey eventually led me into the \emph{ultima Thule }of non-Archimedean analysis, which\textemdash to my surprise\textemdash turned out to be far less barren and far more fruitful than I could have ever suspected. With this monograph, I chronicle my findings. To set the mood\textemdash this \emph{is }a study of the Collatz Conjecture, after all\textemdash let me give my explanations in threes. Besides the obvious goal of securing for myself a hard-earned doctoral degree, the purpose of this dissertation is three-fold: \begin{itemize} \item To present a analytical programme for studying a wide range of Collatz-type arithmetical dynamical systems in a unified way; \item To detail new, unforeseen aspects of the subspecialty of non-archimedean analysis involving the study of functions from the $p$-adic integers to the $q$-adic (complex) numbers (I call this ``\emph{$\left(p,q\right)$-adic analysis}''); \item To use $\left(p,q\right)$-adic analysis to study the dynamics of Collatz-type maps\textemdash primarily the question of whether or not a given integer is a periodic point of such a map. \end{itemize} In this respect\textemdash as indicated by its title\textemdash this dissertation is as much about the novel methods of non-archimedean analysis as it is about using those methods to describe and better understand Collatz-type dynamical systems. The reader has a winding road ahead of them, monotonically increasing in complexity over the course of their six-chapter journey. To me, among the many ideas we will come across, three innovations stand out among the rest: \begin{itemize} \item Given a kind of a Collatz-type map $H:\mathbb{Z}\rightarrow\mathbb{Z}$ which I call a \textbf{Hydra map} (see page \pageref{def:p-Hydra map}), provided $H$ satisfies certain simple qualitative properties, there are distinct primes $p,q$ and a function $\chi_{H}:\mathbb{Z}_{p}\rightarrow\mathbb{Z}_{q}$\textemdash the \textbf{Numen} of $H$ (see page \pageref{def:Chi_H on N_0 in strings})\textemdash with the property that an integer $x\in\mathbb{Z}\backslash\left\{ 0\right\} $ is a periodic point of $H$ \emph{if and only if }there is a $p$-adic integer $\mathfrak{z}_{0}\in\left(\mathbb{Q}\cap\mathbb{Z}_{p}\right)\backslash\left\{ 0,1,2,3,\ldots\right\} $ so that $\chi_{H}\left(\mathfrak{z}_{0}\right)=x$. I call this phenomenon \textbf{the} \textbf{Correspondence Principle}\footnote{Actually, we can say more: modulo certain simple qualitative conditions on $H$, if there is an ``irrational'' $p$-adic integer $\mathfrak{z}_{0}\in\mathbb{Z}_{p}\backslash\mathbb{Q}$ so that $\chi_{H}\left(\mathfrak{z}_{0}\right)\in\mathbb{Z}$, then the sequence of iterates $\chi_{H}\left(\mathfrak{z}_{0}\right),H\left(\chi_{H}\left(\mathfrak{z}_{0}\right)\right),H\left(H\left(\chi_{H}\left(\mathfrak{z}_{0}\right)\right)\right),\ldots$ is unbounded with respect to the standard archimedean absolute value on $\mathbb{R}$. I have not been able to establish the converse of this result, however.} (see page \pageref{cor:CP v4}). \item My notion of \textbf{frames} (presented in Subsection \ref{subsec:3.3.3 Frames}), which provide a formalism for dealing with functions $\chi:\mathbb{Z}_{p}\rightarrow\mathbb{Z}_{q}$ (where $p$ and $q$ are distinct primes) defined by infinite series such that the \emph{topology of convergence} used to sum said series \emph{varies from point to point}. The archetypical example is that of a series in a $p$-adic integer variable $\mathfrak{z}$ which converges in the $q$-adic topology whenever $\mathfrak{z}\in\mathbb{Z}_{p}\backslash\left\{ 0,1,2,3,\ldots\right\} $ and converges in the topology of the reals whenever $\mathfrak{z}\in\left\{ 0,1,2,3,\ldots\right\} $. Using frames, we can significantly enlarge the class of $\left(p,q\right)$-adic functions which can be meaningfully ``integrated''\textemdash I call such functions \textbf{quasi-integrable }functions. With these tools, I then establish corresponding $\left(p,q\right)$-adic generalizations of the venerable \textbf{Wiener Tauberian Theorem }(\textbf{WTT}) from harmonic analysis (Subsection \ref{subsec:3.3.7 -adic-Wiener-Tauberian}). \item A detailed $\left(p,q\right)$-adic Fourier-analytical study of $\chi_{H}$. I show that, for many Hydra maps $H$, the numen $\chi_{H}$ is in fact quasi-integrable. Not only does this come with explicit non-trivial $\left(p,q\right)$-adic series representations of $\chi_{H}$, and formulae for its Fourier transforms ($\hat{\chi}_{H}$), but\textemdash in conjunction with the Correspondence Principle and the $\left(p,q\right)$-adic Wiener Tauberian Theorem\textemdash I show that the question ``is $x\in\mathbb{Z}\backslash\left\{ 0\right\} $ a periodic point of $H$?'' is essentially equivalent to ``is the span of the translates of $\hat{\chi}_{H}\left(t\right)-x\mathbf{1}_{0}\left(t\right)$ dense in the appropriate function space?'' In particular, the answer to the former is ``no'' whenever the density of the span of the translates occurs (\textbf{Theorem \ref{thm:Periodic Points using WTT}}\vpageref{thm:Periodic Points using WTT}). I call this the \textbf{Tauberian Spectral Theorem }for $H$.\footnote{If the spans of the translates are \emph{not }dense, then, at present, my methods then guarantee that $x$ is either a periodic point of $H$ or the sequence of iterates $x,H\left(x\right),H\left(H\left(x\right)\right),\ldots$ is unbounded with respect to the standard archimedean absolute value on $\mathbb{R}$.} \end{itemize} I consider the above my ``main results''. It is worth mentioning that these results hold not just for Collatz-type maps $H:\mathbb{Z}\rightarrow\mathbb{Z}$, but for Collatz-type maps $H:\mathbb{Z}^{d}\rightarrow\mathbb{Z}^{d}$ for any integer $d\geq1$\textemdash the $d\geq2$ case being the ``\textbf{multi-dimensional case}'' (Chapters 5 \& 6). These arise from studying generalizations of Collatz-type maps from $\mathbb{Z}$ to $\mathcal{O}_{\mathbb{F}}$, the ring of integers of $\mathbb{F}$, a finite-dimensional field extension of $\mathbb{Q}$; such maps were first considered by Leigh (1981) and subsequently explored by his colleague K. R. Matthews \cite{Leigh's article,Matthews' Leigh Article,Matthews' slides}. I briefly corresponded with Matthews in 2017, and again in 2019\textemdash the man is a veteran Collatz scholar\textemdash and he informed me that other than the works just cited, he knew of no others that have pursued Collatz-type maps on rings of algebraic integers. My final explanatory triplet addresses the forty-foot dragon in the corner of the room with page after page of \emph{pages }clutched between his claws: \emph{why is this dissertation over four-hundred-fifty ($450$) pages long?} \begin{itemize} \item Unusually for ``hard'' analysis\textemdash generally a flexible, well-mixed, free-ranged discipline\textemdash the \emph{sub}-subspecialty of $\left(p,q\right)$-adic analysis is incredibly arcane, even within the already esoteric setting of non-archimedean analysis as a whole. While it is not anywhere near the level of, say, algebraic geometry, non-archimedean analysis behooves the reader a good deal of legwork. Many readers will find in these pages at least one (and perhaps many) ideas they never would have thought of considering; common courtesy and basic decency demands I give a thorough accounting of it. That my work is left field even by the standards of non-archimedean analysis only strengths my explanatory resolve. \item It is a general conviction of mine\textemdash and one strongly held, at that\textemdash that thought or knowledge without warmth, patience, openness, charity, and accessibility is indistinguishable from Brownian motion or Gaussian noise. Mathematics is difficult and frustrating enough as it is. I refuse to allow my writing\textemdash or, well, any potential \emph{lack} thereof\textemdash to cause or perpetuate pain, suffering, or discomfort which could otherwise have been avoided. Also\textemdash as befits a budding new theory\textemdash many of the most important methods and insights in this document arise directly from demanding computational considerations. The formulas provide the patterns, so it would make little sense (nor kindness, for that matter) not to explore and discuss them at length. \item I am a \emph{wordy} person, and\textemdash in addition to my passion for mathematics\textemdash I have a passion for writing. The Collatz obsession infected me just as I was finishing up my second novel (woefully stillborn), and persisted throughout the writing of my \emph{third} novel, coeval with the research and writing that led to my dissertation. \end{itemize} All I can say is, I hope you enjoy the ride. \subsection{\label{subsec:1.1.3 An-Outline-of}An Outline of this Dissertation} \subsubsection*{Chapter 1} If this dissertation were a video game, Chapter 1 would be the Tutorial level. Aside from my verbal excess, Chapter 1 provides the first significant portion of necessary background material. Section \ref{sec:1.2 Dynamical-Systems-Terminology} reviews elementary facts and terminology from the theory of dynamical systems (periodic points, divergent trajectories, orbit classes, etc.). The largest section of Chapter 1 is \ref{sec:1.3 Crash course in ultrametric analysis}, which is basically a crash course in ultrametric analysis, with a special focus on the $p$-adic numbers in Subsection \ref{subsec:1.3.1. The-p-adics-in-a-nutshell}. Most of the material from Subsection \ref{subsec:1.3.2. Ultrametrics-and-Absolute} is taken from Schikhof's \emph{Ultrametric Calculus }\cite{Ultrametric Calculus}, which I highly recommend for anyone interested in getting a first taste of the subject as a whole. Subsection \ref{subsec:1.3.3 Field-Extensions-=00003D000026} briefly mentions $p$-adic field extensions, Galois actions, and spherical completeness. Subsection \ref{subsec:1.3.4 Pontryagin-Duality-and} introduces Pontryagin duality and the Fourier transform of a \emph{complex-valued }function on a locally compact abelian group, again laser-focused on the $p$-adic case. While Subsections \ref{subsec:1.3.1. The-p-adics-in-a-nutshell} through \ref{subsec:1.3.4 Pontryagin-Duality-and} can be skipped by a reader already familiar with their contents, I must recommend that \emph{everyone }read \ref{subsec:1.3.5 Hensel's-Infamous-Blunder}. Throughout this dissertation, we will skirt grave danger by intermixing and interchanging archimedean and non-archimedean topologies on $\overline{\mathbb{Q}}$. Subsection \ref{subsec:1.3.5 Hensel's-Infamous-Blunder} explains this issue in detail, using Hensel's infamous $p$-adic ``proof'' of the irrationality of $e$ as a springboard. The reason we will not suffer a similar, ignominious fate is due to the so-called ``universality'' of the geometric series. \subsubsection*{Chapter 2} Chapter 2 presents the titular Hydra maps, my name for the class of Collatz-type maps studied in this dissertation, as well as the other Collatz-related work discussed in the Preface. Similar generalizations have been considered by \cite{Matthews and Watts,Moller's paper (german),dgh paper}; the exposition given here is an attempt to consolidate these efforts into a single package. In order to avoid excessive generality, the maps to be considered will be grounded over the ring of integers ($\mathbb{Z}$) and finite-degree extensions thereof. In his slides, Matthews considers Collatz-type maps defined over rings of polynomials with coefficients in finite fields \cite{Matthews' slides}. While, in theory, it may be possible to replicate the $\chi_{H}$ construction by passing from rings of polynomials with coefficients in a finite field $\mathbb{F}$ to the local field of formal Laurent series with coefficients in $\mathbb{F}$, in order to avoid the hassle of dealing with fields of positive characteristic, this dissertation will defer exploration of Hydra maps on fields of positive characteristic to a later date. My presentation of Hydra maps deals with two types: a ``one-dimensional'' variant\textemdash Hydra maps defined on $\mathbb{Z}$\textemdash and a ``multi-dimensional'' variant; given a number field $\mathbb{F}$, a Hydra map defined on the ring of integers $\mathcal{O}_{\mathbb{F}}$ is said to be of dimension $\left[\mathbb{F}:\mathbb{Q}\right]=\left[\mathcal{O}_{\mathbb{F}}:\mathbb{Z}\right]$. Equivalently, the multi-dimensional variant can be viewed as maps on $\mathbb{Z}^{d}$. The study of the multi-dimensional variant is more or less the same as the study of the one-dimensional case, albeit with a sizable dollop of linear algebra. After presenting Hydra maps, Chapter 2 proceeds to demonstrate how, to construct the function $\chi_{H}$\textemdash the ``numen''\footnote{My original instinct was to call $\chi_{H}$ the ``characteristic function'' of $H$ (hence the use of the letter $\chi$). However, in light of Tao's work\textemdash where, in essence, one works with the characteristic function (in the Probability-theoretic sense) of $\chi_{H}$, this name proved awkward. I instead chose to call it the ``numen'' of $H$ (plural: ``numina''), from the Latin, meaning ``the spirit or divine power presiding over a thing or place''.} of the Hydra map $H$\textemdash and establishes the conditions required of $H$ in order for $\chi_{H}$ to exist. Provided that $H\left(0\right)=0$, we will be able to construct $\chi_{H}$ as a function from $\mathbb{N}_{0}$ to $\mathbb{Q}$. Additional conditions (see page \pageref{def:Qualitative conditions on a p-Hydra map}) are then described which will guarantee the existence of a unique extension/continuation of $\chi_{H}$ to a $q$-adic valued function of a $p$-adic integer variable for appropriate $p$ and $q$ determined by $H$ and the corresponding generalization for multi-dimensional Hydra maps. This then leads us to the cornerstone of this dissertation: the \textbf{Correspondence Principle}, which I give in four variants, over pages \pageref{thm:CP v1}-\pageref{cor:CP v4}. This principle is the fact that\textemdash provided $H$ satisfies an extra condition beyond the one required for $\chi_{H}$ to have a non-archimedean extension\textemdash there will be a one-to-one correspondence between the periodic points of $H$ and the rational integer values attained by the non-archimedean extension of $\chi_{H}$. In certain cases, this principle even extends to cover the points in divergent orbits of $H$! Understandably, the Correspondence Principle motivates all the work that follows. Chapter 2 concludes with a brief discussion of the relationship between $\chi_{H}$ and other work, principally Tao's paper \cite{Tao Probability paper} (discussed in this dissertation starting \vpageref{subsec:Connections-to-Tao}), transcendental number theory, the resolution of \textbf{Catalan's Conjecture }by P. Mih\u{a}ilescu in 2002 \cite{Cohen Number Theory}, and the implications thereof for Collatz studies\textemdash in particular, the fundamental inadequacy of the current state of transcendental number theory that precludes it from being able to make meaningful conclusions on the Collatz Conjecture. \subsubsection*{Chapter 3} This is the core theory-building portion of this dissertation, and is dedicated to presenting the facets of non-archimedean analysis which I believe will finally provide Hydra maps and the contents of Chapter 2 with a proper theoretical context. After a historical essay on the diverse realizations of ``non-archimedean analysis'' (Subsection \ref{subsec:3.1.1 Some-Historical-and}), the first section of Chapter 3 is devoted to explaining the essentials of what I call ``$\left(p,q\right)$-adic analysis'', the specific sub-discipline of non-archimedean analysis which concerns functions of a $p$-adic variable taking values in $\mathbb{Q}_{q}$ (or a field extension thereof), where $p$ and $q$ are distinct primes. Since much of the literature in non-archimedean analysis tends to be forbiddingly generalized\textemdash much more so than the ``hard'' analyst is usually comfortable with\textemdash I have taken pains to minimize unnecessary generality as much as possible. As always, the goal is to make the exposition of the material as accessible as possible without compromising rigor. The other subsections of \ref{sec:3.1 A-Survey-of} alternate between the general and the specific. \ref{subsec:3.1.2 Banach-Spaces-over} addresses the essential aspects of the theory of non-archimedean Banach spaces, with an emphasis on which of the major results of the archimedean theory succeed or fail. Subsection \ref{subsec:3.1.3 The-van-der} presents the extremely useful \textbf{van der Put basis }and \textbf{van der Put series representation }for the space of continuous $\mathbb{F}$-valued functions defined on the $p$-adic integers, where $\mathbb{F}$ is a metrically complete valued field (either archimedean or non-archimedean). The existence of this basis makes computations in $\left(p,q\right)$-adic analysis seem much more like its classical, archimedean counterpart than is usually the case in more general non-archimedean analysis. Subsections \ref{subsec:3.1.4. The--adic-Fourier} and \ref{subsec:3.1.5-adic-Integration-=00003D000026} are devoted to the $\left(p,q\right)$-adic Fourier(-Stieltjes) transform. The contents of these subsections are a mix of W. M. Schikhof's 1967 PhD dissertation,\emph{ Non-Archimedean Harmonic Analysis} \cite{Schikhof's Thesis}, and my own independent re-discovery of that information prior to learning that \emph{Non-Archimedean Harmonic Analysis} already existed. For the classical analyst, the most astonishing feature of this subject is the near-total absence of \emph{nuance}: $\left(p,q\right)$-adic functions are integrable (with respect to the $\left(p,q\right)$-adic Haar probability measure, whose construction is detailed in Subsection \ref{subsec:3.1.5-adic-Integration-=00003D000026}) if and only if they are continuous, and are continuous if and only if they have a $\left(p,q\right)$-adic Fourier series representation which converges uniformly \emph{everywhere}. Whereas Subsections \ref{subsec:3.1.4. The--adic-Fourier} and \ref{subsec:3.1.5-adic-Integration-=00003D000026} are primarily oriented toward practical issues of computation, \ref{subsec:3.1.6 Monna-Springer-Integration} gives a thorough exposition of the chief theory of integration used in non-archimedean analysis as a whole, that of\textbf{ }the\textbf{ Monna-Springer Integral}. My primary motivation for introducing Monna-Springer theory is to make it better known, with an eye toward equipping other newcomers to the subject with a guide for decoding the literature. The novel content of Chapter 3 occurs in Sections \ref{sec:3.2 Rising-Continuous-Functions} and \ref{sec:3.3 quasi-integrability}, particularly the latter. Section \ref{sec:3.2 Rising-Continuous-Functions} presents and expands upon material which is given but a couple exercise's\footnote{Exercise 62B, page 192, and its surroundings\cite{Ultrametric Calculus}.} worth of attention in Schikhof's \emph{Ultrametric Calculus}. Instead, in Subsection \ref{subsec:3.2.1 -adic-Interpolation-of}, we will develop into a theory of what I call \textbf{rising-continuous functions}. In brief, letting $\mathbb{F}$ be a metrically complete valued field, these are functions $\chi:\mathbb{Z}_{p}\rightarrow\mathbb{F}$ satisfying the point-wise limit condition: \begin{equation} \chi\left(\mathfrak{z}\right)\overset{\mathbb{F}}{=}\lim_{n\rightarrow\infty}\chi\left(\left[\mathfrak{z}\right]_{p^{n}}\right),\textrm{ }\forall\mathfrak{z}\in\mathbb{Z}_{p} \end{equation} where, as indicated by the $\overset{\mathbb{F}}{=}$, the convergence occurs in the topology of $\mathbb{F}$. These functions arise naturally when one considers interpolating functions on $\mathbb{N}_{0}$ to ones on $\mathbb{Z}_{p}$. After this expos, Subsection \ref{subsec:3.2.2 Truncations-=00003D000026-The} demonstrates that the set of rising-continuous functions forms a non-archimedean Banach algebra which extends the Banach algebra of continuous $\left(p,q\right)$-adic functions. The conditions required for a rising-continuous function to be a unit\footnote{I.E., the reciprocal of the function is also rising-continuous.} of this algebra are investigated, with the \textbf{Square Root Lemma} (see page \pageref{lem:square root lemma}) of Subsection \ref{subsec:3.2.2 Truncations-=00003D000026-The} providing a necessary and sufficient condition. In the \emph{sub}-subsection starting \vpageref{subsec:3.2.2 Truncations-=00003D000026-The}, the theory of Berkovitch spaces from $p$-adic (algebraic) geometry is briefly invoked to prove some topological properties of rising-continuous functions. Subsection \ref{subsec:3.2.2 Truncations-=00003D000026-The} also introduces the crucial construction I call \textbf{truncation}. The $N$th truncation of a $\left(p,q\right)$-adic function $\chi$, denoted $\chi_{N}$, is the function defined by: \begin{equation} \chi_{N}\left(\mathfrak{z}\right)\overset{\textrm{def}}{=}\sum_{n=0}^{p^{N}-1}\chi\left(n\right)\left[\mathfrak{z}\overset{p^{N}}{\equiv}n\right] \end{equation} Even if $\chi$ is merely rising-continuous, its $N$th truncation is continuous\textemdash in fact, locally constant\textemdash for all $N$, and converges point-wise to $\chi$ everywhere as $N\rightarrow\infty$. We also explore the interplay between truncation, van der Put series, and the Fourier transform as it regards continuous $\left(p,q\right)$-adic functions. The heart of section \ref{sec:3.3 quasi-integrability}, however, is the exposition of my chief innovations non-archimedean analysis: \textbf{frames }and \textbf{quasi-integrability}. Because an arbitrary rising-continuous function is not going to be continuous, it will fail to be integrable in the sense of Monna-Springer theory. This non-integrability precludes a general rising-continuous function from possessing a well-defined Fourier transform. Quasi-integrability is the observation that this difficulty can, in certain cases, be overcome, allowing us to significantly enlarge the class of $\left(p,q\right)$-adic functions we can meaningfully integrate. Aside from facilitating a more detailed analysis of $\chi_{H}$, these innovations demonstrate that $\left(p,q\right)$-adic analysis is far more flexible\textemdash and, one would hope, \emph{useful}\textemdash than anyone had previously thought. Instead of diving right into abstract axiomatic definitions, in Subsection \ref{subsec:3.3.1 Heuristics-and-Motivations}, I take the Arnoldian approach (otherwise known as the \emph{sensible} approach) of beginning with an in-depth examination of the specific computational examples that led to and, eventually, necessitated the development of frames and quasi-integrability. To give a minor spoiler, the idea of quasi-integrability, in short, is that even if $\chi:\mathbb{Z}_{p}\rightarrow\mathbb{C}_{q}$ is not continuous, \emph{if we can find }a function $\hat{\chi}$ (defined on $\hat{\mathbb{Z}}_{p}$) so that the partial sums: \begin{equation} \sum_{\left|t\right|_{p}\leq p^{N}}\hat{\chi}\left(t\right)e^{2\pi i\left\{ t\mathfrak{z}\right\} _{p}} \end{equation} converge to $\chi$ as $N\rightarrow\infty$ in an appropriate sense for certain $\mathfrak{z}$, we can then use that $\hat{\chi}$ to define \emph{a}\footnote{Alas, it is not \emph{the} Fourier transform; it is only unique modulo the Fourier-Stieltjes transform of what we will call a \textbf{degenerate measure}.}\textbf{ Fourier transform }of $\chi$. In this case, we say that $\chi$ is \textbf{quasi-integrable}. The name is due to the fact that the existence of $\hat{\chi}$ allows us to define the integral $\int_{\mathbb{Z}_{p}}f\left(\mathfrak{z}\right)\chi\left(\mathfrak{z}\right)d\mathfrak{z}$ for any continuous $\left(p,q\right)$-adic function $f$. We view $\chi$ as the ``derivative'' of the $\left(p,q\right)$-adic measure $\chi\left(\mathfrak{z}\right)d\mathfrak{z}$; this makes $\hat{\chi}$ the Fourier-Stieltjes transform of this measure. The success of this construction is of particular interest, given that, in general \cite{van Rooij - Non-Archmedean Functional Analysis}, the Radon-Nikodym theorem fails to hold in non-archimedean analysis. As a brief\textemdash but fascinating\textemdash tangent, starting from page \pageref{eq:Definition of (p,q)-adic Mellin transform}, I showcase how quasi-integrability can be used to meaningfully define the \textbf{Mellin Transform }of a continuous $\left(p,q\right)$-adic function. Subsection \ref{subsec:3.3.2 The--adic-Dirichlet} shows how the limiting process described above can be viewed as a $\left(p,q\right)$-adic analogue of summability kernels, as per classical Fourier analysis. All the most important classical questions in Fourier Analysis of functions on the unit interval (or circle, or torus) come about as a result of the failure of the \textbf{Dirichlet Kernel }to be a well-behaved approximate identity. On the other hand, $\left(p,q\right)$-adic analysis has no such troubles: the simple, natural $\left(p,q\right)$-adic incarnation of the Dirichlet kernel is, in that setting, an approximate identity\textemdash no questions asked! In subsection \pageref{subsec:3.3.3 Frames}, the reader will be introduced to the notion of \textbf{frames}.\textbf{ }A frame is, at heart, an organizing tool that expedites and facilitates clear, concise discussion of the ``spooky'' convergence behaviors of the partial sums of Fourier series generated by $q$-adically bounded functions $\hat{\mathbb{Z}}_{p}\rightarrow\overline{\mathbb{Q}}$. This subsection is primarily conceptual, as is its counterpart \ref{subsec:3.3.5 Quasi-Integrability}, where quasi-integrability with respect to a given frame is formally defined. Subsection \ref{subsec:3.3.4 Toward-a-Taxonomy} makes the first steps toward what will hopefully one day be an able-bodied theory of $\left(p,q\right)$-adic measures, and introduces several (re-)summation formulae for $\left(p,q\right)$-adic Fourier series that will take central stage in our analysis of $\chi_{H}$. Subsection \ref{subsec:3.3.6 L^1 Convergence} also shows that the hitherto neglected topic of absolute convergence of $\left(p,q\right)$-adic integrals\textemdash that is, $L^{1}$ convergence\textemdash in non-archimedean analysis seems to provide the logical setting for studying quasi-integrability at the theoretical level. Lastly, Subsection \ref{subsec:3.3.7 -adic-Wiener-Tauberian} introduces \textbf{Wiener's Tauberian Theorem }and states and proves two new $\left(p,q\right)$-adic analogues of this chameleonic result, one for continuous functions, and one for measures. Sub-subsection \ref{subsec:A-Matter-of} then explores how the question of the existence and continuity of the reciprocal of a $\left(p,q\right)$-adic function can be stated and explored in terms of the spectral theory of \textbf{circulant matrices}. \subsubsection*{Chapter 4} In Chapter 4, the techniques developed in Chapter 3 are mustered in a comprehensive $\left(p,q\right)$-adic Fourier analysis of $\chi_{H}$. After a bit of preparatory work (Subsection \ref{sec:4.1 Preparatory-Work--}), Subsection \ref{sec:4.2 Fourier-Transforms-=00003D000026} launches into the detailed, edifying computation needed to establish the quasi-integrability of $\chi_{H}$ for a wide range of Hydra maps. The method of proof is a kind of asymptotic analysis: by computing the the Fourier transforms of the \emph{truncations} of $\chi_{H}$, we can identify fine structure which exists independently of the choice of truncation. From this, we derive an explicit formula for Fourier transform of $\chi_{H}$ and demonstrate its convergence with respect to the appropriate frame. A recurring motif of this analysis is the use of functional equation characterizations to confirm that our formulae do, in fact, represent the desired functions. As a capstone, my $\left(p,q\right)$-adic generalizations of Wiener's Tauberian Theorem are then invoked to establish a near-equivalence of the problem of characterizing the periodic points and divergent points of $H$ to determining those values of $x$ for which the span of the translates of $\hat{\chi}_{H}\left(t\right)-x\mathbf{1}_{0}\left(t\right)$ are dense in $c_{0}\left(\hat{\mathbb{Z}}_{p},\mathbb{C}_{q}\right)$; this is the content of the \textbf{Tauberian Spectral Theorem }for $\chi_{H}$ (\textbf{Theorem \ref{thm:Periodic Points using WTT}}, on page \pageref{thm:Periodic Points using WTT}). Section \ref{sec:4.3 Salmagundi} is a potpourri of miscellaneous results about $\chi_{H}$. Some of these build upon the work of Section \ref{sec:4.2 Fourier-Transforms-=00003D000026}; others which do not. I suspect the lines of inquiry covered in Section \ref{sec:4.3 Salmagundi} will be useful for future explorations of this subject. Of particular interest is what I call the ``$L^{1}$ method'' (\textbf{Theorem \ref{thm:The L^1 method}} on \pageref{thm:The L^1 method}), a means for obtaining upper bounds on the \emph{archimedean }absolute value of $\chi_{H}\left(\mathfrak{z}\right)$ over certain appropriately chosen subsets of $\mathbb{Z}_{p}$. \subsubsection*{Chapter 5} Although my methods are applicable to Collatz-type maps on rings of algebraic integers\textemdash equivalently $\mathbb{Z}^{d}$\textemdash I prefer to cover the multi-dimensional case only after the one-dimensional case, so as to let the underlying ideas shine through, unobscured by technical considerations of multi-dimensional linear algebra. Chapter 5 contains the multi-dimensional case of the content of the one-dimensional case as given in Chapters 2 and 3. Section \ref{sec:5.1 Hydra-Maps-on} deals with the issue of converting a Collatz-type on some ring of algebraic integers $\mathcal{O}_{\mathbb{F}}$ to an analogous map acting on a lattice $\mathbb{Z}^{d}$ of appropriate dimension, with Subsection \ref{subsec:5.1.2 Co=00003D0000F6rdinates,-Half-Lattices,-and} providing the working definition for a Hydra map on $\mathbb{Z}^{d}$. Section \ref{sec:5.2 The-Numen-of} is the multi-dimensional counterpart of Chapter 2, and as such constructs $\chi_{H}$ and establishes the multi-dimensional version of the \textbf{Correspondence Principle}. Sections \ref{sec:5.3 Rising-Continuity-in-Multiple} and \ref{sec:5.4. Quasi-Integrability-in-Multiple} treat rising-continuity and $\left(p,q\right)$-adic Fourier theory, respectively, as they occur in the multi-dimensional context, with the tensor product being introduced in Section \ref{sec:5.3 Rising-Continuity-in-Multiple}. \ref{sec:5.4. Quasi-Integrability-in-Multiple} presents multi-dimensional frames and quasi-integrability \subsubsection*{Chapter 6} Chapter 6 is a near-verbatim copy of Chapter 5, extended to the multi-dimensional case, though without a corresponding extension of the salmagundi of Section \ref{sec:4.3 Salmagundi}\textemdash I simply did not have enough time. \newpage{} \section{\label{sec:1.2 Dynamical-Systems-Terminology}Elementary Theory of Discrete Dynamical Systems} In this brief section, we give an overview of the essential terminology from the theory of discrete dynamical systems\index{discrete dynamical systems}. A good reference for a beginner is \cite{Dynamical Systems}; it also covers continuous systems. For our purposes, it suffices to consider dynamical systems on $\mathbb{Z}^{d}$, where $d$ is an integer $\geq1$, which is to say, there is some map $T:\mathbb{Z}^{d}\rightarrow\mathbb{Z}^{d}$ whose iterates we wish to study. Recall that we write $T^{\circ n}$ to denote the $n$-fold iteration of $T$: \begin{equation} T^{\circ n}=\underbrace{T\circ\cdots\circ T}_{n\textrm{ times}} \end{equation} \begin{defn} Given an $\mathbf{x}=\left(x_{1},\ldots,x_{d}\right)\in\mathbb{Z}^{d}$, the \textbf{forward orbit }of $\mathbf{x}$ under $T$ is the sequence of iterates $\mathbf{x},T\left(\mathbf{x}\right),T\left(T\left(\mathbf{x}\right)\right),\ldots,T^{\circ n}\left(\mathbf{x}\right),\ldots$. This is also called the \textbf{trajectory }of $\mathbf{x}$ under $T$. More generally, a \textbf{trajectory }of $T$ refers to the forward orbit of some $\mathbf{x}$ under $T$. \end{defn} \vphantom{} For us, the most important classical construction of the theory of dynamical systems is that of the \textbf{orbit class}\index{orbit class}: \begin{defn} \label{def:orbit class}Define a relation $\sim$ on $\mathbb{Z}^{d}$ by $\mathbf{x}\sim\mathbf{y}$ if and only if there are integers $m,n\geq0$ so that $T^{\circ m}\left(\mathbf{x}\right)=T^{\circ n}\left(\mathbf{y}\right)$, where, recall, $T^{\circ0}$ is the identity map. It can be verified that $\sim$ is an equivalence relation on $\mathbb{Z}^{d}$. The distinct equivalence classes of $\mathbb{Z}^{d}$ under $\sim$ are called the \textbf{irreducible orbit classes }of $T$ in $\mathbb{Z}^{d}$. More generally, an \textbf{orbit class }is the union of irreducible orbit classes of $T$; we say an orbit class is \textbf{reducible }precisely when it can be written as a union of two or more non-empty orbit classes, and say it is \textbf{irreducible }when no such decomposition exists. \end{defn} \vphantom{} For those who have not seen this concept before, it is equivalent to the idea of a watershed in earth sciences and hydrology: all elements of a single irreducible orbit class of $T$ have the same long-term behavior under iteration by $T$. \begin{prop} A set $V\subseteq\mathbb{Z}^{d}$ is an orbit class of $T$ (possibly reducible) if and only if $T^{-1}\left(V\right)=V$. \end{prop} \vphantom{} Using orbit classes, we can distinguish between sets according to how $T$ behaves on them. \begin{defn} We say $\mathbf{x}\in\mathbb{Z}^{d}$ is a \textbf{periodic point} of $T$ whenever there is an integer $n\geq1$ so that $T^{\circ n}\left(\mathbf{x}\right)=\mathbf{x}$. The \textbf{period }of $\mathbf{x}$ is the smallest integer $n\geq1$ for which $T^{\circ n}\left(\mathbf{x}\right)=\mathbf{x}$ holds true. The set $\left\{ T^{\circ m}\left(\mathbf{x}\right):m\geq0\right\} $ (the forward orbit of $\mathbf{x}$) is then called the \textbf{cycle generated by $\mathbf{x}$}. More generally, we say a set $\Omega\subseteq\mathbb{Z}^{d}$ is a \textbf{cycle }of $T$ whenever there is an $\mathbf{x}\in\Omega$ which is a periodic point of $T$ such that $\Omega=\left\{ T^{\circ m}\left(\mathbf{x}\right):m\geq0\right\} $. \end{defn} \vphantom{} The following facts are easily verified directly from the above definitions: \begin{prop} Let $\Omega\subseteq\mathbb{Z}^{d}$ be a cycle of $\Omega$. Then: \vphantom{} I. $\Omega$ is a finite set. \vphantom{} II. Every element of $\Omega$ is a periodic point of $T$ of period $\left|\Omega\right|$. \vphantom{} III. $T\mid_{\Omega}$ (the restriction of $T$ to $\Omega$) is a bijection of $\Omega$. \vphantom{} IV. $\Omega\subseteq T^{-1}\left(\Omega\right)$ \vphantom{} V. Either there exists an $\mathbf{x}\in\mathbb{Z}^{d}\backslash\Omega$ so that $T\left(\mathbf{x}\right)\in\Omega$ or $T^{-1}\left(\Omega\right)=\Omega$. \end{prop} (IV) justifies the next bit of terminology: \begin{defn} Given a cycle $\Omega$ of $T$, we say $\Omega$ is \textbf{attracting }if $T^{-1}\left(\Omega\right)\backslash\Omega$ is non-empty\footnote{That is, if there are points not in $\Omega$ which $T$ sends into $\Omega$.}; we say $\Omega$ is \textbf{isolated }(or, in analogy with continuous dynamics, \textbf{repelling}) whenever $T^{-1}\left(\Omega\right)=\Omega$. Additionally, we say $\mathbf{x}\in\mathbb{Z}^{d}$ is a \textbf{pre-periodic point }of $T$ whenever there is an $n\geq0$ so that $T^{\circ n}\left(\mathbf{x}\right)$ is a periodic point of $T$ (where, recall, $T^{\circ0}$ is defined to be the identity map). Given a cycle $\Omega$, we say $\mathbf{x}$ is \textbf{pre-periodic into $\Omega$ }if $T^{\circ n}\left(\mathbf{x}\right)\in\Omega$ occurs for some $n\geq0$. \end{defn} \begin{prop} Let $\mathbf{x}\in\mathbb{Z}^{d}$ be arbitrary. Then, either $\mathbf{x}$ is a pre-periodic point of $T$, or the forward orbit of $\mathbf{x}$ is unbounded. In particular, if $\mathbf{x}$ is not a pre-periodic point, then: \begin{equation} \lim_{n\rightarrow\infty}\left\Vert T^{\circ n}\left(\mathbf{x}\right)\right\Vert _{\infty}=+\infty\label{eq:Divergent Trajectory Definition} \end{equation} where for any $\mathbf{y}=\left(y_{1},\ldots,y_{d}\right)\in\mathbb{Z}^{d}$: \begin{equation} \left\Vert \mathbf{y}\right\Vert _{\infty}\overset{\textrm{def}}{=}\max\left\{ \left|y_{1}\right|,\ldots,\left|y_{d}\right|\right\} \label{eq:Definition of ell infinity norm on Z^d} \end{equation} \end{prop} Proof: If $\mathbf{x}$ is not pre-periodic, then each element in the forward orbit of $\mathbf{x}$ is unique; that is: for any integers $m,n\geq0$, we have that $T^{\circ n}\left(\mathbf{x}\right)=T^{\circ m}\left(\mathbf{x}\right)$ occurs if and only if $m=n$. So, letting $N>0$ be arbitrary, note that there are only finitely many $\mathbf{y}\in\mathbb{Z}^{d}$ with $\left\Vert \mathbf{y}\right\Vert _{\infty}\leq N$. As such, if $\mathbf{x}$ is not pre-periodic, there can exist at most finitely many $n\geq0$ for which $\left\Vert T^{\circ n}\left(\mathbf{x}\right)\right\Vert _{\infty}\leq N$. So, for any $N$, $\left\Vert T^{\circ n}\left(\mathbf{x}\right)\right\Vert _{\infty}$ will be strictly greater than $N$ for all sufficiently large $n$. This establishes the limit (\ref{eq:Divergent Trajectory Definition}). Q.E.D. \begin{defn} We say $\mathbf{x}$ is a \textbf{divergent point }of $T$, or is \textbf{divergent under $T$ }whenever the limit (\ref{eq:Divergent Trajectory Definition})\textbf{ }occurs. We use the terms \textbf{divergent trajectory}\index{divergent!trajectory}\textbf{ }and\textbf{ divergent orbit }to refer to a set of the form $\left\{ T^{\circ n}\left(\mathbf{x}\right):n\geq0\right\} $ where $\mathbf{x}$ is divergent under $T$. \end{defn} \begin{defn}[\textbf{Types of orbit classes}] Let $V\subseteq\mathbb{Z}^{d}$ be an orbit class of $T$. We say $V$ is: \vphantom{} I. \textbf{attracting},\textbf{ }whenever it contains an attracting cycle of $T$. \vphantom{} II. \textbf{isolated}, whenever it contains an isolated cycle of $T$. \vphantom{} III. \index{divergent!orbit class}\textbf{divergent}, whenever it contains a divergent trajectory of $T$. \end{defn} \vphantom{} It is a straight-forward and useful exercise to prove the following: \begin{lem} If $V\subseteq\mathbb{Z}^{d}$ is an irreducible orbit class of $T$, then one and only one of the following occurs: \vphantom{} I. $V$ is attracting. Additionally, there exists a unique attracting cycle $\Omega\subset V$ so that every $\mathbf{x}\in V$ is pre-periodic into $\Omega$. \vphantom{} II. $V$ is isolated. Additionally, $V$ itself is then an isolated cycle of $T$. \vphantom{} III. $V$ is divergent. Additionally, $\lim_{n\rightarrow\infty}\left\Vert T^{\circ n}\left(\mathbf{x}\right)\right\Vert _{\infty}=+\infty$ for all $\mathbf{x}\in V$. \end{lem} \vphantom{} Since the irreducible orbit classes of $T$ partition $\mathbb{Z}^{d}$, they furnish a complete characterization of $T$'s dynamics on $\mathbb{Z}^{d}$. \begin{thm} \label{thm:orbit classes partition domain}Let $T:\mathbb{Z}^{d}\rightarrow\mathbb{Z}^{d}$ be a map. Then, the irreducible orbit classes constitute a partition of $\mathbb{Z}^{d}$ into at most countably infinitely many pair-wise disjoint sets. In particular, for any $\mathbf{x}\in\mathbb{Z}^{d}$, exactly one of the following occurs: \vphantom{} I. $\mathbf{x}$ is a periodic point of $T$ in an isolated cycle. \vphantom{} II. $\mathbf{x}$ is a pre-periodic point of $T$ which is either in or eventually iterated \emph{into }an attracting cycle of $T$. \vphantom{} III. $\mathbf{x}$ has a divergent trajectory under $T$. \end{thm} \newpage{} \section{\label{sec:1.3 Crash course in ultrametric analysis}$p$-adic Numbers and Ultrametric Analysis} THROUGHOUT THIS SECTION, $p$ DENOTES A PRIME NUMBER. \subsection{\label{subsec:1.3.1. The-p-adics-in-a-nutshell}The $p$-adics in a Nutshell} We owe the conception of the $p$-adic numbers to the mathematical work of\emph{ }Kurt Hensel\index{Hensel, Kurt} in 1897. Although there are many ways of defining these numbers, I find Hensel's own approach to be the most enlightening: the $p$-adics as \emph{power series in the ``variable'' $p$} \cite{Gouvea's introudction to p-adic numbers book}. In fact, many of the fundamental concepts associated with power series turn out to be the inspiration for much of the paradigm-shifting developments in number theory and algebraic geometry during the twentieth century. Likely the most important of these is the simple observation that, in general, a power series for an analytic function $f:\mathbb{C}\rightarrow\mathbb{C}$ about a given point $z_{0}\in\mathbb{C}$ gives us a formula for the function which is valid only in a \emph{neighborhood} of $z_{0}$. Quite often, we need to compute power series expansion about different points (a process known as \emph{analytic continuation}) in order to get formulae for the function on all the points in its domain. That is to say, the function itself is a \emph{global }object, which we study \emph{locally }(near the point $z_{0}$) by expanding it in a power series about a point. \begin{defn} The set of $p$-\textbf{adic integers}\index{$p$-adic!integers}, denoted \nomenclature{$\mathbb{Z}_{p}$}{the set of $p$-adic integers \nopageref}$\mathbb{Z}_{p}$, is the set of all (formal) sums $\mathfrak{z}$ of the form: \begin{equation} \mathfrak{z}=c_{0}+c_{1}p+c_{2}p^{2}+\ldots\label{eq:Definition of a p-adic integer} \end{equation} where the $c_{n}$s are elements of the set $\left\{ 0,1,\ldots,p-1\right\} $. \end{defn} \vphantom{} We can think of $p$-adic integers as ``power series in $p$''. Note that every non-negative integer is automatically a $p$-adic integer, seeing as every non-negative integer $x$ can be uniquely written as a \emph{finite }sum $\mathfrak{z}=\sum_{n=0}^{N}c_{n}p^{n}$, where the $c_{n}$s are the\index{$p$-adic!digits} $p$\textbf{-ary/adic digits }of $\mathfrak{z}$. The $p$-adic representation of $\mathfrak{z}\in\mathbb{Z}_{p}$ is the expression: \begin{equation} \mathfrak{z}=\centerdot_{p}c_{0}c_{1}c_{2}\ldots\label{eq:Definition of the p-adic digit representation} \end{equation} where the subscript $p$ is there to remind us that we are in base $p$; this subscript can (and will) be dropped when there is no confusion as to the value of $p$. $\mathbb{Z}_{p}$ becomes a commutative, unital ring when equipped with the usual addition and multiplication operations, albeit with the caveat that \begin{equation} cp^{n}=\left[c\right]_{p}p^{n}+\left(\frac{c-\left[c\right]_{p}}{p}\right)p^{n+1} \end{equation} for all $n,c\in\mathbb{N}_{0}$. \begin{example} As an example, for $a\in\left\{ 0,p-1\right\} $: \begin{equation} \centerdot_{p}00\left(p+a\right)=\left(p+a\right)p^{2}=ap^{2}+1p^{3}=\centerdot_{p}00a1 \end{equation} where, in $\centerdot_{p}00\left(p+a\right)$, $p+a$ is the value of the third $p$-adic digit, and where $\left[c\right]_{p}\in\left\{ 0,\ldots,p-1\right\} $ denotes the residue class of $c$ mod $p$. That is to say, like an odometer, \emph{we carry over to the next $p$-adic digit's place whenever a digit reaches $p$}. Thus, in the $2$-adics: \begin{eqnarray*} 3 & = & 1\times2^{0}+1\times2^{1}=\centerdot_{2}11\\ 5 & = & 1\times2^{0}+0\times2^{1}+1\times2^{2}=\centerdot_{2}101 \end{eqnarray*} When we add these numbers, we ``carry over the $2$'': \[ 3+5=\centerdot_{2}11+\centerdot_{2}101=\centerdot_{2}211=\centerdot_{2}021=\centerdot_{2}002=\centerdot_{2}0001=1\times2^{3}=8 \] Multiplication is done similarly. \end{example} \vphantom{} Using this arithmetic operation, we can write negative integers $p$-adically. \begin{example} The $p$-adic number $\mathfrak{y}$ whose every $p$-adic digit is equal to $p-1$: \[ \mathfrak{y}=\centerdot_{p}\left(p-1\right)\left(p-1\right)\left(p-1\right)\left(p-1\right)\ldots=\sum_{n=0}^{\infty}\left(p-1\right)p^{n} \] satisfies: \begin{eqnarray*} 1+\mathfrak{y} & = & \centerdot_{p}\left(p\right)\left(p-1\right)\left(p-1\right)\left(p-1\right)\ldots\\ & = & \centerdot_{p}0\left(p-1+1\right)\left(p-1\right)\left(p-1\right)\ldots\\ & = & \centerdot_{p}0\left(p\right)\left(p-1\right)\left(p-1\right)\ldots\\ & = & \centerdot_{p}00\left(p-1+1\right)\left(p-1\right)\ldots\\ & = & \centerdot_{p}00\left(p-1+1\right)\left(p-1\right)\ldots\\ & = & \centerdot_{p}000\left(p-1+1\right)\ldots\\ & \vdots\\ & = & \centerdot_{p}000000\ldots\\ & = & 0 \end{eqnarray*} and thus, $\mathfrak{y}=-1$ in $\mathbb{Z}_{p}$. \end{example} \vphantom{} The beauty of the power series conception of these numbers is that it makes such formulae explicit: \begin{eqnarray*} 1+y & = & 1+\sum_{n=0}^{\infty}\left(p-1\right)p^{n}\\ & = & 1+\sum_{n=0}^{\infty}p^{n+1}-\sum_{n=0}^{\infty}p^{n}\\ & = & p^{0}+\sum_{n=1}^{\infty}p^{n}-\sum_{n=0}^{\infty}p^{n}\\ & = & \sum_{n=0}^{\infty}p^{n}-\sum_{n=0}^{\infty}p^{n}\\ & = & 0 \end{eqnarray*} Consequently, just as when given a power series $\sum_{n=0}^{\infty}a_{n}z^{n}$, we can compute its multiplicative inverse $\sum_{n=0}^{\infty}b_{n}z^{n}$ recursively by way of the equations: \begin{eqnarray*} 1 & = & \left(\sum_{m=0}^{\infty}a_{m}z^{m}\right)\left(\sum_{n=0}^{\infty}b_{n}z^{n}\right)=\sum_{k=0}^{\infty}\left(\sum_{n=0}^{k}a_{n}b_{k-n}\right)z^{n}\\ & \Updownarrow\\ 1 & = & a_{0}b_{0}\\ 0 & = & a_{0}b_{1}+a_{1}b_{0}\\ 0 & = & a_{0}b_{2}+a_{1}b_{1}+a_{2}b_{0}\\ & \vdots \end{eqnarray*} we can use the same formula to compute the multiplicative inverse of a given $p$-adic integer\textemdash \emph{assuming} it exists. Working through the above equations, it can be seen that the $p$-adic integer $\mathfrak{z}=\sum_{n=0}^{\infty}c_{n}p^{n}$ has a multiplicative inverse if and only if $c_{0}\in\left\{ 0,\ldots,p-1\right\} $ is multiplicatively invertible mod $p$ (i.e., $c_{0}$ is co-prime to $p$). This is one of several reasons why we prefer to study $p$-adic integers for prime $p$: every non-zero residue class mod $p$ is multiplicatively invertible modulo $p$. \begin{defn} Just as we can go from the ring of power series to the field of Laurent series, we can pass from the ring of $p$-adic integers $\mathbb{Z}_{p}$ to the field of \textbf{$p$-adic (rational) numbers}\index{$p$-adic!rational numbers} \nomenclature{$\mathbb{Q}_{p}$}{the set of $p$-adic rational numbers \nopageref}$\mathbb{Q}_{p}$\textbf{ }by considering ``Laurent series'' in $p$. Every $\mathfrak{z}\in\mathbb{Q}_{p}$ has a unique representation as: \begin{equation} \mathfrak{z}=\sum_{n=n_{0}}^{\infty}c_{n}p^{n}\label{eq:Laurent series representation of a p-adic rational number} \end{equation} for some $n_{0}\in\mathbb{Z}$. We write the ``fractional part'' of $\mathfrak{z}$ (the terms with negative power of $p$) to the right of the $p$-adic point $\centerdot_{p}$. \end{defn} \begin{example} Thus, the $3$-adic number: \begin{equation} \frac{1}{3^{2}}+\frac{2}{3^{1}}+0\times3^{0}+1\times3^{1}+1\times3^{2}+1\times3^{3}+\ldots \end{equation} would be written as: \begin{equation} 12\centerdot_{3}0\overline{1}=12\centerdot_{3}0111\ldots \end{equation} where, as usual, the over-bar indicates that we keep writing $1$ over and over again forever. \end{example} \begin{defn} We can equip $\mathbb{Q}_{p}$ with the structure of a metric space by defining the \index{$p$-adic!valuation}\textbf{$p$-adic valuation} \textbf{$v_{p}$ }and $p$\textbf{-absolute value}\footnote{Although this text and most texts on non-archimedean analysis by native-English-speaking authors make a clear distinction between the $p$-adic absolute value and the $p$-adic valuation, this distinction is not universal adhered to. A non-negligible proportion of the literature (particularly when authors of Russian extraction are involved) use the word ``valuation'' interchangeably, to refer to either $v_{p}$ or $\left|\cdot\right|_{p}$, depending on the context.}\textbf{ }$\left|\cdot\right|_{p}$: \begin{equation} v_{p}\left(\sum_{n=n_{0}}^{\infty}c_{n}p^{n}\right)\overset{\textrm{def}}{=}n_{0}\label{eq:Definition of the p-adic valuation} \end{equation} \begin{equation} \left|\sum_{n=n_{0}}^{\infty}c_{n}p^{n}\right|_{p}\overset{\textrm{def}}{=}p^{-n_{0}}\label{eq:Definition of the p-adic absolute value} \end{equation} that is, $\left|\mathfrak{z}\right|_{p}=p^{-v_{p}\left(\mathfrak{z}\right)}$. We also adopt the convention that $v_{p}\left(0\right)\overset{\textrm{def}}{=}\infty$. The \textbf{$p$-adic metric}\index{p-adic@\textbf{$p$}-adic!metric}\textbf{ }is then the distance formula defined by the map: \begin{equation} \left(\mathfrak{z},\mathfrak{y}\right)\in\mathbb{Z}_{p}\times\mathbb{Z}_{p}\mapsto\left|\mathfrak{z}-\mathfrak{y}\right|_{p}\label{eq:Definition of the p-adic metric} \end{equation} This metric (also known as an \textbf{ultrametric})\textbf{ }is said to \textbf{non-archimedean} because, in addition to the triangle inequality we all know and love, it also satisfies the \textbf{strong triangle inequality} (also called the \textbf{ultrametric inequality}): \begin{equation} \left|\mathfrak{z}-\mathfrak{y}\right|_{p}\leq\max\left\{ \left|\mathfrak{z}\right|_{p},\left|\mathfrak{y}\right|_{p}\right\} \label{eq:the Strong Triangle Inequality} \end{equation} \emph{Crucially}\textemdash and I \emph{cannot} emphasize this enough\textemdash (\ref{eq:the Strong Triangle Inequality}) holds \emph{with equality} whenever $\left|\mathfrak{z}\right|_{p}\neq\left|\mathfrak{y}\right|_{p}$. The equality of (\ref{eq:the Strong Triangle Inequality}) when $\left|\mathfrak{z}\right|_{p}\neq\left|\mathfrak{y}\right|_{p}$ is one of the most subtly powerful tricks in ultrametric analysis, especially when we are trying to contradict an assumed upper bound. Indeed, this very method is at the heart of my proof of th $\left(p,q\right)$-adic Wiener Tauberian Theorem. \end{defn} \begin{rem} Although it might seem unintuitive that a \emph{large} power of $p$ should have a very small $p$-adic absolute value, this viewpoint becomes extremely natural when viewed through Hensel's original conception of $p$-adic integers as number theoretic analogues of power series \cite{Hensel-s original article,Gouvea's introudction to p-adic numbers book,Journey throughout the history of p-adic numbers}. It is a well-known and fundamental fact of complex analysis that a function $f:U\rightarrow\mathbb{C}$ holomorphic on an open, connected, non-empty set $U\subseteq\mathbb{C}$ which possesses a zero of infinite degree in $U$ must be identically zero on $U$. In algebraic terms, if $f$ has a zero at $z_{0}$ and $f$ is not identically zero, then $f\left(z\right)/\left(z-z_{0}\right)^{n}$ can only be divided by the binomial $z-z_{0}$ finitely many times before we obtain a function which is non-zero at $z_{0}$. As such, a function holomorphic on an open neighborhood of $z_{0}$ which remains holomorphic after being divided by $z-z_{0}$ arbitrarily many times is necessarily the constant function $0$. Hensel's insight was to apply this same reasoning to numbers. Indeed, the uniform convergence to $0$ of the function sequence $\left\{ z^{n}\right\} _{n\geq0}$ on any compact subset of the open unit disk $\mathbb{D}\subset\mathbb{C}$ is spiritually equivalent to the convergence of the sequence $\left\{ p^{n}\right\} _{n\geq0}$ to $0$ in the $p$-adics: just as the only holomorphic function on $\mathbb{D}$ divisible by arbitrarily large powers of $z$ is the zero function, the only $p$-adic integer divisible by arbitrarily high powers of $p$ is the integer $0$. Indeed, in the context of power series and Laurent series\textemdash say, a series $\sum_{n=n_{0}}^{\infty}c_{n}\left(z-z_{0}\right)^{n}$ about some $z_{0}\in\mathbb{C}$\textemdash the $p$-adic valuation corresponds to the \textbf{zero degree }of the series at $z_{0}$. If $n_{0}$ is negative, then the function represented by that power series has a pole of order $-n_{0}$ at $z_{0}$; if $n_{0}$ is positive, then the function represented by that power series has a \emph{zero }of order $n_{0}$ at $z_{0}$. Thus, for all $n\in\mathbb{Z}$: \begin{equation} \left|p^{n}\right|_{p}=p^{-n} \end{equation} This is especially important, seeing as the $p$-adic absolute value is a multiplicative group homomorphism from $\mathbb{Z}_{p}$ to $\mathbb{R}^{+}$: \begin{equation} \left|xy\right|_{p}=\left|x\right|_{p}\left|y\right|_{p}\label{eq:Multiplicativity of p-adic absolute value} \end{equation} as such, the level sets of $\mathbb{Z}_{p}$ are: \begin{equation} \left\{ x\in\mathbb{Z}_{p}:\left|x\right|_{p}=p^{-n}\right\} =p^{n}\mathbb{Z}_{p}\overset{\textrm{def}}{=}\left\{ p^{n}y:y\in\mathbb{Z}_{p}\right\} \label{eq:Definition of p^n times Z_p} \end{equation} for all $n\in\mathbb{N}_{0}$. \end{rem} \vphantom{} Returning to the matter at hand, by using absolute values, we see that the $p$-adic integers are precisely those elements of $\mathbb{Q}_{p}$ with $p$-adic absolute value $\leq1$. Consequentially, the metric space obtained by equipping $\mathbb{Z}_{p}$ with the $p$-adic metric is \emph{compact}. $\mathbb{Q}_{p}$, meanwhile, is locally compact, just like $\mathbb{R}$ and $\mathbb{C}$. \begin{defn} We write \nomenclature{$\mathbb{Z}_{p}^{\times}$}{the group of multiplicatively invertible $p$-adic integers \nopageref}$\mathbb{Z}_{p}^{\times}$ to denote the set of all units of $\mathbb{Z}_{p}$\textemdash that is, elements of $\mathbb{Z}_{p}$ whose reciprocals are contained in $\mathbb{Z}_{p}$. This is an abelian group under multiplication, with $1$ as its identity element. Note, also, that: \begin{equation} \mathbb{Z}_{p}^{\times}=\left\{ \mathfrak{z}\in\mathbb{Z}_{p}:\left|\mathfrak{z}\right|_{p}=1\right\} \end{equation} \end{defn} \begin{defn} \textbf{Congruences }are extremely important when working with $p$-adic integers; this is an intrinsic feature of the ``projective limit'' more algebraic texts frequently use to define the $p$-adic integers. As we saw, every $p$-adic integer $\mathfrak{z}$ can be written uniquely as: \begin{equation} \mathfrak{z}=\sum_{n=0}^{\infty}c_{n}p^{n}\label{eq:p-adic series representation of a p-adic integer} \end{equation} for constants $\left\{ c_{n}\right\} _{n\geq0}\subseteq\left\{ 0,\ldots,p-1\right\} $. The series representation (\ref{eq:p-adic series representation of a p-adic integer}) of a $p$-adic integer $\mathfrak{z}$is sometimes called the \textbf{Hensel series}\index{Hensel series}\index{series!Hensel}series\textbf{ }or \textbf{Henselian series }of $\mathfrak{z}$; Amice uses this terminology, for example \cite{Amice}. With this representation, given any integer $m\geq0$, we can then define \nomenclature{$\left[\mathfrak{z}\right]_{p^{n}}$}{Projection of a $p$-adic integer modulo $p^n$ nomnorefpage} $\left[\mathfrak{z}\right]_{p^{m}}$, the \textbf{projection of $\mathfrak{z}$ mod $p^{m}$} like so: \begin{equation} \left[\mathfrak{z}\right]_{p^{m}}\overset{\textrm{def}}{=}\sum_{n=0}^{m-1}c_{n}p^{n}\label{eq:Definition of the projection of z mod p to the m} \end{equation} where the right-hand side is defined to be $0$ whenever $m=0$. Since $\left[\mathfrak{z}\right]_{p^{m}}$ is a finite sum of integers, it itself is an integer. Given $\mathfrak{z},\mathfrak{y}\in\mathbb{Z}_{p}$, we write $\mathfrak{z}\overset{p^{m}}{\equiv}\mathfrak{y}$ if and only if $\left[\mathfrak{z}\right]_{p^{m}}=\left[\mathfrak{y}\right]_{p^{m}}$. Moreover, there is an equivalence between congruences and absolute values: \begin{equation} \mathfrak{z}\overset{p^{m}}{\equiv}\mathfrak{y}\Leftrightarrow\left|\mathfrak{z}-\mathfrak{y}\right|_{p}\leq p^{-m} \end{equation} There is a very useful notation for denoting subsets (really, ``open neighborhoods'') of $p$-adic integers. Given $\mathfrak{z}\in\mathbb{Z}_{p}$ and $n\in\mathbb{N}_{0}$, we write: \begin{equation} \mathfrak{z}+p^{n}\mathbb{Z}_{p}\overset{\textrm{def}}{=}\left\{ \mathfrak{y}\in\mathbb{Z}_{p}:\mathfrak{y}\overset{p^{n}}{\equiv}\mathfrak{z}\right\} +\left\{ \mathfrak{y}\in\mathbb{Z}_{p}:\left|\mathfrak{z}-\mathfrak{y}\right|_{p}\leq p^{-n}\right\} \label{eq:Definition of co-set notation for p-adic neighborhoods} \end{equation} In particular, note that: \begin{align*} \mathfrak{z}+p^{n}\mathbb{Z}_{p} & =\mathfrak{y}+p^{n}\mathbb{Z}_{p}\\ & \Updownarrow\\ \mathfrak{z} & \overset{p^{n}}{\equiv}\mathfrak{y} \end{align*} Additionally\textemdash as is crucial for performing integration over the $p$-adics\textemdash for any $n\geq0$, we can partition $\mathbb{Z}_{p}$ like so: \begin{equation} \mathbb{Z}_{p}=\bigcup_{k=0}^{p^{n}-1}\left(k+p^{n}\mathbb{Z}_{p}\right) \end{equation} where the $k+p^{n}\mathbb{Z}_{p}$s are pair-wise disjoint with respect to $k$. \end{defn} \vphantom{} Finally, it is worth mentioning that $\mathbb{N}_{0}$ is dense in $\mathbb{Z}_{p}$, and that $\mathbb{Q}$ is dense in $\mathbb{Q}_{p}$. This makes both $\mathbb{Z}_{p}$ and $\mathbb{Q}_{p}$ into separable topological spaces. Moreover, as topological spaces, they\textemdash and any field extension thereof\textemdash are \emph{totally disconnected}. This has profound implications for $p$-adic analysis, and for ultrametric analysis in general. \subsection{\label{subsec:1.3.2. Ultrametrics-and-Absolute}An Introduction to Ultrametric Analysis} While the $p$-adic numbers are surely the most well-known non-archimedean spaces, they are far from the only ones. The study of function theory, calculus, and the like on generic non-archimedean spaces is sometimes called \textbf{Ultrametric analysis}\index{ultrametric!analysis}. As will be addressed at length in the historical essay of Subsection \ref{subsec:3.1.1 Some-Historical-and}, the sub-disciplines that go by the names of non-archimedean analysis, $p$-adic analysis, ultrametric analysis, and the like are sufficiently divers that it is worth being aware of the undercurrent of common terminology. \begin{defn} Let $X$ be a set, and let $d:X\times X\rightarrow\left[0,\infty\right)$ be a metric; that is, a function satisfying: \vphantom{} I. $d\left(x,y\right)\geq0$ $\forall x,y\in X$, with equality if and only if $x=y$; \vphantom{} II. $d\left(x,y\right)=d\left(y,x\right)$ $\forall x,y\in X$; \vphantom{} III. $d\left(x,y\right)\leq d\left(x,z\right)+d\left(y,z\right)$ $\forall x,y,z\in X$. \vphantom{} We say $d$ is a \textbf{non-archimedean} \textbf{metric }or \textbf{ultrametric} whenever it satisfies the \textbf{Strong Triangle Inequality }(a.k.a. \textbf{Ultrametric Inequality}): \begin{equation} d\left(x,y\right)\leq\max\left\{ d\left(x,z\right),d\left(y,z\right)\right\} ,\forall x,y,z\in X\label{eq:Generic Strong Triangle Inequality} \end{equation} Like with the specific case of the $p$-adic ultrametric inequality, the general ultrametric inequality holds \emph{with equality} whenever $d\left(x,z\right)\neq d\left(y,z\right)$. An \index{ultrametric!space}\textbf{ultrametric space }is a pair $\left(X,d\right)$, where $X$ is a set and $d$ is an ultrametric on $X$. \end{defn} \vphantom{} The most important ultrametrics are those that arise from \textbf{absolute values} on abelian groups, particularly fields. \begin{defn} Let $K$ be an abelian group, written additively, and with $0$ as its identity element. An \textbf{absolute value}\index{absolute value (on a field)}\textbf{ }on $K$ is a function $\left|\cdot\right|_{K}:K\rightarrow\left[0,\infty\right)$ satisfying the following properties for all $x,y\in K$: \vphantom{} I. $\left|0\right|_{K}=0$; \vphantom{} II. $\left|x+y\right|_{K}\leq\left|x\right|_{K}+\left|y\right|_{K}$; \vphantom{} III. $\left|x\right|_{K}=0$ if and only if $x=0$; \vphantom{} IV. If $K$ is a ring, we also require $\left|x\cdot y\right|_{K}=\left|x\right|_{K}\cdot\left|y\right|_{K}$. \vphantom{} Finally, we say that $\left|\cdot\right|_{K}$ is a \textbf{non-archimedean absolute value} if, in addition to the above, $\left|\cdot\right|_{K}$ satisfies the \textbf{Ultrametric Inequality}\index{ultrametric!inequality}\index{triangle inequality!strong}: \vphantom{} V. $\left|x+y\right|_{K}\leq\max\left\{ \left|x\right|_{K},\left|y\right|_{K}\right\} $ (with equality whenever $\left|x\right|_{K}\neq\left|y\right|_{K}$),. \vphantom{} If (V) is not satisfied, we call $\left|\cdot\right|_{K}$ an \textbf{archimedean absolute value}. Finally, note that if $K$ is a field, any absolute value $\left|\cdot\right|_{K}$ on a field induces a metric $d$ on $K$ by way of the formula $d\left(x,y\right)=\left|x-y\right|_{K}$. We call the pair $\left(K,\left|\cdot\right|_{K}\right)$ a \textbf{valued group }(resp. \textbf{valued ring}; resp. \index{valued field}\textbf{valued field}) whenever $K$ is an abelian group (resp. ring\footnote{The ring need not be commutative.}; resp., field), $\left|\cdot\right|_{K}$ is an absolute value on and say it is \textbf{archimedean }whenever $\left|\cdot\right|_{K}$ is archimedean, and say that it is \textbf{non-archimedean }whenever $\left|\cdot\right|_{K}$. Let $K$ be a non-archimedean valued ring. \vphantom{} I. Following Schikhof \cite{Schikhof Banach Space Paper}, let $B_{K}\overset{\textrm{def}}{=}\left\{ x\in K:\left|x\right|_{K}\leq1\right\} $ and let $B_{K}^{-}\overset{\textrm{def}}{=}\left\{ x\in K:\left|x\right|_{K}<1\right\} $. Both $B_{K}$ and $B_{K}^{-}$ are rings under the addition and multiplication operations of $K$, with $B_{K}^{-}$ being an ideal\footnote{In fact, $B_{K}^{-}$ is a maximal ideal in $B_{K}$, and is the unique non-zero prime ideal of $B_{K}$. If $K$ is a ring, $K$ is then called a \textbf{local ring}; if $K$ is a field, it is then called a \textbf{local field}, in which case $B_{K}$ is the ring of $K$-integers, and $K$ is the field of fractions of $B_{K}$.} in $B_{K}$. The ring $B_{K}/B_{K}^{-}$ obtained by quotienting $B_{K}$ out by $B_{K}^{-}$ is called the \textbf{residue field }/ \textbf{residue class field }of $K$. We say $K$ is\textbf{ $p$-adic} (where $p$ is a prime) when the residue field of $K$ has characteristic $p$. In the case $K$ is $p$-adic, in an abuse of notation, we will write $\left|\cdot\right|_{p}$ to denote the absolute value on $K$. Also, note that if $K$ is a field, $B_{K}$ is then equal to $\mathcal{O}_{K}$, the \textbf{ring of integers }of $K$. \vphantom{} II. The set: \begin{equation} \left|K\backslash\left\{ 0\right\} \right|_{K}\overset{\textrm{def}}{=}\left\{ \left|x\right|_{K}:x\in K\backslash\left\{ 0\right\} \right\} \label{eq:Definition of the value group of a field} \end{equation} is called the \index{value group}\textbf{value group}\footnote{Not to be confused with a \emph{valued }group.}\textbf{ }of $K$. This group is said to be \textbf{dense }if it is dense in the interval $\left(0,\infty\right)\subset\mathbb{R}$ in the standard topology of $\mathbb{R}$, and is said to be \textbf{discrete }if it is not dense. \end{defn} \vphantom{} Both the residue field and the value group of $K$ play a crucial role in some of the fundamental properties of analysis on $K$ (or on normed vector spaces over $K$). For example, they completely determine whether or not $K$ is locally compact. \begin{thm} A non-archimedean valued field $K$ is locally compact if and only if its residue field is finite and its value group is discrete\footnote{\textbf{Theorem 12.2} from \cite{Ultrametric Calculus}.}. \end{thm} \vphantom{} The term ``non-archimedean'' comes from the failure of the \textbf{archimedean property} of classical analysis, that being the intuitive notion that \emph{lengths add up}. Non-archimedean spaces are precisely those metric spaces where lengths \emph{need not} add up. In particular, we have the following delightful result: \begin{thm} Let $\left(K,\left|\cdot\right|_{K}\right)$ be a valued field, and let $1$ denote the multiplicative identity element of $K$. Then, $\left(K,\left|\cdot\right|_{K}\right)$ is non-archimedean if and only if: \begin{equation} \left|1+1\right|_{K}\leq\left|1\right|_{K} \end{equation} \end{thm} Proof: Exercise. Q.E.D. \vphantom{} The following list, adapted from Robert's book \cite{Robert's Book}, gives an excellent summary of the most important features of ultrametric analysis. \begin{fact}[\textbf{The Basic Principles of Ultrametric Analysis}] \label{fact:Principles of Ultrametric Analysis}Let $\left(K,\left|\cdot\right|_{K}\right)$ be a non-archimedean valued group with additive identity element $0$. Then: \vphantom{} I. \textbf{\emph{The Strongest Wins}}\emph{:} \vphantom{} \begin{equation} \left|x\right|_{K}>\left|y\right|_{K}\Rightarrow\left|x+y\right|_{K}=\left|x\right|_{K}\label{eq:The Strongest Wins} \end{equation} II. \textbf{\emph{Equilibrium}}\emph{:} All triangles are isosceles (or equilateral): \begin{equation} a+b+c=0\textrm{ \& }\left|c\right|_{K}<\left|b\right|_{K}\Rightarrow\left|a\right|_{K}=\left|b\right|_{K}\label{eq:Equilibrium} \end{equation} \vphantom{} III. \textbf{\emph{Competition}}\emph{:} If: \[ a_{1}+\cdots+a_{n}=0 \] then there are distinct $i,j$ so that $\left|a_{i}\right|_{K}=\left|a_{j}\right|_{K}=\max_{k}\left|a_{k}\right|_{K}$. \vphantom{} IV. \textbf{\emph{The Freshman's Dream}}\emph{:} If the metric space $\left(K,\left|\cdot\right|_{K}\right)$ is complete, a series $\sum_{n=0}^{\infty}a_{n}$ converges in $K$ if and only if $\left|a_{n}\right|_{K}\rightarrow0$ as $n\rightarrow\infty$. Consequently: \vphantom{} i. (The infinite version of (III) holds) $\sum_{n=0}^{\infty}a_{n}=0$ implies there are distinct $i,j$ so that $\left|a_{i}\right|_{K}=\left|a_{j}\right|_{K}=\max_{k}\left|a_{k}\right|_{K}$. \vphantom{} ii. The convergence of $\sum_{n=0}^{\infty}\left|a_{n}\right|_{K}$ in $\mathbb{R}$ implies the convergence of $\sum_{n=0}^{\infty}a_{n}$ in $K$, but the converse is not true: $\sum_{n=0}^{\infty}a_{n}$ converging in $K$ need not imply $\sum_{n=0}^{\infty}\left|a_{n}\right|_{K}$ converges in $\mathbb{R}$. \vphantom{} iii. For any $K$-convergent series $\sum_{n=0}^{\infty}a_{n}$: \begin{equation} \left|\sum_{n=0}^{\infty}a_{n}\right|_{K}\leq\sup_{n\geq0}\left|a_{n}\right|_{K}=\max_{n\geq0}\left|a_{n}\right|_{K}\label{eq:Series Estimate} \end{equation} The right-most equality indicates that there is going to be an $n$ for which the absolute value is maximized. \vphantom{} V. \textbf{\emph{The Sophomore's Dream}}\emph{:} A sequence $\left\{ a_{n}\right\} _{n\geq0}$ is Cauchy if and only if $\left|a_{n+1}-a_{n}\right|_{K}\rightarrow0$ as $n\rightarrow\infty$. \vphantom{} \vphantom{} VI. \textbf{\emph{Stationarity of the absolute value}}\emph{:} If $a_{n}$ converges to $a$ in $K$, and if $a\neq0$, then there is an $N$ so that $\left|a_{n}\right|_{K}=\left|a\right|_{K}$ for all $n\geq N$. \end{fact} \vphantom{} We also have the following results regarding infinite series: \begin{prop}[\index{series!re-arrangement}\textbf{Series re-arrangement}\footnote{Given on page 74 of \cite{Robert's Book}.}] \label{prop:series re-arrangement}Let $\left(K,\left|\cdot\right|_{K}\right)$ be a complete non-archimedean valued group, and let $\left\{ a_{n}\right\} _{n\geq0}$ be a sequence in $K$ which tends to $0$ in $K$, so that $\sum_{n=0}^{\infty}a_{n}$ converges in $K$ to $s$. Then, no matter how the terms of the sum are grouped or rearranged, the resultant series will still converge in $K$ to $s$. Specifically: \vphantom{} I. For any bijection $\sigma:\mathbb{N}_{0}\rightarrow\mathbb{N}_{0}$, $\sum_{n=0}^{\infty}a_{\sigma\left(n\right)}$ converges in $K$ to $s$. \vphantom{} II. For any partition of $\mathbb{N}_{0}$ into sets $I_{1},I_{2},\ldots$, the series: \begin{equation} \sum_{k}\left(\sum_{n\in I_{k}}a_{n}\right) \end{equation} converges in $K$ to $s$. \end{prop} \begin{prop}[\index{series!interchange}\textbf{Series interchange}\footnote{Given on page 76 of \cite{Robert's Book}.}] \label{prop:series interchange} Let $\left(K,\left|\cdot\right|_{K}\right)$ be a complete non-archimedean valued group, and let $\left\{ a_{m,n}\right\} _{m,n\geq0}$ be a double-indexed sequence in $K$. If, for any $\epsilon>0$, there are only finitely many pairs $\left(m,n\right)$ so that $\left|a_{m,n}\right|_{K}>\epsilon$, then the double sum: \begin{equation} \sum_{\left(m,n\right)\in\mathbb{N}_{0}^{2}}a_{m,n} \end{equation} converges in $K$, and, moreover: \begin{equation} \sum_{\left(m,n\right)\in\mathbb{N}_{0}^{2}}a_{m,n}=\sum_{m=0}^{\infty}\left(\sum_{n=0}^{\infty}a_{m,n}\right)=\sum_{n=0}^{\infty}\left(\sum_{m=0}^{\infty}a_{m,n}\right) \end{equation} where all equalities are in $K$. \end{prop} \vphantom{} The topological properties of ultrametric spaces are drastically different from Euclidean spaces (ex: $\mathbb{R}^{n}$). \begin{defn} Let $\left(X,d\right)$ be an ultrametric space. \vphantom{} I. A closed ball\index{ball} in $X$ of radius $r$ (where $r$ is a positive real number) centered at $x\in X$, written $B\left(x,r\right)$, is the set: \begin{equation} B\left(x,r\right)\overset{\textrm{def}}{=}\left\{ y\in X:d\left(x,y\right)\leq r\right\} \label{eq:Definition of a closed ball} \end{equation} Open balls are obtained by making the inequality $\leq r$ strict ($<r$). However, as we will see momentarily, this doesn't actually amount to much of a distinction. \vphantom{} II. Given any non-empty subset $Y\subseteq X$, the \textbf{diameter }of $Y$, denoted $d\left(Y\right)$, is defined by: \begin{equation} d\left(Y\right)\overset{\textrm{def}}{=}\sup\left\{ d\left(a,b\right):a,b\in Y\right\} \label{eq:Definition of the diameter of an ultrametric set} \end{equation} \end{defn} \begin{rem} For any ball $B\left(x,r\right)\subseteq X$: \begin{equation} d\left(B\right)=\inf\left\{ r^{\prime}>0:B\left(x,r^{\prime}\right)=B\left(x,r\right)\right\} \label{eq:Diameter of an ultrametric ball in terms of its radius} \end{equation} \end{rem} \begin{prop} In\footnote{\textbf{Propositions 18.4} and \textbf{18.5} from \cite{Ultrametric Calculus}.} an ultrametric space\emph{ \index{ultrametric!balls}}$\left(X,d\right)$: \vphantom{} I. All open balls are closed, and all closed balls are open\footnote{However, it is not necessarily the case that $\left\{ y\in X:d\left(x,y\right)\leq r\right\} $ and $\left\{ y\in X:d\left(x,y\right)<r\right\} $ will be the same set. For example, if $X=\mathbb{Z}_{3}$, then $\left\{ \mathfrak{y}\in\mathbb{Z}_{3}:\left|\mathfrak{z}-\mathfrak{y}\right|_{3}\leq\frac{1}{2}\right\} =\left\{ \mathfrak{y}\in\mathbb{Z}_{3}:\left|\mathfrak{z}-\mathfrak{y}\right|_{3}\leq\frac{1}{3}\right\} $, because the value group of $\mathbb{Z}_{3}$ is $\left\{ 3^{-n}:n\in\mathbb{N}_{0}\right\} $. On the other hand, for the same reason, $\left\{ \mathfrak{y}\in\mathbb{Z}_{3}:\left|\mathfrak{z}-\mathfrak{y}\right|_{3}<\frac{1}{3}\right\} =\left\{ \mathfrak{y}\in\mathbb{Z}_{3}:\left|\mathfrak{z}-\mathfrak{y}\right|_{3}\leq\frac{1}{9}\right\} $.}. \vphantom{} II. All points inside a ball are at the center of the ball; that is, given $x,y\in K$ and $r>0$ such that $d\left(x,y\right)\leq r$, we have that $B\left(x,r\right)=B\left(y,r\right)$. \vphantom{} III. Given two balls $B_{1},B_{2}\subseteq X$, either $B_{1}\cap B_{2}=\varnothing$ or either $B_{1}\subseteq B_{2}$ or $B_{2}\subseteq B_{1}$. \vphantom{} IV. Given any ball $B\left(x,r\right)$, there are infinitely many real numbers $r^{\prime}$ for which $B\left(x,r^{\prime}\right)=B\left(x,r\right)$. \end{prop} \begin{fact} The \textbf{Heine-Borel Property}\footnote{A set is compact if and only if it is both closed and bounded.}\index{Heine-Borel property} also holds in $\mathbb{Q}_{p}$, as well as in any finite-dimensional, locally compact field extension thereof. \end{fact} \vphantom{} We also have the following result regarding open covers in an arbitrary ultrametric space: \begin{thm}[\textbf{Open Set Decomposition Theorem}\footnote{\textbf{Theorem 18.6} on page 48 of \cite{Ultrametric Calculus}.}] Let $X$ be an ultrametric space. Then, every non-empty open set $U\subseteq X$ can be written as the union of countably many pair-wise disjoint balls. \end{thm} \begin{cor} Let $X$ be a compact ultrametric space (such as $\mathbb{Z}_{p}$) can be written as the union of finitely many pair-wise disjoint balls. \end{cor} Proof: Let $U\subseteq\mathbb{Z}_{p}$ be non-empty and clopen. Since $U$ is clopen, it is closed, and since $U$ is in $\mathbb{Z}_{p}$, it is bounded in $\mathbb{Q}_{p}$. Since $\mathbb{Q}_{p}$ possess the Heine-Borel property, the closedness and boundedness of $U$ then force $U$ to be compact. Now, by the theorem, since $U$ is clopen and non-empty, it is open and non-empty, and as such, $U$ can be written as a union of countably many disjoint balls. Since the balls are clopen sets, this collection of balls forms an open cover for $U$. By the compactness of $U$, this open cover must contain a finite sub-cover, which shows that $U$ can, in fact, be written as the union of finitely many clopen balls. Q.E.D. \vphantom{} By this point, the reader might have noticed that this exposition of basic ultrametric analysis has yet to say a word about functions. This is intentional. Non-archimedean function theory is one of the central overarching concerns of this dissertation. Nevertheless, there are two important pieces of non-archimedean function theory I can introduce here and now. \begin{defn}[\textbf{Locally constant functions}\footnote{Taken from \cite{Ultrametric Calculus}.}] Let $X$ be an ultrametric space, and let $K$ be any valued field. We say a function $f:X\rightarrow K$ is \textbf{locally constant }if, for each $x\in X$ there is an open neighborhood $U\subseteq X$ containing $x$ so that $f$ is constant on $U\cap X$. \end{defn} \begin{example} The most important example we will be working with are functions $\mathbb{Z}_{p}\rightarrow K$ of the form: \begin{equation} \sum_{n=0}^{p^{N}-1}a_{n}\left[\mathfrak{z}\overset{p^{N}}{\equiv}n\right] \end{equation} where the $a_{n}$s are scalars in $K$. Such a function's value at any given $\mathfrak{z}$ is entirely determined by the value of $\mathfrak{z}$ modulo $p^{N}$. \end{example} \vphantom{} Our second appetizer in non-archimedean function theory is, in all honesty, a matter of non-archimedean \emph{functional }theory. It is also one of several reasons why integration of $p$-adic-valued functions of one or more $p$-adic variables is ``cursed'' as the kids these days like to say. \begin{defn} Letting $C\left(\mathbb{Z}_{p},K\right)$ denote the space of continuous functions from $\mathbb{Z}_{p}$ to $K$, where $K$ is a metrically complete field extension of $\mathbb{Q}_{p}$. We say a linear functional $\varphi:C\left(\mathbb{Z}_{p},K\right)\rightarrow K$ is\index{translation invariance} \textbf{translation invariant }whenever $\varphi\left(\tau_{1}\left\{ f\right\} \right)=\varphi\left(f\right)$ for all $f\in C\left(\mathbb{Z}_{p},K\right)$ where: \begin{equation} \tau_{1}\left\{ f\right\} \left(\mathfrak{z}\right)\overset{\textrm{def}}{=}f\left(\mathfrak{z}+1\right) \end{equation} \end{defn} \begin{thm} Let $K$ be a metrically complete field extension of $\mathbb{Q}_{p}$. Then, the only translation invariant linear functional $\varphi:C\left(\mathbb{Z}_{p},K\right)\rightarrow K$ is the zero functional ($\varphi\left(f\right)=0$ for all $f$)\footnote{\textbf{Corollary 3 }on page 177 of Robert's book \cite{Robert's Book}.}. \end{thm} \subsection{\label{subsec:1.3.3 Field-Extensions-=00003D000026}Field Extensions and Spherical Completeness} THROUGHOUT THIS SUBSECTION, WE WORK WITH A FIXED PRIME $p$. \vphantom{} It is often said that students of mathematics will not truly appreciate the beauty of analytic functions until after they have endured the pathological menagerie of functions encountered in a typical introductory course in real analysis. In my experience, $p$-adic analysis does much the same, only for fields instead than functions. The fact that adjoining a square root of $-1$ to the real numbers results in a metrically complete algebraically closed field is one of the greatest miracles in all mathematics. Working with the $p$-adic numbers, on the other hand, gives one an appreciation of just how \emph{awful} field extensions can be. The problem begins before we even take our first step up the tower of $p$-adic field extensions. Anyone who has ever struggled with the concept of negative numbers should be thankful that we do not exist in a macroscopically $p$-adic universe. In real life, the existence of negative integers is equivalent to the fact that $\mathbb{Z}^{\times}$\textemdash the group of multiplicative;y invertible elements of $\mathbb{Z}$, a.k.a. $\left\{ 1,-1\right\} $\textemdash is isomorphic to the cyclic group $\mathbb{Z}/2\mathbb{Z}$. The isomorphism $\mathbb{Z}^{\times}\cong\mathbb{Z}/2\mathbb{Z}$ is also responsible for the existence of a meaningful ordering on $\mathbb{Z}$ given by $<$ and friends. Unfortunately, for any grade-school students who happen to live in a $p$-adic universe, things are much less simple, and this has grave implications for the study of field extensions of $\mathbb{Q}_{p}$. To see why, for a moment, let us proceed navely. Let $K$ be a finite-degree Galois extension\footnote{Recall, this means that the set: \[ \left\{ \mathfrak{y}\in K:\sigma\left(\mathfrak{y}\right)=\mathfrak{y},\textrm{ }\forall\sigma\in\textrm{Gal}\left(K/\mathbb{Q}_{p}\right)\right\} \] is equal to $\mathbb{Q}_{p}$.} of $\mathbb{Q}_{p}$. Because we are interested in doing analysis, our first order of business is to understand how we might extend the $p$-adic absolute value on $\mathbb{Q}_{p}$ to include elements of $K$. To do this, observe that since any $\sigma\in\textrm{Gal}\left(K/\mathbb{Q}_{p}\right)$ is a field automorphism of $K$, any candidate for a \emph{useful }absolute value $\left|\cdot\right|_{K}$ on $K$ ought to satisfy: \begin{equation} \left|\sigma\left(\mathfrak{y}\right)\right|_{K}=\left|\mathfrak{y}\right|_{K},\textrm{ }\forall\mathfrak{y}\in K \end{equation} Moreover, because $\mathbb{Q}_{p}$ is a subset of $K$, it is not unreasonable to require $\left|\mathfrak{y}\right|_{K}$ to equal $\left|\mathfrak{y}\right|_{p}$ for all $\mathfrak{y}\in K$ that happen to be elements of $\mathbb{Q}_{p}$. To that end, let us get out the ol' determinant: \begin{defn} Define $N_{K}:K\rightarrow\mathbb{Q}_{p}$ by: \begin{equation} N_{K}\left(\mathfrak{y}\right)\overset{\textrm{def}}{=}\prod_{\sigma\in\textrm{Gal}\left(K/\mathbb{Q}_{p}\right)}\sigma\left(\mathfrak{y}\right)\label{eq:Definition of N_K} \end{equation} \end{defn} \begin{rem} $N_{K}$ is $\mathbb{Q}_{p}$-valued here precisely because $K$ is a Galois extension. \end{rem} \vphantom{} With this, we have that for all $\mathfrak{y}\in K$: \[ \left|N_{K}\left(\mathfrak{y}\right)\right|_{p}=\left|N_{K}\left(\mathfrak{y}\right)\right|_{K}=\left|\prod_{\sigma\in\textrm{Gal}\left(K/\mathbb{Q}_{p}\right)}\sigma\left(\mathfrak{y}\right)\right|_{K}=\prod_{\sigma\in\textrm{Gal}\left(K/\mathbb{Q}_{p}\right)}\left|\sigma\left(\mathfrak{y}\right)\right|_{K}=\left|\mathfrak{y}\right|_{K}^{d} \] where $d$ is the order of $\textrm{Gal}\left(K/\mathbb{Q}_{p}\right)$\textemdash or, equivalently (since $K$ is a Galois extension), the degree of $K$ over $\mathbb{Q}_{p}$. In this way, we can compute $\left|\mathfrak{y}\right|_{K}$ by taking the $d$th root of $\left|N_{K}\left(\mathfrak{y}\right)\right|_{p}$, which is already defined, seeing as $N_{K}\left(\mathfrak{y}\right)$ is an element of $\mathbb{Q}_{p}$. With a little work, making this argument rigorous leads to an extremely useful theorem: \begin{thm} \label{thm:Galois conjugates}Let $K$ be a Galois extension of $\mathbb{Q}_{p}$ of degree $d$, and let $\left|\cdot\right|_{p}$ be the $p$-adic absolute value. Then, the map: \begin{equation} \mathfrak{y}\mapsto\left|N_{K}\left(\mathfrak{y}\right)\right|_{p}^{1/d} \end{equation} defines the unique absolute value $\left|\cdot\right|_{K}$ on $K$ which extends the $p$-adic absolute value of $\mathbb{Q}_{p}$. \end{thm} \begin{rem} In fact, with a slight modification of $N_{K}$, one can show that this method gives the absolute value on $K$ for \emph{any }finite-degree extension $K$ of $\mathbb{Q}_{p}$, not only Galois extensions. Robert does this on page 95 of \cite{Robert's Book}. \end{rem} \begin{example} \label{exa:incorrect galois}As a sample application, consider $\mathbb{Q}_{5}$. Let $\xi$ denote a primitive fourth root of unity, which we adjoin to $\mathbb{Q}_{5}$. Then, since: \begin{align*} \left(1+2\xi\right)\left(1+2\xi^{3}\right) & =1+2\xi^{3}+2\xi+4\xi^{4}\\ \left(\xi+\xi^{3}=-1-\xi^{2}\right); & =5+2\left(-1-\xi^{2}\right)\\ \left(\xi^{2}=-1\right); & =5 \end{align*} and so, by \textbf{Theorem \ref{thm:Galois conjugates}}: \begin{equation} \left|1+2\xi\right|_{5}=\left|1+2\xi^{3}\right|_{5}=\sqrt{\left|5\right|_{5}}=\frac{1}{\sqrt{5}} \end{equation} Alas, this\emph{ }is \emph{wrong!} While this argument would be perfectly valid if our base field was $\mathbb{Q}$ or $\mathbb{R}$, it fails for $\mathbb{Q}_{5}$, due to the fact that $\xi$ was \emph{already an element of $\mathbb{Q}_{5}$.} \end{example} \vphantom{} With the sole exception of $\mathbb{Z}_{2}$, whose only roots of unity are $1$ and $-1$, given any odd prime $p$, $\mathbb{Z}_{p}$ contains more roots of unity than just $\left\{ 1,-1\right\} $, and this ends up complicating the study of field extensions. Fortunately, the situation isn't \emph{too }bad; it is not difficult to determine the roots of unity naturally contained in $\mathbb{Z}_{p}^{\times}$ (and hence, in $\mathbb{Z}_{p}$ and $\mathbb{Q}_{p}$): \begin{thm} When $p$ is an odd prime, $\mathbb{Z}_{p}^{\times}$ contains all $\left(p-1\right)$th roots of unity. $\mathbb{Z}_{2}^{\times}$, meanwhile, contains only second roots of unity ($-1$ and $1$). \end{thm} Proof: (Sketch) If $\mathfrak{z}\in\mathbb{Z}_{p}^{\times}$ satisfies $\mathfrak{z}^{n}=1$ for some $n\geq1$, it must also satisfy $\mathfrak{z}^{n}\overset{p}{\equiv}1$, so, the first $p$-adic digit of $\mathfrak{z}$ must be a primitive root of unity modulo $p$. Conversely, for each primitive root $u\in\left\{ 1,2,\ldots,p-1\right\} $ of unity modulo $p$, with a little work (specifically, \textbf{Hensel's Lemma }(see Section 6 of Chapter 1 of \cite{Robert's Book} for details)), we can construct a unique $\mathfrak{z}\in\mathbb{Z}_{p}^{\times}$ with $u$ as its first digit. Because the group of multiplicative units of $\mathbb{Z}/p\mathbb{Z}$ is isomorphic to $\mathbb{Z}/\left(p-1\right)\mathbb{Z}$, this then yields the theorem. Q.E.D. \vphantom{} Note that in order to compute the $p$-adic absolute value of an expression involving a native root of unity of $\mathbb{Z}_{p}$, we need to use said root's $p$-adic digit expansion. \begin{example} The roots of unity in $\mathbb{Z}_{7}$ are the $6$th roots of unity. Of these, note that only two are primitive. By the argument used to prove the previous theorem, note that these two roots of unity will have $3$ and $5$, respectively, as their first $7$-adic digits, seeing as those are the only two primitive roots of unity in $\mathbb{Z}/7\mathbb{Z}$. If we let $\xi$ and $\zeta$ denote the the $3$-as-first-digit and $5$-as-first-digit roots of unity, we then have that: \begin{equation} 2\xi+1\overset{7}{\equiv}2\cdot3+1\overset{7}{\equiv}0 \end{equation} and so, $\left|2\xi+1\right|_{7}\leq1/7$. By using \textbf{Hensel's Lemma} to compute more and more digits of $\xi$, one can determine the number of $0$s that occur at the start of $2\xi+1$'s $7$-adic expansion and thereby determine its $7$-adic absolute value. On the other hand: \[ 2\zeta+1\overset{7}{\equiv}2\cdot5+1\overset{7}{\equiv}4 \] so, $\left|2\zeta+1\right|_{7}=1$, because $2\zeta+1$ is a $7$-adic integer which is not congruent to $0$ modulo $7$. In addition to Hensel's Lemma, there is a more algebraic approach to computing the $7$-adic absolute values of these quantities. Since $\xi$ is a primitive $6$th root of unity, the other primitive root of unity (the one we denote by $\zeta$) must be $\xi^{5}$; $\xi^{2}$ and $\xi^{4}$ will be primitive $3$rd roots of unity, while $\xi^{3}$ will be the unique primitive square root of unity otherwise known as $-1$. As such, writing $\zeta$ as $\xi^{5}$, observe that: \begin{align*} \left(2\xi+1\right)\left(2\xi^{5}+1\right) & =4\xi^{6}+2\xi+2\xi^{5}+1\\ & =4+2\left(\xi+\xi^{5}\right)+1\\ & =5+2\xi\left(1+\xi^{4}\right)\\ \left(\textrm{let }\omega=\xi^{2}\right); & =5+2\xi\left(1+\omega^{2}\right) \end{align*} Because $\omega$ is a primitive $3$rd root of unity, $1+\omega^{2}=-\omega=-\xi^{2}$, and so: \begin{align*} \left(2\xi+1\right)\left(2\xi^{5}+1\right) & =5+2\xi\left(1+\omega^{2}\right)\\ & =5+2\xi\left(-\xi^{2}\right)\\ & =5-2\xi^{3}\\ \left(\xi^{3}=-1\right); & =5+2\\ & =7 \end{align*} Hence: \[ \left|2\xi+1\right|_{7}\underbrace{\left|2\xi^{5}+1\right|_{7}}_{=\left|2\zeta+1\right|_{7}=1}=\left|7\right|_{7}=\frac{1}{7} \] So, $\left|2\xi+1\right|_{7}$ is precisely $1/7$. What this shows in order to of said root in order to use complex exponential notation to write roots of unity for any given odd prime $p$, we need to specify the $p$-adic digits of $\xi_{p-1}\overset{\textrm{def}}{=}e^{2\pi i/\left(p-1\right)}$ so that we can then have a uniquely defined $p$-adic absolute value for any polynomial in $\xi_{p-1}$ with coefficients in $\mathbb{Q}_{p}$. \end{example} \vphantom{} In general, field extensions of $\mathbb{Q}_{p}$ are obtained in the usual way, by adjoining to $\mathbb{Q}_{p}$ the root of a polynomial with coefficients in $\mathbb{Q}_{p}$. Note that since every algebraic number $\alpha$ is, by definition, the root of a polynomial with coefficients in $\mathbb{Q}$, given any algebraic number $\alpha$, we can create a $p$-adic field containing $\alpha$ by adjoining $\alpha$ to $\mathbb{Q}_{p}$, seeing as $\mathbb{Q}_{p}$ contains $\mathbb{Q}$ as a subfield. As described above, the absolute value on $p$ extends with these extensions in the natural way, maintaining its multiplicativity and ultrametric properties in the extension. Thus, for example, in any extension $K$ of $\mathbb{Q}_{p}$, all roots of unity will have a $p$-adic absolute value of $1$. More generally, in any finite-degree extension $K$ of $\mathbb{Q}_{p}$, the $p$-adic absolute value of an arbitrary element of $K$ will be a number of the form $p^{r}$, where $r\in\mathbb{Z}\cup V$, where $V$ is a finite set whose elements are non-integer rational numbers. Far more problematic, however, is the issue of algebraic closure. \begin{defn} We write \nomenclature{$\overline{\mathbb{Q}}_{p}$}{$p$-adic algebraic numbers} $\overline{\mathbb{Q}}_{p}$, the \textbf{algebraic closure of $\mathbb{Q}_{p}$}, the field obtained by adjoining to $\mathbb{Q}_{p}$ the roots of every polynomial with coefficients in $\mathbb{Q}_{p}$.\index{algebraic closure} \end{defn} \vphantom{} It is not an overgeneralization to say that algebraic number theory is the study of the Galois group $\textrm{Gal}\left(\overline{\mathbb{Q}}/\mathbb{Q}\right)$, and that the continued interest of the subject hinges upon the fact that\textemdash unlike with $\mathbb{R}$\textemdash the algebraic closure of $\mathbb{Q}$ is not a finite dimensional extension of $\mathbb{Q}$. The same is true in the $p$-adic context: $\overline{\mathbb{Q}}_{p}$\textemdash the algebraic closure of $\mathbb{Q}_{p}$\textemdash is an infinite-dimensional extension of $\mathbb{Q}_{p}$. However, the parallels between the two algebraic closures run deeper still. Just as $\overline{\mathbb{Q}}$ is incomplete as a metric space with respect to the complex absolute value, so too is $\overline{\mathbb{Q}}_{p}$ incomplete as a metric spaces with respect to the extension of $\left|\cdot\right|_{p}$. Moreover, as infinite-dimensional extensions of their base fields, neither $\overline{\mathbb{Q}}$ nor $\overline{\mathbb{Q}}_{p}$ is locally compact. Just as taking the metric closure of $\overline{\mathbb{Q}}$ with respect to the usual absolute value yields $\mathbb{C}$, the field of complex numbers, taking the metric closure of $\overline{\mathbb{Q}}_{p}$ with respect to the $p$-adic absolute value yields the $p$-adic complex numbers: \begin{defn} We write \nomenclature{$\mathbb{C}_{p}$}{the set of $p$-adic complex numbers \nomnorefpage} $\mathbb{C}_{p}$ to denote the field of \index{$p$-adic!complex numbers}\textbf{$p$-adic complex numbers}, defined here as the metric closure of $\overline{\mathbb{Q}}_{p}$ \end{defn} \vphantom{}. One of the many reasons complex analysis is the best form of analysis is because $\mathbb{R}$ is locally compact and has $\mathbb{C}$ as a one-dimensional extension of $\mathbb{R}$; this then guarantees that $\mathbb{C}$ is also locally compact. Unfortunately, the same does not hold for $\mathbb{C}_{p}$; $\mathbb{C}_{p}$ is \emph{not }locally compact. Worse still, $\mathbb{C}_{p}$ lacks a property most analysts take for granted: \textbf{spherical completeness}. Ordinarily, an (ultra)metric space is complete if and only if any nested sequence of balls $B_{1}\supseteq B_{2}\supseteq\cdots$ whose diameters/radii tend to $0$, the intersection $\bigcap_{n=1}^{\infty}B_{n}$ is non-empty. In the real/complex world, removing the requirement that the diameters of the $B_{n}$s tend to $0$ would not affect the non-emptiness of the intersection. Bizarrely, this is \emph{not }always the case in an ultrametric space. \begin{defn} An ultrametric space $X$ is said to be \index{spherically complete}\textbf{spherically complete }whenever every nested sequence of balls in $X$ has a non-empty intersection. We say $X$ is \textbf{spherically incomplete }whenever it is not spherically complete. \end{defn} \begin{prop} Any locally compact field is spherically complete. Consequently, for any prime number $p$, $\mathbb{Q}_{p}$ is spherically complete, as is any \emph{finite dimensional }field extension of $\mathbb{Q}_{p}$. \end{prop} Proof: Compactness. Q.E.D. \vphantom{} Despite this, there exist non-locally compact fields which are spherically complete (see \cite{Robert's Book} for more details; Robert denotes the spherical completion of $\mathbb{C}_{p}$ by $\Omega_{p}$). \begin{thm} Let $K$ be a non-archimedean valued field. Then, $K$ is spherically complete whenever it is metrically complete, and has a discrete value group. In particular, all locally compact metrically complete valued fields (archimedean or not) are spherically complete\footnote{\textbf{Corollary 20.3} from \cite{Ultrametric Calculus}.}. \end{thm} As it regards $\mathbb{C}_{p}$, the key properties are as follows: \begin{thm} $\mathbb{C}_{p}$ is \textbf{not}\emph{ }spherically complete\emph{ \cite{Robert's Book}}. \end{thm} \begin{thm} $\mathbb{C}_{p}$ is not\emph{ }separable\footnote{That is, it contains no dense \emph{countable} subset.}\emph{ \cite{Robert's Book}}. \end{thm} \vphantom{} As will be mentioned in Subsection \ref{subsec:3.1.2 Banach-Spaces-over}, whether or not a non-archimedean field is spherically complete has significant impact on the nature of the Banach spaces which can be built over it. At present, it suffices to say that in most ``classical'' applications of $p$-adic analysis (especially to number theory and algebraic geometry), spherical incompleteness is considered an undesirable property, particularly because of its destructive interaction with non-archimedean analogues of the \textbf{Hahn-Banach Theorem}\footnote{The thrust of this theorem, recall, is that we can take a continuous linear functional $\phi$ defined on a subspace $\mathcal{Y}$ of a given Banach space $\mathcal{X}$ and extend $\phi$ to a continuous linear functional on $\mathcal{X}$ itself.}\textbf{ }of functional analysis \cite{Robert's Book}. Despite this, the spherical \emph{incompleteness} of the fields in which our $\left(p,q\right)$-adic functions will take their values plays a pivotal role in our work, because of the effect this spherical incompleteness has upon the Fourier analytic properties of our functions. See, for instance, \textbf{Theorem \ref{thm:FST is an iso from measures to ell infinity}} on page \pageref{thm:FST is an iso from measures to ell infinity} about the $\left(p,q\right)$-adic Fourier-Stieltjes transform. Finally, we must give a brief mention of ``complex exponentials'' in the $p$-adic context. Although one can proceed to define a \textbf{$p$-adic exponential function}\index{$p$-adic!exponential function} by way of the usual power series $\exp_{p}:\mathbb{Q}_{p}\rightarrow\mathbb{Q}_{p}$: \begin{equation} \exp_{p}\left(\mathfrak{z}\right)\overset{\textrm{def}}{=}\sum_{n=0}^{\infty}\frac{\mathfrak{z}^{n}}{n!}\in\mathbb{Q}_{p}\label{eq:Definition of the p-adic exponential function} \end{equation} unlike its classical counterpart, this construction is quite unruly, converging only for those $\mathfrak{z}$ with $\left|\mathfrak{z}\right|_{p}<p^{-\frac{1}{p-1}}$. Even worse, thanks to the properties of $p$-adic power series (specifically, due to \textbf{Strassman's Theorem} \cite{Robert's Book}), $\exp_{p}\left(\mathfrak{z}\right)$ is not a periodic function of $\mathfrak{z}$. As such, instead of working natively, when $p$-adic analysis wants to make use of complex exponentials in a manner comparable to their role in complex analysis, we need to take a different approach, as we discuss in the next subsection. \subsection{\label{subsec:1.3.4 Pontryagin-Duality-and}Pontryagin Duality and Embeddings of Fields} For those who haven't had the pleasure of meeting it, \textbf{Pontryagin duality}\index{Pontryagin duality}\textbf{ }is that lovely corner of mathematics where the concept of representing functions as Fourier series is generalized to study complex-valued functions on spaces\textemdash specifically, \textbf{locally compact abelian groups} (\textbf{LCAG}s)\footnote{Such a space is an abelian group that has a topology in which the group's operations are continuous; moreover, as a topological space, the group is locally compact.}\textemdash other than the circle ($\mathbb{R}/\mathbb{Z}$), the line ($\mathbb{R}$), and their cartesian products. Essentially, given a LCAG $G$, you can decompose functions $G\rightarrow\mathbb{C}$ as series or integrals of ``elementary functions'' (\textbf{characters}\index{character}) which transform in a simple, predictable way when they interact with the group's operation. In classical Fourier analysis, the characters are exponential functions $e^{2\pi ixt}$, $e^{-i\theta}$, and the like, and their ``well-behavedness'' is their periodicity\textemdash their invariance with respect to translations/shifts of the circle / line. In terms of the literature, Folland's book \cite{Folland - harmonic analysis} gives an excellent introduction and is very much worth reading in general. For the reader on the go, Tao's course notes on his blog \cite{Tao Fourier Transform Blog Post} are also quite nice. However, because this dissertation will approach Haar measures and integration theory primarily from the perspective of functional analysis, we will not need to pull out the full workhorse of Pontryagin duality and abstract harmonic analysis. Still, we need to spend at least \emph{some }amount of time discussing characters and our conventions for using them. A character\textbf{ }is, in essence, a generalization of the complex exponential. Classically, given an abelian group $G$, a \textbf{(complex-valued) character }on $G$ is a group homomorphism $\chi:G\rightarrow\mathbb{T}$, where $\left(\mathbb{T},\times\right)$ is the unit circle in $\mathbb{C}$, realized as an abelian group with the usual complex-number multiplication operation. Every character can be written as $\chi\left(g\right)=e^{2\pi i\xi\left(g\right)}$, where $\xi:G\rightarrow\mathbb{R}/\mathbb{Z}$ is a \textbf{frequency}, a group homomorphism from $G$ to $\mathbb{R}/\mathbb{Z}$\textemdash the group of real numbers in $\left[0,1\right)$\textemdash equipped with the operation of addition modulo $1$. The set of all characters of $G$ forms an abelian group under point-wise multiplication, and the set of all frequencies of $G$ forms an abelian group under point-wise addition. These two groups isomorphic to one another, and are both realizations of the \textbf{(Pontryagin) dual }of $G$, denoted $\hat{G}$. Depending on $G$, it can take some work to classify the characters and frequencies and figure out explicit formulae to use for them. Thankfully, since all of our work will be done over the additive group $\mathbb{Z}_{p}$ (where $p$ is a prime), we only need to familiarize ourselves with one set of formulae. \begin{defn}[$\hat{\mathbb{Z}}_{p}$] \ \vphantom{} I. We write \nomenclature{$\hat{\mathbb{Z}}_{p}$}{$\overset{\textrm{def}}{=}\mathbb{Z}\left[\frac{1}{p}\right]/\mathbb{Z}$}$\hat{\mathbb{Z}}_{p}$ to denote the group $\mathbb{Z}\left[\frac{1}{p}\right]/\mathbb{Z}$. This is the set of all $p$-ary rational numbers in $\left[0,1\right)$: \begin{equation} \hat{\mathbb{Z}}_{p}\overset{\textrm{def}}{=}\mathbb{Z}\left[\frac{1}{p}\right]/\mathbb{Z}=\left\{ \frac{k}{p^{n}}:n\in\mathbb{N}_{0},k\in\left\{ 0,\ldots,p^{n}-1\right\} \right\} \label{eq:Definition of Z_p hat} \end{equation} and is a group under the operation of addition modulo $1$. This is the \textbf{additive}\footnote{The \emph{multiplicative} $p$-Prfer group, generally denoted $\mathbb{Z}\left(p^{\infty}\right)$, is the set of all $p$-power roots of unity in $\mathbb{T}$\textemdash that is, all $\xi\in\mathbb{T}$ so that $\xi^{p^{n}}=1$ for some $n\in\mathbb{N}_{0}$\textemdash made into a group by multiplication.}\textbf{ $p$-Prfer group}\index{$p$-Prfer group}. As the notation $\hat{\mathbb{Z}}_{p}$ suggests, this group is one realization of the \textbf{Pontryagin dual} of the additive group of $p$-adic integers: $\left(\mathbb{Z}_{p},+\right)$. \vphantom{} II. Recall that every $p$-adic rational number $\mathfrak{z}\in\mathbb{Q}_{p}$ can be written as: \begin{equation} \mathfrak{z}=\sum_{n=n_{0}}^{\infty}c_{n}p^{n} \end{equation} for constants $n_{0}\in\mathbb{Z}$ and $\left\{ c_{n}\right\} _{n\geq-n_{0}}\subseteq\left\{ 0,\ldots,p-1\right\} $. The\textbf{ $p$-adic fractional part}\index{$p$-adic!fractional part}, denoted \nomenclature{$\left\{ \cdot\right\} _{p}$}{$p$-adic fractional part} $\left\{ \cdot\right\} _{p}$, is the map from $\mathbb{Q}_{p}$ to $\hat{\mathbb{Z}}_{p}$ defined by: \begin{equation} \left\{ \mathfrak{z}\right\} _{p}=\left\{ \sum_{n=-n_{0}}^{\infty}c_{n}p^{n}\right\} _{p}\overset{\textrm{def}}{=}\sum_{n=n_{0}}^{-1}c_{n}p^{n}\label{eq:Definition of the p-adic fractional part} \end{equation} where the sum on the right is defined to be $0$ whenever $n_{0}\geq0$. The \textbf{$p$-adic integer part}\index{$p$-adic!integer part}, denoted $\left[\cdot\right]_{1}$ \nomenclature{$ \left[\cdot\right]_{1}$}{$p$-adic integer part}, is the map from $\mathbb{Q}_{p}$ to $\mathbb{Z}_{p}$ defined by: \begin{equation} \left[\mathfrak{z}\right]_{1}=\left[\sum_{n=-n_{0}}^{\infty}c_{n}p^{n}\right]_{1}\overset{\textrm{def}}{=}\sum_{n=0}^{\infty}c_{n}p^{n}\label{eq:Definition of the p-adic integer part} \end{equation} \end{defn} \vphantom{} Note that $\left[\mathfrak{z}\right]_{1}=\mathfrak{z}-\left\{ \mathfrak{z}\right\} _{p}$. Indeed, the $p$-adic fractional and integer parts are projections from $\mathbb{Q}_{p}$ onto $\mathbb{Q}_{p}/\mathbb{Z}_{p}$ and $\mathbb{Z}_{p}$, respectively. In fact, $\hat{\mathbb{Z}}_{p}$ and $\mathbb{Q}_{p}/\mathbb{Z}_{p}$ are isomorphic as additive groups. Moreover, we can express the group $\left(\mathbb{Q}_{p},+\right)$ as the sum: \begin{equation} \mathbb{Q}_{p}=\mathbb{Z}\left[\frac{1}{p}\right]+\mathbb{Z}_{p}\label{eq:Sum decomposition of Q_p} \end{equation} where the right-hand side is group whose elements are of the form $t+\mathfrak{z}$, where $t\in\mathbb{Z}\left[1/p\right]$ and $\mathfrak{z}\in\mathbb{Z}_{p}$. Note, this sum is \emph{not }direct\footnote{For people who enjoy staring at diagrams of (short) exact sequences, Subsection 5.4 (starting at page 40) of Robert's book \cite{Robert's Book} gives a comprehensive discussion of (\ref{eq:Sum decomposition of Q_p})'s failure to be a direct sum. Even though the \emph{set-theoretic }bijection $\iota:\mathbb{Q}_{p}\rightarrow\hat{\mathbb{Z}}_{p}\times\mathbb{Z}_{p}$ given by $\iota\left(\mathfrak{y}\right)\overset{\textrm{def}}{=}\left(\left\{ \mathfrak{y}\right\} _{p},\left[\mathfrak{y}\right]_{1}\right)$ does more than your ordinary set-theoretic bijection (for example, its restriction to $\mathbb{Z}_{p}$ \emph{is }an isomorphism of groups), $\iota$ itself is not an isomorphism of groups. For example: \[ \iota\left(\frac{1}{p}+\frac{p-1}{p}\right)=\iota\left(1\right)=\left(0,1\right) \] \[ \iota\left(\frac{1}{p}\right)+\iota\left(\frac{p-1}{p}\right)=\left(\frac{1}{p},0\right)+\left(\frac{p-1}{p},0\right)=\left(1,0\right)=\left(0,0\right) \] }. For example: \begin{equation} \frac{1}{p}+2=\left(\frac{1}{p}+1\right)+1 \end{equation} Despite this, every $p$-adic number is uniquely determined by its fractional and integer parts, and we can view $\hat{\mathbb{Z}}_{p}$ as a subset of $\mathbb{Q}_{p}$ by identifying it with the subset of $\mathbb{Q}_{p}$ whose elements all have $0$ as the value of their $p$-adic integer part. In this way we can add and multiply $t$s in $\hat{\mathbb{Z}}_{p}$ with $\mathfrak{z}$s in $\mathbb{Z}_{p}$ or $\mathbb{Q}_{p}$. \begin{example} For $t=\frac{1}{p}$ and $\mathfrak{z}=-3$, $t\mathfrak{z}=-\frac{3}{p}\in\mathbb{Q}_{p}$. \end{example} \vphantom{} Since every $p$-ary rational number $t\in\hat{\mathbb{Z}}_{p}$ is an element of $\mathbb{Q}_{p}$, we can consider $t$'s $p$-adic valuation and its $p$-adic absolute value. Specifically: \begin{fact}[\textbf{Working with $v_{p}$ and $\left|\cdot\right|_{p}$ on $\hat{\mathbb{Z}}_{p}$}] \ \vphantom{} I. $v_{p}\left(t\right)$ will be $\infty$ when $t=0$, and, otherwise, will be $-n$, where $n$ is the unique integer $\geq1$ so that $t$ can be written as $t=k/p^{n}$, where $k\in\left\{ 1,\ldots,p^{n}-1\right\} $ is co-prime to $p$. \vphantom{} II. For non-zero $t$, $\left|t\right|_{p}=p^{-v_{p}\left(t\right)}$ will be the value in the denominator of $t$. Specifically, $\left|t\right|_{p}=p^{n}$ if and only if the irreducible fraction representing $t$ is $t=k/p^{n}$, where $k\in\left\{ 1,\ldots,p^{n}-1\right\} $ is co-prime to $p$. \vphantom{} III. For non-zero $t$, $t\left|t\right|_{p}$ is the numerator $k$ of the irreducible fraction $k/p^{n}$ representing $t$. \vphantom{} IV. The $p$-adic fractional part also encodes information about congruences. In particular, we have the following identity: \begin{equation} \left\{ t\mathfrak{z}\right\} _{p}\overset{1}{\equiv}t\left[\mathfrak{z}\right]_{\left|t\right|_{p}},\textrm{ }\forall t\in\hat{\mathbb{Z}}_{p},\textrm{ }\forall\mathfrak{z}\in\mathbb{Z}_{p}\label{eq:p-adic fractional part identity modulo 1} \end{equation} where, $\left[\mathfrak{z}\right]_{\left|t\right|_{p}}$ is the projection of $\mathfrak{z}$ modulo the power of $p$ in the denominator of $t$, and where, recall, $\overset{1}{\equiv}$ means that the left and the right hand side are equal to one another as real numbers modulo $1$\textemdash that is, their difference is an integer. \end{fact} \begin{example} Let $\mathfrak{z}\in\mathbb{Z}_{2}$. Then, we have the following equality in $\mathbb{C}$ \begin{equation} e^{2\pi i\left\{ \frac{\mathfrak{z}}{4}\right\} _{2}}\overset{\mathbb{C}}{=}e^{2\pi i\frac{\left[\mathfrak{z}\right]_{4}}{4}} \end{equation} More generally, for any $\mathfrak{y}\in\mathbb{Q}_{p}$, we can define: \begin{equation} e^{2\pi i\left\{ \mathfrak{y}\right\} _{p}}\overset{\mathbb{C}}{=}\sum_{n=0}^{\infty}\frac{\left(2\pi i\left\{ \mathfrak{y}\right\} _{p}\right)^{n}}{n!} \end{equation} because $\left\{ \mathfrak{y}\right\} _{p}\in\hat{\mathbb{Z}}_{p}=\mathbb{Z}\left[\frac{1}{p}\right]/\mathbb{Z}\subset\left[0,1\right)\subset\mathbb{R}\subset\mathbb{C}$. \end{example} \vphantom{} There is also the issue of what happens when we ``mix our $p$s and $q$s''. \begin{example} Let $\mathfrak{z}\in\mathbb{Z}_{2}$. Then, since $\left|\frac{1}{3}\right|_{2}=1$, we have that $\frac{1}{3}\in\mathbb{Z}_{2}$, and thus, that $\frac{\mathfrak{z}}{3}\in\mathbb{Z}_{2}$. However: \begin{equation} \left\{ \frac{\mathfrak{z}}{24}\right\} _{2}=\left\{ \frac{1}{8}\frac{\mathfrak{z}}{3}\right\} _{2}\neq\frac{1}{24}\left[\mathfrak{z}\right]_{8} \end{equation} Rather, we have that: \begin{equation} \left\{ \frac{\mathfrak{z}}{24}\right\} _{2}=\left\{ \frac{1}{8}\frac{\mathfrak{z}}{3}\right\} _{2}\overset{1}{\equiv}\frac{1}{8}\left[\frac{\mathfrak{z}}{3}\right]_{8}\overset{1}{\equiv}\frac{1}{8}\left[\mathfrak{z}\right]_{8}\left[\frac{1}{3}\right]_{8}\overset{1}{\equiv}\frac{\left[3\right]_{8}^{-1}}{8}\left[\mathfrak{z}\right]_{8}\overset{1}{\equiv}\frac{3}{8}\left[\mathfrak{z}\right]_{8} \end{equation} where $\left[3\right]_{8}^{-1}$ denotes the unique integer in $\left\{ 1,\ldots,7\right\} $ which is the multiplicative inverse of $3$ modulo $8$\textemdash in this case, $3$, since $3$ is a square root of unity modulo $8$. Lastly, to return to our discussion of characters, it turns out that every complex-valued character on $\mathbb{Z}_{p}$ is a function of the form $\mathfrak{z}\in\mathbb{Z}_{p}\mapsto e^{2\pi i\left\{ t\mathfrak{z}\right\} _{p}}\in\mathbb{C}$ for some unique $t\in\hat{\mathbb{Z}}_{p}$; this is an instance of the isomorphism between the character group of $\mathbb{Z}_{p}$ and $\hat{\mathbb{Z}}_{p}$. \end{example} \vphantom{} Note that all of the above discussion was predicated on the assumption that our characters were \emph{complex-valued}. Importantly, because $\left\{ \cdot\right\} _{p}$ outputs elements of $\hat{\mathbb{Z}}_{p}$, each of which is a rational number, note also that the output of any complex-valued character $\mathfrak{z}\in\mathbb{Z}_{p}\mapsto e^{2\pi i\left\{ t\mathfrak{z}\right\} _{p}}\in\mathbb{C}$ of $\mathbb{Z}_{p}$ is a $p$-power root of unity. Since every $p$-power root of unity is the root of a polynomial of the form $X^{p^{n}}-1$ for some sufficiently large $n\geq1$, every $p$-power root of unity is an \emph{algebraic }number. As such, our $p$-adic characters are not \emph{true} $\mathbb{C}$-valued functions, but $\overline{\mathbb{Q}}$-valued functions. When working with functions taking values in a non-archimedean field like $\mathbb{Q}_{q}$, where $q$ is a prime, it is generally more useful to study them not by using Fourier series involving complex-valued ($\mathbb{C}$-valued) characters, but \textbf{$q$-adic (complex) valued characters}\textemdash those taking values in $\mathbb{C}_{q}$. However, regardless of whether we speak of the complex-valued or $q$-adic-valued characters of $\mathbb{Z}_{p}$, our characters will still end up taking values in $\overline{\mathbb{Q}}$. The reason there even exists a verbal distinction between complex-valued and $q$-adic-valued characters on $\mathbb{Z}_{p}$ is due to the needs of (algebraic) number theorists. Ordinarily, in algebraic number theory, the question of how a given field $\mathbb{F}$ is embedded in a larger field $K$ is very important, and it is not uncommon to allow the embeddings to vary, or to even do work by considering all possible embeddings \emph{simultaneously}. However, in this dissertation, we will do exactly the opposite. Our goal is to use the easy, comforting complex-exponential $e$-to-the-power-of-$2\pi i$-something notation for roots of unity. Because the $\mathbb{C}$-valued characters of $\mathbb{Z}_{p}$ are $\overline{\mathbb{Q}}$-valued, all of the algebraic rules for manipulating expressions like $\left\{ t\mathfrak{z}\right\} _{p}$ and $e^{2\pi i\left\{ t\mathfrak{z}\right\} _{p}}$ will remain valid when we view $e^{2\pi i\left\{ t\mathfrak{z}\right\} _{p}}$ as an element of $\mathbb{C}_{q}$, because we will be identifying $e^{2\pi i\left\{ t\mathfrak{z}\right\} _{p}}$ with its counterpart in the copy of $\overline{\mathbb{Q}}$ that lies in $\mathbb{C}_{q}$\textemdash a copy which is isomorphic to the version of $\overline{\mathbb{Q}}$ that lies in $\mathbb{C}$. If the reader is nervous about this, they can simply think of $e^{2\pi i\left\{ t\mathfrak{z}\right\} _{p}}$ as our notation of writing an abstract unitary character on $\mathbb{Z}_{p}$, without reference to an embedding. \begin{assumption*}[Embedding Convention] For any primes $p$ and $q$, and any integer $n\geq1$, we will write $e^{2\pi i/p^{n}}$ to denote a specific choice of a primitive $p^{n}$th root of unity in $\mathbb{C}_{q}$, and shall then identify $e^{2\pi ik/p^{n}}$ as the $k$th power of that chosen root of unity. We also make these choices so that the are compatible in the natural way, with \begin{equation} \left(e^{2\pi i/p^{n+1}}\right)^{p}=e^{2\pi i/p^{n}},\textrm{ }\forall n\geq1 \end{equation} More generally, when $q\geq5$, since the set of roots of unity in $\mathbb{Q}_{q}$ is generated by a primitive $\left(q-1\right)$th root of unity, we define $e^{2\pi i/\left(q-1\right)}$ to be the primitive $\left(q-1\right)$th root of unity in $\mathbb{Z}_{q}^{\times}$ whose value mod $q$ is $r_{q}$, the smallest element of $\left(\mathbb{Z}/q\mathbb{Z}\right)^{\times}$ which is a primitive $\left(q-1\right)$th root of unity mod $q$. Then, for every other root of unity $\xi$ in $\mathbb{Q}_{q}$, there is a unique integer $k\in\left\{ 0,\ldots,q-2\right\} $ so that $\xi=\left(e^{2\pi i/\left(q-1\right)}\right)^{k}$, and as such, we define $\xi$'s value mod $q$ to be the value of $r_{q}^{k}$ mod $q$. \end{assumption*} In this convention, any \emph{finite }$\overline{\mathbb{Q}}$-linear combination of roots of unity will then be an element of $\overline{\mathbb{Q}}$, and, as such, we can and will view the linear combination as existing in $\mathbb{C}$ and $\mathbb{C}_{q}$ \emph{simultaneously}. When working with infinite sums, the field to which the sum belongs will be indicated, when necessary, by an explicit mention of the field in which the convergence happens to be occurring. \emph{The practical implications of all this is that the reader will not need to worry about considerations of local compactness, spherical completeness, or field embeddings when doing or reading through computations with complex exponentials in this dissertation. }Except for when limits and infinite series come into play (and even then, only rarely), it will be as if everything is happening in $\mathbb{C}$. The reason everything works out is thanks to the fundamental identity upon which all of our Fourier analytic investigations will be based: \begin{equation} \left[\mathfrak{z}\overset{p^{n}}{\equiv}\mathfrak{a}\right]\overset{\overline{\mathbb{Q}}}{=}\frac{1}{p^{n}}\sum_{\left|t\right|_{p}\leq p^{n}}e^{-2\pi i\left\{ t\mathfrak{a}\right\} _{p}}e^{2\pi i\left\{ t\mathfrak{z}\right\} _{p}},\textrm{ }\forall\mathfrak{z},\mathfrak{a}\in\mathbb{Z}_{p} \end{equation} where the sum is taken over all $t\in\hat{\mathbb{Z}}_{p}$ whose denominators are at most $p^{n}$. This is the Fourier series for the indicator function of the co-set $\mathfrak{a}+p^{n}\mathbb{Z}_{p}$. As indicated, the equality holds in $\overline{\mathbb{Q}}$. Moreover, observe that this identity is invariant under the action of $\textrm{Gal}\left(\overline{\mathbb{Q}}/\mathbb{Q}\right)$, seeing as the left-hand side is always rational, evaluating to $1$ when $\mathfrak{z}$ is in $\mathfrak{a}+p^{n}\mathbb{Z}_{p}$ and evaluating to $0$ for all other $\mathfrak{z}\in\mathbb{Z}_{p}$. Because of this, nothing is lost in choosing a particular embedding, and the theories we develop will be equally valid regardless of our choice of an embedding. So, nervous number theorists can set aside their worries and breathe easily. For staying power, we will repeat this discussion in Subsection \ref{subsec:1.3.3 Field-Extensions-=00003D000026}. One of the quirks of our approach is that when we actually go about computing Fourier transforms and integrals for $q$-adic valued functions of one or more $p$-adic variables, the computational formalisms will be identical to their counterparts for real- or complex-valued functions of one or more $p$-adic variables. As such, the overview of non-archimedean Fourier analysis given in Subsections \ref{subsec:3.1.4. The--adic-Fourier} and \ref{subsec:3.1.5-adic-Integration-=00003D000026} will include all the necessary computational tools which will be drawn upon in this dissertation. That being said, I highly recommend the reader consult a source like \cite{Bell - Harmonic Analysis on the p-adics} in case they are not familiar with the nitty-gritty details of doing computations with (real)-valued Haar measures on the $p$-adics and with the Fourier transform of real- or complex-valued functions of a $p$-adic variable. A more comprehensive\textemdash and rigorous\textemdash expositions of this material can be found in the likes of \cite{Taibleson - Fourier Analysis on Local Fields,Automorphic Representations}; \cite{Automorphic Representations} is a book on representation theory which deals with the methods of $p$-adic integration early on because of its use in that subject, whereas \cite{Taibleson - Fourier Analysis on Local Fields} is dedicated entirely to the matter of the Fourier analysis of complex-valued functions on $\mathbb{Q}_{p}$ and other local fields. \subsection{\label{subsec:1.3.5 Hensel's-Infamous-Blunder}Hensel's Infamous Blunder and Geometric Series Universality} In discussions with strangers over the internet, it became apparent that some of the number-theoretic liberties I take in my work are a source of controversy\textemdash and for good reason. Despite the extraordinary importance that $p$-adic numbers would eventually take in number theory, the $p$-adics' immediate reception by the mathematical community of the \emph{fin de sicle }was one of reservation and concerned apprehension. There are a variety of reasons for this: the cultural attitudes of his contemporaries; Hensel\index{Hensel, Kurt}'s unabashed zeal for his discovery; the nascent state of the theory of topology\footnote{Poincar published the installments of \emph{Analysis Situs }from 1895 through to 1904, contemporaneous with Hensel's introduction of the $p$-adics in 1897; \emph{Analysis Situs }is to topology what Newton's \emph{Principia }was to physics.} at that particular day and age; and, especially, Hensel's own fatal blunder: a foundational flawed 1905 $p$-adic ``proof'' that $e$ was a transcendental number \cite{Gouvea's introudction to p-adic numbers book,Gouvea's p-adic number history slides,Koblitz's book,Conrad on p-adic series}. Gouvea presents this blunder like so: \begin{quotation} Hensel's incorrect proof goes like this. Start from the equation: \[ e^{p}=\sum_{n=0}^{\infty}\frac{p^{n}}{n!} \] Hensel checks that this series converges in $\mathbb{Q}_{p}$ and concludes that it satisfies an equation of the form $y^{p}=1+p\varepsilon$ with $\varepsilon$ a $p$-adic unit. If $e$ is algebraic, it follows that $\left[\mathbb{Q}\left(e\right):\mathbb{Q}\right]\geq p$. But $p$ was arbitrary, so we have a contradiction, and $e$ is transcendental \cite{Gouvea's p-adic number history slides}. \end{quotation} Hensel's error is as much \emph{topological} it is number theoretic. Specifically, he \textbf{\emph{incorrectly}} assumes the following statement is true: \begin{assumption} \emph{Let $\mathbb{F}$ and $\mathbb{K}$ be any field extensions of $\mathbb{Q}$, equipped with absolute values $\left|\cdot\right|_{\mathbb{F}}$ and $\left|\cdot\right|$$_{\mathbb{K}}$, respectively, which make $\left(\mathbb{F},\left|\cdot\right|_{\mathbb{F}}\right)$ and $\left(\mathbb{K},\left|\cdot\right|_{\mathbb{K}}\right)$ into complete metric spaces. Let $\left\{ x_{n}\right\} _{n\geq1}$ be a sequence in $\mathbb{Q}$ such that there are elements $\mathfrak{a}\in\mathbb{F}$ and $\mathfrak{b}\in\mathbb{K}$ so that $\lim_{n\rightarrow\infty}\left|\mathfrak{a}-x_{n}\right|_{\mathbb{F}}=0$ and $\lim_{n\rightarrow\infty}\left|\mathfrak{b}-x_{n}\right|_{\mathbb{F}}=0$. Then $\mathfrak{a}=\mathfrak{b}$.} \end{assumption} \vphantom{} So, Hensel presumed that if $\mathbb{F}$ is a field extension of $\mathbb{Q}$ in which the series $\sum_{n=0}^{\infty}\frac{p^{n}}{n!}$ converges, the sum of the series was necessarily $e^{p}$. This, of course, is wrong. One might object and say ``but this is obvious: not every $p$-adic integer can be realized as an ordinary real or complex number!'', however, the situation is even more subtle than that. For example, the statement in the previous paragraph remains false even if we required $\mathfrak{a}\in\mathbb{F}\cap\mathbb{Q}$ and $\mathfrak{b}\in\mathbb{K}\cap\mathbb{Q}$. Consider the following: \begin{example}[\textbf{A Counterexample for Kurt Hensel}\footnote{Taken from \cite{Conrad on p-adic series}.}] Let $a_{n}=\frac{1}{1+p^{n}}$, and let $r_{n}=a_{n}-a_{n-1}$, so that $\sum_{n}r_{n}=\lim_{n\rightarrow\infty}a_{n}$. Then: \begin{align} \sum_{n=0}^{\infty}r_{n} & \overset{\mathbb{R}}{=}0\\ \sum_{n=0}^{\infty}r_{n} & \overset{\mathbb{Q}_{p}}{=}1 \end{align} because $a_{n}$ tends to $0$ in $\mathbb{R}$ and tends to $1$ in $\mathbb{Q}_{p}$. \end{example} \vphantom{} This is, quite literally, the oldest mistake in the subject, and still worth pointing out to neophytes over a century later; I, myself, fell for it in an earlier iteration of this dissertation. Nevertheless, in this work, we are going to do what Hensel \emph{thought }he could do, but in our case, \emph{it will be justified}. Such shenanigans will occur multiple times in this dissertation, so it is worth taking a moment to explain in detail what we are going to do, and why it actually \emph{works}. And the exemplary exception that saves us is none other than the tried and true geometric series formula. \begin{fact}[\textbf{\emph{Geometric Series Universality}}] \textbf{\label{fact:Geometric series universality}} Let\index{geometric series universality} $r\in\mathbb{Q}$, and suppose there is a valued field $\left(\mathbb{F},\left|\cdot\right|_{\mathbb{F}}\right)$ extending $\mathbb{Q}$ so that the series $\sum_{n=0}^{\infty}r^{n}$ converges in $\mathbb{F}$ (i.e., for which $\left|r\right|_{\mathbb{F}}<1$), with the sum converging in $\mathbb{F}$ to $R\in\mathbb{F}$. Then, $R\in\mathbb{Q}$, and, for any valued field $\left(\mathbb{K},\left|\cdot\right|_{\mathbb{K}}\right)$ extending $\mathbb{Q}$ for which the series $\sum_{n=0}^{\infty}r^{n}$ converges in $\mathbb{K}$ (i.e., for which $\left|r\right|_{\mathbb{K}}<1$), the sum converges in $\mathbb{K}$ to $R$. \emph{\cite{Conrad on p-adic series}} \end{fact} \vphantom{} In practice, in our use of this universality, we will have a formal sum $\sum_{n=0}^{\infty}a_{n}$ of rational numbers $\left\{ a_{n}\right\} _{n\geq0}\subseteq\mathbb{Q}$ such that $\sum_{n=0}^{\infty}a_{n}$ can be written as the sum of finitely many geometric series: \begin{equation} \sum_{n=0}^{\infty}b_{1}r_{1}^{n}+\sum_{n=0}^{\infty}b_{2}r_{2}^{n}+\sum_{n=0}^{\infty}b_{M}r_{M}^{n} \end{equation} where the $b_{m}$s and $r_{m}$s are rational numbers. In particular, there will exist an integer $c\geq1$ so that: \begin{equation} \sum_{n=0}^{cN-1}a_{n}\overset{\mathbb{Q}}{=}\sum_{n=0}^{N-1}b_{1}r_{1}^{n}+\sum_{n=0}^{N-1}b_{2}r_{2}^{n}+\sum_{n=0}^{N-1}b_{M}r_{M}^{n},\textrm{ }\forall N\geq1\label{eq:Partial sum decomposed as sum of partial sums of geometric series} \end{equation} Then, if there is a prime $p$ so that $\left|r_{m}\right|<1$ and $\left|r_{m}\right|_{p}<1$ for all $m\in\left\{ 1,\ldots,M\right\} $, the series: \begin{equation} \sum_{n=0}^{N-1}b_{m}r_{m}^{n} \end{equation} will converge to $\frac{b_{m}}{1-r_{m}}$ in both $\mathbb{R}$ \emph{and }$\mathbb{Q}_{p}$ as $N\rightarrow\infty$. \begin{example} The geometric series identity: \[ \sum_{n=0}^{\infty}\left(\frac{3}{4}\right)^{n}=\frac{1}{1-\frac{3}{4}}=4 \] holds in both $\mathbb{R}$ and $\mathbb{Q}_{3}$, in that the series converges in both fields' topologies, and the limit of its partial sum is $4$ in both of those topologies. Indeed: \begin{equation} \sum_{n=0}^{N-1}\left(\frac{3}{4}\right)^{n}=\frac{1-\left(\frac{3}{4}\right)^{N}}{1-\frac{3}{4}}=4-4\left(\frac{3}{4}\right)^{N} \end{equation} and so: \[ \lim_{N\rightarrow\infty}\left|4-\sum_{n=0}^{N-1}\left(\frac{3}{4}\right)^{n}\right|=\lim_{N\rightarrow\infty}4\left(\frac{3}{4}\right)^{N}\overset{\mathbb{R}}{=}0 \] and: \begin{equation} \lim_{N\rightarrow\infty}\left|4-\sum_{n=0}^{N-1}\left(\frac{3}{4}\right)^{n}\right|_{3}=\lim_{N\rightarrow\infty}\left|4\left(\frac{3}{4}\right)^{N}\right|_{3}=\lim_{N\rightarrow\infty}3^{-N}\overset{\mathbb{R}}{=}0 \end{equation} \end{example} \vphantom{} Our second technique for applying universality is the observation that, \emph{for any prime $p$, if a geometric series $\sum_{n=0}^{\infty}r^{n}$ (where $r\in\mathbb{Q}\cap\mathbb{Q}_{p}$) converges in $\mathbb{Q}_{p}$, then its sum, $\frac{1}{1-r}$, is }\textbf{\emph{also}}\emph{ a rational number. }In light of these two tricks, we will consider situations like the following. Here, let $p$ and $q$ be primes (possibly with $p=q$), and consider a function $f:\mathbb{Z}_{p}\rightarrow\mathbb{Q}_{q}$. Then, if there is a subset $U\subseteq\mathbb{Z}_{p}$ such that, for each $\mathfrak{z}\in U$, $f\left(\mathfrak{z}\right)$ is expressible a finite linear combination of geometric series: \begin{equation} f\left(\mathfrak{z}\right)\overset{\mathbb{Q}_{q}}{=}\sum_{m=1}^{M}\sum_{n=0}^{\infty}b_{m}r_{m}^{n} \end{equation} where the $b_{m}$s and the $r_{m}$s are elements of $\mathbb{Q}$, we can view the restriction $f\mid_{U}$ as a rational-valued function $f\mid_{U}:U\rightarrow\mathbb{Q}$. More generally, we can view $f\mid_{U}$ as a real-, or even complex-valued function on $U$. Depending on the values of $\left|r\right|$ and $\left|r\right|_{q}$, we may be able to compute the sum $\sum_{m=1}^{M}\sum_{n=0}^{\infty}b_{m}r_{m}^{n}$ solely in the topology of $\mathbb{R}$ or $\mathbb{C}$, solely in the topology of $\mathbb{Q}_{q}$, or in both, simultaneously. \chapter{\label{chap:2 Hydra-Maps-and}Hydra Maps and their Numina} \includegraphics[scale=0.45]{./PhDDissertationEroica2.png} \vphantom{} Chapter 2 splits neatly (though lop-sidedly) into two halves. Section \ref{sec:2.1 Hydra-Maps-=00003D000026 history} introduces Hydra maps\textemdash my attempt to provide a taxonomy for some of the most important Collatz-type maps, accompanied with numerous examples. Subsection \ref{subsec:2.1.2 It's-Probably-True} is a historical essay on the Collatz Conjecture and most of the most notable efforts to lay bare its mysteries. Section \ref{sec:2.2-the-Numen}, meanwhile, is where we begin with new results. The focus of \ref{sec:2.2-the-Numen} is the construction of the titular \textbf{numen}, a $\left(p,q\right)$-adic function we can associate to any sufficiently well-behaved Hydra map, and the elucidation of its most important properties, these being its characterization as the unique solution of a system of $\left(p,q\right)$-adic functional equations\textemdash subject to a continuity-like condition\textemdash as well as the all-important \textbf{Correspondence Principle}, the first major result of my dissertation. Presented in Subsection \ref{subsec:2.2.3 The-Correspondence-Principle}\textemdash and in four different variants, no less\textemdash the Correspondence Principle establishes a direct correspondence between the values taken by a numen and the periodic points and divergent points of the Hydra map to which it is associated. \ref{sec:2.2-the-Numen} concludes with Subsection \ref{subsec:2.2.4 Other-Avenues}, in which further avenues of exploration are discussed, along with connections to other recent work, principally Mih\u{a}ilescu's resolution of \textbf{Catalan's Conjecture }and the shiny new state-of-the-art result on Collatz due to Tao in 2019-2020 \cite{Tao Probability paper}. \section{\label{sec:2.1 Hydra-Maps-=00003D000026 history}History and Hydra Maps} Much has been written about the Collatz Conjecture, be it efforts to solve it, or efforts to dissuade others from trying. Bibliographically speaking, I would place Lagarias' many writings on the subject\footnote{Such as his papers \cite{Lagarias-Kontorovich Paper,Lagarias' Survey}, and particularly his book \cite{Ultimate Challenge}.}, Wirsching's Book \cite{Wirsching's book on 3n+1}, and\textemdash for this dissertation's purposes, Tao's 2019 paper \cite{Tao Probability paper}\textemdash near the top of the list, along with K.R. Matthews maintains a dedicated $3x+1$ page on his website, \href{http://www.numbertheory.org/3x\%2B1/}{page of Matthews' website}, filled with interesting links, pages, and, of course, his fascinating powerpoint slides \cite{Matthews' slides,Matthews' Leigh Article,Matthews and Watts}. The interested reader can survey these and other sources referenced by this dissertation to get the ``lay of the land''\textemdash to the extent that ``the land'' exists in this particular context. The purpose of this section is two-fold: to introduce this dissertation's fundamental objects of study\textemdash Hydra maps\textemdash and to provide a short, likely inadequate history of the work that has been done the Collatz Conjecture. \subsection{\emph{\label{subsec:2.1.1 Release-the-Hydras!}Release the Hydras! }- An Introduction to Hydra Maps} We begin with the definition: \begin{defn} \label{def:p-Hydra map}\nomenclature{$H$}{a Hydra map}\index{$p$-Hydra map} \index{Hydra map!one-dimensional}\index{Hydra map}Fix an integer $p\geq2$. A \textbf{$p$-Hydra map} is a map $H:\mathbb{Z}\rightarrow\mathbb{Z}$ of the form: \begin{equation} H\left(n\right)=\begin{cases} \frac{a_{0}n+b_{0}}{d_{0}} & \textrm{if }n\overset{p}{\equiv}0\\ \frac{a_{1}n+b_{1}}{d_{1}} & \textrm{if }n\overset{p}{\equiv}1\\ \vdots & \vdots\\ \frac{a_{p-1}n+b_{p-1}}{d_{p-1}} & \textrm{if }n\overset{p}{\equiv}p-1 \end{cases},\textrm{ }\forall n\in\mathbb{Z}\label{eq:Def of a Hydra Map on Z} \end{equation} and where $a_{j}$, $b_{j}$, and $d_{j}$ are integer constants (with $a_{j},d_{j}\geq0$ for all $j$) so that the following two properties hold: \vphantom{} I. (``co-primality'') $a_{j},d_{j}>0$ and $\gcd\left(a_{j},d_{j}\right)=1$ for all $j\in\left\{ 0,\ldots,p-1\right\} $. \vphantom{} II. (``one-sidedness'') $H\left(\mathbb{N}_{0}\right)\subseteq\mathbb{N}_{0}$ and $H\left(-\mathbb{N}_{1}\right)\subseteq-\mathbb{N}_{1}$, where $-\mathbb{N}_{1}=\left\{ -1,-2,-3,\ldots\right\} $. \vphantom{} I call $H$ an \textbf{integral }Hydra map\index{Hydra map!integral}, if, in addition, it satisfies: \vphantom{} III. (``integrality''\footnote{For future or broader study, this condition might be weakened, or even abandoned. It is not needed for the construction of the numen, for example.}) For each $j\in\left\{ 0,\ldots,p-1\right\} $, $\frac{a_{j}n+b_{j}}{d_{j}}$ is an integer if and only if $n\overset{p}{\equiv}j$. \vphantom{} Hydra maps not satisfying (III) are said to be \textbf{non-integral}.\index{Hydra map!non-integral} Finally, $H$ is said to be \textbf{prime }whenever $p$ is prime. \end{defn} \vphantom{} When speaking of or working with these maps, the following notions are quite useful: \begin{defn}[\textbf{Branches of $H$}] For each $j\in\mathbb{Z}/p\mathbb{Z}$, we write $H_{j}:\mathbb{R}\rightarrow\mathbb{R}$ to denote the \textbf{$j$th branch}\index{Hydra map!$j$th branch} of $H$, defined as the function: \begin{equation} H_{j}\left(x\right)\overset{\textrm{def}}{=}\frac{a_{j}x+b_{j}}{d_{j}}\label{eq:Definition of H_j} \end{equation} \end{defn} Next, as a consequence of (\ref{eq:Def of a Hydra Map on Z}), observe that for all $m\in\mathbb{N}_{0}$ and all $j\in\left\{ 0,\ldots,p-1\right\} $, we can write: \[ \underbrace{H\left(pm+j\right)}_{\in\mathbb{N}_{0}}=\frac{a_{j}\left(pm+j\right)+b_{j}}{d_{j}}=\frac{pa_{j}m}{d_{j}}+\frac{ja_{j}+b_{j}}{d_{j}}=\frac{pa_{j}}{d_{j}}m+\underbrace{H\left(j\right)}_{\in\mathbb{N}_{0}} \] Setting $m=1$, we see that the quantity $\frac{pa_{j}}{d_{j}}=H\left(pm+j\right)-H\left(j\right)$ must be an integer for each value of $j$. This quantity occurs often enough in computations to deserve getting a symbol of its own. \begin{defn}[$\mu_{j}$] We write \nomenclature{$\mu_{j}$}{$\overset{\textrm{def}}{=}\frac{pa_{j}}{d_{j}}$}$\mu_{j}$ to denote: \begin{equation} \mu_{j}\overset{\textrm{def}}{=}\frac{pa_{j}}{d_{j}}\in\mathbb{N}_{1},\textrm{ }\forall j\in\mathbb{Z}/p\mathbb{Z}\label{eq:Def of mu_j} \end{equation} In particular, since $a_{j}$ and $d_{j}$ are always co-prime, \textbf{note that $d_{j}$ must be a divisor of $p$}. In particular, this then forces $\mu_{0}/p$ (the derivative of $H_{0}$) to be a positive rational number which is not equal to $1$. \end{defn} \begin{rem} With this notation, we then have that: \begin{equation} H\left(pm+j\right)=\mu_{j}m+H\left(j\right),\textrm{ }\forall m\in\mathbb{Z},\textrm{ }\forall j\in\mathbb{Z}/p\mathbb{Z}\label{eq:Value of rho m + j under H} \end{equation} \end{rem} \vphantom{} For lack of a better or more descriptive name, I have decided to call maps of this form $p$-Hydra maps, in reference to the many-headed monster of Greek myth. Much like their legendary namesake, Hydra maps are not easily conquered. Questions and conjectures regarding these maps' dynamical properties\textemdash such as the number of map's periodic points, the existence of divergent trajectories for a given map, or the total number of the map's orbit classes\textemdash rank among the most difficult problems in all mathematics \cite{Lagarias' Survey}. Case in point, the quintessential example of a Hydra map\textemdash for the handful of people who aren't already aware of it\textemdash is the infamous Collatz map: \begin{defn} \label{def:The-Collatz-map}The\index{Collatz!map}\index{$3x+1$ map} \textbf{Collatz map} $C:\mathbb{Z}\rightarrow\mathbb{Z}$ is the function defined by: \begin{equation} C\left(n\right)\overset{\textrm{def}}{=}\begin{cases} \frac{n}{2} & \textrm{if }n\overset{2}{\equiv}0\\ 3n+1 & \textrm{if }n\overset{2}{\equiv}1 \end{cases}\label{eq:Collatz Map} \end{equation} \end{defn} First proposed by Lothar Collatz\footnote{Actually, Collatz came up with the map in 1929-1930; Matthews' website has a translation of a letter by Collatz himself in which the man states as such. It arose, apparently, as a diversion during time Collatz spent attending classes by Perron and Schur, among others \cite{Collatz Letter}.} in 1937 \cite{Collatz Biography}, the eponymous map's well-deserved mathematical infamy stems from the egregiously simple, yet notoriously difficult conjecture of the same name \cite{Lagarias' Survey,Collatz Biography}: \begin{conjecture}[\textbf{Collatz Conjecture}\footnote{Also known as the $3n+1$ problem, the $3x+1$ problem, the Syracuse Problem, the Hailstone problem, to give but a few of its many monickers.} \index{Collatz!Conjecture}] \label{conj:Collatz}For every $n\in\mathbb{N}_{1}$, there exists a $k\geq0$ so that $C^{\circ k}\left(n\right)=1$. \end{conjecture} \begin{rem} An equivalent reformulation of this result is the statement: \emph{$\mathbb{N}_{1}$ is an irreducible orbit class of $C$.} \end{rem} \vphantom{} A moment's thought reveals that the Collatz Conjecture actually consists of \emph{two} separate statements: \begin{conjecture}[\textbf{Weak Collatz Conjecture}\footnote{Also known as the ``(No) Finite Cycles'' Conjecture.}] \label{conj:Weak Collatz}The only periodic points of $C$ in $\mathbb{N}_{1}$ are $1$, $2$, and $4$. \end{conjecture} \begin{conjecture}[\textbf{Divergent Trajectories Conjecture}] $C$ has no divergent trajectories in $\mathbb{N}_{1}$. That is to say, for each $n\in\mathbb{N}_{1}$: \begin{equation} \sup_{k\geq0}C^{\circ k}\left(n\right)<\infty\label{eq:No divergent trajectories} \end{equation} \end{conjecture} \vphantom{} Since $3n+1$ will be even whenever $n$ is odd, it is generally more expedient to alter $C$ by inserting an extra division by $2$ to obtain the so-called Shortened Collatz map: \begin{defn} \label{def:The-Shortened-Collatz Map}\nomenclature{$T_{3}$}{shortened Collatz map}The\index{$3x+1$ map} \index{Collatz!map}\textbf{Shortened Collatz map }$T_{3}:\mathbb{Z}\rightarrow\mathbb{Z}$ is defined by: \begin{equation} T_{3}\left(n\right)\overset{\textrm{def}}{=}\begin{cases} \frac{n}{2} & \textrm{if }n\overset{2}{\equiv}0\\ \frac{3n+1}{2} & \textrm{if }n\overset{2}{\equiv}1 \end{cases}\label{eq:Definition of T_3} \end{equation} \end{defn} \vphantom{} The general dynamics of which are identical to that of $C$, with $T_{3}\left(n\right)=C\left(n\right)$ for all even $n$ and $T_{3}\left(n\right)=C\left(C\left(n\right)\right)$ for all odd $n$. The use of $T$ or $T_{3}$ to denote this map is standard in the literature (ex. \cite{Lagarias-Kontorovich Paper}, \cite{Wirsching's book on 3n+1}). After the Collatz map $C$ and its shortened counterpart, $T_{3}$, the next most common level of generalization comes from considering the larger family of ``shortened $qx+1$ maps'': \begin{defn} \label{def:Shortened qx plus 1 map}\nomenclature{$T_{q}$}{shortened $qx+1$ map}Let $q$ be an odd integer $\geq1$. Then, the \textbf{Shortened $qx+1$ map}\index{$qx+1$ map} $T_{q}:\mathbb{Z}\rightarrow\mathbb{Z}$ is the function defined by: \begin{equation} T_{q}\left(n\right)\overset{\textrm{def}}{=}\begin{cases} \frac{n}{2} & \textrm{if }n\overset{2}{\equiv}0\\ \frac{qn+1}{2} & \textrm{if }n\overset{2}{\equiv}1 \end{cases}\label{eq:Definition of T_q} \end{equation} \end{defn} \vphantom{} Of the (shortened) $qx+1$ maps, the most well-known are $T_{3}$ and $T_{5}$, with $T_{5}$ being of interest because its behavior seems to be diametrically opposed to that of $T_{3}$: one the one hand, the set of positive integers iterated by $T_{3}$ to $\infty$ has zero density \cite{Lagarias' Survey,Terras 76,Terras 79}; on the other hand, $T_{5}$ iterates almost every positive integer to $\infty$ \cite{Lagarias-Kontorovich Paper}. Nevertheless, despite the probabilistic guarantee that almost every positive integer should belong to a divergent trajectory, it has yet to be proven that any \emph{specific} positive integer belongs to a divergent trajectory \cite{Lagarias-Kontorovich Paper}! A notable feature of Collatz Studies is its overriding single-mindedness. Discussions of $C$ and its various shortened versions\footnote{Tao, for example, considers a variant which sends an $n$ odd integer to the next odd integer $n^{\prime}$ in the forward orbit of $n$ under $C$; that is $n\mapsto\left(3n+1\right)\left|3n+1\right|_{2}$, where $\left|\cdot\right|_{2}$ is the $2$-adic absolute value \cite{Tao Blog}.} predominate, with most in-depth number theoretic studies of Hydra maps being focused on $C$. While other Hydra maps do receive a certain degree of attention, it is generally from a broader, more holistic viewpoint. Some of the best examples of these broader studies can be found works of \index{Matthews, K. R.}K. R. Matthews and his colleagues and collaborators, going all the way back to the 1980s. The \emph{leitmotif} of this body of work is the use of Markov chains to model the maps' dynamics. These models provide a particularly fruitful heuristic basis for conjectures regarding the maps' ``typical'' trajectories, and the expected preponderance of cycles relative to divergent trajectories and vice-versa \cite{dgh paper,Matthews' Conjecture,Matthews' Leigh Article,Matthews' slides,Matthews and Watts}. This line of investigation dates back to the work of H. Mller in the 1970s \cite{Moller's paper (german)}, who formulated a specific type of generalized multiple-branched Collatz map and made conjectures about the existence of cycles and divergent trajectories. Although probabilistic techniques of this sort will not on our agenda here, Matthews and his colleagues' investigations provide ample ``non-classical'' examples of what I call Hydra maps. \begin{example} Our first examples are due to Leigh (\cite{Leigh's article,Matthews' Leigh Article}); we denote these as $L_{1}$ and $L_{2}$, respectively: \begin{equation} L_{1}\left(n\right)\overset{\textrm{def}}{=}\begin{cases} \frac{n}{4} & \textrm{if }n\overset{8}{\equiv}0\\ \frac{n+1}{2} & \textrm{if }n\overset{8}{\equiv}1\\ 20n-40 & \textrm{if }n\overset{8}{\equiv}2\\ \frac{n-3}{8} & \textrm{if }n\overset{8}{\equiv}3\\ 20n+48 & \textrm{if }n\overset{8}{\equiv}4\\ \frac{3n-13}{2} & \textrm{if }n\overset{8}{\equiv}5\\ \frac{11n-2}{4} & \textrm{if }n\overset{8}{\equiv}6\\ \frac{n+1}{8} & \textrm{if }n\overset{8}{\equiv}7 \end{cases} \end{equation} \begin{equation} L_{2}\left(n\right)\overset{\textrm{def}}{=}\begin{cases} 12n-1 & \textrm{if }n\overset{4}{\equiv}0\\ 20n & \textrm{if }n\overset{4}{\equiv}1\\ \frac{3n-6}{4} & \textrm{if }n\overset{4}{\equiv}2\\ \frac{n-3}{4} & \textrm{if }n\overset{4}{\equiv}3 \end{cases} \end{equation} Note that neither $L_{1}$ nor $L_{2}$ satisfy the integrality condition, seeing as they have branches which are integer-valued for all integer inputs. Also, while $L_{1}$ fixes $0$, $L_{2}$ sends $0$ to $-1$, and as such, $L_{2}$ is not a $4$-Hydra map \emph{as written}; this can be rectified by conjugating $L_{2}$ by an appropriately chosen affine linear map\textemdash that is, one of the form $an+b$, where $a$ and $b$ are integers; here, $a$ needs to be co-prime to $p=4$. Setting $a=1$ gives: \begin{equation} L_{2}\left(n-b\right)+b=\begin{cases} 12n-11b-1 & \textrm{if }n\overset{4}{\equiv}b\\ 20n-19b & \textrm{if }n\overset{4}{\equiv}b+1\\ \frac{3n+b-6}{4} & \textrm{if }n\overset{4}{\equiv}b+2\\ \frac{n+3b-3}{4} & \textrm{if }n\overset{4}{\equiv}b+3 \end{cases} \end{equation} Setting $b=1$ then yields the map $\tilde{L}_{2}:\mathbb{N}_{0}\rightarrow\mathbb{N}_{0}$: \begin{equation} \tilde{L}_{2}\left(n\right)\overset{\textrm{def}}{=}L_{2}\left(n-1\right)+1=\begin{cases} \frac{n}{4} & \textrm{if }n\overset{4}{\equiv}0\\ 12n-12 & \textrm{if }n\overset{4}{\equiv}1\\ 20n-19 & \textrm{if }n\overset{4}{\equiv}2\\ \frac{3n-5}{4} & \textrm{if }n\overset{4}{\equiv}3 \end{cases}\label{eq:Conjugated Hydra map} \end{equation} which, unlike $L_{2}$, satisfies conditions (I) and (II) for in (\ref{eq:Def of a Hydra Map on Z}). Conjugating maps in this way is a useful tool to put them in a ``standard form'' of sorts, one in which the conjugated map sends the non-negative integers $\mathbb{N}_{0}$ to $\mathbb{N}_{0}$ and, preferably, also fixes $0$. \end{example} \vphantom{} Our next example of a map is also non-integral, yet, it is significant because it possess \emph{provably} divergent trajectories\index{divergent!trajectory}. Moreover, it may be useful to broaden the notion of Hydra maps to include maps like these, seeing as, despite its integrality failure, we can still construct its numen using the methods of Section \ref{sec:2.2-the-Numen}. \begin{example}[\textbf{Matthews' Map \& Divergent Trajectories}] \label{exa:Matthews' map}The following map is due to Matthews \cite{Matthews' Conjecture}: \begin{equation} M\left(n\right)\overset{\textrm{def}}{=}\begin{cases} 7n+3 & \textrm{if }n\overset{3}{\equiv}0\\ \frac{7n+2}{3} & \textrm{if }n\overset{3}{\equiv}1\\ \frac{n-2}{3} & \textrm{if }n\overset{3}{\equiv}2 \end{cases}\label{eq:Matthews' Conjecture Map} \end{equation} That this map fails to satisfy the integrality requirement can be seen in the fact that its $0$th branch $7n+3$ outputs integers even when $n$ is \emph{not }congruent to $0$ mod $3$. That being said, as defined, note that $-1$ is the only fixed point of $M$ in $\mathbb{Z}$. For our purposes, we will need to work with maps that fix $0$. To do so, we can conjugate $M$ by $f\left(n\right)=n-1$ ($f^{-1}\left(n\right)=n+1$) to obtain: \begin{equation} \tilde{M}\left(n\right)\overset{\textrm{def}}{=}\left(f^{-1}\circ M\circ f\right)\left(n\right)=M\left(n-1\right)+1=\begin{cases} \frac{n}{3} & \textrm{if }n\overset{3}{\equiv}0\\ 7n-3 & \textrm{if }n\overset{3}{\equiv}1\\ \frac{7n-2}{3} & \textrm{if }n\overset{3}{\equiv}2 \end{cases}\label{eq:Matthews' Conjecture Map, conjugated} \end{equation} Conjugating this map by $g\left(n\right)=-n$ ($g^{-1}=g$) then allows us to study $\tilde{T}$'s behavior on the negative integers: \begin{equation} \tilde{T}_{-}\left(n\right)\overset{\textrm{def}}{=}-\tilde{T}\left(-n\right)=\begin{cases} \frac{n}{3} & \textrm{if }n\overset{3}{\equiv}0\\ \frac{7n+2}{3} & \textrm{if }n\overset{3}{\equiv}1\\ 7n+3 & \textrm{if }n\overset{3}{\equiv}2 \end{cases}\label{eq:Matthews' conjecture map, conjugated to the negatives} \end{equation} In general, after conjugating a map (Hydra or not) so that it fixes $0$, conjugating by $g\left(n\right)=-n$ is how one studies the behavior of the maps on the non-positive integers. \end{example} \begin{rem} It may be of interest to note that the $1$st and $2$nd branches $\tilde{M}_{1}\left(x\right)=7x-3$ and $\tilde{M}_{2}\left(x\right)=\frac{7x-2}{3}$ commute with one another: \begin{equation} \tilde{M}_{1}\left(\tilde{M}_{2}\left(x\right)\right)=7\left(\frac{7x-2}{3}\right)-3=\frac{49x-14}{3}-3=\frac{49x-23}{3} \end{equation} \begin{equation} \tilde{M}_{2}\left(\tilde{M}_{1}\left(x\right)\right)=\frac{7\left(7x-3\right)-2}{3}=\frac{49x-21-2}{3}=\frac{49x-23}{3} \end{equation} \end{rem} \vphantom{} As was mentioned above, the import of $M$ lies in the fact that it has provably divergent trajectories: \begin{prop} \label{prop:Matthews' map}$M$ has at least two divergent trajectories over $\mathbb{Z}$: \vphantom{} I. The set of all non-negative integer multiples of $3$ (which are iterated to $+\infty$); \vphantom{} II. The set of all negative integer multiples of three (which are iterated to $-\infty$). \end{prop} Proof: Since $7n+3$ is congruent to $0$ mod $3$ whenever $n$ is congruent to $0$ mod $3$, any non-negative (resp. negative) integer multiple of $3$ is iterated by $M$ to positive (resp. negative) infinity. Q.E.D. \vphantom{} For extra incentive, it is worth mentioning that there is money on the table. \index{Matthews, K. R.}Matthews has put a bounty of $100$ Australian dollars, to be given to anyone who can bring him the severed, fully-proven head of the following conjecture: \begin{conjecture}[\textbf{Matthews' Conjecture}\index{Matthews' Conjecture} \cite{Matthews' Conjecture}] \label{conj:Matthews conjecture}The irreducible orbit classes of $M$ in $\mathbb{Z}$ are the divergent orbit classes associated to the trajectories (I) and (II) described in \textbf{\emph{Proposition \ref{prop:Matthews' map}}}, and the orbit classes corresponding to the attracting cycles $\left\{ -1\right\} $ and $\left\{ -2,-4\right\} $. \end{conjecture} \vphantom{} Matthews' powerpoint slides \cite{Matthews' slides} (freely accessible from his website) contain many more examples, both along these lines, and in further, more generalized forms. The slides are quite comprehensive, detailing generalizations and conjectures due to Hasse and Herbert Mller\index{Mller, Herbert}, as well as delving into the details of the aforementioned Markov-chain-based analysis of the maps and their dynamics. The slides also detail how the Markov methods can be extended along with the maps from $\mathbb{Z}$ to spaces of $p$-adic integers, as well as to the ring of profinite integers\index{profinite integers} (a.k.a,\emph{ polyadic integers}) given by the direct product $\prod_{p\in\mathbb{P}}\mathbb{Z}_{p}$. However, for our purposes, the most significant offering in Matthews' slides are the generalizations given in which simple Collatz-type maps are defined over discrete spaces other than $\mathbb{Z}$. \begin{example}[\textbf{A ``Multi-Dimensional'' Hydra Map}] The first example of such an extension appears to from Leigh in 1983\cite{Matthews' slides}. Leigh considered the map\footnote{The notation $T_{3,\sqrt{2}}$ is my own.} $T_{3,\sqrt{2}}:\mathbb{Z}\left[\sqrt{2}\right]\rightarrow\mathbb{Z}\left[\sqrt{2}\right]$ defined by: \begin{equation} T_{3,\sqrt{2}}\left(z\right)\overset{\textrm{def}}{=}\begin{cases} \frac{z}{\sqrt{2}} & \textrm{if }z\overset{\sqrt{2}}{\equiv}0\\ \frac{3z+1}{\sqrt{2}} & \textrm{if }z\overset{\sqrt{2}}{\equiv}1 \end{cases},\textrm{ }\forall z\in\mathbb{Z}\left[\sqrt{2}\right]\label{eq:Definition of T_3,root 2} \end{equation} which acts on the ring of quadratic integers of the form $a+b\sqrt{2}$, where $a,b\in\mathbb{Z}$. Here, as indicated, a congruence $z\overset{\sqrt{2}}{\equiv}w$ for $z,w\in\mathbb{Z}\left[\sqrt{2}\right]$ means that $z-w$ is of the form $v\sqrt{2}$, where $v\in\mathbb{Z}\left[\sqrt{2}\right]$. Thus, for example, $3+2\sqrt{2}\overset{\sqrt{2}}{\equiv}1$, since: \begin{equation} 3+2\sqrt{2}-1=2+2\sqrt{2}=\left(\sqrt{2}+2\right)\sqrt{2} \end{equation} Leigh's \index{multi-dimensional!Hydra map}map is the prototypical example of what I call a \index{Hydra map!multi-dimensional}\textbf{Multi-Dimensional Hydra Map}. The ``multi-dimensionality'' becomes apparent after applying a bit of linear algebra. We can establish a ring isomorphism of $\mathbb{Z}\left[\sqrt{2}\right]$ and $\mathbb{Z}^{2}$ by associating $z=a+b\sqrt{2}\in\mathbb{Z}\left[\sqrt{2}\right]$ with the column vector $\left(a,b\right)$. A bit of algebra produces: \begin{equation} T_{3,\sqrt{2}}\left(a+b\sqrt{2}\right)=\begin{cases} b+\frac{a}{2}\sqrt{2} & \textrm{if }a=0\mod2\\ 3b+\frac{3a+1}{2}\sqrt{2} & \textrm{if }a=1\mod2 \end{cases},\textrm{ }\forall\left(a,b\right)\in\mathbb{Z}^{2}\label{eq:T_3,root 2 of a plus b root 2} \end{equation} Expressing the effect of $T_{3,\sqrt{2}}$ on $\left(a,b\right)$ in terms of these coordinates then gives us a map $\tilde{T}_{3,\sqrt{2}}:\mathbb{Z}^{2}\rightarrow\mathbb{Z}^{2}$ defined by: \begin{eqnarray} \tilde{T}_{3,\sqrt{2}}\left(\left[\begin{array}{c} a\\ b \end{array}\right]\right) & \overset{\textrm{def}}{=} & \begin{cases} \left[\begin{array}{cc} 1 & 0\\ 0 & 2 \end{array}\right]^{-1}\left(\left[\begin{array}{cc} 0 & 1\\ 1 & 0 \end{array}\right]\left[\begin{array}{c} a\\ b \end{array}\right]\right) & \textrm{if }a=0\mod2\\ \left[\begin{array}{cc} 1 & 0\\ 0 & 2 \end{array}\right]^{-1}\left(\left[\begin{array}{cc} 0 & 3\\ 3 & 0 \end{array}\right]\left[\begin{array}{c} a\\ b \end{array}\right]+\left[\begin{array}{c} 0\\ 1 \end{array}\right]\right) & \textrm{if }a=1\mod2 \end{cases}\label{eq:Definition of the Lattice analogue of T_3,root 2} \end{eqnarray} which is a ``multi-dimensional'' analogue of a Hydra map; the branch: \begin{equation} n\mapsto\frac{an+b}{d} \end{equation} \textemdash an affine linear map on $\mathbb{Q}$\textemdash has been replaced by an affine linear map: \begin{equation} \mathbf{n}\mapsto\mathbf{D}^{-1}\left(\mathbf{A}\mathbf{n}+\mathbf{b}\right) \end{equation} on $\mathbb{Q}^{2}$, for $2\times2$ invertible matrices $\mathbf{A}$ and $\mathbf{D}$ and a $2$-tuple $\mathbf{b}$, all of which have integer entries. In this way, notion of Hydra map defined at the beginning of this subsection is really but a ``one-dimensional'' incarnation of the more general notion of a map on a finitely generated module over a principal ideal domain. Note in (\ref{eq:Definition of the Lattice analogue of T_3,root 2}) the presence of the permutation matrix: \begin{equation} \left[\begin{array}{cc} 0 & 1\\ 1 & 0 \end{array}\right] \end{equation} which swaps the coordinate entries of a given $2$-tuple. As will be seen in Section \ref{sec:5.1 Hydra-Maps-on}, this intertwining of an affine linear map involving diagonal matrices with permutation matrices is characteristic of the matrix representations of multi-dimensional Hydra maps, illustrating the much greater degrees of freedom and flexibility engendered by stepping out of $\mathbb{Z}$ and into spaces of higher dimensionality. \end{example} \vphantom{} Other than one additional example in Leigh's vein\textemdash presented at the very end of Subsection \ref{subsec:5.1.2 Co=00003D0000F6rdinates,-Half-Lattices,-and}\textemdash Matthews told me in our 2019 correspondence that he knew of no other examples of Hydra maps of this type in the literature. For all intents and purposes, the subject is simply unstudied. All the more reason, then, for someone to try to whip it into shape. \cite{Matthews' slides} goes even further, presenting near the end examples of Hydra-like maps defined on a ring of polynomials with coefficients in a finite field. While it may be possible to extend my techniques to study such maps, I have yet to give consideration to how my methods will apply when working with Hydra-like maps on spaces of positive characteristic. Finally, it should be mentioned that, for simplicity's sake, this dissertation only concerns prime Hydra maps\textemdash those for which $p$ is prime. While it remains to be seen whether or note my construction of $\chi_{H}$ holds water in the case where $p$ is an arbitrary composite integer, it seems pretty straightforward to modify the theory presented here to deal with the case where $p$ is the power of a prime\textemdash so, a $p^{n}$-Hydra map, where $p$ is prime and $n\geq1$. \newpage{} \subsection{\emph{\label{subsec:2.1.2 It's-Probably-True}}A Short History of the Collatz Conjecture} \begin{quote} \begin{flushright} \emph{As you know, the $3x+1$ problem is a notorious problem, with perhaps my favorite quote calling it a Soviet conspiracy to slow down American mathematics.} \par\end{flushright} \begin{flushright} \textemdash Stephen J. Miller\footnote{Professor Miller made this remark to me as part of an e-mail correspondence with me back in June of 2021.} \par\end{flushright} \end{quote} \index{Collatz!Conjecture} The Collatz Conjecture is infamous for playing hard to get. With other difficult problems, lack of progress toward a solution often comes with new ideas and insights worth studying in their own right. On the other hand, for Collatz and its ilk, the exact opposite appears to hold: a tool or insight of use for Collatz seems to have little if any applicability to the outside mathematical world. For people\textemdash like myself\textemdash with a vested interest in mainstreaming Collatz studies this is especially problematic. It isn't much help with conquering Collatz, either, given how much mathematical progress owes to the discovery of intriguing new structures, relations, and techniques, even if those novelties might not be immediately applicable to the ``big'' questions we would like to solve. The purpose of this essay is to give an overview of the primary ways in which the mathematical community at large has attempted to frame and understand the Collatz Conjecture and related arithmetical dynamical systems. That my approach is essentially orthogonal to all of the ones we are about to encounter is actually of the utmost relevance to this discussion; I hope it will inspire readers to find other ways of looking beyond the ``standard'' methods, so as to improve our chances of finding exactly the sort of unexpected connections which might be needed to better understand problems like Collatz. To the extent that Collatz studies exists as a coherent subject, however, the most preponderant trends are statistical in nature, where studies center around defining a handful of interesting quantities\textemdash hoping to bottle a sliver of the lightning animating Collatz's behavior\textemdash and then demonstrating that interesting conclusions can be drawn. Arguably the most impactful techniques of the statistical school have their roots in the groundbreaking work done by Riho Terras\index{Terras, Riho} in the 1970s (\cite{Terras 76,Terras 79}; see also Lagarias' survey \cite{Lagarias' Survey}). Terras' paradigm rests upon two statistics: the \textbf{stopping time} and the \textbf{parity sequence} (or \textbf{parity vector}). \begin{defn}[\textbf{Stopping times}\footnote{Adapted from \cite{Lagarias-Kontorovich Paper}.}] Let $\lambda$ be any positive real number, and let $T:\mathbb{Z}\rightarrow\mathbb{Z}$ be a map. \vphantom{} I. For any $n\in\mathbb{Z}$, the \textbf{$\lambda$-decay stopping time}\index{stopping time}\textbf{ }of $n$ under $T$, denoted $\sigma_{T,\lambda}^{-}\left(n\right)$, is the smallest integer $k\geq0$ so that $T^{\circ k}\left(n\right)<\lambda n$: \begin{equation} \sigma_{T,\lambda}^{-}\left(n\right)\overset{\textrm{def}}{=}\inf\left\{ k\geq0:\frac{T^{\circ k}\left(n\right)}{n}<\lambda\right\} \label{eq:Definition of the lambda decay stopping time} \end{equation} \vphantom{} II. For any $n\in\mathbb{Z}$, the \textbf{$\lambda$-growth stopping time }of $n$ under $T$, denoted $\sigma_{T,\lambda}^{+}\left(n\right)$, is the smallest integer $k\geq0$ so that $T^{\circ k}\left(n\right)>\lambda n$: \begin{equation} \sigma_{T,\lambda}^{+}\left(n\right)\overset{\textrm{def}}{=}\inf\left\{ k\geq0:\frac{T^{\circ k}\left(n\right)}{n}>\lambda\right\} \label{eq:Definition of the lambda growth stopping time} \end{equation} \end{defn} \vphantom{} Terras' claim to fame rests on his ingenious implementation of these ideas to prove: \begin{thm}[\textbf{Terras' Theorem} \cite{Terras 76,Terras 79}] Let $S$ be set of positive integers with finite \index{Terras' Theorem}$1$-decay\footnote{i.e., $\lambda$-decay, with $\lambda=1$.} stopping time under the Collatz map $C$. Then, $S$ has a natural density of $1$; that is to say: \begin{equation} \lim_{N\rightarrow\infty}\frac{\left|S\cap\left\{ 1,2,3,\ldots,N\right\} \right|}{N}=1\label{eq:Terras' Theorem} \end{equation} \end{thm} In words, Terras' Theorem is effectively the statement \emph{almost every positive integer does not belong to a divergent trajectory of $C$}; or, equivalently, \emph{the set of divergent trajectories of $C$ in $\mathbb{N}_{1}$ has density $0$}.\emph{ }This result is of double importance: not only does it prove that $S$ \emph{possesses }a density, it also shows that density to be $1$. One of the frustrations of analytic number theory is that the limit (\ref{eq:Terras' Theorem}) defining the natural density of a set $S\subseteq\mathbb{N}_{1}$ does not exist for every\footnote{As an example, the set $S=\bigcup_{n=0}^{\infty}\left\{ 2^{2n},\ldots,2^{2n+1}-1\right\} $ does not have a well-defined natural density. For it, the $\limsup$ of (\ref{eq:Terras' Theorem}) is $2/3$, while the $\liminf$ is $1/3$.} $S\subseteq\mathbb{N}_{1}$. Tao's breakthrough was to improve ``almost every positive integer does not go to infinity'' to ``almost all orbits of the Collatz map attain almost bounded values''\textemdash indeed, that is the very title of his paper \cite{Tao Probability paper}. It is worth noting that by doing little more than replacing every instance of a $3$ in Terras' argument with a $5$\textemdash and adjusting all inequalities accordingly\textemdash an analogous result can be obtained for the Shortened $5x+1$ map\index{$5x+1$ map} (or $T_{q}$, for any odd $q\geq5$) \cite{Lagarias-Kontorovich Paper}: \begin{thm}[\textbf{Terras' Theorem for $qx+1$}] Let $S$ be set of positive integers with finite $1$-growth\footnote{i.e., $\lambda$-growth, with $\lambda=1$.} stopping time under the Shortened $qx+1$ map $T_{q}$. Then, $S$ has a natural density of $1$. \end{thm} \vphantom{} That is to say, while the set of positive integers which belong to divergent trajectories of $T_{3}$ has density $0$, the set of divergent trajectories of $T_{q}$ for $q\geq5$ has density $1$. Surreally, for $q\geq5$, even though we know that almost every positive integer should be iterated to $\infty$ by $T_{q}$, mathematics has yet to prove that any specific positive integer \emph{suspected}\footnote{Such as $7$, for the case of the $5x+1$ map.} of membership in a divergent trajectory actually belongs to one! Next, we have Terras' notion of the \index{parity sequence}\textbf{parity sequence} of a given input $n$. The idea is straightforward: since the behavior of an integer $n$ under iterations of one of the $T_{q}$ maps is determined by the parity of those iterates\textemdash that is, the value mod $2$\textemdash it makes sense to keep track of those values. As such, the parity sequence of $n$ under $T_{q}$ is the sequence $\left[n\right]_{2},\left[T_{q}\left(n\right)\right]_{2},\left[T_{q}^{\circ2}\left(n\right)\right]_{2},\ldots$ consisting of the values (mod $2$) of the iterates of $n$ under the map $T_{q}$ (\ref{eq:Definition of T_q}) \cite{Lagarias' Survey}. More generally, since the iterates of $n$ under an arbitrary $p$-Hydra map $H$ are determined not by their iterates' parities, but by their iterates' values mod $p$, it will be natural to generalize Terras' parity sequence into the \textbf{$p$-ity sequence}\index{$p$-ity sequence}\textbf{ }$\left[n\right]_{p},\left[H\left(n\right)\right]_{p},\left[H^{\circ2}\left(n\right)\right]_{p},\ldots$ consisting of the values mod $p$ of the iterates of $n$ under $H$; doing so will play a crucial role early on in our construction of the numen of $H$. It should not come as much of a surprise that the parity sequence of a given $n$ under $T_{q}$ ends up being intimately connected to the behavior of $T_{q}$'s extension to the $2$-adic integers (see for instance \cite{Parity Sequences}), where we apply the even branch of $T_{q}$ to those $\mathfrak{z}\in\mathbb{Z}_{2}$ with $\mathfrak{z}\overset{2}{\equiv}0$ and apply the odd branch of $T_{q}$ to those $\mathfrak{z}\in\mathbb{Z}_{2}$ with $\mathfrak{z}\overset{2}{\equiv}1$ Extending the environment of study from $\mathbb{Z}$ to $\mathbb{Z}_{2}$ in this manner is another one of Terras' innovations, one which is relevant to our approach for all the obvious reasons. Despite these promising connections, trying to conquer Collatz simply by extending it to the $2$-adics is a quixotic undertaking, thanks to the following astonishing result\footnote{Adapted from \cite{Lagarias-Kontorovich Paper}.}: \begin{thm} \label{thm:shift map}For any odd integer $q\geq1$, the $2$-adic extension of $T_{q}$ is a homeomorphism of $\mathbb{Z}_{2}$, and is topologically and measurably conjugate to the $2$-adic shift map $\theta:\mathbb{Z}_{2}\rightarrow\mathbb{Z}_{2}$ defined by: \begin{equation} \theta\left(c_{0}+c_{1}2^{1}+c_{2}2^{2}+\cdots\right)\overset{\textrm{def}}{=}c_{1}+c_{2}2^{1}+c_{3}2^{2}+\cdots\label{eq:Definition of the 2-adic shift map} \end{equation} meaning that for every odd integer $q\geq1$, there exists a homeomorphism $\Phi_{q}:\mathbb{Z}_{2}\rightarrow\mathbb{Z}_{2}$ which preserves the (real-valued) $2$-adic Haar probability measure\footnote{$\mu\left(\Phi_{q}^{-1}\left(U\right)\right)=\mu\left(U\right)$, for all measurable $U\subseteq\mathbb{Z}_{2}$.} so that: \begin{equation} \left(\Phi_{q}\circ T_{q}\circ\Phi_{q}^{-1}\right)\left(\mathfrak{z}\right)=\theta\left(\mathfrak{z}\right),\textrm{ }\forall\mathfrak{z}\in\mathbb{Z}_{2}\label{eq:Conjugacy of T_q and the shift map} \end{equation} \end{thm} \vphantom{} As this theorem shows, in both a topological and measure theoretic perspective, for any odd integers $p,q\geq1$, the dynamics $T_{p}$ and $T_{q}$ on $\mathbb{Z}_{2}$ are utterly indistinguishable from one another, and from the shift map, which is ergodic on $\mathbb{Z}_{2}$ in a very strong sense \cite{Lagarias-Kontorovich Paper}. Because $\mathbb{Z}$ has zero real-valued Haar measure in $\mathbb{Z}_{2}$, it was already a given that employing tools of \index{ergodic theory}ergodic theory and the measure-theoretic branches of dynamical systems would be something of a wash; those methods cannot detect phenomena occurring in sets of measure zero. However, the conjugate equivalence of the different $T_{q}$ maps would appear to suggest that, at least over the $2$-adics, probabilistic methods will useless for understanding the dynamics of the $qx+1$ maps, let alone the Collatz map. Nevertheless, as shown in \cite{Parity Sequences} and related works, there may yet be fruitful work to be done in the $2$-adic setting, not by directly studying the dynamics of $T_{3}$, but by how certain functions associated to $T_{3}$ affect certain subsets of $\mathbb{Z}_{2}$. Prior to Tao's innovations \cite{Tao Probability paper}, among the motley collection of techniques used to study the Collatz Conjecture, the least \emph{in}effective arguments in support of the Conjecture's truth were based on the so-called ``difference inequalities'' of I. Krasikov\index{Krasikov, Ilia} \cite{Krasikov}. In the 1990s, Applegate and Lagarias\index{Lagarias, Jeffery} used Krasikov's inequalities (along with a computer to assist them with a linear program in $\left(3^{9}-1\right)/2$ variables! \cite{Applegate and Lagarias - Difference inequalities}) to establish the asymptotic: \begin{thm}[\textbf{Applegate-Krasikov-Lagarias} \cite{Applegate and Lagarias - Difference inequalities}] For any integer $a$, define $\pi_{a}:\mathbb{R}\rightarrow\mathbb{N}_{0}$ by: \begin{equation} \pi_{a}\left(x\right)\overset{\textrm{def}}{=}\left|\left\{ n\in\mathbb{Z}:\left|n\right|\leq x\textrm{ and }T^{\circ k}\left(n\right)=a\textrm{ for some }k\in\mathbb{N}_{0}\right\} \right|\label{eq:Definition of Pi_a} \end{equation} Then, for any $a\in\mathbb{Z}$ which is not a multiple of $3$, there is a positive real constant $c_{a}$ so that: \begin{equation} \pi_{a}\left(x\right)\geq c_{a}x^{0.81},\textrm{ }\forall x\geq a\label{eq:Apple-Lag-Kra asymptotic theorem} \end{equation} \end{thm} \vphantom{} This school of work \cite{Applegate and Lagarias - Trees,Applegate and Lagarias - Difference inequalities,Krasikov,Wirsching's book on 3n+1} intersects with graph-theoretic considerations (such as \cite{Applegate and Lagarias - Trees}), a well-established player in probabilistic number theory and additive combinatorics in the twentieth century and beyond. With the exception of Rozier's work in \cite{Parity Sequences}, all of the avenues of inquiry discussed above\textemdash even Tao's!\textemdash suffer the same weakness, inherent to any \emph{probabilistic }approach: they can only establish truths modulo ``small'' sets (zero density, zero measure, etc.). Although results in this tradition provide often deeply insightful, psychologically satisfying substitutes for resolutions of the Collatz Conjecture, in part or in whole, they nevertheless fail to capture or address whatever number-theoretic mechanisms are ultimately responsible for the Collatz's dynamics, and the dynamics of Hydra maps in general. In this respect, the Markov-chain\index{Markov chains}-based work of Mller, Matthews, and their school offers an intriguing contrast, if only because their methods show\textemdash or, at least, \emph{suggest}\textemdash how the dynamical properties of Hydra maps might be related to the constants used to define them. Mller's generalization of Collatz began by fixing co-prime integers $m,d>1$, with $m>d$ and considering the map $T_{\textrm{Mller}}:\mathbb{Z}\rightarrow\mathbb{Z}$ defined by \cite{Moller's paper (german),Matthews' slides}: \begin{equation} T_{\textrm{Mller}}\left(x\right)\overset{\textrm{def}}{=}\begin{cases} \frac{x}{d} & \textrm{if }x\overset{d}{\equiv}0\\ \frac{mx-1}{d} & \textrm{if }x\overset{d}{\equiv}\left[m\right]_{d}^{-1}\\ \frac{mx-2}{d} & \textrm{if }x\overset{d}{\equiv}2\left[m\right]_{d}^{-1}\\ \vdots & \vdots\\ \frac{mx-\left(d-1\right)}{d} & \textrm{if }x\overset{d}{\equiv}\left(d-1\right)\left[m\right]_{d}^{-1} \end{cases},\textrm{ }\forall x\in\mathbb{Z}\label{eq:Moller's map} \end{equation} Mller's analysis led him to conjecture: \begin{conjecture} $T_{\textrm{Mller}}$ eventually iterates every $x\in\mathbb{Z}$ if and only if $m<d^{d/\left(d-1\right)}$. Also, regardless of whether or not $m<d^{d/\left(d-1\right)}$, $T_{\textrm{Mller}}$ has finitely many cycles. \end{conjecture} \vphantom{} Matthews \cite{Matthews' slides} generalizes Mller's work like so. Let $d\geq2$, and let $m_{0},\ldots,m_{d-1}$ be non-zero integers (not necessarily positive). Also, for $j\in\left\{ 0,\ldots,d-1\right\} $, let $r_{j}\in\mathbb{Z}$ satisfy $r_{j}\overset{d}{\equiv}jm_{j}$. Then, define $T_{\textrm{Matthews}}:\mathbb{Z}\rightarrow\mathbb{Z}$ by: \begin{equation} T_{\textrm{Matthews}}\left(x\right)\overset{\textrm{def}}{=}\begin{cases} \frac{m_{0}x-r_{0}}{d} & \textrm{if }x\overset{d}{\equiv}0\\ \vdots & \vdots\\ \frac{m_{d-1}x-r_{d-1}}{d} & \textrm{if }x\overset{d}{\equiv}d-1 \end{cases},\textrm{ }\forall x\in\mathbb{Z}\label{eq:Matthews' Moller-map} \end{equation} He calls this map \textbf{relatively prime }whenever $\gcd\left(m_{j},d\right)=1$ for all $j\in\mathbb{Z}/d\mathbb{Z}$, and then makes the following conjectures (given in \cite{Matthews' slides}): \begin{conjecture} For the relatively prime case: \vphantom{} I. Every trajectory of $T_{\textrm{Matthews}}$ on $\mathbb{Z}$ is eventually periodic whenever: \begin{equation} \prod_{j=0}^{d-1}\left|m_{j}\right|<d^{d} \end{equation} \vphantom{} II. If: \begin{equation} \prod_{j=0}^{d-1}\left|m_{j}\right|>d^{d} \end{equation} then the union of all divergent orbit classes of $T_{\textrm{Matthews}}$ has density $1$ in $\mathbb{Z}$. \vphantom{} III. Regardless of the inequalities in (I) and (II), $T_{\textrm{Matthews}}$ always has a finite, non-zero number of cycles in $\mathbb{Z}$. \vphantom{} IV. Regardless of the inequalities in (I) and (II), for any $x\in\mathbb{Z}$ which belongs to a divergent orbit class of $T_{\textrm{Matthews}}$, the iterates of $x$ under $T$ are uniformly distributed modulo $d^{n}$ for every $n\geq1$; that is: \begin{equation} \lim_{N\rightarrow\infty}\frac{1}{N}\left|\left\{ k\in\left\{ 0,\ldots,N-1\right\} :T^{\circ k}\left(x\right)\overset{d^{n}}{\equiv}j\right\} \right|=\frac{1}{d^{n}}\label{eq:Matthews' Conjecture on uniform distribution modulo powers of rho of iterates of elements of divergent trajectories} \end{equation} \end{conjecture} \vphantom{} Even though these are conjectures, rather than results, they have the appeal of asserting a connection between the maps' parameters and the maps' dynamics. Even though probabilistic methods are not my strong suit, I am fond of approaches like these because of their open-mindedness, as well as the similarities between them and some of my currently unpublished investigations (principally ergodic theoretic) into the actions of Hydra maps on $\check{\mathbb{Z}}=\prod_{p\in\mathbb{P}}\mathbb{Z}_{p}$, the ring of profinite integers (the product is taken over all primes $p$) and its subgroups (ex: for Collatz, $\mathbb{Z}_{2}$ and $\mathbb{Z}_{2}\times\mathbb{Z}_{3}$). As Lagarias has frequently told me (or intimated) in my on-and-off correspondences with him over the past five years, any significant progress on the Collatz Conjecture should either shed light on the dynamics of close relatives like the $5x+1$ map, or give insight into why the arguments used depend on the $3$ in $3x+1$. As such, if we're going to try to surmount an insurmountable obstacle like the Collatz Conjecture, we might as well cast wide nets, in search of broader structures, avoiding any unnecessary dotage over $3x+1$ until we think there might be a way to tackle it. That, of course, is exactly what I intend to do here. \newpage{} \section{\label{sec:2.2-the-Numen}$\chi_{H}$, the Numen of a Hydra Map} THROUGHOUT THIS SECTION, WE ASSUME $p$ IS A PRIME AND THAT $H$ IS A $p$-HYDRA MAP WITH $H\left(0\right)=0$. \vphantom{} In 1978, Bhm and Sontacchi \cite{Bohm and Sontacchi} (see also \cite{Wirsching's book on 3n+1}) established what has since become a minor (and oft-rediscovered) cornerstone of Collatz literature: \begin{thm}[\textbf{Bhm-Sontacchi Criterion}\footnote{A monicker of my own invention.}] \label{thm:Bohm-Sontacchi}Let\index{Bhm-Sontacchi criterion}\index{diophantine equation} $x\in\mathbb{Z}$ be a periodic point of the Collatz map $C$ (\ref{eq:Collatz Map}). Then, there are integers $m,n\geq1$ and a strictly increasing sequence of positive integers $b_{1}<b_{2}<\cdots<b_{n}$ so that: \begin{equation} x=\frac{\sum_{k=1}^{n}2^{m-b_{k}-1}3^{k-1}}{2^{m}-3^{n}}\label{eq:The Bohm-Sontacchi Criterion} \end{equation} \end{thm} \vphantom{} (\ref{eq:The Bohm-Sontacchi Criterion}) is arguably the clearest expression of how the Collatz Conjecture reaches out of the gyre of recreational mathematics and up toward more refined spheres of study. Not only is (\ref{eq:The Bohm-Sontacchi Criterion}) an exponential diophantine equation\textemdash an area of number theory as classical as it is difficult\textemdash but is, worse yet, a \emph{family }of exponential diophantine equations. Especially noteworthy is the denominator term, $2^{m}-3^{n}$, which evidences (\ref{eq:The Bohm-Sontacchi Criterion}) as falling under the jurisdiction of of transcendental number theory\footnote{A slightly more in-depth explanation of this connection is given beginning from page \pageref{subsec:Baker,-Catalan,-and}.}, in which the study of lower bounds for quantities such as $\left|2^{m}-3^{n}\right|$ is of the utmost importance. The most significant result of the present section\textemdash and, one of this dissertation's three principal achievements\textemdash is the \textbf{Correspondence Principle }(proved in Subsection \ref{subsec:2.2.3 The-Correspondence-Principle}), which significantly generalizes the \textbf{Bhm-Sontacchi Criterion}, and in two respects. First, it shows that the integers $m$, $n$, and the $b_{k}$s can all be parameterized as functions of a single variable in $\mathbb{Z}$. Secondly, it generalizes the criterion to apply to a much larger family of Hydra maps. Not only that, but, we will be able to do the same for Hydra maps on $\mathbb{Z}^{d}$ in Chapter 5! It should be noted that a generalized form of the Criterion is given in Matthews' slides \cite{Matthews' slides}, but the version there is not as broadly minded as mine, and lacks the single-variable parameterization of the integer parameters in (\ref{eq:The Bohm-Sontacchi Criterion}) produced my $\chi_{H}$ formalism. The ideas needed to make all this work emerged from considering the $p$-ity vector associated to a given integer $n$ under the particular $p$-Hydra map $H$ we are studying. To illustrate this, let us consider the Shortened Collatz map. We will denote the two branches of $T_{3}$ ($x/2$ and $\left(3x+1\right)/2$) by $H_{0}\left(x\right)$ and $H_{1}\left(x\right)$, respectively. Letting $n\geq1$, note that as we successively apply $T_{3}$ to $n$, the iterates $T_{3}\left(n\right),T_{3}\left(T_{3}\left(n\right)\right),\ldots$ can be expressed in terms of compositions of $H_{0}$ and $H_{1}$: \begin{example} \ \begin{align*} T_{3}\left(1\right) & =2=\frac{3\left(1\right)+1}{2}=H_{1}\left(1\right)\\ T_{3}\left(T_{3}\left(1\right)\right) & =1=\frac{T_{3}\left(1\right)}{2}=H_{0}\left(T_{3}\left(1\right)\right)=H_{0}\left(H_{1}\left(1\right)\right)\\ & \vdots \end{align*} \end{example} \vphantom{} Given integers $m,n\geq1$, when we express the $m$th iterate of $n$ under $T_{3}$: \begin{equation} T_{3}^{\circ m}\left(n\right)=\underbrace{\left(T_{3}\circ T_{3}\circ\cdots\circ T_{3}\right)}_{m\textrm{ times}}\left(n\right) \end{equation} as a composition of $H_{0}$ and $H_{1}$, the sequence of the subscripts $0$ and $1$ that occur in this sequence of maps is clearly equivalent to the parity vector generated by $T_{3}$ over $n$. In this ``classical'' view of a parity vector, the parity vector is \emph{subordinated }to $n$, being obtained \emph{from }$n$. My approach is to turn this relationship on its head: what happens if we reverse it? That is, what happens if we let the \emph{parity vector} (or, more generally, $p$-ity vector) determine $n$? For $T_{3}$, we can see this reversal in action by considering the kinds of composition sequences of $H_{0}$ and $H_{1}$ that occur when we are given an integer $x$ which is a periodic point $x$ of $T_{3}$. For such an $x$, there is an integer $m\geq1$ so that $T_{3}^{\circ m}\left(x\right)=x$, which is to say, there is some length-$m$ composition sequence of $H_{0}$ and $H_{1}$ that has $x$ as a fixed point. Instead of starting with $x$ as the given, however, we treat it as an unknown to be solved for. \begin{example} Suppose $x$ is fixed by the sequence: \begin{equation} x=H_{1}\left(H_{1}\left(H_{0}\left(H_{1}\left(H_{0}\left(x\right)\right)\right)\right)\right) \end{equation} Expanding the right-hand side, we obtain: \begin{equation} x=\frac{1}{2}\left(3\left[\frac{1}{2}\left(3\left[\frac{1}{2}\left(\frac{3\left(\frac{x}{2}\right)+1}{2}\right)\right]+1\right)\right]+1\right)=\frac{3^{3}}{2^{5}}x+\frac{3^{2}}{2^{4}}+\frac{3}{2^{2}}+\frac{1}{2}\label{eq:Bohm-Sontacci Example} \end{equation} and hence: \[ x=\frac{3^{2}\cdot2+3\cdot2^{3}+2^{4}}{2^{5}-3^{3}}=\frac{58}{5} \] Since $58/5$ is not an integer, we can conclude that there is no integer fixed point of $T_{3}$ whose iterates have the parity sequence generated by the string $0,1,0,1,1$. \end{example} \vphantom{} In general, to any \emph{finite }sequence $j_{1},j_{2},j_{3},\ldots,j_{N}$ consisting of $0$s and $1$s, we can solve the equation: \begin{equation} x=\left(H_{j_{1}}\circ H_{j_{2}}\circ\cdots\circ H_{j_{N}}\right)\left(x\right)\label{eq:Periodic point set up for T3} \end{equation} for $x$, and, in doing so, obtain a map\footnote{When we generalize to consider an arbitrary $p$-Hydra map $H$ instead of $T_{3}$, we will obtain a particular $x$ for each $H$.} from the space of all such sequences to the set of rational numbers. That this works is because $H_{0}$ and $H_{1}$ are bijections of $\mathbb{Q}$. Thus, any finite composition sequence $S$ of these two branches is also a bijection, and as such, there is a unique $x\in\mathbb{Q}$ so that $x=S\left(x\right)$, which can be determined in the manner shown above. To put it another way, we can \textbf{\emph{parameterize}} the quantities $m$, $n$, $x$, and $b_{k}$ which appear in the Bhm-Sontacchi Criterion (\ref{eq:Bohm-Sontacci Example}) in terms of the sequence $j_{1},\ldots,j_{N}$ of branches of $T_{3}$ which we applied to map $x$ to itself. Viewing (\ref{eq:Bohm-Sontacci Example}) as a particular case of this formula, note that in the above example, the values of $m$, $n$, the $b_{k}$s in (\ref{eq:Bohm-Sontacci Example}), and\textemdash crucially\textemdash $x$ itself were determined by our choice of the composition sequence: \begin{equation} H_{1}\circ H_{1}\circ H_{0}\circ H_{1}\circ H_{0} \end{equation} and our assumption that this sequence mapped $x$ to $x$. In this way, we will be able to deal with all the possible variants of (\ref{eq:Bohm-Sontacci Example}) in the context of a single object\textemdash to be called $\chi_{3}$. Just as we used sequences $j_{1},\ldots,j_{N}$ in $\left\{ 0,1\right\} $ to deal with the $2$-Hydra map $T_{3}$, for the case of a general $p$-Hydra map, our sequences $j_{1},\ldots,j_{N}$ will consist of integers in the set $\left\{ 0,\ldots,p-1\right\} $, with each sequence corresponding to a particular composition sequences of the $p$ distinct branches of the $p$-Hydra map $H$ under consideration. In doing so, we will realize these expressions as specific cases of a more general functional equation satisfied the function I denote by $\chi_{H}$. We will initially define $\chi_{H}$ as a rational-valued function of finite sequences $j_{1},j_{2},\ldots$. However, by a standard bit of identifications, we will re-contextualize $\chi_{H}$ as a rational function on the non-negative integers $\mathbb{N}_{0}$. By placing certain qualitative conditions on $H$, we can ensure that $\chi_{H}$ will remain well-defined when we extend its inputs from $\mathbb{N}_{0}$ to $\mathbb{Z}_{p}$, thereby allowing us to interpolate $\chi_{H}$ to a $q$-adic valued function over $\mathbb{Z}_{p}$ for an appropriate choice of primes $p$ and $q$. \subsection{\label{subsec:2.2.1 Notation-and-Preliminary}Notation and Preliminary Definitions} Fix a $p$-Hydra map $H$, and \textbf{suppose that $b_{0}=0$} so as to guarantee that $H\left(0\right)=0$; note also that this then makes $H_{0}\left(0\right)=0$. In our study, \emph{it is imperative that }\textbf{\emph{$b_{0}=0$}}. We begin by introducing formalism\textemdash \textbf{strings}\textemdash to used to denote sequences of the numbers $\left\{ 0,\ldots,p-1\right\} $. \begin{defn}[\textbf{Strings}] We write \nomenclature{$\textrm{String}\left(p\right)$}{ }$\textrm{String}\left(p\right)$ to denote the set of all finite sequences whose entries belong to the set $\left\{ 0,\ldots,p-1\right\} $; we call such sequences \textbf{strings}. We also include the empty set ($\varnothing$) as an element of $\textrm{String}\left(p\right)$, and refer to it as the \textbf{empty string}. Arbitrary strings are usually denoted by $\mathbf{j}=\left(j_{1},\ldots,j_{n}\right)$, where $n$ is the \textbf{length }of the string, denoted by \nomenclature{$\left|\mathbf{j}\right|$}{ }$\left|\mathbf{j}\right|$. In this manner, any $\mathbf{j}$ can be written as $\mathbf{j}=\left(j_{1},\ldots,j_{\left|\mathbf{j}\right|}\right)$. We define the empty string as having length $0$, making it the unique string of length $0$. We say the string $\mathbf{j}$ is \textbf{non-zero }if $\mathbf{j}$ contains at least one non-zero entry. \vphantom{} We write \nomenclature{$\textrm{String}_{\infty}\left(p\right)$}{ }$\textrm{String}_{\infty}\left(p\right)$ to denote the set of all all sequences (finite \emph{or }infinite) whose entries belong to the set $\left\{ 0,\ldots,p-1\right\} $. A \textbf{finite }string is one with finite length; on the other hand, any string in $\textrm{String}_{\infty}\left(p\right)$ which is not finite is said to be \textbf{infinite}, and its length is defined to be $+\infty$. We also include $\varnothing$ in $\textrm{String}_{\infty}\left(p\right)$, again referred to as the empty string. \end{defn} \begin{defn} Given any $\mathbf{j}\in\textrm{String}\left(p\right)$, we define the \index{composition sequence}\textbf{composition sequence }$H_{\mathbf{j}}:\mathbb{Q}\rightarrow\mathbb{Q}$ as the affine linear map: \nomenclature{$H_{\mathbf{j}}\left(x\right)$}{$\overset{\textrm{def}}{=}\left(H_{j_{1}}\circ\cdots\circ H_{j_{\left|\mathbf{j}\right|}}\right)\left(x\right)$} \begin{equation} H_{\mathbf{j}}\left(x\right)\overset{\textrm{def}}{=}\left(H_{j_{1}}\circ\cdots\circ H_{j_{\left|\mathbf{j}\right|}}\right)\left(x\right),\textrm{ }\forall\mathbf{j}\in\textrm{String}\left(p\right)\label{eq:Def of composition sequence} \end{equation} When writing by hand, one can use vector notation: $\vec{j}$ instead of $\mathbf{j}$. \end{defn} \begin{rem} Note the near-equivalence of $\mathbf{j}$ with $p$-ity vectors. Indeed, given any $m,n\in\mathbb{N}_{1}$, there exists a unique $\mathbf{j}\in\textrm{String}\left(p\right)$ of length $m$ so that $H^{\circ m}\left(n\right)=H_{\mathbf{j}}\left(n\right)$. Said $\mathbf{j}$ is the $p$-ity vector for the first $m$ iterates of $n$ under $H$, albeit written in reverse order. While, admittedly, this reverse-order convention is somewhat unnatural, it does have one noteworthy advantage. In a moment, we will identify $\mathbf{j}$ with a $p$-adic integer $\mathfrak{z}$ by viewing the entries of $\mathbf{j}$ as the $p$-adic digits of $\mathfrak{z}$, written left-to-right in order of increasing powers of $p$. Our ``reverse'' indexing convention guarantees that the left-to-right order in which we write the $H_{j_{k}}$s, the left-to-right order in which we write the entries of $\mathbf{j}$, and the left-to-write order in which we write the $p$-adic digits of the aforementioned $\mathfrak{z}$ are all the same. \end{rem} \vphantom{} Strings will play a vital role in this chapter. Not only do they simplify many arguments, they also serve as a go-between for the dynamical-system motivations (considering arbitrary composition sequences of $H_{j}$s) and the equivalent reformulations in terms of non-negative and/or $p$-adic integers. As was mentioned above and is elaborated upon below, we will identify elements of $\textrm{String}\left(p\right)$ with the sequences of base $p$ digits of non-negative integers. Likewise, we will identify elements of $\textrm{String}_{\infty}\left(p\right)$ with the sequences of $p$-adic digits of $p$-adic integers. We do this by defining the obvious map for converting strings into ($p$-adic) integers: \begin{defn} We write \nomenclature{$\textrm{Dig}\textrm{Sum}_{p}$}{ }$\textrm{Dig}\textrm{Sum}_{p}:\textrm{String}_{\infty}\left(p\right)\rightarrow\mathbb{Z}_{p}$ by: \begin{equation} \textrm{DigSum}_{p}\left(\mathbf{j}\right)\overset{\textrm{def}}{=}\sum_{k=1}^{\left|\mathbf{j}\right|}j_{k}p^{k-1}\label{eq:Definition of DigSum_p of bold j} \end{equation} \end{defn} \begin{defn}[\textbf{Identifying Strings with Numbers}] Given $\mathbf{j}\in\textrm{String}_{\infty}\left(p\right)$ and $\mathfrak{z}\in\mathbb{Z}_{p}$, we say $\mathbf{j}$ \textbf{represents }(or \textbf{is} \textbf{associated to})\textbf{ }$\mathfrak{z}$ (and vice-versa), written $\mathbf{j}\sim\mathfrak{z}$ or $\mathfrak{z}\sim\mathbf{j}$ whenever $\mathbf{j}$ is the sequence of the $p$-adic digits of $n$; that is: \begin{equation} \mathfrak{z}\sim\mathbf{j}\Leftrightarrow\mathbf{j}\sim\mathfrak{z}\Leftrightarrow\mathfrak{z}=j_{1}+j_{2}p+j_{3}p^{2}+\cdots\label{eq:Definition of n-bold-j correspondence.} \end{equation} equivalently: \begin{equation} \mathbf{j}\sim\mathfrak{z}\Leftrightarrow\textrm{DigSum}_{p}\left(\mathbf{j}\right)=\mathfrak{z} \end{equation} As defined, $\sim$ is then an equivalence relation on $\textrm{String}\left(p\right)$ and $\textrm{String}_{\infty}\left(p\right)$. We write: \begin{equation} \mathbf{i}\sim\mathbf{j}\Leftrightarrow D_{p}\left(\mathbf{i}\right)=D_{p}\left(\mathbf{j}\right)\label{eq:Definition of string rho equivalence relation, rational integer version} \end{equation} and we have that $\mathbf{i}\sim\mathbf{j}$ if and only if both $\mathbf{i}$ and $\mathbf{j}$ represent the same $p$-adic integer. Note that in both $\textrm{String}\left(p\right)$ and $\textrm{String}_{\infty}\left(p\right)$, the shortest string representing the number $0$ is then the empty string. Finally, we write \nomenclature{$\textrm{String}\left(p\right)/\sim$}{ }$\textrm{String}\left(p\right)/\sim$ and \nomenclature{$\textrm{String}_{\infty}\left(p\right)/\sim$}{ }$\textrm{String}_{\infty}\left(p\right)/\sim$ to denote the set of equivalence classes of $\textrm{String}\left(p\right)$ and $\textrm{String}_{\infty}\left(p\right)$ under this equivalence relation. \end{defn} \begin{prop} \label{prop:string number equivalence}\ \vphantom{} I. $\textrm{String}\left(p\right)/\sim$ and $\textrm{String}_{\infty}\left(p\right)/\sim$ are in bijective correspondences with $\mathbb{N}_{0}$ and $\mathbb{Z}_{p}$, respectively, by way of the map $\textrm{Dig}\textrm{Sum}_{p}$. \vphantom{} II. Two finite strings $\mathbf{i}$ and $\mathbf{j}$ satisfy $\mathbf{i}\sim\mathbf{j}$ if and only if either: \vphantom{} a) $\mathbf{i}$ can be obtained from $\mathbf{j}$ by adding finitely many $0$s to the right of $\mathbf{j}$. \vphantom{} b) $\mathbf{j}$ can be obtained from $\mathbf{i}$ by adding finitely many $0$s to the right of $\mathbf{i}$. \vphantom{} III. A finite string $\mathbf{i}$ and an infinite string $\mathbf{j}$ satisfy $\mathbf{i}\sim\mathbf{j}$ if and only if $\mathbf{j}$ contains finitely many non-zero entries. \end{prop} Proof: Immediate from the definitions. Q.E.D. \vphantom{} The principal utility of this formalism is the way in which composition sequences $H_{\mathbf{j}}$ interact with concatenations of strings. \begin{defn}[\textbf{Concatenation}] We introduce the \textbf{concatenation operation}\index{concatenation!operation} $\wedge:\textrm{String}\left(p\right)\times\textrm{String}_{\infty}\left(p\right)\rightarrow\textrm{String}_{\infty}\left(p\right)$, defined by: \begin{equation} \mathbf{i}\wedge\mathbf{j}=\left(i_{1},\ldots,i_{\left|\mathbf{i}\right|}\right)\wedge\left(j_{1},\ldots,j_{\left|\mathbf{j}\right|}\right)\overset{\textrm{def}}{=}\left(i_{1},\ldots,i_{\left|\mathbf{i}\right|},j_{1},\ldots,j_{\left|\mathbf{j}\right|}\right)\label{eq:Definition of Concatenation} \end{equation} with the definition being modified in the obvious way to when $\mathbf{j}$ is of infinite length. \vphantom{} Additionally, for any integer $m\geq1$ and any finite string $\mathbf{j}$, we write $\mathbf{j}^{\wedge m}$ to denote the concatenation of $m$ copies of $\mathbf{j}$: \begin{equation} \mathbf{j}^{\wedge m}\overset{\textrm{def}}{=}\left(\underbrace{j_{1},\ldots,j_{\left|\mathbf{j}\right|},j_{1},\ldots,j_{\left|\mathbf{j}\right|},\ldots,j_{1},\ldots,j_{\left|\mathbf{j}\right|}}_{m\textrm{ times}}\right)\label{eq:Definition of concatenation exponentiation} \end{equation} \end{defn} \vphantom{} In the next subsection, the proposition below will be the foundation for various useful functional equations: \begin{prop} \label{prop:H string formalism}\ \begin{equation} H_{\mathbf{i}\wedge\mathbf{j}}\left(x\right)=H_{\mathbf{i}}\left(H_{\mathbf{j}}\left(x\right)\right),\textrm{ }\forall x\in\mathbb{R},\textrm{ }\forall\mathbf{i},\mathbf{j}\in\textrm{String}\left(p\right)\label{eq:H string formalism} \end{equation} \end{prop} Proof: \begin{equation} H_{\mathbf{i}\wedge\mathbf{j}}\left(x\right)=H_{i_{1},\ldots,i_{\left|\mathbf{i}\right|},j_{1},\ldots,j_{\left|\mathbf{j}\right|}}\left(x\right)=\left(H_{i_{1},\ldots,i_{\left|\mathbf{i}\right|}}\circ H_{j_{1},\ldots,j_{\left|\mathbf{j}\right|}}\right)\left(x\right)=H_{\mathbf{i}}\left(H_{\mathbf{j}}\left(x\right)\right) \end{equation} Q.E.D. \vphantom{} In working with strings and the ($p$-adic) integers they represent, the following functions will be useful to help bridge string-world and number-world. \begin{defn}[$\lambda_{p}$ \textbf{and} $\#_{p:0},\#_{p:1},\ldots,\#_{p:p-1}$] We write $\lambda_{p}:\mathbb{N}_{0}\rightarrow\mathbb{N}_{0}$ to denote the function:\nomenclature{$\lambda_{p}\left(n\right)$}{$\overset{\textrm{def}}{=}\left\lceil \log_{p}\left(n+1\right)\right\rceil$ \nopageref } \begin{equation} \lambda_{p}\left(n\right)\overset{\textrm{def}}{=}\left\lceil \log_{p}\left(n+1\right)\right\rceil ,\textrm{ }\forall n\in\mathbb{N}_{0}\label{eq:definition of lambda rho} \end{equation} which gives the number of $p$-adic digits of $n$. Every $n\in\mathbb{N}_{0}$ can be uniquely written as: \begin{equation} n=c_{0}+c_{1}p+\cdots+c_{\lambda_{p}\left(n\right)-1}p^{\lambda_{p}\left(n\right)-1} \end{equation} where the $c_{j}$s are integers in $\left\{ 0,\ldots,p-1\right\} $. Additionally, note that: \begin{equation} \left\lceil \log_{p}\left(n+1\right)\right\rceil =\left\lfloor \log_{p}n\right\rfloor +1,\textrm{ }\forall n\in\mathbb{N}_{1}\label{eq:Floor and Ceiling expressions for lambda_rho} \end{equation} and that $\lambda_{p}\left(n\right)\leq\left|\mathbf{j}\right|$ is satisfied for any string $\mathbf{j}$ representing $n$. Next, for each $n\geq1$ and each $k\in\left\{ 0,\ldots,p-1\right\} $, we write $\#_{p:k}\left(n\right)$\nomenclature{$\#_{p:k}\left(n\right)$}{the number of $k$s in the $p$-adic digits of $n$ } to denote the number of $k$s present in the $p$-adic expansion of $n$. We define $\#_{p:k}\left(0\right)$ to be $0$ for all $k$. In a minor abuse of notation, we also use $\#_{p:k}$ to denote the number of $k$s which occur in a given string: \begin{equation} \#_{p:k}\left(\mathbf{j}\right)\overset{\textrm{def}}{=}\textrm{number of }k\textrm{s in }\mathbf{j},\textrm{ }\forall k\in\mathbb{Z}/p\mathbb{Z},\textrm{ }\forall\mathbf{j}\in\textrm{String}\left(p\right)\label{eq:Definition of number of ks in rho adic digits of bold j} \end{equation} \end{defn} \begin{rem} $\lambda_{2}\left(n\right)$ is what computer scientists called the number of \textbf{bits }in $n$. \end{rem} \begin{example} For: \begin{align*} n & =1\cdot6^{0}+2\cdot6^{1}+0\cdot6^{2}+5\cdot6^{3}+0\cdot6^{4}+2\cdot6^{5}\\ & =1+2\cdot6+5\cdot6^{3}+2\cdot6^{5}\\ & =16645 \end{align*} we have: \begin{align*} \#_{6:0}\left(16645\right) & =2\\ \#_{6:1}\left(16645\right) & =1\\ \#_{6:2}\left(16645\right) & =2\\ \#_{6:3}\left(16645\right) & =0\\ \#_{6:4}\left(16645\right) & =0\\ \#_{6:5}\left(16645\right) & =1 \end{align*} \end{example} \vphantom{} All important functions in this dissertation satisfy equally important functional equations. Here are the functional equations for $\lambda_{p}$ and the $\#_{p:k}$s. \begin{prop}[\textbf{Functional Equations for $\lambda_{p}$ and the $\#_{p:k}$s}] \ \vphantom{} I. For $k\in\left\{ 1,\ldots,p-1\right\} $: \begin{align} \#_{p:k}\left(p^{n}a+b\right) & =\#_{p:k}\left(a\right)+\#_{p:k}\left(b\right),\textrm{ }\forall a\in\mathbb{N}_{0},\textrm{ }\forall n\in\mathbb{N}_{1},\textrm{ }\forall b\in\left\{ 0,\ldots,p^{n}-1\right\} \label{eq:number-symbol functional equations} \end{align} For $k=0$, we have: \begin{equation} \#_{p:0}\left(p^{n}a+b\right)=\#_{p:0}\left(a\right)+\#_{p:0}\left(b\right)+n-\lambda_{p}\left(b\right),\textrm{ }\forall a\in\mathbb{N}_{1},\textrm{ }\forall n\in\mathbb{N}_{1},\textrm{ }\forall b\in\left\{ 0,\ldots,p^{n}-1\right\} \label{eq:number-symbol functional equations, k is 0} \end{equation} \vphantom{} II. \begin{align} \lambda_{p}\left(p^{n}a+b\right) & =\lambda_{p}\left(a\right)+n,\textrm{ }\forall a\in\mathbb{N}_{1},\textrm{ }\forall n\in\mathbb{N}_{1},\textrm{ }\forall b\in\left\{ 0,\ldots,p^{n}-1\right\} \label{eq:lambda functional equations} \end{align} \vphantom{} III. \begin{equation} \sum_{k=0}^{p-1}\#_{p:k}\left(n\right)=\lambda_{p}\left(n\right)\label{eq:Relation between lambda_rho and the number of rho adic digits} \end{equation} \end{prop} Proof: A straightforward computation. (\ref{eq:number-symbol functional equations, k is 0}) follows from using (\ref{eq:Relation between lambda_rho and the number of rho adic digits}) to write: \[ \#_{p:0}\left(n\right)=\lambda_{p}\left(n\right)-\sum_{k=1}^{p-1}\#_{p:k}\left(n\right) \] and then applying (\ref{eq:number-symbol functional equations}) and (\ref{eq:lambda functional equations}) to compute $\#_{p:0}\left(p^{n}a+b\right)$. Q.E.D. \subsection{\label{subsec:2.2.2 Construction-of}Construction of $\chi_{H}$} Since the $H_{j}$s are affine linear maps, so is any composition sequence $H_{\mathbf{j}}$. As such, for any $\mathbf{j}\in\textrm{String}\left(p\right)$ we can write: \begin{equation} H_{\mathbf{j}}\left(x\right)=H_{\mathbf{j}}^{\prime}\left(0\right)x+H_{\mathbf{j}}\left(0\right),\textrm{ }\forall\mathbf{j}\in\textrm{String}\left(p\right)\label{eq:ax+b formula for h_bold_j} \end{equation} If $\mathbf{j}$ is a string for which $H_{\mathbf{j}}\left(x\right)=x$, this becomes: \begin{equation} x=H_{\mathbf{j}}^{\prime}\left(0\right)x+H_{\mathbf{j}}\left(0\right)\label{eq:affine formula for little x} \end{equation} from which we obtain: \begin{equation} x=\frac{H_{\mathbf{j}}\left(0\right)}{1-H_{\mathbf{j}}^{\prime}\left(0\right)}\label{eq:Formula for little x in terms of bold j} \end{equation} Note that the map which sends a string $\mathbf{j}$ to the rational number $x$ satisfying $H_{\mathbf{j}}\left(x\right)=x$ can then be written as: \begin{equation} \mathbf{j}\mapsto\frac{H_{\mathbf{j}}\left(0\right)}{1-H_{\mathbf{j}}^{\prime}\left(0\right)}\label{eq:Formula for X} \end{equation} While we could analyze this map directly, the resultant function is ill-behaved because of the denominator term. Consequently, instead of studying (\ref{eq:Formula for X}) directly, we will focus on the map $\mathbf{j}\mapsto H_{\mathbf{j}}\left(0\right)$, which we denote by $\chi_{H}$. \begin{defn}[$\chi_{H}$] \label{def:Chi_H on N_0 in strings}Let $H$ be a $p$-Hydra map that fixes $0$. Then, for any $\mathbf{j}\in\textrm{String}\left(p\right)$, we write: \begin{equation} \chi_{H}\left(\mathbf{j}\right)\overset{\textrm{def}}{=}H_{\mathbf{j}}\left(0\right)\label{eq:Definition of Chi_H of bold j} \end{equation} Identifying $H_{\varnothing}$ (the composition sequence associated to the empty string) with the identity map, we have that $\chi_{H}\left(\varnothing\right)=0$. \end{defn} \begin{prop} \label{prop:Chi_H is well defined mod twiddle}$\chi_{H}$ is well-defined on $\textrm{String}\left(p\right)/\sim$. That is, $\chi_{H}\left(\mathbf{i}\right)=\chi_{H}\left(\mathbf{j}\right)$ for all $\mathbf{i},\mathbf{j}\in\textrm{String}\left(p\right)$ for which $\mathbf{i}\sim\mathbf{j}$. \end{prop} Proof: Let $\mathbf{j}\in\textrm{String}\left(p\right)$ be any non-empty string. Then, by \textbf{Proposition \ref{prop:H string formalism}}: \begin{equation} \chi_{H}\left(\mathbf{j}\wedge0\right)=H_{\mathbf{j}\wedge0}\left(0\right)=H_{\mathbf{j}}\left(H_{0}\left(0\right)\right)=H_{\mathbf{j}}\left(0\right)=\chi_{H}\left(\mathbf{j}\right) \end{equation} since $H_{0}\left(0\right)=0$. \textbf{Proposition \ref{prop:string number equivalence}}, we know that given any two equivalent finite strings $\mathbf{i}\sim\mathbf{j}$, we can obtain one of the two by concatenating finitely many $0$s to the right of the other. Without loss of generality, suppose that $\mathbf{i}=\mathbf{j}\wedge0^{\wedge n}$, where $0^{\wedge n}$ is a string of $n$ consecutive $0$s. Then: \begin{equation} \chi_{H}\left(\mathbf{i}\right)=\chi_{H}\left(\mathbf{j}\wedge0^{\wedge n}\right)=\chi_{H}\left(\mathbf{j}\wedge0^{\wedge\left(n-1\right)}\right)=\cdots=\chi_{H}\left(\mathbf{j}\right) \end{equation} Hence, $\chi_{H}$ is well defined on $\textrm{String}\left(p\right)/\sim$. Q.E.D. \begin{lem}[$\chi_{H}$ \textbf{on} $\mathbb{N}_{0}$] \index{chi_{H}@$\chi_{H}$}\label{lem:Chi_H on N_0}We can realize $\chi_{H}$ as a function $\mathbb{N}_{0}\rightarrow\mathbb{Q}$ by defining: \begin{equation} \chi_{H}\left(n\right)\overset{\textrm{def}}{=}\chi_{H}\left(\mathbf{j}\right)=H_{\mathbf{j}}\left(0\right)\label{eq:Definition of Chi_H of n} \end{equation} where $\mathbf{j}\in\textrm{String}\left(p\right)$ is any string representing $n$; we define $\chi_{H}\left(0\right)$ to be $\chi_{H}\left(\varnothing\right)$ (the empty string), which is just $0$. \end{lem} Proof: By \textbf{Proposition \ref{prop:string number equivalence}}, $\textrm{String}\left(p\right)/\sim$ is in a bijection with $\mathbb{N}_{0}$. \textbf{Proposition \ref{prop:Chi_H is well defined mod twiddle} }tells us that $\chi_{H}$ is well-defined on $\textrm{String}\left(p\right)/\sim$, so, by using the aforementioned bijection to identify $\mathbb{N}_{0}$ with $\textrm{String}\left(p\right)/\sim$, the rule $\chi_{H}\left(n\right)\overset{\textrm{def}}{=}\chi_{H}\left(\mathbf{j}\right)$ is well-defined, since it is independent of which $\mathbf{j}\in\textrm{String}\left(p\right)$ we choose to represent $n$. Q.E.D. \begin{defn}[$M_{H}$] \index{$M_{H}$}\label{def:M_H on N_0 in strings}Let $H$ be a $p$-Hydra map. Then, we define $M_{H}:\mathbb{N}_{0}\rightarrow\mathbb{Q}$ by: \begin{equation} M_{H}\left(n\right)\overset{\textrm{def}}{=}M_{H}\left(\mathbf{j}\right)\overset{\textrm{def}}{=}H_{\mathbf{j}}^{\prime}\left(0\right),\textrm{ }\forall n\geq1\label{eq:Definition of M_H} \end{equation} where $\mathbf{j}\in\textrm{String}\left(p\right)$ is the shortest element of $\textrm{String}\left(p\right)$ representing $n$. We define $M_{H}\left(0\right)$ to be $1$. \end{defn} \begin{example} \emph{Unlike} $\chi_{H}$, observe that $M_{H}$ is \emph{not well-defined over} $\textrm{String}\left(p\right)/\sim$ there can be $H$ and distinct strings $\mathbf{i},\mathbf{j}\in\textrm{String}\left(p\right)$ so that $\mathbf{i}\sim\mathbf{j}$, yet $M_{H}\left(\mathbf{i}\right)\neq M_{H}\left(\mathbf{j}\right)$. For example, $H=T_{3}$, the strings $\left(0,1\right)$ and $\left(0,1,0\right)$ both represent the integer $2$. Nevertheless: \begin{align*} H_{0,1}\left(x\right) & =H_{0}\left(H_{1}\left(x\right)\right)=\frac{3}{4}x+\frac{1}{4}\\ H_{0,1,0}\left(x\right) & =H_{0}\left(H_{1}\left(H_{0}\left(x\right)\right)\right)=\frac{3}{8}x+\frac{1}{4} \end{align*} Thus, $\left(0,1\right)\sim\left(0,1,0\right)$, but $M_{T_{3}}\left(0,1\right)=3/4$, while $M_{T_{3}}\left(0,1,0\right)=3/8$. With these definitions in place, we can now use the string formalism to establish vital string-based functional equation identities for $\chi_{H}$ and $M_{H}$. I will refer to these identities as \textbf{concatenation identities}\index{concatenation!identities}. \end{example} \begin{prop}[\textbf{An expression for $H_{\mathbf{j}}$}] \label{prop:H_boldj in terms of M_H and Chi_H}Let $H$ be a $p$-Hydra map. Then: \begin{equation} H_{\mathbf{j}}\left(x\right)=M_{H}\left(\mathbf{j}\right)x+\chi_{H}\left(\mathbf{j}\right),\textrm{ }\forall\mathbf{j}\in\textrm{String}\left(p\right),\textrm{ }\forall x\in\mathbb{Q}\label{eq:Formula for composition sequences of H} \end{equation} \end{prop} Proof: Since the $H_{j}$s are maps of the form $ax+b$, any composition sequence of the $H_{j}$s will also be of that form. Consequently, the venerable slope-intercept formula yields: \[ H_{\mathbf{j}}\left(x\right)=H_{\mathbf{j}}^{\prime}\left(0\right)x+H_{\mathbf{j}}\left(0\right)\overset{\textrm{def}}{=}M_{H}\left(\mathbf{j}\right)x+\chi_{H}\left(\mathbf{j}\right) \] Q.E.D. \vphantom{} The lemmata below give the concatenation identities for $\chi_{H}$ and $M_{H}$. \begin{lem}[$\chi_{H}$ \textbf{Concatenation Identity}] \label{lem:Chi_H concatenation identity}\ \begin{equation} \chi_{H}\left(\mathbf{i}\wedge\mathbf{j}\right)=H_{\mathbf{i}}\left(\chi_{H}\left(\mathbf{j}\right)\right),\textrm{ }\forall\mathbf{i},\mathbf{j}\in\textrm{String}\left(p\right)\label{eq:Chi_H concatenation identity} \end{equation} That is to say, for $\mathbf{i}=\left(i_{1},i_{2},\ldots,i_{m}\right)\in\textrm{String}\left(p\right)$, where $m\geq1$, we have: \begin{equation} \chi_{H}\left(i_{1}+i_{2}p+\cdots+i_{m}p^{m-1}+p^{m}n\right)=H_{\mathbf{i}}\left(\chi_{H}\left(n\right)\right),\textrm{ }\forall n\in\mathbb{N}_{0}\label{eq:Chi_H concatenation identity, alternate version} \end{equation} \end{lem} Proof: Letting $\mathbf{i}$, $\mathbf{j}$, and $x$ be arbitrary, we have that: \begin{align*} H_{\mathbf{i}\wedge\mathbf{j}}\left(x\right) & =H_{\mathbf{i}\wedge\mathbf{j}}^{\prime}\left(0\right)x+H_{\mathbf{i}\wedge\mathbf{j}}\left(0\right)\\ & =H_{\mathbf{i}\wedge\mathbf{j}}^{\prime}\left(0\right)x+H_{\mathbf{i}}\left(H_{\mathbf{j}}\left(0\right)\right) \end{align*} Setting $x=0$ yields: \[ \underbrace{H_{\mathbf{i}\wedge\mathbf{j}}\left(0\right)}_{\chi_{H}\left(\mathbf{i}\wedge\mathbf{j}\right)}=\underbrace{H_{\mathbf{i}}\left(H_{\mathbf{j}}\left(0\right)\right)}_{H_{\mathbf{j}}\left(\chi_{H}\left(\mathbf{j}\right)\right)} \] Q.E.D. \begin{prop}[$M_{H}$ \textbf{Concatenation Identity}] \label{prop:M_H concatenation identity}\ \begin{equation} M_{H}\left(\mathbf{i}\wedge\mathbf{j}\right)=M_{H}\left(\mathbf{i}\right)M_{H}\left(\mathbf{j}\right),\textrm{ }\forall\mathbf{i},\mathbf{j}\in\textrm{String}\left(p\right),\textrm{ }\left|\mathbf{i}\right|,\left|\mathbf{j}\right|\geq1\label{eq:M_H concatenation identity} \end{equation} That is to say, for $\mathbf{i}=\left(i_{1},i_{2},\ldots,i_{m}\right)\in\textrm{String}\left(p\right)$, where $m\geq1$, we have: \begin{equation} M_{H}\left(i_{1}+i_{2}p+\cdots+i_{m}p^{m-1}+p^{m}n\right)=M_{H}\left(\mathbf{i}\right)M_{H}\left(n\right),\textrm{ }\forall n\in\mathbb{N}_{0}\label{eq:Inductive identity for M_H} \end{equation} \end{prop} Proof: We have: \begin{equation} H_{\mathbf{i}\wedge\mathbf{j}}\left(x\right)=H_{\mathbf{i}}\left(H_{\mathbf{j}}\left(x\right)\right) \end{equation} Differentiating both sides with respect to $x$ and applying the Chain rule gives: \[ H_{\mathbf{i}\wedge\mathbf{j}}^{\prime}\left(x\right)=H_{\mathbf{i}}^{\prime}\left(H_{\mathbf{j}}\left(x\right)\right)H_{\mathbf{j}}^{\prime}\left(x\right) \] Now, set $x=0$: \begin{equation} \underbrace{H_{\mathbf{i}\wedge\mathbf{j}}^{\prime}\left(0\right)}_{M_{H}\left(\mathbf{i}\wedge\mathbf{j}\right)}=H_{\mathbf{i}}^{\prime}\left(H_{\mathbf{j}}\left(0\right)\right)\underbrace{H_{\mathbf{j}}^{\prime}\left(0\right)}_{M_{H}\left(\mathbf{j}\right)} \end{equation} Since $H_{\mathbf{i}}\left(x\right)$ is an affine linear map of the form $H_{\mathbf{i}}^{\prime}\left(0\right)x+H_{\mathbf{i}}\left(0\right)$, its derivative is the constant function $H_{\mathbf{i}}^{\prime}\left(y\right)=H_{\mathbf{i}}^{\prime}\left(0\right)$. Thus: \begin{equation} M_{H}\left(\mathbf{i}\wedge\mathbf{j}\right)=H_{\mathbf{i}}^{\prime}\left(H_{\mathbf{j}}\left(0\right)\right)M_{H}\left(\mathbf{j}\right)=H_{\mathbf{i}}^{\prime}\left(0\right)M_{H}\left(\mathbf{j}\right)=M_{H}\left(\mathbf{i}\right)M_{H}\left(\mathbf{j}\right) \end{equation} Q.E.D. \vphantom{} We build a bridge between string-world and number-world by restating the concatenation identities in terms of systems of functional equations involving integer inputs. \begin{prop}[\textbf{Functional Equations for} $M_{H}$] \label{prop:M_H functional equation}\index{functional equation!$M_{H}$}\ \end{prop} \begin{equation} M_{H}\left(pn+j\right)=\frac{\mu_{j}}{p}M_{H}\left(n\right),\textrm{ }\forall n\geq0\textrm{ \& }\forall j\in\mathbb{Z}/p\mathbb{Z}:pn+j\neq0\label{eq:M_H functional equations} \end{equation} \begin{rem} To be clear, this functional equation \emph{does not hold} when $pn+j=0$. \end{rem} Proof: Let $n\geq1$, and let $\mathbf{j}=\left(j_{1},\ldots,j_{m}\right)$ be the shortest string in $\textrm{String}\left(p\right)$ which represents $n$. Then, observe that for $\mathbf{k}=\left(j,j_{1},j_{2},\ldots,j_{m}\right)$, where $j\in\mathbb{Z}/p\mathbb{Z}$ is arbitrary, we have that: \[ \mathbf{k}=j\wedge\mathbf{j}\sim\left(j+pn\right) \] \textbf{Proposition \ref{prop:M_H concatenation identity}} lets us write: \[ M_{H}\left(pn+j\right)=M_{H}\left(j\wedge\mathbf{j}\right)=M_{H}\left(j\right)M_{H}\left(\mathbf{j}\right)=\frac{\mu_{j}}{p}M_{H}\left(n\right) \] When $n=0$, these equalities hold for all $j\in\left\{ 1,\ldots,p-1\right\} $, seeing as: \[ \left(j+p\cdot0\right)\sim j\wedge\varnothing \] As for the one exceptional case\textemdash $n=j=0$\textemdash note that we obtain: \[ M_{H}\left(p\cdot0+0\right)=M_{H}\left(0\right)\overset{\textrm{def}}{=}1 \] but: \[ \frac{\mu_{0}}{p}M_{H}\left(0\right)\overset{\textrm{def}}{=}\frac{\mu_{0}}{p}\times1=\frac{\mu_{0}}{p} \] and, as we saw, $\mu_{0}/p\neq1$. Q.E.D. \vphantom{} Using the concatenation identity for $\chi_{H}$, we can establish the first of two characterizations for it in terms of functional equations. \begin{lem}[\textbf{Functional Equations for} $\chi_{H}$] \label{lem:Chi_H functional equation on N_0 and uniqueness}$\chi_{H}$ is the unique rational-valued function on $\mathbb{N}_{0}$ satisfying the system of functional equations:\index{chi{H}@$\chi_{H}$!functional equation}\index{functional equation!chi_{H}@$\chi_{H}$} \begin{equation} \chi_{H}\left(pn+j\right)=\frac{a_{j}\chi_{H}\left(n\right)+b_{j}}{d_{j}},\textrm{ }\forall n\in\mathbb{N}_{0},\textrm{ }\forall\mathbb{Z}/p\mathbb{Z}\label{eq:Chi_H functional equations} \end{equation} \end{lem} \begin{rem} These functional equations can be written more compactly as: \begin{equation} \chi_{H}\left(pn+j\right)=H_{j}\left(\chi_{H}\left(n\right)\right),\textrm{ }\forall n\in\mathbb{N}_{0},\textrm{ }\forall j\in\mathbb{Z}/p\mathbb{Z}\label{eq:Alternate statement of Chi_H functional equations} \end{equation} \end{rem} Proof: I. Let $\mathbf{i}\sim n$ and let $j\in\mathbb{Z}/p\mathbb{Z}$ be arbitrary. Then $pn+j\sim j\wedge\mathbf{i}$, and hence, by $\chi_{H}$'s concatenation identity (\textbf{Lemma \ref{lem:Chi_H concatenation identity}}): \begin{equation} \chi_{H}\left(pn+j\right)=\chi_{H}\left(j\wedge\mathbf{i}\right)=H_{j}\left(\chi_{H}\left(\mathbf{i}\right)\right)=H_{j}\left(\chi_{H}\left(n\right)\right) \end{equation} Thus, $\chi_{H}$ is a solution of the given system of functional equations. The reason why we need not exclude the case where $n=j=0$ (as we had to do with $M_{H}$) is because $H_{0}\left(\chi_{H}\left(0\right)\right)=H_{0}\left(0\right)=0$. \vphantom{} II. On the other hand, let $f:\mathbb{N}_{0}\rightarrow\mathbb{Q}$ be any function so that: \begin{equation} f\left(pn+j\right)=H_{j}\left(f\left(n\right)\right),\textrm{ }\forall n\in\mathbb{N}_{0},\textrm{ }\forall j\in\mathbb{Z}/p\mathbb{Z} \end{equation} Setting $n=j=0$ gives: \begin{align*} f\left(0\right) & =H_{0}\left(f\left(0\right)\right)\\ \left(H_{0}\left(x\right)=H_{0}^{\prime}\left(0\right)x+\underbrace{H_{0}\left(0\right)}_{0}\right); & =H_{0}^{\prime}\left(0\right)f\left(0\right) \end{align*} Since $H$ is a $p$-Hydra map: \[ H_{0}^{\prime}\left(0\right)=\frac{\mu_{0}}{p}\notin\left\{ 0,1\right\} \] Thus, $f\left(0\right)=H_{0}^{\prime}\left(0\right)f\left(0\right)$ forces $f\left(0\right)=0$. Plugging in $n=0$ then leaves us with: \[ f\left(j\right)=H_{j}\left(f\left(0\right)\right)=H_{j}\left(0\right),\textrm{ }\forall j\in\mathbb{Z}/p\mathbb{Z} \] Writing $n$ $p$-adically as: \begin{equation} n=j_{1}+j_{2}p+\cdots+j_{L}p^{L-1} \end{equation} the identity $f\left(pn+j\right)=H_{j}\left(f\left(n\right)\right)$ is then equivalent to: \begin{align*} f\left(j+j_{1}p+j_{2}p^{2}+\cdots+j_{L}p^{L}\right) & =H_{j}\left(f\left(j_{1}+j_{2}p+\cdots+j_{L}p^{L-1}\right)\right)\\ & =H_{j}\left(H_{j_{1}}\left(f\left(j_{2}+j_{3}p+\cdots+j_{L}p^{L-2}\right)\right)\right)\\ & \vdots\\ & =\left(H_{j}\circ H_{j_{1}}\circ\cdots\circ H_{j_{L}}\right)\left(f\left(0\right)\right)\\ & =\left(H_{j}\circ H_{j_{1}}\circ\cdots\circ H_{j_{L}}\right)\left(0\right)\\ & =H_{j,j_{1},\ldots,j_{L}}\left(0\right) \end{align*} So, for any string $\mathbf{j}^{\prime}=\left(j,j_{1},\ldots,j_{L}\right)$, we have: \[ f\left(\mathbf{j}^{\prime}\right)=H_{\mathbf{j}^{\prime}}\left(0\right)\overset{\textrm{def}}{=}\chi_{H}\left(\mathbf{j}^{\prime}\right) \] where, note: $\mathbf{j}^{\prime}=j\wedge\mathbf{j}$ and $\mathbf{j}^{\prime}\sim n$. In other words, if $f$ solves the given system of functional equations, it is in fact equal to $\chi_{H}$ at every finite string, and hence, at every non-negative integer. This shows the system's solutions are unique. Q.E.D. \vphantom{} Next we compute explicit formulae for $M_{H}$ and $\chi_{H}$ in terms of the numbers present in $H$. These will be needed in to make the $q$-adic estimates needed to establish the extension/interpolation of $\chi_{H}$ from a function $\mathbb{N}_{0}\rightarrow\mathbb{Q}$ to a function $\mathbb{Z}_{p}\rightarrow\mathbb{Z}_{q}$, for an appropriate choice of a prime $q$. \begin{prop} \label{prop:Explicit Formulas for M_H}$M_{H}\left(n\right)$ and $M_{H}\left(\mathbf{j}\right)$ can be explicitly given in terms of the constants associated to $H$ by the formulae: \begin{equation} M_{H}\left(n\right)=\frac{1}{p^{\lambda_{p}\left(n\right)}}\prod_{k=0}^{p-1}\mu_{k}^{\#_{p:k}\left(n\right)}=\prod_{k=0}^{p-1}\frac{a_{k}^{\#_{p:k}\left(n\right)}}{d_{k}^{\#_{p:k}\left(n\right)}}\label{eq:Formula for M_H of n} \end{equation} and: \begin{equation} M_{H}\left(\mathbf{j}\right)=\prod_{k=1}^{\left|\mathbf{j}\right|}\frac{a_{j_{k}}}{d_{j_{k}}}=\frac{1}{p^{\left|\mathbf{j}\right|}}\prod_{k=1}^{\left|\mathbf{j}\right|}\mu_{j_{k}}=\frac{1}{p^{\left|\mathbf{j}\right|}}\prod_{k=0}^{p-1}\mu_{k}^{\#_{p:k}\left(\mathbf{j}\right)}=\prod_{k=0}^{p-1}\frac{a_{k}^{\#_{p:k}\left(\mathbf{j}\right)}}{d_{k}^{\#_{p:k}\left(\mathbf{j}\right)}}\label{eq:formula for M_H of bold-j} \end{equation} respectively. We adopt the convention that the $k$-product in \emph{(\ref{eq:formula for M_H of bold-j})} is defined to be $1$ when $\left|\mathbf{j}\right|=0$. Finally, for any $\mathfrak{z}\in\mathbb{Z}_{p}$ and any $N\geq1$, we have the formula: \begin{equation} M_{H}\left(\left[\mathfrak{z}\right]_{p^{N}}\right)=\frac{\prod_{j=0}^{p-1}\mu_{j}^{\#_{p:j}\left(\left[\mathfrak{z}\right]_{p^{N}}\right)}}{p^{\lambda_{p}\left(\left[\mathfrak{z}\right]_{p^{N}}\right)}}\label{eq:M_H of z mod rho to the N} \end{equation} obtained by setting $n=\left[\mathfrak{z}\right]_{p^{N}}$ in \emph{(\ref{eq:Formula for M_H of n})}. \end{prop} Proof: The proof is purely computational. I omit it, seeing as it occurs in the proof of \textbf{Proposition \ref{prop:Explicit formula for Chi_H of bold j}}, given below. Q.E.D. \begin{prop} \label{prop:Explicit formula for Chi_H of bold j} \begin{equation} \chi_{H}\left(\mathbf{j}\right)=\sum_{m=1}^{\left|\mathbf{j}\right|}\frac{b_{j_{m}}}{d_{j_{m}}}\prod_{k=1}^{m-1}\frac{a_{j_{k}}}{d_{j_{k}}}=\sum_{m=1}^{\left|\mathbf{j}\right|}\frac{b_{j_{m}}}{a_{j_{m}}}\left(\prod_{k=1}^{m}\mu_{j_{k}}\right)p^{-m},\textrm{ }\forall\mathbf{j}\in\textrm{String}\left(p\right)\label{eq:Formula for Chi_H in terms of bold-j} \end{equation} where the $k$-product is defined to be $1$ when $m=1$. \end{prop} Proof: Let $\mathbf{j}=\left(j_{1},\ldots,j_{\left|\mathbf{j}\right|}\right)\in\textrm{String}\left(p\right)$ be arbitrary. Since $\chi_{H}\left(\mathbf{j}\right)=\chi_{H}\left(\mathbf{i}\right)$ for any $\mathbf{i}\in\textrm{String}\left(p\right)$ for which $\mathbf{i}\sim\mathbf{j}$, we can assume without loss of generality that $\mathbf{j}$'s right-most entry is non-zero; that is: $j_{\left|\mathbf{j}\right|}\neq0$. The proof follows by examining: \begin{equation} H_{\mathbf{j}}\left(x\right)=M_{H}\left(\mathbf{j}\right)x+\chi_{H}\left(\mathbf{j}\right),\textrm{ }\forall x\in\mathbb{R} \end{equation} and carefully writing out the composition sequence in full: \begin{align*} H_{\mathbf{j}}\left(x\right) & =H_{\left(j_{1},\ldots,j_{\left|\mathbf{j}\right|}\right)}\left(x\right)\\ & =\frac{a_{j_{1}}\frac{a_{j_{2}}\left(\cdots\right)+b_{j_{2}}}{d_{j_{2}}}+b_{j_{1}}}{d_{j_{1}}}\\ & =\overbrace{\left(\prod_{k=1}^{\left|\mathbf{j}\right|}\frac{a_{j_{k}}}{d_{j_{k}}}\right)}^{M_{H}\left(\mathbf{j}\right)}x+\overbrace{\frac{b_{j_{1}}}{d_{j_{1}}}+\frac{a_{j_{1}}}{d_{j_{1}}}\frac{b_{j_{2}}}{d_{j_{2}}}+\frac{a_{j_{2}}}{d_{j_{2}}}\frac{a_{j_{1}}}{d_{j_{1}}}\frac{b_{j_{3}}}{d_{j_{3}}}+\cdots+\left(\prod_{k=1}^{\left|\mathbf{j}\right|-1}\frac{a_{j_{k}}}{d_{j_{k}}}\right)\frac{b_{j_{\left|\mathbf{j}\right|}}}{d_{j_{\left|\mathbf{j}\right|}}}}^{\chi_{H}\left(\mathbf{j}\right)}\\ & =M_{H}\left(\mathbf{j}\right)x+\underbrace{\sum_{m=1}^{\left|\mathbf{j}\right|}\frac{b_{j_{m}}}{d_{j_{m}}}\prod_{k=1}^{m-1}\frac{a_{j_{k}}}{d_{j_{k}}}}_{\chi_{H}\left(\mathbf{j}\right)} \end{align*} where we use the convention that $\prod_{k=1}^{m-1}\frac{a_{j_{k}}}{d_{j_{k}}}=1$ when $m=1$. This proves: \begin{equation} \chi_{H}\left(\mathbf{j}\right)=\sum_{m=1}^{\left|\mathbf{j}\right|}\frac{b_{j_{m}}}{d_{j_{m}}}\prod_{k=1}^{m-1}\frac{a_{j_{k}}}{d_{j_{k}}} \end{equation} Next, using $\mu_{j}=\frac{pa_{j}}{d_{j}}$, for $m>1$, the $m$th term $\frac{b_{j_{m}}}{d_{j_{m}}}\prod_{k=1}^{m-1}\frac{a_{j_{k}}}{d_{j_{k}}}$ can be written as: \begin{align*} \frac{b_{j_{m}}}{d_{j_{m}}}\prod_{k=1}^{m-1}\frac{a_{j_{k}}}{d_{j_{k}}} & =\frac{1}{p}\cdot\frac{b_{j_{m}}}{a_{j_{m}}}\cdot\frac{pa_{j_{m}}}{d_{j_{m}}}\cdot\prod_{k=1}^{m-1}\left(\frac{pa_{j_{k}}}{d_{j_{k}}}\cdot\frac{1}{p}\right)\\ & =\frac{1}{p}\frac{b_{j_{m}}}{a_{j_{m}}}\frac{\mu_{j_{m}}}{p^{m-1}}\prod_{k=1}^{m-1}\mu_{j_{k}}\\ & =\frac{b_{j_{m}}}{a_{j_{m}}}\left(\prod_{k=1}^{m}\mu_{j_{k}}\right)p^{-m} \end{align*} On the other hand, when $m=1$, by our $k$-product convention, we have: \begin{equation} \frac{b_{j_{1}}}{d_{j_{1}}}\underbrace{\prod_{k=1}^{1-1}\frac{a_{j_{k}}}{d_{j_{k}}}}_{1}=\frac{b_{j_{1}}}{d_{j_{1}}} \end{equation} Thus: \begin{align*} \chi_{H}\left(\mathbf{j}\right) & =\frac{b_{j_{1}}}{d_{j_{1}}}+\sum_{m=2}^{\left|\mathbf{j}\right|}\frac{b_{j_{m}}}{a_{j_{m}}}\left(\prod_{k=1}^{m}\mu_{j_{k}}\right)p^{-m}\\ & =\frac{1}{p}\frac{b_{j_{1}}}{a_{j_{1}}}\frac{pa_{j_{1}}}{d_{j_{1}}}+\sum_{m=2}^{\left|\mathbf{j}\right|}\frac{b_{j_{m}}}{a_{j_{m}}}\left(\prod_{k=1}^{m}\mu_{j_{k}}\right)p^{-m}\\ & =\frac{b_{j_{1}}}{a_{j_{1}}}\cdot\mu_{j_{1}}\cdot p^{-1}+\sum_{m=2}^{\left|\mathbf{j}\right|}\frac{b_{j_{m}}}{a_{j_{m}}}\left(\prod_{k=1}^{m}\mu_{j_{k}}\right)p^{-m}\\ & =\sum_{m=1}^{\left|\mathbf{j}\right|}\frac{b_{j_{m}}}{a_{j_{m}}}\left(\prod_{k=1}^{m}\mu_{j_{k}}\right)p^{-m} \end{align*} as desired. Q.E.D. \vphantom{} To interpolate $\chi_{H}$ from a rational-valued function on $\mathbb{N}_{0}$ to a $q$-adic-valued function on $\mathbb{Z}_{p}$, we need to define some qualitative properties of Hydra maps so as to distinguish those cases where this interpolation will actually exist. \begin{defn}[\textbf{Qualitative Terminology for $p$-Hydra maps}] \label{def:Qualitative conditions on a p-Hydra map}Let $H$ be a $p$-Hydra map. For the definitions below, we DO NOT require $p$ to be prime. \vphantom{} I. We say $H$ is \textbf{simple }if $d_{j}=p$ for all $j$. \vphantom{} II. We say $H$ is \textbf{semi-simple }if $\gcd\left(a_{j},d_{k}\right)=1$ for all $j,k\in\left\{ 0,\ldots,p-1\right\} $. \vphantom{} III. We say $H$ is \textbf{degenerate }if $a_{j}=1$ for some non-zero $j$. If $a_{j}\neq1$ for all non-zero $j$, we say $H$ is \textbf{non-degenerate}. \vphantom{} IV\footnote{This definition will not be relevant until Chapter 3.}. We say $H$ is \textbf{contracting }if $a_{0}/d_{0}$ (a.k.a. $\mu_{0}/p$) is $<1$, and say $H$ is \textbf{expanding }if $a_{0}/d_{0}$ (a.k.a. $\mu_{0}/p$) is $>1$. \vphantom{} V. We say $H$ is \textbf{monogenic}\footnote{This term is based on the Greek for ``one kind/species''.} whenever: \begin{equation} \gcd\left(\left\{ a_{j}:j\in\left\{ 0,\ldots,p-1\right\} \textrm{ \& }a_{j}\neq1\right\} \right)\geq2\label{eq:Monogenic gcd} \end{equation} When this $\textrm{gcd}$ is $1$, we say $H$ is \textbf{polygenic}. If $H$ is monogenic, we then write $q_{H}$\nomenclature{$q_{H}$}{ } to denote the smallest prime divisor of (\ref{eq:Monogenic gcd}). For brevity, sometimes we will drop the $H$ and just write this as $q$ instead of $q_{H}$. \vphantom{} VI. We say $H$ is \textbf{basic }if $H$ is simple, non-degenerate, and monogenic. \vphantom{} VII. We say $H$ is \textbf{semi-basic }if $H$ is semi-simple, non-degenerate, and monogenic. \end{defn} \begin{rem} If $H$ is simple, note that $\mu_{j}=a_{j}$ for all $j$. \end{rem} \begin{rem} It is worth noting that all of the results in this section hold when $q_{H}$ is \emph{any }prime divisor of (\ref{eq:Monogenic gcd}). \end{rem} \vphantom{} Next, a minor technical result which shows that our qualitative definitions are actually useful. \begin{prop}[\textbf{Co-primality of $d_{j}$ and $q_{H}$}] \label{prop:co-primality of d_j and q_H}Let $H$ be a semi-basic $p$-Hydra map. Then, $\gcd\left(d_{j},q_{H}\right)=1$ for all $j\in\left\{ 0,\ldots,p-1\right\} $. \end{prop} Proof: By way of contradiction, suppose there is a $k\in\left\{ 0,\ldots,p-1\right\} $ so that $\gcd\left(d_{k},q_{H}\right)>1$. Since $H$ is semi-basic, $H$ is monogenic and non-degenerate, and so $q_{H}$ is $\geq2$ and, moreover, $a_{j}\neq1$ for any $j\in\left\{ 1,\ldots,p-1\right\} $. Thus, there is a $k$ so that $d_{k}$ and $q_{H}$ have a non-trivial common factor, and there is a $j$ so that $q_{H}$ and $a_{j}$ have a non-trivial common factor (namely, $q_{H}$). Since every factor of $q_{H}$ divides $a_{j}$, the non-trivial common factor of $d_{k}$ and $q_{H}$ must also divide $a_{j}$. This forces $\gcd\left(a_{j},d_{k}\right)>1$. However, as a semi-basic $p$-Hydra map, $H$ is semi-simple, and so $\gcd\left(a_{j},d_{k}\right)=1$ for all $j,k\in\left\{ 0,\ldots,p-1\right\} $\textemdash and that is a contradiction! Thus, it must be that $\gcd\left(d_{k},q_{H}\right)=1$ for all $k\in\left\{ 0,\ldots,p-1\right\} $. Q.E.D. \vphantom{} While our next result is also technical, it is not minor in the least. It demonstrates that $M_{H}\left(\mathbf{j}\right)$ will be $q_{H}$-adically small whenever $\mathbf{j}$ has many non-zero entries. This is the essential ingredient for the proof of the existence of $\chi_{H}$'s $p$-adic interpolation. \begin{prop}[\textbf{$q_{H}$-adic behavior of $M_{H}\left(\mathbf{j}\right)$ as $\left|\mathbf{j}\right|\rightarrow\infty$}] \label{prop:q-adic behavior of M_H of j as the number of non-zero digits tends to infinity}Let $H$ be a semi-basic $p$-Hydra map which fixes $0$. Then: \begin{equation} \left|M_{H}\left(\mathbf{j}\right)\right|_{q_{H}}\leq q_{H}^{-\sum_{k=1}^{p-1}\#_{p:k}\left(\mathbf{j}\right)}\label{eq:M_H of bold j has q-adic valuation at least as large as the number of non-zero digits of bold-j} \end{equation} In particular, for any sequence of strings $\left\{ \mathbf{j}_{n}\right\} _{n\geq1}$ in $\textrm{String}\left(p\right)$ so that $\lim_{n\rightarrow\infty}\left|\mathbf{j}_{n}\right|=\infty$, we have that $\lim_{n\rightarrow\infty}\left|M_{H}\left(\mathbf{j}_{n}\right)\right|_{q_{H}}=0$ whenever the number of non-zero entries in $\mathbf{j}_{n}$ tends to $\infty$ as $n\rightarrow\infty$. \end{prop} Proof: Let $\mathbf{j}$ be a non-zero element of $\textrm{String}\left(p\right)$. Then, by \textbf{Proposition \ref{prop:Explicit Formulas for M_H}}: \[ M_{H}\left(\mathbf{j}\right)=\prod_{\ell=1}^{\left|\mathbf{j}\right|}\frac{a_{j_{\ell}}}{d_{j_{\ell}}} \] Since $H$ is semi-basic, \textbf{Proposition \ref{prop:co-primality of d_j and q_H}} tells us that every $d_{j}$ is co-prime to $q_{H}$, and so $\left|d_{j}\right|_{q_{H}}=1$ for any $j$. On the other hand, the non-degeneracy and monogenicity of $H$ tells us that $a_{j}$ is a non-zero integer multiple of $q_{H}$ for all $j\in\left\{ 1,\ldots,p-1\right\} $; thus, $\left|a_{j}\right|_{q_{H}}\leq1/q_{H}$ for every $\ell$ for which $j_{\ell}\neq0$. Taking $q_{H}$-adic absolute values of $M_{H}\left(\mathbf{j}\right)$ then gives: \begin{equation} \left|M_{H}\left(\mathbf{j}\right)\right|_{q_{H}}=\prod_{\ell=1}^{\left|\mathbf{j}\right|}\left|\frac{a_{j_{\ell}}}{d_{j_{\ell}}}\right|_{q_{H}}=\prod_{\ell:a_{j_{\ell}}\neq0}\left|a_{j_{\ell}}\right|_{q_{H}}\leq q_{H}^{-\left|\left\{ \ell\in\left\{ 1,\ldots,\left|\mathbf{j}\right|\right\} :j_{\ell}\neq0\right\} \right|} \end{equation} Here, $\left|\left\{ \ell\in\left\{ 1,\ldots,\left|\mathbf{j}\right|\right\} :j_{\ell}\neq0\right\} \right|$ is precisely the number of non-zero entries of $\mathbf{j}$, which is: \begin{equation} \left|\left\{ \ell\in\left\{ 1,\ldots,\left|\mathbf{j}\right|\right\} :j_{\ell}\neq0\right\} \right|=\sum_{k=1}^{p-1}\#_{p:k}\left(\mathbf{j}\right) \end{equation} For a sequence of $\mathbf{j}_{n}$s tending to $\infty$ in length, this also shows that $\left|M_{H}\left(\mathbf{j}_{n}\right)\right|_{q_{H}}\rightarrow0$ as $n\rightarrow\infty$, provided that the number of non-zero entries in $\mathbf{j}_{n}$ also tends to $\infty$ as $n\rightarrow\infty$. Q.E.D. \vphantom{} Now the main result of this subsection: the $\left(p,q\right)$-adic characterization of $\chi_{H}$. \begin{lem}[\textbf{$\left(p,q\right)$-adic Characterization of }$\chi_{H}$] \label{lem:Unique rising continuation and p-adic functional equation of Chi_H}Let $H$ be a semi-basic $p$-Hydra map which fixes $0$. Then, the limit: \begin{equation} \chi_{H}\left(\mathfrak{z}\right)\overset{\mathbb{Z}_{q_{H}}}{=}\lim_{n\rightarrow\infty}\chi_{H}\left(\left[\mathfrak{z}\right]_{p^{n}}\right)\label{eq:Rising Continuity Formula for Chi_H} \end{equation} exists for all $\mathfrak{z}\in\mathbb{Z}_{p}$, and thereby defines an interpolation of $\chi_{H}$ to a function $\chi_{H}:\mathbb{Z}_{p}\rightarrow\mathbb{Z}_{q_{H}}$. Moreover: \vphantom{} I. \begin{equation} \chi_{H}\left(p\mathfrak{z}+j\right)=\frac{a_{j}\chi_{H}\left(\mathfrak{z}\right)+b_{j}}{d_{j}},\textrm{ }\forall\mathfrak{z}\in\mathbb{Z}_{p},\textrm{ }\forall j\in\mathbb{Z}/p\mathbb{Z}\label{eq:Functional Equations for Chi_H over the rho-adics} \end{equation} \vphantom{} II. The interpolation $\chi_{H}:\mathbb{Z}_{p}\rightarrow\mathbb{Z}_{q_{H}}$ defined by the limit \emph{(\ref{eq:Rising Continuity Formula for Chi_H})} is the \textbf{unique} function $f:\mathbb{Z}_{p}\rightarrow\mathbb{Z}_{q_{H}}$ satisfying the functional equations: \emph{ \begin{equation} f\left(p\mathfrak{z}+j\right)=\frac{a_{j}f\left(\mathfrak{z}\right)+b_{j}}{d_{j}},\textrm{ }\forall\mathfrak{z}\in\mathbb{Z}_{p},\textrm{ }\forall j\in\mathbb{Z}/p\mathbb{Z}\label{eq:unique p,q-adic functional equation of Chi_H} \end{equation} }along with the \textbf{rising-continuity}\footnote{As discussed in Section \ref{sec:3.2 Rising-Continuous-Functions}, I say a function is \textbf{rising-continuous }whenever it satisfies the limit condition (\ref{eq:unique p,q-adic rising continuity of Chi_H}). (\ref{eq:Rising Continuity Formula for Chi_H}) shows that $\chi_{H}$ is rising-continuous.} \textbf{condition}:\textbf{ } \begin{equation} f\left(\mathfrak{z}\right)\overset{\mathbb{Z}_{q_{H}}}{=}\lim_{n\rightarrow\infty}f\left(\left[\mathfrak{z}\right]_{p^{n}}\right),\textrm{ }\forall\mathfrak{z}\in\mathbb{Z}_{p}\label{eq:unique p,q-adic rising continuity of Chi_H} \end{equation} \end{lem} Proof: First, we show the existence of the limit (\ref{eq:Rising Continuity Formula for Chi_H}). If $\mathfrak{z}\in\mathbb{N}_{0}\cap\mathbb{Z}_{p}$, then $\left[\mathfrak{z}\right]_{p^{N}}=\mathfrak{z}$ for all sufficiently large $N$, showing that the limit (\ref{eq:Rising Continuity Formula for Chi_H}) exists in that case. So, suppose $\mathfrak{z}\in\mathbb{Z}_{p}^{\prime}$. Next, let $\mathbf{j}_{n}=\left(j_{1},\ldots,j_{n}\right)$ be the shortest string representing $\left[\mathfrak{z}\right]_{p^{n}}$; that is: \begin{equation} \left[\mathfrak{z}\right]_{p^{n}}=\sum_{k=1}^{n}j_{k}p^{k-1} \end{equation} By \textbf{Lemma \ref{eq:Definition of Chi_H of n}}, (\ref{eq:Rising Continuity Formula for Chi_H}) can be written as: \begin{equation} \chi_{H}\left(\mathfrak{z}\right)=\lim_{n\rightarrow\infty}\chi_{H}\left(\mathbf{j}_{n}\right) \end{equation} Using \textbf{Proposition \ref{prop:Explicit formula for Chi_H of bold j}}, we can write: \begin{equation} \chi_{H}\left(\mathbf{j}_{n}\right)=H_{\mathbf{j}_{n}}\left(0\right)=\sum_{m=1}^{\left|\mathbf{j}_{n}\right|}\frac{b_{j_{m}}}{d_{j_{m}}}\prod_{k=1}^{m-1}\frac{a_{j_{k}}}{d_{j_{k}}} \end{equation} where the product is defined to be $1$ when $m=1$. Then, using \textbf{Proposition \ref{prop:Explicit Formulas for M_H}}, we get: \begin{equation} \prod_{k=1}^{m-1}\frac{a_{j_{k}}}{d_{j_{k}}}=M_{H}\left(\mathbf{j}_{m-1}\right) \end{equation} where, again, the product is defined to be $1$ when $m=1$. So, our previous equation for $\chi_{H}\left(\mathbf{j}_{n}\right)$ can be written as: \begin{equation} \chi_{H}\left(\mathbf{j}_{n}\right)=\sum_{m=1}^{\left|\mathbf{j}_{n}\right|}\frac{b_{j_{m}}}{d_{j_{m}}}M_{H}\left(\mathbf{j}_{m-1}\right) \end{equation} Taking limits yields: \begin{equation} \lim_{n\rightarrow\infty}\chi_{H}\left(\left[\mathfrak{z}\right]_{p^{n}}\right)=\lim_{n\rightarrow\infty}\chi_{H}\left(\mathbf{j}_{n}\right)=\lim_{n\rightarrow\infty}\sum_{m=1}^{\infty}\frac{b_{j_{m}}}{d_{j_{m}}}M_{H}\left(\mathbf{j}_{m-1}\right)\label{eq:Formal rising limit of Chi_H} \end{equation} Thanks to the ultrametric topology of $\mathbb{Z}_{q_{H}}$, the series on the right will converge in $\mathbb{Z}_{q_{H}}$ if and only if is $m$th term tends to $0$ $q_{H}$-adically: \begin{equation} \lim_{m\rightarrow\infty}\left|\frac{b_{j_{m}}}{d_{j_{m}}}M_{H}\left(\mathbf{j}_{m-1}\right)\right|_{q_{H}}=0 \end{equation} Because the $b_{j}$s and $d_{j}$s belong to a finite set, with the $d_{j}$s being all non-zero: \begin{equation} \sup_{j\in\left\{ 0,\ldots,p-1\right\} }\left|\frac{b_{j}}{d_{j}}\right|_{q_{H}}<\infty \end{equation} and so: \begin{equation} \left|\frac{b_{j_{m}}}{d_{j_{m}}}M_{H}\left(\mathbf{j}_{m-1}\right)\right|_{q_{H}}\ll\left|M_{H}\left(\mathbf{j}_{m-1}\right)\right|_{q_{H}} \end{equation} Because $\mathfrak{z}\in\mathbb{Z}_{p}^{\prime}$, $\mathfrak{z}$ has infinitely many non-zero digits. As such, both the length and number of non-zero entries of $\mathbf{j}_{m-1}$ tend to infinity as $m\rightarrow\infty$. By\textbf{ Proposition \ref{prop:q-adic behavior of M_H of j as the number of non-zero digits tends to infinity}}, this forces $\left|M_{H}\left(\mathbf{j}_{m-1}\right)\right|_{q_{H}}\rightarrow0$ as $m\rightarrow\infty$. So, the $m$th term of our series decays to zero $q_{H}$-adically, which then guarantees the convergence of $\lim_{n\rightarrow\infty}\chi_{H}\left(\left[\mathfrak{z}\right]_{p^{n}}\right)$ in $\mathbb{Z}_{q_{H}}$. \vphantom{} To prove (I), by \textbf{Lemma \ref{lem:Chi_H functional equation on N_0 and uniqueness}}, we know that $\chi_{H}$ satisfies the functional equations (\ref{eq:Functional Equations for Chi_H over the rho-adics}) for $\mathfrak{z}\in\mathbb{N}_{0}$. Taking limits of the functional equations using (\ref{eq:Rising Continuity Formula for Chi_H}) then proves the equations hold for all $\mathfrak{z}\in\mathbb{Z}_{p}$. \vphantom{} For (II), we have just shown that $\chi_{H}$ satisfies the interpolation condition and the functional equations. So, conversely, suppose $f:\mathbb{Z}_{p}\rightarrow\mathbb{Z}_{q_{H}}$ satisfies (\ref{eq:unique p,q-adic functional equation of Chi_H}) and (\ref{eq:unique p,q-adic rising continuity of Chi_H}). Then, by \textbf{Lemma \ref{lem:Chi_H functional equation on N_0 and uniqueness}}, the restriction of $f$ to $\mathbb{N}_{0}$ must be equal to $\chi_{H}$. (\ref{eq:Rising Continuity Formula for Chi_H}) shows that the values $f\left(\mathfrak{z}\right)\overset{\mathbb{Z}_{q_{H}}}{=}\lim_{n\rightarrow\infty}f\left(\left[\mathfrak{z}\right]_{p^{n}}\right)$ obtained by taking limits necessarily forces $f\left(\mathfrak{z}\right)=\chi_{H}\left(\mathfrak{z}\right)$ for all $\mathfrak{z}\in\mathbb{Z}_{p}$. Q.E.D. \subsection{\label{subsec:2.2.3 The-Correspondence-Principle}The Correspondence Principle} THROUGHOUT THIS SUBSECTION, WE ASSUME $H$ IS INTEGRAL. \vphantom{} Our goal\textemdash the \textbf{Correspondence Principle}\textemdash relates the periodic points of $H$ in $\mathbb{Z}$ to the set $\mathbb{Z}\cap\chi_{H}\left(\mathbb{Z}_{p}\right)$, where we view both $\mathbb{Z}$ and $\chi_{H}\left(\mathbb{Z}_{p}\right)$ as being embedded in $\mathbb{Z}_{q_{H}}$. The distinction between basic and semi-basic $p$-Hydra maps will play a key role here, as there is an additional property (\textbf{propriety}, to be defined below) satisfied by semi-basic integral $p$-Hydra maps which is needed to ensure that the set $\mathbb{Z}\cap\chi_{H}\left(\mathbb{Z}_{p}\right)$ fully characterizes the periodic points of $H$. Before proceeding, we will also need a certain self-map of $\mathbb{Z}_{p}$ which I denote by $B_{p}$, so as to formulate one more functional equation satisfied by $\chi_{H}$, one directly implicated in the Correspondence Principle. First, however, a useful\textemdash and motivating\textemdash observation. \begin{prop} \label{prop:Concatenation exponentiation}Let $p$ be an integer $\geq2$, let $n\in\mathbb{N}_{1}$, and let $\mathbf{j}\in\textrm{String}\left(p\right)$ be the shortest string representing $n$. Then, for all $m\in\mathbb{N}_{1}$: \begin{equation} \mathbf{j}^{\wedge m}\sim n\frac{1-p^{m\lambda_{p}\left(n\right)}}{1-p^{\lambda_{p}\left(n\right)}}\label{eq:Proposition 1.2.10} \end{equation} That is to say, the quantity on the right is the integer whose sequence of $p$-adic digits consists of $m$ concatenated copies of the $p$-adic digits of $n$ (or, equivalently, the entries of $\mathbf{j}$). \end{prop} Proof: Since: \[ n=j_{1}+j_{2}p+\cdots+j_{\lambda_{p}\left(n\right)-1}p^{\lambda_{p}\left(n\right)-1} \] we have that: \begin{align*} \mathbf{j}^{\wedge m} & \sim j_{1}+j_{2}p+\cdots+j_{\lambda_{p}\left(n\right)-1}p^{\lambda_{p}\left(n\right)-1}\\ & +p^{\lambda_{p}\left(n\right)}\left(j_{1}+j_{2}p+\cdots+j_{\lambda_{p}\left(n\right)-1}p^{\lambda_{p}\left(n\right)-1}\right)\\ & +\cdots\\ & +p^{\left(m-1\right)\lambda_{p}\left(n\right)}\left(j_{1}+j_{2}p+\cdots+j_{\lambda_{p}\left(n\right)-1}p^{\lambda_{p}\left(n\right)-1}\right)\\ & =n+np^{\lambda_{p}\left(n\right)}+np^{2\lambda_{p}\left(n\right)}+\cdots+np^{\left(m-1\right)\lambda_{p}\left(n\right)}\\ & =n\frac{1-p^{m\lambda_{p}\left(n\right)}}{1-p^{\lambda_{p}\left(n\right)}} \end{align*} as desired. Q.E.D. \begin{defn}[$B_{p}$] \nomenclature{$B_{p}$}{ }Let $p$ be an integer $\geq2$. Then, we define $B_{p}:\mathbb{N}_{0}\rightarrow\mathbb{Q}\cap\mathbb{Z}_{p}$ by: \begin{equation} B_{p}\left(n\right)\overset{\textrm{def}}{=}\begin{cases} 0 & \textrm{if }n=0\\ \frac{n}{1-p^{\lambda_{p}\left(n\right)}} & \textrm{if }n\geq1 \end{cases}\label{eq:Definition of B projection function} \end{equation} \end{defn} \begin{rem} $B_{p}$ has a simple interpretation in terms of $p$-adic expansions: it sends $n$ to the $p$-adic integer whose sequence of $p$-adic digits consists of infinitely many concatenated copies of the sequence of $p$-adic digits of $n$: \begin{equation} B_{p}\left(n\right)\overset{\mathbb{Z}_{p}}{=}\lim_{m\rightarrow\infty}n\frac{1-p^{m\lambda_{p}\left(n\right)}}{1-p^{\lambda_{p}\left(n\right)}}\overset{\mathbb{Z}_{p}}{=}\frac{n}{1-p^{\lambda_{p}\left(n\right)}} \end{equation} In particular, since the sequence of $p$-adic digits of $B_{p}\left(n\right)$ is, by construction, periodic, the geometric series formula in $\mathbb{Z}_{p}$ guarantees the $p$-adic integer $B_{p}\left(n\right)$ is in fact an element of $\mathbb{Q}$. \end{rem} \begin{rem} Note that $B_{p}$ extends to a function on $\mathbb{Z}_{p}$, one whose restriction to $\mathbb{Z}_{p}^{\prime}$ is identity map. \end{rem} \vphantom{} Using $\chi_{H}$ and $B_{p}$, we can compactly and elegantly express the diophantine equation (\ref{eq:The Bohm-Sontacchi Criterion}) of the Bhm-Sontacchi Criterion in terms of a functional equation. The \textbf{Correspondence Principle }emerges almost immediately from this functional equation. \begin{lem}[\textbf{Functional Equation for }$\chi_{H}\circ B_{p}$] \label{lem:Chi_H o B_p functional equation}Let\index{functional equation!chi_{H}circ B_{p}@$\chi_{H}\circ B_{p}$} $H$ be semi-basic. Then: \begin{equation} \chi_{H}\left(B_{p}\left(n\right)\right)\overset{\mathbb{Z}_{qH}}{=}\frac{\chi_{H}\left(n\right)}{1-M_{H}\left(n\right)},\textrm{ }\forall n\in\mathbb{N}_{1}\label{eq:Chi_H B functional equation} \end{equation} \end{lem} Proof: Let $n\in\mathbb{N}_{1}$, and let $\mathbf{j}\in\textrm{String}\left(p\right)$ be the shortest string representing $n$. Now, for all $m\geq1$ and all $k\in\mathbb{N}_{1}$: \begin{equation} H_{\mathbf{j}^{\wedge m}}\left(k\right)=H_{\mathbf{j}}^{\circ m}\left(k\right) \end{equation} Since: \[ H_{\mathbf{j}}\left(k\right)=M_{H}\left(\mathbf{j}\right)k+\chi_{H}\left(\mathbf{j}\right) \] the geometric series formula gives us: \begin{equation} H_{\mathbf{j}^{\wedge m}}\left(k\right)=H_{\mathbf{j}}^{\circ m}\left(k\right)=\left(M_{H}\left(\mathbf{j}\right)\right)^{m}k+\frac{1-\left(M_{H}\left(\mathbf{j}\right)\right)^{m}}{1-M_{H}\left(\mathbf{j}\right)}\chi_{H}\left(\mathbf{j}\right) \end{equation} Since we also have: \begin{equation} H_{\mathbf{j}^{\wedge m}}\left(k\right)=M_{H}\left(\mathbf{j}^{\wedge m}\right)k+\chi_{H}\left(\mathbf{j}^{\wedge m}\right) \end{equation} this then yields: \begin{equation} M_{H}\left(\mathbf{j}^{\wedge m}\right)k+\chi_{H}\left(\mathbf{j}^{\wedge m}\right)=\left(M_{H}\left(\mathbf{j}\right)\right)^{m}k+\frac{1-\left(M_{H}\left(\mathbf{j}\right)\right)^{m}}{1-M_{H}\left(\mathbf{j}\right)}\chi_{H}\left(\mathbf{j}\right) \end{equation} By \textbf{Proposition \ref{prop:M_H concatenation identity}}, we see that $M_{H}\left(\mathbf{j}^{\wedge m}\right)k=\left(M_{H}\left(\mathbf{j}\right)\right)^{m}k$. Cancelling these terms from both sides leaves us with: \begin{equation} \chi_{H}\left(\mathbf{j}^{\wedge m}\right)=\frac{1-\left(M_{H}\left(\mathbf{j}\right)\right)^{m}}{1-M_{H}\left(\mathbf{j}\right)}\chi_{H}\left(\mathbf{j}\right)=\frac{1-\left(M_{H}\left(\mathbf{j}\right)\right)^{m}}{1-M_{H}\left(n\right)}\chi_{H}\left(n\right)\label{eq:Chi_H B functional equation, ready to take limits} \end{equation} Now, by \textbf{Proposition \ref{prop:Concatenation exponentiation}}, we have that: \begin{equation} \chi_{H}\left(\mathbf{j}^{\wedge m}\right)=\chi_{H}\left(n\frac{1-p^{m\lambda_{p}\left(n\right)}}{1-p^{\lambda_{p}\left(n\right)}}\right) \end{equation} where the equality is of rational numbers in $\mathbb{R}$. Since $n\in\mathbb{N}_{1}$, we have that: \begin{equation} B_{p}\left(n\right)=\frac{n}{1-p^{\lambda_{p}\left(n\right)}} \end{equation} is a $p$-adic integer. Moreover, as is immediate from the proof of \textbf{Proposition \ref{prop:Concatenation exponentiation}}, the projection of this $p$-adic integer modulo $p^{m}$ is: \begin{equation} \left[B_{p}\left(n\right)\right]_{p^{m}}=n\frac{1-p^{m\lambda_{p}\left(n\right)}}{1-p^{\lambda_{p}\left(n\right)}} \end{equation} which is, of course, exactly the rational integer represented by the string $\mathbf{j}^{\wedge m}$. In other words: \begin{equation} \frac{1-\left(M_{H}\left(\mathbf{j}\right)\right)^{m}}{1-M_{H}\left(n\right)}\chi_{H}\left(n\right)=\chi_{H}\left(\mathbf{j}^{\wedge m}\right)=\chi_{H}\left(n\frac{1-p^{m\lambda_{p}\left(n\right)}}{1-p^{\lambda_{p}\left(n\right)}}\right)=\chi_{H}\left(\left[B_{p}\left(n\right)\right]_{p^{m}}\right) \end{equation} By \textbf{Lemma \ref{lem:Unique rising continuation and p-adic functional equation of Chi_H}}, we have that: \begin{equation} \chi_{H}\left(B_{p}\left(n\right)\right)\overset{\mathbb{Z}_{q}}{=}\lim_{m\rightarrow\infty}\chi_{H}\left(\left[B_{p}\left(n\right)\right]_{p^{m}}\right)=\lim_{m\rightarrow\infty}\frac{1-\left(M_{H}\left(\mathbf{j}\right)\right)^{m}}{1-M_{H}\left(n\right)}\chi_{H}\left(n\right) \end{equation} Finally, because $H$ is semi-basic, it follows by \textbf{Proposition \ref{prop:q-adic behavior of M_H of j as the number of non-zero digits tends to infinity} }that $\left|M_{H}\left(\mathbf{j}\right)\right|_{q}<1$ (since $\mathbf{j}$ has a non-zero entry), and hence, that $\left(M_{H}\left(\mathbf{j}\right)\right)^{m}$ tends to $0$ in $\mathbb{Z}_{q}$ as $m\rightarrow\infty$. Thus: \begin{equation} \chi_{H}\left(B_{p}\left(n\right)\right)\overset{\mathbb{Z}_{q}}{=}\lim_{m\rightarrow\infty}\frac{1-\left(M_{H}\left(\mathbf{j}\right)\right)^{m}}{1-M_{H}\left(n\right)}\chi_{H}\left(n\right)\overset{\mathbb{Z}_{q}}{=}\frac{\chi_{H}\left(n\right)}{1-M_{H}\left(n\right)} \end{equation} Q.E.D. \begin{rem} As seen in the above proof, (\ref{eq:Chi_H B functional equation}) is really just the geometric series summation formula, convergent in $\mathbb{Z}_{q}$ whenever $\left|M_{H}\left(n\right)\right|_{q_{H}}<1$. In computing $\chi_{H}\left(B_{p}\left(n\right)\right)$ by using (\ref{eq:Rising Continuity Formula for Chi_H}) with $\mathfrak{z}=B_{p}\left(n\right)$ and $n\geq1$, the series in (\ref{eq:Formal rising limit of Chi_H}) will reduce to a geometric series, one which converges in $\mathbb{Z}_{q}$ to $\chi_{H}\left(n\right)/\left(1-M_{H}\left(n\right)\right)$. Additionally, for any $n\geq1$ for which the \emph{archimedean }absolute value $\left|M_{H}\left(n\right)\right|$ is less than $1$, the \emph{universality} of the geometric series formula: \begin{equation} \sum_{n=0}^{\infty}x^{n}=\frac{1}{1-x} \end{equation} guarantees that the limit to which the series in (\ref{eq:Formal rising limit of Chi_H}) converges to in $\mathbb{R}$ in this case is the same as the limit to which it converges in $\mathbb{Z}_{q_{H}}$ \cite{Conrad on p-adic series}. This is worth mentioning because, as we shall see, the values of $n\geq1$ for which $\chi_{H}\left(n\right)/\left(1-M_{H}\left(n\right)\right)$ is a periodic point of $H$ in $\mathbb{N}_{1}$ are exactly those values of $n\geq1$ for which the non-archimedean absolute value $\left|M_{H}\left(n\right)\right|_{q_{H}}$ is $<1$. \end{rem} \vphantom{} Now we introduce the final bit of qualitative controls on $H$'s behavior which we will needed to prove the \textbf{Correspondence Principle}. First, however, a proposition about $H$'s extendibility to the $p$-adics. \begin{prop} \label{prop:p-adic extension of H}Let $H:\mathbb{Z}\rightarrow\mathbb{Z}$ be a $p$-Hydra map. Then, $H$ admits an extension to a continuous map $\mathbb{Z}_{p}\rightarrow\mathbb{Z}_{p}$ defined by: \begin{equation} H\left(\mathfrak{z}\right)\overset{\textrm{def}}{=}\sum_{j=0}^{p-1}\left[\mathfrak{z}\overset{p}{\equiv}j\right]\frac{a_{j}\mathfrak{z}+b_{j}}{d_{j}},\textrm{ }\forall\mathfrak{z}\in\mathbb{Z}_{p}\label{eq:p-adic extension of H} \end{equation} meaning that, $H\left(\mathfrak{z}\right)=H_{j}\left(\mathfrak{z}\right)$ for all $j\in\left\{ 0,\ldots,p-1\right\} $ and all $\mathfrak{z}\in j+p\mathbb{Z}_{p}$. Moreover, the function $f:\mathbb{Z}_{p}\rightarrow\mathbb{Z}_{p}$ defined by: \begin{equation} f\left(\mathfrak{z}\right)=\sum_{j=0}^{p-1}\left[\mathfrak{z}\overset{p}{\equiv}j\right]\frac{a_{j}\mathfrak{z}+b_{j}}{d_{j}},\textrm{ }\forall\mathfrak{z}\in\mathbb{Z}_{p} \end{equation} is the unique continuous function $\mathbb{Z}_{p}\rightarrow\mathbb{Z}_{p}$ whose restriction to $\mathbb{Z}$ is equal to $H$. \end{prop} Proof: Let $f\left(\mathfrak{z}\right)=\sum_{j=0}^{p-1}\left[\mathfrak{z}\overset{p}{\equiv}j\right]\frac{a_{j}\mathfrak{z}+b_{j}}{d_{j}}$; continuity is immediate, as is the fact that the restriction of $f$ to $\mathbb{Z}$ is equal to $H$. As for uniqueness, if $f:\mathbb{Z}_{p}\rightarrow\mathbb{Z}_{p}$ is any continuous function whose restriction on $\mathbb{Z}$ is equal to $H$, the density of $\mathbb{Z}$ in $\mathbb{Z}_{p}$ coupled with the continuity of $f$ then forces $f\left(\mathfrak{z}\right)=\sum_{j=0}^{p-1}\left[\mathfrak{z}\overset{p}{\equiv}j\right]\frac{a_{j}\mathfrak{z}+b_{j}}{d_{j}}$ for all $\mathfrak{z}\in\mathbb{Z}_{p}$. Hence, the extension of $H$ to the $p$-adics is unique. Q.E.D. \vphantom{} As mentioned in \textbf{Subsection \ref{subsec:2.1.2 It's-Probably-True}}, the $p$-adic extension of a Hydra map given by \textbf{Proposition \ref{prop:p-adic extension of H}} has been studied in conjunction with investigations of the Collatz Conjecture and the $qx+1$ maps, however\textemdash paradoxically\textemdash doing so seems to lose most information about the map, viz. \textbf{Theorem \ref{thm:shift map}}, which shows that the $2$-adic extension of the Shortened $qx+1$ map is equivalent to the shift map. For us, however, the utility of the $p$-adic extension of $H$ is in keeping track of what happens if we apply a ``wrong'' choice of branch to a given rational number, as described below. \begin{defn}[\textbf{Wrong Values and Propriety}] \label{def:1D wrong values and properiety}\index{Hydra map!wrong value}\index{Hydra map!proper} Let $H$ be a $p$-Hydra map. \vphantom{} I. We say a $p$-adic number $\mathfrak{y}\in\mathbb{Q}_{p}$ is a \textbf{wrong value for $H$ }whenever there is a $\mathbf{j}\in\textrm{String}\left(p\right)$ and a $\mathfrak{z}\in\mathbb{Z}_{p}$ so that $\mathfrak{y}=H_{\mathbf{j}}\left(\mathfrak{z}\right)$ and $H_{\mathbf{j}}\left(\mathfrak{z}\right)\neq H^{\circ\left|\mathbf{j}\right|}\left(\mathfrak{z}\right)$. We then call $\mathfrak{z}$ a \textbf{seed }of $\mathfrak{y}$. \vphantom{} II. We say $H$ is \textbf{proper }if $\left|H_{j}\left(\mathfrak{z}\right)\right|_{p}>1$ holds for any $\mathfrak{z}\in\mathbb{Z}_{p}$ and any $j\in\mathbb{Z}/p\mathbb{Z}$ so that $\left[\mathfrak{z}\right]_{p}\neq j$. \end{defn} \begin{rem} If $\mathfrak{y}$ is a wrong value of $H$ with seed $\mathfrak{z}$ and string $\mathbf{j}$ so that $\mathfrak{y}=H_{\mathbf{j}}\left(\mathfrak{z}\right)$, note that for any $\mathbf{i}\in\textrm{String}\left(p\right)$, $H_{\mathbf{i}}\left(\mathfrak{y}\right)=H_{\mathbf{i}}\left(H_{\mathbf{j}}\left(\mathfrak{z}\right)\right)=H_{\mathbf{i}\wedge\mathbf{j}}\left(\mathfrak{z}\right)$ will \emph{also }be a wrong value of $H$ with seed $\mathfrak{z}$. Thus, branches of $H$ will always map wrong values to wrong values. \end{rem} \begin{rem} Propriety is a stronger version of the integrality condition. If $H$ is an integral Hydra map, then $H_{j}\left(n\right)\notin\mathbb{Z}$ for any $n\in\mathbb{Z}$ and $j\in\mathbb{Z}/p\mathbb{Z}$ with $\left[n\right]_{p}\neq j$. On the other hand, if $H$ is proper, then $H_{j}\left(\mathfrak{z}\right)\notin\mathbb{Z}_{p}$ for any $\mathfrak{z}\in\mathbb{Z}_{p}$ and $j\in\mathbb{Z}/p\mathbb{Z}$ with $\left[\mathfrak{z}\right]_{p}\neq j$. \end{rem} \vphantom{} Like with the qualitative definitions from the previous subsection, we some technical results to show that our new definitions are useful. \begin{prop} \label{prop:Q_p / Z_p prop}Let $H$ be any $p$-Hydra map. Then, $H_{j}\left(\mathbb{Q}_{p}\backslash\mathbb{Z}_{p}\right)\subseteq\mathbb{Q}_{p}\backslash\mathbb{Z}_{p}$ for all $j\in\mathbb{Z}/p\mathbb{Z}$. \end{prop} \begin{rem} Here, we are viewing the $H_{j}$s as functions $\mathbb{Q}_{p}\rightarrow\mathbb{Q}_{p}$. \end{rem} Proof: Let $\mathfrak{y}\in\mathbb{Q}_{p}\backslash\mathbb{Z}_{p}$ and $j\in\mathbb{Z}/p\mathbb{Z}$ be arbitrary. Note that as a $p$-Hydra map, we have $p\mid d_{j}$ and $\gcd\left(a_{j},d_{j}\right)=1$. Consequently, $p$ does not divide $a_{j}$. and so, $\left|a_{j}\right|_{p}=1$. Thus: \begin{equation} \left|a_{j}\mathfrak{y}\right|_{p}=\left|\mathfrak{y}\right|_{p}>1 \end{equation} So, $a_{j}\mathfrak{y}\in\mathbb{Q}_{p}\backslash\mathbb{Z}_{p}$. Since $b_{j}$ is a rational integer, the $p$-adic ultrametric inequality guarantees that $a_{j}\mathfrak{y}+b_{j}\in\mathbb{Q}_{p}\backslash\mathbb{Z}_{p}$. Finally, $p\mid d_{j}$ implies $\left|d_{j}\right|_{p}<1$, and so: \begin{equation} \left|H_{j}\left(\mathfrak{y}\right)\right|_{p}=\left|\frac{a_{j}\mathfrak{y}+b_{j}}{d_{j}}\right|_{p}\geq\left|a_{j}\mathfrak{y}+b_{j}\right|_{p}>1 \end{equation} which shows that $H_{j}\left(\mathfrak{y}\right)\in\mathbb{Q}_{p}\backslash\mathbb{Z}_{p}$. Q.E.D. \vphantom{} The lemmata given below are the key ingredients in our proofs of the \textbf{Correspondence Principle}. These will utilize \textbf{Proposition \ref{prop:Q_p / Z_p prop}}. \begin{lem} \label{lem:integrality lemma}Let $H$ be a semi-basic $p$-Hydra map, where $p$ is prime. Then, $H$ is proper if and only if $H$ is integral. \end{lem} Proof: Let $H$ be semi-basic. I. (Proper implies integral) Suppose $H$ is proper, and let $n\in\mathbb{Z}$ be arbitrary. Then, clearly, when $j=\left[n\right]_{p}$, we have that $H_{j}\left(n\right)\in\mathbb{Z}$. So, suppose $j\in\mathbb{Z}/p\mathbb{Z}$ satisfies $j\neq\left[n\right]_{p}$. Since $H$ is proper, the fact that $n\in\mathbb{Z}\subseteq\mathbb{Z}_{p}$ and $j\neq\left[n\right]_{p}$ tells us that $\left|H_{j}\left(n\right)\right|_{p}>1$. Hence, $H_{j}\left(n\right)$ is not a $p$-adic integer, and thus, cannot be a rational integer either. This proves $H$ is integral. \vphantom{} II. (Integral implies proper) Suppose $H$ is integral, and\textemdash by way of contradiction\textemdash suppose $H$ is \emph{not}\textbf{ }proper. Then, there is a $\mathfrak{z}\in\mathbb{Z}_{p}$ and a $j\in\mathbb{Z}/p\mathbb{Z}$ with $j\neq\left[\mathfrak{z}\right]_{p}$ so that $\left|H_{j}\left(\mathfrak{z}\right)\right|_{p}\leq1$. Now, writing $\mathfrak{z}=\sum_{n=0}^{\infty}c_{n}p^{n}$: \begin{align*} H_{j}\left(\mathfrak{z}\right) & =\frac{a_{j}\sum_{n=0}^{\infty}c_{n}p^{n}+b_{j}}{d_{j}}\\ & =\frac{a_{j}c_{0}+b_{j}}{d_{j}}+\frac{1}{d_{j}}\sum_{n=1}^{\infty}c_{n}p^{n}\\ \left(c_{0}=\left[\mathfrak{z}\right]_{p}\right); & =H_{j}\left(\left[\mathfrak{z}\right]_{p}\right)+\frac{p}{d_{j}}\underbrace{\sum_{n=1}^{\infty}c_{n}p^{n-1}}_{\textrm{call this }\mathfrak{y}} \end{align*} Because $H$ is a $p$-Hydra map, $d_{j}$ must be a divisor of $p$. The primality of $p$ then forces $d_{j}$ to be either $1$ or $p$. In either case, we have that $p\mathfrak{y}/d_{j}$ is an element of $\mathbb{Z}_{p}$. Our contradictory assumption $\left|H_{j}\left(\mathfrak{z}\right)\right|_{p}\leq1$ tells us that $H_{j}\left(\mathfrak{z}\right)$ is also in $\mathbb{Z}_{p}$, and so: \begin{equation} H_{j}\left(\left[\mathfrak{z}\right]_{p}\right)=H_{j}\left(\mathfrak{z}\right)-\frac{p\mathfrak{y}}{d_{j}}\in\mathbb{Z}_{p} \end{equation} Since $H$ was given to be integral, $j\neq\left[\mathfrak{z}\right]_{p}$ implies $H_{j}\left(\left[\mathfrak{z}\right]_{p}\right)\notin\mathbb{Z}$. As such, $H_{j}\left(\left[\mathfrak{z}\right]_{p}\right)$ is a $p$-adic integer which is \emph{not }a rational integer. Since $H_{j}\left(\left[\mathfrak{z}\right]_{p}\right)$ is a non-integer rational number which is a $p$-adic integer, the denominator of $H_{j}\left(\left[\mathfrak{z}\right]_{p}\right)=\frac{a_{j}\left[\mathfrak{z}\right]_{p}+b_{j}}{d_{j}}$ must be divisible by some prime $q\neq p$, and hence, $q\mid d_{j}$. However, we saw that $p$ being prime forced $d_{j}\in\left\{ 1,p\right\} $\textemdash this is impossible! Thus, it must be that $H$ is proper. Q.E.D. \begin{lem} \label{lem:wrong values lemma}Let $H$ be a proper $p$-Hydra map. All wrong values of $H$ are elements of $\mathbb{Q}_{p}\backslash\mathbb{Z}_{p}$. \end{lem} Proof: Let $H$ be a proper $p$-Hydra map, let $\mathfrak{z}\in\mathbb{Z}_{p}$, and let $i\in\mathbb{Z}/p\mathbb{Z}$ be such that $\left[\mathfrak{z}\right]_{p}\neq i$. Then, by definition of properness, the quantity: \begin{equation} H_{i}\left(\mathfrak{z}\right)=\frac{a_{i}\mathfrak{z}+b_{i}}{d_{i}} \end{equation} has $p$-adic absolute value $>1$. By \textbf{Proposition \ref{prop:Q_p / Z_p prop}}, this then forces $H_{\mathbf{j}}\left(H_{i}\left(\mathfrak{z}\right)\right)$ to be an element of $\mathbb{Q}_{p}\backslash\mathbb{Z}_{p}$ for all $\mathbf{j}\in\textrm{String}\left(p\right)$. Since every wrong value with seed $\mathfrak{z}$ is of the form $H_{\mathbf{j}}\left(H_{i}\left(\mathfrak{z}\right)\right)$ for some $\mathbf{j}\in\textrm{String}\left(p\right)$, some $\mathfrak{z}\in\mathbb{Z}_{p}$, and some $i\in\mathbb{Z}/p\mathbb{Z}$ for which $\left[\mathfrak{z}\right]_{p}\neq i$, this shows that every wrong value of $H$ is in $\mathbb{Q}_{p}\backslash\mathbb{Z}_{p}$. So, $H$ is proper. Q.E.D. \begin{lem} \label{lem:properness lemma}Let $H$ be a proper $p$-Hydra map, let $\mathfrak{z}\in\mathbb{Z}_{p}$, and let $\mathbf{j}\in\textrm{String}\left(p\right)$. If $H_{\mathbf{j}}\left(\mathfrak{z}\right)=\mathfrak{z}$, then $H^{\circ\left|\mathbf{j}\right|}\left(\mathfrak{z}\right)=\mathfrak{z}$. \end{lem} Proof: Let $H$, $\mathfrak{z}$, and $\mathbf{j}$ be as given. By way of contradiction, suppose $H^{\circ\left|\mathbf{j}\right|}\left(\mathfrak{z}\right)\neq\mathfrak{z}$. But then $\mathfrak{z}=H_{\mathbf{j}}\left(\mathfrak{z}\right)$ implies $H^{\circ\left|\mathbf{j}\right|}\left(\mathfrak{z}\right)\neq H_{\mathbf{j}}\left(\mathfrak{z}\right)$. Hence, $H_{\mathbf{j}}\left(\mathfrak{z}\right)$ is a wrong value of $H$ with seed $\mathfrak{z}$. \textbf{Lemma} \ref{lem:wrong values lemma} then forces $H_{\mathbf{j}}\left(\mathfrak{z}\right)\in\mathbb{Q}_{p}\backslash\mathbb{Z}_{p}$. However, $H_{\mathbf{j}}\left(\mathfrak{z}\right)=\mathfrak{z}$, and $\mathfrak{z}$ was given to be in $\mathbb{Z}_{p}$. This is impossible! Consequently, $H_{\mathbf{j}}\left(\mathfrak{z}\right)=\mathfrak{z}$ implies $H^{\circ\left|\mathbf{j}\right|}\left(\mathfrak{z}\right)=\mathfrak{z}$. Q.E.D. \vphantom{} Next, we need an essentially trivial observation: if $H_{\mathbf{j}}\left(n\right)=n$, $H\left(0\right)=0$, and $\left|\mathbf{j}\right|\geq2$, then the $p$-ity vector $\mathbf{j}$ must contain \emph{at least one} non-zero entry. \begin{prop} \label{prop:essentially trivial observation}Let $H$ be a non-degenerate $p$-Hydra map which fixes $0$, let $n\in\mathbb{Z}$ be a periodic point of $H$, and let $\Omega$ be the cycle of $H$ which contains $n$. Letting $\mathbf{j}=\left(j_{1},\ldots,j_{\left|\Omega\right|}\right)\in\left(\mathbb{Z}/p\mathbb{Z}\right)^{\left|\Omega\right|}$ be the unique string so that: \begin{equation} H_{\mathbf{j}}\left(n\right)=H^{\circ\left|\Omega\right|}\left(n\right)=n \end{equation} we have that either $\left|\Omega\right|=1$ (i.e., $n$ is a fixed point of $H$), or, there is an $\ell\in\left\{ 1,\ldots,\left|\Omega\right|\right\} $ so that $j_{\ell}\neq0$. \end{prop} Proof: Let $H$, $n$, and $\mathbf{j}$ be as given, and suppose $\left|\Omega\right|\geq2$. Since $H$ is non-degenerate, $a_{j}\neq1$ for any $j\in\left\{ 1,\ldots,p-1\right\} $. Now, \emph{by way of contradiction}, suppose that $j_{\ell}=0$ for every $\ell$. Since $\mathbf{j}$ contains only $0$s, we can write: \begin{equation} n=H_{\mathbf{j}}\left(n\right)=H_{0}^{\circ\left|\Omega\right|}\left(n\right) \end{equation} Because $H_{0}$ is an affine linear map of the form $H_{0}\left(x\right)=\frac{a_{0}x+b_{0}}{d_{0}}$ where $b_{0}=0$, $x=0$ is the one and only fixed point of $H_{0}$ in $\mathbb{R}$. In fact, the same is true of $H_{0}^{\circ m}\left(x\right)$ for all $m\geq1$. Thus, $n=H_{0}^{\circ\left|\Omega\right|}\left(n\right)$ forces $n=0$. So, $H\left(n\right)=H_{0}\left(n\right)=n$, which is to say, the cycle $\Omega$ to which $n$ belongs contains only one element: $n$ itself. This contradicts the assumption that $\left|\Omega\right|\geq2$. Q.E.D. \vphantom{} Now, at last, we can prove the \textbf{Correspondence Principle}. This is done in four different ways, the last of which (\textbf{Corollary \ref{cor:CP v4}}) is the simplest and most elegant. The first version\textemdash \textbf{Theorem \ref{thm:CP v1}}, proved below\textemdash is more general than \textbf{Corollary \ref{cor:CP v4}}, which is proved by using \textbf{Theorem \ref{thm:CP v1} }as a stepping stone. Whereas \textbf{Corollary \ref{cor:CP v4}} establishes a correspondence between the values attained by $\chi_{H}$ and the periodic \emph{points }of $H$, \textbf{Theorem \ref{thm:CP v1} }establishes a slightly weaker correspondence between the values attained by $\chi_{H}$ and the \emph{cycles }of $H$. \textbf{Corollary \ref{cor:CP v4}} refines this correspondence to one with individual periodic points of $\chi_{H}$. The other two versions of the Correspondence Principle are closer to the Bhm-Sontacchi Criterion, with \textbf{Corollary \ref{cor:CP v3}} providing Bhm-Sontacchi-style diophantine equations for the non-zero periodic points of $H$. In that respect, \textbf{Corollary \ref{cor:CP v3} }is not entirely new; a similar diophantine equation characterization of periodic points for certain general Collatz-type maps on $\mathbb{Z}$ can be found in Matthews' slides \cite{Matthews' slides}. The main innovations of my approach is the reformulation of these diophantine equations in terms of the function $\chi_{H}$, along with the tantalizing half-complete characterization $\chi_{H}$ provides for the divergent trajectories of $H$ (\textbf{Theorem \ref{thm:Divergent trajectories come from irrational z}}). \begin{thm}[\textbf{Correspondence Principle, Ver. 1}] \label{thm:CP v1}Let\index{Correspondence Principle}$H$ be semi-basic. Then: \vphantom{} I. Let $\Omega$ be any cycle of $H$ in $\mathbb{Z}$, with $\left|\Omega\right|\geq2$. Then, there exist $x\in\Omega$ and $n\in\mathbb{N}_{1}$ so that: \begin{equation} \chi_{H}\left(B_{p}\left(n\right)\right)\overset{\mathbb{Z}_{q_{H}}}{=}x \end{equation} That is, there is an $n$ so that the infinite series defining $\chi_{H}\left(B_{p}\left(n\right)\right)$ converges\footnote{This series is obtained by evaluating $\chi_{H}\left(\left[B_{p}\left(n\right)\right]_{p^{m}}\right)$ using \textbf{Proposition \ref{prop:Explicit formula for Chi_H of bold j}}, and then taking the limit in $\mathbb{Z}_{q}$ as $m\rightarrow\infty$.} in $\mathbb{Z}_{q_{H}}$ to $x$. \vphantom{} II. Suppose also that $H$ is integral, and let $n\in\mathbb{N}_{1}$. If the rational number $\chi_{H}\left(B_{p}\left(n\right)\right)$ is in $\mathbb{Z}_{p}$, then $\chi_{H}\left(B_{p}\left(n\right)\right)$ is a periodic point of $H$ in $\mathbb{Z}_{p}$. In particular, if $\chi_{H}\left(B_{p}\left(n\right)\right)$ is in $\mathbb{Z}$, then $\chi_{H}\left(B_{p}\left(n\right)\right)$ is a periodic point of $H$ in $\mathbb{Z}$. \end{thm} Proof: I. Let $\Omega$ be a cycle of $H$ in $\mathbb{Z}$, with $\left|\Omega\right|\geq2$. Given any periodic point $x\in\Omega$ of $H$, there is going to be a string $\mathbf{j}\in\textrm{String}\left(p\right)$ of length $\left|\Omega\right|$ so that $H_{\mathbf{j}}\left(x\right)=x$. In particular, since $\left|\Omega\right|\geq2$,\textbf{ Proposition \ref{prop:essentially trivial observation}} tells us that $\mathbf{j}$ contains \emph{at least one} non-zero entry. A moment's thought shows that for any $x^{\prime}\in\Omega$ \emph{other }than $x$, the entries of the string $\mathbf{i}$ for which $H_{\mathbf{i}}\left(x^{\prime}\right)=x^{\prime}$ must be a cyclic permutation of the entries of $\mathbf{j}$. For example, for the Shortened Collatz Map and the cycle $\left\{ 1,2\right\} $, our branches are: \begin{align*} H_{0}\left(x\right) & =\frac{x}{2}\\ H_{1}\left(x\right) & =\frac{3x+1}{2} \end{align*} The composition sequence which sends $1$ back to itself first applies $H_{1}$ to $1$, followed by $H_{0}$: \begin{equation} H_{0}\left(H_{1}\left(1\right)\right)=H_{0}\left(2\right)=1 \end{equation} On the other hand, the composition sequence which sends $2$ back to itself first applies $H_{0}$ to $2$, followed by $H_{1}$: \begin{equation} H_{1}\left(H_{0}\left(2\right)\right)=H_{1}\left(1\right)=2 \end{equation} and the strings $\left(0,1\right)$ and $\left(1,0\right)$ are cyclic permutations of one another. In this way, note that there must then exist an $x\in\Omega$ and a $\mathbf{j}$ (for which $H_{\mathbf{j}}\left(x\right)=x$) so that \emph{the right-most entry of $\mathbf{j}$ is non-zero}. From this point forward, we will fix $x$ and $\mathbf{j}$ so that this condition is satisfied. That being done, the next observation to make is that because $H_{\mathbf{j}}$ sends $x$ to $x$, further applications of $H_{\mathbf{j}}$ will keep sending $x$ back to $x$. Expressing this in terms of concatenation yields: \begin{equation} x=H_{\mathbf{j}}^{\circ m}\left(x\right)=\underbrace{\left(H_{\mathbf{j}}\circ\cdots\circ H_{\mathbf{j}}\right)}_{m\textrm{ times}}\left(x\right)=H_{\mathbf{j}^{\wedge m}}\left(x\right) \end{equation} Writing the affine map $H_{\mathbf{j}^{\wedge m}}$ in affine form gives us: \begin{equation} x=H_{\mathbf{j}^{\wedge m}}\left(x\right)=M_{H}\left(\mathbf{j}^{\wedge m}\right)x+\chi_{H}\left(\mathbf{j}^{\wedge m}\right)=\left(M_{H}\left(\mathbf{j}\right)\right)^{m}x+\chi_{H}\left(\mathbf{j}^{\wedge m}\right) \end{equation} where the right-most equality follows from $M_{H}$'s concatenation identity (\textbf{Proposition \ref{prop:M_H concatenation identity}}). Since $\mathbf{j}$ the right-most entry of $\mathbf{j}$ is non-zero, \textbf{Proposition \ref{prop:q-adic behavior of M_H of j as the number of non-zero digits tends to infinity}} tells us that $\left|M_{H}\left(\mathbf{j}\right)\right|_{q_{H}}<1$. As such, $\left|\left(M_{H}\left(\mathbf{j}\right)\right)^{m}\right|_{q}\rightarrow0$ as $m\rightarrow\infty$. So, taking limits as $m\rightarrow\infty$, we get: \begin{equation} x\overset{\mathbb{Z}_{q_{H}}}{=}\lim_{m\rightarrow\infty}\left(\left(M_{H}\left(\mathbf{j}\right)\right)^{m}x+\chi_{H}\left(\mathbf{j}^{\wedge m}\right)\right)\overset{\mathbb{Z}_{q_{H}}}{=}\lim_{m\rightarrow\infty}\chi_{H}\left(\mathbf{j}^{\wedge m}\right) \end{equation} Now, let $n$ be the integer represented by $\mathbf{j}$. Since $\mathbf{j}$'s right-most entry is non-zero, $\mathbf{j}$ is then the \emph{shortest} string representing $n$; moreover, $n$ is non-zero. In particular, we have that $\lambda_{p}\left(n\right)=\left|\mathbf{j}\right|>0$, and so, using \textbf{Proposition \ref{prop:Concatenation exponentiation}},\textbf{ }we can write: \begin{equation} \mathbf{j}^{\wedge m}\sim n\frac{1-p^{m\lambda_{p}\left(n\right)}}{1-p^{\lambda_{p}\left(n\right)}},\textrm{ }\forall m\geq1 \end{equation} So, like in the proof of \textbf{Lemma \ref{lem:Chi_H o B_p functional equation}}, we then have: \begin{align*} x & \overset{\mathbb{Z}_{q_{H}}}{=}\lim_{m\rightarrow\infty}\chi_{H}\left(\mathbf{j}^{\wedge m}\right)\\ & \overset{\mathbb{Z}_{q_{H}}}{=}\lim_{m\rightarrow\infty}\chi_{H}\left(n\frac{1-p^{m\lambda_{p}\left(n\right)}}{1-p^{\lambda_{p}\left(n\right)}}\right)\\ & \overset{\mathbb{Z}_{q_{H}}}{=}\chi_{H}\left(\frac{n}{1-p^{\lambda_{p}\left(n\right)}}\right)\\ & =\chi_{H}\left(B_{p}\left(n\right)\right) \end{align*} This proves the existence of the desired $x$ and $n$. \vphantom{} II. Suppose $H$ is integral. First, before we assume the hypotheses of (II), let us do a brief computation. \begin{claim} \label{claim:2.2}Let $n$ be any integer $\geq1$, and let $\mathbf{j}\in\textrm{String}\left(p\right)$ be the shortest string representing $n$. Then, $H_{\mathbf{j}}$ is a continuous map $\mathbb{Z}_{q_{H}}\rightarrow\mathbb{Z}_{q_{H}}$, and, moreover, the $q_{H}$-adic integer $\chi_{H}\left(B_{p}\left(n\right)\right)$ is fixed by $H_{\mathbf{j}}$. Proof of claim: Let $n$ and $\mathbf{j}$ be as given. Using $\chi_{H}$'s concatenation identity (\textbf{Lemma \ref{lem:Chi_H concatenation identity}}), we can write: \begin{equation} \chi_{H}\left(\mathbf{j}^{\wedge k}\right)=H_{\mathbf{j}}\left(\chi_{H}\left(\mathbf{j}^{\wedge\left(k-1\right)}\right)\right)=\cdots=H_{\mathbf{j}^{\wedge\left(k-1\right)}}\left(\chi_{H}\left(\mathbf{j}\right)\right)\label{eq:Concatenation identity for Chi_H} \end{equation} By \textbf{Proposition \ref{prop:Concatenation exponentiation}}, we know that $B_{p}\left(n\right)$ represents $\lim_{k\rightarrow\infty}\mathbf{j}^{\wedge k}$ (i.e. $\textrm{DigSum}_{p}\left(\lim_{k\rightarrow\infty}\mathbf{j}^{\wedge k}\right)=B_{p}\left(n\right)$). In particular, we have that: \begin{equation} \left[B_{p}\left(n\right)\right]_{p^{k}}=\mathbf{j}^{\wedge k}=n\frac{1-p^{k\lambda_{p}\left(n\right)}}{1-p^{\lambda_{p}\left(n\right)}} \end{equation} Letting $k\rightarrow\infty$, the limit (\ref{eq:Rising Continuity Formula for Chi_H}) tells us that: \begin{equation} \lim_{k\rightarrow\infty}H_{\mathbf{j}^{\wedge\left(k-1\right)}}\left(\chi_{H}\left(n\right)\right)\overset{\mathbb{Z}_{q_{H}}}{=}\chi_{H}\left(\frac{n}{1-p^{\lambda_{p}\left(n\right)}}\right)=\chi_{H}\left(B_{p}\left(n\right)\right)\label{eq:Iterating H_bold-j on Chi_H} \end{equation} The heuristic here is that: \begin{equation} H_{\mathbf{j}}\left(\lim_{k\rightarrow\infty}H_{\mathbf{j}^{\wedge\left(k-1\right)}}\right)=\lim_{k\rightarrow\infty}H_{\mathbf{j}^{\wedge\left(k-1\right)}} \end{equation} and, hence, that $\chi_{H}\left(B_{p}\left(n\right)\right)$ will be fixed by $H_{\mathbf{j}}$. To make this rigorous, note that $\gcd\left(a_{j},p\right)=1$ for all $j$, because $H$ is semi-basic. As such, for any element of $\textrm{String}\left(p\right)$\textemdash such as our $\mathbf{j}$\textemdash the quantities $H_{\mathbf{j}}^{\prime}\left(0\right)$ and $H_{\mathbf{j}}\left(0\right)$ are then rational numbers which lie in $\mathbb{Z}_{q_{H}}$. This guarantees that the function $\mathfrak{z}\mapsto H_{\mathbf{j}}^{\prime}\left(0\right)\mathfrak{z}+H_{\mathbf{j}}\left(0\right)$ (which is, of course $H_{\mathbf{j}}\left(\mathfrak{z}\right)$) is then a well-defined \emph{continuous} map $\mathbb{Z}_{q_{H}}\rightarrow\mathbb{Z}_{q_{H}}$. Applying $H_{\mathbf{j}}$ to $\chi_{H}\left(B_{p}\left(n\right)\right)$, we obtain: \begin{align*} H_{\mathbf{j}}\left(\chi_{H}\left(B_{p}\left(n\right)\right)\right) & \overset{\mathbb{Z}_{q_{H}}}{=}H_{\mathbf{j}}\left(\lim_{k\rightarrow\infty}\chi_{H}\left(\mathbf{j}^{\wedge\left(k-1\right)}\right)\right)\\ & \overset{\mathbb{Z}_{q_{H}}}{=}\lim_{k\rightarrow\infty}H_{\mathbf{j}}\left(\chi_{H}\left(\mathbf{j}^{\wedge\left(k-1\right)}\right)\right)\\ & \overset{\mathbb{Z}_{q_{H}}}{=}\lim_{k\rightarrow\infty}H_{\mathbf{j}}\left(H_{\mathbf{j}^{\wedge\left(k-1\right)}}\left(0\right)\right)\\ & \overset{\mathbb{Z}_{q_{H}}}{=}\lim_{k\rightarrow\infty}H_{\mathbf{j}^{\wedge k}}\left(0\right)\\ & \overset{\mathbb{Z}_{q_{H}}}{=}\lim_{k\rightarrow\infty}\chi_{H}\left(\mathbf{j}^{\wedge k}\right)\\ & \overset{\mathbb{Z}_{q_{H}}}{=}\chi_{H}\left(B_{p}\left(n\right)\right) \end{align*} The interchange of $H_{\mathbf{j}}$ and $\lim_{k\rightarrow\infty}$ on the second line is justified by the continuity of $H_{\mathbf{j}}$ on $\mathbb{Z}_{q_{H}}$. This proves the claim. \end{claim} \vphantom{} Now, let us actually \emph{assume} that $n$ satisfies the hypotheses of (II): suppose\emph{ }$\chi_{H}\left(B_{p}\left(n\right)\right)\in\mathbb{Z}_{p}$. By \textbf{Claim \ref{claim:2.2}}, we know that $H_{\mathbf{j}}\left(\chi_{H}\left(B_{p}\left(n\right)\right)\right)=\chi_{H}\left(B_{p}\left(n\right)\right)$. Because $H$ was given to be integral, the primality of $p$ and $H$'s semi-basicness guarantee that $H$ is proper (\textbf{Lemma \ref{lem:integrality lemma}}), and so, we can apply \textbf{Lemma \ref{lem:properness lemma}} to conclude that: \begin{equation} H^{\circ\left|\mathbf{j}\right|}\left(\chi_{H}\left(B_{p}\left(n\right)\right)\right)=\chi_{H}\left(B_{p}\left(n\right)\right) \end{equation} where $\left|\mathbf{j}\right|=\lambda_{p}\left(n\right)\geq1$. So, $\chi_{H}\left(B_{p}\left(n\right)\right)$ is a periodic point of $H$ in $\mathbb{Z}_{p}$. In particular, if $\chi_{H}\left(B_{p}\left(n\right)\right)$ is in $\mathbb{Z}$, then every element of the cycle generated by applying the $p$-adic extension of $H$ to $\chi_{H}\left(B_{p}\left(n\right)\right)$ is an element of $\mathbb{Z}$, and $\chi_{H}\left(B_{p}\left(n\right)\right)$ is then a periodic point of $H$ in $\mathbb{Z}$. Q.E.D. \vphantom{} Using the functional equation in \textbf{Lemma \ref{lem:Chi_H o B_p functional equation}}, we can restate the Correspondence Principle in terms of $\chi_{H}$ and $M_{H}$. \begin{cor}[\textbf{Correspondence Principle, Ver. 2}] \label{cor:CP v2}\index{Correspondence Principle} Suppose that $H$ is semi-basic. \vphantom{} I. Let $\Omega\subseteq\mathbb{Z}$ be any cycle of $H$. Then, viewing $\Omega\subseteq\mathbb{Z}$ as a subset of $\mathbb{Z}_{q_{H}}$, the intersection $\chi_{H}\left(\mathbb{Z}_{p}\right)\cap\Omega$ is non-empty. Moreover, for every $x\in\chi_{H}\left(\mathbb{Z}_{p}\right)\cap\Omega$, there is an $n\in\mathbb{N}_{1}$ so that: \begin{equation} x=\frac{\chi_{H}\left(n\right)}{1-M_{H}\left(n\right)} \end{equation} \vphantom{} II. Suppose in addition that $H$ is integral, and let $n\in\mathbb{N}_{1}$. If the quantity $x$ given by: \begin{equation} x=\frac{\chi_{H}\left(n\right)}{1-M_{H}\left(n\right)} \end{equation} is a $p$-adic integer, then $x$ is a periodic point of $H$ in $\mathbb{Z}_{p}$; if $x$ is in $\mathbb{Z}$, then $x$ is a periodic point of $H$ in $\mathbb{Z}$. Moreover, if $x\in\mathbb{Z}$, then $x$ is positive if and only if $M_{H}\left(n\right)<1$, and $x$ is negative if and only if $M_{H}\left(n\right)>1$. \end{cor} Proof: Re-write the results of \textbf{Theorem \ref{thm:CP v1}} using \textbf{Lemma \ref{lem:Chi_H o B_p functional equation}}. The positivity/negativity of $x$ stipulated in (II) follows by noting that $\chi_{H}\left(n\right)$ and $M_{H}\left(n\right)$ are positive rational numbers for all $n\in\mathbb{N}_{1}$. Q.E.D. \vphantom{} Next, we have Bhm-Sontacchi-style diophantine equations characterizing of $H$'s periodic points. \begin{cor}[\textbf{Correspondence Principle, Ver. 3}] \label{cor:CP v3}Let $H$ be a semi-basic $p$-Hydra map. \index{Bhm-Sontacchi criterion}\index{diophantine equation}Then: \vphantom{} I. Let $\Omega$ be a cycle of $H$ in $\mathbb{Z}$ containing at least two elements. Then, there is an $x\in\Omega$, and a $\mathbf{j}\in\textrm{String}\left(p\right)$ of length $\geq2$ so that: \begin{equation} x=\frac{\sum_{n=1}^{\left|\mathbf{j}\right|}\frac{b_{j_{n}}}{a_{j_{n}}}\left(\prod_{k=1}^{n}\mu_{j_{k}}\right)p^{\left|\mathbf{j}\right|-n}}{p^{\left|\mathbf{j}\right|}-\prod_{k=1}^{\left|\mathbf{j}\right|}\mu_{j_{k}}}\label{eq:Generalized Bohm-Sontacchi criterion-1} \end{equation} \vphantom{} II. Suppose $H$ is integral, and let $x\left(\mathbf{j}\right)=x\left(j_{1},\ldots,j_{N}\right)$ denote the quantity: \begin{equation} x\left(\mathbf{j}\right)\overset{\textrm{def}}{=}\frac{\sum_{n=1}^{\left|\mathbf{j}\right|}\frac{b_{j_{n}}}{a_{j_{n}}}\left(\prod_{k=1}^{n}\mu_{j_{k}}\right)p^{\left|\mathbf{j}\right|-n}}{p^{\left|\mathbf{j}\right|}-\prod_{k=1}^{\left|\mathbf{j}\right|}\mu_{j_{k}}},\textrm{ }\forall\mathbf{j}\in\textrm{String}\left(p\right)\label{eq:definition of x of the js (bold version)} \end{equation} If $\mathbf{j}\in\textrm{String}\left(p\right)$ has $\left|\mathbf{j}\right|\geq2$ and makes $x\left(\mathbf{j}\right)\in\mathbb{Z}$, then $x\left(\mathbf{j}\right)$ is a periodic point of $H$. \end{cor} Proof: Use \textbf{Propositions \ref{prop:Explicit Formulas for M_H}} and \textbf{\ref{prop:Explicit formula for Chi_H of bold j}} on (I) and (II) of \textbf{Corollary \ref{cor:CP v2}}. Q.E.D. \vphantom{} Finally, we can state and prove the most elegant version of the Correspondence Principle for integral semi-basic Hydra maps. This is also the most \emph{powerful }version of the Correspondence Principle, because it will allow us to establish a connection between $\chi_{H}$ and $H$'s \textbf{\emph{divergent points}}. \begin{cor}[\textbf{Correspondence Principle, Ver. 4}] \label{cor:CP v4} \index{Correspondence Principle}Let $H$ be an integral semi-basic $p$-Hydra map. Then, the set of all non-zero periodic points of $H$ in $\mathbb{Z}$ is equal to $\mathbb{Z}\cap\chi_{H}\left(\mathbb{Q}\cap\mathbb{Z}_{p}^{\prime}\right)$, where $\chi_{H}\left(\mathbb{Q}\cap\mathbb{Z}_{p}^{\prime}\right)$ is viewed as a subset of $\mathbb{Z}_{q_{H}}$. \end{cor} \begin{rem} The implication ``If $x\in\mathbb{Z}\backslash\left\{ 0\right\} $ is a periodic point, then $x\in\chi_{H}\left(\mathbb{Q}\cap\mathbb{Z}_{p}^{\prime}\right)$'' \emph{does not }require $H$ to be integral. \end{rem} Proof: First, note that $p$ is prime and since $H$ is integral and semi-basic, \textbf{Lemma \ref{lem:integrality lemma}} tell us that $H$ is proper. I. Let $x$ be a non-zero periodic point of $H$, and let $\Omega$ be the unique cycle of $H$ in $\mathbb{Z}$ which contains $x$. By Version 1 of the Correspondence Principle (\textbf{Theorem \ref{thm:CP v1}}), there exists a $y\in\Omega$ and a $\mathfrak{z}=B_{p}\left(n\right)\subset\mathbb{Z}_{p}$ (for some $n\in\mathbb{N}_{1}$) so that $\chi_{H}\left(\mathfrak{z}\right)=y$. Since $y$ is in $\Omega$, there is an $k\geq1$ so that $x=H^{\circ k}\left(y\right)$. In particular, there is a \emph{unique} length $k$ string $\mathbf{i}\in\textrm{String}\left(p\right)$ so that $H_{\mathbf{i}}\left(y\right)=H^{\circ k}\left(y\right)=x$. Now, let $\mathbf{j}\in\textrm{String}_{\infty}\left(p\right)$ represent $\mathfrak{z}$; note that $\mathbf{j}$ is infinite and that its entries are periodic. Using $\chi_{H}$'s concatenation identity (\textbf{Lemma \ref{lem:Chi_H concatenation identity}}), we can write: \begin{equation} x=H_{\mathbf{i}}\left(y\right)=H_{\mathbf{i}}\left(\chi_{H}\left(\mathfrak{z}\right)\right)=\chi_{H}\left(\mathbf{i}\wedge\mathbf{j}\right) \end{equation} Next, let $\mathfrak{x}$ denote the $p$-adic integer $\mathfrak{x}$ whose sequence of $p$-adic digits is $\mathbf{i}\wedge\mathbf{j}$; note that $\mathfrak{x}$ is then \emph{not} an element of $\mathbb{N}_{0}$. By the above, we have that $\chi_{H}\left(\mathfrak{x}\right)=x$, and hence that $x\in\mathbb{Z}\cap\chi_{H}\left(\mathbb{Z}_{p}\right)$. Finally, since $\mathfrak{z}=B_{p}\left(n\right)$, its $p$-adic digits are periodic, which forces $\mathfrak{z}$ to be an element of $\mathbb{Q}\cap\mathbb{Z}_{p}^{\prime}$. Indeed, $\mathfrak{z}$ is not in $\mathbb{N}_{0}$ because $n\geq1$, and $B_{p}\left(n\right)\in\mathbb{N}_{0}$ if and only if $n=0$. So, letting $m$ be the rational integer represented by length-$k$ string $\mathbf{i}$, we have: \begin{equation} \mathfrak{x}\sim\mathbf{i}\wedge\mathbf{j}\sim m+p^{\lambda_{p}\left(m\right)}\mathfrak{z} \end{equation} This shows that $\mathfrak{x}\in\mathbb{Q}\cap\mathbb{Z}_{p}^{\prime}$, and hence, that $x=\chi_{H}\left(\mathfrak{x}\right)\in\mathbb{Z}\cap\chi_{H}\left(\mathbb{Q}\cap\mathbb{Z}_{p}^{\prime}\right)$. \vphantom{} II. Suppose $x\in\mathbb{Z}\cap\chi_{H}\left(\mathbb{Q}\cap\mathbb{Z}_{p}^{\prime}\right)$, with $x=\chi_{H}\left(\mathfrak{z}\right)$ for some $\mathfrak{z}\in\mathbb{Q}\cap\mathbb{Z}_{p}^{\prime}$. As a rational number which is both a $p$-adic integer \emph{and} not an element of $\mathbb{N}_{0}$, the $p$-adic digits of $\mathfrak{z}$ are \emph{eventually} periodic. As such, there are integers $m$ and $n$ (with $n\neq0$) so that: \begin{equation} \mathfrak{z}=m+p^{\lambda_{p}\left(m\right)}B_{p}\left(n\right) \end{equation} Here, $n$'s $p$-adic digits generate the periodic part of $\mathfrak{z}$'s digits, while $m$'s $p$-adic digits are the finite-length sequence in $\mathfrak{z}$'s digits that occurs before the periodicity sets in. Now, let $\mathbf{i}$ be the finite string representing $m$, and let $\mathbf{j}$ be the infinite string representing $B_{p}\left(n\right)$. Then, $\mathfrak{z}=\mathbf{i}\wedge\mathbf{j}$. So, by \textbf{Lemmata \ref{lem:Chi_H concatenation identity}} and \textbf{\ref{lem:Chi_H o B_p functional equation}} (the concatenation identity and $\chi_{H}\circ B_{p}$ functional equation, respectively): \begin{equation} x=\chi_{H}\left(\mathbf{i}\wedge\mathbf{j}\right)=H_{\mathbf{i}}\left(\chi_{H}\left(\mathbf{j}\right)\right)=H_{\mathbf{i}}\left(\chi_{H}\left(B_{p}\left(n\right)\right)\right)=H_{\mathbf{i}}\left(\underbrace{\frac{\chi_{H}\left(n\right)}{1-M_{H}\left(n\right)}}_{\textrm{call this }y}\right) \end{equation} where $y\overset{\textrm{def}}{=}\frac{\chi_{H}\left(n\right)}{1-M_{H}\left(n\right)}$ is a rational number. \begin{claim} $\left|y\right|_{p}\leq1$. Proof of claim: First, since $y$ is a rational number, it lies in $\mathbb{Q}_{p}$. So, by way of contradiction, suppose $\left|y\right|_{p}>1$. By \textbf{Lemma \ref{lem:wrong values lemma}}, because $H$ is proper, every branch specified by $\mathbf{i}$ maps $\mathbb{Q}_{p}\backslash\mathbb{Z}_{p}$ into $\mathbb{Q}_{p}\backslash\mathbb{Z}_{p}$. Consequently, $\left|H_{\mathbf{i}}\left(y\right)\right|_{p}>1$. However, $H_{\mathbf{i}}\left(y\right)=x$, and $x$ is a rational integer; hence, $1<\left|H_{\mathbf{i}}\left(y\right)\right|_{p}=\left|x\right|_{p}\leq1$. This is impossible! So, it must be that $\left|y\right|_{p}\leq1$. This proves the claim. \end{claim} \begin{claim} \label{claim:2.3}$x=H^{\circ\left|\mathbf{i}\right|}\left(y\right)$ Proof of claim: Suppose the equality failed. Then $H_{\mathbf{i}}\left(y\right)=x\neq H^{\circ\left|\mathbf{i}\right|}\left(y\right)$, and so $x=H_{\mathbf{i}}\left(y\right)$ is a wrong value of $H$ with seed $y$. Because $H$ is proper,\textbf{ Lemma \ref{lem:wrong values lemma}} forces $\left|x\right|_{p}=\left|H_{\mathbf{i}}\left(y\right)\right|_{p}>1$. However, $\left|x\right|_{p}\leq1$. This is just as impossible as it was in the previous paragraph. This proves the claim. \end{claim} \vphantom{} Finally, let $\mathbf{v}$ be the shortest string representing $n$, so that $\mathbf{j}$ (the string representing $B_{p}\left(n\right)$) is obtained by concatenating infinitely many copies of $\mathbf{v}$. Because $q_{H}$ is co-prime to all the $d_{j}$s, \emph{note that $H_{\mathbf{v}}$ is continuous on $\mathbb{Z}_{q_{H}}$}. As such: \begin{equation} \chi_{H}\left(B_{p}\left(n\right)\right)\overset{\mathbb{Z}_{q_{H}}}{=}\lim_{k\rightarrow\infty}\chi_{H}\left(\mathbf{v}^{\wedge k}\right) \end{equation} implies: \begin{align*} H_{\mathbf{v}}\left(\chi_{H}\left(B_{p}\left(n\right)\right)\right) & \overset{\mathbb{Z}_{q_{H}}}{=}\lim_{k\rightarrow\infty}H_{\mathbf{v}}\left(\chi_{H}\left(\mathbf{v}^{\wedge k}\right)\right)\\ & \overset{\mathbb{Z}_{q_{H}}}{=}\lim_{k\rightarrow\infty}\chi_{H}\left(\mathbf{v}^{\wedge\left(k+1\right)}\right)\\ & \overset{\mathbb{Z}_{q_{H}}}{=}\chi_{H}\left(B_{p}\left(n\right)\right) \end{align*} Hence, $H_{\mathbf{v}}\left(y\right)=y$. Since $\left|y\right|_{p}\leq1$, the propriety of $H$ allows us to apply \textbf{Lemma \ref{lem:properness lemma}} to conclude $H_{\mathbf{v}}\left(y\right)=H^{\circ\left|\mathbf{v}\right|}\left(y\right)$. Thus, $y$ is a periodic point of $H:\mathbb{Z}_{p}\rightarrow\mathbb{Z}_{p}$. By \textbf{Claim \ref{claim:2.3}}, $H$ iterates $y$ to $x$. Since $y$ is a periodic point of $H$ in $\mathbb{Z}_{p}$, this forces $x$ and $y$ to belong to the same cycle of $H$ in $\mathbb{Z}_{p}$, with $x$ being a periodic point of $H$. As such, just as $H$ iterates $y$ to $x$, so too does $H$ iterate $x$ to $y$. Likewise, since $x$ is a rational integer, so too is $y$. Thus, $x$ belongs to a cycle of $H$ in $\mathbb{Z}$, as desired. Q.E.D. \begin{example} To illustrate the Correspondence Principle in action, observe that the cycle $1\rightarrow2\rightarrow1$ of the $2$-Hydra map $H=T_{3}$ applies the even branch ($H_{0}$) second and the odd branch ($H_{1}$) first. Thus, the string $\mathbf{j}$ such that $H_{\mathbf{j}}\left(1\right)=1$ is $\mathbf{j}=\left(0,1\right)$: \begin{equation} 1=H_{0}\left(H_{1}\left(1\right)\right)=H_{0}\left(2\right)=1 \end{equation} The integer $n$ represented by $\mathbf{j}$ is $n=0\cdot2^{0}+1\cdot2^{1}=2$. Thus: \begin{equation} \centerdot01010101\ldots\overset{\mathbb{Z}_{2}}{=}B_{2}\left(2\right)=\frac{2}{1-2^{\lambda_{2}\left(2\right)}}=\frac{2}{1-4}=-\frac{2}{3} \end{equation} and so: \begin{equation} \chi_{3}\left(-\frac{2}{3}\right)=\chi_{3}\left(B_{2}\left(2\right)\right)=\frac{\chi_{3}\left(2\right)}{1-M_{3}\left(2\right)}=\frac{\frac{1}{4}}{1-\frac{3}{4}}=\frac{1}{4-3}=1 \end{equation} where: \begin{equation} \chi_{3}\left(2\right)=\frac{1}{2}\chi_{3}\left(1\right)=\frac{1}{2}\left(\frac{3\chi_{3}\left(0\right)+1}{2}\right)=\frac{1}{2}\cdot\frac{0+1}{2}=\frac{1}{4} \end{equation} and: \begin{equation} M_{3}\left(2\right)=\frac{3^{\#_{1}\left(2\right)}}{2^{\lambda_{2}\left(2\right)}}=\frac{3^{1}}{2^{2}}=\frac{3}{4} \end{equation} \end{example} \vphantom{} We end our exploration of the Correspondence Principle with a question mark. As infamous as Collatz-type problems' difficulty might be, a combination of familiarization with the available literature and personal exploration of the problems themselves suggests that, as difficult as the study of these maps' periodic points might be, the question of their divergent trajectories could very well be an order of magnitude more difficult. The diophantine equation characterizations of periodic points given in \textbf{Corollary \ref{cor:CP v3} }show that, at the bare minimum, the question of periodic points can has an \emph{expressible }non-trivial reformulation in terms of an equation of finitely many variables. Yes, there might be infinitely many possible equations to consider, and solving any single one of them\textemdash let alone all of them\textemdash is difficult\textemdash maybe even extraordinarily difficulty. But, at least there is something we can \emph{attempt }to confront. The finitude of any cycle of $H$ guarantees at least this much. For divergent trajectories, on the other hand, the prospects are bleak indeed. The only finite aspect of their existence that come to mind are the fact that they are bounded from below (if divergent to $+\infty$) or above (if divergent to $-\infty$). I am not certain if it has been shown, given an $n\in\mathbb{Z}$ belonging to a divergent trajectory of $T_{3}$, that there are iterates $T_{3}^{\circ k}\left(n\right)$ which are divisible by arbitrarily large powers of $2$; that is: \begin{equation} \sup_{k\geq0}v_{2}\left(T_{3}^{\circ k}\left(n\right)\right)=\infty \end{equation} where $v_{2}$ is the $2$-adic valuation. Even with a positive or negative answer to this question, it is not at all clear how it might be useful for tackling the question of divergent trajectories, assuming it would be useful at all. The extra difficulty of divergent trajectories is all the more startling considering the likes of the shortened $5x+1$ map, $T_{5}$. We know that the set of divergent trajectories of $T_{5}$ in $\mathbb{N}_{1}$ has density $1$, yet not a single positive integer has been proven to belong to a divergent trajectory! \cite{Lagarias-Kontorovich Paper} Late in the writing of this dissertation (March 2022), I was surprised to realize that, for an almost trivial reason, the Correspondence Principle implies a connection between $\chi_{H}$ and $H$'s divergent trajectories, and a particularly beautiful one at that. \textbf{Corollary \ref{cor:CP v4} }shows that the periodic points are controlled by values in $\mathbb{Z}$ attained by $\chi_{H}$ for \emph{rational }$p$-adic inputs; that is, those $p$-adic integers whose $p$-adic digit sequences are eventually periodic. But... what about \emph{irrational }$p$-adic integers?\textemdash that is, $\mathbb{Z}_{p}\backslash\mathbb{Q}$, the set of $\mathfrak{z}\in\mathbb{Z}_{p}$ whose sequence of $p$-adic digits are not eventually periodic. Modulo some simple conditions on $H$, we can prove that irrational $\mathfrak{z}$s which make $\chi_{H}\left(\mathfrak{z}\right)$ an integer then make $\chi_{H}\left(\mathfrak{z}\right)$ an element of a divergent trajectory; this is \textbf{Theorem \ref{thm:Divergent trajectories come from irrational z}}. \begin{prop} \label{prop:Preparing for application to divergent trajectories}Let $H$ be a semi-basic $p$-Hydra map, and suppose that $\left|H_{j}\left(0\right)\right|_{q_{H}}=1$ for all $j\in\left\{ 1,\ldots,p-1\right\} $. Then $\chi_{H}\left(\mathfrak{z}\right)\neq0$ for any $\mathfrak{z}\in\mathbb{Z}_{p}\backslash\mathbb{Q}$. \end{prop} Proof: Let $H$ as given. By way of contradiction, suppose that $\chi_{H}\left(\mathfrak{z}\right)=0$ for some $\mathfrak{z}\in\mathbb{Z}_{p}\backslash\mathbb{Q}$. Now, let $j\in\left\{ 0,\ldots,p-1\right\} $ be the first $p$-adic digit of $\mathfrak{z}$; that is $j=\left[\mathfrak{z}\right]_{p}$. Then, letting $\mathfrak{z}^{\prime}$ denote the $p$-adic integer $\left(\mathfrak{z}-j\right)/p$, we can write: \begin{equation} 0=\chi_{H}\left(\mathfrak{z}\right)=\chi_{H}\left(p\mathfrak{z}^{\prime}+j\right)=H_{j}\left(\chi_{H}\left(\mathfrak{z}^{\prime}\right)\right) \end{equation} Next, suppose $j=0$. Then, since $H\left(0\right)=0$: \begin{equation} 0=H_{0}\left(\chi_{H}\left(\mathfrak{z}^{\prime}\right)\right)=\frac{\mu_{0}}{p}\chi_{H}\left(\mathfrak{z}^{\prime}\right) \end{equation} This forces $\chi_{H}\left(\mathfrak{z}^{\prime}\right)=0$, seeing as $\mu_{0}\neq0$. In this way, if the first $n$ $p$-adic digits of $\mathfrak{z}$ are all $0$, we can remove those digits from $\mathfrak{z}$ to obtain a $p$-adic integer $\mathfrak{z}^{\left(n\right)}\overset{\textrm{def}}{=}p^{-n}\mathfrak{z}$ with the property that $\left|\mathfrak{z}^{\left(n\right)}\right|_{p}=1$ (i.e., $\left[\mathfrak{z}^{\left(n\right)}\right]_{p}\neq0$). If \emph{all }of the digits of $\mathfrak{z}$ are $0$, this then makes $\mathfrak{z}=0$, which would contradict the given that $\mathfrak{z}$ was an element of $\mathbb{Z}_{p}\backslash\mathbb{Q}$. So, without loss of generality, we can assume that $j\neq0$. Hence: \begin{align*} 0 & =H_{j}\left(\chi_{H}\left(\mathfrak{z}^{\prime}\right)\right)\\ & \Updownarrow\\ \chi_{H}\left(\mathfrak{z}^{\prime}\right) & =-\frac{pH_{j}\left(0\right)}{\mu_{j}} \end{align*} Since $H$ is semi-basic, the fact that $j\neq0$ tells us that $\left|\mu_{j}\right|_{q_{H}}<1$ and $\left|p\right|_{q_{H}}=1$. This means $\left|H_{j}\left(0\right)\right|_{q_{H}}<1$; else we would have that $\left|\chi_{H}\left(\mathfrak{z}^{\prime}\right)\right|_{q_{H}}>1$, which is impossible seeing as $\chi_{H}\left(\mathbb{Z}_{p}\right)\subseteq\mathbb{Z}_{q_{H}}$ and every $q_{H}$-adic integer has\textbf{ }$q_{H}$-adic absolute value $\leq1$. So, $\left|H_{j}\left(0\right)\right|_{q_{H}}<1$\textemdash but, we were given that $\left|H_{j}\left(0\right)\right|_{q_{H}}=1$ for all $j\in\left\{ 1,\ldots,p-1\right\} $. There's our first contradiction. So, $\mathfrak{z}$ has no non-zero $p$-adic digits, which forces $\mathfrak{z}$ to be $0$. But, once again, $\mathfrak{z}$ was given to be an element of $\mathbb{Z}_{p}\backslash\mathbb{Q}$. There's our second and final contradiction. So, $\chi_{H}\left(\mathfrak{z}\right)$ must be non-zero. Q.E.D. \begin{thm} \label{thm:Divergent trajectories come from irrational z}\index{Hydra map!divergent trajectories}Let $H$ be a proper, integral, contracting, semi-basic $p$-Hydra map fixing $0$ so that $\left|H_{j}\left(0\right)\right|_{q_{H}}=1$ for all $j\in\left\{ 1,\ldots,p-1\right\} $. Let $\mathfrak{z}\in\mathbb{Z}_{p}\backslash\mathbb{Q}$ be such that $\chi_{H}\left(\mathfrak{z}\right)\in\mathbb{Z}$. Then $\chi_{H}\left(\mathfrak{z}\right)$ belongs to a divergent trajectory of $H$. \end{thm} Proof: Let $H$ and $\mathfrak{z}$ be as given. By \textbf{Proposition \ref{prop:Preparing for application to divergent trajectories}}, $\chi_{H}\left(\mathfrak{z}\right)\neq0$. Now, by way of contradiction, suppose that $\chi_{H}\left(\mathfrak{z}\right)$ did not belong to a divergent trajectory of $H$. By the basic theory of dynamical systems on $\mathbb{Z}$ (\textbf{Theorem \ref{thm:orbit classes partition domain}}), every element of $\mathbb{Z}$ belongs to either a divergent trajectory of $H$, or to an orbit class of $H$ which contains a cycle. Thus, it must be that $\chi_{H}\left(\mathfrak{z}\right)$ is pre-periodic. As such, there is an $n\geq0$ so that $H^{\circ n}\left(\chi_{H}\left(\mathfrak{z}\right)\right)$ is a periodic point of $H$. Using $\chi_{H}$'s functional equations from \textbf{Lemma \ref{lem:Unique rising continuation and p-adic functional equation of Chi_H}}, it then follows that $H^{\circ n}\left(\chi_{H}\left(\mathfrak{z}\right)\right)=\chi_{H}\left(\mathfrak{y}\right)$ where $\mathfrak{y}=m+p^{\lambda_{p}\left(m\right)}\mathfrak{z}$, where $m$ is the unique non-negative integer whose sequence of $p$-adic digits corresponds to the composition sequence of branches of $H$ brought to bear when we apply $H^{\circ n}$ to send $\chi_{H}\left(\mathfrak{z}\right)$ to $\chi_{H}\left(\mathfrak{y}\right)$. Moreover, since $\chi_{H}\left(\mathfrak{z}\right)\neq0$, the fact that $\left\{ 0\right\} $ is an isolated cycle of $H$ guarantees that the periodic point $\chi_{H}\left(\mathfrak{y}\right)$ is non-zero. Since $H$ is proper, we can apply Version 4 of \textbf{Correspondence Principle }(\textbf{Corollary \ref{cor:CP v4}}). This tells us $\mathfrak{y}$ is an element of $\mathbb{Q}\cap\mathbb{Z}_{p}^{\prime}$. Since $\mathfrak{y}$ is rational, its $p$-adic digits are eventually periodic. However, $\mathfrak{y}=m+p^{\lambda_{p}\left(m\right)}\mathfrak{z}$, where $\mathfrak{z}$ is ``irrational'' ($\mathfrak{z}\in\mathbb{Z}_{p}\backslash\mathbb{Q}$)\textemdash the $p$-adic digits of $\mathfrak{z}$ are aperiodic, which means the same is true of $\mathfrak{y}$. This is a contradiction! So, our assumption that $\chi_{H}\left(\mathfrak{z}\right)$ did not belong to a divergent trajectory of $H$ forces the digits of $\mathfrak{y}$ (which are not periodic) to be eventually periodic\textemdash a clear impossibility. This proves that $\chi_{H}\left(\mathfrak{z}\right)$ must be a divergent point of $H$. Q.E.D. \vphantom{} This theorem suggests the following: \begin{conjecture}[\textbf{A Correspondence Principle for Divergent Points?}] \label{conj:correspondence theorem for divergent trajectories}Provided that $H$ satisfies certain prerequisites such as the hypotheses of \textbf{Theorem \ref{thm:Divergent trajectories come from irrational z}}, $x\in\mathbb{Z}$ belongs to a divergent trajectory under $H$ if and only if there is a $\mathfrak{z}\in\mathbb{Z}_{p}\backslash\mathbb{Q}$ so that $\chi_{H}\left(\mathfrak{z}\right)\in\mathbb{Z}$. \end{conjecture} \vphantom{} The difficulty of this conjecture is that, unlike the Correspondence Principle for periodic points of $H$, I have yet to find \emph{constructive }method of producing a $\mathfrak{z}\in\mathbb{Z}_{p}\backslash\mathbb{Q}$ for which $\chi_{H}\left(\mathfrak{z}\right)$ is in $\mathbb{Z}$. Finally, we have: \begin{conjecture}[\textbf{Divergence Conjectures for $qx+1$}] Let $\chi_{q}\left(\mathfrak{z}\right)$ denote the numen of the shortened $qx+1$ map\index{$qx+1$ map}, where $q$ is an odd prime. Then, there exists a $\mathfrak{z}\in\mathbb{Z}_{2}\backslash\mathbb{Q}$ so that $\chi_{q}\left(\mathfrak{z}\right)\in\mathbb{Z}$ if and only if $q=3$. \end{conjecture} \subsection{\label{subsec:2.2.4 Other-Avenues}Other Avenues} \subsubsection{\label{subsec:Relaxing-the-Requirements}Relaxing the Requirements of Monogenicity and Non-Degeneracy} With regard to the convergence of $\chi_{H}$ over the $p$-adics, the key expression is (\ref{eq:Formal rising limit of Chi_H}): \begin{equation} \lim_{m\rightarrow\infty}\chi_{H}\left(\left[\mathfrak{z}\right]_{p^{m}}\right)\overset{\mathbb{Z}_{q}}{=}\sum_{\ell=1}^{\infty}\frac{b_{j_{\ell}\left(\mathfrak{z}\right)}}{d_{j_{\ell}\left(\mathfrak{z}\right)}}M_{H}\left(\mathbf{j}_{\ell-1}\left(\mathfrak{z}\right)\right) \end{equation} where, recall, we write the $p$-adic integer $\mathfrak{z}$ as: \begin{equation} \mathfrak{z}=j_{1}\left(\mathfrak{z}\right)+j_{2}\left(\mathfrak{z}\right)p+j_{3}\left(\mathfrak{z}\right)p^{2}+\cdots \end{equation} and write $\mathbf{j}_{\ell-1}\left(\mathfrak{z}\right)=\left(j_{1}\left(\mathfrak{z}\right),\ldots,j_{\ell-1}\left(\mathfrak{z}\right)\right)\in\left(\mathbb{Z}/p\mathbb{Z}\right)^{\ell-1}$, with the convention that $M_{H}\left(\mathbf{j}_{0}\left(\mathfrak{z}\right)\right)=1$. By the concatenation identity \textbf{Proposition \ref{prop:M_H concatenation identity}}, to obtain $M_{H}\left(\mathbf{j}_{\ell}\left(\mathfrak{z}\right)\right)$, we multiply $M_{H}\left(\mathbf{j}_{\ell-1}\left(\mathfrak{z}\right)\right)$ by a factor of $a_{j_{\ell}\left(\mathfrak{z}\right)}/d_{j_{\ell}\left(\mathfrak{z}\right)}$. The various conditions contained in the notion of semi-basicness were designed so as to ensure that $\left|a_{j}/d_{j}\right|_{q_{H}}\geq1$ occurs if and only if $j=0$. This $q_{H}$-adic condition, in turn, is what guarantees that (\ref{eq:Formal rising limit of Chi_H}) will converge in $\mathbb{Z}_{q_{H}}$ for any $\mathfrak{z}\in\mathbb{Z}_{p}^{\prime}$\textemdash that is, any $\mathfrak{z}$ with infinitely many non-zero $p$-adic digits. For the sake of clarity, rather than continue to work with the encumbrance of our indexing notation and an arbitrary $p$-Hydra map, we will illustrate the obstacles in relaxing the various conditions (monogenicity, (semi)-simplicity, non-degeneracy), by way of an example. \begin{example}[\textbf{A polygenic Hydra map}] \label{exa:Polygenic example, part 1}Consider the $3$-Hydra map $H:\mathbb{Z}\rightarrow\mathbb{Z}$ defined by: \begin{equation} H\left(n\right)\overset{\textrm{def}}{=}\begin{cases} \frac{an}{3} & \textrm{if }n\overset{3}{\equiv}0\\ \frac{bn+b^{\prime}}{3} & \textrm{if }n\overset{3}{\equiv}1\\ \frac{cn+2c^{\prime}}{3} & \textrm{if }n\overset{3}{\equiv}2 \end{cases}\label{eq:Toy model for a polygenic 3-Hydra map} \end{equation} where $a$, $b$, and $c$ are positive integers co-prime to $3$, and where $b^{\prime}$ and $c^{\prime}$ are positive integers so that $b+b^{\prime}\overset{3}{\equiv}0$ and $c+c^{\prime}\overset{3}{\equiv}0$, respectively; $b^{\prime}$ and $c^{\prime}$ guarantee that each of the branches of $H$ outputs a non-negative integer. For this $H$, we have that: \begin{equation} M_{H}\left(\mathbf{j}\right)=\frac{a^{\#_{3:0}\left(\mathbf{j}\right)}\times b^{\#_{3:1}\left(\mathbf{j}\right)}\times c^{\#_{3:2}\left(\mathbf{j}\right)}}{3^{\left|\mathbf{j}\right|}} \end{equation} In order for (\ref{eq:Formal rising limit of Chi_H}) to converge, we need $M_{H}\left(\mathbf{j}\right)$ to become small in some sense as $\left|\mathbf{j}\right|\rightarrow\infty$ with infinitely many non-zero digits. As such, the value of $a$ is irrelevant to our convergence issue; only the constants $b$ and $c$, associated\textemdash respectively\textemdash to the branches $H_{1}$ and $H_{2}$ are of import, because any $\mathbf{j}$ with only finitely many non-zero entries reduces to an integer. In the monogenic case dealt with in this dissertation, we required both $b$ and $c$ to be multiples of some integer $q_{H}\geq2$. \textbf{Proposition \ref{prop:q-adic behavior of M_H of j as the number of non-zero digits tends to infinity}} showed that this condition guaranteed the $q_{H}$-adic decay of $M_{H}\left(\mathbf{j}\right)$ to $0$ as the number of non-zero digits of $\mathbf{j}$ increased to infinity. On the other hand, if this were the \index{Hydra map!polygenic}\textbf{polygenic }case, then $b$ and $c$ would be co-prime to one another. Note that if one or both of $b$ and $c$ are equal to $1$, then we have a degenerate case, because multiplication by $1$ does not decrease $p$-adic size for any prime $p$. So, we can suppose that $b$ and $c$ are co-prime. For simplicity, let $b$ and $c$ be distinct primes, with neither $b$ nor $c$ being $3$. Then, there are several possibilities for $M_{H}\left(\mathbf{j}\right)$ as $\mathbf{j}\rightarrow\infty$ with $\mathbf{j}$ having infinitely many non-zero entries: \end{example} \begin{enumerate} \item If $\mathbf{j}$ has infinitely many $1$s (this corresponds to multiplications by $b$), then $M_{H}\left(\mathbf{j}\right)$ tends to $0$ in $b$-adic magnitude. \item If $\mathbf{j}$ has infinitely many $2$s (this corresponds to multiplications by $c$), then $M_{H}\left(\mathbf{j}\right)$ tends to $0$ in $c$-adic magnitude. \item If only one of $1$ or $2$ occurs infinitely many times in $\mathbf{j}$, then $M_{H}\left(\mathbf{j}\right)$ will tend to $0$ in either $b$-adic or $c$-adic magnitude, \emph{but not in both}\textemdash \emph{that} occurs if and only if $\mathbf{j}$ contains both infinitely many $1$s \emph{and} infinitely many $2$s. \end{enumerate} \begin{rem} In hindsight, I believe my notion of a \textbf{frame} (see Subsection \ref{subsec:3.3.3 Frames}) might be suited for the polygenic case. Considering \textbf{Example \ref{exa:Polygenic example, part 1}}, we could assign the $b$-adic and $c$-adic convergence, respectively, to the sets of $3$-adic integers whose representative strings $\mathbf{j}$ have infinitely many $1$s and $2$s, respectively. This would then allow for a meaningful definition of $\chi_{H}$, and thereby potentially open the door to applying my methods to polygenic Hydra maps. \end{rem} \subsubsection{\label{subsec:Baker,-Catalan,-and}Baker, Catalan, and Collatz} Any proof of the Weak Collatz Conjecture\footnote{Recall, this is the assertion that the only periodic points of the Collatz map in the positive integers are $1$, $2$, and $4$.} will necessarily entail a significant advancement in\index{transcendental number theory} transcendental number theory. A proof of the Weak Collatz Conjecture would necessarily yield a proof of \textbf{Baker's Theorem} far simpler than any currently known method \cite{Tao Blog}. \index{Baker's Theorem}Baker's Theorem, recall, concerns lower bounds on the absolute values of \textbf{linear forms of logarithms}, which are expressions of the form: \begin{equation} \beta_{1}\ln\alpha_{1}+\cdots+\beta_{N}\ln\alpha_{N}\label{eq:linear form in logarithm} \end{equation} where the $\beta_{n}$s and $\alpha_{n}$s are complex algebraic numbers (with all of the $\alpha_{n}$s being non-zero). Most proofs of Baker's Theorem employ a variant of what is known as Baker's Method, in which one constructs of an analytic function (called an \textbf{auxiliary function})\textbf{ }with a large number of zeroes of specified degree so as to obtain contradictions on the assumption that (\ref{eq:linear form in logarithm}) is small in absolute value. The import of Baker's Theorem is that it allows one to obtain lower bounds on expressions such as $\left|2^{m}-3^{n}\right|$, where $m$ and $n$ are positive integers, and it was for applications such as these in conjunction with the study of Diophantine equations that Baker earned the Fields Medal. See \cite{Baker's Transcendental Number Theory} for a comprehensive account of the subject; also, parts of \cite{Cohen Number Theory}. With $\chi_{H}$ and the \textbf{Correspondence Principle }at our disposal, we can get a glimpse at the kinds of advancements in transcendental number theory that might be needed in order to resolve the Weak Collatz Conjecture. To begin, it is instructive to consider the following table of values for $\chi_{H}$ and related functions in the case of the Shortened $qx+1$ maps $T_{q}$, where $q$ is an odd prime. For brevity, we write $\chi_{q}$ to denote $\chi_{T_{q}}:\mathbb{Z}_{2}\rightarrow\mathbb{Z}_{q}$. \begin{center} \begin{table} \begin{centering} \begin{tabular}{|c|c|c|c|c|c|c|} \hline $n$ & $\#_{1}\left(n\right)$ & $\lambda_{2}\left(n\right)$ & $\chi_{q}\left(n\right)$ & $\chi_{q}\left(B_{2}\left(n\right)\right)$ & $\chi_{3}\left(B_{2}\left(n\right)\right)$ & $\chi_{5}\left(B_{2}\left(n\right)\right)$\tabularnewline \hline \hline $0$ & $0$ & $0$ & $0$ & $0$ & $0$ & $0$\tabularnewline \hline $1$ & $1$ & $1$ & $\frac{1}{2}$ & $\frac{1}{2-q}$ & $-1$ & $-\frac{1}{3}$\tabularnewline \hline $2$ & $1$ & $2$ & $\frac{1}{4}$ & $\frac{1}{4-q}$ & $1$ & $-1$\tabularnewline \hline $3$ & $2$ & $2$ & $\frac{2+q}{4}$ & $\frac{2+q}{4-q^{2}}$ & $-1$ & $-\frac{1}{3}$\tabularnewline \hline $4$ & $1$ & $3$ & $\frac{1}{8}$ & $\frac{1}{8-q}$ & $\frac{1}{5}$ & $\frac{1}{3}$\tabularnewline \hline $5$ & $2$ & $3$ & $\frac{4+q}{8}$ & $\frac{4+q}{8-q^{2}}$ & $-7$ & $-\frac{9}{17}$\tabularnewline \hline $6$ & $2$ & $3$ & $\frac{2+q}{8}$ & $\frac{2+q}{8-q^{2}}$ & $-5$ & $-\frac{7}{17}$\tabularnewline \hline $7$ & $3$ & $3$ & $\frac{4+2q+q^{2}}{8}$ & $\frac{4+2q+q^{2}}{8-q^{3}}$ & $-1$ & $-\frac{1}{3}$\tabularnewline \hline $8$ & $1$ & $4$ & $\frac{1}{16}$ & $\frac{1}{16-q}$ & $\frac{1}{13}$ & $\frac{1}{11}$\tabularnewline \hline $9$ & $2$ & $4$ & $\frac{8+q}{16}$ & $\frac{8+q}{16-q^{2}}$ & $\frac{11}{7}$ & $-\frac{13}{9}$\tabularnewline \hline $10$ & $2$ & $4$ & $\frac{4+q}{16}$ & $\frac{4+q}{16-q^{2}}$ & $1$ & $-1$\tabularnewline \hline $11$ & $3$ & $4$ & $\frac{8+4q+q^{2}}{16}$ & $\frac{8+4q+q^{2}}{16-q^{3}}$ & $-\frac{29}{11}$ & $-\frac{53}{109}$\tabularnewline \hline $12$ & $2$ & $4$ & $\frac{2+q}{16}$ & $\frac{2+q}{16-q^{2}}$ & $\frac{5}{7}$ & $-\frac{7}{9}$\tabularnewline \hline $13$ & $3$ & $4$ & $\frac{8+2q+q^{2}}{16}$ & $\frac{8+2q+q^{2}}{16-q^{3}}$ & $-\frac{23}{11}$ & $-\frac{43}{109}$\tabularnewline \hline $14$ & $3$ & $4$ & $\frac{4+2q+q^{2}}{16}$ & $\frac{4+2q+q^{2}}{16-q^{3}}$ & $\frac{19}{7}$ & $-\frac{39}{109}$\tabularnewline \hline $15$ & $4$ & $4$ & $\frac{8+4q+2q^{2}+q^{3}}{16}$ & $\frac{8+4q+2q^{2}+q^{3}}{16-q^{4}}$ & $-1$ & $-\frac{1}{3}$\tabularnewline \hline \end{tabular} \par\end{centering} \caption{Values of $\chi_{q}\left(n\right)$ and related functions} \end{table} \par\end{center} \begin{example} \index{Correspondence Principle}As per the Correspondence Principle,\textbf{ }note that the integer values attained by $\chi_{3}\left(B_{2}\left(n\right)\right)$ and $\chi_{5}\left(B_{2}\left(n\right)\right)$ are all periodic points of the maps $T_{3}$ and $T_{5}$, respectively; this includes fixed points at negative integers, as well. Examining $\chi_{q}\left(B_{2}\left(n\right)\right)$, we see certain patterns, such as: \begin{equation} \chi_{q}\left(B_{2}\left(2^{n}-1\right)\right)=\frac{1}{2-q},\textrm{ }\forall n\in\mathbb{N}_{1} \end{equation} More significantly, $\chi_{3}\left(B_{2}\left(n\right)\right)$ appears to be more likely to be positive than negative, whereas the opposite appears to hold true for $q=5$ (and, heuristically, for all $q\geq5$). Of special interest, however, is: \begin{equation} \chi_{q}\left(B_{2}\left(10\right)\right)=\frac{4+q}{16-q^{2}}=\frac{1}{4-q} \end{equation} \end{example} By Version 1 of the \textbf{Correspondence} \textbf{Principle} (\textbf{Theorem \ref{thm:CP v1}}), every cycle $\Omega\subseteq\mathbb{Z}$ of $T_{q}$ with $\left|\Omega\right|\geq2$ contains an integer $x$ of the form: \begin{equation} x=\chi_{q}\left(B_{2}\left(n\right)\right)=\frac{\chi_{q}\left(n\right)}{1-\frac{q^{\#_{2:1}\left(n\right)}}{2^{\lambda_{2}\left(n\right)}}}=\frac{2^{\lambda_{2}\left(n\right)}\chi_{q}\left(n\right)}{2^{\lambda_{2}\left(n\right)}-q^{\#_{2:1}\left(n\right)}}\label{eq:Rational expression of odd integer periodic points of ax+1} \end{equation} for some $n\in\mathbb{N}_{1}$. In fact, every periodic point $x$ of $T_{q}$ in the odd integers can be written in this form. As such $\left|2^{\lambda_{2}\left(n\right)}-q^{\#_{2:1}\left(n\right)}\right|=1$ is a \emph{sufficient condition }for $\chi_{q}\left(B_{2}\left(n\right)\right)$ to be a periodic point of $T_{q}$. However, as the $n=10$ case shows, this is not\emph{ }a \emph{necessary }condition: there can be values of $n$ where $2^{\lambda_{2}\left(n\right)}-q^{\#_{2:1}\left(n\right)}$ is large in archimedean absolute value, and yet nevertheless divides the numerator on the right-hand side of (\ref{eq:Rational expression of odd integer periodic points of ax+1}), thereby reducing $\chi_{q}\left(B_{2}\left(n\right)\right)$ to an integer. In fact, thanks to P. Mih\u{a}ilescu \index{Mihu{a}ilescu, Preda@Mih\u{a}ilescu, Preda}'s resolution of \index{Catalan's Conjecture}\textbf{ Catalan's Conjecture}, it would seem that Baker's Method-style estimates on the archimedean\emph{ }size of $2^{\lambda_{2}\left(n\right)}-q^{\#_{2:1}\left(n\right)}$ will be of little use in understanding $\chi_{q}\left(B_{2}\left(n\right)\right)$: \begin{thm}[\textbf{Mih\u{a}ilescu's Theorem}\footnote{Presented in \cite{Cohen Number Theory}.}] The only choice of $x,y\in\mathbb{N}_{1}$ and $m,n\in\mathbb{N}_{2}$ for which: \begin{equation} x^{m}-y^{n}=1\label{eq:Mihailescu's Theorem} \end{equation} are $x=3$, $m=2$, $y=2$, $n=3$ (that is, $3^{2}-2^{3}=1$). \end{thm} \vphantom{} With Mih\u{a}ilescu's Theorem, it is easy to see that for any odd integer $q\geq3$, $\left|q^{\#_{2:1}\left(n\right)}-2^{\lambda_{2}\left(n\right)}\right|$ will never be equal to $1$ for any $n\geq8$, because the exponent of $2$ (that is, $\lambda_{2}\left(n\right)$) will be $\geq4$ for all $n\geq8$ (any such $n$ has $4$ or more binary digits). Consequently, for any odd prime $q$, if $n\geq8$ makes $\chi_{q}\left(B_{2}\left(n\right)\right)$ into a rational integer (and hence, a periodic point of $T_{q}$), it \emph{must }be that the numerator $2^{\lambda_{2}\left(n\right)}\chi_{q}\left(n\right)$ in (\ref{eq:Rational expression of odd integer periodic points of ax+1}) is a multiple of $2^{\lambda_{2}\left(n\right)}-q^{\#_{2:1}\left(n\right)}$. The set of $p$-Hydra maps for which conclusions of this sort hold can be enlarged in several ways. First, in general, note that we have: \begin{equation} \chi_{H}\left(B_{p}\left(n\right)\right)=\frac{\chi_{H}\left(n\right)}{1-M_{H}\left(n\right)}=\frac{p^{\lambda_{p}\left(n\right)}\chi_{H}\left(n\right)}{p^{\lambda_{p}\left(n\right)}\left(1-M_{H}\left(n\right)\right)}=\frac{p^{\lambda_{p}\left(n\right)}\chi_{H}\left(n\right)}{p^{\lambda_{p}\left(n\right)}-\prod_{j=0}^{p-1}\mu_{j}^{\#_{p:j}\left(n\right)}}\label{eq:Chi_H o B functional equation with fraction in simplest form} \end{equation} If $\mu_{0}=1$ and there is an integer $\mu\geq2$ so that $\mu_{1},\ldots,\mu_{p-1}$ are all positive integer powers of $\mu$ (i.e., for each $j\in\left\{ 1,\ldots,p-1\right\} $ there is an $r_{j}\in\mathbb{N}_{1}$ so that $\mu_{j}=\mu^{r_{j}}$) then, when $n\geq1$, the denominator of the right-hand side takes the form $p^{a}-\mu^{b}$ for some positive integers $a$ and $b$ depending solely on $n$. By Mih\u{a}ilescu's Theorem, the only way to have $\left|p^{a}-\mu^{b}\right|=1$ is for either $p^{a}-\mu^{b}=3^{2}-2^{3}$ or $p^{a}-\mu^{b}=2^{3}-3^{2}$. As such, for this case, once $n$ is large enough so that both $a$ and $b$ are $\geq3$, the only way for $\chi_{H}\left(B_{p}\left(n\right)\right)$ to be a rational integer for such an $n$ is if $p^{\lambda_{p}\left(n\right)}-\prod_{j=0}^{p-1}\mu_{j}^{\#_{p:j}\left(n\right)}$ is a divisor of the integer $p^{\lambda_{p}\left(n\right)}\chi_{H}\left(n\right)$ of magnitude $\geq2$. In the most general case, if we allow the $\mu_{j}$ to take on different values (while still requiring $H$ to be basic so that the Correspondence Principle applies), in order to replicate the application of Mih\u{a}ilescu's Theorem to the $p^{a}-\mu^{b}$ case, examining the the denominator term $p^{\lambda_{p}\left(n\right)}-\prod_{j=0}^{p-1}\mu_{j}^{\#_{p:j}\left(n\right)}$ from (\ref{eq:Chi_H o B functional equation with fraction in simplest form}) and ``de-parameterizing'' it, the case where the denominator reduces to $1$ or $-1$ can be represented by a Diophantine equation\index{diophantine equation} of the form: \begin{equation} \left|x^{m}-y_{1}^{n_{1}}y_{2}^{n_{2}}\cdots y_{J}^{n_{J}}\right|=1\label{eq:Generalized Catalan Diophantine Equation} \end{equation} where $J$ is a fixed integer $\geq2$, and $x$, $y_{1},\ldots,y_{J}$, $m$, and $n_{1},\ldots,n_{J}$ are positive integer variables. For any fixed choice of $m$ and the $n_{j}$s, \textbf{Faltings' Theorem}\index{Faltings' Theorem}\textbf{ }(the resolution of \textbf{Mordell's Conjecture}) guarantees there are only finitely many rational numbers $x,y_{1},\ldots,y_{J}$ for which (\ref{eq:Generalized Catalan Diophantine Equation}) holds true. On the other hand, for any fixed choice of $x,y_{1},\ldots,y_{J}$ (where we identify $x$ with $p$ and the $y_{j}$s with the $\mu_{j}$s), if it can be shown that (\ref{eq:Generalized Catalan Diophantine Equation}) holds for only finitely many choices of $m$ and the $n_{j}$s, it will follow that the corresponding case for $\left|p^{\lambda_{p}\left(n\right)}-\prod_{j=0}^{p-1}\mu_{j}^{\#_{p:j}\left(n\right)}\right|$ is not equal to $1$ for all sufficiently large positive integers $n$. In that situation, any cycles of $H$ not accounted for by values of $n$ for which $\left|p^{\lambda_{p}\left(n\right)}-\prod_{j=0}^{p-1}\mu_{j}^{\#_{p:j}\left(n\right)}\right|=1$ must occur as a result of $\left|p^{\lambda_{p}\left(n\right)}-\prod_{j=0}^{p-1}\mu_{j}^{\#_{p:j}\left(n\right)}\right|$ being an integer $\geq2$ which divides $p^{\lambda_{p}\left(n\right)}\chi_{H}\left(n\right)$. In light of this, rather than the archimedean/euclidean \emph{size }of $p^{\lambda_{p}\left(n\right)}-\prod_{j=0}^{p-1}\mu_{j}^{\#_{p:j}\left(n\right)}$, it appears we must study its \emph{multiplicative }structure, as well as that of $p^{\lambda_{p}\left(n\right)}\chi_{H}\left(n\right)$\textemdash which is to say, the set of these integers' prime divisors, and how this set depends on $n$. In other words, we ought to study the $p$-adic absolute values of $\left|\chi_{H}\left(n\right)\right|_{p}$ and $\left|1-M_{H}\left(n\right)\right|_{p}$ for various values of $n\geq1$ and primes $p$. It may also be of interest to use $p$-adic methods (Fourier series, Mellin transform) to study $\left|\chi_{H}\left(\mathfrak{z}\right)\right|_{p}$ as a real-valued function of a $p$-adic integer variable for varying values of $p$. \subsubsection{\label{subsec:Connections-to-Tao}Connections to Tao (2019)} The origins of the present paper lie in work the author did in late 2019 and early 2020. Around the same time, in late 2019, Tao published a paper on the Collatz Conjecture that applied probabilistic techniques in a novel way, by use of what Tao\index{Tao, Terence} called ``\index{Syracuse Random Variables}Syracuse Random Variables'' \cite{Tao Probability paper}. Despite the many substantial differences between these two approaches (Tao's is probabilistic, ours is not), they are linked at a fundamental level, thanks to the function $\chi_{3}$\textemdash our shorthand for $\chi_{T_{3}}$, the $\chi_{H}$ associated to the Shortened Collatz map. Given that both papers have their genesis in an examination of the behavior of different combinations of the iterates of branches of $T_{3}$, it is not entirely surprising that both approaches led to $\chi_{3}$. Tao's approach involves constructing his Syracuse Random Variables and then comparing them to a set-up involving tuples of geometric random variables, the comparison in question being an estimate on the ``distance'' between the Syracuse Random Variables and the geometric model, as measured by the total variation norm for discrete random variables. To attain these results, the central challenge Tao overcomes is obtaining explicit estimates for the decay of the characteristic function of the Syracuse Random Variables. In our terminology, Tao establishes decay estimates for the archimedean absolute value of the function $\varphi_{3}:\hat{\mathbb{Z}}_{3}\rightarrow\mathbb{C}$ defined by: \begin{equation} \varphi_{3}\left(t\right)\overset{\textrm{def}}{=}\int_{\mathbb{Z}_{2}}e^{-2\pi i\left\{ t\chi_{3}\left(\mathfrak{z}\right)\right\} _{3}}d\mathfrak{z},\textrm{ }\forall t\in\hat{\mathbb{Z}}_{3}\label{eq:Tao's Characteristic Function} \end{equation} where $\left\{ \cdot\right\} _{3}$ is the $3$-adic fractional part, $d\mathfrak{z}$ is the Haar probability measure on $\mathbb{Z}_{2}$, and $\hat{\mathbb{Z}}_{3}=\mathbb{Z}\left[\frac{1}{3}\right]/\mathbb{Z}=\mathbb{Q}_{3}/\mathbb{Z}_{3}$ is the Pontryagin dual of $\mathbb{Z}_{3}$, identified here with the set of all rational numbers in $\left[0,1\right)$ whose denominators are non-negative integer powers of the prime number $3$. Tao's decay estimate is given in \textbf{Proposition 1.17 }of his paper, where the above integral appears in the form of an expected value \cite{Tao Probability paper}. With this in mind, a natural next step for furthering both this paper and Tao's would be to study the ``characteristic function\index{characteristic function}'' of $\chi_{H}$ for an arbitrary semi-basic $p$-Hydra maps $H$; this is the continuous function $\varphi_{H}:\hat{\mathbb{Z}}_{q_{H}}\rightarrow\mathbb{C}$ defined by: \begin{equation} \varphi_{H}\left(t\right)\overset{\textrm{def}}{=}\int_{\mathbb{Z}_{p}}e^{-2\pi i\left\{ t\chi_{H}\left(\mathfrak{z}\right)\right\} _{q_{H}}}d\mathfrak{z},\textrm{ }\forall t\in\hat{\mathbb{Z}}_{q_{H}}\label{eq:Characteristic function of Chi_H} \end{equation} where $\left\{ \cdot\right\} _{q_{H}}$ is the $q_{H}$-adic fractional part, $d\mathfrak{z}$ is the Haar probability measure on $\mathbb{Z}_{p}$, and $\hat{\mathbb{Z}}_{q_{H}}=\mathbb{Z}\left[\frac{1}{q_{H}}\right]/\mathbb{Z}=\mathbb{Q}_{q_{H}}/\mathbb{Z}_{q_{H}}$ is the Pontryagin dual of $\mathbb{Z}_{q_{H}}$, identified here with the set of all rational numbers in $\left[0,1\right)$ whose denominators are non-negative integer powers of $q_{H}$. We can establish functional equations\index{functional equation} can be established for $\varphi_{H}$. \begin{prop} Let $H$ be a semi-basic $p$-Hydra map which fixes $0$, and write $q=q_{H}$. Then, the characteristic function $\varphi_{H}:\hat{\mathbb{Z}}_{q}\rightarrow\mathbb{C}$ defined by \emph{(\ref{eq:Characteristic function of Chi_H})} satisfies the functional equation: \begin{equation} \varphi_{H}\left(t\right)=\frac{1}{p}\sum_{j=0}^{p-1}e^{-2\pi i\left\{ \frac{b_{j}t}{d_{j}}\right\} _{q}}\varphi_{H}\left(\frac{a_{j}t}{d_{j}}\right),\textrm{ }\forall t\in\hat{\mathbb{Z}}_{q}\label{eq:Phi_H functional equation} \end{equation} \end{prop} \begin{rem} Since $H$ fixes $0$ and is semi-basic, \textbf{Proposition \ref{prop:co-primality of d_j and q_H}} tells us that all of the $d_{j}$s are co-prime to $q$. Thus, the multiplication-by-$d_{j}$ map $t\mapsto d_{j}t$ is a group automorphism of $\hat{\mathbb{Z}}_{q}$, and as such, $b_{j}t/d_{j}$ and $a_{j}t/d_{j}$ denote the images of $b_{j}t$ and $a_{j}t$, respectively, under the inverse of this automorphism. One can remove the denominator terms if desired by writing: \begin{align*} d & \overset{\textrm{def}}{=}\textrm{lcm}\left(d_{0},\ldots,d_{p-1}\right)\\ d_{j}^{\prime} & \overset{\textrm{def}}{=}\frac{d}{d_{j}},\textrm{ }\forall j\in\left\{ 0,\ldots,p-1\right\} \in\mathbb{Z}\\ \alpha_{j} & \overset{\textrm{def}}{=}a_{j}d_{j}^{\prime}\in\mathbb{Z}\\ \beta_{j} & \overset{\textrm{def}}{=}b_{j}d_{j}^{\prime}\in\mathbb{Z} \end{align*} and then replacing $t$ with $dt$ in (\ref{eq:Phi_H functional equation}) to obtain: \begin{equation} \varphi_{H}\left(dt\right)=\frac{1}{p}\sum_{j=0}^{p-1}e^{-2\pi i\beta_{j}t}\varphi_{H}\left(\alpha_{j}t\right),\textrm{ }\forall t\in\hat{\mathbb{Z}}_{q} \end{equation} \end{rem} Proof: The proof is a straight-forward computation. Details regarding Fourier analysis on $p$-adic rings can be found in \cite{Automorphic Representations}. \begin{align*} \varphi_{H}\left(t\right) & =\int_{\mathbb{Z}_{p}}e^{-2\pi i\left\{ t\chi_{H}\left(\mathfrak{z}\right)\right\} _{q}}d\mathfrak{z}\\ & =\sum_{j=0}^{p-1}\int_{j+p\mathbb{Z}_{p}}e^{-2\pi i\left\{ t\chi_{H}\left(\mathfrak{z}\right)\right\} _{q}}d\mathfrak{z}\\ \left(\mathfrak{y}=\frac{\mathfrak{z}-j}{p}\right); & =\frac{1}{p}\sum_{j=0}^{p-1}\int_{\mathbb{Z}_{p}}e^{-2\pi i\left\{ t\chi_{H}\left(p\mathfrak{y}+j\right)\right\} _{q}}d\mathfrak{y}\\ & =\frac{1}{p}\sum_{j=0}^{p-1}\int_{\mathbb{Z}_{p}}e^{-2\pi i\left\{ t\frac{a_{j}\chi_{H}\left(\mathfrak{y}\right)+b_{j}}{d_{j}}\right\} _{q}}d\mathfrak{y}\\ & =\frac{1}{p}\sum_{j=0}^{p-1}e^{-2\pi i\left\{ \frac{b_{j}t}{d_{j}}\right\} _{q}}\int_{\mathbb{Z}_{p}}e^{-2\pi i\left\{ \frac{a_{j}t}{d_{j}}\chi_{H}\left(\mathfrak{y}\right)\right\} _{q}}d\mathfrak{y}\\ & =\frac{1}{p}\sum_{j=0}^{p-1}e^{-2\pi i\left\{ \frac{b_{j}t}{d_{j}}\right\} _{q}}\varphi_{H}\left(\frac{a_{j}t}{d_{j}}\right) \end{align*} Q.E.D. \vphantom{} The Fourier analysis here naturally leads to probabilistic notions. For any measurable function $f:\mathbb{Z}_{p}\rightarrow\mathbb{Z}_{q}$, any $n\in\mathbb{N}_{0}$ and any $m\in\mathbb{Z}$, we write: \begin{equation} \textrm{P}\left(f\overset{q^{n}}{\equiv}m\right)\overset{\textrm{def}}{=}\int_{\mathbb{Z}_{p}}\left[f\left(\mathfrak{z}\right)\overset{q^{n}}{\equiv}m\right]d\mathfrak{z}\label{eq:Definition of the probability that f is congruent to k mod q to the n} \end{equation} where $d\mathfrak{z}$ is the Haar probability measure on $\mathbb{Z}_{p}$ and $\left[f\left(\mathfrak{z}\right)\overset{q^{n}}{\equiv}m\right]$ is $1$ if $f\left(\mathfrak{z}\right)$ is congruent to $m$ mod $q^{n}$ and is $0$ otherwise. We adopt the convention that $\mathfrak{x}\overset{1}{\equiv}\mathfrak{y}$ is true for any $q$-adic integers $\mathfrak{x}$ and $\mathfrak{y}$, and hence, that: \begin{equation} \textrm{P}\left(f\overset{q^{0}}{\equiv}m\right)=\textrm{P}\left(f\overset{1}{\equiv}m\right)=\int_{\mathbb{Z}_{p}}\left[f\left(\mathfrak{z}\right)\overset{1}{\equiv}m\right]d\mathfrak{z}=\int_{\mathbb{Z}_{p}}d\mathfrak{z}=1 \end{equation} for any $f$ and any $m$. Fourier relations follow from the identity: \begin{equation} \left[\mathfrak{x}\overset{q^{n}}{\equiv}\mathfrak{y}\right]=\frac{1}{q^{n}}\sum_{k=0}^{q^{n}-1}e^{2\pi i\left\{ k\frac{\mathfrak{x}-\mathfrak{y}}{q^{n}}\right\} _{q}},\textrm{ }\forall n\in\mathbb{N}_{0},\textrm{ }\forall\mathfrak{x},\mathfrak{y}\in\mathbb{Z}_{q} \end{equation} This gives us the typical Fourier relations between probabilities and characteristic functions: \begin{align} \textrm{P}\left(\chi_{H}\overset{q^{n}}{\equiv}m\right) & =\frac{1}{q^{n}}\sum_{k=0}^{q^{n}-1}e^{-\frac{2\pi ikm}{q^{n}}}\varphi_{H}\left(\frac{k}{q^{n}}\right)\\ \varphi_{H}\left(\frac{m}{q^{n}}\right) & =\sum_{k=0}^{q^{n}-1}e^{\frac{2\pi imk}{q^{n}}}\textrm{P}\left(\chi_{H}\overset{q^{n}}{\equiv}k\right) \end{align} and the Parseval Identity: \begin{equation} \frac{1}{q^{n}}\sum_{k=0}^{q^{n}-1}\left|\varphi_{H}\left(\frac{k}{q^{n}}\right)\right|^{2}=\sum_{k=0}^{q^{n}-1}\left(\textrm{P}\left(\chi_{H}\overset{q^{n}}{\equiv}k\right)\right)^{2} \end{equation} Under certain circumstances (such as the case of the $T_{a}$ maps), one can use (\ref{eq:Phi_H functional equation}) to recursively solve for $\varphi_{H}$\textemdash or for $\textrm{P}\left(\chi_{H}\overset{q^{n}}{\equiv}k\right)$, after performing a discrete Fourier transform. Doing so for $H=T_{3}$ yields the recursive formula given by Tao in \textbf{Lemma 1.12 }of his paper \cite{Tao Probability paper}. That being said, it remains to be seen whether or not recursive formulae can be derived for these probabilities and characteristic functions in the case of a general $p$-Hydra map. It may also be of interest to study the expressions $\varphi_{H,p}:\hat{\mathbb{Z}}_{p}\rightarrow\mathbb{C}$ given by: \begin{equation} \varphi_{H,p}\left(t\right)\overset{\textrm{def}}{=}\int_{\mathbb{Z}_{p}}e^{-2\pi i\left\{ t\chi_{H}\left(\mathfrak{z}\right)\right\} _{p}}d\mathfrak{z}\overset{\textrm{def}}{=}\lim_{N\rightarrow\infty}\frac{1}{p^{N}}\sum_{n=0}^{p^{N}-1}e^{-2\pi i\left\{ t\chi_{H}\left(n\right)\right\} _{p}},\textrm{ }\forall t\in\hat{\mathbb{Z}}_{p}\label{eq:p-adic characteristic function for Chi_H} \end{equation} where $p$ is an arbitrary prime. Doing so, should, conceivably, advance our understanding of the divisibility of $\chi_{H}\left(n\right)$ by $p$ as $n$ varies. \subsubsection{\label{subsec:Dirichlet-Series-and}Dirichlet Series and Complex-Analytic Methods} Like in most situations, given the Correspondence Principle and the associated identity from \textbf{Lemma \ref{lem:Chi_H o B_p functional equation}}: \begin{equation} \chi_{H}\left(B_{p}\left(n\right)\right)=\frac{\chi_{H}\left(n\right)}{1-M_{H}\left(n\right)} \end{equation} one asks: \emph{what can be done with this?} Prior to stumbling upon the $\left(p,q\right)$-adic approach, my intention was to use classical, complex-analytic techniques of analytic number theory, such as Dirichlet Series\index{Dirichlet series}, Mellin transforms, contour integration, and the Residue Theorem. According to the Correspondence Principle, classifying the periodic points of $H$ in $\mathbb{Z}$ is just a matter of finding those integers $x\in\mathbb{Z}$ and $n\geq0$ so that $x=\chi_{H}\left(B_{p}\left(n\right)\right)$; which is to say: \begin{equation} \left(1-M_{H}\left(n\right)\right)x-\chi_{H}\left(n\right)=0\label{eq:nth term of Dirichlet series} \end{equation} Dividing everything by $\left(n+1\right)^{s}$ and summing over $n\geq0$ gives: \begin{equation} \left(\zeta\left(s\right)-\sum_{n=0}^{\infty}\frac{M_{H}\left(n\right)}{\left(n+1\right)^{s}}\right)x-\sum_{n=0}^{\infty}\frac{\chi_{H}\left(n\right)}{\left(n+1\right)^{s}}=0\label{eq:Dirichlet series} \end{equation} for all complex $s$ for which the Dirichlet series converge. By exploiting $M_{H}$ and $\chi_{H}$'s functional equations, the right-hand side of (\ref{eq:nth term of Dirichlet series}) can be written in terms of a contour integral of the right-hand side of (\ref{eq:Dirichlet series}) using a generalized version of \index{Perron's Formula}\textbf{Perron's Formula} (see\footnote{The series of papers by Flajolet et. al. on applications of the Mellin transform in analytic combinatorics (starting with \cite{Flajolet - Mellin Transforms}) provide a comprehensive and illustrative exposition of the Mellin transform\index{Mellin transform!complex} and its many, \emph{many }applications to combinatorics, number theory, algorithm analysis, and more.} \cite{Flajolet - Digital sums}). As usual, the hope was to shift the contour of integration by first analytically continuing (\ref{eq:Dirichlet series}) to the left half-plane, and then using the Residue Theorem to obtain to obtain a more enlightening expression for (\ref{eq:nth term of Dirichlet series}). Unfortunately, while the Dirichlet series in (\ref{eq:Dirichlet series}) \emph{do }admit analytic continuations to meromorphic functions on $\mathbb{C}$\textemdash where they have half-lattices of poles to the left of their abscissae of absolute convergence\textemdash these continuations exhibit hyper-exponential growth as $\textrm{Re}\left(s\right)\rightarrow-\infty$. This appears to greatly limit the usefulness of this complex analytic approach. I have unpublished work in this vein; a messy rough draft of which (containing much overlap with this dissertation) can be found on arXiv (see \cite{Mellin transform paper}). I intend to pursue these issues more delicately and comprehensively at some later date. That being said, I briefly revisit this approach at the end of Section \ref{sec:4.1 Preparatory-Work--}, so the curious reader can turn there for a first taste of that approach. \chapter{\label{chap:3 Methods-of--adic}Methods of $\left(p,q\right)$-adic Analysis} \includegraphics[scale=0.45]{./PhDDissertationEroica3.png} \begin{quote} \vphantom{} \begin{flushright} $\ldots$many people feel that functions $\mathbb{Z}_{p}\rightarrow\mathbb{Q}_{p}$ are more interesting than functions $\mathbb{Z}_{p}\rightarrow\mathbb{Q}_{q}$, which is understandable. \par\end{flushright} \begin{flushright} \textemdash W. M. Schikhof\footnote{From \emph{Ultrametric Calculus} (\cite{Ultrametric Calculus}), on the bottom of page 97.} \par\end{flushright} \begin{flushright} People will want to know: what is the \emph{point} of $\left(p,q\right)$-adic functions? You need a good answer. \par\end{flushright} \begin{flushright} \textemdash K. Conrad\footnote{<https://mathoverflow.net/questions/409806/in-need-of-help-with-parsing-non-archimedean-function-theory>} \par\end{flushright} \end{quote} \vphantom{} The purpose of this chapter is to present and develop an analytical theory capable of dealing with functions like $\chi_{H}$. Section \ref{sec:3.1 A-Survey-of} presents the ``classical'' theory, much of which I independently re-discovered before learning that Schikhof\index{Schikhof, W. M.} had already done it\footnote{One of the dangers of working in isolation as I did is the high probability of ``re-inventing the wheel'', as one of my professors (Nicolai Haydn) put it.} all\textemdash and more\textemdash in his own PhD dissertation, back in 1967. Prototypically, the ``classical'' material concerns the study of $C\left(\mathbb{Z}_{p},\mathbb{Q}_{q}\right)$\textemdash the space of continuous functions from $\mathbb{Z}_{p}$ to $\mathbb{Q}_{q}$\textemdash and the related interplay between functional analysis and Fourier analysis in that setting. Section \ref{sec:3.1 A-Survey-of} explains these theoretical details, as well as providing a primer on how to do $\left(p,q\right)$-adic computations. The other two sections of this chapter, however, are new; they extend the ``classical'' material to include functions like $\chi_{H}$. Section \ref{sec:3.2 Rising-Continuous-Functions} introduces the notion of \textbf{rising-continuity}, which provides a natural extension of the $C\left(\mathbb{Z}_{p},\mathbb{Q}_{q}\right)$ theory. Section \ref{sec:3.3 quasi-integrability}\textemdash the largest of the three\textemdash is the most important section of Chapter 3, containing the most significant innovations, as well as the greatest complexities. The central focus of Section \ref{sec:3.3 quasi-integrability} is the phenomenon I call \textbf{quasi-integrability}. Classically, the range of $\left(p,q\right)$-adic which can be meaningfully integrated is astonishingly narrow, consisting solely of continuous functions. By re-interpreting discontinuous functions like $\chi_{H}$ as \emph{measures}, we can meaningfully integrate them, as well as integrate their product with any continuous $\left(p,q\right)$-adic function; this is done in Subsection \ref{subsec:3.3.5 Quasi-Integrability}; the quasi-integrable functions are precisely those $\left(p,q\right)$-adic functions for which this method applies. However, in formulating quasi-integrability, we will have to deal with an extremely unorthodox situation: infinite series of a $p$-adic integer variable $\mathfrak{z}$ which converge at every $\mathfrak{z}\in\mathbb{Z}_{p}$, but for which the topology used to sum the series is allowed to vary from point to point. I introduce the concept of a \textbf{frame }in order to bring order and rigor to this risky, messy business (see Subsection \ref{subsec:3.3.3 Frames}). Subsections \ref{subsec:3.3.1 Heuristics-and-Motivations} and \ref{subsec:3.3.2 The--adic-Dirichlet} contain motivational examples and a connection to the Dirichlet kernel of classical Fourier analysis, respectively, and thereby serve to set the stage for frames and quasi-integrability. Subsection \ref{subsec:3.3.4 Toward-a-Taxonomy} introduces some tedious terminology for describing noteworthy types of $\left(p,q\right)$-adic measures. Its most important features are the \textbf{Fourier Resummation Lemmata}, which will be used extensively in our analysis of $\chi_{H}$ in Chapter 4. Subsection \ref{subsec:3.3.6 L^1 Convergence} briefly introduces another hitherto-ignored area of non-archimedean analysis: the Banach space of functions $\chi:\mathbb{Z}_{p}\rightarrow\mathbb{C}_{q}$ for which the real-valued function $\mathfrak{z}\in\mathbb{Z}_{p}\mapsto\left|\chi\left(\mathfrak{z}\right)\right|_{q}\in\mathbb{R}$ is integrable with respect to the real-valued Haar probability measure on $\mathbb{Z}_{p}$. As will be shown in Subsection \ref{subsec:4.3.2 A-Wisp-of} of Chapter 4, these spaces provide an approachable setting for studying $\chi_{H}$. Whether or not significant progress can be made in that setting remains to be seen. The final subsection of \ref{sec:3.3 quasi-integrability}, Subsection \ref{subsec:3.3.7 -adic-Wiener-Tauberian}, contains the second of the this dissertation's three main results, a $\left(p,q\right)$-adic generalization of \textbf{Wiener's Tauberian Theorem}. I prove two versions, one for continuous $\left(p,q\right)$-adic functions, and another for $\left(p,q\right)$-adic measures. The latter version will be the one used in Chapter 4\textemdash at the end of Subsection \ref{subsec:4.2.2}\textemdash to establish the \textbf{Tauberian Spectral Theorem }for $p$-Hydra maps, my dissertation's third main result. A \emph{sub}-subsection at the end of \ref{subsec:3.3.7 -adic-Wiener-Tauberian} shows how the study of $\chi_{H}$ can be $q$-adically approximated by the study of the eigenvalues of a family of increasingly large Hermitian matrices. \section{\label{sec:3.1 A-Survey-of}A Survey of the ``Classical'' theory} Due to the somewhat peculiar position currently occupied by $\left(p,q\right)$-adic analysis\index{$p,q$-adic!analysis}\textemdash too exotic to be thoroughly explored, yet too specific to merit consideration at length by specialists in non-archimedean analysis\textemdash I have taken the liberty of providing a brief essay on the historical and theoretical context of the subject and its various flavors; this can be found in Subsection \ref{subsec:3.1.1 Some-Historical-and}. Before that, however, I think it will be helpful to point out the most important (and bizarre) results of $\left(p,q\right)$-adic analysis, particularly for the benefit of readers who are used to working with real- or complex-valued functions. The main features are: \begin{itemize} \item The existence of a translation-invariant $\mathbb{C}_{q}$-valued ``measure'' on $\mathbb{Z}_{p}$, unique up to a choice of a normalization constant, and compatible with an algebra of sets, albeit not the Borel $\sigma$-algebra. \index{Borel!sigma-algebra@$\sigma$-algebra}Moreover, this measure (the \textbf{$\left(p,q\right)$-adic Haar measure}) is a continuous linear functional on $C\left(\mathbb{Z}_{p},\mathbb{Q}_{q}\right)$. This is very good news for us, because it means we can integrate and do Fourier analysis in the $\left(p,q\right)$-adic setting. \item The lack of a concept of ``almost everywhere\emph{''}. Although a substantive theory of $\left(p,q\right)$-adic measures exists, it has no ``almost everywhere\emph{''}.\emph{ }In fact, unless the normalization constant of the $\left(p,q\right)$-adic Haar measure is chosen to be $0$, \emph{the only measurable set of measure $0$ is the empty set!} \item Because there is no way to take a difference quotient for a $\left(p,q\right)$-adic function (how do you divide a $q$-adic number by a $p$-adic number?), there is no differential calculus\footnote{However, my methods allow for us to define a $\left(p,q\right)$-adic Mellin transform (see \textbf{Remark \ref{rem:pq adic mellin transform}} on page \pageref{rem:pq adic mellin transform}). Since the Mellin transform is used to formulate differentiation in the distributional sense for functions $f:\mathbb{Q}_{p}\rightarrow\mathbb{C}$, it seems quite likely that that same approach, done in the $\left(p,q\right)$-adic context, can be used to give $\left(p,q\right)$-adic analysis a concept of distributional derivatives, and thereby open the door to analytical investigation of $\left(p,q\right)$-adic differential equations.} in $\left(p,q\right)$-adic analysis. There are also no such things as polynomials, rational functions, or analytic functions. \item \emph{A function is integrable if and only if it is continuous!} This equivalence is intimately connected to the absence of non-empty sets of measure zero. \item We will have to leave the triangle inequality for integrals to wait for us outside by the door; it has no place in $\left(p,q\right)$-adic analysis. In this subject, there is generally no meaningful relationship between $\left|\int f\right|_{q}$ and $\int\left|f\right|_{q}$. As much as we would want to write $\left|\int f\right|_{q}\leq\int\left|f\right|_{q}$, it simply isn't justified. \textbf{Example \ref{exa:triangle inequality failure} }on page \pageref{exa:triangle inequality failure} provides the unfortunate details. \end{itemize} A first encounter with the theory of $\left(p,q\right)$-adic Fourier analysis is a surreal experience. Indeed, in the ``classical'' $\left(p,q\right)$-adic theory, given a $\left(p,q\right)$-adic function $f:\mathbb{Z}_{p}\rightarrow\mathbb{C}_{q}$ (where, of course, $p\neq q$),\emph{ the following are equivalent}: \begin{enumerate} \item $f$ possesses a well-defined Fourier transform $\hat{f}$. \item $f$ is integrable. \item $f$ is continuous. \item The Fourier series representation of $f$ (using $\hat{f}$) is \emph{absolutely convergent }uniformly over $\mathbb{Z}_{p}$. \end{enumerate} These three wonders\textemdash especially the spectacular equivalence of (1) through (4)\textemdash are, among some other factors, the principal reasons why $\left(p,q\right)$-adic analysis has lain neglected for the over-half-a-century that has elapsed since its inception. The subject's rigid inflexibility and astonishing lack of subtlety goes a long way toward explaining why it has been consigned to eek out a lonely existence in the cabinet of mathematical curiosities. Of course, part of the purpose of this dissertation is to argue that this consignment was premature. Lastly, before freeing the reader to pursue the exposition I have prepared for them, let me first give an exposition for that exposition, which is somewhat non-standard, to the extent that there even \emph{is }a ``standard'' pedagogical approach to $\left(p,q\right)$-adic analysis. One of the main features of non-archimedean analysis as a subject is that the available integration theories will depend on the fields used for the functions' domains and codomains. Of these, the one we will use is what Khrennikov calls the \textbf{Monna-Springer integral} \cite{Quantum Paradoxes}. Schikhof in an appendix of \emph{Ultrametric Analysis} \cite{Ultrametric Calculus} and van Rooij in a chapter of \emph{Non-Archimedean Functional Analysis} \cite{van Rooij - Non-Archmedean Functional Analysis} also give expositions of this theory, though not by that name. For the probability-minded, Khrennikov gives a second, full-throated presentation of the integration theory in his article \cite{Probabilities taking values in non-archimedean fields} on non-archimedean probability theories\index{non-archimedean!probability theory}. As anyone who has gone through a standard course on integration and measures can attest, presentations of measure theory and measure-theoretic integration often lean toward the abstract. The same is true for the non-archimedean case. In certain respect, the non-archimedean abstraction outdoes the archimedean. These presentations typically begin with one of the following: \begin{itemize} \item An explanation of how the set-based approach to measure theory utilized in most university courses is not compatible with the non\textendash archimedean case. Instead, they turn to the functional analysis approach advocated by the Bourbaki group, defining measures as linear functionals satisfying a certain norm condition. \item To get around the fact that Borel sets aren't well-suited to form a set-based theory of measures, the presentation takes as a primitive notion the existence of a collection of sets satisfying most of the properties you would expect, along with a few extra technicalities needed to avoid disaster. \end{itemize} Although we will \emph{eventually }get to this sort of material (see Subsection \ref{subsec:3.1.6 Monna-Springer-Integration}), because of the paramount importance of explicit, specific, \emph{concrete} computations for this dissertation, I have weighted my presentation toward such practical matters. With the exception of \ref{subsec:3.1.6 Monna-Springer-Integration}, I have tried to keep the level of generality to an absolute minimum, so as to allow the basic mechanics to shine through as clearly as possible. After the historical essay in Subsection \ref{subsec:3.1.1 Some-Historical-and}, the exposition begins in Subsection \ref{subsec:3.1.2 Banach-Spaces-over} with a sliver of the more general theory of Banach spaces over a non-archimedean field. Subsection \ref{subsec:3.1.3 The-van-der} introduces the \textbf{van der Put basis} for spaces of functions on $\mathbb{Z}_{p}$, which will be used throughout this section to explicitly construct spaces like $C\left(\mathbb{Z}_{p},\mathbb{Q}_{q}\right)$ and compute integrals and Fourier transforms. After this, we explore the $\left(p,q\right)$-adic Fourier transform of a continuous $\left(p,q\right)$-adic function (Subsection \ref{subsec:3.1.4. The--adic-Fourier}), and then use it to construct $\left(p,q\right)$-adic measures. The initial viewpoint will be of a measure as an object defined by its \textbf{Fourier-Stieltjes transform} (Subsection \ref{subsec:3.1.5-adic-Integration-=00003D000026}). This view of $\left(p,q\right)$-adic measures will be very important in Section \ref{sec:3.3 quasi-integrability} and throughout Chapters \ref{chap:4 A-Study-of} and \ref{chap:6 A-Study-of}. In the process, we will see how to compute continuous $\left(p,q\right)$-adic functions' Fourier transforms\textemdash two different formulae will be given\textemdash along with how to integrate functions with respect to $\left(p,q\right)$-adic measures, convolutions, and the basic\textemdash but fundamental\textemdash change of variable formula for affine transformations ($\mathfrak{y}=\mathfrak{a}\mathfrak{z}+\mathfrak{b}$) in integrals. We will constantly draw from this practical information, knowledge of which is essential for further independent study of $\chi_{H}$ and related matters, We conclude our survey by returning to the abstract with Subsection \ref{subsec:3.1.6 Monna-Springer-Integration}, giving an exposition of Monna-Springer integration theory. This provides the ``correct'' generalization of the explicit, concrete methods of $\left(p,q\right)$-adic integration dealt with in the earlier subsections. As a pedagogical approach, I feel that grounding the Monna-Springer integral in the concrete case of $\left(p,q\right)$-adic Fourier series and $\left(p,q\right)$-adic Fourier-Stieltjes transforms makes the abstract version less intimidating. That being said, my inclusion of the Monna-Springer theory is primarily for completeness' sake. $\left(p,q\right)$-adic analysis is extraordinarily pathological, even by the bizarre standards of non-archimedean analysis. Familiar results such as the Dominated Convergence Theorem or Hlder's Inequality reduce to trivialities in the $\left(p,q\right)$-adic setting. \subsection{\label{subsec:3.1.1 Some-Historical-and}Some Historical and Theoretical Context} In the title of Section \ref{sec:3.1 A-Survey-of}, the word ``classical'' is enclosed in scare quotes. This seems like a fair compromise to me. On the one hand, in $\left(p,q\right)$-adic analysis we have a nearly-sixty-year-old subject ripe for generalization; on the other, to the extent that a well-articulated theory of $\left(p,q\right)$-adic analysis exists, it is too esoteric and (understandably) under-appreciated to truly merit the status of a ``classic''. Until now, the $\left(p,q\right)$-adic analysis chronicled in this Chapter has smoldered in double disregard. Why, indeed, would anyone in their right mind want to investigate a subject which is too exotic and inert for the general community, but also too \emph{insufficiently }general to have earned clear place in the work of the exotic specialists? Somewhat surprisingly, the same could be said for the $p$-adic numbers themselves. Kurt Hensel first published his discovery of $p$-adic numbers in 1897 \cite{Hensel-s original article,Gouvea's p-adic number history slides,Journey throughout the history of p-adic numbers}. At the time, they did not make much of a splash. The eventual inclusion of $p$-adic numbers in the mathematical canon came about purely by serendipity, with Helmut Hasse in the role of the inadvertent midwife. It would make for an interesting scene in a Wes Anderson film: in 1913, Hensel publishes a book on the $p$-adics; sometime between then and 1920, a copy of said book managed to find way to a certain antique shop in Gttingen. In 1920, Hasse\textemdash a then-student at the University of Gttingen\textemdash stumbled across said book in said antique shop while wandering about town as graduate students in Gttingen are wont to do when they feel a hankering for a breath of fresh air. Through this chain of events, Hasse chose to transfer to the University of Marburg, at which Hensel taught, with the express purpose of learning everything he could about Hensel's marvelous, star-crossed innovation. The result? In 1922, Hasse publishes his doctoral thesis, in which he establishes what is now called the \textbf{Hasse-Minkowski Theorem}, otherwise known as the \textbf{local-global principle} \textbf{for quadratic forms }\cite{Gouvea's p-adic number history slides,On the origins of p-adic analysis}. For the uninitiated, the local-global principle is the aspiration that one can find construct a tuple of rational numbers which solve a given polynomial equation in several variables by solving the equation in $\mathbb{R}$ as well as in the $p$-adics for all primes $p$, and then using the Chinese Remainder Theorem to stitch together the real and $p$-adic solutions into a solution over $\mathbb{Q}$. (Conrad's online notes \cite{Conrad - Local Global Principle} give an excellent demonstration of this method in practice.) Fascinatingly, although the Hasse-Minkowski Theorem assures us this approach will work in polynomials for which each variable has an exponent of at most $2$, the local-global principle \emph{does not }apply for polynomial equations of arbitrary degree. A particularly well-known counterexample is due to Selmer \cite{Conrad - Local Global Principle,Local-Global Principle Failure}: \begin{thm}[\textbf{\textit{Selmer's Counterexample}}] The cubic diophantine equation: \begin{equation} 3x^{3}+4y^{3}+5z^{3}=0 \end{equation} has solutions $\left(x,y,z\right)\neq\left(0,0,0\right)$ over $\mathbb{R}$ and over $\mathbb{Q}_{p}$ for every prime $p$, yet its only solution over $\mathbb{Q}$ is $\left(x,y,z\right)=\left(0,0,0\right)$. \end{thm} \vphantom{} Understanding the conditions where it does or does not apply is still an active area of research in arithmetic geometry. Indeed, as Conrad points out, the famous \textbf{Birch and Swinnerton-Dyer Conjecture }concerns, in part, a relation between the rational points of an elliptic curve $E$ over $\mathbb{Q}$ and the points of $E$ over $\mathbb{R}$ and the $p$-adics \cite{Conrad - Local Global Principle}. The local-global philosophy has since had an explosive influence in mathematics, above all in algebraic number theory and adjacent fields, where it opened the door to the introduction of $p$-adic and adlic approaches. A central protagonist of this story is the late, great John Tate (1925 \textendash{} 2019)\index{Tate, John}. In his epochal thesis of 1950 (``\emph{Fourier analysis in number fields and Hecke's zeta functions}''), Tate revolutionized algebraic number theory by demonstrating how the \textbf{Euler Products} for $L$-functions such as the Riemann Zeta Function: \begin{equation} \zeta\left(s\right)\overset{\mathbb{C}}{=}\prod_{p\in\mathbb{P}}\frac{1}{1-p^{-s}},\textrm{ }\forall\textrm{ }\textrm{Re}\left(s\right)>1\label{eq:Euler Product for the RZF} \end{equation} (where $\mathbb{P}$ denotes the set of all prime numbers) could be understood in the local-global spirit as the product of so-called \textbf{local factors} associated to each prime $p$ \cite{Tate's thesis}. This recontextualization occurred by way of Mellin-transform type integrals over the $p$-adic numbers. Letting $d\mathfrak{z}_{p}$ denote the real-valued Haar probability measure on $\mathbb{Z}_{p}$, the aforementioned local factors of (\ref{eq:Euler Product for the RZF}) can be expressed by the integral: \begin{equation} \frac{1}{1-p^{-s}}\overset{\mathbb{C}}{=}\frac{p}{p-1}\int_{\mathbb{Z}_{p}}\left|\mathfrak{z}_{p}\right|_{p}^{s-1}d\mathfrak{z}_{p},\textrm{ }\forall\textrm{Re}\left(s\right)>1 \end{equation} This can also be re-written as: \begin{equation} \frac{1}{1-p^{-s}}\overset{\mathbb{C}}{=}\int_{\mathbb{Z}_{p}}\left|\mathfrak{z}_{p}\right|_{p}^{s-1}d^{\times}\mathfrak{z}_{p},\textrm{ }\forall\textrm{Re}\left(s\right)>1\label{eq:RZF local factors} \end{equation} where: \begin{equation} d^{\times}\mathfrak{z}_{p}\overset{\textrm{def}}{=}\frac{p}{p-1}d\mathfrak{z}_{p} \end{equation} is the real-valued Haar probability measure for the group $\left(\mathbb{Z}_{p}^{\times},\times\right)$ of multiplicatively invertible elements of $\mathbb{Z}_{p}$. Taking the product of the local factors (\ref{eq:RZF local factors}) over all primes $p$, the tensor product transforms the product of the integrals into a single integral over the profinite integers, $\check{\mathbb{Z}}$\nomenclature{$\check{\mathbb{Z}}$}{the ring of profinite integers, $\prod_{p\in\mathbb{P}}\mathbb{Z}_{p}$ \nopageref}: \begin{equation} \zeta\left(s\right)\overset{\mathbb{C}}{=}\int_{\check{\mathbb{Z}}}\left|\mathfrak{z}\right|^{s}d^{\times}\mathfrak{z}\overset{\textrm{def}}{=}\prod_{p\in\mathbb{P}}\int_{\mathbb{Z}_{p}}\left|\mathfrak{z}_{p}\right|_{p}^{s-1}d^{\times}\mathfrak{z}_{p},\textrm{ }\forall\textrm{Re}\left(s\right)>1 \end{equation} Here: \begin{equation} \check{\mathbb{Z}}\overset{\textrm{def}}{=}\prod_{p\in\mathbb{P}}\mathbb{Z}_{p}\label{eq:Definition of the Profinite Integers} \end{equation} and so, every $\mathfrak{z}\in\check{\mathbb{Z}}$ is a $\mathbb{P}$-tuple $\left(\mathfrak{z}_{2},\mathfrak{z}_{3},\mathfrak{z}_{5},\ldots\right)$ of $p$-adic integers, one from each $p$. $d^{\times}\mathfrak{z}$, meanwhile, is the Haar probability measure for the multiplicative group of units of $\check{\mathbb{Z}}$. The significance of this new perspective and its impact on subsequent developments in number theory cannot be overstated. Indeed, in hindsight, it is hard to believe that anyone could have ever doubted the value of the $p$-adics to mathematics. But the story does not end here. If anything, Tate's work is like an intermezzo, bridging two worlds by transmuting the one into the other. Starting in the mid-twentieth century, mathematical analysis began to pick up where Hensel left off \cite{Gouvea's p-adic number history slides,Journey throughout the history of p-adic numbers}, exploring the $p$-adic numbers' ultrametric structure and the implications it would have for analysis and its theories in those settings\textemdash continuity, differentiability, integrability, and so on. Although Hensel himself had taken the first steps toward investigating the properties of functions of his $p$-adic numbers, a general programme for analysis on valued fields other than $\mathbb{R}$ or $\mathbb{C}$\textemdash independent of any number-theoretic investigations was first posed by A. F. Monna\index{Monna, A. F.} in a series of papers in 1943 \cite{van Rooij - Non-Archmedean Functional Analysis}. From this grew the subject that would come to be known as \emph{non-archimedean analysis}. In this sense, non-archimedean analysis\textemdash as opposed to the more number-theoretically loaded term ``$p$-adic analysis''\textemdash has its origins in the activities of graduate students, this time Netherlands in the 1960s, when W. M. Schikhof, A. C. M. van Rooij\index{van Rooij, A. C. M.}, M. van der Put\index{van der Put, Marius}, and J. van Tiel established a so-called ``$p$-adics and fine dining'' society, intent on exploring the possibility of analysis\textemdash particularly \emph{functional} analysis\textemdash over non-archimedean valued fields. The subject is also sometimes called \emph{ultrametric analysis}\footnote{In this vein, one of the subject's principal introductory texts is called \emph{Ultrametric Calculus} \cite{Ultrametric Calculus}.}, to emphasize the fundamental distinction between itself and classical analysis. In the grand scheme of things, non-archimedean valued fields aren't as wild as one might think. As was proved by Ostrowski in 1915, the only valued fields of characteristic zero obtainable as metric completions of $\mathbb{Q}$ are $\mathbb{R}$, $\mathbb{C}$, and the $p$-adics. A necessary and immediate consequence of this straight-forward classification is that non-archimedean analysis comes in four different flavors depending on the nature of the fields we use for our domains and co-domains. Let $\mathbb{F}$ and $K$ be non-archimedean valued fields, complete with respect to their absolute values. Moreover, let the residue fields of $\mathbb{F}$ and $K$ have different characteristic; say, let $\mathbb{F}$ be a $p$-adic field, and let $K$ be a $q$-adic field. By combinatorics, note that there are really only four\emph{ }different types of non-archimedean functions we can consider: functions from an archimedean field to a non-archimedean field ($\mathbb{C}\rightarrow\mathbb{F}$); from a non-archimedean field to an archimedean field ($\mathbb{F}\rightarrow\mathbb{C}$); from \emph{and} to the same non-archimedean field ($\mathbb{F}\rightarrow\mathbb{F}$); and those between two archimedean fields with non-equal residue fields ($\mathbb{F}\rightarrow K$). In order, I refer to these as \textbf{$\left(\infty,p\right)$-adic analysis} ($\mathbb{C}\rightarrow\mathbb{F}$), \textbf{$\left(p,\infty\right)$-adic analysis} ($\mathbb{F}\rightarrow\mathbb{C}$), \textbf{$\left(p,p\right)$-adic analysis} ($\mathbb{F}\rightarrow\mathbb{F}$), and \textbf{$\left(p,q\right)$-adic analysis} ($\mathbb{F}\rightarrow K$). In this terminology, real and complex analysis would fall under the label \textbf{$\left(\infty,\infty\right)$-adic analysis}. The first flavor\textemdash $\left(\infty,p\right)$-adic analysis ($\mathbb{C}\rightarrow\mathbb{F}$)\textemdash I must confess, I know little about, and have seen little, if any, work done with it. Hopefully, this will soon be rectified. Each of the remaining three, however, merit deeper discussion. \index{analysis!left(p,inftyright)-adic@$\left(p,\infty\right)$-adic}The second flavor of non-archimedean analysis\textemdash $\left(p,\infty\right)$-adic analysis ($\mathbb{F}\rightarrow\mathbb{C}$) is the \emph{least} exotic of the three. Provided that $\mathbb{F}$ is locally compact (so, an at most finite-degree extension of $\mathbb{Q}_{p}$), this theory is addressed by modern abstract harmonic analysis, specifically through \textbf{Pontryagin duality} and the theory of Fourier analysis on a locally compact abelian group. Tate's Thesis is of this flavor. In addition to Tate's legacy, the past thirty years have seen a not-insignificant enthusiasm in $\left(p,\infty\right)$-adic analysis from theoretical physics, of all places \cite{First 30 years of p-adic mathematical physics,Quantum Paradoxes,p-adic space-time} (with Volovich's 1987 paper \cite{p-adic space-time} being the ``classic'' of this burgeoning subject). This also includes the adlic variant of $\left(p,\infty\right)$-adic analysis, where $\mathbb{F}$ is instead replaced with $\mathbb{A}_{\mathbb{Q}}$, the \textbf{adle ring }of $\mathbb{Q}$; see \cite{Adelic Harmonic Oscillator}, for instance, for an \emph{adlic} harmonic oscillator. This body of work is generally known by the name $p$-adic (mathematical) physics and $p$-adic quantum mechanics, and is as mathematically intriguing as it is conceptually audacious. The hope in the depths of this Pandora's box is the conviction that non-archimedean mathematics might provide a better model for reality at its smallest scales. On the side of pure mathematics, $\left(p,\infty\right)$-adic analysis is a vital tool in representation theory (see, for instance \cite{Automorphic Representations}). In hindsight, this is not surprising: the abstract harmonic analysis used to formulate Fourier theory over locally compact abelian groups must appeal to representation theory in order to formulate Fourier analysis over an arbitrary compact group. Of the many flavors of non-archimedean analysis, I feel that only the third\textemdash \index{analysis!left(p,pright)-adic@$\left(p,p\right)$-adic}$\left(p,p\right)$-adic anlaysis ($\mathbb{F}\rightarrow\mathbb{F}$)\textemdash truly deserves to be called ``\index{analysis!$p$-adic}$p$-adic analysis''. Of course, even this flavor has its own variations. One can study functions $\mathbb{Z}_{p}\rightarrow\mathbb{F}$, as well as functions $\mathbb{F}\rightarrow\mathbb{F}^{\prime}$, where $\mathbb{F}^{\prime}$ is a field extension of $\mathbb{F}$, possibly of infinite degree. The utility, flexibility, adaptability, and number-theoretic import of $\left(p,p\right)$-adic analysis have all contributed to the high esteem and widespread popularity this subject now enjoys. $\left(p,p\right)$-adic analysis can be studied in its own right for a single, fixed prime $p$, or it can be used in conjunction with the local-global philosophy, wherein one does $\left(p,p\right)$-adic analysis for many\textemdash even infinitely many\textemdash different values of $p$ simultaneously. This synergy is on full display in one of $\left(p,p\right)$-adic analysis most famous early triumphs: Bernard Dwork\index{Dwork, Bernard}'s proof (\cite{Dwork Zeta Rationality}) of the rationality of the zeta function for a finite field\textemdash the first of the three parts of the \textbf{Weil Conjectures }to be proven true \cite{Dwork Zeta Rationality,Koblitz's other book,Robert's Book,p-adic proof for =00003D0003C0}. Much like the local-global principle itself, Dwork's application of $p$-adic analysis is a stratospheric elevation of the Fundamental Theorem of Arithmetic: every integer $\geq2$ is uniquely factorable as a product of primes. This is evident in the so-called \textbf{Product Formula} of algebraic number theory, applied to $\mathbb{Q}$: \begin{equation} \left|a\right|_{\infty}\cdot\prod_{p\in\mathbb{P}}\left|a\right|_{p}=1,\textrm{ }\forall a\in\mathbb{Q}\backslash\left\{ 0\right\} \label{eq:The Product Formula} \end{equation} where $\left|a\right|_{p}$ is the $p$-adic absolute value of $a$, and $\left|a\right|_{\infty}$ is the usual absolute value (on $\mathbb{R}$) of $a$. Indeed, (\ref{eq:The Product Formula}) is but a restatement of the Fundamental Theorem of Arithmetic. The key tool in Dwork's proof was the following theorem: \begin{thm}[\textbf{Dwork}\footnote{Cited in \cite{p-adic proof for =00003D0003C0}.}] Let $f\left(z\right)=\sum_{n=0}^{\infty}a_{n}z^{n}$ be a power series in the complex variable $\mathbb{C}$ with coefficients $\left\{ a_{n}\right\} _{n\geq0}$ in $\mathbb{Q}$. Let $S$ be a set of finitely many places\footnote{For the uninitiated, a ``place'' is a fancy term for a prime number. In algebraic number theory, primes\textemdash and, more generally, prime \emph{ideals}\textemdash of a given field or ring are associated with the valuations they induce. Due to a technicality, much like a Haar measure on a group, valuations are not unique. The absolute value $\mathfrak{z}\in\mathbb{Z}_{2}\mapsto3^{-v_{2}\left(\mathfrak{z}\right)}$, for example, defines a topology on $\mathbb{Z}_{2}$ which is equivalent to that of the standard $2$-adic absolute value. The valuation associated to this modified absolute value would be $\mathfrak{z}\mapsto v_{2}\left(\mathfrak{z}\right)\log_{2}3$. A \textbf{place}, then, is an equivalence class of valuations associated to a given prime ideal, so that any two valuations in the class induce equivalent topologies on the ring or field under consideration.} of $\mathbb{Q}$ which contains the archimedean complex place\footnote{Meaning the valuation corresponding to the ordinary absolute value}. If $S$ satisfies: \vphantom{} I. For any\footnote{Here, $p$ represents \emph{both} a prime number and the associated non-archimedean absolute value.} $p\notin S$, $\left|a_{n}\right|_{p}\leq1$ for all $n\geq0$ (i.e., $a_{n}\in\mathbb{Z}_{p}$); \vphantom{} II. For any\footnote{Here, $v$ is a absolute value on $\mathbb{Q}$, and $\mathbb{C}_{v}$ is the algebraic closure of the $v$-adic completion of $\mathbb{Q}$.} $p\in S$, there is a disk $D_{p}\subseteq\mathbb{C}_{p}$ of radius $R_{p}$ so that $f\left(z\right)$ extends to a meromorphic function $D_{p}\rightarrow\mathbb{C}_{p}$ and that $\prod_{p\in S}R_{p}>1$; \vphantom{} Then $f\left(z\right)$ is a rational function. \end{thm} \vphantom{} This is a $p$-adic generalization of an observation, first made by Borel, that if a power series $\sum_{n=0}^{\infty}a_{n}z^{n}$ with \emph{integer} coefficients has a radius of convergence strictly greater than $1$, the power series is necessarily\footnote{Proof: let $R$ be the radius of convergence, and suppose $R>1$. Then, there is an $r\in\left(1,R\right)$ for which $f\left(z\right)=\sum_{n=0}^{\infty}a_{n}z^{n}$ converges uniformly for $\left|z\right|=r$, and hence, by the standard Cauchy estimate: \[ \left|a_{n}\right|=\left|\frac{f^{\left(n\right)}\left(0\right)}{n!}\right|\leq\frac{1}{r^{n}}\sup_{\left|z\right|=r}\left|f\left(z\right)\right| \] Since $r>1$, the upper bound will be $<1$ for all sufficiently large $n$. However, if the $a_{n}$s are all integers, then any $n$ for which $\left|a_{n}\right|<1$ necessarily forces $a_{n}=0$. Hence $f$ is a polynomial.} a polynomial. It is a testament to the power of Dwork's Theorem\textemdash via a generalization due Bertrandias\textemdash that it can be used to prove the \index{Lindemann-Weierstrass Theorem}\textbf{Lindemann-Weierstrass Theorem}\textemdash the Lindemann-Weierstrass Theorem being the result that proved $\pi$\index{transcendence of pi@transcendence of $\pi$} to be a transcendental number, thereby resolving the millennia-old problem of squaring the circle\textemdash the answer being a resounding \emph{no}. See \cite{p-adic proof for =00003D0003C0} for a lovely exposition of this argument. The fourth, most general flavor of non-archimedean analysis concerns functions between arbitrary non-archimedean valued fields. The subject has gone even further, studying functions $\Omega\rightarrow\mathbb{F}$, where $\Omega$ is any appropriately well-behaved Hausdorff space. Of the different flavors, I feel this is the one most deserving of the title ``\textbf{non-archimedean analysis}\index{analysis!non-archimedean}''.\textbf{ }A good deal of the work done in non-archimedean analysis is in the vein of so-called ``soft analysis'' variety, using the abstract and often heavily algebraified language common to modern real or complex functional analysis\textemdash ``maximal ideals'', ``convex sets'', ``nets'', and the like\textemdash to investigate exotic generalities. Aside from generality's sake, part of the reason for the lack of a significant distinction in the literature between non-archimedean analysis and non-archimedean \emph{functional }analysis is one the central difficulties of non-archimedean analysis: \emph{the absence of a ``nice'' analogue of differentiation in the classical sense.} In the navet of my youth, I dared to dream that calculus in multiple variables\textemdash once I learned it\textemdash would be every bit as elegant and wondrous as its single-variable counterparts. Inevitably, I was disabused of this notion. When it comes to differential calculus, single-variable $\left(p,\infty\right)$-adic analysis (ex: $\mathbb{Z}_{p}\rightarrow\mathbb{C}$) and non-archimedean analysis (such as $\mathbb{Z}_{p}\rightarrow\mathbb{C}_{q}$) manage to surpass multi-variable real analysis in theoretical complexity without needing to draw on even half as much indecency of notation. The main problem in these settings is that there is no way to define a \emph{difference quotient }for functions. The topologies and algebraic operations of functions' inputs and outputs are \emph{completely} different. You can't divide a complex number by a $p$-adic integer, and so on and so forth. On the other hand, the theory of integration rests on much firmer ground. Thanks to Haar measures and Pontryagin Duality, integration in $\left(p,\infty\right)$-adic analysis falls squarely under the jurisdiction of standard measure theory and abstract harmonic analysis. In fact, the parallels between $\left(p,\infty\right)$-adic Fourier analysis and classical Fourier analysis on tori or euclidean space are so nice that, in 1988, Vasilii Vladimirov was able to use Fourier analysis to introduce $\left(p,\infty\right)$-adic differentiation in the sense of distributions. This was done by co-opting the classic \emph{complex-valued }$p$-adic Mellin integral \cite{Vladimirov - the big paper about complex-valued distributions over the p-adics}: \begin{equation} \int_{\mathbb{Q}_{p}}\left|\mathfrak{z}\right|_{p}^{s-1}f\left(\mathfrak{z}\right)d\mathfrak{z} \end{equation} where $d\mathfrak{z}$ is the \emph{real-valued} Haar measure on $\mathbb{Q}_{p}$, normalized to be a probability measure on $\mathbb{Z}_{p}$ and where $s$ is a complex variable. Here, $f$ is a complex-valued function on $\mathbb{Q}_{p}$, and\textemdash for safety\textemdash we assume $f$ is compactly supported. The fruit of this approach is the \index{fractional differentiation}\index{Vladimirov operator}\textbf{ Vladimirov (Fractional) Differentiation / Integration operator} of order $\alpha$ (where $\alpha$ is a real number $>0$). For a function $f:\mathbb{Q}_{p}\rightarrow\mathbb{C}$, this is given by the integral: \begin{equation} D^{\alpha}\left\{ f\right\} \left(\mathfrak{z}\right)\overset{\textrm{def}}{=}\frac{1}{\Gamma_{p}\left(-\alpha\right)}\int_{\mathbb{Q}_{p}}\frac{f\left(\mathfrak{z}\right)-f\left(\mathfrak{y}\right)}{\left|\mathfrak{z}-\mathfrak{y}\right|_{p}^{1+\alpha}}d\mathfrak{y}\label{eq:Definition of the Vladimirov Fractional Differentiation Operator} \end{equation} provided the integral exists. Here, $\Gamma_{p}$ is the physicists' notation for: \begin{equation} \Gamma_{p}\left(-\alpha\right)\overset{\textrm{def}}{=}\frac{p^{\alpha}-1}{1-p^{-1-\alpha}}\label{eq:Physicist's gamma_p} \end{equation} This operator can be extended to negative $\alpha$ by being rewritten so as to produce the $\left(p,\infty\right)$-adic equivalent of the usual, Fourier-transform-based definition of fractional derivatives in fractional-order Sobolev spaces. As for \emph{non-archimedean} analysis (including $\left(p,q\right)$), there is a well developed theory of integration, albeit with a strong functional analytic flavor. Integrals and measures are formulated not as $K$-valued functions of sets, but continuous, $K$-valued linear functionals on the Banach space of continuous functions $X\rightarrow K$, where $X$ is an ultrametric space and $K$ is a metrically complete non-archimedean field. An exposition of this theory is given in Subsection \ref{subsec:3.1.6 Monna-Springer-Integration}. To return to physics for a moment, one of the principal \emph{mathematical }reasons for interest in $p$-adic analysis among quantum physicists has been the hope of realize a theory of probability based on axioms other than Kolmogorov's. The \emph{ide fixe }of this approach is that sequences of rational numbers (particularly those representing frequencies of events) which diverge in $\mathbb{R}$ can be convergent in $\mathbb{Q}_{p}$ for an appropriate choice of $p$. The hope is that this will allow for a resurrection of the pre-Kolmogorov frequency-based probability theory, with the aim of using the frequency interpretation to assign probabilistic meaning to sequences of rational numbers which converge in a $p$-adic topology. Andrei Khrennikov and his associates have spearheaded these efforts; \cite{Measure-theoretic approach to p-adic probability theory} is an excellent (though dense) exposition of such a non-archimedean probability theory, one which strives to establish a more classical measure-based notion of probability. Khrennikov's eclectic monograph \cite{Quantum Paradoxes} also makes for a fascinating read, containing much more information, and\textemdash most importantly\textemdash worked examples from both physics and probability. Now for the bad news. Both $\left(p,\infty\right)$-adic and non-archimedean analysis are effectively incompatible with any notion of analytic functions; there are no power series, nor rational functions. $\left(p,p\right)$-adic analysis \emph{does not }suffer this limitation, which is another reason for its longstanding success. Even so, this theory of $p$-adic analytic functions still managed to get itself bewitched when reality was in the drafting stage: the non-archimedean structure of the $p$-adics makes even the most foundational principles of classical analytic function theory completely untenable in the $p$-adic setting. While one \emph{can} define the derivative of a function, say, $f:\mathbb{Z}_{p}\rightarrow\mathbb{Q}_{p}$ as the limit of difference quotient: \begin{equation} f^{\prime}\left(\mathfrak{z}\right)=\lim_{\mathfrak{y}\rightarrow\mathfrak{z}}\frac{f\left(\mathfrak{y}\right)-f\left(\mathfrak{z}\right)}{\mathfrak{y}-\mathfrak{z}}\label{eq:Naive differentiability} \end{equation} doing so courts disaster. Because there is sense in the world, this notion of ``derivative\index{derivative}'' \emph{does }work for analytic functions. However, it fails \emph{miserably }for everything else. Yes, this kind of differentiability still implies continuity\textemdash that much, we are allowed to keep. Nevertheless, there are simple examples of \emph{differentiable functions $f:\mathbb{Z}_{p}\rightarrow\mathbb{Q}_{p}$ }(in the sense that (\ref{eq:Naive differentiability}) exists at every point)\emph{ which are injective, but whose derivatives are everywhere zero!} \begin{example} \label{exa:p-adic differentiation is crazy}Given\footnote{As a minor spoiler, this example proves to be useful to one of my ideas for studying $\chi_{H}$, because \emph{of course }it would.} any $\mathfrak{z}\in\mathbb{Z}_{p}$, write $\mathfrak{z}$ as $\sum_{n=0}^{\infty}\mathfrak{z}_{n}p^{n}$, where, $\mathfrak{z}_{n}\in\left\{ 0,\ldots,p-1\right\} $ is the $n$th $p$-adic digit of $\mathfrak{z}$, and consider the function\footnote{Amusingly, this function will turn out to be relevant to studying Collatz and the like; see the lead-up to \textbf{Example \ref{exa:L^1 method example}} (page \pageref{exa:L^1 method example}) from Subsection \ref{subsec:4.3.4 Archimedean-Estimates} in Chapter 4 for the details.} $\phi:\mathbb{Z}_{p}\rightarrow\mathbb{Q}_{p}$ defined by: \begin{equation} \phi\left(\mathfrak{z}\right)=\sum_{n=0}^{\infty}\mathfrak{z}_{n}p^{2n}\label{eq:Phi for the L^1 method} \end{equation} Because $\left|\mathfrak{z}-\mathfrak{y}\right|_{p}=1/p^{n+1}$, where $n$ is the largest integer $\geq0$ for which $\mathfrak{z}_{n}=\mathfrak{y}_{n}$ (with the convention that $n=-1$ if no such $n$ exists), we have that: \begin{equation} \left|\phi\left(\mathfrak{z}\right)-\phi\left(\mathfrak{y}\right)\right|_{p}=\frac{1}{p^{2\left(n+1\right)}}=\left|\mathfrak{z}-\mathfrak{y}\right|_{p}^{2} \end{equation} Thus, $\phi$ is continuous, injective, and differentiable, with: \begin{equation} \left|\phi^{\prime}\left(\mathfrak{z}\right)\right|_{p}=\lim_{\mathfrak{y}\rightarrow\mathfrak{z}}\left|\frac{\phi\left(\mathfrak{y}\right)-\phi\left(\mathfrak{z}\right)}{\mathfrak{y}-\mathfrak{z}}\right|_{p}=\lim_{\mathfrak{y}\rightarrow\mathfrak{z}}\left|\mathfrak{y}-\mathfrak{z}\right|_{p}=0,\textrm{ }\forall\mathfrak{z}\in\mathbb{Z}_{p} \end{equation} which is \emph{not} what one would like to see for an injective function! The notion of \textbf{strict differentiability }(see \cite{Robert's Book,Ultrametric Calculus}) is needed to get a non-degenerate notion of differentiability for non-analytic functions. In short, this is the requirement that we take the limit of (\ref{eq:Naive differentiability}) as\emph{ both }$\mathfrak{y}$ and $\mathfrak{z}$ to converge to a single point $\mathfrak{z}_{0}$, and that, regardless of path chosen, the limit exists; this then shows that $f$ is ``strictly differentiable'' at $\mathfrak{z}_{0}$. \end{example} \vphantom{} To continue our tour of the doom and the gloom, the $p$-adic Mean Value Theorem is but a shadow of its real-analytic self; it might as well not exist at all. Unsurprisingly, $p$-adic analysis suffers when it comes to integration. Without a dependable Mean Value Theorem, the relationship between integration and anti-differentiation familiar to us from real and complex analysis fails to hold in the $\left(p,p\right)$-adic context. But the story only gets worse from there. Surely, one of the most beautiful syzygies in modern analysis is the intermingling of measure theory and functional analysis. This is made all the more elegant when Haar measures and harmonic analysis are brought into the mix. Unfortunately, in $\left(p,p\right)$-adic waters, this elegance bubbles, burns, and crumbles into nothingness. Basic considerations of algebra and arithmetic show that any $\left(p,p\right)$-adic ``Haar measure'' (translation-invariant $\mathbb{F}$-valued function of ``nice'' sets in $\mathbb{Z}_{p}$) must assign the rational number $1/p^{n}$ as the measure of sets of the form $k+p^{n}\mathbb{Z}_{p}$, where $k\in\mathbb{Z}$ and $n\in\mathbb{N}_{0}$. Because the $p$-adic absolute value of $1/p^{n}$ tends to $\infty$ as $n\rightarrow\infty$, the only \emph{continuous}, translation-invariant $\mathbb{Q}_{p}$-valued function of sets on $\mathbb{Z}_{p}$ is the constant function $0$\textemdash the zero measure. This same result holds for any ring extension of $\mathbb{Z}_{p}$, as well as for $\mathbb{Q}_{p}$ and any field extension thereof. Fortunately, there ways of getting around this sorry state of affairs, even if the result ends up being completely different than classical integration theory. The first of these is the \textbf{Volkenborn integral}, which\index{integral!Volkenborn} generalizes the classical notion of an integral as the limit of a Riemann sum\index{Riemann sum}. The Volkenborn integral\footnote{Robert covers Volkenborn integration in Section 5 of his chapter on Differentiation in \cite{Robert's Book}, starting at page 263. One can also turn to Borm's dissertation \cite{pp adic Fourier theory}.} of a function $f:\mathbb{Z}_{p}\rightarrow\mathbb{C}_{q}$ is defined by the limit: \begin{equation} \int_{\mathbb{Z}_{p}}f\left(\mathfrak{z}\right)d_{\textrm{Volk}}\mathfrak{z}\overset{\textrm{def}}{=}\lim_{N\rightarrow\infty}\frac{1}{p^{N}}\sum_{n=0}^{p^{N}-1}f\left(n\right)\label{eq:definition of the volkenborn integral} \end{equation} which\textemdash the reader should note\textemdash is \emph{not }translation invariant (see \cite{Robert's Book})! The Volkenborn integral turns out to be intimately connected with the finite differences and the indefinite summation operator. In all likelihood, the most noteworthy property of the Volkenborn integral is that, for any prime $p$ and any integer $n\geq0$, the integral of the function $\mathfrak{z}^{n}$ is equal to the $n$th Bernoulli number (with $B_{1}=-1/2$). In practice, however, the Volkenborn integral is neglected in favor for the Amice-Iwasawa\index{Amice, Yvette} approach, otherwise known as the theory of \index{$p$-adic!distribution}\textbf{$p$-adic distributions}\footnote{Note, these should \textbf{\emph{not}}\emph{ }be confused with the distributions / ``generalized functions'' dealt with by Vladimirov in \cite{Vladimirov - the big paper about complex-valued distributions over the p-adics}. Amice-Iwasawa ``$p$-adic distributions'' are linear functionals taking values in $\mathbb{Q}_{p}$ or a finite degree field extension thereof. Vladimirov's ``$p$-adic distributions'' are linear functionals taking values in $\mathbb{C}$.} (\cite{p-adic L-functions paper,Iwasawa}). Let $\mathbb{F}$ be a metrically complete field extension of $\mathbb{Q}_{p}$. The idea is to exploit the fact that the space $C\left(\mathbb{Z}_{p},\mathbb{F}\right)$ has as a basis the binomial coefficient polynomials ($\left\{ \binom{\mathfrak{z}}{n}\right\} _{n\geq0}$): \begin{align*} \binom{\mathfrak{z}}{0} & =1\\ \binom{\mathfrak{z}}{1} & =\mathfrak{z}\\ \binom{\mathfrak{z}}{2} & =\frac{\mathfrak{z}\left(\mathfrak{z}-1\right)}{2}\\ & \vdots \end{align*} That these functions form a basis of $C\left(\mathbb{Z}_{p},\mathbb{F}\right)$ is a classic result of $p$-adic analysis first proved by Kurt Mahler in 1958\footnote{\cite{Mahler Series} is Mahler's original paper, but his result is now standard material in any worthwhile course on $p$-adc analysis \cite{Mahler,Robert's Book,Ultrametric Calculus}.}. This basis is now known as the\textbf{ Mahler basis}, in his honor. From this, one can define $\mathbb{F}$-valued measures and distributions by specifying what they do to $\binom{\mathfrak{z}}{n}$ for each $n$, and then extending by linearity. This construction is of considerable importance in modern algebraic number theory, where it is one of several ways of defining $p$-adic $L$ functions\index{$p$-adic!$L$-function}; the fact that the several different methods of constructing $p$-adic $L$ functions are all equivalent was a major landmark of twentieth century number theory \cite{p-adic L-functions paper,Iwasawa}. So, it's no wonder the Volkenborn integral has gotten the proverbial cold shoulder. Despite all this, the particular sub-discipline this dissertation focuses on remains essentially untouched.\textbf{ }In terms of the four flavor classification given above, \textbf{$\left(p,q\right)$-adic analysis}\index{analysis!left(p,qright)-adic@$\left(p,q\right)$-adic}\textemdash the study of functions from $\mathbb{Z}_{p}$ (or a metrically complete field extension thereof) to $\mathbb{Z}_{q}$ (or a metrically complete field extension thereof), where $p$ and $q$ are distinct primes, is technically a form of non-archimedean analysis. Even so, this dissertation stands in stark contrast to much of the established literature in non-archimedean analysis not only by its unusual content, but also by virtue of its concreteness. One generally needs to turn to works of mathematical physics like \cite{Quantum Paradoxes} to get comparable levels of computational depth. Also significant, unlike several works going by the title ``non-archimedean analysis'' (or some variation thereof), my dissertation has the distinction of actually \emph{being }a work of mathematical analysis, as opposed to algebra going by a false name\footnote{A particularly egregious example of this is \cite{Bosch lying title}, a text on $p$-adic algebraic geometry and Tate's ``rigid analytic geometry'' which has the \emph{nerve} to call itself \emph{Non-archimedean analysis}, despite not containing so much as a single line of bonafide analysis. Other examples of such wolves-in-sheep's-clothing are \cite{Schneider awful book,More Schneider lies}, both due to Peter Schneider.}. And, by mathematical analysis, I mean \emph{hard} analysis: $\epsilon$s and $\delta$s, approximations, asymptotics, and the like, and all the detailed computations entailed. The past twenty or so years have seen many new strides in $p$-adic and non-archimedean analysis, particularly in the vein of generalizing as many of classical analysis' most useful tools, principally to the setting of $\left(p,\infty\right)$-adic analysis, but also to $\left(p,p\right)$ as well. The tools in question include, asymptotic analysis, Tauberian theorems, distributions, oscillatory integrals (a $p$-adic van der Corput lemma), , and many others; see, \cite{p-adic Tauberian}, \cite{Vladimirov - the big paper about complex-valued distributions over the p-adics,Volo - p-adic Wiener}, \cite{p-adic van der Corput lemma,Real and p-Adic Oscillatory integrals}). However, because $\left(p,q\right)$-adic analysis works with functions taking values in non-archimedean fields, the vast majority of these tools and their associated proofs do not extend to the $\left(p,q\right)$-adic case\textemdash at least, not yet. And it is precisely there where we will begin our work. \subsection{\label{subsec:3.1.2 Banach-Spaces-over}Banach Spaces over a Non-Archimedean Field} THROUGHOUT THIS SUBSECTION, $K$ IS A COMPLETE NON-ARCHIMEDEAN VALUED FIELD. \vphantom{} In classical analysis, Banach spaces frequently trot onto the stage when one moves from asking ``biographical'' questions about specific functions, specific equations, and the like, to ``sociological'' ones\textemdash those regarding whole classes of functions. Much like with real or complex analysis, there is quite a lot one can do in $\left(p,p\right)$-adic analysis without needing to appeal to the general theories of Banach spaces and functional analysis. Power series and Mahler's basis for continuous $\left(p,p\right)$-adic functions give $p$-adic analysis enough machinery to make concreteness just as viable as in the real or complex case. In $\left(p,q\right)$-adic analysis, however, the lack of any notions of differentiation, power series, and the non-applicability of the Mahler Basis make for an entirely different set of circumstances. Rather than drawing from the broader perspectives offered by functional analysis as a means of enriching how we approach what we already know, Banach spaces are foundational to non-archimedean analysis precisely because they can be used to determine what is or is not actually \emph{possible} in the subject. As a simple example, consider Hilbert spaces and orthogonal functions. Investigations into Fourier series and differential equations in the nineteenth century motivated subsequent explorations of function spaces as a whole, especially in the abstract. The Cauchy-Schwarz Inequality came before inner product spaces. In non-archimedean analysis, however, it can be shown that the only inner product spaces over non-archimedean fields which are Hilbert spaces in the classical sense\footnote{Specifically, their norm is induced by a bilinear form, and orthogonal projections exist for every closed subspace.} are necessarily finite-dimensional (\textbf{Theorem 4 }from \cite{Schikhof Banach Space Paper}). Sobering information such as this gives non-archimedean analysis an idea of what they should or should not set their hopes on. Most of the exposition given here is taken from Schikhof's excellent article on Banach spaces over a non-archimedean field \cite{Schikhof Banach Space Paper}, which\textemdash unlike van Rooij's book \cite{van Rooij - Non-Archmedean Functional Analysis}\textemdash is still in print. We begin with the basic definitions. \begin{defn}[S\textbf{emi-norms, normed vector spaces, etc.} \cite{Ultrametric Calculus}] \label{def:seminorms etc.}Let $E$ be a vector space over a non-archimedean valued field $K$ (a.k.a., $E$ is a \textbf{$K$-vector space} or \textbf{$K$-linear space}). A\index{non-archimedean!semi-norm} \textbf{(non-archimedean) semi-norm }on $E$ is a function $\rho:E\rightarrow\mathbb{R}$ such that for all $x,y\in E$ and all $\mathfrak{a}\in K$: \vphantom{} I. $\rho\left(x\right)\geq0$; \vphantom{} II. $\rho\left(\mathfrak{a}x\right)=\left|\mathfrak{a}\right|_{K}\rho\left(x\right)$; \vphantom{} III. $\rho\left(x+y\right)\leq\max\left\{ \rho\left(x\right),\rho\left(y\right)\right\} $. \vphantom{} Additionally, $\rho$ is said to be a \textbf{(non-archimedean) norm }over\index{non-archimedean!norm} $E$ whenever it satisfies the additional condition: \vphantom{} IV. $\rho\left(x\right)=0$ if and only if $x=0$. \vphantom{} Next, given a non-archimedean norm $\left\Vert \cdot\right\Vert $, the pair $\left(E,\left\Vert \cdot\right\Vert \right)$ is called a \textbf{(non-archimedean) normed vector space}; this is then an ultrametric space with respect to the metric induced by the norm. We say $\left(E,\left\Vert \cdot\right\Vert \right)$ is a \textbf{(non-archimedean) Banach space }when\index{non-archimedean!Banach space}\index{Banach algebra!non-archimedean} it is complete as a metric space. Finally, a \textbf{(non-archimedean) Banach algebra }is\index{non-archimedean!Banach algebra} a Banach space $\left(E,\left\Vert \cdot\right\Vert \right)$ with a multiplication operation $m:E\times E\rightarrow E$ so that $\left\Vert m\left(x,y\right)\right\Vert \leq\left\Vert x\right\Vert \left\Vert y\right\Vert $ for all $x,y\in E$. Also, a $K$-Banach space (resp. algebra) is a Banach space (resp. algebra) over the field $K$. \end{defn} \begin{prop} \label{prop:series characterization of a Banach space}Let $E$ be a non-archimedean normed vector space. Then, $E$ is complete if and only if, for every sequence $\left\{ x_{n}\right\} _{n\geq0}$ in $E$ for which $\lim_{n\rightarrow\infty}\left\Vert x_{n}\right\Vert \overset{\mathbb{R}}{=}0$, the series $\sum_{n=0}^{\infty}x_{n}$ converges to a limit in $E$. \end{prop} Proof: I. Suppose $E$ is complete. Then, $\left(E,\left\Vert \cdot\right\Vert \right)$ is a complete ultrametric space, and as such, an infinite series $\sum_{n=0}^{\infty}x_{n}$ in $E$ converges if and only if $\lim_{n\rightarrow\infty}\left\Vert x_{n}\right\Vert \overset{\mathbb{R}}{=}0$. \vphantom{} II. Conversely, suppose an infinite series $\sum_{n=0}^{\infty}x_{n}$ in $E$ converges if and only if $\lim_{n\rightarrow\infty}\left\Vert x_{n}\right\Vert \overset{\mathbb{R}}{=}0$. Then, letting $\left\{ x_{n}\right\} _{n\geq0}$ be a Cauchy sequence in $E$, we can choose a sequence $n_{1}<n_{2}<\cdots$ so that, for any $j$, $\left\Vert x_{m}-x_{n}\right\Vert <2^{-j}$ holds for all $m,n\geq n_{j}$. Setting $y_{1}=x_{n_{1}}$ and $y_{j}=x_{n_{j}}-x_{n_{j-1}}$ gives: \begin{equation} \lim_{J\rightarrow\infty}x_{n_{J}}\overset{E}{=}\lim_{J\rightarrow\infty}\sum_{j=1}^{J}y_{j} \end{equation} By construction, the Cauchyness of the $x_{n}$s guarantees that $\left\Vert y_{j}\right\Vert \rightarrow0$ as $j\rightarrow\infty$. As such, our assumption ``every infinite series in $E$ converges if and only if its terms tend to zero in $E$-norm'' forces $\lim_{J\rightarrow\infty}\sum_{j=1}^{J}y_{j}$ (and hence, $\lim_{J\rightarrow\infty}x_{n_{J}}$) to converge to a limit in $E$. Since the $x_{n}$s are Cauchy, the existence of a convergent subsequence then forces the entire sequence to converge in $E$, with: \begin{equation} \lim_{n\rightarrow\infty}x_{n}\overset{E}{=}\sum_{j=1}^{\infty}y_{j} \end{equation} Because the $x_{n}$s were arbitrary, this shows that $E$ is complete. Q.E.D. \vphantom{} Next, we give the fundamental examples of non-archimedean Banach spaces. Throughout, we will suppose that $K$ is a complete non-archimedean valued field. We address the finite-dimensional cases first. \begin{defn} Given a vector space $V$ over a non-archimedean field $K$, two non-archimedean norms $\left\Vert \cdot\right\Vert $ and $\left\Vert \cdot\right\Vert ^{\prime}$ defined on $V$ are said to be \textbf{equivalent }whenever there are real constants $0<c\leq C<\infty$ so that: \begin{equation} c\left\Vert \mathbf{x}\right\Vert \leq\left\Vert \mathbf{x}\right\Vert ^{\prime}\leq C\left\Vert \mathbf{x}\right\Vert \end{equation} \end{defn} \cite{Robert's Book} gives the following basic results: \begin{thm} Let $V$ and $W$ be normed vector spaces over $\mathbb{Q}_{p}$. \vphantom{} I. If $V$ is finite-dimensional, then all non-archimedean norms on $V$ are equivalent. \vphantom{} II. If $V$ is locally compact, then $V$ is finite dimensional. \vphantom{} III. If $V$ is locally compact, then $V$ has the Heine-Borel property\textemdash sets in $V$ are compact if and only if they are closed and bounded. \vphantom{} IV. If $V$ and $W$ are finite-dimensional, then every linear map $L:V\rightarrow W$ is continuous. \end{thm} \begin{defn} \ I. In light of the above, we shall use only a single norm on finite-dimensional vector spaces over $\mathbb{Q}_{p}$: the $\ell^{\infty}$-norm (written $\left\Vert \cdot\right\Vert _{p}$), which outputs the maximum of the $p$-adic absolute values of the entries of an element of the vector space. \vphantom{} II. Given integers $\rho,c\geq1$, we write \nomenclature{$K^{\rho,c}$}{$\rho\times c$ matrices with entries in $K$ \nopageref}$K^{\rho,c}$ to denote the set of all $\rho\times c$ matrices with entries in $K$. We make this a non-archimedean normed vector space by equipping it with the $\infty$-norm, which, for any $\mathbf{A}\in K^{\rho,c}$ with entries $\left\{ a_{i,j}\right\} _{1\leq i\leq\rho,1,j\leq c}$, is given by: \nomenclature{$\left\Vert \mathbf{A}\right\Vert _{K}$}{max. of the $K$-adic absolute values of a matrix's entries \nopageref}\nomenclature{$\left\Vert \mathbf{a}\right\Vert _{K}$}{max. of the $K$-adic absolute values of a vector's entries \nopageref} \begin{equation} \left\Vert \mathbf{A}\right\Vert _{K}\overset{\textrm{def}}{=}\max_{1\leq i\leq\rho,1\leq j\leq c}\left|a_{i,j}\right|_{K}\label{eq:Definition of the non-archimedean matrix norm} \end{equation} When $c=1$, we identify $K^{\rho,1}$ with the set of all $\rho\times1$ column vectors. \vphantom{} III. Given $d\geq2$, we write $\textrm{GL}_{d}\left(K\right)$\nomenclature{$\textrm{GL}_{d}\left(K\right)$}{set of invertible $d\times d$ matrices with entries in $K$ \nopageref} to denote the set of all invertible $d\times d$ matrices with entries in $K$, made into a non-abelian group by matrix multiplication. \vphantom{} IV. Because we will need it for parts of Chapter 5, we define \nomenclature{$\left\Vert \mathbf{x}\right\Vert _{\infty}$}{$\overset{\textrm{def}}{=}\max\left\{ \left|x_{1}\right|,\ldots,\left|x_{d}\right|\right\}$}$\left\Vert \cdot\right\Vert _{\infty}$ on $\mathbb{C}^{d}$ (or any subset thereof) by: \begin{equation} \left\Vert \mathbf{x}\right\Vert _{\infty}\overset{\textrm{def}}{=}\max\left\{ \left|x_{1}\right|,\ldots,\left|x_{d}\right|\right\} \label{eq:Definition of infinity norm} \end{equation} where $\mathbf{x}=\left(x_{1},\ldots,x_{d}\right)$. Here, the absolute values are those on $\mathbb{C}$. \end{defn} \begin{prop} $K^{\rho,c}$ is a non-archimedean Banach space over $K$. If $\rho=c$, it is also a Banach algebra, with the multiplication operation being matrix multiplication. \end{prop} We will not mention matrix spaces until we get to Chapters 5 and 6, where we they will be ubiquitous. Next, we introduce the fundamental examples of \emph{infinite}-dimensional non-archimedean Banach spaces. \begin{defn} Let $X$ be a set and let $K$ be a metrically-complete non-archimedean valued field. \vphantom{} I. A function $f:X\rightarrow K$ is said to be \textbf{bounded }if: \begin{equation} \sup_{x\in X}\left|f\left(x\right)\right|_{K}<\infty \end{equation} We get a norm \nomenclature{$\left\Vert f\right\Vert _{X,K}$}{$\sup_{x\in X}\left|f\left(x\right)\right|_{K}$} out of this by defining: \begin{equation} \left\Vert f\right\Vert _{X,K}\overset{\textrm{def}}{=}\sup_{x\in X}\left|f\left(x\right)\right|_{K}\label{eq:Definition of X,K norm} \end{equation} We call this the $K$\textbf{-supremum norm }\index{norm!$K$ supremum} on $X$. We write \nomenclature{$B\left(X,K\right)$}{Bounded $K$-valued functions on $X$}$B\left(X,K\right)$ to denote the set of all bounded $K$-valued functions on $X$. This is a non-archimedean Banach space under $\left\Vert \cdot\right\Vert _{X,K}$. When $X=\mathbb{Z}_{p}$, we call the norm $\left\Vert \cdot\right\Vert _{X,K}$ the \textbf{$\left(p,K\right)$-adic norm}\index{norm!left(p,Kright)-adic@$\left(p,K\right)$-adic}, and denote it by $\left\Vert \cdot\right\Vert _{p,K}$\nomenclature{$\left\Vert \cdot\right\Vert _{p,K}$}{ }. When $K$ is a $q$-adic field, we call this the \index{norm!left(p,qright)-adic@$\left(p,q\right)$-adic}\index{$p,q$-adic!norm}\textbf{$\left(p,q\right)$-adic norm}, and denote it by \nomenclature{$\left\Vert \cdot\right\Vert _{p,q}$}{ }$\left\Vert \cdot\right\Vert _{p,q}$. In a minor abuse of notation, we also use $\left\Vert \cdot\right\Vert _{p,K}$ and $\left\Vert \cdot\right\Vert _{p,q}$ to denote the $K$ supremum norm on $X$ when $X$ is $\hat{\mathbb{Z}}_{p}$. \vphantom{} II. If $X$ is a compact Hausdorff space, we write \nomenclature{$C\left(X,K\right)$}{set of continuous $K$-valued functions on $X$}$C\left(X,K\right)$ to denote the set of all continuous functions $f:X\rightarrow K$. This is a non-archimedean Banach space\footnote{This construction is of vital importance to us, because it allows us to consider continuous $K$-valued functions defined on a compact ultrametric space $X$ like $\mathbb{Z}_{p}$, even if $K$'s residue field has characteristic $q\neq p$.} under $\left\Vert \cdot\right\Vert _{X,K}$. \vphantom{} III. We write $\text{\ensuremath{\ell^{\infty}\left(K\right)}}$ (or $\ell^{\infty}$, if $K$ is not in question) to denote the set of all bounded sequences in $K$. This is a non-archimedean Banach space under the norm: \begin{equation} \left\Vert \left(a_{1},a_{2},\ldots\right)\right\Vert _{K}\overset{\textrm{def}}{=}\sup_{n\geq1}\left|a_{n}\right|_{K}\label{eq:K sequence norm} \end{equation} \vphantom{} IV. We write \nomenclature{$c_{0}\left(K\right)$}{set of $K$-valued sequences converging to $0$}$c_{0}\left(K\right)$ (or $c_{0}$, if $K$ is not in question) to denote the set of all sequences in $K$ which converge to $0$ in $K$'s absolute value. This is a Banach space under (\ref{eq:K sequence norm}) and is a closed subspace of $\ell^{\infty}$. \vphantom{} V. We write\footnote{I've adapted this notation from what van Rooij calls ``$c_{0}\left(X\right)$''. \cite{van Rooij - Non-Archmedean Functional Analysis}} $c_{0}\left(X,K\right)$ to denote the set of all $f\in B\left(X,K\right)$ so that, for every $\epsilon>0$, there are only finitely many $x\in X$ for which $\left|f\left(x\right)\right|_{K}\geq\epsilon$. This is a Banach space with respect to $\left\Vert \cdot\right\Vert _{X,K}$, being a closed subspace of $B\left(X,K\right)$. \vphantom{} VI. \nomenclature{$\text{\ensuremath{\ell^{1}\left(K\right)}}$}{set of absolutely summable $K$-valued sequences}We write $\text{\ensuremath{\ell^{1}\left(K\right)}}$ (or $\ell^{1}$, if $K$ is not in question) to denote the set of all sequences $\mathbf{c}:\mathbb{N}_{0}\rightarrow K$ so that: \[ \sum_{n=0}^{\infty}\left|\mathbf{c}\left(n\right)\right|_{K}<\infty \] We write: \begin{equation} \left\Vert \mathbf{c}\right\Vert _{1}\overset{\textrm{def}}{=}\sum_{n=0}^{\infty}\left|\mathbf{c}\left(n\right)\right|_{K}\label{eq:Definition of ell-1 norm} \end{equation} $\ell^{1}$ is an \emph{archimedean }$K$-Banach space under $\left\Vert \cdot\right\Vert _{1}$\textemdash that is, $\left\Vert \mathbf{a}+\mathbf{b}\right\Vert _{1}\leq\left\Vert \mathbf{a}\right\Vert _{1}+\left\Vert \mathbf{b}\right\Vert _{1}$, however, the strong triangle inequality does not hold on it; that is, there exist $\mathbf{a},\mathbf{b}$ for which $\left\Vert \mathbf{a}+\mathbf{b}\right\Vert _{1}>\max\left\{ \left\Vert \mathbf{a}\right\Vert _{1},\left\Vert \mathbf{b}\right\Vert _{1}\right\} $. This also goes to show that not every Banach space over a non-archimedean field is, itself, non-archimedean. \end{defn} \begin{rem} As a rule, when writing expressions involving $\left\Vert \cdot\right\Vert $, I will use a single subscript (ex: $\left\Vert \cdot\right\Vert _{K}$) whenever only the absolute value of $K$ is being used. Expressions with \emph{two} subscripts (ex: $\left\Vert \cdot\right\Vert _{p,K}$, $\left\Vert \cdot\right\Vert _{p,q}$), meanwhile, indicate that we are using the absolute value indicated by the subscript on the right as well as taking a supremum of an input variable with respect to the absolute value indicated by the subscript on the left. \end{rem} \vphantom{} Although $\ell^{1}$ and its continuous counterpart $L^{1}$ are of fundamental importance in real and complex analysis, their non-archimedean analogues are not nearly as distinguished. The first hint that something is amiss comes from the fact that $\ell^{1}$ is archimedean, rather than non-archimedean.\emph{ Traditionally}\footnote{Schikhof himself says as much at the top of page 6 in \cite{Schikhof Banach Space Paper}, writing ``Absolute summability plays no a role in {[}non-archimedean{]} analysis.''}, $\ell^{1}$ is of no interest in non-archimedean analysts. Unsurprisingly, it turns out that $\ell^{1}$\textemdash specifically, its continuous analogue, $L^{1}$) is pertinent to the study of $\chi_{H}$. This is the subject of Subsection \ref{subsec:3.3.6 L^1 Convergence}. Next, we have linear operators and functionals. \begin{defn} Let $E$ and $F$ be non-archimedean Banach spaces, both over the field $K$. Then one writes $\mathcal{L}\left(E,F\right)$ to denote the set of all continuous linear operators from $E$ to $F$. We equip $\mathcal{L}\left(E,F\right)$ with the \textbf{operator norm}: \begin{equation} \left\Vert T\right\Vert \overset{\textrm{def}}{=}\sup_{x\in E}\frac{\left\Vert T\left(x\right)\right\Vert _{F}}{\left\Vert x\right\Vert _{E}}\label{eq:Definition of operator norm} \end{equation} \end{defn} \begin{prop} $\mathcal{L}\left(E,F\right)$ is a non-archimedean Banach space over $K$ with respect to the operator norm. \end{prop} \begin{prop} A linear operator $T:E\rightarrow F$ between two non-archimedean Banach spaces $E$ and $F$ over the field $K$ is continuous if and only if it has finite operator norm. Consequently, \textbf{bounded linear operators }and \textbf{continuous linear operators }are one and the same, just like in the classical case. \end{prop} \begin{defn} Given a non-archimedean $K$-Banach space $E$, the \textbf{dual}\index{Banach space!dual of} of $E$ (denoted $E^{\prime}$) is, as usual, the $K$-Banach space of all continuous linear functionals from $E$ to $K$. Given an $f\in E^{\prime}$, we write $\left\Vert f\right\Vert _{E^{\prime}}$ to denote the norm of $f$; this is defined by the usual variant of (\ref{eq:Definition of operator norm}): \begin{equation} \left\Vert f\right\Vert _{E^{\prime}}\overset{\textrm{def}}{=}\sup_{x\in E}\frac{\left|f\left(x\right)\right|_{K}}{\left\Vert x\right\Vert _{E}}\label{eq:Definition of the norm of a linear functional} \end{equation} \end{defn} \begin{rem} For the case of the space $C\left(\mathbb{Z}_{p},K\right)$, where $K$ is a $q$-adic field, where $p$ and $q$ are distinct primes, note that its dual $C\left(\mathbb{Z}_{p},K\right)^{\prime}$ \nomenclature{$C\left(\mathbb{Z}_{p},K\right)^{\prime}$}{set of continuous $K$-valued linear functionals on $C\left(\mathbb{Z}_{p},K\right)$} will consist of all continuous $K$-valued linear functionals on $C\left(\mathbb{Z}_{p},K\right)$. \end{rem} \begin{fact} \emph{Importantly, because of their proofs' dependence on the }\textbf{\emph{Baire Category Theorem}}\emph{,\index{Uniform Boundedness Principle} the }\textbf{\emph{Uniform Boundedness Principle}}\emph{, the }\textbf{\emph{Closed Graph Theorem}}\emph{, the }\textbf{\emph{Open Mapping Theorem}}\emph{, and the }\textbf{\emph{Banach-Steinhaus Theorem}}\emph{ }all apply to non-archimedean Banach spaces\emph{ \cite{Schikhof Banach Space Paper}. The Hahn-Banach Theorem, however, is more subtle, as is discussed below.} \end{fact} \vphantom{} As we now move to discuss notions of duality, the notion of \index{spherically complete}\textbf{spherical completeness}\footnote{Recall, a valued field $K$ (or, more generally, any metric space) is said to be \textbf{spherically complete} whenever any sequence of nested non-empty balls $B_{1}\supseteq B_{2}\supseteq\cdots$ in $K$ has a non-empty intersection. $\mathbb{Q}_{p}$ and any finite extension thereof are spherically complete; $\mathbb{C}_{p}$, however, is not.} will raise its head in surprising ways. The first instance of this is in the non-archimedean Hahn-Banach Theorem. First, however, another definition, to deal with the fact that non-archimedean fields need not be separable as topological spaces. \begin{defn} A $K$-Banach space $E$ is said\index{Banach space!of countable type} to be \textbf{of countable type }if there is a countable set whose $K$-span is dense in $E$. \end{defn} \begin{thm}[\textbf{Non-Archimedean Hahn-Banach Theorems}\footnote{Theorems 9 and 10 in \cite{Schikhof Banach Space Paper}.}\index{Hahn-Banach Theorem}] Let $E$ be a Banach space over a complete non-archimedean valued field $K$, let $D$ be a subspace of $E$. Let $D^{\prime}$ be the continuous dual of $D$ (continuous linear functionals $D\rightarrow K$) and let $f\in D^{\prime}$. \vphantom{} I. If $K$ is \emph{spherically complete} (for instance, if $K$ is $\mathbb{Q}_{q}$ or a finite, metrically-complete extension thereof), there then exists a continuous linear functional $\overline{f}\in E^{\prime}$ whose restriction to $D$ is equal to $f$, and so that $\left\Vert \overline{f}\right\Vert _{E^{\prime}}=\left\Vert f\right\Vert _{D^{\prime}}$. \vphantom{} II. If $E$ is of countable type, then\textemdash even if $K$ is \textbf{\emph{not }}spherically complete\textemdash for any $\epsilon>0$, there exists a continuous linear functional $\overline{f}:E\rightarrow K$ whose restriction to $D$ is equal to $f$. Moreover, $\left\Vert \overline{f}\right\Vert _{E^{\prime}}\leq\left(1+\epsilon\right)\left\Vert f\right\Vert _{D^{\prime}}$. \end{thm} \begin{rem} The requirement that $E$ be of countable type \emph{cannot} be dropped, nor can $\epsilon$ ever be set to $0$. There exists a two-dimensional non-archimedean Banach space $E$ over a non-spherically-complete $K$ and a subspace $D\subseteq E$ and an $f\in D^{\prime}$ such that no extension $\overline{f}\in E^{\prime}$ exists satisfying $\left\Vert \overline{f}\right\Vert _{E^{\prime}}\leq\left\Vert f\right\Vert _{D^{\prime}}$ \cite{Schikhof Banach Space Paper}. This counterexample can be found on page 68 of van Rooij's book \cite{van Rooij - Non-Archmedean Functional Analysis}. \end{rem} \vphantom{} Spherical completeness is responsible for non-archimedean Banach spaces' most significant deviations from the classical theory, which we now chronicle. The names I give these theorems were taken from \cite{van Rooij - Non-Archmedean Functional Analysis}. \begin{thm}[\textbf{Fleischer's Theorem}\footnote{See \cite{Schikhof Banach Space Paper,van Rooij - Non-Archmedean Functional Analysis}. This theorem is given as \textbf{Theorem}}] Let $K$ be \textbf{spherically complete}. Then, a $K$-Banach space is reflexive\index{Banach space!reflexive} if and only if it is finite-dimensional. \end{thm} \begin{thm}[\textbf{van der Put's Theorem}\footnote{See \cite{Schikhof Banach Space Paper,van Rooij - Non-Archmedean Functional Analysis}. This theorem is a specific part (iii) of \textbf{Theorem 4.21 }on page 118 of \cite{van Rooij - Non-Archmedean Functional Analysis}.}] \label{thm:c_o and ell_infinit are each others duals in a spherically incomplete NA field}Let $K$ be\textbf{\emph{ spherically incomplete}}. Then, for any countable\footnote{To give the reader a taste of the level of generality of van Rooij's text, while I state the theorem for countable $X$, the formulation given in \cite{van Rooij - Non-Archmedean Functional Analysis} states that it holds for any ``small'' set $X$, a definition that involves working with cardinal numbers and arbitrary $\sigma$-additive measures on Boolean rings. On page 31 of the same, van Rooij says that ``obviously'', a set with cardinality $\aleph_{0}$ (that is, a countable set) is ``small''. I choose to believe him, from whence I obtained the version of the theorem given here.} set $X$, the $K$-Banach spaces $c_{0}\left(X,K\right)$ and $B\left(X,K\right)$ are reflexive\index{Banach space!reflexive}, being one another's duals in a natural way. \end{thm} \begin{cor} If $K$ is \textbf{\emph{spherically incomplete}}, then \textbf{every} $K$-Banach space of countable type is reflexive. \cite{van Rooij - Non-Archmedean Functional Analysis} \end{cor} \subsection{\label{subsec:3.1.3 The-van-der}The van der Put Basis} IN THIS SUBSECTION, $p$ IS A PRIME. $K$ IS A METRICALLY COMPLETE VALUED FIELD (NOT NECESSARILY NON-ARCHIMEDEAN) OF CHARACTERISTIC ZERO. \vphantom{} A principal reason for the ``simplicity'' of $\left(p,q\right)$-adic analysis in comparison with more general theories of non-archimedean analysis is that, much like in the $\left(p,p\right)$-adic case, the Banach space of continuous functions $\left(p,q\right)$-adic functions admits a countable basis. This is \textbf{the van der Put basis}. This basis actually works for the space of continuous functions $f:\mathbb{Z}_{p}\rightarrow K$ where $K$ is \emph{any }metrically complete non-archimedean field, unlike the Mahler basis, which only works when $K$ is $\mathbb{Q}_{p}$ or an extension thereof. Because of the van der Put basis, we will be able to explicitly compute integrals and Fourier transforms, which will be of the utmost importance for our analyses of $\chi_{H}$. \begin{defn}[\textbf{The van der Put Basis} \cite{Robert's Book,Ultrametric Calculus}] \label{def:vdP basis, n_minus, vpD coefficients}\ \vphantom{} I. We call the indicator functions $\left[\mathfrak{z}\overset{p^{\lambda_{p}\left(j\right)}}{\equiv}j\right]$ the \index{van der Put!basis}\textbf{van der Put (basis) functions }and refer to the set $\left\{ \left[\mathfrak{z}\overset{p^{\lambda_{p}\left(j\right)}}{\equiv}j\right]\right\} _{j\in\mathbb{N}_{0}}$ as the \textbf{van der Put basis}. \vphantom{} II. Given an integer $n\geq1$, we can express $n$ $p$-adically as: \begin{equation} n=\sum_{k=0}^{\lambda_{p}\left(n\right)-1}n_{k}p^{k} \end{equation} where the $n_{k}$s are the $p$-adic digits of $n$. Following Robert and Schikhof (\cite{Robert's Book,Ultrametric Calculus}), we write \nomenclature{$n_{-}$}{ }$n_{-}$ to denote the integer obtained by deleting the $k=\lambda_{p}\left(n\right)-1$ term from the above series; equivalently, $n_{-}$ is the integer obtained by deleting the right-most non-zero digit in $n$'s $p$-adic representation of $n$. That is: \begin{equation} n_{-}=n-n_{\lambda_{p}\left(n\right)-1}p^{\lambda_{p}\left(n\right)-1}\label{eq:Definition of n minus} \end{equation} We can also write this as the value of $n$ modulo $p^{\lambda_{p}\left(n\right)-1}$: \begin{equation} n_{-}=\left[n\right]_{p^{\lambda_{p}\left(n\right)-1}} \end{equation} \vphantom{} III. Let $f:\mathbb{Z}_{p}\rightarrow K$ be any function. Then, for all $n\in\mathbb{N}_{0}$, we define the constants $\left\{ c_{n}\left(f\right)\right\} _{n\geq0}\in K$ by: \begin{equation} c_{n}\left(f\right)\overset{\textrm{def}}{=}\begin{cases} f\left(0\right) & \textrm{if }n=0\\ f\left(n\right)-f\left(n_{-}\right) & \textrm{if }n\geq1 \end{cases},\textrm{ }\forall n\in\mathbb{N}_{0}\label{eq:Def of c_n of f} \end{equation} We call \nomenclature{$c_{n}\left(f\right)$}{$n$th van der Put coefficient of $f$}$c_{n}\left(f\right)$ the $n$th \index{van der Put!coefficients}\textbf{van der Put coefficient }of $f$. \vphantom{} IV. We write \nomenclature{$\textrm{vdP}\left(\mathbb{Z}_{p},K\right)$}{the set of formal van der Put series with coefficients in $K$}$\textrm{vdP}\left(\mathbb{Z}_{p},K\right)$ to denote the $K$-vector space of all \textbf{formal van der Put series}\index{van der Put!series}. The elements of $\textrm{vdP}\left(\mathbb{Z}_{p},K\right)$ are formal sums: \begin{equation} \sum_{n=0}^{\infty}\mathfrak{a}_{n}\left[\mathfrak{z}\overset{p^{\lambda_{p}\left(n\right)}}{\equiv}n\right] \end{equation} where the $\mathfrak{a}_{n}$s are elements of $K$. The \textbf{domain of convergence }of a formal van der Put series is defined as the set of all $\mathfrak{z}\in\mathbb{Z}_{p}$ for which the series converges in $K$. We call $\textrm{vdP}\left(\mathbb{Z}_{p},\mathbb{C}_{q}\right)$ the space of \textbf{formal $\left(p,q\right)$-adic van der Put series}. \vphantom{} V. Let \nomenclature{$F\left(\mathbb{Z}_{p},K\right)$}{$K$-valued functions on $\mathbb{Z}_{p}$}$F\left(\mathbb{Z}_{p},K\right)$ denote the $K$-linear space of all $K$-valued functions on $\mathbb{Z}_{p}$. Then, we define the linear operator \nomenclature{$S_{p}$}{van-der-Put-series-creating operator}$S_{p}:F\left(\mathbb{Z}_{p},K\right)\rightarrow\textrm{vdP}\left(\mathbb{Z}_{p},K\right)$ by: \begin{equation} S_{p}\left\{ f\right\} \left(\mathfrak{z}\right)\overset{\textrm{def}}{=}\sum_{n=0}^{\infty}c_{n}\left(f\right)\left[\mathfrak{z}\overset{p^{\lambda_{p}\left(n\right)}}{\equiv}n\right],\textrm{ }\forall f\in F\left(\mathbb{Z}_{p},K\right)\label{eq:Definition of S_p of f} \end{equation} We also define partial sum operators \nomenclature{$S_{p:N}$}{$N$th partial van-der-Put-series-creating operator}$S_{p:N}:F\left(\mathbb{Z}_{p},K\right)\rightarrow C\left(\mathbb{Z}_{p},K\right)$ by: \begin{equation} S_{p:N}\left\{ f\right\} \left(\mathfrak{z}\right)\overset{\textrm{def}}{=}\sum_{n=0}^{p^{N}-1}c_{n}\left(f\right)\left[\mathfrak{z}\overset{p^{\lambda_{p}\left(n\right)}}{\equiv}n\right],\textrm{ }\forall f\in F\left(\mathbb{Z}_{p},K\right)\label{eq:Definition of S_p N of f} \end{equation} \end{defn} \begin{rem} Fixing $\mathfrak{z}\in\mathbb{N}_{0}$, observe that $\mathfrak{z}\overset{p^{\lambda_{p}\left(n\right)}}{\equiv}n$ can hold true for at most finitely many $n$. As such, every formal van der Put series necessarily converges in $K$ at every $\mathfrak{z}\in\mathbb{N}_{0}$. \end{rem} \begin{example}[\textbf{$\lambda$-Decomposition}] Given a sum of the form with coefficients in $\mathbb{C}_{q}$: \begin{equation} \sum_{n=0}^{\infty}\mathfrak{a}_{n}\left[\mathfrak{z}\overset{p^{\lambda_{p}\left(n\right)}}{\equiv}n\right] \end{equation} we can decompose it by splitting the sum into distinct pieces, with $\lambda_{p}\left(n\right)$ taking a single value over each individual piece. In particular, noting that for any $m\in\mathbb{N}_{1}$: \begin{equation} \lambda_{p}\left(n\right)=m\Leftrightarrow p^{m-1}\leq n\leq p^{m}-1 \end{equation} we can write: \begin{equation} \sum_{n=0}^{\infty}\mathfrak{a}_{n}\left[\mathfrak{z}\overset{p^{\lambda_{p}\left(n\right)}}{\equiv}n\right]\overset{\mathbb{C}_{q}}{=}\mathfrak{a}_{0}+\sum_{m=1}^{\infty}\sum_{n=p^{m-1}}^{p^{m}-1}\mathfrak{a}_{n}\left[\mathfrak{z}\overset{p^{m}}{\equiv}n\right]\label{eq:Lambda Decomposition} \end{equation} We will use this decomposition frequently enough for it to be worth having its own name: \emph{the Lambda decomposition}, or \emph{$\lambda$-decomposition\index{lambda-decomposition@$\lambda$-decomposition}}, for short. \end{example} \vphantom{} For us, the chief utility of van der Put series is the fact that they tell us how to compute a type of function-limit which we have seen to great effect in Chapter 2. The next proposition gives the details. \begin{prop}[\textbf{\textit{van der Put Identity}}] \label{prop:vdP identity}Let $f\in B\left(\mathbb{Z}_{p},K\right)$. Then, for any $\mathfrak{z}\in\mathbb{Z}_{p}$ which is in the domain of convergence of $S_{p}\left\{ f\right\} $: \begin{equation} S_{p}\left\{ f\right\} \left(\mathfrak{z}\right)\overset{K}{=}\lim_{k\rightarrow\infty}f\left(\left[\mathfrak{z}\right]_{p^{k}}\right)\label{eq:van der Put identity} \end{equation} We call this the \textbf{van der Put identity}\index{van der Put!identity}. We also have: \begin{equation} \sum_{n=0}^{p^{N}-1}c_{n}\left(f\right)\left[\mathfrak{z}\overset{p^{\lambda_{p}\left(n\right)}}{\equiv}n\right]\overset{\mathbb{F}}{=}f\left(\left[\mathfrak{z}\right]_{p^{N}}\right),\textrm{ }\forall\mathfrak{z}\in\mathbb{Z}_{p}\label{eq:truncated van der Put identity} \end{equation} which we call the (\textbf{$N$th}) \textbf{truncated van der Put identity}. \end{prop} \begin{rem} This is part of Exercise 62.B on page 192 of \cite{Ultrametric Calculus}. \end{rem} Proof: Performing a $\lambda$-decomposition yields: \begin{equation} S_{p}\left\{ f\right\} \left(\mathfrak{z}\right)=\sum_{n=0}^{\infty}c_{n}\left(f\right)\left[\mathfrak{z}\overset{p^{\lambda_{p}\left(n\right)}}{\equiv}n\right]\overset{K}{=}c_{0}\left(f\right)+\sum_{k=1}^{\infty}\sum_{n=p^{k-1}}^{p^{k}-1}c_{n}\left(f\right)\left[\mathfrak{z}\overset{p^{k}}{\equiv}n\right] \end{equation} Fixing $\mathfrak{z}$ and letting $k$ be arbitrary, we note that there is at most one value of $n\in\left\{ p^{k-1},\ldots,p^{k}-1\right\} $ which solves the congruence $\mathfrak{z}\overset{p^{k}}{\equiv}n$; that value is $n=\left[\mathfrak{z}\right]_{p^{k}}$. Thus: \begin{equation} \sum_{n=p^{k-1}}^{p^{k}-1}c_{n}\left(f\right)\left[\mathfrak{z}\overset{p^{k}}{\equiv}n\right]\overset{K}{=}c_{\left[\mathfrak{z}\right]_{p^{k}}}\left(f\right)\left[\lambda_{p}\left(\left[\mathfrak{z}\right]_{p^{k}}\right)=k\right]\label{eq:Inner term of vdP lambda decomposition} \end{equation} where the Iverson bracket on the right indicates that the right-hand side is $0$ whenever $\lambda_{p}\left(\left[\mathfrak{z}\right]_{p^{k}}\right)$ is not equal to $k$. This failure of equality occurs precisely when there is no $n\in\left\{ p^{k-1},\ldots,p^{k}-1\right\} $ which solves the congruence $\mathfrak{z}\overset{p^{k}}{\equiv}n$. Next, writing $\mathfrak{z}$ as $\mathfrak{z}=\sum_{j=0}^{\infty}\mathfrak{z}_{j}p^{j}$, where the $\mathfrak{z}_{j}$s are the $p$-adic digits of $\mathfrak{z}$, we have that: \begin{equation} c_{\left[\mathfrak{z}\right]_{p^{k}}}\left(f\right)\overset{K}{=}f\left(\left[\mathfrak{z}\right]_{p^{k}}\right)-f\left(\left[\mathfrak{z}\right]_{p^{k}}-\mathfrak{z}_{\lambda_{p}\left(\left[\mathfrak{z}\right]_{p^{k}}\right)-1}p^{\lambda_{p}\left(\left[\mathfrak{z}\right]_{p^{k}}\right)-1}\right) \end{equation} Note that $\left[\mathfrak{z}\right]_{p^{k}}$ can have \emph{at most} $k$ $p$-adic digits. In particular, $\lambda_{p}\left(\left[\mathfrak{z}\right]_{p^{k}}\right)=j$ if and only if the $\mathfrak{z}_{j-1}$ is the right-most non-zero $p$-adic digit of $\mathfrak{z}$. Consequently, letting $j\in\left\{ 0,\ldots,k\right\} $ denote the integer $\lambda_{p}\left(\left[\mathfrak{z}\right]_{p^{k}}\right)$, we have that $\left[\mathfrak{z}\right]_{p^{k}}=\left[\mathfrak{z}\right]_{p^{j}}$. We can then write: \begin{align*} c_{\left[\mathfrak{z}\right]_{p^{k}}}\left(f\right) & \overset{K}{=}f\left(\left[\mathfrak{z}\right]_{p^{j}}\right)-f\left(\left[\mathfrak{z}\right]_{p^{j}}-\mathfrak{z}_{j-1}p^{j-1}\right)\\ & =f\left(\left[\mathfrak{z}\right]_{p^{j}}\right)-f\left(\sum_{i=0}^{j-1}\mathfrak{z}_{i}p^{i}-\mathfrak{z}_{j-1}p^{j-1}\right)\\ & =f\left(\left[\mathfrak{z}\right]_{p^{j}}\right)-f\left(\sum_{i=0}^{j-2}\mathfrak{z}_{i}p^{i}\right)\\ & =f\left(\left[\mathfrak{z}\right]_{p^{j}}\right)-f\left(\left[\mathfrak{z}\right]_{p^{j-1}}\right) \end{align*} That is to say: \begin{equation} c_{\left[\mathfrak{z}\right]_{p^{k}}}\left(\chi\right)\overset{K}{=}\chi\left(\left[\mathfrak{z}\right]_{p^{\lambda_{p}\left(\left[\mathfrak{z}\right]_{p^{k}}\right)}}\right)-\chi\left(\left[\mathfrak{z}\right]_{p^{\lambda_{p}\left(\left[\mathfrak{z}\right]_{p^{k}}\right)-1}}\right) \end{equation} for all $\mathfrak{z}\in\mathbb{Z}_{p}\backslash\left\{ 0\right\} $, and all $k$ large enough so that $\lambda_{p}\left(\left[\mathfrak{z}\right]_{p^{k}}\right)\geq1$; that is, all $k>v_{p}\left(\mathfrak{z}\right)$. So, we have: \begin{equation} c_{\left[\mathfrak{z}\right]_{p^{k}}}\left(f\right)\left[\lambda_{p}\left(\left[\mathfrak{z}\right]_{p^{k}}\right)=k\right]\overset{K}{=}\left(f\left(\left[\mathfrak{z}\right]_{p^{k}}\right)-f\left(\left[\mathfrak{z}\right]_{p^{k-1}}\right)\right)\left[\lambda_{p}\left(\left[\mathfrak{z}\right]_{p^{k}}\right)=k\right]\label{eq:Iverson Bracket Check for vdP identity} \end{equation} \begin{claim} The Iverson bracket on the right-hand side of (\ref{eq:Iverson Bracket Check for vdP identity}) can be removed. Proof of claim: If $\lambda_{p}\left(\left[\mathfrak{z}\right]_{p^{k}}\right)=k$, then the Iverson bracket is $1$, and it disappears on its own without causing us any trouble. So, suppose $\lambda_{p}\left(\left[\mathfrak{z}\right]_{p^{k}}\right)\neq k$. Then the right-most digit of $\left[\mathfrak{z}\right]_{p^{k}}$\textemdash i.e., $\mathfrak{z}_{k-1}$\textemdash is $0$. But then, $\left[\mathfrak{z}\right]_{p^{k}}=\left[\mathfrak{z}\right]_{p^{k-1}}$, which makes the right-hand side of (\ref{eq:Iverson Bracket Check for vdP identity}) anyway, because $f\left(\left[\mathfrak{z}\right]_{p^{k}}\right)-f\left(\left[\mathfrak{z}\right]_{p^{k-1}}\right)$ is then equal to $0$. So, \emph{any case} which would cause the Iverson bracket to vanish causes $f\left(\left[\mathfrak{z}\right]_{p^{k}}\right)-f\left(\left[\mathfrak{z}\right]_{p^{k-1}}\right)$ to vanish. This tells us that the Iverson bracket in (\ref{eq:Iverson Bracket Check for vdP identity}) isn't actually needed. As such, we are justified in writing: \[ c_{\left[\mathfrak{z}\right]_{p^{k}}}\left(f\right)\left[\lambda_{p}\left(\left[\mathfrak{z}\right]_{p^{k}}\right)=k\right]\overset{K}{=}f\left(\left[\mathfrak{z}\right]_{p^{k}}\right)-f\left(\left[\mathfrak{z}\right]_{p^{k-1}}\right) \] This proves the claim. \end{claim} \vphantom{} That being done, we can write: \begin{align*} \sum_{n=0}^{\infty}c_{n}\left(f\right)\left[\mathfrak{z}\overset{p^{\lambda_{p}\left(n\right)}}{\equiv}n\right] & \overset{K}{=}c_{0}\left(f\right)+\sum_{k=1}^{\infty}\sum_{n=p^{k-1}}^{p^{k}-1}c_{n}\left(f\right)\left[\mathfrak{z}\overset{p^{k}}{\equiv}n\right]\\ & =c_{0}\left(f\right)+\sum_{k=1}^{\infty}c_{\left[\mathfrak{z}\right]_{p^{k}}}\left(f\right)\left[\lambda_{p}\left(\left[\mathfrak{z}\right]_{p^{k}}\right)=k\right]\\ & =c_{0}\left(f\right)+\underbrace{\sum_{k=1}^{\infty}\left(f\left(\left[\mathfrak{z}\right]_{p^{k}}\right)-f\left(\left[\mathfrak{z}\right]_{p^{k-1}}\right)\right)}_{\textrm{telescoping}}\\ \left(c_{0}\left(f\right)=f\left(\left[\mathfrak{z}\right]_{p^{0}}\right)=f\left(0\right)\right); & \overset{K}{=}f\left(0\right)+\lim_{k\rightarrow\infty}f\left(\left[\mathfrak{z}\right]_{p^{k}}\right)-f\left(0\right)\\ & \overset{K}{=}\lim_{k\rightarrow\infty}f\left(\left[\mathfrak{z}\right]_{p^{k}}\right) \end{align*} which is the van der Put identity. If we instead only sum $n$ from $0$ up to $p^{N}-1$, the upper limit of the $k$-sum in the above is changed from $\infty$ to $N$, which yields (\ref{eq:truncated van der Put identity}). Q.E.D. \vphantom{} With the van der Put basis, we can give a satisfying characterization of the $\left(p,K\right)$-adic continuity of a function in terms of the decay of its van der Put coefficients and the uniform convergence of its van der Put series. \begin{thm}[\textbf{van der Put Basis Theorem}\footnote{\cite{Robert's Book} gives this theorem on pages 182 and 183, however, it is stated there only for the case where $K$ is an extension of $\mathbb{Q}_{p}$.}] \label{thm:vdP basis theorem}Let $K$ be non-archimedean, and let $f:\mathbb{Z}_{p}\rightarrow K$ be any function. Then, the following are equivalent: \vphantom{} I. $f$ is continuous. \vphantom{} II. $\lim_{n\rightarrow\infty}\left|c_{n}\left(f\right)\right|_{K}=0$. \vphantom{} III. $S_{p:N}\left\{ f\right\} $ converges uniformly to $f$ in $\left(p,K\right)$-adic norm; that is: \begin{equation} \lim_{N\rightarrow\infty}\sup_{\mathfrak{z}\in\mathbb{Z}_{p}}\left|f\left(\mathfrak{z}\right)-\sum_{n=0}^{p^{N}-1}c_{n}\left(f\right)\left[\mathfrak{z}\overset{p^{\lambda_{p}\left(n\right)}}{\equiv}n\right]\right|_{K}=0 \end{equation} \end{thm} Proof: \textbullet{} ($\textrm{(I)}\Rightarrow\textrm{(II)}$) Suppose $f$ is continuous. Since $n_{-}=n-n_{\lambda_{p}\left(n\right)-1}p^{\lambda_{p}\left(n\right)-1}$ for all $n\geq1$, we have that: \begin{equation} \left|n-n_{-}\right|_{p}=\left|n_{\lambda_{p}\left(n\right)-1}p^{\lambda_{p}\left(n\right)-1}\right|_{p}=\frac{1}{p^{\lambda_{p}\left(n\right)-1}} \end{equation} Since this tends to $0$ as $n\rightarrow\infty$, the continuity of $f$ guarantees that $\left|c_{n}\left(f\right)\right|_{K}=\left|f\left(n\right)-f\left(n_{-}\right)\right|_{K}$ tends to $0$ as $n\rightarrow\infty$. \vphantom{} \textbullet{} ($\textrm{(II)}\Rightarrow\textrm{(III)}$) Suppose $\lim_{n\rightarrow\infty}\left|c_{n}\left(f\right)\right|_{K}=0$. Then, since $K$ is non-archimedean, the absolute value of the difference between $S_{p}\left\{ f\right\} $ and its $N$th partial sum satisfies: \begin{align*} \left|S_{p}\left\{ f\right\} \left(\mathfrak{z}\right)-\sum_{n=0}^{p^{N}-1}c_{n}\left(f\right)\left[\mathfrak{z}\overset{p^{\lambda_{p}\left(n\right)}}{\equiv}n\right]\right|_{K} & =\left|\sum_{n=p^{N}}^{\infty}c_{n}\left(f\right)\left[\mathfrak{z}\overset{p^{\lambda_{p}\left(n\right)}}{\equiv}n\right]\right|_{K}\\ \left(\textrm{ultrametric ineq.}\right); & \leq\sup_{n\geq p^{N}}\left|c_{n}\left(f\right)\left[\mathfrak{z}\overset{p^{\lambda_{p}\left(n\right)}}{\equiv}n\right]\right|_{K}\\ \left(\left[\mathfrak{z}\overset{p^{\lambda_{p}\left(n\right)}}{\equiv}n\right]\in\left\{ 0,1\right\} ,\textrm{ }\forall n,\mathfrak{z}\right); & \leq\sup_{n\geq p^{N}}\left|c_{n}\left(f\right)\right|_{K}\\ & \leq\sup_{n\geq N}\left|c_{n}\left(f\right)\right|_{K} \end{align*} Since $\left|c_{n}\left(f\right)\right|_{K}$ is given to tend to $0$ as $N\rightarrow\infty$, we then have that $\sup_{n\geq N}\left|c_{n}\left(f\right)\right|_{K}\rightarrow0$ as $N\rightarrow\infty$, and hence, that $S_{p}\left\{ f\right\} $ converges uniformly over $\mathbb{Z}_{p}$. The van der Put identity (\ref{eq:van der Put identity}) shows that $f$ is the point-wise limit of $S_{p:N}\left\{ f\right\} $, which in turn shows that $S_{p}\left\{ f\right\} $ converges point-wise to $f$. Since $S_{p}\left\{ f\right\} $ converges uniformly, this proves that $S_{p:N}\left\{ f\right\} $ converges uniformly to $f$. \vphantom{} \textbullet{} ($\textrm{(III)}\Rightarrow\textrm{(I)}$). Suppose $S_{p}\left\{ f\right\} $ converges uniformly to $f$. Since $S_{p}\left\{ f\right\} $ is the limit of a sequence of continuous functions ($\left\{ S_{p:N}\left\{ f\right\} \right\} _{N\geq0}$) which converge uniformly, its limit is necessarily continuous. Thus, $f$ is continuous. Q.E.D. \vphantom{} As an aside, we note the following: \begin{prop} \label{prop:Convergence of real-valued vdP series}Let $f\in C\left(\mathbb{Z}_{p},K\right)$. Then, the van der Put series for the continuous real-valued function $\mathfrak{z}\in\mathbb{Z}_{p}\mapsto\left|f\left(\mathfrak{z}\right)\right|_{q}\in\mathbb{R}$ is uniformly convergent. \end{prop} Proof: Since the $q$-adic absolute value is a continuous function from $K$ to $\mathbb{R}$, the continuity of $f:\mathbb{Z}_{p}\rightarrow K$ guarantees that $\left|f\left(\mathfrak{z}\right)\right|_{q}$ is a continuous\textemdash in fact, \emph{uniformly }continuous\textemdash real-valued function on $\mathbb{Z}_{p}$. The van der Put coefficients of $\left|f\right|_{q}$ are then: \begin{equation} c_{n}\left(\left|f\right|_{q}\right)=\begin{cases} \left|f\left(0\right)\right|_{q} & \textrm{if }n=0\\ \left|f\left(n\right)\right|_{q}-\left|f\left(n_{-}\right)\right|_{q} & \textrm{if }n\geq1 \end{cases} \end{equation} As such, (\ref{eq:van der Put identity}) yields: \begin{equation} \sum_{n=0}^{\infty}c_{n}\left(\left|f\right|_{q}\right)\left[\mathfrak{z}\overset{p^{\lambda_{p}\left(n\right)}}{\equiv}n\right]\overset{\mathbb{R}}{=}\lim_{n\rightarrow\infty}\left|f\left(\left[\mathfrak{z}\right]_{p^{n}}\right)\right|_{q} \end{equation} Since $\left|f\right|_{q}$ is continuous, we see that the van der Put series for $\left|f\right|_{q}$ then converges point-wise to $\left|f\right|_{q}$. Thus, we need only upgrade the convergence from point-wise to uniform. To do this, we show that the sequence $\left\{ \left|f\left(\left[\mathfrak{z}\right]_{p^{n}}\right)\right|_{q}\right\} _{n\geq1}$ is uniformly Cauchy. Let $m$ and $n$ be arbitrary positive integers. Then, by the reverse triangle inequality for the $q$-adic absolute value: \begin{equation} \left|\left|f\left(\left[\mathfrak{z}\right]_{p^{m}}\right)\right|_{q}-\left|f\left(\left[\mathfrak{z}\right]_{p^{n}}\right)\right|_{q}\right|\leq\left|f\left(\left[\mathfrak{z}\right]_{p^{m}}\right)-f\left(\left[\mathfrak{z}\right]_{p^{n}}\right)\right|_{q} \end{equation} Since the van der Put series for $f$ converges uniformly in $K$ over $\mathbb{Z}_{p}$, the $f\left(\left[\mathfrak{z}\right]_{p^{n}}\right)$s are therefore uniformly Cauchy, and hence, so too are the $\left|f\left(\left[\mathfrak{z}\right]_{p^{n}}\right)\right|_{q}$s. This establishes the desired uniform convergence. Q.E.D. \vphantom{} With the van der Put basis, we can completely characterize $C\left(\mathbb{Z}_{p},K\right)$ when $K$ is non-archimedean. \begin{thm} \label{thm:C(Z_p,K) is iso to c_0 K}If $K$ is non-archimedean, then the Banach space $C\left(\mathbb{Z}_{p},K\right)$ is isometrically isomorphic to $c_{0}\left(K\right)$ (the space of sequences in $K$ that converge to $0$ in $K$). \end{thm} Proof: Let $L:C\left(\mathbb{Z}_{p},K\right)\rightarrow c_{0}\left(K\right)$ be the map which sends every $f\in C\left(\mathbb{Z}_{p},K\right)$ to the sequence $\mathbf{c}\left(f\right)=\left(c_{0}\left(f\right),c_{1}\left(f\right),\ldots\right)$ whose entries are the van der Put coefficients of $f$. Since: \begin{equation} c_{n}\left(\alpha f+\beta g\right)=\alpha f\left(n\right)+\beta g\left(n\right)-\left(\alpha f\left(n_{-}\right)+\beta g\left(n_{-}\right)\right)=\alpha c_{n}\left(f\right)+\beta c_{n}\left(g\right) \end{equation} for all $\alpha,\beta\in K$ and all $f,g\in C\left(\mathbb{Z}_{p},K\right)$, we have that $L$ is linear. By the \textbf{van der Put Basis Theorem} (\textbf{Theorem \ref{thm:vdP basis theorem}}), for any $\mathbf{c}=\left(c_{0},c_{1},\ldots\right)$ in $c_{0}\left(K\right)$, the van der Put series $\sum_{n=0}^{\infty}c_{n}\left[\mathfrak{z}\overset{p^{\lambda_{p}\left(n\right)}}{\equiv}n\right]$ is an element of $C\left(\mathbb{Z}_{p},K\right)$ which $L$ sends to $\mathbf{c}$; thus, $L$ is surjective. All that remains is to establish norm-preservation; injectivity will then follow automatically. Since the norm on $c_{0}\left(K\right)$ is the supremum norm: \begin{equation} \left\Vert \mathbf{c}\right\Vert _{K}=\sup_{n\geq0}\left|c_{n}\right|_{K},\textrm{ }\forall\mathbf{c}\in c_{0}\left(K\right) \end{equation} observe that: \begin{equation} \left\Vert L\left\{ f\right\} \right\Vert _{p,K}=\left\Vert \mathbf{c}\left(f\right)\right\Vert _{K}=\sup_{n\geq0}\left|c_{n}\left(f\right)\right|_{K}\geq\underbrace{\left|\sum_{n=0}^{\infty}c_{n}\left(f\right)\left[\mathfrak{z}\overset{p^{\lambda_{p}\left(n\right)}}{\equiv}n\right]\right|_{K}}_{\left|f\left(\mathfrak{z}\right)\right|_{K}} \end{equation} for all $f\in C\left(\mathbb{Z}_{p},K\right)$ and all $\mathfrak{z}\in\mathbb{Z}_{p}$, and hence: \[ \left\Vert L\left\{ f\right\} \right\Vert _{p,K}\geq\sup_{\mathfrak{z}\in\mathbb{Z}_{p}}\left|f\left(\mathfrak{z}\right)\right|_{K}=\left\Vert f\right\Vert _{p,K},\textrm{ }\forall f\in C\left(\mathbb{Z}_{p},K\right) \] Finally, suppose there is an $f\in C\left(\mathbb{Z}_{p},K\right)$ so that the lower bound is strict: \begin{equation} \left\Vert L\left\{ f\right\} \right\Vert _{p,K}>\left\Vert f\right\Vert _{p,K} \end{equation} Since $\lim_{n\rightarrow\infty}\left|c_{n}\left(f\right)\right|_{K}=0$ the principles of ultrametric analysis (see page \pageref{fact:Principles of Ultrametric Analysis}) guarantee the existence of a $\nu\geq0$ so that $\sup_{n\geq0}\left|c_{n}\left(f\right)\right|_{K}=\left|c_{\nu}\left(f\right)\right|_{K}$, and hence, so that: \begin{equation} \left\Vert L\left\{ f\right\} \right\Vert _{p,K}=\left|c_{\nu}\left(f\right)\right|_{K}=\left|f\left(\nu\right)-f\left(\nu_{-}\right)\right|_{K} \end{equation} Thus, our assumption $\left\Vert L\left\{ f\right\} \right\Vert _{p,K}>\left\Vert f\right\Vert _{p,K}$ forces: \begin{equation} \sup_{\mathfrak{z}\in\mathbb{Z}_{p}}\left|f\left(\mathfrak{z}\right)\right|_{K}<\left|f\left(\nu\right)-f\left(\nu_{-}\right)\right|_{K} \end{equation} In particular, we have that the right-hand side is strictly greater than both $\left|f\left(\nu\right)\right|_{K}$ and $\left|f\left(\nu_{-}\right)\right|_{K}$; that is: \begin{equation} \max\left\{ \left|f\left(\nu\right)\right|_{K},\left|f\left(\nu_{-}\right)\right|_{K}\right\} <\left|f\left(\nu\right)-f\left(\nu_{-}\right)\right|_{K} \end{equation} However, the strong triangle inequality allows us to write: \begin{equation} \max\left\{ \left|f\left(\nu\right)\right|_{K},\left|f\left(\nu_{-}\right)\right|_{K}\right\} <\left|f\left(\nu\right)-f\left(\nu_{-}\right)\right|_{K}\leq\max\left\{ \left|f\left(\nu\right)\right|_{K},\left|f\left(\nu_{-}\right)\right|_{K}\right\} \end{equation} which is impossible. Thus, no such $f$ exists, and it must be that $\left\Vert L\left\{ f\right\} \right\Vert _{p,K}=\left\Vert f\right\Vert _{p,K}$, for all $f\in C\left(\mathbb{Z}_{p},K\right)$, and $L$ is therefore an isometry. Q.E.D. \begin{cor} Every $f\in C\left(\mathbb{Z}_{p},K\right)$ is uniquely representable as a van der Put series. \end{cor} Proof: Because map $L$ from the proof of \ref{thm:C(Z_p,K) is iso to c_0 K} is an isometry, it is necessarily injective. Q.E.D. \begin{cor} If $K$ is spherically incomplete, then $C\left(\mathbb{Z}_{p},K\right)$ is a reflexive Banach space, and its dual, is then isometrically isomorphic to $\ell^{\infty}\left(K\right)$. \end{cor} Proof: Since \textbf{Theorem \ref{thm:C(Z_p,K) is iso to c_0 K}} shows that $C\left(\mathbb{Z}_{p},K\right)$ is isometrically isomorphic to $c_{0}\left(K\right)$, by \textbf{van der Put's Theorem} (\textbf{Theorem \ref{thm:c_o and ell_infinit are each others duals in a spherically incomplete NA field}}, page \pageref{thm:c_o and ell_infinit are each others duals in a spherically incomplete NA field}), the spherical incompleteness of $K$ then guarantees that the Banach space $C\left(\mathbb{Z}_{p},K\right)$ is reflexive, and that its dual is isometrically isomorphic to $\ell^{\infty}\left(K\right)$. Q.E.D. \begin{thm} The span of the van der Put basis is dense in $B\left(\mathbb{Z}_{p},K\right)$, the space of bounded functions $f:\mathbb{Z}_{p}\rightarrow K$. \end{thm} Proof: The span (finite $K$-linear combinations) of the indicator functions: \[ \left\{ \left[\mathfrak{z}\overset{p^{n}}{\equiv}k\right]:n\geq0,k\in\left\{ 0,\ldots,p^{n}-1\right\} \right\} \] is precisely the set of locally constant functions $\mathbb{Z}_{p}\rightarrow\mathbb{C}_{q}$. Since each of these indicator functions is continuous, it can be approximated in supremum norm to arbitrary accuracy by the span of the van der Put basis functions. Since the locally constant functions are dense in $B\left(\mathbb{Z}_{p},K\right)$, we have that the $\left\{ \left[\mathfrak{z}\overset{p^{n}}{\equiv}k\right]:n\geq0,k\in\left\{ 0,\ldots,p^{n}-1\right\} \right\} $s are dense in $B\left(\mathbb{Z}_{p},K\right)$. This proves the van der Put basis is dense in $B\left(\mathbb{Z}_{p},K\right)$. Q.E.D. \subsection{\label{subsec:3.1.4. The--adic-Fourier}The $\left(p,q\right)$-adic Fourier Transform\index{Fourier!transform}} \begin{defn} A \textbf{$\left(p,q\right)$-adic Fourier series}\index{Fourier!series}\textbf{ }is a sum of the form: \begin{equation} f\left(\mathfrak{z}\right)\overset{\mathbb{C}_{q}}{=}\sum_{t\in\hat{\mathbb{Z}}_{p}}\hat{f}\left(t\right)e^{2\pi i\left\{ t\mathfrak{z}\right\} _{p}}\label{eq:General Form of a (p,q)-adic Fourier Series} \end{equation} where $\hat{f}$ is any function $\hat{\mathbb{Z}}_{p}\rightarrow\mathbb{C}_{q}$, and where $\mathfrak{z}$ is a $p$-adic variable (in $\mathbb{Z}_{p}$, or, more generally, $\mathbb{Q}_{p}$). We say the Fourier series is\textbf{ convergent at $\mathfrak{z}_{0}\in\mathbb{Q}_{p}$ }whenever the sum $f\left(\mathfrak{z}_{0}\right)$ converges in $\mathbb{C}_{q}$. As per our abuse of notation, we interpret $e^{2\pi i\left\{ t\mathfrak{z}_{0}\right\} _{p}}$ as a function of $t\in\hat{\mathbb{Z}}_{p}$ which, for each $t$, outputs a certain $p$-power root of unity in $\mathbb{C}_{q}$ in accordance with a fixed embedding of $\overline{\mathbb{Q}}$ in both $\mathbb{C}_{q}$ and $\mathbb{C}$. Additionally, we say the Fourier series \textbf{converges} \textbf{(point-wise) }on a set $U\subseteq\mathbb{Q}_{p}$ if it converges at each point in $U$. \end{defn} \vphantom{} For us, the most important Fourier series is the one for indicator functions of sets of the form $\mathfrak{a}+p^{n}\mathbb{Z}_{p}$, where $\mathfrak{a}\in\mathbb{Z}_{p}$ and $n\in\mathbb{N}_{0}$ \begin{prop} \label{prop:Indicator for p-adic coset Fourier identity}Fix $\mathfrak{a}\in\mathbb{Z}_{p}$ and $n\in\mathbb{Z}_{0}$. Then, Fourier series of the indicator function $\left[\mathfrak{z}\overset{p^{n}}{\equiv}\mathfrak{a}\right]$ is: \begin{equation} \left[\mathfrak{z}\overset{p^{n}}{\equiv}\mathfrak{a}\right]\overset{\overline{\mathbb{Q}}}{=}\frac{1}{p^{n}}\sum_{k=0}^{p^{n}-1}e^{2\pi i\left\{ k\frac{\mathfrak{z}-\mathfrak{a}}{p^{n}}\right\} _{p}},\textrm{ }\forall\mathfrak{z}\in\mathbb{Z}_{p}\label{eq:Fourier Identity for indicator functions} \end{equation} \end{prop} \begin{rem} This can also be written as: \begin{equation} \left[\mathfrak{z}\overset{p^{n}}{\equiv}\mathfrak{a}\right]\overset{\overline{\mathbb{Q}}}{=}\frac{1}{p^{n}}\sum_{\left|t\right|_{p}\leq p^{n}}e^{2\pi i\left\{ t\left(\mathfrak{z}-\mathfrak{a}\right)\right\} _{p}},\textrm{ }\forall\mathfrak{z}\in\mathbb{Z}_{p} \end{equation} \end{rem} \begin{rem} As indicated by the $\overline{\mathbb{Q}}$ over the equality, the above identity is valid in $\overline{\mathbb{Q}}$, and hence, in any field extension of $\overline{\mathbb{Q}}$, such as $\mathbb{C}$, or $\mathbb{C}_{q}$ for any prime $q$. \end{rem} Proof: \[ \frac{1}{p^{n}}\sum_{k=0}^{p^{n}-1}e^{2\pi i\left\{ k\frac{\mathfrak{z}-\mathfrak{a}}{p^{n}}\right\} _{p}}=\frac{1}{p^{n}}\sum_{k=0}^{p^{n}-1}e^{2\pi ik\left[\mathfrak{z}-\mathfrak{a}\right]_{p^{n}}/p^{n}}=\begin{cases} 1 & \textrm{if }\left[\mathfrak{z}-\mathfrak{a}\right]_{p^{n}}=0\\ 0 & \textrm{else} \end{cases} \] The formula follows from the fact that $\left[\mathfrak{z}-\mathfrak{a}\right]_{p^{n}}=0$ is equivalent to $\mathfrak{z}\overset{p^{n}}{\equiv}\mathfrak{a}$. Q.E.D. \begin{prop} A $\left(p,q\right)$-adic Fourier series $\sum_{t\in\hat{\mathbb{Z}}_{p}}\hat{f}\left(t\right)e^{2\pi i\left\{ t\mathfrak{z}\right\} _{p}}$ converges in $\mathbb{C}_{q}$ uniformly with respect to $\mathfrak{z}\in\mathbb{Q}_{p}$ if and only if $\hat{f}\in c_{0}\left(\hat{\mathbb{Z}}_{p},\mathbb{C}_{q}\right)$; that is, if and only if: \begin{equation} \lim_{n\rightarrow\infty}\max_{\left|t\right|_{p}=p^{n}}\left|\hat{f}\left(t\right)\right|_{q}\overset{\mathbb{R}}{=}0 \end{equation} \end{prop} Proof: Consider an arbitrary $\left(p,q\right)$-adic Fourier series $\sum_{t\in\hat{\mathbb{Z}}_{p}}\hat{f}\left(t\right)e^{2\pi i\left\{ t\mathfrak{z}\right\} _{p}}$. Due to the ultrametric topology of $\mathbb{C}_{q}$, the series will converge at any given $\mathfrak{z}\in\mathbb{Q}_{p}$ if and only if: \begin{equation} \lim_{n\rightarrow\infty}\max_{\left|t\right|_{p}=p^{n}}\left|\hat{f}\left(t\right)e^{2\pi i\left\{ t\mathfrak{z}\right\} _{p}}\right|_{q}\overset{\mathbb{R}}{=}0 \end{equation} In our abuse of notation, we have that $e^{2\pi i\left\{ t\mathfrak{z}\right\} _{p}}$ is a $p$-power root of unity in $\mathbb{C}_{q}$ for all $t\in\hat{\mathbb{Z}}_{p}$ and all $\mathfrak{z}\in\mathbb{Q}_{p}$; consequently, $\left|e^{2\pi i\left\{ t\mathfrak{z}\right\} _{p}}\right|_{q}=1$ for all $t$ and $\mathfrak{z}$. Hence: \begin{equation} \lim_{n\rightarrow\infty}\max_{\left|t\right|_{p}=p^{n}}\left|\hat{f}\left(t\right)e^{2\pi i\left\{ t\mathfrak{z}\right\} _{p}}\right|_{q}\overset{\mathbb{R}}{=}\lim_{n\rightarrow\infty}\max_{\left|t\right|_{p}=p^{n}}\left|\hat{f}\left(t\right)\right|_{q},\textrm{ }\forall\mathfrak{z}\in\mathbb{Q}_{p} \end{equation} Q.E.D. \vphantom{} Before we prove more on convergence, existence, or uniqueness, let us give the formal definition for what the Fourier coefficients of $f:\mathbb{Z}_{p}\rightarrow\mathbb{C}_{q}$ \emph{should} be, if they happened to exist. \begin{defn} \label{def:pq adic Fourier coefficients}For a function $f:\mathbb{Z}_{p}\rightarrow\mathbb{C}_{q}$, for each $t\in\hat{\mathbb{Z}}_{p}$, we define the $t$th $\left(p,q\right)$\textbf{-adic Fourier coefficient }of\index{Fourier!coefficients} $f$, denoted $\hat{f}\left(t\right)$, by the rule: \begin{equation} \hat{f}\left(t\right)\overset{\mathbb{C}_{q}}{=}\sum_{n=\frac{1}{p}\left|t\right|_{p}}^{\infty}\frac{c_{n}\left(f\right)}{p^{\lambda_{p}\left(n\right)}}e^{-2n\pi it}\label{eq:Definition of (p,q)-adic Fourier Coefficients} \end{equation} provided that the sum on the right is convergent in $\mathbb{C}_{q}$. \end{defn} \vphantom{} The existence criteria for $\hat{f}\left(t\right)$ are extremely simple: \begin{lem} Let $f:\mathbb{Z}_{p}\rightarrow\mathbb{C}_{q}$ be a function. Then, the series defining $\hat{f}\left(t\right)$ will converge in $\mathbb{C}_{q}$ uniformly with respect to $t\in\hat{\mathbb{Z}}_{p}$ if and only if: \begin{equation} \lim_{n\rightarrow\infty}\left|\frac{c_{n}\left(f\right)}{p^{\lambda_{p}\left(n\right)}}\right|_{q}\overset{\mathbb{R}}{=}0 \end{equation} In particular, the uniform convergence of $\hat{f}\left(t\right)$ for all values of $t$ will occur if and only if $f$ is continuous. \end{lem} Proof: $\mathbb{C}_{q}$'s non-archimedean topology guarantees that: \begin{equation} \hat{f}\left(t\right)\overset{\mathbb{C}_{q}}{=}\sum_{n=\frac{1}{p}\left|t\right|_{p}}^{\infty}\frac{c_{n}\left(f\right)}{p^{\lambda_{p}\left(n\right)}}e^{-2n\pi it} \end{equation} converges if and only if: \begin{equation} \lim_{n\rightarrow\infty}\left|\frac{c_{n}\left(f\right)}{p^{\lambda_{p}\left(n\right)}}e^{-2n\pi it}\right|_{q}\overset{\mathbb{R}}{=}\lim_{n\rightarrow\infty}\left|\frac{c_{n}\left(f\right)}{p^{\lambda_{p}\left(n\right)}}\right|_{q}\overset{\mathbb{R}}{=}0 \end{equation} Q.E.D. \begin{defn} We write \nomenclature{$c_{0}\left(\hat{\mathbb{Z}}_{p},\mathbb{C}_{q}\right)$}{set of functions $\hat{f}:\hat{\mathbb{Z}}_{p}\rightarrow\mathbb{C}_{q}$ for which $\lim_{n\rightarrow\infty}\max_{\left|t\right|_{p}=p^{n}}\left|\hat{f}\left(t\right)\right|_{q}\overset{\mathbb{R}}{=}0$}$c_{0}\left(\hat{\mathbb{Z}}_{p},\mathbb{C}_{q}\right)$ to denote the $\mathbb{C}_{q}$-vector space of all functions $\hat{f}:\hat{\mathbb{Z}}_{p}\rightarrow\mathbb{C}_{q}$ for which: \begin{equation} \lim_{n\rightarrow\infty}\max_{\left|t\right|_{p}=p^{n}}\left|\hat{f}\left(t\right)\right|_{q}\overset{\mathbb{R}}{=}0 \end{equation} We make $c_{0}\left(\hat{\mathbb{Z}}_{p},\mathbb{C}_{q}\right)$ into a non-archimedean Banach space by equipping it with the norm: \begin{equation} \left\Vert \hat{f}\right\Vert _{p,q}\overset{\textrm{def}}{=}\sup_{t\in\hat{\mathbb{Z}}_{p}}\left|\hat{f}\left(t\right)\right|_{q} \end{equation} \end{defn} \begin{thm} \label{thm:formula for Fourier series}Let $f:\mathbb{Z}_{p}\rightarrow\mathbb{C}_{q}$ be a function. Then: \vphantom{} I. $f$ admits a uniformly convergent $\left(p,q\right)$-adic Fourier series if and only if $\hat{f}\in c_{0}\left(\hat{\mathbb{Z}}_{p},\mathbb{C}_{q}\right)$. In particular, $f$ is continuous if and only if $\hat{f}\in c_{0}\left(\hat{\mathbb{Z}}_{p},\mathbb{C}_{q}\right)$. \vphantom{} II. If $f$ admits a uniformly convergent $\left(p,q\right)$-adic Fourier series, the series is unique, and is given by: \begin{equation} f\left(\mathfrak{z}\right)\overset{\mathbb{C}_{q}}{=}\sum_{t\in\hat{\mathbb{Z}}_{p}}\hat{f}\left(t\right)e^{2\pi i\left\{ t\mathfrak{z}\right\} _{p}},\textrm{ }\forall\mathfrak{z}\in\mathbb{Z}_{p} \end{equation} \end{thm} Proof: Let $f:\mathbb{Z}_{p}\rightarrow\mathbb{C}_{q}$ be a function. For the moment, let us proceed formally. Using (\ref{eq:Fourier Identity for indicator functions}), we can re-write the crossed van der Put series of $f:\mathbb{Z}_{p}\rightarrow\mathbb{C}_{q}$ as a Fourier series: \begin{align*} S_{p}\left\{ f\right\} \left(\mathfrak{z}\right) & \overset{\mathbb{C}_{q}}{=}c_{0}\left(f\right)+\sum_{n=1}^{\infty}c_{n}\left(f\right)\left[\mathfrak{z}\overset{p^{\lambda_{p}\left(n\right)}}{\equiv}n\right]\\ & =c_{0}\left(f\right)+\sum_{n=1}^{\infty}c_{n}\left(f\right)\frac{1}{p^{\lambda_{p}\left(n\right)}}\sum_{k=0}^{p^{\lambda_{p}\left(n\right)}-1}\exp\left(2k\pi i\left\{ \frac{\mathfrak{z}-n}{p^{\lambda_{p}\left(n\right)}}\right\} _{p}\right)\\ & =c_{0}\left(f\right)+\sum_{n=1}^{\infty}\frac{c_{n}\left(f\right)}{p^{\lambda_{p}\left(n\right)}}\sum_{k=0}^{p^{\lambda_{p}\left(n\right)}-1}\exp\left(-2n\pi i\frac{k}{p^{\lambda_{p}\left(n\right)}}\right)\exp\left(2\pi i\left\{ \frac{k\mathfrak{z}}{p^{\lambda_{p}\left(n\right)}}\right\} _{p}\right) \end{align*} Now, observe that for any function $g$: \begin{align} \sum_{k=0}^{p^{m}-1}g\left(\frac{k}{p^{m}}\right) & =\sum_{k=0}^{p^{m-1}-1}g\left(\frac{k}{p^{m-1}}\right)+\sum_{k\in\left(\mathbb{Z}/p^{m}\mathbb{Z}\right)^{\times}}g\left(\frac{k}{p^{m}}\right)\label{eq:Prufer-group sum decomposition - inductive step} \end{align} As such, by induction: \begin{align} \sum_{k=0}^{p^{m}-1}g\left(\frac{k}{p^{m}}\right) & =g\left(0\right)+\sum_{j=1}^{m}\sum_{k\in\left(\mathbb{Z}/p^{j}\mathbb{Z}\right)^{\times}}g\left(\frac{k}{p^{j}}\right),\textrm{ }\forall m\in\mathbb{N}_{1}\label{eq:Prufer-group sum decomposition} \end{align} The sum on the right is taken over all irreducible non-integer fractions whose denominator is a divisor of $p^{m}$. Adding $0$ into the mix, we see that the sum on the right is then exactly the same as summing $g\left(t\right)$ over every element of $\hat{\mathbb{Z}}_{p}$ whose $p$-adic magnitude is $\leq p^{m}$. Hence: \begin{equation} \sum_{k=0}^{p^{m}-1}g\left(\frac{k}{p^{m}}\right)=\sum_{\left|t\right|_{p}\leq p^{m}}g\left(t\right)\label{eq:Prufer-group sum decomposition - Re-indexing} \end{equation} and so: \[ \sum_{k=0}^{p^{\lambda_{p}\left(n\right)}-1}\exp\left(-2n\pi i\frac{k}{p^{\lambda_{p}\left(n\right)}}\right)\exp\left(2\pi i\left\{ \frac{k\mathfrak{z}}{p^{\lambda_{p}\left(n\right)}}\right\} _{p}\right)=\sum_{\left|t\right|_{p}\leq p^{\lambda_{p}\left(n\right)}}e^{-2n\pi it}e^{2\pi i\left\{ t\mathfrak{z}\right\} _{p}} \] and so: \begin{align*} S_{p}\left\{ f\right\} \left(\mathfrak{z}\right) & \overset{\mathbb{C}_{q}}{=}c_{0}\left(f\right)+\sum_{n=1}^{\infty}\frac{c_{n}\left(f\right)}{p^{\lambda_{p}\left(n\right)}}\sum_{\left|t\right|_{p}\leq p^{\lambda_{p}\left(n\right)}}e^{-2n\pi it}e^{2\pi i\left\{ t\mathfrak{z}\right\} _{p}}\\ & =c_{0}\left(f\right)+\sum_{n=1}^{\infty}\frac{c_{n}\left(f\right)}{p^{\lambda_{p}\left(n\right)}}\left(1+\sum_{0<\left|t\right|_{p}\leq p^{\lambda_{p}\left(n\right)}}e^{-2n\pi it}e^{2\pi i\left\{ t\mathfrak{z}\right\} _{p}}\right)\\ & =\underbrace{c_{0}\left(f\right)+\sum_{n=1}^{\infty}\frac{c_{n}\left(f\right)}{p^{\lambda_{p}\left(n\right)}}}_{\sum_{n=0}^{\infty}\frac{c_{n}\left(f\right)}{p^{\lambda_{p}\left(n\right)}}}+\sum_{n=1}^{\infty}\frac{c_{n}\left(f\right)}{p^{\lambda_{p}\left(n\right)}}\sum_{0<\left|t\right|_{p}\leq p^{\lambda_{p}\left(n\right)}}e^{-2n\pi it}e^{2\pi i\left\{ t\mathfrak{z}\right\} _{p}} \end{align*} Next, observe that a given $t\in\hat{\mathbb{Z}}_{p}\backslash\left\{ 0\right\} $ will occur in the innermost summand if and only if: \begin{align*} p^{\lambda_{p}\left(n\right)} & \geq\left|t\right|_{p}\\ & \Updownarrow\\ n & \geq p^{-\nu_{p}\left(t\right)-1}=\frac{1}{p}\left|t\right|_{p} \end{align*} So, re-indexing in terms of $t$, we have that: \begin{align*} S_{p}\left\{ f\right\} \left(\mathfrak{z}\right) & \overset{\mathbb{C}_{q}}{=}\sum_{n=0}^{\infty}\frac{c_{n}\left(f\right)}{p^{\lambda_{p}\left(n\right)}}+\sum_{t\in\hat{\mathbb{Z}}_{p}\backslash\left\{ 0\right\} }\sum_{n=\frac{1}{p}\left|t\right|_{p}}^{\infty}\frac{c_{n}\left(f\right)}{p^{\lambda_{p}\left(n\right)}}e^{-2n\pi it}e^{2\pi i\left\{ t\mathfrak{z}\right\} _{p}}\\ & =\sum_{t\in\hat{\mathbb{Z}}_{p}}\underbrace{\sum_{n=\frac{1}{p}\left|t\right|_{p}}^{\infty}\frac{c_{n}\left(f\right)}{p^{\lambda_{p}\left(n\right)}}e^{-2n\pi it}}_{\hat{f}\left(t\right)}e^{2\pi i\left\{ t\mathfrak{z}\right\} _{p}}\\ & =\sum_{t\in\hat{\mathbb{Z}}_{p}}\hat{f}\left(t\right)e^{2\pi i\left\{ t\mathfrak{z}\right\} _{p}} \end{align*} As such, if $\hat{f}\in c_{0}\left(\hat{\mathbb{Z}}_{p},\mathbb{C}_{q}\right)$, then: \[ S_{p}\left\{ f\right\} \left(\mathfrak{z}\right)=\sum_{t\in\hat{\mathbb{Z}}_{p}}\hat{f}\left(t\right)e^{2\pi i\left\{ t\mathfrak{z}\right\} _{p}} \] converges in $\mathbb{C}_{q}$ uniformly over $\mathbb{Z}_{p}$ and is equal to $f$, which is necessarily uniformly continuous. Conversely, if $f$ is continuous (which then makes $f$ uniformly continuous, since its domain is compact), then all of the above formal computations count as grouping and re-ordering of the uniformly convergent series: \begin{equation} f\left(\mathfrak{z}\right)\overset{\mathbb{C}_{q}}{=}\sum_{n=0}^{\infty}c_{n}\left(f\right)\left[\mathfrak{z}\overset{p^{\lambda_{p}\left(n\right)}}{\equiv}n\right] \end{equation} This proves that: \begin{equation} S_{p}\left\{ f\right\} \left(\mathfrak{z}\right)=\sum_{t\in\hat{\mathbb{Z}}_{p}}\hat{f}\left(t\right)e^{2\pi i\left\{ t\mathfrak{z}\right\} _{p}} \end{equation} converges in $\mathbb{C}_{q}$ uniformly over $\mathbb{Z}_{p}$, which then forces $\hat{f}\in c_{0}\left(\hat{\mathbb{Z}}_{p},\mathbb{C}_{q}\right)$. Uniqueness follows from the uniqueness of the van der Put coefficients of a continuous $\left(p,q\right)$-adic function. Q.E.D. \begin{cor} \label{cor:pq adic Fourier transform is an isometric isomorphism}The $\left(p,q\right)$-adic Fourier transform: \begin{equation} f\in C\left(\mathbb{Z}_{p},\mathbb{C}_{q}\right)\mapsto\hat{f}\in c_{0}\left(\hat{\mathbb{Z}}_{p},\mathbb{C}_{q}\right) \end{equation} is an isometric isomorphism of the Banach spaces $C\left(\mathbb{Z}_{p},\mathbb{C}_{q}\right)$ and $c_{0}\left(\hat{\mathbb{Z}}_{p},\mathbb{C}_{q}\right)$. \end{cor} Proof: \textbf{Theorem \ref{thm:formula for Fourier series} }shows that the Fourier transform is a bijection between $C\left(\mathbb{Z}_{p},\mathbb{C}_{q}\right)$ and $c_{0}\left(\hat{\mathbb{Z}}_{p},\mathbb{C}_{q}\right)$. The linearity of the Fourier transform then shows that it is an isomorphism of vector space (a.k.a., linear spaces). As for continuity, let $\hat{f}\in c_{0}\left(\hat{\mathbb{Z}}_{p},\mathbb{C}_{q}\right)$ be arbitrary. Then, we can write: \begin{align*} \left\Vert \hat{f}\right\Vert _{p,q} & =\sup_{t\in\hat{\mathbb{Z}}_{p}}\left|\sum_{n=\frac{1}{p}\left|t\right|_{p}}^{\infty}\frac{c_{n}\left(f\right)}{p^{\lambda_{p}\left(n\right)}}e^{-2n\pi it}\right|_{q}\\ & \leq\sup_{t\in\hat{\mathbb{Z}}_{p}}\sup_{n\geq\frac{1}{p}\left|t\right|_{p}}\left|\frac{c_{n}\left(f\right)}{p^{\lambda_{p}\left(n\right)}}e^{-2n\pi it}\right|_{q}\\ & =\sup_{t\in\hat{\mathbb{Z}}_{p}}\sup_{n\geq\frac{1}{p}\left|t\right|_{p}}\left|c_{n}\left(f\right)\right|_{q}\\ & =\sup_{n\geq0}\left|c_{n}\left(f\right)\right|_{q} \end{align*} For $n\geq1$, $c_{n}\left(f\right)=f\left(n\right)-f\left(n_{-}\right)$. As such, the ultrametric inequality implies that: \[ \left\Vert \hat{f}\right\Vert _{p,q}\leq\sup_{n\geq0}\left|c_{n}\left(f\right)\right|_{q}\leq\sup_{n\geq0}\left|f\left(n\right)\right|_{q}\leq\sup_{\mathfrak{z}\in\mathbb{Z}_{p}}\left|f\left(\mathfrak{z}\right)\right|_{q}=\left\Vert f\right\Vert _{p,q} \] On the other hand, if $f\in C\left(\mathbb{Z}_{p},\mathbb{C}_{q}\right)$, then: \begin{align*} \left\Vert f\right\Vert _{p,q} & =\sup_{\mathfrak{z}\in\mathbb{Z}_{p}}\left|\sum_{t\in\hat{\mathbb{Z}}_{p}}\hat{f}\left(t\right)e^{2\pi it\left\{ t\mathfrak{z}\right\} _{p}}\right|_{q}\\ & \leq\sup_{\mathfrak{z}\in\mathbb{Z}_{p}}\sup_{t\in\hat{\mathbb{Z}}_{p}}\left|\hat{f}\left(t\right)\right|_{q}\\ & \leq\sup_{t\in\hat{\mathbb{Z}}_{p}}\left|\hat{f}\left(t\right)\right|_{q}\\ & =\left\Vert \hat{f}\right\Vert _{p,q} \end{align*} Since the inequality holds in both directions, this forces $\left\Vert f\right\Vert _{p,q}=\left\Vert \hat{f}\right\Vert _{p,q}$. Thus, the $\left(p,q\right)$-adic Fourier transform is an isometric isomorphism, as desired. Q.E.D. \subsection{\label{subsec:3.1.5-adic-Integration-=00003D000026}$\left(p,q\right)$-adic Integration and the Fourier-Stieltjes Transform} Because we are focusing on the concrete case of functions on $\mathbb{Z}_{p}$ taking values in a non-archimedean field $K$ with residue field of characteristic $q\neq p$, the van der Put basis for $C\left(\mathbb{Z}_{p},K\right)$ allows us to explicitly define and compute integrals and measures in terms of their effects on this basis, and\textemdash equivalently\textemdash in terms of the $\left(p,q\right)$-adic characters $e^{2\pi i\left\{ t\mathfrak{z}\right\} _{p}}$. \begin{defn} A\textbf{ $\left(p,q\right)$-adic measure}\index{measure!left(p,qright)-adic@$\left(p,q\right)$-adic}\index{measure}\index{$p,q$-adic!measure} $d\mu$ is a continuous $K$-valued linear functional $C\left(\mathbb{Z}_{p},K\right)$; that is, $d\mu\in C\left(\mathbb{Z}_{p},K\right)^{\prime}$. Given a function $f\in C\left(\mathbb{Z}_{p},K\right)$, we write: \begin{equation} \int_{\mathbb{Z}_{p}}f\left(\mathfrak{z}\right)d\mu\left(\mathfrak{z}\right) \end{equation} to denote the image of $f$ under the linear functional $d\mu$. We call this the \textbf{integral }of $f$ with respect to $d\mu$. \vphantom{} Like one would expect, we can multiply measures by continuous functions. Given $g\in C\left(\mathbb{Z}_{p},K\right)$ and a measure $d\mu$, we define the measure $d\nu=gd\mu$ by the rule: \begin{equation} \nu\left(f\right)\overset{K}{=}\int_{\mathbb{Z}_{p}}f\left(\mathfrak{z}\right)d\nu\left(\mathfrak{z}\right)\overset{\textrm{def}}{=}\int_{\mathbb{Z}_{p}}f\left(\mathfrak{z}\right)g\left(\mathfrak{z}\right)d\mu\left(\mathfrak{z}\right)\overset{K}{=}\mu\left(f\times g\right) \end{equation} This definition makes sense since the continuity of $f$ and $g$ guarantees the continuity of their product. \vphantom{} Finally, we say a measure $d\mu$ is\index{translation invariance} \textbf{translation invariant }whenever: \begin{equation} \int_{\mathbb{Z}_{p}}f\left(\mathfrak{z}+\mathfrak{a}\right)d\mu\left(\mathfrak{z}\right)\overset{K}{=}\int_{\mathbb{Z}_{p}}f\left(\mathfrak{z}\right)d\mu\left(\mathfrak{z}\right),\textrm{ }\forall f\in C\left(\mathbb{Z}_{p},K\right),\textrm{ }\forall\mathfrak{a}\in\mathbb{Z}_{p} \end{equation} Note that the zero-measure (which sends every function to $0$) is translation-invariant, so this notion is not vacuous. \end{defn} \begin{prop}[\textbf{Integral-Series Interchange}] \label{prop:pq-adic integral interchange rule}Let $d\mu$ be a $\left(p,q\right)$-adic measure, and let $\left\{ F_{N}\right\} _{N\geq1}$ be a convergent sequence in $C\left(\mathbb{Z}_{p},\mathbb{C}_{q}\right)$. Then: \begin{equation} \lim_{N\rightarrow\infty}\int_{\mathbb{Z}_{p}}F_{N}\left(\mathfrak{z}\right)d\mu\left(\mathfrak{z}\right)\overset{\mathbb{C}_{q}}{=}\int_{\mathbb{Z}_{p}}\left(\lim_{N\rightarrow\infty}F_{N}\left(\mathfrak{z}\right)\right)d\mu\left(\mathfrak{z}\right)\label{eq:pq adic integral interchange rule} \end{equation} Equivalently, limits and integrals can be interchanged whenever the limit converges uniformly. \end{prop} Proof: This follows immediately from the definition of $\left(p,q\right)$-adic measures as continuous linear functionals on $C\left(\mathbb{Z}_{p},\mathbb{C}_{q}\right)$. Q.E.D. \begin{prop}[\textbf{Integration term-by-term}] \label{prop:term-by-term evaluation of integrals}Let $d\mu$ be a $\left(p,q\right)$-adic measure, and let $f\in C\left(\mathbb{Z}_{p},K\right)$. I. The integral of $f$ can be evaluated term-by term using its van der Put series \index{van der Put!series}: \begin{equation} \int_{\mathbb{Z}_{p}}f\left(\mathfrak{z}\right)d\mu\left(\mathfrak{z}\right)\overset{K}{=}\sum_{n=0}^{\infty}c_{n}\left(f\right)\int_{\mathbb{Z}_{p}}\left[\mathfrak{z}\overset{p^{\lambda_{p}\left(n\right)}}{\equiv}n\right]d\mu\left(\mathfrak{z}\right)\label{eq:Integration in terms of vdP basis} \end{equation} II. The integral of $f$ can be evaluated term-by term using its Fourier series: \begin{equation} \int_{\mathbb{Z}_{p}}f\left(\mathfrak{z}\right)d\mu\left(\mathfrak{z}\right)\overset{K}{=}\sum_{t\in\hat{\mathbb{Z}}_{p}}\hat{f}\left(t\right)\int_{\mathbb{Z}_{p}}e^{2\pi i\left\{ t\mathfrak{z}\right\} _{p}}d\mu\left(\mathfrak{z}\right),\forall f\in C\left(\mathbb{Z}_{p},K\right)\label{eq:Integration in terms of Fourier basis} \end{equation} \end{prop} Proof: Let $f$ and $d\mu$ be as given. By \textbf{Theorems \ref{thm:vdP basis theorem}} and \textbf{\ref{thm:formula for Fourier series}}, the continuity of $f$ guarantees the uniform convergence of the van der Put and Fourier series of $f$, respectively. \textbf{Proposition \ref{prop:pq-adic integral interchange rule}} then justifies the interchanges of limits and integrals in (I) and (II). Q.E.D. \vphantom{} Because of the van der Put basis, every $\left(p,q\right)$-adic measure is uniquely determined by what it does to the $n$th van der Put basis function. On the other hand, since every $f\in C\left(\mathbb{Z}_{p},K\right)$ can also be represented by a uniformly continuous $\left(p,q\right)$-adic Fourier series, a measure is therefore uniquely determined by what it does to the $\left(p,q\right)$-adic characters $e^{-2\pi i\left\{ t\mathfrak{z}\right\} _{p}}$. This leads to Fourier-Stieltjes transform of a $\left(p,q\right)$-adic measure. \begin{defn} Let $d\mu$ be a $\left(p,q\right)$-adic measure. Then, the \textbf{Fourier-Stieltjes transform}\index{Fourier-Stieltjes transform} of $d\mu$, denoted $\hat{\mu}$, is the function $\hat{\mu}:\hat{\mathbb{Z}}_{p}\rightarrow\mathbb{C}_{q}$ defined by: \begin{equation} \hat{\mu}\left(t\right)\overset{\mathbb{C}_{q}}{=}\int_{\mathbb{Z}_{p}}e^{-2\pi i\left\{ t\mathfrak{z}\right\} _{p}}d\mu\left(\mathfrak{z}\right),\textrm{ }\forall t\in\hat{\mathbb{Z}}_{p}\label{eq:Fourier-Stieltjes transform of a measure} \end{equation} where, for each $t$, $\int_{\mathbb{Z}_{p}}e^{-2\pi i\left\{ t\mathfrak{z}\right\} _{p}}d\mu\left(\mathfrak{z}\right)$ denotes the image of the function $e^{-2\pi i\left\{ t\mathfrak{z}\right\} _{p}}=e^{2\pi i\left\{ \left(-t\right)\mathfrak{z}\right\} _{p}}$ under $d\mu$. \end{defn} \begin{rem} \label{rem:parseval-plancherel formula for integration against measures}In this notation, (\ref{eq:Integration in terms of Fourier basis}) can be written as: \begin{equation} \int_{\mathbb{Z}_{p}}f\left(\mathfrak{z}\right)d\mu\left(\mathfrak{z}\right)\overset{K}{=}\sum_{t\in\hat{\mathbb{Z}}_{p}}\hat{f}\left(t\right)\hat{\mu}\left(-t\right)\overset{K}{=}\sum_{t\in\hat{\mathbb{Z}}_{p}}\hat{f}\left(-t\right)\hat{\mu}\left(t\right)\label{eq:Integration in terms of Fourier-Stieltjes coefficients} \end{equation} This formula will be \emph{vital }for studying $\chi_{H}$. \end{rem} \begin{thm} \label{thm:FST is an iso from measures to ell infinity}The Fourier-Stieltjes transform defines an isometric isomorphism from the Banach space $C\left(\mathbb{Z}_{p},\mathbb{C}_{q}\right)^{\prime}$ onto the Banach space $B\left(\hat{\mathbb{Z}}_{p},\mathbb{C}_{q}\right)$ of bounded $\mathbb{C}_{q}$-valued functions on $\hat{\mathbb{Z}}_{p}$. \end{thm} \begin{rem} The more general version of this result is so significant to non-archimedean functional analysis that van Rooij put it on the cover of his book \cite{van Rooij - Non-Archmedean Functional Analysis}! \end{rem} Proof: By \textbf{Corollary \ref{cor:pq adic Fourier transform is an isometric isomorphism}}, Fourier transform is an isometric isomorphism of the Banach spaces $C\left(\mathbb{Z}_{p},\mathbb{C}_{q}\right)$ and $c_{0}\left(\hat{\mathbb{Z}}_{p},\mathbb{C}_{q}\right)$. By \textbf{Theorem \ref{thm:c_o and ell_infinit are each others duals in a spherically incomplete NA field}}, since $\hat{\mathbb{Z}}_{p}$ is countable and $\mathbb{C}_{q}$ is spherically incomplete, $c_{0}\left(\hat{\mathbb{Z}}_{p},\mathbb{C}_{q}\right)$'s dual is then $B\left(\hat{\mathbb{Z}}_{p},\mathbb{C}_{q}\right)$. The isometric isomorphism the Fourier transform between the base spaces $C\left(\mathbb{Z}_{p},\mathbb{C}_{q}\right)$ and $c_{0}\left(\hat{\mathbb{Z}}_{p},\mathbb{C}_{q}\right)$ then indices an isometric isomorphism between their duals, as desired. Q.E.D. \begin{prop} \label{prop:Fourier transform of haar measure}Let $d\mu,d\nu\in C\left(\mathbb{Z}_{p},K\right)^{\prime}$ be the measures defined by: \begin{equation} \int_{\mathbb{Z}_{p}}\left[\mathfrak{z}\overset{p^{n}}{\equiv}k\right]d\mu\left(\mathfrak{z}\right)=\frac{1}{p^{n}},\textrm{ }\forall n\in\mathbb{N}_{0},\textrm{ }\forall k\in\mathbb{Z} \end{equation} and: \begin{equation} \int_{\mathbb{Z}_{p}}e^{-2\pi i\left\{ t\mathfrak{z}\right\} _{p}}d\nu\left(\mathfrak{z}\right)=\mathbf{1}_{0}\left(t\right),\textrm{ }\forall t\in\hat{\mathbb{Z}}_{p} \end{equation} Then, $d\mu=d\nu$. \end{prop} Proof: Direct computation using the formula: \begin{equation} \left[\mathfrak{z}\overset{p^{n}}{\equiv}k\right]=\frac{1}{p^{n}}\sum_{\left|t\right|_{p}\leq p^{n}}e^{-2\pi i\left\{ t\left(\mathfrak{z}-k\right)\right\} _{p}} \end{equation} from \textbf{Proposition \ref{prop:Indicator for p-adic coset Fourier identity}}. Q.E.D. \begin{defn} Following \textbf{Proposition \ref{prop:Fourier transform of haar measure}}, I write $d\mathfrak{z}$ to denote the $\left(p,q\right)$-adic measure defined by: \begin{equation} \int_{\mathbb{Z}_{p}}\left[\mathfrak{z}\overset{p^{n}}{\equiv}k\right]d\mathfrak{z}\overset{\textrm{def}}{=}\frac{1}{p^{n}},\textrm{ }\forall n\in\mathbb{N}_{0},\textrm{ }\forall k\in\mathbb{Z}\label{eq:Definition of (p,q)-adic Haar probability measure of k + p^n Z_p} \end{equation} and: \begin{equation} \int_{\mathbb{Z}_{p}}e^{-2\pi i\left\{ t\mathfrak{z}\right\} _{p}}d\mathfrak{z}\overset{\textrm{def}}{=}\mathbf{1}_{0}\left(t\right),\textrm{ }\forall t\in\hat{\mathbb{Z}}_{p}\label{eq:Definition of the Fourier-Stieltjes transform of the (p,q)-adic Haar probability measure} \end{equation} I call $d\mathfrak{z}$ the \textbf{$\left(p,q\right)$-adic Haar (probability) measure}\index{measure!Haar}. Note that this is the \emph{unique} measure satisfying the above two identities. \end{defn} \begin{thm} The set of \index{translation invariance}translation-invariant $\left(p,q\right)$-adic measures is the one-dimensional subspace of $B\left(\hat{\mathbb{Z}}_{p},K\right)$ spanned by $d\mathfrak{z}$. Additionally, $d\mathfrak{z}$ is the unique translation-invariant $\left(p,q\right)$-adic measure satisfying the unit normalization condition: \begin{equation} \int_{\mathbb{Z}_{p}}d\mathfrak{z}\overset{K}{=}1 \end{equation} \end{thm} Proof: First, let $d\mu=\mathfrak{c}d\mathfrak{z}$ for some $\mathfrak{c}\in K$. Then, letting $f\in C\left(\mathbb{Z}_{p},\mathbb{C}_{q}\right)$ and $\mathfrak{a}\in\mathbb{Z}_{p}$ be arbitrary: \begin{align*} \int_{\mathbb{Z}_{p}}f\left(\mathfrak{z}+\mathfrak{a}\right)d\mu\left(\mathfrak{z}\right) & =\mathfrak{c}\int_{\mathbb{Z}_{p}}\sum_{t\in\hat{\mathbb{Z}}_{p}}\hat{f}\left(t\right)e^{2\pi i\left\{ t\left(\mathfrak{z}+\mathfrak{a}\right)\right\} _{p}}d\mathfrak{z}\\ \left(\textrm{\textbf{Proposition }\textbf{\ref{prop:pq-adic integral interchange rule}}}\right); & =\mathfrak{c}\sum_{t\in\hat{\mathbb{Z}}_{p}}\hat{f}\left(t\right)e^{2\pi i\left\{ t\mathfrak{a}\right\} _{p}}\int_{\mathbb{Z}_{p}}e^{2\pi i\left\{ t\mathfrak{z}\right\} _{p}}d\mathfrak{z}\\ & =\mathfrak{c}\sum_{t\in\hat{\mathbb{Z}}_{p}}\hat{f}\left(t\right)e^{2\pi i\left\{ t\mathfrak{a}\right\} _{p}}\mathbf{1}_{0}\left(-t\right)\\ & =\mathfrak{c}\hat{f}\left(0\right)\\ & =\mathfrak{c}\int_{\mathbb{Z}_{p}}f\left(\mathfrak{z}\right)d\mathfrak{z}\\ & =\int_{\mathbb{Z}_{p}}f\left(\mathfrak{z}\right)d\mu\left(\mathfrak{z}\right) \end{align*} So, every scalar multiple of $d\mathfrak{z}$ is translation-invariant. Next, let $d\mu$ be translation-invariant. Then, for all $t\in\hat{\mathbb{Z}}_{p}$: \begin{equation} \hat{\mu}\left(t\right)=\int_{\mathbb{Z}_{p}}e^{-2\pi i\left\{ t\mathfrak{z}\right\} _{p}}d\mu\left(\mathfrak{z}\right)=\int_{\mathbb{Z}_{p}}e^{-2\pi i\left\{ t\left(\mathfrak{z}+1\right)\right\} _{p}}d\mu\left(\mathfrak{z}\right)=e^{-2\pi it}\hat{\mu}\left(t\right) \end{equation} Since $\hat{\mu}\left(t\right)\in\mathbb{C}_{q}$ for all $t$, the above forces $\hat{\mu}\left(t\right)=0$ for all $t\in\hat{\mathbb{Z}}_{p}\backslash\left\{ 0\right\} $. As such: \[ \hat{\mu}\left(t\right)=\hat{\mu}\left(0\right)\mathbf{1}_{0}\left(t\right) \] which shows $d\mu\left(\mathfrak{z}\right)=\hat{\mu}\left(0\right)d\mathfrak{z}$. Thus, $d\mu$ is in the span of $d\mathfrak{z}$. Hence, every translation-invariant measure is a scalar multiple of $d\mathfrak{z}$. Finally, since $\int_{\mathbb{Z}_{p}}d\mu\left(\mathfrak{z}\right)=\hat{\mu}\left(0\right)$, we have that $d\mathfrak{z}$ itself is the unique translation-invariant $\left(p,q\right)$-adic measure $d\mu\left(\mathfrak{z}\right)$ satisfying $\int_{\mathbb{Z}_{p}}d\mu\left(\mathfrak{z}\right)=1$. Q.E.D. \begin{lem}[\textbf{Integral Formula for the Fourier Transform}] For\index{Fourier!transform} any $f\in C\left(\mathbb{Z}_{p},\mathbb{C}_{q}\right)$, its Fourier transform $\hat{f}:\hat{\mathbb{Z}}_{p}\rightarrow\mathbb{C}_{q}$ is given by the integral formula: \begin{equation} \hat{f}\left(t\right)\overset{\mathbb{C}_{q}}{=}\int_{\mathbb{Z}_{p}}f\left(\mathfrak{z}\right)e^{-2\pi i\left\{ t\mathfrak{z}\right\} _{p}}d\mathfrak{z},\textrm{ }\forall t\in\hat{\mathbb{Z}}_{p}\label{eq:Integral Formula for the Fourier transform} \end{equation} \end{lem} Proof: Fix an arbitrary $t\in\hat{\mathbb{Z}}_{p}$. Then, by definition, (\ref{eq:Integral Formula for the Fourier transform}) is the image of the continuous $\left(p,q\right)$-adic function $f\left(\mathfrak{z}\right)e^{-2\pi i\left\{ t\mathfrak{z}\right\} _{p}}$ under the $\left(p,q\right)$-adic Haar measure. Expressing $f$ as a Fourier series, we have that: \[ e^{-2\pi i\left\{ t\mathfrak{z}\right\} _{p}}f\left(\mathfrak{z}\right)\overset{\mathbb{C}_{q}}{=}e^{-2\pi i\left\{ t\mathfrak{z}\right\} _{p}}\sum_{s\in\hat{\mathbb{Z}}_{p}}\hat{f}\left(s\right)e^{2\pi i\left\{ s\mathfrak{z}\right\} _{p}}=\sum_{s\in\hat{\mathbb{Z}}_{p}}\hat{f}\left(s\right)e^{2\pi i\left\{ \left(s-t\right)\mathfrak{z}\right\} _{p}} \] for all $\mathfrak{z}\in\mathbb{Z}_{p}$. Thus, by linearity, since the Fourier series is uniformly convergent in $\mathbb{C}_{q}$, \textbf{Proposition \ref{prop:term-by-term evaluation of integrals}} justifies integrating term-by-term: \begin{align*} \int_{\mathbb{Z}_{p}}f\left(\mathfrak{z}\right)e^{-2\pi i\left\{ t\mathfrak{z}\right\} _{p}}d\mathfrak{z} & =\sum_{s\in\hat{\mathbb{Z}}_{p}}\hat{f}\left(s\right)\int_{\mathbb{Z}_{p}}e^{2\pi i\left\{ \left(s-t\right)\mathfrak{z}\right\} _{p}}d\mathfrak{z}\\ & =\sum_{s\in\hat{\mathbb{Z}}_{p}}\hat{f}\left(s\right)\underbrace{\mathbf{1}_{0}\left(s-t\right)}_{1\textrm{iff }s=t}\\ & =\hat{f}\left(t\right) \end{align*} which gives the desired result. Q.E.D. \vphantom{} With this integral formula, we then get the usual integral formulas for convolution\index{convolution!of left(p,qright)-adic functions@of $\left(p,q\right)$-adic functions}, and they behave exactly as we would expect them to behave. \begin{defn}[\textbf{Convolution}] We define the \textbf{convolution }of functions on $\mathbb{Z}_{p}$ and $\hat{\mathbb{Z}}_{p}$, respectively, by: \begin{equation} \left(f*g\right)\left(\mathfrak{z}\right)\overset{\mathbb{C}_{q}}{=}\int_{\mathbb{Z}_{p}}f\left(\mathfrak{z}-\mathfrak{y}\right)g\left(\mathfrak{y}\right)d\mathfrak{y},\textrm{ }\forall\mathfrak{z}\in\mathbb{Z}_{p},\forall f,g\in C\left(\mathbb{Z}_{p},\mathbb{C}_{q}\right)\label{eq:Continuous Convolution Definition} \end{equation} \begin{equation} \left(\hat{f}*\hat{g}\right)\left(t\right)\overset{\mathbb{C}_{q}}{=}\sum_{\tau\in\hat{\mathbb{Z}}_{p}}\hat{f}\left(t-\tau\right)\hat{g}\left(\tau\right),\textrm{ }\forall t\in\hat{\mathbb{Z}}_{p},\forall\hat{f},\hat{g}\in c_{0}\left(\hat{\mathbb{Z}}_{p},\mathbb{C}_{q}\right)\label{eq:Discrete Convolution Definition} \end{equation} \end{defn} \begin{thm}[\textbf{The} \textbf{Convolution Theorem}] \label{thm:Convolution Theorem}\index{convolution!theorem}For $f,g\in C\left(\mathbb{Z}_{p},\mathbb{C}_{q}\right)$: \begin{align} \widehat{\left(f*g\right)}\left(t\right) & =\hat{f}\left(t\right)\hat{g}\left(t\right)\label{eq:Convolution Theorem 1}\\ \widehat{\left(fg\right)}\left(t\right) & =\left(\hat{f}*\hat{g}\right)\left(t\right)\label{eq:Convolution Theorem 2} \end{align} \end{thm} \vphantom{} We can also take convolutions of measures: \begin{defn} Let $d\mu,d\nu\in C\left(\mathbb{Z}_{p},\mathbb{C}_{q}\right)^{\prime}$ be $\left(p,q\right)$-adic measures. Then, the \textbf{convolution }of $d\mu$ and $d\nu$, denoted $d\mu*d\nu$, is the measure defined by the Fourier-Stieltjes transform: \begin{equation} \widehat{\left(d\mu*d\nu\right)}\left(t\right)\overset{\textrm{def}}{=}\hat{\mu}\left(t\right)\hat{\nu}\left(t\right)\label{eq:Definition of the convolution of two pq adic measures} \end{equation} That is: \begin{equation} \int_{\mathbb{Z}_{p}}f\left(\mathfrak{z}\right)\left(d\mu*d\nu\right)\left(\mathfrak{z}\right)\overset{\mathbb{C}_{q}}{=}\sum_{t\in\hat{\mathbb{Z}}_{p}}\hat{f}\left(-t\right)\hat{\mu}\left(t\right)\hat{\nu}\left(t\right),\textrm{ }\forall f\in C\left(\mathbb{Z}_{p},\mathbb{C}_{q}\right)\label{eq:Definition of the action of the convolution of pq adic measures on a function} \end{equation} \end{defn} \vphantom{} Next up, \textbf{Parseval-Plancherel Identity}:\index{Parseval-Plancherel Identity} \begin{thm}[\textbf{The Parseval-Plancherel Identity}] \label{thm:Parseval-Plancherel Identity}Let $f,g\in C\left(\mathbb{Z}_{p},\mathbb{C}_{q}\right)$. Then: \end{thm} \begin{equation} \int_{\mathbb{Z}_{p}}f\left(\mathfrak{z}\right)g\left(\mathfrak{z}\right)d\mathfrak{z}\overset{\mathbb{C}_{q}}{=}\sum_{t\in\hat{\mathbb{Z}}_{p}}\hat{f}\left(t\right)\hat{g}\left(-t\right),\textrm{ }\forall f,g\in C\left(\mathbb{Z}_{p},\mathbb{C}_{q}\right)\label{eq:Parseval-Plancherel Identity} \end{equation} \vphantom{} In all of these formulae, convergence, rearrangements, and interchanges are justified by the $q$-adic decay of $\hat{f}$ and $\hat{g}$ which is guaranteed to occur thanks to the continuity of $f$ and $g$. \begin{prop} Let $g\in C\left(\mathbb{Z}_{p},\mathbb{C}_{q}\right)$ and let $d\mu\in C\left(\mathbb{Z}_{p},\mathbb{C}_{q}\right)^{\prime}$. Viewing $g$ as the measure: \[ f\mapsto\int_{\mathbb{Z}_{p}}f\left(\mathfrak{z}\right)g\left(\mathfrak{z}\right)d\mathfrak{z} \] we can then define the convolution of $g$ and $d\mu$ as the measure with the $\left(p,q\right)$-adic Fourier-Stieltjes transform: \[ \widehat{\left(g*d\mu\right)}\left(t\right)=\sum_{t\in\hat{\mathbb{Z}}_{p}}\hat{g}\left(t\right)\hat{\mu}\left(t\right) \] The measure is $f*d\mu$ is then absolutely continuous with respect to the $\left(p,q\right)$-adic Haar measure $d\mathfrak{z}$, meaning that there is a continuous $\left(p,q\right)$-adic function $g\in C\left(\mathbb{Z}_{p},\mathbb{C}_{q}\right)$ so that: \[ \left(f*d\mu\right)\left(\mathfrak{z}\right)=g\left(\mathfrak{z}\right)d\mathfrak{z} \] In fact, we have that $\hat{g}\left(t\right)=\hat{f}\left(t\right)\hat{\mu}\left(t\right)$, and hence, we can\footnote{That is, the convolution of a continuous $\left(p,q\right)$-adic function and a $\left(p,q\right)$-adic measure is a continuous function.} view $f*d\mu$ as a continuous function $\mathbb{Z}_{p}\rightarrow\mathbb{C}_{q}$. \end{prop} Proof: Since $f$ is continuous, $\left|\hat{f}\left(t\right)\right|_{q}\rightarrow0$ as $\left|t\right|_{p}\rightarrow\infty$. Since $d\mu$ is a measure, $\left\Vert \hat{\mu}\right\Vert _{p,q}=\sup_{t\in\hat{\mathbb{Z}}_{p}}\left|\hat{\mu}\left(t\right)\right|_{q}<\infty$, hence, defining $\hat{g}\left(t\right)\overset{\textrm{def}}{=}\hat{f}\left(t\right)\hat{\mu}\left(t\right)$, we have that $\hat{g}\in c_{0}\left(\hat{\mathbb{Z}}_{p},\mathbb{C}_{q}\right)$, which shows that $\hat{g}$ is the Fourier transform of the continuous $\left(p,q\right)$-adic function: \[ g\left(\mathfrak{z}\right)\overset{\textrm{def}}{=}\sum_{t\in\hat{\mathbb{Z}}_{p}}\hat{g}\left(t\right)e^{2\pi i\left\{ t\mathfrak{z}\right\} _{p}},\textrm{ }\forall\mathfrak{z}\in\mathbb{Z}_{p} \] Furthermore, for any $h\in C\left(\mathbb{Z}_{p},\mathbb{C}_{q}\right)$: \begin{align*} \int_{\mathbb{Z}_{p}}h\left(\mathfrak{z}\right)\left(f*d\mu\right)\left(\mathfrak{z}\right) & \overset{\mathbb{C}_{q}}{=}\sum_{t\in\hat{\mathbb{Z}}_{p}}\hat{h}\left(-t\right)\hat{f}\left(t\right)\hat{\mu}\left(t\right)\\ & \overset{\mathbb{C}_{q}}{=}\sum_{t\in\hat{\mathbb{Z}}_{p}}\hat{h}\left(-t\right)\hat{g}\left(t\right)\\ & \overset{\mathbb{C}_{q}}{=}\int_{\mathbb{Z}_{p}}h\left(\mathfrak{z}\right)g\left(\mathfrak{z}\right)d\mathfrak{z} \end{align*} Hence, $\left(f*d\mu\right)\left(\mathfrak{z}\right)$ and $g\left(\mathfrak{z}\right)d\mathfrak{z}$ are identical as measures. Q.E.D. \vphantom{} For doing computations with $\left(p,q\right)$-adic integrals, the following simple change-of-variables\index{change of variables!affine substitutions} formula will be of the utmost import: \begin{lem}[\textbf{Change of Variables \textendash{} Affine substitutions}] \label{lem:Affine substitution change of variable formula}Let $f\in C\left(\mathbb{Z}_{p},K\right)$, where $K$ is a complete ring extension of $\mathbb{Z}_{q}$ which is itself contained in $\mathbb{C}_{q}$. Then: \begin{equation} \int_{\mathfrak{a}\mathbb{Z}_{p}}f\left(\mathfrak{z}\right)d\mathfrak{z}=\int_{p^{v_{p}\left(\mathfrak{a}\right)}\mathbb{Z}_{p}}f\left(\mathfrak{z}\right)d\mathfrak{z},\textrm{ }\forall\mathfrak{a}\in\mathbb{Z}_{p}\backslash\left\{ 0\right\} \label{eq:Multiplying domain of integration by a} \end{equation} \begin{equation} \int_{\mathbb{Z}_{p}}f\left(\mathfrak{a}\mathfrak{z}+\mathfrak{b}\right)d\mathfrak{z}\overset{K}{=}\frac{1}{\left|\mathfrak{a}\right|_{p}}\int_{\mathfrak{a}\mathbb{Z}_{p}+\mathfrak{b}}f\left(\mathfrak{z}\right)d\mathfrak{z},\textrm{ }\forall\mathfrak{a}\in\mathbb{Z}_{p}\backslash\left\{ 0\right\} ,\forall\mathfrak{b}\in\mathbb{Z}_{p}\label{eq:Change of variables formula - affine linear substitutions} \end{equation} \end{lem} \begin{rem} When working with these integrals, a key identity is: \[ \int_{p^{n}\mathbb{Z}_{p}+k}f\left(\mathfrak{z}\right)d\mu\left(\mathfrak{z}\right)=\int_{\mathbb{Z}_{p}}\left[\mathfrak{z}\overset{p^{n}}{\equiv}k\right]f\left(\mathfrak{z}\right)d\mu\left(\mathfrak{z}\right) \] which holds for all $f\in C\left(\mathbb{Z}_{p},\mathbb{C}_{q}\right)$, all $d\mu\in C\left(\mathbb{Z}_{p},\mathbb{C}_{q}\right)$, and all $n,k\in\mathbb{Z}$ with $n\geq0$. \end{rem} Proof: (\ref{eq:Multiplying domain of integration by a}) follows from the fact that $\mathfrak{a}\neq0$ implies there are (unique) $m\in\mathbb{N}_{0}$ and $\mathfrak{u}\in\mathbb{Z}_{p}^{\times}$ so that $\mathfrak{a}=p^{m}\mathfrak{u}$. Since multiplication by $\mathfrak{u}$ is a measure-preserving automorphism of the group $\left(\mathbb{Z}_{p},+\right)$, $\mathfrak{a}\mathbb{Z}_{p}=p^{m}\mathfrak{u}\mathbb{Z}_{p}=p^{m}\mathbb{Z}_{p}$. Here, $m=v_{p}\left(\mathfrak{z}\right)$. For (\ref{eq:Change of variables formula - affine linear substitutions}), by the translation-invariance of $d\mathfrak{z}$, it suffices to prove: \[ \int_{\mathbb{Z}_{p}}f\left(\mathfrak{a}\mathfrak{z}\right)d\mathfrak{z}\overset{K}{=}\frac{1}{\left|\mathfrak{a}\right|_{p}}\int_{\mathfrak{a}\mathbb{Z}_{p}}f\left(\mathfrak{z}\right)d\mathfrak{z} \] In fact, using the van der Put basis, we need only verify the formula for the indicator functions $f\left(\mathfrak{z}\right)=\left[\mathfrak{z}\overset{p^{n}}{\equiv}k\right]$. Because the integral of such an $f$ is rational-valued, note that the computation will be the same for the case of the \emph{real-valued }Haar probability measure $d\mathfrak{z}$ on $\mathbb{Z}_{p}$. For any field extension $\mathbb{F}$ of $\mathbb{Q}$, any translation-invariant linear functional on the space of $\mathbb{Q}$-valued functions on $\mathbb{Z}_{p}$ normalized to send the constant function $1$ to the number $1$ necessarily sends the functions $\left[\mathfrak{z}\overset{p^{n}}{\equiv}k\right]$ to the number $1/p^{n}$. Thus, the proof for the real-valued case, such as can be found in \cite{Bell - Harmonic Analysis on the p-adics} or \cite{Automorphic Representations} applies, and we are done. Q.E.D. \begin{rem} (\ref{eq:Change of variables formula - affine linear substitutions}) is \emph{also }valid in the case where $f:\mathbb{Z}_{p}\rightarrow\mathbb{C}$ ($\mathbb{C}$, not $\mathbb{C}_{q}$!) is integrable with respect to the \emph{real-valued }Haar probability measure on $\mathbb{Z}_{p}$. \end{rem} \begin{rem} Seeing as we write $\left[\mathfrak{z}\overset{p^{n}}{\equiv}\mathfrak{b}\right]$ to denote the indicator function for the set $\mathfrak{b}+p^{n}\mathbb{Z}_{p}$, we then have the identities: \begin{equation} \int_{\mathbb{Z}_{p}}\left[\mathfrak{z}\overset{p^{n}}{\equiv}\mathfrak{b}\right]f\left(\mathfrak{z}\right)d\mathfrak{z}=\int_{\mathfrak{b}+p^{n}\mathbb{Z}_{p}}f\left(\mathfrak{z}\right)d\mathfrak{z}=\frac{1}{p^{n}}\int_{\mathbb{Z}_{p}}f\left(p^{n}\mathfrak{y}+\mathfrak{b}\right)d\mathfrak{y}\label{eq:change of variable trifecta} \end{equation} \end{rem} \vphantom{} Lastly, we have the basic integral inequalities. Alas, the triangle inequality is not among them. \begin{prop}[\textbf{Integral Inequalities}] \index{triangle inequality!left(p,qright)-adic@$\left(p,q\right)$-adic}Let $f\in C\left(\mathbb{Z}_{p},\mathbb{C}_{q}\right)$. Then: \vphantom{} I. \begin{equation} \left|\int_{\mathbb{Z}_{p}}f\left(\mathfrak{z}\right)d\mathfrak{z}\right|_{q}\leq\sup_{\mathfrak{z}\in\mathbb{Z}_{p}}\left|f\left(\mathfrak{z}\right)\right|_{q}=\left\Vert f\right\Vert _{p,q}\label{eq:Triangle Inequality for the (p,q)-adic Haar measure} \end{equation} \vphantom{} II. For any measure $d\mu\in C\left(\mathbb{Z}_{p},\mathbb{C}_{q}\right)^{\prime}$: \begin{equation} \left|\int_{\mathbb{Z}_{p}}f\left(\mathfrak{z}\right)d\mu\left(\mathfrak{z}\right)\right|_{q}\leq\sup_{t\in\hat{\mathbb{Z}}_{p}}\left|\hat{f}\left(-t\right)\hat{\mu}\left(t\right)\right|_{q}\label{eq:Triangle inequality for an arbitrary (p,q)-adic measure (with Fourier)} \end{equation} \vphantom{} III.\index{triangle inequality!van der Put series} \begin{equation} \int_{\mathbb{Z}_{p}}\left|f\left(\mathfrak{z}\right)\right|_{q}d\mathfrak{z}\leq\sum_{n=0}^{\infty}\frac{\left|c_{n}\left(f\right)\right|_{q}}{p^{\lambda_{p}\left(n\right)}}\label{eq:Triangle inequality for Integral of q-adic absolute value} \end{equation} \end{prop} \begin{rem} As shown in \textbf{Example} \textbf{\ref{exa:triangle inequality failure}} (page \pageref{exa:triangle inequality failure}), unlike in real or complex analysis, there is not necessarily any relationship between $\left|\int f\right|_{q}$ and $\int\left|f\right|_{q}$. \end{rem} Proof: (I) is the consequence of the Theorem from page 281 of \emph{Ultrametric Calculus }(\cite{Ultrametric Calculus}) and the Exercise that occurs immediately after it. (II), meanwhile, is an immediate consequence of (\ref{eq:Integration in terms of Fourier-Stieltjes coefficients}). (III) is the only one that requires an argument. For (III), let $f\in C\left(\mathbb{Z}_{p},\mathbb{C}_{q}\right)$. Then: \begin{equation} \int_{\mathbb{Z}_{p}}f\left(\mathfrak{z}\right)d\mathfrak{z}\overset{K}{=}\sum_{n=0}^{\infty}c_{n}\left(f\right)\int_{\mathbb{Z}_{p}}\left[\mathfrak{z}\overset{p^{\lambda_{p}\left(n\right)}}{\equiv}n\right]d\mathfrak{z}=\sum_{n=0}^{\infty}\frac{c_{n}\left(f\right)}{p^{\lambda_{p}\left(n\right)}} \end{equation} where the interchange of sum and integral/linear functional is valid due to the convergence of the series, guaranteed by the $q$-adic decay of the $c_{n}\left(f\right)$s. Now, since $f$ is continuous, $\left|f\left(\mathfrak{z}\right)\right|_{q}$ is a continuous function from $\mathbb{Z}_{p}$ to $\mathbb{R}$\textemdash uniformly continuous, in fact, by the compactness of $\mathbb{Z}_{p}$. Thus, by \textbf{Proposition \ref{prop:Convergence of real-valued vdP series}}, $\left|f\left(\mathfrak{z}\right)\right|_{q}$'s van der Put series: \begin{equation} \sum_{n=0}^{\infty}c_{n}\left(\left|f\right|_{q}\right)\left[\mathfrak{z}\overset{p^{\lambda_{p}\left(n\right)}}{\equiv}n\right]=\left|f\left(0\right)\right|_{q}+\sum_{n=0}^{\infty}\underbrace{\left(\left|f\left(n\right)\right|_{q}-\left|f\left(n_{-}\right)\right|_{q}\right)}_{c_{n}\left(\left|f\right|_{q}\right)}\left[\mathfrak{z}\overset{p^{\lambda_{p}\left(n\right)}}{\equiv}n\right] \end{equation} is uniformly convergent in $\mathbb{R}$. As such, when integrating it with respect to the real-valued $p$-adic Haar measure, we may interchange integration and summation: \begin{equation} \int_{\mathbb{Z}_{p}}\left|f\left(\mathfrak{z}\right)\right|_{q}d\mathfrak{z}=\sum_{n=0}^{\infty}c_{n}\left(\left|f\right|_{q}\right)\int_{\mathbb{Z}_{p}}\left[\mathfrak{z}\overset{p^{\lambda_{p}\left(n\right)}}{\equiv}n\right]d\mathfrak{z}=\sum_{n=0}^{\infty}\frac{c_{n}\left(\left|f\right|_{q}\right)}{p^{\lambda_{p}\left(n\right)}} \end{equation} and the infinite series on the right is then necessarily convergent, since $f$'s uniformly continuity guarantees its integrability. Here: \begin{equation} \sum_{n=0}^{\infty}\frac{c_{n}\left(\left|f\right|_{q}\right)}{p^{\lambda_{p}\left(n\right)}}=\sum_{n=0}^{\infty}\frac{\left|f\left(n\right)\right|_{q}-\left|f\left(n_{-}\right)\right|_{q}}{p^{\lambda_{p}\left(n\right)}} \end{equation} Hence: \begin{align*} \int_{\mathbb{Z}_{p}}\left|f\left(\mathfrak{z}\right)\right|_{q}d\mathfrak{z} & =\left|\int_{\mathbb{Z}_{p}}\left|f\left(\mathfrak{z}\right)\right|_{q}d\mathfrak{z}\right|\\ & \leq\sum_{n=0}^{\infty}\frac{\left|\left|f\left(n\right)\right|_{q}-\left|f\left(n_{-}\right)\right|_{q}\right|}{p^{\lambda_{p}\left(n\right)}}\\ \left(\textrm{reverse }\Delta\textrm{-ineq.}\right); & \leq\sum_{n=0}^{\infty}\frac{\left|f\left(n\right)-f\left(n_{-}\right)\right|_{q}}{p^{\lambda_{p}\left(n\right)}}\\ & =\sum_{n=0}^{\infty}\frac{\left|c_{n}\left(f\right)\right|_{q}}{p^{\lambda_{p}\left(n\right)}} \end{align*} as desired. Q.E.D. \vphantom{} The most important practical consequence of this approach to $\left(p,q\right)$-adic integration is that it trivializes most of the basic concerns of classical real analysis. Because of our definition of $\left(p,q\right)$-adic measures as continuous linear functionals on the space of continuous $\left(p,q\right)$-adic functions, the equivalence of continuity with representability by a uniformly convergent Fourier series along with the action of measures on functions in terms of their Fourier series expansions shows that all continuous $\left(p,q\right)$-adic functions are integrable with respect to the $\left(p,q\right)$-adic Haar probability measure. Indeed, the following argument illustrates this quite nicely. \begin{example} \label{exa:end of 3.1.5. example}Let $f:\mathbb{Z}_{p}\rightarrow\mathbb{C}_{q}$ be given by the van der Put series: \begin{equation} f\left(\mathfrak{z}\right)\overset{\mathbb{C}_{q}}{=}\sum_{n=0}^{\infty}\mathfrak{a}_{n}\left[\mathfrak{z}\overset{p^{\lambda_{p}\left(n\right)}}{\equiv}n\right] \end{equation} where the series is assumed to converge merely point-wise. Letting: \begin{equation} f_{N}\left(\mathfrak{z}\right)\overset{\textrm{def}}{=}\sum_{n=0}^{p^{N}-1}\mathfrak{a}_{n}\left[\mathfrak{z}\overset{p^{\lambda_{p}\left(n\right)}}{\equiv}n\right] \end{equation} observe that, like in the classic real-analytical Lebesgue theory, the $f_{N}$s are step functions via (\ref{eq:truncated van der Put identity}): \begin{equation} f_{N}\left(\mathfrak{z}\right)=f\left(\left[\mathfrak{z}\right]_{p^{N}}\right)=\sum_{n=0}^{p^{N}-1}f\left(n\right)\left[\mathfrak{z}\overset{p^{N}}{\equiv}n\right] \end{equation} which converge to $f$. Similar to what is done in the theory of Lebesgue integration of real-valued functions on measure spaces, in Subsection \ref{subsec:3.1.6 Monna-Springer-Integration}, we will construct the class of integrable functions by considering limits of sequences of step functions. To that end, observe that: \begin{equation} \int_{\mathbb{Z}_{p}}f_{N}\left(\mathfrak{z}\right)d\mathfrak{z}\overset{\mathbb{C}_{q}}{=}\sum_{n=0}^{p^{N}-1}\mathfrak{a}_{n}\int_{\mathbb{Z}_{p}}\left[\mathfrak{z}\overset{p^{\lambda_{p}\left(n\right)}}{\equiv}n\right]d\mathfrak{z}=\sum_{n=0}^{p^{N}-1}\frac{\mathfrak{a}_{n}}{p^{\lambda_{p}\left(n\right)}} \end{equation} Since $p$ and $q$ are co-prime, $\left|\mathfrak{a}_{n}p^{-\lambda_{p}\left(n\right)}\right|_{q}=\left|\mathfrak{a}_{n}\right|_{q}$, which shows that: \begin{equation} \lim_{N\rightarrow\infty}\int_{\mathbb{Z}_{p}}f_{N}\left(\mathfrak{z}\right)d\mathfrak{z} \end{equation} exists in $\mathbb{C}_{q}$ if and only if $\lim_{n\rightarrow\infty}\left|\mathfrak{a}_{n}\right|_{q}\overset{\mathbb{R}}{=}0$. However, as we saw in \textbf{Theorem \ref{thm:vdP basis theorem}}, $\left|\mathfrak{a}_{n}\right|_{q}\rightarrow0$ if and only if $f$ is continuous! Schikhof leaves as an exercise in \emph{Ultrametric Calculus}' discussion of the van der Put basis the fact that $f\left(\left[\mathfrak{z}\right]_{p^{N}}\right)$ is the best possible approximation of $f$ in terms of the simple functions $\left[\mathfrak{z}\overset{p^{N}}{\equiv}n\right]$ for $n\in\left\{ 0,\ldots,p^{N}-1\right\} $ \cite{Ultrametric Calculus}. As such, if we are going to use step-function-approximation to construct integrable functions, this analysis of ours tells us that only the continuous functions are going to be integrable. \end{example} \vphantom{} Before we conclude, we need to cover one last special case where integration becomes quite simple. \begin{defn} Let $\mathbb{F}$ be any field, let $f:\mathbb{Z}_{p}\rightarrow\mathbb{F}$, and let $n\in\mathbb{N}_{0}$. We say $f$ is \textbf{constant over inputs modulo $p^{n}$} whenever $f\left(\mathfrak{z}\right)=f\left(\left[\mathfrak{z}\right]_{p^{n}}\right)$ for all $\mathfrak{z}\in\mathbb{Z}_{p}$. \end{defn} \begin{prop} Let $f:\mathbb{Z}_{p}\rightarrow\mathbb{F}$ be constant over inputs modulo $p^{N}$. Then: \begin{equation} f\left(\mathfrak{z}\right)=\sum_{n=0}^{p^{N}-1}f\left(n\right)\left[\mathfrak{z}\overset{p^{N}}{\equiv}n\right]\label{eq:representation of functions constant over inputs mod p^N} \end{equation} \end{prop} Proof: Immediate. Q.E.D. \begin{prop} \label{prop:How to integrate locally constant functions}Let $\mathbb{F}$ be a metrically complete valued field, and let $d\mu$ be a translation invariant $\mathbb{F}$-valued probability measure on $\mathbb{Z}_{p}$; that is: \begin{equation} \int_{\mathbb{Z}_{p}}d\mu\left(\mathfrak{z}\right)\overset{\mathbb{F}}{=}1 \end{equation} and: \begin{equation} \int_{\mathbb{Z}_{p}}f\left(\mathfrak{z}\right)d\mu\left(\mathfrak{z}\right)=\int_{\mathbb{Z}_{p}}f\left(\mathfrak{z}+\mathfrak{a}\right)d\mu\left(\mathfrak{z}\right)\in\mathbb{F},\textrm{ }\forall\mathfrak{a}\in\mathbb{Z}_{p},\textrm{ }\forall f\in C\left(\mathbb{Z}_{p},\mathbb{F}\right) \end{equation} If $f$ is constant over inputs modulo $p^{N}$, then: \begin{equation} \int_{\mathbb{Z}_{p}}f\left(\mathfrak{z}\right)d\mu=\frac{1}{p^{N}}\sum_{n=0}^{p^{N}-1}f\left(n\right) \end{equation} \end{prop} Proof: By the translation invariance of the $\mathbb{F}$-valued probability measure $d\mu$, it follows that: \begin{align*} 1 & =\int_{\mathbb{Z}_{p}}d\mu\left(\mathfrak{z}\right)\\ & =\int_{\mathbb{Z}_{p}}\sum_{n=0}^{p^{N}-1}\left[\mathfrak{z}\overset{p^{N}}{\equiv}n\right]d\mu\left(\mathfrak{z}\right)\\ & =\sum_{n=0}^{p^{N}-1}\int_{\mathbb{Z}_{p}}\left[\mathfrak{z}\overset{p^{N}}{\equiv}0\right]d\mu\left(\mathfrak{z}\right)\\ & =p^{N}\int_{\mathbb{Z}_{p}}\left[\mathfrak{z}\overset{p^{N}}{\equiv}0\right]d\mu\left(\mathfrak{z}\right) \end{align*} Hence: \[ \int_{\mathbb{Z}_{p}}\left[\mathfrak{z}\overset{p^{N}}{\equiv}0\right]d\mu\left(\mathfrak{z}\right)\overset{\mathbb{F}}{=}\frac{1}{p^{N}} \] and so, by translation invariance: \[ \int_{\mathbb{Z}_{p}}\left[\mathfrak{z}\overset{p^{N}}{\equiv}n\right]d\mu\left(\mathfrak{z}\right)\overset{\mathbb{F}}{=}\frac{1}{p^{N}},\textrm{ }\forall n,N \] By linearity, integrating the formula (\ref{eq:representation of functions constant over inputs mod p^N}) yields the desired formula for the integral of $f\left(\mathfrak{z}\right)d\mu\left(\mathfrak{z}\right)$. Q.E.D. \vphantom{} Note that this method can be used to integrate both $\left(p,q\right)$-adic functions \emph{and }$\left(p,\infty\right)$-adic functions, seeing as (provided that $p\neq q$), both those cases admit translation invariant probability measures on $\mathbb{Z}_{p}$. \subsection{\label{subsec:3.1.6 Monna-Springer-Integration}Monna-Springer Integration} FOR THIS SUBSECTION, $K$ IS A COMPLETE ULTRAMETRIC FIELD, $p$ AND $q$ ARE DISTINCT PRIMES, AND $X$ IS AN ARBITRARY ULTRAMETRIC SPACE. \vphantom{} The simplicity, directness, practicality and technical clarity of the basis-based approach to $\left(p,q\right)$-adic integration we have covered so far all speak their usefulness in doing $\left(p,q\right)$-adic analysis. Indeed, it will completely suffice for all of our investigations of $\chi_{H}$. That this approach works out so nicely is all thanks to the translation-invariance of the $\left(p,q\right)$-adic Haar measure. At the same time, note that we have failed to mention many of the issues central to most any theory of integration: measurable sets, measurable functions, and the interchanging of limits and integrals. To do these topics justice, we need Monna-Springer integration theory: the method for defining measures and integration of $K$-valued functions on an arbitrary ultrametric space $X$. Presenting Monna-Springer theory means bombarding the reader with many definitions and almost just as many infima and suprema. As such, to maximize clarity, I have supplemented the standard exposition with examples from the $\left(p,q\right)$-case and additional discussion. To begin, it is worth reflecting on the notion of measure defined in Subsection \ref{subsec:3.1.5-adic-Integration-=00003D000026}. Because we defined measure as continuous linear functionals on the space of continuous $\left(p,q\right)$-adic functions, we will be able to say a set in $\mathbb{Z}_{p}$ is $\left(p,q\right)$-adically measurable'' precisely when the indicator function for that set is $\left(p,q\right)$-adically continuous. In classical integration theory\textemdash particularly when done in the Bourbaki-style functional analysis approach\textemdash indicator functions provide a bridge between the world of functions and the world of sets. Indicator functions for measurable subsets of $\mathbb{R}$ can be approximated in $L^{1}$ to arbitrary accuracy by piece-wise continuous functions. What makes non-archimedean integration theory so different from its archimedean counterpart are the restrictions which $\mathbb{C}_{q}$'s ultrametric structure imposes on continuous functions. Because of the equivalence of integrable functions and continuous functions, the only sets in $\mathbb{Z}_{p}$ which we can call $\left(p,q\right)$-adically measurable are those whose indicator functions are $\left(p,q\right)$-adically continuous. Since indicator functions $\left[\mathfrak{z}\overset{p^{n}}{\equiv}k\right]$ are continuous, it follows that any finite union, intersection, or complement of sets of the form $k+p^{n}\mathbb{Z}_{p}$ ``$\left(p,q\right)$-adically measurable''. Note that all of these sets are compact clopen sets. On the other hand, single points will \emph{not }be measurable. \begin{defn} Fix $\mathfrak{a}\in\mathbb{Z}_{p}$. Then, define the function \nomenclature{$\mathbf{1}_{\mathfrak{a}}$}{ }$\mathbf{1}_{\mathfrak{a}}:\mathbb{Z}_{p}\rightarrow K$ by: \begin{equation} \mathbf{1}_{\mathfrak{a}}\left(\mathfrak{z}\right)=\begin{cases} 1 & \textrm{if }\mathfrak{z}=\mathfrak{a}\\ 0 & \textrm{else} \end{cases}\label{eq:Definition of a one-point function} \end{equation} I call any non-zero scalar multiple of (\ref{eq:Definition of a one-point function}) a \textbf{one-point function} supported at $\mathfrak{a}$. More generally, I call any finite linear combination of the form: \begin{equation} \sum_{n=1}^{N}\mathfrak{c}_{n}\mathbf{1}_{\mathfrak{a}_{n}}\left(\mathfrak{z}\right) \end{equation} an\textbf{ $N$-point function}; here, the $\mathfrak{a}_{n}$s in $\mathbb{Z}_{p}$ and the $\mathfrak{c}_{n}$s in $K\backslash\left\{ 0\right\} $. \end{defn} \begin{rem} It is a simple exercise to prove the formulae: \begin{equation} \mathbf{1}_{0}\left(\mathfrak{z}\right)=1-\sum_{n=0}^{\infty}\sum_{k=1}^{p-1}\left[\mathfrak{z}\overset{p^{n+1}}{\equiv}kp^{n}\right]\label{eq:van der Point series for one point function at 0} \end{equation} \begin{equation} \mathbf{1}_{\mathfrak{a}}\left(\mathfrak{z}\right)=1-\sum_{n=0}^{\infty}\sum_{k=1}^{p-1}\left[\mathfrak{z}-\mathfrak{a}\overset{p^{n+1}}{\equiv}kp^{n}\right]\label{eq:vdP series for a one-point function} \end{equation} \end{rem} \begin{example} The indicator function of a single point in $\mathbb{Z}_{p}$ is not integrable with respect to the $\left(p,q\right)$-adically Haar probability measure. To see this, note that (\ref{eq:van der Point series for one point function at 0}) is actually a van der Put series expression for $\mathbf{1}_{0}\left(\mathfrak{z}\right)$, and the non-zero van der Put coefficients of that series are either $1$ or $-1$. By \textbf{Example \ref{exa:end of 3.1.5. example}} (see page \pageref{exa:end of 3.1.5. example}), since these coefficients do not tend to zero, the integrals of the step-function-approximations of $\mathbf{1}_{0}\left(\mathfrak{z}\right)$ do not converge $q$-adically to a limit. For the case of a general one-point function, the translation invariance of $d\mathfrak{z}$ allows us to extend the argument for $\mathbf{1}_{0}$ to $\mathbf{1}_{\mathfrak{a}}$ for any $\mathfrak{a}\in\mathbb{Z}_{p}$. \end{example} \vphantom{} Seeing as we will \emph{define }the most general class of $\left(p,q\right)$-adically Haar-measurable subsets of $\mathbb{Z}_{p}$ as being precisely those sets whose indicator functions are the limit of a sequence of $\left(p,q\right)$-adically integrable functions, the failure of one-point functions to be integrable tells us that a set consisting of one point (or, more generally, any set consisting of finitely many points) \emph{is not going to be $\left(p,q\right)$-adically Haar integrable!} This is a \emph{fundamental} contrast with the real case, where the vanishing measure of a single point tells us that integrability is unaffected by changing functions' values at finitely many (or even countably many) points. In $\left(p,q\right)$-adic analysis, therefore, there are no notions of ``almost everywhere'' or ``null sets''. From a more measure-theoretic standpoint, this also shows that it is too much to expect an arbitrary Borel set\index{Borel!set} to be $\left(p,q\right)$-adically measurable. In light of this, Monna-Springer integration instead begins by working with a slightly smaller, more manageable algebra of sets: the compact clopen sets. \begin{defn} We write $\Omega\left(X\right)$ to denote the collection of all compact open subsets of $X$ (equivalently, compact clopen sets). We make $\Omega\left(X\right)$ an algebra of sets by equipping it with the operations of unions, intersections, and complements. \end{defn} \vphantom{} In more abstract treatments of non-archimedean integration (such as \cite{van Rooij - Non-Archmedean Functional Analysis,Measure-theoretic approach to p-adic probability theory,van Rooij and Schikhof "Non-archimedean integration theory"}), one begins by fixing a ring of sets $\mathcal{R}$ to use to construct simple functions, which then give rise to a notion of measure. Here, we have chosen $\mathcal{R}$ to be $\Omega\left(X\right)$. Taking limits of simple functions in $L^{1}$ with respect to a given measure $d\mu$ then enlarges $\mathcal{R}$ to an algebra of sets, denoted $\mathcal{R}_{\mu}$, which is the maximal extension of $\mathcal{R}$ containing subsets of $X$ which admit a meaningful notion of $\mu$-measure. This is, in essence, what we will do here, albeit in slightly more concrete terms. For now, though, let us continue through the definitions. Before proceeding, I should also mention that my exposition here is a hybrid of the treatments given by Schikhof in one of the Appendices of \cite{Ultrametric Calculus} and the chapter on Monna-Springer integration theory given by Khrennikov in his book \cite{Quantum Paradoxes} and in the paper \cite{Measure-theoretic approach to p-adic probability theory}. \begin{defn}[\textbf{Integrals and Measures}\footnote{Taken from \cite{Ultrametric Calculus}.}] \ \vphantom{} I. Elements of the dual space $C\left(X,K\right)^{\prime}$ are \index{integral!Monna-Springer} called \textbf{integrals }on $C\left(X,K\right)$. \vphantom{} II. A \index{measure}\textbf{ measure}\index{measure!non-archimedean} is a function $\mu:\Omega\left(X\right)\rightarrow K$ which satisfies the conditions: \vphantom{} i. (Additivity): $\mu\left(U\cup V\right)=\mu\left(U\right)+\mu\left(V\right)$ for all $U,V\in\Omega\left(X\right)$ with $U\cap V=\varnothing$. \vphantom{} ii. (Boundedness): the real number $\left\Vert \mu\right\Vert $ (the\index{measure!total variation} \textbf{total variation }of $\mu$) defined by: \begin{equation} \left\Vert \mu\right\Vert \overset{\textrm{def}}{=}\sup\left\{ \left|\mu\left(U\right)\right|_{K}:U\in\Omega\left(X\right)\right\} \label{eq:Definition of the norm of a measure} \end{equation} is finite. \vphantom{} III. We write $M\left(X,K\right)$ to denote the vector space of $K$-valued measures on $X$. Equipping this vector space with (\ref{eq:Definition of the norm of a measure}) makes it into a non-archimedean normed vector space. \end{defn} \vphantom{} Even though our starting algebra of measurable sets ($\Omega\left(X\right)$) is smaller than its classical counterpart, there is still a correspondence between measures and continuous linear functionals \cite{Ultrametric Calculus}. \begin{thm} For each $\varphi\in C\left(X,K\right)^{\prime}$ (which sends a function $f$ to the scalar $\varphi\left(f\right)$) the map $\mu_{\varphi}:\Omega\left(X\right)\rightarrow K$ defined by: \[ \mu_{\varphi}\left(U\right)\overset{\textrm{def}}{=}\varphi\left(\mathbf{1}_{U}\right),\textrm{ }\forall U\in\Omega\left(X\right) \] (where $\mathbf{1}_{U}$ is the indicator function of $U$) is a measure. Moreover, the map: \[ \varphi\in C\left(X,K\right)^{\prime}\mapsto\mu_{\varphi}\in M\left(X,K\right) \] is a $K$-linear isometry of $C\left(X,K\right)^{\prime}$ onto $M\left(X,K\right)$. We also have that: \begin{equation} \left|\varphi\left(f\right)\right|_{K}\leq\left\Vert f\right\Vert _{X,K}\left\Vert \mu_{\varphi}\right\Vert ,\textrm{ }\forall f\in C\left(X,K\right),\forall\varphi\in C\left(X,K\right)^{\prime}\label{eq:Absolute Value of the output of a functional in terms of the associated measure} \end{equation} \end{thm} \begin{notation} In light of the above theorem, we shall now adopt the following notations for $K$-valued measures and elements of $C\left(X,K\right)^{\prime}$: \vphantom{} I. Elements of $C\left(X,K\right)^{\prime}$ will all be written with a $d$ in front of them: $d\mathfrak{z}$, $d\mu$, $d\mu\left(\mathfrak{z}\right)$, $dA_{3}$, etc. \vphantom{} II. Given $d\mu\in C\left(X,K\right)^{\prime}$ and $f\in C\left(X,K\right)$, we denote the image of $f$ under $d\mu$ by: \[ \int_{X}f\left(\mathfrak{z}\right)d\mu\left(\mathfrak{z}\right) \] \vphantom{} III. Given $d\mu\in C\left(\mathbb{Z}_{p},\mathbb{C}_{q}\right)^{\prime}$ and $U\in\Omega\left(\mathbb{Z}_{p}\right)$, we will denote the $\mu$-measure of $U$ by the symbols $\mu\left(U\right)$ and $\int_{U}d\mu$. \vphantom{} IV. Given any function $g\in C\left(X,K\right)$, the operation of point-wise multiplication: \[ f\in C\left(X,K\right)\mapsto fg\in C\left(X,K\right) \] defines a continuous linear operator on $C\left(X,K\right)$. As such, we can multiply elements of $C\left(X,K\right)^{\prime}$ by continuous functions to obtain new elements of $C\left(X,K\right)^{\prime}$. Given $d\mu\in C\left(X,K\right)^{\prime}$, we write $g\left(\mathfrak{z}\right)d\mu$ and $g\left(\mathfrak{z}\right)d\mu\left(\mathfrak{z}\right)$ to denote the measure which first multiplies a function $f$ by $g$ and then integrates the product against $d\mu$: \begin{equation} \int_{\mathbb{Z}_{p}}f\left(\mathfrak{z}\right)\left(g\left(\mathfrak{z}\right)d\mu\left(\mathfrak{z}\right)\right)\overset{\textrm{def}}{=}\int_{\mathbb{Z}_{p}}\left(f\left(\mathfrak{z}\right)g\left(\mathfrak{z}\right)\right)d\mu\left(\mathfrak{z}\right)\label{eq:Definition of the product of a measure by a continuous function} \end{equation} \vphantom{} V. Given any $f\in C\left(X,K\right)$, any $d\mu\in C\left(X,K\right)^{\prime}$, and any $U\in\Omega\left(X\right)$, we write: \[ \int_{U}f\left(\mathfrak{z}\right)d\mu\left(\mathfrak{z}\right) \] to denote: \[ \int_{\mathbb{Z}_{p}}\left(f\left(\mathfrak{z}\right)\mathbf{1}_{U}\left(\mathfrak{z}\right)\right)d\mu\left(\mathfrak{z}\right) \] \end{notation} \vphantom{} Like with the real case, there is also a Riemann-sum\index{Riemann sum}-type formulation of the integrability of a function with respect to a given measure \cite{Ultrametric Calculus}: \begin{prop}[\textbf{$\left(p,q\right)$-adic integration via Riemann sums}] Let $d\mu\in C\left(\mathbb{Z}_{p},\mathbb{C}_{q}\right)^{\prime}$, and let $f\in C\left(\mathbb{Z}_{p},\mathbb{C}_{q}\right)$. Then, for any $\epsilon>0$, there exists a $\delta>0$, such that for each partition of $\mathbb{Z}_{p}$ into pair-wise disjoint balls $B_{1},\ldots,B_{N}\in\Omega\left(\mathbb{Z}_{p}\right)$ , all of radius $<\delta$, and any choice of $\mathfrak{a}_{1}\in B_{1}$, $\mathfrak{a}_{2}\in B_{2}$,$\ldots$: \begin{equation} \left|\int_{\mathbb{Z}_{p}}f\left(\mathfrak{z}\right)d\mu\left(\mathfrak{z}\right)-\sum_{n=1}^{N}f\left(\mathfrak{a}_{n}\right)\mu\left(B_{j}\right)\right|_{q}<\epsilon\label{eq:Riemann sum criterion for (p,q)-adic integrability} \end{equation} \end{prop} \begin{rem} The pair-wise disjoint balls of radius $<\delta$ must be replaced by pair-wise disjoint open sets of diameter $<\delta$ in the case where $\mathbb{Z}_{p}$ is replaced by a compact ultrametric space $X$ which does not possess the Heine-Borel property. If $X$ possesses the Heine-Borel property, then the open balls may be used instead of the more general open sets. \end{rem} \begin{thm}[\textbf{Fubini's Theorem}\index{Fubini's Theorem} \cite{Ultrametric Calculus}] Let $d\mu,d\nu$ be $K$-valued measures on $C\left(X,K\right)$ and $C\left(Y,K\right)$, respectively, where $X$ and $Y$ are compact ultrametric spaces. Then, for any continuous $f:X\times Y\rightarrow K$: \begin{equation} \int_{X}\int_{Y}f\left(x,y\right)d\mu\left(x\right)d\nu\left(y\right)=\int_{X}\int_{Y}f\left(x,y\right)d\nu\left(y\right)d\mu\left(x\right)\label{eq:Fubini's Theorem} \end{equation} \end{thm} \begin{example}[\textbf{A $\left(p,p\right)$-adic measure of great importance}\footnote{Given as part of an exercise on page 278 in the Appendix of \cite{Ultrametric Calculus}.}] Let $\mathbb{C}_{p}^{+}$ denote the set: \begin{equation} \mathbb{C}_{p}^{+}\overset{\textrm{def}}{=}\left\{ \mathfrak{s}\in\mathbb{C}_{p}:\left|1-\mathfrak{s}\right|_{p}<1\right\} \end{equation} Schikhof calls such $\mathfrak{s}$ ``positive'' \cite{Ultrametric Calculus}, hence the superscript $+$. For any $\mathfrak{s}\in\mathbb{C}_{p}\backslash\mathbb{C}_{p}^{+}$, define $d\mu_{\mathfrak{s}}\in M\left(\mathbb{Z}_{p},\mathbb{C}_{p}\right)$ by: \begin{equation} \int_{\mathbb{Z}_{p}}\left[\mathfrak{z}\overset{p^{n}}{\equiv}k\right]d\mu_{\mathfrak{s}}\left(\mathfrak{z}\right)\overset{\mathbb{C}_{p}}{=}\frac{\mathfrak{s}^{k}}{1-\mathfrak{s}^{p^{n}}},\textrm{ }\forall n\in\mathbb{N}_{0},\textrm{ }\forall k\in\mathbb{Z}/p^{n}\mathbb{Z} \end{equation} This measure satisfies $\left\Vert \mu_{\mathfrak{s}}\right\Vert <1$, and is used by Koblitz (among others) \cite{Koblitz's other book} to define the \textbf{$p$-adic zeta function}\index{$p$-adic!Zeta function}. A modification of this approach is one of the methods used to construct more general $p$-adic $L$-functions\footnote{One of the major accomplishments of $p$-adic analysis ($\left(p,p\right)$, not $\left(p,q\right)$) was to confirm that the measure-based approach to constructing $p$-adic $L$-functions is, in fact, the same as the $p$-adic $L$-functions produced by interpolating the likes of Kummer's famous congruences for the Bernoulli numbers \cite{p-adic L-functions paper}.}\index{$p$-adic!$L$-function}\cite{p-adic L-functions paper}. Note that because $d\mu_{\mathfrak{s}}$ is a $\left(p,p\right)$-adic measure, it has no hope of being translation invariant, seeing as the only translation-invariant $\left(p,p\right)$-adic linear functional is the zero map. \end{example} \vphantom{} It is at this point, however, where we begin to step away from Subsections \ref{subsec:3.1.4. The--adic-Fourier} and \ref{subsec:3.1.5-adic-Integration-=00003D000026} and make our way toward abstraction. The standard construction of a measure-theoretic integral is to first define it for the indicator function of a measurable set, and then for finite linear combinations thereof, and finally taking $\sup$s for the general case. Unfortunately, this does not work well, both due to the shortage of measurable sets, and\textemdash more gallingly\textemdash due to a terrible betrayal by one of analysts' closest companions: the triangle inequality for integrals. \begin{example}[\textbf{Failure of the triangle inequality for $\left(p,q\right)$-adic integrals}] \textbf{\label{exa:triangle inequality failure}}\index{triangle inequality!failure of}Letting $d\mathfrak{z}$ denote the $\left(p,q\right)$-adic Haar measure (which sends the indicator function $\left[\mathfrak{z}\overset{p^{n}}{\equiv}j\right]$ to the scalar $\frac{1}{p^{n}}\in\mathbb{Q}_{q}$), note that for any $n\geq1$, any distinct $j,k\in\left\{ 0,\ldots,p^{n}-1\right\} $, and any $\mathfrak{a},\mathfrak{b}\in\mathbb{Q}_{q}$: \begin{equation} \left|\mathfrak{a}\left[\mathfrak{z}\overset{p^{n}}{\equiv}j\right]+\mathfrak{b}\left[\mathfrak{z}\overset{p^{n}}{\equiv}k\right]\right|_{q}\overset{\mathbb{R}}{=}\left|\mathfrak{a}\right|_{q}\left[\mathfrak{z}\overset{p^{n}}{\equiv}j\right]+\left|\mathfrak{b}\right|_{q}\left[\mathfrak{z}\overset{p^{n}}{\equiv}k\right],\textrm{ }\forall\mathfrak{z}\in\mathbb{Z}_{p} \end{equation} because, for any $\mathfrak{z}$, at most one of $\left[\mathfrak{z}\overset{p^{n}}{\equiv}j\right]$ and $\left[\mathfrak{z}\overset{p^{n}}{\equiv}k\right]$ can be equal to $1$. So, letting: \begin{equation} f\left(\mathfrak{z}\right)=\mathfrak{a}\left[\mathfrak{z}\overset{p^{n}}{\equiv}j\right]+\mathfrak{b}\left[\mathfrak{z}\overset{p^{n}}{\equiv}k\right] \end{equation} we have that: \begin{equation} \left|\int_{\mathbb{Z}_{p}}f\left(\mathfrak{z}\right)d\mathfrak{z}\right|_{q}\overset{\mathbb{R}}{=}\left|\frac{\mathfrak{a}+\mathfrak{b}}{p^{n}}\right|_{q} \end{equation} and: \begin{equation} \int_{\mathbb{Z}_{p}}\left|f\left(\mathfrak{z}\right)\right|_{q}d\mathfrak{z}\overset{\mathbb{R}}{=}\int_{\mathbb{Z}_{p}}\left(\left|\mathfrak{a}\right|_{q}\left[\mathfrak{z}\overset{p^{n}}{\equiv}j\right]+\left|\mathfrak{b}\right|_{q}\left[\mathfrak{z}\overset{p^{n}}{\equiv}k\right]\right)d\mathfrak{z}=\frac{\left|\mathfrak{a}\right|_{q}+\left|\mathfrak{b}\right|_{q}}{p^{n}} \end{equation} where the integral in the middle on the second line is with respect to the standard \emph{real-valued} Haar probability measure on $\mathbb{Z}_{p}$. If we choose $\mathfrak{a}=1$ and $\mathfrak{b}=q^{m}-1$, where $m\in\mathbb{N}_{1}$, then: \begin{equation} \left|\int_{\mathbb{Z}_{p}}f\left(\mathfrak{z}\right)d\mathfrak{z}\right|_{q}\overset{\mathbb{R}}{=}\left|\frac{q^{m}}{p^{n}}\right|_{q}=\frac{1}{q^{m}} \end{equation} \begin{equation} \int_{\mathbb{Z}_{p}}\left|f\left(\mathfrak{z}\right)\right|_{q}d\mathfrak{z}\overset{\mathbb{R}}{=}\frac{\left|1\right|_{q}+\left|q^{m}-1\right|_{q}}{p^{n}}=\frac{2}{p^{n}} \end{equation} Because of this, the symbol $\sim$ in the expression: \begin{equation} \left|\int_{\mathbb{Z}_{p}}f\left(\mathfrak{z}\right)d\mathfrak{z}\right|_{q}\sim\int_{\mathbb{Z}_{p}}\left|f\left(\mathfrak{z}\right)\right|_{q}d\mathfrak{z} \end{equation} can be arranged to be any of the symbols $\leq$, $\geq$, $<$, $>$, or $=$ by making an appropriate choice of the values of $m$, $n$, $p$, and $q$. A very distressing outcome, indeed. \end{example} \vphantom{} The \emph{point} of Monna-Springer theory, one could argue, is to find a work-around for this depressing state of affairs. That work-around takes the form of an observation. What we would like to do is to write: \begin{equation} \left|\int f\right|\leq\int\left|f\right| \end{equation} However, we can't. Instead, we take out the middle man, which, in this case, is the absolute value on the left-hand side. Rather than view the triangle inequality as an operation we apply to an integral already in our possession, we view it as an operation we apply directly to the function $f$ we intend to integrate: \begin{equation} f\mapsto\int\left|f\right| \end{equation} where, for the sake of discussion, we view everything as happening on $\mathbb{R}$; $f:\mathbb{R}\rightarrow\mathbb{R}$, integration is with respect to the Lebesgue measure, etc. Because the vanishing of the integral only forces $f$ to be $0$ \emph{almost }everywhere, the above map is not a true norm, but a semi-norm. Classically, remember, $L^{1}$-norm only becomes a norm after we mod out by functions which are zero almost everywhere. More generally, given\textemdash say\textemdash a Radon measure $d\mu$, the map $f\mapsto\int\left|f\right|d\mu$ defines a semi-norm which we can use to compute the $\mu$-measure of a set $E$ by applying it to continuous functions which approximate $E$'s indicator function. While we will not be able to realize this semi-norm as the integral of $\left|f\right|$ $d\mu$ in the non-archimedean case, we can still obtain a semi-norm by working from the ground up. Given a measure $\mu\in C\left(X,K\right)^{\prime}$, we consider the semi-norm: \begin{equation} f\in C\left(X,Y\right)\mapsto\sup_{g\in C\left(X,K\right)\backslash\left\{ 0\right\} }\frac{1}{\left\Vert g\right\Vert _{X,K}}\left|\int_{X}f\left(\mathfrak{z}\right)g\left(\mathfrak{z}\right)d\mu\left(\mathfrak{z}\right)\right|_{K}\in\mathbb{R} \end{equation} In the appendices of \cite{Ultrametric Calculus}, Schikhof takes this formula as the definition for the semi-norm $N_{\mu}$ induced by the measure $\mu$, which he then defines as a function on $\Omega\left(X\right)$ by defining $N_{\mu}\left(U\right)$ to be $N_{\mu}\left(\mathbf{1}_{U}\right)$, where $\mathbf{1}_{U}$ is the indicator function of $U$. Taking the infimum over all $U\in\Omega\left(X\right)$ containing a given point $\mathfrak{z}$ then allows for $N_{\mu}$ to be extended to a real-valued function on $X$ by way of the formula: \begin{equation} \mathfrak{z}\in X\mapsto\inf_{U\in\Omega\left(X\right):\mathfrak{z}\in U}\sup_{g\in C\left(X,K\right)\backslash\left\{ 0\right\} }\frac{\left|\int_{X}\mathbf{1}_{U}\left(\mathfrak{y}\right)g\left(\mathfrak{y}\right)d\mu\left(\mathfrak{y}\right)\right|_{K}}{\left\Vert g\right\Vert _{X,K}}\in\mathbb{R} \end{equation} While this approach is arguably somewhat more intuitive, the above definitions are not very pleasing to look at, and using them is a hassle. Instead, I follow Khrennikov et. al. \cite{Measure-theoretic approach to p-adic probability theory} in giving the following formulas as the definition of $N_{\mu}$: \begin{defn} \label{def:Khrennikov N_mu definition}Let\footnote{Taken from \cite{Measure-theoretic approach to p-adic probability theory}.} $d\mu\in C\left(X,K\right)^{\prime}$. \vphantom{} I. For any $U\in\Omega\left(X\right)$, we define $N_{\mu}\left(U\right)$ by: \begin{equation} N_{\mu}\left(U\right)=\sup_{V\in\Omega\left(X\right):V\subset U}\left|\mu\left(V\right)\right|_{K},\textrm{ }\forall U\in\Omega\left(X\right)\label{eq:mu seminorm of a compact clopen set} \end{equation} \vphantom{} II. For any $\mathfrak{z}\in X$, we define $N_{\mu}\left(\mathfrak{z}\right)$ by: \begin{equation} N_{\mu}\left(\mathfrak{z}\right)\overset{\textrm{def}}{=}\inf_{U\in\Omega\left(X\right):\mathfrak{z}\in U}N_{\mu}\left(\mathbf{1}_{U}\right)\label{eq:Definition of N_mu at a point} \end{equation} \vphantom{} III. Given any $f:X\rightarrow K$ (not necessarily continuous), we define \begin{equation} N_{\mu}\left(f\right)\overset{\textrm{def}}{=}\sup_{\mathfrak{z}\in X}N_{\mu}\left(\mathfrak{z}\right)\left|f\left(\mathfrak{z}\right)\right|_{K}\label{eq:Definition of N_mu for an arbitrary f} \end{equation} \end{defn} \vphantom{} In \cite{Ultrametric Calculus}, Schikhof proves that the formulas of \textbf{Definition \ref{def:Khrennikov N_mu definition} }as a theorem. Instead, following Khrennikov et. al. in \cite{Measure-theoretic approach to p-adic probability theory}, I state Schikhof's definitions of $N_{\mu}$ as a theorem (\textbf{Theorem \ref{thm:Schikhof's N_mu definition}}), deducible from \textbf{Definition \ref{def:Khrennikov N_mu definition}}. \begin{thm} \label{thm:Schikhof's N_mu definition}\ \vphantom{} I. For all $f\in C\left(X,K\right)$: \begin{equation} N_{\mu}\left(f\right)=\sup_{g\in C\left(X,K\right)\backslash\left\{ 0\right\} }\frac{1}{\left\Vert g\right\Vert _{X,K}}\left|\int_{X}f\left(\mathfrak{z}\right)g\left(\mathfrak{z}\right)d\mu\left(\mathfrak{z}\right)\right|_{K}\label{eq:Schikhof's definition of N_mu} \end{equation} \vphantom{} II. \begin{equation} N_{\mu}\left(\mathfrak{z}\right)=\inf_{U\in\Omega\left(X\right):\mathfrak{z}\in U}\sup_{g\in C\left(X,K\right)\backslash\left\{ 0\right\} }\frac{\left|\int_{U}g\left(\mathfrak{y}\right)d\mu\left(\mathfrak{y}\right)\right|_{K}}{\left\Vert g\right\Vert _{X,K}} \end{equation} \end{thm} \begin{rem} The notation Schikhof uses in \cite{Ultrametric Calculus} for the semi-norms is slightly different than the one given here, which I have adopted from Khrennikov's exposition of the Monna-Springer integral in \cite{Quantum Paradoxes}, which is simpler and more consistent. \end{rem} \vphantom{} In order to be able to extend the notion of $\mu$-integrable functions from $C\left(X,K\right)$ to a large space, we need a non-archimedean version of the triangle inequality for integrals. As we saw above, however, this doesn't really exist in the non-archimedean context. This is where $N_{\mu}$ comes in; it is our replacement for what where $\int\left|f\right|d\mu$ would be in the archimedean case. \begin{prop}[\textbf{Monna-Springer Triangle Inequality} \cite{Ultrametric Calculus,Quantum Paradoxes}] Let $X$ be a compact ultrametric space, and let $d\mu\in C\left(X,K\right)^{\prime}$. Then: \begin{equation} \left|\int_{X}f\left(\mathfrak{z}\right)d\mu\left(\mathfrak{z}\right)\right|_{K}\leq N_{\mu}\left(f\right)\leq\left\Vert \mu\right\Vert \left\Vert f\right\Vert _{X,K}\label{eq:Non-archimedean triangle inequality} \end{equation} \end{prop} \begin{rem} Unfortunately it is often the case\textemdash especially so for $\left(p,q\right)$-adic analysis\textemdash that this estimate is little better than bounding an integral by the product of the $\sup$ of the integrand and the measure of the domain of integration. \end{rem} \vphantom{} Having done all this, the idea is to use $N_{\mu}$ to define what it means for an arbitrary function $f:X\rightarrow K$ to be $\mu$-integrable. This is done in the way one would expect: identifying $f,g\in C\left(X,K\right)$ whenever $N_{\mu}\left(f-g\right)=0$, and then modding $C\left(X,K\right)$ out by this equivalence relation, so as to upgrade $N_{\mu}$ from a semi-norm to a fully-fledged norm. We then take the completion of $C\left(X,K\right)$ with respect to this norm to obtain our space of $\mu$-integrable functions. \cite{Quantum Paradoxes,Measure-theoretic approach to p-adic probability theory} and \cite{Ultrametric Calculus} give us the definitions that will guide us through this process. \begin{defn} \ \vphantom{} I. Because the indicator function for any set $S\subseteq X$ is then a function $X\rightarrow K$, we can use (\ref{eq:Definition of N_mu for an arbitrary f}) to define $N_{\mu}$ for an arbitrary $S\subseteq X$: \begin{equation} N_{\mu}\left(S\right)\overset{\textrm{def}}{=}\inf_{U\in\Omega\left(X\right):S\subseteq U}N_{\mu}\left(U\right)=\inf_{U\in\Omega\left(X\right):S\subseteq U}\sup_{V\in\Omega\left(X\right):V\subset U}\left|\mu\left(V\right)\right|_{K}\label{eq:Definition of N_mu of an arbitrary set} \end{equation} \vphantom{} II.\textbf{ }A function $f:X\rightarrow K$ (not necessarily continuous) is said to be \textbf{$\mu$-integrable }whenever there is a sequence $\left\{ f_{n}\right\} _{n\geq1}$ in $C\left(X,K\right)$ so that $\lim_{n\rightarrow\infty}N_{\mu}\left(f-f_{n}\right)=0$, where we compute $N_{\mu}\left(f-f_{n}\right)$ using (\ref{eq:Definition of N_mu for an arbitrary f}). We then write $\mathcal{L}_{\mu}^{1}\left(X,K\right)$ to denote the vector space of all $\mu$-integrable functions from $X$ to $K$. \vphantom{} III. We say a set $S\subseteq X$ is \textbf{$\mu$-measurable }whenever $N_{\mu}\left(S\right)$ is finite. Equivalently, $S$ is $\mu$-measurable whenever there is a sequence $\left\{ f_{n}\right\} _{n\geq1}$ in $C\left(X,K\right)$ so that $\lim_{n\rightarrow\infty}N_{\mu}\left(\mathbf{1}_{S}-f_{n}\right)=0$. We write $\mathcal{R}_{\mu}$ to denote the collection of all $\mu$-measurable subsets of $X$. \vphantom{} IV. A \textbf{non-archimedean measure space}\index{non-archimedean!measure space}\textbf{ (over $K$) }is a triple $\left(X,d\mu,\mathcal{R}_{\mu}\right)$, where $X$ is an ultrametric space, $d\mu\in C\left(X,K\right)^{\prime}$, and $\mathcal{R}_{\mu}$ is the collection of all $\mu$-measurable subsets of $X$. \vphantom{} V. Given two non-archimedean measure spaces $\left(X_{1},d\mu_{1},\mathcal{R}_{\mu_{1}}\right)$ and $\left(X_{2},d\mu_{2},\mathcal{R}_{\mu_{2}}\right)$ with we say a function $\phi:X_{1}\rightarrow X_{2}$ is \textbf{$\mu$-measurable} whenever $\phi^{-1}\left(V\right)\in\mathcal{R}_{\mu_{1}}$ for all $V\in\mathcal{R}_{\mu_{2}}$. \vphantom{} VI. We define $E_{\mu}$ as the set: \begin{equation} E_{\mu}\overset{\textrm{def}}{=}\left\{ f:X\rightarrow K\textrm{ \& }N_{\mu}\left(f\right)<\infty\right\} \label{eq:Definition of E_mu} \end{equation} Given any $f\in E_{\mu}$, we say $f$ is \textbf{$\mu$-negligible }whenever $N_{\mu}\left(f\right)=0$. Likewise, given any set $S\subseteq X$, we say $S$ is \textbf{$\mu$-negligible} whenever $N_{\mu}\left(S\right)=0$. \vphantom{} VII. Given any real number $\alpha>0$, we define: \begin{equation} X_{\mu:\alpha}\overset{\textrm{def}}{=}\left\{ \mathfrak{z}\in X:N_{\mu}\left(\mathfrak{z}\right)\geq\alpha\right\} \label{eq:Definition of X mu alpha} \end{equation} \begin{equation} X_{\mu:+}\overset{\textrm{def}}{=}\bigcup_{\alpha>0}X_{\mu:\alpha}\label{eq:Definition of X mu plus} \end{equation} \begin{equation} X_{\mu:0}\overset{\textrm{def}}{=}\left\{ \mathfrak{z}\in X:N_{\mu}\left(\mathfrak{z}\right)=0\right\} \label{eq:Definition of X mu 0} \end{equation} Note that $X=X_{\mu:0}\cup X_{\mu:+}$. We omit $\mu$ and just write $X_{\alpha}$, $X_{+}$, and $X_{0}$ whenever there is no confusion as to the measure $\mu$ we happen to be using. \vphantom{} VIII. We write $L_{\mu}^{1}\left(X,K\right)$ to denote the space of equivalence classes of $\mathcal{L}_{\mu}^{1}\left(X,K\right)$ under the relation: \[ f\sim g\Leftrightarrow f-g\textrm{ is }\mu\textrm{-negligible} \] \end{defn} \vphantom{} Using these definitions, we can finally construct an honest-to-goodness space of integrable non-archimedean functions \cite{Ultrametric Calculus}. \begin{thm} Let $d\mu\in C\left(X,K\right)^{\prime}$. Then: \vphantom{} I. $\mathcal{L}_{\mu}^{1}\left(X,K\right)$ contains $C\left(X,K\right)$, as well as all $\mu$-negligible functions from $X$ to $K$. \vphantom{} II. $d\mu$ can be extended from $C\left(X,K\right)$ to $\mathcal{L}_{\mu}^{1}\left(X,K\right)$; denote this extension by $\overline{d\mu}$. Then: \[ \left|\int_{X}f\left(\mathfrak{z}\right)\overline{d\mu}\left(\mathfrak{z}\right)\right|_{K}\leq N_{\mu}\left(f\right),\textrm{ }\forall f\in\mathcal{L}_{\mu}^{1}\left(X,K\right) \] \vphantom{} III. $L_{\mu}^{1}\left(X,K\right)$ is a Banach space over $K$ with respect to the norm induced by $N_{\mu}$. \end{thm} \vphantom{} In $\left(p,q\right)$-adic case, however, these constructions turn out to be overkill. \begin{thm} Let $d\mu$ be the $\left(p,q\right)$-adic Haar probability measure $d\mathfrak{z}$. Then $N_{\mu}\left(\mathfrak{z}\right)=1$ for all $\mathfrak{z}\in\mathbb{Z}_{p}$. Equivalently, for $X=\mathbb{Z}_{p}$, we have\footnote{This is given as an exercise on Page 281 of \cite{Ultrametric Calculus}.}: \begin{align*} X_{0} & =\varnothing\\ X_{+} & =X\\ X_{\alpha} & =\begin{cases} \varnothing & \textrm{if }0<\alpha<1\\ X & \textrm{if }\alpha=1\\ \varnothing & \alpha>1 \end{cases} \end{align*} \end{thm} \vphantom{} Put in words, the only set which is negligible with respect to $d\mathfrak{z}$ is the \emph{empty set!} This is the reason why integrability and continuity are synonymous in $\left(p,q\right)$-adic analysis. \begin{cor}[\textbf{The Fundamental Theorem of $\left(p,q\right)$-adic Analysis}\footnote{The name is my own.}] Letting $d\mathfrak{z}$ denote the $\left(p,q\right)$-adic Haar measure, we have the equality (not just an isomorphism, but an\emph{ }\textbf{equality}!): \begin{equation} L_{d\mathfrak{z}}^{1}\left(\mathbb{Z}_{p},\mathbb{C}_{q}\right)=C\left(\mathbb{Z}_{p},\mathbb{C}_{q}\right)\label{eq:in (p,q)-adic analysis, integrable equals continuous} \end{equation} Consequently: \begin{equation} \left|\int_{\mathbb{Z}_{p}}f\left(\mathfrak{z}\right)d\mathfrak{z}\right|_{q}\leq\left\Vert f\right\Vert _{p,q},\textrm{ }\forall f\in C\left(\mathbb{Z}_{p},\mathbb{C}_{q}\right)\label{eq:(p,q)-adic triangle inequality} \end{equation} \end{cor} Proof: Since $N_{d\mathfrak{z}}=1$, $N_{d\mathfrak{z}}$ is then the $\infty$-norm on $C\left(\mathbb{Z}_{p},\mathbb{C}_{q}\right)$. Hence: \[ \mathcal{L}_{d\mathfrak{z}}^{1}\left(\mathbb{Z}_{p},\mathbb{C}_{q}\right)=L_{d\mathfrak{z}}^{1}\left(\mathbb{Z}_{p},\mathbb{C}_{q}\right)=C\left(\mathbb{Z}_{p},\mathbb{C}_{q}\right) \] Q.E.D. \vphantom{} With a little more work, one can prove many of the basic results familiar to us from classical analysis. While these hold for Monna-Springer theory in general, the simplicity of the $\left(p,q\right)$-adic case makes these theorems significantly less useful than their classical counterparts. I include them primarily for completeness' sake. \begin{thm}[\textbf{Non-Archimedean Dominated Convergence Theorem}\footnote{Given in Exercise 7F on page 263 of \cite{van Rooij - Non-Archmedean Functional Analysis}.}] Let\index{Dominated Convergence Theorem} $X$ be a locally compact ultrametric space. Let $g\in L_{\mu}^{1}\left(X,K\right)$ and let $\left\{ f_{n}\right\} _{n\geq0}$ be a sequence in $L_{\mu}^{1}\left(X,K\right)$ such that: I. For every set $E\in\mathcal{R}_{\mu}$, $f_{n}$ converges uniformly to $f$ on $E$; II. $\left|f_{n}\left(\mathfrak{z}\right)\right|_{K}\leq\left|g\left(\mathfrak{z}\right)\right|_{K}$ for all $\mathfrak{z}\in X$ and all $n$. Then, $f\in L_{\mu}^{1}\left(X,K\right)$, $\lim_{n\rightarrow\infty}N_{\mu}\left(f_{n}-f\right)=0$ and: \begin{equation} \lim_{n\rightarrow\infty}\int_{X}f_{n}\left(\mathfrak{z}\right)d\mu\left(\mathfrak{z}\right)\overset{K}{=}\int_{X}\lim_{n\rightarrow\infty}f_{n}\left(\mathfrak{z}\right)d\mu\left(\mathfrak{z}\right)\overset{K}{=}\int_{X}f\left(\mathfrak{z}\right)d\mu\left(\mathfrak{z}\right)\label{eq:Non-Archimedean Dominated Convergence Theorem} \end{equation} \end{thm} \begin{rem} For $\left(p,q\right)$-analysis, the Dominated Convergence Theorem is simply the statement that: \begin{equation} \lim_{n\rightarrow\infty}\int f_{n}=\int\lim_{n\rightarrow\infty}f_{n} \end{equation} occurs if and only if $f_{n}\rightarrow f$ uniformly on $\mathbb{Z}_{p}$: \begin{equation} \lim_{n\rightarrow\infty}\sup_{\mathfrak{z}\in\mathbb{Z}_{p}}\left|f\left(\mathfrak{z}\right)-f_{n}\left(\mathfrak{z}\right)\right|_{q}=0 \end{equation} \end{rem} \begin{thm}[\textbf{Non-Archimedean Hlder's Inequality} \cite{Quantum Paradoxes}] Let $f,g\in C\left(X,K\right)$, and let $d\mu\in C\left(X,K\right)^{\prime}$ Then: \begin{equation} N_{\mu}\left(fg\right)\leq N_{\mu}\left(f\right)\left\Vert g\right\Vert _{X,K}\label{eq:(p,q)-adic Holder inequality} \end{equation} \end{thm} \begin{rem} For\index{Hlder's Inequality} $\left(p,q\right)$-analysis, Hlder's Inequality is extremely crude, being the statement that $\left|\int fg\right|_{q}\leq\left\Vert f\right\Vert _{X,K}\left\Vert g\right\Vert _{X,K}$. \end{rem} \vphantom{} Finally\textemdash although we shall not use it beyond the affine case detailed in \textbf{Lemma \ref{lem:Affine substitution change of variable formula}}, we also have a general change of variables formula. First, however, the necessary definitional lead-up. \begin{defn} \label{def:pullback measure}Let $\left(X,d\mu,\mathcal{R}_{\mu}\right)$ and $\left(Y,d\nu,\mathcal{R}_{\nu}\right)$ be non-archimedean measure spaces over $K$. For any measurable $\phi:X\rightarrow Y$, we define the measure $d\mu_{\phi}:\mathcal{R}_{\nu}\rightarrow K$ by: \begin{equation} \mu_{\phi}\left(V\right)\overset{\textrm{def}}{=}\mu\left(\phi^{-1}\left(V\right)\right),\textrm{ }\forall V\in\mathcal{R}_{\nu}\label{eq:Definition of change-of-variables measure} \end{equation} \end{defn} \begin{lem} Let $\left(X,d\mu,\mathcal{R}_{\mu}\right)$ and $\left(Y,d\nu,\mathcal{R}_{\nu}\right)$ be non-archimedean measure spaces over $K$. For any measurable $\phi:X\rightarrow Y$, and for every $\mathcal{R}_{\nu}$ continuous $f:Y\rightarrow K$: \begin{equation} N_{\mu_{\phi}}\left(f\right)\leq N_{\mu}\left(f\circ\phi\right) \end{equation} \end{lem} \begin{thm}[\textbf{Change of Variables \textendash{} Monna-Springer Integration }\cite{Measure-theoretic approach to p-adic probability theory}] Let\index{change of variables} $\left(X,d\mu,\mathcal{R}_{\mu}\right)$ and $\left(Y,d\nu,\mathcal{R}_{\nu}\right)$ be non-archimedean measure spaces over $K$. For any measurable $\phi:X\rightarrow Y$, and for every $\mathcal{R}_{\nu}$ continuous\footnote{As remarked in \cite{Measure-theoretic approach to p-adic probability theory}, it would be desirable if the hypothesis of continuity could be weakened.} $f:Y\rightarrow K$ so that $f\circ\phi\in L_{\mu}^{1}\left(X,K\right)$, the function $f$ is in $L_{\nu}^{1}\left(Y,K\right)$ and: \begin{equation} \int_{X}f\left(\phi\left(\mathfrak{z}\right)\right)d\mu\left(\mathfrak{z}\right)=\int_{Y}f\left(\mathfrak{y}\right)d\mu_{\phi}\left(y\right)\label{eq:Monna-Springer Integral Change-of-Variables Formula} \end{equation} \end{thm} \newpage{} \section{\label{sec:3.2 Rising-Continuous-Functions}Rising-Continuous Functions} IN THIS SECTION, $p$ A PRIME AND $K$ IS A COMPLETE NON-ARCHIMEDEAN VALUED FIELD. \vphantom{} Given the somewhat unusual method we used to construct $\chi_{H}$\textemdash strings and all\textemdash you would think that the $\chi_{H}$s would be unusual functions, located out in the left field of ``mainstream'' $\left(p,q\right)$-adic function theory. Surprisingly, this is not the case. The key property here is the limit: \begin{equation} \chi_{H}\left(\mathfrak{z}\right)\overset{\mathbb{Z}_{q_{H}}}{=}\lim_{n\rightarrow\infty}\chi_{H}\left(\left[\mathfrak{z}\right]_{p^{n}}\right) \end{equation} \textbf{Rising-continuous functions} are precisely those functions which satisfy this limit. In this section, we shall get to know these functions as a whole and see how they are easily obtained from the smaller class of continuous functions. \subsection{\label{subsec:3.2.1 -adic-Interpolation-of}$\left(p,q\right)$-adic Interpolation of Functions on $\mathbb{N}_{0}$} It is not an understatement to say that the entire theory of rising-continuous functions emerges (or, should I say, ``\emph{rises}''?) out of the van der Put identity ((\ref{eq:van der Put identity}) from \textbf{Proposition \ref{prop:vdP identity} }on page \pageref{prop:vdP identity}): \begin{equation} \sum_{n=0}^{\infty}c_{n}\left(\chi\right)\left[\mathfrak{z}\overset{p^{\lambda_{p}\left(n\right)}}{\equiv}n\right]\overset{K}{=}\lim_{k\rightarrow\infty}\chi\left(\left[\mathfrak{z}\right]_{p^{k}}\right),\textrm{ }\forall\mathfrak{z}\in\mathbb{Z}_{p} \end{equation} We make two observations: \begin{enumerate} \item The field $K$ can be \emph{any }metrically complete valued field\textemdash archimedean or non-archimedean. \item $\chi$ can be\emph{ any }function $\chi:\mathbb{Z}_{p}\rightarrow K$. \end{enumerate} (1) will be arguably even more important than (2), because it highlights what will quickly become a cornerstone of our approach: the ability to allow the field (and hence, topology)\emph{ }used to sum our series \emph{vary depending on $\mathfrak{z}$}. First, however, some terminology. \begin{defn} Let $p$ be an integer $\geq2$. \vphantom{} I. Recall, we write \nomenclature{$\mathbb{Z}_{p}^{\prime}$}{$\overset{\textrm{def}}{=}\mathbb{Z}_{p}\backslash\left\{ 0,1,2,3,\ldots\right\}$ \nopageref}$\mathbb{Z}_{p}^{\prime}$ to denote the set: \begin{equation} \mathbb{Z}_{p}^{\prime}\overset{\textrm{def}}{=}\mathbb{Z}_{p}\backslash\mathbb{N}_{0}\label{eq:Definition of Z_p prime} \end{equation} \vphantom{} II. A sequence $\left\{ \mathfrak{z}_{n}\right\} _{n\geq0}$ in $\mathbb{Z}_{p}$ is said to be \textbf{($p$-adically) rising }if, as $n\rightarrow\infty$, the number of non-zero $p$-adic digits in $\mathfrak{z}_{n}$ tends to $\infty$. \end{defn} \begin{rem} $\mathbb{Z}_{p}^{\prime}$ is neither open nor closed, and has an empty interior. \end{rem} \begin{rem} Note that for any rising $\left\{ \mathfrak{z}_{n}\right\} _{n\geq0}\subseteq\mathbb{Z}_{p}$, the number of non-zero $p$-adic digits in $\mathfrak{z}_{n}$s tends to $\infty$ as $n\rightarrow\infty$; that is, $\lim_{n\rightarrow\infty}\sum_{j=1}^{p-1}\#_{p:j}\left(\mathfrak{z}_{n}\right)\overset{\mathbb{R}}{=}\infty$. However, the converse of this is not true: there are sequences $\left\{ \mathfrak{z}_{n}\right\} _{n\geq0}\subseteq\mathbb{Z}_{p}$ for which $\lim_{n\rightarrow\infty}\sum_{j=1}^{p-1}\#_{p:j}\left(\mathfrak{z}_{n}\right)\overset{\mathbb{R}}{=}\infty$ but $\lim_{n\rightarrow\infty}\mathfrak{z}_{n}\in\mathbb{N}_{0}$. \end{rem} \begin{example} For any $p$, let $\mathfrak{z}_{n}$'s sequence of $p$-adic digits be $n$ consecutive $0$s followed by infinitely many $1$s; then each $\mathfrak{z}_{n}$ has infinitely many non-zero $p$-adic digits, but the $\mathfrak{z}_{n}$s converge $p$-adically to $0$. \end{example} \vphantom{} In ``classical'' non-archimedean analysis, for a function $\chi$, $\chi$'s continuity is equivalent to the $K$-adic decay of its van der Put coefficients to zero. Rising-continuous functions, in contrast, arise when we relax this requirement of decay. Because we still need for there to be enough regularity to the van der Put coefficients of $\chi$ in order for its van der Put series to converge point-wise, it turns out the ``correct'' definition for rising-continuous functions is the limit condition we established for $\chi_{H}$ in our proof of \textbf{Lemma \ref{lem:Unique rising continuation and p-adic functional equation of Chi_H}} (page \pageref{lem:Unique rising continuation and p-adic functional equation of Chi_H}). \begin{defn} Let $K$ be a metrically complete non-archimedean valued field. A function $\chi:\mathbb{Z}_{p}\rightarrow K$ is said to be \textbf{($\left(p,K\right)$-adically)} \textbf{rising-continuous}\index{rising-continuous!function}\index{rising-continuous!left(p,Kright)-adically@$\left(p,K\right)$-adically}\textbf{ }whenever: \begin{equation} \lim_{n\rightarrow\infty}\chi\left(\left[\mathfrak{z}\right]_{p^{n}}\right)\overset{K}{=}\chi\left(\mathfrak{z}\right),\textrm{ }\forall\mathfrak{z}\in\mathbb{Z}_{p}\label{eq:Definition of a rising-continuous function} \end{equation} We write $\tilde{C}\left(\mathbb{Z}_{p},K\right)$ to denote the $K$-linear space\footnote{Note that this is a subspace of $B\left(\mathbb{Z}_{p},K\right)$.} of all rising-continuous functions $\chi:\mathbb{Z}_{p}\rightarrow K$. We call elements of \nomenclature{$\tilde{C}\left(\mathbb{Z}_{p},K\right)$}{set of $K$-valued rising-continuous functions on $\mathbb{Z}_{p}$}$\tilde{C}\left(\mathbb{Z}_{p},K\right)$ \textbf{rising-continuous functions}; or, more pedantically, \textbf{$\left(p,K\right)$-adic rising-continuous functions}, or $\left(p,q\right)$\textbf{-adic rising-continuous functions}, when $K$ is a $q$-adic field. \end{defn} \begin{rem} The convergence in (\ref{eq:Definition of a rising-continuous function}) only needs to occur \emph{point-wise}. \end{rem} \begin{example} Every continuous $f:\mathbb{Z}_{p}\rightarrow K$ is also rising-continuous, but not every rising-continuous function is continuous. As an example, consider: \begin{equation} f\left(\mathfrak{z}\right)=q^{\textrm{digits}_{p}\left(\mathfrak{z}\right)}\label{eq:Example of a discontinuous rising-continuous function} \end{equation} where $\textrm{digits}_{p}:\mathbb{Z}_{p}\rightarrow\mathbb{N}_{0}\cup\left\{ +\infty\right\} $ outputs the number of non-zero $p$-adic digits in $\mathfrak{z}$. In particular, $\textrm{digits}_{p}\left(\mathfrak{z}\right)=0$ if and only if $\mathfrak{z}=0$, $\textrm{digits}_{p}\left(\mathfrak{z}\right)=+\infty$ if and only if $\mathfrak{z}\in\mathbb{Z}_{p}^{\prime}$. As defined, $f\left(\mathfrak{z}\right)$ then satisfies: \begin{equation} f\left(\mathfrak{z}\right)\overset{K}{=}\begin{cases} q^{\textrm{digits}_{p}\left(\mathfrak{z}\right)} & \textrm{if }\mathfrak{z}\in\mathbb{N}_{0}\\ 0 & \textrm{if }\mathfrak{z}\in\mathbb{Z}_{p}^{\prime} \end{cases} \end{equation} Since $\mathbb{Z}_{p}^{\prime}$ is dense in $\mathbb{Z}_{p}$, if $f$ \emph{were} continuous, the fact that it vanishes on $\mathbb{Z}_{p}$ would force it to be identically zero, which is not the case. Thus, $f$ \emph{cannot} be continuous. \end{example} \begin{example} Somewhat unfortunately, even if $\chi$ is rising continuous, we cannot guarantee that $\chi\left(\mathfrak{z}_{n}\right)$ converges to $\chi\left(\mathfrak{z}\right)$ for every $p$-adically rising sequence $\left\{ \mathfrak{z}_{n}\right\} _{n\geq1}$ with limit $\mathfrak{z}_{n}\rightarrow\mathfrak{z}$. $\chi_{q}$\textemdash the numen of the Shortened $qx+1$ map\textemdash gives us a simple example of this. As we shall soon prove (see \textbf{Theorem \ref{thm:rising-continuability of Generic H-type functional equations}} on page \pageref{thm:rising-continuability of Generic H-type functional equations}), $\chi_{q}:\mathbb{Z}_{2}\rightarrow\mathbb{Z}_{q}$ is rising-continuous. Now, consider the sequence: \begin{equation} \mathfrak{z}_{n}\overset{\textrm{def}}{=}-2^{n}=\centerdot_{2}\underbrace{0\ldots0}_{n}\overline{1}\ldots \end{equation} where $\overline{1}$ indicates that all the remaining $2$-adic digits of $\mathfrak{z}_{n}$ are $1$s; the first $n$ $2$-adic digits of $\mathfrak{z}_{n}$ are $0$. This is a rising sequence, yet it converges $2$-adically to $0$. Moreover, observe that: \begin{equation} \chi_{q}\left(-2^{n}\right)=\frac{1}{2^{n}}\chi_{q}\left(-1\right)=\frac{1}{2^{n}}\chi_{q}\left(B_{2}\left(1\right)\right)=\frac{1}{2^{n}}\frac{\chi_{q}\left(1\right)}{1-M_{q}\left(1\right)}=\frac{1}{2^{n}}\frac{1}{2-q} \end{equation} Thus, $\chi_{q}\left(-2^{n}\right)$ does not even converge $q$-adically to a limit as $n\rightarrow\infty$, even though $\left\{ -2^{n}\right\} _{n\geq1}$ is a $2$-adically rising sequence which converges $2$-adically to $0$. \end{example} \begin{prop} \label{prop:vdP criterion for rising continuity}Let $\chi:\mathbb{Z}_{p}\rightarrow\mathbb{C}_{q}$ be any function. Then, the van der Put series of $\chi$ (that is, $S_{p}\left\{ \chi\right\} $) converges at $\mathfrak{z}\in\mathbb{Z}_{p}$ if and only if: \begin{equation} \lim_{n\rightarrow\infty}c_{\left[\mathfrak{z}\right]_{p^{n}}}\left(\chi\right)\left[\lambda_{p}\left(\left[\mathfrak{z}\right]_{p^{n}}\right)=n\right]\overset{\mathbb{C}_{q}}{=}0\label{eq:vdP criterion for rising-continuity} \end{equation} where $c_{\left[\mathfrak{z}\right]_{p^{n}}}\left(\chi\right)$ is the $\left[\mathfrak{z}\right]_{p^{n}}$th van der Put coefficient of $\chi$. \end{prop} Proof: Using (\ref{eq:Inner term of vdP lambda decomposition}), we can write the van der Put series for $\chi$ as: \begin{equation} S_{p}\left\{ \chi\right\} \left(\mathfrak{z}\right)\overset{\mathbb{C}_{q}}{=}c_{0}\left(\chi\right)+\sum_{n=1}^{\infty}c_{\left[\mathfrak{z}\right]_{p^{n}}}\left(\chi\right)\left[\lambda_{p}\left(\left[\mathfrak{z}\right]_{p^{n}}\right)=n\right],\textrm{ }\forall\mathfrak{z}\in\mathbb{Z}_{p}\label{eq:Lambda-decomposed Chi} \end{equation} The ultrametric properties of $\mathbb{C}_{q}$ tell us that the $q$-adic convergence of this series at any given $\mathfrak{z}\in\mathbb{Z}_{p}$ is equivalent to: \begin{equation} \lim_{k\rightarrow\infty}c_{\left[\mathfrak{z}\right]_{p^{k}}}\left(\chi\right)\left[\lambda_{p}\left(\left[\mathfrak{z}\right]_{p^{k}}\right)=k\right]\overset{\mathbb{C}_{q}}{=}0\label{eq:vdP coefficient decay for lemma} \end{equation} Q.E.D. \begin{thm} \label{thm:S_p}The operator $S_{p}$ which sends a function to its formal van der Put series is a isomorphism from the $K$-linear space $\tilde{C}\left(\mathbb{Z}_{p},K\right)$ onto the subspace of $\textrm{vdP}\left(\mathbb{Z}_{p},K\right)$ consisting of all formal van der Put series\index{van der Put!series} which converge at every $\mathfrak{z}\in\mathbb{Z}_{p}$. In particular, we have that for every $\chi\in\tilde{C}\left(\mathbb{Z}_{p},K\right)$: \vphantom{} I. $\chi=S_{p}\left\{ \chi\right\} $; \vphantom{} II. $\chi$ is uniquely represented by its van der Put series: \begin{equation} \chi\left(\mathfrak{z}\right)\overset{K}{=}\sum_{n=0}^{\infty}c_{n}\left(\chi\right)\left[\mathfrak{z}\overset{p^{\lambda_{p}\left(n\right)}}{\equiv}n\right],\textrm{ }\forall\mathfrak{z}\in\mathbb{Z}_{p}\label{eq:Chi vdP series} \end{equation} where the convergence is point-wise. \end{thm} Proof: Let $\chi\in\tilde{C}\left(\mathbb{Z}_{p},K\right)$ be arbitrary, by the truncated van der Put identity (\ref{eq:truncated van der Put identity}), $\chi\left(\left[\mathfrak{z}\right]_{p^{N}}\right)=S_{p:N}\left\{ \chi\right\} \left(\mathfrak{z}\right)$. Thus, the rising-continuity of $\chi$ guarantees that $S_{p:N}\left\{ \chi\right\} \left(\mathfrak{z}\right)$ converges in $K$ to $\chi\left(\mathfrak{z}\right)$ as $N\rightarrow\infty$ for each $\mathfrak{z}\in\mathbb{Z}_{p}$. By (\ref{eq:vdP criterion for rising-continuity}), this then implies that the van der Put coefficients of $\chi$ satisfy the conditions of \textbf{Proposition \ref{prop:vdP criterion for rising continuity}} for all $\mathfrak{z}\in\mathbb{Z}_{p}$, which then shows that the van der Put series $S_{p}\left\{ \chi\right\} \left(\mathfrak{z}\right)$ converges at every $\mathfrak{z}\in\mathbb{Z}_{p}$, and\textemdash moreover\textemdash that it converges to $\chi\left(\mathfrak{z}\right)$. This proves (I). As for (II), the uniqueness specified therein is equivalent to demonstrating that $S_{p}$ is an isomorphism in the manner described above. The proof of this is like so: \begin{itemize} \item (Surjectivity) Let $V\left(\mathfrak{z}\right)$ be any formal van der Put series which converges $q$-adically at every $\mathfrak{z}\in\mathbb{Z}_{p}$. Then, letting: \begin{equation} \chi\left(\mathfrak{z}\right)\overset{\textrm{def}}{=}\lim_{N\rightarrow\infty}S_{p:N}\left\{ V\right\} \left(\mathfrak{z}\right) \end{equation} we have that $V\left(m\right)=\chi\left(m\right)$, and hence, $V\left(\mathfrak{z}\right)=S_{p}\left\{ \chi\right\} $. Thus, $S_{p:N}\left\{ \chi\right\} \left(\mathfrak{z}\right)=\chi\left(\left[\mathfrak{z}\right]_{p^{N}}\right)$, and so $\chi\left(\mathfrak{z}\right)\overset{\textrm{def}}{=}\lim_{N\rightarrow\infty}S_{p:N}\left\{ V\right\} \left(\mathfrak{z}\right)=\chi\left(\left[\mathfrak{z}\right]_{p^{N}}\right)$ shows that $\chi$ is rising-continuous. This shows that $V=S_{p}\left\{ \chi\right\} $, and thus, that $S_{p}$ is surjective. \item (Injectivity) Let $\chi_{1},\chi_{2}\in\tilde{C}\left(\mathbb{Z}_{p},K\right)$ and suppose $S_{p}\left\{ \chi_{1}\right\} =S_{p}\left\{ \chi_{2}\right\} $. Then, by (I): \begin{equation} \chi_{1}\left(\mathfrak{z}\right)\overset{\textrm{(I)}}{=}S_{p}\left\{ \chi_{1}\right\} \left(\mathfrak{z}\right)=S_{p}\left\{ \chi_{2}\right\} \left(\mathfrak{z}\right)\overset{\textrm{(I)}}{=}\chi_{2}\left(\mathfrak{z}\right),\textrm{ }\forall\mathfrak{z}\in\mathbb{Z}_{p} \end{equation} which proves $\chi_{1}=\chi_{2}$, which proves the injectivity of $S_{p}$. \end{itemize} Thus, $S_{p}$ is an isomorphism. Q.E.D. \vphantom{} While this shows that rising-continuous functions are equivalent to those van der Put series which converge point-wise at every $\mathfrak{z}\in\mathbb{Z}_{p}$, they are also more than that. As suggested by the name, rising-continuous functions naturally occur when we wish to take a function on $\mathbb{N}_{0}$ and interpolate it to one on $\mathbb{Z}_{p}$. This will provide us with an alternative characterization of rising-continuous functions as being precisely those functions on $\mathbb{Z}_{p}$ whose restrictions to $\mathbb{N}_{0}$ admit interpolations to functions on $\mathbb{Z}_{p}$. I call the process underlying \textbf{rising-continuation}. \begin{defn} \label{def:rising-continuation}Let $\mathbb{F}$ be $\mathbb{Q}$ or a field extension thereof, and let $\chi:\mathbb{N}_{0}\rightarrow\mathbb{F}$ be a function. Letting $p,q$ be integers $\geq2$, with $q$ prime, we say $\chi$ has (or ``admits'') a \textbf{$\left(p,q\right)$-adic }\index{rising-continuation}\textbf{rising-continuation} \textbf{(to $K$)} whenever there is a metrically complete $q$-adic field extension $K$ of $\mathbb{F}$ and a rising-continuous function $\chi^{\prime}:\mathbb{Z}_{p}\rightarrow K$ so that $\chi^{\prime}\left(n\right)=\chi\left(n\right)$ for all $n\in\mathbb{N}_{0}$. We call any $\chi^{\prime}$ satisfying this property a \textbf{($\left(p,q\right)$-adic} or \textbf{$\left(p,K\right)$-adic)}\emph{ }\textbf{rising-continuation }of $\chi$ (to $K$). \end{defn} \begin{prop} \label{prop:rising-continuation admission}Let $\chi:\mathbb{N}_{0}\rightarrow\mathbb{F}$ be a function admitting a $\left(p,q\right)$-adic rising-continuation to $K$. Then: \vphantom{} I. The rising-continuation of $\chi$ is unique. As such, we will write $\chi^{\prime}$\nomenclature{$\chi^{\prime}$}{the rising-contination of $\chi:\mathbb{N}_{0}\rightarrow\mathbb{F}$} to denote the rising continuation of $\chi$. \vphantom{} II. \begin{equation} \chi^{\prime}\left(\mathfrak{z}\right)\overset{K}{=}\lim_{j\rightarrow\infty}\chi\left(\left[\mathfrak{z}\right]_{p^{j}}\right)=\sum_{n=0}^{\infty}c_{n}\left(\chi\right)\left[\mathfrak{z}\overset{p^{\lambda_{p}\left(n\right)}}{\equiv}n\right],\textrm{ }\forall\mathfrak{z}\in\mathbb{Z}_{p}\label{eq:Rising continuation limit formula} \end{equation} \end{prop} Proof: I. Suppose $\chi$ admits two (possibly distinct) rising-continuations, $\chi^{\prime}$ and $\chi^{\prime\prime}$. To see that $\chi^{\prime}$ and $\chi^{\prime\prime}$ must be the same, we note that since the restrictions of both $\chi^{\prime}$ and $\chi^{\prime\prime}$ to $\mathbb{N}_{0}$ are, by definition, equal to $\chi$, $\chi^{\prime}\left(n\right)=\chi^{\prime\prime}\left(n\right)$ for all $n\in\mathbb{N}_{0}$. So, let $\mathfrak{z}$ be an arbitrary element of $\mathbb{Z}_{p}^{\prime}$; necessarily, $\mathfrak{z}$ has infinitely many non-zero $p$-adic digits. Consequently, $\left\{ \left[\mathfrak{z}\right]_{p^{j}}\right\} _{j\geq1}$ is a rising sequence of non-negative integers converging to $\mathfrak{z}$. As such, using rising-continuity of $\chi^{\prime}$ and $\chi^{\prime\prime}$, we can write: \[ \lim_{j\rightarrow\infty}\chi^{\prime}\left(\left[\mathfrak{z}\right]_{p^{j}}\right)\overset{K}{=}\chi^{\prime}\left(\mathfrak{z}\right) \] \[ \lim_{j\rightarrow\infty}\chi^{\prime\prime}\left(\left[\mathfrak{z}\right]_{p^{j}}\right)\overset{K}{=}\chi^{\prime\prime}\left(\mathfrak{z}\right) \] Since the $\left[\mathfrak{z}\right]_{p^{j}}$s are integers, this yields: \[ \chi^{\prime}\left(\mathfrak{z}\right)=\lim_{j\rightarrow\infty}\chi^{\prime}\left(\left[\mathfrak{z}\right]_{p^{j}}\right)=\lim_{j\rightarrow\infty}\chi\left(\left[\mathfrak{z}\right]_{p^{j}}\right)=\lim_{j\rightarrow\infty}\chi^{\prime\prime}\left(\left[\mathfrak{z}\right]_{p^{j}}\right)=\chi^{\prime\prime}\left(\mathfrak{z}\right) \] Since $\mathfrak{z}$ was arbitrary, we have that $\chi^{\prime}\left(\mathfrak{z}\right)=\chi^{\prime\prime}\left(\mathfrak{z}\right)$ for all $\mathfrak{z}\in\mathbb{Z}_{p}^{\prime}$. Thus, $\chi^{\prime}$ and $\chi^{\prime\prime}$ are equal to one another on both $\mathbb{Z}_{p}^{\prime}$ and $\mathbb{N}_{0}$, and hence, on all of $\mathbb{Z}_{p}$. This shows that $\chi^{\prime}$ and $\chi^{\prime\prime}$ are in fact the same function, and therefore proves the uniqueness of $\chi$'s rising-continuation. \vphantom{} II. As a rising-continuous function, $\chi^{\prime}\left(\mathfrak{z}\right)$ is uniquely determined by its values on $\mathbb{N}_{0}$. Since $\chi^{\prime}\mid_{\mathbb{N}_{0}}=\chi$, $\chi^{\prime}$ and $\chi$ then have the same van der Put coefficients. Using the van der Put identity (\ref{eq:van der Put identity}) then gives (\ref{eq:Rising continuation limit formula}). Q.E.D. \vphantom{} Now we have our characterization of rising-continuous functions in terms of rising-continuability. \begin{thm} \label{thm:characterization of rising-continuability}Let $\mathbb{F}$ be $\mathbb{Q}$ or a field extension thereof, let $\chi:\mathbb{N}_{0}\rightarrow\mathbb{F}$ be a function, let $p,q$ be integers $\geq2$, with $q$ prime, and let $K$ be a metrically complete $q$-adic field extension of $\mathbb{F}$. Then, the following are equivalent: \vphantom{} I. $\chi$ admits a $\left(p,q\right)$-adic rising-continuation to $K$. \vphantom{} II. For each $\mathfrak{z}\in\mathbb{Z}_{p}$, $\chi\left(\left[\mathfrak{z}\right]_{p^{n}}\right)$ converges to a limit in $K$ as $n\rightarrow\infty$. \end{thm} Proof: i. Suppose (I) holds. Then, by \textbf{Proposition \ref{prop:rising-continuation admission}}, (\ref{eq:Rising continuation limit formula}) holds, which shows that (II) is true. \vphantom{} ii. Conversely, suppose (II) holds. Then, by the van der Put identity (\textbf{Proposition \ref{prop:vdP identity}}), $S_{p}\left\{ \chi\right\} $ is a rising-continuous function whose restriction to $\mathbb{N}_{0}$ is equal to $\chi$, which means that $S_{p}\left\{ \chi\right\} $ is the rising-continuation of $\chi$, and thus, that $\chi$ is rising-continuable. So, (II) and (I) are equivalent. Q.E.D. \vphantom{} As a consequence of this, we then have that $K$-linear space (under point-wise addition) of all $\chi:\mathbb{N}_{0}\rightarrow\mathbb{F}$ which admit $\left(p,q\right)$-adic rising-continuations to $K$ is then isomorphic to $\tilde{C}\left(\mathbb{Z}_{p},K\right)$, with the isomorphism being the act of rising-continuation: $\chi\mapsto\chi^{\prime}$. As such, \textbf{\emph{from now on (for the most part) we will identify $\chi^{\prime}$ and $\chi=\chi^{\prime}\mid_{\mathbb{N}_{0}}$ and treat them as one and the same.}} The exceptions to this convention will be those occasions where we have a $\left(p,q\right)$-adically continuable function $\chi:\mathbb{N}_{0}\rightarrow\mathbb{F}\subseteq K$ and a function $\eta:\mathbb{Z}_{p}\rightarrow K$, and we wish to show that $\mu$ is in fact equal $\chi^{\prime}$. Using $\chi^{\prime}$ in this context will then allow us to distinguish between $\mu$ (our candidate for $\chi^{\prime}$) and the actual ``correct'' rising-continuation of $\chi$. Because many of our rising-continuous functions will emerge as interpolations of solutions of systems of functional equations on $\mathbb{N}_{0}$, our next theorem will be quite the time-saver. \begin{thm} \label{thm:rising-continuability of Generic H-type functional equations}Let $H$ be a semi-basic $p$-Hydra map which fixes $0$, and consider the system of functional equations\index{functional equation}: \begin{equation} f\left(pn+j\right)=\frac{\mu_{j}}{p}f\left(n\right)+c_{j},\textrm{ }\forall j\in\left\{ 0,\ldots,p-1\right\} ,\textrm{ }\forall n\in\mathbb{N}_{0}\label{eq:Generic H-type functional equations} \end{equation} where $\mu_{j}/p=H_{j}^{\prime}\left(0\right)$. Then: \vphantom{} I. There is a unique function $\chi:\mathbb{N}_{0}\rightarrow\overline{\mathbb{Q}}$ such that $f=\chi$ is a solution of \emph{(\ref{eq:Generic H-type functional equations})}. \vphantom{} II. The solution $\chi$ \emph{(\ref{eq:Generic H-type functional equations})} is rising-continuable\index{rising-continuation} to a function $\chi:\mathbb{Z}_{p}\rightarrow\mathbb{C}_{q_{H}}$. Moreover, this continuation satisfies: \begin{equation} \chi\left(p\mathfrak{z}+j\right)=\frac{\mu_{j}}{p}\chi\left(\mathfrak{z}\right)+c_{j},\textrm{ }\forall j\in\left\{ 0,\ldots,p-1\right\} ,\textrm{ }\forall\mathfrak{z}\in\mathbb{Z}_{p}\label{eq:Rising-continuation Generic H-type functional equations} \end{equation} \vphantom{} III. The function $\chi:\mathbb{Z}_{p}\rightarrow\mathbb{C}_{q_{H}}$ described in \emph{(III)} is the unique rising-continuous function $\mathbb{Z}_{p}\rightarrow\mathbb{C}_{q_{H}}$ satisfying \emph{(\ref{eq:Rising-continuation Generic H-type functional equations})}. \end{thm} Proof: I. Let $f:\mathbb{N}_{0}\rightarrow\overline{\mathbb{Q}}$ be any solution of (\ref{eq:Generic H-type functional equations}). Setting $n=j=0$ yields: \begin{align*} f\left(0\right) & =\frac{\mu_{0}}{p}f\left(0\right)+c_{0}\\ & \Updownarrow\\ f\left(0\right) & =\frac{c_{0}}{1-\frac{\mu_{0}}{p}} \end{align*} Since $H$ is semi-basic, $\mu_{0}/p\neq1$, so $f\left(0\right)$ is well-defined. Then, we have that: \begin{equation} f\left(j\right)=\frac{\mu_{j}}{p}f\left(0\right)+c_{j},\textrm{ }\forall j\in\left\{ 0,\ldots,p-1\right\} \end{equation} and, more generally: \begin{equation} f\left(m\right)=\frac{\mu_{\left[m\right]_{p}}}{p}f\left(\frac{m-\left[m\right]_{p}}{p}\right)+c_{\left[m\right]_{p}},\textrm{ }\forall m\in\mathbb{N}_{0}\label{eq:f,m, digit shifting} \end{equation} Since the map $m\mapsto\frac{m-\left[m\right]_{p}}{p}$ sends the non-negative integer: \[ m=m_{0}+m_{1}p+\cdots+m_{\lambda_{p}\left(m\right)-1}p^{\lambda_{p}\left(m\right)-1} \] to the integer: \[ m=m_{1}+m_{2}p+\cdots+m_{\lambda_{p}\left(m\right)-1}p^{\lambda_{p}\left(m\right)-2} \] it follows that $m\mapsto\frac{m-\left[m\right]_{p}}{p}$ eventually iterates every $m\in\mathbb{N}_{0}$ to $0$, and hence (\ref{eq:f,m, digit shifting}) implies that, for every $m\in\mathbb{N}_{0}$, $f\left(m\right)$ is entirely determined by $f\left(0\right)$ and the $c_{j}$s. Since $f\left(0\right)$ is uniquely determined by $c_{0}$ and $H$, the equation (\ref{eq:Generic H-type functional equations}) possesses exactly one solution. Let us denote this solution by $\chi$. \vphantom{} II. Because $H$ is semi-basic, we can repeat for $\chi$ the argument we used to prove the existence of $\chi_{H}$'s rising-continuation in \textbf{Lemma \ref{lem:Unique rising continuation and p-adic functional equation of Chi_H}} (page \pageref{lem:Unique rising continuation and p-adic functional equation of Chi_H}): since any $\mathfrak{z}\in\mathbb{Z}_{p}^{\prime}$ has infinitely many non-zero $p$-adic digits, the product of $\mu_{j}/p$ taken over all the digits of such a $\mathfrak{z}$ will converge $q_{H}$-adically to zero. Then, seeing as (\ref{eq:Generic H-type functional equations}) shows that $\chi\left(\mathfrak{z}\right)$ will be a sum of the form: \begin{equation} \beta_{0}+\alpha_{1}\beta_{1}+\alpha_{1}\alpha_{2}\beta_{2}+\alpha_{1}\alpha_{2}\alpha_{3}\beta_{3}+\cdots \end{equation} (where for each $n$, $\beta_{n}$ is one of the $c_{j}$s and $\alpha_{n}$ is $\mu_{j_{n}}/p$, where $j_{n}$ is the $n$th $p$-adic digit of $\mathfrak{z}$) this sum will converge in $\mathbb{C}_{q_{H}}$ for all $\mathfrak{z}\in\mathbb{Z}_{p}^{\prime}$ because of the estimate guaranteed by $H$'s semi-basicness: \begin{equation} \left|\frac{\mu_{j}}{p}\right|_{q_{H}}<1,\textrm{ }\forall j\in\left\{ 1,\ldots,p-1\right\} \end{equation} This proves $\chi$ is rising-continuable. \vphantom{} III. Because $\chi$ admits a rising continuation, its continuation at any $\mathfrak{z}\in\mathbb{Z}_{p}$ is given by the value of $\chi$'s van der Put series at $\mathfrak{z}$. As such: \begin{equation} \chi\left(\mathfrak{z}\right)\overset{\mathbb{C}_{q_{H}}}{=}S_{p}\left\{ \chi\right\} \left(\mathfrak{z}\right)\overset{\mathbb{C}_{q_{H}}}{=}\lim_{N\rightarrow\infty}\chi\left(\left[\mathfrak{z}\right]_{p^{N}}\right) \end{equation} Since $\chi$ satisfies the functional equations (\ref{eq:Generic H-type functional equations}), we can write: \begin{equation} \chi\left(p\left[\mathfrak{z}\right]_{p^{N}}+j\right)\overset{\overline{\mathbb{Q}}}{=}\frac{\mu_{j}}{p}\chi\left(\left[\mathfrak{z}\right]_{p^{N}}\right)+c_{j} \end{equation} for all $\mathfrak{z}\in\mathbb{Z}_{p}$, all $j\in\left\{ 0,\ldots,p-1\right\} $, and all $N\geq0$. Letting $\mathfrak{z}\in\mathbb{Z}_{p}^{\prime}$ be arbitrary, we let $N\rightarrow\infty$. This gives us: \begin{equation} \chi\left(p\mathfrak{z}+j\right)\overset{\mathbb{C}_{q_{H}}}{=}\frac{\mu_{j}}{p}\chi\left(\mathfrak{z}\right)+c_{j},\textrm{ }\forall j,\textrm{ }\forall\mathfrak{z}\in\mathbb{Z}_{p}^{\prime} \end{equation} Note that these identities automatically hold for $\mathfrak{z}\in\mathbb{N}_{0}$ seeing as those particular cases were governed by (\ref{eq:Generic H-type functional equations}). The uniqueness of $\chi$ as a solution of (\ref{eq:Rising-continuation Generic H-type functional equations}) follows from the fact that any $f:\mathbb{Z}_{p}\rightarrow\mathbb{C}_{q_{H}}$ satisfying (\ref{eq:Rising-continuation Generic H-type functional equations}) has a restriction to $\mathbb{N}_{0}$ which satisfies (\ref{eq:Generic H-type functional equations}). This then forces $f=\chi$. Q.E.D. \vphantom{} A slightly more general version of this type of argument (which will particularly useful in Chapters 4 and 6) is as follows: \begin{lem} \label{lem:rising-continuations preserve functional equations}Fix integers $p,q\geq2$, let $K$ be a metrically complete $q$-adic field, and let $\Phi_{j}:\mathbb{Z}_{p}\times K\rightarrow K$ be continuous for $j\in\left\{ 0,\ldots,p-1\right\} $. If $\chi:\mathbb{N}_{0}\rightarrow K$ is rising-continuable to an element $\chi\in\tilde{C}\left(\mathbb{Z}_{p},K\right)$, and if: \[ \chi\left(pn+j\right)=\Phi_{j}\left(n,\chi\left(n\right)\right),\textrm{ }\forall n\in\mathbb{N}_{0},\textrm{ }\forall j\in\left\{ 0,\ldots,p-1\right\} \] then: \[ \chi\left(p\mathfrak{z}+j\right)=\Phi_{j}\left(\mathfrak{z},\chi\left(\mathfrak{z}\right)\right),\textrm{ }\forall\mathfrak{z}\in\mathbb{Z}_{p},\textrm{ }\forall j\in\left\{ 0,\ldots,p-1\right\} \] \end{lem} Proof: Let everything be as given. Then, since $\chi$ is rising continuous, we have that: \[ \chi\left(\mathfrak{z}\right)\overset{K}{=}\lim_{N\rightarrow\infty}\chi\left(\left[\mathfrak{z}\right]_{p^{N}}\right),\textrm{ }\forall\mathfrak{z}\in\mathbb{Z}_{p} \] Since: \[ \chi\left(p\left[\mathfrak{z}\right]_{p^{N}}+j\right)=\Phi_{j}\left(\left[\mathfrak{z}\right]_{p^{N}},\chi\left(\left[\mathfrak{z}\right]_{p^{N}}\right)\right) \] holds true for all $\mathfrak{z}\in\mathbb{Z}_{p}$ and all $N\geq0$, the rising-continuity of $\chi$ and the continuity of $\Phi_{j}$ then guarantee that: \begin{align*} \chi\left(p\mathfrak{z}+j\right) & \overset{K}{=}\lim_{N\rightarrow\infty}\chi\left(p\left[\mathfrak{z}\right]_{p^{N}}+j\right)\\ & \overset{K}{=}\lim_{N\rightarrow\infty}\Phi_{j}\left(\left[\mathfrak{z}\right]_{p^{N}},\chi\left(\left[\mathfrak{z}\right]_{p^{N}}\right)\right)\\ & \overset{K}{=}\Phi_{j}\left(\mathfrak{z},\chi\left(\mathfrak{z}\right)\right) \end{align*} as desired. Q.E.D. \subsection{\label{subsec:3.2.2 Truncations-=00003D000026-The}Truncations and The Banach Algebra of Rising-Continuous Functions} RECALL THAT WE WRITE $\left\Vert \cdot\right\Vert _{p,q}$ TO DENOTE THE $\left(p,q\right)$-adic SUPREMUM NORM. \vphantom{} In this section, we will examine the structure of the vector space $\tilde{C}\left(\mathbb{Z}_{p},K\right)$ of rising-continuous functions $\chi:\mathbb{Z}_{p}\rightarrow K$. We will demonstrate that $\tilde{C}\left(\mathbb{Z}_{p},K\right)$ is a Banach algebra over $K$\textemdash with the usual ``point-wise multiplication of functions'' as its multiplication operation. Not only that, we will also see that $\tilde{C}\left(\mathbb{Z}_{p},K\right)$ extends $C\left(\mathbb{Z}_{p},K\right)$, containing it as a proper sub-algebra. Before we can even begin our discussion of Banach algebras, however, we need to introduce a simple construction which will be of extraordinary importance in our analyses of $\chi_{H}$ in Chapter 4 and beyond. To begin, as we saw in Subsection \ref{subsec:3.1.4. The--adic-Fourier}, the Fourier Transform of a continuous $\left(p,q\right)$-adic function $f$ can be computed using $f$'s van der Put coefficients via equation (\ref{eq:Definition of (p,q)-adic Fourier Coefficients}) from \textbf{Definition \ref{def:pq adic Fourier coefficients}} \begin{equation} \hat{f}\left(t\right)\overset{\mathbb{C}_{q}}{=}\sum_{n=\frac{1}{p}\left|t\right|_{p}}^{\infty}\frac{c_{n}\left(f\right)}{p^{\lambda_{p}\left(n\right)}}e^{-2n\pi it},\textrm{ }\forall t\in\hat{\mathbb{Z}}_{p} \end{equation} Because the convergence of this infinite series \emph{requires }$\lim_{n\rightarrow\infty}\left|c_{n}\left(f\right)\right|_{q}=0$, this formula for $\hat{f}$ is not compatible with general rising-continuous functions; the van der Put coefficients need not converge $q$-adically to $0$. As will be shown in Chapter 4, we can get around the non-convergence of (\ref{eq:Definition of (p,q)-adic Fourier Coefficients}) for an arbitrary rising-continuous function $\chi$ by replacing $\chi$ with a locally constant approximation. I call these approximations \textbf{truncations}, and\textemdash as locally constant functions\textemdash they have the highly desirable property of $\left(p,q\right)$-adic continuity. \begin{defn}[\textbf{$N$th truncations}] \label{def:Nth truncation}For any $\chi\in B\left(\mathbb{Z}_{p},\mathbb{C}_{q}\right)$ and any $N\in\mathbb{N}_{0}$, the \index{$N$th truncation}\textbf{$N$th truncation of $\chi$}, denoted $\chi_{N}$, is the function $\chi_{N}:\mathbb{Z}_{p}\rightarrow\mathbb{C}_{q}$ defined by: \begin{equation} \chi_{N}\left(\mathfrak{z}\right)\overset{\textrm{def}}{=}\sum_{n=0}^{p^{N}-1}\chi\left(n\right)\left[\mathfrak{z}\overset{p^{N}}{\equiv}n\right],\textrm{ }\forall\mathfrak{z}\in\mathbb{Z}_{p}\label{eq:Definition of Nth truncation} \end{equation} We also extend this notation to negative $N$ by defining $\chi_{N}\left(\mathfrak{z}\right)$ to be identically zero whenever $N<0$. \end{defn} \begin{rem} For any $N\in\mathbb{N}_{0}$: \begin{equation} \chi_{N}\left(\mathfrak{z}\right)=\chi\left(\left[\mathfrak{z}\right]_{p^{N}}\right),\textrm{ }\forall\mathfrak{z}\in\mathbb{Z}_{p}\label{eq:Nth truncation of Chi in terms of Chi} \end{equation} Combining (\ref{eq:Nth truncation of Chi in terms of Chi}) and (\ref{eq:truncated van der Put identity}), we then have that: \begin{equation} \chi_{N}\left(\mathfrak{z}\right)=\sum_{n=0}^{p^{N}-1}\chi\left(n\right)\left[\mathfrak{z}\overset{p^{N}}{\equiv}n\right]=\sum_{n=0}^{p^{N}-1}c_{n}\left(\chi\right)\left[\mathfrak{z}\overset{p^{\lambda_{p}\left(n\right)}}{\equiv}n\right]=\chi\left(\left[\mathfrak{z}\right]_{p^{N}}\right)\label{eq:Nth truncation and truncated van-der-Put identity compared} \end{equation} Moreover, note that $\chi_{N}\in C\left(\mathbb{Z}_{p},\mathbb{C}_{q}\right)$ for all $N\in\mathbb{N}_{0}$. \end{rem} \begin{rem} Although Schikhof did not make any significant investigations with truncations, he was aware of there existence. There is an exercise in \cite{Ultrametric Calculus} in the section on the van der Put basis which asks the reader to show that the $N$th truncation of $\chi$ is the best possible $\left(p,q\right)$-adic approximation of $\chi$ which is constant with respect to inputs modulo $p^{N}$. \end{rem} \begin{prop} \label{prop:Truncations converge pointwise iff rising-continuous}Let $\chi:\mathbb{Z}_{p}\rightarrow K$ be\textbf{ any}\emph{ }function. Then, $\chi\in\tilde{C}\left(\mathbb{Z}_{p},K\right)$ if and only if $\chi_{N}$ converges in $K$ to $\chi$ point-wise on $\mathbb{Z}_{p}$ as $N\rightarrow\infty$. \end{prop} Proof: Since $\chi_{N}\left(\mathfrak{z}\right)=\chi\left(\left[\mathfrak{z}\right]_{p^{N}}\right)$, this proposition is just a restatement of the definition of what it means for $\chi$ to be rising continuous ($\chi\left(\left[\mathfrak{z}\right]_{p^{N}}\right)$ converges to $\chi\left(\mathfrak{z}\right)$ point-wise everywhere). Q.E.D. \begin{prop} \label{prop:Unif. convergence of truncation equals continuity}Let $f:\mathbb{Z}_{p}\rightarrow K$ be \textbf{any} function. Then, $f$ is continuous if and only if $f_{N}$ converges in $K$ to $f$ uniformly on $\mathbb{Z}_{p}$ as $N\rightarrow\infty$. \end{prop} Proof: I. If the $f_{N}$s converge uniformly to $f$, $f$ is continuous, seeing as the $f_{N}$ are locally constant\textemdash and hence, continuous\textemdash and seeing as how the uniform limit of continuous functions is continuous. \vphantom{} II. Conversely, if $f$ is continuous then, since $\mathbb{Z}_{p}$ is compact, $f$ is uniformly continuous. So, letting $\epsilon>0$, pick $\delta$ so that $\left|f\left(\mathfrak{z}\right)-f\left(\mathfrak{y}\right)\right|_{q}<\epsilon$ for all $\mathfrak{z},\mathfrak{y}\in\mathbb{Z}_{p}$ with $\left|\mathfrak{z}-\mathfrak{y}\right|_{p}<\delta$. Note that: \[ \left|\mathfrak{z}-\left[\mathfrak{z}\right]_{p^{N}}\right|_{p}\leq p^{-N},\textrm{ }\forall\mathfrak{z}\in\mathbb{Z}_{p},\textrm{ }\forall N\in\mathbb{N}_{0} \] So, choose $N$ large enough so that $p^{-N}<\delta$. Then $\left|\mathfrak{z}-\left[\mathfrak{z}\right]_{p^{N}}\right|_{p}<\delta$ for all $\mathfrak{z}\in\mathbb{Z}_{p}$, and so: \[ \sup_{\mathfrak{z}\in\mathbb{Z}_{p}}\left|f\left(\mathfrak{z}\right)-f_{N}\left(\mathfrak{z}\right)\right|_{q}=\sup_{\mathfrak{z}\in\mathbb{Z}_{p}}\left|f\left(\mathfrak{z}\right)-f\left(\left[\mathfrak{z}\right]_{p^{N}}\right)\right|_{q}<\epsilon \] which proves that the $f_{N}$s converge uniformly to $f$. Q.E.D. \begin{rem} As is proved in \textbf{Theorem \ref{thm:vdP basis theorem}} (see (\ref{eq:Fourier transform of Nth truncation in terms of vdP coefficients}) on page \pageref{eq:Fourier transform of Nth truncation in terms of vdP coefficients}), the formula (\ref{eq:Definition of (p,q)-adic Fourier Coefficients}) can be applied to: \begin{equation} f_{N}\left(\mathfrak{z}\right)=f\left(\left[\mathfrak{z}\right]_{p^{N}}\right)=\sum_{n=0}^{p^{N}-1}c_{n}\left(f\right)\left[\mathfrak{z}\overset{p^{\lambda_{p}\left(n\right)}}{\equiv}n\right] \end{equation} whereupon we obtain: \begin{equation} \hat{f}_{N}\left(t\right)=\sum_{n=\frac{\left|t\right|_{p}}{p}}^{p^{N}-1}\frac{c_{n}\left(f\right)}{p^{\lambda_{p}\left(n\right)}}e^{-2\pi int} \end{equation} where $\hat{f}_{N}$ is the Fourier transform of the $N$th truncation of $f$. While this formula for $\hat{f}_{N}$ can be used to perform the kind of Fourier analysis we shall do in Chapters 4 and 6, it turns out to be much easier to work with the $N$th truncation directly instead of using the van der Put coefficients in this manner, primarily because the formula for $c_{n}\left(\chi_{H}\right)$ becomes unwieldy\footnote{See\textbf{ Proposition \ref{prop:van der Put series for Chi_H}} on page \pageref{prop:van der Put series for Chi_H} for the details.} when $H$ is a $p$-Hydra map for $p\geq3$. Nevertheless, the van der Put coefficient expression for $\hat{f}_{N}\left(t\right)$ is of interest in its own right, and it may be worth investigating how the above formula might be inverted so as to produce expressions for $c_{n}\left(f\right)$ in terms of $\hat{f}_{N}\left(t\right)$. That being said, both of these realizations of a function's $N$th truncation will be of use to us, both now and in the future. \end{rem} \vphantom{} Now we can begin our approach of the Banach algebra of rising-continuous\index{rising-continuous!Banach algebra of} functions. \begin{defn} For any integer $n\geq0$, we define the operator \nomenclature{$\nabla_{p^{n}}$}{ }$\nabla_{p^{n}}:B\left(\mathbb{Z}_{p},\mathbb{C}_{q}\right)\rightarrow B\left(\mathbb{Z}_{p},\mathbb{C}_{q}\right)$ by: \begin{equation} \nabla_{p^{n}}\left\{ \chi\right\} \left(\mathfrak{z}\right)\overset{\textrm{def}}{=}\begin{cases} \chi\left(\left[\mathfrak{z}\right]_{p^{n}}\right)-\chi\left(\left[\mathfrak{z}\right]_{p^{n-1}}\right) & \textrm{if }n\geq1\\ \chi\left(0\right) & \textrm{if }n=0 \end{cases},\textrm{ }\forall\mathfrak{z}\in\mathbb{Z}_{p},\textrm{ }\forall\chi\in B\left(\mathbb{Z}_{p},\mathbb{C}_{q}\right)\label{eq:Definition of Del p^n of Chi} \end{equation} We then define the \textbf{$\nabla$-norm }(``del norm'') $\left\Vert \cdot\right\Vert _{\nabla}:B\left(\mathbb{Z}_{p},\mathbb{C}_{q}\right)\rightarrow\left[0,\infty\right)$ by: \begin{equation} \left\Vert \chi\right\Vert _{\nabla}\overset{\textrm{def}}{=}\sup_{n\geq0}\left\Vert \nabla_{p^{n}}\left\{ \chi\right\} \right\Vert _{p,q}=\sup_{n\geq0}\left(\sup_{\mathfrak{z}\in\mathbb{Z}_{p}}\left|\nabla_{p^{n}}\left\{ \chi\right\} \left(\mathfrak{z}\right)\right|_{q}\right)\label{eq:Defintion of Del-norm} \end{equation} \end{defn} \begin{prop} Let $\chi\in B\left(\mathbb{Z}_{p},\mathbb{C}_{q}\right)$. Then: \begin{equation} S_{p:N}\left\{ \chi\right\} \left(\mathfrak{z}\right)=\sum_{n=0}^{N}\nabla_{p^{n}}\left\{ \chi\right\} \left(\mathfrak{z}\right),\textrm{ }\forall\mathfrak{z}\in\mathbb{Z}_{p},\textrm{ }\forall N\in\mathbb{N}_{0}\label{eq:Partial vdP series in terms of Del} \end{equation} Moreover, we have the formal identity: \begin{equation} S_{p}\left\{ \chi\right\} \left(\mathfrak{z}\right)=\sum_{n=0}^{\infty}\nabla_{p^{n}}\left\{ \chi\right\} \left(\mathfrak{z}\right)\label{eq:vdP series in terms of Del} \end{equation} This identity holds for any $\mathfrak{z}\in\mathbb{Z}_{p}$ for which either the left- or right-hand side converges in $\mathbb{C}_{q}$. In particular, (\ref{eq:vdP series in terms of Del}) holds in $\mathbb{C}_{q}$ for all $\mathfrak{z}\in\mathbb{Z}_{p}$ whenever $\chi\in\tilde{C}\left(\mathbb{Z}_{p},\mathbb{C}_{q}\right)$. \end{prop} Proof: Sum the telescoping series. Q.E.D. \begin{prop} Let $\chi,\eta\in B\left(\mathbb{Z}_{p},\mathbb{C}_{q}\right)$. Then, for all $n\in\mathbb{N}_{0}$ and all $\mathfrak{z}\in\mathbb{Z}_{p}$: \begin{equation} \nabla_{p^{n}}\left\{ \chi\cdot\eta\right\} \left(\mathfrak{z}\right)=\chi\left(\left[\mathfrak{z}\right]_{p^{n-1}}\right)\nabla_{p^{n}}\left\{ \eta\right\} \left(\mathfrak{z}\right)+\eta\left(\left[\mathfrak{z}\right]_{p^{n}}\right)\nabla_{p^{n}}\left\{ \chi\right\} \left(\mathfrak{z}\right)\label{eq:Quasi-derivation identity for del} \end{equation} using the abuse of notation $\chi\left(\left[\mathfrak{z}\right]_{p^{0-1}}\right)$ to denote the zero function in the $n=0$ case. \end{prop} Proof: Using truncation notation: \begin{align*} \chi_{n-1}\left(\mathfrak{z}\right)\nabla_{p^{n}}\left\{ \eta\right\} \left(\mathfrak{z}\right)+\eta_{n}\left(\mathfrak{z}\right)\nabla_{p^{n}}\left\{ \chi\right\} \left(\mathfrak{z}\right) & =\chi_{n-1}\left(\mathfrak{z}\right)\left(\eta_{n}\left(\mathfrak{z}\right)-\eta_{n-1}\left(\mathfrak{z}\right)\right)\\ & +\eta_{n}\left(\mathfrak{z}\right)\left(\chi_{n}\left(\mathfrak{z}\right)-\chi_{n-1}\left(\mathfrak{z}\right)\right)\\ & =\bcancel{\chi_{n-1}\left(\mathfrak{z}\right)\eta_{n}\left(\mathfrak{z}\right)}-\chi_{n-1}\left(\mathfrak{z}\right)\eta_{n-1}\left(\mathfrak{z}\right)\\ & +\eta_{n}\left(\mathfrak{z}\right)\chi_{n}\left(\mathfrak{z}\right)-\cancel{\eta_{n}\left(\mathfrak{z}\right)\chi_{n-1}\left(\mathfrak{z}\right)}\\ & =\chi_{n}\left(\mathfrak{z}\right)\eta_{n}\left(\mathfrak{z}\right)-\chi_{n-1}\left(\mathfrak{z}\right)\eta_{n-1}\left(\mathfrak{z}\right)\\ & =\nabla_{p^{n}}\left\{ \chi\cdot\eta\right\} \left(\mathfrak{z}\right) \end{align*} Q.E.D. \begin{prop} Let $\chi\in B\left(\mathbb{Z}_{p},K\right)$. Then, $S_{p}\left\{ \chi\right\} $ is identically zero whenever $\chi$ vanishes on $\mathbb{N}_{0}$. \end{prop} Proof: If $\chi\left(n\right)=0$ for all $n\in\mathbb{N}_{0}$, then $c_{n}\left(\chi\right)=0$ for all $n\in\mathbb{N}_{0}$, and hence $S_{p}\left\{ \chi\right\} $ is identically zero. Q.E.D. \vphantom{} The moral of this proposition is that the operator $S_{p}$ behaves like a projection operator on $B\left(\mathbb{Z}_{p},K\right)$. For example, given any rising-continuous function $\chi$ and any function $f\in B\left(\mathbb{Z}_{p},K\right)$ with $f\left(n\right)=0$ for all $n\in\mathbb{N}_{0}$, we have that: \begin{equation} S_{p}\left\{ \chi+f\right\} =S_{p}\left\{ \chi\right\} =\chi \end{equation} The reason why we say $S_{p}$ behaves merely ``like'' a projection operator on $B\left(\mathbb{Z}_{p},K\right)$\textemdash rather than proving that $S_{p}$ actually \emph{is }a projection operator on $B\left(\mathbb{Z}_{p},K\right)$\textemdash is because there exist $f\in B\left(\mathbb{Z}_{p},K\right)$ for which $S_{p}\left\{ f\right\} \left(\mathfrak{z}\right)$ fails to converge at \emph{any} $\mathfrak{z}\in\mathbb{Z}_{p}$. Case in point: \begin{example} Let $f\in B\left(\mathbb{Z}_{p},K\right)$ be defined by: \begin{equation} f\left(\mathfrak{z}\right)\overset{\textrm{def}}{=}\begin{cases} \mathfrak{z} & \textrm{if }\mathfrak{z}\in\mathbb{N}_{0}\\ 0 & \textrm{else} \end{cases}\label{eq:Example of a bounded (p,q)-adic function whose van der Put series is divergent.} \end{equation} Then, we have the formal identity: \begin{align*} S_{p}\left\{ f\right\} \left(\mathfrak{z}\right) & =\sum_{n=1}^{\infty}n\left[\mathfrak{z}\overset{p^{\lambda_{p}\left(n\right)}}{\equiv}n\right]\\ & =\sum_{n=1}^{\infty}\sum_{m=p^{n-1}}^{p^{n}-1}m\left[\mathfrak{z}\overset{p^{n}}{\equiv}m\right]\\ & =\sum_{n=1}^{\infty}\left(\sum_{m=0}^{p^{n}-1}m\left[\mathfrak{z}\overset{p^{n}}{\equiv}m\right]-\sum_{m=0}^{p^{n-1}-1}m\left[\mathfrak{z}\overset{p^{n}}{\equiv}m\right]\right) \end{align*} Here: \[ \sum_{m=0}^{p^{n}-1}m\left[\mathfrak{z}\overset{p^{n}}{\equiv}m\right]=\left[\mathfrak{z}\right]_{p^{n}} \] because $m=\left[\mathfrak{z}\right]_{p^{n}}$ is the unique integer in $\left\{ 0,\ldots,p^{n}-1\right\} $ which is congruent to $\mathfrak{z}$ mod $p^{n}$. On the other hand, for the $m$-sum with $p^{n-1}-1$ as an upper bound, we end up with: \[ \sum_{m=0}^{p^{n-1}-1}m\left[\mathfrak{z}\overset{p^{n}}{\equiv}m\right]=\left[\mathfrak{z}\right]_{p^{n}}\left[\left[\mathfrak{z}\right]_{p^{n}}<p^{n-1}\right] \] and so: \begin{align*} S_{p}\left\{ f\right\} \left(\mathfrak{z}\right) & =\sum_{n=1}^{\infty}\left[\mathfrak{z}\right]_{p^{n}}\left(1-\left[\left[\mathfrak{z}\right]_{p^{n}}<p^{n-1}\right]\right)\\ & =\sum_{n=1}^{\infty}\left[\mathfrak{z}\right]_{p^{n}}\left[\left[\mathfrak{z}\right]_{p^{n}}\geq p^{n-1}\right] \end{align*} Consequently: \[ S_{p}\left\{ f\right\} \left(-1\right)=\sum_{n=1}^{\infty}\left(p^{n}-1\right)\left[p^{n}-1\geq p^{n-1}\right]=\sum_{n=1}^{\infty}\left(p^{n}-1\right) \] which does not converge in $\mathbb{C}_{q}$ because $\left|p^{n}-1\right|_{q}$ does not tend to $0$ in $\mathbb{R}$ as $n\rightarrow\infty$. \end{example} \vphantom{} The above example also shows that $S_{p}$ is as much a continuation-creating operator as it is a projection operator. Bounded functions which do not behave well under limits of sequences in $\mathbb{Z}_{p}$ will have ill-behaved images under $S_{p}$. This motivates the following definition: \begin{defn} We write \nomenclature{$\tilde{B}\left(\mathbb{Z}_{p},\mathbb{C}_{q}\right)$}{set of $f\in B\left(\mathbb{Z}_{p},\mathbb{C}_{q}\right)$ so that $S_{p}\left\{ f\right\} \in B\left(\mathbb{Z}_{p},\mathbb{C}_{q}\right)$.}$\tilde{B}\left(\mathbb{Z}_{p},\mathbb{C}_{q}\right)$ to denote the set of all $f\in B\left(\mathbb{Z}_{p},\mathbb{C}_{q}\right)$ so that $S_{p}\left\{ f\right\} \in B\left(\mathbb{Z}_{p},\mathbb{C}_{q}\right)$ (i.e., $S_{p}\left\{ f\right\} $ converges point-wise in $\mathbb{C}_{q}$). \end{defn} \begin{rem} Note that $\tilde{B}\left(\mathbb{Z}_{p},\mathbb{C}_{q}\right)\neq\tilde{C}\left(\mathbb{Z}_{p},\mathbb{C}_{q}\right)$. The only function in $\tilde{C}\left(\mathbb{Z}_{p},\mathbb{C}_{q}\right)$ which vanishes on $\mathbb{N}_{0}$ is the constant function $0$. Consequently, any function in $B\left(\mathbb{Z}_{p},\mathbb{C}_{q}\right)$ which is supported on $\mathbb{Z}_{p}^{\prime}$ is an element of $\tilde{B}\left(\mathbb{Z}_{p},\mathbb{C}_{q}\right)$ which is not in $\tilde{C}\left(\mathbb{Z}_{p},\mathbb{C}_{q}\right)$. That such a function is in $\tilde{B}\left(\mathbb{Z}_{p},\mathbb{C}_{q}\right)$ is, of course, because its image under $S_{p}$ is identically $0$. \end{rem} \begin{prop} The map: \[ \left(\chi,\eta\right)\in B\left(\mathbb{Z}_{p},\mathbb{C}_{q}\right)\times B\left(\mathbb{Z}_{p},\mathbb{C}_{q}\right)\mapsto\left\Vert \chi-\eta\right\Vert _{\nabla}\in\left[0,\infty\right) \] defines a non-archimedean semi-norm on $B\left(\mathbb{Z}_{p},\mathbb{C}_{q}\right)$ and $\tilde{B}\left(\mathbb{Z}_{p},\mathbb{C}_{q}\right)$. \end{prop} Proof: I. (Positivity) Observe that $\left\Vert \chi\right\Vert _{\nabla}<\infty$ for all $\chi\in B\left(\mathbb{Z}_{p},\mathbb{C}_{q}\right)$. However, $\left\Vert \cdot\right\Vert _{\nabla}$ is \emph{not }positive-definite; let $\chi\in B\left(\mathbb{Z}_{p},\mathbb{C}_{q}\right)$ be any function supported on $\mathbb{Z}_{p}^{\prime}$. Then, $\chi$ vanishes on $\mathbb{N}_{0}$, and so, $\chi\left(\left[\mathfrak{z}\right]_{p^{n}}\right)=0$ for all $\mathfrak{z}\in\mathbb{Z}_{p}$ and all $n\in\mathbb{N}_{0}$. This forces $\left\Vert \chi\right\Vert _{\nabla}$ to be zero; however, $\chi$ is not identically zero. So, $\left\Vert \cdot\right\Vert _{\nabla}$ cannot be a norm on $B\left(\mathbb{Z}_{p},\mathbb{C}_{q}\right)$ \vphantom{} II. Since $\nabla_{p^{n}}$ is linear for all $n\geq0$, for any $\mathfrak{a}\in\mathbb{C}_{q}$, we have that: \[ \left\Vert \mathfrak{a}\chi\right\Vert _{\nabla}=\sup_{n\geq0}\left\Vert \mathfrak{a}\nabla_{p^{n}}\left\{ \chi\right\} \right\Vert _{p,q}=\left|\mathfrak{a}\right|_{q}\sup_{n\geq0}\left\Vert \nabla_{p^{n}}\left\{ \chi\right\} \right\Vert _{p,q}=\left|\mathfrak{a}\right|_{q}\left\Vert \chi\right\Vert _{\nabla} \] for all $\chi\in B\left(\mathbb{Z}_{p},\mathbb{C}_{q}\right)$. As for the ultrametric inequality, observing that: \[ \left\Vert \chi-\eta\right\Vert _{\nabla}=\sup_{n\geq0}\left\Vert \nabla_{p^{n}}\left\{ \chi-\eta\right\} \right\Vert _{p,q}=\sup_{n\geq0}\left\Vert \nabla_{p^{n}}\left\{ \chi\right\} -\nabla_{p^{n}}\left\{ \eta\right\} \right\Vert _{p,q} \] the fact that $d\left(\chi,\eta\right)=\left\Vert \chi-\eta\right\Vert _{p,q}$ defines an ultrametric on $B\left(\mathbb{Z}_{p},\mathbb{C}_{q}\right)$ lets us write: \begin{align*} \left\Vert \chi-\eta\right\Vert _{\nabla} & =\sup_{n\geq0}\left\Vert \nabla_{p^{n}}\left\{ \chi\right\} -\nabla_{p^{n}}\left\{ \eta\right\} \right\Vert _{p,q}\\ & \leq\sup_{n\geq0}\max\left\{ \left\Vert \nabla_{p^{n}}\left\{ \chi\right\} \right\Vert _{p,q},\left\Vert \nabla_{p^{n}}\left\{ \eta\right\} \right\Vert _{p,q}\right\} \\ & =\max\left\{ \left\Vert \chi\right\Vert _{\nabla},\left\Vert \eta\right\Vert _{\nabla}\right\} \end{align*} which proves the ultrametric inequality. Thus, $\left\Vert \cdot\right\Vert _{\nabla}$ is a non-archimedean semi-norm on $B\left(\mathbb{Z}_{p},\mathbb{C}_{q}\right)$. The arguments for $\tilde{B}\left(\mathbb{Z}_{p},\mathbb{C}_{q}\right)$ are identical. Q.E.D. \vphantom{} Next, we show that $\nabla$-(semi)norm dominates the $\left(p,q\right)$-adic supremum norm. \begin{prop} \label{prop:continuous embeds in rising-continuous}If $\chi\in\tilde{C}\left(\mathbb{Z}_{p},\mathbb{C}_{q}\right)$, then: \begin{equation} \left\Vert \chi\right\Vert _{p,q}\leq\left\Vert \chi\right\Vert _{\nabla}\label{eq:Supremum norm is dominated by Del norm} \end{equation} \end{prop} Proof: If $\chi\in\tilde{C}\left(\mathbb{Z}_{p},\mathbb{C}_{q}\right)$, then the van der Put series for $\chi$ converges $q$-adically to $\chi$ point-wise over $\mathbb{Z}_{p}$. As such: \begin{align*} \left|\chi\left(\mathfrak{z}\right)\right|_{q} & =\left|\sum_{n=0}^{\infty}\nabla_{p^{n}}\left\{ \chi\right\} \left(\mathfrak{z}\right)\right|_{q}\\ & \leq\sup_{n\geq0}\left|\nabla_{p^{n}}\left\{ \chi\right\} \left(\mathfrak{z}\right)\right|_{q} \end{align*} Taking suprema over $\mathfrak{z}\in\mathbb{Z}_{p}$ gives: \begin{align*} \left\Vert \chi\right\Vert _{p,q} & =\sup_{\mathfrak{z}\in\mathbb{Z}_{p}}\left|\chi\left(\mathfrak{z}\right)\right|_{q}\\ & \leq\sup_{\mathfrak{z}\in\mathbb{Z}_{p}}\sup_{n\geq0}\left|\nabla_{p^{n}}\left\{ \chi\right\} \left(\mathfrak{z}\right)\right|_{q}\\ \left(\sup_{m}\sup_{n}x_{m,n}=\sup_{n}\sup_{m}x_{m,n}\right); & \leq\sup_{n\geq0}\sup_{\mathfrak{z}\in\mathbb{Z}_{p}}\left|\nabla_{p^{n}}\left\{ \chi\right\} \left(\mathfrak{z}\right)\right|_{q}\\ & =\left\Vert \chi\right\Vert _{\nabla} \end{align*} Q.E.D. \begin{lem} $\left(\tilde{C}\left(\mathbb{Z}_{p},\mathbb{C}_{q}\right),\left\Vert \cdot\right\Vert _{\nabla}\right)$ is a non-archimedean normed linear space. In particular, we can identify $\tilde{C}\left(\mathbb{Z}_{p},\mathbb{C}_{q}\right)$ with the image of $\tilde{B}\left(\mathbb{Z}_{p},\mathbb{C}_{q}\right)$ under $S_{p}$. Moreover, $S_{p}$ is then an isometry of $\tilde{C}\left(\mathbb{Z}_{p},\mathbb{C}_{q}\right)$. \end{lem} Proof: By the First Isomorphism Theorem from abstract algebra, $S_{p}$ maps the $\mathbb{C}_{q}$-linear space $\tilde{B}\left(\mathbb{Z}_{p},\mathbb{C}_{q}\right)$ isomorphically onto $\tilde{B}\left(\mathbb{Z}_{p},\mathbb{C}_{q}\right)/\textrm{Ker}\left(S_{p}\right)$. Since $\left\Vert \cdot\right\Vert _{\nabla}$ is a semi-norm on $\tilde{B}\left(\mathbb{Z}_{p},\mathbb{C}_{q}\right)$, and because the set of non-zero $\chi\in\tilde{B}\left(\mathbb{Z}_{p},\mathbb{C}_{q}\right)$ possessing $\left\Vert \chi\right\Vert _{\nabla}=0$ is equal to $\textrm{Ker}\left(S_{p}\right)$, the non-archimedean semi-norm $\left\Vert \cdot\right\Vert _{\nabla}$ is then a norm on the quotient space $\tilde{B}\left(\mathbb{Z}_{p},\mathbb{C}_{q}\right)/\textrm{Ker}\left(S_{p}\right)$. To conclude, we need only show that $\tilde{B}\left(\mathbb{Z}_{p},\mathbb{C}_{q}\right)/\textrm{Ker}\left(S_{p}\right)$ can be identified with $\tilde{C}\left(\mathbb{Z}_{p},\mathbb{C}_{q}\right)$. To do so, let $\chi\in\tilde{C}\left(\mathbb{Z}_{p},\mathbb{C}_{q}\right)$ be arbitrary. Then, $\chi$ is an element of $B\left(\mathbb{Z}_{p},\mathbb{C}_{q}\right)$. As we saw in the previous subsection (viz., \textbf{Theorem \ref{thm:S_p}}), $S_{p}$ is the identity map on $\tilde{C}\left(\mathbb{Z}_{p},\mathbb{C}_{q}\right)$. Consequently, $S_{p}\left\{ \chi\right\} =\chi$, and so, $\chi\in\tilde{B}\left(\mathbb{Z}_{p},\mathbb{C}_{q}\right)$. For the other direction, let $\chi$ and $\eta$ be two elements of $\tilde{B}\left(\mathbb{Z}_{p},\mathbb{C}_{q}\right)$ which belong to the same equivalence class in $\tilde{B}\left(\mathbb{Z}_{p},\mathbb{C}_{q}\right)/\textrm{Ker}\left(S_{p}\right)$. Then, $S_{p}\left\{ \chi-\eta\right\} $ is the zero function. By the linearity of $S_{p}$, this means $S_{p}\left\{ \chi\right\} =S_{p}\left\{ \eta\right\} $. So, $S_{p}\left\{ \chi\right\} $ and $S_{p}\left\{ \eta\right\} $ have the same van der Put series, and that series defines an element of $B\left(\mathbb{Z}_{p},\mathbb{C}_{q}\right)$. Since this series converges everywhere point-wise, the van der Put identity (\textbf{Proposition \ref{prop:vdP identity}}) shows it is an element of $\tilde{C}\left(\mathbb{Z}_{p},\mathbb{C}_{q}\right)$. Thus, the two spaces are equivalent. Q.E.D. \begin{thm} $\left(\tilde{C}\left(\mathbb{Z}_{p},\mathbb{C}_{q}\right),\left\Vert \cdot\right\Vert _{\nabla}\right)$ is a non-archimedean Banach algebra over $\mathbb{C}_{q}$, with point-wise multiplication of functions as the algebra multiplication identity. Additionally, $\left(\tilde{C}\left(\mathbb{Z}_{p},\mathbb{C}_{q}\right),\left\Vert \cdot\right\Vert _{\nabla}\right)$ contains $\left(C\left(\mathbb{Z}_{p},\mathbb{C}_{q}\right),\left\Vert \cdot\right\Vert _{\nabla}\right)$ as a sub-algebra. \end{thm} Proof: To see that $\left(\tilde{C}\left(\mathbb{Z}_{p},\mathbb{C}_{q}\right),\left\Vert \cdot\right\Vert _{\nabla}\right)$ is a normed algebra, let $\chi,\eta\in\tilde{C}\left(\mathbb{Z}_{p},\mathbb{C}_{q}\right)$. Using (\ref{eq:Quasi-derivation identity for del}) and truncation notation, we have that: \begin{align*} \left\Vert \chi\cdot\eta\right\Vert _{\nabla} & =\sup_{n\geq0}\left\Vert \nabla_{p^{n}}\left\{ \chi\cdot\eta\right\} \right\Vert _{p,q}\\ & =\sup_{n\geq0}\left\Vert \chi_{n-1}\cdot\nabla_{p^{n}}\left\{ \eta\right\} +\eta_{n}\cdot\nabla_{p^{n}}\left\{ \chi\right\} \right\Vert _{p,q}\\ \left(\left\Vert \cdot\right\Vert _{p,q}\textrm{ is non-arch.}\right); & \leq\max\left\{ \sup_{m\geq0}\left\Vert \chi_{m-1}\cdot\nabla_{p^{m}}\left\{ \eta\right\} \right\Vert _{p,q},\sup_{n\geq0}\left\Vert \eta_{n}\cdot\nabla_{p^{n}}\left\{ \chi\right\} \right\Vert _{p,q}\right\} \end{align*} For each $m$ and $n$, the function $\chi_{m-1}$, $\nabla_{p^{m}}\left\{ \eta\right\} $, $\eta_{n}$, and $\nabla_{p^{n}}\left\{ \chi\right\} $ are continuous functions. Then, because $\left(C\left(\mathbb{Z}_{p},\mathbb{C}_{q}\right),\left\Vert \cdot\right\Vert _{p,q}\right)$ is a Banach algebra, we can write: \begin{align*} \left\Vert \chi\cdot\eta\right\Vert _{\nabla} & \leq\max\left\{ \sup_{m\geq0}\left(\left\Vert \chi_{m-1}\right\Vert _{p,q}\left\Vert \nabla_{p^{m}}\left\{ \eta\right\} \right\Vert _{p,q}\right),\sup_{n\geq0}\left(\left\Vert \eta_{n}\right\Vert _{p,q}\left\Vert \nabla_{p^{n}}\left\{ \chi\right\} \right\Vert _{p,q}\right)\right\} \end{align*} Since $\chi_{m-1}$ is rising-continuous, \textbf{Proposition \ref{prop:continuous embeds in rising-continuous}} tells us that $\left\Vert \chi_{m-1}\right\Vert _{p,q}\leq\left\Vert \chi_{m-1}\right\Vert _{\nabla}$. Moreover, because $\chi_{m-1}$ is the restriction of $\chi$ to inputs mod $p^{m-1}$, we have: \begin{align*} \left\Vert \chi_{m-1}\right\Vert _{\nabla} & =\sup_{k\geq0}\sup_{\mathfrak{z}\in\mathbb{Z}_{p}}\left|\nabla_{p^{k}}\left\{ \chi_{m-1}\right\} \left(\mathfrak{z}\right)\right|_{q}\\ & \leq\sup_{k\geq0}\sup_{\mathfrak{z}\in\mathbb{Z}_{p}}\left|\chi_{m-1}\left(\left[\mathfrak{z}\right]_{p^{k}}\right)-\chi_{m-1}\left(\left[\mathfrak{z}\right]_{p^{k-1}}\right)\right|_{q}\\ & \leq\sup_{k\geq0}\sup_{\mathfrak{z}\in\mathbb{Z}_{p}}\left|\chi\left(\left[\left[\mathfrak{z}\right]_{p^{k}}\right]_{p^{m-1}}\right)-\chi_{m-1}\left(\left[\left[\mathfrak{z}\right]_{p^{k-1}}\right]_{p^{m-1}}\right)\right|_{q} \end{align*} Here: \begin{equation} \left[\left[\mathfrak{z}\right]_{p^{k}}\right]_{p^{m-1}}=\left[\mathfrak{z}\right]_{p^{\min\left\{ k,m-1\right\} }} \end{equation} and so: \begin{align*} \left\Vert \chi_{m-1}\right\Vert _{\nabla} & =\sup_{k\geq0}\sup_{\mathfrak{z}\in\mathbb{Z}_{p}}\left|\nabla_{p^{k}}\left\{ \chi_{m-1}\right\} \left(\mathfrak{z}\right)\right|_{q}\\ & \leq\sup_{k\geq0}\sup_{\mathfrak{z}\in\mathbb{Z}_{p}}\left|\chi_{m-1}\left(\left[\mathfrak{z}\right]_{p^{k}}\right)-\chi_{m-1}\left(\left[\mathfrak{z}\right]_{p^{k-1}}\right)\right|_{q}\\ & \leq\sup_{k\geq0}\sup_{\mathfrak{z}\in\mathbb{Z}_{p}}\underbrace{\left|\chi\left(\left[\mathfrak{z}\right]_{p^{\min\left\{ k,m-1\right\} }}\right)-\chi\left(\left[\mathfrak{z}\right]_{p^{\min\left\{ k-1,m-1\right\} }}\right)\right|_{q}}_{0\textrm{ when }k\geq m}\\ & \leq\sup_{0\leq k<m}\sup_{\mathfrak{z}\in\mathbb{Z}_{p}}\left|\chi\left(\left[\mathfrak{z}\right]_{p^{k}}\right)-\chi\left(\left[\mathfrak{z}\right]_{p^{k-1}}\right)\right|_{q}\\ & \leq\sup_{k\geq0}\sup_{\mathfrak{z}\in\mathbb{Z}_{p}}\left|\chi\left(\left[\mathfrak{z}\right]_{p^{k}}\right)-\chi\left(\left[\mathfrak{z}\right]_{p^{k-1}}\right)\right|_{q}\\ & =\left\Vert \chi\right\Vert _{\nabla} \end{align*} Applying this argument to the $\eta_{n}$ yields: \begin{equation} \left\Vert \eta_{n}\right\Vert _{p,q}\leq\left\Vert \eta_{n}\right\Vert _{\nabla}\leq\left\Vert \eta\right\Vert _{\nabla} \end{equation} and hence: \begin{align*} \left\Vert \chi\cdot\eta\right\Vert _{\nabla} & \leq\max\left\{ \sup_{m\geq0}\left(\left\Vert \chi_{m-1}\right\Vert _{p,q}\left\Vert \nabla_{p^{m}}\left\{ \eta\right\} \right\Vert _{p,q}\right),\sup_{n\geq0}\left(\left\Vert \eta_{n}\right\Vert _{p,q}\left\Vert \nabla_{p^{n}}\left\{ \chi\right\} \right\Vert _{p,q}\right)\right\} \\ & \leq\max\left\{ \left\Vert \chi\right\Vert _{\nabla}\sup_{m\geq0}\left\Vert \nabla_{p^{m}}\left\{ \eta\right\} \right\Vert _{p,q},\left\Vert \eta\right\Vert _{\nabla}\sup_{n\geq0}\left\Vert \nabla_{p^{n}}\left\{ \chi\right\} \right\Vert _{p,q}\right\} \\ & =\max\left\{ \left\Vert \chi\right\Vert _{\nabla}\left\Vert \eta\right\Vert _{\nabla},\left\Vert \eta\right\Vert _{\nabla}\left\Vert \chi\right\Vert _{\nabla}\right\} \\ & =\left\Vert \chi\right\Vert _{\nabla}\left\Vert \eta\right\Vert _{\nabla} \end{align*} Thus, $\left(\tilde{C}\left(\mathbb{Z}_{p},\mathbb{C}_{q}\right),\left\Vert \cdot\right\Vert _{\nabla}\right)$ is a normed algebra. The inequality from \textbf{Proposition \ref{prop:continuous embeds in rising-continuous}} then shows that $\left(\tilde{C}\left(\mathbb{Z}_{p},\mathbb{C}_{q}\right),\left\Vert \cdot\right\Vert _{\nabla}\right)$ contains $\left(C\left(\mathbb{Z}_{p},\mathbb{C}_{q}\right),\left\Vert \cdot\right\Vert _{\nabla}\right)$ as a sub-algebra. To conclude, we need only establish the completeness of $\left(\tilde{C}\left(\mathbb{Z}_{p},\mathbb{C}_{q}\right),\left\Vert \cdot\right\Vert _{\nabla}\right)$. We do this using \textbf{Proposition \ref{prop:series characterization of a Banach space}}: it suffices to prove that every sequence in $\tilde{C}\left(\mathbb{Z}_{p},\mathbb{C}_{q}\right)$ which tends to $0$ in $\nabla$-norm is summable to an element of $\tilde{C}\left(\mathbb{Z}_{p},\mathbb{C}_{q}\right)$. So, let $\left\{ \chi_{n}\right\} _{n\geq0}$ be a sequence\footnote{This is \emph{not }an instance truncation notation. I apologize for this abuse of notation.} in $\tilde{C}\left(\mathbb{Z}_{p},\mathbb{C}_{q}\right)$ which tends to $0$ in $\nabla$-norm. Then, by \textbf{Proposition \ref{prop:series characterization of a Banach space}}, upon letting $M$ and $N$ be arbitrary positive integers with $M\leq N$, we can write: \begin{equation} \left\Vert \sum_{n=M}^{N}\chi_{n}\left(\mathfrak{z}\right)\right\Vert _{p,q}\leq\left\Vert \sum_{n=M}^{N}\chi_{n}\left(\mathfrak{z}\right)\right\Vert _{\nabla}\leq\max_{n\geq M}\left\Vert \chi_{n}\right\Vert _{\nabla} \end{equation} Here, the upper bound tends to $0$ as $M\rightarrow\infty$. Consequently the series $\sum_{n=0}^{\infty}\chi_{n}\left(\mathfrak{z}\right)$ converges $q$-adically to a sum $X\left(\mathfrak{z}\right)$, and, moreover, this convergence is \emph{uniform} in $\mathfrak{z}\in\mathbb{Z}_{p}$. Since the $\chi_{n}$s are rising-continuous, we have: \begin{equation} S_{p}\left\{ \sum_{n=0}^{N}\chi_{n}\right\} \left(\mathfrak{z}\right)=\sum_{n=0}^{N}S_{p}\left\{ \chi_{n}\right\} \left(\mathfrak{z}\right)=\sum_{n=0}^{N}\chi_{n}\left(\mathfrak{z}\right) \end{equation} The uniform convergence of the $n$-sum justifies interchanging limits with $S_{p}$: \[ X\left(\mathfrak{z}\right)=\lim_{N\rightarrow\infty}\sum_{n=0}^{N}\chi_{n}=\lim_{N\rightarrow\infty}S_{p}\left\{ \sum_{n=0}^{N}\chi_{n}\right\} \left(\mathfrak{z}\right)=S_{p}\left\{ \lim_{N\rightarrow\infty}\sum_{n=0}^{N}\chi_{n}\right\} \left(\mathfrak{z}\right)=S_{p}\left\{ X\right\} \left(\mathfrak{z}\right) \] Consequently, $X=S_{p}\left\{ X\right\} $. This proves that $X$ is rising-continuous. So, $\sum_{n=0}^{\infty}\chi_{n}\left(\mathfrak{z}\right)$ converges in $\tilde{C}\left(\mathbb{Z}_{p},\mathbb{C}_{q}\right)$, establishing the completeness of $\left(\tilde{C}\left(\mathbb{Z}_{p},\mathbb{C}_{q}\right),\left\Vert \cdot\right\Vert _{\nabla}\right)$. Q.E.D. \begin{problem} Although we shall not explore it here, I think it would be interesting to investigate the behavior of $\tilde{C}\left(\mathbb{Z}_{p},\mathbb{C}_{q}\right)$ under linear operators $T_{\phi}:\tilde{C}\left(\mathbb{Z}_{p},\mathbb{C}_{q}\right)\rightarrow B\left(\mathbb{Z}_{p},\mathbb{C}_{q}\right)$ of the form: \begin{equation} T_{\phi}\left\{ \chi\right\} \left(\mathfrak{z}\right)\overset{\textrm{def}}{=}\chi\left(\phi\left(\mathfrak{z}\right)\right),\textrm{ }\forall\chi\in\tilde{C}\left(\mathbb{Z}_{p},\mathbb{C}_{q}\right),\textrm{ }\forall\mathfrak{z}\in\mathbb{Z}_{p}\label{eq:Definition of T_phi of Chi} \end{equation} where $\phi:\mathbb{Z}_{p}\rightarrow\mathbb{Z}_{p}$. The simplest $T_{\phi}$s are translation operator\index{translation operator}s: \begin{equation} \tau_{\mathfrak{a}}\left\{ \chi\right\} \left(\mathfrak{z}\right)\overset{\textrm{def}}{=}\chi\left(\mathfrak{z}+\mathfrak{a}\right)\label{eq:Definition of the translation operator} \end{equation} where $\mathfrak{a}$ is a fixed element of $\mathbb{Z}_{p}$. The van der Put series (\ref{eq:van der Point series for one point function at 0}) given for $\mathbf{1}_{0}\left(\mathfrak{z}\right)$ in Subsection \ref{subsec:3.1.6 Monna-Springer-Integration} shows that $\mathbf{1}_{0}\left(\mathfrak{z}\right)\in\tilde{C}\left(\mathbb{Z}_{p},\mathbb{C}_{q}\right)$, and that $\tau_{n}\left\{ \mathbf{1}_{0}\right\} =\mathbf{1}_{n}$ is in $\tilde{C}\left(\mathbb{Z}_{p},\mathbb{C}_{q}\right)$ for all $n\geq0$. However, observe that for any $\mathfrak{a}\in\mathbb{Z}_{p}^{\prime}$, $\mathbf{1}_{\mathfrak{a}}\left(\mathfrak{z}\right)$ vanishes on $\mathbb{N}_{0}$, and as a consequence, $S_{p}\left\{ \mathbf{1}_{\mathfrak{a}}\right\} $ is identically $0$; that is to say, $\mathbf{1}_{\mathfrak{a}}\left(\mathfrak{z}\right)$ is rising-continuous \emph{if and only if }$\mathfrak{a}\in\mathbb{N}_{0}$. This shows that $\tilde{C}\left(\mathbb{Z}_{p},\mathbb{C}_{q}\right)$ is not closed under translations. Aside from being a curiosity in its own right, studying operators like (\ref{eq:Definition of T_phi of Chi}) is also relevant to our investigations of Hydra maps. Choosing $\phi=B_{p}$, where: \begin{equation} B_{p}\left(\mathfrak{z}\right)\overset{\mathbb{Z}_{p}}{=}\begin{cases} 0 & \textrm{if }\mathfrak{z}=0\\ \frac{\mathfrak{z}}{1-p^{\lambda_{p}\left(\mathfrak{z}\right)}} & \textrm{if }\mathfrak{z}\in\mathbb{N}_{1}\\ \mathfrak{z} & \textrm{if }\mathfrak{z}\in\mathbb{Z}_{p}^{\prime} \end{cases} \end{equation} is $B_{p}$ as defined as (\ref{eq:Definition of B projection function}), observe that the functional equation (\ref{eq:Chi_H B functional equation}) can be written as: \[ T_{B_{p}}\left\{ \chi_{H}\right\} \left(\mathfrak{z}\right)=\chi_{H}\left(B_{p}\left(\mathfrak{z}\right)\right)=\begin{cases} \chi_{H}\left(0\right) & \textrm{if }\mathfrak{z}=0\\ \frac{\chi_{H}\left(\mathfrak{z}\right)}{1-M_{H}\left(\mathfrak{z}\right)} & \textrm{else} \end{cases} \] where, by the arguments shown in Chapter 2, $M_{H}\left(\mathfrak{z}\right)\overset{\mathbb{Z}_{q}}{=}0$ for all $\mathfrak{z}\in\mathbb{Z}_{p}^{\prime}$ ( every such $\mathfrak{z}\in\mathbb{Z}_{p}^{\prime}$ has infinitely many non-zero $p$-adic digits). As such, understanding the action of the operator $T_{B_{p}}$ may be of use to understanding the periodic points of $H$. \end{problem} \vphantom{} We now move to investigate the units of $\tilde{C}\left(\mathbb{Z}_{p},\mathbb{C}_{q}\right)$. \begin{prop}[\textbf{Unit criteria for} $\tilde{C}\left(\mathbb{Z}_{p},\mathbb{C}_{q}\right)$] \label{prop:criteria for being a rising-continuous unit}\ \vphantom{} I. Let $\chi\in\tilde{C}\left(\mathbb{Z}_{p},K\right)$. Then, $\chi$ is a unit of $\tilde{C}\left(\mathbb{Z}_{p},K\right)$ whenever: \begin{equation} \inf_{n\in\mathbb{N}_{0}}\left|\chi\left(n\right)\right|_{q}>0\label{eq:Sufficient condition for a rising continuous function to have a reciprocal} \end{equation} \vphantom{} II. Let $\chi\in C\left(\mathbb{Z}_{p},K\right)$. Then, $\chi$ is a unit of $\tilde{C}\left(\mathbb{Z}_{p},K\right)$ if and only if $\chi$ has no zeroes. \end{prop} Proof: I. Let $\chi\in\tilde{C}\left(\mathbb{Z}_{p},K\right)$. \begin{align*} \left\Vert \frac{1}{\chi}\right\Vert _{\nabla} & =\sup_{n\geq0}\left\Vert \nabla_{p^{n}}\left\{ \frac{1}{\chi}\right\} \right\Vert _{\nabla}\\ & =\max\left\{ \left|\frac{1}{\chi\left(0\right)}\right|_{q},\sup_{n\geq1}\sup_{\mathfrak{z}\in\mathbb{Z}_{p}}\left|\frac{1}{\chi\left(\left[\mathfrak{z}\right]_{p^{n}}\right)}-\frac{1}{\chi\left(\left[\mathfrak{z}\right]_{p^{n-1}}\right)}\right|_{q}\right\} \\ & =\max\left\{ \left|\frac{1}{\chi\left(0\right)}\right|_{q},\sup_{n\geq1}\sup_{\mathfrak{z}\in\mathbb{Z}_{p}}\left|\frac{\nabla_{p^{n}}\left\{ \chi\right\} \left(\mathfrak{z}\right)}{\chi\left(\left[\mathfrak{z}\right]_{p^{n}}\right)\chi\left(\left[\mathfrak{z}\right]_{p^{n-1}}\right)}\right|_{q}\right\} \\ & \leq\max\left\{ \left|\frac{1}{\chi\left(0\right)}\right|_{q},\sup_{m\geq0}\sup_{\mathfrak{z}\in\mathbb{Z}_{p}}\left|\nabla_{p^{m}}\left\{ \chi\right\} \left(\mathfrak{z}\right)\right|\sup_{k\geq1}\sup_{\mathfrak{z}\in\mathbb{Z}_{p}}\left|\chi\left(\left[\mathfrak{z}\right]_{p^{k}}\right)\right|_{q}^{-1}\cdot\sup_{n\geq1}\sup_{\mathfrak{z}\in\mathbb{Z}_{p}}\left|f\left(\left[\mathfrak{z}\right]_{p^{n-1}}\right)\right|_{q}^{-1}\right\} \\ & \leq\max\left\{ \left|\frac{1}{\chi\left(0\right)}\right|_{q},\frac{\left\Vert \chi\right\Vert _{\nabla}}{\inf_{n\geq1}\left|\chi\left(n\right)\right|_{q}^{2}}\right\} \end{align*} which will be finite whenever $\inf_{n\in\mathbb{N}_{0}}\left|\chi\left(n\right)\right|_{q}>0$. \vphantom{} II. If $\chi\in C\left(\mathbb{Z}_{p},K\right)$ has no zeroes, then $1/\chi\in C\left(\mathbb{Z}_{p},K\right)$. Since $C\left(\mathbb{Z}_{p},K\right)\subset\tilde{C}\left(\mathbb{Z}_{p},K\right)$, we then have that $1/\chi\in\tilde{C}\left(\mathbb{Z}_{p},K\right)$. Conversely, if $1/\chi\in C\left(\mathbb{Z}_{p},K\right)$ has a zero, then $1/\chi\notin C\left(\mathbb{Z}_{p},K\right)$. If $1/\chi$ were in $\tilde{C}\left(\mathbb{Z}_{p},K\right)$, then since the norm on $\tilde{C}\left(\mathbb{Z}_{p},K\right)$ dominates the norm on $C\left(\mathbb{Z}_{p},K\right)$, $1/\chi\in\tilde{C}\left(\mathbb{Z}_{p},K\right)$ force $1/\chi$ to be an element of $C\left(\mathbb{Z}_{p},K\right)$, which would be a contradiction. Q.E.D. \begin{lem} Let $\chi\in\tilde{C}\left(\mathbb{Z}_{p},K\right)$. Then, $1/\chi\in\tilde{C}\left(\mathbb{Z}_{p},K\right)$ if and only if $\chi_{N}^{-1}=1/\chi_{N}$ converges point-wise in $K$ as $N\rightarrow\infty$, in which case, the point-wise limit is equal to $1/\chi$. \end{lem} Proof: Since $\chi_{N}^{-1}$ is the $N$th truncation of $1/\chi_{N}$, we have that $1/\chi\in\tilde{C}\left(\mathbb{Z}_{p},K\right)$ if and only if $\chi_{N}^{-1}$ converges point-wise to a limit, in which case that limit is necessarily $1/\chi$. Q.E.D. \vphantom{} This shows that we can simplify the study of units of $\tilde{C}\left(\mathbb{Z}_{p},K\right)$ by considering their truncations, which is very nice, seeing as the $\left(p,q\right)$-adic continuity of an $N$th truncation allows us to employ the techniques $\left(p,q\right)$-adic Fourier analysis. Our next inversion criterion takes the form of a characterization of the rate at which the truncations of a rising continuous function converge in the limit, and has the appeal of being both necessary \emph{and }sufficient. \begin{lem}[\textbf{The Square Root Lemma}\index{Square Root Lemma}] \label{lem:square root lemma}Let $\chi\in\tilde{C}\left(\mathbb{Z}_{p},K\right)$, let $\mathfrak{z}\in\mathbb{Z}_{p}^{\prime}$, and let $\mathfrak{c}\in K\backslash\chi\left(\mathbb{N}_{0}\right)$. Then, $\chi\left(\mathfrak{z}\right)=\mathfrak{c}$ if and only if: \begin{equation} \liminf_{n\rightarrow\infty}\frac{\left|\chi\left(\left[\mathfrak{z}\right]_{p^{n}}\right)-\mathfrak{c}\right|_{q}}{\left|\nabla_{p^{n}}\left\{ \chi\right\} \left(\mathfrak{z}\right)\right|_{q}^{1/2}}<\infty\label{eq:Square Root Lemma} \end{equation} \end{lem} \begin{rem} Since this is a biconditional statement, it is equivalent to its inverse, which is: \emph{ \begin{equation} \chi\left(\mathfrak{z}\right)\neq\mathfrak{c}\Leftrightarrow\lim_{n\rightarrow\infty}\frac{\left|\chi\left(\left[\mathfrak{z}\right]_{p^{n}}\right)-\mathfrak{c}\right|_{q}}{\left|\nabla_{p^{n}}\left\{ \chi\right\} \left(\mathfrak{z}\right)\right|_{q}^{1/2}}=\infty\label{eq:Square Root Lemma - Inverse} \end{equation} }This is because the $\liminf$ \[ \liminf_{n\rightarrow\infty}\frac{\left|\chi\left(\left[\mathfrak{z}\right]_{p^{n}}\right)-\mathfrak{c}\right|_{q}}{\left|\nabla_{p^{n}}\left\{ \chi\right\} \left(\mathfrak{z}\right)\right|_{q}^{1/2}}=\infty \] forces the true limit to exist and be equal to $\infty$. \end{rem} Proof: Let $\chi$, $\mathfrak{z}$, and $\mathfrak{c}$ be as given. We will prove (\ref{eq:Square Root Lemma - Inverse}). I. Suppose $\chi\left(\mathfrak{z}\right)\neq\mathfrak{c}$. Then, it must be that $\liminf_{n\rightarrow\infty}\left|\chi\left(\left[\mathfrak{z}\right]_{p^{n}}\right)-\mathfrak{c}\right|_{q}>0$. On the other hand, since $\chi\in\tilde{C}\left(\mathbb{Z}_{p},K\right)$, the denominator of (\ref{eq:Square Root Lemma - Inverse}) tends to $0$. This shows (\ref{eq:Square Root Lemma - Inverse}) holds whenever $\chi\left(\mathfrak{z}\right)\neq\mathfrak{c}$. \vphantom{} II. Suppose: \[ \lim_{n\rightarrow\infty}\frac{\left|\chi\left(\left[\mathfrak{z}\right]_{p^{n}}\right)-\mathfrak{c}\right|_{q}}{\left|\nabla_{p^{n}}\left\{ \chi\right\} \left(\mathfrak{z}\right)\right|_{q}^{1/2}}=\infty \] Now, letting $\mathfrak{y}\in\mathbb{Z}_{p}^{\prime}$ be arbitrary, observe that we have the formal identity: \begin{align*} \frac{1}{\chi\left(\mathfrak{y}\right)-\mathfrak{c}} & =\frac{1}{\chi\left(0\right)-\mathfrak{c}}+\sum_{n=1}^{\infty}\left(\frac{1}{\chi\left(\left[\mathfrak{y}\right]_{p^{n}}\right)-\mathfrak{c}}-\frac{1}{\chi\left(\left[\mathfrak{y}\right]_{p^{n-1}}\right)-\mathfrak{c}}\right)\\ & =\frac{1}{\chi\left(0\right)-\mathfrak{c}}-\sum_{n=1}^{\infty}\frac{\overbrace{\chi\left(\left[\mathfrak{y}\right]_{p^{n}}\right)-\chi\left(\left[\mathfrak{y}\right]_{p^{n-1}}\right)}^{\nabla_{p^{n}}\left\{ \chi\right\} \left(\mathfrak{y}\right)}}{\left(\chi\left(\left[\mathfrak{y}\right]_{p^{n}}\right)-\mathfrak{c}\right)\left(\chi\left(\left[\mathfrak{y}\right]_{p^{n-1}}\right)-\mathfrak{c}\right)} \end{align*} Note that we needn't worry about $1/\left(\chi\left(0\right)-\mathfrak{c}\right)$, since $\mathfrak{c}$ was given to not be of the form $\mathfrak{c}=\chi\left(n\right)$ for any $n\in\mathbb{N}_{0}$. Now, observe that $\chi\left(\mathfrak{z}\right)\neq\mathfrak{c}$ if and only if the above series converges in $K$ at $\mathfrak{y}=\mathfrak{z}$. This, in turn, occurs if and only if: \[ \lim_{n\rightarrow\infty}\left|\frac{\nabla_{p^{n}}\left\{ \chi\right\} \left(\mathfrak{z}\right)}{\left(\chi\left(\left[\mathfrak{z}\right]_{p^{n}}\right)-\mathfrak{c}\right)\left(\chi\left(\left[\mathfrak{z}\right]_{p^{n-1}}\right)-\mathfrak{c}\right)}\right|_{q}=0 \] and hence, if and only if: \begin{equation} \lim_{n\rightarrow\infty}\left|\frac{\left(\chi\left(\left[\mathfrak{z}\right]_{p^{n}}\right)-\mathfrak{c}\right)\left(\chi\left(\left[\mathfrak{z}\right]_{p^{n-1}}\right)-\mathfrak{c}\right)}{\nabla_{p^{n}}\left\{ \chi\right\} \left(\mathfrak{z}\right)}\right|_{q}=\infty\label{eq:Desired Limit} \end{equation} So, in order to show that (\ref{eq:Square Root Lemma - Inverse}) implies $\chi\left(\mathfrak{z}\right)\neq\mathfrak{c}$, we need only show that (\ref{eq:Square Root Lemma - Inverse}) implies (\ref{eq:Desired Limit}). Here, note that: \[ \chi\left(\left[\mathfrak{z}\right]_{p^{n-1}}\right)=\chi\left(\left[\mathfrak{z}\right]_{p^{n}}\right)-\nabla_{p^{n}}\left\{ \chi\right\} \left(\mathfrak{z}\right) \] and so: \begin{align*} \left|\frac{\left(\chi\left(\left[\mathfrak{z}\right]_{p^{n}}\right)-\mathfrak{c}\right)\left(\chi\left(\left[\mathfrak{z}\right]_{p^{n-1}}\right)-\mathfrak{c}\right)}{\nabla_{p^{n}}\left\{ \chi\right\} \left(\mathfrak{z}\right)}\right|_{q} & =\left|\frac{\left(\chi\left(\left[\mathfrak{z}\right]_{p^{n}}\right)-\mathfrak{c}\right)\left(\chi\left(\left[\mathfrak{z}\right]_{p^{n}}\right)-\mathfrak{c}-\nabla_{p^{n}}\left\{ \chi\right\} \left(\mathfrak{z}\right)\right)}{\nabla_{p^{n}}\left\{ \chi\right\} \left(\mathfrak{z}\right)}\right|_{q}\\ & =\frac{\left|\chi\left(\left[\mathfrak{z}\right]_{p^{n}}\right)-\mathfrak{c}\right|_{q}}{\left|\nabla_{p^{n}}\left\{ \chi\right\} \left(\mathfrak{z}\right)\right|_{q}^{1/2}}\frac{\left|\chi\left(\left[\mathfrak{z}\right]_{p^{n}}\right)-\mathfrak{c}-\nabla_{p^{n}}\left\{ \chi\right\} \left(\mathfrak{z}\right)\right|_{q}}{\left|\nabla_{p^{n}}\left\{ \chi\right\} \left(\mathfrak{z}\right)\right|_{q}^{1/2}}\\ & \geq\frac{\left|\chi\left(\left[\mathfrak{z}\right]_{p^{n}}\right)-\mathfrak{c}\right|_{q}}{\left|\nabla_{p^{n}}\left\{ \chi\right\} \left(\mathfrak{z}\right)\right|_{q}^{1/2}}\left|\frac{\left|\chi\left(\left[\mathfrak{z}\right]_{p^{n}}\right)-\mathfrak{c}\right|_{q}}{\left|\nabla_{p^{n}}\left\{ \chi\right\} \left(\mathfrak{z}\right)\right|_{q}^{1/2}}-\left|\nabla_{p^{n}}\left\{ \chi\right\} \left(\mathfrak{z}\right)\right|_{q}^{1/2}\right| \end{align*} Now, since $\chi\in\tilde{C}\left(\mathbb{Z}_{p},K\right)$, we have $\left|\nabla_{p^{n}}\left\{ \chi\right\} \left(\mathfrak{z}\right)\right|_{q}^{1/2}\rightarrow0$ as $n\rightarrow\infty$. On the other hand, for this part of the proof, we assumed that: \begin{equation} \lim_{n\rightarrow\infty}\frac{\left|\chi\left(\left[\mathfrak{z}\right]_{p^{n}}\right)-\mathfrak{c}\right|_{q}}{\left|\nabla_{p^{n}}\left\{ \chi\right\} \left(\mathfrak{z}\right)\right|_{q}^{1/2}}=\infty \end{equation} Consequently, since $\mathfrak{z}$ is fixed: \begin{align*} \lim_{n\rightarrow\infty}\left|\frac{\left(\chi\left(\left[\mathfrak{z}\right]_{p^{n}}\right)-\mathfrak{c}\right)\left(\chi\left(\left[\mathfrak{z}\right]_{p^{n-1}}\right)-\mathfrak{c}\right)}{\nabla_{p^{n}}\left\{ \chi\right\} \left(\mathfrak{z}\right)}\right|_{q} & \geq\lim_{n\rightarrow\infty}\frac{\left|\chi\left(\left[\mathfrak{z}\right]_{p^{n}}\right)-\mathfrak{c}\right|_{q}}{\left|\nabla_{p^{n}}\left\{ \chi\right\} \left(\mathfrak{z}\right)\right|_{q}^{1/2}}\left|\frac{\left|\chi\left(\left[\mathfrak{z}\right]_{p^{n}}\right)-\mathfrak{c}\right|_{q}}{\left|\nabla_{p^{n}}\left\{ \chi\right\} \left(\mathfrak{z}\right)\right|_{q}^{1/2}}-\left|\nabla_{p^{n}}\left\{ \chi\right\} \left(\mathfrak{z}\right)\right|_{q}^{1/2}\right|\\ & =\infty\cdot\left|\infty-0\right|\\ & =\infty \end{align*} which is (\ref{eq:Desired Limit}). Q.E.D. \vphantom{} This time, the moral is that $\mathfrak{c}\in K\backslash\chi\left(\mathbb{N}_{0}\right)$ is in the image of $\chi$ if and only if there is a subsequence of $n$s along which $\left|\chi\left(\left[\mathfrak{z}\right]_{p^{n}}\right)-\mathfrak{c}\right|_{q}$ tends to $0$ \emph{at least as quickly as} $\left|\nabla_{p^{n}}\left\{ \chi\right\} \left(\mathfrak{z}\right)\right|_{q}^{1/2}$. \begin{prop} Let $\chi\in\tilde{C}\left(\mathbb{Z}_{p},K\right)$, let $\mathfrak{z}\in\mathbb{Z}_{p}^{\prime}$, and let $\mathfrak{c}\in K\backslash\chi\left(\mathbb{N}_{0}\right)$. If: \[ \liminf_{n\rightarrow\infty}\frac{\left|\chi\left(\left[\mathfrak{z}\right]_{p^{n}}\right)-\mathfrak{c}\right|_{q}}{\left|\nabla_{p^{n}}\left\{ \chi\right\} \left(\mathfrak{z}\right)\right|_{q}^{1/2}}<\infty \] then: \[ \liminf_{n\rightarrow\infty}\frac{\left|\chi\left(\left[\mathfrak{z}\right]_{p^{n}}\right)-\mathfrak{c}\right|_{q}}{\left|\nabla_{p^{n}}\left\{ f\right\} \left(\mathfrak{z}\right)\right|_{q}^{1/2}}>0 \] occurs if and only if, for any strictly increasing sequence $\left\{ n_{j}\right\} _{j\geq1}\subseteq\mathbb{N}_{0}$ so that: \[ \lim_{j\rightarrow\infty}\frac{\left|\chi\left(\left[\mathfrak{z}\right]_{p^{n_{j}}}\right)-\mathfrak{c}\right|_{q}}{\left|\nabla_{p^{n_{j}}}\left\{ \chi\right\} \left(\mathfrak{z}\right)\right|_{q}^{1/2}}=\liminf_{n\rightarrow\infty}\frac{\left|\chi\left(\left[\mathfrak{z}\right]_{p^{n}}\right)-\mathfrak{c}\right|_{q}}{\left|\nabla_{p^{n}}\left\{ \chi\right\} \left(\mathfrak{z}\right)\right|_{q}^{1/2}} \] the congruences: \begin{equation} v_{q}\left(\nabla_{p^{n_{j+1}}}\left\{ \chi\right\} \left(\mathfrak{z}\right)\right)\overset{2}{\equiv}v_{q}\left(\nabla_{p^{n_{j}}}\left\{ \chi\right\} \left(\mathfrak{z}\right)\right) \end{equation} hold for all sufficiently large $j$; that is, the parity of the $q$-adic valuation of $\nabla_{p^{n_{j}}}\left\{ \chi\right\} \left(\mathfrak{z}\right)$ becomes constant for all sufficiently large $j$. \end{prop} Proof: Suppose: \[ \liminf_{n\rightarrow\infty}\frac{\left|\chi\left(\left[\mathfrak{z}\right]_{p^{n}}\right)-\mathfrak{c}\right|_{q}}{\left|\nabla_{p^{n}}\left\{ \chi\right\} \left(\mathfrak{z}\right)\right|_{q}^{1/2}}>0 \] In particular, let us write the value of this $\liminf$ as $q^{C}$, where $C$ is a positive real number. Letting the $n_{j}$s be any sequence as described above, by way of contradiction, suppose there are two disjoint subsequences of $j$s so that $v_{q}\left(\nabla_{p^{n_{j}}}\left\{ \chi\right\} \left(\mathfrak{z}\right)\right)$ is always even for all $j$ in the first subsequence and is always odd for all $j$ in the second subsequence. Now, for \emph{any} $j$, let $v_{j}$ be the positive integer so that $\left|\chi\left(\left[\mathfrak{z}\right]_{p^{n_{j}}}\right)-\mathfrak{c}\right|_{q}=q^{-v_{j}}$. Then: \begin{align*} q^{C}=\lim_{j\rightarrow\infty}q^{\frac{1}{2}v_{q}\left(\nabla_{p^{n_{j}}}\left\{ \chi\right\} \left(\mathfrak{z}\right)\right)}\left|\chi\left(\left[\mathfrak{z}\right]_{p^{n_{j}}}\right)-\mathfrak{c}\right|_{q} & =\lim_{j\rightarrow\infty}q^{\frac{1}{2}v_{q}\left(\nabla_{p^{n_{j}}}\left\{ \chi\right\} \left(\mathfrak{z}\right)\right)-v_{j}} \end{align*} and hence: \begin{equation} C=\lim_{j\rightarrow\infty}\left(\frac{1}{2}v_{q}\left(\nabla_{p^{n_{j}}}\left\{ \chi\right\} \left(\mathfrak{z}\right)\right)-v_{j}\right)\label{eq:Limit formula for K} \end{equation} for \emph{both }subsequences of $j$s. For the $j$s in the first subsequence (where $v_{q}\left(\nabla_{p^{n_{j}}}\left\{ \chi\right\} \left(\mathfrak{z}\right)\right)$ is always even), the limit on the right hand side is of a subsequence in the set $\mathbb{Z}$. For the $j$s in the second subsequence (where $v_{q}\left(\nabla_{p^{n_{j}}}\left\{ \chi\right\} \left(\mathfrak{z}\right)\right)$ is always odd), however, the limit on the right hand side is that of a sequence in $\mathbb{Z}+\frac{1}{2}$. But this is impossible: it forces $C$ to be an element of both $\mathbb{Z}$ and $\mathbb{Z}+\frac{1}{2}$, and those two sets are disjoint. Thus, the parity of $v_{q}\left(\nabla_{p^{n_{j}}}\left\{ \chi\right\} \left(\mathfrak{z}\right)\right)$ must become constant for all sufficiently large $j$ whenever $C>-\infty$. Q.E.D. \vphantom{} Given the criteria described in these last few results, it seems natural to introduce the following definitions: \begin{defn} Let $\chi\in\tilde{C}\left(\mathbb{Z}_{p},K\right)$. A pair $\left(\mathfrak{z},\mathfrak{c}\right)$, where $\mathfrak{z}\in\mathbb{Z}_{p}^{\prime}$ and $\mathfrak{c}\in\chi\left(\mathbb{Z}_{p}\right)\backslash\chi\left(\mathbb{N}_{0}\right)$ is said to be a \index{quick approach}\textbf{quick approach of $\chi$ / quickly approached by $\chi$ }whenever: \[ \liminf_{n\rightarrow\infty}\frac{\left|\chi\left(\left[\mathfrak{z}\right]_{p^{n}}\right)-\mathfrak{c}\right|_{q}}{\left|\nabla_{p^{n}}\left\{ \chi\right\} \left(\mathfrak{z}\right)\right|_{q}^{1/2}}=0 \] We say that the pair is a\textbf{ slow approach of $\chi$ / slowly approached by $\chi$ }whenever the $\liminf$ is finite and non-zero. \end{defn} \begin{question} Let $\chi\in\tilde{C}\left(\mathbb{Z}_{p},K\right)$. What can be said about the speed of approach of $\chi$ to $\mathfrak{c}$ at any given $\mathfrak{z}\in\mathbb{Z}_{p}^{\prime}$? Can there be infinitely many (or at most finitely many\textemdash or none?) points of quick approach? Slow approach? Are there other properties we can use to characterize points of quick or slow approach?\footnote{In Chapter 4, we will show that, for every semi-basic, contracting $2$-Hydra map which fixes $0$, every point in $\chi_{H}\left(\mathbb{Z}_{p}\right)\backslash\chi_{H}\left(\mathbb{N}_{0}\right)$ is a point of quick approach $\chi_{H}$. The more general case of a $p$-Hydra map likely also holds, but I did not have sufficient time to investigate it fully.} \end{question} \begin{rem} To share a bit of my research process, although I discovered the notion of quasi-integrability back in the second half of 2020, it wasn't until the 2021 holiday season that I began to unify my $\left(p,q\right)$-adic work in earnest. With the Correspondence Principle on my mind, I sought to find ways to characterize the image of a $\left(p,q\right)$-adic function. In particular, I found myself gravitating toward the question: \emph{given a rising-continuous function $\chi:\mathbb{Z}_{p}\rightarrow\mathbb{Z}_{q}$, when is $1/\chi$ rising-continuous? }Given a $\chi_{H}$, investigating the singular behavior of $1/\left(\chi_{H}\left(\mathfrak{z}\right)-x\right)$ where $x\in\mathbb{Z}\backslash\left\{ 0\right\} $ would seem to be a natural way to exploit the Correspondence Principle, because an $x\in\mathbb{Z}\backslash\left\{ 0\right\} $ would be a periodic point of $H$ if and only if $1/\left(\chi_{H}\left(\mathfrak{z}\right)-x\right)$ had a singularity for some $\mathfrak{z}\in\mathbb{Z}_{p}^{\prime}$. I drew inspiration from W. Cherry's notes on non-archimedean function theory \cite{Cherry non-archimedean function theory notes}, and the existence of a $p$-adic Nevanlinna theory\textemdash Nevanlinna theory being one of analytic function theorists' most powerful tools for keeping track of the poles of meromorphic functions. Due to the (present) lack of a meaningful notion of an \emph{analytic }$\left(p,q\right)$-adic function, however, I had to go down another route to investigate reciprocals of rising-continuous functions. Things began to pick up speed once I realized that the set of rising-continuous functions could be realized as a non-archimedean Banach algebra under point-wise multiplication. In that context, exploring the Correspondence Principle becomes extremely natural: it is the quest to understand those values of $x\in\mathbb{Z}\backslash\left\{ 0\right\} $ for which $\chi_{H}\left(\mathfrak{z}\right)-x$ is not a unit of the Banach algebra of rising-continuous functions. \end{rem} \subsubsection{A Very Brief Foray Into Berkovitch Space\index{Berkovitch space}} To give yet another reason why $\left(p,q\right)$-adic analysis has been considered ``uninteresting'' by the mathematical community at large: their zeroes do not enjoy any special properties. Case in point: \begin{prop} \label{prop:(p,q)-adic functions are "uninteresting"}Let $A$ be any closed subset of $\mathbb{Z}_{p}$. Then, there is an $f\in C\left(\mathbb{Z}_{p},K\right)$ such that $f\left(\mathfrak{z}\right)=0$ if and only if $\mathfrak{z}\in A$. \end{prop} Proof: Let $A$ be a closed subset of $\mathbb{Z}_{p}$. For any $\mathfrak{z}\in\mathbb{Z}_{p}\backslash A$, the Hausdorff distance: \begin{equation} d\left(\mathfrak{z},A\right)\overset{\textrm{def}}{=}\inf_{\mathfrak{a}\in A}\left|\mathfrak{z}-\mathfrak{a}\right|_{p}\label{eq:Definition of the distance of a point in Z_p from a set in Z_p} \end{equation} of $\mathfrak{z}$ form $A$ will be of the form $\frac{1}{p^{n}}$ for some $n\in\mathbb{N}_{0}$. Consequently, we can define a function $g:\mathbb{Z}_{p}\backslash A\rightarrow\mathbb{N}_{0}$ so that: \begin{equation} d\left(x,A\right)=p^{-g\left(x\right)},\textrm{ }\forall x\in\mathbb{Z}_{p}\backslash A \end{equation} Then, the function: \begin{equation} f\left(x\right)\overset{\textrm{def}}{=}\left[x\notin A\right]q^{g\left(x\right)}\label{eq:Function with zeroes on an arbitrary closed subset of Z_p} \end{equation} will be continuous, and will have $A$ as its vanishing set. Q.E.D. \vphantom{} In this short sub-subsection, we deal with some topological properties of rising-continuous functions which can be proven using \textbf{Berkovitch spaces}. For the uninitiated\textemdash such as myself\textemdash Berkovitch space is the capstone of a series of turbulent efforts by algebraic and arithmetic geometers in the second half of the twentieth century which had as their \emph{ide-fixe }the hope of circumventing the Achilles' heel of the native theory of $\left(p,p\right)$-adic\footnote{i.e., $f:\mathbb{C}_{p}\rightarrow\mathbb{C}_{p}$.} analytic functions: the abysmal failure of Weierstrass' germ-based notions of analytic continuation\index{analytic continuation}. Because of the $p$-adics' ultrametric structure, every $\mathfrak{z}$ in the disk $\left\{ \mathfrak{z}\in\mathbb{C}_{p}:\left|\mathfrak{z}-\mathfrak{a}\right|_{p}<r\right\} $ is, technically, at the center of said disk; here $\mathfrak{a}\in\mathbb{C}_{p}$ and $r>0$. Along with the $p$-adics' stark convergence properties (viz. the principles of ultrametric analysis on \pageref{fact:Principles of Ultrametric Analysis}) this strange property of non-archimedean disks leads to the following unexpected behaviors in a $p$-adic power series $f\left(\mathfrak{z}\right)=\sum_{n=0}^{\infty}\mathfrak{a}_{n}\mathfrak{z}^{n}$. Here, $r$ denotes $f$'s radius of convergence $r>0$. \begin{enumerate} \item Either $f\left(\mathfrak{z}\right)$ converges at every $\mathfrak{z}\in\mathbb{C}_{p}$ with $\left|\mathfrak{z}\right|_{p}=r$, or diverges at every such $\mathfrak{z}$. \item Developing $f\left(\mathfrak{z}\right)$ into a power series at any $\mathfrak{z}_{0}\in\mathbb{C}_{p}$ with $\left|\mathfrak{z}_{0}\right|_{p}<r$ results in a power series whose radius of convergence is equal to $r$. So, no analytic continuation is possible purely by way of rejiggering $f$'s power series. \end{enumerate} The construction of Berkovitch space provides a larger space containing $\mathbb{C}_{p}$ which is Hausdorff, and\textemdash crucially, unlike the totally disconnected space $\mathbb{C}_{p}$\textemdash is actually arc-wise connected.\footnote{Let $X$ be a topological space. Two points $x,y\in X$ are said to be \textbf{topologically distinguishable }if there exists an open set $U\subseteq X$ which contains one and only one of $x$ or $y$. $X$ is then said to be \textbf{arc-wise connected }whenever, for any topologically distinguishable points $x,y\in X$, there is an embedding $\alpha:\left[0,1\right]\rightarrow X$ (continuous and injective) such that $\alpha\left(0\right)=x$ and $\alpha\left(1\right)=y$; such an $\alpha$ is called an \textbf{arc}.} The idea\footnote{I base my exposition here on what is given in \cite{The Berkovitch Space Paper}.} (a minor bit of trepanation) is to replace points in $\mathbb{C}_{p}$ with \index{semi-norm}\emph{semi-norms}. This procedure holds, in fact, for an arbitrary Banach algebra over a non-archimedean field. Before going forward, let me mention that I include the following definitions (taken from \cite{The Berkovitch Space Paper}) primarily for the reader's edification, provided the reader is the sort of person who finds stratospheric abstractions edifying. Meanwhile, the rest of us mere mortals can safely skip over to the proofs at the end of this sub-subsection. \begin{defn} Let $K$ be an algebraically closed non-archimedean field, complete with respect to its ultrametric. Let $A$ be a commutative $K$-Banach algebra with identity $1_{A}$, and with norm $\left\Vert \cdot\right\Vert $. A function $\rho:A\times A\rightarrow\left[0,\infty\right)$ is an \textbf{algebra semi-norm }if\index{Banach algebra!semi-norm}, in addition to being a semi-norm on $A$ when $A$ is viewed as a $K$-Banach space, we have that: \vphantom{} I. $\rho\left(1_{A}\right)=1$; \vphantom{} II. $\rho\left(f\cdot g\right)\leq\rho\left(f\right)\cdot\rho\left(g\right)$ for all $f,g\in A$. $\rho$ is said to be \textbf{multiplicative} whenever\index{semi-norm!multiplicative} (II) holds with equality for all $f,g\in A$. The \textbf{multiplicative spectrum}\index{Banach algebra!multiplicative spectrum}\textbf{ of $A$}, denoted $\mathcal{M}\left(A,\left\Vert \cdot\right\Vert \right)$, is the set of multiplicative semi-norms on $A$ which are continuous with respect to the topology defined by $\left\Vert \cdot\right\Vert $; i.e., for each $\rho\in\mathcal{M}\left(A,\left\Vert \cdot\right\Vert \right)$, there is a real constant $K\geq0$ so that: \begin{equation} \rho\left(f\right)\leq K\left\Vert f\right\Vert ,\textrm{ }\forall f\in A\label{eq:multiplicative semi-norm thing for Berkovitch space} \end{equation} \end{defn} \vphantom{} Using this bizarre setting, algebraic geometers then construct affine spaces over $K$. \begin{defn} We write $\mathbb{A}_{K}^{1}$ to denote the space of all multiplicative semi-norms on the space $K\left[X\right]$ of formal polynomials in the indeterminate $X$ with coefficients in $K$. $\mathbb{A}_{K}^{1}$ is called the \textbf{affine line over $K$}\index{affine line over K@affine line over \textbf{$K$}}. \end{defn} \begin{fact} Both $\mathcal{M}\left(A,\left\Vert \cdot\right\Vert \right)$ and $\mathbb{\mathbb{A}}_{K}^{1}$ are made into topological spaces with the topology of point-wise convergence. \end{fact} \begin{thm} $\mathcal{M}\left(A,\left\Vert \cdot\right\Vert \right)$ is Hausdorff and compact. $\mathbb{A}_{K}^{1}$ is Hausdorff, locally compact, and arc-wise connected \cite{The Berkovitch Space Paper}. \end{thm} \begin{defn} For each $t\in A$, write $\theta_{t}$ to denote the $K$-algbra homomorphism $\theta_{t}:K\left[X\right]\longrightarrow A$ defined by $\theta_{t}\left(X\right)=t$. For each $t\in A$, the \index{Gelfand transform}\textbf{Gelfand transform of $t$} is then defined to be the map $\mathcal{G}_{t}:\mathcal{M}\left(A,\left\Vert \cdot\right\Vert \right)\rightarrow\mathbb{A}_{K}^{1}$ given by: \begin{equation} \mathcal{G}_{t}\left\{ \rho\right\} \overset{\textrm{def}}{=}\rho\circ\theta_{t}\label{Gelfand transform of t} \end{equation} $\mathcal{G}_{t}\left\{ \rho\right\} $ is then a multiplicative semi-norm on $K\left[X\right]$ for all $\rho$ and $t$. The map $\mathcal{G}$ which sends $t\in A$ to the map $\mathcal{G}_{t}$ is called the \textbf{Gelfand transform}. \end{defn} \begin{thm} $\mathcal{G}_{t}$ is continuous for every $t\in A$. \end{thm} \vphantom{} The point of this, apparently, is that $\mathcal{G}$ allows any $t\in A$ to be interpreted as the continuous function $\mathcal{G}_{t}$. Now for some more definitions: \begin{defn} Given $t\in A$: \vphantom{} I. We write $\textrm{s}\left(t\right)$ to denote the \textbf{scalar spectrum }of $t$: the set of $\lambda\in K$ for which $t-\lambda1_{A}$ is not invertible in $A$. \vphantom{} II. We write $\sigma\left(t\right)$ to denote the \textbf{spectrum }of $t$: the image of $\mathcal{M}\left(A,\left\Vert \cdot\right\Vert \right)$ under $\mathcal{G}_{t}$. \end{defn} \vphantom{} Since $K$ is a Banach algebra over itself (with $\left|\cdot\right|_{K}$ as the norm), $\mathcal{M}\left(K,\left|\cdot\right|_{K}\right)$ consists of a single semi-norm: the absolute value on $K$. This allows $K$ to be embedded in $\mathbb{A}_{K}^{1}$ by way of the Gelfand transform $\mathcal{G}$, identifying any $\mathfrak{a}\in K$ with $\mathcal{G}_{\mathfrak{a}}$. Since $\mathcal{M}\left(K,\left|\cdot\right|_{K}\right)$ contains only the single element $\left|\cdot\right|_{K}$, the image of $\mathcal{G}_{\mathfrak{a}}$ ($\sigma\left(\mathfrak{a}\right)$\textemdash the spectrum of $\mathfrak{a}$) is the semi-norm which sends $P\in K\left[X\right]$ to $\left|\theta_{\mathfrak{a}}\left(P\right)\right|_{K}$. That is, for any $\mathfrak{a}\in K$, we we identify with $\mathfrak{a}$ the semi-norm $P\mapsto\left|\theta_{\mathfrak{a}}\left(P\right)\right|_{K}$ on $K\left[X\right]$. \begin{defn} The \textbf{closed disk }in $\mathbb{A}_{K}^{1}$ centered at $\mathfrak{a}\in K$ of radius $r>0$ is defined as the closure in $\mathbb{A}_{K}^{1}$ (with respect to point-wise convergence) of the set of semi-norms $P\in K\left[X\right]\mapsto\left|\theta_{\mathfrak{z}}\left(P\right)\right|_{K}$ as $\mathfrak{z}$ varies over all elements of $K$ with $\left|\mathfrak{z}-\mathfrak{a}\right|_{K}\leq r$. \end{defn} \begin{thm}[\textbf{The First Spectral Radius Formula }\cite{The Berkovitch Space Paper}] Let $A$ be a commutative $K$-Banach algebra with identity $1_{A}$, and with norm $\left\Vert \cdot\right\Vert $, and let $t\in A$. Then: \vphantom{} I. $\sigma\left(t\right)$ is non-empty and compact. \vphantom{} II. The smallest closed disk in $\mathbb{A}_{K}^{1}$ centered at $0$ which contains $\sigma\left(t\right)$ has radius: \begin{equation} \lim_{n\rightarrow\infty}\left\Vert t^{n}\right\Vert ^{1/n} \end{equation} \vphantom{} III. $K\cap\sigma\left(t\right)=s\left(t\right)$, where by $K$, we mean the copy of $K$ embedded in $\mathbb{A}_{K}^{1}$ as described above. \end{thm} \vphantom{} We now recall some basic definitions from topology: \begin{defn} Let $X$ and $Y$ be topological spaces. A map $M:X\rightarrow Y$ is said to be: \vphantom{} I.\textbf{ Proper},\textbf{ }if $M^{-1}\left(C\right)$ is compact in $X$ for all compact sets $C\subseteq Y$. \vphantom{} II. \textbf{Closed}, if $M\left(S\right)$ is closed in $Y$ for all closed sets $S\subseteq X$. \end{defn} \begin{fact} Let $X$ and $Y$ be topological spaces, and let $M:X\rightarrow Y$ be continuous. If $X$ is compact and $Y$ is Hausdorff, then $M$ is both proper and closed. \end{fact} \begin{prop} $s\left(t\right)$ is compact in $K$. \end{prop} Proof: Let $t\in A$ be arbitrary. Since $\mathcal{G}_{t}:\mathcal{M}\left(A,\left\Vert \cdot\right\Vert \right)\rightarrow\mathbb{A}_{K}^{1}$ is continuous, since $\mathcal{M}\left(A,\left\Vert \cdot\right\Vert \right)$ is compact, and since $\mathbb{A}_{K}^{1}$ is Hausdorff, it then follows that $\mathcal{G}_{t}$ is both proper and closed. Consequently, since $K$ is closed, so is its copy in $\mathbb{A}_{K}^{1}$. Since $\sigma\left(t\right)$ is compact in $\mathbb{A}_{K}^{1}$, this means that $s\left(t\right)=K\cap\sigma\left(t\right)$ is compact in $\mathbb{A}_{K}^{1}\cap K$, and hence in $K$. Q.E.D. \vphantom{} The non-algebraic reader can now resume paying attention. \begin{lem} \label{lem:compactness of the image of a rising-continuous function}Let $\chi\in\tilde{C}\left(\mathbb{Z}_{p},K\right)$. Then, $\chi\left(\mathbb{Z}_{p}\right)$ is a compact subset of $K$. \end{lem} Proof: First, we embed $\chi$ in $\tilde{C}\left(\mathbb{Z}_{p},\mathbb{C}_{q}\right)$. Since $\tilde{C}\left(\mathbb{Z}_{p},\mathbb{C}_{q}\right)$ is a commutative Banach algebra over the algebraically closed, metrically complete non-archimedean field $\mathbb{C}_{q}$, it follows by the previous proposition that the scalar spectrum of $\chi$ is compact in $\mathbb{C}_{q}$. Since the scalar spectrum is precisely the set of $\mathfrak{c}\in\mathbb{C}_{q}$ for which $\frac{1}{\chi\left(\mathfrak{z}\right)-\mathfrak{c}}\in\tilde{C}\left(\mathbb{Z}_{p},\mathbb{C}_{q}\right)$, it then follows that the scalar spectrum of $\chi$\textemdash a compact set in $\mathbb{C}_{q}$\textemdash is equal to $\chi\left(\mathbb{Z}_{p}\right)$, the image of $\chi$ over $\mathbb{Z}_{p}$. Since $K$ is closed in $\mathbb{C}_{q}$, the set $\chi\left(\mathbb{Z}_{p}\right)\cap K$ (an intersection of a compact set and a closed set) is compact in both $\mathbb{C}_{q}$ and $K$. Since, by definition of $K$, $\chi\left(\mathbb{Z}_{p}\right)=\chi\left(\mathbb{Z}_{p}\right)\cap K$, we have that $\chi\left(\mathbb{Z}_{p}\right)$ is compact in $K$. Q.E.D. \vphantom{} Before we get to our next results, note the following: \begin{fact} Let $U$ be any clopen subset of $\mathbb{Z}_{p}$. Then, the indicator function for $U$ is continuous as a function from $\mathbb{Z}_{p}$ to any topological ring (where the ``$1$'' taken by the function over $U$ is the multiplicative identity element of the ring). \end{fact} \begin{thm} Let $\chi\in\tilde{C}\left(\mathbb{Z}_{p},K\right)$. Then, $\chi\left(U\right)$ is a compact subset of $K$ for all clopen sets $U\subseteq\mathbb{Z}_{p}$. \end{thm} Proof: Let $U$ be any clopen subset of $\mathbb{Z}_{p}$ with non-empty complement. Then, let $V\overset{\textrm{def}}{=}\mathbb{Z}_{p}\backslash U$ be the complement of $U$, and let $\mathfrak{a}\in\chi\left(U\right)$ be any value attained by $\chi$ on $U$. Then, define: \begin{equation} f\left(\mathfrak{z}\right)\overset{\textrm{def}}{=}\left[\mathfrak{x}\in U\right]\chi\left(\mathfrak{z}\right)+\mathfrak{a}\left[\mathfrak{x}\in V\right]\label{eq:Berkovitch space f construction} \end{equation} Since both $V$ and $U$ are clopen sets, their indicator functions $\left[\mathfrak{x}\in U\right]$ and $\left[\mathfrak{x}\in V\right]$ are continuous functions from $\mathbb{Z}_{p}$ to $K$. Since $\tilde{C}\left(\mathbb{Z}_{p},K\right)$ is an algebra, we have that $f\in\tilde{C}\left(\mathbb{Z}_{p},K\right)$. As such, by \textbf{Lemma \ref{lem:compactness of the image of a rising-continuous function}}, $f\left(\mathbb{Z}_{p}\right)$ is compact in $K$. Observing that: \begin{align*} f\left(\mathbb{Z}_{p}\right) & =f\left(U\right)\cup f\left(V\right)\\ & =\chi\left(U\right)\cup\left\{ \mathfrak{a}\right\} \\ \left(\mathfrak{a}\in\chi\left(U\right)\right); & =\chi\left(U\right) \end{align*} we then have that $\chi\left(U\right)$ is compact in $K$, as desired. Q.E.D. \begin{thm} Let $\chi\in\tilde{C}\left(\mathbb{Z}_{p},K\right)$, and suppose there is a set $S\subseteq\mathbb{Z}_{p}$ and a real constant $v\geq0$ so that $\left|\chi\left(\mathfrak{z}\right)\right|_{q}>q^{-v}$ for all $\mathfrak{z}\in S$. Then, $\chi\left(U\right)$ is compact in $K$ for any set $U\subseteq S$ which is closed in $\mathbb{Z}_{p}$. \end{thm} \begin{rem} In other words, if $\chi$ is bounded away from zero on some set $S$, then the restriction of $\chi$ to $S$ is a proper map. \end{rem} Proof: Since $U$ is closed in $\mathbb{Z}_{p}$, as we saw in \textbf{Proposition \ref{prop:(p,q)-adic functions are "uninteresting"}}, there is a function $g:\mathbb{Z}_{p}\rightarrow\mathbb{N}_{0}$ so that $h:\mathbb{Z}_{p}\rightarrow K$ defined by: \begin{equation} h\left(\mathfrak{z}\right)\overset{\textrm{def}}{=}\left[\mathfrak{z}\notin U\right]q^{g\left(\mathfrak{z}\right)}\label{eq:h construction for Berkovitch space} \end{equation} is continuous and vanishes if and only if $\mathfrak{z}\in U$. Then, letting: \begin{equation} f\left(\mathfrak{z}\right)\overset{\textrm{def}}{=}\left[\mathfrak{x}\in U\right]\chi\left(\mathfrak{z}\right)+q^{v}h\left(\mathfrak{z}\right)\label{eq:Second f construction for Berkovitch space} \end{equation} we have that $f\in\tilde{C}\left(\mathbb{Z}_{p},K\right)$, and that: \begin{equation} f\left(\mathbb{Z}_{p}\right)=\chi\left(U\right)\cup q^{v+g\left(V\right)} \end{equation} is compact in $K$, where $q^{v+g\left(V\right)}=\left\{ q^{v+n}:n\in g\left(V\right)\right\} $. Since $\left|\chi\left(\mathfrak{z}\right)\right|_{q}>q^{-v}$ for all $\mathfrak{z}\in S$, we have; \begin{equation} \chi\left(U\right)\subseteq\chi\left(S\right)\subseteq K\backslash q^{v}\mathbb{Z}_{q} \end{equation} As such: \begin{align*} \left(K\backslash q^{v}\mathbb{Z}_{q}\right)\cap f\left(\mathbb{Z}_{p}\right) & =\left(\chi\left(U\right)\cup q^{v+g\left(V\right)}\right)\cap\left(K\backslash q^{v}\mathbb{Z}_{q}\right)\\ & =\left(\chi\left(U\right)\cap\left(K\backslash q^{v}\mathbb{Z}_{q}\right)\right)\cup\left(q^{v+g\left(V\right)}\cap K\cap\left(q^{v}\mathbb{Z}_{q}\right)^{c}\right)\\ & =\underbrace{\left(\chi\left(U\right)\cap\left(K\backslash q^{v}\mathbb{Z}_{q}\right)\right)}_{\chi\left(U\right)}\cup\underbrace{\left(q^{v+g\left(V\right)}\cap K\cap\left(q^{v}\mathbb{Z}_{q}\right)^{c}\right)}_{\varnothing}\\ \left(q^{v+g\left(V\right)}\cap\left(q^{v}\mathbb{Z}_{q}\right)^{c}=\varnothing\right); & =\chi\left(U\right) \end{align*} Since $K\backslash q^{v}\mathbb{Z}_{q}$ is closed and $f\left(\mathbb{Z}_{p}\right)$ is compact, $\left(K\backslash q^{v}\mathbb{Z}_{q}\right)\cap f\left(\mathbb{Z}_{p}\right)=\chi\left(U\right)$ is compact, as desired. Q.E.D. \begin{lem} $f\in C\left(\mathbb{Z}_{p},K\right)$ is a unit if and only if $f$ has no zeroes. \end{lem} Proof: I. If $f$ is a unit, then $\frac{1}{f}$ is continuous, and hence, $f$ can have no zeroes. \vphantom{} II. Conversely, suppose $f$ has no zeroes. Since $f$ is continuous, its image $f\left(\mathbb{Z}_{p}\right)$ is a compact subset of $K$ (even if $K$ itself is \emph{not} locally compact). Since $K$ is a topological field, the reciprocation map $\iota\left(\mathfrak{y}\right)\overset{\textrm{def}}{=}\frac{1}{\mathfrak{y}}$ is a self-homeomorphism of $K\backslash\left\{ 0\right\} $. Here, we observe that the pre-image of a set $Y$ under $1/f$ is then equal to $f^{-1}\left(\iota^{-1}\left(Y\right)\right)$. So, let $Y$ be an arbitrary closed subset of $K$. Then, the pre-image of $Y$ under $1/f$ is equal to: \begin{equation} f^{-1}\left(\iota^{-1}\left(Y\right)\right)=f^{-1}\left(\iota^{-1}\left(Y\right)\cap f\left(\mathbb{Z}_{p}\right)\right) \end{equation} Since $f\left(\mathbb{Z}_{p}\right)$ is compact and bounded away from zero: \begin{equation} \iota^{-1}\left(Y\right)\cap f\left(\mathbb{Z}_{p}\right) \end{equation} is closed and bounded away from zero, and so, by the continuity of $f$, $f^{-1}\left(\iota^{-1}\left(Y\right)\cap f\left(\mathbb{Z}_{p}\right)\right)$ is closed. Thus, the pre-image of every closed subset of $K$ under $1/f$ is closed; this proves that $1/f$ is continuous whenever $f$ has no zeroes. Q.E.D. \begin{lem} \label{lem:3.17}$\chi\in\tilde{C}\left(\mathbb{Z}_{p},K\right)$ is a unit of $\tilde{C}\left(\mathbb{Z}_{p},K\right)$ whenever the following conditions hold: \vphantom{} I. $\chi$ has no zeroes. \vphantom{} II. For each $\mathfrak{z}\in\mathbb{Z}_{p}$: \begin{equation} \sup_{n\geq1}\left|\frac{\chi\left(\left[\mathfrak{z}\right]_{p^{n}}\right)}{\chi\left(\left[\mathfrak{z}\right]_{p^{n-1}}\right)}\right|_{q}<\infty \end{equation} Note that this bound need not be uniform in $\mathfrak{z}$. \end{lem} Proof: Let $\chi\in\tilde{C}\left(\mathbb{Z}_{p},K\right)$, and suppose $0\notin\chi\left(\mathbb{Z}_{p}\right)$. Then, by the van der Put identity, we have that $\frac{1}{\chi}$ will be rising-continuous if and only if: \[ \lim_{n\rightarrow\infty}\left|\frac{1}{\chi\left(\left[\mathfrak{z}\right]_{p^{n}}\right)}-\frac{1}{\chi\left(\left[\mathfrak{z}\right]_{p^{n-1}}\right)}\right|_{q}=0 \] for all $\mathfrak{z}\in\mathbb{Z}_{p}$, which is the same as: \[ \lim_{n\rightarrow\infty}\left|\frac{\nabla_{p^{n}}\left\{ \chi\right\} \left(\mathfrak{z}\right)}{\chi\left(\left[\mathfrak{z}\right]_{p^{n}}\right)\chi\left(\left[\mathfrak{z}\right]_{p^{n-1}}\right)}\right|_{q}=0 \] By the \textbf{Square-Root Lemma} (\textbf{Lemma \ref{lem:square root lemma}}), since $0$ is not in the image of $\chi$, it must be that: \begin{equation} \liminf_{n\rightarrow\infty}\frac{\left|\chi\left(\left[\mathfrak{z}\right]_{p^{n}}\right)\right|_{q}}{\left|\nabla_{p^{n}}\left\{ \chi\right\} \left(\mathfrak{z}\right)\right|_{q}^{1/2}}=\infty \end{equation} which then forces: \begin{equation} \lim_{n\rightarrow\infty}\frac{\left|\chi\left(\left[\mathfrak{z}\right]_{p^{n}}\right)\right|_{q}}{\left|\nabla_{p^{n}}\left\{ \chi\right\} \left(\mathfrak{z}\right)\right|_{q}^{1/2}}=\infty \end{equation} which forces: \begin{equation} \lim_{n\rightarrow\infty}\frac{\left|\nabla_{p^{n}}\left\{ \chi\right\} \left(\mathfrak{z}\right)\right|_{q}^{1/2}}{\left|\chi\left(\left[\mathfrak{z}\right]_{p^{n}}\right)\right|_{q}}=0 \end{equation} Now, the condition required for $1/\chi$ to be rising-continuous is: \begin{equation} \lim_{n\rightarrow\infty}\left|\frac{\nabla_{p^{n}}\left\{ \chi\right\} \left(\mathfrak{z}\right)}{\chi\left(\left[\mathfrak{z}\right]_{p^{n}}\right)\chi\left(\left[\mathfrak{z}\right]_{p^{n-1}}\right)}\right|_{q}=0 \end{equation} This can be re-written as: \[ \lim_{n\rightarrow\infty}\frac{\left|\nabla_{p^{n}}\left\{ \chi\right\} \left(\mathfrak{z}\right)\right|_{q}^{1/2}}{\left|\chi\left(\left[\mathfrak{z}\right]_{p^{n}}\right)\right|_{q}}\cdot\frac{\left|\chi\left(\left[\mathfrak{z}\right]_{p^{n}}\right)\right|_{q}}{\left|\chi\left(\left[\mathfrak{z}\right]_{p^{n-1}}\right)\right|_{q}}\cdot\frac{\left|\nabla_{p^{n}}\left\{ \chi\right\} \left(\mathfrak{z}\right)\right|_{q}^{1/2}}{\left|\chi\left(\left[\mathfrak{z}\right]_{p^{n}}\right)\right|_{q}}=0 \] The assumptions on $\chi$, meanwhile, guarantee both: \begin{equation} \frac{\left|\nabla_{p^{n}}\left\{ \chi\right\} \left(\mathfrak{z}\right)\right|_{q}^{1/2}}{\left|\chi\left(\left[\mathfrak{z}\right]_{p^{n}}\right)\right|_{q}}\rightarrow0 \end{equation} and: \begin{equation} \sup_{n\geq1}\left|\frac{\chi\left(\left[\mathfrak{z}\right]_{p^{n}}\right)}{\chi\left(\left[\mathfrak{z}\right]_{p^{n-1}}\right)}\right|_{q}<\infty \end{equation} Consequently, the above limit is zero for each $\mathfrak{z}\in\mathbb{Z}_{p}$. This proves $1/\chi$ is rising-continuous. Q.E.D. \begin{thm} Let $\tilde{C}\left(\mathbb{Z}_{p},\mathbb{T}_{q}\right)$ denote the set of all $\chi\in\tilde{C}\left(\mathbb{Z}_{p},\mathbb{C}_{q}\right)$ for which $\left|\chi\left(\mathfrak{z}\right)\right|_{q}=1$ for all $\mathfrak{z}\in\mathbb{Z}_{p}$, where $\mathbb{T}_{q}$ is the set of all $q$-adic complex numbers with a $q$-adic absolute value of $1$. Then, $\tilde{C}\left(\mathbb{Z}_{p},\mathbb{T}_{q}\right)$ is an abelian group under the operation of point-wise multiplication; the identity element is the constant function $1$. \end{thm} Proof: Every $\chi\in\tilde{C}\left(\mathbb{Z}_{p},\mathbb{T}_{q}\right)$ satisfies the conditions of \textbf{Lemma \ref{lem:3.17}}, because $\left|\chi\left(\mathfrak{z}\right)\right|_{q}=\left|\chi\left(\left[\mathfrak{z}\right]_{p^{n}}\right)\right|_{q}=1$ for all $\mathfrak{z}\in\mathbb{Z}_{p}$ and all $n\in\mathbb{N}_{0}$. Q.E.D. \newpage{} \section{\label{sec:3.3 quasi-integrability}$\left(p,q\right)$-adic Measures and Quasi-Integrability} THROUGHOUT THIS SECTION $p$ AND $q$ DENOTE DISTINCT PRIME NUMBERS. \vphantom{} Despite the pronouncements from Schikhof and Konrad used in this chapter's epigraphs, there is more to $\left(p,q\right)$-adic integration than meets the eye. Consider a function $\hat{\mu}:\hat{\mathbb{Z}}_{p}\rightarrow\overline{\mathbb{Q}}$. Viewing $\overline{\mathbb{Q}}$ as being embedded in $\mathbb{C}_{q}$, observe that if $\sup_{t\in\hat{\mathbb{Z}}_{p}}\left|\hat{\mu}\left(t\right)\right|_{q}<\infty$, we can then make sense of $\hat{\mu}$ as the Fourier-Stieltjes transform of the measure $d\mu\in C\left(\mathbb{Z}_{p},\mathbb{C}_{q}\right)^{\prime}$ defined by the formula: \begin{equation} \int_{\mathbb{Z}_{p}}f\left(\mathfrak{z}\right)d\mu\left(\mathfrak{z}\right)=\sum_{t\in\hat{\mathbb{Z}}_{p}}\hat{f}\left(-t\right)\hat{\mu}\left(t\right),\textrm{ }\forall f\in C\left(\mathbb{Z}_{p},\mathbb{C}_{q}\right) \end{equation} Now, for any $g\in C\left(\mathbb{Z}_{p},\mathbb{C}_{q}\right)$, the $\left(p,q\right)$-adic measure $d\mu\left(\mathfrak{z}\right)=g\left(\mathfrak{z}\right)d\mathfrak{z}$ satisfies: \begin{equation} g\left(\mathfrak{z}\right)\overset{\mathbb{C}_{q}}{=}\sum_{t\in\hat{\mathbb{Z}}_{p}}\hat{\mu}\left(t\right)e^{2\pi i\left\{ t\mathfrak{z}\right\} _{p}},\textrm{ }\forall\mathfrak{z}\in\mathbb{Z}_{p}\label{eq:g in terms of mu hat} \end{equation} because the continuity of $g$ guarantees that $\hat{\mu}\in c_{0}\left(\hat{\mathbb{Z}}_{p},\mathbb{C}_{q}\right)$. For a measure like $d\mu\left(\mathfrak{z}\right)=g\left(\mathfrak{z}\right)d\mathfrak{z}$, the Fourier series on the right-hand side of (\ref{eq:g in terms of mu hat}) makes perfect sense. On the other hand, for an arbitrary bounded function $\hat{\mu}:\hat{\mathbb{Z}}_{p}\rightarrow\mathbb{C}_{q}$, there is no guarantee that the right-hand side of (\ref{eq:g in terms of mu hat})\textemdash viewed as the limit\footnote{I will, at times, refer to (\ref{eq:The Limit of Interest}) as the ``limit of interest''.} of the partial sums: \begin{equation} \lim_{N\rightarrow\infty}\sum_{\left|t\right|_{p}\leq p^{N}}\hat{\mu}\left(t\right)e^{2\pi i\left\{ t\mathfrak{z}\right\} _{p}}\label{eq:The Limit of Interest} \end{equation} \textemdash will even exist for any given $\mathfrak{z}\in\mathbb{Z}_{p}$. In this section, we will investigate the classes of $\left(p,q\right)$-adic measures $d\mu$ so that (\ref{eq:The Limit of Interest}) makes sense for some\textemdash or even all\textemdash values of $\mathfrak{z}\in\mathbb{Z}_{p}$\textemdash even if $\hat{\mu}\notin c_{0}\left(\hat{\mathbb{Z}}_{p},\mathbb{C}_{q}\right)$. Because measures are, by definition, ``integrable'', we can enlarge the tent of $\left(p,q\right)$-adically ``integrable'' functions by including those functions which just so happen to be given by (\ref{eq:The Limit of Interest}) for some bounded $\hat{\mu}$. I call these \textbf{quasi-integrable functions}.\textbf{ }Given a quasi-integrable function\index{quasi-integrability} $\chi:\mathbb{Z}_{p}\rightarrow\mathbb{C}_{q}$, upon identifying $\chi$ with the measure $d\mu$, integration against $\chi$ can then be defined as integration against $d\mu$ by way of the formula: \begin{equation} \int_{\mathbb{Z}_{p}}f\left(\mathfrak{z}\right)\chi\left(\mathfrak{z}\right)d\mathfrak{z}\overset{\mathbb{C}_{q}}{=}\sum_{t\in\hat{\mathbb{Z}}_{p}}\hat{f}\left(-t\right)\hat{\mu}\left(t\right),\textrm{ }\forall f\in C\left(\mathbb{Z}_{p},\mathbb{C}_{q}\right) \end{equation} I have two points to make before we begin. First, \emph{the reader should be aware that} \emph{this chapter contains a large number of qualitative definitions}. As will be explained in detail throughout, this is primarily because, at present, there does not appear to be a way to get desirable conclusions properties out of generic $\left(p,q\right)$-adic functions without relying on explicit formulae and computations thereof, such as (\ref{eq:Definition of right-ended p-adic structural equations}), (\ref{eq:Radial-Magnitude Fourier Resummation Lemma - p-adically distributed case}), and (\ref{eq:reciprocal of the p-adic absolute value of z is quasi-integrable}), and for these explicit computations to be feasible, we need our functions to come pre-equipped with at least \emph{some }concrete algebraic structure. By and by, I will indicate where I have been forced to complicate definitions for the sake of subsequent proof. It is obviously of interest to see how much these definitions can be streamlined and simplified. Secondly and finally, I have to say that there are still many basic technical considerations regarding quasi-integrability and quasi-integrable functions which I have not been able to develop to my satisfaction due to a lack of time and/or cleverness. The relevant questions are stated and discussed on pages \pageref{que:3.3}, \pageref{que:3.2}, pages \pageref{que:3.4} through \pageref{que:3.6}, and page \pageref{que:3.7}. \subsection{\label{subsec:3.3.1 Heuristics-and-Motivations}Heuristics and Motivations} To the extent that I have discussed my ideas with other mathematicians who did not already know me personally, the reactions I have gotten from bringing up frames seems to indicate that my ideas are considered ``difficult'', maybe even radical. But they were difficult for me long before they were difficult for anyone else. It took over a year before I finally understood what was going on. My discoveries of quasi-integrability and the necessity of frames occurred completely by chance. In performing a certain lengthy computation, I seemed to get a different answer every time. It was only after breaking up the computations into multiple steps\textemdash performing each separately and in greater generality\textemdash that I realized what was going on. To that end, rather than pull a Bourbaki, I think it will be best if we first consider the main examples that I struggled with. It's really the natural thing to do. Pure mathematics \emph{is} a natural science, after all. Its empirical data are the patterns we notice and the observations we happen to make. The hypotheses are our latest ideas for a problem, and the experiments are our efforts to see if those ideas happen to bear fruit. In this respect, frames and quasi-integrability should not be seen as platonic ideals, but as my effort to describe and speak clearly about an unusual phenomenon. The examples below are the context in which the reader should first try to understand my ideas, because this was the context in which all of my work was originally done. Instead of saving the best for last, let's go right to heart of the matter and consider the example that caused me the most trouble, by far. First, however, I must introduce the main antagonist to feature in our studies of the shortened $qx+1$ maps. \begin{defn}[$\hat{A}_{q}\left(t\right)$] Let \index{hat{A}{q}@$\hat{A}_{q}$}$q$ be an odd prime. \nomenclature{$\hat{A}_{q}\left(t\right)$}{ }I define $\hat{A}_{q}:\hat{\mathbb{Z}}_{2}\rightarrow\overline{\mathbb{Q}}$ by: \begin{equation} \hat{A}_{q}\left(t\right)\overset{\textrm{def}}{=}\begin{cases} 1 & \textrm{if }t=0\\ \prod_{n=0}^{-v_{2}\left(t\right)-1}\frac{1+qe^{-2\pi i\left(2^{n}t\right)}}{4} & \textrm{else} \end{cases},\textrm{ }\forall t\in\hat{\mathbb{Z}}_{2}\label{eq:Definition of A_q hat} \end{equation} Noting that: \begin{equation} \sup_{t\in\hat{\mathbb{Z}}_{2}}\left|\hat{A}_{q}\left(t\right)\right|_{q}=1 \end{equation} it follows that we can define a $\left(2,q\right)$-adic measure \nomenclature{$dA_{q}$}{ }$dA_{q}\in C\left(\mathbb{Z}_{2},\mathbb{C}_{q}\right)^{\prime}$ by way of the formula: \begin{equation} \int_{\mathbb{Z}_{2}}f\left(\mathfrak{z}\right)dA_{q}\left(\mathfrak{z}\right)\overset{\mathbb{C}_{q}}{=}\sum_{t\in\hat{\mathbb{Z}}_{q}}\hat{f}\left(-t\right)\hat{A}_{q}\left(t\right),\textrm{ }\forall f\in C\left(\mathbb{Z}_{2},\mathbb{C}_{q}\right) \end{equation} As defined, $dA_{q}$ has $\hat{A}_{q}$ as its Fourier-Stieltjes transform\index{$dA_{q}$!Fourier-Stieltjes transf.}. \end{defn} \vphantom{} The $\hat{A}_{q}$s will be of \emph{immen}se importance in Chapter 4, where we will see how the behavior of $dA_{q}$ is for $q=3$ is drastically different from all other odd primes. For now, though, let's focus on the $q=3$ case\textemdash the Collatz Case\textemdash seeing as it provides us an example of a particularly pathological $\left(p,q\right)$-adic measure\textemdash what I call a \textbf{degenerate measure}\textemdash as well as a motivation for the topology-mixing inspiration that birthed my concept of a frame. \begin{prop} \label{prop:Nth partial sum of Fourier series of A_3 hat}\index{hat{A}{3}@$\hat{A}_{3}$} \begin{equation} \sum_{\left|t\right|_{2}\leq2^{N}}\hat{A}_{3}\left(t\right)e^{2\pi i\left\{ t\mathfrak{z}\right\} _{2}}=\frac{3^{\#_{1}\left(\left[\mathfrak{z}\right]_{2^{N}}\right)}}{2^{N}},\textrm{ }\forall\mathfrak{z}\in\mathbb{Z}_{2},\textrm{ }\forall N\geq0\label{eq:Convolution of dA_3 with D_N} \end{equation} where $\#_{1}\left(m\right)\overset{\textrm{def}}{=}\#_{2:1}\left(m\right)$ is the number of $1$s in the binary expansion of $m$. \end{prop} Proof: As will be proven in \textbf{Proposition \ref{prop:Generating function identities}} (page \pageref{prop:Generating function identities}), we have a generating function identity: \begin{equation} \prod_{m=0}^{n-1}\left(1+az^{2^{m}}\right)=\sum_{m=0}^{2^{n}-1}a^{\#_{1}\left(m\right)}z^{m}\label{eq:Product-to-sum identity for number of 1s digits} \end{equation} which holds for any field $\mathbb{F}$ of characteristic $0$ and any $a,z\in\mathbb{F}$, and any $n\geq1$. Consequently: \begin{equation} \prod_{n=0}^{-v_{2}\left(t\right)-1}\frac{1+3e^{-2\pi i\left(2^{n}t\right)}}{4}=\frac{1}{\left|t\right|_{2}^{2}}\sum_{m=0}^{\left|t\right|_{2}-1}3^{\#_{1}\left(m\right)}e^{-2\pi imt} \end{equation} As such, for any $n\geq1$: \begin{align*} \sum_{\left|t\right|_{2}=2^{n}}\hat{A}_{3}\left(t\right)e^{2\pi i\left\{ t\mathfrak{z}\right\} _{2}} & =\sum_{\left|t\right|_{2}=2^{n}}\left(\frac{1}{\left|t\right|_{2}^{2}}\sum_{m=0}^{\left|t\right|_{2}-1}3^{\#_{1}\left(m\right)}e^{-2\pi imt}\right)e^{2\pi i\left\{ t\mathfrak{z}\right\} _{2}}\\ & =\frac{1}{4^{n}}\sum_{m=0}^{2^{n}-1}3^{\#_{1}\left(m\right)}\sum_{\left|t\right|_{2}=2^{n}}e^{2\pi i\left\{ t\left(\mathfrak{z}-m\right)\right\} _{2}}\\ & =\frac{1}{4^{n}}\sum_{m=0}^{2^{n}-1}3^{\#_{1}\left(m\right)}\left(2^{n}\left[\mathfrak{z}\overset{2^{n}}{\equiv}m\right]-2^{n-1}\left[\mathfrak{z}\overset{2^{n-1}}{\equiv}m\right]\right)\\ & =\frac{1}{2^{n}}\sum_{m=0}^{2^{n}-1}3^{\#_{1}\left(m\right)}\left[\mathfrak{z}\overset{2^{n}}{\equiv}m\right]-\frac{1}{2^{n+1}}\sum_{m=0}^{2^{n}-1}3^{\#_{1}\left(m\right)}\left[\mathfrak{z}\overset{2^{n-1}}{\equiv}m\right] \end{align*} Here, note that $m=\left[\mathfrak{z}\right]_{2^{n}}$ is the unique integer $m\in\left\{ 0,\ldots,2^{n}-1\right\} $ for which $\mathfrak{z}\overset{2^{n}}{\equiv}m$ holds true. Thus: \begin{equation} \sum_{m=0}^{2^{n}-1}3^{\#_{1}\left(m\right)}\left[\mathfrak{z}\overset{2^{n}}{\equiv}m\right]=3^{\#_{1}\left(\left[\mathfrak{z}\right]_{2^{n}}\right)} \end{equation} Likewise: \begin{align*} \sum_{m=0}^{2^{n}-1}3^{\#_{1}\left(m\right)}\left[\mathfrak{z}\overset{2^{n-1}}{\equiv}m\right] & =\sum_{m=0}^{2^{n-1}-1}3^{\#_{1}\left(m\right)}\left[\mathfrak{z}\overset{2^{n-1}}{\equiv}m\right]+\sum_{m=2^{n-1}}^{2^{n}-1}3^{\#_{1}\left(m\right)}\left[\mathfrak{z}\overset{2^{n-1}}{\equiv}m\right]\\ & =3^{\#_{1}\left(\left[\mathfrak{z}\right]_{2^{n-1}}\right)}+\sum_{m=0}^{2^{n-1}-1}3^{\#_{1}\left(m+2^{n-1}\right)}\left[\mathfrak{z}\overset{2^{n-1}}{\equiv}m+2^{n-1}\right]\\ & =3^{\#_{1}\left(\left[\mathfrak{z}\right]_{2^{n-1}}\right)}+\sum_{m=0}^{2^{n-1}-1}3^{1+\#_{1}\left(m\right)}\left[\mathfrak{z}\overset{2^{n-1}}{\equiv}m\right]\\ & =3^{\#_{1}\left(\left[\mathfrak{z}\right]_{2^{n-1}}\right)}+3\cdot3^{\#_{1}\left(\left[\mathfrak{z}\right]_{2^{n-1}}\right)}\\ & =4\cdot3^{\#_{1}\left(\left[\mathfrak{z}\right]_{2^{n-1}}\right)} \end{align*} and so: \[ \sum_{\left|t\right|_{2}=2^{n}}\hat{A}_{3}\left(t\right)e^{2\pi i\left\{ t\mathfrak{z}\right\} _{2}}=\frac{3^{\#_{1}\left(\left[\mathfrak{z}\right]_{2^{n}}\right)}}{2^{n}}-\frac{4\cdot3^{\#_{1}\left(\left[\mathfrak{z}\right]_{2^{n-1}}\right)}}{2^{n+1}}=\frac{3^{\#_{1}\left(\left[\mathfrak{z}\right]_{2^{n}}\right)}}{2^{n}}-\frac{3^{\#_{1}\left(\left[\mathfrak{z}\right]_{2^{n-1}}\right)}}{2^{n-1}} \] Consequently: \begin{align*} \sum_{\left|t\right|_{2}\leq2^{N}}\hat{A}_{3}\left(t\right)e^{2\pi i\left\{ t\mathfrak{z}\right\} _{2}} & =\hat{A}_{3}\left(0\right)+\sum_{n=1}^{N}\sum_{\left|t\right|_{2}=2^{n}}\hat{A}_{3}\left(t\right)e^{2\pi i\left\{ t\mathfrak{z}\right\} _{2}}\\ & =1+\sum_{n=1}^{N}\left(\frac{3^{\#_{1}\left(\left[\mathfrak{z}\right]_{2^{n}}\right)}}{2^{n}}-\frac{3^{\#_{1}\left(\left[\mathfrak{z}\right]_{2^{n-1}}\right)}}{2^{n-1}}\right)\\ \left(\textrm{telescoping series}\right); & =1+\frac{3^{\#_{1}\left(\left[\mathfrak{z}\right]_{2^{N}}\right)}}{2^{N}}-\underbrace{3^{\#_{1}\left(\left[\mathfrak{z}\right]_{2^{0}}\right)}}_{=3^{0}=1}\\ \left(\left[\mathfrak{z}\right]_{2^{0}}=0\right); & =\frac{3^{\#_{1}\left(\left[\mathfrak{z}\right]_{2^{N}}\right)}}{2^{N}} \end{align*} Q.E.D. \vphantom{} With this formula, the reader will see exactly what I when I say the measure $dA_{3}$ is ``\index{$dA_{3}$!degeneracy}degenerate''. \begin{prop} \label{prop:dA_3 is a degenerate measure}We have: \begin{equation} \lim_{N\rightarrow\infty}\sum_{\left|t\right|_{2}\leq2^{N}}\hat{A}_{3}\left(t\right)e^{2\pi i\left\{ t\mathfrak{z}\right\} _{2}}\overset{\mathbb{C}_{3}}{=}0,\textrm{ }\forall\mathfrak{z}\in\mathbb{Z}_{2}^{\prime}\label{eq:Degeneracy of dA_3 on Z_2 prime} \end{equation} and: \begin{equation} \lim_{N\rightarrow\infty}\sum_{\left|t\right|_{2}\leq2^{N}}\hat{A}_{3}\left(t\right)e^{2\pi i\left\{ t\mathfrak{z}\right\} _{2}}\overset{\mathbb{C}}{=}0,\textrm{ }\forall\mathfrak{z}\in\mathbb{N}_{0}\label{eq:Degeneracy of dA_3 on N_0} \end{equation} \end{prop} \begin{rem} That is to say, the upper limit occurs in the topology of $\mathbb{C}_{3}$ while the lower limit occurs in the topology of $\mathbb{C}$. The reason this works is because, for each $N$, the Fourier series is a sum of finitely many elements of $\overline{\mathbb{Q}}$, which we have embedded in both $\mathbb{C}_{3}$ and $\mathbb{C}$, and that this sum is invariant under the action of $\textrm{Gal}\left(\overline{\mathbb{Q}}/\mathbb{Q}\right)$. \end{rem} Proof: By (\ref{eq:Convolution of dA_3 with D_N}), we have: \begin{equation} \lim_{N\rightarrow\infty}\sum_{\left|t\right|_{2}\leq2^{N}}\hat{A}_{3}\left(t\right)e^{2\pi i\left\{ t\mathfrak{z}\right\} _{2}}=\lim_{N\rightarrow\infty}\frac{3^{\#_{1}\left(\left[\mathfrak{z}\right]_{2^{N}}\right)}}{2^{N}} \end{equation} When $\mathfrak{z}\in\mathbb{Z}_{2}^{\prime}$, the number $\#_{1}\left(\left[\mathfrak{z}\right]_{2^{N}}\right)$ tends to $\infty$ as $N\rightarrow\infty$, thereby guaranteeing that the above limit converges to $0$ in $\mathbb{C}_{3}$. On the other hand, when $\mathfrak{z}\in\mathbb{N}_{0}$, $\#_{1}\left(\left[\mathfrak{z}\right]_{2^{N}}\right)=\#_{1}\left(\mathfrak{z}\right)$ for all $N\geq\lambda_{2}\left(\mathfrak{z}\right)$, and so, the above limit then tends to $0$ in $\mathbb{C}$. Q.E.D. \begin{prop} We have the integral formula: \begin{equation} \int_{\mathbb{Z}_{2}}f\left(\mathfrak{z}\right)dA_{3}\left(\mathfrak{z}\right)\overset{\mathbb{C}_{3}}{=}\lim_{N\rightarrow\infty}\frac{1}{4^{N}}\sum_{n=0}^{2^{N}-1}f\left(n\right)3^{\#_{1}\left(n\right)},\textrm{ }\forall f\in C\left(\mathbb{Z}_{2},\mathbb{C}_{3}\right)\label{eq:Riemann sum formula for the dA_3 integral of a continuous (2,3)-adic function} \end{equation} \end{prop} Proof: Since: \begin{equation} \left[\mathfrak{z}\overset{2^{N}}{\equiv}n\right]=\frac{1}{2^{N}}\sum_{\left|t\right|_{2}\leq2^{N}}e^{2\pi i\left\{ t\left(\mathfrak{z}-n\right)\right\} _{2}}=\frac{1}{2^{N}}\sum_{\left|t\right|_{2}\leq2^{N}}e^{-2\pi i\left\{ tn\right\} _{2}}e^{2\pi i\left\{ t\mathfrak{z}\right\} _{2}} \end{equation} we have: \begin{equation} \int_{\mathbb{Z}_{2}}\left[\mathfrak{z}\overset{2^{N}}{\equiv}n\right]dA_{3}\left(\mathfrak{z}\right)=\frac{1}{2^{N}}\sum_{\left|t\right|_{2}\leq2^{N}}\hat{A}_{3}\left(t\right)e^{2\pi i\left\{ tn\right\} _{2}}=\frac{3^{\#_{1}\left(n\right)}}{4^{N}} \end{equation} for all $N\geq0$ and $n\in\left\{ 0,\ldots,2^{N}-1\right\} $. So, letting $f\in C\left(\mathbb{Z}_{2},\mathbb{C}_{3}\right)$ be arbitrary, taking $N$th truncations yields: \begin{equation} \int_{\mathbb{Z}_{2}}f_{N}\left(\mathfrak{z}\right)dA_{3}\left(\mathfrak{z}\right)=\sum_{n=0}^{2^{N}-1}f\left(n\right)\int_{\mathbb{Z}_{2}}\left[\mathfrak{z}\overset{2^{N}}{\equiv}n\right]dA_{3}\left(\mathfrak{z}\right)=\frac{1}{4^{N}}\sum_{n=0}^{2^{N}-1}f\left(n\right)3^{\#_{1}\left(n\right)} \end{equation} Since $f$ is continuous, the $f_{N}$ converge uniformly to $f$ (\textbf{Proposition \ref{prop:Unif. convergence of truncation equals continuity}}). Since $dA_{3}\in C\left(\mathbb{Z}_{2},\mathbb{C}_{3}\right)^{\prime}$, this guarantees: \begin{equation} \int_{\mathbb{Z}_{2}}f\left(\mathfrak{z}\right)dA_{3}\left(\mathfrak{z}\right)\overset{\mathbb{C}_{3}}{=}\lim_{N\rightarrow\infty}\int_{\mathbb{Z}_{2}}f_{N}\left(\mathfrak{z}\right)dA_{3}\left(\mathfrak{z}\right)=\lim_{N\rightarrow\infty}\frac{1}{4^{N}}\sum_{n=0}^{2^{N}-1}f\left(n\right)3^{\#_{1}\left(n\right)} \end{equation} Q.E.D. \vphantom{} Thus, for $d\mu=dA_{3}$, our limit of interest (\ref{eq:The Limit of Interest}) converges to $0$, yet $dA_{3}$ is not the zero measure: this is $dA_{3}$'s ``degeneracy''. $dA_{3}$ also places us in the extremely unusual position of resorting to \emph{different topologies} for different inputs in order to guarantee the point-wise existence of our limit for every $\mathfrak{z}\in\mathbb{Z}_{2}$. The purpose of Section \ref{sec:3.3 quasi-integrability} is to demonstrate that this procedure can be done consistently, and, moreover, in a way that leads to useful results. Contrary to what Monna-Springer theory would have us believe, our next examples show that there are discontinuous\textemdash even \emph{singular!}\textemdash $\left(p,q\right)$-adic functions which can be meaningfully integrated. \begin{prop} \label{prop:sum of v_p}Let $p$ be an integer $\geq2$. Then, for each $\mathfrak{z}\in\mathbb{Z}_{p}\backslash\left\{ 0\right\} $: \begin{equation} \sum_{0<\left|t\right|_{p}\leq p^{N}}v_{p}\left(t\right)e^{2\pi i\left\{ t\mathfrak{z}\right\} _{p}}\overset{\overline{\mathbb{Q}}}{=}\frac{p\left|\mathfrak{z}\right|_{p}^{-1}-1}{p-1},\textrm{ }\forall N>v_{p}\left(\mathfrak{z}\right)\label{eq:Fourier sum of v_p of t} \end{equation} Here, the use of $\overset{\overline{\mathbb{Q}}}{=}$ is to indicate that the equality is one of elements of $\overline{\mathbb{Q}}$. \end{prop} Proof: Fixing $\mathfrak{z}\in\mathbb{Z}_{p}\backslash\left\{ 0\right\} $, we have: \begin{align*} \sum_{0<\left|t\right|_{p}\leq p^{N}}v_{p}\left(t\right)e^{2\pi i\left\{ t\mathfrak{z}\right\} _{p}} & =\sum_{n=1}^{N}\sum_{\left|t\right|_{p}=p^{n}}\left(-n\right)e^{2\pi i\left\{ t\mathfrak{z}\right\} _{p}}\\ & =\sum_{n=1}^{N}\left(-n\right)\left(p^{n}\left[\mathfrak{z}\overset{p^{n}}{\equiv}0\right]-p^{n-1}\left[\mathfrak{z}\overset{p^{n-1}}{\equiv}0\right]\right) \end{align*} Simplifying gives: \begin{equation} \sum_{0<\left|t\right|_{p}\leq p^{N}}v_{p}\left(t\right)e^{2\pi i\left\{ t\mathfrak{z}\right\} _{p}}=-Np^{N}\left[\mathfrak{z}\overset{p^{N}}{\equiv}0\right]+\sum_{n=0}^{N-1}p^{n}\left[\mathfrak{z}\overset{p^{n}}{\equiv}0\right] \end{equation} Because $\mathfrak{z}$ is non-zero, $\mathfrak{z}=p^{v_{p}\left(\mathfrak{z}\right)}\mathfrak{u}$ for some $\mathfrak{u}\in\mathbb{Z}_{p}^{\times}$, where $v_{p}\left(\mathfrak{z}\right)$ is a non-negative rational integer. As such, the congruence $\mathfrak{z}\overset{p^{n}}{\equiv}0$ will fail to hold for all $n\geq v_{p}\left(\mathfrak{z}\right)+1$. As such, we can write:: \begin{equation} \sum_{0<\left|t\right|_{p}\leq p^{N}}v_{p}\left(t\right)e^{2\pi i\left\{ t\mathfrak{z}\right\} _{p}}=\sum_{n=0}^{v_{p}\left(\mathfrak{z}\right)}p^{n}\left[\mathfrak{z}\overset{p^{n}}{\equiv}0\right],\textrm{ }\forall N\geq v_{p}\left(\mathfrak{z}\right)+1,\textrm{ }\forall\mathfrak{z}\in\mathbb{Z}_{p}\backslash\left\{ 0\right\} \end{equation} Since $\mathfrak{z}\overset{p^{n}}{\equiv}0$ holds true if and only if $n\in\left\{ 0,\ldots,v_{p}\left(\mathfrak{z}\right)\right\} $, the Iverson brackets in the above formula all evaluate to $1$. This leaves us with a finite geometric series: \begin{equation} \sum_{0<\left|t\right|_{p}\leq p^{N}}v_{p}\left(t\right)e^{2\pi i\left\{ t\mathfrak{z}\right\} _{p}}=\sum_{n=0}^{v_{p}\left(\mathfrak{z}\right)}p^{n}\left[\mathfrak{z}\overset{p^{n}}{\equiv}0\right]=\sum_{n=0}^{v_{p}\left(\mathfrak{z}\right)}p^{n}=\frac{p^{v_{p}\left(\mathfrak{z}\right)+1}-1}{p-1} \end{equation} for all $N\geq v_{p}\left(\mathfrak{z}\right)+1$. Noting that $p\left|\mathfrak{z}\right|_{p}^{-1}=p^{v_{p}\left(\mathfrak{z}\right)+1}$ then completes the computation. Q.E.D. \vphantom{} In general, there is no way to define (\ref{eq:Fourier sum of v_p of t}) when $\mathfrak{z}=0$. Indeed: \begin{equation} \sum_{0<\left|t\right|_{p}\leq p^{N}}v_{p}\left(t\right)=\sum_{n=1}^{N}\sum_{\left|t\right|_{p}=p^{n}}\left(-n\right)=-\sum_{n=1}^{N}n\varphi\left(p^{n}\right)=-\left(p-1\right)\sum_{n=1}^{N}np^{n-1} \end{equation} where $\varphi$ is the \textbf{Euler Totient Function}. Since $p$ and $q$ are distinct primes, the $q$-adic absolute value of the $n$th term of the partial sum of the series is: \begin{equation} \left|np^{n-1}\right|_{q}=\left|n\right|_{q} \end{equation} which does not tend to $0$ as $n\rightarrow\infty$. Thus, (\ref{eq:Fourier sum of v_p of t}) does not converge $q$-adically when $\mathfrak{z}=0$. On the other hand, in the topology of $\mathbb{C}$, the series diverges to $\infty$. Nevertheless, because of the $\hat{\mu}$ defined above, we can write: \begin{equation} \frac{p\left|\mathfrak{z}\right|_{p}^{-1}-1}{p-1}d\mathfrak{z} \end{equation} to denote the measure defined by: \begin{equation} \int_{\mathbb{Z}_{p}}\frac{p\left|\mathfrak{z}\right|_{p}^{-1}-1}{p-1}f\left(\mathfrak{z}\right)d\mathfrak{z}\overset{\textrm{def}}{=}\sum_{t\in\hat{\mathbb{Z}}_{p}\backslash\left\{ 0\right\} }\hat{f}\left(-t\right)v_{p}\left(t\right),\textrm{ }\forall f\in C\left(\mathbb{Z}_{p},\mathbb{C}_{q}\right) \end{equation} In this sense, the $\left(p,q\right)$-adic function: \begin{equation} \frac{p\left|\mathfrak{z}\right|_{p}^{-1}-1}{p-1} \end{equation} is ``quasi-integrable'' in that despite having a singularity at $\mathfrak{z}=0$, we can still use the above integral formula to define its integral: \begin{equation} \int_{\mathbb{Z}_{p}}\frac{p\left|\mathfrak{z}\right|_{p}^{-1}-1}{p-1}d\mathfrak{z}\overset{\textrm{def}}{=}0 \end{equation} as the image of the constant function $1$ under the associated measure. With regard to our notational formalism, it is then natural to ask: \begin{equation} \int_{\mathbb{Z}_{p}}\frac{p\left|\mathfrak{z}\right|_{p}^{-1}-1}{p-1}d\mathfrak{z}\overset{?}{=}\frac{p}{p-1}\int_{\mathbb{Z}_{p}}\left|\mathfrak{z}\right|_{p}^{-1}d\mathfrak{z}-\frac{1}{p-1}\underbrace{\int_{\mathbb{Z}_{p}}d\mathfrak{z}}_{1} \end{equation} The answer is \emph{yes}. \begin{prop} Let $p$ be an integer $\geq2$. Then, for all $\mathfrak{z}\in\mathbb{Z}_{p}\backslash\left\{ 0\right\} $, we have: \begin{equation} \left|\mathfrak{z}\right|_{p}^{-1}\overset{\overline{\mathbb{Q}}}{=}\frac{1}{p}+\frac{p-1}{p}\sum_{0<\left|t\right|_{p}\leq p^{N}}v_{p}\left(t\right)e^{2\pi i\left\{ t\mathfrak{z}\right\} _{p}},\textrm{ }\forall N>v_{p}\left(\mathfrak{z}\right)\label{eq:reciprocal of the p-adic absolute value of z is quasi-integrable} \end{equation} \end{prop} Proof: Let: \begin{equation} \frac{1}{p-1}d\mathfrak{z} \end{equation} denote the $\left(p,q\right)$-adic measure which is the scalar multiple of the $\left(p,q\right)$-adic Haar probability measure by $1/\left(p-1\right)$. Then, by definition, for any $f\in C\left(\mathbb{Z}_{p},\mathbb{C}_{q}\right)$: \begin{align*} \int_{\mathbb{Z}_{p}}\frac{p\left|\mathfrak{z}\right|_{p}^{-1}-1}{p-1}f\left(\mathfrak{z}\right)d\mathfrak{z}+\int_{\mathbb{Z}_{p}}\frac{f\left(\mathfrak{z}\right)}{p-1}d\mathfrak{z} & =\sum_{t\in\hat{\mathbb{Z}}_{p}\backslash\left\{ 0\right\} }\hat{f}\left(-t\right)v_{p}\left(t\right)+\frac{\hat{f}\left(0\right)}{p-1}\\ & =\sum_{t\in\hat{\mathbb{Z}}_{p}}\hat{f}\left(-t\right)\hat{\nu}\left(t\right) \end{align*} where $\hat{\nu}:\hat{\mathbb{Z}}_{p}\rightarrow\mathbb{C}_{q}$ is defined by: \begin{equation} \hat{\nu}\left(t\right)\overset{\textrm{def}}{=}\begin{cases} \frac{1}{p-1} & \textrm{if }t=0\\ v_{p}\left(t\right) & \textrm{else} \end{cases},\textrm{ }\forall t\in\hat{\mathbb{Z}}_{p} \end{equation} Running through the computation of $\tilde{\nu}_{N}\left(\mathfrak{z}\right)$ yields: \begin{equation} \sum_{\left|t\right|_{p}\leq p^{N}}\nu\left(t\right)e^{2\pi i\left\{ t\mathfrak{z}\right\} _{p}}\overset{\mathbb{Q}}{=}\frac{p\left|\mathfrak{z}\right|_{p}^{-1}}{p-1},\textrm{ }\forall\mathfrak{z}\in\mathbb{Z}_{p}\backslash\left\{ 0\right\} ,\textrm{ }\forall N>v_{p}\left(\mathfrak{z}\right) \end{equation} Hence, $\mathfrak{z}\in\mathbb{Z}_{p}\backslash\left\{ 0\right\} $ and $N>v_{p}\left(\mathfrak{z}\right)$ imply: \begin{align*} \left|\mathfrak{z}\right|_{p}^{-1} & \overset{\overline{\mathbb{Q}}}{=}\sum_{\left|t\right|_{p}\leq p^{N}}\frac{p-1}{p}\nu\left(t\right)e^{2\pi i\left\{ t\mathfrak{z}\right\} _{p}}\\ & \overset{\overline{\mathbb{Q}}}{=}\frac{1}{p}+\frac{p-1}{p}\sum_{0<\left|t\right|_{p}\leq p^{N}}v_{p}\left(t\right)e^{2\pi i\left\{ t\mathfrak{z}\right\} _{p}} \end{align*} as desired. Q.E.D. \begin{example} Letting $\mathfrak{a}\in\mathbb{Z}_{p}$ be arbitrary, for all $\mathfrak{z}\in\mathbb{Z}_{p}\backslash\left\{ 0\right\} $ and all $N>v_{p}\left(\mathfrak{z}\right)$, we have: \[ \left|\mathfrak{z}-\mathfrak{a}\right|_{p}^{-1}\overset{\mathbb{Q}}{=}\frac{1}{p}+\frac{p-1}{p}\sum_{0<\left|t\right|_{p}\leq p^{N}}v_{p}\left(t\right)e^{-2\pi i\left\{ t\mathfrak{a}\right\} _{p}}e^{2\pi i\left\{ t\mathfrak{z}\right\} _{p}} \] Consequently, letting $\left\{ \mathfrak{a}_{n}\right\} _{n\geq0}$ be any sequence of distinct elements of $\mathbb{Z}_{p}$ and letting $\left\{ \mathfrak{b}_{n}\right\} _{n\geq0}$ be any sequence in $\mathbb{C}_{q}$ tending to $0$ in $q$-adic magnitude, the function: \begin{equation} \sum_{n=0}^{\infty}\frac{\mathfrak{b}_{n}}{\left|\mathfrak{z}-\mathfrak{a}\right|_{p}}\label{eq:"quasi-integrable" function with infinitely many singularities} \end{equation} is ``quasi-integrable'', because we can get a measure out of it by writing: \begin{equation} \int_{\mathbb{Z}_{p}}f\left(\mathfrak{z}\right)\sum_{n=0}^{\infty}\frac{\mathfrak{b}_{n}}{\left|\mathfrak{z}-\mathfrak{a}\right|_{p}}d\mathfrak{z} \end{equation} as: \begin{equation} \frac{\hat{f}\left(0\right)}{p}\sum_{n=0}^{\infty}\mathfrak{b}_{n}+\frac{p-1}{p}\sum_{t\neq0}v_{p}\left(t\right)\left(\sum_{n=0}^{\infty}\mathfrak{b}_{n}e^{-2\pi i\left\{ t\mathfrak{a}_{n}\right\} _{p}}\right)\hat{f}\left(-t\right) \end{equation} for any $f\in C\left(\mathbb{Z}_{p},\mathbb{C}_{q}\right)$. Moreover, because of the failure of (\ref{eq:reciprocal of the p-adic absolute value of z is quasi-integrable}) to converge in either $\mathbb{C}_{q}$ or $\mathbb{C}$ for $\mathfrak{z}=0$, the Fourier series for (\ref{eq:"quasi-integrable" function with infinitely many singularities}) then fails to converge in either $\mathbb{C}_{q}$ or $\mathbb{C}$ for $\mathfrak{z}=\mathfrak{a}_{n}$ for any $n$. Even then, this is not the most pathological case we might have to deal with. Once again, thanks to the invariance of (\ref{eq:reciprocal of the p-adic absolute value of z is quasi-integrable}) under the action of $\textrm{Gal}\left(\overline{\mathbb{Q}}/\mathbb{Q}\right)$, the point-wise convergence of (\ref{eq:reciprocal of the p-adic absolute value of z is quasi-integrable}) on $\mathbb{Z}_{p}\backslash\left\{ 0\right\} $ has the useful property of occurring in \emph{every} field extension of $\overline{\mathbb{Q}}$: for each $\mathfrak{z}$, (\ref{eq:reciprocal of the p-adic absolute value of z is quasi-integrable}) is constant for all $N>v_{p}\left(\mathfrak{z}\right)$. On the other hand, suppose the limit of interest (\ref{eq:The Limit of Interest}) yields a rising-continuous function $\chi\in\tilde{C}\left(\mathbb{Z}_{p},\mathbb{C}_{q}\right)$ which converges in $\mathbb{C}_{q}$ for all $\mathfrak{z}\in\mathbb{Z}_{p}^{\prime}$ and converges in $\mathbb{C}$ for all $\mathfrak{z}\in\mathbb{N}_{0}$. Then, translating by some $\mathfrak{a}\in\mathbb{Z}_{p}$, we can shift the sets on which $\chi$ converges $q$-adically by considering the Fourier series for $\chi\left(\mathfrak{z}+\mathfrak{a}\right)$. But then, letting $\mathfrak{a}$ and $\mathfrak{b}$ be distinct $p$-adic integers, how might we make sense of the convergence of the Fourier series of: \begin{equation} \chi\left(\mathfrak{z}+\mathfrak{a}\right)+\chi\left(\mathfrak{z}+\mathfrak{b}\right) \end{equation} In this set-up, there may be a $\mathfrak{z}_{0}\in\mathbb{Z}_{p}$ for which $\chi\left(\mathfrak{z}+\mathfrak{a}\right)$'s Fourier series converges $q$-adically and $\chi\left(\mathfrak{z}+\mathfrak{b}\right)$'s series converges in $\mathbb{C}$ and \emph{not }in $\mathbb{C}_{q}$! Worse yet, there is a very natural reason to \emph{want} to work with linear combinations of the form: \begin{equation} \sum_{n}\mathfrak{b}_{n}\chi\left(\mathfrak{z}+\mathfrak{a}_{n}\right) \end{equation} for $\mathfrak{b}_{n}$s in $\mathbb{C}_{q}$ and $\mathfrak{a}_{n}$s in $\mathbb{Z}_{p}$; these arise naturally when considering $\left(p,q\right)$-adic analogues of the Wiener Tauberian Theorem. Results of this type describe conditions in which (and the extent to which) a quasi-integrable function $\chi\left(\mathfrak{z}\right)$ will have well-defined reciprocal. Since we want to determine the values $x\in\mathbb{Z}$ for which $\chi_{H}\left(\mathfrak{z}\right)-x$ vanishes for some $\mathfrak{z}\in\mathbb{Z}_{p}$, these issues are, obviously, of the utmost import to us. This particular problem will be dealt with by defining vector spaces of $\left(p,q\right)$-adically bounded functions $\hat{\mathbb{Z}}_{p}\rightarrow\overline{\mathbb{Q}}$, so as to guarantee that we can take linear combinations of translates of functions on $\mathbb{Z}_{p}$ without having to worry about getting our topologies in a twist. \end{example} \vphantom{} Whereas the previous two examples were liberating, the next example will be dour and sobering. It tells us that our newly acquired freedoms \emph{do} come at a price. \begin{example} Having shown that: \begin{equation} \sum_{0<\left|t\right|_{p}\leq p^{N}}v_{p}\left(t\right)e^{2\pi i\left\{ t\mathfrak{z}\right\} _{p}}\overset{\overline{\mathbb{Q}}}{=}\frac{p\left|\mathfrak{z}\right|_{p}^{-1}-1}{p-1},\textrm{ }\forall\mathfrak{z}\in\mathbb{Z}_{p}\backslash\left\{ 0\right\} ,\textrm{ }\forall N>v_{p}\left(\mathfrak{z}\right) \end{equation} upon letting $N\rightarrow\infty$, we can write: \begin{equation} \sum_{t\in\hat{\mathbb{Z}}_{p}\backslash\left\{ 0\right\} }v_{p}\left(t\right)e^{2\pi i\left\{ t\mathfrak{z}\right\} _{p}}\overset{\mathbb{F}}{=}\frac{p\left|\mathfrak{z}\right|_{p}^{-1}-1}{p-1} \end{equation} where $\mathbb{F}$ is any valued field extension of $\overline{\mathbb{Q}}$; note, completeness of $\mathbb{F}$ is \emph{not }required! Now, let $p=2$. Then, we can write: \begin{equation} \sum_{\left|t\right|_{2}\leq2^{N}}\left(1-\mathbf{1}_{0}\left(t\right)\right)v_{2}\left(t\right)e^{2\pi i\left\{ t\mathfrak{z}\right\} _{2}}\overset{\overline{\mathbb{Q}}}{=}2\left|\mathfrak{z}\right|_{2}^{-1}-1,\textrm{ }\forall N\geq v_{2}\left(\mathfrak{z}\right) \end{equation} Here, $1-\mathbf{1}_{0}\left(t\right)$ is $0$ when $t\overset{1}{\equiv}0$ and is $1$ for all other $t$. Now, let us add the Fourier series generated by $\hat{A}_{3}$. This gives: \begin{equation} \sum_{\left|t\right|_{2}\leq2^{N}}\left(\hat{A}_{3}\left(t\right)+\left(1-\mathbf{1}_{0}\left(t\right)\right)v_{2}\left(t\right)\right)e^{2\pi i\left\{ t\mathfrak{z}\right\} _{2}}\overset{\overline{\mathbb{Q}}}{=}2\left|\mathfrak{z}\right|_{2}^{-1}-1+\frac{3^{\#_{1}\left(\left[\mathfrak{z}\right]_{2^{N}}\right)}}{2^{N}} \end{equation} As $N\rightarrow\infty$, when $\mathfrak{z}\in\mathbb{Z}_{2}^{\prime}$, the right-hand side converges to $2\left|\mathfrak{z}\right|_{2}^{-1}-1$ in $\mathbb{C}_{3}$ ; for $\mathbb{\mathfrak{z}}\in\mathbb{N}_{0}$, meanwhile, the right-hand side converges to $2\left|\mathfrak{z}\right|_{2}^{-1}-1$ in $\mathbb{C}$. The same is true if we remove $\hat{A}_{3}\left(t\right)$. As such, we have two different formulas: \begin{equation} f\mapsto\sum_{t\in\hat{\mathbb{Z}}_{2}\backslash\left\{ 0\right\} }\hat{f}\left(t\right)v_{2}\left(-t\right) \end{equation} \begin{equation} f\mapsto\sum_{t\in\hat{\mathbb{Z}}_{2}}\hat{f}\left(t\right)\left(\hat{A}_{3}\left(-t\right)+\left(1-\mathbf{1}_{0}\left(-t\right)\right)v_{2}\left(-t\right)\right) \end{equation} representing two \emph{entirely different }$\left(2,3\right)$-adic measures, and yet, \emph{both }of them constitute perfectly reasonable ways of defining the integral of $\left(2\left|\mathfrak{z}\right|_{2}^{-1}-1\right)f\left(\mathfrak{z}\right)$. These are: \begin{equation} \int_{\mathbb{Z}_{2}}\left(2\left|\mathfrak{z}\right|_{2}^{-1}-1\right)f\left(\mathfrak{z}\right)d\mathfrak{z}\overset{\mathbb{C}_{3}}{=}\sum_{t\in\hat{\mathbb{Z}}_{2}\backslash\left\{ 0\right\} }\hat{f}\left(t\right)v_{2}\left(-t\right) \end{equation} and: \begin{equation} \int_{\mathbb{Z}_{2}}\left(2\left|\mathfrak{z}\right|_{2}^{-1}-1\right)f\left(\mathfrak{z}\right)d\mathfrak{z}\overset{\mathbb{C}_{3}}{=}\sum_{t\in\hat{\mathbb{Z}}_{2}}\hat{f}\left(t\right)\left(\hat{A}_{3}\left(-t\right)+\left(1-\mathbf{1}_{0}\left(-t\right)\right)v_{2}\left(-t\right)\right) \end{equation} Both of these are valid because the $\hat{\mu}$ we are using against $\hat{f}$ are $3$-adically bounded. However, because of $dA_{3}$'s degeneracy, even if $\int fdA_{3}\neq0$, when we try to consider the Fourier series generated by $\hat{\mu}$, the part generated by $\hat{A}_{3}$ vanishes into thin air. So, while quasi-integrability allows us to integrate discontinuous functions, there will be no canonical choice for these integrals' values. Indeed, as we will see in Subsection \ref{subsec:3.3.5 Quasi-Integrability}, the integral of a quasi-integrable function will only be well-defined modulo an arbitrary degenerate measure. \end{example} \vphantom{} Before we begin our study of these matters in earnest, we record the following result, which\textemdash as described below\textemdash can be used to define a ``$p,q$-adic Mellin transform\index{Mellin transform!left(p,qright)-adic@$\left(p,q\right)$-adic}\index{$p,q$-adic!Mellin transform}''. This may be of great interest in expanding the scope of $\left(p,q\right)$-adic analysis beyond what was previously thought possible. \begin{lem} Let $p$ and $q$ be integers $\geq2$ so that $q\mid\left(p-1\right)$. Then, for any non-zero rational number $r$ and any $\mathfrak{z}\in\mathbb{Z}_{p}\backslash\left\{ 0\right\} $: \begin{equation} \left|\mathfrak{z}\right|_{p}^{-r}\overset{\mathbb{C}_{q}}{=}\sum_{\left|t\right|_{p}\leq p^{N}}\left(\frac{1}{p^{r}}\sum_{m=0}^{\infty}\binom{r}{m}\left(p-1\right)^{m}\sum_{\begin{array}{c} 0<\left|s_{1}\right|_{p},\ldots,\left|s_{m}\right|_{p}\leq p^{N}\\ s_{1}+\cdots+s_{m}=t \end{array}}\prod_{j=1}^{m}v_{p}\left(s_{j}\right)\right)e^{2\pi i\left\{ t\mathfrak{z}\right\} _{p}}\label{eq:-rth power of the p-adic absolute value is sometimes quasi-integrable} \end{equation} for all $N>v_{p}\left(\mathfrak{z}\right)$. Here, the condition on $q\mid\left(p-1\right)$ guarantees that the $m$-series converges $q$-adically. Also, the sum over $s_{1},\ldots,s_{m}$ is defined to be $1$ when $m=0$. \emph{Note}: if $r$ is a positive integer, the infinite series reduces to the finite sum $\sum_{m=0}^{r}$, and as such, the condition that $q\mid\left(p-1\right)$ can be dropped. \end{lem} Proof: Letting $p$, $q$, $\mathfrak{z}$, and $N$ be as given, we take (\ref{eq:reciprocal of the p-adic absolute value of z is quasi-integrable}) and raise it to the $r$th power: \begin{align*} \left|\mathfrak{z}\right|_{p}^{-r} & =\left(\frac{1}{p}+\frac{p-1}{p}\sum_{0<\left|t\right|_{p}\leq p^{N}}v_{p}\left(t\right)e^{2\pi i\left\{ t\mathfrak{z}\right\} _{p}}\right)^{r}\\ & =\frac{1}{p^{r}}\left(1+\left(p-1\right)\sum_{0<\left|t\right|_{p}\leq p^{N}}v_{p}\left(t\right)e^{2\pi i\left\{ t\mathfrak{z}\right\} _{p}}\right)^{r}\\ & =\frac{1}{p^{r}}\sum_{m=0}^{\infty}\binom{r}{m}\left(p-1\right)^{m}\underbrace{\sum_{0<\left|t_{1}\right|_{p},\ldots,\left|t_{m}\right|_{p}\leq p^{N}}v_{p}\left(t_{1}\right)\cdots v_{p}\left(t_{m}\right)e^{2\pi i\left\{ \left(t_{1}+\cdots+t_{m}\right)\mathfrak{z}\right\} _{p}}}_{1\textrm{ when }m=0} \end{align*} Note that the use of the binomial series for $\left(1+\mathfrak{y}\right)^{r}$ is valid here, seeing as: \begin{equation} \left(1+\mathfrak{y}\right)^{r}\overset{\mathbb{C}_{q}}{=}\sum_{m=0}^{\infty}\binom{r}{m}\mathfrak{y}^{m},\textrm{ }\forall\mathfrak{y}\in\mathbb{C}_{q}:\left|\mathfrak{y}\right|_{q}<1 \end{equation} and that: \begin{equation} \left|\left(p-1\right)\sum_{0<\left|t\right|_{p}\leq p^{N}}v_{p}\left(t\right)e^{2\pi i\left\{ t\mathfrak{z}\right\} _{p}}\right|_{q}\leq\left|p-1\right|_{q}\cdot\underbrace{\max_{0<\left|t\right|_{p}\leq p^{N}}\left|v_{p}\left(t\right)\right|_{q}}_{\leq1}\leq\left|p-1\right|_{q} \end{equation} where the divisibility of $p-1$ by $q$ then guarantees $\left|p-1\right|_{q}<1$, and hence, that the series in $m$ converges uniformly in $t$ and $\mathfrak{z}$. We then have: \begin{eqnarray*} & \sum_{0<\left|t_{1}\right|_{p},\ldots,\left|t_{m}\right|_{p}\leq p^{N}}v_{p}\left(t_{1}\right)\cdots v_{p}\left(t_{m}\right)e^{2\pi i\left\{ \left(t_{1}+\cdots+t_{m}\right)\mathfrak{z}\right\} _{p}}\\ & =\\ & \sum_{\left|t\right|_{p}\leq p^{N}}\left(\sum_{\begin{array}{c} 0<\left|s_{1}\right|_{p},\ldots,\left|s_{m}\right|_{p}\leq p^{N}\\ s_{1}+\cdots+s_{m}=t \end{array}}v_{p}\left(s_{1}\right)\cdots v_{p}\left(s_{m}\right)\right)e^{2\pi i\left\{ t\mathfrak{z}\right\} _{p}} \end{eqnarray*} Hence: \begin{align*} \left|\mathfrak{z}\right|_{p}^{-r} & =\sum_{\left|t\right|_{p}\leq p^{N}}\left(\frac{1}{p^{r}}\sum_{m=0}^{\infty}\binom{r}{m}\left(p-1\right)^{m}\underbrace{\sum_{\begin{array}{c} 0<\left|s_{1}\right|_{p},\ldots,\left|s_{m}\right|_{p}\leq p^{N}\\ s_{1}+\cdots+s_{m}=t \end{array}}\prod_{j=1}^{m}v_{p}\left(s_{j}\right)}_{1\textrm{ when }m=0}\right)e^{2\pi i\left\{ t\mathfrak{z}\right\} _{p}} \end{align*} for all $N>v_{p}\left(\mathfrak{z}\right)$, as desired. Q.E.D. \begin{rem}[\textbf{A $\left(p,q\right)$-adic Mellin transform}] \label{rem:pq adic mellin transform}The above allows us to define a $\left(p,q\right)$-adic Mellin transform, $\mathscr{M}_{p,q}$, by: \begin{equation} \mathscr{M}_{p,q}\left\{ f\right\} \left(r\right)\overset{\textrm{def}}{=}\int_{\mathbb{Z}_{p}}\left|\mathfrak{z}\right|_{p}^{r-1}f\left(\mathfrak{z}\right)d\mathfrak{z},\textrm{ }\forall r\in\mathbb{Q},\textrm{ }\forall f\in C\left(\mathbb{Z}_{p},\mathbb{C}_{q}\right)\label{eq:Definition of (p,q)-adic Mellin transform} \end{equation} whenever $q\mid\left(p-1\right)$. In particular, we then have: \begin{equation} \int_{\mathbb{Z}_{p}}\left|\mathfrak{z}\right|_{p}^{r}f\left(\mathfrak{z}\right)d\mathfrak{z}=\sum_{t\in\hat{\mathbb{Z}}_{p}}\left(p^{r}\sum_{m=0}^{\infty}\binom{1-r}{m}\left(p-1\right)^{m}\sum_{\begin{array}{c} \mathbf{s}\in\left(\hat{\mathbb{Z}}_{p}\backslash\left\{ 0\right\} \right)^{m}\\ \Sigma\left(\mathbf{s}\right)=t \end{array}}v_{p}\left(\mathbf{s}\right)\right)\hat{f}\left(-t\right)\label{eq:Formula for (p,q)-adic Mellin transform} \end{equation} where the $m$-sum is $1$ when $=0$, and where: \begin{align} \Sigma\left(\mathbf{s}\right) & \overset{\textrm{def}}{=}\sum_{j=1}^{m}s_{j}\\ v_{p}\left(\mathbf{s}\right) & \overset{\textrm{def}}{=}\prod_{j=1}^{m}v_{p}\left(s_{j}\right) \end{align} for all $\mathbf{s}=\left(s_{1},\ldots,s_{m}\right)\in\left(\hat{\mathbb{Z}}_{p}\backslash\left\{ 0\right\} \right)^{m}$. If we allow for $f$ to take values in the universal $q$-adic field $\Omega_{q}$ (the spherical completion of $\mathbb{C}_{q}$, see \cite{Robert's Book} for details), it may be possible to interpret $\mathscr{M}_{p,q}\left\{ f\right\} \left(r\right)$ for an arbitrary real number $r$ (which would mean that $\mathscr{M}_{p,q}\left\{ f\right\} :\mathbb{R}\rightarrow\Omega_{q}$), although care would obviously need to be taken to properly define and interpret (\ref{eq:Formula for (p,q)-adic Mellin transform}) in this case. Furthermore, if we can show that, given a quasi-integrable function $\chi$, the product $\left|\mathfrak{z}\right|_{p}^{r-1}\chi\left(\mathfrak{z}\right)$ will be quasi-integrable for $r\in\mathbb{Q}$ under certain circumstances, we could then define the $\left(p,q\right)$-adic Mellin transform of $\chi$. \end{rem} \begin{rem} It is worth noting that this Mellin transform formalism potentially opens the door for a $\left(p,q\right)$-adic notion of differentiability in the sense of distributions. This notion of differentiability is already well-established for the $\left(p,\infty\right)$-adic case (real/complex-valued functions), having originated a 1988 paper by V. Vladimirov, \emph{Generalized functions over the field of $p$-adic numbers} \cite{Vladimirov - the big paper about complex-valued distributions over the p-adics}. This paper is a comprehensive expos of everything one needs to know to work with distributions in the $\left(p,\infty\right)$-adic context. This method, now known as the \textbf{Vladimirov operator}\index{Vladimirov operator}\textbf{ }or \index{$p$-adic!fractional differentiation}\textbf{$p$-adic (fractional) differentiation }has since become standard in the areas of $\left(p,\infty\right)$-adic mathematical theoretical physics (``$p$-adic mathematical physics'') where it is employed; see, for instance, the article \cite{First 30 years of p-adic mathematical physics}, which gives a summary of the first thirty years' worth of developments in the subject. To borrow from \cite{First 30 years of p-adic mathematical physics} (although, it should be noted there is a typographical error in their statement of the definition), for $\alpha\in\mathbb{C}$, the order $\alpha$ $p$-adic fractional derivative of a function $f:\mathbb{Q}_{p}\rightarrow\mathbb{C}$ is defined\footnote{This definition is slightly more general than the one given in Subsection \ref{subsec:3.1.1 Some-Historical-and}, which was only valid for real $\alpha>0$; we re-obtain (\ref{eq:Definition of the Vladimirov Fractional Differentiation Operator}) from (\ref{eq:First formula for the p-adic fractional derivative}) below.} by: \begin{equation} D^{\alpha}\left\{ f\right\} \left(\mathfrak{z}\right)\overset{\textrm{def}}{=}\int_{\mathbb{Q}_{p}}\left|\mathfrak{y}\right|_{p}^{\alpha}\hat{f}\left(-\mathfrak{y}\right)e^{2\pi i\left\{ \mathfrak{y}\mathfrak{z}\right\} _{p}}d\mathfrak{y}\label{eq:First formula for the p-adic fractional derivative} \end{equation} where $d\mathfrak{y}$ is the real-valued $p$-adic Haar measure on $\mathbb{Q}_{p}$, normalized to be a probability measure on $\mathbb{Z}_{p}$, and where: \begin{equation} \hat{f}\left(\mathfrak{y}\right)\overset{\textrm{def}}{=}\int_{\mathbb{Q}_{p}}f\left(\mathfrak{x}\right)e^{-2\pi i\left\{ \mathfrak{y}\mathfrak{x}\right\} _{p}}d\mathfrak{x},\textrm{ }\forall\mathfrak{y}\in\mathbb{Q}_{p}\label{eq:Fourier transform over Q_p} \end{equation} is the Fourier transform of $f$. All of these require $f$ to be sufficiently well-behaved\textemdash say, compactly supported. When the support of $f$ lies in $\mathbb{Z}_{p}$, (\ref{eq:Fourier transform over Q_p}) reduces to: \[ \hat{f}\left(t\right)\overset{\mathbb{C}}{=}\int_{\mathbb{Z}_{p}}f\left(\mathfrak{x}\right)e^{-2\pi i\left\{ t\mathfrak{x}\right\} _{p}}d\mathfrak{x} \] and (\ref{eq:First formula for the p-adic fractional derivative}) becomes: \begin{equation} D^{\alpha}\left\{ f\right\} \left(\mathfrak{z}\right)=\sum_{t\in\hat{\mathbb{Z}}_{p}}\left|t\right|_{p}^{\alpha}\hat{f}\left(-t\right)e^{2\pi i\left\{ t\mathfrak{z}\right\} _{p}}\label{eq:p-adic fractional derivative over Z_p} \end{equation} When $\alpha>0$, (\ref{eq:First formula for the p-adic fractional derivative}) can be written as: \begin{equation} D^{\alpha}\left\{ f\right\} \left(\mathfrak{z}\right)=\frac{1}{\Gamma_{p}\left(-\alpha\right)}\int_{\mathbb{Q}_{p}}\frac{f\left(\mathfrak{z}\right)-f\left(\mathfrak{y}\right)}{\left|\mathfrak{z}-\mathfrak{y}\right|_{p}^{\alpha+1}}d\mathfrak{y}\label{eq:Second formula for the p-adic fractional derivative} \end{equation} where, recall, $\Gamma_{p}\left(-\alpha\right)$ is the physicist's notation for the normalization constant: \begin{equation} \Gamma_{p}\left(-\alpha\right)=\frac{p^{\alpha}-1}{1-p^{-1-\alpha}} \end{equation} \cite{First 30 years of p-adic mathematical physics} contains no less than \emph{three-hundred forty two} references, describing innumerable applications of this fractional derivative, including ``spectral properties, operators on bounded regions, analogs of elliptic and parabolic equations, a wave-type equation'', and many more. Given that my $\left(p,q\right)$-adic implementation of the Mellin transform (\ref{eq:Definition of (p,q)-adic Mellin transform}) can then be used to make sense of (\ref{eq:First formula for the p-adic fractional derivative}) for any $f\in C\left(\mathbb{Z}_{p},\mathbb{C}_{q}\right)$ and any $r\in\mathbb{Q}$, this seems like a research direction worth exploring. \end{rem} \subsection{\label{subsec:3.3.2 The--adic-Dirichlet}The $p$-adic Dirichlet Kernel and $\left(p,q\right)$-adic Fourier Resummation Lemmata} In this subsection, we will introduce specific types of $\left(p,q\right)$-adic measures for which (\ref{eq:The Limit of Interest}) is sufficiently well-behaved for us to construct useful spaces of quasi-integrable functions. Consequently, this subsection will be low on concepts and high on computations\textemdash though that's nothing in comparison with what we'll confront in Chapters 4 and 6\textemdash but, I digress. A recurring device in Section \ref{sec:3.3 quasi-integrability} is to start with a $\overline{\mathbb{Q}}$-valued function defined on $\hat{\mathbb{Z}}_{p}$ and then use it to create functions on $\mathbb{Z}_{p}$ by summing the associated Fourier series. Delightfully, this procedure is a direct $\left(p,q\right)$-adic analogue of a fundamental construction of classical Fourier analysis: convolution against the Dirichlet kernel. \begin{defn}[\textbf{The $p$-adic Dirichlet Kernel}] \index{Fourier series!$N$th partial sum}\ \vphantom{} I. Let $\mathbb{F}$ be an algebraically closed field extension of $\mathbb{Q}$, and let $\hat{\mu}:\hat{\mathbb{Z}}_{p}\rightarrow\mathbb{F}$. For each $N\in\mathbb{N}_{0}$, we then define \nomenclature{$\tilde{\mu}_{N}$}{$N$th partial sum of Fourier series generated by $\hat{\mu}$}$\tilde{\mu}_{N}:\mathbb{Z}_{p}\rightarrow\mathbb{F}$ by: \begin{equation} \tilde{\mu}_{N}\left(\mathfrak{z}\right)\overset{\textrm{def}}{=}\sum_{\left|t\right|_{p}\leq p^{N}}\hat{\mu}\left(t\right)e^{2\pi i\left\{ t\mathfrak{z}\right\} _{p}}\label{eq:Definition of mu_N twiddle} \end{equation} where the sum is taken over all $t\in\hat{\mathbb{Z}}_{p}$ satisfying $\left|t\right|_{p}\leq p^{N}$. Note that $\tilde{\mu}_{N}$ is then locally constant. As such, for any valued field $K$ extending $\mathbb{F}$, $\tilde{\mu}_{N}$ will be continuous as a function from $\mathbb{Z}_{p}$ to $K$. We call $\tilde{\mu}_{N}$ the \textbf{$N$th partial sum of the Fourier series generated by $\hat{\mu}$}. \vphantom{} II. For each $N\in\mathbb{N}_{0}$, we define the function $D_{p:N}:\mathbb{Z}_{p}\rightarrow\mathbb{Q}$ by: \begin{equation} D_{p:N}\left(\mathfrak{z}\right)\overset{\textrm{def}}{=}p^{N}\left[\mathfrak{z}\overset{p^{N}}{\equiv}0\right]\label{eq:Definition of the p-adic Dirichlet Kernel} \end{equation} We call \nomenclature{$D_{p:N}$}{$N$th $p$-adic Dirichlet kernel}$D_{p:N}$ the $N$th \textbf{$p$-adic Dirichlet kernel}\index{$p$-adic!Dirichlet kernel}. $\left\{ D_{p:N}\right\} _{N\geq0}$ is the \textbf{family of $p$-adic Dirichlet kernels}. Note that since each $D_{p:N}$ is locally constant, each is an element of $C\left(\mathbb{Z}_{p},K\right)$. Moreover, the Fourier transform of $D_{p:N}$ has finite support, with: \begin{equation} \hat{D}_{p:N}\left(t\right)=\mathbf{1}_{0}\left(p^{N}t\right)\overset{\textrm{def}}{=}\begin{cases} 1 & \textrm{if }\left|t\right|_{p}\leq p^{N}\\ 0 & \textrm{if }\left|t\right|_{p}>p^{N} \end{cases}\label{eq:Fourier Transform of the p-adic Dirichlet Kernel} \end{equation} We note here that since the $p$-adic Dirichlet kernels are rational valued, we can also compute their real/complex valued Fourier transforms with respect to the real-valued Haar probability measure on $\mathbb{Z}_{p}$, and\textemdash moreover\textemdash the resulting $\hat{D}_{p:N}\left(t\right)$ will be \emph{the same} as the one given above. In other words, the $\left(p,q\right)$-adic and $\left(p,\infty\right)$-adic Fourier transforms of $D_{p:N}$ and the procedures for computing them are \emph{formally identical}. I use the term \textbf{$\left(p,\infty\right)$-adic Dirichlet Kernel} when viewing the $D_{p:N}$s as taking values in $\mathbb{C}$; I use the term \textbf{$\left(p,K\right)$-adic Dirichlet Kernel }when viewing the $D_{p:N}$s as taking values in a metrically complete $q$-adic field $K$; I use the term \textbf{$\left(p,q\right)$-adic Dirichlet Kernel} for when the specific field $K$ is either not of concern or is $\mathbb{C}_{q}$. \end{defn} \begin{rem} Regarding the name ``$p$-adic Dirichlet Kernel'', it is worth noting that when one is doing $\left(p,\infty\right)$-adic analysis, the family of $p$-adic Dirichlet Kernels then form an approximate identity, in the sense that for any \nomenclature{$L^{1}\left(\mathbb{Z}_{p},\mathbb{C}\right)$}{set of absolutely integrable $f:\mathbb{Z}_{p}\rightarrow\mathbb{C}$}$f\in L^{1}\left(\mathbb{Z}_{p},\mathbb{C}\right)$ (complex-valued function on $\mathbb{Z}_{p}$, integrable with respect to the real-valued Haar probability measure on $\mathbb{Z}_{p}$), it can be shown that: \begin{equation} \lim_{N\rightarrow\infty}\left(D_{p:N}*f\right)\left(\mathfrak{z}\right)\overset{\mathbb{C}}{=}\lim_{N\rightarrow\infty}p^{N}\int_{\mathbb{Z}_{p}}\left[\mathfrak{y}\overset{p^{N}}{\equiv}\mathfrak{z}\right]f\left(\mathfrak{y}\right)d\mathfrak{y}=f\left(\mathfrak{z}\right)\label{eq:p infinity adic Lebesgue differentiation theorem} \end{equation} for \emph{almost} every $\mathfrak{z}\in\mathbb{Z}_{p}$; in particular, the limit holds for all $\mathfrak{z}$ at which $f$ is continuous. Indeed, (\ref{eq:p infinity adic Lebesgue differentiation theorem}) is an instance of the \textbf{Lebesgue Differentiation Theorem}\index{Lebesgue Differentiation Theorem}\footnote{Like in the archimedean case, the theorem is a consequence of a covering lemma and estimates for the ($\left(p,\infty\right)$-adic) Hardy-Littlewood maximal function (see \cite{Geometric Measure Theory}, for instance).}\textbf{ }for real and complex valued functions of a $p$-adic variable. Our use of the $p$-adic Dirichlet Kernels in the $\left(p,q\right)$-adic context therefore falls under the topic of summability kernels and approximate identities\index{approximate identity}. It remains to be seen if convolving other types of kernels with $\left(p,q\right)$-adic measures or $\left(p,q\right)$-adic functions will yield interesting results. \end{rem} \vphantom{} For now, let us establish the identities of import. \begin{prop}[\textbf{$D_{p:N}$ is an approximate identity}] \label{prop:D_p is an approximate identity}\ \vphantom{} I. Let $f\in C\left(\mathbb{Z}_{p},\mathbb{C}_{q}\right)$. Then, for all $N\in\mathbb{N}_{0}$: \begin{equation} \left(D_{p:N}*f\right)\left(\mathfrak{z}\right)\overset{\mathbb{C}_{q}}{=}\int_{\mathbb{Z}_{p}}p^{N}\left[\mathfrak{y}\overset{p^{N}}{\equiv}\mathfrak{z}\right]f\left(\mathfrak{y}\right)d\mathfrak{y}=\sum_{\left|t\right|_{p}\leq p^{N}}\hat{f}\left(t\right)e^{2\pi i\left\{ t\mathfrak{z}\right\} _{p}},\textrm{ }\forall\mathfrak{z}\in\mathbb{Z}_{p} \end{equation} Moreover, as $N\rightarrow\infty$, $D_{p:N}*f$ converges uniformly in $\mathbb{C}_{q}$ to $f$: \begin{equation} \lim_{N\rightarrow\infty}\sup_{\mathfrak{z}\in\mathbb{Z}_{p}}\left|f\left(\mathfrak{z}\right)-\left(D_{p:N}*f\right)\left(\mathfrak{z}\right)\right|_{q}\overset{\mathbb{R}}{=}0 \end{equation} Also, this shows that the evaluation map $f\mapsto f\left(\mathfrak{z}_{0}\right)$ is a continuous linear functional on $C\left(\mathbb{Z}_{p},\mathbb{C}_{q}\right)$. \vphantom{} II. Let $d\mu\in C\left(\mathbb{Z}_{p},\mathbb{C}_{q}\right)^{\prime}$. Then, for all $N\in\mathbb{N}_{0}$: \begin{equation} \left(D_{p:N}*d\mu\right)\left(\mathfrak{z}\right)\overset{\mathbb{C}_{q}}{=}\int_{\mathbb{Z}_{p}}p^{N}\left[\mathfrak{y}\overset{p^{N}}{\equiv}\mathfrak{z}\right]d\mu\left(\mathfrak{y}\right)=\tilde{\mu}_{N}\left(\mathfrak{z}\right),\textrm{ }\forall\mathfrak{z}\in\mathbb{Z}_{p} \end{equation} For fixed $N$, convergence occurs uniformly with respect to $\mathfrak{z}$. Additionally, if $\hat{\mu}$ takes values in $\overline{\mathbb{Q}}$, the the above equality also holds in $\mathbb{C}$, and, for fixed $N$, the convergence there is uniform with respect to $\mathfrak{z}$. \end{prop} Proof: The \textbf{Convolution Theorem }(\textbf{Theorem \ref{thm:Convolution Theorem}}) for the\textbf{ }$\left(p,q\right)$-adic Fourier Transform tells us that: \begin{equation} \left(D_{p:N}*f\right)\left(\mathfrak{z}\right)=\sum_{t\in\hat{\mathbb{Z}}_{p}}\hat{D}_{p:N}\left(t\right)\hat{f}\left(t\right)e^{2\pi i\left\{ t\mathfrak{z}\right\} _{p}} \end{equation} and: \begin{equation} \left(D_{p:N}*d\mu\right)\left(\mathfrak{z}\right)=\sum_{t\in\hat{\mathbb{Z}}_{p}}\hat{D}_{p:N}\left(t\right)\hat{\mu}\left(t\right)e^{2\pi i\left\{ t\mathfrak{z}\right\} _{p}} \end{equation} Using (\ref{eq:Fourier Transform of the p-adic Dirichlet Kernel}) gives the desired results. The uniform convergence of $D_{p:N}*f$ to $f$ follows from: \begin{equation} \sup_{\mathfrak{z}\in\mathbb{Z}_{p}}\left|f\left(\mathfrak{z}\right)-\left(D_{p:N}*f\right)\left(\mathfrak{z}\right)\right|_{q}=\left|\sum_{\left|t\right|_{p}>p^{N}}\hat{f}\left(t\right)e^{2\pi i\left\{ t\mathfrak{z}\right\} _{p}}\right|_{q}\leq\sup_{\left|t\right|_{p}>p^{N}}\left|\hat{f}\left(t\right)\right|_{q} \end{equation} with the upper bound tending to $0$ in $\mathbb{R}$ as $N\rightarrow\infty$, since $f\in C\left(\mathbb{Z}_{p},\mathbb{C}_{q}\right)\Leftrightarrow\hat{f}\in c_{0}\left(\hat{\mathbb{Z}}_{p},\mathbb{C}_{q}\right)$. Finally, fixing $\mathfrak{z}_{0}\in\mathbb{Z}_{p}$, the maps $f\mapsto\left(D_{p:N}*f\right)\left(\mathfrak{z}_{0}\right)$ are a family of continuous linear functionals on $C\left(\mathbb{Z}_{p},\mathbb{C}_{q}\right)$ which are indexed by $N$ which converge to the evaluation map $f\mapsto f\left(\mathfrak{z}_{0}\right)$ in supremum norm as $N\rightarrow\infty$. Since the dual space of $C\left(\mathbb{Z}_{p},\mathbb{C}_{q}\right)$ is a Banach space, it is closed under such limits. This proves the evaluation map $f\mapsto f\left(\mathfrak{z}_{0}\right)$ is an element of $C\left(\mathbb{Z}_{p},\mathbb{C}_{q}\right)^{\prime}$. Q.E.D. \vphantom{} With respect to the above, the introduction of frames in Subsection \ref{subsec:3.3.3 Frames} is little more than a backdrop for studying the point-wise behavior of the limit $\left(D_{p:N}*d\mu\right)\left(\mathfrak{z}\right)$ as $N\rightarrow\infty$ for various $d\mu$. Since we are potentially going to need to utilize different topologies to make sense of this limit, we might as well come up with a consistent set of terminology for describing our choices of topologies, instead of having to repeatedly specify the topologies \emph{every time }we want to talk about a limit\textemdash this is what frames are for. For now, though, let us continue our investigation of $\tilde{\mu}_{N}=D_{p:N}*d\mu$. \begin{prop} \label{prop:Criterion for zero measure in terms of partial sums of Fourier series}Let $d\mu\in C\left(\mathbb{Z}_{p},\mathbb{C}_{q}\right)^{\prime}$. Then $d\mu$ is the zero measure if and only if: \begin{equation} \lim_{N\rightarrow\infty}\left\Vert \tilde{\mu}_{N}\right\Vert _{p,q}\overset{\mathbb{R}}{=}0\label{eq:Criterion for a (p,q)-adic measure to be zero} \end{equation} where, recall, $\left\Vert \tilde{\mu}_{N}\right\Vert _{p,q}=\sup_{\mathfrak{z}\in\mathbb{Z}_{p}}\left|\tilde{\mu}_{N}\left(\mathfrak{z}\right)\right|_{q}$. \end{prop} Proof: I. If $d\mu$ is the zero measure, the $\hat{\mu}$ is identically zero, and hence, so is $\tilde{\mu}_{N}$. \vphantom{} II. Conversely, suppose $\lim_{N\rightarrow\infty}\left\Vert \tilde{\mu}_{N}\right\Vert _{p,q}\overset{\mathbb{R}}{=}0$. Then, let $f\in C\left(\mathbb{Z}_{p},\mathbb{C}_{q}\right)$ be arbitrary. Since $\tilde{\mu}_{N}$ is continuous for each $N$ and has $\mathbf{1}_{0}\left(p^{N}t\right)\hat{\mu}\left(t\right)$ as its Fourier transform, we can write: \begin{equation} \int_{\mathbb{Z}_{p}}f\left(\mathfrak{z}\right)\tilde{\mu}_{N}\left(\mathfrak{z}\right)d\mathfrak{z}\overset{\mathbb{C}_{q}}{=}\sum_{\left|t\right|_{p}\leq p^{N}}\hat{f}\left(-t\right)\hat{\mu}\left(t\right) \end{equation} Then, by the $\left(p,q\right)$-adic triangle inequality (\ref{eq:(p,q)-adic triangle inequality}): \[ \left|\sum_{\left|t\right|_{p}\leq p^{N}}\hat{f}\left(-t\right)\hat{\mu}\left(t\right)\right|_{q}\overset{\mathbb{R}}{=}\left|\int_{\mathbb{Z}_{p}}f\left(\mathfrak{z}\right)\tilde{\mu}_{N}\left(\mathfrak{z}\right)d\mathfrak{z}\right|_{q}\leq\left\Vert f\cdot\tilde{\mu}_{N}\right\Vert _{p,q}\leq\left\Vert f\right\Vert _{p,q}\left\Vert \tilde{\mu}_{N}\right\Vert _{p,q} \] Because the upper bound tends to $0$ in $\mathbb{R}$ as $N\rightarrow\infty$, and because: \begin{equation} \lim_{N\rightarrow\infty}\sum_{\left|t\right|_{p}\leq p^{N}}\hat{f}\left(-t\right)\hat{\mu}\left(t\right)\overset{\mathbb{C}_{q}}{=}\int_{\mathbb{Z}_{p}}f\left(\mathfrak{z}\right)d\mu\left(\mathfrak{z}\right) \end{equation} this shows that: \begin{equation} \left|\int_{\mathbb{Z}_{p}}f\left(\mathfrak{z}\right)d\mu\left(\mathfrak{z}\right)\right|_{q}\overset{\mathbb{R}}{=}\lim_{N\rightarrow\infty}\left|\sum_{\left|t\right|_{p}\leq p^{N}}\hat{f}\left(-t\right)\hat{\mu}\left(t\right)\right|_{q}\leq\left\Vert f\right\Vert _{p,q}\cdot\lim_{N\rightarrow\infty}\left\Vert \tilde{\mu}_{N}\right\Vert _{p,q}=0 \end{equation} Since $f$ was arbitrary, we conclude that $d\mu$ is the zero measure. Q.E.D. \vphantom{}Next, we have the \textbf{Fourier resummation lemmata} which\index{resummation lemmata} will synergize with our analysis of quasi-integrable functions. To make these easier to state and remember, here are some definitions describing the conditions which the lemmata will impose on $\hat{\mu}$. \begin{defn} Consider a field $\mathbb{F}$ of characteristic $0$, a function $\kappa:\mathbb{N}_{0}\rightarrow\mathbb{F}$, and let $K$ be a metrically complete valued field extension of $\mathbb{F}$ which contains $\kappa\left(\mathbb{N}_{0}\right)$. \vphantom{} I. We say $\kappa$ is \index{tame}\textbf{$\left(p,K\right)$-adically tame} on a set $X\subseteq\mathbb{Z}_{p}$ whenever $\lim_{n\rightarrow\infty}\left|\kappa\left(\left[\mathfrak{z}\right]_{p^{n}}\right)\right|_{K}=0$ for all $\mathfrak{z}\in X$. We do not mention $X$ when $X=\mathbb{Z}_{p}$. If $K$ is a $q$-adic field, we say $\kappa$ is $\left(p,q\right)$-adically tame on $X$; if $K$ is archimedean, we say $\kappa$ is $\left(p,\infty\right)$-adically tame on $X$. \vphantom{} II. For a prime $p$, we say $\kappa$ has \index{$p$-adic!structure}\textbf{$p$-adic structure}\footnote{It would seem that this property is necessary in order for our computations to simplify in interesting ways. Life is much more difficult without it.}\textbf{ }whenever there are constants $a_{0},\ldots,a_{p-1}\in\kappa\left(\mathbb{N}_{0}\right)$ so that: \begin{equation} \kappa\left(pn+j\right)=a_{j}\kappa\left(n\right),\textrm{ }\forall n\geq0,\textrm{ }\forall j\in\left\{ 0,\ldots,p-1\right\} \label{eq:Definition of left-ended p-adic structural equations} \end{equation} We call (\ref{eq:Definition of left-ended p-adic structural equations}) the \textbf{left-ended structural equations }of $\kappa$. \end{defn} \begin{rem} Note that the set of functions $\kappa:\mathbb{N}_{0}\rightarrow\mathbb{F}$ with $p$-adic structure is then a linear space over $\mathbb{F}$, as well as over any field extension $K$ of $\mathbb{F}$. \end{rem} \begin{rem} Comparing the above to what we did with $\chi_{H}$, one could distinguish between (\ref{eq:Definition of left-ended p-adic structural equations}) (which could be called \textbf{linear $p$-adic structure}) and a more $\chi_{H}$-like structure: \begin{equation} \kappa\left(pn+j\right)=a_{j}\kappa\left(n\right)+b_{j} \end{equation} which we would call \textbf{affine $p$-adic structure}. For the present purposes however, we shall only concern ourselves with linear $p$-adic structure, so all mentions of ``$p$-adic structure'' in this and any subsequent part of this dissertation means only the \emph{linear }variety of $p$-adic structure defined in (\ref{eq:Definition of left-ended p-adic structural equations}), unless specifically stated otherwise. \end{rem} As defined, functions with $p$-adic structure clearly generalize the systems of functional equations that characterized $\chi_{H}$ in Chapter 2. Because the functional equations satisfied by these functions appear to be crucial when it comes to working with quasi-integrable functions, it is important that we establish their properties as a general class of functions. As for the terminology, I use ``left-ended'' to describe the equations in (\ref{eq:Definition of left-ended p-adic structural equations}) because, given an integer $m=pn+j$ (where $j=\left[m\right]_{p}$), they show what happens when we pull out the left-most $p$-adic digit of $m$. Naturally, there is a ``right-ended'' counterpart to (\ref{eq:Definition of left-ended p-adic structural equations}), obtained by pulling out the right-most $p$-adic digit of $m$. Moreover, as the next proposition shows, $\kappa$ satisfies one of these two systems of functional equations if and only if it satisfies both systems. \begin{prop}[\textbf{Properties of functions with $p$-adic structure}] \label{prop:properties of functions with p-adic structure} Let $\mathbb{F}$ be any field of characteristic zero, and consider a function $\kappa:\mathbb{N}_{0}\rightarrow\mathbb{F}$. Then, $\kappa$ has $p$-adic structure if and only if there are constants $a_{0}^{\prime},\ldots,a_{p-1}^{\prime}$ so that: \begin{equation} \kappa\left(n+jp^{k}\right)=a_{j}^{\prime}\kappa\left(n\right),\textrm{ }\forall n\geq0,\textrm{ }\forall m\in\left\{ 0,\ldots,p^{n}-1\right\} ,\textrm{ }\forall j\in\left\{ 1,\ldots,p-1\right\} \label{eq:Definition of right-ended p-adic structural equations} \end{equation} We call \emph{(\ref{eq:Definition of right-ended p-adic structural equations})} the \textbf{right-ended structural equations }of $\kappa$. \end{prop} Proof: Fix an integer $m\in\mathbb{N}_{0}$. Then, we can write $m$ uniquely in $p$-adic form as: \begin{equation} m=\sum_{\ell=0}^{\lambda_{p}\left(m\right)-1}m_{\ell}p^{\ell} \end{equation} where $m_{\ell}\in\left\{ 0,\ldots,p-1\right\} $ for all $\ell$, with $m_{\lambda_{p}\left(m\right)-1}\neq0$. I. Let $\kappa$ satisfy the left-ended structural equations (\ref{eq:Definition of left-ended p-adic structural equations}). Then: \begin{align*} \kappa\left(m\right) & =\kappa\left(p\left(\sum_{\ell=1}^{\lambda_{p}\left(m\right)-1}m_{\ell}p^{\ell-1}\right)+m_{0}\right)\\ \left(\textrm{use }(\ref{eq:Definition of left-ended p-adic structural equations})\right); & =a_{m_{0}}\kappa\left(\sum_{\ell=1}^{\lambda_{p}\left(m\right)-1}m_{\ell}p^{\ell-1}\right)\\ & \vdots\\ & =\left(\prod_{\ell=0}^{\lambda_{p}\left(m\right)-1}a_{m_{\ell}}\right)\kappa\left(0\right) \end{align*} Pulling out the right-most $p$-adic digit of $m$ gives: \begin{equation} m=\left(\sum_{\ell=0}^{\lambda_{p}\left(m\right)-2}m_{\ell}p^{\ell-1}\right)+m_{\lambda_{p}\left(m\right)-1}p^{\lambda_{p}\left(m\right)-1} \end{equation} Since the above implies: \begin{equation} \kappa\left(\sum_{\ell=0}^{\lambda_{p}\left(m\right)-2}m_{\ell}p^{\ell-1}\right)=\left(\prod_{\ell=0}^{\lambda_{p}\left(m\right)-2}a_{m_{\ell}}\right)\kappa\left(0\right) \end{equation} it follows that: \begin{align*} \left(\prod_{\ell=1}^{\lambda_{p}\left(m\right)-1}a_{m_{\ell}}\right)\kappa\left(0\right) & =\kappa\left(m\right)\\ & =\kappa\left(\sum_{\ell=0}^{\lambda_{p}\left(m\right)-2}m_{\ell}p^{\ell-1}+m_{\lambda_{p}\left(m\right)-1}p^{\lambda_{p}\left(m\right)-1}\right)\\ & =\left(\prod_{\ell=0}^{\lambda_{p}\left(m\right)-1}a_{m_{\ell}}\right)\kappa\left(0\right)\\ & =a_{m_{\lambda_{p}\left(m\right)-1}}\left(\prod_{\ell=0}^{\lambda_{p}\left(m\right)-2}a_{m_{\ell}}\right)\kappa\left(0\right)\\ & =a_{m_{\lambda_{p}\left(m\right)-1}}\kappa\left(\sum_{\ell=0}^{\lambda_{p}\left(m\right)-2}m_{\ell}p^{\ell-1}\right) \end{align*} This proves (\ref{eq:Definition of right-ended p-adic structural equations}). \vphantom{} II. Supposing instead that $\kappa$ satisfies (\ref{eq:Definition of right-ended p-adic structural equations}), perform the same argument as for (I), but with $a^{\prime}$s instead of $a$s, and pulling out $a_{m_{0}}$ instead of $a_{m_{\lambda_{p}\left(m\right)-1}}$. This yields (\ref{eq:Definition of left-ended p-adic structural equations}). Q.E.D. \begin{prop} \label{prop:value of a p-adic structure at 0}Let $\kappa:\mathbb{N}_{0}\rightarrow\mathbb{F}$ have linear $p$-adic structure, and be non-identically zero. Then the constants $a_{j}$ and $a_{j}^{\prime}$ from the left- and right-ended structural equations for $\kappa$ are: \begin{align} a_{j} & =\kappa\left(j\right)\\ a_{j}^{\prime} & =\frac{\kappa\left(j\right)}{\kappa\left(0\right)} \end{align} \end{prop} Proof: Use the structural equations to evaluate $\kappa\left(j\right)$ for $j\in\left\{ 0,\ldots,p-1\right\} $. Q.E.D. \begin{prop} \label{prop:formula for functions with p-adic structure}Let $\left\{ c_{n}\right\} _{n\geq0}$ be a sequence in $\left\{ 0,\ldots,p-1\right\} $. If $\kappa$ is non-zero and has linear $p$-adic structure: \begin{equation} \kappa\left(\sum_{m=0}^{N-1}c_{n}p^{m}\right)=\frac{1}{\left(\kappa\left(0\right)\right)^{N}}\prod_{n=0}^{N-1}\kappa\left(c_{n}\right)\label{eq:Kappa action in terms of p-adic digits} \end{equation} \end{prop} Proof: Use $\kappa$'s right-ended structural equations along with the formula for the $a_{j}^{\prime}$s given in \textbf{Proposition \ref{prop:value of a p-adic structure at 0}}. Q.E.D. \begin{lem} \label{lem:structural equations uniquely determine p-adic structured functions}Let $\mathbb{F}$ be any field of characteristic zero. Then, every $p$-adically structured $\kappa:\mathbb{N}_{0}\rightarrow\mathbb{F}$ is uniquely determined by its structural equations. Moreover: \vphantom{} I. $\kappa$ is identically zero if and only if $\kappa\left(0\right)=0$. \vphantom{} II. There exists an $n\in\mathbb{N}_{0}$ so that $\kappa\left(n\right)\neq0$ if and only if $\kappa\left(0\right)=1$. \end{lem} Proof: We first prove that $\kappa$ is identically zero if and only if $\kappa\left(0\right)=0$. To do this, we need only show that $\kappa\left(0\right)=0$ implies $\kappa$ is identically zero. So, let $x$ be an arbitrary integer $\geq1$. Observe that every positive integer $x$ can be written in the form $x_{-}+jp^{n}$, where $n=\lambda_{p}\left(x\right)$, where $x_{-}$ is the integer obtained by deleting $j$\textemdash the right-most digit of $x$'s $p$-adic representation. Moreover, by construction, $x_{-}$ is in $\left\{ 0,\ldots,p^{n}-1\right\} $. So, since $\kappa$ has linear $p$-adic structure, the right-ended structural equations yield: \begin{equation} \kappa\left(x\right)=\kappa\left(x_{-}+jp^{n}\right)=c_{j}\kappa\left(x_{-}\right) \end{equation} Noting that the map $n\in\mathbb{N}_{0}\mapsto n_{-}\in\mathbb{N}_{0}$ eventually iterates every non-negative integer to $0$, we can write: \begin{equation} \kappa\left(x\right)=\left(\prod_{j}c_{j}\right)\kappa\left(0\right)=0 \end{equation} Here, the product is taken over finitely many $j$s. In particular, these $j$s are precisely the non-zero $p$-adic digits of $x$. Since $x$ was an arbitrary integer $\geq1$, and since $\kappa$ was given to vanish on $\left\{ 0,\ldots,p-1\right\} $, this shows that $\kappa$ is identically zero whenever $\kappa\left(0\right)=0$. Conversely, if $\kappa$ is identically zero, then, necessarily, $\kappa\left(0\right)=0$. Next, suppose $\kappa$ is $p$-adically structured and is \emph{not} identically zero. Then, by the left-ended structural equations (\ref{eq:Definition of left-ended p-adic structural equations}), we have $\kappa\left(0\right)=a_{0}\kappa\left(0\right)$, where $\kappa\left(0\right)\neq0$. This forces $a_{0}=1$. Applying \textbf{Proposition \ref{prop:value of a p-adic structure at 0}} gives $\kappa\left(0\right)=a_{0}$. So, $\kappa\left(0\right)=1$. Finally, let $\kappa_{1}$ and $\kappa_{2}$ be two $p$-adically structured functions satisfying the same set of structural equations. Then, $\kappa_{1}-\kappa_{2}$ is then $p$-adically structured and satisfies: \begin{equation} \left(\kappa_{1}-\kappa_{2}\right)\left(0\right)=\kappa_{1}\left(0\right)-\kappa_{2}\left(0\right)=1-1=0 \end{equation} which, as we just saw, then forces $\kappa_{1}-\kappa_{2}$ to be identically zero. This proves that $\kappa$ is uniquely determined by its structural equations. Q.E.D. \vphantom{} For us, the most important case will be when $\kappa$ is both $\left(p,K\right)$-adically tame over $\mathbb{Z}_{p}^{\prime}$ \emph{and }$p$-adically structured. \begin{defn} Let $\mathbb{F}$ be a field of characteristic $0$, and let $K$ be a metrically complete valued field extension of $\mathbb{F}$. We say $\kappa$ is \textbf{$\left(p,K\right)$-adically regular} whenever\index{$p,q$-adic!regular function} it has linear $p$-adic structure and is $\left(p,K\right)$-adically tame over $\mathbb{Z}_{p}^{\prime}$. In the case $K$ is a $q$-adic field, we call this \textbf{$\left(p,q\right)$-adic regularity};\textbf{ }when $K$ is $\mathbb{R}$ or $\mathbb{C}$, we call this \textbf{$\left(p,\infty\right)$-adic regularity} when $K=\mathbb{C}$. \end{defn} \vphantom{} This might seem like a lot of definitions, but we are really only just beginning to explore the possibilities. The next few definitions describe various important ``standard forms'' for $\left(p,q\right)$-adic measures in terms of their Fourier-Stieltjes coefficients. \begin{defn} Let $K$ be an algebraically closed field of characteristic zero and consider a function $\hat{\mu}:\hat{\mathbb{Z}}_{p}\rightarrow K$. \vphantom{} I. We say $\hat{\mu}$ is \textbf{radially symmetric }whenever\index{radially symmetric}: \begin{equation} \hat{\mu}\left(t\right)=\hat{\mu}\left(\frac{1}{\left|t\right|_{p}}\right),\textrm{ }\forall t\in\hat{\mathbb{Z}}_{p}\backslash\left\{ 0\right\} \label{eq:Definition of a radially symmetric function} \end{equation} That is, the value of $\hat{\mu}\left(t\right)$ depends only on the $p$-adic absolute value of $t$. Similarly, we say a measure $d\mu$ is \textbf{radially symmetric} whenever it has a radially symmetric Fourier-Stieltjes transform. \vphantom{} II. We say $\hat{\mu}$ is \textbf{magnitudinal }whenever\index{magnitudinal} there is a function $\kappa:\mathbb{N}_{0}\rightarrow K$ satisfying: \begin{equation} \hat{\mu}\left(t\right)=\begin{cases} \kappa\left(0\right) & \textrm{if }t=0\\ \sum_{m=0}^{\left|t\right|_{p}-1}\kappa\left(m\right)e^{-2\pi imt} & \textrm{else} \end{cases},\textrm{ }\forall t\in\hat{\mathbb{Z}}_{p}\label{eq:Definition of a Magnitudinal function} \end{equation} Here, we write $e^{-2\pi imt}$ to denote the $m$th power of the particular $\left|t\right|_{p}$th root of unity in $K$ indicated by $e^{-2\pi imt}$. We call an expression of the form of the right-hand side of (\ref{eq:Definition of a Magnitudinal function}) a \textbf{magnitudinal series}. Similarly, we say a measure $d\mu$ is \textbf{magnitudinal} whenever it has a magnitude dependent Fourier-Stieltjes transform. Additionally, for $q$ prime or $q=\infty$, we say that such a $\hat{\mu}$ and $d\mu$ are \textbf{$\left(p,q\right)$-adically regular and magnitudinal} whenever $\kappa$ is $\left(p,q\right)$-adically regular. We say $\hat{\mu}$ and $d\mu$ \textbf{have $p$-adic structure }whenever $\kappa$ does. \vphantom{} III. We say a function $\hat{\mu}:\hat{\mathbb{Z}}_{p}\rightarrow\mathbb{F}$ is \index{radially-magnitudinal}\textbf{radially-magnitudinal }if it is of the form $\hat{\mu}\left(t\right)=\hat{\nu}\left(t\right)\hat{\eta}\left(t\right)$ for a radially symmetric $\hat{\nu}$ and a magnitude\index{$p,q$-adic!regular magnitudinal measure} dependent $\hat{\eta}$. A \textbf{$\left(p,q\right)$-adically regular, radially-magnitudinal }$\hat{\mu}$ is one for which $\hat{\eta}$ is $\left(p,q\right)$-adically magnitudinal. \index{measure!regular, radially-magnitudinal} \end{defn} \begin{lem}[\textbf{Fourier Resummation \textendash{} Radially Symmetric Measures}] \textbf{\label{lem:Rad-Sym Fourier Resum Lemma}}Let\index{resummation lemmata} $\hat{\mu}:\hat{\mathbb{Z}}_{p}\rightarrow\overline{\mathbb{Q}}$ be radially symmetric. Then: \begin{equation} \sum_{\left|t\right|_{p}\leq p^{N}}\hat{\mu}\left(t\right)e^{2\pi i\left\{ t\mathfrak{z}\right\} _{p}}\overset{\overline{\mathbb{Q}}}{=}\begin{cases} \hat{\mu}\left(0\right)+\left(p-1\right)\sum_{n=1}^{N}p^{n-1}\hat{\mu}\left(\frac{1}{p^{n}}\right) & \textrm{for }\mathfrak{z}=0\textrm{ and }N\geq0\\ \sum_{n=0}^{v_{p}\left(\mathfrak{z}\right)}\left(\hat{\mu}\left(p^{-n}\right)-\hat{\mu}\left(p^{-n-1}\right)\right)p^{n} & \textrm{for }\mathfrak{z}\in\mathbb{Z}_{p}^{\prime}\textrm{ \& }N>v_{p}\left(\mathfrak{z}\right) \end{cases}\label{eq:Radial Fourier Resummation Lemma} \end{equation} The lower line can also be written as: \begin{equation} \sum_{n=0}^{v_{p}\left(\mathfrak{z}\right)}\left(\hat{\mu}\left(p^{-n}\right)-\hat{\mu}\left(p^{-n-1}\right)\right)p^{n}=\hat{\mu}\left(0\right)-\left|\mathfrak{z}\right|_{p}^{-1}\hat{\mu}\left(\frac{\left|\mathfrak{z}\right|_{p}}{p}\right)+\left(1-\frac{1}{p}\right)\sum_{n=1}^{v_{p}\left(\mathfrak{z}\right)}\hat{\mu}\left(\frac{1}{p^{n}}\right)p^{n}\label{eq:Radial Fourier Resummation - Lower Line Simplification} \end{equation} \end{lem} Proof: When $\mathfrak{z}=0$: \begin{align*} \sum_{\left|t\right|_{p}\leq p^{N}}\hat{\mu}\left(t\right)e^{2\pi i\left\{ t\mathfrak{z}\right\} _{p}} & =\sum_{\left|t\right|_{p}\leq p^{N}}\hat{\mu}\left(t\right)\\ & =\hat{\mu}\left(0\right)+\sum_{n=1}^{N}\sum_{\left|t\right|_{p}=p^{n}}\hat{\mu}\left(t\right)\\ \left(\hat{\mu}\textrm{ is radially symm.}\right); & =\hat{\mu}\left(0\right)+\sum_{n=1}^{N}\sum_{\left|t\right|_{p}=p^{n}}\hat{\mu}\left(\frac{1}{p^{n}}\right)\\ \left(\sum_{\left|t\right|_{p}=p^{n}}1=\varphi\left(p^{n}\right)=\left(p-1\right)p^{n-1}\right); & =\hat{\mu}\left(0\right)+\left(p-1\right)\sum_{n=1}^{N}p^{n-1}\hat{\mu}\left(\frac{1}{p^{n}}\right) \end{align*} Next, letting $\mathfrak{z}$ be non-zero: \begin{align*} \sum_{\left|t\right|_{p}\leq p^{N}}\hat{\mu}\left(t\right)e^{2\pi i\left\{ t\mathfrak{z}\right\} _{p}} & =\hat{\mu}\left(0\right)+\sum_{n=1}^{N}\sum_{\left|t\right|_{p}=p^{n}}\hat{\mu}\left(t\right)e^{2\pi i\left\{ t\mathfrak{z}\right\} _{p}}\\ \left(\hat{\mu}\textrm{ is radially symm.}\right); & =\hat{\mu}\left(0\right)+\sum_{n=1}^{N}\sum_{\left|t\right|_{p}=p^{n}}\hat{\mu}\left(\frac{1}{\left|t\right|_{p}}\right)e^{2\pi i\left\{ t\mathfrak{z}\right\} _{p}}\\ & =\hat{\mu}\left(0\right)+\sum_{n=1}^{N}\hat{\mu}\left(p^{-n}\right)\sum_{\left|t\right|_{p}=p^{n}}e^{2\pi i\left\{ t\mathfrak{z}\right\} _{p}}\\ & =\hat{\mu}\left(0\right)+\sum_{n=1}^{N}\hat{\mu}\left(p^{-n}\right)\left(p^{n}\left[\mathfrak{z}\overset{p^{n}}{\equiv}0\right]-p^{n-1}\left[\mathfrak{z}\overset{p^{n-1}}{\equiv}0\right]\right)\\ & =\hat{\mu}\left(0\right)+\sum_{n=1}^{N}\hat{\mu}\left(p^{-n}\right)p^{n}\left[\mathfrak{z}\overset{p^{n}}{\equiv}0\right]-\sum_{n=1}^{N}\hat{\mu}\left(p^{-n}\right)p^{n-1}\left[\mathfrak{z}\overset{p^{n-1}}{\equiv}0\right]\\ & =\hat{\mu}\left(0\right)+\sum_{n=1}^{N}\hat{\mu}\left(p^{-n}\right)p^{n}\left[\mathfrak{z}\overset{p^{n}}{\equiv}0\right]-\sum_{n=0}^{N-1}\hat{\mu}\left(p^{-n-1}\right)p^{n}\left[\mathfrak{z}\overset{p^{n}}{\equiv}0\right]\\ \left(\hat{\mu}\left(t+1\right)=\hat{\mu}\left(t\right)\right); & =p^{N}\hat{\mu}\left(p^{-N}\right)\left[\mathfrak{z}\overset{p^{N}}{\equiv}0\right]+\sum_{n=0}^{N-1}p^{n}\left(\hat{\mu}\left(p^{-n}\right)-\hat{\mu}\left(p^{-n-1}\right)\right)\left[\mathfrak{z}\overset{p^{n}}{\equiv}0\right]\\ \left(\mathfrak{z}\overset{p^{n}}{\equiv}0\Leftrightarrow n\leq v_{p}\left(\mathfrak{z}\right)\right); & =p^{N}\hat{\mu}\left(p^{-N}\right)\left[\mathfrak{z}\overset{p^{N}}{\equiv}0\right]+\sum_{n=0}^{\min\left\{ v_{p}\left(\mathfrak{z}\right)+1,N\right\} -1}\left(\hat{\mu}\left(p^{-n}\right)-\hat{\mu}\left(p^{-n-1}\right)\right)p^{n} \end{align*} Since $\mathfrak{z}\neq0$, $v_{p}\left(\mathfrak{z}\right)$ is an integer, and, as such, $\min\left\{ v_{p}\left(\mathfrak{z}\right)+1,N\right\} =v_{p}\left(\mathfrak{z}\right)+1$ for all sufficiently large $N$\textemdash specifically, all $N\geq1+v_{p}\left(\mathfrak{z}\right)$. For such $N$, we also have that $\mathfrak{z}\overset{p^{N}}{\cancel{\equiv}}0$, and thus, that: \begin{equation} \sum_{\left|t\right|_{p}\leq p^{N}}\hat{\mu}\left(t\right)e^{2\pi i\left\{ t\mathfrak{z}\right\} _{p}}=\sum_{n=0}^{v_{p}\left(\mathfrak{z}\right)}\left(\hat{\mu}\left(p^{-n}\right)-\hat{\mu}\left(p^{-n-1}\right)\right)p^{n},\textrm{ }\forall\mathfrak{z}\in\mathbb{Z}_{p}^{\prime},\textrm{ }\forall N>v_{p}\left(\mathfrak{z}\right) \end{equation} As for (\ref{eq:Radial Fourier Resummation - Lower Line Simplification}), we can write: \begin{align*} \sum_{n=0}^{v_{p}\left(\mathfrak{z}\right)}\left(\hat{\mu}\left(p^{-n}\right)-\hat{\mu}\left(p^{-n-1}\right)\right)p^{n} & =\sum_{n=0}^{v_{p}\left(\mathfrak{z}\right)}\hat{\mu}\left(\frac{1}{p^{n}}\right)p^{n}-\frac{1}{p}\sum_{n=0}^{v_{p}\left(\mathfrak{z}\right)}\hat{\mu}\left(\frac{1}{p^{n+1}}\right)p^{n+1}\\ & =\sum_{n=0}^{v_{p}\left(\mathfrak{z}\right)}\hat{\mu}\left(\frac{1}{p^{n}}\right)p^{n}-\frac{1}{p}\sum_{n=1}^{v_{p}\left(\mathfrak{z}\right)+1}\hat{\mu}\left(\frac{1}{p^{n}}\right)p^{n}\\ & =\hat{\mu}\left(0\right)-p^{v_{p}\left(\mathfrak{z}\right)}\hat{\mu}\left(\frac{p^{-v_{p}\left(\mathfrak{z}\right)}}{p}\right)+\left(1-\frac{1}{p}\right)\sum_{n=1}^{v_{p}\left(\mathfrak{z}\right)}\hat{\mu}\left(\frac{1}{p^{n}}\right)p^{n}\\ & =\hat{\mu}\left(0\right)-\left|\mathfrak{z}\right|_{p}^{-1}\hat{\mu}\left(\frac{\left|\mathfrak{z}\right|_{p}}{p}\right)+\left(1-\frac{1}{p}\right)\sum_{n=1}^{v_{p}\left(\mathfrak{z}\right)}\hat{\mu}\left(\frac{1}{p^{n}}\right)p^{n} \end{align*} Q.E.D. \begin{lem}[\textbf{Fourier Resummation \textendash{} Magnitudinal measures}] Let \index{resummation lemmata}$\hat{\mu}:\hat{\mathbb{Z}}_{p}\rightarrow\overline{\mathbb{Q}}$ be magnitudinal and not identically $0$. Then, for all $N\in\mathbb{N}_{1}$ and all $\mathfrak{z}\in\mathbb{Z}_{p}$: \begin{equation} \sum_{\left|t\right|_{p}\leq p^{N}}\hat{\mu}\left(t\right)e^{2\pi i\left\{ t\mathfrak{z}\right\} _{p}}\overset{\overline{\mathbb{Q}}}{=}p^{N}\kappa\left(\left[\mathfrak{z}\right]_{p^{N}}\right)-\sum_{n=0}^{N-1}\sum_{j=1}^{p-1}p^{n}\kappa\left(\left[\mathfrak{z}\right]_{p^{n}}+jp^{n}\right)\label{eq:Magnitude Fourier Resummation Lemma} \end{equation} If, in addition, $\hat{\mu}$ has $p$-adic structure, then: \begin{equation} \sum_{\left|t\right|_{p}\leq p^{N}}\hat{\mu}\left(t\right)e^{2\pi i\left\{ t\mathfrak{z}\right\} _{p}}\overset{\overline{\mathbb{Q}}}{=}p^{N}\kappa\left(\left[\mathfrak{z}\right]_{p^{N}}\right)-\left(\sum_{j=1}^{p-1}\kappa\left(j\right)\right)\sum_{n=0}^{N-1}p^{n}\kappa\left(\left[\mathfrak{z}\right]_{p^{n}}\right)\label{eq:Magnitudinal Fourier Resummation Lemma - p-adically distributed case} \end{equation} \end{lem} Proof: \begin{align*} \sum_{\left|t\right|_{p}\leq p^{N}}\hat{\mu}\left(t\right)e^{2\pi i\left\{ t\mathfrak{z}\right\} _{p}} & =\hat{\mu}\left(0\right)+\sum_{n=1}^{N}\sum_{\left|t\right|_{p}=p^{n}}\hat{\mu}\left(t\right)e^{2\pi i\left\{ t\mathfrak{z}\right\} _{p}}\\ & =\hat{\mu}\left(0\right)+\sum_{n=1}^{N}\sum_{\left|t\right|_{p}=p^{n}}\left(\sum_{m=0}^{p^{n}-1}\kappa\left(m\right)e^{-2\pi imt}\right)e^{2\pi i\left\{ t\mathfrak{z}\right\} _{p}}\\ & =\hat{\mu}\left(0\right)+\sum_{n=1}^{N}\sum_{m=0}^{p^{n}-1}\kappa\left(m\right)\sum_{\left|t\right|_{p}=p^{n}}e^{2\pi i\left\{ t\left(\mathfrak{z}-m\right)\right\} _{p}}\\ & =\hat{\mu}\left(0\right)+\sum_{n=1}^{N}\sum_{m=0}^{p^{n}-1}\kappa\left(m\right)\left(p^{n}\left[\mathfrak{z}\overset{p^{n}}{\equiv}m\right]-p^{n-1}\left[\mathfrak{z}\overset{p^{n-1}}{\equiv}m\right]\right) \end{align*} Next, note that as $m$ varies in $\left\{ 0,\ldots,p^{N}-1\right\} $, the only value of $m$ satisfying $\mathfrak{z}\overset{p^{N}}{\equiv}m$ is $m=\left[\mathfrak{z}\right]_{p^{N}}$. Consequently: \begin{equation} \sum_{m=0}^{p^{n}-1}\kappa\left(m\right)p^{n}\left[\mathfrak{z}\overset{p^{n}}{\equiv}m\right]=p^{n}\kappa\left(\left[\mathfrak{z}\right]_{p^{n}}\right) \end{equation} On the other hand: \begin{align*} \sum_{m=0}^{p^{n}-1}\kappa\left(m\right)p^{n-1}\left[\mathfrak{z}\overset{p^{n-1}}{\equiv}m\right] & =\sum_{m=p^{n-1}}^{p^{n}-1}\kappa\left(m\right)p^{n-1}\left[\mathfrak{z}\overset{p^{n-1}}{\equiv}m\right]+\sum_{m=0}^{p^{n-1}-1}\kappa\left(m\right)p^{n-1}\left[\mathfrak{z}\overset{p^{n-1}}{\equiv}m\right]\\ & =\sum_{m=0}^{p^{n}-p^{n-1}-1}\kappa\left(m+p^{n-1}\right)p^{n-1}\left[\mathfrak{z}\overset{p^{n-1}}{\equiv}m+p^{n-1}\right]+p^{n-1}\kappa\left(\left[\mathfrak{z}\right]_{p^{n-1}}\right)\\ & =\sum_{m=0}^{\left(p-1\right)p^{n-1}-1}\kappa\left(m+p^{n-1}\right)p^{n-1}\left[\mathfrak{z}\overset{p^{n-1}}{\equiv}m\right]+p^{n-1}\kappa\left(\left[\mathfrak{z}\right]_{p^{n-1}}\right)\\ & =\sum_{j=1}^{p-1}\sum_{m=\left(j-1\right)p^{n-1}}^{jp^{n-1}-1}\kappa\left(m+p^{n-1}\right)p^{n-1}\left[\mathfrak{z}\overset{p^{n-1}}{\equiv}m\right]+p^{n-1}\kappa\left(\left[\mathfrak{z}\right]_{p^{n-1}}\right)\\ & =\sum_{j=1}^{p-1}\sum_{m=0}^{p^{n-1}-1}\kappa\left(m+jp^{n-1}\right)p^{n-1}\left[\mathfrak{z}\overset{p^{n-1}}{\equiv}m\right]+p^{n-1}\kappa\left(\left[\mathfrak{z}\right]_{p^{n-1}}\right)\\ & =\sum_{j=1}^{p-1}p^{n-1}\kappa\left(\left[\mathfrak{z}\right]_{p^{n-1}}+jp^{n-1}\right)+p^{n-1}\kappa\left(\left[\mathfrak{z}\right]_{p^{n-1}}\right)\\ & =\sum_{j=0}^{p-1}p^{n-1}\kappa\left(\left[\mathfrak{z}\right]_{p^{n-1}}+jp^{n-1}\right) \end{align*} Thus: \begin{align*} \sum_{\left|t\right|_{p}\leq p^{N}}\hat{\mu}\left(t\right)e^{2\pi i\left\{ t\mathfrak{z}\right\} _{p}} & =\hat{\mu}\left(0\right)+\sum_{n=1}^{N}\sum_{m=0}^{p^{n}-1}\kappa\left(m\right)\left(p^{n}\left[\mathfrak{z}\overset{p^{n}}{\equiv}m\right]-p^{n-1}\left[\mathfrak{z}\overset{p^{n-1}}{\equiv}m\right]\right)\\ & =\hat{\mu}\left(0\right)+\sum_{n=1}^{N}\left(p^{n}\kappa\left(\left[\mathfrak{z}\right]_{p^{n}}\right)-\sum_{j=0}^{p-1}p^{n-1}\kappa\left(\left[\mathfrak{z}\right]_{p^{n-1}}+jp^{n-1}\right)\right)\\ & =\hat{\mu}\left(0\right)+\underbrace{\sum_{n=1}^{N}\left(p^{n}\kappa\left(\left[\mathfrak{z}\right]_{p^{n}}\right)-p^{n-1}\kappa\left(\left[\mathfrak{z}\right]_{p^{n-1}}\right)\right)}_{\textrm{telescoping}}-\sum_{n=1}^{N}\sum_{j=1}^{p-1}p^{n-1}\kappa\left(\left[\mathfrak{z}\right]_{p^{n-1}}+jp^{n-1}\right)\\ & =\hat{\mu}\left(0\right)-\underbrace{\kappa\left(0\right)}_{\hat{\mu}\left(0\right)}+p^{N}\kappa\left(\left[\mathfrak{z}\right]_{p^{N}}\right)-\sum_{n=0}^{N-1}\sum_{j=1}^{p-1}p^{n}\kappa\left(\left[\mathfrak{z}\right]_{p^{n}}+jp^{n}\right) \end{align*} as desired. Finally, when $\kappa$ has $p$-adic structure, $\kappa$'s right-ended structural equations let us write: \[ \sum_{n=0}^{N-1}\sum_{j=1}^{p-1}p^{n}\kappa\left(\left[\mathfrak{z}\right]_{p^{n}}+jp^{n}\right)=\left(\sum_{j=1}^{p-1}\kappa\left(j\right)\right)\sum_{n=0}^{N-1}p^{n}\kappa\left(\left[\mathfrak{z}\right]_{p^{n}}\right) \] Q.E.D. \begin{lem}[\textbf{Fourier Resummation \textendash{} Radially-Magnitudinal measures}] \textbf{\label{lem:Radially-Mag Fourier Resummation Lemma}} Let $\hat{\mu}:\hat{\mathbb{Z}}_{p}\rightarrow\overline{\mathbb{Q}}$ be a function which is not identically $0$, and suppose that $\hat{\mu}\left(t\right)=\hat{\nu}\left(t\right)\hat{\eta}\left(t\right)$, where $\hat{\nu},\hat{\eta}:\hat{\mathbb{Z}}_{p}\rightarrow\overline{\mathbb{Q}}$ are radially symmetric and magnitude-dependent, respectively. Then, for all $N\in\mathbb{N}_{1}$ and all $\mathfrak{z}\in\mathbb{Z}_{p}$: \begin{align} \sum_{\left|t\right|_{p}\leq p^{N}}\hat{\mu}\left(t\right)e^{2\pi i\left\{ t\mathfrak{z}\right\} _{p}} & \overset{\overline{\mathbb{Q}}}{=}\hat{\nu}\left(0\right)\kappa\left(0\right)+\sum_{n=1}^{N}\hat{\nu}\left(p^{-n}\right)\left(p^{n}\kappa\left(\left[\mathfrak{z}\right]_{p^{n}}\right)-p^{n-1}\kappa\left(\left[\mathfrak{z}\right]_{p^{n-1}}\right)\right)\label{eq:Radial-Magnitude Fourier Resummation Lemma}\\ & -\sum_{j=1}^{p-1}\sum_{n=1}^{N}p^{n-1}\hat{\nu}\left(p^{-n}\right)\kappa\left(\left[\mathfrak{z}\right]_{p^{n-1}}+jp^{n-1}\right)\nonumber \end{align} If, in addition, $\kappa$ has $p$-adic structure, then: \begin{align} \sum_{\left|t\right|_{p}\leq p^{N}}\hat{\mu}\left(t\right)e^{2\pi i\left\{ t\mathfrak{z}\right\} _{p}} & \overset{\overline{\mathbb{Q}}}{=}p^{N}\hat{\nu}\left(p^{-N}\right)\kappa\left(\left[\mathfrak{z}\right]_{p^{N}}\right)\label{eq:Radial-Magnitude Fourier Resummation Lemma - p-adically distributed case}\\ & +\sum_{n=0}^{N-1}\left(\hat{\nu}\left(p^{-n}\right)-\left(\sum_{j=0}^{p-1}\kappa\left(j\right)\right)\hat{\nu}\left(p^{-n-1}\right)\right)\kappa\left(\left[\mathfrak{z}\right]_{p^{n}}\right)p^{n}\nonumber \end{align} \end{lem} \begin{rem} In order to make things more compact, we will frequently adopt the notation: \begin{align} A & \overset{\textrm{def}}{=}\sum_{j=1}^{p-1}\kappa\left(j\right)\label{eq:Definition of Big A}\\ v_{n} & \overset{\textrm{def}}{=}\hat{\nu}\left(\frac{1}{p^{n}}\right)\label{eq:Definition of v_n} \end{align} so that (\ref{eq:Radial-Magnitude Fourier Resummation Lemma - p-adically distributed case}) becomes: \begin{equation} \sum_{\left|t\right|_{p}\leq p^{N}}\hat{\mu}\left(t\right)e^{2\pi i\left\{ t\mathfrak{z}\right\} _{p}}\overset{\overline{\mathbb{Q}}}{=}v_{N}\kappa\left(\left[\mathfrak{z}\right]_{p^{N}}\right)p^{N}+\sum_{n=0}^{N-1}\left(v_{n}-\left(1+A\right)v_{n+1}\right)\kappa\left(\left[\mathfrak{z}\right]_{p^{n}}\right)p^{n}\label{eq:Radial-Magnitude p-adic distributed Fourier Resummation Identity, simplified} \end{equation} \end{rem} Proof: \begin{align*} \sum_{\left|t\right|_{p}\leq p^{N}}\hat{\mu}\left(t\right)e^{2\pi i\left\{ t\mathfrak{z}\right\} _{p}} & =\hat{\mu}\left(0\right)+\sum_{n=1}^{N}\sum_{\left|t\right|_{p}=p^{n}}\hat{\nu}\left(t\right)\hat{\eta}\left(t\right)e^{2\pi i\left\{ t\mathfrak{z}\right\} _{p}}\\ \left(\hat{\nu}\left(t\right)=\hat{\nu}\left(\frac{1}{\left|t\right|_{p}}\right)\right); & =\hat{\mu}\left(0\right)+\sum_{n=1}^{N}\hat{\nu}\left(p^{-n}\right)\sum_{\left|t\right|_{p}=p^{n}}\hat{\eta}\left(t\right)e^{2\pi i\left\{ t\mathfrak{z}\right\} _{p}} \end{align*} From (\ref{eq:Magnitude Fourier Resummation Lemma}), we have that: \[ \sum_{\left|t\right|_{p}=p^{n}}\hat{\eta}\left(t\right)e^{2\pi i\left\{ t\mathfrak{z}\right\} _{p}}=p^{n}\kappa\left(\left[\mathfrak{z}\right]_{p^{n}}\right)-\sum_{j=0}^{p-1}p^{n-1}\kappa\left(\left[\mathfrak{z}\right]_{p^{n-1}}+jp^{n-1}\right) \] and hence: \begin{align*} \sum_{\left|t\right|_{p}\leq p^{N}}\hat{\mu}\left(t\right)e^{2\pi i\left\{ t\mathfrak{z}\right\} _{p}} & =\hat{\mu}\left(0\right)+\sum_{n=1}^{N}\hat{\nu}\left(p^{-n}\right)\sum_{\left|t\right|_{p}=p^{n}}\hat{\eta}\left(t\right)e^{2\pi i\left\{ t\mathfrak{z}\right\} _{p}}\\ & =\hat{\mu}\left(0\right)+\sum_{n=1}^{N}\hat{\nu}\left(p^{-n}\right)\kappa\left(\left[\mathfrak{z}\right]_{p^{n}}\right)p^{n}\\ & -\sum_{j=0}^{p-1}\sum_{n=1}^{N}\hat{\nu}\left(p^{-n}\right)\kappa\left(\left[\mathfrak{z}\right]_{p^{n-1}}+jp^{n-1}\right)p^{n-1} \end{align*} If, in addition, $\hat{\mu}$ is $p$-adically distributed, then: \begin{align*} \sum_{\left|t\right|_{p}\leq p^{N}}\hat{\mu}\left(t\right)e^{2\pi i\left\{ t\mathfrak{z}\right\} _{p}} & =\hat{\mu}\left(0\right)+\sum_{n=1}^{N}\hat{\nu}\left(p^{-n}\right)\kappa\left(\left[\mathfrak{z}\right]_{p^{n}}\right)p^{n}\\ & -\left(\sum_{j=0}^{p-1}\kappa\left(j\right)\right)\sum_{n=1}^{N}\hat{\nu}\left(p^{-n}\right)\kappa\left(\left[\mathfrak{z}\right]_{p^{n-1}}\right)p^{n-1}\\ \left(\hat{\mu}\left(0\right)=\hat{\nu}\left(0\right)\underbrace{\hat{\eta}\left(0\right)}_{\kappa\left(0\right)}\right); & =\underbrace{\hat{\nu}\left(0\right)\kappa\left(0\right)-\hat{\nu}\left(\frac{1}{p}\right)\kappa\left(0\right)\left(\sum_{j=0}^{p-1}\kappa\left(j\right)\right)}_{n=0\textrm{ term of the series on the bottom line}}+p^{N}\hat{\nu}\left(p^{-N}\right)\kappa\left(\left[\mathfrak{z}\right]_{p^{N}}\right)\\ & +\sum_{n=1}^{N-1}\left(\hat{\nu}\left(p^{-n}\right)-\left(\sum_{j=0}^{p-1}\kappa\left(j\right)\right)\hat{\nu}\left(p^{-n-1}\right)\right)\kappa\left(\left[\mathfrak{z}\right]_{p^{n}}\right)p^{n} \end{align*} Q.E.D. \begin{prop}[\textbf{Convolution with $v_{p}$}] \label{prop:v_p of t times mu hat sum}Let $\hat{\mu}:\hat{\mathbb{Z}}_{p}\rightarrow\mathbb{C}_{q}$ be any function. Then: \begin{equation} \sum_{0<\left|t\right|_{p}\leq p^{N}}v_{p}\left(t\right)\hat{\mu}\left(t\right)e^{2\pi i\left\{ t\mathfrak{z}\right\} _{p}}\overset{\mathbb{C}_{q}}{=}-N\tilde{\mu}_{N}\left(\mathfrak{z}\right)+\sum_{n=0}^{N-1}\tilde{\mu}_{n}\left(\mathfrak{z}\right)\label{eq:Fourier sum of v_p times mu-hat} \end{equation} \end{prop} Proof: \begin{align*} \sum_{0<\left|t\right|_{p}\leq p^{N}}v_{p}\left(t\right)\hat{\mu}\left(t\right)e^{2\pi i\left\{ t\mathfrak{z}\right\} _{p}} & =\sum_{n=1}^{N}\sum_{\left|t\right|_{p}=p^{n}}\left(-n\right)\hat{\mu}\left(t\right)e^{2\pi i\left\{ t\mathfrak{z}\right\} _{p}}\\ & =-\sum_{n=1}^{N}n\left(\sum_{\left|t\right|_{p}\leq p^{n}}\hat{\mu}\left(t\right)e^{2\pi i\left\{ t\mathfrak{z}\right\} _{p}}-\sum_{\left|t\right|_{p}\leq p^{n-1}}\hat{\mu}\left(t\right)e^{2\pi i\left\{ t\mathfrak{z}\right\} _{p}}\right)\\ & =-\sum_{n=1}^{N}n\left(\tilde{\mu}_{n}\left(\mathfrak{z}\right)-\tilde{\mu}_{n-1}\left(\mathfrak{z}\right)\right)\\ & =\sum_{n=1}^{N}n\tilde{\mu}_{n-1}\left(\mathfrak{z}\right)-\sum_{n=1}^{N}n\tilde{\mu}_{n}\left(\mathfrak{z}\right)\\ & =\sum_{n=1}^{N}\left(\tilde{\mu}_{n-1}\left(\mathfrak{z}\right)+\left(n-1\right)\tilde{\mu}_{n-1}\left(\mathfrak{z}\right)\right)-\sum_{n=1}^{N}n\tilde{\mu}_{n}\left(\mathfrak{z}\right)\\ & =\sum_{n=1}^{N}\tilde{\mu}_{n-1}\left(\mathfrak{z}\right)+\sum_{n=0}^{N-1}\underbrace{n\tilde{\mu}_{n}\left(\mathfrak{z}\right)}_{0\textrm{ when }n=0}-\sum_{n=1}^{N}n\tilde{\mu}_{n}\left(\mathfrak{z}\right)\\ & =\sum_{n=0}^{N-1}\tilde{\mu}_{n}\left(\mathfrak{z}\right)-N\tilde{\mu}_{N}\left(\mathfrak{z}\right) \end{align*} Q.E.D. \subsection{\label{subsec:3.3.3 Frames}Frames} \begin{rem} In order to be fully comprehensive, this sub-section takes a significant dive into abstraction. This is meant primarily for readers with a good background in non-archimedean analysis, as well as anyone who can handle a significant number of definitions being thrown their way. For readers more interested in how all of this relates to $\chi_{H}$ and Hydra maps, they can safely skip this section so long as the keep the following concepts in mind: Consider a function $f:\mathbb{Z}_{p}\rightarrow\mathbb{C}_{q}$. A sequence of functions $\left\{ f_{n}\right\} _{n\geq1}$ on $\mathbb{Z}_{p}$ is said to \textbf{converge} \textbf{to $f$ with respect to the standard $\left(p,q\right)$-adic frame }if\index{frame!standard left(p,qright)-adic@standard $\left(p,q\right)$-adic}: \vphantom{} I. For all $n$, $f_{n}\left(\mathfrak{z}\right)\in\overline{\mathbb{Q}}$ for all $\mathfrak{z}\in\mathbb{Z}_{p}$. \vphantom{} II. For all $\mathfrak{z}\in\mathbb{N}_{0}$, $f\left(\mathfrak{z}\right)\in\overline{\mathbb{Q}}$ and $\lim_{n\rightarrow\infty}f_{n}\left(\mathfrak{z}\right)\overset{\mathbb{C}}{=}f\left(\mathfrak{z}\right)$ (meaning the convergence is in the topology of $\mathbb{C}$). \vphantom{} III. For all $\mathfrak{z}\in\mathbb{Z}_{p}^{\prime}$, $f\left(\mathfrak{z}\right)\in\mathbb{C}_{q}$ and $\lim_{n\rightarrow\infty}f_{n}\left(\mathfrak{z}\right)\overset{\mathbb{C}_{q}}{=}f\left(\mathfrak{z}\right)$ (meaning the convergence is in the topology of $\mathbb{C}_{q}$). \vphantom{} We write $\mathcal{F}_{p,q}$ to denote the standard $\left(p,q\right)$-adic frame and write $\lim_{n\rightarrow\infty}f_{n}\left(\mathfrak{z}\right)\overset{\mathcal{F}_{p,q}}{=}f\left(\mathfrak{z}\right)$ to denote convergence with respect to the standard frame. Also, if the $f_{n}$s are the partial sums of an infinite series, we say that $\lim_{n\rightarrow\infty}f_{n}\left(\mathfrak{z}\right)$ is an \textbf{$\mathcal{F}_{p,q}$-series }for/of\index{mathcal{F}-@$\mathcal{F}$-!series} $f$. \end{rem} \vphantom{} HERE BEGINS THE COMPREHENSIVE ACCOUNT OF FRAMES \vphantom{} To begin, let us recall the modern notion of what a ``function'' actually \emph{is}. Nowadays a ``function'' is merely a rule which to each element of a designated input set $A$ associates a single element of a designated output set, $B$. The first appearance of this type of understanding is generally attributed to Dirichlet. Prior to his work (and, honestly, for a good time after that), functions were often synonymous with what we now call analytic functions\textemdash those which are expressible as a power series, or some combination of functions of that type. This synonymy is understandable: power series are wonderful. Not only do they give us something concrete to manipulate, they even allow us to compute functions' values to arbitrary accuracy\textemdash{} provided that the series converges, of course. For example, suppose we wish to use the geometric series formula: \begin{equation} \sum_{n=0}^{\infty}z^{n}=\frac{1}{1-z} \end{equation} to compute the value of the function $1/\left(1-z\right)$. As we all know, for any real or complex number $z$ with $\left|z\right|<1$, the series will converge in $\mathbb{R}$ or $\mathbb{C}$ to the right hand side. However, \emph{what if we plug in $z=3/2$}? There, the topologies of $\mathbb{R}$ and $\mathbb{C}$ do not help us. While we \emph{could} use analytic continuation\textemdash rejiggering the series until we get something which converges at $3/2$\textemdash that particular method doesn't make much sense in non-archimedean analysis, let alone $\left(p,q\right)$-adic analysis, so that route isn't available to us. Instead, we can observe that our obstacle isn't the geometric series formula itself, but rather the topologies of $\mathbb{R}$ and $\mathbb{C}$. As long as we stubbornly insist on living in those archimedean universes and no others, we cannot sum the series as written for any $\left|z\right|\geq1$. However, just like with speculative fiction, there is no reason we must confine ourselves to a single universe. If we go through the magic portal to the world of the $3$-adic topology, we can easily compute the true value of $1/\left(1-z\right)$ at $z=3/2$ using the power series formula. There: \begin{equation} \sum_{n=0}^{\infty}\left(\frac{3}{2}\right)^{n}\overset{\mathbb{Z}_{3}}{=}\frac{1}{1-\frac{3}{2}} \end{equation} is a perfectly sensible statement, both ordinary and rigorously meaningful. Most importantly, even though we have \emph{summed }the series in an otherworldly topology, the answer ends up being an ordinary real number, one which agrees with the value of $1/\left(1-z\right)$ at $z=3/2$ in $\mathbb{R}$ or $\mathbb{C}$. More generally, for any rational number $p/q$ where $p$ and $q$ are co-prime integers with $q\neq0$, we can go about computing $1/\left(1-z\right)$ at $z=p/q$ by summing the series: \begin{equation} \sum_{n=0}^{\infty}\left(\frac{p}{q}\right)^{n} \end{equation} in $\mathbb{Z}_{p}$. That this all works is thanks to the universality of the geometric series universality \index{geometric series universality}(see page \pageref{fact:Geometric series universality}). In this way, we can get more mileage out of our series formulae. Even though the geometric series converges in the topology of $\mathbb{C}$ only for complex numbers $\left|z\right|<1$, we can make the formula work at every complex algebraic number $\alpha$ of absolute value $>1$ by summing: \begin{equation} \sum_{n=0}^{\infty}\alpha^{n} \end{equation} in the topology of the $\alpha$-adic completion of the field $\mathbb{Q\left(\alpha\right)}$. This then gives us the correct value of the complex-valued function $1/\left(1-z\right)$ at $z=\alpha$. The only unorthodoxy is the route we took to get there. Nevertheless, even this is fully in line with the modern notion of functions: the output a function assigns to a given input should be defined \emph{independently} of any method (if any) we use to explicitly compute it. Treating $1/\left(1-z\right)$ as a function from $\overline{\mathbb{Q}}$ to $\overline{\mathbb{Q}}$, we can use the geometric series to compute the value of $1/\left(1-z\right)$ at any $\alpha\in\overline{\mathbb{Q}}$ by choosing a topological universe (the space of $\alpha$-adic numbers) where the series formula happens to make sense. This discussion of $1/\left(1-z\right)$ contains in a nutshell all the key concepts that come into play when defining frames. In this dissertation, the $\left(p,q\right)$-adic functions like $\chi_{H}$ we desire to study \emph{will not }possess series representations which converge in a single topology at every point of the function's domain. However, if we allow ourselves to vary from point to point the specific topology we use to sum these series, we can arrange things so that the series is meaningful at every possible input value. That this all \emph{works} is thanks to the observation that the values we get by varying the topology in this way are the correct outputs that the function assigns to its inputs, even if\textemdash as stated above\textemdash the route we took to compute them happened to be unusual. So, when working in a certain topology $\mathcal{T}$, instead of admitting defeat and giving up hope when our series become non-convergent\textemdash or, worse, \emph{divergent}\textemdash in $\mathcal{T}$'s topology, we will instead hop over to a \emph{different} topological universe $\mathcal{T}^{\prime}$ in which the series \emph{is} convergent. It almost goes without saying that world-hopping in this way is fraught with pitfalls. As Hensel's famous blunder (see Subsection \ref{subsec:1.3.5 Hensel's-Infamous-Blunder}) perfectly illustrates, just because there exists \emph{some }topology in which a series converges, it does not mean that the sum of the series has meaning from the perspective of a different topology\textemdash the one exception, of course, being the universality of the geometric series. Though cumbersome, the abstractions presented in this subsection are a necessary security measure. We need them to ensure that the series and functions we work with will be compatible with the itinerary of world-hopping we will use to make sense of them. As alluded to in Subsection \ref{subsec:3.3.1 Heuristics-and-Motivations}, the general idea behind a quasi-integrable function $\chi$ on $\mathbb{Z}_{p}$ is that we can express $\chi$ as the limit of interest (\ref{eq:The Limit of Interest}) for some $\hat{\mu}:\hat{\mathbb{Z}}_{p}\rightarrow\overline{\mathbb{Q}}$. Moreover, as demonstrated by $\hat{A}_{3}$ and the discussion of the geometric series above, given a function $\chi:\mathbb{Z}_{p}\rightarrow\mathbb{K}$, it will be too restrictive to mandate that the partial sums of our Fourier series: \begin{equation} \sum_{\left|t\right|_{p}\leq p^{N}}\hat{\chi}\left(t\right)e^{2\pi i\left\{ t\mathfrak{z}\right\} _{p}} \end{equation} converge point-wise on $\mathbb{Z}_{p}$ with respect to a \emph{single }topology. Instead, we will only require that at every $\mathfrak{z}\in\mathbb{Z}_{p}$ for which the function $\chi\left(\mathfrak{z}\right)$ takes a finite value, either the sequence of partial sums in the limit of interest (\ref{eq:The Limit of Interest}) equals its limit for all sufficiently large $N$ (as was the case in \textbf{Proposition \ref{prop:sum of v_p}}), or, that there exists\emph{ }a valued field $K_{\mathfrak{z}}$ with absolute value $\left|\cdot\right|_{K_{\mathfrak{z}}}$ for which: \begin{equation} \lim_{N\rightarrow\infty}\left|\chi\left(\mathfrak{z}\right)-\sum_{\left|t\right|_{p}\leq p^{N}}\hat{\chi}\left(t\right)e^{2\pi i\left\{ t\mathfrak{z}\right\} _{p}}\right|_{K_{\mathfrak{z}}}=0\label{eq:Quasi-integrability in a nutshell} \end{equation} With this procedure in mind, frames will be formulated as a collection of pairs $\left(\mathfrak{z},K_{\mathfrak{z}}\right)$, where $\mathfrak{z}$ is a point in $\mathbb{Z}_{p}$ and $K_{\mathfrak{z}}$ is a field so that (\ref{eq:The Limit of Interest}) converges at $\mathfrak{z}$ in the topology of $K_{\mathfrak{z}}$. That being said, we still have to have \emph{some} limitations. In applying $K_{\mathfrak{z}}$-absolute values to an equation like (\ref{eq:Quasi-integrability in a nutshell}), it is implicitly required that all of the terms the sum are elements of $K_{\mathfrak{z}}$. To surmount this, note that for any $t$ and $\mathfrak{z}$, $e^{2\pi i\left\{ t\mathfrak{z}\right\} _{p}}$ is some root of unity. Roots of unity exist abstractly in any algebraically closed field. So, we can fit these complex exponentials inside all of the $K_{\mathfrak{z}}$s by requiring the $K_{\mathfrak{z}}$s to be \textbf{algebraically closed}. There is also the issue of $\chi$ itself. Suppose that there are $\mathfrak{a},\mathfrak{b}\in\mathbb{Z}_{p}$ so that $K_{\mathfrak{a}}=\mathbb{C}_{q}$ and $K_{\mathfrak{b}}=\mathbb{C}_{r}$, for distinct primes $p,q,r$? Unless it just so happens that $\chi\left(\mathfrak{a}\right)$ and $\chi\left(\mathfrak{b}\right)$ both lie in $\overline{\mathbb{Q}}$, we will not be able to deal with all of $\chi$ by forcing $\chi$'s co-domain to be a single field. So, we will need to be flexible. Just as we allow for the topology of convergence to vary from point to point, so too will we allow for the co-domain of our functions to be more than just one field. This then suggests the following set up: \begin{itemize} \item A ``big'' space in which the limits of our (Fourier) series shall live. While for us, this will end up always being in $\mathbb{C}_{q}$, in general, it can be more than a single field\footnote{Originally, I required the ``big'' space to be a single field, but, upon contemplating what would need to be done in order to make frames applicable to the study of $\chi_{H}$ in the case of a polygenic Hydra map, I realized I would need to allow for $\chi_{H}$ to take values in more than one field.}. \item A choice of topologies of convergence $\left(\mathfrak{z},K_{\mathfrak{z}}\right)$ for every point in $\mathbb{Z}_{p}$. These shall be topological universes we use to sum our Fourier series so as to arrive at elements of the big space. \item We should require both the big space and the $K_{\mathfrak{z}}$s, to contain $\overline{\mathbb{Q}}$, and will require the coefficients of our Fourier series to be contained \emph{in }$\overline{\mathbb{Q}}$. This way, we will be able to compute partial sums of Fourier series in all of the $K_{\mathfrak{z}}$s simultaneously by working in $\overline{\mathbb{Q}}$, the space they share in common. \end{itemize} To make this work, we will first start with a choice of the pairs $\left(\mathfrak{z},K_{\mathfrak{z}}\right)$. This will get us a ``$p$-adic frame'', denoted $\mathcal{F}$. We will then define $I\left(\mathcal{F}\right)$\textemdash the \textbf{image }of $\mathcal{F}$\textemdash to denote the big space; this will just be the union of all the $K_{\mathfrak{z}}$s. Lastly, we will have $C\left(\mathcal{F}\right)$, the $\overline{\mathbb{Q}}$-linear space of all the functions which are compatible with this set up; the archetypical elements of these spaces will be partial sums of Fourier series generated by functions $\hat{\mathbb{Z}}_{p}\rightarrow\overline{\mathbb{Q}}$. \begin{defn}[\textbf{Frames}] \label{def:Frames}A \textbf{$p$-adic quasi-integrability frame }\index{frame}(or just ``frame'', for short) $\mathcal{F}$ consists of the following pieces of data: \vphantom{} I. A non-empty set \nomenclature{$U_{\mathcal{F}}$}{domain of $\mathcal{F}$}$U_{\mathcal{F}}\subseteq\mathbb{Z}_{p}$, called the \textbf{domain }of $\mathcal{F}$.\index{frame!domain} \vphantom{} II. For every $\mathfrak{z}\in U_{\mathcal{F}}$, a topological field $K_{\mathfrak{z}}$ \nomenclature{$K_{\mathfrak{z}}$}{ } with $K_{\mathfrak{z}}\supseteq\overline{\mathbb{Q}}$. We allow for $K_{\mathfrak{z}}=\overline{\mathbb{Q}}$. Additionally, for any $\mathfrak{z}\in U_{\mathcal{F}}$, we require the topology of $K_{\mathfrak{z}}$ to be either the discrete topology\emph{ or} the topology induced by an absolute value, denoted $\left|\cdot\right|_{K_{\mathfrak{z}}}$\nomenclature{$\left|\cdot\right|_{K_{\mathfrak{z}}}$}{ }, so that $\left(K_{\mathfrak{z}},\left|\cdot\right|_{\mathfrak{z}}\right)$ is then a metrically complete valued field. \end{defn} \vphantom{} The two most important objects associated with a given frame are its image and the space of compatible functions: \begin{defn}[\textbf{Image and Compatible Functions}] \label{def:Frame terminology}Let $\mathcal{F}$ be a $p$-adic frame. \vphantom{} I. The \textbf{image }of $\mathcal{F}$, denoted $I\left(\mathcal{F}\right)$, is the \emph{set }defined by: \begin{equation} I\left(\mathcal{F}\right)\overset{\textrm{def}}{=}\bigcup_{\mathfrak{z}\in U_{\mathcal{F}}}K_{\mathfrak{z}}\label{eq:The image of a frame} \end{equation} \nomenclature{$I\left(\mathcal{F}\right)$}{The image of $\mathcal{F}$}\index{frame!image} \vphantom{} II. A function $\chi:U_{\mathcal{F}}\rightarrow I\left(\mathcal{F}\right)$ is said to be \textbf{$\mathcal{F}$-compatible} / \textbf{compatible }(\textbf{with $\mathcal{F}$}) whenever \index{mathcal{F}-@$\mathcal{F}$-!compatible}\index{frame!compatible functions} $\chi\left(\mathfrak{z}\right)\in K_{\mathfrak{z}}$ for all $\mathfrak{z}\in U_{\mathcal{F}}$. I write $C\left(\mathcal{F}\right)$ \nomenclature{$C\left(\mathcal{F}\right)$}{set of $\mathcal{F}$-compatible functions} to denote the set of all $\mathcal{F}$-compatible functions. \end{defn} \begin{rem} Note that $C\left(\mathcal{F}\right)$ is a linear space over $\overline{\mathbb{Q}}$ with respect to point-wise addition of functions and scalar multiplication. \end{rem} \vphantom{} Next, we have some terminology which is of use when working with a frame. \begin{defn}[\textbf{Frame Terminology}] Let $\mathcal{F}$ be a $p$-adic frame. \vphantom{} I. I call $\mathbb{Z}_{p}\backslash U_{\mathcal{F}}$ the \textbf{set of singularities }of the frame. I say that $\mathcal{F}$ is \textbf{non-singular }whenever $U_{\mathcal{F}}=\mathbb{Z}_{p}$.\index{frame!non-singular} \vphantom{} II. I write $\textrm{dis}$ to denote the discrete topology. I write $\infty$ to denote the topology of $\mathbb{C}$ (induced by $\left|\cdot\right|$), and I write $p$ to denote the $p$-adic topology (that of $\mathbb{C}_{p}$, induced by $\left|\cdot\right|_{p}$). I write $\textrm{non}$ to denote an arbitrary non-archimedean topology (induced by some non-archimedean absolute value on $\overline{\mathbb{Q}}$). \vphantom{} III. Given a topology $\tau$ (so, $\tau$ could be $\textrm{dis}$, $\infty$, $\textrm{non}$, or $p$), I write\footnote{So, we can write $U_{\textrm{dis}}\left(\mathcal{F}\right)$, $U_{\infty}\left(\mathcal{F}\right)$, $U_{\textrm{non}}\left(\mathcal{F}\right)$, or $U_{p}\left(\mathcal{F}\right)$.} $U_{\tau}\left(\mathcal{F}\right)$ to denote the set of $\mathfrak{z}\in U_{\mathcal{F}}$ so that $K_{\mathfrak{z}}$ has been equipped with $\tau$. I call the $U_{\tau}\left(\mathcal{F}\right)$\textbf{ $\tau$-convergence domain }or \textbf{domain of $\tau$ convergence }of $\mathcal{F}$. In this way, we can speak of the \textbf{discrete convergence domain}, the \textbf{archimedean convergence domain}, the \textbf{non-archimedean convergence domain}, and the \textbf{$p$-adic convergence domain} of a given frame $\mathcal{F}$. \vphantom{} IV. I say $\mathcal{F}$ is \textbf{simple} \index{frame!simple}if either $U_{\textrm{non}}\left(\mathcal{F}\right)$ is empty or there exists a single metrically complete non-archimedean field extension $K$ of $\overline{\mathbb{Q}}$ so that $K=K_{\mathfrak{z}}$ for all $\mathfrak{z}\in U_{\textrm{non}}\left(\mathcal{F}\right)$. (That is to say, for a simple frame, we use at most one non-archimedean topology.) \vphantom{} V. I say $\mathcal{F}$ is \textbf{proper }\index{frame!proper}proper whenever $K_{\mathfrak{z}}$ is not a $p$-adic field for any $\mathfrak{z}\in U_{\textrm{non}}\left(\mathcal{F}\right)$. \end{defn} \vphantom{} UNLESS STATED OTHERWISE, ALL FRAMES ARE ASSUMED TO BE PROPER. \vphantom{} Only one frame will ever be used in this dissertation. It is described below. \begin{defn} The \textbf{standard ($\left(p,q\right)$-adic) frame}\index{frame!standard left(p,qright)-adic@standard $\left(p,q\right)$-adic}, denoted $\mathcal{F}_{p,q}$, is the frame for which the topology of $\mathbb{C}$ is associated to $\mathbb{N}_{0}$ and the topology of $\mathbb{C}_{q}$ is associated to $\mathbb{Z}_{p}^{\prime}$. \end{defn} \vphantom{} With frames, we can avoid circumlocution when discussing topologies of convergence for $\left(p,q\right)$-adic functions and sequences thereof. \begin{defn}[\textbf{$\mathcal{F}$-convergence}] Given a frame a $\mathcal{F}$ and a function $\chi\in C\left(\mathcal{F}\right)$, we say a sequence $\left\{ \chi_{n}\right\} _{n\geq1}$ in $C\left(\mathcal{F}\right)$ \textbf{converges to $\chi$ over $\mathcal{F}$ }(or \textbf{is $\mathcal{F}$-convergent to $\chi$}) whenever, for each $\mathfrak{z}\in U_{\mathcal{F}}$, we have: \begin{equation} \lim_{n\rightarrow\infty}\chi_{n}\left(\mathfrak{z}\right)\overset{K_{\mathfrak{z}}}{=}\chi\left(\mathfrak{z}\right) \end{equation} Note that if $K_{\mathfrak{z}}$ has the discrete topology, the limit implies that $\chi_{n}\left(\mathfrak{z}\right)=\chi\left(\mathfrak{z}\right)$ for all sufficiently large $n$.\index{mathcal{F}-@$\mathcal{F}$-!convergence}\index{frame!convergence} The convergence is point-wise with respect to $\mathfrak{z}$. We then call $\chi$ the \index{mathcal{F}-@$\mathcal{F}$-!limit}\textbf{$\mathcal{F}$-limit of the $\chi_{n}$s}. \index{frame!limit}More generally, we say $\left\{ \chi_{n}\right\} _{n\geq1}$ is \textbf{$\mathcal{F}$-convergent / converges over $\mathcal{F}$} whenever there is a function $\chi\in C\left(\mathcal{F}\right)$ so that the $\chi_{n}$s are $\mathcal{F}$-convergent to $\chi$. In symbols, we denote $\mathcal{F}$-convergence by: \begin{equation} \lim_{n\rightarrow\infty}\chi_{n}\left(\mathfrak{z}\right)\overset{\mathcal{F}}{=}\chi\left(\mathfrak{z}\right),\textrm{ }\forall\mathfrak{z}\in U_{\mathcal{F}}\label{eq:Definition-in-symbols of F-convergence} \end{equation} or simply: \begin{equation} \lim_{n\rightarrow\infty}\chi_{n}\overset{\mathcal{F}}{=}\chi\label{eq:Simplified version of expressing f as the F-limit of f_ns} \end{equation} \end{defn} \begin{rem} With respect to the standard $\left(p,q\right)$-adic frame, $\mathcal{F}_{p,q}$-convergence means $\chi_{n}\left(\mathfrak{z}\right)$ converges to $\chi\left(\mathfrak{z}\right)$ in the topology of $\mathbb{C}_{q}$ for all $\mathfrak{z}\in\mathbb{Z}_{p}^{\prime}$ and converges to $\chi\left(\mathfrak{z}\right)$ in the topology of $\mathbb{C}$ for all $\mathfrak{z}\in\mathbb{N}_{0}$. \end{rem} \begin{rem} In an abuse of notation, we will sometimes write $\mathcal{F}$ convergence as: \begin{equation} \lim_{n\rightarrow\infty}\left|\chi_{n}\left(\mathfrak{z}\right)-\chi\left(\mathfrak{z}\right)\right|_{K_{\mathfrak{z}}},\textrm{ }\forall\mathfrak{z}\in U_{\mathcal{F}} \end{equation} The abuse here is that the absolute value $\left|\cdot\right|_{K_{\mathfrak{z}}}$ will not exist if $\mathfrak{z}\in U_{\textrm{dis}}\left(\mathcal{F}\right)$. As such, for any $\mathfrak{z}\in U_{\textrm{dis}}\left(\mathcal{F}\right)$, we define: \begin{equation} \lim_{n\rightarrow\infty}\left|\chi_{n}\left(\mathfrak{z}\right)-\chi\left(\mathfrak{z}\right)\right|_{K_{\mathfrak{z}}}=0 \end{equation} to mean that, for all sufficiently large $n$, the equality $\chi_{n}\left(\mathfrak{z}\right)=\chi_{n+1}\left(\mathfrak{z}\right)$ holds in $K_{\mathfrak{z}}$. \end{rem} \begin{prop} Let $\mathcal{F}$ be a $p$-adic frame, and let $\hat{\mu}:\hat{\mathbb{Z}}_{p}\rightarrow\overline{\mathbb{Q}}$. Then, $\tilde{\mu}_{N}\left(\mathfrak{z}\right)$\textemdash the $N$th partial sum of the Fourier series generated by $\hat{\mu}$\textemdash is an element of $C\left(\mathcal{F}\right)$. \end{prop} Proof: Because $\hat{\mu}$ takes values in $\overline{\mathbb{Q}}$, $\tilde{\mu}_{N}$ is a function from $\mathbb{Z}_{p}$ to $\overline{\mathbb{Q}}$. As such, restricting $\tilde{\mu}_{N}$ to $U_{\mathcal{F}}$ then gives us a function $U_{\mathcal{F}}\rightarrow I\left(\mathcal{F}\right)$, because $\overline{\mathbb{Q}}\subseteq I\left(\mathcal{F}\right)$. Q.E.D. \vphantom{} Now that we have the language of frames at our disposal, we can begin to define classes of $\left(p,q\right)$-adic measures which will be of interest to us. \begin{defn} For any prime $q\neq p$, we say $d\mu\in C\left(\mathbb{Z}_{p},\mathbb{C}_{q}\right)^{\prime}$ is\index{measure!algebraic} \textbf{algebraic},\textbf{ }if its Fourier-Stieltjes transform $\hat{\mu}$ takes values in $\overline{\mathbb{Q}}$. \end{defn} \begin{defn}[\textbf{$\mathcal{F}$-measures}] Given a $p$-adic frame $\mathcal{F}$, we write \nomenclature{$M\left(\mathcal{F}\right)$}{set of $\mathcal{F}$-measures}$M\left(\mathcal{F}\right)$ to denote the set of all functions $\hat{\mu}:\hat{\mathbb{Z}}_{p}\rightarrow\overline{\mathbb{Q}}$ so that $\hat{\mu}\in B\left(\hat{\mathbb{Z}}_{p},K_{\mathfrak{z}}\right)$ for all $\mathfrak{z}\in U_{\textrm{non}}\left(\mathcal{F}\right)$; that is: \begin{equation} \left\Vert \hat{\mu}\right\Vert _{p,K_{\mathfrak{z}}}<\infty,\textrm{ }\forall\mathfrak{z}\in U_{\textrm{non}}\left(\mathcal{F}\right)\label{eq:Definition of an F-measure} \end{equation} where, recall $\left\Vert \hat{\mu}\right\Vert _{p,K_{\mathfrak{z}}}$ denotes $\sup_{t\in\hat{\mathbb{Z}}_{p}}\left|\hat{\mu}\left(t\right)\right|_{K_{\mathfrak{z}}}$. Next, observe that for every $\mathfrak{z}\in U_{\textrm{non}}\left(\mathcal{F}\right)$, the map: \begin{equation} f\in C\left(\mathbb{Z}_{p},K_{\mathfrak{z}}\right)\mapsto\sum_{t\in\hat{\mathbb{Z}}_{p}}\hat{f}\left(t\right)\hat{\mu}\left(-t\right)\in K_{\mathfrak{z}}\label{eq:Definition of the action of an F-measure} \end{equation} then defines an element of $C\left(\mathbb{Z}_{p},K_{\mathfrak{z}}\right)^{\prime}$. As such, we will identify $M\left(\mathcal{F}\right)$ with elements of: \begin{equation} \bigcap_{\mathfrak{z}\in U_{\textrm{non}}\left(\mathcal{F}\right)}C\left(\mathbb{Z}_{p},K_{\mathfrak{z}}\right)^{\prime} \end{equation} and refer to elements of $M\left(\mathcal{F}\right)$ as \textbf{$\mathcal{F}$-measures}\index{mathcal{F}-@$\mathcal{F}$-!measure}, writing them as $d\mu$. Thus, the statement ``$d\mu\in M\left(\mathcal{F}\right)$'' means that $d\mu$ is an algebraic measure on $C\left(\mathbb{Z}_{p},K_{\mathfrak{z}}\right)$ for all $\mathfrak{z}\in U_{\textrm{non}}\left(\mathcal{F}\right)$, which necessarily forces $\hat{\mu}$ to be bounded on $\hat{\mathbb{Z}}_{p}$ in $K_{\mathfrak{z}}$-absolute-value for all $\mathfrak{z}\in U_{\textrm{non}}\left(\mathcal{F}\right)$. \end{defn} \begin{rem} If $\mathcal{F}$ is simple and there is a prime $q\neq p$ so that $K_{\mathfrak{z}}=\mathbb{C}_{q}$ for all $\mathfrak{z}\in U_{\textrm{non}}\left(\mathcal{F}\right)$, observe that $M\left(\mathcal{F}\right)$ is then precisely the set of all algebraic $\left(p,q\right)$-adic measures. This is also the case for the standard $\left(p,q\right)$-adic frame. \end{rem} \begin{rem} $M\left(\mathcal{F}\right)$ is a linear space over $\overline{\mathbb{Q}}$. \end{rem} \vphantom{} Next, we introduce definitions to link frames with the limits of Fourier series, by way of the convolution of a measure with the $p$-adic Dirichlet Kernels. \begin{defn}[\textbf{$\mathcal{F}$-rising measure}] Given a $p$-adic frame $\mathcal{F}$, we say $d\mu\in M\left(\mathcal{F}\right)$ \textbf{rises over $\mathcal{F}$} / is \textbf{$\mathcal{F}$-rising} whenever\index{measure!mathcal{F}-rising@$\mathcal{F}$-rising} \index{frame!rising measure}the sequence $\left\{ \tilde{\mu}_{N}\right\} _{N\geq0}=\left\{ D_{p:N}*d\mu\right\} _{N\geq0}$ is $\mathcal{F}$-convergent: \begin{equation} \lim_{N\rightarrow\infty}\tilde{\mu}_{N}\left(\mathfrak{z}\right)\textrm{ exists in }K_{\mathfrak{z}},\textrm{ }\forall\mathfrak{z}\in U_{\mathcal{F}}\label{eq:Definition of an F-rising measure} \end{equation} In the case of the standard $\left(p,q\right)$-adic frame, this means: \vphantom{} I. For every $\mathfrak{z}\in\mathbb{Z}_{p}^{\prime}$, $\lim_{N\rightarrow\infty}\tilde{\mu}_{N}\left(\mathfrak{z}\right)$ converges in $\mathbb{C}_{q}$. \vphantom{} II. For every $\mathfrak{z}\in\mathbb{N}_{0}$, $\lim_{N\rightarrow\infty}\tilde{\mu}_{N}\left(\mathfrak{z}\right)$ converges in $\mathbb{C}$. \vphantom{} In all cases, the convergence is point-wise. We then write $M_{\textrm{rise}}\left(\mathcal{F}\right)$\nomenclature{$M_{\textrm{rise}}\left(\mathcal{F}\right)$}{the set of $\mathcal{F}$-rising measures} to denote the set of all $\mathcal{F}$-rising measures. Note that $M_{\textrm{rise}}\left(\mathcal{F}\right)$ forms a linear space over $\overline{\mathbb{Q}}$. Given a $p$-adic frame $\mathcal{F}$ and an $\mathcal{F}$-rising measure $d\mu\in M_{\textrm{rise}}\left(\mathcal{F}\right)$, the \textbf{$\mathcal{F}$-derivative }of \index{measure!mathcal{F}-derivative@$\mathcal{F}$-derivative} $d\mu$ (or simply \textbf{derivative }of $d\mu$), denoted $\tilde{\mu}$, is the function on $U_{\mathcal{F}}$ defined by the $\mathcal{F}$-limit of the $\tilde{\mu}_{N}$s: \begin{equation} \tilde{\mu}\left(\mathfrak{z}\right)\overset{\mathcal{F}}{=}\lim_{N\rightarrow\infty}\tilde{\mu}_{N}\left(\mathfrak{z}\right)\label{eq:Definition of mu-twiddle / the derivative of mu} \end{equation} That is, for each $\mathfrak{z}\in U_{\mathcal{F}}$, $\tilde{\mu}\left(\mathfrak{z}\right)$ is defined by the limit of the $\tilde{\mu}_{N}\left(\mathfrak{z}\right)$s in $K_{\mathfrak{z}}$. When $\mathcal{F}$ is the standard $\left(p,q\right)$-adic frame, we have that: \begin{equation} \tilde{\mu}\left(\mathfrak{z}\right)=\begin{cases} \lim_{N\rightarrow\infty}\tilde{\mu}_{N}\left(\mathfrak{z}\right)\textrm{ in }\mathbb{C}_{q} & \textrm{if }\mathfrak{z}\in\mathbb{Z}_{p}^{\prime}\\ \lim_{N\rightarrow\infty}\tilde{\mu}_{N}\left(\mathfrak{z}\right)\textrm{ in }\mathbb{C} & \textrm{if }\mathfrak{z}\in\mathbb{N}_{0} \end{cases}\label{eq:F-limit in the standard frame} \end{equation} We then write $\left(\tilde{\mu}\right)_{N}\left(\mathfrak{z}\right)$ to denote the $N$th truncation of the derivative of $d\mu$: \begin{equation} \left(\tilde{\mu}\right)_{N}\left(\mathfrak{z}\right)\overset{\textrm{def}}{=}\sum_{n=0}^{p^{N}-1}\tilde{\mu}\left(n\right)\left[\mathfrak{z}\overset{p^{N}}{\equiv}n\right]\label{eq:Definition of the Nth truncation of the derivative of a rising measure} \end{equation} More generally, we have: \begin{equation} \left(\tilde{\mu}_{M}\right)_{N}\left(\mathfrak{z}\right)\overset{\textrm{def}}{=}\sum_{n=0}^{p^{N}-1}\tilde{\mu}_{M}\left(n\right)\left[\mathfrak{z}\overset{p^{N}}{\equiv}n\right]\label{eq:Nth truncation of the Mth partial sum of mu's derivative} \end{equation} is the $N$th truncation of $\tilde{\mu}_{M}$. \end{defn} \begin{defn} We say $d\mu\in\mathcal{M}_{\textrm{rise}}\left(\mathcal{F}_{p,q}\right)$ is \textbf{rising-continuous }if\index{measure!rising-continuous} it\index{rising-continuous!measure} whenever its $\mathcal{F}_{p,q}$-derivative $\tilde{\mu}\left(\mathfrak{z}\right)$ is a rising-continuous function. \end{defn} \vphantom{} Next up: \emph{degenerate }measures. \begin{defn}[\textbf{$\mathcal{F}$-degenerate measure}] \label{def:degenerate measure}Given a $p$-adic frame $\mathcal{F}$, an $\mathcal{F}$-rising measure $d\mu\in M_{\textrm{rise}}\left(\mathcal{F}\right)$ is said to be\textbf{ degenerate }whenever\index{measure!degenerate} its\index{mathcal{F}-@$\mathcal{F}$-!degenerate measure} $\mathcal{F}$-derivative\index{frame!degenerate measure} is identically zero. We drop the modifier $\mathcal{F}$ when there is no confusion. We write $M_{\textrm{dgen}}\left(\mathcal{F}\right)$\nomenclature{$M_{\textrm{dgen}}\left(\mathcal{F}\right)$}{the set of $\mathcal{F}$-degenerate measures} to denote the set of all $\mathcal{F}$-degenerate measures. Note that $M_{\textrm{dgen}}\left(\mathcal{F}\right)$ is a linear subspace of $M_{\textrm{rise}}\left(\mathcal{F}\right)$. \end{defn} \begin{defn} Given a $p$-adic frame $\mathcal{F}$, an $\mathcal{F}$-rising measure $d\mu\in M_{\textrm{rise}}\left(\mathcal{F}\right)$ is\index{measure!mathcal{F}-proper@$\mathcal{F}$-proper} said\index{mathcal{F}-@$\mathcal{F}$-!proper measure} to be \textbf{proper / $\mathcal{F}$-proper }whenever: \begin{equation} \lim_{N\rightarrow\infty}\left(\tilde{\mu}_{N}-\left(\tilde{\mu}\right)_{N}\right)\overset{\mathcal{F}}{=}0\label{eq:Definition of a proper measure} \end{equation} If this property fails to hold, we call $d\mu$ \textbf{improper / $\mathcal{F}$-improper}.\textbf{ }We then write $M_{\textrm{prop}}\left(\mathcal{F}\right)$ to denote the set of all $\mathcal{F}$-proper measures. Note that $M_{\textrm{prop}}\left(\mathcal{F}\right)$ is a linear subspace of $M_{\textrm{rise}}\left(\mathcal{F}\right)$. \end{defn} \begin{question} \label{que:3.2}Is the notion of a non-proper measure a vacuous concept, or does there exist a frame $\mathcal{F}$ and an $\mathcal{F}$-rising measure $d\mu$ which is improper? \end{question} \vphantom{} Before we move on to the next subsection, we record one last result: \begin{prop} Let $d\mu$ be a rising-continuous $\left(p,q\right)$-adic measure. Then, the derivative $\tilde{\mu}$ is a rising-continuous function if and only if: \vphantom{} I. $d\mu$ is proper. \vphantom{} II. $S_{p}\left\{ \tilde{\mu}\right\} \left(\mathfrak{z}\right)$ converges in $\mathbb{C}_{q}$ for every $\mathfrak{z}\in\mathbb{Z}_{p}$. \end{prop} Proof: We begin by defining a function $\mu^{\prime}$ by: \begin{equation} \mu^{\prime}\left(\mathfrak{z}\right)\overset{\textrm{def}}{=}S_{p}\left\{ \tilde{\mu}\right\} \left(\mathfrak{z}\right)\overset{\textrm{def}}{=}\sum_{n=0}^{\infty}c_{n}\left(\tilde{\mu}\right)\left[\mathfrak{z}\overset{p^{\lambda_{p}\left(n\right)}}{\equiv}n\right]\label{eq:Chi prime} \end{equation} That is to say, $\mu^{\prime}$ is obtained by computing the values $\tilde{\mu}\left(m\right)$ via the limits of $\tilde{\mu}_{N}\left(m\right)$, and then using those values to construct a van der Put series. On the other hand, $\tilde{\mu}\left(\mathfrak{z}\right)$ is constructed by computing the $\mathcal{F}_{p,q}$-limits of $\tilde{\mu}_{N}\left(\mathfrak{z}\right)$ for each $\mathfrak{z}\in\mathbb{Z}_{p}$. To complete the proof, we just need to show that $\tilde{\mu}$ and $\mu^{\prime}$ are the same function. In fact, we need only show that they are equal to one another over $\mathbb{Z}_{p}^{\prime}$; that they are equal over $\mathbb{N}_{0}$ is because $\mu^{\prime}$ is the function represented by the van der Put series of $\tilde{\mu}$, and a van der Put series of a function \emph{always} converges on $\mathbb{N}_{0}$ to said function. To do this, we write: \begin{equation} \tilde{\mu}\left(\mathfrak{z}\right)-\mu^{\prime}\left(\mathfrak{z}\right)=\tilde{\mu}\left(\mathfrak{z}\right)-\tilde{\mu}_{N}\left(\mathfrak{z}\right)+\tilde{\mu}_{N}\left(\mathfrak{z}\right)-\left(\tilde{\mu}\right)_{N}\left(\mathfrak{z}\right)+\left(\tilde{\mu}\right)_{N}\left(\mathfrak{z}\right)-\mu^{\prime}\left(\mathfrak{z}\right) \end{equation} Since: \begin{equation} S_{p:N}\left\{ \tilde{\mu}\right\} \left(\mathfrak{z}\right)=\tilde{\mu}\left(\left[\mathfrak{z}\right]_{p^{N}}\right)\overset{\textrm{def}}{=}\left(\tilde{\mu}\right)_{N}\left(\mathfrak{z}\right) \end{equation} we then have: \begin{equation} \tilde{\mu}\left(\mathfrak{z}\right)-\mu^{\prime}\left(\mathfrak{z}\right)=\underbrace{\tilde{\mu}\left(\mathfrak{z}\right)-\tilde{\mu}_{N}\left(\mathfrak{z}\right)}_{\textrm{A}}+\underbrace{\tilde{\mu}_{N}\left(\mathfrak{z}\right)-\left(\tilde{\mu}\right)_{N}\left(\mathfrak{z}\right)}_{\textrm{B}}+\underbrace{S_{p:N}\left\{ \tilde{\mu}\right\} \left(\mathfrak{z}\right)-\mu^{\prime}\left(\mathfrak{z}\right)}_{\textrm{C}}\label{eq:A,B,C,} \end{equation} i. Suppose $d\mu$ is proper and that $S_{p}\left\{ \tilde{\mu}\right\} \left(\mathfrak{z}\right)\overset{\mathbb{C}_{q}}{=}\lim_{N\rightarrow\infty}S_{p:N}\left\{ \tilde{\mu}\right\} \left(\mathfrak{z}\right)$ converges for every $\mathfrak{z}\in\mathbb{Z}_{p}$. Since $d\mu$ is rising-continuous, $\lim_{N\rightarrow\infty}\left|\tilde{\mu}\left(\mathfrak{z}\right)-\tilde{\mu}_{N}\left(\mathfrak{z}\right)\right|_{q}=0$ for all $\mathfrak{z}\in\mathbb{Z}_{p}^{\prime}$, which proves that (A) tends to zero $q$-adically. Additionally, because $d\mu$ is proper, the fact that $d\mu$ is proper then guarantees that $\lim_{N\rightarrow\infty}\left|\tilde{\mu}_{N}\left(\mathfrak{z}\right)-\left(\tilde{\mu}\right)_{N}\left(\mathfrak{z}\right)\right|_{q}=0$ for all $\mathfrak{z}\in\mathbb{Z}_{p}^{\prime}$, seeing as $\mathbb{Z}_{p}^{\prime}$ is the domain of non-archimedean convergence for the standard $\left(p,q\right)$-adic frame. This shows that (B) tends to zero $q$-adically. Finally, (C) tends to zero $q$-adically as $N\rightarrow\infty$ for all $\mathfrak{z}\in\mathbb{Z}_{p}^{\prime}$ because firstly, by definition, $\mu^{\prime}$ is the limit of $S_{p:N}\left\{ \tilde{\mu}\right\} \left(\mathfrak{z}\right)$ as $N\rightarrow\infty$, and, secondly, because $S_{p:N}\left\{ \tilde{\mu}\right\} \left(\mathfrak{z}\right)$ converges to a limit $q$-adically for every $\mathfrak{z}\in\mathbb{Z}_{p}$. This shows that the right hand side of (\ref{eq:A,B,C,}) is identically zero for all $\mathfrak{z}\in\mathbb{Z}_{p}^{\prime}$. So, $\tilde{\mu}=\mu^{\prime}$ on $\mathbb{Z}_{p}^{\prime}$, as desired. \vphantom{} ii. Suppose $\tilde{\mu}$ is rising-continuous. Then, $\mu^{\prime}$\textemdash the van der Put series of $\tilde{\mu}$\textemdash is necessarily equal to $\tilde{\mu}$, and converges everywhere, thereby forcing (II) to hold true. Next, by the van der Put identity (\textbf{Proposition \ref{prop:vdP identity}}), we can write: \begin{equation} \tilde{\mu}\left(\mathfrak{z}\right)=\mu^{\prime}\left(\mathfrak{z}\right)\overset{\mathbb{C}_{q}}{=}\lim_{N\rightarrow\infty}\left(\tilde{\mu}\right)_{N}\left(\mathfrak{z}\right),\textrm{ }\forall\mathfrak{z}\in\mathbb{Z}_{p} \end{equation} Moreover, $\left(\tilde{\mu}\right)_{N}\left(\mathfrak{z}\right)=\left(\tilde{\mu}\right)_{N+1}\left(\mathfrak{z}\right)$ for all sufficiently large $N$ whenever $\mathfrak{z}\in\mathbb{N}_{0}$, so $\left(\tilde{\mu}\right)_{N}\left(\mathfrak{z}\right)$ converges to $\tilde{\mu}\left(\mathfrak{z}\right)$ in $\mathbb{C}$ for $\mathfrak{z}\in\mathbb{N}_{0}$ and in $\mathbb{C}_{q}$ for $\mathfrak{z}\in\mathbb{Z}_{p}^{\prime}$. This shows that $\left(\tilde{\mu}\right)_{N}$ is $\mathcal{F}_{p,q}$-convergent to $\tilde{\mu}$. Since, $d\mu$ is rising-continuous, by definition, the $\tilde{\mu}_{N}$s are $\mathcal{F}_{p,q}$-convergent to $\tilde{\mu}$. Since $\left(\tilde{\mu}\right)_{N}$ and $\tilde{\mu}_{N}$ then $\mathcal{F}_{p,q}$-converge to the same function, they then necessarily $\mathcal{F}_{p,q}$-converge to one another. This proves $d\mu$ is proper, and hence, proves (I). Q.E.D. \subsection{\label{subsec:3.3.4 Toward-a-Taxonomy}Toward a Taxonomy of $\left(p,q\right)$-adic Measures} Although frames lubricate our discussion of the convergence behavior of Fourier series generated by the Fourier-Stieltjes transforms of $\left(p,q\right)$-adic measures, we still do not have many practical examples of these measures. In the present subsection, I have gathered the few results about $\left(p,q\right)$-adic measures which I \emph{have }been able to prove, in the hopes of being able to give more substantial descriptions of them. Somewhat vexingly, the methods in this subsection depend nigh-entirely on the structural properties of whichever type of measure we happen to be considering. Without assuming such properties hold, it becomes difficult to say anything meaningful about the measures and the Fourier series they generate\textemdash this is one of the main challenges we have to deal with when doing theory with frames. In order to keep this subsection text from drowning in adjectives, we will use the following abbreviations: \begin{defn}[\textbf{Measure Type Codes}] \ \vphantom{} I.\textbf{ }(\textbf{ED1}) We say \index{measure!elementary type D1}$d\mu\in M_{\textrm{alg}}\left(\mathbb{Z}_{p},\mathbb{C}_{q}\right)$ is an \textbf{elementary D1 measure }(ED1) whenever it can be written as $d\mu=d\nu*d\eta$ for $d\nu,d\eta\in M_{\textrm{alg}}\left(\mathbb{Z}_{p},\mathbb{C}_{q}\right)$, where $d\nu$ is radially symmetric and where $d\eta$ is $\left(p,q\right)$-adically regular magnitudinal measure. That is to say, $\hat{\nu}\left(t\right)$ is a $\overline{\mathbb{Q}}$-valued function which depends only on $\left|t\right|_{p}$, and $\hat{\eta}\left(t\right)$ is of the form: \begin{equation} \hat{\eta}\left(t\right)=\begin{cases} \kappa\left(0\right) & \textrm{if }t=0\\ \sum_{m=0}^{\left|t\right|_{p}-1}\kappa\left(m\right)e^{-2\pi imt} & \textrm{else} \end{cases},\textrm{ }\forall t\in\hat{\mathbb{Z}}_{p} \end{equation} for some function $\kappa:\mathbb{N}_{0}\rightarrow\overline{\mathbb{Q}}$ such that: \begin{equation} \lim_{n\rightarrow\infty}\left|\kappa\left(\left[\mathfrak{z}\right]_{p^{n}}\right)\right|_{q}=0,\textrm{ }\forall\mathfrak{z}\in\mathbb{Z}_{p}^{\prime} \end{equation} and, moreover, for which there are constants $a_{1},\ldots,a_{p-1}\in\overline{\mathbb{Q}}$ so that: \[ \kappa\left(\left[\mathfrak{z}\right]_{p^{n}}+jp^{n}\right)=a_{j}\kappa\left(\left[\mathfrak{z}\right]_{p^{n}}\right),\textrm{ }\forall\mathfrak{z}\in\mathbb{Z}_{p},\textrm{ }\forall n\geq1,\textrm{ }\forall j\in\left\{ 1,\ldots,p-1\right\} \] Note that this then implies $\kappa\left(0\right)=1$. \vphantom{} II. (\textbf{D1}) We say $d\mu\in M_{\textrm{alg}}\left(\mathbb{Z}_{p},\mathbb{C}_{q}\right)$ a \textbf{type D1 measure }whenever\index{measure!type D1} it is a $\overline{\mathbb{Q}}$-linear combination of finitely many ED1 measures. \vphantom{} III. (\textbf{ED2}) We say $d\mu\in M_{\textrm{alg}}\left(\mathbb{Z}_{p},\mathbb{C}_{q}\right)$ an \textbf{elementary D2 measure} whenever it is ED1, with\index{measure!elementary type D2}: \begin{equation} v_{n}=\left(1+A\right)v_{n+1},\textrm{ }\forall n\geq0\label{eq:ED2 measure} \end{equation} where, recall: \begin{equation} A=\sum_{j=1}^{p-1}a_{j}=\sum_{j=1}^{p-1}\kappa\left(j\right) \end{equation} is the sum of the constants from the structural equation of $\kappa$ and $v_{n}$ denotes $\hat{\nu}\left(p^{-n}\right)$. \vphantom{} IV. (\textbf{D2}) We say $d\mu\in M_{\textrm{alg}}\left(\mathbb{Z}_{p},\mathbb{C}_{q}\right)$ a \textbf{type D2 measure }whenever it is a $\overline{\mathbb{Q}}$-linear combination of finitely many ED2 measures. \index{measure!type D2} \end{defn} \vphantom{} Obviously, there are many other variants we could consider. For example, w measures of the form: \begin{equation} \hat{\mu}\left(t\right)=\begin{cases} \kappa\left(0\right) & \textrm{if }t=0\\ \sum_{n=0}^{-v_{p}\left(t\right)-1}\kappa\left(n\right)e^{-2\pi int} & \textrm{else} \end{cases},\textrm{ }\forall t\in\hat{\mathbb{Z}}_{p} \end{equation} for some $\left(p,q\right)$-adically regular $\kappa:\mathbb{N}_{0}\rightarrow\overline{\mathbb{Q}}$; also, spaces generated by convolutions of measures with this type, or any combination of convolutions of radially-symmetric and/or magnitudinal measures, ED1 measures for which: \begin{equation} \lim_{n\rightarrow\infty}\hat{\nu}\left(p^{-N}\right)p^{N}\overset{\mathbb{C}}{=}0 \end{equation} and so on and so forth. \begin{prop} \label{prop:ED1 measures are zero whenever kappa of 0 is 0}An\textbf{ }ED1\textbf{ }measure is the zero measure whenever $\kappa\left(0\right)=0$. \end{prop} Proof: If $\kappa\left(0\right)=0$, then \textbf{Lemma \ref{lem:structural equations uniquely determine p-adic structured functions}} applies, telling us that $\kappa$ is identically zero. This makes $\hat{\eta}$ identically zero, which makes $\hat{\mu}=\hat{\nu}\cdot\hat{\eta}$ identically zero. This proves that $d\mu$ is the zero measure. Q.E.D. \begin{lem} \label{lem:degenerate rad sym measures are zero}The only degenerate radially symmetric measure is the zero measure. \end{lem} Proof: Let $d\mu$ be a degenerate radially-symmetric measure. Using the Radial Fourier Resummation formula (\ref{eq:Radial Fourier Resummation Lemma}), the degeneracy of $d\mu$ allows us to write: \begin{equation} 0\overset{\mathbb{C}_{q}}{=}\lim_{N\rightarrow\infty}\tilde{\mu}_{N}\left(\mathfrak{z}\right)\overset{\overline{\mathbb{Q}}}{=}\hat{\mu}\left(0\right)-\left|\mathfrak{z}\right|_{p}^{-1}\hat{\mu}\left(\frac{\left|\mathfrak{z}\right|_{p}}{p}\right)+\left(1-\frac{1}{p}\right)\sum_{n=1}^{v_{p}\left(\mathfrak{z}\right)}\hat{\mu}\left(\frac{1}{p^{n}}\right)p^{n},\textrm{ }\forall\mathfrak{z}\in\mathbb{Z}_{p}\backslash\left\{ 0\right\} \label{eq:Degeneracy Lemma - Algebraic Vanishing} \end{equation} Hence, it must be that the algebraic number: \begin{equation} \hat{\mu}\left(0\right)-\left|\mathfrak{z}\right|_{p}^{-1}\hat{\mu}\left(\frac{\left|\mathfrak{z}\right|_{p}}{p}\right)+\left(1-\frac{1}{p}\right)\sum_{n=1}^{v_{p}\left(\mathfrak{z}\right)}\hat{\mu}\left(\frac{1}{p^{n}}\right)p^{n} \end{equation} is $0$ for all non-zero $p$-adic integers $\mathfrak{z}$. Letting $\left|\mathfrak{z}\right|_{p}=p^{k}$ for $k\geq1$, and letting $\hat{\mu}\left(p^{-n}\right)=c_{n}$, the above becomes: \begin{equation} 0\overset{\overline{\mathbb{Q}}}{=}c_{0}-c_{k+1}p^{k}+\left(1-\frac{1}{p}\right)\sum_{n=1}^{k}c_{n}p^{n},\textrm{ }\forall k\geq1 \end{equation} Next, we subtract the $\left(k-1\right)$th case from the $k$th case. \begin{equation} 0\overset{\overline{\mathbb{Q}}}{=}c_{k}p^{k-1}-c_{k+1}p^{k}+\left(1-\frac{1}{p}\right)c_{k}p^{k},\textrm{ }\forall k\geq2 \end{equation} and hence: \begin{equation} c_{k+1}\overset{\overline{\mathbb{Q}}}{=}c_{k},\textrm{ }\forall k\geq2 \end{equation} Meanwhile, when $k=1$, we get: \begin{equation} c_{2}\overset{\overline{\mathbb{Q}}}{=}\frac{1}{p}c_{0}+\left(1-\frac{1}{p}\right)c_{1} \end{equation} As for the $\mathfrak{z}=0$ case, using the \textbf{Radial Fourier Resummation Lemma }(\textbf{Lemma \ref{lem:Rad-Sym Fourier Resum Lemma}}), the degeneracy of $d\mu$ tell us that: \[ 0\overset{\mathbb{C}}{=}\lim_{N\rightarrow\infty}\tilde{\mu}_{N}\left(0\right)=c_{0}+\left(p-1\right)\lim_{N\rightarrow\infty}\sum_{k=1}^{N}c_{k}p^{k-1}\overset{\mathbb{C}}{=}c_{0}+\left(1-\frac{1}{p}\right)\sum_{k=1}^{\infty}c_{k}p^{k} \] This then forces $c_{n}p^{n}$ to tend to $0$ in $\mathbb{C}$ as $n\rightarrow\infty$. However, we just showed that $c_{k}=c_{k+1}$ for all $k\geq2$. So, the only way convergence occurs is if $c_{k}=0$ for all $k\geq2$. Consequently, $\hat{\mu}\left(t\right)=0$ for all $\left|t\right|_{p}\geq p^{2}$. Using the \textbf{Radial Fourier Resummation Lemma} once more, we obtain: \begin{equation} \tilde{\mu}\left(\mathfrak{z}\right)\overset{\overline{\mathbb{Q}}}{=}\begin{cases} \hat{\mu}\left(0\right)-\hat{\mu}\left(\frac{1}{p}\right) & \textrm{if }\left|\mathfrak{z}\right|_{p}=1\\ \hat{\mu}\left(0\right)+\left(p-1\right)\hat{\mu}\left(\frac{1}{p}\right) & \textrm{if }\left|\mathfrak{z}\right|_{p}\leq\frac{1}{p} \end{cases} \end{equation} Since $d\mu$ is degenerate, $\tilde{\mu}\left(\mathfrak{z}\right)$ is identically zero. This forces $\hat{\mu}\left(0\right)=\hat{\mu}\left(\frac{1}{p}\right)$. So, we can write: \begin{equation} 0=\hat{\mu}\left(0\right)+\left(p-1\right)\hat{\mu}\left(\frac{1}{p}\right)=p\hat{\mu}\left(0\right) \end{equation} which forces $\hat{\mu}\left(0\right)=\hat{\mu}\left(\frac{1}{p}\right)=\hat{\mu}\left(\frac{1}{p^{2}}\right)=\cdots=0$. This proves that $d\mu$ is the zero measure. Q.E.D. \begin{prop} Let $d\mu=d\nu*d\eta$ be an ED2 measure. Then, $d\mu$ is degenerate if and only if: \begin{equation} \lim_{N\rightarrow\infty}\hat{\nu}\left(p^{-N}\right)p^{N}\overset{\mathbb{C}}{=}0\label{eq:Degeneracy criterion for elementary D4 measures} \end{equation} \end{prop} Proof: Since $d\mu$ is elementary and non-zero, the fact that every ED2 measure is ED1 means that \textbf{Proposition \ref{prop:ED1 measures are zero whenever kappa of 0 is 0} }applies, guaranteeing that $\kappa\left(0\right)\neq0$. Now, supposing $d\mu$ satisfies (\ref{eq:Degeneracy criterion for elementary D4 measures}), the \textbf{Radially-Magnitudinal Fourier Resummation Lemma }(\textbf{Lemma \ref{lem:Radially-Mag Fourier Resummation Lemma}}) becomes: \begin{equation} \tilde{\mu}_{N}\left(\mathfrak{z}\right)=\sum_{\left|t\right|_{p}\leq p^{N}}\hat{\mu}\left(t\right)e^{2\pi i\left\{ t\mathfrak{z}\right\} _{p}}\overset{\overline{\mathbb{Q}}}{=}\hat{\nu}\left(p^{-N}\right)\kappa\left(\left[\mathfrak{z}\right]_{p^{N}}\right)p^{N} \end{equation} The $\left(p,q\right)$-adic regularity of $\kappa$ and along with the assumptions then guarantee that $\tilde{\mu}_{N}\left(\mathfrak{z}\right)$ tends to $0$ in $\mathbb{C}$ (resp. $\mathbb{C}_{q}$) for $\mathfrak{z}\in\mathbb{N}_{0}$ (resp. $\mathbb{Z}_{p}^{\prime}$) as $N\rightarrow\infty$, which shows that $d\mu$ is then degenerate. Conversely, suppose that the non-zero ED2 measure $d\mu$ is degenerate. Using \textbf{Lemma \ref{lem:Radially-Mag Fourier Resummation Lemma}} once again, we write: \begin{align} \sum_{\left|t\right|_{p}\leq p^{N}}\hat{\mu}\left(t\right)e^{2\pi i\left\{ t\mathfrak{z}\right\} _{p}} & \overset{\mathbb{\overline{\mathbb{Q}}}}{=}v_{N}\kappa\left(\left[\mathfrak{z}\right]_{p^{N}}\right)p^{N}+\sum_{n=0}^{N-1}\left(v_{n}-\left(1+A\right)v_{n+1}\right)\kappa\left(\left[\mathfrak{z}\right]_{p^{n}}\right)p^{n}\label{eq:Radial Magnitudinal identity for the Degeneracy Lemma} \end{align} where, recall, $v_{n}=\hat{\nu}\left(p^{-n}\right)$. Since $d\mu$ is ED2, $v_{n}=Av_{n+1}$ for all $n\geq0$, and hence: \begin{equation} \sum_{\left|t\right|_{p}\leq p^{N}}\hat{\mu}\left(t\right)e^{2\pi i\left\{ t\mathfrak{z}\right\} _{p}}\overset{\mathbb{\overline{\mathbb{Q}}}}{=}v_{N}\kappa\left(\left[\mathfrak{z}\right]_{p^{N}}\right)p^{N} \end{equation} Since $d\mu$ is ED2, it is also ED1, and as such, the assumption that $d\mu$ is non-zero guarantees that $\kappa\left(0\right)\neq0$. So, setting $\mathfrak{z}=0$, the degeneracy of $d\mu$ tells us that: \begin{equation} 0\overset{\mathbb{C}}{=}\lim_{N\rightarrow\infty}\sum_{\left|t\right|_{p}\leq p^{N}}\hat{\mu}\left(t\right)\overset{\mathbb{\overline{\mathbb{Q}}}}{=}\kappa\left(0\right)\lim_{N\rightarrow\infty}v_{N}p^{N} \end{equation} Since $\kappa\left(0\right)\neq0$, this forces $\lim_{N\rightarrow\infty}v_{N}p^{N}$ to converge to $0$ in $\mathbb{C}$, which is exactly what we wished to show (equation (\ref{eq:Degeneracy criterion for elementary D4 measures})). Q.E.D. \begin{prop} Let $d\mu=d\nu*d\eta$ be an ED1 measure. Then: \end{prop} \begin{equation} \tilde{\mu}_{N}\left(\mathfrak{z}\right)\overset{\overline{\mathbb{Q}}}{=}\hat{\nu}\left(0\right)\kappa\left(0\right)+\sum_{n=1}^{N}\hat{\nu}\left(p^{-n}\right)\left(\kappa\left(\left[\mathfrak{z}\right]_{p^{n}}\right)-\kappa\left(\left[\mathfrak{z}\right]_{p^{n-1}}\right)\right)p^{n}\label{eq:radial-magnitude Fourier resummation w/ SbP} \end{equation} Proof: Apply summation by parts to equation (\ref{eq:Radial-Magnitude p-adic distributed Fourier Resummation Identity, simplified}) from \textbf{Lemma \ref{lem:Radially-Mag Fourier Resummation Lemma}}. Q.E.D. \begin{prop} \label{prop:non-zero degenerate ED1}Let $d\mu=d\nu*d\eta$ be a non-zero degenerate ED1 measure. Then $\left|\kappa\left(j\right)\right|_{q}<1$ for all $j\in\left\{ 1,\ldots,p-1\right\} $. In particular, there is a $j\in\left\{ 1,\ldots,p-1\right\} $ so that $\left|\kappa\left(j\right)\right|_{q}>0$. \end{prop} Proof: Suppose that the non-zero ED1\textbf{ }measure $d\mu$ is degenerate. Then, by equation (\ref{eq:Radial Magnitudinal identity for the Degeneracy Lemma}): \begin{align} \sum_{\left|t\right|_{p}\leq p^{N}}\hat{\mu}\left(t\right)e^{2\pi i\left\{ t\mathfrak{z}\right\} _{p}} & \overset{\mathbb{\overline{\mathbb{Q}}}}{=}v_{N}\kappa\left(\left[\mathfrak{z}\right]_{p^{N}}\right)p^{N}+\sum_{n=0}^{N-1}\left(v_{n}-\left(1+A\right)v_{n+1}\right)\kappa\left(\left[\mathfrak{z}\right]_{p^{n}}\right)p^{n}\label{eq:Radial Magnitudinal identity for the Degeneracy Lemma-1} \end{align} where, again, $v_{n}=\hat{\nu}\left(p^{-n}\right)$. Since $p$ and $q$ are distinct primes, and since $d\nu$ is a $\left(p,q\right)$-adic measure, we have that: \begin{equation} \sup_{N\geq1}\left|v_{N}p^{N}\right|_{q}<\infty \end{equation} Also, since $\kappa$ is $\left(p,q\right)$-adically regular, letting $\mathfrak{z}\in\mathbb{Z}_{p}^{\prime}$ be arbitrary, taking the limit of (\ref{eq:Radial Magnitudinal identity for the Degeneracy Lemma-1}) in $\mathbb{C}_{q}$ as $N\rightarrow\infty$ gives: \begin{equation} \sum_{n=0}^{\infty}\left(v_{n}-\left(1+A\right)v_{n+1}\right)\kappa\left(\left[\mathfrak{z}\right]_{p^{n}}\right)p^{n}\overset{\mathbb{C}_{q}}{=}0,\textrm{ }\forall\mathfrak{z}\in\mathbb{Z}_{p}^{\prime}\label{eq:Degeneracy lemma, equation 1-1} \end{equation} which is equal to $0$, by the presumed degeneracy of $d\mu$. Now, letting $j\in\left\{ 1,\ldots,p-1\right\} $, observe that $\left[-j\right]_{p^{n}}$ has exactly $n$ $p$-adic digits, all of which are $p-j$. That is: \begin{equation} \left[-j\right]_{p^{n+1}}=\left[-j\right]_{p^{n}}+\left(p-j\right)p^{n},\textrm{ }\forall n\geq0 \end{equation} Because $\kappa$ is $\left(p,q\right)$-adically regular and non-zero, we can then write: \[ \kappa\left(\left[-j\right]_{p^{n+1}}\right)=\kappa\left(\left[-j\right]_{p^{n}}+\left(p-j\right)p^{n}\right)=\kappa\left(p-j\kappa\right)\left(\left[-j\right]_{p^{n}}\right),\textrm{ }\forall n\geq0 \] Thus, by induction: \begin{equation} \kappa\left(\left[-j\right]_{p^{n}}\right)=\left(\kappa\left(p-j\right)\right)^{n}\kappa\left(0\right),\textrm{ }\forall j\in\left\{ 1,\ldots,p-1\right\} ,\textrm{ }\forall n\geq0 \end{equation} Moreover, because $\left|\kappa\left(\left[\mathfrak{z}\right]_{p^{n}}\right)\right|_{q}\rightarrow0$ in $\mathbb{R}$ for all $\mathfrak{z}\in\mathbb{Z}_{p}^{\prime}$, it must be that: \begin{equation} 0\overset{\mathbb{R}}{=}\lim_{n\rightarrow\infty}\left|\kappa\left(\left[-j\right]_{p^{n}}\right)\right|_{q}=\lim_{n\rightarrow\infty}\left|\kappa\left(p-j\right)\right|_{q}^{n}\left|\kappa\left(0\right)\right|_{q} \end{equation} Since $\kappa\left(0\right)\neq0$, this forces $\left|\kappa\left(p-j\right)\right|_{q}<1$ for all $j\in\left\{ 1,\ldots p-1\right\} $. So, for $\mathfrak{z}=-j$, equation (\ref{eq:Degeneracy lemma, equation 1-1}) becomes: \begin{equation} \kappa\left(0\right)\sum_{n=0}^{\infty}\left(v_{n}-\left(1+A\right)v_{n+1}\right)\left(\kappa\left(p-j\right)\right)^{n}p^{n}\overset{\mathbb{C}_{q}}{=}0,\textrm{ }\forall j\in\left\{ 1,\ldots,p-1\right\} \end{equation} Since $\kappa\left(0\right)\neq0$, it must be that the series is zero. Re-indexing the $j$s then gives us: \begin{equation} \sum_{n=0}^{\infty}\left(v_{n}-\left(1+A\right)v_{n+1}\right)\left(\kappa\left(j\right)\right)^{n}p^{n}\overset{\mathbb{C}_{q}}{=}0,\textrm{ }\forall j\in\left\{ 1,\ldots,p-1\right\} \end{equation} \begin{claim} There exists a $j\in\left\{ 1,\ldots,p-1\right\} $ so that $\left|\kappa\left(j\right)\right|_{q}\in\left(0,1\right)$. Proof of claim: We already know that all of the $\kappa\left(j\right)$s have $q$-adic absolute value $<1$. So by way of contradiction, suppose $\kappa\left(j\right)=0$ for all $j\in\left\{ 1,\ldots,p-1\right\} $. This tells us that $\kappa\left(n\right)$ is $1$ when $n=0$ and is $0$ for all $n\geq1$. Consequently, $\hat{\eta}\left(t\right)=\kappa\left(0\right)$ for all $t\in\hat{\mathbb{Z}}_{p}$. With this, we see that the degenerate measure $d\mu$ must be radially-symmetric, and so, $\hat{\mu}\left(t\right)=\kappa\left(0\right)\hat{\nu}\left(t\right)=\hat{\nu}\left(t\right)$ for some radially-symmetric $\hat{\nu}$. Using \textbf{Lemma \ref{lem:degenerate rad sym measures are zero}}, $d\mu$ is then the zero measure, which contradicts the fact that $d\mu$ was given to be non-zero. This proves the claim. \end{claim} \vphantom{} This shows that $\left|\kappa\left(j\right)\right|_{q}<1$ for all $j\in\left\{ 1,\ldots,p-1\right\} $, and that there is one such $j$ for which $0<\left|\kappa\left(j\right)\right|_{q}<1$. Q.E.D. \begin{prop} Let $d\mu=d\nu*d\eta$ be a non-zero degenerate ED1 measure. Then $A\neq p-1$, where, recall, $A=\sum_{j=1}^{p-1}\kappa\left(j\right)$. \end{prop} Proof: Let $d\mu$ be as given. By way of contradiction, suppose $A=p-1$. Note that since $d\mu$ is non-zero, we have that $\kappa\left(0\right)=1$. Now, letting $\mathfrak{z}=0$, in (\ref{eq:radial-magnitude Fourier resummation w/ SbP}) we get: \begin{equation} \tilde{\mu}_{N}\left(\mathfrak{z}\right)\overset{\overline{\mathbb{Q}}}{=}v_{0}\kappa\left(0\right)+\sum_{n=1}^{N}v_{n}\left(\kappa\left(0\right)-\kappa\left(0\right)\right)p^{n}=v_{0}\kappa\left(0\right) \end{equation} Letting $N\rightarrow\infty$, we have: \begin{equation} 0=v_{0}\kappa\left(0\right) \end{equation} Since $\kappa\left(0\right)=1$, this then implies $v_{0}=0$. Next, by \textbf{Proposition \ref{prop:non-zero degenerate ED1}}, we can fix a $j\in\left\{ 1,\ldots,p-1\right\} $ so that $0<\left|\kappa\left(j\right)\right|_{q}<1$. Since $\kappa$ is $\left(p,q\right)$-adically regular, we have: \begin{equation} \kappa\left(\left[j+jp+\cdots+jp^{m-1}\right]_{p^{n}}\right)=\kappa\left(j\right)\kappa\left(\left[j+jp+\cdots+jp^{m-2}\right]_{p^{n}}\right) \end{equation} As such: \begin{equation} \kappa\left(\left[j\frac{p^{m}-1}{p-1}\right]_{p^{n}}\right)=\kappa\left(\left[j+jp+\cdots+jp^{m-1}\right]_{p^{n}}\right)=\left(\kappa\left(j\right)\right)^{\min\left\{ n,m\right\} }\label{eq:kappa of j times (p to the m -1 over p -1)} \end{equation} and so, letting $\mathfrak{z}=j\frac{p^{m}-1}{p-1}\in\mathbb{N}_{1}$ for $m\geq1$, (\ref{eq:radial-magnitude Fourier resummation w/ SbP}) becomes: \begin{align*} \tilde{\mu}_{N}\left(j\frac{p^{m}-1}{p-1}\right) & \overset{\overline{\mathbb{Q}}}{=}v_{0}+\sum_{n=1}^{N}v_{n}\left(\kappa\left(\left[j\frac{p^{m}-1}{p-1}\right]_{p^{n}}\right)-\kappa\left(\left[j\frac{p^{m}-1}{p-1}\right]_{p^{n-1}}\right)\right)p^{n}\\ & =v_{0}+\sum_{n=1}^{N}v_{n}\left(\kappa\left(j\right)^{\min\left\{ n,m\right\} }-\left(\kappa\left(j\right)\right)^{\min\left\{ n-1,m\right\} }\right)p^{n}\\ & =v_{0}+\sum_{n=1}^{m-1}v_{n}\left(\kappa\left(j\right)\right)^{n}p^{n}-\sum_{n=1}^{m}v_{n}\left(\kappa\left(j\right)\right)^{n-1}p^{n}\\ & +\left(\kappa\left(j\right)\right)^{m}\left(\sum_{n=m}^{N}v_{n}p^{n}-\sum_{n=m+1}^{N}v_{n}p^{n}\right)\\ & =v_{0}+\sum_{n=1}^{m}v_{n}\left(\left(\kappa\left(j\right)\right)^{n}-\left(\kappa\left(j\right)\right)^{n-1}\right)p^{n} \end{align*} Letting $N\rightarrow\infty$ gives: \begin{align} v_{0}+\sum_{n=1}^{m}v_{n}\left(\left(\kappa\left(j\right)\right)^{n}-\left(\kappa\left(j\right)\right)^{n-1}\right)p^{n} & \overset{\overline{\mathbb{Q}}}{=}0,\textrm{ }\forall m\geq1\label{eq:m cases} \end{align} Subtracting the $\left(m-1\right)$th case from the $m$th case for $m\geq2$ leaves us with: \begin{equation} v_{m}\left(\left(\kappa\left(j\right)\right)^{m}-\left(\kappa\left(j\right)\right)^{m-1}\right)p^{m}\overset{\overline{\mathbb{Q}}}{=}0,\textrm{ }\forall m\geq2 \end{equation} Multiplying through by the non-zero algebraic number $\left(\kappa\left(j\right)\right)^{1-m}$ yields: \begin{equation} v_{m}\left(\kappa\left(j\right)-1\right)p^{m}\overset{\overline{\mathbb{Q}}}{=}0,\textrm{ }\forall m\geq2 \end{equation} Here, $0<\left|\kappa\left(j\right)\right|_{q}<1$ guarantees that $\left(\kappa\left(j\right)-1\right)p^{m}\neq0$, and hence, it must be that $v_{m}=0$ for all $m\geq2$. So, only the $m=1$ case of (\ref{eq:m cases}) is left: \begin{equation} v_{0}+v_{1}\left(\kappa\left(j\right)-1\right)p\overset{\overline{\mathbb{Q}}}{=}0 \end{equation} Since $v_{0}=0$, this gives: \begin{equation} v_{1}\left(\kappa\left(j\right)-1\right)p\overset{\overline{\mathbb{Q}}}{=}0 \end{equation} Since $0<\left|\kappa\left(j\right)\right|_{q}<1$, this then shows that $\left(\kappa\left(j\right)-1\right)p\neq0$, and hence that $v_{1}=0$. So, $v_{n}=0$ for all $n\geq0$. This means the radially symmetric measure $d\nu$ is identically zero. Thus, so is $d\mu$\textemdash but $d\mu$ was given to be non-zero. There's our contradiction. Thus, if $d\mu$ is non-zero, it must be that $A\neq p-1$. Q.E.D. \begin{prop} Let $d\nu\in M_{\textrm{dgen}}\left(\mathbb{Z}_{p},K\right)$. Then $e^{2\pi i\left\{ \tau\mathfrak{z}\right\} _{p}}d\nu\left(\mathfrak{z}\right)$ is a degenerate measure for all $\tau\in\hat{\mathbb{Z}}_{p}$. \end{prop} Proof: The Fourier-Stieltjes transform of $d\mu\left(\mathfrak{z}\right)\overset{\textrm{def}}{=}e^{2\pi i\left\{ \tau\mathfrak{z}\right\} _{p}}d\nu\left(\mathfrak{z}\right)$ is $\hat{\nu}\left(t-\tau\right)$. Then, letting $N$ be large enough so that $\left|\tau\right|_{p}\leq p^{N}$, we have that the map $t\mapsto t-\tau$ is then a bijection of the set $\left\{ t\in\hat{\mathbb{Z}}_{p}:\left|t\right|_{p}\leq p^{N}\right\} $. For all such $N$: \begin{align*} \tilde{\mu}_{N}\left(\mathfrak{z}\right) & =\sum_{\left|t\right|_{p}\leq p^{N}}\hat{\nu}\left(t-\tau\right)e^{2\pi i\left\{ t\mathfrak{z}\right\} _{p}}\\ & =\sum_{\left|t\right|_{p}\leq p^{N}}\hat{\nu}\left(t\right)e^{2\pi i\left\{ \left(t+\tau\right)\mathfrak{z}\right\} _{p}}\\ & =e^{2\pi i\left\{ \tau\mathfrak{z}\right\} _{p}}\tilde{\nu}_{N}\left(\mathfrak{z}\right) \end{align*} Since $e^{2\pi i\left\{ \tau\mathfrak{z}\right\} _{p}}$ is bounded in $\mathbb{C}$ for all $\mathfrak{z}\in\mathbb{N}_{0}$, the degeneracy of $d\nu$ guarantees that $\tilde{\mu}_{N}\left(\mathfrak{z}\right)\overset{\mathbb{C}}{\rightarrow}0$ as $N\rightarrow\infty$ for all $\mathfrak{z}\in\mathbb{N}_{0}$. Likewise, since $e^{2\pi i\left\{ \tau\mathfrak{z}\right\} _{p}}$ is bounded in $\mathbb{C}_{q}$ for all $\mathfrak{z}\in\mathbb{Z}_{p}^{\prime}$, the degeneracy of $d\nu$ guarantees that $\tilde{\mu}_{N}\left(\mathfrak{z}\right)\overset{\mathbb{C}_{q}}{\rightarrow}0$ as $N\rightarrow\infty$ for all $\mathfrak{z}\in\mathbb{Z}_{p}^{\prime}$. Hence, $d\mu=e^{2\pi i\left\{ \tau\mathfrak{z}\right\} _{p}}d\nu\left(\mathfrak{z}\right)$ is degenerate. Q.E.D. \vphantom{} For our purposes, the following proposition will be of great import. \begin{prop} Let $d\mu$ be a degenerate ED2 measure. Then, either $d\mu$ is the zero measure, or $\hat{\mu}\left(0\right)\neq0$. \end{prop} Proof: Let $d\mu$ be as given. If $d\mu$ is the zero measure, then $\hat{\mu}\left(0\right)=0$. Thus, to complete the proof, we need only show that $\hat{\mu}\left(0\right)\neq0$ whenever $d\mu$ is a non-zero degenerate ED2 measure. First, let $d\mu$ be elementary, and suppose $\hat{\mu}\left(0\right)=0$. Since $\hat{\mu}\left(0\right)=\hat{\nu}\left(0\right)\hat{\eta}\left(0\right)=\hat{\nu}\left(1\right)\kappa\left(0\right)$, there are two possibilities: either $\kappa\left(0\right)=0$ or $\hat{\nu}\left(1\right)=0$. If $\kappa\left(0\right)=0$, \textbf{Lemma \ref{lem:structural equations uniquely determine p-adic structured functions}} shows that $d\mu$ is then the zero measure. Alternatively, suppose $\hat{\nu}\left(1\right)=0$. Then, since $d\nu$ is an ED2 measure, we have that: \begin{equation} \hat{\nu}\left(p^{-n}\right)=\left(1+A\right)\hat{\nu}\left(p^{-\left(n+1\right)}\right),\textrm{ }\forall n\geq0 \end{equation} If $A=-1$, then $\hat{\nu}\left(p^{-n}\right)$ is zero for all $n\geq1$. Since $\hat{\nu}\left(1\right)=\hat{\nu}\left(p^{-0}\right)$ was assumed to be $0$, the radial symmetry of $\hat{\nu}$ forces $\hat{\nu}\left(t\right)=0$ for all $t\in\hat{\mathbb{Z}}_{p}$. Since $\hat{\mu}\left(t\right)=\hat{\nu}\left(t\right)\hat{\eta}\left(t\right)$, this shows that $d\mu$ is the zero measure. On the other hand, if $A\neq-1$, then: \begin{equation} \hat{\nu}\left(p^{-\left(n+1\right)}\right)=\frac{1}{A+1}\hat{\nu}\left(p^{-n}\right),\textrm{ }\forall n\geq0 \end{equation} and thus, the assumption that $\hat{\nu}\left(1\right)=0$ then forces $\hat{\nu}\left(p^{-n}\right)=0$ for all $n\geq0$. Like with the $A=-1$ case, this forces $d\mu$ to be the zero measure. Thus, either $\hat{\mu}\left(0\right)\neq0$ or $d\mu$ is the zero measure. Q.E.D. \begin{question} \label{que:3.3}Although we shall not explore it here, there is also the question of how spaces of $\left(p,q\right)$-adic measures behave under pre-compositions by maps $\phi:\mathbb{Z}_{p}\rightarrow\mathbb{Z}_{p}$. For example, given a rising-continuous $d\mu$, consider the measure $d\nu$ defined by: \begin{equation} \hat{\nu}\left(t\right)\overset{\textrm{def}}{=}\hat{\mu}\left(t\right)e^{-2\pi i\left\{ t\mathfrak{a}\right\} _{p}} \end{equation} where $\mathfrak{a}\in\mathbb{Z}_{p}$ is a constant (i.e., $d\nu\left(\mathfrak{z}\right)=d\mu\left(\mathfrak{z}-\mathfrak{a}\right)$). Will $d\nu$ still be rising continuous? How will the frame of convergence for $\lim_{N\rightarrow\infty}\tilde{\mu}_{N}\left(\mathfrak{z}\right)$ be affected by the map $d\mu\left(\mathfrak{z}\right)\mapsto d\mu\left(\mathfrak{z}-\mathfrak{a}\right)$? The most general case of this kind of question would be to consider the map $d\mu\left(\mathfrak{z}\right)\mapsto d\mu_{\phi}\left(\mathfrak{z}\right)$ where $\phi:\mathbb{Z}_{p}\rightarrow\mathbb{Z}_{p}$ is any sufficiently nice map, as in \textbf{\emph{Definition \ref{def:pullback measure}}}\emph{ on page \pageref{def:pullback measure}}. These all appear to be worth exploring. \end{question} \subsection{\label{subsec:3.3.5 Quasi-Integrability}Quasi-Integrability} \begin{rem} Like in Subsection \ref{subsec:3.3.3 Frames}, this subsection exists primarily to give a firm foundation for my concept of quasi-integrability. With regard to the analysis of $\chi_{H}$ to be performed in Chapter 4, the only thing the reader needs to know is that I say a function $\chi:\mathbb{Z}_{p}\rightarrow\mathbb{C}_{q}$ is \textbf{quasi-integrable }with\index{quasi-integrability} respect to the standard $\left(p,q\right)$-adic frame whenever there exists a function $\hat{\chi}:\hat{\mathbb{Z}}_{p}\rightarrow\overline{\mathbb{Q}}$ so that: \begin{equation} \chi\left(\mathfrak{z}\right)\overset{\mathcal{F}_{p,q}}{=}\lim_{N\rightarrow\infty}\sum_{\left|t\right|_{p}\leq p^{N}}\hat{\chi}\left(t\right)e^{2\pi i\left\{ t\mathfrak{z}\right\} _{p}},\textrm{ }\forall\mathfrak{z}\in\mathbb{Z}_{p} \end{equation} where, recall, the $\mathcal{F}_{p,q}$ means that we interpret the right-hand side in the topology of $\mathbb{C}$ when $\mathfrak{z}\in\mathbb{N}_{0}$ and in the topology of $\mathbb{C}_{q}$ whenever $\mathfrak{z}\in\mathbb{Z}_{p}^{\prime}$. We call the function $\hat{\chi}$ \emph{a }\textbf{Fourier transform}\footnote{The reason we cannot say \emph{the }Fourier transform of $\chi$ is because of the existence of degenerate measures. For those who did not read all of Subsection \ref{subsec:3.3.3 Frames}, a $\left(p,q\right)$-adic measure $d\mu$ is said to \index{measure!degenerate}be \textbf{degenerate }(with respect to the standard frame) whenever its Fourier-Stieltjes transform $\hat{\mu}$ takes values in $\overline{\mathbb{Q}}$ and satisfies the limit condition: \begin{equation} \lim_{N\rightarrow\infty}\underbrace{\sum_{\left|t\right|_{p}\leq p^{N}}\hat{\mu}\left(t\right)e^{2\pi i\left\{ t\mathfrak{z}\right\} _{p}}}_{\tilde{\mu}_{N}\left(\mathfrak{z}\right)}\overset{\mathcal{F}_{p,q}}{=}0,\textrm{ }\forall\mathfrak{z}\in\mathbb{Z}_{p} \end{equation} As a result, given any Fourier transform $\hat{\chi}$ of the quasi-integrable function $\chi$, the function $\hat{\chi}+\hat{\mu}$ will \emph{also }be a Fourier transform of $\chi$ for any degenerate measure $d\mu$. This prevents $\hat{\chi}$ from being unique.}\textbf{ }of $\chi$. Finally, given a Fourier transform $\hat{\chi}$ of a quasi-integrable function $\chi$, we write $\tilde{\chi}_{N}$ to denote the $N$th partial Fourier series generated by $\hat{\chi}$. Interested readers can continue with this subsection to learn more (although, in that case, they should probably read Subsection \ref{subsec:3.3.3 Frames}, assuming they haven't already done so). Otherwise, the reader can safely proceed to the next subsection. \vphantom{} \end{rem} HERE BEGINS THE DISCUSSION OF QUASI-INTEGRABILITY. \vphantom{} Given the issues discussed in Subsection \ref{subsec:3.3.3 Frames}, it would be a bit unreasonable to expect a comprehensive definition of the phenomenon of quasi-integrability given the current novelty of $\left(p,q\right)$-adic analysis. As such, the definition I am about to give should come with an asterisk attached; it is in desperate need of deeper examination, especially regarding interactions with the many types of measures described in \ref{subsec:3.3.3 Frames}. \begin{defn} Consider a $p$-adic frame $\mathcal{F}$. We say a function $\chi\in C\left(\mathcal{F}\right)$ is\textbf{ quasi-integrable} \textbf{with respect to $\mathcal{F}$} / $\mathcal{F}$\textbf{-quasi-integrable }whenever $\chi:U_{\mathcal{F}}\rightarrow I\left(\mathcal{F}\right)$ is the derivative of some $\mathcal{F}$-rising measure $d\mu\in M_{\textrm{rise}}\left(\mathcal{F}\right)$. That is, there is a measure $d\mu$ whose Fourier-Stieltjes transform $\hat{\mu}$ satisfies: \vphantom{} I. $\hat{\mu}\left(t\right)\in\overline{\mathbb{Q}}$, $\forall t\in\hat{\mathbb{Z}}_{p}$; \vphantom{} II. $\left\Vert \hat{\mu}\right\Vert _{p,K_{\mathfrak{z}}}<\infty$ all $\mathfrak{z}\in U_{\textrm{non}}\left(\mathcal{F}\right)$; \vphantom{} II. $\tilde{\mu}_{N}$ $\mathcal{F}$-converges to $\chi$: \begin{equation} \lim_{N\rightarrow\infty}\left|\chi\left(\mathfrak{z}\right)-\tilde{\mu}_{N}\left(\mathfrak{z}\right)\right|_{K_{\mathfrak{z}}}\overset{\mathbb{R}}{=}0,\textrm{ }\forall\mathfrak{z}\in U_{\mathcal{F}}\label{eq:definition of quasi-integrability} \end{equation} We call any $d\mu$ satisfying these properties an\textbf{ $\mathcal{F}$-quasi-integral}\index{mathcal{F}-@$\mathcal{F}$-!quasi-integral}\textbf{ }of $\chi$, dropping the $\mathcal{F}$ when there is no confusion. More generally, we say a function $\chi$ is quasi-integrable\index{quasi-integrability}\textbf{ }if it is quasi-integrable with respect to some frame $\mathcal{F}$. We then write \nomenclature{$\tilde{L}^{1}\left(\mathcal{F}\right)$}{set of $\mathcal{F}$-quasi-integrable functions}$\tilde{L}^{1}\left(\mathcal{F}\right)$ to denote the set of all $\mathcal{F}$-quasi-integrable functions. (Note that this is a vector space over $\overline{\mathbb{Q}}$.) When $\mathcal{F}$ is the standard $\left(p,q\right)$-adic frame, this definition becomes: \vphantom{} i. $\chi$ is a function from $\mathbb{Z}_{p}\rightarrow\mathbb{C}_{q}$ so that $\chi\left(\mathfrak{z}\right)\in\mathbb{C}$ for all $\mathfrak{z}\in\mathbb{N}_{0}$ and $\chi\left(\mathfrak{z}\right)\in\mathbb{C}_{q}$ for all $\mathfrak{z}\in\mathbb{Z}_{p}^{\prime}$. \vphantom{} ii. For each $\mathfrak{z}\in\mathbb{N}_{0}$, as $N\rightarrow\infty$, $\tilde{\mu}_{N}\left(\mathfrak{z}\right)$ converges to $\chi\left(\mathfrak{z}\right)$ in the topology of $\mathbb{C}$. \vphantom{} iii. For each $\mathfrak{z}\in\mathbb{Z}_{p}^{\prime}$, as $N\rightarrow\infty$, $\tilde{\mu}_{N}\left(\mathfrak{z}\right)$ converges to $\chi\left(\mathfrak{z}\right)$ in the topology of $\mathbb{C}_{q}$. \vphantom{} We then write \nomenclature{$\tilde{L}^{1}\left(\mathcal{F}_{p,q}\right)$}{ }$\tilde{L}^{1}\left(\mathcal{F}_{p,q}\right)$ and $\tilde{L}^{1}\left(\mathbb{Z}_{p},\mathbb{C}_{q}\right)$ \nomenclature{$\tilde{L}^{1}\left(\mathbb{Z}_{p},\mathbb{C}_{q}\right)$}{ } to denote the space of all functions $\mathbb{Z}_{p}\rightarrow\mathbb{C}_{q}$ which are quasi-integrable with respect to the standard frame $\left(p,q\right)$-adic frame. \end{defn} \vphantom{} Using quasi-integrability, we can define a generalization of the $\left(p,q\right)$-adic Fourier transform. The proposition which guarantees this is as follows: \begin{prop} Let $\chi\in\tilde{L}^{1}\left(\mathcal{F}\right)$. Then, the quasi-integral of $\chi$ is unique modulo $M_{\textrm{dgen}}\left(\mathcal{F}\right)$. That is, if two measures $d\mu$ and $d\nu$ are both quasi-integrals of $\chi$, then the measure $d\mu-d\nu$ is $\mathcal{F}$-degenerate\index{measure!mathcal{F}-degenerate@$\mathcal{F}$-degenerate}\index{mathcal{F}-@$\mathcal{F}$-!degenerate}. Equivalently, we have an isomorphism of $\overline{\mathbb{Q}}$-linear spaces: \begin{equation} \tilde{L}^{1}\left(\mathcal{F}\right)\cong M_{\textrm{rise}}\left(\mathcal{F}\right)/M_{\textrm{dgen}}\left(\mathcal{F}\right)\label{eq:L1 twiddle isomorphism to M alg / M dgen} \end{equation} where the isomorphism associates a given $\chi\in\tilde{L}^{1}\left(\mathcal{F}\right)$ to the equivalence class of $\mathcal{F}$-rising measures which are quasi-integrals of $\chi$. \end{prop} Proof: Let $\mathcal{F}$, $\chi$, $d\mu$, and $d\nu$ be as given. Then, by the definition of what it means to be a quasi-integral of $\chi$, for the measure $d\eta\overset{\textrm{def}}{=}d\mu-d\nu$, we have that $\tilde{\eta}_{N}\left(\mathfrak{z}\right)$ tends to $0$ in $K_{\mathfrak{z}}$ for all $\mathfrak{z}\in U_{\mathcal{F}}$. Hence, $d\mu-d\nu$ is indeed $\mathcal{F}$-degenerate. Q.E.D. \begin{defn} Let $\chi$ be quasi-integrable with respect to $\mathcal{F}$. Then, \textbf{a} \textbf{Fourier Transform} of $\chi$ is a function $\hat{\mu}:\hat{\mathbb{Z}}_{p}\rightarrow\overline{\mathbb{Q}}$ which is the Fourier-Stieltjes transform of some $\mathcal{F}$-quasi-integral $d\mu$ of $\chi$. We write these functions as $\hat{\chi}$ and write the associated measures as $\chi\left(\mathfrak{z}\right)d\mathfrak{z}$. \end{defn} \vphantom{} Because there exist degenerate measures which are not the zero measure, our use of the notation $\chi\left(\mathfrak{z}\right)d\mathfrak{z}$ to denote a given quasi-integral $d\mu\left(\mathfrak{z}\right)$ of $\chi$ is \emph{not} well-defined, being dependent on our particular choice of $\hat{\chi}$. So, provided we are working with a fixed choice of $\hat{\chi}$, this then allows us to define the integral $\int_{\mathbb{Z}_{p}}f\left(\mathfrak{z}\right)\chi\left(\mathfrak{z}\right)d\mathfrak{z}$ for any $f\in C\left(\mathbb{Z}_{p},\mathbb{C}_{q}\right)$ by interpreting it as the image of $f$ under integration against the measure $\chi\left(\mathfrak{z}\right)d\mathfrak{z}$: \begin{equation} \int_{\mathbb{Z}_{p}}f\left(\mathfrak{z}\right)\chi\left(\mathfrak{z}\right)d\mathfrak{z}\overset{\mathbb{C}_{q}}{=}\sum_{t\in\hat{\mathbb{Z}}_{p}}\hat{f}\left(-t\right)\hat{\chi}\left(t\right),\textrm{ }\forall f\in C\left(\mathbb{Z}_{p},\mathbb{C}_{q}\right)\label{eq:Integral of a quasi-integrable function} \end{equation} More pedantically, we could say that ``the'' Fourier transform of $\chi$ is an equivalence class of functions $\hat{\mathbb{Z}}_{p}\rightarrow\overline{\mathbb{Q}}$ so that the difference of any two functions in the equivalence class is the Fourier-Stieltjes transform of an $\mathcal{F}$-degenerate algebraic measure. Additionally, because the Fourier transform of $\chi$ is unique only modulo degenerate measures, there is (at least at present) no \emph{canonical }choice for what $\hat{\chi}$ should be. In this dissertation, the symbol $\hat{\chi}$ will be used in one of two senses: either to designate an \emph{arbitrary }choice of a Fourier transform for a quasi-integrable function $\chi$\textemdash as is the case in Chapters 3 \& 5\textemdash OR, to designate a \emph{specific }choice of a Fourier transform for a quasi-integrable function $\chi$, as is the case in Chapters 4 \& 6. \begin{defn} Given a choice of Fourier transform $\hat{\chi}$ for \nomenclature{$\tilde{\chi}_{N}\left(\mathfrak{z}\right)$}{$N$th partial sum of Fourier series generated by $\hat{\chi}$.} $\chi\in\tilde{L}^{1}\left(\mathcal{F}\right)$, we write: \begin{equation} \tilde{\chi}_{N}\left(\mathfrak{z}\right)\overset{\textrm{def}}{=}\sum_{\left|t\right|_{p}\leq p^{N}}\hat{\chi}\left(t\right)e^{2\pi i\left\{ t\mathfrak{z}\right\} _{p}},\textrm{ }\forall\mathfrak{z}\in\mathbb{Z}_{p}\label{eq:Definition of Chi_N twiddle} \end{equation} to denote the \textbf{$N$th partial sum of the Fourier series generated by $\hat{\chi}$}. \end{defn} \vphantom{} Working with quasi-integrable functions in the abstract is particularly difficult because it is not at all clear how to actually \emph{prove }anything without being able to draw upon identities like the Fourier Resummation Lemmata. Moreover, for \emph{specific }quasi-integrable functions\textemdash those given by a particular formula\textemdash one can often easily prove many results which seem nearly impossible to address in the abstract. Conversely, those results which \emph{can }be proven in the abstract are generally unimpressive; case in point: \begin{prop} \label{prop:Chi_N and Chi_twiddle N F converge to one another}Let $\chi\in\tilde{L}^{1}\left(\mathcal{F}\right)$. Then, for any Fourier transform $\hat{\chi}$ of $\chi$, the difference $\chi_{N}-\tilde{\chi}_{N}$ (where $\chi_{N}$ is the $N$th truncation of $\chi$) $\mathcal{F}$-converges to $0$. \end{prop} Proof: Immediate from the definitions of $\hat{\chi}$, $\tilde{\chi}_{N}$, and $\chi_{N}$. Q.E.D. \vphantom{} Both because it is not my purpose to do so here, and as a simple matter of time constraints, there are quite a few issues in the theory of quasi-integrable functions that I have not yet been able to resolve to my satisfaction. Below, I have outlined what I feel are the most outstanding issues. \begin{question} \label{que:3.4}Fix a $\left(p,q\right)$-adic frame $\mathcal{F}$. Given a function $\chi:\mathbb{Z}_{p}\rightarrow\mathbb{C}_{q}$, is there a practical criterion for determining whether or not $\chi$ is $\mathcal{F}$-quasi-integrable? Similarly, given a function $\hat{\chi}:\hat{\mathbb{Z}}_{p}\rightarrow\mathbb{C}_{q}$, is there a practical criterion for determining whether or not $\hat{\chi}$ is the Fourier transform of an $\mathcal{F}$-quasi-integrable function (other than directly summing the Fourier series generated by $\hat{\chi}$)? \end{question} \vphantom{} Question \ref{que:3.4} probably bothers me more than any other outstanding issue\footnote{The verification of when the convolution of quasi-integrable functions is, itself, quasi-integrable comes in at a close second.} in this fledgling theory. At present, the only means I have of verifying quasi-integrability is by performing the rather intensive computational regimen of Chapters 4 and 6 on a given $\chi$ in the hopes of being able to find a function $\hat{\chi}$ which serves as the Fourier transform of $\chi$ with respect to some frame. Arguably even worse is the fact that there currently doesn't appear to be any reasonable criterion for showing that a function is \emph{not }quasi-integrable. In particular, despite being nearly certain that the following conjecture is true, I have no way to prove it: \begin{conjecture} Let $\mathfrak{a}\in\mathbb{Z}_{p}$. Then, the one-point function $\mathbf{1}_{\mathfrak{a}}\left(\mathfrak{z}\right)$ is not quasi-integrable with respect to any $\left(p,q\right)$-adic quasi-integrability frame. \end{conjecture} \vphantom{} The methods of Chapters 4 and 6 do not seem applicable here. These methods are best described as a kind of ``asymptotic analysis'' in which we truncate the function $\chi$ being studied and then explicitly compute the Fourier transform $\hat{\chi}_{N}\left(t\right)$ of the locally constant (and hence, continuous) $\left(p,q\right)$-adic function $\chi_{N}\left(\mathfrak{z}\right)$. Careful computational manipulations allow us to ``untangle'' the dependence of $N$ and $t$ in $\hat{\chi}_{N}\left(t\right)$ and to express $\hat{\chi}_{N}\left(t\right)$ in the form: \begin{equation} \hat{\chi}_{N}\left(t\right)=\hat{F}_{N}\left(t\right)+\mathbf{1}_{0}\left(p^{N}t\right)\hat{G}\left(t\right) \end{equation} where $\hat{F}_{N},\hat{G}:\hat{\mathbb{Z}}_{p}\rightarrow\overline{\mathbb{Q}}$ are $q$-adically bounded, and with the Fourier series generated by $\hat{F}_{N}$ converging to zero in the standard frame as $N\rightarrow\infty$. $\hat{G}\left(t\right)$ can be thought of as the ``fine structure'' hidden beneath the tumult of the divergent $\hat{F}_{N}\left(t\right)$ term. This suggests that $\hat{G}$ should be a Fourier transform of $\hat{\chi}$\textemdash and, with some work, this intuition will be proved correct. Even though the Fourier series generated by $\hat{F}_{N}$ only $\mathcal{F}_{p,q}$-converges to $0$ when $\chi$ satisfies certain arithmetical properties\footnote{For example, of the $\chi_{q}$s (the numina of the shortened $qx+1$ maps), the $\hat{F}_{N}$ term's Fourier series is $\mathcal{F}_{2,q}$-convergent to $0$ if and only if $q=3$.}, we can bootstrap this special case and use it to tackle the more general case where the Fourier series generated by $\hat{F}_{N}$ \emph{doesn't} $\mathcal{F}_{p,q}$-converge to $0$. Applying this procedure to, say, $\mathbf{1}_{0}\left(\mathfrak{z}\right)$, the $N$th truncation of $\mathbf{1}_{0}\left(\mathfrak{z}\right)$ is: \begin{equation} \mathbf{1}_{0,N}\left(\mathfrak{z}\right)=\left[\mathfrak{z}\overset{p^{N}}{\equiv}0\right]\label{eq:Nth truncation of 1_0} \end{equation} which has the Fourier transform: \begin{equation} \hat{\mathbf{1}}_{0,N}\left(t\right)=\frac{1}{p^{N}}\mathbf{1}_{0}\left(p^{N}t\right)=\begin{cases} \frac{1}{p^{N}} & \textrm{if }\left|t\right|_{p}\leq p^{N}\\ 0 & \textrm{if }\left|t\right|_{p}>p^{N} \end{cases}\label{eq:Fourier transform of Nth truncation of 1_0} \end{equation} On the one hand, there appears to be no way to replicate Chapter 4's argument to obtain a candidate for a Fourier transform of $\mathbf{1}_{0}$ and thereby establish its quasi-integrability. But how might we make this \emph{rigorous}? At present, I have no idea. Only time will tell if it will stay that way. The next major issue regards the convolution of quasi-integrable functions. \begin{question} Fix a frame $\mathcal{F}$, and let $\chi,\kappa\in\tilde{L}^{1}\left(\mathcal{F}\right)$. The natural definition for the \textbf{convolution }of $\chi$ and $\kappa$ would be: \begin{equation} \lim_{N\rightarrow\infty}\sum_{\left|t\right|_{p}\leq p^{N}}\hat{\chi}\left(t\right)\hat{\kappa}\left(t\right)e^{2\pi i\left\{ t\mathfrak{z}\right\} _{p}}\label{eq:Limit formula for the convolution of two quasi-integrable functions} \end{equation} for a choice $\hat{\chi}$ and $\hat{\kappa}$ of Fourier transforms of $\chi$ and $\kappa$, respectively. However, this immediately leads to difficulties: (1) As defined, $\hat{\chi}$ and $\hat{\kappa}$ are unique only up to $\mathcal{F}$-degenerate measures. However, as is shown in \emph{(\ref{eq:Limit of Fourier sum of v_p A_H hat}) }from \emph{Chapter 4's} \textbf{\emph{Lemma \ref{lem:v_p A_H hat summation formulae}}} \emph{(page \pageref{lem:v_p A_H hat summation formulae})}, there exists a degenerate measure $d\mu$ and a bounded $\hat{\nu}:\hat{\mathbb{Z}}_{p}\rightarrow\overline{\mathbb{Q}}$ so that: \[ \lim_{N\rightarrow\infty}\sum_{\left|t\right|_{p}\leq p^{N}}\hat{\nu}\left(t\right)\hat{\mu}\left(t\right)e^{2\pi i\left\{ t\mathfrak{z}\right\} _{p}} \] is \textbf{\emph{not}}\emph{ }$\mathcal{F}$-convergent to $0$ everywhere. As a result, it appears that \emph{(\ref{eq:Limit formula for the convolution of two quasi-integrable functions})} will not be well defined modulo $\mathcal{F}$-degenerate measures; that is, replacing $\hat{\chi}$ in \emph{(\ref{eq:Limit formula for the convolution of two quasi-integrable functions})} with $\hat{\chi}+\hat{\mu}$ where $d\mu$ is $\mathcal{F}$-degenerate may cause the $\mathcal{F}$-limit of \emph{(\ref{eq:Limit formula for the convolution of two quasi-integrable functions})} to change. This prompts the question: there a way to \uline{uniquely} define the convolution of two quasi-integrable functions? Or will we have to make everything contingent upon the particular choice of Fourier transforms in order to work with quasi-integrable functions? (2) As an extension of the present lack of a criterion for determining whether or not a given $\chi$ is quasi-integrable (or whether a given $\hat{\chi}$ is a Fourier transform of a quasi-integrable function) are there any useful conditions we can impose on $\chi$, $\kappa$, $\hat{\chi}$, and $\hat{\kappa}$ to ensure that \emph{(\ref{eq:Limit formula for the convolution of two quasi-integrable functions})} exists and is quasi-integrable? The one obvious answer is when one or both of $\hat{\chi}$ or $\hat{\kappa}$ is an element of $c_{0}\left(\hat{\mathbb{Z}}_{p},\mathbb{C}_{q}\right)$, in which case $\chi*\kappa$ will then be a continuous function, due to the $q$-adic decay of $\hat{\chi}\cdot\hat{\kappa}$. However, that would be quite an unsatisfying state of affairs! Worse, the computations done in \emph{Chapter 4} involve convolving quasi-integrable functions; for example \emph{(\ref{eq:Limit of Fourier sum of v_p A_H hat} }from \textbf{\emph{Lemma \ref{lem:v_p A_H hat summation formulae}}} on page \emph{\pageref{lem:v_p A_H hat summation formulae})} arises in the analysis of $\chi_{3}$, in which we convolve $dA_{3}$ with the quasi-integrable function obtained by letting $N\rightarrow\infty$ in \emph{(\ref{eq:Fourier sum of v_p of t})}. So, there are clearly cases where convolution \emph{\uline{is}}\emph{ }valid! \end{question} \begin{question} \label{que:3.6}How, if at all, can we work with functions $\chi$ and $\kappa$ which are quasi-integrable with respect to \uline{different frames}? \end{question} \subsection{\label{subsec:3.3.6 L^1 Convergence}$L^{1}$ Convergence} When doing real or complex analysis, few settings are as fundamental as $L^{p}$ spaces, with $L^{1}$ and $L^{2}$ possessing a mutual claim to primacy. As\index{non-archimedean!$L^{1}$ theory} was mentioned in Subsection \ref{subsec:3.1.2 Banach-Spaces-over}, however, neither $L^{1}$ nor $L^{2}$\textemdash nor any $L^{p}$ space, for that matter\textemdash have much significance when working with functions taking values in a non-archimedean valued field. The magic of the complex numbers is that they are a finite-dimensional field extension of $\mathbb{R}$ which is nonetheless algebraically complete. Because $\mathbb{C}_{q}$ is an infinite dimensional extension of $\mathbb{Q}_{q}$, the Galois-theoretic identity $z\overline{z}=\left|z\right|^{2}$ from complex analysis does not hold, and the elegant duality of $L^{2}$ spaces gets lost as a result in the $q$-adic context. From an analytical standpoint, the issue with non-archimedean $L^{1}$ spaces is arguably even more fundamental. For infinite series of complex numbers, we need absolute convergence in order to guarantee the invariance of the sum under groupings and rearrangements of its terms. In the ultrametric setting, however, this is no longer the case: all that matters is that the $n$th term of the series tend to $0$ in non-archimedean absolute value. Because of this, as we saw in our discussion of Monna-Springer Integration in Subsection \ref{subsec:3.1.6 Monna-Springer-Integration}, when working with non-archimedean absolute values, there is no longer a meaningful connection between $\left|\int f\right|$ and $\int\left|f\right|$. Despite this, it appears that a non-archimedean $L^{1}$ theory \emph{is} implicated in $\left(p,q\right)$-adic analysis. Rather than begin with theory or definitions, let us start with a concrete, Collatz-adjacent example involving $\chi_{5}$\index{chi{5}@$\chi_{5}$}, the numen of \index{$5x+1$ map} the shortened $5x+1$ map. \begin{example} For a given $\mathcal{F}$-quasi-integrable $\chi$, \textbf{Proposition \ref{prop:Chi_N and Chi_twiddle N F converge to one another}} tells us that $\chi_{N}-\tilde{\chi}_{N}$ is $\mathcal{F}$-convergent to zero. Nevertheless, for any given choice of $\hat{\chi}$, the limit of the differences $\hat{\chi}_{N}\left(t\right)-\hat{\chi}\left(t\right)$ as $N\rightarrow\infty$ need not be well-behaved. As we will see in Chapter 4, $\chi_{5}\in\tilde{L}^{1}\left(\mathbb{Z}_{2},\mathbb{C}_{5}\right)$ (that is, $\chi_{5}$ is the derivative of a rising-continuous $\left(2,5\right)$-adic measure); moreover, one possible choice for a Fourier transform of $\chi_{5}$ is: \begin{equation} \hat{\chi}_{5}\left(t\right)=-\frac{1}{4}\mathbf{1}_{0}\left(t\right)-\frac{1}{4}\hat{A}_{5}\left(t\right),\textrm{ }\forall t\in\hat{\mathbb{Z}}_{2}\label{eq:Choice of Fourier Transform for Chi_5} \end{equation} where $\hat{A}_{5}$ is as defined in (\ref{eq:Definition of A_q hat}), and where $\mathbf{1}_{0}$ is the indicator function of $0$ in $\hat{\mathbb{Z}}_{2}$. Letting $\hat{\chi}_{5,N}$ denote the Fourier transform of the $N$th truncation of $\chi_{5}$ ($\chi_{5,N}$) it can be shown that for this choice of $\hat{\chi}_{5}$: \begin{equation} \hat{\chi}_{5,N}\left(t\right)-\hat{\chi}_{5}\left(t\right)=\begin{cases} \frac{1}{2}\left(\frac{3}{2}\right)^{N+\min\left\{ v_{2}\left(t\right),0\right\} }\hat{A}_{5}\left(t\right) & \textrm{if }\left|t\right|_{2}\leq2^{N}\\ \frac{1}{4}\hat{A}_{5}\left(t\right) & \textrm{if }\left|t\right|_{2}>2^{N} \end{cases},\textrm{ }\forall t\in\hat{\mathbb{Z}}_{2}\label{eq:Difference between Chi_5,N hat and Chi_5 hat} \end{equation} For any given $t$, as $N\rightarrow\infty$, the limit of this difference exists neither in $\mathbb{C}_{5}$ nor in $\mathbb{C}$, despite the fact that the difference of $\chi_{5,N}$ and $\tilde{\chi}_{5,N}$ (the $N$th partial sum of the Fourier series generated by $\hat{\chi}_{5}$) is $\mathcal{F}$-convergent to $0$. Nevertheless, going through the motions and applying Fourier inversion to the above (multiplying by $e^{2\pi i\left\{ t\mathfrak{z}\right\} _{2}}$ and summing over $\left|t\right|_{2}\leq2^{N}$), we obtain: \begin{equation} \chi_{5,N}\left(\mathfrak{z}\right)-\tilde{\chi}_{5,N}\left(\mathfrak{z}\right)=\frac{5^{\#_{1}\left(\left[\mathfrak{z}\right]_{2^{N}}\right)}}{2^{N+1}}\label{eq:Fourier inversion of Chi_5,N hat - Chi_5 hat} \end{equation} which, as expected, converges to zero $5$-adically as $N\rightarrow\infty$ for all $\mathfrak{z}\in\mathbb{Z}_{2}^{\prime}$ and converges to zero in $\mathbb{C}$ for all $\mathfrak{z}\in\mathbb{N}_{0}$. Note that this convergence is \emph{not }uniform over $\mathbb{Z}_{2}^{\prime}$: we can make the rate of convergence with respect to $N$ arbitrarily slow by choosing a $\mathfrak{z}\in\mathbb{Z}_{2}\backslash\mathbb{N}_{0}$ whose $1$s digits in its $2$-adic expansion are spaced sufficiently far apart. Despite the lack of the \emph{uniform} convergence of $\chi_{5,N}\left(\mathfrak{z}\right)-\tilde{\chi}_{5,N}\left(\mathfrak{z}\right)$ to zero, we can get $L^{1}$ convergence for certain cases, and in the classical sense, no less. We have: \begin{align*} \int_{\mathbb{Z}_{2}}\left|\chi_{5,N}\left(\mathfrak{z}\right)-\tilde{\chi}_{5,N}\left(\mathfrak{z}\right)\right|_{5}d\mathfrak{z} & \overset{\mathbb{R}}{=}\int_{\mathbb{Z}_{2}}\left(\frac{1}{5}\right)^{\#_{1}\left(\left[\mathfrak{z}\right]_{2^{N}}\right)}d\mathfrak{z}\\ & =\frac{1}{2^{N}}\sum_{n=0}^{2^{N}-1}\left(\frac{1}{5}\right)^{\#_{1}\left(n\right)}\\ & =\frac{1}{2^{N}}\underbrace{\prod_{n=0}^{N-1}\left(1+\frac{1}{5}\left(1^{2^{n}}\right)\right)}_{\left(6/5\right)^{N}}\\ & =\left(\frac{3}{5}\right)^{N} \end{align*} which tends to $0$ in $\mathbb{R}$ as $N\rightarrow\infty$. By using the non-trivial explicit formula for $\chi_{5}$ and $\chi_{5,N}$ from Subsection \ref{subsec:4.2.2}'s\textbf{ Corollary \ref{cor:F-series for Chi_H for p equals 2}} and \textbf{Theorem \ref{thm:F-series for Nth truncation of Chi_H, alpha is 1}},\textbf{ }we obtain: \begin{equation} \chi_{5}\left(\mathfrak{z}\right)-\chi_{5,N}\left(\mathfrak{z}\right)=-\frac{1}{4}\frac{5^{\#_{1}\left(\left[\mathfrak{z}\right]_{2^{N}}\right)}}{2^{N}}+\frac{1}{8}\sum_{n=N}^{\infty}\frac{5^{\#_{1}\left(\left[\mathfrak{z}\right]_{2^{n}}\right)}}{2^{n}}\label{eq:Difference between Chi_5 and Chi_5,N} \end{equation} where the convergence is in $\mathbb{C}$ for $\mathfrak{z}\in\mathbb{N}_{0}$ and is in $\mathbb{C}_{5}$ for $\mathfrak{z}\in\mathbb{Z}_{2}^{\prime}$. Again, regardless of the topology, the convergence is point-wise. Nevertheless, using the standard $5$-adic absolute value estimate on the right-hand side of (\ref{eq:Difference between Chi_5 and Chi_5,N}), we have: \begin{align*} \int_{\mathbb{Z}_{2}}\left|\chi_{5}\left(\mathfrak{z}\right)-\chi_{5,N}\left(\mathfrak{z}\right)\right|_{5}d\mathfrak{z} & \overset{\mathbb{R}}{\leq}\int_{\mathbb{Z}_{2}}\max_{n\geq N}\left(5^{-\#_{1}\left(\left[\mathfrak{z}\right]_{2^{n}}\right)}\right)d\mathfrak{z}\\ & =\int_{\mathbb{Z}_{2}}\left(\frac{1}{5}\right)^{\#_{1}\left(\left[\mathfrak{z}\right]_{2^{N}}\right)}d\mathfrak{z}\\ & =\left(\frac{3}{5}\right)^{N} \end{align*} which, once again, tends to $0$ in $\mathbb{R}$ as $N\rightarrow\infty$. Finally, when working with the $\left(2,5\right)$-adic integral of the difference, we obtain: \begin{align*} \int_{\mathbb{Z}_{2}}\left(\chi_{5,N}\left(\mathfrak{z}\right)-\tilde{\chi}_{5,N}\left(\mathfrak{z}\right)\right)d\mathfrak{z} & \overset{\mathbb{C}_{5}}{=}\int_{\mathbb{Z}_{2}}\frac{5^{\#_{1}\left(\left[\mathfrak{z}\right]_{2^{N}}\right)}}{2^{N+1}}d\mathfrak{z}\\ & \overset{\mathbb{C}_{5}}{=}\frac{1}{2^{N}}\sum_{n=0}^{2^{N}-1}\frac{5^{\#_{1}\left(n\right)}}{2^{N+1}}\\ & \overset{\mathbb{C}_{5}}{=}\frac{1/2}{4^{N}}\prod_{n=0}^{N-1}\left(1+5\left(1^{2^{n}}\right)\right)\\ & \overset{\mathbb{C}_{5}}{=}\frac{1}{2}\left(\frac{3}{2}\right)^{N} \end{align*} which, just like $\hat{\chi}_{5,N}\left(t\right)-\hat{\chi}_{5}\left(t\right)$, fails to converge $5$-adically as $N\rightarrow\infty$. Indeed, we actually have that both of these quantities fail to converge $5$-adically in precisely the same way: \[ \left(\frac{2}{3}\right)^{N}\int_{\mathbb{Z}_{2}}\left(\chi_{5,N}\left(\mathfrak{z}\right)-\tilde{\chi}_{5,N}\left(\mathfrak{z}\right)\right)d\mathfrak{z}\overset{\mathbb{C}_{5}}{=}\frac{1}{2},\textrm{ }\forall N\geq1 \] \[ \lim_{N\rightarrow\infty}\left(\frac{2}{3}\right)^{N}\left(\hat{\chi}_{5,N}\left(t\right)-\hat{\chi}_{5}\left(t\right)\right)\overset{\mathbb{C}_{5}}{=}\frac{1}{2}\left(\frac{3}{2}\right)^{N+\min\left\{ v_{2}\left(t\right),0\right\} }\hat{A}_{5}\left(t\right),\textrm{ }\forall t\in\hat{\mathbb{Z}}_{2} \] and hence: \[ \lim_{N\rightarrow\infty}\frac{\hat{\chi}_{5,N}\left(t\right)-\hat{\chi}_{5}\left(t\right)}{\int_{\mathbb{Z}_{2}}\left(\chi_{5,N}\left(\mathfrak{z}\right)-\tilde{\chi}_{5,N}\left(\mathfrak{z}\right)\right)d\mathfrak{z}}\overset{\mathbb{C}_{5}}{=}\left(\frac{3}{2}\right)^{\min\left\{ v_{2}\left(t\right),0\right\} }\hat{A}_{5}\left(t\right),\textrm{ }\forall t\in\hat{\mathbb{Z}}_{2} \] \end{example} \vphantom{} At present, the stringent restrictions demanded by $\mathcal{F}$-convergence seems to prevent us from equipping spaces such as $M\left(\mathcal{F}\right)$ or $\tilde{L}^{1}\left(\mathcal{F}\right)$ with a norm to induce a topology. Non-archimedean $L^{1}$ theory\index{non-archimedean!$L^{1}$ theory} might very well provided a viable work-around. \begin{defn} We write$\mathcal{L}_{\mathbb{R}}^{1}\left(\mathbb{Z}_{p},\mathbb{C}_{q}\right)$ to denote the set of all functions $f:\mathbb{Z}_{p}\rightarrow\mathbb{C}_{q}$ so that the real-valued function $\mathfrak{z}\in\mathbb{Z}_{p}\mapsto\left|f\left(\mathfrak{z}\right)\right|_{q}\in\mathbb{R}$ is integrable with respect to the real-valued Haar probability measure on $\mathbb{Z}_{p}$. As is customary, we then define an equivalence relation $\sim$ on $\mathcal{L}_{\mathbb{R}}^{1}\left(\mathbb{Z}_{p},\mathbb{C}_{q}\right)$, and say that $f\sim g$ whenever $\left|f\left(\mathfrak{z}\right)-g\left(\mathfrak{z}\right)\right|_{q}=0$ for almost every $\mathfrak{z}\in\mathbb{Z}_{p}$. We then write $L_{\mathbb{R}}^{1}\left(\mathbb{Z}_{p},\mathbb{C}_{q}\right)$\nomenclature{$L_{\mathbb{R}}^{1}\left(\mathbb{Z}_{p},\mathbb{C}_{q}\right)$}{set of $f:\mathbb{Z}_{p}\rightarrow\mathbb{C}_{q}$ so that $\left|f\right|_{q}\in L^{1}\left(\mathbb{Z}_{p},\mathbb{C}\right)$} to denote the space of equivalence classes: \begin{equation} L_{\mathbb{R}}^{1}\left(\mathbb{Z}_{p},\mathbb{C}_{q}\right)\overset{\textrm{def}}{=}\mathcal{L}_{\mathbb{R}}^{1}\left(\mathbb{Z}_{p},\mathbb{C}_{q}\right)/\sim\label{eq:Definition of L^1_R} \end{equation} and we make this an\emph{ archimedean}\footnote{That is, $\left\Vert \mathfrak{a}f+\mathfrak{b}g\right\Vert _{1}\leq\left|\mathfrak{a}\right|_{q}\left\Vert f\right\Vert _{1}+\left|\mathfrak{b}\right|_{q}\left\Vert g\right\Vert _{1}$ for all $f,g\in L_{\mathbb{R}}^{1}\left(\mathbb{Z}_{p},\mathbb{C}_{q}\right)$ and all $\mathfrak{a},\mathfrak{b}\in\mathbb{C}_{q}$.}\emph{ }normed linear space over $\mathbb{C}_{q}$ by defining: \begin{equation} \left\Vert f\right\Vert _{1}\overset{\textrm{def}}{=}\int_{\mathbb{Z}_{p}}\left|f\left(\mathfrak{z}\right)\right|_{q}d\mathfrak{z}\in\mathbb{R},\textrm{ }\forall f\in L_{\mathbb{R}}^{1}\left(\mathbb{Z}_{p},\mathbb{C}_{q}\right)\label{eq:Definition of L^1 norm} \end{equation} \end{defn} \begin{lem} \label{lem:Egorov lemma}Let $\left\{ f_{n}\right\} _{n\geq1}$ be a sequence of functions $f_{n}:\mathbb{Z}_{p}\rightarrow\mathbb{C}_{q}$ so that: \vphantom{} i. The real-valued functions $\mathfrak{z}\mapsto\left|f_{n}\left(\mathfrak{z}\right)\right|_{q}$ are measurable with respect to the real-valued Haar probability measure on $\mathbb{Z}_{p}$; \vphantom{} ii. There is a constant $C\in\mathbb{R}>0$ so that for each $n$, $\left|f_{n}\left(\mathfrak{z}\right)\right|_{q}<C$ holds almost everywhere. \vphantom{} Then: \vphantom{} I. If $\lim_{n\rightarrow\infty}\left|f_{n}\left(\mathfrak{z}\right)\right|_{q}\overset{\mathbb{R}}{=}0$ holds for almost every $\mathfrak{z}\in\mathbb{Z}_{p}$, then $\lim_{n\rightarrow\infty}\int_{\mathbb{Z}_{p}}\left|f_{n}\left(\mathfrak{z}\right)\right|_{q}d\mathfrak{z}\overset{\mathbb{R}}{=}0$. \vphantom{} II. If $\lim_{n\rightarrow\infty}\int_{\mathbb{Z}_{p}}\left|f_{n}\left(\mathfrak{z}\right)\right|_{q}d\mathfrak{z}\overset{\mathbb{R}}{=}0$ then there is a subsequence $\left\{ f_{n_{j}}\right\} _{j\geq1}$ so that $\lim_{j\rightarrow0}\left|f_{n_{j}}\left(\mathfrak{z}\right)\right|_{q}\overset{\mathbb{R}}{=}0$ holds for almost every $\mathfrak{z}\in\mathbb{Z}_{p}$. \end{lem} Proof: I. Suppose the $\left|f_{n}\right|_{q}$s converge point-wise to $0$ almost everywhere. Then, letting $\epsilon>0$, by \textbf{Egorov's Theorem}\footnote{Let $\left(X,\Omega,\mu\right)$ be a measure space with $\mu\left(X\right)<\infty$, and let $f$ and $\left\{ f_{N}\right\} _{N\geq0}$ be measurable, complex-valued functions on $X$ so that $f_{N}$ converges to $f$ almost everywhere. Then, for every $\epsilon>0$, there is a measurable set $E\subseteq X$ with $\mu\left(E\right)<\epsilon$ such that $f_{N}\rightarrow f$ uniformly on $X\backslash E$. \cite{Folland - real analysis}}, there is a measurable set $E\subseteq\mathbb{Z}_{p}$ of real-valued Haar measure $<\epsilon$ so that the $\left|f_{n}\right|_{q}$s converge uniformly to $0$ on $\mathbb{Z}_{p}\backslash E$. \begin{align*} \int_{\mathbb{Z}_{p}}\left|f_{n}\left(\mathfrak{z}\right)\right|_{q}d\mathfrak{z} & =\int_{E}\left|f_{n}\left(\mathfrak{z}\right)\right|_{q}d\mathfrak{z}+\int_{\mathbb{Z}_{p}\backslash E}\left|f_{n}\left(\mathfrak{z}\right)\right|_{q}d\mathfrak{z}\\ \left(\left|f_{n}\right|_{q}<C\textrm{ a.e.}\right); & \leq C\epsilon+\int_{\mathbb{Z}_{p}\backslash E}\left|f_{n}\left(\mathfrak{z}\right)\right|_{q}d\mathfrak{z} \end{align*} Since the $\left|f_{n}\right|_{q}$s converge uniformly to $0$ on $\mathbb{Z}_{p}\backslash E$, $\int_{\mathbb{Z}_{p}\backslash E}\left|f_{n}\left(\mathfrak{z}\right)\right|_{q}d\mathfrak{z}$ tends to $0$ in $\mathbb{R}$ as $n\rightarrow\infty$, and we are left with: \begin{equation} \limsup_{n\rightarrow\infty}\int_{\mathbb{Z}_{p}}\left|f_{n}\left(\mathfrak{z}\right)\right|_{q}d\mathfrak{z}\ll\epsilon \end{equation} Since $\epsilon$ was arbitrary, we conclude that: \begin{equation} \lim_{n\rightarrow\infty}\int_{\mathbb{Z}_{p}}\left|f_{n}\left(\mathfrak{z}\right)\right|_{q}d\mathfrak{z}=0 \end{equation} \vphantom{} II. This is a standard result of measure theory. See Chapter 2.4 in (\cite{Folland - real analysis}) for details. Q.E.D. \begin{thm} \label{thm:L^1_R is an archimedean Banach space}$L_{\mathbb{R}}^{1}\left(\mathbb{Z}_{p},\mathbb{C}_{q}\right)$ is an archimedean Banach space over $\mathbb{C}_{q}$. \end{thm} Proof: To show that the archimedean normed vector space $L_{\mathbb{R}}^{1}\left(\mathbb{Z}_{p},\mathbb{C}_{q}\right)$ is a Banach space, we use \textbf{Theorem 5.1 }from \cite{Folland - real analysis} (the archimedean analogue of \textbf{Proposition \ref{prop:series characterization of a Banach space}}): it suffices to show that every absolutely convergent series in $L_{\mathbb{R}}^{1}\left(\mathbb{Z}_{p},\mathbb{C}_{q}\right)$ converges to a limit in $L_{\mathbb{R}}^{1}\left(\mathbb{Z}_{p},\mathbb{C}_{q}\right)$. To do this, let $\left\{ f_{n}\right\} _{n\geq0}$ be a sequence in $L_{\mathbb{R}}^{1}\left(\mathbb{Z}_{p},\mathbb{C}_{q}\right)$ so that: \begin{equation} \sum_{n=0}^{\infty}\left\Vert f_{n}\right\Vert _{1}<\infty \end{equation} That is: \begin{equation} \lim_{N\rightarrow\infty}\sum_{n=0}^{N-1}\int_{\mathbb{Z}_{p}}\left|f_{n}\left(\mathfrak{z}\right)\right|_{q}d\mathfrak{z} \end{equation} exists in $\mathbb{R}$. Consequently: \begin{equation} \lim_{N\rightarrow\infty}\sum_{n=0}^{N-1}\int_{\mathbb{Z}_{p}}\left|f_{n}\left(\mathfrak{z}\right)\right|_{q}d\mathfrak{z}\overset{\mathbb{R}}{=}\int_{\mathbb{Z}_{p}}\lim_{N\rightarrow\infty}\sum_{n=0}^{N-1}\left|f_{n}\left(\mathfrak{z}\right)\right|_{q}d\mathfrak{z} \end{equation} and hence, the function defined by $\lim_{N\rightarrow\infty}\sum_{n=0}^{N-1}\left|f_{n}\left(\mathfrak{z}\right)\right|_{q}$ is in $L^{1}\left(\mathbb{Z}_{p},\mathbb{R}\right)$, and the point-wise limit exists $\sum_{n=0}^{\infty}\left|f_{n}\left(\mathfrak{z}\right)\right|_{q}$ for almost every $\mathfrak{z}\in\mathbb{Z}_{p}$. By the ordinary $q$-adic triangle inequality (not the ultrametric inequality, just the ordinary triangle inequality), this then shows that $\sum_{n=0}^{\infty}f_{n}\left(\mathfrak{z}\right)$ converges in $\mathbb{C}_{q}$ for almost every $\mathfrak{z}\in\mathbb{Z}_{p}$. As such, let $F:\mathbb{Z}_{p}\rightarrow\mathbb{C}_{q}$ be defined by: \begin{equation} F\left(\mathfrak{z}\right)\overset{\textrm{def}}{=}\begin{cases} \sum_{n=0}^{\infty}f_{n}\left(\mathfrak{z}\right) & \textrm{if \ensuremath{\sum_{n=0}^{\infty}f_{n}\left(\mathfrak{z}\right)} converges in }\mathbb{C}_{q}\\ 0 & \textrm{if \ensuremath{\sum_{n=0}^{\infty}f_{n}\left(\mathfrak{z}\right)} does not converge in }\mathbb{C}_{q} \end{cases},\textrm{ }\forall\mathfrak{z}\in\mathbb{Z}_{p} \end{equation} Then: \begin{equation} \left|F\left(\mathfrak{z}\right)-\sum_{n=0}^{N-1}f_{n}\left(\mathfrak{z}\right)\right|_{q}=\left|\sum_{n=N}^{\infty}f_{n}\left(\mathfrak{z}\right)\right|_{q} \end{equation} holds for almost every $\mathfrak{z}\in\mathbb{Z}_{p}$, and so: \begin{align*} \lim_{N\rightarrow\infty}\left\Vert F-\sum_{n=0}^{N-1}f_{n}\right\Vert _{1} & \overset{\mathbb{R}}{=}\lim_{N\rightarrow\infty}\int_{\mathbb{Z}_{p}}\left|F\left(\mathfrak{z}\right)-\sum_{n=0}^{N-1}f_{n}\left(\mathfrak{z}\right)\right|_{q}d\mathfrak{z}\\ & =\lim_{N\rightarrow\infty}\int_{\mathbb{Z}_{p}}\left|\sum_{n=N}^{\infty}f_{n}\left(\mathfrak{z}\right)\right|_{q}d\mathfrak{z}\\ & \leq\lim_{N\rightarrow\infty}\int_{\mathbb{Z}_{p}}\sum_{n=N}^{\infty}\left|f_{n}\left(\mathfrak{z}\right)\right|_{q}d\mathfrak{z}\\ & =\lim_{N\rightarrow\infty}\sum_{n=N}^{\infty}\left\Vert f_{n}\right\Vert _{1}\\ \left(\sum_{n=0}^{\infty}\left\Vert f_{n}\right\Vert _{1}<\infty\right); & =0 \end{align*} which shows that $F$ is the limit of the series $\sum_{n=0}^{N-1}f_{n}\left(\mathfrak{z}\right)$ in $L_{\mathbb{R}}^{1}\left(\mathbb{Z}_{p},\mathbb{C}_{q}\right)$. Thus, every absolutely convergent series in $L_{\mathbb{R}}^{1}\left(\mathbb{Z}_{p},\mathbb{C}_{q}\right)$ converges, which proves that $L_{\mathbb{R}}^{1}\left(\mathbb{Z}_{p},\mathbb{C}_{q}\right)$ is a Banach space. Q.E.D. \begin{prop} Let $\chi:\mathbb{Z}_{p}\rightarrow\mathbb{C}_{q}$ be a function so that: \begin{equation} \chi\left(\mathfrak{z}\right)\overset{\mathbb{C}_{q}}{=}\lim_{N\rightarrow\infty}\chi\left(\left[\mathfrak{z}\right]_{p^{N}}\right) \end{equation} holds for almost every $\mathfrak{z}\in\mathbb{Z}_{p}$. Then, $\chi\in L_{\mathbb{R}}^{1}\left(\mathbb{Z}_{p},\mathbb{C}_{q}\right)$ whenever: \begin{equation} \sum_{N=1}^{\infty}\frac{1}{p^{N}}\sum_{n=p^{N-1}}^{p^{N}-1}\left|c_{n}\left(\chi\right)\right|_{q}<\infty\label{eq:van der Put criterion for real L^1 integrability} \end{equation} \end{prop} Proof: Using the truncated van der Put identity (\ref{eq:truncated van der Put identity}), we have: \begin{align*} \chi_{N}\left(\mathfrak{z}\right)-\chi_{N-1}\left(\mathfrak{z}\right) & \overset{\mathbb{C}_{q}}{=}\chi\left(\left[\mathfrak{z}\right]_{p^{N}}\right)-\chi\left(\left[\mathfrak{z}\right]_{p^{N-1}}\right)\\ & =\sum_{n=p^{N-1}}^{p^{N}-1}c_{n}\left(\chi\right)\left[\mathfrak{z}\overset{p^{\lambda_{p}\left(n\right)}}{\equiv}n\right]\\ \left(\lambda_{p}\left(n\right)=N\Leftrightarrow p^{N-1}\leq n\leq p^{N}-1\right); & =\sum_{n=p^{N-1}}^{p^{N}-1}c_{n}\left(\chi\right)\left[\mathfrak{z}\overset{p^{N}}{\equiv}n\right] \end{align*} Now, fixing $N$, note that for any $\mathfrak{z}\in\mathbb{Z}_{p}$: \begin{equation} \sum_{n=p^{N-1}}^{p^{N}-1}c_{n}\left(\chi\right)\left[\mathfrak{z}\overset{p^{N}}{\equiv}n\right]\overset{\mathbb{R}}{=}\begin{cases} c_{\left[\mathfrak{z}\right]_{p^{N}}}\left(\chi\right) & \textrm{if }p^{N-1}\leq\left[\mathfrak{z}\right]_{p^{N}}\leq p^{N}-1\\ 0 & \textrm{else} \end{cases} \end{equation} Hence: \begin{align*} \left|\chi\left(\left[\mathfrak{z}\right]_{p^{N}}\right)-\chi\left(\left[\mathfrak{z}\right]_{p^{N-1}}\right)\right|_{q} & \overset{\mathbb{R}}{=}\left|c_{\left[\mathfrak{z}\right]_{p^{N}}}\left(\chi\right)\right|_{q}\left[p^{N-1}\leq\left[\mathfrak{z}\right]_{p^{N}}\leq p^{N}-1\right]\\ & =\left|c_{\left[\mathfrak{z}\right]_{p^{N}}}\left(\chi\right)\right|_{q}\sum_{n=p^{N-1}}^{p^{N}-1}\left[\mathfrak{z}\overset{p^{N}}{\equiv}n\right]\\ & =\sum_{n=p^{N-1}}^{p^{N}-1}\left|c_{n}\left(\chi\right)\right|_{q}\left[\mathfrak{z}\overset{p^{N}}{\equiv}n\right] \end{align*} Since the integral of $\left[\mathfrak{z}\overset{p^{N}}{\equiv}n\right]$ is $p^{-N}$, this yields: \begin{equation} \left\Vert \chi_{N}-\chi_{N-1}\right\Vert _{1}\overset{\mathbb{R}}{=}\frac{1}{p^{N}}\sum_{n=p^{N-1}}^{p^{N}-1}\left|c_{n}\left(\chi\right)\right|_{q}\label{eq:Real Integral of difference of Nth truncations of Chi} \end{equation} If the right-hand side is summable in $\mathbb{R}$ with respect to $N$, then: \begin{equation} \sum_{N=1}^{\infty}\left\Vert \chi_{N}-\chi_{N-1}\right\Vert _{1}<\infty \end{equation} and hence, the series: \[ \sum_{N=1}^{\infty}\left(\chi_{N}\left(\mathfrak{z}\right)-\chi_{N-1}\left(\mathfrak{z}\right)\right)\overset{\mathbb{C}_{q}}{=}\lim_{N\rightarrow\infty}\chi_{N}\left(\mathfrak{z}\right)-\chi\left(0\right)\overset{\mathbb{C}_{q}}{=}\lim_{N\rightarrow\infty}\chi\left(\left[\mathfrak{z}\right]_{p^{N}}\right)-\chi\left(0\right) \] converges in $L_{\mathbb{R}}^{1}\left(\mathbb{Z}_{p},\mathbb{C}_{q}\right)$. As such, since $\chi\left(\mathfrak{z}\right)\overset{\mathbb{C}_{q}}{=}\lim_{N\rightarrow\infty}\chi\left(\left[\mathfrak{z}\right]_{p^{N}}\right)$ almost everywhere, it then follows that $\chi\left(\mathfrak{z}\right)-\chi\left(0\right)$ is an element of $L_{\mathbb{R}}^{1}\left(\mathbb{Z}_{p},\mathbb{C}_{q}\right)$, which shows that $\chi\in L_{\mathbb{R}}^{1}\left(\mathbb{Z}_{p},\mathbb{C}_{q}\right)$, since the constant function $\chi\left(0\right)$ is in $L_{\mathbb{R}}^{1}\left(\mathbb{Z}_{p},\mathbb{C}_{q}\right)$. Q.E.D. \begin{prop} Let $d\mu\in C\left(\mathbb{Z}_{p},\mathbb{C}_{q}\right)^{\prime}$. Then: \begin{equation} \lim_{N\rightarrow\infty}\int_{\mathbb{Z}_{p}}\left|\tilde{\mu}_{N}\left(\mathfrak{z}\right)\right|_{q}d\mathfrak{z}\overset{\mathbb{R}}{=}0 \end{equation} is a necessary\emph{ }condition for all of the following: \vphantom{} I. For $d\mu$ to be the zero measure; \vphantom{} II. For $d\mu$ to be a degenerate rising-continuous measure; \vphantom{} III. For $d\mu$ to be $\mathcal{F}$-rising, where the non-archimedean domain of $\mathcal{F}$ is a set of full measure in $\mathbb{Z}_{p}$ (that is, its complement has measure zero in $\mathbb{Z}_{p}$). \end{prop} Proof: If $d\mu$ is the zero measure, then, by \textbf{Proposition \ref{prop:Criterion for zero measure in terms of partial sums of Fourier series}}, $\left\Vert \tilde{\mu}_{N}\right\Vert _{p,q}\rightarrow0$, and hence, the $\tilde{\mu}_{N}$s converge $q$-adically to $0$ as $N\rightarrow\infty$ uniformly over $\mathbb{Z}_{p}$. Consequently, they are uniformly bounded in $q$-adic absolute value. Moreover, since the $\tilde{\mu}_{N}$s are $\left(p,q\right)$-adically continuous, the functions $\mathfrak{z}\mapsto\left|\tilde{\mu}_{N}\left(\mathfrak{z}\right)\right|_{q}$ are continuous, and hence measurable. Thus, by \textbf{Lemma \ref{lem:Egorov lemma}}, $\lim_{N\rightarrow\infty}\int_{\mathbb{Z}_{p}}\left|\tilde{\mu}_{N}\left(\mathfrak{z}\right)\right|_{q}d\mathfrak{z}\overset{\mathbb{R}}{=}0$. So, this limit is necessary for $d\mu$ to be the zero measure. (II) is a more specific case of (III). (III) holds true because if the non-archimedean domain of $\mathcal{F}$ has full measure in $\mathbb{Z}_{p}$, then the continuous (and hence measurable) real-valued functions $\mathfrak{z}\mapsto\left|\tilde{\mu}_{N}\left(\mathfrak{z}\right)\right|_{q}$ converge to $0$ almost everywhere on $\mathbb{Z}_{p}$. Moreover, because $d\mu$ is a measure, we have that: \[ \left|\tilde{\mu}_{N}\left(\mathfrak{z}\right)\right|_{q}\leq\max_{\left|t\right|_{p}\leq p^{N}}\left|\hat{\mu}\left(t\right)\right|_{q}\leq\sup_{t\in\hat{\mathbb{Z}}_{p}}\left|\hat{\mu}\left(t\right)\right|_{q}<\infty,\textrm{ }\forall N\geq0 \] As such, we can apply\textbf{ Lemma \ref{lem:Egorov lemma}} to obtain $\lim_{N\rightarrow\infty}\int_{\mathbb{Z}_{p}}\left|\tilde{\mu}_{N}\left(\mathfrak{z}\right)\right|_{q}d\mathfrak{z}\overset{\mathbb{R}}{=}0$. Q.E.D. \vphantom{} I find it interesting that everything I would like to have true for quasi-integrable rising-continuous functions is easily verified for \emph{continuous }$\left(p,q\right)$-adic functions. To show this, we need the following formula:\index{van der Put!coefficients!Fourier coeffs.} \begin{prop} \label{prop:3.78}\ \begin{equation} \hat{f}_{N}\left(t\right)=\sum_{n=\frac{\left|t\right|_{p}}{p}}^{p^{N}-1}\frac{c_{n}\left(f\right)}{p^{\lambda_{p}\left(n\right)}}e^{-2\pi int},\textrm{ }\forall t\in\hat{\mathbb{Z}}_{p}\label{eq:Fourier transform of Nth truncation in terms of vdP coefficients} \end{equation} \end{prop} Proof: Identical to the computation in \textbf{Theorem \ref{thm:formula for Fourier series}}, but with the $N$th partial van der Put series of $f$ ($S_{p:N}\left\{ f\right\} $) used in place of the full van der Put series of $f$ ($S_{p}\left\{ f\right\} $). Q.E.D. \vphantom{}Before proceeding, we note that the above identity also holds for rising-continuous functions. \begin{prop} Let $\chi\in\tilde{C}\left(\mathbb{Z}_{p},\mathbb{C}_{q}\right)$. Then, the Fourier transform of the $N$th truncation of $\chi$ is given by: \end{prop} \begin{equation} \hat{\chi}_{N}\left(t\right)=\sum_{n=\frac{\left|t\right|_{p}}{p}}^{p^{N}-1}\frac{c_{n}\left(\chi\right)}{p^{\lambda_{p}\left(n\right)}}e^{-2\pi int},\textrm{ }\forall t\in\hat{\mathbb{Z}}_{p}\label{eq:Fourier transform of Nth truncation in terms of vdP coefficients for a rising continuous function} \end{equation} where the sum is defined to be $0$ whenever $\frac{\left|t\right|_{p}}{p}>p^{N}-1$ (i.e., whenever $\left|t\right|_{p}\geq p^{N+1}$). Note that this equality holds in $\overline{\mathbb{Q}}$ whenever $\chi\mid_{\mathbb{N}_{0}}$ takes values in $\overline{\mathbb{Q}}$. Proof: Since $\chi_{N}\in C\left(\mathbb{Z}_{p},\mathbb{C}_{q}\right)$ for all $N$, this immediately follows by \textbf{Proposition \ref{prop:3.78}}. Q.E.D. \vphantom{} Here are the aforementioned properties provable for continuous $\left(p,q\right)$-adic functions: \begin{lem} Let $f\in C\left(\mathbb{Z}_{p},\mathbb{C}_{q}\right)$. Recall that $f_{N}$ is the $N$th truncation of $f$, $\hat{f}_{N}$ is the Fourier transform of $f_{N}$, $\hat{f}$ is the Fourier transform of $f$, and write: \begin{equation} \tilde{f}_{N}\left(\mathfrak{z}\right)\overset{\textrm{def}}{=}\sum_{\left|t\right|_{p}\leq p^{N}}\hat{f}\left(t\right)e^{2\pi i\left\{ t\mathfrak{z}\right\} _{p}}\label{eq:Definition of f_N twiddle} \end{equation} Then: \vphantom{} I. \begin{equation} \hat{f}\left(t\right)-\hat{f}_{N}\left(t\right)=\sum_{n=p^{N}}^{\infty}\frac{c_{n}\left(f\right)}{p^{\lambda_{p}\left(n\right)}}e^{-2\pi int},\textrm{ }\forall t\in\hat{\mathbb{Z}}_{p}\label{eq:Difference between f-hat and f_N-hat} \end{equation} with: \begin{equation} \left\Vert \hat{f}-\hat{f}_{N}\right\Vert _{p,q}\leq\sup_{n\geq p^{N}}\left|c_{n}\left(f\right)\right|_{q},\textrm{ }\forall N\geq0\label{eq:Infinit norm of f-hat - f_N-hat} \end{equation} \vphantom{} II. \begin{equation} \tilde{f}_{N}\left(\mathfrak{z}\right)-f_{N}\left(\mathfrak{z}\right)=p^{N}\sum_{n=p^{N}}^{\infty}\frac{c_{n}\left(f\right)}{p^{\lambda_{p}\left(n\right)}}\left[\mathfrak{z}\overset{p^{N}}{\equiv}n\right],\textrm{ }\forall\mathfrak{z}\in\mathbb{Z}_{p},\textrm{ }\forall N\geq0\label{eq:Difference between f_N-twiddle and f_N} \end{equation} with: \begin{equation} \left\Vert \tilde{f}_{N}-f_{N}\right\Vert _{p,q}\leq\sup_{n\geq p^{N}}\left|c_{n}\left(f\right)\right|_{q}\label{eq:Infinity norm of difference between f_N twiddle and f_N} \end{equation} and: \begin{equation} \int_{\mathbb{Z}_{p}}\left|\tilde{f}_{N}\left(\mathfrak{z}\right)-f_{N}\left(\mathfrak{z}\right)\right|_{q}d\mathfrak{z}\leq\sup_{n\geq p^{N}}\left|c_{n}\left(f\right)\right|_{q}\label{eq:L^1 norm of the difference between f_N-twiddle and f_N} \end{equation} \vphantom{} III. \begin{equation} \left\Vert f-\tilde{f}_{N}\right\Vert _{p,q}\leq\sup_{\left|t\right|_{p}>p^{N}}\left|\hat{f}\left(t\right)\right|_{q}\leq\sup_{n\geq p^{N}}\left|c_{n}\left(f\right)\right|_{q}\label{eq:Supremum norm of difference between f and f_N twiddle} \end{equation} and: \begin{equation} \int_{\mathbb{Z}_{p}}\left|f\left(\mathfrak{z}\right)-\tilde{f}_{N}\left(\mathfrak{z}\right)\right|_{q}d\mathfrak{z}\leq\sup_{\left|t\right|_{p}>p^{N}}\left|\hat{f}\left(t\right)\right|_{q}\leq\sup_{n\geq p^{N}}\left|c_{n}\left(f\right)\right|_{q}\label{eq:L^1 norm of difference between f and f_N twiddle} \end{equation} \end{lem} Proof: Since the Banach space $C\left(\mathbb{Z}_{p},\mathbb{C}_{q}\right)$ is isometrically isomorphic to $c_{0}\left(\hat{\mathbb{Z}}_{p},\mathbb{C}_{q}\right)$ by way of the $\left(p,q\right)$-adic Fourier transform (\ref{eq:Fourier transform of Nth truncation in terms of vdP coefficients}), we have that: \begin{align*} \hat{f}\left(t\right)-\hat{f}_{N}\left(t\right) & \overset{\mathbb{C}_{q}}{=}\sum_{n=\frac{\left|t\right|_{p}}{p}}^{\infty}\frac{c_{n}\left(f\right)}{p^{\lambda_{p}\left(n\right)}}e^{-2\pi int}-\sum_{n=\frac{\left|t\right|_{p}}{p}}^{p^{N}-1}\frac{c_{n}\left(f\right)}{p^{\lambda_{p}\left(n\right)}}e^{-2\pi int}\\ & \overset{\mathbb{C}_{q}}{=}\sum_{n=p^{N}}^{\infty}\frac{c_{n}\left(f\right)}{p^{\lambda_{p}\left(n\right)}}e^{-2\pi int} \end{align*} This proves (\ref{eq:Difference between f-hat and f_N-hat}). Taking $\left(p,q\right)$-adic supremum norm over $\hat{\mathbb{Z}}_{p}$ yields (\ref{eq:Infinit norm of f-hat - f_N-hat}). Next, performing Fourier inversion, we have: \begin{align*} \tilde{f}_{N}\left(\mathfrak{z}\right)-f_{N}\left(\mathfrak{z}\right) & =\sum_{\left|t\right|_{p}\leq p^{N}}\left(\hat{f}\left(t\right)-\hat{f}_{N}\left(t\right)\right)e^{2\pi i\left\{ t\mathfrak{z}\right\} _{p}}\\ & =\sum_{\left|t\right|_{p}\leq p^{N}}\sum_{n=p^{N}}^{\infty}\frac{c_{n}\left(f\right)}{p^{\lambda_{p}\left(n\right)}}e^{-2\pi int}e^{2\pi i\left\{ t\mathfrak{z}\right\} _{p}}\\ & =p^{N}\sum_{n=p^{N}}^{\infty}\frac{c_{n}\left(f\right)}{p^{\lambda_{p}\left(n\right)}}\left[\mathfrak{z}\overset{p^{N}}{\equiv}n\right] \end{align*} Hence: \[ \tilde{f}_{N}\left(\mathfrak{z}\right)-f_{N}\left(\mathfrak{z}\right)=p^{N}\sum_{n=p^{N}}^{\infty}\frac{c_{n}\left(f\right)}{p^{\lambda_{p}\left(n\right)}}\left[\mathfrak{z}\overset{p^{N}}{\equiv}n\right] \] which proves (\ref{eq:Difference between f_N-twiddle and f_N}). Taking $\left(p,q\right)$-adic supremum norm over $\mathbb{Z}_{p}$ yields (\ref{eq:Infinity norm of difference between f_N twiddle and f_N}). On the other hand, taking $q$-adic absolute values, integrating gives us: \begin{align*} \int_{\mathbb{Z}_{p}}\left|\tilde{f}_{N}\left(\mathfrak{z}\right)-f_{N}\left(\mathfrak{z}\right)\right|_{q}d\mathfrak{z} & =\int_{\mathbb{Z}_{p}}\left|p^{N}\sum_{n=p^{N}}^{\infty}\frac{c_{n}\left(f\right)}{p^{\lambda_{p}\left(n\right)}}\left[\mathfrak{z}\overset{p^{N}}{\equiv}n\right]\right|_{q}d\mathfrak{z}\\ & \leq\int_{\mathbb{Z}_{p}}\sup_{n\geq p^{N}}\left|\frac{c_{n}\left(f\right)}{p^{\lambda_{p}\left(n\right)}}\left[\mathfrak{z}\overset{p^{N}}{\equiv}n\right]\right|_{q}d\mathfrak{z}\\ & \leq\int_{\mathbb{Z}_{p}}\sup_{n\geq p^{N}}\left(\left|c_{n}\left(f\right)\right|_{q}\left[\mathfrak{z}\overset{p^{N}}{\equiv}n\right]\right)d\mathfrak{z}\\ & \leq\left(\sup_{n\geq p^{N}}\left|c_{n}\left(f\right)\right|_{q}\right)\int_{\mathbb{Z}_{p}}d\mathfrak{z}\\ & =\sup_{n\geq p^{N}}\left|c_{n}\left(f\right)\right|_{q} \end{align*} which proves (\ref{eq:L^1 norm of the difference between f_N-twiddle and f_N}). Lastly: \begin{align*} f\left(\mathfrak{z}\right)-\tilde{f}_{N}\left(\mathfrak{z}\right) & =\sum_{t\in\hat{\mathbb{Z}}_{p}}\hat{f}\left(t\right)e^{2\pi i\left\{ t\mathfrak{z}\right\} _{p}}-\sum_{\left|t\right|_{p}\leq p^{N}}\hat{f}\left(t\right)e^{2\pi i\left\{ t\mathfrak{z}\right\} _{p}}\\ & =\sum_{\left|t\right|_{p}>p^{N}}\hat{f}\left(t\right)e^{2\pi i\left\{ t\mathfrak{z}\right\} _{p}} \end{align*} and so: \begin{align*} \left\Vert f-\tilde{f}_{N}\right\Vert _{p,q} & \leq\sup_{\left|t\right|_{p}>p^{N}}\left|\hat{f}\left(t\right)\right|_{q}\\ & \leq\sup_{\left|t\right|_{p}>p^{N}}\left|\sum_{n=\frac{\left|t\right|_{p}}{p}}^{\infty}\frac{c_{n}\left(f\right)}{p^{\lambda_{p}\left(n\right)}}e^{-2\pi int}\right|_{q}\\ & \leq\sup_{\left|t\right|_{p}>p^{N}}\sup_{n\geq\frac{\left|t\right|_{p}}{p}}\left|c_{n}\left(f\right)\right|_{q}\\ & \leq\sup_{\left|t\right|_{p}>p^{N}}\sup_{n\geq\frac{p^{N+1}}{p}}\left|c_{n}\left(f\right)\right|_{q}\\ & =\sup_{n\geq p^{N}}\left|c_{n}\left(f\right)\right|_{q} \end{align*} This proves (\ref{eq:Supremum norm of difference between f and f_N twiddle}). If we instead integrate, we get: \begin{align*} \int_{\mathbb{Z}_{p}}\left|f\left(\mathfrak{z}\right)-\tilde{f}_{N}\left(\mathfrak{z}\right)\right|_{q}d\mathfrak{z} & =\int_{\mathbb{Z}_{p}}\left|\sum_{\left|t\right|_{p}>p^{N}}\hat{f}\left(t\right)e^{2\pi i\left\{ t\mathfrak{z}\right\} _{p}}\right|_{q}d\mathfrak{z}\\ & \leq\int_{\mathbb{Z}_{p}}\left(\sup_{\left|t\right|_{p}>p^{N}}\left|\hat{f}\left(t\right)\right|_{q}\right)d\mathfrak{z}\\ & =\sup_{\left|t\right|_{p}>p^{N}}\left|\hat{f}\left(t\right)\right|_{q}\\ & \leq\sup_{n\geq p^{N}}\left|c_{n}\left(f\right)\right|_{q} \end{align*} Q.E.D. \vphantom{} The last result of this subsection is a sufficient condition for a rising-continuous function to be an element of $L_{\mathbb{R}}^{1}\left(\mathbb{Z}_{p},\mathbb{C}_{q}\right)$, formulated as a decay condition on the $q$-adic absolute value of the function's van der Put coefficient. We need two propositions first. \begin{prop} Let $\chi:\mathbb{Z}_{p}\rightarrow\mathbb{C}_{q}$ be any function. Then, for integers $N,k\geq1$ the difference between the $\left(N+k\right)$th and $N$th truncations of $\chi$ is: \begin{align} \chi_{N+k}\left(\mathfrak{z}\right)-\chi_{N}\left(\mathfrak{z}\right) & =\sum_{j=1}^{p^{k}-1}\sum_{n=0}^{p^{N}-1}\left(\chi\left(n+jp^{N}\right)-\chi\left(n\right)\right)\left[\mathfrak{z}\overset{p^{N+k}}{\equiv}n+jp^{N}\right]\label{eq:Difference between N+kth and Nth truncations of Chi} \end{align} \end{prop} Proof: Note that: \begin{equation} \left[\mathfrak{z}\overset{p^{N}}{\equiv}n\right]=\sum_{j=0}^{p^{k}-1}\left[\mathfrak{z}\overset{p^{N+k}}{\equiv}n+p^{N}j\right],\textrm{ }\forall N,k\geq0,\textrm{ }\forall n\in\left\{ 0,\ldots,p^{N}-1\right\} ,\textrm{ }\forall\mathfrak{z}\in\mathbb{Z}_{p} \end{equation} Hence: \begin{align*} \chi_{N+k}\left(\mathfrak{z}\right)-\chi_{N}\left(\mathfrak{z}\right) & =\sum_{n=0}^{p^{N+k}-1}\chi\left(n\right)\left[\mathfrak{z}\overset{p^{N+k}}{\equiv}n\right]-\sum_{n=0}^{p^{N}-1}\chi\left(n\right)\left[\mathfrak{z}\overset{p^{N}}{\equiv}n\right]\\ & =\sum_{n=0}^{p^{N+k}-1}\chi\left(n\right)\left[\mathfrak{z}\overset{p^{N+k}}{\equiv}n\right]-\sum_{n=0}^{p^{N}-1}\chi\left(n\right)\sum_{j=0}^{p^{k}-1}\left[\mathfrak{z}\overset{p^{N+k}}{\equiv}n+p^{N}j\right]\\ & =\sum_{n=0}^{p^{N}-1}\chi\left(n\right)\underbrace{\left(\left[\mathfrak{z}\overset{p^{N+k}}{\equiv}n\right]-\sum_{j=0}^{p^{k}-1}\left[\mathfrak{z}\overset{p^{N+k}}{\equiv}n+p^{N}j\right]\right)}_{j=0\textrm{ term cancels}}\\ & +\sum_{n=p^{N}}^{p^{N+k}-1}\chi\left(n\right)\left[\mathfrak{z}\overset{p^{N+k}}{\equiv}n\right]\\ & =\sum_{n=p^{N}}^{p^{N+k}-1}\chi\left(n\right)\left[\mathfrak{z}\overset{p^{N+k}}{\equiv}n\right]-\sum_{n=0}^{p^{N}-1}\chi\left(n\right)\sum_{j=1}^{p^{k}-1}\left[\mathfrak{z}\overset{p^{N+k}}{\equiv}n+p^{N}j\right] \end{align*} Here: \begin{align*} \sum_{n=p^{N}}^{p^{N+k}-1}\chi\left(n\right)\left[\mathfrak{z}\overset{p^{N+k}}{\equiv}n\right] & =\sum_{n=0}^{\left(p^{k}-1\right)p^{N}-1}\chi\left(n+p^{N}\right)\left[\mathfrak{z}\overset{p^{N+k}}{\equiv}n+p^{N}\right]\\ & =\sum_{j=1}^{p^{k}-1}\sum_{n=\left(j-1\right)p^{N}}^{jp^{N}-1}\chi\left(n+p^{N}\right)\left[\mathfrak{z}\overset{p^{N+k}}{\equiv}n+p^{N}\right]\\ & =\sum_{j=1}^{p^{k}-1}\sum_{n=0}^{p^{N}-1}\chi\left(n+jp^{N}\right)\left[\mathfrak{z}\overset{p^{N+k}}{\equiv}n+jp^{N}\right] \end{align*} and so: \begin{align*} \chi_{N+k}\left(\mathfrak{z}\right)-\chi_{N}\left(\mathfrak{z}\right) & =\sum_{n=p^{N}}^{p^{N+k}-1}\chi\left(n\right)\left[\mathfrak{z}\overset{p^{N+k}}{\equiv}n\right]-\sum_{n=0}^{p^{N}-1}\chi\left(n\right)\sum_{j=1}^{p^{k}-1}\left[\mathfrak{z}\overset{p^{N+k}}{\equiv}n+jp^{N}\right]\\ & =\sum_{j=1}^{p^{k}-1}\sum_{n=0}^{p^{N}-1}\left(\chi\left(n+jp^{N}\right)-\chi\left(n\right)\right)\left[\mathfrak{z}\overset{p^{N+k}}{\equiv}n+jp^{N}\right] \end{align*} Q.E.D. \begin{prop} Let $\chi:\mathbb{Z}_{p}\rightarrow\mathbb{C}_{q}$ be any function. Then, for all integers $N,k\geq1$: \begin{equation} \int_{\mathbb{Z}_{p}}\left|\chi_{N+k}\left(\mathfrak{z}\right)-\chi_{N}\left(\mathfrak{z}\right)\right|_{q}d\mathfrak{z}=\sum_{j=1}^{p^{k}-1}\sum_{n=0}^{p^{N}-1}\frac{\left|\chi\left(n+jp^{N}\right)-\chi\left(n\right)\right|_{q}}{p^{N+k}}\label{eq:L^1_R norm of Chi_N+k minus Chi_N} \end{equation} \end{prop} Proof: Let $N$ and $k$ be arbitrary. Note that the brackets in (\ref{eq:Difference between N+kth and Nth truncations of Chi}) have supports which are pair-wise disjoint with respect to $n$ and $j$, and that each of the brackets has Haar measure $1/p^{N+k}$. As such: \[ \int_{\mathbb{Z}_{p}}\left|\chi_{N+k}\left(\mathfrak{z}\right)-\chi_{N}\left(\mathfrak{z}\right)\right|_{q}d\mathfrak{z}=\sum_{j=1}^{p^{k}-1}\sum_{n=0}^{p^{N}-1}\frac{\left|\chi\left(n+jp^{N}\right)-\chi\left(n\right)\right|_{q}}{p^{N+k}} \] Q.E.D. \vphantom{} Here, then, is the $L_{\mathbb{R}}^{1}$ criterion: \begin{lem} Let\index{van der Put!coefficients!L_{mathbb{R}}^{1} criterion@$L_{\mathbb{R}}^{1}$ criterion} $\chi\in\tilde{C}\left(\mathbb{Z}_{p},\mathbb{C}_{q}\right)$. If there exists an $r\in\left(0,1\right)$ so that for all sufficiently large integers $N\geq0$: \begin{equation} \max_{0\leq n<p^{N}}\left|c_{n+jp^{N}}\left(\chi\right)\right|_{q}\ll r^{j},\textrm{ }\forall j\geq1\label{eq:van der Put coefficient asymptotic condition for L^1_R integrability} \end{equation} Then $\chi\in L_{\mathbb{R}}^{1}\left(\mathbb{Z}_{p},\mathbb{C}_{q}\right)$. \end{lem} Proof: Letting $\chi$ and $r$ be as given, note that: \begin{equation} c_{n+jp^{N}}\left(\chi\right)=\chi\left(n+jp^{N}\right)-\chi\left(n\right),\textrm{ }\forall n<p^{N},\textrm{ }\forall j\geq1 \end{equation} By (\ref{eq:L^1_R norm of Chi_N+k minus Chi_N}), we have that: \begin{align*} \int_{\mathbb{Z}_{p}}\left|\chi_{N+k}\left(\mathfrak{z}\right)-\chi_{N}\left(\mathfrak{z}\right)\right|_{q}d\mathfrak{z} & \ll\sum_{n=0}^{p^{N}-1}\sum_{j=1}^{p^{k}-1}\frac{r^{j}}{p^{N+k}}\\ & =\sum_{n=0}^{p^{N}-1}\frac{1}{p^{N+k}}\left(-1+\frac{1-r^{p^{k}}}{1-r}\right)\\ & =\frac{1}{p^{k}}\frac{r-r^{p^{k}}}{1-r} \end{align*} which tends to $0$ in $\mathbb{R}$ as $k\rightarrow\infty$. Since $k$ was arbitrary, it follows that for any $\epsilon>0$, choosing $N$ and $M=N+k$ sufficiently large, we can guarantee that: \begin{equation} \int_{\mathbb{Z}_{p}}\left|\chi_{M}\left(\mathfrak{z}\right)-\chi_{N}\left(\mathfrak{z}\right)\right|_{q}d\mathfrak{z}<\epsilon \end{equation} which shows that the $\chi_{N}$s are Cauchy in $L_{\mathbb{R}}^{1}\left(\mathbb{Z}_{p},\mathbb{C}_{q}\right)$ norm. Since $L_{\mathbb{R}}^{1}\left(\mathbb{Z}_{p},\mathbb{C}_{q}\right)$ is complete, and since, as a rising-continuous function, the $\chi_{N}$s converge point-wise to $\chi$, it follows that $\chi$ is then an element of $L_{\mathbb{R}}^{1}\left(\mathbb{Z}_{p},\mathbb{C}_{q}\right)$. Q.E.D. \vphantom{} We end this subsection with two questions: \begin{question} \label{que:3.7}Is every function in $\tilde{C}\left(\mathbb{Z}_{p},\mathbb{C}_{q}\right)\cap L_{\mathbb{R}}^{1}\left(\mathbb{Z}_{p},\mathbb{C}_{q}\right)$ and element of $\tilde{L}^{1}\left(\mathcal{F}_{p,q}\right)$? \end{question} \begin{question} Let $\mathcal{F}$ be a $p$-aduc frame. Is $\tilde{C}\left(\mathbb{Z}_{p},\mathbb{C}_{q}\right)\cap\tilde{L}^{1}\left(\mathcal{F}\right)\subseteq L_{\mathbb{R}}^{1}\left(\mathbb{Z}_{p},\mathbb{C}_{q}\right)$? Is $\tilde{L}^{1}\left(\mathcal{F}\right)\subseteq L_{\mathbb{R}}^{1}\left(\mathbb{Z}_{p},\mathbb{C}_{q}\right)$? \end{question} \subsection{\label{subsec:3.3.7 -adic-Wiener-Tauberian}$\left(p,q\right)$-adic Wiener Tauberian Theorems} One of the great mathematical accomplishments of the twentieth century was the transfiguration of the study of divergent series from the realm of foolish fancy to one of serious, rigorous study. Asymptotic analysis\textemdash be it Karamata's theory of functions of regular variation\index{regular variation}\footnote{Even though it has little to do with the topic at hand, I cannot give too strong of a recommendation of Bingham, Goldie, and Teugels' book \emph{Regular Variation} \cite{Regular Variation}. It should be required reading for anyone working in analysis. It is filled with a thousand and one useful beauties that you never knew you wanted to know.}, or Abelian or Tauberian summability theorems\textemdash are powerful and widely useful in number theory, probability theory, and nearly everything in between. The \index{Wiener!Tauberian Theorem}Wiener Tauberian Theorem (WTT) is a capstone of these investigations. This theorem has a protean diversity of forms and restatements, all of which are interesting in their own particular way. Jacob Korevaar\index{Korevaar, Jacob}'s book on Tauberian theory gives an excellent exposition of most of them \cite{Korevaar}. The version of the WTT that matters to us sits at the boarder between Fourier analysis and Functional analysis, where it characterizes the relationship between a $1$-periodic function $\phi:\mathbb{R}/\mathbb{Z}\rightarrow\mathbb{C}$ (where $\mathbb{R}/\mathbb{Z}$ is identified with $\left[0,1\right)$ with addition modulo $1$), its Fourier coefficients $\hat{\phi}\left(n\right)$, and its reciprocal $1/\phi$. \begin{defn} The \index{Wiener!algebra}\textbf{Wiener algebra (on the circle)}, denote $A\left(\mathbb{T}\right)$, is the $\mathbb{C}$-linear space of all absolutely convergent Fourier series: \begin{equation} A\left(\mathbb{T}\right)\overset{\textrm{def}}{=}\left\{ \sum_{n\in\mathbb{Z}}\hat{\phi}\left(n\right)e^{2\pi int}:\hat{\phi}\in\ell^{1}\left(\mathbb{Z},\mathbb{C}\right)\right\} \label{eq:Definition of the Wiener algebra} \end{equation} where: \begin{equation} \ell^{1}\left(\mathbb{Z},\mathbb{C}\right)\overset{\textrm{def}}{=}\left\{ \hat{\phi}:\mathbb{Z}\rightarrow\mathbb{C}:\sum_{n\in\mathbb{Z}}\left|\hat{\phi}\left(n\right)\right|<\infty\right\} \label{eq:Definition of ell 1 of Z, C} \end{equation} $A\left(\mathbb{T}\right)$ is then made into a Banach space with the norm: \begin{equation} \left\Vert \phi\right\Vert _{A}\overset{\textrm{def}}{=}\left\Vert \hat{\phi}\right\Vert _{1}\overset{\textrm{def}}{=}\sum_{n\in\mathbb{Z}}\left|\hat{\phi}\left(n\right)\right|,\textrm{ }\forall\phi\in A\left(\mathbb{T}\right)\label{eq:Definition of Wiener algebra norm} \end{equation} \end{defn} \vphantom{} As defined, observe that $A\left(\mathbb{T}\right)$ is then isometrically isomorphic to the Banach space $\ell^{1}\left(\mathbb{Z},\mathbb{C}\right)$. Because $\left(\mathbb{Z},+\right)$ is a locally compact abelian group, $\ell^{1}\left(\mathbb{Z},\mathbb{C}\right)$ can be made into a Banach algebra (the \textbf{group algebra}\index{group algebra}\textbf{ }of $\mathbb{Z}$) by equipping it with the convolution operation: \begin{equation} \left(\hat{\phi}*\hat{\psi}\right)\left(n\right)\overset{\textrm{def}}{=}\sum_{m\in\mathbb{Z}}\hat{\phi}\left(n-m\right)\hat{\psi}\left(m\right)\label{eq:ell 1 convolution} \end{equation} In this way, $A\left(\mathbb{T}\right)$ becomes a Banach algebra under point-wise multiplication, thanks to \textbf{Young's Convolution Inequality} and the fact that the Fourier transform turns multiplication into convolution: \begin{equation} \left\Vert \phi\cdot\psi\right\Vert _{A}=\left\Vert \widehat{\phi\cdot\psi}\right\Vert _{1}=\left\Vert \hat{\phi}*\hat{\psi}\right\Vert _{1}\overset{\textrm{Young}}{\leq}\left\Vert \hat{\phi}\right\Vert _{1}\left\Vert \hat{\psi}\right\Vert _{1}=\left\Vert \phi\right\Vert _{A}\left\Vert \psi\right\Vert _{A} \end{equation} With this terminology, we can state three versions of the WTT. \begin{thm}[\textbf{Wiener's Tauberian Theorem, Ver. 1}] Let $\phi\in A\left(\mathbb{T}\right)$. Then, $1/\phi\in A\left(\mathbb{T}\right)$ if and only if\footnote{That is to say, the units of the Wiener algebra are precisely those functions $\phi\in A\left(\mathbb{T}\right)$ which have no zeroes.} $\phi\left(t\right)\neq0$ for all $t\in\mathbb{R}/\mathbb{Z}$. \end{thm} At first glance, this statement might seem almost tautological, until you realize that we are requiring more than just the well-definedness of $1/\phi$: we are also demanding it to have an absolutely convergent Fourier series. Because the Fourier transform turns multiplication into convolution, the existence of a multiplicative inverse for $\phi\in A\left(\mathbb{T}\right)$ is equivalent to the existence of a \index{convolution!inverse}\textbf{convolution inverse}\emph{ }for $\hat{\phi}\in\ell^{1}\left(\mathbb{Z},\mathbb{C}\right)$, by which I mean a function $\hat{\phi}^{-1}\in\ell^{1}\left(\mathbb{Z},\mathbb{C}\right)$ so that: \begin{equation} \left(\hat{\phi}*\hat{\phi}^{-1}\right)\left(n\right)=\left(\hat{\phi}^{-1}*\hat{\phi}\right)\left(n\right)=\mathbf{1}_{0}\left(n\right)=\begin{cases} 1 & \textrm{if }n=0\\ 0 & \textrm{else} \end{cases}\label{eq:Definition of a convolution inverse} \end{equation} because the function $\mathbf{1}_{0}\left(n\right)$ which is $1$ at $n=0$ and $0$ for all other $n$ is identity element of the convolution operation on $\ell^{1}\left(\mathbb{Z},\mathbb{C}\right)$. This gives us a second version of the WTT. \begin{thm}[\textbf{Wiener's Tauberian Theorem, Ver. 2}] Let $\hat{\phi}\in\ell^{1}\left(\mathbb{Z},\mathbb{C}\right)$. Then $\hat{\phi}$ has a convolution inverse $\hat{\phi}^{-1}\in\ell^{1}\left(\mathbb{Z},\mathbb{C}\right)$ if and only if $\phi\left(t\right)\neq0$ for all $t\in\mathbb{R}/\mathbb{Z}$. \end{thm} \vphantom{} To get a more analytic statement of the theorem, observe that the existence of a convolution inverse $\hat{\phi}^{-1}\in\ell^{1}\left(\mathbb{Z},\mathbb{C}\right)$ for $\hat{\phi}\in\ell^{1}\left(\mathbb{Z},\mathbb{C}\right)$ means that for any $\hat{\psi}\in\ell^{1}\left(\mathbb{Z},\mathbb{C}\right)$, we can convolve $\hat{\phi}$ with a function in $\ell^{1}\left(\mathbb{Z},\mathbb{C}\right)$ to obtain $\hat{\psi}$: \begin{equation} \hat{\phi}*\left(\hat{\phi}^{-1}*\hat{\psi}\right)=\hat{\psi} \end{equation} Writing $\hat{\chi}$ to denote $\hat{\phi}^{-1}*\hat{\psi}$, observe the limit: \begin{equation} \left(\hat{\phi}*\hat{\chi}\right)\left(n\right)\overset{\mathbb{C}}{=}\lim_{M\rightarrow\infty}\sum_{\left|m\right|\leq M}\hat{\chi}\left(m\right)\hat{\phi}\left(n-m\right) \end{equation} where, the right-hand side is a linear combination of translates of $\hat{\phi}$. In fact, the convergence is in $\ell^{1}$-norm: \begin{equation} \lim_{M\rightarrow\infty}\sum_{n\in\mathbb{Z}}\left|\hat{\psi}\left(n\right)-\sum_{\left|m\right|\leq M}\hat{\chi}\left(m\right)\hat{\phi}\left(n-m\right)\right|\overset{\mathbb{C}}{=}0 \end{equation} This gives us a third version of the WTT: \begin{thm}[\textbf{Wiener's Tauberian Theorem, Ver. 3}] Let $\phi\in A\left(\mathbb{T}\right)$. Then, the translates of $\hat{\phi}$ are dense in $\ell^{1}\left(\mathbb{Z},\mathbb{C}\right)$ if and only if $\phi\left(t\right)\neq0$ for any $t\in\mathbb{R}/\mathbb{Z}$. \end{thm} \vphantom{} Because the \textbf{Correspondence Principle }tells us that periodic points of a Hydra map $H$ are precisely those $x\in\mathbb{Z}$ which lie in the image of $\chi_{H}$, a $\left(p,q\right)$-adic analogue of WTT will allow us to study the multiplicative inverse of $\mathfrak{z}\mapsto\chi_{H}\left(\mathfrak{z}\right)-x$ (for any fixed $x$) by using Fourier transforms of $\chi_{H}$. And while this hope is eventually borne out, it does not come to pass without incident. In this subsection, I establish two realizations of a $\left(p,q\right)$-adic WTT: one for continuous functions; another for measures $d\mu$. First, however, a useful notation for translates: \begin{defn} For $s\in\hat{\mathbb{Z}}_{p}$, $\mathfrak{a}\in\mathbb{Z}_{p}$, $f:\mathbb{Z}_{p}\rightarrow\mathbb{C}_{q}$, and $\hat{f}:\mathbb{Z}_{p}\rightarrow\mathbb{C}_{q}$, we write: \begin{equation} \tau_{s}\left\{ \hat{f}\right\} \left(t\right)\overset{\textrm{def}}{=}\hat{f}\left(t+s\right)\label{eq:definition of the translate of f hat} \end{equation} and: \begin{equation} \tau_{\mathfrak{a}}\left\{ f\right\} \left(\mathfrak{z}\right)\overset{\textrm{def}}{=}f\left(\mathfrak{z}+\mathfrak{a}\right)\label{eq:Definition of the translate of f} \end{equation} \end{defn} \begin{thm}[\textbf{Wiener Tauberian Theorem for Continuous $\left(p,q\right)$-adic Functions}] \index{Wiener!Tauberian Theorem!left(p,qright)-adic@$\left(p,q\right)$-adic}\label{thm:pq WTT for continuous functions}Let $f\in C\left(\mathbb{Z}_{p},\mathbb{C}_{q}\right)$. Then, the following are equivalent: \vphantom{} I. $\frac{1}{f}\in C\left(\mathbb{Z}_{p},\mathbb{C}_{q}\right)$; \vphantom{} II. $\hat{f}$ has a convolution inverse in $c_{0}\left(\hat{\mathbb{Z}}_{p},\mathbb{C}_{q}\right)$; \vphantom{} III. $\textrm{span}_{\mathbb{C}_{q}}\left\{ \tau_{s}\left\{ \hat{f}\right\} \left(t\right):s\in\hat{\mathbb{Z}}_{p}\right\} $ is dense in $c_{0}\left(\hat{\mathbb{Z}}_{p},\mathbb{C}_{q}\right)$; \vphantom{} IV. $f$ has no zeroes. \end{thm} Proof: \textbullet{} ($\textrm{I}\Rightarrow\textrm{II}$) Suppose $\frac{1}{f}$ is continuous. Then, since the $\left(p,q\right)$-adic Fourier transform is an isometric isomorphism of the Banach algebra $C\left(\mathbb{Z}_{p},\mathbb{C}_{q}\right)$ onto the Banach algebra $c_{0}\left(\hat{\mathbb{Z}}_{p},\mathbb{C}_{q}\right)$, it follows that: \begin{align*} f\left(\mathfrak{z}\right)\cdot\frac{1}{f\left(\mathfrak{z}\right)} & =1,\textrm{ }\forall\mathfrak{z}\in\mathbb{Z}_{p}\\ \left(\textrm{Fourier transform}\right); & \Updownarrow\\ \left(\hat{f}*\widehat{\left(\frac{1}{f}\right)}\right)\left(t\right) & =\mathbf{1}_{0}\left(t\right),\textrm{ }\forall t\in\hat{\mathbb{Z}}_{p} \end{align*} where both $\hat{f}$ and $\widehat{\left(1/f\right)}$ are in $c_{0}$. $\hat{f}$ has a convolution inverse in $c_{0}$, and this inverse is $\widehat{\left(1/f\right)}$. \vphantom{} \textbullet{} ($\textrm{II}\Rightarrow\textrm{III}$) Suppose $\hat{f}$ has a convolution inverse $\hat{f}^{-1}\in c_{0}\left(\hat{\mathbb{Z}}_{p},\mathbb{C}_{q}\right)$. Then, letting $\hat{g}\in c_{0}\left(\hat{\mathbb{Z}}_{p},\mathbb{C}_{q}\right)$ be arbitrary, we have that: \begin{equation} \left(\hat{f}*\left(\hat{f}^{-1}*\hat{g}\right)\right)\left(t\right)=\left(\left(\hat{f}*\hat{f}^{-1}\right)*\hat{g}\right)\left(t\right)=\left(\mathbf{1}_{0}*\hat{g}\right)\left(t\right)=\hat{g}\left(t\right),\textrm{ }\forall t\in\hat{\mathbb{Z}}_{p} \end{equation} In particular, letting $\hat{h}$ denote $\hat{f}^{-1}*\hat{g}$, we have that: \begin{equation} \hat{g}\left(t\right)=\left(\hat{f}*\hat{h}\right)\left(t\right)\overset{\mathbb{C}_{q}}{=}\lim_{N\rightarrow\infty}\sum_{\left|s\right|_{p}\leq p^{N}}\hat{h}\left(s\right)\hat{f}\left(t-s\right) \end{equation} Since $\hat{f}$ and $\hat{h}$ are in $c_{0}$, note that: \begin{align*} \sup_{t\in\hat{\mathbb{Z}}_{p}}\left|\sum_{s\in\hat{\mathbb{Z}}_{p}}\hat{h}\left(s\right)\hat{f}\left(t-s\right)-\sum_{\left|s\right|_{p}\leq p^{N}}\hat{h}\left(s\right)\hat{f}\left(t-s\right)\right|_{q} & \leq\sup_{t\in\hat{\mathbb{Z}}_{p}}\sup_{\left|s\right|_{p}>p^{N}}\left|\hat{h}\left(s\right)\hat{f}\left(t-s\right)\right|_{q}\\ \left(\left|\hat{f}\right|_{q}<\infty\right); & \leq\sup_{\left|s\right|_{p}>p^{N}}\left|\hat{h}\left(s\right)\right|_{q} \end{align*} and hence: \begin{equation} \lim_{N\rightarrow\infty}\sup_{t\in\hat{\mathbb{Z}}_{p}}\left|\sum_{s\in\hat{\mathbb{Z}}_{p}}\hat{h}\left(s\right)\hat{f}\left(t-s\right)-\sum_{\left|s\right|_{p}\leq p^{N}}\hat{h}\left(s\right)\hat{f}\left(t-s\right)\right|_{q}\overset{\mathbb{R}}{=}\lim_{N\rightarrow\infty}\sup_{\left|s\right|_{p}>p^{N}}\left|\hat{h}\left(s\right)\right|_{q}\overset{\mathbb{R}}{=}0 \end{equation} which shows that the $q$-adic convergence of $\sum_{\left|s\right|_{p}\leq p^{N}}\hat{h}\left(s\right)\hat{f}\left(t-s\right)$ to $\hat{g}\left(t\right)=\sum_{s\in\hat{\mathbb{Z}}_{p}}\hat{h}\left(s\right)\hat{f}\left(t-s\right)$ is uniform in $t$. Hence: \begin{equation} \lim_{N\rightarrow\infty}\sup_{t\in\hat{\mathbb{Z}}_{p}}\left|\hat{g}\left(t\right)-\sum_{\left|s\right|_{p}\leq p^{N}}\hat{h}\left(s\right)\hat{f}\left(t-s\right)\right|_{q}=0 \end{equation} which is precisely the definition of what it means for the sequence $\left\{ \sum_{\left|s\right|_{p}\leq p^{N}}\hat{h}\left(s\right)\hat{f}\left(t-s\right)\right\} _{N\geq0}$ to converge in $c_{0}\left(\hat{\mathbb{Z}}_{p},\mathbb{C}_{q}\right)$ to $\hat{g}$. Since $\hat{g}$ was arbitrary, and since this sequence is in the span of the translates of $\hat{f}$ over $\mathbb{C}_{q}$, we see that said span is dense in $c_{0}\left(\hat{\mathbb{Z}}_{p},\mathbb{C}_{q}\right)$. \vphantom{} \textbullet{} ($\textrm{III}\Rightarrow\textrm{IV}$) Suppose $\textrm{span}_{\mathbb{C}_{q}}\left\{ \tau_{s}\left\{ \hat{f}\right\} \left(t\right):s\in\hat{\mathbb{Z}}_{p}\right\} $ is dense in $c_{0}\left(\hat{\mathbb{Z}}_{p},\mathbb{C}_{q}\right)$. Since $\mathbf{1}_{0}\left(t\right)\in c_{0}\left(\hat{\mathbb{Z}}_{p},\mathbb{C}_{q}\right)$, given any $\epsilon\in\left(0,1\right)$, we can then choose constants $\mathfrak{c}_{1},\ldots,\mathfrak{c}_{M}\in\mathbb{C}_{q}$ and $t_{1},\ldots,t_{M}\in\hat{\mathbb{Z}}_{p}$ so that: \begin{equation} \sup_{t\in\hat{\mathbb{Z}}_{p}}\left|\mathbf{1}_{0}\left(t\right)-\sum_{m=1}^{M}\mathfrak{c}_{m}\hat{f}\left(t-t_{m}\right)\right|_{q}<\epsilon \end{equation} Now, letting $N\geq\max\left\{ -v_{p}\left(t_{1}\right),\ldots,-v_{p}\left(t_{M}\right)\right\} $ be arbitrary (note that $-v_{p}\left(t\right)=-\infty$ when $t=0$), the maps $t\mapsto t+t_{m}$ are then bijections of the set $\left\{ t\in\hat{\mathbb{Z}}_{p}:\left|t\right|_{p}\leq p^{N}\right\} $. Consequently: \begin{align*} \sum_{\left|t\right|_{p}\leq p^{N}}\left(\sum_{m=1}^{M}\mathfrak{c}_{m}\hat{f}\left(t-t_{m}\right)\right)e^{2\pi i\left\{ t\mathfrak{z}\right\} _{p}} & =\sum_{m=1}^{M}\mathfrak{c}_{m}\sum_{\left|t\right|_{p}\leq p^{N}}\hat{f}\left(t\right)e^{2\pi i\left\{ \left(t+t_{m}\right)\mathfrak{z}\right\} _{p}}\\ & =\left(\sum_{m=1}^{M}\mathfrak{c}_{m}e^{2\pi i\left\{ t_{m}\mathfrak{z}\right\} _{p}}\right)\sum_{\left|t\right|_{p}\leq p^{N}}\hat{f}\left(t\right)e^{2\pi i\left\{ t\mathfrak{z}\right\} _{p}} \end{align*} Letting $N\rightarrow\infty$, we obtain: \begin{equation} \lim_{N\rightarrow\infty}\sum_{\left|t\right|_{p}\leq p^{N}}\left(\sum_{m=1}^{M}\mathfrak{c}_{m}\hat{f}\left(t-t_{m}\right)\right)e^{2\pi i\left\{ t\mathfrak{z}\right\} _{p}}\overset{\mathbb{C}_{q}}{=}g_{m}\left(\mathfrak{z}\right)f\left(\mathfrak{z}\right) \end{equation} where $g_{m}:\mathbb{Z}_{p}\rightarrow\mathbb{C}_{q}$ is defined by: \begin{equation} g_{m}\left(\mathfrak{z}\right)\overset{\textrm{def}}{=}\sum_{m=1}^{M}\mathfrak{c}_{m}e^{2\pi i\left\{ t_{m}\mathfrak{z}\right\} _{p}},\textrm{ }\forall\mathfrak{z}\in\mathbb{Z}_{p}\label{eq:Definition of g_m} \end{equation} Moreover, this convergence is uniform with respect to $\mathfrak{z}$. Consequently: \begin{align*} \left|1-g_{m}\left(\mathfrak{z}\right)f\left(\mathfrak{z}\right)\right|_{q} & \overset{\mathbb{R}}{=}\lim_{N\rightarrow\infty}\left|\sum_{\left|t\right|_{p}\leq p^{N}}\left(\mathbf{1}_{0}\left(t\right)-\sum_{m=1}^{M}\mathfrak{c}_{m}\hat{f}\left(t-t_{m}\right)\right)e^{2\pi i\left\{ t\mathfrak{z}\right\} _{p}}\right|_{q}\\ & \leq\sup_{t\in\hat{\mathbb{Z}}_{p}}\left|\mathbf{1}_{0}\left(t\right)-\sum_{m=1}^{M}\mathfrak{c}_{m}\hat{f}\left(t-t_{m}\right)\right|_{q}\\ & <\epsilon \end{align*} for all $\mathfrak{z}\in\mathbb{Z}_{p}$. If $f\left(\mathfrak{z}_{0}\right)=0$ for some $\mathfrak{z}_{0}\in\mathbb{Z}_{p}$, we would then have: \begin{equation} \epsilon>\left|1-g_{m}\left(\mathfrak{z}_{0}\right)\cdot0\right|_{q}=1 \end{equation} which would contradict the fact that $\epsilon\in\left(0,1\right)$. As such, $f$ cannot have any zeroes whenever the span of $\hat{f}$'s translates are dense in $c_{0}\left(\hat{\mathbb{Z}}_{p},\mathbb{C}_{q}\right)$. \vphantom{} \textbullet{} ($\textrm{IV}\Rightarrow\textrm{I}$) Suppose $f$ has no zeroes. Since $\mathfrak{y}\mapsto\frac{1}{\mathfrak{y}}$ is a continuous map on $\mathbb{C}_{q}\backslash\left\{ 0\right\} $, and since compositions of continuous maps are continuous, to show that $1/f$ is continuous, it suffices to show that $\left|f\left(\mathfrak{z}\right)\right|_{q}$ is bounded away from $0$. To see this, suppose by way of contradiction that there was a sequence $\left\{ \mathfrak{z}_{n}\right\} _{n\geq0}\subseteq\mathbb{Z}_{p}$ such that for all $\epsilon>0$, $\left|f\left(\mathfrak{z}_{n}\right)\right|_{q}<\epsilon$ holds for all sufficiently large $n$\textemdash say, for all $n\geq N_{\epsilon}$. Since $\mathbb{Z}_{p}$ is compact, the $\mathfrak{z}_{n}$s have a subsequence $\mathfrak{z}_{n_{k}}$ which converges in $\mathbb{Z}_{p}$ to some limit $\mathfrak{z}_{\infty}$. The continuity of $f$ forces $f\left(\mathfrak{z}_{\infty}\right)=0$, which contradicts the hypothesis that $f$ was given to have no zeroes. Thus, if $f$ has no zeroes, $\left|f\left(\mathfrak{z}\right)\right|_{q}$ is bounded away from zero. This prove that $1/f$ is $\left(p,q\right)$-adically continuous. Q.E.D. \vphantom{} The proof of WTT for $\left(p,q\right)$-adic measures is significantly more involved than the continuous case. While proving the non-vanishing of the limit of $\tilde{\mu}_{N}\left(\mathfrak{z}\right)$ when $\hat{\mu}$'s translates have dense span is as simple as in the continuous case, the other direction requires a more intricate argument. The idea is this: by way of contradiction, suppose there is a $\mathfrak{z}_{0}\in\mathbb{Z}_{p}$ so that $\lim_{N\rightarrow\infty}\tilde{\mu}_{N}\left(\mathfrak{z}_{0}\right)$ converges in $\mathbb{C}_{q}$ to zero, yet the span of the translates of $\hat{\mu}$ \emph{are} dense in $c_{0}\left(\hat{\mathbb{Z}}_{p},\mathbb{C}_{q}\right)$. We get a contradiction by showing that, with these assumptions, we can construct two functions $c_{0}\left(\hat{\mathbb{Z}}_{p},\mathbb{C}_{q}\right)$, both of which produce interesting results when convolved with $\hat{\mu}$. The first function is constructed so as to give us something close (in the sense of $c_{0}\left(\hat{\mathbb{Z}}_{p},\mathbb{C}_{q}\right)$'s norm, the $\left(p,q\right)$-adic sup norm on $\hat{\mathbb{Z}}_{p}$) to the constant function $0$ when we convolve it with $\hat{\mu}$, whereas the second is constructed so as to give us something close to $\mathbf{1}_{0}$ upon convolution with $\hat{\mu}$. Applications of the ultrametric inequality (a.k.a., the strong triangle inequality) show that these two estimates tear each other to pieces, and thus cannot \emph{both }be true; this yields the desired contradiction. While the idea is straightforward, the approach is somewhat technical, because our argument will require restricting our attention to bounded neighborhoods of zero in $\hat{\mathbb{Z}}_{p}$. The contradictory part of estimates we need follow from delicate manipulations of convolutions in tandem with these domain restrictions. As such, it will be very helpful to have a notation for the supremum norm of a function $\hat{\mathbb{Z}}_{p}\rightarrow\mathbb{C}_{q}$ taken over a bounded neighborhood of zero, rather than all of $\hat{\mathbb{Z}}_{p}$. \begin{defn} \label{def:norm notation definition}For any integer $n\geq1$ and any function $\hat{\chi}:\hat{\mathbb{Z}}_{p}\rightarrow\mathbb{C}_{q}$, we write $\left\Vert \hat{\chi}\right\Vert _{p^{n},q}$ to denote the non-archimedean norm: \begin{equation} \left\Vert \hat{\chi}\right\Vert _{p^{n},q}\overset{\textrm{def}}{=}\sup_{\left|t\right|_{p}\leq p^{n}}\left|\hat{\chi}\left(t\right)\right|_{q}\label{eq:Definition of truncated norm} \end{equation} \end{defn} \begin{rem} \textbf{WARNING }\textendash{} For the statement and proof of \textbf{Theorem \ref{thm:pq WTT for measures}} and \textbf{Lemma \ref{lem:three convolution estimate}}, we will write $\left\Vert \cdot\right\Vert _{p^{\infty},q}$ to denote the $\left(p,q\right)$-adic supremum norm for functions $\hat{\mathbb{Z}}_{p}\rightarrow\mathbb{C}_{q}$. writing ``$\left\Vert \cdot\right\Vert _{p,q}$'' would potentially cause confusion with $\left\Vert \cdot\right\Vert _{p^{1},q}$. \end{rem} \begin{thm}[\textbf{Wiener Tauberian Theorem for $\left(p,q\right)$-adic Measures}] \index{Wiener!Tauberian Theorem!left(p,qright)-adic@$\left(p,q\right)$-adic}\label{thm:pq WTT for measures}Let $d\mu\in C\left(\mathbb{Z}_{p},\mathbb{C}_{q}\right)^{\prime}$. Then, $\textrm{span}_{\mathbb{C}_{q}}\left\{ \tau_{s}\left\{ \hat{\mu}\right\} \left(t\right):s\in\hat{\mathbb{Z}}_{p}\right\} $ is dense in $c_{0}\left(\hat{\mathbb{Z}}_{p},\mathbb{C}_{q}\right)$ if and only if, for any $\mathfrak{z}\in\mathbb{Z}_{p}$ for which the limit $\lim_{N\rightarrow\infty}\tilde{\mu}_{N}\left(\mathfrak{z}\right)$ converges in $\mathbb{C}_{q}$, said limit is necessarily non-zero. \end{thm} \vphantom{} While much of the intricate business involving the intermingling of restrictions and convolutions occurs in the various claims that structure our proof of \textbf{Theorem \ref{thm:pq WTT for measures}}, one particular result in this vein\textemdash the heart of \textbf{Theorem \ref{thm:pq WTT for measures}}'s proof\textemdash is sufficiently non-trivial as to merit separate consideration, so as to avoid cluttering the flow of the argument. This is detailed in the following lemma: \begin{lem} \label{lem:three convolution estimate}Let $M,N\in\mathbb{N}_{0}$ be arbitrary, and let $\hat{\phi}\in B\left(\hat{\mathbb{Z}}_{p},\mathbb{C}_{q}\right)$ be supported on $\left|t\right|_{p}\leq p^{N}$. Then, for any $\hat{f},\hat{g}:\hat{\mathbb{Z}}_{p}\rightarrow\mathbb{C}_{q}$ where $\hat{g}$ is supported on $\left|t\right|_{p}\leq p^{M}$, we have: \begin{equation} \left\Vert \hat{\phi}*\hat{f}\right\Vert _{p^{M},q}\leq\left\Vert \hat{\phi}\right\Vert _{p^{\infty},q}\left\Vert \hat{f}\right\Vert _{p^{\max\left\{ M,N\right\} },q}\label{eq:phi_N hat convolve f hat estimate-1} \end{equation} \begin{equation} \left\Vert \hat{\phi}*\hat{f}*\hat{g}\right\Vert _{p^{M},q}\leq\left\Vert \hat{\phi}*\hat{f}\right\Vert _{p^{\max\left\{ M,N\right\} },q}\left\Vert \hat{g}\right\Vert _{p^{\infty},q}\label{eq:phi_N hat convolve f hat convolve g hat estimate-1} \end{equation} \end{lem} Proof: (\ref{eq:phi_N hat convolve f hat estimate-1}) is the easier of the two: \begin{align*} \left\Vert \hat{\phi}*\hat{f}\right\Vert _{p^{M},q} & =\sup_{\left|t\right|_{p}\leq p^{M}}\left|\sum_{s\in\hat{\mathbb{Z}}_{p}}\hat{\phi}\left(s\right)\hat{f}\left(t-s\right)\right|_{q}\\ \left(\hat{\phi}\left(s\right)=0,\textrm{ }\forall\left|s\right|_{p}>p^{N}\right); & \leq\sup_{\left|t\right|_{p}\leq p^{M}}\left|\sum_{\left|s\right|_{p}\leq p^{N}}\hat{\phi}\left(s\right)\hat{f}\left(t-s\right)\right|_{q}\\ \left(\textrm{ultrametric ineq.}\right); & \leq\sup_{\left|t\right|_{p}\leq p^{M}}\sup_{\left|s\right|_{p}\leq p^{N}}\left|\hat{\phi}\left(s\right)\hat{f}\left(t-s\right)\right|_{q}\\ & \leq\sup_{\left|s\right|_{p}\leq p^{N}}\left|\hat{\phi}\left(s\right)\right|_{q}\sup_{\left|t\right|_{p}\leq p^{\max\left\{ M,N\right\} }}\left|\hat{f}\left(t\right)\right|_{q}\\ & =\left\Vert \hat{\phi}\right\Vert _{p^{\infty},q}\cdot\left\Vert \hat{f}\right\Vert _{p^{\max\left\{ M,N\right\} },q} \end{align*} and we are done. Proving (\ref{eq:phi_N hat convolve f hat convolve g hat estimate-1}) is similar, but more involved. We start by writing out the convolution of $\hat{\phi}*\hat{f}$ and $\hat{g}$: \begin{align*} \left\Vert \hat{\phi}*\hat{f}*\hat{g}\right\Vert _{p^{M},q} & =\sup_{\left|t\right|_{p}\leq p^{M}}\left|\sum_{s\in\hat{\mathbb{Z}}_{p}}\left(\hat{\phi}*\hat{f}\right)\left(t-s\right)\hat{g}\left(s\right)\right|_{q}\\ & \leq\sup_{\left|t\right|_{p}\leq p^{M}}\sup_{s\in\hat{\mathbb{Z}}_{p}}\left|\left(\hat{\phi}*\hat{f}\right)\left(t-s\right)\hat{g}\left(s\right)\right|_{q}\\ \left(\hat{g}\left(s\right)=0,\textrm{ }\forall\left|s\right|_{p}>p^{M}\right); & \leq\sup_{\left|t\right|_{p}\leq p^{M}}\sup_{\left|s\right|_{p}\leq p^{M}}\left|\left(\hat{\phi}*\hat{f}\right)\left(t-s\right)\hat{g}\left(s\right)\right|_{q}\\ & \leq\left\Vert \hat{g}\right\Vert _{p^{\infty},q}\sup_{\left|t\right|_{p},\left|s\right|_{p}\leq p^{M}}\left|\left(\hat{\phi}*\hat{f}\right)\left(t-s\right)\right|_{q}\\ \left(\textrm{write out }\hat{\phi}*\hat{f}\right); & =\left\Vert \hat{g}\right\Vert _{p^{\infty},q}\sup_{\left|t\right|_{p},\left|s\right|_{p}\leq p^{M}}\left|\sum_{\tau\in\hat{\mathbb{Z}}_{p}}\hat{\phi}\left(t-s-\tau\right)\hat{f}\left(\tau\right)\right|_{q}\\ \left(\textrm{let }u=s+\tau\right); & =\left\Vert \hat{g}\right\Vert _{p^{\infty},q}\sup_{\left|t\right|_{p},\left|s\right|_{p}\leq p^{M}}\left|\sum_{u-s\in\hat{\mathbb{Z}}_{p}}\hat{\phi}\left(t-u\right)\hat{f}\left(u-s\right)\right|_{q}\\ \left(s+\hat{\mathbb{Z}}_{p}=\hat{\mathbb{Z}}_{p}\right); & =\left\Vert \hat{g}\right\Vert _{p^{\infty},q}\sup_{\left|t\right|_{p},\left|s\right|_{p}\leq p^{M}}\left|\sum_{u\in\hat{\mathbb{Z}}_{p}}\hat{\phi}\left(t-u\right)\hat{f}\left(u-s\right)\right|_{q} \end{align*} Now we use the fact that $\hat{\phi}\left(t-u\right)$ vanishes for all $\left|t-u\right|_{p}>p^{N}$. Because $t$ is restricted to $\left|t\right|_{p}\leq p^{M}$, observe that for $\left|u\right|_{p}>p^{\max\left\{ M,N\right\} }$, the ultrametric inequality allows us to write: \begin{equation} \left|t-u\right|_{p}=\max\left\{ \left|t\right|_{p},\left|u\right|_{p}\right\} >p^{\max\left\{ M,N\right\} }>p^{N} \end{equation} So, for $\left|t\right|_{p},\left|s\right|_{p}\leq p^{M}$, the summand $\hat{\phi}\left(t-u\right)\hat{f}\left(u-s\right)$ vanishes whenever $\left|u\right|_{p}>p^{\max\left\{ M,N\right\} }$. This gives us: \begin{equation} \left\Vert \hat{\phi}*\hat{f}*\hat{g}\right\Vert _{p^{M},q}\leq\left\Vert \hat{g}\right\Vert _{p^{\infty},q}\sup_{\left|t\right|_{p},\left|s\right|_{p}\leq p^{M}}\left|\sum_{\left|u\right|_{p}\leq p^{\max\left\{ M,N\right\} }}\hat{\phi}\left(t-u\right)\hat{f}\left(u-s\right)\right|_{q} \end{equation} Next, we expand the range of $\left|s\right|_{p}$ and $\left|t\right|_{p}$ from $\leq p^{M}$ in $p$-adic absolute value to $\leq p^{\max\left\{ M,N\right\} }$ in $p$-adic absolute value: \begin{equation} \left\Vert \hat{\phi}*\hat{f}*\hat{g}\right\Vert _{p^{M},q}\leq\left\Vert \hat{g}\right\Vert _{p^{\infty},q}\sup_{\left|t\right|_{p},\left|s\right|_{p}\leq p^{\max\left\{ M,N\right\} }}\left|\sum_{\left|u\right|_{p}\leq p^{\max\left\{ M,N\right\} }}\hat{\phi}\left(t-u\right)\hat{f}\left(u-s\right)\right|_{q} \end{equation} In doing so, we have put $s$, $t$, and $u$ all in the same $p$-adic neighborhood of $0$: \begin{equation} \left\{ x\in\hat{\mathbb{Z}}_{p}:\left|x\right|_{p}\leq p^{\max\left\{ M,N\right\} }\right\} \end{equation} We do this because this set is closed under addition: for any $\left|s\right|_{p}\leq p^{\max\left\{ M,N\right\} }$, our $u$-sum is invariant under the change of variables $u\mapsto u+s$, and we obtain: \begin{align*} \left\Vert \hat{\phi}*\hat{f}*\hat{g}\right\Vert _{p^{M},q} & \leq\left\Vert \hat{g}\right\Vert _{p^{\infty},q}\sup_{\left|t\right|_{p},\left|s\right|_{p}\leq p^{\max\left\{ M,N\right\} }}\left|\sum_{\left|u\right|_{p}\leq p^{\max\left\{ M,N\right\} }}\hat{\phi}\left(t-\left(u+s\right)\right)\hat{f}\left(u\right)\right|_{q}\\ & =\left\Vert \hat{g}\right\Vert _{p^{\infty},q}\sup_{\left|t\right|_{p},\left|s\right|_{p}\leq p^{\max\left\{ M,N\right\} }}\left|\sum_{\left|u\right|_{p}\leq p^{\max\left\{ M,N\right\} }}\hat{\phi}\left(t-s-u\right)\hat{f}\left(u\right)\right|_{q} \end{align*} Finally, observing that: \begin{equation} \left\{ t-s:\left|t\right|_{p},\left|s\right|_{p}\leq p^{\max\left\{ M,N\right\} }\right\} =\left\{ t:\left|t\right|_{p}\leq p^{\max\left\{ M,N\right\} }\right\} \end{equation} we can write: \begin{align*} \left\Vert \hat{\phi}*\hat{f}*\hat{g}\right\Vert _{p^{M},q} & \leq\left\Vert \hat{g}\right\Vert _{p^{\infty},q}\sup_{\left|t\right|_{p}\leq p^{\max\left\{ M,N\right\} }}\left|\sum_{\left|u\right|_{p}\leq p^{\max\left\{ M,N\right\} }}\hat{\phi}\left(t-u\right)\hat{f}\left(u\right)\right|_{q}\\ & \leq\left\Vert \hat{g}\right\Vert _{p^{\infty},q}\sup_{\left|t\right|_{p}\leq p^{\max\left\{ M,N\right\} }}\left|\underbrace{\sum_{u\in\hat{\mathbb{Z}}_{p}}\hat{\phi}\left(t-u\right)\hat{f}\left(u\right)}_{\hat{\phi}*\hat{f}}\right|_{q}\\ & =\left\Vert \hat{g}\right\Vert _{p^{\infty},q}\sup_{\left|t\right|_{p}\leq p^{\max\left\{ M,N\right\} }}\left|\left(\hat{\phi}*\hat{f}\right)\left(u\right)\right|_{q}\\ \left(\textrm{by definition}\right); & =\left\Vert \hat{g}\right\Vert _{p^{\infty},q}\left\Vert \hat{\phi}*\hat{f}\right\Vert _{p^{\max\left\{ M,N\right\} },q} \end{align*} This proves the desired estimate, and with it, the rest of the Lemma. Q.E.D. \vphantom{} \textbf{Proof} \textbf{of Theorem \ref{thm:pq WTT for measures}}: We start with the simpler of the two directions. I. Suppose the span of the translates of $\hat{\mu}$ are dense. Just as in the proof of the WTT for continuous $\left(p,q\right)$-adic functions, we let $\epsilon\in\left(0,1\right)$ and then choose $\mathfrak{c}_{m}$s and $t_{m}$s so that: \begin{equation} \sup_{t\in\hat{\mathbb{Z}}_{p}}\left|\mathbf{1}_{0}\left(t\right)-\sum_{m=1}^{M}\mathfrak{c}_{m}\hat{\mu}\left(t-t_{m}\right)\right|_{q}<\epsilon \end{equation} Picking sufficiently large $N$, we obtain: \begin{align*} \left|1-\left(\sum_{m=1}^{M}\mathfrak{c}_{m}e^{2\pi i\left\{ t_{m}\mathfrak{z}\right\} _{p}}\right)\tilde{\mu}_{N}\left(\mathfrak{z}\right)\right|_{q} & \leq\max_{\left|t\right|_{p}\leq p^{N}}\left|\mathbf{1}_{0}\left(t\right)-\sum_{m=1}^{M}\mathfrak{c}_{m}\hat{\mu}\left(t-t_{m}\right)\right|_{q}\\ & \leq\sup_{t\in\hat{\mathbb{Z}}_{p}}\left|\mathbf{1}_{0}\left(t\right)-\sum_{m=1}^{M}\mathfrak{c}_{m}\hat{\mu}\left(t-t_{m}\right)\right|_{q}\\ & <\epsilon \end{align*} Now, let $\mathfrak{z}_{0}\in\mathbb{Z}_{p}$ be a point so that $\mathfrak{L}\overset{\textrm{def}}{=}\lim_{N\rightarrow\infty}\tilde{\mu}_{N}\left(\mathfrak{z}_{0}\right)$ converges in $\mathbb{C}_{q}$. We need to show $\mathfrak{L}\neq0$. To do this, plugging $\mathfrak{z}=\mathfrak{z}_{0}$ into the above yields: \[ \epsilon>\lim_{N\rightarrow\infty}\left|1-\left(\sum_{m=1}^{M}\mathfrak{c}_{m}e^{2\pi i\left\{ t_{m}\mathfrak{z}_{0}\right\} _{p}}\right)\tilde{\mu}_{N}\left(\mathfrak{z}_{0}\right)\right|_{q}=\left|1-\left(\sum_{m=1}^{M}\mathfrak{c}_{m}e^{2\pi i\left\{ t_{m}\mathfrak{z}_{0}\right\} _{p}}\right)\cdot\mathfrak{L}\right|_{q} \] If $\mathfrak{L}=0$, the right-most expression will be $1$, and hence, we get $\epsilon>1$, but this is impossible; $\epsilon$ was given to be less than $1$. So, if $\lim_{N\rightarrow\infty}\tilde{\mu}_{N}\left(\mathfrak{z}_{0}\right)$ converges in $\mathbb{C}_{q}$ to $\mathfrak{L}$, $\mathfrak{L}$ must be non-zero. \vphantom{} II. Let $\mathfrak{z}_{0}\in\mathbb{Z}_{p}$ be a zero of $\tilde{\mu}\left(\mathfrak{z}\right)$ such that $\tilde{\mu}_{N}\left(\mathfrak{z}_{0}\right)\rightarrow0$ in $\mathbb{C}_{q}$ as $N\rightarrow\infty$. Then, by way of contradiction, suppose the span of the translates of $\hat{\mu}$ is dense in $c_{0}\left(\hat{\mathbb{Z}}_{p},\mathbb{C}_{q}\right)$, despite the zero of $\tilde{\mu}$ at $\mathfrak{z}_{0}$. Thus, we can use linear combinations of translates of $\hat{\mu}$ approximate any function in $c_{0}\left(\hat{\mathbb{Z}}_{p},\mathbb{C}_{q}\right)$'s sup norm. In particular, we choose to approximate $\mathbf{1}_{0}$: letting $\epsilon\in\left(0,1\right)$ be arbitrary, there is then a choice of $\mathfrak{c}_{k}$s in $\mathbb{C}_{q}$ and $t_{k}$s in $\hat{\mathbb{Z}}_{p}$ so that: \begin{equation} \sup_{t\in\hat{\mathbb{Z}}_{p}}\left|\mathbf{1}_{0}\left(t\right)-\sum_{k=1}^{K}\mathfrak{c}_{k}\hat{\mu}\left(t-t_{k}\right)\right|_{q}<\epsilon \end{equation} Letting: \begin{equation} \hat{\eta}_{\epsilon}\left(t\right)\overset{\textrm{def}}{=}\sum_{k=1}^{K}\mathfrak{c}_{k}\mathbf{1}_{t_{k}}\left(t\right)\label{eq:Definition of eta_epsilon hat} \end{equation} we can express the above linear combination as the convolution: \begin{equation} \left(\hat{\mu}*\hat{\eta}_{\epsilon}\right)\left(t\right)=\sum_{\tau\in\hat{\mathbb{Z}}_{p}}\hat{\mu}\left(t-\tau\right)\sum_{k=1}^{K}\mathfrak{c}_{k}\mathbf{1}_{t_{k}}\left(\tau\right)=\sum_{k=1}^{K}\mathfrak{c}_{k}\hat{\mu}\left(t-t_{k}\right) \end{equation} So, we get: \begin{equation} \left\Vert \mathbf{1}_{0}-\hat{\mu}*\hat{\eta}_{\epsilon}\right\Vert _{p^{\infty},q}\overset{\textrm{def}}{=}\sup_{t\in\hat{\mathbb{Z}}_{p}}\left|\mathbf{1}_{0}\left(t\right)-\left(\hat{\mu}*\hat{\eta}_{\epsilon}\right)\left(t\right)\right|_{q}<\epsilon\label{eq:Converse WTT - eq. 1} \end{equation} Before proceeding any further, it is vital to note that we can (and must) assume that $\hat{\eta}_{\epsilon}$ is not identically\footnote{This must hold whenever $\epsilon\in\left(0,1\right)$, because\textemdash were $\hat{\eta}_{\epsilon}$ identically zero\textemdash the supremum: \[ \sup_{t\in\hat{\mathbb{Z}}_{p}}\left|\mathbf{1}_{0}\left(t\right)-\left(\hat{\mu}*\hat{\eta}_{\epsilon}\right)\left(t\right)\right|_{q} \] would then be equal to $1$, rather than $<\epsilon$.} $0$. That being done, equation (\ref{eq:Converse WTT - eq. 1}) shows the assumption we want to contradict: the existence of a function which produces something close to $\mathbf{1}_{0}$ after convolution with $\hat{\mu}$. We will arrive at our contradiction by showing that the zero of $\tilde{\mu}$ at $\mathfrak{z}_{0}$ allows us to construct a second function which yields something close to $0$ after convolution with $\hat{\mu}$. By convolving \emph{both} of these functions with $\hat{\mu}$, we will end up with something which is close to both $\mathbf{1}_{0}$ \emph{and }close to $0$, which is, of course, impossible. Our make-things-close-to-zero-by-convolution function is going to be: \begin{equation} \hat{\phi}_{N}\left(t\right)\overset{\textrm{def}}{=}\mathbf{1}_{0}\left(p^{N}t\right)e^{-2\pi i\left\{ t\mathfrak{z}_{0}\right\} _{p}},\textrm{ }\forall t\in\hat{\mathbb{Z}}_{p}\label{eq:Definition of Phi_N hat} \end{equation} Note that $\hat{\phi}_{N}\left(t\right)$ is only supported for $\left|t\right|_{p}\leq p^{N}$. Now, as defined, we have that: \begin{align*} \left(\hat{\mu}*\hat{\phi}_{N}\right)\left(\tau\right) & =\sum_{s\in\hat{\mathbb{Z}}_{p}}\hat{\mu}\left(\tau-s\right)\hat{\phi}_{N}\left(s\right)\\ \left(\hat{\phi}_{N}\left(s\right)=0,\textrm{ }\forall\left|s\right|_{p}>p^{N}\right); & =\sum_{\left|s\right|_{p}\leq p^{N}}\hat{\mu}\left(\tau-s\right)\hat{\phi}_{N}\left(s\right)\\ & =\sum_{\left|s\right|_{p}\leq p^{N}}\hat{\mu}\left(\tau-s\right)e^{-2\pi i\left\{ s\mathfrak{z}_{0}\right\} _{p}} \end{align*} Fixing $\tau$, observe that the map $s\mapsto\tau-s$ is a bijection of the set $\left\{ s\in\hat{\mathbb{Z}}_{p}:\left|s\right|_{p}\leq p^{N}\right\} $ whenever $\left|\tau\right|_{p}\leq p^{N}$. So, for any $N\geq-v_{p}\left(\tau\right)$, we obtain: \begin{align*} \left(\hat{\mu}*\hat{\phi}_{N}\right)\left(\tau\right) & =\sum_{\left|s\right|_{p}\leq p^{N}}\hat{\mu}\left(\tau-s\right)e^{-2\pi i\left\{ s\mathfrak{z}_{0}\right\} _{p}}\\ & =\sum_{\left|s\right|_{p}\leq p^{N}}\hat{\mu}\left(s\right)e^{-2\pi i\left\{ \left(\tau-s\right)\mathfrak{z}_{0}\right\} _{p}}\\ & =e^{-2\pi i\left\{ \tau\mathfrak{z}_{0}\right\} _{p}}\sum_{\left|s\right|_{p}\leq p^{N}}\hat{\mu}\left(s\right)e^{2\pi i\left\{ s\mathfrak{z}_{0}\right\} _{p}}\\ & =e^{-2\pi i\left\{ \tau\mathfrak{z}_{0}\right\} _{p}}\tilde{\mu}_{N}\left(\mathfrak{z}_{0}\right) \end{align*} Since this holds for all $N\geq-v_{p}\left(\tau\right)$, upon letting $N\rightarrow\infty$, $\tilde{\mu}_{N}\left(\mathfrak{z}_{0}\right)$ converges to $0$ in \emph{$\mathbb{C}_{q}$}, by our assumption. So, for any $\epsilon^{\prime}>0$, there exists an $N_{\epsilon^{\prime}}$ so that $\left|\tilde{\mu}_{N}\left(\mathfrak{z}_{0}\right)\right|_{q}<\epsilon^{\prime}$ for all $N\geq N_{\epsilon^{\prime}}$. Combining this with the above computation (after taking $q$-adic absolute values), we have established the following: \begin{claim} \label{claim:phi_N hat claim}Let $\epsilon^{\prime}>0$ be arbitrary. Then, there exists an $N_{\epsilon^{\prime}}\geq0$ (depending only on $\hat{\mu}$ and $\epsilon^{\prime}$) so that, for all $\tau\in\hat{\mathbb{Z}}_{p}$: \begin{equation} \left|\left(\hat{\mu}*\hat{\phi}_{N}\right)\left(\tau\right)\right|_{q}=\left|e^{-2\pi i\left\{ \tau\mathfrak{z}_{0}\right\} _{p}}\tilde{\mu}_{N}\left(\mathfrak{z}_{0}\right)\right|_{q}<\epsilon^{\prime},\textrm{ }\forall N\geq\max\left\{ N_{\epsilon^{\prime}},-v_{p}\left(\tau\right)\right\} ,\tau\in\hat{\mathbb{Z}}_{p}\label{eq:WTT - First Claim} \end{equation} \end{claim} \vphantom{} As stated, the idea is to convolve $\hat{\mu}*\hat{\phi}_{N}$ with $\hat{\eta}_{\epsilon}$ so as to obtain a function (via the associativity of convolution) which is both close to $0$ and close to $\mathbf{1}_{0}$. However, our present situation is less than ideal because the lower bound on $N$ in \textbf{Claim \ref{claim:phi_N hat claim}} depends on $\tau$, and so, the convergence of $\left|\left(\hat{\mu}*\hat{\phi}_{N}\right)\left(\tau\right)\right|_{q}$ to $0$ as $N\rightarrow\infty$ will \emph{not }be uniform in $\tau$. This is where the difficulty of this direction of the proof lies. To overcome this obstacle, instead of convolving $\hat{\mu}*\hat{\phi}_{N}$ with $\hat{\eta}_{\epsilon}$, we will convolve $\hat{\mu}*\hat{\phi}_{N}$ with a truncated version of $\hat{\eta}_{\epsilon}$, whose support has been restricted to a finite subset of $\hat{\mathbb{Z}}_{p}$. This is the function $\hat{\eta}_{\epsilon,M}:\hat{\mathbb{Z}}_{p}\rightarrow\mathbb{C}_{q}$ given by: \begin{equation} \hat{\eta}_{\epsilon,M}\left(t\right)\overset{\textrm{def}}{=}\mathbf{1}_{0}\left(p^{M}t\right)\hat{\eta}_{\epsilon}\left(t\right)=\begin{cases} \hat{\eta}_{\epsilon}\left(t\right) & \textrm{if }\left|t\right|_{p}\leq p^{M}\\ 0 & \textrm{else} \end{cases}\label{eq:Definition of eta epsilon M hat} \end{equation} where $M\geq0$ is arbitrary. With this, we can attain the desired ``close to both $0$ and $\mathbf{1}_{0}$'' contradiction for $\hat{\mu}*\hat{\phi}_{N}*\hat{\eta}_{\epsilon,M}$. The proof will be completed upon demonstrating that this contradiction will remain even as $M\rightarrow\infty$. The analogue of the estimate (\ref{eq:Converse WTT - eq. 1}) for this truncated case is: \begin{align*} \left(\hat{\mu}*\hat{\eta}_{\epsilon,M}\right)\left(t\right) & =\sum_{\tau\in\hat{\mathbb{Z}}_{p}}\hat{\mu}\left(t-\tau\right)\mathbf{1}_{0}\left(p^{M}\tau\right)\sum_{k=1}^{K}\mathfrak{c}_{k}\mathbf{1}_{t_{k}}\left(\tau\right)\\ & =\sum_{\left|\tau\right|_{p}\leq p^{M}}\hat{\mu}\left(t-\tau\right)\sum_{k=1}^{K}\mathfrak{c}_{k}\mathbf{1}_{t_{k}}\left(\tau\right)\\ & =\sum_{k:\left|t_{k}\right|_{p}\leq p^{M}}\mathfrak{c}_{k}\hat{\mu}\left(t-t_{k}\right) \end{align*} where, \emph{note}, instead of summing over all the $t_{k}$s that came with $\hat{\eta}_{\epsilon}$, we only sum over those $t_{k}$s in the set $\left\{ t\in\hat{\mathbb{Z}}_{p}:\left|t\right|_{p}\leq p^{M}\right\} $. Because the $t_{k}$s came with the un-truncated $\hat{\eta}_{\epsilon}$, observe that we can make $\hat{\mu}*\hat{\eta}_{\epsilon,M}$ \emph{equal} to $\hat{\mu}*\hat{\eta}_{\epsilon}$ by simply choosing $M$ to be large enough so that all the $t_{k}$s lie in $\left\{ t\in\hat{\mathbb{Z}}_{p}:\left|t\right|_{p}\leq p^{M}\right\} $. The lower bound on such $M$s is given by: \begin{equation} M_{0}\overset{\textrm{def}}{=}\max\left\{ -v_{p}\left(t_{1}\right),\ldots,-v_{p}\left(t_{K}\right)\right\} \label{eq:WTT - Choice for M_0} \end{equation} Then, we have: \begin{equation} \left(\hat{\mu}*\hat{\eta}_{\epsilon,M}\right)\left(t\right)=\sum_{k=1}^{K}\mathfrak{c}_{k}\hat{\mu}\left(t-t_{k}\right)=\left(\hat{\mu}*\hat{\eta}_{\epsilon}\right)\left(t\right),\textrm{ }\forall M\geq M_{0},\textrm{ }\forall t\in\hat{\mathbb{Z}}_{p}\label{eq:Effect of M bigger than M0} \end{equation} So, applying $\left\Vert \cdot\right\Vert _{p^{M},q}$ norm: \begin{align*} \left\Vert \mathbf{1}_{0}-\hat{\mu}*\hat{\eta}_{\epsilon,M}\right\Vert _{p^{M},q} & =\sup_{\left|t\right|_{p}\leq p^{M}}\left|\mathbf{1}_{0}\left(t\right)-\left(\hat{\mu}*\hat{\eta}_{\epsilon,M}\right)\left(t\right)\right|_{q}\\ \left(\textrm{if }M\geq M_{0}\right); & =\sup_{\left|t\right|_{p}\leq p^{M}}\left|\mathbf{1}_{0}\left(t\right)-\left(\hat{\mu}*\hat{\eta}_{\epsilon}\right)\left(t\right)\right|_{q}\\ & \leq\left\Vert \mathbf{1}_{0}-\left(\hat{\mu}*\hat{\eta}_{\epsilon}\right)\right\Vert _{p^{\infty},q}\\ & <\epsilon \end{align*} Finally, note that\emph{ $M_{0}$ }depends on\emph{ only $\hat{\eta}_{\epsilon}$ }and\emph{ $\epsilon$}. \textbf{Claim \ref{claim:truncated convolution estimates for 1d WTT}}, given below, summarizes these findings: \begin{claim} \label{claim:truncated convolution estimates for 1d WTT}Let $\epsilon>0$ be arbitrary. Then, there exists an integer $M_{0}$ depending only on $\hat{\eta}_{\epsilon}$ and $\epsilon$, so that: I. $\hat{\mu}*\hat{\eta}_{\epsilon,M}=\hat{\mu}*\hat{\eta}_{\epsilon},\textrm{ }\forall M\geq M_{0}$. \vphantom{} II. $\left\Vert \mathbf{1}_{0}-\hat{\mu}*\hat{\eta}_{\epsilon,M}\right\Vert _{p^{\infty},q}<\epsilon,\textrm{ }\forall M\geq M_{0}$. \vphantom{} III. $\left\Vert \mathbf{1}_{0}-\hat{\mu}*\hat{\eta}_{\epsilon,M}\right\Vert _{p^{M},q}<\epsilon,\textrm{ }\forall M\geq M_{0}$. \end{claim} \vphantom{} The next step is to refine our ``make $\hat{\mu}$ close to zero'' estimate by taking into account $\left\Vert \cdot\right\Vert _{p^{m},q}$. \begin{claim} \label{claim:p^m, q norm of convolution of mu-hat and phi_N hat }Let $\epsilon^{\prime}>0$. Then, there exists $N_{\epsilon^{\prime}}$ depending only on $\epsilon^{\prime}$ and $\hat{\mu}$ so that: \begin{equation} \left\Vert \hat{\phi}_{N}*\hat{\mu}\right\Vert _{p^{m},q}<\epsilon^{\prime},\textrm{ }\forall m\geq1,\textrm{ }\forall N\geq\max\left\{ N_{\epsilon^{\prime}},m\right\} \label{eq:WTT - eq. 4} \end{equation} Proof of claim: Let $\epsilon^{\prime}>0$. \textbf{Claim \ref{claim:phi_N hat claim}} tells us that there is an $N_{\epsilon^{\prime}}$ (depending on $\epsilon^{\prime}$, $\hat{\mu}$) so that: \begin{equation} \left|\left(\hat{\phi}_{N}*\hat{\mu}\right)\left(\tau\right)\right|_{q}<\epsilon^{\prime},\textrm{ }\forall N\geq\max\left\{ N_{\epsilon^{\prime}},-v_{p}\left(\tau\right)\right\} ,\textrm{ }\forall\tau\in\hat{\mathbb{Z}}_{p} \end{equation} So, letting $m\geq1$ be arbitrary, note that $\left|\tau\right|_{p}\leq p^{m}$ implies $-v_{p}\left(\tau\right)\leq m$. As such, we can make the result of \textbf{Claim \ref{claim:phi_N hat claim}} hold for all $\left|\tau\right|_{p}\leq p^{m}$ by choosing $N\geq\max\left\{ N_{\epsilon^{\prime}},m\right\} $: \begin{equation} \underbrace{\sup_{\left|\tau\right|_{p}\leq p^{m}}\left|\left(\hat{\phi}_{N}*\hat{\mu}\right)\left(\tau\right)\right|_{q}}_{\left\Vert \hat{\mu}*\hat{\phi}_{N}\right\Vert _{p^{m},q}}<\epsilon^{\prime},\textrm{ }\forall N\geq\max\left\{ N_{\epsilon^{\prime}},m\right\} \end{equation} This proves the claim. \end{claim} \vphantom{} Using \textbf{Lemma \ref{lem:three convolution estimate}}, we can now set up the string of estimates we need to arrive at the desired contradiction. First, let us choose an $\epsilon\in\left(0,1\right)$ and a function $\hat{\eta}_{\epsilon}:\hat{\mathbb{Z}}_{p}\rightarrow\mathbb{C}_{q}$ which is not identically zero, so that: \begin{equation} \left\Vert \mathbf{1}_{0}-\hat{\mu}*\hat{\eta}_{\epsilon}\right\Vert _{p^{\infty},q}<\epsilon \end{equation} Then, by \textbf{Claim \ref{claim:truncated convolution estimates for 1d WTT}}, we can choose a $M_{\epsilon}$ depending only on $\epsilon$ and $\hat{\eta}_{\epsilon}$ so that: \begin{equation} M\geq M_{\epsilon}\Rightarrow\left\Vert \mathbf{1}_{0}-\hat{\mu}*\hat{\eta}_{\epsilon,M}\right\Vert _{p^{M},q}<\epsilon\label{eq:Thing to contradict} \end{equation} This shows that $\hat{\mu}$ can be convolved to become close to $\mathbf{1}_{0}$. The contradiction will follow from convolving this with $\hat{\phi}_{N}$, where $N$, at this point, is arbitrary: \begin{equation} \left\Vert \hat{\phi}_{N}*\left(\mathbf{1}_{0}-\left(\hat{\mu}*\hat{\eta}_{\epsilon,M}\right)\right)\right\Vert _{p^{M},q}=\left\Vert \hat{\phi}_{N}-\left(\hat{\phi}_{N}*\hat{\mu}*\hat{\eta}_{\epsilon,M}\right)\right\Vert _{p^{M},q}\label{eq:WTT - Target of attack} \end{equation} Our goal here is to show that (\ref{eq:WTT - Target of attack}) is close to both $0$ and $1$ simultaneously. First, writing: \begin{equation} \left\Vert \hat{\phi}_{N}*\left(\mathbf{1}_{0}-\left(\hat{\mu}*\hat{\eta}_{\epsilon,M}\right)\right)\right\Vert _{p^{M},q} \end{equation} the fact that $\hat{\phi}_{N}\left(t\right)$ is supported on $\left|t\right|_{p}\leq p^{N}$ allows us to apply equation (\ref{eq:phi_N hat convolve f hat estimate}) from \textbf{Lemma \ref{lem:three convolution estimate}}. This gives the estimate: \begin{equation} \left\Vert \hat{\phi}_{N}*\left(\mathbf{1}_{0}-\left(\hat{\mu}*\hat{\eta}_{\epsilon,M}\right)\right)\right\Vert _{p^{M},q}\leq\underbrace{\left\Vert \hat{\phi}_{N}\right\Vert _{p^{\infty},q}}_{1}\left\Vert \mathbf{1}_{0}-\left(\hat{\mu}*\hat{\eta}_{\epsilon,M}\right)\right\Vert _{p^{\max\left\{ M,N\right\} },q} \end{equation} for all $M$ and $N$. Letting $M\geq M_{\epsilon}$, we can apply \textbf{Claim \ref{claim:truncated convolution estimates for 1d WTT}} and write $\hat{\mu}*\hat{\eta}_{\epsilon,M}=\hat{\mu}*\hat{\eta}_{\epsilon}$. So, for $M\geq M_{\epsilon}$ and arbitrary $N$, we have: \begin{align*} \left\Vert \hat{\phi}_{N}*\left(\mathbf{1}_{0}-\left(\hat{\mu}*\hat{\eta}_{\epsilon,M}\right)\right)\right\Vert _{p^{M},q} & \leq\left\Vert \mathbf{1}_{0}-\left(\hat{\mu}*\hat{\eta}_{\epsilon}\right)\right\Vert _{p^{\max\left\{ M,N\right\} },q}\\ & \leq\left\Vert \mathbf{1}_{0}-\left(\hat{\mu}*\hat{\eta}_{\epsilon}\right)\right\Vert _{p^{\infty},q}\\ \left(\textrm{\textbf{Claim \ensuremath{\ref{claim:truncated convolution estimates for 1d WTT}}}}\right); & <\epsilon \end{align*} Thus, the \emph{left-hand side} of (\ref{eq:WTT - Target of attack}) is $<\epsilon$. This is the first estimate. Keeping $M\geq M_{\epsilon}$ and $N$ arbitrary, we will obtain the desired contradiction by showing that the \emph{right-hand side} of (\ref{eq:WTT - Target of attack}) is $>\epsilon$. Since $\left\Vert \cdot\right\Vert _{p^{M},q}$ is a non-archimedean norm, it satisfies the ultrametric inequality. Applying this to the right-hand side of (\ref{eq:WTT - Target of attack}) yields: \begin{equation} \left\Vert \hat{\phi}_{N}-\left(\hat{\phi}_{N}*\hat{\mu}*\hat{\eta}_{\epsilon,M}\right)\right\Vert _{p^{M},q}\leq\max\left\{ \left\Vert \hat{\phi}_{N}\right\Vert _{p^{M},q},\left\Vert \hat{\phi}_{N}*\hat{\mu}*\hat{\eta}_{\epsilon,M}\right\Vert _{p^{M},q}\right\} \label{eq:WTT - Ultrametric inequality} \end{equation} Because $\hat{\eta}_{\epsilon,M}\left(t\right)$ and $\hat{\phi}_{N}$ are supported on $\left|t\right|_{p}\leq p^{M}$ and $\left|t\right|_{p}\leq p^{N}$, respectively, we can apply (\ref{eq:phi_N hat convolve f hat convolve g hat estimate}) from \textbf{Lemma \ref{lem:three convolution estimate}} and write: \begin{equation} \left\Vert \hat{\phi}_{N}*\hat{\mu}*\hat{\eta}_{\epsilon,M}\right\Vert _{p^{M},q}\leq\left\Vert \hat{\phi}_{N}*\hat{\mu}\right\Vert _{p^{\max\left\{ M,N\right\} },q}\cdot\left\Vert \hat{\eta}_{\epsilon}\right\Vert _{p^{\infty},q}\label{eq:Ready for epsilon prime} \end{equation} Since $\hat{\eta}_{\epsilon}$ was given to \emph{not }be identically zero, the quantity $\left\Vert \hat{\eta}_{\epsilon}\right\Vert _{p^{\infty},q}$ must be positive. Consequently, for our given $\epsilon\in\left(0,1\right)$, let us define $\epsilon^{\prime}$ by: \begin{equation} \epsilon^{\prime}=\frac{\epsilon}{2\left\Vert \hat{\eta}_{\epsilon}\right\Vert _{p^{\infty},q}} \end{equation} So far, $N$ is still arbitrary. By \textbf{Claim \ref{claim:p^m, q norm of convolution of mu-hat and phi_N hat }}, for this $\epsilon^{\prime}$, we know there exists an $N_{\epsilon^{\prime}}$ (depending only on $\epsilon$, $\hat{\eta}_{\epsilon}$, and $\hat{\mu}$) so that:\textbf{ \begin{equation} \left\Vert \hat{\phi}_{N}*\hat{\mu}\right\Vert _{p^{m},q}<\epsilon^{\prime},\textrm{ }\forall m\geq1,\textrm{ }\forall N\geq\max\left\{ N_{\epsilon^{\prime}},m\right\} \end{equation} }Choosing $m=N_{\epsilon^{\prime}}$ gives us: \begin{equation} N\geq N_{\epsilon^{\prime}}\Rightarrow\left\Vert \hat{\phi}_{N}*\hat{\mu}\right\Vert _{p^{N},q}<\epsilon^{\prime} \end{equation} So, choose $N\geq\max\left\{ N_{\epsilon^{\prime}},M\right\} $. Then, $\max\left\{ M,N\right\} =N$, and so (\ref{eq:Ready for epsilon prime}) becomes: \begin{equation} \left\Vert \hat{\phi}_{N}*\hat{\mu}*\hat{\eta}_{\epsilon,M}\right\Vert _{p^{M},q}\leq\left\Vert \hat{\phi}_{N}*\hat{\mu}\right\Vert _{p^{N},q}\cdot\left\Vert \hat{\eta}_{\epsilon}\right\Vert _{p^{\infty},q}<\frac{\epsilon}{2\left\Vert \hat{\eta}_{\epsilon}\right\Vert _{p^{\infty},q}}\cdot\left\Vert \hat{\eta}_{\epsilon}\right\Vert _{p^{\infty},q}=\frac{\epsilon}{2} \end{equation} Since $\left\Vert \hat{\phi}_{N}\right\Vert _{p^{M},q}=1$ for all $M,N\geq0$, this shows that: \begin{equation} \left\Vert \hat{\phi}_{N}*\hat{\mu}*\hat{\eta}_{\epsilon,M}\right\Vert _{p^{M},q}<\frac{\epsilon}{2}<1=\left\Vert \hat{\phi}_{N}\right\Vert _{p^{M},q} \end{equation} Now comes the hammer: by the ultrametric inequality, (\ref{eq:WTT - Ultrametric inequality}) is an \emph{equality }whenever one of $\left\Vert \hat{\phi}_{N}\right\Vert _{p^{M},q}$ or $\left\Vert \hat{\phi}_{N}*\hat{\mu}*\hat{\eta}_{\epsilon,M}\right\Vert _{p^{M},q}$ is strictly greater than the other. Having proved that to be the case, (\ref{eq:WTT - Target of attack}) becomes: \begin{equation} \epsilon>\left\Vert \hat{\phi}_{N}*\left(\mathbf{1}_{0}-\left(\hat{\mu}*\hat{\eta}_{\epsilon,M}\right)\right)\right\Vert _{p^{M},q}=\left\Vert \hat{\phi}_{N}-\left(\hat{\phi}_{N}*\hat{\mu}*\hat{\eta}_{\epsilon,M}\right)\right\Vert _{p^{M},q}=1>\epsilon \end{equation} for all $M\geq M_{\epsilon}$ and all $N\geq\max\left\{ N_{\epsilon}^{\prime},M\right\} $. The left-hand side is our first estimate, and the right-hand side is our second. Since $\epsilon<1$, this is clearly impossible. This proves that the existence of the zero $\mathfrak{z}_{0}$ precludes the translates of $\hat{\mu}$ from being dense in $c_{0}\left(\hat{\mathbb{Z}}_{p},\mathbb{C}_{q}\right)$. Q.E.D. \subsubsection{A Matter of \label{subsec:A-Matter-of}Matrices} Along with the \textbf{Correspondence Principle }and the derivation of a Formula for the Fourier transforms of the $\chi_{H}$s, \textbf{Theorem \ref{thm:pq WTT for measures}} (and its multi-dimensional analogue in Subsection \ref{subsec:5.4.4More-Fourier-Resummation}) is one of this dissertation's central results. Combined, these three theorems can (and will) be used to show that an integer $x$ is a periodic point of a contracting, semi-basic $p$-Hydra map $H$ if and only if the translates of $\hat{\chi}_{H}\left(t\right)-x\mathbf{1}_{0}\left(t\right)$ are dense in $c_{0}\left(\mathbb{Z}_{p},\mathbb{C}_{q_{H}}\right)$. Aside from the mere aesthetic gratification of this equivalence, it also appears to be fertile ground for future investigations into the periodic points of Hydra maps, thanks to the algebraic structure of $\hat{\mathbb{Z}}_{p}$ and a spoonful of linear algebra. Identifying the group $\mathbb{Z}/p^{N}\mathbb{Z}$ with the set: \begin{equation} \left\{ 0,\frac{1}{p^{N}},\frac{2}{p^{N}}\ldots,\frac{p^{N}-1}{p^{N}}\right\} \end{equation} equipped with addition modulo $1$, it is a standard fact of higher algebra that $\hat{\mathbb{Z}}_{p}$ is the so-called \index{direct limit}\textbf{direct limit }of the $\mathbb{Z}/p^{N}\mathbb{Z}$s. In terms of functions, this is just the fact that any function $\hat{\mathbb{Z}}_{p}\rightarrow\mathbb{C}_{q}$ can be restricted to a function $\mathbb{Z}/p^{N}\mathbb{Z}\rightarrow\mathbb{C}_{q}$ by multiplication by $\mathbf{1}_{0}\left(p^{N}t\right)$. This makes it feasible to study functions $\hat{\mathbb{Z}}_{p}\rightarrow\mathbb{C}_{q}$ by considering functions $\mathbb{Z}/p^{N}\mathbb{Z}\rightarrow\mathbb{C}_{q}$ as $N\rightarrow\infty$. This is particularly nice, seeing as the question of dense translations over $\mathbb{Z}/p^{N}\mathbb{Z}$ reduces to a matter of matrices. \begin{defn} \label{def:Conv inv}Let $\hat{\chi}:\hat{\mathbb{Z}}_{p}\rightarrow\mathbb{C}_{q}$. Then, a \index{convolution!inverse}\textbf{convolution inverse }of $\hat{\chi}$ is a function $\hat{\chi}^{-1}:\hat{\mathbb{Z}}_{p}\rightarrow\mathbb{C}_{q}$ so that: \begin{equation} \underbrace{\lim_{N\rightarrow\infty}\sum_{\left|s\right|_{p}\leq p^{N}}\hat{\chi}\left(t-s\right)\hat{\chi}^{-1}\left(s\right)}_{\overset{\textrm{def}}{=}\left(\hat{\chi}*\hat{\chi}^{-1}\right)\left(t\right)}\overset{\mathbb{C}_{q}}{=}\mathbf{1}_{0}\left(t\right),\textrm{ }\forall t\in\hat{\mathbb{Z}}_{p}\label{eq:Definition of Convolution Inverse} \end{equation} Note that $\mathbf{1}_{0}$ is the identity element with respect to convolution of functions $\hat{\mathbb{Z}}_{p}\rightarrow\mathbb{C}_{q}$. Given $N\geq0$, we say that $\hat{\chi}^{-1}$ is an \textbf{$N$th partial convolution inverse }of $\hat{\chi}$ if: \begin{equation} \sum_{\left|s\right|_{p}\leq p^{N}}\hat{\chi}\left(t-s\right)\hat{\chi}^{-1}\left(s\right)\overset{\mathbb{C}_{q}}{=}\mathbf{1}_{0}\left(t\right),\textrm{ }\forall\left|t\right|_{p}\leq p^{N}\label{eq:Definition of Nth Partial Convolution Inverse} \end{equation} \end{defn} \begin{prop} Let $\hat{\chi}:\hat{\mathbb{Z}}_{p}\rightarrow\mathbb{C}_{q}$, and suppose there is a function $\hat{\chi}^{-1}:\hat{\mathbb{Z}}_{p}\rightarrow\mathbb{C}_{q}$ which is an $N$th partial convolution inverse of $\hat{\chi}$ for all sufficiently large $N$. Then, $\hat{\chi}^{-1}$ is a convolution inverse of $\hat{\chi}$. \end{prop} Proof: Take limits of (\ref{eq:Definition of Nth Partial Convolution Inverse}) as $N\rightarrow\infty$. (Note: the convergence is only guaranteed to be point-wise with respect to $t$.) Q.E.D. \vphantom{} Now, given any $N$, observe that the equation (\ref{eq:Definition of Nth Partial Convolution Inverse}) defining an $N$th partial convolution inverse of $\hat{\chi}$ is actually a system of linear equations with the values of $\hat{\chi}^{-1}\left(s\right)$ on $\left|s\right|_{p}\leq p^{N}$ as its unknowns: \begin{align*} 1 & =\sum_{\left|s\right|_{p}\leq p^{N}}\hat{\chi}\left(0-s\right)\hat{\chi}^{-1}\left(s\right)\\ 0 & =\sum_{\left|s\right|_{p}\leq p^{N}}\hat{\chi}\left(\frac{1}{p^{N}}-s\right)\hat{\chi}^{-1}\left(s\right)\\ 0 & =\sum_{\left|s\right|_{p}\leq p^{N}}\hat{\chi}\left(\frac{2}{p^{N}}-s\right)\hat{\chi}^{-1}\left(s\right)\\ & \vdots\\ 0 & =\sum_{\left|s\right|_{p}\leq p^{N}}\hat{\chi}\left(\frac{p^{N}-1}{p^{N}}-s\right)\hat{\chi}^{-1}\left(s\right) \end{align*} We can express this linear system as a matrix. In fact, we can do so in two noteworthy ways. \begin{defn} Let $\hat{\chi}:\hat{\mathbb{Z}}_{p}\rightarrow\mathbb{C}_{q}$ and let $N\geq1$. \vphantom{} I. We write $\mathbf{M}_{N}\left(\hat{\chi}\right)$ to denote the $p^{N}\times p^{N}$matrix: \begin{equation} \mathbf{M}_{N}\left(\hat{\chi}\right)\overset{\textrm{def}}{=}\left(\begin{array}{ccccc} \hat{\chi}\left(0\right) & \hat{\chi}\left(\frac{p^{N}-1}{p^{N}}\right) & \cdots & \hat{\chi}\left(\frac{2}{p^{N}}\right) & \hat{\chi}\left(\frac{1}{p^{N}}\right)\\ \hat{\chi}\left(\frac{1}{p^{N}}\right) & \hat{\chi}\left(0\right) & \hat{\chi}\left(\frac{p^{N}-1}{p^{N}}\right) & & \hat{\chi}\left(\frac{2}{p^{N}}\right)\\ \vdots & \hat{\chi}\left(\frac{1}{p^{N}}\right) & \hat{\chi}\left(0\right) & \ddots & \vdots\\ \hat{\chi}\left(\frac{p^{N}-2}{p^{N}}\right) & & \ddots & \ddots & \hat{\chi}\left(\frac{p^{N}-1}{p^{N}}\right)\\ \hat{\chi}\left(\frac{p^{N}-1}{p^{N}}\right) & \hat{\chi}\left(\frac{p^{N}-2}{p^{N}}\right) & \cdots & \hat{\chi}\left(\frac{1}{p^{N}}\right) & \hat{\chi}\left(0\right) \end{array}\right)\label{eq:Definition of bold M_N of Chi hat} \end{equation} \vphantom{} II. By the \textbf{radial enumeration of $\mathbb{Z}/p^{N}\mathbb{Z}$}, we mean the sequence: \begin{equation} 0,\frac{1}{p},\ldots,\frac{p-1}{p},\frac{1}{p^{2}},\ldots,\frac{p-1}{p^{2}},\frac{p+1}{p^{2}},\ldots,\frac{p^{2}-1}{p^{2}},\frac{1}{p^{3}},\ldots,\frac{p^{N}-1}{p^{N}}\label{eq:Definition of the radial enumeration of Z mod p^N Z} \end{equation} that is to say, the radial enumeration of $\mathbb{Z}/p^{N}\mathbb{Z}$ starts with $0$, which is then followed by the list of all $t\in\hat{\mathbb{Z}}_{p}$ with $\left|t\right|_{p}=p$ in order of increasing value in the numerator, which is then followed by the list of all $t\in\hat{\mathbb{Z}}_{p}$ with $\left|t\right|_{p}=p^{2}$ in order of increasing value in the numerator, so on and so forth, until we have listed all $t$ with $\left|t\right|_{p}\leq p^{N}$. By the \textbf{standard enumeration of }$\mathbb{Z}/p^{N}\mathbb{Z}$, on the other hand, we mean the enumeration: \begin{equation} 0,\frac{1}{p^{N}},\ldots,\frac{p^{N}-1}{p^{N}}\label{eq:Definition of the standard enumeration of Z mod p^N Z} \end{equation} \vphantom{} III. We write $\mathbf{R}_{N}\left(\hat{\chi}\right)$ to denote the $p^{N}\times p^{N}$ matrix: \[ \mathbf{R}_{N}\left(\hat{\chi}\right)\overset{\textrm{def}}{=}\left(\begin{array}{cccccc} \hat{\chi}\left(0-0\right) & \hat{\chi}\left(0-\frac{1}{p}\right) & \cdots & \hat{\chi}\left(0-\frac{p-1}{p}\right) & \cdots & \hat{\chi}\left(0-\frac{p^{N}-1}{p^{N}}\right)\\ \hat{\chi}\left(\frac{1}{p}-0\right) & \hat{\chi}\left(0\right) & \cdots & \hat{\chi}\left(\frac{1}{p}-\frac{p-1}{p}\right) & \cdots & \hat{\chi}\left(\frac{1}{p}-\frac{p^{N}-1}{p^{N}}\right)\\ \vdots & \vdots & \ddots & \vdots & & \vdots\\ \hat{\chi}\left(\frac{p-1}{p}-0\right) & \hat{\chi}\left(\frac{p-1}{p}-\frac{1}{p}\right) & \cdots & \hat{\chi}\left(0\right) & \cdots & \hat{\chi}\left(\frac{p-1}{p}-\frac{p^{N}-1}{p^{N}}\right)\\ \vdots & \vdots & & \vdots & \ddots & \vdots\\ \hat{\chi}\left(\frac{p^{N}-1}{p^{N}}-0\right) & \hat{\chi}\left(\frac{p^{N}-1}{p^{N}}-\frac{1}{p}\right) & \cdots & \hat{\chi}\left(\frac{p^{N}-1}{p^{N}}-\frac{p-1}{p}\right) & \cdots & \hat{\chi}\left(0\right) \end{array}\right) \] That is, the entries of the $n$th row of $\mathbf{R}_{N}\left(\hat{\chi}\right)$, where $n\in\left\{ 0,\ldots,p^{N}-1\right\} $ are: \[ \hat{\chi}\left(t_{n}-s_{0}\right),\hat{\chi}\left(t_{n}-s_{1}\right),\hat{\chi}\left(t_{n}-s_{2}\right)\ldots \] where $t_{n}$ and $s_{k}$ are the $n$th and $k$th elements of the radial enumeration of $\mathbb{Z}/p^{N}\mathbb{Z}$, respectively ($s_{0}=t_{0}=0$, $s_{1}=t_{1}=1/p$, $s_{2}=t_{2}=2/p$, etc.). \end{defn} \begin{rem} If $\hat{\chi}$ is a Fourier transform of some function $\chi:\mathbb{Z}_{p}\rightarrow\mathbb{C}_{q}$, we will also write $\mathbf{M}_{N}\left(\chi\right)$ and $\mathbf{R}_{N}\left(\chi\right)$ to denote $\mathbf{M}_{N}\left(\hat{\chi}\right)$ and $\mathbf{R}_{N}\left(\hat{\chi}\right)$, respectively. If $\chi$ and/or $\hat{\chi}$ are not in question, we will just write $\mathbf{M}_{N}$ and $\mathbf{R}_{N}$. \end{rem} \begin{rem} As defined, we have that (\ref{eq:Definition of Nth Partial Convolution Inverse}) can be written in matrix form as: \begin{equation} \mathbf{M}_{N}\left(\hat{\chi}\right)\left(\begin{array}{c} \hat{\chi}^{-1}\left(0\right)\\ \hat{\chi}^{-1}\left(\frac{1}{p^{N}}\right)\\ \vdots\\ \hat{\chi}^{-1}\left(\frac{p^{N}-1}{p^{N}}\right) \end{array}\right)=\underbrace{\left(\begin{array}{c} 1\\ 0\\ \vdots\\ 0 \end{array}\right)}_{\mathbf{e}_{1}}\label{eq:Matrix Equation - Circulant Form} \end{equation} and as: \begin{equation} \mathbf{R}_{N}\left(\hat{\chi}\right)\left(\begin{array}{c} \hat{\chi}^{-1}\left(0\right)\\ \hat{\chi}^{-1}\left(\frac{1}{p}\right)\\ \vdots\\ \hat{\chi}^{-1}\left(\frac{p-1}{p}\right)\\ \vdots\\ \hat{\chi}^{-1}\left(\frac{p^{N}-1}{p^{N}}\right) \end{array}\right)=\mathbf{e}_{1}\label{eq:Matrix Equation - Radial Form} \end{equation} where the column of $\hat{\chi}^{-1}$ is listed according to the radial enumeration of $\hat{\mathbb{Z}}_{p}$. \end{rem} \begin{prop} \label{prop:permutation similarity}For each $N$ for which at least one of $\mathbf{R}_{N}\left(\hat{\chi}\right)$ and $\mathbf{M}_{N}\left(\hat{\chi}\right)$ is invertible, there exists a permutation matrix $\mathbf{E}_{N}$ (depending only on $N$ and $p$) so that: \begin{equation} \mathbf{R}_{N}\left(\hat{\chi}\right)=\mathbf{E}_{N}\mathbf{M}_{N}\left(\hat{\chi}\right)\mathbf{E}_{N}^{-1}\label{eq:Permutation Similarity of M and R} \end{equation} That is to say, $\mathbf{M}_{N}\left(\hat{\chi}\right)$ and $\mathbf{R}_{N}\left(\hat{\chi}\right)$ are \textbf{permutation similar}. \end{prop} Proof: Let $\mathbf{E}_{N}$ be the permutation matrix which sends the standard enumeration of $\mathbb{Z}/p^{N}\mathbb{Z}$ to the radial enumeration of $\mathbb{Z}/p^{N}\mathbb{Z}$: \begin{equation} \mathbf{E}_{N}\left(\begin{array}{c} 0\\ \frac{1}{p^{N}}\\ \vdots\\ \frac{p^{N}-1}{p^{N}} \end{array}\right)=\left(\begin{array}{c} 0\\ \frac{1}{p}\\ \vdots\\ \frac{p-1}{p}\\ \vdots\\ \frac{p^{N}-1}{p^{N}} \end{array}\right)\label{eq:Definition of bold E_N} \end{equation} Consequently, $\mathbf{E}_{N}^{-1}$ sends the radial enumeration to the standard enumeration. Now, without loss of generality, suppose $\mathbf{M}_{N}\left(\hat{\chi}\right)$ is invertible. The proof for when $\mathbf{E}_{N}\left(\hat{\chi}\right)$ is nearly identical, just replace every instance of $\mathbf{E}_{N}^{-1}$ below with $\mathbf{E}_{N}$, and vice-versa, and replace every standard enumeration with the radial enumeration, and vice-versa. Then, taking (\ref{eq:Matrix Equation - Radial Form}), letting $\mathbf{x}$ denote the column vector with $\hat{\chi}^{-1}$ in the standard enumeration, and letting $\mathbf{y}$ denote the column vector with $\hat{\chi}^{-1}$ in the radial enumeration, observe that: \begin{equation} \mathbf{R}_{N}\left(\hat{\chi}\right)\mathbf{y}=\mathbf{R}_{N}\left(\hat{\chi}\right)\mathbf{E}_{N}\mathbf{x}=\mathbf{e}_{1} \end{equation} Left-multiplying both sides by $\mathbf{E}_{N}^{-1}$ yields: \begin{equation} \mathbf{E}_{N}^{-1}\mathbf{R}_{N}\left(\hat{\chi}\right)\mathbf{E}_{N}\mathbf{x}=\mathbf{E}_{N}^{-1}\mathbf{e}_{1}=\mathbf{e}_{1} \end{equation} where the right-most equality follows from the fact that the $1$ at the top of $\mathbf{e}_{1}$ is fixed by the permutation encoded by $\mathbf{E}_{N}^{-1}$. Since $\mathbf{M}_{N}\left(\hat{\chi}\right)$ was assumed to be invertible, (\ref{eq:Matrix Equation - Circulant Form}) shows that $\mathbf{M}_{N}\left(\hat{\chi}\right)$ is the \emph{unique }matrix so that: \[ \mathbf{M}_{N}\left(\hat{\chi}\right)\mathbf{x}=\mathbf{e}_{1} \] The above shows that $\left(\mathbf{E}_{N}^{-1}\mathbf{R}_{N}\left(\hat{\chi}\right)\mathbf{E}_{N}\right)\mathbf{x}=\mathbf{e}_{1}$, which then forces $\mathbf{E}_{N}^{-1}\mathbf{R}_{N}\left(\hat{\chi}\right)\mathbf{E}_{N}=\mathbf{M}_{N}\left(\hat{\chi}\right)$, and hence, (\ref{eq:Permutation Similarity of M and R}). Q.E.D. \vphantom{} Observe that $\mathbf{M}_{N}\left(\hat{\chi}\right)$ is a matrix of the form: \begin{equation} \left(\begin{array}{ccccc} c_{0} & c_{p^{N}-1} & \cdots & c_{2} & c_{1}\\ c_{1} & c_{0} & c_{p^{N}-1} & & c_{2}\\ \vdots & c_{1} & c_{0} & \ddots & \vdots\\ c_{p^{N}-2} & & \ddots & \ddots & c_{p^{N}-1}\\ c_{p^{N}-1} & c_{p^{N}-2} & \cdots & c_{1} & c_{0} \end{array}\right)\label{eq:Bold M_N as a p^N by p^N Circulant Matrix} \end{equation} Our analysis is greatly facilitated by the elegant properties of properties satisfied by matrices of the form (\ref{eq:Bold M_N as a p^N by p^N Circulant Matrix}), which are called \textbf{circulant matrices}\index{circulant matrix}. A standard reference for this subject is \cite{Circulant Matrices}. \begin{defn}[\textbf{Circulant matrices}] Let $\mathbb{F}$ be an algebraically closed field of characteristic $0$, and let $N$ be an integer $\geq1$. Then, an\textbf{ $N\times N$ circulant matrix with entries in $\mathbb{F}$} is a matrix $\mathbf{M}$ of the form: \begin{equation} \left(\begin{array}{ccccc} c_{0} & c_{N-1} & \cdots & c_{2} & c_{1}\\ c_{1} & c_{0} & c_{N-1} & & c_{2}\\ \vdots & c_{1} & c_{0} & \ddots & \vdots\\ c_{N-2} & & \ddots & \ddots & c_{N-1}\\ c_{N-1} & c_{N-2} & \cdots & c_{1} & c_{0} \end{array}\right)\label{eq:Definition of a Circulant Matrix-1} \end{equation} where $c_{0},\ldots,c_{N-1}\in\mathbb{F}$. The polynomial $f_{\mathbf{M}}:\mathbb{F}\rightarrow\mathbb{F}$ defined by: \begin{equation} f_{\mathbf{M}}\left(x\right)\overset{\textrm{def}}{=}\sum_{n=0}^{N-1}c_{n}x^{n}\label{eq:Definition of the associated polynomial of a circulant matrix} \end{equation} is called the \textbf{associated polynomial}\index{circulant matrix!associated polynomial}\textbf{ }of the matrix $\mathbf{M}$. \end{defn} \begin{rem} $\mathbf{M}_{N}\left(\hat{\chi}\right)$ is a circulant matrix for all $\hat{\chi}:\hat{\mathbb{Z}}_{p}\rightarrow\mathbb{C}_{q}$. \end{rem} \begin{defn} Let $\mathbb{F}$ be a field of characteristic $0$. For a function $\hat{\chi}:\hat{\mathbb{Z}}_{p}\rightarrow\mathbb{F}$, we then write $f_{\hat{\chi},N}:\mathbb{F}\rightarrow\mathbb{F}$ to denote the associated polynomial of $\mathbf{M}_{N}\left(\hat{\chi}\right)$: \begin{equation} f_{\hat{\chi},N}\left(z\right)=\sum_{n=0}^{p^{N}-1}\hat{\chi}\left(\frac{n}{p^{N}}\right)z^{n}\label{eq:Definition/Notation for the associated polynomial of bold M_N of Chi hat} \end{equation} \end{defn} \begin{rem} Like with $\mathbf{M}_{N}$ and $\mathbf{R}_{N}$, if $\hat{\chi}$ is a Fourier transform of some function $\chi$, we will write $f_{\chi,N}$ to denote $f_{\hat{\chi},N}$. \end{rem} \vphantom{} The chief properties of circulant matrices stem from their intimate connection with convolution inverses and the finite Fourier transform. \begin{thm} \label{thm:3.52}Let $\mathbf{M}$ be an $N\times N$ circulant matrix\index{circulant matrix!determinant} with coefficients in an algebraically closed field $\mathbb{F}$, and let $\omega\in\mathbb{F}$ be any primitive $N$th root of unity. Then: \vphantom{} I. \begin{equation} \det\mathbf{M}=\prod_{n=0}^{N-1}f_{\mathbf{M}}\left(\omega^{n}\right)\label{eq:Determinant of a Circulant Matrix} \end{equation} \vphantom{} II. The\index{circulant matrix!eigenvalues} eigenvalues of $\mathbf{M}$ are $f_{\mathbf{M}}\left(\omega^{n}\right)$ for $n\in\left\{ 0,\ldots,N-1\right\} $. \end{thm} \vphantom{} We employ this result to compute the characteristic polynomial of $\mathbf{M}_{N}\left(\hat{\chi}\right)$ for any $\hat{\chi}$. \begin{prop} \label{prop:3.71}Let $\hat{\chi}:\hat{\mathbb{Z}}_{p}\rightarrow\mathbb{C}_{q}$ be any function. Then: \begin{equation} \det\left(\mathbf{M}_{N}\left(\hat{\chi}\right)-\lambda\mathbf{I}_{p^{N}}\right)\overset{\mathbb{C}_{q}}{=}\prod_{n=0}^{p^{N}-1}\left(\tilde{\chi}_{N}\left(n\right)-\lambda\right),\textrm{ }\forall\lambda\in\mathbb{C}_{q}\label{eq:Characteristic polynomial of bold M_N of Chi hat} \end{equation} where, $\mathbf{I}_{p^{N}}$ is a $p^{N}\times p^{N}$ identity matrix, and where, as usual, $\tilde{\chi}_{N}\left(\mathfrak{z}\right)=\sum_{\left|t\right|_{p}\leq p^{N}}\hat{\chi}\left(t\right)e^{2\pi i\left\{ t\mathfrak{z}\right\} _{p}}$. \end{prop} Proof: First, observe that: \begin{equation} \mathbf{M}_{N}\left(\hat{\chi}\right)-\lambda\mathbf{I}_{p^{N}}=\mathbf{M}_{N}\left(\hat{\chi}-\lambda\mathbf{1}_{0}\right) \end{equation} where: \begin{equation} \hat{\chi}\left(t\right)-\lambda\mathbf{1}_{0}\left(t\right)=\begin{cases} \hat{\chi}\left(0\right)-\lambda & \textrm{if }t=0\\ \hat{\chi}\left(t\right) & \textrm{else} \end{cases},\textrm{ }\forall t\in\hat{\mathbb{Z}}_{p} \end{equation} Here, $\mathbf{M}_{N}\left(\hat{\chi}-\lambda\mathbf{1}_{0}\right)$ is a circulant matrix with the associated polynomial: \begin{align*} f_{\hat{\chi}-\lambda\mathbf{1}_{0},N}\left(\mathfrak{z}\right) & =\sum_{n=0}^{p^{N}-1}\left(\hat{\chi}\left(\frac{n}{p^{N}}\right)-\lambda\mathbf{1}_{0}\left(\frac{n}{p^{N}}\right)\right)\mathfrak{z}^{n}\\ \left(\mathbf{1}_{0}\left(\frac{n}{p^{N}}\right)=\begin{cases} 1 & \textrm{if }n=0\\ 0 & \textrm{else} \end{cases}\right); & =-\lambda+\sum_{n=0}^{p^{N}-1}\hat{\chi}\left(\frac{n}{p^{N}}\right)\mathfrak{z}^{n}\\ & =f_{\hat{\chi},N}\left(\mathfrak{z}\right)-\lambda \end{align*} By (\ref{eq:Determinant of a Circulant Matrix}), since $\mathbf{M}_{N}\left(\hat{\chi}-\lambda\mathbf{1}_{0}\right)=\mathbf{M}_{N}\left(\hat{\chi}\right)-\lambda\mathbf{I}_{p^{N}}$ is an $p^{N}\times p^{N}$ circulant matrix, we have that: \begin{align*} \det\left(\mathbf{M}_{N}\left(\hat{\chi}\right)-\lambda\mathbf{I}_{p^{N}}\right) & \overset{\mathbb{C}_{q}}{=}\prod_{n=0}^{p^{N}-1}\left(f_{\hat{\chi},N}\left(e^{2\pi in/p^{N}}\right)-\lambda\right)\\ & \overset{\mathbb{C}_{q}}{=}\prod_{n=0}^{p^{N}-1}\left(\sum_{k=0}^{p^{N}-1}\hat{\chi}\left(\frac{k}{p^{N}}\right)e^{\frac{2\pi ikn}{p^{N}}}-\lambda\right)\\ & \overset{\mathbb{C}_{q}}{=}\prod_{n=0}^{p^{N}-1}\left(\sum_{\left|t\right|_{p}\leq p^{N}}\hat{\chi}\left(t\right)e^{2\pi itn}-\lambda\right)\\ \left(e^{2\pi itn}=e^{2\pi i\left\{ tn\right\} _{p}}\right); & =\prod_{n=0}^{p^{N}-1}\left(\tilde{\chi}_{N}\left(n\right)-\lambda\right) \end{align*} Q.E.D. \begin{thm}[\textbf{$\chi_{H}$ as an eigenvalue problem}] \label{thm:Eigenvalue problem}Let $\hat{\chi}:\hat{\mathbb{Z}}_{p}\rightarrow\mathbb{C}_{q}$ be any function. Then: \vphantom{} I. For any $\lambda\in\mathbb{C}_{q}$, $\mathbf{M}_{N}-\lambda\mathbf{I}_{p^{N}}$ is invertible if and only if $\tilde{\chi}_{N}\left(n\right)\neq\lambda$ for any $n\in\left\{ 0,\ldots,p^{N}-1\right\} $. \vphantom{} II. The eigenvalues\index{matrix!eigenvalues} of $\mathbf{M}_{N}$ are $\left\{ \tilde{\chi}_{N}\left(n\right)\right\} _{0\leq n\leq p^{N}-1}$. \vphantom{} III. Let $N\geq1$ satisfy $\tilde{\chi}_{N}\left(n\right)\neq0$ for any $n\in\left\{ 0,\ldots,p^{N}-1\right\} $. Then $\mathbf{M}_{N}$ and $\mathbf{R}_{N}$ are permutation similar, and (\ref{eq:Permutation Similarity of M and R}) holds true. \end{thm} Proof: Use \textbf{Proposition \ref{prop:permutation similarity}}, \textbf{Theorem \ref{thm:3.52}}, and \textbf{Proposition \ref{prop:3.71}}. Q.E.D. \chapter{\label{chap:4 A-Study-of}A Study of $\chi_{H}$ - The One-Dimensional Case} \includegraphics[scale=0.45]{./PhDDissertationEroica4.png} \vphantom{} THROUGHOUT THIS CHAPTER, UNLESS STATED OTHERWISE, WE ASSUME $p$ IS A PRIME NUMBER, AND THAT $H$ IS A CONTRACTING, SEMI-BASIC $p$-HYDRA MAP WHICH FIXES $0$. \vphantom{} Assuming the reader hasn't already figured it out, the musical epigrams accompanying each of this dissertation's chapters are excerpts from the fourth movement of Beethoven's Symphony No. 3\textemdash the \emph{Eroica}. The excerpt for Chapter 4 is from one of the movement's fugal sections. For the uninitiated, a fugue is a form of western classical composition based around strict, rigorous, academic applications of imitative counterpoint among multiple interacting musical voices, and it is the perfect analogy for the work we are about to undertake. It is this chapter that we will finally take the techniques recounted or developed in Chapter 3 and deploy them to analyze $\chi_{H}$. The main goal is to obtain a meaningful formula for a Fourier transform ($\hat{\chi}_{H}$) of $\chi_{H}$, and thereby show that $\chi_{H}$ is quasi-integrable. With a formula for $\hat{\chi}_{H}$, we can utilize the Correspondence Principle in conjunction with our $\left(p,q\right)$-adic generalization of Wiener's Tauberian Theorem (WTT) to reformulate questions about periodic points and divergent points of $H$ into Fourier-analytic questions about $\hat{\chi}_{H}$. Roughly speaking, the equivalences are as follows: \begin{eqnarray*} & x\in\mathbb{Z}\backslash\left\{ 0\right\} \textrm{ is a periodic point or divergent point of }H\\ & \Updownarrow\\ & \chi_{H}\left(\mathfrak{z}\right)-x\textrm{ vanishes at some }\mathfrak{z}\in\mathbb{Z}_{p}^{\prime}\\ & \Updownarrow\\ & \textrm{translates of }\hat{\chi}_{H}\left(t\right)-x\mathbf{1}_{0}\left(t\right)\textrm{ are not dense in }c_{0}\left(\hat{\mathbb{Z}}_{p},\mathbb{C}_{q}\right) \end{eqnarray*} where the upper $\Updownarrow$ is the Correspondence Principle and the lower $\Updownarrow$ is the WTT. I conjecture that \emph{both} of the $\Updownarrow$s are, in fact, rigorous if-and-only-if equivalences. At present, however, each $\Updownarrow$ is only three-quarters true, and requires that $H$ satisfy the hypotheses of \textbf{Theorem \ref{thm:Divergent trajectories come from irrational z}} (page \pageref{thm:Divergent trajectories come from irrational z}). For such an $H$, the actual implications of the Correspondence Principal are: \begin{eqnarray*} & x\in\mathbb{Z}\backslash\left\{ 0\right\} \textrm{ is a periodic point of }H\\ & \Updownarrow\\ & \chi_{H}\left(\mathfrak{z}\right)-x\textrm{ vanishes at some }\mathfrak{z}\in\mathbb{Q}\cap\mathbb{Z}_{p}^{\prime} \end{eqnarray*} and: \begin{eqnarray*} & x\in\mathbb{Z}\backslash\left\{ 0\right\} \textrm{ is a divergent point of }H\\ & \Uparrow\\ & \chi_{H}\left(\mathfrak{z}\right)-x\textrm{ vanishes at some }\mathfrak{z}\in\mathbb{Z}_{p}\backslash\mathbb{Q}_{p} \end{eqnarray*} With the WTT and the help of a a condition on $H$ called \textbf{non-singularity }we will prove the \textbf{Tauberian Spectral Theorem }for $\chi_{H}$ (\textbf{Theorem \ref{thm:Periodic Points using WTT}}, page \pageref{thm:Periodic Points using WTT}) then yields: \begin{eqnarray*} & \chi_{H}\left(\mathfrak{z}\right)-x\textrm{ vanishes at some }\mathfrak{z}\in\mathbb{Z}_{p}^{\prime}\\ & \Updownarrow\\ & \textrm{translates of }\hat{\chi}_{H}\left(t\right)-x\mathbf{1}_{0}\left(t\right)\textrm{ are not dense in }c_{0}\left(\hat{\mathbb{Z}}_{p},\mathbb{C}_{q}\right) \end{eqnarray*} To keep the computations in this chapter as simple and manageable as possible, Section \ref{sec:4.1 Preparatory-Work--} contains some additional notational conventions (definitions of functions, constants, etc.) which will streamline the presentation. Section \ref{sec:4.1 Preparatory-Work--} also includes some identities and asymptotic estimates which will be used throughout the rest of the chapter. \section{\label{sec:4.1 Preparatory-Work--}Preparatory Work \textendash{} Conventions, Identities, etc.} We begin by introducing some new notations. \begin{defn} \label{def:alpha, beta, gamma 1D}We define \nomenclature{$\alpha_{H}\left(t\right)$}{ }\nomenclature{$\beta_{H}\left(t\right)$}{ }$\alpha_{H},\beta_{H}:\hat{\mathbb{Z}}_{p}\rightarrow\overline{\mathbb{Q}}$ by: \begin{equation} \alpha_{H}\left(t\right)\overset{\textrm{def}}{=}\frac{1}{p}\sum_{j=0}^{p-1}H_{j}^{\prime}\left(0\right)e^{-2\pi ijt}=\frac{1}{p}\sum_{j=0}^{p-1}\frac{\mu_{j}}{p}e^{-2\pi ijt}=\frac{1}{p}\sum_{j=0}^{p-1}\frac{a_{j}}{d_{j}}e^{-2\pi ijt}\label{eq:Definition of alpha_H} \end{equation} \begin{equation} \beta_{H}\left(t\right)\overset{\textrm{def}}{=}\frac{1}{p}\sum_{j=0}^{p-1}H_{j}\left(0\right)e^{-2\pi ijt}=\frac{1}{p}\sum_{j=0}^{p-1}\frac{b_{j}}{d_{j}}e^{-2\pi ijt}\label{eq:Definition of beta_H} \end{equation} where, recall, $H_{j}$ denotes the $j$th branch of $H$. We also adopt the notation: \nomenclature{$\gamma_{H}\left(t\right)$}{ } \begin{equation} \gamma_{H}\left(t\right)\overset{\textrm{def}}{=}\frac{\beta_{H}\left(t\right)}{\alpha_{H}\left(t\right)}\label{eq:Definition of gamma_H} \end{equation} Additionally, we write $\sigma_{H}$ to denote the constant: \begin{equation} \sigma_{H}\overset{\textrm{def}}{=}\log_{p}\left(\sum_{j=0}^{p-1}H_{j}^{\prime}\left(0\right)\right)=\log_{p}\left(\frac{1}{p}\sum_{j=0}^{p-1}\mu_{j}\right)=1+\log_{p}\left(\alpha_{H}\left(0\right)\right)\label{eq:Definition of sigma_H} \end{equation} where $\log_{p}x=\frac{\ln x}{\ln p}$. Equivalently: \begin{equation} \alpha_{H}\left(0\right)=p^{\sigma_{H}-1}\label{eq:alpha_H of 0 in terms of sigma_H} \end{equation} Finally, we define the function \nomenclature{$\kappa_{H}\left(n\right)$}{ }$\kappa_{H}:\mathbb{N}_{0}\rightarrow\mathbb{Q}$ by:\index{alpha{H}left(tright)@$\alpha_{H}\left(t\right)$}\index{beta{H}left(tright)@$\beta_{H}\left(t\right)$}\index{gamma{H}left(tright)@$\gamma_{H}\left(t\right)$} \begin{equation} \kappa_{H}\left(n\right)\overset{\textrm{def}}{=}M_{H}\left(n\right)\left(\frac{\mu_{0}}{p}\right)^{-\lambda_{p}\left(n\right)}\label{eq:Definition of Kappa_H} \end{equation} \end{defn} \begin{defn} We say $H$ is \textbf{non-singular }whenever\index{Hydra map!non-singular} $\alpha_{H}\left(j/p\right)\neq0$ for any $j\in\mathbb{Z}/p\mathbb{Z}$. \end{defn} \vphantom{} The most important identities for this Chapter involve generating functions for the $\#_{p:j}\left(n\right)$; recall that these functions tell us the number of $j$s in the $p$-adic digits of $n$. \begin{prop}[\textbf{A Generating Function Identity}] \label{prop:Generating function identities}Let $\mathbb{F}$ be a field of characteristic zero, let $p$ be an integer $\geq2$, and let $c_{0},\ldots,c_{p-1}$ be any non-zero elements of $\mathbb{F}$. Then, for all $z\in\mathbb{F}$, we have the identities: \vphantom{} I. For all $n\geq1$: \begin{equation} \prod_{m=0}^{n-1}\left(\sum_{j=0}^{p-1}c_{j}z^{jp^{m}}\right)=c_{0}^{n}\sum_{k=0}^{p^{n}-1}c_{0}^{-\lambda_{p}\left(k\right)}\left(\prod_{j=0}^{p-1}c_{j}^{\#_{p:j}\left(k\right)}\right)z^{k}\label{eq:M_H partial sum generating identity} \end{equation} \vphantom{} II. If $c_{0}=1$, for all we have the identity: \begin{equation} \sum_{k=p^{n-1}}^{p^{n}-1}\left(\prod_{j=0}^{p-1}c_{j}^{\#_{p:j}\left(k\right)}\right)z^{k}=\left(\sum_{k=1}^{p-1}c_{k}z^{kp^{n-1}}\right)\prod_{m=0}^{n-2}\left(\sum_{j=0}^{p-1}c_{j}z^{jp^{m}}\right)\label{eq:Lambda-restricted partial M_H sum} \end{equation} The $n$-product is defined to be $1$ whenever $n=1$. \end{prop} Proof: I. Each term on the right of (\ref{eq:M_H partial sum generating identity}) is obtained by taking a product of the form: \begin{equation} \prod_{\ell=0}^{n-1}c_{j_{\ell}}z^{j_{\ell}p^{\ell}}=\left(\prod_{h=0}^{n-1}c_{j_{h}}\right)z^{\sum_{\ell=0}^{n-1}j_{\ell}p^{\ell}} \end{equation} for some choice of $j_{1},\ldots,j_{n-1}\in\mathbb{Z}/p\mathbb{Z}$. Note, however, that if there is an $m\in\left\{ 0,\ldots,n-1\right\} $ so that $j_{\ell}=0$ for all $\ell\in\left\{ m,\ldots,n-1\right\} $, the $c_{j_{\ell}}$s will still keep contributing to the product even for those values of $\ell$. So, in order to use our $\#_{p:j}\left(k\right)$ notation\textemdash which stops counting the $p$-adic digits of $k$ after the last \emph{non-zero} digit in $k$'s $p$-adic expansion\textemdash we need to modify the above product to take into account the number of extra $0$s that occur past the last non-zero digit in $k$'s $p$-adic expansion. So, let $k\geq1$, and let: \begin{equation} k=\sum_{\ell=0}^{n-1}j_{\ell}p^{\ell} \end{equation} Then, $j_{\ell}=0$ for all $\ell\geq\lambda_{p}\left(k\right)$. As such: \begin{equation} \prod_{h=0}^{n-1}c_{j_{h}}=c_{0}^{\left|\left\{ h\in\left\{ 0,\ldots,n-1\right\} :j_{h}=0\right\} \right|}\times\prod_{\ell=1}^{p-1}c_{\ell}^{\#_{p:\ell}\left(k\right)} \end{equation} Now, $\#_{p:0}\left(k\right)$ is the number of $\ell\leq\lambda_{p}\left(k\right)-1$ for which $j_{\ell}=0$. Since $j_{\ell}=0$ for all $\ell\geq\lambda_{p}\left(k\right)$, and because there are $n-\lambda_{p}\left(k\right)$ values of $h\in\left\{ 0,\ldots,n-1\right\} $ which are in the interval $\left[\lambda_{p}\left(k\right),n-1\right]$, we can write: \begin{equation} \left|\left\{ h\in\left\{ 0,\ldots,n-1\right\} :j_{h}=0\right\} \right|=\#_{p:0}\left(k\right)+n-\lambda_{p}\left(k\right) \end{equation} So: \begin{equation} \prod_{h=0}^{n-1}c_{j_{h}}=c_{0}^{\#_{p:0}\left(k\right)+n-\lambda_{p}\left(k\right)}\times\prod_{\ell=1}^{p-1}c_{\ell}^{\#_{p:\ell}\left(k\right)}=c_{0}^{n-\lambda_{p}\left(k\right)}\prod_{\ell=0}^{p-1}c_{\ell}^{\#_{p:\ell}\left(k\right)} \end{equation} Hence, every term on the right hand side of (\ref{eq:M_H partial sum generating identity}) is of the form: \begin{equation} \prod_{\ell=0}^{n-1}c_{j_{\ell}}z^{j_{\ell}p^{\ell}}=\left(\prod_{h=0}^{n-1}c_{j_{h}}\right)z^{\sum_{\ell=0}^{n-1}j_{\ell}p^{\ell}}=c_{0}^{n-\lambda_{p}\left(k\right)}\left(\prod_{\ell=0}^{p-1}c_{\ell}^{\#_{p:\ell}\left(k\right)}\right)z^{k} \end{equation} where: \begin{equation} k=\sum_{\ell=0}^{n-1}j_{\ell}p^{\ell}\leq\sum_{\ell=0}^{n-1}\left(p-1\right)p^{\ell}=p^{n}-1 \end{equation} Summing over $k\in\left\{ 0,\ldots,p^{n}-1\right\} $ then proves the identity in (I). \vphantom{} II. Suppose $c_{0}=1$. Then (I) can be written as: \[ \prod_{m=0}^{n-1}\left(\sum_{j=0}^{p-1}c_{j}z^{jp^{m}}\right)=\sum_{k=0}^{p^{n}-1}\left(\prod_{\ell=0}^{p-1}c_{\ell}^{\#_{p:\ell}\left(k\right)}\right)z^{k} \] Thus, subtracting the $\left(n-2\right)$nd case from the $\left(n-1\right)$st case gives: \begin{align*} \sum_{k=p^{n-1}}^{p^{n}-1}\left(\prod_{\ell=0}^{p-1}c_{\ell}^{\#_{p:\ell}\left(k\right)}\right)z^{k} & =\prod_{m=0}^{n-1}\left(\sum_{j=0}^{p-1}c_{j}z^{jp^{m}}\right)-\prod_{m=0}^{n-2}\left(\sum_{j=0}^{p-1}c_{j}z^{jp^{m}}\right)\\ & =\left(\sum_{k=0}^{p-1}c_{k}z^{kp^{n-1}}-1\right)\prod_{m=0}^{n-2}\left(\sum_{j=0}^{p-1}c_{j}z^{jp^{m}}\right) \end{align*} Note that the $k=0$ term is $c_{0}z^{0\cdot p^{n-1}}=1\cdot1=1$; this cancels out the $-1$, thereby proving (II). Q.E.D. \vphantom{} Next, we will need some additional formulae, properties, and estimates for $\kappa_{H}$. \begin{prop}[\textbf{Properties of} $\kappa_{H}$] \label{prop:Properties of Kappa_H}\ \vphantom{} I. \index{kappa{H}@$\kappa_{H}$!properties} \begin{equation} \kappa_{H}\left(n\right)=\prod_{k=1}^{p-1}\left(\frac{\mu_{k}}{\mu_{0}}\right)^{\#_{p:k}\left(n\right)},\textrm{ }\forall n\in\mathbb{N}_{0}\label{eq:Kappa_H explicit formula} \end{equation} \vphantom{} II. $\kappa_{H}$ is the unique function $\mathbb{N}_{0}\rightarrow\mathbb{Q}$ satisfying the functional equations \index{kappa{H}@$\kappa_{H}$!functional equation}: \begin{equation} \kappa_{H}\left(pn+j\right)=\frac{\mu_{j}}{\mu_{0}}\kappa_{H}\left(n\right),\textrm{ }\forall n\geq0,\textrm{ }\forall j\in\mathbb{Z}/p\mathbb{Z}\label{eq:Kappa_H functional equations} \end{equation} subject to the initial condition $\kappa_{H}\left(0\right)=1$. \vphantom{} III. $\kappa_{H}$ has $p$-adic structure, with: \begin{equation} \kappa_{H}\left(m+jp^{n}\right)=\frac{\mu_{j}}{\mu_{0}}\kappa_{H}\left(m\right),\textrm{ }\forall n\in\mathbb{N}_{0},\textrm{ }\forall j\in\mathbb{Z}/p\mathbb{Z},\textrm{ }\forall m\in\mathbb{Z}/p^{n}\mathbb{Z}\label{eq:Kappa_H is rho-adically distributed} \end{equation} \vphantom{} IV. If $H$ is semi-basic, then $\kappa_{H}$ is $\left(p,q_{H}\right)$-adically regular, with: \begin{equation} \lim_{n\rightarrow\infty}\left|\kappa_{H}\left(\left[\mathfrak{z}\right]_{p^{n}}\right)\right|_{q_{H}}\overset{\mathbb{R}}{=}0,\textrm{ }\forall\mathfrak{z}\in\mathbb{Z}_{p}^{\prime}\label{eq:Semi-basic q-adic decay for Kappa_H} \end{equation} \vphantom{} V. If $H$ is contracting\index{Hydra map!contracting} ($\mu_{0}/p<1$), then: \begin{equation} \lim_{N\rightarrow\infty}\left(\frac{\mu_{0}}{p}\right)^{N}\kappa_{H}\left(\left[\mathfrak{z}\right]_{p^{N}}\right)\overset{\mathbb{R}}{=}0,\textrm{ }\forall\mathfrak{z}\in\mathbb{N}_{0}\label{eq:Kappa_H decay when H is contracting} \end{equation} If $H$ is expanding ($\mu_{0}/p>1$), then: \begin{equation} \lim_{N\rightarrow\infty}\left(\frac{\mu_{0}}{p}\right)^{N}\kappa_{H}\left(\left[\mathfrak{z}\right]_{p^{N}}\right)\overset{\mathbb{R}}{=}+\infty,\textrm{ }\forall\mathfrak{z}\in\mathbb{N}_{0}\label{eq:Kappa_H growth when H is expanding} \end{equation} \end{prop} Proof: I. Using \textbf{Proposition \ref{prop:Explicit Formulas for M_H} }(page \pageref{prop:Explicit Formulas for M_H}), we have: \begin{equation} \prod_{j=0}^{p-1}\mu_{j}^{\#_{p:j}\left(\left[\mathfrak{z}\right]_{p^{N}}\right)}=p^{\lambda_{p}\left(\left[\mathfrak{z}\right]_{p^{N}}\right)}M_{H}\left(\left[\mathfrak{z}\right]_{p^{N}}\right)=\frac{M_{H}\left(\left[\mathfrak{z}\right]_{p^{N}}\right)}{p^{-\lambda_{p}\left(\left[\mathfrak{z}\right]_{p^{N}}\right)}} \end{equation} Multiplying through by: \begin{equation} \left(\frac{\mu_{0}}{p}\right)^{N}\mu_{0}^{-\lambda_{p}\left(\left[\mathfrak{z}\right]_{p^{N}}\right)} \end{equation} makes the right-hand side into: \begin{equation} \left(\frac{\mu_{0}}{p}\right)^{N}\kappa_{H}\left(\left[\mathfrak{z}\right]_{p^{N}}\right) \end{equation} So: \begin{align*} \left(\frac{\mu_{0}}{p}\right)^{N}\kappa_{H}\left(\left[\mathfrak{z}\right]_{p^{N}}\right) & =\left(\frac{\mu_{0}}{p}\right)^{N}\mu_{0}^{-\lambda_{p}\left(\left[\mathfrak{z}\right]_{p^{N}}\right)}\prod_{j=0}^{p-1}\mu_{j}^{\#_{p:j}\left(\left[\mathfrak{z}\right]_{p^{N}}\right)}\\ & =\left(\frac{\mu_{0}}{p}\right)^{N}\mu_{0}^{\#_{p:0}\left(\left[\mathfrak{z}\right]_{p^{N}}\right)-\lambda_{p}\left(\left[\mathfrak{z}\right]_{p^{N}}\right)}\prod_{j=1}^{p-1}\mu_{j}^{\#_{p:j}\left(\left[\mathfrak{z}\right]_{p^{N}}\right)}\\ \left(\lambda_{p}\left(k\right)=\sum_{j=0}^{p-1}\#_{p:j}\left(k\right)\right); & =\left(\frac{\mu_{0}}{p}\right)^{N}\mu_{0}^{-\sum_{j=1}^{p-1}\#_{p:j}\left(\left[\mathfrak{z}\right]_{p^{N}}\right)}\prod_{j=1}^{p-1}\mu_{j}^{\#_{p:j}\left(\left[\mathfrak{z}\right]_{p^{N}}\right)}\\ & =\left(\frac{\mu_{0}}{p}\right)^{N}\prod_{j=1}^{p-1}\left(\frac{\mu_{j}}{\mu_{0}}\right)^{\#_{p:j}\left(\left[\mathfrak{z}\right]_{p^{N}}\right)} \end{align*} Now, divide by $\left(\mu_{0}/p\right)^{N}$, pick $\mathfrak{z}=n\in\mathbb{N}_{0}$, and let $N\geq\lambda_{p}\left(n\right)$. This yields (I). \vphantom{} II \& III. Let $n\geq1$ and let $j\in\mathbb{Z}/p\mathbb{Z}$. Then: \begin{align*} \lambda_{p}\left(pn+j\right) & =\lambda_{p}\left(n\right)+1\\ M_{H}\left(pn+j\right) & =\frac{\mu_{j}}{p}M_{H}\left(n\right) \end{align*} Consequently, we have: \begin{align*} \kappa_{H}\left(pn+j\right) & =\left(\frac{p}{\mu_{0}}\right)^{\lambda_{p}\left(pn+j\right)}M_{H}\left(pn+j\right)\\ & =\frac{p}{\mu_{0}}\cdot\frac{\mu_{j}}{p}\cdot\left(\frac{p}{\mu_{0}}\right)^{\lambda_{p}\left(n\right)}M_{H}\left(n\right)\\ & =\frac{\mu_{j}}{\mu_{0}}\kappa_{H}\left(n\right) \end{align*} Next: \begin{align*} \kappa_{H}\left(j\right) & =\begin{cases} \left(\frac{p}{\mu_{0}}\right)^{\lambda_{p}\left(0\right)}M_{H}\left(0\right) & \textrm{if }j=0\\ \left(\frac{p}{\mu_{0}}\right)^{\lambda_{p}\left(pn+j\right)}M_{H}\left(j\right) & \textrm{if }j\in\left\{ 1,\ldots,p-1\right\} \end{cases}\\ & =\begin{cases} 1 & \textrm{if }j=0\\ \frac{\mu_{j}}{\mu_{0}} & \textrm{if }j\in\left\{ 1,\ldots,p-1\right\} \end{cases} \end{align*} So, $\kappa_{H}\left(0\right)=1$. Consequently \begin{equation} \kappa_{H}\left(j\right)=\frac{\mu_{j}}{\mu_{0}}\kappa_{H}\left(0\right),\textrm{ }\forall j\in\mathbb{Z}/p\mathbb{Z} \end{equation} Combining this with the other cases computed above proves (\ref{eq:Kappa_H functional equations}). As for (III), replace $n$ in (\ref{eq:Kappa_H explicit formula}) with $m+jp^{n}$, where $m\in\left\{ 0,\ldots,p^{n}-1\right\} $. This gives us: \begin{align*} \kappa_{H}\left(m+jp^{n}\right) & =\prod_{k=1}^{p-1}\left(\frac{\mu_{k}}{\mu_{0}}\right)^{\#_{p:k}\left(m+jp^{n}\right)}\\ \left(\#_{p:k}\left(m+jp^{n}\right)=\#_{p:k}\left(m\right)+\left[j=k\right]\right); & =\frac{\mu_{j}}{\mu_{0}}\prod_{k=1}^{p-1}\left(\frac{\mu_{k}}{\mu_{0}}\right)^{\#_{p:k}\left(m\right)}\\ & =\frac{\mu_{j}}{\mu_{0}}\kappa_{H}\left(m\right) \end{align*} So: \[ \kappa_{H}\left(pn+j\right)=\frac{\mu_{j}}{\mu_{0}}\kappa_{H}\left(n\right),\textrm{ }\forall n\geq0 \] \[ \kappa_{H}\left(jp^{n}+m\right)=\frac{\mu_{m}}{\mu_{0}}\kappa_{H}\left(n\right),\textrm{ }\forall n\geq0 \] Finally, by \textbf{Proposition \ref{prop:formula for functions with p-adic structure}}, we then have that $\kappa_{H}\left(n\right)$ is uniquely determined by the $\mu_{j}$s and $\kappa_{H}\left(0\right)$. This proves that any function $\mathbb{N}_{0}\rightarrow\mathbb{Q}$ satisfying (\ref{eq:Kappa_H functional equations}) and $\kappa_{H}\left(0\right)=1$ is necessarily equal to $\kappa_{H}$. This shows the desired uniqueness, thereby proving the rest of (II). \vphantom{} IV. Let $\mathfrak{z}\in\mathbb{Z}_{p}^{\prime}$. If $H$ is semi-basic, we have that $\mu_{0}$ and $p$ are both co-prime to $q_{H}$. Taking $q$-adic absolute values of (I) yields: \begin{equation} \left|\kappa_{H}\left(\left[\mathfrak{z}\right]_{p^{N}}\right)\right|_{q}=\left|\prod_{j=1}^{p-1}\left(\frac{\mu_{j}}{\mu_{0}}\right)^{\#_{p:j}\left(\left[\mathfrak{z}\right]_{p^{N}}\right)}\right|_{q}=\prod_{j=1}^{p-1}\left|\mu_{j}\right|_{q}^{\#_{p:j}\left(\left[\mathfrak{z}\right]_{p^{N}}\right)} \end{equation} With $H$ being semi-basic, $\mu_{j}$ is a multiple of $q_{H}$ for all non-zero $j$. As such, by the pigeonhole principle, since $\mathfrak{z}\in\mathbb{Z}_{p}^{\prime}$, there is at least one $j\in\left\{ 1,\ldots,p-1\right\} $ so that infinitely many of $\mathfrak{z}$'s $p$-adic digits are $j$. This makes $\#_{p:j}\left(\left[\mathfrak{z}\right]_{p^{N}}\right)\rightarrow\infty$ in $\mathbb{R}$ as $N\rightarrow\infty$, and demonstrates that (\ref{eq:Semi-basic q-adic decay for Kappa_H}) holds. This proves (IV). \vphantom{} V. Let $\mathfrak{z}\in\mathbb{N}_{0}$. Then, $\left[\mathfrak{z}\right]_{p^{N}}=\mathfrak{z}$ for all $N\geq\lambda_{p}\left(\mathfrak{z}\right)$, and so: \begin{equation} \left(\frac{\mu_{0}}{p}\right)^{N}\kappa_{H}\left(\left[\mathfrak{z}\right]_{p^{N}}\right)=\left(\frac{\mu_{0}}{p}\right)^{N}\kappa_{H}\left(\mathfrak{z}\right),\textrm{ }\forall N\geq\lambda_{p}\left(\mathfrak{z}\right) \end{equation} where $\kappa_{H}\left(\mathfrak{z}\right)\neq0$ is then a non-zero rational constant $c$. Thus, as $N\rightarrow\infty$ the limit in $\mathbb{R}$ of the left-hand side will be $0$ if and only if $H$ is contracting ($\mu_{0}/p<1$) and will be $+\infty$ if and only if $H$ is expanding ($\mu_{0}/p>1$). This proves (V). Q.E.D. \vphantom{} Next, we compute the value of the summatory function\index{chi{H}@$\chi_{H}$!summatory function} $\sum_{n=0}^{N}\chi_{H}\left(n\right)$ via a recursive method. \begin{lem}[\textbf{Summatory function of} $\chi_{H}$] \label{lem:Summatory function of Chi_H}Let $H$ be a $p$-Hydra map (we \emph{do not }require that $\mu_{0}=1$). Then: \begin{equation} \sum_{n=0}^{p^{N}-1}\chi_{H}\left(n\right)=\begin{cases} \beta_{H}\left(0\right)Np^{N} & \textrm{if }\sigma_{H}=1\\ \frac{\beta_{H}\left(0\right)}{\alpha_{H}\left(0\right)-1}\left(p^{\sigma_{H}N}-p^{N}\right) & \textrm{else} \end{cases},\textrm{ }\forall N\geq1\label{eq:rho-N minus 1 sum of Chi_H} \end{equation} \end{lem} Proof: For any $N\geq0$, define: \begin{equation} S_{H}\left(N\right)\overset{\textrm{def}}{=}\frac{1}{p^{N}}\sum_{n=0}^{p^{N}-1}\chi_{H}\left(n\right)\label{eq:Definition of S_H of N} \end{equation} Letting $N\geq1$, note that: \begin{align*} p^{N}S_{H}\left(N\right) & =\sum_{n=0}^{p^{N}-1}\chi_{H}\left(n\right)\\ & =\sum_{j=0}^{p-1}\sum_{n=0}^{p^{N-1}-1}\chi_{H}\left(pn+j\right)\\ \left(\textrm{use }\chi_{H}\textrm{'s functional eqs.}\right); & =\sum_{j=0}^{p-1}\sum_{n=0}^{p^{N-1}-1}\frac{a_{j}\chi_{H}\left(n\right)+b_{j}}{d_{j}}\\ & =\sum_{j=0}^{p-1}\left(\frac{a_{j}}{d_{j}}\sum_{n=0}^{p^{N-1}-1}\chi_{H}\left(n\right)+\frac{b_{j}}{d_{j}}p^{N-1}\right)\\ & =\left(\sum_{j=0}^{p-1}\frac{a_{j}}{d_{j}}\right)p^{N-1}S_{H}\left(N-1\right)+\left(\sum_{j=0}^{p-1}\frac{b_{j}}{d_{j}}\right)p^{N-1} \end{align*} Dividing through by $p^{N}$, note that: \begin{align*} \alpha_{H}\left(0\right) & =\frac{1}{p}\sum_{j=0}^{p-1}\frac{a_{j}}{d_{j}}=\frac{1}{p}\left(\frac{1}{p}\sum_{j=0}^{p-1}\mu_{j}\right)=p^{\sigma_{H}-1}\\ \beta_{H}\left(0\right) & =\frac{1}{p}\sum_{j=0}^{p-1}\frac{b_{j}}{d_{j}} \end{align*} As such, we can write: \begin{equation} S_{H}\left(N\right)=p^{\sigma_{H}-1}S_{H}\left(N-1\right)+\beta_{H}\left(0\right)\label{eq:Recursive formula for S_H} \end{equation} With this, we see that $S_{H}\left(N\right)$ is the image of $S_{H}\left(0\right)$ under $N$ iterates of the affine linear map: \begin{equation} x\mapsto p^{\sigma_{H}-1}x+\beta_{H}\left(0\right)\label{eq:Affine linear map generating S_H} \end{equation} This leaves us with two cases to investigate: \vphantom{} i. Suppose $\sigma_{H}=1$. Then, (\ref{eq:Affine linear map generating S_H}) reduces to a translation: \begin{equation} x\mapsto x+\beta_{H}\left(0\right) \end{equation} in which case: \begin{equation} S_{H}\left(N\right)=S_{H}\left(0\right)+\beta_{H}\left(0\right)N\label{eq:Explicit formula for S_H of N when sigma_H =00003D00003D 1} \end{equation} \vphantom{} ii. Suppose $\sigma_{H}\neq1$. Then, (\ref{eq:Affine linear map generating S_H}) is not a translation, and so, using the explicit formula for this $N$th iterate, we obtain: \begin{equation} S_{H}\left(N\right)=p^{\left(\sigma_{H}-1\right)N}S_{H}\left(0\right)+\frac{p^{\left(\sigma_{H}-1\right)N}-1}{p^{\sigma_{H}-1}-1}\beta_{H}\left(0\right)\label{eq:Explicit formula for S_H of N} \end{equation} Noting that: \begin{align} S_{H}\left(0\right) & =\sum_{n=0}^{p^{0}-1}\chi_{H}\left(n\right)=\chi_{H}\left(0\right)=0 \end{align} we then have: \begin{equation} \frac{1}{p^{N}}\sum_{n=0}^{p^{N}-1}\chi_{H}\left(n\right)=S_{H}\left(N\right)=\begin{cases} \beta_{H}\left(0\right)N & \textrm{if }\sigma_{H}=1\\ \frac{p^{\left(\sigma_{H}-1\right)N}-1}{p^{\sigma_{H}-1}-1}\beta_{H}\left(0\right) & \textrm{else} \end{cases} \end{equation} Multiplying through on all sides by $p^{N}$ and re-writing $p^{\sigma_{H}-1}-1$ as $\alpha_{H}\left(0\right)-1$ then yields: \begin{equation} \sum_{n=0}^{p^{N}-1}\chi_{H}\left(n\right)=\begin{cases} \beta_{H}\left(0\right)Np^{N} & \textrm{if }\sigma_{H}=1\\ \frac{\beta_{H}\left(0\right)}{\alpha_{H}\left(0\right)-1}\left(p^{\sigma_{H}N}-p^{N}\right) & \textrm{else} \end{cases} \end{equation} which proves (\ref{eq:rho-N minus 1 sum of Chi_H}). Q.E.D. \vphantom{} Although not of any particular use (at least at the time of writing), we can use the formula for the summatory function to obtain simple upper and lower (archimedean) bounds. We do this with help of the following trick: \begin{prop}[\textbf{Replacing $N$ with }$\log_{p}\left(N+1\right)$] Let $\left\{ a_{n}\right\} _{n\geq0}$ be a sequence of non-negative real numbers, and let $p$ be an integer $\geq2$. Suppose there is a non-decreasing function $A:\left[0,\infty\right)\rightarrow\left[0,\infty\right)$ such that: \[ \sum_{n=0}^{p^{N}-1}a_{n}=A\left(N\right),\textrm{ }\forall N\in\mathbb{N}_{1} \] Then: \begin{equation} A\left(\log_{p}N\right)\leq\sum_{n=0}^{N}a_{n}\leq2A\left(\log_{p}\left(N\right)+1\right)-A\left(\log_{p}\left(N\right)-1\right),\textrm{ }\forall N\geq1\label{eq:Replacement Lemma} \end{equation} \end{prop} Proof: As just remarked, for each $k\in\mathbb{N}_{1}$, $\left\{ p^{k-1},p^{k-1}+1,\ldots,p^{k}-1\right\} $ is the set of all positive integers $n$ for which $\lambda_{p}\left(n\right)=k$. As such, letting $N\in\mathbb{N}_{1}$ we have that $p^{\lambda_{p}\left(N\right)-1}$ is the smallest positive integer whose $p$-adic representation has the same number of digits as the $p$-adic representation of $N$. Consequently, we can write: \begin{equation} \sum_{n=0}^{N}a_{n}=\sum_{n=0}^{p^{\lambda_{p}\left(N\right)-1}-1}a_{n}+\sum_{n=p^{\lambda_{p}\left(N\right)-1}}^{N}a_{n}=A\left(\lambda_{p}\left(N\right)\right)+\sum_{n=p^{\lambda_{p}\left(N\right)-1}}^{N}a_{n}\label{eq:Lambda decomposition of Nth partial sum} \end{equation} Since the $a_{n}$s are non-negative, this then yields the lower bound: \begin{equation} \sum_{n=0}^{N}a_{n}\geq A\left(\lambda_{p}\left(N\right)\right) \end{equation} Since: \begin{equation} \lambda_{p}\left(N\right)=1+\left\lfloor \log_{p}N\right\rfloor \geq\log_{p}N \end{equation} and since $A$ is a non-decreasing function, this then implies: \begin{equation} \sum_{n=0}^{N}a_{n}\geq A\left(\log_{p}N\right) \end{equation} As for an upper bound, we note that the non-negativity of the $a_{n}$s allows us to write: \begin{align*} \sum_{n=p^{\lambda_{p}\left(N\right)-1}}^{N}a_{n} & \leq\sum_{n=p^{\lambda_{p}\left(N\right)-1}}^{p^{\lambda_{p}\left(N\right)}-1}a_{n}\\ & =\sum_{n=0}^{p^{\lambda_{p}\left(N\right)}-1}a_{n}-\sum_{n=0}^{p^{\lambda_{p}\left(N\right)-1}-1}a_{n}\\ & =A\left(\lambda_{p}\left(N\right)\right)-A\left(\lambda_{p}\left(N\right)-1\right) \end{align*} seeing as $p^{\lambda_{p}\left(N\right)}-1$ is the largest positive integer with the same number of $p$-adic digits as $N$. With this, (\ref{eq:Lambda decomposition of Nth partial sum}) becomes: \begin{align} \sum_{n=0}^{N}a_{n} & \leq2A\left(\lambda_{p}\left(N\right)\right)-A\left(\lambda_{p}\left(N\right)-1\right) \end{align} Since $A$ is non-decreasing, and since: \begin{equation} \log_{p}N\leq1+\left\lfloor \log_{p}N\right\rfloor =\lambda_{p}\left(N\right)=1+\left\lfloor \log_{p}N\right\rfloor \leq1+\log_{p}N \end{equation} we then have that: \begin{align} \sum_{n=0}^{N}a_{n} & \leq2A\left(\log_{p}\left(N\right)+1\right)-A\left(\log_{p}\left(N\right)-1\right) \end{align} Q.E.D. \begin{prop}[\textbf{Archimedean Bounds for $\chi_{H}$'s Summatory Function}] \label{prop:archimedean bounds on Chi_H summatory function}Let $H$ be a $p$-Hydra map (we \emph{do not }require that $\mu_{0}=1$). Then\index{chi{H}@$\chi_{H}$!summatory function!estimates}: \vphantom{} I. If $\sigma_{H}=1$: \begin{equation} \beta_{H}\left(0\right)N\log_{p}N\leq\sum_{n=0}^{N}\chi_{H}\left(n\right)\leq C_{H,1}N+\frac{2p^{2}-1}{p}\beta_{H}\left(0\right)N\log_{p}N\label{eq:Chi_H partial sum asympotitcs for when sigma_H is 1} \end{equation} where: \begin{equation} C_{H,1}\overset{\textrm{def}}{=}\frac{2p^{2}+1}{p}\beta_{H}\left(0\right) \end{equation} \vphantom{} II. If $\sigma_{H}>1$: \begin{equation} C_{H,2}N^{\sigma_{H}}-\gamma_{H}\left(0\right)N\leq\sum_{n=0}^{N}\chi_{H}\left(n\right)\leq\frac{2p^{2\sigma_{H}}-1}{p^{\sigma_{H}}}C_{H,2}N^{\sigma_{H}}-\frac{2p^{2}-1}{p}\gamma_{H}\left(0\right)N\label{eq:Chi_H partial sum asympotitcs for when sigma_H is greater than 1} \end{equation} where: \begin{equation} C_{H,2}\overset{\textrm{def}}{=}\gamma_{H}\left(0\right) \end{equation} \begin{equation} \gamma_{H}\left(0\right)\overset{\textrm{def}}{=}\frac{\beta_{H}\left(0\right)}{\alpha_{H}\left(0\right)-1} \end{equation} \end{prop} Proof: When $\sigma_{H}\geq1$, the formulae in (\ref{eq:rho-N minus 1 sum of Chi_H}) are non-decreasing functions of $N$. As such, we may use (\ref{eq:Replacement Lemma}) which, upon simplification, yields (\ref{eq:Chi_H partial sum asympotitcs for when sigma_H is 1}) and (\ref{eq:Chi_H partial sum asympotitcs for when sigma_H is greater than 1}). Q.E.D. \vphantom{} Finally, we have the truncations of $\chi_{H}$: \begin{notation} We write $q$ to denote $q_{H}$. We write $\chi_{H,N}:\mathbb{Z}_{p}\rightarrow\mathbb{Q}\subset\mathbb{Q}_{q}$ to denote the $N$th truncation\index{chi{H}@$\chi_{H}$!$N$th truncation} of $\chi_{H}$: \begin{equation} \chi_{H,N}\left(\mathfrak{z}\right)\overset{\textrm{def}}{=}\sum_{n=0}^{p^{N}-1}\chi_{H}\left(n\right)\left[\mathfrak{z}\overset{p^{N}}{\equiv}n\right],\textrm{ }\forall\mathfrak{z}\in\mathbb{Z}_{p},\textrm{ }\forall N\in\mathbb{N}_{0}\label{eq:Definition of Nth truncation of Chi_H} \end{equation} Recall that: \begin{equation} \chi_{H,N}\left(n\right)=\chi_{H}\left(n\right),\textrm{ }\forall n\in\left\{ 0,\ldots,p^{N}-1\right\} \end{equation} In particular: \begin{equation} \chi_{H}\left(n\right)=\chi_{H,\lambda_{p}\left(n\right)}\left(n\right),\textrm{ }\forall n\in\mathbb{N}_{0} \end{equation} Because $\chi_{H,N}$ is a rational-valued function which is a linear combination of finitely many indicator functions for clopen subsets of $\mathbb{Z}_{p}$, $\chi_{H,N}$ is continuous both as a function $\mathbb{Z}_{p}\rightarrow\mathbb{C}_{q}$ and as a function $\mathbb{Z}_{p}\rightarrow\mathbb{C}$. As a result of this, in writing $\hat{\chi}_{H,N}:\hat{\mathbb{Z}}_{p}\rightarrow\overline{\mathbb{Q}}$ to denote the Fourier Transform of $\chi_{H,N}$: \begin{equation} \hat{\chi}_{H,N}\left(t\right)\overset{\textrm{def}}{=}\int_{\mathbb{Z}_{p}}\chi_{H,N}\left(\mathfrak{z}\right)e^{-2\pi i\left\{ t\mathfrak{z}\right\} _{p}}d\mathfrak{z},\textrm{ }\forall t\in\hat{\mathbb{Z}}_{p}\label{eq:Definition of the Fourier Coefficients of Chi_H,N} \end{equation} observe that this integral is convergent in both the topology of $\mathbb{C}$ and the topology of $\mathbb{C}_{q}$. In fact, because $\chi_{H,N}$ is locally constant, the integral will reduce to a finite sum (see (\ref{eq:Chi_H,N hat transform formula - sum form})): \begin{equation} \hat{\chi}_{H,N}\left(t\right)\overset{\overline{\mathbb{Q}}}{=}\frac{\mathbf{1}_{0}\left(p^{N}t\right)}{p^{N}}\sum_{n=0}^{p^{N}-1}\chi_{H}\left(n\right)e^{-2\pi int},\textrm{ }\forall t\in\hat{\mathbb{Z}}_{p} \end{equation} where: \nomenclature{$\mathbf{1}_{0}\left(t\right)$}{ } \begin{equation} \mathbf{1}_{0}\left(p^{N}t\right)=\left[\left|t\right|_{p}\leq p^{N}\right],\textrm{ }\forall t\in\hat{\mathbb{Z}}_{p} \end{equation} As such, all of the computations and formal manipulations we will perform hold simultaneously in $\mathbb{C}$ and in $\mathbb{C}_{q}$; they both occur in $\overline{\mathbb{Q}}\subset\mathbb{C}\cap\mathbb{C}_{q}$. The difference between the archimedean and non-archimedean topologies will only emerge when we consider what happens as $N$ tends to $\infty$. Additionally, note that because of $\chi_{H,N}$'s local constant-ness and the finitude of its range, we have that, for each $N$, $\hat{\chi}_{H,N}$ has finite support ($\left|t\right|_{p}\geq p^{N+1}$): \begin{equation} \hat{\chi}_{H,N}\left(t\right)=0,\textrm{ }\forall\left|t\right|_{p}\geq p^{N+1},\textrm{ }\forall N\in\mathbb{N}_{0}\label{eq:Vanishing of Chi_H,N hat for all t with sufficiently large denominators} \end{equation} and thus, that the Fourier series: \begin{equation} \chi_{H,N}\left(\mathfrak{z}\right)=\sum_{t\in\hat{\mathbb{Z}}_{p}}\hat{\chi}_{H,N}\left(t\right)e^{2\pi i\left\{ t\mathfrak{z}\right\} _{p}}\label{eq:Fourier series for Chi_H,N} \end{equation} will be uniformly convergent in both $\mathbb{C}$ and $\mathbb{C}_{q}$ with respect to $\mathfrak{z}\in\mathbb{Z}_{p}$, reducing to a finite sum in all cases. Finally, with regard to algebraic-number-theoretic issues of picking embeddings of $\overline{\mathbb{Q}}$ in $\mathbb{C}$ and $\mathbb{C}_{q}$, there is no need to worry. This is because all of our work hinges on the Fourier series identity: \begin{equation} \left[\mathfrak{z}\overset{p^{N}}{\equiv}n\right]\overset{\overline{\mathbb{Q}}}{=}\frac{1}{p^{N}}\sum_{\left|t\right|_{p}\leq p^{N}}e^{2\pi i\left\{ t\left(\mathfrak{z}-n\right)\right\} _{p}} \end{equation} which\textemdash \emph{crucially}\textemdash is invariant under the action of elements of the Galois group $\textrm{Gal}\left(\overline{\mathbb{Q}}/\mathbb{Q}\right)$, seeing as the sum evaluates to a rational number (the indicator function, which is either $0$ or $1$). As such, all finite operations (sums, multiplication, series rearrangements) involving this identity are \emph{independent }of our choices of embeddings of $\overline{\mathbb{Q}}$ in $\mathbb{C}$ and $\mathbb{C}_{q}$. \end{notation} \vphantom{} Lastly, we need to make is the following lemma regarding the interaction between truncation and functional equations. This result will play a critical role in actually \emph{proving }that the functions $\hat{\mathbb{Z}}_{p}\rightarrow\overline{\mathbb{Q}}$ which we will derive are, in fact, Fourier transforms of $\chi_{H}$. \begin{lem} \label{lem:Functional equations and truncation}Let $\chi:\mathbb{N}_{0}\rightarrow\mathbb{Q}$, and suppose that for $j\in\left\{ 0,\ldots,p-1\right\} $ there are functions $\Phi_{j}:\mathbb{N}_{0}\times\mathbb{Q}\rightarrow\mathbb{Q}$ such that: \begin{equation} \chi\left(pn+j\right)=\Phi_{j}\left(n,\chi\left(n\right)\right),\textrm{ }\forall n\in\mathbb{N}_{0},\textrm{ }\forall j\in\mathbb{Z}/p\mathbb{Z}\label{eq:Relation between truncations and functional equations - Hypothesis} \end{equation} Then, the $N$th truncations $\chi_{N}$ satisfy the functional equations\index{functional equation}: \begin{equation} \chi_{N}\left(pn+j\right)=\Phi_{j}\left(\left[n\right]_{p^{N-1}},\chi_{N-1}\left(n\right)\right),\textrm{ }\forall n\in\mathbb{N}_{0},\textrm{ }\forall j\in\mathbb{Z}/p\mathbb{Z}\label{eq:Relation between truncations and functional equations, version 1} \end{equation} Equivalently: \begin{equation} \chi\left(\left[pn+j\right]_{p^{N}}\right)=\Phi_{j}\left(\left[n\right]_{p^{N-1}},\chi\left(\left[n\right]_{p^{N-1}}\right)\right),\textrm{ }\forall n\in\mathbb{N}_{0},\textrm{ }\forall j\in\mathbb{Z}/p\mathbb{Z}\label{eq:Relation between truncations and functional equations, version 2} \end{equation} \end{lem} Proof: Fix $N\geq0$, $n\in\mathbb{N}_{0}$, and $j\in\mathbb{Z}/p\mathbb{Z}$. Then: \begin{align*} \chi\left(\left[pn+j\right]_{p^{N}}\right) & =\chi_{N}\left(pn+j\right)\\ & =\sum_{m=0}^{p^{N}-1}\chi\left(m\right)\left[pn+j\overset{p^{N}}{\equiv}m\right]\\ \left(\textrm{split }m\textrm{ mod }p\right); & =\sum_{\ell=0}^{p^{N-1}-1}\sum_{k=0}^{p-1}\chi\left(p\ell+k\right)\left[pn+j\overset{p^{N}}{\equiv}p\ell+k\right]\\ & =\sum_{\ell=0}^{p^{N-1}-1}\sum_{k=0}^{p-1}\Phi_{k}\left(\ell,\chi\left(\ell\right)\right)\underbrace{\left[n\overset{p^{N-1}}{\equiv}\ell+\frac{k-j}{p}\right]}_{0\textrm{ }\forall n\textrm{ if }k\neq j}\\ & =\sum_{\ell=0}^{p^{N-1}-1}\Phi_{j}\left(\ell,\chi\left(\ell\right)\right)\left[n\overset{p^{N-1}}{\equiv}\ell\right]\\ & =\Phi_{j}\left(\left[n\right]_{p^{N-1}},\chi\left(\left[n\right]_{p^{N-1}}\right)\right) \end{align*} Q.E.D. \begin{rem} As an aside, it is worth mentioning that $\sigma_{H}$ is implicated in the abscissa of convergence of the Dirichlet series\index{Dirichlet series}\index{chi{H}@$\chi_{H}$!Dirichlet series}: \begin{equation} \zeta_{M_{H}}\left(s\right)\overset{\textrm{def}}{=}\sum_{n=0}^{\infty}\frac{M_{H}\left(n\right)}{\left(n+1\right)^{s}} \end{equation} and: \begin{equation} \zeta_{\chi_{H}}\left(s\right)\overset{\textrm{def}}{=}\sum_{n=0}^{\infty}\frac{\chi_{H}\left(n\right)}{\left(n+1\right)^{s}} \end{equation} For instance, using \textbf{Abel's Summation Formula }in the standard way yields: \begin{equation} \sum_{n=0}^{p^{N}-1}\frac{\chi_{H}\left(n\right)}{\left(1+n\right)^{s}}=\frac{1}{p^{Ns}}\sum_{n=0}^{p^{N}-1}\chi_{H}\left(n\right)+s\int_{0}^{p^{N}-1}\frac{\sum_{n=0}^{\left\lfloor x\right\rfloor }\chi_{H}\left(n\right)}{\left(1+x\right)^{s+1}}dx \end{equation} So, by \textbf{Proposition \ref{lem:Summatory function of Chi_H}}, when $\sigma_{H}=1$, we obtain: \[ \sum_{n=0}^{p^{N}-1}\frac{\chi_{H}\left(n\right)}{\left(1+n\right)^{s}}=\frac{N\beta_{H}\left(0\right)}{p^{N\left(s-1\right)}}+s\int_{0}^{p^{N}-1}\frac{\sum_{n=0}^{\left\lfloor x\right\rfloor }\chi_{H}\left(n\right)}{\left(1+x\right)^{s+1}}dx \] whereas for $\sigma_{H}\neq1$, we obtain: \[ \sum_{n=0}^{p^{N}-1}\frac{\chi_{H}\left(n\right)}{\left(1+n\right)^{s}}=\frac{\beta_{H}\left(0\right)}{\alpha_{H}\left(0\right)-1}\left(\frac{1}{p^{N\left(s-\sigma_{H}\right)}}-\frac{1}{p^{N\left(s-1\right)}}\right)+s\int_{0}^{p^{N}-1}\frac{\sum_{n=0}^{\left\lfloor x\right\rfloor }\chi_{H}\left(n\right)}{\left(1+x\right)^{s+1}}dx \] which then gives: \begin{equation} \zeta_{\chi_{H}}\left(s\right)=s\int_{0}^{\infty}\frac{\sum_{n=0}^{\left\lfloor x\right\rfloor }\chi_{H}\left(n\right)}{\left(1+x\right)^{s+1}}dx,\textrm{ }\forall\textrm{Re}\left(s\right)>\max\left\{ 1,\sigma_{H}\right\} \end{equation} The (archimedean) bounds on $\chi_{H}$'s summatory function given in \textbf{Proposition \ref{prop:archimedean bounds on Chi_H summatory function} }then show that $\zeta_{\chi_{H}}\left(s\right)$ converges absolutely for all $\textrm{Re}\left(s\right)>\sigma_{H}$ for all $H$ with $\sigma_{H}\geq1$. As mentioned at the end of Chapter 2, this Dirichlet series can be analytically continued to meromorphic function on $\mathbb{C}$ with a half-lattice of poles in the half-plane $\textrm{Re}\left(s\right)\leq\sigma_{H}$, however, its growth rate as $\textrm{Re}\left(s\right)\rightarrow-\infty$ is hyper-exponential, making it seemingly ill-suited for analysis via techniques of contour integration. \end{rem} \newpage{} \section{\label{sec:4.2 Fourier-Transforms-=00003D000026}Fourier Transforms and Quasi-Integrability} THROUGHOUT THIS SECTION, WE ALSO ASSUME $H$ IS NON-SINGULAR. \vphantom{} This section is the core of our analysis of $\chi_{H}$. Understandably, it is also the most computationally intensive portion of this dissertation\textemdash at least until Chapter 6. Our work will be broken up into three parts. In Subsection \ref{subsec:4.2.1}, we use the functional equations (\ref{eq:Functional Equations for Chi_H over the rho-adics}) to establish a recursion relation between the Fourier transforms of the $N$th truncations of $\chi_{H}$. Solving this yields an explicit formula for $\hat{\chi}_{H,N}\left(t\right)$ (\ref{eq:Explicit formula for Chi_H,N hat}) which is a sum of products of $\beta_{H}$ and a partial product involving $\alpha_{H}$. The partial products involving $\alpha_{H}$ depend on both $N$ and $t$, and by carefully manipulating the product, we can extract from the product the part that depends on $N$. Doing this yields a function $\hat{A}_{H}:\hat{\mathbb{Z}}_{p}\rightarrow\overline{\mathbb{Q}}$ (\vref{eq:Definition of A_H hat}) whose properties we then study in detail, establishing various formulae along the way. Subsection \ref{subsec:4.2.1} concludes with greatly simplified expressions for $\hat{\chi}_{H,N}\left(t\right)$ given by\textbf{ Theorem} \textbf{\ref{thm:(N,t) asymptotic decomposition of Chi_H,N hat}}. These can be considered explicit ``asymptotic formulae'' for $\hat{\chi}_{H,N}\left(t\right)$ in terms of $t$ and $N$ as $N\rightarrow\infty$. The asymptotic formulae of \textbf{Theorem} \textbf{\ref{thm:(N,t) asymptotic decomposition of Chi_H,N hat}} demonstrate the sensitive dependence of $\hat{\chi}_{H,N}\left(t\right)$ on $p$ and, most of all, on $\alpha_{H}\left(0\right)$. The dependence on $p$ is a consequence of the fact that the multiplicative group $\left(\mathbb{Z}/p\mathbb{Z}\right)^{\times}$ of multiplicatively invertible integers modulo $p$ satisfies $\left|\left(\mathbb{Z}/p\mathbb{Z}\right)^{\times}\right|=1$ if and only if $p=2$. Much more significant, however, is the dependence on $\alpha_{H}\left(0\right)$. The essence of this dependence was already on display in the formula for $\chi_{H}$'s summatory function as given in \textbf{Lemma \ref{lem:Summatory function of Chi_H}}, where we saw two distinct behaviors, one for $\sigma_{H}=1$ (equivalently, for $\alpha_{H}\left(0\right)=1$) and one for all other values. This dependency will appear again in our analysis of $\hat{A}_{H}$, where it yields two cases: $\alpha_{H}\left(0\right)=1$ and $\alpha_{H}\left(0\right)\neq1$. In Subsection \ref{subsec:4.2.2}, we first show that $\chi_{H}$ is quasi-integrable with respect to the standard $\left(p,q_{H}\right)$-adic frame in the $\alpha_{H}\left(0\right)=1$ case. We will then exploit the functional equations (\ref{eq:Functional Equations for Chi_H over the rho-adics}) to extend this result to arbitrary values of $\alpha_{H}\left(0\right)$. As a consequence, we will be able to provide Fourier transforms for $\chi_{H}$\textemdash thereby proving $\chi_{H}$'s quasi-integrability\textemdash along with interesting non-trivial explicit series expressions for $\chi_{H}$ itself. \subsection{\label{subsec:4.2.1}$\hat{\chi}_{H,N}$ and $\hat{A}_{H}$} We begin by computing a recursive formula\index{hat{chi}{H,N}@$\hat{\chi}_{H,N}$!recursive formula} for $\hat{\chi}_{H,N}$ in terms of $\hat{\chi}_{H,N-1}$. Solving this produces an explicit formula for $\hat{\chi}_{H,N}$ in terms of $t$ and $N$. \begin{prop}[\textbf{Formulae for }$\hat{\chi}_{H,N}$] \label{prop:Computation of Chi_H,N hat}\ \vphantom{} I. \begin{equation} \hat{\chi}_{H,N}\left(t\right)\overset{\overline{\mathbb{Q}}}{=}\frac{\mathbf{1}_{0}\left(p^{N}t\right)}{p^{N}}\sum_{n=0}^{p^{N}-1}\chi_{H}\left(n\right)e^{-2\pi int}\label{eq:Chi_H,N hat transform formula - sum form} \end{equation} \vphantom{} II. \begin{equation} \hat{\chi}_{H,N}\left(t\right)=\mathbf{1}_{0}\left(p^{N}t\right)\alpha_{H}\left(t\right)\hat{\chi}_{H,N-1}\left(pt\right)+\mathbf{1}_{0}\left(pt\right)\beta_{H}\left(t\right),\textrm{ }\forall N\geq1,\textrm{ }\forall t\in\hat{\mathbb{Z}}_{p}\label{eq:Chi_H,N hat functional equation} \end{equation} the nesting of which yields: \begin{equation} \hat{\chi}_{H,N}\left(t\right)=\sum_{n=0}^{N-1}\mathbf{1}_{0}\left(p^{n+1}t\right)\beta_{H}\left(p^{n}t\right)\prod_{m=0}^{n-1}\alpha_{H}\left(p^{m}t\right)\label{eq:Explicit formula for Chi_H,N hat} \end{equation} where the $m$-product is defined to be $1$ when $n=0$. In particular, note that $\hat{\chi}_{H,N}\left(t\right)=0$ for all $t\in\hat{\mathbb{Z}}_{p}$ with $\left|t\right|_{p}>p^{N}$. \end{prop} Proof: We use the Fourier series for $\left[\mathfrak{z}\overset{p^{N}}{\equiv}n\right]$ to re-write $\chi_{H,N}$ as a Fourier series: \begin{align*} \chi_{H,N}\left(\mathfrak{z}\right) & =\sum_{n=0}^{p^{N}-1}\chi_{H}\left(n\right)\left[\mathfrak{z}\overset{p^{N}}{\equiv}n\right]\\ & =\sum_{n=0}^{p^{N}-1}\chi_{H}\left(n\right)\frac{1}{p^{N}}\sum_{\left|t\right|_{p}\leq p^{N}}e^{2\pi i\left\{ t\left(\mathfrak{z}-n\right)\right\} _{p}}\\ & =\sum_{\left|t\right|_{p}\leq p^{N}}\left(\frac{1}{p^{N}}\sum_{n=0}^{p^{N}-1}\chi_{H}\left(n\right)e^{-2\pi int}\right)e^{2\pi i\left\{ t\mathfrak{z}\right\} _{p}} \end{align*} which is the Fourier series representation of $\chi_{H,N}$, with $\hat{\chi}_{H,N}\left(t\right)$ being the coefficient of $e^{2\pi i\left\{ t\mathfrak{z}\right\} _{p}}$ in the series. As such: \[ \hat{\chi}_{H,N}\left(t\right)=\frac{\mathbf{1}_{0}\left(p^{N}t\right)}{p^{N}}\sum_{n=0}^{p^{N}-1}\chi_{H}\left(n\right)e^{-2\pi int} \] which proves (I). Next, we split the $n$-sum modulo $p$ and utilize $\chi_{H}$'s functional equation (here $\left|t\right|_{p}\leq p^{N}$): \begin{align*} \hat{\chi}_{H,N}\left(t\right) & =\frac{\mathbf{1}_{0}\left(p^{N}t\right)}{p^{N}}\sum_{j=0}^{p-1}\sum_{n=0}^{p^{N-1}-1}\chi_{H}\left(pn+j\right)e^{-2\pi i\left(pn+j\right)t}\\ & =\frac{\mathbf{1}_{0}\left(p^{N}t\right)}{p^{N}}\sum_{j=0}^{p-1}\sum_{n=0}^{p^{N-1}-1}\frac{a_{j}\chi_{H}\left(n\right)+b_{j}}{d_{j}}e^{-2\pi i\left(pn+j\right)t}\\ & =\frac{\mathbf{1}_{0}\left(p^{N}t\right)}{p^{N}}\sum_{n=0}^{p^{N-1}-1}\left(\sum_{j=0}^{p-1}\frac{a_{j}\chi_{H}\left(n\right)+b_{j}}{d_{j}}e^{-2\pi ijt}\right)e^{-2\pi in\left(pt\right)} \end{align*} The right-hand side can be re-written like so: \[ \frac{\mathbf{1}_{0}\left(p^{N}t\right)}{p^{N-1}}\sum_{n=0}^{p^{N-1}-1}\left(\underbrace{\frac{1}{p}\sum_{j=0}^{p-1}\frac{a_{j}}{d_{j}}e^{-2\pi ijt}}_{\alpha_{H}\left(t\right)}\chi_{H}\left(n\right)+\underbrace{\frac{1}{p}\sum_{j=0}^{p-1}\frac{b_{j}}{d_{j}}e^{-2\pi ijt}}_{\beta_{H}\left(t\right)}\right)e^{-2\pi in\left(pt\right)} \] which leaves us with: \begin{align*} \hat{\chi}_{H,N}\left(t\right) & =\frac{\mathbf{1}_{0}\left(p^{N}t\right)}{p^{N-1}}\sum_{n=0}^{p^{N-1}-1}\left(\alpha_{H}\left(t\right)\chi_{H}\left(n\right)+\beta_{H}\left(t\right)\right)e^{-2\pi in\left(pt\right)}\\ & =\mathbf{1}_{0}\left(p^{N}t\right)\left(\underbrace{\frac{\alpha_{H}\left(t\right)}{p^{N-1}}\sum_{n=0}^{p^{N-1}-1}\chi_{H}\left(n\right)e^{-2\pi in\left(pt\right)}}_{\alpha_{H}\left(t\right)\hat{\chi}_{H,N-1}\left(pt\right)}+\frac{\beta_{H}\left(t\right)}{p^{N-1}}\sum_{n=0}^{p^{N-1}-1}e^{-2\pi in\left(pt\right)}\right) \end{align*} and so: \begin{equation} \hat{\chi}_{H,N}\left(t\right)=\mathbf{1}_{0}\left(p^{N}t\right)\left(\alpha_{H}\left(t\right)\hat{\chi}_{H,N-1}\left(pt\right)+\frac{\beta_{H}\left(t\right)}{p^{N-1}}\sum_{n=0}^{p^{N-1}-1}e^{-2\pi in\left(pt\right)}\right)\label{eq:Chi_H N hat, almost ready to nest} \end{equation} Simplifying the remaining $n$-sum, we find that: \begin{align*} \frac{1}{p^{N-1}}\sum_{n=0}^{p^{N-1}-1}e^{-2\pi in\left(pt\right)} & =\begin{cases} \frac{1}{p^{N-1}}\sum_{n=0}^{p^{N-1}-1}1 & \textrm{if }\left|t\right|_{p}\leq p\\ \frac{1}{p^{N-1}}\frac{\left(e^{-2\pi i\left(pt\right)}\right)^{p^{N-1}}-1}{e^{-2\pi i\left(pt\right)}-1} & \textrm{if }\left|t\right|_{p}\geq p^{2} \end{cases}\\ & =\begin{cases} 1 & \textrm{if }\left|t\right|_{p}\leq p\\ 0 & \textrm{if }\left|t\right|_{p}\geq p^{2} \end{cases}\\ & =\mathbf{1}_{0}\left(pt\right) \end{align*} Hence: \begin{align*} \hat{\chi}_{H,N}\left(t\right) & =\mathbf{1}_{0}\left(p^{N}t\right)\left(\alpha_{H}\left(t\right)\hat{\chi}_{H,N-1}\left(pt\right)+\beta_{H}\left(t\right)\mathbf{1}_{0}\left(pt\right)\right)\\ & =\mathbf{1}_{0}\left(p^{N}t\right)\alpha_{H}\left(t\right)\hat{\chi}_{H,N-1}\left(pt\right)+\mathbf{1}_{0}\left(pt\right)\beta_{H}\left(t\right) \end{align*} This proves (II). With (II) proven, we can then nest (\ref{eq:Chi_H,N hat functional equation}) to derive an explicit formula for $\hat{\chi}_{H,N}\left(t\right)$: \begin{align*} \hat{\chi}_{H,N}\left(t\right) & =\mathbf{1}_{0}\left(p^{N}t\right)\alpha_{H}\left(t\right)\hat{\chi}_{H,N-1}\left(pt\right)+\mathbf{1}_{0}\left(pt\right)\beta_{H}\left(t\right)\\ & =\mathbf{1}_{0}\left(p^{N}t\right)\alpha_{H}\left(t\right)\left(\mathbf{1}_{0}\left(p^{N-1}pt\right)\alpha_{H}\left(pt\right)\hat{\chi}_{H,N-2}\left(p^{2}t\right)+\mathbf{1}_{0}\left(p^{2}t\right)\beta_{H}\left(pt\right)\right)+\mathbf{1}_{0}\left(pt\right)\beta_{H}\left(t\right)\\ & =\mathbf{1}_{0}\left(p^{N}t\right)\alpha_{H}\left(t\right)\alpha_{H}\left(pt\right)\hat{\chi}_{H,N-2}\left(p^{2}t\right)+\mathbf{1}_{0}\left(p^{2}t\right)\alpha_{H}\left(t\right)\beta_{H}\left(pt\right)+\mathbf{1}_{0}\left(pt\right)\beta_{H}\left(t\right)\\ & \vdots\\ & =\mathbf{1}_{0}\left(p^{N}t\right)\left(\prod_{n=0}^{N-2}\alpha_{H}\left(p^{n}t\right)\right)\hat{\chi}_{H,1}\left(p^{N-1}t\right)+\sum_{n=0}^{N-2}\beta_{H}\left(p^{n}t\right)\mathbf{1}_{0}\left(p^{n+1}t\right)\prod_{m=0}^{n-1}\alpha_{H}\left(p^{m}t\right) \end{align*} where the $m$-product is $1$ whenever $n=0$. Finally: \begin{align*} \hat{\chi}_{H,1}\left(t\right) & =\frac{\mathbf{1}_{0}\left(pt\right)}{p}\sum_{j=0}^{p-1}\chi_{H}\left(j\right)e^{-2\pi ijt} \end{align*} Since $\chi_{H}\left(0\right)=0$, $\chi_{H}$'s functional equation gives: \[ \chi_{H}\left(j\right)=\chi_{H}\left(p\cdot0+j\right)=\frac{a_{j}\chi_{H}\left(0\right)+b_{j}}{d_{j}}=\frac{b_{j}}{d_{j}} \] So: \begin{align*} \hat{\chi}_{H,1}\left(t\right) & =\frac{\mathbf{1}_{0}\left(pt\right)}{p}\sum_{j=0}^{p-1}\chi_{H}\left(j\right)e^{-2\pi ijt}=\frac{\mathbf{1}_{0}\left(pt\right)}{p}\sum_{j=0}^{p-1}\frac{b_{j}}{d_{j}}e^{-2\pi ijt}=\mathbf{1}_{0}\left(pt\right)\beta_{H}\left(t\right) \end{align*} Consequently: \begin{align*} \hat{\chi}_{H,N}\left(t\right) & =\mathbf{1}_{0}\left(p^{N}t\right)\left(\prod_{n=0}^{N-2}\alpha_{H}\left(p^{n}t\right)\right)\hat{\chi}_{H,1}\left(p^{N-1}t\right)+\sum_{n=0}^{N-2}\mathbf{1}_{0}\left(p^{n+1}t\right)\beta_{H}\left(p^{n}t\right)\prod_{m=0}^{n-1}\alpha_{H}\left(p^{m}t\right)\\ & =\mathbf{1}_{0}\left(p^{N}t\right)\beta_{H}\left(p^{N-1}t\right)\prod_{n=0}^{N-2}\alpha_{H}\left(p^{n}t\right)+\sum_{n=0}^{N-2}\mathbf{1}_{0}\left(p^{n+1}t\right)\beta_{H}\left(p^{n}t\right)\prod_{m=0}^{n-1}\alpha_{H}\left(p^{m}t\right)\\ & =\sum_{n=0}^{N-1}\mathbf{1}_{0}\left(p^{n+1}t\right)\beta_{H}\left(p^{n}t\right)\prod_{m=0}^{n-1}\alpha_{H}\left(p^{m}t\right) \end{align*} This proves (\ref{eq:Explicit formula for Chi_H,N hat}). Q.E.D. \vphantom{} Because $\chi_{H}$ is rising-continuous, rather than continuous, for any fixed $t\in\hat{\mathbb{Z}}_{p}$, the limit of $\hat{\chi}_{H,N}\left(t\right)$ as $N\rightarrow\infty$ need not converge in $\mathbb{C}_{q}$. To overcome this, we perform a kind of asymptotic analysis, excising as much as possible the dependence of (\ref{eq:Explicit formula for Chi_H,N hat}) on $N$. Instead, we \emph{reverse} the dependence of $t$ and $n$ or $N$ in (\ref{eq:Explicit formula for Chi_H,N hat}). That this can be done hinges on the observation that, for any fixed $t\in\hat{\mathbb{Z}}_{p}$, the congruence $p^{m}t\overset{1}{\equiv}0$ will be satisfied for all sufficiently large $m$\textemdash namely, all $m\geq-v_{p}\left(t\right)$. So, as $n\rightarrow\infty$, the $m$th term of the product: \begin{equation} \prod_{m=0}^{n-1}\alpha_{H}\left(p^{m}t\right) \end{equation} will be $\alpha_{H}\left(0\right)$ for all $m\geq-v_{p}\left(t\right)+1$. This suggests that we can untangle the dependency of this product on $n$ by breaking it up into the partial product from $m=0$ to $m=-v_{p}\left(t\right)-1$, and a remainder product which will be independent of $t$. This partial product is precisely $\hat{A}_{H}\left(t\right)$. \begin{defn}[$\hat{A}_{H}\left(t\right)$] We\index{hat{A}{H}left(tright)@$\hat{A}_{H}\left(t\right)$} define the function \nomenclature{$\hat{A}_{H}\left(t\right)$}{ }$\hat{A}_{H}:\hat{\mathbb{Z}}_{p}\rightarrow\overline{\mathbb{Q}}$ by: \begin{equation} \hat{A}_{H}\left(t\right)\overset{\textrm{def}}{=}\prod_{m=0}^{-v_{p}\left(t\right)-1}\alpha_{H}\left(p^{m}t\right),\textrm{ }\forall t\in\hat{\mathbb{Z}}_{p}\label{eq:Definition of A_H hat} \end{equation} where the $m$-product is defined to be $1$ whenever $t=0$; that is, $\hat{A}_{H}\left(0\right)\overset{\textrm{def}}{=}1$. \end{defn} \vphantom{} Our next proposition actually does the work of untangling $\prod_{m=0}^{n-1}\alpha_{H}\left(p^{m}t\right)$. \begin{prop}[\textbf{$\alpha_{H}$-product in terms of $\hat{A}_{H}$}] \label{prop:alpha product in terms of A_H hat}Fix $t\in\hat{\mathbb{Z}}_{p}\backslash\left\{ 0\right\} $. Then: \begin{align} \mathbf{1}_{0}\left(p^{n+1}t\right)\prod_{m=0}^{n-1}\alpha_{H}\left(p^{m}t\right) & =\begin{cases} 0 & \textrm{if }n\leq-v_{p}\left(t\right)-2\\ \frac{\hat{A}_{H}\left(t\right)}{\alpha_{H}\left(\frac{t\left|t\right|_{p}}{p}\right)} & \textrm{if }n=-v_{p}\left(t\right)-1\\ \left(\alpha_{H}\left(0\right)\right)^{n+v_{p}\left(t\right)}\hat{A}_{H}\left(t\right) & \textrm{if }n\geq-v_{p}\left(t\right) \end{cases}\label{eq:alpha product in terms of A_H hat} \end{align} \end{prop} Proof: Fix $t\in\hat{\mathbb{Z}}_{p}\backslash\left\{ 0\right\} $. Then, we can write: \begin{align*} \mathbf{1}_{0}\left(p^{n+1}t\right)\prod_{m=0}^{n-1}\alpha_{H}\left(p^{m}t\right) & =\begin{cases} 0 & \textrm{if }\left|t\right|_{p}\geq p^{n+2}\\ \prod_{m=0}^{n-1}\alpha_{H}\left(p^{m}t\right) & \textrm{if }\left|t\right|_{p}\leq p^{n+1} \end{cases}\\ & =\begin{cases} 0 & \textrm{if }n\leq-v_{p}\left(t\right)-2\\ \prod_{m=0}^{n-1}\alpha_{H}\left(p^{m}t\right) & \textrm{if }n\geq-v_{p}\left(t\right)-1 \end{cases} \end{align*} As remarked above $m\geq-v_{p}\left(t\right)$ makes the congruence $p^{m}t\overset{1}{\equiv}0$ hold true, guaranteeing that $\alpha_{H}\left(p^{m}t\right)=\alpha_{H}\left(0\right)$ for all $m\geq-v_{p}\left(t\right)$. So, for fixed $t$, we prepare our decomposition like so: \[ \prod_{m=0}^{n-1}\alpha_{H}\left(p^{m}t\right)=\begin{cases} \prod_{m=0}^{-v_{p}\left(t\right)-2}\alpha_{H}\left(p^{m}t\right) & \textrm{if }n=-v_{p}\left(t\right)-1\\ \prod_{m=0}^{-v_{p}\left(t\right)-1}\alpha_{H}\left(p^{m}t\right) & \textrm{if }n=-v_{p}\left(t\right)\\ \prod_{m=0}^{-v_{p}\left(t\right)-1}\alpha_{H}\left(p^{m}t\right)\times\prod_{k=-v_{p}\left(t\right)}^{n-1}\alpha_{H}\left(p^{k}t\right) & \textrm{if }n\geq-v_{p}\left(t\right)+1 \end{cases} \] Since $\alpha_{H}\left(p^{k}t\right)=\alpha_{H}\left(0\right)$ for all terms of the $k$-product on the bottom line, we can write: \begin{align*} \prod_{m=0}^{n-1}\alpha_{H}\left(p^{m}t\right) & =\begin{cases} \prod_{m=0}^{-v_{p}\left(t\right)-2}\alpha_{H}\left(p^{m}t\right) & \textrm{if }n=-v_{p}\left(t\right)-1\\ \left(\alpha_{H}\left(0\right)\right)^{n+v_{p}\left(t\right)}\prod_{m=0}^{-v_{p}\left(t\right)-1}\alpha_{H}\left(p^{m}t\right) & \textrm{if }n\geq-v_{p}\left(t\right) \end{cases}\\ \left(\times\&\div\textrm{ by }\alpha_{H}\left(p^{-v_{p}\left(t\right)-1}t\right)\right); & =\begin{cases} \frac{\hat{A}_{H}\left(t\right)}{\alpha_{H}\left(p^{-v_{p}\left(t\right)-1}t\right)} & \textrm{if }n=-v_{p}\left(t\right)-1\\ \left(\alpha_{H}\left(0\right)\right)^{n+v_{p}\left(t\right)}\hat{A}_{H}\left(t\right) & \textrm{if }n\geq-v_{p}\left(t\right) \end{cases} \end{align*} which gives us the desired formula. Q.E.D. \vphantom{} Now, we learn how to express the $\alpha_{H}$ product as a series. \begin{prop}[$\alpha_{H}$\textbf{ series formula}] \label{prop:alpha product series expansion} \end{prop} \begin{equation} \prod_{m=0}^{n-1}\alpha_{H}\left(p^{m}t\right)=\left(\frac{\mu_{0}}{p^{2}}\right)^{n}\sum_{m=0}^{p^{n}-1}\kappa_{H}\left(m\right)e^{-2\pi imt},\textrm{ }\forall n\geq0,\textrm{ }\forall t\in\hat{\mathbb{Z}}_{p}\label{eq:alpha_H product expansion} \end{equation} Proof: We start by writing: \[ \prod_{m=0}^{n-1}\alpha_{H}\left(p^{m}t\right)=\prod_{m=0}^{n-1}\left(\sum_{j=0}^{p-1}\frac{\mu_{j}}{p^{2}}e^{-2\pi ij\left(p^{m}t\right)}\right) \] and then apply (\ref{eq:M_H partial sum generating identity}) from \textbf{Proposition \ref{prop:Generating function identities}} with $c_{j}=\mu_{j}/p^{2}$ and $z=e^{-2\pi it}$: \begin{align*} \prod_{m=0}^{n-1}\alpha_{H}\left(p^{m}t\right) & =\left(\frac{\mu_{0}}{p^{2}}\right)^{n}\sum_{m=0}^{p^{n}-1}\left(\frac{\mu_{0}}{p^{2}}\right)^{-\lambda_{p}\left(m\right)}\left(\prod_{j=0}^{p-1}\left(\frac{\mu_{j}}{p^{2}}\right)^{\#_{p:j}\left(m\right)}\right)e^{-2\pi imt}\\ \left(\sum_{j=0}^{p-1}\#_{p:j}\left(m\right)=\lambda_{p}\left(m\right)\right); & =\left(\frac{\mu_{0}}{p^{2}}\right)^{n}\sum_{m=0}^{p^{n}-1}\left(\frac{\mu_{0}}{p^{2}}\right)^{-\lambda_{p}\left(m\right)}\left(\frac{\prod_{j=0}^{p-1}\mu_{j}^{\#_{p:j}\left(m\right)}}{p^{2\lambda_{p}\left(m\right)}}\right)e^{-2\pi imt}\\ \left(M_{H}\left(m\right)=\frac{\prod_{j=0}^{p-1}\mu_{j}^{\#_{p:j}\left(m\right)}}{p^{\lambda_{p}\left(m\right)}}\right); & =\left(\frac{\mu_{0}}{p^{2}}\right)^{n}\sum_{m=0}^{p^{n}-1}\left(\frac{\mu_{0}}{p^{2}}\right)^{-\lambda_{p}\left(m\right)}\frac{M_{H}\left(m\right)}{p^{\lambda_{p}\left(m\right)}}e^{-2\pi imt}\\ & =\left(\frac{\mu_{0}}{p^{2}}\right)^{n}\sum_{m=0}^{p^{n}-1}\left(\frac{p}{\mu_{0}}\right)^{\lambda_{p}\left(m\right)}M_{H}\left(m\right)e^{-2\pi imt}\\ & =\left(\frac{\mu_{0}}{p^{2}}\right)^{n}\sum_{m=0}^{p^{n}-1}\kappa_{H}\left(m\right)e^{-2\pi imt} \end{align*} Q.E.D. \vphantom{} This formula reveals $\hat{A}_{H}$ as being a radially-magnitudinal function. A simple estimate shows that that $\hat{A}_{H}$ is the Fourier-Stieltjes transform of a $\left(p,q\right)$-adic measure. \begin{prop} $\hat{A}_{H}\left(t\right)$ is the Fourier-Stieltjes transform of a $\left(p,q\right)$-adic measure. \end{prop} Proof: Let $H$ be semi-basic. Then, $p$ and the $d_{j}$s are co-prime to both $q$ and the $a_{j}$s. So, we get: \[ \left|\alpha_{H}\left(t\right)\right|_{q}=\left|\sum_{j=0}^{p-1}\frac{\mu_{j}}{p^{2}}e^{-2\pi ijt}\right|_{q}=\left|\sum_{j=0}^{p-1}\frac{a_{j}}{pd_{j}}e^{-2\pi ijt}\right|_{q}\leq\max_{0\leq j\leq p-1}\left|\frac{a_{j}}{pd_{j}}\right|_{q}\leq1 \] Hence: \begin{align*} \sup_{t\in\hat{\mathbb{Z}}_{p}}\left|\hat{A}_{H}\left(t\right)\right|_{q} & \leq\max\left\{ 1,\sup_{t\in\hat{\mathbb{Z}}_{p}\backslash\left\{ 0\right\} }\prod_{n=0}^{-v_{p}\left(t\right)-1}\left|\alpha_{H}\left(p^{n}t\right)\right|_{q}\right\} \\ & \leq\max\left\{ 1,\sup_{t\in\hat{\mathbb{Z}}_{p}\backslash\left\{ 0\right\} }\prod_{n=0}^{-v_{p}\left(t\right)-1}1\right\} \\ & =1 \end{align*} This shows $\hat{A}_{H}$ is then in $B\left(\hat{\mathbb{Z}}_{p},\mathbb{C}_{q}\right)$. As such, the map: \begin{equation} f\in C\left(\mathbb{Z}_{p},\mathbb{C}_{q}\right)\mapsto\sum_{t\in\hat{\mathbb{Z}}_{p}}\hat{f}\left(-t\right)\hat{A}_{H}\left(t\right)\in\mathbb{C}_{q} \end{equation} is a continuous linear functional on $C\left(\mathbb{Z}_{p},\mathbb{C}_{q}\right)$\textemdash that is, a $\left(p,q\right)$-adic measure. Q.E.D. \vphantom{} Here, we introduce the notation for the partial sums of the Fourier series generated by $\hat{A}_{H}$. \begin{defn} We\index{$dA_{H}$} write \nomenclature{$dA_{H}$}{ }$dA_{H}$ to denote the $\left(p,q\right)$-adic measure whose Fourier-Stieltjes transform is $\hat{A}_{H}\left(t\right)$. We then have \nomenclature{$\tilde{A}_{H,N}\left(\mathfrak{z}\right)$}{$N$th partial Fourier series generated by $\hat{A}_{H}\left(t\right)$}: \begin{equation} \tilde{A}_{H,N}\left(\mathfrak{z}\right)\overset{\textrm{def}}{=}\sum_{\left|t\right|_{p}\leq p^{N}}\hat{A}_{H}\left(t\right)e^{2\pi i\left\{ t\mathfrak{z}\right\} _{p}}\label{eq:Definition of A_H,N twiddle} \end{equation} \end{defn} \vphantom{} The theorem given below summarizes the main properties of $dA_{H}$: \begin{thm}[\textbf{Properties of $dA_{H}$}] \label{thm:Properties of dA_H}\index{$dA_{H}$!properties of} \ \vphantom{} I. ($dA_{H}$ is radially-magnitudinal and $\left(p,q\right)$-adically regular) \begin{equation} \hat{A}_{H}\left(t\right)=\begin{cases} 1 & \textrm{if }t=0\\ \frac{1}{\left|t\right|_{p}^{2-\log_{p}\mu_{0}}}\sum_{m=0}^{\left|t\right|_{p}-1}\kappa_{H}\left(m\right)e^{-2\pi imt} & \textrm{else} \end{cases},\textrm{ }\forall t\in\hat{\mathbb{Z}}_{p}\label{eq:A_H hat as the product of radially symmetric and magnitude-dependent measures} \end{equation} \vphantom{} II. (Formula for $\tilde{A}_{H,N}\left(\mathfrak{z}\right)$) For any $N\in\mathbb{N}_{0}$ and $\mathfrak{z}\in\mathbb{Z}_{p}$: \begin{equation} \tilde{A}_{H,N}\left(\mathfrak{z}\right)\overset{\overline{\mathbb{Q}}}{=}\left(\frac{\mu_{0}}{p}\right)^{N}\kappa_{H}\left(\left[\mathfrak{z}\right]_{p^{N}}\right)+\left(1-\alpha_{H}\left(0\right)\right)\sum_{n=0}^{N-1}\left(\frac{\mu_{0}}{p}\right)^{n}\kappa_{H}\left(\left[\mathfrak{z}\right]_{p^{n}}\right)\label{eq:Convolution of dA_H and D_N} \end{equation} \vphantom{} III. ($q$-adic convergence of $\tilde{A}_{H,N}\left(\mathfrak{z}\right)$) As $N\rightarrow\infty$, \emph{(\ref{eq:Convolution of dA_H and D_N})} converges in $\mathbb{C}_{q}$ (in fact, $\mathbb{Z}_{q}$) for all $\mathfrak{z}\in\mathbb{Z}_{p}^{\prime}$, with: \begin{equation} \lim_{N\rightarrow\infty}\tilde{A}_{H,N}\left(\mathfrak{z}\right)\overset{\mathbb{C}_{q}}{=}\left(1-\alpha_{H}\left(0\right)\right)\sum_{n=0}^{\infty}\left(\frac{\mu_{0}}{p}\right)^{n}\kappa_{H}\left(\left[\mathfrak{z}\right]_{p^{n}}\right),\textrm{ }\forall\mathfrak{z}\in\mathbb{Z}_{p}^{\prime}\label{eq:Derivative of dA_H on Z_rho prime} \end{equation} This convergence is point-wise. \vphantom{} IV. (Convergence of $\tilde{A}_{H,N}\left(\mathfrak{z}\right)$ in $\mathbb{C}$) As $N\rightarrow\infty$, \emph{(\ref{eq:Convolution of dA_H and D_N})} either converges in $\mathbb{C}$ for all $\mathfrak{z}\in\mathbb{N}_{0}$, which occurs if and only if $H$ is contracting ($\mu_{0}/p<1$); or diverges in $\mathbb{C}$ for all $\mathfrak{z}\in\mathbb{N}_{0}$, which occurs if and only if $H$ is expanding ($\mu_{0}/p>1$). The convergence/divergence in both cases is point-wise. Moreover, if convergence occurs, the limit in $\mathbb{C}$ is given by \begin{equation} \lim_{N\rightarrow\infty}\tilde{A}_{H,N}\left(m\right)\overset{\mathbb{C}}{=}\left(1-\alpha_{H}\left(0\right)\right)\sum_{n=0}^{\infty}\left(\frac{\mu_{0}}{p}\right)^{n}\kappa_{H}\left(\left[m\right]_{p^{n}}\right),\textrm{ }\forall m\in\mathbb{N}_{0}\label{eq:derivative of dA_H on N_0} \end{equation} \vphantom{} V. (Characterization of degeneracy)\index{$dA_{H}$!degeneracy} If $H$ is contracting, then $dA_{H}$ is degenerate if and only if $\alpha_{H}\left(0\right)=1$. \end{thm} Proof: I. By \textbf{Proposition \ref{prop:alpha product series expansion}}, we have that: \begin{equation} \hat{A}_{H}\left(t\right)=\prod_{m=0}^{-v_{p}\left(t\right)-1}\alpha_{H}\left(p^{m}t\right)=\left(\frac{\mu_{0}}{p^{2}}\right)^{-v_{p}\left(t\right)}\sum_{m=0}^{p^{-v_{p}\left(t\right)}-1}\kappa_{H}\left(m\right)e^{-2\pi imt} \end{equation} and hence: \begin{equation} \hat{A}_{H}\left(t\right)=\prod_{m=0}^{-v_{p}\left(t\right)-1}\alpha_{H}\left(p^{m}t\right)=\frac{1}{\left|t\right|_{p}^{2-\log_{p}\mu_{0}}}\sum_{m=0}^{\left|t\right|_{p}-1}\kappa_{H}\left(m\right)e^{-2\pi imt} \end{equation} \vphantom{} II. Since: \[ \hat{A}_{H}\left(t\right)=\prod_{n=0}^{-v_{p}\left(t\right)-1}\alpha_{H}\left(p^{n}t\right)=\left(\frac{\mu_{0}}{p^{2}}\right)^{-v_{p}\left(t\right)}\sum_{m=0}^{\left|t\right|_{p}-1}\kappa_{H}\left(m\right)e^{-2\pi imt} \] we can use the Radial-Magnitudinal Fourier Resummation Lemma (\textbf{Lemma \ref{lem:Radially-Mag Fourier Resummation Lemma}}) to write: \begin{align*} \sum_{\left|t\right|_{p}\leq p^{N}}\hat{A}_{H}\left(t\right)e^{2\pi i\left\{ t\mathfrak{z}\right\} _{p}} & \overset{\overline{\mathbb{Q}}}{=}\hat{A}_{H}\left(0\right)+\sum_{n=1}^{N}\left(\frac{\mu_{0}}{p^{2}}\right)^{n}\left(p^{n}\kappa_{H}\left(\left[\mathfrak{z}\right]_{p^{n}}\right)-p^{n-1}\kappa_{H}\left(\left[\mathfrak{z}\right]_{p^{n-1}}\right)\right)\\ & -\sum_{j=1}^{p-1}\sum_{n=1}^{N}p^{n-1}\left(\frac{\mu_{0}}{p^{2}}\right)^{n}\kappa_{H}\left(\left[\mathfrak{z}\right]_{p^{n-1}}+jp^{n-1}\right) \end{align*} By the $\left(p,q\right)$-adic regularity of $\kappa_{H}$, we then have: \[ \sum_{\left|t\right|_{p}\leq p^{N}}\hat{A}_{H}\left(t\right)e^{2\pi i\left\{ t\mathfrak{z}\right\} _{p}}\overset{\overline{\mathbb{Q}}}{=}1+\sum_{n=1}^{N}\left(\frac{\mu_{0}}{p}\right)^{n}\kappa_{H}\left(\left[\mathfrak{z}\right]_{p^{n}}\right)-\underbrace{\left(\frac{1}{p}\sum_{j=0}^{p-1}\frac{\mu_{j}}{p}\right)}_{\alpha_{H}\left(0\right)}\sum_{n=0}^{N-1}\left(\frac{\mu_{0}}{p}\right)^{n}\kappa_{H}\left(\left[\mathfrak{z}\right]_{p^{n}}\right) \] and hence (since $\kappa_{H}\left(0\right)=1$): \begin{equation} \tilde{A}_{H,N}\left(\mathfrak{z}\right)\overset{\overline{\mathbb{Q}}}{=}\left(\frac{\mu_{0}}{p}\right)^{N}\kappa_{H}\left(\left[\mathfrak{z}\right]_{p^{N}}\right)+\left(1-\alpha_{H}\left(0\right)\right)\sum_{n=0}^{N-1}\left(\frac{\mu_{0}}{p}\right)^{n}\kappa_{H}\left(\left[\mathfrak{z}\right]_{p^{n}}\right) \end{equation} as desired. \vphantom{} III. Fix $\mathfrak{z}\in\mathbb{Z}_{p}^{\prime}$. Then, since $H$ is semi-basic, we have: \begin{equation} \left|\left(\frac{\mu_{0}}{p}\right)^{n}\kappa_{H}\left(\left[\mathfrak{z}\right]_{p^{n}}\right)\right|_{q}=\left|\kappa_{H}\left(\left[\mathfrak{z}\right]_{p^{n}}\right)\right|_{q} \end{equation} So, by \textbf{Proposition \ref{prop:Properties of Kappa_H}}, $\kappa_{H}\left(\left[\mathfrak{z}\right]_{p^{n}}\right)$ converges $q$-adically to $0$ as $n\rightarrow\infty$.The ultrametric structure of $\mathbb{C}_{q}$ then guarantees the convergence of (\ref{eq:Derivative of dA_H on Z_rho prime}). \vphantom{} IV. Let $\mathfrak{z}\in\mathbb{N}_{0}$. Then, $\kappa_{H}\left(\mathfrak{z}\right)=\kappa_{H}\left(\left[\mathfrak{z}\right]_{p^{n}}\right)=\kappa_{H}\left(\left[\mathfrak{z}\right]_{p^{n+1}}\right)\in\mathbb{Q}\backslash\left\{ 0\right\} $ for all $n\geq\lambda_{p}\left(\mathfrak{z}\right)$. So, let $c$ denote the value of $M_{H}\left(\left[\mathfrak{z}\right]_{p^{n}}\right)$ for all such $n$. Examining the tail of the $n$-series in (\ref{eq:Convolution of dA_H and D_N}), $N\geq\lambda_{p}\left(\mathfrak{z}\right)+1$ implies: \begin{align*} \sum_{n=0}^{N-1}\left(\frac{\mu_{0}}{p}\right)^{n}\kappa_{H}\left(\left[\mathfrak{z}\right]_{p^{n}}\right) & \overset{\mathbb{R}}{=}\sum_{n=0}^{\lambda_{p}\left(\mathfrak{z}\right)-1}\left(\frac{\mu_{0}}{p}\right)^{n}\kappa_{H}\left(\left[\mathfrak{z}\right]_{p^{n}}\right)+\sum_{n=\lambda_{p}\left(\mathfrak{z}\right)}^{N-1}\left(\frac{\mu_{0}}{p}\right)^{n}\kappa_{H}\left(\left[\mathfrak{z}\right]_{p^{n}}\right)\\ & \overset{\mathbb{R}}{=}O\left(1\right)+\kappa_{H}\left(\mathfrak{z}\right)\sum_{n=\lambda_{p}\left(\mathfrak{z}\right)}^{N-1}\left(\frac{\mu_{0}}{p}\right)^{n} \end{align*} The series on the bottom line is a \emph{geometric} series, and hence, is convergent in $\mathbb{C}$ if and only if $\mu_{0}/p<1$. Likewise, by \textbf{Proposition \ref{prop:Properties of Kappa_H}},\textbf{ }$\left(\frac{\mu_{0}}{p}\right)^{N}\kappa_{H}\left(\left[\mathfrak{z}\right]_{p^{N}}\right)$ tends to $0$ as $N\rightarrow\infty$ if and only if $\mu_{0}/p<1$, and tends to $\infty$ if and only if $\mu_{0}/p>1$. \vphantom{} V. Letting $H$ be semi-basic and contracting, (\ref{eq:Derivative of dA_H on Z_rho prime}) shows that $\lim_{N\rightarrow\infty}\tilde{A}_{H,N}$ will be identically zero if and only if $\alpha_{H}\left(0\right)=1$. Q.E.D. \vphantom{} Using the above, we can give the formulae for integrating $\left(p,q\right)$-adic functions against $dA_{H}$. \begin{cor}[\textbf{$dA_{H}$ Integration Formulae}] \ \vphantom{} I. \index{$dA_{H}$!integration}($dA_{H}$ is a probability measure\footnote{In the sense of \cite{Probabilities taking values in non-archimedean fields,Measure-theoretic approach to p-adic probability theory}.}): \begin{equation} \int_{\mathbb{Z}_{p}}dA_{H}\left(\mathfrak{z}\right)\overset{\mathbb{C}_{q}}{=}1\label{eq:dA_H is a probabiity measure} \end{equation} Also, for all $n\in\mathbb{N}_{1}$ and all $k\in\left\{ 0,\ldots,p^{n}-1\right\} $: \begin{align} \int_{\mathbb{Z}_{p}}\left[\mathfrak{z}\overset{p^{n}}{\equiv}k\right]dA_{H}\left(\mathfrak{z}\right) & \overset{\overline{\mathbb{Q}}}{=}\left(\frac{\mu_{0}}{p}\right)^{n}\kappa_{H}\left(k\right)+\left(1-\alpha_{H}\left(0\right)\right)\sum_{m=0}^{n-1}\left(\frac{\mu_{0}}{p}\right)^{m}\kappa_{H}\left(\left[k\right]_{p^{m}}\right)\label{eq:integration of indicator functions dA_H} \end{align} \vphantom{} II. Let $f\in C\left(\mathbb{Z}_{p},\mathbb{C}_{q}\right)$. Then: \begin{align} \int_{\mathbb{Z}_{p}}f\left(\mathfrak{z}\right)dA_{H}\left(\mathfrak{z}\right) & \overset{\mathbb{C}_{q}}{=}\lim_{N\rightarrow\infty}\left(\left(\frac{\mu_{0}}{p^{2}}\right)^{N}\sum_{n=0}^{p^{N}-1}\kappa_{H}\left(n\right)f\left(n\right)\right.\label{eq:Truncation-based formula for integral of f dA_H}\\ & +\left.\frac{1-\alpha_{H}\left(0\right)}{p^{N}}\sum_{n=0}^{p^{N}-1}\sum_{m=0}^{N-1}\left(\frac{\mu_{0}}{p}\right)^{m}\kappa_{H}\left(\left[n\right]_{p^{m}}\right)f\left(n\right)\right)\nonumber \end{align} \end{cor} \begin{rem} One can also compute $\int_{\mathbb{Z}_{p}}f\left(\mathfrak{z}\right)dA_{H}\left(\mathfrak{z}\right)$ by expressing $f$ as a van der Put series and then integrating term-by-term; this yields formulae nearly identical to the ones given above, albeit in terms of $c_{n}\left(f\right)$ (the $n$th van der Put coefficient of $f$), rather than $f\left(n\right)$. \end{rem} Proof: (I) follows by using (\ref{eq:Convolution of dA_H and D_N}) along with the fact that, by definition: \begin{equation} \int_{\mathbb{Z}_{p}}\left[\mathfrak{z}\overset{p^{n}}{\equiv}k\right]dA_{H}\left(\mathfrak{z}\right)=\frac{1}{p^{n}}\sum_{\left|t\right|_{p}\leq p^{n}}\hat{A}_{H}\left(t\right)e^{-2\pi i\left\{ tk\right\} _{p}} \end{equation} and that $\left[k\right]_{p^{n}}=k$ if and only if $k\in\left\{ 0,\ldots,p^{n}-1\right\} $. As for (II), let $f\in C\left(\mathbb{Z}_{p},\mathbb{C}_{q}\right)$ be arbitrary. Then, the truncations $f_{N}$ converge $q$-adically to $f$ uniformly over $\mathbb{Z}_{p}$. Consequently: \begin{align*} \int_{\mathbb{Z}_{p}}f\left(\mathfrak{z}\right)dA_{H}\left(\mathfrak{z}\right) & \overset{\mathbb{C}_{q}}{=}\lim_{N\rightarrow\infty}\sum_{n=0}^{p^{N}-1}f\left(n\right)\int_{\mathbb{Z}_{p}}\left[\mathfrak{z}\overset{p^{N}}{\equiv}n\right]dA_{H}\left(\mathfrak{z}\right)\\ \left(\textrm{Use (I)}\right); & =\lim_{N\rightarrow\infty}\sum_{n=0}^{p^{N}-1}f\left(n\right)\left(\frac{1}{p^{N}}\left(\frac{\mu_{0}}{p}\right)^{N}\kappa_{H}\left(n\right)\right)\\ & +\lim_{N\rightarrow\infty}\sum_{n=0}^{p^{N}-1}f\left(n\right)\frac{1-\alpha_{H}\left(0\right)}{p^{N}}\sum_{m=0}^{N-1}\left(\frac{\mu_{0}}{p}\right)^{m}\kappa_{H}\left(\left[n\right]_{p^{m}}\right) \end{align*} Q.E.D. \vphantom{} Given the import of the shortened $qx+1$ map\index{$qx+1$ map}s, let me give the details for what happens with their associated $A_{H}$s. \begin{cor}[\textbf{$dA_{H}$ for $qx+1$}] \label{cor:Formula for Nth partial sum of the Fourier series generated by A_q hat}Let $q$ be an odd integer $\geq3$, and let $A_{q}$ denote $A_{H}$ where $H=T_{q}$ is the shortened $qx+1$ map. Finally, write: \begin{equation} \tilde{A}_{q,N}\left(\mathfrak{z}\right)\overset{\textrm{def}}{=}\sum_{\left|t\right|_{2}\leq2^{N}}\hat{A}_{q}\left(t\right)e^{2\pi i\left\{ t\mathfrak{z}\right\} _{2}}=\left(D_{2:N}*dA_{q}\right)\left(\mathfrak{z}\right)\label{eq:Definition of A_q,n twiddle} \end{equation} Then: \begin{equation} \tilde{A}_{q,N}\left(\mathfrak{z}\right)=\frac{q^{\#_{1}\left(\left[\mathfrak{z}\right]_{2^{N}}\right)}}{2^{N}}+\left(1-\frac{q+1}{4}\right)\sum_{n=0}^{N-1}\frac{q^{\#_{1}\left(\left[\mathfrak{z}\right]_{2^{n}}\right)}}{2^{n}}\label{eq:Convolution of dA_q and D_N} \end{equation} where, for any integer $m\geq0$, $\#_{1}\left(m\right)$ is the number of $1$s in the binary representation of $m$ (a shorthand for $\#_{2:1}\left(m\right)$). Integration against $dA_{q}$ is then given by: \begin{align} \int_{\mathbb{Z}_{2}}f\left(\mathfrak{z}\right)dA_{q}\left(\mathfrak{z}\right) & \overset{\mathbb{C}_{q}}{=}\lim_{N\rightarrow\infty}\frac{1}{4^{N}}\sum_{n=0}^{2^{N}-1}f\left(n\right)q^{\#_{1}\left(n\right)}\\ & +\frac{3-q}{4}\lim_{N\rightarrow\infty}\frac{1}{2^{N}}\sum_{n=0}^{2^{N}-1}f\left(n\right)\sum_{m=0}^{N-1}\frac{q^{\#_{1}\left(\left[n\right]_{2^{m}}\right)}}{2^{m}}\nonumber \end{align} In particular, for $q=3$ (the Shortened Collatz map\index{Collatz!map}), we have: \begin{equation} \tilde{A}_{3,N}\left(\mathfrak{z}\right)=\frac{3^{\#_{1}\left(\left[\mathfrak{z}\right]_{2^{N}}\right)}}{2^{N}}\label{eq:A_3 measure of a coset in Z_2} \end{equation} and hence: \begin{equation} \int_{\mathbb{Z}_{2}}f\left(\mathfrak{z}\right)dA_{3}\left(\mathfrak{z}\right)\overset{\mathbb{C}_{3}}{=}\lim_{N\rightarrow\infty}\frac{1}{4^{N}}\sum_{n=0}^{2^{N}-1}3^{\#_{1}\left(n\right)}f\left(n\right),\textrm{ }\forall f\in C\left(\mathbb{Z}_{2},\mathbb{C}_{3}\right)\label{eq:Integral of f dA_3} \end{equation} \end{cor} Proof: Use (\ref{eq:Truncation-based formula for integral of f dA_H}). Q.E.D. \vphantom{} By using $\hat{A}_{H}$ and \textbf{Proposition \ref{prop:alpha product in terms of A_H hat}}, we can re-write our explicit formula for $\hat{\chi}_{H,N}$\textemdash equation (\ref{eq:Explicit formula for Chi_H,N hat}) from \textbf{Proposition \ref{prop:Computation of Chi_H,N hat}}\textemdash in a way that separates the dependence of $t$ and $N$. Like in previous computations, the formulae for \index{hat{chi}{H,N}@$\hat{\chi}_{H,N}$!left(N,tright) asymptotic decomposition@$\left(N,t\right)$ asymptotic decomposition}$\hat{\chi}_{H,N}$ come in two forms, based on whether or not $\alpha_{H}\left(0\right)=1$. \begin{thm}[\textbf{$\left(N,t\right)$-Asymptotic Decomposition of $\hat{\chi}_{H,N}$}] \label{thm:(N,t) asymptotic decomposition of Chi_H,N hat}Recall that we write $\gamma_{H}\left(t\right)$ to denote $\beta_{H}\left(t\right)/\alpha_{H}\left(t\right)$. \vphantom{} I. If $\alpha_{H}\left(0\right)=1$, then: \begin{equation} \hat{\chi}_{H,N}\left(t\right)=\begin{cases} \beta_{H}\left(0\right)N\hat{A}_{H}\left(t\right) & \textrm{if }t=0\\ \left(\gamma_{H}\left(\frac{t\left|t\right|_{p}}{p}\right)+\beta_{H}\left(0\right)\left(N+v_{p}\left(t\right)\right)\right)\hat{A}_{H}\left(t\right) & \textrm{if }0<\left|t\right|_{p}<p^{N}\\ \gamma_{H}\left(\frac{t\left|t\right|_{p}}{p}\right)\hat{A}_{H}\left(t\right) & \textrm{if }\left|t\right|_{p}=p^{N}\\ 0 & \textrm{if }\left|t\right|_{p}>p^{N} \end{cases},\textrm{ }\forall t\in\hat{\mathbb{Z}}_{p}\label{eq:Fine Structure Formula for Chi_H,N hat when alpha is 1} \end{equation} \vphantom{} II. If $\alpha_{H}\left(0\right)\neq1$. Then: \begin{equation} \hat{\chi}_{H,N}\left(t\right)=\begin{cases} \beta_{H}\left(0\right)\frac{\left(\alpha_{H}\left(0\right)\right)^{N}-1}{\alpha_{H}\left(0\right)-1}\hat{A}_{H}\left(t\right) & \textrm{if }t=0\\ \left(\gamma_{H}\left(\frac{t\left|t\right|_{p}}{p}\right)+\beta_{H}\left(0\right)\frac{\left(\alpha_{H}\left(0\right)\right)^{N+v_{p}\left(t\right)}-1}{\alpha_{H}\left(0\right)-1}\right)\hat{A}_{H}\left(t\right) & \textrm{if }0<\left|t\right|_{p}<p^{N}\\ \gamma_{H}\left(\frac{t\left|t\right|_{p}}{p}\right)\hat{A}_{H}\left(t\right) & \textrm{if }\left|t\right|_{p}=p^{N}\\ 0 & \textrm{if }\left|t\right|_{p}>p^{N} \end{cases},\textrm{ }\forall t\in\hat{\mathbb{Z}}_{p}\label{eq:Fine Structure Formula for Chi_H,N hat when alpha is not 1} \end{equation} \end{thm} Proof: For brevity, we write: \begin{equation} \hat{A}_{H,n}\left(t\right)\overset{\textrm{def}}{=}\begin{cases} 1 & \textrm{if }n=0\\ \mathbf{1}_{0}\left(p^{n+1}t\right)\prod_{m=0}^{n-1}\alpha_{H}\left(p^{m}t\right) & \textrm{if }n\geq1 \end{cases}\label{eq:Definition of A_H,n+1 hat} \end{equation} so that (\ref{eq:Explicit formula for Chi_H,N hat}) becomes: \[ \hat{\chi}_{H,N}\left(t\right)=\sum_{n=0}^{N-1}\beta_{H}\left(p^{n}t\right)\hat{A}_{H,n}\left(t\right) \] Then, letting $t\in\hat{\mathbb{Z}}_{p}$ be non-zero and satisfy $\left|t\right|_{p}\leq p^{N}$, we use \textbf{Proposition \ref{prop:alpha product in terms of A_H hat}} to write: \begin{equation} \hat{\chi}_{H,N}\left(t\right)\overset{\overline{\mathbb{Q}}}{=}\sum_{n=0}^{N-1}\beta_{H}\left(p^{n}t\right)\hat{A}_{H,n}\left(t\right)=\sum_{n=-v_{p}\left(t\right)-1}^{N-1}\beta_{H}\left(p^{n}t\right)\hat{A}_{H,n}\left(t\right)\label{eq:Chi_H,N hat as Beta_H plus A_H,n hat - ready for t,n analysis} \end{equation} Here, we have used the fact that: \begin{equation} \hat{A}_{H,n}\left(t\right)=0,\textrm{ }\forall n\leq-v_{p}\left(t\right)-2 \end{equation} because $\mathbf{1}_{0}\left(p^{n+1}t\right)$ vanishes whenever $n\leq-v_{p}\left(t\right)-2$. This leaves us with two cases. \vphantom{} I. First, suppose $\left|t\right|_{p}=p^{N}$. Then $N=-v_{p}\left(t\right)$, and so, (\ref{eq:Chi_H,N hat as Beta_H plus A_H,n hat - ready for t,n analysis}) becomes: \begin{align*} \hat{\chi}_{H,N}\left(t\right) & =\sum_{n=-v_{p}\left(t\right)-1}^{N-1}\beta_{H}\left(p^{n}t\right)\hat{A}_{H,n}\left(t\right)\\ & =\beta_{H}\left(p^{-v_{p}\left(t\right)-1}t\right)\hat{A}_{H,-v_{p}\left(t\right)-1}\left(t\right)\\ & =\beta_{H}\left(p^{-v_{p}\left(t\right)-1}t\right)\cdot\mathbf{1}_{0}\left(p^{-v_{p}\left(t\right)-1+1}t\right)\prod_{m=0}^{-v_{p}\left(t\right)-1-1}\alpha_{H}\left(p^{m}t\right)\\ & =\beta_{H}\left(p^{-v_{p}\left(t\right)-1}t\right)\cdot\frac{\mathbf{1}_{0}\left(p^{-v_{p}\left(t\right)}t\right)}{\alpha_{H}\left(p^{-v_{p}\left(t\right)-1}t\right)}\prod_{m=0}^{-v_{p}\left(t\right)-1}\alpha_{H}\left(p^{m}t\right) \end{align*} Since $p^{-v_{p}\left(t\right)}t=\left|t\right|_{p}t$ is the integer in the numerator of $t$, $\mathbf{1}_{0}\left(p^{-v_{p}\left(t\right)}t\right)=1$, and we are left with: \begin{align*} \hat{\chi}_{H,N}\left(t\right) & =\frac{\beta_{H}\left(p^{-v_{p}\left(t\right)-1}t\right)}{\alpha_{H}\left(p^{-v_{p}\left(t\right)-1}t\right)}\underbrace{\prod_{m=0}^{-v_{p}\left(t\right)-1}\alpha_{H}\left(p^{m}t\right)}_{\hat{A}_{H}\left(t\right)}\\ & =\frac{\beta_{H}\left(\frac{t\left|t\right|_{p}}{p}\right)}{\alpha_{H}\left(\frac{t\left|t\right|_{p}}{p}\right)}\hat{A}_{H}\left(t\right)\\ & =\gamma_{H}\left(\frac{t\left|t\right|_{p}}{p}\right)\hat{A}_{H}\left(t\right) \end{align*} \vphantom{} II. Second, suppose $0<\left|t\right|_{p}<p^{N}$. Then $-v_{p}\left(t\right)-1$ is strictly less than $N-1$, and so $p^{n}t\overset{1}{\equiv}0$ for all $n\geq-v_{p}\left(t\right)$. With this, (\ref{eq:Chi_H,N hat as Beta_H plus A_H,n hat - ready for t,n analysis}) becomes: \begin{align*} \hat{\chi}_{H,N}\left(t\right) & =\beta_{H}\left(p^{-v_{p}\left(t\right)-1}t\right)\hat{A}_{H,-v_{p}\left(t\right)-1}\left(t\right)+\sum_{n=-v_{p}\left(t\right)}^{N-1}\beta_{H}\left(p^{n}t\right)\hat{A}_{H,n}\left(t\right)\\ \left(p^{n}t\overset{1}{\equiv}0\textrm{ }\forall n\geq-v_{p}\left(t\right)\right); & =\gamma_{H}\left(\frac{t\left|t\right|_{p}}{p}\right)\hat{A}_{H}\left(t\right)+\beta_{H}\left(0\right)\sum_{n=-v_{p}\left(t\right)}^{N-1}\hat{A}_{H,n}\left(t\right) \end{align*} Using \textbf{Proposition \ref{prop:alpha product in terms of A_H hat}}, we get: \begin{equation} \hat{A}_{H,n}\left(t\right)=\left(\alpha_{H}\left(0\right)\right)^{n+v_{p}\left(t\right)}\hat{A}_{H}\left(t\right),\textrm{ }\forall n\geq-v_{p}\left(t\right) \end{equation} and hence: \begin{align*} \hat{\chi}_{H,N}\left(t\right) & =\gamma_{H}\left(\frac{t\left|t\right|_{p}}{p}\right)\hat{A}_{H}\left(t\right)+\beta_{H}\left(0\right)\sum_{n=-v_{p}\left(t\right)}^{N-1}\left(\alpha_{H}\left(0\right)\right)^{n+v_{p}\left(t\right)}\hat{A}_{H}\left(t\right)\\ & =\gamma_{H}\left(\frac{t\left|t\right|_{p}}{p}\right)\hat{A}_{H}\left(t\right)+\beta_{H}\left(0\right)\left(\sum_{n=0}^{N+v_{p}\left(t\right)-1}\left(\alpha_{H}\left(0\right)\right)^{n}\right)\hat{A}_{H}\left(t\right)\\ & =\begin{cases} \gamma_{H}\left(\frac{t\left|t\right|_{p}}{p}\right)\hat{A}_{H}\left(t\right)+\beta_{H}\left(0\right)\left(N+v_{p}\left(t\right)\right)\hat{A}_{H}\left(t\right) & \textrm{if }\alpha_{H}\left(0\right)=1\\ \gamma_{H}\left(\frac{t\left|t\right|_{p}}{p}\right)\hat{A}_{H}\left(t\right)+\beta_{H}\left(0\right)\frac{\left(\alpha_{H}\left(0\right)\right)^{N+v_{p}\left(t\right)}-1}{\alpha_{H}\left(0\right)-1}\hat{A}_{H}\left(t\right) & \textrm{if }\alpha_{H}\left(0\right)\neq1 \end{cases} \end{align*} Finally, when $t=0$: \begin{align*} \hat{\chi}_{H,N}\left(0\right) & =\sum_{n=0}^{N-1}\beta_{H}\left(0\right)\hat{A}_{H,n}\left(0\right)\\ & =\beta_{H}\left(0\right)+\sum_{n=1}^{N-1}\beta_{H}\left(0\right)\mathbf{1}_{0}\left(0\right)\prod_{m=0}^{n-1}\alpha_{H}\left(0\right)\\ & =\beta_{H}\left(0\right)+\sum_{n=1}^{N-1}\beta_{H}\left(0\right)\left(\alpha_{H}\left(0\right)\right)^{n}\\ & =\beta_{H}\left(0\right)\sum_{n=0}^{N-1}\left(\alpha_{H}\left(0\right)\right)^{n}\\ & =\begin{cases} \beta_{H}\left(0\right)N & \textrm{if }\alpha_{H}\left(0\right)=1\\ \beta_{H}\left(0\right)\frac{\left(\alpha_{H}\left(0\right)\right)^{N}-1}{\alpha_{H}\left(0\right)-1} & \textrm{if }\alpha_{H}\left(0\right)\neq1 \end{cases} \end{align*} Since $\hat{A}_{H}\left(0\right)=1$, we have that $\beta_{H}\left(0\right)N=\beta_{H}\left(0\right)N\hat{A}_{H}\left(0\right)$ for the $\alpha_{H}\left(0\right)=1$ case and $\beta_{H}\left(0\right)\frac{\left(\alpha_{H}\left(0\right)\right)^{N}-1}{\alpha_{H}\left(0\right)-1}=\beta_{H}\left(0\right)\frac{\left(\alpha_{H}\left(0\right)\right)^{N}-1}{\alpha_{H}\left(0\right)-1}\hat{A}_{H}\left(0\right)$ for the $\alpha_{H}\left(0\right)\neq1$ case. Q.E.D. \vphantom{} Subtracting $\beta_{H}\left(0\right)N\hat{A}_{H}\left(t\right)\mathbf{1}_{0}\left(p^{N-1}t\right)$ (which is $0$ for all $\left|t\right|_{p}\geq p^{N}$) from the $\alpha_{H}\left(0\right)=1$ case (equation (\ref{eq:Fine Structure Formula for Chi_H,N hat when alpha is 1}))\index{hat{chi}{H,N}@$\hat{\chi}_{H,N}$!fine structure} we obtain: \begin{equation} \hat{\chi}_{H,N}\left(t\right)-\beta_{H}\left(0\right)N\hat{A}_{H}\left(t\right)\mathbf{1}_{0}\left(p^{N-1}t\right)=\begin{cases} 0 & \textrm{if }t=0\\ \left(\gamma_{H}\left(\frac{t\left|t\right|_{p}}{p}\right)+\beta_{H}\left(0\right)v_{p}\left(t\right)\right)\hat{A}_{H}\left(t\right) & \textrm{if }0<\left|t\right|_{p}<p^{N}\\ \gamma_{H}\left(\frac{t\left|t\right|_{p}}{p}\right)\hat{A}_{H}\left(t\right) & \textrm{if }\left|t\right|_{p}=p^{N}\\ 0 & \textrm{if }\left|t\right|_{p}>p^{N} \end{cases}\label{eq:Fine structure of Chi_H,N hat when alpha is 1} \end{equation} Even though the ranges of $t$ assigned to the individual pieces of this formula depends on $N$, note that the actual \emph{values} of the pieces on the right-hand side are independent of $N$ for all $\left|t\right|_{p}<p^{N}$. So, while the Fourier transform $\hat{\chi}_{H}\left(t\right)$ might not exist in a natural way, by having subtracted off the ``divergent'' $\beta_{H}\left(0\right)N\hat{A}_{H}\left(t\right)\mathbf{1}_{0}\left(p^{N-1}t\right)$ term from $\hat{\chi}_{H,N}\left(t\right)$, we have discovered fine-scale behavior of $\hat{\chi}_{H,N}\left(t\right)$ which was hidden beneath the tumult. Indeed, for fixed $t$, the $q$-adic limit of the left-hand side of (\ref{eq:Fine structure of Chi_H,N hat when alpha is 1}) as $N\rightarrow\infty$ is: \begin{equation} \begin{cases} 0 & \textrm{if }t=0\\ \left(\gamma_{H}\left(\frac{t\left|t\right|_{p}}{p}\right)+\beta_{H}\left(0\right)v_{p}\left(t\right)\right)\hat{A}_{H}\left(t\right) & \textrm{if }\left|t\right|_{p}>0 \end{cases}\label{eq:Point-wise limit of the left-hand side of the fine structure formula when alpha is 1} \end{equation} Moreover, for any given $t$, the left-hand side of (\ref{eq:Fine structure of Chi_H,N hat when alpha is 1}) is actually \emph{equal} to the above limit provided that $N>-v_{p}\left(t\right)$. This strongly suggests that (\ref{eq:Point-wise limit of the left-hand side of the fine structure formula when alpha is 1}) is, or is very close to a ``valid'' formula for a Fourier transform of $\chi_{H}$. The next subsection is dedicated to making this intuition rigorous. \subsection{\label{subsec:4.2.2}$\hat{\chi}_{H}$ and $\tilde{\chi}_{H,N}$} We begin by computing the Fourier series generated by $v_{p}\left(t\right)\hat{A}_{H}\left(t\right)$. \begin{lem}[\textbf{$v_{p}\hat{A}_{H}$ summation formulae}] \label{lem:v_p A_H hat summation formulae}\ \vphantom{} I. \begin{equation} \sum_{0<\left|t\right|_{p}\leq p^{N}}v_{p}\left(t\right)\hat{A}_{H}\left(t\right)e^{2\pi i\left\{ t\mathfrak{z}\right\} _{p}}\overset{\overline{\mathbb{Q}}}{=}-N\left(\frac{\mu_{0}}{p}\right)^{N}\kappa_{H}\left(\left[\mathfrak{z}\right]_{p^{N}}\right)+\sum_{n=0}^{N-1}\left(\alpha_{H}\left(0\right)\left(n+1\right)-n\right)\left(\frac{\mu_{0}}{p}\right)^{n}\kappa_{H}\left(\left[\mathfrak{z}\right]_{p^{n}}\right)\label{eq:Fourier sum of A_H hat v_p} \end{equation} \vphantom{} II. If $H$ is semi-basic and contracting: \begin{equation} \sum_{t\in\hat{\mathbb{Z}}_{p}\backslash\left\{ 0\right\} }v_{p}\left(t\right)\hat{A}_{H}\left(t\right)e^{2\pi i\left\{ t\mathfrak{z}\right\} _{p}}\overset{\mathbb{C}_{q}}{=}\sum_{n=0}^{\infty}\left(\alpha_{H}\left(0\right)\left(n+1\right)-n\right)\left(\frac{\mu_{0}}{p}\right)^{n}\kappa_{H}\left(\left[\mathfrak{z}\right]_{p^{n}}\right),\textrm{ }\forall\mathfrak{z}\in\mathbb{Z}_{p}^{\prime}\label{eq:Limit of Fourier sum of v_p A_H hat} \end{equation} where the convergence is point-wise. \end{lem} Proof: For (I), use (\ref{eq:Convolution of dA_H and D_N}) from \textbf{Theorem \ref{thm:Properties of dA_H}}, along with \textbf{Proposition \ref{prop:v_p of t times mu hat sum}}. If $H$ is semi-basic and contracting, the decay estimates on $\kappa_{H}$ and $\left(\frac{\mu_{0}}{p}\right)^{N}\kappa_{H}\left(\left[\mathfrak{z}\right]_{p^{N}}\right)$ given by \textbf{Proposition \ref{prop:Properties of Kappa_H}} then guarantee the convergence of (I) to (II) in $\mathbb{C}_{q}$ for $\mathfrak{z}\in\mathbb{Z}_{p}^{\prime}$ as $N\rightarrow\infty$. Q.E.D. \vphantom{} Next, we need to deal with how $p$ affects the situation. The primary complication in our computations comes from $\gamma_{H}\left(t\left|t\right|_{p}/p\right)$, whose behavior bifurcates at $p=2$. The function: \begin{equation} t\in\hat{\mathbb{Z}}_{p}\mapsto\frac{t\left|t\right|_{p}}{p}\in\hat{\mathbb{Z}}_{p}\label{eq:Projection onto 1 over rho} \end{equation} is a projection which sends every non-zero element $t=k/p^{n}$ in $\hat{\mathbb{Z}}_{p}$ and outputs the fraction $\left[k\right]_{p}/p$ obtained by reducing $k$ mod $p$ and then sticking it over $p$. When $p=2$, $\left[k\right]_{p}=1$ for all $t\in\hat{\mathbb{Z}}_{2}$, making (\ref{eq:Projection onto 1 over rho}) constant on $\hat{\mathbb{Z}}_{2}\backslash\left\{ 0\right\} $, where it takes the value $1/2$. When $p\geq3$, however, (\ref{eq:Projection onto 1 over rho}) is no longer constant on $\hat{\mathbb{Z}}_{p}\backslash\left\{ 0\right\} $, which is going to significantly complicate our computations. This can be made much more manageable, provided the reader will put up with another bit of notation. \begin{defn}[$\varepsilon_{n}\left(\mathfrak{z}\right)$] \nomenclature{$\varepsilon_{n}\left(\mathfrak{z}\right)$}{ }For each $n\in\mathbb{N}_{0}$, we define $\varepsilon_{n}:\mathbb{Z}_{p}\rightarrow\overline{\mathbb{Q}}$ by: \begin{equation} \varepsilon_{n}\left(\mathfrak{z}\right)\overset{\textrm{def}}{=}e^{\frac{2\pi i}{p^{n+1}}\left(\left[\mathfrak{z}\right]_{p^{n+1}}-\left[\mathfrak{z}\right]_{p^{n}}\right)}=e^{2\pi i\left\{ \frac{\mathfrak{z}}{p^{n+1}}\right\} _{p}}e^{-\frac{2\pi i}{p}\left\{ \frac{\mathfrak{z}}{p^{n}}\right\} _{p}}\label{eq:Definition of epsilon_n} \end{equation} \end{defn} \vphantom{} As with nearly every other noteworthy function in this dissertation, the $\varepsilon_{n}$s satisfy functional equations, which we record below. \begin{prop}[\textbf{Properties of $\varepsilon_{n}$}] \ \vphantom{} I. \begin{equation} \varepsilon_{0}\left(\mathfrak{z}\right)=e^{2\pi i\left\{ \frac{\mathfrak{z}}{p}\right\} _{p}},\textrm{ }\forall\mathfrak{z}\in\mathbb{Z}_{p}\label{eq:Epsilon 0 of z} \end{equation} \begin{equation} \varepsilon_{n}\left(j\right)=1,\textrm{ }\forall j\in\mathbb{Z}/p\mathbb{Z},\textrm{ }\forall n\geq1\label{eq:Epsilon_n of j} \end{equation} \vphantom{} II. \begin{equation} \varepsilon_{n}\left(pm+j\right)=\begin{cases} \varepsilon_{0}\left(j\right) & \textrm{if }n=0\\ \varepsilon_{n-1}\left(m\right) & \textrm{if }n\geq1 \end{cases},\textrm{ }\forall m\in\mathbb{N}_{0},\textrm{ }\forall j\in\mathbb{Z}/p\mathbb{Z},\textrm{ }\forall n\geq1\label{eq:epsilon_n functional equations} \end{equation} \vphantom{} III. Let $\mathfrak{z}\neq0$. Then $\varepsilon_{n}\left(\mathfrak{z}\right)=1\textrm{ }$ for all $n<v_{p}\left(\mathfrak{z}\right)$. \end{prop} Proof: I. The identity (\ref{eq:Epsilon 0 of z}) is immediate from the definition of $\varepsilon_{n}$. As for the other identity, note that for $j\in\mathbb{Z}/p\mathbb{Z}$, $\left[j\right]_{p^{n}}=j$ for all $n\geq1$. Hence, for $n\geq1$: \begin{equation} \varepsilon_{n}\left(j\right)=e^{\frac{2\pi i}{p^{n+1}}\left(\left[j\right]_{p^{n+1}}-\left[j\right]_{p^{n}}\right)}=e^{\frac{2\pi i}{p^{n+1}}\cdot0}=1 \end{equation} \vphantom{} II. \begin{align*} \varepsilon_{n}\left(pm+j\right) & =e^{2\pi i\left\{ \frac{pm+j}{p^{n+1}}\right\} _{p}}e^{-\frac{2\pi i}{p}\left\{ \frac{pm+j}{p^{n}}\right\} _{p}}\\ & =e^{2\pi i\left\{ \frac{j}{p^{n+1}}\right\} _{p}}e^{-\frac{2\pi i}{p}\left\{ \frac{j}{p^{n}}\right\} _{p}}\cdot e^{2\pi i\left\{ \frac{m}{p^{n}}\right\} _{p}}e^{-\frac{2\pi i}{p}\left\{ \frac{m}{p^{n-1}}\right\} _{p}}\\ & =\varepsilon_{n}\left(j\right)\varepsilon_{n-1}\left(m\right)\\ \left(\textrm{by (I)}\right); & =\begin{cases} \varepsilon_{0}\left(j\right) & \textrm{if }n=0\\ \varepsilon_{n-1}\left(m\right) & \textrm{if }n\geq1 \end{cases} \end{align*} \vphantom{} III. Let $\mathfrak{z}$ be non-zero. When $n<v_{p}\left(\mathfrak{z}\right)$, we have that $p^{-n}\mathfrak{z}$ and $p^{-\left(n+1\right)}\mathfrak{z}$ are then $p$-adic integers, and hence: \begin{equation} \varepsilon_{n}\left(\mathfrak{z}\right)=e^{2\pi i\left\{ \frac{\mathfrak{z}}{p^{n+1}}\right\} _{p}}e^{-\frac{2\pi i}{p}\left\{ \frac{\mathfrak{z}}{p^{n}}\right\} _{p}}=e^{0}\cdot e^{-0}=1 \end{equation} Q.E.D. \vphantom{} Now we compute the sum of the Fourier series generated by $\gamma_{H}\left(t\left|t\right|_{p}/p\right)\hat{A}_{H}\left(t\right)$. \begin{lem}[\textbf{$\gamma_{H}\hat{A}_{H}$ summation formulae}] \label{lem:1D gamma formula}Let $p\geq2$. Then: \vphantom{} I. \begin{align} \sum_{0<\left|t\right|_{p}\leq p^{N}}\gamma_{H}\left(\frac{t\left|t\right|_{p}}{p}\right)\hat{A}_{H}\left(t\right)e^{2\pi i\left\{ t\mathfrak{z}\right\} _{p}} & \overset{\overline{\mathbb{Q}}}{=}\sum_{n=0}^{N-1}\left(\sum_{j=1}^{p-1}\beta_{H}\left(\frac{j}{p}\right)\varepsilon_{n}^{j}\left(\mathfrak{z}\right)\right)\left(\frac{\mu_{0}}{p}\right)^{n}\kappa_{H}\left(\left[\mathfrak{z}\right]_{p^{n}}\right)\label{eq:Gamma formula} \end{align} \vphantom{} II. If $H$ is a contracting semi-basic $p$-Hydra map, then, as $N\rightarrow\infty$, \emph{(\ref{eq:Gamma formula})} is $\mathcal{F}_{p,q_{H}}$ convergent to: \begin{equation} \sum_{t\in\hat{\mathbb{Z}}_{p}\backslash\left\{ 0\right\} }\gamma_{H}\left(\frac{t\left|t\right|_{p}}{p}\right)\hat{A}_{H}\left(t\right)e^{2\pi i\left\{ t\mathfrak{z}\right\} _{p}}\overset{\mathcal{F}_{p,q_{H}}}{=}\sum_{n=0}^{\infty}\left(\sum_{j=1}^{p-1}\beta_{H}\left(\frac{j}{p}\right)\varepsilon_{n}^{j}\left(\mathfrak{z}\right)\right)\left(\frac{\mu_{0}}{p}\right)^{n}\kappa_{H}\left(\left[\mathfrak{z}\right]_{p^{n}}\right)\label{eq:F limit of Gamma_H A_H hat Fourier series when alpha is 1} \end{equation} whenever $H$ is semi-basic and contracting. \end{lem} \begin{rem} Note that the non-singularity of $H$ ($\alpha_{H}\left(j/p\right)\neq0$ for all $j\in\mathbb{Z}/p\mathbb{Z}$) is \emph{essential} for this result. \end{rem} Proof: I. Note that the map $t\in\hat{\mathbb{Z}}_{p}\mapsto\frac{t\left|t\right|_{p}}{p}\in\hat{\mathbb{Z}}_{p}$ takes fractions $k/p^{n}$ and sends them to $\left[k\right]_{p}/p$. Now, for brevity, let: \begin{align} \gamma_{j} & \overset{\textrm{def}}{=}\gamma_{H}\left(\frac{j}{p}\right)\\ F_{N}\left(\mathfrak{z}\right) & \overset{\textrm{def}}{=}\sum_{0<\left|t\right|_{p}\leq p^{N}}\gamma_{H}\left(\frac{t\left|t\right|_{p}}{p}\right)\hat{A}_{H}\left(t\right)e^{2\pi i\left\{ t\mathfrak{z}\right\} _{p}} \end{align} Observing that: \begin{equation} \left\{ t\in\hat{\mathbb{Z}}_{p}:\left|t\right|_{p}=p^{n}\right\} =\left\{ \frac{pk+j}{p^{n}}:k\in\left\{ 0,\ldots,p^{n-1}-1\right\} ,\textrm{ }j\in\left\{ 1,\ldots,p-1\right\} \right\} \label{eq:Decomposition of level sets} \end{equation} we can then express $F_{N}$ as a sum involving the $\gamma_{j}$s, like so: \begin{align*} F_{N}\left(\mathfrak{z}\right) & =\sum_{n=1}^{N}\sum_{\left|t\right|_{p}=p^{n}}\gamma_{H}\left(\frac{t\left|t\right|_{p}}{p}\right)\hat{A}_{H}\left(t\right)e^{2\pi i\left\{ t\mathfrak{z}\right\} _{p}}\\ \left(\textrm{use }(\ref{eq:Decomposition of level sets})\right); & =\sum_{n=1}^{N}\sum_{j=1}^{p-1}\sum_{k=0}^{p^{n-1}-1}\gamma_{H}\left(\frac{j}{p}\right)\hat{A}_{H}\left(\frac{pk+j}{p^{n}}\right)e^{2\pi i\left\{ \frac{pk+j}{p^{n}}\mathfrak{z}\right\} _{p}} \end{align*} Using the formal identity: \begin{equation} \sum_{k=0}^{p^{n-1}-1}f\left(\frac{pk+j}{p^{n}}\right)=\sum_{\left|t\right|_{p}\leq p^{n-1}}f\left(t+\frac{j}{p^{n}}\right) \end{equation} we can then write: \begin{align} F_{N}\left(\mathfrak{z}\right) & =\sum_{n=1}^{N}\sum_{\left|t\right|_{p}\leq p^{n-1}}\sum_{j=1}^{p-1}\gamma_{j}e^{2\pi i\left\{ \frac{j\mathfrak{z}}{p^{n}}\right\} _{p}}\hat{A}_{H}\left(t+\frac{j}{p^{n}}\right)e^{2\pi i\left\{ t\mathfrak{z}\right\} _{p}}\label{eq:Halfway through the gamma computation} \end{align} To deal with the $j$-sum, we express $\hat{A}_{H}$ in product form, changing: \begin{equation} \sum_{j=1}^{p-1}\gamma_{j}e^{2\pi i\left\{ \frac{j\mathfrak{z}}{p^{n}}\right\} _{p}}\hat{A}_{H}\left(t+\frac{j}{p^{n}}\right)e^{2\pi i\left\{ t\mathfrak{z}\right\} _{p}} \end{equation} into: \begin{equation} \sum_{j=1}^{p-1}\gamma_{j}e^{2\pi i\left\{ \frac{j\mathfrak{z}}{p^{n}}\right\} _{p}}\left(\prod_{m=0}^{n-1}\alpha_{H}\left(p^{m}\left(t+\frac{j}{p^{n}}\right)\right)\right)e^{2\pi i\left\{ t\mathfrak{z}\right\} _{p}} \end{equation} Using\textbf{ Proposition \ref{prop:alpha product series expansion}} to write the $\alpha_{H}$-product out as a series, the above becomes: \begin{equation} \sum_{j=1}^{p-1}\gamma_{j}e^{2\pi i\left\{ \frac{j\mathfrak{z}}{p^{n}}\right\} _{p}}\left(\left(\frac{\mu_{0}}{p^{2}}\right)^{n}\sum_{m=0}^{p^{n}-1}\kappa_{H}\left(m\right)e^{-2\pi im\left(t+\frac{j}{p^{n}}\right)}\right)e^{2\pi i\left\{ t\mathfrak{z}\right\} _{p}} \end{equation} Hence: \begin{equation} \sum_{j=1}^{p-1}\gamma_{j}\left(\frac{\mu_{0}}{p^{2}}\right)^{n}\sum_{m=0}^{p^{n}-1}\kappa_{H}\left(m\right)e^{2\pi i\left\{ \frac{j\left(\mathfrak{z}-m\right)}{p^{n}}\right\} _{p}}e^{2\pi i\left\{ t\left(\mathfrak{z}-m\right)\right\} _{p}} \end{equation} Summing over $\left|t\right|_{p}\leq p^{n-1}$, and using: \begin{equation} \sum_{\left|t\right|_{p}\leq p^{n-1}}e^{2\pi i\left\{ t\left(\mathfrak{z}-m\right)\right\} _{p}}=p^{n-1}\left[\mathfrak{z}\overset{p^{n-1}}{\equiv}m\right] \end{equation} we obtain: \begin{equation} \sum_{j=1}^{p-1}\gamma_{j}\left(\frac{\mu_{0}}{p^{2}}\right)^{n}\sum_{m=0}^{p^{n}-1}\kappa_{H}\left(m\right)e^{2\pi i\left\{ \frac{j\left(\mathfrak{z}-m\right)}{p^{n}}\right\} _{p}}p^{n-1}\left[\mathfrak{z}\overset{p^{n-1}}{\equiv}m\right] \end{equation} In summary, so far, we have; \begin{eqnarray} & \sum_{\left|t\right|_{p}\leq p^{n-1}}\sum_{j=1}^{p-1}\gamma_{j}e^{2\pi i\left\{ \frac{j\mathfrak{z}}{p^{n}}\right\} _{p}}\hat{A}_{H}\left(t+\frac{j}{p^{n}}\right)e^{2\pi i\left\{ t\mathfrak{z}\right\} _{p}}\nonumber \\ & =\label{eq:2/3rds of the way through the gamma computation}\\ & \sum_{j=1}^{p-1}\frac{\gamma_{j}}{p}\left(\frac{\mu_{0}}{p}\right)^{n}\sum_{m=0}^{p^{n}-1}\kappa_{H}\left(m\right)e^{2\pi i\left\{ \frac{j\left(\mathfrak{z}-m\right)}{p^{n}}\right\} _{p}}\left[\mathfrak{z}\overset{p^{n-1}}{\equiv}m\right]\nonumber \end{eqnarray} Next, using the formal identity: \begin{equation} \sum_{m=0}^{p^{n}-1}f\left(m\right)=\sum_{k=0}^{p-1}\sum_{m=0}^{p^{n-1}-1}f\left(m+kp^{n-1}\right)\label{eq:rho to the n formal identity} \end{equation} and the functional equation identity for $\kappa_{H}$ (equation (\ref{eq:Kappa_H functional equations}) from \textbf{Proposition \ref{prop:Properties of Kappa_H}}), we have: \begin{eqnarray*} & \sum_{m=0}^{p^{n}-1}\kappa_{H}\left(m\right)e^{2\pi i\left\{ \frac{j\left(\mathfrak{z}-m\right)}{p^{n}}\right\} _{p}}\left[\mathfrak{z}\overset{p^{n-1}}{\equiv}m\right]\\ & =\\ & \sum_{k=0}^{p-1}\sum_{m=0}^{p^{n-1}-1}\frac{\mu_{k}}{\mu_{0}}\kappa_{H}\left(m\right)e^{2\pi i\left\{ \frac{j\left(\mathfrak{z}-m-kp^{n-1}\right)}{p^{n}}\right\} _{p}}\left[\mathfrak{z}\overset{p^{n-1}}{\equiv}m\right] \end{eqnarray*} Here, note that $\left[\mathfrak{z}\right]_{p^{n-1}}$ is the unique integer $m\in\left\{ 0,\ldots,p^{n-1}-1\right\} $ satisfying $\mathfrak{z}\overset{p^{n-1}}{\equiv}m$. This leaves us with: \begin{align*} \sum_{m=0}^{p^{n}-1}\kappa_{H}\left(m\right)e^{2\pi i\left\{ \frac{j\left(\mathfrak{z}-m\right)}{p^{n}}\right\} _{p}}\left[\mathfrak{z}\overset{p^{n-1}}{\equiv}m\right] & =\kappa_{H}\left(\left[\mathfrak{z}\right]_{p^{n-1}}\right)\left(\varepsilon_{n-1}\left(\mathfrak{z}\right)\right)^{j}\overbrace{\sum_{k=0}^{p-1}\frac{\mu_{k}}{\mu_{0}}e^{-2\pi i\frac{jk}{p}}}^{\frac{p^{2}}{\mu_{0}}\alpha_{H}\left(\frac{j}{p}\right)}\\ & =\frac{p^{2}}{\mu_{0}}\alpha_{H}\left(\frac{j}{p}\right)\kappa_{H}\left(\left[\mathfrak{z}\right]_{p^{n-1}}\right)\left(\varepsilon_{n-1}\left(\mathfrak{z}\right)\right)^{j} \end{align*} Returning with this to (\ref{eq:2/3rds of the way through the gamma computation}), the equation \begin{equation} \sum_{\left|t\right|_{p}\leq p^{n-1}}\sum_{j=1}^{p-1}\gamma_{j}e^{2\pi i\left\{ \frac{j\mathfrak{z}}{p^{n}}\right\} _{p}}\hat{A}_{H}\left(t+\frac{j}{p^{n}}\right)e^{2\pi i\left\{ t\mathfrak{z}\right\} _{p}} \end{equation} transforms into: \begin{equation} \sum_{j=1}^{p-1}\frac{\gamma_{j}}{p}\left(\frac{\mu_{0}}{p}\right)^{n}\frac{p^{2}}{\mu_{0}}\alpha_{H}\left(\frac{j}{p}\right)\left(\varepsilon_{n-1}\left(\mathfrak{z}\right)\right)^{j}\kappa_{H}\left(\left[\mathfrak{z}\right]_{p^{n-1}}\right) \end{equation} Almost finished, recall that: \begin{equation} \gamma_{j}=\gamma_{H}\left(\frac{j}{p}\right)=\frac{\beta_{H}\left(\frac{j}{p}\right)}{\alpha_{H}\left(\frac{j}{p}\right)} \end{equation} With this, we can write the previous line as: \begin{equation} \left(\sum_{j=1}^{p-1}\beta_{H}\left(\frac{j}{p}\right)\left(\varepsilon_{n-1}\left(\mathfrak{z}\right)\right)^{j}\right)\left(\frac{\mu_{0}}{p}\right)^{n-1}\kappa_{H}\left(\left[\mathfrak{z}\right]_{p^{n-1}}\right) \end{equation} Applying this to the right-hand side of (\ref{eq:Halfway through the gamma computation}) gives us: \begin{align} F_{N}\left(\mathfrak{z}\right) & =\sum_{n=1}^{N}\left(\sum_{j=1}^{p-1}\beta_{H}\left(\frac{j}{p}\right)\left(\varepsilon_{n-1}\left(\mathfrak{z}\right)\right)^{j}\right)\left(\frac{\mu_{0}}{p}\right)^{n-1}\kappa_{H}\left(\left[\mathfrak{z}\right]_{p^{n-1}}\right) \end{align} Re-indexing $n$ by a shift of $1$ produces equation (\ref{eq:Gamma formula}). \vphantom{} II. For each $\mathfrak{z}\in\mathbb{Z}_{p}$, the algebraic number $\sum_{j=1}^{p-1}\beta_{H}\left(\frac{j}{p}\right)\left(\varepsilon_{n}\left(\mathfrak{z}\right)\right)^{j}$ is uniformly bounded with respect to $n\in\mathbb{N}_{0}$ in both $\mathbb{C}$ and $\mathbb{C}_{q}$. Like in the $p=2$ case, since $H$ is semi-basic, the sequence $\left(\frac{\mu_{0}}{p}\right)^{n}\kappa_{H}\left(\left[\mathfrak{z}\right]_{p^{n}}\right)$ is then $\mathcal{F}_{p,q_{H}}$-convergent to $0$. The $q$-adic convergence to $0$ over $\mathbb{Z}_{p}^{\prime}$ guarantees the $q$-adic convergence of (\ref{eq:Gamma formula}) as $N\rightarrow\infty$ for $\mathfrak{z}\in\mathbb{Z}_{p}^{\prime}$. On the other hand, for $\mathfrak{z}\in\mathbb{N}_{0}$, as we have seen: \begin{equation} \left(\frac{\mu_{0}}{p}\right)^{n}\kappa_{H}\left(\left[\mathfrak{z}\right]_{p^{n}}\right)=\left(\frac{\mu_{0}}{p}\right)^{n}\kappa_{H}\left(\mathfrak{z}\right),\textrm{ }\forall n\geq\lambda_{p}\left(\mathfrak{z}\right) \end{equation} and the fact that $H$ is contracting then guarantees (by the ratio test, no less) that (\ref{eq:Gamma formula}) is convergent in $\mathbb{C}$ for $\mathfrak{z}\in\mathbb{N}_{0}$. This proves (II). Q.E.D. \vphantom{} With these formulae, we can sum the Fourier series generated by (\ref{eq:Fine structure of Chi_H,N hat when alpha is 1}) to obtain a non-trivial formula for $\chi_{H,N}$. \begin{thm} \label{thm:F-series for Nth truncation of Chi_H, alpha is 1}Suppose\index{chi{H}@$\chi_{H}$!$N$th truncation} $\alpha_{H}\left(0\right)=1$. Then, for all $N\geq1$ and all $\mathfrak{z}\in\mathbb{Z}_{p}$: \begin{align} \chi_{H,N}\left(\mathfrak{z}\right) & \overset{\overline{\mathbb{Q}}}{=}\sum_{n=0}^{N-1}\left(\sum_{j=0}^{p-1}\beta_{H}\left(\frac{j}{p}\right)\left(\varepsilon_{n}\left(\mathfrak{z}\right)\right)^{j}\right)\left(\frac{\mu_{0}}{p}\right)^{n}\kappa_{H}\left(\left[\mathfrak{z}\right]_{p^{n}}\right)\label{eq:Chi_H,N when alpha is 1 and rho is arbitrary} \end{align} In particular, when $p=2$: \begin{equation} \chi_{H,N}\left(\mathfrak{z}\right)\overset{\overline{\mathbb{Q}}}{=}-\gamma_{H}\left(\frac{1}{2}\right)+\gamma_{H}\left(\frac{1}{2}\right)\left(\frac{\mu_{0}}{2}\right)^{N}\kappa_{H}\left(\left[\mathfrak{z}\right]_{2^{N}}\right)+\beta_{H}\left(0\right)\sum_{n=0}^{N-1}\left(\frac{\mu_{0}}{2}\right)^{n}\kappa_{H}\left(\left[\mathfrak{z}\right]_{2^{n}}\right)\label{eq:Chi_H,N when alpha is 1 and rho is 2} \end{equation} \end{thm} Proof: We start by multiplying (\ref{eq:Fine structure of Chi_H,N hat when alpha is 1}) by $e^{2\pi i\left\{ t\mathfrak{z}\right\} _{p}}$ and then summing over all $\left|t\right|_{p}\leq p^{N}$. The left-hand side of (\ref{eq:Fine structure of Chi_H,N hat when alpha is 1}) becomes: \begin{equation} \chi_{H,N}\left(\mathfrak{z}\right)-\beta_{H}\left(0\right)N\left(D_{p:N-1}*dA_{H}\right)\left(\mathfrak{z}\right) \end{equation} whereas the right-hand side is: \begin{align*} \sum_{0<\left|t\right|_{p}\leq p^{N-1}}\left(\gamma_{H}\left(\frac{t\left|t\right|_{p}}{p}\right)+\beta_{H}\left(0\right)v_{p}\left(t\right)\right)\hat{A}_{H}\left(t\right)e^{2\pi i\left\{ t\mathfrak{z}\right\} _{p}}\\ +\sum_{\left|t\right|_{p}=p^{N}}\gamma_{H}\left(\frac{t\left|t\right|_{p}}{p}\right)\hat{A}_{H}\left(t\right)e^{2\pi i\left\{ t\mathfrak{z}\right\} _{p}} \end{align*} Simplifying produces: \begin{align} \chi_{H,N}\left(\mathfrak{z}\right) & \overset{\overline{\mathbb{Q}}}{=}\beta_{H}\left(0\right)N\left(D_{p:N-1}*dA_{H}\right)\left(\mathfrak{z}\right)\label{eq:Chi_H,N rho not equal to 2, ready to simplify}\\ & +\beta_{H}\left(0\right)\sum_{0<\left|t\right|_{p}\leq p^{N-1}}v_{p}\left(t\right)\hat{A}_{H}\left(t\right)e^{2\pi i\left\{ t\mathfrak{z}\right\} _{p}}\nonumber \\ & +\sum_{0<\left|t\right|_{p}\leq p^{N}}\gamma_{H}\left(\frac{t\left|t\right|_{p}}{p}\right)\hat{A}_{H}\left(t\right)e^{2\pi i\left\{ t\mathfrak{z}\right\} _{p}}\nonumber \end{align} Now we call upon our legion of formulae: (\ref{eq:Convolution of dA_H and D_N}), \textbf{Lemma \ref{lem:v_p A_H hat summation formulae}}, and \textbf{Lemma \ref{lem:1D gamma formula}}. Using them (whilst remembering that $\alpha_{H}\left(0\right)=1$) makes (\ref{eq:Chi_H,N rho not equal to 2, ready to simplify}) into: \begin{align*} \chi_{H,N}\left(\mathfrak{z}\right) & \overset{\overline{\mathbb{Q}}}{=}\beta_{H}\left(0\right)N\left(\frac{\mu_{0}}{p}\right)^{N-1}\kappa_{H}\left(\left[\mathfrak{z}\right]_{p^{N-1}}\right)\\ & -\beta_{H}\left(0\right)\left(N-1\right)\left(\frac{\mu_{0}}{p}\right)^{N-1}\kappa_{H}\left(\left[\mathfrak{z}\right]_{p^{N-1}}\right)\\ & +\beta_{H}\left(0\right)\sum_{n=0}^{N-2}\left(\frac{\mu_{0}}{p}\right)^{n}\kappa_{H}\left(\left[\mathfrak{z}\right]_{p^{n}}\right)\\ & \sum_{n=0}^{N-1}\left(\sum_{j=1}^{p-1}\beta_{H}\left(\frac{j}{p}\right)\left(\varepsilon_{n}\left(\mathfrak{z}\right)\right)^{j}\right)\left(\frac{\mu_{0}}{p}\right)^{n}\kappa_{H}\left(\left[\mathfrak{z}\right]_{p^{n}}\right)\\ & \overset{\overline{\mathbb{Q}}}{=}\beta_{H}\left(0\right)\sum_{n=0}^{N-1}\left(\frac{\mu_{0}}{p}\right)^{n}\kappa_{H}\left(\left[\mathfrak{z}\right]_{p^{n}}\right)\\ & +\sum_{n=0}^{N-1}\left(\sum_{j=1}^{p-1}\beta_{H}\left(\frac{j}{p}\right)\left(\varepsilon_{n}\left(\mathfrak{z}\right)\right)^{j}\right)\left(\frac{\mu_{0}}{p}\right)^{n}\kappa_{H}\left(\left[\mathfrak{z}\right]_{p^{n}}\right) \end{align*} The bottom two lines combine to form a single sum by noting that the upper most of the two is the $j=0$ case of the bottom-most of the two. Finally, when $p=2$, rather than simplify (\ref{eq:Chi_H,N when alpha is 1 and rho is arbitrary}), it will actually be easier to compute it from scratch all over again. Multiplying (\ref{eq:Fine structure of Chi_H,N hat when alpha is 1}) by $e^{2\pi i\left\{ t\mathfrak{z}\right\} _{2}}$ and summing over all $\left|t\right|_{2}\leq2^{N}$ gives: \begin{align*} \chi_{H,N}\left(\mathfrak{z}\right)-\beta_{H}\left(0\right)N\left(D_{2:N-1}*dA_{H}\right)\left(\mathfrak{z}\right) & \overset{\overline{\mathbb{Q}}}{=}\gamma_{H}\left(\frac{1}{2}\right)\sum_{0<\left|t\right|_{2}\leq2^{N-1}}\hat{A}_{H}\left(t\right)e^{2\pi i\left\{ t\mathfrak{z}\right\} _{2}}\\ & +\beta_{H}\left(0\right)\sum_{0<\left|t\right|_{2}\leq2^{N-1}}v_{2}\left(t\right)\hat{A}_{H}\left(t\right)e^{2\pi i\left\{ t\mathfrak{z}\right\} _{2}}\\ & +\gamma_{H}\left(\frac{1}{2}\right)\sum_{\left|t\right|_{2}=2^{N}}\hat{A}_{H}\left(t\right)e^{2\pi i\left\{ t\mathfrak{z}\right\} _{2}} \end{align*} This simplifies to: \begin{align*} \chi_{H,N}\left(\mathfrak{z}\right)-\beta_{H}\left(0\right)N\left(D_{2:N-1}*dA_{H}\right)\left(\mathfrak{z}\right) & \overset{\overline{\mathbb{Q}}}{=}\gamma_{H}\left(\frac{1}{2}\right)\sum_{0<\left|t\right|_{2}\leq2^{N}}\hat{A}_{H}\left(t\right)e^{2\pi i\left\{ t\mathfrak{z}\right\} _{2}}\\ & +\beta_{H}\left(0\right)\sum_{0<\left|t\right|_{2}\leq2^{N-1}}v_{2}\left(t\right)\hat{A}_{H}\left(t\right)e^{2\pi i\left\{ t\mathfrak{z}\right\} _{2}}\\ & \overset{\overline{\mathbb{Q}}}{=}-\gamma_{H}\left(\frac{1}{2}\right)\hat{A}_{H}\left(0\right)\\ & +\gamma_{H}\left(\frac{1}{2}\right)\underbrace{\sum_{\left|t\right|_{2}\leq2^{N}}\hat{A}_{H}\left(t\right)e^{2\pi i\left\{ t\mathfrak{z}\right\} _{2}}}_{\left(D_{2:N}*dA_{H}\right)\left(\mathfrak{z}\right)}\\ & +\beta_{H}\left(0\right)\sum_{0<\left|t\right|_{2}\leq2^{N-1}}v_{2}\left(t\right)\hat{A}_{H}\left(t\right)e^{2\pi i\left\{ t\mathfrak{z}\right\} _{2}} \end{align*} Applying (\ref{eq:Convolution of dA_H and D_N}) and \textbf{Lemma \ref{lem:v_p A_H hat summation formulae}}, the above becomes: \begin{align*} \chi_{H,N}\left(\mathfrak{z}\right)-\beta_{H}\left(0\right)N\left(\frac{\mu_{0}}{2}\right)^{N-1}\kappa_{H}\left(\left[\mathfrak{z}\right]_{2^{N-1}}\right) & \overset{\overline{\mathbb{Q}}}{=}-\gamma_{H}\left(\frac{1}{2}\right)\\ & +\gamma_{H}\left(\frac{1}{2}\right)\left(\frac{\mu_{0}}{2}\right)^{N}\kappa_{H}\left(\left[\mathfrak{z}\right]_{2^{N}}\right)\\ & -\beta_{H}\left(0\right)\left(N-1\right)\left(\frac{\mu_{0}}{2}\right)^{N-1}\kappa_{H}\left(\left[\mathfrak{z}\right]_{2^{N-1}}\right)\\ & +\beta_{H}\left(0\right)\sum_{n=0}^{N-2}\left(\frac{\mu_{0}}{2}\right)^{n}\kappa_{H}\left(\left[\mathfrak{z}\right]_{2^{n}}\right) \end{align*} Simplifying yields: \begin{equation} \chi_{H,N}\left(\mathfrak{z}\right)\overset{\overline{\mathbb{Q}}}{=}-\gamma_{H}\left(\frac{1}{2}\right)+\gamma_{H}\left(\frac{1}{2}\right)\left(\frac{\mu_{0}}{2}\right)^{N}\kappa_{H}\left(\left[\mathfrak{z}\right]_{2^{N}}\right)+\beta_{H}\left(0\right)\sum_{n=0}^{N-1}\left(\frac{\mu_{0}}{2}\right)^{n}\kappa_{H}\left(\left[\mathfrak{z}\right]_{2^{n}}\right) \end{equation} Q.E.D. \begin{cor}[\textbf{$\mathcal{F}$-series for $\chi_{H}$ when $\alpha_{H}\left(0\right)=1$}] \label{cor:Chi_H, F-convergent formula}Suppose\index{chi{H}@$\chi_{H}$!mathcal{F}-series@$\mathcal{F}$-series} For the given $p$-Hydra map $H$ (semi-basic, contracting, non-singular, fixes $0$), suppose $\alpha_{H}\left(0\right)=1$. Then: \begin{equation} \chi_{H}\left(\mathfrak{z}\right)\overset{\mathcal{F}_{p,q_{H}}}{=}\sum_{n=0}^{\infty}\left(\sum_{j=0}^{p-1}\beta_{H}\left(\frac{j}{p}\right)\left(\varepsilon_{n}\left(\mathfrak{z}\right)\right)^{j}\right)\left(\frac{\mu_{0}}{p}\right)^{n}\kappa_{H}\left(\left[\mathfrak{z}\right]_{p^{n}}\right),\textrm{ }\forall\mathfrak{z}\in\mathbb{Z}_{p}\label{eq:Explicit Formula for Chi_H when alpha is 1 and rho is arbitrary} \end{equation} with the special case: \begin{equation} \chi_{H}\left(\mathfrak{z}\right)\overset{\mathcal{F}_{2,q_{H}}}{=}-\gamma_{H}\left(\frac{1}{2}\right)+\beta_{H}\left(0\right)\sum_{n=0}^{\infty}\left(\frac{\mu_{0}}{2}\right)^{n}\kappa_{H}\left(\left[\mathfrak{z}\right]_{2^{n}}\right),\textrm{ }\forall\mathfrak{z}\in\mathbb{Z}_{2}\label{eq:Explicit Formula for Chi_H when alpha is 1 and rho is 2} \end{equation} for when $p=2$. In both cases, as indicated, the topology of convergence of the series on the right-hand side is that of the standard $\left(p,q_{H}\right)$-adic frame: the topology of $\mathbb{C}$ when $\mathfrak{z}\in\mathbb{N}_{0}$, and the topology of $\mathbb{C}_{q_{H}}$ when $\mathfrak{z}\in\mathbb{Z}_{p}^{\prime}$. \end{cor} Proof: The given properties of $H$ tell us that $\left(\frac{\mu_{0}}{p}\right)^{N}\kappa_{H}\left(\left[\mathfrak{z}\right]_{p^{N}}\right)$ has $0$ as a $\mathcal{F}_{p,q_{H}}$-limit. Appealing to the limits formulae in \textbf{Lemmata \ref{lem:1D gamma formula}} and \textbf{\ref{lem:v_p A_H hat summation formulae}} then allows us to take the $\mathcal{F}_{p,q_{H}}$-limits of (\ref{eq:Chi_H,N when alpha is 1 and rho is arbitrary}) and (\ref{eq:Chi_H,N when alpha is 1 and rho is 2}) as $N\rightarrow\infty$, which produces (\ref{eq:Explicit Formula for Chi_H when alpha is 1 and rho is arbitrary}) and (\ref{eq:Explicit Formula for Chi_H when alpha is 1 and rho is 2}), respectively. Q.E.D. \vphantom{} Next, we sum in $\mathbb{C}$ for $\mathfrak{z}\in\mathbb{N}_{0}$. \begin{cor} \label{cor:Chi_H explicit formula on N_0}If $\alpha_{H}\left(0\right)=1$, then: \begin{equation} \chi_{H}\left(n\right)\overset{\mathbb{C}}{=}\sum_{k=0}^{\lambda_{p}\left(n\right)-1}\left(\sum_{j=0}^{p-1}\beta_{H}\left(\frac{j}{p}\right)\left(\varepsilon_{k}\left(n\right)\right)^{j}\right)\left(\frac{\mu_{0}}{p}\right)^{k}\kappa_{H}\left(\left[n\right]_{p^{k}}\right),\textrm{ }\forall n\in\mathbb{N}_{0}\label{eq:archimedean Chi_H when rho is arbitrary and alpha_H of 0 is 1} \end{equation} with the special case: \begin{equation} \chi_{H}\left(n\right)\overset{\mathbb{C}}{=}-\gamma_{H}\left(\frac{1}{2}\right)+\frac{2\beta_{H}\left(0\right)}{2-\mu_{0}}M_{H}\left(n\right)+\beta_{H}\left(0\right)\sum_{k=0}^{\lambda_{2}\left(n\right)-1}\left(\frac{\mu_{0}}{2}\right)^{k}\kappa_{H}\left(\left[n\right]_{2^{k}}\right),\textrm{ }\forall n\in\mathbb{N}_{0}\label{eq:archimedean Chi_H when rho is 2 and alpha_H of 0 is 1} \end{equation} when $p=2$. Regardless of the value of $p$, the $k$-sums are defined to be $0$ when $n=0$. \end{cor} Proof: Let $n\in\mathbb{N}_{0}$. Since $\kappa_{H}\left(\left[n\right]_{p^{k}}\right)=\kappa_{H}\left(n\right)$ and $\varepsilon_{k}\left(n\right)=1$ for all $k\geq\lambda_{p}\left(n\right)$, the equation (\ref{eq:Explicit Formula for Chi_H when alpha is 1 and rho is arbitrary}) becomes: \begin{align*} \chi_{H}\left(n\right) & \overset{\mathbb{C}}{=}\sum_{k=0}^{\lambda_{p}\left(n\right)-1}\left(\sum_{j=0}^{p-1}\beta_{H}\left(\frac{j}{p}\right)\left(\varepsilon_{k}\left(n\right)\right)^{j}\right)\left(\frac{\mu_{0}}{p}\right)^{k}\kappa_{H}\left(\left[n\right]_{p^{k}}\right)\\ & +\left(\sum_{k=\lambda_{p}\left(n\right)}^{\infty}\left(\sum_{j=0}^{p-1}\beta_{H}\left(\frac{j}{p}\right)\cdot1\right)\left(\frac{\mu_{0}}{p}\right)^{k}\right)\kappa_{H}\left(n\right) \end{align*} Since $H$ is contracting, the geometric series: \begin{equation} \sum_{k=\lambda_{p}\left(n\right)}^{\infty}\left(\frac{\mu_{0}}{p}\right)^{k} \end{equation} converges to a limit in $\mathbb{C}$. However, this does not matter much, because: \[ \sum_{j=0}^{p-1}\beta_{H}\left(\frac{j}{p}\right)=\sum_{j=0}^{p-1}\frac{1}{p}\sum_{\ell=0}^{p-1}\frac{b_{\ell}}{d_{\ell}}e^{-2\pi i\ell\frac{j}{p}}=\sum_{\ell=0}^{p-1}\frac{b_{\ell}}{d_{\ell}}\sum_{j=0}^{p-1}\frac{1}{p}e^{-2\pi i\ell\frac{j}{p}}=\frac{b_{\ell}}{d_{\ell}}\left[\ell\overset{p}{\equiv}0\right]=\frac{b_{0}}{d_{0}} \] Since $b_{0}=H\left(0\right)=0$, we are then left with: \begin{align*} \chi_{H}\left(n\right) & \overset{\mathbb{C}}{=}\sum_{k=0}^{\lambda_{p}\left(n\right)-1}\left(\sum_{j=0}^{p-1}\beta_{H}\left(\frac{j}{p}\right)\left(\varepsilon_{k}\left(n\right)\right)^{j}\right)\left(\frac{\mu_{0}}{p}\right)^{k}\kappa_{H}\left(\left[n\right]_{p^{k}}\right) \end{align*} which is the desired formula. As for the $p=2$ case, applying the same argument given above to (\ref{eq:Explicit Formula for Chi_H when alpha is 1 and rho is 2}) yields: \begin{align*} \chi_{H}\left(n\right) & \overset{\mathbb{C}}{=}-\gamma_{H}\left(\frac{1}{2}\right)+\beta_{H}\left(0\right)\sum_{k=0}^{\lambda_{2}\left(n\right)-1}\left(\frac{\mu_{0}}{2}\right)^{n}\kappa_{H}\left(\left[n\right]_{2^{k}}\right)\\ & +\beta_{H}\left(0\right)\kappa_{H}\left(n\right)\sum_{k=\lambda_{2}\left(n\right)}^{\infty}\left(\frac{\mu_{0}}{2}\right)^{k}\\ \left(H\textrm{ is contracting}\right); & \overset{\mathbb{C}}{=}-\gamma_{H}\left(\frac{1}{2}\right)+\beta_{H}\left(0\right)\sum_{k=0}^{\lambda_{2}\left(n\right)-1}\left(\frac{\mu_{0}}{2}\right)^{k}\kappa_{H}\left(\left[n\right]_{2^{k}}\right)\\ & +\beta_{H}\left(0\right)\frac{\left(\frac{\mu_{0}}{2}\right)^{\lambda_{2}\left(n\right)}\kappa_{H}\left(n\right)}{1-\frac{\mu_{0}}{2}} \end{align*} Finally, since: \[ \kappa_{H}\left(n\right)=\left(\frac{2}{\mu_{0}}\right)^{\lambda_{2}\left(n\right)}M_{H}\left(n\right) \] we then obtain: \begin{align*} \chi_{H}\left(n\right) & \overset{\mathbb{C}}{=}-\gamma_{H}\left(\frac{1}{2}\right)+\beta_{H}\left(0\right)\frac{M_{H}\left(n\right)}{1-\frac{\mu_{0}}{2}}+\beta_{H}\left(0\right)\sum_{k=0}^{\lambda_{2}\left(n\right)-1}\left(\frac{\mu_{0}}{2}\right)^{k}\kappa_{H}\left(\left[n\right]_{2^{k}}\right) \end{align*} Q.E.D. \vphantom{} Taken together, these two corollaries then establish the quasi-integrability of $\chi_{H}$\index{chi{H}@$\chi_{H}$!quasi-integrability} for $p\geq2$ and $\alpha_{H}\left(0\right)=1$. \begin{cor}[\textbf{Quasi-Integrability of $\chi_{H}$ when $\alpha_{H}\left(0\right)=1$}] \label{cor:Quasi-integrability of Chi_H for alpha equals 1}If $\alpha_{H}\left(0\right)=1$, then, $\chi_{H}$ is quasi-integrable with respect to the standard $\left(p,q_{H}\right)$-adic frame. In\index{chi{H}@$\chi_{H}$!Fourier transform} particular, when $p=2$, the function $\hat{\chi}_{H}:\hat{\mathbb{Z}}_{2}\rightarrow\overline{\mathbb{Q}}$ defined by: \begin{equation} \hat{\chi}_{H}\left(t\right)\overset{\textrm{def}}{=}\begin{cases} -\gamma_{H}\left(\frac{1}{2}\right) & \textrm{if }t=0\\ \beta_{H}\left(0\right)v_{2}\left(t\right)\hat{A}_{H}\left(t\right) & \textrm{else } \end{cases},\textrm{ }\forall t\in\hat{\mathbb{Z}}_{2}\label{eq:Formula for Chi_H hat when rho is 2 and alpha is 1} \end{equation} is then a Fourier transform of $\chi_{H}$. In this case, the function defined by \emph{(\ref{eq:Point-wise limit of the left-hand side of the fine structure formula when alpha is 1})} is also a Fourier transform of $\chi_{H}$, differing from the $\hat{\chi}_{H}$ given above by $\gamma_{H}\left(\frac{1}{2}\right)\hat{A}_{H}\left(t\right)$, which, by \textbf{\emph{Theorem \ref{thm:Properties of dA_H}}}, is a degenerate measure, seeing as $\alpha_{H}\left(0\right)=1$. For $p\geq3$, we can obtain a Fourier transform for $\chi_{H}$ by defining a function $\hat{\chi}_{H}:\hat{\mathbb{Z}}_{p}\rightarrow\overline{\mathbb{Q}}$ by: \begin{equation} \hat{\chi}_{H}\left(t\right)\overset{\textrm{def}}{=}\begin{cases} 0 & \textrm{if }t=0\\ \left(\gamma_{H}\left(\frac{t\left|t\right|_{p}}{p}\right)+\beta_{H}\left(0\right)v_{p}\left(t\right)\right)\hat{A}_{H}\left(t\right) & \textrm{else} \end{cases}\label{eq:Chi_H hat when rho is not 2 and when alpha is 1} \end{equation} \end{cor} Proof: Corollaries\textbf{ \ref{cor:Chi_H explicit formula on N_0}} and\textbf{ \ref{cor:Chi_H, F-convergent formula}} show that the $N$th partial sums of the Fourier series generated by (\ref{eq:Point-wise limit of the left-hand side of the fine structure formula when alpha is 1}) are $\mathcal{F}_{p,q_{H}}$-convergent to (\ref{eq:Formula for Chi_H hat when rho is 2 and alpha is 1}) and (\ref{eq:Chi_H hat when rho is not 2 and when alpha is 1}) for $p=2$ and $p\geq3$, respectively, thereby establishing the quasi-integrability of $\chi_{H}$ with respect to the standard $\left(p,q_{H}\right)$-adic frame. Finally, letting $\hat{\chi}_{H}^{\prime}\left(t\right)$ denote (\ref{eq:Point-wise limit of the left-hand side of the fine structure formula when alpha is 1}), observe that when $\alpha_{H}\left(0\right)=1$ and $p=2$: \begin{equation} \hat{\chi}_{H}^{\prime}\left(t\right)\overset{\overline{\mathbb{Q}}}{=}\begin{cases} 0 & \textrm{if }t=0\\ \left(\gamma_{H}\left(\frac{1}{2}\right)+\beta_{H}\left(0\right)v_{2}\left(t\right)\right)\hat{A}_{H}\left(t\right) & \textrm{if }\left|t\right|_{2}>0 \end{cases} \end{equation} Since $\hat{A}_{H}\left(0\right)=1$, we have that: \begin{equation} \hat{\chi}_{H}^{\prime}\left(t\right)-\gamma_{H}\left(\frac{1}{2}\right)\hat{A}_{H}\left(t\right)\overset{\overline{\mathbb{Q}}}{=}\begin{cases} -\gamma_{H}\left(\frac{1}{2}\right) & \textrm{if }t=0\\ \beta_{H}\left(0\right)v_{2}\left(t\right)\hat{A}_{H}\left(t\right) & \textrm{if }\left|t\right|_{2}>0 \end{cases}=\hat{\chi}_{H}\left(t\right) \end{equation} which shows that $\hat{\chi}_{H}^{\prime}\left(t\right)$ and $\hat{\chi}_{H}\left(t\right)$ differ by a factor of $\gamma_{H}\left(\frac{1}{2}\right)\hat{A}_{H}\left(t\right)$, which is a degenerate measure because $\alpha_{H}\left(0\right)=1$. Q.E.D. \vphantom{} This completes the $\alpha_{H}\left(0\right)=1$ case. By using $\chi_{H}$'s functional equations (\ref{eq:Functional Equations for Chi_H over the rho-adics}), we can extend all of the above work to cover all $p$-Hydra maps, regardless of the value of $\alpha_{H}\left(0\right)$. The key to this are \textbf{Corollary \ref{cor:Chi_H explicit formula on N_0} }and \textbf{Lemma \ref{lem:Functional equations and truncation}}. First, however, a definition: \begin{defn}[\textbf{Little Psi-$H$ \& Big Psi-$H$}] \ \vphantom{} I. We define \nomenclature{$\psi_{H}\left(m\right)$}{ }$\psi_{H}:\mathbb{N}_{0}\rightarrow\mathbb{Q}$ (``Little Psi-$H$'') by: \begin{equation} \psi_{H}\left(m\right)\overset{\textrm{def}}{=}\frac{M_{H}\left(m\right)}{1-\frac{\mu_{0}}{p}}+\sum_{n=0}^{\lambda_{p}\left(m\right)-1}\left(\frac{\mu_{0}}{p}\right)^{n}\kappa_{H}\left(\left[m\right]_{p^{n}}\right),\textrm{ }\forall m\in\mathbb{N}_{0}\label{eq:Definition of Little Psi_H} \end{equation} where the $n$-sum is defined to be $0$ when $m=0$. \vphantom{} II. We define \nomenclature{$\Psi_{H}\left(m\right)$}{ }$\Psi_{H}:\mathbb{N}_{0}\rightarrow\mathbb{Q}$ (``Big Psi-$H$'') by: \begin{equation} \Psi_{H}\left(m\right)\overset{\textrm{def}}{=}-\beta_{H}\left(0\right)\frac{M_{H}\left(m\right)}{1-\frac{\mu_{0}}{p}}+\sum_{n=0}^{\lambda_{p}\left(m\right)-1}\left(\sum_{j=1}^{p-1}\beta_{H}\left(\frac{j}{p}\right)\varepsilon_{n}^{j}\left(m\right)\right)\left(\frac{\mu_{0}}{p}\right)^{n}\kappa_{H}\left(\left[m\right]_{p^{n}}\right)\label{eq:Definition of Big Psi_H} \end{equation} \end{defn} \vphantom{} As a simple computation will immediately verify, (\ref{eq:Definition of Little Psi_H}) and (\ref{eq:Definition of Big Psi_H}) are merely the sums in $\mathbb{C}$ of (\ref{eq:Limit of Fourier sum of v_p A_H hat}) and (\ref{eq:F limit of Gamma_H A_H hat Fourier series when alpha is 1}) from \textbf{Lemmata \ref{lem:v_p A_H hat summation formulae}} and \textbf{\ref{lem:1D gamma formula}}, respectively, for $\mathfrak{z}\in\mathbb{N}_{0}$. To derive Fourier transforms and quasi-integrability for $\chi_{H}$ when $\alpha_{H}\left(0\right)\neq1$, we will first show that $\psi_{H}$ and $\Psi_{H}$ are rising-continuable to standard-frame-quasi-integrable $\left(p,q_{H}\right)$-adic functions, and that these continuations are characterized by systems of functional equations very to near the ones satisfied by $\chi_{H}$ (\ref{eq:Functional Equations for Chi_H over the rho-adics}). With a bit of linear algebra, we can then \emph{solve }for the appropriate linear combinations of $\psi_{H}$ and $\Psi_{H}$ to needed to obtain $\chi_{H}$, regardless of the value of $\alpha_{H}\left(0\right)$. \begin{lem}[\textbf{Rising-Continuability and Functional Equations for $\psi_{H}$ \& $\Psi_{H}$}] \label{lem:Rising continuation and uniquenss of the psi_Hs}For $H$ as given at the start of \emph{Section \pageref{sec:4.2 Fourier-Transforms-=00003D000026}}: \vphantom{} I. $\psi_{H}$ is rising-continuable to a $\left(p,q_{H}\right)$-adic function $\psi_{H}:\mathbb{Z}_{p}\rightarrow\mathbb{Z}_{q_{H}}$ given by: \begin{equation} \psi_{H}\left(\mathfrak{z}\right)\overset{\mathcal{F}_{p,q_{H}}}{=}\sum_{n=0}^{\infty}\left(\frac{\mu_{0}}{p}\right)^{n}\kappa_{H}\left(\left[\mathfrak{z}\right]_{p^{n}}\right),\textrm{ }\forall\mathfrak{z}\in\mathbb{Z}_{p}\label{eq:Rising-continuation of Little Psi_H} \end{equation} Moreover, $\psi_{H}\left(\mathfrak{z}\right)$ is the unique rising-continuous $\left(p,q_{H}\right)$-adic function satisfying the system of \index{functional equation!psi_{H}@$\psi_{H}$}functional equations: \begin{equation} \psi_{H}\left(p\mathfrak{z}+j\right)=\frac{\mu_{j}}{p}\psi_{H}\left(\mathfrak{z}\right)+1,\textrm{ }\forall\mathfrak{z}\in\mathbb{Z}_{p}\textrm{ \& }\forall j\in\mathbb{Z}/p\mathbb{Z}\label{eq:Little Psi_H functional equations} \end{equation} \vphantom{} II. \index{functional equation!Psi_{H}@$\Psi_{H}$}$\Psi_{H}$ is rising-continuable to a $\left(p,q_{H}\right)$-adic function $\Psi_{H}:\mathbb{Z}_{p}\rightarrow\mathbb{C}_{q_{H}}$ given by: \begin{equation} \Psi_{H}\left(\mathfrak{z}\right)\overset{\mathcal{F}_{p,q_{H}}}{=}\sum_{n=0}^{\infty}\left(\sum_{j=1}^{p-1}\beta_{H}\left(\frac{j}{p}\right)\left(\varepsilon_{n}\left(\mathfrak{z}\right)\right)^{j}\right)\left(\frac{\mu_{0}}{p}\right)^{n}\kappa_{H}\left(\left[\mathfrak{z}\right]_{p^{n}}\right),\textrm{ }\forall\mathfrak{z}\in\mathbb{Z}_{p}\label{eq:Rising-continuation of Big Psi_H} \end{equation} Moreover, $\Psi_{H}\left(\mathfrak{z}\right)$ is the unique rising-continuous $\left(p,q_{H}\right)$-adic function satisfying the system of functional equations: \begin{equation} \Psi_{H}\left(p\mathfrak{z}+j\right)=\frac{\mu_{j}}{p}\Psi_{H}\left(\mathfrak{z}\right)+H_{j}\left(0\right)-\beta_{H}\left(0\right),\textrm{ }\forall\mathfrak{z}\in\mathbb{Z}_{p}\textrm{ \& }\forall j\in\mathbb{Z}/p\mathbb{Z}\label{eq:Big Psi_H functional equations} \end{equation} \end{lem} \begin{rem} (\ref{eq:Little Psi_H functional equations}) is the most significant part of this result. It shows that $\psi_{H}$ is a shifted form of $\chi_{H}$, and\textemdash crucially\textemdash that this relation between $\chi_{H}$ and $\psi_{H}$ is \emph{independent }of the value of $\alpha_{H}\left(0\right)$. Indeed, it is solely a consequence of the properties of the expression (\ref{eq:Definition of Little Psi_H}). \end{rem} Proof: For both parts, we use (\ref{eq:Relation between truncations and functional equations, version 2}) from \textbf{Lemma \ref{lem:Functional equations and truncation}}. With this, observe that the functional equations (\ref{eq:Kappa_H functional equations}): \begin{equation} \kappa_{H}\left(pm+j\right)=\frac{\mu_{j}}{\mu_{0}}\kappa_{H}\left(m\right) \end{equation} then imply that: \begin{equation} \kappa_{H}\left(\left[pm+j\right]_{p^{n}}\right)=\frac{\mu_{j}}{\mu_{0}}\kappa_{H}\left(\left[m\right]_{p^{n-1}}\right),\textrm{ }\forall m\in\mathbb{N}_{0},\textrm{ }\forall n\in\mathbb{N}_{1},\textrm{ }\forall j\in\mathbb{Z}/p\mathbb{Z}\label{eq:functional equation truncation Lemma applied to kappa_H} \end{equation} where the function $\Phi_{j}$ from (\ref{eq:Relation between truncations and functional equations, version 1}) is, in this case: \begin{equation} \Phi_{j}\left(m,n\right)=\frac{\mu_{j}}{\mu_{0}}n \end{equation} The rising-continuability of $\psi_{H}$ and $\psi_{H}$ to the given series follow by the givens on $H$, which guarantee that, for each $\mathfrak{z}\in\mathbb{Z}_{p}$, $M_{H}\left(\left[\mathfrak{z}\right]_{p^{n}}\right)=\left(\frac{\mu_{0}}{p}\right)^{n}\kappa_{H}\left(\left[\mathfrak{z}\right]_{p^{n}}\right)$ tends to $0$ in the standard $\left(p,q_{H}\right)$-adic frame as $n\rightarrow\infty$, and convergence is then guaranteed by the same arguments used for (\ref{eq:Limit of Fourier sum of v_p A_H hat}) and (\ref{eq:F limit of Gamma_H A_H hat Fourier series when alpha is 1}) from \textbf{Lemmata \ref{lem:v_p A_H hat summation formulae}} and \textbf{\ref{lem:1D gamma formula}}, respectively. All that remains is to verify the functional equations. \textbf{Theorem \ref{thm:rising-continuability of Generic H-type functional equations}} from Subsection \ref{subsec:3.2.1 -adic-Interpolation-of} then guarantees the uniqueness of $\psi_{H}$ and $\Psi_{H}$ as rising-continuous solutions of their respective systems of functional equations. \vphantom{} I. We pull out the $n=0$ term from $\psi_{H}\left(pm+j\right)$: \begin{align*} \psi_{H}\left(pm+j\right) & =\frac{\mu_{j}}{p}\frac{M_{H}\left(m\right)}{1-\frac{\mu_{0}}{p}}+\overbrace{\kappa_{H}\left(0\right)}^{1}+\sum_{n=1}^{\lambda_{p}\left(m\right)}\left(\frac{\mu_{0}}{p}\right)^{n}\kappa_{H}\left(\left[pm+j\right]_{p^{n}}\right)\\ & =\frac{\mu_{j}}{p}\frac{M_{H}\left(m\right)}{1-\frac{\mu_{0}}{p}}+1+\sum_{n=1}^{\lambda_{p}\left(m\right)}\left(\frac{\mu_{0}}{p}\right)^{n}\frac{\mu_{j}}{p}\kappa_{H}\left(\left[m\right]_{p^{n-1}}\right)\\ & =1+\frac{\mu_{j}}{p}\left(\frac{M_{H}\left(m\right)}{1-\frac{\mu_{0}}{p}}+\sum_{n=1}^{\lambda_{p}\left(m\right)}\left(\frac{\mu_{0}}{p}\right)^{n-1}\kappa_{H}\left(\left[m\right]_{p^{n-1}}\right)\right)\\ & =1+\frac{\mu_{j}}{p}\underbrace{\left(\frac{M_{H}\left(m\right)}{1-\frac{\mu_{0}}{p}}+\sum_{n=0}^{\lambda_{p}\left(m\right)-1}\left(\frac{\mu_{0}}{p}\right)^{n}\kappa_{H}\left(\left[m\right]_{p^{n}}\right)\right)}_{\psi_{H}\left(m\right)}\\ & =1+\frac{\mu_{j}}{p}\psi_{H}\left(m\right) \end{align*} Consequently: \begin{equation} \psi_{H}\left(pm+j\right)=\frac{\mu_{j}}{p}\psi_{H}\left(m\right)+1,\textrm{ }\forall m\geq0\textrm{ \& }\forall j\in\mathbb{Z}/p\mathbb{Z}\label{eq:Little Psi_H functional equation on the integers} \end{equation} shows that (\ref{eq:Little Psi_H functional equation on the integers}) extends to hold for the rising-continuation of $\psi_{H}$, and that this rising-continuation is the \emph{unique }$\left(p,q_{H}\right)$-adic function satisfying (\ref{eq:Little Psi_H functional equations}). Finally, letting $m\in\mathbb{N}_{0}$ and setting $\mathfrak{z}=m$, the right-hand side of (\ref{eq:Rising-continuation of Little Psi_H}) becomes: \begin{align*} \sum_{n=0}^{\infty}\left(\frac{\mu_{0}}{p}\right)^{n}\kappa_{H}\left(\left[m\right]_{p^{n}}\right) & =\sum_{n=0}^{\lambda_{p}\left(m\right)-1}\left(\frac{\mu_{0}}{p}\right)^{n}\kappa_{H}\left(\left[m\right]_{p^{n}}\right)+\sum_{n=\lambda_{p}\left(m\right)}^{\infty}\left(\frac{\mu_{0}}{p}\right)^{n}\kappa_{H}\left(m\right)\\ & \overset{\mathbb{C}}{=}\sum_{n=0}^{\lambda_{p}\left(m\right)-1}\left(\frac{\mu_{0}}{p}\right)^{n}\kappa_{H}\left(\left[m\right]_{p^{n}}\right)+\frac{\kappa_{H}\left(m\right)\left(\frac{\mu_{0}}{p}\right)^{\lambda_{p}\left(m\right)}}{1-\frac{\mu_{0}}{p}}\\ & =\frac{M_{H}\left(m\right)}{1-\frac{\mu_{0}}{p}}+\sum_{n=0}^{\lambda_{p}\left(m\right)-1}\left(\frac{\mu_{0}}{p}\right)^{n}\kappa_{H}\left(\left[m\right]_{p^{n}}\right)\\ & =\psi_{H}\left(m\right) \end{align*} Hence, (\ref{eq:Rising-continuation of Little Psi_H}) converges to $\psi_{H}$ in the standard frame. \vphantom{} II. Pulling out $n=0$ from (\ref{eq:Definition of Big Psi_H}) yields: \begin{align*} \Psi_{H}\left(m\right) & =-\beta_{H}\left(0\right)\frac{M_{H}\left(m\right)}{1-\frac{\mu_{0}}{p}}+\sum_{k=1}^{p-1}\beta_{H}\left(\frac{k}{p}\right)\varepsilon_{0}^{k}\left(m\right)\\ & +\sum_{n=1}^{\lambda_{p}\left(m\right)-1}\left(\sum_{k=1}^{p-1}\beta_{H}\left(\frac{k}{p}\right)\varepsilon_{n}^{k}\left(m\right)\right)\left(\frac{\mu_{0}}{p}\right)^{n}\kappa_{H}\left(\left[m\right]_{p^{n}}\right) \end{align*} Then: \begin{align*} \sum_{k=0}^{p-1}\beta_{H}\left(\frac{k}{p}\right)\varepsilon_{0}^{k}\left(m\right) & =\sum_{k=0}^{p-1}\left(\frac{1}{p}\sum_{j=0}^{p-1}H_{j}\left(0\right)e^{-\frac{2\pi ijk}{p}}\right)\left(e^{\frac{2\pi i}{p}\left(\left[m\right]_{p}-\left[m\right]_{p^{0}}\right)}\right)^{k}\\ & =\sum_{j=0}^{p-1}H_{j}\left(0\right)\frac{1}{p}\sum_{k=0}^{p-1}e^{-\frac{2\pi ijk}{p}}e^{\frac{2\pi ikm}{p}}\\ & =\sum_{j=0}^{p-1}H_{j}\left(0\right)\frac{1}{p}\sum_{k=0}^{p-1}e^{\frac{2\pi ik\left(m-j\right)}{p}}\\ & =\sum_{j=0}^{p-1}H_{j}\left(0\right)\left[j\overset{p}{\equiv}m\right]\\ & =H_{\left[m\right]_{p}}\left(0\right) \end{align*} and so: \begin{equation} \sum_{k=1}^{p-1}\beta_{H}\left(\frac{k}{p}\right)\varepsilon_{0}^{k}\left(m\right)=H_{\left[m\right]_{p}}\left(0\right)-\beta_{H}\left(0\right)\label{eq:Fourier sum (but not with 0) of beta_H} \end{equation} Consequently: \begin{align*} \Psi_{H}\left(m\right) & =-\beta_{H}\left(0\right)\frac{M_{H}\left(m\right)}{1-\frac{\mu_{0}}{p}}+H_{\left[m\right]_{p}}\left(0\right)-\beta_{H}\left(0\right)\\ & +\sum_{n=1}^{\lambda_{p}\left(m\right)-1}\left(\sum_{k=1}^{p-1}\beta_{H}\left(\frac{k}{p}\right)\varepsilon_{n}^{k}\left(m\right)\right)\left(\frac{\mu_{0}}{p}\right)^{n}\kappa_{H}\left(\left[m\right]_{p^{n}}\right) \end{align*} Replacing $m$ with $pm+j$ (where at least one of $m$ and $j$ is non-zero), we use (\ref{eq:functional equation truncation Lemma applied to kappa_H}) and the functional equations for $M_{H}$, $\varepsilon_{n}$, and $\lambda_{p}$ to obtain: \begin{align*} \Psi_{H}\left(m\right) & =-\beta_{H}\left(0\right)\frac{\mu_{j}}{p}\frac{M_{H}\left(m\right)}{1-\frac{\mu_{0}}{p}}+H_{j}\left(0\right)-\beta_{H}\left(0\right)\\ & +\sum_{n=1}^{\lambda_{p}\left(m\right)}\left(\sum_{k=1}^{p-1}\beta_{H}\left(\frac{k}{p}\right)\varepsilon_{n-1}^{k}\left(m\right)\varepsilon_{n}^{k}\left(j\right)\right)\left(\frac{\mu_{0}}{p}\right)^{n}\frac{\mu_{j}}{\mu_{0}}\kappa_{H}\left(\left[m\right]_{p^{n-1}}\right) \end{align*} Because $j\in\left\{ 0,\ldots,p-1\right\} $, note that $\left[j\right]_{p^{n}}=j$ for all $n\geq1$. As such: \[ \varepsilon_{n}\left(j\right)=e^{\frac{2\pi i}{p^{n+1}}\left(\left[j\right]_{p^{n+1}}-\left[j\right]_{p^{n}}\right)}=e^{\frac{2\pi i}{p^{n+1}}\left(j-j\right)}=1,\textrm{ }\forall n\geq1 \] Re-indexing the $n$-sum then gives us: \begin{align*} \Psi_{H}\left(m\right) & =-\beta_{H}\left(0\right)\frac{\mu_{j}}{p}\frac{M_{H}\left(m\right)}{1-\frac{\mu_{0}}{p}}+H_{j}\left(0\right)-\beta_{H}\left(0\right)\\ & +\sum_{n=0}^{\lambda_{p}\left(m\right)-1}\left(\sum_{k=1}^{p-1}\beta_{H}\left(\frac{k}{p}\right)\varepsilon_{n}^{k}\left(m\right)\right)\left(\frac{\mu_{0}}{p}\right)^{n}\frac{\mu_{0}}{p}\frac{\mu_{j}}{\mu_{0}}\kappa_{H}\left(\left[m\right]_{p^{n}}\right)\\ & =H_{j}\left(0\right)-\beta_{H}\left(0\right)\\ & +\frac{\mu_{j}}{p}\underbrace{\left(-\beta_{H}\left(0\right)\frac{M_{H}\left(m\right)}{1-\frac{\mu_{0}}{p}}+\sum_{n=0}^{\lambda_{p}\left(m\right)-1}\left(\sum_{k=1}^{p-1}\beta_{H}\left(\frac{k}{p}\right)\varepsilon_{n}^{k}\left(m\right)\right)\left(\frac{\mu_{0}}{p}\right)^{n}\kappa_{H}\left(\left[m\right]_{p^{n}}\right)\right)}_{\Psi_{H}\left(m\right)}\\ & =\frac{\mu_{j}}{p}\Psi_{H}\left(m\right)+H_{j}\left(0\right)-\beta_{H}\left(0\right) \end{align*} Finally, letting $\mathfrak{z}=m$ where $m\in\mathbb{N}_{0}$, the right-hand side of (\ref{eq:Rising-continuation of Big Psi_H}) becomes: \begin{align*} \sum_{n=0}^{\lambda_{p}\left(m\right)}\left(\sum_{j=1}^{p-1}\beta_{H}\left(\frac{j}{p}\right)\left(\varepsilon_{n}\left(m\right)\right)^{j}\right)\left(\frac{\mu_{0}}{p}\right)^{n}\kappa_{H}\left(\left[m\right]_{p^{n}}\right)\\ +\sum_{n=\lambda_{p}\left(m\right)}^{\infty}\left(\sum_{j=1}^{p-1}\beta_{H}\left(\frac{j}{p}\right)\left(\varepsilon_{n}\left(m\right)\right)^{j}\right)\left(\frac{\mu_{0}}{p}\right)^{n}\kappa_{H}\left(m\right) \end{align*} Here: \begin{equation} \varepsilon_{n}\left(m\right)=e^{\frac{2\pi i}{p^{n+1}}\left(\left[m\right]_{p^{n+1}}-\left[m\right]_{p^{n}}\right)}=e^{\frac{2\pi i}{p^{n+1}}\left(m-m\right)}=1,\textrm{ }\forall n\geq\lambda_{p}\left(m\right) \end{equation} So: \begin{align*} \sum_{n=0}^{\lambda_{p}\left(m\right)}\left(\sum_{j=1}^{p-1}\beta_{H}\left(\frac{j}{p}\right)\left(\varepsilon_{n}\left(m\right)\right)^{j}\right)\left(\frac{\mu_{0}}{p}\right)^{n}\kappa_{H}\left(\left[m\right]_{p^{n}}\right)\\ +\left(\sum_{j=1}^{p-1}\beta_{H}\left(\frac{j}{p}\right)\right)\sum_{n=\lambda_{p}\left(m\right)}^{\infty}\left(\frac{\mu_{0}}{p}\right)^{n}\kappa_{H}\left(m\right) \end{align*} Summing this in the topology of $\mathbb{C}$ yields: \begin{align*} \sum_{n=0}^{\lambda_{p}\left(m\right)}\left(\sum_{j=1}^{p-1}\beta_{H}\left(\frac{j}{p}\right)\left(\varepsilon_{n}\left(m\right)\right)^{j}\right)\left(\frac{\mu_{0}}{p}\right)^{n}\kappa_{H}\left(\left[m\right]_{p^{n}}\right)\\ +\left(\sum_{j=1}^{p-1}\beta_{H}\left(\frac{j}{p}\right)\right)\frac{M_{H}\left(m\right)}{1-\frac{\mu_{0}}{p}} \end{align*} Using (\ref{eq:Fourier sum (but not with 0) of beta_H}) with $m=0$ (which makes $\varepsilon_{0}^{k}\left(m\right)$ identically equal to $1$) gives us: \begin{equation} \sum_{j=1}^{p-1}\beta_{H}\left(\frac{j}{p}\right)=H_{0}\left(0\right)-\beta_{H}\left(0\right)=-\beta_{H}\left(0\right) \end{equation} and so, we are left with: \begin{equation} -\beta_{H}\left(0\right)\frac{M_{H}\left(m\right)}{1-\frac{\mu_{0}}{p}}+\sum_{n=0}^{\lambda_{p}\left(m\right)}\left(\sum_{j=1}^{p-1}\beta_{H}\left(\frac{j}{p}\right)\left(\varepsilon_{n}\left(m\right)\right)^{j}\right)\left(\frac{\mu_{0}}{p}\right)^{n}\kappa_{H}\left(\left[m\right]_{p^{n}}\right) \end{equation} This, of course, is precisely $\Psi_{H}\left(m\right)$. This proves that (\ref{eq:Rising-continuation of Big Psi_H}) converges to $\Psi_{H}$ in the standard frame. Q.E.D. \begin{prop}[\textbf{Quasi-Integrability of $\psi_{H}$ \& $\Psi_{H}$}] Let $H$ be as given at the start of \emph{Section \pageref{sec:4.2 Fourier-Transforms-=00003D000026}}. Then: \vphantom{} I. \begin{equation} \psi_{H}\left(\mathfrak{z}\right)\overset{\mathcal{F}_{p,q_{H}}}{=}\begin{cases} \lim_{N\rightarrow\infty}\sum_{0<\left|t\right|_{p}\leq p^{N}}v_{p}\left(t\right)\hat{A}_{H}\left(t\right)e^{2\pi i\left\{ t\mathfrak{z}\right\} _{p}} & \textrm{if }\alpha_{H}\left(0\right)=1\\ \lim_{N\rightarrow\infty}\sum_{\left|t\right|_{p}\leq p^{N}}\frac{\hat{A}_{H}\left(t\right)}{1-\alpha_{H}\left(0\right)}e^{2\pi i\left\{ t\mathfrak{z}\right\} _{p}} & \textrm{if }\alpha_{H}\left(0\right)\neq1 \end{cases},\textrm{ }\forall\mathfrak{z}\in\mathbb{Z}_{p}\label{eq:Little Psi_H as an F-limit} \end{equation} As such, $\psi_{H}$ is quasi-integrable with respect to the standard $\left(p,q_{H}\right)$-adic quasi-integrability frame, and the function $\hat{\psi}_{H}:\hat{\mathbb{Z}}_{p}\rightarrow\overline{\mathbb{Q}}$ defined by: \begin{equation} \hat{\psi}_{H}\left(t\right)\overset{\textrm{def}}{=}\begin{cases} \begin{cases} 0 & \textrm{if }t=0\\ v_{p}\left(t\right)\hat{A}_{H}\left(t\right) & \textrm{if }t\neq0 \end{cases} & \textrm{if }\alpha_{H}\left(0\right)=1\\ \frac{\hat{A}_{H}\left(t\right)}{1-\alpha_{H}\left(0\right)} & \textrm{if }\alpha_{H}\left(0\right)\neq1 \end{cases},\textrm{ }\forall t\in\hat{\mathbb{Z}}_{p}\label{eq:Fourier Transform of Little Psi_H} \end{equation} is a Fourier transform of $\psi_{H}$. Hence: \begin{equation} \hat{\psi}_{H,N}\left(\mathfrak{z}\right)\overset{\overline{\mathbb{Q}}}{=}-N\left(\frac{\mu_{0}}{p}\right)^{N}\kappa_{H}\left(\left[\mathfrak{z}\right]_{p^{N}}\right)+\sum_{n=0}^{N-1}\left(\frac{\mu_{0}}{p}\right)^{n}\kappa_{H}\left(\left[\mathfrak{z}\right]_{p^{n}}\right)\label{eq:Little Psi H N twiddle when alpha is 1} \end{equation} when $\alpha_{H}\left(0\right)=1$ and: \begin{equation} \tilde{\psi}_{H,N}\left(\mathfrak{z}\right)\overset{\overline{\mathbb{Q}}}{=}\left(\frac{\mu_{0}}{p}\right)^{N}\frac{\kappa_{H}\left(\left[\mathfrak{z}\right]_{p^{N}}\right)}{1-\alpha_{H}\left(0\right)}+\sum_{n=0}^{N-1}\left(\frac{\mu_{0}}{p}\right)^{n}\kappa_{H}\left(\left[\mathfrak{z}\right]_{p^{n}}\right)\label{eq:Little Psi H N twiddle when alpha is not 1} \end{equation} when $\alpha_{H}\left(0\right)\neq1$. \vphantom{} II. \begin{equation} \Psi_{H}\left(\mathfrak{z}\right)\overset{\mathcal{F}_{p,q_{H}}}{=}\lim_{N\rightarrow\infty}\sum_{0<\left|t\right|_{p}\leq p^{N}}\gamma_{H}\left(\frac{t\left|t\right|_{p}}{p}\right)\hat{A}_{H}\left(t\right)e^{2\pi i\left\{ t\mathfrak{z}\right\} _{p}},\textrm{ }\forall\mathfrak{z}\in\mathbb{Z}_{p}\label{eq:Big Psi_H as an F-limit} \end{equation} As such, $\Psi_{H}$ is quasi-integrable with respect to the standard $\left(p,q_{H}\right)$-adic quasi-integrability frame, and the function $\hat{\Psi}_{H}:\hat{\mathbb{Z}}_{p}\rightarrow\overline{\mathbb{Q}}$ defined by: \begin{equation} \hat{\Psi}_{H}\left(t\right)\overset{\textrm{def}}{=}\begin{cases} 0 & \textrm{if }t=0\\ \gamma_{H}\left(\frac{t\left|t\right|_{p}}{p}\right)\hat{A}_{H}\left(t\right) & \textrm{if }t\neq0 \end{cases},\textrm{ }\forall t\in\hat{\mathbb{Z}}_{p}\label{eq:Fourier Transform of Big Psi_H} \end{equation} is a Fourier transform of $\Psi_{H}$. Hence: \begin{equation} \tilde{\Psi}_{H,N}\left(\mathfrak{z}\right)\overset{\overline{\mathbb{Q}}}{=}\sum_{n=0}^{N-1}\left(\sum_{j=1}^{p-1}\beta_{H}\left(\frac{j}{p}\right)\varepsilon_{n}^{j}\left(\mathfrak{z}\right)\right)\left(\frac{\mu_{0}}{p}\right)^{n}\kappa_{H}\left(\left[\mathfrak{z}\right]_{p^{n}}\right)\label{eq:Big Psi H N twiddle} \end{equation} \end{prop} Proof: I. When $\alpha_{H}\left(0\right)=1$, (\ref{eq:Little Psi_H as an F-limit}) follows from the limit (\ref{eq:Limit of Fourier sum of v_p A_H hat}) from \textbf{Lemma \ref{lem:v_p A_H hat summation formulae}}. So, the $\alpha_{H}\left(0\right)=1$ case of (\ref{eq:Fourier Transform of Little Psi_H}) is then indeed a Fourier transform of $\psi_{H}$. This establishes the $\mathcal{F}_{p,q_{H}}$-quasi-integrability of $\psi_{H}$ when $\alpha_{H}\left(0\right)=1$. When $\alpha_{H}\left(0\right)\neq1$, comparing (\ref{eq:Convolution of dA_H and D_N}) and (\ref{eq:Rising-continuation of Little Psi_H}) we observe that $\left(1-\alpha_{H}\left(0\right)\right)\psi_{H}\left(\mathfrak{z}\right)$ then takes the values: \[ \lim_{N\rightarrow\infty}\sum_{\left|t\right|_{p}\leq p^{N}}\hat{A}_{H}\left(t\right)e^{2\pi i\left\{ t\mathfrak{z}\right\} _{p}} \] at every $\mathfrak{z}\in\mathbb{Z}_{p}^{\prime}$. Since $H$ is semi-basic, applying our standard argument involving the $q$-adic decay of: \[ \left(\frac{\mu_{0}}{p}\right)^{n}\kappa_{H}\left(\left[\mathfrak{z}\right]_{p^{n}}\right) \] to (\ref{eq:Little Psi_H functional equation on the integers}) shows that $\psi_{H}\left(\left[\mathfrak{z}\right]_{p^{N}}\right)$ converges in $\mathbb{Z}_{q_{H}}$ as $N\rightarrow\infty$ for all $\mathfrak{z}\in\mathbb{Z}_{p}$. This establishes the rising-continuity of $\psi_{H}$. Consequently, the function $A_{H}:\mathbb{Z}_{p}\rightarrow\mathbb{Z}_{q_{H}}$ defined by: \begin{equation} A_{H}\left(\mathfrak{z}\right)\overset{\textrm{def}}{=}\begin{cases} \left(1-\alpha_{H}\left(0\right)\right)\psi_{H}\left(\mathfrak{z}\right) & \textrm{if }\mathfrak{z}\in\mathbb{N}_{0}\\ \left(1-\alpha_{H}\left(0\right)\right)\lim_{N\rightarrow\infty}\sum_{\left|t\right|_{p}\leq p^{N}}\hat{A}_{H}\left(t\right)e^{2\pi i\left\{ t\mathfrak{z}\right\} _{p}} & \textrm{if }\mathfrak{z}\in\mathbb{Z}_{p}^{\prime} \end{cases}\label{eq:definition of A_H} \end{equation} is then rising-continuous, and its restriction to $\mathbb{N}_{0}$ is equal to $\left(1-\alpha_{H}\left(0\right)\right)\psi_{H}\left(\mathfrak{z}\right)$. By (\ref{eq:Little Psi_H as an F-limit}), it then follows that $\hat{A}_{H}$ is a Fourier transform of $A_{H}$ with respect to the standard $\left(p,q_{H}\right)$-adic frame, and thus, that $A_{H}$ is $\mathcal{F}_{p,q_{H}}$-quasi-integrable. So, when $\alpha_{H}\left(0\right)\neq1$, $\psi_{H}\left(\mathfrak{z}\right)=\left(1-\alpha_{H}\left(0\right)\right)A_{H}\left(\mathfrak{z}\right)$ is also $\mathcal{F}_{p,q_{H}}$-quasi-integrable, and the $\alpha_{H}\left(0\right)\neq1$ case of (\ref{eq:Fourier Transform of Little Psi_H}) is then a Fourier transform of $\psi_{H}$. As for formulae (\ref{eq:Little Psi H N twiddle when alpha is 1}) and (\ref{eq:Little Psi H N twiddle when alpha is not 1}), these are nothing more than restatements of \textbf{Lemma \ref{lem:v_p A_H hat summation formulae}} and (\ref{eq:Convolution of dA_H and D_N}), respectively. \vphantom{} II. (\ref{eq:Big Psi_H as an F-limit}) and (\ref{eq:Fourier Transform of Big Psi_H}) are exactly what we proved in \textbf{Lemma \ref{lem:1D gamma formula}}; (\ref{eq:Big Psi H N twiddle}) is just (\ref{eq:Gamma formula}). Q.E.D. \vphantom{} Now that we know that $\psi_{H}$ and $\Psi_{H}$ are quasi-integrable \emph{regardless} of the value of $\alpha_{H}\left(0\right)$ (or $p$, for that matter), we can then reverse-engineer formulae for the Fourier transform of $\chi_{H}$ \emph{regardless }of the value of $\alpha_{H}\left(0\right)$; we need only solve a system of linear equations to find the constants which relate $\psi_{H}$, $\Psi_{H}$ and $\chi_{H}$. We do the $p=2$ and $p\geq3$ cases separately. Both of the following theorems can then be immediately applied to obtain formulae for $\hat{\chi}_{H}$, $\tilde{\chi}_{H,N}\left(\mathfrak{z}\right)$ \emph{and} $\chi_{H}\left(\mathfrak{z}\right)$. \begin{thm}[\textbf{Quasi-Integrability of $\chi_{H}$ for a $2$-Hydra map}] \label{thm:Quasi-integrability of an arbitrary 2-Hydra map}Let $H$ be any contracting non-singular semi-basic $2$-Hydra map which fixes $0$. Then:\index{chi{H}@$\chi_{H}$!quasi-integrability} \begin{equation} \chi_{H}\left(\mathfrak{z}\right)\overset{\mathcal{F}_{2,q_{H}}}{=}-\gamma_{H}\left(\frac{1}{2}\right)+\left(1-H^{\prime}\left(0\right)\right)\gamma_{H}\left(\frac{1}{2}\right)\psi_{H}\left(\mathfrak{z}\right),\textrm{ }\forall\mathfrak{z}\in\mathbb{Z}_{2}\label{eq:Chi_H in terms of Little Psi_H for a 2-Hydra map} \end{equation} In particular, by \emph{(\ref{eq:Fourier Transform of Little Psi_H})}, this shows that $\chi_{H}$ is then quasi-integrable with respect to the standard $\left(2,q_{H}\right)$-adic quasi-integrability frame, with the function $\hat{\chi}_{H}:\hat{\mathbb{Z}}_{2}\rightarrow\overline{\mathbb{Q}}$ defined by: \index{chi{H}@$\chi_{H}$!Fourier transform} \begin{equation} \hat{\chi}_{H}\left(t\right)\overset{\textrm{def}}{=}\begin{cases} \begin{cases} -\gamma_{H}\left(\frac{1}{2}\right) & \textrm{if }t=0\\ \left(1-H^{\prime}\left(0\right)\right)\gamma_{H}\left(\frac{1}{2}\right)v_{2}\left(t\right)\hat{A}_{H}\left(t\right) & \textrm{if }t\neq0 \end{cases} & \textrm{if }\alpha_{H}\left(0\right)=1\\ -\gamma_{H}\left(\frac{1}{2}\right)\mathbf{1}_{0}\left(t\right)+\gamma_{H}\left(\frac{1}{2}\right)\frac{1-H^{\prime}\left(0\right)}{1-\alpha_{H}\left(0\right)}\hat{A}_{H}\left(t\right) & \textrm{if }\alpha_{H}\left(0\right)\neq1 \end{cases},\textrm{ }\forall t\in\hat{\mathbb{Z}}_{2}\label{eq:Fourier Transform of Chi_H for a contracting semi-basic 2-Hydra map} \end{equation} being a Fourier transform of $\chi_{H}$. \end{thm} Proof: Let $A,B$ be undetermined constants, and let $\psi\left(\mathfrak{z}\right)=A\chi_{H}\left(\mathfrak{z}\right)+B$. Then, multiplying $\chi_{H}$'s functional equations (\ref{eq:Functional Equations for Chi_H over the rho-adics}) by $A$ and adding $B$ gives: \begin{align*} A\chi_{H}\left(2\mathfrak{z}+j\right)+B & =\frac{Aa_{j}\chi_{H}\left(\mathfrak{z}\right)+Ab_{j}}{d_{j}}+B\\ & \Updownarrow\\ \psi\left(2\mathfrak{z}+j\right) & =\frac{a_{j}\left(\psi\left(\mathfrak{z}\right)-B\right)+Ab_{j}}{d_{j}}+B\\ & =\frac{\mu_{j}}{2}\psi\left(\mathfrak{z}\right)+A\frac{b_{j}}{d_{j}}+B\left(1-\frac{a_{j}}{d_{j}}\right) \end{align*} for $j\in\left\{ 0,1\right\} $. Since: \[ \psi_{H}\left(2\mathfrak{z}+j\right)=\frac{\mu_{j}}{2}\psi\left(\mathfrak{z}\right)+1 \] it then follows that to make $\psi=\psi_{H}$, we need for $A$ and $B$ to satisfy: \begin{align*} \frac{b_{0}}{d_{0}}A+\left(1-\frac{a_{0}}{d_{0}}\right)B & =1\\ \frac{b_{1}}{d_{1}}A+\left(1-\frac{a_{1}}{d_{1}}\right)B & =1 \end{align*} Now, since: \begin{align*} \psi_{H}\left(\mathfrak{z}\right) & =A\chi_{H}\left(\mathfrak{z}\right)+B\\ & \Updownarrow\\ \chi_{H}\left(\mathfrak{z}\right) & =\frac{1}{A}\psi_{H}\left(\mathfrak{z}\right)-\frac{B}{A} \end{align*} upon letting $C=\frac{1}{A}$ and $D=-\frac{B}{A}$, we have that $A=\frac{1}{C}$, $B=-\frac{D}{C}$, and hence, our linear system becomes: \begin{align*} d_{0}C+\left(d_{0}-a_{0}\right)D & =b_{0}\\ d_{1}C+\left(d_{1}-a_{1}\right)D & =b_{1} \end{align*} where: \begin{equation} \chi_{H}\left(\mathfrak{z}\right)=C\psi_{H}\left(\mathfrak{z}\right)+D\label{eq:Chi_H =00003D00003D C Psi_H + D} \end{equation} The determinant of the coefficient matrix is: \begin{equation} a_{0}d_{1}-a_{1}d_{0} \end{equation} So, as long as $a_{0}d_{1}\neq a_{1}d_{0}$, the system admits a unique solution, which is easily computed to be: \begin{align*} C & =\frac{a_{0}b_{1}-a_{1}b_{0}+b_{0}d_{1}-b_{1}d_{0}}{a_{0}d_{1}-a_{1}d_{0}}\\ D & =\frac{b_{1}d_{0}-d_{1}b_{0}}{a_{0}d_{1}-a_{1}d_{0}} \end{align*} Finally, note that: \begin{align*} a_{0}d_{1} & \neq a_{1}d_{0}\\ & \Updownarrow\\ 0 & \neq\frac{a_{0}}{d_{0}}-\frac{a_{1}}{d_{1}}\\ & =2\alpha_{H}\left(\frac{1}{2}\right) \end{align*} and so: \[ D=\frac{b_{1}d_{0}-d_{1}b_{0}}{a_{0}d_{1}-a_{1}d_{0}}=-\frac{\frac{1}{2}\left(\frac{b_{0}}{d_{0}}-\frac{b_{1}}{d_{1}}\right)}{\frac{1}{2}\left(\frac{a_{0}}{d_{0}}-\frac{a_{1}}{d_{1}}\right)}=-\frac{\beta_{H}\left(\frac{1}{2}\right)}{\alpha_{H}\left(\frac{1}{2}\right)}=-\gamma_{H}\left(\frac{1}{2}\right) \] As for $C$, note that our requirement that $H\left(0\right)=0$ then forces $b_{0}=0$, and so: \begin{align*} C & =\frac{a_{0}b_{1}-a_{1}b_{0}+b_{0}d_{1}-b_{1}d_{0}}{a_{0}d_{1}-a_{1}d_{0}}\\ & =\frac{a_{0}-d_{0}}{d_{0}}\frac{\frac{b_{1}}{d_{1}}}{\frac{a_{0}}{d_{0}}-\frac{a_{1}}{d_{1}}}\\ \left(\begin{array}{c} \alpha_{H}\left(\frac{1}{2}\right)=\frac{1}{2}\left(\frac{a_{0}}{a_{0}}-\frac{a_{1}}{a_{1}}\right)\\ \beta_{H}\left(\frac{1}{2}\right)=\frac{1}{2}\left(\frac{b_{0}}{d_{0}}-\frac{b_{1}}{d_{1}}\right)=-\frac{1}{2}\frac{b_{1}}{d_{1}} \end{array}\right); & =\frac{a_{0}-d_{0}}{d_{0}}\frac{-2\beta_{H}\left(\frac{1}{2}\right)}{2\alpha_{H}\left(\frac{1}{2}\right)}\\ & =\frac{d_{0}-a_{0}}{d_{0}}\gamma_{H}\left(\frac{1}{2}\right)\\ \left(H^{\prime}\left(0\right)=\frac{a_{0}}{d_{0}}\right); & =\left(1-H^{\prime}\left(0\right)\right)\gamma_{H}\left(\frac{1}{2}\right) \end{align*} Plugging these values for $C$ and $D$ into (\ref{eq:Chi_H =00003D00003D C Psi_H + D}) produces (\ref{eq:Chi_H in terms of Little Psi_H for a 2-Hydra map}). Q.E.D. \vphantom{} Next up, our non-trivial formulae for $\tilde{\chi}_{H,N}$. \begin{cor} \label{cor:Non-trivial formula for Chi_H,N twiddle for a 2-hydra map, alpha arbitrary}Let\index{chi{H}@$\chi_{H}$!$N$th partial Fourier series} $H$ be a $2$-Hydra map like in \textbf{\emph{Theorem \ref{thm:Quasi-integrability of an arbitrary 2-Hydra map}}}, with $\hat{\chi}_{H}$ being as defined by \emph{(\ref{eq:Fourier Transform of Chi_H for a contracting semi-basic 2-Hydra map})}. Then: \begin{align} \tilde{\chi}_{H,N}\left(\mathfrak{z}\right) & \overset{\overline{\mathbb{Q}}}{=}-\gamma_{H}\left(\frac{1}{2}\right)-\left(1-H^{\prime}\left(0\right)\right)\gamma_{H}\left(\frac{1}{2}\right)N\left(\frac{\mu_{0}}{2}\right)^{N}\kappa_{H}\left(\left[\mathfrak{z}\right]_{2^{N}}\right)\label{eq:Explicit Formula for Chi_H,N twiddle when rho is 2 and alpha is 1}\\ & +\left(1-H^{\prime}\left(0\right)\right)\gamma_{H}\left(\frac{1}{2}\right)\sum_{n=0}^{N-1}\left(\frac{\mu_{0}}{2}\right)^{n}\kappa_{H}\left(\left[\mathfrak{z}\right]_{2^{n}}\right)\nonumber \end{align} if $\alpha_{H}\left(0\right)=1$, and: \begin{align} \tilde{\chi}_{H,N}\left(\mathfrak{z}\right) & \overset{\overline{\mathbb{Q}}}{=}-\gamma_{H}\left(\frac{1}{2}\right)+\gamma_{H}\left(\frac{1}{2}\right)\frac{1-H^{\prime}\left(0\right)}{1-\alpha_{H}\left(0\right)}\left(\frac{\mu_{0}}{2}\right)^{N}\kappa_{H}\left(\left[\mathfrak{z}\right]_{2^{N}}\right)\label{eq:Explicit Formula for Chi_H,N twiddle when rho is 2 and alpha is not 1}\\ & +\gamma_{H}\left(\frac{1}{2}\right)\left(1-H^{\prime}\left(0\right)\right)\sum_{n=0}^{N-1}\left(\frac{\mu_{0}}{2}\right)^{n}\kappa_{H}\left(\left[\mathfrak{z}\right]_{2^{n}}\right)\nonumber \end{align} if $\alpha_{H}\left(0\right)\neq1$. \end{cor} Proof: Take (\ref{eq:Fourier Transform of Chi_H for a contracting semi-basic 2-Hydra map}) and use formulas from \textbf{Lemma \ref{lem:1D gamma formula}}, \textbf{Lemma (\ref{lem:v_p A_H hat summation formulae}}, and \textbf{Theorem \ref{thm:Properties of dA_H}}. Q.E.D. \begin{cor}[\textbf{$\mathcal{F}$-series for $\chi_{H}$ when $H$ is a $2$-Hydra map}] Let \label{cor:F-series for Chi_H for p equals 2}\index{chi{H}@$\chi_{H}$!mathcal{F}-series@$\mathcal{F}$-series}$H$ be as given in \textbf{\emph{Theorem \ref{thm:Quasi-integrability of an arbitrary 2-Hydra map}}}. Then: \begin{equation} \chi_{H}\left(\mathfrak{z}\right)\overset{\mathcal{F}_{2,q_{H}}}{=}-\gamma_{H}\left(\frac{1}{2}\right)+\left(1-H^{\prime}\left(0\right)\right)\gamma_{H}\left(\frac{1}{2}\right)\sum_{n=0}^{\infty}\left(\frac{\mu_{0}}{2}\right)^{n}\left(\frac{a_{1}/d_{1}}{a_{0}/d_{0}}\right)^{\#_{1}\left(\left[\mathfrak{z}\right]_{2^{n}}\right)},\textrm{ }\forall\mathfrak{z}\in\mathbb{Z}_{2}\label{eq:Chi_H non-trivial series formula, rho =00003D00003D 2} \end{equation} \end{cor} Proof: Write out (\ref{eq:Chi_H in terms of Little Psi_H for a 2-Hydra map}), then use (\ref{eq:Definition of Kappa_H}) and $M_{H}$'s explicit formula (\textbf{Proposition \ref{prop:Explicit Formulas for M_H}}) to write: \begin{align*} \left(\frac{\mu_{0}}{2}\right)^{n}\kappa_{H}\left(m\right) & =\left(\frac{\mu_{0}}{2}\right)^{n}\left(\frac{2}{\mu_{0}}\right)^{\lambda_{2}\left(m\right)}M_{H}\left(m\right)\\ & =\left(\frac{\mu_{0}}{2}\right)^{n}\left(\frac{2}{\mu_{0}}\right)^{\lambda_{2}\left(m\right)}\frac{\mu_{0}^{\#_{2:0}\left(m\right)}\mu_{1}^{\#_{1}\left(m\right)}}{2^{\lambda_{2}\left(m\right)}}\\ \left(\#_{2:0}\left(m\right)=\lambda_{2}\left(m\right)-\#_{1}\left(m\right)\right); & =\left(\frac{\mu_{0}}{2}\right)^{n}\left(\frac{\mu_{1}}{\mu_{0}}\right)^{\#_{1}\left(m\right)}\\ & =\left(\frac{\mu_{0}}{2}\right)^{n}\left(\frac{a_{1}/d_{1}}{a_{0}/d_{0}}\right)^{\#_{1}\left(m\right)} \end{align*} Q.E.D. \vphantom{} Now we move to the general case. \begin{thm}[\textbf{Quasi-Integrability of $\chi_{H}$ for a $p$-Hydra map}] \index{chi{H}@$\chi_{H}$!mathcal{F}-series@$\mathcal{F}$-series}\label{thm:F-series for an arbitrary 1D Chi_H}Let $H$ be an arbitrary contracting, non-singular, semi-basic $p$-Hydra map which fixes $0$, where $p\geq2$. Suppose additionally that $H_{j}\left(0\right)\neq0$ for any $j\in\left\{ 1,\ldots,p-1\right\} $. Then\footnote{I call the following identity for $\chi_{H}$ the \textbf{$\mathcal{F}$-series} of $\chi_{H}$.}: \begin{equation} \chi_{H}\left(\mathfrak{z}\right)\overset{\mathcal{F}_{p,q_{H}}}{=}\beta_{H}\left(0\right)\psi_{H}\left(\mathfrak{z}\right)+\Psi_{H}\left(\mathfrak{z}\right),\textrm{ }\forall\mathfrak{z}\in\mathbb{Z}_{p}\label{eq:Chi_H in terms of Little Psi_H and Big Psi_H for arbitrary rho} \end{equation} In particular, using \emph{(\ref{eq:Fourier Transform of Little Psi_H})}, this shows that $\chi_{H}$ is then quasi-integrable with respect to the standard $\left(p,q_{H}\right)$-adic quasi-integrability frame, with the function $\hat{\chi}_{H}:\hat{\mathbb{Z}}_{p}\rightarrow\overline{\mathbb{Q}}$ defined by: \index{chi{H}@$\chi_{H}$!Fourier transform} \begin{equation} \hat{\chi}_{H}\left(t\right)\overset{\textrm{def}}{=}\begin{cases} \begin{cases} 0 & \textrm{if }t=0\\ \left(\beta_{H}\left(0\right)v_{p}\left(t\right)+\gamma_{H}\left(\frac{t\left|t\right|_{p}}{p}\right)\right)\hat{A}_{H}\left(t\right) & \textrm{if }t\neq0 \end{cases} & \textrm{if }\alpha_{H}\left(0\right)=1\\ \frac{\beta_{H}\left(0\right)\hat{A}_{H}\left(t\right)}{1-\alpha_{H}\left(0\right)}+\begin{cases} 0 & \textrm{if }t=0\\ \gamma_{H}\left(\frac{t\left|t\right|_{p}}{p}\right)\hat{A}_{H}\left(t\right) & \textrm{if }t\neq0 \end{cases} & \textrm{if }\alpha_{H}\left(0\right)\neq1 \end{cases},\textrm{ }\forall t\in\hat{\mathbb{Z}}_{p}\label{eq:Fourier Transform of Chi_H for a contracting semi-basic rho-Hydra map} \end{equation} being a Fourier transform of $\chi_{H}$. \end{thm} Proof: We make the ansatz: \begin{equation} \chi_{H}\left(\mathfrak{z}\right)=A\psi_{H}\left(\mathfrak{z}\right)+B\Psi_{H}\left(\mathfrak{z}\right) \end{equation} for constants $A$ and $B$. Using the functional equations (\ref{eq:Little Psi_H functional equations}) and (\ref{eq:Big Psi_H functional equations}), we then have that: \begin{align*} \chi_{H}\left(p\mathfrak{z}+j\right) & =A\psi_{H}\left(p\mathfrak{z}+j\right)+B\Psi_{H}\left(p\mathfrak{z}+j\right)\\ & =A\frac{\mu_{j}}{p}\psi_{H}\left(\mathfrak{z}\right)+B\frac{\mu_{j}}{p}\Psi_{H}\left(\mathfrak{z}\right)+A+B\left(H_{j}\left(0\right)-\beta_{H}\left(0\right)\right) \end{align*} and: \[ \chi_{H}\left(p\mathfrak{z}+j\right)=\frac{\mu_{j}}{p}\chi_{H}\left(\mathfrak{z}\right)+H_{j}\left(0\right)=A\frac{\mu_{j}}{p}\psi_{H}\left(\mathfrak{z}\right)+B\frac{\mu_{j}}{p}\Psi_{H}\left(\mathfrak{z}\right)+H_{j}\left(0\right) \] Combining these two formulae for $\chi_{H}\left(p\mathfrak{z}+j\right)$ yields: \[ A\frac{\mu_{j}}{p}\psi_{H}\left(\mathfrak{z}\right)+B\frac{\mu_{j}}{p}\Psi_{H}\left(\mathfrak{z}\right)+A+BH_{j}\left(0\right)-B\beta_{H}\left(0\right)=A\frac{\mu_{j}}{p}\psi_{H}\left(\mathfrak{z}\right)+B\frac{\mu_{j}}{p}\Psi_{H}\left(\mathfrak{z}\right)+H_{j}\left(0\right) \] and hence: \begin{equation} A+\left(H_{j}\left(0\right)-\beta_{H}\left(0\right)\right)B=H_{j}\left(0\right),\textrm{ }\forall j\in\left\{ 0,\ldots,p-1\right\} \label{eq:Seemingly over-determined system} \end{equation} Note that this is a system of $j$ linear equations in \emph{two }unknowns ($A$ and $B$). Most such systems are usually overdetermined and admit no solution. Not here, however. Since $H$ fixes $0$, $H_{0}\left(0\right)=0$, and the $j=0$ equation becomes: \begin{equation} A-\beta_{H}\left(0\right)B=0 \end{equation} Letting $j\in\left\{ 1,\ldots,p-1\right\} $, we plug $A=\beta_{H}\left(0\right)B$ into (\ref{eq:Seemingly over-determined system}) to obtain: \begin{align*} \beta_{H}\left(0\right)B+\left(H_{j}\left(0\right)-\beta_{H}\left(0\right)\right)B & =H_{j}\left(0\right)\\ & \Updownarrow\\ H_{j}\left(0\right)B & =H_{j}\left(0\right)\\ \left(\textrm{if }H_{j}\left(0\right)\neq0\textrm{ for any }j\in\left\{ 1,\ldots,p-1\right\} \right); & \Updownarrow\\ B & =1 \end{align*} Hence, our condition on the $H_{j}\left(0\right)$s guarantees that (\ref{eq:Seemingly over-determined system}) has $A=\beta_{H}\left(0\right)$ and $B=1$ as its unique solution. Using (\ref{eq:Fourier Transform of Little Psi_H}) and (\ref{eq:Fourier Transform of Big Psi_H}) for the Fourier transforms of $\psi_{H}$ and $\Psi_{H}$ yields (\ref{eq:Chi_H in terms of Little Psi_H and Big Psi_H for arbitrary rho}). Q.E.D. \begin{cor} \label{cor:Chi_H,N twiddle explicit formula, arbitrary p, arbitrary alpha}Let $H$ be as given in \textbf{\emph{Theorem \ref{thm:F-series for an arbitrary 1D Chi_H}}}, with $p\geq2$. Then: \begin{align} \tilde{\chi}_{H,N}\left(\mathfrak{z}\right) & \overset{\overline{\mathbb{Q}}}{=}-\beta_{H}\left(0\right)N\left(\frac{\mu_{0}}{p}\right)^{N}\kappa_{H}\left(\left[\mathfrak{z}\right]_{p^{N}}\right)+\sum_{n=0}^{N-1}\left(\sum_{j=0}^{p-1}\beta_{H}\left(\frac{j}{p}\right)\varepsilon_{n}^{j}\left(\mathfrak{z}\right)\right)\left(\frac{\mu_{0}}{p}\right)^{n}\kappa_{H}\left(\left[\mathfrak{z}\right]_{p^{n}}\right)\label{eq:Explicit Formula for Chi_H,N twiddle for arbitrary rho and alpha equals 1} \end{align} if $\alpha_{H}\left(0\right)=1$, and: \begin{align} \tilde{\chi}_{H,N}\left(\mathfrak{z}\right) & \overset{\overline{\mathbb{Q}}}{=}\frac{\beta_{H}\left(0\right)}{1-\alpha_{H}\left(0\right)}\left(\frac{\mu_{0}}{p}\right)^{N}\kappa_{H}\left(\left[\mathfrak{z}\right]_{p^{N}}\right)+\sum_{n=0}^{N-1}\left(\sum_{j=0}^{p-1}\beta_{H}\left(\frac{j}{p}\right)\varepsilon_{n}^{j}\left(\mathfrak{z}\right)\right)\left(\frac{\mu_{0}}{p}\right)^{n}\kappa_{H}\left(\left[\mathfrak{z}\right]_{p^{n}}\right)\label{eq:Explicit Formula for Chi_H,N twiddle for arbitrary rho and alpha not equal to 1} \end{align} if $\alpha_{H}\left(0\right)\neq1$. \end{cor} Proof: Take (\ref{eq:Fourier Transform of Chi_H for a contracting semi-basic rho-Hydra map}) and use the formulas from \textbf{Lemmata \ref{lem:1D gamma formula}} and \textbf{\ref{lem:v_p A_H hat summation formulae}}. Q.E.D. \begin{cor}[\textbf{$\mathcal{F}$-series for $\chi_{H}$}] \label{cor:F-series for Chi_H, arbitrary p and alpha}Let\index{chi{H}@$\chi_{H}$!mathcal{F}-series@$\mathcal{F}$-series} $H$ be as given in \textbf{\emph{Theorem \ref{thm:F-series for an arbitrary 1D Chi_H}}}. Then: \begin{equation} \chi_{H}\left(\mathfrak{z}\right)\overset{\mathcal{F}_{p,q_{H}}}{=}\sum_{n=0}^{\infty}\left(\sum_{j=0}^{p-1}\beta_{H}\left(\frac{j}{p}\right)\left(\varepsilon_{n}\left(\mathfrak{z}\right)\right)^{j}\right)\left(\frac{\mu_{0}}{p}\right)^{n}\kappa_{H}\left(\left[\mathfrak{z}\right]_{p^{n}}\right)\label{eq:Chi_H non-trivial series formula, rho not equal to 2} \end{equation} \end{cor} Proof: Take the $\mathcal{F}_{p,q_{H}}$-limit of the formulae in \textbf{Corollary \ref{cor:Chi_H,N twiddle explicit formula, arbitrary p, arbitrary alpha}}. Q.E.D. \vphantom{} Finally, using the Wiener Tauberian Theorem for $\left(p,q\right)$-adic measures (\textbf{Theorem \ref{thm:pq WTT for measures}}), we can prove the third chief result of this dissertation: the \textbf{Tauberian Spectral Theorem }for $p$-Hydra maps. \begin{thm}[\textbf{Tauberian Spectral Theorem for $p$-Hydra Maps}] \index{Hydra map!periodic points}\index{Hydra map!Tauberian Spectral Theorem}\label{thm:Periodic Points using WTT}\index{Wiener!Tauberian Theorem!left(p,qright)-adic@$\left(p,q\right)$-adic}\index{Hydra map!divergent trajectories}Let $H$ be as given in \textbf{\emph{Theorem \ref{thm:F-series for an arbitrary 1D Chi_H}}}. Let $x\in\mathbb{Z}\backslash\left\{ 0\right\} $, and let: \begin{equation} \hat{\chi}_{H}\left(t\right)=\begin{cases} \begin{cases} 0 & \textrm{if }t=0\\ \left(\beta_{H}\left(0\right)v_{p}\left(t\right)+\gamma_{H}\left(\frac{t\left|t\right|_{p}}{p}\right)\right)\hat{A}_{H}\left(t\right) & \textrm{if }t\neq0 \end{cases} & \textrm{if }\alpha_{H}\left(0\right)=1\\ \frac{\beta_{H}\left(0\right)\hat{A}_{H}\left(t\right)}{1-\alpha_{H}\left(0\right)}+\begin{cases} 0 & \textrm{if }t=0\\ \gamma_{H}\left(\frac{t\left|t\right|_{p}}{p}\right)\hat{A}_{H}\left(t\right) & \textrm{if }t\neq0 \end{cases} & \textrm{if }\alpha_{H}\left(0\right)\neq1 \end{cases},\textrm{ }\forall t\in\hat{\mathbb{Z}}_{p} \end{equation} \vphantom{} I. If $x$ is a periodic point of $H$, then the translates of the function $\hat{\chi}_{H}\left(t\right)-x\mathbf{1}_{0}\left(t\right)$ are \emph{not }dense in $c_{0}\left(\hat{\mathbb{Z}}_{p},\mathbb{C}_{q_{H}}\right)$. \vphantom{} II. Suppose in addition that $H$ is integral\footnote{Propriety is defined alongside Hydra maps themselves on \pageref{def:p-Hydra map}; all the shortened $qx+1$ maps are integral.}, and that $\left|H_{j}\left(0\right)\right|_{q_{H}}=1$ for all $j\in\left\{ 1,\ldots,p-1\right\} $. If the translates of the function $\hat{\chi}_{H}\left(t\right)-x\mathbf{1}_{0}\left(t\right)$ are \emph{not }dense in $c_{0}\left(\hat{\mathbb{Z}}_{p},\mathbb{C}_{q_{H}}\right)$, then either $x$ is a periodic point of $H$ or $x$ belongs to a divergent trajectory of $H$. When $\alpha_{H}\left(0\right)=1$ and $p=2$, we can also work with: \begin{equation} \hat{\chi}_{H}\left(t\right)=\begin{cases} -\gamma_{H}\left(\frac{1}{2}\right) & \textrm{if }t=0\\ \beta_{H}\left(0\right)v_{2}\left(t\right)\hat{A}_{H}\left(t\right) & \textrm{else } \end{cases},\textrm{ }\forall t\in\hat{\mathbb{Z}}_{2} \end{equation} \end{thm} Proof: Let $H$ and $\hat{\chi}_{H}$ be as given. In particular, since $H$ is semi-basic, $\chi_{H}$ exists and the \index{Correspondence Principle} \textbf{Correspondence Principle }applies. Moreover, since $H$ is contracting and $H_{j}\left(0\right)\neq0$ for any $j\in\left\{ 1,\ldots,p-1\right\} $, $\chi_{H}$ is quasi-integrable with respect to the standard $\left(p,q\right)$-adic frame, and $\hat{\chi}_{H}$ is a Fourier transform of $\chi_{H}$. Note that the alternative formula for $\hat{\chi}_{H}$ when $p=2$ and $\alpha_{H}\left(0\right)=1$ follows from the fact that it and the first formula differ by the Fourier-Stieltjes transform of a degenerate measure, as was shown in \textbf{Corollary \ref{cor:Quasi-integrability of Chi_H for alpha equals 1}}. Now, let $x\in\mathbb{Z}\backslash\left\{ 0\right\} $. Since the constant function $x$ has $x\mathbf{1}_{0}\left(t\right)$ as its Fourier transform, the quasi-integrability of $\chi_{H}$ tells us that the difference $\chi_{H}\left(\mathfrak{z}\right)-x$ is quasi-integrable, and that $\hat{\chi}_{H}\left(t\right)-x\mathbf{1}_{0}\left(t\right)$ is then a Fourier transform of $\chi_{H}\left(\mathfrak{z}\right)-x$. So, $\hat{\chi}_{H}\left(t\right)-x\mathbf{1}_{0}\left(t\right)$ is then the Fourier-Stieltjes transform of a $\left(p,q\right)$-adic measure, and the Fourier series it generates converges in $\mathbb{C}_{q_{H}}$ (to $\chi_{H}\left(\mathfrak{z}\right)-x$) if and only if $\mathfrak{z}\in\mathbb{Z}_{p}^{\prime}$. As such, the \textbf{Wiener Tauberian Theorem for $\left(p,q\right)$-adic measures} (\textbf{Theorem \ref{thm:pq WTT for measures}}) guarantees that the translates of $\hat{\chi}_{H}\left(t\right)-x\mathbf{1}_{0}\left(t\right)$ will be dense in $c_{0}\left(\hat{\mathbb{Z}}_{p},\mathbb{C}_{q_{H}}\right)$ if and only if $\chi_{H}\left(\mathfrak{z}\right)-x\neq0$ for every $\mathfrak{z}\in\mathbb{Z}_{p}^{\prime}$. So: \vphantom{} I. Suppose $x$ is a periodic point of $H$. By the \textbf{Correspondence Principle} (specifically, \textbf{Corollary \ref{cor:CP v4}}), there exists a $\mathfrak{z}_{0}\in\mathbb{Q}\cap\mathbb{Z}_{p}^{\prime}$ so that $\chi_{H}\left(\mathfrak{z}_{0}\right)=x$. Then, the WTT tells us that the translates of $\hat{\chi}_{H}\left(t\right)-x\mathbf{1}_{0}\left(t\right)$ are \emph{not }dense in $c_{0}\left(\hat{\mathbb{Z}}_{p},\mathbb{C}_{q}\right)$. \vphantom{} II. Let $H$ be integral and let $\left|H_{j}\left(0\right)\right|_{q_{H}}=1$ for all $j\in\left\{ 1,\ldots,p-1\right\} $. Since $H$ is integral, this implies $H$ is proper (\textbf{Lemma \ref{lem:integrality lemma}} from page \pageref{lem:integrality lemma}). Next, suppose the translates of $\hat{\chi}_{H}\left(t\right)-x\mathbf{1}_{0}\left(t\right)$ are not dense in $c_{0}\left(\hat{\mathbb{Z}}_{p},\mathbb{C}_{q}\right)$. Then, there is a $\mathfrak{z}_{0}\in\mathbb{Z}_{p}^{\prime}$ so that $\chi_{H}\left(\mathfrak{z}_{0}\right)=x$. If the $p$-adic digits of $\mathfrak{z}_{0}$ are eventually periodic (i.e., if $\mathfrak{z}_{0}\in\mathbb{Q}\cap\mathbb{Z}_{p}^{\prime}$), since $H$ is proper, \textbf{Corollary \ref{cor:CP v4}} guarantees that $x$ is a periodic point of $H$. If the $p$-adic digits of $\mathfrak{z}_{0}$ are not eventually periodic ($\mathfrak{z}_{0}\in\mathbb{Z}_{p}\backslash\mathbb{Q}$), \textbf{Theorem \ref{thm:Divergent trajectories come from irrational z}}\textemdash applicable because of the hypotheses placed on $H$\textemdash then guarantees that $x$ belongs to a divergent trajectory of $H$. Q.E.D. \vphantom{} Finally, for the reader's benefit, here are the formulae associated to $\chi_{3}$\index{chi{3}@$\chi_{3}$!mathcal{F}-series@$\mathcal{F}$-series}. \index{chi{3}@$\chi_{3}$}We have its $\mathcal{F}$-series: \begin{equation} \chi_{3}\left(\mathfrak{z}\right)\overset{\mathcal{F}_{2,3}}{=}-\frac{1}{2}+\frac{1}{4}\sum_{k=0}^{\infty}\frac{3^{\#_{1}\left(\left[\mathfrak{z}\right]_{2^{k}}\right)}}{2^{k}},\textrm{ }\forall\mathfrak{z}\in\mathbb{Z}_{2}\label{eq:Explicit formula for Chi_3} \end{equation} Plugging $n\in\mathbb{N}_{0}$ in for $\mathfrak{z}$ and summing the resultant geometric series in $\mathbb{R}$ yields a formula for $\chi_{3}$ on the integers: \begin{equation} \chi_{3}\left(n\right)=-\frac{1}{2}+\frac{1}{4}\frac{3^{\#_{1}\left(n\right)}}{2^{\lambda_{2}\left(n\right)}}+\frac{1}{4}\sum_{k=0}^{\lambda_{2}\left(n\right)}\frac{3^{\#_{1}\left(\left[n\right]_{2^{k}}\right)}}{2^{k}},\textrm{ }\forall n\in\mathbb{N}_{0}\label{eq:Explicit Formula for Chi_3 on the integers} \end{equation} A nice choice of Fourier transform for $\chi_{3}$ is\index{chi{3}@$\chi_{3}$!Fourier transform}: \begin{equation} \hat{\chi}_{3}\left(t\right)=\begin{cases} -\frac{1}{2} & \textrm{if }t=0\\ \frac{1}{4}v_{2}\left(t\right)\hat{A}_{3}\left(t\right) & \textrm{if }t\neq0 \end{cases},\textrm{ }\forall t\in\hat{\mathbb{Z}}_{2}\label{eq:Formula for Chi_3 hat} \end{equation} Since it is the most natural case for comparison, here are corresponding formulae for $\chi_{5}$:\index{chi{5}@$\chi_{5}$} \begin{equation} \hat{\chi}_{5}\left(t\right)=\begin{cases} -\frac{1}{2} & \textrm{if }t=0\\ -\frac{1}{4}\hat{A}_{5}\left(t\right) & \textrm{if }t\neq0 \end{cases}\label{eq:Chi_5 Fourier transform} \end{equation} \index{chi{5}@$\chi_{5}$!Fourier transform}\index{chi{5}@$\chi_{5}$!mathcal{F}-series@$\mathcal{F}$-series} \begin{equation} \chi_{5}\left(\mathfrak{z}\right)\overset{\mathcal{F}_{2,5}}{=}-\frac{1}{4}+\frac{1}{8}\sum_{k=0}^{\infty}\frac{5^{\#_{1}\left(\left[\mathfrak{z}\right]_{2^{k}}\right)}}{2^{k}}\label{eq:Chi_5 F-series} \end{equation} \begin{equation} \chi_{5}\left(n\right)=-\frac{1}{4}+\frac{1}{4}\frac{5^{\#_{1}\left(n\right)}}{2^{\lambda_{2}\left(n\right)}}+\frac{1}{8}\sum_{k=0}^{\lambda_{2}\left(n\right)}\frac{5^{\#_{1}\left(\left[n\right]_{2^{k}}\right)}}{2^{k}},\textrm{ }\forall n\in\mathbb{N}_{0}\label{eq:Explicit Formula for Chi_5 on the integers} \end{equation} \index{chi{q}@$\chi_{q}$}\index{chi{q}@$\chi_{q}$!Fourier transform}\index{chi{q}@$\chi_{q}$!mathcal{F}-series@$\mathcal{F}$-series} More generally, for any odd prime $q\geq5$, letting $\chi_{q}$ be the numen of the Shortened $qx+1$ map, we have that: \begin{equation} \hat{\chi}_{q}\left(t\right)=\begin{cases} -\frac{1}{q-3} & \textrm{if }t=0\\ -\frac{2\hat{A}_{q}\left(t\right)}{\left(q-1\right)\left(q-3\right)} & \textrm{if }t\neq0 \end{cases},\textrm{ }\forall t\in\hat{\mathbb{Z}}_{2}\label{eq:Fourier transform of the numen of the shortened qx+1 map} \end{equation} with the $\mathcal{F}$-series: \begin{equation} \chi_{q}\left(\mathfrak{z}\right)\overset{\mathcal{F}_{2,q}}{=}-\frac{1}{q-1}+\frac{1}{2\left(q-1\right)}\sum_{k=0}^{\infty}\frac{q^{\#_{1}\left(\left[\mathfrak{z}\right]_{2^{k}}\right)}}{2^{k}},\textrm{ }\forall\mathfrak{z}\in\mathbb{Z}_{2}\label{eq:F-series for Chi_q} \end{equation} and: \begin{equation} \chi_{q}\left(n\right)\overset{\mathbb{Q}}{=}-\frac{1}{q-1}+\frac{1}{2\left(q-1\right)}\frac{q^{\#_{1}\left(n\right)}}{2^{\lambda_{2}\left(n\right)}}+\frac{1}{2\left(q-1\right)}\sum_{k=0}^{\lambda_{2}\left(n\right)}\frac{q^{\#_{1}\left(\left[n\right]_{2^{k}}\right)}}{2^{k}},\textrm{ }\forall n\in\mathbb{N}_{0}\label{eq:Chi_q on N_0} \end{equation} Even though (\ref{eq:Fourier transform of the numen of the shortened qx+1 map}) is not the correct formula for $\hat{\chi}_{q}$ when $q=3$, it seems impossible that it is merely coincidental that the right-hand side of (\ref{eq:Fourier transform of the numen of the shortened qx+1 map}) is singular when $q\in\left\{ 1,3\right\} $. When $q=1$, it is easy to show that the shortened $qx+1$ map iterates every positive integer to $1$. Meanwhile, when $q=3$, it is conjectured that every positive integer goes to $1$. This suggests that for a given family $\mathcal{H}$ of Hydra maps with quasi-integrable numina $\hat{\chi}_{H}$ for each $H\in\mathcal{H}$, we have something along the lines of ``a given map $T\in\mathcal{H}$ has finitely many orbit classes (possibly all of which contain cycles) in $\mathbb{Z}$ if and only if the map $H\mapsto\hat{\chi}_{H}$ is 'singular' at $T$.'' \newpage{} \section{\label{sec:4.3 Salmagundi}Salmagundi} FOR THIS SUBSECTION, UNLESS STATED OTHERWISE, $H$ IS ALSO ASSUMED TO BE INTEGRAL AND NON-SINGULAR. \vphantom{} The title of this section is a wonderful word which my dictionary informs me, means either ``a dish of chopped meat, anchovies, eggs, onions, and seasoning'', or ``a general mixture; a miscellaneous collection''. It is the latter meaning that we are going to invoke here. This section contains a general mixture of results on $\chi_{H}$. Subsection \ref{subsec:4.3.1 p-adic-Estimates} demonstrates how the $\mathcal{F}$-series formula for $\chi_{H}$ (\textbf{Corollaries \ref{cor:F-series for Chi_H for p equals 2}} and \textbf{\ref{cor:F-series for Chi_H, arbitrary p and alpha}}) can be used to obtain a crude lower bound on the $q_{H}$-adic absolute value of $\chi_{H}\left(\mathfrak{z}\right)-\lambda$. It would be interesting to see if (and, if, then \emph{how}) this approach to estimation could be refined. \ref{subsec:4.3.2 A-Wisp-of} takes us back to the content of Subsection \ref{subsec:3.3.6 L^1 Convergence}\textemdash $L^{1}$ integrability of $\left(p,q\right)$-adic functions\textemdash and shows that the real-valued function $\left|\chi_{H}\left(\mathfrak{z}\right)\right|_{q_{H}}$ will be integrable with respect to the real-valued Haar probability measure on $\mathbb{Z}_{p}$ whenever $\gcd\left(\mu_{j},q_{H}\right)>1$ for at least one $j\in\left\{ 0,\ldots,p-1\right\} $ (\textbf{Theorem \ref{thm:L^1 criterion for Chi_H}}). I also show that: \[ \lim_{N\rightarrow\infty}\int_{\mathbb{Z}_{p}}\left|\chi_{H,N}\left(\mathfrak{z}\right)-\tilde{\chi}_{H,N}\left(\mathfrak{z}\right)\right|_{q_{H}}d\mathfrak{z}\overset{\mathbb{R}}{=}0 \] whenever $H$ is semi-basic and contracting (\textbf{Theorem \ref{thm:L^1 convergence of Chi_H,N minus Chi_H,N twiddle}}). In Subsection \ref{subsec:4.3.3 Quick-Approach-of}, we revisit the circle of ideas around the \textbf{Square Root Lemma }(page \pageref{lem:square root lemma}). Finally, Subsection \ref{subsec:4.3.4 Archimedean-Estimates} establishes what I have termed the ``$L^{1}$-method'', a means of using geometric series universality to straddle the chasm between archimedean and non-archimedean topologies so as to obtain upper bounds on the ordinary, archimedean absolute value of $\chi_{H}\left(\mathfrak{z}\right)$ for appropriately chosen values of $\mathfrak{z}$ at which $\chi_{H}\left(\left[\mathfrak{z}\right]_{p^{N}}\right)$ converges to $\chi_{H}\left(\mathfrak{z}\right)$ in $\mathbb{Z}_{q_{H}}$ \emph{and }$\mathbb{R}$ as $N\rightarrow\infty$ (\textbf{Theorem \ref{thm:The L^1 method}}). I also sketch a sample application of this method to $\chi_{3}$. \subsection{\label{subsec:4.3.1 p-adic-Estimates}$p$-adic Estimates} While it is my belief that the most fruitful investigations into $\chi_{H}$ will come from studying $\hat{\chi}_{H}$ in the context of the $\left(p,q\right)$-adic Wiener Tauberian Theorem\textemdash assuming, of course, that a proper converse for the case of $\left(p,q\right)$-adic measures / quasi-integrable functions can be formulated and proven\textemdash the explicit series formulae from \textbf{Corollaries \ref{cor:F-series for Chi_H for p equals 2}} and \textbf{\ref{cor:F-series for Chi_H, arbitrary p and alpha}} may be of use in their own right. With sufficient knowledge of the $p$-adic digits of a given $\mathfrak{z}$\textemdash equivalently, with sufficient knowledge on the pattern of applications of branches of $H$ needed to ensure that a trajectory cycles\textemdash one can use the explicit series formulae to obtain $\left(p,q\right)$-adic estimates for $\chi_{H}$ which can be used to rule out periodic points. The more we know about $\mathfrak{z}$, the more refined the estimates can be. In that respect, even though the estimates detailed below are relatively simple, the method may be of use, so I might as well give the details. First, two notations to make our life easier: \begin{defn}[$u_{p}\left(\mathfrak{z}\right)$ \textbf{and} $K_{H}$] \ \vphantom{} I. We write \nomenclature{$u_{p}\left(\mathfrak{z}\right)$}{$\mathfrak{z}\left|\mathfrak{z}\right|_{p}$ }$u_{p}:\mathbb{Z}_{p}\rightarrow\mathbb{Z}_{p}^{\times}\cup\left\{ 0\right\} $ to denote the function: \begin{equation} u_{p}\left(\mathfrak{z}\right)\overset{\textrm{def}}{=}\mathfrak{z}\left|\mathfrak{z}\right|_{p}\label{eq:Definition of u_p} \end{equation} \vphantom{} II. Define:\nomenclature{$K_{H}$}{ } \begin{equation} K_{H}\overset{\textrm{def}}{=}\max_{j\in\left\{ 1,\ldots,p-1\right\} }\left|\kappa_{H}\left(j\right)\right|_{q_{H}}=\max_{j\in\left\{ 1,\ldots,p-1\right\} }\left|\frac{\mu_{j}}{\mu_{0}}\right|_{q_{H}}\label{eq:Definition of K_H} \end{equation} Note that since $H$ is semi-basic, $\mu_{j}/\mu_{0}$ is a $q_{H}$-adic integer with $q_{H}$-adic absolute value $\leq1/q_{H}$. \end{defn} \begin{prop} We have: \begin{equation} \left|\kappa_{H}\left(n\right)\right|_{q_{H}}\leq K_{H}\leq\frac{1}{q_{H}}<1,\textrm{ }\forall n\in\mathbb{N}_{1}\label{eq:Kappa_H K_H bound} \end{equation} \end{prop} Proof: Use (\ref{eq:Kappa_H is rho-adically distributed}). Q.E.D. \vphantom{} Using the explicit formulae from \textbf{Proposition \ref{prop:formula for functions with p-adic structure}} we can now establish the promised estimates. There isn't much thought behind these estimates; it's mostly a matter of computation. So, to keep things from getting too unwieldy, I think it appropriate to show one concrete example\textemdash say, for $\chi_{3}$\textemdash before diving into the details. \begin{example} Using \textbf{Corollary \ref{cor:F-series for Chi_H for p equals 2}}, for the case of $\chi_{3}$, we have that: \begin{equation} \tilde{\chi}_{3,N}\left(\mathfrak{z}\right)\overset{\overline{\mathbb{Q}}}{=}-\frac{1}{2}+\frac{N}{4}\frac{3^{\#_{1}\left(\left[\mathfrak{z}\right]_{2^{N}}\right)}}{2^{N}}+\frac{1}{4}\sum_{n=0}^{N-1}\frac{3^{\#_{1}\left(\left[\mathfrak{z}\right]_{2^{n}}\right)}}{2^{n}},\textrm{ }\forall N\in\mathbb{N}_{1},\textrm{ }\forall\mathfrak{z}\in\mathbb{Z}_{2}\label{eq:Explicit formula for Chi_3,N twiddle} \end{equation} Now, observe that $\left[\mathfrak{z}\right]_{2^{n}}=0$ for all $n\leq v_{2}\left(\mathfrak{z}\right)$; ex. $\left[8\right]_{2}=\left[8\right]_{4}=\left[8\right]_{8}=0$. As such, $n=v_{2}\left(\mathfrak{z}\right)+1$ is the smallest non-negative integer for which $\left[\mathfrak{z}\right]_{2^{n}}>0$, and hence, is the smallest integer for which $\kappa_{3}\left(\left[\mathfrak{z}\right]_{2^{n}}\right)=3^{\#_{1}\left(\left[\mathfrak{z}\right]_{2^{n}}\right)}$ is $>1$. Indeed, $\kappa_{3}\left(\left[\mathfrak{z}\right]_{2^{v_{2}\left(\mathfrak{z}\right)+1}}\right)=3$ for all $\mathfrak{z}\in\mathbb{Z}_{2}\backslash\left\{ 0\right\} $. So, $3^{\#_{1}\left(\left[\mathfrak{z}\right]_{2^{n}}\right)}$ will en be a multiple of $3$ for all $n\geq v_{2}\left(\mathfrak{z}\right)+1$. This tells us we can pull out terms from the $n$-sum which have a $3$-adic magnitude of $1$: \begin{align*} \sum_{n=0}^{N-1}\frac{3^{\#_{1}\left(\left[\mathfrak{z}\right]_{2^{n}}\right)}}{2^{n}} & =\sum_{n=0}^{v_{2}\left(\mathfrak{z}\right)}\frac{1}{2^{n}}+\sum_{n=v_{2}\left(\mathfrak{z}\right)+1}^{N-1}\frac{3^{\#_{1}\left(\left[\mathfrak{z}\right]_{2^{n}}\right)}}{2^{n}}\\ \left(3^{\#_{1}\left(\left[\mathfrak{z}\right]_{2^{n}}\right)}=1\textrm{ }\forall n\leq v_{2}\left(\mathfrak{z}\right)\right); & =\sum_{n=0}^{v_{2}\left(\mathfrak{z}\right)}\frac{1}{2^{n}}+\sum_{n=v_{2}\left(\mathfrak{z}\right)+1}^{N-1}\frac{3^{\#_{1}\left(\left[\mathfrak{z}\right]_{2^{n}}\right)}}{2^{n}}\\ & =\frac{1-\frac{1}{2^{v_{2}\left(\mathfrak{z}\right)+1}}}{1-\frac{1}{2}}+\sum_{n=v_{2}\left(\mathfrak{z}\right)+1}^{N-1}\frac{3^{\#_{1}\left(\left[\mathfrak{z}\right]_{2^{n}}\right)}}{2^{n}}\\ \left(\frac{1}{2^{v_{2}\left(\mathfrak{z}\right)}}=\left|\mathfrak{z}\right|_{2}\right); & =2-\left|\mathfrak{z}\right|_{2}+\sum_{n=v_{2}\left(\mathfrak{z}\right)+1}^{N-1}\frac{3^{\#_{1}\left(\left[\mathfrak{z}\right]_{2^{n}}\right)}}{2^{n}} \end{align*} which, for any $\mathfrak{z}\in\mathbb{Z}_{2}\backslash\left\{ 0\right\} $, holds for all $N-1\geq v_{2}\left(\mathfrak{z}\right)+1$. So, (\ref{eq:Explicit formula for Chi_3,N twiddle}) becomes: \begin{equation} \tilde{\chi}_{3,N}\left(\mathfrak{z}\right)\overset{\overline{\mathbb{Q}}}{=}\frac{N}{4}\frac{3^{\#_{1}\left(\left[\mathfrak{z}\right]_{2^{N}}\right)}}{2^{N}}-\frac{1}{4}\left|\mathfrak{z}\right|_{2}+\frac{1}{4}\sum_{n=v_{2}\left(\mathfrak{z}\right)+1}^{N-1}\frac{3^{\#_{1}\left(\left[\mathfrak{z}\right]_{2^{n}}\right)}}{2^{n}}\label{eq:Explicit formula for Chi_3,N twiddle with z pulled out} \end{equation} In terms of strings and composition sequence of trajectories, if $\mathfrak{z}\in\mathbb{Z}_{2}^{\prime}\cap\mathbb{Q}$ made $\chi_{3}\left(\mathfrak{z}\right)$ into a periodic point (belonging to the cycle $\Omega$) of the shortened Collatz map $T_{3}$ , then $v_{2}\left(\mathfrak{z}\right)$ would be the number of times we had to divide by $2$ to get from the penultimate element of $\Omega$ and return to $\chi_{3}\left(\mathfrak{z}\right)$. In this light, (\ref{eq:Explicit formula for Chi_3,N twiddle with z pulled out}) shows how we can obtain greater detail by using our knowledge of the number of times we divide by $2$ on the final ``fall'' in a given cycle. Channeling the Correspondence Principle, let $\mathfrak{z}\in\mathbb{Q}\cap\mathbb{Z}_{2}^{\prime}$. Then, (\ref{eq:Explicit formula for Chi_3,N twiddle with z pulled out}) converges in $\mathbb{Z}_{3}$ to $\chi_{3}\left(\mathfrak{z}\right)$ as $N\rightarrow\infty$, and we obtain: \begin{equation} \chi_{3}\left(\mathfrak{z}\right)\overset{\mathbb{Z}_{3}}{=}-\frac{1}{4}\left|\mathfrak{z}\right|_{2}+\frac{1}{4}\sum_{n=v_{2}\left(\mathfrak{z}\right)+1}^{\infty}\frac{3^{\#_{1}\left(\left[\mathfrak{z}\right]_{2^{n}}\right)}}{2^{n}},\textrm{ }\forall\mathfrak{z}\in\mathbb{Z}_{2}^{\prime} \end{equation} If we knew that the first $1$ in the sequence of $\mathfrak{z}$'s $2$-adic digits was followed by $m$ consecutive zeroes, we would then be able to pull out all of the terms of the $n$-sum of $3$-adic magnitude $1/3$, and so on and so forth\textemdash but doing already takes us beyond the simple estimate I had in mind. \emph{That }(admittedly crude) estimate consists of taking the above, subtracting $\lambda\in\mathbb{Z}$, and then applying the $3$-adic absolute value: \[ \left|\chi_{3}\left(\mathfrak{z}\right)-\lambda\right|_{3}\overset{\mathbb{Z}_{3}}{=}\left|\lambda+\frac{1}{4}\left|\mathfrak{z}\right|_{2}-\frac{1}{4}\underbrace{\sum_{n=v_{2}\left(\mathfrak{z}\right)+1}^{\infty}\frac{3^{\#_{1}\left(\left[\mathfrak{z}\right]_{2^{n}}\right)}}{2^{n}}}_{\textrm{has }\left|\cdot\right|_{3}\leq1/3}\right|_{3} \] By the $3$-adic ultrametric inequality, observe that $\left|\lambda+\frac{1}{4}\left|\mathfrak{z}\right|_{2}\right|_{3}=1$ then forces: \[ \left|\chi_{3}\left(\mathfrak{z}\right)-\lambda\right|_{3}=\left|\lambda+\frac{1}{4}\left|\mathfrak{z}\right|_{2}\right|_{3}=1 \] So, we have that $\chi_{3}\left(\mathfrak{z}\right)\neq\lambda$ for any $\lambda\in\mathbb{Z}$ and $\mathfrak{z}\in\mathbb{Z}_{2}^{\prime}$ such that $\left|\lambda+\frac{1}{4}\left|\mathfrak{z}\right|_{2}\right|_{3}=1$. For example, if $\left|\mathfrak{z}\right|_{2}=2^{-n}$, then $\left|2^{n+2}\lambda+1\right|_{3}=1$ implies $\chi_{3}\left(\mathfrak{z}\right)\neq\lambda$. Hence, $2^{n+2}\lambda+1\overset{3}{\equiv}0$ is necessary in order for $\lambda$ to be a non-zero periodic point of Collatz with $\chi_{3}\left(\mathfrak{z}\right)=\lambda$ where $\left|\mathfrak{z}\right|_{2}=2^{-n}$. \end{example} \vphantom{} The following Corollary of the explicit formulae from Section \ref{sec:4.2 Fourier-Transforms-=00003D000026} gives the above result for all of the kinds of Hydra maps we considered in that section. \begin{cor} \label{cor:Non-archimedean estimates of Chi_H}Let $\lambda\in\mathbb{Z}_{q_{H}}$ and $\mathfrak{z}\in\mathbb{Z}_{p}^{\prime}$.\index{chi{H}@$\chi_{H}$!non-archimedean estimates} Then: \begin{equation} \left|\chi_{H}\left(\mathfrak{z}\right)-\lambda\right|_{q_{H}}=\left|\chi_{H}\left(\left[u_{p}\left(\mathfrak{z}\right)\right]_{p}\right)\left|\mathfrak{z}\right|_{p}^{1-\log_{p}\mu_{0}}-\lambda\right|_{q_{H}}>0\label{eq:Crude Estimate} \end{equation} whenever: \begin{equation} \left|\chi_{H}\left(\left[u_{p}\left(\mathfrak{z}\right)\right]_{p}\right)\left|\mathfrak{z}\right|_{p}^{1-\log_{p}\mu_{0}}-\lambda\right|_{q_{H}}>K_{H}\sup_{n>v_{p}\left(\mathfrak{z}\right)}\left|\sum_{j=0}^{p-1}\beta_{H}\left(\frac{j}{p}\right)\varepsilon_{n}^{j}\left(\mathfrak{z}\right)\right|_{q_{H}}\label{eq:Crude Estimate Prerequisite} \end{equation} In particular, $\chi_{H}\left(\mathfrak{z}\right)\neq\lambda$ whenever \emph{(\ref{eq:Crude Estimate Prerequisite})} holds true. \end{cor} Proof: Fix $\mathfrak{z}\in\mathbb{Z}_{p}^{\prime}$ and recall that: \begin{equation} \kappa_{H}\left(\left[\mathfrak{z}\right]_{p^{n}}\right)=\kappa_{H}\left(0\right)=1,\textrm{ }\forall n\leq v_{p}\left(\mathfrak{z}\right) \end{equation} \begin{equation} \varepsilon_{n}\left(\mathfrak{z}\right)=e^{2\pi i\left\{ \frac{\mathfrak{z}}{p^{n+1}}\right\} _{p}}e^{-\frac{2\pi i}{p}\left\{ \frac{\mathfrak{z}}{p^{n}}\right\} _{p}}=1,\textrm{ }\forall n<v_{p}\left(\mathfrak{z}\right) \end{equation} Now, let $N-1\geq v_{p}\left(\mathfrak{z}\right)+1$. Then, there are two cases. \vphantom{} I. Suppose $\alpha_{H}\left(0\right)=1$. Then, (\ref{eq:Explicit Formula for Chi_H,N twiddle for arbitrary rho and alpha equals 1}) becomes: \begin{align*} \tilde{\chi}_{H,N}\left(\mathfrak{z}\right) & \overset{\overline{\mathbb{Q}}}{=}-\beta_{H}\left(0\right)N\left(\frac{\mu_{0}}{p}\right)^{N}\kappa_{H}\left(\left[\mathfrak{z}\right]_{p^{N}}\right)+\beta_{H}\left(0\right)\sum_{n=0}^{v_{p}\left(\mathfrak{z}\right)}\left(\frac{\mu_{0}}{p}\right)^{n}\kappa_{H}\left(0\right)\\ & +\sum_{n=0}^{v_{p}\left(\mathfrak{z}\right)-1}\left(\sum_{j=1}^{p-1}\beta_{H}\left(\frac{j}{p}\right)\left(1\right)^{j}\right)\left(\frac{\mu_{0}}{p}\right)^{n}\kappa_{H}\left(0\right)\\ & +\left(\sum_{j=1}^{p-1}\beta_{H}\left(\frac{j}{p}\right)\varepsilon_{v_{p}\left(\mathfrak{z}\right)}^{j}\left(\mathfrak{z}\right)\right)\left(\frac{\mu_{0}}{p}\right)^{v_{p}\left(\mathfrak{z}\right)}\kappa_{H}\left(0\right)\\ & +\beta_{H}\left(0\right)\sum_{n=v_{p}\left(\mathfrak{z}\right)+1}^{N-1}\left(\frac{\mu_{0}}{p}\right)^{n}\kappa_{H}\left(\left[\mathfrak{z}\right]_{p^{n}}\right)\\ & +\sum_{n=v_{p}\left(\mathfrak{z}\right)+1}^{N-1}\left(\sum_{j=1}^{p-1}\beta_{H}\left(\frac{j}{p}\right)\varepsilon_{n}^{j}\left(\mathfrak{z}\right)\right)\left(\frac{\mu_{0}}{p}\right)^{n}\kappa_{H}\left(\left[\mathfrak{z}\right]_{p^{n}}\right) \end{align*} Here: \begin{align*} \sum_{j=0}^{p-1}\beta_{H}\left(\frac{j}{p}\right) & =\sum_{j=0}^{p-1}\left(\frac{1}{p}\sum_{k=0}^{p-1}H_{k}\left(0\right)e^{-\frac{2\pi ijk}{p}}\right)\\ & =\sum_{k=0}^{p-1}H_{k}\left(0\right)\frac{1}{p}\sum_{j=0}^{p-1}e^{-\frac{2\pi ijk}{p}}\\ & =\sum_{k=0}^{p-1}H_{k}\left(0\right)\left[k\overset{p}{\equiv}0\right]\\ & =H_{0}\left(0\right)\\ & =0 \end{align*} So: \begin{equation} \sum_{j=1}^{p-1}\beta_{H}\left(\frac{j}{p}\right)=-\beta_{H}\left(0\right)+\underbrace{\sum_{j=0}^{p-1}\beta_{H}\left(\frac{j}{p}\right)}_{0}=-\beta_{H}\left(0\right)\label{eq:Sum of beta H of j/p for j in 1,...,p-1} \end{equation} On the other hand: \begin{align*} \sum_{j=0}^{p-1}\beta_{H}\left(\frac{j}{p}\right)\varepsilon_{v_{p}\left(\mathfrak{z}\right)}^{j}\left(\mathfrak{z}\right) & =\sum_{j=0}^{p-1}\left(\frac{1}{p}\sum_{k=0}^{p-1}H_{k}\left(0\right)e^{-\frac{2\pi ijk}{p}}\right)\left(e^{2\pi i\left\{ \frac{\mathfrak{z}}{p^{v_{p}\left(\mathfrak{z}\right)+1}}\right\} _{p}-\frac{2\pi i}{p}\left\{ \frac{\mathfrak{z}}{p^{v_{p}\left(\mathfrak{z}\right)}}\right\} _{p}}\right)^{j}\\ & =\sum_{j=0}^{p-1}\left(\frac{1}{p}\sum_{k=0}^{p-1}H_{k}\left(0\right)e^{-\frac{2\pi ijk}{p}}\right)e^{2\pi i\frac{j\left[\left|\mathfrak{z}\right|_{p}\mathfrak{z}\right]_{p}}{p}}\\ & =\frac{1}{p}\sum_{k=0}^{p-1}H_{k}\left(0\right)\sum_{j=0}^{p-1}e^{\frac{2\pi ij}{p}\left(\left[u_{p}\left(\mathfrak{z}\right)\right]_{p}-k\right)}\\ & =\sum_{k=0}^{p-1}H_{k}\left(0\right)\left[\left|\mathfrak{z}\right|_{p}\mathfrak{z}\overset{p}{\equiv}k\right]\\ & =H_{\left[u_{p}\left(\mathfrak{z}\right)\right]_{p}}\left(0\right) \end{align*} and so: \begin{equation} \sum_{j=1}^{p-1}\beta_{H}\left(\frac{j}{p}\right)\varepsilon_{v_{p}\left(\mathfrak{z}\right)}^{j}\left(\mathfrak{z}\right)=-\beta_{H}\left(0\right)+\sum_{j=0}^{p-1}\beta_{H}\left(\frac{j}{p}\right)\varepsilon_{v_{p}\left(\mathfrak{z}\right)}^{j}\left(\mathfrak{z}\right)=H_{\left[u_{p}\left(\mathfrak{z}\right)\right]_{p}}\left(0\right)-\beta_{H}\left(0\right)\label{eq:Sum of beta H of j/p times the epsilon for j in 1,...,p-1} \end{equation} With this, $\tilde{\chi}_{H,N}\left(\mathfrak{z}\right)$ becomes: \begin{align*} \tilde{\chi}_{H,N}\left(\mathfrak{z}\right) & \overset{\overline{\mathbb{Q}}}{=}H_{\left[u_{p}\left(\mathfrak{z}\right)\right]_{p}}\left(0\right)\left(\frac{\mu_{0}}{p}\right)^{v_{p}\left(\mathfrak{z}\right)}-\beta_{H}\left(0\right)N\left(\frac{\mu_{0}}{p}\right)^{N}\kappa_{H}\left(\left[\mathfrak{z}\right]_{p^{N}}\right)\\ & +\beta_{H}\left(0\right)\sum_{n=v_{p}\left(\mathfrak{z}\right)+1}^{N-1}\left(\frac{\mu_{0}}{p}\right)^{n}\kappa_{H}\left(\left[\mathfrak{z}\right]_{p^{n}}\right)\\ & +\sum_{n=v_{p}\left(\mathfrak{z}\right)+1}^{N-1}\left(\sum_{j=1}^{p-1}\beta_{H}\left(\frac{j}{p}\right)\varepsilon_{n}^{j}\left(\mathfrak{z}\right)\right)\left(\frac{\mu_{0}}{p}\right)^{n}\kappa_{H}\left(\left[\mathfrak{z}\right]_{p^{n}}\right) \end{align*} Letting $N\rightarrow\infty$, the above converges in $\mathbb{C}_{q_{H}}$ to: \begin{align*} \chi_{H}\left(\mathfrak{z}\right) & \overset{\mathbb{C}_{q_{H}}}{=}H_{\left[u_{p}\left(\mathfrak{z}\right)\right]_{p}}\left(0\right)\left(\frac{\mu_{0}}{p}\right)^{v_{p}\left(\mathfrak{z}\right)}+\beta_{H}\left(0\right)\sum_{n=v_{p}\left(\mathfrak{z}\right)+1}^{\infty}\left(\frac{\mu_{0}}{p}\right)^{n}\kappa_{H}\left(\left[\mathfrak{z}\right]_{p^{n}}\right)\\ & +\sum_{n=v_{p}\left(\mathfrak{z}\right)+1}^{\infty}\left(\sum_{j=1}^{p-1}\beta_{H}\left(\frac{j}{p}\right)\varepsilon_{n}^{j}\left(\mathfrak{z}\right)\right)\left(\frac{\mu_{0}}{p}\right)^{n}\kappa_{H}\left(\left[\mathfrak{z}\right]_{p^{n}}\right) \end{align*} The $n$-sum on the top line is the $j=0$th term of the sum on the bottom line. This leaves us with: \[ \chi_{H}\left(\mathfrak{z}\right)\overset{\mathbb{C}_{q_{H}}}{=}H_{\left[u_{p}\left(\mathfrak{z}\right)\right]_{p}}\left(0\right)\left(\frac{\mu_{0}}{p}\right)^{v_{p}\left(\mathfrak{z}\right)}+\sum_{n=v_{p}\left(\mathfrak{z}\right)+1}^{\infty}\left(\sum_{j=0}^{p-1}\beta_{H}\left(\frac{j}{p}\right)\varepsilon_{n}^{j}\left(\mathfrak{z}\right)\right)\left(\frac{\mu_{0}}{p}\right)^{n}\kappa_{H}\left(\left[\mathfrak{z}\right]_{p^{n}}\right) \] Because $\mathfrak{z}\in\mathbb{Z}_{p}^{\prime}$, observe that $\left|\kappa_{H}\left(\left[\mathfrak{z}\right]_{p^{n}}\right)\right|_{q_{H}}\leq K_{H}$ for all $n>v_{p}\left(\mathfrak{z}\right)$. As such, subtracting $\lambda\in\mathbb{Z}_{q_{H}}$ from both sides, we can apply the $q_{H}$-adic ultrametric inequality. This gives us the equality: \begin{equation} \left|\chi_{H}\left(\mathfrak{z}\right)-\lambda\right|_{q_{H}}=\left|H_{\left[u_{p}\left(\mathfrak{z}\right)\right]_{p}}\left(0\right)\left(\frac{\mu_{0}}{p}\right)^{v_{p}\left(\mathfrak{z}\right)}-\lambda\right|_{q_{H}} \end{equation} provided that the condition: \begin{equation} \left|H_{\left[u_{p}\left(\mathfrak{z}\right)\right]_{p}}\left(0\right)\left(\frac{\mu_{0}}{p}\right)^{v_{p}\left(\mathfrak{z}\right)}-\lambda\right|_{q_{H}}>K_{H}\sup_{n>v_{p}\left(\mathfrak{z}\right)}\left|\sum_{j=0}^{p-1}\beta_{H}\left(\frac{j}{p}\right)\varepsilon_{n}^{j}\left(\mathfrak{z}\right)\right|_{q_{H}} \end{equation} is satisfied. Finally, writing: \begin{equation} \left(\frac{\mu_{0}}{p}\right)^{v_{p}\left(\mathfrak{z}\right)}=\left(p^{-v_{p}\left(\mathfrak{z}\right)}\right)^{1-\log_{p}\mu_{0}}=\left|\mathfrak{z}\right|_{p}^{1-\log_{p}\mu_{0}} \end{equation} we obtain (I) by noting that: \begin{equation} H_{\left[u_{p}\left(\mathfrak{z}\right)\right]_{p}}\left(0\right)=\chi_{H}\left(\left[u_{p}\left(\mathfrak{z}\right)\right]_{p}\right) \end{equation} \vphantom{} II. Suppose $\alpha_{H}\left(0\right)\neq1$. Then (\ref{eq:Explicit Formula for Chi_H,N twiddle for arbitrary rho and alpha not equal to 1}) becomes: \begin{align*} \tilde{\chi}_{H,N}\left(\mathfrak{z}\right) & \overset{\overline{\mathbb{Q}}}{=}\frac{\beta_{H}\left(0\right)}{1-\alpha_{H}\left(0\right)}\left(\frac{\mu_{0}}{p}\right)^{N}\kappa_{H}\left(\left[\mathfrak{z}\right]_{p^{N}}\right)+\beta_{H}\left(0\right)\sum_{n=0}^{N-1}\left(\frac{\mu_{0}}{p}\right)^{n}\kappa_{H}\left(\left[\mathfrak{z}\right]_{p^{n}}\right)\\ & +\sum_{n=0}^{N-1}\left(\sum_{j=1}^{p-1}\beta_{H}\left(\frac{j}{p}\right)\varepsilon_{n}^{j}\left(\mathfrak{z}\right)\right)\left(\frac{\mu_{0}}{p}\right)^{n}\kappa_{H}\left(\left[\mathfrak{z}\right]_{p^{n}}\right) \end{align*} The only term in this equation that differs from the $\alpha_{H}\left(0\right)=1$ case is the initial term: \begin{equation} \frac{\beta_{H}\left(0\right)}{1-\alpha_{H}\left(0\right)}\left(\frac{\mu_{0}}{p}\right)^{N}\kappa_{H}\left(\left[\mathfrak{z}\right]_{p^{N}}\right) \end{equation} The corresponding term of the $\alpha_{H}\left(0\right)=1$ case was: \begin{equation} -\beta_{H}\left(0\right)N\left(\frac{\mu_{0}}{p}\right)^{N}\kappa_{H}\left(\left[\mathfrak{z}\right]_{p^{N}}\right) \end{equation} Since both of these terms converge $q_{H}$-adically to zero as $N\rightarrow\infty$ whenever $\mathfrak{z}\in\mathbb{Z}_{p}^{\prime}$, all of the computations done for the $\alpha_{H}\left(0\right)=1$ case will apply here\textemdash the only term that differs between the two cases ends up vanishing as $N\rightarrow\infty$. Q.E.D. \subsection{\label{subsec:4.3.2 A-Wisp-of}A Wisp of $L^{1}$\index{non-archimedean!$L^{1}$ theory}} One of my goals for future research is to pursue $\left(p,q\right)$-adic analysis in the setting of $L_{\mathbb{R}}^{1}$. To that end, in this subsection, a simple sufficient condition is establish for determining when $\chi_{H}\in L_{\mathbb{R}}^{1}\left(\mathbb{Z}_{p},\mathbb{C}_{q_{H}}\right)$. We begin by computing the van der Put series for $\chi_{H}$. When $p=2$, this is easily done. For $p\geq3$, we will have to settle on a mutated sort of van der Put series, seeing as the van der Put coefficients of $\chi_{H}$ are not as well-behaved for $p\geq3$ as they are for $p=2$. \begin{prop}[\textbf{``van der Put'' series for $\chi_{H}$}] \index{chi{H}@$\chi_{H}$!van der Put series}\label{prop:van der Put series for Chi_H}When $p=2$: \begin{equation} \chi_{H}\left(\mathfrak{z}\right)\overset{\mathbb{Z}_{q_{H}}}{=}\frac{b_{1}}{a_{1}}\sum_{n=1}^{\infty}M_{H}\left(n\right)\left[\mathfrak{z}\overset{2^{\lambda_{2}\left(n\right)}}{\equiv}n\right],\textrm{ }\forall\mathfrak{z}\in\mathbb{Z}_{2}\label{eq:Simplified vdP series for Chi_H when rho is 2} \end{equation} More generally, for any $p$: \begin{equation} \chi_{H}\left(\mathfrak{z}\right)\overset{\mathbb{Z}_{q_{H}}}{=}\chi_{H}\left(\left[\mathfrak{z}\right]_{p}\right)+\sum_{n=1}^{\infty}\left(\sum_{j=1}^{p-1}H_{j}\left(0\right)\left[u_{p}\left(\mathfrak{z}-n\right)\overset{p}{\equiv}j\right]\right)M_{H}\left(n\right)\left[\mathfrak{z}\overset{p^{\lambda_{p}\left(n\right)}}{\equiv}n\right],\textrm{ }\forall\mathfrak{z}\in\mathbb{Z}_{p}\label{eq:Quasi vdP series for Chi__H} \end{equation} \end{prop} Proof: Let $n\in\mathbb{N}_{1}$ be arbitrary, and let $\mathbf{j}=\left(j_{1},\ldots,j_{\lambda_{p}\left(n\right)}\right)$ be the shortest string in $\textrm{String}\left(p\right)$ representing $n$. Since $n\neq0$, observe that $j_{\lambda_{p}\left(n\right)}\in\left\{ 1,\ldots,p-1\right\} $. Using \textbf{Proposition \ref{prop:Explicit formula for Chi_H of bold j} }(page \pageref{prop:Explicit formula for Chi_H of bold j}), we have that: \begin{equation} \chi_{H}\left(n\right)=\sum_{m=1}^{\lambda_{p}\left(n\right)}\frac{b_{j_{m}}}{a_{j_{m}}}\left(\prod_{k=1}^{m}\mu_{j_{k}}\right)p^{-m} \end{equation} Note that $n_{-}$ is the integer represented by the string $\mathbf{j}^{\prime}=\left(j_{1},\ldots,j_{\lambda_{p}\left(n\right)-1}\right)$. Nevertheless, since $H\left(0\right)=0$, $\chi_{H}\left(\mathbf{j}^{\prime}\right)=\chi_{H}\left(n_{-}\right)$, since removing zeroes at the right end of a string does not affect the value of $\chi_{H}$. Consequently: \begin{equation} \chi_{H}\left(n_{-}\right)=\sum_{m=1}^{\lambda_{p}\left(n\right)-1}\frac{b_{j_{m}}}{a_{j_{m}}}\left(\prod_{k=1}^{m}\mu_{j_{k}}\right)p^{-m} \end{equation} Thus: \begin{align*} c_{n}\left(\chi_{H}\right) & =\chi_{H}\left(n\right)-\chi_{H}\left(n_{-}\right)=\frac{b_{j_{\lambda_{p}\left(n\right)}}}{a_{j_{\lambda_{p}\left(n\right)}}}\left(\prod_{k=1}^{\lambda_{p}\left(n\right)}\mu_{j_{k}}\right)p^{-\lambda_{p}\left(n\right)}=\frac{b_{j_{\lambda_{p}\left(n\right)}}}{a_{j_{\lambda_{p}\left(n\right)}}}M_{H}\left(n\right) \end{align*} which tends to $0$ $q_{H}$-adically as the number of non-zero $p$-adic digits in $n$ tends to $\infty$, which gives another proof of the rising-continuity of $\chi_{H}$; our first proof of this fact was implicit in \textbf{Lemma \ref{lem:Unique rising continuation and p-adic functional equation of Chi_H}}. Since $\chi_{H}$ is rising-continuous, it is represented everywhere by its van der Put series: \begin{equation} \chi_{H}\left(\mathfrak{z}\right)\overset{\mathbb{Z}_{q_{H}}}{=}\sum_{n=1}^{\infty}\frac{b_{j_{\lambda_{p}\left(n\right)}}}{a_{j_{\lambda_{p}\left(n\right)}}}M_{H}\left(n\right)\left[\mathfrak{z}\overset{p^{\lambda_{p}\left(n\right)}}{\equiv}n\right],\textrm{ }\forall\mathfrak{z}\in\mathbb{Z}_{p}\label{eq:Unsimplified van der Put series for Chi_H} \end{equation} where the $n=0$ term is $0$ because $c_{0}\left(\chi_{H}\right)=\chi_{H}\left(0\right)=0$. When $p=2$, note that $j_{\lambda_{p}\left(n\right)}=1$ for all $n\geq1$; that is to say, the right-most $2$-adic digit of every non-zero integer is $1$. This allows us to write: \begin{equation} \chi_{H}\left(\mathfrak{z}\right)\overset{\mathbb{Z}_{q_{H}}}{=}\frac{b_{1}}{a_{1}}\sum_{n=1}^{\infty}M_{H}\left(n\right)\left[\mathfrak{z}\overset{2^{\lambda_{2}\left(n\right)}}{\equiv}n\right],\textrm{ }\forall\mathfrak{z}\in\mathbb{Z}_{2} \end{equation} When $p\geq3$, we use the formal identity: \begin{equation} \sum_{n=1}^{p^{N}-1}f\left(n\right)=\sum_{n=1}^{p^{N-1}-1}f\left(n\right)+\sum_{\ell=1}^{p-1}\sum_{n=1}^{p^{N-1}-1}f\left(n+\ell p^{N-1}\right) \end{equation} When applying this to $\chi_{H}$'s van der Put series, observe that $j_{\lambda_{p}\left(n+\ell p^{N-1}\right)}$, the right-most $p$-adic digit of $n+\ell p^{N-1}$, is then equal to $\ell$. Also: \begin{equation} M_{H}\left(n+\ell p^{N-1}\right)=\frac{\mu_{\ell}}{p}M_{H}\left(n\right) \end{equation} and: \begin{equation} \left[\mathfrak{z}\overset{p^{\lambda_{p}\left(n+\ell p^{N-1}\right)}}{\equiv}n+\ell p^{N-1}\right]=\left[\mathfrak{z}\overset{p^{N}}{\equiv}n+\ell p^{N-1}\right] \end{equation} These hold for all $n\in\left\{ 1,\ldots,p^{N-1}-1\right\} $ and all $\ell\in\left\{ 1,\ldots,p-1\right\} $. As such: \begin{align*} \sum_{n=1}^{p^{N}-1}\frac{b_{j_{\lambda_{p}\left(n\right)}}}{a_{j_{\lambda_{p}\left(n\right)}}}M_{H}\left(n\right)\left[\mathfrak{z}\overset{p^{\lambda_{p}\left(n\right)}}{\equiv}n\right] & =\sum_{n=1}^{p^{N-1}-1}\frac{b_{j_{\lambda_{p}\left(n\right)}}}{a_{j_{\lambda_{p}\left(n\right)}}}M_{H}\left(n\right)\left[\mathfrak{z}\overset{p^{\lambda_{p}\left(n\right)}}{\equiv}n\right]\\ & +\sum_{\ell=1}^{p-1}\sum_{n=1}^{p^{N-1}-1}\frac{b_{\ell}}{a_{\ell}}\frac{\mu_{\ell}}{p}M_{H}\left(n\right)\left[\mathfrak{z}\overset{p^{N}}{\equiv}n+\ell p^{N-1}\right]\\ & =\sum_{n=1}^{p^{N-1}-1}\frac{b_{j_{\lambda_{p}\left(n\right)}}}{a_{j_{\lambda_{p}\left(n\right)}}}M_{H}\left(n\right)\left[\mathfrak{z}\overset{p^{\lambda_{p}\left(n\right)}}{\equiv}n\right]\\ & +\sum_{\ell=1}^{p-1}\frac{b_{\ell}}{d_{\ell}}\sum_{n=1}^{p^{N-1}-1}M_{H}\left(n\right)\left[\mathfrak{z}\overset{p^{N}}{\equiv}n+\ell p^{N-1}\right] \end{align*} Noting that: \begin{equation} \sum_{n=1}^{p^{N}-1}\frac{b_{j_{\lambda_{p}\left(n\right)}}}{a_{j_{\lambda_{p}\left(n\right)}}}M_{H}\left(n\right)\left[\mathfrak{z}\overset{p^{\lambda_{p}\left(n\right)}}{\equiv}n\right]=S_{p:N}\left\{ \chi_{H}\right\} \left(\mathfrak{z}\right) \end{equation} we can simplify the previous equation to: \[ S_{p:N}\left\{ \chi_{H}\right\} \left(\mathfrak{z}\right)=S_{p:N-1}\left\{ \chi_{H}\right\} \left(\mathfrak{z}\right)+\sum_{\ell=1}^{p-1}\frac{b_{\ell}}{d_{\ell}}\sum_{n=1}^{p^{N-1}-1}M_{H}\left(n\right)\left[\mathfrak{z}\overset{p^{N}}{\equiv}n+\ell p^{N-1}\right] \] Next, we apply the truncated van der Put identity (\textbf{Proposition \ref{prop:vdP identity}}) to note that: \begin{equation} S_{p:N}\left\{ \chi_{H}\right\} \left(\mathfrak{z}\right)-S_{p:N-1}\left\{ \chi_{H}\right\} \left(\mathfrak{z}\right)=\chi_{H}\left(\left[\mathfrak{z}\right]_{p^{N}}\right)-\chi_{H}\left(\left[\mathfrak{z}\right]_{p^{N-1}}\right) \end{equation} So: \begin{align*} \chi_{H}\left(\left[\mathfrak{z}\right]_{p^{K}}\right)-\chi_{H}\left(\left[\mathfrak{z}\right]_{p}\right) & =\sum_{N=2}^{K}\left(\chi_{H}\left(\left[\mathfrak{z}\right]_{p^{N}}\right)-\chi_{H}\left(\left[\mathfrak{z}\right]_{p^{N-1}}\right)\right)\\ & =\sum_{N=2}^{K}\sum_{\ell=1}^{p-1}\frac{b_{\ell}}{d_{\ell}}\sum_{n=1}^{p^{N-1}-1}M_{H}\left(n\right)\left[\mathfrak{z}\overset{p^{N}}{\equiv}n+\ell p^{N-1}\right]\\ & =\sum_{\ell=1}^{p-1}\frac{b_{\ell}}{d_{\ell}}\sum_{N=1}^{K-1}\sum_{n=1}^{p^{N}-1}M_{H}\left(n\right)\left[\mathfrak{z}\overset{p^{N+1}}{\equiv}n+\ell p^{N}\right]\\ & =\sum_{\ell=1}^{p-1}\frac{b_{\ell}}{d_{\ell}}\sum_{n=1}^{p^{K-1}-1}\sum_{N=\lambda_{p}\left(n\right)}^{K-1}M_{H}\left(n\right)\left[\mathfrak{z}\overset{p^{N+1}}{\equiv}n+\ell p^{N}\right]\\ \left(H_{\ell}\left(0\right)=\frac{b_{\ell}}{d_{\ell}}\right); & =\sum_{\ell=1}^{p-1}H_{\ell}\left(0\right)\sum_{n=1}^{p^{K-1}-1}M_{H}\left(n\right)\sum_{N=\lambda_{p}\left(n\right)+1}^{K}\left[\mathfrak{z}\overset{p^{N}}{\equiv}n+\ell p^{N-1}\right] \end{align*} Letting $K\rightarrow\infty$ yields: \begin{equation} \chi_{H}\left(\mathfrak{z}\right)-\chi_{H}\left(\left[\mathfrak{z}\right]_{p}\right)\overset{\mathbb{Z}_{q_{H}}}{=}\sum_{j=1}^{p-1}H_{j}\left(0\right)\sum_{n=1}^{\infty}M_{H}\left(n\right)\sum_{N=\lambda_{p}\left(n\right)+1}^{\infty}\left[\mathfrak{z}\overset{p^{N}}{\equiv}n+jp^{N-1}\right]\label{eq:Nearly finished with van-der-put computation for Chi_H} \end{equation} \[ \chi_{H}\left(\mathfrak{z}\right)-\chi_{H}\left(\left[\mathfrak{z}\right]_{p}\right)\overset{\mathbb{Z}_{q_{H}}}{=}\sum_{j=1}^{p-1}H_{j}\left(0\right)\sum_{n=1}^{\infty}M_{H}\left(n\right)\sum_{N=\lambda_{p}\left(n\right)+1}^{\infty}\left[\mathfrak{z}\overset{p^{N}}{\equiv}n+jp^{N-1}\right] \] Now, fix $n$ and $j$. Then: \[ \sum_{N=\lambda_{p}\left(n\right)+1}^{\infty}\left[\mathfrak{z}\overset{p^{N}}{\equiv}n+jp^{N-1}\right]=\sum_{N=\lambda_{p}\left(n\right)+1}^{\infty}\left[\mathfrak{z}\overset{p^{N}}{\equiv}n+jp^{N-1}\right]\left[\mathfrak{z}\overset{p^{N-1}}{\equiv}n\right] \] Since $j$ is co-prime to $p$, observe that the congruence $\mathfrak{z}\overset{p^{N}}{\equiv}n+jp^{N-1}$ is equivalent to the following pair of conditions: i. $\mathfrak{z}\overset{p^{N-1}}{\equiv}n$; ii. $v_{p}\left(\frac{\mathfrak{z}-n}{p^{N}}-\frac{j}{p}\right)\geq0$. So: \begin{equation} \left[\mathfrak{z}\overset{p^{N}}{\equiv}n+jp^{N-1}\right]=\left[\mathfrak{z}\overset{p^{N-1}}{\equiv}n\right]\left[v_{p}\left(\frac{\mathfrak{z}-n}{p^{N}}-\frac{j}{p}\right)\geq0\right] \end{equation} Here: \begin{align*} v_{p}\left(\frac{\mathfrak{z}-n}{p^{N}}-\frac{j}{p}\right) & \geq0\\ \left(1+v_{p}\left(\mathfrak{y}\right)=v_{p}\left(p\mathfrak{y}\right)\right); & \Updownarrow\\ v_{p}\left(\frac{\mathfrak{z}-n}{p^{N-1}}-j\right) & \geq1\\ & \Updownarrow\\ \left|\frac{\mathfrak{z}-n}{p^{N-1}}-j\right|_{p} & \leq\frac{1}{p} \end{align*} Because $\left|j\right|_{p}=1$, observe that if $\left|\frac{\mathfrak{z}-n}{p^{N-1}}\right|_{p}<1$, we can then use $p$-adic ultrametric inequality to write: \begin{equation} \frac{1}{p}\geq\left|\frac{\mathfrak{z}-n}{p^{N-1}}-j\right|_{p}=\max\left\{ \left|\frac{\mathfrak{z}-n}{p^{N-1}}\right|_{p},\left|j\right|_{p}\right\} =1 \end{equation} Of course, this equality is impossible. So, if we have $\mathfrak{z}\overset{p^{N}}{\equiv}n+jp^{N-1}$, it \emph{must} be that $\left|\frac{\mathfrak{z}-n}{p^{N-1}}\right|_{p}=1$. This latter condition occurs if and only if $N=v_{p}\left(\mathfrak{z}-n\right)+1$. Putting it all together, (\ref{eq:Nearly finished with van-der-put computation for Chi_H}) becomes: \begin{align*} \chi_{H}\left(\mathfrak{z}\right)-\chi_{H}\left(\left[\mathfrak{z}\right]_{p}\right) & \overset{\mathbb{Z}_{q_{H}}}{=}\sum_{j=1}^{p-1}H_{j}\left(0\right)\sum_{n=1}^{\infty}M_{H}\left(n\right)\sum_{N=\lambda_{p}\left(n\right)+1}^{\infty}\left[\mathfrak{z}\overset{p^{N}}{\equiv}n+jp^{N-1}\right]\\ & \overset{\mathbb{Z}_{q_{H}}}{=}\sum_{j=1}^{p-1}H_{j}\left(0\right)\sum_{n=1}^{\infty}M_{H}\left(n\right)\left[\lambda_{p}\left(n\right)\leq v_{p}\left(\mathfrak{z}-n\right)\right]\left[\mathfrak{z}\overset{p^{v_{p}\left(\mathfrak{z}-n\right)+1}}{\equiv}n+jp^{v_{p}\left(\mathfrak{z}-n\right)}\right]\\ & \overset{\mathbb{Z}_{q_{H}}}{=}\sum_{j=1}^{p-1}H_{j}\left(0\right)\sum_{n=1}^{\infty}M_{H}\left(n\right)\left[\left|\mathfrak{z}-n\right|_{p}\leq p^{-\lambda_{p}\left(n\right)}\right]\left[\mathfrak{z}\overset{p^{v_{p}\left(\mathfrak{z}-n\right)+1}}{\equiv}n+jp^{v_{p}\left(\mathfrak{z}-n\right)}\right]\\ & \overset{\mathbb{Z}_{q_{H}}}{=}\sum_{j=1}^{p-1}H_{j}\left(0\right)\sum_{n=1}^{\infty}M_{H}\left(n\right)\left[\mathfrak{z}\overset{p^{\lambda_{p}\left(n\right)}}{\equiv}n\right]\left[\mathfrak{z}\overset{p^{v_{p}\left(\mathfrak{z}-n\right)+1}}{\equiv}n+jp^{v_{p}\left(\mathfrak{z}-n\right)}\right] \end{align*} Here: \begin{align*} \mathfrak{z} & \overset{p^{v_{p}\left(\mathfrak{z}-n\right)+1}}{\equiv}n+jp^{v_{p}\left(\mathfrak{z}-n\right)}\\ & \Updownarrow\\ \mathfrak{z}-n & \in jp^{v_{p}\left(\mathfrak{z}-n\right)}+p^{v_{p}\left(\mathfrak{z}-n\right)+1}\mathbb{Z}_{p}\\ & \Updownarrow\\ \left(\mathfrak{z}-n\right)\left|\mathfrak{z}-n\right|_{p} & \in j+p\mathbb{Z}_{p}\\ & \Updownarrow\\ u_{p}\left(\mathfrak{z}-n\right) & \overset{p}{\equiv}j \end{align*} and so: \begin{align} \chi_{H}\left(\mathfrak{z}\right)-\chi_{H}\left(\left[\mathfrak{z}\right]_{p}\right) & \overset{\mathbb{Z}_{q_{H}}}{=}\sum_{n=1}^{\infty}M_{H}\left(n\right)\left[\mathfrak{z}\overset{p^{\lambda_{p}\left(n\right)}}{\equiv}n\right]\sum_{j=1}^{p-1}H_{j}\left(0\right)\left[u_{p}\left(\mathfrak{z}-n\right)\overset{p}{\equiv}j\right] \end{align} Q.E.D. \vphantom{} With these formulae, we can directly estimate the $L_{\mathbb{R}}^{1}\left(\mathbb{Z}_{p},\mathbb{C}_{q_{H}}\right)$ norm of $\chi_{H}$. \begin{prop} \label{prop:L_R^1 norm of Chi_H}\index{chi{H}@$\chi_{H}$!L{mathbb{R}}{1}-norm@$L_{\mathbb{R}}^{1}$-norm} \begin{equation} \int_{\mathbb{Z}_{p}}\left|\chi_{H}\left(\mathfrak{z}\right)\right|_{q_{H}}d\mathfrak{z}\leq\frac{1}{p}\left(\sum_{j=1}^{p-1}\left|\chi_{H}\left(j\right)\right|_{q_{H}}\right)\sum_{n=0}^{\infty}\frac{\left|M_{H}\left(n\right)\right|_{q_{H}}}{p^{\lambda_{p}\left(n\right)}}\label{eq:real L^1 estimate for Chi_H} \end{equation} \end{prop} Proof: For brevity, let $q=q_{H}$. We use \textbf{Proposition \ref{prop:van der Put series for Chi_H}}. This gives: \begin{align*} \int_{\mathbb{Z}_{p}}\left|\chi_{H}\left(\mathfrak{z}\right)\right|_{q}d\mathfrak{z} & \leq\sum_{n=1}^{\infty}\sum_{j=1}^{p-1}\left|H_{j}\left(0\right)M_{H}\left(n\right)\right|_{q}\int_{\mathbb{Z}_{p}}\left[u_{p}\left(\mathfrak{z}-n\right)\overset{p}{\equiv}j\right]\left[\mathfrak{z}\overset{p^{\lambda_{p}\left(n\right)}}{\equiv}n\right]d\mathfrak{z}\\ & +\int_{\mathbb{Z}_{p}}\left|\chi_{H}\left(\left[\mathfrak{z}\right]_{p}\right)\right|_{q}d\mathfrak{z} \end{align*} Using the translation invariance of $d\mathfrak{z}$, we can write: \begin{align*} \int_{\mathbb{Z}_{p}}\left[u_{p}\left(\mathfrak{z}-n\right)\overset{p}{\equiv}j\right]\left[\mathfrak{z}\overset{p^{\lambda_{p}\left(n\right)}}{\equiv}n\right]d\mathfrak{z} & \overset{\mathbb{R}}{=}\int_{\mathbb{Z}_{p}}\left[u_{p}\left(\mathfrak{z}\right)\overset{p}{\equiv}j\right]\left[\mathfrak{z}+n\overset{p^{\lambda_{p}\left(n\right)}}{\equiv}n\right]d\mathfrak{z}\\ & =\int_{\mathbb{Z}_{p}}\left[u_{p}\left(\mathfrak{z}\right)\overset{p}{\equiv}j\right]\left[\mathfrak{z}\overset{p^{\lambda_{p}\left(n\right)}}{\equiv}0\right]d\mathfrak{z}\\ & =\int_{p^{\lambda_{p}\left(n\right)}\mathbb{Z}_{p}}\left[u_{p}\left(\mathfrak{z}\right)\overset{p}{\equiv}j\right]d\mathfrak{z}\\ \left(\mathfrak{y}=\frac{\mathfrak{z}}{p^{\lambda_{p}\left(n\right)}}\right); & =\frac{1}{p^{\lambda_{p}\left(n\right)}}\int_{\mathbb{Z}_{p}}\left[u_{p}\left(p^{\lambda_{p}\left(n\right)}\mathfrak{y}\right)\overset{p}{\equiv}j\right]d\mathfrak{z}\\ \left(u_{p}\left(p\mathfrak{y}\right)=u_{p}\left(\mathfrak{y}\right)\right); & =\frac{1}{p^{\lambda_{p}\left(n\right)}}\int_{\mathbb{Z}_{p}}\left[u_{p}\left(\mathfrak{y}\right)\overset{p}{\equiv}j\right]d\mathfrak{z}\\ \left(j\in\left\{ 1,\ldots,p-1\right\} :u_{p}\left(\mathfrak{y}\right)\overset{p}{\equiv}j\Leftrightarrow\mathfrak{y}\overset{p}{\equiv}j\right); & =\frac{1}{p^{\lambda_{p}\left(n\right)}}\int_{\mathbb{Z}_{p}}\left[\mathfrak{y}\overset{p}{\equiv}j\right]d\mathfrak{z}\\ & =\frac{1}{p^{\lambda_{p}\left(n\right)+1}} \end{align*} Thus: \begin{align*} \int_{\mathbb{Z}_{p}}\left|\chi_{H}\left(\mathfrak{z}\right)\right|_{q}d\mathfrak{z} & \leq\sum_{n=1}^{\infty}\sum_{j=1}^{p-1}\frac{\left|H_{j}\left(0\right)M_{H}\left(n\right)\right|_{q}}{p^{\lambda_{p}\left(n\right)+1}}+\int_{\mathbb{Z}_{p}}\left|\chi_{H}\left(\left[\mathfrak{z}\right]_{p}\right)\right|_{q}d\mathfrak{z}\\ & =\sum_{n=1}^{\infty}\sum_{j=1}^{p-1}\frac{\left|H_{j}\left(0\right)M_{H}\left(n\right)\right|_{q}}{p^{\lambda_{p}\left(n\right)+1}}+\frac{1}{p}\sum_{j=0}^{p-1}\left|\chi_{H}\left(j\right)\right|_{q}\\ \left(\chi_{H}\left(0\right)=0\right); & =\frac{1}{p}\sum_{j=1}^{p-1}\left(\left|\chi_{H}\left(j\right)\right|_{q}+\sum_{n=1}^{\infty}\left|H_{j}\left(0\right)\right|_{q}\frac{\left|M_{H}\left(n\right)\right|_{q}}{p^{\lambda_{p}\left(n\right)}}\right) \end{align*} Finally, since $\chi_{H}\left(j\right)=H_{j}\left(0\right)$ for all $j\in\left\{ 0,\ldots,p-1\right\} $, we have: \begin{align*} \int_{\mathbb{Z}_{p}}\left|\chi_{H}\left(\mathfrak{z}\right)\right|_{q}d\mathfrak{z} & \leq\frac{1}{p}\sum_{j=1}^{p-1}\left(\left|\chi_{H}\left(j\right)\right|_{q}+\sum_{n=1}^{\infty}\left|H_{j}\left(0\right)\right|_{q}\frac{\left|M_{H}\left(n\right)\right|_{q}}{p^{\lambda_{p}\left(n\right)}}\right)\\ & =\frac{1}{p}\left(\sum_{j=1}^{p-1}\left|\chi_{H}\left(j\right)\right|_{q}\right)\left(1+\sum_{n=1}^{\infty}\frac{\left|M_{H}\left(n\right)\right|_{q}}{p^{\lambda_{p}\left(n\right)}}\right)\\ \left(M_{H}\left(0\right)=1\right); & =\frac{1}{p}\left(\sum_{j=1}^{p-1}\left|\chi_{H}\left(j\right)\right|_{q}\right)\sum_{n=0}^{\infty}\frac{\left|M_{H}\left(n\right)\right|_{q}}{p^{\lambda_{p}\left(n\right)}} \end{align*} Q.E.D. \vphantom{} With this, we can now state the sufficient condition for $\chi_{H}$ to be in $L_{\mathbb{R}}^{1}$. \begin{thm}[\textbf{Criterion for $\chi_{H}\in L_{\mathbb{R}}^{1}$}] \label{thm:L^1 criterion for Chi_H}$\chi_{H}$ is an element of $L_{\mathbb{R}}^{1}\left(\mathbb{Z}_{p},\mathbb{C}_{q_{H}}\right)$ whenever: \begin{equation} \sum_{j=0}^{p-1}\left|\mu_{j}\right|_{q_{H}}<p\label{eq:Criterion for Chi_H to be real L^1 integrable} \end{equation} In particular, since the $\mu_{j}$s are integers, this shows that $\chi_{H}\in L_{\mathbb{R}}^{1}\left(\mathbb{Z}_{p},\mathbb{C}_{q_{H}}\right)$ occurs whenever $\gcd\left(\mu_{j},q_{H}\right)>1$ for at least one $j\in\left\{ 0,\ldots,p-1\right\} $. \end{thm} Proof: For brevity, let $q=q_{H}$. By \textbf{Proposition \ref{prop:L_R^1 norm of Chi_H}}, to show that $\chi_{H}$ has finite $L_{\mathbb{R}}^{1}\left(\mathbb{Z}_{p},\mathbb{C}_{q}\right)$ norm, it suffices to prove that: \begin{equation} \sum_{n=0}^{\infty}\frac{\left|M_{H}\left(n\right)\right|_{q}}{p^{\lambda_{p}\left(n\right)}}<\infty \end{equation} To do this, we use a lambda decomposition: \begin{equation} \sum_{n=0}^{\infty}\frac{\left|M_{H}\left(n\right)\right|_{q}}{p^{\lambda_{p}\left(n\right)}}\overset{\mathbb{R}}{=}1+\sum_{m=1}^{\infty}\frac{1}{p^{m}}\sum_{n=p^{m-1}}^{p^{m}-1}\left|M_{H}\left(n\right)\right|_{q}\label{eq:M_H q-adic absolute value lambda decomposition} \end{equation} Once again, we proceed recursively: \begin{equation} S_{m}\overset{\textrm{def}}{=}\sum_{n=0}^{p^{m}-1}\left|M_{H}\left(n\right)\right|_{q} \end{equation} Then, using $M_{H}$'s functional equations: \begin{align*} \sum_{n=0}^{p^{m}-1}\left|M_{H}\left(n\right)\right|_{q} & =\sum_{j=0}^{p-1}\left|M_{H}\left(j\right)\right|_{q}+\sum_{j=0}^{p-1}\sum_{n=1}^{p^{m-1}-1}\left|M_{H}\left(pn+j\right)\right|_{q}\\ & =\sum_{j=0}^{p-1}\left|M_{H}\left(j\right)\right|_{q}+\sum_{j=0}^{p-1}\sum_{n=1}^{p^{m-1}-1}\left|\frac{\mu_{j}}{p}M_{H}\left(n\right)\right|_{q}\\ \left(\gcd\left(p,q\right)=1\right); & =\sum_{j=0}^{p-1}\left|M_{H}\left(j\right)\right|_{q}+\sum_{n=1}^{p^{m-1}-1}\left(\sum_{j=0}^{p-1}\left|\mu_{j}\right|_{q}\right)\left|M_{H}\left(n\right)\right|_{q}\\ \left(M_{H}\left(j\right)=\mu_{j}\textrm{ }\forall j\in\left\{ 1,\ldots,p-1\right\} \right); & =1+\sum_{j=1}^{p-1}\left|\mu_{j}\right|_{q}+\left(\sum_{j=0}^{p-1}\left|\mu_{j}\right|_{q}\right)\sum_{n=1}^{p^{m-1}-1}\left|M_{H}\left(n\right)\right|_{q}\\ & =1-\left|\mu_{0}\right|_{q}+\left(\sum_{j=0}^{p-1}\left|\mu_{j}\right|_{q}\right)\left(1+\sum_{n=1}^{p^{m-1}-1}\left|M_{H}\left(n\right)\right|_{q}\right)\\ & =1-\left|\mu_{0}\right|_{q}+\left(\sum_{j=0}^{p-1}\left|\mu_{j}\right|_{q}\right)\sum_{n=0}^{p^{m-1}-1}\left|M_{H}\left(n\right)\right|_{q} \end{align*} So: \begin{equation} S_{m}=1-\left|\mu_{0}\right|_{q}+\left(\sum_{j=0}^{p-1}\left|\mu_{j}\right|_{q}\right)S_{m-1},\textrm{ }\forall m\geq1 \end{equation} where: \begin{equation} S_{0}=\left|M_{H}\left(0\right)\right|_{q}=1 \end{equation} Finally, defining $A$ and $B$ by: \begin{align} A & \overset{\textrm{def}}{=}1-\left|\mu_{0}\right|_{q}\\ B & \overset{\textrm{def}}{=}\sum_{j=0}^{p-1}\left|\mu_{j}\right|_{q} \end{align} we can write: \begin{align*} S_{m} & =A+BS_{m-1}\\ & =A+AB+B^{2}S_{m-2}\\ & =A+AB+AB^{2}+B^{3}S_{m-3}\\ & \vdots\\ & =A\sum_{k=0}^{m-1}B^{k}+B^{m}S_{0}\\ & =B^{m}+A\sum_{k=0}^{m-1}B^{k} \end{align*} Using this, our lambda decomposition (\ref{eq:M_H q-adic absolute value lambda decomposition}) becomes: \begin{align*} \sum_{n=0}^{\infty}\frac{\left|M_{H}\left(n\right)\right|_{q}}{p^{\lambda_{p}\left(n\right)}} & \overset{\mathbb{R}}{=}1+\sum_{m=1}^{\infty}\frac{1}{p^{m}}\left(\sum_{n=0}^{p^{m}-1}\left|M_{H}\left(n\right)\right|_{q}-\sum_{n=0}^{p^{m-1}-1}\left|M_{H}\left(n\right)\right|_{q}\right)\\ & =1+\sum_{m=1}^{\infty}\frac{1}{p^{m}}\left(S_{m}-S_{m-1}\right)\\ & =1+\sum_{m=1}^{\infty}\frac{S_{m}}{p^{m}}-p\sum_{m=1}^{\infty}\frac{S_{m-1}}{p^{m-1}}\\ & =1-p\frac{S_{0}}{p^{0}}+\sum_{m=1}^{\infty}\frac{S_{m}}{p^{m}}-p\sum_{m=1}^{\infty}\frac{S_{m}}{p^{m}}\\ & =1-p+\left(1-p\right)\sum_{m=1}^{\infty}\frac{S_{m}}{p^{m}}\\ & =1-p+\left(1-p\right)\sum_{m=1}^{\infty}\frac{\left(B^{m}+A\sum_{k=0}^{m-1}B^{k}\right)}{p^{m}}\\ & \overset{\mathbb{R}}{=}\begin{cases} 1-p+\left(1-p\right)\sum_{m=1}^{\infty}\frac{Am+1}{p^{m}} & \textrm{if }B=1\\ 1-p+\left(1-p\right)\sum_{m=1}^{\infty}\frac{B^{m}+A\frac{B^{m}-1}{B-1}}{p^{m}} & \textrm{else} \end{cases} \end{align*} The $B=1$ case is finite since $p\geq2$. As for $B\neq1$, summing the geometric series yields: \begin{equation} \sum_{m=1}^{\infty}\frac{B^{m}+A\frac{B^{m}-1}{B-1}}{p^{m}}\overset{\mathbb{R}}{=}-\frac{A}{B-1}\frac{1}{p-1}+\frac{A+B-1}{B-1}\sum_{m=1}^{\infty}\left(\frac{B}{p}\right)^{m} \end{equation} Hence, $0\leq B<p$ is sufficient to guarantee convergence in the topology of $\mathbb{R}$. Q.E.D. \vphantom{} We end our wisp of $L_{\mathbb{R}}^{1}$ by proving in general what was shown for $\chi_{5}$ in Subsection \ref{subsec:3.3.6 L^1 Convergence}: that $\tilde{\chi}_{H,N}-\chi_{H,N}$ converges to $0$ in $L_{\mathbb{R}}^{1}$ as $N\rightarrow\infty$. The first step is another computation\textemdash this time outsourced to previous work. \begin{prop} \label{prop:Convolution of alpha and A_H hat}Suppose $\alpha_{H}\left(0\right)\notin\left\{ 0,1\right\} $. Then: \begin{equation} \sum_{0<\left|t\right|_{p}\leq p^{N}}\left(\alpha_{H}\left(0\right)\right)^{v_{p}\left(t\right)}\hat{A}_{H}\left(t\right)e^{2\pi i\left\{ t\mathfrak{z}\right\} _{p}}\overset{\overline{\mathbb{Q}}}{=}\left(\frac{\mu_{0}}{p\alpha_{H}\left(0\right)}\right)^{N}\kappa_{H}\left(\left[\mathfrak{z}\right]_{p^{N}}\right)-1\label{eq:Partial Fourier Sum of A_H hat times alpha to the v_p(t)} \end{equation} \end{prop} Proof: Use (\ref{eq:Convolution of dA_H and D_N}) from \textbf{Theorem \ref{thm:Properties of dA_H}}. Q.E.D. \vphantom{} With this, we can compute simple closed-form expressions for $\tilde{\chi}_{H,N}\left(\mathfrak{z}\right)-\chi_{H,N}\left(\mathfrak{z}\right)$. It should be noted that the formulae given below only hold for the indicated choice of $\hat{\chi}_{H}$. Choosing a different Fourier transform of $\hat{\chi}_{H}$ will alter the formula for $\tilde{\chi}_{H,N}\left(\mathfrak{z}\right)-\chi_{H,N}\left(\mathfrak{z}\right)$. \begin{thm}[\textbf{$\chi_{H,N}\left(\mathfrak{z}\right)-\tilde{\chi}_{H,N}\left(\mathfrak{z}\right)$}] \label{thm:Chi_H,N / Chi_H,N twiddle error}Choose $\hat{\chi}_{H}\left(t\right)$ to be the Fourier Transform of $\chi_{H}$ defined by \emph{(\ref{eq:Fourier Transform of Chi_H for a contracting semi-basic rho-Hydra map})}: \[ \hat{\chi}_{H}\left(t\right)\overset{\textrm{def}}{=}\begin{cases} \begin{cases} 0 & \textrm{if }t=0\\ \left(\beta_{H}\left(0\right)v_{p}\left(t\right)+\gamma_{H}\left(\frac{t\left|t\right|_{p}}{p}\right)\right)\hat{A}_{H}\left(t\right) & \textrm{if }t\neq0 \end{cases} & \textrm{if }\alpha_{H}\left(0\right)=1\\ \frac{\beta_{H}\left(0\right)\hat{A}_{H}\left(t\right)}{1-\alpha_{H}\left(0\right)}+\begin{cases} 0 & \textrm{if }t=0\\ \gamma_{H}\left(\frac{t\left|t\right|_{p}}{p}\right)\hat{A}_{H}\left(t\right) & \textrm{if }t\neq0 \end{cases} & \textrm{if }\alpha_{H}\left(0\right)\neq1 \end{cases},\textrm{ }\forall t\in\hat{\mathbb{Z}}_{p} \] Then: \vphantom{} I. If $\alpha_{H}\left(0\right)=1$: \begin{equation} \chi_{H,N}\left(\mathfrak{z}\right)-\tilde{\chi}_{H,N}\left(\mathfrak{z}\right)\overset{\overline{\mathbb{Q}}}{=}\beta_{H}\left(0\right)\left(\frac{\mu_{0}}{p}\right)^{N-1}\left(\frac{2\mu_{0}N}{p}\kappa_{H}\left(\left[\mathfrak{z}\right]_{p^{N}}\right)-\left(N-1\right)\kappa_{H}\left(\left[\mathfrak{z}\right]_{p^{N-1}}\right)\right)\label{eq:Chi_H,N / Chi_H,N twiddle identity for alpha equals 1} \end{equation} \vphantom{} II. If $\alpha_{H}\left(0\right)\neq1$: \begin{equation} \chi_{H,N}\left(\mathfrak{z}\right)-\tilde{\chi}_{H,N}\left(\mathfrak{z}\right)\overset{\overline{\mathbb{Q}}}{=}\frac{\beta_{H}\left(0\right)}{\alpha_{H}\left(0\right)-1}\left(\frac{\mu_{0}}{p}\right)^{N-1}\left(\frac{2\mu_{0}}{p}\kappa_{H}\left(\left[\mathfrak{z}\right]_{p^{N}}\right)-\alpha_{H}\left(0\right)\kappa_{H}\left(\left[\mathfrak{z}\right]_{p^{N-1}}\right)\right)\label{eq:Chi_H,N / Chi_H,N twiddle identity for alpha is not 1} \end{equation} \end{thm} Proof: I. Let $\alpha_{H}\left(0\right)=1$. Then, we have that: \begin{equation} \hat{\chi}_{H}\left(t\right)\overset{\overline{\mathbb{Q}}}{=}\begin{cases} 0 & \textrm{if }t=0\\ \left(\beta_{H}\left(0\right)v_{p}\left(t\right)+\gamma_{H}\left(\frac{t\left|t\right|_{p}}{p}\right)\right)\hat{A}_{H}\left(t\right) & \textrm{if }t\neq0 \end{cases} \end{equation} Turning to \textbf{Theorem \ref{thm:(N,t) asymptotic decomposition of Chi_H,N hat}}), the associated ``fine-structure'' formula for $\hat{\chi}_{H,N}$ in this case is: \[ \hat{\chi}_{H,N}\left(t\right)-\beta_{H}\left(0\right)N\hat{A}_{H}\left(t\right)\mathbf{1}_{0}\left(p^{N-1}t\right)=\begin{cases} 0 & \textrm{if }t=0\\ \left(\gamma_{H}\left(\frac{t\left|t\right|_{p}}{p}\right)+\beta_{H}\left(0\right)v_{p}\left(t\right)\right)\hat{A}_{H}\left(t\right) & \textrm{if }0<\left|t\right|_{p}<p^{N}\\ \gamma_{H}\left(\frac{t\left|t\right|_{p}}{p}\right)\hat{A}_{H}\left(t\right) & \textrm{if }\left|t\right|_{p}=p^{N}\\ 0 & \textrm{if }\left|t\right|_{p}>p^{N} \end{cases} \] Using use our chosen $\hat{\chi}_{H}$ (and remembering that $\hat{A}_{H}\left(0\right)=1$), this can be rewritten as: \begin{align} \hat{\chi}_{H,N}\left(t\right)-\beta_{H}\left(0\right)N\hat{A}_{H}\left(t\right)\mathbf{1}_{0}\left(p^{N-1}t\right) & \overset{\overline{\mathbb{Q}}}{=}\mathbf{1}_{0}\left(p^{N-1}t\right)\hat{\chi}_{H}\left(t\right)\label{eq:Chi_H,N / Chi_H,N twiddle proof: eq 1}\\ & +\left[\left|t\right|_{p}=p^{N}\right]\gamma_{H}\left(\frac{t\left|t\right|_{p}}{p}\right)\hat{A}_{H}\left(t\right),\textrm{ }\forall\left|t\right|_{p}\leq p^{N}\nonumber \end{align} which is to say: \begin{align} \hat{\chi}_{H,N}\left(t\right)-\beta_{H}\left(0\right)N\hat{A}_{H}\left(t\right)\mathbf{1}_{0}\left(p^{N-1}t\right) & \overset{\overline{\mathbb{Q}}}{=}\mathbf{1}_{0}\left(p^{N}t\right)\hat{\chi}_{H}\left(t\right)\label{eq:Chi_H,N / Chi_H,N twiddle proof: eq 2}\\ & +\left[\left|t\right|_{p}=p^{N}\right]\left(\gamma_{H}\left(\frac{t\left|t\right|_{p}}{p}\right)\hat{A}_{H}\left(t\right)-\hat{\chi}_{H}\left(t\right)\right),\textrm{ }\forall\left|t\right|_{p}\leq p^{N}\nonumber \end{align} Multiplying through by $e^{2\pi i\left\{ t\mathfrak{z}\right\} _{p}}$ and summing over $\left|t\right|_{p}\leq p^{N}$ yields: \begin{align} \chi_{H,N}\left(\mathfrak{z}\right)-\beta_{H}\left(0\right)N\tilde{A}_{H,N-1}\left(\mathfrak{z}\right) & \overset{\overline{\mathbb{Q}}}{=}\tilde{\chi}_{H,N}\left(\mathfrak{z}\right)\label{eq:Chi_H,N / Chi_H,N twiddle proof: eq 3}\\ & +\sum_{\left|t\right|_{p}=p^{N}}\left(\gamma_{H}\left(\frac{t\left|t\right|_{p}}{p}\right)\hat{A}_{H}\left(t\right)-\hat{\chi}_{H}\left(t\right)\right)e^{2\pi i\left\{ t\mathfrak{z}\right\} _{p}}\nonumber \end{align} Using \textbf{Lemma \ref{lem:1D gamma formula}}, we have: \begin{align*} \sum_{\left|t\right|_{p}=p^{N}}\gamma_{H}\left(\frac{t\left|t\right|_{p}}{p}\right)\hat{A}_{H}\left(t\right)e^{2\pi i\left\{ t\mathfrak{z}\right\} _{p}} & =\sum_{0<\left|t\right|_{p}\leq p^{N}}\gamma_{H}\left(\frac{t\left|t\right|_{p}}{p}\right)\hat{A}_{H}\left(t\right)e^{2\pi i\left\{ t\mathfrak{z}\right\} _{p}}\\ & -\sum_{0<\left|t\right|_{p}\leq p^{N-1}}\gamma_{H}\left(\frac{t\left|t\right|_{p}}{p}\right)\hat{A}_{H}\left(t\right)e^{2\pi i\left\{ t\mathfrak{z}\right\} _{p}}\\ & =\sum_{n=0}^{N-1}\left(\sum_{j=1}^{p-1}\beta_{H}\left(\frac{j}{p}\right)\varepsilon_{n}^{j}\left(\mathfrak{z}\right)\right)\left(\frac{\mu_{0}}{p}\right)^{n}\kappa_{H}\left(\left[\mathfrak{z}\right]_{p^{n}}\right)\\ & -\sum_{n=0}^{N-2}\left(\sum_{j=1}^{p-1}\beta_{H}\left(\frac{j}{p}\right)\varepsilon_{n}^{j}\left(\mathfrak{z}\right)\right)\left(\frac{\mu_{0}}{p}\right)^{n}\kappa_{H}\left(\left[\mathfrak{z}\right]_{p^{n}}\right)\\ & =\left(\sum_{j=1}^{p-1}\beta_{H}\left(\frac{j}{p}\right)\varepsilon_{N-1}^{j}\left(\mathfrak{z}\right)\right)\left(\frac{\mu_{0}}{p}\right)^{N-1}\kappa_{H}\left(\left[\mathfrak{z}\right]_{p^{N-1}}\right) \end{align*} Using (\ref{eq:Explicit Formula for Chi_H,N twiddle for arbitrary rho and alpha equals 1}) from \textbf{Corollary \ref{cor:Chi_H,N twiddle explicit formula, arbitrary p, arbitrary alpha}} gives us: \begin{align*} \sum_{\left|t\right|_{p}=p^{N}}\hat{\chi}_{H}\left(t\right)e^{2\pi i\left\{ t\mathfrak{z}\right\} _{p}} & =\tilde{\chi}_{H,N}\left(\mathfrak{z}\right)-\tilde{\chi}_{H,N-1}\left(\mathfrak{z}\right)\\ & \overset{\overline{\mathbb{Q}}}{=}-\beta_{H}\left(0\right)N\left(\frac{\mu_{0}}{p}\right)^{N}\kappa_{H}\left(\left[\mathfrak{z}\right]_{p^{N}}\right)+\beta_{H}\left(0\right)\sum_{n=0}^{N-1}\left(\frac{\mu_{0}}{p}\right)^{n}\kappa_{H}\left(\left[\mathfrak{z}\right]_{p^{n}}\right)\\ & +\sum_{n=0}^{N-1}\left(\sum_{j=1}^{p-1}\beta_{H}\left(\frac{j}{p}\right)\varepsilon_{n}^{j}\left(\mathfrak{z}\right)\right)\left(\frac{\mu_{0}}{p}\right)^{n}\kappa_{H}\left(\left[\mathfrak{z}\right]_{p^{n}}\right)\\ & +\beta_{H}\left(0\right)\left(N-1\right)\left(\frac{\mu_{0}}{p}\right)^{N-1}\kappa_{H}\left(\left[\mathfrak{z}\right]_{p^{N-1}}\right)\\ & -\beta_{H}\left(0\right)\sum_{n=0}^{N-2}\left(\frac{\mu_{0}}{p}\right)^{n}\kappa_{H}\left(\left[\mathfrak{z}\right]_{p^{n}}\right)\\ & -\sum_{n=0}^{N-2}\left(\sum_{j=1}^{p-1}\beta_{H}\left(\frac{j}{p}\right)\varepsilon_{n}^{j}\left(\mathfrak{z}\right)\right)\left(\frac{\mu_{0}}{p}\right)^{n}\kappa_{H}\left(\left[\mathfrak{z}\right]_{p^{n}}\right)\\ & \overset{\overline{\mathbb{Q}}}{=}\beta_{H}\left(0\right)\left(N-1\right)\left(\frac{\mu_{0}}{p}\right)^{N-1}\kappa_{H}\left(\left[\mathfrak{z}\right]_{p^{N-1}}\right)\\ & -\beta_{H}\left(0\right)N\left(\frac{\mu_{0}}{p}\right)^{N}\kappa_{H}\left(\left[\mathfrak{z}\right]_{p^{N}}\right)\\ & +\left(\sum_{j=0}^{p-1}\beta_{H}\left(\frac{j}{p}\right)\varepsilon_{N-1}^{j}\left(\mathfrak{z}\right)\right)\left(\frac{\mu_{0}}{p}\right)^{N-1}\kappa_{H}\left(\left[\mathfrak{z}\right]_{p^{N-1}}\right) \end{align*} Finally, employing our familiar formula for for $\tilde{A}_{H,N}\left(\mathfrak{z}\right)$ (equation (\ref{eq:Convolution of dA_H and D_N}) from \textbf{Theorem \ref{thm:Properties of dA_H}}) and remembering that $\alpha_{H}\left(0\right)=1$, (\ref{eq:Chi_H,N / Chi_H,N twiddle proof: eq 3}) becomes: \begin{align*} \chi_{H,N}\left(\mathfrak{z}\right)-\beta_{H}\left(0\right)N\left(\frac{\mu_{0}}{p}\right)^{N}\kappa_{H}\left(\left[\mathfrak{z}\right]_{p^{N}}\right) & \overset{\overline{\mathbb{Q}}}{=}\tilde{\chi}_{H,N}\left(\mathfrak{z}\right)\\ & -\beta_{H}\left(0\right)\left(N-1\right)\left(\frac{\mu_{0}}{p}\right)^{N-1}\kappa_{H}\left(\left[\mathfrak{z}\right]_{p^{N-1}}\right)\\ & +\beta_{H}\left(0\right)N\left(\frac{\mu_{0}}{p}\right)^{N}\kappa_{H}\left(\left[\mathfrak{z}\right]_{p^{N}}\right) \end{align*} and hence: \[ \chi_{H,N}\left(\mathfrak{z}\right)-\tilde{\chi}_{H,N}\left(\mathfrak{z}\right)\overset{\overline{\mathbb{Q}}}{=}\beta_{H}\left(0\right)\left(\frac{\mu_{0}}{p}\right)^{N-1}\left(\frac{2\mu_{0}N}{p}\kappa_{H}\left(\left[\mathfrak{z}\right]_{p^{N}}\right)-\left(N-1\right)\kappa_{H}\left(\left[\mathfrak{z}\right]_{p^{N-1}}\right)\right) \] which proves (I). \vphantom{} II. Suppose $\alpha_{H}\left(0\right)\neq1$. The ``fine-structure'' equation from our $\left(N,t\right)$-asymptotic analysis of $\hat{\chi}_{H,N}$ ((\ref{eq:Fine Structure Formula for Chi_H,N hat when alpha is not 1}) from \textbf{Theorem \ref{thm:(N,t) asymptotic decomposition of Chi_H,N hat}}) is: \[ \hat{\chi}_{H,N}\left(t\right)=\begin{cases} \beta_{H}\left(0\right)\frac{\left(\alpha_{H}\left(0\right)\right)^{N}-1}{\alpha_{H}\left(0\right)-1} & \textrm{if }t=0\\ \left(\gamma_{H}\left(\frac{t\left|t\right|_{p}}{p}\right)+\beta_{H}\left(0\right)\frac{\left(\alpha_{H}\left(0\right)\right)^{N+v_{p}\left(t\right)}-1}{\alpha_{H}\left(0\right)-1}\right)\hat{A}_{H}\left(t\right) & \textrm{if }0<\left|t\right|_{p}<p^{N}\\ \gamma_{H}\left(\frac{t\left|t\right|_{p}}{p}\right)\hat{A}_{H}\left(t\right) & \textrm{if }\left|t\right|_{p}=p^{N}\\ 0 & \textrm{if }\left|t\right|_{p}>p^{N} \end{cases},\textrm{ }\forall t\in\hat{\mathbb{Z}}_{p} \] Meanwhile, $\hat{\chi}_{H}\left(t\right)$ is: \[ \hat{\chi}_{H}\left(t\right)=\begin{cases} \frac{\beta_{H}\left(0\right)}{1-\alpha_{H}\left(0\right)} & \textrm{if }t=0\\ \left(\frac{\beta_{H}\left(0\right)}{1-\alpha_{H}\left(0\right)}+\gamma_{H}\left(\frac{t\left|t\right|_{p}}{p}\right)\right)\hat{A}_{H}\left(t\right) & \textrm{if }t\neq0 \end{cases} \] Subtracting yields: \begin{equation} \hat{\chi}_{H,N}\left(t\right)-\hat{\chi}_{H}\left(t\right)=\begin{cases} \frac{\beta_{H}\left(0\right)\left(\alpha_{H}\left(0\right)\right)^{N}}{\alpha_{H}\left(0\right)-1} & \textrm{if }t=0\\ \frac{\beta_{H}\left(0\right)\left(\alpha_{H}\left(0\right)\right)^{N+v_{p}\left(t\right)}}{\alpha_{H}\left(0\right)-1}\hat{A}_{H}\left(t\right) & \textrm{if }0<\left|t\right|_{p}<p^{N}\\ \frac{\beta_{H}\left(0\right)}{\alpha_{H}\left(0\right)-1}\hat{A}_{H}\left(t\right) & \textrm{if }\left|t\right|_{p}=p^{N}\\ \left(\frac{\beta_{H}\left(0\right)}{\alpha_{H}\left(0\right)-1}-\gamma_{H}\left(\frac{t\left|t\right|_{p}}{p}\right)\right)\hat{A}_{H}\left(t\right) & \textrm{if }\left|t\right|_{p}>p^{N} \end{cases},\textrm{ }\forall t\in\hat{\mathbb{Z}}_{p}\label{eq:Chi_H,N / Chi_H,N twiddle proof: eq 4} \end{equation} Since $\hat{\chi}_{H,N}\left(t\right)$ is supported on $\left|t\right|_{p}\leq p^{N}$, when summing our Fourier series, we need only sum over $\left|t\right|_{p}\leq p^{N}$. Doing so yields: \begin{align*} \chi_{H,N}\left(\mathfrak{z}\right)-\tilde{\chi}_{H,N}\left(\mathfrak{z}\right) & =\frac{\beta_{H}\left(0\right)\left(\alpha_{H}\left(0\right)\right)^{N}}{\alpha_{H}\left(0\right)-1}+\frac{\beta_{H}\left(0\right)}{\alpha_{H}\left(0\right)-1}\sum_{\left|t\right|_{p}=p^{N}}\hat{A}_{H}\left(t\right)e^{2\pi i\left\{ t\mathfrak{z}\right\} _{p}}\\ & +\frac{\beta_{H}\left(0\right)\left(\alpha_{H}\left(0\right)\right)^{N}}{\alpha_{H}\left(0\right)-1}\sum_{0<\left|t\right|_{p}<p^{N}}\left(\alpha_{H}\left(0\right)\right)^{v_{p}\left(t\right)}\hat{A}_{H}\left(t\right)e^{2\pi i\left\{ t\mathfrak{z}\right\} _{p}} \end{align*} Using (\ref{eq:Convolution of dA_H and D_N}) and \textbf{Proposition \ref{prop:Convolution of alpha and A_H hat}}, we obtain: \begin{align*} \chi_{H,N}\left(\mathfrak{z}\right)-\tilde{\chi}_{H,N}\left(\mathfrak{z}\right) & =\frac{\beta_{H}\left(0\right)\left(\alpha_{H}\left(0\right)\right)^{N}}{\alpha_{H}\left(0\right)-1}\\ & +\frac{\beta_{H}\left(0\right)}{\alpha_{H}\left(0\right)-1}\left(\left(\frac{\mu_{0}}{p}\right)^{N}\kappa_{H}\left(\left[\mathfrak{z}\right]_{p^{N}}\right)-\alpha_{H}\left(0\right)\left(\frac{\mu_{0}}{p}\right)^{N-1}\kappa_{H}\left(\left[\mathfrak{z}\right]_{p^{N-1}}\right)\right)\\ & +\frac{\beta_{H}\left(0\right)\left(\alpha_{H}\left(0\right)\right)^{N}}{\alpha_{H}\left(0\right)-1}\left(\left(\frac{\mu_{0}}{p\alpha_{H}\left(0\right)}\right)^{N}\kappa_{H}\left(\left[\mathfrak{z}\right]_{p^{N}}\right)-1\right)\\ & =\frac{\beta_{H}\left(0\right)}{\alpha_{H}\left(0\right)-1}\left(\frac{\mu_{0}}{p}\right)^{N-1}\left(\frac{2\mu_{0}}{p}\kappa_{H}\left(\left[\mathfrak{z}\right]_{p^{N}}\right)-\alpha_{H}\left(0\right)\kappa_{H}\left(\left[\mathfrak{z}\right]_{p^{N-1}}\right)\right) \end{align*} Q.E.D. \vphantom{} The result just proved is of independent interest, seeing as it provides us with an explicit form for the error between $\chi_{H,N}\left(\mathfrak{z}\right)$ and $\tilde{\chi}_{H,N}\left(\mathfrak{z}\right)$. A significant issue can be seen by examining (\ref{eq:Chi_H,N / Chi_H,N twiddle identity for alpha equals 1}) and (\ref{eq:Chi_H,N / Chi_H,N twiddle identity for alpha is not 1}) for $\mathfrak{z}=0$. Since $\chi_{H,N}\left(0\right)=\chi_{H}\left(\left[0\right]_{p^{N}}\right)=0$ for all $N\geq0$, we obtain: \begin{equation} \tilde{\chi}_{H,N}\left(0\right)\overset{\overline{\mathbb{Q}}}{=}\begin{cases} \beta_{H}\left(0\right)\left(\left(\frac{2\mu_{0}}{p}-1\right)N+1\right)\left(\frac{\mu_{0}}{p}\right)^{N-1} & \textrm{if }\alpha_{H}\left(0\right)=1\\ \frac{\beta_{H}\left(0\right)}{\alpha_{H}\left(0\right)-1}\left(\frac{2\mu_{0}}{p}-\alpha_{H}\left(0\right)\right)\left(\frac{\mu_{0}}{p}\right)^{N-1} & \textrm{if }\alpha_{H}\left(0\right)\neq1 \end{cases},\textrm{ }\forall N\geq1\label{eq:Formula for Chi_H,N twiddle at 0} \end{equation} When $\alpha_{H}\left(0\right)=1$, observe that $\tilde{\chi}_{H,N}\left(0\right)$ is non-zero for all $N\geq1$, with it tending to $0$ in $\mathbb{R}$ as $N\rightarrow\infty$. The same is true when $\alpha_{H}\left(0\right)\neq1$, unless $2\mu_{0}=p\alpha_{H}\left(0\right)$. Because the $N$th truncation $\chi_{H,N}\left(\mathfrak{z}\right)$ is continuous and vanishes for $\mathfrak{z}\overset{p^{N}}{\equiv}0$, the WTT for continuous $\left(p,q\right)$-adic functions tells us that $\hat{\chi}_{H,N}\left(t\right)$ does not have a convolution inverse. However, this statement need not be true for $\tilde{\chi}_{H,N}\left(\mathfrak{z}\right)$. That being said, using the previous proposition, we can establish the $L_{\mathbb{R}}^{1}$ convergence of $\tilde{\chi}_{H,N}$ to $\chi_{H,N}$ and vice-versa. We just need one last computation under our belts: the $L_{\mathbb{R}}^{1}$-norm of $\kappa_{H,N}\left(\mathfrak{z}\right)$ \begin{prop}[\textbf{$L_{\mathbb{R}}^{1}$-norm of $\kappa_{H,N}$}] \label{prop:L^1_R norm of kapp_H,N}\index{kappa{H}@$\kappa_{H}$!truncation and L{mathbb{R}}{1}-norm@truncation and $L_{\mathbb{R}}^{1}$-norm}Let $H$ be a contracting semi-basic $p$-Hydra map, and let $q=q_{H}$. Then: \begin{equation} \int_{\mathbb{Z}_{p}}\left|\kappa_{H,N}\left(\mathfrak{z}\right)\right|_{q}d\mathfrak{z}=\int_{\mathbb{Z}_{p}}\left|\kappa_{H}\left(\left[\mathfrak{z}\right]_{p^{N}}\right)\right|_{q}d\mathfrak{z}\leq\left(\frac{p+q-1}{pq}\right)^{N},\textrm{ }\forall N\geq1\label{eq:L^1_R norm estimate of Nth truncation of Kappa_H} \end{equation} In particular, since $p+q-1<pq$ for all $p,q\geq2$, we have that the $N$th truncations of $\kappa_{H}$ converge to $0$ in $L_{\mathbb{R}}^{1}\left(\mathbb{Z}_{p},\mathbb{C}_{q}\right)$ as $N\rightarrow\infty$. \end{prop} Proof: Since $\left|\kappa_{H}\left(\left[\mathfrak{z}\right]_{p^{N}}\right)\right|_{q}$ is a locally constant real valued function of $\mathfrak{z}$ whose output is determined by the value of $\mathfrak{z}$ mod $p^{N}$, we have that: \begin{equation} \int_{\mathbb{Z}_{p}}\left|\kappa_{H}\left(\left[\mathfrak{z}\right]_{p^{N}}\right)\right|_{q}d\mathfrak{z}=\frac{1}{p^{N}}\sum_{n=0}^{p^{N}-1}\left|\kappa_{H}\left(n\right)\right|_{q} \end{equation} Once again, we proceed recursively. Let: \begin{equation} S_{N}=\frac{1}{p^{N}}\sum_{n=0}^{p^{N}-1}\left|\kappa_{H}\left(n\right)\right|_{q} \end{equation} where, note, $S_{0}=\left|\kappa_{H}\left(0\right)\right|_{q}=\left|1\right|_{q}=1$. Then, splitting the $n$-sum modulo $p$: \begin{align*} S_{N} & =\frac{1}{p^{N}}\sum_{n=0}^{p^{N-1}-1}\sum_{j=0}^{p-1}\left|\kappa_{H}\left(pn+j\right)\right|_{q}\\ & =\frac{1}{p^{N}}\sum_{n=0}^{p^{N-1}-1}\sum_{j=0}^{p-1}\left|\frac{\mu_{j}}{\mu_{0}}\kappa_{H}\left(n\right)\right|_{q}\\ & =\left(\sum_{j=0}^{p-1}\left|\frac{\mu_{j}}{\mu_{0}}\right|_{q}\right)\frac{1}{p^{N}}\sum_{n=0}^{p^{N-1}-1}\left|\kappa_{H}\left(n\right)\right|_{q}\\ & =\left(\frac{1}{p}\sum_{j=0}^{p-1}\left|\frac{\mu_{j}}{\mu_{0}}\right|_{q}\right)S_{N-1} \end{align*} Hence: \begin{equation} S_{N}=\left(\frac{1}{p}\sum_{j=0}^{p-1}\left|\frac{\mu_{j}}{\mu_{0}}\right|_{q}\right)^{N}S_{0}=\left(\frac{1}{p}\sum_{j=0}^{p-1}\left|\frac{\mu_{j}}{\mu_{0}}\right|_{q}\right)^{N} \end{equation} Since $H$ is given to be semi-basic, we have that: \begin{equation} \left|\frac{\mu_{j}}{\mu_{0}}\right|_{q}\leq\frac{1}{q},\textrm{ }\forall j\in\left\{ 1,\ldots,q-1\right\} \end{equation} and so: \[ S_{N}\leq\left(\frac{1}{p}\left(1+\sum_{j=1}^{p-1}\frac{1}{q}\right)\right)^{N}=\left(\frac{p+q-1}{pq}\right)^{N} \] Q.E.D. \vphantom{} Here, then, is the desired theorem: \begin{thm} \label{thm:L^1 convergence of Chi_H,N minus Chi_H,N twiddle}Let $H$ be a contracting semi-basic $p$-Hydra map. Then: \begin{equation} \lim_{N\rightarrow\infty}\int_{\mathbb{Z}_{p}}\left|\chi_{H,N}\left(\mathfrak{z}\right)-\tilde{\chi}_{H,N}\left(\mathfrak{z}\right)\right|_{q}d\mathfrak{z}\overset{\mathbb{R}}{=}0\label{eq:L^1 converges of the difference between Chi_H,N and Chi_H,N twiddle} \end{equation} \end{thm} Proof: Regardless of whether or not $\alpha_{H}\left(0\right)=1$, the co-primality of $p$ and $q$ along with an application of the \emph{ordinary} $q$-adic triangle inequality, (\ref{eq:Chi_H,N / Chi_H,N twiddle identity for alpha equals 1}) and (\ref{eq:Chi_H,N / Chi_H,N twiddle identity for alpha is not 1}) from \textbf{Theorem \ref{thm:Chi_H,N / Chi_H,N twiddle error}} both become: \begin{align*} \int_{\mathbb{Z}_{p}}\left|\chi_{H,N}\left(\mathfrak{z}\right)-\tilde{\chi}_{H,N}\left(\mathfrak{z}\right)\right|_{q}d\mathfrak{z} & \ll\int_{\mathbb{Z}_{p}}\left|\kappa_{H}\left(\left[\mathfrak{z}\right]_{p^{N}}\right)\right|_{q}d\mathfrak{z}+\int_{\mathbb{Z}_{p}}\left|\kappa_{H}\left(\left[\mathfrak{z}\right]_{p^{N-1}}\right)\right|_{q}d\mathfrak{z} \end{align*} By \textbf{Proposition \ref{prop:L^1_R norm of kapp_H,N}}, both of the terms in the upper bound go to $0$ as $N\rightarrow\infty$. Q.E.D. \subsection{\label{subsec:4.3.3 Quick-Approach-of}\index{quick approach}Quick Approach of $\chi_{H}$} In Subsection \ref{subsec:3.2.2 Truncations-=00003D000026-The}, we proved the \textbf{Square Root Lemma }(page \pageref{eq:Square Root Lemma}), which\textemdash recall\textemdash asserted that for any $\chi\in\tilde{C}\left(\mathbb{Z}_{p},K\right)$, any $\mathfrak{z}\in\mathbb{Z}_{p}^{\prime}$, and any $\mathfrak{c}\in K\backslash\chi\left(\mathbb{N}_{0}\right)$, the equality $\chi\left(\mathfrak{z}\right)=\mathfrak{c}$ held if and only if: \begin{equation} \liminf_{n\rightarrow\infty}\frac{\left|\chi\left(\left[\mathfrak{z}\right]_{p^{n}}\right)-\mathfrak{c}\right|_{q}}{\left|\nabla_{p^{n}}\left\{ \chi\right\} \left(\mathfrak{z}\right)\right|_{q}^{1/2}}<\infty \end{equation} With that in mind, we then said the pair $\left(\mathfrak{z},\mathfrak{c}\right)$ was quickly (resp. slowly) approached by $\chi$ whenever the above $\liminf$ was $0$ (resp. $\in\left(0,\infty\right)$). Below, we prove that the pair $\left(\mathfrak{z},\chi_{H}\left(\mathfrak{z}\right)\right)$ is approached quickly by $\chi_{H}$ for all $\mathfrak{z}\in\mathbb{Z}_{p}$ whenever $H$ is a contracting, semi-basic $p$-Hydra map which fixes $0$. \begin{thm}[\textbf{\textit{Quick Approach of $\chi_{H}$ on $\mathbb{Z}_{p}^{\prime}$}}] Let $H$ be a contracting, semi-basic $p$-Hydra map which fixes $0$, and write $q=q_{H}$. Then: \begin{equation} \lim_{n\rightarrow\infty}\frac{\left|\chi_{H}\left(\left[\mathfrak{z}\right]_{p^{n}}\right)-\chi_{H}\left(\mathfrak{z}\right)\right|_{q}}{\left|M_{H}\left(\left[\mathfrak{z}\right]_{p^{n}}\right)\right|_{q}^{1/2}}=0,\textrm{ }\forall\mathfrak{z}\in\mathbb{Z}_{p}^{\prime}\label{eq:Quickness of a p-Hydra map on Z_p prime} \end{equation} \end{thm} Proof: Suppose that $\alpha_{H}\left(0\right)=1$, and fix $\mathfrak{z}\in\mathbb{Z}_{p}^{\prime}$. We wish to investigate $\chi_{H}\left(\mathfrak{z}\right)-\chi_{H}\left(\left[\mathfrak{z}\right]_{p^{N}}\right)$, or\textemdash which is the same\textemdash $\chi_{H}\left(\mathfrak{z}\right)-\chi_{H,N}\left(\mathfrak{z}\right)$. To do this, we write: \begin{equation} \chi_{H}\left(\mathfrak{z}\right)-\chi_{H,N}\left(\mathfrak{z}\right)=\left(\chi_{H}\left(\mathfrak{z}\right)-\tilde{\chi}_{H,N}\left(\mathfrak{z}\right)\right)-\left(\chi_{H,N}\left(\mathfrak{z}\right)-\tilde{\chi}_{H,N}\left(\mathfrak{z}\right)\right) \end{equation} Subtracting \textbf{Corollary \ref{cor:Chi_H,N twiddle explicit formula, arbitrary p, arbitrary alpha}}'s formula for $\tilde{\chi}_{H,N}\left(\mathfrak{z}\right)$) from \textbf{Corollary \ref{cor:F-series for Chi_H, arbitrary p and alpha}}'s $\mathcal{F}$-series formula for $\chi_{H}\left(\mathfrak{z}\right)$) gives: \begin{align*} \chi_{H}\left(\mathfrak{z}\right)-\tilde{\chi}_{H,N}\left(\mathfrak{z}\right) & \overset{\mathcal{F}_{p,q_{H}}}{=}\beta_{H}\left(0\right)N\left(\frac{\mu_{0}}{p}\right)^{N}\kappa_{H}\left(\left[\mathfrak{z}\right]_{p^{N}}\right)\\ & +\beta_{H}\left(0\right)\sum_{n=N}^{\infty}\left(\frac{\mu_{0}}{p}\right)^{n}\kappa_{H}\left(\left[\mathfrak{z}\right]_{p^{n}}\right)\\ & +\sum_{n=N}^{\infty}\left(\sum_{j=1}^{p-1}\beta_{H}\left(\frac{j}{p}\right)\left(\varepsilon_{n}\left(\mathfrak{z}\right)\right)^{j}\right)\left(\frac{\mu_{0}}{p}\right)^{n}\kappa_{H}\left(\left[\mathfrak{z}\right]_{p^{n}}\right) \end{align*} Since $\mathfrak{z}\in\mathbb{Z}_{p}^{\prime}$, the topology of convergence as $N\rightarrow\infty$ is the $q$-adic topology, and as such, estimating with the $q$-adic absolute value gives: \begin{equation} \left|\chi_{H}\left(\mathfrak{z}\right)-\tilde{\chi}_{H,N}\left(\mathfrak{z}\right)\right|_{q}\ll\left|\kappa_{H}\left(\left[\mathfrak{z}\right]_{p^{N}}\right)\right|_{q} \end{equation} On the other hand, because $\alpha_{H}\left(0\right)=1$,\textbf{ Theorem \ref{thm:Chi_H,N / Chi_H,N twiddle error}} tells us that: \begin{equation} \chi_{H,N}\left(\mathfrak{z}\right)-\tilde{\chi}_{H,N}\left(\mathfrak{z}\right)=\beta_{H}\left(0\right)\left(\frac{\mu_{0}}{p}\right)^{N-1}\left(\frac{2\mu_{0}N}{p}\kappa_{H}\left(\left[\mathfrak{z}\right]_{p^{N}}\right)-\left(N-1\right)\kappa_{H}\left(\left[\mathfrak{z}\right]_{p^{N-1}}\right)\right) \end{equation} and so: \begin{equation} \left|\chi_{H,N}\left(\mathfrak{z}\right)-\tilde{\chi}_{H,N}\left(\mathfrak{z}\right)\right|_{q}\ll\left|\kappa_{H}\left(\left[\mathfrak{z}\right]_{p^{N-1}}\right)\right|_{q} \end{equation} Since $\mathfrak{z}\in\mathbb{Z}_{p}^{\prime}$, $\left|\kappa_{H}\left(\left[\mathfrak{z}\right]_{p^{N-1}}\right)\right|_{q}\geq\left|\kappa_{H}\left(\left[\mathfrak{z}\right]_{p^{N}}\right)\right|_{q}$, which leaves us with: \[ \max\left\{ \left|\chi_{H}\left(\mathfrak{z}\right)-\tilde{\chi}_{H,N}\left(\mathfrak{z}\right)\right|_{q},\left|\chi_{H,N}\left(\mathfrak{z}\right)-\tilde{\chi}_{H,N}\left(\mathfrak{z}\right)\right|_{q}\right\} \ll\left|\kappa_{H}\left(\left[\mathfrak{z}\right]_{p^{N-1}}\right)\right|_{q} \] Hence, by the ultrametric inequality: \begin{equation} \left|\chi_{H}\left(\mathfrak{z}\right)-\chi_{H,N}\left(\mathfrak{z}\right)\right|_{q}\ll\left|\kappa_{H}\left(\left[\mathfrak{z}\right]_{p^{N-1}}\right)\right|_{q} \end{equation} Since: \begin{equation} \kappa_{H}\left(m\right)=M_{H}\left(m\right)\left(\frac{\mu_{0}}{p}\right)^{-\lambda_{p}\left(m\right)} \end{equation} we have that: \begin{equation} \left|\kappa_{H}\left(m\right)\right|_{q}=\left|M_{H}\left(m\right)\left(\frac{\mu_{0}}{p}\right)^{-\lambda_{p}\left(m\right)}\right|_{q}=\left|M_{H}\left(m\right)\right|_{q} \end{equation} and so: \begin{equation} \left|\chi_{H}\left(\mathfrak{z}\right)-\chi_{H,N}\left(\mathfrak{z}\right)\right|_{q}\ll\left|M_{H}\left(\left[\mathfrak{z}\right]_{p^{N-1}}\right)\right|_{q} \end{equation} Dividing by $\left|M_{H}\left(\left[\mathfrak{z}\right]_{p^{N}}\right)\right|_{q}^{1/2}$ gives: \begin{equation} \frac{\left|\chi_{H}\left(\mathfrak{z}\right)-\chi_{H,N}\left(\mathfrak{z}\right)\right|_{q}}{\left|M_{H}\left(\left[\mathfrak{z}\right]_{p^{N}}\right)\right|_{q}^{1/2}}\ll\frac{\left|M_{H}\left(\left[\mathfrak{z}\right]_{p^{N-1}}\right)\right|_{q}}{\left|M_{H}\left(\left[\mathfrak{z}\right]_{p^{N}}\right)\right|_{q}^{1/2}} \end{equation} Since the string of $p$-adic digits of $\left[\mathfrak{z}\right]_{p^{N}}$ is either equal to that of $\left[\mathfrak{z}\right]_{p^{N-1}}$ or differs by a single digit on the right, $M_{H}$'s functional equations (\textbf{Proposition \ref{prop:M_H concatenation identity}}) tell us that: \begin{equation} M_{H}\left(\left[\mathfrak{z}\right]_{p^{N}}\right)=\begin{cases} M_{H}\left(\left[\mathfrak{z}\right]_{p^{N-1}}\right) & \textrm{if }\left[\mathfrak{z}\right]_{p^{N-1}}=\left[\mathfrak{z}\right]_{p^{N}}\\ M_{H}\left(\left[\mathfrak{z}\right]_{p^{N-1}}\right)\cdot M_{H}\left(\textrm{coefficient of }p^{N}\textrm{ in }\mathfrak{z}\right) & \textrm{if }\left[\mathfrak{z}\right]_{p^{N-1}}\neq\left[\mathfrak{z}\right]_{p^{N}} \end{cases} \end{equation} Seeing $\left[\mathfrak{z}\right]_{p^{N-1}}\neq\left[\mathfrak{z}\right]_{p^{N}}$ occurs if and only if the coefficient of $p^{N}$ in $\mathfrak{z}$ is non-zero, the semi-basicness of $H$ then guarantees that: \begin{equation} \left|M_{H}\left(\textrm{coefficient of }p^{N}\textrm{ in }\mathfrak{z}\right)\right|_{p}\leq\max_{1\leq j\leq p-1}\left|M_{H}\left(j\right)\right|_{p}=p^{-\nu} \end{equation} occurs for some integer constant $\nu\geq1$. Hence: \begin{equation} \left|M_{H}\left(\left[\mathfrak{z}\right]_{p^{N}}\right)\right|_{q}\geq p^{-\nu}\left|M_{H}\left(\left[\mathfrak{z}\right]_{p^{N-1}}\right)\right|_{q} \end{equation} and so: \begin{align*} \frac{\left|\chi_{H}\left(\mathfrak{z}\right)-\chi_{H,N}\left(\mathfrak{z}\right)\right|_{q}}{\left|M_{H}\left(\left[\mathfrak{z}\right]_{p^{N}}\right)\right|_{q}^{1/2}} & \ll\frac{\left|M_{H}\left(\left[\mathfrak{z}\right]_{p^{N-1}}\right)\right|_{q}}{\left|M_{H}\left(\left[\mathfrak{z}\right]_{p^{N}}\right)\right|_{q}^{1/2}}\\ & \leq\frac{\left|M_{H}\left(\left[\mathfrak{z}\right]_{p^{N-1}}\right)\right|_{q}}{p^{-\frac{\nu}{2}}\left|M_{H}\left(\left[\mathfrak{z}\right]_{p^{N-1}}\right)\right|_{q}^{1/2}}\\ & =p^{\nu/2}\left|M_{H}\left(\left[\mathfrak{z}\right]_{p^{N-1}}\right)\right|_{q}^{1/2} \end{align*} which tends to $0$ as $N\rightarrow\infty$, since $\mathfrak{z}\in\mathbb{Z}_{p}^{\prime}$. The case where $\alpha_{H}\left(0\right)\neq1$ works in almost exactly the same way. Q.E.D. \subsection{\label{subsec:4.3.4 Archimedean-Estimates}Archimedean Estimates} As we discussed in Chapter 1, mixing up convergence in $\mathbb{Q}_{p}$ or $\mathbb{C}_{p}$ with convergence in $\mathbb{R}$ or $\mathbb{C}$ is, quite literally, the oldest mistake in the big book of $p$-adic analysis. As a rule, mixing topologies in this way is fraught with danger. So far, however, the formalism of frames has been a helpful guard rail, keeping us on track. However, in this subsection, we are going to simply dive off the edge of the cliff head-first. Let $\hat{\eta}:\hat{\mathbb{Z}}_{p}\rightarrow\overline{\mathbb{Q}}$ be an element of $c_{0}\left(\hat{\mathbb{Z}}_{p},\mathbb{C}_{q}\right)$, so that the $\left(p,q\right)$-adic Fourier series: \begin{equation} \eta\left(\mathfrak{z}\right)=\sum_{t\in\hat{\mathbb{Z}}_{p}}\hat{\eta}\left(t\right)e^{2\pi i\left\{ t\mathfrak{z}\right\} _{p}}\label{eq:Getting Ready to Jump off the Cliff} \end{equation} converges in $\mathbb{C}_{q}$ uniformly with respect to $\mathfrak{z}\in\mathbb{Z}_{p}$. In particular, for any $\mathfrak{z}$, the ultrametric structure of $\mathbb{C}_{q}$ allows us to freely group and rearrange the terms of (\ref{eq:Getting Ready to Jump off the Cliff}) however we please without affecting the convergence. Because every term of the series is, technically, an element of $\overline{\mathbb{Q}}$, note that the $N$th partial sums of this series are perfectly well-defined\emph{ }complex-valued functions (complex, not $q$-adic complex) on $\mathbb{Z}_{p}$. Moreover, it may just so happen that there are $\mathfrak{z}_{0}\in\mathbb{Z}_{p}$ so that the partial sums: \begin{equation} \tilde{\eta}_{N}\left(\mathfrak{z}_{0}\right)=\sum_{\left|t\right|_{p}\leq p^{N}}\hat{\eta}\left(t\right)e^{2\pi i\left\{ t\mathfrak{z}_{0}\right\} _{p}}\label{eq:Jumping off the Cliff} \end{equation} converge to a limit in $\mathbb{C}$ as $N\rightarrow\infty$, in addition to the known limit to which they converge in $\mathbb{C}_{q}$. As we saw when discussing Hensel's error, \textbf{\emph{we cannot, in general, assume that the limit of }}(\ref{eq:Jumping off the Cliff})\textbf{\emph{ in $\mathbb{C}$ has, in any way, shape, or form, a meaningful relation to its limit in $\mathbb{C}_{q}$}}\textemdash \textbf{except}, of course, when (\ref{eq:Jumping off the Cliff}) happens to be a \textbf{\emph{geometric series}}, thanks to the \emph{universality }of the geometric series\index{geometric series universality} (page \pageref{fact:Geometric series universality}). Specifically, for any $\mathfrak{z}_{0}$, we can characterize the limiting behavior in $\mathbb{C}$ (not $\mathbb{C}_{q}$!) of (\ref{eq:Jumping off the Cliff}) as $N\rightarrow\infty$ like so: \begin{fact} Let $\mathfrak{z}_{0}\in\mathbb{Z}_{p}$. Then, exactly one of the following occurs for \emph{(\ref{eq:Jumping off the Cliff})} as $N\rightarrow\infty$: \vphantom{} I. \emph{(\ref{eq:Jumping off the Cliff})} fails to converge to a limit in $\mathbb{C}$. \vphantom{} II. (\emph{\ref{eq:Jumping off the Cliff})} converges to a limit in $\mathbb{C}$, and \emph{(\ref{eq:Getting Ready to Jump off the Cliff})}\textbf{ }\textbf{\emph{can}} be rearranged into a geometric series of the form $\sum_{n=0}^{\infty}c_{n}r^{n}$ for $c_{n},r\in\overline{\mathbb{Q}}$, where $r$ has archimedean absolute value $<1$. \vphantom{} III. \emph{(\ref{eq:Jumping off the Cliff})} converges to a limit in $\mathbb{C}$, but \emph{(\ref{eq:Getting Ready to Jump off the Cliff})}\textbf{ }\textbf{\emph{cannot}} be rearranged into a geometric series of the form $\sum_{n=0}^{\infty}c_{n}r^{n}$ for $c_{n},r\in\overline{\mathbb{Q}}$, where $r$ has archimedean absolute value $<1$. \end{fact} \vphantom{} By the universality of the geometric series, (II) is \emph{precisely }the case where we \emph{can }conclude that the limit of (\ref{eq:Jumping off the Cliff}) in $\mathbb{C}$ as $N\rightarrow\infty$ is the same number as the limit of (\ref{eq:Jumping off the Cliff}) in $\mathbb{C}_{q}$ as $N\rightarrow\infty$. For any $\mathfrak{z}$ satisfying (II), the value of (\ref{eq:Getting Ready to Jump off the Cliff}) is the same regardless of the topology (archimedean or $q$-adic) we use to make sense of it. This then leads to the following observation: let $U$ be the set of all $\mathfrak{z}\in\mathbb{Z}_{p}$ for which either (II) or (III) holds true. Then, we can define a function $\eta^{\prime}:U\rightarrow\mathbb{C}$ by the limit of (\ref{eq:Jumping off the Cliff}) in $\mathbb{C}$ (not $\mathbb{C}_{q}$!). For any $\mathfrak{z}_{0}$ in $U$ satisfying (III), there is no reason for the complex number $\eta^{\prime}\left(\mathfrak{z}_{0}\right)$ to have anything to do with the $q$-adic complex number $\eta\left(\mathfrak{z}_{0}\right)$ defined by the limit of (\ref{eq:Jumping off the Cliff}) in $\mathbb{C}_{q}$. However$\ldots$ for $\mathfrak{z}_{0}\in U$ satisfying (II), the universality of the geometric series \emph{guarantees} that $\eta^{\prime}\left(\mathfrak{z}_{0}\right)=\eta\left(\mathfrak{z}_{0}\right)$, where the equality holds \emph{simultaneously }in $\mathbb{C}$ and in $\mathbb{C}_{q}$. The general implementation of this idea is as following. First\textemdash of course\textemdash some terminology: \begin{notation} We write $L^{1}\left(\hat{\mathbb{Z}}_{p},\mathbb{C}\right)$ to denote the Banach space of all complex-valued functions (not $q$-adic complex valued!) $\hat{\eta}:\hat{\mathbb{Z}}_{p}\rightarrow\mathbb{C}$ with finite $L^{1}$ norm: \begin{equation} \left\Vert \hat{\eta}\right\Vert _{L^{1}}\overset{\textrm{def}}{=}\sum_{t\in\hat{\mathbb{Z}}_{p}}\left|\hat{\eta}\left(t\right)\right|\label{eq:Definition of complex L^1 norm on Z_p hat} \end{equation} For $r\in\left[1,2\right]$, we write $L^{r}\left(\mathbb{Z}_{p},\mathbb{C}\right)$ to denote the Banach space of all complex-valued functions (not $q$-adic complex valued!) $\eta:\mathbb{Z}_{p}\rightarrow\mathbb{C}$ with finite $L^{r}$ norm: \begin{equation} \left\Vert \eta\right\Vert _{L^{r}}\overset{\textrm{def}}{=}\left(\int_{\mathbb{Z}_{p}}\left|\eta\left(\mathfrak{z}\right)\right|^{r}d\mathfrak{z}\right)^{1/r}\label{eq:Definition of complex L^r norm on Z_p} \end{equation} \end{notation} \begin{defn} Let $\chi\in\tilde{C}$$\left(\mathbb{Z}_{p},\mathbb{C}_{q}\right)$, and suppose that $\chi\left(\mathbb{N}_{0}\right)\subseteq\overline{\mathbb{Q}}$ (so that every van der Put coefficient of $\chi$ is in $\overline{\mathbb{Q}}$). Let $\mathfrak{z}_{0}\in\mathbb{Z}_{p}$. \vphantom{} I. We\index{double convergence} say $\chi$ is \textbf{doubly convergent }at $\mathfrak{z}_{0}$ whenever the van der Put series $S_{p}\left\{ \chi\right\} \left(\mathfrak{z}_{0}\right)$ can be rearranged into a geometric series of the form $\sum_{n=0}^{\infty}c_{n}r^{n}$ where $r$ and the $c_{n}$s are elements of $\overline{\mathbb{Q}}$ so that both the archimedean and $q$-adic absolute values of $r$ are $<1$. We call $\mathfrak{z}_{0}$ a \textbf{double convergence point }of $\chi$ whenever $\chi$ is doubly convergent at $\mathfrak{z}_{0}$. \vphantom{} II. We say $\chi$ is \textbf{spurious }at $\mathfrak{z}_{0}$ whenever $S_{p}\left\{ \chi\right\} \left(\mathfrak{z}_{0}\right)$ converges in $\mathbb{C}$, yet cannot be rearranged into a geometric series which is $q$-adically convergent. Likewise, we call $\mathfrak{z}_{0}$ a \textbf{spurious point }of $\chi$ whenever $\chi$ is spurious at $\mathfrak{z}_{0}$. \vphantom{} III. We say $\chi$ is \textbf{mixed }at $\mathfrak{z}_{0}$ whenever $\chi$ is either doubly convergent or spurious at $\mathfrak{z}_{0}$. Likewise, we call $\mathfrak{z}_{0}$ a \textbf{mixed point }of $\chi$ whenever $\chi$ is either doubly convergent or spurious at $\mathfrak{z}_{0}$. \end{defn} \begin{rem} Observe that the complex number value attained by $S_{p}\left\{ \chi\right\} \left(\mathfrak{z}_{0}\right)$ at a mixed point $\mathfrak{z}_{0}$ is equal to the $q$-adic value attained by $\chi$ at $\mathfrak{z}_{0}$ \emph{if and only if} $\mathfrak{z}_{0}$ is a point of double convergence. \end{rem} \vphantom{} The next proposition shows that, however audacious this approach might be, it is nevertheless sound in mind and body. \begin{prop} \label{prop:measurability proposition}$\chi\in\tilde{C}\left(\mathbb{Z}_{p},\mathbb{C}_{q}\right)$, suppose that $\chi\left(\mathbb{N}_{0}\right)\subseteq\overline{\mathbb{Q}}$, and let $\phi:\mathbb{Z}_{p}\rightarrow\mathbb{Z}_{p}$ be a function which is measurable with respect to the \textbf{\emph{real-valued}}\emph{ }Haar probability measure on $\mathbb{Z}_{p}$ so that every $\mathfrak{z}\in\phi\left(\mathbb{Z}_{p}\right)$ is a mixed point of $\chi$. Then, the function $\eta:\mathbb{Z}_{p}\rightarrow\mathbb{C}$ given by: \begin{equation} \eta\left(\mathfrak{z}\right)\overset{\textrm{def}}{=}\lim_{N\rightarrow\infty}S_{p,N}\left\{ \chi\right\} \left(\phi\left(\mathfrak{z}\right)\right) \end{equation} is measurable with respect to the \textbf{\emph{real-valued}}\emph{ }Haar probability measure on $\mathbb{Z}_{p}$. Moreover, we have that $\eta\left(\mathfrak{z}\right)=\chi\left(\phi\left(\mathfrak{z}\right)\right)$ for all $\mathfrak{z}$ for which $\chi$ is doubly convergent at $\phi\left(\mathfrak{z}\right)$. \end{prop} Proof: Let $\chi$, $\phi$, and $\eta$ be as given. Then, for each $N$, the $N$th partial van der Put series $S_{p,N}\left\{ \chi\right\} \left(\mathfrak{z}\right)$ is a locally-constant $\overline{\mathbb{Q}}$-valued function on $\mathbb{Z}_{p}$, and, as such, is a \emph{continuous }$\overline{\mathbb{Q}}$-valued function on $\mathbb{Z}_{p}$. Since $\phi$ is measurable, this means that $\mathfrak{z}\mapsto S_{p,N}\left\{ \chi\right\} \left(\phi\left(\mathfrak{z}\right)\right)$ is a measurable $\overline{\mathbb{Q}}$-valued function on $\mathbb{Z}_{p}$. Since $\phi\left(\mathfrak{z}\right)$ is a mixed point of $\chi$ for every $\mathfrak{z}$, the van der Put series $S_{p,N}\left\{ \chi\right\} \left(\phi\left(\mathfrak{z}\right)\right)$ converge point-wise in $\mathbb{C}$ to limit, and this limit is, by definition, $\eta\left(\mathfrak{z}\right)$. Since $\eta$ is the point-wise limit of a sequence of measurable complex-valued functions on $\mathbb{Z}_{p}$, $\eta$ itself is then measurable. Finally, note that for any $\mathfrak{z}$ for which $\phi\left(\mathfrak{z}\right)$ is a double convergent point of $\chi$, the limit of $S_{p,N}\left\{ \chi\right\} \left(\phi\left(\mathfrak{z}\right)\right)$ is a geometric series which converges simultaneously in $\mathbb{C}$ and $\mathbb{C}_{q}$. The universality of the geometric series then guarantees that the limit of the series in $\mathbb{C}$ is equal to the limit of the series in $\mathbb{C}_{q}$. Since $\chi$ is rising-continuous, the $N$th partial van der Put series $S_{p,N}\left\{ \chi\right\} \left(\phi\left(\mathfrak{z}\right)\right)$ converges in $\mathbb{C}_{q}$ to $\chi\left(\phi\left(\mathfrak{z}\right)\right)$ for all $\mathfrak{z}$. So, $\eta\left(\mathfrak{z}\right)=\chi\left(\phi\left(\mathfrak{z}\right)\right)$ for all $\mathfrak{z}$ for which $\phi\left(\mathfrak{z}\right)$ is a double convergent point of $\chi$. Q.E.D. \begin{thm}[\textbf{$L^{1}$ Method for Archimedean Estimates}] \label{thm:The L^1 method} Let\index{$L^{1}$ method} $\chi\in\tilde{C}$$\left(\mathbb{Z}_{p},\mathbb{C}_{q}\right)$, suppose that $\chi\left(\mathbb{N}_{0}\right)\subseteq\overline{\mathbb{Q}}$, and let $\phi:\mathbb{Z}_{p}\rightarrow\mathbb{Z}_{p}$ be a function which is measurable with respect to the \textbf{\emph{real-valued}}\emph{ }Haar probability measure on $\mathbb{Z}_{p}$ so that: \vphantom{} I. Every $\mathfrak{z}\in\phi\left(\mathbb{Z}_{p}\right)$ is a mixed point of $\chi$. \vphantom{} II. The measurable function $\eta:\mathbb{Z}_{p}\rightarrow\mathbb{C}$ defined by: \begin{equation} \eta\left(\mathfrak{z}\right)\overset{\textrm{def}}{=}\lim_{N\rightarrow\infty}S_{p,N}\left\{ \chi\right\} \left(\phi\left(\mathfrak{z}\right)\right) \end{equation} is in $L^{r}\left(\mathbb{Z}_{p},\mathbb{C}_{q}\right)$ for some $r\in\left[1,2\right]$, so that the Fourier transform $\hat{\eta}:\hat{\mathbb{Z}}_{p}\rightarrow\mathbb{C}$: \begin{equation} \hat{\eta}\left(t\right)\overset{\textrm{def}}{=}\int_{\mathbb{Z}_{p}}\eta\left(\mathfrak{z}\right)e^{2\pi i\left\{ t\mathfrak{z}\right\} _{p}}d\mathfrak{z} \end{equation} is an element of $L^{1}\left(\mathbb{\hat{Z}}_{p},\mathbb{C}_{q}\right)$. Then: \begin{equation} \left|\chi\left(\phi\left(\mathfrak{z}\right)\right)\right|\leq\left\Vert \hat{\eta}\right\Vert _{L^{1}}\label{eq:The L^1 Method} \end{equation} for all $\mathfrak{z}\in\mathbb{Z}_{p}$ so that $\chi$ is doubly convergent at $\phi\left(\mathfrak{z}\right)$. \end{thm} \begin{rem} For any $\mathfrak{z}\in\mathbb{Z}_{p}$ so that $\chi$ is doubly convergent at $\phi\left(\mathfrak{z}\right)$, it necessarily follows that the value of $\chi\left(\mathfrak{y}\right)$ at $\mathfrak{y}=\phi\left(\mathfrak{z}\right)$ is then an element of $\mathbb{C}_{q}\cap\overline{\mathbb{Q}}$. As such, the $L^{1}$ method allows us to extract \emph{archimedean upper bounds }on outputs of $\chi$ that happen to be both $q$-adic complex numbers and ordinary algebraic numbers. \end{rem} Proof: Let everything be as given. By \textbf{Proposition \ref{prop:measurability proposition}}, $\eta$ is measurable. It is a well-known fact that the Fourier transform of $\eta$ (a complex-valued function on the compact abelian group $\mathbb{Z}_{p}$) is defined whenever $\eta$ is in $L^{r}\left(\mathbb{Z}_{p},\mathbb{C}\right)$ for some $r\in\left[1,2\right]$ (see \cite{Folland - harmonic analysis}, for example). Consequently, the hypothesis that $\hat{\eta}$ be in $L^{1}$ tells us that $\eta\left(\mathfrak{z}\right)$'s Fourier series is absolutely convergent, and hence, that: \begin{equation} \left|\eta\left(\mathfrak{z}\right)\right|=\left|\sum_{t\in\hat{\mathbb{Z}}_{p}}\hat{\eta}\left(t\right)e^{2\pi i\left\{ t\mathfrak{z}\right\} _{p}}\right|\leq\sum_{t\in\hat{\mathbb{Z}}_{p}}\left|\hat{\eta}\left(t\right)\right|=\left\Vert \hat{\eta}\right\Vert _{L^{1}} \end{equation} Since $\eta\left(\mathfrak{z}\right)=\chi\left(\phi\left(\mathfrak{z}\right)\right)$ for all $\mathfrak{z}\in\mathbb{Z}_{p}$ for which $\chi$ is doubly convergent at $\phi\left(\mathfrak{z}\right)$, the result follows. Q.E.D. \vphantom{} Originally, I employed this method to obtain archimedean bounds on $\chi_{q}\left(\mathfrak{z}\right)$ for odd $q\geq3$. However, the bounds were not very good, in that my choice of $\phi$ was relatively poor\textemdash the exact meaning of this will be clear in a moment. Like with most of my original work in this dissertation, the bounds followed only after a lengthy, involved Fourier analytic computation. As such, rather than continue to draw out this three-hundred-plus-page-long behemoth, I hope the reader will forgive me for merely sketching the argument. The idea for $\phi$ comes from how the $2$-adic digits of a given $\mathfrak{z}\in\mathbb{Z}_{2}$ affect the value of $\chi_{q}\left(\mathfrak{z}\right)$. Consider, for instance the explicit formula for $\chi_{3}\left(\mathfrak{z}\right)$ (equation (\ref{eq:Explicit formula for Chi_3})): \begin{equation} \chi_{3}\left(\mathfrak{z}\right)\overset{\mathcal{F}_{2,3}}{=}-\frac{1}{2}+\frac{1}{4}\sum_{n=0}^{\infty}\frac{3^{\#_{1}\left(\left[\mathfrak{z}\right]_{2^{n}}\right)}}{2^{n}},\textrm{ }\forall\mathfrak{z}\in\mathbb{Z}_{2} \end{equation} For fixed $\mathfrak{z}\in\mathbb{Z}_{2}^{\prime}$, as $n$ increases to $n+1$, $3^{\#_{1}\left(\left[\mathfrak{z}\right]_{2^{n}}\right)}$ will increase only if the coefficient of $2^{n}$ in the $2$-adic expansion of $\mathfrak{z}$ is $1$. If that coefficient is $0$, the denominator $2^{n}$ in the geometric series will increase, but the numerator will not increase. A simple analysis shows that, for $\chi_{3}$, we can guarantee that the series will become a convergent geometric series for $\mathfrak{z}\in\mathbb{Q}\cap\mathbb{Z}_{2}^{\prime}$ by requiring that any two $1$s in the $2$-adic digits of $\mathfrak{z}$ are separated by at least one $0$. There is, in fact, a function $\phi:\mathbb{Z}_{2}\rightarrow\mathbb{Z}_{2}$ which can guarantee this for us, and\textemdash amusingly\textemdash we have actually already encountered this particular $\phi$ before: it was the function from \textbf{Example \ref{exa:p-adic differentiation is crazy}} (page \pageref{exa:p-adic differentiation is crazy}) which altered the $2$-adic expansion of a given $2$-adic integer like so: \begin{equation} \phi_{3}\left(\sum_{n=0}^{\infty}c_{n}2^{n}\right)\overset{\textrm{def}}{=}\sum_{n=0}^{\infty}c_{n}2^{2n}\label{eq:Definition of Phi_2} \end{equation} As we saw in Subsection \ref{subsec:3.1.1 Some-Historical-and}, $\phi_{2}$ is injective, continuous (and hence, measurable), differentiable, and in fact \emph{continuously} differentiable, and its derivative is identically $0$. Observe that for a $\mathfrak{z}$ with a $2$-adic digit sequence of $\mathfrak{z}=\centerdot c_{0}c_{1}c_{2}\ldots$, we have: \begin{align*} \phi_{2}\left(\mathfrak{z}\right) & =c_{0}+c_{1}2^{2}+c_{2}2^{4}+\cdots\\ & =c_{0}+0\cdot2^{1}+c_{1}2^{2}+0\cdot2^{3}+c_{2}2^{4}+\cdots\\ & =\centerdot c_{0}0c_{1}0c_{2}0\ldots \end{align*} Thus, $\phi_{2}\left(\mathfrak{z}\right)$ is a mixed point of $\chi_{3}$ for all $\mathfrak{z}\in\mathbb{Z}_{2}$. To use the $L^{1}$ method, like with nearly everything else in this dissertation, we once again appeal to functional equations\index{functional equation}. Specifically: \begin{prop} Let $d$ be an integer $\geq2$, and let $\phi_{d}:\mathbb{Z}_{p}\rightarrow\mathbb{Z}_{p}$ be defined by: \begin{equation} \phi_{d}\left(\sum_{n=0}^{\infty}c_{n}p^{n}\right)\overset{\textrm{def}}{=}\sum_{n=0}^{\infty}c_{n}p^{dn}\label{eq:Definition of phi_d} \end{equation} for all $p$-adic integers $\sum_{n=0}^{\infty}c_{n}p^{n}$. Then: \begin{equation} \phi_{d}\left(p\mathfrak{z}+j\right)=p^{d}\phi_{d}\left(\mathfrak{z}\right)+j,\textrm{ }\forall\mathfrak{z}\in\mathbb{Z}_{p},\textrm{ }\forall j\in\mathbb{Z}/p\mathbb{Z}\label{eq:phi_d functional equations} \end{equation} \end{prop} Proof: Let $j\in\left\{ 0,\ldots,p-1\right\} $, and let $\mathfrak{z}=\sum_{n=0}^{\infty}c_{n}p^{n}$. Then: \begin{align*} \phi_{d}\left(p\mathfrak{z}+j\right) & =\phi_{d}\left(j+\sum_{n=0}^{\infty}c_{n}p^{n+1}\right)\\ & =j+\sum_{n=0}^{\infty}c_{n}p^{d\left(n+1\right)}\\ & =j+p^{d}\sum_{n=0}^{\infty}c_{n}p^{dn}\\ & =j+p^{d}\phi_{d}\left(\mathfrak{z}\right) \end{align*} Q.E.D. \vphantom{} The $\phi_{d}$s are just one example of a possible choice of $\phi$ for which we can apply the $L^{1}$ method. The goal is to choose a $\phi$ satisfying desirable functional equations like these which will then allow us to exploit $\chi_{H}$'s functional equation to show that $\eta=\chi\circ\phi$ satisfies a functional equation as well. This will allow us to recursively solve for an explicit formula of $\hat{\text{\ensuremath{\eta}}}$ whose $L^{1}$ convergence can then be proven by direct estimation. \begin{example}[\textbf{A sample application of the $L^{1}$ method to $\chi_{3}$}] \label{exa:L^1 method example}Let $\eta=\chi_{3}\circ\phi_{2}$. Then, combining the functional equations of $\chi_{3}$ and $\phi_{2}$ yields: \begin{equation} \eta\left(2\mathfrak{z}\right)=\frac{1}{4}\eta\left(\mathfrak{z}\right) \end{equation} \begin{equation} \eta\left(2\mathfrak{z}+1\right)=\frac{3\eta\left(\mathfrak{z}\right)+2}{4} \end{equation} Since $\eta$ will be integrable over $\mathbb{Z}_{2}$ whenever $\hat{\eta}$ is in $L^{1}$, we can proceed by formally computing $\hat{\eta}$ and then showing that it is in $L^{1}$. Formally: \begin{align*} \hat{\eta}\left(t\right) & \overset{\mathbb{C}}{=}\int_{\mathbb{Z}_{2}}\eta\left(\mathfrak{z}\right)e^{-2\pi i\left\{ t\mathfrak{z}\right\} _{2}}d\mathfrak{z}\\ & =\int_{2\mathbb{Z}_{2}}\eta\left(\mathfrak{z}\right)e^{-2\pi i\left\{ t\mathfrak{z}\right\} _{2}}d\mathfrak{z}+\int_{2\mathbb{Z}_{2}+1}\eta\left(\mathfrak{z}\right)e^{-2\pi i\left\{ t\mathfrak{z}\right\} _{2}}d\mathfrak{z}\\ \left(\mathfrak{y}=\frac{\mathfrak{z}}{2},\frac{\mathfrak{z}-1}{2}\right); & =\frac{1}{2}\int_{\mathbb{Z}_{2}}\eta\left(2\mathfrak{y}\right)e^{-2\pi i\left\{ 2t\mathfrak{y}\right\} _{2}}d\mathfrak{y}+\frac{1}{2}\int_{\mathbb{Z}_{2}}\eta\left(2\mathfrak{y}+1\right)e^{-2\pi i\left\{ t\left(2\mathfrak{y}+1\right)\right\} _{2}}d\mathfrak{y}\\ & =\frac{1}{8}\int_{\mathbb{Z}_{2}}\eta\left(\mathfrak{y}\right)e^{-2\pi i\left\{ 2t\mathfrak{y}\right\} _{2}}d\mathfrak{y}+\frac{1}{8}\int_{\mathbb{Z}_{2}}\left(3\eta\left(\mathfrak{z}\right)+2\right)e^{-2\pi i\left\{ t\left(2\mathfrak{y}+1\right)\right\} _{2}}d\mathfrak{y}\\ & =\frac{1}{8}\hat{\eta}\left(2t\right)+\frac{3e^{-2\pi i\left\{ t\right\} _{2}}}{8}\hat{\eta}\left(2t\right)+\frac{e^{-2\pi i\left\{ t\right\} _{2}}}{4}\mathbf{1}_{0}\left(2t\right) \end{align*} and hence: \begin{equation} \hat{\eta}\left(t\right)\overset{\mathbb{C}}{=}\frac{1+3e^{-2\pi i\left\{ t\right\} _{2}}}{8}\hat{\eta}\left(2t\right)+\frac{e^{-2\pi i\left\{ t\right\} _{2}}}{4}\mathbf{1}_{0}\left(2t\right) \end{equation} Just like we did with $\hat{\chi}_{H,N}\left(t\right)$, nesting this formula will lead to an explicit formula for $\hat{\eta}\left(t\right)$, which can then be used to prove $\left\Vert \hat{\eta}\right\Vert _{L^{1}}<\infty$ and thereby obtain a bound on $\left|\chi_{3}\left(\phi_{2}\left(\mathfrak{z}\right)\right)\right|$ via \textbf{Theorem \ref{thm:The L^1 method}}. The reason why this particular choice of $\phi$ was relatively poor is because by inserting a $0$ between \emph{every }two consecutive $2$-adic digits of $\mathfrak{z}$, it forces us to consider $\mathfrak{z}$s associated to composition sequences of the branches of the Shortened Collatz Map where every application of $\frac{3x+1}{2}$ is followed by at least one application of $\frac{x}{2}$. By experimenting with more flexible choices of $\phi$ (say, those satisfying approximate functional equations / inequalities, rather than \emph{exact} ones), it may be possible to obtain non-trivial archimedean bounds on $\chi_{3}$\textemdash and, on $\chi_{H}$s in general. \end{example} \chapter{\label{chap:5 The-Multi-Dimensional-Case}The Multi-Dimensional Case} \includegraphics[scale=0.45]{./PhDDissertationEroica5.png} \vphantom{} As stated in the Introduction (Chapter 1), Chapter 5 is essentially a compactification of the contents of Chapters 2 and 3, presenting most of their contents as their occur in the context of multi-dimensional Hydra maps and the techniques of multi-variable $\left(p,q\right)$-adic analysis employed to study them. Section \ref{sec:5.1 Hydra-Maps-on} introduces multi-dimensional Hydra maps and the two equivalent contexts we use to study them: $\mathbb{Z}^{d}$ and $\mathcal{O}_{\mathbb{F}}$. \section{\label{sec:5.1 Hydra-Maps-on}Hydra Maps on Lattices and Number Rings} When attempting to generalize Hydra maps to ``higher-dimensional'' spaces, there are two possible approaches. The first is to view the higher-dimensional analogue of $\mathbb{Z}$ as being the ring of integers (a.k.a. \textbf{number ring}) of a given \textbf{number field }$\mathbb{F}$\textemdash a degree $d<\infty$ extension of $\mathbb{Q}$. As is traditional in algebraic number theory, we write $\mathcal{O}_{\mathbb{F}}$ to denote the number ring of integers of $\mathbb{F}$ (``$\mathbb{F}$-integers''). The second approach is to work with the lattice $\mathbb{Z}^{d}$ instead of $\mathcal{O}_{\mathbb{F}}$. In theory, these two approaches are ultimately equivalent, with $\mathcal{O}_{\mathbb{F}}$ being $\mathbb{Z}$-module-isomorphic to $\mathbb{Z}^{d}$ after making a choice of a basis. In practice, however, the way of number rings introduces an irksome complication by way of the \textbf{Structure Theorem for Finitely Generated Modules over a Principal Ideal Domain}. Before we begin, it is informative to consider how we might translate the one-dimensional case into the number ring setting. In the one-dimensional case, a $p$-Hydra map $H$ was obtained by stitching together a finite number of affine-linear maps $H_{0},\ldots,H_{p-1}$ on $\mathbb{Q}$ by defining $H\left(n\right)$ to be the image of the integer $n$ under $H_{j}$, where $j$ was the value of $n$ modulo $p$. The essential features of this construction were: \begin{itemize} \item $\mathbb{F}$, a number field; previously, this was $\mathbb{Q}$. \item $\mathcal{O}_{\mathbb{F}}$, the ring of integers of $\mathbb{F}$; previously, this was $\mathbb{Z}$. \item \nomenclature{$\mathfrak{I}$}{a non-zero proper ideal of $\mathcal{O}_{\mathbb{F}}$}A proper ideal\index{ideal!of mathcal{O}{mathbb{F}}@of $\mathcal{O}_{\mathbb{F}}$} of $\mathcal{O}_{\mathbb{F}}$; previously, this was $p\mathbb{Z}$. \item The different co-sets of $\mathfrak{I}$ in $\mathcal{O}_{\mathbb{F}}$; previously, these were the different equivalence classes modulo $p$. \item For each co-set of $\mathfrak{I}$, an affine linear map which sends elements of that particular co-set to some (possibly different) co-set of $\mathfrak{I}$; previously, these were the $H_{j}$s. \item Congruences which tell us the co-set of $\mathfrak{I}$ to which a given element of $\mathcal{O}_{\mathbb{F}}$ belongs; previously, these were $\overset{p}{\equiv}j$ for $j\in\mathbb{Z}/p\mathbb{Z}$. \end{itemize} \begin{example} \label{exa:root 2 example}Let $\mathbb{F}=\mathbb{Q}\left(\sqrt{2}\right)$, $\mathcal{O}_{\mathbb{F}}=\mathbb{Z}\left[\sqrt{2}\right]$, and consider the ideal $\mathfrak{I}=\left\langle \sqrt{2}\right\rangle $ generated by $\sqrt{2}$ along with the quotient ring $\mathbb{Z}\left[\sqrt{2}\right]/\left\langle \sqrt{2}\right\rangle $. We $\left\{ 1,\sqrt{2}\right\} $ as our basis for $\mathbb{Z}\left[\sqrt{2}\right]$. Since $\left\langle 2\right\rangle $ is a proper ideal of $\left\langle \sqrt{2}\right\rangle $, it follows that everything in $\mathbb{Z}\left[\sqrt{2}\right]$ which is congruent to $0$ mod $2$ will also be congruent to $0$ mod $\sqrt{2}$. Additionally, since $2\in\mathfrak{I}$, it follows that $a+b\sqrt{2}\overset{\sqrt{2}}{\equiv}a$ for all $a,b\in\mathbb{Z}$. If $a$ is even, then $a\overset{\sqrt{2}}{\equiv}0$. If $a$ is odd, then $a\overset{2}{\equiv}1$. Moreover, since $\sqrt{2}$ divides $2$, note that $a\overset{2}{\equiv}1$ forces $a\overset{\sqrt{2}}{\equiv}1$. Consequently, given any number $a+b\sqrt{2}\in\mathbb{Z}\left[\sqrt{2}\right]$, the equivalence class in $\mathbb{Z}\left[\sqrt{2}\right]/\left\langle \sqrt{2}\right\rangle $ to which belongs\textemdash a.k.a, the value of $a+b\sqrt{2}$ ``mod $\sqrt{2}$''\textemdash is then uniquely determined by the value of $a$ mod $2$. In particular, every $a+b\sqrt{2}\in\mathbb{Z}\left[\sqrt{2}\right]$ is congruent to either $0$ or $1$ mod $\sqrt{2}$, with: \[ a+b\sqrt{2}\overset{\sqrt{2}}{\equiv}0\Leftrightarrow a\overset{2}{\equiv}0 \] \[ a+b\sqrt{2}\overset{\sqrt{2}}{\equiv}1\Leftrightarrow a\overset{2}{\equiv}1 \] As such, if we write $\mathbb{Z}\left[\sqrt{2}\right]$ in $\left\{ 1,\sqrt{2}\right\} $-coordinates, the $2$-tuple $\mathbf{m}=\left(a,b\right)\in\mathbb{Z}^{2}$ corresponding to $a+b\sqrt{2}\in\mathbb{Z}\left[\sqrt{2}\right]$ represents a number congruent to $0$ (resp. $1$) mod $\sqrt{2}$ if and only if the first component of $\mathbf{m}$ ($a$) is congruent to $0$ (resp. $1$) mod $2$. \end{example} \vphantom{} The reason \textbf{Example \ref{exa:root 2 example}} works out so nicely is because quotienting the number ring $\mathbb{Z}\left[\sqrt{2}\right]$ by the ideal $\left\langle \sqrt{2}\right\rangle $ yields a ring isomorphic to $\mathbb{Z}/2\mathbb{Z}$, which is cyclic as an additive group. However, as any algebraic number theorist will readily tell you, for a general number field $\mathbb{F}$ and an arbitrary non-zero proper ideal $\mathfrak{I}$ of $\mathcal{O}_{\mathbb{F}}$, there is no guarantee that the ring $\mathcal{O}_{\mathbb{F}}/\mathfrak{I}$ will be cyclic as an additive group. Rather, the Structure Theorem for Finitely-Generated Modules over a Principal Ideal Domain tells us that the most we can expect is for there to be an isomorphism of additive groups: \begin{equation} \mathcal{O}_{\mathbb{F}}/\mathfrak{I}\cong\mathfrak{C}_{p_{1}}\times\cdots\times\mathfrak{C}_{p_{r}}\label{eq:Direct Product Representation of O_F / I} \end{equation} where $\mathfrak{C}_{p_{n}}$ is the cyclic group of order $p_{n}$ \nomenclature{$\mathfrak{C}_{n}$}{cyclic group of order $n$ \nopageref}, where $r\in\left\{ 1,\ldots,\left|\mathcal{O}_{\mathbb{F}}/\mathfrak{I}\right|\right\} $, and where the $p_{n}$s are integers $\geq2$ so that $p_{n}\mid p_{n+1}$ for all $n\in\left\{ 1,\ldots,r-1\right\} $. In analogy to the one-dimensional case, $\mathcal{O}_{\mathbb{F}}/\mathfrak{I}$ was $\mathbb{Z}/p\mathbb{Z}$, the set to which the branch-determining parameter $j$ belonged. As (\ref{eq:Direct Product Representation of O_F / I}) shows, however, for the multi-dimensional case, we cannot assume that the branch-determining parameter will take integer values like in \textbf{Example \ref{exa:root 2 example}}, where the parameter was $j\in\left\{ 0,1\right\} $. Instead, the branch-determining parameter we be an \textbf{$r$-tuple of integers }$\mathbf{j}\in\left(\mathbb{Z}/p_{1}\mathbb{Z}\right)\times\cdots\times\left(\mathbb{Z}/p_{r}\mathbb{Z}\right)$. In the one-dimensional case, we obtained an extension of $\chi_{H}$ by considering infinitely long lists (strings) of parameters. In the multi-dimensional case, an infinitely long list of $r$-tuples $\mathbf{j}\in\left(\mathbb{Z}/p_{1}\mathbb{Z}\right)\times\cdots\times\left(\mathbb{Z}/p_{r}\mathbb{Z}\right)$ would then be identified with an $r$-tuple $\mathbf{z}\in\mathbb{Z}_{p_{1}}\times\cdots\times\mathbb{Z}_{p_{r}}$ whose $n$th entry was a $p_{n}$-adic integer corresponding to the string whose elements are the $n$th entries of the $\mathbf{j}$s. Although this situation creates no problems when performing Fourier analysis on multi-dimensional $\chi_{H}$, it causes trouble for our proof of the multi-dimensional analogue of the Correspondence Principle, because it potentially makes it impossible for us to define a $p$-adic extension of the multi-dimensional Hydra map $H$. As such, for Subsection \ref{sec:5.2 The-Numen-of}, we will need to restrict to the case where every element of $P$ is a single prime, $p$. \subsection{\label{subsec:5.1.1. Algebraic-Conventions}Algebraic Conventions} To minimize any cumbersome aspects of notation, we introduce the following conventions\footnote{The one exception to this will be for multi-dimensional analogues of functions from the one-dimensional case, such as $\chi_{H}$, $\hat{A}_{H}$, $\alpha_{H}$, and the rest. For them, I retain as much of the one-dimensional notation as possible to emphasize the parallels between the two cases. Indeed, most of the computations will be repeated nearly verbatim.}: \begin{itemize} \item \textbf{Bold, lower case letters }(ex: $\mathbf{j}$, $\mathbf{a}$, $\mathbf{n}$, $\mathbf{x}$, $\mathbf{z}$, etc.) are reserved to denote tuples of finite length. In computations involving matrices, such tuples will always be treated as column vectors. Row vectors are written with a superscript $T$ to denote the transpose of column vector (ex: \textbf{$\mathbf{j}^{T}$}). \item \textbf{BOLD, UPPER CASE LETTERS }(ex: $\mathbf{A},\mathbf{D},\mathbf{P}$, etc.) are reserved to denote $d\times d$ matrices. In particular, $\mathbf{D}$ will be used for \textbf{diagonal matrices}, while $\mathbf{P}$ will be used for \textbf{permutation matrices}\textemdash matrices (whose entries are $0$s and $1$s) that give the so-called ``defining representation'' of the symmetric group $\mathfrak{S}_{d}$ acting on $\mathbb{R}^{d}$. \end{itemize} It is not an exaggeration to say that the primary challenge of the multi-dimensional case is its notation. As a rule of thumb, whenever vectors are acting on vectors, reader should assume that the operations are being done entry-wise, unless state otherwise. That being said, there is quite a lot of notation we will have to cover. In order to avoid symbol overload, the notation will only be introduced as needed. The primary chunks of notation occur in the segment given below, and then again at the start of Subsections \ref{subsec:5.3.1 Tensor-Products} and \ref{subsec:5.4.1 Multi-Dimensional--adic-Fourier}. \begin{notation}[\textbf{Multi-Dimensional Notational Conventions} \index{multi-dimensional!notation}] \label{nota:First MD notation batch}Let $d$ be an integer, let $\mathbb{F}$ be a field, let $s\in\mathbb{F}$, and let $\mathbf{a}=\left(a_{1},\ldots,a_{d}\right)$, $\mathbf{b}=\left(b_{1},\ldots,b_{d}\right)$, and $\mathbf{c}=\left(c_{1},\ldots,c_{d}\right)$ be elements of $\mathbb{F}^{d}$. Then:\index{multi-dimensional!notation} \vphantom{} I. $\mathbf{a}+\mathbf{b}\overset{\textrm{def}}{=}\left(a_{1}+b_{1},\ldots,a_{d}+b_{d}\right)$ \vphantom{} II. $\mathbf{a}\mathbf{b}\overset{\textrm{def}}{=}\left(a_{1}b_{1},\ldots,a_{d}b_{d}\right)$ \vphantom{} III. $\frac{\mathbf{a}}{\mathbf{b}}\overset{\textrm{def}}{=}\left(\frac{a_{1}}{b_{1}},\ldots,\frac{a_{d}}{b_{d}}\right)$ \vphantom{} IV. $s\mathbf{a}\overset{\textrm{def}}{=}\left(sa_{1},\ldots,sa_{d}\right)$ \vphantom{} V. $\mathbf{a}+s\overset{\textrm{def}}{=}\left(a_{1}+s,\ldots,a_{d}+s\right)$ \vphantom{} VI. \[ \sum_{\mathbf{k}=\mathbf{a}}^{\mathbf{b}}\overset{\textrm{def}}{=}\sum_{k_{1}=a_{1}}^{b_{1}}\cdots\sum_{k_{d}=a_{d}}^{b_{d}} \] \vphantom{} VII. $\mathbf{I}_{d}$ \nomenclature{$\mathbf{I}_{d}$}{$d\times d$ identity matrix \nopageref} denotes the $d\times d$ identity matrix. $\mathbf{O}_{d}$\nomenclature{$\mathbf{O}_{d}$}{$d\times d$ zero matrix \nopageref} denotes the $d\times d$ zero matrix. $\mathbf{0}$\nomenclature{$\mathbf{0}$}{zero vector \nopageref} denotes a column vector of $0$s (a.k.a., the zero vector). Unfortunately, the length of this $\mathbf{0}$ will usually depend on context. \vphantom{} VIII. Given $d\times d$ matrices $\mathbf{A}$ and $\mathbf{B}$ with entries in a field $\mathbb{F}$, if $\mathbf{B}$ is invertible, we write: \nomenclature{$\frac{\mathbf{A}}{\mathbf{B}}$}{$\mathbf{B}^{-1}\mathbf{A}$ \nopageref} \begin{equation} \frac{\mathbf{A}}{\mathbf{B}}\overset{\textrm{def}}{=}\mathbf{B}^{-1}\mathbf{A}\label{eq:Matrix fraction convention} \end{equation} \end{notation} \begin{defn} We write\nomenclature{$P$}{$\left(p_{1},\ldots,p_{r}\right)$}: \begin{equation} P\overset{\textrm{def}}{=}\left(p_{1},\ldots,p_{r}\right)\label{eq:Definition of Big P} \end{equation} be to denote $r$-tuple of integers (all of which are $\geq2$), where $r$ is a positive integer, and where $p_{n}\mid p_{n+1}$ for all $n\in\left\{ 1,\ldots,r-1\right\} $. We then write\nomenclature{$\mathbb{Z}^{r}/P\mathbb{Z}^{r}$}{$\overset{\textrm{def}}{=}\prod_{m=1}^{r}\left(\mathbb{Z}/p_{m}\mathbb{Z}\right)$ } to denote the direct product of rings: \begin{equation} \mathbb{Z}^{r}/P\mathbb{Z}^{r}\overset{\textrm{def}}{=}\prod_{m=1}^{r}\left(\mathbb{Z}/p_{m}\mathbb{Z}\right) \end{equation} That is, $\mathbb{Z}^{r}/P\mathbb{Z}^{r}$ is set of all $r$-tuples $\mathbf{j}=\left(j_{1},\ldots,j_{r}\right)$ so that $j_{m}\in\mathbb{Z}/p_{m}\mathbb{Z}$ for all $m$, equipped with component-wise addition and multiplication operations on the $\mathbb{Z}/p_{m}\mathbb{Z}$s. We also write $\mathbb{Z}^{r}/p\mathbb{Z}^{r}\overset{\textrm{def}}{=}\left(\mathbb{Z}/p\mathbb{Z}\right)^{r}$ to denote the case where $p_{n}=p$ (for some integer $p\geq2$) for all $n\in\left\{ 1,\ldots,r\right\} $. Given a $\mathbf{j}=\left(j_{1},\ldots,j_{r}\right)\in\mathbb{Z}^{r}/P\mathbb{Z}^{r}$ and any tuple $\mathbf{x}=\left(x_{1},\ldots,x_{\left|\mathbf{x}\right|}\right)$ of length $\geq1$, we write: \begin{align*} \mathbf{x} & \overset{P}{\equiv}\mathbf{j}\\ & \Updownarrow\\ x_{n} & \overset{p_{n}}{\equiv}j_{n},\textrm{ }\forall n\in\left\{ 1,\ldots,\min\left\{ r,\left|\mathbf{x}\right|\right\} \right\} \end{align*} When there is an integer $p\geq2$ so that $p_{n}=p$ for all $n$, we write: \begin{align*} \mathbf{x} & \overset{p}{\equiv}\mathbf{j}\\ & \Updownarrow\\ x_{n} & \overset{p}{\equiv}j_{n},\textrm{ }\forall n\in\left\{ 1,\ldots,\min\left\{ r,\left|\mathbf{x}\right|\right\} \right\} \end{align*} \end{defn} \subsection{Multi-Dimensional Hydra Maps \label{subsec:5.1.2 Co=00003D0000F6rdinates,-Half-Lattices,-and}} \begin{defn} Fix a number field $\mathbb{F}$ of degree $d$ over $\mathbb{Q}$, and a non-zero proper ideal $\mathfrak{I}\subset\mathcal{O}_{\mathbb{F}}$. \nomenclature{$\iota$}{$\left|\mathcal{O}_{\mathbb{F}}/\mathfrak{I}\right|$}We write: \begin{equation} \iota\overset{\textrm{def}}{=}\left|\mathcal{O}_{\mathbb{F}}/\mathfrak{I}\right|\label{eq:definition of iota_I} \end{equation} In algebraic terminology, $\iota$ is the \textbf{index }of\index{ideal!index of a} $\mathfrak{I}$ in\emph{ $\mathcal{O}_{\mathbb{F}}$. }As discussed above, there is an isomorphism of additive groups: \begin{equation} \mathcal{O}_{\mathbb{F}}/\mathfrak{I}\cong\mathfrak{C}_{p_{1}}\times\cdots\times\mathfrak{C}_{p_{r}} \end{equation} where $\mathfrak{C}_{p_{i}}$ is the cyclic group of order $p_{i}$, where $r\in\left\{ 1,\ldots,\iota\right\} $, where $p_{n}\mid p_{n+1}$ for all $n\in\left\{ 1,\ldots,r-1\right\} $ and: \begin{equation} \prod_{n=1}^{r}p_{n}=\iota\label{eq:Relation between the rho_is and iota_I} \end{equation} I call $r$ the \textbf{depth}\footnote{From an algebraic perspective, the decomposition (\ref{eq:Direct Product Representation of O_F / I-1}) is relatively unremarkable, being a specific case of the structure theorem for finitely generated modules over a principal ideal domain. To an algebraist, $r_{\mathfrak{I}}$ would be ``the number of invariant factors of the $\mathbb{Z}$-module $\mathcal{O}_{\mathbb{F}}/\mathfrak{I}$''; ``depth'', though not standard terminology, is certainly pithier.}\textbf{ }of\index{ideal!depth of a} $\mathfrak{I}$. In addition to the above, note that there is a set $\mathcal{B}\subset\mathcal{O}_{\mathbb{F}}$ which is a $\mathbb{Z}$-basis of the group $\left(\mathcal{O}_{\mathbb{F}},+\right)$, so that, for any $d$-tuple $\mathbf{x}=\left(x_{1},\ldots,x_{d}\right)\in\mathbb{Z}^{d}$ representing an element $z\in\mathcal{O}_{\mathbb{F}}$, the equivalence class of $\mathcal{O}_{\mathbb{F}}/\mathfrak{I}$ to which $z$ belongs is then completely determined by the values $x_{1}$ mod $p_{1}$, $x_{2}$ mod $p_{2}$, $\ldots$, $x_{r_{\mathfrak{I}}}$ mod $p_{r}$. \emph{From here on out, we will work with a fixed choice of such a basis $\mathcal{B}$}. We write $\mathbf{R}$ to denote the $d\times d$ diagonal matrix: \begin{equation} \mathbf{R}\overset{\textrm{def}}{=}\left[\begin{array}{cccccc} p_{1}\\ & \ddots\\ & & p_{r}\\ & & & 1\\ & & & & \ddots\\ & & & & & 1 \end{array}\right]\label{eq:Definition of bold R} \end{equation} where there are $d-r$ $1$s after $p_{r}$. Lastly, when adding vectors of unequal length, we do everything from left to right: \begin{equation} \mathbf{v}+\mathbf{j}\overset{\textrm{def}}{=}\left(v_{1}+j_{1},\ldots,v_{r}+j_{r},v_{r+1}+0,\ldots,v_{d}+0\right),\textrm{ }\forall\mathbf{v}\in\mathbb{Z}^{d},\textrm{ }\forall\mathbf{j}\in\mathbb{Z}^{r}/P\mathbb{Z}^{r}\label{eq:Definition of the sum of bold_m and bold_j} \end{equation} \end{defn} \begin{defn} Letting everything be as described above, enumerate the elements of the basis $\mathcal{B}$ as $\left\{ \gamma_{1},\ldots,\gamma_{d}\right\} $. We then write $\mathcal{O}_{\mathbb{F},\mathcal{B}}^{+}$ to denote the \textbf{half-lattice}\index{half-lattice}\textbf{ of }$\mathcal{B}$\textbf{-positive $\mathbb{F}$-integers} (or ``positive half-lattice''), defined by: \begin{equation} \mathcal{O}_{\mathbb{F},\mathcal{B}}^{+}\overset{\textrm{def}}{=}\left\{ \sum_{\ell=1}^{d}x_{\ell}\gamma_{\ell}:x_{1},\ldots,x_{d}\in\mathbb{N}_{0}\right\} \label{eq:Def of O-plus_F,B} \end{equation} The \textbf{half-lattice of $\mathcal{B}$-negative $\mathbb{F}$-integers }(or ``negative half-lattice''), denoted $\mathcal{O}_{\mathbb{F},\mathcal{B}}^{-}$, is defined by: \begin{equation} \mathcal{O}_{\mathbb{F},\mathcal{B}}^{-}\overset{\textrm{def}}{=}\left\{ -\sum_{\ell=1}^{d}x_{\ell}\gamma_{\ell}:x_{1},\ldots,x_{d}\in\mathbb{N}_{0}\right\} \label{eq:Def of O_F,B minus} \end{equation} Finally, we write \nomenclature{$\varphi_{\mathcal{B}}$}{ }$\varphi_{\mathcal{B}}:\mathbb{Q}^{d}\rightarrow\mathbb{F}$ to denote the vector space isomorphism that sends each $\mathbf{x}\in\mathbb{Q}^{d}$ to the unique element of $\mathbb{F}$ whose representation in $\mathcal{B}$-cordinates is $\mathbf{x}$, and which satisfies the property that the restriction $\varphi_{\mathcal{B}}\mid_{\mathbb{Z}^{d}}$ of $\varphi_{\mathcal{B}}$ to $\mathbb{Z}^{d}$ is an isomorphism of the groups $\left(\mathbb{Z}^{d},+\right)$ and $\left(\mathcal{O}_{\mathbb{F}},+\right)$. We write $\varphi_{\mathcal{B}}^{-1}$ to denote the inverse of $\varphi_{\mathcal{B}}$; $\varphi_{\mathcal{B}}^{-1}$ outputs the $d$-tuple $\mathcal{B}$-cordinate representation of the inputted $z\in\mathbb{F}$. We then write $\mathbb{Z}_{\mathbb{F},\mathcal{B}}^{+}$ \nomenclature{$\mathbb{Z}_{\mathbb{F},\mathcal{B}}^{+}$}{$\varphi_{\mathcal{B}}\left(\mathcal{O}_{\mathbb{F},\mathcal{B}}^{+}\right)$} and $\mathbb{Z}_{\mathbb{F},\mathcal{B}}^{-}$ \nomenclature{$\mathbb{Z}_{\mathbb{F},\mathcal{B}}^{-}$}{$\varphi_{\mathcal{B}}\left(\mathcal{O}_{\mathbb{F},\mathcal{B}}^{-}\right)$} to denote $\varphi_{\mathcal{B}}\left(\mathcal{O}_{\mathbb{F},\mathcal{B}}^{+}\right)\subset\mathbb{Z}^{d}$ and $\varphi_{\mathcal{B}}\left(\mathcal{O}_{\mathbb{F},\mathcal{B}}^{-}\right)\subset\mathbb{Z}^{d}$, respectively. Note that, as subsets of $\mathbb{Z}^{d}$, we have: \begin{align*} \mathbb{Z}_{\mathbb{F},\mathcal{B}}^{+} & =\mathbb{N}_{0}^{d}\\ \mathbb{Z}_{\mathbb{F},\mathcal{B}}^{-} & =-\mathbb{N}_{0}^{d} \end{align*} \end{defn} \begin{defn}[\textbf{$\left(\mathbb{F},\mathfrak{I},\mathcal{B}\right)$-Hydra maps}] \label{def:(F,I,B)-Hydra map}Let $\mathbb{F}$ be a number field of dimension $d$, let $\mathfrak{I}$ be a non-zero proper ideal of $\mathcal{O}_{\mathbb{F}}$ of index $\iota$, and let $\mathcal{B}$ be a basis as discussed above. We write $\mathfrak{I}_{0},\ldots,\mathfrak{I}_{\iota}$ to denote the co-sets of $\mathfrak{I}$ in $\mathcal{O}_{\mathbb{F}}$, with $\mathfrak{I}_{0}$ denoting $\mathfrak{I}$ itself. Then, a \textbf{$\left(\mathbb{F},\mathfrak{I},\mathcal{B}\right)$-Hydra map} \textbf{on $\mathcal{O}_{\mathbb{F}}$} is a surjective map $\tilde{H}:\mathcal{O}_{\mathbb{F}}\rightarrow\mathcal{O}_{\mathbb{F}}$ of the form: \begin{equation} \tilde{H}\left(z\right)=\begin{cases} \frac{a_{0}z+b_{0}}{d_{0}} & \textrm{if }z\in\mathfrak{I}_{0}\\ \vdots & \vdots\\ \frac{a_{\iota-1}z+b_{\iota-1}}{d_{\iota-1}} & \textrm{if }z\in\mathfrak{I}_{\iota} \end{cases}\label{eq:Definition of I-hydra map} \end{equation} We call $r$ the \textbf{depth }of $\tilde{H}$. Here, the $a_{j}$s, $b_{j}$s, and $d_{j}$s are elements of $\mathcal{O}_{\mathbb{F}}$ so that: \vphantom{} I. $a_{j},d_{j}\neq0$ for all $j\in\left\{ 0,\ldots,\iota-1\right\} $. \vphantom{} II. $\gcd\left(a_{j},d_{j}\right)=1$ for all $j\in\left\{ 0,\ldots,\iota-1\right\} $. \vphantom{} III. $H\left(\mathcal{O}_{\mathbb{F},\mathcal{B}}^{+}\right)\subseteq H\left(\mathcal{O}_{\mathbb{F},\mathcal{B}}^{+}\right)$ and $H\left(\mathcal{O}_{\mathbb{F},\mathcal{B}}^{-}\backslash\mathcal{O}_{\mathbb{F},\mathcal{B}}^{+}\right)\subseteq H\left(\mathcal{O}_{\mathbb{F},\mathcal{B}}^{-}\backslash\mathcal{O}_{\mathbb{F},\mathcal{B}}^{+}\right)$, where $\mathcal{O}_{\mathbb{F},\mathcal{B}}^{-}\backslash\mathcal{O}_{\mathbb{F},\mathcal{B}}^{+}$ is the set of all elements of $\mathcal{O}_{\mathbb{F},\mathcal{B}}^{-}$ which are not elements of $\mathcal{O}_{\mathbb{F},\mathcal{B}}^{+}$. \vphantom{} IV. For all $j\in\left\{ 0,\ldots,\iota-1\right\} $, the ideal $\left\langle d_{j}\right\rangle _{\mathcal{O}_{\mathbb{F}}}$ in $\mathcal{O}_{\mathbb{F}}$ generated by $d_{j}$ is contained in $\mathfrak{I}$. \vphantom{} V. For all $j\in\left\{ 0,\ldots,\iota-1\right\} $, the the matrix representation in $\mathcal{B}$-coordinates on $\mathbb{Q}^{d}$ of the ``multiplication by $a_{j}/d_{j}$'' map on $\mathbb{F}$ is of the form: \begin{equation} \frac{\mathbf{A}}{\mathbf{D}}\overset{\textrm{def}}{=}\mathbf{D}^{-1}\mathbf{A}\label{eq:A / D notation} \end{equation} where $\mathbf{A},\mathbf{D}$ are invertible $d\times d$ matrices so that: \vphantom{} V-i. $\mathbf{A}=\tilde{\mathbf{A}}\mathbf{P}$, where $\mathbf{P}$ is a permutation matrix\footnote{That is, a matrix of $0$s and $1$s which is a representation of the action on $\mathbb{Z}^{d}$ of an element of the symmetric group on $d$ objects by way of a permutation of the coordinate entries of the $d$-tuples in $\mathbb{Z}^{d}$.} and where $\mathbf{\tilde{\mathbf{A}}}$ is a diagonal matrix whose non-zero entries are positive integers. \vphantom{} V-ii. $\mathbf{D}$ is a diagonal matrix whose non-zero entries are positive integers such that every entry on the diagonal of $\mathbf{R}\mathbf{D}^{-1}$ is a positive integer. Note that this forces the $\left(r+1\right)$th through $d$th diagonal entries of $\mathbf{D}$ to be equal to $1$. Moreover, for each $\ell\in\left\{ 1,\ldots,r\right\} $, this forces the $\ell$th entry of the diagonal of $\mathbf{D}$ to be a divisor of $p_{\ell}$. \vphantom{} V-iii. Every non-zero element of $\mathbf{A}$ is co-prime to every non-zero element of $\mathbf{D}$. \vphantom{} Additionally, we say that $\tilde{H}$ is \textbf{integral }if it satisfies\index{Hydra map!integral}: \vphantom{} VI. For all $j\in\left\{ 0,\ldots,\iota-1\right\} $, the number $\frac{a_{j}z+b_{j}}{d_{j}}$ is an element of $\mathcal{O}_{\mathbb{F}}$ if and only if $z\in\mathcal{O}_{\mathbb{F}}\cap\mathfrak{I}_{j}$. \vphantom{} If $\tilde{H}$ does not satisfy (VI), we say $\tilde{H}$ is \textbf{non-integral}. \index{Hydra map!non-integral} \end{defn} \begin{defn}[\textbf{$P$-Hydra maps}] \label{def:P-Hydra map}Let $\mathbb{F}$ be a number field of dimension $d$, let $\mathfrak{I}$ be a non-zero proper ideal of $\mathcal{O}_{\mathbb{F}}$ of index $\iota$, and let $\mathcal{B}$ be a basis as discussed above. A $P$\textbf{-Hydra map} \textbf{on $\mathbb{Z}^{d}$}\index{$P$-Hydra map}\textbf{ }is a map $H:\mathbb{Z}^{d}\rightarrow\mathbb{Z}^{d}$ so that: \begin{equation} H=\varphi_{\mathcal{B}}\circ\tilde{H}\circ\varphi_{\mathcal{B}}^{-1}\label{eq:Definition of a Hydra map on Zd} \end{equation} for some $\left(\mathbb{F},\mathfrak{I},\mathcal{B}\right)$-Hydra map $\tilde{H}$ on $\mathcal{O}_{\mathbb{F}}$ for which $P=\left(p_{1},\ldots,p_{r}\right)$. We\index{Hydra map!field analogue} \index{Hydra map!lattice analogue}call $\tilde{H}$ the \textbf{field analogue }of $H$, and call $H$ the \textbf{lattice analogue }of $\tilde{H}$. We say $H$ has \textbf{depth} $r$ whenever $\tilde{H}$ has depth $r$.\index{Hydra map!depth} \end{defn} \begin{prop}[Formula for $P$-Hydra maps] Let $\tilde{H}:\mathcal{O}_{\mathbb{F}}\rightarrow\mathcal{O}_{\mathbb{F}}$ be an $\left(\mathbb{F},\mathfrak{I},\mathcal{B}\right)$-Hydra map, and let $H$ be its lattice analogue (a $P$-Hydra map). Then, there is a unique collection of integer-entry matrices: \[ \left\{ \mathbf{A}_{\mathbf{j}}\right\} _{\mathbf{j}\in\mathbb{Z}^{r}/P\mathbb{Z}^{r}},\left\{ \mathbf{D}_{\mathbf{j}}\right\} _{\mathbf{j}\in\mathbb{Z}^{r}/P\mathbb{Z}^{r}}\subseteq\textrm{GL}_{d}\left(\mathbb{Q}\right) \] satisfying conditions \emph{(V-i)}, \emph{(V-ii), and (V-iii) }from\textbf{ }\emph{(\ref{def:(F,I,B)-Hydra map})} for each $\mathbf{j}$, respectively, and a unique collection of $d\times1$ column vectors $\left\{ \mathbf{b}_{\mathbf{j}}\right\} _{\mathbf{j}\in\mathbb{Z}^{r}/P\mathbb{Z}^{r}}\subseteq\mathbb{Z}^{d}$ so that: \nomenclature{$\mathbf{A}_{\mathbf{j}}$}{ } \nomenclature{$\mathbf{D}_{\mathbf{j}}$}{ } \nomenclature{$\mathbf{b}_{\mathbf{j}}$}{ } \index{Hydra map}\index{Hydra map!on mathbb{Z}{d}@on $\mathbb{Z}^{d}$} \index{$P$-Hydra map} \index{multi-dimensional!Hydra map} \begin{equation} H\left(\mathbf{x}\right)=\sum_{\mathbf{j}\in\mathbb{Z}^{r}/P\mathbb{Z}^{r}}\left[\mathbf{x}\overset{P}{\equiv}\mathbf{j}\right]\frac{\mathbf{A}_{\mathbf{j}}\mathbf{x}+\mathbf{b}_{\mathbf{j}}}{\mathbf{D}_{\mathbf{j}}},\textrm{ }\forall\mathbf{x}\in\mathbb{Z}^{d}\label{eq:MD Hydra Map Formula} \end{equation} In particular, for all $\mathbf{j}\in\mathbb{Z}^{r}/P\mathbb{Z}^{r}$, the $d$-tuple $\mathbf{D}_{\mathbf{j}}^{-1}\left(\mathbf{A}_{\mathbf{j}}\mathbf{x}+\mathbf{b}_{\mathbf{j}}\right)$ will be an element of $\mathbb{Z}_{\mathbb{F},\mathcal{B}}^{+}$ for all $\mathbf{x}\in\mathbb{Z}_{\mathbb{F},\mathcal{B}}^{+}$ for which $\mathbf{x}\overset{P}{\equiv}\mathbf{j}$. \end{prop} \begin{rem} Recall that the congruence $\mathbf{x}\overset{P}{\equiv}\mathbf{j}$ means that for each $n\in\left\{ 1,\ldots,r\right\} $, the $n$th entry of $\mathbf{x}$ is congruent mod $p_{n}$ to the $n$th entry of $\mathbf{j}$. Moreover, $\mathbf{x}\overset{P}{\equiv}\mathbf{j}$ is completely independent of the $\left(r+1\right)$th through $d$th entries of $\mathbf{x}$. \end{rem} Proof: Let everything be as described above. Fix $z=\sum_{n=1}^{d}c_{n}\gamma_{n}\in\mathcal{O}_{\mathbb{F},\mathcal{B}}^{+}$, where the $c_{n}$s are non-negative integers. Letting $\mathfrak{I}_{j}$ denote the unique equivalence class of $\mathfrak{I}$ in $\mathcal{O}_{\mathbb{F}}$ to which $z$ belongs, it follows by definition of $\tilde{H}$ that: \begin{equation} \tilde{H}\left(z\right)=\frac{a_{j}z+b_{j}}{d_{j}} \end{equation} Now, consider $\mathbb{F}$ as a $d$-dimensional linear space over $\mathbb{Q}$, equipped with the coordinate system given by the basis $\mathcal{B}=\left\{ \gamma_{1},\ldots,\gamma_{d}\right\} $. In these coordinates, the ``multiplication by $a_{j}/d_{j}$'' map on $\mathbb{F}$ can be uniquely represented as left-multiplication by some $\mathbf{C}\in\textrm{GL}_{d}\left(\mathbb{Q}\right)$ with rational entries. So, letting $\mathbf{x}$ denote the coordinate $d$-tuple representing $z$ (that is, $\mathbf{x}=\varphi_{\mathcal{B}}^{-1}\left(z\right)$), it follows that: \begin{equation} \varphi_{\mathcal{B}}^{-1}\left(\frac{a_{j}z}{d_{j}}\right)=\mathbf{C}\mathbf{x} \end{equation} Letting $\mathbf{k}=\varphi_{\mathcal{B}}^{-1}\left(b_{j}/d_{j}\right)$ be unique the coordinate $d$-tuple representing $b_{j}/d_{j}$, we then have that: \begin{equation} \varphi_{\mathcal{B}}^{-1}\left(\tilde{H}\left(z\right)\right)=\varphi_{\mathcal{B}}^{-1}\left(\frac{a_{j}z+b_{j}}{d_{j}}\right)=\varphi_{\mathcal{B}}^{-1}\left(\frac{a_{j}z}{d_{j}}\right)+\varphi_{\mathcal{B}}^{-1}\left(\frac{b_{j}}{d_{j}}\right)=\mathbf{C}\mathbf{x}+\mathbf{k} \end{equation} where we used the fact that, as defined, $\varphi_{\mathcal{B}}$ is an isomorphism of the linear spaces $\mathbb{F}$ and $\mathbb{Q}^{d}$. By (\ref{eq:A / D notation}), we can write $\mathbf{C}$ as: \begin{equation} \mathbf{C}=\mathbf{D}^{-1}\mathbf{A} \end{equation} for $\mathbf{D},\mathbf{A}$ as described in (V) of (\ref{def:(F,I,B)-Hydra map}). We can make $\mathbf{D}$ and $\mathbf{A}$ unique by choosing them so as to make their non-zero entires (all of which are positive integers) as small as possible. Consequently, the vector: \begin{equation} \mathbf{b}\overset{\textrm{def}}{=}\mathbf{D}\mathbf{k} \end{equation} will have integer entries, and we can express the action of the $j$th branch of $\tilde{H}$ on an arbitrary $\mathbf{x}$ by: \begin{equation} \varphi_{\mathcal{B}}^{-1}\left(\frac{a_{j}z+b_{j}}{d_{j}}\right)=\frac{\mathbf{A}\mathbf{x}+\mathbf{b}}{\mathbf{D}} \end{equation} Since $\mathcal{O}_{\mathbb{F}}/\mathfrak{I}\cong\mathbb{Z}^{r}/P\mathbb{Z}^{r}$ for each $\mathbf{j}\in\mathbb{Z}^{r}/P\mathbb{Z}^{r}$, by the argument given in the previous paragraph, there are unique invertible $\mathbf{A}_{\mathbf{j}},\mathbf{D}_{\mathbf{j}}$ satisfying (V-i), (V-ii), and (V-iii) respectively and a unique $d\times1$ column vector $\mathbf{b}_{\mathbf{j}}$ with integer entries so that: \begin{equation} \varphi_{\mathcal{B}}^{-1}\left(\tilde{H}\left(z\right)\right)=\frac{\mathbf{A}_{\mathbf{j}}\varphi_{\mathcal{B}}^{-1}\left(z\right)+\mathbf{b}_{\mathbf{j}}}{\mathbf{D}_{\mathbf{j}}} \end{equation} holds for all $z\in\mathcal{O}_{\mathbb{F}}$ that are congruent to $\varphi_{\mathcal{B}}\left(\mathbf{j}\right)$ mod $\mathfrak{I}$. Multiplying by the Iverson bracket $\left[\varphi_{\mathcal{B}}^{-1}\left(z\right)\overset{P}{\equiv}\mathbf{j}\right]$ and summing over all $\mathbf{j}\in\mathbb{Z}^{r}/P\mathbb{Z}^{r}$ gives: \begin{eqnarray*} \sum_{\mathbf{j}\in\mathbb{Z}^{r}/P\mathbb{Z}^{r}}\left[\varphi_{\mathcal{B}}^{-1}\left(z\right)\overset{P}{\equiv}\mathbf{j}\right]\left(\frac{\mathbf{A}_{\mathbf{j}}\varphi_{\mathcal{B}}^{-1}\left(z\right)+\mathbf{b}_{\mathbf{j}}}{\mathbf{D}_{\mathbf{j}}}\right) & = & \sum_{\mathbf{j}\in\mathbb{Z}^{r}/P\mathbb{Z}^{r}}\left[\varphi_{\mathcal{B}}^{-1}\left(z\right)\overset{P}{\equiv}\mathbf{j}\right]\varphi_{\mathcal{B}}^{-1}\left(\tilde{H}\left(z\right)\right)\\ & = & \sum_{\mathbf{j}\in\mathbb{Z}^{r}/P\mathbb{Z}^{r}}\underbrace{\left[z\overset{\mathfrak{I}}{\equiv}\varphi_{\mathcal{B}}\left(\mathbf{j}\right)\right]}_{\in\left\{ 0,1\right\} }\varphi_{\mathcal{B}}^{-1}\left(\tilde{H}\left(z\right)\right)\\ & = & \varphi_{\mathcal{B}}^{-1}\left(\sum_{\mathbf{j}\in\mathcal{O}_{\mathbb{F}}/\mathfrak{I}}\left[z\overset{\mathfrak{I}}{\equiv}\varphi_{\mathcal{B}}\left(\mathbf{j}\right)\right]\tilde{H}\left(z\right)\right)\\ \left(\varphi_{\mathcal{B}}\left(\mathbf{j}\right)\textrm{s partition }\mathcal{O}_{\mathbb{F},\mathcal{B}}^{+}\right); & = & \varphi_{\mathcal{B}}^{-1}\left(\tilde{H}\left(z\right)\right) \end{eqnarray*} Hence: \begin{equation} \varphi_{\mathcal{B}}^{-1}\left(\tilde{H}\left(z\right)\right)=\sum_{\mathbf{j}\in\mathbb{Z}^{r}/P\mathbb{Z}^{r}}\left[\varphi_{\mathcal{B}}^{-1}\left(z\right)\overset{P}{\equiv}\mathbf{j}\right]\frac{\mathbf{A}_{\mathbf{j}}\varphi_{\mathcal{B}}^{-1}\left(z\right)+\mathbf{b}_{\mathbf{j}}}{\mathbf{D}_{\mathbf{j}}},\forall z\in\mathcal{O}_{\mathbb{F},\mathcal{B}}^{+} \end{equation} Replacing $z$ with $\varphi_{\mathcal{B}}\left(\mathbf{x}\right)$ (where $\mathbf{x}\in\mathbb{Z}_{\mathbb{F},\mathcal{B}}^{+}$) then gives the desired formula for $H\left(\mathbf{x}\right)$: \begin{equation} \underbrace{\left(\varphi_{\mathcal{B}}^{-1}\circ\tilde{H}\circ\varphi_{\mathcal{B}}\right)\left(\mathbf{x}\right)}_{H\left(\mathbf{x}\right)}=\sum_{\mathbf{j}\in\mathbb{Z}^{r}/P\mathbb{Z}^{r}}\left[\mathbf{x}\overset{P}{\equiv}\mathbf{j}\right]\frac{\mathbf{A}_{\mathbf{j}}\mathbf{x}+\mathbf{b}_{\mathbf{j}}}{\mathbf{D}_{\mathbf{j}}},\forall\mathbf{x}\in\mathbb{Z}_{\mathbb{F},\mathcal{B}}^{+} \end{equation} Q.E.D. \vphantom{} Finally, we introduce notation to take on the roles played by $\mu_{j}$ and $p$ in the one-dimensional case. \begin{defn} Let $H$ be a $P$-Hydra map on $\mathbb{Z}^{d}$. \vphantom{} I. For each $\mathbf{j}\in\mathbb{Z}^{r}/P\mathbb{Z}^{r}$, we write: \begin{equation} \mathbf{M}_{\mathbf{j}}\overset{\textrm{def}}{=}\mathbf{R}\frac{\mathbf{A}_{\mathbf{j}}}{\mathbf{D}_{\mathbf{j}}}\overset{\textrm{def}}{=}\mathbf{R}\mathbf{D}_{\mathbf{j}}^{-1}\mathbf{A}_{\mathbf{j}}\label{eq:Definition of bold M bold j} \end{equation} \vphantom{} II. \nomenclature{$H_{\mathbf{j}}\left(\mathbf{x}\right)$}{$\mathbf{j}$th branch of a multi-dimensional Hydra map}For each $\mathbf{j}\in\mathbb{Z}^{r}/P\mathbb{Z}^{r}$, we call the affine linear map $H_{\mathbf{j}}:\mathbb{Q}^{d}\rightarrow\mathbb{Q}^{d}$ defined by: \begin{equation} H_{\mathbf{j}}\left(\mathbf{x}\right)\overset{\textrm{def}}{=}\frac{\mathbf{A}_{\mathbf{j}}\mathbf{x}+\mathbf{b}_{\mathbf{j}}}{\mathbf{D}_{\mathbf{j}}}\label{eq:Definition of the bold jth branch of H} \end{equation} the \textbf{$\mathbf{j}$th branch }of $H$. Note that this overrides the definition used for $H_{\mathbf{j}}$ in Chapter 2. Using $\mathbf{M}_{\mathbf{j}}$, we can write: \begin{equation} H_{\mathbf{j}}\left(\mathbf{x}\right)=\frac{\mathbf{M}_{\mathbf{j}}}{\mathbf{R}}\mathbf{x}+H_{\mathbf{j}}\left(\mathbf{0}\right)\label{eq:bold jth branch of H in terms of bold M bold j} \end{equation} We then define $H_{\mathbf{j}}^{\prime}\left(\mathbf{0}\right)$ as: \begin{equation} H_{\mathbf{j}}^{\prime}\left(\mathbf{0}\right)\overset{\textrm{def}}{=}\frac{\mathbf{A}_{\mathbf{j}}}{\mathbf{D}_{\mathbf{j}}}=\frac{\mathbf{M}_{\mathbf{j}}}{\mathbf{R}}\label{eq:Definition of H_bold j prime of bold 0} \end{equation} Also, note then that $H^{\prime}\left(\mathbf{0}\right)=H_{\mathbf{0}}^{\prime}\left(\mathbf{0}\right)$. \vphantom{} III. We say $H$ is integral (resp. non-integral) if its field analogue is integral (resp. non-integral). \end{defn} \begin{example} Consider the map $\tilde{H}:\mathbb{Z}\left[\sqrt{3}\right]\rightarrow\mathbb{Z}\left[\sqrt{3}\right]$ defined by: \begin{equation} \tilde{H}\left(z\right)\overset{\textrm{def}}{=}\begin{cases} \frac{z}{\sqrt{3}} & \textrm{if }z=0\mod\sqrt{3}\\ \frac{z-1}{\sqrt{3}} & \textrm{if }z=1\mod\sqrt{3}\\ \frac{4z+1}{\sqrt{3}} & \textrm{if }z=2\mod\sqrt{3} \end{cases} \end{equation} Here $\gamma_{1}=1$ and $\gamma_{2}=\sqrt{3}$, $\mathfrak{I}=\left\langle \sqrt{3}\right\rangle $. Since: \begin{equation} \tilde{H}\left(a+b\sqrt{3}\right)=\begin{cases} b+\frac{a}{3}\sqrt{3} & \textrm{if }a=0\mod3\\ b+\frac{a-1}{3}\sqrt{3} & \textrm{if }a=1\mod3\\ 4b+\frac{4a+1}{3}\sqrt{3} & \textrm{if }a=2\mod3 \end{cases} \end{equation} we have: \begin{align*} H\left(\left[\begin{array}{c} v_{1}\\ v_{2} \end{array}\right]\right) & =\begin{cases} \left[\begin{array}{c} v_{2}\\ \frac{v_{1}}{3} \end{array}\right] & \textrm{if }v_{1}=0\mod3\\ \left[\begin{array}{c} v_{2}\\ \frac{v_{1}-1}{3} \end{array}\right] & \textrm{if }v_{1}=1\mod3\\ \left[\begin{array}{c} 4v_{2}\\ \frac{4v_{1}+1}{3} \end{array}\right] & \textrm{if }v_{1}=2\mod3 \end{cases}\\ & =\begin{cases} \left[\begin{array}{cc} 0 & 1\\ \frac{1}{3} & 0 \end{array}\right]\left[\begin{array}{c} v_{1}\\ v_{2} \end{array}\right] & \textrm{if }v_{1}=0\mod3\\ \left[\begin{array}{cc} 0 & 1\\ \frac{1}{3} & 0 \end{array}\right]\left[\begin{array}{c} v_{1}\\ v_{2} \end{array}\right]+\left[\begin{array}{c} 0\\ -\frac{1}{3} \end{array}\right] & \textrm{if }v_{1}=1\mod3\\ \left[\begin{array}{cc} 0 & 4\\ \frac{4}{3} & 0 \end{array}\right]\left[\begin{array}{c} v_{1}\\ v_{2} \end{array}\right]+\left[\begin{array}{c} 0\\ \frac{1}{3} \end{array}\right] & \textrm{if }v_{1}=2\mod3 \end{cases} \end{align*} and so: \begin{align*} H\left(\mathbf{v}\right) & =\left[\mathbf{v}\overset{3}{\equiv}\left[\begin{array}{c} 0\\ 0 \end{array}\right]\right]\left[\begin{array}{cc} 0 & 1\\ \frac{1}{3} & 0 \end{array}\right]\mathbf{v}+\left[\mathbf{v}\overset{3}{\equiv}\left[\begin{array}{c} 1\\ 0 \end{array}\right]\right]\left(\left[\begin{array}{cc} 0 & 1\\ \frac{1}{3} & 0 \end{array}\right]\mathbf{v}+\left[\begin{array}{c} 0\\ -\frac{1}{3} \end{array}\right]\right)\\ & +\left[\mathbf{v}\overset{3}{\equiv}\left[\begin{array}{c} 2\\ 0 \end{array}\right]\right]\left(\left[\begin{array}{cc} 0 & 4\\ \frac{4}{3} & 0 \end{array}\right]\mathbf{v}+\left[\begin{array}{c} 0\\ \frac{1}{3} \end{array}\right]\right) \end{align*} where, in all three branches, the permutation is: \begin{equation} \left[\begin{array}{cc} 0 & 1\\ 1 & 0 \end{array}\right] \end{equation} \end{example} \vphantom{} Lastly, we will need to distinguish those $P$-Hydra maps for which $p_{n}=p$ for all $n\in\left\{ 1,\ldots,r\right\} $ for some prime $p$. \begin{defn} \index{ideal!smooth}I say a non-zero proper ideal $\mathfrak{I}$ of $\mathcal{O}_{\mathbb{F}}$ is \textbf{smooth }if the $p_{n}$s from the factorization: \[ \mathcal{O}_{\mathbb{F}}/\mathfrak{I}\cong\mathfrak{C}_{p_{1}}\times\cdots\times\mathfrak{C}_{p_{r}} \] are all equal, with there being some integer $p\geq2$ so that $p_{n}=p$ for all $n\in\left\{ 1,\ldots,r\right\} $. I call a Multi-Dimensional Hydra map \textbf{smooth}\index{Hydra map!smooth}\index{Hydra map!$p$-smooth}\textbf{ }(or, more specifically, \textbf{$p$-smooth})\textbf{ }whenever either of the equivalent conditions is satisfied: \vphantom{} i $H$ is an $\left(\mathbb{F},\mathcal{B},\mathfrak{I}\right)$-Hydra map and $\mathfrak{I}$ is smooth. \vphantom{} ii. $H$ is a $d$-dimensional $P$-Hydra map of depth $r$, and there is an integer $p\geq2$ so that $p_{n}=p$ for all $n\in\left\{ 1,\ldots,r\right\} $. In this case, I refer to $H$ is a \textbf{$d$-dimensional $p$-Hydra map of depth $r$}. \vphantom{} Finally, I say $H$ (equivalently, $\tilde{H}$) is \textbf{prime }whenever it is $p$-smooth for some prime $p$. \end{defn} \newpage{} \section{\label{sec:5.2 The-Numen-of}The Numen of a Multi-Dimensional Hydra Map} THROUGHOUT THIS SECTION, UNLESS STATED OTHERWISE, $H$ DENOTES A PRIME ($p$-SMOOTH) $d$-DIMENSIONAL HYDRA MAP OF DEPTH $r$ WHICH FIXES $\mathbf{0}$. \vphantom{} In this section, we will do for multi-dimensional Hydra maps what we did for their one-dimensional cousins in Chapter 2. The exposition here will be more briskly paced than in that earlier chapter. Throughout, we will keep the one-dimensional case as a guiding analogy. Of course, we will pause as needed to discuss deviations particular to the multi-dimensional setting. \subsection{\label{subsec:5.2.1 Notation-and-Preliminary}Notation and Preliminary Definitions} Much like as in the one-dimensional case, we will restrict our attention to $\mathbf{m}\in\mathbb{N}_{0}^{d}$, the multi-dimensional analogue of $\mathbb{N}_{0}$. Whereas our composition sequences in the one-dimensional case were of the form: \begin{equation} H_{j_{1}}\circ H_{j_{2}}\circ\cdots \end{equation} for integers $j_{1},j_{2},\ldots\in\mathbb{Z}/p\mathbb{Z}$, in the multi-dimensional case, our composition sequences will be of the form: \begin{equation} H_{\mathbf{j}_{1}}\circ H_{\mathbf{j}_{2}}\circ\cdots \end{equation} where $\mathbf{j}_{1},\mathbf{j}_{2},\ldots\in\mathbb{Z}^{r}/p\mathbb{Z}^{r}$ are \emph{tuples}, rather than integers.\emph{ }What we denoted by $H_{\mathbf{j}}$ in the one-dimensional case will now be written as $H_{\mathbf{J}}$. As indicated by the capital letter, $\mathbf{J}$ denotes a \emph{matrix} whose columns are $\mathbf{j}_{1},\mathbf{j}_{2},\ldots$. The particular definitions are as follows: \begin{defn}[\textbf{Block strings}] \ \vphantom{} I. A \textbf{$p$-block string }of \textbf{depth }$r$ is a\index{block string}\index{block string!depth} finite tuple $\mathbf{J}=\left(\mathbf{j}_{1},\ldots,\mathbf{j}_{\left|\mathbf{J}\right|}\right)^{T}$, where: \vphantom{} i. \nomenclature{$\left|\mathbf{J}\right|$}{Length of the block string $\mathbf{J}$}$\left|\mathbf{J}\right|$ is an integer $\geq1$ which we call the \textbf{length }of $\mathbf{J}$. \vphantom{} ii. For each $m\in\left\{ 1,\ldots,\left|\mathbf{J}\right|\right\} $, $\mathbf{j}_{m}$ is an $r$-tuple: \begin{equation} \mathbf{j}_{m}=\left(j_{m,1},j_{m,2},\ldots,j_{m,r}\right)\in\mathbb{Z}^{r}/p\mathbb{Z}^{r} \end{equation} That is, for each $m$, the $n$th entry of $\mathbf{j}_{m}$ is an element of $\mathbb{Z}/p\mathbb{Z}$. We can also view $\mathbf{J}$ as a $\left|\mathbf{J}\right|\times r$ matrix, where the $m$th row of $\mathbf{J}$ consists of the entries of $\mathbf{j}_{m}$: \begin{equation} \mathbf{J}=\left(\mathbf{j}_{1},\ldots,\mathbf{j}_{\left|\mathbf{J}\right|}\right)^{T}=\left(\begin{array}{cccc} j_{1,1} & j_{1,2} & \cdots & j_{1,r}\\ j_{2,1} & j_{2,2} & \cdots & j_{2,r}\\ \vdots & \vdots & \ddots & \vdots\\ j_{\left|\mathbf{J}\right|,1} & j_{\left|\mathbf{J}\right|,2} & \cdots & j_{\left|\mathbf{J}\right|,r} \end{array}\right)\label{eq:J as a matrix} \end{equation} \vphantom{} II. We write $\textrm{String}^{r}\left(p\right)$\nomenclature{$\textrm{String}^{r}\left(p\right)$}{ } to denote the set of all finite length $p$-block strings of depth $r$. \vphantom{} III. Given any $\mathbf{J}\in\textrm{String}^{r}\left(p\right)$, the \textbf{composition}\index{composition sequence!multi-dimensional}\textbf{ sequence }$H_{\mathbf{J}}:\mathbb{Q}^{d}\rightarrow\mathbb{Q}^{d}$ \nomenclature{$H_{\mathbf{J}}\left(\mathbf{x}\right)$}{$\overset{\textrm{def}}{=}\left(H_{\mathbf{j}_{1}}\circ\cdots\circ H_{\mathbf{j}_{\left|\mathbf{J}\right|}}\right)\left(\mathbf{x}\right) $}is the map: \begin{equation} H_{\mathbf{J}}\left(\mathbf{x}\right)\overset{\textrm{def}}{=}\left(H_{\mathbf{j}_{1}}\circ\cdots\circ H_{\mathbf{j}_{\left|\mathbf{J}\right|}}\right)\left(\mathbf{x}\right),\textrm{ }\forall\mathbf{J}\in\textrm{String}^{r}\left(p\right),\textrm{ }\forall\mathbf{x}\in\mathbb{R}^{d}\label{eq:Def of composition sequence-1} \end{equation} \vphantom{} IV. We write $\textrm{String}_{\infty}^{r}\left(p\right)$ \nomenclature{$\textrm{String}_{\infty}^{r}\left(p\right)$}{ }to denote the set of all $p$-block strings of finite or infinite length. We also include the empty set as an element of $\textrm{String}_{\infty}^{r}\left(p\right)$, and call it the \textbf{empty $p$-block string}. \end{defn} \begin{rem} As $\left|\mathbf{J}\right|\rightarrow\infty$, we will have that the sequence consisting of the $m$th entry of $\mathbf{j}_{1}$ followed by the $m$th entry of $\mathbf{j}_{2}$, followed by $m$th entry of $\mathbf{j}_{3}$, and so on will be the digits of a $p_{m}$-adic integer. \end{rem} \begin{defn}[\textbf{Concatenation}] The\index{block string!concatenation} \textbf{concatenation operator} $\wedge:\textrm{String}^{r}\left(p\right)\times\textrm{String}_{\infty}^{r}\left(p\right)\rightarrow\textrm{String}_{\infty}^{r}\left(p\right)$ is defined by: \index{concatenation!operation!multi-dimensional} \begin{equation} \mathbf{J}\wedge\mathbf{K}=\left(\mathbf{j}_{1},\ldots,\mathbf{j}_{\left|\mathbf{J}\right|}\right)\wedge\left(\mathbf{k}_{1},\ldots,\mathbf{k}_{\left|\mathbf{K}\right|}\right)\overset{\textrm{def}}{=}\left(\mathbf{j}_{1},\ldots,\mathbf{j}_{\left|\mathbf{J}\right|},\mathbf{k}_{1},\ldots,\mathbf{k}_{\left|\mathbf{K}\right|}\right)\label{eq:Definition of Concatenation-1} \end{equation} for all $\mathbf{J}\in\textrm{String}^{r}\left(p\right)$ and $\mathbf{K}\in\textrm{String}_{\infty}^{r}\left(p\right)$. Also, for any integer $m\geq1$, if $\mathbf{J}\in\textrm{String}^{r}\left(p\right)$, we write $\mathbf{J}^{\wedge m}$ to denote the concatenation of $m$ copies of $\mathbf{J}$: \begin{equation} \mathbf{J}^{\wedge m}\overset{\textrm{def}}{=}\left(\underbrace{\mathbf{j}_{1},\ldots,\mathbf{j}_{\left|\mathbf{J}\right|},\mathbf{j}_{1},\ldots,\mathbf{j}_{\left|\mathbf{J}\right|},\ldots,\mathbf{j}_{1},\ldots,\mathbf{j}_{\left|\mathbf{J}\right|}}_{m\textrm{ times}}\right)^{T}\label{eq:Definition of block string concatenation} \end{equation} We shall only use this notation when $\mathbf{J}$ has finite length. Note that the empty block string is the identity element with respect to $\wedge$: \begin{equation} \mathbf{J}\wedge\varnothing=\varnothing\wedge\mathbf{J}=\mathbf{J} \end{equation} \end{defn} \begin{defn} We write $\left\Vert \cdot\right\Vert _{p}$ to denote the non-archimedean norm:\nomenclature{$\left\Vert \mathbf{z}\right\Vert _{p}$}{$\max\left\{ \left|\mathfrak{z}_{1}\right|_{p},\ldots,\left|\mathfrak{z}_{r}\right|_{p}\right\}$ } \begin{equation} \left\Vert \mathbf{z}\right\Vert _{p}\overset{\textrm{def}}{=}\max\left\{ \left|\mathfrak{z}_{1}\right|_{p},\ldots,\left|\mathfrak{z}_{r}\right|_{p}\right\} ,\textrm{ }\forall\mathbf{z}\in\mathbb{Z}_{p}^{r}\label{eq:Definition of p norm} \end{equation} which outputs the maximum of the $p$-adic absolute values of the entries of $\mathbf{z}$. Here \begin{equation} \left\Vert \mathbf{z}-\mathbf{w}\right\Vert _{p}\leq\max\left\{ \left\Vert \mathbf{z}\right\Vert _{p},\left\Vert \mathbf{w}\right\Vert _{p}\right\} \label{eq:p norm ultrametric inequality} \end{equation} with equality whenever $\left\Vert \mathbf{z}\right\Vert _{p}\neq\left\Vert \mathbf{w}\right\Vert _{p}$, and: \begin{equation} \left\Vert \mathbf{z}\mathbf{w}\right\Vert _{p}\leq\left\Vert \mathbf{z}\right\Vert _{p}\left\Vert \mathbf{w}\right\Vert _{p}\label{eq:Product property of p norm} \end{equation} \end{defn} \vphantom{} Next, we have notations for projections mod $p^{k}$ and for the multi-dimensional analogue of $\mathbb{Z}_{p}^{\prime}\backslash\mathbb{N}_{0}$. \begin{defn} For any $\mathbf{z}=\left(\mathfrak{z}_{1},\ldots,\mathfrak{z}_{r}\right)\in\mathbb{Z}_{p}^{r}$, we write: \nomenclature{$\left[\mathbf{z}\right]_{p^{k}}$}{$\overset{\textrm{def}}{=}\left(\left[\mathfrak{z}_{1}\right]_{p^{k}},\ldots,\left[\mathfrak{z}_{r}\right]_{p^{k}}\right)$ \nopageref} \begin{equation} \left[\mathbf{z}\right]_{p^{k}}\overset{\textrm{def}}{=}\left(\left[\mathfrak{z}_{1}\right]_{p^{k}},\ldots,\left[\mathfrak{z}_{r}\right]_{p^{k}}\right)\label{eq:definition of z-bar mod P^k} \end{equation} We also use the notation:\nomenclature{$\left(\mathbb{Z}_{p}^{r}\right)^{\prime}$}{$\overset{\textrm{def}}{=}\mathbb{Z}_{p}^{r}\backslash\mathbb{N}_{0}^{r}$ \nopageref} \begin{equation} \left(\mathbb{Z}_{p}^{r}\right)^{\prime}\overset{\textrm{def}}{=}\mathbb{Z}_{p}^{r}\backslash\mathbb{N}_{0}^{r}\label{eq:Definition of Z_P prime} \end{equation} \end{defn} \begin{defn} Given $\mathbf{J}\in\textrm{String}_{\infty}^{r}\left(p\right)$ and $\mathbf{z}=\left(\mathfrak{z}_{1},\ldots,\mathfrak{z}_{r}\right)\in\mathbb{Z}_{p}^{r}$ we say $\mathbf{J}$ \textbf{represents }(or \textbf{is} \textbf{associated to})\textbf{ }$\mathbf{z}$ and vice-versa\textemdash written $\mathbf{J}\sim\mathbf{z}$ or $\mathbf{z}\sim\mathbf{J}$\textemdash whenever the entries of the $k$th column of $\mathbf{J}$ represent the $p$-adic integer $\mathfrak{z}_{k}$ for all $k\in\left\{ 1,\ldots,r\right\} $ in the sense of Chapter 2: that is, for each $k$, the entries of the $k$th column of $\mathbf{J}$ are the sequence of the $p$-adic digits of $\mathfrak{z}_{k}$: \begin{equation} \mathbf{J}\sim\mathbf{z}\Leftrightarrow\mathbf{z}\sim\mathbf{J}\Leftrightarrow\mathfrak{z}_{k}=j_{1,k}+j_{2,k}p+j_{3,k}p^{2}+\cdots\textrm{ }\forall k\in\left\{ 1,\ldots,r\right\} \label{eq:Definition of n-bold-j correspondence.-1} \end{equation} We extend the relation $\sim$ to a binary relation on $\textrm{String}_{\infty}^{r}\left(p\right)$ by writing $\mathbf{J}\sim\mathbf{K}$ if and only if both $\mathbf{J}$ and $\mathbf{K}$ represent the same $d$-tuple $\mathbf{z}\in\mathbb{Z}_{p}^{r}$, and write \nomenclature{$\textrm{String}_{\infty}^{r}\left(p\right)/\sim$}{ } $\textrm{String}_{\infty}^{r}\left(p\right)/\sim$ to denote the set of all equivalence classes of $\textrm{String}_{\infty}^{r}\left(p\right)$ with respect to $\sim$. In particular, we identify any $p$-block string $\mathbf{J}$ of depth $r$ whose entries are all $0$s (such a string is called a \textbf{zero $p$-block string}) with the empty set, and view this particular equivalence class of $\textrm{String}_{\infty}^{r}\left(p\right)/\sim$ as representing the zero vector in $\mathbb{Z}_{p}^{r}$. \end{defn} \vphantom{} Similar to what we did in Chapter 2, every function on $\mathbb{Z}_{p}^{r}$ can be defined as a function of block strings. \begin{defn} For each $\mathbf{k}\in\mathbb{Z}^{r}/p\mathbb{Z}^{r}$, we define $\#_{\mathfrak{I}:\mathbf{k}}:\textrm{String}^{r}\left(p\right)\rightarrow\mathbb{N}_{0}$ by: \begin{equation} \#_{\mathfrak{I}:\mathbf{k}}\left(\mathbf{J}\right)\overset{\textrm{def}}{=}\left|\left\{ \mathbf{j}\in\mathbf{J}:\mathbf{j}=\mathbf{k}\right\} \right|,\textrm{ }\forall\mathbf{J}\in\textrm{String}^{r}\left(p\right)\label{eq:Definition of pound of big bold j} \end{equation} We also define $\#_{\mathfrak{I}:\mathbf{j}}$ as a function on $\mathbb{N}_{0}^{r}$ by writing: \begin{equation} \#_{\mathfrak{I}:\mathbf{k}}\left(\mathbf{n}\right)\overset{\textrm{def}}{=}\#_{\mathfrak{I}:\mathbf{k}}\left(\mathbf{J}\right)\label{eq:Definition of pound bold n} \end{equation} where $\mathbf{J}$ is the shortest element of $\textrm{String}^{r}\left(p\right)$ representing $\mathbf{n}$. \end{defn} \begin{defn} We define the function \nomenclature{$\lambda_{p}\left(\mathbf{n}\right)$}{ } $\lambda_{p}:\mathbb{N}_{0}^{r}\rightarrow\mathbb{N}_{0}$ by: \begin{equation} \lambda_{p}\left(\mathbf{n}\right)\overset{\textrm{def}}{=}\max_{1\leq\ell\leq r}\lambda_{p}\left(n_{\ell}\right)\label{eq:Definition of lambda P of bold n} \end{equation} Note that $\lambda_{p}\left(\mathbf{n}\right)$ is then the length of the shortest $p$-block string representing $\mathbf{n}$. \end{defn} \begin{prop} For any $\mathbf{z}\in\mathbb{Z}_{p}^{r}$ there exists a unique $\mathbf{J}\in\textrm{String}_{\infty}^{r}\left(p\right)$ of minimal length which represents $\mathbf{z}$. This $\mathbf{J}$ has finite length if and only if $\mathbf{z}=\mathbf{n}\in\mathbb{N}_{0}^{r}$, in which case, $\left|\mathbf{J}\right|=\min\left\{ 1,\lambda_{p}\left(\mathbf{n}\right)\right\} $ (the minimum is necessary to account for the case when $\mathbf{n}=\mathbf{0}$). \end{prop} Proof: We begin with the following special case: \begin{claim} Let $\mathbf{n}\in\mathbb{N}_{0}^{r}$. Then, there is a unique $\mathbf{J}\in\textrm{String}_{\infty}^{r}\left(p\right)$ of minimal length which represents $\mathbf{n}$. Proof of claim:\textbf{ }If $\mathbf{n}$ is the zero vector, then we can choose $\mathbf{J}$ to have length $1$ and contain only $0$s. So, suppose $\mathbf{n}$ is not the zero vector. Then $\lambda_{p}\left(\mathbf{n}\right)=\max_{1\leq k\leq r}\lambda_{p}\left(n_{k}\right)$ is positive. Using this, we construct the desired $\mathbf{J}$ like so: for any $k$ for which $\lambda_{p}\left(n_{k}\right)=L$, we write out in the $k$th column of $\mathbf{J}$ (starting from the top, proceeding downward) all of the $p$-adic digits of $n_{k}$. Next, for any $k$ for which $\lambda_{p}\left(n_{k}\right)<L$, we have that $j_{\ell,k}=0$ for all $\ell\in\left\{ \lambda_{p}\left(n_{k}\right)+1,\ldots,L\right\} $; that is, after we have written out all of the $p$-adic digits of $n_{k}$ in the $k$th column of $\mathbf{J}$, if there are still spaces further down in that column without entries, we fill those spaces with $0$s. As constructed, this $\mathbf{J}$ represents $\mathbf{n}$. Moreover, its length is minimal: if $\mathbf{J}^{\prime}$ is a block string of length $\left|\mathbf{J}\right|-1=L-1$, then, for any $k\in\left\{ 1,\ldots,r\right\} $ for which $\lambda_{p}\left(n_{k}\right)=L$, the $k$th column of $\mathbf{J}^{\prime}$ will fail to represent $n_{k}$. This also establishes $\mathbf{J}$'s uniqueness. This proves the claim. \vphantom{} \end{claim} Having proved the claim, using the density of $\mathbb{N}_{0}^{r}$ in $\mathbb{Z}_{p}^{r}$, given any $\mathbf{z}\in\mathbb{Z}_{p}^{r}$, let $\mathbf{J}$ be the limit (in the projective sense) of the sequence $\mathbf{J}_{1},\mathbf{J}_{2},\ldots\in\textrm{String}_{\infty}^{r}\left(p\right)$, where $\mathbf{J}_{m}$ is the unique minimal-length block string representing $\left[\mathbf{z}\right]_{p^{m}}$, the $r$-tuple whose entries are $\left[\mathfrak{z}_{k}\right]_{p^{m}}$. $\mathbf{J}$ will then be the unique block string representing $\mathbf{z}$. Q.E.D. \vphantom{} We can restate the above as: \begin{prop} The map which sends each $\mathbf{z}\in\mathbb{Z}_{p}^{r}$ to the unique string $\mathbf{J}\in\textrm{String}_{\infty}^{r}\left(p\right)$ of minimal length which represents $\mathbf{z}$ is a bijection from $\mathbb{Z}_{p}^{r}$ onto $\textrm{String}_{\infty}^{r}\left(p\right)/\sim$. \end{prop} \begin{rem} We identify the zero vector in $\mathbb{Z}_{p}^{r}$ with the empty string. On the other hand, for any non-zero vector $\mathbf{z}=\left(\mathfrak{z}_{1},\ldots,\mathfrak{z}_{r}\right)$ so that $\mathfrak{z}_{k}=0$ for some $k$, we have that the $k$th column of the unique $\mathbf{J}$ representing $\mathbf{z}$ will contain only $0$s as entries. \end{rem} \begin{prop} \label{prop:MD lambda and digit-number functional equations}$\lambda_{p}$ and the $\#_{\mathfrak{I}:\mathbf{k}}$s satisfy the \index{functional equation!lambda{p}@$\lambda_{p}$}functional equations: \begin{equation} \lambda_{p}\left(p\mathbf{n}+\mathbf{j}\right)=\lambda_{p}\left(\mathbf{n}\right)+1,\textrm{ }\forall\mathbf{n}\in\mathbb{N}_{0}^{r},\textrm{ }\forall\mathbf{j}\in\mathbb{Z}^{r}/p\mathbb{Z}^{r}\label{eq:lambda_P functional equation} \end{equation} \begin{equation} \#_{\mathfrak{I}:\mathbf{k}}\left(p\mathbf{n}+\mathbf{j}\right)=\begin{cases} \#_{\mathfrak{I}:\mathbf{k}}\left(\mathbf{n}\right)+1 & \textrm{if }\mathbf{j}=\mathbf{k}\\ \#_{\mathfrak{I}:\mathbf{k}}\left(\mathbf{n}\right) & \textrm{else} \end{cases},\textrm{ }\forall\mathbf{n}\in\mathbb{N}_{0}^{r},\textrm{ }\forall\mathbf{j}\in\mathbb{Z}^{r}/p\mathbb{Z}^{r}\label{eq:Pound of bold n functional equation} \end{equation} More generally, for any $k\geq1$, any $\mathbf{m},\mathbf{n}\in\mathbb{N}_{0}^{r}$ with $\lambda_{p}\left(\mathbf{n}\right)\leq k$: \begin{equation} \lambda_{p}\left(\mathbf{m}+p^{k}\mathbf{n}\right)=\lambda_{p}\left(\mathbf{m}\right)+\lambda_{p}\left(\mathbf{n}\right)\label{eq:lambda_P pseudoconcatenation identity} \end{equation} \begin{equation} \#_{\mathfrak{I}:\mathbf{k}}\left(\mathbf{m}+p^{k}\mathbf{n}\right)=\#_{\mathfrak{I}:\mathbf{k}}\left(\mathbf{m}\right)+\#_{\mathfrak{I}:\mathbf{k}}\left(\mathbf{n}\right)\label{eq:Pound of bold n pseudoconcatenation identity} \end{equation} \end{prop} \subsection{\label{subsec:5.2.2 Construction-of-the}Construction of the Numen} For any affine linear map $L:\mathbb{R}^{d}\rightarrow\mathbb{R}^{d}$ defined by: \begin{equation} L\left(\mathbf{x}\right)=\mathbf{A}\mathbf{x}+\mathbf{b} \end{equation} for a $d\times d$ matrix $\mathbf{A}$ and a $d\times1$ column vector $\mathbf{b}$ (both with real entries), we will define the derivative of $L$ to be the matrix $\mathbf{A}$. Thus, given any such $L$, we will write $L^{\prime}\left(\mathbf{0}\right)$ to denote the derivative of $L$. $L^{\prime}\left(\mathbf{0}\right)$ is then the unique $d\times d$ matrix so that: \begin{equation} L\left(\mathbf{x}\right)=L^{\prime}\left(\mathbf{0}\right)\mathbf{x}+\mathbf{b} \end{equation} In fact, we can write: \begin{equation} L\left(\mathbf{x}\right)=L^{\prime}\left(\mathbf{0}\right)\mathbf{x}+L\left(\mathbf{0}\right) \end{equation} Applying this formalism to a composition sequence $H_{\mathbf{J}}$\textemdash which is valid, since, as a composition of a sequence of the affine linear maps $H_{\mathbf{j}}:\mathbb{R}^{d}\rightarrow\mathbb{R}^{d}$\textemdash we see that $H_{\mathbf{J}}$ is an affine linear map on $\mathbb{R}^{d}$, with: \begin{equation} H_{\mathbf{J}}\left(\mathbf{x}\right)=H_{\mathbf{J}}^{\prime}\left(\mathbf{0}\right)\mathbf{x}+H_{\mathbf{J}}\left(\mathbf{0}\right),\textrm{ }\forall\mathbf{J}\in\textrm{String}^{r}\left(p\right)\label{eq:ax+b formula for H_bold_J} \end{equation} If $\mathbf{J}$ is a $p$-block string for which $H_{\mathbf{J}}\left(\mathbf{x}\right)=\mathbf{x}$, this becomes: \begin{equation} \mathbf{x}=H_{\mathbf{J}}^{\prime}\left(\mathbf{0}\right)\mathbf{x}+H_{\mathbf{J}}\left(\mathbf{0}\right)\label{eq:affine formula for bold x} \end{equation} So: \begin{equation} \mathbf{x}=\left(\mathbf{I}_{d}-H_{\mathbf{J}}^{\prime}\left(\mathbf{0}\right)\right)^{-1}H_{\mathbf{J}}\left(\mathbf{0}\right)\label{eq:Formula for bold x in terms of bold J} \end{equation} where $\mathbf{I}_{d}$ is a $d\times d$ identity matrix. Now for the definitions: \begin{defn}[\textbf{Multi-Dimensional $\chi_{H}$}] \label{def:MD Chi_H} \index{chi{H}@$\chi_{H}$!multi-dimensional}\nomenclature{$\chi_{H}\left(\mathbf{J}\right)$}{ }\nomenclature{$\chi_{H}\left(\mathbf{n}\right)$}{ }\ \vphantom{} I. We define $\chi_{H}:\textrm{String}^{r}\left(p\right)\rightarrow\mathbb{Q}^{d}$ by: \begin{equation} \chi_{H}\left(\mathbf{J}\right)\overset{\textrm{def}}{=}H_{\mathbf{J}}\left(\mathbf{0}\right),\textrm{ }\forall\mathbf{J}\in\textrm{String}^{r}\left(p\right)\label{eq:MD Definition of Chi_H of bold J} \end{equation} \vphantom{} II. For any $\mathbf{n}\in\mathbb{N}_{0}^{r}$, we write: \begin{equation} \chi_{H}\left(\mathbf{n}\right)\overset{\textrm{def}}{=}\chi_{H}\left(\mathbf{J}\right)=\begin{cases} H_{\mathbf{J}}\left(\mathbf{0}\right) & \textrm{if }\mathbf{n}\neq\mathbf{0}\\ \mathbf{0} & \textrm{else} \end{cases}\label{eq:MD Definition of Chi_H of n} \end{equation} where $\mathbf{J}$ is any element of $\textrm{String}^{r}\left(p\right)$ which represents $\mathbf{n}$. \end{defn} \begin{rem} Much like in the one-dimensional case, since $H_{\mathbf{0}}\left(\mathbf{0}\right)=\mathbf{0}$, it then follows that $\chi_{H}$ is then a well-defined function from $\textrm{String}^{r}\left(p\right)/\sim$ (equivalently, $\mathbb{N}_{0}^{r}$) to $\mathbb{Q}^{d}$. \end{rem} \begin{defn}[\textbf{Multi-Dimensional $M_{H}$}] We define $M_{H}:\mathbb{N}_{0}^{r}\rightarrow\textrm{GL}_{d}\left(\mathbb{Q}\right)$ by: \nomenclature{$M_{H}\left(\mathbf{J}\right)$}{$\overset{\textrm{def}}{=}H_{\mathbf{J}}^{\prime}\left(\mathbf{0}\right)$ }\nomenclature{$M_{H}\left(\mathbf{n}\right)$}{ }\index{$M_{H}$!multi-dimensional} \begin{equation} M_{H}\left(\mathbf{n}\right)\overset{\textrm{def}}{=}M_{H}\left(\mathbf{J}\right)\overset{\textrm{def}}{=}H_{\mathbf{J}}^{\prime}\left(\mathbf{0}\right),\textrm{ }\forall\mathbf{n}\in\mathbb{N}_{0}^{r}\backslash\left\{ \mathbf{0}\right\} \label{eq:MD Definition of M_H} \end{equation} where $\mathbf{J}\in\textrm{String}^{r}\left(p\right)$ is the unique minimal length block string representing $\mathbf{n}$. We define $M_{H}\left(\mathbf{0}\right)$ and $M_{H}\left(\varnothing\right)$ to be the $d\times d$ identity matrix $\mathbf{I}_{d}$, where $\mathbf{0}$ is the $r\times1$ zero vector and where $\varnothing$ is the empty block string. \end{defn} \vphantom{} In the one-dimensional case, the $q$-adic convergence of $\chi_{H}$ over $\mathbb{Z}_{p}$ was due to the $q$-adic decay of $M_{H}\left(\mathbf{j}\right)$ as $\left|\mathbf{j}\right|\rightarrow\infty$ with infinitely many non-zero entries. We will do the same in the multi-dimensional case, except with matrices. Unlike the one-dimensional case, however, the multi-dimensional case has to deal with the various ways in which the $\mathbf{A}_{\mathbf{j}}$s might interact with respect to multiplication. Like in the one-dimensional case, $M_{H}$ and $\chi_{H}$ will be characterized by functional equations and concatenation identities. \begin{prop}[\textbf{Functional Equations for Multi-Dimensional $M_{H}$}] \label{prop:MD M_H functional equations}\index{functional equation!$M_{H}$!multi-dimensional}\ \begin{equation} M_{H}\left(p\mathbf{n}+\mathbf{j}\right)=\mathbf{D}_{\mathbf{j}}^{-1}\mathbf{A}_{\mathbf{j}}M_{H}\left(\mathbf{n}\right),\textrm{ }\forall\mathbf{n}\in\mathbb{N}_{0}^{r},\textrm{ }\forall\mathbf{j}\in\mathbb{Z}^{r}/p\mathbb{Z}^{r}:p\mathbf{n}+\mathbf{j}\neq\mathbf{0}\label{eq:MD M_H functional equation} \end{equation} In terms of block strings and concatenations, we have: \begin{equation} M_{H}\left(\mathbf{J}\wedge\mathbf{K}\right)=M_{H}\left(\mathbf{J}\right)M_{H}\left(\mathbf{K}\right),\textrm{ }\forall\mathbf{J},\mathbf{K}\in\textrm{String}^{r}\left(p\right)\label{eq:MD M_H concatenation identity} \end{equation} \end{prop} \begin{rem} Because $M_{H}$ is matrix-valued, $M_{H}\left(\mathbf{J}\right)M_{H}\left(\mathbf{K}\right)$ is not generally equal to $M_{H}\left(\mathbf{K}\right)M_{H}\left(\mathbf{J}\right)$. However, if $H$ is $\mathbf{A}$-scalar, then: \begin{equation} M_{H}\left(\mathbf{J}\right)M_{H}\left(\mathbf{K}\right)=M_{H}\left(\mathbf{K}\right)M_{H}\left(\mathbf{J}\right) \end{equation} does hold for all $\mathbf{J},\mathbf{K}$. Consequently, \emph{when $H$ is $\mathbf{A}$-scalar, permuting the rows of $\mathbf{J}$ (a $\left|\mathbf{J}\right|\times r$ matrix) does not change the value of $M_{H}\left(\mathbf{J}\right)$}.\index{concatenation!identities!multi-dimensional} \end{rem} \begin{lem}[\textbf{Functional Equations for Multi-Dimensional $\chi_{H}$}] \label{lem:MD Chi_H functional equations and characterization}$\chi_{H}$ is the unique function $\mathbb{N}_{0}^{r}\rightarrow\mathbb{Q}^{d}$ satisfying the system of functional equations\index{functional equation!chi_{H}@$\chi_{H}$!multi-dimensional}: \begin{equation} \chi_{H}\left(p\mathbf{n}+\mathbf{j}\right)=\frac{\mathbf{A_{j}}\chi_{H}\left(\mathbf{n}\right)+\mathbf{b}_{\mathbf{j}}}{\mathbf{D}_{\mathbf{j}}},\textrm{ }\forall\mathbf{n}\in\mathbb{N}_{0}^{r},\textrm{ }\forall\mathbf{j}\in\mathbb{Z}^{r}/p\mathbb{Z}^{r}\label{eq:MD Chi_H functional equation} \end{equation} Equivalently: \begin{equation} \chi_{H}\left(p\mathbf{n}+\mathbf{j}\right)=H_{\mathbf{j}}\left(\chi_{H}\left(\mathbf{n}\right)\right),\textrm{ }\forall\mathbf{n}\in\mathbb{N}_{0}^{r},\textrm{ }\forall\mathbf{j}\in\mathbb{Z}^{r}/p\mathbb{Z}^{r}\label{eq:MD Chi_H functional equation, alternate statement} \end{equation} In terms of concatenations of block strings, the above are written as: \begin{equation} \chi_{H}\left(\mathbf{J}\wedge\mathbf{K}\right)=H_{\mathbf{J}}\left(\chi_{H}\left(\mathbf{K}\right)\right),\textrm{ }\forall\mathbf{J},\mathbf{K}\in\textrm{String}^{r}\left(p\right)\label{eq:MD Chi_H concatenation identity} \end{equation} \end{lem} Proof: For once, I'll leave this to the reader as an exercise. Q.E.D. \vphantom{} Like in the one-dimensional case, we will also need explicit formulae for $M_{H}$ and $\chi_{H}$. \begin{prop} \label{prop:H_bold J in terms of M_H and Chi_H, explicitly}For all $\mathbf{x}\in\mathbb{N}_{0}^{d}$ and all $\mathbf{J}\in\textrm{String}^{r}\left(p\right)$: \begin{equation} H_{\mathbf{J}}\left(\mathbf{x}\right)=\left(\prod_{k=1}^{\left|\mathbf{J}\right|}\frac{\mathbf{M}_{\mathbf{j}_{k}}}{\mathbf{R}}\right)\mathbf{x}+\sum_{n=1}^{\left|\mathbf{J}\right|}\left(\prod_{k=1}^{n-1}\frac{\mathbf{M}_{\mathbf{j}_{k}}}{\mathbf{R}}\right)H_{\mathbf{j}_{k}}\left(\mathbf{0}\right)\label{eq:MD H composition sequence formula} \end{equation} where the products are defined to be $\mathbf{I}_{d}$ whenever their upper limits of multiplication are $0$. \end{prop} Proof: Induction. Q.E.D. \vphantom{} Next up, the explicit, string-based formulae for $M_{H}$ and $\chi_{H}$. \begin{prop} \label{prop:Bold J formulae for M_H and Chi_H}\ \vphantom{} I. For all non-zero $\mathbf{n}\in\mathbb{N}_{0}^{r}$ and any $\mathbf{J}\in\textrm{String}^{r}\left(p\right)$ representing $\mathbf{n}$: \begin{equation} M_{H}\left(\mathbf{n}\right)=M_{H}\left(\mathbf{J}\right)=\prod_{k=1}^{\left|\mathbf{J}\right|}\frac{\mathbf{M}_{\mathbf{j}_{k}}}{\mathbf{R}}=\prod_{k=1}^{\left|\mathbf{J}\right|}\frac{\mathbf{A}_{\mathbf{j}_{k}}}{\mathbf{D}_{\mathbf{j}_{k}}}\label{eq:MD M_H formula} \end{equation} \vphantom{} II. For all $\mathbf{J}\in\textrm{String}^{r}\left(p\right)$: \begin{equation} \chi_{H}\left(\mathbf{J}\right)=\sum_{n=1}^{\left|\mathbf{J}\right|}\left(\prod_{k=1}^{n-1}\frac{\mathbf{M}_{\mathbf{j}_{k}}}{\mathbf{R}}\right)H_{\mathbf{j}_{n}}\left(\mathbf{0}\right)\label{eq:MD Chi_H formula} \end{equation} \end{prop} \begin{rem} All the above equalities hold in $\mathbb{Q}^{d,d}$, the space of $d\times d$ matrices with rational entries. \end{rem} Proof: Use (\ref{eq:MD H composition sequence formula}) along with the fact that: \begin{equation} H_{\mathbf{J}}\left(\mathbf{x}\right)=H_{\mathbf{J}}^{\prime}\left(\mathbf{0}\right)\mathbf{x}+H_{\mathbf{J}}\left(\mathbf{0}\right)=M_{H}\left(\mathbf{J}\right)\mathbf{x}+\chi_{H}\left(\mathbf{J}\right) \end{equation} Q.E.D. \vphantom{} Now we deal with the rising-continuation of $\chi_{H}$. Like in the one-dimensional case, $\chi_{H}$ will end up being a rising-continuous function, albeit multi-dimensionally so. The specifics of multi-dimensional rising-continuous functions are dealt with in Section \ref{sec:5.3 Rising-Continuity-in-Multiple}. Our current goal is to state the multi-dimensional generalizations of conditions like semi-basicness and basicness which were needed to guaranteed the rising-continuability of $\chi_{H}$ in the one-dimensional case. To motivate these, let us examine (\ref{eq:MD Chi_H formula}). Recall that the \emph{columns }of $\mathbf{J}$ are going to represent $p$-adic integers as $\left|\mathbf{J}\right|\rightarrow\infty$; that is: \begin{equation} \mathbf{J}\rightarrow\left.\left[\begin{array}{cccc} \mid & \mid & & \mid\\ \mathfrak{z}_{1} & \mathfrak{z}_{2} & \cdots & \mathfrak{z}_{r}\\ \mid & \mid & & \mid \end{array}\right]\right\} \textrm{ infinitely many rows} \end{equation} where the $m$th column on the right contains the digits of the $p$-adic integer $\mathfrak{z}_{m}$; the $k$th row on the right, recall, is $\mathbf{j}_{k}$. In particular, the length of $\mathbf{J}$ is infinite if and only if at least one of the $\mathfrak{z}_{m}$s is an element of $\mathbb{Z}_{p}^{\prime}$. This, in turn, will occur if and only if said $\mathfrak{z}_{m}$ has \emph{infinitely many non-zero $p$-adic digits}, which occurs if and only if $\lambda_{p}\left(\mathbf{J}\right)\rightarrow\infty$. Applying $\chi_{H}$ to $\mathbf{J}$, we see that: \begin{equation} \chi_{H}\left(\left[\begin{array}{cccc} \mid & \mid & & \mid\\ \mathfrak{z}_{1} & \mathfrak{z}_{2} & \cdots & \mathfrak{z}_{r}\\ \mid & \mid & & \mid \end{array}\right]\right)=\sum_{n=1}^{\left|\mathbf{J}\right|}\left(\prod_{k=1}^{n-1}\frac{\mathbf{M}_{\mathbf{j}_{k}}}{\mathbf{R}}\right)H_{\mathbf{j}_{k}}\left(\mathbf{0}\right) \end{equation} So, $\chi_{H}$ will extend to a function on $\mathbb{Z}_{p}^{r}$, with the convergence of the series on the right occurring because at least one of the $\mathfrak{z}_{m}$s has infinitely many non-zero $p$-adic digits. To deal with the convergence of sums of products of matrices, let briefly recall how convergence issues like this work in spaces of matrices. \begin{prop}[\textbf{Convergence for Matrix Sums and Matrix Products}] Let $K$ be a metrically complete non-archimedean valued field, let $d\geq2$, and let $\left\{ \mathbf{M}_{n}\right\} _{n\geq1}$ be a sequence in $K^{d,d}$ ($d\times d$ matrices with entries in $K$). Then, the partial products: \begin{equation} \prod_{n=1}^{N}\mathbf{M}_{n} \end{equation} converge in $K^{d,d}$ to the $d\times d$ zero matrix $\mathbf{O}_{d}$ whenever $\prod_{n=1}^{N}\left\Vert \mathbf{M}_{n}\right\Vert _{K}\rightarrow0$ in $\mathbb{R}$ as $N\rightarrow\infty$, where $\left\Vert \mathbf{M}_{n}\right\Vert _{K}$ is the maximum of the $K$-absolute-values of the entries of $\mathbf{M}_{n}$.\index{matrix!infinite product}\index{series!of matrices}\index{matrix!infinite series} Likewise, since $K$ is non-archimedean, we have that a matrix series: \begin{equation} \sum_{n=1}^{\infty}\mathbf{M}_{n} \end{equation} converges entry-wise in $K$ to an element of $K^{d,d}$ if and only if $\left\Vert \mathbf{M}_{n}\right\Vert _{K}\rightarrow0$. \end{prop} Proof: $\left(K^{d,d},\left\Vert \cdot\right\Vert _{K}\right)$ is a finite-dimensional Banach space over $K$. Q.E.D. \vphantom{} To cleanly state the multi-dimensional analogue of what it means for $H$ to be contracting, we will need to use the proposition given below. Recall that we write $\left\Vert \cdot\right\Vert _{\infty}$ to denote the maximum of the ordinary, complex absolute value of the entries of a matrix or vector. \begin{prop} \label{prop:MD contracting proposition}Let $\mathbf{D}$ and $\mathbf{P}$ be invertible $d\times d$ matrices with rational entries, with $\mathbf{D}$ being diagonal and $\mathbf{P}$ being a permutation matrix, representing a permutation of order $\omega$. Then, $\lim_{n\rightarrow\infty}\left\Vert \left(\mathbf{DP}\right)^{n}\right\Vert _{\infty}=0$ whenever $\left\Vert \left(\mathbf{D}\mathbf{P}\right)^{\omega}\right\Vert _{\infty}<1$. \end{prop} Proof: As given, $\omega$ is the smallest positive integer so that $\mathbf{P}^{\omega}=\mathbf{I}_{d}$. Now, letting $\mathbf{v}\in\mathbb{Q}^{d}$ be arbitrary observe that there will be an invertible diagonal matrix $\mathbf{D}_{\omega}$ so that: \begin{equation} \left(\mathbf{D}\mathbf{P}\right)^{\omega}\mathbf{v}=\mathbf{D}_{\omega}\mathbf{v} \end{equation} This is because each time we apply $\mathbf{D}\mathbf{P}$ to $\mathbf{v}$, we permute the entries of $\mathbf{v}$ by $\mathbf{P}$ and then multiply by $\mathbf{D}$. Since $\mathbf{P}$ has order $\omega$, if we label the entries of $\mathbf{v}$ as $v_{1},\ldots,v_{d}$, the positions of the $v_{j}$s in $\mathbf{v}$ will return to $v_{1},\ldots,v_{d}$ only after we have multiplied $\mathbf{v}$ by $\mathbf{P}$ $\omega$ times. In each application of $\mathbf{D}\mathbf{P}$, the $v_{j}$s accrue a non-zero multiplicative factor\textemdash{} courtesy of $\mathbf{D}$. So, after $\omega$ applications of $\mathbf{D}\mathbf{P}$, all of the entries of $\mathbf{v}$ will be returned to the original places, albeit with their original values having been multiplied by constants chosen from the diagonal entries of $\mathbf{D}$. These constants are then precisely the diagonal entries of $\mathbf{D}_{\omega}$. Since $\mathbf{v}$ was arbitrary, we conclude that $\left(\mathbf{D}\mathbf{P}\right)^{\omega}=\mathbf{D}_{\omega}$. Now, letting the integer $n\geq0$ be arbitrary, we can write $n$ as $\omega\left\lfloor \frac{n}{\omega}\right\rfloor +\left[n\right]_{\omega}$, where $\left\lfloor \frac{n}{\omega}\right\rfloor $ is the largest integer $\leq n/\omega$. As such: \begin{equation} \left\Vert \left(\mathbf{D}\mathbf{P}\right)^{n}\right\Vert _{\infty}\leq\left\Vert \underbrace{\left(\mathbf{D}\mathbf{P}\right)^{\omega}}_{\mathbf{D}_{\omega}}\right\Vert _{\infty}^{\left\lfloor \frac{n}{\omega}\right\rfloor }\left\Vert \mathbf{D}\right\Vert _{\infty}^{\left[n\right]_{\omega}}\underbrace{\left\Vert \mathbf{P}\right\Vert _{\infty}^{\left[n\right]_{\omega}}}_{1} \end{equation} Since $\left[n\right]_{\omega}$ only takes values in $\left\{ 0,\ldots,\omega-1\right\} $, the quantity $\left\Vert \mathbf{D}\right\Vert _{\infty}^{\left[n\right]_{\omega}}$ is uniformly bounded with respect to $n$, we can write: \begin{equation} \left\Vert \left(\mathbf{D}\mathbf{P}\right)^{n}\right\Vert _{\infty}\ll\left\Vert \mathbf{D}_{\omega}\right\Vert _{\infty}^{\left\lfloor \frac{n}{\omega}\right\rfloor } \end{equation} The upper bounds tends to zero as $n\rightarrow\infty$ whenever $\left\Vert \mathbf{D}_{\omega}\right\Vert _{\infty}<1$. Q.E.D. \vphantom{} Below, we formulate the analogues of properties such as monogenicity, degeneracy, simplicity, and the like for multi-dimensional Hydra maps. \begin{defn}[\textbf{Qualitative Terminology for Multi-Dimensional Hydra Maps}] \label{def:qualitative definitions for MD hydra maps}Let $H$ be a $P$-Hydra map, where $P=\left(p_{1},\ldots,p_{r}\right)$ where the $p_{\ell}$s are possibly distinct, and possibly non-prime. \vphantom{} I. The \textbf{ground }of $H$ is\index{Hydra map!ground} the set: \begin{equation} \textrm{Gr}\left(H\right)\overset{\textrm{def}}{=}\left\{ p\textrm{ prime}:p\mid p_{m}\textrm{ for some }m\in\left\{ 1,\ldots,r\right\} \right\} \label{eq:Definition of the Ground of H} \end{equation} \vphantom{} II. I say $H$ is \textbf{rough }if $H$ is not smooth. In particular, $H$ is called: i. \textbf{Weakly rough }if $H$ is rough and $\left|\textrm{Gr}\left(H\right)\right|=1$; \vphantom{} ii. \textbf{Strongly rough }if $H$ is rough and $\left|\textrm{Gr}\left(H\right)\right|\geq2$. \vphantom{} III.\index{Hydra map!degenerate} We say $H$ is \textbf{degenerate }if $\mathbf{A}_{\mathbf{j}}$ is a permutation matrix (including possibly the identity matrix) for some $\mathbf{j}\in\left(\mathbb{Z}^{r}/P\mathbb{Z}^{r}\right)\backslash\left\{ \mathbf{0}\right\} $. We say $H$ is \textbf{non-degenerate }whenever it is not degenerate. \vphantom{} IV. We say $H$ is \textbf{semi-simple }\index{Hydra map!semi-simple}whenever, for any $\mathbf{j},\mathbf{k}\in\mathbb{Z}^{r}/P\mathbb{Z}^{r}$, every non-zero entry in $\mathbf{A}_{\mathbf{j}}$ is co-prime to every non-zero entry in $\mathbf{D}_{\mathbf{k}}$. If, in addition: \begin{equation} \mathbf{D}_{\mathbf{j}}=\left(\begin{array}{cccccc} p_{1}\\ & \ddots\\ & & p_{r}\\ & & & 1\\ & & & & \ddots\\ & & & & & 1 \end{array}\right),\textrm{ }\forall\mathbf{j}\in\mathbb{Z}^{r}/P\mathbb{Z}^{r}\label{eq:MD Definition of simplicity} \end{equation} we then say $H$ is \textbf{simple}. \vphantom{} V. We say $H$ is \textbf{monogenic }\index{Hydra map!monogenic}if there exists a prime $q\notin\textrm{Gr}\left(H\right)$ so that $\left\Vert \mathbf{A}_{\mathbf{j}}\right\Vert _{q}\leq1/q$ for all $\mathbf{j}\in\left(\mathbb{Z}^{r}/P\mathbb{Z}^{r}\right)\backslash\left\{ \mathbf{0}\right\} $. We write $q_{H}$ to denote the smallest such prime. We say $H$ \index{Hydra map!polygenic} is \textbf{polygenic }if $H$ is not monogenic, but there exists a set $Q$ primes, with $Q\cap\textrm{Gr}\left(H\right)=\varnothing$ so that for each $\mathbf{j}\in\left(\mathbb{Z}^{r}/P\mathbb{Z}^{r}\right)\backslash\left\{ \mathbf{0}\right\} $, there exists a $q\in Q$ for which $\left\Vert \mathbf{A}_{\mathbf{j}}\right\Vert _{q}\leq1/q$. \vphantom{} VI.\footnote{This definition will not be relevant until Chapter 6.} For each $\mathbf{j}\in\mathbb{Z}^{r}/P\mathbb{Z}^{r}$, we write $\omega_{\mathbf{j}}$ to denote the order of the permutation represented by the permutation matrix $\mathbf{P}_{\mathbf{j}}$, where $\mathbf{A}_{\mathbf{j}}=\tilde{\mathbf{A}}_{\mathbf{j}}\mathbf{P}_{\mathbf{j}}$, where $\tilde{\mathbf{A}}_{\mathbf{j}}$ is diagonal. We say $H$ \index{Hydra map!contracting} is \textbf{contracting }whenever $\lim_{n\rightarrow\infty}\left\Vert \left(H^{\prime}\left(\mathbf{0}\right)\right)^{n}\right\Vert _{\infty}=0$; recall that $H^{\prime}\left(\mathbf{0}\right)=\mathbf{D}_{\mathbf{0}}^{-1}\mathbf{A}_{\mathbf{0}}$. By \textbf{Proposition \ref{prop:MD contracting proposition}}, it follows that $H$ is contracting whenever $\left\Vert \left(H^{\prime}\left(\mathbf{0}\right)\right)^{\omega_{\mathbf{0}}}\right\Vert _{\infty}<1$. \vphantom{} VII. We say $H$ \index{Hydra map!basic}is \textbf{basic }if $H$ is simple, non-degenerate, and monogenic. \vphantom{} VIII. We say $H$ is \textbf{semi-basic }if \index{Hydra map!semi-basic}$H$ is semi-simple, non-degenerate, and monogenic. \end{defn} \begin{rem} For brevity, we will sometimes write $q$ to denote $q_{H}$. More generally, in any context where a Hydra map is being worked with\textemdash unless explicitly stated otherwise\textemdash $q$ will \emph{always }denote $q_{H}$. \end{rem} \begin{rem} Like in the one-dimensional case, everything given in this subsection holds when $q_{H}$ is replaced by any prime $q\neq p$ such that $\left\Vert \mathbf{A}_{\mathbf{j}}\right\Vert _{q}\leq1/q$ for all $\mathbf{j}\in\left(\mathbb{Z}^{r}/p\mathbb{Z}^{r}\right)\backslash\left\{ \mathbf{0}\right\} $. \end{rem} \begin{rem} Like with polygenicity in the one-dimensional case, I would like to believe that frames can be used to provide a setting where we can speak meaningfully about the $p$-adic interpolation of $\chi_{H}$ in the case of a $p$-smooth polygenic multi-dimensional Hydra map. \end{rem} \begin{prop} \label{prop:MD Contracting H proposition}If $H$ is contracting, the matrix $\mathbf{I}_{d}-H^{\prime}\left(\mathbf{0}\right)$ is invertible. \end{prop} Proof: If $H$ is contracting, there is an integer $\omega_{\mathbf{0}}\geq1$ so that the entries of the matrix $\left(H^{\prime}\left(\mathbf{0}\right)\right)^{\omega_{\mathbf{0}}}$ are all rational numbers with archimedean absolute value $<1$. Consequently, letting $\mathbf{X}$ denote $H^{\prime}\left(\mathbf{0}\right)$: \begin{align*} \sum_{n=0}^{\infty}\mathbf{X}^{n} & =\sum_{k=0}^{\omega_{\mathbf{0}}-1}\sum_{n=0}^{\infty}\mathbf{X}^{\omega_{\mathbf{0}}n+k}\\ & =\sum_{k=0}^{\omega_{\mathbf{0}}-1}\mathbf{X}^{k}\sum_{n=0}^{\infty}\mathbf{X}^{\omega_{\mathbf{0}}n}\\ \left(\left\Vert \mathbf{X}^{\omega_{\mathbf{0}}}\right\Vert _{\infty}<1\right); & \overset{\mathbb{R}^{d,d}}{=}\sum_{k=0}^{\omega_{\mathbf{0}}-1}\mathbf{X}^{k}\left(\mathbf{I}_{d}-\mathbf{X}^{\omega_{\mathbf{0}}}\right)^{-1} \end{align*} As such, the series $\sum_{n=0}^{\infty}\mathbf{X}^{n}$ converges in $\mathbb{R}^{d,d}$, and therefore defines a matrix $\mathbf{M}\in\mathbb{R}^{d,d}$. Since: \[ \left(\mathbf{I}_{d}-\mathbf{X}\right)\mathbf{M}=\mathbf{M}\left(\mathbf{I}_{d}-\mathbf{X}\right)=\mathbf{I}_{d} \] this proves that $\mathbf{M}=\left(\mathbf{I}_{d}-\mathbf{X}\right)^{-1}$, showing that $\mathbf{I}_{d}-H^{\prime}\left(\mathbf{0}\right)$ is invertible. Q.E.D. \vphantom{} Like in the one-dimensional case, monogenicity will be needed to ensure the existence of a rising-continuation of $\chi_{H}$. \begin{prop} \label{prop:MD M_H decay estimate}Let $H$ be monogenic. Then: \textup{ \begin{equation} \left\Vert M_{H}\left(\mathbf{J}\right)\right\Vert _{q_{H}}=\left\Vert \prod_{k=1}^{\left|\mathbf{J}\right|}\frac{\mathbf{M}_{\mathbf{j}_{k}}}{\mathbf{R}}\right\Vert _{q_{H}}\leq q_{H}^{-\#\left(\mathbf{J}\right)},\textrm{ }\forall\mathbf{J}\in\textrm{String}^{r}\left(p\right)\label{eq:Decay estimate on M_H of big bold j} \end{equation} }This result also holds when we replace $\mathbf{J}$ by the unique $\mathbf{n}\in\mathbb{N}_{0}^{r}$ for which $\mathbf{J}\sim\mathbf{n}$. \end{prop} Proof: Since $\mathbf{M}_{\mathbf{j}}=\mathbf{R}\mathbf{D}_{\mathbf{j}}^{-1}\mathbf{A}_{\mathbf{j}}$, and since the entries of $\mathbf{R}$ and $\mathbf{D}_{\mathbf{j}}$ are co-prime to $q_{H}$, we have that: \begin{equation} \left\Vert \frac{\mathbf{M}_{\mathbf{j}}}{\mathbf{R}}\right\Vert _{q_{H}}\leq\left\Vert \mathbf{A}_{\mathbf{j}}\right\Vert _{q_{H}} \end{equation} Since $H$ is monogenic, $\left\Vert \mathbf{A}_{\mathbf{j}}\right\Vert _{q_{H}}\leq1/q_{H}$ whenever $\mathbf{j}\neq\mathbf{0}$. Since $M_{H}\left(\mathbf{J}\right)=\prod_{k=1}^{\left|\mathbf{J}\right|}\frac{\mathbf{M}_{\mathbf{j}_{k}}}{\mathbf{R}}$, (\ref{eq:Decay estimate on M_H of big bold j}) then holds. Q.E.D. \vphantom{} Now, the all-important characterization of $\chi_{H}$'s interpolation as the unique rising-continuous solution of a certain system of functional equations. \begin{lem}[\textbf{$\left(p,q\right)$-adic Characterization of $\chi_{H}$}] \label{lem:MD rising-continuation and functional equations of Chi_H}Let $H$ be monogenic. Then, the limit:\nomenclature{$\chi_{H}\left(\mathbf{z}\right)$}{ } \begin{equation} \chi_{H}\left(\mathbf{z}\right)\overset{\mathbb{Z}_{q_{H}}^{d}}{=}\lim_{n\rightarrow\infty}\chi_{H}\left(\left[\mathbf{z}\right]_{p^{n}}\right)\label{eq:MD Rising Continuity Formula for Chi_H} \end{equation} exists for all $\mathbf{z}\in\mathbb{Z}_{p}^{r}$, and thereby defines a continuation of $\chi_{H}$ to a function $\mathbb{Z}_{p}^{r}\rightarrow\mathbb{Z}_{q_{H}}^{d}$. Moreover:\index{functional equation!chi_{H}@$\chi_{H}$!multi-dimensional} \vphantom{} I. \begin{equation} \chi_{H}\left(p\mathbf{z}+\mathbf{j}\right)\overset{\mathbb{Z}_{q_{H}}^{d}}{=}\frac{\mathbf{A_{j}}\chi_{H}\left(\mathbf{z}\right)+\mathbf{b}_{\mathbf{j}}}{\mathbf{D}_{\mathbf{j}}},\textrm{ }\forall\mathbf{z}\in\mathbb{Z}_{p}^{r},\textrm{ }\forall\mathbf{j}\in\mathbb{Z}^{r}/p\mathbb{Z}^{r}\label{eq:MD Functional Equations for Chi_H over the rho-bar-adics} \end{equation} \vphantom{} II. \emph{(\ref{eq:MD Rising Continuity Formula for Chi_H})} is the unique function $\mathbf{f}:\mathbb{Z}_{p}^{r}\rightarrow\mathbb{Z}_{q_{H}}^{d}$ satisfying both\emph{: \begin{equation} \mathbf{f}\left(p\mathbf{z}+\mathbf{j}\right)\overset{\mathbb{Z}_{q_{H}}^{d}}{=}\frac{\mathbf{A_{j}}\mathbf{f}\left(\mathbf{z}\right)+\mathbf{b}_{\mathbf{j}}}{\mathbf{D}_{\mathbf{j}}},\textrm{ }\forall\mathbf{z}\in\mathbb{Z}_{p}^{r},\textrm{ }\forall\mathbf{j}\in\mathbb{Z}^{r}/p\mathbb{Z}^{r}\label{eq:MD extension Lemma for Chi_H - functional equation} \end{equation} and: \begin{equation} \mathbf{f}\left(\mathbf{z}\right)\overset{\mathbb{Z}_{q_{H}}^{d}}{=}\lim_{n\rightarrow\infty}\mathbf{f}\left(\left[\mathbf{z}\right]_{p^{n}}\right)\label{eq:MD extension Lemma for Chi_H - rising continuity} \end{equation} } \end{lem} Proof: Let $\mathbf{z}\in\mathbb{Z}_{p}^{r}$ and $N\geq0$ be arbitrary. Then, choose: \begin{equation} \mathbf{J}=\left[\begin{array}{ccc} \mathfrak{z}_{1,0} & \cdots & \mathfrak{z}_{r,0}\\ \mathfrak{z}_{1,1} & \cdots & \mathfrak{z}_{r,1}\\ \vdots & & \vdots\\ \mathfrak{z}_{1,N-1} & \cdots & \mathfrak{z}_{r,N-1} \end{array}\right]=\left[\begin{array}{ccc} \mid & & \mid\\ \left[\mathfrak{z}_{1}\right]_{p^{N}} & \cdots & \left[\mathfrak{z}_{r}\right]_{p^{N}}\\ \mid & & \mid \end{array}\right]\sim\left[\mathbf{z}\right]_{^{N}} \end{equation} where the $m$th column's entries are the first $N$ $p$-adic digits of $\mathfrak{z}_{m}$. Then, taking (\ref{eq:MD Chi_H formula}) from \textbf{Proposition \ref{prop:Bold J formulae for M_H and Chi_H}}: \begin{equation} \chi_{H}\left(\mathbf{J}\right)=\sum_{n=1}^{\left|\mathbf{J}\right|}\left(\prod_{k=1}^{n-1}\frac{\mathbf{M}_{\mathbf{j}_{k}}}{\mathbf{R}}\right)H_{\mathbf{j}_{k}}\left(\mathbf{0}\right) \end{equation} we obtain: \begin{equation} \chi_{H}\left(\left[\mathbf{z}\right]_{p^{N}}\right)=\sum_{n=1}^{N}\left(\prod_{k=1}^{n-1}\frac{\mathbf{M}_{\left(\mathfrak{z}_{1,k},\ldots,\mathfrak{z}_{r,k}\right)}}{\mathbf{R}}\right)H_{\left(\mathfrak{z}_{1,n},\ldots,\mathfrak{z}_{r,n}\right)}\left(\mathbf{0}\right)\label{eq:MD Chi_H formula for truncation of z-arrow} \end{equation} If $\mathbf{z}\in\mathbb{N}_{0}^{r}$, then $\left(\mathfrak{z}_{1,n},\ldots,\mathfrak{z}_{r,n}\right)$ must be a tuple of $0$s for all sufficiently large $n$. Since $H$ fixes $\mathbf{0}$, we know that $\mathbf{b}_{\mathbf{0}}=\mathbf{0}$. As such, the $n$th terms of (\ref{eq:MD Chi_H formula for truncation of z-arrow}) are then $\mathbf{0}$ for all sufficiently large $n$, which shows that (\ref{eq:MD Rising Continuity Formula for Chi_H}) holds for $\mathbf{z}\in\mathbb{N}_{0}^{r}$. On the other hand, suppose at least one entry of $\mathbf{z}$ is not in $\mathbb{N}_{0}$. Then, as $n\rightarrow\infty$, in the matrix product: \begin{equation} \prod_{k=1}^{n-1}\frac{\mathbf{M}_{\mathbf{j}_{k}}}{\mathbf{R}} \end{equation} there will be infinitely many values of $k$ for which the $r$-tuple $\mathbf{j}=\left(\mathfrak{z}_{1,k},\ldots,\mathfrak{z}_{r,k}\right)$ has a non-zero entry. Since $H$ is monogenic, the decay estimate given by \textbf{Proposition \ref{prop:MD M_H decay estimate}} guarantees that: \begin{equation} \left\Vert \prod_{k=1}^{n-1}\frac{\mathbf{M}_{\mathbf{j}_{k}}}{\mathbf{R}}\right\Vert _{q_{H}}\leq q_{H}^{-\left|\left\{ k\in\left\{ 1,\ldots,n-1\right\} :\mathbf{j}_{k}\neq0\right\} \right|} \end{equation} which tends to $0$ as $n\rightarrow\infty$. This shows that (\ref{eq:MD Chi_H formula for truncation of z-arrow}) converges point-wise in $\mathbb{Z}_{q_{H}}^{d}$ to a limit. Since (\ref{eq:MD Functional Equations for Chi_H over the rho-bar-adics}) holds for all $\mathbf{z}\in\mathbb{N}_{0}^{r}$, the limit (\ref{eq:MD Functional Equations for Chi_H over the rho-bar-adics}) then shows that (\ref{eq:MD Functional Equations for Chi_H over the rho-bar-adics}) in fact holds for all $\mathbf{z}\in\mathbb{Z}_{p}^{r}$, thereby proving (I). Finally, \textbf{Lemma \ref{lem:MD Chi_H functional equations and characterization} }shows that any solution $\mathbf{f}$ of (\ref{eq:MD extension Lemma for Chi_H - functional equation}) must be equal to $\chi_{H}$ on $\mathbb{N}_{0}^{r}$. If $\mathbf{f}$ also satisfies (\ref{eq:MD extension Lemma for Chi_H - rising continuity}), this then shows that, just like $\chi_{H}$'s extension to $\mathbb{Z}_{p}^{r}$, the extension of $\mathbf{f}$ to $\mathbb{Z}_{p}^{r}$ is uniquely determined by $\mathbf{f}$'s values on $\mathbb{N}_{0}^{r}$. Since $\mathbf{f}=\chi_{H}$ on $\mathbb{N}_{0}^{r}$, this then forces $\mathbf{f}=\chi_{H}$ on $\mathbb{Z}_{p}^{r}$, proving the uniqueness of $\chi_{H}$. Q.E.D. \subsection{\label{subsec:5.2.3 The-Correspondence-Principle}The Correspondence Principle Revisited} THROUGHOUT THIS SUBSECTION, WE ALSO ASSUME $H$ IS INTEGRAL, UNLESS STATED OTHERWISE. \vphantom{} Like in the one-dimensional case, we start with a self-concatenation identity. \begin{prop} \label{prop:MD self-concatenation identity}Let $\mathbf{n}\in\mathbb{N}_{0}^{r}$ and let $\mathbf{J}\in\textrm{String}^{r}\left(p\right)$ be the shortest block string representing. Then, for all $k\in\mathbb{N}_{1}$: \begin{equation} \left(\mathbf{j}_{1}^{\wedge k},\ldots,\mathbf{j}_{\left|\mathbf{J}\right|}^{\wedge k}\right)^{T}=\mathbf{J}^{\wedge k}\sim\frac{1-p^{k\lambda_{p}\left(\mathbf{n}\right)}}{1-p^{\lambda_{p}\left(\mathbf{n}\right)}}\mathbf{n}\overset{\textrm{def}}{=}\left(n_{1}\frac{1-p^{k\lambda_{p}\left(n_{1}\right)}}{1-p^{\lambda_{p_{}}\left(n_{1}\right)}},\ldots,n_{r}\frac{1-p^{k\lambda_{p}\left(n_{r}\right)}}{1-p^{\lambda_{p}\left(n_{r}\right)}}\right)\label{eq:MD Self-Concatentation Identity} \end{equation} \end{prop} Proof: Apply \textbf{Proposition \ref{prop:Concatenation exponentiation}} to each row of $\mathbf{J}^{\wedge k}$. Q.E.D. \vphantom{}Next come the multi-dimensional analogues of $B_{p}$ and the functional equation satisfied by $\chi_{H}\circ B_{p}$. \begin{defn}[$\mathbf{B}_{p}$] We define $\mathbf{B}_{p}:\mathbb{Z}_{p}^{r}\rightarrow\mathbb{Z}_{p}^{r}$ by:\nomenclature{$\mathbf{B}_{p}\left(\mathbf{z}\right)$}{$\overset{\textrm{def}}{=}\left(B_{p}\left(\mathfrak{z}_{1}\right),\ldots,B_{p}\left(\mathfrak{z}_{m}\right)\right)$} \begin{equation} \mathbf{B}_{p}\left(\mathbf{z}\right)\overset{\textrm{def}}{=}\left(B_{p}\left(\mathfrak{z}_{1}\right),\ldots,B_{p}\left(\mathfrak{z}_{m}\right)\right)\label{eq:MD Definition of B projection function} \end{equation} where, recall: \begin{equation} B_{p}\left(\mathfrak{z}\right)=\begin{cases} \mathfrak{z} & \textrm{if }\mathfrak{z}\in\mathbb{Z}_{p}^{\prime}\\ \frac{\mathfrak{z}}{1-p^{\lambda_{p}\left(\mathfrak{z}\right)}} & \textrm{if }\mathfrak{z}\in\mathbb{N}_{0} \end{cases} \end{equation} \end{defn} \begin{lem}[\textbf{Functional Equation for $\chi_{H}\circ\mathbf{B}_{p}$}] \label{lem:MD Chi_H o B functional equation}Let\index{functional equation!chi{H}circmathbf{B}{p}@$\chi_{H}\circ\mathbf{B}_{p}$} $H$ be monogenic. Then: \begin{equation} \chi_{H}\left(\mathbf{B}_{p}\left(\mathbf{n}\right)\right)\overset{\mathbb{Z}_{q_{H}}^{d}}{=}\left(\mathbf{I}_{d}-M_{H}\left(\mathbf{n}\right)\right)^{-1}\chi_{H}\left(\mathbf{n}\right),\textrm{ }\forall\mathbf{n}\in\mathbb{N}_{0}^{r}\backslash\left\{ \mathbf{0}\right\} \label{eq:MD Chi_H B functional equation} \end{equation} Moreover, both sides of the above identity are $d\times1$ column vectors with entries in $\mathbb{Q}$. \end{lem} Proof: Let $\mathbf{n}\in\mathbb{N}_{0}^{r}\backslash\left\{ \mathbf{0}\right\} $, and let $\mathbf{J}\in\textrm{String}^{r}\left(p\right)$ be the shortest block string representing $\mathbf{n}$. Then: \begin{equation} H_{\mathbf{J}^{\wedge k}}\left(\mathbf{m}\right)=H_{\mathbf{J}}^{\circ k}\left(\mathbf{m}\right),\textrm{ }\forall\mathbf{m}\in\mathbb{Z}^{d},\textrm{ }\forall k\geq1 \end{equation} Since: \begin{equation} H_{\mathbf{J}}\left(\mathbf{m}\right)=M_{H}\left(\mathbf{J}\right)\mathbf{m}+\chi_{H}\left(\mathbf{J}\right) \end{equation} observe that: \begin{align*} H_{\mathbf{J}^{\wedge k}}\left(\mathbf{m}\right) & \overset{\mathbb{Q}^{d}}{=}H_{\mathbf{J}}^{\circ k}\left(\mathbf{m}\right)\\ & =\left(M_{H}\left(\mathbf{J}\right)\right)^{k}\mathbf{m}+\left(\sum_{\ell=0}^{k-1}\left(M_{H}\left(\mathbf{J}\right)\right)^{\ell}\right)\chi_{H}\left(\mathbf{J}\right)\\ & =\left(M_{H}\left(\mathbf{n}\right)\right)^{k}\mathbf{m}+\left(\sum_{\ell=0}^{k-1}\left(M_{H}\left(\mathbf{n}\right)\right)^{\ell}\right)\chi_{H}\left(\mathbf{n}\right) \end{align*} Since $\mathbf{n}\neq\mathbf{0}$, there is an $m\in\left\{ 1,\ldots,r\right\} $ so that the $m$th entry of $\mathbf{n}$ has at least one non-zero $p$-adic digit. Because $H$ is monogenic, we then have that at least \emph{one} of the matrices in the product $M_{H}\left(\mathbf{n}\right)$ has $q_{H}$-adic norm $<1$. Because all the matrices in that product have $q$-adic norm $\leq1$, this shows that $\left\Vert M_{H}\left(\mathbf{n}\right)\right\Vert _{q_{H}}<1$. Consequently, the series: \begin{equation} \sum_{\ell=0}^{\infty}\left(M_{H}\left(\mathbf{n}\right)\right)^{\ell} \end{equation} converges in $\mathbb{Q}_{q_{H}}^{d,d}$, thereby defining the inverse of $\mathbf{I}_{d}-M_{H}\left(\mathbf{n}\right)$. In fact, because $\mathbf{I}_{d}-M_{H}\left(\mathbf{n}\right)$ has entries in $\mathbb{Q}$, note that that $\left(\mathbf{I}_{d}-M_{H}\left(\mathbf{n}\right)\right)^{-1}\in\mathbb{Q}_{q_{H}}^{d,d}$. Moreover, we can use the geometric series formula to write: \begin{equation} \left(\mathbf{I}_{d}-M_{H}\left(\mathbf{n}\right)\right)\sum_{\ell=0}^{k-1}\left(M_{H}\left(\mathbf{n}\right)\right)^{\ell}=\sum_{\ell=0}^{k-1}\left(M_{H}\left(\mathbf{n}\right)\right)^{\ell}-\sum_{\ell=0}^{k-1}\left(M_{H}\left(\mathbf{n}\right)\right)^{\ell+1}=\mathbf{I}_{d}-\left(M_{H}\left(\mathbf{n}\right)\right)^{k} \end{equation} So: \begin{equation} \sum_{\ell=0}^{k-1}\left(M_{H}\left(\mathbf{n}\right)\right)^{\ell}=\frac{\mathbf{I}_{d}-\left(M_{H}\left(\mathbf{n}\right)\right)^{k}}{\mathbf{I}_{d}-M_{H}\left(\mathbf{n}\right)}\overset{\textrm{def}}{=}\left(\mathbf{I}_{d}-\left(M_{H}\left(\mathbf{n}\right)\right)^{k}\right)^{-1}\left(\mathbf{I}_{d}-\left(M_{H}\left(\mathbf{n}\right)\right)^{k}\right) \end{equation} Consequently: \begin{equation} H_{\mathbf{J}^{\wedge k}}\left(\mathbf{m}\right)\overset{\mathbb{Q}^{d}}{=}\left(M_{H}\left(\mathbf{n}\right)\right)^{k}\mathbf{m}+\frac{\mathbf{I}_{d}-\left(M_{H}\left(\mathbf{n}\right)\right)^{k}}{\mathbf{I}_{d}-M_{H}\left(\mathbf{n}\right)}\chi_{H}\left(\mathbf{n}\right) \end{equation} and so: \begin{equation} M_{H}\left(\mathbf{J}^{\wedge k}\right)\mathbf{m}+\chi_{H}\left(\mathbf{J}^{\wedge k}\right)\overset{\mathbb{Q}^{d}}{=}\left(M_{H}\left(\mathbf{n}\right)\right)^{k}\mathbf{m}+\frac{\mathbf{I}_{d}-\left(M_{H}\left(\mathbf{n}\right)\right)^{k}}{\mathbf{I}_{d}-M_{H}\left(\mathbf{n}\right)}\chi_{H}\left(\mathbf{n}\right) \end{equation} Now, using $M_{H}$'s concatenation identity (\textbf{Proposition \ref{prop:MD M_H functional equations}}): \begin{equation} M_{H}\left(\mathbf{J}^{\wedge k}\right)\mathbf{m}=\left(M_{H}\left(\mathbf{J}\right)\right)^{k}\mathbf{m}=\left(M_{H}\left(\mathbf{n}\right)\right)^{k}\mathbf{m} \end{equation} we have: \begin{equation} \left(M_{H}\left(\mathbf{n}\right)\right)^{k}\mathbf{m}+\chi_{H}\left(\mathbf{J}^{\wedge k}\right)\overset{\mathbb{Q}^{d}}{=}\left(M_{H}\left(\mathbf{n}\right)\right)^{k}\mathbf{m}+\frac{\mathbf{I}_{d}-\left(M_{H}\left(\mathbf{n}\right)\right)^{k}}{\mathbf{I}_{d}-M_{H}\left(\mathbf{n}\right)}\chi_{H}\left(\mathbf{n}\right) \end{equation} and so: \begin{equation} \chi_{H}\left(\mathbf{J}^{\wedge k}\right)=\frac{\mathbf{I}_{d}-\left(M_{H}\left(\mathbf{n}\right)\right)^{k}}{\mathbf{I}_{d}-M_{H}\left(\mathbf{n}\right)}\chi_{H}\left(\mathbf{n}\right)\label{eq:MD Chi_H B functional equation, ready to take limits} \end{equation} Using \textbf{Proposition \ref{prop:MD self-concatenation identity}} gives us: \begin{align*} \chi_{H}\left(\mathbf{J}^{\wedge k}\right) & =\chi_{H}\left(\mathbf{n}\frac{1-p^{k\lambda_{p}\left(\mathbf{n}\right)}}{1-p^{\lambda_{p}\left(\mathbf{n}\right)}}\right)\\ & =\chi_{H}\left(n_{1}\frac{1-p^{k\lambda_{p}\left(n_{1}\right)}}{1-p^{\lambda_{p}\left(n_{1}\right)}},\ldots,n_{r}\frac{1-p^{k\lambda_{p}\left(n_{r}\right)}}{1-p^{\lambda_{p}\left(n_{r}\right)}}\right) \end{align*} $\chi_{H}$'s rising-continuity property (\ref{eq:MD Rising Continuity Formula for Chi_H}) allows us to take the limit as $k\rightarrow\infty$: \begin{equation} \lim_{k\rightarrow\infty}\chi_{H}\left(\mathbf{J}^{\wedge k}\right)\overset{\mathbb{Z}_{q_{H}}^{d}}{=}\chi_{H}\left(\frac{n_{1}}{1-p^{\lambda_{p}\left(n_{1}\right)}},\ldots,\frac{n_{r}}{1-p^{\lambda_{p}\left(n_{r}\right)}}\right)=\chi_{H}\left(\mathbf{B}_{p}\left(\mathbf{n}\right)\right) \end{equation} On the other hand, (\ref{eq:MD Chi_H B functional equation, ready to take limits}) yields; \begin{align*} \lim_{k\rightarrow\infty}\chi_{H}\left(\mathbf{J}^{\wedge k}\right) & \overset{\mathbb{Z}_{q_{H}}^{d}}{=}\lim_{k\rightarrow\infty}\frac{\mathbf{I}_{d}-\left(M_{H}\left(\mathbf{n}\right)\right)^{k}}{\mathbf{I}_{d}-M_{H}\left(\mathbf{n}\right)}\chi_{H}\left(\mathbf{n}\right)\\ \left(\left\Vert M_{H}\left(\mathbf{n}\right)\right\Vert _{q}<1\right); & \overset{\mathbb{Z}_{q_{H}}^{d}}{=}\frac{\mathbf{I}_{d}-\mathbf{O}_{d}}{\mathbf{I}_{d}-M_{H}\left(\mathbf{n}\right)}\chi_{H}\left(\mathbf{n}\right)\\ & =\frac{\chi_{H}\left(\mathbf{n}\right)}{\mathbf{I}_{d}-M_{H}\left(\mathbf{n}\right)}\\ & =\left(\mathbf{I}_{d}-M_{H}\left(\mathbf{n}\right)\right)^{-1}\chi_{H}\left(\mathbf{n}\right) \end{align*} Putting the two together gives: \begin{equation} \chi_{H}\left(\mathbf{B}_{p}\left(\mathbf{n}\right)\right)\overset{\mathbb{Z}_{q_{H}}^{d}}{=}\left(\mathbf{I}_{d}-M_{H}\left(\mathbf{n}\right)\right)^{-1}\chi_{H}\left(\mathbf{n}\right) \end{equation} Lastly, because the right-hand side is the product of $\left(\mathbf{I}_{d}-M_{H}\left(\mathbf{n}\right)\right)^{-1}$ (a $d\times d$ matrix with entries in $\mathbb{Q}$) and $\chi_{H}\left(\mathbf{n}\right)$ (a $d\times1$ column vector with entries in $\mathbb{Q}$), we see that both sides of (\ref{eq:MD Chi_H B functional equation}) are $d\times1$ column vectors with entries in $\mathbb{Q}$. Q.E.D. \vphantom{}Next up: the multi-dimensional notion of a ``wrong value''. But first, let us establish the $p$-adic extendibility of $H$. \begin{prop} \label{prop:p-adic extension of H-1}Let $H:\mathbb{Z}^{d}\rightarrow\mathbb{Z}^{d}$ be any $p$-Hydra map of depth $r$, not necessarily integral. Then, $H$ admits an extension to a continuous map $\mathbb{Z}_{p}^{d}\rightarrow\mathbb{Z}_{p}^{d}$ defined by: \begin{equation} H\left(\mathbf{z}\right)\overset{\textrm{def}}{=}\sum_{\mathbf{j}\in\mathbb{Z}^{r}/p\mathbb{Z}^{r}}\left[\mathbf{z}\overset{p}{\equiv}\mathbf{j}\right]\mathbf{D}_{\mathbf{j}}^{-1}\left(\mathbf{A}_{\mathbf{j}}\mathbf{z}+\mathbf{b}_{\mathbf{j}}\right),\textrm{ }\forall\mathbf{z}\in\mathbb{Z}_{p}^{r}\label{eq:MD p-adic extension of H} \end{equation} Moreover, the function $\mathbf{f}:\mathbb{Z}_{p}^{d}\rightarrow\mathbb{Z}_{p}^{d}$ defined by: \begin{equation} \mathbf{f}\left(\mathbf{z}\right)=\sum_{\mathbf{j}\in\mathbb{Z}^{r}/p\mathbb{Z}^{r}}\left[\mathbf{z}\overset{p}{\equiv}\mathbf{j}\right]\mathbf{D}_{\mathbf{j}}^{-1}\left(\mathbf{A}_{\mathbf{j}}\mathbf{z}+\mathbf{b}_{\mathbf{j}}\right),\textrm{ }\forall\mathbf{z}\in\mathbb{Z}_{p}^{r} \end{equation} is the unique continuous function $\mathbb{Z}_{p}^{d}\rightarrow\mathbb{Z}_{p}^{d}$ whose restriction to $\mathbb{Z}^{d}$ is equal to $H$. \end{prop} Proof: Immediate, just like the one-dimensional case. Q.E.D. \begin{defn}[\textbf{Wrong Values and Propriety}] \label{def:MD Wrong values and propriety}\index{Hydra map!wrong value}\index{Hydra map!proper}Let $H$ be any $p$-Hydra map, not necessarily integral. \vphantom{} I. We say $\mathbf{y}=\left(\mathfrak{y}_{1},\ldots,\mathfrak{y}_{d}\right)\in\mathbb{Q}_{p}^{d}$ is a \textbf{wrong value for $H$ }whenever there are $\mathbf{z}\in\mathbb{Z}_{p}^{d}$ and $\mathbf{J}\in\textrm{String}^{r}\left(p\right)$ so that $\mathbf{y}=H_{\mathbf{J}}\left(\mathbf{z}\right)$ and $H_{\mathbf{J}}\left(\mathbf{z}\right)\neq H^{\circ\left|\mathbf{J}\right|}\left(\mathbf{z}\right)$. We call $\mathbf{z}$ a \textbf{seed }of $\mathbf{y}$. \vphantom{} II.\index{Hydra map!proper} We say $H$ is \textbf{proper }whenever every wrong value of $H$ is an element of $\mathbb{Q}_{p}^{d}\backslash\mathbb{Z}_{p}^{d}$; that is, every wrong value of $H$ is a $d$-tuple of $p$-adic rational numbers at least one of which is \emph{not} a $p$-adic integer. \end{defn} \vphantom{} We now repeat the lead-up to the Correspondence Principle from the one-dimensional case. \begin{prop} \label{prop:MD Q_p / Z_p prop}Let $H$ be any $p$-Hydra map of dimension $d$ and depth $r$. Then, $H_{\mathbf{j}}\left(\mathbb{Q}_{p}^{d}\backslash\mathbb{Z}_{p}^{d}\right)\subseteq\mathbb{Q}_{p}^{d}\backslash\mathbb{Z}_{p}^{d}$ for all $\mathbf{j}\in\mathbb{Z}^{r}/p\mathbb{Z}^{r}$. \end{prop} \begin{rem} Here, we are viewing the $H_{\mathbf{j}}$s as functions $\mathbb{Q}_{p}^{d}\rightarrow\mathbb{Q}_{p}^{d}$. \end{rem} Proof: Let $\mathbf{y}\in\mathbb{Q}_{p}^{d}\backslash\mathbb{Z}_{p}^{d}$ and $\mathbf{j}\in\mathbb{Z}^{r}/p\mathbb{Z}^{r}$ be arbitrary. Note that as a $p$-Hydra map, $p$ divides every element of $\mathbf{D}_{\mathbf{j}}$ which is not $1$. Moreover, as a multi-dimensional $p$-Hydra map, every non-zero element of $\mathbf{A}_{\mathbf{j}}$ is co-prime to every non-zero element of $\mathbf{D}_{\mathbf{j}}$. Consequently, $p$ does not divide any non-zero element of $\mathbf{A}_{\mathbf{j}}$, and so, $\left\Vert \mathbf{A}_{\mathbf{j}}\right\Vert _{p}=1$. Thus: \begin{equation} \left\Vert \mathbf{A}_{\mathbf{j}}\mathbf{y}\right\Vert _{p}=\left\Vert \mathbf{y}\right\Vert _{p}>1 \end{equation} So, $\mathbf{A}_{\mathbf{j}}\mathbf{y}\in\mathbb{Q}_{p}^{d}\backslash\mathbb{Z}_{p}^{d}$. Since every entry of the $d$-tuple $\mathbf{b}_{\mathbf{j}}$ is a rational integer, the $p$-adic ultrametric inequality guarantees that $\left\Vert \mathbf{A}_{\mathbf{j}}\mathbf{y}+\mathbf{b}_{\mathbf{j}}\right\Vert >1$, and thus, $\mathbf{A}_{\mathbf{j}}\mathbf{y}+\mathbf{b}_{\mathbf{j}}$ is in $\mathbb{Q}_{p}^{d}\backslash\mathbb{Z}_{p}^{d}$. Finally, $p$ dividing the diagonal elements of $\mathbf{D}_{\mathbf{j}}$ which are \emph{not }$1$ implies $\left\Vert D_{\mathbf{j}}\right\Vert _{p}<1$, and so: \begin{equation} \left\Vert H_{\mathbf{j}}\left(\mathbf{y}\right)\right\Vert _{p}=\left\Vert \frac{\mathbf{A}_{\mathbf{j}}\mathbf{y}+\mathbf{b}_{\mathbf{j}}}{\mathbf{D}_{\mathbf{j}}}\right\Vert _{p}\geq\left\Vert \mathbf{A}_{\mathbf{j}}\mathbf{y}+\mathbf{b}_{\mathbf{j}}\right\Vert _{p}>1 \end{equation} which shows that $H_{\mathbf{j}}\left(\mathbf{y}\right)\in\mathbb{Q}_{p}^{d}\backslash\mathbb{Z}_{p}^{d}$. Q.E.D. \begin{lem} \label{lem:MD integrality lemma}Let $H$ be a semi-basic $p$-Hydra map of dimension $d$ and depth $r$, where $p$ is prime; we do not require $H$ to be integral. Then, $H$ is proper if and only if $H$ is integral. \end{lem} Proof: Let $H$ be semi-basic. I. (Proper implies integral) Suppose $H$ is proper, and let $\mathbf{n}\in\mathbb{Z}^{d}$ be arbitrary. Then, clearly, when $\mathbf{j}\in\mathbb{Z}^{r}/p\mathbb{Z}^{r}$ satisfies $\mathbf{j}=\left[\mathbf{n}\right]_{p}$ (recall, this means the first $r$ entries of $\mathbf{n}$ are congruent mod $p$ to the first $r$ entries of $\mathbf{j}$), we have that $H_{\mathbf{j}}\left(\mathbf{n}\right)\in\mathbb{Z}^{d}$. So, suppose instead that $\mathbf{j}\neq\left[\mathbf{n}\right]_{p}$. Since $H$ is proper, the fact that $\mathbf{n}\in\mathbb{Z}^{d}\subseteq\mathbb{Z}_{p}^{d}$ tells us that $\left\Vert H_{\mathbf{j}}\left(\mathbf{n}\right)\right\Vert _{p}>1$. So, at least one entry of $H_{\mathbf{j}}\left(\mathbf{n}\right)$ is not a $p$-adic integer, and thus, is not a rational integer, either. This proves $H$ is integral. \vphantom{} II. (Integral implies proper) Suppose $H$ is integral, and\textemdash by way of contradiction\textemdash suppose $H$ is \emph{not}\textbf{ }proper. Then, there is a $\mathbf{z}\in\mathbb{Z}_{p}^{d}$ and a $\mathbf{j}\in\mathbb{Z}^{r}/p\mathbb{Z}^{r}$ with $\mathbf{j}\neq\left[\mathbf{z}\right]_{p}$ (again, only the first $r$ entries are affected) so that $\left\Vert H_{\mathbf{j}}\left(\mathbf{z}\right)\right\Vert _{p}\leq1$. Now, writing \[ \mathbf{z}=\sum_{m=1}^{d}\sum_{n=0}^{\infty}c_{m,n}p^{n}\mathbf{e}_{m} \] where $\mathbf{e}_{m}$ is the $m$th standard basis vector in $\mathbb{Z}_{p}^{d}$: \begin{align*} H_{\mathbf{j}}\left(\mathbf{z}\right) & =\frac{\mathbf{A}_{\mathbf{j}}\left(\sum_{m=1}^{d}\sum_{n=0}^{\infty}c_{m,n}p^{n}\mathbf{e}_{m}\right)+\mathbf{b}_{\mathbf{j}}}{\mathbf{D}_{\mathbf{j}}}\\ & =\frac{\mathbf{A}_{\mathbf{j}}\left(\sum_{m=1}^{d}c_{m,0}\mathbf{e}_{m}\right)+\mathbf{b}_{\mathbf{j}}}{\mathbf{D}_{\mathbf{j}}}+\sum_{m=1}^{d}\sum_{n=1}^{\infty}c_{m,n}p^{n-1}\left(p\mathbf{D}_{\mathbf{j}}^{-1}\right)\mathbf{e}_{m}\\ \left(\sum_{m=1}^{d}c_{m,0}\mathbf{e}_{m}=\left[\mathbf{z}\right]_{p}\right); & =H_{\mathbf{j}}\left(\left[\mathbf{z}\right]_{p}\right)+\sum_{m=1}^{d}\sum_{n=1}^{\infty}c_{m,n}p^{n-1}\left(p\mathbf{D}_{\mathbf{j}}^{-1}\right)\mathbf{e}_{m} \end{align*} Because $H$ is a $p$-Hydra map, every diagonal entry of $\mathbf{D}_{\mathbf{j}}$ which is not $1$ must be a divisor of $p$$d_{j}$ must be a divisor of $p$. The primality of $p$ then forces $d_{j}$ to be either $1$ or $p$. In either case, we have that $p\mathfrak{y}/d_{j}$ is an element of $\mathbb{Z}_{p}$. Our contradictory assumption $\left|H_{j}\left(\mathfrak{z}\right)\right|_{p}\leq1$ tells us that $H_{j}\left(\mathfrak{z}\right)$ is also in $\mathbb{Z}_{p}$, and so: \begin{equation} H_{j}\left(\left[\mathfrak{z}\right]_{p}\right)=H_{j}\left(\mathfrak{z}\right)-\frac{p\mathfrak{y}}{d_{j}}\in\mathbb{Z}_{p} \end{equation} Since $H$ was given to be integral, $j\neq\left[\mathfrak{z}\right]_{p}$ implies $H_{j}\left(\left[\mathfrak{z}\right]_{p}\right)\notin\mathbb{Z}$. As such, $H_{j}\left(\left[\mathfrak{z}\right]_{p}\right)$ is a $p$-adic integer which is \emph{not }a rational integer. Since $H_{j}\left(\left[\mathfrak{z}\right]_{p}\right)$ is a non-integer rational number which is a $p$-adic integer, the denominator of $H_{j}\left(\left[\mathfrak{z}\right]_{p}\right)=\frac{a_{j}\left[\mathfrak{z}\right]_{p}+b_{j}}{d_{j}}$ must be divisible by some prime $q\neq p$, and hence, $q\mid d_{j}$. However, we saw that $p$ being prime forced $d_{j}\in\left\{ 1,p\right\} $\textemdash this is impossible! Thus, it must be that $H$ is proper. Q.E.D. \vphantom{} Next, we use \textbf{Proposition \ref{prop:MD Q_p / Z_p prop} }to establish a result about the wrong values of $H$ when $H$ is proper. \begin{lem} \label{lem:MD wrong values lemma}Let $H$ be any proper $p$-Hydra map, not necessarily integral or semi-basic. Then, all wrong values of $H$ are elements of $\mathbb{Q}_{p}^{d}\backslash\mathbb{Z}_{p}^{d}$. \end{lem} Proof: Let $H$ be as given, let $\mathbf{z}\in\mathbb{Z}_{p}^{d}$, and let $\mathbf{i}\in\mathbb{Z}^{r}/p\mathbb{Z}^{r}$ be such that $\left[\mathbf{z}\right]_{p}\neq\mathbf{i}$. Then, by definition of properness, the quantity: \begin{equation} H_{\mathbf{i}}\left(\mathbf{z}\right)=\frac{\mathbf{A}_{\mathbf{i}}\mathbf{z}+\mathbf{b}_{\mathbf{i}}}{\mathbf{D}_{\mathbf{i}}} \end{equation} has $p$-adic norm $>1$. By \textbf{Proposition \ref{prop:MD Q_p / Z_p prop}}, this then forces $H_{\mathbf{J}}\left(H_{\mathbf{i}}\left(\mathbf{z}\right)\right)$ to be an element of $\mathbb{Q}_{p}^{d}\backslash\mathbb{Z}_{p}^{d}$ for all $\mathbf{J}\in\textrm{String}^{r}\left(p\right)$. Since every wrong value with seed $\mathbf{z}$ is of the form $H_{\mathbf{J}}\left(H_{\mathbf{i}}\left(\mathbf{z}\right)\right)$ for some $\mathbf{J}\in\textrm{String}^{r}\left(p\right)$, some $\mathbf{z}\in\mathbb{Z}_{p}^{d}$, and some $\mathbf{i}\in\mathbb{Z}^{r}/p\mathbb{Z}^{r}$ for which $\left[\mathbf{z}\right]_{p}\neq\mathbf{i}$, this shows that every wrong value of $H$ is in $\mathbb{Q}_{p}^{d}\backslash\mathbb{Z}_{p}^{d}$. So, $H$ is proper. Q.E.D. \begin{lem} \label{lem:MD properness lemma}Let $H$ be any proper $p$-Hydra map, not necessarily semi-basic or integral. Let $\mathbf{z}\in\mathbb{Z}_{p}^{d}$, and let $\mathbf{J}\in\mathrm{String}^{r}\left(p\right)$. If $H_{\mathbf{J}}\left(\mathbf{z}\right)=\mathbf{z}$, then $\ensuremath{H^{\circ\left|\mathbf{J}\right|}\left(\mathbf{z}\right)=\mathbf{z}}$. \end{lem} Proof: Let $H$, $\mathbf{z}$, and $\mathbf{J}$ be as given. By way of contradiction, suppose $H^{\circ\left|\mathbf{J}\right|}\left(\mathbf{z}\right)\neq\mathbf{z}$. But then $\mathbf{z}=H_{\mathbf{J}}\left(\mathbf{z}\right)$ implies $H^{\circ\left|\mathbf{J}\right|}\left(\mathbf{z}\right)\neq H_{\mathbf{J}}\left(\mathbf{z}\right)$. Hence, $H_{\mathbf{J}}\left(\mathbf{z}\right)$ is a wrong value of $H$ with seed $\mathbf{z}$. \textbf{Lemma} \ref{lem:MD wrong values lemma} then forces $H_{\mathbf{J}}\left(\mathbf{z}\right)\in\mathbb{Q}_{p}^{d}\backslash\mathbb{Z}_{p}^{d}$. However, $H_{\mathbf{J}}\left(\mathbf{z}\right)=\mathbf{z}$, and $\mathbf{z}$ was given to be in $\mathbb{Z}_{p}^{d}$. This is impossible! Consequently, $H_{\mathbf{J}}\left(\mathbf{z}\right)=\mathbf{z}$ implies $H^{\circ\left|\mathbf{J}\right|}\left(\mathbf{z}\right)=\mathbf{z}$. Q.E.D. \vphantom{} Now comes the multi-dimensional Correspondence Principle. Like in the one-dimensional case, we start by proving a correspondence between cycles of $H$ and rational-integer values attained by $\chi_{H}$. The task of refining this correspondence to one involving the individual periodic points of $H$ will be dealt with in a Corollary. \begin{thm}[\textbf{Multi-Dimensional Correspondence Principle, Ver. 1}] \label{thm:MD CP v1}Let $H$ be a semi-basic $p$-Hydra map of dimension $d$ and depth $r$ that fixes $\mathbf{0}$. \index{Correspondence Principle} Then: \vphantom{} I. Let $\Omega$ be any cycle of $H$ in $\mathbb{Z}^{d}$, with $\left|\Omega\right|\geq2$. Then, there exist $\mathbf{x}\in\Omega$ and an $\mathbf{n}\in\mathbb{N}_{0}^{r}\backslash\left\{ \mathbf{0}\right\} $ so that: \begin{equation} \chi_{H}\left(\mathbf{B}_{p}\left(\mathbf{n}\right)\right)\overset{\mathbb{Z}_{q_{H}}^{d}}{=}\mathbf{x} \end{equation} \vphantom{} II. Suppose also that $H$ is integral, and let $\mathbf{n}\in\mathbb{N}_{0}^{r}$. If: \begin{equation} \chi_{H}\left(\mathbf{B}_{p}\left(\mathbf{n}\right)\right)\in\mathbb{Z}_{q_{H}}^{d}\cap\mathbb{Z}^{d} \end{equation} then $\chi_{H}\left(\mathbf{B}_{p}\left(\mathbf{n}\right)\right)$ is a periodic point of $H$ in $\mathbb{Z}^{d}$. \end{thm} Proof: I. Let $\Omega$ be a cycle of $H$ in $\mathbb{Z}^{d}$ with $\left|\Omega\right|\geq2$. Observe that for any $\mathbf{x}\in\Omega$ and any $\mathbf{J}\in\textrm{String}^{r}\left(p\right)$ for which $\left|\mathbf{J}\right|=\left|\Omega\right|\geq2$ and $H_{\mathbf{J}}\left(\mathbf{x}\right)=\mathbf{x}$, it must be that $\mathbf{J}$ contains at least one non-zero $r$-tuple; that is, in writing $\mathbf{J}=\left(\mathbf{j}_{1},\ldots,\mathbf{j}_{\left|\mathbf{J}\right|}\right)$, there is at least one $\mathbf{j}_{k}$ which is not $\mathbf{0}$. As in the one-dimensional case, we can assume without loss of generality that we have chosen $\mathbf{x}\in\Omega$ for which the known non-zero $r$-tuple of $\mathbf{J}$ occurs in the bottom-most row of $\mathbf{J}$ (that is, $\mathbf{j}_{\left|\mathbf{J}\right|}\neq\mathbf{0}$), viewing $\mathbf{J}$ as a matrix. Writing the affine map $H_{\mathbf{J}}$ in affine form gives: \begin{equation} \mathbf{x}=H_{\mathbf{J}}\left(\mathbf{x}\right)=H_{\mathbf{J}}^{\prime}\left(\mathbf{0}\right)\mathbf{x}+H_{\mathbf{J}}\left(\mathbf{0}\right)=M_{H}\left(\mathbf{J}\right)\mathbf{x}+\chi_{H}\left(\mathbf{J}\right) \end{equation} where all the equalities are in $\mathbb{Q}^{d}$. Then, noting the implication: \begin{equation} H_{\mathbf{J}}\left(\mathbf{x}\right)=\mathbf{x}\Rightarrow H_{\mathbf{J}^{\wedge m}}\left(\mathbf{x}\right)=\mathbf{x} \end{equation} we have: \begin{equation} \mathbf{x}=H_{\mathbf{J}^{\wedge m}}\left(\mathbf{x}\right)=M_{H}\left(\mathbf{J}^{\wedge m}\right)\mathbf{x}+\chi_{H}\left(\mathbf{J}^{\wedge m}\right)=\left(M_{H}\left(\mathbf{J}\right)\right)^{m}\mathbf{x}+\chi_{H}\left(\mathbf{J}^{\wedge m}\right) \end{equation} Since $\mathbf{J}$ contains a tuple which is \emph{not} the zero tuple, the decay estimate from \textbf{Proposition \ref{prop:MD M_H decay estimate}} tells us that that $\left\Vert \left(M_{H}\left(\mathbf{J}\right)\right)^{m}\right\Vert _{q_{H}}$ tends to $0$ in $\mathbb{R}$ as $m\rightarrow\infty$. Moreover, the $\mathbf{n}\in\mathbb{N}_{0}^{r}$ represented by $\mathbf{J}$ is necessarily non-$\mathbf{0}$. So, we can apply \textbf{Proposition \ref{prop:MD self-concatenation identity}}: \begin{equation} \mathbf{J}^{\wedge m}\sim\frac{1-p^{m\lambda_{p}\left(\mathbf{n}\right)}}{1-p^{\lambda_{p}\left(\mathbf{n}\right)}}\mathbf{n} \end{equation} and, like in the proof of \textbf{Lemma \ref{lem:MD Chi_H o B functional equation}}, we can write: \begin{align*} \lim_{m\rightarrow\infty}\chi_{H}\left(\mathbf{J}^{\wedge m}\right) & \overset{\mathbb{Z}_{q_{H}}^{d}}{=}\lim_{m\rightarrow\infty}\chi_{H}\left(\frac{1-p^{m\lambda_{p}\left(\mathbf{n}\right)}}{1-p^{\lambda_{p}\left(\mathbf{n}\right)}}\mathbf{n}\right)\\ & \overset{\mathbb{Z}_{q_{H}}^{d}}{=}\chi_{H}\left(\frac{\mathbf{n}}{1-p^{\lambda_{p}\left(\mathbf{n}\right)}}\right)\\ & =\chi_{H}\left(\mathbf{B}_{p}\left(\mathbf{n}\right)\right) \end{align*} Finally, letting $m\rightarrow\infty$ in the equation: \begin{equation} \mathbf{x}=\left(M_{H}\left(\mathbf{J}\right)\right)^{m}\mathbf{x}+\chi_{H}\left(\mathbf{J}^{\wedge m}\right)=\left(M_{H}\left(\mathbf{J}\right)\right)^{m}\mathbf{x}+\chi_{H}\left(\frac{1-p^{m\lambda_{p}\left(\mathbf{n}\right)}}{1-p^{\lambda_{p}\left(\mathbf{n}\right)}}\mathbf{n}\right) \end{equation} we obtain the equality: \begin{equation} \mathbf{x}\overset{\mathbb{Z}_{q_{H}}^{d}}{=}\chi_{H}\left(\frac{\mathbf{n}}{1-p^{\lambda_{p}\left(\mathbf{n}\right)}}\right)=\chi_{H}\left(\mathbf{B}_{p}\left(\mathbf{n}\right)\right) \end{equation} This proves the existence of the desired $\mathbf{x}$ and $\mathbf{n}$. \vphantom{} II. Let $H$ be integral; then, by \textbf{Lemma \ref{lem:MD integrality lemma}}, $H$ is proper. Next, let $\mathbf{n}\in\mathbb{N}_{0}^{r}\backslash\left\{ \mathbf{0}\right\} $ be such that $\chi_{H}\left(\mathbf{B}_{p}\left(\mathbf{n}\right)\right)$ is in $\mathbb{Z}_{q_{H}}^{d}\cap\mathbb{Z}^{d}$, and let $\mathbf{J}\in\textrm{String}^{r}\left(p\right)$ be the shortest block string representing $\mathbf{n}$. Then, by $\chi_{H}$'s concatenation identity (equation \ref{eq:MD Chi_H concatenation identity} from \textbf{Lemma \ref{lem:MD Chi_H functional equations and characterization}}), we can write: \begin{equation} \chi_{H}\left(\frac{1-p^{k\lambda_{p}\left(\mathbf{n}\right)}}{1-p^{\lambda_{p}\left(\mathbf{n}\right)}}\mathbf{n}\right)=\chi_{H}\left(\mathbf{J}^{\wedge k}\right)=H\left(\chi_{H}\left(\mathbf{J}^{\wedge\left(k-1\right)}\right)\right)=\cdots=H^{\circ\left(k-1\right)}\left(\chi_{H}\left(\mathbf{J}\right)\right)\label{eq:MD self-concatenation identity for Chi_H} \end{equation} Letting $k\rightarrow\infty$, \textbf{Lemma \ref{lem:MD Chi_H functional equations and characterization}} along with the convergence of $\frac{1-p^{k\lambda_{p}\left(\mathbf{n}\right)}}{1-p^{\lambda_{p}\left(\mathbf{n}\right)}}\mathbf{n}$ to $\frac{\mathbf{n}}{1-p^{\lambda_{p}\left(\mathbf{n}\right)}}$ in $\mathbb{Z}_{p}^{r}$ then yields the $q$-adic equality: \begin{equation} \lim_{k\rightarrow\infty}H_{\mathbf{J}}^{\circ\left(k-1\right)}\left(\chi_{H}\left(\mathbf{n}\right)\right)\overset{\mathbb{Z}_{q_{H}}^{d}}{=}\chi_{H}\left(\frac{\mathbf{n}}{1-p^{\lambda_{p}\left(\mathbf{n}\right)}}\right)=\chi_{H}\left(\mathbf{B}_{p}\left(\mathbf{n}\right)\right)\label{eq:Iterating H_bold-J on Chi_H} \end{equation} Because $H$ is semi-basic, the non-zero entries of any $\mathbf{A}_{\mathbf{j}}$ are co-prime to the non-zero entries of any $\mathbf{D}_{\mathbf{k}}$. Moreover, since $\left\Vert \mathbf{A}_{\mathbf{j}}\right\Vert _{q_{H}}\leq1/q_{H}$ for all $\mathbf{j}\in\left(\mathbb{Z}^{r}/p\mathbb{Z}^{r}\right)\backslash\left\{ \mathbf{0}\right\} $, this guarantees that for any finite length block string such as our $\mathbf{J}$, the matrix $H_{\mathbf{J}}^{\prime}\left(\mathbf{0}\right)$ and the $d$-tuple $H_{\mathbf{J}}\left(\mathbf{0}\right)$ have entries in $\mathbb{Q}$ whose $q_{H}$-adic absolute values are $\leq1$; that is, their entries are in $\mathbb{Q}\cap\mathbb{Z}_{q_{H}}$. Consequently, the affine linear map: \begin{equation} \mathbf{w}\in\mathbb{Z}_{q_{H}}^{d}\mapsto H_{\mathbf{J}}\left(\mathbf{w}\right)=H_{\mathbf{J}}^{\prime}\left(\mathbf{0}\right)\mathbf{w}+H_{\mathbf{J}}\left(\mathbf{0}\right)\in\mathbb{Z}_{q_{H}}^{d} \end{equation} defines a continuous self-map of $\mathbb{Z}_{q_{H}}^{d}$. With this, we can then write: \begin{align*} H_{\mathbf{J}}\left(\chi_{H}\left(\mathbf{B}_{p}\left(\mathbf{n}\right)\right)\right) & \overset{\mathbb{Z}_{q_{H}}^{d}}{=}H_{\mathbf{j}}\left(\lim_{k\rightarrow\infty}H_{\mathbf{J}}^{\circ\left(k-1\right)}\left(\chi_{H}\left(\mathbf{n}\right)\right)\right)\\ & \overset{\mathbb{Z}_{q_{H}}^{d}}{=}\lim_{k\rightarrow\infty}H_{\mathbf{J}}^{\circ k}\left(\chi_{H}\left(\mathbf{n}\right)\right)\\ & \overset{\mathbb{Z}_{q_{H}}^{d}}{=}\lim_{k\rightarrow\infty}H_{\mathbf{J}}^{\circ\left(k-1\right)}\left(\chi_{H}\left(\mathbf{n}\right)\right)\\ & \overset{\mathbb{Z}_{q_{H}}^{d}}{=}\chi_{H}\left(\mathbf{B}_{p}\left(\mathbf{n}\right)\right) \end{align*} This proves that the rational $d$-tuple $\chi_{H}\left(\mathbf{B}_{p}\left(\mathbf{n}\right)\right)$ is a fixed point of the composition sequence $H_{\mathbf{J}}$ as an element of $\mathbb{Z}_{q_{H}}^{d}$. Now, since we assumed that $\chi_{H}\left(\mathbf{B}_{p}\left(\mathbf{n}\right)\right)$ was in $\mathbb{Z}^{d}$, the fact that $H_{\mathbf{J}}$ is a map on $\mathbb{Q}^{d}$, tells us that the equality $H_{\mathbf{J}}\left(\chi_{H}\left(\mathbf{B}_{p}\left(\mathbf{n}\right)\right)\right)=\chi_{H}\left(\mathbf{B}_{p}\left(\mathbf{n}\right)\right)$\textemdash nominally occurring in $\mathbb{Z}_{q_{H}}^{d}$\textemdash is actually valid in $\mathbb{Q}^{d}$. Consequently, this equality is also valid in $\mathbb{Q}_{p}^{d}$, and so, $\chi_{H}\left(\mathbf{B}_{p}\left(\mathbf{n}\right)\right)$ is a fixed point of the map $H_{\mathbf{J}}:\mathbb{Q}_{p}^{d}\rightarrow\mathbb{Q}_{p}^{d}$. Next, because the assumed integrality of $H$ makes $H$ proper, we can apply \textbf{Lemma \ref{lem:MD properness lemma}} to conclude that $\chi_{H}\left(\mathbf{B}_{p}\left(\mathbf{n}\right)\right)\in\mathbb{Z}^{d}\subseteq\mathbb{Z}_{p}^{d}$, as a fixed point of $H_{\mathbf{J}}$, is in fact a fixed point of $H^{\circ\left|\mathbf{J}\right|}$: \begin{equation} H^{\circ\left|\mathbf{J}\right|}\left(\chi_{H}\left(\mathbf{B}_{p}\left(\mathbf{n}\right)\right)\right)=\chi_{H}\left(\mathbf{B}_{p}\left(\mathbf{n}\right)\right) \end{equation} Lastly, because $\chi_{H}\left(\mathbf{B}_{p}\left(\mathbf{n}\right)\right)$ is in $\mathbb{Z}^{d}$, the fact that $H:\mathbb{Z}^{d}\rightarrow\mathbb{Z}^{d}$ is equal to the restriction to $\mathbb{Z}^{d}$ of $H:\mathbb{Z}_{p}^{d}\rightarrow\mathbb{Z}_{p}^{d}$ necessarily forces $\chi_{H}\left(\mathbf{B}_{p}\left(\mathbf{n}\right)\right)$ to be a periodic point of $H$ in $\mathbb{Z}^{d}$, as desired. Q.E.D. \vphantom{} Like with the one-dimensional case, we now have several alternative versions. \begin{cor}[\textbf{Multi-Dimensional Correspondence Principle, Ver. 2}] \label{cor:MD CP v2}Suppose that $H$ is semi-basic\index{Correspondence Principle} $p$-Hydra map of dimension $d$ and depth $r$ that fixes $\mathbf{0}$. Then: \vphantom{} I. For every cycle $\Omega\subseteq\mathbb{Z}^{d}$ of $H$, viewing $\Omega\subseteq\mathbb{Z}^{d}$ as a subset of $\mathbb{Z}_{q_{H}}^{d}$, the intersection $\chi_{H}\left(\mathbb{Z}_{p}^{r}\right)\cap\Omega$ is non-empty. Moreover, for every $\mathbf{x}\in\chi_{H}\left(\mathbb{Z}_{p}^{r}\right)\cap\Omega$, there is an $\mathbf{n}\in\mathbb{N}_{0}^{r}\backslash\left\{ \mathbf{0}\right\} $ so that: \[ \mathbf{x}=\left(\mathbf{I}_{d}-M_{H}\left(\mathbf{n}\right)\right)^{-1}\chi_{H}\left(\mathbf{n}\right)=\chi_{H}\left(\mathbf{B}_{p}\left(\mathbf{n}\right)\right) \] where the equality occurs in $\mathbb{Q}^{d}$. \vphantom{} II. Suppose in addition that $H$ is integral. Let $\mathbf{n}\in\mathbb{N}_{0}^{r}\backslash\left\{ \mathbf{0}\right\} $. If the tuple $\mathbf{x}\in\mathbb{Z}_{q_{H}}^{d}$ given by: \[ \mathbf{x}=\left(\mathbf{I}_{d}-M_{H}\left(\mathbf{n}\right)\right)^{-1}\chi_{H}\left(\mathbf{n}\right)=\chi_{H}\left(\mathbf{B}_{p}\left(\mathbf{n}\right)\right) \] is an element of $\mathbb{Z}^{d}$, then $\mathbf{x}$ is necessarily a periodic point of $H$. \end{cor} Proof: Re-write the results of \textbf{Theorem \ref{thm:MD CP v1}} using \textbf{Lemma \ref{lem:MD Chi_H o B functional equation}}. Q.E.D. \begin{cor}[\textbf{Multi-Dimensional Correspondence Principle, Ver. 3}] \label{cor:MD CP v3}Suppose that $H$ is an integral semi-basic \index{Correspondence Principle}$p$-Hydra map of dimension $d$ and depth $r$ that fixes $\mathbf{0}$. Then, the set of non-zero periodic points of $H$ is equal to $\mathbb{Z}^{d}\cap\chi_{H}\left(\mathbb{Q}^{r}\cap\left(\mathbb{Z}_{p}^{r}\right)^{\prime}\right)$ (where, recall, $\left(\mathbb{Z}_{p}^{r}\right)^{\prime}=\mathbb{Z}_{p}^{\prime}\backslash\mathbb{N}_{0}^{r}$). \end{cor} \begin{rem} The implication ``If $\mathbf{x}\in\mathbb{Z}^{d}\backslash\left\{ \mathbf{0}\right\} $ is a periodic point, then $\mathbf{x}\in\chi_{H}\left(\mathbb{Q}^{r}\cap\left(\mathbb{Z}_{p}^{r}\right)^{\prime}\right)$'' \emph{does not }require $H$ to be integral. \end{rem} Proof: Before we do anything else, note that because $H$ is integral and semi-basic, \textbf{Lemma \ref{lem:MD integrality lemma} }guarantees $H$ will be proper. I. Let $\mathbf{x}$ be a non-zero periodic point of $H$, and let $\Omega$ be the unique cycle of $H$ in $\mathbb{Z}$ which contains $\mathbf{x}$. By Version 1 of the Correspondence Principle (\textbf{Theorem \ref{thm:MD CP v1}}), there exists a $\mathbf{y}\in\Omega$ and a $\mathbf{z}=B_{p}\left(\mathbf{n}\right)\subset\mathbb{Z}_{p}^{r}$ (for some $\mathbf{n}\in\mathbb{N}_{0}^{r}$ containing at least one non-zero entry) so that $\chi_{H}\left(\mathbf{z}\right)=\mathbf{y}$. Because $\mathbf{y}$ is in $\Omega$, there is an $k\geq1$ so that $\mathbf{x}=H^{\circ k}\left(\mathbf{y}\right)$. As such, there is a \emph{unique} length $k$ block-string $\mathbf{I}\in\textrm{String}^{r}\left(p\right)$ so that $H_{\mathbf{I}}\left(\mathbf{y}\right)=H^{\circ k}\left(\mathbf{y}\right)=\mathbf{x}$. Now, let $\mathbf{J}\in\textrm{String}_{\infty}^{r}\left(p\right)$ represent $\mathbf{z}$; note that $\mathbf{J}$ is infinite and that its columns are periodic. Using $\chi_{H}$'s concatenation identity (\textbf{Lemma \ref{eq:MD Chi_H concatenation identity}}), we have: \begin{equation} \mathbf{x}=H_{\mathbf{I}}\left(\mathbf{y}\right)=H_{\mathbf{I}}\left(\chi_{H}\left(\mathbf{z}\right)\right)=\chi_{H}\left(\mathbf{I}\wedge\mathbf{J}\right) \end{equation} Next, let $\mathbf{w}\in\mathbb{Z}_{p}^{r}$ be the $r$-tuple of $p$-adic integers represented by the infinite block-string $\mathbf{I}\wedge\mathbf{J}$; note that $\mathbf{w}$ is \emph{not} an element of $\mathbb{N}_{0}^{r}$. By the above, $\chi_{H}\left(\mathbf{w}\right)=\mathbf{x}$, and hence that $\mathbf{x}\in\mathbb{Z}^{d}\cap\chi_{H}\left(\mathbb{Z}_{p}^{r}\right)$. Finally, since $\mathbf{z}=B_{p}\left(\mathbf{n}\right)$ and $\mathbf{n}\neq\mathbf{0}$, the $p$-adic digits of $\mathbf{z}$'s entries are periodic, which forces $\mathbf{z}$ to be an element of $\mathbb{Q}^{r}\cap\left(\mathbb{Z}_{p}^{r}\right)^{\prime}$. So, letting $\mathbf{m}$ be the rational integer $r$-tuple represented by length-$k$ block-string $\mathbf{I}$, we have: \begin{equation} \mathbf{w}\sim\mathbf{I}\wedge\mathbf{J}\sim\mathbf{m}+p^{\lambda_{p}\left(\mathbf{m}\right)}\mathbf{z} \end{equation} This shows that $\mathbf{w}\in\mathbb{Q}^{r}\cap\left(\mathbb{Z}_{p}^{r}\right)^{\prime}$, and hence, that: \[ \mathbf{x}=\chi_{H}\left(\mathbf{w}\right)\in\mathbb{Z}^{d}\cap\chi_{H}\left(\mathbb{Q}^{r}\cap\left(\mathbb{Z}_{p}^{r}\right)^{\prime}\right) \] \vphantom{} II. Suppose $\mathbf{x}\in\mathbb{Z}^{d}\cap\chi_{H}\left(\mathbb{Q}^{r}\cap\left(\mathbb{Z}_{p}^{r}\right)^{\prime}\right)$, with $\mathbf{x}=\chi_{H}\left(\mathbf{z}\right)$ for some $\mathbf{z}\in\mathbb{Q}^{r}\cap\left(\mathbb{Z}_{p}^{r}\right)^{\prime}$. As a $r$-tuple of rational numbers are all $p$-adic integers which are \emph{not} elements of $\mathbb{N}_{0}$, the $p$-adic digits of the entries of $\mathbf{z}$ are all eventually periodic. As such, there are $\mathbf{m},\mathbf{n}\in\mathbb{N}_{0}^{r}$ (with $\mathbf{n}\neq\mathbf{0}$) so that: \begin{equation} \mathbf{z}=\mathbf{m}+p^{\lambda_{p}\left(\mathbf{m}\right)}B_{p}\left(\mathbf{n}\right) \end{equation} Here, the $p$-adic digits of the entries of $\mathbf{n}$ generate the periodic parts of the digits of $\mathbf{z}$'s entries. On the other hand, the $p$-adic digits of $\mathbf{m}$'s entries are the finite-length sequences in the digits of the entries of $\mathbf{z}$ that occur before the periodicity sets in. Now, let $\mathbf{I}$ be the finite block string representing $\mathbf{m}$, and let $\mathbf{J}$ be the infinite block-string representing $B_{p}\left(\mathbf{n}\right)$. Then, $\mathbf{z}=\mathbf{I}\wedge\mathbf{J}$. Using \textbf{Lemmata \ref{eq:MD Chi_H concatenation identity}} and \textbf{\ref{lem:MD Chi_H o B functional equation}} (the concatenation identity and $\chi_{H}\circ\mathbf{B}_{p}$ functional equation, respectively), we have: \begin{equation} \mathbf{x}=\chi_{H}\left(\mathbf{I}\wedge\mathbf{J}\right)=H_{\mathbf{I}}\left(\chi_{H}\left(\mathbf{J}\right)\right)=H_{\mathbf{I}}\left(\chi_{H}\left(B_{p}\left(\mathbf{n}\right)\right)\right)=H_{\mathbf{I}}\left(\left(\mathbf{I}_{d}-M_{H}\left(\mathbf{n}\right)\right)^{-1}\chi_{H}\left(\mathbf{n}\right)\right) \end{equation} Here, let $\mathbf{y}\overset{\textrm{def}}{=}\left(\mathbf{I}_{d}-M_{H}\left(\mathbf{n}\right)\right)^{-1}\chi_{H}\left(\mathbf{n}\right)$ is a $d$-tuple of rational numbers. \begin{claim} $\left\Vert \mathbf{y}\right\Vert _{p}\leq1$. Proof of claim: First, since $\mathbf{y}\in\mathbb{Q}^{d}$, it also lies in $\mathbb{Q}_{p}^{d}$. So, by way of contradiction, suppose $\left\Vert \mathbf{y}\right\Vert _{p}>1$. Since $H$ is proper, \textbf{Lemma \ref{lem:MD wrong values lemma}} tells us that no matter which branches of $H$ are specified by $\mathbf{I}$, the composition sequence $H_{\mathbf{I}}$ maps $\mathbf{y}\in\mathbb{Q}_{p}^{d}\backslash\mathbb{Z}_{p}^{d}$ to $H_{\mathbf{I}}\left(\mathbf{y}\right)\in\mathbb{Q}_{p}^{d}\backslash\mathbb{Z}_{p}^{d}$, and so $\left\Vert H_{\mathbf{i}}\left(\mathbf{y}\right)\right\Vert _{p}>1$. However, $H_{\mathbf{i}}\left(\mathbf{y}\right)=\mathbf{x}$, and $\mathbf{x}\in\mathbb{N}_{0}^{d}$; hence, $1<\left\Vert H_{\mathbf{i}}\left(\mathbf{y}\right)\right\Vert _{p}=\left\Vert \mathbf{x}\right\Vert _{p}\leq1$. This is impossible! So, it must be that $\left\Vert \mathbf{y}\right\Vert _{p}\leq1$. This proves the claim. \end{claim} \begin{claim} \label{claim:5}$\mathbf{x}=H^{\circ\left|\mathbf{I}\right|}\left(\mathbf{y}\right)$ Proof of claim: Suppose the equality failed. Then $H_{\mathbf{I}}\left(\mathbf{y}\right)=\mathbf{x}\neq H^{\circ\left|\mathbf{I}\right|}\left(\mathbf{y}\right)$, and so $\mathbf{x}=H_{\mathbf{I}}\left(\mathbf{y}\right)$ is a wrong value of $H$ with seed $\mathbf{y}$. Because $H$ is proper,\textbf{ Lemma \ref{lem:MD wrong values lemma}} forces $\left\Vert \mathbf{x}\right\Vert _{p}=\left\Vert H_{\mathbf{I}}\left(\mathbf{y}\right)\right\Vert _{p}>1$. However, like in the one-dimensional case, this is just as impossible as it was in the previous paragraph, seeing as $\left\Vert \mathbf{x}\right\Vert _{p}\leq1$. This proves the claim. \end{claim} \vphantom{} Finally, let $\mathbf{V}$ be the shortest block-string representing $\mathbf{n}$, so that $\mathbf{J}$ (the block-string representing $B_{p}\left(\mathbf{n}\right)$) is obtained by concatenating infinitely many copies of $\mathbf{v}$. Because $q_{H}$ is co-prime to all the non-zero entries of $\mathbf{D}_{\mathbf{j}}$ for all $\mathbf{j}\in\mathbb{Z}^{r}/p\mathbb{Z}^{r}$, \emph{note that $H_{\mathbf{V}}$ is continuous on $\mathbb{Z}_{q_{H}}^{d}$}. As such: \begin{equation} \chi_{H}\left(B_{p}\left(\mathbf{n}\right)\right)\overset{\mathbb{Z}_{q_{H}}^{d}}{=}\lim_{k\rightarrow\infty}\chi_{H}\left(\mathbf{V}^{\wedge k}\right) \end{equation} implies: \begin{align*} H_{\mathbf{V}}\left(\chi_{H}\left(B_{p}\left(\mathbf{n}\right)\right)\right) & \overset{\mathbb{Z}_{q_{H}}^{d}}{=}\lim_{k\rightarrow\infty}H_{\mathbf{V}}\left(\chi_{H}\left(\mathbf{V}^{\wedge k}\right)\right)\\ & \overset{\mathbb{Z}_{q_{H}}^{d}}{=}\lim_{k\rightarrow\infty}\chi_{H}\left(\mathbf{V}^{\wedge\left(k+1\right)}\right)\\ & \overset{\mathbb{Z}_{q_{H}}^{d}}{=}\chi_{H}\left(B_{p}\left(\mathbf{n}\right)\right) \end{align*} Hence, $H_{\mathbf{V}}\left(\mathbf{y}\right)=\mathbf{y}$. Since we showed that $\left\Vert \mathbf{y}\right\Vert _{p}\leq1$, the propriety of $H$ allows us to apply \textbf{Lemma \ref{lem:properness lemma}} and conclude that $H_{\mathbf{V}}\left(\mathbf{y}\right)=H^{\circ\left|\mathbf{V}\right|}\left(\mathbf{y}\right)$. Thus, $\mathbf{y}$ is a periodic point of $H:\mathbb{Z}_{p}^{d}\rightarrow\mathbb{Z}_{p}^{d}$. By \textbf{Claim \ref{claim:5}}, $H$ iterates $\mathbf{y}$ to $\mathbf{x}$. Since $\mathbf{y}$ is a periodic point of $H$ in $\mathbb{Z}_{p}^{d}$, this forces $\mathbf{x}$ and $\mathbf{y}$ to belong to the same cycle of $H$ in $\mathbb{Z}_{p}^{d}$, with $\mathbf{x}$ being a periodic point of $H$. As such, just as $H$ iterates $\mathbf{y}$ to $\mathbf{x}$, so too does $H$ iterate $\mathbf{x}$ to $\mathbf{y}$. Finally, since $\mathbf{x}\in\mathbb{N}_{0}^{d}$, so too is $\mathbf{y}$. integer as well. Thus, $\mathbf{x}$ belongs to a cycle of $H$ in $\mathbb{Z}^{d}$, as desired. Q.E.D. \vphantom{} Like in the one-dimensional case, the fact that the orbit classes of $H$'s attracting cycles, isolated cycles, and divergent trajectories constitute a partition of $\mathbb{Z}^{d}$ allows us to use the Correspondence Principle to make conclusions about the divergent trajectories. \begin{prop} \label{prop:MD Preparing for application to divergent trajectories}Let $H$ be as given in \textbf{\emph{Corollary \ref{cor:MD CP v3}}}. Additionally, suppose that $\left\Vert H_{\mathbf{j}}\left(\mathbf{0}\right)\right\Vert _{q_{H}}=1$ for all $\mathbf{j}\in\left(\mathbb{Z}^{r}/p\mathbb{Z}^{r}\right)\backslash\left\{ \mathbf{0}\right\} $. Then $\chi_{H}\left(\mathbf{z}\right)\neq\mathbf{0}$ for any $\mathbf{z}\in\mathbb{Z}_{p}^{r}\backslash\mathbb{Q}^{r}$. \end{prop} Proof: Let $H$ as given. By way of contradiction, suppose that $\chi_{H}\left(\mathbf{z}\right)=\mathbf{0}$ for some $\mathbf{z}=\left(\mathfrak{z}_{1},\ldots,\mathfrak{z}_{r}\right)\in\mathbb{Z}_{p}^{r}\backslash\mathbb{Q}^{r}$. Let $\mathbf{j}\in\mathbb{Z}^{r}/p\mathbb{Z}^{r}$ denote $\left[\mathbf{z}\right]_{p}$, and let $\mathbf{z}^{\prime}$ denote: \begin{equation} \mathbf{z}^{\prime}\overset{\textrm{def}}{=}\frac{\mathbf{z}-\mathbf{j}}{p}=\left(\frac{\mathfrak{z}_{1}-j_{1}}{p},\ldots,\frac{\mathfrak{z}_{r}-j_{r}}{p}\right) \end{equation} Then, we can write: \[ \mathbf{0}=\chi_{H}\left(\mathbf{z}\right)=\chi_{H}\left(p\mathbf{z}^{\prime}+\mathbf{j}\right)=H_{\mathbf{j}}\left(\chi_{H}\left(\mathbf{z}^{\prime}\right)\right) \] Next, suppose $\mathbf{j}=\mathbf{0}$. Because $H\left(\mathbf{0}\right)=\mathbf{0}$, we have: \[ \mathbf{0}=H_{\mathbf{0}}\left(\chi_{H}\left(\mathbf{z}^{\prime}\right)\right)=\mathbf{D}_{\mathbf{0}}^{-1}\mathbf{A}_{\mathbf{0}}\chi_{H}\left(\mathbf{z}^{\prime}\right) \] Since $\mathbf{A}_{\mathbf{0}}$ and $\mathbf{D}_{\mathbf{0}}$ are invertible, this forces $\chi_{H}\left(\mathbf{z}^{\prime}\right)=\mathbf{0}$. In this way, if the the first $n$ $p$-adic digits of $\mathfrak{z}_{m}$ are $0$ for all $m\in\left\{ 1,\ldots,r\right\} $, we can pull those digits out from $\mathbf{z}$ to obtain a $p$-adic integer tuple of unit $p$-adic norm ($\left\Vert \cdot\right\Vert _{p}$). If \emph{all }of the digits of all the $\mathfrak{z}_{m}$s are $0$, this then makes $\mathbf{z}=\mathbf{0}$, contradicting that $\mathbf{z}$ was given to be an element of $\mathbb{Z}_{p}^{r}\backslash\mathbb{Q}^{r}$. So, without loss of generality, we can assume that $\mathbf{j}\neq\mathbf{0}$. Hence: \begin{align*} \mathbf{0} & =H_{\mathbf{j}}\left(\chi_{H}\left(\mathbf{z}^{\prime}\right)\right)\\ & =H_{\mathbf{j}}^{\prime}\left(\mathbf{0}\right)\chi_{H}\left(\mathbf{z}^{\prime}\right)+H_{\mathbf{j}}\left(\mathbf{0}\right)\\ \left(H_{\mathbf{j}}^{\prime}\left(\mathbf{0}\right)=\mathbf{D}_{\mathbf{j}}^{-1}\mathbf{A}_{\mathbf{j}}\right); & \Updownarrow\\ \mathbf{D}_{\mathbf{j}}^{-1}\mathbf{A}_{\mathbf{j}}\chi_{H}\left(\mathbf{z}^{\prime}\right) & =-H_{\mathbf{j}}\left(\mathbf{0}\right) \end{align*} Because $H$ is semi-basic, $\mathbf{j}\neq\mathbf{0}$ tells us that $\left\Vert \mathbf{A}_{\mathbf{j}}\right\Vert _{q_{H}}<1$ and $\left\Vert \mathbf{D}_{\mathbf{j}}\right\Vert _{q_{H}}=1$. So: \begin{align*} \left\Vert H_{\mathbf{j}}\left(\mathbf{0}\right)\right\Vert _{q_{H}} & =\left\Vert \mathbf{D}_{\mathbf{j}}^{-1}\mathbf{A}_{\mathbf{j}}\chi_{H}\left(\mathbf{z}^{\prime}\right)\right\Vert _{q_{H}}\\ & \leq\underbrace{\left\Vert \mathbf{D}_{\mathbf{j}}^{-1}\right\Vert _{q_{H}}\left\Vert \mathbf{A}_{\mathbf{j}}\right\Vert _{q_{H}}}_{<1}\left\Vert \chi_{H}\left(\mathbf{z}^{\prime}\right)\right\Vert _{q_{H}}\\ & <\left\Vert \chi_{H}\left(\mathbf{z}^{\prime}\right)\right\Vert _{q_{H}} \end{align*} Since $\chi_{H}\left(\mathbb{Z}_{p}^{r}\right)\subseteq\mathbb{Z}_{q_{H}}^{d}$, we see that $\left\Vert \chi_{H}\left(\mathbf{z}^{\prime}\right)\right\Vert _{q_{H}}\leq1$, which forces $\left\Vert H_{\mathbf{j}}\left(\mathbf{0}\right)\right\Vert _{q_{H}}<1$ for our non-zero $\mathbf{j}$. However, we were given that $\left\Vert H_{\mathbf{j}}\left(\mathbf{0}\right)\right\Vert _{q_{H}}=1$ for all $\mathbf{j}\in\left(\mathbb{Z}^{r}/p\mathbb{Z}^{r}\right)\backslash\left\{ \mathbf{0}\right\} $. This is a contradiction! So, for each $m$, $\mathfrak{z}_{m}$ has no non-zero $p$-adic digits. This forces all of the $\mathfrak{z}_{m}$s to be $0$, which forces $\mathbf{z}=\mathbf{0}$. But $\mathbf{z}$ was given to be an element of $\mathbb{Z}_{p}^{r}\backslash\mathbb{Q}^{r}$, yet $\mathbf{0}\in\mathbb{Q}^{r}$\textemdash and that's our contradiction. So, it must be that $\chi_{H}\left(\mathbf{z}\right)\neq\mathbf{0}$. Q.E.D. \begin{thm} \label{thm:MD Divergent trajectories come from irrational z}Let\index{Hydra map@\emph{Hydra map!divergent trajectories}} $H$ be as given in \textbf{\emph{Corollary \ref{cor:MD CP v3}}}. Additionally, suppose that: \vphantom{} I. $H$ is contracting; \vphantom{} II. $\left\Vert H_{\mathbf{j}}\left(\mathbf{0}\right)\right\Vert _{q_{H}}=1$ for all $\mathbf{j}\in\left(\mathbb{Z}^{r}/p\mathbb{Z}^{r}\right)\backslash\left\{ \mathbf{0}\right\} $. \vphantom{} Under these hypotheses, let $\mathbf{z}\in\mathbb{Z}_{p}^{r}\backslash\mathbb{Q}^{r}$. If $\chi_{H}\left(\mathbf{z}\right)\in\mathbb{Z}^{d}$, then $\chi_{H}\left(\mathbf{z}\right)$ belongs to a divergent orbit class of $H$. \end{thm} Proof: Let $H$ and $\mathbf{z}$ be as given. By \textbf{Proposition \ref{prop:MD Preparing for application to divergent trajectories}}, $\chi_{H}\left(\mathbf{z}\right)\neq\mathbf{0}$. Now, by way of contradiction, suppose that $\chi_{H}\left(\mathbf{z}\right)$ did not belong to a divergent orbit class of $H$. By \textbf{Theorem \ref{thm:orbit classes partition domain}}, every element of $\mathbb{Z}^{d}$ belongs to either a divergent orbit class of $H$, or to an orbit class of $H$ which contains a cycle of $H$. This forces $\chi_{H}\left(\mathbf{z}\right)$ to be a pre-periodic point of $H$. As such, there is an $n\geq0$ so that $H^{\circ n}\left(\chi_{H}\left(\mathbf{z}\right)\right)$ is a periodic point of $H$. Using the functional equations of $\chi_{H}$ (\textbf{Lemma \ref{lem:MD Chi_H functional equations and characterization}}), we can write $H^{\circ n}\left(\chi_{H}\left(\mathbf{z}\right)\right)=\chi_{H}\left(\mathbf{y}\right)$ where $\mathbf{y}=\mathbf{m}+p^{\lambda_{p}\left(\mathbf{m}\right)}\mathbf{z}$, and where $\mathbf{m}\in\mathbb{N}_{0}^{r}$ is the unique $r$-tuple of non-negative integers which represents the composition sequence of branches used in iterating $H$ $n$ times at $\chi_{H}\left(\mathbf{z}\right)$ to produce $\chi_{H}\left(\mathbf{y}\right)$. Because $\chi_{H}\left(\mathbf{z}\right)\neq\mathbf{0}$, we can use the integrality of $H$ along with the fact that $\mathbf{0}=H\left(\mathbf{0}\right)=H_{\mathbf{0}}\left(\mathbf{0}\right)$ to conclude that $\left\{ \mathbf{0}\right\} $ is an isolated cycle of $H$. This then guarantees that the periodic point $\chi_{H}\left(\mathbf{y}\right)$ is non-zero. Consequently, the multi-dimensional \textbf{Correspondence Principle }tells us that $\mathbf{y}$ must be an element of $\mathbb{Q}^{r}\cap\left(\mathbb{Z}_{p}^{r}\right)^{\prime}$. So, \emph{for each} $\ell\in\left\{ 1,\ldots,r\right\} $, the $\ell$th entry of $\mathbf{y}$ is a rational number; therefore, its sequence of $p$-adic digits is eventually periodic. However, since $\mathbf{y}=\mathbf{m}+p^{\lambda_{p}\left(\mathbf{m}\right)}\mathbf{z}$ where $\mathbf{z}\in\mathbb{Z}_{p}^{r}\backslash\mathbb{Q}^{r}$, there is at least one $\ell\in\left\{ 1,\ldots,r\right\} $ so that $\mathfrak{z}_{\ell}$'s $p$-adic digits are \emph{aperiodic}. This is a contradiction! So, it must be that $\chi_{H}\left(\mathbf{z}\right)$ is a divergent point of $H$. Q.E.D. \vphantom{} Like with the one-dimensional case, it would be highly desirable to prove the following conjecture true: \begin{conjecture}[\textbf{Correspondence Principle for Multi-Dimensional Divergent Points?}] \label{conj:MD correspondence theorem for divergent trajectories}Provided that $H$ satisfies certain prerequisites such as the hypotheses of \textbf{\emph{Theorem \ref{thm:MD Divergent trajectories come from irrational z}}}, $\mathbf{x}\in\mathbb{Z}^{d}$ belongs to a divergent trajectory under $H$ if and only if there is a $\mathbf{z}\in\mathbb{Z}_{p}^{r}\backslash\mathbb{Q}^{r}$ so that $\chi_{H}\left(\mathbf{z}\right)\in\mathbb{Z}^{d}$. \end{conjecture} \newpage{} \section{\label{sec:5.3 Rising-Continuity-in-Multiple}Rising-Continuity in Multiple Dimensions} IN THIS SECTION, WE FIX INTEGERS $r,d\geq1$ (WITH $r\leq d$) ALONG WITH TWO DISTINCT PRIME NUMBERS $p$ AND $q$. UNLESS SAID OTHERWISE, $\mathbb{K}$ DENOTES A METRICALLY COMPLETE VALUED FIELD, POSSIBLY ARCHIMEDEAN; $K$, MEANWHILE, DENOTES A METRICALLY COMPLETE VALUED \emph{NON-ARCHIMEDEAN} FIELD, UNLESS STATED OTHERWISE. \subsection{\label{subsec:5.3.1 Tensor-Products}Multi-Dimensional Notation and Tensor Products} \begin{notation} \index{multi-dimensional!notation}\ \vphantom{} I. Like with $\left\Vert \mathbf{z}\right\Vert _{p}$ for $\mathbf{z}\in\mathbb{Z}_{p}^{r}$, for $\hat{\mathbb{Z}}_{p}^{r}$, we write: \begin{equation} \left\Vert \mathbf{t}\right\Vert _{p}\overset{\textrm{def}}{=}\max\left\{ p^{-v_{p}\left(t_{1}\right)},\ldots,p^{-v_{p}\left(t_{m}\right)}\right\} ,\textrm{ }\forall\mathbf{t}\in\hat{\mathbb{Z}}_{p}^{r}\label{eq:Definition of bold t p norm} \end{equation} \begin{equation} v_{p}\left(\mathbf{t}\right)\overset{\textrm{def}}{=}\min\left\{ v_{p}\left(t_{1}\right),\ldots,v_{p}\left(t_{m}\right)\right\} ,\textrm{ }\forall\mathbf{t}\in\hat{\mathbb{Z}}_{p}^{r}\label{eq:Definition of v_P of bold t} \end{equation} so that $\left\Vert \mathbf{t}\right\Vert _{p}=p^{-v_{p}\left(\mathbf{t}\right)}$. In this notation, $\left\{ \mathbf{t}:\left\Vert \mathbf{t}\right\Vert _{p}=p^{n}\right\} $ (equivalently $\left\{ \mathbf{t}:v_{p}\left(\mathbf{t}\right)=-n\right\} $) tells us that for each $\mathbf{t}$, the inequality $\left|t_{m}\right|_{p}\leq p^{n}$ occurs for all $m$, with equality actually being achieved for \emph{at least} one $m\in\left\{ 1,\ldots,r\right\} $. $\left\{ \mathbf{t}:\left\Vert \mathbf{t}\right\Vert _{p}\leq p^{N}\right\} $, meanwhile, tells us that, for each $\mathbf{t}$, $\left|t_{m}\right|_{p}\leq p^{N}$ occurs for all $m$. In particular: \begin{equation} \sum_{\left\Vert \mathbf{t}\right\Vert _{p}\leq p^{N}}=\sum_{\left|t_{1}\right|_{p}\leq p^{N}}\cdots\sum_{\left|t_{r}\right|_{p}\leq p^{N}}\label{eq:Definition of sum of bold t norm _p lessthanorequalto p^N} \end{equation} That being said, the reader should take care to notice that: \begin{equation} \sum_{\left\Vert \mathbf{t}\right\Vert _{p}=p^{n}}\neq\sum_{\left|t_{1}\right|_{p}=p^{n}}\cdots\sum_{\left|t_{r}\right|_{p}=p^{n}}\label{eq:Warning about summing bold t norm _p equal to p^N} \end{equation} \vphantom{} II. We write: \begin{equation} \sum_{\mathbf{n}=\mathbf{0}}^{p^{N}-1}\overset{\textrm{def}}{=}\sum_{n_{1}=0}^{p^{N}-1}\cdots\sum_{n_{r}=0}^{p^{N}-1}\label{eq:Definition of sum from bold n eq zero to P^N minus 1} \end{equation} and define: \begin{equation} \mathbf{n}\leq p^{m}-1\label{eq:Definition of bold n lessthanoreqto P^m minus 1} \end{equation} to indicate that $0\leq n_{\ell}\leq p^{m}-1$ for all $\ell\in\left\{ 1,\ldots,r\right\} $. Note then that: \begin{equation} \sum_{\mathbf{j}=\mathbf{0}}^{p-1}=\sum_{\mathbf{j}\in\mathbb{Z}^{r}/p\mathbb{Z}^{r}}\label{eq:Definition of sum from bold j is 0 to P minus 1} \end{equation} and also: \begin{equation} \sum_{\mathbf{j}>\mathbf{0}}^{p-1}\overset{\textrm{def}}{=}\sum_{\mathbf{j}\in\left(\mathbb{Z}^{r}/p\mathbb{Z}^{r}\right)\backslash\left\{ \mathbf{0}\right\} }\label{eq:Definition of sum from bold j greaterthan 0 to P minus 1} \end{equation} Consequently, $\forall\mathbf{j}\in\mathbb{Z}^{r}/\mathbb{Z}^{r}$ and $\forall\mathbf{j}\leq p-1$ denote the same sets of $\mathbf{j}$s. \vphantom{} III. $\mathbf{1}_{\mathbf{0}}\left(\mathbf{t}\right)$\nomenclature{$\mathbf{1}_{\mathbf{0}}\left(\mathbf{t}\right)$}{ } denotes the function on $\hat{\mathbb{Z}}_{p}^{r}$ which is $1$ if $\mathbf{t}\overset{1}{\equiv}\mathbf{0}$ and is $0$ otherwise. For $\mathbf{s}\in\hat{\mathbb{Z}}_{p}^{r}$, I write $\mathbf{1}_{\mathbf{s}}\left(\mathbf{t}\right)\overset{\textrm{def}}{=}\mathbf{1}_{\mathbf{0}}\left(\mathbf{t}-\mathbf{s}\right)$. \vphantom{} IV. For a matrix $\mathbf{A}$ with entries in $\mathbb{Z}_{q}$, recall we write $\left\Vert \mathbf{A}\right\Vert _{q}$ to denote the maximum of the $q$-adic absolute values of $\mathbf{A}$'s entries. When $\mathbf{A}$ has entries in $\overline{\mathbb{Q}}$, we write $\left\Vert \mathbf{A}\right\Vert _{\infty}$ to denote the maximum of the complex absolute values of $\mathbf{A}$'s entries. \vphantom{} V. For any $\mathbf{t}\in\hat{\mathbb{Z}}_{p}^{r}$, we write \nomenclature{$\left|\mathbf{t}\right|_{p}$}{$\left(p^{-v_{p}\left(\mathbf{t}\right)},\ldots,p^{-v_{p}\left(\mathbf{t}\right)}\right)$}$\left|\mathbf{t}\right|_{p}$ to denote the $r$\emph{-tuple}: \begin{equation} \left|\mathbf{t}\right|_{p}\overset{\textrm{def}}{=}\left(p^{-v_{p}\left(\mathbf{t}\right)},\ldots,p^{-v_{p}\left(\mathbf{t}\right)}\right)\label{eq:Definition of single bars of bold t _P} \end{equation} Consequently: \begin{equation} \frac{\mathbf{t}\left|\mathbf{t}\right|_{p}}{p}=p^{-v_{p}\left(\mathbf{t}\right)-1}\mathbf{t}=\left(p^{-v_{p}\left(\mathbf{t}\right)-1}t_{1},\ldots,p^{-v_{p}\left(\mathbf{t}\right)-1}t_{r}\right),\textrm{ }\forall\mathbf{t}\in\hat{\mathbb{Z}}_{p}^{r}\label{eq:Definition of bold t projection onto first Z_P hat level set} \end{equation} \end{notation} \vphantom{} Recall our earlier notation for writing $\mathbb{K}^{\rho,c}$ to denote the $\mathbb{K}$-linear space of all $\rho\times c$ matrices with entries in $\mathbb{K}$ for integers $\rho,c\geq1$. When $c=1$, we identify $\mathbb{K}^{\rho,1}$ with $\mathbb{K}^{\rho}$. \begin{defn} \label{nota:Second batch}For a function $\mathbf{F}:\mathbb{Z}_{p}^{r}\rightarrow\mathbb{K}^{\rho,c}$, we write \nomenclature{$\left\Vert \mathbf{F}\right\Vert _{p,\mathbb{K}}$}{ } $\left\Vert \mathbf{F}\right\Vert _{p,\mathbb{K}}$ to denote: \begin{equation} \left\Vert \mathbf{F}\right\Vert _{p,\mathbb{K}}\overset{\textrm{def}}{=}\sup_{\mathbf{z}\in\mathbb{Z}_{p}^{r}}\left\Vert \mathbf{F}\left(\mathbf{z}\right)\right\Vert _{\mathbb{K}}\label{eq:Definition of P,K norm} \end{equation} Here, $\left\Vert \mathbf{F}\left(\mathbf{z}\right)\right\Vert _{\mathbb{K}}$ is the maximum of the $\mathbb{K}$ absolute values of the entries of the matrix $\mathbf{F}\left(\mathbf{z}\right)$. We write $\left\Vert \cdot\right\Vert _{p,q}$ in the case where $\mathbb{K}$ is a $q$-adic field, and write $\left\Vert \cdot\right\Vert _{p,\infty}$ when $\mathbb{K}$ is $\mathbb{C}$. We \emph{also} use $\left\Vert \cdot\right\Vert _{p,\mathbb{K}}$ to denote the corresponding norm of a matrix-valued function on $\hat{\mathbb{Z}}_{p}^{r}$. \end{defn} \begin{defn} Let\index{matrix!Hadamard product} $\mathbf{A}=\left\{ a_{j,k}\right\} _{1\leq j\leq\rho,1\leq k\leq c}$ and $\mathbf{B}=\left\{ b_{j,k}\right\} _{1\leq j\leq\rho,1\leq k\leq c}$ be elements of $\mathbb{K}^{\rho,c}$. Then, we write \nomenclature{$\mathbf{A}\odot\mathbf{B}$}{the Hadamard product of matrices}$\mathbf{A}\odot\mathbf{B}$ to denote the matrix: \begin{equation} \mathbf{A}\odot\mathbf{B}\overset{\textrm{def}}{=}\left\{ a_{j,k}\cdot b_{j,k}\right\} _{1\leq j\leq\rho,1\leq k\leq c}\label{eq:Definition of the Hadamard product of two matrices} \end{equation} This is called the \textbf{Hadamard product }of $\mathbf{A}$ and $\mathbf{B}$. Given $\mathbf{A}_{1},\ldots,\mathbf{A}_{N}$, where: \begin{equation} \mathbf{A}_{n}=\left\{ a_{j,k}\left(n\right)\right\} _{1\leq j\leq\rho,1\leq k\leq c} \end{equation} for each $n$, we then define write: \begin{equation} \bigodot_{n=1}^{N}\mathbf{A}_{n}\overset{\textrm{def}}{=}\left\{ \prod_{n=1}^{N}a_{j,k}\left(n\right)\right\} _{1\leq j\leq\rho,1\leq k\leq c}\label{eq:Definition of the tensor product of n matrices} \end{equation} Note that $\odot$ reduces to standard multiplication when $\rho=c=1$. \end{defn} \begin{defn} We say a function $\mathbf{F}$ (resp. $\hat{\mathbf{F}}$) defined on $\mathbb{Z}_{p}^{r}$ (resp. $\hat{\mathbb{Z}}_{p}^{r}$) taking values in $\mathbb{K}^{\rho,c}$ is \textbf{elementary }if\index{elementary!function} there are functions $\mathbf{F}_{1},\ldots,\mathbf{F}_{r}$ (resp. $\hat{\mathbf{F}}_{1},\ldots,\hat{\mathbf{F}}_{r}$) defined on $\mathbb{Z}_{p}$ (resp. $\hat{\mathbb{Z}}_{p}$) so that: \begin{equation} \mathbf{F}\left(\mathbf{z}\right)=\bigodot_{m=1}^{r}\mathbf{F}\left(\mathfrak{z}_{m}\right),\textrm{ }\forall\mathbf{z}\in\mathbb{Z}_{p}^{r} \end{equation} and: \begin{equation} \hat{\mathbf{F}}\left(\mathbf{t}\right)=\bigodot_{m=1}^{r}\hat{\mathbf{F}}\left(t_{m}\right),\textrm{ }\forall\mathbf{t}\in\hat{\mathbb{Z}}_{p}^{r} \end{equation} respectively. We write \nomenclature{$E\left(\mathbb{Z}_{p}^{r},\mathbb{K}^{\left(\rho,c\right)}\right)$}{set of elementary tensors $\mathbb{Z}_{p}^{r}\rightarrow\mathbb{K}^{\rho,c}$}$E\left(\mathbb{Z}_{p}^{r},\mathbb{K}^{\rho,c}\right)$ (resp. \nomenclature{$E\left(\hat{\mathbb{Z}}_{p}^{r},\mathbb{K}^{\left(\rho,c\right)}\right)$}{set of elementary tensors $\hat{\mathbb{Z}}_{p}^{r}\rightarrow\mathbb{K}^{\rho,c}$}$E\left(\hat{\mathbb{Z}}_{p}^{r},\mathbb{K}^{\rho,c}\right)$) to denote the $\mathbb{K}$-span of all elementary functions (a.k.a., \textbf{elementary tensors}\index{elementary!tensors}) on $\mathbb{Z}_{p}^{r}$ (resp. $\hat{\mathbb{Z}}_{p}^{r}$). Note that the Hadamard products reduce to entry-wise products of vectors when $c=1$. \end{defn} \begin{defn}[\textbf{Multi-Dimensional Function Spaces}] \ \vphantom{} I. We write \nomenclature{$B\left(\mathbb{Z}_{p}^{r},K\right)$}{set of bounded functions $\mathbb{Z}_{p}^{r}\rightarrow K$}$B\left(\mathbb{Z}_{p}^{r},K\right)$ to denote the $K$-linear space of all bounded functions $f:\mathbb{Z}_{p}^{r}\rightarrow K$. This is a non-archimedean Banach space under the norm: \begin{equation} \left\Vert f\right\Vert _{p,K}\overset{\textrm{def}}{=}\sup_{\mathbf{z}\in\mathbb{Z}_{p}^{r}}\left|f\left(\mathbf{z}\right)\right|_{K}=\sup_{\mathfrak{z}_{1},\ldots,\mathfrak{z}_{r}\in\mathbb{Z}_{p}}\left|f\left(\mathfrak{z}_{1},\ldots,\mathfrak{z}_{r}\right)\right|_{K}\label{eq:Definition of scalar valued C Z_P norm} \end{equation} We write \nomenclature{$C\left(\mathbb{Z}_{p}^{r},K\right)$}{set of continuous functions $\mathbb{Z}_{p}^{r}\rightarrow K$ }$C\left(\mathbb{Z}_{p}^{r},K\right)$ to denote the space of all continuous $f:\mathbb{Z}_{p}^{r}\rightarrow K$. \vphantom{} II. We write \nomenclature{$B\left(\mathbb{Z}_{p}^{r},K^{\left(\rho,c\right)}\right)$}{set of bounded functions $\mathbb{Z}_{p}^{r}\rightarrow K^{\rho,c}$ }$B\left(\mathbb{Z}_{p}^{r},K^{\rho,c}\right)$ to denote the $K$-linear space of all bounded functions $\mathbf{F}:\mathbb{Z}_{p}^{r}\rightarrow K^{\rho,c}$, where: \begin{equation} \mathbf{F}\left(\mathbf{z}\right)=\left\{ F_{j,k}\left(\mathbf{z}\right)\right\} _{1\leq j\leq\rho,1\leq k\leq c}\in K^{\rho,c},\textrm{ }\forall\mathbf{z}\in\mathbb{Z}_{p}^{r} \end{equation} where, for each $\left(j,k\right)$, $F_{j,k}\in B\left(\mathbb{Z}_{p}^{r},K\right)$. $B\left(\mathbb{Z}_{p}^{r},K^{\rho,c}\right)$ becomes a non-archimedean Banach space under the norm $\left\Vert \mathbf{F}\right\Vert _{p,K}$ (the maximum of the supremum-over-$\mathbb{Z}_{p}^{r}$ of the $K$-adic absolute values of each entry of $\mathbf{F}$). We write \nomenclature{$C\left(\mathbb{Z}_{p}^{r},K^{\left(\rho,c\right)}\right)$}{set of continuous functions $\mathbb{Z}_{p}^{r}\rightarrow K^{\rho,c}$ }$C\left(\mathbb{Z}_{p}^{r},K^{\rho,c}\right)$ to denote the space of all continuous functions $\mathbf{F}:\mathbb{Z}_{p}^{r}\rightarrow K^{\rho,c}$. \vphantom{} III. We write \nomenclature{$B\left(\hat{\mathbb{Z}}_{p}^{r},K\right)$}{set of bounded functions $\hat{\mathbb{Z}}_{p}^{r}\rightarrow K$}$B\left(\hat{\mathbb{Z}}_{p}^{r},K\right)$ to denote the $K$-linear space of all bounded functions $\hat{f}:\hat{\mathbb{Z}}_{p}^{r}\rightarrow K$. This is a non-archimedean Banach space under the norm: \begin{equation} \left\Vert \hat{f}\right\Vert _{p,K}\overset{\textrm{def}}{=}\sup_{\mathbf{t}\in\hat{\mathbb{Z}}_{p}^{r}}\left|\hat{f}\left(\mathbf{t}\right)\right|_{K}=\sup_{t_{1},\ldots,t_{r}\in\hat{\mathbb{Z}}_{p}}\left|\hat{f}\left(\hat{t}_{1},\ldots,\hat{t}_{r}\right)\right|_{K}\label{eq:Definition of scalar valued C Z_P hat norm} \end{equation} We write $c_{0}\left(\hat{\mathbb{Z}}_{p}^{r},K\right)$\nomenclature{$c_{0}\left(\hat{\mathbb{Z}}_{p}^{r},K\right)$}{set of $f\in B\left(\hat{\mathbb{Z}}_{p}^{r},K\right)$ so that $\lim_{\left\Vert \mathbf{t}\right\Vert _{p}\rightarrow\infty}\left|\hat{f}\left(\mathbf{t}\right)\right|_{K}=0$} to denote the subspace of $B\left(\hat{\mathbb{Z}}_{p}^{r},K\right)$ consisting of all $\hat{f}$ for which: \begin{equation} \lim_{\left\Vert \mathbf{t}\right\Vert _{p}\rightarrow\infty}\left|\hat{f}\left(\mathbf{t}\right)\right|_{K}=0 \end{equation} \vphantom{} IV. We write \nomenclature{$B\left(\hat{\mathbb{Z}}_{p}^{r},K^{\left(\rho,c\right)}\right)$}{set of bounded functions $\hat{\mathbb{Z}}_{p}^{r}\rightarrow K^{\rho,c}$}$B\left(\hat{\mathbb{Z}}_{p}^{r},K^{\rho,c}\right)$ to denote the $K$-linear space of all bounded functions $\hat{\mathbf{F}}:\hat{\mathbb{Z}}_{p}^{r}\rightarrow K^{\rho,c}$, where: \begin{equation} \hat{\mathbf{F}}\left(\mathbf{t}\right)=\left\{ \hat{F}_{j,k}\left(\mathbf{t}\right)\right\} _{1\leq j\leq\rho,1\leq k\leq c}\in K^{\rho,c},\textrm{ }\forall\mathbf{t}\in\hat{\mathbb{Z}}_{p}^{r} \end{equation} where, for each $\left(j,k\right)$, $\hat{F}_{m}\in B\left(\hat{\mathbb{Z}}_{p}^{r},K^{\rho,c}\right)$. $B\left(\hat{\mathbb{Z}}_{p}^{r},K^{\rho,c}\right)$ becomes a non-archimedean Banach space under the norm $\left\Vert \hat{\mathbf{F}}\right\Vert _{p,K}^{r}$. We then write \nomenclature{$c_{0}\left(\hat{\mathbb{Z}}_{p}^{r},K^{\left(\rho,c\right)}\right)$}{set of bounded functions $\hat{\mathbb{Z}}_{p}^{r}\rightarrow K^{\rho,c}$ so that $\lim_{\left\Vert \mathbf{t}\right\Vert _{p}^{r}\rightarrow\infty}\left\Vert \hat{\mathbf{F}}\left(\mathbf{t}\right)\right\Vert _{K}=0$}$c_{0}\left(\hat{\mathbb{Z}}_{p}^{r},K^{\rho,c}\right)$ to denote the Banach subspace of $B\left(\hat{\mathbb{Z}}_{p}^{r},K^{\rho,c}\right)$ consisting of all those $\hat{\mathbf{F}}$ so that: \begin{equation} \lim_{\left\Vert \mathbf{t}\right\Vert _{p}^{r}\rightarrow\infty}\left\Vert \hat{\mathbf{F}}\left(\mathbf{t}\right)\right\Vert _{K}=0 \end{equation} \end{defn} \begin{rem} The Fourier transform will send functions $\mathbb{Z}_{p}^{r}\rightarrow K$ to functions $\hat{\mathbb{Z}}_{p}^{r}\rightarrow K$. Fourier transforms on $C\left(\mathbb{Z}_{p}^{r},K^{\rho,c}\right)$ are done component-wise. \end{rem} \vphantom{} Now, we need to introduce the tensor product. Fortunately, in all of the cases we shall be working with, the tensor product is nothing more than point-wise multiplication of functions. A reader desirous for greater abstract detail can refer to \cite{van Rooij - Non-Archmedean Functional Analysis} to slake their thirst. The fourth chapter of van Rooij's book has an \emph{entire section} dedicated to the tensor product\index{tensor!product} ($\otimes$\nomenclature{$\otimes$}{tensor product}). \begin{thm} \cite{van Rooij - Non-Archmedean Functional Analysis} Let $E$, $F$, and $G$ be Banach spaces over metrically complete non-archimedean fields. Then: \begin{equation} \left\Vert f\otimes g\right\Vert =\left\Vert f\right\Vert _{E}\left\Vert g\right\Vert _{F},\textrm{ }\forall f\in E,\textrm{ }\forall g\in F\label{eq:Norm of a tensor product} \end{equation} Additionally, for any linear operator $T:E\otimes F\rightarrow G$, the operator norm\index{operator norm} of $T$ is given by: \begin{equation} \left\Vert T\right\Vert \overset{\textrm{def}}{=}\sup_{\left\Vert f\right\Vert _{E}\leq1,\left\Vert g\right\Vert _{F}\leq1}\frac{\left\Vert T\left\{ f\otimes g\right\} \right\Vert }{\left\Vert f\right\Vert _{E}\left\Vert g\right\Vert _{F}}\label{eq:Definition of the norm of a linear operator on a tensor product} \end{equation} \end{thm} \begin{thm} \cite{van Rooij - Non-Archmedean Functional Analysis} Let $E_{1},F_{1},E_{2},F_{2}$ be Banach spaces over metrically complete non-archimedean fields. Given continuous linear operators $T_{j}:E_{j}\rightarrow F_{j}$ for $j\in\left\{ 1,2\right\} $, the operator $T_{1}\otimes T_{2}:E_{1}\otimes F_{1}\rightarrow E_{2}\otimes F_{2}$ is given by: \begin{equation} \left(T_{1}\otimes T_{2}\right)\left\{ f\otimes g\right\} =T_{1}\left\{ f\right\} \otimes T_{2}\left\{ g\right\} \label{eq:Action of tensor products of linear operators} \end{equation} \end{thm} \begin{cor} \cite{van Rooij - Non-Archmedean Functional Analysis} Let $E$ and $F$ be Banach spaces over a metrically complete non-archimedean field $K$. Then, the dual of $E\otimes F$ is $E^{\prime}\otimes F^{\prime}$, with elementary tensors $\mu\otimes\nu\in E^{\prime}\otimes F^{\prime}$ acting on elementary tensors $f\otimes g\in E\otimes F$ by: \begin{equation} \left(\mu\otimes\nu\right)\left(f\otimes g\right)\overset{K}{=}\mu\left(f\right)\cdot\nu\left(g\right)\label{eq:action of tensor product of linear functionals} \end{equation} \end{cor} \begin{thm} \cite{van Rooij - Non-Archmedean Functional Analysis}\ \vphantom{} I. \begin{equation} B\left(\mathbb{Z}_{p}^{r},K\right)=\bigotimes_{m=1}^{r}B\left(\mathbb{Z}_{p},K\right) \end{equation} \index{tensor!elementary}Elementary tensors in $B\left(\mathbb{Z}_{p}^{r},K\right)$ are of the form: \begin{equation} \left(\bigotimes_{m=1}^{r}f_{m}\right)\left(\mathbf{z}\right)=\left(\bigotimes_{m=1}^{r}f_{m}\right)\left(\mathfrak{z}_{1},\ldots,\mathfrak{z}_{r}\right)\overset{\textrm{def}}{=}\prod_{m=1}^{r}f_{m}\left(\mathfrak{z}_{m}\right) \end{equation} This also holds for $C\left(\mathbb{Z}_{p}^{r},K\right)$. \vphantom{} II. \begin{equation} B\left(\hat{\mathbb{Z}}_{p}^{r},K\right)=\bigotimes_{m=1}^{r}B\left(\hat{\mathbb{Z}}_{p},K\right) \end{equation} Elementary tensors in $B\left(\hat{\mathbb{Z}}_{p}^{r},K\right)$ are of the form: \begin{equation} \left(\bigotimes_{m=1}^{r}\hat{f}_{m}\right)\left(\mathbf{t}\right)=\left(\bigotimes_{m=1}^{r}\hat{f}_{m}\right)\left(t_{1},\ldots,t_{r}\right)\overset{\textrm{def}}{=}\prod_{m=1}^{r}\hat{f}_{m}\left(t_{m}\right) \end{equation} This also holds for $c_{0}\left(\hat{\mathbb{Z}}_{p}^{r},K\right)$. \vphantom{} III. \begin{equation} B\left(\mathbb{Z}_{p}^{r},K^{\rho,c}\right)=\bigotimes_{m=1}^{r}B\left(\mathbb{Z}_{p},K^{\rho,c}\right) \end{equation} where an elementary tensor in $B\left(\mathbb{Z}_{p}^{r},K^{\rho,c}\right)$ is of the form: \begin{equation} \mathbf{F}\left(\mathbf{z}\right)=\bigodot_{m=1}^{r}\mathbf{F}_{m}\left(\mathfrak{z}_{m}\right) \end{equation} where $\mathbf{F}_{m}$ is in $B\left(\mathbb{Z}_{p},K^{\rho,c}\right)$ for each $m$. This also holds for $C\left(\mathbb{Z}_{p}^{r},K^{\rho,c}\right)$. \vphantom{} IV. \begin{equation} B\left(\hat{\mathbb{Z}}_{p}^{r},K^{\rho,c}\right)=\bigotimes_{m=1}^{r}B\left(\hat{\mathbb{Z}}_{p},K^{\rho,c}\right) \end{equation} where an elementary tensor in $B\left(\hat{\mathbb{Z}}_{p}^{r},K^{\rho,c}\right)$ is of the form: \begin{equation} \hat{\mathbf{F}}\left(\mathbf{t}\right)=\bigodot_{m=1}^{r}\hat{\mathbf{F}}_{m}\left(t_{m}\right) \end{equation} where $\hat{\mathbf{F}}_{m}$ is in $B\left(\hat{\mathbb{Z}}_{p},K^{\rho,c}\right)$ for each $m$. This also holds for $c_{0}\left(\hat{\mathbb{Z}}_{p}^{r},K^{\rho,c}\right)$. \end{thm} \begin{rem} Every function in $C\left(\mathbb{Z}_{p}^{r},K\right)$ can be written as a linear combination of (possibly infinitely many) elementary functions. If the linear combination is infinite, then elements of the linear combination can be enumerated in a sequence which converges to $0$ in norm, so as to guarantee the convergence of the resulting infinite sum. \end{rem} \begin{defn}[\textbf{Multi-Dimensional van der Put Basis}] Given a prime $p$, we write $\mathcal{B}_{p}$ to denote the set of van der Put basis functions for $\mathbb{Z}_{p}$: \begin{equation} \mathcal{B}_{p}\overset{\textrm{def}}{=}\left\{ \left[\mathfrak{z}\overset{p^{\lambda_{p}\left(n\right)}}{\equiv}n\right]:n\in\mathbb{N}_{0}\right\} \label{eq:Definition of the van der Put basis} \end{equation} Next, for any $\mathbf{n}\in\mathbb{N}_{0}^{r}$, we write: \begin{equation} \left[\mathbf{z}\overset{p^{\lambda_{p}\left(\mathbf{n}\right)}}{\equiv}\mathbf{n}\right]\overset{\textrm{def}}{=}\prod_{\ell=1}^{r}\left[\mathfrak{z}_{\ell}\overset{p^{\lambda_{p}\left(n_{\ell}\right)}}{\equiv}n_{\ell}\right],\textrm{ }\forall\mathbf{z}\in\mathbb{Z}_{p}^{r}\label{eq:Notation for elementary P-adic van der Put tensors} \end{equation} We call (\ref{eq:Notation for elementary P-adic van der Put tensors}) an \textbf{elementary $p$-adic van der Put tensor}. \nomenclature{$\mathcal{B}_{p}^{\otimes r}$}{$\left\{ \left[\mathbf{z}\overset{p^{\lambda_{p}\left(\mathbf{n}\right)}}{\equiv}\mathbf{n}\right]:\mathbf{n}\in\mathbb{N}_{0}^{r}\right\}$}We then define the \textbf{$p$-adic van der Put basis of depth $r$ }as the set: \begin{equation} \mathcal{B}_{p}^{\otimes r}\overset{\textrm{def}}{=}\left\{ \left[\mathbf{z}\overset{p^{\lambda_{p}\left(\mathbf{n}\right)}}{\equiv}\mathbf{n}\right]:\mathbf{n}\in\mathbb{N}_{0}^{r}\right\} \label{eq:definition of elementary tensor van der Put basis} \end{equation} \end{defn} \begin{thm} $\mathcal{B}_{p}^{\otimes r}$ is a basis for $C\left(\mathbb{Z}_{p}^{r},K\right)$. \end{thm} Proof: $\mathcal{B}_{p}$ is a basis for $C\left(\mathbb{Z}_{p},K\right)$, so, tensoring the $\mathcal{B}_{p}$s to get $\mathcal{B}_{p}^{\otimes r}$ yields a basis for $C\left(\mathbb{Z}_{p}^{r},K\right)$. Q.E.D. \begin{defn}[\textbf{Multi-Dimensional van der Put Series}] We write \nomenclature{$\textrm{vdP}\left(\mathbb{Z}_{p}^{r},K\right)$}{the set of formal $\left(p,K\right)$-adic van der Put series of depth $r$ } to denote the $K$-linear space of all\index{van der Put!series!formal} \textbf{formal $\left(p,K\right)$-adic van der Put series of depth $r$}. The elements of $\textrm{vdP}\left(\mathbb{Z}_{p}^{r},K\right)$ are formal sums: \begin{equation} \sum_{\mathbf{n}\in\mathbb{N}_{0}^{r}}\mathfrak{a}_{\mathbf{n}}\left[\mathbf{z}\overset{p^{\lambda_{p}\left(\mathbf{n}\right)}}{\equiv}\mathbf{n}\right]\label{eq:Definition of a formal (P,K)-adic van der Put series} \end{equation} where the $\mathfrak{a}_{\mathbf{n}}$s are elements of $K$. The \textbf{domain of convergence }of a formal van der Put series is the set of all $\mathbf{z}\in\mathbb{Z}_{p}^{r}$ for which the series converges\footnote{Note: since $\mathbf{z}\overset{p^{\lambda_{p}\left(\mathbf{n}\right)}}{\equiv}\mathbf{n}$ can hold true for at most finitely many $\mathbf{n}$ whenever $\mathbf{z}\in\mathbb{N}_{0}^{r}$, observe that every formal van der Put series necessarily converges in $K$ at every $\mathbf{z}\in\mathbb{N}_{0}^{r}$.} in $K$. We call $\textrm{vdP}\left(\mathbb{Z}_{p}^{r},\mathbb{C}_{q}\right)$ the space of \textbf{formal $\left(p,q\right)$-adic van der Put series of depth $r$}. \end{defn} \begin{defn} Recall that for an integer $n\geq0$ and a prime $p$, we write: \begin{equation} n_{-}=\begin{cases} 0 & \textrm{if }n=0\\ n-n_{\lambda_{p}\left(n\right)-1}p^{\lambda_{p}\left(n\right)-1} & \textrm{if }n\geq1 \end{cases} \end{equation} where $n_{\lambda_{p}\left(n\right)-1}$ is the coefficient of the largest power of $p$ present in the $p$-adic expansion of $n$. \vphantom{} I. Given $\mathbf{n}\in\mathbb{N}_{0}^{d}$, we write \nomenclature{$\mathbf{n}_{-}$}{multi-dimensional analogue of $n_{-}$}$\mathbf{n}_{-}$ to denote the $d$-tuple obtained by applying the subscript $-$ operation to each entry of $\mathbf{n}$: $\mathbf{n}_{-}\overset{\textrm{def}}{=}\left(\left(n_{1}\right)_{-},\ldots,\left(n_{d}\right)_{-}\right)$. \vphantom{} II. Let $f:\mathbb{Z}_{p}^{r}\rightarrow K$ be an elementary function, with $f=\prod_{m=1}^{r}f_{m}$. Then, we define the \textbf{van der Put coefficients}\index{van der Put!coefficients}\textbf{ }\nomenclature{$c_{\mathbf{n}}\left(f\right)$}{$\mathbf{n}$th van der Put coefficient of $f$}$c_{\mathbf{n}}\left(f\right)$ of $f$ by: \begin{equation} c_{\mathbf{n}}\left(f\right)\overset{\textrm{def}}{=}\prod_{m=1}^{r}c_{n_{m}}\left(f_{m}\right),\textrm{ }\forall\mathbf{n}\in\mathbb{N}_{0}^{r}\label{eq:Def of the van der put coefficients of an elementary function} \end{equation} \end{defn} \begin{prop} \label{prop:vdP series of an elementary function}Every elementary\index{elementary!function} $f\in C\left(\mathbb{Z}_{p}^{r},K\right)$ is uniquely representable as a $\left(p,K\right)$-adic van der Put series of depth $r$: \begin{equation} f\left(\mathbf{z}\right)\overset{K}{=}\sum_{\mathbf{n}\in\mathbb{N}_{0}^{r}}c_{\mathbf{n}}\left(f\right)\left[\mathbf{z}\overset{p^{\lambda_{p}\left(\mathbf{n}\right)}}{\equiv}\mathbf{n}\right],\textrm{ }\mathbf{z}\in\mathbb{Z}_{p}^{r} \end{equation} Moreover, the $K$-convergence of this series is uniform over $\mathbb{Z}_{p}^{r}$. \end{prop} Proof: Let $f$ be elementary. Then, $f$ is a product of the form: \begin{equation} f\left(\mathbf{z}\right)=f\left(\mathfrak{z}_{1},\ldots,\mathfrak{z}_{r}\right)=\prod_{m=1}^{r}f_{m}\left(\mathfrak{z}_{m}\right) \end{equation} for $f_{m}\in C\left(\mathbb{Z}_{p},K\right)$. As such, each $f_{m}$ is uniquely representable by a uniformly convergent van der Put series: \begin{equation} f_{m}\left(\mathfrak{z}_{m}\right)=\sum_{n_{m}=0}^{\infty}c_{n_{m}}\left(f_{m}\right)\left[\mathfrak{z}_{m}\overset{p^{\lambda_{p}\left(n_{m}\right)}}{\equiv}n_{m}\right] \end{equation} with $\left|c_{n_{m}}\left(f_{m}\right)\right|_{K}\rightarrow0$ as $n_{m}\rightarrow\infty$. So: \begin{align*} f\left(\mathfrak{z}_{1},\ldots,\mathfrak{z}_{r}\right) & =\prod_{m=1}^{r}\left(\sum_{n_{m}=0}^{\infty}c_{n_{m}}\left(f_{m}\right)\left[\mathfrak{z}_{m}\overset{p^{\lambda_{p}\left(n_{m}\right)}}{\equiv}n_{m}\right]\right)\\ & =\sum_{n_{1}=0}^{\infty}\cdots\sum_{n_{r}=0}^{\infty}\left(\prod_{m=1}^{r}c_{n_{m}}\left(f_{m}\right)\left[\mathfrak{z}_{m}\overset{p^{\lambda_{p}\left(n_{m}\right)}}{\equiv}n_{m}\right]\right)\\ & =\sum_{\mathbf{n}\in\mathbb{N}_{0}^{r}}c_{\mathbf{n}}\left(f\right)\left[\mathbf{z}\overset{p^{\lambda_{p}\left(\mathbf{n}\right)}}{\equiv}\mathbf{n}\right] \end{align*} Q.E.D. \begin{prop} Let $g_{1},\ldots g_{N}$ be elementary functions in $C\left(\mathbb{Z}_{p}^{r},K\right)$. Then, for any $\alpha_{1},\ldots,\alpha_{N}\in K$, the linear combination $\sum_{k=1}^{N}\alpha_{k}g_{k}$ is represented by a uniformly convergent $\left(p,K\right)$-adic van der Put series of depth $r$, the coefficients of which are given by: \begin{equation} c_{\mathbf{n}}\left(\sum_{k=1}^{N}\alpha_{k}g_{k}\right)=\sum_{k=1}^{N}\alpha_{k}c_{\mathbf{n}}\left(g_{k}\right)\label{eq:Linear extension of MD van der Put coefficients} \end{equation} where $c_{\mathbf{n}}\left(g_{k}\right)$ is computed by \emph{(\ref{eq:Def of the van der put coefficients of an elementary function})}. This result also extends to infinite linear combinations, provided that they converge in norm. \end{prop} Proof: Each $g_{k}$ is representable by a uniformly convergent $\left(p,K\right)$-adic van der Put series of depth $r$. The proposition follows from the fact that the space of van der Put series is closed under linear combinations. Q.E.D. \begin{rem} Consequently, we can use (\ref{eq:Linear extension of MD van der Put coefficients}) to compute the van der Put coefficients of \emph{any }function in $C\left(\mathbb{Z}_{p}^{r},K\right)$, seeing as the $K$-span of $C\left(\mathbb{Z}_{p}^{r},K\right)$'s elementary functions is dense in $C\left(\mathbb{Z}_{p}^{r},K\right)$. \end{rem} \begin{lem} \label{lem:van der Put series for an arbitrary K-valued function on Z_P}Let $f\in C\left(\mathbb{Z}_{p}^{r},K\right)$. Then, there are unique constants $\left\{ c_{\mathbf{n}}\left(f\right)\right\} _{\mathbf{n}\in\mathbb{N}_{0}^{r}}$ so that: \begin{equation} f\left(\mathbf{z}\right)\overset{K}{=}\sum_{\mathbf{n}\in\mathbb{N}_{0}^{r}}c_{\mathbf{n}}\left(f\right)\left[\mathbf{z}\overset{p^{\lambda_{p}\left(\mathbf{n}\right)}}{\equiv}\mathbf{n}\right]\label{eq:van der Put series for an arbitrary K-valued function on Z_P} \end{equation} where the convergence is uniform in $\mathbf{z}$; specifically: \begin{equation} \lim_{\left\Vert \mathbf{n}\right\Vert _{\infty}\rightarrow\infty}\left|c_{\mathbf{n}}\left(f\right)\right|_{K}=0 \end{equation} where, recall (see \emph{(\ref{eq:Definition of infinity norm})} on page \emph{\pageref{eq:Definition of infinity norm}}): \begin{equation} \left\Vert \mathbf{n}\right\Vert _{\infty}=\max\left\{ n_{1},\ldots,n_{r}\right\} \end{equation} In particular, letting $\left\{ g_{k}\right\} _{k\geq1}$ and $\left\{ \alpha_{k}\right\} _{k\geq1}$ be any sequences of \index{elementary!function}elementary functions in $C\left(\mathbb{Z}_{p}^{r},K\right)$ and scalars in $K$, respectively, so that: \begin{equation} \lim_{N\rightarrow\infty}\left\Vert f-\sum_{k=1}^{N}\alpha_{k}g_{k}\right\Vert _{p,q}=0 \end{equation} then: \begin{equation} c_{\mathbf{n}}\left(f\right)\overset{K}{=}\lim_{N\rightarrow\infty}c_{\mathbf{n}}\left(\sum_{k=1}^{N}\alpha_{k}g_{k}\right)=\lim_{N\rightarrow\infty}\sum_{k=1}^{N}\alpha_{k}c_{\mathbf{n}}\left(g_{k}\right)\label{eq:Limit formula for the van der Put coefficients of an abitrary continuous (P,K)-adic function} \end{equation} We call the $c_{\mathbf{n}}\left(f\right)$s the \index{van der Put!coefficients}\textbf{van der Put coefficients }of $f$. \end{lem} Proof: I. (Existence) Since the $C\left(\mathbb{Z}_{p}^{r},K\right)$-elementary functions are dense in $C\left(\mathbb{Z}_{p}^{r},K\right)$, given any $f\in C\left(\mathbb{Z}_{p}^{r},K\right)$, we can choose $\left\{ g_{k}\right\} _{k\geq1}$ and $\left\{ \alpha_{k}\right\} _{k\geq1}$ as described above so that the norm convergence occurs. Writing: \begin{equation} \sum_{k=1}^{N}\alpha_{k}g_{k}\left(\mathbf{z}\right)\overset{K}{=}\sum_{\mathbf{n}\in\mathbb{N}_{0}^{r}}\left(\sum_{k=1}^{N}\alpha_{k}c_{\mathbf{n}}\left(g_{k}\right)\right)\left[\mathbf{z}\overset{p^{\lambda_{p}\left(\mathbf{n}\right)}}{\equiv}\mathbf{n}\right] \end{equation} \textemdash note, the convergence is uniform in $\mathbf{z}$\textemdash the assumed norm convergence allows us to interchange limits and sums: \begin{equation} f\left(\mathbf{z}\right)\overset{K}{=}\lim_{N\rightarrow\infty}\sum_{k=1}^{N}\alpha_{k}g_{k}\left(\mathbf{z}\right)\overset{K}{=}\sum_{\mathbf{n}\in\mathbb{N}_{0}^{r}}\underbrace{\left(\lim_{N\rightarrow\infty}\sum_{k=1}^{N}\alpha_{k}c_{\mathbf{n}}\left(g_{k}\right)\right)}_{c_{\mathbf{n}}\left(f\right)}\left[\mathbf{z}\overset{p^{\lambda_{p}\left(\mathbf{n}\right)}}{\equiv}\mathbf{n}\right] \end{equation} \vphantom{} II. (Uniqueness) Let $\left\{ g_{k}\right\} _{k\geq1}$ and $\left\{ \alpha_{k}\right\} _{k\geq1}$ be one choice of $g$s and $\alpha$s whose linear combinations converge in norm to $f$; let $\left\{ g_{k}^{\prime}\right\} _{k\geq1}$ and $\left\{ \alpha_{k}^{\prime}\right\} _{k\geq1}$ be another choice of $g$s and $\alpha$s satisfying the same. Then: \begin{align*} 0 & \overset{K}{=}f\left(\mathbf{z}\right)-f\left(\mathbf{z}\right)\\ & \overset{K}{=}\lim_{N\rightarrow\infty}\left(\sum_{k=1}^{N}\left(\alpha_{k}g_{k}\left(\mathbf{z}\right)-\alpha_{k}^{\prime}g_{k}^{\prime}\left(\mathbf{z}\right)\right)\right)\\ & \overset{K}{=}\sum_{\mathbf{n}\in\mathbb{N}_{0}^{r}}\lim_{N\rightarrow\infty}\left(\sum_{k=1}^{N}\left(\alpha_{k}c_{\mathbf{n}}\left(g_{k}\right)-\alpha_{k}^{\prime}c_{\mathbf{n}}\left(g_{k}^{\prime}\right)\right)\right)\left[\mathbf{z}\overset{p^{\lambda_{p}\left(\mathbf{n}\right)}}{\equiv}\mathbf{n}\right] \end{align*} Since $0$ is an elementary function, by \textbf{Proposition \ref{prop:vdP series of an elementary function},} it is uniquely represented by the van der Put series whose coefficients are all $0$. This forces: \begin{equation} \lim_{N\rightarrow\infty}\sum_{k=1}^{N}\alpha_{k}c_{\mathbf{n}}\left(g_{k}\right)\overset{K}{=}\lim_{N\rightarrow\infty}\sum_{k=1}^{N}\alpha_{k}^{\prime}c_{\mathbf{n}}\left(g_{k}^{\prime}\right) \end{equation} which shows that $c_{\mathbf{n}}\left(f\right)$ is, indeed, unique. Q.E.D. \begin{defn} \label{def:MD S_p}We write $F\left(\mathbb{Z}_{p}^{r},K\right)$\nomenclature{$F\left(\mathbb{Z}_{p}^{r},K\right)$}{$K$-valued functions on $\mathbb{Z}_{p}^{r}$}, to denote the space of all $K$-valued functions on $\mathbb{Z}_{p}^{r}$. We then define the linear operator $S_{p}:F\left(\mathbb{Z}_{p}^{r},K\right)\rightarrow\textrm{vdP}\left(\mathbb{Z}_{p}^{r},K\right)$ by: \begin{equation} S_{p}\left\{ f\right\} \left(\mathbf{z}\right)\overset{\textrm{def}}{=}\sum_{\mathbf{n}\in\mathbb{N}_{0}^{r}}c_{\mathbf{n}}\left(f\right)\left[\mathbf{z}\overset{p^{\lambda_{p}\left(\mathbf{n}\right)}}{\equiv}\mathbf{n}\right],\textrm{ }\forall f\in F\left(\mathbb{Z}_{p}^{r},K\right)\label{eq:MD Definition of S_p of f} \end{equation} We also define partial sum operators: $S_{p:N}:F\left(\mathbb{Z}_{p}^{r},K\right)\rightarrow C\left(\mathbb{Z}_{p}^{r},K\right)$ by: \begin{equation} S_{p:N}\left\{ f\right\} \left(\mathbf{z}\right)\overset{\textrm{def}}{=}\sum_{\mathbf{n}=\mathbf{0}}^{p^{N}-1}c_{\mathbf{n}}\left(f\right)\left[\mathbf{z}\overset{p^{\lambda_{p}\left(\mathbf{n}\right)}}{\equiv}\mathbf{n}\right],\textrm{ }\forall f\in F\left(\mathbb{Z}_{p}^{r},K\right)\label{eq:MD Definition of S_p N of f} \end{equation} Recall, the sum here is taken over all $\mathbf{n}\in\mathbb{N}_{0}^{r}$ so that, for each $m\in\left\{ 1,\ldots,r\right\} $, $0\leq n_{m}\leq p^{N}-1$. \end{defn} \begin{prop} We can extend $S_{p}$ from $F\left(\mathbb{Z}_{p}^{r},K\right)$ to $B\left(\mathbb{Z}_{p}^{r},K\right)$ by taking limits in $F\left(\mathbb{Z}_{p}^{r},K\right)$ with respect to $B\left(\mathbb{Z}_{p}^{r},K\right)$'s norm. We can then compute $c_{\mathbf{n}}\left(f\right)$ for any $f\in B\left(\mathbb{Z}_{p}^{r},K\right)$ by using the construction given in \textbf{\emph{Lemma \ref{lem:van der Put series for an arbitrary K-valued function on Z_P}}}, albeit with elementary functions in $B\left(\mathbb{Z}_{p}^{r},K\right)$, rather than in $C\left(\mathbb{Z}_{p}^{r},K\right)$. \end{prop} Proof: Essentially the same as the proof of \textbf{Lemma \ref{lem:van der Put series for an arbitrary K-valued function on Z_P}}. Q.E.D. \begin{lem}[\textbf{Multi-Dimensional van der Put Identities}] \label{lem:MD vdP identities}\ \vphantom{} I. For any $f:\mathbb{Z}_{p}^{r}\rightarrow K$ and any $N\in\mathbb{N}_{0}$: \begin{equation} S_{p:N}\left\{ f\right\} \left(\mathbf{z}\right)=\sum_{\mathbf{n}=\mathbf{0}}^{p^{N}-1}c_{\mathbf{n}}\left(f\right)\left[\mathbf{z}\overset{p^{\lambda_{p}\left(\mathbf{n}\right)}}{\equiv}\mathbf{n}\right]\overset{K}{=}f\left(\left[\mathbf{z}\right]_{p^{N}}\right),\textrm{ }\forall\mathbf{z}\in\mathbb{Z}_{p}^{r}\label{eq:MD truncated vdP identity} \end{equation} \vphantom{} II. For any $f\in B\left(\mathbb{Z}_{p}^{r},K\right)$: \begin{equation} S_{p}\left\{ f\right\} \left(\mathbf{z}\right)=\sum_{\mathbf{n}\in\mathbb{N}_{0}^{r}}c_{\mathbf{n}}\left(f\right)\left[\mathbf{z}\overset{p^{\lambda_{p}\left(\mathbf{n}\right)}}{\equiv}\mathbf{n}\right]\overset{K}{=}\lim_{k\rightarrow\infty}f\left(\left[\mathbf{z}\right]_{p^{k}}\right),\textrm{ }\forall\mathbf{z}\in\mathbb{Z}_{p}^{r}\label{eq:MD vdP identity} \end{equation} \end{lem} Proof: I. Let $f:\mathbb{Z}_{p}^{r}\rightarrow K$ be elementary, with $f=\prod_{m=1}^{r}f_{m}$. Then, by the one-dimensional truncated van der Put identity (\ref{eq:truncated van der Put identity}): \begin{align*} f\left(\left[\mathbf{z}\right]_{p^{N}}\right) & =\prod_{m=1}^{r}f_{m}\left(\left[\mathfrak{z}_{m}\right]_{p^{N}}\right)\\ & =\prod_{m=1}^{r}S_{p:N}\left\{ f_{m}\right\} \left(\mathfrak{z}_{m}\right)\\ \left(\textrm{Computation from \textbf{Prop. \ref{prop:vdP series of an elementary function}})}\right); & =S_{p:N}\left\{ \prod_{m=1}^{r}f_{m}\right\} \left(\mathfrak{z}_{1},\ldots,\mathfrak{z}_{r}\right)\\ & =S_{p:N}\left\{ f\right\} \left(\mathbf{z}\right) \end{align*} Using the linearity of $S_{p:N}$ then gives the desired result. \vphantom{} II. Take limits with (I). Q.E.D. \begin{thm}[\textbf{Multi-Dimensional van der Put Basis Theorem}] \label{thm:MD vdP basis theorem}$\mathcal{B}_{p}^{\otimes r}$ is a basis for $C\left(\mathbb{Z}_{p}^{r},K\right)$. In particular, for any $f:\mathbb{Z}_{p}^{r}\rightarrow K$, the following are equivalent: \vphantom{} I. $f$ is continuous. \vphantom{} II. $\lim_{\left\Vert \mathbf{n}\right\Vert _{\infty}\rightarrow\infty}\left|c_{\mathbf{n}}\left(f\right)\right|_{K}=0$. \vphantom{} III. $\lim_{N\rightarrow\infty}\left\Vert f-S_{p:N}\left\{ f\right\} \right\Vert _{p,K}=0$ \end{thm} Proof: Formally equivalent to its one-dimensional analogue\textemdash \textbf{Theorem \ref{thm:vdP basis theorem}}. Q.E.D. \begin{cor} If $K$ is non-archimedean, then the Banach space $C\left(\mathbb{Z}_{p}^{r},K\right)$ is isometrically isomorphic to $c_{0}\left(\mathbb{N}_{0}^{r},K\right)$ (the space of sequences $\left\{ c_{\mathbf{n}}\right\} _{\mathbf{n}\in\mathbb{N}_{0}^{r}}$ in $K$ that converge to $0$ in $K$ as $\left\Vert \mathbf{n}\right\Vert _{\infty}\rightarrow\infty$). \end{cor} Proof: Use the tensor product along with the isometric isomorphism $C\left(\mathbb{Z}_{p},K\right)\cong c_{0}\left(K\right)$ proven in \textbf{Theorem \ref{thm:C(Z_p,K) is iso to c_0 K}} (page \pageref{thm:C(Z_p,K) is iso to c_0 K}). Q.E.D. \begin{rem} Before we move on to the next subsection, note that everything we have done so far extends in the obvious way to vector\emph{-} or matrix-valued\emph{ }functions on $\mathbb{Z}_{p}^{r}$ by working component-wise. To that end, for $\mathbf{F}:\mathbb{Z}_{p}^{r}\rightarrow K^{\rho,c}$ given by $\mathbf{F}\left(\mathbf{z}\right)=\left\{ F_{j,k}\right\} _{j,k}$, \nomenclature{$c_{\mathbf{n}}\left(\mathbf{F}\right)$}{$\mathbf{n}$th van der Put coefficient of $\mathbf{F}$} we write: \begin{equation} c_{\mathbf{n}}\left(\mathbf{F}\right)\overset{\textrm{def}}{=}\left\{ c_{\mathbf{n}}\left(F_{j,k}\right)\right\} _{j,k} \end{equation} \begin{equation} S_{p:N}\left\{ \mathbf{F}\right\} \left(\mathbf{z}\right)\overset{\textrm{def}}{=}\left\{ S_{p:N}\left\{ F_{j,k}\right\} \left(\mathbf{z}\right)\right\} _{j,k} \end{equation} \end{rem} \subsection{\label{subsec:5.3.2. Interpolation-Revisited}Interpolation Revisited} We begin with rising sequences and rising-continuous functions. \begin{defn} A sequence $\left\{ \mathbf{z}_{n}\right\} _{n\geq0}$ in $\mathbb{Z}_{p}^{r}$ (where $\mathbf{z}_{n}=\left(\mathfrak{z}_{n,1},\ldots,\mathfrak{z}_{n,r}\right)$) is said to be \textbf{($p$-adically) rising }if there exists an $\ell\in\left\{ 1,\ldots,r\right\} $ so that the number of non-zero $p$-adic digits in $\mathfrak{z}_{n,\ell}$ tends to $\infty$ as $n\rightarrow\infty$. \end{defn} \begin{defn} \label{def:MD rising-continuity}A function $\chi:\mathbb{Z}_{p}^{r}\rightarrow K^{d}$ is said to be \textbf{($\left(p,K\right)$-adically)} \textbf{rising-continuous}\index{rising-continuous!left(p,Kright)-adically@$\left(p,K\right)$-adically}\textbf{ }\index{rising-continuous!function}whenever: \begin{equation} \lim_{n\rightarrow\infty}\chi\left(\left[\mathbf{z}\right]_{p^{n}}\right)\overset{K^{d}}{=}\chi\left(\mathbf{z}\right),\textrm{ }\forall\mathbf{z}\in\mathbb{Z}_{p}^{r}\label{eq:MD Definition of a rising-continuous function} \end{equation} where the convergence is point-wise. We write \nomenclature{$\tilde{C}\left(\mathbb{Z}_{p}^{r},K^{d}\right)$}{set of $K^{d}$-valued rising-continuous functions on $\mathbb{Z}_{p}^{r}$ }$\tilde{C}\left(\mathbb{Z}_{p}^{r},K^{d}\right)$ to denote the $K$-linear space of all rising-continuous functions $\mathbb{Z}_{p}^{r}\rightarrow K^{d}$. \end{defn} \begin{prop} \label{prop:MD vdP criterion for rising continuity}Let $\chi\in B\left(\mathbb{Z}_{p}^{r},\mathbb{C}_{q}^{d}\right)$ be any function. Then, $S_{p}\left\{ \chi\right\} $ (the van der Put series\index{van der Put!series} of $\chi$) converges at $\mathbf{z}\in\mathbb{Z}_{p}^{r}$ if and only if: \begin{equation} \lim_{k\rightarrow\infty}c_{\left[\mathbf{z}\right]_{p^{k}}}\left(\chi\right)\left[\lambda_{p}\left(\left[\mathbf{z}\right]_{p^{k}}\right)=k\right]\overset{\mathbb{C}_{q}^{d}}{=}\mathbf{0}\label{eq:MD vdP criterion for rising-continuity} \end{equation} where $c_{\left[\mathbf{z}\right]_{p^{k}}}\left(\chi\right)$ is the $\left[\mathbf{z}\right]_{p^{k}}=\left(\left[\mathfrak{z}_{1}\right]_{p^{k}},\ldots,\left[\mathfrak{z}_{r}\right]_{p^{k}}\right)$ th van der Put coefficient of $\chi$. \end{prop} Proof: We start by writing: \begin{align*} S_{p}\left\{ \chi\right\} \left(\mathbf{z}\right) & \overset{\mathbb{C}_{q}^{d}}{=}\sum_{\mathbf{n}\in\mathbb{N}_{0}^{r}}c_{\mathbf{n}}\left(\chi\right)\left[\mathbf{z}\overset{p^{\lambda_{p}\left(\mathbf{n}\right)}}{\equiv}\mathbf{n}\right]\\ & =c_{\mathbf{0}}\left(\chi\right)+\sum_{k=1}^{\infty}\sum_{\mathbf{n}:\lambda_{p}\left(\mathbf{n}\right)=k}c_{\mathbf{n}}\left(\chi\right)\left[\mathbf{z}\overset{p^{k}}{\equiv}\mathbf{n}\right] \end{align*} For each $\mathbf{n}$, observe that if $\lambda_{p}\left(\left[\mathbf{z}\right]_{p^{k}}\right)=k$, then: \[ \mathbf{n}\overset{\textrm{def}}{=}\left[\mathbf{z}\right]_{p^{k}}=\left(\left[\mathfrak{z}_{1}\right]_{p^{k}},\ldots,\left[\mathfrak{z}_{r}\right]_{p^{k}}\right) \] is the unique element of $\mathbb{N}_{0}^{r}$ satisfying both $\lambda_{p}\left(\mathbf{n}\right)=k$ and $\mathbf{z}\overset{p^{k}}{\equiv}\mathbf{n}$. Hence: \begin{align} S_{p}\left\{ \chi\right\} \left(\mathbf{z}\right) & \overset{\mathbb{C}_{q}^{d}}{=}c_{\mathbf{0}}\left(\chi\right)+\sum_{k=1}^{\infty}c_{\left[\mathbf{z}\right]_{p^{k}}}\left(\chi\right)\left[\lambda_{p}\left(\left[\mathbf{z}\right]_{p^{k}}\right)=k\right] \end{align} The ultrametric properties of $\mathbb{C}_{q}^{d}$ tell us that the $q$-adic convergence of this series at any given $\mathbf{z}\in\mathbb{Z}_{p}^{r}$ is equivalent to: \begin{equation} \lim_{k\rightarrow\infty}c_{\left[\mathbf{z}\right]_{p^{k}}}\left(\chi\right)\left[\lambda_{p}\left(\left[\mathbf{z}\right]_{p^{k}}\right)=k\right]\overset{\mathbb{C}_{q}^{d}}{=}\mathbf{0} \end{equation} Q.E.D. \begin{thm} The operator $S_{p}$ which sends a function to its formal van der Put series is a isomorphism of $K$-linear spaces. This isomorphism map $\tilde{C}\left(\mathbb{Z}_{p}^{r},K^{d}\right)$ onto the subspace of $\textrm{vdP}\left(\mathbb{Z}_{p}^{r},K^{d}\right)$ consisting of all van der Put series which converge at every $\mathbf{z}\in\mathbb{Z}_{p}^{r}$. Additionally, for every $\chi\in\tilde{C}\left(\mathbb{Z}_{p}^{r},K^{d}\right)$: \vphantom{} I. $\chi=S_{p}\left\{ \chi\right\} $; \vphantom{} II. $\chi$ is uniquely represented by its van der Put series: \begin{equation} \chi\left(\mathbf{z}\right)\overset{K^{d}}{=}\sum_{\mathbf{n}\in\mathbb{N}_{0}^{r}}c_{\mathbf{n}}\left(\chi\right)\left[\mathbf{z}\overset{p^{\lambda_{p}\left(\mathbf{n}\right)}}{\equiv}\mathbf{n}\right],\textrm{ }\forall\mathbf{z}\in\mathbb{Z}_{p}^{r}\label{eq:MD Chi vdP series} \end{equation} where the convergence is point-wise. \end{thm} Proof: Let $\chi\in\tilde{C}\left(\mathbb{Z}_{p}^{r},K^{d}\right)$ be arbitrary. By the truncated van der Put identity (\ref{eq:MD truncated vdP identity}), $\chi\left(\left[\mathbf{z}\right]_{p^{N}}\right)=S_{p:N}\left\{ \chi\right\} \left(\mathbf{z}\right)$. Here, the rising-continuity of $\chi$ guarantees the point-wise convergence of $S_{p:N}\left\{ \chi\right\} \left(\mathbf{z}\right)$ in $K^{d}$ to $\chi\left(\mathbf{z}\right)$ as $N\rightarrow\infty$. By \textbf{Proposition \ref{prop:MD vdP criterion for rising continuity}}, this implies the van der Put coefficients of $\chi$ satisfy (\ref{eq:MD vdP criterion for rising-continuity}) for all $\mathbf{z}\in\mathbb{Z}_{p}^{r}$. Consequently, the van der Put series $S_{p}\left\{ \chi\right\} \left(\mathbf{z}\right)$ converges at every $\mathbf{z}\in\mathbb{Z}_{p}^{r}$, where it is equal to $\chi\left(\mathbf{z}\right)$. This proves (I). As for (II), the uniqueness specified therein is equivalent to demonstrating that $S_{p}$ is an isomorphism in the manner described above. We do this below: \begin{itemize} \item (Surjectivity) Let $V\left(\mathbf{z}\right)\in\textrm{vdP}\left(\mathbb{Z}_{p}^{r},K^{d}\right)$ be any formal van der Put series which converges $q$-adically at every $\mathbf{z}\in\mathbb{Z}_{p}^{r}$. Letting: \begin{equation} \chi\left(\mathbf{z}\right)\overset{\textrm{def}}{=}\lim_{N\rightarrow\infty}S_{p:N}\left\{ V\right\} \left(\mathbf{z}\right) \end{equation} we have $V\left(\mathbf{m}\right)=\chi\left(\mathbf{m}\right)$ for all $\mathbf{m}\in\mathbb{N}_{0}^{r}$, and hence, $V\left(\mathbf{z}\right)=S_{p}\left\{ \chi\right\} $. Thus, $S_{p:N}\left\{ \chi\right\} \left(\mathbf{z}\right)=\chi\left(\left[\mathbf{z}\right]_{p^{N}}\right)$. Since $\chi\left(\mathbf{z}\right)$ is defined by $\lim_{N\rightarrow\infty}S_{p:N}\left\{ V\right\} \left(\mathbf{z}\right)$, this gives: \begin{equation} \chi\left(\mathbf{z}\right)=\lim_{N\rightarrow\infty}S_{p:N}\left\{ V\right\} \left(\mathbf{z}\right)=\chi\left(\left[\mathbf{z}\right]_{p^{N}}\right) \end{equation} which establishes the rising-continuity of $\chi$. This proves $V=S_{p}\left\{ \chi\right\} $, and thus, that $S_{p}$ is surjective. \item (Injectivity) Let $\chi_{1},\chi_{2}\in\tilde{C}\left(\mathbb{Z}_{p}^{r},K^{d}\right)$ and suppose $S_{p}\left\{ \chi_{1}\right\} =S_{p}\left\{ \chi_{2}\right\} $. Then, by (I): \[ \chi_{1}\left(\mathbf{z}\right)\overset{\textrm{(I)}}{=}S_{p}\left\{ \chi_{1}\right\} \left(\mathbf{z}\right)=S_{p}\left\{ \chi_{2}\right\} \left(\mathbf{z}\right)\overset{\textrm{(I)}}{=}\chi_{2}\left(\mathbf{z}\right),\textrm{ }\forall\mathbf{z}\in\mathbb{Z}_{p}^{r} \] This proves $\chi_{1}=\chi_{2}$, which establishes $S_{p}$'s injectivity. \end{itemize} Thus, $S_{p}$ is an isomorphism. Q.E.D. \begin{defn} Let $\mathbb{F}$ be $\mathbb{Q}$ or a field extension thereof, and let $\chi:\mathbb{N}_{0}^{r}\rightarrow\mathbb{F}^{d}$ be a function. We say $\chi$ has (or ``admits'') a \textbf{$\left(p,q\right)$-adic rising-continuation} \textbf{(to $K^{d}$)} whenever\index{rising-continuation} there is a metrically complete $q$-adic field extension $K$ of $\mathbb{F}$ and a rising-continuous function $\chi^{\prime}:\mathbb{Z}_{p}^{r}\rightarrow K^{d}$ so that $\chi^{\prime}\left(\mathbf{n}\right)=\chi\left(\mathbf{n}\right)$ for all $\mathbf{n}\in\mathbb{N}_{0}^{r}$. We call any $\chi^{\prime}$ satisfying this property a \textbf{($\left(p,q\right)$-adic)}\emph{ }\textbf{rising-continuation }of $\chi$ (to $K^{d}$). \end{defn} \begin{prop} \label{prop:Uniqueness of rising-continuation, MD}Let $\chi:\mathbb{N}_{0}^{r}\rightarrow\mathbb{F}^{d}$ be a function admitting a $\left(p,q\right)$-adic rising-continuation to $K^{d}$. Then: \vphantom{} I. The rising-continuation of $\chi$ is unique. Consequently, we write $\chi^{\prime}$ to denote the rising continuation of $\chi$. \vphantom{} II. \begin{equation} \chi^{\prime}\left(\mathbf{z}\right)\overset{K^{d}}{=}\lim_{k\rightarrow\infty}\chi\left(\left[\mathbf{z}\right]_{p^{k}}\right)=\sum_{\mathbf{n}\in\mathbb{N}_{0}^{r}}c_{\mathbf{n}}\left(\chi\right)\left[\mathbf{z}\overset{p^{\lambda_{p}\left(\mathbf{n}\right)}}{\equiv}\mathbf{n}\right],\textrm{ }\forall\mathbf{z}\in\mathbb{Z}_{p}^{r}\label{eq:MD Rising continuation limit formula} \end{equation} \end{prop} Proof: I. Suppose $\chi$ admits two (possibly distinct) rising-continuations, $\chi^{\prime}$ and $\chi^{\prime\prime}$. To see that $\chi^{\prime}$ and $\chi^{\prime\prime}$ must be the same, we note that since the restrictions of both $\chi^{\prime}$ and $\chi^{\prime\prime}$ to $\mathbb{N}_{0}^{r}$ are, by definition, equal to $\chi$, these restrictions must be equal to one another. So, let $\mathbf{z}$ be an arbitrary element of $\left(\mathbb{Z}_{p}^{r}\right)^{\prime}$. Then, we note that at least one entry of $\mathbf{z}$ (say, the $\ell$th entry) necessarily has infinitely many non-zero $p$-adic digits. Consequently, $\left\{ \left[\mathbf{z}\right]_{p^{k}}\right\} _{k\geq1}$ is a rising sequence of non-negative integer $r$-tuples converging to $\mathbf{z}$. As such, by the rising-continuity of $\chi^{\prime}$ and $\chi^{\prime\prime}$: \begin{equation} \lim_{k\rightarrow\infty}\chi^{\prime}\left(\left[\mathbf{z}\right]_{p^{k}}\right)\overset{K^{d}}{=}\chi^{\prime}\left(\mathbf{z}\right) \end{equation} \begin{equation} \lim_{k\rightarrow\infty}\chi^{\prime\prime}\left(\left[\mathbf{z}\right]_{p^{k}}\right)\overset{K^{d}}{=}\chi^{\prime\prime}\left(\mathbf{z}\right) \end{equation} Because the $\left[\mathbf{z}\right]_{p^{k}}$s are $r$-tuples of integers, we can then write: \begin{equation} \chi^{\prime}\left(\mathbf{z}\right)=\lim_{k\rightarrow\infty}\chi^{\prime}\left(\left[\mathbf{z}\right]_{p^{k}}\right)=\lim_{k\rightarrow\infty}\chi\left(\left[\mathbf{z}\right]_{p^{k}}\right)=\lim_{k\rightarrow\infty}\chi^{\prime\prime}\left(\left[\mathbf{z}\right]_{p^{k}}\right)=\chi^{\prime\prime}\left(\mathbf{z}\right) \end{equation} Since $\mathbf{z}$ was arbitrary, we conclude $\chi^{\prime}\left(\mathbf{z}\right)=\chi^{\prime\prime}\left(\mathbf{z}\right)$ for all $\mathbf{z}\in\left(\mathbb{Z}_{p}^{r}\right)^{\prime}$. Thus, $\chi^{\prime}$ and $\chi^{\prime\prime}$ are equal to one another on both $\left(\mathbb{Z}_{p}^{r}\right)^{\prime}$ and $\mathbb{N}_{0}^{r}$, which is all of $\mathbb{Z}_{p}^{r}$. So, $\chi^{\prime}$ and $\chi^{\prime\prime}$ are, in fact, the same function. This proves the uniqueness of $\chi$'s rising-continuation. \vphantom{} II. As a rising-continuous function, $\chi^{\prime}\left(\mathbf{z}\right)$ is uniquely determined by its values on $\mathbb{N}_{0}^{r}$. Since $\chi^{\prime}\mid_{\mathbb{N}_{0}^{r}}=\chi$, $\chi^{\prime}$ and $\chi$ then have the same van der Put coefficients. Applying the van der Put identity (\ref{eq:van der Put identity}) then yields (\ref{eq:MD Rising continuation limit formula}). Q.E.D. \begin{thm} Let $\mathbb{F}$ be $\mathbb{Q}$ or a field extension thereof, let $\chi:\mathbb{N}_{0}^{r}\rightarrow\mathbb{F}^{d}$ be a function, and let $K$ be a metrically complete $q$-adic field extension of $\mathbb{F}$, where $q$ is prime. Then, the following are equivalent: \vphantom{} I. $\chi$ admits a $\left(p,q\right)$-adic rising-continuation to $K^{d}$. \vphantom{} II. For each $\mathbf{z}\in\mathbb{Z}_{p}^{r}$, $\chi\left(\left[\mathbf{z}\right]_{p^{n}}\right)$ converges to a limit in $K^{d}$ as $n\rightarrow\infty$. \end{thm} Proof: i. Suppose (I) holds. Then, by \textbf{Proposition \ref{prop:Uniqueness of rising-continuation, MD}} we have that (\ref{eq:MD Rising continuation limit formula}) holds, which shows that (II) is true. \vphantom{} ii. Conversely, suppose (II) holds. Then, by the van der Put identity (\ref{eq:van der Put identity}), $S_{p}\left\{ \chi\right\} $ is a rising-continuous function whose restriction to $\mathbb{N}_{0}^{r}$ is equal to $\chi$. So, $S_{p}\left\{ \chi\right\} $ is the rising-continuation of $\chi$, and hence, $\chi$ is rising-continuable. This shows the equivalence of (II) and (I). Q.E.D. \begin{rem} Like in the one-dimensional case, we will now identify a function $\chi:\mathbb{N}_{0}^{r}\rightarrow K^{d}$ with its rising-continuation $\chi^{\prime}$. \end{rem} \vphantom{} Finally, we have the analogues of the functional equation results from the end of Subsection \ref{subsec:3.2.1 -adic-Interpolation-of}. \begin{thm} \label{thm:generic Chi_type functional equations, MD}Let $H$ be a semi-basic $p$-smooth $d$-dimensional depth-$r$ Hydra map which fixes $\mathbf{0}$, and consider the system of functional equations\index{functional equation!rising-continuability}: \begin{equation} \mathbf{f}\left(p\mathbf{n}+\mathbf{j}\right)=H_{\mathbf{j}}^{\prime}\left(\mathbf{0}\right)\mathbf{f}\left(\mathbf{n}\right)+\mathbf{c}_{\mathbf{j}},\textrm{ }\forall\mathbf{j}\in\mathbb{Z}^{r}/p\mathbb{Z}^{r},\textrm{ }\forall\mathbf{n}\in\mathbb{N}_{0}^{r}\label{eq:MD Generic H-type functional equations} \end{equation} where $\left\{ \mathbf{c}_{\mathbf{j}}\right\} _{\mathbf{j}\in\mathbb{Z}^{r}/p\mathbb{Z}^{r}}$ are vector constants in $\overline{\mathbb{Q}}^{d}$. Additionally, suppose $\mathbf{I}_{d}-H^{\prime}\left(\mathbf{0}\right)$ is invertible. Then: \vphantom{} I. There is a unique function $\chi:\mathbb{N}_{0}^{r}\rightarrow\overline{\mathbb{Q}}^{d}$ such that $\mathbf{f}=\chi$ is a solution of \emph{(\ref{eq:MD Generic H-type functional equations})}. \vphantom{} II. The solution $\chi$ \emph{(\ref{eq:MD Generic H-type functional equations})} is rising-continuable to a function $\chi:\mathbb{Z}_{p}^{r}\rightarrow\mathbb{C}_{q_{H}}^{d}$ which satisfies: \begin{equation} \chi\left(p\mathbf{z}+\mathbf{j}\right)=H_{\mathbf{j}}^{\prime}\left(\mathbf{0}\right)\chi\left(\mathbf{n}\right)+\mathbf{c}_{\mathbf{j}},\textrm{ }\forall\mathbf{j}\in\mathbb{Z}^{r}/p\mathbb{Z}^{r},\textrm{ }\forall\mathbf{z}\in\mathbb{Z}_{p}^{r}\label{eq:MD Rising-continuation Generic H-type functional equations} \end{equation} \vphantom{} III. The function $\chi:\mathbb{Z}_{p}^{r}\rightarrow\mathbb{C}_{q_{H}}^{d}$ described in \emph{(III)} is the unique rising-continuous function $\mathbf{f}:\mathbb{Z}_{p}^{r}\rightarrow\mathbb{C}_{q_{H}}^{d}$ satisfying: \begin{equation} \mathbf{f}\left(p\mathbf{z}+\mathbf{j}\right)=H_{\mathbf{j}}^{\prime}\left(\mathbf{0}\right)\mathbf{f}\left(\mathbf{n}\right)+\mathbf{c}_{\mathbf{j}},\textrm{ }\forall\mathbf{j}\in\mathbb{Z}^{r}/p\mathbb{Z}^{r},\textrm{ }\forall\mathbf{z}\in\mathbb{Z}_{p}^{r} \end{equation} \end{thm} Proof: I. Let $\mathbf{f}:\mathbb{N}_{0}^{r}\rightarrow\overline{\mathbb{Q}}^{d}$ be any solution of (\ref{eq:MD Generic H-type functional equations}). Setting $\mathbf{n}=\mathbf{j}=\mathbf{0}$ yields: \begin{align*} \mathbf{f}\left(\mathbf{0}\right) & =H^{\prime}\left(\mathbf{0}\right)\mathbf{f}\left(\mathbf{0}\right)+\mathbf{c}_{\mathbf{0}}\\ & \Updownarrow\\ \mathbf{f}\left(\mathbf{0}\right) & =\left(\mathbf{I}_{d}-H^{\prime}\left(\mathbf{0}\right)\right)^{-1}\mathbf{c}_{\mathbf{0}} \end{align*} which is well-defined since $\mathbf{I}_{d}-H^{\prime}\left(\mathbf{0}\right)$ was given to be invertible. Then, we have that: \begin{equation} \mathbf{f}\left(\mathbf{j}\right)=H_{\mathbf{j}}^{\prime}\left(\mathbf{0}\right)\mathbf{f}\left(\mathbf{0}\right)+\mathbf{c}_{\mathbf{j}},\textrm{ }\forall\mathbf{j}\in\mathbb{Z}^{r}/p\mathbb{Z}^{r} \end{equation} and, more generally: \begin{equation} \mathbf{f}\left(\mathbf{m}\right)=H_{\left[\mathbf{m}\right]_{p}}^{\prime}\left(\mathbf{0}\right)\mathbf{f}\left(\frac{\mathbf{m}-\left[\mathbf{m}\right]_{p}}{p}\right)+\mathbf{c}_{\left[\mathbf{m}\right]_{p}},\textrm{ }\forall\mathbf{m}\in\mathbb{N}_{0}^{r}\label{eq:MD f,m, digit shifting} \end{equation} Next, because the map $\mathbf{m}\mapsto\frac{\mathbf{m}-\left[\mathbf{m}\right]_{p}}{p}$ sends the non-negative integer tuple: \begin{equation} \mathbf{m}=\left(\sum_{k=0}^{\lambda_{p}\left(m_{1}\right)-1}m_{1,k}p^{k},\ldots,\sum_{k=0}^{\lambda_{p}\left(m_{r}\right)-1}m_{r,k}p^{k}\right) \end{equation} to the integer tuple: \begin{equation} \mathbf{m}=\left(\sum_{k=0}^{\lambda_{p}\left(m_{1}\right)-2}m_{1,k}p^{k},\ldots,\sum_{k=0}^{\lambda_{p}\left(m_{r}\right)-2}m_{r,k}p^{k}\right) \end{equation} it follows that $\mathbf{m}\mapsto\frac{\mathbf{m}-\left[\mathbf{m}\right]_{p}}{p}$ eventually iterates every $\mathbf{m}\in\mathbb{N}_{0}^{r}$ to $\mathbf{0}$. So, (\ref{eq:MD f,m, digit shifting}) implies that, for every $\mathbf{m}\in\mathbb{N}_{0}^{r}$, $\mathbf{f}\left(\mathbf{m}\right)$ is entirely determined by $\mathbf{f}\left(\mathbf{0}\right)$ and the $\mathbf{c}_{\mathbf{j}}$s. Since $\mathbf{f}\left(\mathbf{0}\right)$ is uniquely determined by $\mathbf{c}_{\mathbf{0}}$ and $H$, (\ref{eq:MD Generic H-type functional equations}) then possesses exactly one solution, which we shall denote by $\chi$. \vphantom{} II. Rehashing the argument for existence of the numen $\chi_{H}$ when $H$ is semi-basic and fixes $\mathbf{0}$, because $H$ is here semi-basic, any $\mathbf{z}\in\left(\mathbb{Z}_{p}^{r}\right)^{\prime}$ will have at least one entry infinitely many non-zero $p$-adic digits, and hence, the product of $H_{\mathbf{j}}^{\prime}\left(\mathbf{0}\right)$ taken over all the digits of such a $\mathbf{z}$ will converge $q_{H}$-adically to zero. Then, using (\ref{eq:MD Generic H-type functional equations}), we see that $\chi\left(\mathbf{z}\right)$ will be a sum of the form: \begin{equation} \beta_{0}+\alpha_{1}\beta_{1}+\alpha_{1}\alpha_{2}\beta_{2}+\alpha_{1}\alpha_{2}\alpha_{3}\beta_{3}+\cdots \end{equation} where, for each $n$, $\beta_{n}$ is one of the $\mathbf{c}_{\mathbf{j}}$s and $\alpha_{n}$ is $H_{\mathbf{j}_{n}}^{\prime}\left(\mathbf{0}\right)$, where $\mathbf{j}_{n}$ is the $r$-tuple of the $n$th $p$-adic digit of $\mathfrak{z}_{1}$ through the $n$th $p$-adic digit of $\mathfrak{z}_{r}$, where $\mathbf{z}=\left(\mathfrak{z}_{1},\ldots,\mathfrak{z}_{r}\right)$. Consequently, this series converges in $\mathbb{C}_{q_{H}}^{d}$ for all $\mathbf{z}\in\left(\mathbb{Z}_{p}^{r}\right)^{\prime}$, seeing as $\left\Vert H_{\mathbf{j}}^{\prime}\left(\mathbf{0}\right)\right\Vert _{q_{H}}<1$ for all $\mathbf{j}\in\left(\mathbb{Z}^{r}/p\mathbb{Z}^{r}\right)\backslash\left\{ \mathbf{0}\right\} $ because $H$ is semi-basic. This guarantees the rising-continuability of $\chi$. \vphantom{} III. Because $\chi$ admits a rising continuation, this continuation is given at every $\mathbf{z}\in\mathbb{Z}_{p}^{r}$ by the van der Put series $S_{p}\left\{ \chi\right\} \left(\mathbf{z}\right)$ . As such: \begin{equation} \chi\left(\mathbf{z}\right)\overset{\mathbb{C}_{q_{H}}^{d}}{=}S_{p}\left\{ \chi\right\} \left(\mathbf{z}\right)\overset{\mathbb{C}_{q_{H}}^{d}}{=}\lim_{N\rightarrow\infty}\chi\left(\left[\mathbf{z}\right]_{p^{N}}\right) \end{equation} Because $\chi$ satisfies (\ref{eq:MD Generic H-type functional equations}), we can write: \begin{equation} \chi\left(p\left[\mathbf{z}\right]_{p^{N}}+\mathbf{j}\right)\overset{\overline{\mathbb{Q}}^{d}}{=}H_{\mathbf{j}}^{\prime}\left(\mathbf{0}\right)\chi\left(\left[\mathbf{z}\right]_{p^{N}}\right)+\mathbf{c}_{\mathbf{j}} \end{equation} for all $\mathbf{z}\in\mathbb{Z}_{p}^{r}$, all $\mathbf{j}\in\mathbb{Z}^{r}/p\mathbb{Z}^{r}$, and all $N\geq0$. So, for $\mathbf{z}\in\left(\mathbb{Z}_{p}^{r}\right)^{\prime}$, letting $N\rightarrow\infty$ yields: \begin{equation} \chi\left(p\mathbf{z}+\mathbf{j}\right)\overset{\mathbb{C}_{q_{H}}^{d}}{=}H_{\mathbf{j}}^{\prime}\left(\mathbf{0}\right)\chi\left(\mathbf{z}\right)+\mathbf{c}_{\mathbf{j}},\textrm{ }\forall\mathbf{j}\in\mathbb{Z}^{r}/p\mathbb{Z}^{r},\textrm{ }\forall\mathbf{z}\in\left(\mathbb{Z}_{p}^{r}\right)^{\prime} \end{equation} Note that these identities hold automatically for $\mathbf{z}\in\mathbb{N}_{0}^{r}$, because those are the cases governed by (\ref{eq:MD Generic H-type functional equations}). The uniqueness of $\chi$ as a solution of (\ref{eq:MD Rising-continuation Generic H-type functional equations}) occurs because any $\mathbf{f}:\mathbb{Z}_{p}^{r}\rightarrow\mathbb{C}_{q_{H}}^{d}$ satisfying (\ref{eq:MD Rising-continuation Generic H-type functional equations}) has a restriction to $\mathbb{N}_{0}^{r}$ which satisfies (\ref{eq:MD Generic H-type functional equations}), thereby forcing $\mathbf{f}=\chi$. Q.E.D. \vphantom{}A slightly more general version of this type of argument is as follows: \begin{lem} Fix an integer $q\geq2$, let $K$ be a metrically complete $q$-adic field, and let $\Phi_{\mathbf{j}}:\mathbb{Z}_{p}^{r}\times K^{d}\rightarrow K^{d}$ be continuous for $\mathbf{j}\in\mathbb{Z}^{r}/p\mathbb{Z}^{r}$. Suppose $\chi:\mathbb{N}_{0}^{r}\rightarrow K^{d}$ is $\left(p,K\right)$-adically rising-continuable. If $\chi$ satisfies the functional equations: \begin{equation} \chi\left(p\mathbf{n}+\mathbf{j}\right)=\Phi_{\mathbf{j}}\left(\mathbf{n},\chi\left(\mathbf{n}\right)\right),\textrm{ }\forall\mathbf{n}\in\mathbb{N}_{0}^{r},\textrm{ }\forall\mathbf{j}\in\mathbb{Z}^{r}/p\mathbb{Z}^{r} \end{equation} then the rising-continuation of $\chi$ satisfies: \begin{equation} \chi\left(p\mathbf{z}+\mathbf{j}\right)=\Phi_{\mathbf{j}}\left(\mathbf{z},\chi\left(\mathbf{z}\right)\right),\textrm{ }\forall\mathbf{z}\in\mathbb{Z}_{p},\textrm{ }\forall\mathbf{j}\in\mathbb{Z}^{r}/p\mathbb{Z}^{r} \end{equation} \end{lem} Proof: Let everything be as given. Since: \begin{equation} \chi\left(p\left[\mathbf{z}\right]_{p^{N}}+\mathbf{j}\right)=\Phi_{\mathbf{j}}\left(\left[\mathbf{z}\right]_{p^{N}},\chi\left(\left[\mathbf{z}\right]_{p^{N}}\right)\right) \end{equation} holds true for all $\mathbf{z}\in\mathbb{Z}_{p}$ and all $N\geq0$, the rising-continuability of $\chi$ and the continuity of $\Phi_{\mathbf{j}}$ guarantee that: \begin{align*} \chi\left(p\mathbf{z}+\mathbf{j}\right) & \overset{K^{d}}{=}\lim_{N\rightarrow\infty}\chi\left(p\left[\mathbf{z}\right]_{p^{N}}+\mathbf{j}\right)\\ & \overset{K^{d}}{=}\lim_{N\rightarrow\infty}\Phi_{\mathbf{j}}\left(\left[\mathbf{z}\right]_{p^{N}},\chi\left(\left[\mathbf{z}\right]_{p^{N}}\right)\right)\\ & \overset{K^{d}}{=}\Phi_{\mathbf{j}}\left(\mathbf{z},\chi\left(\mathbf{z}\right)\right) \end{align*} as desired. Q.E.D. \vphantom{} \begin{rem} Like in Subsection \ref{subsec:5.3.2. Interpolation-Revisited}, all of the above holds when functions taking values in $\mathbb{C}_{q}^{d}$ (vectors) are replaced with functions taking values in $\mathbb{C}_{q}^{\rho,c}$ (matrices). \end{rem} \newpage{} \section{\label{sec:5.4. Quasi-Integrability-in-Multiple}Quasi-Integrability in Multiple Dimensions} I see little reason to cover the multi-dimensional analogue of the results on the Banach algebra of rising-continuous functions, seeing as we will not use them in our analysis of the multi-dimensional $\chi_{H}$. As such, we will instead proceed directly to the multi-dimensional $\left(p,q\right)$-adic Fourier transform, and from there to frames and quasi-integrability. \subsection{Multi-Dimensional $\left(p,q\right)$-adic Fourier Analysis and Thick Measures\label{subsec:5.4.1 Multi-Dimensional--adic-Fourier}} With regard to multi-dimensional Fourier analysis, I will be less pedantic than the reader has probably come to expect me to be. So long as the reader understands how to do $\left(p,q\right)$-adic Fourier analysis in one dimension, the formalism for Fourier analysis of scalar or vector valued functions over $\mathbb{R}^{n}$ will provide enough information for the reader to do multi-dimensional $\left(p,q\right)$-adic Fourier analysis\index{multi-dimensional!Fourier transform}, with the help of the notation provided below, of course. \begin{notation}[\textbf{Formalism for Multi-Dimensional $\left(p,q\right)$-adic Fourier Analysis}] \label{nota:third batch}\ \vphantom{} I. Given $\mathbf{x}\in\mathbb{Q}_{p}^{r}$, where $\mathbf{x}=\left(\mathfrak{x}_{1},\ldots,\mathfrak{x}_{r}\right)$, we write: \begin{equation} \left\{ \mathbf{x}\right\} _{p}\overset{\textrm{def}}{=}\sum_{m=1}^{r}\left\{ \mathfrak{x}_{m}\right\} _{p}\label{eq:Definition of P-adic fractional part (MD)} \end{equation} Consequently, for $\mathbf{t}\in\hat{\mathbb{Z}}_{p}^{r}$ and $\mathbf{z}\in\mathbb{Z}_{p}^{r}$: \begin{equation} \left\{ \mathbf{t}\cdot\mathbf{z}\right\} _{p}=\left\{ \mathbf{t}\mathbf{z}\right\} _{p}=\sum_{m=1}^{r}\left\{ t_{m}\mathfrak{z}_{m}\right\} _{p}\label{eq:P-adic fractional part of bold t times bold z (MD)} \end{equation} \vphantom{} II. Let $f\in C\left(\mathbb{Z}_{p}^{r},\mathbb{C}_{q}\right)$ be elementary, with $f\left(\mathbf{z}\right)=\prod_{m=1}^{r}f_{m}\left(\mathfrak{z}_{m}\right)$. Then, the Fourier transform of $f$ is defined by: \begin{equation} \hat{f}\left(\mathbf{t}\right)\overset{\textrm{def}}{=}\int_{\mathbb{Z}_{p}^{r}}f\left(\mathbf{z}\right)e^{-2\pi i\left\{ \mathbf{t}\mathbf{z}\right\} _{p}}d\mathbf{z}=\prod_{m=1}^{r}\int_{\mathbb{Z}_{p}^{r}}f\left(\mathfrak{z}_{m}\right)e^{-2\pi i\left\{ t_{m}\mathfrak{z}_{m}\right\} _{p}}d\mathfrak{z}_{m}\label{eq:Definition of Fourier transform of an elementary scalar function on Z_P} \end{equation} We then define $\hat{f}\left(\mathbf{t}\right)$ for any $C\left(\mathbb{Z}_{p}^{r},\mathbb{C}_{q}\right)$ by extending via linearity. \vphantom{} III. Let $\mathbf{f}\in C\left(\mathbb{Z}_{p},\mathbb{C}_{q}^{d}\right)$, with $\mathbf{f}\left(\mathfrak{z}\right)=\left(f_{1}\left(\mathfrak{z}\right),\ldots,f_{d}\left(\mathfrak{z}\right)\right)$ for $f_{1},\ldots,f_{d}\in C\left(\mathbb{Z}_{p},\mathbb{C}_{q}\right)$. Then, we define $\hat{\mathbf{f}}\left(t\right)$ as the $d$-tuple of the Fourier transforms of the $f_{m}$s: \begin{equation} \hat{\mathbf{f}}\left(t\right)\overset{\textrm{def}}{=}\left(\int_{\mathbb{Z}_{p}}f_{1}\left(\mathfrak{z}\right)e^{-2\pi i\left\{ t\mathfrak{z}\right\} _{p}}d\mathfrak{z},\ldots,\int_{\mathbb{Z}_{p}}f_{d}\left(\mathfrak{z}\right)e^{-2\pi i\left\{ t\mathfrak{z}\right\} _{p}}d\mathfrak{z}\right)\label{eq:Definition of the Fourier transform of a vector-valued function on Z_p} \end{equation} \vphantom{} IV. Let $\mathbf{f}\in C\left(\mathbb{Z}_{p}^{r},\mathbb{C}_{q}^{d}\right)$, with $\mathbf{f}\left(\mathbf{z}\right)=\left(f_{1}\left(\mathbf{z}\right),\ldots,f_{d}\left(\mathbf{z}\right)\right)$ with $f_{1},\ldots,f_{d}\in C\left(\mathbb{Z}_{p}^{r},\mathbb{C}_{q}\right)$. Then, we define: \begin{equation} \hat{\mathbf{f}}\left(\mathbf{t}\right)\overset{\textrm{def}}{=}\left(\int_{\mathbb{Z}_{p}^{r}}f_{1}\left(\mathbf{z}\right)e^{-2\pi i\left\{ \mathbf{t}\mathbf{z}\right\} _{p}}d\mathbf{z},\ldots,\int_{\mathbb{Z}_{p}^{r}}f_{d}\left(\mathbf{z}\right)e^{-2\pi i\left\{ \mathbf{t}\mathbf{z}\right\} _{p}}d\mathbf{z}\right)\label{eq:Definition of the Fourier transform of a vector-valued function on Z_P} \end{equation} This will also be written as: \begin{equation} \hat{\mathbf{f}}\left(\mathbf{t}\right)=\int_{\mathbb{Z}_{p}^{r}}\mathbf{f}\left(\mathbf{z}\right)e^{-2\pi i\left\{ \mathbf{t}\mathbf{z}\right\} _{p}}d\mathbf{z}\label{eq:Non-tuple formula for the Fourier transform of a vector-valued function on Z_P} \end{equation} \vphantom{} V. Most generally, let $\mathbf{F}\in C\left(\mathbb{Z}_{p}^{r},\mathbb{C}_{q}^{\rho,c}\right)$, with $\mathbf{F}\left(\mathbf{z}\right)=\left\{ F_{j,k}\left(\mathbf{z}\right)\right\} _{1\leq j\leq\rho,1\leq k\leq c}$ where each $F_{j,k}$ is in $C\left(\mathbb{Z}_{p}^{r},\mathbb{C}_{q}\right)$. Then: \begin{equation} \hat{\mathbf{F}}\left(\mathbf{t}\right)\overset{\textrm{def}}{=}\left\{ \int_{\mathbb{Z}_{p}^{r}}F_{j,k}\left(\mathbf{z}\right)e^{-2\pi i\left\{ \mathbf{t}\mathbf{z}\right\} _{p}}d\mathbf{z}\right\} _{1\leq j\leq\rho,1\leq k\leq c}\label{eq:Definition of the Fourier transform of a matrix-valued function on Z_P} \end{equation} This will also be written as: \begin{equation} \hat{\mathbf{F}}\left(\mathbf{t}\right)=\int_{\mathbb{Z}_{p}^{r}}\mathbf{F}\left(\mathbf{z}\right)e^{-2\pi i\left\{ \mathbf{t}\mathbf{z}\right\} _{p}}d\mathbf{z}\label{eq:Non-tuple formula for the Fourier transform of a matrix-valued function on Z_P} \end{equation} \end{notation} Like in the one-dimensional case, the most important identity for us is the Fourier series for an indicator function for the clopen set $\mathbf{n}+p^{N}\mathbb{Z}_{p}^{r}$: \begin{prop}[\textbf{Fourier Series for the Multi-Dimensional Indicator Function}] \label{prop:Multi-Dimensional indicator function Fourier Series}For $r$-tuples $\mathbf{z}$ and $\mathbf{n}$: \begin{equation} \left[\mathbf{z}\overset{p^{N}}{\equiv}\mathbf{n}\right]=\frac{1}{p^{Nr}}\sum_{\left\Vert \mathbf{t}\right\Vert _{p}\leq p^{N}}e^{2\pi i\left\{ \mathbf{t}\left(\mathbf{z}-\mathbf{n}\right)\right\} _{p}}\label{eq:MD Fourier series for indicator function} \end{equation} \end{prop} Proof: \begin{align*} \left[\mathbf{z}\overset{p^{N}}{\equiv}\mathbf{n}\right] & =\prod_{\ell=1}^{r}\left[\mathfrak{z}_{\ell}\overset{p^{N}}{\equiv}n_{\ell}\right]\\ & =\prod_{\ell=1}^{r}\frac{1}{p^{N}}\sum_{k_{\ell}=0}^{p^{N}-1}e^{2\pi i\left\{ \frac{k_{\ell}}{p^{N}}\left(\mathfrak{z}_{\ell}-n_{\ell}\right)\right\} _{p}}\\ & =\frac{1}{p^{Nr}}\prod_{\ell=1}^{r}\sum_{\left|t_{\ell}\right|_{p}\leq p^{N}}e^{2\pi i\left\{ t_{\ell}\left(\mathfrak{z}_{\ell}-n_{\ell}\right)\right\} _{p}}\\ & =\frac{1}{p^{Nr}}\sum_{\left\Vert \mathbf{t}\right\Vert _{p}\leq p^{N}}e^{2\pi i\left\{ \mathbf{t}\left(\mathbf{z}-\mathbf{n}\right)\right\} _{p}} \end{align*} Q.E.D. \vphantom{} Just to be extra cautious, let us prove that the following formal summation identities \emph{do }in fact hold true: \begin{equation} \sum_{\left\Vert \mathbf{t}\right\Vert _{p}\leq p^{N}}f\left(\mathbf{t}\right)=f\left(\mathbf{0}\right)+\sum_{n=1}^{N}\sum_{\left\Vert \mathbf{t}\right\Vert _{p}=p^{n}}f\left(\mathbf{t}\right)\label{eq:First MD level set summation identity} \end{equation} \begin{equation} \sum_{\left\Vert \mathbf{t}\right\Vert _{p}=p^{n}}f\left(\mathbf{t}\right)=\sum_{\left\Vert \mathbf{t}\right\Vert _{p}\leq p^{n}}f\left(\mathbf{t}\right)-\sum_{\left\Vert \mathbf{t}\right\Vert _{p}\leq p^{n-1}}f\left(\mathbf{t}\right)\label{eq:Second MD level set summation identity} \end{equation} so as to justify all this hullabaloo over notation. \begin{prop} For all $n\geq1$: \begin{equation} \left\{ \left\Vert \mathbf{t}\right\Vert _{p}=p^{n}\right\} =\left\{ \left\Vert \mathbf{t}\right\Vert _{p}\leq p^{n}\right\} \backslash\left\{ \left\Vert \mathbf{t}\right\Vert _{p}\leq p^{n-1}\right\} \label{eq:MD Level set decomposition} \end{equation} \end{prop} Proof: I. If $\left\Vert \mathbf{t}\right\Vert _{p}=p^{n}$, then $\left\Vert \mathbf{t}\right\Vert _{p}\leq p^{n}$. However, there is at least one $m\in\left\{ 1,\ldots,r\right\} $ so that $-v_{p}\left(t_{m}\right)=n>n-1$. As such, $\left\Vert \mathbf{t}\right\Vert _{p}>p^{n-1}$, and so: \begin{equation} \left\{ \left\Vert \mathbf{t}\right\Vert _{p}=p^{n}\right\} \subseteq\left\{ \left\Vert \mathbf{t}\right\Vert _{p}\leq p^{n}\right\} \backslash\left\{ \left\Vert \mathbf{t}\right\Vert _{p}\leq p^{n-1}\right\} \end{equation} \vphantom{} II. If $\left\Vert \mathbf{t}\right\Vert _{p}\leq p^{n}$ and $\left\Vert \mathbf{t}\right\Vert _{p}\nleq p^{n-1}$, then $-v_{p}\left(t_{m}\right)\leq n$ occurs for all $m\in\left\{ 1,\ldots,r\right\} $. Moreover, there is at least one such $m$ for which $-v_{p}\left(t_{m}\right)=n$. This means $\left\Vert \mathbf{t}\right\Vert _{p}=p^{n}$, and so: \begin{equation} \left\{ \left\Vert \mathbf{t}\right\Vert _{p}\leq p^{n}\right\} \backslash\left\{ \left\Vert \mathbf{t}\right\Vert _{p}\leq p^{n-1}\right\} \subseteq\left\{ \left\Vert \mathbf{t}\right\Vert _{p}=p^{n}\right\} \end{equation} Hence, the two sets are equal. Q.E.D. \vphantom{}Consequently, we have: \begin{prop} \label{prop:MD ramanujan sum} \begin{equation} \sum_{\left\Vert \mathbf{t}\right\Vert _{p}=p^{n}}e^{2\pi i\left\{ \mathbf{t}\mathbf{z}\right\} _{p}}=p^{rn}\left[\mathbf{z}\overset{p^{n}}{\equiv}\mathbf{0}\right]-p^{r\left(n-1\right)}\left[\mathbf{z}\overset{p^{n-1}}{\equiv}\mathbf{0}\right],\textrm{ }\forall n\geq1,\textrm{ }\forall\mathbf{z}\in\mathbb{Z}_{p}^{r}\label{eq:MD Ramanujan sum} \end{equation} \end{prop} Proof: Use (\ref{eq:Second MD level set summation identity}) and \textbf{Proposition \ref{prop:Multi-Dimensional indicator function Fourier Series}}. Q.E.D. \begin{thm} Let $\mathbf{F}\left(\mathbf{t}\right)=\bigodot_{m=1}^{r}\mathbf{F}_{m}\left(t_{m}\right)$ be an elementary function in $C\left(\mathbb{Z}_{p}^{r},\mathbb{C}_{q}^{\rho,c}\right)$. Then: \begin{equation} \hat{\mathbf{F}}\left(\mathbf{t}\right)=\hat{\mathbf{F}}\left(t_{1},\ldots,t_{r}\right)=\bigodot_{m=1}^{r}\hat{\mathbf{F}}_{m}\left(t_{m}\right)\label{eq:Fourier transform mixes with tensoring} \end{equation} \end{thm} Proof: The complex exponentials turn multiplication into addition. Q.E.D. \begin{thm} The $\mathbb{C}_{q}$-valued functions $\left\{ \mathbf{z}\mapsto e^{2\pi i\left\{ \mathbf{t}\mathbf{z}\right\} _{p}}\right\} _{\mathbf{t}\in\hat{\mathbb{Z}}_{p}^{r}}$ form a basis of $C\left(\mathbb{Z}_{p}^{r},\mathbb{C}_{q}\right)$. More generally, the vectors: \[ \left[\begin{array}{c} e^{2\pi i\left\{ \mathbf{t}\mathbf{z}\right\} _{p}}\\ 0\\ \vdots\\ 0 \end{array}\right],\left[\begin{array}{c} 0\\ e^{2\pi i\left\{ \mathbf{t}\mathbf{z}\right\} _{p}}\\ \vdots\\ 0 \end{array}\right],\ldots,\left[\begin{array}{c} 0\\ 0\\ \vdots\\ e^{2\pi i\left\{ \mathbf{t}\mathbf{z}\right\} _{p}} \end{array}\right] \] form a basis of $C\left(\mathbb{Z}_{p}^{r},\mathbb{C}_{q}^{d}\right)$. \end{thm} Proof: $C\left(\mathbb{Z}_{p}^{r},\mathbb{C}_{q}^{d}\right)$ is the closure in $\left\Vert \cdot\right\Vert _{p,q}$-norm of the set of linear combinations of elementary continuous functions $\mathbf{f}:\mathbb{Z}_{p}^{r}\rightarrow\mathbb{C}_{q}^{d}$. By the properties of the tensor product and the Fourier transform, every such elementary function $\mathbf{f}\left(\mathbf{z}\right)=\prod_{m=1}^{r}\mathbf{f}_{m}\left(\mathfrak{z}_{m}\right)$ is then uniquely expressible as: \begin{align*} \mathbf{f}\left(\mathbf{z}\right) & =\prod_{m=1}^{r}\sum_{t_{m}\in\hat{\mathbb{Z}}_{p}}\hat{\mathbf{f}}\left(t_{m}\right)e^{2\pi i\left\{ t_{m}\mathfrak{z}_{m}\right\} _{p}}\\ & =\sum_{t_{1}\in\hat{\mathbb{Z}}_{p}}\cdots\sum_{t_{r}\in\hat{\mathbb{Z}}_{p}}\left(\prod_{m=1}^{r}\hat{\mathbf{f}}\left(t_{m}\right)\right)e^{2\pi i\sum_{\ell=1}^{r}\left\{ t_{\ell}\mathfrak{z}_{\ell}\right\} _{p}}\\ & =\sum_{\mathbf{t}\in\hat{\mathbb{Z}}_{p}}\hat{\mathbf{f}}\left(\mathbf{t}\right)e^{2\pi i\left\{ \mathbf{t}\mathbf{z}\right\} _{p}} \end{align*} The theorem follows by the density of these elementary functions in $C\left(\mathbb{Z}_{p}^{r},\mathbb{C}_{q}^{d}\right)$. Q.E.D. \begin{thm} The Fourier transform $C\left(\mathbb{Z}_{p}^{r},\mathbb{C}_{q}^{\rho,c}\right)\rightarrow c_{0}\left(\hat{\mathbb{Z}}_{p}^{r},\mathbb{C}_{q}^{\rho,c}\right)$ is an isometric isomorphism of non-archimedean Banach spaces. \end{thm} Proof: Use linearity and tensors to extend the one-dimensional case (\textbf{Corollary \ref{cor:pq adic Fourier transform is an isometric isomorphism}}). Q.E.D. \begin{defn} Given a function $\hat{\mathbf{F}}:\hat{\mathbb{Z}}_{p}^{r}\rightarrow\mathbb{C}_{q}^{\rho,c}$ and an $N\geq0$, we write $\tilde{\mathbf{F}}_{N}$ to denote the function \nomenclature{$\tilde{\mathbf{F}}_{N}$}{$N$th partial Fourier series generated by $\hat{\mathbf{F}}\left(\mathbf{t}\right)$}$\tilde{\mathbf{F}}_{N}:\mathbb{Z}_{p}^{r}\rightarrow\mathbb{C}_{q}^{\rho,c}$ defined by: \begin{equation} \tilde{\mathbf{F}}_{N}\left(\mathbf{z}\right)\overset{\textrm{def}}{=}\sum_{\left\Vert \mathbf{t}\right\Vert _{p}\leq p^{N}}\hat{\mathbf{F}}\left(\mathbf{t}\right)e^{2\pi i\left\{ \mathbf{t}\mathbf{z}\right\} _{p}}\label{eq:Definition of bold F_N twiddle} \end{equation} We call $\tilde{\mathbf{F}}_{N}$ the \textbf{$N$th partial Fourier series }generated by $\hat{\mathbf{F}}$. \end{defn} \vphantom{} At this point, were we in the one-dimensional case, we would introduce measures. However, the situation is not as simple in the multi-dimensional case. Let us review our set-up so far. $\chi_{H}$\textemdash the function we want to study\textemdash goes from $\mathbb{Z}_{p}^{r}$ to $\mathbb{Z}_{q}^{d}$. As such, our setting will be functions $\mathbb{Z}_{p}^{r}\rightarrow\mathbb{C}_{q}^{d}$. The goal is to interpret integration of $\chi_{H}$ as the action of \emph{some sort }of linear map acting on $C\left(\mathbb{Z}_{p}^{r},\mathbb{C}_{q}^{d}\right)$. In the one-dimensional case, this linear map was a continuous, \emph{scalar-valued} linear operator $C\left(\mathbb{Z}_{p},K\right)\rightarrow K$ (a.k.a. a \emph{measure}), which acted upon continuous functions by way of the formula: \begin{equation} f\in C\left(\mathbb{Z}_{p},K\right)\mapsto\sum_{t\in\hat{\mathbb{Z}}_{p}}\hat{f}\left(-t\right)\hat{\mu}\left(t\right)\in K \end{equation} Because $\chi_{H}$ is vector-valued, in order to replicate this Parseval-Plancherel identity, we will need to find a way for the vector $\hat{\chi}_{H}\left(\mathbf{t}\right)$ to act on $\hat{\mathbf{f}}\left(\mathbf{t}\right)\in C\left(\mathbb{Z}_{p}^{r},\mathbb{C}_{q}^{d}\right)$. However, because we want to use this action to integrate $\chi_{H}\left(\mathbf{z}\right)$ entry-by-entry, the \emph{result }of the action cannot be scalar: it needs to be a vector. Thus, our linear operator will not send $\hat{\mathbf{f}}\left(\mathbf{t}\right)$ to a scalar, but to a \emph{vector}. So, instead of linear functionals, we will work with linear operators on finite-dimensional vector spaces. This is where a subtlety emerges: \emph{we will need to distinguish between different realizations of linear operators}.\emph{ }For example, we could have a linear operator which sends $\mathbf{f}\left(\mathbf{z}\right)\in C\left(\mathbb{Z}_{p}^{r},\mathbb{C}_{q}^{d}\right)$ to the sum: \begin{equation} \sum_{\mathbf{t}\in\hat{\mathbb{Z}}_{p}^{r}}\hat{\mathbf{f}}\left(-\mathbf{t}\right)\hat{\mathbf{m}}\left(\mathbf{t}\right) \end{equation} where $\hat{\mathbf{m}}:\hat{\mathbb{Z}}_{p}^{r}\rightarrow\mathbb{C}_{q}^{d}$ is a column-vector valued function on $\hat{\mathbb{Z}}_{p}^{r}$ and the juxtaposition $\hat{\mathbf{f}}\left(-\mathbf{t}\right)\hat{\mathbf{m}}\left(\mathbf{t}\right)$ denotes the entry-wise product of $\hat{\mathbf{f}}\left(-\mathbf{t}\right)$ and $\mathbf{\hat{m}}\left(\mathbf{t}\right)$. Alternatively, we could have a linear operator which sends $\mathbf{f}\left(\mathbf{z}\right)$ to: \begin{equation} \sum_{\mathbf{t}\in\hat{\mathbb{Z}}_{p}^{r}}\hat{\mathbf{M}}\left(\mathbf{t}\right)\hat{\mathbf{f}}\left(-\mathbf{t}\right) \end{equation} where $\hat{\mathbf{M}}:\hat{\mathbb{Z}}_{p}^{r}\rightarrow\mathbb{C}_{q}^{d,d}$ is a $d\times d$-matrix valued function and the juxtaposition $\hat{\mathbf{M}}\left(\mathbf{t}\right)\hat{\mathbf{f}}\left(-\mathbf{t}\right)$ denotes ordinary left-multiplication of the vector $\hat{\mathbf{f}}\left(-\mathbf{t}\right)$ by the matrix $\hat{\mathbf{M}}\left(\mathbf{t}\right)$. Note that $\hat{\mathbf{f}}\left(-\mathbf{t}\right)\hat{\mathbf{m}}\left(\mathbf{t}\right)$ could also be written as left-multiplication of $\hat{\mathbf{f}}\left(-\mathbf{t}\right)$ by the $d\times d$ diagonal matrix whose diagonal entries are precisely the entries of the vector $\hat{\mathbf{m}}\left(-\mathbf{t}\right)$. However, the object we would obtain by summing the Fourier series generated by said matrix would be a $d\times d$ matrix rather than a $d\times1$ vector. With all of this in mind, instead of forcing the reader to keep different context-based identifications in mind, I feel it will be simpler to distinguish between those linear operators which are represented by entry-wise multiplication against a vector and those which are represented by left-multiplication by a matrix. These considerations motivate our next definition, along with those elaborated in the examples that follow. \begin{defn}[\textbf{Thick measures}] \index{measure!thick}\index{thick measure}A\textbf{ $\left(p,q\right)$-adic} \textbf{thick measure of depth $r$ }is a continuous linear operator $\mathcal{M}:C\left(\mathbb{Z}_{p}^{r},\mathbb{C}_{q}^{d}\right)\rightarrow\mathbb{C}_{q}^{d}$. I write\footnote{In this way, we can reserve the notation $C\left(\mathbb{Z}_{p}^{r},\mathbb{C}_{q}^{d}\right)^{\prime}$ to denote the dual space of $C\left(\mathbb{Z}_{p}^{r},\mathbb{C}_{q}^{d}\right)$\textemdash continuous \emph{scalar}-valued linear operators on $C\left(\mathbb{Z}_{p}^{r},\mathbb{C}_{q}^{d}\right)$.} \nomenclature{$C\left(\mathbb{Z}_{p}^{r},\mathbb{C}_{q}^{d}\right)^{*}$}{Thick measures on $C\left(\mathbb{Z}_{p}^{r},\mathbb{C}_{q}^{d}\right)$}$C\left(\mathbb{Z}_{p}^{r},\mathbb{C}_{q}^{d}\right)^{*}$ to denote the space of vector measures on $C\left(\mathbb{Z}_{p}^{r},\mathbb{C}_{q}^{d}\right)$. Note that this is a $\mathbb{C}_{q}$-Banach space under operator norm. Finally, we will write $\mathcal{M}\left(\mathbf{f}\right)$ to denote the $d\times1$ vector obtained by applying $\mathcal{M}$ to $\mathbf{f}\in C\left(\mathbb{Z}_{p}^{r},\mathbb{C}_{q}^{d}\right)$. \end{defn} \begin{rem} I use the asterisk $*$ for the space of thick measures rather than a $\prime$, because, for the sake of consistency, the space denoted $C\left(\mathbb{Z}_{p}^{r},\mathbb{C}_{q}^{d}\right)^{\prime}$ should be the dual of $C\left(\mathbb{Z}_{p}^{r},\mathbb{C}_{q}^{d}\right)$, which is the space of continuous \emph{scalar-valued }linear maps on $C\left(\mathbb{Z}_{p}^{r},\mathbb{C}_{q}^{d}\right)$, as opposed to $C\left(\mathbb{Z}_{p}^{r},\mathbb{C}_{q}^{d}\right)^{*}$, which is the space of continuous \emph{vector-valued }linear maps on $C\left(\mathbb{Z}_{p}^{r},\mathbb{C}_{q}^{d}\right)$. \end{rem} \begin{example}[\textbf{Thick measures of matrix type}] \label{exa:thick matrix measures}Consider a bounded $d\times d$-matrix-valued function $\hat{\mathbf{M}}\in B\left(\hat{\mathbb{Z}}_{p}^{r},\mathbb{C}_{q}^{d,d}\right)$. Then, the map: \begin{equation} \mathbf{f}\in C\left(\mathbb{Z}_{p}^{r},\mathbb{C}_{q}^{d}\right)\mapsto\sum_{\mathbf{t}\in\hat{\mathbb{Z}}_{p}^{r}}\hat{\mathbf{M}}\left(\mathbf{t}\right)\hat{\mathbf{f}}\left(-\mathbf{t}\right)\in\mathbb{C}_{q}^{d}\label{eq:Action of a matrix-valued multiplier} \end{equation} defines a continuous linear operator $\mathcal{M}:C\left(\mathbb{Z}_{p}^{r},\mathbb{C}_{q}^{d}\right)\rightarrow\mathbb{C}_{q}^{d}$, and hence, a $\mathbb{C}_{q}^{d}$-valued thick measure. I call such an $\mathcal{M}$ a \textbf{thick measure of} \textbf{matrix type}\index{thick measure!matrix type}.\textbf{ }Consequently, we define $\hat{\mathbf{M}}:\hat{\mathbb{Z}}_{p}^{r}\rightarrow\mathbb{C}_{q}^{d,d}$ as the \textbf{Fourier-Stieltjes transform} of $\mathcal{M}$, and write: \begin{equation} \hat{\mathcal{M}}\left(\mathbf{t}\right)\overset{\textrm{def}}{=}\hat{\mathbf{M}}\left(\mathbf{t}\right)\label{eq:Fourier-Stieltjes transform of a matrix type thick measure} \end{equation} \index{Fourier-Stieltjes transform} \end{example} \begin{example}[\textbf{Thick measures of vector type}] \label{exa:thick vector measures}Consider a bounded $d\times1$-vector-valued function $\hat{\mathbf{m}}\in B\left(\hat{\mathbb{Z}}_{p}^{r},\mathbb{C}_{q}^{d}\right)$, as well as the map: \begin{equation} \mathbf{f}\in C\left(\mathbb{Z}_{p}^{r},\mathbb{C}_{q}^{d}\right)\mapsto\sum_{\mathbf{t}\in\hat{\mathbb{Z}}_{p}^{r}}\hat{\mathbf{m}}\left(\mathbf{t}\right)\hat{\mathbf{f}}\left(-\mathbf{t}\right)\in\mathbb{C}_{q}^{d}\label{eq:Action of a vector-valued multiplier} \end{equation} Here $\hat{\mathbf{m}}\left(\mathbf{t}\right)=\left(\hat{m}_{1}\left(\mathbf{t}\right),\ldots,\hat{m}_{d}\left(\mathbf{t}\right)\right)$ and we write: \begin{equation} \hat{\mathbf{m}}\left(\mathbf{t}\right)\hat{\mathbf{f}}\left(-\mathbf{t}\right)=\left(\begin{array}{c} \hat{m}_{1}\left(\mathbf{t}\right)\hat{f}_{1}\left(-\mathbf{t}\right)\\ \vdots\\ \hat{m}_{d}\left(\mathbf{t}\right)\hat{f}_{d}\left(-\mathbf{t}\right) \end{array}\right)\in\mathbb{C}_{q}^{d} \end{equation} to denote the entry-wise product of the column vectors $\hat{\mathbf{m}}\left(\mathbf{t}\right)$ and $\hat{\mathbf{f}}\left(-\mathbf{t}\right)$. As written, (\ref{eq:Action of a vector-valued multiplier}) defines a a continuous linear operator $\mathcal{M}:C\left(\mathbb{Z}_{p}^{r},\mathbb{C}_{q}^{d}\right)\rightarrow\mathbb{C}_{q}^{d}$, and hence, a $\mathbb{C}_{q}^{d}$-valued Banach measure. I call such an $\mathcal{M}$ a \textbf{thick measure of vector type}\index{thick measure!vector type}.\textbf{ }The \textbf{Fourier-Stieltjes transform} of $\mathcal{M}$ is given by: \begin{equation} \hat{\mathcal{M}}\left(\mathbf{t}\right)\overset{\textrm{def}}{=}\hat{\mathbf{m}}\left(\mathbf{t}\right)\label{eq:Fourier transform of a vector-type multiplier} \end{equation} \end{example} \begin{defn} Given a thick measure $\mathcal{M}:C\left(\mathbb{Z}_{p}^{r},\mathbb{C}_{q}^{d}\right)\rightarrow\mathbb{C}_{q}^{d}$ of matrix or vector type, the \textbf{Fourier-Stieltjes transform }of $\mathcal{M}$ is the function $\hat{\mathcal{M}}$ defined in the above two examples, respectively. We say a thick measure $\mathcal{M}$ is \textbf{algebraic }whenever the entries of $\hat{\mathcal{M}}\left(\mathbf{t}\right)$ are elements of $\overline{\mathbb{Q}}$ for all $\mathbf{t}\in\hat{\mathbb{Z}}_{p}^{r}$. \end{defn} \begin{defn} We write $M\left(\mathbb{Z}_{p}^{r},\mathbb{C}_{q}^{d}\right)$ and $M\left(\mathbb{Z}_{p}^{r},\mathbb{C}_{q}^{d,d}\right)$ \nomenclature{$M\left(\mathbb{Z}_{p}^{r},\mathbb{C}_{q}^{d}\right)$}{thick vector-type measures} \nomenclature{$M\left(\mathbb{Z}_{p}^{r},\mathbb{C}_{q}^{\left(d,d\right)}\right)$}{thick matrix-type measures} to denote the sets (in fact, $\mathbb{C}_{q}$-linear spaces) of vector- and matrix-type thick measures on $C\left(\mathbb{Z}_{p}^{r},\mathbb{C}_{q}^{d}\right)$, respectively. By definition, $M\left(\mathbb{Z}_{p}^{r},\mathbb{C}_{q}^{d}\right)$ and $M\left(\mathbb{Z}_{p}^{r},\mathbb{C}_{q}^{d,d}\right)$ are the operators induced by Fourier-Stieltjes transforms in $B\left(\hat{\mathbb{Z}}_{p}^{r},\mathbb{C}_{q}^{d}\right)$ and $B\left(\hat{\mathbb{Z}}_{p}^{r},\mathbb{C}_{q}^{d,d}\right)$, respectively, in the manner described in \textbf{Examples \ref{exa:thick vector measures}} and \textbf{\ref{exa:thick matrix measures}}. \end{defn} \begin{defn}[\textbf{$N$th partial kernel of a thick measure}] Consider a thick $\left(p,q\right)$-adic measure $\mathcal{M}$ with Fourier-Stieltjes transform $\hat{\mathcal{M}}\left(\mathbf{t}\right)$. For each $N\geq0$, we write \nomenclature{$\tilde{\mathcal{M}}_{N}\left(\mathbf{z}\right)$}{$N$th partial kernel of $\mathcal{M}$}$\tilde{\mathcal{M}}_{N}$ to denote the \textbf{$N$th partial kernel }of\index{thick measure!partial kernel} $\mathcal{M}$, which is defined as the continuous (in fact, locally constant)\emph{ }function: \begin{equation} \tilde{\mathcal{M}}_{N}\left(\mathbf{z}\right)\overset{\textrm{def}}{=}\sum_{\left\Vert \mathbf{t}\right\Vert _{p}\leq p^{N}}\hat{\mathcal{M}}\left(\mathbf{t}\right)e^{2\pi i\left\{ \mathbf{t}\mathbf{z}\right\} _{p}},\textrm{ }\forall\mathbf{z}\in\mathbb{Z}_{p}^{r}\label{eq:Concrete Definition of the Nth Partial Kernel of a Fourier Multiplier} \end{equation} This function is vector-valued when $\mathcal{M}$ is vector-type and is matrix-valued when $\mathcal{M}$ is matrix-type. \end{defn} \subsection{\label{subsec:5.4.2 Multi-Dimensional-Frames}Multi-Dimensional Frames} \begin{rem} Like with \ref{subsec:3.3.3 Frames}, the present subsection is going to be heavy on abstract definitions. Practical-minded readers can safely skip this section so long as the keep the following concepts in mind: Consider a function $\mathbf{F}:\mathbb{Z}_{p}^{r}\rightarrow\mathbb{C}_{q}^{\rho,c}$. A sequence of functions $\left\{ \mathbf{F}_{n}\right\} _{n\geq1}$ on $\mathbb{Z}_{p}^{r}$ is said to \textbf{converge} \textbf{to $f$ with respect to the standard $\left(p,q\right)$-adic frame }if\index{frame!standard left(p,qright)-adic@standard $\left(p,q\right)$-adic}: \vphantom{} I. For all $n$, $\mathbf{F}_{n}\left(\mathbf{z}\right)\in\overline{\mathbb{Q}}^{\rho,c}$ for all $\mathbf{z}\in\mathbb{Z}_{p}^{r}$. \vphantom{} II. For all $\mathbf{z}\in\mathbb{N}_{0}^{r}$, $\mathbf{F}\left(\mathbf{z}\right)\in\overline{\mathbb{Q}}^{\rho,c}$ and $\lim_{n\rightarrow\infty}\mathbf{F}_{n}\left(\mathbf{z}\right)\overset{\mathbb{C}^{\rho,c}}{=}\mathbf{F}\left(\mathbf{z}\right)$ (meaning the convergence is in the topology of $\mathbb{C}$). \vphantom{} III. For all $\mathbf{z}\in\left(\mathbb{Z}_{p}^{r}\right)^{\prime}$, $\mathbf{F}\left(\mathbf{z}\right)\in\mathbb{C}_{q}^{\rho,c}$ and $\lim_{n\rightarrow\infty}\mathbf{F}_{n}\left(\mathbf{z}\right)\overset{\mathbb{C}_{q}^{\rho,c}}{=}\mathbf{F}\left(\mathbf{z}\right)$ (meaning the convergence is in the topology of $\mathbb{C}_{q}$). \vphantom{} We write $\mathcal{F}_{p,q}$ to denote the standard $\left(p,q\right)$-adic frame. In particular, we write: \vphantom{} i. $\mathcal{F}_{p,q}^{d}$, to denote the case where $\mathbf{F}\left(\mathbf{z}\right)$ is a $d\times1$-column-vector-valued function (the $d$-dimensional standard frame\index{frame!standard left(p,qright)-adic@standard $\left(p,q\right)$-adic!$d$-dimensional}); \vphantom{} ii. $\mathcal{F}_{p,q}^{d,d}$ to denote the case where $\mathbf{F}\left(\mathbf{z}\right)$ is a $d\times d$-matrix-valued function (the $d,d$-dimensional standard frame \index{frame!standard left(p,qright)-adic@standard $\left(p,q\right)$-adic!left(d,dright) -dimensional@$d,d$-dimensional}). \vphantom{} We then write $\lim_{n\rightarrow\infty}\mathbf{f}_{n}\left(\mathbf{z}\right)\overset{\mathcal{F}_{p,q}^{d}}{=}\mathbf{f}\left(\mathbf{z}\right)$ and $\lim_{n\rightarrow\infty}\mathbf{F}_{n}\left(\mathbf{z}\right)\overset{\mathcal{F}_{p,q}^{d,d}}{=}\mathbf{F}\left(\mathbf{z}\right)$ to denote convergence with respect to the $d$-dimensional and $d,d$-dimensional standard frames, respectively. \end{rem} \vphantom{} HERE BEGINS THE DISCUSSION OF MULTI-DIMENSIONAL FRAMES \vphantom{} The set-up for the multi-dimensional case will be a tad bit more involved than the one-dimensional case. The most significant difference is that because our functions can now take vector or matrix values, instead of assigning topological fields to each point of the domain, we will assign topological \emph{vector spaces}, all of which contain a finite-dimensional $\overline{\mathbb{Q}}$-vector-space of an appropriate dimension. These will come in two flavors: those with the discrete topology, and those which are Banach spaces with respect to a norm induced by an absolute value. \begin{defn}[\textbf{Multi-Dimensional Frames}] \label{def:MD frame}Fix an integer $d\geq1$ . A $d$-dimensional $p$-adic \index{frame!$p$-adic}\index{frame}(or just ``frame'', for short) $\mathcal{F}$ of depth $r$ is the following collection of data: \vphantom{} I. A set $U_{\mathcal{F}}\subseteq\mathbb{Z}_{p}^{r}$, called the \textbf{domain }of $\mathcal{F}$. \vphantom{} II. For each $\mathbf{z}\in U_{\mathcal{F}}$, a topological field $K_{\mathbf{z}}$ containing $\overline{\mathbb{Q}}$ and a $d$-dimensional topological vector space $V_{\mathbf{z}}$ over $K_{\mathbf{z}}$. \nomenclature{$K_{\mathbf{z}}^{d}$}{ }\nomenclature{$V_{\mathbf{z}}$}{ } We allow for $K_{\mathbf{z}}=\overline{\mathbb{Q}}$. In particular, for any $\mathbf{z}\in U_{\mathcal{F}}$, we require the topology of $K_{\mathbf{z}}$ to be either the discrete topology\emph{ or} the topology induced by an absolute value,\nomenclature{$K_{\mathbf{z}}$}{ }, denoted \nomenclature{$\left|\cdot\right|_{K_{\mathbf{z}}}$}{ }$\left|\cdot\right|_{K_{\mathbf{z}}}$. For any $\mathbf{z}$ for which $K_{\mathbf{z}}$ is given the discrete topology, we endow $V_{\mathbf{z}}$ with the discrete topology as well. If $K_{\mathbf{z}}$ is given the topology induced by an absolute value, we require $\left(K_{\mathbf{z}},\left|\cdot\right|_{K_{\mathbf{z}}}\right)$ to be a complete metric space, and then endow $V_{\mathbf{z}}$ with the topology induced by the norm $\left\Vert \cdot\right\Vert _{K_{\mathbf{z}}}$\nomenclature{$\left\Vert \cdot\right\Vert _{K_{\mathbf{z}}}$}{ }, defined here as outputting the maximum of the $K_{\mathbf{z}}$-absolute values of the entries of a given element of $V_{\mathbf{z}}$. This makes $V_{\mathbf{z}}$ into a finite-dimensional Banach space. \end{defn} \begin{defn} I adopt the convention of writing $\mathcal{F}^{d}$ when I mean that the $V_{\mathbf{z}}$s are spaces of $d\times1$ column vectors. In that case, I write $V_{\mathbf{z}}$ as $K_{\mathbf{z}}^{d}$. On the other hand, when working with $\rho\times c$ matrices (where $d=\rho c$) instead of $d\times1$ column vectors, I write $\mathcal{F}^{\rho,c}$ instead of $\mathcal{F}$, and say that this frame has dimension $\rho,c$. For this case, I write $K_{\mathbf{z}}^{\rho,c}$ instead of $V_{\mathbf{z}}$. \end{defn} \vphantom{} The two most important objects associated with a given frame are its image and the space of compatible functions, defined below. \begin{defn}[\textbf{Image and Compatible Functions}] \label{def:MD Frame terminology}Let $\mathcal{F}$ be a $p$-adic frame. \vphantom{} I. The \textbf{image }of $\mathcal{F}$, denoted $I\left(\mathcal{F}\right)$, is the \emph{set }defined by: \begin{equation} I\left(\mathcal{F}\right)\overset{\textrm{def}}{=}\bigcup_{\mathbf{z}\in U_{\mathcal{F}}}V_{\mathbf{z}}\label{eq:MD The image of a frame} \end{equation} \index{frame!image} \vphantom{} II. A function $\chi:U_{\mathcal{F}}\rightarrow I\left(\mathcal{F}\right)$ is said to be \textbf{$\mathcal{F}$-compatible} / \textbf{compatible }(\textbf{with $\mathcal{F}$}) whenever \index{mathcal{F}-@$\mathcal{F}$-!compatible}\index{frame!compatible functions} $\chi\left(\mathfrak{z}\right)\in V_{\mathbf{z}}$ for all $\mathbf{z}\in U_{\mathcal{F}}$. I write $C\left(\mathcal{F}\right)$ to denote the set of all $\mathcal{F}$-compatible functions. \end{defn} \begin{rem} Note that $C\left(\mathcal{F}\right)$ is a vector space over $\overline{\mathbb{Q}}$ with respect to point-wise addition of functions and scalar multiplication. \end{rem} \vphantom{} In addition to this, we also have the following bits of terminology: \begin{defn}[\textbf{Frame Terminology}] Let $\mathcal{F}$ be a $p$-adic frame of dimension $d$ and depth $r$. \vphantom{} I. I call $\mathbb{Z}_{p}^{r}\backslash U_{\mathcal{F}}$ the \textbf{set of singularities }of the frame. I say that $\mathcal{F}$ is \textbf{non-singular }whenever $U_{\mathcal{F}}=\mathbb{Z}_{p}^{r}$.\index{frame!non-singular} \vphantom{} II. Given a topology $\tau$ (so, $\tau$ could be $\textrm{dis}$, $\infty$, $\textrm{non}$, or $p$), I writ $U_{\tau}\left(\mathcal{F}\right)$ to denote the set of $\mathbf{z}\in U_{\mathcal{F}}$ so that $K_{\mathbf{z}}$ has been equipped with $\tau$. I call the $U_{\tau}\left(\mathcal{F}\right)$\textbf{ $\tau$-convergence domain }or \textbf{domain of $\tau$ convergence }of $\mathcal{F}$. In this way, we can speak of the \textbf{discrete convergence domain}, the \textbf{archimedean convergence domain}, the \textbf{non-archimedean convergence domain}, and the \textbf{$p$-adic convergence domain} of a given frame $\mathcal{F}$. \vphantom{} IV. I say $\mathcal{F}$ is \textbf{simple} \index{frame!simple}if either $U_{\textrm{non}}\left(\mathcal{F}\right)$ is empty or there exists a single metrically complete non-archimedean field extension $K$ of $\overline{\mathbb{Q}}$ so that $K=K_{\mathbf{z}}$ for all $\mathbf{z}\in U_{\textrm{non}}\left(\mathcal{F}\right)$. (That is to say, for a simple frame, we use at most one non-archimedean topology.) \vphantom{} V. I say $\mathcal{F}$ is \textbf{proper }\index{frame!proper}proper whenever $K_{\mathbf{z}}$ is not a $p$-adic field for any $\mathbf{z}\in U_{\textrm{non}}\left(\mathcal{F}\right)$. \end{defn} \vphantom{} UNLESS STATED OTHERWISE, ALL FRAMES ARE ASSUMED TO BE PROPER. \vphantom{} Like in the one-dimensional case, only one $p$-adic frame will ever be used in this dissertation: the standard one. \begin{defn} The \textbf{standard $d$-dimensional ($\left(p,q\right)$-adic) frame}\index{frame!standard left(p,qright)-adic@standard $\left(p,q\right)$-adic}, denoted $\mathcal{F}_{p,q}$, is the frame for which the topology of $\mathbb{C}$ is associated to $\mathbb{N}_{0}^{d}$ and the topology of $\mathbb{C}_{q}$ is associated to $\left(\mathbb{Z}_{p}^{r}\right)^{\prime}$. In particular, I write $\mathcal{F}_{p,q}^{d}$ to denote the case where the topological vector spaces are spaces of $d\times1$ column vectors; I write $\mathcal{F}_{p,q}^{\rho,c}$ to denote the case where the topological vector spaces are spaces of $\rho\times c$ column vectors. \end{defn} \vphantom{} Next we have the all-important notion of $\mathcal{F}$-convergence. \begin{defn}[\textbf{$\mathcal{F}$-convergence}] Given a frame a $\mathcal{F}$ and a function $\chi\in C\left(\mathcal{F}\right)$, we say a sequence $\left\{ \chi_{n}\right\} _{n\geq1}$ in $C\left(\mathcal{F}\right)$ \textbf{converges to $\chi$ over $\mathcal{F}$ }(or \textbf{is $\mathcal{F}$-convergent to $\chi$}) whenever, for each $\mathbf{z}\in U_{\mathcal{F}}$, we have: \begin{equation} \lim_{n\rightarrow\infty}\chi_{n}\left(\mathbf{z}\right)\overset{V_{\mathbf{z}}}{=}\chi\left(\mathbf{z}\right) \end{equation} Note that if $K_{\mathbf{z}}$ has the discrete topology, the limit implies that $\chi_{n}\left(\mathbf{z}\right)=\chi\left(\mathbf{z}\right)$ for all sufficiently large $n$.\index{mathcal{F}-@$\mathcal{F}$-!convergence}\index{frame!convergence} On the other hand, if $K_{\mathbf{z}}$ has the topology of a valued field, we then require: \begin{equation} \lim_{n\rightarrow\infty}\left\Vert \chi\left(\mathbf{z}\right)-\chi_{n}\left(\mathbf{z}\right)\right\Vert _{K_{\mathbf{z}}}\overset{\mathbb{R}}{=}0 \end{equation} I call $\chi$ the \index{mathcal{F}-@$\mathcal{F}$-!limit}\textbf{$\mathcal{F}$-limit of the $\chi_{n}$s}. \index{frame!limit}More generally, I say $\left\{ \chi_{n}\right\} _{n\geq1}$ is \textbf{$\mathcal{F}$-convergent / converges over $\mathcal{F}$} whenever there is a function $\chi\in C\left(\mathcal{F}\right)$ so that the $\chi_{n}$s are $\mathcal{F}$-convergent to $\chi$. In symbols, I denote $\mathcal{F}$-convergence by: \begin{equation} \lim_{n\rightarrow\infty}\chi_{n}\left(\mathbf{z}\right)\overset{\mathcal{F}}{=}\chi\left(\mathbf{z}\right),\textrm{ }\forall\mathbf{z}\in U_{\mathcal{F}}\label{eq:MD Definition-in-symbols of F-convergence} \end{equation} or simply: \begin{equation} \lim_{n\rightarrow\infty}\chi_{n}\overset{\mathcal{F}}{=}\chi\label{eq:MD Simplified version of expressing f as the F-limit of f_ns} \end{equation} \end{defn} \begin{rem} In an abuse of notation, we will sometimes write $\mathcal{F}$ convergence as: \begin{equation} \lim_{n\rightarrow\infty}\left\Vert \chi_{n}\left(\mathbf{z}\right)-\chi\left(\mathbf{z}\right)\right\Vert _{K_{\mathbf{z}}},\textrm{ }\forall\mathfrak{z}\in U_{\mathcal{F}} \end{equation} The abuse here is that the norm $\left\Vert \cdot\right\Vert _{K_{\mathbf{z}}}$ will not exist if $\mathbf{z}\in U_{\textrm{dis}}\left(\mathcal{F}\right)$. As such, for any $\mathbf{z}\in U_{\textrm{dis}}\left(\mathcal{F}\right)$, I define the expression: \begin{equation} \lim_{n\rightarrow\infty}\left\Vert \chi_{n}\left(\mathbf{z}\right)-\chi\left(\mathbf{z}\right)\right\Vert _{K_{\mathbf{z}}}=0 \end{equation} as meaning that for all sufficiently large $n$, the equality $\chi_{n}\left(\mathbf{z}\right)=\chi_{n+1}\left(\mathbf{z}\right)$ holds in $K_{\mathbf{z}}^{d}$. \end{rem} \vphantom{} Now that we have the language of frames at our disposal, we can begin to define classes of $\left(p,q\right)$-adic measures which will be of interest to us. \begin{defn}[\textbf{Thick} \textbf{$\mathcal{F}$-measures}] Given a $d$-dimensional depth $r$ $p$-adic frame $\mathcal{F}$, let $V=\overline{\mathbb{Q}}^{d}$ if $\mathcal{F}=\mathcal{F}^{d}$ and let $V=\overline{\mathbb{Q}}^{\rho,c}$ if $\mathcal{F}=\mathcal{F}^{\rho,c}$. We write \nomenclature{$M\left(\mathcal{F}\right)$}{$\mathcal{F}$-measures}$M\left(\mathcal{F}\right)$ to denote the set of all functions $\hat{\mathcal{M}}:\hat{\mathbb{Z}}_{p}^{r}\rightarrow V$ so that $\hat{\mathcal{M}}\in B\left(\hat{\mathbb{Z}}_{p}^{r},V_{\mathbf{z}}\right)$ for all $\mathbf{z}\in U_{\textrm{non}}\left(\mathcal{F}\right)$; that is: \begin{equation} \left\Vert \hat{\mathcal{M}}\right\Vert _{p,K_{\mathbf{z}}}<\infty,\textrm{ }\forall\mathbf{z}\in U_{\textrm{non}}\left(\mathcal{F}\right)\label{eq:Definition of a thick F-measure} \end{equation} where: \[ \left\Vert \hat{\mathcal{M}}\right\Vert _{p,K_{\mathbf{z}}}=\sup_{\mathbf{t}\in\hat{\mathbb{Z}}_{p}^{r}}\left\Vert \hat{\mathcal{M}}\left(\mathbf{t}\right)\right\Vert _{K_{\mathbf{z}}} \] where $\left\Vert \cdot\right\Vert _{K_{\mathbf{z}}}$, recall, is the norm on $V_{\mathbf{z}}$, being the maximum of the $K_{\mathbf{z}}$-absolute-values of the entries of $\hat{\mathcal{M}}\left(\mathbf{t}\right)$. \vphantom{} Next observe that for every $\mathbf{z}\in U_{\textrm{non}}\left(\mathcal{F}\right)$, the map: \begin{equation} \mathbf{f}\in C\left(\mathbb{Z}_{p}^{r},K_{\mathbf{z}}\right)\mapsto\sum_{\mathbf{t}\in\hat{\mathbb{Z}}_{p}^{r}}\hat{\mathcal{M}}\left(\mathbf{t}\right)\hat{\mathbf{f}}\left(-\mathbf{t}\right)\in K_{\mathbf{z}}\label{eq:Definition of the action of a thick F-measure} \end{equation} then defines an element of $C\left(\mathbb{Z}_{p}^{r},K_{\mathbf{z}}^{d}\right)^{*}$, the space of all continuous $K_{\mathbf{z}}^{d}$-valued linear maps on the space of continuous functions $\mathbf{f}:\mathbb{Z}_{p}^{r}\rightarrow K_{\mathbf{z}}^{d}$. As such, we will identify $M\left(\mathcal{F}\right)$ with elements of $\bigcap_{\mathbf{z}\in U_{\textrm{non}}\left(\mathcal{F}\right)}C\left(\mathbb{Z}_{p}^{r},K_{\mathbf{z}}\right)^{*}$, and refer to elements of $M\left(\mathcal{F}\right)$ as \textbf{thick} \textbf{$\mathcal{F}$-measures}\index{mathcal{F}-@$\mathcal{F}$-!thick measure}\index{measure!thick}. Note that the specific type of multiplication signified by the juxtaposition of $\hat{\mathcal{M}}\left(\mathbf{t}\right)$ and $\hat{\mathbf{f}}\left(-\mathbf{t}\right)$ in (\ref{eq:Definition of the action of a thick F-measure}) depends on whether $\hat{\mathcal{M}}$ is of vector type or matrix type. We write $M\left(\mathcal{F}^{d}\right)$\nomenclature{$M\left(\mathcal{F}^{d}\right)$}{ } and $M\left(\mathcal{F}^{\rho,c}\right)$\nomenclature{$M\left(\mathcal{F}^{\rho,c}\right)$}{ } to denote the vector type and matrix type cases, respectively. Regardless, we then identify $\hat{\mathcal{M}}$ with the Fourier-Stieltjes transform of a thick measure $\mathcal{M}$. Thus, the statement ``$\mathcal{M}\in M\left(\mathcal{F}^{d}\right)$'' means that, for all $\mathbf{z}\in U_{\textrm{non}}\left(\mathcal{F}\right)$, $\mathcal{M}$ is an element of $C\left(\mathbb{Z}_{p}^{r},K_{\mathbf{z}}\right)^{*}$ whose action on a function is given by $\sum_{\mathbf{t}\in\hat{\mathbb{Z}}_{p}^{r}}\hat{\mathcal{M}}\left(\mathbf{t}\right)\hat{\mathbf{f}}\left(-\mathbf{t}\right)$, where $\hat{\mathcal{M}}\in B\left(\hat{\mathbb{Z}}_{p}^{r},K_{\mathbf{z}}^{d}\right)$. ``$\mathcal{M}\in M\left(\mathcal{F}^{\rho,c}\right)$'', meanwhile, means that, for all $\mathbf{z}\in U_{\textrm{non}}\left(\mathcal{F}\right)$, $\mathcal{M}$ is an element of $C\left(\mathbb{Z}_{p}^{r},K_{\mathbf{z}}\right)^{*}$ whose action on a function is given by $\sum_{\mathbf{t}\in\hat{\mathbb{Z}}_{p}^{r}}\hat{\mathcal{M}}\left(\mathbf{t}\right)\hat{\mathbf{f}}\left(-\mathbf{t}\right)$, where $\hat{\mathcal{M}}\in B\left(\hat{\mathbb{Z}}_{p}^{r},K_{\mathbf{z}}^{d,d}\right)$. Both cases force $\left\Vert \hat{\mathcal{M}}\right\Vert _{p,K_{\mathbf{z}}}$ to be finite for all $\mathbf{z}\in U_{\textrm{non}}\left(\mathcal{F}\right)$. \end{defn} \begin{prop} Let $\hat{\mathcal{M}}\in M\left(\mathcal{F}\right)$. Then, the $N$th partial sum of the Fourier series generated by $\hat{\mathcal{M}}$: \begin{equation} \tilde{\mathcal{M}}_{N}\left(\mathbf{z}\right)\overset{\textrm{def}}{=}\sum_{\left\Vert \mathbf{t}\right\Vert _{p}\leq p^{N}}\hat{\mathcal{M}}\left(\mathbf{t}\right)e^{2\pi i\left\{ \mathbf{t}\mathbf{z}\right\} _{p}},\textrm{ }\forall\mathbf{z}\in\mathbb{Z}_{p}^{r} \end{equation} is an element of $C\left(\mathcal{F}\right)$. \end{prop} Proof: A straight-forward multi-dimensional analogue of the one-dimensional case. Q.E.D. \begin{defn}[\textbf{$\mathcal{F}$-rising thick measure}] Given a $p$-adic frame $\mathcal{F}$ of dimension $d$ and depth $r$, we say $\mathcal{M}\in M\left(\mathcal{F}\right)$ \index{thick measure!mathcal{F}-rising@$\mathcal{F}$-rising} \textbf{rises over $\mathcal{F}$} / is \textbf{$\mathcal{F}$-rising} whenever the sequence $\left\{ \tilde{\mathcal{M}}_{N}\right\} _{N\geq0}$ is $\mathcal{F}$-convergent. That is: \begin{equation} \tilde{\mathcal{M}}\left(\mathbf{z}\right)\overset{\mathcal{F}}{=}\lim_{N\rightarrow\infty}\tilde{\mathcal{M}}_{N}\left(\mathbf{z}\right),\textrm{ }\forall\mathbf{z}\in U_{\mathcal{F}} \end{equation} which, recall, means: \vphantom{} I. For every $\mathbf{z}\in U_{\textrm{dis}}\left(\mathcal{F}\right)$, $\tilde{\mathcal{M}}_{N}\left(\mathbf{z}\right)=\tilde{\mathcal{M}}_{N+1}\left(\mathbf{z}\right)$ for all sufficiently large $N$. \vphantom{} II. For every $\mathbf{z}\in U_{\textrm{arch}}\left(\mathcal{F}\right)\cup U_{\textrm{non}}\left(\mathcal{F}\right)$, $\lim_{N\rightarrow\infty}\tilde{\mathcal{M}}_{N}\left(\mathbf{z}\right)$ converges to a limit in the topology of $V_{\mathbf{z}}$. \vphantom{} We write $\mathcal{M}_{\textrm{rise}}\left(\mathcal{F}\right)$ to denote the space of thick $\mathcal{F}$-rising measures, with $\mathcal{M}_{\textrm{rise}}\left(\mathcal{F}^{d}\right)$ and $\mathcal{M}_{\textrm{rise}}\left(\mathcal{F}^{\rho,c}\right)$ being used to remind the reader that the thick measures in question are of vector type and matrix type, respectively. Additionally, we call a thick measure \textbf{rising-continuous }whenever it rises over the standard frame. \index{thick measure!rising-continuous} \end{defn} \begin{defn}[\textbf{Kernel of an $\mathcal{F}$-rising measure}] Given a $p$-adic frame $\mathcal{F}$ of dimension $d$ and depth $r$ along with an $\mathcal{F}$-rising thick measure $\mathcal{M}$, the\index{mathcal{F}-@$\mathcal{F}$-!kernel} \textbf{kernel }of $\mathcal{M}$\textemdash denoted\index{thick measure!mathcal{F}-kernel@$\mathcal{F}$-kernel} \nomenclature{$\tilde{\mathcal{M}}\left(\mathbf{z}\right)$}{kernel of $\mathcal{M}$}\textemdash is the vector- or matrix-valued function on $U_{\mathcal{F}}$ defined by the $\mathcal{F}$-limit of the $\tilde{\mathcal{M}}_{N}$s: \begin{equation} \tilde{\mathcal{M}}\left(\mathbf{z}\right)\overset{\mathcal{F}}{=}\lim_{N\rightarrow\infty}\tilde{\mathcal{M}}_{N}\left(\mathbf{z}\right),\textrm{ }\forall\mathbf{z}\in U_{\mathcal{F}}\label{eq:Definition of M-twiddle / the kernel of M} \end{equation} We write $\left(\tilde{\mathcal{M}}\right)_{N}\left(\mathbf{z}\right)$\nomenclature{$\left(\tilde{\mathcal{M}}\right)_{N}\left(\mathbf{z}\right)$}{$N$th truncation of the kernel of $\mathcal{M}$} to denote the $N$th truncation of the kernel of $\mathcal{M}$: \begin{equation} \left(\tilde{\mathcal{M}}\right)_{N}\left(\mathbf{z}\right)\overset{\textrm{def}}{=}\sum_{\mathbf{n}=\mathbf{0}}^{p^{N}-1}\tilde{\mathcal{M}}\left(\mathbf{n}\right)\left[\mathbf{z}\overset{p^{N}}{\equiv}\mathbf{n}\right]\label{eq:Definition of the Nth truncation of the full kernel of a rising multiplier} \end{equation} More generally, we write: \begin{equation} \left(\tilde{\mathcal{M}}_{M}\right)_{N}\left(\mathbf{z}\right)\overset{\textrm{def}}{=}\sum_{\mathbf{n}=\mathbf{0}}^{p^{N}-1}\tilde{\mathcal{M}}_{M}\left(\mathbf{n}\right)\left[\mathbf{z}\overset{p^{N}}{\equiv}\mathbf{n}\right]\label{eq:Nth truncation of the Mth partial Kernel of M} \end{equation} to denote the $N$th truncation of $\tilde{\mathcal{M}}_{M}$. \end{defn} \begin{rem} In this terminology, observe that for $\mathbf{f}\in C\left(\mathbb{Z}_{p}^{r},\mathbb{C}_{q}^{d}\right)$, we can express the image of $\mathbf{f}$ under $\mathcal{M}$ as: \begin{equation} \mathcal{M}\left(\mathbf{f}\right)\overset{\mathbb{C}_{q}^{d}}{=}\lim_{N\rightarrow\infty}\int_{\mathbb{Z}_{p}^{r}}\tilde{\mathcal{M}}_{N}\left(\mathbf{z}\right)\mathbf{f}\left(\mathbf{z}\right)d\mathbf{z}=\lim_{N\rightarrow\infty}\sum_{\left\Vert \mathbf{t}\right\Vert _{p}\leq p^{N}}\hat{\mathcal{M}}\left(\mathbf{t}\right)\hat{\mathbf{f}}\left(-\mathbf{t}\right) \end{equation} \end{rem} \begin{defn}[\textbf{$\mathcal{F}$-degenerate thick measure}] For a $p$-adic frame $\mathcal{F}$, a thick $\mathcal{F}$-rising measure \index{thick measure!mathcal{F}-degenerate@$\mathcal{F}$-degenerate}$\mathcal{M}$ is said to be \textbf{($\mathcal{F}$-)degenerate }whenever its kernel is identically zero. We write $M_{\textrm{dgen}}\left(\mathcal{F}\right)$ to denote the set of $\mathcal{F}$-degenerate thick measures. We write this as $M_{\textrm{dgen}}\left(\mathcal{F}^{d}\right)$ and $M_{\textrm{dgen}}\left(\mathcal{F}^{\rho,c}\right)$ when we wish to single out vector-type and matrix-type thick measures, respectively. \end{defn} \subsection{\label{subsec:5.4.2 Quasi-Integrability-Revisited}Quasi-Integrability and Thick Measures} \begin{rem} Like in \ref{subsec:3.3.5 Quasi-Integrability}, the purpose of Subsection \ref{subsec:5.4.3 -adic-Wiener-Tauberian} is to provide a solid foundation for understanding and discussing quasi-integrability in its multi-dimensional incarnation. As it regards Chapter \ref{chap:6 A-Study-of}'s analysis of multi-dimensional $\chi_{H}$, the main thing the reader needs to know is that I say a function $\chi:\mathbb{Z}_{p}^{r}\rightarrow\mathbb{C}_{q}^{d}$ is \textbf{quasi-integrable }with\index{quasi-integrability} respect to the standard $\left(p,q\right)$-adic frame whenever there exists a function $\hat{\chi}:\hat{\mathbb{Z}}_{p}^{r}\rightarrow\overline{\mathbb{Q}}^{d}$ so that: \begin{equation} \chi\left(\mathbf{z}\right)\overset{\mathcal{F}_{p,q}^{d}}{=}\lim_{N\rightarrow\infty}\sum_{\left\Vert \mathbf{t}\right\Vert _{p}\leq p^{N}}\hat{\chi}\left(\mathbf{t}\right)e^{2\pi i\left\{ \mathbf{t}\mathbf{z}\right\} _{p}},\textrm{ }\forall\mathbf{z}\in\mathbb{Z}_{p}^{r} \end{equation} We call the function $\hat{\chi}$ a\emph{ }\textbf{Fourier transform }of $\chi$. Once again, this Fourier transform is only unique modulo the Fourier-Stieltjes transform of a thick measure of vector type which is degenerate with respect to the standard $\left(p,q\right)$-adic frame. Like in the one-dimensional case, given a Fourier transform $\hat{\chi}$ of a quasi-integrable function $\chi$, we write $\tilde{\chi}_{N}$ to denote the $N$th partial Fourier series generated by $\hat{\chi}$. \end{rem} \vphantom{} HERE BEGINS THE DISCUSSION OF MULTI-DIMENSIONAL QUASI-INTEGRABILITY \vphantom{} \begin{defn} Consider a $d$-dimensional $p$-adic frame $\mathcal{F}$ of depth $r$. \vphantom{} We say a function $\chi\in C\left(\mathcal{F}\right)$ is\textbf{ quasi-integrable} \textbf{with respect to $\mathcal{F}$} / $\mathcal{F}$\textbf{-quasi-integrable }whenever $\chi:U_{\mathcal{F}}\rightarrow I\left(\mathcal{F}\right)$ is the kernel of some $\mathcal{F}$-rising measure $\mathcal{M}\in M\left(\mathcal{F}^{d}\right)$ (that is, of \emph{vector }type). In other words: \vphantom{} I. There is a vector-type thick measure $\mathcal{M}$ with a Fourier-Stieltjes transform $\hat{\mathcal{M}}:\hat{\mathbb{Z}}_{p}^{r}\rightarrow\overline{\mathbb{Q}}^{d}$ which is bounded in $\left\Vert \cdot\right\Vert _{p,K_{\mathbf{z}}}$-norm for all $\mathbf{z}\in U_{\textrm{non}}\left(\mathcal{F}\right)$; \vphantom{} II. $\tilde{\mathcal{M}}_{N}$ $\mathcal{F}$-converges to $\chi$ as $N\rightarrow\infty$: \begin{equation} \lim_{N\rightarrow\infty}\left\Vert \chi\left(\mathbf{z}\right)-\tilde{\mathcal{M}}_{N}\left(\mathbf{z}\right)\right\Vert _{K_{\mathbf{z}}}\overset{\mathbb{R}}{=}0,\textrm{ }\forall\mathbf{z}\in U_{\mathcal{F}}\label{eq:MD definition of quasi-integrability} \end{equation} \vphantom{} I call any $\mathcal{M}$ satisfying these properties an\textbf{ $\mathcal{F}$-quasi-integral}\index{mathcal{F}-@$\mathcal{F}$-!quasi-integral}. I write $\tilde{L}^{1}\left(\mathcal{F}\right)$ to denote the set of all $\mathcal{F}$-quasi-integrable functions. (Note that $\tilde{L}^{1}\left(\mathcal{F}\right)$ is then a vector space over $\overline{\mathbb{Q}}$.) We write $\tilde{L}^{1}\left(\mathcal{F}^{d}\right)$ when we wish to remind our audience that the elements of $\tilde{L}^{1}\left(\mathcal{F}^{d}\right)$ are $d\times1$-vector-valued functions, rather than scalar valued functions. When $\mathcal{F}$ is the standard $d$-dimensional depth $r$ $\left(p,q\right)$-adic frame, this definition becomes: \vphantom{} i. $\chi$ is a function from $\mathbb{Z}_{p}^{r}\rightarrow\mathbb{C}_{q}^{d}$ so that $\chi\left(\mathbf{z}\right)\in\mathbb{C}^{d}$ for all $\mathbf{z}\in\mathbb{N}_{0}^{r}$ and $\chi\left(\mathbf{z}\right)\in\mathbb{C}_{q}^{d}$ for all $\mathbf{z}\in\left(\mathbb{Z}_{p}^{r}\right)^{\prime}$; \vphantom{} ii. For each $\mathbf{z}\in\mathbb{N}_{0}^{r}$, as $N\rightarrow\infty$, $\tilde{\mathcal{M}}_{N}\left(\mathbf{z}\right)$ converges to $\chi\left(\mathbf{z}\right)$ in $\mathbb{C}^{d}$. \vphantom{} iii. For each $\mathbf{z}\in\left(\mathbb{Z}_{p}^{r}\right)^{\prime}$, as $N\rightarrow\infty$, $\tilde{\mathcal{M}}_{N}\left(\mathbf{z}\right)$ converges to $\chi\left(\mathbf{z}\right)$ in $\mathbb{C}_{q}^{d}$. \vphantom{} We then write \nomenclature{$\tilde{L}^{1}\left(\mathbb{Z}_{p}^{r},\mathbb{C}_{q}^{d}\right)$}{ }$\tilde{L}^{1}\left(\mathbb{Z}_{p}^{r},\mathbb{C}_{q}^{d}\right)$ and \nomenclature{$\tilde{L}^{1}\left(\mathcal{F}_{p,q}^{d}\right)$}{ }$\tilde{L}^{1}\left(\mathcal{F}_{p,q}^{d}\right)$ to denote the space of all functions $\mathbb{Z}_{p}^{r}\rightarrow\mathbb{C}_{q}^{d}$ which are quasi-integrable with respect to the standard frame. \end{defn} \vphantom{} Once again, we will end up with a generalization of the Fourier transform of a function $\chi:\mathbb{Z}_{p}^{r}\rightarrow\mathbb{C}_{q}^{d}$ by treating $\chi$ as the kernel of some thick vector-type measure. \begin{defn} Let $\chi$ be $\mathcal{F}$-quasi-integrable. Then, \textbf{a} \textbf{Fourier Transform} of $\chi$ is a function $\hat{\mathcal{M}}:\hat{\mathbb{Z}}_{p}^{r}\rightarrow\overline{\mathbb{Q}}^{d}$ which is the Fourier-Stieltjes transform of some $\mathcal{F}$-quasi-integral $\mathcal{M}$ of $\chi$. We write these functions as $\hat{\chi}$ and write the associated thick measure as $\chi\left(\mathbf{z}\right)d\mathbf{z}$. Given $\mathbf{f}\in C\left(\mathbb{Z}_{p}^{r},\mathbb{C}_{q}^{d}\right)$, we then define: \begin{equation} \int_{\mathbb{Z}_{p}^{r}}\mathbf{f}\left(\mathbf{z}\right)\chi\left(\mathbf{z}\right)d\mathbf{z}\overset{\textrm{def}}{=}\sum_{\mathbf{t}\in\hat{\mathbb{Z}}_{p}^{r}}\hat{\mathbf{f}}\left(-\mathbf{t}\right)\hat{\chi}\left(\mathbf{t}\right)\label{eq:MD Definition of the integral of a quasi-integrable function against a continuous function} \end{equation} Note that because $\mathcal{M}$ is a thick measure of vector type, its Fourier-Stieltjes transform $\hat{\mathcal{M}}\left(\mathbf{t}\right)=\hat{\chi}\left(\mathbf{t}\right)$ is a $d\times1$ column vector with entries in $\overline{\mathbb{Q}}$. The above integral is then the same thing as the image of $\mathbf{f}$ under $\mathcal{M}$: \begin{equation} \mathcal{M}\left(\mathbf{f}\right)\overset{\mathbb{C}_{q}^{d}}{=}\sum_{\mathbf{t}\in\hat{\mathbb{Z}}_{p}^{r}}\hat{\mathbf{f}}\left(-\mathbf{t}\right)\hat{\mathcal{M}}\left(\mathbf{t}\right)\overset{\mathbb{C}_{q}^{d}}{=}\sum_{\mathbf{t}\in\hat{\mathbb{Z}}_{p}^{r}}\hat{\mathcal{M}}\left(\mathbf{t}\right)\hat{\mathbf{f}}\left(-\mathbf{t}\right) \end{equation} where the order of juxtaposition of the $d$-tuples $\hat{\mathbf{f}}$ and $\hat{\mathcal{M}}=\hat{\chi}$ is irrelevant because the juxtaposition signifies \emph{entry-wise} multiplication\textemdash a commutative operation. \end{defn} \begin{defn} Given a choice of Fourier transform $\hat{\chi}$ for $\chi\in\tilde{L}^{1}\left(\mathcal{F}^{d}\right)$, we write: \begin{equation} \tilde{\chi}_{N}\left(\mathbf{z}\right)\overset{\textrm{def}}{=}\sum_{\left\Vert \mathbf{t}\right\Vert _{p}\leq p^{N}}\hat{\chi}\left(\mathbf{t}\right)e^{2\pi i\left\{ \mathbf{t}\mathbf{z}\right\} _{p}},\textrm{ }\forall\mathbf{z}\in\mathbb{Z}_{p}^{r}\label{eq:MD Definition of Chi_N twiddle} \end{equation} to denote the \textbf{$N$th partial sum of the Fourier series generated by $\hat{\chi}$}. \end{defn} \vphantom{} Just like with the one-dimensional case, Fourier transforms of quasi-integrable functions are only unique modulo the Fourier-Stieltjes transform of a degenerate thick measure. \begin{prop} Let $\chi\in\tilde{L}^{1}\left(\mathcal{F}^{d}\right)$. The quasi-integral of $\chi$ is unique modulo $M_{\textrm{dgen}}\left(\mathcal{F}^{d}\right)$. That is, if two thick measures $\mathcal{M}$ and $\mathcal{N}$ are both quasi-integrals of $\chi$, then the thick measure $\mathcal{M}-\mathcal{N}$ is $\mathcal{F}$-degenerate\index{thick measure!mathcal{F}-degenerate@$\mathcal{F}$-degenerate}\index{mathcal{F}-@$\mathcal{F}$-!degenerate}. Equivalently, we have an isomorphism of $\overline{\mathbb{Q}}$-linear spaces: \begin{equation} \tilde{L}^{1}\left(\mathcal{F}^{d}\right)\cong M_{\textrm{rise}}\left(\mathcal{F}^{d}\right)/M_{\textrm{dgen}}\left(\mathcal{F}^{d}\right)\label{eq:MD L1 twiddle isomorphism to M alg / M dgen} \end{equation} where the isomorphism associates a given $\chi\in\tilde{L}^{1}\left(\mathcal{F}^{d}\right)$ to the equivalence class of $\mathcal{F}$-rising thick measures which are quasi-integrals of $\chi$. \end{prop} Proof: Let $\mathcal{F}$, $\chi$, $\mathcal{M}$, and $\mathcal{N}$ be as given. Then, by the definition of what it means to be a quasi-integral of $\chi$, for the thick vector-type measure $\mathcal{P}\overset{\textrm{def}}{=}\mathcal{M}-\mathcal{N}$, we have that $\tilde{\mathcal{P}}_{N}\left(\mathbf{z}\right)$ tends to $\mathbf{0}$ in $K_{\mathbf{z}}^{d}$ for all $\mathbf{z}\in U_{\mathcal{F}}$. Hence, $\mathcal{P}=\mathcal{M}-\mathcal{N}$ is indeed $\mathcal{F}$-degenerate. Q.E.D. \begin{prop} Let $\chi\in\tilde{L}^{1}\left(\mathcal{F}^{d}\right)$. Then, for any Fourier transform $\hat{\chi}$ of $\chi$, the difference $\chi_{N}-\tilde{\chi}_{N}$ (where $\chi_{N}$ is the $N$th truncation of $\chi$) $\mathcal{F}$-converges to $\mathbf{0}$. \end{prop} Proof: Immediate from the definitions of $\hat{\chi}$, $\tilde{\chi}_{N}$, and $\chi_{N}$. Q.E.D. \begin{rem} Note that all of the main unresolved issues regarding one-dimensional quasi-integrable functions (convolutions, a criterion for determining non-quasi-integrability, etc.) also apply to the multi-dimensional case. \end{rem} \subsection{\label{subsec:5.4.3 -adic-Wiener-Tauberian}$\left(p,q\right)$-adic Wiener Tauberian Theorems Revisited} Here, we quickly establish the multi-dimensional analogues of the results of Subsection \ref{subsec:3.3.7 -adic-Wiener-Tauberian}. \begin{defn} For \nomenclature{$\tau_{\mathbf{s}}$}{Multi-dimensional translation operator}$\mathbf{s}\in\hat{\mathbb{Z}}_{p}^{r}$, and for a thick $\left(p,q\right)$-adic measure $\mathcal{M}$ with Fourier-Stieltjes transform $\hat{\mathcal{M}}$, we write: \begin{equation} \tau_{\mathbf{s}}\left\{ \hat{\mathcal{M}}\right\} \left(\mathbf{t}\right)\overset{\textrm{def}}{=}\hat{\mathcal{M}}_{\mathbf{t}+\mathbf{s}},\textrm{ }\forall\mathbf{t}\in\hat{\mathbb{Z}}_{p}^{r}\label{eq:definition of the translate of M hat} \end{equation} We also write: \begin{equation} \tau_{\mathbf{s}}\left\{ \hat{\mathbf{f}}\right\} \left(\mathbf{t}\right)\overset{\textrm{def}}{=}\hat{\mathbf{f}}\left(\mathbf{t}+\mathbf{s}\right),\textrm{ }\forall\mathbf{t}\in\hat{\mathbb{Z}}_{p}^{r}\label{eq:MD Definition of the translate of f hat} \end{equation} \end{defn} \begin{defn} We write $\left(\mathbb{C}_{q}^{d}\right)^{\times}$\nomenclature{$\left(\mathbb{C}_{q}^{d}\right)^{\times}$}{$d\times1$ vectors whose entries are non-zero $q$-adic complex numbers } to denote the set of all $d\times1$ vectors whose entries are non-zero $q$-adic complex numbers. We make $\left(\mathbb{C}_{q}^{d}\right)^{\times}$ an abelian group with respect to the operation of entry-wise multiplication; i.e.: \begin{equation} \left(\begin{array}{c} \mathfrak{a}_{1}\\ \mathfrak{a}_{2} \end{array}\right)\left(\begin{array}{c} \mathfrak{b}_{1}\\ \mathfrak{b}_{2} \end{array}\right)=\left(\begin{array}{c} \mathfrak{a}_{1}\mathfrak{b}_{1}\\ \mathfrak{a}_{2}\mathfrak{b}_{2} \end{array}\right) \end{equation} We write \nomenclature{$\mathbf{1}$}{$d\times 1$ vector whose every entry is $1$}$\mathbf{1}$ to denote the identity element of $\left(\mathbb{C}_{q}^{d}\right)^{\times}$, the $d\times1$ vector whose entries are all $1$s. We also write $\vec{\mathbf{1}}_{\mathbf{0}}\left(\mathbf{t}\right)$\nomenclature{$\vec{\mathbf{1}}_{\mathbf{0}}\left(\mathbf{t}\right)$}{function which is $\mathbf{1}$ when $\mathbf{t}=\mathbf{0}$ and which is $\mathbf{0}$ otherwise.} to denote the function $\hat{\mathbb{Z}}_{p}^{r}\rightarrow\mathbb{C}_{q}^{d}$ which is equal to $\mathbf{1}$ when $\mathbf{t}=\mathbf{0}$ and which is equal to $\mathbf{0}$ for all other $\mathbf{t}$. Note that for any $\hat{\mathbf{f}}:\hat{\mathbb{Z}}_{p}^{r}\rightarrow\mathbb{C}_{q}^{d}$: \begin{equation} \left(\hat{\mathbf{f}}*\vec{\mathbf{1}}_{\mathbf{0}}\right)\left(\mathbf{t}\right)=\sum_{\mathbf{s}\in\hat{\mathbb{Z}}_{p}^{r}}\hat{\mathbf{f}}\left(\mathbf{t}-\mathbf{s}\right)\vec{\mathbf{1}}_{\mathbf{0}}\left(\mathbf{s}\right)=\hat{\mathbf{f}}\left(\mathbf{t}\right),\textrm{ }\forall\mathbf{t}\in\hat{\mathbb{Z}}_{p}^{r} \end{equation} so $\vec{\mathbf{1}}_{\mathbf{0}}\left(\mathbf{t}\right)$ is the identity element with respect to the convolution operation on $B\left(\hat{\mathbb{Z}}_{p}^{r},\mathbb{C}_{q}^{d}\right)$. Incidentally, this also proves that the constant function $\mathbf{z}\mapsto\mathbf{1}$ and the function $\mathbf{t}\mapsto\vec{\mathbf{1}}_{\mathbf{0}}\left(\mathbf{t}\right)$ are a Fourier transform pair. More generally, we write $\vec{\mathbf{1}}_{\mathbf{s}}\left(\mathbf{t}\right)$ to denote $\vec{\mathbf{1}}_{\mathbf{0}}\left(\mathbf{t}-\mathbf{s}\right)$, which is $\mathbf{1}$ for $\mathbf{t}\overset{\mathbf{1}}{\equiv}\mathbf{s}$ and $\mathbf{0}$ otherwise. The reason for introducing the vector notation is to avoid confusing the vector-valued indicator function for the point $\left\{ \mathbf{s}\right\} $ ($\vec{\mathbf{1}}_{\mathbf{s}}\left(\mathbf{t}\right)$) and the \emph{scalar}-valued indicator function for the point $\left\{ \mathbf{s}\right\} $ ($\mathbf{1}_{\mathbf{s}}\left(\mathbf{t}\right)$). \end{defn} \vphantom{} As in the one-dimensional case, we establish two WTTs: one for continuous functions, and another for thick measures. \begin{thm}[\textbf{Multi-Dimensional Wiener Tauberian Theorem for Continuous $\left(p,q\right)$-adic Functions}] \label{thm:MD pq-adic WTT for continuous functions}Let\index{Wiener!Tauberian Theorem!left(p,qright)-adic@$\left(p,q\right)$-adic} $\mathbf{f}=\left(f_{1}\left(\mathbf{z}\right),\ldots,f_{d}\left(\mathbf{z}\right)\right)\in C\left(\mathbb{Z}_{p}^{r},\mathbb{C}_{q}^{d}\right)$, and let: \begin{equation} \mathbf{f}^{-1}\left(\mathbf{z}\right)=\left(\frac{1}{f_{1}\left(\mathbf{z}\right)},\ldots,\frac{1}{f_{d}\left(\mathbf{z}\right)}\right) \end{equation} denote the multiplicative inverse\footnote{If $\mathbf{F}$ is matrix-valued, then $\mathbf{F}^{-1}$ is the matrix inverse; if $\mathbf{F}$ is scalar-valued, then $\mathbf{F}^{-1}$ is the reciprocal of $\mathbf{F}$; if $\mathbf{F}$ is vector-valued, then $\mathbf{F}^{-1}$ is the vector whose entries are the reciprocals of the entries of $\mathbf{F}$.} of $\mathbf{f}$, if it exists. Then, the following are equivalent: \vphantom{} I. $\mathbf{f}^{-1}$ exists and is an element of $C\left(\mathbb{Z}_{p}^{r},\mathbb{C}_{q}^{d}\right)$; \vphantom{} II. $\hat{\mathbf{f}}$ has a convolution inverse in $c_{0}\left(\hat{\mathbb{Z}}_{p}^{r},\mathbb{C}_{q}^{d}\right)$; \vphantom{} III. $\textrm{span}_{\mathbb{C}_{q}}\left\{ \tau_{\mathbf{s}}\left\{ \hat{\mathbf{f}}\right\} \left(\mathbf{t}\right):\mathbf{s}\in\hat{\mathbb{Z}}_{p}^{r}\right\} $ is dense in $c_{0}\left(\hat{\mathbb{Z}}_{p}^{r},\mathbb{C}_{q}^{d}\right)$; \vphantom{} IV. $\mathbf{f}\left(\mathbf{z}\right)\in\left(\mathbb{C}_{q}^{d}\right)^{\times}$ for all $\mathbf{z}\in\mathbb{Z}_{p}^{r}$, . \end{thm} Proof: \textbullet{} ($\textrm{I}\Rightarrow\textrm{II}$) Suppose $\mathbf{f}^{-1}$ exists and is continuous. Because the $\left(p,q_{H}\right)$-adic Fourier transform isometrically and isomorphically maps the Banach algebra $C\left(\mathbb{Z}_{p}^{r},\mathbb{C}_{q}^{d}\right)$ onto the Banach algebra $c_{0}\left(\hat{\mathbb{Z}}_{p}^{r},\mathbb{C}_{q}^{d}\right)$, we can write: \begin{align*} \mathbf{f}\left(\mathbf{z}\right)\cdot\mathbf{f}^{-1}\left(\mathbf{z}\right) & =\mathbf{1},\textrm{ }\forall\mathbf{z}\in\mathbb{Z}_{p}^{r}\\ \left(\textrm{Fourier transform}\right); & \Updownarrow\\ \left(\hat{\mathbf{f}}*\widehat{\mathbf{f}^{-1}}\right)\left(\mathbf{t}\right) & =\vec{\mathbf{1}}_{\mathbf{0}}\left(\mathbf{t}\right),\textrm{ }\forall\mathbf{t}\in\hat{\mathbb{Z}}_{p}^{r} \end{align*} where both $\hat{\mathbf{f}}$ and $\widehat{\mathbf{f}^{-1}}$ are in $c_{0}$. This shows that $\widehat{\mathbf{f}^{-1}}$ is $\hat{\mathbf{f}}^{-1}$\textemdash the convolution inverse of $\hat{\mathbf{f}}$. \vphantom{} \textbullet{} ($\textrm{II}\Rightarrow\textrm{III}$) Suppose $\hat{\mathbf{f}}$ has a convolution inverse $\hat{\mathbf{f}}^{-1}\in c_{0}\left(\hat{\mathbb{Z}}_{p}^{r},\mathbb{C}_{q}^{d}\right)$. Then, letting $\hat{\mathbf{g}}\in c_{0}\left(\hat{\mathbb{Z}}_{p}^{r},\mathbb{C}_{q}^{d}\right)$ be arbitrary, we can write: \begin{equation} \left(\hat{\mathbf{f}}*\left(\hat{\mathbf{f}}^{-1}*\hat{\mathbf{g}}\right)\right)\left(\mathbf{t}\right)=\left(\left(\hat{\mathbf{f}}*\hat{\mathbf{f}}^{-1}\right)*\hat{\mathbf{g}}\right)\left(\mathbf{t}\right)=\left(\mathbf{1}_{\mathbf{0}}*\hat{\mathbf{g}}\right)\left(\mathbf{t}\right)=\hat{\mathbf{g}}\left(\mathbf{t}\right),\textrm{ }\forall\mathbf{t}\in\hat{\mathbb{Z}}_{p}^{r} \end{equation} Letting $\hat{\mathbf{h}}$ denote $\hat{\mathbf{f}}^{-1}*\hat{\mathbf{g}}$, we then have: \begin{equation} \hat{\mathbf{g}}\left(\mathbf{t}\right)=\left(\hat{\mathbf{f}}*\hat{\mathbf{h}}\right)\left(\mathbf{t}\right)\overset{\mathbb{C}_{q}^{d}}{=}\lim_{N\rightarrow\infty}\sum_{\left\Vert \mathbf{s}\right\Vert _{p}\leq p^{N}}\hat{\mathbf{h}}\left(\mathbf{s}\right)\hat{\mathbf{f}}\left(\mathbf{t}-\mathbf{s}\right) \end{equation} Since $\hat{\mathbf{f}}$ and $\hat{\mathbf{h}}$ are in $c_{0}$, note that: \begin{align*} \sup_{\mathbf{t}\in\hat{\mathbb{Z}}_{p}^{r}}\left\Vert \sum_{\mathbf{s}\in\hat{\mathbb{Z}}_{p}^{r}}\hat{\mathbf{h}}\left(\mathbf{s}\right)\hat{\mathbf{f}}\left(\mathbf{t}-\mathbf{s}\right)-\sum_{\left\Vert \mathbf{s}\right\Vert _{p}\leq p^{N}}\hat{\mathbf{h}}\left(\mathbf{s}\right)\hat{\mathbf{f}}\left(\mathbf{t}-\mathbf{s}\right)\right\Vert _{q} & \leq\sup_{\mathbf{t}\in\hat{\mathbb{Z}}_{p}^{r}}\sup_{\left\Vert \mathbf{s}\right\Vert _{p}>p^{N}}\left\Vert \hat{\mathbf{h}}\left(\mathbf{s}\right)\hat{\mathbf{f}}\left(\mathbf{t}-\mathbf{s}\right)\right\Vert _{q}\\ \left(\left\Vert \hat{\mathbf{f}}\right\Vert _{p,q}<\infty\right); & \leq\sup_{\left\Vert \mathbf{s}\right\Vert _{p}>p^{N}}\left\Vert \hat{\mathbf{h}}\left(\mathbf{s}\right)\right\Vert _{q} \end{align*} and hence: \[ \lim_{N\rightarrow\infty}\sup_{\mathbf{t}\in\hat{\mathbb{Z}}_{p}^{r}}\left\Vert \sum_{\mathbf{s}\in\hat{\mathbb{Z}}_{p}^{r}}\hat{\mathbf{h}}\left(\mathbf{s}\right)\hat{\mathbf{f}}\left(\mathbf{t}-\mathbf{s}\right)-\sum_{\left\Vert \mathbf{s}\right\Vert _{p}\leq p^{N}}\hat{\mathbf{h}}\left(\mathbf{s}\right)\hat{\mathbf{f}}\left(\mathbf{t}-\mathbf{s}\right)\right\Vert _{q} \] is equal to $\lim_{N\rightarrow\infty}\sup_{\left\Vert \mathbf{s}\right\Vert _{p}>p^{N}}\left\Vert \hat{\mathbf{h}}\left(\mathbf{s}\right)\right\Vert _{q}$, which tends to $0$. This shows that the $q$-adic convergence of $\sum_{\left\Vert \mathbf{s}\right\Vert _{p}>p^{N}}\hat{\mathbf{h}}\left(\mathbf{s}\right)\hat{\mathbf{f}}\left(\mathbf{t}-\mathbf{s}\right)$ to $\hat{\mathbf{g}}\left(\mathbf{t}\right)=\sum_{\mathbf{s}\in\hat{\mathbb{Z}}_{p}^{r}}\hat{\mathbf{h}}\left(\mathbf{s}\right)\hat{\mathbf{f}}\left(\mathbf{t}-\mathbf{s}\right)$ is uniform in $\mathbf{t}$. Hence: \begin{equation} \lim_{N\rightarrow\infty}\sup_{\mathbf{t}\in\hat{\mathbb{Z}}_{p}^{r}}\left\Vert \hat{\mathbf{g}}\left(\mathbf{t}\right)-\sum_{\left\Vert \mathbf{s}\right\Vert _{p}\leq p^{N}}\hat{\mathbf{h}}\left(\mathbf{s}\right)\hat{\mathbf{f}}\left(\mathbf{t}-\mathbf{s}\right)\right\Vert _{q}=0 \end{equation} which is precisely the definition of what it means for the sequence: \begin{equation} \left\{ \sum_{\left\Vert \mathbf{s}\right\Vert _{p}\leq p^{N}}\hat{\mathbf{h}}\left(\mathbf{s}\right)\hat{\mathbf{f}}\left(\mathbf{t}-\mathbf{s}\right)\right\} _{N\geq0} \end{equation} to converge in $c_{0}\left(\hat{\mathbb{Z}}_{p}^{r},\mathbb{C}_{q}^{d}\right)$ to $\hat{\mathbf{g}}$. Because our sequence converging to the arbitrary $\hat{\mathbf{g}}\in c_{0}\left(\hat{\mathbb{Z}}_{p}^{r},\mathbb{C}_{q}^{d}\right)$ is an element of: \begin{equation} \textrm{span}_{\mathbb{C}_{q}}\left\{ \tau_{\mathbf{s}}\left\{ \hat{\mathbf{f}}\right\} \left(\mathbf{t}\right):\mathbf{s}\in\hat{\mathbb{Z}}_{p}^{r}\right\} \end{equation} we have proven the density of $\textrm{span}_{\mathbb{C}_{q}}\left\{ \tau_{\mathbf{s}}\left\{ \hat{\mathbf{f}}\right\} \left(\mathbf{t}\right):\mathbf{s}\in\hat{\mathbb{Z}}_{p}^{r}\right\} $ in $c_{0}\left(\hat{\mathbb{Z}}_{p}^{r},\mathbb{C}_{q}^{d}\right)$. \vphantom{} \textbullet{} ($\textrm{III}\Rightarrow\textrm{IV}$) Suppose $\textrm{span}_{\mathbb{C}_{q}}\left\{ \tau_{\mathbf{s}}\left\{ \hat{\mathbf{f}}\right\} \left(\mathbf{t}\right):\mathbf{s}\in\hat{\mathbb{Z}}_{p}^{r}\right\} $ is dense in $c_{0}\left(\hat{\mathbb{Z}}_{p}^{r},\mathbb{C}_{q}^{d}\right)$. Now, let $\epsilon>0$. Because $\vec{\mathbf{1}}_{\mathbf{0}}\left(\mathbf{t}\right)\in c_{0}\left(\hat{\mathbb{Z}}_{p}^{r},\mathbb{C}_{q}^{d}\right)$, the assumed density of the span of translates guarantees the existence of constant vectors $\mathbf{c}_{1},\ldots,\mathbf{c}_{M}\in\mathbb{C}_{q}^{d}$ and points $\mathbf{t}_{1},\ldots,\mathbf{t}_{M}\in\hat{\mathbb{Z}}_{p}^{r}$ so that: \begin{equation} \sup_{\mathbf{t}\in\hat{\mathbb{Z}}_{p}^{r}}\left\Vert \vec{\mathbf{1}}_{\mathbf{0}}\left(\mathbf{t}\right)-\sum_{m=1}^{M}\mathbf{c}_{m}\hat{\mathbf{f}}\left(\mathbf{t}-\mathbf{t}_{m}\right)\right\Vert _{q}<\epsilon \end{equation} Here, note that $\mathbf{c}_{m}\hat{\mathbf{f}}\left(\mathbf{t}-\mathbf{t}_{m}\right)$ is the entry-wise product of the $d\times1$ vectors $\mathbf{c}_{m}$ and $\hat{\mathbf{f}}\left(\mathbf{t}-\mathbf{t}_{m}\right)$. Next, letting $N\geq\max\left\{ -v_{p}\left(\mathbf{t}_{1}\right),\ldots,-v_{p}\left(\mathbf{t}_{M}\right)\right\} $ be arbitrary\footnote{Note that this choice of $N$ is larger than the negative $p$-adic valuations of all of the entries of all of the $\mathbf{t}_{m}$s.}, we have that for all $m$, the map $\mathbf{t}\mapsto\mathbf{t}+\mathbf{t}_{m}$ is a bijection of $\left\{ \mathbf{t}\in\hat{\mathbb{Z}}_{p}^{r}:\left\Vert \mathbf{t}\right\Vert _{p}\leq p^{N}\right\} $. Consequently: \begin{align*} \sum_{\left\Vert \mathbf{t}\right\Vert _{p}\leq p^{N}}\left(\sum_{m=1}^{M}\mathbf{c}_{m}\hat{\mathbf{f}}\left(\mathbf{t}-\mathbf{t}_{m}\right)\right)e^{2\pi i\left\{ \mathbf{t}\mathbf{z}\right\} _{p}} & =\sum_{m=1}^{M}\mathbf{c}_{m}\sum_{\left\Vert \mathbf{t}\right\Vert _{p}\leq p^{N}}\hat{\mathbf{f}}\left(\mathbf{t}\right)e^{2\pi i\left\{ \left(\mathbf{t}+\mathbf{t}_{m}\right)\mathbf{z}\right\} _{p}}\\ & =\left(\sum_{m=1}^{M}\mathbf{c}_{m}e^{2\pi i\left\{ \mathbf{t}_{m}\mathbf{z}\right\} _{p}}\right)\sum_{\left\Vert \mathbf{t}\right\Vert _{p}\leq p^{N}}\hat{\mathbf{f}}\left(\mathbf{t}\right)e^{2\pi i\left\{ \mathbf{t}\mathbf{z}\right\} _{p}} \end{align*} Since $N$ was arbitrary, we may let it tend to $\infty$. This gives: \begin{equation} \lim_{N\rightarrow\infty}\sum_{\left\Vert \mathbf{t}\right\Vert _{p}\leq p^{N}}\left(\sum_{m=1}^{M}\mathbf{c}_{m}\hat{\mathbf{f}}\left(\mathbf{t}-\mathbf{t}_{m}\right)\right)e^{2\pi i\left\{ \mathbf{t}\mathbf{z}\right\} _{p}}\overset{\mathbb{C}_{q}^{d}}{=}\mathbf{g}_{m}\left(\mathbf{z}\right)\mathbf{f}\left(\mathbf{z}\right) \end{equation} where $\mathbf{g}_{m}=\left(g_{m,1},\ldots,g_{m,d}\right):\mathbb{Z}_{p}^{r}\rightarrow\mathbb{C}_{q}^{d}$ is defined by: \begin{equation} \mathbf{g}_{m}\left(\mathbf{z}\right)\overset{\textrm{def}}{=}\sum_{m=1}^{M}\mathbf{c}_{m}e^{2\pi i\left\{ \mathbf{t}_{m}\mathbf{z}\right\} _{p}},\textrm{ }\forall\mathbf{z}\in\mathbb{Z}_{p}^{r}\label{eq:MD Definition of G_m} \end{equation} Moreover, the convergence of the $N$-limit is uniform in $\mathbf{z}$. Consequently: \begin{align*} \left\Vert \mathbf{1}-\mathbf{g}_{m}\left(\mathbf{z}\right)\mathbf{f}\left(\mathbf{z}\right)\right\Vert _{q} & \overset{\mathbb{R}}{=}\lim_{N\rightarrow\infty}\left\Vert \sum_{\left\Vert \mathbf{t}\right\Vert _{p}\leq p^{N}}\left(\vec{\mathbf{1}}_{\mathbf{0}}\left(\mathbf{t}\right)-\sum_{m=1}^{M}\mathbf{c}_{m}\hat{\mathbf{f}}\left(\mathbf{t}-\mathbf{t}_{m}\right)\right)e^{2\pi i\left\{ \mathbf{t}\mathbf{z}\right\} _{p}}\right\Vert _{q}\\ & \leq\sup_{\mathbf{t}\in\hat{\mathbb{Z}}_{p}^{r}}\left\Vert \vec{\mathbf{1}}_{\mathbf{0}}\left(\mathbf{t}\right)-\sum_{m=1}^{M}\mathbf{c}_{m}\hat{\mathbf{f}}\left(\mathbf{t}-\mathbf{t}_{m}\right)\right\Vert _{q}\\ & <\epsilon \end{align*} for all $\mathbf{z}\in\mathbb{Z}_{p}^{r}$. Finally, by way of contradiction, suppose there is an $\ell\in\left\{ 1,\ldots,d\right\} $ so that $f_{\ell}\left(\mathbf{z}_{0}\right)=0$. Then, the $\ell$th entry of the $d\times1$ vector $\mathbf{1}-\mathbf{g}_{m}\left(\mathbf{z}_{0}\right)\mathbf{f}\left(\mathbf{z}_{0}\right)$ is: \[ 1-g_{m,\ell}\left(\mathbf{z}_{0}\right)f_{\ell}\left(\mathbf{z}_{0}\right)=1-g_{m,\ell}\left(\mathbf{z}_{0}\right)\cdot0=1 \] Because $\left\Vert \mathbf{1}-\mathbf{g}_{m}\left(\mathbf{z}_{0}\right)\mathbf{f}\left(\mathbf{z}_{0}\right)\right\Vert _{q}\geq\left\Vert 1-g_{m,k}\left(\mathbf{z}_{0}\right)f_{k}\left(\mathbf{z}_{0}\right)\right\Vert _{q}$ for all $k\in\left\{ 1,\ldots,d\right\} $, we then have: \begin{equation} \epsilon>\left\Vert \mathbf{1}-\mathbf{g}_{m}\left(\mathbf{z}_{0}\right)\mathbf{f}\left(\mathbf{z}_{0}\right)\right\Vert _{q}\geq\left\Vert 1-g_{m,\ell}\left(\mathbf{z}_{0}\right)\cdot0\right\Vert _{q}=1 \end{equation} However, $\epsilon$ was given to be in $\left(0,1\right)$. This is a contradiction! As such, the entries of $\mathbf{f}$ cannot have any zeroes whenever the span of $\hat{\mathbf{f}}$'s translates are dense in $c_{0}\left(\hat{\mathbb{Z}}_{p}^{r},\mathbb{C}_{q}^{d}\right)$. \vphantom{} \textbullet{} ($\textrm{IV}\Rightarrow\textrm{I}$) Suppose $\mathbf{f}\left(\mathbf{z}\right)\in\left(\mathbb{C}_{q}^{d}\right)^{\times}$ for all $\mathbf{z}\in\mathbb{Z}_{p}^{r}$; that is, suppose no entry of $\mathbf{f}\left(\mathbf{z}\right)$ is zero for any $\mathbf{z}\in\mathbb{Z}_{p}^{r}$. Because the map which sends $\mathbf{y}=\left(\mathfrak{y}_{1},\ldots,\mathfrak{y}_{d}\right)\in\left(\mathbb{C}_{q}^{d}\right)^{\times}$ to: \[ \frac{1}{\mathbf{y}}=\left(\frac{1}{\mathfrak{y}_{1}},\ldots,\frac{1}{\mathfrak{y}_{d}}\right)\in\left(\mathbb{C}_{q}^{d}\right)^{\times} \] is continuous on $\left(\mathbb{C}_{q}^{d}\right)^{\times}$, the continuity of compositions of continuous maps tells us that we can show $1/\mathbf{f}$ is continuous by establishing that show that $\inf_{\mathbf{z}\in\mathbb{Z}_{p}^{r}}\left|f_{k}\left(\mathbf{z}\right)\right|_{q}>0$ for each $k\in\left\{ 1,\ldots,d\right\} $. So, by way of contradiction, suppose there is a $k\in\left\{ 1,\ldots,d\right\} $ so that $\inf_{\mathbf{z}\in\mathbb{Z}_{p}^{r}}\left|f_{k}\left(\mathbf{z}\right)\right|_{q}=0$. Then, there exists a sequence $\left\{ \mathbf{z}_{n}\right\} _{n\geq0}\subseteq\mathbb{Z}_{p}^{r}$ so that $\lim_{n\rightarrow\infty}\left|f_{k}\left(\mathbf{z}_{n}\right)\right|_{q}=0$. By the compactness of $\mathbb{Z}_{p}^{r}$, the $\mathbf{z}_{n}$s have a subsequence $\mathbf{z}_{n_{\ell}}$ which converges in $\mathbb{Z}_{p}^{r}$ to some limit $\mathbf{z}_{\infty}\in\mathbb{Z}_{p}^{r}$. Because $\mathbf{f}$ is continuous, so is $f_{k}$; this forces $f_{k}\left(\mathbf{z}_{\infty}\right)=0$. Hence, $\mathbf{z}_{\infty}$ is an element of $\mathbb{Z}_{p}^{r}$ for which $\mathbf{f}\left(\mathbf{z}_{\infty}\right)$ has a non-zero entry\textemdash which contradicts our given hypothesis on $\mathbf{f}$. Thus, if none of the entries of $\mathbf{f}$ are ever zero on $\mathbb{Z}_{p}^{r}$, $\left|f_{k}\left(\mathbf{z}\right)\right|_{q}$ is bounded away from zero for all $k$, which then shows that $1/\mathbf{f}$ is $\left(p,q\right)$-adically continuous. Q.E.D. \begin{thm}[\textbf{Wiener Tauberian Theorem for Vector-Type Thick $\left(p,q\right)$-adic Measures}] \label{thm:MD WTT for pq adic vector type thick measures}Let $\mathcal{M}\in M\left(\mathbb{Z}_{p}^{r},\mathbb{C}_{q}^{d}\right)$ be a $\left(p,q\right)$-adic thick measure of vector type, with Fourier-Stieltjes transform $\hat{\mathcal{M}}:\hat{\mathbb{Z}}_{p}^{r}\rightarrow\mathbb{C}_{q}^{d}$. Then, $\textrm{span}_{\mathbb{C}_{q}}\left\{ \tau_{\mathbf{s}}\left\{ \hat{\mathcal{M}}\right\} \left(\mathbf{t}\right):\mathbf{s}\in\hat{\mathbb{Z}}_{p}^{r}\right\} $ is dense in $c_{0}\left(\hat{\mathbb{Z}}_{p}^{r},\mathbb{C}_{q}^{d}\right)$ if and only if for any $\mathbf{z}\in\mathbb{Z}_{p}^{r}$ for which the limit $\lim_{N\rightarrow\infty}\tilde{\mathcal{M}}_{N}\left(\mathbf{z}\right)$ converges in $\mathbb{C}_{q}^{d}$, said limit is necessarily an element of $\left(\mathbb{C}_{q}^{d}\right)^{\times}$. \index{Wiener!Tauberian Theorem!left(p,qright)-adic@$\left(p,q\right)$-adic} \end{thm} \begin{rem} \textbf{WARNING \textendash{} }Like in the one-dimensional case (\textbf{Theorem \ref{thm:pq WTT for measures}}), for the proof of this theorem\textemdash and only this theorem\textemdash we will need to modify our notation for the $\left(p,q\right)$-adic norm. Instead of writing $\left\Vert \cdot\right\Vert _{p,q}$ to denote the supremum over $\hat{\mathbb{Z}}_{p}^{r}$ of a vector-valued function with entries in $\mathbb{C}_{q}$, we write that norm as $\left\Vert \cdot\right\Vert _{p^{\infty},q}$. \end{rem} Proof: I. Suppose the span of the translates of $\hat{\mathcal{M}}$ are dense. Like in the one-dimensional case, we let $\epsilon\in\left(0,1\right)$ and then, using the assumed density, choose constant vectors $\mathbf{c}_{m}\in\mathbb{C}_{q}^{d}$ and $\mathbf{t}_{m}\in\hat{\mathbb{Z}}_{p}^{r}$ so that: \begin{equation} \sup_{\mathbf{t}\in\hat{\mathbb{Z}}_{p}^{r}}\left\Vert \vec{\mathbf{1}}_{\mathbf{0}}\left(\mathbf{t}\right)-\sum_{m=1}^{M}\mathbf{c}_{m}\hat{\mathcal{M}}\left(\mathbf{t}-\mathbf{t}_{m}\right)\right\Vert _{q}<\epsilon \end{equation} When $N$ is sufficiently large, we obtain: \begin{align*} \left\Vert \mathbf{1}-\left(\sum_{m=1}^{M}\mathbf{c}_{m}e^{2\pi i\left\{ \mathbf{t}_{m}\mathbf{z}\right\} _{p}}\right)\tilde{\mathcal{M}}_{N}\left(\mathbf{z}\right)\right\Vert _{q} & \leq\max_{\left\Vert \mathbf{t}\right\Vert _{p}\leq p^{N}}\left\Vert \vec{\mathbf{1}}_{\mathbf{0}}\left(\mathbf{t}\right)-\sum_{m=1}^{M}\mathbf{c}_{m}\hat{\mathcal{M}}\left(\mathbf{t}-\mathbf{t}_{m}\right)\right\Vert _{q}\\ & \leq\sup_{\mathbf{t}\in\hat{\mathbb{Z}}_{p}^{r}}\left\Vert \vec{\mathbf{1}}_{\mathbf{0}}\left(\mathbf{t}\right)-\sum_{m=1}^{M}\mathbf{c}_{m}\hat{\mathcal{M}}\left(\mathbf{t}-\mathbf{t}_{m}\right)\right\Vert _{q}\\ & <\epsilon \end{align*} So, let $\mathbf{z}_{0}\in\mathbb{Z}_{p}^{r}$ be a point so that $\mathfrak{L}=\lim_{N\rightarrow\infty}\tilde{\mathcal{M}}_{N}\left(\mathbf{z}_{0}\right)$ converges in $\mathbb{C}_{q}^{d}$. Then, plugging $\mathbf{z}=\mathbf{z}_{0}$ in the above ends yields: \[ \epsilon>\lim_{N\rightarrow\infty}\left\Vert \mathbf{1}-\left(\sum_{m=1}^{M}\mathbf{c}_{m}e^{2\pi i\left\{ \mathbf{t}_{m}\mathbf{z}_{0}\right\} _{p}}\right)\tilde{\mathcal{M}}_{N}\left(\mathbf{z}_{0}\right)\right\Vert _{q}=\left\Vert \mathbf{1}-\left(\sum_{m=1}^{M}\mathbf{c}_{m}e^{2\pi i\left\{ \mathbf{t}_{m}\mathbf{z}_{0}\right\} _{p}}\right)\cdot\mathfrak{L}\right\Vert _{q} \] If $\mathfrak{L}$ had an entry which is $0$ (i.e., if $\mathfrak{L}\in\mathbb{C}_{q}^{d}\backslash\left(\mathbb{C}_{q}^{d}\right)^{\times}$), then we end up with $\epsilon>1$, which contradicts the fact that $\epsilon\in\left(0,1\right)$. Thus, if $\lim_{N\rightarrow\infty}\tilde{\mathcal{M}}_{N}\left(\mathbf{z}_{0}\right)$ converges in $\mathbb{C}_{q}^{d}$, it must converge to an element of $\left(\mathbb{C}_{q}^{d}\right)^{\times}$. \vphantom{} II. Let $\mathbf{z}_{0}\in\mathbb{Z}_{p}^{r}$ be such that $\tilde{\mathcal{M}}\left(\mathbf{z}_{0}\right)\in\mathbb{C}_{q}^{d}\backslash\left(\mathbb{C}_{q}^{d}\right)^{\times}$ and that $\tilde{\mathcal{M}}_{N}\left(\mathbf{z}_{0}\right)\rightarrow\tilde{\mathcal{M}}\left(\mathbf{z}_{0}\right)$ in $\mathbb{C}_{q}^{d}$ as $N\rightarrow\infty$; in an abuse of terminology, let us call this $\mathbf{z}_{0}$ a ``zero'' of $\tilde{\mathcal{M}}$. Next, by way of contradiction, suppose the span of the translates of $\hat{\mathcal{M}}$ is dense in $c_{0}\left(\hat{\mathbb{Z}}_{p}^{r},\mathbb{C}_{q}^{d}\right)$ in spite of the zero of $\tilde{\mathcal{M}}$ at $\mathbf{z}_{0}$. So, letting $\epsilon\in\left(0,1\right)$ be arbitrary, by the assumed density,, there exists a linear combination of translates of $\hat{\mathcal{M}}$ which approximates $\vec{\mathbf{1}}_{\mathbf{0}}$ in sup norm: \[ \sup_{\mathbf{t}\in\hat{\mathbb{Z}}_{p}^{r}}\left\Vert \vec{\mathbf{1}}_{\mathbf{0}}\left(\mathbf{t}\right)-\sum_{k=1}^{K}\mathbf{c}_{k}\hat{\mathcal{M}}\left(\mathbf{t}-\mathbf{t}_{k}\right)\right\Vert _{q}<\epsilon \] where the $\mathbf{c}_{k}$s and $\mathbf{t}_{k}$s are constants in $\mathbb{C}_{q}^{d}$ and $\hat{\mathbb{Z}}_{p}^{r}$, respectively. Writing: \begin{equation} \hat{\mathbf{h}}_{\epsilon}\left(\mathbf{t}\right)\overset{\textrm{def}}{=}\sum_{k=1}^{K}\mathbf{c}_{k}\vec{\mathbf{1}}_{\mathbf{t}_{k}}\left(\mathbf{t}\right)\label{eq:Definition of bold h_epsilon hat} \end{equation} we can represent our approximating linear combination as a convolution: \[ \left(\hat{\mathcal{M}}*\hat{\mathbf{h}}_{\epsilon}\right)\left(\mathbf{t}\right)=\sum_{\mathbf{u}\in\hat{\mathbb{Z}}_{p}^{r}}\hat{\mathcal{M}}\left(\mathbf{t}-\mathbf{u}\right)\sum_{k=1}^{K}\mathbf{c}_{k}\vec{\mathbf{1}}_{\mathbf{t}_{k}}\left(\mathbf{u}\right)=\sum_{k=1}^{K}\mathbf{c}_{k}\hat{\mathcal{M}}\left(\mathbf{t}-\mathbf{t}_{k}\right) \] and so: \begin{equation} \left\Vert \vec{\mathbf{1}}_{\mathbf{0}}-\hat{\mathcal{M}}*\hat{\mathbf{h}}_{\epsilon}\right\Vert _{p^{\infty},q}\overset{\textrm{def}}{=}\sup_{\mathbf{t}\in\hat{\mathbb{Z}}_{p}^{r}}\left\Vert \vec{\mathbf{1}}_{\mathbf{0}}\left(\mathbf{t}\right)-\left(\hat{\mathcal{M}}*\hat{\mathbf{h}}_{\epsilon}\right)\left(\mathbf{t}\right)\right\Vert _{q}<\epsilon\label{eq:MD Converse WTT - eq. 1} \end{equation} Before we proceed, exactly like in the one-dimensional case, it is \emph{vital }to note that we can and \emph{must} assume that $\hat{\mathbf{h}}_{\epsilon}$ is \emph{not identically zero}. In fact, we can assume this must hold for any $\epsilon\in\left(0,1\right)$. Indeed, if there was an $\epsilon\in\left(0,1\right)$ for which $\hat{\mathbf{h}}_{\epsilon}$ was identically zero, the supremum $\sup_{\mathbf{t}\in\hat{\mathbb{Z}}_{p}^{r}}\left\Vert \vec{\mathbf{1}}_{\mathbf{0}}\left(\mathbf{t}\right)-\left(\hat{\mathcal{M}}*\hat{\mathbf{h}}_{\epsilon}\right)\left(\mathbf{t}\right)\right\Vert _{q}$ would be equal to $1$, which is quite impossible, given that the supremum is, by construction, less than $\epsilon$. Exactly like before, we will obtain a contradiction by showing that we can convolve $\hat{\mathcal{M}}$ by two different functions; this first makes the resultant vector-valued function close to $\vec{\mathbf{1}}_{\mathbf{0}}$; the second makes one of the entries of the resultant vector-valued function close to $0$. Our close-to-zero convolving function is going to be: \begin{equation} \hat{\phi}_{N}\left(\mathbf{t}\right)\overset{\textrm{def}}{=}\vec{\mathbf{1}}_{\mathbf{0}}\left(p^{N}\mathbf{t}\right)e^{-2\pi i\left\{ \mathbf{t}\mathbf{z}_{0}\right\} _{p}}\label{eq:MD Definition of Phi_N hat} \end{equation} Note that we can then write: \begin{align*} \left(\hat{\mathcal{M}}*\hat{\phi}_{N}\right)\left(\mathbf{u}\right) & =\sum_{\mathbf{s}\in\hat{\mathbb{Z}}_{p}^{r}}\hat{\mathcal{M}}\left(\mathbf{u}-\mathbf{s}\right)\hat{\phi}_{N}\left(\mathbf{s}\right)\\ \left(\hat{\phi}_{N}\left(\mathbf{s}\right)=\mathbf{0},\textrm{ }\forall\left\Vert \mathbf{s}\right\Vert _{p}>p^{N}\right); & =\sum_{\left\Vert \mathbf{s}\right\Vert _{p}\leq p^{N}}\hat{\mathcal{M}}\left(\mathbf{u}-\mathbf{s}\right)\hat{\phi}_{N}\left(\mathbf{s}\right)\\ & =\sum_{\left\Vert \mathbf{s}\right\Vert _{p}\leq p^{N}}\hat{\mathcal{M}}\left(\mathbf{u}-\mathbf{s}\right)e^{-2\pi i\left\{ \mathbf{s}\mathbf{z}_{0}\right\} _{p}} \end{align*} Now, fixing $\mathbf{u}$ with $\left\Vert \mathbf{u}\right\Vert _{p}\leq p^{N}$, observe that the map $\mathbf{s}\mapsto\mathbf{u}-\mathbf{s}$ is a bijection of the set $\left\{ \mathbf{s}\in\hat{\mathbb{Z}}_{p}^{r}:\left\Vert \mathbf{s}\right\Vert _{p}\leq p^{N}\right\} $; equivalently, this map is a bijection whenever $N\geq-v_{p}\left(\mathbf{u}\right)$. So, picking $N\geq-v_{p}\left(\mathbf{u}\right)$, we can write: \begin{align*} \sum_{\left\Vert \mathbf{s}\right\Vert _{p}\leq p^{N}}\hat{\mathcal{M}}\left(\mathbf{u}-\mathbf{s}\right)e^{-2\pi i\left\{ \mathbf{s}\mathbf{z}_{0}\right\} _{p}} & =\sum_{\left\Vert \mathbf{s}\right\Vert _{p}\leq p^{N}}\hat{\mathcal{M}}\left(\mathbf{s}\right)e^{-2\pi i\left\{ \left(\mathbf{u}-\mathbf{s}\right)\mathbf{z}_{0}\right\} _{p}}\\ & =e^{-2\pi i\left\{ \mathbf{u}\mathbf{z}_{0}\right\} _{p}}\sum_{\left\Vert \mathbf{s}\right\Vert _{p}\leq p^{N}}\hat{\mathcal{M}}\left(\mathbf{s}\right)e^{2\pi i\left\{ \mathbf{s}\mathbf{z}_{0}\right\} _{p}}\\ & =e^{-2\pi i\left\{ \mathbf{u}\mathbf{z}_{0}\right\} _{p}}\tilde{\mathcal{M}}_{N}\left(\mathbf{z}_{0}\right) \end{align*} Denoting the entries of $\tilde{\mathcal{M}}_{N}\left(\mathbf{z}_{0}\right)$ as $\tilde{\mathcal{M}}_{N}\left(\mathbf{z}_{0}\right)=\left(\tilde{\mathcal{M}}_{N,1}\left(\mathbf{z}_{0}\right),\ldots,\tilde{\mathcal{M}}_{N,d}\left(\mathbf{z}_{0}\right)\right)$, the given assumption about the zero at $\mathbf{z}_{0}$ tells us there is an $\ell\in\left\{ 1,\ldots,d\right\} $ so that $\tilde{\mathcal{M}}_{N}\left(\mathbf{z}_{0}\right)$'s $\ell$th entry ($\tilde{\mathcal{M}}_{N,\ell}\left(\mathbf{z}_{0}\right)$) converges to $0$ in\emph{ $\mathbb{C}_{q}$} as $N\rightarrow\infty$. Thus, for any $\epsilon^{\prime}>0$, there exists an $N_{\epsilon^{\prime}}$ so that $\left|\tilde{\mathcal{M}}_{N,\ell}\left(\mathbf{z}_{0}\right)\right|_{q}<\epsilon^{\prime}$ for all $N\geq N_{\epsilon^{\prime}}$. Next, let $\mathbf{e}_{\ell}$ denote the $\ell$th standard basis vector of $\mathbb{C}_{q}^{d}$. Then, for any $\hat{\mathbf{f}}=\left(\hat{f}_{1},\ldots,\hat{f}_{d}\right):\hat{\mathbb{Z}}_{p}^{r}\rightarrow\mathbb{C}_{q}^{d}$, the entry-wise product $\mathbf{e}_{\ell}\hat{\mathbf{f}}$ is: \begin{equation} \left(\mathbf{e}_{\ell}\hat{\mathbf{f}}\right)\left(\mathbf{t}\right)=\left(0,\ldots,0,\hat{f}_{\ell}\left(\mathbf{t}\right),0,\ldots,0\right)\label{eq:e_ell f hat} \end{equation} Moreover, since the convolution of two vector-valued functions on $\hat{\mathbb{Z}}_{p}^{r}$ is done entry-wise: \begin{align*} \left(\hat{\mathbf{f}}*\hat{\mathbf{g}}\right)\left(\mathbf{t}\right) & =\sum_{\mathbf{s}\in\hat{\mathbb{Z}}_{p}^{r}}\hat{\mathbf{f}}\left(\mathbf{t}-\mathbf{s}\right)\hat{\mathbf{g}}\left(\mathbf{s}\right)\\ & =\left(\sum_{\mathbf{s}\in\hat{\mathbb{Z}}_{p}^{r}}\hat{f}_{1}\left(\mathbf{t}-\mathbf{s}\right)\hat{g}_{1}\left(\mathbf{s}\right),\ldots,\sum_{\mathbf{s}\in\hat{\mathbb{Z}}_{p}^{r}}\hat{f}_{d}\left(\mathbf{t}-\mathbf{s}\right)\hat{g}_{d}\left(\mathbf{s}\right)\right) \end{align*} we obtain: \begin{equation} \left(\left(\mathbf{e}_{\ell}\hat{\mathbf{f}}\right)*\hat{\mathbf{g}}\right)\left(\mathbf{t}\right)=\left(\mathbf{e}_{\ell}\left(\hat{\mathbf{f}}*\hat{\mathbf{g}}\right)\right)\left(\mathbf{t}\right)=\left(\left(\hat{f}_{1}*\hat{g}_{1}\right)\left(\mathbf{t}\right),0,\ldots,0\right)\label{eq:e_ell f hat convolve with g hat} \end{equation} With this notation, we can write: \begin{align*} \left(\mathbf{e}_{\ell}\hat{\mathcal{M}}*\hat{\phi}_{N}\right)\left(\mathbf{u}\right) & =\mathbf{e}_{\ell}\left(\hat{\mathcal{M}}*\hat{\phi}_{N}\right)\left(\mathbf{u}\right)\\ & =\mathbf{e}_{\ell}\left(e^{-2\pi i\left\{ \mathbf{u}\mathbf{z}_{0}\right\} _{p}}\tilde{\mathcal{M}}_{N}\left(\mathbf{z}_{0}\right)\right)\\ & =e^{-2\pi i\left\{ \mathbf{u}\mathbf{z}_{0}\right\} _{p}}\left(\mathbf{e}_{\ell}\tilde{\mathcal{M}}_{N}\right)\left(\mathbf{z}_{0}\right)\\ & =\left(0,\ldots,0,e^{-2\pi i\left\{ \mathbf{u}\mathbf{z}_{0}\right\} _{p}}\tilde{\mathcal{M}}_{N,\ell}\left(\mathbf{z}_{0}\right),0,\ldots,0\right) \end{align*} Taking $q$-adic absolute values, we have thereby proven the following claim: \begin{claim} \label{claim:5.11}For any $\epsilon^{\prime}>0$, there is an $N_{\epsilon^{\prime}}\geq0$ (depending only on $\hat{\mathcal{M}}$, $\ell$ and $\epsilon^{\prime}$) so that, for any $\mathbf{u}\in\hat{\mathbb{Z}}_{p}$: \end{claim} \begin{equation} \left\Vert \left(\mathbf{e}_{\ell}\hat{\mathcal{M}}*\hat{\phi}_{N}\right)\left(\mathbf{u}\right)\right\Vert _{p^{\infty},q}<\epsilon^{\prime},\textrm{ }\forall N\geq\max\left\{ N_{\epsilon^{\prime}},-v_{p}\left(\mathbf{u}\right)\right\} \label{eq:MD WTT - First Claim} \end{equation} \vphantom{} For our next step, we restrict the support of $\hat{\mathbf{h}}_{\epsilon}$ by defining the function $\hat{\mathbf{h}}_{\epsilon,M}:\hat{\mathbb{Z}}_{p}^{r}\rightarrow\mathbb{C}_{q}^{d}$ as: \begin{equation} \hat{\mathbf{h}}_{\epsilon,M}\left(\mathbf{t}\right)\overset{\textrm{def}}{=}\vec{\mathbf{1}}_{\mathbf{0}}\left(p^{M}\mathbf{t}\right)\hat{\mathbf{h}}_{\epsilon}\left(\mathbf{t}\right)=\begin{cases} \hat{\mathbf{h}}_{\epsilon}\left(\mathbf{t}\right) & \textrm{if }\left\Vert \mathbf{t}\right\Vert _{p}\leq p^{M}\\ \mathbf{0} & \textrm{else} \end{cases}\label{eq:MD Definition of h epsilon M hat} \end{equation} Here $M$ is an arbitrary integer $\geq0$. Then: \begin{align*} \left(\hat{\mathcal{M}}*\hat{\mathbf{h}}_{\epsilon,M}\right)\left(\mathbf{t}\right) & =\sum_{\mathbf{u}\in\mathbb{Z}_{p}}\hat{\mathcal{M}}\left(\mathbf{t}-\mathbf{u}\right)\vec{\mathbf{1}}_{\mathbf{0}}\left(p^{M}\mathbf{u}\right)\sum_{k=1}^{K}\mathbf{c}_{k}\vec{\mathbf{1}}_{\mathbf{t}_{k}}\left(\mathbf{u}\right)\\ & =\sum_{\left\Vert \mathbf{u}\right\Vert _{p}\leq p^{M}}\hat{\mathcal{M}}\left(\mathbf{t}-\mathbf{u}\right)\sum_{k=1}^{K}\mathbf{c}_{k}\vec{\mathbf{1}}_{\mathbf{t}_{k}}\left(\mathbf{u}\right)\\ & =\sum_{k:\left\Vert \mathbf{t}_{k}\right\Vert _{p}\leq p^{M}}\mathbf{c}_{k}\hat{\mathcal{M}}\left(\mathbf{t}-\mathbf{t}_{k}\right) \end{align*} Since there are finitely many $\mathbf{t}_{k}$s, we can choose the non-negative integer: \begin{equation} M_{\epsilon}\overset{\textrm{choose}}{=}\max\left\{ -v_{p}\left(\mathbf{t}_{1}\right),\ldots,-v_{p}\left(\mathbf{t}_{K}\right)\right\} \label{eq:MD WTT - Choice for M_e} \end{equation} In particular, note that\emph{ $M_{\epsilon}$ depends only on $\mathbf{h}_{\epsilon}$ and $\epsilon$. }Then: \begin{equation} \left(\hat{\mathcal{M}}*\hat{\mathbf{h}}_{\epsilon,M}\right)\left(\mathbf{t}\right)=\sum_{k=1}^{K}\mathbf{c}_{k}\hat{\mathcal{M}}\left(\mathbf{t}-\mathbf{t}_{k}\right)=\left(\hat{\mathcal{M}}*\hat{\mathbf{h}}_{\epsilon}\right)\left(\mathbf{t}\right),\textrm{ }\forall M\geq M_{\epsilon}\label{eq:MD Effect of M bigger than M0} \end{equation} Next, for any $\hat{\mathbf{f}}:\hat{\mathbb{Z}}_{p}^{r}\rightarrow\mathbb{C}_{q}^{d}$ and any $n\in\mathbb{N}_{0}$, let us write: \nomenclature{$\left\Vert \hat{\mathbf{f}}\right\Vert _{p^{n},q}$}{ } \begin{equation} \left\Vert \hat{\mathbf{f}}\right\Vert _{p^{n},q}\overset{\textrm{def}}{=}\sup_{\left\Vert \mathbf{t}\right\Vert _{p}\leq p^{n}}\left\Vert \hat{\mathbf{f}}\left(\mathbf{t}\right)\right\Vert _{q}\label{eq:MD Definition of infinity,p^n norm} \end{equation} Then: \begin{align*} \left\Vert \vec{\mathbf{1}}_{\mathbf{0}}-\hat{\mathcal{M}}*\hat{\mathbf{h}}_{\epsilon,M}\right\Vert _{p^{M},q} & =\sup_{\left\Vert \mathbf{t}\right\Vert _{p}\leq p^{M}}\left\Vert \vec{\mathbf{1}}_{\mathbf{0}}\left(\mathbf{t}\right)-\left(\hat{\mathcal{M}}*\hat{\mathbf{h}}_{\epsilon,M}\right)\left(\mathbf{t}\right)\right\Vert _{q}\\ \left(\textrm{if }M\geq M_{\epsilon}\right); & =\sup_{\left\Vert \mathbf{t}\right\Vert _{p}\leq p^{M}}\left\Vert \vec{\mathbf{1}}_{\mathbf{0}}\left(\mathbf{t}\right)-\left(\hat{\mathcal{M}}*\hat{\mathbf{h}}_{\epsilon}\right)\left(\mathbf{t}\right)\right\Vert _{q}\\ & \leq\left\Vert \vec{\mathbf{1}}_{\mathbf{0}}-\left(\hat{\mathcal{M}}*\hat{\mathbf{h}}_{\epsilon}\right)\right\Vert _{p^{\infty},q}\\ & <\epsilon \end{align*} In summary, we have shown: \begin{claim} \label{claim:5.12}Let $\epsilon>0$ be arbitrary. Then, there exists an $M_{\epsilon}$ (depending only on $\hat{\mathcal{M}}$ and $\epsilon$ so that: \vphantom{} I. $\hat{\mathcal{M}}*\hat{\mathbf{h}}_{\epsilon,M}=\hat{\mathcal{M}}*\hat{\mathbf{h}}_{\epsilon},\textrm{ }\forall M\geq M_{\epsilon}$ \vphantom{} II. $\left\Vert \vec{\mathbf{1}}_{\mathbf{0}}-\hat{\mathcal{M}}*\hat{\mathbf{h}}_{\epsilon,M}\right\Vert _{p^{\infty},q}<\epsilon,\textrm{ }\forall M\geq M_{\epsilon}$ \vphantom{} III. $\left\Vert \vec{\mathbf{1}}_{\mathbf{0}}-\hat{\mathcal{M}}*\hat{\mathbf{h}}_{\epsilon,M}\right\Vert _{p^{M},q}<\epsilon,\textrm{ }\forall M\geq M_{\epsilon}$ \end{claim} \vphantom{} We now modify these estimates to take into account the norm $\left\Vert \cdot\right\Vert _{p^{m},q}$. \begin{claim} \label{claim:5.13}Let $\epsilon^{\prime}>0$ and $m\in\mathbb{N}_{1}$ be arbitrary. Then, there exists $N_{\epsilon^{\prime}}$ (depending only on $\epsilon^{\prime}$ and $\hat{\mathcal{M}}$) so that: \begin{equation} \left\Vert \left(\mathbf{e}_{\ell}\hat{\mathcal{M}}\right)*\hat{\phi}_{N}\right\Vert _{p^{m},q}<\epsilon^{\prime},\textrm{ }\forall N\geq\max\left\{ N_{\epsilon^{\prime}},m\right\} \label{eq:MD WTT - eq. 4} \end{equation} Proof of claim: For arbitrary $\epsilon^{\prime}>0$, \textbf{Claim \ref{claim:5.11}} tells us there is an $N_{\epsilon^{\prime}}$ with the stated dependencies so that, for any $\mathbf{u}\in\hat{\mathbb{Z}}_{p}^{r}$: \begin{equation} \left\Vert \left(\left(\mathbf{e}_{\ell}\hat{\mathcal{M}}\right)*\hat{\phi}_{N}\right)\left(\mathbf{u}\right)\right\Vert _{p^{\infty},q}<\epsilon^{\prime},\textrm{ }\forall N\geq\max\left\{ N_{\epsilon},-v\left(\mathbf{u}\right)\right\} \end{equation} So, letting $m\geq1$ be arbitrary, note that $\left\Vert \mathbf{u}\right\Vert _{p}\leq p^{m}$ implies $-v_{p}\left(\mathbf{u}\right)\leq m$. As such, by choosing $N\geq\max\left\{ N_{\epsilon},m\right\} $, we can make the result of \textbf{Claim \ref{claim:5.11}} hold for all $\left\Vert \mathbf{u}\right\Vert _{p}\leq p^{m}$: \begin{equation} \underbrace{\sup_{\left\Vert \mathbf{u}\right\Vert _{p}\leq p^{m}}\left\Vert \left(\left(\mathbf{e}_{\ell}\hat{\mathcal{M}}\right)*\hat{\phi}_{N}\right)\left(\mathbf{u}\right)\right\Vert _{q}}_{\left\Vert \left(\mathbf{e}_{\ell}\hat{\mathcal{M}}\right)*\hat{\phi}_{N}\right\Vert _{p^{m},q}}<\epsilon^{\prime},\textrm{ }\forall N\geq\max\left\{ N_{\epsilon},m\right\} \end{equation} This proves the claim. \end{claim} \vphantom{} We need two more estimates before we can wrap up the argument. \begin{claim} \label{claim:5.14}Let $M,N\in\mathbb{N}_{0}$ be arbitrary. Then, for any $\hat{\varphi},\hat{\mathbf{f}},\hat{\mathbf{g}}:\hat{\mathbb{Z}}_{p}^{r}\rightarrow\mathbb{C}_{q}^{d}$ satisfying: \vphantom{} i. $\hat{\varphi}\left(\mathbf{t}\right)=\mathbf{0}$ for all $\left\Vert \mathbf{t}\right\Vert _{p}>p^{N}$; \vphantom{} ii. $\left\Vert \hat{\varphi}\right\Vert _{p^{\infty},q}\leq1$; \vphantom{} iii. $\hat{\mathbf{g}}\left(\mathbf{t}\right)=\mathbf{0}$ for all $\left\Vert \mathbf{t}\right\Vert _{p}>p^{M}$; \vphantom{} the following estimates hold: \begin{equation} \left\Vert \hat{\varphi}*\hat{\mathbf{f}}\right\Vert _{p^{M},q}\leq\left\Vert \hat{\mathbf{f}}\right\Vert _{p^{\max\left\{ M,N\right\} },q}\label{eq:MD phi_N hat convolve f hat estimate} \end{equation} \begin{equation} \left\Vert \hat{\varphi}*\hat{\mathbf{f}}*\hat{\mathbf{g}}\right\Vert _{p^{M},q}\leq\left\Vert \hat{\mathbf{g}}\right\Vert _{p^{\infty},q}\left\Vert \hat{\varphi}*\hat{\mathbf{f}}\right\Vert _{p^{\max\left\{ M,N\right\} },q}\label{eq:MD phi_N hat convolve f hat convolve g hat estimate} \end{equation} Proof of claim: As in the one-dimensional case, we start with the top-most inequality: \begin{align*} \left\Vert \hat{\varphi}*\hat{\mathbf{f}}\right\Vert _{p^{M},q} & =\sup_{\left\Vert \mathbf{t}\right\Vert _{p}\leq p^{M}}\left|\sum_{\mathbf{s}\in\hat{\mathbb{Z}}_{p}^{r}}\hat{\varphi}\left(\mathbf{s}\right)\hat{\mathbf{f}}\left(\mathbf{t}-\mathbf{s}\right)\right|_{q}\\ \left(\hat{\varphi}\left(\mathbf{s}\right)=\mathbf{0},\textrm{ }\forall\left\Vert \mathbf{s}\right\Vert _{p}>p^{N}\right); & \leq\sup_{\left\Vert \mathbf{t}\right\Vert _{p}\leq p^{M}}\left\Vert \sum_{\left\Vert \mathbf{s}\right\Vert _{p}\leq p^{N}}\hat{\varphi}\left(\mathbf{s}\right)\hat{\mathbf{f}}\left(\mathbf{t}-\mathbf{s}\right)\right\Vert _{q}\\ \left(\textrm{ultrametric ineq.}\right); & \leq\sup_{\left\Vert \mathbf{t}\right\Vert _{p}\leq p^{M}}\sup_{\left\Vert \mathbf{s}\right\Vert _{p}\leq p^{N}}\left\Vert \hat{\varphi}\left(\mathbf{s}\right)\hat{\mathbf{f}}\left(\mathbf{t}-\mathbf{s}\right)\right\Vert _{q}\\ & \leq\sup_{\left\Vert \mathbf{s}\right\Vert _{p}\leq p^{N}}\left\Vert \hat{\varphi}\left(\mathbf{s}\right)\right\Vert _{q}\sup_{\left\Vert \mathbf{t}\right\Vert _{p}\leq p^{\max\left\{ M,N\right\} }}\left\Vert \hat{\mathbf{f}}\left(\mathbf{t}\right)\right\Vert _{q}\\ & =\left\Vert \hat{\varphi}\right\Vert _{p^{\infty},q}\cdot\left\Vert \hat{\mathbf{f}}\right\Vert _{p^{\max\left\{ M,N\right\} },q}\\ \left(\left\Vert \hat{\varphi}\right\Vert _{p^{\infty},q}\leq1\right); & \leq\left\Vert \hat{\mathbf{f}}\right\Vert _{p^{\max\left\{ M,N\right\} },q} \end{align*} This proves (\ref{eq:MD phi_N hat convolve f hat estimate}). Next, we deal with (\ref{eq:MD phi_N hat convolve f hat convolve g hat estimate}). For that, we start by writing out the convolution of $\hat{\varphi}*\hat{\mathbf{f}}$ with $\hat{\mathbf{g}}$: \begin{align*} \left\Vert \hat{\varphi}*\hat{\mathbf{f}}*\hat{\mathbf{g}}\right\Vert _{p^{M},q} & =\sup_{\left\Vert \mathbf{t}\right\Vert _{p}\leq p^{M}}\left\Vert \sum_{\mathbf{s}\in\hat{\mathbb{Z}}_{p}^{r}}\left(\hat{\varphi}*\hat{\mathbf{f}}\right)\left(\mathbf{t}-\mathbf{s}\right)\hat{\mathbf{g}}\left(\mathbf{s}\right)\right\Vert _{q}\\ \left(\textrm{ultrametric ineq.}\right); & \leq\sup_{\left\Vert \mathbf{t}\right\Vert _{p}\leq p^{M}}\sup_{\mathbf{s}\in\hat{\mathbb{Z}}_{p}^{r}}\left\Vert \left(\hat{\varphi}*\hat{\mathbf{f}}\right)\left(\mathbf{t}-\mathbf{s}\right)\hat{\mathbf{g}}\left(\mathbf{s}\right)\right\Vert _{q}\\ \left(\hat{\mathbf{g}}\left(\mathbf{s}\right)=\mathbf{0},\textrm{ }\forall\left\Vert \mathbf{s}\right\Vert _{p}>p^{M}\right); & \leq\sup_{\left\Vert \mathbf{t}\right\Vert _{p}\leq p^{M}}\sup_{\left\Vert \mathbf{s}\right\Vert _{p}\leq p^{M}}\left\Vert \left(\hat{\varphi}*\hat{\mathbf{f}}\right)\left(\mathbf{t}-\mathbf{s}\right)\hat{\mathbf{g}}\left(\mathbf{s}\right)\right\Vert _{q}\\ & \leq\left\Vert \hat{\mathbf{g}}\right\Vert _{p^{\infty},q}\sup_{\left\Vert \mathbf{t}\right\Vert _{p},\left\Vert \mathbf{s}\right\Vert _{p}\leq p^{M}}\left\Vert \left(\hat{\varphi}*\hat{\mathbf{f}}\right)\left(\mathbf{t}-\mathbf{s}\right)\right\Vert _{q} \end{align*} Next, we write out the convolution $\hat{\varphi}*\hat{\mathbf{f}}$. This gives us: \begin{align*} \left\Vert \hat{\varphi}*\hat{\mathbf{f}}*\hat{\mathbf{g}}\right\Vert _{p^{M},q} & \leq\left\Vert \hat{\mathbf{g}}\right\Vert _{p^{\infty},q}\sup_{\left\Vert \mathbf{t}\right\Vert _{p},\left\Vert \mathbf{s}\right\Vert _{p}\leq p^{M}}\left\Vert \sum_{\mathbf{v}\in\hat{\mathbb{Z}}_{p}}\hat{\varphi}\left(\mathbf{t}-\mathbf{s}-\mathbf{v}\right)\hat{\mathbf{f}}\left(\mathbf{v}\right)\right\Vert _{q}\\ \left(\textrm{let }\mathbf{u}=\mathbf{s}+\mathbf{v}\right); & =\left\Vert \hat{\mathbf{g}}\right\Vert _{p^{\infty},q}\sup_{\left\Vert \mathbf{t}\right\Vert _{p},\left\Vert \mathbf{s}\right\Vert _{p}\leq p^{M}}\left\Vert \sum_{\mathbf{u}-\mathbf{s}\in\hat{\mathbb{Z}}_{p}^{r}}\hat{\varphi}\left(\mathbf{t}-\mathbf{u}\right)\hat{\mathbf{f}}\left(\mathbf{u}-\mathbf{s}\right)\right\Vert _{q}\\ \left(\mathbf{s}+\hat{\mathbb{Z}}_{p}^{r}=\hat{\mathbb{Z}}_{p}^{r}\right); & =\left\Vert \hat{\mathbf{g}}\right\Vert _{p^{\infty},q}\sup_{\left\Vert \mathbf{t}\right\Vert _{p},\left\Vert \mathbf{s}\right\Vert _{p}\leq p^{M}}\left\Vert \sum_{\mathbf{u}\in\hat{\mathbb{Z}}_{p}^{r}}\hat{\varphi}\left(\mathbf{t}-\mathbf{u}\right)\hat{\mathbf{f}}\left(\mathbf{u}-\mathbf{s}\right)\right\Vert _{q} \end{align*} We now invoke the vanishing of $\hat{\varphi}\left(\mathbf{t}-\mathbf{u}\right)$ for all $\left\Vert \mathbf{t}-\mathbf{u}\right\Vert _{p}>p^{N}$. Indeed, because $\mathbf{t}$ is restricted to $\left\Vert \mathbf{t}\right\Vert _{p}\leq p^{M}$, observe that when $\left\Vert \mathbf{u}\right\Vert _{p}>p^{\max\left\{ M,N\right\} }$, the ultrametric inequality yields: \begin{equation} \left\Vert \mathbf{t}-\mathbf{u}\right\Vert _{p}=\max\left\{ \left\Vert \mathbf{t}\right\Vert _{p},\left\Vert \mathbf{u}\right\Vert _{p}\right\} >p^{\max\left\{ M,N\right\} }>p^{N} \end{equation} Hence, for $\left\Vert \mathbf{t}\right\Vert _{p},\left\Vert \mathbf{s}\right\Vert _{p}\leq p^{M}$, the summand $\hat{\varphi}\left(\mathbf{t}-\mathbf{u}\right)\hat{\mathbf{f}}\left(\mathbf{u}-\mathbf{s}\right)$ vanishes whenever $\left\Vert \mathbf{u}\right\Vert >p^{\max\left\{ M,N\right\} }$. This gives us: \begin{equation} \left\Vert \hat{\varphi}*\hat{\mathbf{f}}*\hat{\mathbf{g}}\right\Vert _{p^{M},q}\leq\left\Vert \hat{\mathbf{g}}\right\Vert _{p^{\infty},q}\sup_{\left\Vert \mathbf{t}\right\Vert _{p},\left\Vert \mathbf{s}\right\Vert _{p}\leq p^{M}}\left\Vert \sum_{\left\Vert \mathbf{u}\right\Vert _{p}\leq p^{\max\left\{ M,N\right\} }}\hat{\varphi}\left(\mathbf{t}-\mathbf{u}\right)\hat{\mathbf{f}}\left(\mathbf{u}-\mathbf{s}\right)\right\Vert _{q} \end{equation} Next, we enlarge things slightly by expanding the range of $\left\Vert \mathbf{s}\right\Vert _{p}$ and $\left\Vert \mathbf{t}\right\Vert _{p}$ from $\leq p^{M}$ to $\leq p^{\max\left\{ M,N\right\} }$: \begin{equation} \left\Vert \hat{\varphi}*\hat{\mathbf{f}}*\hat{\mathbf{g}}\right\Vert _{p^{M},q}\leq\left\Vert \hat{\mathbf{g}}\right\Vert _{p^{\infty},q}\sup_{\left\Vert \mathbf{t}\right\Vert _{p},\left\Vert \mathbf{s}\right\Vert _{p}\leq p^{\max\left\{ M,N\right\} }}\left\Vert \sum_{\left\Vert \mathbf{u}\right\Vert _{p}\leq p^{\max\left\{ M,N\right\} }}\hat{\varphi}\left(\mathbf{t}-\mathbf{u}\right)\hat{\mathbf{f}}\left(\mathbf{u}-\mathbf{s}\right)\right\Vert _{q} \end{equation} Like in the one-dimensional case, everything is now on the same level: $\mathbf{t}$, $\mathbf{s}$, and $\mathbf{u}$ are all bounded in $p$-adic norm by $p^{\max\left\{ M,N\right\} }$. Exploiting the closure of the set: \[ \left\{ \mathbf{x}\in\hat{\mathbb{Z}}_{p}^{r}:\left\Vert \mathbf{x}\right\Vert _{p}\leq p^{\max\left\{ M,N\right\} }\right\} \] under addition allows us to note that our $\mathbf{u}$-sum is invariant under the change of variables $\mathbf{u}\mapsto\mathbf{u}+\mathbf{s}$ for any $\left\Vert \mathbf{s}\right\Vert _{p}\leq p^{\max\left\{ M,N\right\} }$. Making this change of variables then gives us: \begin{align*} \left\Vert \hat{\varphi}*\hat{\mathbf{f}}*\hat{\mathbf{g}}\right\Vert _{p^{M},q} & \leq\left\Vert \hat{\mathbf{g}}\right\Vert _{p^{\infty},q}\sup_{\left\Vert \mathbf{t}\right\Vert _{p},\left\Vert \mathbf{s}\right\Vert _{p}\leq p^{\max\left\{ M,N\right\} }}\left\Vert \sum_{\left\Vert \mathbf{u}\right\Vert _{p}\leq p^{\max\left\{ M,N\right\} }}\hat{\varphi}\left(\mathbf{t}-\left(\mathbf{u}+\mathbf{s}\right)\right)\hat{\mathbf{f}}\left(\mathbf{u}\right)\right\Vert _{q}\\ & =\left\Vert \hat{\mathbf{g}}\right\Vert _{p^{\infty},q}\sup_{\left\Vert \mathbf{t}\right\Vert _{p},\left\Vert \mathbf{s}\right\Vert _{p}\leq p^{\max\left\{ M,N\right\} }}\left\Vert \sum_{\left\Vert \mathbf{u}\right\Vert _{p}\leq p^{\max\left\{ M,N\right\} }}\hat{\varphi}\left(\mathbf{t}-\mathbf{s}-\mathbf{u}\right)\hat{\mathbf{f}}\left(\mathbf{u}\right)\right\Vert _{q} \end{align*} Because: \begin{equation} \left\{ \mathbf{t}-\mathbf{s}:\left\Vert \mathbf{t}\right\Vert _{p},\left\Vert \mathbf{s}\right\Vert _{p}\leq p^{\max\left\{ M,N\right\} }\right\} =\left\{ \mathbf{t}:\left\Vert \mathbf{t}\right\Vert _{p}\leq p^{\max\left\{ M,N\right\} }\right\} \end{equation} we then have: \begin{align*} \left\Vert \hat{\varphi}*\hat{\mathbf{f}}*\hat{\mathbf{g}}\right\Vert _{p^{M},q} & \leq\left\Vert \hat{\mathbf{g}}\right\Vert _{p^{\infty},q}\sup_{\left\Vert \mathbf{t}\right\Vert _{p}\leq p^{\max\left\{ M,N\right\} }}\left\Vert \sum_{\left\Vert \mathbf{u}\right\Vert _{p}\leq p^{\max\left\{ M,N\right\} }}\hat{\varphi}\left(\mathbf{t}-\mathbf{u}\right)\hat{\mathbf{f}}\left(\mathbf{u}\right)\right\Vert _{q}\\ & \leq\left\Vert \hat{\mathbf{g}}\right\Vert _{p^{\infty},q}\sup_{\left\Vert \mathbf{t}\right\Vert _{p}\leq p^{\max\left\{ M,N\right\} }}\left\Vert \sum_{\mathbf{u}\in\hat{\mathbb{Z}}_{p}^{r}}\hat{\varphi}\left(\mathbf{t}-\mathbf{u}\right)\hat{\mathbf{f}}\left(\mathbf{u}\right)\right\Vert _{q}\\ & =\left\Vert \hat{\mathbf{g}}\right\Vert _{p^{\infty},q}\sup_{\left\Vert \mathbf{t}\right\Vert _{p}\leq p^{\max\left\{ M,N\right\} }}\left\Vert \left(\hat{\varphi}*\hat{\mathbf{f}}\right)\left(\mathbf{u}\right)\right\Vert _{q}\\ \left(\textrm{by definition}\right); & =\left\Vert \hat{\mathbf{g}}\right\Vert _{p^{\infty},q}\left\Vert \hat{\varphi}*\hat{\mathbf{f}}\right\Vert _{p^{\max\left\{ M,N\right\} },q} \end{align*} which proves the desired estimate. \end{claim} \vphantom{} Now, let's line everything up: first, we choose $\epsilon\in\left(0,1\right)$ and a non-identically-zero $\hat{\mathbf{h}}_{\epsilon}$ so that: \begin{equation} \left\Vert \vec{\mathbf{1}}_{\mathbf{0}}-\hat{\mathcal{M}}*\hat{\mathbf{h}}_{\epsilon}\right\Vert _{p^{\infty},q}<\epsilon \end{equation} Then, by \textbf{Claim \ref{claim:5.12}}, we can choose an integer $M_{\epsilon}$ (depending only on $\epsilon$ and $\hat{\mathbf{h}}_{\epsilon}$) so that: \begin{equation} \left\Vert \vec{\mathbf{1}}_{\mathbf{0}}-\hat{\mathcal{M}}*\hat{\mathbf{h}}_{\epsilon,M}\right\Vert _{p^{M},q}<\epsilon,\textrm{ }\forall M\geq M_{\epsilon} \end{equation} Since the norm is the maximum of the components, multiplying the left-hand side entry-wise by the standard basis vector $\mathbf{e}_{\ell}$ can only decrease the norm: \begin{equation} \left\Vert \mathbf{e}_{\ell}\left(\vec{\mathbf{1}}_{\mathbf{0}}-\hat{\mathcal{M}}*\hat{\mathbf{h}}_{\epsilon,M}\right)\right\Vert _{p^{M},q}\leq\left\Vert \vec{\mathbf{1}}_{\mathbf{0}}-\hat{\mathcal{M}}*\hat{\mathbf{h}}_{\epsilon,M}\right\Vert _{p^{M},q}<\epsilon,\textrm{ }\forall M\geq M_{\epsilon} \end{equation} Next, we observe that: \begin{equation} \left\Vert \mathbf{e}_{\ell}\left(\hat{\phi}_{N}*\left(\vec{\mathbf{1}}_{\mathbf{0}}-\left(\hat{\mathcal{M}}*\hat{\mathbf{h}}_{\epsilon,M}\right)\right)\right)\right\Vert _{p^{M},q}=\left\Vert \mathbf{e}_{\ell}\hat{\phi}_{N}*\mathbf{e}_{\ell}\left(\vec{\mathbf{1}}_{\mathbf{0}}-\left(\hat{\mathcal{M}}*\hat{\mathbf{h}}_{\epsilon,M}\right)\right)\right\Vert _{p^{M},q}\label{eq:MD WTT - Target of attack} \end{equation} Like in the one-dimensional case, we will estimate this in two ways. First, we apply (\ref{eq:MD phi_N hat convolve f hat estimate}) from \textbf{Claim \ref{claim:5.14}} with $\hat{\varphi}=\mathbf{e}_{\ell}\hat{\phi}_{N}$ and $\hat{\mathbf{f}}=\mathbf{e}_{\ell}\left(\vec{\mathbf{1}}_{\mathbf{0}}-\left(\hat{\mathcal{M}}*\hat{\mathbf{h}}_{\epsilon,M}\right)\right)$. This gives us: \begin{align*} \left\Vert \mathbf{e}_{\ell}\hat{\phi}_{N}*\mathbf{e}_{\ell}\left(\vec{\mathbf{1}}_{\mathbf{0}}-\left(\hat{\mathcal{M}}*\hat{\mathbf{h}}_{\epsilon,M}\right)\right)\right\Vert _{p^{M},q} & \leq\left\Vert \mathbf{e}_{\ell}\left(\vec{\mathbf{1}}_{\mathbf{0}}-\left(\hat{\mathcal{M}}*\hat{\mathbf{h}}_{\epsilon,M}\right)\right)\right\Vert _{p^{\max\left\{ M,N\right\} },q}\\ & \leq\left\Vert \vec{\mathbf{1}}_{\mathbf{0}}-\left(\hat{\mathcal{M}}*\hat{\mathbf{h}}_{\epsilon,M}\right)\right\Vert _{p^{\max\left\{ M,N\right\} },q}\\ \left(M\geq M_{\epsilon}\Rightarrow\textrm{use \textbf{Claim \ref{claim:5.12}}}\right); & <\epsilon \end{align*} So, the \emph{left-hand side} of (\ref{eq:MD WTT - Target of attack}) is $<\epsilon$. To contradict this, we use the ultrametric inequality for the non-archimedean norm $\left\Vert \cdot\right\Vert _{p^{M},q}$ on the \emph{right-hand side} of (\ref{eq:MD WTT - Target of attack}). This yields: \begin{equation} \max\left\{ \left\Vert \mathbf{e}_{\ell}\hat{\phi}_{N}\right\Vert _{p^{M},q},\left\Vert \hat{\phi}_{N}*\mathbf{e}_{\ell}\hat{\mathcal{M}}*\hat{\mathbf{h}}_{\epsilon,M}\right\Vert _{p^{M},q}\right\} \label{eq:WTT - Ultrametric inequality-1} \end{equation} as an upper bound for: \begin{equation} \left\Vert \mathbf{e}_{\ell}\left(\hat{\phi}_{N}-\left(\hat{\phi}_{N}*\hat{\mathcal{M}}*\hat{\mathbf{h}}_{\epsilon,M}\right)\right)\right\Vert _{p^{M},q} \end{equation} (Note that we could have also written $\mathbf{e}_{\ell}$ next to $\hat{\phi}_{N}$ and $\hat{\mathbf{h}}_{\epsilon,M}$, though it would have been redundant.) Using (\ref{eq:MD phi_N hat convolve f hat convolve g hat estimate}) from \textbf{Claim \ref{claim:5.14}}, we get: \begin{align*} \left\Vert \hat{\phi}_{N}*\mathbf{e}_{\ell}\hat{\mathcal{M}}*\hat{\mathbf{h}}_{\epsilon,M}\right\Vert _{p^{M},q} & \leq\left\Vert \hat{\phi}_{N}*\mathbf{e}_{\ell}\hat{\mathcal{M}}\right\Vert _{p^{M},q}\left\Vert \hat{\mathbf{h}}_{\epsilon,M}\right\Vert _{p^{M},q}\\ \left(M\geq M_{\epsilon}\right); & \leq\left\Vert \hat{\phi}_{N}*\mathbf{e}_{\ell}\hat{\mathcal{M}}\right\Vert _{p^{M},q}\cdot\left\Vert \hat{\mathbf{h}}_{\epsilon}\right\Vert _{p^{\infty},q} \end{align*} Now, let $\epsilon^{\prime}=\frac{\epsilon}{2\left\Vert \hat{\mathbf{h}}_{\epsilon}\right\Vert _{p^{\infty},q}}$; we are allowed to do this, thanks to our assumption that $\hat{\mathbf{h}}_{\epsilon}$ was not identically zero. Seeing as $N$ is still arbitrary, let $N$ be larger than both $N_{\epsilon^{\prime}}$ and our choice for $M$; \textbf{Claim \ref{claim:5.13}} tells us that $\left\Vert \mathbf{e}_{\ell}\hat{\mathcal{M}}*\hat{\phi}_{N}\right\Vert _{p^{M},q}<\epsilon^{\prime}$. Since the size of the $\ell$th component of $\mathbf{e}_{\ell}\hat{\mathcal{M}}*\hat{\phi}_{N}$ is: \begin{equation} \left\Vert \mathbf{e}_{\ell}\hat{\mathcal{M}}*\hat{\phi}_{N}\right\Vert _{p^{M},q}<\epsilon^{\prime} \end{equation} we can write: \begin{align*} \left\Vert \hat{\phi}_{N}*\mathbf{e}_{\ell}\hat{\mathcal{M}}*\hat{\mathbf{h}}_{\epsilon,M}\right\Vert _{p^{M},q} & \leq\left\Vert \hat{\phi}_{N}*\mathbf{e}_{\ell}\hat{\mathcal{M}}\right\Vert _{p^{M},q}\cdot\left\Vert \hat{\mathbf{h}}_{\epsilon}\right\Vert _{p^{\infty},q}\\ & <\epsilon^{\prime}\cdot\left\Vert \hat{\mathbf{h}}_{\epsilon}\right\Vert _{p^{\infty},q}\\ & =\frac{\epsilon}{2\left\Vert \hat{\mathbf{h}}_{\epsilon}\right\Vert _{p^{\infty},q}}\cdot\left\Vert \hat{\mathbf{h}}_{\epsilon}\right\Vert _{p^{\infty},q}\\ & =\frac{\epsilon}{2} \end{align*} for our large choice of $N$. Because $\left\Vert \mathbf{e}_{\ell}\hat{\phi}_{N}\right\Vert _{p^{M},q}=1$ for all $M,N\geq0$, this shows that: \begin{equation} \left\Vert \mathbf{e}_{\ell}\hat{\phi}_{N}\right\Vert _{p^{M},q}=1>\frac{\epsilon}{2}>\left\Vert \hat{\phi}_{N}*\mathbf{e}_{\ell}\hat{\mathcal{M}}*\hat{\mathbf{h}}_{\epsilon,M}\right\Vert _{p^{M},q} \end{equation} Hence, by the ultrametric inequality, the upper bound (\ref{eq:WTT - Ultrametric inequality-1}) is in fact an equality: \begin{align*} \left\Vert \mathbf{e}_{\ell}\left(\hat{\phi}_{N}-\left(\hat{\phi}_{N}*\hat{\mathcal{M}}*\hat{\mathbf{h}}_{\epsilon,M}\right)\right)\right\Vert _{p^{M},q} & =\max\left\{ \left\Vert \mathbf{e}_{\ell}\hat{\phi}_{N}\right\Vert _{p^{M},q},\left\Vert \hat{\phi}_{N}*\mathbf{e}_{\ell}\hat{\mathcal{M}}*\hat{\mathbf{h}}_{\epsilon,M}\right\Vert _{p^{M},q}\right\} \\ & =1 \end{align*} With this, (\ref{eq:MD WTT - Target of attack}) becomes: \begin{align*} \epsilon & >\left\Vert \mathbf{e}_{\ell}\left(\hat{\phi}_{N}*\left(\vec{\mathbf{1}}_{\mathbf{0}}-\left(\hat{\mathcal{M}}*\hat{\mathbf{h}}_{\epsilon,M}\right)\right)\right)\right\Vert _{p^{M},q}\\ & =\left\Vert \mathbf{e}_{\ell}\hat{\phi}_{N}-\mathbf{e}_{\ell}\left(\hat{\phi}_{N}*\hat{\mathcal{M}}*\hat{\mathbf{h}}_{\epsilon,M}\right)\right\Vert _{p^{M},q}\\ & =1 \end{align*} But $\epsilon<1$!\textemdash we have arrived at our contradiction. Consequently, the existence of the ``zero'' $\mathbf{z}_{0}$ precludes the translates of $\hat{\mathcal{M}}$ from being dense in $c_{0}\left(\hat{\mathbb{Z}}_{p}^{r},\mathbb{C}_{q}^{d}\right)$. Q.E.D. \begin{rem} Versions of \textbf{Theorem \ref{thm:MD pq-adic WTT for continuous functions}} likely hold for arbitrary thick $\left(p,q\right)$-adic measures, but I will not pursue the matter here. \end{rem} \subsection{\emph{\label{subsec:5.4.4More-Fourier-Resummation}More} Fourier Resummation Lemmata} As in the one-dimensional case, we will need to avail ourselves of various Fourier Resummation Lemmata. First, of course, we need the multi-dimensional definitions: \begin{defn} A $\left(p,q\right)$-adic function $\hat{\mu}:\hat{\mathbb{Z}}_{p}^{r}\rightarrow\mathbb{C}_{q}^{\rho,c}$ is said to be: \vphantom{} I. \textbf{Radial} whenever:\index{thick measure!radial} \[ \hat{\mu}\left(\mathbf{t}\right)=\hat{\mu}\left(\left|t_{1}\right|_{p_{1}}^{-1},\ldots,\left|t_{r}\right|_{p_{r}}^{-1}\right),\textrm{ }\forall\mathbf{t}\in\hat{\mathbb{Z}}_{p}^{r}\backslash\left\{ \mathbf{0}\right\} \] \vphantom{} II. \textbf{Magnitudinal} \index{thick measure!magnitudinal}whenever there is a function $\kappa:\mathbb{N}_{0}^{r}\rightarrow\mathbb{C}_{q}^{\rho,c}$ so that: \[ \hat{\mu}\left(\mathbf{t}\right)=\begin{cases} \kappa\left(\mathbf{0}\right) & \textrm{if }\mathbf{t}=\mathbf{0}\\ \sum_{\mathbf{m}=\mathbf{0}}^{p^{-v_{p}\left(\mathbf{t}\right)}-1}\kappa\left(\mathbf{m}\right)e^{-2\pi i\left(\mathbf{m}\cdot\mathbf{t}\right)} & \textrm{else} \end{cases},\textrm{ }\forall\mathbf{t}\in\hat{\mathbb{Z}}_{p}^{r} \] \vphantom{} III. \textbf{Radially-Magnitudinal} \index{thick measure!radial-magnitudinal} whenever there is a radial function $\hat{\nu}:\hat{\mathbb{Z}}_{p}^{r}\rightarrow\mathbb{C}_{q}^{\left(r,c\right)}$ and a magnitudinal function $\hat{\eta}:\hat{\mathbb{Z}}_{p}^{r}\rightarrow\mathbb{C}_{q}^{\left(\rho,r\right)}$ so that: \begin{equation} \hat{\mu}\left(\mathbf{t}\right)=\hat{\eta}\left(\mathbf{t}\right)\hat{\nu}\left(\mathbf{t}\right),\textrm{ }\forall\mathbf{t}\in\hat{\mathbb{Z}}_{p}^{r} \end{equation} where the right-hand side, as a product of a $\rho\times r$ matrix-valued function and an $r\times c$ matrix-valued function is a $\rho\times c$ matrix-valued function. \end{defn} \begin{defn} We say a thick measure $\mathcal{M}$ is radial (resp. magnitudinal, radially-magnitudinal) whenever $\hat{\mathcal{M}}\left(\mathbf{t}\right)$ is radial (resp. magnitudinal, radially-magnitudinal). \end{defn} \begin{defn} Consider a function $\kappa:\mathbb{N}_{0}^{r}\rightarrow\mathbb{C}_{q}^{d,d}$. \vphantom{} I. We say $\kappa$ is \textbf{$p$-adically structured} / \textbf{has $p$-adic structure }whenever:\index{$p$-adic!structure} \begin{equation} \kappa\left(p\mathbf{n}+\mathbf{j}\right)=\mathbf{M}_{\mathbf{j}}\kappa_{H}\left(\mathbf{n}\right)\mathbf{M}_{\mathbf{0}}^{-1},\textrm{ }\forall\mathbf{j}\in\mathbb{Z}^{r}/p\mathbb{Z}^{r},\textrm{ }\forall\mathbf{n}\in\mathbb{N}_{0}^{r}\label{eq:Definition of P-adic structure} \end{equation} where $\left\{ \mathbf{M}_{\mathbf{j}}\right\} _{\mathbf{j}\in\mathbb{Z}^{r}/p\mathbb{Z}^{r}}$ are invertible matrices in $\mathbb{C}_{q}^{d,d}$. \vphantom{} II. We say $\kappa$\textbf{ }is \index{tame}\textbf{$\left(p,K\right)$-adically tame} on a set $X\subseteq\mathbb{Z}_{p}^{r}$ whenever $\lim_{n\rightarrow\infty}\left\Vert \kappa\left(\left[\mathbf{z}\right]_{p^{n}}\right)\right\Vert _{K}=0$ for all $\mathbf{z}\in X$. We do not mention $X$ when $X=\mathbb{Z}_{p}^{r}$. If $K$ is a $q$-adic field, we say $\kappa$ is $\left(p,q\right)$-adically tame on $X$; if $K$ is archimedean, we say $\kappa$ is $\left(p,\infty\right)$-adically tame on $X$. \end{defn} \begin{prop} \label{prop:MD p-adic structure prop}A function $\kappa:\mathbb{N}_{0}^{r}\rightarrow\mathbb{C}_{q}^{d,d}$ is $p$-adically structured if and only if, for every $m\geq0$: \begin{equation} \kappa\left(\mathbf{n}+\mathbf{j}p^{m}\right)=\mathbf{M}_{\mathbf{n}}\kappa_{H}\left(\mathbf{j}\right)\mathbf{M}_{\mathbf{0}}^{-1},\textrm{ }\forall\mathbf{j}\in\mathbb{Z}^{r}/p\mathbb{Z}^{r},\textrm{ }\forall\mathbf{n}\leq p^{m}-1 \end{equation} where $\mathbf{M}_{\mathbf{n}}=\mathbf{M}_{\mathbf{j}_{1}}\times\cdots\times\mathbf{M}_{\mathbf{j}_{\lambda_{p}\left(\mathbf{n}\right)}}$, where $\mathbf{J}=\left(\mathbf{j}_{1},\ldots,\mathbf{j}_{\lambda_{p}\left(\mathbf{n}\right)}\right)\in\textrm{String}^{r}\left(p\right)$ is the shortest block string representing $\mathbf{n}$. \end{prop} Proof: Write: \[ \mathbf{n}=\left(n_{1},\ldots,n_{r}\right) \] \[ \mathbf{j}=\left(j_{1},\ldots,j_{r}\right) \] where: \[ n_{\ell}=n_{\ell,0}+n_{\ell,1}p+\cdots+n_{\ell,\lambda_{p}\left(n_{\ell}\right)-1}p^{\lambda_{p}\left(n_{\ell}\right)-1},\textrm{ }\forall\ell\in\left\{ 1,\ldots,r\right\} \] and: \[ \mathbf{n}+\mathbf{j}p^{m}=\left(n_{1}+j_{1}p^{m},\ldots,n_{r}+j_{r}p^{m}\right) \] where, for each $\ell$, $\lambda_{p}\left(n_{\ell}\right)-1<m$. The rest is just like the one-dimensional case. Q.E.D. \vphantom{}Next up, the multi-dimensional resummation lemmata. \begin{lem} \label{lem:MD magnitude F Resum Lemma}Let $\hat{\mathcal{M}}:\hat{\mathbb{Z}}_{p}^{r}\rightarrow\mathbb{C}_{q}^{d,d}$ be the Fourier-Stieltjes transform of a magnitudinal thick measure, and let $\kappa$ have $p$-adic structure. Then, for all $N\geq0$ and all $\mathbf{z}\in\mathbb{Z}_{p}^{r}$: \begin{align} \tilde{\mathcal{M}}_{N}\left(\mathbf{z}\right) & =p^{rN}\kappa\left(\left[\mathbf{z}\right]_{p^{N}}\right)-\sum_{n=0}^{N-1}\sum_{\mathbf{j}>\mathbf{0}}^{p-1}p^{rn}\kappa\left(\left[\mathbf{z}\right]_{p^{n}}+\mathbf{j}p^{n+1}\right)\label{eq:MD magnitudinal resummation formula} \end{align} \end{lem} Proof: We begin with: \[ \hat{\mathcal{M}}\left(\mathbf{t}\right)=\sum_{\mathbf{m}=\mathbf{0}}^{p^{-v_{p}\left(\mathbf{t}\right)}-1}\kappa\left(\mathbf{m}\right)e^{-2\pi i\left(\mathbf{m}\cdot\mathbf{t}\right)} \] Consequently: \begin{align*} \tilde{\mathcal{M}}_{N}\left(\mathbf{z}\right) & =\sum_{\left\Vert \mathbf{t}\right\Vert _{p}\leq p^{N}}\hat{\mathcal{M}}\left(\mathbf{t}\right)e^{2\pi i\left\{ \mathbf{t}\mathbf{z}\right\} _{p}}\\ & =\kappa\left(\mathbf{0}\right)+\sum_{n=1}^{N}\sum_{\left\Vert \mathbf{t}\right\Vert _{p}=p^{n}}\left(\sum_{\mathbf{m}=\mathbf{0}}^{p^{n}-1}\kappa\left(\mathbf{m}\right)e^{-2\pi i\left(\mathbf{m}\cdot\mathbf{t}\right)}\right)e^{2\pi i\left\{ \mathbf{t}\mathbf{z}\right\} _{p}}\\ & =\kappa\left(\mathbf{0}\right)+\sum_{n=1}^{N}\sum_{\mathbf{m}=\mathbf{0}}^{p^{n}-1}\kappa\left(\mathbf{m}\right)\sum_{\left\Vert \mathbf{t}\right\Vert _{p}=p^{n}}e^{2\pi i\left\{ \mathbf{t}\left(\mathbf{z}-\mathbf{m}\right)\right\} _{p}}\\ & =\kappa\left(\mathbf{0}\right)+\sum_{n=1}^{N}\sum_{\mathbf{m}=\mathbf{0}}^{p^{n}-1}\kappa\left(\mathbf{m}\right)\left(p^{rn}\left[\mathbf{z}\overset{p^{n}}{\equiv}\mathbf{m}\right]-p^{r\left(n-1\right)}\left[\mathbf{z}\overset{p^{n-1}}{\equiv}\mathbf{m}\right]\right)\\ & =\kappa\left(\mathbf{0}\right)+\sum_{n=1}^{N}\left(p^{rn}\kappa\left(\left[\mathbf{z}\right]_{p^{n}}\right)-p^{r\left(n-1\right)}\sum_{\mathbf{m}=\mathbf{0}}^{p^{n}-1}\kappa\left(\mathbf{m}\right)\left[\mathbf{z}\overset{p^{n-1}}{\equiv}\mathbf{m}\right]\right) \end{align*} Because $\kappa$ has $p$-adic structure, we can then write: \begin{align*} \sum_{\mathbf{m}=\mathbf{0}}^{p^{n}-1}\kappa\left(\mathbf{m}\right)\left[\mathbf{z}\overset{p^{n-1}}{\equiv}\mathbf{m}\right] & =\sum_{\mathbf{j}=\mathbf{0}}^{p-1}\sum_{\mathbf{m}=\mathbf{0}}^{p^{n-1}-1}\kappa\left(\mathbf{m}+\mathbf{j}p^{n}\right)\left[\mathbf{z}\overset{p^{n-1}}{\equiv}\mathbf{m}+\mathbf{j}p^{n}\right]\\ & =\sum_{\mathbf{j}=\mathbf{0}}^{p-1}\kappa\left(\left[\mathbf{z}\right]_{p^{n-1}}+\mathbf{j}p^{n}\right) \end{align*} and so: \begin{equation} \sum_{\left\Vert \mathbf{t}\right\Vert _{p}=p^{n}}\sum_{\mathbf{m}=\mathbf{0}}^{p^{n}-1}\kappa\left(\mathbf{m}\right)e^{-2\pi i\mathbf{t}\cdot\mathbf{m}}e^{2\pi i\left\{ \mathbf{t}\mathbf{z}\right\} _{p}}=p^{rn}\kappa\left(\left[\mathbf{z}\right]_{p^{n}}\right)-p^{r\left(n-1\right)}\sum_{\mathbf{j}=\mathbf{0}}^{p-1}\kappa\left(\left[\mathbf{z}\right]_{p^{n-1}}+\mathbf{j}p^{n}\right)\label{eq:MD Level set Fourier sum of a magnitudinal multiplier} \end{equation} Consequently: \begin{align*} \tilde{\mathcal{M}}_{N}\left(\mathbf{z}\right) & =\kappa\left(\mathbf{0}\right)+\sum_{n=1}^{N}\left(p^{rn}\kappa\left(\left[\mathbf{z}\right]_{^{n}}\right)-p^{r\left(n-1\right)}\sum_{\mathbf{m}=\mathbf{0}}^{p^{n}-1}\kappa\left(\mathbf{m}\right)\left[\mathbf{z}\overset{p^{n-1}}{\equiv}\mathbf{m}\right]\right)\\ & =\kappa\left(\mathbf{0}\right)+\sum_{n=1}^{N}\left(p^{rn}\kappa\left(\left[\mathbf{z}\right]_{p^{n}}\right)-p^{r\left(n-1\right)}\sum_{\mathbf{j}=\mathbf{0}}^{p-1}\kappa\left(\left[\mathbf{z}\right]_{p^{n-1}}+\mathbf{j}p^{n}\right)\right)\\ & =\sum_{n=0}^{N}p^{rn}\kappa\left(\left[\mathbf{z}\right]_{p^{n}}\right)-\sum_{n=0}^{N-1}\sum_{\mathbf{j}=\mathbf{0}}^{p-1}p^{rn}\kappa\left(\left[\mathbf{z}\right]_{p^{n}}+\mathbf{j}p^{n+1}\right)\\ & =\sum_{n=0}^{N}p^{rn}\kappa\left(\left[\mathbf{z}\right]_{p^{n}}\right)-\sum_{n=0}^{N-1}p^{rn}\kappa\left(\left[\mathbf{z}\right]_{p^{n}}\right)-\sum_{n=0}^{N-1}\sum_{\mathbf{j}>\mathbf{0}}^{p-1}p^{rn}\kappa\left(\left[\mathbf{z}\right]_{p^{n}}+\mathbf{j}p^{n+1}\right)\\ & =p^{rN}\kappa\left(\left[\mathbf{z}\right]_{p^{N}}\right)-\sum_{n=0}^{N-1}\sum_{\mathbf{j}>\mathbf{0}}^{p-1}p^{rn}\kappa\left(\left[\mathbf{z}\right]_{p^{n}}+\mathbf{j}p^{n+1}\right) \end{align*} Q.E.D. \begin{lem} \label{lem:MD radial-magnitude F Resum Lemma}Let $\hat{\mathcal{M}}:\hat{\mathbb{Z}}_{p}^{r}\rightarrow\mathbb{C}_{q}^{d,d}$ be the Fourier-Stieltjes transform of a radially-magnitudinal thick measure, and let $\kappa$ have $p$-adic structure. Then, for all $N\geq0$ and all $\mathbf{z}\in\mathbb{Z}_{p}^{r}$: \begin{align} \tilde{\mathcal{M}}_{N}\left(\mathbf{z}\right) & =p^{rN}\kappa\left(\left[\mathbf{z}\right]_{p^{N}}\right)\hat{\nu}\left(\frac{1}{p^{N}}\right)+\sum_{n=0}^{N-1}p^{rn}\kappa\left(\left[\mathbf{z}\right]_{p^{n}}\right)\left(\hat{\nu}\left(\frac{1}{p^{n}}\right)-\hat{\nu}\left(\frac{1}{p^{n+1}}\right)\right)\label{eq:MD radially-magnitudinal resummation formula}\\ & -\sum_{n=0}^{N-1}p^{rn}\sum_{\mathbf{j}>\mathbf{0}}^{p-1}\kappa\left(\left[\mathbf{z}\right]_{p^{n}}+\mathbf{j}p^{n}\right)\hat{\nu}\left(\frac{1}{p^{n+1}}\right)\nonumber \end{align} \end{lem} Proof: Write: \[ \hat{\mathcal{M}}\left(\mathbf{t}\right)=\sum_{\mathbf{m}=\mathbf{0}}^{p^{-v_{p}\left(\mathbf{t}\right)}-1}\kappa\left(\mathbf{m}\right)\hat{\nu}\left(\mathbf{t}\right)e^{-2\pi i\left(\mathbf{m}\cdot\mathbf{t}\right)} \] Then: \begin{align*} \tilde{\mathcal{M}}_{N}\left(\mathbf{z}\right) & =\sum_{\left\Vert \mathbf{t}\right\Vert _{p}\leq p^{N}}\left(\sum_{\mathbf{m}=\mathbf{0}}^{p^{-v_{p}\left(\mathbf{t}\right)}-1}\kappa\left(\mathbf{m}\right)\hat{\nu}\left(\mathbf{t}\right)e^{-2\pi i\left(\mathbf{m}\cdot\mathbf{t}\right)}\right)e^{2\pi i\left\{ \mathbf{t}\mathbf{z}\right\} _{p}}\\ & =\kappa\left(\mathbf{0}\right)\hat{\nu}\left(\mathbf{0}\right)+\sum_{n=1}^{N}\sum_{\left\Vert \mathbf{t}\right\Vert _{p}=p^{n}}\left(\sum_{\mathbf{m}=\mathbf{0}}^{p^{-v_{p}\left(\mathbf{t}\right)}-1}\kappa\left(\mathbf{m}\right)\hat{\nu}\left(\mathbf{t}\right)e^{-2\pi i\left(\mathbf{m}\cdot\mathbf{t}\right)}\right)e^{2\pi i\left\{ \mathbf{t}\mathbf{z}\right\} _{p}}\\ \left(\textrm{use }(\ref{eq:First MD level set summation identity})\right); & =\kappa\left(\mathbf{0}\right)\hat{\nu}\left(\mathbf{0}\right)+\sum_{n=1}^{N}\sum_{\left\Vert \mathbf{t}\right\Vert _{p}=p^{n}}\left(\sum_{\mathbf{m}=\mathbf{0}}^{p^{n}-1}\kappa\left(\mathbf{m}\right)e^{-2\pi i\left(\mathbf{m}\cdot\mathbf{t}\right)}\right)e^{2\pi i\left\{ \mathbf{t}\mathbf{z}\right\} _{p}}\hat{\nu}\left(\frac{1}{p^{n}}\right) \end{align*} Using (\ref{eq:MD Level set Fourier sum of a magnitudinal multiplier}) from the proof of \textbf{Lemma \ref{lem:MD magnitude F Resum Lemma}} gives us: \begin{align*} \tilde{\mathcal{M}}_{N}\left(\mathbf{z}\right) & =\kappa\left(\mathbf{0}\right)\hat{\nu}\left(\mathbf{0}\right)+\sum_{n=1}^{N}\sum_{\left\Vert \mathbf{t}\right\Vert _{p}=p^{n}}\left(\sum_{\mathbf{m}=\mathbf{0}}^{p^{n}-1}\kappa\left(\mathbf{m}\right)e^{-2\pi i\left(\mathbf{m}\cdot\mathbf{t}\right)}\right)e^{2\pi i\left\{ \mathbf{t}\mathbf{z}\right\} _{p}}\hat{\nu}\left(\frac{1}{p^{n}}\right)\\ & =\kappa\left(\mathbf{0}\right)\hat{\nu}\left(\mathbf{0}\right)+\sum_{n=1}^{N}\left(p^{rn}\kappa\left(\left[\mathbf{z}\right]_{p^{n}}\right)-p^{r\left(n-1\right)}\sum_{\mathbf{j}=\mathbf{0}}^{p-1}\kappa\left(\left[\mathbf{z}\right]_{p^{n-1}}+\mathbf{j}p^{n-1}\right)\right)\hat{\nu}\left(\frac{1}{p^{n}}\right)\\ & =\sum_{n=0}^{N}p^{rn}\kappa\left(\left[\mathbf{z}\right]_{p^{n}}\right)\hat{\nu}\left(\frac{1}{p^{n}}\right)-\sum_{n=0}^{N-1}p^{rn}\sum_{\mathbf{j}=\mathbf{0}}^{p-1}\kappa\left(\left[\mathbf{z}\right]_{p^{n}}+\mathbf{j}p^{n}\right)\hat{\nu}\left(\frac{1}{p^{n+1}}\right)\\ & =\sum_{n=0}^{N}p^{rn}\kappa\left(\left[\mathbf{z}\right]_{p^{n}}\right)\hat{\nu}\left(\frac{1}{p^{n}}\right)-\sum_{n=0}^{N-1}p^{rn}\kappa\left(\left[\mathbf{z}\right]_{p^{n}}\right)\hat{\nu}\left(\frac{1}{p^{n+1}}\right)\\ & -\sum_{n=0}^{N-1}p^{rn}\sum_{\mathbf{j}>\mathbf{0}}^{p-1}\kappa\left(\left[\mathbf{z}\right]_{p^{n}}+\mathbf{j}p^{n}\right)\hat{\nu}\left(\frac{1}{p^{n+1}}\right) \end{align*} and so: \begin{align*} \tilde{\mathcal{M}}_{N}\left(\mathbf{z}\right) & =p^{rN}\kappa\left(\left[\mathbf{z}\right]_{p^{N}}\right)\hat{\nu}\left(\frac{1}{p^{N}}\right)+\sum_{n=0}^{N-1}p^{rn}\kappa\left(\left[\mathbf{z}\right]_{p^{n}}\right)\left(\hat{\nu}\left(\frac{1}{p^{n}}\right)-\hat{\nu}\left(\frac{1}{p^{n+1}}\right)\right)\\ & -\sum_{n=0}^{N-1}p^{rn}\sum_{\mathbf{j}>\mathbf{0}}^{p-1}\kappa\left(\left[\mathbf{z}\right]_{p^{n}}+\mathbf{j}p^{n}\right)\hat{\nu}\left(\frac{1}{p^{n+1}}\right) \end{align*} Q.E.D. \begin{prop} \label{prop:MD convolution with v_p identity}Let $\hat{\mathcal{M}}:\hat{\mathbb{Z}}_{p}^{r}\rightarrow\mathbb{C}_{q}^{d,d}$ be any function. Then: \begin{equation} \sum_{0<\left\Vert \mathbf{t}\right\Vert _{p}\leq p^{N}}v_{p}\left(\mathbf{t}\right)\hat{\mathcal{M}}\left(\mathbf{t}\right)e^{2\pi i\left\{ \mathbf{t}\mathbf{z}\right\} _{p}}\overset{\mathbb{C}_{q}^{d,d}}{=}-N\tilde{\mathcal{M}}_{N}\left(\mathbf{z}\right)+\sum_{n=0}^{N-1}\tilde{\mathcal{M}}_{n}\left(\mathbf{z}\right)\label{eq:MD Fourier sum of v_p times mu-hat} \end{equation} \end{prop} Proof: \begin{align*} \sum_{0<\left\Vert \mathbf{t}\right\Vert _{p}\leq p^{N}}v_{p}\left(\mathbf{t}\right)\hat{\mathcal{M}}\left(\mathbf{t}\right)e^{2\pi i\left\{ \mathbf{t}\mathbf{z}\right\} } & =\sum_{n=1}^{N}\sum_{\left\Vert \mathbf{t}\right\Vert _{p}=p^{n}}v_{p}\left(\mathbf{t}\right)\hat{\mathcal{M}}\left(\mathbf{t}\right)e^{2\pi i\left\{ \mathbf{t}\mathbf{z}\right\} _{p}}\\ & =-\sum_{n=1}^{N}n\sum_{\left\Vert \mathbf{t}\right\Vert _{p}=p^{n}}\hat{\mathcal{M}}\left(\mathbf{t}\right)e^{2\pi i\left\{ \mathbf{t}\mathbf{z}\right\} _{p}}\\ & =-\sum_{n=1}^{N}n\left(\tilde{\mathcal{M}}_{n}\left(\mathbf{z}\right)-\tilde{\mathcal{M}}_{n-1}\left(\mathbf{z}\right)\right) \end{align*} The rest of the proof is formally identical to its one-dimensional counterpart (\textbf{Proposition \ref{prop:v_p of t times mu hat sum}}). Q.E.D. \chapter{\label{chap:6 A-Study-of}A Study of $\chi_{H}$ - The Multi-Dimensional Case} \includegraphics[scale=0.45]{./PhDDissertationEroica6.png} \vphantom{} IN THIS CHAPTER, UNLESS STATED OTHERWISE, WE ASSUME $p$ IS A PRIME AND THAT $H$ IS A CONTRACTING, SEMI-BASIC $p$-SMOOTH $d$-DIMENSIONAL DEPTH $r$ HYDRA MAP WHICH FIXES $\mathbf{0}$. \vphantom{} The layout of Chapter 6 is much like that of its one-dimensional predecessor\textemdash Chapter 4. After an initial string of notational definitions and preparatory work, we investigate $\chi_{H}$ by squeezing out explicit ``asymptotic formulae'' for the Fourier transform of the $N$th truncation of $\chi_{H}$; this is the content of Subsection \ref{subsec:6.2.1 -and-}, with the formulae themselves occurring in \textbf{Theorem \ref{thm:MD N,t asympotics for Chi_H,N hat}}. Just as $1-\alpha_{H}\left(0\right)$ was the key determinative quantity for the one-dimensional case, so too will $\mathbf{I}_{d}-\alpha_{H}\left(\mathbf{0}\right)$ be a key quantity for the multi-dimensional case, where $\alpha_{H}$ (defined at the start of Section \ref{sec:6.1. Preparatory-Work-(Again)}) is the multi-dimensional generalization of the one-dimensional $\alpha_{H}$. However, the non-commutativity of matrix multiplication will, unfortunately, complicate matters somewhat. As a result, at the beginning of \ref{sec:6.1. Preparatory-Work-(Again)}, I introduce a qualitative notion of the \textbf{commutativity }of a $p$-Hydra map $H$. This condition turns out to be exactly what is needed in order to minimize the computational differences between the one-dimensional case and the multi-dimensional case currently under consideration. The primary consequence of this issue is that I have only been able to establish quasi-integrability results and formulae for Fourier transforms of the multi-dimensional $\chi_{H}$ in the case where $H$ is commutative. A treatment of this case is given in Subsection \ref{subsec:6.2.3 Multi-Dimensional--=00003D000026}, following the $\alpha_{H}\left(\mathbf{0}\right)=\mathbf{I}_{d}$ case dealt with in \ref{subsec:6.2.2 Multi-Dimensional--=00003D000026}. That being said, as a headache-preventing prophylaxis for future explorers of this subject, \emph{I have taken the liberty of doing all the major computations without relying on the assumption that $H$ is commutative}. For ease of readability, alongside these non-commutative cases, I also state the simpler commutative cases for all significant formulae. Subsection \ref{subsec:6.2.4 Multi-Dimensional--=00003D000026} contains additional computations leading all the way up to the doorstep of a proof of the quasi-integrability of $\chi_{H}$ for non-commutative $H$, leaving the final step as a conjecture to be tackled in future work. \section{\label{sec:6.1. Preparatory-Work-(Again)}Preparatory Work (Again)} This section is just a multi-dimensional copy of its predecessor in Section \pageref{sec:4.1 Preparatory-Work--}. \begin{defn} We define the functions $\alpha_{H}:\hat{\mathbb{Z}}_{p}^{r}\rightarrow\textrm{GL}_{d}\left(\overline{\mathbb{Q}}\right)$ and $\beta_{H}:\hat{\mathbb{Z}}_{p}^{r}\rightarrow\overline{\mathbb{Q}}^{d}$ by:\nomenclature{$\alpha_{H}\left(\mathbf{t}\right)$}{ }\nomenclature{$\beta_{H}\left(\mathbf{t}\right)$}{ } \begin{equation} \alpha_{H}\left(\mathbf{t}\right)\overset{\textrm{def}}{=}\frac{1}{p^{r}}\sum_{\mathbf{j}=\mathbf{0}}^{p-1}\mathbf{D}_{\mathbf{j}}^{-1}\mathbf{A}_{\mathbf{j}}e^{-2\pi i\mathbf{j}\cdot\mathbf{t}}\label{eq:MD definition of alpha_H} \end{equation} \begin{equation} \beta_{H}\left(\mathbf{t}\right)\overset{\textrm{def}}{=}\frac{1}{p^{r}}\sum_{\mathbf{j}=\mathbf{0}}^{p-1}\mathbf{D}_{\mathbf{j}}^{-1}\mathbf{b}_{\mathbf{j}}e^{-2\pi i\mathbf{j}\cdot\mathbf{t}}\label{eq:MD definition of beta_H} \end{equation} We then write $\gamma_{H}:\hat{\mathbb{Z}}_{p}^{r}\rightarrow\overline{\mathbb{Q}}^{d}$ to denote:\nomenclature{$\gamma_{H}\left(\mathbf{t}\right)$}{ } \begin{equation} \gamma_{H}\left(\mathbf{t}\right)\overset{\textrm{def}}{=}\left(\alpha_{H}\left(\mathbf{t}\right)\right)^{-1}\beta_{H}\left(\mathbf{t}\right)\label{eq:MD definition of gamma_H} \end{equation} \end{defn} \begin{defn} We say $H$ is \textbf{non-singular }whenever the matrix $\alpha_{H}\left(\mathbf{j}/p\right)$ is invertible for all $\mathbf{j}\in\mathbb{Z}^{r}/p\mathbb{Z}^{r}$. \index{Hydra map!non-singular} \end{defn} \vphantom{} The primary distinction between the work we will do here and the one-dimensional case comes from the non-commutativity of matrix multiplication. Despite this, there is a simple\textemdash and not unreasonable\textemdash qualitative condition we can place on $H$ to make the computational distinctions between the one-dimensional and multi-dimensional cases all-but-trivial: \begin{defn} We say $H$ is\index{Hydra map!commutative} \textbf{commutative }whenever $\alpha_{H}\left(\mathbf{0}\right)$ commutes with $H^{\prime}\left(\mathbf{0}\right)$: \begin{equation} \alpha_{H}\left(\mathbf{0}\right)H^{\prime}\left(\mathbf{0}\right)=H^{\prime}\left(\mathbf{0}\right)\alpha_{H}\left(\mathbf{0}\right)\label{eq:Definition of H commutativity} \end{equation} \end{defn} \vphantom{} Next, we introduce the multi-dimensional analogue of $\kappa_{H}$, here a matrix-valued function. \begin{defn} \nomenclature{$\kappa_{H}\left(\mathbf{n}\right)$}{ }We define the function $\kappa_{H}:\mathbb{N}_{0}^{r}\rightarrow\textrm{GL}_{d}\left(\mathbb{Q}\right)$ by: \begin{equation} \kappa_{H}\left(\mathbf{n}\right)\overset{\textrm{def}}{=}M_{H}\left(\mathbf{n}\right)\left(H^{\prime}\left(\mathbf{0}\right)\right)^{-\lambda_{p}\left(\mathbf{n}\right)},\textrm{ }\forall\mathbf{n}\in\mathbb{N}_{0}^{r}\label{eq:MD definition of Kappa_H} \end{equation} \end{defn} \vphantom{} \textbf{Proposition \ref{prop:MD generating function}}, given below, contains the multi-dimensional analogue of the generating function identities from Chapter \ref{chap:4 A-Study-of}'s \textbf{Proposition \ref{prop:Generating function identities}}. \begin{prop}[\textbf{A Generating Function Identity}] \label{prop:MD generating function}For all $n\geq1$, all scalars $z$, and all $\mathbf{t}\in\hat{\mathbb{Z}}_{p}^{r}$: \begin{equation} \prod_{m=0}^{n-1}\left(\sum_{\mathbf{j}=\mathbf{0}}^{p-1}\frac{\mathbf{A}_{\mathbf{j}}}{\mathbf{D}_{\mathbf{j}}}z^{\mathbf{j}\cdot p^{m}\mathbf{t}}\right)=\sum_{\mathbf{m}=\mathbf{0}}^{p^{n}-1}M_{H}\left(\mathbf{m}\right)\left(H^{\prime}\left(\mathbf{0}\right)\right)^{n-\lambda_{p}\left(\mathbf{m}\right)}z^{\mathbf{m}\cdot\mathbf{t}}\label{eq:MD M_H partial sum generating identity} \end{equation} \end{prop} Proof: Each term on the right of (\ref{eq:MD M_H partial sum generating identity}) is obtained by taking a product of the form: \begin{equation} \prod_{\ell=0}^{n-1}\frac{\mathbf{A}_{\mathbf{j}_{\ell}}}{\mathbf{D}_{\mathbf{j}_{\ell}}}z^{\mathbf{j}_{\ell}\cdot p^{\ell}\mathbf{t}}=\left(\prod_{\ell=0}^{n-1}\frac{\mathbf{A}_{\mathbf{j}_{\ell}}}{\mathbf{D}_{\mathbf{j}_{\ell}}}\right)z^{\mathbf{t}\cdot\sum_{\ell=0}^{n-1}p^{\ell}\mathbf{j}_{\ell}} \end{equation} for some choice of $\mathbf{j}_{1},\ldots,\mathbf{j}_{n-1}\in\mathbb{Z}^{r}/p\mathbb{Z}^{r}$. Here, writing $\mathbf{j}_{\ell}=\left(j_{\ell,1},\ldots,j_{\ell,r}\right)$ we have: \begin{equation} \sum_{\ell=0}^{n-1}\mathbf{j}_{\ell}p^{\ell}=\left(\sum_{\ell=0}^{n-1}j_{\ell,1}p^{\ell},\ldots,\sum_{\ell=0}^{n-1}j_{\ell,r}p^{\ell}\right) \end{equation} So, writing $\mathbf{J}$ to denote the block string $\left(\mathbf{j}_{0},\ldots,\mathbf{j}_{n-1}\right)$, we define $\mathbf{m}\in\mathbb{N}_{0}^{r}$ by: \begin{equation} \mathbf{m}\overset{\textrm{def}}{=}\sum_{\ell=0}^{n-1}\mathbf{j}_{\ell}p^{\ell}\label{eq:definition of bold m} \end{equation} so that $\mathbf{m}$ is then represented by $\mathbf{J}$. As such: \begin{equation} \prod_{m=0}^{n-1}\left(\sum_{\mathbf{j}=\mathbf{0}}^{-1}\frac{\mathbf{A}_{\mathbf{j}}}{\mathbf{D}_{\mathbf{j}}}z^{\mathbf{j}\cdot\left(p^{m}\mathbf{t}\right)}\right)=\sum_{\begin{array}{c} \mathbf{J}\in\textrm{String}^{r}\left(p\right)\\ \left|\mathbf{J}\right|=n \end{array}}\left(\frac{\mathbf{A}_{\mathbf{j}_{0}}}{\mathbf{D}_{\mathbf{j}_{0}}}\times\cdots\times\frac{\mathbf{A}_{\mathbf{j}_{n-1}}}{\mathbf{D}_{\mathbf{j}_{n-1}}}\right)z^{\mathbf{J}\cdot\mathbf{t}} \end{equation} As an illustrative example, let $n=4$, consider: \begin{equation} \mathbf{J}=\left(\mathbf{j}_{0},\mathbf{j}_{1},\mathbf{0},\mathbf{0}\right) \end{equation} and let $\mathbf{m}$ be the tuple represented by this $\mathbf{J}$. Note that this $\mathbf{J}$ is the unique \emph{length $4$ }block string representing $\mathbf{m}$; any other length $4$ block string will have a non-zero tuple in either the $\mathbf{j}_{2}$ slot or the $\mathbf{j}_{3}$ slot. More generally, there will be a bijection between the set of all length $n$ block strings and the set of all $r$-tuples of integers \emph{representable }by said block strings. This set of $r$-tuples of integers is precisely: \begin{equation} \left\{ \left(m_{1},\ldots,m_{r}\right):0\leq m_{\ell}\leq p^{n}-1\textrm{ }\forall\ell\in\left\{ 1,\ldots,r\right\} \right\} \end{equation} More compactly, this is exactly the range of tuples indicated by the summation notation: \begin{equation} \sum_{\mathbf{m}=\mathbf{0}}^{p^{N}-1} \end{equation} The problem here is that any terminal $\mathbf{0}$-tuples in $\mathbf{J}$ they affect the matrix product associated to $\mathbf{J}$: \begin{equation} \frac{\mathbf{A}_{\mathbf{j}_{0}}}{\mathbf{D}_{\mathbf{j}_{0}}}\times\cdots\times\frac{\mathbf{A}_{\mathbf{j}_{n-1}}}{\mathbf{D}_{\mathbf{j}_{n-1}}} \end{equation} even though those $\mathbf{0}$-tuples do \emph{not} affect $z^{\mathbf{J}\cdot\mathbf{t}}$. For example, for $\mathbf{J}=\left(\mathbf{j}_{0},\mathbf{j}_{1},\mathbf{0},\mathbf{0}\right)$, the product would be: \begin{align*} \frac{\mathbf{A}_{\mathbf{j}_{0}}}{\mathbf{D}_{\mathbf{j}_{0}}}\times\frac{\mathbf{A}_{\mathbf{j}_{1}}}{\mathbf{D}_{\mathbf{j}_{1}}}\times\frac{\mathbf{A}_{\mathbf{0}}}{\mathbf{D}_{\mathbf{0}}}\times\frac{\mathbf{A}_{\mathbf{0}}}{\mathbf{D}_{\mathbf{0}}} & =M_{H}\left(\left(\mathbf{j}_{0},\mathbf{j}_{1}\right)\right)\times\left(\frac{\mathbf{A}_{\mathbf{0}}}{\mathbf{D}_{\mathbf{0}}}\right)^{2}\\ \left(\mathbf{m}\textrm{ is represented by }\left(\mathbf{j}_{0},\mathbf{j}_{1}\right)\right); & =M_{H}\left(\mathbf{m}\right)\times\left(H^{\prime}\left(\mathbf{0}\right)\right)^{2} \end{align*} So, we need to modify the product to take into account terminal $\mathbf{0}$s in $\mathbf{J}$. To do this, consider the worst-case scenario where $\mathbf{J}$ is a length-$n$ block string whose every $\mathbf{j}$ is $\mathbf{0}$. Letting $\mathbf{m}$ be the unique $r$-tuple of integers represented by an arbitrary length-$n$ $\mathbf{J}$, observe that $\lambda_{p}\left(\mathbf{m}\right)$ is the number of tuples in $\mathbf{J}$ which are \emph{not }part of a possible run of consecutive terminal $\mathbf{0}$s. This is because $\lambda_{p}\left(\mathbf{m}\right)$ is the length of the shortest block string representing $\mathbf{m}$. Consequently, for each $\mathbf{J}$, we have: \begin{equation} \frac{\mathbf{A}_{\mathbf{j}_{0}}}{\mathbf{D}_{\mathbf{j}_{0}}}\times\cdots\times\frac{\mathbf{A}_{\mathbf{j}_{n-1}}}{\mathbf{D}_{\mathbf{j}_{n-1}}}=M_{H}\left(\mathbf{m}\right)\left(H^{\prime}\left(\mathbf{0}\right)\right)^{n-\lambda\left(\mathbf{m}\right)} \end{equation} which leaves us with: \begin{align*} \prod_{m=0}^{n-1}\left(\sum_{\mathbf{j}=\mathbf{0}}^{-1}\frac{\mathbf{A}_{\mathbf{j}}}{\mathbf{D}_{\mathbf{j}}}z^{\mathbf{j}\cdot\left(^{m}\mathbf{t}\right)}\right) & =\sum_{\begin{array}{c} \mathbf{J}\in\textrm{String}^{r}\left(p\right)\\ \left|\mathbf{J}\right|=n \end{array}}\left(\frac{\mathbf{A}_{\mathbf{j}_{0}}}{\mathbf{D}_{\mathbf{j}_{0}}}\times\cdots\times\frac{\mathbf{A}_{\mathbf{j}_{n-1}}}{\mathbf{D}_{\mathbf{j}_{n-1}}}\right)z^{\mathbf{J}\cdot\mathbf{t}}\\ & =\sum_{\mathbf{m}=\mathbf{0}}^{^{n}-1}M_{H}\left(\mathbf{m}\right)\left(H^{\prime}\left(\mathbf{0}\right)\right)^{n-\lambda\left(\mathbf{m}\right)}z^{\mathbf{m}\cdot\mathbf{t}} \end{align*} which is, of course, our sought-after identity. Q.E.D. \vphantom{} In the process of writing this chapter, I took pains to devise a notation which would make multi-dimensional case's computations as near as possible to verbatim repeats of their one-dimensional predecessors. The principal difficulty our notation must overcome is the non-commutativity of matrix multiplication. This manifests most strongly in the functional equations for the multi-dimensional $\kappa_{H}$. In order to keep our computations from spilling out into the margins of the page\textemdash or beyond\textemdash we will need to introduce a notation for the linear operator which conjugates $d\times d$ matrices by the $n$th power of $H^{\prime}\left(\mathbf{0}\right)$. \begin{defn} For any $n\in\mathbb{N}_{0}$ and any $\mathbf{A}\in\textrm{GL}_{d}\left(\overline{\mathbb{Q}}\right)$, $\textrm{GL}_{d}\left(\mathbb{C}\right)$, or $\textrm{GL}_{d}\left(\mathbb{C}_{q}\right)$, we define: \nomenclature{$\mathcal{C}_{H}\left(\mathbf{A}:n\right)$}{$\overset{\textrm{def}}{=}\left(H^{\prime}\left(\mathbf{0}\right)\right)^{n}\mathbf{A}\left(H^{\prime}\left(\mathbf{0}\right)\right)^{-n}$} \begin{equation} \mathcal{C}_{H}\left(\mathbf{A}:n\right)\overset{\textrm{def}}{=}\left(H^{\prime}\left(\mathbf{0}\right)\right)^{n}\mathbf{A}\left(H^{\prime}\left(\mathbf{0}\right)\right)^{-n}\label{eq:Definition of script C_H} \end{equation} \end{defn} \begin{rem} The most important thing to note is that: \begin{equation} \mathcal{C}_{H}\left(\mathbf{I}_{d}:n\right)=\mathbf{I}_{d}\label{eq:Script C_H of I_d} \end{equation} This property is responsible for the impact had on the dynamics of $H$ when $\alpha_{H}\left(\mathbf{0}\right)=\mathbf{I}_{d}$ (the multi-dimensional analogue of the ``$\alpha_{H}\left(0\right)=1$'' case.). Additionally, note that $\mathcal{C}_{H}\left(\alpha_{H}\left(\mathbf{0}\right):n\right)=\alpha_{H}\left(\mathbf{0}\right)$ whenever $H$ is commutative. \end{rem} \begin{lem}[\textbf{Properties of Multi-Dimensional $\kappa_{H}$}] \label{lem:properties of MD kappa_H}\ \vphantom{} I. \begin{equation} \sum_{\mathbf{j}>\mathbf{0}}^{p-1}\kappa_{H}\left(\mathbf{j}\right)=\alpha_{H}\left(\mathbf{0}\right)\left(\frac{H^{\prime}\left(\mathbf{0}\right)}{p^{r}}\right)^{-1}-\mathbf{I}_{d}\label{eq:MD kappa H sum in terms of MD alpha} \end{equation} \vphantom{} II. $\kappa_{H}$ is the unique function $\mathbb{N}_{0}^{r}\rightarrow\textrm{GL}_{d}\left(\mathbb{Q}\right)$ satisfying the functional equations: \begin{equation} \kappa_{H}\left(p\mathbf{n}+\mathbf{j}\right)=H_{\mathbf{j}}^{\prime}\left(\mathbf{0}\right)\kappa_{H}\left(\mathbf{n}\right)\left(H^{\prime}\left(\mathbf{0}\right)\right)^{-1},\textrm{ }\forall\mathbf{n}\in\mathbb{N}_{0}^{d},\textrm{ }\forall\mathbf{j}\leq p-1\label{eq:MD Kappa_H functional equations} \end{equation} subject to the initial condition $\kappa_{H}\left(\mathbf{0}\right)=\mathbf{I}_{d}$. \vphantom{} III. $\kappa_{H}$ satisfies: \begin{equation} \kappa_{H}\left(\mathbf{m}+p^{k}\mathbf{j}\right)=\kappa_{H}\left(\mathbf{m}\right)\left(H^{\prime}\left(\mathbf{0}\right)\right)^{\lambda_{p}\left(\mathbf{m}\right)}\kappa_{H}\left(\mathbf{j}\right)\left(H^{\prime}\left(\mathbf{0}\right)\right)^{-\lambda_{p}\left(\mathbf{m}\right)}\label{eq:MD Kappa_H has P-adic structure} \end{equation} for all $k\in\mathbb{N}_{1}$, all $\mathbf{m}\leq p^{k}-1$, and all $\mathbf{j}\leq p-1$. Equivalently, for these parameters: \begin{equation} \kappa_{H}\left(\mathbf{m}+p^{k}\mathbf{j}\right)=\kappa_{H}\left(\mathbf{m}\right)\mathcal{C}_{H}\left(\kappa_{H}\left(\mathbf{j}\right):\lambda_{p}\left(\mathbf{m}\right)\right)\label{eq:MD Kappa_H has P-adic structure, with script C_H} \end{equation} \vphantom{} IV. If $H$ is semi-basic, then $\kappa_{H}$ is $\left(p,q_{H}\right)$-adically regular, with: \begin{equation} \lim_{n\rightarrow\infty}\left\Vert \kappa_{H}\left(\left[\mathbf{z}\right]_{p^{n}}\right)\right\Vert _{q_{H}}\overset{\mathbb{R}}{=}0,\textrm{ }\forall\mathbf{z}\in\left(\mathbb{Z}_{p}^{r}\right)^{\prime}\label{eq:MD Semi-basic q-adic decay for Kappa_H} \end{equation} \vphantom{} V. If $H$ is contracting, then: \begin{equation} \lim_{N\rightarrow\infty}\kappa_{H}\left(\left[\mathbf{z}\right]_{p^{N}}\right)\left(H^{\prime}\left(\mathbf{0}\right)\right)^{N}\overset{\mathbb{R}^{d,d}}{=}\mathbf{O}_{d},\textrm{ }\forall\mathbf{z}\in\mathbb{N}_{0}^{r}\label{eq:MD Kappa_H decay when H is contracting} \end{equation} \end{lem} Proof: I. \begin{equation} \sum_{\mathbf{j}>\mathbf{0}}^{p-1}\kappa_{H}\left(\mathbf{j}\right)=\sum_{\mathbf{j}>\mathbf{0}}^{p-1}M_{H}\left(\mathbf{j}\right)\left(H^{\prime}\left(\mathbf{0}\right)\right)^{-\lambda_{p}\left(\mathbf{j}\right)} \end{equation} Since $\lambda_{p}\left(\mathbf{j}\right)=1$ for all $\mathbf{j}\in\left(\mathbb{Z}^{r}/p\mathbb{Z}^{r}\right)\backslash\left\{ \mathbf{0}\right\} $, and since: \begin{equation} M_{H}\left(\mathbf{j}\right)=\mathbf{D}_{\mathbf{j}}^{-1}\mathbf{A}_{\mathbf{j}} \end{equation} we have: \begin{align*} \sum_{\mathbf{j}>\mathbf{0}}^{p-1}\kappa_{H}\left(\mathbf{j}\right) & =\sum_{\mathbf{j}>\mathbf{0}}^{p-1}\mathbf{D}_{\mathbf{j}}^{-1}\mathbf{A}_{\mathbf{j}}\left(H^{\prime}\left(\mathbf{0}\right)\right)^{-1}\\ & =\frac{1}{p^{r}}\sum_{\mathbf{j}>\mathbf{0}}^{p-1}\mathbf{D}_{\mathbf{j}}^{-1}\mathbf{A}_{\mathbf{j}}\left(\frac{H^{\prime}\left(\mathbf{0}\right)}{p^{r}}\right)^{-1}\\ \left(\mathbf{D}_{\mathbf{0}}^{-1}\mathbf{A}_{\mathbf{0}}=H^{\prime}\left(\mathbf{0}\right)\right); & =\frac{1}{p^{r}}\left(-H^{\prime}\left(\mathbf{0}\right)+\sum_{\mathbf{j}=\mathbf{0}}^{-1}\mathbf{D}_{\mathbf{j}}^{-1}\mathbf{A}_{\mathbf{j}}\right)\left(\frac{H^{\prime}\left(\mathbf{0}\right)}{p^{r}}\right)^{-1}\\ & =-\frac{H^{\prime}\left(\mathbf{0}\right)}{p^{r}}\left(\frac{H^{\prime}\left(\mathbf{0}\right)}{p^{r}}\right)^{-1}+\underbrace{\left(\frac{1}{p^{r}}\sum_{\mathbf{j}=\mathbf{0}}^{p-1}\mathbf{D}_{\mathbf{j}}^{-1}\mathbf{A}_{\mathbf{j}}\right)}_{\alpha_{H}\left(\mathbf{0}\right)}\left(\frac{H^{\prime}\left(\mathbf{0}\right)}{p^{r}}\right)^{-1}\\ & =-\mathbf{I}_{d}+\alpha_{H}\left(\mathbf{0}\right)\left(\frac{H^{\prime}\left(\mathbf{0}\right)}{p^{r}}\right)^{-1} \end{align*} \vphantom{} II. Let $\mathbf{n}\in\mathbb{N}_{0}^{r}\backslash\left\{ \mathbf{0}\right\} $ and let $\mathbf{j}\in\mathbb{Z}^{r}/p\mathbb{Z}^{r}$. Then: \begin{align*} \lambda_{p}\left(p\mathbf{n}+\mathbf{j}\right) & =\lambda_{p}\left(\mathbf{n}\right)+1\\ M_{H}\left(p\mathbf{n}+\mathbf{j}\right) & =M_{H}\left(\mathbf{n}\right)\frac{\mathbf{A}_{\mathbf{j}}}{\mathbf{D}_{\mathbf{j}}}=M_{H}\left(\mathbf{n}\right)\mathbf{D}_{\mathbf{j}}^{-1}\mathbf{A}_{\mathbf{j}} \end{align*} As such: \begin{align*} \kappa_{H}\left(p\mathbf{n}+\mathbf{j}\right) & =M_{H}\left(p\mathbf{n}+\mathbf{j}\right)\left(H^{\prime}\left(\mathbf{0}\right)\right)^{-\lambda_{p}\left(p\mathbf{n}+\mathbf{j}\right)}\\ & =\left(\frac{\mathbf{A}_{\mathbf{j}}}{\mathbf{D}_{\mathbf{j}}}M_{H}\left(\mathbf{n}\right)\right)\left(H^{\prime}\left(\mathbf{0}\right)\right)^{-\lambda_{p}\left(\mathbf{n}\right)-1}\\ & =\mathbf{D}_{\mathbf{j}}^{-1}\mathbf{A}_{\mathbf{j}}\overbrace{\left(M_{H}\left(\mathbf{n}\right)\left(H^{\prime}\left(\mathbf{0}\right)\right)^{-\lambda_{p}\left(\mathbf{n}\right)}\right)}^{\kappa_{H}\left(\mathbf{n}\right)}\left(H^{\prime}\left(\mathbf{0}\right)\right)^{-1}\\ \left(H^{\prime}\left(\mathbf{0}\right)=\mathbf{D}_{\mathbf{0}}^{-1}\mathbf{A}_{\mathbf{0}}\right); & =\mathbf{D}_{\mathbf{j}}^{-1}\mathbf{A}_{\mathbf{j}}\kappa_{H}\left(\mathbf{n}\right)\mathbf{A}_{\mathbf{0}}^{-1}\mathbf{D}_{\mathbf{0}} \end{align*} Next: \begin{align*} \kappa_{H}\left(\mathbf{j}\right) & =\begin{cases} M_{H}\left(\mathbf{0}\right)\left(H^{\prime}\left(\mathbf{0}\right)\right)^{-0} & \textrm{if }\mathbf{j}=\mathbf{0}\\ M_{H}\left(\mathbf{j}\right)\left(H^{\prime}\left(\mathbf{0}\right)\right)^{-1} & \textrm{if }\mathbf{j}\in\left(\mathbb{Z}^{r}/p\mathbb{Z}^{r}\right)\backslash\left\{ \mathbf{0}\right\} \end{cases}\\ & =\begin{cases} \mathbf{I}_{d} & \textrm{if }\mathbf{j}=\mathbf{0}\\ \frac{\mathbf{A}_{\mathbf{j}}}{\mathbf{D}_{\mathbf{j}}}\frac{\mathbf{A}_{\mathbf{0}}}{\mathbf{D}_{\mathbf{0}}} & \textrm{if }\mathbf{j}\in\left(\mathbb{Z}^{r}/p\mathbb{Z}^{r}\right)\backslash\left\{ \mathbf{0}\right\} \end{cases} \end{align*} where: \begin{equation} \frac{\mathbf{A}_{\mathbf{j}}}{\mathbf{D}_{\mathbf{j}}}\frac{\mathbf{A}_{\mathbf{0}}}{\mathbf{D}_{\mathbf{0}}}=\mathbf{D}_{\mathbf{j}}^{-1}\mathbf{A}_{\mathbf{j}}\mathbf{D}_{\mathbf{0}}^{-1}\mathbf{A}_{\mathbf{0}} \end{equation} Since this shows $\kappa_{H}\left(\mathbf{0}\right)=\mathbf{I}_{d}$, we can then write: \begin{equation} \kappa_{H}\left(\mathbf{j}\right)=\frac{\mathbf{A}_{\mathbf{j}}}{\mathbf{D}_{\mathbf{j}}}\kappa_{H}\left(\mathbf{0}\right),\textrm{ }\forall\mathbf{j}\leq p-1 \end{equation} Combining this last equation with the other cases computed above yields (\ref{eq:MD Kappa_H functional equations}). The uniqueness follows by an inductive argument, showing that the initial condition $\kappa_{H}\left(\mathbf{0}\right)=\mathbf{I}_{d}$ uniquely determines solutions of the functional equation (\ref{eq:MD Kappa_H functional equations}) on $\mathbb{N}_{0}^{r}$. Finally, we put things in the desired form by recalling that $H_{\mathbf{j}}^{\prime}\left(\mathbf{0}\right)=\mathbf{D}_{\mathbf{j}}^{-1}\mathbf{A}_{\mathbf{j}}$ for all $\mathbf{j}$, and that $H^{\prime}\left(\mathbf{0}\right)=H_{\mathbf{0}}^{\prime}\left(\mathbf{0}\right)$. \vphantom{} III. Let $\mathbf{j}\in\mathbb{Z}^{r}/p\mathbb{Z}^{r}$ and $k\geq1$ be arbitrary, and let $\mathbf{m}\in\mathbb{N}_{0}^{r}$ satisfy $\lambda\left(\mathbf{m}\right)\leq k$ By definition of multi-dimensional $\kappa_{H}$ (equation (\ref{eq:MD definition of Kappa_H}) and using the functional equations for $M_{H}$ (\textbf{Proposition \ref{prop:MD M_H functional equations}}) and $\lambda_{p}$ (\textbf{Proposition \ref{prop:MD lambda and digit-number functional equations}}) \begin{align*} \kappa_{H}\left(\mathbf{m}+p^{k}\mathbf{j}\right) & =M_{H}\left(\mathbf{m}+p^{k}\mathbf{j}\right)\left(H^{\prime}\left(\mathbf{0}\right)\right)^{-\lambda_{p}\left(\mathbf{m}+p^{k}\mathbf{j}\right)}\\ & =M_{H}\left(\mathbf{m}\right)M_{H}\left(p^{k}\mathbf{j}\right)\left(H^{\prime}\left(\mathbf{0}\right)\right)^{-\lambda_{p}\left(\mathbf{m}\right)-\lambda_{p}\left(\mathbf{j}\right)}\\ \left(M_{H}\left(p^{k}\mathbf{j}\right)=M_{H}\left(\mathbf{j}\right)\right); & =M_{H}\left(\mathbf{m}\right)M_{H}\left(\mathbf{j}\right)\left(H^{\prime}\left(\mathbf{0}\right)\right)^{-\lambda_{p}\left(\mathbf{m}\right)-\lambda_{p}\left(\mathbf{j}\right)}\\ & =M_{H}\left(\mathbf{m}\right)\underbrace{M_{H}\left(\mathbf{j}\right)\left(H^{\prime}\left(\mathbf{0}\right)\right)^{-\lambda_{p}\left(\mathbf{j}\right)}}_{\kappa_{H}\left(\mathbf{j}\right)}\left(H^{\prime}\left(\mathbf{0}\right)\right)^{-\lambda_{p}\left(\mathbf{m}\right)}\\ & =M_{H}\left(\mathbf{m}\right)\kappa_{H}\left(\mathbf{j}\right)\left(H^{\prime}\left(\mathbf{0}\right)\right)^{-\lambda_{p}\left(\mathbf{m}\right)} \end{align*} Inserting $\mathbf{I}_{d}=\left(H^{\prime}\left(\mathbf{0}\right)\right)^{-\lambda_{p}\left(\mathbf{m}\right)}\left(H^{\prime}\left(\mathbf{0}\right)\right)^{\lambda_{p}\left(\mathbf{m}\right)}$ in between $M_{H}\left(\mathbf{m}\right)$ and $\kappa_{H}\left(\mathbf{j}\right)$ yields: \begin{align*} \kappa_{H}\left(\mathbf{m}+p^{k}\mathbf{j}\right) & =\underbrace{M_{H}\left(\mathbf{m}\right)\left(H^{\prime}\left(\mathbf{0}\right)\right)^{-\lambda_{p}\left(\mathbf{m}\right)}}_{\kappa_{H}\left(\mathbf{m}\right)}\left(H^{\prime}\left(\mathbf{0}\right)\right)^{\lambda_{p}\left(\mathbf{m}\right)}\kappa_{H}\left(\mathbf{j}\right)\left(H^{\prime}\left(\mathbf{0}\right)\right)^{-\lambda_{p}\left(\mathbf{m}\right)}\\ & =\kappa_{H}\left(\mathbf{m}\right)\left(H^{\prime}\left(\mathbf{0}\right)\right)^{\lambda_{p}\left(\mathbf{m}\right)}\kappa_{H}\left(\mathbf{j}\right)\left(H^{\prime}\left(\mathbf{0}\right)\right)^{-\lambda_{p}\left(\mathbf{m}\right)} \end{align*} as desired. \vphantom{} IV. Applying $q$-adic norm to (\ref{eq:MD Kappa_H has P-adic structure}) gives us: \begin{align*} \left\Vert \kappa_{H}\left(\mathbf{m}+p^{k}\mathbf{j}\right)\right\Vert _{q} & \leq\left\Vert \kappa_{H}\left(\mathbf{m}\right)\right\Vert _{q}\left\Vert H^{\prime}\left(\mathbf{0}\right)\right\Vert _{q}^{\lambda_{p}\left(\mathbf{m}\right)}\left\Vert \kappa_{H}\left(\mathbf{j}\right)\right\Vert _{q}\left\Vert H^{\prime}\left(\mathbf{0}\right)\right\Vert _{q}^{-\lambda_{p}\left(\mathbf{m}\right)}\\ & =\left\Vert \kappa_{H}\left(\mathbf{m}\right)\right\Vert _{q}\left\Vert \kappa_{H}\left(\mathbf{j}\right)\right\Vert _{q}\\ & =\left\Vert M_{H}\left(\mathbf{m}\right)\left(H^{\prime}\left(\mathbf{0}\right)\right)^{-\lambda_{p}\left(\mathbf{m}\right)}\right\Vert _{q}\left\Vert M_{H}\left(\mathbf{j}\right)\left(H^{\prime}\left(\mathbf{0}\right)\right)^{-\lambda_{p}\left(\mathbf{j}\right)}\right\Vert _{q}\\ & \leq\left\Vert M_{H}\left(\mathbf{m}\right)\right\Vert _{q}\left\Vert M_{H}\left(\mathbf{j}\right)\right\Vert _{q}\left\Vert H^{\prime}\left(\mathbf{0}\right)\right\Vert _{q}^{-\lambda_{p}\left(\mathbf{m}\right)-\lambda_{p}\left(\mathbf{j}\right)} \end{align*} Since $H$ is semi-basic, $\left\Vert H^{\prime}\left(\mathbf{0}\right)\right\Vert _{q}=1$, and so: \begin{equation} \left\Vert \kappa_{H}\left(\mathbf{m}+p^{k}\mathbf{j}\right)\right\Vert _{q}\leq\left\Vert M_{H}\left(\mathbf{m}\right)\right\Vert _{q}\left\Vert M_{H}\left(\mathbf{j}\right)\right\Vert _{q} \end{equation} From this, it follows that for a block string $\mathbf{J}$, the $q$-adic norm $\left\Vert \kappa_{H}\left(\mathbf{J}\right)\right\Vert _{q}$ tends to $0$ as $\mathbf{J}$ converges ($\left|\mathbf{J}\right|\rightarrow\infty$) to any infinite block string representing an element of $\left(\mathbb{Z}_{p}^{r}\right)^{\prime}$. This proves (IV). \vphantom{} V. Let $\mathbf{z}\in\mathbb{N}_{0}^{r}$. Then, $\left[\mathbf{z}\right]_{p^{N}}=\mathbf{z}$ for all $N\geq\lambda_{p}\left(\mathbf{z}\right)$, and so: \begin{equation} \kappa_{H}\left(\left[\mathbf{z}\right]_{p^{N}}\right)\left(H^{\prime}\left(\mathbf{0}\right)\right)^{N}=\kappa_{H}\left(\mathbf{z}\right)\left(H^{\prime}\left(\mathbf{0}\right)\right)^{N},\textrm{ }\forall N\geq\lambda_{p}\left(\mathbf{z}\right) \end{equation} the right-hand side of which tends to the zero matrix $\mathbf{O}_{d}$ in the topology of $\mathbb{R}^{d,d}$ as $N\rightarrow\infty$, seeing as $H$ is contracting. This proves (V). Q.E.D. \begin{prop}[\textbf{Summatory Function of $\chi_{H}$}] \label{prop:MD summatory function formula for Chi_H} \begin{equation} \sum_{\mathbf{n}=\mathbf{0}}^{p^{N}-1}\chi_{H}\left(\mathbf{n}\right)=\begin{cases} \beta_{H}\left(\mathbf{0}\right)Np^{rN} & \textrm{if }\alpha_{H}\left(\mathbf{0}\right)=\mathbf{I}_{d}\\ \frac{\left(\alpha_{H}\left(\mathbf{0}\right)\right)^{N}-1}{\alpha_{H}\left(\mathbf{0}\right)-1}\beta_{H}\left(\mathbf{0}\right)p^{rN} & \textrm{else} \end{cases}\label{eq:MD Summatory formula for Chi_H} \end{equation} \end{prop} Proof: Let: \[ S\left(N\right)=\frac{1}{p^{rN}}\sum_{\mathbf{n}=\mathbf{0}}^{p^{N}-1}\chi_{H}\left(\mathbf{n}\right) \] Then, using $\chi_{H}$'s functional equation (\textbf{Lemma \ref{lem:MD Chi_H functional equations and characterization}}): \begin{align*} p^{rN}S\left(N\right) & =\sum_{\mathbf{n}=\mathbf{0}}^{p^{N}-1}\chi_{H}\left(\mathbf{n}\right)\\ & =\sum_{\mathbf{j}=\mathbf{0}}^{p-1}\sum_{\mathbf{n}=\mathbf{0}}^{p^{N-1}-1}\chi_{H}\left(p\mathbf{n}+\mathbf{j}\right)\\ & =\sum_{\mathbf{j}=\mathbf{0}}^{p-1}\sum_{\mathbf{n}=\mathbf{0}}^{p^{N-1}-1}\frac{\mathbf{A_{j}}\chi_{H}\left(\mathbf{n}\right)+\mathbf{b}_{\mathbf{j}}}{\mathbf{D}_{\mathbf{j}}}\\ & =\sum_{\mathbf{j}=\mathbf{0}}^{p-1}\left(p^{r\left(N-1\right)}\mathbf{D}_{\mathbf{j}}^{-1}\mathbf{A}_{\mathbf{j}}\frac{1}{p^{r\left(N-1\right)}}\sum_{\mathbf{n}=\mathbf{0}}^{p^{N-1}-1}\chi_{H}\left(\mathbf{n}\right)+p^{r\left(N-1\right)}\mathbf{D}_{\mathbf{j}}^{-1}\mathbf{b}_{\mathbf{j}}\right)\\ & =p^{r\left(N-1\right)}\left(\sum_{\mathbf{j}=\mathbf{0}}^{p-1}\mathbf{D}_{\mathbf{j}}^{-1}\mathbf{A}_{\mathbf{j}}\right)S\left(N-1\right)+p^{rN}\sum_{\mathbf{j}=\mathbf{0}}^{p-1}\mathbf{D}_{\mathbf{j}}^{-1}\mathbf{b}_{\mathbf{j}} \end{align*} Dividing through by $p^{rN}$ gives us: \begin{equation} S_{H}\left(N\right)=\alpha_{H}\mathbf{\left(0\right)}S_{H}\left(N-1\right)+\beta_{H}\left(\mathbf{0}\right)\label{eq:MD Recursive formula for S_H} \end{equation} Much like the one-dimensional case, this shows that $S_{H}\left(N\right)$ is the image of $S_{H}\left(0\right)$ under $N$ iterates of an affine linear map (this time, on $\mathbb{Q}^{d}$); specifically: \begin{equation} \mathbf{x}\mapsto\alpha_{H}\left(\mathbf{0}\right)\mathbf{x}+\beta_{H}\left(\mathbf{0}\right)\label{eq:Affine linear map generating S_H-1} \end{equation} This leaves us with two cases: \vphantom{} i. Suppose $\alpha_{H}\left(\mathbf{0}\right)=\mathbf{I}_{d}$. Then, (\ref{eq:Affine linear map generating S_H-1}) reduces to a translation: \begin{equation} \mathbf{x}\mapsto\mathbf{x}+\beta_{H}\left(\mathbf{0}\right) \end{equation} in which case: \begin{equation} S_{H}\left(N\right)=S_{H}\left(0\right)+\beta_{H}\left(\mathbf{0}\right)N\label{eq:MD Explicit formula for S_H of N when alpha =00003D00003D 1} \end{equation} \vphantom{} ii. Suppose $\alpha_{H}\left(\mathbf{0}\right)\neq\mathbf{I}_{d}$, so that (\ref{eq:Affine linear map generating S_H-1}) is not a translation. Using the explicit formula for this $N$th iterate, we obtain: \begin{equation} S_{H}\left(N\right)=\left(\alpha_{H}\left(\mathbf{0}\right)\right)^{N}S_{H}\left(0\right)+\left(\alpha_{H}\left(\mathbf{0}\right)-1\right)^{-1}\left(\left(\alpha_{H}\left(\mathbf{0}\right)\right)^{N}-1\right)\beta_{H}\left(\mathbf{0}\right)\label{eq:MD Explicit formula for S_H of N} \end{equation} Noting that: \begin{align} S_{H}\left(0\right) & =\sum_{\mathbf{n}=\mathbf{0}}^{p^{0}-1}\chi_{H}\left(\mathbf{n}\right)=\chi_{H}\left(\mathbf{0}\right)=\mathbf{0} \end{align} we then have: \begin{equation} \frac{1}{p^{rN}}\sum_{\mathbf{n}=\mathbf{0}}^{p^{N}-1}\chi_{H}\left(\mathbf{n}\right)=S_{H}\left(N\right)=\begin{cases} \beta_{H}\left(\mathbf{0}\right)N & \textrm{if }\alpha_{H}\left(\mathbf{0}\right)=\mathbf{I}_{d}\\ \frac{\left(\alpha_{H}\left(\mathbf{0}\right)\right)^{N}-1}{\alpha_{H}\left(\mathbf{0}\right)-1}\beta_{H}\left(\mathbf{0}\right) & \textrm{else} \end{cases} \end{equation} Q.E.D. \begin{notation} Just as a reminder, we write $q$ to denote $q_{H}$. Like in the one-dimensional case, we write $\chi_{H,N}:\mathbb{Z}_{p}^{r}\rightarrow\mathbb{Q}^{d}\subset\mathbb{Q}_{q}^{d}$ to denote the $N$th truncation of $\chi_{H}$: \begin{equation} \chi_{H,N}\left(\mathbf{z}\right)\overset{\textrm{def}}{=}\sum_{\mathbf{n}=\mathbf{0}}^{p^{N}-1}\chi_{H}\left(\mathbf{n}\right)\left[\mathbf{z}\overset{p^{N}}{\equiv}\mathbf{n}\right],\textrm{ }\forall\mathbf{z}\in\mathbb{Z}_{p}^{r},\textrm{ }\forall N\in\mathbb{N}_{0}\label{eq:MD Definition of Nth truncation of Chi_H} \end{equation} Recall that: \begin{equation} \chi_{H,N}\left(\mathbf{n}\right)\overset{\mathbb{Q}^{d}}{=}\chi_{H}\left(\mathbf{n}\right),\textrm{ }\forall\mathbf{n}\leq p^{N}-1 \end{equation} In particular: \begin{equation} \chi_{H}\left(\mathbf{n}\right)=\chi_{H,\lambda_{p}\left(\mathbf{n}\right)}\left(\mathbf{n}\right),\textrm{ }\forall\mathbf{n}\in\mathbb{N}_{0}^{r} \end{equation} Here, $\chi_{H,N}$ is a $\mathbb{Q}^{d}$-valued function which is a linear combination of finitely many indicator functions for clopen subsets of $\mathbb{Z}_{p}^{r}$. As such, $\chi_{H,N}$ is continuous both as a function $\mathbb{Z}_{p}^{r}\rightarrow\mathbb{C}_{q}^{d}$ and as a function $\mathbb{Z}_{p}^{r}\rightarrow\mathbb{C}^{d}$; in fact, $\chi_{H,N}$ is continuous on $K^{d}$ for any topological field $K$ containing $\mathbb{Q}$. As a result, of this, in writing $\hat{\chi}_{H,N}:\hat{\mathbb{Z}}_{p}^{r}\rightarrow\overline{\mathbb{Q}}^{d}$ to denote the Fourier Transform of $\chi_{H,N}$: \begin{equation} \hat{\chi}_{H,N}\left(\mathbf{t}\right)\overset{\textrm{def}}{=}\int_{\mathbb{Z}_{p}^{r}}\chi_{H,N}\left(\mathbf{z}\right)e^{-2\pi i\left\{ \mathbf{t}\mathbf{z}\right\} _{p}}d\mathbf{z},\textrm{ }\forall\mathbf{t}\in\hat{\mathbb{Z}}_{p}^{r}\label{eq:MD Definition of the Fourier Coefficients of Chi_H,N} \end{equation} observe that this integral is convergent in both the topology of $\mathbb{C}$ \emph{and} the topology of $\mathbb{C}_{q}$. As before, because $\chi_{H,N}$ is locally constant, it will reduce to a finite sum (see \textbf{Proposition \ref{prop:MD Recursive formula for Chi_H,N hat}} in Subsection \ref{subsec:6.2.1 -and-}, below)): \begin{equation} \hat{\chi}_{H,N}\left(\mathbf{t}\right)\overset{\overline{\mathbb{Q}}^{d}}{=}\frac{\mathbf{1}_{\mathbf{0}}\left(p^{N}\mathbf{t}\right)}{p^{rN}}\sum_{\mathbf{n}=\mathbf{0}}^{p^{N}-1}\chi_{H}\left(\mathbf{n}\right)e^{-2\pi i\mathbf{n}\cdot\mathbf{t}},\textrm{ }\forall\mathbf{t}\in\hat{\mathbb{Z}}_{p}^{r} \end{equation} where: \begin{equation} \mathbf{1}_{\mathbf{0}}\left(p^{N}\mathbf{t}\right)=\left[\left\Vert \mathbf{t}\right\Vert _{p}\leq p^{N}\right]=\prod_{\ell=1}^{r}\left[\left|t_{\ell}\right|_{p}\leq p^{N}\right],\textrm{ }\forall\mathbf{t}\in\hat{\mathbb{Z}}_{p}^{r} \end{equation} is the \emph{scalar-valued }indicator function for the set $\left\{ \left\Vert \mathbf{t}\right\Vert _{p}\leq p^{N}\right\} $. Once more, all of the computations and formal manipulations we will perform hold simultaneously in the topologies of $\mathbb{C}$ and in $\mathbb{C}_{q}$; they both occur in $\overline{\mathbb{Q}}\subset\mathbb{C}\cap\mathbb{C}_{q}$. The difference between the archimedean and non-archimedean topologies will only emerge when we consider what happens as $N$ tends to $\infty$. Much like the one-dimensional case, the reason this all works because of the invariance of the Fourier series formula for $\left[\mathbf{z}\overset{p^{N}}{\equiv}\mathbf{n}\right]$ under the action of elements of the Galois group $\textrm{Gal}\left(\overline{\mathbb{Q}}/\mathbb{Q}\right)$. Additionally, note that because of $\chi_{H,N}$'s local constant-ness and the finitude of its range, we have that, for each $N$, $\hat{\chi}_{H,N}$ has finite support: \begin{equation} \hat{\chi}_{H,N}\left(\mathbf{t}\right)=\mathbf{0},\textrm{ }\forall\left\Vert \mathbf{t}\right\Vert _{p}\geq p^{N+1},\textrm{ }\forall N\in\mathbb{N}_{0}\label{eq:MD Vanishing of Chi_H,N hat for all t with sufficiently large denominators} \end{equation} Consequently, the Fourier series: \begin{equation} \chi_{H,N}\left(\mathbf{z}\right)\overset{\overline{\mathbb{Q}}^{d}}{=}\sum_{\mathbf{t}\in\hat{\mathbb{Z}}_{p}^{r}}\hat{\chi}_{H,N}\left(\mathbf{t}\right)e^{2\pi i\left\{ \mathbf{t}\mathbf{z}\right\} _{p}}\label{eq:MD Fourier series for Chi_H,N} \end{equation} will converge in both $\mathbb{C}^{d}$ and $\mathbb{C}_{q}^{d}$ uniformly with respect to $\mathbf{z}\in\mathbb{Z}_{p}^{r}$, reducing to a finite sum in all cases. \end{notation} Lastly, like in the one-dimensional case, we will need to express the interaction between $p$-adic structure functional equations and $N$th truncations. For full generality (matrix-type and vector-type), this result is given in terms of functions taking values in an arbitrary linear space $V$ over $\mathbb{Q}$. \begin{lem} \label{lem:MD functional equations and truncation lemma}Let $V$ be a $\mathbb{Q}$-linear space, and consider a function $\chi:\mathbb{N}_{0}^{r}\rightarrow V$. Suppose that for all $\mathbf{j}\in\mathbb{Z}^{r}/p\mathbb{Z}^{r}$ there are functions $\Phi_{\mathbf{j}}:\mathbb{N}_{0}^{r}\times V\rightarrow V$ so that: \begin{equation} \chi\left(p\mathbf{n}+\mathbf{j}\right)\overset{V}{=}\Phi_{\mathbf{j}}\left(\mathbf{n},\chi\left(\mathbf{n}\right)\right),\textrm{ }\forall\mathbf{n}\in\mathbb{N}_{0}^{r},\textrm{ }\forall\mathbf{j}\in\mathbb{Z}^{r}/p\mathbb{Z}^{r}\label{eq:MD Relation between truncations and functional equations - Hypothesis} \end{equation} Then, the $N$th truncations $\chi_{N}$ satisfy the functional equations: \begin{equation} \chi_{N}\left(p\mathbf{n}+\mathbf{j}\right)\overset{V}{=}\Phi_{\mathbf{j}}\left(\left[\mathbf{n}\right]_{p^{N-1}},\chi_{N-1}\left(\mathbf{n}\right)\right),\textrm{ }\forall\mathbf{n}\in\mathbb{N}_{0}^{r},\textrm{ }\forall\mathbf{j}\in\mathbb{Z}^{r}/p\mathbb{Z}^{r}\label{eq:MD Relation between truncations and functional equations, version 1} \end{equation} Equivalently: \begin{equation} \chi\left(\left[p\mathbf{n}+\mathbf{j}\right]_{p^{N}}\right)\overset{V}{=}\Phi_{\mathbf{j}}\left(\left[\mathbf{n}\right]_{p^{N-1}},\chi\left(\left[\mathbf{n}\right]_{p^{N-1}}\right)\right),\textrm{ }\forall\mathbf{n}\in\mathbb{N}_{0}^{r},\textrm{ }\forall\mathbf{j}\in\mathbb{Z}^{r}/p\mathbb{Z}^{r}\label{eq:MD Relation between truncations and functional equations, version 2} \end{equation} \end{lem} Proof: Fix $N\geq0$, $\mathbf{n}\in\mathbb{N}_{0}^{r}$, and $\mathbf{j}\in\mathbb{Z}^{r}/p\mathbb{Z}^{r}$. Then: \begin{align*} \chi\left(\left[p\mathbf{n}+\mathbf{j}\right]_{p^{N}}\right) & \overset{V}{=}\chi_{N}\left(p\mathbf{n}+\mathbf{j}\right)\\ & =\sum_{\mathbf{m}=\mathbf{0}}^{p^{N}-1}\chi\left(\mathbf{m}\right)\left[p\mathbf{n}+\mathbf{j}\overset{p^{N}}{\equiv}\mathbf{m}\right]\\ \left(\textrm{split }\mathbf{m}\textrm{ mod }p\right); & =\sum_{\mathbf{m}=\mathbf{0}}^{p^{N-1}-1}\sum_{\mathbf{k}=\mathbf{0}}^{p-1}\chi\left(p\mathbf{m}+\mathbf{k}\right)\left[p\mathbf{n}+\mathbf{j}\overset{p^{N}}{\equiv}p\mathbf{m}+\mathbf{k}\right]\\ & =\sum_{\mathbf{m}=\mathbf{0}}^{p^{N-1}-1}\sum_{\mathbf{k}=\mathbf{0}}^{p-1}\Phi_{\mathbf{k}}\left(\mathbf{m},\chi\left(\mathbf{m}\right)\right)\underbrace{\left[\mathbf{n}\overset{p^{N-1}}{\equiv}\mathbf{m}+\frac{\mathbf{k}-\mathbf{j}}{p}\right]}_{0\textrm{ }\forall\mathbf{n}\textrm{ if }\mathbf{k}\neq\mathbf{j}}\\ & =\sum_{\mathbf{m}=\mathbf{0}}^{p^{N-1}-1}\Phi_{\mathbf{j}}\left(\mathbf{m},\chi\left(\mathbf{m}\right)\right)\left[\mathbf{n}\overset{p^{N-1}}{\equiv}\mathbf{m}\right]\\ & =\Phi_{\mathbf{j}}\left(\left[\mathbf{n}\right]_{p^{N-1}},\chi\left(\left[\mathbf{n}\right]_{p^{N-1}}\right)\right) \end{align*} Q.E.D. \newpage{} \section{\label{sec:6.2 Fourier-Transforms-and}Fourier Transforms and Quasi-Integrability (Again)} IN THIS SECTION, WE ASSUME THAT $H$ IS NON-SINGULAR. \vphantom{} Section \pageref{sec:6.2 Fourier-Transforms-and} is mostly the same as its one-dimensional predecessor in Section \pageref{sec:4.2 Fourier-Transforms-=00003D000026}. As mentioned earlier, the primary distinction of the multi-dimensional case is the non-commutativity of matrix multiplication. This eventually leads to three distinct cases: those where $\alpha_{H}\left(\mathbf{0}\right)=\mathbf{I}_{d}$, those where $\mathbf{I}_{d}-\alpha_{H}\left(\mathbf{0}\right)$ is invertible and $H$ is commutative, and those where $\mathbf{I}_{d}-\alpha_{H}\left(\mathbf{0}\right)$ is invertible and $H$ is \emph{not }commutative. Although an $\mathcal{F}$-series representation of $\chi_{H}$ is obtained for all three cases (\textbf{Theorem \ref{thm:MD F-series for Chi_H}}), it is only for the first two cases that this series can be used to deduce a closed-form expression for a Fourier transform of $\hat{\chi}_{H}$. Subsection \pageref{subsec:6.2.4 Multi-Dimensional--=00003D000026} takes the reader right up to the computational obstacle that appears in the case of a non-commutative $H$. That being said, for the two cases where we \emph{can }deduce formulae for $\hat{\chi}_{H}$, our multi-dimensional $\left(p,q\right)$-adic Wiener Tauberian Theorem can then be applied to yield a \textbf{Tauberian Spectral Theorem }for suitable multi-dimensional Hydra maps, exactly like in the one-dimensional case. \subsection{\label{subsec:6.2.1 -and-}$\hat{\chi}_{H,N}$ and $\hat{A}_{H}$ (Again)} We begin by computing a recursive formula for $\hat{\chi}_{H,N}$ in terms of $\hat{\chi}_{H,N-1}$, which we then solve to obtain an explicit formula for $\hat{\chi}_{H,N}$. \begin{prop}[\textbf{Formulae for Multi-Dimensional} $\hat{\chi}_{H,N}$] \label{prop:MD Recursive formula for Chi_H,N hat}\ \vphantom{} I. \begin{equation} \hat{\chi}_{H,N}\left(\mathbf{t}\right)\overset{\overline{\mathbb{Q}}^{d}}{=}\frac{\mathbf{1}_{\mathbf{0}}\left(p^{N}\mathbf{t}\right)}{p^{rN}}\sum_{\mathbf{n}=\mathbf{0}}^{p^{N}-1}\chi_{H}\left(\mathbf{n}\right)e^{-2\pi i\mathbf{n}\cdot\mathbf{t}}\label{eq:MD Chi_H,N hat transform formula - sum form} \end{equation} \vphantom{} II. \begin{equation} \mathbf{1}_{\mathbf{0}}\left(p^{N}\mathbf{t}\right)\alpha_{H}\left(\mathbf{t}\right)\hat{\chi}_{H,N-1}\left(p\mathbf{t}\right)+\mathbf{1}_{\mathbf{0}}\left(p\mathbf{t}\right)\beta_{H}\left(\mathbf{t}\right),\textrm{ }\forall N\geq1,\textrm{ }\forall\mathbf{t}\in\hat{\mathbb{Z}}_{p}^{r}\label{eq:MD Chi_H,N hat functional equation} \end{equation} the nesting of which yields: \begin{equation} \hat{\chi}_{H,N}\left(\mathbf{t}\right)=\sum_{n=0}^{N-1}\mathbf{1}_{\mathbf{0}}\left(p^{n+1}\mathbf{t}\right)\left(\prod_{m=0}^{n-1}\alpha_{H}\left(p^{m}\mathbf{t}\right)\right)\beta_{H}\left(p^{n}\mathbf{t}\right)\label{eq:MD Explicit formula for Chi_H,N hat} \end{equation} where the $m$-product is defined to be $1$ when $n=0$. In particular, note that $\hat{\chi}_{H,N}\left(\mathbf{t}\right)=\mathbf{0}$ for all $\mathbf{t}\in\hat{\mathbb{Z}}_{p}^{r}$ with $\left\Vert \mathbf{t}\right\Vert _{p}>p^{N}$. \end{prop} Proof: Using \textbf{Proposition \ref{prop:Multi-Dimensional indicator function Fourier Series}}, we have: \begin{align*} \chi_{H,N}\left(\mathbf{z}\right) & =\sum_{\mathbf{n}=\mathbf{0}}^{p^{N}-1}\chi_{H}\left(\mathbf{n}\right)\left[\mathbf{z}\overset{p^{N}}{\equiv}\mathbf{n}\right]\\ & =\sum_{\mathbf{n}=\mathbf{0}}^{p^{N}-1}\chi_{H}\left(\mathbf{n}\right)\frac{1}{p^{rN}}\sum_{\left\Vert \mathbf{t}\right\Vert _{p}\leq p^{N}}e^{2\pi i\left\{ \mathbf{t}\left(\mathbf{z}-\mathbf{n}\right)\right\} _{p}}\\ & =\sum_{\left\Vert \mathbf{t}\right\Vert _{p}\leq p^{N}}\left(\frac{1}{p^{rN}}\sum_{\mathbf{n}=\mathbf{0}}^{p^{N}-1}\chi_{H}\left(\mathbf{n}\right)e^{-2\pi i\mathbf{n}\cdot\mathbf{t}}\right)e^{2\pi i\left\{ \mathbf{t}\mathbf{z}\right\} _{p}} \end{align*} This is the Fourier series representation of $\chi_{H,N}$; as such, the coefficient of $e^{2\pi i\left\{ \mathbf{t}\mathbf{z}\right\} _{p}}$ in the series is precisely the formula for $\hat{\chi}_{H,N}\left(\mathbf{t}\right)$: \[ \hat{\chi}_{H,N}\left(\mathbf{t}\right)=\frac{\mathbf{1}_{\mathbf{0}}\left(p^{N}\mathbf{t}\right)}{p^{rN}}\sum_{\mathbf{n}=\mathbf{0}}^{p^{N}-1}\chi_{H}\left(\mathbf{n}\right)e^{-2\pi i\mathbf{n}\cdot\mathbf{t}} \] This proves (I). To prove (II), just like in the one-dimensional case, we split the $\mathbf{n}$-sum modulo $p$ and utilize $\chi_{H}$'s functional equation. Here\textbf{ }$\mathbf{t}$ satisfies $\left\Vert \mathbf{t}\right\Vert _{p}\leq p^{N}$. \begin{align*} \hat{\chi}_{H,N}\left(\mathbf{t}\right) & =\frac{\mathbf{1}_{\mathbf{0}}\left(p^{N}\mathbf{t}\right)}{p^{rN}}\sum_{\mathbf{j}=\mathbf{0}}^{p-1}\sum_{\mathbf{n}=\mathbf{0}}^{p^{N-1}-1}\chi_{H}\left(p\mathbf{n}+\mathbf{j}\right)e^{-2\pi i\left(p\mathbf{n}+\mathbf{j}\right)\cdot\mathbf{t}}\\ & =\frac{\mathbf{1}_{\mathbf{0}}\left(p^{N}\mathbf{t}\right)}{p^{rN}}\sum_{\mathbf{j}=\mathbf{0}}^{p-1}\sum_{\mathbf{n}=\mathbf{0}}^{p^{N-1}-1}\frac{\mathbf{A}_{\mathbf{j}}\chi_{H}\left(\mathbf{n}\right)+\mathbf{b}_{\mathbf{j}}}{\mathbf{D}_{\mathbf{j}}}e^{-2\pi i\left(p\mathbf{n}+\mathbf{j}\cdot\mathbf{t}\right)} \end{align*} Interchanging the $\mathbf{j}$ and $\mathbf{n}$ sums and breaking things up yields \begin{align*} \frac{\mathbf{1}_{\mathbf{0}}\left(p^{N}\mathbf{t}\right)}{p^{r\left(N-1\right)}}\sum_{\mathbf{n}=\mathbf{0}}^{p^{N-1}-1}\underbrace{\left(\frac{1}{p^{r}}\sum_{\mathbf{j}=\mathbf{0}}^{p-1}\mathbf{D}_{\mathbf{j}}^{-1}\mathbf{A}_{\mathbf{j}}e^{-2\pi i\mathbf{j}\cdot\mathbf{t}}\right)}_{\alpha_{H}\left(\mathbf{t}\right)}\chi_{H}\left(\mathbf{n}\right)e^{-2\pi i\left(p\mathbf{n}\right)\cdot\mathbf{t}}\\ +\frac{\mathbf{1}_{\mathbf{0}}\left(p^{N}\mathbf{t}\right)}{p^{r\left(N-1\right)}}\sum_{\mathbf{n}=\mathbf{0}}^{p^{N-1}-1}\underbrace{\left(\frac{1}{p^{r}}\sum_{\mathbf{j}=\mathbf{0}}^{p-1}\mathbf{D}_{\mathbf{j}}^{-1}\mathbf{b}_{\mathbf{j}}e^{-2\pi i\mathbf{j}\cdot\mathbf{t}}\right)}_{\beta_{H}\left(\mathbf{t}\right)}e^{-2\pi i\left(p\mathbf{n}\right)\cdot\mathbf{t}} \end{align*} Hence: \begin{equation} \hat{\chi}_{H,N}\left(\mathbf{t}\right)=\frac{\mathbf{1}_{\mathbf{0}}\left(p^{N}\mathbf{t}\right)}{p^{r\left(N-1\right)}}\sum_{\mathbf{n}=\mathbf{0}}^{p^{N-1}-1}\left(\alpha_{H}\left(\mathbf{t}\right)\chi_{H}\left(\mathbf{n}\right)+\beta_{H}\left(\mathbf{t}\right)\right)e^{-2\pi i\mathbf{n}\cdot\left(p\mathbf{t}\right)} \end{equation} Re-writing the right-hand side in terms of $\chi_{H,N-1}$ yields: \begin{equation} \hat{\chi}_{H,N}\left(\mathbf{t}\right)=\mathbf{1}_{\mathbf{0}}\left(p^{N}\mathbf{t}\right)\alpha_{H}\left(\mathbf{t}\right)\hat{\chi}_{H,N-1}\left(p\mathbf{t}\right)+\beta_{H}\left(\mathbf{t}\right)\frac{\mathbf{1}_{\mathbf{0}}\left(p^{N}\mathbf{t}\right)}{p^{r\left(N-1\right)}}\sum_{\mathbf{n}=\mathbf{0}}^{p^{N-1}-1}e^{-2\pi i\mathbf{n}\cdot\left(p\mathbf{t}\right)}\label{eq:Chi_H N hat, almost ready to nest-1} \end{equation} Simplifying the remaining $n$-sum, we obtain: \begin{align*} \frac{1}{p^{r\left(N-1\right)}}\sum_{\mathbf{n}=\mathbf{0}}^{p^{N-1}-1}e^{-2\pi i\mathbf{n}\cdot\left(p\mathbf{t}\right)} & =\begin{cases} \frac{1}{p^{r\left(N-1\right)}}\sum_{\mathbf{n}=\mathbf{0}}^{p^{N-1}-1}1 & \textrm{if }\left\Vert \mathbf{t}\right\Vert _{p}\leq p\\ \frac{1}{p^{r\left(N-1\right)}}\prod_{m=1}^{r}\frac{\left(e^{-2\pi ipt_{m}}\right)^{p^{N-1}}-1}{e^{-2\pi ipt_{m}}-1} & \textrm{if }\left\Vert \mathbf{t}\right\Vert _{p}\geq p^{2} \end{cases}\\ & =\begin{cases} 1 & \textrm{if }\left\Vert \mathbf{t}\right\Vert _{p}\leq p\\ 0 & \textrm{if }\left\Vert \mathbf{t}\right\Vert _{p}\geq p^{2} \end{cases}\\ & =\mathbf{1}_{\mathbf{0}}\left(p\mathbf{t}\right) \end{align*} Hence: \begin{align*} \hat{\chi}_{H,N}\left(\mathbf{t}\right) & =\mathbf{1}_{\mathbf{0}}\left(p^{N}\mathbf{t}\right)\left(\alpha_{H}\left(\mathbf{t}\right)\hat{\chi}_{H,N-1}\left(p\mathbf{t}\right)+\beta_{H}\left(\mathbf{t}\right)\mathbf{1}_{\mathbf{0}}\left(p\mathbf{t}\right)\right)\\ & =\mathbf{1}_{\mathbf{0}}\left(p^{N}\mathbf{t}\right)\alpha_{H}\left(\mathbf{t}\right)\hat{\chi}_{H,N-1}\left(p\mathbf{t}\right)+\mathbf{1}_{\mathbf{0}}\left(p\mathbf{t}\right)\beta_{H}\left(\mathbf{t}\right) \end{align*} which proves (II). With (II) proven, we can then nest (\ref{eq:MD Chi_H,N hat functional equation}) to derive an explicit formula for $\hat{\chi}_{H,N}\left(\mathbf{t}\right)$: \begin{align*} \hat{\chi}_{H,N}\left(\mathbf{t}\right) & =\mathbf{1}_{\mathbf{0}}\left(p^{N}\mathbf{t}\right)\alpha_{H}\left(\mathbf{t}\right)\hat{\chi}_{H,N-1}\left(p\mathbf{t}\right)+\mathbf{1}_{\mathbf{0}}\left(p\mathbf{t}\right)\beta_{H}\left(\mathbf{t}\right)\\ & =\mathbf{1}_{\mathbf{0}}\left(p^{N}\mathbf{t}\right)\alpha_{H}\left(\mathbf{t}\right)\left(\mathbf{1}_{\mathbf{0}}\left(p^{N-1}p\mathbf{t}\right)\alpha_{H}\left(p\mathbf{t}\right)\hat{\chi}_{H,N-2}\left(p^{2}\mathbf{t}\right)+\mathbf{1}_{0}\left(p^{2}\mathbf{t}\right)\beta_{H}\left(p\mathbf{t}\right)\right)\\ & +\mathbf{1}_{0}\left(p\mathbf{t}\right)\beta_{H}\left(\mathbf{t}\right)\\ & =\mathbf{1}_{\mathbf{0}}\left(p^{N}\mathbf{t}\right)\alpha_{H}\left(\mathbf{t}\right)\alpha_{H}\left(p\mathbf{t}\right)\hat{\chi}_{H,N-2}\left(p^{2}\mathbf{t}\right)\\ & +\mathbf{1}_{\mathbf{0}}\left(p^{2}\mathbf{t}\right)\alpha_{H}\left(\mathbf{t}\right)\beta_{H}\left(p\mathbf{t}\right)+\mathbf{1}_{\mathbf{0}}\left(p\mathbf{t}\right)\beta_{H}\left(\mathbf{t}\right)\\ & \vdots\\ & =\mathbf{1}_{\mathbf{0}}\left(p^{N}\mathbf{t}\right)\left(\prod_{n=0}^{N-2}\alpha_{H}\left(p^{n}\mathbf{t}\right)\right)\hat{\chi}_{H,1}\left(p^{N-1}\mathbf{t}\right)\\ & +\sum_{n=0}^{N-2}\left(\prod_{m=0}^{n-1}\alpha_{H}\left(p^{m}\mathbf{t}\right)\right)\beta_{H}\left(p^{n}\mathbf{t}\right)\mathbf{1}_{\mathbf{0}}\left(p^{n+1}\mathbf{t}\right) \end{align*} where the $m$-product is $1$ whenever $n=0$. Finally, using (\ref{eq:MD Chi_H,N hat transform formula - sum form}) to compute $\hat{\chi}_{H,1}\left(\mathbf{t}\right)$, we have: \begin{align} \hat{\chi}_{H,1}\left(\mathbf{t}\right) & =\frac{\mathbf{1}_{\mathbf{0}}\left(p\mathbf{t}\right)}{p^{r}}\sum_{\mathbf{j}=\mathbf{0}}^{p-1}\chi_{H}\left(\mathbf{j}\right)e^{-2\pi i\mathbf{j}\cdot\mathbf{t}} \end{align} Since $\chi_{H}\left(\mathbf{0}\right)=\mathbf{0}$, $\chi_{H}$'s functional equation gives: \begin{equation} \chi_{H}\left(\mathbf{j}\right)=\chi_{H}\left(p\mathbf{0}+\mathbf{j}\right)=\frac{\mathbf{A}_{\mathbf{j}}\chi_{H}\left(\mathbf{0}\right)+\mathbf{b}_{\mathbf{j}}}{\mathbf{D}_{\mathbf{j}}}=\mathbf{D}_{\mathbf{j}}^{-1}\mathbf{b}_{\mathbf{j}} \end{equation} and so: \begin{align*} \hat{\chi}_{H,1}\left(\mathbf{t}\right) & =\frac{\mathbf{1}_{\mathbf{0}}\left(p\mathbf{t}\right)}{p^{r}}\sum_{\mathbf{j}=\mathbf{0}}^{p-1}\chi_{H}\left(\mathbf{j}\right)e^{-2\pi i\mathbf{j}\cdot\mathbf{t}}\\ & =\frac{\mathbf{1}_{\mathbf{0}}\left(p\mathbf{t}\right)}{p^{r}}\sum_{\mathbf{j}=\mathbf{0}}^{p-1}\mathbf{D}_{\mathbf{j}}^{-1}\mathbf{b}_{\mathbf{j}}e^{-2\pi i\mathbf{j}\cdot\mathbf{t}}\\ & =\mathbf{1}_{\mathbf{0}}\left(p\mathbf{t}\right)\beta_{H}\left(\mathbf{t}\right) \end{align*} Consequently: \begin{align*} \hat{\chi}_{H,N}\left(\mathbf{t}\right) & =\mathbf{1}_{\mathbf{0}}\left(p^{N}\mathbf{t}\right)\left(\prod_{n=0}^{N-2}\alpha_{H}\left(p^{n}\mathbf{t}\right)\right)\hat{\chi}_{H,1}\left(p^{N-1}\mathbf{t}\right)\\ & +\sum_{n=0}^{N-2}\left(\prod_{m=0}^{n-1}\alpha_{H}\left(p^{m}\mathbf{t}\right)\right)\beta_{H}\left(p^{n}\mathbf{t}\right)\mathbf{1}_{\mathbf{0}}\left(p^{n+1}\mathbf{t}\right)\\ & =\mathbf{1}_{\mathbf{0}}\left(p^{N}t\right)\beta_{H}\left(p^{N-1}\mathbf{t}\right)\prod_{n=0}^{N-2}\alpha_{H}\left(p^{n}\mathbf{t}\right)\\ & +\sum_{n=0}^{N-2}\mathbf{1}_{\mathbf{0}}\left(p^{n+1}\mathbf{t}\right)\left(\prod_{m=0}^{n-1}\alpha_{H}\left(p^{m}\mathbf{t}\right)\right)\beta_{H}\left(p^{n}\mathbf{t}\right)\\ & =\sum_{n=0}^{N-1}\mathbf{1}_{\mathbf{0}}\left(p^{n+1}\mathbf{t}\right)\left(\prod_{m=0}^{n-1}\alpha_{H}\left(p^{m}\mathbf{t}\right)\right)\beta_{H}\left(p^{n}\mathbf{t}\right) \end{align*} which proves (\ref{eq:MD Explicit formula for Chi_H,N hat}). Q.E.D. \vphantom{} Next, we introduce our multi-dimensional analogue $\hat{A}_{H}$. \begin{defn}[\textbf{Multi-Dimensional $\hat{A}_{H}$}] \nomenclature{$\hat{A}_{H}\left(\mathbf{t}\right)$}{ }\textbf{ }We define the function $\hat{A}_{H}:\hat{\mathbb{Z}}_{p}^{r}\rightarrow\textrm{GL}_{d}\left(\overline{\mathbb{Q}}\right)$ by: \begin{equation} \hat{A}_{H}\left(\mathbf{t}\right)\overset{\textrm{def}}{=}\prod_{m=0}^{-v_{p}\left(\mathbf{t}\right)-1}\alpha_{H}\left(p^{m}\mathbf{t}\right),\textrm{ }\forall\mathbf{t}\in\hat{\mathbb{Z}}_{p}^{r}\label{eq:MD Definition of A_H hat} \end{equation} where the $m$-product is defined to be $\mathbf{I}_{d}$ whenever $\mathbf{t}=\mathbf{0}$\textemdash so, $\hat{A}_{H}\left(\mathbf{0}\right)\overset{\textrm{def}}{=}\mathbf{I}_{d}$. \end{defn} \begin{prop}[\textbf{Matrix $\alpha_{H}$ product in terms of $\hat{A}_{H}$}] \label{prop:MD alpha_H A_H hat product} Fix $\mathbf{t}\in\hat{\mathbb{Z}}_{p}^{r}\backslash\left\{ \mathbf{0}\right\} $. Then: \begin{align} \mathbf{1}_{\mathbf{0}}\left(p^{n+1}\mathbf{t}\right)\prod_{m=0}^{n-1}\alpha_{H}\left(p^{m}\mathbf{t}\right) & =\begin{cases} \mathbf{O}_{d} & \textrm{if }n\leq-v_{p}\left(\mathbf{t}\right)-2\\ \hat{A}_{H}\left(\mathbf{t}\right)\left(\alpha_{H}\left(\frac{\mathbf{t}\left|\mathbf{t}\right|_{p}}{p}\right)\right)^{-1} & \textrm{if }n=-v_{p}\left(\mathbf{t}\right)-1\\ \hat{A}_{H}\left(\mathbf{t}\right)\left(\alpha_{H}\left(\mathbf{0}\right)\right)^{n+v_{p}\left(\mathbf{t}\right)} & \textrm{if }n\geq-v_{p}\left(\mathbf{t}\right) \end{cases}\label{eq:MD alpha product in terms of A_H hat} \end{align} \end{prop} Proof: Fix $\mathbf{t}\in\hat{\mathbb{Z}}_{p}^{r}\backslash\left\{ \mathbf{0}\right\} $. Then, we can write: \begin{align*} \mathbf{1}_{\mathbf{0}}\left(p^{n+1}\mathbf{t}\right)\prod_{m=0}^{n-1}\alpha_{H}\left(p^{m}\mathbf{t}\right) & =\begin{cases} \mathbf{O}_{d} & \textrm{if }\left\Vert \mathbf{t}\right\Vert _{p}\geq p^{n+2}\\ \prod_{m=0}^{n-1}\alpha_{H}\left(p^{m}\mathbf{t}\right) & \textrm{if }\left\Vert \mathbf{t}\right\Vert _{p}\leq p^{n+1} \end{cases}\\ & =\begin{cases} \mathbf{O}_{d} & \textrm{if }\exists\ell:n\leq-v_{p}\left(t_{\ell}\right)-2\\ \prod_{m=0}^{n-1}\alpha_{H}\left(p^{m}\mathbf{t}\right) & \textrm{if }n\geq-v_{p}\left(t_{\ell}\right)-1,\textrm{ }\forall\ell \end{cases} \end{align*} So, we get the zero matrix whenever $n+1$ is not greater than the negatives of the valuations of each of $\mathbf{t}$s components. Fixing $\mathbf{t}$, we have that $p^{m}\mathbf{t}\overset{1}{\equiv}\mathbf{0}$ only when $m$ is larger than the negative valuations of each of the components of $\mathbf{t}$: \begin{equation} m\geq\max_{1\leq\ell\leq r}\left(-v_{p}\left(t_{\ell}\right)\right)=-\min_{1\leq\ell\leq r}v_{p}\left(t_{\ell}\right)=-v_{p}\left(\mathbf{t}\right) \end{equation} Hence, $\alpha_{H}\left(p^{m}\mathbf{t}\right)=\alpha_{H}\left(\mathbf{0}\right)$ for all $m\geq-v_{p}\left(\mathbf{t}\right)$. Thus, for fixed $\mathbf{t}$: \begin{equation} \prod_{m=0}^{n-1}\alpha_{H}\left(p^{m}\mathbf{t}\right)=\begin{cases} \prod_{m=0}^{-v_{p}\left(\mathbf{t}\right)-2}\alpha_{H}\left(p^{m}\mathbf{t}\right) & \textrm{if }n=-v_{p}\left(\mathbf{t}\right)-1\\ \prod_{m=0}^{-v_{p}\left(\mathbf{t}\right)-1}\alpha_{H}\left(p^{m}\mathbf{t}\right) & \textrm{if }n=-v_{p}\left(\mathbf{t}\right)\\ \prod_{m=0}^{-v_{p}\left(\mathbf{t}\right)-1}\alpha_{H}\left(p^{m}\mathbf{t}\right)\times\prod_{k=-v_{p}\left(\mathbf{t}\right)}^{n-1}\alpha_{H}\left(p^{k}\mathbf{t}\right) & \textrm{if }n\geq-v_{p}\left(\mathbf{t}\right)+1 \end{cases} \end{equation} Because $\alpha_{H}\left(p^{k}\mathbf{t}\right)=\alpha_{H}\left(\mathbf{0}\right)$ for all $k\geq-v_{p}\left(\mathbf{t}\right)$, this can be simplified to: \begin{align*} \prod_{m=0}^{n-1}\alpha_{H}\left(p^{m}\mathbf{t}\right) & =\begin{cases} \prod_{m=0}^{-v_{p}\left(\mathbf{t}\right)-2}\alpha_{H}\left(p^{m}\mathbf{t}\right) & \textrm{if }n=-v_{p}\left(\mathbf{t}\right)-1\\ \left(\prod_{m=0}^{-v_{p}\left(\mathbf{t}\right)-1}\alpha_{H}\left(p^{m}\mathbf{t}\right)\right)\left(\alpha_{H}\left(\mathbf{0}\right)\right)^{n+v_{p}\left(\mathbf{t}\right)} & \textrm{if }n\geq-v_{p}\left(\mathbf{t}\right) \end{cases}\\ \left(\times\&\div\textrm{ by }\alpha_{H}\left(p^{-v_{p}\left(\mathbf{t}\right)-1}\mathbf{t}\right)\right); & =\begin{cases} \hat{A}_{H}\left(\mathbf{t}\right)\left(\alpha_{H}\left(p^{-v_{p}\left(\mathbf{t}\right)-1}\mathbf{t}\right)\right)^{-1} & \textrm{if }n=-v_{p}\left(\mathbf{t}\right)-1\\ \hat{A}_{H}\left(\mathbf{t}\right)\left(\alpha_{H}\left(\mathbf{0}\right)\right)^{n+v_{p}\left(\mathbf{t}\right)} & \textrm{if }n\geq-v_{p}\left(\mathbf{t}\right) \end{cases} \end{align*} which gives us the desired formula. Q.E.D. \vphantom{} Next, we express the $\alpha_{H}$ product as a series. \begin{prop}[\textbf{Matrix $\alpha_{H}$ series formula}] \label{prop:MD alpha H series} \end{prop} \begin{equation} \prod_{m=0}^{n-1}\alpha_{H}\left(p^{m}\mathbf{t}\right)\overset{\overline{\mathbb{Q}}^{d,d}}{=}\sum_{\mathbf{m}=\mathbf{0}}^{p^{n}-1}\kappa_{H}\left(\mathbf{m}\right)\left(\frac{H^{\prime}\left(\mathbf{0}\right)}{p^{r}}\right)^{n}e^{-2\pi i\left(\mathbf{m}\cdot\mathbf{t}\right)},\textrm{ }\forall n\geq0,\textrm{ }\forall\mathbf{t}\in\hat{\mathbb{Z}}_{p}^{r}\label{eq:MD alpha_H product expansion} \end{equation} Proof: We start by writing: \begin{equation} \prod_{m=0}^{n-1}\alpha_{H}\left(p^{m}\mathbf{t}\right)=\prod_{m=0}^{n-1}\left(\sum_{\mathbf{j}=\mathbf{0}}^{p-1}\frac{1}{p^{r}}\frac{\mathbf{A}_{\mathbf{j}}}{\mathbf{D}_{\mathbf{j}}}e^{-2\pi i\mathbf{j}\cdot\left(p^{m}\mathbf{t}\right)}\right) \end{equation} and then apply (\ref{eq:MD M_H partial sum generating identity}) from \textbf{Proposition \ref{prop:MD generating function} }with $z^{\mathbf{j}\cdot\mathbf{t}}$ replaced by $e^{-2\pi i\left(\mathbf{j}\cdot\mathbf{t}\right)}$: \begin{align*} \prod_{m=0}^{n-1}\alpha_{H}\left(p^{m}\mathbf{t}\right) & =\prod_{m=0}^{n-1}\left(\sum_{\mathbf{j}=\mathbf{0}}^{p-1}\frac{1}{p^{r}}\frac{\mathbf{A}_{\mathbf{j}}}{\mathbf{D}_{\mathbf{j}}}e^{-2\pi i\mathbf{j}\cdot\left(p^{m}\mathbf{t}\right)}\right)\\ & =\frac{1}{p^{rn}}\sum_{\mathbf{m}=\mathbf{0}}^{p^{n}-1}M_{H}\left(\mathbf{m}\right)\left(H^{\prime}\left(\mathbf{0}\right)\right)^{n-\lambda_{p}\left(\mathbf{m}\right)}e^{-2\pi i\left(\mathbf{m}\cdot\mathbf{t}\right)}\\ \left(\kappa_{H}\left(\mathbf{m}\right)=M_{H}\left(\mathbf{m}\right)\left(H^{\prime}\left(\mathbf{0}\right)\right)^{-\lambda_{p}\left(\mathbf{m}\right)}\right); & =\frac{1}{p^{rn}}\sum_{\mathbf{m}=\mathbf{0}}^{p^{n}-1}\kappa_{H}\left(\mathbf{m}\right)\left(H^{\prime}\left(\mathbf{0}\right)\right)^{n}e^{-2\pi i\left(\mathbf{m}\cdot\mathbf{t}\right)}\\ & =\sum_{\mathbf{m}=\mathbf{0}}^{p^{n}-1}\kappa_{H}\left(\mathbf{m}\right)\left(\frac{H^{\prime}\left(\mathbf{0}\right)}{p^{r}}\right)^{n}e^{-2\pi i\left(\mathbf{m}\cdot\mathbf{t}\right)} \end{align*} Q.E.D. \vphantom{} Just like the one-dimensional case, we now use this formula to exhibit $\hat{A}_{H}$ as a radially-magnitudinal function, and then\textemdash with the help of our multi-dimensional Fourier Resummation Lemmata\textemdash show that $\hat{A}_{H}$ is the Fourier-Stieltjes transform of a rising-continuous $\left(p,q\right)$-adic thick measure of matrix-type. \begin{prop} Let $H$ be semi-basic. Then, $\hat{A}_{H}\left(\mathbf{t}\right)$ is a depth-$r$ $\left(p,q\right)$-adic thick measure of matrix type. \end{prop} Proof: Let $H$ be semi-basic. Then, $p$ divides the diagonal entires of the $\mathbf{D}_{\mathbf{j}}$s which are not $1$, and the entries of any $\mathbf{D}_{\mathbf{j}}$ are co-prime to the entries of every $\mathbf{A}_{\mathbf{j}}$. As such, taking $q_{H}$-adic matrix norms yields: \begin{equation} \left\Vert \alpha_{H}\left(\mathbf{t}\right)\right\Vert _{q_{H}}=\left\Vert \frac{1}{p^{r}}\sum_{\mathbf{j}=\mathbf{0}}^{p-1}\frac{\mathbf{A}_{\mathbf{j}}}{\mathbf{D}_{\mathbf{j}}}e^{-2\pi i\mathbf{j}\cdot\mathbf{t}}\right\Vert _{q_{H}}\leq\max_{\mathbf{j}\leq p-1}\left|\frac{1}{p^{r}}\frac{\mathbf{A}_{\mathbf{j}}}{\mathbf{D}_{\mathbf{j}}}\right|_{q_{H}}\leq1 \end{equation} Hence: \begin{align*} \sup_{\mathbf{t}\in\hat{\mathbb{Z}}_{p}^{r}}\left\Vert \hat{A}_{H}\left(\mathbf{t}\right)\right\Vert _{q_{H}} & \leq\max\left\{ 1,\sup_{\mathbf{t}\in\hat{\mathbb{Z}}_{p}^{r}\backslash\left\{ \mathbf{0}\right\} }\left\Vert \prod_{m=0}^{-v_{p}\left(\mathbf{t}\right)-1}\alpha_{H}\left(p^{m}\mathbf{t}\right)\right\Vert _{q_{H}}\right\} \\ & \leq\max\left\{ 1,\sup_{\mathbf{t}\in\hat{\mathbb{Z}}_{p}^{r}\backslash\left\{ \mathbf{0}\right\} }\prod_{m=0}^{-v_{p}\left(\mathbf{t}\right)-1}1\right\} \\ & =1 \end{align*} and so, $\hat{A}_{H}$ is then in $B\left(\hat{\mathbb{Z}}_{p}^{r},\mathbb{C}_{q}^{d,d}\right)$. As such, the map: \begin{equation} \mathbf{f}\in C\left(\mathbb{Z}_{p}^{r},\mathbb{C}_{q}^{d}\right)\mapsto\sum_{\mathbf{t}\in\hat{\mathbb{Z}}_{p}^{r}}\hat{A}_{H}\left(\mathbf{t}\right)\hat{\mathbf{f}}\left(-\mathbf{t}\right)\in\mathbb{C}_{q}^{d} \end{equation} is then a continuous, defining a $\left(p,q\right)$-adic thick measure of matrix type. Q.E.D. \vphantom{} Next, some obligatory abbreviations: \begin{defn} \nomenclature{$\tilde{A}_{H,N}\left(\mathbf{z}\right)$}{$N$th partial sum of the Fourier series generated by $\hat{A}_{H}\left(\mathbf{t}\right)$.}For each $N\in\mathbb{N}_{0}$, we write $\tilde{A}_{H,N}:\mathbb{Z}_{p}^{r}\rightarrow\overline{\mathbb{Q}}^{d,d}$ to denote the matrix-valued function: \begin{equation} \tilde{A}_{H,N}\left(\mathbf{z}\right)\overset{\textrm{def}}{=}\sum_{\left\Vert \mathbf{t}\right\Vert _{p}\leq p^{N}}\hat{A}_{H}\left(\mathbf{t}\right)e^{2\pi i\left\{ \mathbf{t}\mathbf{z}\right\} _{p}}\label{eq:MD Definition of A_H,N twiddle} \end{equation} \end{defn} \begin{defn} We write $\mathbf{I}_{H,n}\left(n\right)$ \nomenclature{$\mathbf{I}_{H,n}\left(n\right)$}{$\overset{\textrm{def}}{=}\mathbf{I}_{d}-\left(H^{\prime}\left(\mathbf{0}\right)\right)^{n}\alpha_{H}\left(\mathbf{0}\right)\left(H^{\prime}\left(\mathbf{0}\right)\right)^{-n}$ \nopageref}to denote the $d\times d$ matrix: \begin{align} \mathbf{I}_{H}\left(n\right) & \overset{\textrm{def}}{=}\mathbf{I}_{d}-\mathcal{C}_{H}\left(\alpha_{H}\left(\mathbf{0}\right):n\right)\label{eq:Definition of I_H}\\ & =\mathbf{I}_{d}-\left(H^{\prime}\left(\mathbf{0}\right)\right)^{n}\alpha_{H}\left(\mathbf{0}\right)\left(H^{\prime}\left(\mathbf{0}\right)\right)^{-n}\nonumber \end{align} \end{defn} \begin{rem} Note that $\mathbf{I}_{H}\left(n\right)=\mathbf{I}_{d}-\alpha_{H}\left(\mathbf{0}\right)$ for all $n\in\mathbb{N}_{0}$ whenever $H$ is commutative. Also, $\mathbf{I}_{H}\left(n\right)=\mathbf{O}_{d}$ for all $n\in\mathbb{N}_{0}$ whenever $\alpha_{H}\left(\mathbf{0}\right)=\mathbf{I}_{d}$. \end{rem} Our next theorem details the properties of $A_{H}$: \begin{thm}[\textbf{Properties of Multi-Dimensional $\hat{A}_{H}$}] \label{thm:MD properties of A_H hat}\ \vphantom{} I. ($\hat{A}_{H}$ is radially-magnitudinal and $\left(p,q\right)$-adically regular) \begin{equation} \hat{A}_{H}\left(\mathbf{t}\right)\overset{\overline{\mathbb{Q}}^{d,d}}{=}\begin{cases} \mathbf{I}_{d} & \textrm{if }\mathbf{t}=0\\ \left(\sum_{\mathbf{m}=\mathbf{0}}^{p^{-v_{p}\left(\mathbf{t}\right)}-1}\kappa_{H}\left(\mathbf{m}\right)e^{-2\pi i\left(\mathbf{m}\cdot\mathbf{t}\right)}\right)\left(\frac{H^{\prime}\left(\mathbf{0}\right)}{p^{r}}\right)^{-v_{p}\left(\mathbf{t}\right)} & \textrm{else} \end{cases},\textrm{ }\forall\mathbf{t}\in\hat{\mathbb{Z}}_{p}^{r}\label{eq:A_H hat as the product of radially symmetric and magnitude-dependent measures-1} \end{equation} Here, in an abuse of notation, we write: \[ \left(\frac{H^{\prime}\left(\mathbf{0}\right)}{p^{r}}\right)^{-v_{p}\left(\mathbf{0}\right)}\overset{\textrm{def}}{=}\mathbf{I}_{d} \] \vphantom{} II. (Formula for $\tilde{A}_{H,N}\left(\mathbf{z}\right)$) For any $N\in\mathbb{N}_{0}$ and $\mathbf{z}\in\mathbb{Z}_{p}^{r}$: \begin{align} \tilde{A}_{H,N}\left(\mathbf{z}\right)\overset{\overline{\mathbb{Q}}^{d,d}}{=} & \kappa_{H}\left(\left[\mathbf{z}\right]_{p^{N}}\right)\left(H^{\prime}\left(\mathbf{0}\right)\right)^{N}+\sum_{n=0}^{N-1}\kappa_{H}\left(\left[\mathbf{z}\right]_{p^{n}}\right)\mathbf{I}_{H}\left(\lambda_{p}\left(\left[\mathbf{z}\right]_{p^{n}}\right)\right)\left(H^{\prime}\left(\mathbf{0}\right)\right)^{n}\label{eq:MD Convolution of dA_H and D_N} \end{align} In particular, if $H$ is commutative: \begin{equation} \tilde{A}_{H,N}\left(\mathbf{z}\right)\overset{\overline{\mathbb{Q}}^{d,d}}{=}\kappa_{H}\left(\left[\mathbf{z}\right]_{p^{N}}\right)\left(H^{\prime}\left(\mathbf{0}\right)\right)^{N}+\sum_{n=0}^{N-1}\kappa_{H}\left(\left[\mathbf{z}\right]_{p^{n}}\right)\left(H^{\prime}\left(\mathbf{0}\right)\right)^{n}\left(\mathbf{I}_{d}-\alpha_{H}\left(\mathbf{0}\right)\right)\label{eq:MD Convolution of dA_H and D_N when H is commutative} \end{equation} \vphantom{} III. As $N\rightarrow\infty$, \emph{(\ref{eq:MD Convolution of dA_H and D_N})} converges in the standard $\left(p,q_{H}\right)$-adic frame to: \begin{equation} \lim_{N\rightarrow\infty}\tilde{A}_{H,N}\left(\mathbf{z}\right)\overset{\mathcal{F}_{p,q_{H}}^{d,d}}{=}\sum_{n=0}^{\infty}\kappa_{H}\left(\left[\mathbf{z}\right]_{p^{n}}\right)\mathbf{I}_{H}\left(\lambda_{p}\left(\left[\mathbf{z}\right]_{p^{n}}\right)\right)\left(H^{\prime}\left(\mathbf{0}\right)\right)^{n},\textrm{ }\forall\mathbf{z}\in\mathbb{Z}_{p}^{r}\label{eq:MD Fourier Limit of A_H,N twiddle in standard frame} \end{equation} There are two particular cases: i If $\alpha_{H}\left(\mathbf{0}\right)=\mathbf{I}_{d}$, then: \begin{equation} \lim_{N\rightarrow\infty}\tilde{A}_{H,N}\left(\mathbf{z}\right)\overset{\mathcal{F}_{p,q_{H}}^{d,d}}{=}\mathbf{O}_{d},\textrm{ }\forall\mathbf{z}\in\mathbb{Z}_{p}^{r}\label{eq:Full kernel of dA_H on when alpha is 1} \end{equation} ii. If $H$ is commutative, then: \begin{equation} \lim_{N\rightarrow\infty}\tilde{A}_{H,N}\left(\mathbf{z}\right)\overset{\mathcal{F}_{p,q_{H}}^{d,d}}{=}\sum_{n=0}^{\infty}\kappa_{H}\left(\left[\mathbf{z}\right]_{p^{n}}\right)\left(H^{\prime}\left(\mathbf{0}\right)\right)^{n}\left(\mathbf{I}_{d}-\alpha_{H}\left(\mathbf{0}\right)\right),\textrm{ }\forall\mathbf{z}\in\mathbb{Z}_{p}^{r}\label{eq:MD Fourier Limit of A_H,N twiddle in standard frame, commutative} \end{equation} \end{thm} Proof: I. By (\ref{eq:MD alpha_H product expansion}), we have that: \[ \hat{A}_{H}\left(\mathbf{t}\right)\overset{\overline{\mathbb{Q}}^{d,d}}{=}\prod_{m=0}^{-v_{p}\left(\mathbf{t}\right)-1}\alpha_{H}\left(p^{m}\mathbf{t}\right)=\left(\sum_{\mathbf{m}=\mathbf{0}}^{p^{-v_{p}\left(\mathbf{t}\right)}-1}\kappa_{H}\left(\mathbf{m}\right)e^{-2\pi i\left(\mathbf{m}\cdot\mathbf{t}\right)}\right)\left(\frac{H^{\prime}\left(\mathbf{0}\right)}{p^{r}}\right)^{-v_{p}\left(\mathbf{t}\right)} \] \vphantom{} II. By (\ref{eq:MD radially-magnitudinal resummation formula}), we have: \begin{align*} \tilde{A}_{H,N}\left(\mathbf{z}\right) & =p^{rN}\kappa_{H}\left(\left[\mathbf{z}\right]_{p^{N}}\right)\left(\frac{H^{\prime}\left(\mathbf{0}\right)}{p^{r}}\right)^{-v_{p}\left(p^{-N}\right)}\\ & +\sum_{n=0}^{N-1}p^{rn}\kappa_{H}\left(\left[\mathbf{z}\right]_{p^{n}}\right)\left(\left(\frac{H^{\prime}\left(\mathbf{0}\right)}{p^{r}}\right)^{-v_{p}\left(p^{-n}\right)}-\left(\frac{H^{\prime}\left(\mathbf{0}\right)}{p^{r}}\right)^{-v_{p}\left(p^{-\left(n+1\right)}\right)}\right)\\ & -\sum_{n=0}^{N-1}p^{rn}\sum_{\mathbf{j}>\mathbf{0}}^{p-1}\kappa_{H}\left(\left[\mathbf{z}\right]_{p^{n}}+\mathbf{j}p^{n}\right)\left(\frac{H^{\prime}\left(\mathbf{0}\right)}{p^{r}}\right)^{-v_{p}\left(p^{-\left(n+1\right)}\right)} \end{align*} Here, $-v_{p}\left(p^{-n}\right)=n$. Since we are using the convention: \begin{equation} \left(\frac{H^{\prime}\left(\mathbf{0}\right)}{p^{r}}\right)^{-v_{p}\left(\frac{1}{p^{0}}\right)}=\left(\frac{H^{\prime}\left(\mathbf{0}\right)}{p^{r}}\right)^{-v_{p}\left(\mathbf{0}\right)}\overset{\textrm{def}}{=}\mathbf{I}_{d} \end{equation} and since $H^{\prime}\left(\mathbf{0}\right)/p^{r}$ is an invertible $d\times d$ matrix, we have that: \begin{equation} \left(\frac{H^{\prime}\left(\mathbf{0}\right)}{p^{r}}\right)^{-v_{p}\left(1/p^{n}\right)}=\left(\frac{H^{\prime}\left(\mathbf{0}\right)}{p^{r}}\right)^{n},\textrm{ }\forall n\in\mathbb{N}_{0} \end{equation} Consequently: \begin{align*} \tilde{A}_{H,N}\left(\mathbf{z}\right) & =p^{rN}\kappa_{H}\left(\left[\mathbf{z}\right]_{p^{n}}\right)\left(\frac{H^{\prime}\left(\mathbf{0}\right)}{p^{r}}\right)^{N}\\ & +\sum_{n=0}^{N-1}p^{rn}\kappa_{H}\left(\left[\mathbf{z}\right]_{p^{n}}\right)\left(\left(\frac{H^{\prime}\left(\mathbf{0}\right)}{p^{r}}\right)^{n}-\left(\frac{H^{\prime}\left(\mathbf{0}\right)}{p^{r}}\right)^{n+1}\right)\\ & -\sum_{n=0}^{N-1}p^{rn}\sum_{\mathbf{j}>\mathbf{0}}^{p-1}\kappa_{H}\left(\left[\mathbf{z}\right]_{p^{n}}+\mathbf{j}p^{n}\right)\left(\frac{H^{\prime}\left(\mathbf{0}\right)}{p^{r}}\right)^{n+1} \end{align*} Here, (\ref{eq:MD Kappa_H has P-adic structure}) from \textbf{Lemma \ref{lem:properties of MD kappa_H}} yields: \begin{align*} \kappa_{H}\left(\left[\mathbf{z}\right]_{p^{n}}+p^{n}\mathbf{j}\right) & =\kappa_{H}\left(\left[\mathbf{z}\right]_{p^{n}}\right)\left(H^{\prime}\left(\mathbf{0}\right)\right)^{\lambda_{p}\left(\left[\mathbf{z}\right]_{p^{n}}\right)}\kappa_{H}\left(\mathbf{j}\right)\left(H^{\prime}\left(\mathbf{0}\right)\right)^{-\lambda_{p}\left(\left[\mathbf{z}\right]_{p^{n}}\right)}\\ & =M_{H}\left(\left[\mathbf{z}\right]_{p^{n}}\right)\kappa_{H}\left(\mathbf{j}\right)\left(H^{\prime}\left(\mathbf{0}\right)\right)^{-\lambda_{p}\left(\left[\mathbf{z}\right]_{p^{n}}\right)} \end{align*} As such: \begin{align*} \tilde{A}_{H,N}\left(\mathbf{z}\right) & =\kappa_{H}\left(\left[\mathbf{z}\right]_{p^{N}}\right)\left(H^{\prime}\left(\mathbf{0}\right)\right)^{N}\\ & +\sum_{n=0}^{N-1}p^{rn}\kappa_{H}\left(\left[\mathbf{z}\right]_{p^{n}}\right)\left(\left(\frac{H^{\prime}\left(\mathbf{0}\right)}{p^{r}}\right)^{n}-\left(\frac{H^{\prime}\left(\mathbf{0}\right)}{p^{r}}\right)^{n+1}\right)\\ & -\sum_{n=0}^{N-1}p^{rn}M_{H}\left(\left[\mathbf{z}\right]_{p^{n}}\right)\left(\sum_{\mathbf{j}>\mathbf{0}}^{p-1}\kappa_{H}\left(\mathbf{j}\right)\right)\left(H^{\prime}\left(\mathbf{0}\right)\right)^{-\lambda_{p}\left(\left[\mathbf{z}\right]_{p^{n}}\right)}\left(\frac{H^{\prime}\left(\mathbf{0}\right)}{p^{r}}\right)^{n+1} \end{align*} Using (\ref{eq:MD kappa H sum in terms of MD alpha}) gives: \begin{align*} \tilde{A}_{H,N}\left(\mathbf{z}\right) & =\kappa_{H}\left(\left[\mathbf{z}\right]_{p^{N}}\right)\left(H^{\prime}\left(\mathbf{0}\right)\right)^{N}\\ & +\sum_{n=0}^{N-1}p^{rn}\kappa_{H}\left(\left[\mathbf{z}\right]_{p^{n}}\right)\left(\left(\frac{H^{\prime}\left(\mathbf{0}\right)}{p^{r}}\right)^{n}-\left(\frac{H^{\prime}\left(\mathbf{0}\right)}{p^{r}}\right)^{n+1}\right)\\ & -\sum_{n=0}^{N-1}p^{rn}M_{H}\left(\left[\mathbf{z}\right]_{p^{n}}\right)\left(\alpha_{H}\left(\mathbf{0}\right)\left(\frac{H^{\prime}\left(\mathbf{0}\right)}{p^{r}}\right)^{-1}-\mathbf{I}_{d}\right)\left(H^{\prime}\left(\mathbf{0}\right)\right)^{-\lambda_{p}\left(\left[\mathbf{z}\right]_{p^{n}}\right)}\left(\frac{H^{\prime}\left(\mathbf{0}\right)}{p^{r}}\right)^{n+1} \end{align*} Re-writing things to make some cancellations evident: \begin{align*} \tilde{A}_{H,N}\left(\mathbf{z}\right) & =\kappa_{H}\left(\left[\mathbf{z}\right]_{p^{N}}\right)\left(H^{\prime}\left(\mathbf{0}\right)\right)^{N}\\ & +\sum_{n=0}^{N-1}p^{rn}\kappa_{H}\left(\left[\mathbf{z}\right]_{p^{n}}\right)\left(\frac{H^{\prime}\left(\mathbf{0}\right)}{p^{r}}\right)^{n}-\overbrace{\sum_{n=0}^{N-1}p^{rn}\kappa_{H}\left(\left[\mathbf{z}\right]_{p^{n}}\right)\left(\frac{H^{\prime}\left(\mathbf{0}\right)}{p^{r}}\right)^{n+1}}^{\textrm{these two}}\\ & -\sum_{n=0}^{N-1}p^{rn}M_{H}\left(\left[\mathbf{z}\right]_{p^{n}}\right)\alpha_{H}\left(\mathbf{0}\right)\left(H^{\prime}\left(\mathbf{0}\right)\right)^{-\lambda_{p}\left(\left[\mathbf{z}\right]_{p^{n}}\right)}\left(\frac{H^{\prime}\left(\mathbf{0}\right)}{p^{r}}\right)^{n}\\ & +\underbrace{\sum_{n=0}^{N-1}p^{rn}\overbrace{M_{H}\left(\left[\mathbf{z}\right]_{p^{n}}\right)\left(H^{\prime}\left(\mathbf{0}\right)\right)^{-\lambda_{p}\left(\left[\mathbf{z}\right]_{p^{n}}\right)}}^{\kappa_{H}\left(\left[\mathbf{z}\right]_{p^{n}}\right)}\left(\frac{H^{\prime}\left(\mathbf{0}\right)}{p^{r}}\right)^{n+1}}_{\textrm{cancel out}} \end{align*} we end up with: \begin{align*} \tilde{A}_{H,N}\left(\mathbf{z}\right) & =\kappa_{H}\left(\left[\mathbf{z}\right]_{p^{N}}\right)\left(H^{\prime}\left(\mathbf{0}\right)\right)^{N}+\sum_{n=0}^{N-1}p^{rn}\kappa_{H}\left(\left[\mathbf{z}\right]_{p^{n}}\right)\left(\frac{H^{\prime}\left(\mathbf{0}\right)}{p^{r}}\right)^{n}\\ & -\sum_{n=0}^{N-1}p^{rn}M_{H}\left(\left[\mathbf{z}\right]_{p^{n}}\right)\alpha_{H}\left(\mathbf{0}\right)\left(H^{\prime}\left(\mathbf{0}\right)\right)^{-\lambda_{p}\left(\left[\mathbf{z}\right]_{p^{n}}\right)}\left(\frac{H^{\prime}\left(\mathbf{0}\right)}{p^{r}}\right)^{n} \end{align*} Now we utilize our $\mathcal{C}_{H}$ notation to make the lower line manageable. First, recall that: \begin{equation} M_{H}\left(\left[\mathbf{z}\right]_{p^{n}}\right)=\kappa_{H}\left(\left[\mathbf{z}\right]_{p^{n}}\right)\left(H^{\prime}\left(\mathbf{0}\right)\right)^{\lambda_{p}\left(\left[\mathbf{z}\right]_{p^{n}}\right)} \end{equation} With this, the expression: \begin{equation} M_{H}\left(\left[\mathbf{z}\right]_{p^{n}}\right)\alpha_{H}\left(\mathbf{0}\right)\left(H^{\prime}\left(\mathbf{0}\right)\right)^{-\lambda_{p}\left(\left[\mathbf{z}\right]_{p^{n}}\right)} \end{equation} becomes: \begin{equation} \kappa_{H}\left(\left[\mathbf{z}\right]_{p^{n}}\right)\left(H^{\prime}\left(\mathbf{0}\right)\right)^{\lambda_{p}\left(\left[\mathbf{z}\right]_{p^{n}}\right)}\alpha_{H}\left(\mathbf{0}\right)\left(H^{\prime}\left(\mathbf{0}\right)\right)^{-\lambda_{p}\left(\left[\mathbf{z}\right]_{p^{n}}\right)} \end{equation} which, using $\mathcal{C}_{H}$, is just: \begin{equation} \kappa_{H}\left(\left[\mathbf{z}\right]_{p^{n}}\right)\mathcal{C}_{H}\left(\alpha_{H}\left(\mathbf{0}\right):\lambda_{p}\left(\left[\mathbf{z}\right]_{p^{n}}\right)\right) \end{equation} Finally, writing out the formula for $\tilde{A}_{H,N}\left(\mathbf{z}\right)$, we have: \begin{align*} \tilde{A}_{H,N}\left(\mathbf{z}\right) & =\kappa_{H}\left(\left[\mathbf{z}\right]_{p^{N}}\right)\left(H^{\prime}\left(\mathbf{0}\right)\right)^{N}+\sum_{n=0}^{N-1}p^{rn}\kappa_{H}\left(\left[\mathbf{z}\right]_{p^{n}}\right)\left(\frac{H^{\prime}\left(\mathbf{0}\right)}{p^{r}}\right)^{n}\\ & -\sum_{n=0}^{N-1}p^{rn}\kappa_{H}\left(\left[\mathbf{z}\right]_{p^{n}}\right)\mathcal{C}_{H}\left(\alpha_{H}\left(\mathbf{0}\right):\lambda_{p}\left(\left[\mathbf{z}\right]_{p^{n}}\right)\right)\left(\frac{H^{\prime}\left(\mathbf{0}\right)}{p^{r}}\right)^{n}\\ & =\kappa_{H}\left(\left[\mathbf{z}\right]_{p^{N}}\right)\left(H^{\prime}\left(\mathbf{0}\right)\right)^{N}\\ & +\sum_{n=0}^{N-1}\kappa_{H}\left(\left[\mathbf{z}\right]_{p^{n}}\right)\underbrace{\left(\mathbf{I}_{d}-\mathcal{C}_{H}\left(\alpha_{H}\left(\mathbf{0}\right):\lambda_{p}\left(\left[\mathbf{z}\right]_{p^{n}}\right)\right)\right)}_{\mathbf{I}_{H}\left(\lambda_{p}\left(\left[\mathbf{z}\right]_{p^{n}}\right)\right)}\left(H^{\prime}\left(\mathbf{0}\right)\right)^{n} \end{align*} which is the desired identity. \vphantom{} III. Let $H$ be semi-basic, and fix $\mathbf{z}\in\left(\mathbb{Z}_{p}^{r}\right)^{\prime}$. Then: \begin{equation} \left\Vert \kappa_{H}\left(\left[\mathbf{z}\right]_{p^{N}}\right)\left(H^{\prime}\left(\mathbf{0}\right)\right)^{N}\right\Vert _{q}\leq\left\Vert \kappa_{H}\left(\left[\mathbf{z}\right]_{p^{N}}\right)\right\Vert _{q} \end{equation} Applying \textbf{Lemma \ref{lem:properties of MD kappa_H}}, $\kappa_{H}\left(\left[\mathbf{z}\right]_{p^{N}}\right)$ converges $q$-adically to $\mathbf{O}_{d}$ as $N\rightarrow\infty$. By the ultrametric structure of $\mathbb{C}_{q}$, this then guarantees the convergence of (\ref{eq:MD Fourier Limit of A_H,N twiddle in standard frame}). Next, let $\mathfrak{\mathbf{z}}=\mathbf{m}\in\mathbb{N}_{0}^{r}$. Then, for all $n\geq\lambda_{p}\left(\mathbf{m}\right)$: \begin{equation} \kappa_{H}\left(\mathbf{m}\right)=\kappa_{H}\left(\left[\mathbf{m}\right]_{p^{n}}\right)=\kappa_{H}\left(\left[\mathbf{m}\right]_{p^{n+1}}\right)\in\textrm{Gl}_{d}\left(\mathbb{Q}\right)\backslash\left\{ \mathbf{O}_{d}\right\} \end{equation} Likewise, keeping $n\geq\lambda_{p}\left(\mathbf{m}\right)$, we have: \begin{align*} \mathbf{I}_{H}\left(\lambda_{p}\left(\left[\mathbf{m}\right]_{p^{n}}\right)\right) & =\mathbf{I}_{d}-\left(H^{\prime}\left(\mathbf{0}\right)\right)^{\lambda_{p}\left(\left[\mathbf{m}\right]_{p^{n}}\right)}\alpha_{H}\left(\mathbf{0}\right)\left(H^{\prime}\left(\mathbf{0}\right)\right)^{-\lambda_{p}\left(\left[\mathbf{m}\right]_{p^{n}}\right)}\\ & =\mathbf{I}_{d}-\left(H^{\prime}\left(\mathbf{0}\right)\right)^{\lambda_{p}\left(\mathbf{m}\right)}\alpha_{H}\left(\mathbf{0}\right)\left(H^{\prime}\left(\mathbf{0}\right)\right)^{-\lambda_{p}\left(\mathbf{m}\right)}\\ & =\mathbf{I}_{d}-\mathcal{C}_{H}\left(\alpha_{H}\left(\mathbf{0}\right):\lambda_{p}\left(\mathbf{m}\right)\right)\\ & =\mathbf{I}_{H}\left(\lambda_{p}\left(\mathbf{m}\right)\right) \end{align*} Examining the tail of the $n$-series in (\ref{eq:MD Convolution of dA_H and D_N}), note that: \[ \sum_{n=0}^{N-1}\kappa_{H}\left(\left[\mathbf{m}\right]_{p^{n}}\right)\mathbf{I}_{H}\left(\lambda_{p}\left(\mathbf{m}\right)\right)\left(H^{\prime}\left(\mathbf{0}\right)\right)^{n}\overset{\overline{\mathbb{Q}}^{d,d}}{=}O\left(\mathbf{I}_{d}\right)+\kappa_{H}\left(\mathbf{m}\right)\mathbf{I}_{H}\left(\lambda_{p}\left(\mathbf{m}\right)\right)\sum_{n=\lambda_{p}\left(\mathbf{m}\right)}^{N-1}\left(H^{\prime}\left(\mathbf{0}\right)\right)^{n} \] holds whenever $N\geq\lambda_{p}\left(\mathbf{m}\right)+1$. Since the $n$-series on the right is geometric in the matrix $H^{\prime}\left(\mathbf{0}\right)$, it will converge in the topology of in $\mathbb{C}$ as $N\rightarrow\infty$ if and only if $H$ is contracting. Likewise, by \textbf{Lemma \ref{lem:properties of MD kappa_H}},\textbf{ }$\kappa_{H}\left(\left[\mathbf{z}\right]_{p^{N}}\right)\left(H^{\prime}\left(\mathbf{0}\right)\right)^{N}$ will tend to $\mathbf{O}_{d}$ in the topology of $\mathbb{C}$ as $N\rightarrow\infty$ if and only if $H$ is contracting, and will diverge in the topology of $\mathbb{C}$ if and only if $H$ is expanding. This shows that (\ref{eq:MD Fourier Limit of A_H,N twiddle in standard frame}) holds true. Q.E.D. \vphantom{} Now we can show that $\hat{A}_{H}$ is the Fourier-Stieltjes transform of a thick measure of matrix type. \begin{cor}[\textbf{$\hat{A}_{H}$ Induces a Matrix-Type Thick Measure}] \ \vphantom{} I. The matrix-valued function $\hat{A}_{H}\left(\mathbf{t}\right)$ is the Fourier-Stieltjes transform of a $\left(p,q_{H}\right)$-adic matrix-type thick measure of depth $r$. This measure acts on functions by way of the formula: \begin{equation} \mathbf{f}\in C\left(\mathbb{Z}_{p}^{r},\mathbb{C}_{q}^{d}\right)\mapsto\sum_{\mathbf{t}\in\hat{\mathbb{Z}}_{p}^{r}}\hat{A}_{H}\left(-\mathbf{t}\right)\hat{\mathbf{f}}\left(\mathbf{t}\right)\in\mathbb{C}_{q}^{d} \end{equation} Moreover, this thick measure is degenerate with respect to the standard $\left(p,q_{H}\right)$-adic frame if and only if $\alpha_{H}\left(\mathbf{0}\right)=\mathbf{I}_{d}$. \vphantom{} II. For any $\mathbf{v}\in\mathbb{C}_{q}^{d}$, the vector-valued function $\hat{A}_{H}\left(\mathbf{t}\right)\mathbf{v}$ is the Fourier-Stieltjes transform of a $\left(p,q_{H}\right)$-adic thick measure of vector type. This measure acts on functions by way of the formula: \begin{equation} \mathbf{f}\in C\left(\mathbb{Z}_{p}^{r},\mathbb{C}_{q}^{d}\right)\mapsto\sum_{\mathbf{t}\in\hat{\mathbb{Z}}_{p}^{r}}\left(\hat{A}_{H}\left(-\mathbf{t}\right)\mathbf{v}\right)\hat{\mathbf{f}}\left(\mathbf{t}\right)\in\mathbb{C}_{q}^{d} \end{equation} Moreover, this thick measure is degenerate with respect to the standard $\left(p,q_{H}\right)$-adic frame if and only if $\alpha_{H}\left(\mathbf{0}\right)=\mathbf{I}_{d}$. \end{cor} Proof: I. Follows by (II) and (III) of \textbf{Theorem \ref{thm:MD properties of A_H hat}}. \vphantom{} II. (I) shows that the matrix-type thick measure associated to $\hat{A}_{H}\left(\mathbf{t}\right)$ is degenerate if and only if $\alpha_{H}\left(\mathbf{0}\right)=\mathbf{I}_{d}$. Noting that the vector-valued Fourier series generated by $\hat{A}_{H}\left(\mathbf{t}\right)\mathbf{v}$ has an $\mathcal{F}_{p,q}$ limit of: \begin{equation} \lim_{N\rightarrow\infty}\sum_{\left\Vert \mathbf{t}\right\Vert _{p}\leq p^{N}}\left(\hat{A}_{H}\left(\mathbf{t}\right)\mathbf{v}\right)e^{2\pi i\left\{ \mathbf{t}\mathbf{z}\right\} _{p}} \end{equation} we can write this as: \begin{equation} \left(\lim_{N\rightarrow\infty}\sum_{\left\Vert \mathbf{t}\right\Vert _{p}\leq p^{N}}\hat{A}_{H}\left(\mathbf{t}\right)e^{2\pi i\left\{ \mathbf{t}\mathbf{z}\right\} _{p}}\right)\mathbf{v} \end{equation} Note that (I) is exactly the statement that the limit in parenthesis (taken with respect to $\mathcal{F}_{p,q}$) is $\mathbf{O}_{d}$ for every $\mathbf{z}\in\mathbb{Z}_{p}^{r}$ if and only if $\alpha_{H}\left(\mathbf{0}\right)=\mathbf{I}_{d}$. Hence: \begin{align*} \lim_{N\rightarrow\infty}\sum_{\left\Vert \mathbf{t}\right\Vert _{p}\leq p^{N}}\left(\hat{A}_{H}\left(\mathbf{t}\right)\mathbf{v}\right)e^{2\pi i\left\{ \mathbf{t}\mathbf{z}\right\} _{p}} & =\left(\lim_{N\rightarrow\infty}\sum_{\left\Vert \mathbf{t}\right\Vert _{p}\leq p^{N}}\hat{A}_{H}\left(\mathbf{t}\right)e^{2\pi i\left\{ \mathbf{t}\mathbf{z}\right\} _{p}}\right)\mathbf{v}\\ \left(\textrm{iff }\alpha_{H}\left(\mathbf{0}\right)=\mathbf{I}_{d}\right); & =\mathbf{O}_{d}\mathbf{v}\\ & =\mathbf{0} \end{align*} This shows that the vector-type thick measure induced by $\hat{A}_{H}\left(\mathbf{t}\right)\mathbf{v}$ is degenerate if and only if $\alpha_{H}\left(\mathbf{0}\right)=\mathbf{I}_{d}$. Q.E.D. \begin{defn}[$\tilde{A}_{H}\left(\mathbf{z}\right)$] We write $\tilde{A}_{H}\left(\mathbf{z}\right)$ to denote the kernel of the matrix-type thick measure induced by $\hat{A}_{H}$; that is, $\tilde{A}_{H}\left(\mathbf{z}\right)$ is the $\mathcal{F}_{p,q_{H}}^{d,d}$-limit of $\tilde{A}_{H,N}\left(\mathbf{z}\right)$ as $N\rightarrow\infty$. \end{defn} \vphantom{} Like in the one-dimensional case, we now can write out our asymptotic decomposition of $\hat{\chi}_{H,N}$. \begin{thm}[\textbf{$\left(N,\mathbf{t}\right)$ Asymptotic Decomposition of $\hat{\chi}_{H,N}$}] \label{thm:MD N,t asympotics for Chi_H,N hat}\ \vphantom{} I. If $\alpha_{H}\left(\mathbf{0}\right)=\mathbf{I}_{d}$, then: \begin{equation} \hat{\chi}_{H,N}\left(\mathbf{t}\right)\overset{\overline{\mathbb{Q}}^{d}}{=}\begin{cases} N\hat{A}_{H}\left(\mathbf{t}\right)\beta_{H}\left(\mathbf{0}\right) & \textrm{if }\mathbf{t}=\mathbf{0}\\ \hat{A}_{H}\left(\mathbf{t}\right)\left(\left(N+v_{p}\left(\mathbf{t}\right)\right)\beta_{H}\left(\mathbf{0}\right)+\gamma_{H}\left(\frac{\mathbf{t}\left|\mathbf{t}\right|_{p}}{p}\right)\right) & \textrm{if }0<\left\Vert \mathbf{t}\right\Vert _{p}<p^{N}\\ \hat{A}_{H}\left(\mathbf{t}\right)\gamma_{H}\left(\frac{\mathbf{t}\left|\mathbf{t}\right|_{p}}{p}\right) & \textrm{if }\left\Vert \mathbf{t}\right\Vert _{p}=p^{N}\\ \mathbf{0} & \textrm{if }\left\Vert \mathbf{t}\right\Vert _{p}>p^{N} \end{cases},\textrm{ }\forall\mathbf{t}\in\hat{\mathbb{Z}}_{p}^{r}\label{eq:MD Formula for Chi_H,N hat when alpha is 1} \end{equation} \vphantom{} II. If $\mathbf{I}_{d}-\alpha_{H}\left(\mathbf{0}\right)$ is invertible, then: \begin{equation} \hat{\chi}_{H,N}\left(\mathbf{t}\right)\overset{\overline{\mathbb{Q}}^{d}}{=}\begin{cases} \hat{A}_{H}\left(\mathbf{t}\right)\frac{\mathbf{I}_{d}-\left(\alpha_{H}\left(\mathbf{0}\right)\right)^{N}}{\mathbf{I}_{d}-\alpha_{H}\left(\mathbf{0}\right)}\beta_{H}\left(\mathbf{0}\right) & \textrm{if }\mathbf{t}=\mathbf{0}\\ \hat{A}_{H}\left(\mathbf{t}\right)\left(\gamma_{H}\left(\frac{\mathbf{t}\left|\mathbf{t}\right|_{p}}{p}\right)+\frac{\mathbf{I}_{d}-\left(\alpha_{H}\left(\mathbf{0}\right)\right)^{N+v_{p}\left(\mathbf{t}\right)}}{\mathbf{I}_{d}-\alpha_{H}\left(\mathbf{0}\right)}\beta_{H}\left(\mathbf{0}\right)\right) & \textrm{if }0<\left\Vert \mathbf{t}\right\Vert _{p}<p^{N}\\ \hat{A}_{H}\left(\mathbf{t}\right)\gamma_{H}\left(\frac{\mathbf{t}\left|\mathbf{t}\right|_{p}}{p}\right) & \textrm{if }\left\Vert \mathbf{t}\right\Vert _{p}=p^{N}\\ \mathbf{0} & \textrm{if }\left\Vert \mathbf{t}\right\Vert _{p}>p^{N} \end{cases},\textrm{ }\forall\mathbf{t}\in\hat{\mathbb{Z}}_{p}^{r}\label{eq:MD Formula for Chi_H,N hat when alpha is not 1} \end{equation} \end{thm} Proof: For brevity, we write: \begin{equation} \hat{A}_{H,n}\left(\mathbf{t}\right)\overset{\textrm{def}}{=}\begin{cases} \mathbf{I}_{d} & \textrm{if }n=0\\ \mathbf{1}_{\mathbf{0}}\left(p^{n+1}\mathbf{t}\right)\prod_{m=0}^{n-1}\alpha_{H}\left(p^{m}\mathbf{t}\right) & \textrm{if }n\geq1 \end{cases}\label{eq:MD Definition of A_H,n+1 hat} \end{equation} so that (\ref{eq:MD Chi_H,N hat functional equation}) becomes: \begin{equation} \hat{\chi}_{H,N}\left(\mathbf{t}\right)=\sum_{n=0}^{N-1}\hat{A}_{H,n}\left(\mathbf{t}\right)\beta_{H}\left(p^{n}\mathbf{t}\right) \end{equation} Now, letting $\mathbf{t}\in\hat{\mathbb{Z}}_{p}^{r}$ be non-zero and satisfy $\left\Vert \mathbf{t}\right\Vert _{p}\leq p^{N}$, we use \textbf{Proposition \ref{prop:MD alpha_H A_H hat product}} to write: \begin{equation} \hat{A}_{H,n}\left(\mathbf{t}\right)=\begin{cases} \mathbf{O}_{d} & \textrm{if }n\leq-v_{p}\left(\mathbf{t}\right)-2\\ \frac{\hat{A}_{H}\left(\mathbf{t}\right)}{\alpha_{H}\left(p^{-v_{p}\left(\mathbf{t}\right)-1}\mathbf{t}\right)} & \textrm{if }n=-v_{p}\left(\mathbf{t}\right)-1\\ \left(\alpha_{H}\left(\mathbf{0}\right)\right)^{n+v_{p}\left(\mathbf{t}\right)}\hat{A}_{H}\left(\mathbf{t}\right) & \textrm{if }n\geq-v_{p}\left(\mathbf{t}\right) \end{cases} \end{equation} So: \begin{equation} \hat{\chi}_{H,N}\left(\mathbf{t}\right)\overset{\overline{\mathbb{Q}}^{d}}{=}\sum_{n=0}^{N-1}\hat{A}_{H,n}\left(\mathbf{t}\right)\beta_{H}\left(p^{n}\mathbf{t}\right)=\sum_{n=-v_{p}\left(\mathbf{t}\right)-1}^{N-1}\hat{A}_{H,n}\left(\mathbf{t}\right)\beta_{H}\left(p^{n}\mathbf{t}\right)\label{eq:MD Chi_H,N hat as Beta_H plus A_H,n hat - ready for t,n analysis} \end{equation} Note that in obtaining (\ref{eq:MD Chi_H,N hat as Beta_H plus A_H,n hat - ready for t,n analysis}), we have used the fact: \begin{equation} \hat{A}_{H,n}\left(\mathbf{t}\right)=\mathbf{O}_{d},\textrm{ }\forall n\leq-v_{p}\left(\mathbf{t}\right)-2 \end{equation} which is, itself, a consequence of the vanishing of $\mathbf{1}_{\mathbf{0}}\left(p^{n+1}\mathbf{t}\right)$ whenever $n\leq-v_{p}\left(\mathbf{t}\right)-2$. With that done, we are left with two cases. \vphantom{} \textbullet{} First, suppose $\left\Vert \mathbf{t}\right\Vert _{p}=p^{N}$. Then $N=-v_{p}\left(\mathbf{t}\right)$, and so, (\ref{eq:MD Chi_H,N hat as Beta_H plus A_H,n hat - ready for t,n analysis}) becomes: \begin{align*} \hat{\chi}_{H,N}\left(\mathbf{t}\right) & =\sum_{n=-v_{p}\left(\mathbf{t}\right)-1}^{N-1}\hat{A}_{H,n}\left(\mathbf{t}\right)\beta_{H}\left(p^{n}\mathbf{t}\right)\\ & =\hat{A}_{H,-v_{p}\left(\mathbf{t}\right)-1}\left(\mathbf{t}\right)\beta_{H}\left(p^{-v_{p}\left(\mathbf{t}\right)-1}\mathbf{t}\right)\\ & =\left(\mathbf{1}_{\mathbf{0}}\left(p^{-v_{p}\left(\mathbf{t}\right)-1+1}\mathbf{t}\right)\prod_{m=0}^{-v_{p}\left(\mathbf{t}\right)-1-1}\alpha_{H}\left(p^{m}\mathbf{t}\right)\right)\beta_{H}\left(p^{-v_{p}\left(\mathbf{t}\right)-1}\mathbf{t}\right)\\ & =\left(\mathbf{1}_{\mathbf{0}}\left(p^{-v_{p}\left(\mathbf{t}\right)}\mathbf{t}\right)\prod_{m=0}^{-v_{p}\left(\mathbf{t}\right)-1}\alpha_{H}\left(p^{m}\mathbf{t}\right)\right)\left(\alpha_{H}\left(p^{-v_{p}\left(\mathbf{t}\right)-1}\mathbf{t}\right)\right)^{-1}\beta_{H}\left(p^{-v_{p}\left(\mathbf{t}\right)-1}\mathbf{t}\right) \end{align*} Because $p^{-v_{p}\left(\mathbf{t}\right)}\mathbf{t}$ contains only integer entries, note that $\mathbf{1}_{\mathbf{0}}\left(p^{-v_{p}\left(\mathbf{t}\right)}\mathbf{t}\right)=1$. This leaves us with: \begin{align*} \hat{\chi}_{H,N}\left(\mathbf{t}\right) & =\left(\prod_{m=0}^{-v_{p}\left(\mathbf{t}\right)-1}\alpha_{H}\left(p^{m}\mathbf{t}\right)\right)\left(\alpha_{H}\left(p^{-v_{p}\left(\mathbf{t}\right)-1}\mathbf{t}\right)\right)^{-1}\beta_{H}\left(p^{-v_{p}\left(\mathbf{t}\right)-1}\mathbf{t}\right)\\ & =\hat{A}_{H}\left(\mathbf{t}\right)\gamma_{H}\left(\frac{\mathbf{t}\left|\mathbf{t}\right|_{p}}{p}\right) \end{align*} \vphantom{} \textbullet{} Second, suppose $0<\left\Vert \mathbf{t}\right\Vert _{p}<p^{N}$. Then $-v_{p}\left(\mathbf{t}\right)-1<N-1$, and so $p^{n}\mathbf{t}\overset{1}{\equiv}\mathbf{0}$ for all $n\geq-v_{p}\left(\mathbf{t}\right)$. Consequently, (\ref{eq:MD Chi_H,N hat as Beta_H plus A_H,n hat - ready for t,n analysis}) becomes: \begin{align*} \hat{\chi}_{H,N}\left(\mathbf{t}\right) & =\hat{A}_{H,-v_{p}\left(\mathbf{t}\right)-1}\left(\mathbf{t}\right)\beta_{H}\left(p^{-v_{p}\left(\mathbf{t}\right)-1}\mathbf{t}\right)+\sum_{n=-v_{p}\left(\mathbf{t}\right)}^{N-1}\hat{A}_{H,n}\left(\mathbf{t}\right)\beta_{H}\left(p^{n}\mathbf{t}\right)\\ \left(p^{n}\mathbf{t}\overset{1}{\equiv}\mathbf{0}\textrm{ }\forall n\geq-v_{p}\left(\mathbf{t}\right)\right); & =\hat{A}_{H}\left(\mathbf{t}\right)\gamma_{H}\left(\frac{\mathbf{t}\left|\mathbf{t}\right|_{p}}{p}\right)+\sum_{n=-v_{p}\left(\mathbf{t}\right)}^{N-1}\hat{A}_{H,n}\left(\mathbf{t}\right)\beta_{H}\left(\mathbf{0}\right) \end{align*} Now, using \textbf{Proposition \ref{prop:MD alpha_H A_H hat product}}, we write: \begin{equation} \hat{A}_{H,n}\left(\mathbf{t}\right)=\hat{A}_{H}\left(\mathbf{t}\right)\left(\alpha_{H}\left(\mathbf{0}\right)\right)^{n+v_{p}\left(\mathbf{t}\right)},\textrm{ }\forall n\geq-v_{p}\left(\mathbf{t}\right) \end{equation} and hence: \begin{align*} \hat{\chi}_{H,N}\left(\mathbf{t}\right) & =\hat{A}_{H}\left(\mathbf{t}\right)\gamma_{H}\left(\frac{\mathbf{t}\left|\mathbf{t}\right|_{p}}{p}\right)+\sum_{n=-v_{p}\left(\mathbf{t}\right)}^{N-1}\hat{A}_{H}\left(\mathbf{t}\right)\left(\alpha_{H}\left(\mathbf{0}\right)\right)^{n+v_{p}\left(\mathbf{t}\right)}\beta_{H}\left(\mathbf{0}\right)\\ & =\hat{A}_{H}\left(\mathbf{t}\right)\gamma_{H}\left(\frac{\mathbf{t}\left|\mathbf{t}\right|_{p}}{p}\right)+\hat{A}_{H}\left(\mathbf{t}\right)\sum_{n=0}^{N+v_{p}\left(\mathbf{t}\right)-1}\left(\alpha_{H}\left(\mathbf{0}\right)\right)^{n}\beta_{H}\left(\mathbf{0}\right) \end{align*} When $\alpha_{H}\left(\mathbf{0}\right)=\mathbf{I}_{d}$, this gives: \begin{equation} \hat{\chi}_{H,N}\left(\mathbf{t}\right)=\hat{A}_{H}\left(\mathbf{t}\right)\gamma_{H}\left(\frac{\mathbf{t}\left|\mathbf{t}\right|_{p}}{p}\right)+\left(N+v_{p}\left(\mathbf{t}\right)\right)\hat{A}_{H}\left(\mathbf{t}\right)\beta_{H}\left(\mathbf{0}\right)\label{eq:MD asymptotic analysis, nearly done} \end{equation} On the other hand, suppose that $\mathbf{I}_{d}-\alpha_{H}\left(\mathbf{0}\right)$ is invertible. Then, since: \begin{align} \left(\mathbf{I}_{d}-\alpha_{H}\left(\mathbf{0}\right)\right)\sum_{m=0}^{M-1}\left(\alpha_{H}\left(\mathbf{0}\right)\right)^{m} & =\mathbf{I}_{d}-\left(\alpha_{H}\left(\mathbf{0}\right)\right)^{M}=\sum_{m=0}^{M-1}\left(\alpha_{H}\left(\mathbf{0}\right)\right)^{m}\left(\mathbf{I}_{d}-\alpha_{H}\left(\mathbf{0}\right)\right) \end{align} it follows that: \begin{align*} \sum_{m=0}^{M-1}\left(\alpha_{H}\left(\mathbf{0}\right)\right)^{m} & =\left(\mathbf{I}_{d}-\alpha_{H}\left(\mathbf{0}\right)\right)^{-1}\left(\mathbf{I}_{d}-\left(\alpha_{H}\left(\mathbf{0}\right)\right)^{M}\right)\\ & =\left(\mathbf{I}_{d}-\left(\alpha_{H}\left(\mathbf{0}\right)\right)^{M}\right)\left(\mathbf{I}_{d}-\alpha_{H}\left(\mathbf{0}\right)\right)^{-1} \end{align*} With this, the right-hand side of (\ref{eq:MD asymptotic analysis, nearly done}) becomes: \[ \begin{cases} \hat{A}_{H}\left(\mathbf{t}\right)\gamma_{H}\left(\frac{\mathbf{t}\left|\mathbf{t}\right|_{p}}{p}\right)+\left(N+v_{p}\left(\mathbf{t}\right)\right)\hat{A}_{H}\left(\mathbf{t}\right)\beta_{H}\left(\mathbf{0}\right) & \textrm{if }\alpha_{H}\left(\mathbf{0}\right)=\mathbf{I}_{d}\\ \hat{A}_{H}\left(\mathbf{t}\right)\gamma_{H}\left(\frac{\mathbf{t}\left|\mathbf{t}\right|_{p}}{p}\right)+\hat{A}_{H}\left(\mathbf{t}\right)\frac{\mathbf{I}_{d}-\left(\alpha_{H}\left(\mathbf{0}\right)\right)^{N+v_{p}\left(\mathbf{t}\right)}}{\mathbf{I}_{d}-\alpha_{H}\left(\mathbf{0}\right)}\beta_{H}\left(\mathbf{0}\right) & \textrm{if }\det\left(\mathbf{I}_{d}-\alpha_{H}\left(\mathbf{0}\right)\right)\neq0 \end{cases} \] Finally, when $\mathbf{t}=\mathbf{0}$: \begin{align*} \hat{\chi}_{H,N}\left(\mathbf{0}\right) & =\sum_{n=0}^{N-1}\hat{A}_{H,n}\left(\mathbf{0}\right)\beta_{H}\left(\mathbf{0}\right)\\ & =\beta_{H}\left(\mathbf{0}\right)+\sum_{n=1}^{N-1}\mathbf{1}_{\mathbf{0}}\left(\mathbf{0}\right)\left(\prod_{m=0}^{n-1}\alpha_{H}\left(\mathbf{0}\right)\right)\beta_{H}\left(\mathbf{0}\right)\\ & =\beta_{H}\left(\mathbf{0}\right)+\sum_{n=1}^{N-1}\left(\alpha_{H}\left(\mathbf{0}\right)\right)^{n}\beta_{H}\left(\mathbf{0}\right)\\ & =\begin{cases} \beta_{H}\left(\mathbf{0}\right)N & \textrm{if }\alpha_{H}\left(\mathbf{0}\right)=\mathbf{I}_{d}\\ \frac{\mathbf{I}_{d}-\left(\alpha_{H}\left(\mathbf{0}\right)\right)^{N}}{\mathbf{I}_{d}-\alpha_{H}\left(\mathbf{0}\right)}\beta_{H}\left(\mathbf{0}\right) & \textrm{if }\det\left(\mathbf{I}_{d}-\alpha_{H}\left(\mathbf{0}\right)\right)\neq0 \end{cases} \end{align*} Since $\hat{A}_{H}\left(\mathbf{0}\right)=\mathbf{I}_{d}$, we have that $\beta_{H}\left(\mathbf{0}\right)N=\hat{A}_{H}\left(\mathbf{t}\right)\beta_{H}\left(\mathbf{0}\right)N$ when $\mathbf{t}=\mathbf{0}$ (in the $\alpha_{H}\left(\mathbf{0}\right)=\mathbf{I}_{d}$ case) and: \[ \frac{\mathbf{I}_{d}-\left(\alpha_{H}\left(\mathbf{0}\right)\right)^{N}}{\mathbf{I}_{d}-\alpha_{H}\left(\mathbf{0}\right)}\beta_{H}\left(\mathbf{0}\right)=\hat{A}_{H}\left(\mathbf{t}\right)\frac{\mathbf{I}_{d}-\left(\alpha_{H}\left(\mathbf{0}\right)\right)^{N}}{\mathbf{I}_{d}-\alpha_{H}\left(\mathbf{0}\right)}\beta_{H}\left(\mathbf{0}\right) \] when $\mathbf{t=0}$ in the $\det\left(\mathbf{I}_{d}-\alpha_{H}\left(\mathbf{0}\right)\right)\neq0$ case. Q.E.D. \vphantom{} When $\alpha_{H}\left(\mathbf{0}\right)=\mathbf{I}_{d}$, we have that \[ \hat{\chi}_{H,N}\left(\mathbf{t}\right)-N\mathbf{1}_{\mathbf{0}}\left(p^{N-1}\mathbf{t}\right)\hat{A}_{H}\left(\mathbf{t}\right)\beta_{H}\left(\mathbf{0}\right) \] is given by: \begin{equation} \begin{cases} \mathbf{0} & \textrm{if }\mathbf{t}=\mathbf{0}\\ \hat{A}_{H}\left(\mathbf{t}\right)\left(v_{p}\left(\mathbf{t}\right)\beta_{H}\left(\mathbf{0}\right)+\gamma_{H}\left(\frac{\mathbf{t}\left|\mathbf{t}\right|_{p}}{p}\right)\right) & \textrm{if }0<\left\Vert \mathbf{t}\right\Vert _{p}<p^{N}\\ \hat{A}_{H}\left(\mathbf{t}\right)\gamma_{H}\left(\frac{\mathbf{t}\left|\mathbf{t}\right|_{p}}{p}\right) & \textrm{if }\left\Vert \mathbf{t}\right\Vert _{p}=p^{N}\\ \mathbf{0} & \textrm{if }\left\Vert \mathbf{t}\right\Vert _{p}>p^{N} \end{cases}\label{eq:MD Fine structure of Chi_H,N hat when alpha is 1} \end{equation} \subsection{\label{subsec:6.2.2 Multi-Dimensional--=00003D000026}Multi-Dimensional $\hat{\chi}_{H}$ and $\tilde{\chi}_{H,N}$ for $\alpha_{H}\left(\mathbf{0}\right)=\mathbf{I}_{d}$} We begin by computing the Fourier series generated by $v_{p}\left(\mathbf{t}\right)\hat{A}_{H}\left(\mathbf{t}\right)$. \begin{lem}[\textbf{$v_{p}\hat{A}_{H}$ Summation Formulae}] \label{lem:MD v_p A_H hat summation formulae}\ \vphantom{} I. \begin{align} \sum_{0<\left\Vert \mathbf{t}\right\Vert _{p}\leq p^{N}}v_{p}\left(\mathbf{t}\right)\hat{A}_{H}\left(\mathbf{t}\right)e^{2\pi i\left\{ \mathbf{t}\mathbf{z}\right\} _{p}}\overset{\overline{\mathbb{Q}}^{d,d}}{=} & -N\kappa_{H}\left(\left[\mathbf{z}\right]_{p^{N}}\right)\left(H^{\prime}\left(\mathbf{0}\right)\right)^{N}\label{eq:MD Fourier sum of A_H hat v_rho}\\ & +\sum_{n=0}^{N-1}\kappa_{H}\left(\left[\mathbf{z}\right]_{p^{n}}\right)\left(H^{\prime}\left(\mathbf{0}\right)\right)^{n}\nonumber \\ & -\sum_{n=0}^{N-1}\left(n+1\right)\kappa_{H}\left(\left[\mathbf{z}\right]_{p^{n}}\right)\mathbf{I}_{H}\left(\lambda_{p}\left(\left[\mathbf{z}\right]_{p^{n}}\right)\right)\left(H^{\prime}\left(\mathbf{0}\right)\right)^{n}\nonumber \end{align} Additionally, if $H$ is commutative: \begin{align} \sum_{0<\left\Vert \mathbf{t}\right\Vert _{p}\leq p^{N}}v_{p}\left(\mathbf{t}\right)\hat{A}_{H}\left(\mathbf{t}\right)e^{2\pi i\left\{ \mathbf{t}\mathbf{z}\right\} _{p}}\overset{\overline{\mathbb{Q}}^{d,d}}{=} & -N\kappa_{H}\left(\left[\mathbf{z}\right]_{p^{N}}\right)\left(H^{\prime}\left(\mathbf{0}\right)\right)^{N}\label{eq:MD Fourier sum of A_H hat v_rho, commutative}\\ & +\sum_{n=0}^{N-1}\kappa_{H}\left(\left[\mathbf{z}\right]_{p^{n}}\right)\left(H^{\prime}\left(\mathbf{0}\right)\right)^{n}\nonumber \\ & -\sum_{n=0}^{N-1}\left(n+1\right)\kappa_{H}\left(\left[\mathbf{z}\right]_{p^{n}}\right)\left(H^{\prime}\left(\mathbf{0}\right)\right)^{n}\left(\mathbf{I}_{d}-\alpha_{H}\left(\mathbf{0}\right)\right)\nonumber \end{align} \vphantom{} II. \begin{align} \sum_{\mathbf{t}\in\hat{\mathbb{Z}}_{p}^{r}\backslash\left\{ \mathbf{0}\right\} }v_{p}\left(\mathbf{t}\right)\hat{A}_{H}\left(\mathbf{t}\right)e^{2\pi i\left\{ \mathbf{t}\mathbf{z}\right\} _{p}}\overset{\mathcal{F}_{p,q}^{d,d}}{=} & \sum_{n=0}^{\infty}\kappa_{H}\left(\left[\mathbf{z}\right]_{p^{n}}\right)\left(H^{\prime}\left(\mathbf{0}\right)\right)^{n}\label{eq:MD Limit of Fourier sum of v_rho A_H hat}\\ & -\sum_{n=0}^{\infty}\left(n+1\right)\kappa_{H}\left(\left[\mathbf{z}\right]_{p^{n}}\right)\mathbf{I}_{H}\left(\left[\mathbf{z}\right]_{p^{n}}\right)\left(H^{\prime}\left(\mathbf{0}\right)\right)^{n}\nonumber \end{align} for all $\mathbf{z}\in\mathbb{Z}_{p}^{r}$, where the convergence is point-wise. The right-hand side can also be written as: \begin{equation} \sum_{n=0}^{\infty}\kappa_{H}\left(\left[\mathbf{z}\right]_{p^{n}}\right)\left(\left(n+1\right)\mathcal{C}_{H}\left(\alpha_{H}\left(\mathbf{0}\right):\lambda_{p}\left(\left[\mathbf{z}\right]_{p^{n}}\right)\right)-n\mathbf{I}_{d}\right)\left(H^{\prime}\left(\mathbf{0}\right)\right)^{n}\label{eq:MD Limit of Fourier sum of v_rho A_H hat, Alt} \end{equation} When $H$ is commutative, we get: \begin{equation} \sum_{\mathbf{t}\in\hat{\mathbb{Z}}_{p}^{r}\backslash\left\{ \mathbf{0}\right\} }v_{p}\left(\mathbf{t}\right)\hat{A}_{H}\left(\mathbf{t}\right)e^{2\pi i\left\{ \mathbf{t}\mathbf{z}\right\} _{p}}\overset{\mathcal{F}_{p,q}^{d,d}}{=}\sum_{n=0}^{\infty}\kappa_{H}\left(\left[\mathbf{z}\right]_{p^{n}}\right)\left(\left(n+1\right)\alpha_{H}\left(\mathbf{0}\right)-n\mathbf{I}_{d}\right)\left(H^{\prime}\left(\mathbf{0}\right)\right)^{n}\label{eq:MD Limit of Fourier sum of v_rho A_H hat, Alt, commutative} \end{equation} \end{lem} Proof: For (I), using (\ref{eq:MD Convolution of dA_H and D_N}) gives: \begin{align*} \sum_{0<p\left\Vert \mathbf{t}\right\Vert \leq p^{N}}v_{p}\left(\mathbf{t}\right)\hat{A}_{H}\left(\mathbf{t}\right)e^{2\pi i\left\{ \mathbf{t}\mathbf{z}\right\} _{p}} & =\sum_{n=0}^{N-1}\kappa_{H}\left(\left[\mathbf{z}\right]_{p^{n}}\right)\left(H^{\prime}\left(\mathbf{0}\right)\right)^{n}\\ & -N\kappa_{H}\left(\left[\mathbf{z}\right]_{p^{N}}\right)\left(H^{\prime}\left(\mathbf{0}\right)\right)^{N}\\ & -N\sum_{m=0}^{N-1}\kappa_{H}\left(\left[\mathbf{z}\right]_{p^{m}}\right)\mathbf{I}_{H}\left(\lambda_{p}\left(\left[\mathbf{z}\right]_{p^{m}}\right)\right)\left(H^{\prime}\left(\mathbf{0}\right)\right)^{m}\\ & +\sum_{n=1}^{N-1}\sum_{m=0}^{n-1}\kappa_{H}\left(\left[\mathbf{z}\right]_{p^{m}}\right)\mathbf{I}_{H}\left(\lambda_{p}\left(\left[\mathbf{z}\right]_{p^{m}}\right)\right)\left(H^{\prime}\left(\mathbf{0}\right)\right)^{m} \end{align*} where we used the fact that $\tilde{A}_{H,0}\left(\mathbf{z}\right)=\hat{A}_{H}\left(\mathbf{0}\right)=\mathbf{I}_{d}$. Next, we note the formal identity (provable by summation by parts): \begin{equation} \sum_{n=1}^{N-1}\sum_{m=0}^{n-1}f\left(m\right)=\sum_{m=0}^{N-2}\sum_{n=m+1}^{N-1}f\left(m\right)=\sum_{m=0}^{N-2}\left(N-1-m\right)f\left(m\right) \end{equation} Applying this to our sum yields: \begin{align*} \sum_{0<\left\Vert \mathbf{t}\right\Vert _{p}\leq p^{N}}v_{p}\left(\mathbf{t}\right)\hat{A}_{H}\left(\mathbf{t}\right)e^{2\pi i\left\{ \mathbf{t}\mathbf{z}\right\} _{p}} & =\sum_{n=0}^{N-1}\kappa_{H}\left(\left[\mathbf{z}\right]_{p^{n}}\right)\left(H^{\prime}\left(\mathbf{0}\right)\right)^{n}\\ & -N\kappa_{H}\left(\left[\mathbf{z}\right]_{p^{N}}\right)\left(H^{\prime}\left(\mathbf{0}\right)\right)^{N}\\ & -N\sum_{m=0}^{N-1}\kappa_{H}\left(\left[\mathbf{z}\right]_{p^{m}}\right)\mathbf{I}_{H}\left(\lambda_{p}\left(\left[\mathbf{z}\right]_{p^{m}}\right)\right)\left(H^{\prime}\left(\mathbf{0}\right)\right)^{m}\\ & +\sum_{m=0}^{N-2}\left(N-1-m\right)\kappa_{H}\left(\left[\mathbf{z}\right]_{p^{m}}\right)\mathbf{I}_{H}\left(\lambda_{p}\left(\left[\mathbf{z}\right]_{p^{m}}\right)\right)\left(H^{\prime}\left(\mathbf{0}\right)\right)^{m} \end{align*} and hence: \begin{align*} \sum_{0<\left\Vert \mathbf{t}\right\Vert _{p}\leq p^{N}}v_{p}\left(\mathbf{t}\right)\hat{A}_{H}\left(\mathbf{t}\right)e^{2\pi i\left\{ \mathbf{t}\mathbf{z}\right\} _{p}} & =-N\kappa_{H}\left(\left[\mathbf{z}\right]_{p^{N}}\right)\left(H^{\prime}\left(\mathbf{0}\right)\right)^{N}\\ & +\sum_{n=0}^{N-1}\kappa_{H}\left(\left[\mathbf{z}\right]_{p^{n}}\right)\left(H^{\prime}\left(\mathbf{0}\right)\right)^{n}\\ & -\sum_{n=0}^{N-1}\left(n+1\right)\kappa_{H}\left(\left[\mathbf{z}\right]_{p^{n}}\right)\mathbf{I}_{H}\left(\lambda_{p}\left(\left[\mathbf{z}\right]_{p^{n}}\right)\right)\left(H^{\prime}\left(\mathbf{0}\right)\right)^{n} \end{align*} The $\mathcal{F}_{p,q}^{d,d}$-convergence of this sum as $N\rightarrow\infty$ in the case where $H$ is semi-basic and contracting follows from the decay properties ($q$-adic and archimedean, respectively) of $\kappa_{H}$ and $\left(H^{\prime}\left(\mathbf{0}\right)\right)^{n}$, as well as the boundedness of $\mathbf{I}_{H}\left(\lambda_{p}\left(\left[\mathbf{z}\right]_{p^{n}}\right)\right)$ in both topologies with respect to $n$. Finally, writing: \begin{equation} \mathbf{I}_{H}\left(\lambda_{p}\left(\left[\mathbf{z}\right]_{p^{n}}\right)\right)=\mathbf{I}_{d}-\mathcal{C}_{H}\left(\alpha_{H}\left(\mathbf{0}\right):\lambda_{p}\left(\left[\mathbf{z}\right]_{p^{n}}\right)\right) \end{equation} yields (\ref{eq:MD Limit of Fourier sum of v_rho A_H hat, Alt}). Q.E.D. \vphantom{} Like in the one-dimensional case, we now introduce $\varepsilon_{n}$. \begin{defn}[\textbf{Multi-Dimensional $\varepsilon_{n}$}] For each $n\in\mathbb{N}_{0}$, \nomenclature{$\varepsilon_{n}\left(\mathbf{z}\right)$}{ }we define $\varepsilon_{n}:\mathbb{Z}_{p}^{r}\rightarrow\overline{\mathbb{Q}}$ by: \begin{equation} \varepsilon_{n}\left(\mathbf{z}\right)\overset{\textrm{def}}{=}e^{\frac{2\pi i}{p^{n+1}}\left(\left[\mathbf{z}\right]_{p^{n+1}}-\left[\mathbf{z}\right]_{p^{n}}\right)}=e^{2\pi i\left\{ \frac{\mathbf{z}}{p^{n+1}}\right\} _{p}}\cdot e^{-\frac{2\pi i}{p}\left\{ \frac{\mathbf{z}}{p^{n}}\right\} _{p}}\label{eq:MD Definition of epsilon_n} \end{equation} \end{defn} Once again, we have functional equations, among other things. \begin{prop}[\textbf{Properties of Multi-Dimensional $\varepsilon_{n}$}] \label{prop:properties of MD epsilon n}\ \vphantom{} I. \begin{equation} \varepsilon_{0}\left(\mathbf{z}\right)=e^{2\pi i\left\{ \frac{\mathbf{z}}{p}\right\} _{p}},\textrm{ }\forall\mathbf{z}\in\mathbb{Z}_{p}^{r}\label{eq:MD Epsilon 0 of z} \end{equation} \begin{equation} \varepsilon_{n}\left(\mathbf{j}\right)=1,\textrm{ }\forall\mathbf{j}\in\mathbb{Z}^{r}/p\mathbb{Z}^{r},\textrm{ }\forall n\geq1\label{eq:MD Epsilon_n of j} \end{equation} \vphantom{} II. \begin{equation} \varepsilon_{n}\left(p\mathbf{m}+\mathbf{j}\right)=\begin{cases} \varepsilon_{0}\left(\mathbf{j}\right) & \textrm{if }n=0\\ \varepsilon_{n-1}\left(\mathbf{m}\right) & \textrm{if }n\geq1 \end{cases},\textrm{ }\forall\mathbf{m}\in\mathbb{N}_{0}^{r},\textrm{ }\forall\mathbf{j}\in\mathbb{Z}^{r}/p\mathbb{Z}^{r},\textrm{ }\forall n\geq1\label{eq:MD epsilon_n functional equations} \end{equation} \vphantom{} III. Let $\mathbf{z}\neq\mathbf{0}$. Then $\varepsilon_{n}\left(\mathbf{z}\right)=1\textrm{ }$ for all $n<v_{p}\left(\mathbf{z}\right)$. \end{prop} Proof: I. The identity (\ref{eq:MD Epsilon 0 of z}) is immediate from the definition of $\varepsilon_{n}$. As for the other one, note that $\left[\mathbf{j}\right]_{p^{n}}=\mathbf{j}$ for all $n\geq1$ and all $\mathbf{j}\in\mathbb{Z}^{r}/p\mathbb{Z}^{r}$. So, letting $n\geq1$, we get: \begin{equation} \varepsilon_{n}\left(\mathbf{j}\right)=e^{\frac{2\pi i}{p^{n+1}}\left(\left[\mathbf{j}\right]_{p^{n+1}}-\left[\mathbf{j}\right]_{p^{n}}\right)}=e^{\frac{2\pi i}{p^{n+1}}\cdot\mathbf{0}}=1 \end{equation} as desired. \vphantom{} II. \begin{align*} \varepsilon_{n}\left(p\mathbf{m}+\mathbf{j}\right) & =e^{2\pi i\left\{ \frac{p\mathbf{m}+\mathbf{j}}{p^{n+1}}\right\} _{p}}e^{-\frac{2\pi i}{p}\left\{ \frac{p\mathbf{m}+\mathbf{j}}{p^{n}}\right\} _{p}}\\ & =e^{2\pi i\left\{ \frac{\mathbf{j}}{p^{n+1}}\right\} _{p}}e^{-\frac{2\pi i}{p}\left\{ \frac{\mathbf{j}}{p^{n}}\right\} _{p}}\cdot e^{2\pi i\left\{ \frac{\mathbf{m}}{p^{n}}\right\} _{p}}e^{-\frac{2\pi i}{p}\left\{ \frac{\mathbf{m}}{p^{n-1}}\right\} _{p}}\\ & =\varepsilon_{n}\left(\mathbf{j}\right)\varepsilon_{n-1}\left(\mathbf{m}\right)\\ \left(\textrm{by (I)}\right); & =\begin{cases} \varepsilon_{0}\left(\mathbf{j}\right) & \textrm{if }n=0\\ \varepsilon_{n-1}\left(\mathbf{m}\right) & \textrm{if }n\geq1 \end{cases} \end{align*} \vphantom{} III. Let $\mathbf{z}$ be non-zero. When $n<v_{p}\left(\mathbf{z}\right)$, we have that $p^{-n}\mathbf{z}$ and $p^{-\left(n+1\right)}\mathbf{z}$ are then $p$-adic integer tuples. As such: \begin{equation} \varepsilon_{n}\left(\mathbf{z}\right)=e^{2\pi i\left\{ \frac{\mathbf{z}}{p^{n+1}}\right\} _{p}}e^{-\frac{2\pi i}{p}\left\{ \frac{\mathbf{z}}{p^{n}}\right\} _{p}}=e^{0}\cdot e^{-0}=1 \end{equation} Q.E.D. \vphantom{}Now we compute the sum of the Fourier series generated by $\hat{A}_{H}\left(\mathbf{t}\right)\gamma_{H}\left(\frac{\mathbf{t}\left|\mathbf{t}\right|_{p}}{p}\right)$. Once again, the non-singularity of $H$ will be necessary for this to work out, as will $H$'s semi-basicness and contracting-ness. \begin{lem}[\textbf{Multi-Dimensional $\gamma_{H}\hat{A}_{H}$ Summation Formulae}] \label{lem:MD gamma formulae}\ \vphantom{} I. The column vector: \begin{equation} \sum_{0<\left\Vert \mathbf{t}\right\Vert _{p}\leq p^{N}}\hat{A}_{H}\left(\mathbf{t}\right)\gamma_{H}\left(\frac{\mathbf{t}\left|\mathbf{t}\right|_{p}}{p}\right)e^{2\pi i\left\{ \mathbf{t}\mathbf{z}\right\} _{p}}\in\overline{\mathbb{Q}}^{d}\label{eq:Left Hand Side of MD Gamma Formulae (partial Fourier sum)} \end{equation} is given by: \begin{equation} \sum_{n=0}^{N-1}\kappa_{H}\left(\left[\mathbf{z}\right]_{p^{n}}\right)\sum_{\mathbf{j}>\mathbf{0}}^{p-1}\mathcal{C}_{H}\left(\alpha_{H}\left(\frac{\mathbf{j}}{p}\right)\varepsilon_{n}\left(\mathbf{j}\mathbf{z}\right):\lambda_{p}\left(\left[\mathbf{z}\right]_{p^{n}}\right)\right)\left(H^{\prime}\left(\mathbf{0}\right)\right)^{n}\gamma_{H}\left(\frac{\mathbf{j}}{p}\right)\label{eq:MD Gamma formula} \end{equation} \vphantom{} II. As $N\rightarrow\infty$, \emph{(\ref{eq:Left Hand Side of MD Gamma Formulae (partial Fourier sum)})} is $\mathcal{F}_{p,q}^{d}$ convergent to: \begin{equation} \sum_{n=0}^{\infty}\kappa_{H}\left(\left[\mathbf{z}\right]_{p^{n}}\right)\sum_{\mathbf{j}>\mathbf{0}}^{p-1}\mathcal{C}_{H}\left(\alpha_{H}\left(\frac{\mathbf{j}}{p}\right)\varepsilon_{n}\left(\mathbf{j}\mathbf{z}\right):\lambda_{p}\left(\left[\mathbf{z}\right]_{p^{n}}\right)\right)\left(H^{\prime}\left(\mathbf{0}\right)\right)^{n}\gamma_{H}\left(\frac{\mathbf{j}}{p}\right)\label{eq:F limit of MD Gamma_H A_H hat Fourier series} \end{equation} since $H$ is semi-basic and contracting. \end{lem} Proof: Throughout this proof, we write $\mathbf{X}$ to denote $H^{\prime}\left(\mathbf{0}\right)$. \vphantom{} I. Like in the one-dimensional case, we first note that the map: \begin{equation} \mathbf{t}\in\hat{\mathbb{Z}}_{p}^{r}\mapsto\frac{\mathbf{t}\left|\mathbf{t}\right|_{p}}{p}=\left(\frac{t_{1}\left|t_{1}\right|_{p}}{p},\ldots,\frac{t_{r}\left|t_{r}\right|_{p}}{p}\right)\in\hat{\mathbb{Z}}_{p}^{r} \end{equation} takes tuples: \begin{equation} \mathbf{t}=\left(\frac{k_{1}}{p^{n_{1}}},\ldots,\frac{k_{r}}{p^{n_{r}}}\right) \end{equation} and outputs: \begin{equation} \frac{\mathbf{t}\left|\mathbf{t}\right|_{p}}{p}=\left(\frac{\left[k_{1}\right]_{p}}{p},\ldots,\frac{\left[k_{r}\right]_{p}}{p}\right) \end{equation} Now, for brevity, let: \begin{align} \gamma_{\mathbf{j}} & \overset{\textrm{def}}{=}\gamma_{H}\left(\frac{\mathbf{j}}{p}\right)=\gamma_{H}\left(\frac{j_{1}}{p},\ldots,\frac{j_{r}}{p}\right),\textrm{ }\forall\mathbf{j}\in\mathbb{Z}^{r}/p\mathbb{Z}^{r}\\ F_{N}\left(\mathbf{z}\right) & \overset{\textrm{def}}{=}\sum_{0<\left\Vert \mathbf{t}\right\Vert _{p}\leq p^{N}}\hat{A}_{H}\left(\mathbf{t}\right)\gamma_{H}\left(\frac{\mathbf{t}\left|\mathbf{t}\right|_{p}}{p}\right)e^{2\pi i\left\{ \mathbf{t}\mathbf{z}\right\} _{p}} \end{align} Finally, for each $\mathbf{k}\leq p^{n-1}-1$, each $\mathbf{j}\in\mathbb{Z}^{r}/p\mathbb{Z}^{r}$, and each $n\geq1$, we write: \begin{equation} \frac{p\mathbf{k}+\mathbf{j}}{p^{n}}=\left(\frac{pk_{1}+j_{1}}{p^{n}},\ldots,\frac{pk_{r}+j_{r}}{p^{n}}\right) \end{equation} \begin{claim} In the above notation, we have: \begin{equation} \left\{ \mathbf{t}\in\hat{\mathbb{Z}}_{p}^{r}:\left\Vert \mathbf{t}\right\Vert _{p}=p^{n}\right\} =\left\{ \frac{p\mathbf{k}+\mathbf{j}}{p^{n}}:\mathbf{k}\leq p^{n-1}-1,\textrm{ }\mathbf{j}\in\left(\mathbb{Z}^{r}/p\mathbb{Z}^{r}\right)\backslash\left\{ \mathbf{0}\right\} \right\} \label{eq:MD Decomposition of level sets} \end{equation} Proof of claim: \vphantom{} I. Let $\mathbf{t}=\frac{p\mathbf{k}+\mathbf{j}}{p^{n}}$. Then, for each $m\in\left\{ 1,\ldots,r\right\} $, $t_{m}=\frac{pk_{m}+j_{m}}{p^{n}}$ where $0\leq k_{m}\leq p^{n-1}-1$ and $0\leq j_{m}\leq p-1$; moreover, there exists an $\ell\in\left\{ 1,\ldots,r\right\} $ so that $j_{\ell}>0$. Since $pk_{\ell}+j_{\ell}$ is then co-prime to $p_{\ell}$, it follows that $\left|t_{\ell}\right|_{p}=p^{n}$. Since $\left|t_{m}\right|_{p}\leq p^{n}$ for all $m$, this shows that $\left\Vert \mathbf{t}\right\Vert _{p}=p^{n}$. \vphantom{} II. Let $\left\Vert \mathbf{t}\right\Vert _{p}=p^{n}$. Then, for every $m\in\left\{ 1,\ldots,r\right\} $, we can write $t_{m}=\frac{\nu_{m}}{p^{n}}$, where $\nu_{m}\in\left\{ 0,\ldots,p^{n}-1\right\} $. Moreover, there is an $\ell\in\left\{ 1,\ldots,r\right\} $ so that $\nu_{\ell}$ is co-prime to $p$, so as to guarantee that $\left|t_{\ell}\right|_{p}=p^{n}$ Each $\nu_{m}$ can be written as $pk_{m}-j_{m}$, where we have chosen $j_{m}=\left[\nu_{m}\right]_{p}\leq p-1$ and $k_{m}=\left(\nu_{m}-j_{m}\right)/p$. Since $\nu_{m}\leq p^{n}-1$, this forces $k_{m}\leq p^{n-1}-1$. Consequently, for these $j_{m}$s, the tuple $\mathbf{j}=\left(j_{1},\ldots,j_{r}\right)$ is then an element of $\mathbb{Z}^{r}/p\mathbb{Z}^{r}$, and $\mathbf{k}$ is a tuple $\leq p^{n-1}-1$ so that $\mathbf{t}=\left(p\mathbf{k}+\mathbf{j}\right)/p^{n}$. Finally, since $\nu_{\ell}$ is co-prime to $p$, $j_{\ell}\neq0$, which shows that $\mathbf{j}\in\left(\mathbb{Z}^{r}/p\mathbb{Z}^{r}\right)\backslash\left\{ \mathbf{0}\right\} $. \vphantom{} This proves the claim. \vphantom{} \end{claim} Consequently, we can express $F_{N}$ as a sum involving the $\gamma_{\mathbf{j}}$s like so: \begin{align*} F_{N}\left(\mathbf{z}\right) & =\sum_{n=1}^{N}\sum_{\left\Vert \mathbf{t}\right\Vert _{p}=p^{n}}\hat{A}_{H}\left(\mathbf{t}\right)\gamma_{H}\left(\frac{\mathbf{t}\left|\mathbf{t}\right|_{p}}{p}\right)e^{2\pi i\left\{ \mathbf{t}\mathbf{z}\right\} _{p}}\\ \left(\textrm{use }(\ref{eq:MD Decomposition of level sets})\right); & =\sum_{n=1}^{N}\sum_{\mathbf{j}>\mathbf{0}}^{p-1}\sum_{\mathbf{k}=\mathbf{0}}^{p^{n-1}-1}\hat{A}_{H}\left(\frac{p\mathbf{k}+\mathbf{j}}{p^{n}}\right)\gamma_{H}\left(\frac{\mathbf{j}}{p}\right)e^{2\pi i\left\{ \frac{p\mathbf{k}+\mathbf{j}}{p^{n}}\mathbf{z}\right\} _{p}} \end{align*} Using the formal identity: \begin{equation} \sum_{\mathbf{k}=\mathbf{0}}^{p^{n-1}-1}f\left(\frac{p\mathbf{k}+\mathbf{j}}{p^{n}}\right)=\sum_{\left\Vert \mathbf{t}\right\Vert _{p}\leq p^{n-1}}f\left(\mathbf{t}+\frac{\mathbf{j}}{p^{n}}\right) \end{equation} we can then write: \begin{align} F_{N}\left(\mathbf{z}\right) & =\sum_{n=1}^{N}\sum_{\left\Vert \mathbf{t}\right\Vert _{p}\leq p^{n-1}}\sum_{\mathbf{j}>\mathbf{0}}^{p-1}\hat{A}_{H}\left(\mathbf{t}+\frac{\mathbf{j}}{p^{n}}\right)\gamma_{\mathbf{j}}e^{2\pi i\left\{ \frac{\mathbf{j}\mathbf{z}}{p^{n}}\right\} _{p}}e^{2\pi i\left\{ \mathbf{t}\mathbf{z}\right\} _{p}}\label{eq:MD 1/3rd of the way through the gamma computation} \end{align} To deal with the $\mathbf{j}$-sum, we express $\hat{A}_{H}$ in product form, changing: \[ \sum_{\mathbf{j}>\mathbf{0}}^{p-1}\hat{A}_{H}\left(\mathbf{t}+\frac{\mathbf{j}}{p^{n}}\right)\gamma_{\mathbf{j}}e^{2\pi i\left\{ \frac{\mathbf{j}\mathbf{z}}{p^{n}}\right\} _{p}}e^{2\pi i\left\{ \mathbf{t}\mathbf{z}\right\} _{p}} \] into: \[ \sum_{\mathbf{j}>\mathbf{0}}^{p-1}\left(\prod_{\mathbf{m}=\mathbf{0}}^{n-1}\alpha_{H}\left(p^{m}\left(\mathbf{t}+\frac{\mathbf{j}}{p^{n}}\right)\right)\right)\gamma_{\mathbf{j}}e^{2\pi i\left\{ \frac{\mathbf{j}\mathbf{z}}{p^{n}}\right\} _{p}}e^{2\pi i\left\{ \mathbf{t}\mathbf{z}\right\} _{p}} \] Using \textbf{Proposition \ref{prop:MD alpha H series}} to write the $\alpha_{H}$-product out as a series gives: \begin{equation} \sum_{\mathbf{j}>\mathbf{0}}^{p-1}\left(\sum_{\mathbf{m}=\mathbf{0}}^{p^{n}-1}\kappa_{H}\left(\mathbf{m}\right)\left(\frac{\mathbf{X}}{p^{r}}\right)^{n}e^{-2\pi i\mathbf{m}\cdot\left(\mathbf{t}+\frac{\mathbf{j}}{p^{n}}\right)}\right)\gamma_{\mathbf{j}}e^{2\pi i\left\{ \frac{\mathbf{j}\mathbf{z}}{p^{n}}\right\} _{p}}e^{2\pi i\left\{ \mathbf{t}\mathbf{z}\right\} _{p}} \end{equation} and hence: \begin{equation} \sum_{\mathbf{j}>\mathbf{0}}^{p-1}\sum_{\mathbf{m}=\mathbf{0}}^{p^{n}-1}\kappa_{H}\left(\mathbf{m}\right)\left(\frac{\mathbf{X}}{p^{r}}\right)^{n}\gamma_{\mathbf{j}}e^{2\pi i\left\{ \frac{\mathbf{j}\left(\mathbf{z}-\mathbf{m}\right)}{p^{n}}\right\} _{p}}e^{2\pi i\left\{ \mathbf{t}\left(\mathbf{z}-\mathbf{m}\right)\right\} _{p}} \end{equation} Summing over $\left\Vert \mathbf{t}\right\Vert _{p}\leq p^{n-1}$ and using the Fourier series for $\left[\mathbf{z}\overset{p^{n-1}}{\equiv}\mathbf{m}\right]$: \begin{equation} \sum_{\left\Vert \mathbf{t}\right\Vert _{p}\leq p^{n-1}}e^{2\pi i\left\{ \mathbf{t}\left(\mathbf{z}-\mathbf{m}\right)\right\} _{p}}=p^{r\left(n-1\right)}\left[\mathbf{z}\overset{p^{n-1}}{\equiv}\mathbf{m}\right] \end{equation} we obtain: \begin{equation} \sum_{\mathbf{j}>\mathbf{0}}^{p-1}\sum_{\mathbf{m}=\mathbf{0}}^{p^{n}-1}\kappa_{H}\left(\mathbf{m}\right)\left(\frac{\mathbf{X}}{p^{r}}\right)^{n}\gamma_{\mathbf{j}}e^{2\pi i\left\{ \frac{\mathbf{j}\left(\mathbf{z}-\mathbf{m}\right)}{p^{n}}\right\} _{p}}p^{r\left(n-1\right)}\left[\mathbf{z}\overset{p^{n-1}}{\equiv}\mathbf{m}\right] \end{equation} In summary: \begin{eqnarray} & \sum_{\left\Vert \mathbf{t}\right\Vert _{p}\leq p^{n-1}}\sum_{\mathbf{j}>\mathbf{0}}^{p-1}\hat{A}_{H}\left(\mathbf{t}+\frac{\mathbf{j}}{p^{n}}\right)\gamma_{\mathbf{j}}e^{2\pi i\left\{ \frac{\mathbf{j}\mathbf{z}}{p^{n}}\right\} _{p}}e^{2\pi i\left\{ \mathbf{t}\mathbf{z}\right\} _{p}}\nonumber \\ & =\label{eq:MD 2/3rds of the way through the gamma computation}\\ & \frac{1}{p^{r}}\sum_{\mathbf{j}>\mathbf{0}}^{p-1}\left(\sum_{\mathbf{m}=\mathbf{0}}^{p^{n}-1}\kappa_{H}\left(\mathbf{m}\right)e^{2\pi i\left\{ \frac{\mathbf{j}\left(\mathbf{z}-\mathbf{m}\right)}{p^{n}}\right\} _{p}}\left[\mathbf{z}\overset{p^{n-1}}{\equiv}\mathbf{m}\right]\right)\mathbf{X}^{n}\gamma_{\mathbf{j}}\nonumber \end{eqnarray} Next, using the formal summation identity: \begin{equation} \sum_{\mathbf{m}=\mathbf{0}}^{p^{n}-1}f\left(\mathbf{m}\right)=\sum_{\mathbf{k}=\mathbf{0}}^{p-1}\sum_{\mathbf{m}=\mathbf{0}}^{p^{n-1}-1}f\left(\mathbf{m}+p^{n-1}\mathbf{k}\right)\label{eq:MD rho to the n formal identity} \end{equation} the expression: \begin{equation} \sum_{\mathbf{m}=\mathbf{0}}^{p^{n}-1}\kappa_{H}\left(\mathbf{m}\right)e^{2\pi i\left\{ \frac{\mathbf{j}\left(\mathbf{z}-\mathbf{m}\right)}{p^{n}}\right\} _{p}}\left[\mathbf{z}\overset{p^{n-1}}{\equiv}\mathbf{m}\right] \end{equation} becomes: \begin{equation} \sum_{\mathbf{k}=\mathbf{0}}^{p-1}\sum_{\mathbf{m}=\mathbf{0}}^{p^{n-1}-1}\kappa_{H}\left(\mathbf{m}+p^{n-1}\mathbf{k}\right)e^{-2\pi i\frac{\mathbf{j}\cdot\mathbf{k}}{p}}e^{2\pi i\left\{ \frac{\mathbf{j}\left(\mathbf{z}-\mathbf{m}\right)}{p^{n}}\right\} _{p}}\left[\mathbf{z}\overset{p^{n-1}}{\equiv}\mathbf{m}\right] \end{equation} Applying the functional equation identity for $\kappa_{H}$ (equation (\ref{eq:MD Kappa_H has P-adic structure}) from \textbf{Lemma \ref{lem:properties of MD kappa_H}}) yields: \begin{equation} \sum_{\mathbf{k}=\mathbf{0}}^{p-1}\sum_{\mathbf{m}=\mathbf{0}}^{p^{n-1}-1}\kappa_{H}\left(\mathbf{m}\right)\mathcal{C}_{H}\left(\kappa_{H}\left(\mathbf{k}\right):\lambda_{p}\left(\mathbf{m}\right)\right)e^{-2\pi i\frac{\mathbf{j}\cdot\mathbf{k}}{p}}e^{2\pi i\left\{ \frac{\mathbf{j}\left(\mathbf{z}-\mathbf{m}\right)}{p^{n}}\right\} _{p}}\left[\mathbf{z}\overset{p^{n-1}}{\equiv}\mathbf{m}\right] \end{equation} Here, note that $\left[\mathbf{z}\right]_{p^{n-1}}$ is the unique integer $r$-tuple $\mathbf{m}$ so that $\mathbf{m}\leq p^{n-1}-1$ and $\mathbf{z}\overset{p^{n-1}}{\equiv}\mathbf{m}$. This leaves us with: \begin{equation} \sum_{\mathbf{k}=\mathbf{0}}^{p-1}\kappa_{H}\left(\left[\mathbf{z}\right]_{p^{n-1}}\right)\mathcal{C}_{H}\left(\kappa_{H}\left(\mathbf{k}\right):\lambda_{p}\left(\left[\mathbf{z}\right]_{p^{n-1}}\right)\right)e^{-2\pi i\frac{\mathbf{j}\cdot\mathbf{k}}{p}}e^{2\pi i\left\{ \frac{\mathbf{j}\left(\mathbf{z}-\left[\mathbf{z}\right]_{p^{n-1}}\right)}{p^{n}}\right\} _{p}} \end{equation} which is: \begin{equation} \sum_{\mathbf{k}=\mathbf{0}}^{p-1}\kappa_{H}\left(\left[\mathbf{z}\right]_{p^{n-1}}\right)\mathcal{C}_{H}\left(\kappa_{H}\left(\mathbf{k}\right):\lambda_{p}\left(\left[\mathbf{z}\right]_{p^{n-1}}\right)\right)e^{-2\pi i\frac{\mathbf{j}\cdot\mathbf{k}}{p}}\varepsilon_{n-1}\left(\mathbf{j}\mathbf{z}\right)\label{eq:gamma proof - the above} \end{equation} Next, observe that for fixed $\left[\mathbf{z}\right]_{p^{n-1}}$, the map $\mathbf{A}\mapsto\mathcal{C}_{H}\left(\mathbf{A}:\lambda_{p}\left(\left[\mathbf{z}\right]_{p^{n-1}}\right)\right)$ is linear. So, (\ref{eq:gamma proof - the above}) can be written as: \begin{equation} \kappa_{H}\left(\left[\mathbf{z}\right]_{p^{n-1}}\right)\mathcal{C}_{H}\left(\sum_{\mathbf{k}=\mathbf{0}}^{p-1}\kappa_{H}\left(\mathbf{k}\right)e^{-2\pi i\frac{\mathbf{j}\cdot\mathbf{k}}{p}}:\lambda_{p}\left(\left[\mathbf{z}\right]_{p^{n-1}}\right)\right)\varepsilon_{n-1}\left(\mathbf{j}\mathbf{z}\right) \end{equation} With this, (\ref{eq:MD 2/3rds of the way through the gamma computation}) becomes: \begin{eqnarray} & \sum_{\left\Vert \mathbf{t}\right\Vert _{p}\leq p^{n-1}}\sum_{\mathbf{j}=\mathbf{0}}^{p-1}\hat{A}_{H}\left(\mathbf{t}+\frac{\mathbf{j}}{p^{n}}\right)\gamma_{\mathbf{j}}e^{2\pi i\left\{ \frac{\mathbf{j}\mathbf{z}}{p^{n}}\right\} _{p}}e^{2\pi i\left\{ \mathbf{t}\mathbf{z}\right\} _{p}}\nonumber \\ & =\label{eq:MD 2/3rds and 1/4th of the way through the gamma computation}\\ & \frac{1}{p^{r}}\sum_{\mathbf{j}>\mathbf{0}}^{p-1}\kappa_{H}\left(\left[\mathbf{z}\right]_{p^{n-1}}\right)\mathcal{C}_{H}\left(\sum_{\mathbf{k}=\mathbf{0}}^{p-1}\kappa_{H}\left(\mathbf{k}\right)e^{-2\pi i\frac{\mathbf{j}\cdot\mathbf{k}}{p}}:\lambda_{p}\left(\left[\mathbf{z}\right]_{p^{n-1}}\right)\right)\varepsilon_{n-1}\left(\mathbf{j}\mathbf{z}\right)\mathbf{X}^{n}\gamma_{\mathbf{j}}\nonumber \end{eqnarray} Our next step is to simplify the expression with $\mathcal{C}_{H}$. Here, we use $\kappa_{H}$'s functional equation ((\ref{eq:MD Kappa_H functional equations}) from \textbf{Lemma \ref{lem:properties of MD kappa_H}}), setting $\mathbf{j}=\mathbf{k}$ and $\mathbf{n}=\mathbf{0}$ in it. This gives us: \begin{align*} \sum_{\mathbf{k}=\mathbf{0}}^{p-1}\kappa_{H}\left(\mathbf{k}\right)e^{-2\pi i\frac{\mathbf{j}\cdot\mathbf{k}}{p}} & =\sum_{\mathbf{k}=\mathbf{0}}^{p-1}\mathbf{D}_{\mathbf{k}}^{-1}\mathbf{A}_{\mathbf{k}}\kappa_{H}\left(\mathbf{0}\right)\mathbf{A}_{\mathbf{0}}^{-1}\mathbf{D}_{\mathbf{0}}e^{-2\pi i\frac{\mathbf{j}\cdot\mathbf{k}}{p}}\\ \left(\kappa_{H}\left(\mathbf{0}\right)=\mathbf{I}_{d}\right); & =\left(\sum_{\mathbf{k}=\mathbf{0}}^{p-1}\mathbf{D}_{\mathbf{k}}^{-1}\mathbf{A}_{\mathbf{k}}e^{-2\pi i\frac{\mathbf{j}\cdot\mathbf{k}}{p}}\right)\mathbf{A}_{\mathbf{0}}^{-1}\mathbf{D}_{\mathbf{0}}\\ & =p^{r}\alpha_{H}\left(\frac{\mathbf{j}}{p}\right)\mathbf{A}_{\mathbf{0}}^{-1}\mathbf{D}_{\mathbf{0}}\\ & =p^{r}\alpha_{H}\left(\frac{\mathbf{j}}{p}\right)\mathbf{X}^{-1} \end{align*} Hence, (\ref{eq:MD 2/3rds and 1/4th of the way through the gamma computation}) becomes: \begin{eqnarray*} & \sum_{\left\Vert \mathbf{t}\right\Vert _{p}\leq p^{n-1}}\sum_{\mathbf{j}>\mathbf{0}}^{p-1}\hat{A}_{H}\left(\mathbf{t}+\frac{\mathbf{j}}{p^{n}}\right)\gamma_{\mathbf{j}}e^{2\pi i\left\{ \frac{\mathbf{j}\mathbf{z}}{p^{n}}\right\} _{p}}e^{2\pi i\left\{ \mathbf{t}\mathbf{z}\right\} _{p}}\\ & =\\ & \frac{1}{p^{r}}\sum_{\mathbf{j}>\mathbf{0}}^{p-1}\kappa_{H}\left(\left[\mathbf{z}\right]_{p^{n-1}}\right)\mathcal{C}_{H}\left(p\alpha_{H}\left(\frac{\mathbf{j}}{p}\right)\mathbf{X}^{-1}:\lambda_{p}\left(\left[\mathbf{z}\right]_{p^{n-1}}\right)\right)\underbrace{\varepsilon_{n-1}\left(\mathbf{j}\mathbf{z}\right)}_{\textrm{scalar}}\mathbf{X}^{n}\gamma_{\mathbf{j}}\\ & =\\ & \kappa_{H}\left(\left[\mathbf{z}\right]_{p^{n-1}}\right)\sum_{\mathbf{j}>\mathbf{0}}^{p-1}\varepsilon_{n-1}\left(\mathbf{j}\mathbf{z}\right)\mathcal{C}_{H}\left(\alpha_{H}\left(\frac{\mathbf{j}}{p}\right)\mathbf{X}^{-1}:\lambda_{p}\left(\left[\mathbf{z}\right]_{p^{n-1}}\right)\right)\mathbf{X}^{n}\gamma_{\mathbf{j}} \end{eqnarray*} Next, we note the following formal identity for matrices $\mathbf{A}$ and $\mathbf{X}$ and integers $m,n\geq0$: \begin{align*} \mathcal{C}_{H}\left(\mathbf{A}\mathbf{X}^{-1}:m\right)\mathbf{X}^{n} & =\left(\mathbf{X}^{m}\mathbf{A}\mathbf{X}^{-1}\mathbf{X}^{-m}\right)\mathbf{X}^{n}\\ & =\mathbf{X}^{m}\mathbf{A}\mathbf{X}^{-m+n-1}\\ & =\mathbf{X}^{-n+1}\mathbf{X}^{m-n+1}\mathbf{A}\mathbf{X}^{-\left(m-n+1\right)}\\ & =\mathbf{X}^{-n+1}\mathcal{C}_{H}\left(\mathbf{A}:m-n+1\right) \end{align*} As such: \begin{eqnarray*} & \sum_{\left\Vert \mathbf{t}\right\Vert _{p}\leq p^{n-1}}\sum_{\mathbf{j}>\mathbf{0}}^{p-1}\hat{A}_{H}\left(\mathbf{t}+\frac{\mathbf{j}}{p^{n}}\right)\gamma_{\mathbf{j}}e^{2\pi i\left\{ \frac{\mathbf{j}\cdot\mathbf{z}}{p^{n}}\right\} _{p}}e^{2\pi i\left\{ \mathbf{t}\cdot\mathbf{z}\right\} _{p}}\\ & =\\ & \kappa_{H}\left(\left[\mathbf{z}\right]_{p^{n-1}}\right)\sum_{\mathbf{j}>\mathbf{0}}^{p-1}\underbrace{\varepsilon_{n-1}\left(\mathbf{j}\mathbf{z}\right)}_{\textrm{a scalar}}\mathbf{X}^{-n+1}\mathcal{C}_{H}\left(\alpha_{H}\left(\frac{\mathbf{j}}{p}\right):\lambda_{p}\left(\left[\mathbf{z}\right]_{p^{n-1}}\right)-n+1\right)\gamma_{\mathbf{j}}\\ & =\\ & \kappa_{H}\left(\left[\mathbf{z}\right]_{p^{n-1}}\right)\mathbf{X}^{-n+1}\sum_{\mathbf{j}>\mathbf{0}}^{p-1}\mathcal{C}_{H}\left(\alpha_{H}\left(\frac{\mathbf{j}}{p}\right)\varepsilon_{n-1}\left(\mathbf{j}\mathbf{z}\right):\lambda_{p}\left(\left[\mathbf{z}\right]_{p^{n-1}}\right)-n+1\right)\gamma_{\mathbf{j}} \end{eqnarray*} Since: \begin{align*} \mathbf{X}^{-n+1}\mathcal{C}_{H}\left(\mathbf{A}:m-\left(n-1\right)\right) & =\mathbf{X}^{-n+1}\left(\mathbf{X}^{m-\left(n-1\right)}\mathbf{A}\mathbf{X}^{-m+n-1}\right)\\ & =\mathbf{X}^{-n+1}\mathbf{X}^{-\left(n-1\right)}\left(\mathbf{x}^{m}\mathbf{A}\mathbf{x}^{-m}\right)\mathbf{X}^{n-1}\\ & =\mathcal{C}_{H}\left(\mathbf{A}:m\right)\mathbf{X}^{n-1} \end{align*} we then have: \begin{eqnarray} & \sum_{\left\Vert \mathbf{t}\right\Vert _{p}\leq p^{n-1}}\sum_{\mathbf{j}>\mathbf{0}}^{p-1}\hat{A}_{H}\left(\mathbf{t}+\frac{\mathbf{j}}{p^{n}}\right)\gamma_{\mathbf{j}}e^{2\pi i\left\{ \frac{\mathbf{j}\mathbf{z}}{p^{n}}\right\} _{p}}e^{2\pi i\left\{ \mathbf{t}\mathbf{z}\right\} _{p}}\nonumber \\ & =\label{eq:MD 2/3rds and 1/4th and a bit of the way through the gamma computation}\\ & \kappa_{H}\left(\left[\mathbf{z}\right]_{p^{n-1}}\right)\sum_{\mathbf{j}>\mathbf{0}}^{p-1}\mathcal{C}_{H}\left(\alpha_{H}\left(\frac{\mathbf{j}}{p}\right)\varepsilon_{n-1}\left(\mathbf{j}\mathbf{z}\right):\lambda_{p}\left(\left[\mathbf{z}\right]_{p^{n-1}}\right)\right)\mathbf{X}^{n-1}\gamma_{\mathbf{j}}\nonumber \end{eqnarray} Finally, returning with this to (\ref{eq:MD 1/3rd of the way through the gamma computation}), we get: \begin{align} F_{N}\left(\mathbf{z}\right) & =\sum_{n=1}^{N}\kappa_{H}\left(\left[\mathbf{z}\right]_{p^{n-1}}\right)\sum_{\mathbf{j}>\mathbf{0}}^{p-1}\mathcal{C}_{H}\left(\alpha_{H}\left(\frac{\mathbf{j}}{p}\right)\varepsilon_{n-1}\left(\mathbf{j}\mathbf{z}\right):\lambda_{p}\left(\left[\mathbf{z}\right]_{p^{n-1}}\right)\right)\left(H^{\prime}\left(\mathbf{0}\right)\right)^{n-1}\gamma_{\mathbf{j}} \end{align} which gives (\ref{eq:MD Gamma formula}) after re-indexing $n$ by a shift of $1$. \vphantom{} II. Taking the $n$th term from (\ref{eq:MD Gamma formula}), we first apply $q$-adic norm to get and upper bound of: \begin{equation} \left\Vert \kappa_{H}\left(\left[\mathbf{z}\right]_{p^{n}}\right)\right\Vert _{q}\left\Vert H^{\prime}\left(\mathbf{0}\right)\right\Vert _{q}^{n}\max_{\mathbf{0}<\mathbf{j}\leq p-1}\left\Vert \beta_{H}\left(\frac{\mathbf{j}}{p}\right)\right\Vert _{q} \end{equation} In this, we used: \begin{equation} \gamma_{\mathbf{j}}=\gamma_{H}\left(\frac{\mathbf{j}}{p}\right)=\left(\alpha_{H}\left(\frac{\mathbf{j}}{p}\right)\right)^{-1}\beta_{H}\left(\frac{\mathbf{j}}{p}\right) \end{equation} Since the $q$-adic bound on the $\beta_{H}$s is independent of $n$, and since $\left\Vert H^{\prime}\left(\mathbf{0}\right)\right\Vert _{q}=1$, the fact that $\left\Vert \kappa_{H}\left(\left[\mathbf{z}\right]_{p^{n}}\right)\right\Vert _{q}$ tends to $0$ as $n\rightarrow\infty$ for all $\mathbf{z}\in\left(\mathbb{Z}_{p}^{r}\right)^{\prime}$ then guarantees the convergence of (\ref{eq:F limit of MD Gamma_H A_H hat Fourier series}) for $\mathbf{z}\in\left(\mathbb{Z}_{p}^{r}\right)^{\prime}$. For $\mathbf{z}\in\mathbb{N}_{0}^{r}$, applying the archimedean complex norm to the $n$th term of (\ref{eq:MD Gamma formula}) gives an upper bound of: \begin{equation} \sum_{n=0}^{N-1}\left\Vert \kappa_{H}\left(\left[\mathbf{z}\right]_{p^{n}}\right)\right\Vert _{\infty}\left\Vert H^{\prime}\left(\mathbf{0}\right)\right\Vert _{\infty}^{n}\sum_{\mathbf{j}>\mathbf{0}}^{p-1}\left\Vert \beta_{H}\left(\frac{\mathbf{j}}{p}\right)\right\Vert _{\infty} \end{equation} Here, $\sum_{\mathbf{j}>\mathbf{0}}^{p-1}\left\Vert \beta_{H}\left(\frac{\mathbf{j}}{p}\right)\right\Vert _{\infty}$ is uniformly bounded with respect to $n$. Moreover, since $\mathbf{z}\in\mathbb{N}_{0}^{r}$, we have that $\kappa_{H}\left(\left[\mathbf{z}\right]_{p^{n}}\right)=\kappa_{H}\left(\mathbf{z}\right)$ for all sufficiently large $n$. So, if we let $N\rightarrow\infty$, we obtain: \begin{equation} \sum_{n=0}^{N-1}\left\Vert \kappa_{H}\left(\left[\mathbf{z}\right]_{p^{n}}\right)\right\Vert _{\infty}\left\Vert H^{\prime}\left(\mathbf{0}\right)\right\Vert _{\infty}^{n}\sum_{\mathbf{j}>\mathbf{0}}^{p-1}\left\Vert \beta_{H}\left(\frac{\mathbf{j}}{p}\right)\right\Vert _{\infty}\ll O\left(1\right)+\sum_{n}\left\Vert H^{\prime}\left(\mathbf{0}\right)\right\Vert _{\infty}^{n} \end{equation} Here, the upper bound is a convergent geometric series because $H$ is contracting. This then guarantees the convergence of (\ref{eq:F limit of MD Gamma_H A_H hat Fourier series}) for $\mathbf{z}\in\mathbb{N}_{0}^{r}$. Q.E.D. \vphantom{} With these formulae, we can now sum the Fourier series generated by (\ref{eq:MD Fine structure of Chi_H,N hat when alpha is 1}) to obtain a non-trivial formula for $\chi_{H,N}$. \begin{thm} Suppose $\alpha_{H}\left(\mathbf{0}\right)=\mathbf{I}_{d}$. Then, for all $N\geq1$ and all $\mathbf{z}\in\mathbb{Z}_{p}^{r}$: \begin{align} \chi_{H,N}\left(\mathbf{z}\right) & \overset{\overline{\mathbb{Q}}^{d}}{=}\sum_{n=0}^{N-1}\kappa_{H}\left(\left[\mathbf{z}\right]_{p^{n}}\right)\sum_{\mathbf{j}=\mathbf{0}}^{p-1}\mathcal{C}_{H}\left(\alpha_{H}\left(\frac{\mathbf{j}}{p}\right)\varepsilon_{n}\left(\mathbf{j}\mathbf{z}\right):\lambda_{p}\left(\left[\mathbf{z}\right]_{p^{n}}\right)\right)\left(H^{\prime}\left(\mathbf{0}\right)\right)^{n}\gamma_{H}\left(\frac{\mathbf{j}}{p}\right)\label{eq:MD Chi_H,N when alpha is 1 and rho is arbitrary} \end{align} In particular, when $p=2$: \begin{equation} \chi_{H,N}\left(\mathbf{z}\right)=-\gamma_{H}\left(\mathbf{\frac{1}{2}}\right)+\kappa_{H}\left(\left[\mathbf{z}\right]_{2^{N}}\right)\left(H^{\prime}\left(\mathbf{0}\right)\right)^{N}\gamma_{H}\left(\mathbf{\frac{1}{2}}\right)+\sum_{n=0}^{N-1}\kappa_{H}\left(\left[\mathbf{z}\right]_{2^{n}}\right)\left(H^{\prime}\left(\mathbf{0}\right)\right)^{n}\beta_{H}\left(\mathbf{0}\right)\label{eq:MD Chi_H,N when alpha is 1 and every prime in P is 2} \end{equation} where: \begin{equation} \gamma_{H}\left(\mathbf{\frac{1}{2}}\right)\overset{\textrm{def}}{=}\gamma_{H}\left(\left(\frac{1}{2},\ldots,\frac{1}{2}\right)\right)\label{eq:Definition of MD gamma_H of 1/2} \end{equation} \end{thm} Proof: We start by multiplying (\ref{eq:MD Fine structure of Chi_H,N hat when alpha is 1}) by $e^{2\pi i\left\{ \mathbf{t}\mathbf{z}\right\} _{p}}$ and then summing over all $\left\Vert \mathbf{t}\right\Vert _{p}\leq p^{N}$. The left-hand side of (\ref{eq:MD Fine structure of Chi_H,N hat when alpha is 1}) becomes: \begin{equation} \chi_{H,N}\left(\mathbf{z}\right)-N\tilde{A}_{H,N-1}\left(\mathbf{z}\right)\beta_{H}\left(\mathbf{0}\right) \end{equation} while the right-hand side becomes: \begin{align*} & \sum_{0<\left\Vert \mathbf{t}\right\Vert _{p}\leq p^{N-1}}\hat{A}_{H}\left(\mathbf{t}\right)\left(v_{p}\left(\mathbf{t}\right)\beta_{H}\left(\mathbf{0}\right)+\gamma_{H}\left(\frac{\mathbf{t}\left|\mathbf{t}\right|_{p}}{p}\right)\right)e^{2\pi i\left\{ \mathbf{t}\mathbf{z}\right\} _{p}}\\ & +\sum_{\left\Vert \mathbf{t}\right\Vert _{p}=p^{N}}\hat{A}_{H}\left(\mathbf{t}\right)\gamma_{H}\left(\frac{\mathbf{t}\left|\mathbf{t}\right|_{p}}{p}\right)e^{2\pi i\left\{ \mathbf{t}\mathbf{z}\right\} _{p}} \end{align*} Simplifying produces: \begin{align} \chi_{H,N}\left(\mathbf{z}\right) & \overset{\overline{\mathbb{Q}}^{d}}{=}N\tilde{A}_{H,N-1}\left(\mathbf{z}\right)\beta_{H}\left(\mathbf{0}\right)\label{eq:MD Chi_H,N rho not equal to 2, ready to simplify}\\ & +\sum_{0<\left\Vert \mathbf{t}\right\Vert _{p}\leq p^{N-1}}\hat{A}_{H}\left(\mathbf{t}\right)v_{p}\left(\mathbf{t}\right)\beta_{H}\left(\mathbf{0}\right)e^{2\pi i\left\{ \mathbf{t}\mathbf{z}\right\} _{p}}\nonumber \\ & +\sum_{0<\left\Vert \mathbf{t}\right\Vert _{p}\leq p^{N}}\hat{A}_{H}\left(\mathbf{t}\right)\gamma_{H}\left(\frac{\mathbf{t}\left|\mathbf{t}\right|_{p}}{p}\right)e^{2\pi i\left\{ \mathbf{t}\mathbf{z}\right\} _{p}}\nonumber \end{align} Once again, we call upon our legion of formulae: (\ref{eq:MD Convolution of dA_H and D_N}), \textbf{Lemmata \ref{lem:MD v_p A_H hat summation formulae}}, and \textbf{\ref{lem:MD gamma formulae}}. Using them (while remembering that $\alpha_{H}\left(\mathbf{0}\right)=\mathbf{I}_{d}$ makes $\mathbf{I}_{H}$ identically $\mathbf{O}_{d}$) turns (\ref{eq:MD Chi_H,N rho not equal to 2, ready to simplify}) into: \begin{align*} \chi_{H,N}\left(\mathbf{z}\right) & \overset{\overline{\mathbb{Q}}^{d}}{=}N\kappa_{H}\left(\left[\mathbf{z}\right]_{p^{N-1}}\right)\left(H^{\prime}\left(\mathbf{0}\right)\right)^{N-1}\beta_{H}\left(\mathbf{0}\right)\\ & -\left(N-1\right)\kappa_{H}\left(\left[\mathbf{z}\right]_{p^{N-1}}\right)\left(H^{\prime}\left(\mathbf{0}\right)\right)^{N-1}\beta_{H}\left(\mathbf{0}\right)\\ & +\sum_{n=0}^{N-2}\kappa_{H}\left(\left[\mathbf{z}\right]_{p^{n}}\right)\left(H^{\prime}\left(\mathbf{0}\right)\right)^{n}\beta_{H}\left(\mathbf{0}\right)\\ & +\sum_{n=0}^{N-1}\kappa_{H}\left(\left[\mathbf{z}\right]_{p^{n}}\right)\sum_{\mathbf{j}>\mathbf{0}}^{p-1}\mathcal{C}_{H}\left(\alpha_{H}\left(\frac{\mathbf{j}}{p}\right)\varepsilon_{n}\left(\mathbf{j}\mathbf{z}\right):\lambda_{p}\left(\left[\mathbf{z}\right]_{p^{n}}\right)\right)\left(H^{\prime}\left(\mathbf{0}\right)\right)^{n}\gamma_{H}\left(\frac{\mathbf{j}}{p}\right) \end{align*} Simplifying yields: \begin{align*} \chi_{H,N}\left(\mathbf{z}\right) & \overset{\overline{\mathbb{Q}}^{d}}{=}\sum_{n=0}^{N-1}\kappa_{H}\left(\left[\mathbf{z}\right]_{p^{n}}\right)\left(H^{\prime}\left(\mathbf{0}\right)\right)^{n}\beta_{H}\left(\mathbf{0}\right)\\ & +\sum_{n=0}^{N-1}\kappa_{H}\left(\left[\mathbf{z}\right]_{p^{n}}\right)\sum_{\mathbf{j}>\mathbf{0}}^{p-1}\mathcal{C}_{H}\left(\alpha_{H}\left(\frac{\mathbf{j}}{p}\right)\varepsilon_{n}\left(\mathbf{j}\mathbf{z}\right):\lambda_{p}\left(\left[\mathbf{z}\right]_{p^{n}}\right)\right)\left(H^{\prime}\left(\mathbf{0}\right)\right)^{n}\gamma_{H}\left(\frac{\mathbf{j}}{p}\right) \end{align*} Now, because $\alpha_{H}\left(\mathbf{0}\right)=\mathbf{I}_{d}$, the vector: \begin{equation} \mathcal{C}_{H}\left(\alpha_{H}\left(\frac{\mathbf{0}}{p}\right)\varepsilon_{n}\left(\mathbf{0}\mathbf{z}\right):\lambda_{p}\left(\left[\mathbf{z}\right]_{p^{n}}\right)\right)\left(H^{\prime}\left(\mathbf{0}\right)\right)^{n}\gamma_{H}\left(\frac{\mathbf{0}}{p}\right) \end{equation} becomes: \begin{equation} \underbrace{\mathcal{C}_{H}\left(\mathbf{I}_{d}:\lambda_{p}\left(\left[\mathbf{z}\right]_{p^{n}}\right)\right)}_{\mathbf{I}_{d}}\left(H^{\prime}\left(\mathbf{0}\right)\right)^{n}\beta_{H}\left(\mathbf{0}\right)=\left(H^{\prime}\left(\mathbf{0}\right)\right)^{n}\beta_{H}\left(\mathbf{0}\right) \end{equation} So, the two lines on the right-hand side of the above formula for $\chi_{H,N}\left(\mathbf{z}\right)$ combine to form a single sum because the upper line is the $\mathbf{j}=\mathbf{0}$ case of the bottom line. Finally, when $p=2$, we can compute everything directly from (\ref{eq:MD Fine structure of Chi_H,N hat when alpha is 1}) by multiplying by $e^{2\pi i\left\{ \mathbf{t}\mathbf{z}\right\} _{2}}$ and summing over all $\left\Vert \mathbf{t}\right\Vert _{2}\leq2^{N}$. This gives: \begin{align*} \chi_{H,N}\left(\mathbf{z}\right)-N\tilde{A}_{H,N-1}\left(\mathbf{z}\right)\beta_{H}\left(\mathbf{0}\right)= & \sum_{0<\left\Vert \mathbf{t}\right\Vert _{2}\leq2^{N-1}}\hat{A}_{H}\left(\mathbf{t}\right)v_{2}\left(\mathbf{t}\right)e^{2\pi i\left\{ \mathbf{t}\mathbf{z}\right\} _{2}}\beta_{H}\left(\mathbf{0}\right)\\ & +\sum_{0<\left\Vert \mathbf{t}\right\Vert _{2}\leq2^{N}}\hat{A}_{H}\left(\mathbf{t}\right)e^{2\pi i\left\{ \mathbf{t}\mathbf{z}\right\} _{2}}\gamma_{H}\left(\mathbf{\frac{1}{2}}\right) \end{align*} which simplifies to: \begin{align*} \chi_{H,N}\left(\mathbf{z}\right)= & N\tilde{A}_{H,N-1}\left(\mathbf{z}\right)\beta_{H}\left(\mathbf{0}\right)-\gamma_{H}\left(\mathbf{\frac{1}{2}}\right)+\sum_{\left\Vert \mathbf{t}\right\Vert _{2}\leq2^{N}}\hat{A}_{H}\left(\mathbf{t}\right)e^{2\pi i\left\{ \mathbf{t}\mathbf{z}\right\} _{2}}\gamma_{H}\left(\mathbf{\frac{1}{2}}\right)\\ & +\sum_{0<\left\Vert \mathbf{t}\right\Vert _{2}\leq2^{N-1}}\hat{A}_{H}\left(\mathbf{t}\right)v_{2}\left(\mathbf{t}\right)e^{2\pi i\left\{ \mathbf{t}\mathbf{z}\right\} _{2}}\beta_{H}\left(\mathbf{0}\right) \end{align*} Applying (\ref{eq:MD Convolution of dA_H and D_N}) and \textbf{Lemma \ref{lem:MD v_p A_H hat summation formulae}}\textemdash and, again remembering that $\alpha_{H}\left(\mathbf{0}\right)=\mathbf{I}_{d}$ makes $\mathbf{I}_{H}$ identically $\mathbf{O}_{d}$\textemdash the above becomes: \begin{align*} \chi_{H,N}\left(\mathbf{z}\right)= & N\kappa_{H}\left(\left[\mathbf{z}\right]_{2^{N-1}}\right)\left(H^{\prime}\left(\mathbf{0}\right)\right)^{N-1}\beta_{H}\left(\mathbf{0}\right)-\gamma_{H}\left(\mathbf{\frac{1}{2}}\right)\\ & +\kappa_{H}\left(\left[\mathbf{z}\right]_{2^{N}}\right)\left(H^{\prime}\left(\mathbf{0}\right)\right)^{N}\gamma_{H}\left(\mathbf{\frac{1}{2}}\right)\\ & -\left(N-1\right)\kappa_{H}\left(\left[\mathbf{z}\right]_{2^{N-1}}\right)\left(H^{\prime}\left(\mathbf{0}\right)\right)^{N-1}\beta_{H}\left(\mathbf{0}\right)\\ & +\sum_{n=0}^{N-2}\kappa_{H}\left(\left[\mathbf{z}\right]_{2^{n}}\right)\left(H^{\prime}\left(\mathbf{0}\right)\right)^{n}\beta_{H}\left(\mathbf{0}\right) \end{align*} Simplifying then gives us what we wanted: \[ \chi_{H,N}\left(\mathbf{z}\right)=-\gamma_{H}\left(\mathbf{\frac{1}{2}}\right)+\kappa_{H}\left(\left[\mathbf{z}\right]_{2^{N}}\right)\left(H^{\prime}\left(\mathbf{0}\right)\right)^{N}\gamma_{H}\left(\mathbf{\frac{1}{2}}\right)+\sum_{n=0}^{N-1}\kappa_{H}\left(\left[\mathbf{z}\right]_{2^{n}}\right)\left(H^{\prime}\left(\mathbf{0}\right)\right)^{n}\beta_{H}\left(\mathbf{0}\right) \] Q.E.D. \begin{cor}[\textbf{$\mathcal{F}$-Series for $\chi_{H}$ when $\alpha_{H}\left(\mathbf{0}\right)=\mathbf{I}_{d}$}] \label{cor:MD alpha is 1 case, F-series}Suppose $\alpha_{H}\left(\mathbf{0}\right)=\mathbf{I}_{d}$. Then: \begin{equation} \chi_{H}\left(\mathbf{z}\right)\overset{\mathcal{F}_{p,q_{H}}^{d}}{=}\sum_{n=0}^{\infty}\kappa_{H}\left(\left[\mathbf{z}\right]_{p^{n}}\right)\sum_{\mathbf{j}=\mathbf{0}}^{p-1}\mathcal{C}_{H}\left(\alpha_{H}\left(\frac{\mathbf{j}}{p}\right)\varepsilon_{n}\left(\mathbf{j}\mathbf{z}\right):\lambda_{p}\left(\left[\mathbf{z}\right]_{p^{n}}\right)\right)\left(H^{\prime}\left(\mathbf{0}\right)\right)^{n}\gamma_{H}\left(\frac{\mathbf{j}}{p}\right)\label{eq:MD Explicit Formula for Chi_H when alpha is 1 and rho is arbitrary} \end{equation} for all $\mathbf{z}\in\mathbb{Z}_{p}^{r}$, with the special case: \begin{equation} \chi_{H}\left(\mathbf{z}\right)\overset{\mathcal{F}_{2,q_{H}}^{d}}{=}-\gamma_{H}\left(\mathbf{\frac{1}{2}}\right)+\sum_{n=0}^{\infty}\kappa_{H}\left(\left[\mathbf{z}\right]_{2^{n}}\right)\left(H^{\prime}\left(\mathbf{0}\right)\right)^{n}\beta_{H}\left(\mathbf{0}\right)\label{eq:MD Explicit Formula for Chi_H when alpha is 1 and every prime in P is 2} \end{equation} for all $\mathbf{z}\in\mathbb{Z}_{2}$ when $p=2$. \end{cor} Proof: Same as in the one-dimensional case, but with vector norms. Q.E.D. \vphantom{} Next, we sum in $\mathbb{C}$ for $\mathbf{z}\in\mathbb{N}_{0}^{r}$. \begin{cor} \label{cor:MD alpha is 1, F-series on N_0^r}Suppose that $\alpha_{H}\left(\mathbf{0}\right)=\mathbf{I}_{d}$. Then: \begin{align} \chi_{H}\left(\mathbf{n}\right) & \overset{\mathbb{C}^{d}}{=}\sum_{k=0}^{\lambda_{p}\left(\mathbf{n}\right)-1}\kappa_{H}\left(\left[\mathbf{n}\right]_{p^{k}}\right)\sum_{\mathbf{j}=\mathbf{0}}^{p-1}\mathcal{C}_{H}\left(\alpha_{H}\left(\frac{\mathbf{j}}{p}\right)\varepsilon_{k}\left(\mathbf{j}\mathbf{n}\right):\lambda_{p}\left(\left[\mathbf{n}\right]_{p^{k}}\right)\right)\left(H^{\prime}\left(\mathbf{0}\right)\right)^{k}\gamma_{H}\left(\frac{\mathbf{j}}{p}\right)\label{eq:MD archimedean Chi_H when rho is arbitrary and alpha_H of 0 is 1}\\ & +M_{H}\left(\mathbf{n}\right)\sum_{\mathbf{j}=\mathbf{0}}^{p-1}\alpha_{H}\left(\frac{\mathbf{j}}{p}\right)\left(\mathbf{I}_{d}-H^{\prime}\left(\mathbf{0}\right)\right)^{-1}\gamma_{H}\left(\frac{\mathbf{j}}{p}\right) \end{align} for all $\mathbf{n}\in\mathbb{N}_{0}^{r}$, with the special case: \begin{align} \chi_{H}\left(\mathbf{n}\right) & \overset{\mathbb{C}^{d}}{=}-\gamma_{H}\left(\mathbf{\frac{1}{2}}\right)+M_{H}\left(\mathbf{n}\right)\left(\mathbf{I}_{d}-H^{\prime}\left(\mathbf{0}\right)\right)^{-1}\beta_{H}\left(\mathbf{0}\right)\label{eq:MD archimedean Chi_H when every prime in P is 2 and alpha_H of 0 is 1}\\ & +\sum_{k=0}^{\lambda_{2}\left(\mathbf{n}\right)-1}\kappa_{H}\left(\left[\mathbf{n}\right]_{2^{k}}\right)\left(H^{\prime}\left(\mathbf{0}\right)\right)^{k}\beta_{H}\left(\mathbf{0}\right)\chi_{H}\left(\mathbf{n}\right) \end{align} when $p=2$. Regardless of the value of $p$, the $k$-sums are defined to be $\mathbf{0}$ when $\mathbf{n}=\mathbf{0}$. \end{cor} Proof: Let $\mathbf{n}\in\mathbb{N}_{0}^{r}$. Since $\left[\mathbf{n}\right]_{p^{k}}=\mathbf{n}$ and $\varepsilon_{k}\left(\mathbf{n}\right)=1$ for all $k\geq\lambda_{p}\left(\mathbf{n}\right)$, (\ref{eq:MD Explicit Formula for Chi_H when alpha is 1 and rho is arbitrary}) becomes: \begin{align*} \chi_{H}\left(\mathbf{n}\right) & \overset{\mathbb{C}^{d}}{=}\sum_{k=0}^{\lambda_{p}\left(\mathbf{n}\right)-1}\kappa_{H}\left(\left[\mathbf{n}\right]_{p^{k}}\right)\sum_{\mathbf{j}=\mathbf{0}}^{p-1}\mathcal{C}_{H}\left(\alpha_{H}\left(\frac{\mathbf{j}}{p}\right)\varepsilon_{k}\left(\mathbf{j}\mathbf{n}\right):\lambda_{p}\left(\left[\mathbf{n}\right]_{p^{k}}\right)\right)\left(H^{\prime}\left(\mathbf{0}\right)\right)^{k}\gamma_{H}\left(\frac{\mathbf{j}}{p}\right)\\ & +\kappa_{H}\left(\mathbf{n}\right)\sum_{k=\lambda_{p}\left(\mathbf{n}\right)}^{\infty}\sum_{\mathbf{j}=\mathbf{0}}^{p-1}\mathcal{C}_{H}\left(\alpha_{H}\left(\frac{\mathbf{j}}{p}\right):\lambda_{p}\left(\mathbf{n}\right)\right)\left(H^{\prime}\left(\mathbf{0}\right)\right)^{k}\gamma_{H}\left(\frac{\mathbf{j}}{p}\right) \end{align*} Here: \[ \sum_{k=\lambda_{p}\left(\mathbf{n}\right)}^{\infty}\sum_{\mathbf{j}=\mathbf{0}}^{p-1}\mathcal{C}_{H}\left(\alpha_{H}\left(\frac{\mathbf{j}}{p}\right):\lambda_{p}\left(\mathbf{n}\right)\right)\left(H^{\prime}\left(\mathbf{0}\right)\right)^{k}\gamma_{H}\left(\frac{\mathbf{j}}{p}\right) \] becomes: \[ \sum_{k=\lambda_{p}\left(\mathbf{n}\right)}^{\infty}\sum_{\mathbf{j}=\mathbf{0}}^{p-1}\left(H^{\prime}\left(\mathbf{0}\right)\right)^{\lambda_{p}\left(\mathbf{n}\right)}\alpha_{H}\left(\frac{\mathbf{j}}{p}\right)\left(H^{\prime}\left(\mathbf{0}\right)\right)^{k-\lambda_{p}\left(\mathbf{n}\right)}\gamma_{H}\left(\frac{\mathbf{j}}{p}\right) \] which simplifies to : \[ \sum_{\mathbf{j}=\mathbf{0}}^{p-1}\left(H^{\prime}\left(\mathbf{0}\right)\right)^{\lambda_{p}\left(\mathbf{n}\right)}\alpha_{H}\left(\frac{\mathbf{j}}{p}\right)\sum_{k=0}^{\infty}\left(H^{\prime}\left(\mathbf{0}\right)\right)^{k}\gamma_{H}\left(\frac{\mathbf{j}}{p}\right) \] Since $H$ is contracting, we get a $\mathbb{C}$-convergent geometric series: \[ \sum_{k=0}^{\infty}\left(H^{\prime}\left(\mathbf{0}\right)\right)^{k}\overset{\mathbb{C}}{=}\left(\mathbf{I}_{d}-H^{\prime}\left(\mathbf{0}\right)\right)^{-1} \] and so, we are left with: \[ \sum_{\mathbf{j}=\mathbf{0}}^{p-1}\left(H^{\prime}\left(\mathbf{0}\right)\right)^{\lambda_{p}\left(\mathbf{n}\right)}\alpha_{H}\left(\frac{\mathbf{j}}{p}\right)\left(\mathbf{I}_{d}-H^{\prime}\left(\mathbf{0}\right)\right)^{-1}\gamma_{H}\left(\frac{\mathbf{j}}{p}\right) \] Consequently: \begin{align*} \chi_{H}\left(\mathbf{n}\right) & \overset{\mathbb{C}^{d}}{=}\sum_{k=0}^{\lambda_{p}\left(\mathbf{n}\right)-1}\kappa_{H}\left(\left[\mathbf{n}\right]_{p^{k}}\right)\sum_{\mathbf{j}=\mathbf{0}}^{p-1}\mathcal{C}_{H}\left(\alpha_{H}\left(\frac{\mathbf{j}}{p}\right)\varepsilon_{k}\left(\mathbf{j}\mathbf{n}\right):\lambda_{p}\left(\left[\mathbf{n}\right]_{p^{k}}\right)\right)\left(H^{\prime}\left(\mathbf{0}\right)\right)^{k}\gamma_{H}\left(\frac{\mathbf{j}}{p}\right)\\ & +\kappa_{H}\left(\mathbf{n}\right)\left(H^{\prime}\left(\mathbf{0}\right)\right)^{\lambda_{p}\left(\mathbf{n}\right)}\sum_{\mathbf{j}=\mathbf{0}}^{p-1}\alpha_{H}\left(\frac{\mathbf{j}}{p}\right)\left(\mathbf{I}_{d}-H^{\prime}\left(\mathbf{0}\right)\right)^{-1}\gamma_{H}\left(\frac{\mathbf{j}}{p}\right) \end{align*} Since: \[ \kappa_{H}\left(\mathbf{n}\right)=M_{H}\left(\mathbf{n}\right)\left(H^{\prime}\left(\mathbf{0}\right)\right)^{-\lambda_{p}\left(\mathbf{n}\right)} \] the above can be simplified to: \begin{align*} \chi_{H}\left(\mathbf{n}\right) & \overset{\mathbb{C}^{d}}{=}\sum_{k=0}^{\lambda_{p}\left(\mathbf{n}\right)-1}\kappa_{H}\left(\left[\mathbf{n}\right]_{p^{k}}\right)\sum_{\mathbf{j}=\mathbf{0}}^{p-1}\mathcal{C}_{H}\left(\alpha_{H}\left(\frac{\mathbf{j}}{p}\right)\varepsilon_{k}\left(\mathbf{j}\mathbf{n}\right):\lambda_{p}\left(\left[\mathbf{n}\right]_{p^{k}}\right)\right)\left(H^{\prime}\left(\mathbf{0}\right)\right)^{k}\gamma_{H}\left(\frac{\mathbf{j}}{p}\right)\\ & +M_{H}\left(\mathbf{n}\right)\sum_{\mathbf{j}=\mathbf{0}}^{p-1}\alpha_{H}\left(\frac{\mathbf{j}}{p}\right)\left(\mathbf{I}_{d}-H^{\prime}\left(\mathbf{0}\right)\right)^{-1}\gamma_{H}\left(\frac{\mathbf{j}}{p}\right) \end{align*} which is the desired formula. As for the case where $p=2$, applying the above argument to (\ref{eq:MD Explicit Formula for Chi_H when alpha is 1 and every prime in P is 2}) given produces: \begin{align*} \chi_{H}\left(\mathbf{n}\right) & \overset{\mathbb{C}^{d}}{=}-\gamma_{H}\left(\mathbf{\frac{1}{2}}\right)+\sum_{k=0}^{\lambda_{2}\left(\mathbf{n}\right)-1}\kappa_{H}\left(\left[\mathbf{n}\right]_{2^{k}}\right)\left(H^{\prime}\left(\mathbf{0}\right)\right)^{k}\beta_{H}\left(\mathbf{0}\right)\\ & +\sum_{k=\lambda_{2}\left(\mathbf{n}\right)}^{\infty}\kappa_{H}\left(\mathbf{n}\right)\left(H^{\prime}\left(\mathbf{0}\right)\right)^{k}\beta_{H}\left(\mathbf{0}\right)\\ \left(H\textrm{ is contracting}\right); & \overset{\mathbb{C}^{d}}{=}-\gamma_{H}\left(\mathbf{\frac{1}{2}}\right)+\sum_{k=0}^{\lambda_{2}\left(\mathbf{n}\right)-1}\kappa_{H}\left(\left[\mathbf{n}\right]_{2^{k}}\right)\left(H^{\prime}\left(\mathbf{0}\right)\right)^{k}\beta_{H}\left(\mathbf{0}\right)\\ & +\underbrace{\kappa_{H}\left(\mathbf{n}\right)\left(H^{\prime}\left(\mathbf{0}\right)\right)^{\lambda_{2}\left(\mathbf{n}\right)}}_{M_{H}\left(\mathbf{n}\right)}\left(\mathbf{I}_{d}-H^{\prime}\left(\mathbf{0}\right)\right)^{-1}\beta_{H}\left(\mathbf{0}\right) \end{align*} Q.E.D. \vphantom{} Taken together, these two corollaries establish the quasi-integrability of $\chi_{H}$ for arbitrary $p$, provided that $\alpha_{H}\left(\mathbf{0}\right)=\mathbf{I}_{d}$: \begin{cor}[\textbf{Quasi-Integrability of $\chi_{H}$ When $\alpha_{H}\left(\mathbf{0}\right)=\mathbf{I}_{d}$}] If $\alpha_{H}\left(\mathbf{0}\right)=\mathbf{I}_{d}$, then, $\chi_{H}$ is quasi-integrable with respect to the standard $\left(p,q_{H}\right)$-adic frame. In particular, when $p=2$, the function $\hat{\chi}_{H}:\hat{\mathbb{Z}}_{2}^{r}\rightarrow\overline{\mathbb{Q}}^{d}$ defined by: \begin{equation} \hat{\chi}_{H}\left(\mathbf{t}\right)\overset{\textrm{def}}{=}\begin{cases} -\gamma_{H}\left(\mathbf{\frac{1}{2}}\right) & \textrm{if }\mathbf{t}=\mathbf{0}\\ \hat{A}_{H}\left(\mathbf{t}\right)v_{2}\left(\mathbf{t}\right)\beta_{H}\left(\mathbf{0}\right) & \textrm{else } \end{cases},\textrm{ }\forall\mathbf{t}\in\hat{\mathbb{Z}}_{2}\label{eq:MD Formula for Chi_H hat when every prime in P is 2 and alpha is 1} \end{equation} is then a Fourier transform of $\chi_{H}$. In this case, the function defined by: \begin{equation} \hat{\chi}_{H}\left(\mathbf{t}\right)=\begin{cases} \mathbf{0} & \textrm{if }\mathbf{t}=\mathbf{0}\\ \hat{A}_{H}\left(\mathbf{t}\right)\left(v_{2}\left(\mathbf{t}\right)\beta_{H}\left(\mathbf{0}\right)+\gamma_{H}\left(\mathbf{\frac{1}{2}}\right)\right) & \textrm{if }\textrm{else} \end{cases},\textrm{ }\forall\mathbf{t}\in\hat{\mathbb{Z}}_{2}^{r}\label{eq:MD Formula for Chi_H hat when every prime in P is 2 and alpha is 1, alt} \end{equation} is also a Fourier transform of $\chi_{H}$, differing from the $\hat{\chi}_{H}$ given above by $\hat{A}_{H}\left(\mathbf{t}\right)\gamma_{H}\left(\mathbf{\frac{1}{2}}\right)$. \emph{By}\textbf{\emph{ Theorem \ref{thm:MD properties of A_H hat}}},\textbf{ }since $\alpha_{H}\left(\mathbf{0}\right)=\mathbf{I}_{d}$, the function $\hat{A}_{H}\left(\mathbf{t}\right)\gamma_{H}\left(\mathbf{\frac{1}{2}}\right)$ is then the Fourier-Stieltjes transform of a degenerate thick measure of vector type. For odd primes $p$, we can obtain a Fourier transform for $\chi_{H}$ by defining a function $\hat{\chi}_{H}:\hat{\mathbb{Z}}_{p}^{r}\rightarrow\overline{\mathbb{Q}}^{d}$ by: \begin{equation} \hat{\chi}_{H}\left(\mathbf{t}\right)\overset{\textrm{def}}{=}\begin{cases} \mathbf{0} & \textrm{if }\mathbf{t}=\mathbf{0}\\ \hat{A}_{H}\left(\mathbf{t}\right)\left(v_{p}\left(\mathbf{t}\right)\beta_{H}\left(\mathbf{0}\right)+\gamma_{H}\left(\frac{\mathbf{t}\left|\mathbf{t}\right|_{p}}{p}\right)\right) & \textrm{else} \end{cases},\textrm{ }\forall\mathbf{t}\in\hat{\mathbb{Z}}_{p}^{r}\label{eq:MD Chi_H hat when rho is not 2 and when alpha is 1} \end{equation} \end{cor} Proof: \textbf{Corollaries \ref{cor:MD alpha is 1 case, F-series}} and \textbf{\ref{cor:MD alpha is 1, F-series on N_0^r}} show that the $N$th partial sums of the Fourier series generated by (\ref{eq:MD Formula for Chi_H hat when every prime in P is 2 and alpha is 1}) and (\ref{eq:MD Chi_H hat when rho is not 2 and when alpha is 1}) are $\mathcal{F}_{p,q_{H}}$-convergent to (\ref{eq:MD Explicit Formula for Chi_H when alpha is 1 and every prime in P is 2}) and (\ref{eq:MD Explicit Formula for Chi_H when alpha is 1 and rho is arbitrary}) for the case where $p=2$ and the case $p\geq3$, respectively, thereby establishing the quasi-integrability of $\chi_{H}$ with respect to the standard $\left(p,q_{H}\right)$-adic frame. Finally, letting $\hat{\chi}_{H}^{\prime}\left(\mathbf{t}\right)$ denote (\ref{eq:MD Formula for Chi_H hat when every prime in P is 2 and alpha is 1, alt}), observe that when $\alpha_{H}\left(\mathbf{0}\right)=\mathbf{I}_{d}$ and $p=2$: \begin{equation} \hat{\chi}_{H}^{\prime}\left(\mathbf{t}\right)-\hat{A}_{H}\left(\mathbf{t}\right)\gamma_{H}\left(\mathbf{\frac{1}{2}}\right)\overset{\overline{\mathbb{Q}}^{d}}{=}\begin{cases} -\gamma_{H}\left(\mathbf{\frac{1}{2}}\right) & \textrm{if }\mathbf{t}=\mathbf{0}\\ \hat{A}_{H}\left(\mathbf{t}\right)v_{2}\left(\mathbf{t}\right)\beta_{H}\left(\mathbf{0}\right) & \textrm{if }\left\Vert \mathbf{t}\right\Vert _{2}>0 \end{cases}=\hat{\chi}_{H}\left(\mathbf{t}\right) \end{equation} which shows that $\hat{\chi}_{H}^{\prime}\left(\mathbf{t}\right)$ and $\hat{\chi}_{H}\left(\mathbf{t}\right)$ differ by a factor of $\hat{A}_{H}\left(\mathbf{t}\right)\gamma_{H}\left(\mathbf{\frac{1}{2}}\right)$, which is the Fourier-Stieltjes transform of a degenerate vector-type thick measure, by \textbf{Theorem \ref{thm:MD properties of A_H hat}}, since $\alpha_{H}\left(\mathbf{0}\right)=\mathbf{I}_{d}$. Q.E.D. \subsection{\label{subsec:6.2.3 Multi-Dimensional--=00003D000026}Multi-Dimensional $\hat{\chi}_{H}$ and $\tilde{\chi}_{H,N}$ \textendash{} The Commutative Case} Like in the one-dimensional case, we now introduce $\psi_{H}$ and $\Psi_{H}$, which we will use in conjunction with $\chi_{H}$'s functional equations (\ref{eq:Functional Equations for Chi_H over the rho-adics}) to extend what we have done to cover all \textbf{\emph{commutative}} $p$-Hydra maps, regardless of the value of $\alpha_{H}\left(\mathbf{0}\right)$. \begin{defn}[\textbf{Multi-Dimensional Little Psi-$H$ \& Big Psi-$H$}] \ \vphantom{} I. We define $\psi_{H}:\mathbb{N}_{0}^{r}\rightarrow\overline{\mathbb{Q}}^{d,d}$ (``Little Psi-$H$'') by: \begin{equation} \psi_{H}\left(\mathbf{n}\right)\overset{\textrm{def}}{=}M_{H}\left(\mathbf{n}\right)\left(\mathbf{I}_{d}-H^{\prime}\left(\mathbf{0}\right)\right)^{-1}+\sum_{k=0}^{\lambda_{p}\left(\mathbf{n}\right)-1}\kappa_{H}\left(\left[\mathbf{n}\right]_{p^{k}}\right)\left(H^{\prime}\left(\mathbf{0}\right)\right)^{k}\label{eq:MD Definition of Little Psi_H} \end{equation} \vphantom{} II. We define $\Psi_{H}:\mathbb{N}_{0}^{r}\rightarrow\overline{\mathbb{Q}}^{d}$ (``Big Psi-$H$'') by: \begin{align} \Psi_{H}\left(\mathbf{n}\right) & \overset{\overline{\mathbb{Q}}^{d}}{=}M_{H}\left(\mathbf{n}\right)\sum_{\mathbf{j}>\mathbf{0}}^{p-1}\alpha_{H}\left(\frac{\mathbf{j}}{p}\right)\left(\mathbf{I}_{d}-H^{\prime}\left(\mathbf{0}\right)\right)^{-1}\gamma_{H}\left(\frac{\mathbf{j}}{p}\right)\label{eq:MD Definition of Big Psi_H}\\ & \sum_{k=0}^{\lambda_{p}\left(\mathbf{n}\right)-1}\kappa_{H}\left(\left[\mathbf{n}\right]_{p^{k}}\right)\sum_{\mathbf{j}>\mathbf{0}}^{p-1}\mathcal{C}_{H}\left(\alpha_{H}\left(\frac{\mathbf{j}}{p}\right)\varepsilon_{k}\left(\mathbf{j}\mathbf{n}\right):\lambda_{p}\left(\left[\mathbf{n}\right]_{p^{k}}\right)\right)\left(H^{\prime}\left(\mathbf{0}\right)\right)^{k}\gamma_{H}\left(\frac{\mathbf{j}}{p}\right) \end{align} In both cases, the $k$-sums are defined to be $\mathbf{0}$ when $\mathbf{n}=\mathbf{0}$. \end{defn} \begin{lem}[\textbf{Rising-Continuability and Functional Equations for Multi-Dimensional $\psi_{H}$ \& $\Psi_{H}$}] \label{lem:MD Rising-continuability of Psi_Hs}\ \vphantom{} I. $\psi_{H}$ is rising-continuable to a $d\times d$-matrix-valued $\left(p,q_{H}\right)$-adic function $\psi_{H}:\mathbb{Z}_{p}^{r}\rightarrow\mathbb{Z}_{q_{H}}^{d,d}$ given by: \begin{equation} \psi_{H}\left(\mathbf{z}\right)\overset{\mathcal{F}_{p,q_{H}}^{d,d}}{=}\sum_{n=0}^{\infty}\kappa_{H}\left(\left[\mathbf{z}\right]_{p^{n}}\right)\left(H^{\prime}\left(\mathbf{0}\right)\right)^{n},\textrm{ }\forall\mathbf{z}\in\left(\mathbb{Z}_{p}^{r}\right)^{\prime}\label{eq:MD Rising-continuation of Little Psi_H} \end{equation} Moreover, $\psi_{H}\left(\mathbf{z}\right)$ is the unique rising-continuous $d\times d$-matrix-valued $\left(p,q_{H}\right)$-adic function satisfying the system of functional equations: \begin{equation} \psi_{H}\left(p\mathbf{z}+\mathbf{j}\right)\overset{\mathbb{C}_{q_{H}}^{d,d}}{=}H_{\mathbf{j}}^{\prime}\left(\mathbf{0}\right)\psi_{H}\left(\mathbf{z}\right)+\mathbf{I}_{d},\textrm{ }\forall\mathbf{z}\in\mathbb{Z}_{p}^{r}\textrm{ \& }\forall\mathbf{j}\in\mathbb{Z}^{r}/p\mathbb{Z}^{r}\label{eq:MD Little Psi_H functional equations} \end{equation} \index{functional equation!psi_{H}@$\psi_{H}$!multi-dimensional} \vphantom{} II. $\Psi_{H}$ is rising-continuable to a $d\times1$-vector-valued $\left(p,q_{H}\right)$-adic function $\Psi_{H}:\mathbb{Z}_{p}^{r}\rightarrow\mathbb{C}_{q_{H}}^{d}$ given by:\index{functional equation!Psi_{H}@$\Psi_{H}$!multi-dimensional} \begin{equation} \Psi_{H}\left(\mathbf{z}\right)\overset{\mathcal{F}_{p,q_{H}}^{d}}{=}\sum_{n=0}^{\infty}\kappa_{H}\left(\left[\mathbf{z}\right]_{p^{n}}\right)\sum_{\mathbf{j}>\mathbf{0}}^{p-1}\mathcal{C}_{H}\left(\alpha_{H}\left(\frac{\mathbf{j}}{p}\right)\varepsilon_{n}\left(\mathbf{j}\mathbf{z}\right):\lambda_{p}\left(\left[\mathbf{z}\right]_{p^{n}}\right)\right)\left(H^{\prime}\left(\mathbf{0}\right)\right)^{n}\gamma_{H}\left(\frac{\mathbf{j}}{p}\right)\label{eq:MD Rising-continuation of Big Psi_H} \end{equation} for all $\mathbf{z}\in\left(\mathbb{Z}_{p}^{r}\right)^{\prime}$. Moreover, $\Psi_{H}\left(\mathbf{z}\right)$ is the unique rising-continuous $d\times1$-vector-valued $\left(p,q_{H}\right)$-adic function satisfying the system of functional equations: \begin{equation} \Psi_{H}\left(p\mathbf{z}+\mathbf{j}\right)\overset{\mathbb{C}_{q_{H}}^{d}}{=}H_{\mathbf{j}}^{\prime}\left(\mathbf{0}\right)\Psi_{H}\left(\mathbf{z}\right)+H_{\mathbf{j}}\left(\mathbf{0}\right)-\beta_{H}\left(\mathbf{0}\right),\textrm{ }\forall\mathbf{z}\in\mathbb{Z}_{p}^{r},\textrm{ }\forall\mathbf{j}\in\mathbb{Z}^{r}/p\mathbb{Z}^{r}\label{eq:MD Big Psi_H functional equations} \end{equation} \end{lem} Proof: For both parts, we use (\ref{eq:MD Relation between truncations and functional equations, version 2}) and (\ref{eq:MD Kappa_H functional equations}) from \textbf{Lemma \ref{lem:MD functional equations and truncation lemma}}. With these, observe that $\kappa_{H}$'s functional equations: \begin{equation} \kappa_{H}\left(p\mathbf{m}+\mathbf{j}\right)=\underbrace{\mathbf{D}_{\mathbf{j}}^{-1}\mathbf{A}_{\mathbf{j}}}_{H_{\mathbf{j}}^{\prime}\left(\mathbf{0}\right)}\kappa_{H}\left(\mathbf{m}\right)\underbrace{\mathbf{A}_{\mathbf{0}}^{-1}\mathbf{D}_{\mathbf{0}}}_{\left(H_{\mathbf{j}}\left(\mathbf{0}\right)\right)^{-1}} \end{equation} imply that: \begin{equation} \kappa_{H}\left(\left[p\mathbf{m}+\mathbf{j}\right]_{p^{n}}\right)=\mathbf{D}_{\mathbf{j}}^{-1}\mathbf{A}_{\mathbf{j}}\kappa_{H}\left(\left[\mathbf{m}\right]_{p^{n-1}}\right)\mathbf{A}_{\mathbf{0}}^{-1}\mathbf{D}_{\mathbf{0}},\textrm{ }\forall n\in\mathbb{N}_{1},\textrm{ }\forall\mathbf{j}\in\mathbb{Z}^{r}/p\mathbb{Z}^{r},\textrm{ }\forall\mathbf{m}\in\mathbb{N}_{0}^{r}\label{eq:MD functional equation truncation Lemma applied to kappa_H} \end{equation} In this case, the function $\Phi_{\mathbf{j}}$ from (\ref{eq:MD Relation between truncations and functional equations, version 1}) is, in this case: \begin{equation} \Phi_{\mathbf{j}}\left(\mathbf{m},\mathbf{X}\right)=\mathbf{D}_{\mathbf{j}}^{-1}\mathbf{A}_{\mathbf{j}}\mathbf{X}\mathbf{A}_{\mathbf{0}}^{-1}\mathbf{D}_{\mathbf{0}} \end{equation} where $\mathbf{X}$ is a $d\times d$ matrix. The rising-continuability of $\psi_{H}$ and $\psi_{H}$ to the given series follow by the givens on $H$, which guarantee that, for each $\mathbf{z}\in\mathbb{Z}_{p}^{r}$, $M_{H}\left(\left[\mathbf{z}\right]_{p^{n}}\right)=\kappa_{H}\left(\left[\mathbf{z}\right]_{p^{n}}\right)\left(H^{\prime}\left(\mathbf{0}\right)\right)^{\lambda_{p}\left(n\right)}$ tends to $0$ in the standard $\left(p,q_{H}\right)$-adic frame as $n\rightarrow\infty$, and convergence is then guaranteed by the same arguments used for \textbf{Lemma \ref{lem:MD v_p A_H hat summation formulae}} and equation (\ref{eq:F limit of MD Gamma_H A_H hat Fourier series}). All that remains is to verify the functional equations; \textbf{Theorem \ref{thm:rising-continuability of Generic H-type functional equations}} from Subsection \ref{subsec:5.3.2. Interpolation-Revisited} then guarantees the uniqueness of $\psi_{H}$ and $\Psi_{H}$ as rising-continuous solutions of their respective systems of functional equations. In what follows, recall that since we have assumed at the start of this chapter that $H$ is contracting, \textbf{Proposition \ref{prop:MD Contracting H proposition}} (page \pageref{prop:MD Contracting H proposition}) shows that the matrix $\mathbf{I}_{d}-H^{\prime}\left(\mathbf{0}\right)$ is invertible. \vphantom{} I. We pull out the $k=0$ term from $\psi_{H}\left(p\mathbf{n}+\mathbf{j}\right)$: \begin{align*} \psi_{H}\left(p\mathbf{n}+\mathbf{j}\right) & \overset{\overline{\mathbb{Q}}^{d,d}}{=}M_{H}\left(p\mathbf{n}+\mathbf{j}\right)\left(\mathbf{I}_{d}-H^{\prime}\left(\mathbf{0}\right)\right)^{-1}\mathbf{I}_{d}\\ & +\sum_{k=1}^{\lambda_{p}\left(p\mathbf{n}+\mathbf{j}\right)-1}\kappa_{H}\left(\left[p\mathbf{n}+\mathbf{j}\right]_{p^{k}}\right)\left(H^{\prime}\left(\mathbf{0}\right)\right)^{k}\\ & =\mathbf{D}_{\mathbf{j}}^{-1}\mathbf{A}_{\mathbf{j}}M_{H}\left(\mathbf{n}\right)\left(\mathbf{I}_{d}-H^{\prime}\left(\mathbf{0}\right)\right)^{-1}+\mathbf{I}_{d}\\ & +\sum_{k=1}^{\lambda_{p}\left(\mathbf{n}\right)}\mathbf{D}_{\mathbf{j}}^{-1}\mathbf{A}_{\mathbf{j}}\kappa_{H}\left(\left[\mathbf{n}\right]_{p^{k-1}}\right)\underbrace{\mathbf{A}_{\mathbf{0}}^{-1}\mathbf{D}_{\mathbf{0}}}_{\left(H^{\prime}\left(\mathbf{0}\right)\right)^{-1}}\left(H^{\prime}\left(\mathbf{0}\right)\right)^{k}\\ & =\mathbf{D}_{\mathbf{j}}^{-1}\mathbf{A}_{\mathbf{j}}\underbrace{\left(M_{H}\left(\mathbf{n}\right)\left(\mathbf{I}_{d}-H^{\prime}\left(\mathbf{0}\right)\right)^{-1}+\sum_{k=0}^{\lambda_{p}\left(\mathbf{n}\right)-1}\kappa_{H}\left(\left[\mathbf{n}\right]_{p^{k}}\right)\left(H^{\prime}\left(\mathbf{0}\right)\right)^{k}\right)}_{\psi_{H}\left(\mathbf{n}\right)}+\mathbf{I}_{d} \end{align*} Consequently: \begin{equation} \psi_{H}\left(p\mathbf{n}+\mathbf{j}\right)\overset{\overline{\mathbb{Q}}^{d,d}}{=}\underbrace{\mathbf{D}_{\mathbf{j}}^{-1}\mathbf{A}_{\mathbf{j}}}_{H_{\mathbf{j}}^{\prime}\left(\mathbf{0}\right)}\psi_{H}\left(\mathbf{n}\right)+\mathbf{I}_{d},\textrm{ }\forall\mathbf{n}\in\mathbb{N}_{0}^{r}\textrm{ \& }\forall j\in\mathbb{Z}^{r}/p\mathbb{Z}^{r}\label{eq:MD Little Psi_H functional equation on the integer tuples} \end{equation} then shows that (\ref{eq:MD Little Psi_H functional equation on the integer tuples}) extends to hold for the rising-continuation of $\psi_{H}$, and that this rising-continuation is then the \emph{unique }$\left(p,q_{H}\right)$-adic function satisfying (\ref{eq:MD Little Psi_H functional equations}). Finally, letting $\mathbf{m}\in\mathbb{N}_{0}^{r}$, setting $\mathbf{z}=\mathbf{m}$, the right-hand side of (\ref{eq:MD Rising-continuation of Little Psi_H}) becomes: \begin{align*} \sum_{n=0}^{\infty}\kappa_{H}\left(\left[\mathbf{m}\right]_{p^{n}}\right)\left(H^{\prime}\left(\mathbf{0}\right)\right)^{n} & \overset{\mathbb{C}^{d}}{=}\sum_{n=0}^{\lambda_{p}\left(\mathbf{m}\right)-1}\kappa_{H}\left(\left[\mathbf{m}\right]_{p^{n}}\right)\left(H^{\prime}\left(\mathbf{0}\right)\right)^{n}\\ & +\kappa_{H}\left(\mathbf{m}\right)\left(H^{\prime}\left(\mathbf{0}\right)\right)^{\lambda_{p}\left(\mathbf{m}\right)}\left(\mathbf{I}_{d}-H^{\prime}\left(\mathbf{0}\right)\right)\\ & =M_{H}\left(\mathbf{n}\right)\left(\mathbf{I}_{d}-H^{\prime}\left(\mathbf{0}\right)\right)^{-1}+\sum_{n=0}^{\lambda_{p}\left(\mathbf{m}\right)-1}\kappa_{H}\left(\left[\mathbf{m}\right]_{p^{n}}\right)\left(H^{\prime}\left(\mathbf{0}\right)\right)^{n}\\ & =\psi_{H}\left(\mathbf{m}\right) \end{align*} Hence, (\ref{eq:MD Rising-continuation of Little Psi_H}) converges to $\psi_{H}$ in the standard frame. \vphantom{} II. Pulling out $k=0$ from (\ref{eq:MD Definition of Big Psi_H}) yields: \begin{align*} \Psi_{H}\left(\mathbf{n}\right) & =M_{H}\left(\mathbf{n}\right)\sum_{\mathbf{j}>\mathbf{0}}^{p-1}\alpha_{H}\left(\frac{\mathbf{j}}{p}\right)\left(\mathbf{I}_{d}-H^{\prime}\left(\mathbf{0}\right)\right)^{-1}\gamma_{H}\left(\frac{\mathbf{j}}{p}\right)\\ & +\kappa_{H}\left(\mathbf{0}\right)\sum_{\mathbf{j}>\mathbf{0}}^{p-1}\mathcal{C}_{H}\left(\alpha_{H}\left(\frac{\mathbf{j}}{p}\right)\varepsilon_{0}\left(\mathbf{j}\mathbf{n}\right):0\right)\gamma_{H}\left(\frac{\mathbf{j}}{p}\right)\\ & +\sum_{k=1}^{\lambda_{p}\left(\mathbf{n}\right)-1}\kappa_{H}\left(\left[\mathbf{n}\right]_{p^{k}}\right)\sum_{\mathbf{j}>\mathbf{0}}^{p-1}\mathcal{C}_{H}\left(\alpha_{H}\left(\frac{\mathbf{j}}{p}\right)\varepsilon_{k}\left(\mathbf{j}\mathbf{n}\right):\lambda_{p}\left(\left[\mathbf{n}\right]_{p^{k}}\right)\right)\left(H^{\prime}\left(\mathbf{0}\right)\right)^{k}\gamma_{H}\left(\frac{\mathbf{j}}{p}\right) \end{align*} Here: \begin{equation} \kappa_{H}\left(\mathbf{0}\right)\sum_{\mathbf{j}>\mathbf{0}}^{p-1}\mathcal{C}_{H}\left(\alpha_{H}\left(\frac{\mathbf{j}}{p}\right)\varepsilon_{0}\left(\mathbf{j}\mathbf{n}\right):\lambda_{p}\left(0\right)\right)\gamma_{H}\left(\frac{\mathbf{j}}{p}\right) \end{equation} is: \begin{equation} \sum_{\mathbf{j}>\mathbf{0}}^{p-1}\alpha_{H}\left(\frac{\mathbf{j}}{p}\right)\gamma_{H}\left(\frac{\mathbf{j}}{p}\right)\varepsilon_{0}\left(\mathbf{j}\mathbf{n}\right) \end{equation} Now: \begin{align*} \sum_{\mathbf{j}=\mathbf{0}}^{p-1}\alpha_{H}\left(\frac{\mathbf{j}}{p}\right)\gamma_{H}\left(\frac{\mathbf{j}}{p}\right)\varepsilon_{0}\left(\mathbf{j}\mathbf{n}\right) & =\sum_{\mathbf{j}=\mathbf{0}}^{p-1}\beta_{H}\left(\frac{\mathbf{j}}{p}\right)\varepsilon_{0}\left(\mathbf{j}\mathbf{n}\right)\\ & =\sum_{\mathbf{k}=\mathbf{0}}^{p-1}\mathbf{D}_{\mathbf{k}}^{-1}\mathbf{b}_{\mathbf{k}}\frac{1}{p^{r}}\sum_{\mathbf{j}=\mathbf{0}}^{p-1}e^{2\pi i\frac{\mathbf{j}\cdot\left(\mathbf{n}-\mathbf{k}\right)}{p}}\\ & =\sum_{\mathbf{k}=\mathbf{0}}^{p-1}\mathbf{D}_{\mathbf{k}}^{-1}\mathbf{b}_{\mathbf{k}}\left[\mathbf{n}\overset{p}{\equiv}\mathbf{k}\right]\\ & =\mathbf{D}_{\left[\mathbf{n}\right]_{p}}^{-1}\mathbf{b}_{\left[\mathbf{n}\right]_{p}}\\ & =H_{\left[\mathbf{n}\right]_{p}}\left(\mathbf{0}\right) \end{align*} and so: \begin{equation} \sum_{\mathbf{j}>\mathbf{0}}^{p-1}\alpha_{H}\left(\frac{\mathbf{j}}{p}\right)\gamma_{H}\left(\frac{\mathbf{j}}{p}\right)\varepsilon_{0}\left(\mathbf{j}\mathbf{n}\right)=H_{\left[\mathbf{n}\right]_{p}}\left(\mathbf{0}\right)-\beta_{H}\left(\mathbf{0}\right) \end{equation} Consequently: \begin{align*} \Psi_{H}\left(\mathbf{n}\right) & =M_{H}\left(\mathbf{n}\right)\sum_{\mathbf{j}>\mathbf{0}}^{p-1}\alpha_{H}\left(\frac{\mathbf{j}}{p}\right)\left(\mathbf{I}_{d}-H^{\prime}\left(\mathbf{0}\right)\right)^{-1}\gamma_{H}\left(\frac{\mathbf{j}}{p}\right)+H_{\left[\mathbf{n}\right]_{p}}\left(\mathbf{0}\right)-\beta_{H}\left(\mathbf{0}\right)\\ & +\sum_{k=1}^{\lambda_{p}\left(\mathbf{n}\right)-1}\kappa_{H}\left(\left[\mathbf{n}\right]_{p^{k}}\right)\sum_{\mathbf{j}>\mathbf{0}}^{p-1}\mathcal{C}_{H}\left(\alpha_{H}\left(\frac{\mathbf{j}}{p}\right)\varepsilon_{k}\left(\mathbf{j}\mathbf{n}\right):\lambda_{p}\left(\left[\mathbf{n}\right]_{p^{k}}\right)\right)\left(H^{\prime}\left(\mathbf{0}\right)\right)^{k}\gamma_{H}\left(\frac{\mathbf{j}}{p}\right) \end{align*} Now, replacing $\mathbf{n}$ with $p\mathbf{n}+\mathbf{i}$ (where at least one of $\mathbf{n}$ and $\mathbf{i}$ is not the zero tuple), we use (\ref{eq:MD functional equation truncation Lemma applied to kappa_H}) and the functional equations for $M_{H}$, $\varepsilon_{n}$, and $\lambda_{p}$ (along with using (\ref{eq:MD Relation between truncations and functional equations, version 2}) for $\lambda_{p}\left(\left[\mathbf{n}\right]_{p^{k}}\right)$) and the definition of $\mathcal{C}_{H}$ to obtain: \begin{align*} & \mathbf{D}_{\mathbf{i}}^{-1}\mathbf{A}_{\mathbf{i}}M_{H}\left(\mathbf{n}\right)\sum_{\mathbf{j}>\mathbf{0}}^{p-1}\alpha_{H}\left(\frac{\mathbf{j}}{p}\right)\left(\mathbf{I}_{d}-H^{\prime}\left(\mathbf{0}\right)\right)^{-1}\gamma_{H}\left(\frac{\mathbf{j}}{p}\right)+H_{\mathbf{i}}\left(\mathbf{0}\right)-\beta_{H}\left(\mathbf{0}\right)\\ & +\mathbf{D}_{\mathbf{i}}^{-1}\mathbf{A}_{\mathbf{i}}\sum_{k=0}^{\lambda_{p}\left(\mathbf{n}\right)-1}\kappa_{H}\left(\left[\mathbf{n}\right]_{p^{k}}\right)\sum_{\mathbf{j}>\mathbf{0}}^{p-1}\mathcal{C}_{H}\left(\alpha_{H}\left(\frac{\mathbf{j}}{p}\right)\varepsilon_{k}\left(\mathbf{j}\mathbf{n}\right):\lambda_{p}\left(\left[\mathbf{n}\right]_{p^{k}}\right)\right)\left(H^{\prime}\left(\mathbf{0}\right)\right)^{k}\gamma_{H}\left(\frac{\mathbf{j}}{p}\right) \end{align*} as the formula for $\Psi_{H}\left(p\mathbf{n}+\mathbf{i}\right)$. This is: \[ \Psi_{H}\left(p\mathbf{n}+\mathbf{i}\right)=\underbrace{\mathbf{D}_{\mathbf{i}}^{-1}\mathbf{A}_{\mathbf{i}}}_{H_{\mathbf{i}}^{\prime}\left(\mathbf{0}\right)}\Psi_{H}\left(p\mathbf{n}+\mathbf{i}\right)+H_{\mathbf{i}}\left(\mathbf{0}\right)-\beta_{H}\left(\mathbf{0}\right) \] Finally, letting $\mathbf{X}$ denote $H^{\prime}\left(\mathbf{0}\right)$ and letting $\mathbf{m}\in\mathbb{N}_{0}^{r}$, we have that for $\mathbf{z}=\mathbf{m}$, the right-hand side of (\ref{eq:MD Rising-continuation of Big Psi_H}) becomes: \begin{align*} \sum_{n=0}^{\lambda_{p}\left(\mathbf{m}\right)-1}\kappa_{H}\left(\left[\mathbf{m}\right]_{p^{n}}\right)\sum_{\mathbf{j}>\mathbf{0}}^{p-1}\mathcal{C}_{H}\left(\alpha_{H}\left(\frac{\mathbf{j}}{p}\right)\varepsilon_{n}\left(\mathbf{j}\mathbf{m}\right):\lambda_{p}\left(\left[\mathbf{m}\right]_{p^{n}}\right)\right)\mathbf{X}^{n}\gamma_{H}\left(\frac{\mathbf{j}}{p}\right)\\ +\sum_{n=\lambda_{p}\left(\mathbf{m}\right)}^{\infty}\kappa_{H}\left(\mathbf{m}\right)\sum_{\mathbf{j}>\mathbf{0}}^{p-1}\mathcal{C}_{H}\left(\alpha_{H}\left(\frac{\mathbf{j}}{p}\right)\varepsilon_{n}\left(\mathbf{j}\mathbf{m}\right):\lambda_{p}\left(\mathbf{m}\right)\right)\mathbf{X}^{n}\gamma_{H}\left(\frac{\mathbf{j}}{p}\right) \end{align*} Here: \[ \varepsilon_{n}\left(\mathbf{j}\mathbf{m}\right)=e^{\frac{2\pi i}{p^{n+1}}\left(\left[\mathbf{j}\mathbf{m}\right]_{p^{n+1}}-\left[\mathbf{j}\mathbf{m}\right]_{p^{n}}\right)}=e^{\frac{2\pi i}{p^{n+1}}\left(\mathbf{j}\mathbf{m}-\mathbf{j}\mathbf{m}\right)}=1,\textrm{ }\forall n\geq\lambda_{p}\left(\mathbf{m}\right) \] and so: \begin{align*} \sum_{n=0}^{\lambda_{p}\left(\mathbf{m}\right)-1}\kappa_{H}\left(\left[\mathbf{m}\right]_{p^{n}}\right)\sum_{\mathbf{j}>\mathbf{0}}^{p-1}\mathcal{C}_{H}\left(\alpha_{H}\left(\frac{\mathbf{j}}{p}\right)\varepsilon_{n}\left(\mathbf{j}\mathbf{m}\right):\lambda_{p}\left(\left[\mathbf{m}\right]_{p^{n}}\right)\right)\mathbf{X}^{n}\gamma_{H}\left(\frac{\mathbf{j}}{p}\right)\\ +\sum_{n=\lambda_{p}\left(\mathbf{m}\right)}^{\infty}\kappa_{H}\left(\mathbf{m}\right)\sum_{\mathbf{j}>\mathbf{0}}^{p-1}\mathbf{X}^{\lambda_{p}\left(\mathbf{m}\right)}\alpha_{H}\left(\frac{\mathbf{j}}{p}\right)\mathbf{X}^{-\lambda_{p}\left(\mathbf{m}\right)}\mathbf{X}^{n}\gamma_{H}\left(\frac{\mathbf{j}}{p}\right) \end{align*} Summed in the topology of $\mathbb{C}$, this becomes: \begin{align*} \sum_{n=0}^{\lambda_{p}\left(\mathbf{m}\right)-1}\kappa_{H}\left(\left[\mathbf{m}\right]_{p^{n}}\right)\sum_{\mathbf{j}>\mathbf{0}}^{p-1}\mathcal{C}_{H}\left(\alpha_{H}\left(\frac{\mathbf{j}}{p}\right)\varepsilon_{n}\left(\mathbf{j}\mathbf{m}\right):\lambda_{p}\left(\left[\mathbf{m}\right]_{p^{n}}\right)\right)\mathbf{X}^{n}\gamma_{H}\left(\frac{\mathbf{j}}{p}\right)\\ +\kappa_{H}\left(\mathbf{m}\right)\sum_{\mathbf{j}>\mathbf{0}}^{p-1}\mathbf{X}^{\lambda_{p}\left(\mathbf{m}\right)}\alpha_{H}\left(\frac{\mathbf{j}}{p}\right)\mathbf{X}^{-\lambda_{p}\left(\mathbf{m}\right)}\mathbf{X}^{\lambda_{p}\left(\mathbf{m}\right)}\left(\mathbf{I}_{d}-\mathbf{X}\right)^{-1}\gamma_{H}\left(\frac{\mathbf{j}}{p}\right) \end{align*} which simplifies to: \begin{align*} \sum_{n=0}^{\lambda_{p}\left(\mathbf{m}\right)-1}\kappa_{H}\left(\left[\mathbf{m}\right]_{p^{n}}\right)\sum_{\mathbf{j}>\mathbf{0}}^{p-1}\mathcal{C}_{H}\left(\alpha_{H}\left(\frac{\mathbf{j}}{p}\right)\varepsilon_{n}\left(\mathbf{j}\mathbf{m}\right):\lambda_{p}\left(\left[\mathbf{m}\right]_{p^{n}}\right)\right)\mathbf{X}^{n}\gamma_{H}\left(\frac{\mathbf{j}}{p}\right)\\ +M_{H}\left(\mathbf{m}\right)\sum_{\mathbf{j}>\mathbf{0}}^{p-1}\alpha_{H}\left(\frac{\mathbf{j}}{p}\right)\left(\mathbf{I}_{d}-\mathbf{X}\right)^{-1}\gamma_{H}\left(\frac{\mathbf{j}}{p}\right) \end{align*} which is precisely $\Psi_{H}\left(\mathbf{m}\right)$ as given in (\ref{eq:MD Definition of Big Psi_H}). Hence, (\ref{eq:MD Rising-continuation of Big Psi_H}) converges to $\Psi_{H}$ in the standard frame. Q.E.D. \begin{prop}[\textbf{Quasi-Integrability of $\psi_{H}$ and $\Psi_{H}$}] \label{prop:quasi-integrability of MD Psis}Let $H$ be commutative. Then: \vphantom{} I. $\psi_{H}$ is quasi-integrable with respect to the standard $\left(p,q_{H}\right)$-adic frame whenever either $\alpha_{H}\left(\mathbf{0}\right)=\mathbf{I}_{d}$ or the matrix $\mathbf{I}_{d}-\alpha_{H}\left(\mathbf{0}\right)$ is invertible. For these conditions, the function $\hat{\psi}_{H}:\hat{\mathbb{Z}}_{p}^{r}\rightarrow\overline{\mathbb{Q}}^{d,d}$ defined by: \begin{equation} \hat{\psi}_{H}\left(\mathbf{t}\right)\overset{\textrm{def}}{=}\begin{cases} \begin{cases} \mathbf{O}_{d} & \textrm{if }\mathbf{t}=\mathbf{0}\\ v_{p}\left(\mathbf{t}\right)\hat{A}_{H}\left(\mathbf{t}\right) & \textrm{if }\mathbf{t}\neq\mathbf{0} \end{cases} & \textrm{if }\alpha_{H}\left(\mathbf{0}\right)=\mathbf{I}_{d}\\ \hat{A}_{H}\left(\mathbf{t}\right)\left(\mathbf{I}_{d}-\alpha_{H}\left(\mathbf{0}\right)\right)^{-1} & \textrm{if }\mathbf{I}_{d}-\alpha_{H}\left(\mathbf{0}\right)\textrm{ is invertible} \end{cases},\textrm{ }\forall\mathbf{t}\in\hat{\mathbb{Z}}_{p}^{r}\label{eq:MD Fourier Transform of Little Psi_H, commutative} \end{equation} is then a Fourier transform of $\psi_{H}$. Hence: \begin{equation} \hat{\psi}_{H,N}\left(\mathbf{z}\right)\overset{\overline{\mathbb{Q}}^{d,d}}{=}-N\kappa_{H}\left(\left[\mathbf{z}\right]_{p^{N}}\right)\left(H^{\prime}\left(\mathbf{0}\right)\right)^{N}+\sum_{n=0}^{N-1}\kappa_{H}\left(\left[\mathbf{z}\right]_{p^{n}}\right)\left(H^{\prime}\left(\mathbf{0}\right)\right)^{n}\label{eq:MD Little Psi H N twiddle when alpha is 1, commutative} \end{equation} when $\alpha_{H}\left(\mathbf{0}\right)=\mathbf{I}_{d}$ and: \begin{equation} \tilde{\psi}_{H,N}\left(\mathbf{z}\right)\overset{\overline{\mathbb{Q}}^{d,d}}{=}\kappa_{H}\left(\left[\mathbf{z}\right]_{p^{N}}\right)\left(H^{\prime}\left(\mathbf{0}\right)\right)^{N}\left(\mathbf{I}_{d}-\alpha_{H}\left(\mathbf{0}\right)\right)^{-1}+\sum_{n=0}^{N-1}\kappa_{H}\left(\left[\mathbf{z}\right]_{p^{n}}\right)\left(H^{\prime}\left(\mathbf{0}\right)\right)^{n}\label{eq:MD Little Psi H N twiddle when 1 minus alpha is invertible, commutative} \end{equation} when $\mathbf{I}_{d}-\alpha_{H}\left(\mathbf{0}\right)$ is invertible. \vphantom{} II. $\Psi_{H}$ is quasi-integrable with respect to the standard $\left(p,q_{H}\right)$-adic frame for all $\alpha_{H}\left(\mathbf{0}\right)$, and the function $\hat{\Psi}_{H}:\hat{\mathbb{Z}}_{p}^{r}\rightarrow\overline{\mathbb{Q}}^{d}$ defined by: \begin{equation} \hat{\Psi}_{H}\left(t\right)\overset{\textrm{def}}{=}\begin{cases} 0 & \textrm{if }\mathbf{t}=\mathbf{0}\\ \hat{A}_{H}\left(\mathbf{t}\right)\gamma_{H}\left(\frac{\mathbf{t}\left|\mathbf{t}\right|_{p}}{p}\right) & \textrm{if }\mathbf{t}\neq\mathbf{0} \end{cases},\textrm{ }\forall\mathbf{t}\in\hat{\mathbb{Z}}_{p}^{r}\label{eq:MD Fourier Transform of Big Psi_H} \end{equation} is a Fourier transform of $\Psi_{H}$. Hence: \begin{equation} \tilde{\Psi}_{H,N}\left(\mathbf{z}\right)\overset{\overline{\mathbb{Q}}^{d}}{=}\sum_{n=0}^{N-1}\kappa_{H}\left(\left[\mathbf{z}\right]_{p^{n}}\right)\sum_{\mathbf{j}>\mathbf{0}}^{p-1}\mathcal{C}_{H}\left(\alpha_{H}\left(\frac{\mathbf{j}}{p}\right)\varepsilon_{n}\left(\mathbf{j}\mathbf{z}\right):\lambda_{p}\left(\left[\mathbf{z}\right]_{p^{n}}\right)\right)\left(H^{\prime}\left(\mathbf{0}\right)\right)^{n}\gamma_{H}\left(\frac{\mathbf{j}}{p}\right)\label{eq:MD Big Psi H N twiddle} \end{equation} \end{prop} Proof: I. When $\alpha_{H}\left(\mathbf{0}\right)=\mathbf{I}_{d}$, (\ref{eq:MD Fourier Transform of Little Psi_H, commutative}) follows from \textbf{Lemma \ref{lem:MD v_p A_H hat summation formulae}}, and hence, the $\alpha_{H}\left(\mathbf{0}\right)=\mathbf{I}_{d}$ case of (\ref{eq:MD Fourier Transform of Little Psi_H, commutative}) is then indeed a Fourier transform of $\psi_{H}$, thus proving the $\mathcal{F}_{p,q_{H}}$-quasi-integrability of $\psi_{H}$ when $\alpha_{H}\left(\mathbf{0}\right)=\mathbf{I}_{d}$. When $\mathbf{I}_{d}-\alpha_{H}\left(\mathbf{0}\right)$ is invertible, comparing (\ref{eq:MD Fourier Limit of A_H,N twiddle in standard frame, commutative}): \begin{align*} \tilde{A}_{H}\left(\mathbf{z}\right) & \overset{\mathcal{F}_{p,q_{H}}^{d,d}}{=}\sum_{n=0}^{\infty}\kappa_{H}\left(\left[\mathbf{z}\right]_{p^{n}}\right)\left(H^{\prime}\left(\mathbf{0}\right)\right)^{n}\left(\mathbf{I}_{d}-\alpha_{H}\left(\mathbf{0}\right)\right) \end{align*} and (\ref{eq:MD Rising-continuation of Little Psi_H}): \[ \psi_{H}\left(\mathbf{z}\right)\overset{\mathcal{F}_{p,q_{H}}^{d,d}}{=}\sum_{n=0}^{\infty}\kappa_{H}\left(\left[\mathbf{z}\right]_{p^{n}}\right)\left(H^{\prime}\left(\mathbf{0}\right)\right)^{n} \] we observe that: \[ \tilde{A}_{H}\left(\mathbf{z}\right)=\psi_{H}\left(\mathbf{z}\right)\left(\mathbf{I}_{d}-\alpha_{H}\left(\mathbf{0}\right)\right) \] Since \textbf{Theorem \ref{thm:MD properties of A_H hat}} shows that $\tilde{A}_{H}\left(\mathbf{z}\right)$ is $\mathcal{F}_{p,q_{H}}$-quasi-integrable when $\mathbf{I}_{d}-\alpha_{H}\left(\mathbf{0}\right)$ is invertible (which necessarily forces $\alpha_{H}\left(\mathbf{0}\right)\neq\mathbf{I}_{d}$), it then follows that: \[ \psi_{H}\left(\mathbf{z}\right)=\tilde{A}_{H}\left(\mathbf{z}\right)\left(\mathbf{I}_{d}-\alpha_{H}\left(\mathbf{0}\right)\right)^{-1} \] and hence, that: \[ \hat{\psi}_{H}\left(\mathbf{t}\right)\overset{\textrm{def}}{=}\hat{A}_{H}\left(\mathbf{t}\right)\left(\mathbf{I}_{d}-\alpha_{H}\left(\mathbf{0}\right)\right)^{-1} \] is a Fourier transform of $\psi_{H}$. This proves that $\psi_{H}$ is quasi-integrable with respect to the standard frame when $\mathbf{I}_{d}-\alpha_{H}\left(\mathbf{0}\right)$ is invertible, and that (\ref{eq:MD Fourier Transform of Little Psi_H, commutative}) is then a Fourier transform of $\psi_{H}$ in this case. Finally, since $H$ is commutative, for the case $\alpha_{H}\left(\mathbf{0}\right)=\mathbf{I}_{d}$, (\ref{eq:MD Fourier sum of A_H hat v_rho, commutative}) becomes: \begin{align*} \sum_{0<\left\Vert \mathbf{t}\right\Vert _{p}\leq p^{N}}v_{p}\left(\mathbf{t}\right)\hat{A}_{H}\left(\mathbf{t}\right)e^{2\pi i\left\{ \mathbf{t}\mathbf{z}\right\} _{p}} & \overset{\overline{\mathbb{Q}}^{d,d}}{=}-N\kappa_{H}\left(\left[\mathbf{z}\right]_{p^{N}}\right)\left(H^{\prime}\left(\mathbf{0}\right)\right)^{N}\\ & +\sum_{n=0}^{N-1}\kappa_{H}\left(\left[\mathbf{z}\right]_{p^{n}}\right)\left(H^{\prime}\left(\mathbf{0}\right)\right)^{n} \end{align*} the left-hand side of which is exactly (\ref{eq:MD Little Psi H N twiddle when alpha is 1, commutative}). On the other hand, for the case where $\mathbf{I}_{d}-\alpha_{H}\left(\mathbf{0}\right)$ is invertible, the commutativity of $H$ tells us that (\ref{eq:MD Convolution of dA_H and D_N when H is commutative}) holds: \[ \tilde{A}_{H,N}\left(\mathbf{z}\right)=\kappa_{H}\left(\left[\mathbf{z}\right]_{p^{N}}\right)\left(H^{\prime}\left(\mathbf{0}\right)\right)^{N}+\sum_{n=0}^{N-1}\kappa_{H}\left(\left[\mathbf{z}\right]_{p^{n}}\right)\left(H^{\prime}\left(\mathbf{0}\right)\right)^{n}\left(\mathbf{I}_{d}-\alpha_{H}\left(\mathbf{0}\right)\right) \] Right-multiplying by $\mathbf{I}_{d}-\alpha_{H}\left(\mathbf{0}\right)$ then yields (\ref{eq:MD Little Psi H N twiddle when 1 minus alpha is invertible, commutative}). \vphantom{} II. (\ref{eq:MD Fourier Transform of Big Psi_H}) is exactly what we proved in \textbf{Lemma \ref{lem:MD Rising-continuability of Psi_Hs}}. (\ref{eq:MD Big Psi H N twiddle}) is precisely what we proved in (\ref{eq:MD Gamma formula}). Q.E.D. \vphantom{} Like in the one-dimensional case, we can bootstrap a Fourier transform for $\chi_{H}$ for the case where $H$ is commutative. However, this follows from a more general result which tells us how to express $\chi_{H}$ in terms of $\psi_{H}$ and $\Psi_{H}$: \begin{thm}[\textbf{$\mathcal{F}$-Series for Multi-Dimensional $\chi_{H}$}] \label{thm:MD F-series for Chi_H}Let $H$ be a $p$-smooth $d$-dimensional depth $r$ Hydra map dimension $d$ which is contracting, semi-basic, non-singular, and fixes $\mathbf{0}$. Regardless of: \vphantom{} I. Whether or not $\alpha_{H}\mathbf{\left(0\right)}=\mathbf{I}_{d}$; \vphantom{} II. Whether or not $\mathbf{I}_{d}-\alpha_{H}\left(\mathbf{0}\right)$ is invertible; \vphantom{} III. Whether or not $H$ is commutative; \vphantom{} the numen $\chi_{H}$ admits the $\mathcal{F}$-series representation: \begin{equation} \chi_{H}\left(\mathbf{z}\right)\overset{\mathcal{F}_{p,q_{H}}^{d}}{=}\psi_{H}\left(\mathbf{z}\right)\beta_{H}\left(\mathbf{0}\right)+\Psi_{H}\left(\mathbf{z}\right),\textrm{ }\forall\mathbf{z}\in\mathbb{Z}_{p}^{r}\label{eq:MD Chi_H in terms of Little Psi_H and Big Psi_H} \end{equation} \end{thm} Proof: Let $f\left(\mathbf{z}\right)$ denote the function $\psi_{H}\left(\mathbf{z}\right)\beta_{H}\left(\mathbf{0}\right)+\Psi_{H}\left(\mathbf{z}\right)$. Then, using the functional equations for $\psi_{H}$ and $\Psi_{H}$: \begin{equation} \psi_{H}\left(p\mathbf{z}+\mathbf{j}\right)=H_{\mathbf{j}}^{\prime}\left(\mathbf{0}\right)\psi_{H}\left(\mathbf{z}\right)+\mathbf{I}_{d} \end{equation} \begin{equation} \Psi_{H}\left(p\mathbf{z}+\mathbf{j}\right)=H_{\mathbf{j}}^{\prime}\left(\mathbf{0}\right)\Psi_{H}\left(\mathbf{z}\right)+H_{\mathbf{j}}\left(\mathbf{0}\right)-\beta_{H}\left(\mathbf{0}\right) \end{equation} we have that: \begin{align*} f\left(p\mathbf{z}+\mathbf{j}\right) & =\psi_{H}\left(p\mathbf{z}+\mathbf{j}\right)\beta_{H}\left(\mathbf{0}\right)+\Psi_{H}\left(p\mathbf{z}+\mathbf{j}\right)\\ & =\left(H_{\mathbf{j}}^{\prime}\left(\mathbf{0}\right)\psi_{H}\left(\mathbf{z}\right)+\mathbf{I}_{d}\right)\beta_{H}\left(\mathbf{0}\right)+\left(H_{\mathbf{j}}^{\prime}\left(\mathbf{0}\right)\Psi_{H}\left(\mathbf{z}\right)+H_{\mathbf{j}}\left(\mathbf{0}\right)-\beta_{H}\left(\mathbf{0}\right)\right)\\ & =H_{\mathbf{j}}^{\prime}\left(\mathbf{0}\right)\psi_{H}\left(\mathbf{z}\right)\beta_{H}\left(\mathbf{0}\right)+H_{\mathbf{j}}^{\prime}\left(\mathbf{0}\right)\Psi_{H}\left(\mathbf{z}\right)+H_{\mathbf{j}}\left(\mathbf{0}\right)\\ & =H_{\mathbf{j}}^{\prime}\left(\mathbf{0}\right)\left(\psi_{H}\left(\mathbf{z}\right)\beta_{H}\left(\mathbf{0}\right)+\Psi_{H}\left(\mathbf{z}\right)\right)+H_{\mathbf{j}}\left(\mathbf{0}\right)\\ & =H_{\mathbf{j}}^{\prime}\left(\mathbf{0}\right)f\left(\mathbf{z}\right)+H_{\mathbf{j}}\left(\mathbf{0}\right)\\ & =H_{\mathbf{j}}\left(f\left(\mathbf{z}\right)\right) \end{align*} Thus, $f$ satisfies $\chi_{H}$'s functional equation. Since $\chi_{H}$ is the unique $\left(p,q_{H}\right)$-adic function satisfying its functional equation asa described in \textbf{Lemma \ref{lem:MD rising-continuation and functional equations of Chi_H}}, this forces $f=\chi_{H}$. Q.E.D. \begin{cor}[\textbf{Quasi-Integrability of $\chi_{H}$ When $\mathbf{I}_{d}-\alpha_{H}\left(\mathbf{0}\right)$ is Invertible}] \label{cor:MD Quasi-integrability of Chi_H for I minus alpha invertible}Let $H$ be as given in \textbf{\emph{Theorem \ref{thm:MD F-series for Chi_H}}}. In addition, suppose $H$ is commutative, and that $\mathbf{I}_{d}-\alpha_{H}\left(\mathbf{0}\right)$ is invertible. Then, $\chi_{H}$ is quasi-integrable with respect to the standard $\left(p,q_{H}\right)$-adic frame, and the function $\hat{\chi}_{H}:\hat{\mathbb{Z}}_{p}^{r}\rightarrow\overline{\mathbb{Q}}^{d}$ defined by: \begin{equation} \hat{\chi}_{H}\left(\mathbf{t}\right)\overset{\textrm{def}}{=}\hat{A}_{H}\left(\mathbf{t}\right)\left(\mathbf{I}_{d}-\alpha_{H}\left(\mathbf{0}\right)\right)^{-1}\beta_{H}\left(\mathbf{0}\right)+\begin{cases} \mathbf{0} & \textrm{if }\mathbf{t}=\mathbf{0}\\ \hat{A}_{H}\left(\mathbf{t}\right)\gamma_{H}\left(\frac{\mathbf{t}\left|\mathbf{t}\right|_{p}}{p}\right) & \textrm{else} \end{cases},\textrm{ }\forall\mathbf{t}\in\hat{\mathbb{Z}}_{p}^{r}\label{eq:MD Chi_H hat for a contracting semi-basic commutative P Hydra map where alpha minus I is invertible} \end{equation} is a Fourier transform of $\chi_{H}$. \end{cor} Proof: The given hypotheses tell us that both $\psi_{H}$ and $\Psi_{H}$ are quasi-integrable. As such, by the linearity of the Fourier transform, (\ref{eq:MD Chi_H in terms of Little Psi_H and Big Psi_H}) becomes: \begin{equation} \hat{\chi}_{H}\left(\mathbf{t}\right)=\hat{\psi}_{H}\left(\mathbf{t}\right)\beta_{H}\left(\mathbf{0}\right)+\hat{\Psi}_{H}\left(\mathbf{t}\right) \end{equation} Using (\ref{eq:MD Fourier Transform of Little Psi_H, commutative}) and (\ref{eq:MD Fourier Transform of Big Psi_H}) from \textbf{Proposition \ref{prop:quasi-integrability of MD Psis}} to express the right-hand side of this yields (\ref{eq:MD Chi_H hat for a contracting semi-basic commutative P Hydra map where alpha minus I is invertible}). Q.E.D. \begin{cor}[\textbf{Formulae for $\tilde{\chi}_{H,N}$}] Let $H$ and $\hat{\chi}_{H}$ be as given in \textbf{\emph{Corollary \ref{cor:MD Quasi-integrability of Chi_H for I minus alpha invertible}}}. \vphantom{} I. If $p=2$, then: \begin{align} \tilde{\chi}_{H,N}\left(\mathbf{z}\right) & \overset{\overline{\mathbb{Q}}^{d}}{=}-\gamma_{H}\left(\mathbf{\frac{1}{2}}\right)+\kappa_{H}\left(\left[\mathbf{z}\right]_{2^{N}}\right)\left(H^{\prime}\left(\mathbf{0}\right)\right)^{N}\left(\left(\mathbf{I}_{d}-\alpha_{H}\left(\mathbf{0}\right)\right)^{-1}\beta_{H}\left(\mathbf{0}\right)+\gamma_{H}\left(\mathbf{\frac{1}{2}}\right)\right)\label{eq:MD Chi_H,N twiddle formula when P is 2 and alpha minus 1 is invertible, commutative}\\ & +\sum_{n=0}^{N-1}\kappa_{H}\left(\left[\mathbf{z}\right]_{2^{n}}\right)\left(H^{\prime}\left(\mathbf{0}\right)\right)^{n}\left(\beta_{H}\left(\mathbf{0}\right)+\left(\mathbf{I}_{d}-\alpha_{H}\left(\mathbf{0}\right)\right)\gamma_{H}\left(\mathbf{\frac{1}{2}}\right)\right)\nonumber \end{align} Hence, passing to the $\mathcal{F}_{2,q_{H}}$-limit as $N\rightarrow\infty$: \begin{equation} \chi_{H}\left(\mathbf{z}\right)\overset{\mathcal{F}_{2,q_{H}}^{d}}{=}-\gamma_{H}\left(\mathbf{\frac{1}{2}}\right)+\sum_{n=0}^{\infty}\kappa_{H}\left(\left[\mathbf{z}\right]_{2^{n}}\right)\left(H^{\prime}\left(\mathbf{0}\right)\right)^{n}\left(\beta_{H}\left(\mathbf{0}\right)+\left(\mathbf{I}_{d}-\alpha_{H}\left(\mathbf{0}\right)\right)\gamma_{H}\left(\mathbf{\frac{1}{2}}\right)\right)\label{eq:MD Chi_H explicit formula when P is 2 and alpha minus 1 is invertible, commutative} \end{equation} \vphantom{} II. If $p$ is odd: \begin{align} \tilde{\chi}_{H,N}\left(\mathbf{z}\right) & \overset{\overline{\mathbb{Q}}^{d}}{=}\kappa_{H}\left(\left[\mathbf{z}\right]_{p^{N}}\right)\left(H^{\prime}\left(\mathbf{0}\right)\right)^{N}+\sum_{n=0}^{N-1}\kappa_{H}\left(\left[\mathbf{z}\right]_{p^{n}}\right)\left(H^{\prime}\left(\mathbf{0}\right)\right)^{n}\left(\mathbf{I}_{d}-\alpha_{H}\left(\mathbf{0}\right)\right)\label{eq:MD Chi_H,N twiddle formula for arbitrary P and alpha minus 1 is invertible, commutative}\\ & +\sum_{n=0}^{N-1}\kappa_{H}\left(\left[\mathbf{z}\right]_{p^{n}}\right)\sum_{\mathbf{j}>\mathbf{0}}^{p-1}\mathcal{C}_{H}\left(\alpha_{H}\left(\frac{\mathbf{j}}{p}\right)\varepsilon_{n}\left(\mathbf{j}\mathbf{z}\right):\lambda_{p}\left(\left[\mathbf{z}\right]_{p^{n}}\right)\right)\left(H^{\prime}\left(\mathbf{0}\right)\right)^{n}\gamma_{H}\left(\frac{\mathbf{j}}{p}\right)\nonumber \end{align} Hence, passing to the $\mathcal{F}_{p,q_{H}}$-limit as $N\rightarrow\infty$: \begin{align} \chi_{H}\left(\mathbf{z}\right) & \overset{\mathcal{F}_{p,q_{H}}^{d}}{=}\sum_{n=0}^{\infty}\kappa_{H}\left(\left[\mathbf{z}\right]_{p^{n}}\right)\left(H^{\prime}\left(\mathbf{0}\right)\right)^{n}\left(\mathbf{I}_{d}-\alpha_{H}\left(\mathbf{0}\right)\right)\label{eq:MD Chi_H explicit formula for arbitrary P and alpha minus 1 is invertible, commutative}\\ & +\sum_{n=0}^{\infty}\kappa_{H}\left(\left[\mathbf{z}\right]_{p^{n}}\right)\sum_{\mathbf{j}>\mathbf{0}}^{p-1}\mathcal{C}_{H}\left(\alpha_{H}\left(\frac{\mathbf{j}}{p}\right)\varepsilon_{n}\left(\mathbf{j}\mathbf{z}\right):\lambda_{p}\left(\left[\mathbf{z}\right]_{p^{n}}\right)\right)\left(H^{\prime}\left(\mathbf{0}\right)\right)^{n}\gamma_{H}\left(\frac{\mathbf{j}}{p}\right)\nonumber \end{align} \end{cor} Proof: I. Suppose $p=2$. Then, by \textbf{Corollary \ref{cor:MD Quasi-integrability of Chi_H for I minus alpha invertible}}, we have that: \begin{equation} \hat{\chi}_{H}\left(\mathbf{t}\right)=\hat{A}_{H}\left(\mathbf{t}\right)\left(\mathbf{I}_{d}-\alpha_{H}\left(\mathbf{0}\right)\right)^{-1}\beta_{H}\left(\mathbf{0}\right)+\begin{cases} \mathbf{0} & \textrm{if }\mathbf{t}=\mathbf{0}\\ \hat{A}_{H}\left(\mathbf{t}\right)\gamma_{H}\left(\mathbf{\frac{1}{2}}\right) & \textrm{else} \end{cases} \end{equation} Hence, by (\ref{eq:MD Convolution of dA_H and D_N when H is commutative}) from\textbf{ Theorem \ref{thm:MD properties of A_H hat}}: \begin{align*} \tilde{\chi}_{H,N}\left(\mathbf{z}\right) & =\sum_{\left\Vert \mathbf{t}\right\Vert _{2}\leq2^{N}}\hat{A}_{H}\left(\mathbf{t}\right)e^{2\pi i\left\{ \mathbf{t}\mathbf{z}\right\} _{2}}\left(\mathbf{I}_{d}-\alpha_{H}\left(\mathbf{0}\right)\right)^{-1}\beta_{H}\left(\mathbf{0}\right)\\ & +\sum_{0<\left\Vert \mathbf{t}\right\Vert _{2}\leq2^{N}}\hat{A}_{H}\left(\mathbf{t}\right)e^{2\pi i\left\{ \mathbf{t}\mathbf{z}\right\} _{2}}\gamma_{H}\left(\mathbf{\frac{1}{2}}\right)\\ & =-\gamma_{H}\left(\mathbf{\frac{1}{2}}\right)+\kappa_{H}\left(\left[\mathbf{z}\right]_{2^{N}}\right)\left(H^{\prime}\left(\mathbf{0}\right)\right)^{N}\left(\left(\mathbf{I}_{d}-\alpha_{H}\left(\mathbf{0}\right)\right)^{-1}\beta_{H}\left(\mathbf{0}\right)+\gamma_{H}\left(\frac{1}{2}\right)\right)\\ & +\sum_{n=0}^{N-1}\kappa_{H}\left(\left[\mathbf{z}\right]_{2^{n}}\right)\left(H^{\prime}\left(\mathbf{0}\right)\right)^{n}\left(\beta_{H}\left(\mathbf{0}\right)+\left(\mathbf{I}_{d}-\alpha_{H}\left(\mathbf{0}\right)\right)\gamma_{H}\left(\mathbf{\frac{1}{2}}\right)\right) \end{align*} Letting $N\rightarrow\infty$, the above $\mathcal{F}_{2,q_{H}}^{d}$-converges to: \begin{align*} \chi_{H}\left(\mathbf{z}\right) & \overset{\mathcal{F}_{2,q_{H}}^{d}}{=}-\gamma_{H}\left(\mathbf{\frac{1}{2}}\right)+\sum_{n=0}^{\infty}\kappa_{H}\left(\left[\mathbf{z}\right]_{2^{n}}\right)\left(H^{\prime}\left(\mathbf{0}\right)\right)^{n}\left(\beta_{H}\left(\mathbf{0}\right)+\left(\mathbf{I}_{d}-\alpha_{H}\left(\mathbf{0}\right)\right)\gamma_{H}\left(\mathbf{\frac{1}{2}}\right)\right) \end{align*} \vphantom{} II. Letting $p\geq3$, \textbf{Corollary \ref{cor:MD Quasi-integrability of Chi_H for I minus alpha invertible}} gives us: \[ \hat{\chi}_{H}\left(\mathbf{t}\right)=\hat{A}_{H}\left(\mathbf{t}\right)\left(\mathbf{I}_{d}-\alpha_{H}\left(\mathbf{0}\right)\right)^{-1}\beta_{H}\left(\mathbf{0}\right)+\begin{cases} \mathbf{0} & \textrm{if }\mathbf{t}=\mathbf{0}\\ \hat{A}_{H}\left(\mathbf{t}\right)\gamma_{H}\left(\frac{\mathbf{t}\left|\mathbf{t}\right|_{p}}{p}\right) & \textrm{else} \end{cases} \] Using (\ref{eq:MD Convolution of dA_H and D_N when H is commutative}) and \textbf{Lemma \ref{lem:MD gamma formulae}}, we then have: \begin{align*} \tilde{\chi}_{H,N}\left(\mathbf{z}\right) & =\kappa_{H}\left(\left[\mathbf{z}\right]_{p^{N}}\right)\left(H^{\prime}\left(\mathbf{0}\right)\right)^{N}+\sum_{n=0}^{N-1}\kappa_{H}\left(\left[\mathbf{z}\right]_{p^{n}}\right)\left(H^{\prime}\left(\mathbf{0}\right)\right)^{n}\left(\mathbf{I}_{d}-\alpha_{H}\left(\mathbf{0}\right)\right)\\ & +\sum_{n=0}^{N-1}\kappa_{H}\left(\left[\mathbf{z}\right]_{p^{n}}\right)\sum_{\mathbf{j}>\mathbf{0}}^{p-1}\mathcal{C}_{H}\left(\alpha_{H}\left(\frac{\mathbf{j}}{p}\right)\varepsilon_{n}\left(\mathbf{j}\mathbf{z}\right):\lambda_{p}\left(\left[\mathbf{z}\right]_{p^{n}}\right)\right)\left(H^{\prime}\left(\mathbf{0}\right)\right)^{n}\gamma_{H}\left(\frac{\mathbf{j}}{p}\right) \end{align*} Taking limits as $N\rightarrow\infty$ gives the desired formula. Q.E.D. \begin{thm}[\textbf{Tauberian Spectral Theorem for Multi-Dimensional $p$-Hydra Maps}] \index{Hydra map!Tauberian Spectral Theorem}\label{thm:MD Periodic Points using WTT} \index{$p,q$-adic!Wiener Tauberian Theorem}\index{Hydra map!divergent trajectories}Let $H$ be as given in \textbf{\emph{Theorem \ref{thm:MD F-series for Chi_H}}}. If $\alpha_{H}\left(\mathbf{0}\right)=\mathbf{I}_{d}$, let: \begin{equation} \hat{\chi}_{H}\left(\mathbf{t}\right)=\begin{cases} -\gamma_{H}\left(\mathbf{\frac{1}{2}}\right) & \textrm{if }\mathbf{t}=\mathbf{0}\\ \hat{A}_{H}\left(\mathbf{t}\right)v_{2}\left(\mathbf{t}\right)\beta_{H}\left(\mathbf{0}\right) & \textrm{else } \end{cases},\textrm{ }\forall\mathbf{t}\in\hat{\mathbb{Z}}_{2}^{r} \end{equation} if every $p=2$. If $p\geq3$, let: \begin{equation} \hat{\chi}_{H}\left(\mathbf{t}\right)=\begin{cases} \mathbf{0} & \textrm{if }\mathbf{t}=\mathbf{0}\\ \hat{A}_{H}\left(\mathbf{t}\right)\left(v_{p}\left(\mathbf{t}\right)\beta_{H}\left(\mathbf{0}\right)+\gamma_{H}\left(\frac{\mathbf{t}\left|\mathbf{t}\right|_{p}}{p}\right)\right) & \textrm{else} \end{cases},\textrm{ }\forall\mathbf{t}\in\hat{\mathbb{Z}}_{p}^{r} \end{equation} Alternatively, if $\alpha_{H}\left(\mathbf{0}\right)\neq\mathbf{I}_{d}$, suppose $\mathbf{I}_{d}-\alpha_{H}\left(\mathbf{0}\right)$ is invertible and that $H$ is commutative. Then, let: \begin{equation} \hat{\chi}_{H}\left(\mathbf{t}\right)=\hat{A}_{H}\left(\mathbf{t}\right)\left(\mathbf{I}_{d}-\alpha_{H}\left(\mathbf{0}\right)\right)^{-1}\beta_{H}\left(\mathbf{0}\right)+\begin{cases} \mathbf{0} & \textrm{if }\mathbf{t}=\mathbf{0}\\ \hat{A}_{H}\left(\mathbf{t}\right)\gamma_{H}\left(\frac{\mathbf{t}\left|\mathbf{t}\right|_{p}}{p}\right) & \textrm{else} \end{cases},\textrm{ }\forall\mathbf{t}\in\hat{\mathbb{Z}}_{p}^{r} \end{equation} Finally, let $\mathbf{x}\in\mathbb{Z}^{d}\backslash\left\{ \mathbf{0}\right\} $. Then, for the formula for $\hat{\chi}_{H}$ chosen according to the situations described above: \vphantom{} I. If $\mathbf{x}$ is a periodic point of $H$, the translates of the function $\hat{\chi}_{H}\left(\mathbf{t}\right)-\mathbf{x}\mathbf{1}_{\mathbf{0}}\left(\mathbf{t}\right)$ are \emph{not }dense in $c_{0}\left(\hat{\mathbb{Z}}_{p}^{r},\mathbb{C}_{q_{H}}^{d}\right)$. \vphantom{} II. Suppose further that $H$ is integral, and that $\left\Vert H_{\mathbf{j}}\left(\mathbf{0}\right)\right\Vert _{q_{H}}=1$ for all $\mathbf{j}\in\left(\mathbb{Z}^{r}/p\mathbb{Z}^{r}\right)\backslash\left\{ \mathbf{0}\right\} $. If the translates of the function $\hat{\chi}_{H}\left(\mathbf{t}\right)-\mathbf{x}\mathbf{1}_{\mathbf{0}}\left(\mathbf{t}\right)$ are \emph{not }dense in $c_{0}\left(\hat{\mathbb{Z}}_{p}^{r},\mathbb{C}_{q_{H}}^{d}\right)$, then either $\mathbf{x}$ is a periodic point of $H$ or $\mathbf{x}$ belongs to a divergent trajectory of $H$. \end{thm} Proof: Essentially identical to the one-dimensional case (\textbf{Theorem \ref{thm:MD pq-adic WTT for continuous functions}}). Q.E.D. \subsection{\label{subsec:6.2.4 Multi-Dimensional--=00003D000026}Multi-Dimensional $\hat{\chi}_{H}$ and $\tilde{\chi}_{H,N}$ \textendash{} The Non-Commutative Case} Suppose that $H$ is non-commutative. Then, the multi-dimensional analogue of the factor $1-\alpha_{H}\left(0\right)$ is now: \begin{equation} \mathbf{I}_{H}\left(\lambda_{p}\left(\left[\mathbf{z}\right]_{p^{n}}\right)\right)=\mathbf{I}_{d}-\left(H^{\prime}\left(\mathbf{0}\right)\right)^{\lambda_{p}\left(\left[\mathbf{z}\right]_{p^{n}}\right)}\alpha_{H}\left(\mathbf{0}\right)\left(H^{\prime}\left(\mathbf{0}\right)\right)^{-\lambda_{p}\left(\left[\mathbf{z}\right]_{p^{n}}\right)} \end{equation} In the multi-dimensional case, the $\mathcal{F}_{p,q_{H}}$-limit of the Fourier series generated by $\hat{A}_{H}\left(t\right)$ is: \begin{equation} \tilde{A}_{H}\left(\mathbf{z}\right)\overset{\textrm{def}}{=}\lim_{N\rightarrow\infty}\tilde{A}_{H,N}\left(\mathbf{z}\right)\overset{\mathcal{F}_{p,q_{H}}^{d,d}}{=}\sum_{n=0}^{\infty}\kappa_{H}\left(\left[\mathbf{z}\right]_{p^{n}}\right)\mathbf{I}_{H}\left(\lambda_{p}\left(\left[\mathbf{z}\right]_{p^{n}}\right)\right)\left(H^{\prime}\left(\mathbf{0}\right)\right)^{n}\label{eq:MD Definition of A_H twiddle} \end{equation} (equation (\ref{eq:MD Fourier Limit of A_H,N twiddle in standard frame}) from \textbf{Theorem \ref{thm:MD properties of A_H hat}}). To speak of the multi-dimensional case for a moment using the terminology of the one-dimensional case, because $\psi_{H}\left(\mathfrak{z}\right)$ satisfied: \begin{equation} \psi_{H}\left(\mathfrak{z}\right)\overset{\mathcal{F}_{p,q_{H}}}{=}\sum_{n=0}^{\infty}\kappa_{H}\left(\left[\mathfrak{z}\right]_{p^{n}}\right)\left(H^{\prime}\left(0\right)\right)^{n} \end{equation} (\textbf{Lemma \ref{eq:Rising-continuation of Little Psi_H}}) the method of the one-dimensional case was to exploit the fact that the one-dimensional counterpart of $\mathbf{I}_{H}\left(\lambda_{p}\left(\left[\mathbf{z}\right]_{p^{n}}\right)\right)$ reduced to a constant which could then be factored out of the $n$-sum. To overcome this obstacle in the $\alpha_{H}\left(0\right)\neq1$ case, we ended up writing: \begin{align} \hat{\psi}_{H}\left(t\right) & =\frac{\hat{A}_{H}\left(t\right)}{1-\alpha_{H}\left(0\right)}\\ \psi_{H}\left(\mathfrak{z}\right) & =\frac{A_{H}\left(\mathfrak{z}\right)}{1-\alpha_{H}\left(0\right)} \end{align} the right-hand sides of which were defined solely because $\alpha_{H}\left(0\right)\neq1$. In particular, the non-vanishing of $\alpha_{H}\left(0\right)$ makes multiplication by $1/\left(1-\alpha_{H}\left(0\right)\right)$ into an \emph{invertible linear transformation}, with the linear transformation: \begin{equation} \mathcal{L}\left\{ f\right\} \left(\mathfrak{z}\right)\overset{\textrm{def}}{=}\left(1-\alpha_{H}\left(0\right)\right)f\left(\mathfrak{z}\right) \end{equation} then satisfying $\mathcal{L}\left\{ \psi_{H}\right\} \left(\mathfrak{z}\right)=A_{H}\left(\mathfrak{z}\right)$. Turning now to the multi-dimensional case, this suggests that to replicate this argument, we will need to find an invertible linear transformation which sends $\psi_{H}\left(\mathbf{z}\right)$ to $\tilde{A}_{H}\left(\mathbf{z}\right)$. In theory, by computing the effects of this linear transformation on the Fourier transforms of $\psi_{H}$ and $\tilde{A}_{H}$, we can then reverse-engineer a formula for $\hat{\psi}_{H}$ from our formula for $\hat{A}_{H}$. \begin{defn} We define the following operators on the space of functions $\mathbb{Z}_{p}^{r}\rightarrow\mathbb{C}_{q}^{d,d}$: \vphantom{} I. For all $n\geq0$: \begin{equation} \mathcal{T}_{n}\left\{ \mathbf{F}\right\} \left(\mathbf{z}\right)\overset{\textrm{def}}{=}\mathbf{F}\left(\left[\mathbf{z}\right]_{p^{n}}\right)\label{eq:Definition of the nth MD truncation operator} \end{equation} \vphantom{} II. For all $n\geq0$: \begin{equation} \mathcal{L}_{H,1,n}\left\{ \mathbf{F}\right\} \left(\mathbf{z}\right)\overset{\textrm{def}}{=}\mathcal{T}_{n}\left\{ \mathbf{F}\right\} \left(\mathbf{z}\right)\left(\mathbf{I}_{d}-H^{\prime}\left(\mathbf{0}\right)\right)\left(H^{\prime}\left(\mathbf{0}\right)\right)^{-n}\label{eq:Definition of L_H,1,n} \end{equation} \vphantom{} III. For all $n\geq1$: \begin{equation} \mathcal{L}_{H,2,n}\left\{ \mathbf{F}\right\} \left(\mathbf{z}\right)\overset{\textrm{def}}{=}\mathcal{L}_{H,1,n}\left\{ \mathbf{F}\right\} \left(\mathbf{z}\right)-\mathcal{L}_{H,1,n-1}\left\{ \mathbf{F}\right\} \left(\mathbf{z}\right)\left(H^{\prime}\left(\mathbf{0}\right)\right)^{-1}\label{eq:Definition of L_H,2,n} \end{equation} \vphantom{} IV. \begin{equation} \mathcal{E}_{H}\left\{ \mathbf{F}\right\} \left(\mathbf{z}\right)\overset{\textrm{def}}{=}\mathbf{F}\left(\mathbf{0}\right)\left(\mathbf{I}_{d}-H^{\prime}\left(\mathbf{0}\right)\right)\label{eq:Definition of E_H} \end{equation} \vphantom{} V. For all $n\geq0$: \begin{equation} \mathcal{L}_{H,3,n}\left\{ \mathbf{F}\right\} \left(\mathbf{z}\right)\overset{\textrm{def}}{=}\begin{cases} \mathcal{E}_{0}\left\{ \mathbf{F}\right\} \left(\mathbf{z}\right) & \textrm{if }n=0\\ \mathcal{E}_{0}\left\{ \mathbf{F}\right\} \left(\mathbf{z}\right)+\sum_{m=1}^{n}\mathcal{L}_{H,2,m}\left\{ \mathbf{F}\right\} \left(\mathbf{z}\right) & \textrm{if }n\geq1 \end{cases}\label{eq:Definition of L_H,3,n} \end{equation} \vphantom{} VI. For all $n\geq0$: \begin{equation} \mathcal{L}_{H,4,n}\left\{ \mathbf{F}\right\} \left(\mathbf{z}\right)\overset{\textrm{def}}{=}\mathcal{L}_{H,3,n}\left\{ \mathbf{F}\right\} \left(\mathbf{z}\right)\left(\mathbf{I}_{H}\left(\lambda_{p}\left(\left[\mathbf{z}\right]_{p^{n}}\right)\right)\right)^{-1}\left(H^{\prime}\left(\mathbf{0}\right)\right)^{n}\label{eq:Definition of L_H,4,n} \end{equation} \vphantom{} VII. \begin{equation} \mathcal{L}_{H}\left\{ \mathbf{F}\right\} \left(\mathbf{z}\right)\overset{\textrm{def}}{=}\sum_{n=0}^{\infty}\mathcal{L}_{H,4,n}\left\{ \mathbf{F}\right\} \left(\mathbf{z}\right)\label{eq:Definition of L_H} \end{equation} \end{defn} \begin{rem} Note that all of these operators are \emph{linear}. \end{rem} \begin{prop} If $\mathbf{I}_{d}-\alpha_{H}\left(\mathbf{0}\right)$ is invertible, then $\mathbf{I}_{H}\left(n\right)$ is invertible for all $n\geq0$. \end{prop} Proof: \begin{align*} \mathbf{I}_{H}\left(n\right) & =\mathbf{I}_{d}-\left(H^{\prime}\left(\mathbf{0}\right)\right)^{n}\alpha_{H}\left(\mathbf{0}\right)\left(H^{\prime}\left(\mathbf{0}\right)\right)^{-n}\\ & =\left(H^{\prime}\left(\mathbf{0}\right)\right)^{n}\left(\mathbf{I}_{d}-\alpha_{H}\left(\mathbf{0}\right)\right)\left(H^{\prime}\left(\mathbf{0}\right)\right)^{-n} \end{align*} Since $H$ is a $p$-Hydra map, $H^{\prime}\left(\mathbf{0}\right)$ is invertible, and hence: \[ \det\mathbf{I}_{H}\left(n\right)=\det\left(\mathbf{I}_{d}-\alpha_{H}\left(\mathbf{0}\right)\right) \] which is non-zero if and only if $\mathbf{I}_{d}-\alpha_{H}\left(\mathbf{0}\right)$ is invertible. Q.E.D. \begin{prop} Suppose $\mathbf{I}_{d}-\alpha_{H}\left(\mathbf{0}\right)$ is invertible. Then: \begin{equation} \mathcal{L}_{H}\left\{ \tilde{A}_{H}\right\} \left(\mathbf{z}\right)\overset{\mathcal{F}_{p,q_{H}}^{d,d}}{=}\psi_{H}\left(\mathbf{z}\right)\label{eq:L_H sends A_H twiddle to Little Psi_H} \end{equation} where, as indicated, we compute the value of either side by summing the infinite series on either side in accordance with the standard $\left(p,q_{H}\right)$-adic frame. \end{prop} \begin{rem} The linear operator which sends $\psi_{H}$ to $\tilde{A}_{H}$ is nearly identical to $\mathcal{L}_{H}$; all you do is change the multiplication by $\left(I_{H}\left(\lambda_{p}\left(\left[\mathbf{z}\right]_{p^{n}}\right)\right)\right)^{-1}$ in $\mathcal{L}_{H,4,n}$ to multiplication by $I_{H}\left(\lambda_{p}\left(\left[\mathbf{z}\right]_{p^{n}}\right)\right)$. \end{rem} Proof: First, we observe that: \begin{align*} \mathcal{T}_{m}\left\{ \tilde{A}_{H}\right\} \left(\mathbf{z}\right) & =\tilde{A}_{H}\left(\left[\mathbf{z}\right]_{p^{m}}\right)\\ & \overset{\mathbb{C}^{d,d}}{=}\sum_{n=0}^{\infty}\kappa_{H}\left(\left[\left[\mathbf{z}\right]_{p^{m}}\right]_{p^{n}}\right)\mathbf{I}_{H}\left(\lambda_{p}\left(\left[\left[\mathbf{z}\right]_{p^{m}}\right]_{p^{n}}\right)\right)\left(H^{\prime}\left(\mathbf{0}\right)\right)^{n}\\ & \overset{\mathbb{C}^{d,d}}{=}\kappa_{H}\left(\left[\mathbf{z}\right]_{p^{m}}\right)\mathbf{I}_{H}\left(\lambda_{p}\left(\left[\mathbf{z}\right]_{p^{m}}\right)\right)\sum_{n=m}^{\infty}\left(H^{\prime}\left(\mathbf{0}\right)\right)^{n}\\ & +\sum_{n=0}^{m-1}\kappa_{H}\left(\left[\mathbf{z}\right]_{p^{n}}\right)\mathbf{I}_{H}\left(\lambda_{p}\left(\left[\mathbf{z}\right]_{p^{n}}\right)\right)\left(H^{\prime}\left(\mathbf{0}\right)\right)^{n} \end{align*} Summing the geometric series yields: \begin{align*} \mathcal{T}_{m}\left\{ \tilde{A}_{H}\right\} \left(\mathbf{z}\right) & \overset{\mathbb{C}^{d,d}}{=}\kappa_{H}\left(\left[\mathbf{z}\right]_{p^{m}}\right)\mathbf{I}_{H}\left(\lambda_{p}\left(\left[\mathbf{z}\right]_{p^{m}}\right)\right)\left(H^{\prime}\left(\mathbf{0}\right)\right)^{m}\left(\mathbf{I}_{d}-H^{\prime}\left(\mathbf{0}\right)\right)^{-1}\\ & +\sum_{n=0}^{m-1}\kappa_{H}\left(\left[\mathbf{z}\right]_{p^{n}}\right)\mathbf{I}_{H}\left(\lambda_{p}\left(\left[\mathbf{z}\right]_{p^{n}}\right)\right)\left(H^{\prime}\left(\mathbf{0}\right)\right)^{n} \end{align*} Thus: \begin{align*} \mathcal{L}_{H,1,m}\left\{ \tilde{A}_{H}\right\} \left(\mathbf{z}\right) & \overset{\overline{\mathbb{Q}}^{d,d}}{=}\kappa_{H}\left(\left[\mathbf{z}\right]_{p^{m}}\right)\mathbf{I}_{H}\left(\lambda_{p}\left(\left[\mathbf{z}\right]_{p^{m}}\right)\right)\\ & +\sum_{n=0}^{m-1}\kappa_{H}\left(\left[\mathbf{z}\right]_{p^{n}}\right)\mathbf{I}_{H}\left(\lambda_{p}\left(\left[\mathbf{z}\right]_{p^{n}}\right)\right)\left(H^{\prime}\left(\mathbf{0}\right)\right)^{n}\left(\mathbf{I}_{d}-H^{\prime}\left(\mathbf{0}\right)\right)\left(H^{\prime}\left(\mathbf{0}\right)\right)^{-m} \end{align*} and so: \begin{equation} \mathcal{L}_{H,2,m}\left\{ \tilde{A}_{H}\right\} \left(\mathbf{z}\right)\overset{\overline{\mathbb{Q}}^{d,d}}{=}\kappa_{H}\left(\left[\mathbf{z}\right]_{p^{m}}\right)\mathbf{I}_{H}\left(\lambda_{p}\left(\left[\mathbf{z}\right]_{p^{m}}\right)\right)-\kappa_{H}\left(\left[\mathbf{z}\right]_{p^{m-1}}\right)\mathbf{I}_{H}\left(\lambda_{p}\left(\left[\mathbf{z}\right]_{p^{m-1}}\right)\right) \end{equation} Next, because: \begin{equation} \kappa_{H}\left(\mathbf{0}\right)=\mathbf{I}_{d} \end{equation} and: \begin{equation} \mathbf{I}_{H}\left(0\right)=\mathbf{I}_{d}-\alpha_{H}\left(\mathbf{0}\right) \end{equation} we have: \[ \sum_{m=1}^{n}\mathcal{L}_{H,2,m}\left\{ \tilde{A}_{H}\right\} \left(\mathbf{z}\right)=\kappa_{H}\left(\left[\mathbf{z}\right]_{p^{n}}\right)\mathbf{I}_{H}\left(\lambda_{p}\left(\left[\mathbf{z}\right]_{p^{n}}\right)\right)-\left(\mathbf{I}_{d}-\alpha_{H}\left(\mathbf{0}\right)\right) \] Now: \begin{equation} \mathcal{E}_{H}\left\{ \tilde{A}_{H}\right\} \left(\mathbf{z}\right)=\tilde{A}_{H}\left(\mathbf{0}\right)\left(\mathbf{I}_{d}-H^{\prime}\left(\mathbf{0}\right)\right) \end{equation} Since $\alpha_{H}\left(\mathbf{0}\right)\neq\mathbf{I}_{d}$: \begin{align*} \tilde{A}_{H}\left(\mathbf{0}\right) & =\sum_{n=0}^{\infty}\kappa_{H}\left(\mathbf{0}\right)\mathbf{I}_{H}\left(0\right)\left(H^{\prime}\left(\mathbf{0}\right)\right)^{n}\\ & =\left(\mathbf{I}_{d}-\alpha_{H}\left(\mathbf{0}\right)\right)\sum_{n=0}^{\infty}\left(H^{\prime}\left(\mathbf{0}\right)\right)^{n}\\ & =\left(\mathbf{I}_{d}-\alpha_{H}\left(\mathbf{0}\right)\right)\left(\mathbf{I}_{d}-H^{\prime}\left(\mathbf{0}\right)\right)^{-1} \end{align*} and so: \begin{align*} \mathcal{E}_{H}\left\{ \tilde{A}_{H}\right\} \left(\mathbf{z}\right) & =\tilde{A}_{H}\left(\mathbf{0}\right)\left(\mathbf{I}_{d}-H^{\prime}\left(\mathbf{0}\right)\right)\\ & =\left(\mathbf{I}_{d}-\alpha_{H}\left(\mathbf{0}\right)\right)\left(\mathbf{I}_{d}-H^{\prime}\left(\mathbf{0}\right)\right)^{-1}\left(\mathbf{I}_{d}-H^{\prime}\left(\mathbf{0}\right)\right)\\ & =\mathbf{I}_{d}-\alpha_{H}\left(\mathbf{0}\right) \end{align*} So, for $n\geq1$: \begin{align*} \mathcal{L}_{H,3,n}\left\{ \tilde{A}_{H}\right\} \left(\mathbf{z}\right) & =\mathcal{E}_{H}\left\{ \tilde{A}_{H}\right\} \left(\mathbf{z}\right)+\sum_{m=1}^{n}\mathcal{L}_{H,2,m}\left\{ \tilde{A}_{H}\right\} \left(\mathbf{z}\right)\\ & =\left(\mathbf{I}_{d}-\alpha_{H}\left(\mathbf{0}\right)\right)+\kappa_{H}\left(\left[\mathbf{z}\right]_{p^{n}}\right)\mathbf{I}_{H}\left(\lambda_{p}\left(\left[\mathbf{z}\right]_{p^{n}}\right)\right)-\left(\mathbf{I}_{d}-\alpha_{H}\left(\mathbf{0}\right)\right)\\ & =\kappa_{H}\left(\left[\mathbf{z}\right]_{p^{n}}\right)\mathbf{I}_{H}\left(\lambda_{p}\left(\left[\mathbf{z}\right]_{p^{n}}\right)\right) \end{align*} Finally: \begin{align*} \mathcal{L}_{H}\left\{ \tilde{A}_{H}\right\} \left(\mathbf{z}\right) & \overset{\mathcal{F}_{p,q_{H}}^{d,d}}{=}\sum_{n=0}^{\infty}\mathcal{L}_{H,4,n}\left\{ \tilde{A}_{H}\right\} \left(\mathbf{z}\right)\\ & \overset{\mathcal{F}_{p,q_{H}}^{d,d}}{=}\sum_{n=0}^{\infty}\mathcal{L}_{H,3,n}\left\{ \tilde{A}_{H}\right\} \left(\mathbf{z}\right)\left(\mathbf{I}_{H}\left(\lambda_{p}\left(\left[\mathbf{z}\right]_{p^{n}}\right)\right)\right)^{-1}\left(H^{\prime}\left(\mathbf{0}\right)\right)^{n}\\ & \overset{\mathcal{F}_{p,q_{H}}^{d,d}}{=}\sum_{n=0}^{\infty}\kappa_{H}\left(\left[\mathbf{z}\right]_{p^{n}}\right)\mathbf{I}_{H}\left(\lambda_{p}\left(\left[\mathbf{z}\right]_{p^{n}}\right)\right)\left(\mathbf{I}_{H}\left(\lambda_{p}\left(\left[\mathbf{z}\right]_{p^{n}}\right)\right)\right)^{-1}\left(H^{\prime}\left(\mathbf{0}\right)\right)^{n}\\ & \overset{\mathcal{F}_{p,q_{H}}^{d,d}}{=}\sum_{n=0}^{\infty}\kappa_{H}\left(\left[\mathbf{z}\right]_{p^{n}}\right)\left(H^{\prime}\left(\mathbf{0}\right)\right)^{n}\\ & \overset{\mathcal{F}_{p,q_{H}}^{d,d}}{=}\psi_{H}\left(\mathbf{z}\right) \end{align*} where $\left(\mathbf{I}_{H}\left(\lambda_{p}\left(\left[\mathbf{z}\right]_{p^{n}}\right)\right)\right)^{-1}$ is defined for all $\mathbf{z}$ and all $n$ because the invertibility of $\mathbf{I}_{d}-\alpha_{H}\left(\mathbf{0}\right)$ guarantees the invertibility of $\mathbf{I}_{H}\left(m\right)$ for all $m\geq0$. Q.E.D. \vphantom{} So, to obtain a Fourier transform for $\psi_{H}$, we just need only compute the effect $\mathcal{L}_{H}$ has on $\tilde{A}_{H}$'s Fourier transform ($\hat{A}_{H}$). This, however, is easier said than done. I have broken down the computation into several steps; to begin, we need to following definition: \begin{defn} For each $m\in\mathbb{N}_{0}$, let $\hat{K}_{m}:\hat{\mathbb{Z}}_{p}^{r}\rightarrow\mathbb{C}_{q}$ be defined by: \begin{equation} \hat{K}_{m}\left(\mathbf{t}\right)\overset{\textrm{def}}{=}\frac{1}{p^{rm}}\sum_{\mathbf{n}=\mathbf{0}}^{p^{m}-1}e^{-2\pi i\mathbf{n}\cdot\mathbf{t}}\label{eq:Definition of K_m hat} \end{equation} \end{defn} \begin{prop} \label{prop:A_H hat convolve K_H hat MD}Let $m\geq0$. Then, for all $\mathbf{t}\in\hat{\mathbb{Z}}_{p}^{r}$: \begin{equation} \left(\hat{A}_{H}*\hat{K}_{m}\right)\left(\mathbf{t}\right)=\sum_{n=0}^{m-1}\mathbf{1}_{\mathbf{0}}\left(p^{n}\mathbf{t}\right)\prod_{k=0}^{n-1}\alpha_{H}\left(p^{k}\mathbf{t}\right)+\left(\prod_{k=0}^{m-1}\alpha_{H}\left(p^{k}\mathbf{t}\right)\right)\left(\mathbf{I}_{d}-H^{\prime}\left(\mathbf{0}\right)\right)^{-1}\label{eq:Convolution of A_H hat and K_m hat} \end{equation} where the $k$ product is defined to be $\mathbf{I}_{d}$ whenever its upper limit is $<0$. \end{prop} Proof: We begin with the definition of $\hat{A}_{H}*\hat{K}_{m}$: \begin{equation} \left(\hat{A}_{H}*\hat{K}_{m}\right)\left(\mathbf{t}\right)=\sum_{\mathbf{s}\in\hat{\mathbb{Z}}_{p}^{r}}\hat{A}_{H}\left(\mathbf{s}\right)\frac{1}{p^{rm}}\sum_{\mathbf{n}=\mathbf{0}}^{p^{m}-1}e^{-2\pi i\left\{ \mathbf{n}\left(\mathbf{t}-\mathbf{s}\right)\right\} _{p}} \end{equation} We then pull out the $\mathbf{s}=\mathbf{0}$ term: \begin{align*} \hat{A}_{H}\left(\mathbf{0}\right)\frac{1}{p^{rm}}\sum_{\mathbf{n}=\mathbf{0}}^{p^{m}-1}e^{-2\pi i\left\{ \mathbf{n}\left(\mathbf{t}-\mathbf{0}\right)\right\} _{p}} & =\frac{1}{p^{rm}}\sum_{\mathbf{n}=\mathbf{0}}^{p^{m}-1}e^{-2\pi i\left\{ \mathbf{n}\mathbf{t}\right\} _{p}}\\ & =\frac{1}{p^{rm}}\sum_{\mathbf{n}=\mathbf{0}}^{p^{m}-1}e^{-2\pi i\left\{ \frac{\mathbf{n}}{p^{m}}p^{m}\mathbf{t}\right\} _{p}}\\ \left(\left\{ \mathbf{s}\in\hat{\mathbb{Z}}_{p}^{r}:\left\Vert \mathbf{s}\right\Vert _{p}\leq p^{m}\right\} =\left\{ \frac{\mathbf{n}}{p^{m}}:\mathbf{n}\leq p^{m}-1\right\} \right); & =\frac{1}{p^{rm}}\sum_{\left\Vert \mathbf{s}\right\Vert _{p}\leq p^{m}}e^{-2\pi i\left\{ \mathbf{s}p^{m}\mathbf{t}\right\} _{p}}\\ \left(\textrm{\textbf{proposition \ref{prop:Multi-Dimensional indicator function Fourier Series}}}\right); & =\left[p^{m}\mathbf{t}\overset{p^{m}}{\equiv}\mathbf{0}\right] \end{align*} Since congruence $p^{m}\mathbf{t}\overset{p^{m}}{\equiv}\mathbf{0}$ means that $p^{m}t_{\ell}\overset{p^{m}}{\equiv}0$ for all $\ell\in\left\{ 1,\ldots,r\right\} $, note that this occurs if and only if $\left|t_{\ell}\right|_{p}\leq p^{m}$. As such, our $\mathbf{s}=\mathbf{0}$ term becomes: \[ \hat{A}_{H}\left(\mathbf{0}\right)\frac{1}{p^{rm}}\sum_{\mathbf{n}=\mathbf{0}}^{p^{m}-1}e^{-2\pi i\left\{ \mathbf{n}\left(\mathbf{t}-\mathbf{0}\right)\right\} _{p}}=\left[p^{m}\mathbf{t}\overset{p^{m}}{\equiv}\mathbf{0}\right]=\mathbf{1}_{\mathbf{0}}\left(p^{m}\mathbf{t}\right) \] Next, using \textbf{Proposition \ref{prop:MD alpha H series}}, we express $\hat{A}_{H}\left(\mathbf{s}\right)$ as a series: \begin{equation} \hat{A}_{H}\left(\mathbf{s}\right)=\prod_{m=0}^{-v_{p}\left(\mathbf{s}\right)-1}\alpha_{H}\left(p^{m}\mathbf{s}\right)=\sum_{\mathbf{k}=\mathbf{0}}^{p^{-v_{p}\left(\mathbf{s}\right)}-1}\kappa_{H}\left(\mathbf{k}\right)\left(\frac{H^{\prime}\left(\mathbf{0}\right)}{p^{r}}\right)^{-v_{p}\left(\mathbf{s}\right)}e^{-2\pi i\left(\mathbf{k}\cdot\mathbf{s}\right)} \end{equation} and so: \begin{equation} \left(\hat{A}_{H}*\hat{K}_{m}\right)\left(\mathbf{t}\right)=\sum_{\mathbf{s}\in\hat{\mathbb{Z}}_{p}^{r}}\sum_{\mathbf{k}=\mathbf{0}}^{p^{-v_{p}\left(\mathbf{s}\right)}-1}\kappa_{H}\left(\mathbf{k}\right)\left(\frac{H^{\prime}\left(\mathbf{0}\right)}{p^{r}}\right)^{-v_{p}\left(\mathbf{s}\right)}\frac{e^{-2\pi i\left(\mathbf{k}\cdot\mathbf{s}\right)}}{p^{rm}}\sum_{\mathbf{n}=\mathbf{0}}^{p^{m}-1}e^{-2\pi i\left\{ \mathbf{n}\left(\mathbf{t}-\mathbf{s}\right)\right\} _{p}} \end{equation} Like many times before, we now split the $\mathbf{s}$-sum into a sum over level sets $\left\Vert \mathbf{s}\right\Vert _{p}=p^{h}$. This gives: \begin{equation} \mathbf{1}_{\mathbf{0}}\left(p^{m}\mathbf{t}\right)+\sum_{h=1}^{\infty}\sum_{\left\Vert \mathbf{s}\right\Vert _{p}=p^{h}}\sum_{\mathbf{k}=\mathbf{0}}^{p^{h}-1}\kappa_{H}\left(\mathbf{k}\right)\left(\frac{H^{\prime}\left(\mathbf{0}\right)}{p^{r}}\right)^{h}\frac{e^{-2\pi i\left(\mathbf{k}\cdot\mathbf{s}\right)}}{p^{rm}}\sum_{\mathbf{n}=\mathbf{0}}^{p^{m}-1}e^{-2\pi i\left\{ \mathbf{n}\left(\mathbf{t}-\mathbf{s}\right)\right\} _{p}} \end{equation} as our expression for $\left(\hat{A}_{H}*\hat{K}_{m}\right)\left(\mathbf{t}\right)$. Pulling the $\left\Vert \mathbf{s}\right\Vert _{p}$-sum to the far right, we have: \begin{align*} e^{-2\pi i\left(\mathbf{k}\cdot\mathbf{s}\right)}\sum_{\left\Vert \mathbf{s}\right\Vert _{p}=p^{h}}e^{-2\pi i\left\{ \mathbf{n}\left(\mathbf{t}-\mathbf{s}\right)\right\} _{p}} & =e^{-2\pi i\mathbf{n}\cdot\mathbf{t}}\sum_{\left\Vert \mathbf{s}\right\Vert _{p}=p^{h}}e^{2\pi i\left(\mathbf{n}-\mathbf{k}\right)\cdot\mathbf{s}}\\ & =p^{rh}e^{-2\pi i\mathbf{n}\cdot\mathbf{t}}\left[\mathbf{k}\overset{p^{h}}{\equiv}\mathbf{n}\right] \end{align*} and so: \begin{align*} \left(\hat{A}_{H}*\hat{K}_{m}\right)\left(\mathbf{t}\right) & =\mathbf{1}_{\mathbf{0}}\left(p^{m}\mathbf{t}\right)+\sum_{h=1}^{\infty}\sum_{\mathbf{k}=\mathbf{0}}^{p^{h}-1}\frac{1}{p^{rm}}\sum_{\mathbf{n}=\mathbf{0}}^{p^{m}-1}\kappa_{H}\left(\mathbf{k}\right)\left(H^{\prime}\left(\mathbf{0}\right)\right)^{h}e^{-2\pi i\mathbf{n}\cdot\mathbf{t}}\left[\mathbf{k}\overset{p^{h}}{\equiv}\mathbf{n}\right]\\ & =\mathbf{1}_{\mathbf{0}}\left(p^{m}\mathbf{t}\right)+\sum_{h=1}^{\infty}\frac{1}{p^{rm}}\sum_{\mathbf{n}=\mathbf{0}}^{p^{m}-1}\kappa_{H}\left(\left[\mathbf{n}\right]_{p^{h}}\right)\left(H^{\prime}\left(\mathbf{0}\right)\right)^{h}e^{-2\pi i\mathbf{n}\cdot\mathbf{t}}\\ & =\sum_{h=0}^{\infty}\frac{1}{p^{rm}}\sum_{\mathbf{n}=\mathbf{0}}^{p^{m}-1}\kappa_{H}\left(\left[\mathbf{n}\right]_{p^{h}}\right)\left(H^{\prime}\left(\mathbf{0}\right)\right)^{h}e^{-2\pi i\mathbf{n}\cdot\mathbf{t}} \end{align*} Because $m$ is finite, $\mathbf{n}\leq p^{m}-1$ tells us that $\left[\mathbf{n}\right]_{p^{h}}=\mathbf{n}$ for all $h\geq m$. Consequently: \begin{align*} \left(\hat{A}_{H}*K_{m}\right)\left(\mathbf{t}\right) & =\sum_{h=0}^{m-1}\frac{1}{p^{rm}}\sum_{\mathbf{n}=\mathbf{0}}^{p^{m}-1}\kappa_{H}\left(\left[\mathbf{n}\right]_{p^{h}}\right)\left(H^{\prime}\left(\mathbf{0}\right)\right)^{h}e^{-2\pi i\mathbf{n}\cdot\mathbf{t}}\\ & +\frac{1}{p^{rm}}\sum_{\mathbf{n}=\mathbf{0}}^{p^{m}-1}\kappa_{H}\left(\mathbf{n}\right)e^{-2\pi i\mathbf{n}\cdot\mathbf{t}}\sum_{h=m}^{\infty}\left(H^{\prime}\left(\mathbf{0}\right)\right)^{h} \end{align*} Since $H$ is contracting, by \textbf{Proposition \ref{prop:MD Contracting H proposition}} (page \pageref{prop:MD Contracting H proposition}) the series $\sum_{h=m}^{\infty}\left(H^{\prime}\left(\mathbf{0}\right)\right)^{h}$ converges in $\mathbb{R}$ to the matrix: \[ \sum_{h=m}^{\infty}\left(H^{\prime}\left(\mathbf{0}\right)\right)^{h}\overset{\mathbb{R}^{d,d}}{=}\left(H^{\prime}\left(\mathbf{0}\right)\right)^{m}\left(\mathbf{I}_{d}-H^{\prime}\left(\mathbf{0}\right)\right)^{-1} \] which has entries in $\mathbb{Q}$. Thus: \begin{align*} \left(\hat{A}_{H}*K_{m}\right)\left(\mathbf{t}\right) & =\sum_{h=0}^{m-1}\frac{1}{p^{rm}}\sum_{\mathbf{n}=\mathbf{0}}^{p^{m}-1}\kappa_{H}\left(\left[\mathbf{n}\right]_{p^{h}}\right)e^{-2\pi i\mathbf{n}\cdot\mathbf{t}}\left(H^{\prime}\left(\mathbf{0}\right)\right)^{h}\\ & +\frac{1}{p^{rm}}\sum_{\mathbf{n}=\mathbf{0}}^{p^{m}-1}\kappa_{H}\left(\mathbf{n}\right)e^{-2\pi i\mathbf{n}\cdot\mathbf{t}}\left(H^{\prime}\left(\mathbf{0}\right)\right)^{m}\left(\mathbf{I}_{d}-H^{\prime}\left(\mathbf{0}\right)\right)^{-1} \end{align*} Next, we split the $\mathbf{n}$-sum on the upper line modulo $p^{h}$: \begin{align*} \sum_{\mathbf{n}=\mathbf{0}}^{p^{m}-1}\kappa_{H}\left(\left[\mathbf{n}\right]_{p^{h}}\right)e^{-2\pi i\mathbf{n}\cdot\mathbf{t}} & =\sum_{\mathbf{k}=\mathbf{0}}^{p^{h}-1}\sum_{\mathbf{n}=\mathbf{0}}^{p^{m-h}-1}\kappa_{H}\left(\left[p^{h}\mathbf{n}+\mathbf{k}\right]_{p^{h}}\right)e^{-2\pi i\left(p^{h}\mathbf{n}+\mathbf{k}\right)\cdot\mathbf{t}}\\ & =\sum_{\mathbf{n}=\mathbf{0}}^{p^{m-h}-1}\left(\sum_{\mathbf{k}=\mathbf{0}}^{p^{h}-1}\kappa_{H}\left(\mathbf{k}\right)e^{-2\pi i\mathbf{k}\cdot\mathbf{t}}\right)e^{-2\pi i\mathbf{n}\cdot p^{h}\mathbf{t}} \end{align*} and so: \begin{align} \left(\hat{A}_{H}*K_{m}\right)\left(\mathbf{t}\right) & =\sum_{h=0}^{m-1}\frac{1}{p^{rm}}\sum_{\mathbf{n}=\mathbf{0}}^{p^{m-h}-1}\left(\sum_{\mathbf{k}=\mathbf{0}}^{p^{h}-1}\kappa_{H}\left(\mathbf{k}\right)e^{-2\pi i\mathbf{k}\cdot\mathbf{t}}\right)e^{-2\pi i\mathbf{n}\cdot p^{h}\mathbf{t}}\left(H^{\prime}\left(\mathbf{0}\right)\right)^{h}\label{eq:MD Halfway done with K_m convolution computation}\\ & +\frac{1}{p^{rm}}\sum_{\mathbf{n}=\mathbf{0}}^{p^{m}-1}\kappa_{H}\left(\mathbf{n}\right)e^{-2\pi i\mathbf{n}\cdot\mathbf{t}}\left(H^{\prime}\left(\mathbf{0}\right)\right)^{m}\left(\mathbf{I}_{d}-H^{\prime}\left(\mathbf{0}\right)\right)^{-1}\nonumber \end{align} To finish, we yet again apply the familiar recursive evaluation technique. First, define: \begin{equation} S_{h}\left(\mathbf{t}\right)\overset{\textrm{def}}{=}\sum_{\mathbf{k}=\mathbf{0}}^{p^{h}-1}\kappa_{H}\left(\mathbf{k}\right)e^{-2\pi i\left(\mathbf{k}\cdot\mathbf{t}\right)}\label{eq:S_h for MD K_m convolution computation} \end{equation} Then: \begin{align*} S_{h}\left(\mathbf{t}\right) & =\sum_{\mathbf{k}=\mathbf{0}}^{p^{h}-1}\kappa_{H}\left(\mathbf{k}\right)e^{-2\pi i\left(\mathbf{k}\cdot\mathbf{t}\right)}\\ & =\sum_{\mathbf{j}=\mathbf{0}}^{p-1}\sum_{\mathbf{k}=\mathbf{0}}^{p^{h-1}-1}\kappa_{H}\left(p\mathbf{k}+\mathbf{j}\right)e^{-2\pi i\left(\left(p\mathbf{k}+\mathbf{j}\right)\cdot\mathbf{t}\right)}\\ & =\sum_{\mathbf{j}=\mathbf{0}}^{p-1}\sum_{\mathbf{k}=\mathbf{0}}^{p^{h-1}-1}H_{\mathbf{j}}^{\prime}\left(\mathbf{0}\right)\kappa_{H}\left(\mathbf{k}\right)\left(H^{\prime}\left(\mathbf{0}\right)\right)^{-1}e^{-2\pi i\left(\left(p\mathbf{k}+\mathbf{j}\right)\cdot\mathbf{t}\right)}\\ & =\left(\sum_{\mathbf{j}=\mathbf{0}}^{p-1}H_{\mathbf{j}}^{\prime}\left(\mathbf{0}\right)e^{-2\pi i\mathbf{j}\cdot\mathbf{t}}\right)\sum_{\mathbf{k}=\mathbf{0}}^{p^{h-1}-1}\kappa_{H}\left(\mathbf{k}\right)e^{-2\pi i\left(\mathbf{k}\cdot p\mathbf{t}\right)}\left(H^{\prime}\left(\mathbf{0}\right)\right)^{-1}\\ & =p^{r}\alpha_{H}\left(\mathbf{t}\right)S_{h-1}\left(p\mathbf{t}\right)\left(H^{\prime}\left(\mathbf{0}\right)\right)^{-1} \end{align*} This yields the recursion relation: \begin{equation} S_{h}\left(\mathbf{t}\right)=p^{r}\alpha_{H}\left(\mathbf{t}\right)S_{h-1}\left(p\mathbf{t}\right)\left(H^{\prime}\left(\mathbf{0}\right)\right)^{-1}\label{eq:S_h recursion relation for K_m convolution computation} \end{equation} and so: \begin{equation} S_{h}\left(\mathbf{t}\right)=p^{rh}\left(\prod_{k=0}^{h-1}\alpha_{H}\left(p^{k}\mathbf{t}\right)\right)S_{0}\left(p^{h}\mathbf{t}\right)\left(H^{\prime}\left(\mathbf{0}\right)\right)^{-h} \end{equation} Here: \begin{equation} S_{0}\left(\mathbf{t}\right)=\kappa_{H}\left(\mathbf{0}\right)=\mathbf{I}_{d} \end{equation} which leaves us with: \begin{equation} S_{h}\left(\mathbf{t}\right)=p^{hr}\left(\prod_{k=0}^{h-1}\alpha_{H}\left(p^{k}\mathbf{t}\right)\right)\left(H^{\prime}\left(\mathbf{0}\right)\right)^{-h}\label{eq:Explicit formula for S_h in K_m convolution computation} \end{equation} Returning to (\ref{eq:MD Halfway done with K_m convolution computation}), we have: \begin{align*} \left(\hat{A}_{H}*K_{m}\right)\left(\mathbf{t}\right) & =\sum_{h=0}^{m-1}\frac{1}{p^{rm}}\sum_{\mathbf{n}=\mathbf{0}}^{p^{m-h}-1}\left(p^{rh}\left(\prod_{k=0}^{h-1}\alpha_{H}\left(p^{k}\mathbf{t}\right)\right)\left(H^{\prime}\left(\mathbf{0}\right)\right)^{-h}\right)e^{-2\pi i\mathbf{n}\cdot p^{h}\mathbf{t}}\left(H^{\prime}\left(\mathbf{0}\right)\right)^{h}\\ & +\frac{1}{p^{rm}}p^{rm}\left(\prod_{k=0}^{m-1}\alpha_{H}\left(p^{k}\mathbf{t}\right)\right)\left(H^{\prime}\left(\mathbf{0}\right)\right)^{-m}\left(H^{\prime}\left(\mathbf{0}\right)\right)^{m}\left(\mathbf{I}_{d}-H^{\prime}\left(\mathbf{0}\right)\right)^{-1}\\ & =\sum_{h=0}^{m-1}\left(\prod_{k=0}^{h-1}\alpha_{H}\left(p^{k}\mathbf{t}\right)\right)\frac{1}{p^{r\left(m-h\right)}}\sum_{\mathbf{n}=\mathbf{0}}^{p^{m-h}-1}e^{-2\pi i\mathbf{n}\cdot p^{h}\mathbf{t}}\\ & +\left(\prod_{k=0}^{m-1}\alpha_{H}\left(p^{k}\mathbf{t}\right)\right)\left(\mathbf{I}_{d}-H^{\prime}\left(\mathbf{0}\right)\right)^{-1} \end{align*} Here: \begin{align*} \sum_{\mathbf{n}=\mathbf{0}}^{p^{m-h}-1}e^{-2\pi i\mathbf{n}\cdot p^{h}\mathbf{t}} & =\sum_{\mathbf{n}=\mathbf{0}}^{p^{m-h}-1}e^{-2\pi i\frac{\mathbf{n}}{p^{m-h}}\cdot p^{m}\mathbf{t}}\\ & =\sum_{\left\Vert \mathbf{s}\right\Vert _{p}\leq p^{m-h}}e^{-2\pi i\left(\mathbf{s}\cdot p^{m}\mathbf{t}\right)}\\ & =p^{r\left(m-h\right)}\left[p^{m}\mathbf{t}\overset{p^{m-h}}{\equiv}\mathbf{0}\right] \end{align*} The congruence says that $p^{m}\mathbf{t}\in p^{m-h}\mathbb{Z}^{r}$. This implies $p^{h}\mathbf{t}\in\mathbb{Z}^{r}$, which means: \begin{equation} \left[p^{m}\mathbf{t}\overset{p^{m-h}}{\equiv}\mathbf{0}\right]=\left[p^{h}\mathbf{t}\overset{1}{\equiv}\mathbf{0}\right]=\mathbf{1}_{\mathbf{0}}\left(p^{h}\mathbf{t}\right) \end{equation} Hence: \begin{align*} \left(\hat{A}_{H}*K_{m}\right)\left(\mathbf{t}\right) & =\sum_{h=0}^{m-1}\left(\prod_{k=0}^{h-1}\alpha_{H}\left(p^{k}\mathbf{t}\right)\right)\frac{1}{p^{r\left(m-h\right)}}p^{m-h}\mathbf{1}_{\mathbf{0}}\left(p^{h}\mathbf{t}\right)\\ & +\left(\prod_{k=0}^{m-1}\alpha_{H}\left(p^{k}\mathbf{t}\right)\right)\left(\mathbf{I}_{d}-H^{\prime}\left(\mathbf{0}\right)\right)^{-1}\\ & =\sum_{h=0}^{m-1}\mathbf{1}_{\mathbf{0}}\left(p^{h}\mathbf{t}\right)\left(\prod_{k=0}^{h-1}\alpha_{H}\left(p^{k}\mathbf{t}\right)\right)+\left(\prod_{k=0}^{m-1}\alpha_{H}\left(p^{k}\mathbf{t}\right)\right)\left(\mathbf{I}_{d}-H^{\prime}\left(\mathbf{0}\right)\right)^{-1} \end{align*} Q.E.D. \begin{lem} \label{lem:T_m hat of A_H hat}Let $m\in\mathbb{N}_{0}$. Then: \begin{equation} \hat{\mathcal{T}}_{m}\left\{ \hat{A}_{H}\right\} \left(\mathbf{t}\right)=\sum_{n=0}^{m-1}\mathbf{1}_{\mathbf{0}}\left(p^{n}\mathbf{t}\right)\prod_{k=0}^{n-1}\alpha_{H}\left(p^{k}\mathbf{t}\right)+\left(\prod_{k=0}^{m-1}\alpha_{H}\left(p^{k}\mathbf{t}\right)\right)\left(\mathbf{I}_{d}-H^{\prime}\left(\mathbf{0}\right)\right)^{-1}\label{eq:T_m hat of A_H hat} \end{equation} where the $k$ product is defined to be $\mathbf{I}_{d}$ whenever its upper limit is $<0$, and where the $m$-sum is defined to be $\mathbf{O}_{d}$ when $m=0$. \end{lem} Proof: Note that: \begin{align*} \mathcal{T}_{m}\left\{ \tilde{A}_{H}\right\} \left(\mathbf{z}\right) & =\sum_{\mathbf{n}=\mathbf{0}}^{p^{m}-1}\tilde{A}_{H}\left(\mathbf{n}\right)\left[\mathbf{z}\overset{p^{m}}{\equiv}\mathbf{n}\right]\\ & =\frac{1}{p^{rm}}\sum_{\mathbf{n}=\mathbf{0}}^{p^{m}-1}\tilde{A}_{H}\left(\mathbf{n}\right)\sum_{\left\Vert \mathbf{s}\right\Vert _{p}\leq p^{m}}e^{2\pi i\left\{ \mathbf{s}\left(\mathbf{z}-\mathbf{n}\right)\right\} _{p}}\\ & =\frac{1}{p^{rm}}\sum_{\mathbf{n}=\mathbf{0}}^{p^{m}-1}\sum_{\mathbf{t}\in\hat{\mathbb{Z}}_{p}^{r}}\hat{A}_{H}\left(\mathbf{t}\right)e^{2\pi i\left\{ \mathbf{t}\mathbf{n}\right\} _{p}}\sum_{\left\Vert \mathbf{s}\right\Vert _{p}\leq p^{m}}e^{2\pi i\left\{ \mathbf{s}\left(\mathbf{z}-\mathbf{n}\right)\right\} _{p}}\\ & =\sum_{\left\Vert \mathbf{s}\right\Vert _{p}\leq p^{m}}\sum_{\mathbf{t}\in\hat{\mathbb{Z}}_{p}^{r}}\hat{A}_{H}\left(\mathbf{t}\right)\frac{1}{p^{rm}}\sum_{\mathbf{n}=\mathbf{0}}^{p^{m}-1}e^{-2\pi i\left\{ \mathbf{n}\left(\mathbf{s}-\mathbf{t}\right)\right\} _{p}}e^{2\pi i\left\{ \mathbf{s}\mathbf{z}\right\} _{p}}\\ & =\sum_{\left\Vert \mathbf{s}\right\Vert _{p}\leq p^{m}}\left(\hat{A}_{H}*\hat{K}_{m}\right)\left(\mathbf{s}\right)e^{2\pi i\left\{ \mathbf{s}\mathbf{z}\right\} _{p}} \end{align*} Then use \textbf{Proposition \ref{prop:A_H hat convolve K_H hat MD}}. Q.E.D. \vphantom{} The last ingredient is the proposition given below. However, we will need another bit of notation to help us along the way: \begin{notation} Let $\mathbf{J}=\left(\mathbf{j}_{1},\ldots,\mathbf{j}_{\left|\mathbf{J}\right|}\right)\in\textrm{String}^{r}\left(p\right)$ be a $p$-block string. Then, we define: \begin{equation} \mathbf{t}\cdot\mathbf{J}\overset{\textrm{def}}{=}\mathbf{t}\cdot\sum_{k=1}^{\left|\mathbf{J}\right|}\mathbf{j}_{k}p^{k-1}\label{eq:Definition of t dot big bold J} \end{equation} Writing each $\mathbf{j}_{k}$ as $\left(j_{k,1},\ldots,j_{k,r}\right)$, we have that: \begin{equation} \sum_{k=1}^{\left|\mathbf{J}\right|}\mathbf{j}_{k}p^{k-1}=\left(j_{1,1}+\cdots+j_{\left|\mathbf{J}\right|,1}p^{\left|\mathbf{J}\right|-1},\ldots,j_{1,r}+\cdots+j_{\left|\mathbf{J}\right|,r}p^{\left|\mathbf{J}\right|-1}\right) \end{equation} In particular, $\sum_{k=1}^{\left|\mathbf{J}\right|}\mathbf{j}_{k}p^{k-1}=\mathbf{n}$, where $\mathbf{n}\in\mathbb{N}_{0}^{r}$ is the $r$-tuple represented by $\mathbf{J}$. Additionally, we write: \begin{equation} \sum_{\left|\mathbf{J}\right|=n}=\sum_{\mathbf{j}_{1},\ldots,\mathbf{j}_{n}\in\mathbb{Z}^{r}/p\mathbb{Z}^{r}} \end{equation} to denote a sum taken over all $\mathbf{J}\in\textrm{String}^{r}\left(p\right)$ of length exactly $n$. \end{notation} \begin{prop} Let $n\geq0$. Then, the Fourier transform of the locally constant $\overline{\mathbb{Q}}^{d,d}$-valued function: \begin{equation} \mathbf{z}\mapsto\mathbf{I}_{H}\left(\lambda_{p}\left(\left[\mathbf{z}\right]_{p^{n}}\right)\right) \end{equation} is given by: \begin{align} \int_{\mathbb{Z}_{p}^{r}}\mathbf{I}_{H}\left(\lambda_{p}\left(\left[\mathbf{z}\right]_{p^{n}}\right)\right)e^{-2\pi i\left\{ \mathbf{t}\mathbf{z}\right\} _{p}}d\mathbf{z} & =\mathbf{1}_{\mathbf{0}}\left(\mathbf{t}\right)\mathbf{I}_{d}-\mathcal{C}_{H}\left(\alpha_{H}\left(\mathbf{0}\right):n\right)\hat{K}_{n}\left(\mathbf{t}\right)\label{eq:Fourier transform of I_H of lambda_P of z mod P^n} \end{align} \end{prop} Proof: To keep the computations from running off the right side of the page, let us write $\mathbf{A}=\alpha_{H}\left(\mathbf{0}\right)$ and $\mathbf{X}=H^{\prime}\left(\mathbf{0}\right)$. Then, the integral: \begin{equation} \int_{\mathbb{Z}_{p}^{r}}\mathbf{I}_{H}\left(\lambda_{p}\left(\left[\mathbf{z}\right]_{p^{n}}\right)\right)e^{-2\pi i\left\{ \mathbf{t}\mathbf{z}\right\} _{p}}d\mathbf{z} \end{equation} is: \begin{equation} \int_{\mathbb{Z}_{p}^{r}}\left(\mathbf{I}_{d}-\mathbf{X}^{\lambda_{p}\left(\left[\mathbf{z}\right]_{p^{n}}\right)}\mathbf{A}\mathbf{X}^{-\lambda_{p}\left(\left[\mathbf{z}\right]_{p^{n}}\right)}\right)e^{-2\pi i\left\{ \mathbf{t}\mathbf{z}\right\} _{p}}d\mathbf{z} \end{equation} Because the main part of the integrand is a constant with respect to values of $\mathbf{z}$ modulo $p^{n}$, the integral reduces to a finite sum: \begin{equation} \mathbf{1}_{\mathbf{0}}\left(\mathbf{t}\right)\mathbf{I}_{d}-\frac{1}{p^{rn}}\sum_{\mathbf{m}=\mathbf{0}}^{p^{n}-1}\mathbf{X}^{\lambda_{p}\left(\mathbf{m}\right)}\mathbf{A}\mathbf{X}^{-\lambda_{p}\left(\mathbf{m}\right)}e^{-2\pi i\mathbf{t}\cdot\mathbf{\mathbf{m}}} \end{equation} Hopefully, the reader should not be surprised that we evaluate the $\mathbf{m}$-sum using the recursive method. This time around, we define: \begin{equation} S_{n}\left(\mathbf{t}\right)\overset{\textrm{def}}{=}\frac{1}{p^{rn}}\sum_{\mathbf{m}=\mathbf{0}}^{p^{n}-1}\mathbf{X}^{\lambda_{p}\left(\mathbf{m}\right)}\mathbf{A}\mathbf{X}^{-\lambda_{p}\left(\mathbf{m}\right)}e^{-2\pi i\mathbf{t}\cdot\mathbf{m}}\label{eq:Definition of S_n for I_H computation} \end{equation} Then: \begin{align*} S_{n}\left(\mathbf{t}\right) & =\frac{1}{p^{rn}}\sum_{\mathbf{j}=\mathbf{0}}^{p-1}\sum_{\mathbf{m}=\mathbf{0}}^{p^{n-1}-1}\mathbf{X}^{\lambda_{p}\left(p\mathbf{m}+\mathbf{j}\right)}\mathbf{A}\mathbf{X}^{-\lambda_{p}\left(p\mathbf{m}+\mathbf{j}\right)}e^{-2\pi i\mathbf{t}\cdot\left(p\mathbf{m}+\mathbf{j}\right)}\\ & =\mathbf{X}\underbrace{\left(\frac{1}{p^{r\left(n-1\right)}}\sum_{\mathbf{m}=\mathbf{0}}^{p^{n-1}-1}\mathbf{X}^{\lambda_{p}\left(\mathbf{m}\right)}\mathbf{A}\mathbf{X}^{-\lambda_{p}\left(\mathbf{m}\right)}e^{-2\pi i\left(\mathbf{m}\cdot p\mathbf{t}\right)}\right)}_{S_{n-1}\left(p\mathbf{t}\right)}\mathbf{X}^{-1}\frac{1}{p^{r}}\sum_{\mathbf{j}=\mathbf{0}}^{p-1}e^{-2\pi i\mathbf{t}\cdot\mathbf{j}} \end{align*} So: \begin{equation} S_{n}\left(\mathbf{t}\right)=\mathbf{X}\left(S_{n-1}\left(p\mathbf{t}\right)\right)\mathbf{X}^{-1}\frac{1}{p^{r}}\sum_{\mathbf{j}=\mathbf{0}}^{p-1}e^{-2\pi i\mathbf{t}\cdot\mathbf{j}}\label{eq:S_n recursion identity for I_H computation} \end{equation} Nesting this yields: \begin{equation} S_{n}\left(\mathbf{t}\right)=\mathbf{X}^{n}\left(S_{0}\left(p^{n}\mathbf{t}\right)\right)\mathbf{X}^{-n}\frac{1}{p^{rn}}\sum_{\mathbf{j}_{1},\ldots,\mathbf{j}_{n}\leq p-1}e^{-2\pi i\mathbf{t}\cdot\sum_{k=1}^{n}\mathbf{j}_{k}p^{k}} \end{equation} Because: \begin{equation} S_{0}\left(\mathbf{s}\right)=\mathbf{X}^{\lambda_{p}\left(\mathbf{0}\right)}\mathbf{A}\mathbf{X}^{-\lambda_{p}\left(\mathbf{0}\right)}e^{-2\pi i\mathbf{s}\cdot\mathbf{0}}=\mathbf{A} \end{equation} we are then left with: \begin{equation} S_{n}\left(\mathbf{t}\right)=\frac{1}{p^{rn}}\mathbf{X}^{n}\mathbf{A}\mathbf{X}^{-n}\sum_{\left|\mathbf{J}\right|=n}e^{-2\pi i\mathbf{t}\cdot\mathbf{J}} \end{equation} Finally, recall that the set of all $p$-block strings of length $n$ is in a bijective correspondence with the set of all $r$-tuples $\mathbf{k}\in\mathbb{N}_{0}^{r}$ satisfying $\mathbf{k}\leq p^{n}-1$. As such, we can write: \begin{equation} \sum_{\left|\mathbf{J}\right|=n}e^{-2\pi i\mathbf{t}\cdot\mathbf{J}}=\sum_{\mathbf{k}=\mathbf{0}}^{p^{n}-1}e^{-2\pi i\mathbf{t}\cdot\mathbf{k}} \end{equation} and hence: \begin{equation} S_{n}\left(\mathbf{t}\right)=\frac{1}{p^{rn}}\mathbf{X}^{n}\mathbf{A}\mathbf{X}^{-n}\sum_{\mathbf{k}=\mathbf{0}}^{p^{n}-1}e^{-2\pi i\mathbf{t}\cdot\mathbf{k}}\label{eq:Explicit formula for S_n for I_H computation} \end{equation} Putting everything together yields: \begin{align*} \int_{\mathbb{Z}_{p}^{r}}\mathbf{I}_{H}\left(\lambda_{p}\left(\left[\mathbf{z}\right]_{p^{n}}\right)\right)e^{-2\pi i\left\{ \mathbf{t}\mathbf{z}\right\} _{p}}d\mathbf{z} & =\mathbf{1}_{\mathbf{0}}\left(\mathbf{t}\right)\mathbf{I}_{d}-\mathbf{X}^{n}\mathbf{A}\mathbf{X}^{-n}\frac{1}{p^{rn}}\sum_{\mathbf{k}=\mathbf{0}}^{p^{n}-1}e^{-2\pi i\mathbf{t}\cdot\mathbf{k}}\\ & =\mathbf{1}_{\mathbf{0}}\left(\mathbf{t}\right)\mathbf{I}_{d}-\mathbf{X}^{n}\mathbf{A}\mathbf{X}^{-n}\hat{K}_{n}\left(\mathbf{t}\right)\\ & =\mathbf{1}_{\mathbf{0}}\left(\mathbf{t}\right)\mathbf{I}_{d}-\mathcal{C}_{H}\left(\mathbf{A}:n\right)\hat{K}_{n}\left(\mathbf{t}\right) \end{align*} Q.E.D. \vphantom{} The way forward is then like so: we start our computation with: \begin{equation} \hat{\mathcal{L}}_{H,1,n}\left\{ \hat{A}_{H}\right\} \left(\mathbf{t}\right)=\hat{\mathcal{T}}_{n}\left\{ \hat{A}_{H}\right\} \left(\mathbf{t}\right)\left(\mathbf{I}_{d}-H^{\prime}\left(\mathbf{0}\right)\right)\left(H^{\prime}\left(\mathbf{0}\right)\right)^{-n} \end{equation} Using \textbf{Lemma \ref{lem:T_m hat of A_H hat}}, we get: \begin{align*} \hat{\mathcal{L}}_{H,1,n}\left\{ \hat{A}_{H}\right\} \left(\mathbf{t}\right) & =\sum_{m=0}^{n-1}\mathbf{1}_{\mathbf{0}}\left(p^{m}\mathbf{t}\right)\left(\prod_{k=0}^{m-1}\alpha_{H}\left(p^{k}\mathbf{t}\right)\right)\left(H^{\prime}\left(\mathbf{0}\right)\right)^{-n}\left(\mathbf{I}_{d}-H^{\prime}\left(\mathbf{0}\right)\right)\\ & +\left(\prod_{k=0}^{n-1}\alpha_{H}\left(p^{k}\mathbf{t}\right)\right)\left(H^{\prime}\left(\mathbf{0}\right)\right)^{-n} \end{align*} Next: \begin{equation} \hat{\mathcal{L}}_{H,2,n}\left\{ \hat{A}_{H}\right\} \left(\mathbf{t}\right)=\mathcal{L}_{H,1,n}\left\{ \hat{A}_{H}\right\} \left(\mathbf{t}\right)-\mathcal{L}_{H,1,n-1}\left\{ \hat{A}_{H}\right\} \left(\mathbf{t}\right)\left(H^{\prime}\left(\mathbf{0}\right)\right)^{-1} \end{equation} and so: \begin{align*} \hat{\mathcal{L}}_{H,2,n}\left\{ \hat{A}_{H}\right\} \left(\mathbf{t}\right) & =\mathbf{1}_{\mathbf{0}}\left(p^{n-1}\mathbf{t}\right)\left(\prod_{k=0}^{n-2}\alpha_{H}\left(p^{k}\mathbf{t}\right)\right)\left(H^{\prime}\left(\mathbf{0}\right)\right)^{-n}\left(\mathbf{I}_{d}-H^{\prime}\left(\mathbf{0}\right)\right)\\ & +\left(\prod_{k=0}^{n-1}\alpha_{H}\left(p^{k}\mathbf{t}\right)\right)\left(H^{\prime}\left(\mathbf{0}\right)\right)^{-n}-\left(\prod_{k=0}^{n-2}\alpha_{H}\left(p^{k}\mathbf{t}\right)\right)\left(H^{\prime}\left(\mathbf{0}\right)\right)^{-n}\\ & =\mathbf{1}_{\mathbf{0}}\left(p^{n-1}\mathbf{t}\right)\left(\prod_{k=0}^{n-2}\alpha_{H}\left(p^{k}\mathbf{t}\right)\right)\left(H^{\prime}\left(\mathbf{0}\right)\right)^{-n}\left(\mathbf{I}_{d}-H^{\prime}\left(\mathbf{0}\right)\right)\\ & +\left(\prod_{k=0}^{n-2}\alpha_{H}\left(p^{k}\mathbf{t}\right)\right)\left(\alpha_{H}\left(p^{n-1}\mathbf{t}\right)-\mathbf{I}_{d}\right)\left(H^{\prime}\left(\mathbf{0}\right)\right)^{-n} \end{align*} Then: \begin{equation} \hat{\mathcal{L}}_{H,3,n}\left\{ \hat{A}_{H}\right\} \left(\mathbf{t}\right)=\hat{\mathcal{E}}_{0}\left\{ \hat{A}_{H}\right\} \left(\mathbf{t}\right)+\sum_{m=1}^{n}\hat{\mathcal{L}}_{H,2,m}\left\{ \hat{A}_{H}\right\} \left(\mathbf{t}\right) \end{equation} where the $m$-sum is $\mathbf{O}_{d}$ when $n=0$. Recalling that: \begin{equation} \mathcal{E}_{0}\left\{ \tilde{A}_{H}\right\} \left(\mathbf{z}\right)=\mathbf{I}_{d}-\alpha_{H}\left(\mathbf{0}\right) \end{equation} we then have: \begin{equation} \hat{\mathcal{E}}_{0}\left\{ \hat{A}_{H}\right\} \left(\mathbf{t}\right)=\left(\mathbf{I}_{d}-\alpha_{H}\left(\mathbf{0}\right)\right)\mathbf{1}_{\mathbf{0}}\left(\mathbf{t}\right) \end{equation} which gives: \begin{align*} \hat{\mathcal{L}}_{H,3,n}\left\{ \hat{A}_{H}\right\} \left(\mathbf{t}\right) & =\left(\mathbf{I}_{d}-\alpha_{H}\left(\mathbf{0}\right)\right)\mathbf{1}_{\mathbf{0}}\left(\mathbf{t}\right)\\ & +\sum_{m=1}^{n}\mathbf{1}_{\mathbf{0}}\left(p^{m-1}\mathbf{t}\right)\left(\prod_{k=0}^{m-2}\alpha_{H}\left(p^{k}\mathbf{t}\right)\right)\left(H^{\prime}\left(\mathbf{0}\right)\right)^{-m}\left(\mathbf{I}_{d}-H^{\prime}\left(\mathbf{0}\right)\right)\\ & +\sum_{m=1}^{n}\left(\prod_{k=0}^{m-2}\alpha_{H}\left(p^{k}\mathbf{t}\right)\right)\left(\alpha_{H}\left(p^{m-1}\mathbf{t}\right)-\mathbf{I}_{d}\right)\left(H^{\prime}\left(\mathbf{0}\right)\right)^{-m} \end{align*} where the $k$-products are $\mathbf{I}_{d}$ whenever $m=1$. \begin{equation} \prod_{k=0}^{n-1}\alpha_{H}\left(p^{k}\mathbf{t}\right)=\begin{cases} \hat{A}_{H}\left(\mathbf{t}\right)\left(\alpha_{H}\left(\frac{\mathbf{t}\left|\mathbf{t}\right|_{p}}{p}\right)\right)^{-1} & \textrm{if }n=-v_{p}\left(\mathbf{t}\right)-1\\ \hat{A}_{H}\left(\mathbf{t}\right)\left(\alpha_{H}\left(\mathbf{0}\right)\right)^{n+v_{p}\left(\mathbf{t}\right)} & \textrm{if }n\geq-v_{p}\left(\mathbf{t}\right) \end{cases} \end{equation} Finally, note that: \begin{equation} \mathcal{L}_{H,4,n}\left\{ \tilde{A}_{H}\right\} \left(\mathbf{z}\right)=\mathcal{L}_{H,3,n}\left\{ \tilde{A}_{H}\right\} \left(\mathbf{z}\right)\left(\mathbf{I}_{H}\left(\lambda_{p}\left(\left[\mathbf{z}\right]_{p^{n}}\right)\right)\right)^{-1}\left(H^{\prime}\left(\mathbf{0}\right)\right)^{n} \end{equation} Since the Fourier transform turns multiplication into convolution, we get: \begin{equation} \hat{\mathcal{L}}_{H,4,n}\left\{ \hat{A}_{H}\right\} \left(\mathbf{t}\right)=\left(\hat{\mathcal{L}}_{H,3,n}\left\{ \hat{A}_{H}\right\} *\hat{\mathbf{G}}_{n}^{-1}\right)\left(\mathbf{t}\right)\left(H^{\prime}\left(\mathbf{0}\right)\right)^{n} \end{equation} where $\hat{\mathbf{G}}_{n}^{-1}$ denotes the convolution inverse of the Fourier transform of: \begin{equation} \mathbf{G}_{n}\left(\mathbf{z}\right)\overset{\textrm{def}}{=}\mathbf{I}_{H}\left(\lambda_{p}\left(\left[\mathbf{z}\right]_{p^{n}}\right)\right)\label{eq:Definition of bold G_n} \end{equation} It is here that we hit a rocky obstacle. Because $\hat{\mathbf{G}}_{n}^{-1}\left(\mathbf{t}\right)$ does not appear to have a manageably simple explicit formula, determining whether or not the sum: \begin{equation} \hat{\mathcal{L}}_{H}\left\{ \hat{A}_{H}\right\} \left(\mathbf{t}\right)=\sum_{n=0}^{\infty}\hat{\mathcal{L}}_{H,4,n}\left\{ \hat{A}_{H}\right\} \left(\mathbf{t}\right)\label{eq:L_H hat of A_H hat} \end{equation} converges with respect to $\mathcal{F}_{p,q_{H}}^{d,d}$ becomes problematic. Without an explicit formula with which convergence of the right-hand side of (\ref{eq:L_H hat of A_H hat}) could be directly ascertained, it would seem we are at the mercy of foremost weakness of fledgling frame theory: the lack of a method of \emph{indirectly }verifying convergence with respect to a given frame by way of estimates, approximation arguments, and the like. So, while it is intuitively clear that (\ref{eq:L_H hat of A_H hat}) should then furnish a formula for $\hat{\psi}_{H}\left(\mathbf{t}\right)$ in the case of non-commutative $H$ for which $\mathbf{I}_{d}-\alpha_{H}\left(\mathbf{0}\right)$ is invertible, at the time of this writing\textemdash the third day of Putin's invasion of Ukraine (26 February, 2022)\textemdash I do not have an argument to prove this rigorously. If and when such an argument is obtained, it would immediately follow by \textbf{Theorem \ref{thm:MD F-series for Chi_H}}'s formula $\chi_{H}\left(\mathbf{z}\right)=\psi_{H}\left(\mathbf{z}\right)\beta_{H}\left(\mathbf{0}\right)+\Psi_{H}\left(\mathbf{z}\right)$ that $\chi_{H}$ would be quasi-integrable. As such, I end with a conjecture: \begin{conjecture}[\textbf{Quasi-Integrability of $\chi_{H}$ in the Non-Commutative Case}] \label{conj:MD Non-Commutative case conjecture}Let $H$ be as given in \textbf{\emph{Theorem \ref{thm:MD F-series for Chi_H}}}, and suppose that $\mathbf{I}_{d}-\alpha_{H}\left(\mathbf{0}\right)$ is invertible. Then,\emph{ (\ref{eq:L_H hat of A_H hat})} gives a formula $\hat{\mathbb{Z}}_{p}^{r}\rightarrow\overline{\mathbb{Q}}^{d,d}$ for a Fourier transform of $\psi_{H}$ with respect to the standard $\left(p,q_{H}\right)$-adic frame. \end{conjecture} \chapter*{Coda} \pagestyle{fancy}\fancyfoot{}\fancyhead[L]{\sl CODA}\fancyhead[R]{\thepage}\addcontentsline{toc}{chapter}{Coda} \includegraphics[scale=0.4]{./PhDDissertationEroicaCoda.png} \section*{Conclusion - A Call to Arms} \addcontentsline{toc}{section}{Conclusion - A Call to Arms} Even though insight drives mathematical progress, the utility and efficacy of mathematics is only knowable in hindsight. Will the methods I have presented in my dissertation be of any use in solving the Collatz Conjecture once and for all? The answer\textemdash if it is to ever be found\textemdash lies in posterity. I would very much like to meet it, if I can. In writing this dissertation, the two biggest surprises were the connection to eigenvalues of matrices in Sub-subsection \ref{subsec:A-Matter-of}\textemdash although, in hindsight this feels obvious (of course)\textemdash and, most of all, the result that divergent trajectories of $H$ were associated with values in $\mathbb{Z}$ attained by $\chi_{H}$ over $\mathbb{Z}_{p}\backslash\mathbb{Q}$ (\textbf{Theorem \ref{thm:Divergent trajectories come from irrational z}} on page \pageref{thm:Divergent trajectories come from irrational z}, and \textbf{Theorem \ref{thm:MD Divergent trajectories come from irrational z}} on page \pageref{thm:MD Divergent trajectories come from irrational z}). I only discovered $\chi_{H}$'s involvement in divergent trajectories in February 2022 while I was beginning my polishing and editing runs on this monograph. Prior to that, I was firmly convinced that my methods had nothing to say about divergent trajectories. For once, I was \emph{thrilled} to be proven wrong. Regarding the theory of Hydra maps as I have presented it, the most significant outstanding issue are the \textbf{Conjectures} \textbf{\ref{conj:correspondence theorem for divergent trajectories}} (page \pageref{conj:correspondence theorem for divergent trajectories}) and \textbf{\ref{conj:MD correspondence theorem for divergent trajectories}} (page \pageref{conj:MD correspondence theorem for divergent trajectories}) regarding a Correspondence Principle for divergent trajectories. If these Conjectures could be proven true, the \textbf{Tauberian Spectral Theorems} (pages \pageref{thm:Periodic Points using WTT} \& \pageref{thm:MD Periodic Points using WTT}) for $\chi_{H}$ would then \emph{completely} characterize the dynamics of the Hydra maps under consideration\textemdash that is, both periodic points \emph{and }divergent trajectories. The entire matter of the Collatz Conjecture (not to mention most any comparable conjectures for other Hydra maps) would then be reduced to the study of the density of translates of $\hat{\chi}_{H}\left(t\right)-x\mathbf{1}_{0}\left(t\right)$ in $c_{0}\left(\hat{\mathbb{Z}}_{p},\mathbb{C}_{q}\right)$ and its multi-dimensional analogue. At a lesser level, there are the matters of one-dimensional $p^{n}$-Hydra maps (where $p$ is a prime and $n\in\mathbb{N}_{1}$) and multi-dimensional $P$-Hydra maps where $P=\left(p^{n_{1}},\ldots,p^{n_{r}}\right)$ where $p$ is prime and $n_{1}\leq\ldots\leq n_{r}$ is a non-decreasing sequence of positive integers. While I certainly \emph{like} to think that my Tauberian Spectral reformulation of Hydra maps' dynamics is aesthetically pleasing, I have yet to explore whether or not it is actually \emph{useful}. The question of the density of the translates of $\hat{\chi}_{H}\left(t\right)-x\mathbf{1}_{0}\left(t\right)$ might very well turn out to be just as intractable as the original question of whether or not $x$ is a periodic point or divergent trajectory of $H$. Although, obviously, I am biased in favor of believing that my work \emph{will} turn out to be useful, I cannot help but feel that the distinctions between, say, Fourier transforms of the Shortened $3x+1$ map (for which $\alpha_{H}\left(0\right)=1$) and those of the Shortened $5x+1$ map (for which $\alpha_{H}\left(0\right)\neq1$) have the look of a smoking gun. For reference, valid choices of Fourier transforms for these numina are: \begin{equation} \hat{\chi}_{3}\left(t\right)=\begin{cases} -\frac{1}{2} & \textrm{if }t=0\\ \frac{1}{4}v_{2}\left(t\right)\hat{A}_{3}\left(t\right) & \textrm{if }t\neq0 \end{cases},\textrm{ }\forall t\in\hat{\mathbb{Z}}_{2} \end{equation} \begin{equation} \hat{\chi}_{5}\left(t\right)=\begin{cases} -\frac{1}{2} & \textrm{if }t=0\\ -\frac{1}{4}\hat{A}_{5}\left(t\right) & \textrm{if }t\neq0 \end{cases}=-\frac{1}{4}\mathbf{1}_{0}\left(t\right)-\frac{1}{4}\hat{A}_{5}\left(t\right),\textrm{ }\forall t\in\hat{\mathbb{Z}}_{2} \end{equation} Because $v_{2}\left(t\right)$ takes values in $\left\{ -1,-2,-3,\ldots\right\} $ for $t\in\hat{\mathbb{Z}}_{2}\backslash\left\{ 0\right\} $, the $3$-adic absolute value of $\hat{\chi}_{3}\left(t\right)$ will become arbitrarily small infinitely often: \begin{equation} \liminf_{\left|t\right|_{2}\rightarrow\infty}\left|\hat{\chi}_{3}\left(t\right)\right|_{3}=0 \end{equation} whereas: \begin{equation} \left|\hat{\chi}_{5}\left(t\right)\right|_{5}=1,\textrm{ }\forall t\in\hat{\mathbb{Z}}_{2} \end{equation} Unlike the probabilistic heuristics usually given to argue that $3x+1$ sends all positive integers to $1$ or that $5x+1$ sends almost all positive integers to $\infty$, this observation about the behaviors of $\hat{\chi}_{3}$ and $\hat{\chi}_{5}$ holds with \emph{absolute} certainty. This is but one reason for my my conviction that $\hat{\chi}_{3}\left(t\right)$'s failure to be $3$-adically bounded away from $0$ \emph{must }be a key feature in any proof of the Collatz Conjecture, should one arise. To that end, it would be wonderful if we could establish something along the lines of: \begin{conjecture} \label{conj:Implication of Tauberian Spectral Theorem}Let $H:\mathbb{Z}\rightarrow\mathbb{Z}$ be a contracting, semi-basic $p$-Hydra map, and let $\hat{\chi}_{H}$ be a Fourier transform of $\chi_{H}$. \vphantom{} I. $H$ has finitely many periodic points if and only if $\liminf_{\left|t\right|_{p}\rightarrow\infty}\left|\hat{\chi}_{H}\left(t\right)\right|_{q_{H}}=0$. \vphantom{} II. If $\liminf_{\left|t\right|_{p}\rightarrow\infty}\left|\hat{\chi}_{H}\left(t\right)\right|_{q_{H}}>0$, then $H$ has a divergent trajectory. \end{conjecture} \vphantom{} While the particulars of the hypotheses and conclusions of this conjecture might very well need to be tinkered with, my hope in making it will be that questions about the dynamics of $H$ can be reduced to number-theoretic ($q$-adic) properties of the formulae for $\hat{\chi}_{H}$. \begin{example}[\textbf{Matthews' Map, Revisited}] \index{Matthews' Conjecture}Of special interest is the $3$-Hydra map obtained by conjugating the map featured in \textbf{Matthews' Conjecture} (\textbf{Conjecture \ref{conj:Matthews conjecture}} on page \pageref{conj:Matthews conjecture}); specifically, the conjugated version $\tilde{M}$ defined in equation (\ref{eq:Matthews' Conjecture Map, conjugated}) on page \pageref{exa:Matthews' map}: \begin{equation} \tilde{M}\left(n\right)=\begin{cases} \frac{n}{3} & \textrm{if }n\overset{3}{\equiv}0\\ 7n-3 & \textrm{if }n\overset{3}{\equiv}1\\ \frac{7n-2}{3} & \textrm{if }n\overset{3}{\equiv}2 \end{cases} \end{equation} The reason this map is of interest, recall, is because it is easily proved that every integer congruent to $1$ mod $3$ belongs to a divergent trajectory of $\tilde{M}$ (a simple modification of page \pageref{prop:Matthews' map}'s \textbf{Proposition \ref{prop:Matthews' map}}). Though \emph{non-integral}, this $3$-Hydra is nevertheless semi-basic (with $\mu_{0}=1$, $\mu_{1}=21$, $\mu_{2}=7$, and $q=7$) and so it possesses a numen $\chi_{\tilde{M}}$, and every periodic point of $\tilde{M}$ is of the form $\chi_{\tilde{M}}\left(B_{3}\left(n\right)\right)$ for some $n\in\mathbb{N}_{1}$. However, because $\tilde{M}$ is non-integrable, the versions of the Correspondence Principal established in this dissertation do not apply to conclude that every value in $\mathbb{Z}$ attained by $\chi_{\tilde{M}}$ over $\mathbb{Q}\cap\mathbb{Z}_{3}^{\prime}$ is a periodic point of $\tilde{M}$, or that $\mathfrak{z}\in\mathbb{Z}_{3}\backslash\mathbb{Q}$ for which $\chi_{\tilde{M}}\left(\mathfrak{z}\right)\in\mathbb{Z}$ necessarily make $\chi_{\tilde{M}}\left(\mathfrak{z}\right)$ into a divergent point of $\tilde{M}$. Nevertheless, I have not explored this issue in detail\textemdash my goal in my dissertation was to obtain as broad a theory as possible, rather than whittle away at any particular Hydra map\textemdash so there may still be a way to make these results applicable to $\tilde{M}$. Despite these obstacles, nothing prevents us from going through with Chapter 4's Fourier analysis of $\chi_{\tilde{M}}$. Doing so, we find that: \begin{equation} \alpha_{\tilde{M}}\left(t\right)\overset{\textrm{def}}{=}\frac{1+21e^{-2\pi it}+7e^{-4\pi it}}{9},\textrm{ }\forall t\in\hat{\mathbb{Z}}_{3}\label{eq:alpha for conjugated matthews map} \end{equation} \begin{equation} \beta_{\tilde{M}}\left(t\right)\overset{\textrm{def}}{=}e^{-2\pi it}-\frac{2}{9}e^{-4\pi it},\textrm{ }\forall t\in\hat{\mathbb{Z}}_{3}\label{eq:Beta for conjugated Matthews map} \end{equation} This gives: \begin{align} \alpha_{\tilde{M}}\left(0\right) & =\frac{29}{9}\\ \beta_{\tilde{M}}\left(0\right) & =\frac{7}{9} \end{align} with: \begin{equation} \hat{A}_{\tilde{M}}\left(t\right)\overset{\textrm{def}}{=}\begin{cases} 1 & \textrm{if }t=0\\ \prod_{n=0}^{-v_{3}\left(t\right)-1}\frac{1+21e^{-2\pi i3^{m}t}+7e^{-4\pi i3^{m}t}}{9} & \textrm{else} \end{cases},\textrm{ }\forall t\in\hat{\mathbb{Z}}_{3}\label{eq:A_H-hat for Matthews map} \end{equation} and: \begin{equation} \kappa_{\tilde{M}}\left(n\right)=\left(21\right)^{\#_{3:1}\left(n\right)}7^{\#_{3:2}\left(n\right)}\label{eq:Kappa_H for matthews map} \end{equation} Using \textbf{Theorem \ref{thm:F-series for an arbitrary 1D Chi_H}} (page \pageref{thm:F-series for an arbitrary 1D Chi_H}), a Fourier transform for $\chi_{\tilde{M}}:\mathbb{Z}_{3}\rightarrow\mathbb{Z}_{7}$ is given by: \begin{equation} \hat{\chi}_{\tilde{M}}\left(t\right)=\begin{cases} -\frac{7}{20} & \textrm{if }t=0\\ \left(\frac{9e^{-2\pi i\frac{t\left|t\right|_{3}}{3}}-2e^{-4\pi i\frac{t\left|t\right|_{3}}{3}}}{1+21e^{-2\pi i\frac{t\left|t\right|_{3}}{3}}+7e^{-4\pi i\frac{t\left|t\right|_{3}}{3}}}-\frac{7}{20}\right)\hat{A}_{\tilde{M}}\left(t\right) & \textrm{if }t\neq0 \end{cases}\label{eq:Fourier Transform for Chi_H for Matthews map} \end{equation} Here, number theory comes into play. Noting that: \begin{equation} \alpha_{\tilde{M}}\left(t\right)=\frac{1+\overbrace{21e^{-2\pi it}+7e^{-4\pi it}}^{\textrm{a multiple of }7}}{9}\in\frac{1}{9}+7\mathbb{C}_{7},\textrm{ }\forall t\in\hat{\mathbb{Z}}_{3} \end{equation} the $7$-adic ultrametric inequality yields: \begin{equation} \left|\alpha_{\tilde{M}}\left(t\right)\right|_{7}=1,\textrm{ }\forall t\in\hat{\mathbb{Z}}_{3} \end{equation} As such: \begin{equation} \left|\hat{A}_{\tilde{M}}\left(t\right)\right|_{7}=\prod_{n=0}^{-v_{3}\left(t\right)-1}\left|\alpha_{\tilde{M}}\left(3^{n}t\right)\right|_{7}=1,\textrm{ }\forall t\in\hat{\mathbb{Z}}_{3} \end{equation} So, the $7$-adic absolute value of $\hat{\chi}_{\tilde{M}}\left(t\right)$ for non-zero $t$ is entirely determined by the function: \begin{equation} \frac{9e^{-2\pi i\frac{t\left|t\right|_{3}}{3}}-2e^{-4\pi i\frac{t\left|t\right|_{3}}{3}}}{1+21e^{-2\pi i\frac{t\left|t\right|_{3}}{3}}+7e^{-4\pi i\frac{t\left|t\right|_{3}}{3}}}-\frac{7}{20}\label{eq:7-adic magnitude determinator for Chi_H for Matthews map} \end{equation} Because $t\left|t\right|_{3}/3$ is congruent mod $1$ to either $1/3$ (if the numerator of $t$ is $1$ mod $3$) or $2/3$ (if the numerator of $t$ is $2$ mod $3$) for any $t\in\hat{\mathbb{Z}}_{3}\backslash\left\{ 0\right\} $, (\ref{eq:7-adic magnitude determinator for Chi_H for Matthews map}) takes precisely two values on $\hat{\mathbb{Z}}_{3}\backslash\left\{ 0\right\} $. Letting $\xi$ denote $e^{2\pi i/6}$, and letting $\zeta$ denote $\xi^{2}$, these are: \begin{equation} \frac{9e^{-2\pi i\frac{t\left|t\right|_{3}}{3}}-2e^{-4\pi i\frac{t\left|t\right|_{3}}{3}}}{1+21e^{-2\pi i\frac{t\left|t\right|_{3}}{3}}+7e^{-4\pi i\frac{t\left|t\right|_{3}}{3}}}-\frac{7}{20}=\begin{cases} \frac{17-47\zeta^{2}}{1580} & \textrm{if }t\left|t\right|_{3}\overset{3}{\equiv}1\\ \frac{17-47\zeta}{1580} & \textrm{if }t\left|t\right|_{3}\overset{3}{\equiv}2 \end{cases} \end{equation} Recalling that the torsion subgroup of $\mathbb{Z}_{7}^{\times}$ is isomorphic to $\mathbb{Z}/6\mathbb{Z}$, all $6$th roots of unity are occur naturally as $7$-adic integers. As such, to compute the $7$-adic absolute values of these expressions, we will need to fall back on our convention that, for an odd prime $p$, $\xi\in\mathbb{Z}_{p}$ denotes the unique primitive $\left(p-1\right)$th root of unity whose first $p$-adic digit is the smallest primitive root of unity in $\mathbb{Z}/p\mathbb{Z}$. Since $p=7$, that digit is $3$. Consequently, $\zeta=\xi^{2}$ has $3^{2}\overset{7}{\equiv}2$ as its first $7$-adic digit and $\zeta^{2}=\xi^{4}$ has $3^{4}\overset{7}{\equiv}4$ as its first $7$-adic digit. Observing the identity: \begin{equation} \left(17-47\zeta^{2}\right)\left(17-47\zeta\right)=\left(17\right)^{2}+\left(47\right)^{2}+17\cdot47=3297=3\cdot7\cdot157 \end{equation} we have that: \begin{equation} 17-47\zeta^{2}\overset{7}{\equiv}17-\left(47\right)\left(4\right)\overset{7}{\equiv}3-\left(5\right)\left(2\right)=-17\overset{7}{\equiv}4 \end{equation} So, $\left|17-47\zeta^{2}\right|_{7}=1$. As such: \begin{equation} \left|\left(17-47\zeta^{2}\right)\left(17-47\zeta\right)\right|_{7}=\frac{1}{7} \end{equation} then forces: \begin{equation} \left|17-47\zeta\right|_{7}=\frac{1}{7} \end{equation} Finally, noting that: \begin{equation} 1580=2^{2}\cdot5\cdot79 \end{equation} we have: \begin{align} \left|\frac{17-47\zeta}{1580}\right|_{7} & =\frac{1}{7}\\ \left|\frac{17-47\zeta^{2}}{1580}\right|_{7} & =1 \end{align} Since $\left|\hat{\chi}_{\tilde{M}}\left(0\right)\right|_{7}=\left|-7/20\right|_{7}=1/7$, we then conclude that: \begin{equation} \left|\hat{\chi}_{\tilde{M}}\left(t\right)\right|_{7}=\begin{cases} \frac{1}{7} & \textrm{if }t=0\\ 1 & \textrm{if }t\left|t\right|_{3}\overset{3}{\equiv}1\\ \frac{1}{7} & \textrm{if }t\left|t\right|_{3}\overset{3}{\equiv}2 \end{cases},\textrm{ }\forall t\in\hat{\mathbb{Z}}_{7}\label{eq:7-adic absolute value of the fourier transform of the numen of Matthews map} \end{equation} With this, we see that $\chi_{\tilde{M}}$ is more like $\chi_{q}$ for $q\geq5$, in that its $7$-adic absolute value is bounded away from $0$. This is consistent with \textbf{Conjecture \ref{conj:Implication of Tauberian Spectral Theorem}}\textemdash or, at least, with this particular formulation of the Conjecture. As mentioned above, the precise statement of that Conjecture is still subject to revision; more specific cases should be investigated to help see what the ``correct'' conjecture might be. Subject to the verification of \textbf{Conjecture \ref{conj:Implication of Tauberian Spectral Theorem}}, the method described above\textemdash checking $\hat{\chi}_{H}$ and the values of $\gamma_{H}\left(j/p\right)$ for $j\in\left\{ 1,\ldots,p-1\right\} $ over the $q$-adics\textemdash would then be an extremely elegant means for determining the dynamical properties of $H$. \end{example} \vphantom{} The two other most pressing issues I can think of are the \textbf{exploration of Hydra maps on fields of positive characteristic}, such as those presented near the end of Matthews' slides \cite{Matthews' slides}, and the \textbf{exploration of the polygenic case} as discussed in \textbf{Example \ref{exa:Polygenic example, part 1}} on page \pageref{exa:Polygenic example, part 1}. As mentioned in the remark given after \textbf{Example \ref{exa:Polygenic example, part 1}}, I believe it will be possible to use frames to put $\chi_{H}$ on rigorous footing in the polygenic case. Because the $N$th truncations of $\chi_{H}$ take values in $\mathbb{Q}$, the asymptotic analysis of $\chi_{H,N}$\textemdash untangling the link between $N$ and $t$\textemdash will yield Fourier transforms for polygenic $\chi_{H}$. The only possible issue I can foresee is in verifying that the resultant formulae for $\hat{\chi}_{H}$ form continuous linear functionals\textemdash although, there, the question becomes \emph{on what space?} Provided this can be answered in a straight-forward matter, the $\left(p,q\right)$-adic Wiener Tauberian Theorem for measures will almost certainly apply as in the monogenic case, thereby furnishing a Tauberian Spectral Theorem for polygenic $\chi_{H}$, seeing as how they already depend on the points at which the measure's Fourier series converge $q$-adically. Aside from these obvious next steps, it also seems worthwhile to explore if $L_{\mathbb{R}}^{1}$ can, in fact, be used as a ``base of operations'' for analytic investigations of quasi-integrability and frames. \subsection*{A Soapbox Moment} Before I begin the obligatory bibliographic essay, I would like to make a ``call to arms'' for fellow travelers in mathematical analysis and its many subspecialties. Although I am pleased to have broken new ground in non-archimedean analysis\textemdash without it, I would not have attained my hard-earned PhD\textemdash it troubles me that some of the phenomena I have discovered appear to haven't already been discovered, especially considering their astonishing simplicity. The identity (\ref{eq:Fourier sum of v_p of t}) (page \pageref{eq:Fourier sum of v_p of t}): \[ \sum_{0<\left|t\right|_{p}\leq p^{N}}v_{p}\left(t\right)e^{2\pi i\left\{ t\mathfrak{z}\right\} _{p}}\overset{\mathbb{Q}}{=}\frac{p\left|\mathfrak{z}\right|_{p}^{-1}-1}{p-1},\textrm{ }\forall N>v_{p}\left(\mathfrak{z}\right) \] requires no extraordinary machinery to prove. The computation is as near as mindless as can be, yet\textemdash to my bemusement\textemdash no one seems to have noticed it until now. The mathematical community's failure to notice this identity, as well as the implication it has for $\left(p,q\right)$-adic integration ($\left(p,q\right)$-adic Mellin transforms, distributional derivatives, etc.) cannot be due to a lack of genius or ingenuity; it can only be attributed to a lack of curiosity and an aversion to simplicity. It frustrates me to no end that this state of affairs is allowed to stand. I have abiding empathy for Vladimir Arnold's protests against contemporary mathematical pedagogy \cite{Arnold}. As the venerable Russian analyst put it: \begin{quote} Attempts to create ``pure'' deductive-axiomatic mathematics have led to the rejection of the scheme used in physics (observation - model - investigation of the model - conclusions - testing by observations) and its substitution by the scheme: definition - theorem - proof. \end{quote} I could not agree with this sentiment more strongly. The deductive-axiomatic scheme is a powerful organizational tool, but I feel it horribly misrepresents the reality of mathematical inquiry. In the research that led to my dissertation, I proceeded very much like a physicist of old, testing different ideas and formulas in the hopes of finding something that managed to hold water and say something useful about the objects of my investigations. Make no mistake: I am not a mindless iconoclast. The trend toward abstraction has and always will be a vital part of mathematical inquiry. In clearing the road of debris and obstacles, they make it easier to discern deep patterns and unexpected symmetries. But, it is my conviction that flights of fancy such as these need to be matched and tempered by good old-fashioned concrete play. We need to be able to get our hands dirty and loamy with tedious, self-limiting specificity. Euler spent a decade trying to prove his famous (and beautiful) generating function identity for pentagonal numbers. If only us moderns had the tenacity to give simple, specific questions that same level of consideration. Progress comes about when people look in places they shouldn't, and when ideas take root far from home. Theories stagnate when all their voices sound the same. Practical-minded analysts should not let topics like non-archimedean analysis fall by the wayside and become sole purview of the algebraists. The existence of the topological, open-set-based definition of continuity does not diminish the $\epsilon$-$\delta$ definition, nor render it obsolete. The same ought to be true of non-archimedean analysis and classical analysis. Just because a thing can be tied to the cross of commutative algebra does not mean there is nothing to be gained from an independent examination of the thing from a purely concrete, analytical perspective. Even if this dissertation of mine ends up being a mere tangent on the long road to Collatz, if it can succeed at enticing other scholars to consider topics or settings they might not have bothered to ever explore, I can say I will be happy with it, with my efforts, and with myself. \vphantom{} \begin{quote} \emph{Oh me! Oh life! of the questions of these recurring,} \emph{Of the endless trains of the faithless, of cities fill\textquoteright d with the foolish,} \emph{Of myself forever reproaching myself, (for who more foolish than I, and who more faithless?)} \emph{Of eyes that vainly crave the light, of the objects mean, of the struggle ever renew\textquoteright d,} \emph{Of the poor results of all, of the plodding and sordid crowds I see around me,} \emph{Of the empty and useless years of the rest, with the rest me intertwined,} \emph{The question, O me! so sad, recurring\textemdash What good amid these, O me, O life?} \begin{center} Answer. \par\end{center} \emph{That you are here\textemdash that life exists and identity,} \emph{That the powerful play goes on, and you may contribute a verse.} \textemdash Walt Whitman (1892) \end{quote} \newpage{} \section*{Bibliographic Essay} \addcontentsline{toc}{section}{Bibliographic Essay}As explained in the Preface, the $\left(p,q\right)$-adic analytic methods used in this dissertation were not the first path of attack I tried on the Collatz Conjecture, and I doubt they will be the last. To that end, I have not exercised much restraint with regard to the references catalogued in the Bibliography below; it is my hope that others will find the listed sources as interesting as I did. Like most bibliographic essays, the next few paragraphs are intended to assist the reader in assessing the resources I have left for them. For ease of access, I have organized my discussion by topic. \subsubsection*{Collatz Studies} At the time of writing, ``Collatz studies'' has yet to become a well-established mathematical discipline, and the scattered, amorphous state of the literature reflects this. Lagarias' \emph{Ultimate Challenge }book \cite{Ultimate Challenge} is an wonderful start, as are his annotated bibliographies of Collatz research \cite{Collatz Biography}. As of the first quarter of the twenty-first century, Lagarias is generally considered the top authority in Collatz studies, and to that end, all of his publications on the topic are worth reading (\cite{Applegate and Lagarias - Trees,Applegate and Lagarias - Difference inequalities,3x+1 semigroup,natural boundaries,Lagarias-Kontorovich Paper,Lagarias' Survey,Lagarias - rational cycles for 3x+1}). There are doubtless others to be found, should the reader spend the time searching for them. Matthews' \href{http://www.numbertheory.org/3x\%2B1/}{Collatz webpage}\footnote{http://www.numbertheory.org/3x+1/} website contains links to his slides and related articles (\cite{Matthews' Conjecture,Matthews' Leigh Article,Matthews' slides,Matthews and Watts}); these focus primarily on the Markov-chain approach. Also of note, the late Meinardus and Berg \cite{Meinardus and Berg,Meinardus} attempted analytic approaches using functional equations, as did their colleague G. Opfer (\cite{Berg =00003D000026 Opfer,Opfer}), with Opfer's 2011 paper \cite{Opfer} being notable as having been thought to have actually\emph{ proved }Collatz until a gap was discovered (see, for instance, \cite{de Weger on Opfer}). There is also my own (albeit somewhat messy) work in this vein \cite{Dreancatchers for Hydra Maps}. My bibliography only scratches the surface of the extant literature. Eric Roosendaal maintains a $3x+1$ website dedicated to chronicling various computational phenomena and statistics thereof found in the iteration of the Collatz map \cite{Roosendaal's Website}. Then, of course, there is Tao's recent work \cite{Tao Probability paper}. The reader should be aware that most Collatz studies tend to focus solely on the $3x+1$ map itself. One notable exception to this trend is \cite{RCWAG}, which is part of a computational package for studying more general Collatz-type maps\textemdash the ungainly (though accurately) termed ``residue-class-wise affine maps'' (RCWA)\textemdash all due to Stephan Kohl. \cite{RCWAG} is a freely available computational software for studying Collatz type maps, written using the GAP programming language. It strikes me as the sort of thing a person might fool around with\footnote{Particularly if it can be used to create attractive, eye-catching visualizations. As they say, a picture is worth a thousand words. It also makes for a very effective tool for marketing and recruitment.} while lounging about at home, either on a rainy day or a golden Sunday afternoon. \subsubsection*{Non-Archimedean Analysis (Including $\left(p,q\right)$)} W. M. Schikhof PhD dissertation \cite{Schikhof's Thesis} establishes the fundamentals of harmonic analysis in the general non-archimedean setting, including the $\left(p,q\right)$-adic case. A particularly valuable\textemdash and accessible!\textemdash introduction to non-archimedean analysis is Schikhof's \emph{Ultrametric Calculus }\cite{Ultrametric Calculus}, a classic of the subject. The bulk of it is focused on $\left(p,p\right)$-adic analysis, though. A treatment of the Monna-Springer integral (though not by that name) is given in one of \emph{Ultrametric Calculus'} appendices. A \emph{much }more exhaustive collection of non-archimedean material can be found in van Rooij's \emph{Non-Archimedean Functional Analysis }\cite{van Rooij - Non-Archmedean Functional Analysis}, though\textemdash as I have mentioned elsewhere\textemdash this book is out of print\footnote{It \emph{can}, however, be pirated.}. van Rooij's vantage is quite general, to the point that his book is a bit bewildering to work with. Those without photographic memories are advised to keep a glossary of his definitions and notations. Khrennikov's writings (especially the wild ride of a book in \cite{Quantum Paradoxes}) are very much worth the read, both for sheer entertainment value, and because of their tendency to be more concrete than van Rooij's presentation or that of other scholars: Aguayo (\cite{Aguayo 1,Aguayo 2,Aguayo and Moraga radon-nikodym theorem paper}), and\textemdash most egregiously\textemdash Ludkovsky terribly dense writing (see \cite{Ludkovsky on non-archimedean measures}, for example). Khrennikov's co-authored paper \cite{Measure-theoretic approach to p-adic probability theory} on non-archimedean probability theory is also an excellent, self-contained introduction to the Monna-Springer integration technique presented here, supplemented with additional considerations regarding the formulation of measure-theoretic approaches to probability theory for probabilities taking values in non-archimedean fields. As a rule of thumb, much of the literature in non-archimedean analysis (as opposed to $p$-adic analysis proper) is couched in the language of abstract functional analysis. Some may find that register of presentation elegant or high-brow; I find it frustratingly un-welcoming, especially for newcomers. This is particularly important for the reader to keep in mind, seeing as the nature of the work I have done in my dissertation is foundationally \emph{concrete}. As mentioned in Subsection \ref{subsec:3.1.1 Some-Historical-and}, a would-be non-archimedean analyst should be aware of 'false friends' like \cite{Bosch lying title,Schneider awful book,More Schneider lies} which are works of algebraic geometry prancing about with analysis-sounding titles. \subsubsection*{$\left(p,\infty\right)$-Adic Analysis} An excellent reference for the methods of Fourier analysis for complex-valued functions on local fields is Taibleson's book \cite{Taibleson - Fourier Analysis on Local Fields}. I also recommend perusing the matter of harmonic analysis on locally compact groups. Jordan Bell's lecture notes \cite{Bell - Harmonic Analysis on the p-adics,Bell - Pontryagin Duals of Q/Z and Q} are a great resource for anyone who share my ``just shut up and tell me how to compute stuff!'' attitude and the lack of patience implicit therein. Folland's text on abstract harmonic analysis \cite{Folland - harmonic analysis} makes for an excellent chaser to introduce the theory as a whole\textemdash Pontryagin duality, connections to representation theory, and all. And although \cite{Automorphic Representations} is a text on the representation theory of the general linear group, it nevertheless presents a thorough account of the tools of $\left(p,\infty\right)$-adic analysis\textemdash Fourier transform \emph{and }Mellin transform\textemdash in its opening chapters. Tate's thesis \cite{Tate's thesis}, is of course, a classic, and a beautiful display of these analytical methods at work in a number-theoretic application. The connections with $p$-adic mathematical physics are also worth exploring, particularly for the reader interested in learning more about the nitty-gritty of integration and the theory of distributions. Vladimirov's article \cite{Vladimirov - the big paper about complex-valued distributions over the p-adics} contains a comprehensive account of $\left(p,\infty\right)$-adic distributions and the accompanying notion of distributional derivatives, going so far as to even solve some differential equations by this method. \cite{Real and p-Adic Oscillatory integrals,p-adic van der Corput lemma} deal with $\left(p,\infty\right)$-adic oscillatory integrals. \subsubsection*{$\left(p,p\right)$-Adic Analysis} After reading the first chapter of Gouvea's undergraduate-level text \cite{Gouvea's introudction to p-adic numbers book} in $\left(p,p\right)$-adic analysis, I used Robert's \emph{A course in $p$-adic analysis }(\cite{Robert's Book}) to acquaint myself with the ins and outs of the titular subject, along with a little help from the seminar in algebraic number theory taught at the University of Southern California by my eventual co-advisor\footnote{Along with Nicolai Haydn}, Prof. Sheldon Kamienny. Robert's book gives an excellent, balanced treatment of the subject; I never had the feeling that he had to tie and gag his inner number theorist or algebraic geometer to keep it from trying to monopolize the discussion. Curious undergraduates should probably peruse \cite{Gouvea's introudction to p-adic numbers book} before attempting Robert, if only for the pleasure of reading Gouvea's beautiful, lucid explanations. Koblitz's books \cite{Koblitz's book,Koblitz's other book} are more biased toward number theoretical concerns, but are still very accessible. More advanced\footnote{In other words, more \emph{algebraic}.} references include Kedlaya's book on $p$-adic differential equations \cite{Kedlaya}, and\textemdash diving straight in to the algebra and number theory\textemdash Iwasawa's classic treatise on $p$-adic $L$-functions \cite{Iwasawa} or Colmez's exposition of the same \cite{p-adic L-functions paper}, the chapter on $p$-adic distributions in Washington's \emph{Introduction to Cyclotomic Fields} \cite{Cyclotomic fields}. Readers with a desire to force-feed themselves the theory of Berkovitch spaces need only turn to \cite{Berkovich spaces and applications} and \cite{Bosch lying title} for algebraic geometry in all its haunting, malefic glory. On the analytical side, Anashi has a free e-text on $p$-adic ergodic theory \cite{Anashi - p-adic ergodic theory} and Adams' work on $p$-adic transcendental number theory contains an exposition of the $p$-adic analogue of contour integration (the \index{integral!Shnirelmann}\textbf{Shnirelmann integral}) \cite{Adams' paper on p-adic transcendental numbers with an appendix on Schnirelman integration}\textemdash though, to be clear, this is \emph{not }the same as the $p$-adic line integral used in rigid analytic spaces. Cherry's notes \cite{Cherry non-archimedean function theory notes} give an excellent introduction to the theory of analytic and meromorphic $\left(p,p\right)$-adic functions, especially for readers who don't have the patience for Robert's more completionist presentation, or Escassut more advanced, specialized treatment of the topic in \cite{value distribution in p-adic analysis}. George Brom's thesis \cite{pp adic Fourier theory} covers Fourier analysis in the $\left(p,p\right)$-adic setting, a fascinating oddity, seeing as\textemdash recall\textemdash there is no non-trivial translation-invariant $p$-adic-valued linear functional on the space of continuous functions $\mathbb{Z}_{p}\rightarrow\mathbb{C}_{p}$. \subsubsection*{Miscellany} For completeness' sake, I have included in the bibliography much of the material that I had on mind from 2017 to 2019, back when I was still trying to use complex analytic methods and their $\left(p,\infty\right)$-adic extensions to study Hydra maps. To reiterate, the guiding light of behind my investigations of those two years was the functional equation approach which I discovered independently of Meinardus and Berg. This line of research had me trying to learn as much as I could about holomorphic functions on the open unit disk and the boundary values of power series. Garnett's\footnote{My undergraduate analysis professor!} book \cite{Bounded analytic functions} gives a comprehensive account of harmonic analysis on the disk (and also half-plane), including\textemdash but not limited to\textemdash the Poisson Kernel and Hardy spaces. \cite{Ross et al} is an expository text on the Cauchy transform; \cite{Fractional Cauchy transforms} is a textbook on the Fractional Cauchy transform. Pavlovi\'{c}'s book\footnote{Not to be confused with the less-advanced but similarly-titled \cite{Baby Pavlovic}, also by Pavlovi\'{c}.} \cite{function classes on the unit disc} is an \emph{encyclopedic} account of spaces of complex-valued functions on the open unit disk in $\mathbb{C}$. One of the central themes of my complex-analytic investigations was the study of generating functions I called \textbf{set-series}\index{set-series}; these are of the form: \begin{equation} \varsigma_{V}\left(z\right)\overset{\textrm{def}}{=}\sum_{v\in V}z^{v}\label{eq:Definition of a set-series} \end{equation} where $V\subseteq\mathbb{N}_{0}$ is a set of interest. For my work, $V$ was an orbit class (or union thereof) of a given Hydra map on $\mathbb{Z}$. H. S. Wilf's delightfully titled \emph{generatingfunctionology }\cite{generatingfunctionology} is a thrilling read, as is Flajolet and Sedgewick's \emph{Analytic Combinatorics} \cite{analytic combinatorics}. For a thorough accounting of Tauberian methods used to study the coefficients of functions represented by power series or integrals, I cannot give too strong of a recommendation of Korevaar's book \cite{Korevaar} or Bingham, Goldie, and Teugels' \emph{magisterial }tome, the \emph{Regular Variation} \cite{Regular Variation}. For a more general background in (divergent) infinite series, Hardy's \emph{Divergent Series} \cite{Hardy - Divergent Series} is a classic. Flajolet's fascinating series of expository articles on the Mellin transform (\cite{Flajolet - Mellin Transforms,Flajolet - Digital sums}, etc.) provide powerful analytical tools for anyone wishing to study generating functions and power series asymptotics. It is also noteworthy that generating functions have a fascinating connection to transcendental number theory all their own. The study of this connection often goes by the name of \textbf{Mahler Theory}\footnote{Kurt, not Gustav.}\textbf{ }\index{Mahler Theory}\index{Mahler, Kurt}, after the German-Australian mathematician Kurt Mahler (of $p$-adic Mahler basis fame). Letting $d$ be an integer $\geq2$, the function\footnote{The notation $\phi_{d}$ is my own.}: \begin{equation} \phi_{d}\left(z\right)\overset{\textrm{def}}{=}\sum_{n=0}^{\infty}z^{d^{n}}\label{eq:Definition of Mahler's phi_d} \end{equation} is a \textbf{lacunary}\footnote{Meaning ``gap''.}\textbf{ series }\index{series!lacunary} with a natural boundary on the unit circle in $\mathbb{C}$. This function satisfies the functional equation: \begin{equation} \phi_{d}\left(z^{d}\right)=\phi_{d}\left(z\right)-z\label{eq:Mahler functional equation} \end{equation} By using this equation, Mahler was able to prove that the complex number $\phi_{d}\left(\alpha\right)$ is transcendental for any algebraic number $\alpha$ with complex absolute value $0<\left|\alpha\right|<1$ \cite{Mahler theory}. Mahler Theory uses functions characterized by polynomial functional equations like (\ref{eq:Mahler functional equation}) to show that said functions' take transcendental values at algebraic numbers \cite{mahler functions,Loxton and van der Poorten - arithemtic properties of functional equations}. Mahler theory is also connected with the theory of automata and automatic sequences\textemdash see \cite{mahler functions} and the sources cited therein.\newpage{} \pagestyle{fancy}\fancyfoot{}\fancyhead[L]{\sl LIST OF SYMBOLS}\fancyhead[R]{\thepage}\printnomenclature{} \begin{thebibliography}{100} \bibitem{mahler functions}\addcontentsline{toc}{chapter}{Bibliography} \pagestyle{fancy}\fancyfoot{}\fancyhead[L]{\sl BIBLIOGRAPHY}\fancyhead[R]{\thepage}Adamczewski, Boris, and Jason Bell. ``A problem around Mahler functions.'' (2012). <https://hal.archives-ouvertes.fr/hal-00798303/document> \bibitem{Adams' paper on p-adic transcendental numbers with an appendix on Schnirelman integration}Adams, William Wells. ``Transcendental numbers in the $p$-adic domain''. Columbia University, 1964. \bibitem{Aguayo 1}Aguayo, J. ``Vector measures and integral operators''. Ultrametric Functional Analysis, Contemp. Math., vol. 384, Amer. Math. Soc., Providence, RI (2005), pp. 1-13. \bibitem{Aguayo 2}Aguayo, J. and T. E. Gilsdorf. ``Non-Archimedean vector measures and integral operators'' $p$-Adic Functional Analysis, Lect. Notes Pure Appl. Math., vol. 222, Marcel Dekker, New York (2001), pp. 1-11. \bibitem{Aguayo and Moraga radon-nikodym theorem paper}Jos Aguayo and Mirta Moraga. ``A radon nikodym theorem in the non-archimedean setting.'' Proyecciones (Antofagasta) 20.3 (2001): 263-279. \bibitem{key-10}Akin, Ethan (2004), ``Why is the $3x+1$ Problem Hard?'', In: Chapel Hill Ergodic Theory Workshops (I. Assani, Ed.), Contemp. Math. vol 356, Amer. Math. Soc. 2004, pp. 1\textendash 20. (MR 2005f:37031). \bibitem{Amice}Amice, Yvette. \emph{Les nombres $p$-adiques}. Vol. 14. Presses universitaires de France, 1975. \bibitem{Anashi - p-adic ergodic theory}Anashi, Vladimir Sergeevich (2014). \emph{The p-adic ergodic theory and applications}. <https://www.researchgate.net/publication/269571423\_The\_p-adic\_ergodic\_theory\_and\_applications>/ \bibitem{Andaloro - first paper on sufficient sets}Andaloro, Paul. ``On total stopping times under $3x+1$ iteration'', Fibonacci Quart. 38 (2000), no. 1, 73\textendash 78. MR 1738650. \bibitem{Applegate and Lagarias - Trees}Applegate, David, and Jeffrey C. Lagarias. ``Density bounds for the $3x+1$ problem. I. Tree-search method.'' mathematics of computation 64.209 (1995): 411-426. \bibitem{Applegate and Lagarias - Difference inequalities}Applegate, David, and Jeffrey C. Lagarias. ``Density bounds for the $3x+1$ problem. II. Krasikov inequalities.'' Mathematics of computation 64.209 (1995): 427-438. \bibitem{3x+1 semigroup}David Applegate, Jeffrey C. Lagarias, ``The $3x+1$ semigroup'', Journal of Number Theory, Volume 117, Issue 1, 2006, Pages 146-159, ISSN 0022-314X. <https://www.sciencedirect.com/science/article/pii/S0022314X05001459> \bibitem{Arens =00003D000026 Singer - Generalized Analytic Functions}Arens, Richard and I. M. Singer (1956). ``Generalized Analytic Functions''. Transactions of the American Mathematical Society, Vol. 81, No. 2 (Mar., 1956), pp. 379-393. Published by: American Mathematical Society. <http://www.jstor.org/stable/1992923>. \bibitem{Arnold}Arnold, V. I., ``On teaching mathematics''. Translated from the original Russian by A.V. Goryunov. Published in: Uspekhi Mat. Nauk 53 (1998), no. 1, 229-234; English translation: Russian Math. Surveys 53 (1998), no. 1, 229-236. <http://www.ceremade.dauphine.fr/\textasciitilde msfr/articles/arnold/PRE\_anglais.ps> \bibitem{key-2-1}Axelsson, Ekaterina Yurova. ``On the sub-coordinate representation of $p$-adic functions''. \emph{Advances in Ultrametric Analysis}, edited by Alain Escassut, et al., American Mathematical Society, 2018. ProQuest Ebook Central, <https://ebookcentral.proquest.com/lib/socal/detail.action?docID=5347081>. \bibitem{Baker's Transcendental Number Theory}Baker, Alan (1990). \emph{Transcendental number theory}. Cambridge Mathematical Library (2nd ed.), Cambridge University Press, ISBN 978-0-521-39791-9, MR 0422171. \bibitem{p-adic proof for =00003D0003C0}Baker, Matt. ``A p-adic proof that pi is transcendental''. \emph{Matt Baker's Math Blog}. 20 March. 2015. <https://mattbaker.blog/2015/03/20/a-p-adic-proof-that-pi-is-transcendental/> \bibitem{Simon =00003D000026 Breuer - Natural Boundaries =00003D000026 Spectral Theory}Barry Simon and Jonathan Breuer (2011).\emph{ }``Natural Boundaries and Spectral Theory''. \bibitem{natural boundaries}Bell, Jason P., and Jeffrey C. Lagarias. ``$3x+1$ inverse orbit generating functions almost always have natural boundaries.'' arXiv preprint arXiv:1408.6884 (2014). \bibitem{Bell - Harmonic Analysis on the p-adics}Bell, Jordan. (2016) ``Harmonic analysis on the $p$-adic numbers''.\emph{ }<https://pdfs.semanticscholar.org/9d4f/f3efad867fe867331bb00402807e0856170c.pdf> \bibitem{Bell - Pontryagin Duals of Q/Z and Q}Bell, Jordan. (2015) ``The Pontryagin duals of $\mathbb{Q}/\mathbb{Z}$ and $\mathbb{Q}$''. <https://pdfs.semanticscholar.org/bffd/643d972eab01c0b063c34ad1560791899d3a.pdf> \bibitem{Color Picture Book}Alek Bellos and Edmund Harriss. \emph{Visions of the Universe: A Coloring Journey Through Math's Great Mysteries}.\emph{ }Experiment; Clr Csm edition. 29 Nov. 2016. ISBN-10: 1615193677. \bibitem{Berg =00003D000026 Opfer}Berg, Lothar, and Gerhard Opfer. ``An Analytic Approach to the Collatz $3n+1$ Problem for Negative Start Values.'' Computational Methods and Function Theory 13.2 (2013): 225-236. \bibitem{Berg =00003D000026 Meinardus}Berg, Lothar, and Gnter Meinardus. ``The $3n+1$ Collatz Problem and Functional Equations.'' (1995). \bibitem{Meinardus and Berg}Lothar Berg and Gnter Meinardus. ``Functional equations connected with the Collatz problem.'' Results in Mathematics 25, no. 1-2 (1994): 1-12. \bibitem{Besikovitch - Almost Periodic Functions}Besikovitch, A. S. (1954) \emph{Almost Periodic Functions}. Dover Publications, Inc., through special permission of Cambridge University Press. \bibitem{Regular Variation}Bingham, Nicholas H., Charles M. Goldie, and Jozef L. Teugels. \emph{Regular variation}. No. 27. Cambridge university press, 1989. \bibitem{pp adic Fourier theory}Borm, George Florimond. \emph{$p$-adic Fourier theory}. Diss. {[}Sl: sn{]}, 1988. \bibitem{Bosch lying title}S. Bosch, U. Gntzer, and R. Remmert, Non-Archimedean Analysis, Grundlehren der Math. Wiss. 261, Springer-Verlag, Berlin, 1984. <https://www.math.arizona.edu/\textasciitilde cais/scans/BGR-Non\_Archimedean\_Analysis.pdf> \bibitem{Introduction to Langlands}Bump, Daniel, et al. \emph{An introduction to the Langlands program}. Vol. 140. Springer Science \& Business Media, 2003. \bibitem{key-20}Chabrera, Vctor Martn. ``An algebraic fractal approach to Collatz Conjecture''. BS thesis. Universitat Politcnica de Catalunya, 2019. \bibitem{Cherry non-archimedean function theory notes}Cherry, William. ``Lectures on non-archimedean function theory.'' arXiv preprint arXiv:0909.4509 (2009). <https://arxiv.org/pdf/0909.4509.pdf> \bibitem{Cohen Number Theory}Cohen, H. (2008). \emph{Number theory: Volume II: Analytic and modern tools} (Vol. 240). Springer Science and Business Media. \bibitem{Collatz Letter}Collatz, Lothar. ``\href{http://www.cecm.sfu.ca/organics/papers/lagarias/paper/goodies/ubersetzung/html/ubersetzung.html}{Letter to Professor Mays}''. Translated from the German by Marion Meudt and John Read\footnote{<http://www.cecm.sfu.ca/organics/papers/lagarias/paper/goodies/ubersetzung/html/ubersetzung.html>}. \bibitem{p-adic L-functions paper}Colmez, Pierre. ``Fontaine's rings and $p$-adic $L$-functions.'' Lecture notes 32 (2004): 33. <http://people.math.jussieu.fr/\textasciitilde colmez/tsinghua.pdf> \bibitem{Conrad on p-adic series}Conrad, K. ``Infinite Series in $p$-Adic Fields''. \bibitem{Conrad - Local Global Principle}Conrad, K. ``The Local-Global Principle''. \bibitem{Conway - Unpredictable Iterations}Conway, John H. (1972). ``Unpredictable Iterations''. Proc. 1972 Number Theory Conf., Univ. Colorado, Boulder, pp. 49\textendash 52. \bibitem{Bohm and Sontacchi}Corrado Bhm and Giovanna Sontacchi (1978). ``On the existence of cycles of given length in integer sequences like $x_{n}+1=x_{n}/2$ if $x_{n}$ even, and $x_{n+1}=3x_{n}+1$ otherwise'', Atti Ac- cad. Naz. Lincei Rend. Cl. Sci. Fis. Mat. Natur. 64 (1978), 260\textendash 264. (MR 83h:10030) \bibitem{Circulant Matrices}Davis, Philip J. \emph{Circulant matrices}. American Mathematical Soc., 2013. \bibitem{Dynamical Systems}Devaney, Robert L. \emph{An introduction to chaotic dynamical systems}. CRC press, 2018. \bibitem{Non-archimedean operator theory}Diagana, Toka, and Franois Ramaroson. \emph{Non-Archimedean operator theory}. Springer, 2016. \bibitem{First 30 years of p-adic mathematical physics}Dragovich, B., et al. ``$p$-Adic mathematical physics: the first 30 years.'' P-Adic numbers, ultrametric analysis and applications 9.2 (2017): 87-121. \bibitem{Adelic Harmonic Oscillator}Dragovich, Branko. ``Adelic harmonic oscillator.'' International Journal of Modern Physics A 10.16 (1995): 2349-2365. \bibitem{Berkovich spaces and applications}Ducros, Antoine, Charles Favre, and Johannes Nicaise, eds. \emph{Berkovich spaces and applications}. Springer, 2015. \bibitem{Dwork Zeta Rationality}Dwork, Bernard. ``On the rationality of the zeta function of an algebraic variety''. Amer. J. Math., 82:631\textendash 648, 1960. \bibitem{value distribution in p-adic analysis}Escassut, Alain. \emph{Value Distribution in p-adic Analysis}. World Scientific, 2015. \bibitem{n-furcations}M. R. Feix, M. Muriel and J. L. Rouet. ``Statistical Properties of an Iterated Arithmetic Mapping'', J. Stat. Phys., 76 (1994), 725-741 \bibitem{Geometric Measure Theory}Federer, Herbert. \emph{Geometric measure theory}. Springer, 2014. \bibitem{analytic combinatorics}Flajolet, Philippe, and Robert Sedgewick. \emph{Analytic combinatorics}. Cambridge University press, 2009. \bibitem{Flajolet - Mellin Transforms}Philippe Flajolet, Xavier Gourdon, and Philippe Dumas. ``Mellin transforms and asymptotics: Harmonic sums\textquotedblright . In: Theoretical Computer Science 144 (1995), pp. 3\textendash 58. \bibitem{Flajolet - Digital sums}Philippe Flajolet, et al. ``Mellin transforms and asymptotics: digital sums\textquotedblright . In: Theoretical Computer Science 123 (1994), pp. 291\textendash 314. \bibitem{Folland - real analysis}Folland, Gerald B. \emph{Real analysis: modern techniques and their applications}. Vol. 40. John Wiley and Sons, 1999. \bibitem{Folland - harmonic analysis}Folland, Gerald B. \emph{A course in abstract harmonic analysis}. Vol. 29. CRC press, 2016. \bibitem{ergodic theory of p-adic transformations}Furno, Joanna. 2013. \emph{Ergodic Theory of P-Adic Transformations}. PhD Thesis. University of North Carolina at Chapel Hill. <https://doi.org/10.17615/8p5w-9v68> \bibitem{Bounded analytic functions}Garnett, John. \emph{Bounded analytic functions}. Vol. 236. Springer Science \& Business Media, 2007. \bibitem{George collatz visualization}Georgiou, George M. ``Encoding and Visualization in the Collatz Conjecture.'' arXiv preprint arXiv:1811.00384 (2018). \bibitem{Automorphic Representations}Goldfeld, D., and Hundley, J. (2011). \emph{Automorphic Representations and L-Functions for the General Linear Group: Volume 1} (Vol. 129). Cambridge University Press. \bibitem{Gouvea's introudction to p-adic numbers book}Gouva, Fernando Quadros. \emph{$p$-adic Numbers: An Introduction}. Springer Science and Business Media, 2012. \bibitem{Gouvea's p-adic number history slides}Gouva, Fernando Quadros. ``Hensel's $p$-adic Numbers: early history'' <https://www-fourier.ujf-grenoble.fr/\textasciitilde panchish/Mag2009L3/GouveaHensel2.pdf> \bibitem{Walsh}B. Golubov, A. Efimov, and V. Skvortsov. ``Walsh Functions and Their Generalizations.'' \emph{Walsh Series and Transforms}. Springer, Dordrecht, 1991. 1-33. \bibitem{key-18}Hashimoto, Yukihiro. ``A fractal set associated with the Collatz problem.'' Bull. of Aichi Univ. of Education, Natural Science 56 (2007): 1-6. \bibitem{key-21}Hassan, Sk. ``Discrete dynamics of one dimensional Collatz like integral value transformations.'' Journal of Applied Mathematics and Computing 49.1 (2015): 91-105. \bibitem{Hardy - Divergent Series}Hardy, G. H. (1949). \emph{Divergent Series}. Oxford University Press. Oxford: 1949. \bibitem{Dirichlet Series (Hardy)}G. H. Hardy and Marcel Reisz. (1915). \emph{The general theory of Dirichlet's series}. Cambridge {[}Eng.{]} University Press, 1915. <http://hdl.handle.net/2027/mdp.39015064521282> \bibitem{key-27}Helson, Henry (1955). ``On a theorem of Szeg\H{o}''. Proceedings of the American Mathematical Society. \bibitem{Hensel-s original article}Hensel, Kurt. \textquotedbl Ueber die Zurckfhrung der Divisorensysteme auf eine reducirte Form.\textquotedbl{} (1897): 234-250. \bibitem{Fractional Cauchy transforms}Hibschweiler, Rita A., and Thomas H. MacGregor. \emph{Fractional Cauchy Transforms}. Chapman and Hall/CRC, 2020. \bibitem{Iwasawa}Iwasawa, Kinkichi. \emph{Lectures on $P$-Adic $L$-Functions}.(AM-74), Volume 74. Princeton University Press, 2016. \bibitem{Jackson}Jackson, Glenn Micah Jr., ``Explorations of the Collatz Conjecture (mod $m$)'' (2014). Honors College Theses. 70. <https://digitalcommons.georgiasouthern.edu/honors-theses/70> \bibitem{key-19}Joseph, L. Pe. ``The $3x+1$ fractal.'' Computers \& Graphics 28.3 (2004): 431-435. \bibitem{Non-archimedean function spaces and the dominated convergence theorem}J. K\k{a}kol. C. Perez-Garcia. W. \'{S}liwa. ``Non-archimedean function spaces and the Lebesgue dominated convergence theorem.'' Bull. Belg. Math. Soc. Simon Stevin 19 (1) 173 - 184, march 2012. https://doi.org/10.36045/bbms/1331153417 \bibitem{Katsaras}Katsaras. A. K. ``On $p$-adic vector measure spaces'', J. Math. Anal. Appl. ,365(2010), 342-357. \bibitem{Kedlaya}Kedlaya, Kiran S. \emph{p-adic Differential Equations}. Vol. 125. Cambridge University Press, 2010. \bibitem{Quantum Paradoxes}Khrennikov, Andrei. \emph{Non-Archimedean Analysis: Quantum Paradoxes, Dynamical Systems and Biological Models}. Springer, Dordrecht, 1997. 101-129. \bibitem{Probabilities taking values in non-archimedean fields}Khrennikov, A. Yu. ``Generalized probabilities taking values in non-Archimedean fields and in topological groups.''\textquotedbl{} Russian Journal of Mathematical Physics 14.2 (2007): 142-159. \bibitem{Measure-theoretic approach to p-adic probability theory}Khrennikov, Andrei; Yamada, Shinichi; van Rooij, Arnoud. ``The measure-theoretical approach to $p$-adic probability theory''. Annales Mathmatiques Blaise Pascal, Tome 6 (1999) no. 1, pp. 21-32. <http://www.numdam.org/item/AMBP\_1999\_\_6\_1\_21\_0/> \bibitem{p-adic Tauberian}Khrennikov, A. Yu, and V. M. Shelkovich. ``Distributional asymptotics and $p$-adic Tauberian and Shannon\textendash Kotelnikov theorems.'' Asymptotic Analysis 46.2 (2006): 163-187. \bibitem{Koblitz's book}Koblitz, Neal. \emph{$p$-adic analysis: A short course on recent work}. Vol. 46. Cambridge University Press, 1980. \bibitem{Koblitz's other book}Koblitz, Neal. \emph{$p$-adic Numbers, $p$-adic Analysis, and Zeta-Functions}. Vol. 58. Springer Science \& Business Media, 2012. \bibitem{Kochubei}Kochubei, Anatoly N. ``Non-Archimedean Duality: Algebras, Groups, and Multipliers''. 2016 <https://arxiv.org/pdf/1510.06876.pdf> \bibitem{RCWAG}Kohl, Stefan (2016).\emph{ RCWA: Residue-Class-Wise Affine Groups}. Version 4.0.0 <https://www.gap-system.org/Manuals/pkg/rcwa/doc/manual.pdf> \bibitem{dgh paper}A. V. Kontorovich and Y. G. Sinai. ``Structure Theorem for $\left(d,g,h\right)$-Maps'', 2006 <https://arxiv.org/pdf/math/0601622.pdf> \bibitem{Lagarias-Kontorovich Paper}Alex V. Kontorovich and Jeffery C. Lagarias. (2009) ``Stochastic Models for the 3x+1 and 5x+1 Problems''. <https://arxiv.org/pdf/0910.1944> \bibitem{Korevaar}Korevaar, Jacob. \emph{Tauberian theory: a century of developments}. Vol. 329. Springer Science and Business Media, 2013. \bibitem{Krasikov}Krasikov, Ilia. ``How many numbers satisfy the $3x+1$ conjecture?'' International Journal of Mathematics and Mathematical Sciences 12.4 (1989): 791-796. \bibitem{key-4}Kudla, Stephen S. ``Tate's thesis.'' An introduction to the Langlands program. Birkhuser, Boston, MA, 2004. 109-131. \bibitem{key-4-1}Kyung Soo Rim. ``Gibbs phenomenon for Fourier partial sums on $\mathbb{Z}_{p}$'', \emph{Journal of Mathematical Analysis and Applications}, Volume 433, Issue 1, 2016, Pages 392-404, ISSN 0022-247X, <https://doi.org/10.1016/j.jmaa.2015.08.005> \bibitem{Ultimate Challenge}Lagarias, Jeffrey C., ed. \emph{The ultimate challenge: The $3x+1$ problem}. American Mathematical Soc., 2010. \bibitem{Lagarias' Survey}Lagarias, J.C. ``The $3x+1$ problem and its generalizations'', Amer. Math. Monthly 92 (1985), 3\textendash 23. \bibitem{Lagarias - rational cycles for 3x+1}Lagarias, Jeffrey C. ``The set of rational cycles for the $3x+1$ problem'', Acta Arith. 56 (1990), no. 1, 33\textendash 53. MR 1067980, DOI <https://doi.org/10.4064/aa-56-1-33-53> \bibitem{Lang - Algebraic Number Theory}Lang, Serge. \emph{Algebraic number theory}. Vol. 110. Springer Science and Business Media, 2013. \bibitem{key-9}Laarhoven, Thijs, and Benne de Weger. ``The Collatz conjecture and De Bruijn graphs.'' Indagationes Mathematicae 24.4 (2013): 971-983. \bibitem{Leigh's article}Leigh, G.M. ``A Markov process underlying the generalized Syracuse algorithm'', Acta Arith. 46 (1986) 125-143. \bibitem{Liepins}Liepins, Gunar E. (1976). ``A Paley-Wiener Theorem for Locally Compact Abelian Groups''. Transactions of the American Mathematical Society, Vol. 222 (Sep., 1976), pp. 193-210 Published by: American Mathematical Society. <http://www.jstor.org/stable/1997665> \bibitem{Loxton and van der Poorten - arithemtic properties of functional equations}J. H. Loxton and A. J. van der Poorten (1981). ``Arithmetic properties of the solutions of a class of functional equations''. \bibitem{Ludkovsky on non-archimedean measures}Ludkovsky, S. V. ``Quasi-Invariant and Pseudo-Differentiable Measures with Values in Non-Archimedean Fields on a Non-Archimedean Banach Space''. J Math Sci 128, 3428\textendash 3460 (2005). <https://doi.org/10.1007/s10958-005-0280-2> \bibitem{Mackey - Laplace Transform for Locally Compact Abelian groups}Mackey, George W. (1948). ``The Laplace Transform for Locally Compact Abelian Groups''. Proceedings of the National Academy of Sciences, Volume 34, Number 4, pp. 156-162. <http://www.pnas.org/content/34/4/156> \bibitem{Mahler}Mahler, Kurt. \emph{Introduction to p-adic numbers and their functions}. No. 64. CUP Archive, 1973. \bibitem{Mahler Series}Mahler, Kurt. ``An Interpolation Series for Continuous Functions of a $p$-adic Variable.'' (1958): 23-34. \bibitem{Mandelbrojt's lacunary series paper}Mandelbrojt, Szolem. ``Lacunary series''. Rice Institute Pamphlet - Rice University Studies, 14, no. 4 (1927) Rice University: <https://hdl.handle.net/1911/8511>. \bibitem{Mandelbrojt's Dirichlet Series book}Mandelbrojt, S. (1972) \emph{Dirichlet Series: Principals and Methods}. D. Reidel Publishing Company / Dordrecht-Holland. \bibitem{The Berkovitch Space Paper}Manetti, Nicolas. ``Gefland transform and spectral radius formulae for ultrametric Banach algebras'' Ultrametric Functional Analysis: Seventh International Conference on P-Adic Functional Analysis, June 17-21, 2002, University of Nijmegen, the Netherlands. Vol. 319. American Mathematical Soc., 2003. \bibitem{Matthews' Conjecture}Matthews, K. R. ``A Generalized $3x+1$ Conjecture''. <https://www.numbertheory.org/gnubc/challenge> \bibitem{Matthews' Leigh Article}Matthews, K. R. ``The generalized $3x+1$ mapping: George Leigh's approach''. <http://www.numbertheory.org/keith/george.html> \bibitem{Matthews' slides}Matthews, K. R. ``Generalizations of the $3x+1$ problem and connections with Markov matrices and chains'' (2010) <http://www.numbertheory.org/PDFS/3x+1\_slides.pdf> \bibitem{Matthews and Watts}Matthews, K. R., and Watts, A.M. (1985). ``A Markov approach to the generalized Syracuse algorithm''. Acta Arithmetica, 45(1), 29-42. \bibitem{Meinardus}Meinardus, Gnter. ``Some Analytic Aspects Concerning the Collatz Problem.'' (2001). \bibitem{key-6-2-1}Memic, Nacima. ``Multiplicative Systems on Ultra-Metric Spaces.'' Mathematica Balkanica (2010). \bibitem{Moller's paper (german)}Mller, Herbert. ``ber Hasses Verallgemeinerung des Syracuse-Algorithmus (Kakutanis Problem)'', Acta Arith. 34 (1978) 219-226. \bibitem{Monks' sufficiency paper for 3x+1}Monks, Kenneth. ``The sufficiency of arithmetic progressions for the $3x+1$ Conjecture.'' Proceedings of the American mathematical society 134.10 (2006): 2861-2872. \bibitem{The many Monks paper}Monks, Keenan, et al. ``On the distribution of arithmetic sequences in the Collatz graph.'' <http://www.mathematicalgemstones.com/maria/slides/SlidesV4.pdf> \bibitem{Monna and Springer}Monna, A. and Springer, T. ``Integration non-Archimedienne'', 1,2. Indag. Math., 25, 634-653 (1963) \bibitem{Mahler theory}Nishioka, Kumiko. \emph{Mahler functions and transcendence}. Springer, 2006. \bibitem{Schikhof memorial}Ochsenius, H., E. Olivos, and C. Perez-Garcia. ``Remembering WH Schikhof.'' P-Adic Numbers, Ultrametric Analysis, and Applications 7.2 (2015): 81-95. \bibitem{Collatz Biography}J. J. O'Connor and E. F. Robertson. ``Lothar Collatz''. <https://mathshistory. st-andrews.ac.uk/Biographies/Collatz/>, 2006. St Andrews University School of Mathematics and Statistics, Scotland. \bibitem{Oliveira}T. Oliveira e Silva ``Maximum excursion and stopping time record-holders for the problem: computational results''. Math. Comp., 68 (1) (1999), pp. 371-384 (see website: <http://www.ieeta.pt/\textasciitilde tos/3x+1.html> for additional computations) \bibitem{Opfer}Opfer, Gerhard. ``An Analytic Approach to the Collatz $3n+1$ Problem''. Hamburger Beitrge zur Angewandten Mathematik Nr. 2011-09, May 2011. \bibitem{Baby Pavlovic}Pavlovi\'{c}, Miroslav. \emph{Introduction to function spaces on the disk}. Belgrade, 2004. \bibitem{function classes on the unit disc}Pavlovi\'{c}, Miroslav. \emph{Function classes on the unit disc}. de Gruyter, 2019. \bibitem{collatz harmonic oscillator}Perelman, C. C., Carb-Dorca, R. ``The Collatz conjecture and the quantum mechanical harmonic oscillator''. J Math Chem 60, 145\textendash 160 (2022). <https://doi.org/10.1007/s10910-021-01296-6> \bibitem{key-4-2}Perez-Garcia, C. and W. H. Schikhof. ``The Orlicz-Pettis property in $p$-adic analysis''. Collect. Math., 43 (1992), 225-233. \bibitem{Journey throughout the history of p-adic numbers}Perrin, Yvette. ``A journey throughout the history of $p$-adic numbers.'' \emph{Advances in Ultrametric Analysis}, edited by Alain Escassut, et al., American Mathematical Society, 2018. ProQuest Ebook Central, <https://ebookcentral.proquest.com/lib/socal/detail.action?docID=5347081> \bibitem{Remmert}Remmert, Reinhold. \emph{Classical topics in complex function theory}. Vol. 172. Springer Science and Business Media, 2013. \bibitem{Robert's Book}Robert, A. M. (2013). \emph{A course in p-adic analysis} (Vol. 198). Springer Science and Business Media. Chicago. \bibitem{Real and p-Adic Oscillatory integrals}Rogers, Keith McKenzie. \emph{Real and p-Adic Oscillatory Integrals}. Diss. The University of New South Wales, 2004. \bibitem{p-adic van der Corput lemma}Rogers, Keith M. ``A van der Corput lemma for the $p$-adic numbers.'' Proceedings of the American Mathematical Society (2005): 3525-3534. \bibitem{van Rooij - Non-Archmedean Functional Analysis}van Rooij, A.C.M. \emph{Non-Archimedean functional analysis.} Pure and Applied Math., vol. 51, Marcel Dekker, New York, 1978. \bibitem{van Rooij and Schikhof - "Non-archimedean analysis"}A. C. M. van Rooij and W. H. Schikhof. ``Non-Archimedean analysis.\textquotedblright{} Nieuw Arch. Wisk. (3)19, 120\textendash 160 (1971). \bibitem{van Rooij and Schikhof "Non-archimedean integration theory"}Arnoud C. M van Rooij, and Wim H. Schikhof. ``Non-Archimedean integration theory.'' Indagationes Mathematicae (Proceedings). Vol. 72. No. 2. North-Holland, 1969. \bibitem{Roosendaal's Website}Roosendaal, E. ``On the $3x+1$ problem'' <www.ericr.nl/wondrous> \bibitem{Ross et al}Ross, William T., Joseph A. Cima, and Alec L. Matheson. ``The Cauchy Transform.'' (2006). \bibitem{Parity Sequences}Rozier, O. (2018). ``Parity sequences of the $3x+1$ map on the $2$-adic integers and Euclidean embedding''. arXiv preprint arXiv:1805.00133. \bibitem{Ultrametric Calculus}Schikhof, W. (1985). \emph{Ultrametric Calculus: An Introduction to p-Adic Analysis} (Cambridge Studies in Advanced Mathematics). Cambridge: Cambridge University Press. doi:10.1017/CBO9780511623844 \bibitem{Schikhof's Thesis}Schikhof, W. H. \emph{Non-Archimedean Harmonic Analysis}, Ph.D. Thesis, pp. 1\textendash 80 (Catholic Univ. of Nijmegen, The Netherlands, 1967). \bibitem{Schikhof - Radon-Nikodym}Schikhof, W. H. ``A Radon-Nikodym theorem for non-Archimedean integrals and absolutely continuous measures on groups,\textquotedblright{} Indag. Math. 33, 78\textendash 85 (1971). \bibitem{key-8-1}Schikhof, W. H. ``An Approach to the Ultrametric Moment Problem'', Trudy Mat. Inst. Steklova, 2004, Volume 245, 251\textendash 256 \bibitem{Schikhof Banach Space Paper}Schikhof, W. H. ``Banach spaces over non-Archimedean valued fields,\textquotedblright{} Topology Proc. 24, 547\textendash 581 (2001). \bibitem{Schneider awful book}Schneider, Peter. \emph{Nonarchimedean functional analysis}. Springer Science \& Business Media, 2001. \bibitem{More Schneider lies}Schneider, Peter, and Jeremy Teitelbaum. ``$p$-adic Fourier theory.'' arXiv preprint math/0102012 (2001). \bibitem{On Wiener's Lemma}Schulte, Eike. ``On Wiener's lemma for locally compact abelian groups''. arXiv:2005.05212 {[}math.FA{]} <https://arxiv.org/abs/2005.05212> (2020) \bibitem{Local-Global Principle Failure}Selmer, E. ``The Diophantine equation $ax^{3}+by^{3}+cz^{3}=0$'', Acta Arithmetica 85 (1951), 203\textendash 362. \bibitem{Dreancatchers for Hydra Maps}Siegel, M. C. ``Conservation of Singularities in Functional Equations Associated to Collatz-Type Dynamical Systems; or, Dreamcatchers for Hydra Maps'' (2019) arXiv:1909.09733v3 {[}math.GM{]} <https://arxiv.org/abs/1909.09733> \bibitem{Mellin transform paper}Siegel, M. C. ``Syracuse Random Variables and the Periodic Points of Collatz-type maps'' (2020) <https://arxiv.org/abs/2007.15936> \bibitem{Sultanow}Sultanow, Eldar, Christian Koch, and Sean Cox. ``Collatz sequences in the light of graph theory.'' (2020). \bibitem{Taibleson - Fourier Analysis on Local Fields}Taibleson, M. H. \emph{Fourier Analysis on Local Fields}. Notes, vol. 15, Princeton Univ. Press (1975) \bibitem{Tao Blog}Tao, Terence. (2011) ``The Collatz conjecture, Littlewood-Offord theory, and powers of $2$ and $3$'' <https://terrytao.wordpress.com/2011/08/25/the-collatz-conjecture-littlewood-offord-theory-and-powers-of-2-and-3/> \bibitem{Tao Fourier Transform Blog Post}Tao, Terence. (2009) ``245C, Notes 2: The Fourier transform'' <https://terrytao.wordpress.com/2009/04/06/the-fourier-transform/> \bibitem{Tao Probability paper}Tao, Terence. (2019) ``Almost all orbits of the Collatz map attain almost bounded values.\textquotedblright{} <https://arxiv.org/abs/1909.03562> \bibitem{Tate's thesis}Tate, John (1950), \emph{Fourier analysis in number fields and Hecke's zeta functions}, Princeton University Ph.D. thesis under Emil Artin. Reprinted in Cassels, J. W. S.; Frhlich, Albrecht, eds. (1967), Algebraic number theory, London: Academic Press, pp. 305\textendash 347, MR 0215665 \bibitem{Terras 76}Terras, Riho. (1976) ``A stopping time problem on the positive integers''. Acta Arithmetica. XXX. \bibitem{Terras 79}Terras, Riho. (1979) ``On the existence of a density''. Acta Arithmetica. XXXV. \bibitem{On the origins of p-adic analysis}Ullrich, Peter. ``On the origins of p-adic analysis.'' Symposia Gaussiana, Proc. of the 2nd Gauss Symposium (Mnchen 1993), Conference A: Mathematics and Theoretical Physics, ed. M. Behara, R. Fritsch und RG Lintz, (Berlin 1995). 1995. \bibitem{Vladimirov - the big paper about complex-valued distributions over the p-adics}Vladimirov, Vasilii S. ``Generalized functions over the field of $p$-adic numbers.'' Russian Mathematical Surveys 43.5 (1988): 19. \bibitem{key-5-1}Volosivets, S. S. ``Fourier-Vilenkin series and analogs of Besov and Sobolev classes.'' Annales Univ. Sci. Budapest, Sect. Comp. Vol. 33. 2010. \bibitem{Volo - p-adic Wiener}Volosivets, S. S. ``A Distributional Proof of $p$-Adic Wiener Tauberian Theorem and Approximation by Translates of a Function.'' p-Adic Numbers, Ultrametric Analysis and Applications 13.4 (2021): 308-315. \bibitem{p-adic space-time}Volovich, Igor V.. ``$p$-adic space-time and string theory.\textquotedblright{} Theoretical and Mathematical Physics 71 (1987): 574-576. \bibitem{Cyclotomic fields}Washington, Lawrence C. \emph{Introduction to Cyclotomic Fields}. Second Edition. Vol. 83. New York: Springer New York, 1997 \bibitem{de Weger on Opfer}de Weger, Benne. ``Comments on Opfer's alleged proof of the $3n+1$ Conjecture''. Tech. rep. Eindhoven University of Technology, 2011. \bibitem{generatingfunctionology}Wilf, Herbert S. \emph{generatingfunctionology}. CRC press, 2005. \bibitem{Wirsching's book on 3n+1}Wirsching, Gnther J. \emph{The dynamical system generated by the $3n+1$ function}, Lecture Notes in Mathematics, vol. 1681, Springer-Verlag, Berlin, 1998. MR 1612686\newpage{} \end{thebibliography} \addcontentsline{toc}{chapter}{Index}\pagestyle{fancy}\fancyfoot{}\fancyhead[L]{\sl INDEX}\fancyhead[R]{\thepage} \printindex \end{document}
2412.03135v1
http://arxiv.org/abs/2412.03135v1
A new look at the classiffication of the tri-covectors of a 6-dimensional symplectic space
\documentclass[leqno,a4paper]{article} \usepackage{amssymb,amsmath,amsthm,verbatim} \title{\textbf{A new look at the classification }\\ \textbf{of the tri-covectors of a} $6$\textbf{-dimensional symplectic space }} \author{\textsc{J. Mu\~{n}oz Masqu\'e, L. M. Pozo Coronado}} \date{} \newtheorem{theorem}{Theorem}[section] \newtheorem{proposition}[theorem]{Proposition} \newtheorem{lemma}[theorem]{Lemma} \newtheorem{corollary}[theorem]{Corollary} \newtheorem{definition}[theorem]{Definition} \begin{document} \maketitle \begin{abstract} Let $\mathbb{F}$ be a field of characteristic $\neq 2$ and $3$, let $V$ be a $\mathbb{F}$-vector space of dimension $6$, and let $\Omega \in \wedge ^2V^\ast $ be a non-degenerate form. A system of generators for polynomial invariant functions under the tensorial action of the group $Sp(\Omega )$ on $\wedge ^3 V^\ast $, is given explicitly. Applications of these results to the normal forms of De Bruyn-Kwiatkowski and Popov are given. \end{abstract} \bigskip \noindent\emph{Mathematics Subject Classification 2010:\/} Primary: 15A21; Secondary: 15A63, 15A75, 20G15 \medskip \noindent\emph{Key words and phrases:\/} Algebraic invariant function, derivation, linear representation, normal forms, symplectic group. \section{Symplectic invariants\label{symplectic}} Below, $\mathbb{F}$ denotes a field of characteristic $\neq 2$ and $3$, and $V$ is a $\mathbb{F}$-vector space of dimension $6$. Notations and elementary properties of algebraic sets and groups have been taken from Fogarty's book \cite{Fogarty}. The group $GL(V)$ acts on $\otimes ^rV^\ast $ by \[ (A\cdot \xi )(x_1,\dotsc,x_r)=\xi (A^{-1}x_1,\dotsc,A^{-1}x_r), \] $\forall\xi \in \otimes ^rV^\ast ,\;\forall x_1,\dotsc,x_r\in V$, and $GL(V)$ acts on $\wedge ^rV^\ast $ by the same formula. In particular, the $\mathbb{F}$-homomorphism induced by the action of $GL(V)$ on $\wedge ^3 V^\ast $ is denoted by $\rho\colon GL(V)\to GL(\wedge ^3V^\ast )$. We denote by $\rho^{\prime}\colon Sp(\Omega ) \to GL(\wedge ^3 V^\ast )$ the restriction of $\rho $ to the symplectic group of a non-degenerate $2$-covector $\Omega \in \wedge ^2V^\ast $. Furthermore, $GL(V)$ acts on $\otimes ^rV^\ast \otimes V$ by $(A\cdot \eta )(x_1,\dotsc,x_r) =A[\eta (A^{-1}x_1,\dotsc,A^{-1}x_r)]$, for all $\eta $ in $\otimes ^rV^\ast \otimes V$, and all $x_1,\dotsc,x_r\in V $. If $(v_i)_{i=1}^6$ is a basis of $V$ such that $\Omega =\sum _{i=1}^6v^i\wedge v^{i+3}$ and $(v^i)_{i=1}^6$ is the dual basis, then we define a system of coordinate functions $y_{abc}$, $1\leq a<b<c\leq6$, on $\wedge ^3 V^\ast $ by setting \begin{equation*} \theta =\sum\nolimits_{1\leq a<b<c\leq 6}y_{abc}(\theta ) \left( v^a\wedge v^b\wedge v^c\right) \in \wedge ^3 V^\ast . \end{equation*} For every $A\in GL(V)$ and every $1\leq a<b<c\leq 6$ we have \begin{align} A\cdot \left( v^a\wedge v^b\wedge v^c\right) & =(A^{-1})^\ast v^a\wedge (A^{-1})^\ast v^b \wedge (A^{-1})^\ast v^c \nonumber \\ & =\left( v^a\circ A^{-1}\right) \wedge \left( v^b\circ A^{-1}\right) \wedge \left( v^{c}\circ A^{-1}\right) \nonumber \\ & =\left( \lambda _{ah}v^h\right) \wedge \left( \lambda _{bi}v^i\right) \wedge \left( \lambda _{cj}v^{j}\right) \nonumber \\ & =\sum\nolimits_{1\leq h<i<j\leq 6}\left\vert \begin{array} [c]{ccc} \lambda _{ah} & \lambda _{bh} & \lambda _{ch}\\ \lambda _{ai} & \lambda _{bi} & \lambda _{bi}\\ \lambda _{aj} & \lambda _{bj} & \lambda _{cj} \end{array} \right\vert v^h\wedge v^i\wedge v^j, \nonumber \end{align} where $(\lambda _{ij})_{i,j=1}^6$ is the matrix of $(A^{-1})^T$ in the basis $(v_i)_{i=1}^6$. Therefore, we have $\mathbb{F}[\wedge ^3 V^\ast ] =\mathbb{F}[y_{abc}]_{1\leq a<b<c\leq 6}$, and hence, $\mathbb{F}(\wedge ^3V^\ast ) =\mathbb{F}(y_{abc})_{1\leq a<b<c\leq 6}$. A function $I\in\mathbb{F}[\wedge ^3 V^\ast ]$ (resp.\ $I\in \mathbb{F}(\wedge ^3 V^\ast )$) is $Sp(\Omega )$-invariant if \[ I\left( A\cdot \theta \right) =I(\theta ),\quad \forall \theta \in \wedge ^3 V^\ast, \; \forall A\in Sp(\Omega ). \] \section{The basic invariants defined\label{invariants}} \subsection{$I_1$ defined\label{I_1}} For every $\theta \in \wedge ^3 V^\ast $ there exists a unique $J^\theta \in\wedge ^2V^\ast \otimes V$ such that \begin{equation} \theta (x,y,z)=\Omega (J^\theta (x,y),z), \quad \forall x,y,z\in V.\label{J} \end{equation} Given $A\in GL(V)$ and replacing $\theta $ by $A\cdot \theta $ in the formula \eqref{J}, we have \[ (A\cdot \theta )(x,y,z)=\Omega (J^{A\cdot \theta }(x,y),z). \] Expanding on the right-hand side, we deduce \begin{align*} (A\cdot \theta )(x,y,z) & =\theta (A^{-1}x,A^{-1}y,A^{-1}z)\\ & =\Omega (J^\theta (A^{-1}x,A^{-1}y),A^{-1}z)\\ & =(A\cdot \Omega)(A[J^\theta (A^{-1}x,A^{-1}y)],z))\\ & =(A\cdot \Omega)((A\cdot J^\theta )(x,y),z). \end{align*} Furthermore, if $A\in Sp(\Omega )$, then $A\cdot \Omega =\Omega $ and consequently \[ (A\cdot \theta )(x,y,z)=\Omega ((A\cdot J^\theta )(x,y),z) =\Omega (J^{A\cdot \theta }(x,y),z), \quad \forall x,y,z\in V. \] Hence $J^{A\cdot \theta }=A\cdot J^\theta , \quad \forall A\in Sp(\Omega )$. If \begin{equation} \theta =\sum _{1\leq i<j<k\leq 6} \lambda _{ijk}v^i\wedge v^{j}\wedge v^k \label{theta} \end{equation} and $J^\theta =\sum_{b<c}\mu _{bc}^av^b\wedge v^c\otimes v_a$, then by letting $x=v_i$, $y=v_j$, $z=z^kv_k$ in the formula \eqref{J} for every pair $1\leq i<j\leq 5$, it follows: \[ \sum _{k\neq i,j}\theta \left( v_i,v_j,v_k\right) z^k =-\mu_{ij}^4z^1-\mu_{ij}^5z^2-\mu_{ij}^6z^3+\mu_{ij}^1z^4 +\mu_{ij}^2z^5+\mu_{ij}^3z^6, \] and comparing the coefficients of $z^1,\dotsc,z^6$ in both sides, we deduce \[ \begin{array} [c]{lll} \mu_{ij}^1=\lambda _{ij4}, & \mu_{ij}^2=\lambda _{ij5}, & \mu_{ij}^3=\lambda _{ij6}, \medskip\\ \mu_{ij}^4=-\lambda _{ij1}, & \mu_{ij}^{5}=-\lambda _{ij2}, & \mu_{ij}^6=-\lambda _{ij3}, \end{array} \] with the usual agreement: $\lambda _{ijk}=\varepsilon _\sigma \lambda _{abc}$, where $a<b<c$, $\{ a,b,c\} =\{ i,j,k\} $, $\sigma $ being the permutation $a\mapsto i$, $b\mapsto j$, $c\mapsto k$. Let $\Omega =\sum _{i=1}^6v^i\wedge v^{i+3}$ be as in section \ref{symplectic}. Each $\theta \in \wedge ^3 V^\ast $ determines a vector $v_\theta \in V$ defined by the following equation: \begin{equation} i_{v_\theta }(\Omega \wedge \Omega \wedge \Omega ) =\theta\wedge \Omega .\label{v_theta} \end{equation} Transforming the equation \eqref{v_theta} by $A\in Sp(\Omega )$ and recalling that $A\cdot \Omega =\Omega $, we obtain $A\cdot \left[ i_{v_\theta }(\Omega \wedge \Omega \wedge \Omega ) \right] =(A\cdot \theta )\wedge \Omega =i_{v_{A\cdot \theta }}(\Omega \wedge \Omega \wedge \Omega )$. Moreover, for every system $x_1,\dotsc,x_5\in V$ one has: \[ \begin{array} [c]{rl} \left( A\cdot \left[ i_{v_\theta }(\Omega \wedge \Omega \wedge \Omega ) \right] \right) \left( x_1,\dotsc,x_5\right) =\! & \! \left[ i_{v_\theta }(\Omega \wedge \Omega \wedge \Omega) \right] \left( A^{-1}x_1,\dotsc,A^{-1}x_5\right) \\ =\! & \!(\Omega \wedge \Omega \wedge \Omega ) \left( A^{-1}Av_\theta ,A^{-1} x_1,\dotsc,A^{-1}x_5\right) \\ =\! & \!\! \left( i_{Av_\theta }\left[ A\cdot (\Omega \wedge \Omega \wedge \Omega )\right] \right) \left( x_1,\dotsc,x_5\right) . \end{array} \] Hence $A\cdot \left[ i_{v_\theta }(\Omega \wedge \Omega \wedge \Omega ) \right] =i_{Av_\theta } \left[ A\cdot (\Omega \wedge \Omega \wedge \Omega ) \right] =i_{Av_\theta }(\Omega \wedge \Omega\wedge \Omega)$. Accordingly: $v_{A\cdot \theta }=Av_\theta $. Letting $x=v_\theta $ in \eqref{J}, it follows: \[ \Omega \left[ \left( i_{v_\theta }J^\theta \right) (y),z\right] =\Omega\left[ J^\theta (v_\theta ,y),z\right] =\theta (v_\theta ,y,z), \] and replacing $\theta $ by $A\cdot \theta $, $A\in Sp(\Omega )$, we have \begin{align*} \Omega \left[ \left( i_{v_{A\cdot \theta }}J^{A\cdot \theta } \right) (y),z \right] & =\Omega \left[ J^{A\cdot \theta }(v_{A\cdot \theta },y),z \right] =(A\cdot \theta )(v_{A\cdot \theta },y,z)\\ & =\theta \left( A^{-1}v_{A\cdot \theta },A^{-1}y,A^{-1}z \right) =\theta (v_\theta ,A^{-1}y,A^{-1}z)\\ & =\Omega \left[ J^\theta (v_\theta ,A^{-1}y),A^{-1}z \right] =\Omega \left[ (i_{v_\theta }J^\theta )(A^{-1}y),A^{-1}z \right] \\ & =\left( A\cdot \Omega \right) \left[ (i_{v_\theta }J^\theta )(y),z \right] \\ & =\Omega \left[ (i_{v_\theta }J^\theta )(y),z \right] . \end{align*} Hence $i_{v_{A\cdot \theta }}J^{A\cdot \theta } =i_{v_\theta }J^\theta $ for all $A\in Sp(\Omega )$. Accordingly, the endomorphism $L_\theta =i_{v_\theta }J^\theta \in V^\ast \otimes V$ is $Sp(\Omega )$-invariant. If $\theta $ is as in \eqref{theta} and $v_\theta =\sum _{h=1}^6x^hv_h$, then, as a computation shows, we have \begin{equation*} \begin{array} [c]{lll} x^1=-\tfrac{1}{6}(\lambda _{245}+\lambda _{346}), & x^2=\tfrac{1}{6}(\lambda _{145}-\lambda _{356}), & x^3=\tfrac{1}{6}(\lambda _{146}+\lambda _{256}), \medskip\\ x^4=-\tfrac{1}{6}(\lambda _{125}+\lambda _{136}), & x^5=-\tfrac{1}{6}(\lambda _{236}-\lambda _{124}), & x^6=\tfrac{1}{6}(\lambda _{134}+\lambda _{235}). \end{array} \end{equation*} Again writing $J^\theta =\sum _{b<c}\mu _{bc}^av^b\wedge v^c\otimes v_a$, we obtain \begin{equation*} \begin{array} [c]{lll} \mu_{ij}^1=\lambda _{ij4}, & \mu _{ij}^2=\lambda _{ij5}, & \mu _{ij}^3=\lambda _{ij6},\medskip\\ \mu _{ij}^4=-\lambda _{ij1}, & \mu _{ij}^{5}=-\lambda _{ij2}, & \mu_{ij}^6=-\lambda _{ij3}, \end{array} \end{equation*} or equivalently $\mu_{ij}^{h}=\lambda _{ij,h+1}$, $\mu_{ij}^{h+3}=-\lambda _{ij,h}$ for $1\leq h\leq3$. Therefore \[ L_\theta =i_{v_\theta }J^\theta =\sum\nolimits_{b<c}\mu_{bc}^a\left( x^bv^c-x^cv^b\right) \otimes v_a. \] Hence, the matrix $M=(M_{ia})_{i,a=1}^6$ of $L_\theta $ in the basis $v_1,\dotsc,v_6$ is given by $M_{ia}=\sum\nolimits_{b=1}^6\mu_{bi}^ax^b$, and as a computation shows, the characteristic polynomial of $L_\theta $ is written as follows: \[ \det \left( xI-L_\theta \right) =x^6+c_4(\theta)x^4+c_2(\theta )x^2, \] where \[ \begin{array} [c]{ll} c_2(\theta )=\frac{1}{2^4\cdot 3^4}(I_1)^2, & c_4(\theta )=-\frac{1}{2\cdot3^2}I_1, \end{array} \] and the explicit expression of $I_1$ is given in the Appendix. \subsection{$I_2$ defined\label{I_2}} For every $\theta \in \wedge ^3 V^\ast $ let $J^\theta \barwedge J^\theta \in\wedge ^3 V^\ast \otimes V$ be the tensor defined as follows: \[ \begin{array} [c]{ll} \left( J^\theta \barwedge J^\theta \right) \left( x_1,x_2,x_3\right) = & J^\theta \left( J^\theta \left( x_1,x_2\right) ,x_3\right) +J^\theta \left( J^\theta \left( x_2,x_3\right) ,x_1\right) \\ \multicolumn{1}{r}{} & \multicolumn{1}{r}{+J^\theta \left( J^\theta \left( x_3,x_1\right) ,x_2\right) ,}\\ \multicolumn{1}{r}{} & \multicolumn{1}{r}{\forall x_1,x_2,x_3\in V.} \end{array} \] As $J^\theta \left( v_i,v_j\right) =\mu_{ij}^hv_h$, we have $J^\theta \left( J^\theta \left( v_i,v_j\right) ,v_k\right) =\mu_{ij}^hJ^\theta \left( v_h,v_k\right) =\mu_{ij}^h\mu _{hk}^lv_l$. Hence \[ \left( J^\theta \barwedge J^\theta \right) \left( v_i,v_j,v_k\right) =\left( \mu_{ij}^h\mu_{hk}^l+\mu_{jk}^h\mu_{hi}^l +\mu_{ki}^h\mu_{hj}^l\right) v_l. \] Let $\Omega ^\theta \in\wedge ^6V^\ast $ be defined by $\Omega ^\theta =\tfrac{1}{3!^2}\operatorname{alt} \left[ \tilde{\Omega }\circ \left( J^\theta \barwedge J^\theta \otimes J^\theta \barwedge J^\theta \right) \right] $, where $\tilde{\Omega }\colon V\otimes V\to \mathbb{F}$ is the linear map attached to $\Omega $; i.e., $\tilde{\Omega }(x\otimes y)=\Omega (x,y)$, $\forall x,y\in V$, and $(J^\theta \barwedge J^\theta ) \otimes (J^\theta \barwedge J^\theta ) \colon \wedge ^3 V\otimes\wedge ^3V\to V\otimes V$ is the tensor product of $J^\theta \barwedge J^\theta $ and itself, $J^\theta \barwedge J^\theta $ being understood as a linear map $\wedge ^3 V\to V$, via the canonical isomorphism $\wedge ^3 V^\ast \otimes V=\operatorname{Hom} \left( \wedge ^3 V,V\right) $. Letting $\xi _{ijk}^l=\mu_{ij}^h\mu_{hk}^l+\mu_{jk}^h\mu_{hi}^l +\mu_{ki}^h\mu_{hj}^l$, we have \[ J^\theta \barwedge J^\theta =\sum_{i<j<k}\xi _{ijk}^lv^i\wedge v^j\wedge v^k\otimes v_l. \] Hence $\Omega ^\theta =\sum_{i<j<k}\sum_{a<b<c}\xi _{ijk}^l\xi _{abc}^d \left( v^i\wedge v^j\wedge v^k\right) \wedge \left( v^a\wedge v^b\wedge v^c\right) \Omega (v_l,v_d)$. The product $\left( v^i\wedge v^j\wedge v^k\right) \wedge \left( v^a\wedge v^b\wedge v^c\right) $ does not vanish if and only if the indices $i,j,k,a,b,c$ are pairwise distinct, i.e., $\{ i,j,k,a,b,c\} =\{ 1,\dotsc,6\} $, and in that case, $v^i\wedge v^j\wedge v^k\wedge v^a\wedge v^b\wedge v^c =\varepsilon _\sigma v^1\wedge v^2\wedge v^3\wedge v^4\wedge v^5\wedge v^6$, $\sigma $ being the permutation $\sigma (1)=i$, $\sigma (2)=j$, $\sigma (3)=k$, $\sigma (4)=a$, $\sigma (5)=b$, $\sigma (6)=c$. Therefore $\Omega ^\theta =I_2(\theta)\Omega \wedge \Omega \wedge \Omega $, where \begin{align*} I_2(\theta )\!& =\! -\tfrac{1}{6}\sum _{\sigma \in S_6} \varepsilon _\sigma \xi _{\sigma (1)\sigma (2)\sigma (3)}^l \xi _{\sigma (4)\sigma (5)\sigma (6)}^{d}\Omega (v_{l},v_{d}) \\ \!& =\! \tfrac{1}{6}\sum _{\sigma \in S_6} \varepsilon _\sigma \xi _{\sigma (1)\sigma (2)\sigma (3)}^4 \xi _{\sigma (4)\sigma (5)\sigma (6)}^{1} -\tfrac{1}{6}\sum _{\sigma \in S_6} \varepsilon _\sigma \xi _{\sigma (1)\sigma (2)\sigma (3)}^1 \xi _{\sigma (4)\sigma (5)\sigma (6)}^4 \\ & \!\! +\tfrac{1}{6}\sum_{\sigma \in S_{6}} \varepsilon _{\sigma }\xi _{\sigma (1)\sigma (2)\sigma (3)}^5 \xi _{\sigma (4)\sigma (5)\sigma (6)}^2 -\tfrac{1}{6}\sum_{\sigma \in S_6} \varepsilon _{\sigma }\xi _{\sigma (1)\sigma (2)\sigma (3)}^2 \xi _{\sigma (4)\sigma (5)\sigma (6)}^5 \\ & \!\! +\tfrac{1}{6}\sum _{\sigma \in S_6} \varepsilon _\sigma \xi _{\sigma (1)\sigma (2)\sigma (3)}^6 \xi _{\sigma (4)\sigma (5)\sigma (6)}^3 -\tfrac{1}{6}\sum_{\sigma \in S_{6}} \varepsilon _\sigma \xi _{\sigma (1)\sigma (2)\sigma (3)}^3 \xi _{\sigma (4)\sigma (5)\sigma (6)}^6. \end{align*} If $\sigma _0$ is the permutation $1\mapsto 4$, $2\mapsto 5$, $3\mapsto 6$, $4\mapsto 1$, $5\mapsto 2$, $6\mapsto3$, then letting $\sigma ^\prime =\sigma\circ\sigma_{0}$, we have $\sigma _0\circ \sigma _0=1$, $\varepsilon _{\sigma ^\prime} =\varepsilon _\sigma \varepsilon _{\sigma _0} =-\varepsilon _\sigma $, \[ \begin{array} [c]{lll} \sigma ^\prime (1)=\sigma (4), & \sigma ^\prime (2) =\sigma (5), & \sigma ^\prime (3)=\sigma (6),\\ \sigma ^\prime (4)=\sigma (1), & \sigma ^\prime (5) =\sigma (2), & \sigma ^\prime (6)=\sigma (3). \end{array} \] and from the previous formula for $I_2(\theta )$ we obtain \begin{equation*} \begin{array} [c]{rl} I_2(\theta)= & -\tfrac{1}{3}\sum _{\sigma \in S_6} \varepsilon _\sigma \xi _{\sigma (1)\sigma (2)\sigma (3)}^1 \xi _{\sigma (4)\sigma (5)\sigma (6)} ^4\smallskip\\ & -\tfrac{1}{3}\sum_{\sigma\in S_6} \varepsilon _\sigma \xi _{\sigma (1)\sigma (2)\sigma (3)}^2 \xi _{\sigma (4)\sigma (5)\sigma (6)}^5 \smallskip\\ & -\tfrac{1}{3}\sum_{\sigma \in S_6} \varepsilon _\sigma \xi _{\sigma (1)\sigma (2)\sigma (3)}^3 \xi _{\sigma (4)\sigma (5)\sigma (6)}^6. \end{array} \end{equation*} The explicit expression for $I_2$ is also given in the Appendix. \section{Infinitesimal criterion of invariance} The derivations \[ \begin{array} [c]{l} \tfrac{\partial }{\partial y_{abc}}\colon \mathbb{F}[\wedge ^3 V^\ast] \to \mathbb{F}[\wedge ^3 V^\ast ]\quad(\mathrm{resp.}\;\tfrac {\partial}{\partial y_{abc}}\colon\mathbb{F}(\wedge ^3 V^\ast ) \to \mathbb{F}(\wedge ^3 V^\ast )),\\ 1\leq a<b<c\leq 6, \end{array} \] are a basis of the $\mathbb{F}[\wedge ^3 V^\ast ]$-module (resp.\ $\mathbb{F}(\wedge ^3 V^\ast )$-vector space) $\operatorname{Der}\nolimits_{\mathbb{F}}\mathbb{F}[\wedge ^3 V^\ast ]$ (resp.\ $\operatorname{Der} \nolimits_{\mathbb{F}}\mathbb{F}(\wedge ^3 V^\ast )$). The subrings of $Sp(\Omega )$-invariant functions are denoted by $\mathbb{F}[\wedge ^3 V^\ast ]^{Sp(\Omega )}$ and $\mathbb{F}(\wedge ^3 V^\ast )^{Sp(\Omega )}$, respectively. \begin{lemma} \label{lemma_bis} Assume $V$ is a $\mathbb{F}$-vector space of dimension $6$ and let $\Omega\in\wedge ^2V^\ast $ be a non-degenerate $2$-covector. If $I\in\mathbb{F}[\wedge ^3 V^\ast ]$ (resp.\ $I\in\mathbb{F}(\wedge ^3V^\ast )$) is a $Sp(\Omega )$-invariant function, then $I$ is a common first integral of the following derivations: \begin{equation} \begin{array} [c]{l} U^\ast =\sum\nolimits_{1\leq h<i<j\leq 6} \left( \sum\nolimits_{1\leq a<b<c\leq 6}U_{hij}^{abc}y_{abc} \right) \tfrac{\partial}{\partial y_{hij}},\\ \forall U=(u_{ij})_{i,j=1}^6\in \mathfrak{sp}(6,\mathbb{F}), \end{array} \label{U^ast} \end{equation} where the functions $U_{hij}^{abc}$ are defined by \begin{equation*} U_{hij}^{abc}=-\left\vert \begin{array} [c]{ccc} u_{ha} & \delta_{hb} & \delta_{hc}\\ u_{ia} & \delta_{ib} & \delta_{ic}\\ u_{ja} & \delta_{jb} & \delta_{jc} \end{array} \right\vert -\left\vert \begin{array} [c]{ccc} \delta_{ha} & u_{hb} & \delta_{hc}\\ \delta_{ia} & u_{ib} & \delta_{ic}\\ \delta_{ja} & u_{jb} & \delta_{jc} \end{array} \right\vert -\left\vert \begin{array} [c]{ccc} \delta_{ha} & \delta_{hb} & u_{hc}\\ \delta_{ia} & \delta_{ib} & u_{ic}\\ \delta_{ja}^{{}} & \delta_{jb} & u_{jc} \end{array} \right\vert , \end{equation*} and $\delta$ denotes the Kronecker delta. \end{lemma} \begin{proof} We first observe that the derivations in \eqref{U^ast} are the image of the $\mathbb{F}$-homomorphism of Lie algebras $\rho _\ast ^\prime \colon \mathfrak{sp}(6,\mathbb{F})\to \operatorname{Der} \nolimits_{\mathbb{F}}\mathbb{F}[\wedge ^3 V^\ast ]$ induced by $\rho ^\prime $. Accordingly, we only need to show that $U^\ast (I)=0$ for the elements $U$ in a basis of the algebra $\mathfrak{sp}(6,\mathbb{F})$. If $(E_{ij})_{i,j=1}^6$ is the standard basis of $\mathfrak{gl}(6,\mathbb{F})$, then the matrices \begin{equation} \begin{array} [c]{llll} E_{11}+E_{14}-E_{41}-E_{44}, & E_{41}, & E_{52}, & E_{14},\\ E_{22}-E_{25}+E_{52}-E_{55}, & E_{42}+E_{51}, & E_{53}+E_{62}, & E_{25},\\ E_{33}-E_{66}+E_{36}-E_{63}, & E_{43}+E_{61}, & E_{63}, & E_{36},\\ E_{12}-E_{54}, & E_{31}-E_{46}, & E_{15}+E_{24}, & E_{26}+E_{35},\\ E_{21}-E_{45}, & E_{23}-E_{65}, & E_{16}+E_{34}, & E_{13}-E_{64},\\ E_{32}-E_{56}, & & & \end{array} \label{basis} \end{equation} are a basis $\mathcal{B}$ of $\mathfrak{sp}(6,\mathbb{F})$ with the following property: For every $U\in\mathcal{B}$, we have $U^2=0$, as a simple computation proves. Hence, for every $U\in\mathcal{B}$ and $t\in\mathbb{F}$, the endomorphism $I+tU$ ($I$ denoting the identity map of $V$) is symplectic. In fact, if $M_\Omega $, $M_U$ are the matrices of $\Omega $, $U$, respectively, then \begin{align*} \left( M_{I+tU}\right) ^{T}M_{\Omega}M_{I+tU} & =\left( I+t\left( M_U\right) ^T\right) M_\Omega \left( I+tM_U\right) \\ & =M_\Omega +t\left[ \left( M_U\right) ^TM_\Omega +M_\Omega M_U\right] +t^2\left( M_U\right) ^TM_\Omega M_U, \end{align*} but on one hand, we have $\left( M_U\right) ^TM_\Omega +M_\Omega M_{U}=0$, as $U\in\mathfrak{sp}(6,\mathbb{F})$, and on the other: $\left( M_U\right) ^TM_\Omega M_U=-M_\Omega M_{U^2}=0$. Hence $\left( M_{I+tU}\right) ^TM_\Omega M_{I+tU} =M_\Omega $, thus proving that $I+tU\in Sp(\Omega )$, $\forall t\in \mathbb{F}$. Therefore $I\left( (I+tU)\cdot \theta \right) =I(\theta )$, $\forall t\in\mathbb{F}$, $\forall U=(u_{ij})\in \mathcal{B}$ and $\forall t\in \mathbb{F} $. If $\Lambda(t)=(\lambda _{ij}(t))_{i,j=1}^6=I-tU^T$, then \[ I\Bigl( \sum\nolimits_{\substack{1\leq a<b<c\leq 6 \\1\leq h<i<j\leq 6}}~y_{abc} \left\vert \begin{array} [c]{ccc} \lambda _{ah}(t) & \lambda _{bh}(t) & \lambda _{ch}(t)\\ \lambda _{ai}(t) & \lambda _{bi}(t) & \lambda _{ci}(t)\\ \lambda _{aj}(t) & \lambda _{bj}(t) & \lambda _{cj}(t) \end{array} \right\vert v^{h}\wedge v^i\wedge v^{j} \Bigr) =I(\theta ), \] and taking derivatives at $t=0$, we obtain \[ 0=\sum\nolimits_{1\leq a<b<c\leq 6,1\leq h<i<j\leq6} U_{hij}^{abc}y_{abc} \tfrac{\partial I}{\partial y_{hij}}(\theta). \] \end{proof} \begin{definition} Let $F$ be the field of fractions of an entire ring $R$. The \emph{generic rank} of a finitely-generated $R$-module $\mathcal{M}$ is the dimension of the $F$-vector space $F\otimes_r\mathcal{M}$. \end{definition} \begin{theorem} \label{theorem} The generic rank of the $\mathbb{F}[\wedge ^3 V^\ast ]$-module $\mathcal{M}$ spanned by the derivations in the formula \emph{\eqref{U^ast}} of \emph{Lemma \ref{lemma_bis}} is $18$. \end{theorem} \begin{proof} If $U=(u_{ij})_{i,j=1}^6\in\mathfrak{sp}(\Omega)$ then \[ \begin{array} [c]{lllll} u_{24}=u_{15}, & u_{34}=u_{16}, & u_{35}=u_{26}, & u_{51}=u_{42}, & u_{61}=u_{43},\\ u_{62}=u_{53}, & u_{44}=-u_{11}, & u_{45}=-u_{21}, & u_{46}=-u_{31}, & u_{54}=-u_{12},\\ \multicolumn{1}{r}{u_{55}=-u_{22},} & \multicolumn{1}{r}{u_{56}=-u_{32},} & \multicolumn{1}{r}{u_{64}=-u_{13},} & \multicolumn{1}{r}{u_{65}=-u_{23},} & \multicolumn{1}{r}{u_{66}=-u_{33},} \end{array} \] and the following $21$ functions are coordinates on the symplectic algebra $\mathfrak{sp}(\Omega )$: $u_{11}$, $u_{12}$, $u_{13}$, $u_{14}$, $u_{15}$, $u_{16}$, $u_{21}$, $u_{22}$, $u_{23}$, $u_{25}$, $u_{26}$, $u_{31}$, $u_{32}$, $u_{33}$, $u_{36}$, $u_{41}$, $u_{42}$, $u_{43}$, $u_{52}$, $u_{53}$, $u_{63}$. Hence we can write: \[ \begin{array} [c]{rl} U^\ast \! = & \!\!\!\! u_{11}Z_{11}+u_{12}Z_{12}+u_{13}Z_{13}+u_{14}Z_{14} +u_{15}Z_{15}+u_{16}Z_{16}+u_{21}Z_{21}\\ \! & \multicolumn{1}{r}{\!\!\!\! +u_{22}Z_{22}+u_{23}Z_{23}+u_{25}Z_{25}+u_{26}Z_{26} +u_{31}Z_{31}+u_{32}Z_{32}+u_{33}Z_{33}}\\ \! & \multicolumn{1}{r}{\!\!\!\!+u_{36}Z_{36} +u_{41}Z_{41}+u_{42}Z_{42}+u_{43}Z_{43} +u_{52}Z_{52}+u_{53}Z_{53}+u_{63}Z_{63},} \end{array} \] where \begin{equation*} \begin{array}{rc} Z_{11}= & -y_{123}Y_{123}-y_{125}Y_{125}-y_{126}Y_{126} -y_{135}Y_{135}-y_{136}Y_{136}-y_{156}Y_{156} \\ & +y_{234}Y_{234}+y_{245}Y_{245}+y_{246}Y_{246} +y_{345}Y_{345}+y_{346}Y_{346}+y_{456}Y_{456}, \\ Z_{12}= & y_{124}Y_{125}-y_{234}Y_{134}+y_{134}Y_{135} -y_{235}Y_{135}-y_{236}Y_{136}-y_{245}Y_{145} \\ & -y_{246}Y_{146}+y_{146}Y_{156}-y_{256}Y_{156} +y_{234}Y_{235}+y_{246}Y_{256}+y_{346}Y_{356}, \\ Z_{13}= & y_{234}Y_{124}+y_{235}Y_{125}+y_{124}Y_{126} +y_{236}Y_{126}+y_{134}Y_{136}-y_{345}Y_{145} \\ & -y_{346}Y_{146}-y_{145}Y_{156}-y_{356}Y_{156} +y_{234}Y_{236}-y_{245}Y_{256}-y_{345}Y_{356}, \\ Z_{14}= & -y_{234}Y_{123}+y_{245}Y_{125}+y_{246}Y_{126} +y_{345}Y_{135}+y_{346}Y_{136}-y_{456}Y_{156}, \\ Z_{15}= & y_{134}Y_{123}-y_{235}Y_{123}-y_{245}Y_{124} -y_{145}Y_{125}-y_{146}Y_{126}+y_{256}Y_{126} \\ & -y_{345}Y_{134}+y_{356}Y_{136}+y_{456}Y_{146} +y_{345}Y_{235}+y_{346}Y_{236}-y_{456}Y_{256}, \end{array} \end{equation*} \begin{equation*} \begin{array}{rc} Z_{16}= & -y_{124}Y_{123}-y_{236}Y_{123}-y_{246}Y_{124} -y_{256}Y_{125}-y_{346}Y_{134}-y_{145}Y_{135} \\ & -y_{356}Y_{135}-y_{146}Y_{136}-y_{456}Y_{145} -y_{245}Y_{235}-y_{246}Y_{236}-y_{456}Y_{356}, \\ Z_{21}= & y_{125}Y_{124}+y_{135}Y_{134}+y_{156}Y_{146} -y_{134}Y_{234}+y_{235}Y_{234}-y_{135}Y_{235} \\ & -y_{136}Y_{236}-y_{145}Y_{245}-y_{146}Y_{246} +y_{256}Y_{246}-y_{156}Y_{256}+y_{356}Y_{346}, \\ Z_{22}= & -y_{123}Y_{123}-y_{124}Y_{124}-y_{126}Y_{126} +y_{135}Y_{135}+y_{145}Y_{145}+y_{156}Y_{156} \\ & -y_{234}Y_{234}-y_{236}Y_{236}-y_{246}Y_{246} +y_{345}Y_{345}+y_{356}Y_{356}+y_{456}Y_{456}, \\ Z_{23}= & -y_{134}Y_{124}-y_{135}Y_{125}+y_{125}Y_{126} -y_{136}Y_{126}+y_{135}Y_{136}+y_{145}Y_{146} \\ & +y_{235}Y_{236}-y_{345}Y_{245}+y_{245}Y_{246} -y_{346}Y_{246}-y_{356}Y_{256}+y_{345}Y_{346}, \\ Z_{25}= & y_{135}Y_{123}+y_{145}Y_{124}-y_{156}Y_{126} -y_{345}Y_{234}+y_{356}Y_{236}+y_{456}Y_{246}, \\ Z_{26}= & -y_{125}Y_{123}+y_{136}Y_{123}+y_{146}Y_{124} +y_{156}Y_{125}+y_{145}Y_{134}-y_{156}Y_{136} \\ & +y_{245}Y_{234}-y_{346}Y_{234}-y_{356}Y_{235} -y_{256}Y_{236}-y_{456}Y_{245}+y_{456}Y_{346}, \\ Z_{31}= & y_{126}Y_{124}+y_{136}Y_{134}-y_{156}Y_{145} +y_{124}Y_{234}+y_{236}Y_{234}+y_{125}Y_{235} \\ & +y_{126}Y_{236}-y_{256}Y_{245}-y_{145}Y_{345} -y_{356}Y_{345}-y_{146}Y_{346}-y_{156}Y_{356}, \end{array} \end{equation*} \[ \begin{array} [c]{rl} Z_{32}= & y_{126}Y_{125}-y_{124}Y_{134}-y_{125}Y_{135} +y_{136}Y_{135}-y_{126}Y_{136}+y_{146}Y_{145}\\ & \multicolumn{1}{r}{+y_{236}Y_{235}+y_{246}Y_{245}-y_{245}Y_{345} +y_{346}Y_{345}-y_{246}Y_{346}-y_{256}Y_{356},}\\ Z_{33}= & -y_{123}Y_{123}+y_{126}Y_{126}-y_{134}Y_{134} -y_{135}Y_{135}+y_{146}Y_{146}+y_{156}Y_{156}\\ & \multicolumn{1}{r}{-y_{234}Y_{234}-y_{235}Y_{235}+y_{246}Y_{246} +y_{256}Y_{256}-y_{345}Y_{345}+y_{456}Y_{456},}\\ Z_{36}= & -y_{126}Y_{123}+y_{146}Y_{134}+y_{156}Y_{135}+y_{246}Y_{234} +y_{256}Y_{235}-y_{456}Y_{345},\\ Z_{41}= & -y_{123}Y_{234}+y_{125}Y_{245}+y_{126}Y_{246}+y_{135}Y_{345} +y_{136}Y_{346}-y_{156}Y_{456},\\ Z_{42}= & y_{123}Y_{134}-y_{125}Y_{145}-y_{126}Y_{146}-y_{123}Y_{235} -y_{124}Y_{245}+y_{126}Y_{256}\\ & \multicolumn{1}{r}{-y_{134}Y_{345}+y_{235}Y_{345}+y_{236}Y_{346} +y_{136}Y_{356}+y_{146}Y_{456}-y_{256}Y_{456},}\\ Z_{43}= & -y_{123}Y_{124}-y_{135}Y_{145}-y_{136}Y_{146}-y_{123}Y_{236} -y_{235}Y_{245}-y_{124}Y_{246}\\ & \multicolumn{1}{r}{-y_{236}Y_{246}-y_{125}Y_{256}-y_{134}Y_{346} -y_{135}Y_{356}-y_{145}Y_{456}-y_{356}Y_{456},}\\ Z_{52}= & y_{123}Y_{135}+y_{124}Y_{145}-y_{126}Y_{156}-y_{234}Y_{345} +y_{236}Y_{356}+y_{246}Y_{456},\\ Z_{53}= & -y_{123}Y_{125}+y_{123}Y_{136}+y_{134}Y_{145}+y_{124}Y_{146} +y_{125}Y_{156}-y_{136}Y_{156}\\ & +y_{234}Y_{245}-y_{236}Y_{256}-y_{234}Y_{346}-y_{235}Y_{356}-y_{245} Y_{456}+y_{346}Y_{456},\\ Z_{63}= & -y_{123}Y_{126}+y_{134}Y_{146}+y_{135}Y_{156}+y_{234}Y_{246} +y_{235}Y_{256}-y_{345}Y_{456}, \end{array} \] where $Y_{abc}=\frac{\partial }{\partial y_{abc}}$, $1\leq a<b<c\leq6$, is the standard basis of derivations. Moreover, the invariant functions $I_1$ and $I_2$ are algebraically independent. In fact, by using the formulas for $I_1$ and $I_2$ in the Appendix, after a computation, it follows that the determinant \[ \left\vert \begin{array} [c]{cc} dI_1(Y_{123}) & dI_1(Y_{126})\\ dI_2(Y_{123}) & dI_2(Y_{126}) \end{array} \right\vert \] at the point \[ \begin{array} [c]{ccccccc} \lambda _{123}=1, & \lambda _{124}=0, & \lambda _{125}=1, & \lambda _{126}=0, & \lambda _{134}=1, & \lambda _{135}=0, & \lambda _{136}=0,\\ \lambda _{145}=0, & \lambda _{146}=0, & \lambda _{156}=0, & \lambda _{234}=0, & \lambda _{235}=0, & \lambda _{236}=0, & \lambda _{245}=0,\\ \lambda _{246}=0, & \lambda _{256}=0, & \lambda _{345}=0, & \lambda _{346}=0, & \lambda _{356}=0, & \lambda _{456}=1, & \end{array} \] takes the value $2^4\cdot 3^2$. As the differentials $dI_1$ and $dI_2$ vanish over all the vector fields $Z_{11},\dotsc,Z_{63}$, we deduce that the generic rank of $\mathcal{M}$ is $\leq 18$. Moreover, it is easy to obtain values of the variables $y_{abc}$, $1\leq a<b<c\leq 6$, for which the matrix of $Z_{11},\dotsc,Z_{63}$ in the basis $Y_{abc}$, $1\leq a<b<c\leq6$, is $18$, and thus we can conclude the proof. \end{proof} \begin{corollary} If $\mathbb{F}$ is an algebraically closed field of characteristic zero, then $\mathbb{F}[\wedge ^3 V^\ast ]^{Sp(\Omega )} =\mathbb{F}[I_1,I_2]$. \end{corollary} \begin{proof} The result is a direct consequence of Theorem \ref{theorem} and \cite[\textsc{Th\'eor\`{e}me} 1-I]{KPV}. \end{proof} \section{Normal forms and invariants} In the series of papers \cite{BK1}, \cite{BK2}, \cite{BK3}, \cite{BK4} and \cite{BK5}, the normal forms for equivalence classes of trivectors over a $6$-dimensional vector space over an arbitrary field $\mathbb{F}$ of characteristic distinct from $2$ and $3$ under the symplectic group, is given. According to \cite[Theorem 2.1]{BK5} every non-zero trivector of $V$ is equivalent with (at least) one of the following trivectors: \[ \begin{array} [c]{ll} \chi _{A_1}=\bar{e}_1^\ast \wedge \bar{e}_2^\ast \wedge \bar{e}_3^\ast , & \chi _{A_2}=\bar{e}_1^\ast \wedge \bar{e}_2^\ast \wedge \bar{f}_2^\ast , \end{array} \] \[ \begin{array} [c]{rl} \chi _{B_1}= & \bar{e}_1^\ast \wedge \bar{e}_2^\ast \wedge \bar{e}_3^\ast +\bar{e}_1^\ast \wedge \bar{f}_1^\ast \wedge \bar{f}_3^\ast ,\\ \chi _{B_2}= & \bar{e}_1^\ast \wedge \bar{e}_2^\ast \wedge \bar{f}_2^\ast +\bar{e}_1^\ast \wedge \bar{f}_1^\ast \wedge \bar{e}_3^\ast,\\ \chi _{B_3}= & \bar{e}_1^\ast \wedge \bar{e}_2^\ast \wedge \bar{f}_2^\ast +\bar{e}_1^\ast \wedge \bar{e}_3^\ast \wedge \bar{f}_3^\ast ,\\ \chi _{B_4}(\lambda )= & \bar{e}_1^\ast \wedge \bar{e}_2^\ast \wedge \bar{e}_3^\ast +\lambda\bar{e}_1^\ast \wedge \bar{f}_2^\ast \wedge \bar{f}_3^\ast ,\\ \chi _{B_5}(\lambda )= & \lambda\bar{e}_1^\ast \wedge \bar{e}_2^\ast \wedge \bar{f}_2^\ast +\bar{e}_1^\ast \wedge (\bar{e}_2^\ast -\bar{e}_3^\ast )\wedge (\bar{f}_2^\ast +\bar{f}_3^\ast ), \end{array} \] \[ \begin{array} [c]{rl} \chi _{C_1}(\lambda )= & \bar{e}_1^\ast \wedge \bar{e}_2^\ast \wedge \bar{e}_3^\ast +\lambda\bar{f}_1^\ast \wedge \bar{f}_2^\ast \wedge \bar{f}_3^\ast ,\\ \chi _{C_2}(\lambda)= & \bar{f}_1^\ast \wedge (\bar{e}_2^\ast +\bar{e}_3^\ast ) \wedge (\bar{f}_2^\ast -\bar{f}_3^\ast ) +\lambda \bar{e}_1^\ast \wedge \bar{e}_2^\ast \wedge \bar{f}_2^\ast ,\\ \chi _{C_3}(\lambda)= & \bar{e}_1^\ast \wedge \bar{e}_2^\ast \wedge\bar{f}_2^\ast +\lambda\bar{f}_1^\ast \wedge \bar{e}_3^\ast \wedge\bar{f}_3^\ast ,\\ \chi _{C_4}(\lambda)= & \bar{f}_1^\ast \wedge \bar{e}_3^\ast \wedge(\bar{e}_2^\ast +\bar{f}_3^\ast )+\lambda\bar{e}_1^\ast \wedge \bar{e}_2^\ast \wedge \bar{f}_2^\ast ,\\ \chi _{C_5}(\lambda )= & \bar{e}_1^\ast \wedge \bar{e}_3^\ast \wedge (\bar{f}_2^\ast +\bar{f}_3^\ast ) +\lambda \bar{e}_2^\ast \wedge \bar{f}_3^\ast \wedge (\bar{f}_1^\ast +\bar{e}_3^\ast ),\\ \chi _{C_6}(\lambda,\varepsilon)= & \bar{f}_1^\ast \wedge (\bar{e}_2^\ast +\bar{e}_3^\ast ) \wedge (\bar{f}_2^\ast +\varepsilon \bar{f}_3^\ast) +\lambda \bar{e}_1^\ast \wedge \bar{e}_2^\ast \wedge \bar{f}_2^\ast , \end{array} \] \[ \begin{array} [c]{rl} \chi _{D_1}= & \bar{e}_1^\ast \wedge \bar{e}_2^\ast \wedge \bar{f}_2^\ast +\bar{e}_2^\ast \wedge \bar{f}_1^\ast \wedge \bar{e}_3^\ast +\bar{f}_1^\ast \wedge \bar{e}_1^\ast \wedge \bar{f}_3^\ast ,\\ \chi _{D_2}(\lambda)= & \lambda \bar{e}_1^\ast \wedge \bar{e}_2^\ast \wedge \bar{f}_3^\ast +\bar{e}_2^\ast \wedge \bar{f}_1^\ast \wedge \bar{e}_3^\ast +\bar{f}_1^\ast \wedge \bar{e}_1^\ast \wedge \bar{f}_2^\ast ,\\ \chi _{D_3}(\lambda _1,\lambda _2)= & \bar{e}_1^\ast \wedge \bar{e}_2^\ast \wedge \bar{f}_3^\ast +\lambda _1\bar{e}_2^\ast \wedge \bar{e}_3^\ast \wedge \bar{f}_1^\ast +\lambda _2\bar{e}_3^\ast \wedge \bar{e}_1^\ast \wedge \bar{f}_2^\ast ,\\ \chi _{D_4}(\lambda _1,\lambda _2)= & \bar{e}_1^\ast \wedge \bar{e}_2^\ast \wedge \bar{f}_3^\ast +\lambda _1\bar{e}_2^\ast \wedge \bar{e}_3^\ast \wedge (\bar{f}_1^\ast +\bar{f}_3^\ast ) +\lambda _2\bar{e}_3^\ast \wedge \bar{e}_1^\ast \wedge \bar{f}_2^\ast ,\\ \chi _{D_5}(\lambda)= & \bar{e}_1^\ast \wedge \bar{e}_2^\ast \wedge \bar{f}_3^\ast +\lambda \bar{e}_2^\ast \wedge \bar{e}_3^\ast \wedge (\bar{f}_1^\ast +\bar{f}_2^\ast +\bar{f}_3^\ast ) -\bar{e}_3^\ast \wedge \bar{e}_1^\ast \wedge \bar{f}_2^\ast ,\\ \chi _{D_6}= & -\bar{e}_1^\ast \wedge \bar{e}_2^\ast \wedge \bar{f}_2^\ast +\bar{e}_2^\ast \wedge \bar{e}_3^\ast \wedge \bar{f}_1^{\ast }+\bar{e}_3^\ast \wedge \bar{e}_1^\ast \wedge \bar{f}_3^\ast , \end{array} \] \[ \begin{array} [c]{rl} \chi _{E_1}(a,b,h_1,h_2,h_3) = & 2\bar{e}_1^\ast \wedge \bar{e}_2^\ast \wedge \bar{e}_3^\ast \\ & \multicolumn{1}{r}{+a\left( h_1\bar{f}_1^\ast \wedge \bar{e}_2^\ast \wedge \bar{e}_3^\ast +h_2\bar{e}_1^\ast \wedge \bar{f}_2^\ast \wedge \bar{e}_3^\ast +h_3\bar{e}_1^\ast \wedge \bar{e}_2^\ast \wedge \bar{f}_3^\ast \right) }\\ & \multicolumn{1}{r}{+(a^2+2b)\left( h_1h_2\bar{f}_1^\ast \wedge \bar{f}_2^\ast \wedge \bar{e}_3^\ast +h_1h_3\bar{f}_1^\ast \wedge \bar{e}_2^\ast \wedge \bar{f}_3^\ast \right. }\\ & \multicolumn{1}{r}{\left. +h_2h_3\bar{e}_1^\ast \wedge \bar{f}_2^\ast \wedge \bar{f}_3^\ast \right) +h_1h_2h_3(a^2+3b)\bar{f}_1^\ast \wedge \bar{f}_2^\ast \wedge \bar{f}_3^\ast ,} \end{array} \] \[ \begin{array} [c]{rl} \chi _{E_2}(a,b,k)= & \bar{e}_1^\ast \wedge \bar{e}_2^\ast \wedge \bar{f}_2^\ast +\bar{e}_1^\ast \wedge \bar{e}_3^\ast \wedge \bar{f}_3^\ast \\ & \multicolumn{1}{r}{+k\left( \bar{f}_1^\ast \wedge \bar{e}_2^\ast \wedge \bar{f}_3^\ast -b\bar{f}_1^\ast \wedge \bar{f}_2^\ast \wedge \bar{e}_3^\ast +a\bar{f}_1^\ast \wedge \bar{e}_3^\ast \wedge \bar{f}_3^\ast \right) ,\medskip}\\ \chi _{E_3}(a,b,k,h)= & \bar{e}_1^\ast \wedge \bar{e}_2^\ast \wedge \bar{f}_2^\ast +\bar{e}_1^\ast \wedge \bar{e}_3^\ast \wedge \bar{f}_3^\ast +k\left( \bar{f}_1^\ast \wedge \bar{e}_2^\ast \wedge \bar{f}_3^\ast -b\bar{f}_1^\ast \wedge \bar{f}_2^\ast \wedge \bar{e}_3^\ast \right. \\ & \multicolumn{1}{r}{\left. +a\bar{f}_1^\ast \wedge \bar{e}_3^\ast \wedge \bar{f}_3^\ast \right) +h\bar{e}_1^\ast \wedge \bar{f}_2^\ast \wedge \bar{f}_3^\ast ,} \end{array} \] \[ \begin{array} [c]{rl} \chi _{E_4}(a,b,k,h_1,h_2)= & \left[ 1-h_1h_2(a^2+4b)\right] \bar{e}_1^\ast \wedge \bar{e}_2^\ast \wedge \bar{f}_2^\ast \\ & +\left[ 1+h_1h_2(a^2+4b)\right] \bar{e}_1^\ast \wedge \bar{e}_3^\ast \wedge \bar{f}_3^\ast \\ & +k\left[ \bar{f}_1^\ast \wedge \bar{e}_2^\ast \wedge \bar{f}_3^\ast -b(1-h_1h_2(a^2+4b)) \bar{f}_1^\ast \wedge \bar{f}_2^\ast \wedge \bar{e}_3^\ast \right. \\ & \left. +a\bar{f}_1^\ast \wedge \bar{e}_3^\ast \wedge \bar{f}_3^\ast \right] +h_1\left[ 1-h_1h_2(a^2+4b)\right] \bar{e}_1^\ast \wedge \bar{f}_2^\ast \wedge \bar{f}_3^\ast \\ & \multicolumn{1}{r}{+(a^2+4b)h_2\bar{e}_1^\ast \wedge \bar{e}_2^\ast \wedge \bar{e}_3^\ast ,}\\ & h_1h_2(a^2+4b)\neq 1, \end{array} \] \[ \begin{array} [c]{rl} \chi _{E_5}(a,b,k)= & \bar{f}_1^\ast \wedge \bar{e}_2^\ast \wedge \bar{f}_3^\ast +2\bar{e}_1^\ast \wedge \bar{f}_1^\ast \wedge \bar{e}_2^\ast -a\bar{f}_1^\ast \wedge \bar{e}_2^\ast \wedge \bar{f}_2^\ast \\ & \multicolumn{1}{r}{+a\bar{f}_1^\ast \wedge \bar{e}_3^\ast \wedge \bar{f}_3^\ast +a\bar{e}_1^\ast \wedge \bar{f}_1^\ast \wedge \bar{e}_3^\ast +(a^2+b)\bar{f}_1^\ast \wedge \bar{f}_2^\ast \wedge \bar{e}_3^\ast }\\ & \multicolumn{1}{r}{+k\left( a\bar{e}_1^\ast \wedge \bar{f}_2^{\ast }\wedge \bar{e}_3^\ast -\bar{e}_1^\ast \wedge \bar{e}_2^\ast \wedge \bar{f}_2^\ast +\bar{e}_1^\ast \wedge \bar{e}_3^\ast \wedge \bar{f}_3^\ast \right) ,} \end{array} \] (As we are assuming that $\mathbb{F}$ has characteristic distinct from $2$ and $3$, we do not consider the trivectors of types $(E1^\prime )$, $(E2^\prime )$, and $(E3^\prime )$, because they only exist when the characteristic is $2$; indeed, for such trivectors one needs separable quadratic extensions of $\mathbb{F}$, which can only exist in the case of characteristic $2$.) With the same notations as above, we have $\bar{e}_1^\ast =v^1$, $\bar{e}_2^\ast =v^2$, $\bar{e}_3^\ast =v^3$, $\bar{f}_1^\ast =v^4$, $\bar{f}_2^\ast =v^{5}$, $\bar{f}_3^\ast =v^6$. In this section we study the evaluation map $(I_1,I_2)\colon \wedge ^3V^\ast /GL(V) \to \mathbb{F}^2$, by using the normal forms given in \cite{BK5}. This throws light on the quotient space. We have \[ \begin{array}{llll} I_j(\chi _{A_h})=0, \quad I_j(\chi _{B_i})=0, & h,j=1,2, & 1\leq i\leq 5,\\ I_1(\chi _{C_h})=0, & I_2(\chi _{C_h})=-2^3\cdot 3^2\lambda ^2, & 1\leq h\leq 2, & \\ I_1(\chi _{C_h})=\lambda ^2, & I_2(\chi _{C_h}) =2^4\cdot 3\lambda ^2, & 3\leq h\leq 4, & \\ I_1(\chi _{C_5})=0, & I_2(\chi _{C_5}) =-2^3\cdot 3^2\lambda ^2, & & \\ I_1(\chi _{C_6})=\lambda ^2 \varepsilon (\varepsilon +1), & I_2(\chi _{C_6})=2^3\cdot 3\lambda ^2 \varepsilon (2\varepsilon +5), & \varepsilon \neq 0,-1, & \\ I_j(\chi _{D_h})=0, & h\in \{ 1,3,4,5,6\} , & 1\leq j\leq 2, & \\ I_1(\chi _{D_2})=\lambda , & I_2(\chi _{D_2}) =2^3\cdot 3\cdot 5\lambda , & & \\ I_1(\chi _{E_1})=0, & & & \end{array} \] \begin{align*} I_2(\chi _{E_1})&=-2^3\cdot 3^2(h_1h_2h_3)^2 \left( 2^2 (a^2+3b)^2(1-2a)+(a^2+2b)^2(5a^2+2^4 b) \right) , \quad \\ I_1(\chi _{E_{j}})&=k^2(a^2+4 b), \qquad \qquad I_2(\chi _{E_j}) =2^4\cdot 3k^2(a^2+4b), \quad 2\leq j\leq 3,\\ I_1(\chi _{E_4}) & =k^2 (a^2+4b)\left( 1-(a^2+4 b) h_1 h_2 \right) , \qquad \qquad \qquad \qquad\\ I_2(\chi _{E_4}) & =2^3\cdot 3 k^2 (a^2+4 b) \left( 1-(a^2+4 b)h_1 h_2 \right) (2+3 (a^2 + 4 b) h_1 h_2), \\ I_1(\chi _{E_5})&=0, \qquad I_2(\chi _{E_5})=-2^3 \cdot 3^2 k^2 (a^2+4 b) \qquad \qquad \qquad \end{align*} When $\mathbb{F}$ is an algebraically closed field of characteristic distinct from $2$, Popov \cite{P} gave another alternative for the normal forms for equivalence classes of trivectors over a $6$-dimensional vector space over $\mathbb{F}$ under the symplectic group. We list these normal forms, following the notations in \cite[Theorem 2.1]{BK5}, with $q,p\in\mathbb{F}^\ast $: \[ \begin{array} [c]{rl} \chi _{P_1}= & 0,\\ \chi _{P_2}= & \bar{e}_1^\ast \wedge \bar{e}_2^\ast \wedge \bar{f}_2^\ast +\bar{e}_1^\ast \wedge \bar{e}_3^\ast \wedge \bar{f}_3^\ast ,\\ \chi _{P_3}(q)= & \bar{e}_1^\ast \wedge \bar{e}_2^\ast \wedge \bar{e}_3^\ast +q\bar{f}_1^\ast \wedge \bar{f}_2^\ast \wedge \bar{f}_3^\ast ,\\ \chi _{P_4}(q)= & \bar{e}_1^\ast \wedge \bar{e}_2^\ast \wedge \bar{e}_3^\ast +q\bar{f}_1^\ast \wedge \bar{f}_2^\ast \wedge \bar{f}_3^\ast +\bar{e}_1^\ast \wedge \bar{e}_2^\ast \wedge \bar{f}_2^\ast +\bar{e}_1^\ast \wedge \bar{e}_3^\ast \wedge \bar{f}_3^\ast ,\\ \chi _{P_5}(q)= & \bar{e}_1^\ast \wedge \bar{e}_2^\ast \wedge \bar{e} _3^\ast +q\bar{f}_1^\ast \wedge \bar{f}_2^\ast \wedge \bar{f}_3^\ast +\bar{e}_1^\ast \wedge \bar{e}_2^\ast \wedge \bar{f}_2^\ast +\bar{e}_1^\ast \wedge \bar{e}_3^\ast \wedge \bar{f}_3^\ast \\ & +\bar{f}_2^\ast \wedge \bar{e}_1^\ast \wedge \bar{f}_1^\ast +\bar{f}_2^\ast \wedge \bar{e}_3^\ast \wedge \bar{f}_3^\ast ,\\ \chi _{P_6}(q,p)= & \bar{e}_1^\ast \wedge \bar{e}_2^\ast \wedge \bar{e}_3^\ast +q\bar{f}_1^\ast \wedge \bar{f}_2^\ast \wedge \bar{f}_3^\ast +\bar{e}_1^\ast \wedge \bar{e}_2^\ast \wedge \bar{f}_2^\ast +\bar{e}_1^\ast \wedge \bar{e}_3^\ast \wedge \bar{f}_3^\ast \\ & +p\bar{f}_1^\ast \wedge \bar{e}_2^\ast \wedge \bar{f}_2^\ast +p\bar{f}_1^\ast \wedge \bar{e}_3^\ast \wedge \bar{f}_3^\ast ,\\ \chi _{P_{7}}= & \bar{e}_1^\ast \wedge \bar{e}_2^\ast \wedge \bar{e}_3^\ast ,\\ \chi _{P_{8}}= & \bar{e}_1^\ast \wedge \bar{e}_2^\ast \wedge \bar{e}_3^\ast +\bar{e}_1^\ast \wedge \bar{e}_2^\ast \wedge \bar{f}_2^\ast +\bar{e}_1^\ast \wedge \bar{e}_3^\ast \wedge \bar{f}_3^\ast ,\\ \chi _{P_{9}}= & \bar{e}_1^\ast \wedge \bar{e}_2^\ast \wedge \bar{e} _3^\ast +\bar{f}_1^\ast \wedge \bar{e}_2^\ast \wedge \bar{f}_2^\ast+\bar{f}_1^\ast \wedge \bar{e}_3^\ast \wedge \bar{f}_3^\ast , \end{array} \] \[ \begin{array} [c]{rl} \chi _{P_{10}}= & \bar{e}_1^\ast \wedge \bar{f}_1^\ast \wedge \bar{e}_3^\ast +\bar{f}_2^\ast \wedge \bar{e}_2^\ast \wedge \bar{e}_3^\ast +\bar{e}_1^\ast \wedge \bar{e}_2^\ast \wedge \bar{f}_3^\ast ,\\ \chi _{P_{11}}= & \bar{e}_1^\ast \wedge \bar{f}_1^\ast \wedge \bar{e}_3^\ast +\bar{f}_2^\ast \wedge \bar{e}_2^\ast \wedge \bar{e}_3^\ast +\bar{e}_1^\ast \wedge \bar{e}_2^\ast \wedge \bar{f}_3^\ast +\bar{e}_1^\ast \wedge \bar{e}_2^\ast \wedge \bar{f}_2^\ast \\ & +\bar{e}_1^\ast \wedge \bar{e}_3^\ast \wedge \bar{f}_3^\ast ,\\ \chi _{P_{12}}(q)= & \bar{e}_1^\ast \wedge \bar{f}_1^\ast \wedge \bar{e}_3^\ast +\bar{f}_2^\ast \wedge \bar{e}_2^\ast \wedge \bar{e} _3^\ast +\bar{e}_1^\ast \wedge \bar{e}_2^\ast \wedge \bar{f}_3^\ast +q\bar{e}_3^\ast \wedge \bar{e}_1^\ast \wedge \bar{f}_1^\ast \\ & \multicolumn{1}{r}{+q\bar{e}_3^\ast \wedge \bar{e}_2^\ast \wedge \bar{f}_2^\ast ,}\\ \chi _{P_{13}}= & \bar{e}_1^\ast \wedge \bar{f}_1^\ast \wedge \bar{e}_3^\ast +\bar{f}_2^\ast \wedge \bar{e}_2^\ast \wedge \bar{e}_3^\ast +\bar{e}_1^\ast \wedge \bar{e}_2^\ast \wedge \bar{f}_3^\ast +\bar{f}_1^\ast \wedge \bar{e}_2^\ast \wedge \bar{f}_2^\ast \\ & \multicolumn{1}{r}{+\bar{f}_1^\ast \wedge \bar{e}_3^\ast \wedge \bar{f}_3^\ast ,}\\ \chi _{P_{14}}(q)= & \bar{e}_1^\ast \wedge \bar{f}_1^\ast \wedge \bar {e}_3^\ast +\bar{f}_2^\ast \wedge \bar{e}_2^\ast \wedge \bar{e}_3^\ast +\bar{e}_1^\ast \wedge \bar{e}_2^\ast \wedge \bar{f}_3^\ast +q\bar{f}_3^\ast \wedge \bar{e}_1^\ast \wedge \bar{f}_1^\ast \\ & \multicolumn{1}{r}{+q\bar{f}_3^\ast \wedge \bar{e}_2^\ast \wedge \bar{f}_2^\ast ,} \end{array} \] \[ \begin{array} [c]{rl} \chi _{P_{15}}= & \bar{e}_1^\ast \wedge \bar{f}_1^\ast \wedge \bar{e}_3^\ast +\bar{f}_2^\ast \wedge \bar{e}_2^\ast \wedge \bar{e}_3^\ast ,\\ \chi _{P_{16}}(q)= & \bar{e}_1^\ast \wedge \bar{f}_1^\ast \wedge \bar{e}_3^\ast +\bar{f}_2^\ast \wedge \bar{e}_2^\ast \wedge \bar{e}_3^\ast +q\bar{f}_3^\ast \wedge \bar{e}_1^\ast \wedge \bar{f}_1^\ast +q\bar{f}_3^\ast \wedge \bar{e}_2^\ast \wedge \bar{f}_2^\ast ,\\ \chi _{P_{17}}(q)= & \bar{e}_1^\ast \wedge \bar{f}_1^\ast \wedge \bar{e}_3^\ast +\bar{f}_2^\ast \wedge \bar{e}_2^\ast \wedge \bar{e}_3^\ast +q\bar{e}_3^\ast \wedge \bar{e}_1^\ast \wedge \bar{f}_1^\ast +q\bar{e}_3^\ast \wedge \bar{e}_2^\ast \wedge \bar{f}_2^\ast ,\\ \chi _{P_{18}}= & \bar{e}_1^\ast \wedge \bar{f}_1^\ast \wedge \bar{e}_3^\ast +\bar{f}_2^\ast \wedge \bar{e}_2^\ast \wedge \bar{e}_3^\ast +\bar{e}_1^\ast \wedge \bar{e}_2^\ast \wedge \bar{f}_2^\ast +\bar{e}_1^\ast \wedge \bar{e}_2^\ast \wedge \bar{f}_3^\ast ,\\ \chi _{P_{19}}= & \bar{e}_1^\ast \wedge \bar{f}_1^\ast \wedge \bar{e}_3^\ast +\bar{f}_2^\ast \wedge \bar{e}_2^\ast \wedge \bar{e}_3^{\ast }+\bar{e}_1^\ast \wedge \bar{e}_2^\ast \wedge \bar{f}_2^\ast +\bar{e}_1^\ast \wedge \bar{e}_2^\ast \wedge \bar{f}_3^\ast \\ & +\bar{e}_2^\ast \wedge \bar{e}_1^\ast \wedge \bar{f}_1^\ast +\bar{e}_2^\ast \wedge \bar{e}_3^\ast \wedge \bar{f}_3^\ast . \end{array} \] Then, we obtain \[ \begin{array} [c]{lll} I_{j}(\chi _{P_h})=0, & 1\leq j\leq2, & h\in \{ 2,7,8,9,10,11,12,13,15,17,18,19\} , \end{array} \] \[ \begin{array} [c]{lll} I_1(\chi _{P_h})=0, & I_2(\chi _{P_h})=-2^3\cdot 3^2q^2, & h\in\{ 3,4,5\} ,\\ I_1(\chi _{P_6})=-2^2pq, & I_2(\chi _{P_6}) =-2^3\cdot 3q(3q+2^3p), & \\ I_1(\chi _{P_h})=2^2q^2, & I_2(\chi _{P_h})=2^6\cdot 3 q^2, & h\in \{ 14,16\} . \end{array} \] \subsubsection*{Acknowledgment} The authors wish to thank the unknown referee for his careful and thorough read of our manuscript and his valuable comments. \section{Appendix} \begin{align*} I_1 & =y_{135}y_{234}y_{256}^2-y_{126}y_{234}y_{356}^2-y_{134} y_{136}y_{236}y_{456}-y_{126}y_{134}y_{145}y_{346}\\ & +2y_{126}y_{135}y_{245}y_{346}+y_{124}y_{135}y_{245}y_{256}-y_{126} y_{145}y_{235}y_{245}\\ & -3y_{126}y_{145}y_{235}y_{346}+y_{124}y_{135}y_{146}y_{346}+3y_{124} y_{135}y_{256}y_{346}\\ & -y_{124}y_{135}y_{146}y_{245}+y_{145}y_{156}y_{234}y_{236}+2y_{135} y_{146}y_{234}y_{256}\\ & +y_{124}y_{156}y_{234}y_{356}+y_{156}y_{235}y_{236}y_{346}-y_{156} y_{235}y_{236}y_{245}\\ & -y_{146}y_{156}y_{234}y_{235}+2y_{126}y_{145}y_{234}y_{356}-y_{156} y_{234}y_{236}y_{356}\\ & +y_{136}y_{156}y_{234}y_{346}-y_{156}y_{234}y_{235}y_{256}+y_{136} y_{156}y_{234}y_{245}\\ & -y_{124}y_{156}y_{235}y_{245}+y_{146}^2y_{235}^2+y_{145}^2y_{236} ^2+y_{124}^2y_{356}^2\\ & +2y_{124}y_{156}y_{236}y_{345}-3y_{124}y_{156}y_{235}y_{346}-y_{125} y_{126}y_{345}y_{346}\\ & +y_{123}y_{136}y_{346}y_{456}-y_{123}y_{145}y_{236}y_{456}-y_{125} y_{126}y_{245}y_{345}\\ & +2y_{125}y_{146}y_{235}y_{346}-2y_{136}y_{146}y_{235}y_{245}-2y_{136} y_{145}y_{236}y_{245}\\ & -3y_{123}y_{146}y_{245}y_{356}-2y_{145}y_{146}y_{235}y_{236}-3y_{125} y_{134}y_{236}y_{456}\\ & +2y_{125}y_{136}y_{234}y_{456}+2y_{123}y_{156}y_{245}y_{346}+2y_{123} y_{145}y_{246}y_{356}\\ & -3y_{125}y_{146}y_{236}y_{345}-y_{123}y_{145}y_{146}y_{346}+y_{123} y_{145}y_{146}y_{245}\\ & -y_{123}y_{145}y_{245}y_{256}-y_{123}y_{146}y_{346}y_{356}-y_{125} y_{234}y_{256}y_{356}\\ & +y_{134}y_{136}y_{145}y_{246}+y_{134}y_{136}y_{246}y_{356}-y_{125} y_{145}y_{234}y_{256}\\ & -3y_{125}y_{146}y_{234}y_{356}-3y_{123}y_{145}y_{256}y_{346}+y_{125} y_{145}y_{146}y_{234}\\ & +y_{123}y_{256}y_{346}y_{356}-y_{123}y_{245}y_{256}y_{356}+y_{123} y_{136}y_{245}y_{456}\\ & +y_{123}y_{134}y_{256}y_{456}+y_{135}y_{236}y_{245}y_{256}+2y_{134} y_{136}y_{245}y_{256}\\ & +y_{136}^2y_{245}^2+y_{125}^2y_{346}^2-y_{125}y_{236}y_{256} y_{345}-y_{135}y_{236}y_{256}y_{346}\\ & +3y_{135}y_{146}y_{236}y_{245}+y_{123}y_{235}y_{256}y_{456}-y_{125} y_{235}y_{236}y_{456}\\ & +y_{135}y_{146}y_{236}y_{346}+y_{123}y_{236}y_{356}y_{456}-3y_{124} y_{136}y_{235}y_{456}\\ & +y_{124}y_{125}y_{134}y_{456}+2y_{124}y_{136}y_{245}y_{356}-2y_{124} y_{145}y_{236}y_{356}\\ & -2y_{124}y_{134}y_{256}y_{356}+y_{125}y_{235}y_{246}y_{356}-y_{125} y_{134}y_{145}y_{246}\\ & +3y_{125}y_{134}y_{246}y_{356}+2y_{124}y_{135}y_{236}y_{456}-3y_{124} y_{136}y_{256}y_{345}\\ & +y_{125}y_{145}y_{235}y_{246}+y_{123}y_{146}y_{235}y_{456}+2y_{123} y_{146}y_{256}y_{345}\\ & +y_{123}y_{125}y_{346}y_{456}-y_{124}y_{136}y_{146}y_{345}+y_{123} y_{134}y_{146}y_{456}\\ & +y_{136}y_{236}y_{256}y_{345}+y_{136}y_{235}y_{236}y_{456}-y_{124} y_{126}y_{145}y_{345}\\ & +y_{125}y_{156}y_{234}y_{245}-y_{125}y_{135}y_{246}y_{346}+y_{126} y_{235}y_{346}y_{356}\\ & -y_{125}y_{135}y_{245}y_{246}-2y_{125}y_{136}y_{245}y_{346}-2y_{134} y_{146}y_{235}y_{256} \end{align*} \begin{align*} & +y_{134}^2y_{256}^2+2y_{134}y_{145}y_{236}y_{256}-y_{135}y_{235} y_{246}y_{256}+y_{136}y_{234}y_{256}y_{356}\\ & -y_{136}y_{145}y_{146}y_{234}-y_{126}y_{235}y_{245}y_{356}-y_{136} y_{146}y_{236}y_{345}\\ & -y_{136}y_{146}y_{234}y_{356}-3y_{136}y_{145}y_{234}y_{256}+y_{125} y_{156}y_{234}y_{346}\\ & +y_{126}y_{145}y_{236}y_{345}-y_{126}y_{136}y_{245}y_{345}+y_{126} y_{134}y_{146}y_{345} \end{align*} \begin{align*} & +2y_{125}y_{136}y_{246}y_{345}+y_{126}y_{134}y_{256}y_{345}+y_{126} y_{235}y_{256}y_{345}\\ & -y_{134}y_{135}y_{246}y_{256}-y_{126}y_{236}y_{345}y_{356}-y_{124} y_{135}y_{246}y_{356}\\ & -y_{124}y_{145}y_{156}y_{234}+y_{126}y_{146}y_{235}y_{345}-y_{126} y_{136}y_{345}y_{346}\\ & +y_{124}y_{135}y_{145}y_{246}-y_{124}y_{134}y_{156}y_{346}-y_{134} y_{156}y_{234}y_{256}\\ & +2y_{134}y_{156}y_{235}y_{246}+2y_{125}y_{145}y_{236}y_{346}-2y_{125} y_{134}y_{256}y_{346}\\ & +2y_{124}y_{146}y_{235}y_{356}-3y_{134}y_{156}y_{236}y_{245}-y_{134} y_{135}y_{146}y_{246}\\ & -y_{135}y_{136}y_{245}y_{246}-y_{134}y_{146}y_{156}y_{234}-y_{135} y_{145}y_{236}y_{246}\\ & -y_{135}y_{136}y_{246}y_{346}-y_{134}y_{156}y_{236}y_{346}+2y_{126} y_{134}y_{235}y_{456}\\ & -2y_{124}y_{125}y_{346}y_{356}+3y_{136}y_{145}y_{235}y_{246}+y_{126} y_{134}y_{145}y_{245}\\ & -3y_{126}y_{134}y_{245}y_{356}-y_{126}y_{134}y_{346}y_{356}-y_{124} y_{134}y_{136}y_{456}\\ & -y_{124}y_{125}y_{235}y_{456}+y_{124}y_{125}y_{146}y_{345}-y_{124} y_{125}y_{256}y_{345}\\ & -y_{136}y_{235}y_{246}y_{356}+y_{123}y_{124}y_{145}y_{456}-y_{123} y_{124}y_{356}y_{456}\\ & +y_{125}^2y_{246}y_{345}+y_{123}y_{256}^2y_{345}+y_{123}y_{156} y_{245}^2+y_{126}y_{135}y_{245}^2\\ & -y_{135}y_{236}^2y_{456}+y_{135}y_{146}^2y_{234}+y_{125}^2 y_{234}y_{456}-y_{123}y_{145}^2y_{246}\\ & -y_{156}y_{236}^2y_{345}+y_{156}y_{235}^2y_{246}+y_{134}^2 y_{156}y_{246}+y_{123}y_{156}y_{346}^2\\ & +y_{126}y_{135}y_{346}^2-y_{124}^2y_{156}y_{345}+y_{123}y_{146} ^2y_{345}+y_{126}y_{235}^2y_{456}\\ & +y_{126}y_{134}^2y_{456}-y_{124}^2y_{135}y_{456}+y_{136}^2 y_{246}y_{345}-y_{123}y_{246}y_{356}^2\\ & -y_{126}y_{145}^2y_{234}+y_{136}^2y_{234}y_{456}-y_{135}y_{146} y_{235}y_{246}\\ & +y_{123}y_{125}y_{245}y_{456}+y_{124}y_{134}y_{156}y_{245}+y_{135} y_{236}y_{246}y_{356}\\ & +y_{124}y_{126}y_{345}y_{356}. \end{align*} \begin{align*} I_2 & =24\left( -5y_{124}y_{136}y_{146}y_{345}-5y_{126}y_{145} y_{235}y_{245}-5y_{124}y_{156}y_{235}y_{245}\right. \\ & -4y_{136}y_{146}y_{235}y_{245}-5y_{136}y_{146}y_{236}y_{345}-5y_{136} y_{146}y_{234}y_{356}\\ & -5y_{124}y_{125}y_{256}y_{345}-5y_{125}y_{126}y_{245}y_{345}-2y_{125} y_{136}y_{246}y_{345}\\ & +y_{125}y_{126}y_{345}y_{346}-5y_{124}y_{135}y_{146}y_{245}+5y_{124} y_{135}y_{245}y_{256}\\ & -5y_{125}y_{135}y_{245}y_{246}+y_{135}y_{136}y_{245}y_{246}-2y_{126} y_{135}y_{245}y_{346}\\ & +4y_{124}y_{146}y_{235}y_{356}-5y_{126}y_{235}y_{245}y_{356}-5y_{136} y_{235}y_{246}y_{356}\\ & +6y_{123}y_{156}y_{234}y_{456}+6y_{135}y_{156}y_{234}y_{246}+6y_{126} y_{156}y_{234}y_{345}\\ & +5y_{126}y_{134}y_{146}y_{345}+6y_{123}y_{126}y_{345}y_{456}+6y_{126} y_{135}y_{246}y_{345}\\ & -y_{123}y_{136}y_{245}y_{456}+4y_{124}y_{136}y_{245}y_{356}+y_{126} y_{136}y_{245}y_{345}\\ & -4y_{134}y_{146}y_{235}y_{256}+12y_{123}y_{156}y_{246}y_{345} +12y_{126}y_{135}y_{234}y_{456}\\ & +5y_{123}y_{146}y_{145}y_{245}-5y_{123}y_{145}y_{245}y_{256}-2y_{126} y_{134}y_{235}y_{456}\\ & +5y_{136}y_{235y_{236}}y_{456}-3y_{124}y_{136}y_{235}y_{456}-5y_{125} y_{235}y_{236}y_{456}\\ & -2y_{123}y_{146}y_{256}y_{345}-3y_{123}y_{146}y_{245}y_{356}-5y_{123} y_{245}y_{256}y_{356}\\ & -5y_{134}y_{136}y_{236}y_{456}+5y_{123}y_{256}y_{346}y_{356}-5y_{124} y_{134}y_{136}y_{456}\\ & -2y_{124}y_{135}y_{236}y_{456}-2y_{123}y_{145}y_{246}y_{356}+5y_{124} y_{125}y_{134}y_{456}\\ & -3y_{123}y_{145}y_{256}y_{346}-3y_{125}y_{134}y_{236}y_{456}-2y_{123} y_{156}y_{245}y_{346}\\ & -2y_{125}y_{136}y_{234}y_{456}-5y_{124}y_{125}y_{456}y_{235}+5y_{123} y_{125}y_{456}y_{245}\\ & -4y_{124}y_{134}y_{256}y_{356}-5y_{123}y_{145}y_{146}y_{346}-5y_{123} y_{146}y_{346}y_{356}\\ & +5y_{123}y_{136}y_{456}y_{346}-5y_{126}(y_{145})^2y_{234}-5y_{156} (y_{236})^2y_{345}\\ & -5(y_{124})^2y_{156}y_{345}-5y_{126}y_{234}(y_{356})^2+5y_{135} (y_{146})^2y_{234}\\ & +\!5y_{156}(y_{235})^2y_{246}+5(y_{134})^2y_{156}y_{246}+2(y_{124} )^2(y_{356})^2\\ & -4(y_{156})^2(y_{234})^2-3(y_{126})^2(y_{345})^2-3(y_{123} )^2(y_{456})^2\\ & -3(y_{135})^2(y_{246})^2+2(y_{136})^2(y_{245})^2+5y_{135} y_{234}(y_{256})^2\\ & +5y_{125}^2y_{345}y_{246}+5y_{135}y_{245}^2y_{126}+5y_{126}y_{346} ^2y_{135}\\ & +5(y_{136})^2y_{246}y_{345}+5y_{123}y_{156}(y_{245})^2+5(y_{136} )^2y_{234}y_{456}\\ & +5y_{126}(y_{235})^2y_{456}+5y_{123}(y_{256})^2y_{345}-5y_{123} y_{246}(y_{356})^2\\ & -5y_{135}(y_{236})^2y_{456}-5y_{123}(y_{145})^2y_{246}-5(y_{124} )^2y_{135}y_{456}\\ & +5(y_{125})^2y_{234}y_{456}+5y_{126}(y_{134})^2y_{456}+5y_{123} (y_{146})^2y_{345}\\ & +5y_{123}y_{156}(y_{346})^2+5y_{123}y_{235}y_{256}y_{456}+5y_{126} y_{256}y_{235}y_{345}\\ & +5y_{123}y_{134}y_{146}y_{456}+y_{123}y_{145}y_{236}y_{456}+y_{123} y_{124}y_{356}y_{456} \end{align*} \begin{align*} & +6y_{123}y_{135}y_{246}y_{456}+y_{135}y_{145}y_{236}y_{246}+y_{124} y_{135}y_{246}y_{356}\\ & +4y_{125}y_{145}y_{236}y_{346}-y_{123}y_{125}y_{346}y_{456}+y_{125} y_{135}y_{246}y_{346}\\ & -5y_{125}y_{134}y_{145}y_{246}+5y_{125}y_{145}y_{235}y_{246}-5y_{125} y_{145}y_{234}y_{256}\\ & -5y_{134}y_{156}y_{236}y_{346}+5y_{135}y_{146}y_{236}y_{346}-4y_{125} y_{134}y_{256}y_{346}\\ & +5y_{124}y_{135}y_{145}y_{246}-4y_{124}y_{145}y_{236}y_{356}+5y_{123} y_{236}y_{356}y_{456}\\ & +5y_{135}y_{236}y_{246}y_{356}+5y_{123}y_{124}y_{145}y_{456}-y_{123} y_{146}y_{235}y_{456}\\ & -y_{123}y_{134}y_{256}y_{456}-y_{126}y_{146}y_{235}y_{345}-y_{126} y_{134}y_{256}y_{345}\\ & -5y_{126}y_{134}y_{145}y_{346}-5y_{126}y_{136}y_{345}y_{346}-5y_{126} y_{134}y_{346}y_{356}\\ & -5y_{135}y_{136}y_{246}y_{346}+5y_{134}y_{136}y_{246}y_{356}+4y_{134} y_{145}y_{236}y_{256}\\ & -5y_{125}y_{236}y_{256}y_{345}-5y_{135}y_{236}y_{256}y_{346}+5y_{125} y_{156}y_{234}y_{245}\\ & -y_{136}y_{156}y_{234}y_{245}-y_{125}y_{156}y_{234}y_{346}+5y_{136} y_{156}y_{234}y_{346}\\ & +5y_{125}y_{145}y_{146}y_{234}-y_{145}y_{156}y_{234}y_{236}-5y_{124} y_{145}y_{156}y_{234}\\ & -3y_{136}y_{145}y_{234}y_{256}-2y_{126}y_{145}y_{234}y_{356}-3y_{125} y_{146}y_{236}y_{345}\\ & -y_{126}y_{145}y_{236}y_{345}-2y_{124}y_{156}y_{236}y_{345}+5y_{136} y_{236}y_{345}y_{256}\\ & -5y_{126}y_{236}y_{345}y_{356}+4y_{125}y_{146}y_{235}y_{346}-3y_{126} y_{145}y_{235}y_{346}\\ & +5y_{156}y_{236}y_{235}y_{346}-3y_{124}y_{156}y_{235}y_{346}+5y_{126} y_{235}y_{346}y_{356}\\ & +5y_{245}y_{134}y_{126}y_{145}-3y_{245}y_{134}y_{236}y_{156}+5y_{245} y_{134}y_{156}y_{124}\\ & +4y_{245}y_{134}y_{136}y_{256}-3y_{245}y_{134}y_{126}y_{356}+5y_{345} y_{124}y_{146}y_{125}\\ & -5y_{345}y_{124}y_{126}y_{145}-3y_{345}y_{124}y_{136}y_{256}-y_{345} y_{124}y_{126}y_{356}\\ & -3y_{356}y_{234}y_{146}y_{125}-5y_{356}y_{234}y_{236}y_{156}-y_{356} y_{234}y_{156}y_{124}\\ & +5y_{356}y_{234}y_{136}y_{256}-5y_{146}y_{234}y_{136}y_{145}+y_{146} y_{234}y_{156}y_{235}\\ & -5y_{146}y_{234}y_{156}y_{134}-2y_{146}y_{234}y_{256}y_{135}+y_{235} y_{246}y_{135}y_{146}\\ & +3y_{235}y_{246}y_{136}y_{145}-2y_{235}y_{246}y_{156}y_{134}-5y_{235} y_{246}y_{256}y_{135}\\ & +5y_{125}y_{235}y_{246}y_{356}+3y_{135}y_{146}y_{236}y_{245}-4y_{136} y_{145}y_{236}y_{245}\\ & -5y_{156}y_{235}y_{236}y_{245}+5y_{135}y_{236}y_{245}y_{256}-5y_{134} y_{135}y_{146}y_{246}\\ & +5y_{134}y_{136}y_{145}y_{246}+y_{134}y_{135}y_{246}y_{256}+3y_{125} y_{134}y_{246}y_{356}\\ & +5y_{124}y_{135}y_{146}y_{346}-5y_{124}y_{134}y_{156}y_{346}+3y_{124} y_{135}y_{346}y_{256}\\ & -4y_{346}y_{124}y_{356}y_{125}-5y_{256}y_{234}y_{156}y_{235}+y_{134} y_{156}y_{234}y_{256}\\ & -5y_{125}y_{234}y_{256}y_{356}-4y_{145}y_{146}y_{236}y_{235}-4y_{125} y_{136}y_{245}y_{346}\\ & \left. +2(y_{146})^2(y_{235})^2+2(y_{145})^2(y_{236})^2 +2(y_{134})^2(y_{256}^2)+2(y_{125})^2(y_{346})^2\right) . \end{align*} \begin{thebibliography}{9} \bibitem {BK1} B. De Bruyn, M. Kwiatkowski, \emph{On the trivectors of a }$6$\emph{-dimensional symplectic vector space}, Linear Alg.\ Appl.\ \textbf{435} (2011), 289--306. \bibitem {BK2} B. De Bruyn, M. Kwiatkowski, \emph{On the trivectors of a }$6$\emph{-dimensional symplectic vector space. II}, Linear Alg.\ Appl.\ \textbf{437} (2012), 1215--1233. \bibitem {BK3} B. De Bruyn, M. Kwiatkowski, \emph{On the trivectors of a }$6$\emph{-dimensional symplectic vector space. III}, Linear Alg.\ Appl.\ \textbf{438} (2013), 374--398. \bibitem {BK4} B. De Bruyn, M. Kwiatkowski, \emph{On the trivectors of a }$6$\emph{-dimensional symplectic vector space. IV}, Linear Alg.\ Appl.\ \textbf{438} (2013), 2405--2429. \bibitem {BK5} B. De Bruyn, M. Kwiatkowski, \emph{The classification of the trivectors of a }$6$\emph{-dimensional symplectic space: Summary, consequences and connections}, Linear Alg.\ Appl.\ \textbf{438} (2013), 3516--3529. \bibitem {Fogarty} J. Fogarty, \emph{Invariant theory}, W. A. Benjamin, Inc., New York, Amsterdam, 1969. \bibitem {KPV} V. G. Kac, V. L. Popov, E. B. Vinberg, \emph{Sur les groupes lin\'eaires alg\'ebriques dont l'alg\`{e}bre des invariants est libre}, C. R. Acad. Sci. Paris S\'er. A-B \textbf{283} (1976), no. 12, Ai, A875--A878. \bibitem {P} V. L. Popov, \emph{Classification of spinors of dimension fourteen}, Trans.\ Mosc.\ Math. Soc.\ \textbf{1} (1980), 181--232. \end{thebibliography} \noindent\textbf{Authors' addresses} \smallskip \noindent(J.M.M.) \textsc{Instituto de Tecnolog\'{\i}as F\'{\i}sicas y de la Informaci\'on, CSIC, C/ Serrano 144, 28006-Madrid, Spain.} \noindent\emph{E-mail:\/} \texttt{[email protected]} \medskip \noindent(L.M.P.C.) \textsc{Departamento de Matem\'atica Aplicada a las T.I.C., E.T.S.I. Sistemas Inform\'aticos, Universidad Polit\'ecnica de Madrid, Carrete\-ra de Valencia, Km.\ 7, 28031-Madrid, Spain.} \noindent\emph{E-mail:\/} \texttt{[email protected]} \end{document}
2412.03164v1
http://arxiv.org/abs/2412.03164v1
Lebesgue constants for the Walsh system and the discrepancy of the van der Corput sequence
\documentclass[12pt]{article} \setlength{\bigskipamount}{5ex plus1.5ex minus 2ex} \setlength{\textheight}{24cm} \setlength{\textwidth}{16cm} \setlength{\hoffset}{-1.3cm} \setlength{\voffset}{-1.8cm} \newtheorem{theorem}{Theorem} \newtheorem{proposition}{Proposition} \newtheorem{lemma}{Lemma} \newtheorem{definition}{Definition} \newtheorem{coro}{Corollary} \newtheorem{remark}{Remark} \newtheorem{example}{Example} \newtheorem{problem}{Problem} \newtheorem{construction}{Construction} \newenvironment{proof}{\begin{trivlist} \item[\hskip\labelsep{\it Proof.}]}{$\hfill\Box$\end{trivlist}} \newcommand{\RR}{{\Bbb R}} \newcommand{\NN}{{\Bbb N}} \newcommand{\ZZ}{{\Bbb Z}} \newcommand{\wal}{{\rm wal}} \newcommand{\qed} {\hfill \Box \vspace{0.5cm}} \newcommand{\lt}{L_{2,N}} \newcommand{\disc}{D_N^{\ast}} \newcommand{\rd}{\,{\rm d}} \usepackage{latexsym,amsfonts,amsmath,amssymb} \title{Lebesgue constants for the Walsh system and the discrepancy of the van der Corput sequence} \author{Josef Dick and Friedrich Pillichshammer} \date{} \begin{document} \maketitle \begin{abstract} In this short note we report on a coincidence of two mathematical quantities that, at first glance, have little to do with each other. On the one hand, there are the Lebesgue constants of the Walsh function system that play an important role in approximation theory, and on the other hand there is the star discrepancy of the van der Corput sequence that plays a prominent role in uniform distribution theory. Over the decades, these two quantities have been examined in great detail independently of each other and important results have been proven. Work in these areas has been carried out independently, but as we show here, they actually coincide. Interestingly, many theorems have been discovered in both areas independently, but some results have only been known in one area but not in the other. \end{abstract} \paragraph{Lebesgue Constants.} For a given system of orthonormal functions $\Phi=\{\phi_k \ : \ k =0,1,\ldots\}$ in $L_2([a,b])$ the $n$-th Lebesgue function is defined as \begin{equation*} L_n^{\Phi}(u):=\int_a^b \left|\sum_{k=0}^{n-1} \phi_k(x) \phi_k(u) \right| \rd x\qquad \mbox{for $u \in [a,b]$.} \end{equation*} It follows easily from the fact $S_n(f)(u)=\int_a^b \left(\sum_{k=0}^{n-1} \phi_k(x) \phi_k(u)\right) f(u)\rd u$ that \begin{equation*} L_n^{\Phi}(u)=\sup_{\|f\|_{C([a,b])} \le 1} |S_n(f)(u)|\qquad \mbox{for $u \in [a,b]$,} \end{equation*} where $\|\cdot \|_{C([a,b])}$ is the uniform norm and $S_n(f)(u)=\sum_{k=0}^{n-1} \widehat{f}(k) \phi_k(u)$ is the $n$-th partial sum of the Fourier expansion of $f$ with respect to the system $\Phi$. If the $n$-th Lebesgue function $L_n^{\Phi}$ is constant over $[a,b]$, then its function value is called the $n$-th Lebesgue constant, which is denoted by $L_n^{\Phi}$ or simply by $L_n$. Lebesgue functions/constants are a fundamental tool in approximation theory; see, e.g., \cite{AT,KS,SWS,zyg}. In this note we consider as an orthonormal system the Walsh functions and report a connection of the corresponding Lebesgue functions to the star discrepancy of the van der Corput sequence, which is an important quantity in uniform distribution and discrepancy theory. Both Lebesgue functions for the Walsh system and star discrepancy of van der Corput sequence are well-studied objects in literature, but so far it has not been observed that both quantities coincide. This is, at least for us, a very interesting connection between concepts from different branches of mathematics that allows to transfer results for one measure to the other and vice versa. \paragraph{Walsh functions.} For a nonnegative integer $k$ with base $2$ representation $k=k_0+k_1 2+\cdots+k_m 2^m$ the $k$-th {\it Walsh function} $\wal_k:\RR \rightarrow \RR$, periodic with period one, is defined by \begin{eqnarray} \wal_k(x):=(-1)^{x_1 k_0 +x_2 k_1+\cdots + x_{m+1} k_m},\nonumber \end{eqnarray} when $x \in [0,1)$ has (canonical) base $2$ representation $x=\frac{x_1}{2}+\frac{x_2}{2^2}+\cdots$. The system $\mathcal{W}=\{\wal_k \ : \ k=0,1,\ldots \}$ is a complete orthonormal system in $L_2([0,1])$. For information about Walsh functions see \cite{fine,SWS} or \cite[Appendix~A]{DP10}. In \cite[Section~5]{fine} Fine introduced the Walsh Dirichlet kernel $$D_n(x,u):= \sum_{k=0}^{n-1} \wal_k(x) \wal_k(u)$$ and the so-called Lebesgue functions $$L_n(x):=\int_0^1|D_n(x,u)| \rd u$$ (for simplicity we write $L_n(x)$ rather than $L_n^{\mathcal{W}}(x)$) for the Walsh systems and proved several interesting results. In the first place he proved that the $L_n(x)$ are in fact independent of $x$ (see \cite[p.386]{fine}) and for this reason they coincide with their Lebesgue constants $L_n$. In \cite[Theorem~IX]{fine} several properties of the Lebesgue constants $L_n$ are summarized. \begin{theorem}[Fine]\label{thm:fine} The Lebesgue constants $L_n$ of the Walsh system satisfy \begin{equation}\label{fo:LCW} L_n=\nu-\sum_{1 \le j < i \le \nu} 2^{n_i-n_{j}}, \end{equation} where $n$ is of the form $n=2^{n_1}+2^{n_2}+\cdots+2^{n_{\nu}}$ with integer exponents $n_1 > n_2 > \cdots > n_{\nu}\ge 0$. Furthermore, the following properties hold: \begin{enumerate} \item $L_{2n}=L_n$ and $L_{2n+1}=(1+L_n+L_{n+1})/2$. \item $L_n=O(\log n)$. \item $\frac{1}{n} \sum_{k=1}^n L_k =\frac{\log n}{4 \log 2}+O(1)$. \item $\limsup_{n \rightarrow \infty} \left[L_n-\left(\frac{4}{9}+\frac{\log 3}{3 \log 2}+\frac{\log n}{3 \log 2}\right)\right]=0$. \item the generating function for $L_n$ is $\sum_{n=1}^{\infty} L_n z^n =\frac{1}{2} \frac{z}{(1-z)^2} \sum_{k=0}^{\infty}\frac{1}{2^k} \frac{1-z^{2^k}}{1+z^{2^k}}$ for $|z|<1$. \end{enumerate} \end{theorem} Further results on the Lebesgue constants for the Walsh system are shown in \cite{AS1,AS2}. \paragraph{Star discrepancy.} For a sequence $X=(x_k)_{k \ge 0}$ in $[0,1)$ the $n$-th discrepancy function is defined as $$\Delta_{X,n}(t) :=\frac{\#\{k \in \NN_0\ : \ k < n,\ x_k \in [0,t)\}}{n}-t \qquad \mbox{for $t \in [0,1]$}.$$ The star discrepancy of the initial $n$ terms of $X$ is then defined as $$D_n^*(X):=\sup_{t \in [0,1]}|\Delta_{X,n}(t)|.$$ This is a quantitative measure for the irregularity of distribution of the sequence $X$ in $[0,1)$. The sequence $X$ is uniformly distributed in the sense of Weyl \cite{weyl} iff $\lim_{n \rightarrow \infty}D_n^{\ast}(X)=0$. The smaller $D_n^{\ast}(X)$, the better the $n$ initial terms of $X$ are distributed in $[0,1]$. For an introduction to uniform distribution and discrepancy, see, e.g., \cite{kuinie}. \paragraph{Star discrepancy of the van der Corput sequence.} The prototype of a uniformly distributed sequence is the van der Corput sequence (see \cite{vdc35} or \cite{FKP}) whose construction is based on the reflection of the binary digits of nonnegative integers $k$ around the binary point. If $k$ has binary expansion $k=k_0+k_1 2+k_2 2^2+\cdots$ (which is of course finite) with $k_j \in \{0,1\}$ for $j=0,1,2,\ldots$, then the $k$-th element $y_k$ of the van der Corput sequence $Y^{{\rm vdC}}=(y_k)_{k \ge 0}$ is given by \begin{equation*} y_k := \frac{k_0}{2}+\frac{k_1}{2^2}+\frac{k_2}{2^3}+\cdots. \end{equation*} The star discrepancy of the van der Corput sequence is intensively studied in literature, see, e.g., \cite{befa,befa77,dlp04,fau1990,hab1966,spi}. See also the survey \cite{FKP} for many other references. Consider the star discrepancy $D_n^{\ast}(Y^{{\rm vdC}})$ of the first $n$ terms of the van der Corput sequence and set $d_n:=n D_n^{\ast}(Y^{{\rm vdC}})$ to be the non-normalized star discrepancy. \begin{theorem}\label{thm2} The $n$-th Lebesgue constant $L_n$ for the Walsh system is exactly the nonnormalized star discrepancy $d_n$ of the first $n$ terms of the van der Corput sequence, that is $$d_n=L_n.$$ \end{theorem} \begin{proof} Let $n \in \NN$ be of the form $n=2^{n_1}+2^{n_2}+\cdots+2^{n_{\nu}}$ with integer exponents $n_1 > n_2 > \cdots > n_{\nu}\ge 0$. For the initial $n$ terms of $Y^{\rm vdC}$ we have \begin{align*} \{y_0,y_1,\ldots,y_{n-1}\} = & \bigcup_{i=1}^{\nu}\left\{y_{\ell}\ : \ \ell \in \{2^{n_1}+\cdots+2^{n_{i-1}},\ldots, 2^{n_1}+\cdots+2^{n_{i-1}}+2^{n_i}-1\}\right\}\\ = & \bigcup_{i=1}^{\nu} \left\{\frac{k}{2^{n_i}}+\frac{1}{2^{n_{i-1}+1}}+\frac{1}{2^{n_{i-2}+1}}+\cdots +\frac{1}{2^{n_1+1}} \ : \ k \in \{0,1,\ldots,2^{n_i}-1\}\right\}, \end{align*} where we put $2^{n_1}+\cdots+2^{n_{i-1}}:=0$ if $i=1$. It is well known that the star discrepancy of the van der Corput sequence is nonnegative (see \cite[Remark~3]{p04}) and twice the $L_1([0,1])$-norm of the discrepancy function (see, for example, \cite{PA}). Hence \begin{align}\label{stl1} d_n = & 2 n \int_0^1 \Delta_{Y^{\rm vdc},n}(t) \rd t = 2 \int_0^1 \sum_{\ell=0}^{n-1} (\boldsymbol{1}_{[0,t)}(y_{\ell}) -t) \rd t\\ = & 2 \sum_{\ell=0}^{n-1} \left(\frac{1}{2}-y_{\ell}\right) = 2 \sum_{i=1}^{\nu} \sum_{k=0}^{2^{n_i}-1} \left(\frac{1}{2}-\frac{k}{2^{n_i}}- \sum_{j=1}^{i-1} \frac{1}{2^{n_j+1}}\right)\nonumber\\ = & 2 \left(\frac{\nu}{2}-\sum_{i=1}^{\nu}2^{n_i} \sum_{j=1}^{i-1} \frac{1}{2^{n_j+1}} \right) = \nu-\sum_{1\le j < i \le \nu}2^{n_i-n_j} =L_n,\nonumber \end{align} where the last equality is \eqref{fo:LCW} from Theorem~\ref{thm:fine}. \end{proof} The representation $d_n=\nu-\sum_{1\le j < i \le \nu}2^{n_i-n_j}$ for the star discrepancy of $Y^{\rm vdC}$ is also mentioned (in an equivalent form) in \cite[p.~61]{spi}. \paragraph{Applications.} Theorem~\ref{thm2} can now be used to transfer results for the star discrepancy of the van der Corput sequence to results for the Lebesgue constants for the Walsh system and vice versa. In particular Items~1-4 from Theorem~\ref{thm:fine} by Fine have also been proven in the context of discrepancy of the van der Corput sequence by B\'{e}jian and Faure \cite{befa,befa77}. Therein, the estimate $d_n \le \frac{\log n}{3 \log 2} +1$ from \cite[Th\'eor\`eme~3]{befa77} yields a refinement of item~{\it 2} in Theorem~\ref{thm:fine}. Furthermore, it is known (see \cite[Lemme~4.1]{fau1990}) that for every $r \in \mathbb{N}$ we have $$\max_{n \in [2^{r-1},2^r]} d_n = \frac{r}{3}+\frac{7}{9}+\frac{(-1)^r}{9 \cdot 2^{r-1}}$$ and the maximum is attained for $n=(2^{r+1}+(-1)^r)/3$. Thus, for all $n$ of this form we have $d_n \ge \frac{\log n}{3 \log 2} +O(1)$. This result has been proven (in a slightly weaker form) for the Lebesgue constants for the Walsh system in \cite[Theorem~2]{AS2}. From Theorem~\ref{thm2} we also obtain new representations of the Lebesgue constant or star discrepancy of the van der Corput sequence that have not been known so far. For instance, the definition of $L_n$ leads to a new representation of $D_n^{\ast}(Y^{{\rm vdC}})$ via \begin{equation*} D_n^{\ast}(Y^{{\rm vdC}})=\frac{d_n}{n}=\frac{L_n}{n}=\int_0^1 \left|\frac{1}{n}\sum_{k=0}^{n-1} \wal_k(x)\right| \rd x. \end{equation*} Using this formula, we can derive another new formula for the star discrepancy of the van der Corput sequence. We have \begin{align*} D_n^{\ast}(Y^{{\rm vdC}}) = & \sum_{m=0}^{2^{n_1+1}-1} \int_{m 2^{-n_1-1}}^{(m+1) 2^{-n_1-1}} \left|\frac{1}{n} \sum_{k=0}^{n-1} \wal_k(x) \right| \rd x. \end{align*} For $x \in [m 2^{-n_1-1}, (m+1) 2^{-n_1-1})$ we can write $\wal_k(x) = \wal_{m'}(y_k)$, where $m'=m'(m)=m_{n_1}+m_{n_1-1} 2 +\cdots+m_0 2^{n_1}$ for $m$ with binary expansion $m=m_0+m_1 2+\cdots+m_{n_1} 2^{n_1}$. Then \begin{align*} D^\ast_n(Y^{{\rm vdC}}) = & \frac{1}{2^{n_1+1}} \sum_{m'=0}^{2^{n_1+1}-1} \left| \frac{1}{n} \sum_{k=0}^{n-1} \wal_{m'}(y_k) \right|. \end{align*} On the other hand, a well known representation for the star discrepancy of the van der Corput sequence according to \cite[Th\'eor\`eme~1]{befa77} leads to a new representation for $L_n$ via $$L_n=d_n=n D_n^{\ast}(Y^{{\rm vdC}}) =\sum_{r=1}^{\infty} \left\|\frac{n}{2^r}\right\| = \sum_{r=1}^m \left\|\frac{n}{2^r}\right\|+\frac{n}{2^m}\ \ \ \mbox{ whenever $1 \le n \le 2^m$},$$ where $\|x\|=\min(\{x\},1-\{x\})$ is the distance of a real $x$ to the nearest integer. This formula immediately implies the recursion in Item~1 of Theorem~\ref{thm:fine} (note that $d_1=L_1=1$), which was already known for $d_n$ before (see~\cite[p.~13-07]{befa77}). Also the generating function for $L_n$ (see Item~5 in Theorem~\ref{thm:fine}) was unknown in terms of discrepancy, which can now be formulated as $$\sum_{n=1}^{\infty} d_n z^n =\frac{1}{2} \frac{z}{(1-z)^2} \sum_{k=0}^{\infty}\frac{1}{2^k} \frac{1-z^{2^k}}{1+z^{2^k}} \qquad \mbox{for $|z|<1$.}$$ For the star discrepancy of the van der Corput sequence we know a central limit theorem from \cite{dlp04}, which may now be formulated in terms of Lebesgue constants for the Walsh system. Accordingly, for every real $y$ and for $N \rightarrow \infty$ we have $$\frac{1}{N} \# \left\{ n < N \ : \ L_n \le \frac{\log n}{4 \log 2} + \frac{y}{4} \sqrt{ \frac{\log n}{3 \log 2}} \right\} = \Phi(y)+ o(1),$$ where $\Phi(y) = \frac1{\sqrt{2\pi}} \int_{-\infty}^y \exp(-t^2/2) \rd t $ is the Gaussian cumulative distribution function. That is, the Lebesgue constants for the Walsh system satisfy a central limit theorem. As the final example of this note we reformulate another result for the Lebesgue constants to obtain a so far unknown result for the star discrepancy of the van der Corput sequence. For $t \in [0,1]$ and $m \in \mathbb{N}$ let $n_t(m):= \lfloor 2^m(1+t)\rfloor$. Then \cite[Theorem~5]{AS2} in terms of discrepancy of the van der Corput sequence states: \begin{enumerate} \item For almost all $t \in [0,1]$ we have $$\lim_{m \rightarrow \infty} \frac{d_{n_t(m)}}{\log n_t(m) }=\frac{1}{4 \log 2}.$$ \item For all dyadic rational $t \in [0,1]$ we have $$\lim_{m \rightarrow \infty} \frac{d_{n_t(m)}}{\log n_t(m)}=0.$$ \item There exists a dense subset $A \subseteq [0,1]$ such that $$\liminf_{m \rightarrow \infty} \frac{d_{n_t(m)}}{\log n_t(m)}=0 \quad \mbox{ and }\quad \limsup_{m \rightarrow \infty} \frac{d_{n_t(m)}}{\log n_t(m)}=\frac{1}{3 \log 2}$$ for all $t \in A$. \end{enumerate} \begin{thebibliography}{10} \bibitem{AS1} S.V.~Astashkin, E.M.~Semenov: Lebesgue constants of a Walsh system. (Russian) Dokl. Akad. Nauk 462(5): 509--511, 2015; translation in Dokl. Math. 91(3): 344–346, 2015. \bibitem{AS2} S.V.~Astashkin, E.M.~Semenov: Lebesgue constants of a Walsh system and Banach limits. (Russian) Sibirsk. Mat. Zh. 57(3): 512--526, 2016; translation in Sib. Math. J. 57(3): 398--410, 2016. \bibitem{AT} A.P.~Austin, L.N.~Trefethen: Trigonometric interpolation and quadrature in perturbed points. SIAM J. Numer. Anal. 55(5): 2113--2122, 2017. \bibitem{befa} R.~B\'{e}jian, H.~Faure: Discr\'{e}pance de la suite de van der Corput. (French) C. R. Acad. Sci., Paris, S\'{e}r. A 285: 313--316, 1977. \bibitem{befa77} R.~B\'{e}jian, H.~Faure: Discr\'{e}pance de la suite de van der Corput. (French) S\'{e}minaire Delange-Pisot-Poitou (Th\'{e}orie des nombres) 13: 1--14,1977/78. \bibitem{DP10} J.~Dick, F. Pillichshammer: Digital Nets and Sequences. Discrepancy Theory and Quasi-Monte Carlo Integration. Cambridge University Press, Cambridge, 2010. \bibitem{dlp04} M.~Drmota, G.~Larcher, F.~Pillichshammer: Precise distribution properties of the van der Corput sequence and related sequences. Manuscripta Math. 118: 11--41, 2005. \bibitem{fau1990} H.~Faure: Discr\'{e}pance quadratique de la suite de van der Corput et de sa sym\'{e}trique. (French) Acta Arith. 55: 333--350, 1990. \bibitem{FKP} H.~Faure, P.~Kritzer, F.~Pillichshammer: From van der Corput to modern constructions of sequences for quasi-Monte Carlo rules. Indag. Math. 26: 760--822, 2015. \bibitem{fine} N.J.~Fine: On the Walsh functions. Trans. Amer. Math. Soc. 65: 372--414, 1949. \bibitem{hab1966} S.~Haber: On a sequence of points of interest for numerical quadrature. J. Res. Nat. Bur. Standards Sect. B70: 127--136, 1966. \bibitem{KS} S.~Kaczmarz, H.~Steinhaus: Theorie der Orthogonalreihen. (German) Monografie Matematyczne, Chelsea Publishing Company, New York, 1951. \bibitem{kuinie} L.~Kuipers, H.~Niederreiter: Uniform Distribution of Sequences. John Wiley, New York, 1974. \bibitem{p04} F.~Pillichshammer: On the discrepancy of $(0,1)$-sequences. J. Number Theory 104: 301--314, 2004. \bibitem{PA} P.D.~Pro\u{\i}nov, E.Y.~Atanassov: On the distribution of the van der Corput generalized sequences. C. R. Acad. Sci. Paris S\'er. I Math. 307(18): 895--900, 1988. \bibitem{SWS} F.~Schipp, W.R.~Wade, P.~Simon: Walsh Series. An Introduction to Dyadic Harmonic Analysis. Adam Hilger Ltd., Bristol, 1990. \bibitem{spi} L.~Spiegelhofer: Discrepancy results for the van der Corput sequence. Unif. Distrib. Theory 13(2): 57--69, 2018. \bibitem{vdc35} J.G.~van der Corput: Verteilungsfunktionen I-II. Proc. Akad. Amsterdam 38: 813--821, 1058--1066, 1935. \bibitem{weyl} H.~Weyl: \"Uber die Gleichverteilung mod. Eins. (German) Math. Ann. 77: 313--352, 1916. \bibitem{zyg} A.~Zygmund: Trigonometric Series. Cambridge University Press, New York, 1959. \end{thebibliography} \end{document}
2412.03234v3
http://arxiv.org/abs/2412.03234v3
$\mathrm{PGL}_2(\mathbb{C})$-character stacks and Langlands duality over finite fields
\documentclass[11pt, twoside]{article} \usepackage[a4paper,width=150mm,top=25mm,bottom=30mm]{geometry} \usepackage[all,cmtip]{xy} \usepackage{comment} \usepackage[pdftex]{pict2e} \usepackage{amsmath,amssymb,amscd,amsthm} \usepackage{a4wide,pstricks} \usepackage{pdfsync} \usepackage{fancyhdr} \usepackage{makebox, bigstrut} \pagestyle{fancy} \setlength{\headheight}{20pt} \fancyhead{} \fancyhead[LO,RE]{\thepage} \fancyhead[LE]{\leftmark} \fancyhead[RO]{\rightmark} \fancyfoot{} \setlength\parindent{0 pt} \usepackage[T1]{fontenc} \usepackage[utf8]{inputenc} \usepackage[english]{babel} \usepackage{setspace} \usepackage{bbm} \usepackage{leftindex} \usepackage{tasks} \usepackage{dsfont} \usepackage{amsthm} \usepackage{amsmath} \usepackage{amssymb} \usepackage{mathrsfs} \usepackage{mathtools} \usepackage{enumitem} \usepackage{hyperref} \usepackage{amssymb} \usepackage{calligra} \usepackage{ytableau} \usepackage{marginnote} \usepackage{cleveref} \usepackage{babel} \usepackage{bm} \usepackage{tikz-cd} \usepackage{faktor} \usepackage{stmaryrd} \renewcommand{\phi}{\varphi} \newcommand{\eps}{\varepsilon} \newcommand{\al}{\alpha} \newcommand{\ls}{\lesssim} \newcommand{\Ll}{\left} \newcommand{\Rr}{\right} \newcommand*\diff{\mathop{}\!\mathrm{d}} \let\oldemptyset\emptyset \let\emptyset\varnothing \newcommand\id{\ensuremath{\mathbbm{1}}} \newcommand{\C}{\mathbb{C}} \newcommand{\D}{\mathbf{D}} \newcommand{\E}{\mathbb{E}} \newcommand{\N}{\mathbb{N}} \renewcommand{\P}{{\mathbb P}} \newcommand{\Q}{{\mathbb Q}} \newcommand{\R}{\mathbb{R}} \newcommand{\T}{\mathbb{T}} \newcommand{\Z}{\mathbb{Z}} \newcommand{\F}{\mathbb{F}} \newcommand{\tens}[1]{ \mathbin{\mathop{\otimes}\displaylimits_{#1}}} \newcommand{\bigzero}{\mbox{\normalfont\Large\bfseries 0}} \newcommand{\rvline}{\hspace*{-\arraycolsep}\vline\hspace*{-\arraycolsep}} \xyoption{all} \input{xypic} \renewcommand{\baselinestretch}{1.1} \newcommand{\Cc}{\mathcal{C}} \newcommand{\Ch}{\mathcal{X}} \newcommand{\G}{\mathbb{G}_m} \newcommand{\Ss}{\mathcal{S}} \newcommand{\Bb}{\mathcal{B}} \newcommand{\Ff}{\mathcal{F}} \newcommand{\Hh}{\mathcal{H}} \theoremstyle{plain} \numberwithin{equation}{subsection} \crefformat{section}{\S#2#1#3} \crefformat{subsection}{\S#2#1#3} \crefformat{subsubsection}{\S#2#1#3} \setlength{\marginparwidth}{1.2in} \let\oldmarginpar\marginpar \renewcommand\marginpar[1]{\-\oldmarginpar[\raggedleft\footnotesize #1] {\raggedright\footnotesize #1}} \renewcommand{\sharp}{\#} \makeindex \newenvironment{parag}[1]{ \noindent{\bf\boldmath\stepcounter{parag}\arabic{section}.\arabic{parag} #1.}} \newenvironment{conjecturemark}{\noindent {\it Remark} \stepcounter{remark}\arabic{section}.\arabic{conjecture}.\arabic{remark}}{\vskip0.4cm} \newtheorem{teorema}{Theorem}[subsection] \newtheorem{prop}[teorema]{Proposition} \newtheorem{corollary}[teorema]{Corollary} \newtheorem{conjecture}[teorema]{Conjecture} \newtheorem{lemma}[teorema]{Lemma} \newtheorem{property}[teorema]{Property} \newtheorem{problem}[teorema]{Problem} \theoremstyle{remark} \newtheorem{oss}[teorema]{Remark} \newtheorem{remark}[teorema]{Remark} \newtheorem{esempio}[teorema]{Example} \newtheorem{conj}[teorema]{Conjecture} \newtheorem{construction}[teorema]{Construction} \theoremstyle{definition} \newtheorem{definizione}[teorema]{Definition} \newtheorem{nothing}[teorema]{} \newcounter{margin} \newenvironment{margin} {\stepcounter{margin} \bigskip \par\noindent\textbf{Note \arabic{margin}.} \begin{itshape}} {\end{itshape} \bigskip} \usepackage{tikz} \DeclareMathOperator{\X}{{\bf X}} \DeclareMathOperator{\Hom}{Hom} \DeclareMathOperator{\sz}{size} \DeclareMathOperator{\Mod}{Mod} \DeclareMathOperator{\Conn}{Conn} \DeclareMathOperator{\Loc}{Loc} \DeclareMathOperator{\Mat}{Mat} \DeclareMathOperator{\Dim}{Dim} \DeclareMathOperator{\Deg}{deg} \DeclareMathOperator{\an}{an} \DeclareMathOperator{\ord}{ord} \DeclareMathOperator{\Res}{Res} \DeclareMathOperator{\Ker}{Ker} \DeclareMathOperator{\Sh}{Sh} \DeclareMathOperator{\Coker}{Coker} \DeclareMathOperator{\Supp}{supp} \DeclareMathOperator{\Gl}{GL} \DeclareMathOperator{\End}{End} \DeclareMathOperator{\Imm}{Im} \DeclareMathOperator{\reg}{reg} \DeclareMathOperator{\Moob}{mod} \DeclareMathOperator{\Quot}{Quot} \DeclareMathOperator{\sm}{sm} \DeclareMathOperator{\sing}{sing} \DeclareMathOperator{\Sol}{Sol} \DeclareMathOperator{\Mon}{Mon} \DeclareMathOperator{\Ann}{Ann} \DeclareMathOperator{\mero}{mero} \DeclareMathOperator{\su}{su} \DeclareMathOperator{\alg}{alg} \DeclareMathOperator{\ide}{id} \DeclareMathOperator{\curv}{Curv} \DeclareMathOperator{\Aut}{Aut} \DeclareMathOperator{\Der}{Der} \DeclareMathOperator{\tr}{Tr} \DeclareMathOperator{\Dn}{D} \DeclareMathOperator{\ch}{char} \DeclareMathOperator{\spec}{Spec} \DeclareMathOperator{\Sl}{SL} \DeclareMathOperator{\pic}{Pic} \DeclareMathOperator{\Proj}{Proj} \DeclareMathOperator{\Span}{Span} \DeclareMathOperator{\Stab}{Stab} \DeclareMathOperator{\Infl}{Infl} \DeclareMathOperator{\grr}{gr} \DeclareMathOperator{\Rep}{Rep} \DeclareMathOperator{\Lie}{Lie} \DeclareMathOperator{\Ext}{Ext} \DeclareMathOperator{\topp}{top} \DeclareMathOperator{\sgn}{sgn} \DeclareMathOperator{\Rad}{Rad} \DeclareMathOperator{\Gal}{Gal} \DeclareMathOperator{\Con}{Con} \DeclareMathOperator{\Ad}{Ad} \DeclareMathOperator{\Real}{Re} \DeclareMathOperator{\Fr}{Fr} \DeclareMathOperator{\Sym}{Sym} \DeclareMathOperator{\Plexp}{Exp} \DeclareMathOperator{\Plelog}{Log} \DeclareMathOperator{\St}{St} \DeclareMathOperator{\Ind}{Ind} \DeclareMathOperator{\Degpar}{degpar} \DeclareMathOperator{\Coeff}{Coeff} \DeclareMathOperator{\grrep}{GrRep} \DeclareMathOperator{\rank}{rank} \DeclareMathOperator{\chh}{ch} \DeclareMathOperator{\PGl}{PGL} \DeclareMathOperator{\Cl}{Cl} \DeclareMathOperator{\Sch}{Sch} \DeclareMathOperator{\IC}{IC} \DeclareMathOperator{\codim}{codim} \DeclareMathOperator{\triv}{triv} \DeclareMathOperator*{\rightoverleft}{\parbox{2em}{\centerline{$\longrightarrow$}\vskip -6pt\centerline{$\longleftarrow$}}} \begin{document} \title{${\rm PGL}_2(\C)$-character stacks and Langlands duality over finite fields} \author{ Emmanuel Letellier \\ {\it Universit\'e Paris Cit\'e, IMJ-PRG CNRS UMR 7586} \\{\tt [email protected] } \and Tommaso Scognamiglio \\ {\it University of Heidelberg} \\ {\tt [email protected]} } \pagestyle{myheadings} \maketitle \begin{abstract}In this paper we study the mixed Poincar\'e polynomials of generic $\PGl_2(\C)$-character stacks with coefficients in some local systems arising from the conjugacy class of $\PGl_2(\C)$ which has a non-connected stabiliser. We give a conjectural formula that we prove to be true under the Euler specialisation. We then prove that these conjectured formulas interpolate the structure coefficients of the two following based rings: $$ \left(\mathcal{C}(\PGl_2(\F_q)),Loc(\PGl_2),*\right),\hspace{1cm}\left(\mathcal{C}(\Sl_2(\F_q)), CS(\Sl_2),\cdot\right) $$ where for a group $H$, $\mathcal{C}(H)$ denotes the space of complex valued class functions on $H$, $Loc(\PGl_2)$ denotes the basis of characteristic functions of intermediate extensions of equivariant local systems on conjugacy classes of $\PGl_2$ and $CS(\Sl_2)$ the basis of characteristic functions of Lusztig's character-sheaves on $\Sl_2$. Our result reminds us of a non-abelian Fourier transform. \end{abstract} \tableofcontents \maketitle \section{Introduction} \bigskip Given a finite group $H$ we denote by $\mathcal{C}(H)$ the $\C$-vector space of $\C$-valued class functions on $H$. It is equipped with the basis $Cl(H)$ of characteristic functions of the conjugacy classes of $H$ and the basis $\widehat{H}$ of irreducible characters of $H$. We then consider the two rings $$ \mathcal{A}=(\mathcal{C}(H),Cl(H),*),\hspace{.5cm}\text{and }\hspace{.5cm}\mathcal{B}=(\mathcal{C}(H),\widehat{H},\cdot) $$ where $*$ is the convolution product and $\cdot$ the pointwise multiplication. The first ring can be identified with the center of the group algebra of $H$ and the second one is the character ring of $H$. \bigskip These rings are determined by their structure coefficients $$ \left\langle 1_{C_1}*1_{C_2},1_{C_3}\right\rangle_H,\hspace{1cm}\left\langle\chi_1\otimes\chi_2,\chi_3\right\rangle_H $$ where $\langle\,,\,\rangle_H$ is the usual inner product on $\mathcal{C}(H)$ making the basis $\widehat{H}$ orthonormal and where $C_1, C_2, C_3$ run over the set of conjugacy classes of $H$ and $\chi_1,\chi_2,\chi_3$ run over $\widehat{H}$. \bigskip When $H$ is abelian, these two rings are isomorphic, namely there is a (non-canonical) automorphism of $\mathcal{C}(H)$ mapping $Cl(H)$ to $\widehat{H}$ and transforming $*$ into $\cdot$ (which can be thought as a Fourier transform). \bigskip We are interested in the case where $H$ is the group $G(\F_q)$ of rational points of a connected reductive group $G$ defined over a finite field $\F_q$. \bigskip For that purpose we need a base other than $Cl(G^F)$ and $\widehat{G^F}$ : \bigskip Denote by $F:G\rightarrow G$ the geometric Frobenius (then $G^F=G(\F_q)$). \bigskip Instead of the basis $Cl(G^F)$ we consider the basis $Loc(G)^F$ formed by the characteristic functions of the intermediate extensions of the irreducible $F$-equivariant $G$-equivariant local systems on the $F$-stable conjugacy classes of $G$, and instead of the basis $\widehat{G^F}$ we consider the basis $CS(G)^F$ formed by the characteristic functions of the $F$-equivariant Lusztig's character-sheaves on $G$ (which are closely related to irreducible characters and coincide often with them). \bigskip We are interested in the relationship between the structure coefficients of the two based rings $$ \mathcal{A}_G=\left(\mathcal{C}(G^F),Loc(G)^F,*\right),\hspace{1cm}\mathcal{B}_{G^*}=\left(\mathcal{C}(G^*{^F}), CS(G^*)^F,\cdot\right) $$ where $(G^*,F)$ is the dual group of $(G,F)$ in the sense of Deligne-Lusztig \cite{DL}. \bigskip \subsection{Review of the case $G=\mathrm{GL}_n$} \bigskip In this case $G=G^*$ and the basis $CS(G)^F$ coincides with $\widehat{G^F}$. The stabilisers of the elements of $G$ being all connected, the irreducible $G$-equivariant local systems on the conjugacy classes are the constant sheaf. Therefore, the elements of $Loc(G)^F$ and $Cl(G^F)$ attached to semisimple conjugacy classes coincide. \bigskip Given a \emph{generic} $k$-tuple $\mathcal{C}=(\mathcal{C}_1,\dots,\mathcal{C}_k)$ of conjugacy classes of $G(\C)$ we consider the quotient stack (possibly empty) $$ \mathcal{M}_{\overline{\mathcal{C}}}=\left[\{(x_1,\dots,x_k)\in\overline{\mathcal{C}_1}\times\cdots\times\overline{\mathcal{C}_k}\,|\, x_1\cdots x_k=1\}/G\right]. $$ The genericity assumption ensures that the diagonal action of $G$ induces a free action of $\PGl_n$. In particular, the stack $\mathcal{M}_{\overline{\mathcal{C}}}$ is a $\mathbb{G}_m$-gerbe over the smooth variety $M_{\overline{\mathcal{C}}}$ obtained by taking the quotient by $\PGl_n$. We still take the quotient by $G=\Gl_n$ in the definition since otherwise we would have to to correct some formulas (like Formula (\ref{eq-intro}) by a factor $q-1$. \bigskip The stack $\mathcal{M}_{\overline{\mathcal{C}}}$, called a $G$-character stack, is the moduli stack of $G$-local systems on the Riemann sphere $\mathbb{P}^1_{\C}$ minus $k$ points $a_1,\dots,a_k$ with local monodromies around $a_i$ in $\overline{\mathcal{C}_i}$. \bigskip We consider the compactly supported intersection cohomology $IH_c^i(\mathcal{M}_{\overline{\mathcal{C}}},\C)$ of $\mathcal{M}_{\overline{\mathcal{C}}}$. \bigskip It is equipped with a weight filtration (increasing) $W^i_\bullet$ from which we define the mixed Poincar\'e series $$ IH_c(\mathcal{M}_{\overline{\mathcal{C}}};q,t)=\sum_{i,r}{\rm dim}\left(W^i_r/W^i_{r-1}\right)q^{r/2}t^i. $$ Our conjugacy classes are defined over a subring $R$ of $\C$ which is finitely generated as a $\mathbb{Z}$-algebra. We can choose this subring $R$ such that for any finite field $\F_q$ and any ring homomorphism $\varphi:R\rightarrow \F_q$, the conjugacy classes of $G(\overline{\F}_q)$ obtained from $\mathcal{C}_1,\dots,\mathcal{C}_k$ by base change are of same Jordan type and form a generic $k$-tuple. To ease the notation we denote them again by $\mathcal{C}_1,\dots,\mathcal{C}_k$. \bigskip Then we have \cite[Theorem 4.13]{L} \begin{align} E(\mathcal{M}_{\overline{\mathcal{C}}};q):&=IH_c(\mathcal{M}_{\overline{\mathcal{C}}};q,-1)\\ &=\left\langle {\bf Y}_{\mathcal{C}_1}*\cdots*{\bf Y}_{\mathcal{C}_k},1_{\{1\}}\right\rangle_{G(\F_q)}\label{eq-intro} \end{align} where ${\bf Y}_\mathcal{C}:G(\F_q)\rightarrow \overline{\Q}_\ell\simeq\C$ denotes the characteristic function of the intersection cohomology complex of $\overline{\mathcal{C}}$ (in the $\ell$-adic setting). \bigskip In \cite[Conjecture 4.5]{L} we give a conjecture formula for $IH_c(\mathcal{M}_{\overline{\mathcal{C}}};q,t)$ in terms of Macdonald symmetric functions. When the conjugacy classes are semisimple, this was first conjectured in \cite{HA} (see \S \ref{GL2} for a review in the semisimple case). The conjecture is known to be true in some non-trivial examples where we can compute explicitly the weight filtration (see \cite[\S 1.5.3]{HA}). \bigskip This conjectural formula is proved after the specialisation $t\mapsto -1$ (see \cite[Theorem 4.8]{L} and \cite[Theorem 1.2.3]{HA} in the semisimple case). It is also proved after the specialisation $q\mapsto 1$ which gives the Poincar\'e series. In the semisimple case this is due to by A. Mellit \cite{Mellit} who followed a strategy used by O. Schiffmann to compute the Poincar\'e polynomial of the moduli space of semistable Higgs bundles over a smooth projective curve \cite{schiffmann}. In the general case, this is due to M. Ballandras \cite{ballandras}. \bigskip We define the "pure part" of $IH_c(\mathcal{M}_{\overline{\mathcal{C}}};q,t)$ as $$ PIH_c(\mathcal{M}_{\overline{\mathcal{C}}};q)=\sum_i{\rm dim}\, \left(W^i_i/W^i_{i-1}\right) q^{i/2}. $$ The conjectural formula for $IH_c(\mathcal{M}_{\overline{\mathcal{C}}};q,t)$ implies a conjectural formula for $PIH_c(\mathcal{M}_{\overline{\mathcal{C}}};q)$ in terms of Hall-Littlewood symmetric functions. In \cite[Theorem 6.10.1]{letellier2} we prove that the conjectured formula for the pure part coincides with the inner product $$ \left\langle \chi_1\otimes\cdots\otimes\chi_k,1\right\rangle_{G(\F_q)} $$ where $(\chi_1,\dots,\chi_k)$ is a generic $k$-tuple of irreducible characters of $G(\F_q)$ of same Jordan type (in the sense of Lusztig) as the conjugacy classes $\mathcal{C}_1,\dots,\mathcal{C}_k$. \bigskip Therefore, it follows from the main conjecture for the mixed Poincar\'e series of character stacks that the latter interpolate (generically) the structure coefficients of the two rings $\mathcal{A}_{\Gl_n}$ and $\mathcal{B}_{\Gl_n}$. \bigskip The non-generic situation is more complicated and will not be discussed in this paper. However we refer to \cite{scognamiglio1}\cite{scognamiglio2} for results in the non-generic semisimple case. \bigskip Finally to conclude this part on the $G=\Gl_n$ case, notice that there is a notion of character-sheaves on the Lie algebra of $G$ due to Lusztig \cite{Lu-Fourier} (see also \cite{Let}). The Lie algebra analogues $\mathcal{A}^{Lie}$ and $\mathcal{B}^{Lie}$ of these two rings $\mathcal{A}$ and $\mathcal{B}$ are then isomorphic by an arithmetic Fourier transform (the character-sheaves are precisely the Deligne-Fourier transforms of the intersection cohomology complexes on the Zariski closures of the adjoint orbits of the Lie algebra). The generic structure coefficients of $\mathcal{A}^{Lie}$ and $\mathcal{B}^{Lie}$ are thus equal. Moreover we can prove that the generic structure coefficients of $\mathcal{B}^{Lie}$ and $\mathcal{B}$ are also equal \cite[Theorem 6.9.1]{letellier2} and coincide \cite{HA}\cite{letellier2} with the Poincar\'e series (for intersection cohomology) of $$ \mathcal{Q}=\left[\{(x_1,\dots,x_k)\in\overline{\mathcal{O}_1}\times\dots\times\overline{\mathcal{O}_k}\,|\, x_1+\cdots+x_k=0\}/G\right] $$ where $(\mathcal{O}_1,\dots,\mathcal{O}_k)$ runs over the set of \emph{generic} $k$-tuple of adjoint orbits of $\mathfrak{gl}_n(\C)$ (unlike the multiplicative case, generic $k$-tuples of adjoint orbits of given Jordan types do not always exist). Finally, unlike character stacks, the intersection cohomology of $\mathcal{Q}$ is pure \cite[Proposition 2.2.6]{HA}\cite[Theorem 7.3.2]{letellier2} and its Poincar\'e series is conjectured to be the pure part of the mixed Poincar\'e series of character stacks. This latter conjecture is also a consequence of the conjectural formula for the mixed Poincar\'e series of generic character stacks. \bigskip \subsection{The case where $G=\mathrm{PGL}_2$} \bigskip The aim of this paper is to verify for a group $G\neq G^*$ the assumption that the mixed Poincar\'e series of $G$-character stacks interpolate structure coefficients of the two rings $\mathcal{A}_G$ and $\mathcal{B}_{G^*}$. We are especially interested in the situation where local monodromies are in conjugacy classes which carry non-trivial $G$-equivariant local systems. \bigskip We now assume that $G=\PGl_2$. \bigskip When $K$ is an algebraically closed field of characteristic $\neq 2$, there is only one conjugacy class of $G(K)$ which has a disconnected centraliser. This is the $G(K)$-conjugacy class $\mathcal{C}_{-1}$ of the image in $\PGl_2(K)$ of the diagonal matrix with coefficients $1$ and $-1$. There are thus two irreducible (up to isomorphism) $G(K)$-equivariant local systems on $\mathcal{C}_{-1}$ which are naturally parametrised by the set $\widehat{\Z/2\Z}=\{{\rm Id},\epsilon\}$ of irreducible characters of the group of connected components of the centraliser. We will denote by $\mathcal{L}_\epsilon$ the non-trivial equivariant irreducible local systems on $\mathcal{C}_{-1}$ (when the characteristic of $K$ is positive, the local systems considered are in the $\ell$-adic setting). \bigskip We will call $\mathcal{C}_{-1}$ the \emph{degenerate} conjugacy class. The other conjugacy classes (non-degenerate) have all a connected centraliser and so the constant sheaf is the only irreducible $G(K)$-equivariant local system on them. \bigskip {\bf $\PGl_2$-character stacks} \bigskip Consider a \emph{generic} $k$-tuple $\mathcal{C}=(\mathcal{C}_1,\dots,\mathcal{C}_k)$ of semisimple regular conjugacy classes of $G(\C)$ (see Definition \ref{definitiongenericity}) and consider the $G(\C)$-character stack $$ \mathcal{M}_\mathcal{C}=\mathcal{M}_\mathcal{C}(\C):=[\{(x_1,\dots,x_k)\in\mathcal{C}_1\times\cdots\times\mathcal{C}_k\,|\, x_1\cdots x_k=1\}/G]. $$ Choose on each $\mathcal{C}_i$ an irreducible $G$-equivariant local system $\mathcal{L}_i$ (it is the trivial local system if $\mathcal{C}_i$ is non-degenerate and can be either the trivial local system or $\mathcal{L}_\epsilon$ if $\mathcal{C}_i=\mathcal{C}_{-1}$). This defines a local system $\mathcal{L}_\mathcal{C}$ on $\mathcal{M}_\mathcal{C}$. \bigskip We consider the mixed Poincar\'e polynomial $$ H_c(\mathcal{M}_\mathcal{C},\mathcal{L}_\mathcal{C};q,t)=\sum_{i,r}{\rm dim}\left(W^i_r/W^i_{r-1}\right)q^{r/2}t^i $$ where $W^i_\bullet$ is the weight filtration on $H_c^i(\mathcal{M}_\mathcal{C},\mathcal{L}_\mathcal{C})$. \bigskip In this paper we give an explicit conjectural formula for $H_c(\mathcal{M}_\mathcal{C},\mathcal{L}_\mathcal{C};q,t)$ (see Conjecture \ref{conjEnondegenerate} for the non-degenerate case and Conjecture \ref{conjmhslocalsystems} when at least one conjugacy class is degenerate) and we prove that it is true after the specialisation $t\mapsto -1$ (see Theorem \ref{theoremEnondegenerate} and Theorem \ref{Epolynomialgenericdegenerate}). \bigskip Our conjectural formula for $H_c(\mathcal{M}_\mathcal{C},\mathcal{L}_\mathcal{C};q,t)$ is a consequence of the conjectural formula for the mixed Poincar\'e polynomials of $\Gl_n$-character varieties for which we have many evidences. \bigskip {\bf Langlands duality over finite fields (Lusztig) when $G=\PGl_2$} \bigskip Assume that $K=\overline{\F}_q$ with odd $q$ and that $F:G\rightarrow G$ is the standard Frobenius ($G(\F_q)=G^F$). \bigskip We now explain the correspondence between $F$-stable local systems on $F$-stable semisimple conjugacy classes of $\PGl_2(K)$ and $F$-stable (semisimple) character-sheaves on $\Sl_2(K)$. \bigskip For an $F$-stable $G$-equivariant local system $\mathcal{E}$ on an $F$-stable semisimple conjugacy class $\mathcal{C}$ of $G$, we denote by ${\bf Y}_{(\mathcal{C},\mathcal{E})}:G^F\rightarrow\overline{\Q}_\ell$ the characteristic function of $\mathcal{E}$ defined by $$ {\bf Y}_{(\mathcal{C},\mathcal{E})}(x)=\tr(Frob, \mathcal{E}_x).$$ (1) If $\mathcal{E}=\overline{\Q}_\ell$, then ${\bf Y}_{(\mathcal{C},\overline{\Q}_\ell)}$ is the function $1_{\mathcal{C}^F}$ that takes the value $1$ on $\mathcal{C}^F$ and $0$ elsewhere. \bigskip (2) If $(\mathcal{C},\mathcal{E})=(\mathcal{C}_{-1},\mathcal{L}_\epsilon)$, then $$ {\bf Y}_{(\mathcal{C},\mathcal{E})}=1_{\mathcal{O}_{e}}-1_{\mathcal{O}_{\sigma}} $$ where $$ (\mathcal{C}_{-1})^F=\mathcal{O}_{e}\sqcup\mathcal{O}_{\sigma} $$ is the decomposition into $G^F$-conjugacy classes (see Remark \ref{rmdecomp}). \bigskip Denote by $T$ and $T^*$ the maximal tori respectively of $\PGl_2$ and $\Sl_2$ of diagonal matrices. We have a non-canonical group isomorphism $$ T^F\simeq\F_q^*\overset{\sim}{\longrightarrow}\widehat{\F_q^*}\simeq\widehat{T^*{^F}} $$ under which $-1$, the non-trivial square of $1$, corresponds to the non-trivial square $\alpha_o$ of the identity character of $T^*{^F}$. The character $\alpha_o$ takes the value $1$ at squares elements and $-1$ at non-square elements. \bigskip Now the group of linear characters of $T^*{^F}$ is isomorphic with the group of isomorphism classes of the $F$-stable Kummer local systems on $T^*$ (see Proposition \ref{bijectionkummer}). \bigskip We thus have an isomorphism $x\mapsto \mathcal{E}_x$ from $\F_q^*\simeq T^F$ to the group of isomorphism classes of $F$-stable Kummer local systems on $T^*$. \bigskip Denote by $\mathcal{I}_{T^*}^{G^*}:\mathcal{M}(T^*;F)\rightarrow\mathcal{M}(G^*;F)$ the geometric induction functor between categories of $F$-equivariant perverse sheaves (see \S \ref{sectionspringerresolution}). \bigskip Let $\mathcal{E}_x$ be an $F$-stable Kummer local system on $T^*$. We have the following two situations : \bigskip (1) If $x\neq -1,1$, then $\mathcal{I}_{T^*}^{G^*}(\mathcal{E}_x)$ is an $F$-stable irreducible $G^*$-equivariant perverse sheaf on $G^*$ which is an example of a character-sheaf (its characteristic function is an irreducible character of $G^*{^F}$). \bigskip (2) If $x=-1$, then $\mathcal{I}_{T^*}^{G^*}(\mathcal{E}_x)$ is an $F$-stable $G^*$-equivariant semisimple perverse sheaf whose simple direct factors are naturally parametrised by $\widehat{\Z/2\Z}=\{{\rm Id},\epsilon\}$ : $$ \mathcal{I}_{T^*}^{G^*}(\mathcal{E}_x)=\mathcal{X}_{{\rm Id}}[3]\oplus\mathcal{X}_\epsilon[3]. $$ The perverse sheaves $\mathcal{X}_{{\rm Id}}[3]$ and $\mathcal{X}_\epsilon[3]$ are other examples of character-sheaves. However unlike the ones above their characteristic functions are not irreducible characters (see the end of \S \ref{CS-SL}). \bigskip We thus have a bijection between irreducible equivariant local systems on semisimple conjugcy classes of $G$ and certain character sheaves on $G^*$ : $$ \Phi:\left\{(\mathcal{C}_x,\overline{\Q}_\ell)\,|\, x\neq 1,-1\right\}\sqcup\{(\mathcal{C}_{-1},\mathcal{L}_r)\,|\, r={\rm Id},\epsilon\}\simeq\left\{\mathcal{I}_{T^*}^{G^*}(\mathcal{E}_x)\,|\, x\neq 1,-1\right\}\sqcup\left\{\mathcal{X}_r[3]\,|\, r={\rm Id},\epsilon\right\} $$ where $\mathcal{C}_x$ is the (semisimple) $G$-conjugacy class of $x\in\F_q^*\simeq T^F$, and where $\mathcal{L}_{{\rm Id}}$ is the constant sheaf on $\mathcal{C}_{-1}$. \bigskip Notice that on the left hand side we are missing the unipotent conjugacy classes and also the non-split ones (i.e. the ones with eigenvalues in $\F_{q^2}\backslash\F_q$) and on the right hand side we are missing the unipotent character-sheaves (the simple direct factors of $\mathcal{I}_{T^*}^{G^*}(\overline{\Q}_\ell)$ whose characteristic functions are the irreducible unipotent characters of $G^*{^F}$) and the two cuspidal character-sheaves (see \S \ref{CS-SL}). \bigskip {\bf Main result} \bigskip Let $\mathcal{C}=(\mathcal{C}_1,\dots,\mathcal{C}_k)$ be a generic $k$-tuple of semisimple regular conjugacy classes of $G(\C)$. Choose for each $i$ an irreducible $G(\C)$-equivariant local system $\mathcal{L}_i$ on $\mathcal{C}_ i$ and denote by $\mathcal{L}_\mathcal{C}$ the resulting local system on $\mathcal{M}_\mathcal{C}(\C)$. \bigskip Choose a generic $k$-tuple of $F$-stable semisimple regular conjugacy classes of $G(\overline{\F}_q)$ whose $i$-th coordinate is degenerate (i.e. equals to $\mathcal{C}_{-1}$) if and only if $\mathcal{C}_i$ is degenerate. To ease the notation we will denote this $k$-tuple again by $\mathcal{C}=(\mathcal{C}_1,\dots,\mathcal{C}_k)$. On each conjugacy class we choose the same local system as in the complex case and denote it again by $\mathcal{L}_i$. We then denote by ${\bf Y}_i:G^F\rightarrow\overline{\Q}_\ell\simeq\C$ the characteristic function ${\bf Y}_{(\mathcal{C}_i,\mathcal{L}_i)}$ and by ${\bf X}_i$ the characteristic function of the character-sheaf of $G^*$ which corresponds to $(\mathcal{C}_i,\mathcal{L}_i)$ under the isomorphism $\Phi$. \bigskip Our main theorem is the following one (see \S \ref{statement}): \begin{teorema} We have (1) $$H_c(\mathcal{M}_\mathcal{C}(\mathbb{C}),\mathcal{L}_\mathcal{C};q,-1)=\left\langle {\bf Y}_1*\cdots*{\bf Y}_k,1_{\{1\}}\right\rangle_{G^F}.$$ (2) The conjectured formula for the pure part $$ PH_c(\mathcal{M}_\mathcal{C}(\C),\mathcal{L}_\mathcal{C};q):=\sum_i{\rm dim}\,\left(W^i_i/W^i_{i-1}\right)\, q^{i/2} $$ equals $$ q^{k-3}\left\langle{\bf X}_1\cdots {\bf X}_k,1\right\rangle_{G^*{^F}}. $$ \end{teorema} Our result reminds us of a Fourier transform. \bigskip \section{$\mathrm{GL}_2$-character varieties } \subsection{Preliminaries on cohomology}\label{polynomialcount-paragraph} In the following, $K$ is an algebraically closed field of $\ch \neq 2$, which is either $\C$ or $\overline{\F}_q$ and $X$ an algebraic variety over $K$. We denote by $H^*_c(X)$ its compactly supported cohomology, with $\C$ coefficients if $K=\C$ and $\overline{\Q}_{\ell}$-coefficients if $K=\overline{\F}_q$. If $K=\C$, each cohomology group $H^k_c(X)$ is equipped with the weight filtration $W^i_{\bullet}$, introduced by Deligne in \cite{HodgeIII}. If $K=\overline{\F}_q$ and if we assume given a geometric Frobenius $F:X\rightarrow X$ (or, equivalently, an $\F_q$-scheme $X_o$ such that $X=X_o\times_{\F_q}\overline{\F}_q$) we have a weight filtration $W^k_{\bullet}H^k_c(X)$, where $W^k_{m}H^k_c(X)$ is the subspace on which the eigenvalues of the Frobenius $F$ are of absolute value $\leq q^{\frac{m}{2}}$. \vspace{4 pt} In both cases, we define the mixed Poincar\'e polynomial $H_c(X;q,t) \in \Z[\sqrt{q},t]$ $$ H_c(X;q,t)\coloneqq\sum_{i,k}{\rm dim}(W^k_i/W^k_{i-1})q^{i/2}t^k. $$ Then $ H_c(X;1,t)=\sum_k{\rm dim}\, H_c^k(X)\, t^k$ is the compactly supported Poincar\'e polynomial and $H_c(X;q,-1)$ is the so-called $E$-polynomial denoted by $E(X;q)$. In our cases, $H_c(X,q,t)$ and $E(X,q)$ will be actual polynomial in $q$, i.e. if $W^k_i/W^k_{i-1} \neq 0$ then $i$ is even. We also define the pure part $PH_c(X;q)$ as $$PH_c(X;q)\coloneqq \sum_{k}\dim(W^k_k/W^k_{k-1})q^{k/2}.$$ \bigskip Given a variety $X/_{\overline{\F}_q}$ with Frobenius $F$, we say that $X/_{\overline{\F}_q}$ has polynomial count if there exists $P_{X}(t) \in \Z[t]$ such that $$P_{X}(q^m)=|X^{F^m}|=|X_o(\F_{q^m})|$$ for all integer $m>0$. In this case, we have the following \cite[Theorem 2.2.6]{LRV}. \begin{teorema} \label{countingfq} If $X/_{\overline{\F}_q}$ has polynomial count with counting polynomial $P_{X}(t)$, we have $$P_{X}(q)=E(X;q) .$$ \end{teorema} \vspace{4 pt} \begin{esempio} The variety $X=\overline{\F}_q^*$, with the Frobenius $F(z)=z^q$ has polynomial count with counting polynomial $P_{X}(t)=t-1$. Indeed, in this case we have $$|X^{F^m}|=|\F_{q^m}^*|=q^m-1 .$$ Notice that $H_c(X;q,t)=qt^2+t$ and thus $E(X;q)=P_X(q)$. \end{esempio} \bigskip For a variety $X/_{\C}$, we say that $X$ has polynomial count with counting polynomial $P_X(t)$ if there exists a finitely generated $\Z$-subalgebra $R \subseteq \C$ and an $R$-scheme $X_R$ such that $$X_R \times_R \C \cong X ,$$and such that, for any ring homomorphism $\phi:R \to \F_q$, the variety $X^{\phi}=X_R\times_R\overline{\F}_q$ has polynomial count with counting polynomial $P_X(t)$. We have the following result. \begin{teorema}\cite[Appendix by Katz]{HRV} If $X/_\C$ has polynomial count with counting polynomial $P_X(t)$, then \begin{equation} \label{katzformula} E(X;q)=P_X(q). \end{equation} \label{Katz}\end{teorema} Katz's formulation of the above theorem gives also information on the Hodge filtration. If we do not bother about the Hodge filtration (as it is the case in this paper) we can prove the above theorem in a simple way as follows. \vspace{4 pt} Fix an isomorphism $\C \cong \overline{\Q}_{\ell}$ and identify $H^*_c(X,\C) \cong H^*_c(X,\overline{\Q}_{\ell})$ through this isomorphism. Recall that, if $\ch(\overline{\F}_q)$ is sufficiently large, we have a natural isomorphism \begin{equation} \label{comparisonisom} H^*_c(X^{\phi}) \cong H^*_c(X). \end{equation} Indeed, let $f:X_R \to \spec(R)$ be the structural morphism. The complex $Rf_!\overline{\Q}_{\ell}$ is constructible, see for instance \cite[Chapter 2]{Deligne-etale}. In particular, there exists a non-empty open $U \subseteq \spec(R)$ such that $Rf_!\overline{\Q}_{\ell}$ is constant. Denote by $\xi:\spec(\C) \to \spec(R)$ the (geometric) generic point of $\spec(R)$ coming from the embedding $R \subseteq \C$ and, for any $\phi:R \to \F_q$, denote by $\xi_{\phi}:\spec(\overline{\F}_q) \to \spec(R)$ the corresponding geometric point. If $\Imm(\xi_{\phi}) \in U$, from the fact that $Rf_!\overline{\Q}_{\ell}$ is constant and from the proper base change theorem, we have the following chain of isomorphisms: \begin{equation} \label{isomorphism-cohomology-constant} H^*_c(X^{\phi}) \cong (Rf_!\overline{\Q}_{\ell})_{\xi_{\phi}} \cong (Rf_!\overline{\Q}_{\ell})_{\xi} \cong H^*_c(X) .\end{equation} The results of \cite[Theorem 14]{Deligne} show that the isomorphism (\ref{comparisonisom}) preserves the weight filtration on both sides. In particular, we have an equality \begin{equation} \label{comparisonmhp} H_c(X;q,t)=H_c(X^{\phi};q,t). \end{equation} We deduce thus Formula (\ref{katzformula}) from Theorem \ref{countingfq} and formula (\ref{comparisonmhp}). \subsection{Review on cohomology of $\mathrm{GL}_n$-character varieties}\label{GL2} \label{paragraphcohomologyGl2characterstacks} To alleviate the notation we will put $\Gl_n=\Gl_n(K)$. If $K=\overline{\F}_q$, we denote by $F:\Gl_n \to \Gl_n$ the standard Frobenius $F((a_{i,j}))=(a_{i,j}^q)$. \bigskip Let $C$ be a $k$-tuple $C=(C_1,\dots,C_k)$ of semisimple conjugacy classes of $\Gl_n$. If $K=\overline{\F}_q$, we assume that each $C_i$ is $F$-stable and split, i.e. its eigenvalues belong to $\F_q^*$. We define the corresponding $\Gl_n$-character stack $\mathcal{M}_C$ as the quotient stack $$ \mathcal{M}_C=\left[\{(x_1,\dots,x_k)\in C_1\times\cdots\times C_k\,|\, x_1\cdots x_k=1\}/\PGl_n\right] $$ for the diagonal conjugation action of $\PGl_n$. \bigskip Following \cite[Definition 2.1.1]{HA}, we have the following definition of a \emph{generic} $k$-tuple $C$. \begin{definizione} The $k$-tuple $C$ is generic if \begin{equation} \prod_{i=1}^k\det(C_i)=1, \label{det}\end{equation} and for any integer $1\leq r<n$, if we select $r$ eigenvalues of $C_i$ (for each $i=1,\dots,k$), the product of the $nr$ selected eigenvalues is different from $1$. \end{definizione} \bigskip For each $i$ let $\mu^i$ be the partition of $n$ whose parts are given by the multiplicities of the eigenvalues of $C_i$. We say that $C_i$ is of type $\mu^i$ and that the $k$-tuple $C$ is of type ${\bm \mu}=(\mu^1,\dots,\mu^k)$. \bigskip \begin{remark}Recall \cite[Lemma 2.1.2]{HA} that for any $k$-tuple ${\bm \mu}=(\mu^1,\dots,\mu^k)$ of partitions of $n$, if $\ch(K)$ does not divide $\gcd(\mu^j_i)_{i,j}$, there always exists a generic $k$-tuple of semisimple conjugacy classes of $\Gl_n$ of type $\bm\mu$. \end{remark} Recall the following result \cite[Proposition 2.1.4, Theorem 2.1.5]{HA} \cite[Theorem 1.1.1]{HA1}. \begin{teorema}If $C=(C_1,\dots,C_k)$ is generic, then, if not empty, $\mathcal{M}_C$ is a nonsingular irreducible affine algebraic variety of dimension $$ d_{\bm\mu}:=(k-2)n ^2-\sum_{i,j}(\mu^i_j)^2+2 $$ where $\mu^i=(\mu^i_1,\mu^i_2,\dots)$. \label{irreducible}\end{teorema} \bigskip In \cite[Section 2.3]{HA} the authors defined a rational function $\mathbb{H}_{\bm\mu}(z,w)$ which conjecturally computes the polynomial $H_c(\mathcal{M}_C;q,t)$. It is defined as follows. Consider $k$ sets ${\bm x}_1,\dots,{\bm x}_k$ of infinitely many independent variables and let $\Lambda({\bm x}_1,\dots,{\bm x}_k)$ be the ring of functions separately symmetric in each of the set of variables. We then put $$ \Omega(z,w):=\sum_{\lambda\in\mathcal{P}}\mathcal{H}_\lambda(z,w)\prod_{i=1}^k\tilde{H}_\lambda({\bm x}_i;z^2,w^2) $$ where $\mathcal{P}$ is the set of all partitions, $\tilde{H}_\lambda({\bm x}_i;z,w)$ are the modified Macdonald polynomials and $$ \mathcal{H}_\lambda(z,w):=\prod_s\frac{1}{(z^{2a(s)+2}-w^{2l(s)})(z^{2a(s)}-w^{2l(s)+2})} $$ where $s$ runs over the boxes of the Young diagram of $\lambda$ with $a(s)$ and $l(s)$ its arm and leg length. Then $$ \mathbb{H}_{\bm\mu}(z,w):=(z^2-1)(1-w^2)\left\langle {\rm Log}\,\Omega(z,w),h_{\bm\mu}\right\rangle $$ where $h_{\bm \mu}:=h_{\mu^1}({\bm x}_1)\cdots h_{\mu^k}({\bm x}_k)\in\Lambda({\bm x}_1,\dots,{\bm x}_k)$ is the product of complete symmetric functions and ${\rm Log}$ is the plethystic logarithm. \bigskip It is proved in \cite{MellitD} that these rational functions $\mathbb{H}_{\bf \mu}(z,w)$ are polynomials in $z$, $w$ with integer coefficients as it was conjectured in \cite{HA}. \bigskip We have the following conjecture \cite[Conjecture 1.2.1]{HA}. \begin{conjecture} \label{conjecturemhsgln} The polynomial $H_c(\mathcal{M}_C;q,t)$ depends only on the type ${\bm\mu}$ of $C$ and not the choice of generic eigenvalues and \begin{equation}H_c(\mathcal{M}_C;q,t)=(t\sqrt{q})^{d_{\bm\mu}}\mathbb{H}_{\bm\mu}\left(\frac{1}{\sqrt{q}},-t\sqrt{q}\right). \label{conj}\end{equation} In particular, \begin{equation} PH_c(\mathcal{M}_C;q)=q^{d_{\bm \mu}/2}\mathbb{H}_{\bm \mu}(0,\sqrt{q}). \label{pure}\end{equation} \end{conjecture} \bigskip Notice that since we are working with the Riemann sphere, the sign in the second coordinate could have been omitted because in this case $\mathbb{H}_{\bm\mu}(z,w)$ is a function of $z^2$ and $w^2$ (which is not the case in higher genus). We decided to keep the formulas as there are in \cite{HA} for the convenience of the reader. \bigskip The Euler specialization $t\mapsto -1$ of Formula (\ref{conj}) is proved \cite[Theorem 1.2.3]{HA} by counting points of $\mathcal{M}_C$ over finite fields and using Theorem \ref{Katz}, more precisely we have the following result. \begin{teorema} $$ E(\mathcal{M}_C;q)=q^{d_{\bm\mu}/2}\mathbb{H}_{\bm\mu}\left(\frac{1}{\sqrt{q}},\sqrt{q}\right). $$ In particular the polynomial $E(\mathcal{M}_C;q)$ depends only on the type ${\bm\mu}$ of $C$ and not the choice of generic eigenvalues. \label{E-poly}\end{teorema} \bigskip Let us now write a more explicit formula when $n=2$. \begin{prop}Assume that $k\geq 1$ and ${\bm\mu}=((1^2),\dots,(1^2))$, then $$ \mathbb{H}_{\bm\mu}(z,w)=\frac{(w^2+1)^k}{(z^2-w^2)(1-w^4)}+\frac{(z^2+1)^k}{(z^4-1)(z^2-w^2)}-\frac{2^{k-1}}{(z^2-1)(1-w^2)}$$ \label{H}\end{prop} \begin{remark} As mentioned earlier, we know from \cite{MellitD} that these rational functions are in fact polynomials in $z$ and $w$ with integer coefficients which a priori is not obvious. \end{remark} \bigskip \begin{proof}[Proof of Proposition \ref{H}] The conjugacy classes of $\Gl_n(\mathbb{F}_q)$ are parametrised by the set of maps from the set of Frobenius-orbits of $\overline{\mathbb{F}}_q^*$ to the set of all partitions. The type of a such a map consists of the collection of pairs $(d,\lambda)$ with $d$ a positive integer and $\lambda$ a non-zero partition which is the image of a Frobenius-orbit of size $d$. There are 4 types when $n=2$, i.e. $(1,1)^2,(2,1), (1,(1^2)), (1,(2^1))$. The first one is the type of a conjugacy class with two distincts eigenvalues in $\mathbb{F}_q^*$, the second one is the type of a conjugacy class whose characteristic polynomial is irreducible over $\mathbb{F}_q$ (i.e. it has two distinct eigenvalues $x, x^q$ in $\mathbb{F}_{q^2}^*$), and the last two types are the types respectively of central matrices and matrices with one Jordan block. Then from \cite[Formula (2.3.9)]{HA} we have $$ \mathbb{H}_{\bm \mu}(z,w)=(z^2-1)(1-w^2)\left\langle \sum_{\omega}C_\omega^o\mathcal{H}_\omega(z,w)\prod_{i=1}^k\tilde{H}_\omega({\bm x}_i;z^2,w^2),h_{\bm \mu}\right\rangle $$ where the sum runs over the types of size $2$. A direct calculation shows that $$ C_{(2^1)}^o=C_{(1^2)}^o=1,\hspace{1cm}C_{(2,1)}^o=C_{(1,1)^2}^o=-\frac{1}{2}$$ and \begin{align*} \mathcal{H}_{(1^2)}(z,w)&=\frac{1}{(z^2-w^2)(1-w^4)(z^2-1)(1-w^2)}\\ \mathcal{H}_{(2^1)}(z,w)&=\frac{1}{(z^4-1)(z^2-w^2)(z^2-1)(1-w^2)}\\ \mathcal{H}_{(1,1)^2}(z,w)&=\frac{1}{(z^2-1)^2(1-w^2)^2}\\ \mathcal{H}_{(2,1)}(z,w)&=\frac{1}{(z^4-1)(1-w^4)} \end{align*} To compute the pairing we need also to decompose the modified Macdonald symmetric functions into the basis of monomial symmetric functions $\{m_\lambda({\bm x})\}_\lambda$ which is orthogonal to the basis of complete symmetric functions $\{h_\lambda({\bm x})\}_\lambda$ : \begin{align*} \tilde{H}_{(1^2)}({\bm x};z,w)&=(w+1)m_{(1^2)}({\bm x})+m_{(2^1)}({\bm x})\\ \tilde{H}_{(2^1)}({\bm x};z,w)&=(z+1)m_{(1^2)}({\bm x})+m_{(2^1)}({\bm x})\\ \tilde{H}_{(1,1)^2}({\bm x};z,w)&=m_{(2^1)}({\bm x})+2m_{(1^2)}({\bm x})\\ \tilde{H}_{(2,1)}({\bm x};z,w)&=m_{(2^1)}({\bm x}). \end{align*} \end{proof} Hence in the case $n=2$, Conjecture \ref{conj} reads. \begin{conjecture}For $k\geq 1$ and ${\bm\mu}=((1^2),\dots,(1^2))$ we have $$ H_c(\mathcal{M}_C;q,t)=\frac{(q^2t^4+qt^2)^k}{q^2t^6(q^2t^2-1)(q^2t^4-1)}+\frac{t^{2k-6}(q+1)^k}{(q^2-1)(q^2t^2-1)}-\frac{(2qt^2)^{k-1}}{qt^4(q-1)(qt^2-1)}. $$ \end{conjecture} \bigskip \subsection{Twisted mixed Poincar\'e polynomials}\label{twisted} Assume that $X$ is a $K$-variety endowed with an action of a finite group $W$. If $K=\overline{\F}_q$, we assume that the action of $W$ commutes with $F$. The group action preserves the weight filtration on $H_c^\bullet(X)$ and for $w\in W$, we define the $w$-twisted mixed Poincar\'e polynomial as $$ H_c^w(X;q,t):=\sum_{i,k}{\rm Tr}\left(w\,|\, W^k_i/W^k_{i-1}\right)\, q^{i/2}t^k. $$ If $K=\overline{\F}_q,$ we say that the pair $(X/_{\overline{\F}_q}, W)$ has polynomial count with (twisted) counting polynomials $\{P_w(t)\}_{w\in W}$ in $\mathbb{Z}[t]$ if $$ |X^{wF^r}|=P_w(q^r). $$ for any integer $r \geq 1$. We have the following twisted analogue of Theorem \ref{countingfq}. \begin{teorema} \label{theoremtwistedEFq} If $(X/_{\overline{\F}_q},W)$ has polynomial count with (twisted) counting polynomials $\{P_w(t)\}_{w\in W}$ for any $w\in W$ we have \begin{equation} \label{katztwistedformula} E^w(X;q)=P_w(q). \end{equation} where $E^{w}(X;q)\coloneqq H^w_c(X;q,-1)$. \end{teorema} The proof of this Theorem is very similar to that of \cite[Theorem 2.8]{LRV}. We give it below after Theorem \ref{theoremtwistedE}. \vspace{4 pt} \begin{oss} Given $(X/_{\overline{\F}_q},W)$ as above, for any $w \in W$, the map $wF:X \to X$ is a Frobenius morphism and gives thus another $\F_q$-structure of the $\overline{\F}_q$-variety $X$. In particular, there exists an $\F_q$-scheme $X_w$ with an isomorphism $X_w \times_{\F_q} \overline{\F}_q \cong X$ such that, through this isomorphism, the geometric Frobenius $F_w$ of $X_w$ is identified with $wF$. In general, the polynomial $P_w(t)$ is not the counting polynomial of $X_w$, i.e. it is not true that $|X_w(\F_{q^r})|=P_w(q^r)$. Notice indeed that the LHS of the latter equality is $|X^{(wF)^r}|$, while the RHS is $|X^{wF^r}|$. \vspace{2 pt} For a concrete example, consider , $X=\overline{\F}_q^*$ with $F(z)=z^q$ and $W=\bm \mu_2=\{1,\sigma\}$, with the action $\sigma \cdot x=x^{-1}$. The pair $(\overline{\F}_q^*,\bm \mu_2)$ has polynomial count with counting polynomials $P_1(t)=(t-1)$ and $P_{\sigma}(t)=(t+1)$. Indeed, we have $\sigma F(x)=x^{-q}$ and thus $X^{\sigma F}=\bm \mu_{q+1}$. Notice that, if $4$ does not divide $q+1$, i.e. if $-1$ is not a square in $\F_q^*$, we can consider $X_{\sigma}=\spec(\F_q[s,t]/(s^2+t^2=1))$, with the isomorphism $X_{\sigma} \times_{\F_q}\overline{\F}_q \to \overline{\F}_q^*$ given by $(s,t) \to s+it$. We have that $$\#X_{\sigma}(\F_{q^r})=\begin{cases} q^r+1 \text{ if } r \text{ is odd}\\ q^r-1 \text{ if } r \text{ is even} \end{cases} $$ \end{oss} \bigskip If $K=\C$, we say that $(X/_{\C},W)$ has polynomial count with (twisted) counting polynomials $\{P_w(t)\}_{w\in W}$ if there exists a finitely generated $\mathbb{Z}$-subalgebra $R$ of $\C$, a separated $R$-scheme $X_R$ equipped with a $W$-action which gives back $X$ with its $W$-action after scalar extensions from $R$ to $\C$, such that for any ring homomorphism $\varphi: R \to \mathbb{F}_q$, $(X^{\phi},W)$ has polynomial counting with (twisted) counting polynomials $\{P_{w}(t)\}_{w \in W}$. \vspace{2 pt} \bigskip We have the following twisted version of Theorem \ref{Katz}. \begin{teorema} \label{theoremtwistedE} Assume that $(X/_{\C},W)$ has polynomial count with (twisted) counting polynomials $\{P_w(t)\}_{w\in W}$. Then for any $w\in W$ we have \begin{equation} \label{katztwistedformulaC} E^w(X;q)=P_w(q). \end{equation} \end{teorema} \vspace{2 pt} Theorem \ref{theoremtwistedE} above can be deduced from Theorem \ref{theoremtwistedEFq} as follows. Consider a variety $X/_{\C}$ and $R$ as above and $U \subseteq \spec(R)$ as in \cref{polynomialcount-paragraph}. Since the action of $W$ is defined over $R$, the isomorphism (\ref{isomorphism-cohomology-constant}) is compatible with the $W$-action. In particular, since the action of $W$ on $H^*_c(X)$ is defined over the rationals, through the isomorphism (\ref{comparisonisom}), we have $$\tr\left(\,w\,|\, W_{i}^kH^k_c(X)/W^k_{i-1}H^k_c(X)\right)=\tr\left(w\,|\, W_{i}^kH^k_c(X^{\phi})/W^k_{i-1}H^k_c(X^{\phi})\right)$$ and so $$H^{w}_c(X;q,t)=H^{w}_c(X^{\phi};q,t) $$ from which we get $$E(X;q)=E(X^{\phi};q) .$$ \bigskip \begin{proof}[Proof of Theorem \ref{theoremtwistedEFq}] \label{prooftwistedEFq} By Grothendieck's trace formula, for any $r$, we have \begin{equation} \label{grothendiecktrace} P_w(q^r)=|X^{wF^r}|=\sum_{k}(-1)^k \tr\left(wF^r\,|\,H^k_c(X,\overline{\Q}_{\ell})\right) .\end{equation} Let $\lambda_{i,k,1}q^{\frac{i}{2}},\dots,\lambda_{i,k,s_{k,i}}q^{\frac{i}{2}}$ and $\alpha^w_{i,k,1},\dots,\alpha^w_{i,k,s_{k,i}}$ be the eigenvalues, counted with multiplicities, of $F$ and $w$ on $W^k_i/W^k_{i-1}$. Since $w$ and $F$ commute, up to reordering, we can assume that, for any $r \geq 1$, $$\tr\left(wF^r\,|\,W^k_{i}/W^k_{i-1}\right)=\sum_{h=1}^{s_{k,i}}\alpha^w_{i,k,h}\lambda^r_{i,k,h}q^{\frac{ri}{2}}$$ and thus $$\sum_{k}(-1)^k \tr\left(wF^r\,|\,H^k_c(X)\right)=\sum_{i}\left(\sum_k (-1)^k \sum_{h=1}^{s_{k,i}}\alpha^w_{i,k,h}\lambda^r_{i,k,h}\right)q^{\frac{ir}{2}} .$$ If $\displaystyle P_w(t)=\sum_{i}t^i c_{w,i}$, from Formula (\ref{grothendiecktrace}) we deduce that \begin{equation} \sum_k (-1)^k \sum_{h=1}^{s_{k,i}}\alpha^w_{i,k,h}\lambda^r_{i,k,h}=\begin{cases} c_{w,\frac{i}{2}} \text{ if } i \text{ is even }\\ 0 \text{ otherwise} \end{cases}. \end{equation} From \cite[Lemma 2.9]{LRV}, we deduce that we have $$\sum_k (-1)^k \sum_{h=1}^{s_{k,i}}\alpha^w_{i,k,h}=\begin{cases} c_{w,\frac{i}{2}} \text{ if } i \text{ is even }\\ 0 \text{ otherwise} \end{cases}.$$ Since $\displaystyle \sum_k (-1)^k \sum_{h=1}^{s_{k,i}}\alpha^w_{i,k,h}=\sum_k (-1)^k \tr(w|W^{k}_{i}/W^k_{i-1})$, we have the desired equality (\ref{katztwistedformula}). \end{proof} We keep the notation of the previous section with $n=2$. We will apply the above theorem to the following situation. We assume that $C=(C_1,\dots,C_k)$ is a generic $k$-tuple of semisimple regular conjugacy classes of $\Gl_2$ where the $m$-th first conjugacy classes $C_1,\dots,C_m$ (for some integer $m<k$) are the conjugacy class with eigenvalues $1$ and $-1$ and we let $H_m$ be the kernel of the morphism $$ \epsilon_m:({\bm\mu}_2)^m\longrightarrow \C^*,\hspace{1cm}(y_1,\dots,y_m)\mapsto y_1\cdots y_m $$ where ${\bm\mu}_2$ is the multiplicative group $\{1,-1\}$. \bigskip The group $H_m$ acts on the character variety $\mathcal{M}_C$ by multiplication on the first $m$-th coordinates. \bigskip Put $\bm\mu=((1^2),\dots,(1^2))$ and fix $y=(y_1,\dots,y_m)\in H_m$. For each $i=1,\dots,m$, define $$ h^{y_i}_{(1^2)}({\bm x}_i)=\begin{cases}h_{(1^2)}({\bm x}_i)&\text{ if }y_i=1,\\ p_{2}({\bm x}_i)& \text{ otherwise}\end{cases} $$ where $p_2({\bm x}_i)=x_{i1}^2+x_{i2}^2+\cdots$ is the power sum symmetric function in the variable ${\bm x}_i=\{x_{i1},x_{i2},\dots\}$ and put $$ \mathbb{H}^y_{\bm\mu}(z,w):=(z^2-1)(1-w^2)\left\langle{\rm Log}\,\Omega(z,w),h^y_{\bm\mu}\right\rangle $$ where $$ h^y_{\bm\mu}:=h^{y_1}_{(1^2)}({\bm x}_1)\cdots h^{y_m}_{(1^2)}({\bm x}_m)h_{(1^2)}({\bm x}_{m+1})\cdots h_{(1^2)}({\bm x}_k)\in\Lambda({\bm x}_1,\dots,{\bm x}_k). $$ \bigskip \begin{prop}Let $r=r(y):=\#\{i=1,\dots,m\,|\, y_i=-1\}>0$, then $$ \mathbb{H}^y_{\bm\mu}(z,w)=\frac{(w^2+1)^{k-r}(1-w^2)^r}{(z^2-w^2)(1-w^4)}+\frac{(1-z^2)^r(z^2+1)^{k-r}}{(z^4-1)(z^2-w^2)}. $$ \label{H-twisted}\end{prop} \bigskip \begin{proof}Similar calculation as for the proof of Proposition \ref{H}, noticing that $$ \langle \tilde{H}_{(1^2)}({\bm x};z,w),p_2({\bm x})\rangle=\langle \tilde{H}_{(2,1)}({\bm x};z,w),h_{(1^2)}({\bm x})\rangle=0 $$ which explains why we have only two terms. \end{proof} We make the following conjecture. \begin{conjecture}We have $$ H_c^y(\mathcal{M}_C;q,t)=(t\sqrt{q})^{d_{\bm\mu}}\mathbb{H}_{\bm \mu}^y\left(\frac{1}{\sqrt{q}},-t\sqrt{q}\right). $$ \label{conj-twisted}\end{conjecture} We prove that the above conjecture is true under the Euler specialization $t\mapsto -1$. \begin{teorema} We have $$ E^y(\mathcal{M}_C;q)=q^{d_{\bm\mu}/2}\mathbb{H}^y_{\bm \mu}\left(\frac{1}{\sqrt{q}},\sqrt{q}\right). $$ In particular, the polynomial $E^y(\mathcal{M}_C;q)$ depends only on the type ${\bm\mu}$ of $C$ and not the choice of generic eigenvalues. \label{Ey}\end{teorema} \begin{proof} From Formula \ref{comparisonmhp}, it is enough to show the case of $K=\overline{\F}_q$. Take thus a finite field $\mathbb{F}_q$ of characteristic $\neq 2$ and consider a generic $k$-tuple $ C=( C_1,\dots, C_k)$ of semisimple regular conjugacy classes of $\Gl_2(\overline{\mathbb{F}}_q)$ with eigenvalues in $\mathbb{F}_q^*$ and such that the first $m$ conjugacy classes are the conjugacy class with eigenvalues $1,-1$. \bigskip Then the group $H_m$ acts on $\mathcal{M}_{ C}$ by multiplication coordinate by coordinate on the first $m$-th coordinates. By Theorem \ref{katztwistedformula} we need to show that $$ \# (\mathcal{M}_{C})^{yF}=q^{d_{\bm\mu}/2}\mathbb{H}^y_{\bm \mu}\left(\frac{1}{\sqrt{q}},\sqrt{q}\right) $$ where $F$ is the standard Frobenius that raises matrix coefficients to their $q$-th power. Fix $i=1,\dots,m$, let $\alpha_i\in\overline{\mathbb{F}}_q^*$ be defined as $\alpha_i=1$ if $y_i=1$ and as an element of $\mathbb{F}_{q^2}^*$ such that $\alpha_i^q=-\alpha_i$ if $y_i=-1$. We then let ${O}_i$ be the conjugacy class of $\Gl_2(\overline{\mathbb{F}}_q)$ with eigenvalues $\alpha_i,-\alpha_i$ (it is the conjugacy class ${ C}_i$ when $y_i=1$). Notice that the standard Frobenius $F$ leaves stable the conjugacy classes ${O}_i$ and that we have the following commutative diagram $$ \xymatrix{{C}_i\ar[d]_{y_iF}\ar[rr]^{f_i}&&{ O}_i\ar[d]^F\\ { C}_i\ar[rr]^{f_i}&&{ O}_i} $$ where $f_i$ is the multiplication by the scalar $\alpha_i$. Therefore via $\prod_i f_i$, the pair $(\mathcal{M}_{ C},yF)$ can be identified with $(\mathcal{M}_{ O},F)$ where ${ O}$ is the $k+1$-tuple of conjugacy classes $$ {O}=\left({ O}_1,\dots,{O}_k, \{(\alpha_1\cdots\alpha_m)^{-1} I_2\}\right)$$ where ${ O}_i={ C}_i$ for $i>m$. Notice that $\alpha_1\cdots\alpha_m\in\mathbb{F}_q^*$ as there is an even number of non-trivial $y_i$. We thus have $$ \#(\mathcal{M}_{ C})^{yF}=\#(\mathcal{M}_{ O})^F. $$ Now for an $F$-stable conjugacy class ${ O}$ of $\Gl_2(\overline{\mathbb{F}}_q)$ denote by $1_{ O}:\Gl_2(\mathbb{F}_q)\rightarrow \C^*$ the function that takes the value $1$ at ${O}^F$ and $0$ elsewhere. Then $$ \#(\mathcal{M}_{ O})^F=\frac{1}{|\PGl_n(\mathbb{F}_q)|}\left(1_{{ O}_1}*\cdots*1_{{ O}_k}*1_{(\alpha_1\cdots\alpha_k)^{-1} I_2}\right)(1) $$ It follows from \cite[Theorem 4.14]{L} that the RHS equals to $$ q^{d_{\bm\mu}/2}\mathbb{H}^y_{\bm \mu}\left(\frac{1}{\sqrt{q}},\sqrt{q}\right). $$ Indeed, since the conjugacy classes ${ O}_i$ are semisimple, the function ${\bm X}_{\overline{ O}_i}$ in \cite[Theorem 4.14]{L} is our function $1_{{ O}_i}$. In loc. cit. we introduced the notion of type of a conjugacy class of $\Gl_n(\mathbb{F}_q)$ (see \cite[\S 4.1.2]{L}) which we could avoid here as we are working with semisimple conjugacy classes of $\Gl_2$. The type of a split semisimple regular conjugacy class of $\Gl_2(\mathbb{F}_q)$ is $\omega=(1,1)(1,1)$ and the symmetric function $s_\omega$ (see \cite[Remark 4.1]{L}) is our symmetric function $h_{(1^2)}$. The type of a non-split semisimple regular conjugacy class of $\Gl_2(\mathbb{F}_q)$ like our conjugacy class ${ O}_i$ with $y_i=-1$ (it has eigenvalues in $\mathbb{F}_{q^2}-\mathbb{F}_q$) is $\omega=(2,1)$ and $s_\omega$ is our symmetric function $p_2$. Finally in the formula for $\mathbb{H}_{\bm\omega}(z,w)$ in \cite[Above Lemma 4.2]{L}, there is a sign which in our case is always $1$ as we have an even number of twisted conjugacy classes. \end{proof} \section{$\mathrm{PGL}_2$-character stacks } In this section we work over an algebraically closed field $K$ of characteristic $\neq 2$ (for us $K$ will be either $\mathbb{C}$ or $\overline{\F}_q$). If $K=\overline{\F}_q$ we will work with $\ell$-adic sheaves and if $K=\mathbb{C}$ we can work either with $\ell$-adic sheaves or complex sheaves for the analytic topology. We thus let $\kappa$ be the field $\C$ or $\overline{\Q}_\ell$ depending on the context and use the same letter to denote the constant sheaf. \subsection{Local systems on semisimple conjugacy classes of $\mathrm{PGL}_2(K)$} \label{premilinariesconjclassesC} We denote by $$ p:\Gl_2(K) \to \PGl_2(K) $$ the canonical projection map. Let $T \subseteq \Gl_2(K)$ and $\mathcal{T}\subseteq\PGl_2(K)$ be the maximal tori of diagonal matrices. Identify $W_{\Gl_2}(T)=\Z/2\Z$ and $W_{\PGl_2}(\mathcal{T})=\Z/2\Z$ in the classical way, through permutation matrices. Notice that $p(W_{\Gl_2}(T))=W_{\PGl_2}(\mathcal{T})$. We now describe the irreducible $\PGl_2$-equivariant local systems on the regular semisimple conjugacy classes of $\PGl_2(K)$. \vspace{6 pt} For $x \in K^*\setminus{1}$, let $g_x$ be the matrix $$ g_x=\begin{pmatrix}1 &0 \\ 0 &x \end{pmatrix}. $$ The centraliser $C_{\Gl_2}(g_x)$ of $g_x$ in $\Gl_2(K)$ is $T$ and so in particular it is connected. Therefore the only irreducible $\Gl_2(K)$-equivariant local system on the $\Gl_2(K)$-conjugacy class $C_x$ of $g_x$ is the constant sheaf $\kappa$. \bigskip $\bullet$ If $x \neq -1$, then $C_{\PGl_2(K)}(p(g_x))=\mathcal{T}$ is connected and so the only irreducible $\PGl_2(K)$-equivariant local system on the $\PGl_2(K)$-conjugacy class $\mathcal{C}_x$ of $p(g_x)$ is the constant sheaf. Notice that $p$ restricts to an isomorphism $$ p:C_x \to \mathcal{C}_x. $$ $\bullet$ If $x=-1$, then $$ C_{\PGl_2(K)}(p(g_{-1}))=\mathcal{T} \rtimes W_{\PGl_2}(\mathcal{T})=\mathcal{T} \rtimes \Z/2\Z $$ has two connected components and so there are exactly two (up to isomorphism) irreducible $\PGl_2(K)$-equivariant local systems on the $\PGl_2(K)$-conjugacy class $\mathcal{C}_{-1}$ of $p(g_{-1})$. The non-trivial local system $\mathcal{L}_\epsilon$ is obtained from the $2$-covering $$ p:C_{-1} \to \mathcal{C}_{-1} $$ as $$ p_*(\kappa) \cong \kappa \oplus \mathcal{L}_\epsilon. $$ The corresponding non-trivial covering automorphism $\sigma: C_{-1} \to C_{-1}$ is given by $\sigma(x)=-x$. The local system $\mathcal{L}_\epsilon$ has thus the following explicit description. For an open $U \subseteq \mathcal{C}_{-1} $, we have \begin{equation} \label{explicitwritinglocalsystem} \mathcal{L}_\epsilon(U)=\{f \in \kappa(p^{-1}(U)) \ | \ f \circ \sigma=-f\}. \end{equation} We will call the conjugacy classes $\mathcal{C}_x$, with $x\neq -1$, \emph{non-degenerate} and $\mathcal{C}_{-1}$ \emph{degenerate}. \subsection{Geometry of $\mathrm{PGL}_2$-character stacks} \label{paragraphgeometry} Let $p^k:(\Gl_2)^k \to (\PGl_2)^k$ where $p:\Gl_2\rightarrow\PGl_2$ is the quotient map. Fix a $k$-tuple $\mathcal{C}=(\mathcal{C}_1,\dots,\mathcal{C}_k)$ of regular semisimple conjugacy classes of $\PGl_2(K)$ and consider a $k$-tuple $C=(C_1,\dots,C_k)$ of conjugacy classes of $\Gl_2(K)$ such that $p(C_i)=\mathcal{C}_i$ for each $i=1,\dots,k$. Fix a square root $$ \lambda_C=\sqrt{\prod_{i=1}^k \det(C_i)} $$ and consider the following affine algebraic varieties $$ X_{\mathcal{C}}\coloneqq \left\{(x_1,\dots,x_k) \in \mathcal{C}_1 \times \cdots \times \mathcal{C}_k \ | \ x_1 \cdots x_k=1\right\} $$ $$X^+_C\coloneqq \{(x_1,\dots,x_k) \in C_1 \times \cdots \times C_k \ | \ x_1 \cdots x_k=\lambda_C I_2\} $$ $$X^-_C\coloneqq \{(x_1,\dots,x_k) \in C_1 \times \cdots \times C_k \ | \ x_1 \cdots x_k=-\lambda_C I_2\} .$$ We have a decomposition $$(p^k)^{-1}(X_{\mathcal{C}})= X^+_{\mathcal{C}} \bigsqcup X^-_{\mathcal{C}} $$ Notice that $\PGl_2$ acts diagonally by conjugation on each of the above varieties. We consider the following GIT quotients $$ M^{\pm}_C\coloneqq X^{\pm}_C /\!/ \PGl_2$$ $$ M_{\mathcal{C}}\coloneqq X_{\mathcal{C}} /\!/ \PGl_2 $$ and the quotient stacks $$\mathcal{M}^{\pm}_C\coloneqq [X^{\pm}_C /\PGl_2]. $$ $$\mathcal{M}_{\mathcal{C}}\coloneqq [X_{\mathcal{C}} /\PGl_2] $$ We have the following Proposition. \begin{prop} \label{propdescriptiongeometry} (1) If the $\Gl_2$-conjugacy classes $C_1,\dots,C_k$ are all non-degenerate, we have isomorphisms: \begin{equation} \label{isom1} M_{\mathcal{C}} \cong M^+_C \bigsqcup M^-_C \end{equation} and \begin{equation} \label{isom2} \mathcal{M}_{\mathcal{C}}=\mathcal{M}^{+}_C\bigsqcup \mathcal{M}^{-}_C \end{equation} (2) Suppose that there are $m \geq 1$ degenerate conjugacy classes among $C_1,\dots,C_k$. We have isomorphisms \begin{equation}\label{isom3} M_C^+ \cong M_C^-\end{equation} \begin{equation}\label{isom4} \mathcal{M}_C^+ \cong \mathcal{M}^-_C. \end{equation} Moreover, denote by $p^+:\mathcal{M}_C^+ \to \mathcal{M}_{\mathcal{C}}$ the map obtained by restricting $p^k$ to $X_C^+$. The map $p^+$ is a Galois covering with Galois group $H_m={\rm Ker}(\epsilon_m)$ (see \S \ref{twisted}). \end{prop} \begin{proof} We have a Cartesian diagram \begin{center} \begin{tikzcd} X^+_C \bigsqcup X^-_C \arrow[r," "] \arrow[d," "] & C_1 \times \cdots \times C_k \arrow[d," p^k"] \\ X_{\mathcal{C}} \arrow[r," "] & \mathcal{C}_1 \times \cdots \times\mathcal{C}_k. \end{tikzcd} \end{center} \item From the description of the conjugacy classes given in \cref{premilinariesconjclassesC}, if each the conjugacy classes $\mathcal{C}_i$ are all non-degenerate, $p^k$ is an isomorphism and \begin{equation} X^+_{\mathcal{C}} \bigsqcup X^-_{\mathcal{C}} \cong X_{\mathcal{C}} \end{equation} Taking the quotient by $\PGl_2$, we obtain the isomorphisms (\ref{isom1}) and (\ref{isom2}). \item In the rest of proof, we assume that $\mathcal{C}_1,\dots,\mathcal{C}_m$ are the degenerate classes. From the description of the conjugacy classes given in \cref{premilinariesconjclassesC}, if there are $m$ degenerate conjugacy classes, we see that $p^k$ is a Galois covering with group $({\bm \mu}_2)^m$. The $({\bm\mu}_2)^m$-action is defined as follows. For $y=(y_1,\dots,y_m) \in ({\bm\mu}_2)^m$ and $(x_1,\dots,x_k) \in C_1 \times \cdots \times C_k$, we have $$y \cdot (x_1,\dots,x_k)=(y_1x_1,y_2x_2,\dots,y_mx_m,x_{m+1},\dots,x_k) .$$ By pullback, we see that the map $$X_C^+ \sqcup X_C^- \to X_{\mathcal{C}}$$ is a $({\bm\mu}_2)^m$-Galois covering too. Consider the element $\tau=(-1,1,\dots,1) \in ({\bm\mu}_2)^m$ and let $H_m'$ be the subgroup of $({\bm\mu}_2)^m$ generated by $\tau$. Notice that \begin{equation} \label{splittinggroup} ({\bm\mu}_2)^m \cong H_m \oplus H_m' \end{equation} The automorphism associated to $\tau$ restricts to an isomorphism $$\tau:X_C^+ \to X_C^-$$ $$(x_1,\dots,x_k) \to (-x_1,\dots,x_k) .$$ It is $\PGl_2$-equivariant from which we get the isomorphisms (\ref{isom3}) and (\ref{isom4}). \vspace{2 pt} From the splitting (\ref{splittinggroup}), we see that $$X_C^+ \to X_{\mathcal{C}}$$ is a Galois covering with Galois group $H_m$. Since the Galois covering structure is $\PGl_2$-equivariant, we deduce that the quotient map $p^+$ is also a Galois covering with Galois group $H_m$. \end{proof} \begin{remark} Consider the case in which $m=1$, i.e. there is exactly one degenerate class among $\mathcal{C}_1,\dots,\mathcal{C}_k$. From Proposition \ref{propdescriptiongeometry}, we see that we have an isomorphism $$\mathcal{M}^+_C \cong \mathcal{M}_{\mathcal{C}}.$$ \end{remark} Consider the $k+1$-tuples of conjugacy classes $$ C^+=(C_1,\dots,C_k,\lambda_C^{-1} I_2),\,\,\,\,C^-=(C_1,\dots,C_k,-\lambda_C^{-1} I_2) $$ of $\Gl_2$. \bigskip \begin{definizione} \label{definitiongenericity} The $k$-tuple $\mathcal{C}$ of conjugacy classes of $\PGl_2$ is said to be \emph{generic} if the $(k+1)$-tuples $C^+,C^-$ are both generic in the sense of \S \ref{GL2}. \end{definizione} \vspace{2 pt} \begin{oss} Notice that, if at least one of the conjugacy class $\mathcal{C}_1,\dots,\mathcal{C}_k$ is degenerate (i.e. if $m \geq 1$), then $C^+$ is generic if and only if $C^-$ is generic. \end{oss} \vspace{2 pt} \begin{oss} In \cite[Definition 9]{KNWG}, the authors give a definition of a generic $k$-tuple of conjugacy classes for any reductive group $G$. It is not hard to see that their definition agrees with our definition in the case of $\PGl_2$. \end{oss} From \cite[Theorem 2.1.5]{HA} and Proposition \ref{propdescriptiongeometry} we deduce the following Theorem. \begin{teorema} \label{genericity} (1) If $\mathcal{C}$ is generic and $\mathcal{C}_1,\dots,\mathcal{C}_k$ are non degenerate, the $\PGl_2$-character stack $\mathcal{M}_{\mathcal{C}}$ is a smooth algebraic variety (i.e. the canonical morphism $\mathcal{M}_{\mathcal{C}}\rightarrow M_{\mathcal{C}}$ is an isomorphism of stacks) of dimension $2k-6$. (2) If $\mathcal{C}$ is generic and there are $m$ degenerate classes among $\mathcal{C}_1,\dots,\mathcal{C}_k$ with $m \geq 1$, the $\PGl_2$-character stack $\mathcal{M}_{\mathcal{C}}$ is a smooth Deligne-Mumford stack of dimension $2k-6$. More precisely, we have an isomorphism $\mathcal{M}_{\mathcal{C}} \cong [\mathcal{M}_{C}^+/H_m]$. \end{teorema} \vspace{4 pt} \begin{oss} If there are $m$ degenerate classes between $\mathcal{C}_1,\dots,\mathcal{C}_k$ with $m \geq 1$, it is not always true that the $\PGl_2$-action on $X_{\mathcal{C}}$ is free, i.e. that the canonical morphism $\mathcal{M}_{\mathcal{C}} \to M_{\mathcal{C}}$ is an isomorphism. Consider for example the case of $k=5$, $m=4$ and where $\mathcal{C}_5$ is the $\PGl_2$-conjugacy class of $p(g_4)$. Pick $\lambda_{\mathcal{C}}=2$. It is not hard to see that the $5$-tuple $\mathcal{C}$ is generic. We have that $$(X_1,\dots,X_5)=\left(\begin{pmatrix}1 &0\\ 0 &-1 \end{pmatrix},\begin{pmatrix}0 &\frac{1}{2}\\ 2 &0 \end{pmatrix},\begin{pmatrix}-1 &0\\ 0 &1 \end{pmatrix},\begin{pmatrix}0 &1\\ 1 &0 \end{pmatrix},\begin{pmatrix}4 &0\\ 0 &1 \end{pmatrix}\right) \in X^+_C $$ and therefore $(p(X_1),\dots,p(X_5)) \in X_{\mathcal{C}}$. Notice that \begin{equation} g_{-1}\cdot (X_1,\dots,X_5)=(X_1,-X_2,X_3,-X_4,X_5) \end{equation} from which we deduce that $p(g_{-1}) \cdot (p(X_1),\dots,p(X_5))=(p(X_1),\dots,p(X_5))$, i.e. $p(g_{-1}) \in \Stab_{\PGl_2}((p(X_1),\dots,p(X_5)))$. \end{oss} \subsection{Local systems on $\mathrm{PGL}_2$-character stacks} \label{sectionlocalsystemscharacterstacks} We keep the notation of the previous section and we fix a non-negative integer $m<k$ and we assume that the first $m$ conjugacy classes of $\mathcal{C}_1,\dots,\mathcal{C}_k$ are degenerate and the last $k-m$ conjugacy classes are non-degenerate. From Proposition \ref{propdescriptiongeometry}, the map $p^+:\mathcal{M}^+_C \to \mathcal{M}_{\mathcal{C}}$ is a Galois covering with Galois group $H_m$. We have therefore a decomposition \begin{equation} \label{decompositionsplitting} (p^+)_*\kappa \cong \bigoplus_{\chi \in \widehat{H_m}} \mathcal{F}_{\chi}, \end{equation} where $\widehat{H_m}$ denotes the set of irreducible characters of $H_m$ and where each $\mathcal{F}_{\chi}$ is an irreducible local system. We have for any open subset $U$ of $\mathcal{M}_{\mathcal{C}}$ \begin{equation} \label{descriptionlocalsystemgalois} \mathcal{F}_{\chi}(U)=\{f \in \kappa((p^+)^{-1}(U)) \ | \ y^*(f)=\chi(h)f \ \ \text{ for each }y \in H_m\} \end{equation} where, for $y \in H_m$, we denote by $y:\mathcal{M}^+_C \to \mathcal{M}^+_C$ the corresponding covering automorphism. \bigskip Let us now describes the set of irreducible characters of $H_m$. For each subset $A \subseteq \{1,\dots,m\}$, define $\chi_A \in \widehat{({\bm\mu}_2)^m}$ as $$\chi_A:({\bm\mu}_2)^m \to \C^\times$$ $$(y_1,\dots,y_m) \mapsto \prod_{j \in A}y_j .$$ \begin{esempio} With the notation just introduced, we have that $\chi_{\emptyset}=1$ and $\chi_{\{1,\dots,m\}}=\epsilon_m$. \end{esempio} \vspace{4 pt} Notice that $\widehat{({\bm\mu}_2)^{m}}=\{\chi_A\}_{A \subseteq \{1,\dots,m\}}$ and we have the following Proposition. \begin{prop} The restriction map $$\Res_{H_m}:\widehat{({\bm\mu}_2)^{m}} \to \widehat{H_m} $$ is surjective and we have $\Res_{H_m}(\chi_A)=\Res_{H_m}(\chi_B)$ if and only if $\chi_A=\chi_B$ or $\chi_A=\epsilon_m \chi_B$, i.e. if and only if $A=B$ or $B=A^c$. \end{prop} For $B\subseteq\{2,\dots,m\}$ we denote $\Res_{H_m}(\chi_{B\cup\{1\}})$ by $\overline{\chi_B}$. We have therefore the following bijection $$\widehat{H_m} \longleftrightarrow \{\overline{\chi_B} \ | \ B \subseteq \{2,\dots,m\}\} .$$ \bigskip The decomposition of (\ref{decompositionsplitting}) can therefore be rewritten as \begin{equation} \label{decompositionsplitting1} (p^+)_*\kappa \cong \bigoplus_{B \subseteq \{2,\dots,m\}} \mathcal{F}_{\overline{\chi_B}}. \end{equation} We now give a concrete description of the local systems $\mathcal{F}_{\overline{\chi_B}}$. Consider the closed embedding $$i_{\mathcal{C}}:X_{\mathcal{C}} \to \mathcal{C}_1 \times \cdots \times\mathcal{C}_k. $$ \vspace{2 pt} For any $B \subseteq \{2,\dots,m\}$, put $$\mathcal{L}_B \coloneqq i_{\mathcal{C}}^*\left(\mathcal{E}_1\boxtimes\mathcal{E}_2\boxtimes\cdots\boxtimes\mathcal{E}_k \right)$$ where $$ \mathcal{E}_i=\begin{cases}\mathcal{L}_\epsilon&\text{ if }i\in \{1\}\cup B,\\\kappa&\text{ otherwise}.\end{cases} $$ Notice that $\mathcal{L}_B$ is $\PGl_2$-equivariant and defines therefore a local system $\mathcal{F}_B$ on $\mathcal{M}_{\mathcal{C}}$. From (\ref{explicitwritinglocalsystem}) and (\ref{descriptionlocalsystemgalois}), we see that for any $B \subseteq \{2,\dots,m\}$, we have an equality \begin{equation} \label{equalitylocalsytems} \mathcal{F}_B=\mathcal{F}_{\overline{\chi_B}}. \end{equation} The decomposition (\ref{decompositionsplitting1}) can therefore be rewritten as \begin{equation} \label{decompositionsplitting2} (p^+)_*\kappa \cong \bigoplus_{B \subseteq \{2,\dots,m\}} \mathcal{F}_B. \end{equation} \begin{remark}Notice that if $m=1$, then $B$ is necessarily the empty set and $$ \mathcal{L}_B=(i_{\mathcal{C}})^*(\mathcal{L}_\epsilon\boxtimes\kappa\boxtimes\cdots\boxtimes\kappa) $$ is the constant sheaf. \end{remark} \subsection{Cohomology of generic $\mathrm{PGL}_2$-character stacks}\label{cohomology} In this section, we give our main results concerning the cohomology of $\PGl_2$-character stacks. \vspace{2 pt} If $K=\overline{\F}_q$, we pick a generic $k$-tuple $\mathcal{C}$ of regular semisimple conjugacy classes of $\PGl_2(\overline{\F}_q)$ which are $F$-stable and split, i.e. each $\mathcal{C}_i$ is the conjugacy class of $p(g_{x_i})$ with $x_i \in \F_q^*$. Notice that, for such a $\mathcal{C}$, we can find a $k$-tuple $C$ of semisimple conjugacy classes of $\Gl_2(\overline{\F}_q)$ which are $F$-stable and split and such that $p(C_i)=\mathcal{C}_i$. For instance we can take $C_i$ to be the conjugacy class of $g_{x_i}$ for each $i$. In this case, we assume in what follows that $\lambda_{C} \in \F_q^*$, or, equivalently, that $$\prod_{i} \det(C_i) \in (\F_q^*)^2 ,$$ where $(\F_q^*)^2 \subseteq \F_q^*$ is the subgroup of squares. Notice that, under these assumptions, the constructions of \cref{paragraphgeometry} and \cref{sectionlocalsystemscharacterstacks} are all compatible with $F$. \begin{remark} If $\lambda_C \not\in \F_q^*$, then $|\mathcal{M}_{\mathcal{C}}^F|=0$, see \cref{eulerspecializationproof} for more details. \end{remark} \subsubsection{The non-degenerate case} Let us first discuss the simpler case where the conjugacy classes $\mathcal{C}_1,\dots,\mathcal{C}_k$ are all non-degenerate. From Proposition \ref{propdescriptiongeometry} and Theorem \ref{E-poly}, we deduce the following Theorem. \begin{teorema} \label{theoremEnondegenerate} If $\mathcal{C}$ is a generic $k$-tuple of non-degenerate regular semisimple conjugacy classes of $\PGl_2$, we have \begin{equation} E(\mathcal{M}_{\mathcal{C}};q)=2q^{k-3}\mathbb{H}_{\bm \mu}\left(\frac{1}{\sqrt{q}},\sqrt{q}\right) \end{equation} \end{teorema} \begin{remark}We know from Theorem \ref{irreducible} and Theorem \ref{E-poly} that the coefficient of the highest power of $q$ in $\mathbb{H}_{\bm \mu}\left(\frac{1}{\sqrt{q}},\sqrt{q}\right)$ equals $1$. Therefore the coefficient of the highest power of $q$ in $E(\mathcal{M}_{\mathcal{C}};q)$ equals $2$ which is also the number of connected components of the center of the dual group $\Sl_2$ of $\PGl_2$. This has been previously observed for an arbitrary connected reductive group \cite[Remark 3 (iii)]{KNWG}. \end{remark} The following conjecture is a consequence of the conjectural formula (\ref{conj}) and Proposition \ref{propdescriptiongeometry}. \begin{conjecture} \label{conjEnondegenerate} If $\mathcal{C}$ is a generic $k$-tuple of non-degenerate regular semisimple conjugacy classes of $\PGl_2$, we have \begin{equation} H_c(\mathcal{M}_{\mathcal{C}};q,t)=2(qt^2)^{k-3}\mathbb{H}_{\bm \mu}\left(\frac{1}{\sqrt{q}},-t\sqrt{q}\right) \end{equation} In particular, the pure part is given by the formula $$ PH_c(\mathcal{M}_\mathcal{C};q)=2q^{k-3}\mathbb{H}_{\bm \mu}(0,\sqrt{q}).$$ \end{conjecture} \bigskip \subsubsection{The degenerate case} Fix now $1\leq m<k$ and consider the case where the first $m$ conjugacy classes of $\mathcal{C}_1,\dots,\mathcal{C}_k$ are degenerate. \bigskip Taking cohomology in the decomposition (\ref{decompositionsplitting2}) we have the following isomorphism of $H_m$-representations: \begin{equation} \label{decompositioncohomologyirred} H_c^\bullet(\mathcal{M}^+_C) \cong \bigoplus_{B\subseteq\{2,\dots,m\}}H_c^\bullet(\mathcal{M}_{\mathcal{C}},\mathcal{F}_B) \otimes \overline{\chi_B}. \end{equation} i.e. $H^\bullet_c(\mathcal{M}_{\mathcal{C}},\mathcal{F}_B)$ is the subspace of $H^\bullet_c(\mathcal{M}_C^+)$ on which the group $H_m$ acts by the character $\overline{\chi_B}$. \bigskip The decomposition (\ref{decompositioncohomologyirred}) respects the weight filtrations on both sides and so we have an equality of twisted mixed Poincar\'e series (see \S \ref{twisted}), i.e. for any $y\in H_m$, \begin{equation} \label{directequalityequivariant} H_c^y(\mathcal{M}^+_C;q,t)=\sum_{B\subseteq\{2,\dots,m\}} H_c(\mathcal{M}_{\mathcal{C}},\mathcal{F}_B;q,t)\,\overline{\chi_B}(y), \end{equation} where for a local system $\mathcal{L}$ on $X$ we put $$ H_c(X,\mathcal{L};q,t):=\sum_{i,k}{\rm dim}(W^k_i/W^k_{i-1})q^{i/2}t^k $$ with $W^k_\bullet$ the weight filtration on $H^k_c(X,\mathcal{L})$. \vspace{2 pt} Given the orthogonality relations in the character ring of $H_m$, we can "invert" the isomorphism (\ref{decompositioncohomologyirred}) and obtain that for each $B\subseteq\{2,\dots,m\}$, we have \begin{equation} \label{invertedrelationcharacters} H_c(\mathcal{M}_{\mathcal{C}},\mathcal{F}_B;q,t)=\frac{1}{|H_m|}\sum_{y \in H_m}H^y_c(\mathcal{M}^+_C;q,t)\,\overline{\chi_B}(y) \end{equation} The equation (\ref{invertedrelationcharacters}) allows to express the cohomology of the $\PGl_2$-character stack $\mathcal{M}_{\mathcal{C}}$ with coefficient in the local system $\mathcal{F}_B$ just in terms of the cohomology of $\mathcal{M}^+_C$ and its $H_m$-action. \bigskip For any $m_1,m_2,r \in \N$, denote by $C_{m_1,m_2,r}$ the coefficient of $y^rx^{m_1+m_2-r}$ in the product $(x-y)^{m_1}(x+y)^{m_2}$, and define $$ \mathbb{A}_r(z,w):=\begin{cases}\frac{(w^2+1)^{k-r}(1-w^2)^r}{(z^2-w^2)(1-w^4)}+\frac{(1-z^2)^r(z^2+1)^{k-r}}{(z^4-1)(z^2-w^2)}&\text{ if }0<r\leq m,\\ &\\ \frac{(w^2+1)^k}{(z^2-w^2)(1-w^4)}+\frac{(z^2+1)^k}{(z^4-1)(z^2-w^2)}-\frac{2^{k-1}}{(z^2-1)(1-w^2)}&\text{ if }r=0. \end{cases} $$ \bigskip \begin{remark}Notice that $\mathbb{A}_0(z,w)=\mathbb{H}_{\bm \mu}(z,w)$ for ${\bm \mu}=((1^2),\dots,(1^2))$ (see Proposition \ref{H}) and for $r>0$, $\mathbb{A}_r(z,w)=\mathbb{H}_{\bm \mu}^y(z,w)$ with $y\in H_m$ such that $r(y)=r$ (see Proposition \ref{H-twisted}). \label{remA}\end{remark} We have the following results concerning the cohomology of the local systems $\mathcal{F}_B$. \begin{teorema} \label{Epolynomialgenericdegenerate} For any $B \subseteq \{2,\dots,m\}$, we have \begin{align*} E(\mathcal{M}_{\mathcal{C}},\mathcal{F}_B;q):&=H_c(\mathcal{M}_{\mathcal{C}},\mathcal{F}_ B;q,-1)\\&=\frac{q^{k-3}}{2^{m-1}}\sum_{\substack{r=0\\r \text{ even}}}^{m}C_{|B|+1,m-|B|-1,r}\,\,\mathbb{A}_r\left(\frac{1}{\sqrt{q}},\sqrt{q}\right). \end{align*} \end{teorema} \begin{proof} From Formula (\ref{invertedrelationcharacters}) and Theorem \ref{Ey} we have \begin{align*} E(\mathcal{M}_{\mathcal{C}},\mathcal{F}_B;q)&=\frac{1}{|H_m|}\sum_{y \in H_m}E^y(\mathcal{M}^+_C;q)\,\overline{\chi_B}(y)\\ &=\frac{q^{d_{\bm \mu}/2}}{|H_m|}\sum_{y\in H_m}\mathbb{H}^y_{{\bm \mu}}\left(\frac{1}{\sqrt{q}},\sqrt{q}\right) \,\overline{\chi_B}(y) \end{align*} First of all notice that $d_{\bm\mu}=2k-6$ when ${\bm\mu}=((1^2),\dots,(1^2))$ and from Proposition \ref{H-twisted} we see that $\mathbb{H}^y_{\bm\mu}(z,w)=\mathbb{A}_{r(y)}(z,w)$ depends only on the number $r(y)$ of $i=1,\dots,m$ such that $y_i=-1$ (notice that $r(y)$ must be even to have $y\in H_m$). Fix an even number $r$ and $y \in H_m$ such that $r(y)=r$. The value $\overline{\chi_B}(y)$ depends on the parity of $\#\{i\in B \cup \{1\}\,|\, y_i=-1\}$, it equals $1$ if this number is even and it equals $-1$ is this number is odd. For any even $r \in \{0,\dots,m\}$, we thus have \begin{align*} \sum_{\substack{y \in H_m\\ r(y)=r}}\overline{\chi_B}(y)\,\mathbb{H}^y_{\bm \mu}(z,w)&=\mathbb{A}_r(z,w)\sum_{\substack{y \in H_m\\ r(y)=r}}(-1)^{\#\{i\in B \cup \{1\}\,|\, y_i=-1\}}\\ &=\mathbb{A}_r(z,w)\sum_{\substack{A \subseteq \{1,\dots,m\}\\|A|=r}}(-1)^{|A \cap (B \cup \{1\})|} \\ &=C_{|B|+1,m-|B|-1,r}\, \mathbb{A}_r(z,w). \end{align*} \end{proof} The following conjecture is a consequence of Conjecture \ref{conj-twisted} and Formula (\ref{invertedrelationcharacters}). \begin{conjecture} \label{conjmhslocalsystems} For any $B \subseteq \{2,\dots,m\}$, we have \begin{equation} H_c(\mathcal{M}_\mathcal{C},\mathcal{F}_B;q,t)=\frac{(qt^2)^{k-3}}{2^{m-1}}\sum_{\substack{r=0\\r \text{ even}}}^{m}C_{|B|+1,m-|B|-1,r}\,\,\mathbb{A}_r\left(\frac{1}{\sqrt{q}},-t\sqrt{q}\right). \end{equation} In particular, the Poincar\'e polynomial of the pure part is given by \begin{equation} PH_c(\mathcal{M}_\mathcal{C},\mathcal{F}_B;q)=\frac{(qt^2)^{k-3}}{2^{m-1}}\sum_{\substack{r=0\\r \text{ even}}}^{m}C_{|B|+1,m-|B|-1,r}\,\,\mathbb{A}_r\left(0,\sqrt{q}\right). \label{pure-conj}\end{equation} \end{conjecture} \section{Review on Deligne-Lusztig theory and Lusztig's character-sheaves theory for ${\rm GL}_2$ and ${\rm SL}_2$} In this section we review the construction of the complex character table of $\Gl_2(\F_q)$, $\Sl_2(\F_q)$ as well as the construction of the character-sheaves by Lusztig \cite{Lusztig}\cite{CS2}\cite{mars}. \bigskip We assume that $q$ is odd. \bigskip On $G=\Gl_n(\overline{\F}_q),\,\Sl_n(\overline{\F}_q)$ we will only consider the standard Frobenius $F:G\rightarrow G$ that raises matrix coefficients to their $q$-th power so that the finite group $G^F$ of points fixed by $F$ is exactly matrices with coefficients in the finite field $\F_q$. \bigskip As we will work with $\ell$-adic sheaves, we need to choose a prime $\ell$ which does not divide $q$. \subsection{Deligne-Lusztig theory for $\mathrm{GL}_2$ and $\mathrm{SL}_2$}\label{table-SL} Consider a connected reductive algebraic group $G$ over $\overline{\F}_q$ with a geometric Frobenius $F:G\rightarrow G$, an $F$-stable maximal torus $T$ of $G$, a Borel subgroup $B$ containing $T$ and we denote by $U$ the unipotent radical of $B$. We have $B=T\ltimes U$. For $G=\Gl_n$ or $\Sl_n$, an example of such a pair would be the upper triangular matrices for $B$ and the diagonal matrices for $T$ in which case both $T$ and $B$ are $F$-stable if $F$ is the standard Frobenius, but we will need also $F$-stable maximal tori which are not contained in $F$-stable Borel subgroups. \bigskip Denote by $$ L:G\rightarrow G,\hspace{.5cm}g\mapsto g^{-1}F(g) $$ the Lang map (it is surjective by a theorem of Steinberg). \bigskip The affine algebraic variety $L^{-1}(U)$ is equipped with an action of $G^F$ by left multiplication and with an action of $T^F$ by right multiplication (because $U$ is a normal subgroup of $B$). These actions induce actions on the compactly supported $\ell$-adic cohomology groups $H_c^i(L^{-1}(U),\overline{\Q}_{\ell})$. For a linear character $\theta$ of $T^F$ we define the Deligne-Lusztig virtual character $R_T^G(\theta)$ of $G^F$ $$ R_T^G(\theta)(g)=\sum_ i(-1)^i\tr\left(g,\, H_c^i(L^{-1}(U),\overline{\Q}_\ell)\otimes_{\overline{\Q}_\ell[T^F]}\theta\right) $$ for all $g\in G^F$. These virtual characters were defined by Deligne and Lusztig \cite{DL} (see also \cite{DM}). These are very important in the classification of the irreducible characters of $G^F$ (analogue of Jordan decomposition of irreducible characters by Lusztig). When $G=\Gl_n$, we can express all irreducible characters of $G^F$ as a linear combination of these Deligne-Lusztig characters (this is not the case for $\Sl_2$ as we will see below). \bigskip When the maximal torus $T$ is contained in some $F$-stable Borel subgroup $B$ (we say that $T$ is \emph{split}), then $R_T^G(\theta)$ has the following more concrete description (see \cite[below Definition 11.1]{DM}): Consider the canonical projection $\pi:B^F\rightarrow T^F$, then $$ R_T^G(\theta)={\rm Ind}_{B^F}^{G^F}(\pi^*(\theta)) $$ where ${\rm Ind}_{B^F}^{G^F}$ is the classical induction in representations of finite groups. If moreover $\theta$ is the trivial character then $R_T^G(\theta)$ is the character afforded by the $G^F$-module $\overline{\Q}_\ell[G^F/B^F]$ where $G^F$ acts by left multiplication on $G^F/B^F$. \bigskip We have the following result. \begin{prop}\cite[Proposition 13.22]{DM} Let $T$ be an $F$-stable maximal torus of $\Gl_n$ and denote by $T'$ the maximal torus $T\cap\Sl_n$ of $\Sl_n$. Let $\theta$ be a linear character of $T^F$, then $$ R_{T'}^{\Sl_n}(\theta|_{T'{^F}})=R_T^{\Gl_n}(\theta)|_{\Sl_n^F}. $$ \end{prop} When $G=\Gl_2$, we have two $G^F$-conjugacy classes of $F$-stable maximal tori whose representatives are chosen as follows $$ T_1=\left.\left\{\left(\begin{array}{cc}a&0\\0&b\end{array}\right)\right|\, a,b\in\overline{\F}_ q^*\right\},\hspace{0.5cm}T_\sigma= \left.\Biggl\{\dfrac{1}{\alpha^q-\alpha}\begin{pmatrix} a\alpha^q-b\alpha &-a+b\\ (a-b)\alpha^{q+1} &-a\alpha+b\alpha^q \end{pmatrix} \ \right| \ a,b \in \overline{\F}_q^* \Biggr\} . $$ where $\alpha$ is a fixed element of $\F_{q^2}$ which is not in $\F_ q$. Here $\sigma$ denotes the non-trivial element of the Weyl group $W=S_2$ of $\Gl_2$ with respect to $T_1$. Then $T_1^F\simeq \F_q^*\times\F_q^*$ and $$T_\sigma^F=\Biggl\{\dfrac{1}{\alpha^q-\alpha}\begin{pmatrix} a\alpha^q-b\alpha &-a+b\\ (a-b)\alpha^{q+1} &-a\alpha+b\alpha^q \end{pmatrix} \ | \ a \in \F_{q^2}^* \text{ and } b=a^q \Biggr\}\simeq\F_{q^2}^*. $$ Notice that the diagram \begin{equation} \xymatrix{T_1\ar[rr]^{\sigma F:=\sigma\circ F}\ar[d]^g&& T_1\ar[d]^g\\ T_\sigma\ar[rr]^F&&T_\sigma} \label{conj1}\end{equation} where the vertical arrows are the conjugation $t\mapsto gtg^{-1}$ by $g=\left(\begin{array}{cc}1&1\\\alpha&\alpha^q\end{array}\right)$, commutes since $g^{-1}F(g)=\sigma$. A matrix in $T_\sigma^F$ is thus geometrically conjugate to a diagonal matrix with eigenvalues of the form $x$ and $x^q$ with $x\in\F_{q^2}^*$. \bigskip When $G=\Sl_2$, we have also two $G^F$-conjugacy classes of $F$-stable maximal tori whose representatives can be chosen as $T'_w:=T_w\cap\Sl_n$ with $w=1,\sigma$. \bigskip The following table gives the values of Deligne-Lusztig characters $R_{T_w}^G(\theta)$ in the case of $G=\Gl_2$ (and by restriction we get also the values in the $G=\Sl_2$ case), see \cite[\S 15.9]{DM} \begin{scriptsize} \begin{equation} \label{tableGL} \begin{array}{|c|c|c|c|c|} \hline &&&&\\ &\left(\begin{array}{cc}a&0\\0&a\end{array}\right)&\left(\begin{array}{cc}a&0\\0&b\end{array}\right)& \left(\begin{array}{cc}x&0\\0& x^q\end{array}\right)&\left(\begin{array}{cc}a&1\\0&a\end{array}\right)\\ &a\in\F_q^*&a\neq b\in\F_q^*&x\neq x^q\in\F_{q^2}^*&a\in\F_q^*\\ \hline &&&&\\ R_{T_1}^{\Gl_2}(\alpha,\beta)&(q+1)\alpha(a)\beta(a)&\alpha(a)\beta(b)+\alpha(b)\beta(a)&0&\alpha(a)\beta(a) \\ \alpha,\beta\in\widehat{\F_q^*}&&&&\\ \hline &&&&\\ R_{T_\sigma}^{\Gl_2}(\omega)&(1-q)\omega(a)&0&\omega(x)+\omega(x^q)&\omega(a)\\ \omega\in\widehat{\F_{q^2}^*}&&&&\\ \hline \end{array} \end{equation} \end{scriptsize} \bigskip The character tables of $\Gl_2(\F_q)$ and $\Sl_2(\mathbb{F}_q)$ from Deligne-Lusztig theory can be found for instance in \cite[15.9]{DM}. \bigskip {\bf List of the irreducible characters of $\Gl_2(\F_q)$ :} \bigskip (1) If $\alpha\neq\beta\in\widehat{\F_q^*}$, then the Deligne-Lusztig characters $R_{T_1}^{\Gl_2}(\alpha,\beta)$ are irreducible (from Lusztig-Jordan decomposition they correspond to conjugacy classes of the second column). (2) If $\omega\neq\omega^q\in\widehat{\F_{q^2}^*}$, then $-R_{T_\sigma}^{\Gl_2}(\omega)$ is an irreducible character (it corresponds to conjugacy classes of the third column). (3) The irreducible characters of the form $\alpha\circ{\rm det}$ and $(\alpha\circ{\rm det})\otimes{\rm St}$ where $\alpha\in\widehat{\F_q^*}$ and where ${\rm St}$ (called the Steinberg character) is the non-trivial irreducible constituent of the character $R_{T_1}^{\Gl_2}({\rm Id})$. They decomposes into Deligne-Lusztig characters as $$ \alpha\circ{\rm det}=\frac{1}{2}\left(R_{T_1}^{\Gl_2}(\alpha\circ{\rm det})+R_{T_\sigma}^{\Gl_2}(\alpha\circ{\rm det})\right)$$ $$ (\alpha\circ{\rm det})\otimes{\rm St}=\frac{1}{2}\left(R_{T_1}^{\Gl_2}(\alpha\circ{\rm det})-R_{T_\sigma}^{\Gl_2}(\alpha\circ{\rm det})\right)$$ \bigskip {\bf List of the irreducible characters of $\Sl_2(\F_q)$ :} \bigskip First of all notice that $$ T'_1{^F}\simeq \F_q^*,\hspace{1cm}T'_\sigma{^F}\simeq \mu_{q+1}\subset\F_{q^2}^*. $$ (1) If $\alpha\in\widehat{\F_q^*}$ satisfies $\alpha^2\neq 1$, then $R_{T'_1}^{\Sl_2}(\alpha)$ is an irreducible character of $\Sl_2^F$. (2) If $\omega\in\widehat{\mu_{q+1}}$ satisfies $\omega^2\neq 1$, then $-R_{T'_\sigma}^{\Sl_2}(\omega)$ is irreducible. (3) We have the unipotent characters ${\rm Id}$ and ${\rm St}$ which are the restrictions of the unipotent characters of $\Gl_2(\F_q)$. (4) If $\alpha_o$ is the nontrivial square of $1$ in $\widehat{\F_q^*}$, then $R_{T'_1}^{\Sl_2}(\alpha_o)$ splits as a sum of two irreducible characters $\chi_{\alpha_o}^+$ and $\chi_{\alpha_o}^-$ of same degree $\frac{q+1}{2}$. (5) If $\omega_o$ is the non-trivial square of $1$ in $\widehat{\mu_{q+1}}$, then $-R_{T_\sigma'}^{\Sl_2}(\omega_o)$ splits as a sum of two irreducible characters $\chi_{\omega_o}^+$ and $\chi_{\omega_o}^-$ both of same degree $\frac{q-1}{2}$. \bigskip All irreducible characters of $\Sl_2(\F_q)$ in cases (1), (2) and (3) arise as the restriction of irreducible characters of $\Gl_2(\F_q)$ and so can be written as a linear combination of Deligne-Lusztig characters of $\Sl_2(\F_q)$. The irreducible characters $\chi_{\alpha_o}^{\pm}$ and $\chi_{\omega_o}^\pm$ are not restrictions of irreducible characters of $\Gl_2$ and can not be written as a linear combination of Deligne-Lusztig characters of $\Sl_2$. \bigskip Recall that we have an exact sequence $$ \xymatrix{1\ar[r]&\mu_{q+1}\ar[r]&\F_{q^2}^*\ar[r]^N&\F_q^*\ar[r]&1} $$ where $N:\F_{q^2}^*\rightarrow\F_q^*, x\mapsto x^{q+1}$ is the norm map. \bigskip This dualizes into an exact sequence $$ \xymatrix{1\ar[r]&\widehat{\F_q^*}\ar[r]^{N^*}&\widehat{\F_{q^2}^*}\ar[r]^-{\rm res}&\widehat{\mu_{q+1}}\ar[r]&1} $$ where ${\rm res}$ is the restriction. The embedding $N^*$ identifies $\widehat{\F_q^*}$ with the linear characters $\omega$ of $\F_{q^2}^*$ such that $\omega^q=\omega$. Therefore, any $\omega\in {\rm res}^{-1}(\omega_o)$ satisfies $\omega^q\neq \omega$. \bigskip We thus have the following lemma. \begin{lemma} The (non-irreducible) characters $R_{T'_1}^{\Sl_2}(\alpha_o)$ and $-R_{T'_\sigma}^{\Sl_2}(\omega_o)$ are the restrictions respectively of the irreducible characters $R_{T_1}^{\Gl_2}(1,\alpha_o)$ and $-R_{T_\sigma}^{\Gl_2}(\omega)$ (for $\omega\in{\rm res}^{-1}(\omega_o)$). \label{restrict}\end{lemma} We will see in the next sections that all the irreducible characters of $\Gl_2$ and those of $\Sl_2$ in cases (1), (2) and (3) have a geometric nature which is not the case of the irreducible characters of $\Sl_2$ in case (4). More precisely Lusztig has defined certain simple perverse sheaves (called \emph{character-sheaves}) on $\Sl_2(\overline{\F}_q)$ whose characteristic functions coincide with the irreducible characters of $\Sl_2(\F_q)$ in cases (1), (2) and (3) but differs from the irreducible characters in case (4) and (5). \subsection{Characteristic functions of orbital complexes} In what follows, $X$ is an algebraic variety over $\overline{\F}_q$ equipped with a geometric Frobenius $F:X\rightarrow X$ (or equivalently, an $\F_q$-scheme $X_o$ such that $X=X_o\times_{\F_q}\overline{\F}_q$), for more details see for instance \cite[Chapter 4]{DM}. Let $D^b_c(X)$ be the category of $\overline{\Q}_{\ell}$-complexes with constructible cohomology. An $F$-equivariant structure (or Weil structure) on $\mathcal{F}\in D^b_c(X)$ is an isomorphism $$\phi:F^*(\mathcal{F}) \to \mathcal{F} .$$ \begin{remark}If $\mathcal{F}$ is the pullback of a complex $\mathcal{F}_o$ on $X_o$, then it admits a canonical $F$-equivariant structure, see for instance \cite[Chapter 1]{Weil}. \end{remark} \bigskip We then say that $(\mathcal{F},\phi)$ is an $F$-\emph{equivariant complex} on $X$. Given two $F$-equivariant complexes $(\mathcal{F},\phi)$ and $(\mathcal{F}',\phi')$, the Frobenius $F$ acts on ${\rm Hom}(\mathcal{F},\mathcal{F}')$ as $$ f\mapsto \phi'\circ F^*(f)\circ\phi^{-1}. $$ We denote by $D_c^b(X;F)$ the category of $F$-equivariant complexes on $X$ with ${\rm Hom}(\mathcal{F},\mathcal{F'})^F$ as set of morphisms $(\mathcal{F},\phi)\rightarrow(\mathcal{F}',\phi')$. The characteristic function of $(\mathcal{F},\phi)\in D_c^b(X;F)$ is the function $\X_{\mathcal{F},\phi}:X^F \to \overline{\Q}_{\ell}$ defined as $$\X_{\mathcal{F},\phi}(x)\coloneqq \sum_{i \in \Z}(-1)^i \tr(\phi_x^i: \mathcal{H}^i_x(\mathcal{F}) \to \mathcal{H}^i_x(\mathcal{F})) .$$ The function $\X_{\mathcal{F},\phi}$ does depend on the choice of the isomorphism $\phi$. However, in all the cases of relevance for this article, we can make a canonical choice of the isomorphism $\phi$ and we will often drop it from the notation. In particular, if $X$ is an algebraic group, we will always assume that $\phi_e=Id$. \bigskip Assume that $G$ is a connected linear algebraic group over $\overline{\F}_q$ equipped with a geometric Frobenius $F:G\rightarrow G$. For an $F$-stable conjugacy class $C$ of $G$ together with an $F$-stable $G$-equivariant irreducible local system $\mathcal{E}$ on $C$, we denote by $IC^\bullet_{\overline{C},\mathcal{E}}$ the intersection cohomology complex on the Zariski closure $\overline{C}$ of $C$ with coefficients in $\mathcal{E}$. We also fix an $F$-equivariant structure $\phi:F^*(\mathcal{E})\simeq\mathcal{E}$ and we denote again by $\phi$ the induced $F$-equivariant structure on $IC^\bullet_{\overline{C},\mathcal{E}}$ (as $IC^\bullet_{\overline{C},\mathcal{E}}$ is irreducible, any two $F$-equivariant structure differ by a scalar). \begin{prop} \cite[Proposition 4.4.13]{Let} The set $\{{\bf X}_{IC^\bullet_{\overline{C},\mathcal{E}},\phi}\}$, where $(C,\mathcal{E})$ runs over the pairs as above, forms a basis of the space $\mathcal{C}(G^F)$ of class functions $G^F/G^F\rightarrow\overline{\Q}_\ell$. \end{prop} The above basis is a geometric counterpart of the basis of characteristic functions of conjugacy classes of $G^F$. When $G$ is reductive, the characteristic functions of the $F$-stable character-sheaves will be the geometric counter-part of the basis of irreducible characters of $G^F$. \subsection{Character-sheaves on tori} Let $T$ be a torus defined over $\F_q$, with geometric Frobenius $F:T \to T$. A \textit{Kummer local system} $\mathcal{E}$ is a $\overline{\Q}_{\ell}$-local system on $T$ such that $\mathcal{E}^{\otimes m}\simeq\overline{\Q}_\ell$ for some $m \in \N$ such that $(m,q)=1$. Notice that in particular every Kummer local system is of rank $1$ and thus simple. For any $F$-stable Kummer local system $\mathcal{E}$ on $T$, the characteristic function $\X_{\mathcal{E}}$ is a linear character of the finite group $T^F$ and any linear character of $T^F$ is obtained in this way, i.e. \begin{prop}\cite[Proposition 2.3.1]{mars} \label{bijectionkummer} The map $\mathcal{E} \to \X_{\mathcal{E}}$ is an isomorphism between the group of $F$-stable isomorphism classes of Kummer local systems on $T$ and the group $\widehat{T^F}$ of linear characters of $T^F$. \end{prop} The Kummer local systems are the character-sheaves for $T$. \begin{esempio} \label{examplekummer} Consider $T=\mathbb{G}_m$ with the Frobenius $F(x)=x^q$ for $x \in \mathbb{G}_m$. In this case, we have $T^F=\F_q^*$. Consider a linear character $\alpha:\F_q^* \to \C^*$ and let $n$ be the order of $\alpha$. In particular, $n$ divides $q-1$. Fix a surjection $q_n:\F_q^* \to \Z/n\Z$ (by sending a generator $\zeta$ of the cyclic group $\F_q^*$ to its subgroup of order $n$ generated by $\zeta^{\frac{q-1}{n}}$). Since $\alpha^n=1$, there exists a linear character $\mu:\Z/n\Z \to \C^*$ such that $\mu \circ q_n=\alpha$. Consider now the $\Z/n\Z$-Galois cover $f_n:\mathbb{G}_m \to \mathbb{G}_m$ given by $f_n(z)=z^n$. We have a splitting $$(f_n)_*(\overline{\Q}_{\ell})=\bigoplus_{\xi \in \widehat{\Z/n\Z}} \mathcal{E}_{\xi} .$$ Since $f_n$ commutes with $F$, the local systems $\mathcal{E}_{\xi}$ are defined over $\F_q$ and have thus a canonical $F$-equivariant structure, and we have $\X_{\mathcal{E}_{\mu}}=\alpha$. \end{esempio} \subsection{Grothendieck-Springer resolution and geometric induction} \label{sectionspringerresolution} For more details and references on this section, see \cite[\S 6.1, 6.2]{laumonletellier} and also \cite[\S 2.4, 2.7]{laumonletellier2}. \bigskip Consider a connected reductive algebraic group $G$ over $\overline{\F}_q$, a maximal torus $T \subseteq G$ and a Borel subgroup $B \supseteq T$. Let $\pi:B \to T$ be the canonical projection and $W=W_G(T)$ be the Weyl group of $G$ with respect to $T$. We denote by $\widetilde{G}$ the variety $$\widetilde{G}\coloneqq \{(xB,g)\in (G/B) \times G \ | \ x^{-1}gx \in B\} .$$ We denote by $p:\widetilde{G} \to G$ and $q:\widetilde{G} \to T$ the maps given by $p((xB,g))=g$ and $q((xB,g))=\pi(x^{-1}gx)$. For any $\mathcal{F} \in D^b_c(T)$, we define the geometric induction $\mathcal{I}_T^G(\mathcal{F}) \in D^b_c(G)$ as $$\mathcal{I}_T^G(\mathcal{F})\coloneqq p_!q^*(\mathcal{F})[{\rm dim}\,G-{\rm dim}\, T] . $$ By \cite{BY}, the functor $\mathcal{I}_T^G$ preserves perverse sheaves and induces thus a functor $\mathcal{I}_T^G:\mathcal{M}(T)\rightarrow\mathcal{M}(G)$ between categories of perverse sheaves. \bigskip Consider the morphism of correspondences \begin{equation} \xymatrix{&&\ar[lld]_-q\widetilde{G}\ar[rrd]^-p\ar[d]^{(q,p)}&&\\ T&&S:=T\times_{T/\!/W}G\ar[rr]^{{\rm pr}_2}\ar[ll]_{{\rm pr}_1}&&G} \label{cor}\end{equation} The morphism $(q,p)$ is small and so $(q,p)_!\overline{\Q}_\ell$ is the intersection cohomology complex $IC^\bullet_{S,\overline{\Q}_\ell}$ with coefficients in the constant sheaf $\overline{\Q}_\ell$. From the projection formula we deduce that $$ \mathcal{I}_T^G(\mathcal{F})={\rm pr}_{2\,!}\left({\rm pr}_1^*(\mathcal{F})\otimes IC^\bullet_{S,\overline{\Q}_\ell}\right). $$ Notice also that ${\rm pr}_1$ and ${\rm pr}_2$ are $W$-equivariant for the natural action of $W$ on $T$ and the trivial action of $W$ on $G$. Therefore, if $\mathcal{F}$ is equipped with a $W$-equivariant structure, namely if we are given a collection $\theta=(\theta_w)_{w\in W}$ of isomorphisms $$ \theta_w:w^*(\mathcal{F})\rightarrow\mathcal{F} $$ such that $\theta_{ww'}=\theta_w\circ w^*(\theta_{w'})$ and $\theta_1={\rm Id}$, then we get an action of $W$ on $\mathcal{I}_T^G(\mathcal{F})$, i.e. we have a group homomorphism $$ \theta^G:W\rightarrow{\rm Aut}(\mathcal{I}_T^G(\mathcal{F})). $$ \begin{remark}Notice that $W$ does not act on $\widetilde{G}$ so, to construct $\theta^G$, we need the factorization through $S$ (Stein factorization) together with the fact that $IC^\bullet_{S,\overline{\Q}_\ell}$ is naturally $W$-equivariant. \end{remark} Therefore if $\mathcal{F}$ is equipped with a $W$-equivariant structure $\theta$, then we have a decomposition \begin{equation} \mathcal{I}_T^G(\mathcal{F})=\bigoplus_{\chi\in\widehat{W}}\mathcal{I}_T^G(\mathcal{F})_\chi \label{W-decomp}\end{equation} where $\mathcal{I}_T^G(\mathcal{F})_\chi\rightarrow\mathcal{I}_T^G(\mathcal{F})$ is the kernel of the idempotent $1-e_\chi\in{\rm End}\left(\mathcal{I}_T^G(\mathcal{F})\right)$ with \begin{align*} e_\chi&=\frac{\chi(1)}{|W|}\sum_{w\in W}\overline{\chi(w)}\,\theta^G(w)\\&=\frac{\chi(1)}{|W|}\sum_{w\in W}\chi(w)\,\theta^G(w) \end{align*} (the last equality follows from the fact that the irreducible characters of Weyl groups have integer values). \bigskip Now we assume that $T$ is $F$-stable and that $F$ acts trivially on $W$ (this is will be our case since we will consider $\Gl_2$, $\Sl_2$ and $\PGl_2$). We do not assume that $B$ is $F$-stable. \bigskip The morphisms ${\rm pr}_1$ and ${\rm pr}_2$ are $F$-equivariant as well as the complex $IC^\bullet_{S,\overline{\Q}}$. Therefore the functor $\mathcal{I}_T^G$ induces functors $D_c^b(T;F)\rightarrow D_c^b(G;F)$ and $\mathcal{M}(T;F)\rightarrow\mathcal{M}(G;F)$ between $F$-equivariant objects. \bigskip Consider $\mathcal{F}\in D_c^b(T)$ equipped with both a $W$-equivariant structure $\theta$ and an $F$-equivariant structure $\phi:F^*(\mathcal{F})\simeq \mathcal{F}$ such that the two are compatible, namely the following diagram commutes for all $w\in W$, see \cite[Diagram (6.1)]{laumonletellier}: $$ \xymatrix{w^*F^*(\mathcal{F})\ar[rr]^{F^*(\theta_w)}\ar[d]_{w^*(\phi)}&&F^*(\mathcal{F})\ar[d]^{\phi}\\ w^*(\mathcal{F})\ar[rr]^{\theta_w}&&\mathcal{F}}. $$ Then we have the following compatibility $$ \xymatrix{F^*\left(\mathcal{I}_T^G(\mathcal{F})\right)\ar[rr]^{F^*(\theta^G(w)))}\ar[d]_{\phi^G}&&F^*\left(\mathcal{I}_T^G(\mathcal{F})\right)\ar[d]^{\phi^G}\\ \mathcal{I}_T^G(\mathcal{F})\ar[rr]^{\theta^G(w)}&&\mathcal{I}_T^G(\mathcal{F})} $$ For any $\chi\in\widehat{W}$, we see from this compatibility condition (see \cite[Remark 6.1.3]{laumonletellier} for more details) that the canonical $F$-equivariant structure $\phi^G:F^*\left(\mathcal{I}_T^G(\mathcal{F})\right)\simeq\mathcal{I}_T^G(\mathcal{F})$ restricts to an $F$-equivariant structure $\phi^G_\chi:F^*(\mathcal{I}_T^G(\mathcal{F})_\chi)\simeq\mathcal{I}_T^G(\mathcal{F})_\chi$ and $$ {\bf X}_{\mathcal{I}_T^G(\mathcal{F}),\phi^G}=\sum_{\chi\in\widehat{W}}{\bf X}_{\mathcal{I}_T^G(\mathcal{F})_\chi,\phi^G_\chi}. $$ We have also a formula for ${\bf X}_{\mathcal{I}_T^G(\mathcal{F}),\theta^G(w)\circ\phi^G}$ for any $w\in W$ which simplifies when the group $W$ is commutative (which will be our case), i.e. \begin{prop}Assume that $G=\Gl_2,\Sl_2$ or $\PGl_2$. Then $${\bf X}_{\mathcal{I}_T^G(\mathcal{F}),\theta^G(w)\circ\phi^G}=\sum_{\chi\in\widehat{W}}\chi(w)\,{\bf X}_{\mathcal{I}_T^G(\mathcal{F})_\chi,\phi^G_\chi} $$ for all $w\in W$. \label{Lu}\end{prop} \begin{proof} $\theta^G$ induces an action of $W$ on $\mathcal{I}_T^G(\mathcal{F})_\chi$ for all $\chi\in\widehat{W}$ which we also denote by $\theta^G$. Then $${\bf X}_{\mathcal{I}_T^G(\mathcal{F}),\theta^G(w)\circ\phi^G}=\sum_{\chi\in\widehat{W}}{\bf X}_{\mathcal{I}_T^G(\mathcal{F})_\chi,\theta^G(w)\circ\phi^G_\chi} $$ We thus need to see that \begin{equation} {\bf X}_{\mathcal{I}_T^G(\mathcal{F})_\chi,\theta^G(w)\circ \phi^G_\chi}=\chi(w)\,{\bf X}_{\mathcal{I}_T^G(\mathcal{F})_\chi,\phi^G_\chi}. \label{w}\end{equation} But this follows from the fact that for a representation $$ \rho:H\rightarrow \Gl(V) $$ of a commutative group $H$ in a finite-dimensional complex vector space $V$, then $H$ acts on $V_\chi={\rm Ker}(1-e_\chi)$, with $e_\chi=\frac{1}{|H|}\sum_{h\in H}\overline{\chi(h)}\rho(h)$, by the character $\chi$. \end{proof} \begin{remark}Note that Formula (\ref{w}) for $w=1$ is obviously false if $\chi$ is of degree $>1$. Hence the proof works for commutative $W$ only. For general $W$ we need to decompose further the complexes $\mathcal{I}_T^G(\mathcal{F})_\chi$ which may not be indecomposable. \end{remark} From the orthogonality relation on irreducible characters of $W$ we deduce. \begin{prop}Under the assumption of Proposition \ref{Lu} we have $$ {\bf X}_{\mathcal{I}_T^G(\mathcal{F})_\chi,\phi^G_\chi}=\frac{1}{|W|}\sum_{w\in W}\chi(w)\,{\bf X}_{\mathcal{I}_T^G(\mathcal{F}),\theta^G(w)\circ\phi^G}. $$ \label{inv}\end{prop} \subsection{Character-sheaves on $\mathrm{SL}_2$}\label{CS-SL} Let us start with some generalities: $G$ is an arbitrary connected reductive algebraic group with Frobenius $F$ and $T$ is a maximal torus of $G$. \begin{teorema}[Lusztig] Let $\mathcal{L}$ be a Kummer local system (character-sheaf up to a shift) on $T$, then $\mathcal{I}_T^G(\mathcal{L}[{\rm dim}\, T])$ is a semisimple perverse sheaf on $G$. \end{teorema} \begin{proof}See for instance \cite[\S 4]{shoji}. \end{proof} \bigskip If $\mathcal{L}$ is a Kummer local system on $T$, then the irreducible constituents of $\mathcal{I}_T^G(\mathcal{L}[{\rm dim}\, T])$ are examples of character-sheaves on $G$. When $G=\Gl_n$, we get all character-sheaves in this way. We will see below that this is not true for $\Sl_2$. \bigskip Assume that $T$ is $F$-stable. Since the morphism ${\rm pr}_2$ of Diagram (\ref{cor}) is $G$-equivariant, for any $\mathcal{F}\in D_c^b(T)$, the complex $\mathcal{I}_T^G(\mathcal{F})$ is $G$-equivariant and so $\mathcal{I}_T^G:D_c^b(T;F)\rightarrow D_c^b(G;F)$ induces a $\overline{\Q}_\ell$-linear map $$ I_T^G:\mathcal{C}(T^F)\rightarrow\mathcal{C}(G^F),\hspace{.5cm} I_T^G({\bf X}_{\mathcal{F},\phi})={\bf X}_{\mathcal{I}_T^G(\mathcal{F}),\phi^G}. $$ We have the following particular case of the main result of \cite{LUG}. \begin{teorema}[Lusztig] The geometric induction coincides with the Deligne-Lusztig induction, i.e. we have $$ I_T^G=R_T^G. $$ \label{GreenLu}\end{teorema} \bigskip Thanks to the above theorem, the characteristic functions of the $F$-stable character-sheaves are closely related to the irreducible characters of $G^F$. When $G=\Gl_n$, the characteristic functions of the $F$-stable character-sheaves are exactly (up to a sign) the irreducible characters of $\Gl_n(\F_q)$. This is not true in general (as we will see in details with $\Sl_2$). Notice that there is in fact no reason that the decomposition of $\mathcal{I}_T^G(\mathcal{L})$ as a direct sum of simple objects in the category of perverse sheaves would agree with the decomposition of the character $R_T^G({\bf X}_\mathcal{L})$ as a direct sum of irreducible characters. \bigskip We assume now that $G=\Sl_2$ with $W=S_2=\{1,\sigma\}$. The $F$-stable maximal torus $T$ is either the maximal torus of diagonal matrices $T_1$ or the twisted $F$-stable maximal torus $T_\sigma$ (see \S \ref{table-SL}). Recall that $\sigma\in W$ acts by $x\mapsto x^{-1}$ on $T\simeq \Gl_1$. \bigskip Fix a an $F$-stable Kummer local system $\mathcal{L}$ on $T$. \begin{remark}Notice that if $\mathcal{L}$ is $W$-equivariant on $T=T_1$, then $\mathcal{L}$ will be also $\sigma F$-stable. Since the pairs $(T_1,\sigma F)$ is $G$-conjugate to $(T_\sigma, F)$ (see Diagram (\ref{conj1})), we get an $F$-stable Kummer local system $\mathcal{L}_\sigma$ on $T_\sigma$ and we have $$ \mathcal{I}_T^G(\mathcal{L}[1])\simeq\mathcal{I}_{T_\sigma}^G(\mathcal{L}_\sigma[1]) $$ as perverse sheaves (not as $F$-equivariant perverse sheaves). To avoid redundancy in the list of character-sheaves we will thus consider only the torus $T_1$ in this situation. \label{Wequiv}\end{remark} \bigskip We therefore have the following cases : \bigskip (1) $T=T_1$ or $T_\sigma$ and $\mathcal{L}^{\otimes 2}\not\simeq \overline{\Q}_\ell$, then $\sigma^*(\mathcal{L})\not\simeq\mathcal{L}$. The perverse sheaf $\mathcal{I}_T^G(\mathcal{L}[{\rm dim}\, T])$ is irreducible and so is a character-sheaf. (2) $T=T_1$ and $\mathcal{L}=\overline{\Q}_\ell$, then $\mathcal{L}$ is naturally $W$-equivariant and the decomposition (\ref{W-decomp}) reads $$ \mathcal{I}_T^G(\overline{\Q}_\ell[1])\simeq\overline{\Q}_\ell[3]\oplus{\rm St}[3] $$ where ${\rm St}[3]$ is the character-sheaf corresponding to the sign character of $W$. (3) $T=T_1$ and $\mathcal{L}^{\otimes 2}\simeq\overline{\Q}_\ell$ with $\mathcal{L}\not\simeq\overline{\Q}_\ell$, then, by Example \ref{examplekummer}, $\mathcal{L}$ is the non-trivial constituent $\mathcal{A}_o$ of $$ (f_2)_*(\overline{\Q}_\ell)=\overline{\Q}_\ell\oplus\mathcal{A}_o $$ where $f_2:T_1\rightarrow T_1$, $z\mapsto z^2$. The Kummer local system $\mathcal{A}_o$ corresponds to the character $\alpha_o$ of $\F_q^*\simeq T_1^F$ (which take the value $1$ at squares and $-1$ at non-squares). Since $f_2$ is $W$-equivariant, the local system $\mathcal{A}_o$ must be $W$-equivariant and by (\ref{W-decomp}) we have $$ \mathcal{I}_T^G(\mathcal{A}_o[1])=\mathcal{X}_1[3]\oplus \mathcal{X}_{\epsilon}[3] $$ where $X_\epsilon$ is the constituent corresponding to the sign character of $W$. \bigskip Notice that we do not get all character-sheaves here, we are missing the two \emph{cuspidals} which are the two non-trivial $G$-equivariant irreducible local systems on the conjugacy classes of $$ \left(\begin{array}{cc}1&1\\0&1\end{array}\right),\hspace{1cm}\text{and}\hspace{1cm}\left(\begin{array}{cc}-1&1\\0&-1\end{array}\right) $$ In the first two cases (1) and (2), the characteristic functions of the character-sheaves coincide with the irreducible "characters" $R_T^G({\bf X}_{\mathcal{L}})$, ${\rm Id}$ and ${\rm St}$ of $\Sl_2(\F_q)$ (here by "character" we mean up to a sign, since $R_T^G(\theta)$ is the negative of a character when $T$ is the twisted torus). This is not true in the last case as we now see. \bigskip Denote by $X_1$ and $X_\epsilon$ the characteristic functions of $\mathcal{X}_1$ and $\mathcal{X}_\epsilon$. \begin{teorema} \begin{align*} X_1&=\frac{1}{2}\left(R_{T_1}^G(\alpha_o)+R_{T_\sigma}^G(\omega_o)\right)\\ X_\epsilon&=\frac{1}{2}\left(R_{T_1}^G(\alpha_o)-R_{T_\sigma}^G(\omega_o)\right) \end{align*} \label{unif}\end{teorema} \begin{proof}Proposition \ref{inv} says that \begin{align*} X_1&=\frac{1}{2}\left({\bf X}_{\mathcal{I}_{T_1}^G(\mathcal{A}_o),\phi^G}+{\bf X}_{\mathcal{I}_{T_1}^G(\mathcal{A}_o), \theta^G(\sigma)\circ\phi^G}\right)\\ X_\epsilon&=\frac{1}{2}\left({\bf X}_{\mathcal{I}_{T_1}^G(\mathcal{A}_o),\phi^G}-{\bf X}_{\mathcal{I}_{T_1}^G(\mathcal{A}_o), \theta^G(\sigma)\circ\phi^G}\right) \end{align*} By Theorem \ref{GreenLu} applied to $T=T_1$ we have $$ {\bf X}_{\mathcal{I}_{T_1}^G(\mathcal{A}_o),\phi^G}=R_{T_1}^G(\alpha_o). $$ As in Remark \ref{Wequiv}, the $F$-equivariant local system $\mathcal{A}_o$ (affording the character $\alpha_o$ on $T_1^F$) being naturally $W$-equivariant (with $W$-equivariant structure compatible with $F$-equivariant structure) is transported to an $F$-equivariant local system $\mathcal{A}_\sigma$ on $T_\sigma$ which corresponds to the non-trivial square $\omega_o$ in $\widehat{\mu_{q+1}}$ of the identity character. Therefore by Theorem \ref{GreenLu} applied to $T=T_\sigma$ we have \begin{align*} {\bf X}_{\mathcal{I}_{T_1}^G(\mathcal{A}_o),\theta^G(\sigma)\circ\phi^G}&={\bf X}_{\mathcal{I}_{T_\sigma}^G(\mathcal{A}_\sigma)}\\ &=R_{T_\sigma}^G(\omega_o) \end{align*} (the first equality being formal). \end{proof} Now notice that \begin{lemma} $$\omega_o(-1)=-\alpha_o(-1).$$ \label{alphaomega}\end{lemma} \begin{proof} This follows from the fact that $-1$ is a square in $\F_q^*$ if and only if it is a non-square in $\mu_{q+1}\subset\F_{q^2}^*$. \end{proof} From Theorem \ref{unif}, Lemma \ref{restrict} and the Table (\ref{tableGL}) we deduce the following table for the values of $X_1$ and $X_\epsilon$: \begin{scriptsize} \begin{equation} \label{tableSL} \begin{array}{|c|c|c|c|c|c|c|} \hline &&&&&&\\ &\left(\begin{array}{cc}1&0\\0&1\end{array}\right)&\left(\begin{array}{cc}-1&0\\0&-1\end{array}\right)&\left(\begin{array}{cc}a&0\\0&a^{-1}\end{array}\right)& \left(\begin{array}{cc}x&0\\0& x^q\end{array}\right)&\left(\begin{array}{cc}1&1\\0&1\end{array}\right)&\left(\begin{array}{cc}-1&1\\0&-1\end{array}\right)\\ &&&a\neq a^{-1}\in\F_q^\times&x^{q+1}=1, x\neq x^q&&\\ \hline &&&&&&\\ X_1&1&q\alpha_o(-1)&\alpha_o(a)&0&1&0 \\ &&&&&&\\ \hline &&&&&&\\ X_\epsilon&q&\alpha_o(-1)&0&\omega_o(x)&1&\alpha_o(-1)\\ &&&&&&\\ \hline \end{array} \end{equation} \end{scriptsize} We saw in \S \ref{table-SL}, that the irreducible characters $\chi_{\alpha_o}^\pm$ and $\chi_{\omega_o}^\pm$ have degrees $\frac{q+1}{2}$ and $\frac{q-1}{2}$ and so $X_1$ and $X_\epsilon$ can not be irreducible characters of $\Sl_2(\F_q)$. \section{Main results} \subsection{Statements}\label{statement} Let $K$ be an algebraically closed field of characteristic $\neq 2$ ($K=\mathbb{C}$ or $\overline{\F}_q$) and $\mathcal{C}=(\mathcal{C}_1,\dots,\mathcal{C}_k)$ be a $k$-tuple of regular semisimple conjugacy classes of $\PGl_2(K)$. When $K=\overline{\F}_q$, we assume that $\mathcal{C}$ respects the conditions explained at the beginning of \cref{cohomology}. We denote by $\mathcal{M}_C(K)$ the corresponding character stack. \bigskip {\bf The non-degenerate case} \bigskip We assume that all conjugacy classes $\mathcal{C}_i$ are non-degenerate (i.e. that their stabilisers are connected). \bigskip We fix a \emph{generic} $k$-tuple $R'=(R'_1,\dots,R'_k)$ of irreducible characters of $\Sl_2(\F_q)$ of the form $$ R'=\left(R_{T'_1}^{\Sl_2}(\gamma_1),\dots,R_{T'_1}^{\Sl_2}(\gamma_k)\right) $$ (see \S \ref{generic-char} for the definition of generic $R'$). \begin{teorema}We have (1) \begin{equation} E(\mathcal{M}_{\mathcal{C}}(\C);q)=E(\mathcal{M}_\mathcal{C}(\overline{\F}_q);q)=\left\langle1_{(\mathcal{C}_1)^F}*\cdots*1_{(\mathcal{C}_k)^F},1_{\{1\}}\right\rangle_{\PGl_2(\F_q)}.\label{Eseries-ndegen} \end{equation} (2) \begin{equation} \left\langle R'_1\otimes\cdots\otimes R'_k,1\right\rangle_{\Sl_2(\F_q)}=2\, \mathbb{A}_0(0,\sqrt{q}). \label{pure-part-ndegen}\end{equation} where $\mathbb{A}_r(z,w)$ is defined in \S\ref{cohomology}. \end{teorema} \bigskip We will see that these polynomials do not depend on the choice of the eigenvalues as long as this choice is generic and the conjugacy classes regular non-degenerate. \bigskip The proof of the first assertion is completely similar to the analogous statement for $\Gl_n(\C)$-character varieties (see \cite{HA}). Namely we first notice that the RHS counts the number of points of $\mathcal{M}_\mathcal{C}$ over $\F_q$ and we conclude by Theorem \ref{Katz}. \bigskip The second assertion will be a particular case of Theorem \ref{theoR'}(2) with $k_2=0$. \bigskip It follows from Conjecture \ref{conjEnondegenerate} and Remark \ref{remA} that \bigskip \begin{center} \begin{tikzcd} \label{diag3} & & & H_c(\mathcal{M}_\mathcal{C}(\C);q,t) \arrow[d,"t \mapsto -1"] \arrow[rd, "\text{ pure part}"]\\ & & & \langle 1_{(\mathcal{C}_1)^F}*\cdots*1_{(\mathcal{C}_k)^F},1_1 \rangle_{\PGl_2(\F_q)} & q^{k-3} \langle R'_1\otimes \cdots\otimes R'_k,1 \rangle_{\Sl_2(\F_q)} \end{tikzcd} \end{center} \bigskip {\bf The degenerate case} \bigskip We fix a non-negative integer $1\leq m<k$ and we assume that $\mathcal{C}_1=\dots=\mathcal{C}_m=\mathcal{C}_{-1}$ and that the $\mathcal{C}_i$'s are non-degenerate for $i>m$. \bigskip Recall that $\mathcal{L}_\epsilon$ denotes the non-trivial irreducible $\PGl_2(K)$-equivariant local system on $\mathcal{C}_{-1}$. \bigskip If $K=\overline{\F}_q$, it is equipped with a natural $F$-equivariant structure and we denote by $$ Y_\epsilon:\PGl_2(\F_q)\rightarrow\overline{\Q}_\ell $$ the characteristic function of $\mathcal{L}_\epsilon$. It is supported on $(\mathcal{C}_{-1})^F$. \bigskip \begin{remark} The set $(\mathcal{C}_{-1})^F$ is not a single conjugacy class of $\PGl_2(\F_q)$. Since the stabiliser of an element of $\mathcal{C}_{-1}$ has two connected components, it decomposes as $$ (\mathcal{C}_{-1})^F=\mathcal{O}_{e}\sqcup\mathcal{O}_{\sigma}, $$ the disjoint union of two $\PGl_2(\F_q)$-conjugacy classes. Here $\mathcal{O}_{e}$ is the $\PGl_2(\F_q)$-conjugacy class of the image $D_e=p(g_{-1})$ of the matrix $g_{-1}=\left(\begin{array}{cc}1&0\\0&-1\end{array}\right)$ in $\PGl_2$. For the other one we fix $x\in\F_{q^2}^*$ such that $x^q=-x$. The matrix $$\left(\begin{array}{cc}x&0\\0&-x\end{array}\right) $$ is $\sigma F$-stable and so is $\Gl_2(\overline{\F}_q)$-conjugate to an $F$-stable matrix $g_{\sigma}$ in $T_\sigma$. The image $D_\sigma=p(g_{\sigma})$ in $\PGl_2$ of this latter matrix is $F$-stable and is geometrically conjugate to $D_e$ (but not rationally as $x\notin\F_q$). The second conjugacy class $\mathcal{O}_{\sigma}$ is the $\PGl_2(\F_q)$-conjugacy class of $D_\sigma$. Then $$ Y_\epsilon=1_{\mathcal{O}_e}-1_{\mathcal{O}_\sigma}, $$ i.e. it takes the value $1$ at $\mathcal{O}_e$ and $-1$ at $\mathcal{O}_\sigma$. \label{rmdecomp}\end{remark} \bigskip Fix a subset $B\subset\{2,\dots,m\}$ (possibly empty) and let $({\bf Y}_1,\dots,{\bf Y}_k)$ be the $k$-tuple of functions ${\bf Y}_i:\PGl_2(\F_q)\rightarrow\overline{\Q}_\ell$ given by $$ {\bf Y}_i=\begin{cases}Y_\epsilon&\text{ if }i\in \{1\}\cup B,\\ 1_{(\mathcal{C}_i)^F}&\text{ otherwise.}\end{cases} $$ where $1_{(\mathcal{C}_i)^F}$ is the characteristic function of $(\mathcal{C}_i)^F$ that takes the value $1$ on $(\mathcal{C}_i)^F$ and $0$ elsewhere. \bigskip \begin{teorema} We have \label{theoremEulerspeciaz} $$ E(\mathcal{M}_{\mathcal{C}}(\C),\mathcal{F}_B;q)=E(\mathcal{M}_\mathcal{C}(\overline{\F}_q),\mathcal{F}_B;q)=\left\langle {\bf Y}_1*\cdots*{\bf Y}_k,1_{\{1\}}\right\rangle_{\PGl_2(\F_q)}$$ \end{teorema} \bigskip Consider a \emph{generic} $k$-tuple $({\bold X}_1,\dots, {\bf X}_k)$ of characteristic functions of character-sheaves on $\Sl_2$ (for the definition of genericity, see in \S \ref{proofB}) such that $$ {\bf X}_i=\begin{cases}X_\epsilon&\text { if }i\in \{1\}\cup B,\\ X_1&\text{ if }i\in \{2,\dots,m\}\backslash B,\\ R_{T'_2}^{\Sl_2}(\gamma_i)&\text{ otherwise.} \end{cases} $$ \begin{teorema}We have \begin{equation} \left\langle {\bf X}_1 \cdots {\bf X}_k,1\right\rangle_{\Sl_2}=\frac{1}{2^{m-1}}\sum_{\substack{r=0\\r \text{ even}}}^{m}C_{|B|+1,m-|B|-1,r}\,\,\mathbb{A}_r\left(0,\sqrt{q}\right). \label{B'}\end{equation} \label{ThB}\end{teorema} \bigskip We deduce from the above theorems together with the conjectural formula (\ref{pure-conj}) the following conjectural picture : \begin{center} \begin{tikzcd} \label{diag1} & & & H_c(\mathcal{M}_\mathcal{C}(\C),\mathcal{F}_B;q,t) \arrow[d,"t \mapsto -1"] \arrow[rd, "\text{ pure part}"]\\ & & & \langle {\bf Y}_1*\cdots*{\bf Y}_k,1_1 \rangle_{\PGl_2(\F_q)} & q^{k-3} \langle {\bf X}_1\cdots{\bf X}_k,1 \rangle_{\Sl_2(\F_q)} \end{tikzcd} \end{center} We will start with the proof of Theorem \ref{ThB}. Notice that the computation of the LHS of (\ref{B'}) will reduce to some computations in $\Gl_2$ thanks to the Frobenius reciprocity formula $$ \left\langle {\rm Res}^{\Gl_2(\F_q)}_{\Sl_2(\F_q)}(f), 1\right\rangle_{\Sl_2}=\left\langle f,{\rm Ind}_{\Sl_2(\F_q)}^{\Gl_2(\F_q)}(1)\right\rangle_{\Gl_2} $$ for any class function $f$ on $\Gl_2(\F_q)$. \subsection{Some result for $\mathrm{GL}_2$} Consider a $k$-tuple $R=(R_1,\dots,R_k)$ of Deligne-Lusztig characters of $\Gl_2(\F_q)$ (possible not irreducible) of the form $$ R=\left(R_{T_1}^{\Gl_2}(\alpha_1^1,\alpha^1_2),\dots,R_{T_1}^{\Gl_2}(\alpha^{k_1}_1,\alpha^{k_1}_2),-R_{T_\sigma}^{\Gl_2}(\omega_1),\dots,-R_{T_\sigma}^{\Gl_2}(\omega_{k_2})\right) $$ with $k_1>0$ (but we allow $k_2$ to be zero). \bigskip We have the following definition \cite[Definition 6.8.6]{letellier2}. \begin{definizione} We say that $R$ is \emph{generic} if \bigskip (1) $$\prod_{i=1}^{k_1}(\alpha^i_1\alpha^i_2)\prod_{j=1}^{k_2}(\omega_j|_{\F_q^*})=1.$$ (2) If $k_2=0$, we further ask that for any $w_1,\dots,w_k\in S_2$, we have $$ \prod_{i=1}^k\alpha^i_{w_i(1)}\neq 1. $$ \label{def-generic}\end{definizione} \begin{teorema}\cite[Theorem 6.10.1]{letellier2} If $R$ is generic then $$ \left\langle R_1\otimes\cdots\otimes R_k,1\right\rangle_{\Gl_2}=(-1)^{k_2}\mathbb{A}_{k_2}(0,\sqrt{q}). $$ \label{JEMS}\end{teorema} \bigskip \begin{remark} It follows from the theorem that $$ \left\langle R_1\otimes\cdots\otimes R_k,1\right\rangle_{\Gl_2} $$ does not depend on the choice of the characters $(\alpha_1^i,\alpha_2^i)$ and $\omega^j$ as long as the choice is generic. \label{independ}\end{remark} \bigskip \begin{remark} If the condition (1) in Definition \ref{def-generic} is not satisfied, then $$ \left\langle R_1\otimes\cdots\otimes R_k,1\right\rangle_{\Gl_2}=0. $$ This is analoguous to the condition (\ref{det}) for the character stack $\mathcal{M}_{\mathcal{C}}$ to be non-empty. \label{cond1}\end{remark} \subsection{From $\mathrm{GL}_2$ to $\mathrm{SL}_2$} Consider a $k$-tuple $R'=(R'_1,\dots,R'_k)$ of Deligne-Lusztig characters of $\Sl_2(\F_q)$ (possibly not irreducible) of the form $$ R'=\left(R_{T'_1}^{\Sl_2}(\gamma_1),\dots,R_{T'_1}^{\Sl_2}(\gamma_{k_1}),-R_{T'_\sigma}^{\Sl_2}(\eta_1),\dots,-R_{T'_\sigma}^{\Sl_2}(\eta_{k_2})\right). $$ Each $R'_i$ is the restriction of a Deligne-Lusztig character $R_i$ of $\Gl_2(\F_q)$ of the form $$ R_i=\begin{cases}R_{T_1}^{\Gl_2}(1,\gamma_i)&\text{ if } 1\leq i\leq k_1,\\-R_{T_\sigma}^{\Gl_2}(\omega_i)&\text{ if }k_1+1\leq i\leq k_2. \end{cases} $$ By the Frobenius reciprocity formula we have $$ \left\langle R'_1\otimes\cdots\otimes R'_k\right\rangle_{\Sl_2}=\left\langle R_1\otimes\cdots\otimes R_k,{\rm Ind}_{\Sl_2}^{\Gl_2}(1)\right\rangle_{\Gl_2}. $$ Since $$ {\rm Ind}_{\Sl_2}^{\Gl_2}(1)=\sum_{\delta\in\widehat{\F_q^*}}\delta\circ\det $$ we have $$ \left\langle R'_1\otimes\cdots\otimes R'_k\right\rangle_{\Sl_2}=\sum_{\delta\in\widehat{\F_q^*}}\left\langle R_1\otimes\cdots\otimes R_k\otimes(\delta^{-1}\circ\det),1\right\rangle_{\Gl_2} $$ By Remark \ref{cond1}, the multiplicity $$ \left\langle R_1\otimes\cdots\otimes R_k\otimes(\delta^{-1}\circ\det),1\right\rangle_{\Gl_2} $$ vanishes unless $$ \gamma_1\cdots\gamma_{k_1}(\omega_1|_{\F_q^*})\cdots(\omega_{k_2}|_{\F_q^*})=\delta^2. $$ We need the following lemma. \begin{lemma}Given $\gamma\in\widehat{\F_q^*}$, the equation $$ \delta^2=\gamma $$ has a solution if and only if $\gamma(-1)=1$. In this case the two solutions differ by the character $\alpha_o$. \end{lemma} \begin{proof} The group homomorphism $$ \phi:\widehat{\F_q^*}\rightarrow\{-1,1\},\hspace{1cm}\gamma\mapsto \gamma(-1) $$ is surjective and we see that the square elements of $\widehat{\F_q^*}$ live in the kernel of $\varphi$. The subgroup ${\rm Ker}(\varphi)$ is thus of index $2$ in $\widehat{\F_q^*}$ and it contains the subgroup of square elements of $\widehat{\F_q^*}$ which is also of index $2$ hence the result. \end{proof} We thus have the following result. \begin{prop} (1) If $\left(\gamma_1\cdots\gamma_{k_1}(\omega_1|_{\F_q^*})\cdots(\omega_{k_2}|_{\F_q^*})\right)(-1)=-1$, then $$ \left\langle R'_1\otimes\cdots\otimes R'_k,1\right\rangle_{\Sl_2}=0. $$ (2) If $\left(\gamma_1\cdots\gamma_{k_1}(\omega_1|_{\F_q^*})\cdots(\omega_{k_2}|_{\F_q^*})\right)(-1)=1$, then we choose a square root $$ \lambda_R=\sqrt{\gamma_1\cdots\gamma_{k_1}(\omega_1|_{\F_q^*})\cdots(\omega_{k_2}|_{\F_q^*})}$$and we have \bigskip $\left\langle R'_1\otimes\cdots\otimes R'_k,1\right\rangle_{\Sl_2}$ $$=\left\langle R_1\otimes\cdots\otimes R_k\otimes(\lambda_R^{-1}\circ\det),1\right\rangle_{\Gl_2}+\left\langle R_1\otimes\cdots\otimes R_k\otimes(\lambda_R^{-1}\alpha_o\circ\det),1\right\rangle_{\Gl_2}.$$ \label{GL-SL}\end{prop} \subsection{Generic $k$-tuples of semisimple characters of $\mathrm{SL}_2(\F_q)$}\label{generic-char} Let $R'=(R'_1,\dots,R'_k)$ be as in the previous section. \begin{definizione}The $k$-tuple is said to be \emph{generic} if we have : (1) $$\prod_{i=1}^{k_1}\gamma_i(-1)\prod_{j=1}^{k_2}\eta_i(-1)=1.$$ (2) If $k_2=0$, we further ask that for all $w_1,\dots,w_k\in W=S_2=\{1,\sigma\}$, $$ {^{w_1}}\gamma_1\cdots{^{w_k}}\gamma_k\neq 1 $$ where ${^\sigma}\gamma$ is the character $\gamma^{-1}$. \label{gen-SL}\end{definizione} \begin{prop} The $k$-tuple $R'$ is generic if and only if the two $(k+1)$-tuples of irreducible characters of $\Gl_2(\F_q)$ $$R^+=(R_1,\dots,R_k, \lambda_R^{-1}\circ\det),\hspace{.5cm}\text{ and }\hspace{.5cm} R^-=(R_1,\dots,R_k, \lambda_R^{-1}\alpha_o\circ\det) $$ are both generic. \label{gen}\end{prop} \begin{remark} If $T$ is an $F$-stable maximal torus of $\Gl_2$, $\theta$ a character of $T^F$ and $\lambda$ a linear character of $\F_q^*$, then $$ R_T^{\Gl_2}(\theta)\otimes (\lambda\circ\det)=R_T^{\Gl_2}(\theta\otimes(\lambda\circ\det)).$$ The $(k+1)$-tuple $R^+$ is generic if and only if the $k$-tuple $(R^\lambda_1,R_2,\dots,R_k)$, where $$ R^\lambda_1=R_{T_1}^{\Gl_2}\left((1,\gamma_1)\otimes(\lambda^{-1}_R\circ\det)\right),$$ is generic. \label{DL-gen}\end{remark} \begin{proof} From our choice of $\lambda_R$, we have that $$\lambda_R^{-2}\prod_{i=1}^{k_1}\gamma_i \prod_{j=1}^{k_2}(\omega_j|_{\F_q^*})=(\lambda_R\alpha_o)^{-2}\prod_{i=1}^{k_1}\gamma_i \prod_{j=1}^{k_2}(\omega_j|_{\F_q^*})=1 .$$ We see thus that the condition $(1)$ of Definition \ref{def-generic} is verified for the $(k+1)$-tuples $R^+,R^-$. Assume now that $k_2=0$ and consider $w_1,\dots,w_k \in W$. Let $I\subseteq \{1,\dots,k\}$ be the subset $$I \coloneqq \{i \in \{1,\dots,k\}\ | \ w_i=\sigma\} .$$ Condition $(2)$ of Definition \ref{def-generic} for the $(k+1)$-tuples $R^+,R^-$ and the elements $w_1,\dots,w_k$ is thus \begin{equation} \label{inequalities-genericity} \lambda_R^{-1}\prod_{i \in I}\gamma_i \neq 1 \ \ \text{ and } \ \ \lambda_R^{-1}\alpha_o\prod_{i \in I}\gamma_i \neq 1. \end{equation} Notice that (\ref{inequalities-genericity}) above holds if and only if \begin{equation} \label{inequalities-genericity1} \lambda_R^{-2}\prod_{i \in I}\gamma^2_i \neq 1. \end{equation} We have now $$\lambda_R^{-2}\prod_{i \in I}\gamma^2_i=\prod_{i \in I}\gamma_i \prod_{j \in I^{c}}\gamma_j^{-1}=({^{w_1}}\gamma_1\cdots{^{w_k}}\gamma_k)^{-1} ,$$ i.e. (\ref{inequalities-genericity}) is equivalent to the Condition $(2)$ of Definition \ref{gen-SL} for the $k$-tuple $R^{'}$ and the elements $w_1,\dots,w_k$. \end{proof} \begin{teorema}If $R'$ is generic then (1) $$\left\langle R_1\otimes\cdots\otimes R_k\otimes(\lambda_R^{-1}\circ\det),1\right\rangle_{\Gl_2}=\left\langle R_1\otimes\cdots\otimes R_k\otimes(\lambda_R^{-1}\alpha_o\circ\det),1\right\rangle_{\Gl_2}$$ (2) $$ \left\langle R'_1\otimes\cdots\otimes R'_k,1\right\rangle_{\Sl_2}=2(-1)^{k_2}\mathbb{A}_{k_2}(0,\sqrt{q}). $$ \label{theoR'}\end{teorema} \begin{proof}The first assertion is a consequence of Proposition \ref{gen}, Remark \ref{DL-gen} and Remark \ref{independ}. The second assertion is a consequence of the assertion (1) together with Proposition \ref{GL-SL} and Theorem \ref{JEMS}. \end{proof} \subsection{Proof of Theorem \ref{ThB}}\label{proofB} We use the notation of \S \ref{statement}. We assume that the $k$-tuple $({\bf X}_1,\dots,{\bf X}_k)$ is \emph{generic}, meaning that the $k$-tuple $$ \left(\underbrace{R_{T'_1}^{\Sl_2}(\alpha_o),\dots,R_{T'_1}^{\Sl_2}(\alpha_o)}_m,{\bf X}_{m+1},\dots,{\bf X}_k\right)$$ of Deligne-Lusztig characters of $\Sl_2(\F_q)$ is generic. \bigskip Put $m_1=|B|+1$ and $m_2=m-m_1$. We have $$ \left\langle {\bf X}_1\otimes\cdots\otimes{\bf X}_k,1\right\rangle_{\Sl_2}=\left\langle (X_\epsilon)^{\otimes m_1}\otimes (X_1)^{\otimes m_2}\otimes\bigotimes_{j=m+1}^kR_{T'_1}^{\Sl_2}(\gamma_j),1\right\rangle_{\Sl_2} $$ From Theorem \ref{unif}, we have \bigskip $\left\langle {\bf X}_1\otimes\cdots\otimes{\bf X}_k,1\right\rangle_{\Sl_2}$ \begin{align*}&=\frac{1}{2^m}\left\langle (R_{T_1}^{\Sl_2}(\alpha_o)-R_{T'_\sigma}^{\Sl_2}(\omega_o))^{\otimes m_1}\otimes (R_{T_1}^{\Sl_2}(\alpha_o)+R_{T'_\sigma}^{\Sl_2}(\omega_o))^{\otimes m_2}\otimes\bigotimes_{j=m+1}^k R_{T'_1}^{\Sl_2}(\gamma_j),1\right\rangle_{\Sl_2}\\ &=\frac{1}{2^m}\sum_{r=0}^m C_{|B|+1,m-|B|-1,r}\left\langle R_{T'_\sigma}^{\Sl_2}(\omega_o)^{\otimes r}\otimes R_{T'_1}^{\Sl_2}(\alpha_o)^{\otimes m-r}\otimes\bigotimes_{j=m+1}^kR_{T'_1}^{\Sl_2}(\gamma_j),1\right\rangle_{\Sl_2} \end{align*} By the genericity assumption we have $$ \alpha_o(-1)^m(\gamma_{m+1}\cdots\gamma_k)(-1)=1. $$ Since $\omega_o(-1)=-\alpha_o(-1)$ by Lemma \ref{alphaomega} we deduce that \begin{equation} \omega_o(-1)^r\alpha_o(-1)^{m-r}(\gamma_{m+1}\cdots\gamma_k)(-1)=(-1)^r. \label{eq-r}\end{equation} By Proposition \ref{GL-SL}(1) we see that $$ \left\langle R_{T'_\sigma}^{\Sl_2}(\omega_o)^{\otimes r}\otimes R_{T'_1}^{\Sl_2}(\alpha_o)^{\otimes m-r}\otimes\bigotimes_{j=m+1}^kR_{T'_1}^{\Sl_2}(\gamma_j),1\right\rangle_{\Sl_2} $$ vanishes unless $r$ is even. Therefore \bigskip $\left\langle {\bf X}_1\otimes\cdots\otimes{\bf X}_k,1\right\rangle_{\Sl_2}$ $$ =\frac{1}{2^m}\sum_{\substack{r=0\\r \text{ even}}}^m C_{|B|+1,m-|B|-1,r}\left\langle R_{T'_\sigma}^{\Sl_2}(\omega_o)^{\otimes r}\otimes R_{T'_1}^{\Sl_2}(\alpha_o)^{\otimes m-r}\otimes\bigotimes_{j=m+1}^kR_{T'_1}^{\Sl_2}(\gamma_j),1\right\rangle_{\Sl_2} $$ Now notice that the $k$-tuple $$ \left(\underbrace{R_{T'_\sigma}^{\Sl_2}(\omega_o),\dots,R_{T'_\sigma}^{\Sl_2}(\omega_o)}_r,\underbrace{R_{T'_1}^{\Sl_2}(\alpha_o),\dots,R_{T'_1}^{\Sl_2}(\alpha_o)}_{m-r},R_{T'_1}^{\Sl_2}(\gamma_{m+1}),\dots,R_{T'_1}^{\Sl_2}(\gamma_k)\right) $$ is generic for all even integer $r$ ($0\leq r\leq m$). Indeed, if $r=0$, then this is by assumption, if $0<r$ the condition (1) of Definition \ref{gen-SL} is satisfied by Formula (\ref{eq-r}) since $r$ is even (the condition (2) also since $k_2=r>0$). \bigskip We deduce Theorem \ref{ThB} from Theorem \ref{theoR'}(2). \subsection{The Euler specialization} \label{eulerspecializationproof} In this section, we prove Theorem \ref{theoremEulerspeciaz}. We give two proofs : a formal one and a computational one. The second proof is "dual" to the proof of Theorem \ref{ThB} given in \cref{proofB}. \subsubsection{A formal proof} Recall that $\mathcal{C}$ is a generic $k$-tuple of conjugacy classes of $\PGl_2(\overline{\F}_q)$ such that $\mathcal{C}_1=\cdots=\mathcal{C}_m$ are degenerate and $\mathcal{C}_j$ is non-degenerate for $j >m$. Fix $C=(C_1,\dots,C_k)$ a corresponding $k$-tuple of $F$-stable conjugacy classes of $\Gl_2(\overline{\F}_q)$ with the assumptions made at the beginning of \cref{cohomology}. \vspace{2 pt} Notice that, for any $(x_1,\dots,x_k) \in X_{\mathcal{C}}(\F_q)$, we have that \begin{equation} {\bf X_{\mathcal{L}_B}}(x_1,\dots,x_k)={\bf Y}_1(x_1) \cdots {\bf Y}_k(x_k). \end{equation} We thus have \begin{equation} \left\langle {\bf Y}_1 \ast \cdots \ast {\bf Y}_k,1_{\{1\}} \right\rangle=\dfrac{1}{|\PGl_2(\F_q)|}\sum_{(x_1,\dots,x_k) \in X_{\mathcal{C}}(\F_q)} {\bf Y}_1(x_1) \cdots {\bf Y}_k(x_k)= \end{equation} \begin{equation} \label{computationconvolutionfq} =\dfrac{1}{|\PGl_2(\F_q)|}\sum_{(x_1,\dots,x_k) \in X_{\mathcal{C}}(\F_q)}{\bf X_{\mathcal{L}_B}}(x_1,\dots,x_k)=\sum_{z \in \mathcal{M}_{\mathcal{C}(\F_q)}}{\bf X}_{\mathcal{F}_B}(z). \end{equation} From the last equality of formula (\ref{computationconvolutionfq}) and Grothendieck's trace formula, we deduce that $$\left\langle {\bf Y}_1 \ast \cdots \ast {\bf Y}_k,1_{\{1\}} \right\rangle=\sum_{k} (-1)^k\tr\left(F\,|\,H^k_c(\mathcal{M}_{\mathcal{C}},\mathcal{F}_B)\right)$$ Notice that, since the action of $H_m$ and $F$ on $H^*_c(\mathcal{M}_{\mathcal{C}})$ commute, the decompositon (\ref{decompositioncohomologyirred}) and the inversion formula in the character ring of $H_m$ imply that $$ \tr\left(F\,|\,H^k_c(\mathcal{M}_{\mathcal{C}},\mathcal{F}_B)\right)=\frac{1}{|H_m|}\sum_{y \in H_m}\tr\left(yF\,|\,H^k_c(\mathcal{M}^+_C)\right)\,\overline{\chi_B}(y) $$ for any $k$ and thus \begin{equation} \sum_{k} (-1)^k \tr\left(F\,|\,H^k_c(\mathcal{M}_{\mathcal{C}},\mathcal{F}_B)\right)=\dfrac {1}{2^{m-1}}\sum_{y \in H_m}\left(\sum_k (-1)^k \tr\left(yF\,|\,H^k_c(\mathcal{M}_C^+)\right)\right)\overline{\chi_B}(y) \end{equation} Theorem \ref{theoremEulerspeciaz} is now a consequence of Theorem \ref{theoremtwistedE} together with Grothendieck's trace formula and Formula (\ref{invertedrelationcharacters}). \subsubsection{A computational proof} \bigskip We give here another proof of Theorem \ref{theoremEulerspeciaz}, by direct computations of convolution products of class functions of $\PGl_2(\F_q)$. \vspace{2 pt} We start by the following more general remark. Consider $k$ semisimple conjugacy classes of $\PGl_2(\F_q)$ denoted by $\mathcal{O}_1,\dots,\mathcal{O}_k$ and corresponding conjugacy classes $O_1,\dots,O_k$ of $\Gl_2(\F_q)$ such that $\pi(O_j)=\mathcal{O}_j$. We assume that $\mathcal{O}_1,\dots,\mathcal{O}_m$ are degenerate (i.e. $\mathcal{O}_i=\mathcal{O}_{e}$ or $\mathcal{O}_i=\mathcal{O}_{\sigma}$ for $i=1,\dots,m$) and $\mathcal{O}_{m+1},\dots,\mathcal{O}_k$ are non-degenerate. If $\mathcal{O}_j=\mathcal{O}_e$, we take as $O_j$ the conjugacy class of $g_{-1}$ and if $\mathcal{O}_j=\mathcal{O}_{\sigma}$ we take as $O_j$ the conjugacy class of the matrix $g_{\sigma}$. We have the following equality \begin{equation} \label{convolutionfinitegroups} \langle 1_{\mathcal{O}_1} \ast \cdots \ast 1_{\mathcal{O}_k},1_{\{1\}} \rangle_{\PGl_2(\F_q)}=\dfrac{(q-1)}{2^m}\sum_{z \in \F_q^*}\langle 1_{O_1} \ast \cdots \ast 1_{O_k},1_{\{z\}} \rangle_{\Gl_2(\F_q)}. \end{equation} Indeed, the LHS of Formula (\ref{convolutionfinitegroups}) is $$\dfrac{\Big|\{(x_1,\dots,x_k) \in \mathcal{O}_1 \times \cdots \times \mathcal{O}_k \ | \ x_1\cdots x_k=1\}\Big|}{|\PGl_2(\F_q)|} $$ and the RHS of Formula (\ref{convolutionfinitegroups}) is $$\dfrac{\Bigg|\displaystyle\bigsqcup_{z \in \F_q^*} \{(y_1,\dots,y_k) \in O_1 \times \cdots \times O_k \ | \ y_1 \cdots y_k=z\} \Bigg|}{2^{m}|\PGl_2(\F_q)|} .$$ A similar argument to the one used in Proposition \ref{propdescriptiongeometry} in the geometric context, shows now that the projection map $$\displaystyle\bigsqcup_{z \in \F_q^*} \{(y_1,\dots,y_k) \in O_1 \times \cdots \times O_k \ | \ y_1 \cdots y_k=z\} \to \{(x_1,\dots,x_k) \in \mathcal{O}_1 \times \cdots \times \mathcal{O}_k \ | \ x_1\cdots x_k=1\} $$ $$(y_1,\dots,y_k) \to (p(y_1),\dots,p(y_k)) $$ is a quotient map for the free $(\Z/2\Z)^m$-action obtained by multiplication on the first $m$ coordinates. Notice that $\langle 1_{O_1} \ast \cdots \ast 1_{O_k},1_{\{z\}} \rangle_{\Gl_2(\F_q)}=0$ if $\det(O_1)\cdots \det(O_k) \neq z^2$. In particular, we deduce that \begin{equation} \label{oddr} \det(O_1) \cdots \det(O_k) \not \in (\F_q^*)^2 \Rightarrow \langle 1_{\mathcal{O}_1} \ast \cdots \ast 1_{\mathcal{O}_k},1_{\{1\}} \rangle_{\PGl_2(\F_q)}=0 \end{equation} where $(\F_q^*)^2$ is the subgroup of squares and otherwise, if $$ \det(O_1)\cdots \det(O_k)=\lambda_O^2 $$ with $\lambda_O \in \F_q^*$, then \begin{equation} \label{eqconvolutionGL2} \langle 1_{\mathcal{O}_1} \ast \cdots \ast 1_{\mathcal{O}_k},1_{\{1\}} \rangle_{\PGl_2(\F_q)}=\dfrac{(q-1)\left(\langle 1_{O_1} \ast \cdots \ast 1_{O_k},1_{\{\lambda_O\}}\rangle_{\Gl_2(\F_q)} +\langle 1_{O_1} \ast \cdots \ast 1_{O_k},1_{\{-\lambda_O\}} \rangle_{\Gl_2(\F_q)}\right)}{2^{m-1}} \end{equation} \bigskip We apply Formula (\ref{eqconvolutionGL2}) in the following way. Recall that by hypothesis we are assuming that $$\det(C_1)\cdots \det(C_k)=(-1)^m \det(C_{m+1})\cdots \det(C_{k}) \in (\F_q^*)^2 .$$ \vspace{4 pt} Fix a subset $B \subseteq \{2,\dots,m\}$ and the corresponding class functions $\bf{Y}_1,\dots,{\bf{Y}}_k$ as in \cref{statement}. Put $m_1=|B|+1$ and $m_2=m-m_1$. Since the convolution product is commutative, we have \begin{equation} \left\langle {\bf Y}_1 \ast \cdots \ast {\bf Y}_k,1_{\{1\}} \right\rangle=\left\langle Y_{\epsilon}^{\ast m_1} \ast Y_{1}^{\ast m_2} \ast 1_{\mathcal{C}_{m+1}^F} \ast \cdots \ast 1_{\mathcal{C}_{k}^F},1_{\{1\}} \right\rangle= \end{equation} \begin{equation} \left\langle (1_{\mathcal{O}_e}-1_{\mathcal{O}_{\sigma}})^{\ast m_1} \ast (1_{\mathcal{O}_e}+1_{\mathcal{O}_{\sigma}})^{\ast m_2} \ast 1_{\mathcal{C}_{m+1}^F} \ast \cdots \ast 1_{\mathcal{C}_{k}^F},1_{\{1\}} \right\rangle= \end{equation} \begin{equation} \sum_{r=0}^{m}C_{|B|+1,m-|B|-1,r}\left \langle 1_{\mathcal{O}_{\sigma}}^{\ast r} \ast 1_{\mathcal{O}_e}^{\ast m-r}\ast 1_{\mathcal{C}_{m+1}^F} \ast \cdots \ast 1_{\mathcal{C}_{k}^F},1_{\{1\}}\right\rangle. \end{equation} For each $r$, we have that $$\det(O_{\sigma})^r \det(O_{e})^{m-r}\det(C_{m+1})\cdots \det(C_{k})=x^{2r}(-1)^m \det(C_{m+1})\cdots \det(C_{k}) .$$ Since $x^2 \in \F_q^* \setminus (\F_q^*)^2$, from (\ref{oddr}), we have $$ \left \langle 1_{\mathcal{O}_{\sigma}}^{\ast r} \ast 1_{\mathcal{O}_e}^{\ast m-r}\ast 1_{\mathcal{C}_{m+1}^F} \ast \cdots \ast 1_{\mathcal{C}_{k}^F},1_{\{1\}}\right\rangle=0 ,$$ unless $r$ is even. Given the genericity of $\mathcal{C}$, Formula (\ref{eqconvolutionGL2}) and an argument similar to the one used at the end of the proof of Theorem \ref{Ey} show that, for each $r$ even, we have $$\left \langle 1_{\mathcal{O}_{\sigma}}^{\ast r} \ast 1_{\mathcal{O}_e}^{\ast m-r}\ast 1_{\mathcal{C}_{m+1}^F} \ast \cdots \ast 1_{\mathcal{C}_{k}^F},1_{\{1\}}\right\rangle=\dfrac{1}{2^{m-1}}\mathbb{A}_r\left(\frac{1}{\sqrt{q}},\sqrt{q}\right) .$$ Theorem \ref{theoremEulerspeciaz} is now a consequence of Theorem \ref{Epolynomialgenericdegenerate}. \begin{thebibliography}{} {\small \bibitem{ballandras}{\sc Ballandras, M.}: Intersection cohomology of character varieties for punctured Riemann surfaces, {\em J. \'Ec. polytech. Math.} {\bf 10} (2023), 141--198. \bibitem{BY}{\sc Bezrukavnikov, R. {\rm and }Yon Din, A.}: On parabolic restriction of perverse sheaves, {\em Publ. Res. Inst. Math. Sci} {\bf 57} (2021), 1089--1107. \bibitem{Deligne-etale}{\sc Deligne, P.}: Cohomologie etale, SGA 4-1/2 IV, {\em Lecture Notes in Mathematics SpringerVerlag}, {\bf 569}, (1977), pp. 233–261. \bibitem{Deligne}{\sc Deligne, P.}: Poids dans la cohomologie des variétés algèbriques, {\em Actes du congrès international des mathématiciens (Vancouver)} (1974). \bibitem{HodgeIII}{\sc Deligne, P.}: Th\'eorie de Hodge III, \emph{Inst. Hautes Etudes Sci. Publ. Math.} {\bf 44} \bibitem{DL}{\sc Deligne, P. {\rm and } Lusztig, G.}: \newblock Representations of reductive groups over finite fields. {\em Ann. of Math. (2)}, {\bf 103} (1976), 103--161. \bibitem{DM}{\sc Digne, F. {\rm and } Michel, J.} : Representations of Finite Groups of Lie Type, {\em London Math. Soc. Student Texts} {\bf 21}, Cambridge Univ. Press, Cambridge (1991). \bibitem{KNWG}{\sc Giannini, S., Kamgarpour, M., Nam, G. {\rm and} Whitbread, B. }: Counting points on character varieties, arXiv:2409.04735. \bibitem{HRV} {\sc Hausel, T. {\rm and} Rodriguez-Villegas, F.}: {Mixed Hodge polynomials of character varieties}, {\em Inv. Math.} { \bf 174}, No. 3, (2008), 555--624. \bibitem{HA} {\sc Hausel, T., Letellier, E. {\rm and} Rodriguez-Villegas, F.}: Arithmetic harmonic analysis on character and quiver varieties, {\em Duke Math. J.} {\bf 160} (2011), 323--400. \bibitem{HA1}{\sc Hausel, T., Letellier, E. {\rm and} Rodriguez-Villegas, F.}: Arithmetic harmonic analysis on character and quiver varieties II, {\em Adv. Math.} {\bf 234} (2013), 85--128. \bibitem{Weil}{\sc Kiehl, R. {\rm and} Weissaeur, R.}: Weil conjectures, perverse sheaves and $\ell$-adic Fourier transform, {\em Ergebnisse der Mathematik und ihrer Grenzgebiete. 3. Folge / A Series of Modern Surveys in Mathematics}, {\bf 42} (2001). \bibitem{laumonletellier}{\sc Laumon, G. {\rm and} Letellier, E.}: Note on a conjecture of Braverman-Kazhdan, {\em Advances in Mathematics} {\bf 419} (2023). \bibitem{laumonletellier2}{\sc Laumon, G. {\rm and} Letellier, E.}: On the derived Lusztig correspondence, {\em Forum Math. Sigma} {\bf 11} (2023). \bibitem{Let}{\sc Letellier, E.}: Fourier transforms of invariant functions on finite reductive Lie algebras, {\em Lectures Notes in Mathematics} {\bf 1859} (2005). \bibitem{L}{\sc Letellier, E.}: Character varieties with Zariski closures of $\Gl_n$-conjugacy classes at punctures, {\em Selecta Math. (N.S.)} {\bf 21} (2015), 293--344. \bibitem{letellier2}{\sc Letellier, E.}: Quiver varieties and the character ring of general linear groups over finite fields, {\em J. Eur. Math. Soc. (JEMS)} {\bf 15} (2013), no. 4, 1375--1455. \bibitem{LRV}{\sc Letellier, E. {\rm and} Rodriguez-Villegas, F.}: E-series of character varieties of non-orientable surfaces, {\em Ann. Inst. Fourier} {\bf 73} (2023), 1385--1420. \bibitem{Lusztig}{\sc Lusztig, G.}: Character sheaves $I$, {\em Advances in Mathematics} {\bf 56} (1985), 193-237. \bibitem{CS2}{\sc Lusztig, G.}: Character sheaves $II$, {\em Advances in Mathematics} {\bf 57} (1985), 226-265. \bibitem{LUG}{\sc Lusztig, G.}: Green functions and character-sheaves, {\em Ann. of . Math (2)} {\bf 131} (1990), 355--408. \bibitem{Lu-Fourier}{\sc Lusztig, G.}: Fourier transform on a semisimple Lie algebra over $\F_q$, \emph{Algebraic groups, Ultrech 1986, Springer Berlin} (1987), 177--188. \bibitem{mars}{\sc Mars, J. G. M. {\rm and} Springer, T. A.}: Character sheaves. {\em Orbites unipotentes et représentations III. Orbites et faisceaux pervers, Astérisque} {\bf 173-174} (1989), 111-198. \bibitem{MellitD}{\sc Mellit, A.}: Integrality of Hausel-Letellier-Villegas kernels, \emph{Duke Math. J.} {\bf 167} (2018), 3171--3205. \bibitem{Mellit}{\sc Mellit, A.}: Poincar\'e polynomials of character varieties, Macdonald polynomials and affine Springer fibers, \emph{Ann. of Math. (2)} 1{\bf 92} (2020), no. 1, 165--228. \bibitem{scognamiglio1}{\sc Scognamiglio, T.}: A generalization of Kac polynomials and tensor product of representations of $\Gl_n(\F_q)$, {\em Transformation Groups} (2024). \bibitem{scognamiglio2}{\sc Scognamiglio, T.}: Cohomology of non-generic character stacks, {\em J. \'Ec. polytech. Math.} {\bf 11} (2024), 1287--1371. \bibitem{schiffmann}{\sc Schiffmann, O.}: Indecomposable vector bundles and stable Higgs bundles over smooth projective curves, {\em Ann. of Math. (2)} {\bf 183} (2016), no. 1, 297--362. \bibitem{shoji}{\sc Shoji, T.}: Geometry of orbits and Springer correspondence, {\em Ast\'erisque} {\bf 168} (1988), 61-140. } \end{thebibliography} \end{document} \vspace{4 pt} \begin{oss} In the cases of interest to this article, i.e. when $G=\Gl_n,\Sl_n$, the unipotent characters of $G^F$ have the following description. Denote by $\mathcal{P}_n$ the set of partitions of $n$. Recall that $\mathcal{P}_n$ is in bijection with the set of irreducible characters $\widehat{S_n}$. We denote by $\chi^{\lambda} \in \widehat{S_n}$ the character associated to $\lambda \in \mathcal{P}_n$. In our bijection, $(n)$ is associated to the trivial character. \vspace{4 pt} For $G=\Gl_n,\Sl_n$, put $T=T_n,T_n'$ respectively, where $T_n,T'n$ are the corresponding $F$-stable maximal tori of diagonal matrices. In both cases, we have that $W_G(T)=S_n$ and the corresponding Frobenius action on $S_n$ is trivial. Denote now by $R_{\lambda}$ the following class function of $G^F$: $$R_{\lambda}=\dfrac{1}{n!}\sum_{\sigma \in S_n}R^G_{T_{\sigma}}(1)\chi^{\lambda}(\sigma) .$$ For $G=\Gl_n,\Sl_n$, each $R_{\lambda}$ is an irreducible character and, more precisely, the $R_{\lambda}$s are the unipotent irreducible characters of $G^F$. Moreover, we have a decomposition: $$R^G_T(1)=\sum_{\lambda \in \mathcal{P}_n}R_{\lambda}\chi^{\lambda}(e) ,$$ i.e. the unipotent characters of $G^F$ all appear in the decomposition of $R^G_T(1)$. \end{oss} \begin{oss} In the bijection (\ref{bijectionKummer}), the trivial character of $\widehat{T^F}$ is sent to the trivial local system $\overline{\Q}_{\ell}$. We have in particular, from eq.(\ref{charactersheavesbuilding}), an equality \begin{equation} R^G_T(1)=\chi_{r_!\psi^*\overline{\Q_{\ell}}}. \end{equation} Consider now the case of $G=\Gl_n,\Sl_n$ and $T$ the torus $T_n,T'_n$ respectively. Since from Lemma \ref{springermaplemma} the map $r$ is a $S_n$-Galois covering over $G_{rs}$ and $r$ is small, we have a decomposition $$\chi_{r_!\psi^*\overline{\Q_{\ell}}}=\bigoplus_{\lambda \in \mathcal{P}_n}\mathcal{R}_{\lambda}^{\oplus \chi^{\lambda}(e)} ,$$ where each $\mathcal{R}_{\lambda}$ is a (shifted) simple perverse sheaf. In this case, we have an equality \begin{equation} R_{\lambda}=\chi_{\mathcal{R}_{\lambda}}, \end{equation} i.e. the unipotent characters coincide with the characteristic functions of the character sheaves appearing in the decomposition of the trivial local system. This is not true in general for other reductive groups. \end{oss} \subsection{Preliminaries: $\mathrm{PGL}_2(\mathbb{F}_q)$} In this paragraph, we describe some geometrical properties of projective linear groups over $\F_q$. In what follows, $\PGl_2$ is $\PGl_2(\overline{\F_q})$ and $\Gl_2=\Gl_2(\overline{\F_q})$ and we denote by $F:\PGl_2 \to \PGl_2$, $F:\Gl_2 \to \Gl_2$ the respective canonical Frobenius morphisms, raising each entry of a matrix to the $q$-th power. Denote by $\widetilde{M_{-1}}$ the analogous element of $\PGl_2(\F_q)$ and by $\widetilde{\mathcal{O}}_{\widetilde{M_{-1}}}$ the corresponding conjugacy class in $\PGl_2$ and similarly for $M_{-1}$ and $\mathcal{O}_{M_{-1}}$. In the following, it will be clear whether the base field is $\C$ or $\F_q$ from the context. The conjugacy class $\widetilde{\mathcal{O}}_{\widetilde{M_{-1}}}$ is $F$-stable. However, unlike the case of $\Gl_2$ it is not true that $\widetilde{\mathcal{O}}_{\widetilde{M_{-1}}}$ is the conjugacy class of $\widetilde{M_{-1}}$ in $\PGl_2(\F_q)$. More precisely, we have a decomposition $$\widetilde{\mathcal{O}}_{\widetilde{M_{-1}}}^F=\widetilde{\mathcal{O}}_{\widetilde{M_{-1}}}^{+,F} \bigsqcup \widetilde{\mathcal{O}}_{\widetilde{M_{-1}}}^{-,F} ,$$ where $\widetilde{\mathcal{O}}_{\widetilde{M_{-1}}}^{+,F}$ is the conjugacy class of $\widetilde{M_{-1}}$ in $\PGl_2(\F_q)$ and $\widetilde{\mathcal{O}}_{\widetilde{M_{-1}}}^{-,F}$ is the conjugacy class of $$p_2\left(\begin{pmatrix}0 &-1\\ \eta^{q+1} &0 \end{pmatrix} \right),$$ where $\eta \in \F_{q^2}^*\setminus \F_q^*$ and $F(\eta)=-\eta$. \vspace{2 pt} As in the complex case, the morphism $p_2$ restricts to a $2$-covering $$p_2:\mathcal{O}_{M_{-1}} \to \widetilde{\mathcal{O}}_{\widetilde{M_{-1}}}$$ and therefore we have a splitting of $\ell$-adic sheaves over $\widetilde{\mathcal{O}}_{\widetilde{M_{-1}}}$ \begin{equation} \label{splittingFq} p_2(\overline{\Q}_{\ell}) \cong \overline{\Q}_{\ell} \oplus \mathcal{L}_{\F_q} .\end{equation} Taking the characteristic functions of the sheaves appearing in the RHS of eq.(\ref{splittingFq}), we get two functions $$\phi^+,\phi^-:\widetilde{\mathcal{O}}_{\widetilde{M_{-1}}}^F \to \overline{\Q}_{\ell} .$$ Extending the two functions by $0$ outside $\widetilde{\mathcal{O}}_{\widetilde{M_{-1}}}^F$ and fixing an isomorphism $\overline{\Q}_{\ell} \cong \C$, we obtain two functions $$\phi^+,\phi^-:\PGl_2(\F_q) \to \C .$$ Notice that $\phi^+,\phi^-$ are class functions since the decomposition of eq.(\ref{splittingFq}) is $\PGl_2$-equivariant. It is not difficult to verify that we have the following equalities in $\mathcal{C}(\PGl_2(\F_q))$ \begin{equation} \phi^+=1_{\widetilde{\mathcal{O}}_{\widetilde{M_{-1}}}^{+,F}}+1_{\widetilde{\mathcal{O}}_{\widetilde{M_{-1}}}^{-,F}} \end{equation} \begin{equation} \phi^-=1_{\widetilde{\mathcal{O}}_{\widetilde{M_{-1}}}^{+,F}}-1_{\widetilde{\mathcal{O}}_{\widetilde{M_{-1}}}^{-,F}} \end{equation} \section{Correspondence conjugacy class and local system and character sheaves} Consider the following more general situation. We have $(G,F)$ and $(G^*,F^*)$ two dual reductive groups over $\F_q$. Fix a split $F$-stable maximal torus $T \subseteq G$ and a corresponding dual split $F^*$-stable maximal torus $T^* \subseteq G^*$. Let $W=W_{G}(T)=W_{G^*}(T^*)$. Notice that $F$ acts trivially on $W$. Recall that we have the following correspondence \begin{equation}(T^F)^{\vee} \longleftrightarrow (T^*)^{F^*} ,\end{equation} see for example \cite[Proposition 11.1.14]{DM}. Recall moreover that we have another bijection \begin{equation} (T^F)^{\vee} \longleftrightarrow \{F-\text{stable Kummer local systems on } T \text{ up to isom. }\}. \end{equation} \vspace{2 pt} Fix now an $F^*$-stable regular semisimple conjugacy class $C^* \subseteq G^*$ and a representative $s^* \in (T^* \cap C^*)(\F_q)$. Put $$W_{s^*}=\{w \in W \ | \ w \cdot s^*=s^*\} .$$ Since $s^*$ is regular semisimple, we have that $$W_{s^*}=C_{G^*}(s^*)/C^{\circ}_{G^*}(s^*) .$$ Recall that we have the following bijection \begin{equation} (C_{G^*}(s^*)/C^{\circ}_{G^*}(s^*))^{\vee}=(W_{s^*})^{\vee} \longleftrightarrow \{\text{ Irreducible and } G^*-\text{equivariant local systems on } C^*\}. \end{equation} \vspace{2 pt} The element $s^*$ determines a character $s:T^F \to \C^*$ and therefore an $F$-stable Kummer local system $\mathcal{L}$. Put $$W_{\mathcal{L}}=\{w \in W \ | \ w^*\mathcal{L} \cong \mathcal{L}\} .$$ We have that $$W_{s^*}=W_{\mathcal{L}} .$$ Assume that $m$ is the smallest integer such that $\mathcal{L}^{\otimes m} \cong \overline{\Q}_{\ell}$. Denote by $\theta_m:T \to T$ the covering given by $\theta_m(t)=t^m$ and put $T[m]\coloneqq \Ker(\theta_m)$, i.e. $T[m]$ is given by the element whose order divides $m$. We have a decomposition \begin{equation} (\theta_m)_!\overline{\Q}_{\ell} \cong \bigoplus_{\phi:T[m] \to \C^*} \mathcal{L}_{\phi} \end{equation} analogous to that of eq.(\ref{decompositionsplitting}) and there exists $\phi:T[m] \to \C^*$ such that $\mathcal{L}_{\phi}=\mathcal{L}$. The character $\phi$ is $W_{\mathcal{L}}$ invariant (for the conjugation action). \vspace{2 pt} Fix an $F$-stable Borel $B \supseteq T$ and consider $\widetilde{G}=\{(x,gB) \in G \times G/B \ | \ g^{-1}xg \in B\}$ and the usual maps $p:\widetilde{G} \to G$ and $q:\widetilde{G} \to T$. Consider the perverse sheaf $$\mathcal{F}=p_!q^*\mathcal{L} .$$ Let $G_{reg} \subseteq G$ be the subset of semisimple regular elements. Recall that we can identify $$G_{reg}=G/T \times_W T_{reg}$$ and $$p^{-1}(G_{reg})=G/T \times T_{reg}$$ in such a way that $p(gT,t)=[(gT,t)]$. Since $p$ is a $W$-covering over $G_{reg}$, we have that $\mathcal{F}|_{G_{reg}}$ is local system and $$p_!q^*\mathcal{L}=\IC(\mathcal{F}_{G_{reg}}) .$$ Let now $p_m\coloneqq p(id \times \theta_m):G/T \times T_{reg} \to G_{reg}$. The map $p_m$ is a Galois covering with group $W \rtimes T[m]$ and we have a decomposition $$(p_m)_!(\overline{\Q}_{\ell}) \cong \bigoplus_{\rho \in (W \rtimes T[m])^{\vee}} \mathcal{M}_{\rho}^{\oplus d_{\rho}}$$ where $d_{\rho}=\dim(\rho)$. Moreover, if $\displaystyle \Ind_{T[m]}^{W \rtimes T[m]}(\phi)=\bigoplus_{\rho \in (W \rtimes T[m])^{\vee}}\rho^{\oplus m_{\rho}}$, we have that $$\mathcal{F}_{G_{reg}} \cong \mathcal{M}_{\rho}^{\oplus m_{\rho}} .$$ Notice that $$\Ind_{T[m]}^{W \rtimes T[m]}(\phi)=\Ind_{W_{\mathcal{L}} \rtimes T[m]}^{W \rtimes T[m]}(\Ind_{T[m]}^{W_{\mathcal{L}} \rtimes T[m]}(\phi)) .$$ Since the character $\phi$ is $W_{\mathcal{L}}$-invariant, we can extend it to a character $\phi :W_{\mathcal{L}} \rtimes T[m] \to \C^*$. We have therefore that $$\Ind_{T[m]}^{W_{\mathcal{L}} \rtimes T[m]}(\phi)=\C[W_{\mathcal{L}}] \otimes \phi$$ as $W_{\mathcal{L}} \rtimes T[m]$-modules. Moreover, for any $\chi \in W_{\mathcal{L}}^{\vee}$, the character $\Ind_{W_{\mathcal{L}} \rtimes T[m]}^{W \rtimes T[m]}(\chi \otimes \phi)$ is an irreducible character of $W \rtimes T[m]$. Let $d_{\chi}=\deg(\chi)$. In particular, we have a decomposition $$\Ind_{T[m]}^{W \rtimes T[m]}(\phi)=\bigoplus_{\chi \in W_{\mathcal{L}}^{\vee}} \Ind_{W_{\mathcal{L}} \rtimes T[m]}^{W \rtimes T[m]}(\chi \otimes \phi)^{\oplus d_{\chi}}$$ and therefore $$\mathcal{F}_{G_{reg}}=\bigoplus_{\chi \in W_{\mathcal{L}}^{\vee}} \mathcal{M}_{\Ind_{W_{\mathcal{L}} \rtimes T[m]}^{W \rtimes T[m]}(\chi \otimes \phi)}^{\oplus d_{\chi}} .$$ In particular, the irreducible perverse sheaves (i.e. the character sheaves) appearing in the decomposition of $\mathcal{F}$ are indexed by the irreducible local systems appearing in the decomposition of $\mathcal{F}_{G_{reg}}$, i.e. are indexed by $W_{\mathcal{L}}^{\vee}=\mathcal{W}_{s^*}^{\vee}$. ------------------------------------ \subsection{Pure cohomology of $\mathrm{PGL}_2(\C)$-character stacks and Langlands duality over finite fields} In this section, we explay how Theorem \ref{multiplicitiescharactersheaves} generalize the results of \cref{purecohomologyparagraph} through a sort of Langlands duality argument. We start by the following more general considerations. \vspace{2 pt} Consider $(G,F)$ and $(G^*,F^*)$ two dual split reductive groups over $\F_q$. For a definition, see for instance \cite[Definition 11.1.10]{DM}. Fix a split $F$-stable maximal torus $T \subseteq G$ and a corresponding dual split $F^*$-stable maximal torus $T^* \subseteq G^*$. Recall the following result, see for example \cite[Proposition 11.1.14]{DM}. \begin{prop} \label{bijectionelementscharacters1} We have an isomorphism of groups \begin{equation} \label{bijectionelementscharacters} \widehat{T^F} \longleftrightarrow (T^*)^{F^*} .\end{equation} \end{prop} \vspace{2 pt} In what follows, we will consider the case where $(G,F)=\Sl_2$ and $T=T_2'$ the torus of diagonal matrices and $(G^*,F^*)=\PGl_2$ and $T^*=\mathcal{T}$, the torus obtained as the projection of the torus of diagonal matrices. Fix a generator $\zeta_{q-1}$ of $\F_q^*$. The bijection (\ref{bijectionelementscharacters}) can be explicitly written down as follows $$\Phi:\Hom(\F_q^*,\C^*) \to \mathcal{T}(\F_q)$$ \begin{equation} \ \ \ \ \ \ \ \ \ \alpha \longrightarrow p_2\left(\begin{pmatrix} 1 &0\\ 0 &\alpha(\zeta_{q-1}) \end{pmatrix}\right) \end{equation} In particular, we have $$\Phi(\alpha_0)=p_2\left(\begin{pmatrix} 1 &0\\ 0 &-1 \end{pmatrix}\right) .$$ This correspondence has the following geometric interpretation. For each $\alpha \in \Hom(\F_q^*,\C^*)$, pick the $F$-stable Kummer local system $\mathcal{A}$ on $T_2'$ such that $\chi_{\mathcal{A}}=\alpha.$ Let now $s=\Phi(\alpha)$ and denote by $\mathcal{G}_s$ the skyscraper sheaf $(i_s)_*(\overline{\Q}_{\ell})$, where $i_s:s \to \mathcal{T}$ is the closed embedding. Through Proposition \ref{bijectionkummer} and bijection (\ref{bijectionelementscharacters}), we are thus associating to each Kummer local system $\mathcal{A}$ on $T_2'$ a corresponding skyscraper sheaf $\mathcal{G}_s$ on $\mathcal{T}$. We now compare their respective geometric inductions $\mathcal{I}^{\Sl_2}_{T_2'}(\mathcal{A})$ and $\mathcal{I}^{\PGl_2}_{\mathcal{T}}(\mathcal{G}_s)$. Notice that $\mathcal{I}^{\PGl_2}_{\mathcal{T}}(\mathcal{G}_s)=r_!\widetilde{\mathcal{G}}_{s}$, where $\widetilde{\mathcal{G}}_{s}=(i_{\psi^{-1}(s)})_*\overline{\Q}_{\ell}$ and $i_{\psi^{-1}(s)}:\psi^{-1}(s) \to \widetilde{\PGl_2}$ is the closed embedding. We assume that $\mathcal{A} \neq \overline{\Q}_{\ell}$, i.e. that $s$ is a regular element. In this case, we have that $r(\psi^{-1}(s))=\mathcal{C}_s$, the $\PGl_2$-conjugacy class of $s$ and through the identifications of Lemma \ref{lemmaspringerresolution}, we have $\psi^{-1}(s)=\PGl_2/\mathcal{T}$. We have thus two possibilities: \begin{itemize} \item $\mathcal{A} \neq \mathcal{A}_0$. In this case, on the one side, the (shifted) perverse sheaf $\mathcal{I}^{\Sl_2}_{T_2'}(\mathcal{A})$ is irreducible and, taking characteristic function, $\chi_{\mathcal{I}^{\Sl_2}_{T_2'}(\mathcal{A})}=R^{\Sl_2}_{T_{\epsilon'}}(\alpha)$. On the other side, since $s \neq p(g_{-1})$, we have that $r:\PGl_2/\mathcal{T} \to \mathcal{C}_{s} $ is an isomorphism, i.e. $\mathcal{I}^{\PGl_2}_{\mathcal{T}}(\mathcal{G}_s)$ is $(i_{\mathcal{C}_{s}})_{*}(\overline{\Q}_{\ell})$. In particular, we have $\chi_{\mathcal{I}^{\PGl_2}_{\mathcal{T}}(\mathcal{G}_s)}=1_{\mathcal{C}_{s}(\F_q)}$, i.e. the characteristic function of the conjugacy class $\mathcal{C}_{s}(\F_q)$. \item $\mathcal{A}=\mathcal{A}_0$. In this case, on the one side, as explained in \cref{sectioncharactersheaves}, we have $\mathcal{I}_{T_2'}^{\Sl_2}(\mathcal{A}_0)=X \oplus Y$. On the other side, we have that $r:\PGl_2/\mathcal{T} \to \mathcal{C}_{-1}$ is the analog over $\F_q$ of the $\Z/2\Z$-covering introduced in \cref{premilinariesconjclassesC} over $\C$ for the degenerate class $\mathcal{C}_{-1}$. Similarly to the complex setting, we have therefore a splitting $\mathcal{I}^{\PGl_2}_{\mathcal{T}}(\mathcal{G}_{p(g_{-1})})=\overline{\Q}_{\ell} \oplus \mathcal{L}_{\F_q}$, where $\mathcal{L}_{\F_q}$ is associated to the character $\sgn$. In particular, $\chi_{\mathcal{I}^{\PGl_2}_{\mathcal{T}}(\mathcal{G}_{p(g_{-1})})}=1_{\mathcal{C}_{-1}}+\chi_{\mathcal{L}_{\F_q}}$. Notice that, since $\mathcal{L}_{\F_q}$ is $\PGl_2$-equivariant, $\chi_{\mathcal{L}_{\F_q}}$ is a class function. \end{itemize} The bijection of Proposition \ref{bijectionelementscharacters} can therefore be extended to the following correspondence between characters of $\Sl_2(\F_q)$ and local systems on conjugacy classes of $\PGl_2(\overline{\F_q})$ (and their characteristic functions): $$\alpha \neq 1,\alpha_0 ,\ \ R^{\Sl_2}_{T'_2}(\alpha) \rightsquigarrow \text{local system } \overline{\Q}_{\ell} \text{ over } \mathcal{C}_s \ \ (1_{\mathcal{C}_s}) $$ $$\chi_Y \rightsquigarrow \text{ local system } \overline{\Q}_{\ell} \text{ over the degenerate conjugacy class } \mathcal{C}_{-1} \ \ (1_{\mathcal{C}_{-1}})$$ $$\chi_X \rightsquigarrow \text{ local system } \mathcal{L}_{\F_q}\text{ over the degenerate conjugacy class } \mathcal{C}_{-1} \ \ (\chi_{\mathcal{L}_{\F_q}}) .$$ \vspace{4 pt} In this framework, Conjecture \ref{conjmhslocalsystems} and Theorem \ref{multiplicitiescharactersheaves} have the following interpretation in terms of the duality here explained. \subsubsection{Preliminaries representations} We denote by $\alpha_0$ be the non-trivial character $$\alpha_0:\F_q^* \to \{\pm 1 \} .$$ We have the following. \begin{prop} Given $\gamma \in \Hom(\F_q^*,\C^*)$, there exists a solution to the equation \begin{equation} \delta^2=\gamma \end{equation} if and only if $\gamma(-1)=1$. In this case, the solutions are $\{\delta,\alpha_0 \delta\}$. \end{prop} \subsubsection{Character varieties} We propose thus the following definition of genericity for $k$-tuples of conjugacy classes of $\PGl_2(\C)$. \begin{definizione} Consider a $k$-tuple $\widetilde{\mathcal{C}}$ with $\widetilde{C}_i=\widetilde{C}_{x_i}$ and $x_1,\dots,x_k \in \C^*\setminus\{ 1\}$. We say that $\widetilde{C}$ is generic if and only if $\mathcal{C}(1),\mathcal{C}(-1)$ are generic in the usual sense. \end{definizione} \begin{oss} Notice that if there exists $j$ such that $x_j=-1$, the $k$-tuple $\mathcal{C}(1)$ is generic if and only if $\mathcal{C}(-1)$ is generic. \end{oss} \begin{oss} Notice that if $\mathcal{C}(1),\mathcal{C}(-1)$ are generic, we expect $M_{\mathcal{C}}$ and $ M_{\mathcal{C}(-1)}$ to have the same cohomology. If $x_1,\dots,x_k \neq -1$ we should thus have \begin{equation} \label{genericity1} H_c(M_{\widetilde{\mathcal{C}}},q,t)=2H_c(M_{\mathcal{C}},q,t). \end{equation} \end{oss} \begin{oss} Let $\sigma_{\widetilde{\mathcal{C}}} \in (\C^*)^I$ be the element $$\sigma_{\widetilde{\mathcal{C}}}\coloneqq(x_1\cdots x_k,x_1^{-2},\dots,x_k^{-2}) .$$ It is not difficult to check that $\mathcal{C}(1),\mathcal{C}(-1)$ are generic if and only if $\sigma_{\widetilde{\mathcal{C}}}$ is generic with respect to $\alpha$, i.e if and only if $$\mathcal{H}^{*}_{\sigma_{\widetilde{\mathcal{C}}},\alpha}=\{\delta \in (\N^I)^* \ | \ \sigma_{\widetilde{\mathcal{C}}}^{\delta}=1 \text{ and } \delta \leq \alpha \}=\{\alpha\} .$$ \end{oss} \vspace{6 pt} \subsubsection{Representations} Consider a $k$-tuple $$\widetilde{\Ch}=(R^{\Sl_2}_{T'}(\gamma_1),\dots,R^{\Sl_2}_{T'}(\gamma_k)) $$ and a $k$-tuple $$\Ch=(R^{\Gl_2}_{T}((\alpha_1,\beta_1),\dots,R^{\Gl_2}_{T}((\alpha_k,\beta_k)) $$ such that, for any $i$, we have $R^{\Sl_2}_{T_2'}(\gamma_i)=\Res^{T_2}_{T_2'}R^{\Gl_2}_{T_2}((\alpha_i,\beta_i))$. For any such $k$-tuple $\Ch$ $=(R^{\Gl_2}_{T}((\alpha_1,\beta_1)),\dots,R^{\Gl_2}_{T}((\alpha_k,\beta_k)))$ put $$\lambda_{\Ch}=\prod_{i=1}^k \alpha_i \beta_i .$$ \vspace{2 pt} From eq.(\ref{restriction1}) and Frobenius reciprocity, we deduce that, for any $\widetilde{\Ch}=(R^{\Sl_2}_{T'}(\gamma_1),\dots,R^{\Sl_2}_{T'}(\gamma_k))$ we have \begin{equation} \langle \bigotimes_{i=1}^k R^{\Sl_2}_{T'}(\gamma_i),1 \rangle= \langle \bigotimes_{i=1}^k R^{\Gl_2}_{T}((\gamma_i,1)), \bigoplus_{\delta \in \Hom(\F_q^*,\C^*)} \delta \circ \det \rangle. \end{equation} Let $\Ch=(R^{\Gl_2}_{T}(\gamma_1,1),\dots,R^{\Gl_2}_{T}(\gamma_k,1))$. Notice that, for any $\delta \in \Hom(\F_q^*,\C^*)$, we have that $\langle \bigotimes_{i=1}^k R^{\Gl_2}_{T}((\gamma_i,1)), \delta \circ \det \rangle \neq 0$ if and only if $$\delta^2=\lambda_{\Ch} .$$ We deduce therefore that $$ \langle \bigotimes_{i=1}^k R^{\Sl_2}_{T'}(\gamma_i),1 \rangle \neq 0$$ if and only if $\gamma_1 \cdots \gamma_k$ is a square, i.e if and only if $\gamma_1\cdot \gamma_k(-1)=1$. In this case, if we pick $\delta$ such that $\delta^2=\gamma_1 \cdots \gamma_k$, we have \begin{equation} \langle \bigotimes_{i=1}^k R^{\Sl_2}_{T'}(\gamma_i),1 \rangle=\langle \bigotimes_{i=1}^k R^{\Gl_2}_{T}((\gamma_i,1)), \delta \circ \det \rangle+ \langle \bigotimes_{i=1}^k R^{\Gl_2}_{T}((\gamma_i,1)), \alpha_0\delta \circ \det \rangle. \end{equation} We propose the following Definition of genericity for a $k$-tuple $\widetilde{\Ch}$ as above. \begin{definizione} The $k$-tuple $\widetilde{\Ch}$ is generic if and only if $(\Ch,\delta^{-1} \circ \det)$ and $(\Ch,\alpha_0 \delta^{-1} \circ \det)$ are generic in the usual sense. \end{definizione} \begin{oss} Let $\sigma_{\widetilde{\Ch}} \in \Hom(\F_q^*,\C^*)^I$ be the element $$\sigma_{\widetilde{\Ch}}=(\gamma_1,\dots,\gamma_k,\gamma_1^{-2},\dots,\gamma_k^{-2}) .$$ It is not difficult to check that $(\Ch,\delta^{-1} \circ \det)$ and $(\Ch,\alpha_0 \delta^{-1} \circ \det)$ are generic if and only if $\sigma_{\widetilde{\Ch}}$ is generic with respect to $\alpha$, i.e if and only if $$\mathcal{H}^*_{\sigma_{\widetilde{\Ch}},\alpha}=\{\alpha\} .$$ \end{oss} \begin{oss} If $(\Ch,\delta^{-1} \circ \det)$ and $(\Ch,\alpha_0 \delta^{-1} \circ \det)$ are both generic, the associated multiplicities are equal and we find therefore $$\langle \bigotimes_{i=1}^k R^{\Sl_2}_{T'}(\gamma_i),1 \rangle=2 \langle \bigotimes_{i=1}^k R^{\Gl_2}_{T}((\gamma_i,1)) \otimes \delta^{-1} \circ \det, 1 \rangle .$$ Consider generic $k$-tuples $\mathcal{C},\widetilde{\mathcal{C}}$ as above, with $\widetilde{\mathcal{C}}_i=\widetilde{\mathcal{C}}_{x_i}$ and $x_1,\dots,x_k \neq -1$. Conjecturally, we have that $$\langle \bigotimes_{i=1}^k R^{\Gl_2}_{T}((\gamma_i,1)) \otimes \delta^{-1} \circ \det, 1 \rangle=PH_c(M_{\mathcal{C}},q)$$ adn thus we should have $$\langle \bigotimes_{i=1}^k R^{\Sl_2}_{T'}(\gamma_i),1 \rangle=2PH_c(M_{\mathcal{C}},q)=PH_c(M_{\widetilde{\mathcal{C}}},q) .$$ \end{oss} \subsection{Character sheaves} Recall that we have a decomposition $$R^{\Sl_2}_{T'}(\alpha_0)=\chi^{+}_{\alpha_0}+\chi^{-}_{\alpha_0}$$ where $\chi^{+}_{\alpha_0},\chi^{-}_{\alpha_0}$ are irreducible. There are $4$ character sheaves which are not irreducible characters for $\Sl_2(\F_q)$. \begin{proof} From the first point of Definition \ref{definitiongenericitycharacterSln}, we have that $\gamma_1\cdots \gamma_k(-1)=1$ and therefore from eq.(\ref{eqmult5}), we have that \begin{equation} \langle R^{\Sl_2}_{T'_2}(\gamma_1) \otimes \cdots \otimes R^{\Sl_2}_{T'_2}(\gamma_k),1 \rangle= \end{equation} \begin{equation} \langle R^{\Gl_2}_{T_2}((\gamma_1,1))\otimes \cdots \otimes R^{\Gl_2}_{T_2}((\gamma_k,1))\otimes \lambda_{\Ch}^{-1}\circ \det,1 \rangle+\langle R^{\Gl_2}_{T_2}((\gamma_1,1))\otimes \cdots \otimes R^{\Gl_2}_{T_2}((\gamma_k,1)) \otimes \lambda_{\Ch}^{-1}\alpha_0\circ \det,1 \rangle. \end{equation} From Lemma \ref{lemmagenericitysl2}, we have that the $(k+1)$-tuples $ R^{\Gl_2}_{T_2}((\gamma_1,1)),\dots, R^{\Gl_2}_{T_2}((\gamma_k,1)),\lambda_{\Ch}^{-1}\circ \det)$ and $ R^{\Gl_2}_{T_2}((\gamma_1,1)),\dots, R^{\Gl_2}_{T_2}((\gamma_k,1)),\lambda_{\Ch}^{-1}\alpha_0\circ \det)$ are both generic. From \cite[Theorem 6.1.1]{HA}, we deduce that we have $$\langle R^{\Gl_2}_{T_2}((\gamma_1,1))\otimes \cdots \otimes R^{\Gl_2}_{T_2}((\gamma_k,1))\otimes \lambda_{\Ch}^{-1}\circ \det,1 \rangle=\langle R^{\Gl_2}_{T_2}((\gamma_1,1))\otimes \cdots \otimes R^{\Gl_2}_{T_2}((\gamma_k,1)) \otimes \lambda_{\Ch}^{-1}\alpha_0\circ \det,1 \rangle=$$ $$\mathbb{H}_{\mathbb{\mu}}(0,\sqrt{q}) .$$ We deduce therefore eq.(\ref{theoremmultiplicitiesformula}) \end{proof} Multiplicities for generic $k$-tuples have a well-understood combinatorial description in terms of the generating series $\Omega(z,w)$ introduced before. More precisely, in \cite{letellier2} to any $k$-tuple $\Ch$, it is associated a function $s_{\omega_{\Ch}} \in \Lambda(\mathbf{x}_1,\dots,\mathbf{x}_k)$. We have the following Theorem, see \cite[Theorem 6.10.1]{letellier2} \begin{teorema} \label{theoremgenericmultiplicitiesgln} If $\Ch$ is generic, we have \begin{equation} \label{genericmultiplicitiesglnformula} \langle \Ch_1 \otimes \cdots \otimes \Ch_k,1 \rangle=\mathbb{H}_{\omega_{\Ch}}(0,\sqrt{q}) \end{equation} where $\mathbb{H}_{\omega_{\Ch}}(z,w):=(z^2-1)(1-w^2)\left\langle {\rm Log}\,\Omega(z,w),s_{\omega_{\Ch}}\right\rangle$ \end{teorema} Fix now an $F^*$-stable regular semisimple conjugacy class $C^* \subseteq G^*$ and a representative $s^* \in (T^* \cap C^*)(\F_q)$. Put $$W_{s^*}=\{w \in W \ | \ w \cdot s^*=s^*\} .$$ Since $s^*$ is regular semisimple, we have that $$W_{s^*}=C_{G^*}(s^*)/C^{\circ}_{G^*}(s^*) .$$ Recall that we have the following bijection \begin{equation} (C_{G^*}(s^*)/C^{\circ}_{G^*}(s^*))^{\vee}=(W_{s^*})^{\vee} \longleftrightarrow \{\text{ Irreducible and } G^*-\text{equivariant local systems on } C^*\}. \end{equation} \vspace{2 pt} From the bijections \ref{bijectionelementscharacters},\ref{bijectionKummer}, the element $s^*$ determines a character $s:T^F \to \C^*$ and therefore an $F$-stable Kummer local system $\mathcal{E}$. Notice that we have $$W_{s^*}=W_{\mathcal{E}} .$$ From the arguments given at the end of \cref{sectioncharactersheaves}, we have therefore a bijection \begin{equation} \label{bijectionlocalsystems} \{\text{ Irreducible and } G^*-\text{equivariant local systems on } C^*\} \Leftrightarrow \{\text{Irreducible components of } r_!\psi^*\mathcal{E}\} .\end{equation} Apply this to the case where $(G,F)=\Sl_2$ and $(G^*,F^*)=\PGl_2$, as in Example \ref{exampleLanglandsduality}. Put $C^*=\mathcal{C}_{-1,\F_q}$ and $s^*=M_{-1}$, where we denote by $\mathcal{C}_{-1,\F_q}$ the conjugacy class of $M_{-1}$ in $\PGl_2$. As remarked in the Exmaple \ref{exampleLanglandsduality}, in this case $\mathcal{E}=\mathcal{A}_0$. The same reasoning applied in \cref{premilinariesconjclassesC}, in that case over $\C$, translates to the case of finite fields and we have therefore $$\{\text{ Irreducible and } \PGl_2-\text{equivariant local systems on } \mathcal{C}_{-1}\} =\{\overline{\Q}_{\ell},\mathcal{L}_{\F_q}\} ,$$ where $\mathcal{L}_{\F_q}$ is the analogous of $\mathcal{L}$ over $\F_q$. The bijection of eq.(\ref{bijectionlocalsystems}) becomes therefore the bijection $$\{\overline{\Q}_{\ell},\mathcal{L}_{\F_q}\} \Leftrightarrow \{X,Y\} $$ $$\mathcal{L}_{\F_q} \to X .$$
2412.03383v1
http://arxiv.org/abs/2412.03383v1
Optimal estimation of three parallel spins with genuine and restricted collective measurements
\documentclass[aps,pra,superscriptaddress,longbibliography,showpacs,twocolumn]{revtex4-2} \newif\ifapp \newcommand{\lsp}{\hspace{0.1em}} \usepackage{makecell} \usepackage{hyperref} \usepackage{physics} \usepackage{xfrac} \usepackage{graphicx} \usepackage{subfigure} \usepackage{mathdots} \usepackage{amsfonts,amssymb,amsmath} \usepackage{mathrsfs} \usepackage[]{graphics,graphicx,epsfig} \usepackage{amsthm} \usepackage{csquotes} \MakeOuterQuote{"} \usepackage{epstopdf} \usepackage{tikz} \usepackage{paralist} \usepackage{diagbox} \usepackage[inline]{enumitem} \usepackage{tabularx} \usepackage{setspace} \usepackage{mathtools} \usepackage{dsfont} \DeclarePairedDelimiter{\parens}{(}{)} \DeclarePairedDelimiter{\bracks}{[}{]} \DeclarePairedDelimiter{\bracx}{\{}{\}} \hypersetup{ breaklinks=true, colorlinks=true, linkcolor=blue, citecolor=red, filecolor=magenta, urlcolor=blue, runcolor=cyan } \addtolength{\textwidth}{-0.5in} \addtolength{\oddsidemargin}{0.25in} \addtolength{\evensidemargin}{0.25in} \def\identity{\leavevmode\hbox{\small1\kern-3.8pt\normalsize1}} \newtheorem{theorem}{Theorem} \newtheorem{lemma}{Lemma} \newtheorem{proposition}{Proposition} \newtheorem{definition}{Definition} \newtheorem{corollary}{Corollary} \newcommand{\Cl}{\mathrm{Cl}} \newcommand{\Sym}{\mathrm{Sym}} \newcommand{\bbA}{\mathbb{A}} \newcommand{\bbC}{\mathbb{C}} \newcommand{\bbE}{\mathbb{E}} \newcommand{\bbF}{\mathbb{F}} \newcommand{\bbI}{\mathbb{I}} \newcommand{\bbN}{\mathbb{N}} \newcommand{\bbR}{\mathbb{R}} \newcommand{\bbS}{\mathbb{S}} \newcommand{\bbW}{\mathbb{W}} \newcommand{\bbZ}{\mathbb{Z}} \newcommand{\bQ}{\bar{Q}} \newcommand{\bPhi}{\bar{\Phi}} \newcommand{\bcaQ}{\bar{\mathcal{Q}}} \newcommand{\caA}{\mathcal{A}} \newcommand{\caB}{\mathcal{B}} \newcommand{\caD}{\mathcal{D}} \newcommand{\caE}{\mathcal{E}} \newcommand{\caG}{\mathcal{G}} \newcommand{\caH}{\mathcal{H}} \newcommand{\caI}{\mathcal{I}} \newcommand{\caJ}{\mathcal{J}} \newcommand{\caL}{\mathcal{L}} \newcommand{\caM}{\mathcal{M}} \newcommand{\caN}{\mathcal{N}} \newcommand{\caO}{\mathcal{O}} \newcommand{\caP}{\mathcal{P}} \newcommand{\caQ}{\mathcal{Q}} \newcommand{\caR}{\mathcal{R}} \newcommand{\caS}{\mathcal{S}} \newcommand{\caT}{\mathcal{T}} \newcommand{\caU}{\mathcal{U}} \newcommand{\caV}{\mathcal{V}} \newcommand{\caW}{\mathcal{W}} \newcommand{\fkB}{\mathfrak{B}} \newcommand{\hka}{\hat{\kappa}} \newcommand{\hR}{\hat{R}} \newcommand{\rma}{\mathrm{a}} \newcommand{\rmd}{\mathrm{d}} \newcommand{\rme}{\operatorname{e}} \newcommand{\rmi}{\mathrm{i}} \newcommand{\rms}{\mathrm{s}} \newcommand{\bs}{\mathrm{bs}} \newcommand{\rmA}{\mathrm{A}} \newcommand{\rmB}{\mathrm{B}} \newcommand{\rmC}{\mathrm{C}} \newcommand{\rmG}{\mathrm{G}} \newcommand{\rmH}{\mathrm{H}} \newcommand{\rmI}{\mathrm{I}} \newcommand{\rmS}{\mathrm{S}} \newcommand{\rmT}{\mathrm{T}} \newcommand{\rmU}{\mathrm{U}} \newcommand{\rmW}{\mathrm{W}} \newcommand{\sgn}{\mathrm{sgn}} \newcommand{\col}{\mathrm{col}} \newcommand{\scrA}{\mathscr{A}} \newcommand{\scrB}{\mathscr{B}} \newcommand{\scrC}{\mathscr{C}} \newcommand{\scrK}{\mathscr{K}} \newcommand{\scrI}{\mathscr{I}} \newcommand{\scrL}{\mathscr{L}} \newcommand{\scrM}{\mathscr{M}} \newcommand{\scrO}{\mathscr{O}} \newcommand{\scrP}{\mathscr{P}} \newcommand{\scrS}{\mathscr{S}} \newcommand{\scrT}{\mathscr{T}} \newcommand{\scrV}{\mathscr{V}} \newcommand{\scrX}{\mathscr{X}} \newcommand{\scrY}{\mathscr{Y}} \newcommand{\scrZ}{\mathscr{Z}} \newcommand{\tscrE}{\tilde{\mathscr{E}}} \newcommand{\tscrM}{\tilde{\mathscr{M}}} \newcommand{\tf}{\tilde{f}} \newcommand{\tbfx}{\tilde{\mathbf{x}}} \newcommand{\tbfy}{\tilde{\mathbf{y}}} \newcommand{\tbfz}{\tilde{\mathbf{z}}} \newcommand{\tx}{\tilde{x}} \newcommand{\ty}{\tilde{y}} \newcommand{\tz}{\tilde{z}} \newcommand{\tg}{\tilde{g}} \newcommand{\tL}{\tilde{L}} \newcommand{\tN}{\tilde{N}} \newcommand{\tR}{\tilde{R}} \newcommand{\tT}{\tilde{T}} \newcommand{\tW}{\tilde{W}} \newcommand{\tka}{\tilde{\kappa}} \newcommand{\tomega}{\tilde{\omega}} \newcommand{\tgamma}{\tilde{\gamma}} \newcommand{\tmu}{\tilde{\mu}} \newcommand{\tDelta}{{\tilde{\Delta}}} \newcommand{\tPhi}{\tilde{\Phi}} \newcommand{\tPsi}{\tilde{\Psi}} \newcommand{\tcaN}{\tilde{\mathcal{N}}} \newcommand{\tcaT}{\tilde{\mathcal{T}}} \newcommand{\tcaW}{\tilde{\mathcal{W}}} \newcommand{\ocaW}{\overline{\mathcal{W}}} \newcommand{\tscrP}{\tilde{\mathscr{P}}} \newcommand{\proj}[1]{\ket{#1}\bra{#1}} \renewcommand{\epsilon}{\varepsilon} \newcommand{\vertiii}[1]{{\left\vert\kern-0.25ex\left\vert\kern-0.25ex\left\vert #1 \right\vert\kern-0.25ex\right\vert\kern-0.25ex\right\vert}} \newcommand{\lref}[1]{Lemma~\ref{#1}} \newcommand{\lsref}[1]{Lemmas~\ref{#1}} \newcommand{\Lref}[1]{Lemma~\ref{#1}} \newcommand{\Lsref}[1]{Lemmas~\ref{#1}} \newcommand{\thref}[1]{Theorem~\ref{#1}} \newcommand{\thsref}[1]{Theorems~\ref{#1}} \newcommand{\Thref}[1]{Theorem~\ref{#1}} \newcommand{\Thsref}[1]{Theorems~\ref{#1}} \newcommand{\pref}[1]{Proposition~\ref{#1}} \newcommand{\psref}[1]{Propositions~\ref{#1}} \newcommand{\Pref}[1]{Proposition~\ref{#1}} \newcommand{\Psref}[1]{Propositions~\ref{#1}} \newcommand{\coref}[1]{Corollary~\ref{#1}} \newcommand{\cosref}[1]{Corollaries~\ref{#1}} \newcommand{\Coref}[1]{Corollary~\ref{#1}} \newcommand{\Cosref}[1]{Corollaries~\ref{#1}} \newcommand{\cref}[1]{Conjecture~\ref{#1}} \newcommand{\csref}[1]{Conjectures~\ref{#1}} \newcommand{\Cref}[1]{Conjecture~\ref{#1}} \newcommand{\Csref}[1]{Conjectures~\ref{#1}} \def\eqref#1{\textup{(\ref{#1})}} \newcommand{\eref}[1]{Eq.~\textup{(\ref{#1})}} \newcommand{\eqsref}[2]{Eqs.~(\ref{#1}) and (\ref{#2})} \newcommand{\Eref}[1]{Equation~\textup{(\ref{#1})}} \newcommand{\Eqsref}[2]{Equations~(\ref{#1}) and (\ref{#2})} \newcommand{\tref}[1]{Table~\ref{#1}} \newcommand{\Tref}[1]{Table~\ref{#1}} \newcommand{\tsref}[1]{Tables~\ref{#1}} \newcommand{\Tsref}[1]{Tables~\ref{#1}} \newcommand{\sref}[1]{Sec.~\ref{#1}} \newcommand{\Sref}[1]{Section~\ref{#1}} \newcommand{\ssref}[1]{Secs.~\ref{#1}} \newcommand{\Ssref}[1]{Sections~\ref{#1}} \newcommand{\fref}[1]{Fig.~\ref{#1}} \newcommand{\Fref}[1]{Figure~\ref{#1}} \newcommand{\fsref}[1]{Figs.~\ref{#1}} \newcommand{\Fsref}[1]{Figures~\ref{#1}} \newcommand{\aref}[1]{Appendix~\ref{#1}} \newcommand{\asref}[1]{Appendices~\ref{#1}} \newcommand{\Aref}[1]{Appendix~\ref{#1}} \newcommand{\Asref}[1]{Appendices~\ref{#1}} \setcounter{equation}{0} \setcounter{figure}{0} \setcounter{table}{0} \setcounter{theorem}{0} \setcounter{lemma}{0} \setcounter{section}{0} \def\<{\langle} \def\>{\rangle} \newcommand{\rcite}[1]{Ref.~\cite{#1}} \newcommand{\rscite}[1]{Refs.~\cite{#1}} \newcommand{\equad}{\,\hphantom{=}\,} \begin{document} \title{Optimal estimation of three parallel spins with genuine and restricted collective measurements} \author{Changhao Yi} \affiliation{State Key Laboratory of Surface Physics, Department of Physics, and Center for Field Theory and Particle Physics, Fudan University, Shanghai 200433, China} \affiliation{Institute for Nanoelectronic Devices and Quantum Computing, Fudan University, Shanghai 200433, China} \affiliation{Shanghai Research Center for Quantum Sciences, Shanghai 201315, China} \author{Kai Zhou} \affiliation{CAS Key Laboratory of Quantum Information, University of Science and Technology of China, Hefei 230026, People's Republic of China} \affiliation{CAS Center For Excellence in Quantum Information and Quantum Physics, University of Science and Technology of China, Hefei 230026, People's Republic of China} \author{Zhibo Hou} \affiliation{CAS Key Laboratory of Quantum Information, University of Science and Technology of China, Hefei 230026, People's Republic of China} \affiliation{CAS Center For Excellence in Quantum Information and Quantum Physics, University of Science and Technology of China, Hefei 230026, People's Republic of China} \affiliation{Hefei National Laboratory, University of Science and Technology of China, Hefei 230088, People's Republic of China} \author{Guo-Yong Xiang} \affiliation{CAS Key Laboratory of Quantum Information, University of Science and Technology of China, Hefei 230026, People's Republic of China} \affiliation{CAS Center For Excellence in Quantum Information and Quantum Physics, University of Science and Technology of China, Hefei 230026, People's Republic of China} \affiliation{Hefei National Laboratory, University of Science and Technology of China, Hefei 230088, People's Republic of China} \author{Huangjun~Zhu} \email{[email protected]} \affiliation{State Key Laboratory of Surface Physics, Department of Physics, and Center for Field Theory and Particle Physics, Fudan University, Shanghai 200433, China} \affiliation{Institute for Nanoelectronic Devices and Quantum Computing, Fudan University, Shanghai 200433, China} \affiliation{Shanghai Research Center for Quantum Sciences, Shanghai 201315, China} \begin{abstract} Collective measurements on identical and independent quantum systems are advantageous in information extraction than individual measurements. However, little is known about the distinction between restricted collective measurements and genuine collective measurements in the multipartite setting. In this work we establish a rigorous performance gap based on a simple and old estimation problem, the estimation of a random spin state given three parallel spins. Notably, we derive an analytical formula for the maximum estimation fidelity of biseparable measurements and clarify its fidelity gap from genuine collective measurements. Moreover, we show that this estimation fidelity can be achieved by two- and one-copy measurements assisted by one-way communication in one direction, but not the other way. Our work reveals a rich landscape of multipartite nonclassicality in quantum measurements instead of quantum states and is expected to trigger further studies. \end{abstract} \date{\today} \maketitle \section{Introduction} Quantum measurements are the key for extracting information from quantum systems \cite{NielC10book} and play crucial roles in various tasks in quantum information processing, such as quantum state estimation, quantum metrology, quantum communication, and quantum computation. When two or more quantum systems are available, collective measurements on all quantum systems together may extract more information than individual measurements \cite{PereW91,MassP95,GisiP99,Mass00,BagaBGM06S,Zhu12the,ZhuH18U}, even if there is no entanglement or correlation among these quantum systems. This intriguing phenomenon is a manifestation of nonclassicality in quantum measurements rather than quantum states. Moreover, collective measurements are quite useful in many practical applications, including quantum state estimation \cite{MassP95,BagaBGM06S,HaahHJW16,ODonW16,Zhu12the,ZhuH18U}, direction estimation \cite{GisiP99,Mass00}, multi-parameter estimation \cite{VidrDGJ14,LuW21,ChenCY22}, shadow estimation \cite{Aaro18,GriePS24,LiuLYZ24,LiYZZ24}, quantum state discrimination \cite{PereW91,HiggDBP11,MartHSS21}, quantum learning \cite{HuanKP21,ChenCHL22,AharCQ22,GrewIKL24}, entanglement detection and distillation \cite{Horo03,LindMP98,DehaVDV03}, and nonlocality distillation \cite{EftaWC23}. The power of collective measurements have been demonstrated in a number of experiments \cite{HouTSZ18,TangHSZ20,ZhouYYH23,ConlVMP23,ConlELA23,TianYHX24}. Although collective measurements are advantageous for many applications, their realization in experiments is quite challenging, especially for multi-copy collective measurements. Actually, almost all experiments in this direction are restricted to two-copy collective measurements, and a genuine three-copy collective measurement was realized only very recently \cite{ZhouYYH23}. In view of this situation, it is natural to ask whether there is a fundamental gap between restricted collective measurements on limited copies of quantum states and genuine collective measurements, which represent the ultimate limit. This problem is of interest to both foundational studies and practical applications. Unfortunately, little is known about this problem, although the counterpart for quantum states has been well studied \cite{HoroHHH09,BrunCPS13}. Conceptually, the very basic definitions remain to be clarified. Technically, it is substantially more difficult to analyze the performances of restricted collective measurements. In this work we start to explore the rich territory of multipartite nonclassicality in quantum measurements by virtue of a simple and old estimation problem, the estimation of a random spin state given three parallel spins \cite{MassP95,DerkBE98,LatoPT98,Haya98,BrusM99,GisiP99,Mass00,AcinLP00,BagaBM02,HayaHH05}. To set the stage, we first introduce rigorous definitions of biseparable measurements (which encompass all restricted collective measurements) and genuine collective measurements. Then, we derive an analytical formula for the maximum estimation fidelity of biseparable measurements, which clearly demonstrates a fidelity gap from genuine collective measurements. In addition, we determine the maximum estimation fidelity based on one- and two-copy collective measurements assisted by one-way communication. Surprisingly, such strategies can reach the maximum estimation fidelity of biseparable measurements if the communication direction is chosen properly. By contrast, the maximum estimation fidelity achievable is strictly smaller if the communication direction is reversed. Our work reveals a strict hierarchy of multi-copy collective measurements and a plethora of nonclassical phenomena rooted in quantum measurements, which deserve further studies. The rest of this paper is organized as follows. We begin with the formal definitions positive operator-valued measures (POVMs), biseparable POVMs and genuine collective POVMs in \sref{sec:definition}. In \sref{sec:state_estimation}, we review an old estimation problem and the concept of estimation fidelity. In \sref{sec:CollectiveOpt} we review an optimal POVM for estimating three parallel spins and show that it is genuinely collective, although all its POVM elements are biseparable. In \sref{sec:RestrictedOpt}, we determine the maximum estimation fidelities of biseparable measurements, 2+1 adaptive measurements, and 1+2 adaptive measurements, respectively, and construct optimal estimation strategies explicitly. \Sref{sec:Summary} summarizes this paper. \section{Separable and collective measurements}\label{sec:definition} \subsection{Quantum states and quantum measurements} Let $\caH$ be a given finite-dimensional Hilbert space and $\caL(\caH)$ the space of linear operators on $\caH$. Quantum states on $\caH$ are represented by positive (semidefinite) operators of trace 1. Quantum measurements on $\caH$ can be described by POVMs when post-measurement quantum states are irrelevant \cite{NielC10book}. Mathematically, a POVM is composed of a set of positive operators that sum up to the identity operator, which is denoted by $I$ henceforth. If we perform the POVM $\scrA=\{A_j\}_j$ on the quantum state $\rho$, then the probability of obtaining outcome $j$ is $\tr(\rho A_j)$ according to the Born rule. Given two POVMs $\scrA=\{A_j\}_j$ and $\scrB=\{B_k\}_k$ on $\caH$, $\scrA$ is a \textit{coarse graining} of $\scrB$ if it can be realized by performing $\scrB$ followed by data processing \cite{MartM90,Zhu22}. In other words, the POVM elements $A_j$ of $\scrA$ can be expressed as follows, \begin{equation} A_j = \sum_{B_k\in\scrB}\Lambda_{jk}B_k \quad \forall A_j\in\scrA, \end{equation} where $\Lambda$ is a stochastic matrix satisfying $\Lambda_{jk}\geq 0$ and $\sum_j \Lambda_{jk} = 1$. A convex combination of $\scrA$ and $\scrB$ is the disjoint union of $\{wA_j\}_j$ and $\{(1-w)B_k\}_k$ with $0\leq w\leq 1$. Convex combinations of three or more POVMs can be defined in a similar way. \subsection{Separable and collective measurements} Next, we turn to quantum states and POVMs on a bipartite system shared by Alice and Bob, where the total Hilbert space is a tensor product of the form $\caH_\rmT=\caH_\rmA\otimes \caH_\rmB$. A quantum state $\rho$ on $\caH_\rmT$ is a product state if it is a tensor product of two states on $\caH_\rmA$ and $\caH_\rmB$, respectively. The state $\rho$ is separable if it can be expressed as a convex sum of product states; otherwise, it is entangled. Note that a pure state on $\caH_\rmT$ is separable if and only if (iff) it is a product state. A positive operator on $\caH_\rmT$ is separable if it is proportional to a separable state. A POVM on $\caH_\rmT$ is separable if every POVM element is separable. Let $\scrA=\{A_j\}_j$ and $\scrB=\{B_k\}_k$ be two POVMs on $\caH_\rmA$ and $\caH_\rmB$, respectively. The tensor product of $\scrA$ and $\scrB$ is defined as $\scrA\otimes \scrB:=\{A_j\otimes B_k\}_{j,k}$. Such product POVMs are prominent examples of separable POVMs, but there are more interesting examples. A POVM is $\rmA \rightarrow \rmB$ one-way adaptive if it has the form $\{A'_j\otimes B_{jk}'\}_{j,k}$, where $\scrA'=\{A_j'\}_j$ is a POVM on $\caH_\rmA$ and $\scrB_j'=\{B_{jk}'\}_k$ for each $j$ is a POVM on $\caH_\rmB$. Such a POVM can be realized by first performing the POVM $\scrA'$ on $\caH_\rmA$ and then performing the POVM $\scrB_j'$ on $\caH_\rmB$ if the first measurement yields outcome $j$. \subsection{Biseparable and genuine collective measurements} Next, we turn to a tripartite quantum system, which represents the simplest setting that can manifest multivariate quantum correlations. Although multipartite quantum correlations in quantum states have been studied by numerous researchers, the counterparts in quantum measurements remain uncharted. Now, the total Hilbert space is a tensor product of the form $\caH_\rmT=\caH_\rmA\otimes \caH_\rmB\otimes \caH_\rmC$, which is shared by Alice, Bob, and Charlie. A quantum state on $\caH_\rmT$ is biseparable with respect to the bipartition $\rmA\rmB|\rmC$ if $\rho$ is separable when $\rmA\rmB$ is regarded as whole. Biseparable states with respect to the bipartition $\rmA\rmC|\rmB$ or $\rmB\rmC|\rmA$ can be defined in a similar way. The quantum state $\rho$ is biseparable if it can be expressed as a convex sum of three states $\rho_{\rmA\rmB|\rmC}$, $\rho_{\rmA\rmC|\rmB}$, and $\rho_{\rmB\rmC|\rmA}$, which are biseparable with respect to three bipartitions, respectively. Otherwise, the state $\rho$ has genuine multipartite entanglement. By contrast, a POVM $\scrM=\{M_j\}_j$ on $\caH_\rmT$ is biseparable with respect to the bipartition $\rmA\rmB|\rmC$ if every POVM element $M_j$ is biseparable with respect to the bipartition, which means $\scrM$ is separable when $\rmA\rmB$ is regarded as whole. The POVM is 2+1 adaptive (with respect to the bipartition $\rmA\rmB|\rmC$) if it can be realized by one-way communication from $\rmA\rmB$ to $\rmC$; it is 1+2 adaptive if it can be realized by one-way communication from $\rmC$ to $\rmA\rmB$; see \fref{fig:POVMRC} for an illustration. Generalization to other bipartitions is immediate. The POVM $\scrM$ is biseparable if it can be expressed as a coarse graining of a convex combination of three POVMs $\scrM_{\rmA\rmB|\rmC}$, $\scrM_{\rmA\rmC|\rmB}$, and $\scrM_{\rmB\rmC|\rmA}$, which are biseparable with respect to three bipartitions, respectively. Otherwise, the POVM $\scrM$ is genuinely collective. \begin{figure}[tbp] \centering \includegraphics[width = 0.45\textwidth]{biseparable.pdf} \caption{Three types of biseparable measurements on three qubits: two-way adaptive (a), 2+1 adaptive (b), and 1+2 adaptive (c). Note that (a) contains (b) and (c) as special cases.} \label{fig:POVMRC} \end{figure} \section{Optimal quantum state estimation} \label{sec:state_estimation} \subsection{A simple estimation problem and estimation fidelity} Here we reexamine an old estimation problem: A quantum device produces $N$ copies of a Haar random pure state $\rho=|\psi\>\<\psi|$ on $\caH$, where $\caH$ has dimension~$d$, and our task is to estimate the identity of $\rho$ based on quantum measurements \cite{MassP95,DerkBE98,LatoPT98,Haya98,BrusM99,GisiP99,Mass00,AcinLP00,BagaBM02,HayaHH05}. The performance of an estimation protocol is quantified by the average fidelity. Suppose we perform a POVM $\scrM=\{M_j\}_j$ on $\rho^{\otimes N}$, then the probability of obtaining outcome $j$ reads $p_j = \tr(M_j \rho^{\otimes N})$. If we choose $\hat{\rho}_j$ as the estimator associated with outcome $j$, then the average estimation fidelity achieved by this protocol reads \begin{equation} \overline{F} = \sum_j \int_{\text{Haar}}d\psi \tr\bigl[(|\psi\>\<\psi|)^{\otimes N}M_j\bigr]\<\psi|\hat{\rho}_j|\psi\>, \end{equation} where the integral means taking the average over the ensemble of Haar random pure states. Let $\Sym_N(\caH)$ be the symmetric subspace in $\caH^{\otimes N}$ and $P_N$ the projector onto $\Sym_N(\caH)$. Define \begin{align} \caQ(M_j) &:= (N+1)!\tr_{1,2,\ldots, N}[P_{N+1}(M_j\otimes I)], \label{eq:Qmap}\\ F(\scrM)&:= \sum_j\frac{\|\caQ(M_j)\|}{d(d+1)\cdots (d+N)},\label{eq:EstimationFidDef} \end{align} where $\|\cdot\|$ is the spectral norm. Then $\overline{F}\leq F(\scrM)$, and the inequality is saturated if each $\hat{\rho}_j$ is supported in the eigenspace of $\caQ(M_j)$ with the maximum eigenvalue by \rcite{Zhu22}. In view of this fact, $F(\scrM)$ is called the estimation fidelity of $\scrM$ henceforth. The definition of the estimation fidelity is still applicable when $\scrM$ is an incomplete POVM, which means $\sum_j M_j\leq I^{\otimes N}$. If there is no restriction on the POVMs that can be performed, then the maximum estimation fidelity is $(N+1)/(N+d)$, and optimal POVMs can be constructed from $t$-designs with $t=N$ \cite{MassP95,BrusM99,HayaHH05,Zhu22}. \subsection{Properties of the map $\caQ$ and estimation fidelity} The basic properties of the map $\caQ$ and estimation fidelity $F(\scrM)$ are clarified in \rcite{Zhu22}. Here we introduce some additional results that are relevant to the following discussion. Note that the argument of $\caQ$ is not restricted to POVM elements and is not necessarily Hermitian. In analogy to $P_N$, let $P_N^\rmA$ be the projector onto the antisymmetric subspace in $\caH^{\otimes N}$. Let $\caS_N$ be the symmetric group of the $N$ parties associated with the $N$ copies of $\rho$. For each $\sigma\in \caS_N$, let $\bbW_\sigma$ be the unitary operator on $\caH^{\otimes N}$ tied to the permutation $\sigma$. Then \begin{align} P_N = \frac{1}{N!}\sum_{\sigma\in \caS_N}\bbW_\sigma,\; P^\rmA_N = \frac{1}{N!}\sum_{\sigma\in \caS_N}\sgn(\sigma)\bbW_\sigma, \end{align} where $\sgn(\sigma)=1$ when $\sigma$ is an even permutation and $\sgn(\sigma)=-1$ when $\sigma$ is an odd permutation. The following lemma is a simple corollary of the definition of $\caQ$ in \eref{eq:Qmap}. \begin{lemma}\label{lem:Q} Suppose $M\in \caL(\caH^{\otimes N})$; then \begin{gather} \label{eq:B1} \caQ(\bbW_\sigma M \bbW_\tau) = \caQ(M) \quad \forall \sigma,\tau \in \caS_N,\\ \label{eq:B2} \caQ[(P_{N-1}\otimes I)M(P_{N-1}\otimes I)] = \caQ(M). \end{gather} If in addition $N\geq 3$, then \begin{equation} \label{eq:B3} \caQ\bigl[\bigl(P_{N-1}^\rmA\otimes I\bigr)M\bigl(P_{N-1}^\rmA\otimes I\bigr)\bigr] =0. \end{equation} \end{lemma} Given any POVM $\scrM=\{M_j\}_j$ on $\caH^{\otimes N}$, define \begin{equation}\label{eq:POVMsymAsym} \begin{aligned} \scrM_\rmS&:=\{(P_{N-1}\otimes I) M_j (P_{N-1}\otimes I)\}_j,\\ \scrM_\rmA&:=\bigl\{\bigl(P_{N-1}^\rmA \otimes I\bigr) M_j \bigl(P_{N-1}^\rmA \otimes I\bigr)\bigr\}_j. \end{aligned} \end{equation} Here $\scrM_\rmS$ ($\scrM_\rmA$) is called the symmetric (antisymmetric) part of $\scrM$. The following lemma is a simple corollary of \lref{lem:Q}. \begin{lemma}\label{lem:POVMsym} Suppose $\scrM$ is a POVM on $\caH^{\otimes N}$. Then \begin{align} F(\scrM_\rmS)=F(\scrM),\quad F(\scrM_\rmA)=0. \label{eq:POVMsym} \end{align} \end{lemma} \section{\label{sec:CollectiveOpt}Optimal estimation of three parallel spins with genuine collective measurements} From now on we turn to the estimation of three parallel spins, which means $d=2$, $N=3$, and $\caH$ is a single-qubit Hilbert space. In the following discussion, we label the three copies of $\caH$ by $\rmA$, $\rmB$, and $\rmC$, respectively. To benchmark the performances of restricted collective measurements, we first reexamine an optimal estimation strategy when there is no restriction on the POVMs that can be performed. In this case, the maximum estimation fidelity is $4/5$, \cite{MassP95,LatoPT98}. Consider the three Pauli operators \begin{align} X=\begin{pmatrix} 0 &1\\ 1 &0 \end{pmatrix}, \;\; Y=\begin{pmatrix} 0 &-\rmi\\ \rmi &0 \end{pmatrix}, \;\; Z=\begin{pmatrix} 1 &0\\ 0 &-1 \end{pmatrix}, \end{align} and let $|\psi_j\>$ for $j=1,2,...,6$ be the six eigenstates, which form a regular octahedron when represented on the Bloch sphere. Then an optimal POVM $\scrM_\col$ can be constructed from the following seven POVM elements \cite{LatoPT98}, \begin{equation}\label{eq:POVMcol} \begin{aligned} E_j &:= \frac{2}{3}|\psi_j\>\<\psi_j|^{\otimes 3},\quad j = 1,2,\ldots,6,\\ E_7 &:= I^{\otimes 3} - \sum_{j=1}^6 E_j = I^{\otimes 3} - P_3, \end{aligned} \end{equation} where $P_3$ is the projector onto $\Sym_3(\caH)$. Although this POVM was constructed more than 20 years ago, its intriguing properties have not been fully appreciated. Let $\Pi=P_2^\rmA\otimes I$ and let $\bbW$ be the unitary operator on $\caH^{\otimes 3}$ that is associated with a cyclic permutation. Then $\bbW^2=\bbW^\dag$ and the POVM element $E_7$ can be expressed as follows, \begin{align} E_7=\frac{2}{3}\bigl(\Pi +\bbW \Pi\bbW^\dag + \bbW^\dag \Pi\bbW \bigr), \end{align} which means $E_7$ is biseparable. So all POVM elements in the optimal POVM $\scrM_\col$ are biseparable. However, the POVM $\scrM_\col$ itself is not biseparable as shown in the following proposition and proved in \aref{app:POVMgcolProof}. \begin{proposition}\label{pro:POVMgcol} The POVM $\scrM_\col$ defined in \eref{eq:POVMcol} is genuinely collective. \end{proposition} Collective measurements are in general not easy to realize. If we can only perform local measurements on individual copies, then the maximum estimation fidelity is $(3 +\sqrt{3}\lsp)/6$, and the maximum can be attained when Alice, Bob, and Charlie perform Pauli $X$, $Y$, and $Z$ measurements, respectively. \section{\label{sec:RestrictedOpt}Optimal estimation of three parallel spins with restricted collective measurements} Although the maximum estimation fidelity of general collective measurements has been well known for a long time, the performances of restricted collective measurements are still poorly understood. To fill this gap, here we shall determine the maximum estimation fidelities of biseparable measurements, 2+1 adaptive measurements, and 1+2 adaptive measurements, respectively, and construct optimal estimation strategies explicitly. It turns out 2+1 adaptive measurements can achieve the same estimation fidelity as biseparable measurements. By symmetry, the maximum estimation fidelity of biseparable POVMs with respect to the bipartition $\rmA\rmB|\rmC$ is identical to the counterpart with respect to the bipartition $\rmA\rmC|\rmB$ and the counterpart with respect to the bipartition $\rmB\rmC|\rmA$. Moreover, this maximum estimation fidelity is also the maximum estimation fidelity of general biseparable POVMs, given that coarse graining cannot increase the estimation fidelity \cite{Zhu22}. In addition, it suffices to consider rank-1 POVMs to determine the maximum estimation fidelity. Therefore, we can focus on rank-1 POVMs that are biseparable with respect to the bipartition $\rmA\rmB|\rmC$ in the following discussion. Furthermore, the maximum estimation fidelity of biseparable POVMs on $\caH^{\otimes 2}\otimes \caH$ is identical to the maximum estimation fidelity of separable POVMs on $\Sym_2(\caH)\otimes \caH$ thanks to \lref{lem:POVMsym}. If $\scrM$ is an optimal biseparable POVM on $\caH^{\otimes 2}\otimes \caH$, then $\scrM_\rmS$ defined in \eref{eq:POVMsymAsym} is an optimal separable POVM on $\Sym_2(\caH)\otimes \caH$. Conversely, if $\scrM$ is an optimal separable POVM on $\Sym_2(\caH)\otimes \caH$, then an optimal biseparable POVM on $\caH^{\otimes 2}\otimes \caH$ can be constructed as follows, \begin{align} \scrM\cup \bigl\{P_2^\rmA\otimes I\bigr\}. \end{align} Similar remarks apply to 2+1 adaptive POVMs and 1+2 adaptive POVMs. \subsection{Biseparable measurements} According to the previous discussion, to determine the maximum estimation fidelity of biseparable POVMs on $\caH^{\otimes 3}$, it suffices to consider separable rank-1 POVMs on $\Sym_2(\caH)\otimes \caH$. Suppose $\scrM=\{M_j\}_j$ is such a POVM, then $M_j$ have the form \begin{align}\label{eq:povm} M_j=w_j|\Psi_j\>\<\Psi_j|,\quad |\Psi_j\>= |\Phi_j\>\otimes|\varphi_j\>, \end{align} where $|\Phi_j\>\in \Sym_2(\caH)$ and $|\varphi_j\>\in \caH$. In addition, \begin{equation}\label{eq:POVMbs} \begin{gathered} w_j > 0, \quad \sum_j w_j =6,\\ \sum_j w_j|\Phi_j\>\<\Phi_j|\otimes|\varphi_j\>\<\varphi_j| = P_2\otimes I, \end{gathered} \end{equation} where $P_2$ is the projector onto $\Sym_2(\caH)$. In conjunction with \eref{eq:EstimationFidDef}, the maximum estimation fidelity of biseparable POVMs can be expressed as follows, \begin{gather} F_\bs = \max_{\scrM} \frac{1}{120}\sum_j w_j \|\caQ(|\Phi_j\>\<\Phi_j|\otimes|\varphi_j\>\<\varphi_j|)\|,\label{eq:BisepOpt} \end{gather} where the maximization is subjected to the constraints in \eref{eq:POVMbs}. Based on \eqsref{eq:POVMbs}{eq:BisepOpt} we can derive an analytical formula for the maximum estimation fidelity, as shown in \thref{thm:bisep} below and proved in \aref{app:bisepProof}. Note that biseparable measurements can achieve a higher estimation fidelity than local measurements, but there is a fundamental gap from genuine collective measurements, as summarized in \tref{tab:summary}. \begin{theorem} \label{thm:bisep} In the estimation of three parallel spins, the maximum estimation fidelity of biseparable POVMs reads \begin{equation} F_\bs=\frac{1}{2} + \frac{\sqrt{22}}{16}. \end{equation} \end{theorem} \begin{table}[htbp] \centering \caption{\label{tab:summary}Maximum estimation fidelities of four different types of measurements.} \renewcommand\arraystretch{2} \begin{tabular}{ c | c } \hline \hline Measurement & Maximum estimation fidelity \\ \hline \hline Local & $\frac{1}{2} + \frac{\sqrt{3}}{6} \approx 0.78868$ \cite{BagaMM05}\\ \hline 1+2 adaptive & $\frac{1}{2} + \frac{11 + \sqrt{41}}{60} \approx 0.79005$ \\ \hline \makecell{Biseparable\\ (or 2+1 adaptive)} & $\frac{1}{2} + \frac{\sqrt{22}}{16} \approx 0.79315$ \\ \hline Genuine collective & $\frac{4}{5}$ \cite{MassP95}\\ \hline \hline \end{tabular} \end{table} \subsection{2+1 adaptive measurements} Here we show that 2+1 adaptive measurements can achieve the same estimation fidelity as biseparable measurements. \begin{theorem}\label{thm:2+1} In the estimation of three parallel spins, the maximum estimation fidelity of 2+1 adaptive POVMs is \begin{equation}\label{eq:2+1} F_{2\rightarrow 1}= \frac{1}{2} + \frac{\sqrt{22}}{16}. \end{equation} \end{theorem} Since 2+1 adaptive POVMs are biseparable, to prove \thref{thm:2+1}, it suffices to construct a 2+1 adaptive POVM that can attain the estimation fidelity in \eref{eq:2+1}. A direct proof can be found in \aref{app:2+1}. Our construction is based on a subgroup of the single-qubit Clifford group $\Cl_1$. Recall that the Pauli group is generated by the three Pauli operators $X,Y,Z$. The Clifford group $\Cl_1$ is the normalizer of the Pauli group. Up to overall phase factors, it is generated by the Hadamard gate $H$ and phase gate~$S$, where \begin{align}\label{eq:HS} H=\frac{1}{\sqrt{2}}\begin{pmatrix} 1 &1\\ 1 &-1 \end{pmatrix} , \quad S=\begin{pmatrix} 1 &0\\ 0 &\rmi \end{pmatrix}. \end{align} Let $V=HS$ and let $\caG$ be the group generated by $X$ and $V$, then $\caG$ is a subgroup of $\Cl_1$ that contains the Pauli group. Let \begin{align} \overline{\caG}:=\bigl\{I, V, V^2\bigr\}\times \{I, X, Y, Z\}. \end{align} Then $\overline{\caG}$ can be identified as the quotient group of $\caG$ after identifying operators that are proportional to each other. Define \begin{equation} \begin{aligned} |\tPhi\> &:= \frac{\sqrt{8+3\sqrt{7}}}{4} |00\> + \frac{\sqrt{8 - 3\sqrt{7}}}{4}|11\>,\\ |\tPsi\> &:=|\tPhi\>\otimes |+\>, \end{aligned} \end{equation} where $|+\>=(|0\>+|1\>)/\sqrt{2}$ is the eigenstate of $X$ with eigenvalue 1. Then an optimal biseparable POVM can be constructed as follows, \begin{align}\label{eq:OptPOVMbs2+1} \scrM_\bs:=\Bigl\{\frac{1}{2}U^{\otimes 3}|\tPsi\>\<\tPsi|U^{\dag\otimes 3}\,\Big|\, U\in \overline{\caG} \Bigr\}\cup \bigl\{P_2^\rmA\otimes I\bigr\}. \end{align} Moreover, this optimal POVM can be realized by a 2+1 adaptive strategy. To be specific, let \begin{equation} \begin{aligned} \!\!\scrM_2&:= \Bigl\{\frac{1}{2}U^{\otimes 2}|\tPhi\>\<\tPhi|U^{\dag\otimes 2}\Big| U\in \overline{\caG}_2\Bigr\}\cup\bigl\{P_2^\rmA\bigr\},\\ \overline{\caG}_2&:=\bigl\{I, V, V^2\bigr\}\times\{I,X\}; \end{aligned} \end{equation} then $\scrM_2$ is a POVM on $\caH^{\otimes 2}$. Now, $\scrM_\bs$ can be realized as follows: Alice and Bob first perform the POVM $\scrM_2$ and send the outcome to Charlie; if they obtain outcome $U^{\otimes 2}|\tPhi\>\<\tPhi|U^{\dag\otimes 2}/2$ (note that the outcome $P_2^\rmA$ can never occur), then Charlie performs the projective measurement on the eigenbasis of $UXU^\dag$. \subsection{1+2 adaptive measurements} In this section, we determine the maximum estimation fidelity of 1+2 adaptive measurements and devise an optimal strategy. \begin{theorem} \label{thm:1+2} In the estimation of three parallel spins, the maximum estimation fidelity of 1+2 adaptive POVMs is \begin{equation}\label{eq:1+2} F_{1\rightarrow 2}= \frac{1}{2} + \frac{11 + \sqrt{41}}{60}. \end{equation} \end{theorem} \Thref{thm:1+2} is proved in \aref{app:1+2}. Here we construct an optimal POVM that can attain the maximum estimation fidelity in \eref{eq:1+2}. Let \begin{equation}\label{eq:pSPhiW} \begin{aligned} p & := \frac{47 - 3\sqrt{41}}{216},\quad |\bbS\> :=\frac{|01\> + |10\>}{\sqrt{2}}, \\ |\Upsilon\> &:= \sqrt{\frac{1 - 3p}{3-3p}}\,|00\> + \sqrt{\frac{1}{3-3p}}\left(|\bbS\> + |11\>\right),\\ W_j &:=(S\otimes S)^j,\quad j=0,1,2,3, \end{aligned} \end{equation} where $S$ is the phase gate defined in \eref{eq:HS}. Then we can construct two POVMs $\scrK_0, \scrK_1$ on $\caH^{\otimes 2}$ as follows, \begin{equation}\label{eq:scrE01} \begin{aligned} K_j&:=\frac{3-3p}{4}W_j|\Upsilon\>\<\Upsilon|W_j^\dag,\quad j=0,1,2,3,\\ K_4 &:=3p |00\>\< 00|,\quad K_5:=P_2^\rmA,\\ \scrK_0 & := \{K_j\}_{j=0}^5,\quad \scrK_1:=\bigl\{X^{\otimes 2}K_jX^{\otimes 2}\bigr\}_{j=0}^5. \end{aligned} \end{equation} On this basis, we can construct an optimal 1+2 adaptive POVM, \begin{align}\label{eq:OptPOVM1+2} \scrM_{1\rightarrow 2} :=\{\scrK_0\otimes |0\>\<0|\}\cup \{\scrK_1\otimes |1\>\<1|\}. \end{align} This POVM can be realized as follows: Charlie first performs the $Z$-basis measurement on his qubit and sends the measurement outcome to Alice and Bob. If the outcome is 0, then Alice and Bob perform the POVM $\scrK_0$ on their qubits; if the outcome is 1, then they perform the POVM $\scrK_1$ instead. \section{Summary} \label{sec:Summary} In this work we introduced rigorous definitions of biseparable measurements and genuine collective measurements, thereby setting the stage for exploring the rich territory of multipartite nonclassicality in quantum measurements instead of quantum states. By virtue of a simple estimation problem, we established a rigorous fidelity gap between biseparable measurements and collective measurements. Moreover, we showed that the maximum estimation fidelity of biseparable measurements can be attained by 2+1 adaptive measurements, but not by 1+2 adaptive measurements. Optimal estimation protocols in all these settings are constructed explicitly. Our work shows that quantum measurements in the multipartite setting may exhibit a rich hierarchy of nonclassical phenomena, which offer exciting opportunities for future studies. \section*{Acknowledgments} The work at Fudan University is supported by the National Natural Science Foundation of China (Grant No.~92165109), National Key Research and Development Program of China (Grant No.~2022YFA1404204), and Shanghai Municipal Science and Technology Major Project (Grant No.~2019SHZDZX01). The work at the University of Science and Technology of China is supported by the Innovation Program for Quantum Science and Technology (Grant No.~2023ZD0301400), the National Natural Science Foundation of China (Grants Nos. 62222512, 12104439, and 12134014), the Anhui Provincial Natural Science Foundation (Grant No.~2208085J03), and USTC Research Funds of the Double First-Class Initiative (Grant Nos.~YD2030002007 and YD2030002011). \onecolumngrid \bigskip \appendix \section*{Appendix} In this Appendix we prove the key results on optimal estimation of three parallel spins presented in the main text, namely, \pref{pro:POVMgcol} and \thsref{thm:bisep}-\ref{thm:1+2}. Throughout this appendix, we assume that $\caH$ is a single-qubit Hilbert space. \section{\label{app:POVMgcolProof}Proof of \pref{pro:POVMgcol}} \begin{proof}[Proof of \pref{pro:POVMgcol}] Suppose on the contrary that $\scrM_\col$ is biseparable. Then $\scrM_\col$ can be expressed as a coarse graining of three POVMs $\scrM_{\rmA\rmB|\rmC}$, $\scrM_{\rmA\rmC|\rmB}$, and $\scrM_{\rmB\rmC|\rmA}$, which are biseparable with respect to three bipartitions, respectively. Without loss of generality, we can assume that the three POVMs are all rank-1. Note that every POVM element in $\scrM_\col$ is supported either in $\Sym_3(\caH)$ or in its orthogonal complement $\Sym_3(\caH)^\perp$, so every POVM element in $\scrM_{\rmA\rmB|\rmC}$ is also supported either in $\Sym_3(\caH)$ or in $\Sym_3(\caH)^\perp$. Let $\scrM_{\rmA\rmB|\rmC}'$ be the subset of POVM elements in $\scrM_{\rmA\rmB|\rmC}$ that are supported in $\Sym_3(\caH)^\perp$; then $\scrM_{\rmA\rmB|\rmC}'$ forms a POVM on $\Sym_3(\caH)^\perp$, which means \begin{align}\label{eq:POVMgcolProof} \sum_{M\in \scrM_{\rmA\rmB|\rmC}'}M=I^{\otimes 3}-P_3. \end{align} On the other hand, by assumption each POVM element in $\scrM_{\rmA\rmB|\rmC}'$ has the form $w_j|\Phi_j\>\<\Phi_j|\otimes |\varphi_j\>\<\varphi_j|$, where $w_j\geq 0$, $|\Phi_j\>\in \caH^{\otimes 2}$, and $|\varphi_j\>\in\caH$. According to \lref{lem:P3orthogonal} below, we have $|\Phi_j\>\<\Phi_j|=P_2^\rmA$, so every POVM element in $\scrM_{\rmA\rmB|\rmC}'$ is supported in $\Sym_2(\caH)^\perp\otimes \caH$, which has dimension 2. Consequently, $\sum_{M\in \scrM_{\rmA\rmB|\rmC}'}M$ has rank at most 2, which contradicts \eref{eq:POVMgcolProof}. This contradiction shows that $\scrM_\col$ cannot be biseparable and is thus genuinely collective. \end{proof} Next, we prove an auxiliary lemma employed in the proof of \pref{pro:POVMgcol}. \begin{lemma}\label{lem:P3orthogonal} Suppose $\caH$ has dimension 2, $|\Phi\>\in \caH^{\otimes 2}$, and $|\varphi\>\in\caH$. Then $\tr[P_3(|\Phi\>\<\Phi|\otimes |\varphi\>\<\varphi|)]=0$ iff $|\Phi\>\<\Phi|=P_2^\rmA$. \end{lemma} \begin{proof} Let $\bbW_{(12)}$ be the swap operator acting on the first two parties. If $|\Phi\>\<\Phi|=P_2^\rmA$, then \begin{equation} \tr[P_3(|\Phi\>\<\Phi|\otimes |\varphi\>\<\varphi|)]=\tr\bigl[P_3\bbW_{(12)}(|\Phi\>\<\Phi|\otimes |\varphi\>\<\varphi|)\bigr]=-\tr[P_3(|\Phi\>\<\Phi|\otimes |\varphi\>\<\varphi|)], \end{equation} which implies that $\tr[P_3(|\Phi\>\<\Phi|\otimes |\varphi\>\<\varphi|)]=0$. Next, suppose $\tr[P_3(|\Phi\>\<\Phi|\otimes |\varphi\>\<\varphi|)]=0$. Let $\rmU(\caH)$ be the group of all unitary operators on $\caH$. Choose $U\in \rmU(\caH)$ such that $U|\varphi\>=|0\>$ and let $|\Phi'\>=U^{\otimes 2}|\Phi\>$. Then \begin{align} 0&=\tr[P_3(|\Phi\>\<\Phi|\otimes |\varphi\>\<\varphi|)]=\tr\bigl[P_3U^{\otimes 3}(|\Phi\>\<\Phi|\otimes |\varphi\>\<\varphi|)U^{\dag\otimes 3}\bigr]=\tr[P_3(|\Phi'\>\<\Phi'|\otimes |0\>\<0|)]\nonumber\\ &=\tr(|\Phi'\>\<\Phi'| R), \end{align} where \begin{align} R=\tr_\rmC[P_3(I\otimes I\otimes |0\>\<0|)]=|00\>\<00|+\frac{2|\bbS\>\<\bbS|}{3}+\frac{|11\>\<11|}{3}, \end{align} with $|\bbS\>=(|01\> + |10\>)/\sqrt{2}$ as defined in \eref{eq:pSPhiW}. Note that $R$ has full support in $\Sym_2(\caH)$, so $|\Phi'\>$ is necessarily supported in the orthogonal complement of $\Sym_2(\caH)$, which is spanned by $(|01\>-|10\>)/\sqrt{2}$. Therefore, $|\Phi\>\<\Phi|=|\Phi'\>\<\Phi'|=P_2^\rmA$, which completes the proof of \lref{lem:P3orthogonal}. \end{proof} \section{Proof of \thref{thm:bisep}} \label{app:bisepProof} \subsection{Auxiliary results} Suppose $a,b,c,x,y,\phi$ are real numbers. Define \begin{align} \eta(a,b,c,\phi)&:= 15 a^2 + 10b^2 + 5c^2 + \sqrt{8b^2|3a + 2c\rme^{\rmi\phi}|^2 + (9a^2 + 2b^2 - c^2)^2}, \label{eq:etaabcphiDef}\\ \eta(a,b,c)&:= 15 a^2 + 10b^2 + 5c^2 + \sqrt{8b^2(3|a| + 2|c|)^2 + (9a^2 + 2b^2 - c^2)^2},\\ f(x,y)& := \sqrt{4(1-x)(3\sqrt{x + y} + 2\sqrt{x - y}\lsp)^2 + (2x + 5y + 2)^2}. \label{eq:fxy} \end{align} Given any state $|\Psi \>$ in $\caH^{\otimes 3}$, define \begin{align} \caI_\Psi := \int_{\mathrm{Haar}}\rmd U U^{\otimes 3} |\Psi\>\<\Psi| U^{\dag\otimes 3}, \label{eq:IpsiDef} \end{align} where the integration is taken over the normalized Haar measure on the unitary group $\rmU(\caH)$. Alternatively, the integration can be replaced by summation over a unitary 3-design. Recall that a set of unitaries $\caU = \{U_k\}_k$ is a unitary $t$-design \cite{GrosAE07} if the following equation holds \begin{equation} \frac{1}{|\caU|}\sum_{U_k\in\caU}U_k^{\otimes t}O U_k^{\dag\otimes t} = \int_{\mathrm{Haar}} dU U^{\otimes t}O U^{\dag\otimes t}\quad \forall O\in \caL\bigl(\caH^{\otimes t}\bigr). \end{equation} \begin{lemma}\label{lem:q_abc} Suppose $|\Phi \> = a|00\> + b|\bbS\> + c\rme^{\rmi\phi}|11\>\in \caH^{\otimes 2}$ and $|\Psi\>=|\Phi\>\otimes |0\>\in \caH^{\otimes 3}$, where $a,b,c,\phi$ are real numbers and $|\bbS\>=(|01\>+|10\>)/\sqrt{2}$. Then \begin{gather} 6\tr(P_3 |\Psi\>\<\Psi|) = 6a^2+4b^2+2c^2, \label{eq:P3Psi0}\\ \caI_\Psi = \frac{3a^2+2b^2+c^2}{12}P_3+\frac{b^2+2c^2}{6}(P_2\otimes I-P_3), \label{eq:IPsi} \\ \|\caQ(|\Psi\>\<\Psi|)\| = \eta(a,b,c,\phi)\le \eta(a,b,c)=15 a^2 + 10b^2 + 5c^2 + f\bigl(a^2+c^2, a^2-c^2\bigr), \label{eq:QPsinorm2} \end{gather} and the last inequality is saturated when $\phi=0$. \end{lemma} When $a^2 = c^2$, \eref{eq:IPsi} implies that \begin{equation}\label{eq:a2c2} \caI_\Psi = \frac{1}{6}P_2\otimes I. \end{equation} In this case, if $\{U_k\}_{k=1}^m$ is a unitary 3-design, then $\bigl\{6 U_k|\Psi\>\<\Psi|U_k^\dag/m \bigr\}_{k=1}^m$ is a POVM on $\Sym_2(\caH)\otimes \caH$. This conclusion will be useful to constructing optimal biseparable POVMs. \begin{proof}[Proof of \lref{lem:q_abc}] \Eref{eq:P3Psi0} can be verified by straightforward calculation. By Schur-Weyl duality, $\caI_\Psi$ is a linear combination of $\bbW_\sigma$ with $\sigma\in \caS_3$. In addition $\caI_\Psi=\bbW_{(12)}\caI_\Psi= \caI_\Psi\bbW_{(12)}$, where $(1 2)$ denotes the transposition of the first two parties. So $\caI_\Psi$ can only be a linear combination of $P_3$ and $P_2\otimes I$. Now \eref{eq:IPsi} is a simple corollary of \eref{eq:P3Psi0}. Next, straightforward calculation yields \begin{align} \caQ(|\Psi\>\<\Psi|) &= \bigl(15a^2 + 10b^2 +5c^2\bigr)I + 2\sqrt{2}\lsp b[3a + 2c \cos (\phi)] X + 4\sqrt{2}\lsp b c \sin(\phi) Y + \bigl(9a^2 + 2b^2 - c^2\bigr)Z, \end{align} which implies \eref{eq:QPsinorm2} given the definitions in Eqs.~\eqref{eq:etaabcphiDef}-\eqref{eq:fxy}. \end{proof} \begin{lemma}\label{lem:fxyUB} Suppose $0 \le x \le 1$ and $-x \le y \le x$. Then the function $f(x,y)$ defined in \eref{eq:fxy} satisfies \begin{equation}\label{eq:fxyUB} f(x,y) \le \frac{5}{2}\sqrt{\frac{11}{2}} + 8\sqrt{\frac{2}{11}}\,y; \end{equation} and the inequality is saturated iff $x=9/16$ and $y=0$. \end{lemma} \begin{proof} Due to continuity, it suffices to prove \eref{eq:fxyUB} when $0<x<1$ and $-x<y<x$. Direct calculation yields \begin{gather} \frac{5}{2}\sqrt{\frac{11}{2}} + 8\sqrt{\frac{2}{11}}\,y\geq \frac{5}{2}\sqrt{\frac{11}{2}} - 8\sqrt{\frac{2}{11}}>0,\quad \left(\frac{5}{2}\sqrt{\frac{11}{2}} + 8\sqrt{\frac{2}{11}}\,y\right)^2 - f^2(x,y) = r(x,z), \end{gather} where $z := y^2<x^2$ and \begin{equation} r(x,z): = \frac{243}{8} + 48x^2 - \frac{147z}{11} + 48(x-1)\sqrt{x^2 - z} - 60x. \end{equation} The partial derivative of $r(x,z)$ over $z$ reads \begin{equation} \frac{\partial r}{\partial z} = \frac{24(1-x)}{\sqrt{x^2 - z}} - \frac{147}{11}. \end{equation} If $0<x< 88/137$, then this derivative is always positive. Therefore, \begin{equation}\label{eq:fxyUBproof} r(x,z) \ge r(x,0) = \frac{3}{8}(9-16x)^2\geq 0, \end{equation} which implies \eref{eq:fxyUB}. If instead $88/137\leq x< 1$, then the partial derivative $\partial r/\partial z$ has a unique zero, denoted by $z_0$ henceforth. In addition, $z_0$ satisfies the equation \begin{equation} \sqrt{x^2 - z_0} = \frac{264}{147}(1-x), \end{equation} which means \begin{align} z_0=\frac{-5343x^2+15488x-7744}{2401}. \end{align} Therefore, \begin{equation} r(x,z) \ge r(x,z_0)=-\frac{3(12168x^2-37664x+18293)}{4312}\geq \frac{91875}{150152}>0, \end{equation} which implies \eref{eq:fxyUB}. Here the last inequality follows from the assumption $88/137\leq x< 1$. If the inequality in \eref{eq:fxyUB} is saturated, then $0<x< 88/137$ and the two inequalities in \eref{eq:fxyUBproof} are saturated simultaneously, which means $x=9/16$ and $y=z=0$, in which case the inequality in \eref{eq:fxyUB} is indeed saturated. \end{proof} \subsection{Proof of \thref{thm:bisep}} To determine the maximum estimation fidelity of biseparable POVMs on $\caH^{\otimes 3}$, it suffices to consider separable rank-1 POVMs on $\Sym_2(\caH)\otimes \caH$. More specifically, it suffices to solve the optimization problem in \eref{eq:BisepOpt} subject to the constraints in \eref{eq:POVMbs}. Note that $|\Psi_j\> =|\Phi_j\>\otimes |\varphi_j\>$ can be expressed as follows, \begin{align} |\Psi_j\>=U_j^{\otimes 3} \bigl[\bigl(a_j|00\> + b_j|\bbS\> + c_j\rme^{\rmi \phi_j}|11\>\bigr)\otimes |0\>\bigr], \end{align} where $U_j \in \rmU(\caH)$, $a_j, b_j,c_j\geq0$, $a_j^2+b_j^2+c_j^2=1$, and $0\leq\phi_j<2\pi$. By virtue of \lref{lem:q_abc} we can deduce that \begin{align} 4=\sum_j w_j\tr\left(P_3 |\Psi_j\>\<\Psi_j|\right)&=\sum_j \frac{w_j\bigl(3a_j^2+2b_j^2+c_j^2\bigr)}{3}=\sum_j\frac{w_j\bigl(2+a_j^2-c_j^2\bigr)}{3}=4+\sum_j \frac{w_j\bigl(a_j^2-c_j^2\bigr)}{3}, \end{align} which means $\sum_j w_j a_j^2=\sum_j w_j c_j^2$. By virtue of \eref{eq:EstimationFidDef} and \lref{lem:q_abc} we can further deduce that \begin{align} F(\scrM)&=\frac{1}{120}\|\caQ(w_j|\Psi_j\>\<\Psi_j|)\|= \frac{1}{120}\sum_j w_j \eta(a_j,b_j,c_j,\phi_j)\leq \frac{1}{120}\sum_j w_j \eta(a_j,b_j,c_j) \nonumber\\ &=\frac{1}{20}\sum_{j} p_j[10 + 5y_j + f(x_j,y_j)]=\frac{1}{2}+\frac{1}{20}\sum_{j} p_jf(x_j,y_j), \label{eq:bsProof1} \end{align} where \begin{align} p_j :=\frac{w_j}{6},\quad x_j := a_j^2+c_j^2,\quad y_j := a_j^2-c_j^2, \quad \sum_j p_j=1,\quad \sum_j p_jy_j=0. \end{align} Now, in conjunction with \lref{lem:fxyUB}, we can conclude that \begin{align} F(\scrM)&\leq \frac{1}{2}+\frac{1}{20}\sum_{j} p_jf(x_j,y_j) \le \frac{1}{2}+\frac{1}{20}\sum_j p_j \left(\frac{5}{2}\sqrt{\frac{11}{2}} + 8\sqrt{\frac{2}{11}}\, y_j\right) = \frac{1}{2} + \frac{\sqrt{22}}{16}. \label{eq:bsProof2} \end{align} If the upper bound for $F(\scrM)$ in \eref{eq:bsProof2} is saturated, then the two inequalities are saturated simultaneously. The saturation of the second inequality in \eref{eq:bsProof2} means $x_j=9/16$ and $y_j=0$, that is, $a_j=c_j=3\sqrt{2}/8$ and $b_j =\sqrt{7}/4$, for all $j$ by \lref{lem:fxyUB}. The saturation of the first inequality in \eref{eq:bsProof2}, which means the saturation of the inequality in \eref{eq:bsProof1}, further implies that $\phi_j=0$ for all $j$ according to the definitions in Eqs.~\eqref{eq:etaabcphiDef}-\eqref{eq:fxy} (cf. \lref{lem:q_abc}). Conversely, if $a_j=c_j=3\sqrt{2}/8$, $b_j =\sqrt{7}/4$, and $\phi_j=0$ for all $j$, then $F(\scrM)$ can attain the upper bound in \eref{eq:bsProof2}. Suppose $\{U_j\}_{j=1}^m$ is a unitary 3-design (say the Clifford group \cite{Zhu17,Webb16}), $w_j=6/m$, and $|\Psi_j\>$ have the form \begin{equation} |\Psi_j\> = U_j^{\otimes 3}\left[\left(\frac{3\sqrt{2}}{8}|00\> + \frac{\sqrt{7}}{4}|\bbS\> + \frac{3\sqrt{2}}{8}|11\>\right)\otimes |0\>\right],\quad j=1,2,\ldots,m; \end{equation} then $\scrM=\{w_j |\Psi_j\>\<\Psi_j|\}_{j=1}^m$ is an optimal separable POVM on $\Sym_2(\caH)\otimes \caH$ that saturates the upper bound in \eref{eq:bsProof2}. Accordingly, $\scrM\cup \{P_2^\rmA\otimes I\} $ is an optimal biseparable POVM on $\caH^{\otimes 3}$. This observation completes the proof of \thref{thm:bisep}. \section{Direct proof of \thref{thm:2+1}} \label{app:2+1} \subsection{Auxiliary results} \begin{lemma}\label{lem:Sym2CanonicalForm} Suppose $|\Phi\>\in \Sym_2(\caH)$. Then there exists $U\in \rmU(\caH)$ and $\xi \in [0,\pi/2]$ such that \begin{equation}\label{eq:Sym2CanonicalForm} U^{\otimes 2}|\Phi\> =\cos\frac{\xi}{2}|00\> + \sin\frac{\xi}{2}|11\>. \end{equation} \end{lemma} \begin{proof} By assumption $|\Phi\>$ can be expressed as follows, \begin{equation} |\Phi\> = W^{\dag\otimes 2}\bigl(a|00\> + b|\bbS\> + c \rme^{\rmi\chi}|11\>\bigr),\quad a,b,c \ge 0,\ a^2 + b^2 + c^2=1, \ \chi \in [0,2\pi), \end{equation} where $W\in \rmU(\caH)$. Consider a unitary operator of the form \begin{equation} U_1(\theta,\phi) = \left(\begin{array}{cc} \cos\theta & \sin\theta \rme^{\rmi\phi} \\ -\sin\theta \rme^{-\rmi\phi} & \cos\theta \end{array}\right). \end{equation} Hence, we have \begin{equation} [U_1(\theta,\phi)W]^{\otimes 2}|\Phi\> = u(\theta,\phi)|00\> + v(\theta,\phi)|\bbS\> + w(\theta,\phi)|11\>,\\ \end{equation} where \begin{equation} \begin{aligned} u(\theta,\phi) &= a\cos^2\theta + \frac{1}{\sqrt{2}}b\rme^{\rmi\phi} \sin 2\theta + c\rme^{2\rmi(\phi + \chi)}\sin^2\theta,\\ v(\theta,\phi) &= b\cos(2\theta) + \frac{1}{\sqrt{2}}\left[c\rme^{\rmi(\phi + \chi)} - a\rme^{-\rmi\phi}\right]\sin(2\theta)\\ &=b \cos(2\theta) + \frac{1}{\sqrt{2}}\left[c \cos(\phi + \chi) - a \cos(\phi)\right] \sin(2\theta) + \frac{\rmi}{\sqrt{2}}\left[c \sin(\phi + \chi) + a \sin(\phi)\right] \sin(2\theta),\\ w(\theta,\phi) &= c\rme^{\rmi\chi}\cos^2\theta - \frac{1}{\sqrt{2}}b\rme^{-\rmi\phi}\sin(2\theta) + a\rme^{-\rmi2\phi}\sin^2\theta. \end{aligned} \end{equation} Let $\phi_0$ be a solution of the equation \begin{equation} c \sin(\phi_0 + \chi) + a \sin(\phi_0) = 0, \end{equation} and let $\theta_0$ be a solution of the equation \begin{equation} b \cos(2\theta_0) + \frac{1}{\sqrt{2}}\left[c \cos(\phi_0 + \chi) - a \cos\phi_0\right] \sin(2\theta_0) = 0. \end{equation} Then $v(\theta_0,\phi_0)=0$ and \begin{equation} [U_1(\theta_0,\phi_0)W]^{\otimes 2}|\Phi\> = u(\theta_0,\phi_0) |00\> + w(\theta_0,\phi_0) |11\>. \end{equation} Now it is easy to find a diagonal unitary operator $U_2$ (with respect to the computational basis) such that $U_2^{\otimes 2}[u(\theta_0,\phi_0) |00\> + w(\theta_0,\phi_0) |11\>]=\cos(\xi/2)|00\> + \sin(\xi/2)|11\>$ with $\xi \in [0,\pi/2]$. Let $U=U_2 U_1(\theta_0,\phi_0) W$, then \eref{eq:Sym2CanonicalForm} holds, which completes the proof of \lref{lem:Sym2CanonicalForm}. \end{proof} \begin{lemma}\label{lem:proofofthm2+1} Suppose $|\Phi\>\in \Sym_2(\caH)$, and $\scrM = \{M_j\}_j$ is a POVM on $\caH$. Then \begin{equation}\label{eq:PsiPOVM2+1} \sum_j \|\caQ(|\Phi\>\<\Phi|\otimes M_j)\| \le 20 + \frac{5\sqrt{22}}{2}, \end{equation} and the upper bound is saturated when \begin{equation}\label{eq:PsiPOVM2+1Saturate} |\Phi\>= \cos\frac{\xi_0}{2}|00\> + \sin\frac{\xi_0}{2}|11\>,\quad \xi_0 := \arcsin\left(1/8\right),\quad \scrM = \{|+\>\< +|, |-\>\< -|\}. \end{equation} \end{lemma} \begin{proof} Thanks to \lref{lem:Sym2CanonicalForm}, we can assume that $|\Phi\>$ has the form $|\Phi\> = \cos(\xi/2)|00\> + \sin(\xi/2)|11\>$ with $\xi \in [0,\pi/2]$ without loss of generality. According to \rcite{BagaMM05}, we can further assume that $\scrM$ is a rank-1 projective measurement that has the form $\scrM = \{|\varphi_+\>\<\varphi_+|,|\varphi_-\>\<\varphi_-|\}$, where \begin{equation} |\varphi_+\> = \cos\frac{\theta}{2}|0\> + \sin\frac{\theta}{2} \rme^{\rmi \phi}|1\>,\quad |\varphi_-\> = \sin\frac{\theta}{2}|0\> - \cos\frac{\theta}{2} \rme^{\rmi \phi}|1\>,\quad \theta\in[0,\pi],\; \phi\in[0,2\pi). \end{equation} Then \begin{align} \sum_j \|\caQ(|\Phi\>\<\Phi|\otimes M_j)\| & = \|\caQ(|\Phi\>\<\Phi|\otimes|\varphi_+\>\<\varphi_+|)\| + \|\caQ(|\Phi\>\<\Phi|\otimes|\varphi_-\>\<\varphi_-|)\| \nonumber\\ &= 20 + \sqrt{\sin^2\theta(9 + \sin^2\xi + 6\sin\xi \cos 2\phi) + (5\cos\xi + 4\cos\theta)^2}\nonumber \\ &\equad + \sqrt{\sin^2\theta(9 + \sin^2\xi + 6\sin\xi \cos 2\phi) + (5\cos\xi - 4\cos\theta)^2}\nonumber \\ &\le 20+q(\xi,\theta), \label{eq:PsiPOVM2+1Proof} \end{align} where \begin{equation} q(\xi,\theta) := \sqrt{h_+}+\sqrt{h_-},\quad h_\pm := \sin^2\theta(3 + \sin\xi)^2 + (5\cos\xi \pm 4\cos\theta)^2, \end{equation} and the inequality above is saturated when $\phi = 0$. To prove \eref{eq:PsiPOVM2+1}, it suffices to prove the following inequality \begin{align}\label{eq:qxithetaUB} q(\xi,\theta)\leq\frac{5\sqrt{22}}{2}. \end{align} If $\theta=0$ or $\theta=\pi$, then \begin{align} q(\xi,\theta) &= |5 \cos \xi + 4| + |5 \cos \xi - 4| \le 10<\frac{5\sqrt{22}}{2}, \label{eq:qxithetaUB1} \end{align} which confirms \eref{eq:qxithetaUB}. Next, suppose $0<\theta<\pi$, then $\sin\theta>0$ and $h_\pm>0$. To determine the extremal points of $q(\xi,\theta)$, we can evaluate the partial derivative of $q(\xi,\theta)$ over $\theta$, with the result \begin{gather} \frac{\partial q(\xi,\theta)}{\partial \theta} = \sin\theta\left(-\frac{g_+}{\sqrt{h_+}} + \frac{g_-}{\sqrt{h_-}}\right) = \frac{\sin\theta \bigl(g_- \sqrt{h_+} - g_+ \sqrt{h_-}\lsp\bigr)}{\sqrt{h_+ h_-}},\\ g_\pm := 20\cos\xi \mp \cos\theta(\sin^2\xi + 6\sin\xi - 7) . \end{gather} In addition, \begin{equation} g^2_-h_+ - g^2_+ h_- = 480\cos\theta \left(\cos\frac{\xi}{2} - \sin\frac{\xi}{2}\right)^3\left(\cos\frac{\xi}{2} + \sin\frac{\xi}{2}\right)(3+\sin\xi)^2 (3+4\sin\xi). \end{equation} If $\partial q(\xi,\theta)/\partial \theta = 0$, then $g^2_-h_+ - g^2_+ h_-=0$, which means $\cos\theta = 0$ or $\cos(\xi/2)=\sin(\xi/2)$, that is, $\theta=\pi/2$ or $\xi = \pi/2$, given that $0<\theta<\pi$ and $0\leq \xi\leq \pi/2$ by assumption. In the latter case, we have \begin{equation} q(\xi,\theta)= q(\pi/2, \theta) = 8 \quad \forall \theta. \label{eq:qxithetaUB2} \end{equation} In the former case, we have \begin{align}\label{eq:qxithetaUB3} q(\xi,\theta) = q(\xi,\pi/2) &= 2\sqrt{(3+\sin\xi)^2 + 25\cos^2\xi} \le \frac{5\sqrt{22}}{2}, \end{align} where the inequality is saturated when $\xi =\xi_0= \arcsin(1/8)$. In conjunction with \eqsref{eq:qxithetaUB1}{eq:qxithetaUB2}, this observation completes the proof of \eref{eq:qxithetaUB}. Now, \eref{eq:PsiPOVM2+1} is a simple corollary of \eqsref{eq:PsiPOVM2+1Proof}{eq:qxithetaUB}. If $|\Phi\>$ and $\scrM$ have the form in \eref{eq:PsiPOVM2+1Saturate}, then the inequalities in \eqsref{eq:PsiPOVM2+1Proof}{eq:qxithetaUB3} are saturated, so the upper bound in \eref{eq:PsiPOVM2+1} is saturated accordingly, which can also be verified by straightforward calculation. \end{proof} \subsection{Direct proof of \thref{thm:2+1}} Thanks to \lref{lem:POVMsym}, to determine the maximum estimation fidelity of 2+1 adaptive POVMs on $\caH^{\otimes 3}$, it suffices to consider 2+1 adaptive rank-1 POVMs on $\Sym_2(\caH)\otimes \caH$. A general 2+1 adaptive rank-1 POVM $\scrM$ on $\Sym_2(\caH)\otimes \caH$ can be expressed as follows, \begin{align} \scrM=\bigcup_j w_j|\Phi_j\>\<\Phi_j|\otimes \scrM_j, \end{align} where $\{w_j|\Phi_j\>\<\Phi_j|\}_j$ forms a POVM on $\Sym_2(\caH)$, which means $|\Phi_j\>\in \Sym_2(\caH)$, $w_j\geq 0$, and $\sum_j w_j=3$; in addition, each $\scrM_j$ is a POVM on $\caH$. By virtue of \eref{eq:EstimationFidDef} and \lref{lem:proofofthm2+1} we can deduce that \begin{align} F(\scrM) &= \frac{1}{120}\sum_{j}\sum_{M\in \scrM_j} \|\caQ(w_j |\Phi_j\>\<\Phi_j|\otimes M)\| \le \frac{1}{120}\left(20 + \frac{5}{2}\sqrt{22}\right)\sum_{j}w_j = \frac{1}{2} + \frac{\sqrt{22}}{16}. \end{align} An optimal POVM that saturates the upper bound is presented in \eref{eq:OptPOVMbs2+1} in the main text, which completes the proof of \thref{thm:2+1}. \section{Proof of \thref{thm:1+2}} \label{app:1+2} \subsection{Auxiliary results} As a complement to \lref{lem:fxyUB}, here we first derive another tight linear upper bound for the function $f(x,y)$ defined in \eref{eq:fxy}. Let \begin{equation}\label{eq:p0gamma} \begin{gathered} p := \frac{47 - 3\sqrt{41}}{216},\quad x_0 := \frac{\frac{2}{3} - p}{1 - p}=\frac{2003 + 27 \sqrt{41}}{3524},\quad y_0 := \frac{-p}{1 - p}=\frac{-1039 + 81 \sqrt{41}}{3524},\\ \alpha := f_x(x_0,y_0) = \frac{5}{2} - \frac{43}{2\sqrt{41}},\;\; \beta := f_y(x_0,y_0) = \frac{9}{2} - \frac{13}{2\sqrt{41}},\;\; \gamma := f(x_0, y_0) - \alpha x_0 - \beta y_0 = 2 + \frac{28}{\sqrt{41}}. \end{gathered} \end{equation} Here $p$ is reproduced from \eref{eq:pSPhiW}, $f_x=\partial f/\partial x$, and $f_y=\partial f/\partial y$. Note that $\alpha$ and $y_0$ are negative, while the other four numbers are positive. \begin{lemma}\label{lem:fxyABC} Suppose $0 \le x \le 1$ and $-x \le y \le x$. Then the function $f(x,y)$ defined in \eref{eq:fxy} satisfies \begin{equation}\label{eq:fxyABC} f(x,y) \le \alpha x + \beta y + \gamma, \end{equation} where $\alpha,\beta,\gamma$ are defined in \eref{eq:p0gamma}, and the inequality is saturated iff $x = y = 1$ or $x = x_0, y = y_0$. \end{lemma} \begin{proof} Note that $\alpha x + \beta y + \gamma\geq \gamma+\alpha-\beta=13/\sqrt{41}>0$. Define the difference function \begin{align} \Delta(x,y) &:= (\alpha x + \beta y + \gamma)^2 - f(x,y)^2\nonumber\\ &\;= -4 - 108 x + 96 x^2 - 40 y - 25 y^2 + 48 (-1 + x)\bigl(-x + \sqrt{x^2 - y^2}\lsp\bigr)\nonumber\\ &\;\equad + \left[ \left(\frac{5}{2} - \frac{43}{2\sqrt{41}}\right)x + \left(\frac{9}{2} - \frac{13}{2\sqrt{41}}\right)y + 2 + \frac{28}{\sqrt{41}}\right]^2. \end{align} Then to prove \eref{eq:fxyABC}, it suffices to prove the inequality $\Delta(x,y) \ge 0$ for $0 \le x \le 1$ and $-x \le y \le x$. When $x=0$, which means $y=0$, it is straightforward to verify that $\Delta(x,y)>0$ and $f(x,y)<\alpha x + \beta y + \gamma$. By assumption $y$ can be expressed as $y = x\cos \zeta$ with $\zeta\in [0,\pi]$, and $\zeta$ is uniquely determined by $x$ and $y$ when $x\neq 0$. Accordingly, $\Delta(x,y)$ can be expressed as \begin{gather} \Delta(x,y) =\Delta(x,x\cos \zeta)= g_2(\zeta) x^2-2g_1(\zeta)x + \gamma^2 - 4, \end{gather} where \begin{equation}\label{eq:g12} g_2(\zeta): =48 + (\alpha + \beta\cos \zeta)^2 - 25\cos^2 \zeta + 48\sin \zeta ,\quad g_1(\zeta):=30 - \alpha\gamma - (\beta\gamma - 20)\cos \zeta + 24\sin \zeta. \end{equation} Note that \begin{equation}\label{eq:g12LB} g_2(\zeta) \ge 23,\quad g_1(\zeta)\geq g_1(0)=50-\alpha\gamma - \beta\gamma>33, \end{equation} given that $\beta\gamma-20>0$. Let $x^*(\zeta) : =g_1(\zeta)/g_2(\zeta)$; then $\Delta(x,x\cos \zeta)\geq \Delta(x^*(\zeta),x^*(\zeta)\cos \zeta)$, and the inequality is saturated iff $x= x^*(\zeta)\leq 1$. Let \begin{equation}\label{eq:ellzeta} \ell(\zeta) :=g_2(\zeta)-g_1(\zeta)= 18 + \alpha^2 + \alpha \gamma - (20 - \beta\gamma - 2\alpha\beta)\cos \zeta -\bigl(25 - \beta^2\bigr)\cos^2 \zeta + 24\sin \zeta; \end{equation} then $x^*(\zeta)\leq 1$ iff $\ell(\zeta) \geq 0$. If $\pi/2\leq \zeta\leq \pi$, then \begin{align} \ell(\zeta)\geq 18 + \alpha^2 + \alpha \gamma-\bigl(25 - \beta^2\bigr)=\frac{907-139\sqrt{41}}{41}>0,\quad x^*(\zeta)<1, \end{align} given that $(20 - \beta\gamma - 2\alpha\beta)>0$ and $25 - \beta^2>0$. If instead $0\leq \zeta\leq \pi/2$, then $\ell(\zeta)$ is monotonically increasing in $\zeta$. Meanwhile, $\ell(0)<0$ and $\ell(\pi/2)>0$. Therefore, $\ell(\zeta)$ has a unique zero for $\zeta\in [0,\pi]$. Let $\zeta_*$ be this unique zero; numerically, we have $\zeta_* \approx 0.12988$ and $\cos \zeta_* \approx 0.99158$. Then $x^*(\zeta_*)=1$, $x^*(\zeta)>1$ for $\zeta\in [0,\zeta_*)$, and $x^*(\zeta)<1$ for $\zeta\in (\zeta_*, \pi]$. Define \begin{equation} \Delta^*(\zeta) := \begin{cases} \Delta(1,\cos \zeta) & \zeta \in [0,\zeta_*),\\ \Delta(x^*(\zeta),x^*(\zeta)\cos \zeta) & \zeta \in [\zeta_*,\pi]; \end{cases} \end{equation} then $\Delta(x,x\cos \zeta) \ge \Delta^*(\zeta)$, and the inequality is saturated iff \begin{align}\label{eq:xzetaMin} x=\begin{cases} 1 & \zeta \in [0,\zeta_*),\\ x^*(\zeta) & \zeta \in [\zeta_*,\pi]. \end{cases} \end{align} To prove \eref{eq:fxyABC}, it suffices to prove the inequality $\Delta^*(\zeta) \ge 0$ for $\zeta\in [0,\pi]$. If $\zeta \in [0,\zeta_*)$, then \begin{align} \Delta^*(\zeta)=\Delta(1,\cos \zeta)=\frac{1}{41}\bigl[433 + 117 \sqrt{41} + \bigl(305 + 117 \sqrt{41}\lsp\bigr) \cos\zeta\bigr]\sin^2\frac{\zeta}{2}\geq 0. \label{eq:Delta*zetaLB1} \end{align} If instead $\zeta \in [\zeta_*,\pi]$, then \begin{align} \Delta^*(\zeta)=\Delta(x^*(\zeta),x^*(\zeta)\cos \zeta)=\frac{(\gamma^2-4)g_2(\zeta) -g_1(\zeta)^2}{g_2(\zeta)}=\frac{h(\zeta)}{41g_2(\zeta)}, \end{align} where \begin{align} h(\zeta) &:= c_0 \cos (2\zeta) + c_1 \sin (2\zeta) + c_2 \cos \zeta + c_3 \sin \zeta + c_4\nonumber\\ &\;=2c_0\cos^2\zeta+ c_2 \cos \zeta+c_4-c_0+(2c_1\cos\zeta+c_3)\sin\zeta, \label{eq:hzeta} \end{align} with \begin{equation} \begin{gathered} c_0 = -4197 + 977 \sqrt{41},\quad c_1 = 24 \bigl(-633 + 113 \sqrt{41}\lsp\bigr),\quad c_2 = 4\bigl(-14667 + 2191 \sqrt{41}\lsp\bigr),\\ c_3 = 48 \bigl(-843 + 139 \sqrt{41}\lsp\bigr),\quad c_4 = -53775 + 8403 \sqrt{41}. \end{gathered} \end{equation} Note that $c_0,c_1, c_3,c_4>0$ and $c_2<0$; in addition, $g_2(\zeta)>0$ by \eref{eq:g12LB}. To prove the inequality $\Delta^*(\zeta)\geq 0$ for $\zeta \in [\zeta_*,\pi]$, it suffices to prove the inequality $h(\zeta)\geq 0$. Let $u=\cos\zeta$ and define \begin{equation} h_2(u) := \bigl(2c_0 u^2 + c_2 u + c_4 - c_0\bigr)^2 - \left(2c_1 u + c_3\right)^2\bigl(1-u^2\bigr). \end{equation} Then $h_2(u)=0$ whenever $h(\zeta)=0$ according to \eref{eq:hzeta}. Calculation shows that $h_2(u)$ has the following three distinct zeros: \begin{equation}\label{eq:u0pm} u_0 := \frac{-308 + 27\sqrt{41}}{565},\quad u_\pm:=\frac{37529139\sqrt{41}-239145719\pm 576\sqrt{-35743460158+5587351798\sqrt{41}}}{294550033-45301173\sqrt{41}}, \end{equation} where $u_-<u_0<u_+$ and the zero $u_0$ has multiplicity 2. By contrast, $h(\zeta)$ has two distinct zeros, namely, $\zeta_0:=\arccos u_0\approx 1.81228$ and $\zeta_+:=\arccos u_+\approx 0.07235$; note that $\arccos u_-$ is not a zero of $h(\zeta)$. In addition, $0<\zeta_+<\zeta_*$ and $\zeta_*<\zeta_0<\pi$, so $\zeta_0$ is the unique zero of $h(\zeta)$ within the interval $[\zeta_*,\pi]$. Straightforward calculation shows that $h(\zeta_*), h(\pi)>0$ as illustrated in \fref{fig:hzeta}, which implies that $h(\zeta),\Delta^*(\zeta) \geq 0$ for $\zeta\in [\zeta_*,\pi]$ by continuity. In conjunction with \eref{eq:Delta*zetaLB1} we can deduce that $\Delta(x,x\cos \zeta)\geq \Delta^*(\zeta)\geq 0$ for $\zeta\in [0,\pi]$, which implies \eref{eq:fxyABC}; in addition, $\Delta^*(\zeta)$ has only two zeros in this interval, namely, 0 and $\zeta_0$. If $x = y = 1$ or if $x = x_0$ and $ y = y_0$, then the inequality in \eref{eq:fxyABC} is saturated by straightforward calculation. Conversely, if the inequality in \eref{eq:fxyABC} is saturated, then $\Delta^*(\zeta)=\Delta(x,x\cos \zeta)=\Delta(x,y)=0$, which means $\zeta=0$ or $\zeta=\zeta_0$. According to \eref{eq:xzetaMin}, if $\zeta=0$, then $y=x=1$; if instead $\zeta=\zeta_0$, then \begin{align} x=x^*(\zeta_0)=x_0, \quad y=x^*(\zeta_0)\cos\zeta_0=y_0. \end{align} This observation completes the proof of \lref{lem:fxyABC}. \end{proof} \begin{figure} \centering \includegraphics[width = 9cm]{hz.pdf} \caption{\label{fig:hzeta}A plot of the function $h(\zeta)$ defined in \eref{eq:hzeta} for $\zeta\in [0,\pi]$. Here $\zeta_+=\arccos u_+$ and $\zeta_0=\arccos u_0$ are the two zeros of $h(\zeta)$ [see \eref{eq:u0pm}], while $\zeta_*$ is the unique zero of the function $\ell(\zeta)$ defined in \eref{eq:ellzeta}.} \end{figure} \begin{lemma}\label{lem:canonical1+2} Suppose $\scrM = \{w_j |\Psi_j\>\<\Psi_j|\}_j$ is a 1+2 adaptive POVM on $\Sym_2(\caH)\otimes\caH$, where $|\Psi_j\>$ have the form \begin{equation} |\Psi_j\> = U_j^{\otimes 3}\bigl[\bigl(a_j|00\> + b_j|\bbS\> + c_j \rme^{\rmi \phi_j}|11\>\bigr)\otimes|0\>\bigr], \end{equation} with $U_j\in \rmU(\caH), a_j, b_j, c_j \ge 0, a_j^2 + b_j^2 + c_j^2 = 1 $, and $\phi_j\in[0,2\pi)$. Then \begin{equation} \sum_j w_j a_j^2 = \sum_j w_j b_j^2 = \sum_j w_j c_j^2 = 2,\quad \sum_j w_j a_j b_j = \sum_j a_j c_j \rme^{\rmi\phi_j} = \sum_j b_j c_j \rme^{\rmi\phi_j} = 0. \label{eq:constraint2} \end{equation} \end{lemma} \begin{proof} Let $|\Phi_j\>=a_j|00\> + b_j|\bbS\> + c_j \rme^{\rmi \phi_j}|11\>$, then $\{(w_j/2) |\Phi_j\>\<\Phi_j|\}_j$ forms a POVM on $\Sym_2(\caH)$. Therefore, \begin{align} \sum_j w_j|\Phi_j\>\<\Phi_j|=2P_2 =2|00\>\<00|+2|\bbS\>\<\bbS|+2|11\>\<11|, \end{align} which implies \eref{eq:constraint2} and completes the proof of \lref{lem:canonical1+2}. \end{proof} \subsection{Proof of \thref{thm:1+2}} Thanks to \lref{lem:POVMsym}, to determine the maximum estimation fidelity of 1+2 adaptive POVMs on $\caH^{\otimes 3}$, it suffices to consider 1+2 adaptive rank-1 POVMs on $\Sym_2(\caH)\otimes \caH$. Suppose $\scrM = \bigl\{M_j\}_j$ is an arbitrary 1+2 rank-1 POVM on $\Sym_2(\caH)\otimes\caH$. Then $M_j$ can be expressed as follows, $M_j=w_j U_j^{\otimes 3}|\Psi_j\>\<\Psi_j|U_j^{\dag\otimes 3}$, where $w_j> 0$, $\sum_j w_j=6$, $U_j\in \rmU(\caH)$, and $|\Psi_j\>$ have the form \begin{equation} |\Psi_j\> = \bigl(a_j|00\> + b_j|\bbS\> + c_j \rme^{\rmi \phi_j}|11\>\bigr)\otimes|0\> \end{equation} with $a_j,b_j,c_j\ge 0, a_j^2 + b_j^2 + c_j^2 = 1$, and $\phi_j\in[0,2\pi)$. By virtue of \eref{eq:EstimationFidDef} and \lsref{lem:q_abc}, \ref{lem:canonical1+2} we can deduce that \begin{align} F(\scrM)&=\frac{1}{120}\sum_j w_j \caQ(|\Psi_j\>\<\Psi_j|)=\frac{1}{120}\sum_j w_j \eta(a_j,b_j,c_j,\phi_j) \le \frac{1}{120}\sum_j w_j\bigl[15 a_j^2+10b_j^2+5c_j^2 +f(x_j,y_j)\bigr]\nonumber\\ &= \frac{1}{2}+\frac{1}{20}\sum_{j} p_jf(x_j,y_j), \label{eq:1+2sat} \end{align} where the relevant parameters satisfy the following constraints (see \lref{lem:canonical1+2}): \begin{equation} p_j := \frac{w_j}{6},\quad x_j := a_j^2 + c_j^2,\quad y_j := a_j^2 - c_j^2,\quad \sum_{j} p_j = 1, \quad \sum_{j} p_j y_j = 0,\quad \sum_j p_j x_j = \frac{2}{3}. \label{eq:opt3} \end{equation} Note that the inequality in \eref{eq:1+2sat} is saturated when $\phi_j=0$ for all $j$. Next, in conjunction with \eref{eq:p0gamma} and \lref{lem:fxyABC} we can deduce that \begin{align}\label{eq:proofthe4} F(\scrM) \le \frac{1}{2}+\frac{1}{20}\sum_j p_j \left(\alpha x_j + \beta y_j + \gamma\right) = \frac{1}{2} + \frac{1}{30}\alpha + \frac{1}{20}\gamma = \frac{1}{2} + \frac{11 + \sqrt{41}}{60}. \end{align} The saturation of the above inequality means either $(x_j,y_j)=(x_0,y_0)$ or $(x_j,y_j)=(1,1)$ for each $j$, where $x_0,y_0$ are defined in \eref{eq:p0gamma}. In conjunction with \eref{eq:opt3} we can deduce that \begin{align} \sum_{j\,|\,(x_j,y_j)=(1,1)} p_j=p, \end{align} where $p$ is defined in \eref{eq:pSPhiW} and is reproduced in \eref{eq:p0gamma}. Moreover, the upper bound in \eref{eq:proofthe4} is saturated when $\scrM$ has the form \begin{equation} \scrM=\{\scrK_0'\otimes |0\>\<0|\}\cup \{\scrK_1'\otimes |1\>\<1|\},\quad \scrK_0' := \{K_j\}_{j=0}^4,\quad \scrK_1':=\bigl\{X^{\otimes 2}K_jX^{\otimes 2}\bigr\}_{j=0}^4, \end{equation} where $K_j$ for $j=0,1,2,3,4$ are defined in \eref{eq:scrE01}. Note that $\scrK_0'$ and $\scrK_1'$ are POVMs on $\Sym_2(\caH)$. Accordingly, we can construct an optimal 1+2 adaptive POVM on $\caH^{\otimes 3}$ as shown in \eref{eq:OptPOVM1+2} in the main text. This observation completes the proof of \thref{thm:1+2}. \bibliography{all_references} \end{document}
2412.03358v1
http://arxiv.org/abs/2412.03358v1
Galois groups of low dimensional abelian varieties over finite fields
\documentclass[11pt]{amsart} \pdfoutput=1 \usepackage[dvipsnames, table, xcdraw]{xcolor} \usepackage{upgreek} \usepackage{amssymb,amsmath,mathtools,amsfonts,amsthm} \usepackage[mathscr]{eucal} \usepackage{fullpage} \usepackage{color} \usepackage{caption} \usepackage{subcaption} \usepackage{enumitem} \usepackage[colorlinks]{hyperref} \definecolor{headercolor}{RGB}{255,255,240} \definecolor{mylinkcolor}{RGB}{0,0,255} \definecolor{mycitecolor}{RGB}{169,169,169} \definecolor{myurlcolor}{RGB}{255,20,147} \hypersetup{colorlinks=true, urlcolor=myurlcolor, citecolor=mycitecolor, linkcolor=mylinkcolor, linktoc=page, breaklinks=true} \usepackage{url} \urlstyle{tt} \usepackage[numbers]{natbib} \usepackage{setspace} \setstretch{1.2} \setlength{\bibsep}{0.0pt} \usepackage{indentfirst} \usepackage{colonequals} \usepackage{nicefrac} \usepackage{tikz-cd,tikz} \usepackage{cleveref} \usetikzlibrary{positioning} \usepackage{float} \usepackage{adjustbox} \newcommand{\PreserveBackslash}[1]{\let\temp=\\#1\let\\=\temp} \newcolumntype{C}[1]{>{\PreserveBackslash\centering}p{#1}} \usepackage{diagbox} \usepackage{multirow} \usepackage{hhline} \usepackage{booktabs} \usepackage{placeins} \numberwithin{equation}{section} \newtheorem{thmx}{Theorem} \renewcommand{\thethmx}{\Alph{thmx}} \newtheorem{corox}{Corollary} \renewcommand{\thecorox}{\Alph{corox}} \newtheorem{theorem}{Theorem}[section] \newtheorem{hypothesis}[theorem]{Hypothesis} \newtheorem{lemma}[theorem]{Lemma} \newtheorem{coro}[theorem]{Corollary} \newtheorem{conjecture}[theorem]{Conjecture} \newtheorem{proposition}[theorem]{Proposition} \theoremstyle{definition} \newtheorem{defn}[theorem]{Definition} \newtheorem{example}[theorem]{Example} \newtheorem{question}{Question} \newtheorem{examples}[theorem]{Examples} \newtheorem{remark}[theorem]{Remark} \def \ifempty#1{\def\temp{#1} \ifx\temp\empty } \newcommand{\claim}[2][]{\ifempty{#1} \par\vspace{2mm}\noindent\underline{Claim}:\;\textit{#2}\\\else \par\vspace{2mm}\noindent\underline{Claim~#1}:\;\textit{#2}\\ } \numberwithin{table}{section} \numberwithin{figure}{section} \makeatletter \setlength{\@fptop}{0pt plus 1fil} \setlength{\@fpsep}{8pt plus 2fil} \makeatother \addtolength{\textfloatsep}{0mm} \newcommand{\balpha}{\boldsymbol\alpha} \newcommand{\oalpha}{\overline{\alpha}} \newcommand{\oq}{\overline{\mathfrak{q}}} \newcommand{\op}{\overline{\mathfrak{p}}} \newcommand{\bd}[1]{\mathbf{#1}} \newcommand{\msf}[1]{\mathsf{#1}} \newcommand{\RR}{\mathbf{R}} \newcommand{\NN}{\mathbf{N}} \newcommand{\ZZ}{\mathbf{Z}} \newcommand{\QQ}{\mathbf{Q}} \newcommand{\CC}{\mathbf{C}} \newcommand{\FF}{\mathbf{F}} \newcommand{\Fp}{\FF_p} \newcommand{\Fq}{\FF_q} \newcommand{\ZmodnZ}[1]{\mathbb{Z} / #1 \mathbb{Z}} \newcommand{\Qp}{\mathbf{Q}_p} \newcommand{\Ql}{\mathbf{Q}_\ell} \newcommand{\Qbar}{\overline{\mathbf{Q}}} \newcommand{\XX}{\mathscr{X}} \newcommand{\YY}{\mathscr{Y}} \newcommand{\OO}{\mathscr{O}} \newcommand{\mfp}{\mathfrak{p}} \newcommand{\mfq}{\mathfrak{q}} \newcommand{\Pone}{\mathbb{P}^1} \newcommand{\Aone}{\mathbb{A}^1} \newcommand{\degr}{\operatorname{deg }} \newcommand{\md}{\text{ mod }} \newcommand{\krn}{\text{ker }} \newcommand{\legendre}[2]{\genfrac{(}{)}{}{}{#1}{#2}} \newcommand{\ceil}[1]{\lceil #1 \rceil} \newcommand{\floor}[1]{\left\lfloor #1 \right\rfloor} \newcommand{\mand}{\text{ and }} \newcommand{\mforall}{ \text{ for all }} \newcommand{\fs}{\text{ for some }} \newcommand{\mwith}{\text{ with }} \newcommand{\mof}{\text{ of }} \newcommand{\mfor}{\text{ for }} \newcommand{\mif}{\text{ if }} \newcommand{\tth}{^{\text{th}}} \newcommand{\inv}{^{-1}} \newcommand{\unit}{^{\times}} \newcommand{\brk}[1]{ \mathopen{}\left\lbrace #1 \right\rbrace\mathclose{}} \newcommand{\paren}[1]{ \mathopen{}\left( #1 \right)\mathclose{}} \newcommand{\abs}[1]{\left| #1 \right|} \newcommand{\cdef}[1]{{\color{blue}{\textit{#1}}}} \newcommand{\Magma}{\textsc{Magma} } \newcommand{\SageMath}{\textsc{SageMath} } \newcommand{\GitHub}{\textsc{GitHub }} \newcommand{\Stpm}{S_3^\pm} \newcommand{\pii}{\uppi} \newcommand{\Isog}{\mathcal{I}} \newcommand{\ol}[1]{\overline{#1}} \newcommand{\ideal}[1]{\langle #1 \rangle} \newcommand{\pmone}{\brk{\pm 1}} \newcommand{\conjto}[1]{\propto_{#1}} \newcommand{\indx}{\mathcal{I}} \newcommand{\rts}{\mathcal{R}} \newcommand{\cO}{\mathcal{O}} \newcommand{\cP}{\mathcal{P}} \newcommand{\vecsp}[2]{\ensuremath#1\langle#2\rangle} \DeclareMathOperator{\disc}{disc} \DeclareMathOperator{\Aut}{Aut} \DeclareMathOperator{\id}{id} \DeclareMathOperator{\Frac}{Frac} \DeclareMathOperator{\Frob}{Frob} \DeclareMathOperator{\Br}{Br} \DeclareMathOperator{\codim}{codim} \DeclareMathOperator{\tr}{tr} \DeclareMathOperator{\Hom}{Hom} \DeclareMathOperator{\rank}{rk} \DeclareMathOperator{\im}{im} \DeclareMathOperator{\coker}{coker} \DeclareMathOperator{\Gal}{Gal} \DeclareMathOperator{\Jac}{J} \DeclareMathOperator{\Spec}{Spec} \DeclareMathOperator{\Proj}{Proj} \DeclareMathOperator{\lcm}{lcm} \DeclareMathOperator{\sgn}{sgn} \DeclareMathOperator{\Sym}{Sym} \DeclareMathOperator{\Pic}{Pic} \renewcommand{\div}{\operatorname{div}} \DeclareMathOperator{\Div}{Div} \DeclareMathOperator{\supp}{supp} \DeclareMathOperator{\res}{res} \DeclareMathOperator{\vol}{vol} \DeclareMathOperator{\ev}{ev} \DeclareMathOperator{\trdeg}{tr.deg} \DeclareMathOperator{\Res}{Res} \DeclareMathOperator{\Log}{Log} \DeclareMathOperator{\End}{End} \DeclareMathOperator{\Tr}{Tr} \DeclareMathOperator{\sqf}{sqf} \DeclareMathOperator{\Conf}{Conf} \DeclareMathOperator{\Stab}{Stab} \DeclareMathOperator{\Span}{span} \newcommand{\arXiv}[2]{\href{https://arxiv.org/abs/#1}{arXiv:#1v#2}} }}} \usepackage{underscore} \newcommand{\avlink}[1]{\href{http://www.lmfdb.org/Variety/Abelian/Fq/#1}{\texttt{#1}}} \makeatletter \newcommand\newtag[2]{#1\def\@currentlabel{#1}\label{#2}} \makeatother \definecolor{wlabcol}{rgb}{0,0,0.3} \newcommand{\wgrouplabel}[1]{\textnormal{\texttt{#1}}} \newcommand{\mkwl}[1]{\newtag{\wgrouplabel{#1}}{#1}} \newcommand{\wl}[1]{ \begingroup\hypersetup{linkcolor=mylinkcolor}\ref{#1}\endgroup } \newcommand{\cclass}[1]{\mathcal{G}_{#1}} \newcommand{\thmemph}[1]{\textbf{#1}} \setlist[itemize]{leftmargin=30pt, itemsep=2pt} \setlist[enumerate]{leftmargin=30pt, itemsep=2pt} \renewcommand{\o}{{\textsf{o}}} \newcommand{\ao}{{\textsf{ao}}} \renewcommand{\ss}{{\textsf{ss}}} \newcommand{\GL}{\msf{GL}} \newcommand{\SL}{\msf{SL}} \newcommand{\GSp}{\msf{GSp}} \newcommand{\Sp}{\msf{Sp}} \newcommand{\UU}{\msf{U}} \newcommand{\USp}{\msf{USp}} \newcommand{\SF}{\msf{SF}} \newcommand{\ste}{\texttt{6T11}} \newcommand{\sts}{\texttt{6T6}} \newcommand{\stt}{\texttt{6T3}} \newcommand{\sto}{\texttt{6T1}} \usetikzlibrary{shapes.geometric, arrows.meta, bending, positioning} \definecolor{transitive}{rgb}{0.0, 0.8, 0.6} \definecolor{non-transitive}{rgb}{0.9, 0.17, 0.31} \tikzstyle{start} = [rectangle, rounded corners, minimum width = 2cm, minimum height = 1cm, draw = blue, fill = white, text centered, very thick] \tikzstyle{stop} = [rectangle, rounded corners, minimum width = 2cm, minimum height = 1cm, draw = blue, fill = white, text width = 2cm, text centered, very thick] \tikzstyle{stop-t} = [rectangle, rounded corners, minimum width = 2cm, minimum height = 1cm, draw = transitive, fill = white, text width = 2cm, text centered, very thick] \tikzstyle{stop-nt} = [rectangle, rounded corners, minimum width = 2cm, minimum height = 1cm, draw = non-transitive, fill = white, text width = 2cm, text centered, very thick] \tikzstyle{stage} = [rectangle, minimum width = 1.5cm, minimum height = 1cm, draw = blue, fill = white, text centered, very thick] \tikzstyle{stage-NP} = [rectangle, minimum width = 2.1cm, minimum height = 1cm, draw = white, fill = white, text centered, very thick] \tikzstyle{tag} = [rectangle, draw = white, fill = white, text centered] \tikzstyle{decision} = [diamond, minimum width = 1.6cm, minimum height = 1cm, draw = black, fill = white, text width = 1.9cm, text centered, thick] \tikzstyle{input} = [trapezium, trapezium left angle = 70, trapezium right angle = 110, draw = cyan, fill = white, text width = 2cm, text centered] \tikzstyle{arrow} = [thick, ->, >=stealth, gray] \title[Galois groups of low dimensional Abelian varieties over finite fields]{Galois groups of low dimensional Abelian\\ varieties over finite fields} \date{December 4, 2024} \author{Santiago Arango-Piñeros} \address{Department of Mathematics, Emory University, Atlanta, GA 30322, USA} \email{[email protected]} \urladdr{\url{https://sarangop1728.github.io/}} \author{Sam Frengley} \address{School of Mathematics, University of Bristol, Bristol, BS8 1UG, UK} \email{[email protected]} \urladdr{\url{https://samfrengley.github.io/}} \author{Sameera Vemulapalli} \address{Department of Mathematics, Harvard University} \email{[email protected]} \urladdr{\url{https://web.math.princeton.edu/~sameerav/}} \allowdisplaybreaks \begin{document} \begin{abstract} We consider three isogeny invariants of abelian varieties over finite fields: the Galois group, Newton polygon, and the angle rank. Motivated by work of Dupuy, Kedlaya, and Zureick-Brown, we define a new invariant called the \emph{weighted permutation representation} which encompasses all three of these invariants and use it to study the subtle relationships between them. We use this permutation representation to classify the triples of invariants that occur for abelian surfaces and simple abelian threefolds. \end{abstract} \maketitle \vspace{-5mm} \setcounter{tocdepth}{1} \tableofcontents \vspace{-5mm} \section{Introduction} \label{sec:intro} The purpose of this article is to study the surprisingly subtle interactions between three isogeny invariants of abelian varieties over finite fields. We analyze which triples of invariants may occur and what restrictions they impose on the abelian variety in question. The interaction of these invariants is discussed in a letter from Serre to Ribet \cite[pp.~6]{Serre89} and has gained renewed interest following the publication of the database of abelian varieties over finite fields in the LMFDB \cite{DupuyKedlayaRoeVincent22}. The availability of this data has demonstrated that the interaction between these invariants is more intricate than initially thought \cite{DupuyKedlayaRoeVincent21}, prompting the development of more refined invariants to better understand these relationships \cite{DupuyKedlayaZureick-Brown22}. This article makes progress towards that goal. Even though this subject is interesting in its own right, it has applications to the Tate conjecture \cite{Zarhin15, Zarhin22}, monodromy groups of abelian varieties over number fields \cite{Zywina22}, Frobenius distributions \cite{AhmadiShparlinsky10, APBS2023}, and prime number races and Chebyshev biases in the context of function fields \cite{keliher2024}. By the Honda--Tate theorem \cite{Tate1966, Honda1968, Tate1971}, the isogeny class of an abelian variety $A$ is determined by its Frobenius polynomial, which is the characteristic polynomial of the Frobenius endomorphism acting on the $\ell$-adic Tate module of $A$ (where $\ell$ is a prime number which is not equal to the characteristic of the base field $\Fq$). All of our invariants are derived from the Frobenius polynomial; the first invariant is the \cdef{Newton polygon} of the Frobenius polynomial, which determines the $p$-adic valuations of the roots, the second invariant is the \cdef{angle rank} (see \Cref{defn:angle-rank}), which measures the nontrivial multiplicative relations between the roots of the Frobenius polynomial, and finally, we have the \cdef{Galois group} of the Frobenius polynomial as our third invariant. We classify triples of invariants that occur for abelian varieties of dimension $\leq 3$; the dimension $3$ case already exhibits some subtleties that should be expected in general. In \cite{DupuyKedlayaZureick-Brown22}, the authors noticed that the Galois group, Newton polygon, and angle rank are not independent. The Galois group acts on the $p$-adic valuations of the roots (we visualize each root as a ball of radius proportional to its $p$-adic valuation) and this \cdef{weighted permutation representation} (see \Cref{def:perm-rep}) determines the Galois group, Newton polygon, and angle rank. However, this does \emph{not} imply that the Galois group and Newton polygon determine the angle rank. For example, the isogeny classes of abelian threefolds over $\FF_2$ with LMFDB \cite{lmfdb} labels \avlink{3.2.ac_a_d} and \avlink{3.2.a_a_ad} have the same Newton polygon and Galois group, but different angle ranks. This example illustrates the need to consider more than just the isomorphism class of the Galois group. \subsection{Statement of main results} The authors of \cite{DupuyKedlayaZureick-Brown22} defined the \emph{Newton hyperplane representation} of a geometrically simple abelian variety to encode this information. The central contribution of this paper is to reinterpret the Newton hyperplane representation in terms of a \emph{weighted permutation representation} and prove additional constraints upon it (see \Cref{sec:divisor-map}). We leverage these results to classify weighted permutation representations for elliptic curves (\Cref{lemma:main-thm-ecs}), abelian surfaces (\Cref{thm:main-thm-surfaces}) and simple abelian threefolds (\Cref{thm:main-thm-3folds}). The flowcharts in Figures~\ref{fig:flowchart2-simple}--\ref{fig:flowchart3} distill information from the tables in our main theorems. The purpose of these flowcharts is to serve as a ``user's guide'' to the tables in \Cref{thm:main-thm-surfaces} and \Cref{thm:main-thm-3folds}; in particular, if one has in hand an abelian surface or threefold, then the flowchart rules out certain Galois groups. \begin{theorem} \label{thm:main-surfaces} If $A$ is an abelian surface, then the possible isomorphism classes of the Galois group $G_A$ are determined by \Cref{fig:flowchart2-simple} in the simple case, and by \Cref{fig:flowchart2-non-simple} in the nonsimple case. Moreover, every possibility occurs. \end{theorem} \begin{theorem} \label{thm:main-simple-threefolds} If $A$ is a simple abelian threefold, then the possible isomorphism classes of the Galois group $G_A$ are determined by \Cref{fig:flowchart3}. Moreover, every possibility occurs. \end{theorem} \begin{figure}[p] \centering \resizebox{0.8\textwidth}{!}{ \begin{tikzpicture}[node distance = 2.7cm] \node (start) [start] {Simple abelian surface over $\mathbf{F}_q$}; \node (B) [stage-NP, below of = start, yshift=-1cm] {\includegraphics[scale=0.45]{images/ao_2.png}}; \node (B-tag) [tag, above of=B, yshift=-1cm, xshift=-1cm] {(B)}; \node (A) [stage-NP, left of = B, xshift = -3cm] {\includegraphics[scale=0.45]{images/o_2.png}}; \node (A-tag) [tag, above of=A, yshift=-1cm] {(A)}; \node (C) [stage-NP, right of = B, xshift = 3cm] {\includegraphics[scale=0.45]{images/ss_2.png}}; \node (C-tag) [tag, above of=C, yshift=-1cm] {(C)}; \node (C-delta0) [stage, below of=C,yshift=-1cm] {$\delta_A = 0$}; \node (A-delta2) [stage, below of=A,xshift=-1.5cm,yshift=-1cm] {$\delta_A = 2$}; \node (A-delta1) [stage, below of=A,xshift=1.5cm,yshift=-1cm] {$\delta_A = 1$}; \node (B-delta2) [stage, right of=A-delta1,xshift=1.6cm] {$\delta_A = 2$}; \node (W4) [stop, below of=B, yshift=-4cm, xshift = -6cm] {$C_2\wr S_2$}; \node (C4) [stop, below of=B, yshift=-4cm, xshift=-2cm] {$C_4$}; \node (V4) [stop, below of=B, yshift=-4cm, xshift = 2cm] {$V_4$}; \node (C2) [stop, below of=B, yshift=-4cm, xshift = 6cm] {$C_2$}; \draw[arrow] (start) -- (A); \draw[arrow] (start) -- (B); \draw[arrow] (start) -- (C); \draw[arrow] (B) -- (B-delta2); \draw[arrow] (C) -- (C-delta0); \draw[arrow] (A) -- (A-delta2); \draw[arrow] (A) -- (A-delta1); \draw[arrow] (B-delta2) -- (W4); \draw[arrow] (A-delta2) -- (W4); \draw[arrow] (A-delta2) -- (C4); \draw[arrow] (A-delta1) -- (V4); \draw[arrow] (C-delta0) -- (V4); \draw[arrow] (C-delta0) -- (C2); \end{tikzpicture} } \caption{Possible isomorphism classes of Galois groups of simple abelian surfaces in terms of their Newton polygon, and angle rank $\delta_A$.} \label{fig:flowchart2-simple} \end{figure} \begin{figure}[p] \centering \resizebox{0.8\textwidth}{!}{ \begin{tikzpicture}[node distance = 2.7cm] \node (start) [start] {Nonsimple abelian surface $E_1\times E_2$ over $\mathbf{F}_q$}; \node (B) [stage-NP, below of = start, yshift=-1cm] {\includegraphics[scale=0.45]{images/ao_2.png}}; \node (B-tag) [tag, above of=B, yshift=-1cm, xshift=-1cm] {(B)}; \node (A) [stage-NP, left of = B, xshift = -3cm] {\includegraphics[scale=0.45]{images/o_2.png}}; \node (A-tag) [tag, above of=A, yshift=-1cm] {(A)}; \node (C) [stage-NP, right of = B, xshift = 3cm] {\includegraphics[scale=0.45]{images/ss_2.png}}; \node (C-tag) [tag, above of=C, yshift=-1cm] {(C)}; \node (B-delta1) [stage, below of=B,yshift=-1cm] {$\delta_A = 1$}; \node (C-delta0) [stage, below of=C,yshift=-1cm] {$\delta_A = 0$}; \node (A-delta2) [stage, below of=A,xshift=-1.5cm,yshift=-1cm] {$\delta_A = 2$}; \node (A-delta1) [stage, below of=A,xshift=1.5cm,yshift=-1cm] {$\delta_A = 1$}; \node (V4) [stop, below of=B, yshift=-4cm, xshift = -4cm] {$V_4$}; \node (C2) [stop, below of=B, yshift=-4cm] {$C_2$}; \node (C1) [stop, below of=B, yshift=-4cm, xshift = 4cm] {$C_1$}; \draw[arrow] (start) -- (A); \draw[arrow] (start) -- (B); \draw[arrow] (start) -- (C); \draw[arrow] (B) -- (B-delta1); \draw[arrow] (C) -- (C-delta0); \draw[arrow] (A) -- (A-delta2); \draw[arrow] (A) -- (A-delta1); \draw[arrow] (A-delta2) -- (V4); \draw[arrow] (A-delta1) -- (C2); \draw[arrow] (B-delta1) -- (V4); \draw[arrow] (B-delta1) -- (C2); \draw[arrow] (C-delta0) -- (V4); \draw[arrow] (C-delta0) -- (C2); \draw[arrow] (C-delta0) -- (C1); \end{tikzpicture} } \caption{Possible isomorphism classes of Galois groups of simple abelian surfaces in terms of their Newton polygon, and angle rank $\delta_A$.} \label{fig:flowchart2-non-simple} \end{figure} \begin{figure} \centering \resizebox{\textwidth}{!}{ \begin{tikzpicture}[node distance = 2.7cm] \node (start) [start] {Simple abelian threefold over $\mathbf{F}_q$}; \node (C) [stage-NP, below of = start, yshift=-1cm] {\includegraphics[scale=0.36]{images/k3type_3.png}}; \node (C-tag) [tag, above of=C, yshift=-1.3cm] {(C)}; \node (B) [stage-NP, left of = C, xshift = -1.5cm] {\includegraphics[scale=0.36]{images/ao_3.png}}; \node (B-tag) [tag, above of=B, yshift=-1.3cm] {(B)}; \node (A) [stage-NP, left of = B, xshift = -1.5cm] {\includegraphics[scale=0.36]{images/o_3.png}}; \node (A-tag) [tag, above of=A, yshift=-1.3cm] {(A)}; \node (D) [stage-NP, right of = C, xshift = 1.5cm] {\includegraphics[scale=0.36]{images/prk0_3.png}}; \node (D-tag) [tag, above of=D, yshift=-1.3cm] {(D)}; \node (E) [stage-NP, right of = D, xshift = 1.5cm] {\includegraphics[scale=0.36]{images/ss_3.png}}; \node (E-tag) [tag, above of=E, yshift=-1.3cm] {(E)}; \node (A-delta3) [stage, below of = A, xshift = -1cm,yshift=-1cm]{$\delta_A = 3$}; \node (A-delta1) [stage, below of = A, xshift=1cm,yshift=-1cm]{$\delta_A = 1$}; \node (B-delta3) [stage, below of = B, xshift = -1cm,yshift=-1cm]{$\delta_A = 3$}; \node (B-delta2) [stage, below of = B, xshift = 1cm,yshift=-1cm]{$\delta_A = 2$}; \node (C-delta3) [stage, right of = B-delta2, xshift=0.5cm]{$\delta_A = 3$}; \node (D-delta3) [stage, below of = D, xshift = -1cm,yshift=-1cm]{$\delta_A = 3$}; \node (D-delta1) [stage, below of = D, xshift = 1cm,yshift=-1cm]{$\delta_A = 1$}; \node (E-delta0) [stage, right of = D-delta1]{$\delta_A = 0$}; \node (W6) [stop, below of = C, yshift = -6cm, xshift = -6cm]{$C_2\wr S_3$}; \node (C2wrC3) [stop, below of = C, yshift = -6cm, xshift = -2cm]{$C_2\wr C_3$}; \node (D6) [stop, below of = C, yshift = -6cm, xshift = 2cm]{$D_6$}; \node (C6) [stop, below of = C, yshift = -6cm, xshift = 6cm]{$C_6$}; \draw[arrow] (start) -- (A-tag); \draw[arrow] (start) -- (B-tag); \draw[arrow] (start) -- (C-tag); \draw[arrow] (start) -- (D-tag); \draw[arrow] (start) -- (E-tag); \draw[arrow] (E) -- (E-delta0); \draw[arrow] (A) -- (A-delta3); \draw[arrow] (A) -- (A-delta1); \draw[arrow] (B) -- (B-delta3); \draw[arrow] (B) -- (B-delta2); \draw[arrow] (C) -- (C-delta3); \draw[arrow] (D) -- (D-delta3); \draw[arrow] (D) -- (D-delta1); \draw[arrow] (A-delta3) .. controls + (down:2cm) .. (W6); \draw[arrow] (A-delta3) .. controls + (down:2cm) .. (C2wrC3); \draw[arrow] (A-delta3) .. controls + (down:2cm) .. (D6); \draw[arrow] (A-delta3) .. controls + (down:2cm) .. (C6); \draw[arrow] (A-delta1) .. controls + (down:1cm) .. (D6); \draw[arrow] (A-delta1) .. controls + (down:1cm) .. (C6); \draw[arrow] (B-delta3) -- (W6); \draw[arrow] (B-delta3) -- (C2wrC3); \draw[arrow] (B-delta2) -- (C6); \draw[arrow] (C-delta3) -- (W6); \draw[arrow] (C-delta3) -- (C2wrC3); \draw[arrow] (C-delta3) -- (D6); \draw[arrow] (D-delta3) -- (W6); \draw[arrow] (D-delta3) -- (C2wrC3); \draw[arrow] (D-delta1) -- (D6); \draw[arrow] (D-delta1) -- (C6); \draw[arrow] (E-delta0) -- (C6); \end{tikzpicture} } \caption{Possible isomorphism classes of Galois groups of simple abelian threefolds, in terms of their Newton polygon and angle rank $\delta_A$.} \label{fig:flowchart3} \end{figure} \subsection{Outline} In \Cref{sec:background} we introduce background and notation used throughout this paper. A reader familiar with abelian varieties may choose to skip directly to \Cref{sec:weighted-perm-rep}, where we introduce a key tool in our paper, the weighted permutation representation. In \Cref{sec:warmup} we warm up by classifying weighted permutation representations of elliptic curves. In \Cref{sec:surfaces} and \Cref{sec:threefolds} we classify weighted permutation representations of abelian surfaces and simple abelian threefolds respectively. We finish by listing further inverse Galois questions in \Cref{sec:questions}. The code associated to this article is written in \texttt{Magma}~\cite{Magma} and is publicly available from the \GitHub repository \cite{OurElectronic}. \subsection{Acknowledgements} We thank Raymond van Bommel, Deewang Bhamidipati, Soumya Sankar, John Voight, and David Zureick-Brown for helpful conversations about this project. We also thank Everett Howe for several useful correspondences. SF was supported by the Woolf Fisher and Cambridge Trusts through a Woolf Fisher scholarship and by C{\'e}line Maistret's Royal Society Dorothy Hodgkin Fellowship. SV was supported by the National Science Foundation under grant number DMS2303211. \section{Background and notation}\label{sec:background} \subsection{Honda--Tate theory}\label{sec:HT-theory} Let $A/\FF_q$ be an abelian variety. A celebrated theorem of Honda and Tate classifies isogeny classes of abelian varieties over finite fields. Let $g \colonequals \dim(A) > 0$ and recall that $P_A(T) \in \ZZ[T]$ is the characteristic polynomial of Frobenius. The roots of $P_A(T)$ have absolute value $\sqrt{q}$ in all their complex embeddings; an algebraic integer with this property is called \cdef{$q$-Weil number}. A \cdef{$q$-Weil polynomial} is a monic integral polynomial whose roots are all $q$-Weil numbers. Fix an algebraic closure $\Qbar$ of $\QQ$ inside of $\CC$, and an embedding $\Qbar \hookrightarrow \Qbar_p$. We write $\nu$ for the $p$-adic valuation on $\Qbar_p$, normalized so that $\nu(q) = 1$. The statement below is the one presented in \cite[Theorem 4.2.12]{Poonen06}. \begin{theorem}[Honda--Tate Theorem] \label{thm:HT} \noindent \begin{enumerate} \item \label{enum:HT1} If $A$ is a simple abelian variety, then $P_A(T) = h_A(T)^e$ for some irreducible polynomial $h_A(T) \in \mathbb{Z}[T]$ and some $e \geq 1$. \item \label{enum:HT2} There is a bijection between isogeny classes of simple abelian varieties over $\mathbb{F}_q$ and conjugacy classes of $q$-Weil numbers. \item \label{enum:HT3} Given $\Gal(\Qbar/\QQ)$-conjugacy class of $q$-Weil numbers, let $h_A(T)$ be the minimal polynomial of any element of this conjugacy class. Then there exists a unique integer $e_A \colonequals e \geq 1$ such that $h_A(T)^e = P_A(T)$ for some simple abelian variety $A$ over $\Fq$. Moreover, $e$ is the smallest positive integer such that: \begin{enumerate} \item $h_A(0)^e > 0$, and \item For each monic $\QQ_p$-irreducible factor $g(T) \in \QQ_p[T]$ of $h_A(T)$, the valuation $\nu(g(0)^e)$ is in $\ZZ$. \end{enumerate} \end{enumerate} \end{theorem} The polynomial $h_A(T)$ is the \cdef{minimal polynomial of the $q$-Frobenius endomorphism of $A$}. When $P_A(T)$ is totally complex (i.e., has no real roots), the degree of $h_A(T)$ is equal to $2d$ for some positive integer $d$. In general, the isogeny factorization of $A$ yields a factorization of the Frobenius polynomial. In particular, by the Honda--Tate theorem, two abelian varieties $A$ and $B$ are isogenous if and only if $P_A(T)$ is equal to $P_B(T)$. This observation is sufficient to understand the classification of our three isogeny invariants in the case of elliptic curves (see \Cref{sec:warmup}). We now proceed to give more background that will be useful for higher dimensional abelian varieties. We will use the following as a running example throughout the article to consolidate our definitions and notations. This example appears in \cite[Example 6.1]{Shioda81} and \cite[Example 1.7]{Zywina22} (see also \cite[Example 4.2.9]{Gallese2024}). \begin{example}[Shioda's example] \label{example:Shioda} Let $q = p = 19$, and let $A$ be the Jacobian of the hyperelliptic curve $C$ with affine equation $y^2 = x^9 - 1$, defined over the field $\FF_{19}$. The curve $C$ has genus $g = 4$ and therefore $A$ is an abelian fourfold. By calculating $\#C(\FF_{19^r})$ for $r=1,2,3,4$, we are able to estimate the zeta function of $C$ to enough precision to recover the Frobenius polynomial $P_A(T)$. It is given by: \begin{equation*} P_A(T) = T^8 + 8T^7 + 28T^6 + 8T^5 - 170T^4 + 152T^3 + 10108T^2 + 54872T + 130321. \end{equation*} This polynomial factors as $P_A(T) = P_E(T)P_B(T)$, where $E$ is the elliptic curve $y^2 = x^3-1$ in the isogeny class \avlink{1.19.i} with Frobenius polynomial $P_E(T) = T^2 + 8T + 19$, and $B$ is an abelian threefold in the isogeny class \avlink{3.19.a_j_acm} with Frobenius polynomial $P_B(T) = T^6 + 9T^4 - 64T^3 + 171T^2 + 6859$. By the Honda--Tate theorem $A$ is isogenous to the product $E \times B$. \end{example} \subsection{Newton polygons of Frobenius polynomials} Let $A$ be a $g$-dimensional abelian variety over $\Fq$. At this moment, we do not assume that $A$ is simple. \begin{defn}[$q$-Newton polygon] \label{def:NP} The \cdef{$q$-Newton polygon} of $A$ is the $\nu$-adic Newton polygon of $P_A(T)$. More precisely, if $P_A(T) = \sum_{j = 0}^{2g} a_{2g-j}T^j$, then the $q$-Newton polygon of $A$ is the lower convex hull of the finite set \begin{equation*} \brk{(j,\nu(a_j)) : 0 \leq j \leq 2g,\mand a_j \neq 0} \subset \RR^2. \end{equation*} \end{defn} \begin{example} Continuing with \Cref{example:Shioda}, we note that both $E$ and $B$ are ordinary varieties. Since $A \sim E \times B$, the Newton polygon of $A$ is also ordinary, and it is obtained by concatenating those of $E$ and $B$. Alternatively, one can notice that $-170$ (the middle coefficient of $P_A(T)$) is not divisible by $p=19$. \end{example} \subsection{Angle rank of Frobenius polynomials} We recall the following definition from \cite{DupuyKedlayaRoeVincent22,DupuyKedlayaZureick-Brown22,APBS2023}. \begin{defn} \label{defn:angle-rank} Consider the multiplicative subgroup $U_A \subset \Qbar^\times$ generated by the normalized Frobenius eigenvalues $u \coloneqq \alpha/\sqrt{q}$ where $\alpha$ ranges over the roots of $h_A(T)$. The \cdef{angle rank of $A$} is denoted $\delta_A$ and is defined to be dimension of $U_A \otimes \QQ$. \end{defn} \subsection{Galois groups of Frobenius polynomials} Given an abelian variety $A$, denote by $R_A$ the \cdef{set of roots}, without multiplicity, of the Frobenius polynomial $P_A(T)$. \begin{defn}[Galois group] \label{def:gal-grp} The \cdef{Galois group} of $A$ is the Galois group of the minimal polynomial of Frobenius $h_A(T)$. Equivalently, $G_A$ is the largest quotient of $\Gal(\Qbar/\QQ)$ over which the permutation action on $R_A$ factors. \end{defn} \begin{defn} \label{def:K} In the case that $A$ is simple, we will denote by $K_A$ the center of the endomorphism algebra $\End(A)\otimes\QQ$. We have that $K_A \cong \QQ[T]/(h_A(T))$ is a number field. \end{defn} For ``most'' abelian varieties the polynomial $P_A(T)$ is totally complex (i.e., has no real roots), and $K_A$ is a \cdef{complex multiplication} (CM) number field. See \cite{Dodson1984} for some background on Galois groups of CM number fields, and \cite[Section 2.2]{DupuyKedlayaZureick-Brown22} for a discussion on Galois groups of $q$-Weil polynomials. \begin{example} \label{example:part-2} Continuing with \Cref{example:Shioda}, we have that the splitting field of $P_A(T)$ is the degree $6$ field $\QQ(\zeta_9)$, where $\zeta_9$ is a primitive $9^\text{th}$-root of unity. This already implies that the $8$ roots of $P_A(T)$ are algebraically dependent. Recalling that $A \sim E \times B$, observe that $K_E = \QQ(\sqrt{-3})$, which is contained in $K_B = \QQ(\zeta_9)$. Note that $G_A$ is a permutation group acting on the $8$ element set $R_A$. But as abstract groups, $G_A \cong \Gal(\QQ(\zeta_9)/\QQ) \cong C_6$. Denote by $R_E = \brk{\eta, 19\eta^{-1}}$ and $R_B = \brk{\alpha,\beta,\gamma,19\alpha^{-1},19\beta^{-1}, 19\gamma^{-1}}$ the sets of roots of $P_E(T)$ and $P_B(T)$ respectively. For an appropriate such labelling, we have \begin{equation} \label{eq:mult-rel-example} \eta = \dfrac{\alpha\cdot\beta\cdot\gamma}{19}. \end{equation} In \Cref{example:angle-rank} that the $\QQ$-vector space $U_A\otimes \QQ$ has dimension 3, i.e., $\delta_A = 3$. \end{example} Instead of viewing $G_A$ as a permutation subgroup of $S_n$ for $n = \# R_A$, it will be convenient to use a ``CM specific'' permutation representation. \subsection{The group of signed permutations}\label{sec:W2d} We now describe the abstract group which Galois groups of totally complex $q$-Weil numbers are naturally contained in. First note that when $h_A(T)$ is totally complex of degree $2d$ and the roots of $h_A(T)$ come in complex conjugate pairs. Moreover, the action of the Galois group $G_A$ respects this partition. Let $X_{2d}$ be the set consisting of the symbols $1, \bar{1}, \dots, d, \bar{d}$. Define $W_{2d}$ to be the subgroup of $\Sym(X_{2d})$ which preserves the partition \begin{equation*} X_{2d} = \brk{1,\bar{1}}\sqcup \cdots \sqcup \brk{d,\bar{d}}. \end{equation*} Upon a choice of labelling of roots, the Galois group of $h_A(T)$ may naturally be embedded in $W_{2d}$. We refer to the element $\iota \colonequals (1\bar{1})\ldots (d\bar{d})$ as \cdef{complex conjugation}. \subsubsection{Subgroup labelling} \label{sec:subgroup-labelling} We now briefly describe our naming conventions for subgroups of $W_{2d}$. A group $H$ is denoted $\texttt{G.d.t.letter.k}$ if it is isomorphic to $G$, contained in $W_{2d}$, and acts transitively on $X_{2d}$. The $W_{2d}$ conjugacy class is indexed by $\texttt{letter}$, and the groups in that conjugacy class are indexed by the tiebreaker $\texttt{k}$ (a positive integer). The use of $\texttt{nt}$ instead of $\texttt{t}$ indicates that $H$ acts intransitively on $X_{2d}$. For example, the group $\wl{C2.4.nt.c.1}$ refers to a group which is isomorphic to $C_2$, contained in $W_4$, and is intransitive. The label $\texttt{c}$ refers to the conjugacy class in $W_4$ and the index $\texttt{1}$ means that it is the first listed in its conjugacy class. \begin{remark} In the code associated to this article \cite{OurElectronic} we provide functions which compute and label the transitive subgroups of $W_{2d}$ which contain complex conjugation in the file \href{https://github.com/sarangop1728/Galois-Frob-Polys/blob/main/src/W2d-subgroups.m}{\texttt{src/W2d-subgroups.m}}. Our labelling convention is well defined and essentially follows lexicographic ordering of the subgroups of the symmetric group $S_{2d} \supset W_{2d}$ (as described in \cite{HulpkeLinton:TotalOrdering}). See the file \href{https://github.com/sarangop1728/Galois-Frob-Polys/blob/main/src/subgroup-labelling.m}{\texttt{src/subgroup-labelling.m}}. \end{remark} \section{The weighted permutation representation} \label{sec:weighted-perm-rep} In this section we introduce a key tool in this paper -- the notion of a weighted permutation representation. This construction is heavily inspired by the definition of \emph{Newton hyperplane arrangement} of Dupuy, Kedlaya, and Zureick-Brown \cite{DupuyKedlayaZureick-Brown22}. \subsection{The weighted permutation representation} \label{sec:perm-rep} We now describe the weighted permutation representation associated to an abelian variety, which is the central isogeny invariant in this article. It determines the Galois group, angle rank, and Newton polygon. The main purpose of our paper is to determine which weighted permutation representations occur from low dimensional abelian varieties. The weighted permutation representation is a reinterpretation of the \emph{Newton hyperplane representation} discussed in \cite{DupuyKedlayaZureick-Brown22}. \begin{defn}[Weighted permutation representations] \label{def:weighted-perm-rep} Given a finite group $G$, a \cdef{weighted permutation representation of $G$} is a pair $(w,\rho)$, where $w \colon X_{2d} \rightarrow \QQ_{\geq 0}$ is a map of sets and $\rho \colon G \xhookrightarrow{} W_{2d}$ is an inclusion of groups. We say that a pair of weighted permutation representations $(w, \rho)$ and $(w, \rho')$ of $G$ are \cdef{$w$-conjugate} if they are conjugate by an element of $\Stab(w)$, i.e., if there exists an element $\sigma \in W_{2d}$ such that $w = w \circ \sigma$ and $\rho' = \sigma^{-1} \rho(g) \sigma$ for all $g \in G$. \end{defn} \begin{defn}[Roots] \label{def:roots} Recall that for an abelian variety $A$ we write $R_A$ for the set of roots of $h_A(T)$ without multiplicity. We define $\rts_A$ to be the \cdef{multiset} of roots of $h_A(T)$, that is: \begin{enumerate} \item When $h_A(T)$ is totally complex, $\rts_A = R_A$. \item When $h_A(T)$ has a real root $\alpha$, it is counted with multiplicity two, and its duplicate is denoted by $\oalpha$. \end{enumerate} We define $d_A = \#\rts_A / 2$. \end{defn} \begin{defn}[Indexing and weighting] \label{def:indexing} An \cdef{indexing of roots} of $h_A(T)$ is a bijection $\indx\colon X_{2d} \to \rts_A$ which satisfies the following conditions: \begin{enumerate} \item $\indx$ respects complex conjugation, i.e., $\indx(\overline{k}) = \overline{\indx(k)}$ for each $1 \leq k \leq d$, and \item the indices climb the Newton polygon, i.e., \[ \nu(\alpha_i) \leq \nu(\alpha_j) \leq \nu(\oalpha_j) \leq \nu(\oalpha_i) \] for each pair of indices $1 \leq i \leq j \leq d$. \end{enumerate} \end{defn} Any indexing of the roots naturally gives a \cdef{weighting} $w_A \colon X_{2d} \rightarrow \QQ_{\geq 0}$ given by $k \mapsto \nu(\alpha_k)$ and $\bar{k} \mapsto \nu(\oalpha_k)$ for $k \in \brk{1,\dots,d}$. We omit the choice of indexing from the notation, since every choice of indexing yields the same weighting $w_A$. Note that the weighting $w_A$ associated to an abelian variety $A$ is uniquely determined by the $q$-Newton polygon of $A$. \begin{defn}[Weighted permutation representation, totally complex case] \label{def:perm-rep} Suppose $h_A(T)$ is totally complex. Given an indexing $\mathcal{I}$, we obtain representation \begin{equation*} \rho_{\mathcal{I}} \colon G_A \xhookrightarrow{} \Sym(X_{2d}) \cong S_{2d} \end{equation*} whose image lies in $W_{2d}$. The pair $(w_{A}, \rho_{\mathcal{I}})$ is the \cdef{weighted permutation representation} associated to $A$ with respect to the indexing $\mathcal{I}$. \end{defn} The definition above \emph{canonically} defines the weighted permutation representation associated to $A$ up to $w_A$-conjugacy when $P_A(T)$ is totally complex. The following definition gives a notion of weighted permutation representation when $P_A(T)$ is totally real; although the definition does not appear canonical, we will show later that it is in fact unique up to isomorphism. The simple isogeny classes of abelian varieties with real eigenvalues are highly constrained. \begin{lemma}[{\cite[pp.~528]{Waterhouse1969}}] \label{lem:real-eigenvalues} Let $A$ be a simple abelian variety whose Frobenius eigenvalues are real, then: \begin{enumerate}[label=(\arabic*)] \item \label{enum:real-ECs} If $q$ is a square, $A$ is a supersingular elliptic curve, and either $h_A(T) = (T \pm \sqrt{q})$ for some choice of sign, or \item \label{enum:complex-ECs} If $q$ is not a square, $A$ is a supersingular abelian surface, and $h_A(T) = (T^2-q)$. \end{enumerate} Moreover, in case~\ref{enum:real-ECs} we have $\rts_A = \brk{\sqrt{q}, \overline{\sqrt{q}}}$ or $\brk{-\sqrt{q}, -\overline{\sqrt{q}}}$, and in case~\ref{enum:complex-ECs} we have $\rts_A = \brk{\sqrt{q}, \overline{\sqrt{q}}, -\sqrt{q}, -\overline{\sqrt{q}}}$. \end{lemma} \begin{defn}[Weighted permutation representation, totally real case] \label{def:perm-rep-real} Suppose that $A$ has only real Frobenius eigenvalues. \begin{enumerate} \item[(1a)] If $q$ is a square and $A$ has one eigenvalue, then $G_A$ is trivial. Define $\rho_{\mathcal{I}} \colon G_A \to W_2$ to be the trivial map for all indexings. \item[(1b)] If $q$ is a square and $A$ has two eigenvalues, then $G_A$ is trivial. Define $\rho_{\mathcal{I}} \colon G_A \to W_4$ to be the trivial map for all indexings. \item[(2)] If $q$ is not a square, then $G_A \cong C_2$. Define $\rho_{\mathcal{I}} \colon G_A \to W_4$ to be the homomorphism sending the nontrivial element on $G_A$ to $(12)(\bar{1}\bar{2})$ for all indexings. \end{enumerate} The pair $(w_A, \rho_\mathcal{I})$ is the \cdef{weighted permutation representation} associated to $A$ with respect to the indexing $\mathcal{I}$. \end{defn} We now define the weighted permutation representation associated to an abelian variety $A$ for which $h_A(T)$ need not be totally real or totally complex; it is essentially the direct sum of its totally real and totally complex parts. In this case $A$ is isogenous to a product $B \times C$ where $h_B(T)$ is totally real and $h_C(T)$ is totally complex. Let $(w_B,\rho_B)$ and $(w_C,\rho_C)$ be the weighted permutation representations corresponding to the factors $B$ and $C$ respectively. Let $(w_B \oplus w_C, \rho_B \oplus \rho_C)$ be the direct sum of these weighted permutation representations; conjugate by $W_{2d}$ so that the weight function is nondecreasing, i.e., so that \[ w(i) \leq w(j) \leq w(\bar{j}) \leq w(\bar{i}) \] for $i \leq j$. We define the weighted permutation representation associated to $A$ to be the resulting weighted permutation representation. \Cref{lemma:well-def-conj-class} follows by construction. \begin{lemma} \label{lemma:well-def-conj-class} The weighted permutation representation associated to an abelian variety $A$ is well defined up to $w_A$-conjugacy (irrespective of indexing). In particular, its image in $W_{2d}$ is a subgroup $\cclass{A}$ which is well-defined up to $w_A$-conjugacy. \end{lemma} \subsection{The angle rank}\label{sec:newton-rep} It turns out that the angle rank can be computed from the weighted permutation representation of an abelian variety in the following way. \begin{defn} Given a weighted permutation representation $(w,\rho \colon G \xhookrightarrow{} W_{2d})$, define the \cdef{angle rank of $(w,\rho)$} to be $\rank(M) - 1$ where $M$ is the $(d \times |G|)$-matrix whose $i^\text{th}$-column has entries $w(\sigma(i))$ where $\sigma$ ranges over elements of $G$. \end{defn} The angle rank of $(w,\rho)$ is equal to the rank of the $(d \times |G|)$-matrix whose $i^\text{th}$-column has entries $w(\sigma(i)) - w(\sigma(\bar{i}))$, which is referred to as the Newton hyperplane matrix \cite[Remark 3.4]{DupuyKedlayaZureick-Brown22}. We show in \Cref{prop:newton-matrix-rank} that the angle rank of an abelian variety is equal to the angle rank of its weighted permutation. \begin{example}\label{example:angle-rank} Continuing with \Cref{example:part-2}, fix a prime $\mfp$ of the splitting field $K = \QQ(\zeta_9)$ above $p = 19$, and let $\nu$ be the extension of the $19$-adic valuation of $\QQ$ extended to $K$. An indexing of the roots $R_A$ according to \Cref{def:indexing} is for example \begin{equation} \alpha_1 = \alpha, \quad \alpha_2 = \beta, \quad \alpha_3 = \eta, \quad \alpha_4 = \gamma, \end{equation} if we ensure that $\nu(\alpha) = \nu(\beta) = \nu(\gamma) = \nu(\eta) = 0$. With this indexing, the weighted permutation representation $\rho\colon \Gal(\QQ(\zeta_9)/\QQ) \to W_8$ has image $H = \langle h \rangle$, where $h = (1\bar{2}\bar{4}\bar{1}24)(3\bar{3})$. With respect to this indexing, the multiplicative relation in \Cref{eq:mult-rel-example} becomes \begin{equation} \alpha_3 = \dfrac{\alpha_1\alpha_2\oalpha_4}{19} =\dfrac{\alpha_1\alpha_2}{\alpha_4}. \end{equation} One can find this multiplicative relation by considering the Newton hyperplane matrix of this weighted permutation representation, and computing its kernel. By direct calculation we see that the rank of the Newton hyperplane matrix (which is equal to $\delta_A$) is three. \end{example} \subsection{The divisor map} \label{sec:divisor-map} Many of the proofs in this paper make use of the same key idea, which we elucidate here. Given a simple totally complex $g$-dimensional abelian variety $A$ with Galois group $G_A$, we may construct the following. Let $L$ be the Galois closure of the field $K$, i.e., the field generated by the roots of $P_A(T)$. Let $\cP_L$ be the set of primes in $L$ above $p$. Let $\vecsp{\QQ}{R_A}$ denote the $\QQ$-vector space of dimension $2d$ whose basis consists of the formal symbols $[\alpha]$ where $\alpha$ ranges over the set of Frobenius eigenvalues. Let $\Div_{\QQ}(\OO_L)$ be the free $\QQ$-module supported on the prime ideals of $\OO_L$ and let $\div_A \colon \OO_L \to \Div_{\QQ}(\OO_L)$ be the map sending each element $x \in \OO_L$ to $ \sum_{\mfp} a_{\mfp} \mfp$ where $(x) = \prod_\mfp \mfp^{a_{\mfp}}$ is the prime factorization of the ideal generated by $x$. By abuse of notation we write \[ \div_A \colon \vecsp{\QQ}{R_A} \rightarrow \vecsp{\QQ}{\cP_L} \] for the $\QQ$-linear map given by linearly extending $\div_A$ on the roots $\alpha \in R_A$. Note that $\vecsp{\QQ}{R_A}$ naturally inherits the structure of a $G_A$-module from the action of $G_A$ on $R_A$, and similarly $\vecsp{\QQ}{\cP_L}$ has an action of $G_A$ inherited from the action of $G_A$ on $\cP_L$. The map $\div_A$ is $G_A$-equivariant. Note that $\div_A$ determines the weighted permutation representation, and thus also determines the angle rank, Newton polygon, and Galois group. It is natural to ask which $G_A$-module homomorphisms are permissible as the divisor map of a simple abelian variety. We list some necessary conditions which follow immediately from the construction. \begin{proposition} \label{prop:div-properties} Let $A$ be a simple abelian variety with no real Frobenius eigenvalues. Then: \begin{enumerate}[label=(\arabic*)] \item $\div_A([\alpha] + [\overline{\alpha}]) = \div_A(q)$ for all $\alpha \in R_A$; \item the $G_A$-action on $R_A$ is transitive; \item the $G_A$-action on $\cP_L$ is transitive; and \item \label{enum:div-property-4} if the Newton polygon has a segment of length $m$ which contains exactly two lattice points, then $\# \cP_L \mid \frac{1}{m}\# G_A$. \end{enumerate} \end{proposition} We now quickly use the divisor map to show the following lemma. \begin{lemma} \label{prop:newton-matrix-rank} The angle rank $\delta_A$ of an abelian variety is equal to the angle rank of its weighted permutation representation. Moreover, $\delta_A = \rank(\div_A) - 1$. \end{lemma} \begin{proof} Consider the subspace $V_p = \Span(\div_A(p)) \subset \vecsp{\QQ}{\cP_L}$. We first claim that the angle rank of an abelian variety is precisely the rank of the linear map $\vecsp{\QQ}{R_A} \rightarrow \vecsp{\QQ}{\cP_L}/ V_p$ induced by $\div_A$. Clearly a multiplicative relation between the Frobenius eigenvalues gives rise to an element in the kernel of this map. Conversely, given an element $\sum_{i = 1}^{2g} k_i\alpha_i$ in the kernel, we have \[ \bigg (\prod_i\alpha_i^{k_i}\bigg ) = (q)^{\sum k_i /2}, \] so there exists $u \in \OO_L^{\times}$ such that \[ \prod_iq^{-k_i/2}\alpha_i^{k_i} = u. \] Because $u$ has length $1$ in all complex embeddings, $u$ is a root of unity. Now observe that the rank of $\vecsp{\QQ}{R_A} \rightarrow \vecsp{\QQ}{\cP_L} / V_p$ is precisely one less than the rank of $\div_A \colon \vecsp{\QQ}{R_A} \rightarrow \vecsp{\QQ}{\cP_L}$. After removing duplicate rows and columns, the matrix representing the latter is precisely the matrix given in the definition of the angle rank of a weighted permutation representation. \end{proof} Observe that every such map $\vecsp{\QQ}{X_{2d}} \rightarrow \QQ^{\ell}$ satisfying the properties above gives rise to a weighted permutation representation. To prove that a certain weighted representation cannot occur, it suffices to show that there does not exist a lift $\QQ^{2d} \rightarrow \QQ^{\ell}$ satisfying the conditions above. \section{Warm-up: elliptic curves}\label{sec:elliptic-curves} \label{sec:warmup} We begin with the case when $A$ is an elliptic curve. In accordance with the labelling convention described in \Cref{sec:W2d} we write $\texttt{W2.2.t.a.1}$ and $\texttt{C1.2.t.a.1}$ for the subgroups of order $2$ and $1$ of $W_2$. The following \namecref{lemma:main-thm-ecs} is immediate. \begin{proposition} \label{lemma:main-thm-ecs} Let $A$ be an elliptic curve. Then the image, $\cclass{A}$, of the weighted permutation representation associated to $A$ is equal to $W_2$ except when $q$ is a square and $P_A(T) = (T \pm \sqrt{q})^2$, in which case $A$ is supersingular and $\cclass{A} = \{ \operatorname{id} \}$. More precisely, the possible combinations of Galois group, Newton polygon, and angle rank that occur are displayed in Tables~\ref{tab:EC-A}~and~\ref{tab:EC-B}. \end{proposition} \begin{table}[H] \setlength{\arrayrulewidth}{0.3mm} \setlength{\tabcolsep}{5pt} \renewcommand{\arraystretch}{1.2} \centering \begin{tabular}{|c|c|c|c|} \hline \rowcolor{headercolor} $w_A$-conjugacy class & Angle rank & Occurs & Example(s) \\ \hline $\texttt{W2.2.t.a.1}$ & $1$ & Yes & \avlink{1.2.ab} \\ \hline $\texttt{C1.2.nt.a.1}$ & $0$ & No & \\ \hline \end{tabular} \caption{The $w_A$-conjugacy classes of subgroups $G \subset W_2$ which occur as the image of the weighted permutation representation associated to an ordinary elliptic curve.} \label{tab:EC-A} \end{table} \begin{table}[H] \setlength{\arrayrulewidth}{0.3mm} \setlength{\tabcolsep}{5pt} \renewcommand{\arraystretch}{1.2} \centering \begin{tabular}{|c|c|c|c|} \hline \rowcolor{headercolor} $w_A$-conjugacy class & Angle rank & Occurs & Example(s) \\ \hline $\texttt{W2.2.t.a.1}$ & $0$ & Yes & \avlink{1.2.ac} \\ \hline $\texttt{C1.2.nt.a.1}$ & $0$ & Yes & \avlink{1.4.ae} \\ \hline \end{tabular} \caption{The $w_A$-conjugacy classes of subgroups $G \subset W_2$ which occur as the image of the weighted permutation representation associated to a supersingular elliptic curve.} \label{tab:EC-B} \end{table} \FloatBarrier \section{Abelian surfaces}\label{sec:surfaces} In this section we prove the following theorem. \begin{theorem} \label{thm:main-thm-surfaces} Let $A$ be an abelian surface. \begin{itemize} \item \thmemph{Permutation representations in $W_4$:} If $A$ has permutation representation contained in $W_4$, then the possible images $\cclass{A}$ of the weighted permutation representation associated to $A$ are given in Tables~\ref{tab:dim2-A}~and~\ref{tab:dim2-A-ns} when $A$ is ordinary, in Tables~\ref{tab:dim2-B}~and~\ref{tab:dim2-B-ns} when $A$ is almost ordinary, and in Tables~\ref{tab:dim2-C}~and~\ref{tab:dim2-C-ns} when $A$ is supersingular. The permutation representation determines whether $A$ is geometrically simple; this information can be seen in the tables as well. \item \thmemph{Permutation representations in $W_2$ when $A$ is simple:} If $A$ has a permutation representation contained in $W_2$ and is simple, then $A$ is supersingular and hence the angle rank is $0$. Both trivial Galois group and Galois group $C_2$ occur. $A$ is not geometrically simple. \item \thmemph{Permutation representations in $W_2$ when $A$ is not simple:} Now suppose $A$ has a permutation representation contained in $W_2$ and is not simple. Then $A$ is isogenous (over $\Fq$) to $E^2$ for some elliptic curve $E$ then the weighted permutation representation associated to $A$ takes values in $W_2$, and its image is equal to that associated to $E$ (and classified in \Cref{lemma:main-thm-ecs}). \end{itemize} \end{theorem} The rest of the section is dedicated to proving \Cref{thm:main-thm-surfaces}. The third point follows from the definition and the section on elliptic curves. The second point follows from the classification given in \cite{Xing1994}; see also \cite[Theorem~2.9~(SS2)]{MaisnerNart02}. In the remainder of this section we assume $A$ has permutation representation contained in $W_4$. \subsection{Permutation representations of subgroups of \texorpdfstring{$W_4$}{W4}} The group $W_4$ is isomorphic to $D_4$, the dihedral group of order $8$. The following lemma classifies the $w$-conjugacy classes of subgroups of $W_4$. \begin{lemma} \label{lemma:w-conj-w4} There are exactly $3$ transitive and $7$ intransitive subgroups of $W_4$ and each is recorded in \Cref{tab:W4-and-W6-subgroups}. Moreover, the only distinct subgroups which are $w_A$-conjugate are: \begin{enumerate} \item $\wl{C2.4.nt.b.1}$ and $\wl{C2.4.nt.b.2}$ when $A$ is either ordinary or supersingular, and \item $\wl{C2.4.nt.c.1}$ and $\wl{C2.4.nt.c.2}$ when $A$ is supersingular. \end{enumerate} Each $w_A$-conjugacy class gives rise to an isomorphism class of weighted permutation representation, and the angle ranks of these $w_A$-conjugacy classes $\cclass{} \subset W_4$ are recorded in Tables~\ref{tab:dim2-A}--\ref{tab:dim2-C-ns}. \end{lemma} \begin{proof} The first claim then follows by a direct calculation. The angle ranks are computed using our implementation of \Cref{prop:newton-matrix-rank} in the file \href{https://github.com/sarangop1728/Galois-Frob-Polys/blob/main/src/weighted-perm-rep.m}{\texttt{src/weighted-perm-rep.m}} of our \GitHub repository \cite{OurElectronic}. \end{proof} To prove \Cref{thm:main-thm-surfaces} it suffices to show that: \begin{enumerate} \item the cases we claim do not occur, actually do not occur; and \item in the cases that do occur, the weighted permutation representation determines whether the abelian surface is geometrically simple. \end{enumerate} In the remaining cases, we provide an example in Tables~\ref{tab:dim2-A}--\ref{tab:dim2-C-ns}, which realizes the given weighted permutation representation. We remark that the angle ranks displayed in the LMFDB are numerical approximations, but we verify these examples explicitly via a slower (but deterministic) algorithm; see the file \href{https://github.com/sarangop1728/Galois-Frob-Polys/blob/main/tables/verify-angle-rank.m}{\texttt{tables/verify-angle-rank.m}} in our \GitHub repository \cite{OurElectronic}. \subsection{Proof of \texorpdfstring{\Cref{thm:main-thm-surfaces}}{Theorem ??} in the simple case} For a simple abelian surface, the permutation representation acts transitively except when $A$ is supersingular with real Frobenius eigenvalues. \subsubsection{The ordinary case} In this case, every possible transitive weighted permutation representation occurs. Therefore, to prove the theorem in this case, it suffices to show that a simple ordinary abelian surface is not geometrically simple if and only if its weighted permutation representation is \wl{V4.4.t.a.1}. \begin{proposition} \label{lemma:simple-splits-power-of-EC} A simple abelian variety has a unique simple factor over every finite extension of the base field. In particular, if $A$ is simple of prime dimension and it splits over a finite extension of $\FF_q$, then it does so as the power of an elliptic curve. \end{proposition} \begin{proof} This follows immediately from \cite[Proposition 1.2.6.1]{chai-conrad-oort}. \end{proof} \begin{coro} \label{coro:not-geom-simple=>angle-rank-1} If $A$ is a simple abelian variety of prime dimension which is not geometrically simple and not supersingular, then $A$ is ordinary and has angle rank $1$. \end{coro} \begin{proof} Since $A$ is simple but not geometrically simple by \Cref{lemma:simple-splits-power-of-EC} there exists an extension $\FF_{q^k}$ of $\FF_q$ over which $A$ becomes isogenous to $E^g$ for some ordinary elliptic curve $E/\FF_{q^k}$ (in particular $A$ is ordinary). Angle rank is invariant under base change, so it follows that $\delta_A = 1$. \end{proof} \begin{lemma} \label{lemma:abs-simp-ord=>max-angle-rank} An ordinary geometrically simple abelian surface $A$ has angle rank $2$. \end{lemma} \begin{proof} We prove the contrapositive. Assume that $\delta_A < 2$. This implies that there exists a multiplicative relation among the normalized Frobenius eigenvalues $u_1^{r_1}u_2^{r_2} = 1$. Note that $r_1r_2 = 0$ implies that some $u_i$ is a root of unity, which would imply that $A$ is not ordinary. Thus, we have that both $r_1$ and $r_2$ are nonzero integers. Let $\alpha_1$ and $\alpha_2$ be the two Frobenius eigenvalues with $\nu(\alpha_1) = \nu(\alpha_2) = 0$. From the multiplicative relation we deduce that $r_1 = -r_2$, so that $(u_1/u_2)^{r_1} = 1$ and $\alpha_2 = \zeta_k\alpha_1$ where $\zeta_k$ is a $k^{\text{th}}$ root of unity. The Frobenius eigenvalues of the base change of $A$ to $\FF_{q^k}$ are precisely the $k$-th powers of the Frobenius eigenvalues of $A$. Because $\alpha_1^k = \alpha_2^k$, the Honda--Tate theorem implies that the base change of $A$ to $\FF_{q^k}$ is isogenous to the square of an elliptic curve, so $A$ is not geometrically simple. \end{proof} \begin{lemma} Let $A$ be a simple ordinary abelian surface. Then, exactly one of the following conditions holds. \begin{enumerate}[label=(\arabic*)] \item \label{enum:o-dim2-gs} $A$ is geometrically simple and $G_A \cong C_4$ or $W_4$. \item \label{enum:o-dim2-ngs} $A$ is not geometrically simple and $G_A \cong V_4$. \end{enumerate} \end{lemma} \begin{proof} By \Cref{lemma:abs-simp-ord=>max-angle-rank} if $A$ is geometrically simple then $A$ has angle rank 2, and by \Cref{coro:not-geom-simple=>angle-rank-1} if $A$ is not geometrically simple the it has angle rank $1$. The claim follows from the angle ranks computed in \Cref{lemma:w-conj-w4} (and recorded in \Cref{tab:dim2-A}). \end{proof} \subsubsection{The almost ordinary case} Every simple almost ordinary abelian surface is geometrically simple by \Cref{coro:not-geom-simple=>angle-rank-1}, so the following lemma completes the proof. \begin{lemma} \label{lemma:ao-dim2} A simple almost ordinary abelian surface has Galois group $W_4$. \end{lemma} \begin{proof} Suppose for the sake of contradiction that $A$ is an almost ordinary abelian variety with Galois group $C_4$ or $V_4$. Let $\div_A$ be the corresponding divisor map as discussed in \Cref{prop:div-properties}. By point \ref{enum:div-property-4}, we have $\# \cP_L \mid 2$, where $\cP_L$ is the set of primes above $p$ in $L$. From \Cref{prop:newton-matrix-rank}, we have $\delta_A = \rank(\div_A) -1 \leq \#\cP_L - 1$, which contradicts \Cref{tab:dim2-B}. \end{proof} \begin{remark} One can also see \Cref{lemma:ao-dim2} as follows: by the theory of Newton polygons and the Honda--Tate theorem, $P_A(T)$ has an irreducible linear and quadratic factor over $\QQ_p$, so the quartic extension $K/\QQ$ is not Galois. \end{remark} \subsubsection{The supersingular case} It follows from the Honda--Tate theorem that every supersingular abelian variety is geometrically isogenous to the power of an elliptic curve. The proof is concluded with the following well known result which shows that no supersingular abelian variety of dimension $g > 1$ can have Galois group $W_{2g}$. \begin{lemma} \label{lemma:big-G} Suppose $A$ has Galois group $G_A \cong W_{2d}$. Then either: \begin{enumerate} \item $A$ is a power of a supersingular elliptic curve $E$ with $P_E(T) \neq (T\pm\sqrt{q})^2$, or \item $A$ has angle rank $\delta_A = d$. \end{enumerate} \end{lemma} \begin{proof} Suppose that $A$ is not supersingular. Then, every normalized Frobenius eigenvalue $u = \alpha/\sqrt{q}$ satisfies $\nu(u) \neq 0$. Fix an indexing $\mathcal{I}\colon X_{2d} \to R_A$ and consider the vector $\mathbf{v} \colonequals (v_1, \dots, v_d)$ with nonzero entries $v_j \colonequals \nu(u_j)$. Acting on $\mathbf{v}$ by the transpositions $(k\bar{k})\in W_{2d}$, we see that the Newton hyperplane matrix contains the $(d \times d)$-minor \begin{equation*} \begin{bmatrix} v_1 & v_1 & \cdots & v_1 \\ v_2 & -v_2 & \cdots & v_2 \\ \vdots & \vdots & \ddots & \vdots \\ v_d & v_d & \cdots & -v_d \end{bmatrix}, \end{equation*} which is visibly similar to the diagonal matrix $\mathrm{diag}(v_1,\dots,v_d)$. \end{proof} \subsection{Proof of \texorpdfstring{\Cref{thm:main-thm-surfaces}}{Theorem ??} in the nonsimple case} If $A$ is isogenous to the square of an elliptic curve, then the claim follows immediately from \Cref{lemma:main-thm-ecs}. Therefore suppose that $A$ is isogenous over $\Fq$ to a product $E_1 \times E_2$ of non-isogenous elliptic curves. Note that when $A$ is supersingular, there is nothing to show so it suffices to consider the ordinary and almost ordinary cases. Let $\alpha_i$ and $\oalpha_i$ be the Frobenius eigenvalues of $E_i$ for each $i = 1, 2$. \subsubsection{Nonsimple ordinary abelian surfaces} In this case, both $E_1$ and $E_2$ are ordinary elliptic curves and the Frobenius eigenvalues $\alpha_i,\oalpha_i$ are not real, and therefore $\cclass{A}$ contains complex conjugation. It is now easy to see that the Galois group is $\wl{V4.4.nt.a.1}$ if $\QQ(\alpha_1) \neq \QQ(\alpha_2)$ and $\wl{C2.4.nt.a.1}$ otherwise. \begin{remark} In fact, using Lemma 5.3 of \cite{KrajicekScanlon00}, it is possible to show that in this case, the two elliptic curves are geometrically isogeneous if and only if the Galois group is $\wl{C2.4.nt.a.1}$, but we will not need this. \end{remark} \subsubsection{Nonsimple almost ordinary abelian surfaces} In this case we may assume without loss of generality that $E_1$ is ordinary and $E_2$ is supersingular. First note that $\cclass{A}$ cannot be the $w_A$-conjugate to $\wl{C2.4.nt.b.1}$ or $\wl{C1.4.nt.a.1}$ since in this case the Frobenius eigenvalues of $E_1$ are fixed by $G_A$, which cannot occur. \begin{lemma} Let $E_1$ and $E_2$ be elliptic curves over $\FF_q$. Suppose that $E_1$ is ordinary and $E_2$ is supersingular. Then, the corresponding number fields satisfy $K_1 \cap K_2 = \QQ$. \end{lemma} \begin{proof} If $K_2 = \QQ$ there is nothing to show. Suppose that this is not the case, and assume in search of a contradiction that $K_1 = K_2 = K$. Then $\alpha_1 = u \alpha_2$ for $u \in K$ of absolute value $1$. Note that $u$ can't be a root of unity, since that would imply that $\alpha_1$ is a supersingular $q$-Weil number. Since $\alpha_2$ is a supersingular $q$-Weil number, some power of $u$ is an \emph{algebraic integer}. This implies that $u$ is also an algebraic integer which has length $1$ in all complex embeddings, and thus $u \in \OO_K^\times$. But $K$ is quadratic imaginary, implying that $u$ is a root of unity, a contradiction. \end{proof} \FloatBarrier \section{Abelian threefolds}\label{sec:threefolds} \begin{theorem} \label{thm:main-thm-3folds} Let $A$ be a simple abelian threefold. \begin{itemize} \item \thmemph{Permutation representations in $W_6$}. If $A$ has permutation representation contained in $W_6$, then the possible images of the weighted permutation representation associated to a simple abelian threefold is given in Tables~\ref{tab:threefoldA}--\ref{tab:threefoldE}. Each table corresponds to one Newton polygon. The permutation representation determines whether $A$ is geometrically simple or not, and this information can be found in the tables. \item \thmemph{Permutation representations in $W_2$}. If the permutation representation of $A$ is not contained in $W_6$, then it is contained in $W_2$. In this case $e_A = 1$, where $e_A$ is as in the statement of the Honda Tate--theorem. Such abelian varieties have Galois group $C_2$, angle rank $1$, and Newton polygon type $(D)$. They are geometrically simple. \end{itemize} \end{theorem} The second point follows from \cite{Xing1994}; see \cite[Theorem 6.1.1, Lemma 6.1.2]{APBS2023} for more detail. In the remainder of this section we assume all abelian threefolds in question are simple and have permutation representation contained in $W_6$. We first classify the weighted permutation representations that occur, and then in \Cref{subsec:geom-simple} classify whether weighted permutation representations that occur are geometrically simple or not. The rest of this section is dedicated to proving \Cref{thm:main-thm-3folds}. \subsection{Signed permutations on three elements}\label{sec:signed-permutations-3} The following lemma follows by a direct calculation in \texttt{Magma}. See the file \href{https://github.com/sarangop1728/Galois-Frob-Polys/blob/main/src/W2d-subgroups.m}{\texttt{src/W2d-subgroups.m}} in our \GitHub repository \cite{OurElectronic}. \begin{lemma} \label{prop:trans-subs} There are exactly $10$ transitive subgroups in $W_6$ which contain the complex conjugation element $\iota \in W_6$, and each is listed in \Cref{tab:W4-and-W6-subgroups}. These $10$ subgroups are contained in exactly $4$ $W_6$-conjugacy classes, namely those of: \begin{enumerate} \item $W_6$, \item $C_2 \wr C_3$ (transitive label \texttt{6T6}) generated by $(123)(\bar{1}\bar{2}\bar{3})$, $(1\bar{1})$, $(2\bar{2}),$ and $(3\bar{3})$, \item $D_6$ generated by $(123)(\bar{1}\bar{2}\bar{3})$ and $(12)(\bar{1}\bar{2})$, \item $C_6$ generated by $(123\bar{1}\bar{2}\bar{3})$. \end{enumerate} Moreover, for each possible Newton polygon of an abelian threefold $A$, the $w_A$-conjugacy classes of transitive subgroups of $W_6$ are recorded in Tables~\ref{tab:threefoldA}--\ref{tab:threefoldE}. \end{lemma} Similarly to the case of surfaces, to prove \Cref{thm:main-thm-3folds} it suffices to show that: \begin{enumerate} \item the cases we claim do not occur, actually do not occur; and \item in the cases that do occur, the weighted permutation representation determines whether the abelian surface is geometrically simple (this is in \Cref{subsec:geom-simple}). \end{enumerate} In the remaining cases, we provide an example in Tables~\ref{tab:threefoldA}--\ref{tab:threefoldE} which realizes the given weighted permutation representation. We now prove $(1)$. \subsection{Proof of \texorpdfstring{\Cref{thm:main-thm-3folds}}{Theorem ??} in the ordinary case}\label{sec:o-3} In this case, the table shows that every possible weighted permutation representation occurs, so there is nothing to prove. \subsection{Proof of \texorpdfstring{\Cref{thm:main-thm-3folds}}{Theorem ??} in the almost ordinary case}\label{sec:ao-3} We first classify weighted permutation representations that occur, and then give proofs of geometric simplicity. The following two lemmas complete the classification of weighted permutation representations in the case of simple almost ordinary abelian threefolds. \begin{lemma} \label{lem:ao-not-C6} An almost ordinary abelian threefold cannot have Galois group $C_6$. \end{lemma} \begin{proof} Suppose for the sake of contradiction that $A$ is an almost ordinary abelian threefold with Galois group $G_A \cong C_6$. Note that all weighted permutation representations of $C_6$ are conjugate in the almost ordinary case, so it suffices to show that the image of the weighted permutation representation is not conjugate to $\wl{C6.6.t.a.2}$, which is generated by $\sigma = (123\bar{1}\bar{2}\bar{3})$. We now show that this weighted permutation representation doesn't lift to a divisor map with the properties listed in \Cref{prop:div-properties}. By \Cref{prop:div-properties}\ref{enum:div-property-4}, we have $\# \cP_L \mid 3$ where $\cP_L$ is the set of primes of $K = L$ dividing $p$. But then the action of $G_A$ on $\cP_L$ factors through $C_3$ and in particular the sequence \[ (\nu(\alpha_1), \nu(\sigma \alpha_1), ..., \nu(\sigma^5 \alpha_1)) = \left(0, 0, \tfrac{1}{2}, 1, 1, \tfrac{1}{2} \right) \] should be $3$-periodic, a contradiction. \end{proof} \begin{lemma} \label{lem:aoC6} An almost ordinary abelian threefold with Galois group $D_6$ has angle rank $2$. \end{lemma} \begin{proof} From \Cref{tab:threefoldB}, it suffices to show that the weighted permutation representation \wl{D6.6.t.a.1}, which has angle rank $3$, does not occur. Suppose for the sake of contradiction that $A$ is an almost ordinary abelian threefold with weighted permutation representation \wl{D6.6.t.a.1}. We exhibit a nontrivial multiplicative relation between the Frobenius eigenvalues (a contradiction to the angle rank being maximal) -- in particular we show that $\alpha_1\oalpha_3/q$ is a root of unity. \newcounter{claimcount}\setcounter{claimcount}{1} \claim[\theclaimcount]{$\#\cP_L = 6$.} \addtocounter{claimcount}{1} By \Cref{prop:div-properties}\ref{enum:div-property-4}, we have $\#\cP_L \mid 6$. The order $6$ cyclic subgroup of \wl{D6.6.t.a.1} is generated by $\sigma = (1 \bar{2} 3 \bar{1} 2 \bar{3})$. As in the proof of \Cref{lem:aoC6}, because the sequence \[ (\nu(\alpha_1), \nu(\sigma \alpha_1), ..., \nu(\sigma^5 \alpha_1 )) = \left( 0, 1, \tfrac{1}{2}, 1, 0, \tfrac{1}{2} \right) \] is not periodic, we must have $\#\cP_L = 6$. \claim[\theclaimcount]{$\alpha_1 \oalpha_3/q$ is a root of unity.} Let $\mfp_1$ be the prime of $\OO_L$ corresponding to the valuation $\nu$ and let \[ (\mfp_1,\op_2,\mfp_3,\op_1,\mfp_2,\op_3) = (\mfp_1, \sigma(\mfp_1),\dots,\sigma^5(\mfp_1)). \] Because $D_6$ has a unique transitive permutation representation on a $6$ element set, the action of $D_6$ on $\cP_L$ is exactly the rigid symmetries of the hexagon as depicted in \Cref{fig:hexagon}. Because \[ (\nu(\alpha_1), \nu(\sigma \alpha_1), ..., \nu(\sigma^5 \alpha_1)) = \left( 0, 1, \tfrac{1}{2}, 1, 0, \tfrac{1}{2} \right), \] we have \[ \div_A(\alpha_1) = en \big( [\op_3] + \tfrac{1}{2}[\mfp_2] + [\op_1] + \tfrac{1}{2} [\op_2] \big) \] where $q = p^n$ and $e$ is the ramification index of $p$ in $L$. Now consider the element $\tau = (1\bar{3})(2\bar{2})(\bar{1}3) \in \wl{D6.6.t.a.1}$ depicted in \Cref{fig:hexagon}. Note that $\tau$ is \emph{not} complex conjugation and in fact $\tau(\alpha_1) = \oalpha_3$. The action of $D_6$ on $\mathcal{P}_L$ allows us to compute $\div_A(\oalpha_3)$, and it is: \[ \div_A(\oalpha_3) = \tau(\div_A(\alpha_1)) = en \left( [\mfp_1] + \tfrac{1}{2}[\oq_2] + [\mfp_3] + \tfrac{1}{2}[\mfp_2] \right). \] We have $(\oalpha_3\alpha_1)\OO_L = q\OO_L$, so $\oalpha_3\alpha_1 = qu$ for some unit $u \in \OO_L^\times$. Because $\alpha_1$ and $\oalpha_3$ are both $q$-Weil numbers, the unit $u=\alpha_1\oalpha_3/q$ has absolute value $1$ over all complex places, thus it is a root of unity. \end{proof} \begin{figure}[ht] \centering \scalebox{0.99}{ \begin{tikzpicture} \draw[thick] (30:1) node[anchor=south west] {$\op_1$} -- (90:1) node[anchor=south] {$\mfp_3$} -- (150:1) node[anchor=south east] {$\op_2$} -- (210:1) node[anchor=north east] {$\mfp_1$} -- (270:1) node[anchor=north] {$\op_3$} -- (330:1) node[anchor=north west] {$\mfp_2$} -- cycle; \draw[dotted, thick, blue] (60:1.5) -- (240:1.5); \end{tikzpicture} } \caption{The action of $G_A$ on $\cP_L$. The permutation $\tau \in G_A$ is reflection with respect to the dotted line.} \label{fig:hexagon} \end{figure} \FloatBarrier \subsection{Proof of \texorpdfstring{\Cref{thm:main-thm-3folds}}{Theorem ??} in the Newton polygon (C) case} The following lemma completes the proof. \begin{lemma} An abelian threefold with Newton polygon (C) cannot have Galois group $G_A \cong C_6$. \end{lemma} \begin{proof} Suppose for the sake of contradiction that $G_A \cong C_6$. It suffices to show that the image of the weighted permutation representation is not conjugate to $\wl{C6.6.t.a.2}$, which is generated by $\sigma = (123\bar{1}\bar{2}\bar{3})$. By \Cref{prop:div-properties}\ref{enum:div-property-4}, we have $\# \cP_L \mid 3$. But then the action of $G_A$ on $\cP_L$ factors through $C_3$ and in particular the sequence \[ (\nu(\alpha_1), \nu(\sigma \alpha_1), ..., \nu(\sigma^5 \alpha_1)) = \left(0, \tfrac{1}{2}, \tfrac{1}{2}, 1, \tfrac{1}{2}, \tfrac{1}{2} \right) \] should be $3$-periodic, a contradiction. \end{proof} \subsection{Proof of \texorpdfstring{\Cref{thm:main-thm-3folds}}{Theorem ??} in the Newton polygon (D) case}\label{sec:non-ss-prank0-3} The following lemmas complete the proof. \begin{lemma} A type (D) abelian threefold cannot have weighted permutation representation whose image is $w_A$-conjugate to either $\wl{C6.6.t.a.2}$ or $\wl{D6.6.t.a.4}$. \end{lemma} \begin{proof} Suppose for the sake of contradiction that the image of the weighted permutation representation is $w_A$-conjugate to either \wl{C6.6.t.a.2} or \wl{D6.6.t.a.4}. By \Cref{prop:div-properties}\ref{enum:div-property-4}, we have $\# \cP_L \mid 4$ where $\cP_L$ is the set of primes of $L$ dividing $p$. But then the action of the order $6$ element $\sigma = (123\bar{1}\bar{2}\bar{3}) \in G_A$ on $\cP_L$ factors through $C_2$. But the sequence \[ (\nu(\alpha_1), \nu(\sigma\alpha_1), ..., \nu(\sigma^5\alpha_1)) = \left( \tfrac{1}{3}, \tfrac{1}{3}, \tfrac{1}{3}, \tfrac{2}{3}, \tfrac{2}{3}, \tfrac{2}{3} \right) \] is not $2$-periodic, a contradiction. \end{proof} \subsection{Proof of \texorpdfstring{\Cref{thm:main-thm-3folds}}{Theorem ??} in the supersingular case}\label{sec:ss-3} \begin{lemma} The sextic field generated by the Frobenius eigenvalues of a simple supersingular abelian threefold must be $\QQ(\zeta_7)$ or $\QQ(\zeta_9)$, both of which have Galois group $C_6$. \end{lemma} \begin{proof} We follow the argument in \cite[Proposition~2.1]{NartRitzenthaler08} (note that the characteristic of the base field in \textit{loc. cit.} is $2$). Supersingular abelian varieties have angle rank $0$, so the sextic field $K$ (as defined in \Cref{def:K}) is of the form $K \cong \QQ(\zeta_m \sqrt{q})$ where $\zeta_m$ is a primitive $m^\text{th}$-root of unity. Choose $m$ to be the smallest integer such that $\zeta_m\sqrt{q}$ generates $K$. If $q$ is a square, then the only cyclotomic fields of degree $6$ are $\QQ(\zeta_7)$ or $\QQ(\zeta_9)$, so we are done. Suppose now that $q$ is not a square. If $m$ is odd then $K$ contains the field $\QQ(\zeta_m)$. Thus $[\QQ(\zeta_m) : \QQ] \leq 6$ and $m = 3,7,9$. If $m = 3$ then $[K : \QQ] \leq [\QQ(\zeta_3,\sqrt{q}) : \QQ] = 4$. If $m = 7,9$, then $6 = [K : \QQ] \geq [\QQ(\zeta_m) : \QQ] = 6$, so $K = \QQ(\zeta_m)$. Now suppose $m = 2n$ is even. Since $K$ contains $\QQ(\zeta_m^2) = \QQ(\zeta_{n})$ and in particular we have $n = 3,4,6,7,9,14$. If $n = 7,9,14$ then $6 = [K : \QQ] \geq [\QQ(\zeta_{n}) : \QQ] = 6$, so $K = \QQ(\zeta_{n})$. If $n = 3$, then $[K : \QQ] \leq [\QQ(\zeta_6, \sqrt{q}) : \QQ] = 4$, contradiction. If $n = 6$, then $K = \QQ(\zeta_{12} \sqrt{q})$. However, this is a quadratic extension of the quadratic field $\QQ(\zeta_6)$, so $[K : \QQ] = 4$, which is a contradiction. \end{proof} \subsection{When are simple abelian threefolds geometrically simple?} \label{subsec:geom-simple} By the Honda--Tate theorem every supersingular abelian variety is not geometrically simple. Moreover, by \Cref{coro:not-geom-simple=>angle-rank-1} a simple ordinary threefold which is neither supersingular nor geometrically simple must be ordinary. Suppose that $A$ is a simple ordinary abelian threefold. In this case all possible weighted permutation representations occur, and it suffices to show that if $A$ is geometrically simple if it has angle rank $3$ and not geometrically simple if it has angle rank $1$. \begin{lemma} \label{lem:geom-simple<=>max-angle-rank} Let $A$ be a simple ordinary abelian threefold. Then the angle rank of $A$ is $3$ if $A$ is geometrically simple and $1$ otherwise. \end{lemma} \begin{proof} If $A$ is geometrically simple, then $\delta_A = 3$ by \cite[Lemma~6.2.2]{APBS2023}. If $A$ is not geometrically simple then $A$ has angle rank $1$ by \Cref{coro:not-geom-simple=>angle-rank-1}. \end{proof} \FloatBarrier \section{Inverse Galois Problems}\label{sec:questions} We state a slight refinement of a conjecture Dupuy, Kedlaya, Roe, and Vincent \cite[Conjecture 2.7]{DupuyKedlayaRoeVincent21}. \begin{conjecture}[Refined inverse Galois problem for abelian varieties] \label{conj:IGP} Fix a prime number $p$ and let $G \subset W_{2d}$ be a transitive subgroup containing the complex conjugation element. Then: \begin{enumerate} \item there exists an integer $r \geq 1$ and a simple abelian variety $A/\FF_{p^r}$ of dimension $d$ such that $G$ is $w_A$-conjugate to the image of the weighted permutation representation associated to $A$, and \item the abelian variety $A$ may be taken to be ordinary. \end{enumerate} In particular, $G$ is isomorphic to the Galois group of some abelian variety $A$. \end{conjecture} We note that \Cref{conj:IGP} is a very strong statement. In particular it implies the inverse Galois problem holds even when we are restricted to totally real fields. \begin{proposition} Assume \Cref{conj:IGP}, then the inverse Galois problem holds for totally real fields. More precisely, let $d \geq 1$ and let $G \subset S_d$ be a transitive subgroup, then $G$ is the Galois group of a polynomial $P^+(T)$ of degree $d$ over $\QQ$ and moreover the splitting field of $P^+(T)$ may be taken to be totally real. \end{proposition} \begin{proof} Let $G \subset S_d$ be a transitive subgroup, and consider the group $\widetilde{G} = C_2 \wr G$ equipped with its natural embedding $\widetilde{G} \hookrightarrow W_{2d}$. But $\widetilde{G}$ is a transitive subgroup of $W_{2d}$ which contains the complex conjugation element, so by assumption it occurs as the Galois group of a $q$-Weil polynomial $P_A(T)$. Let $P_A^+(T)$ be the \cdef{trace polynomial} of $P_A(T)$ defined by the equation \begin{equation*} P_A^+(T) = \prod_{\alpha}(T - (\alpha + \oalpha)), \end{equation*} where $\alpha$ ranges over the roots of $P_A(T)$. It follows by construction that $\Gal(P_A^+(T))$ is isomorphic to $G$ and that every root of $P_A^+(T)$ is real. \end{proof} \appendix\section{Tables} In \Cref{tab:W4-and-W6-subgroups} we record our labelling convention for subgroups of $W_4$ and $W_6$. In particular, we list every subgroup of $W_4$ and every transitive subgroup of $W_6$ containing complex conjugation. \subsection{Abelian surfaces} Tables~\ref{tab:dim2-A}--\ref{tab:dim2-C-ns} record the possible subgroups $G \subset W_4$ which may occur as the ($w_A$-conjugacy class of the) images of weighted permutation representations associated to abelian surfaces $A$. For each case which does occur, we provide an example from the LMFDB~\cite{lmfdb}. The tables are separated by Newton polygon (according to the conventions in \Cref{fig:flowchart2-simple}) and by whether $A$ is simple. \subsubsection{Simple abelian surfaces} These cases are treated in Tables~\ref{tab:dim2-A}--\ref{tab:dim2-C}. We do not list intransitive subgroups which do not occur as the image of the weighted permutation representation associated to a simple abelian surface -- note that an intransitive subgroup can only occur for the supersingular abelian surface in \Cref{lem:real-eigenvalues}\ref{enum:complex-ECs}. In each case we record whether every isogeny class of abelian surfaces with the recorded image of the weighted permutation representation is geometrically simple. \subsubsection{Non-simple abelian surfaces} These cases are treated in Tables~\ref{tab:dim2-A-ns}--\ref{tab:dim2-C-ns}. Since transitive subgroups of $W_4$ cannot occur as the image of the weighted permutation representation associated to an abelian surface, we do not record them. \subsection{Abelian threefolds} Tables~\ref{tab:threefoldA}--\ref{tab:threefoldE} record the possible subgroups $G \subset W_6$ which may occur as the ($w_A$-conjugacy class of the) images of weighted permutation representations associated to simple abelian threefolds $A$. For each case which does occur, we provide an example from the LMFDB~\cite{lmfdb}. The tables are separated by Newton polygon (according to the conventions in \Cref{fig:flowchart3}). \FloatBarrier { \vfill \begin{table}[ht] \setlength{\arrayrulewidth}{0.3mm} \setlength{\tabcolsep}{5pt} \renewcommand{\arraystretch}{1.15} \centering \begin{tabular}{|c|C{4.5cm}||c|C{4.5cm}|} \hline \rowcolor{headercolor} Label of $G$ & Generators of $G$ & Label of $G$ & Generators of $G$ \\ \hline \mkwl{W4.4.t.a.1} & $W_4$ & \mkwl{W6.6.t.a.1} & $W_6$ \\ \hline \mkwl{V4.4.t.a.1} & $\iota, (12)(\bar{1}\bar{2})$ & \mkwl{6T6.6.t.a.1} & $(123)(\bar{1}\bar{2}\bar{3})$, $(1\bar{1})$, $(2\bar{2})$, $(3\bar{3})$ \\ \hline \mkwl{C4.4.t.a.1} & $(12\bar{1}\bar{2})$ & \mkwl{D6.6.t.a.1} & $(1\bar{2}3\bar{1}2\bar{3}), (23)(\bar{2}\bar{3})$ \\ \hline \mkwl{V4.4.nt.a.1} & $(1\bar{1}), (2\bar{2})$ & \mkwl{D6.6.t.a.2} & $(12\bar{3}\bar{1}\bar{2}3)$, $(23)(\bar{2}\bar{3})$ \\ \hline \mkwl{C2.4.nt.a.1} & $\iota$ & \mkwl{D6.6.t.a.3} & $(1\bar{2}\bar{3} \bar{1}23)$, $(2\bar{3})(\bar{2}3)$ \\ \hline \mkwl{C2.4.nt.b.1} & $(1\bar{1})$ & \mkwl{D6.6.t.a.4} & $(123\bar{1}\bar{2}\bar{3})$, $(2\bar{3})(\bar{2}3)$ \\ \hline \mkwl{C2.4.nt.b.2} & $(2\bar{2})$ & \mkwl{C6.6.t.a.1} & $(1\bar{2}3\bar{1}2\bar{3})$ \\ \hline \mkwl{C2.4.nt.c.1} & $(12)(\bar{1}\bar{2})$ & \mkwl{C6.6.t.a.2} & $(123\bar{1}\bar{2}\bar{3})$ \\ \hline \mkwl{C2.4.nt.c.2} & $(1\bar{2})(\bar{1}2)$ & \mkwl{C6.6.t.a.3} & $(12\bar{3}\bar{1}\bar{2}3)$ \\ \hline \mkwl{C1.4.nt.a.1} & $\operatorname{id}$ &\mkwl{C6.6.t.a.4} & $(1\bar{2}\bar{3}\bar{1}23)$ \\ \hline \end{tabular} \caption{Labels for subgroups of $W_4$ (left) and subgroups of $W_6$ (right).} \label{tab:W4-and-W6-subgroups} \end{table} \begin{table}[ht] \setlength{\arrayrulewidth}{0.3mm} \setlength{\tabcolsep}{5pt} \renewcommand{\arraystretch}{1.2} \centering \begin{tabular}{|c|c|c|c|c|} \hline \rowcolor{headercolor} $w_A$-conjugacy class & Angle rank & Occurs & Geometrically simple & Example(s) \\ \hline \wl{W4.4.t.a.1} & $2$ & Yes & Yes & \avlink{2.2.ac_d} \\ \hline \wl{V4.4.t.a.1} & $1$ & Yes & No & \avlink{2.2.ad_f} \\ \hline \wl{C4.4.t.a.1} & $2$ & Yes & Yes & \avlink{2.3.ad_f} \\ \hline \end{tabular} \caption{The images of the weighted permutation representations associated to a simple ordinary abelian surface (Newton polygon (A) in \Cref{fig:flowchart2-simple}).} \label{tab:dim2-A} \end{table} \vfill } \begin{table}[ht] \setlength{\arrayrulewidth}{0.3mm} \setlength{\tabcolsep}{5pt} \renewcommand{\arraystretch}{1.2} \centering \begin{tabular}{|c|c|c|c|c|} \hline \rowcolor{headercolor} $w_A$-conjugacy class & Angle rank & Occurs & Geometrically simple & Example(s) \\ \hline \wl{W4.4.t.a.1} & $2$ & Yes & Yes & \avlink{2.2.ab_a} \\ \hline \wl{V4.4.t.a.1} & $2$ & No & & \\ \hline \wl{C4.4.t.a.1} & $2$ & No & & \\ \hline \end{tabular} \caption{The images of the weighted permutation representations associated to simple almost ordinary abelian surfaces (Newton polygon (B) in \Cref{fig:flowchart2-simple}).} \label{tab:dim2-B} \end{table} \begin{table}[p] \setlength{\arrayrulewidth}{0.3mm} \setlength{\tabcolsep}{5pt} \renewcommand{\arraystretch}{1.2} \centering \begin{tabular}{|c|c|c|c|c|} \hline \rowcolor{headercolor} $w_A$-conjugacy class & Angle rank & Occurs & Geometrically simple & Example(s) \\ \hline \wl{W4.4.t.a.1} & $2$ & No & & \\ \hline \wl{V4.4.t.a.1} & $0$ & Yes & No & \avlink{2.2.ac_c} \\ \hline \wl{C4.4.t.a.1} & $0$ & Yes & No & \avlink{2.4.ac_e} \\ \hline \wl{C2.4.nt.c.1} & \multirow{2}{*}{$0$} & \multirow{2}{*}{Yes} & \multirow{2}{*}{No} & \multirow{2}{*}{\avlink{2.2.a_ae}} \\ \wl{C2.4.nt.c.2} & & & & \\ \hline \end{tabular} \caption{The images of the weighted permutation representations associated to simple supersingular abelian surfaces (Newton polygon (C) in \Cref{fig:flowchart2-simple}).} \label{tab:dim2-C} \end{table} \begin{table}[p] \setlength{\arrayrulewidth}{0.3mm} \setlength{\tabcolsep}{5pt} \renewcommand{\arraystretch}{1.2} \centering \begin{tabular}{|c|c|c|c|} \hline \rowcolor{headercolor} $w_A$-conjugacy class & Angle rank & Occurs & Example(s) \\ \hline \wl{V4.4.nt.a.1} & $1$ & Yes & \avlink{2.3.ad_i} \\ \hline \wl{C2.4.nt.a.1} & $1$ & Yes & \avlink{2.2.a_d} \\ \hline \wl{C2.4.nt.b.1} & \multirow{2}{*}{$1$} & \multirow{2}{*}{No} & \\ \wl{C2.4.nt.b.2} & & & \\ \hline \wl{C2.4.nt.c.1} & $1$ & No & \\ \hline \wl{C2.4.nt.c.2} & $1$ & No & \\ \hline \wl{C1.4.nt.a.1} & $0$ & No & \\ \hline \end{tabular} \caption{The images of the weighted permutation representations associated to non-simple ordinary abelian surfaces (Newton polygon (A) in \Cref{fig:flowchart2-non-simple}).} \label{tab:dim2-A-ns} \end{table} \begin{table}[p] \setlength{\arrayrulewidth}{0.3mm} \setlength{\tabcolsep}{5pt} \renewcommand{\arraystretch}{1.2} \centering \begin{tabular}{|c|c|c|c|} \hline \rowcolor{headercolor} $w_A$-conjugacy class & Angle rank & Occurs & Example(s) \\ \hline \wl{V4.4.nt.a.1} & $1$ & Yes & \avlink{2.2.ad_g} \\ \hline \wl{C2.4.nt.a.1} & $1$ & No & \\ \hline \wl{C2.4.nt.b.1} & $1$ & No & \\ \hline \wl{C2.4.nt.b.2} & $1$ & Yes & \avlink{2.4.ah_u} \\ \hline \wl{C2.4.nt.c.1} & $1$ & No & \\ \hline \wl{C2.4.nt.c.2} & $1$ & No & \\ \hline \wl{C1.4.nt.a.1} & $0$ & No & \\ \hline \end{tabular} \caption{The images of the weighted permutation representations associated to a non-simple almost ordinary abelian surfaces (Newton polygon (B) in \Cref{fig:flowchart2-non-simple}).} \label{tab:dim2-B-ns} \end{table} \begin{table}[p] \setlength{\arrayrulewidth}{0.3mm} \setlength{\tabcolsep}{5pt} \renewcommand{\arraystretch}{1.2} \centering \begin{tabular}{|c|c|c|c|c|} \hline \rowcolor{headercolor} $w_A$-conjugacy class & Angle rank & Occurs & Example(s) \\ \hline \wl{V4.4.nt.a.1} & $0$ & Yes & \avlink{2.2.ac_e} \\ \hline \wl{C2.4.nt.a.1} & $0$ & Yes & \avlink{2.2.a_a} \\ \hline \wl{C2.4.nt.b.1} & \multirow{2}{*}{$0$} & \multirow{2}{*}{Yes} & \multirow{2}{*}{\avlink{2.4.ag_q}} \\ \wl{C2.4.nt.b.2} & & & \\ \hline \wl{C2.4.nt.c.1} & \multirow{2}{*}{$0$} & \multirow{2}{*}{No} & \\ \wl{C2.4.nt.c.2} & & & \\ \hline \wl{C1.4.nt.a.1} & $0$ & Yes & \avlink{2.4.a_ai} \\ \hline \end{tabular} \caption{The images of the weighted permutation representations associated to non-simple supersingular abelian surfaces (Newton polygon (C) in \Cref{fig:flowchart2-non-simple}).} \label{tab:dim2-C-ns} \end{table} \begin{table}[p] \setlength{\arrayrulewidth}{0.3mm} \setlength{\tabcolsep}{5pt} \renewcommand{\arraystretch}{1.2} \centering \begin{tabular}{|c|c|c|c|c|} \hline \rowcolor{headercolor} $w_A$-conjugacy class & Angle rank & Occurs & Geometrically simple & Example(s) \\ \hline \wl{W6.6.t.a.1} & $3$ & Yes & Yes & \avlink{3.2.ad_f_ah} \\ \hline \wl{6T6.6.t.a.1} & $3$ & Yes & Yes & \avlink{3.2.ad_g_aj} \\ \hline \wl{D6.6.t.a.1} & $1$ & Yes & No & \avlink{3.2.a_a_ad} \\ \hline \wl{D6.6.t.a.2} & \multirow{3}{*}{$3$} & \multirow{3}{*}{Yes} & \multirow{3}{*}{Yes} & \multirow{3}{*}{\avlink{3.2.ac_a_d}} \\ \wl{D6.6.t.a.3} & & & & \\ \wl{D6.6.t.a.4} & & & & \\ \hline \wl{C6.6.t.a.1} & $1$ & Yes & No & \avlink{3.2.ae_j_ap} \\ \hline \wl{C6.6.t.a.2} & \multirow{3}{*}{$3$} & \multirow{3}{*}{Yes} & \multirow{3}{*}{Yes} & \multirow{3}{*}{\avlink{3.7.ak_bw_afv}} \\ \wl{C6.6.t.a.3} & & & & \\ \wl{C6.6.t.a.4} & & & & \\ \hline \end{tabular} \caption{The images of the weighted permutation representations associated to simple ordinary abelian threefolds (Newton polygon (A) in \Cref{fig:flowchart3}).} \label{tab:threefoldA} \end{table} \begin{table}[p] \setlength{\arrayrulewidth}{0.3mm} \setlength{\tabcolsep}{5pt} \renewcommand{\arraystretch}{1.2} \centering \begin{tabular}{|c|c|c|c|c|} \hline \rowcolor{headercolor} $w_A$-conjugacy class & Angle rank & Occurs & Geometrically simple & Example \\\hline \wl{W6.6.t.a.1} & $3$ & Yes & Yes & \avlink{3.2.ab_ab_c} \\ \hline \wl{6T6.6.t.a.1} & $3$ & Yes & Yes & \avlink{3.4.ac_ab_g} \\ \hline \wl{D6.6.t.a.1} & \multirow{2}{*}{$3$} & \multirow{2}{*}{No} & & \\ \wl{D6.6.t.a.3} & & & & \\ \hline \wl{D6.6.t.a.2} & \multirow{2}{*}{$2$} & \multirow{2}{*}{Yes} & \multirow{2}{*}{Yes} & \multirow{2}{*}{\avlink{3.2.ac_b_a}} \\ \wl{D6.6.t.a.4} & & & & \\ \hline \wl{C6.6.t.a.1} & \multirow{2}{*}{$3$} & \multirow{2}{*}{No} & & \\ \wl{C6.6.t.a.4} & & & & \\ \hline \wl{C6.6.t.a.2} & \multirow{2}{*}{$2$} & \multirow{2}{*}{No} & & \\ \wl{C6.6.t.a.3} & & & & \\ \hline \end{tabular} \caption{The images of the weighted permutation representations associated to simple almost ordinary abelian threefold (Newton polygon (B) in \Cref{fig:flowchart3}).} \label{tab:threefoldB} \end{table} \begin{table}[p] \setlength{\arrayrulewidth}{0.3mm} \setlength{\tabcolsep}{5pt} \renewcommand{\arraystretch}{1.2} \centering \begin{tabular}{|c|c|c|c|c|} \hline \rowcolor{headercolor} $w$-conjugacy class & Angle rank & Occurs & Geometrically simple & Example \\ \hline \wl{W6.6.t.a.1} & $3$ & Yes & Yes & \avlink{3.2.ab_a_a} \\ \hline \wl{6T6.6.t.a.1} & $3$ & Yes & Yes & \avlink{3.4.ab_c_a} \\ \hline \wl{D6.6.t.a.1} & \multirow{4}{*}{$3$} & \multirow{4}{*}{Yes} & \multirow{4}{*}{Yes} & \multirow{4}{*}{\avlink{3.4.ab_a_ae}} \\ \wl{D6.6.t.a.2} & & & & \\ \wl{D6.6.t.a.3} & & & & \\ \wl{D6.6.t.a.4} & & & & \\ \hline \wl{C6.6.t.a.1} & \multirow{4}{*}{$3$} & \multirow{4}{*}{No} & & \\ \wl{C6.6.t.a.2} & & & & \\ \wl{C6.6.t.a.3} & & & & \\ \wl{C6.6.t.a.4} & & & & \\ \hline \end{tabular} \caption{The images of the weighted permutation representations associated to a simple abelian threefolds with Newton polygon (C) in \Cref{fig:flowchart3}.} \label{tab:threefoldC} \end{table} \begin{table}[p] \setlength{\arrayrulewidth}{0.3mm} \setlength{\tabcolsep}{5pt} \renewcommand{\arraystretch}{1.2} \centering \begin{tabular}{|c|c|c|c|c|} \hline \rowcolor{headercolor} $w$-conjugacy class & Angle rank & Occurs & Geometrically simple & Example \\ \hline \wl{W6.6.t.a.1} & $3$ & Yes & Yes & \avlink{3.2.ac_c_ac} \\ \hline \wl{6T6.6.t.a.1} & $3$ & Yes & Yes & \avlink{3.3.ad_j_ap} \\ \hline \wl{D6.6.t.a.1} & $1$ & Yes & Yes & \avlink{3.2.a_a_ac} \\ \hline \wl{D6.6.t.a.2} & \multirow{3}{*}{$3$} & \multirow{3}{*}{No} & & \\ \wl{D6.6.t.a.3} & & & & \\ \wl{D6.6.t.a.4} & & & & \\ \hline \wl{C6.6.t.a.1} & $1$ & Yes & Yes & \avlink{3.7.a_a_abj} \\ \hline \wl{C6.6.t.a.2} & \multirow{3}{*}{$3$} & \multirow{3}{*}{No} & & \\ \wl{C6.6.t.a.3} & & & & \\ \wl{C6.6.t.a.4} & & & & \\ \hline \end{tabular} \caption{The images of the weighted permutation representations associated to simple abelian threefolds with Newton polygon (D) in \Cref{fig:flowchart3}.} \label{tab:threefoldD} \end{table} \FloatBarrier \begin{table}[H] \setlength{\arrayrulewidth}{0.3mm} \setlength{\tabcolsep}{5pt} \renewcommand{\arraystretch}{1.2} \centering \begin{tabular}{|c|c|c|c|c|} \hline \rowcolor{headercolor} $w$-conjugacy class & Angle rank & Occurs & Geometrically simple & Example \\ \hline \wl{W6.6.t.a.1} & $0$ & No & & \\ \hline \wl{6T6.6.t.a.1} & $0$ & No & & \\ \hline \wl{D6.6.t.a.1} & \multirow{4}{*}{$0$} & \multirow{4}{*}{No} & & \\ \wl{D6.6.t.a.2} & & & & \\ \wl{D6.6.t.a.3} & & & & \\ \wl{D6.6.t.a.4} & & & & \\ \hline \wl{C6.6.t.a.1} & \multirow{4}{*}{$0$} & \multirow{4}{*}{Yes} & \multirow{4}{*}{No} & \multirow{4}{*}{\avlink{3.3.a_a_aj}} \\ \wl{C6.6.t.a.2} & & & & \\ \wl{C6.6.t.a.3} & & & & \\ \wl{C6.6.t.a.4} & & & & \\ \hline \end{tabular} \caption{The images of the weighted permutation representations associated to simple almost ordinary abelian threefolds (Newton polygon (E) in \Cref{fig:flowchart3}).} \label{tab:threefoldE} \end{table} \FloatBarrier \providecommand{\bysame}{\leavevmode\hbox to3em{\hrulefill}\thinspace} MR } \providecommand{\MRhref}[2]{ \href{http://www.ams.org/mathscinet-getitem?mr=#1}{#2} } \providecommand{\href}[2]{#2} \begin{thebibliography}{DKRV21b} \bibitem[ABS24]{APBS2023} Santiago {Arango-Pi{\~n}eros}, Deewang {Bhamidipati}, and Soumya {Sankar}, \emph{Frobenius {D}istributions of {L}ow {D}imensional {A}belian {V}arieties {O}ver {F}inite {F}ields}, Int. Math. Res. Not. IMRN (2024), no.~16, 11989--12020. \MR{4789105} \bibitem[AFV]{OurElectronic} Santiago {Arango-Piñeros}, Sam {Frengley}, and Sameera {Vemulapalli}, \emph{Git{H}ub repository}, \url{https://github.com/sarangop1728/Galois-Frob-Polys}. \bibitem[AS10]{AhmadiShparlinsky10} Omran Ahmadi and Igor~E. Shparlinski, \emph{On the distribution of the number of points on algebraic curves in extensions of finite fields}, Math. Res. Lett. \textbf{17} (2010), no.~4, 689--699. \MR{2661173} \bibitem[BCP97]{Magma} W.~{Bosma}, J.~{Cannon}, and C.~{Playoust}, \emph{The {M}agma algebra system. {I}. {T}he user language}, J. Symbolic Comput. \textbf{24} (1997), no.~3-4, 235--265, Computational algebra and number theory (London, 1993). \MR{1484478} \bibitem[BDKL24]{keliher2024} Alexandre Bailleul, Lucile Devin, Daniel Keliher, and Wanlin Li, \emph{Exceptional biases in counting primes over function fields}, J. Lond. Math. Soc. (2) \textbf{109} (2024), no.~3, Paper No. e12876, 32. \MR{4709829} \bibitem[CCO14]{chai-conrad-oort} Ching-Li Chai, Brian Conrad, and Frans Oort, \emph{Complex multiplication and lifting problems}, Mathematical Surveys and Monographs, vol. 195, American Mathematical Society, Providence, RI, 2014. \MR{3137398} \bibitem[DKRV21a]{DupuyKedlayaRoeVincent21} Taylor Dupuy, Kiran Kedlaya, David Roe, and Christelle Vincent, \emph{Counterexamples to a {C}onjecture of {A}hmadi and {S}hparlinski}, Experimental Mathematics (2021), 1--5. \bibitem[DKRV21b]{DupuyKedlayaRoeVincent22} \bysame, \emph{Isogeny {C}lasses of {A}belian {V}arieties over {F}inite {F}ields in the {LMFDB}}, Arithmetic Geometry, Number Theory, and Computation (Cham) (Jennifer~S. Balakrishnan, Noam Elkies, Brendan Hassett, Bjorn Poonen, Andrew~V. Sutherland, and John Voight, eds.), Springer International Publishing, 2021, pp.~375--448. \bibitem[DKZ24]{DupuyKedlayaZureick-Brown22} Taylor Dupuy, Kiran~S. Kedlaya, and David {Zureick-Brown}, \emph{Angle ranks of abelian varieties}, Math. Ann. \textbf{389} (2024), no.~1, 169--185. \MR{4735944} \bibitem[Dod84]{Dodson1984} B.~Dodson, \emph{The structure of {G}alois groups of {${\rm CM}$}-fields}, Trans. Amer. Math. Soc. \textbf{283} (1984), no.~1, 1--32. \MR{735406} \bibitem[GGL24]{Gallese2024} Andrea Gallese, Heidi Goodson, and Davide Lombardo, \emph{Monodromy groups and exceptional {H}odge classes}, 2024. \bibitem[HL03]{HulpkeLinton:TotalOrdering} Alexander Hulpke and Steve Linton, \emph{Total ordering on subgroups and cosets}, Proceedings of the 2003 {I}nternational {S}ymposium on {S}ymbolic and {A}lgebraic {C}omputation, ACM, New York, 2003, pp.~156--160. \MR{2035207} \bibitem[Hon68]{Honda1968} Taira Honda, \emph{Isogeny classes of abelian varieties over finite fields}, Journal of the Mathematical Society of Japan \textbf{20} (1968), no.~1-2, 83--95. \bibitem[KS00]{KrajicekScanlon00} Jan Kraj\'{\i}\v{c}ek and Thomas Scanlon, \emph{Combinatorics with definable sets: {E}uler characteristics and {G}rothendieck rings}, Bull. Symbolic Logic \textbf{6} (2000), no.~3, 311--330. \MR{1803636} \bibitem[{LMF}24]{lmfdb} The {LMFDB Collaboration}, \emph{The {L}-functions and modular forms database}, \url{https://www.lmfdb.org}, 2024, [Online; accessed 20 October 2024]. \bibitem[MN02]{MaisnerNart02} Daniel Maisner and Enric Nart, \emph{Abelian surfaces over finite fields as {J}acobians}, Experiment. Math. \textbf{11} (2002), no.~3, 321--337, With an appendix by Everett W. Howe. \MR{1959745} \bibitem[NR08]{NartRitzenthaler08} Enric Nart and Christophe Ritzenthaler, \emph{Jacobians in isogeny classes of supersingular abelian threefolds in characteristic 2}, Finite Fields Appl. \textbf{14} (2008), no.~3, 676--702. \MR{2435055} \bibitem[Poo06]{Poonen06} Bjorn Poonen, \emph{Lectures on rational points on curves}, \url{https://math.mit.edu/~poonen/papers/curves.pdf}, March 2006. \bibitem[Ser89]{Serre89} Jean-Pierre Serre, \emph{Abelian {$l$}-adic representations and elliptic curves}, second ed., Advanced Book Classics, Addison-Wesley Publishing Company, Advanced Book Program, Redwood City, CA, 1989, With the collaboration of Willem Kuyk and John Labute. \MR{1043865} \bibitem[Shi82]{Shioda81} Tetsuji Shioda, \emph{Algebraic cycles on abelian varieties of {F}ermat type}, Math. Ann. \textbf{258} (1981/82), no.~1, 65--80. \MR{641669} \bibitem[Tat66]{Tate1966} John Tate, \emph{Endomorphisms of abelian varieties over finite fields}, Inventiones mathematicae \textbf{2} (1966), no.~2, 134--144. \bibitem[Tat71]{Tate1971} \bysame, \emph{Classes d'isog\'{e}nie des vari\'{e}t\'{e}s ab\'{e}liennes sur un corps fini (d'apr\`es {T}. {H}onda)}, S\'{e}minaire {B}ourbaki. {V}ol. 1968/69: {E}xpos\'{e}s 347--363, Lecture Notes in Math., vol. 175, Springer, Berlin, 1971, pp.~Exp. No. 352, 95--110. \MR{3077121} \bibitem[Wat69]{Waterhouse1969} William~C. Waterhouse, \emph{Abelian varieties over finite fields}, Annales scientifiques de l'{\'E}cole normale sup{\'e}rieure, vol.~2, 1969, pp.~521--560. \bibitem[Xin94]{Xing1994} Chaoping Xing, \emph{The characteristic polynomials of abelian varieties of dimensions three and four over finite fields}, Sci. China Ser. A \textbf{37} (1994), no.~2, 147--150. \MR{1275799} \bibitem[Zar15]{Zarhin15} Yuri~G. Zarhin, \emph{Eigenvalues of {F}robenius endomorphisms of abelian varieties of low dimension}, Journal of Pure and Applied Algebra \textbf{219} (2015), no.~6, 2076--2098. \bibitem[Zar22]{Zarhin22} \bysame, \emph{Tate classes on self-products of abelian varieties over finite fields}, Ann. Inst. Fourier (Grenoble) \textbf{72} (2022), no.~6, 2339--2383. \MR{4500358} \bibitem[Zyw22]{Zywina22} David Zywina, \emph{Determining monodromy groups of abelian varieties}, Res. Number Theory \textbf{8} (2022), no.~4, Paper No. 89, 53. \MR{4496693} \end{thebibliography} \end{document}
2412.03500v2
http://arxiv.org/abs/2412.03500v2
$(σ, τ)$-Derivations of Number Rings with Coding Theory Applications
\documentclass[12pt]{article} \usepackage{graphicx} \usepackage{epstopdf}\usepackage{hyperref} \hypersetup{ colorlinks = true, urlcolor = blue, linkcolor = blue, citecolor = red } \usepackage[caption=false]{subfig} \usepackage{amsmath} \usepackage{csquotes} \usepackage{tikz} \usepackage{upgreek} \usepackage{amssymb} \usepackage{amsthm} \usepackage{amsfonts} \numberwithin{equation}{section} \usepackage{booktabs,caption} \usepackage[flushleft]{threeparttable} \usepackage{stmaryrd} \usepackage{wasysym} \usepackage{makecell} \usepackage{float} \usepackage{orcidlink} \usepackage{longtable} \newtheorem{theorem}{Theorem}[section] \newtheorem{definition}[theorem]{Definition} \newtheorem{proposition}[theorem]{Proposition} \newtheorem{lemma}[theorem]{Lemma} \newtheorem{algorithm}[theorem]{Algorithm} \newtheorem{corollary}[theorem]{Corollary} \newtheorem{notation}[theorem]{Notation} \newtheorem{remark}[theorem]{Remark} \newtheorem{note}[theorem]{Note} \theoremstyle{definition} \newtheorem{example}[theorem]{Example} \newtheorem{conjecture}[theorem]{Conjecture} \usepackage{nccmath} \usepackage{mathtools} \usepackage[reftex]{theoremref} \usepackage{tikz-cd} \usepackage[left=1in, right=1in, top=1in, bottom=1in]{geometry} \setcounter{MaxMatrixCols}{20} \begin{document} \title{$(\sigma, \tau)$-Derivations of Number Rings with Coding Theory Applications} \author{Praveen Manju and Rajendra Kumar Sharma} \date{} \maketitle \begin{center} \noindent{\small Department of Mathematics, \\Indian Institute of Technology Delhi, \\ Hauz Khas, New Delhi-110016, India$^{1}$} \end{center} \footnotetext[1]{{\em E-mail addresses:} \url{[email protected]}(Corresponding Author: Praveen Manju), \url{[email protected]}(Rajendra Kumar Sharma).} \medskip \begin{abstract} In this article, we study $(\sigma, \tau)$-derivations of number rings by considering them as commutative unital $\mathbb{Z}$-algebras. We begin by characterizing all $(\sigma, \tau)$-derivations and inner $(\sigma, \tau)$-derivations of the ring of algebraic integers of a quadratic number field. Then we characterize all $(\sigma, \tau)$-derivations of the ring of algebraic integers $\mathbb{Z}[\zeta]$ of a $p^{\text{th}}$-cyclotomic number field $\mathbb{Q}(\zeta)$ ($p$ odd rational prime and $\zeta$ a primitive $p^{\text{th}}$-root of unity). We also conjecture (using SAGE and MATLAB) an \enquote{if and only if} condition for a $(\sigma, \tau)$-derivation $D$ on $\mathbb{Z}[\zeta]$ to be inner. We further characterize all $(\sigma, \tau)$-derivations and inner $(\sigma, \tau)$-derivations of the bi-quadratic number ring $\mathbb{Z}[\sqrt{m}, \sqrt{n}]$ ($m$, $n$ distinct square-free rational integers). In each of the above cases, we also determine the rank and an explicit basis of the derivation algebra consisting of all $(\sigma, \tau)$-derivations of the number ring. Finally, we give the applications of the above work in coding theory by giving the notion of a Hom-IDD code. \end{abstract} \textbf{Keywords:} $(\sigma, \tau)$-derivation, inner $(\sigma, \tau)$-derivation, number ring, ring of algebraic integers, quadratic, cyclotomic, bi-quadratic \textbf{Mathematics Subject Classification (2010):} Primary: 13N15, 11R04; Secondary: 11C20, 11R11, 11R16, 11R18. \section{Introduction}\label{section 1} Let us denote a commutative ring with unity by $R$, an algebra over $R$ by $\mathcal{A}$, and two different non-zero $R$-algebra endomorphisms of $\mathcal{A}$ by $\sigma$, $\tau$. An $R$-linear map $d:\mathcal{A} \rightarrow \mathcal{A}$ that satisfies $d(\alpha \beta) = d(\alpha) \beta + \alpha d(\beta)$ for every $\alpha, \beta \in \mathcal{A}$, is called a derivation on $\mathcal{A}$. It is called inner if $d(\alpha) = \beta \alpha - \alpha \beta$ for all $\alpha \in \mathcal{A}$, for some $\beta \in \mathcal{A}$. An $R$-linear map $D:\mathcal{A} \rightarrow \mathcal{A}$ that satisfies $D(\alpha \beta) = D(\alpha) \tau(\beta) + \sigma(\alpha) D(\beta)$ for all $\alpha, \beta \in \mathcal{A}$, is called a $(\sigma, \tau)$-derivation on $\mathcal{A}$. Again, it is called inner if $D(\alpha) = \beta \tau(\alpha) - \sigma(\alpha) \beta$ for every $\alpha \in \mathcal{A}$, for some $\beta \in \mathcal{A}$. $\mathcal{D}_{(\sigma, \tau)}(\mathcal{A})$ denotes the set of all $(\sigma, \tau)$-derivations on $\mathcal{A}$. Defining componentwise sum and module action, $\mathcal{D}_{(\sigma, \tau)}(\mathcal{A})$ becomes an $R$- as well as $\mathcal{A}$-module. If $1$ is the unity in $\mathcal{A}$, $\sigma$, $\tau$ are unital, that is, $\sigma(1) = \tau(1) = 1$ and $D$ is a $(\sigma, \tau)$-derivation of $\mathcal{A}$, then $D(1) = 0$. We shall need some definitions and results from algebraic number theory for which we refer the reader to \cite{Marcus2018}, \cite{IanStewart2002}. Still, we summarize a few, as follows: A complex number is an algebraic number if it is a zero of some non-constant polynomial over $\mathbb{Q}$. A number field is a finite field extension of $\mathbb{Q}$ that contains algebraic numbers. A number ring is any subring of a number field. An algebraic integer is a complex number that is a zero of some monic polynomial over $\mathbb{Z}$. The ring of algebraic integers of a number field $K$, denoted by $O_{K}$, is defined as $O_{K} = K \cap \mathbb{B}$, where $\mathbb{B}$ is the ring of all algebraic integers. Thus $O_{K}$ contains precisely all those algebraic integers which belong to $K$. A subring of $O_{K}$ which is also a $\mathbb{Z}$-module of rank $[K: \mathbb{Q}]$ is called an order of $K$. A set of algebraic numbers $\{\alpha_{1}, \alpha_{2}, ..., \alpha_{r}\}$ is called an integral basis of $K$ (or $O_{K}$) if each element in $O_{K}$ is uniquely expressible as a $\mathbb{Z}$-linear combination of $\alpha_{1}, \alpha_{2}, ..., \alpha_{r}$. Every integral basis is a $\mathbb{Q}$-basis. An integral basis having form $\{1, \theta, ..., \theta^{n-1}\}$ for some $\theta \in O_{K}$, is called a power basis of $K$. A number field $K$ that has a power basis is called monogenic. A number field $K$ that has degree $2$ over $\mathbb{Q}$ is called quadratic. A number field that has the form $K = \mathbb{Q}(\zeta)$, where $\zeta = e^{2 \pi i / m}$ ($m \geq 1$) is a primitive complex $m^{\text{th}}$ root of unity, is called an $m^{\text{th}}$ cyclotomic field. We state below some results that we may need later. \begin{theorem}[{\cite[Theorem 2.2]{IanStewart2002}}]\th\label{theorem 1.1} Let $K$ be a number field. Then $K = \mathbb{Q}(\theta)$ for some algebraic number $\theta$, or in other words, number fields are finite simple extensions of $\mathbb{Q}$. In fact, $K = \mathbb{Q}(\theta)$ for some algebraic integer $\theta$. \end{theorem} \begin{theorem}[{\cite[Theorem 2.16]{IanStewart2002}}]\th\label{theorem 1.2} A number field $K$ always has an integral basis. Further, the additive group of $O_{K}$ is free abelian having rank equal to the degree of $K$. \end{theorem} \begin{theorem}[{\cite[Proposition 3.1]{IanStewart2002}}]\th\label{theorem 1.3} Every quadratic field is of the form $\mathbb{Q}(\sqrt{d})$ for some square-free rational integer $d$. \end{theorem} \begin{theorem}[{\cite[Theorem 3.2]{IanStewart2002}}]\th\label{theorem 1.4} Let $K = \mathbb{Q}(\sqrt{d})$ be a quadratic field. Then \begin{itemize} \item[(i)] $O_{K} = \mathbb{Z}[\sqrt{d}]$ when $d \not\equiv 1 \hspace{0.1cm} (\text{mod} \hspace{0.1cm} 4)$, \item[(ii)] $O_{K} = \mathbb{Z}[\frac{1 + \sqrt{d}}{2}]$ when $d \equiv 1 \hspace{0.1cm} (\text{mod} \hspace{0.1cm} 4)$. \end{itemize} \end{theorem} \begin{theorem}[{\cite[Theorem 3.3]{IanStewart2002}}]\th\label{theorem 1.5} Let $K = \mathbb{Q}(\sqrt{d})$ be a quadratic field. Then $\mathcal{B}$ is an integral basis of $K$, where \begin{itemize} \item[(a)] $\mathcal{B} = \{1, \sqrt{d}\}$ when $d \not\equiv 1 \hspace{0.1cm} (\text{mod} \hspace{0.1cm} 4)$. \item[(b)] $\mathcal{B} = \{1, \frac{1 + \sqrt{d}}{2}\}$ when $d \equiv 1 \hspace{0.1cm} (\text{mod} \hspace{0.1cm} 4)$. \end{itemize} \end{theorem} \begin{theorem}[{\cite[Lemma 3.4]{IanStewart2002}}]\th\label{theorem 1.6} If $p$ is an odd prime and $\zeta$ is a primitive $p^{\text{th}}$ root of unity, then the minimal polynomial of $\zeta$ over $\mathbb{Q}$ is $\phi_{p}(x) = x^{p-1} + x^{p-2} + ... + 1$. Hence $[\mathbb{Q}(\zeta):Q] = p-1$. \end{theorem} \begin{theorem}[{\cite[Theorem 3.5]{IanStewart2002}}]\th\label{theorem 1.7} Let $p$ be an odd prime and $K = \mathbb{Q}(\zeta)$ be a $p^{\text{th}}$ cyclotomic field. Then $O_{K} = \mathbb{Z}[\zeta]$. Hence, $\{1, \zeta, \zeta^{2}, ..., \zeta^{p-2}\}$ is an integral basis of $K = \mathbb{Q}(\zeta)$. \end{theorem} \begin{theorem}[{\cite[Chapter 2]{Marcus2018}}]\th\label{theorem 1.8} Let $m \in \mathbb{N}$ and $K = \mathbb{Q}(\zeta)$ be an $m^{\text{th}}$ cyclotomic field, then $O_{K} = \mathbb{Z}[\zeta]$. \end{theorem} Derivations have been studied in various fields like algebra, functional analysis, Fourier analysis, measure theory, operator theory, harmonic analysis, differential equations, differential geometry, etc. They play an essential role in many important branches of mathematics and physics. The notion of derivation in rings has been applied to and studied in various algebras to produce their theory of derivations. For example, BCI-algebras \cite{Muhiuddin2012}, von Neumann algebras \cite{Brear1992}, incline algebras which have many applications \cite{Kim2014}, MV-algebras \cite{KamaliArdakani2013}, \cite{Mustafa2013}, Banach algebras \cite{Raza2016}, lattices that have a very crucial role in various fields like information theory: information recovery, management of information access and cryptanalysis \cite{Chaudhry2011}. Differentiable manifolds, operator algebras, $\mathbb{C}^{*}$-algebras, and representation theory of Lie groups are continued to be studied using derivations \cite{Klimek2021}. Derivations on rings and algebras help in the study of their structure. For a historical account and more applications of derivations, we refer the reader to \cite{MohammadAshraf2006}, \cite{Atteya2019}, \cite{Haetinger2011}. The idea of a $(s_{1}, s_{2})$-derivation was introduced by Jacobson \cite{Jacobson1964}. These derivations were later on commonly called as $(\sigma, \tau)$ or $(\theta, \phi)$-derivation. These derivations have been highly studied in prime and semiprime rings and have been principally used in solving functional equations \cite{Brear1992}. Derivations, and especially $(\sigma, \tau)$-derivations of rings have various applications in coding theory \cite{Boucher2014}, \cite{Creedon2019}. For more applications of $(\sigma, \tau)$-derivations, we refer the reader to \cite{AleksandrAlekseev2020}. There is no literature available on twisted derivations of number rings except in \cite{Chaudhuri} where the author studies $(\sigma, \tau)$-derivations of the ring of algebraic integers of a quadratic number field. In this article, we mainly study $(\sigma, \tau)$-derivations of number rings. We partition the article into three sections. In Section \ref{section 2}, we first obtain results on $(\sigma, \tau)$-derivations and inner $(\sigma, \tau)$-derivations of a commutative unital algebra $\mathcal{A}$ over a commutative ring $R$ with unity $1$. We find a necessary condition for an $R$-linear map $D$ on $\mathcal{A}$ to be a $(\sigma, \tau)$-derivation and give counterexamples showing that the condition is not sufficient. This motivates us to study these derivations in certain number rings where this condition is both necessary as well as sufficient. We also determine a necessary and sufficient condition for a $(\sigma, \tau)$-derivation on $\mathcal{A}$ to be inner. In Section \ref{section 3}, we apply our results obtained in Section \ref{section 2} to study $(\sigma, \tau)$-derivations and inner $(\sigma, \tau)$-derivations of number rings by considering them as commutative unital $\mathbb{Z}$-algebras. Section \ref{section 3} is further subdivided into three subsections. In Subsection \ref{subsection 3.1}, we study $(\sigma, \tau)$-derivations and inner $(\sigma, \tau)$-derivations of the ring of algebraic integers of a quadratic number field. We obtain necessary and sufficient conditions for a $\mathbb{Z}$-linear map $D$ on the rings of algebraic integers of a quadratic field to be a $(\sigma, \tau)$-derivation. We also classify the inner $(\sigma, \tau)$-derivations of the ring of algebraic integers of a quadratic field. In Subsection \ref{subsection 3.2}, we study $(\sigma, \tau)$-derivations and inner $(\sigma, \tau)$-derivations of the ring of algebraic integers of a cyclotomic number field. We obtain necessary and sufficient conditions for a $\mathbb{Z}$-linear map $D$ on the rings of algebraic integers $\mathbb{Z}[\zeta]$ of a $p^{\text{th}}$ cyclotomic field $\mathbb{Q}(\zeta)$ to be a $(\sigma, \tau)$-derivation. We prove that $\mathcal{D}_{(\sigma, \tau)}(\mathbb{Z}[\zeta])$ is a $\mathbb{Z}$-module having rank $p-1$. Further, in this process, we also propose two conjectures giving a necessary and sufficient condition on a $(\sigma, \tau)$-derivation of $\mathbb{Z}[\zeta]$ to be inner. This is done with the help of SAGE and MATLAB. In Subsection \ref{subsection 3.3}, a beautiful application of \th\ref{lemma 2.1} is given in determining all $(\sigma, \tau)$-derivations of the number ring $\mathbb{Z}[\sqrt{m}, \sqrt{n}]$ for every pair $(\sigma, \tau)$ of distinct ring endomorphisms $\sigma$ and $\tau$ of $\mathbb{Z}[\sqrt{m}, \sqrt{n}]$. As a result, we establish that $D_{(\sigma, \tau)}(\mathbb{Z}[\sqrt{m}, \sqrt{n}])$ is a $\mathbb{Z}$-module having rank $4$. Further, we obtain a necessary and sufficient condition for a $(\sigma, \tau)$-derivation of $\mathbb{Z}[\sqrt{m}, \sqrt{n}]$ to be inner. We discuss the applications of our work in coding theory in Section \ref{section 4} by giving the notion of a Hom-IDD code. In Section \ref{section 5}, we conclude our findings. \section{Some useful results on \texorpdfstring{$(\sigma, \tau)$}{Lg}-derivations of commutative algebras}\label{section 2} $\mathcal{A}$, in this section, denotes a commutative algebra with unity $1$ over a commutative ring $R$ having unity $1$ and $\sigma$, $\tau$ are two different non-zero unital $R$-algebra endomorphisms of $\mathcal{A}$. First, we have an important lemma below. \begin{lemma}\th\label{lemma 2.1} Let $\mathcal{A}$ be of finite rank $n$ as an $R$-module and let $\{\alpha_{1}, \alpha_{2}, ..., \alpha_{n}\}$ be an $R$-basis of $\mathcal{A}$. Then an $R$-linear map $D:\mathcal{A} \rightarrow \mathcal{A}$ is a $(\sigma, \tau)$-derivation if and only if $$D(\alpha_{i} \alpha_{j}) = D(\alpha_{i}) \tau(\alpha_{j}) + \sigma(\alpha_{i}) D(\alpha_{j})$$ for every $i, j \in \{1, 2, ..., n\}$. \end{lemma} \begin{proof} Let $D:\mathcal{A} \rightarrow \mathcal{A}$ be an $R$-linear map, and $x, y \in \mathcal{A}$. Then $x = \sum_{i=1}^{n} a_{i} \alpha_{i}$ and $y = \sum_{j=1}^{n} b_{j} \alpha_{j}$ for some $a_{i}, b_{j} \in R$ ($i, j \in \{1, 2, ..., n\}$). Observe that $$xy = \left(\sum_{i=1}^{n} a_{i} \alpha_{i}\right) \left(\sum_{j=1}^{n} b_{j} \alpha_{j} \right) = \sum_{i, j} (a_{i}b_{j}) (\alpha_{i} \alpha_{j}),$$ and \begin{eqnarray*} D(xy) & = & \sum_{i,j=1}^{n} (a_{i}b_{j}) D(\alpha_{i} \alpha_{j}) = \sum_{i,j=1}^{n} (a_{i}b_{j}) (D(\alpha_{i}) \tau(\alpha_{j}) + \sigma(\alpha_{i}) D(\alpha_{j})) \\ & = & D \left(\sum_{i=1}^{n} a_{i}\alpha_{i}\right) \tau \left(\sum_{j=1}^{n} b_{j} \alpha_{j}\right) + \sigma \left(\sum_{i=1}^{n} a_{i}\alpha_{i}\right) D \left(\sum_{j=1}^{n} b_{j} \alpha_{j}\right) \\ & = & D(x) \tau(y) + \sigma(x) D(y). \end{eqnarray*} Hence $D$ is a $(\sigma, \tau)$-derivation. The converse is straightforward. \end{proof} For a non-negative integer $k$, define $S_{k}$ as the set containing all ordered pairs $(i,j)$ for which $i+j = k$. Note that the sets $S_{k} \text{'s}$ are pairwise disjoint. Therefore, $|\cup_{i=0}^{m} S_{i}| = \sum_{i=0}^{m} |S_{i}| = 1 + 2 + 3 + ... + m + (m+1) = \frac{(m+1)(m+2)}{2}$ for every non-negative integer $m$. \begin{lemma}\th\label{lemma 2.2} For a $(\sigma, \tau)$-derivation $D$ of $\mathcal{A}$, $$D(\alpha^{k}) = \left(\sum_{(i,j) \in S_{k-1}} \sigma(\alpha^{i}) \tau(\alpha^{j})\right)D(\alpha),$$ for every $\alpha \in \mathcal{A}$ and for every $k \in \mathbb{N}$. \end{lemma} \begin{proof} We use induction. For $k = 1$, the equality trivially holds. Let the result hold for $k=n$, that is, $D(\alpha^{n}) = \left(\sum_{(i,j) \in S_{n-1}} \sigma(\alpha^{i}) \tau(\alpha^{j})\right)D(\alpha)$. Then \begin{eqnarray*} D(\alpha^{n+1}) = D(\alpha^{n} \alpha) & = & D(\alpha^{n}) \tau(\alpha) + \sigma(\alpha^{n}) D(\alpha) \\ & = & \left(\sum_{i+j = n-1} \sigma(\alpha^{i}) \tau(\alpha^{j+1}) + \sigma(\alpha^{n}) \tau(\alpha^{0})\right) D(\alpha) \\ & = & \left(\sum_{i+j = n} \sigma(\alpha^{i}) \tau(\alpha^{j})\right)D(\alpha) = \left(\sum_{(i,j) \in S_{n}} \sigma(\alpha^{i}) \tau(\alpha^{j})\right)D(\alpha). \end{eqnarray*} Induction is complete and so is proof. \end{proof} The theorem is now straightforward. \begin{theorem}\th\label{theorem 2.3} Let $\mathcal{A}$ be of finite rank $n$ and suppose that $\mathcal{A}$ has an $R$-basis of the form $\{1, \alpha, \alpha^{2}, ..., \alpha^{n-1}\}$ for some $\alpha \in \mathcal{A}$. If an $R$-linear map $D:\mathcal{A} \rightarrow \mathcal{A}$ is a $(\sigma, \tau)$-derivation, then \begin{equation}D(\alpha^{k}) = \left(\sum_{(i,j) \in S_{k-1}} \sigma(\alpha^{i}) \tau(\alpha^{j})\right)D(\alpha)\end{equation} for all $k \in \{1, 2, ..., n-1\}$. \end{theorem} The converse of the \th\ref{theorem 2.3} is not true. Some examples are given below.\vspace{10pt} \noindent \textbf{Example 1.} Let $R[X]$ be the polynomial ring over a commutative unital ring $R$. Let $f(X) \in R[X]$ be monic with degree $n$ and $\mathcal{A} = \frac{R[X]}{\langle f(X) \rangle}$. Then $\mathcal{A}$ is a commutative $R$-algebra with unity $\bar{1} = 1 + f(X)$. Suppose $\alpha = X + f(X)$. Then $\{1, \alpha, \alpha^{2}, ..., \alpha^{n-1}\}$ is an $R$-basis of the $R$-module $\mathcal{A}$. We denote the zero element $0 + \langle f(X) \rangle$ of $\mathcal{A}$ by $\bar{0}$. In particular, take $R = \mathbb{Z}$ and $f(X) = X^{4} - 1$ so that $\{\bar{1}, \alpha, \alpha^{2}, \alpha^{3}\}$ is a $\mathbb{Z}$-basis of $\mathcal{A} = \frac{R[X]}{\langle f(X) \rangle}$. Let $\sigma$ and $\tau$ be $\mathbb{Z}$-algebra endomorphisms of $\mathcal{A}$ given by $\sigma(\alpha) = \alpha$ and $\tau(\alpha) = \alpha^{2}$. Then $\sigma(\bar{1}) = \bar{1}$ and $\tau(\bar{1}) = \bar{1}$. Define $D:\mathcal{A} \rightarrow \mathcal{A}$ as a $\mathbb{Z}$-linear map with $D(\bar{1}) = \bar{0}$ and $$D(\alpha^{r}) = \left(\sum_{(i,j) \in S_{r-1}} \sigma(\alpha^{i}) \tau(\alpha^{j})\right)D(\alpha), \hspace{0.1cm} \forall \hspace{0.1cm} r \in \{1, 2, 3\}.$$ $\alpha^{4} = \bar{1}$ so that $D(\alpha^{4}) = \bar{0}$. Also, \begin{eqnarray*} \left(\sum_{(i,j) \in S_{3}} \sigma(\alpha^{i}) \tau(\alpha^{j})\right)D(\alpha) & = & \left(\sigma(\alpha^{3}) + \sigma(\alpha^{2}) \tau(\alpha) + \sigma(\alpha) \tau(\alpha^{2}) + \tau(\alpha^{3})\right)D(\alpha) \\ & = & \left(\alpha^{3} + \alpha^{4} + \alpha^{5} + \alpha^{6}\right)D(\alpha) = \left(\bar{1} + \alpha + \alpha^{2} + \alpha^{3}\right)D(\alpha). \end{eqnarray*} Now $\bar{1} + \alpha + \alpha^{2} + \alpha^{3} \neq \bar{0}$ since $\{\bar{1}, \alpha, \alpha^{2}, \alpha^{3}\}$ is a basis of $\mathcal{A}$. Also, $\mathcal{A}$ is an integral domain, so if $D(\alpha) \neq 0$, then the above expression cannot be zero. Further, observe that $D(\alpha^{4}) \neq \left(\sum_{(i,j) \in S_{3}} \sigma(\alpha^{i}) \tau(\alpha^{j})\right)D(\alpha)$. Hence by \th\ref{lemma 2.2}, $D$ cannot be a $(\sigma, \tau)$-derivation of $\mathcal{A}$. \vspace{10pt} \noindent \textbf{Example 2.} Consider the set $$\mathcal{A} = \{aI + bA + cA^{2} \mid a, b, c \in \mathbb{Z}\},$$ where $A = \begin{pmatrix} 0 & 0 & 1 \\ 1 & 0 & 0 \\ 0 & 1 & 0 \end{pmatrix}$. Then $\mathcal{A}$ is a commutative subalgebra of the $\mathbb{Z}$-algebra $M_{3}(\mathbb{Z})$ with unity $I$ and $\{I, A, A^{2}\}$ is a $\mathbb{Z}$-basis of $\mathcal{A}$. Let $\sigma$ and $\tau$ be $\mathbb{Z}$-algebra endomorphisms of $\mathcal{A}$ given by $\sigma(aI + bA + cA^{2}) = aI + bA + cA^{2}$ and $\tau(aI + bA + cA^{2}) = aI + bA^{2} + cA$ for all $a, b, c \in \mathbb{Z}$. Then $\sigma(I) = \tau(I) = I$. Define $D:\mathcal{A} \rightarrow \mathcal{A}$ as a $\mathbb{Z}$-linear map with $D(I) = 0$ and such that $$D(A^{r}) = \left(\sum_{(i,j) \in S_{r-1}} \sigma(A^{i}) \tau(A^{j})\right)D(A), \hspace{0.8cm} \text{for} \hspace{0.2cm} r = 1, 2.$$ Then $D(A^{3}) = D(I) = 0$. Also, \begin{eqnarray*}\left(\sum_{(i,j) \in S_{2}} \sigma(A^{i}) \tau(A^{j})\right)D(A) & = & \left(\sigma(A^{2}) + \sigma(A) \tau(A) + \tau(A^{2})\right)D(A) = \left(I + A + A^{2}\right)D(A).\end{eqnarray*} Again, $I + A + A^{2} \neq 0$, so if $D(A) \in \mathcal{A}$ is such that $\left(I + A + A^{2}\right)D(A) \neq 0$, for example, $D(A) = A$ or $A^{2}$, then $D(A^{3}) \neq \left(\sum_{(i,j) \in S_{2}} \sigma(A^{i}) \tau(A^{j})\right)D(A)$. Hence, $D$ is not a $(\sigma, \tau)$-derivation. \vspace{10pt} \noindent \textbf{Example 3.} Let $n \in \mathbb{N}$ and $A \in M_{n \times n}(\mathbb{Z})$ be an idempotent matrix. Then the subset $$\mathcal{A} = \{aI + bA \mid a, b \in \mathbb{Z}\}$$ of $M_{n \times n}(\mathbb{Z})$ is a commutative $\mathbb{Z}$-algebra with unity $I$ and $\{I, A\}$ is a basis of $\mathcal{A}$. Let $\sigma$ and $\tau$ be $\mathbb{Z}$-algebra endomorphisms of $\mathcal{A}$ given by $\sigma(aI + bA) = (a+b)I$ and $\tau(aI + bA) = aI + bA$ for all $a, b \in \mathbb{Z}$. Define $D:\mathcal{A} \rightarrow \mathcal{A}$ as a $\mathbb{Z}$-linear map with $D(I) = 0$ and such that $$D(A^{r}) = \left(\sum_{(i,j) \in S_{r-1}} \sigma(A^{i}) \tau(A^{j})\right)D(A), \hspace{0.8cm} \text{for} \hspace{0.2cm} r = 1.$$ $$\left(\sum_{(i,j) \in S_{1}} \sigma(A^{i}) \tau(A^{j})\right)D(A) = \left(\sigma(A) + \tau(A)\right)D(A) = \left(I + A\right)D(A).$$ $D(A) \neq \left(I + A\right)D(A)$ provided $A D(A) \neq 0$. Hence, $D$ is not a $(\sigma, \tau)$-derivation of $\mathcal{A}$ provided $D(A)$ is chosen in $M_{n \times n}(\mathbb{Z})$ such that $A D(A) \neq 0$.\vspace{10pt} \noindent \textbf{Example 4.} Let $n \in \mathbb{N}$ and $A \in M_{n \times n}(\mathbb{Z})$ be a nilpotent matrix, that is, $A^{k} = 0$ for some least positive integer $k$. Then the subset $$\mathcal{A} = \{\sum_{i=0}^{k-1} a_{i} A^{i} \mid a_{i} \in \mathbb{Z}, \hspace{0.1cm} \forall \hspace{0.1cm} i \in \{0, 1, ..., k-1\}\}$$ of $M_{n \times n}(\mathbb{Z})$ is a commutative $\mathbb{Z}$-algebra with unity $I$ and $\{A^{0}=I, A, ..., A^{k-1}\}$ forms a basis of the $\mathbb{Z}$-module $\mathcal{A}$. Let $\sigma$ and $\tau$ be $\mathbb{Z}$-algebra endomorphisms of $\mathcal{A}$ given by $\sigma(\sum_{i=0}^{k-1} a_{i} A^{i}) = \sum_{i=0}^{k-1} a_{i} A^{i}$ and $\tau(\sum_{i=0}^{k-1} a_{i} A^{i}) = \sum_{i=0}^{k-1} a_{i} B^{i}$ for all $a_{i} \in \mathbb{Z}$ $(0 \leq i \leq k-1),$ where $B = m A$ for some $m \in \mathbb{N}$ and $m \neq 1$. Define $D:\mathcal{A} \rightarrow \mathcal{A}$ as a $\mathbb{Z}$-linear map with $D(I) = 0$ and such that $$D(A^{r}) = \left(\sum_{(i,j) \in S_{r-1}} \sigma(A^{i}) \tau(A^{j})\right)D(A), \hspace{0.8cm} \text{for} \hspace{0.2cm} r \in \{1, ..., k-1\}.$$ Then $D(A^{k}) = 0$. Also, \begin{eqnarray*}\left(\sum_{(i,j) \in S_{k-1}} \sigma(A^{i}) \tau(A^{j})\right)D(A) = \left(\sum_{i+j=k-1} A^{i} B^{j}\right)D(A) & = & \left(\sum_{j=0}^{k-1} m^{j} A^{k-1} \right)D(A) \\ & = & \left(\frac{1-m^{k}}{1-m}\right)A^{k-1}D(A).\end{eqnarray*} $0 \neq \left(\frac{1-m^{k}}{1-m}\right)A^{k-1}D(A)$ provided $A^{k-1} D(A) \neq 0$. Hence, $D$ cannot be a $(\sigma, \tau)$-derivation of $\mathcal{A}$ provided $D(A)$ is chosen in $M_{n \times n}(\mathbb{Z})$ such that $A^{k-1} D(A) \neq 0$.\vspace{10pt} If $R$ is a ring and $G$ is a group, then the group ring of $G$ over $R$ is defined as the set $$RG = \{\sum_{g \in G} a_{g} g \mid a_{g} \in R, \forall g \in G \hspace{0.2cm} \text{and} \hspace{0.2cm} |\text{supp}(\alpha)| < \infty \},$$ where for $\alpha = \sum_{g \in G} a_{g} g$, $\text{supp}(\alpha)$ is the support of $\alpha$ that consists of elements from $G$ that appear in the expression of $\alpha$. $RG$ is a ring with respect to the componentwise addition and multiplication defined respectively by: For $\alpha = \sum_{g \in G} a_{g} g$, $\beta = \sum_{g \in G} b_{g} g$ in $RG$, $$(\sum_{g \in G} a_{g} g ) + (\sum_{g \in G} b_{g} g) = \sum_{g \in G}(a_{g} + b_{g}) g \hspace{0.2cm} \text{and} \hspace{0.2cm} \alpha \beta = \sum_{g, h \in G} a_{g} b_{h} gh.$$ If the ring $R$ is commutative having unity $1$ and the group $G$ is abelian having identity $e$, then $RG$ becomes a commutative unital algebra over $R$ with unity $1 = 1e$. If $\sigma$ and $\tau$ are algebra endomorphisms of $RG$ which are $R$-linear extensions of group homomorphisms of $G$ and $D:RG \rightarrow RG$ is a $(\sigma, \tau)$-derivation, then $$D(e) = D(ee) = D(e) \tau(e) + D(e) \sigma(e) = D(e) + D(e)$$ so that $D(e) = 0$.\vspace{10pt} \noindent \textbf{Example 5.} Let $C_{n} = \langle g \mid g^{n} = 1\rangle$ be a cyclic group having order $n \geqslant 2$ and consider the group ring $\mathbb{Z}C_{n}$. Let $\sigma$ and $\tau$ be $\mathbb{Z}$-algebra endomorphisms of $\mathbb{Z}G$ which are $\mathbb{Z}$-linear extensions of group homomorphisms of $G$, say, $\sigma(g) = 1$ and $\tau(g) = g$. Let $D:\mathbb{Z}C_{n} \rightarrow \mathbb{Z}C_{n}$ be a $\mathbb{Z}$-linear map with $D(1) = 0$ and $$D(g^{r}) = \left(\sum_{(i,j) \in S_{r-1}} \sigma(g^{i}) \tau(g^{j})\right)D(g), \hspace{0.1cm} \forall \hspace{0.1cm} r \in \{1, ..., n-1\}.$$ So $D(g^{n}) = D(1) = 0$. Also, \begin{equation*} \begin{aligned} \left(\sum_{(i,j) \in S_{n-1}} \sigma(g^{i}) \tau(g^{j})\right)D(g) & = (\sigma(g^{n-1}) + \sigma(g^{n-2})\tau(g) + \sigma(g^{n-3})\tau(g^{2}) + ... \\ &\quad + \sigma(g^{2}) \tau(g^{n-3}) + \sigma(g) \tau(g^{n-2}) + \tau(g^{n-1}))D(g) \\ & = \left(e + g + ... + g^{n-1}\right)D(g) \end{aligned} \end{equation*} Note that $e + g + ... + g^{n-1} \neq 0$ since $e, g, ..., g^{n-1}$ are linearly independent being basis elements of $\mathbb{Z}C_{n}$. So if $D(g) \in \mathbb{Z}C_{n}$ is such that $\left(e + g + ... + g^{n-1}\right)D(g) \neq 0$, then $D(g^{n}) \neq \left(\sum_{(i,j) \in S_{n-1}} \sigma(g^{i}) \tau(g^{j})\right)D(g)$. Hence, $D$ cannot be a $(\sigma, \tau)$-derivation of $\mathbb{Z}C_{n}$. \begin{lemma}\th\label{lemma 2.4} Let $\mathcal{A}$ be of finite rank $n$ and $\{\alpha_{1}, \alpha_{2}, ..., \alpha_{n}\}$ be an $R$-basis of $\mathcal{A}$. Let $D:\mathcal{A} \rightarrow \mathcal{A}$ be a $(\sigma, \tau)$-derivation. Suppose that there exists some $\beta \in \mathcal{A}$ such that $D(\alpha_{i}) = \beta (\tau - \sigma)(\alpha_{i})$ for all $i \in \{1, 2, ..., n\}$. Then $D(\alpha) = \beta (\tau - \sigma)(\alpha)$ for all $\alpha \in \mathcal{A}$. \end{lemma} \begin{proof} For $\alpha = \sum_{i=1}^{n} a_{i} \alpha_{i} \in \mathcal{A}$, \begin{eqnarray*}D(\alpha) = \sum_{i=1}^{n} a_{i} D(\alpha_{i}) = \sum_{i=1}^{n} a_{i} \beta (\tau - \sigma)(\alpha_{i}) & = & \beta \left(\sum_{i=1}^{n} a_{i} \tau(\alpha_{i}) - \sum_{i=1}^{n} a_{i} \sigma(\alpha_{i})\right) \\ & = & \beta \left(\tau\left(\sum_{i=1}^{n} a_{i} \alpha_{i}\right) - \sigma\left(\sum_{i=1}^{n} a_{i} \alpha_{i}\right)\right) \\ & = & \beta (\tau - \sigma)(\alpha)\end{eqnarray*} Since $\alpha \in \mathcal{A}$ is arbitrary, therefore, $D$ is inner. \end{proof} The theorem now follows immediately. \begin{theorem}\th\label{theorem 2.5} Let $\mathcal{A}$ be of finite rank $n$ and $\{\alpha_{1}, \alpha_{2}, ..., \alpha_{n}\}$ be an $R$-basis of $\mathcal{A}$. Then a $(\sigma, \tau)$-derivation $D:\mathcal{A} \rightarrow \mathcal{A}$ is inner if and only if there exists some $\beta \in \mathcal{A}$ such that $D(\alpha_{i}) = \beta (\tau - \sigma)(\alpha_{i})$ for all $i \in \{1, 2, ..., n\}$. \end{theorem} \begin{lemma}\th\label{lemma 2.6} Let $D: \mathcal{A} \rightarrow \mathcal{A}$ be a $(\sigma, \tau)$-derivation and $\alpha \in \mathcal{A}$. If there exists some $\beta \in \mathcal{A}$ such that $D(\alpha) = \beta (\tau - \sigma)(\alpha)$, then $$D(\alpha^{n}) = \beta (\tau - \sigma)(\alpha^{n})$$ for all $n \in \mathbb{N}$. \end{lemma} \begin{proof} We again use induction on $n$. If $n=1$, the result holds trivially. Now suppose that $D(\alpha^{k}) = \beta (\tau - \sigma)(\alpha^{k})$. Then \begin{eqnarray*} D(\alpha^{k+1}) & = & D(\alpha^{k}) \tau(\alpha) + \sigma(\alpha^{k}) D(\alpha) = \beta (\tau - \sigma)(\alpha^{k}) \tau(\alpha) + \sigma(\alpha^{k}) \beta (\tau - \sigma)(\alpha) \\ & = & \beta \left(\tau(\alpha^{k+1}) - \sigma(\alpha^{k}) \tau(\alpha) + \sigma(\alpha^{k}) \tau(\alpha) - \sigma(\alpha^{k+1})\right) \\ & = & \beta (\tau - \sigma)(\alpha^{k+1}) \end{eqnarray*} Induction is complete and so is proof. \end{proof} \begin{lemma}\th\label{lemma 2.7} Let $\mathcal{A}$ be of finite rank $n$ and suppose that $\mathcal{A}$ has an $R$-basis of the form $\{1, \alpha, \alpha^{2}, ..., \alpha^{n-1}\}$ for some $\alpha \in \mathcal{A}$. Let $D: \mathcal{A} \rightarrow \mathcal{A}$ be a $(\sigma, \tau)$-derivation. If there exists some $\beta \in \mathcal{A}$ such that $D(\alpha) = \beta (\tau - \sigma)(\alpha)$, then $D$ is inner. \end{lemma} \begin{proof} Since $D(1) = 0$ and $\sigma(1) = \tau(1) = 1$, so $D(1) = \beta (\tau - \sigma)(1)$. Further since $D(\alpha) = \beta (\tau - \sigma)(\alpha)$, therefore, by \th\ref{lemma 2.6}, $$D(\alpha^{k}) = \beta (\tau - \sigma)(\alpha^{k})$$ for all $k \in \mathbb{N}$. So $D(\alpha^{k}) = \beta (\tau - \sigma)(\alpha^{k})$ for every $k \in \{0, 1, ..., n-1\}$. Now we use \th\ref{lemma 2.4}. \end{proof} The theorem below follows immediately. \begin{theorem}\th\label{theorem 2.8} Let $\mathcal{A}$ be of finite rank $n$ and suppose that $\mathcal{A}$ has an $R$-basis of the form $\{1, \alpha, \alpha^{2}, ..., \alpha^{n-1}\}$ for some $\alpha \in \mathcal{A}$. Then a $(\sigma, \tau)$-derivation $D:\mathcal{A} \rightarrow \mathcal{A}$ is inner if and only if there exists some $\beta \in \mathcal{A}$ such that $D(\alpha) = \beta (\tau - \sigma)(\alpha)$. \end{theorem} \section{\texorpdfstring{$(\sigma, \tau)$}{Lg}-derivations of number rings}\label{section 3} We know by \th\ref{theorem 1.1} that if $K$ is a number field, then $K = \mathbb{Q}(\theta)$ for some algebraic integer $\theta$. \th\ref{theorem 2.3} gives the lemma below: \begin{lemma}\th\label{lemma 3.1} Let $K = \mathbb{Q}(\theta)$ ($\theta$ an algebraic integer) be a number field of degree $n$ such that $\{1, \theta, ..., \theta^{n-1}\}$ is a power basis of $K$. Let $\sigma$ and $\tau$ be two different non-zero $\mathbb{Z}$-algebra endomorphisms of $O_{K}$. If a $\mathbb{Z}$-linear map $D:O_{K} \rightarrow O_{K}$ is a $(\sigma, \tau)$-derivation, then $$D(\theta^{k}) = \left(\sum_{(i,j) \in S_{k-1}} \sigma(\theta^{i}) \tau(\theta^{j})\right)D(\theta)$$ for all $k \in \{1, 2, ..., n-1\}$. \end{lemma} \subsection{\texorpdfstring{$(\sigma, \tau)$}{Lg}-derivations of quadratic number rings}\label{subsection 3.1} By \th\ref{theorem 1.3}, if $K$ is a quadratic field, then $K = \mathbb{Q}(\sqrt{d})$ for some square-free rational integer $d$. In this section, we take $K = \mathbb{Q}(\sqrt{d})$ and $\sigma$, $\tau$ as two different non-zero ring endomorphisms of $O_{K}$. Note that any ring endomorphism of the ring $O_{K}$ is a unital $\mathbb{Z}$-algebra endomorphism of the unital $\mathbb{Z}$-algebra $O_{K}$. As an application of \th\ref{lemma 2.1}, we present a different proof of a result of \cite{Chaudhuri} stated as follows. \begin{theorem}[{\cite[Theorem 4.2]{Chaudhuri}}] \th\label{theorem 3.2} Every $\mathbb{Z}$-linear map $D:O_{K} \rightarrow O_{K}$ with $D(1)=0$ is a $(\sigma, \tau)$-derivation of $O_{K}$. \end{theorem} \begin{proof} Only two possibilities arise: $d \not\equiv 1 \hspace{0.1cm} (\text{mod} \hspace{0.1cm} 4)$ and $d \equiv 1 \hspace{0.1cm} (\text{mod} \hspace{0.1cm} 4)$. Since $(\sqrt{d})^{2} = d$, so if $\theta$ is a non-zero ring endomorphism of $O_{K}$, then $(\theta(\sqrt{d}))^{2} = d$ or that $\theta(\sqrt{d}) = \pm \sqrt{d}$. When $d \not\equiv 1 \hspace{0.1cm} (\text{mod} \hspace{0.1cm} 4)$, then from \th\ref{theorem 1.4}, $O_{K} = \mathbb{Z}[\sqrt{d}]$ and from \th\ref{theorem 1.5}, $\{1, \sqrt{d}\}$ is a $\mathbb{Z}$-basis of $O_{K}$. In this case, exactly two possibilities arise. For every $a, b \in \mathbb{Z}$, \begin{itemize} \item[(a)] $\sigma(a + b \sqrt{d}) = a + b \sqrt{d}$; $\tau(a + b \sqrt{d}) = a - b \sqrt{d}$, \item[(b)] $\sigma(a + b \sqrt{d}) = a - b \sqrt{d}$; $\tau(a + b \sqrt{d}) = a + b \sqrt{d}.$ \end{itemize} When $d \equiv 1 \hspace{0.1cm} (\text{mod} \hspace{0.1cm} 4)$, then by \th\ref{theorem 1.4}, $O_{K} = \mathbb{Z}[\frac{1+\sqrt{d}}{2}]$ and by \th\ref{theorem 1.5}, $\{1,\frac{1+\sqrt{d}}{2}\}$ is a $\mathbb{Z}$-basis of $O_{K}$. Thus in this case, too, exactly two possibilities arise. For all $a, b \in \mathbb{Z}$, \begin{itemize} \item[(a)] $\sigma(a + b (\frac{1+\sqrt{d}}{2})) = a + b (\frac{1+\sqrt{d}}{2})$; $\tau(a + b (\frac{1+\sqrt{d}}{2})) = a + b (\frac{1-\sqrt{d}}{2})$, \item[(b)] $\sigma(a + b (\frac{1+\sqrt{d})}{2})) = a + b (\frac{1-\sqrt{d}}{2})$; $\tau(a + b (\frac{1+\sqrt{d}}{2})) = a + b (\frac{1+\sqrt{d}}{2})$. \end{itemize} Now the proof trivially follows from \th\ref{lemma 2.1}. \end{proof} Further, in the same paper \cite{Chaudhuri}, the author has proved the theorem stated below. \begin{theorem}[{\cite[Theorem 4.2]{Chaudhuri}}] \th\label{theorem 3.3} Let $d \not\equiv 1 \hspace{0.1cm} (\text{mod} \hspace{0.1cm} 4)$. Let $D:O_{K} \rightarrow O_{K}$ be a $(\sigma, \tau)$-derivation and $D(\sqrt{d}) = c_{0} + c_{1} \sqrt{d}$ for some $c_{0}, c_{1} \in \mathbb{Z}$. If $2 d$ divides $c_{0}$ and $c_{1}$ is even, then $D$ is inner. \end{theorem} We present a different proof of this theorem using \th\ref{theorem 2.8} and also prove that the conditions in the above theorem are necessary. \begin{theorem}\th\label{theorem 3.4} Let $d \not\equiv 1 \hspace{0.1cm} (\text{mod} \hspace{0.1cm} 4)$. Let $D:O_{K} \rightarrow O_{K}$ be a $(\sigma, \tau)$-derivation and $D(\sqrt{d}) = c_{0} + c_{1} \sqrt{d}$ for some $c_{0}, c_{1} \in \mathbb{Z}$. Then $D$ is inner if and only if $c_{0}$ is divisible by $2 d$ and $c_{1}$ is even. In particular, $O_{K}$ has non-trivial outer $(\sigma, \tau)$-derivations. \end{theorem} \begin{proof} Since $d \not\equiv 1 \hspace{0.1cm} (\text{mod} \hspace{0.1cm} 4)$, so $\{1, \sqrt{d}\}$ is a basis of the $\mathbb{Z}$-module $O_{K} = \mathbb{\mathbb{Z}}[\sqrt{d}]$. $O_{K} = \mathbb{\mathbb{Z}}[\sqrt{d}]$ has precisely two different non-zero ring endomorphisms: $\phi_{1}(a + b \sqrt{d}) \\ = a + b \sqrt{d}$ and $\phi_{2}(a + b \sqrt{d}) = a - b \sqrt{d}$ ($a, b \in \mathbb{Z}$). Therefore, $(\sigma, \tau) = (\phi_{1}, \phi_{2})$ or $(\phi_{2}, \phi_{1})$. We prove the result for $(\sigma, \tau) = (\phi_{2}, \phi_{1})$ since the other follows similarly. First, let $D$ be inner. Then there exists some $\beta = b_{0} + b_{1} \sqrt{d}$ for some $b_{0}, b_{1} \in \mathbb{Z}$ such that $D(\sqrt{d}) = \beta (\tau - \sigma)(\sqrt{d})$. Therefore, \begin{eqnarray*} c_{0} + c_{1} \sqrt{d} = D(\sqrt{d}) & = & \beta (\tau - \sigma)(\sqrt{d}) \\ & = & (b_{0} + b_{1} \sqrt{d}) (\tau(\sqrt{d}) - \sigma(\sqrt{d})) \\ & = & 2d b_{1} + 2b_{0} \sqrt{d} \end{eqnarray*} Since $\{1, \sqrt{d}\}$ is a basis of the $\mathbb{Z}$-module $O_{K} = \mathbb{\mathbb{Z}}[\sqrt{d}]$, therefore, $$2d b_{1} = c_{0} \hspace{0.2cm} \text{and} \hspace{0.2cm} 2 b_{0} = c_{1}.$$ Obviously, $b_{0} = \frac{c_{1}}{2}$ and $b_{1} = \frac{c_{0}}{2d}$. Since $b_{0}, b_{1} \in \mathbb{Z}$, therefore, $c_{1}$ is divisible by $2$ and $c_{0}$ is divisible by $2d$. Conversely, let $c_{0}$ be divisible by $2d$ and $c_{1}$ be even. Define $\beta = \left(\frac{c_{0}}{2d}\right) + \left(\frac{c_{1}}{2} \right) \sqrt{d}$. Then it can be verified that $\beta (\tau - \sigma)(\sqrt{d}) = D(\sqrt{d})$ as $D(\sqrt{d}) = c_{0} + c_{1} \sqrt{d}$. Now the result follows immediately by \th\ref{theorem 2.8}. \end{proof} \begin{theorem} Let $d \equiv 1 \hspace{0.1cm} (\text{mod} \hspace{0.1cm} 4)$. Let $D:O_{K} \rightarrow O_{K}$ be a $(\sigma, \tau)$-derivation and $D(\sqrt{d}) = c_{0} + c_{1} (\frac{1+\sqrt{d}}{2})$ for some $c_{0}, c_{1} \in \mathbb{Z}$. Then $D$ is inner if and only if $d$ divides $-c_{0} + c_{1} \left(\frac{d-1}{2}\right)$ and $2 c_{0} + c_{1}$. In particular, the following conditions hold: \begin{enumerate} \item[(i)] $O_{K}$ has non-trivial outer $(\sigma, \tau)$-derivations. \item[(ii)] $D$ is inner if $d$ divides both $c_{0}$ and $c_{1}$. \end{enumerate} \end{theorem} \begin{proof} Since $d \equiv 1 \hspace{0.1cm} (\text{mod} \hspace{0.1cm} 4)$, so $\{1, \frac{1+\sqrt{d}}{2}\}$ is a basis of the $\mathbb{Z}$-module $O_{K} = \mathbb{\mathbb{Z}}[\frac{1+\sqrt{d}}{2}]$. $O_{K}$ has precisely two different non-zero ring endomorphisms, namely, $\phi_{1}(a + b (\frac{1+\sqrt{d}}{2})) = a + b (\frac{1+\sqrt{d}}{2})$ and $\phi_{2}(a + b (\frac{1+\sqrt{d}}{2})) = a + b (\frac{1-\sqrt{d}}{2})$ ($a, b \in \mathbb{Z}$). Therefore, $(\sigma, \tau) = (\phi_{1}, \phi_{2})$ or $(\phi_{2}, \phi_{1})$. We prove the result for $(\sigma, \tau) = (\phi_{2}, \phi_{1})$ since the other follows similarly. First, let $D$ be inner. Then there exists some $\beta = b_{0} + b_{1} (\frac{1+\sqrt{d}}{2})$ for some $b_{0}, b_{1} \in \mathbb{Z}$ such that $D(\frac{1+\sqrt{d}}{2}) = \beta (\tau - \sigma)(\frac{1+\sqrt{d}}{2})$. Therefore, \begin{eqnarray*} c_{0} + c_{1} \left(\frac{1+\sqrt{d}}{2}\right) & = & D\left(\frac{1+\sqrt{d}}{2}\right) = \beta \left(\tau - \sigma \right)\left(\frac{1+\sqrt{d}}{2}\right) \\ & = & \left(b_{0} + b_{1} \left(\frac{1+\sqrt{d}}{2}\right)\right) \left(\tau \left(\frac{1+\sqrt{d}}{2} \right) - \sigma \left(\frac{1+\sqrt{d}}{2}\right) \right) \\ & = & \left(b_{0} + b_{1} \left(\frac{1+\sqrt{d}}{2} \right) \right) \left( \left(\frac{1+\sqrt{d}}{2}\right) - \left(\frac{1-\sqrt{d}}{2}\right) \right) \\ & = & \left(-b_{0} + b_{1} \left( \frac{d-1}{2} \right)\right) + (2b_{0} + b_{1}) \left(\frac{1+\sqrt{d}}{2}\right) \end{eqnarray*} Since $\{1, \frac{1+ \sqrt{d}}{2}\}$ is a basis of $O_{K} = \mathbb{\mathbb{Z}}[\frac{1+ \sqrt{d}}{2}]$, therefore, $$-b_{0} + b_{1} \left( \frac{d-1}{2} \right) = c_{0} \hspace{0.2cm} \text{and} \hspace{0.2cm} 2b_{0} + b_{1} = c_{1}.$$ Note that since $d \equiv 1 \hspace{0.1cm} (\text{mod} \hspace{0.1cm} 4)$, so $\frac{d-1}{2} \in \mathbb{Z}$. Solving, we get, $$b_{0} = \frac{1}{d} \left(- c_{0} + c_{1} \left(\frac{d-1}{2} \right)\right) \hspace{0.1cm} \text{and} \hspace{0.1cm} b_{1} = \frac{1}{d} \left(2 c_{0} + c_{1}\right).$$ Since $b_{0}, b_{1} \in \mathbb{Z}$, therefore, $d$ divides $- c_{0} + c_{1} \left(\frac{d-1}{2} \right)$ and $2 c_{0} + c_{1}$. Conversely, let $d$ divide $- c_{0} + c_{1} \left(\frac{d-1}{2} \right)$ and $2 c_{0}+c_{1}$. Define $$\beta = \left(\frac{1}{d} \left(- c_{0} + c_{1} \left(\frac{d-1}{2} \right)\right)\right) + \left(\frac{1}{d} \left(2 c_{0} + c_{1}\right)\right)\left(\frac{1+\sqrt{d}}{2}\right).$$ Then it can be verified that $\beta \left(\tau - \sigma \right)\left(\frac{1+\sqrt{d}}{2}\right) = D\left(\frac{1+\sqrt{d}}{2}\right)$ as $D\left(\frac{1+\sqrt{d}}{2}\right) = c_{0} + c_{1} \left(\frac{1+\sqrt{d}}{2}\right)$. The result now follows immediately by \th\ref{theorem 2.8}. \end{proof} \subsection{\texorpdfstring{$(\sigma, \tau)$}{Lg}-derivations of cyclotomic number rings}\label{subsection 3.2} In this section, $K = \mathbb{Q}(\zeta)$ denotes a $p^{\text{th}}$ cyclotomic field, where $p$ is an odd rational prime. By \th\ref{theorem 1.7}, the ring of algebraic integers of $K$ is $O_{K} = \mathbb{Z}[\zeta]$ and $\{1, \zeta, \zeta^{2}, ..., \zeta^{p-2}\}$ is an integral basis of $K$. Further, $\sigma$ and $\tau$ denote any two different non-zero ring endomorphisms of $O_{K} = \mathbb{Z}[\zeta]$. \begin{lemma}\th\label{lemma 3.6} Let for each $k \in \{0, 1, ..., p-2\}$, $S_{k}$'s be sets as defined in Section \ref{section 2}. Then $$\sum_{(i,j) \in \cup_{k=0}^{p-2} S_{k}} \sigma(\zeta^{i}) \tau(\zeta^{j}) = 0.$$ \end{lemma} \begin{proof} All the non-zero ring endomorphisms $\phi_{i}$ of $O_{K}$ are given by $\phi_{i}(\zeta) = \zeta^{i},$ where $i \in \{1, 2, ..., p-1\}$. Since $\sigma$ and $\tau$ are non-zero and different, therefore, $$\sigma(\zeta) = \zeta^{u} \hspace{0.2cm} \text{and} \hspace{0.2cm} \tau(\zeta) = \zeta^{u+v}$$ for some $u, v \in \{1, 2, ..., p-1\}$ such that $u+v \neq p$. Now, $$\sum_{(i,j) \in \cup_{k=0}^{p-2} S_{k}} \sigma(\zeta^{i}) \tau(\zeta^{j}) = \sum_{k=0}^{p-2} \sum_{(i,j) \in S_{k}} \sigma(\zeta^{i}) \tau(\zeta^{j}).$$ For any $k \in \{0, 1, ..., p-2\}$, $$\sum_{(i,j) \in S_{k}} \sigma(\zeta^{i}) \tau(\zeta^{j}) = \sum_{(i,j) \in S_{k}} \zeta^{ku} \zeta^{jv}.$$ There are three observations for each $k \in \{0, 1, ..., p-2\}$: \begin{enumerate} \item[(a)] The sum $\sum_{(i,j) \in S_{k}} \sigma(\zeta^{i}) \tau(\zeta^{j})$ contains $k+1$ terms and all these $k+1$ terms are different from each other since $u+v \neq p$. \item[(b)] The sum $\sum_{(i,j) \in \cup_{k=0}^{p-2} S_{k}} \sigma(\zeta^{i}) \tau(\zeta^{j})$ contains exactly $\frac{p(p-1)}{2}$ terms. \item[(c)] Since the prime $p$ is odd, therefore, for each $k \in \{1, 2, ..., p-1\}$, $\zeta^{k}$ is a primitive $p^{\text{th}}$ root of unity. \end{enumerate} These together imply that each term from the set $\{1, \zeta, \zeta^{2}, ..., \zeta^{p-1}\}$ is being repeated $\frac{p-1}{2}$ times in the sum $\sum_{(i,j) \in \cup_{k=0}^{p-2} S_{k}} \sigma(\zeta^{i}) \tau(\zeta^{j})$. Hence, $$\sum_{(i,j) \in \cup_{m=0}^{p-2} S_{m}} \sigma(\zeta^{i}) \tau(\zeta^{j}) = \frac{p-1}{2} \left(1 + \zeta + \zeta^{2} + ... + \zeta^{p-1}\right) = 0,$$ since $1 + \zeta + \zeta^{2} + ... + \zeta^{p-1} = 0$ by \th\ref{theorem 1.6}. This proves the required result. \end{proof} \begin{lemma}\th\label{lemma 3.7} Let $D:O_{K} \rightarrow O_{K}$ be a $\mathbb{Z}$-linear map with $D(1) = 0$ and \begin{equation}\label{eq 3.1}D(\zeta^{k}) = \left(\sum_{(i,j) \in S_{k-1}} \sigma(\zeta^{i}) \tau(\zeta^{j})\right)D(\zeta)\end{equation} for all $k \in \{1, 2, ..., p-2\}$. Then $D$ is a $(\sigma, \tau)$-derivation. \end{lemma} \begin{proof} According to \th\ref{lemma 2.1}, the result can be concluded by establishing that \begin{equation}\label{eq 3.2}D(\zeta^{i+j}) = D(\zeta^{i}) \tau(\zeta^{j}) + \sigma(\zeta^{i}) D(\zeta^{j})\end{equation} for all $i, j \in \{0, 1, ..., p-2\}$. The relations (\ref{eq 3.2}) hold trivially when atleast one of $i$ or $j$ is $0$, using the fact that $\sigma(1) = \tau(1) = 1$ and $D(1) = 0$. So now let $i, j \in \{1, 2, ..., p-2\}$. Using (\ref{eq 3.1}), we get: \begin{eqnarray*} D(\zeta^{i}) \tau(\zeta^{j}) = \left(\sum_{(s,t) \in S_{i-1}} \sigma(\zeta^{s}) \tau(\zeta^{t})\right)D(\zeta) \tau(\zeta^{j}) & = & \left(\sum_{(s,t) \in S_{i-1}} \sigma(\zeta^{s}) \tau(\zeta^{t+j})\right)D(\zeta) \\ & = & \left(\sum_{s=0}^{i-1} \sigma(\zeta^{s}) \tau(\zeta^{i-1-s+j})\right)D(\zeta)\end{eqnarray*} and \begin{eqnarray*} \sigma(\zeta^{i}) D(\zeta^{j}) = \sigma(\zeta^{i})\left(\sum_{(s,t) \in S_{j-1}} \sigma(\zeta^{s}) \tau(\zeta^{t})\right)D(\zeta) & = & \left(\sum_{(s,t) \in S_{j-1}} \sigma(\zeta^{i+s}) \tau(\zeta^{t})\right)D(\zeta) \\ & = & \left(\sum_{s=0}^{j-1} \sigma(\zeta^{i+s}) \tau(\zeta^{j-1-s})\right)D(\zeta) \\ & = & \left(\sum_{s=i}^{i+j-1} \sigma(\zeta^{s}) \tau(\zeta^{j-1+i-s})\right)D(\zeta). \end{eqnarray*} Therefore, \begin{equation}\label{eq 3.3} D(\zeta^{i}) \tau(\zeta^{j}) + \sigma(\zeta^{i}) D(\zeta^{j}) = \left(\sum_{(s,t) \in S_{i+j-1}} \sigma(\zeta^{s}) \tau(\zeta^{t})\right)D(\zeta). \end{equation} Since $i, j \in \{1, 2,..., p-2\}$, so $i+j \in \{2, 3 ..., 2(p-2)\}$. We partition the proof into the following four cases.\vspace{10pt} Case 1: $i+j \leq p-2$. Since $i+j \leqslant p-2$, so by (\ref{eq 3.1}), $D(\zeta^{i+j}) = \left(\sum_{(s,t) \in S_{i+j-1}} \sigma(\zeta^{s}) \tau(\zeta^{t})\right)D(\zeta)$. Then by (\ref{eq 3.3}), $D(\zeta^{i}) \tau(\zeta^{j}) + \sigma(\zeta^{i}) D(\zeta^{j}) = D(\zeta^{i+j})$. Therefore, in this case, relations (\ref{eq 3.2}) hold.\vspace{10pt} Case 2: $i+j = p-1$. By \th\ref{lemma 3.6}, $\sum_{(i,j) \in \cup_{k=0}^{p-2} S_{k}} \sigma(\zeta^{i}) \tau(\zeta^{j}) = 0$. $\Rightarrow \sum_{k=0}^{p-2} \sum_{(i,j) \in S_{k}} \sigma(\zeta^{i}) \tau(\zeta^{j}) = 0$. $\Rightarrow \sum_{(i,j) \in S_{p-2}} \sigma(\zeta^{i}) \tau(\zeta^{j}) = - \left(\sum_{k=0}^{p-3} \sum_{(i,j) \in S_{k}} \sigma(\zeta^{i}) \tau(\zeta^{j})\right)$. $\Rightarrow \left(\sum_{(i,j) \in S_{p-2}} \sigma(\zeta^{i}) \tau(\zeta^{j})\right)D(\zeta) = - \left(\sum_{k=0}^{p-3} \left(\sum_{(i,j) \in S_{k}} \sigma(\zeta^{i}) \tau(\zeta^{j})\right)D(\zeta)\right)$. Therefore, using (\ref{eq 3.1}) and (\ref{eq 3.3}), $$D(\zeta^{i}) \tau(\zeta^{j}) + \sigma(\zeta^{i}) D(\zeta^{j}) = -\left(\sum_{k=0}^{p-3} D(\zeta^{k+1})\right) = -D(1 + \zeta + \zeta^{2} + ... + \zeta^{p-2}) = D(\zeta^{p-1}).$$ So (\ref{eq 3.2}) holds in this case too.\vspace{10pt} Case 3: $i+j = p$. Since the ring endomorphisms $\sigma$ and $\tau$ are non-zero and different, therefore, $\sigma(\zeta) = \zeta^{u} \hspace{0.2cm} \text{and} \hspace{0.2cm} \tau(\zeta) = \zeta^{u+v}$ for some $u, v \in \{1, 2, ..., p-1\}$ such that $u+v \neq p$. Now by (\ref{eq 3.3}), \begin{equation*} \begin{aligned} D(\zeta^{i}) \tau(\zeta^{j}) + \sigma(\zeta^{i}) D(\zeta^{j}) & = \left(\sum_{(s,t) \in S_{i+j-1}} \sigma(\zeta^{s}) \tau(\zeta^{t})\right)D(\zeta) = \left(\sum_{(s,t) \in S_{p-1}} \sigma(\zeta^{s}) \tau(\zeta^{t})\right)D(\zeta) \\ & = \left(\sum_{s=0}^{p-1} \sigma(\zeta^{s}) \tau(\zeta^{p-1-s})\right)D(\zeta) = \zeta^{(p-1)u} \left(\sum_{s=0}^{p-1} \zeta^{(p-1-s)v})\right)D(\zeta) \\ & = \zeta^{(p-1)u} \left(1 + \zeta^{v} + \zeta^{2v} + ... + \zeta^{(p-1)v}\right)D(\zeta) = 0, \end{aligned} \end{equation*} as $\zeta^{v}$ satisfies the cyclotomic polynomial. Therefore, $D(\zeta^{i}) \tau(\zeta^{j}) + \sigma(\zeta^{i}) D(\zeta^{j}) = D(\zeta^{p})$ as $\zeta^{p} = 1$ and $D(1) = 0$. So (\ref{eq 3.2}) holds in this case as well.\vspace{10pt} Case 4: $p < i+j \leq 2(p-2)$. Then $i+j = m + p$ for some $m \in \{1, 2, ..., p-2\}$. By (\ref{eq 3.1}), \begin{eqnarray*}D(\zeta^{i+j}) = D(\zeta^{m}) = \left(\sum_{(s,t) \in S_{m-1}} \sigma(\zeta^{s}) \tau(\zeta^{t})\right)D(\zeta) & = & \left(\sum_{t=0}^{m-1} \sigma(\zeta^{m-1-t}) \tau(\zeta^{t})\right)D(\zeta) \\ & = & \zeta^{(m-1)u} \left(\sum_{t=0}^{m-1} \zeta^{tv}\right)D(\zeta).\end{eqnarray*} Further, by (\ref{eq 3.3}), $D(\zeta^{i}) \tau(\zeta^{j}) + \sigma(\zeta^{i}) D(\zeta^{j}) = \left(\sum_{(s,t) \in S_{i+j-1}} \sigma(\zeta^{s}) \tau(\zeta^{t})\right)D(\zeta)$ \begin{equation*} \begin{aligned} & = \left(\sum_{(s,t) \in S_{m+p-1}} \sigma(\zeta^{s}) \tau(\zeta^{t})\right)D(\zeta) = \left(\sum_{t=0}^{m+p-1} \sigma(\zeta^{m+p-1-t}) \tau(\zeta^{t})\right)D(\zeta) \\ & = \zeta^{(m+p-1)u} \left(\sum_{t=0}^{m+p-1} \zeta^{tv}\right)D(\zeta) = \zeta^{(m+p-1)u} \left(\sum_{t=0}^{m-1} \zeta^{tv} + \sum_{t=m}^{m+p-1} \zeta^{tv}\right)D(\zeta) \\ & = \zeta^{(m-1)u} \left(\sum_{t=0}^{m-1} \zeta^{tv} + \zeta^{mv} \left(\sum_{t=0}^{p-1} \zeta^{tv}\right)\right)D(\zeta) = \zeta^{(m-1)u} \left(\sum_{t=0}^{m-1} \zeta^{tv}\right)D(\zeta) \end{aligned} \end{equation*} since $\sum_{t=0}^{p-1} \zeta^{tv} = 0$ as $v \in \{1, 2, ..., p-1\}.$ Therefore, $D(\zeta^{i+j}) = D(\zeta^{i}) \tau(\zeta^{j}) + \sigma(\zeta^{i}) D(\zeta^{j})$. This gives again the relations (\ref{eq 3.2}). We conclude that the relations (\ref{eq 3.2}) hold for all $i, j \in \{0, 1, ..., p-2\}$. The result now can be concluded from \th\ref{lemma 2.1}. \end{proof} As a consequence of \th\ref{lemma 3.1} and \th\ref{lemma 3.7}, we have the main theorem. \begin{theorem}\th\label{theorem 3.8} Let $D:O_{K} \rightarrow O_{K}$ be a $\mathbb{Z}$-linear map with $D(1) = 0$. Then $D$ is a $(\sigma, \tau)$-derivation if and only if the following relations hold for all $k \in \{1, 2, ..., p-2\}$: $$D(\zeta^{k}) = \left(\sum_{(i,j) \in S_{k-1}} \sigma(\zeta^{i}) \tau(\zeta^{j})\right)D(\zeta).$$ \end{theorem} \begin{corollary}\th\label{corollary 3.9} The $\mathbb{Z}$-module $\mathcal{D}_{(\sigma, \tau)}(O_{K})$ is finitely generated of rank $p-1$. \end{corollary} \begin{proof} For every $i \in \{0, 1, ..., p-2\}$, define $D_{i}:O_{K} \rightarrow O_{K}$ as a $\mathbb{Z}$-linear map with $D(1) = 0$ and $$D_{i}(\zeta) = \zeta^{i}.$$ More precisely, for each $i \in \{0, 1, ..., p-2\}$, let $D_{i}:O_{K} \rightarrow O_{K}$ be a map defined by $$D_{i}\left(\sum_{j=0}^{p-2} a_{j} \zeta^{j}\right) = \sum_{j=1}^{p-2} a_{j} D_{i}(\zeta^{j}),$$ where for each $j \in \{1, 2, ..., p-2\}$, $D_{i}(\zeta^{j})$ is defined as $$D_{i}(\zeta^{j}) = \left(\sum_{(s,t) \in S_{j-1}} \sigma(\zeta^{s}) \tau(\zeta^{t})\right)D_{i}(\zeta).$$ Then by \th\ref{theorem 3.8}, for each $i \in \{0, 1, ..., p-2\}$, $D_{i}:O_{K} \rightarrow O_{K}$ is a $(\sigma, \tau)$-derivation. Further, it can be easily verified that $\{D_{0}, D_{1}, ..., D_{p-2}\}$ forms a linearly independent subset of the $\mathbb{Z}$-module $\mathcal{D}_{(\sigma, \tau)}(O_{K})$ that generates it. Hence the result is proved. \end{proof} We observe from the preceding corollary that the rank of the $\mathbb{Z}$-module $\mathcal{D}_{(\sigma, \tau)}(O_{K})$ is equal to $p-1$, the degree of $K = \mathbb{Q}(\zeta)$ and which, by \th\ref{theorem 1.2}, is also equal to the rank of the free abelian (additive) group $O_{K} = \mathbb{Z}[\zeta]$. Some immediate corollaries can be stated below. \begin{corollary}\th\label{corollary 3.10} Let $K = Q(\zeta)$ be a $3^{\text{th}}$ cyclotomic field. Then any $\mathbb{Z}$-linear map $D:O_{K} \rightarrow O_{K}$ with $D(1)=0$ is a $(\sigma, \tau)$-derivation. \end{corollary} \begin{corollary}\th\label{corollary 3.11} There always exists a non-zero $(\sigma, \tau)$-derivation of $O_{K} = \mathbb{Z}[\zeta]$. \end{corollary} We propose the following conjecture. \begin{conjecture}\th\label{conjecture 3.12} Suppose $\beta = \sum_{i=0}^{p-2} b_{i} \zeta^{i} \in O_{K}$ and $\beta (\tau - \sigma) (\zeta) = \sum_{i=0}^{p-2} \left( \sum_{j=0}^{p-2} a_{ij} b_{j} \right) \zeta^{i}$. Then $A = [a_{ij}]$ is a $(p-1) \times (p-1)$ matrix with determinant $p$. \end{conjecture} We have verified the conjecture for all odd primes less than $100$ using SAGE and MATLAB. Calculations for primes $3, 5, 7, 11, 13$ were done on SAGE (explicit matrix $A$ and $\text{det}(A)$ were found for all possible values of the pair $(\sigma(\zeta), \tau(\zeta))$ (see the appendix)). We then developed a code on MATLAB and calculations for the remaining primes were done by running that MATLAB code. For larger primes, MATLAB took a long time to provide the output after running the code. Nevertheless, we believe the conjecture to be true for all odd primes. As a consequence, we propose another conjecture which too will hold once the above conjecture is proved. Let $\beta = \sum_{i=0}^{p-2} b_{i} \zeta^{i} \in K$, $D$ be a $(\sigma, \tau)$-derivation of $O_{K}$ and $D(\zeta) = \sum_{i=0}^{p-2} c_{i} \zeta^{i} \in O_{K}$. Put $X^{T} = (b_{0} ~ b_{1} ~ ... ~ b_{p-2})$ and $C^{T} = (c_{0} ~ c_{1} ~ ... ~ c_{p-2})$. We denote by $\mathbb{Z}^{p-1}$ the set of all $(p-1) \times 1$ column matrices with entries from $\mathbb{Z}$. Also, $Adj(A)$ denotes the adjoint of the matrix $A$. Note that since $\sigma \neq \tau$, therefore, $D(\zeta) = \beta(\tau - \sigma)(\zeta)$ always has a solution $\beta$ in $K$. Furthermore, $D(\zeta) = \beta(\tau - \sigma)(\zeta)$ has a solution $\beta$ in $O_{K}$ if and only if $AX = C$ has a solution in $\mathbb{Z}^{p-1}$. \begin{conjecture}\th\label{conjecture 3.13} With the above notations, $D$ is inner if and only if $\frac{1}{p}(Adj(A)C) \in \mathbb{Z}^{p-1}$. In particular, the following conditions hold: \begin{enumerate} \item[(i)] $O_{K}$ has non-trivial outer $(\sigma, \tau)$-derivations. \item[(ii)] If $p$ divides $c_{i}$ for each $i \in \{0, 1, ..., p-2\}$, then $D$ is inner. \end{enumerate} \end{conjecture} Given below are some interesting examples. \begin{example} Let $p \geq 5$. Then not every $\mathbb{Z}$-linear map $D:\mathbb{Z}[\zeta] \longrightarrow \mathbb{Z}[\zeta]$ with $D(1)=0$ is a $(\sigma, \tau)$-derivation. In fact, there exists a non-zero $\mathbb{Z}$-linear map $D:\mathbb{Z}[\zeta] \longrightarrow \mathbb{Z}[\zeta]$ with $D(1)=0$ which is not a $(\sigma, \tau)$-derivation for every pair $(\sigma, \tau)$ ($\sigma \neq 0, \tau \neq 0, \sigma \neq \tau$). As an example, define $D:\mathbb{Z}[\zeta] \longrightarrow \mathbb{Z}[\zeta]$ by $D(\sum_{i=0}^{p-2} a_{i} \zeta^{i}) = a_{1} D(\zeta),$ where $D(\zeta) \in O_{K} \setminus \{0\}$, say, $D(\zeta) = \zeta$. Then $D$ is a non-zero well-defined $\mathbb{Z}$-linear map with $D(1) = 0.$ But in view of \th\ref{theorem 3.8}, $D$ is not a $(\sigma, \tau)$-derivation, since $$D(\zeta^{2}) \neq \left(\sum_{(i,j) \in S_{1}} \sigma(\zeta^{i}) \tau(\zeta^{j}) \right) D(\zeta).$$ \end{example} \begin{example} Let $p = 5$. Let $\sigma$ and $\tau$ be given by $\sigma(\zeta) = \zeta$ and $\tau(\zeta) = \zeta^{2}$. (i) Define $D:O_{K} \rightarrow O_{K}$ by $$D(\sum_{i=0}^{3} a_{i} \zeta^{i}) = \sum_{i=1}^{3} a_{i} D(\zeta^{i}), \hspace{0.2cm} \forall \hspace{0.2cm} a_{i} \in \mathbb{Z} \hspace{0.1cm} (i \in \{0, 1, 2, 3\}),$$ where for each $i \in \{1, 2, 3\}$, $D(\zeta^{i})$ is defined as $$D(\zeta^{i}) = \left(\sum_{(s,t) \in S_{i-1}} \sigma(\zeta^{s}) \tau(\zeta^{t})\right)D(\zeta).$$ Note that $D$ is a $\mathbb{Z}$-linear map with $D(1) = 0$. Take $D(\zeta) \in O_{K} \setminus \{0\}$. Then by \th\ref{theorem 3.8}, $D$ is a non-zero $(\sigma, \tau)$-derivation. But as shown below, $D$ is not inner if in particular, we take $D(\zeta) = \zeta$. Thus in a cyclotomic field $K$, it is not necessary that every $(\sigma, \tau)$-derivation of its ring of algebraic integers $O_{K}$ is inner. (ii) If possible, suppose that the $(\sigma, \tau)$-derivation $D:O_{K} \rightarrow O_{K}$ is inner. Then there exists some $\beta \in O_{K}$ such that $D(\alpha) = \beta(\sigma - \tau)(\alpha), \hspace{0.1cm} \forall \hspace{0.1cm} \alpha \in O_{K}$. Since $\beta \in O_{K}$, so $\beta = \sum_{i=0}^{3} b_{i} \zeta^{i}$ for some unique $b_{i} \in \mathbb{Z}$ for all $i \in \{0, 1, 2, 3\}$. \begin{eqnarray*} \beta(\sigma - \tau)(\zeta) & = & \beta (\sigma(\zeta) - \tau(\zeta)) \\ & = & (b_{0} + b_{1} \zeta + b_{2} \zeta^{2} + b_{3} \zeta^{3})(\zeta - \zeta^{2}) \\ & = & (b_{2} - 2b_{3}) + (b_{0} + b_{2} - b_{3}) \zeta + (-b_{0} + b_{1} + b_{2} - b_{3}) \zeta^{2} + (-b_{1} + 2b_{2} - b_{3})\zeta^{3} \end{eqnarray*} Now $\beta(\sigma - \tau)(\zeta) = D(\zeta)$ with $D(\zeta) = \zeta$ gives $$(b_{2} - 2b_{3}) + (b_{0} + b_{2} - b_{3}) \zeta + (-b_{0} + b_{1} + b_{2} - b_{3}) \zeta^{2} + (-b_{1} + 2b_{2} - b_{3})\zeta^{3} = 0 + 1 \zeta + 0 \zeta^{2} + 0 \zeta^{3}.$$ $\Rightarrow b_{2} - 2b_{3} = 0$; $b_{0} + b_{2} - b_{3} = 1$; $-b_{0} + b_{1} + b_{2} - b_{3} = 0$; $-b_{1} + 2b_{2} - b_{3} = 0$. \noindent This system of linear equations can be written as $AX = C$, where $$A = \begin{pmatrix} 0 & 0 & 1 & -2 \\ 1 & 0 & 1 & -1 \\ -1 & 1 & 1 & -1 \\ 0 & -1 & 2 & -1 \end{pmatrix}, ~ X^{T} = \begin{pmatrix} b_{0} & b_{1} & b_{2} & b_{3} \end{pmatrix} ~ \text{and} ~ C^{T} = \begin{pmatrix} 0 & 1 & 0 & 0 \end{pmatrix}.$$ This can have a (unique) integral solution if and only if $A$ is a unimodular matrix, that is, the determinant of $A$ is either $1$ or $-1$. But the determinant of $A$ is $5$. Therefore, there is no integer solution. Hence a contradiction. Therefore, the given $(\sigma, \tau)$-derivation $D:O_{K} \rightarrow O_{K}$ is not inner for $D(\zeta) = \zeta$. \end{example} \subsection{\texorpdfstring{$(\sigma, \tau)$}{Lg}-derivations of bi-quadratic number rings}\label{subsection 3.3} Here we present an elegant application of \th\ref{lemma 2.1} in classifying all $(\sigma, \tau)$-derivations and inner $(\sigma, \tau)$-derivations of the bi-quadratic number ring $\mathbb{Z}[\sqrt{m}, \sqrt{n}]$. Throughout this subsection, $K$ denotes the quartic number field $K = \mathbb{Q}(\sqrt{m}, \sqrt{n})$, where $m$ and $n$ are distinct square-free rational integers, $\mathcal{A}$ the number ring $\mathcal{A} = \mathbb{Z}[\sqrt{m}, \sqrt{n}]$ with $\mathbb{Z}$-basis $\{1, \sqrt{m}, \sqrt{n}, \sqrt{mn}\}$, and $\sigma$ and $\tau$ any two non-zero ring endomorphisms of $\mathcal{A}$. \begin{lemma}\th\label{lemma 3.16} If $D:\mathcal{A} \rightarrow \mathcal{A}$ is a non-zero $(\sigma, \tau)$-derivation of $\mathcal{A}$, then precisely one case amongst (i), (ii) or (iii) holds. \begin{enumerate} \item[(i)] $D(\sqrt{m}) = 0$, $D(\sqrt{n}) \neq 0$, $D(\sqrt{mn}) \neq 0$ and $D(\sqrt{mn}) = \pm \sqrt{m} D(\sqrt{n})$. Further, in this case, the possible values of the pair $(\sigma, \tau)$ are $(\phi_{1}, \phi_{2}), (\phi_{2}, \phi_{1}), (\phi_{3}, \phi_{4}), (\phi_{4}, \phi_{3})$. In fact, $D(\sqrt{mn}) = \sqrt{m} D(\sqrt{n})$ if $(\sigma, \tau) = (\phi_{1}, \phi_{2})$ or $(\phi_{2}, \phi_{1})$ and $D(\sqrt{mn}) \\ = - \sqrt{m} D(\sqrt{n})$ if $(\sigma, \tau) = (\phi_{3}, \phi_{4})$ or $(\phi_{4}, \phi_{3})$.\vspace{0.2cm} \item[(ii)] $D(\sqrt{m}) \neq 0$, $D(\sqrt{n}) = 0$, $D(\sqrt{mn}) \neq 0$ and $D(\sqrt{mn}) = \pm \sqrt{n} D(\sqrt{m})$. Further, in this case, the possible values of the pair $(\sigma, \tau)$ are $(\phi_{1}, \phi_{3}), (\phi_{2}, \phi_{4}), (\phi_{3}, \phi_{1}), (\phi_{4}, \phi_{2})$. In fact, $D(\sqrt{mn}) = \sqrt{n} D(\sqrt{m})$ if $(\sigma, \tau) = (\phi_{1}, \phi_{3})$ or $(\phi_{3}, \phi_{1})$ and $D(\sqrt{mn}) \\ = - \sqrt{n} D(\sqrt{m})$ if $(\sigma, \tau) = (\phi_{2}, \phi_{4})$ or $(\phi_{4}, \phi_{2})$.\vspace{0.2cm} \item[(iii)] $D(\sqrt{m}) \neq 0$, $D(\sqrt{n}) \neq 0$, $D(\sqrt{mn}) = 0$ and $\sqrt{m} D(\sqrt{n}) = \pm \sqrt{n} D(\sqrt{m})$. Further, in this case, the possible values of the pair $(\sigma, \tau)$ are $(\phi_{1}, \phi_{4}), (\phi_{2}, \phi_{3}), (\phi_{4}, \phi_{1}), (\phi_{3}, \phi_{2})$. In fact, $\sqrt{m} D(\sqrt{n}) = \sqrt{n} D(\sqrt{m})$ if $(\sigma, \tau) = (\phi_{1}, \phi_{4})$ or $(\phi_{4}, \phi_{1})$ and $\sqrt{m} D(\sqrt{n}) \\ = - \sqrt{n} D(\sqrt{m})$ if $(\sigma, \tau) = (\phi_{2}, \phi_{3})$ or $(\phi_{3}, \phi_{2})$. \end{enumerate} Here $\phi_{1}, \phi_{2}, \phi_{3}, \phi_{4}$ are the non-zero ring endomorphisms of $\mathcal{A} = \mathbb{Z}[\sqrt{m}, \sqrt{n}]$ given by \begin{enumerate} \item[(a)] $\phi_{1}(\sqrt{m}) = \sqrt{m}$, $\phi_{1}(\sqrt{n}) = \sqrt{n}$. \item[(b)] $\phi_{2}(\sqrt{m}) = \sqrt{m}$, $\phi_{2}(\sqrt{n}) = - \sqrt{n}$. \item[(c)] $\phi_{3}(\sqrt{m}) = - \sqrt{m}$, $\phi_{3}(\sqrt{n}) = \sqrt{n}$. \item[(d)] $\phi_{4}(\sqrt{m}) = - \sqrt{m}$, $\phi_{4}(\sqrt{n}) = - \sqrt{n}$. \end{enumerate} \end{lemma} \begin{proof} If $\phi$ is a non-zero ring endomorphisms of $\mathcal{A}$, then $(\phi(\sqrt{m}))^{2} = m \hspace{0.2cm} \text{and} \hspace{0.2cm} (\phi(\sqrt{n}))^{2} = n$. So $\phi(\sqrt{m}) = \pm \sqrt{m}$ and $\phi(\sqrt{n}) = \pm \sqrt{n}.$ Therefore, $\mathcal{A}$ has precisely the four non-zero ring endomorphisms $\phi_{1}, \phi_{2}, \phi_{3}, \phi_{4}$ given in the table below. \begin{table}[H] \caption{} \centering \begin{tabular}{|c||c|c|c|c|} \hline & $1$ & $\sqrt{m}$ & $\sqrt{n}$ & $\sqrt{mn}$ \\ \hline \hline $\phi_{1}$ & $1$ & $\sqrt{m}$ & $\sqrt{n}$ & $\sqrt{mn}$ \\ \hline $\phi_{2}$ & $1$ & $\sqrt{m}$ & $- \sqrt{n}$ & $- \sqrt{mn}$ \\ \hline $\phi_{3}$ & $1$ & $- \sqrt{m}$ & $\sqrt{n}$ & $- \sqrt{mn}$ \\ \hline $\phi_{4}$ & $1$ & $- \sqrt{m}$ & $- \sqrt{n}$ & $\sqrt{mn}$ \\ \hline \end{tabular} \label{table 1} \end{table} Put $\alpha_{1} = 1$, $\alpha_{2} = \sqrt{m}$, $\alpha_{3} = \sqrt{n}$ and $\alpha_{4} = \sqrt{mn}$. Then by \th\ref{lemma 2.1}, $D(\alpha_{i} \alpha_{j}) = D(\alpha_{i}) \tau(\alpha_{j}) + \sigma(\alpha_{i}) D(\alpha_{j}), \hspace{0.1cm} \forall \hspace{0.1cm} i, j \in \{1, 2, 3, 4\}$. Also, $D$ is $\mathbb{Z}$-linear. So we get the following table \ref{table 2}. \begin{table}[H] \caption{} \centering \begin{tabular}{|c|c|c|c|c|} \hline $(i,j)$ & $\alpha_{i}$ & $\alpha_{j}$ & $D(\alpha_{i} \alpha_{j})$ & $D(\alpha_{i}) \tau(\alpha_{j}) + \sigma(\alpha_{i}) D(\alpha_{j})$ \\ \hline \hline $(1, 1)$ & $1$ & $1$ & $0$ & $0$ \\ \hline $(1, 2)$ & $1$ & $\sqrt{m}$ & $D(\sqrt{m})$ & $D(\sqrt{m})$ \\ \hline $(1, 3)$ & $1$ & $\sqrt{n}$ & $D(\sqrt{n})$ & $D(\sqrt{n})$ \\ \hline $(1, 4)$ & $1$ & $\sqrt{mn}$ & $D(\sqrt{mn})$ & $D(\sqrt{mn})$ \\ \hline $(2, 1)$ & $\sqrt{m}$ & $1$ & $D(\sqrt{m})$ & $D(\sqrt{m})$ \\ \hline $(2, 2)$ & $\sqrt{m}$ & $\sqrt{m}$ & $0$ & $(\sigma(\sqrt{m}) + \tau(\sqrt{m}))D(\sqrt{m})$ \\ \hline $(2, 3)$ & $\sqrt{m}$ & $\sqrt{n}$ & $D(\sqrt{mn})$ & $D(\sqrt{m}) \tau(\sqrt{n}) + \sigma(\sqrt{m}) D(\sqrt{n})$ \\ \hline $(2, 4)$ & $\sqrt{m}$ & $\sqrt{mn}$ & $mD(\sqrt{n})$ & $D(\sqrt{m}) \tau(\sqrt{mn}) + \sigma(\sqrt{m}) D(\sqrt{mn})$ \\ \hline $(3, 1)$ & $\sqrt{n}$ & $1$ & $D(\sqrt{n})$ & $D(\sqrt{n})$ \\ \hline $(3, 2)$ & $\sqrt{n}$ & $\sqrt{m}$ & $D(\sqrt{mn})$ & $D(\sqrt{n}) \tau(\sqrt{m}) + \sigma(\sqrt{n}) D(\sqrt{m})$ \\ \hline $(3, 3)$ & $\sqrt{n}$ & $\sqrt{n}$ & $0$ & $(\sigma(\sqrt{n}) + \tau(\sqrt{n}))D(\sqrt{n})$ \\ \hline $(3, 4)$ & $\sqrt{n}$ & $\sqrt{mn}$ & $n D(\sqrt{m})$ & $D(\sqrt{n}) \tau(\sqrt{mn}) + \sigma(\sqrt{n}) D(\sqrt{mn})$ \\ \hline $(4, 1)$ & $\sqrt{mn}$ & $1$ & $D(\sqrt{mn})$ & $D(\sqrt{mn})$ \\ \hline $(4, 2)$ & $\sqrt{mn}$ & $\sqrt{m}$ & $mD(\sqrt{n})$ & $D(\sqrt{mn}) \tau(\sqrt{m}) + \sigma(\sqrt{mn}) D(\sqrt{m})$ \\ \hline $(4, 3)$ & $\sqrt{mn}$ & $\sqrt{n}$ & $nD(\sqrt{m})$ & $D(\sqrt{mn}) \tau(\sqrt{n}) + \sigma(\sqrt{mn}) D(\sqrt{n})$ \\ \hline $(4, 4)$ & $\sqrt{mn}$ & $\sqrt{mn}$ & $0$ & $(\sigma(\sqrt{mn}) + \tau(\sqrt{mn}))D(\sqrt{mn})$ \\ \hline \end{tabular} \label{table 2} \end{table} From Table \ref{table 2} containing the values of $D(\alpha_{i} \alpha_{j})$ and $D(\alpha_{i}) \tau(\alpha_{j}) + \sigma(\alpha_{i}) D(\alpha_{j})$ ($i, j \in \{1, 2, 3, 4\}$), we see that the relation $D(\alpha_{i} \alpha_{j}) = D(\alpha_{i}) \tau(\alpha_{j}) + \sigma(\alpha_{i}) D(\alpha_{j})$ is being satisfied by the pairs $$(i,j) = (1,1), (1,2), (1,3), (1,4), (2,1), (3,1), (4,1).$$ Then using the fact that the $\mathbb{Z}$-algebra $\mathcal{A} = \mathbb{Z}[\sqrt{m}, \sqrt{n}]$ is commutative, we get that $D$ will be a $(\sigma, \tau)$-derivation if and only if it satisfies the following nine equations simultaneously. \begin{equation} \label{eq 3.4} (\sigma(\sqrt{m}) + \tau(\sqrt{m}))D(\sqrt{m}) = 0 \end{equation} \begin{equation} \label{eq 3.5} D(\sqrt{m}) \tau(\sqrt{n}) + \sigma(\sqrt{m}) D(\sqrt{n}) = D(\sqrt{mn}) \end{equation} \begin{equation} \label{eq 3.6} D(\sqrt{m}) \tau(\sqrt{mn}) + \sigma(\sqrt{m}) D(\sqrt{mn}) = m D(\sqrt{n}) \end{equation} \begin{equation} \label{eq 3.7} (\sigma(\sqrt{n}) + \tau(\sqrt{n}))D(\sqrt{n}) = 0 \end{equation} \begin{equation} \label{eq 3.8} D(\sqrt{n}) \tau(\sqrt{mn}) + \sigma(\sqrt{n}) D(\sqrt{mn}) = n D(\sqrt{m}) \end{equation} \begin{equation} \label{eq 3.9} (\sigma(\sqrt{mn}) + \tau(\sqrt{mn}))D(\sqrt{mn}) = 0 \end{equation} \begin{equation} \label{eq 3.10} D(\sqrt{n}) \tau(\sqrt{m}) + \sigma(\sqrt{n}) D(\sqrt{m}) = D(\sqrt{mn}) \end{equation} \begin{equation} \label{eq 3.11} D(\sqrt{mn}) \tau(\sqrt{m}) + \sigma(\sqrt{mn}) D(\sqrt{m}) = m D(\sqrt{n}) \end{equation} \begin{equation} \label{eq 3.12} D(\sqrt{mn}) \tau(\sqrt{n}) + \sigma(\sqrt{mn}) D(\sqrt{n}) = n D(\sqrt{m}) \end{equation} Since $\mathcal{A} = \mathbb{Z}[\sqrt{m}, \sqrt{n}]$ is an integral domain, so in view of equations (\ref{eq 3.4}), (\ref{eq 3.7}) and (\ref{eq 3.9}), there are four possible cases.\vspace{0.2cm} Case 1: All of $D(\sqrt{m})$, $D(\sqrt{n})$ and $D(\sqrt{mn})$ are non-zero. Since $\mathcal{A} = \mathbb{Z}[\sqrt{m}, \sqrt{n}]$ is an integral domain, therefore, from (\ref{eq 3.4}), (\ref{eq 3.7}) and (\ref{eq 3.9}), $$\tau(\sqrt{m}) = - \sigma(\sqrt{m}), \hspace{0.2cm} \tau(\sqrt{n}) = - \sigma(\sqrt{n}) \hspace{0.2cm} \text{and} \hspace{0.2cm} \tau(\sqrt{mn}) = - \sigma(\sqrt{mn}).$$ But from Table \ref{table 1}, we observe that there is no such pair $(\sigma, \tau)$ which satisfies the above three equations simultaneously. Therefore, this case is not possible.\vspace{0.2cm} Case 2: Exactly one of $D(\sqrt{m})$, $D(\sqrt{n})$ and $D(\sqrt{mn})$ is non-zero. Then there are three possible subcases. Subcase 2.1: $D(\sqrt{m}) = 0$, $D(\sqrt{n}) \neq 0$, $D(\sqrt{mn}) \neq 0$. By equations (\ref{eq 3.7}) and (\ref{eq 3.9}) respectively, $\tau(\sqrt{n}) = - \sigma(\sqrt{n})$ and $\tau(\sqrt{mn}) = - \sigma(\sqrt{mn})$. Also, by (\ref{eq 3.5}) and (\ref{eq 3.10}), $\tau(\sqrt{m}) = \sigma(\sqrt{m})$. Therefore, in view of Table \ref{table 1}, the possible values of the pair $(\sigma, \tau)$ are $$(\phi_{1}, \phi_{2}), \hspace{0.2cm} (\phi_{2}, \phi_{1}), \hspace{0.2cm} (\phi_{3}, \phi_{4}), \hspace{0.2cm} (\phi_{4}, \phi_{3}).$$ Further, from (\ref{eq 3.5}), $D(\sqrt{mn}) = \sigma(\sqrt{m}) D(\sqrt{n})$ and $\sigma(\sqrt{m}) = \pm \sqrt{m}$, therefore, $D$ must satisfy $D(\sqrt{mn}) = \pm \sqrt{m} D(\sqrt{n})$. More precisely, we have the following table \ref{table 3}: \begin{table}[H] \caption{} \centering \begin{tabular}{|c|c|c|c|} \hline $(\sigma, \tau)$ & $D(\sqrt{mn}) = \pm \sqrt{m} D(\sqrt{n})?$ \\ \hline \hline $(\phi_{1}, \phi_{2})$ & $D(\sqrt{mn}) = \sqrt{m} D(\sqrt{n})$ \\ \hline $(\phi_{2}, \phi_{1})$ & $D(\sqrt{mn}) = \sqrt{m} D(\sqrt{n})$ \\ \hline $(\phi_{3}, \phi_{4})$ & $D(\sqrt{mn}) = - \sqrt{m} D(\sqrt{n})$ \\ \hline $(\phi_{4}, \phi_{3})$ & $D(\sqrt{mn}) = - \sqrt{m} D(\sqrt{n})$ \\ \hline \end{tabular} \label{table 3} \end{table} Subcase 2.2: $D(\sqrt{m}) \neq 0$, $D(\sqrt{n}) = 0$, $D(\sqrt{mn}) \neq 0$. By equations (\ref{eq 3.4}) and (\ref{eq 3.9}) respectively, $\tau(\sqrt{m}) = - \sigma(\sqrt{m})$ and $\tau(\sqrt{mn}) = - \sigma(\sqrt{mn})$. Also, by (\ref{eq 3.5}) and (\ref{eq 3.10}), $\tau(\sqrt{n}) = \sigma(\sqrt{n})$. Therefore, in view of Table \ref{table 1}, the possible values of the pair $(\sigma, \tau)$ are $$(\phi_{1}, \phi_{3}), \hspace{0.2cm} (\phi_{2}, \phi_{4}), \hspace{0.2cm} (\phi_{3}, \phi_{1}), \hspace{0.2cm} (\phi_{4}, \phi_{2}).$$ Further from (\ref{eq 3.10}), $D(\sqrt{mn}) = \sigma(\sqrt{n}) D(\sqrt{m})$ and $\sigma(\sqrt{n}) = \pm \sqrt{n}$, therefore, $D$ must satisfy $D(\sqrt{mn}) = \pm \sqrt{n} D(\sqrt{m})$. More precisely, we have the following table \ref{table 4}: \begin{table}[H] \caption{} \centering \begin{tabular}{|c|c|c|c|} \hline $(\sigma, \tau)$ & $D(\sqrt{mn}) = \pm \sqrt{n} D(\sqrt{m})?$ \\ \hline \hline $(\phi_{1}, \phi_{3})$ & $D(\sqrt{mn}) = \sqrt{n} D(\sqrt{m})$ \\ \hline $(\phi_{2}, \phi_{4})$ & $D(\sqrt{mn}) = - \sqrt{n} D(\sqrt{m})$ \\ \hline $(\phi_{3}, \phi_{1})$ & $D(\sqrt{mn}) = \sqrt{n} D(\sqrt{m})$ \\ \hline $(\phi_{4}, \phi_{2})$ & $D(\sqrt{mn}) = - \sqrt{n} D(\sqrt{m})$ \\ \hline \end{tabular} \label{table 4} \end{table} Subcase 2.3: $D(\sqrt{m}) \neq 0$, $D(\sqrt{n}) \neq 0$, $D(\sqrt{mn}) = 0$. By equations (\ref{eq 3.4}) and (\ref{eq 3.7}) respectively, $\tau(\sqrt{m}) = - \sigma(\sqrt{m})$ and $\tau(\sqrt{n}) = - \sigma(\sqrt{n})$. Also, by equations (\ref{eq 3.6}) and (\ref{eq 3.11}), $\tau(\sqrt{mn}) = \sigma(\sqrt{mn})$. Therefore, in view of Table \ref{table 1}, the possible values of the pair $(\sigma, \tau)$ are $$(\phi_{1}, \phi_{4}), \hspace{0.2cm} (\phi_{2}, \phi_{3}), \hspace{0.2cm} (\phi_{4}, \phi_{1}), \hspace{0.2cm} (\phi_{3}, \phi_{2}).$$ Further from equation (\ref{eq 3.11}), $m D(\sqrt{n}) = \sigma(\sqrt{mn}) D(\sqrt{m})$ and $\sigma(\sqrt{mn}) = \pm \sqrt{mn}$, therefore, $D$ must satisfy $m D(\sqrt{n}) = \pm \sqrt{mn} D(\sqrt{m})$ so that $\sqrt{m} D(\sqrt{n}) = \pm \sqrt{n} D(\sqrt{m})$. More precisely, we have the following table \ref{table 5}: \begin{table}[H] \caption{} \centering \begin{tabular}{|c|c|c|c|} \hline $(\sigma, \tau)$ & $\sqrt{m} D(\sqrt{n}) = \sqrt{n} D(\sqrt{m})?$ \\ \hline \hline $(\phi_{1}, \phi_{4})$ & $\sqrt{m} D(\sqrt{n}) = \sqrt{n} D(\sqrt{m})$ \\ \hline $(\phi_{2}, \phi_{3})$ & $\sqrt{m} D(\sqrt{n}) = - \sqrt{n} D(\sqrt{m})$ \\ \hline $(\phi_{3}, \phi_{2})$ & $\sqrt{m} D(\sqrt{n}) = - \sqrt{n} D(\sqrt{m})$ \\ \hline $(\phi_{4}, \phi_{1})$ & $\sqrt{m} D(\sqrt{n}) = \sqrt{n} D(\sqrt{m})$ \\ \hline \end{tabular} \label{table 5} \end{table} Case 3: Exactly two of $D(\sqrt{m})$, $D(\sqrt{n})$ or $D(\sqrt{mn})$ are $0$. Then there are three possible subcases. Subcase 3.1: $D(\sqrt{m}) = 0$, $D(\sqrt{n}) = 0$, $D(\sqrt{mn}) \neq 0$. Subcase 3.2: $D(\sqrt{m}) = 0$, $D(\sqrt{n}) \neq 0$, $D(\sqrt{mn}) = 0$. Subcase 3.3: $D(\sqrt{m}) \neq 0$, $D(\sqrt{n}) = 0$, $D(\sqrt{mn}) = 0$. Then in each of the three possible subcases, $D$ will become a zero derivation using equations (\ref{eq 3.4}) to (\ref{eq 3.12}) and the facts that $\{1, \sqrt{m}, \sqrt{n}, \sqrt{mn}\}$ is a $\mathbb{Z}$-basis of the $\mathbb{Z}$-module $\mathcal{A} = \mathbb{Z}[\sqrt{m}, \sqrt{n}]$ and $D:\mathcal{A} \rightarrow \mathcal{A}$ is $\mathbb{Z}$-linear with $D(1)=0$. But this gives a contradiction because $D$ is non-zero. Therefore, this case is not possible.\vspace{0.2cm} Case 4: All $D(\sqrt{p})$, $D(\sqrt{q})$ and $D(\sqrt{pq})$ are zero. The proof is similar to Case 3. \end{proof} The proof of the following lemma is trivial using \th\ref{lemma 2.1}. \begin{lemma}\th\label{lemma 3.17} Let $D:\mathcal{A} \rightarrow \mathcal{A}$ be any $\mathbb{Z}$-linear map with $D(1)=0$. Then $D$ is a $(\sigma, \tau)$-derivation in the following cases. \begin{enumerate} \item[(i)] $D(\sqrt{m}) = 0$, $D(\sqrt{n}) \neq 0$ and $D(\sqrt{mn}) \neq 0$. \begin{itemize} \item[(a)] If $D(\sqrt{mn}) = \sqrt{m} D(\sqrt{n})$ and $(\sigma, \tau) = (\phi_{1}, \phi_{2})$ or $(\phi_{2}, \phi_{1})$. \item[(b)] If $D(\sqrt{mn}) = - \sqrt{m} D(\sqrt{n})$ and $(\sigma, \tau) = (\phi_{3}, \phi_{4})$ or $(\phi_{4}, \phi_{3})$. \end{itemize} \item[(ii)] $D(\sqrt{m}) \neq 0$, $D(\sqrt{n}) = 0$ and $D(\sqrt{mn}) \neq 0$. \begin{itemize} \item[(a)] If $D(\sqrt{mn}) = \sqrt{n} D(\sqrt{m})$ and $(\sigma, \tau) = (\phi_{1}, \phi_{3})$ or $(\phi_{3}, \phi_{1})$. \item[(b)] If $D(\sqrt{mn}) = - \sqrt{n} D(\sqrt{m})$ and $(\sigma, \tau) = (\phi_{2}, \phi_{4})$ or $(\phi_{4}, \phi_{2})$. \end{itemize} \item[(iii)] $D(\sqrt{m}) \neq 0$, $D(\sqrt{n}) \neq 0$ and $D(\sqrt{mn}) = 0$. \begin{itemize} \item[(a)] If $\sqrt{m} D(\sqrt{n}) = \sqrt{n} D(\sqrt{m})$ and $(\sigma, \tau) = (\phi_{1}, \phi_{4})$ or $(\phi_{4}, \phi_{1})$. \item[(b)] If $\sqrt{m} D(\sqrt{n}) = - \sqrt{n} D(\sqrt{m})$ and $(\sigma, \tau) = (\phi_{2}, \phi_{3})$ or $(\phi_{3}, \phi_{2})$. \end{itemize} \end{enumerate} \end{lemma} \begin{theorem}\th\label{theorem 3.18} A non-zero $\mathbb{Z}$-linear map $D:\mathcal{A} \rightarrow \mathcal{A}$ with $D(1)=0$ is a $(\sigma, \tau)$-derivation if and only if exactly one of the cases (i), (ii) or (iii) of \th\ref{lemma 3.16} holds. \end{theorem} \begin{corollary}\th\label{corollary 3.19} The $\mathbb{Z}$-module $\mathcal{D}_{(\sigma, \tau)}(\mathcal{A})$ is finitely generated of rank $4$. \end{corollary} \begin{proof} $\mathcal{A} = \mathbb{Z}[\sqrt{m}, \sqrt{n}]$ has exactly four distinct ring endomorphisms as given in Table \ref{table 1}. In view of \th\ref{theorem 3.18}, there are three possible cases. Case 1: $(\sigma, \tau) \in \{(\phi_{1}, \phi_{2}), (\phi_{2}, \phi_{1}), (\phi_{3}, \phi_{4}), (\phi_{4}, \phi_{3})\}$. For each $i \in \{1, 2, 3, 4\}$, define $D_{i}:\mathcal{A} \rightarrow \mathcal{A}$ as a $\mathbb{Z}$-linear map with $D(1) = 0$ such that $$D_{1}(\sqrt{n}) = 1, \hspace{0.2cm} D_{2}(\sqrt{n}) = \sqrt{m}, \hspace{0.2cm} D_{3}(\sqrt{n}) = \sqrt{n}, \hspace{0.2cm} D_{4}(\sqrt{n}) = \sqrt{mn}.$$ More precisely, for each $i \in \{1, 2, 3, 4\}$, define $D_{i}:\mathcal{A} \rightarrow \mathcal{A}$ by $$D_{i}(a_{1} + a_{2} \sqrt{m} + a_{3} \sqrt{n} + a_{4} \sqrt{mn}) = \begin{cases} (a_{3} + a_{4} \sqrt{m}) D_{i}(\sqrt{n}) & \text{if $(\sigma, \tau) = (\phi_{1}, \phi_{2}) \hspace{0.2cm} \text{or} \hspace{0.2cm} (\phi_{2}, \phi_{1})$} \\ (a_{3} - a_{4} \sqrt{m}) D_{i}(\sqrt{n}) & \text{if $(\sigma, \tau) = (\phi_{3}, \phi_{4}) \hspace{0.2cm} \text{or} \hspace{0.2cm} (\phi_{4}, \phi_{3})$} \end{cases}$$ where for each $i \in \{1, 2, 3, 4\}$, $D_{i}(\sqrt{n})$ is defined as above.\vspace{10pt} Case 2: $(\sigma, \tau) \in \{(\phi_{1}, \phi_{3}), (\phi_{3}, \phi_{1}), (\phi_{2}, \phi_{4}), (\phi_{4}, \phi_{2})\}$. For each $i \in \{1, 2, 3, 4\}$, define $D_{i}:\mathcal{A} \rightarrow \mathcal{A}$ as a $\mathbb{Z}$-linear map with $D(1) = 0$ such that $$D_{1}(\sqrt{m}) = 1, \hspace{0.2cm} D_{2}(\sqrt{m}) = \sqrt{m}, \hspace{0.2cm} D_{3}(\sqrt{m}) = \sqrt{n}, \hspace{0.2cm} D_{4}(\sqrt{m}) = \sqrt{mn}.$$ More precisely, for each $i \in \{1, 2, 3, 4\}$, define $D_{i}:\mathcal{A} \rightarrow \mathcal{A}$ by $$D_{i}(a_{1} + a_{2} \sqrt{m} + a_{3} \sqrt{n} + a_{4} \sqrt{mn}) = \begin{cases} (a_{2} + a_{4} \sqrt{n}) D_{i}(\sqrt{m}) & \text{if $(\sigma, \tau) = (\phi_{1}, \phi_{3}) \hspace{0.2cm} \text{or} \hspace{0.2cm} (\phi_{3}, \phi_{1})$} \\ (a_{2} - a_{4} \sqrt{n}) D_{i}(\sqrt{m}) & \text{if $(\sigma, \tau) = (\phi_{2}, \phi_{4}) \hspace{0.2cm} \text{or} \hspace{0.2cm} (\phi_{4}, \phi_{2})$} \end{cases}$$ where for each $i \in \{1, 2, 3, 4\}$, $D_{i}(\sqrt{m})$ is defined as above.\vspace{10pt} Case 3: $(\sigma, \tau) \in \{(\phi_{1}, \phi_{4}), (\phi_{4}, \phi_{1}), (\phi_{2}, \phi_{3}), (\phi_{3}, \phi_{2})\}$. Suppose $\text{gcd}(m, n) = k$. Then $m = kr$ and $n = ks$ for some $r, s \in \mathbb{Z}$. For each $i \in \{1, 2, 3, 4\}$, define $D_{i}:\mathcal{A} \rightarrow \mathcal{A}$ as a $\mathbb{Z}$-linear map with $D(1) = 0$ such that $$D_{1}(\sqrt{m}) = m, \hspace{0.2cm} D_{2}(\sqrt{m}) = \sqrt{m}, \hspace{0.2cm} D_{3}(\sqrt{m}) = r \sqrt{n}, \hspace{0.2cm} D_{4}(\sqrt{m}) = \sqrt{mn}$$ so that $$D_{1}(\sqrt{n}) = \sqrt{mn}, \hspace{0.2cm} D_{2}(\sqrt{n}) = \sqrt{n}, \hspace{0.2cm} D_{3}(\sqrt{n}) = s \sqrt{m}, \hspace{0.2cm} D_{4}(\sqrt{n}) = n.$$ More precisely, for each $i \in \{1, 2, 3, 4\}$, define $D_{i}:\mathcal{A} \rightarrow \mathcal{A}$ by $$D_{i}(a_{1} + a_{2} \sqrt{m} + a_{3} \sqrt{n} + a_{4} \sqrt{mn}) = a_{2} D_{i}(\sqrt{m}) + a_{3} D_{i}(\sqrt{n})$$ where for each $i \in \{1, 2, 3, 4\}$, $D_{i}(\sqrt{m})$ and $D_{i}(\sqrt{n})$ are defined as above. In all three cases, it can be easily verified that $\{D_{1}, D_{2}, D_{3}, D_{4}\}$ is a linearly independent subset of the $\mathbb{Z}$-module $\mathcal{D}_{(\sigma, \tau)}(\mathcal{A})$ that generates it. \end{proof} We note from the preceding corollary that the rank of the $\mathbb{Z}$-module $\mathcal{D}_{(\sigma, \tau)}(\mathcal{A})$ is equal to $4$, the degree of $K = \mathbb{Q}(\sqrt{m}, \sqrt{n})$ and which, by \th\ref{theorem 1.2}, is also equal to the rank of the free abelian group $\mathcal{A} = \mathbb{Z}[\sqrt{m}, \sqrt{n}]$ as a $\mathbb{Z}$-module. \begin{corollary}\th\label{corollary 3.20} There always exists a non-zero $(\sigma, \tau)$-derivation of $\mathcal{A}$. \end{corollary} \begin{corollary}\th\label{corollary 3.21} There always exists a non-zero $\mathbb{Z}$-linear map $D:\mathcal{A} \rightarrow \mathcal{A}$ with $D(1)=0$ that is not a $(\sigma, \tau)$-derivation. In fact, there exists a non-zero $\mathbb{Z}$-linear map $D:\mathcal{A} \rightarrow \mathcal{A}$ with $D(1)=0$ which is not a $(\sigma, \tau)$-derivation for every pair $(\sigma, \tau)$ of any two different non-zero ring endomorphisms of $\mathcal{A}$. \end{corollary} \begin{theorem}\th\label{theorem 3.22} Let $D:\mathcal{A} \rightarrow \mathcal{A}$ be a non-zero $(\sigma, \tau)$-derivation. \begin{enumerate} \item[(i)] If $D(\sqrt{m}) = 0$, $D(\sqrt{n}) \neq 0$ and $D(\sqrt{mn}) \neq 0$, then $D$ is inner if and only if $\frac{D(\sqrt{n})}{2 \sqrt{n}} \in \mathcal{A} = \mathbb{Z}[\sqrt{m}, \sqrt{n}]$. In particular, $\mathcal{A}$ has non-trivial outer $(\sigma, \tau)$-derivations, where $(\sigma, \tau) \in \{(\phi_{1}, \phi_{2}), (\phi_{2}, \phi_{1}), (\phi_{3}, \phi_{4}), (\phi_{4}, \phi_{3})\}$. \item[(ii)] If $D(\sqrt{m}) \neq 0$, $D(\sqrt{n}) = 0$ and $D(\sqrt{mn}) \neq 0$, then $D$ is inner if and only if $\frac{D(\sqrt{}m)}{2 \sqrt{m}} \in \mathcal{A} = \mathbb{Z}[\sqrt{m}, \sqrt{n}]$. In particular, $\mathcal{A}$ has non-trivial outer $(\sigma, \tau)$-derivations, where $(\sigma, \tau) \in \{(\phi_{1}, \phi_{3}), (\phi_{2}, \phi_{4}), (\phi_{3}, \phi_{1}), (\phi_{4}, \phi_{2})\}$. \item[(iii)] If $D(\sqrt{m}) \neq 0$, $D(\sqrt{n}) \neq 0$ and $D(\sqrt{mn}) = 0$, then $D$ is inner if and only if either $\frac{D(\sqrt{}m)}{2 \sqrt{m}} \in \mathcal{A} = \mathbb{Z}[\sqrt{m}, \sqrt{n}]$ or $\frac{D(\sqrt{n})}{2 \sqrt{n}} \in \mathcal{A} = \mathbb{Z}[\sqrt{m}, \sqrt{n}]$. In particular, $\mathcal{A}$ has non-trivial outer $(\sigma, \tau)$-derivations, where $(\sigma, \tau) \in \{(\phi_{1}, \phi_{4}), (\phi_{2}, \phi_{3}), (\phi_{4}, \phi_{1}), (\phi_{3}, \phi_{2})\}$. \end{enumerate} \end{theorem} \begin{proof} $\mathcal{A} = \mathbb{Z}[\sqrt{m}, \sqrt{n}]$ has exactly four distinct ring endomorphisms as given in Table \ref{table 1}. In view of \th\ref{theorem 3.18}, there are three possible cases. Case 1: $D(\sqrt{m}) = 0$, $D(\sqrt{n}) \neq 0$ and $D(\sqrt{mn}) \neq 0$. In this case, $(\sigma, \tau) \in \{(\phi_{1}, \phi_{2}), (\phi_{2}, \phi_{1}), (\phi_{3}, \phi_{4}), (\phi_{4}, \phi_{3})\}$. In view of Subcase 2.1 in the proof of \th\ref{lemma 3.16}, in this case, $$\tau(\sqrt{m}) = \sigma(\sqrt{m}), \hspace{0.2cm} \tau(\sqrt{n}) = - \sigma(\sqrt{n}) \hspace{0.2cm} \text{and} \hspace{0.2cm} \tau(\sqrt{mn}) = - \sigma(\sqrt{mn}).$$ Note that $(\tau - \sigma)(1) = 0$, $(\tau - \sigma)(\sqrt{m}) = 0$, $(\tau - \sigma)(\sqrt{n}) = 2 \tau(\sqrt{n})$ and $(\tau - \sigma)(\sqrt{mn}) = 2 \tau(\sqrt{mn}) = 2 \tau(\sqrt{m}) \tau(\sqrt{n})$. By \th\ref{theorem 2.5}, $D$ is inner if and only if there is a $\beta \in \mathcal{A}$ with $$D(1) = \beta (\tau - \sigma)(1), \hspace{0.2cm} D(\sqrt{m}) = \beta (\tau - \sigma)(\sqrt{m}),$$ $$D(\sqrt{n}) = \beta (\tau - \sigma)(\sqrt{n}), \hspace{0.2cm} D(\sqrt{mn}) = \beta (\tau - \sigma)(\sqrt{mn}).$$ Note that $D(1) = \alpha (\tau - \sigma)(1)$ and $D(\sqrt{m}) = \alpha (\tau - \sigma)(\sqrt{m})$ hold for all $\alpha \in \mathcal{A}$. So $D$ will be inner if and only if $$D(\sqrt{n}) = \beta (\tau - \sigma)(\sqrt{n}) \hspace{0.2cm} \text{and} \hspace{0.2cm} D(\sqrt{mn}) = \beta (\tau - \sigma)(\sqrt{mn}),$$ that is, if and only if $$D(\sqrt{n}) = 2 \beta \tau (\sqrt{n}) \hspace{0.2cm} \text{and} \hspace{0.2cm} D(\sqrt{mn}) = 2 \beta \tau (\sqrt{m}) \tau (\sqrt{n}).$$ \noindent Further since in this case, $D(\sqrt{mn}) = \begin{cases} \sqrt{m} D(\sqrt{n}) \hspace{0.2cm} \text{if $\tau = \phi_{1}$ or $\phi_{2}$} \\ - \sqrt{m} D(\sqrt{n}) \hspace{0.2cm} \text{if $\tau = \phi_{3}$ or $\phi_{4}$} \end{cases}$ and \\ $2 \beta \tau(\sqrt{m}) \tau(\sqrt{n}) = \begin{cases} 2 \beta \sqrt{mn} \hspace{0.2cm} \text{if $\tau = \phi_{1}$ or $\tau = \phi_{4}$} \\ - 2 \beta \sqrt{mn} \hspace{0.2cm} \text{if $\tau = \phi_{2}$ or $\tau = \phi_{3}$} \end{cases}$ therefore, \\ $D(\sqrt{mn}) = 2 \beta \tau (\sqrt{m}) \tau (\sqrt{n})$ if and only if $D(\sqrt{n}) = \begin{cases} 2 \beta \sqrt{n} \hspace{0.2cm} \text{if $\tau = \phi_{1}$ or $\phi_{3}$} \\ - 2 \beta \sqrt{n} \hspace{0.2cm} \text{if $\tau = \phi_{2}$ or $\phi_{4}$} \end{cases}.$ From the above discussion, it can be easily concluded that $D$ is inner if and only if $$D(\sqrt{n}) = \begin{cases} 2 \beta \sqrt{n} \hspace{0.2cm} \text{if $\tau = \phi_{1}$ or $\phi_{3}$} \\ - 2 \beta \sqrt{n} \hspace{0.2cm} \text{if $\tau = \phi_{2}$ or $\phi_{4}$} \end{cases}.$$ Therefore, $D$ is inner if and only if there is a $\beta \in \mathcal{A}$ with $$\beta = \begin{cases} \frac{D(\sqrt{n})}{2 \sqrt{n}} \hspace{0.2cm} \text{if $\tau = \phi_{1}$ or $\phi_{3}$} \\ - \frac{D(\sqrt{n})}{2 \sqrt{n}} \hspace{0.2cm} \text{if $\tau = \phi_{2}$ or $\phi_{4}$} \end{cases}.$$ Therefore, in this case, $D$ is inner if and only if $\frac{D(\sqrt{n})}{2 \sqrt{n}} \in \mathcal{A}$.\vspace{10pt} Case 2: $D(\sqrt{m}) \neq 0$, $D(\sqrt{n}) = 0$ and $D(\sqrt{mn}) \neq 0$. As in Case 1, it can be shown that $D$ is inner if and only if there is a $\beta \in \mathcal{A}$ with $$\beta = \begin{cases} \frac{D(\sqrt{m})}{2 \sqrt{m}} \hspace{0.2cm} \text{if $\tau = \phi_{1}$ or $\phi_{2}$} \\ - \frac{D(\sqrt{m})}{2 \sqrt{m}} \hspace{0.2cm} \text{if $\tau = \phi_{3}$ or $\phi_{4}$} \end{cases}.$$ Therefore, in this case, $D$ is inner if and only if $\frac{D(\sqrt{m})}{2 \sqrt{m}} \in \mathcal{A}$.\vspace{10pt} Case 3: $D(\sqrt{m}) \neq 0$, $D(\sqrt{n}) \neq 0$ and $D(\sqrt{mn}) = 0$. On similar lines, it can be shown that in this case, $D$ is inner if and only if either $\frac{D(\sqrt{m})}{2 \sqrt{m}} \in \mathcal{A}$ or $\frac{D(\sqrt{n})}{2 \sqrt{n}} \in \mathcal{A}$ (as $\frac{D(\sqrt{m})}{2 \sqrt{m}} = \frac{D(\sqrt{n})}{2 \sqrt{n}}$). Hence the result. \end{proof} The corollary is immediate. \begin{corollary}\th\label{corollary 3.23} There always exists a non-zero $(\sigma, \tau)$-inner derivation of $\mathcal{A}$. \end{corollary} \section{Coding Theory Applications}\label{section 4} In this section, we give the applications of the work of previous sections in coding theory. We give the notion of a Hom-IDD code over finite fields. \subsection{IDD Code in $O_{K}$}\label{subsection 4.1} Let $K = \mathbb{Q}(\theta)$ ($\theta$ an algebraic integer) be a number field of degree $n$ over $\mathbb{Q}$, which is a normal extension of $\mathbb{Q}$ and $\mathcal{B} = \{\theta_{1} = \theta, \theta_{2}, ..., \theta_{n}\}$ be an ordered integral basis of $O_{K}$. Let $\sigma, \tau:O_{K} \rightarrow O_{K}$ be two different $\mathbb{Z}$-algebra endomorphisms of $O_{K}$. Let $D$ be a $(\sigma, \tau)$-derivation of $O_{K}$. The range $\text{Im}(D)$ of $D$ is a $\mathbb{Z}$-submodule of the $\mathbb{Z}$-module $O_{K}$ with the generating set $D(\mathcal{B}) = \{D(\theta_{i}) | 1 \leq i \leq n\}$. It generates an $(n,r)$-code in $O_{K}$, where $r = \text{rank}(Im(D))$, the rank of the $\mathbb{Z}$-module $\text{Im}(D)$. Suppose $S = \{D(\theta_{i_{1}}),D(\theta_{i_{2}}),...,D(\theta_{i_{s}})\}$ is a $\mathbb{Z}$-linearly independent subset of $\text{Im}(D)$. Then $S$ generates a submodule of $\text{Im}(D)$ with rank $s$. Thus, $S$ generates an $(n,s)$-code. \begin{definition} Let $T = \{\theta_{k_{1}}, \theta_{k_{2}}, ..., \theta_{k_{t}}\}$ ($\{k_{1}, k_{2}, ..., k_{t}\} \subseteq \{1, 2, ..., n\}$) be a subset of the integral basis $\{\theta_{1}, \theta_{2}, ..., \theta_{n}\}$ such that $D(T)$ is $\mathbb{Z}$-linearly independent subset of $O_{K}$ (or $\text{Im}(D)$). Then the submodule $W = \langle D(T) \rangle$ generates an $(n,t)$-code, and is called as Image of Derivation-Derived Code, or simply an IDD Code. \end{definition} \subsection{Equivalent IDD Code in $\mathbb{Z}^{n}$}\label{subsection 4.2} In this subsection, we construct a code in $\mathbb{Z}^{n}$ equivalent to an IDD code. Suppose $\Delta:\mathbb{Z}^{n} \rightarrow O_{K}$ be the mapping given by $$\Delta(a_{1}, a_{2}, ..., a_{n}) = \sum_{i=1}^{n} a_{i} \theta_{i}.$$ \noindent For each $i \in \{1, 2, ..., n\}$, suppose $D(\theta_{i}) = \sum_{j=1}^{n} a_{ij}\theta_{j}$. Then $\Delta^{-1}(D(\theta_{i})) = (a_{i1}, a_{i2}, ..., a_{in})$. Construct an $n \times n$ matrix $B$ with the rows $\Delta^{-1}(D(\theta_{1})), \Delta^{-1}(D(\theta_{2})), ..., \Delta^{-1}(D(\theta_{n}))$. More precisely, $$B = \begin{pmatrix} a_{11} & a_{12} & \cdots & a_{1n} \\ a_{21} & a_{22} & \cdots & a_{2n} \\ \vdots & \vdots & \ddots & \vdots \\ a_{n1} & a_{n2} & ... & a_{nn} \\ \end{pmatrix}.$$ \begin{theorem} Any rows $i_{1}, i_{2}, ..., i_{t}$ of $B$ are $\mathbb{Z}$-linearly independent if and only if the set $\{D(\theta_{i_{1}}), D(\theta_{i_{2}}), ..., D(\theta_{i_{t}})\}$ is $\mathbb{Z}$-linearly independent. \end{theorem} Below, we give the steps of construction: \begin{enumerate} \item[(i)] Suppose $W$ is an Image of Derivations-Derived (IDD) code. Then $W = \langle S \rangle$, for some $\mathbb{Z}$-linearly independent subset $S$ of $D(\mathcal{B})$, say, $S = \{D(\theta_{k_{1}}), D(\theta_{k_{2}}), ..., D(\theta_{k_{s}})\}$.\vspace{6pt} \item[(ii)] Pick $\underline{w} \in \mathbb{Z}^{s}$ and $\underline{w} = (\lambda_{1}, \lambda_{2}, ..., \lambda_{s})$.\vspace{6pt} \item[(iii)] Write $\underline{w}$ as $\underline{x}$ in $\mathbb{Z}^{n}$ with $\lambda_{j}$ in position $k_{j}$ and zero elsewhere.\vspace{6pt} \item[(iv)] $\underline{x}$ can be mapped to an element $O_{K}$ by $x = \Delta(\underline{x}) = \sum_{j=1}^{s}\lambda_{j}\theta_{k_{j}}$ and a codeword $$D(x) = D(\Delta(\underline{x})) = D(\sum_{j=1}^{s}\lambda_{j}\theta_{k_{j}}) = \sum_{j=1}^{s}\lambda_{j}D(\theta_{k_{j}})$$ equated with a codeword in $\mathcal{E}$ given by $$\Delta^{-1}(D(x)) = \underline{x}B.$$ \item[(v)] Thus, we obtain an $(n,s)$ code $$\mathcal{E} = \{\underline{x}B \mid \underline{x} ~ \text{extends} ~ \underline{w} \in \mathbb{Z}^{s} ~ \text{to} ~ \mathbb{Z}^{n}\}.$$ \end{enumerate} \subsection{Hom-IDD Code in $(\mathbb{Z}_{q})^{n}$}\label{subsection 4.3} For a positive integer $q$ and for an equivalent IDD code $C$ in $\mathbb{Z}^{n}$, we construct a Hom-IDD code $\Omega(C)$ in $(\mathbb{Z}_{q})^{n}$. \begin{lemma}\label{lemma 4.3} Let $q$ be a positive integer. Then the map $\Omega:\mathbb{Z}^{n} \rightarrow (\mathbb{Z}_{q})^{n}$ defined by $$\Omega(\lambda_{1}, \lambda_{2}, ..., \lambda_{n}) = (\overline{\lambda_{1}}, \overline{\lambda_{2}}, ..., \overline{\lambda_{n}}),$$ where $\overline{\lambda_{i}} = \lambda_{i}$ (mod $q$) for each $i \in \{1, 2, ..., n\}$ is a $\mathbb{Z}$-module homomorphism. \end{lemma} \begin{proof} Both $\mathbb{Z}^{n}$ and $(\mathbb{Z}_{q})^{n}$ are abelian groups with respect to the usual operations of componentwise addition. And since every abelian group is a $\mathbb{Z}$-module with respect to the usual operation of scalar multiplication. More precisely, if $M$ is an additive abelian group, then $M$ is a $\mathbb{Z}$-module with respect to the scalar multiplication operation defined by: $\mathbb{Z} \times M \rightarrow M$, where $$(\lambda, m) \mapsto \lambda m,$$ where $$\lambda m = \begin{cases} \underbrace{m + m + ... + m}_{\lambda ~ \text{times}} & \text{if} ~ \lambda > 0 \\ \underbrace{(-m) + (-m) + ... + (-m)}_{-\lambda ~ \text{times}} & \text{if} ~ \lambda < 0 \\ 0 & \text{if} ~ \lambda = 0 \end{cases}.$$ First note that $\Omega$ is indeed a well-defined map from $\mathbb{Z}^{n}$ to $(\mathbb{Z}_{q})^{n}$. Let $x = (\lambda_{1}, \lambda_{2}, ..., \lambda_{n}), y = (\mu_{1}, \mu_{2}, ..., \mu_{n}) \in \mathbb{Z}^{n}$ and $\lambda \in \mathbb{Z}$. Then \begin{align*} \Omega(x +_{1} y) & = \Omega(\lambda_{1} + \mu_{1}, \lambda_{2} + \mu_{2}, ..., \lambda_{n} + \mu_{n}) \\ & = (\overline{\lambda_{1} + \mu_{1}}, \overline{\lambda_{2} + \mu_{2}}, ..., \overline{\lambda_{n} + \mu_{n}}), \end{align*} where for each $i \in \{1, 2, ..., n\}$, $\overline{\lambda_{i} + \mu_{i}} = (\lambda_{i} + \mu_{i}) (\text{mod} ~ q)$. Now, for $i \in \{1, 2, ..., n\}$, we have that \begin{align*} \overline{\lambda_{i} + \mu_{i}} & = (\lambda_{i} + \mu_{i}) (\text{mod} ~ q) \\ & = (\lambda_{i} (\text{mod} ~ q) + \mu_{i} (\text{mod} ~ n))(\text{mod} ~ q) \\ & = (\overline{\lambda_{i}} + \overline{\mu_{i}})(\text{mod} ~ q) \\ & = \overline{\lambda_{i}} \oplus_{q} \overline{\mu_{i}} \end{align*} So from above, we get that \begin{align*} \Omega(x +_{1} y) & = (\overline{\lambda_{1} + \mu_{1}}, \overline{\lambda_{2} + \mu_{2}}, ..., \overline{\lambda_{n} + \mu_{n}}) \\ & = (\overline{\lambda_{1}} \oplus_{q} \overline{\mu_{1}}, \overline{\lambda_{2}} \oplus_{q} \overline{\mu_{2}}, ..., \overline{\lambda_{n}} \oplus_{q} \overline{\mu_{n}}) \\ & = (\overline{\lambda_{1}}, \overline{\lambda_{2}}, ..., \overline{\lambda_{n}}) +_{2} (\overline{\mu_{1}}, \overline{\mu_{2}}, ..., \overline{\mu_{n}}) \\ & = \Omega(x) +_{2} \Omega(y) \end{align*} Now, first suppose that $\lambda > 0$. \begin{align*} \lambda \odot_{1} x & = \underbrace{x +_{1} x +_{1} ... +_{1} x}_{\lambda ~ \text{times}} \\ & = (\underbrace{\lambda_{1} + \lambda_{1} + ... + \lambda_{1}}_{\lambda ~ \text{times}}, \underbrace{\lambda_{2} + \lambda_{2} + ... + \lambda_{2}}_{\lambda ~ \text{times}}, ..., \underbrace{\lambda_{n} + \lambda_{n} + ... + \lambda_{n}}_{\lambda ~ \text{times}}) \\ & = (\lambda \lambda_{1}, \lambda \lambda_{2}, ..., \lambda \lambda_{n}) \end{align*} Now, \begin{align*} \Omega(\lambda \odot_{1} x) & = \Omega(\lambda \lambda_{1}, \lambda \lambda_{2}, ..., \lambda \lambda_{n}) \\ & = (\overline{\lambda \lambda_{1}}, \overline{\lambda \lambda_{2}}, ..., \overline{\lambda \lambda_{n}}) \end{align*} Now, for each $i \in \{1, 2, ..., n\}$, \begin{align*} \overline{\lambda \lambda_{i}} & = (\lambda \lambda_{i})(\text{mod} ~ q) \\ & = (((\lambda (\text{mod} ~ q)) (\lambda_{i} (\text{mod} ~ q))) (\text{mod} ~ q) \\ & = (\overline{\lambda} ~ \overline{\lambda_{i}}) (\text{mod} ~ q) \\ & = (\lambda \lambda_{i}) (\text{mod} ~ q). \end{align*} \begin{align*} \lambda \odot_{2} \Omega(x) & = \underbrace{\Omega(x) +_{2} \Omega(x) +_{2} ... +_{2} \Omega(x)}_{\lambda ~ \text{times}} \\ & = (\underbrace{\overline{\lambda_{1}} \oplus_{q} \overline{\lambda_{1}} \oplus_{q} ... \oplus_{q} \overline{\lambda_{1}}}_{\lambda ~ \text{times}}, \underbrace{\overline{\lambda_{2}} \oplus_{q} \overline{\lambda_{2}} \oplus_{q} ... \oplus_{q} \overline{\lambda_{2}}}_{\lambda ~ \text{times}}, ..., \underbrace{\overline{\lambda_{n}} \oplus_{q} \overline{\lambda_{n}} \oplus_{q} ... \oplus_{q} \overline{\lambda_{n}}}_{\lambda ~ \text{times}}) \end{align*} Now, let $i \in \{1, 2, ..., n\}$. Then we get that \begin{align*} \underbrace{\overline{\lambda_{i}} \oplus_{q} \overline{\lambda_{i}} \oplus_{q} ... \oplus_{q} \overline{\lambda_{i}}}_{\lambda ~ \text{times}} & = (\underbrace{\overline{\lambda_{i}} + \overline{\lambda_{i}} + ... + \overline{\lambda_{i}}}_{\lambda ~ \text{times}}) (\text{mod} ~ q) \\ & = (\underbrace{\lambda_{i} + \lambda_{i} + ... + \lambda_{i}}_{\lambda ~ \text{times}}) (\text{mod} ~ q) \\ & = (\lambda \lambda_{i}) (\text{mod} ~ q) \end{align*} So we have proved that $\Omega(\lambda \odot_{1} x) = \lambda \odot_{2} \Omega(x)$ for $\lambda > 0$. The above equation holds trivially for $\lambda = 0$. Now, suppose $\lambda < 0$. Then \begin{align*} \lambda \odot_{1} x & = \underbrace{(-x) +_{1} (-x) +_{1} ... +_{1} (-x)}_{- \lambda ~ \text{times}} \\ & = (\underbrace{(-\lambda_{1}) + (-\lambda_{1}) + ... + (-\lambda_{1})}_{- \lambda ~ \text{times}}, \underbrace{(- \lambda_{2}) + (- \lambda_{2}) + ... + (-\lambda_{2})}_{- \lambda ~ \text{times}}, ..., \\ &\quad \underbrace{(- \lambda_{n}) + (- \lambda_{n}) + ... + (- \lambda_{n})}_{- \lambda ~ \text{times}}) \\ & = ((- \lambda) (- \lambda_{1}), (- \lambda) (- \lambda_{2}), ..., (- \lambda) (- \lambda_{n})) \\ & = (\lambda \lambda_{1}, \lambda \lambda_{2}, ..., \lambda \lambda_{n}) \end{align*} Then as shown above, we will have $$\Omega(\lambda \odot_{1} x) = (\overline{\lambda \lambda_{1}}, \overline{\lambda \lambda_{2}}, ..., \overline{\lambda \lambda_{n}}),$$ where for each $i \in \{1, 2, ..., n\}$, $\overline{\lambda \lambda_{i}} = (\lambda \lambda_{i}) (\text{mod} ~ q)$. Using the fact that $\Omega$ is a group homomorphisms from the additive group $\mathbb{Z}^{n}$ to the additive group $(\mathbb{Z}_{q})^{n}$, we get that \begin{align*} \lambda \odot_{2} \Omega(x) & = \underbrace{(- \Omega(x)) +_{2} (- \Omega(x)) +_{2} ... +_{2} (- \Omega(x))}_{- \lambda ~ \text{times}} \\ & = \underbrace{\Omega(-x) +_{2} \Omega(-x) +_{2} ... +_{2} \Omega(-x)}_{- \lambda ~ \text{times}} \\ & = (\underbrace{\overline{- \lambda_{1}} \oplus_{q} \overline{- \lambda_{1}} \oplus_{q} ... \oplus_{q} \overline{- \lambda_{1}}}_{- \lambda ~ \text{times}}, \underbrace{\overline{- \lambda_{2}} \oplus_{q} \overline{- \lambda_{2}} \oplus_{q} ... \oplus_{q} \overline{- \lambda_{2}}}_{- \lambda ~ \text{times}}, ..., \\ & \quad \underbrace{\overline{- \lambda_{n}} \oplus_{q} \overline{- \lambda_{n}} \oplus_{q} ... \oplus_{q} \overline{- \lambda_{n}}}_{- \lambda ~ \text{times}}) \end{align*} Now, for $i \in \{1, 2, ..., n\}$, \begin{align*} \underbrace{\overline{- \lambda_{i}} \oplus_{q} \overline{- \lambda_{i}} \oplus_{q} ... \oplus_{q} \overline{- \lambda_{i}}}_{- \lambda ~ \text{times}} & = (\underbrace{(\overline{- \lambda_{i}}) + (\overline{- \lambda_{i}}) + ... + (\overline{- \lambda_{i}})}_{- \lambda ~ \text{times}}) (\text{mod} ~ q) \\ & = (\underbrace{(- \lambda_{i}) + (- \lambda_{i}) + ... + (- \lambda_{i})}_{- \lambda ~ \text{times}}) (\text{mod} ~ q) \\ & = ((-\lambda) (- \lambda_{i})) (\text{mod} ~ q) \\ & = (\lambda \lambda_{i}) (\text{mod} ~ q) \end{align*} So we have proved that $\Omega(\lambda \odot_{1} x) = \lambda \odot_{2} \Omega(x)$ for $\lambda < 0$. In all possible cases, we have proved that $\Omega(\lambda \odot_{1} x) = \lambda \odot_{2} \Omega(x)$ for all $\lambda \in \mathbb{Z}$. Therefore, $\Omega$ is a $\mathbb{Z}$-module homomorphism from the $\mathbb{Z}$-module $\mathbb{Z}^{n}$ to the $\mathbb{Z}$-module $(\mathbb{Z}_{q})^{n}$. Hence proved. \end{proof} \begin{lemma}\label{lemma 4.4} \begin{enumerate} \item[(i)] If $W$ is a $\mathbb{Z}$-submodule of $\mathbb{Z}^{n}$, then $\Omega(W)$ is a $\mathbb{Z}$-submodule of $(\mathbb{Z}_{q})^{n}$. \item[(ii)] If $V$ is a $\mathbb{Z}$-submodule of $(\mathbb{Z}_{q})^{n}$, then $\Omega^{-1}(V) = \{w \in \mathbb{Z}^{n} \mid \Omega(w) \in V\}$ is a $\mathbb{Z}$-submodule of $\mathbb{Z}^{n}$. \end{enumerate} \end{lemma} \begin{proof} \textbf{(i)} Let $W$ be a $\mathbb{Z}$-submodule of the $\mathbb{Z}$-module $\mathbb{Z}^{n}$. Since $\Omega: \mathbb{Z}^{n} \rightarrow (\mathbb{Z}_{q})^{n}$, therefore, $\Omega(W) \subseteq (\mathbb{Z}_{q})^{n}$. Also, $\Omega(W)$ is non-empty as $W$ being a $\mathbb{Z}$-submodule of $\mathbb{Z}^{n}$ is non-empty. Let $x, y \in \Omega(W)$ and $\lambda \in \mathbb{Z}$. Then $x = \Omega(w)$ and $y = \Omega(w')$ for some $w, w' \in W$. $x + y = \Omega(w) + \Omega(w') = \Omega(w + w')$ as $\Omega: \mathbb{Z}^{n} \rightarrow (\mathbb{Z}_{q})^{n}$ is a $\mathbb{Z}$-module homomorphism and $W \subseteq \mathbb{Z}^{n}$. As $w, w' \in W$ and $W$ is a $\mathbb{Z}$-submodule of $\mathbb{Z}^{n}$, therefore, $w + w' \in W$. This implies that $x+y = \Omega(w + w') \in \Omega(W)$. Further, since $\Omega: \mathbb{Z}^{n} \rightarrow (\mathbb{Z}_{q})^{n}$ is a $\mathbb{Z}$-module homomorphism, therefore, $\lambda x = \lambda \Omega(w) = \Omega(\lambda w)$. Since $\lambda \in \mathbb{Z}$, $w \in W$ and $W$ is a $\mathbb{Z}$-submodule of $\mathbb{Z}^{n}$, therefore, $\lambda w \in W$. This implies that $\lambda x = \Omega(\lambda w) \in \Omega(W)$. Therefore, $\Omega(W)$ is a $\mathbb{Z}$-submodule of $(\mathbb{Z}_{q})^{n}$. Hence proved.\vspace{16pt} \textbf{(ii)} Let $V$ be a $\mathbb{Z}$-submodule of the $\mathbb{Z}$-module $(\mathbb{Z}_{q})^{n}$. Clearly, $\Omega^{-1}(V)$ is a subset of $\mathbb{Z}^{n}$. Also, $\Omega^{-1}(V)$ is non-empty as $V$ being a $\mathbb{Z}$-submodule of $(\mathbb{Z}_{q})^{n}$ is non-empty (note that $0 \in V$ and $\Omega(0) = 0$). Let $w, w' \in \Omega^{-1}(V)$ and $\lambda \in \mathbb{Z}$. Then $w, w' \in \mathbb{Z}^{n}$ such that $\Omega(w), \Omega(w') \in V$. But then the fact that $V$ is a $\mathbb{Z}$-submodule of $(\mathbb{Z}_{q})^{n}$ implies that $\Omega(w) + \Omega(w') \in V$. Since by Lemma \ref{lemma 4.3}, $\Omega : \mathbb{Z}^{n} \rightarrow (\mathbb{Z}_{q})^{n}$ is a $\mathbb{Z}$-module homomorphism and $w, w' \in \mathbb{Z}^{n}$, so $\Omega(w + w') = \Omega(w) + \Omega(w')$. Therefore, from above, we get that $\Omega(w + w') \in V$. $\Rightarrow w + w' \in \Omega^{-1}(V)$. Also, since $\lambda \in \mathbb{Z}$, $\Omega(w) \in V$ and $V$ is a $\mathbb{Z}$-submodule of $(\mathbb{Z}_{q})^{n}$, therefore, $\lambda \Omega(w) \in V$. Since by Lemma \ref{lemma 4.3}, $\Omega : \mathbb{Z}^{n} \rightarrow (\mathbb{Z}_{q})^{n}$ is a $\mathbb{Z}$-module homomorphism, $w \in \mathbb{Z}^{n}$ and $\lambda \in \mathbb{Z}$, therefore, $\Omega(\lambda w) = \lambda \Omega(w)$. Therefore, from above, we get that $\Omega(\lambda w) \in V$. $\Rightarrow \lambda w \in \Omega^{-1}(V)$. Therefore, $\Omega^{-1}(V)$ is a $\mathbb{Z}$-submodule of $\mathbb{Z}^{n}$. Hence proved. \end{proof} \begin{lemma}\label{lemma 4.5} A subset $W$ is a $\mathbb{Z}$-submodule of $(\mathbb{Z}_{q})^{n}$ if and only if $W$ is a $\mathbb{Z}_{q}$-submodule of $(\mathbb{Z}_{q})^{n}$. \end{lemma} \begin{proof} Since $(\mathbb{Z}_{q})^{n}$ is a commutative unital ring with respect to the usual operations of componentwise addition module $q$ and multiplication module $q$, therefore, $(\mathbb{Z}_{q})^{n}$ is also a $\mathbb{Z}_{q}$-module with respect to the usual operation of componentwise scalar multiplication module $q$. Let $\odot_{2}$ denote the scalar multiplication of the $\mathbb{Z}$-module $(\mathbb{Z}_{q})^{n}$ and $\odot$ denote the scalar multiplication of the $\mathbb{Z}_{q}$-module $(\mathbb{Z}_{q})^{n}$. First, let $W$ be a $\mathbb{Z}$-submodule of $(\mathbb{Z}_{q})^{n}$. We show that $W$ is a $\mathbb{Z}_{q}$-submodule of $(\mathbb{Z}_{q})^{n}$. $W$ being a $\mathbb{Z}$-submodule of $(\mathbb{Z}_{q})^{n}$ is a non-empty subset of $(\mathbb{Z}_{q})^{n}$. Let $w, w' \in W$ and $\lambda \in \mathbb{Z}_{q}$. Suppose $w = (\lambda_{1}, \lambda_{2}, ..., \lambda_{n})$ for some $\lambda_{i} \in \mathbb{Z}_{q}$ ($1 \leq i \leq n$). Since $w, w' \in W$ and $W$ is a $\mathbb{Z}$-submodule of $(\mathbb{Z}_{q})^{n}$, therefore, $w+w' \in W$. $\lambda \in \mathbb{Z}_{q}$ implies that $\lambda \in \mathbb{Z}$. Now, \begin{align*} \lambda \odot_{2} w & = \underbrace{w +_{2} w +_{2} ... +_{2} w}_{\lambda ~ \text{times}} \\ & = (\underbrace{\lambda_{1} \oplus_{q} \lambda_{1} \oplus_{q} ... \oplus_{q} \lambda_{1}}_{\lambda ~ \text{times}}, \underbrace{\lambda_{2} \oplus_{q} \lambda_{2} \oplus_{q} ... \oplus_{q} \lambda_{2}}_{\lambda ~ \text{times}}, ..., \underbrace{\lambda_{n} \oplus_{q} \lambda_{n} \oplus_{q} ... \oplus_{q} \lambda_{n}}_{\lambda ~ \text{times}}) \end{align*} Also, \begin{align*} \lambda \odot w & = (\lambda \otimes_{q} \lambda_{1}, \lambda \otimes_{q} \lambda_{2}, ..., \lambda \otimes_{q} \lambda_{n}) \\ \end{align*} \textbf{Claim: For each $i \in \{1, 2, ..., n\}$, $\underbrace{\lambda_{i} \oplus_{q} \lambda_{i} \oplus_{q} ... \oplus_{q} \lambda_{i}}_{\lambda ~ \text{times}} = \lambda \otimes_{q} \lambda_{1}$.} So let $i \in \{1, 2, ..., n\}$. Then we get \begin{align*} \underbrace{\lambda_{i} \oplus_{q} \lambda_{i} \oplus_{q} ... \oplus_{q} \lambda_{i}}_{\lambda ~ \text{times}} & = (\underbrace{\lambda_{i} + \lambda_{i} + ... + \lambda_{i}}_{\lambda ~ \text{times}}) (\text{mod} ~ q) \\ & = (\lambda \lambda_{i}) (\text{mod} ~ q) \\ & = \lambda \otimes_{q} \lambda_{i} \end{align*} So we have proved that $\lambda \odot_{2} w = \lambda \odot w$. Since $W$ is a $\mathbb{Z}$-submodule, therefore, $\lambda \odot_{2} w \in W$. Further, since $\lambda \odot_{2} w = \lambda \odot w$, therefore, $\lambda \odot w \in W$. Since $w, w' \in W$ and $\lambda \in \mathbb{Z}_{q}$ are arbitrary, therefore, $w+w', \lambda \odot w \in W$ for all $w, w' \in W$ and $\lambda \in \mathbb{Z}_{q}$. Therefore, $W$ is also a $\mathbb{Z}_{q}$-submodule of $(\mathbb{Z}_{q})^{n}$. Conversely, let $W$ be a $\mathbb{Z}_{q}$-submodule of $(\mathbb{Z}_{q})^{n}$. We show that $W$ is also a $\mathbb{Z}$-submodule of $(\mathbb{Z}_{q})^{n}$. Being a $\mathbb{Z}_{q}$-submodule of $(\mathbb{Z}_{q})^{n}$, $W$ is a non-empty subset of $(\mathbb{Z}_{q})^{n}$. Let $w, w' \in W$ and $\lambda \in \mathbb{Z}$. Since $W$ is a $\mathbb{Z}_{q}$-submodule of $(\mathbb{Z}_{q})^{n}$, therefore, $w + w' \in W$. Since $w \in W \subseteq (\mathbb{Z}_{q})^{n}$, so $w = (\lambda_{1}, \lambda_{2}, ..., \lambda_{n})$ for some $\lambda_{i} \in \mathbb{Z}_{q}$ ($1 \leq i \leq n$). Now, if $\lambda > 0$, then \begin{align*} \lambda \odot_{2} w & = \underbrace{w +_{2} w +_{2} ... +_{2} w}_{\lambda ~ \text{times}} \\ & = (\underbrace{\lambda_{1} \oplus_{q} \lambda_{1} \oplus_{q} ... \oplus_{q} \lambda_{1}}_{\lambda ~ \text{times}}, \underbrace{\lambda_{2} \oplus_{q} \lambda_{2} \oplus_{q} ... \oplus_{q} \lambda_{2}}_{\lambda ~ \text{times}}, ..., \underbrace{\lambda_{n} \oplus_{q} \lambda_{n} \oplus_{q} ... \oplus_{q} \lambda_{n}}_{\lambda ~ \text{times}}). \end{align*} $\overline{\lambda} = \lambda (\text{mod} ~ q)$. Then \begin{align*} \overline{\lambda} \odot w & = (\overline{\lambda} \otimes_{q} \lambda_{1}, \overline{\lambda} \otimes_{q} \lambda_{2}, ..., \overline{\lambda} \otimes_{q} \lambda_{n}). \end{align*} \textbf{Claim: For each $i \in \{1, 2, ..., n\}$, $\underbrace{\lambda_{i} \oplus_{q} \lambda_{i} \oplus_{q} ... \oplus_{q} \lambda_{i}}_{\lambda ~ \text{times}} = \overline{\lambda} \otimes_{q} \lambda_{i}$.} We get \begin{align*} \underbrace{\lambda_{i} \oplus_{q} \lambda_{i} \oplus_{q} ... \oplus_{q} \lambda_{i}}_{\lambda ~ \text{times}} & = (\underbrace{\lambda_{i} + \lambda_{i} + ... + \lambda_{i}}_{\lambda ~ \text{times}}) (\text{mod} ~ q) \\ & = (\lambda \lambda_{i}) (\text{mod} ~ q) \\ & = (\overline{\lambda} \overline{\lambda_{i}}) (\text{mod} ~ q) \\ & = \overline{\lambda} \otimes_{q} \lambda_{i} \end{align*} because $\overline{\lambda_{i}} = \lambda_{i}$ as $\lambda_{i} \in \mathbb{Z}_{q}$. So we have proved that for $\lambda > 0$, $\lambda \odot_{2} w = \overline{\lambda} \odot w$. For $\lambda = 0$, the result holds trivially. If $\lambda < 0$, then \begin{align*} \lambda \odot_{2} w & = \underbrace{(-w) +_{2} (-w) +_{2} ... +_{2} (-w)}_{- \lambda ~ \text{times}} \\ & = (\underbrace{(- \lambda_{1}) \oplus_{q} (- \lambda_{1}) \oplus_{q} ... \oplus_{q} (- \lambda_{1})}_{- \lambda ~ \text{times}}, \underbrace{(- \lambda_{2}) \oplus_{q} (- \lambda_{2}) \oplus_{q} ... \oplus_{q} (- \lambda_{2})}_{- \lambda ~ \text{times}}, ..., \\ &\quad \underbrace{(- \lambda_{n}) \oplus_{q} (- \lambda_{n}) \oplus_{q} ... \oplus_{q} (- \lambda_{n})}_{- \lambda ~ \text{times}}) \end{align*} \textbf{Claim: For each $i \in \{1, 2, ..., n\}$, $\underbrace{(- \lambda_{i}) \oplus_{q} (- \lambda_{i}) \oplus_{q} ... \oplus_{q} (- \lambda_{i})}_{- \lambda ~ \text{times}} = \overline{\lambda} \otimes_{q} \lambda_{i}$.} We get \begin{align*} \underbrace{(- \lambda_{i}) \oplus_{q} (- \lambda_{i}) \oplus_{q} ... \oplus_{q} (- \lambda_{i})}_{- \lambda ~ \text{times}} & = (\underbrace{(- \lambda_{i}) + (- \lambda_{i}) + ... + (- \lambda_{i})}_{- \lambda ~ \text{times}}) (\text{mod} ~ q) \\ & = ((- \lambda) (- \lambda_{i})) (\text{mod} ~ q) \\ & = (\lambda \lambda_{i}) (\text{mod} ~ q) \\ & = (\overline{\lambda} \overline{\lambda_{i}}) (\text{mod} ~ q) \\ & = \overline{\lambda} \otimes_{q} \lambda_{i} \end{align*} because $\overline{\lambda_{i}} = \lambda_{i}$ as $\lambda_{i} \in \mathbb{Z}_{q}$. So we have proved that for $\lambda < 0$, $\lambda \odot_{2} w = \overline{\lambda} \odot w$. Therefore, for any $\lambda$, $\lambda \odot_{2} w = \overline{\lambda} \odot w$. But since $\overline{\lambda} \in \mathbb{Z}_{q}$, $w \in W$ and $W$ is a $\mathbb{Z}_{q}$-submodule of $(\mathbb{Z}_{q})^{n}$, therefore, $\overline{\lambda} \odot w \in W$. And then the fact that $\lambda \odot_{2} w = \overline{\lambda} \odot w$ implies that $\lambda \odot_{2} w \in W$. Therefore, $W$ is also a $\mathbb{Z}$-submodule of $(\mathbb{Z}_{q})^{n}$. \end{proof} Suppose $W$ is an IDD code and $\mathcal{E}$ is its corresponding equivalent IDD code in $\mathbb{Z}^{n}$. Then $\mathcal{E}$ is a $\mathbb{Z}$-submodule of the $\mathbb{Z}$-module $(\mathbb{Z})^{n}$. By Lemma \ref{lemma 4.4} (i), $\Omega(\mathcal{E})$ is a $\mathbb{Z}$-submodule of $(\mathbb{Z}_{q})^{n}$. Then by Lemma \ref{lemma 4.5}, $\Omega(\mathcal{E})$ is a $\mathbb{Z}_{q}$-submodule of $(\mathbb{Z}_{q})^{n}$. Hence, $\Omega(\mathcal{E})$ is a code over $\mathbb{Z}_{q}$ and in $(\mathbb{Z}_{q})^{n}$. So we have the following definition. \begin{definition} Let $q$ be a positive integer. With the notations of Subsection \ref{subsection 4.1}, let $W$ be an IDD code and $\mathcal{E}$ be its equivalent IDD code in $\mathbb{Z}^{n}$. Then the $\mathbb{Z}_{q}$-submodule $\Omega(\mathcal{E})$ of $(\mathbb{Z}_{q})^{n}$ is called Hom-IDD code in $(\mathbb{Z}_{q})^{n}$. \end{definition} \subsection{Example}\label{subsection 4.4} \begin{example} Let $K = \mathbb{Q}(\zeta)$, $\zeta$ primitive $p^{\text{th}}$-root of unity, where $p=17$. Let $\sigma(\zeta) = \zeta$ and $\tau(\zeta) = \zeta^{3}$. Then by Theorem \ref{theorem 3.8}, the map $D:O_{K} \rightarrow O_{K}$ given by $$D(\zeta) = 1 + \zeta + \zeta^{2} + \zeta^{3} + \zeta^{5} + \zeta^{7} + \zeta^{8} + \zeta^{11}$$ is a $(\sigma, \tau)$-derivation of $O_{K} = \mathbb{Z}[\zeta]$. Below, we have obtained (binary) Hom-IDD codes of length $n=16$ over the finite field $\mathbb{Z}_{2}$ with various parameters, using the construction discussed above. \begin{center} \begin{tabular}{|c|c|c|c|} \hline \textbf{Basis $S_{i}$} & \thead{\textbf{Code Description:}\\ $[n,k,d]$} & \textbf{Code Properties} & \thead{\textbf{Dual Code} \\ \textbf{Description:} \\ $[n,k,d]$} \\ \hline \hline $S_{1}$ & $[16,8,4]$ & LCD & $[16,8,4]$ \\ \hline $S_{2}$ & $[16,8,4]$ & non-LCD & $[16,8,4]$ \\ \hline $S_{3}$ & $[16,7,4]$ & LCD & $[16,9,3]$ \\ \hline $S_{4}$ & $[16,9,3]$ & non-LCD & $[16,7,4]$ \\ \hline $S_{5}$ & $[16,7,4]$ & non-LCD & $[16,9,3]$ \\ \hline $S_{6}$ & $[16,6,4]$ & non-LCD & $[16,10,3]$ \\ \hline $S_{7}$ & $[16,5,5]$ & non-LCD & $[16,11,2]$ \\ \hline $S_{8}$ & $[16,4,7]$ & non-LCD & $[16,12,2]$ \\ \hline $S_{9}$ & $[16,5,5]$ & LCD & $[16,11,2]$ \\ \hline $S_{10}$ & $[16,8,3]$ & non-LCD & $[16,8,4]$ \\ \hline $S_{11}$ & $[16,9,3]$ & LCD & $[16,7,4]$ \\ \hline $S_{12}$ & $[16,8,4]$ & LCD & $[16,8,3]$ \\ \hline $S_{13}$ & $[16,6,5]$ & LCD & $[16,10,3]$ \\ \hline \end{tabular} \end{center} \begin{center} \begin{tabular}{|c|} \hline \textbf{Basis $S_{i}$} \\ \hline \hline $S_{1} = \{D(\zeta), D(\zeta^{2}), D(\zeta^{4}), D(\zeta^{5}), D(\zeta^{6}), D(\zeta^{7}), D(\zeta^{10}), D(\zeta^{13})\}$ \\ \hline $S_{2} = \{D(\zeta), D(\zeta^{2}), D(\zeta^{4}), D(\zeta^{5}), D(\zeta^{6}), D(\zeta^{7}), D(\zeta^{9}), D(\zeta^{12})\}$ \\ \hline $S_{3} = \{D(\zeta), D(\zeta^{2}), D(\zeta^{5}), D(\zeta^{6}), D(\zeta^{7}), D(\zeta^{9}), D(\zeta^{12})\}$ \\ \hline $S_{4} = \{D(\zeta), D(\zeta^{2}), D(\zeta^{5}), D(\zeta^{6}), D(\zeta^{7}), D(\zeta^{9}), D(\zeta^{10}), D(\zeta^{12}), D(\zeta^{14})\}$ \\ \hline $S_{5} = \{D(\zeta), D(\zeta^{2}), D(\zeta^{6}), D(\zeta^{7}), D(\zeta^{9}), D(\zeta^{12}), D(\zeta^{14})\}$ \\ \hline $S_{6} = \{D(\zeta), D(\zeta^{2}), D(\zeta^{6}), D(\zeta^{7}), D(\zeta^{9}), D(\zeta^{12})\}$ \\ \hline $S_{7} = \{D(\zeta), D(\zeta^{2}), D(\zeta^{5}), D(\zeta^{6}), D(\zeta^{7})\}$ \\ \hline $S_{8} = \{D(\zeta), D(\zeta^{2}), D(\zeta^{6}), D(\zeta^{7})\}$ \\ \hline $S_{9} = \{D(\zeta), D(\zeta^{2}), D(\zeta^{6}), D(\zeta^{7}), D(\zeta^{15})\}$ \\ \hline $S_{10} = \{D(\zeta), D(\zeta^{2}), D(\zeta^{6}), D(\zeta^{7}), D(\zeta^{10}), D(\zeta^{13}), D(\zeta^{14}), D(\zeta^{15})\}$ \\ \hline $S_{11} = \{D(\zeta), D(\zeta^{2}), D(\zeta^{4}), D(\zeta^{5}), D(\zeta^{6}), D(\zeta^{10}), D(\zeta^{13}), D(\zeta^{14}), D(\zeta^{15})\}$ \\ \hline $S_{12} = \{D(\zeta), D(\zeta^{4}), D(\zeta^{5}), D(\zeta^{6}), D(\zeta^{10}), D(\zeta^{13}), D(\zeta^{14}), D(\zeta^{15})\}$ \\ \hline $S_{13} = \{D(\zeta), D(\zeta^{4}), D(\zeta^{5}), D(\zeta^{6}), D(\zeta^{10}), D(\zeta^{13})\}$ \\ \hline \end{tabular} \end{center} \end{example} \section{Conclusion}\label{section 5} In this article, we studied the $(\sigma, \tau)$-derivations of number rings and their applications to coding theory. First, in Section \ref{section 2}, we obtained some useful results for the forthcoming sections. In Section \ref{section 3}, we studied $(\sigma, \tau)$-derivations of the rings of algebraic integers of quadratic and cyclotomic number fields, and that of the bi-quadratic number ring. In quadratic case, we reformulated and reinvented the results formed in \cite{Chaudhuri}. We obtained a characterization in Subsection \ref{subsection 3.2} for a $\mathbb{Z}$-linear map vanishing on unity to be a $(\sigma, \tau)$-derivation of the ring of algebraic integers of a $p^{\text{th}}$-cyclotomic number field. We also conjectured a criterion under which a $(\sigma, \tau)$-derivation of the ring of algebraic integers of a $p^{\text{th}}$ cyclotomic number field is inner. In Subsection \ref{subsection 3.3}, we explicitly obtained all $(\sigma, \tau)$-derivations of a bi-quadratic number ring. We also obtained a criterion under which a $(\sigma, \tau)$-derivation of a bi-quadratic number ring is inner. Finally, in Section \ref{section 4}, we discussed the applications of our work in coding theory by giving the notion of Hom-IDD codes. \vspace{20pt} \noindent \textbf{Declaration of Interest Statement}\vspace{8pt} \noindent The authors report there are no competing interests to declare. \begin{thebibliography}{10} \bibitem{AleksandrAlekseev2020} Aleksandr Alekseev, Andronick Arutyunov, and Sergei Silvestrov. \newblock On $(\sigma, \tau)$-derivations of group algebra as category characters. \newblock {\em arXiv preprint arXiv:2008.00390}, 2020. \bibitem{Atteya2019} Mehsin~Jabel Atteya. \newblock New types of permuting n-derivations with their applications on associative rings. \newblock {\em Symmetry}, 12(1):46, 2019. \bibitem{Brear1992} M.~Bresar. \newblock On the composition of $(\alpha, \beta)$-derivations of rings and applications to von {N}eumann algebras. \newblock {\em Acta Sct. Math}, 56:369--375, 1992. \bibitem{Chaudhry2011} Muhammad~Anwar Chaudhry and Zafar Ullah. \newblock On generalized ($\alpha$, $\beta$)-derivations on lattices. \newblock {\em Quaestiones Mathematicae}, 34(4):417--424, 2011. \bibitem{Chaudhuri} Dishari Chaudhuri. \newblock A note on $(\sigma, \tau)$-derivations on commutative algebras. \newblock {\em arXiv preprint arXiv:1912.12812}, 2019. \bibitem{Creedon2019} Leo Creedon and Kieran Hughes. \newblock Derivations on group algebras with coding theory applications. \newblock {\em Finite Fields and Their Applications}, 56:247--265, 2019. \bibitem{Boucher2014} Boucher Delphine and Felix Ulmer. \newblock Linear codes using skew polynomials with automorphisms and derivations. \newblock {\em Designs, codes and cryptography}, 70:405--431, 2014. \bibitem{Haetinger2011} M.~Haetinger, C.~Ashraf and S.~Ali. \newblock On higher derivations: a survey. \newblock {\em Int. J. Math., Game Theory and Algebra}, 18:359--379, 2011. \bibitem{Jacobson1964} N.~Jacobson. \newblock {\em Structure of Rings}. \newblock CP37. American Mathematical Society, 1964. \bibitem{KamaliArdakani2013} L.~Kamali~Ardakani and Bijan Davvaz. \newblock f-derivations and (f,g)-derivations of {MV}-algebras. \newblock {\em Journal of Algebraic Systems}, 1(1):11--31, 2013. \bibitem{Kim2014} Kyung~Ho Kim. \newblock On (f,g)-derivations of incline algebras. \newblock {\em Journal of the Chungcheong Mathematical Society}, 27(4):643--649, 2014. \bibitem{Klimek2021} Slawomir Klimek and Matt McBride. \newblock Unbounded derivations in algebras associated with monothetic groups. \newblock {\em Journal of the Australian Mathematical Society}, 111(3):345--371, 2021. \bibitem{Marcus2018} Daniel~A. Marcus. \newblock {\em Number fields}. \newblock Universitext. Springer, 2nd edition, 2018. \bibitem{MohammadAshraf2006} Ashraf Mohammad, Ali Shakir, and Haetinger Claus. \newblock On derivations in rings and their applications. \newblock {\em The Aligarh Bull of Math}, 25(2):79--107, 2006. \bibitem{Muhiuddin2012} G~Muhiuddin and Abdullah~M Al-Roqi. \newblock On $(\alpha, \beta)$-derivations in {BCI}-algebras. \newblock {\em Discrete Dynamics in Nature and Society}, 2012:11, 2012. \bibitem{Mustafa2013} A{\c{S}}CI Mustafa and {\c{S}}ahin Ceran. \newblock Generalized $(f,g)$-derivations of lattices. \newblock {\em Mathematical Sciences and Applications E-Notes}, 1(2):56--62, 2013. \bibitem{Raza2016} Mohd~Arif Raza and Nadeem~Ur Rehman. \newblock On prime and semiprime rings with generalized derivations and non-commutative {B}anach algebras. \newblock {\em Proceedings-Mathematical Sciences}, 126(3):389--398, 2016. \bibitem{IanStewart2002} Ian Stewart and David Tall. \newblock {\em Algebraic Number Theory and Fermat's Last Theorem}. \newblock A. K. Peters, 3rd edition, 2002. \end{thebibliography} \bibliographystyle{plain} \end{document}
2412.03475v1
http://arxiv.org/abs/2412.03475v1
On the long-time limit of the mean curvature flow in closed manifolds
\documentclass[11pt, oneside,reqno]{amsart} \usepackage{geometry} \usepackage{float} \allowdisplaybreaks \geometry{letterpaper} \usepackage{parskip} \usepackage{graphicx} \usepackage{amssymb} \usepackage{amsmath} \usepackage[all]{xy} \usepackage{mathtools} \usepackage{tikz-cd} \usepackage{enumitem} \usepackage{tikz-cd} \usepackage{appendix} \usepackage[T1]{fontenc} \usepackage{esint} \headheight=6.15pt \textheight=8in \textwidth=6.5in \oddsidemargin=0in \evensidemargin=0in \topmargin=0in \setlength{\parskip}{6pt} \usepackage[bookmarks=true, colorlinks, linkcolor=blue, urlcolor=blue, citecolor=blue, plainpages=false, pdfpagelabels, final, breaklinks=true\between ]{hyperref} \usepackage{hyperref} \usepackage[numbers,sort&compress]{natbib} \renewcommand{\bf}{\textbf} \renewcommand{\sc}{\textsc} \newcommand{\intprod}{\mbox{\rule{5pt}{.6pt}\rule{.6pt}{6pt}}~} \newtheorem{thm}{Theorem}[section] \newtheorem{lem}[thm]{Lemma} \newtheorem{cor}[thm]{Corollary} \newtheorem{prop}[thm]{Proposition} \newtheorem{conj}[thm]{Conjecture} \newtheorem{observation}[thm]{Observation} \newtheorem{Question}[thm]{Question} \newtheorem{rem}[thm]{Remark} \newtheorem{exmp}[thm]{Example} \newtheorem{defn}[thm]{Definition} \numberwithin{equation}{section} \newcommand{\Balpha}{\mbox{$\hspace{0.12em}\shortmid\hspace{-0.62em}\alpha$}} \renewcommand{\a}{\alpha} \renewcommand{\b}{\beta} \renewcommand{\d}{\delta} \newcommand{\D}{\Delta} \newcommand{\e}{\varepsilon} \newcommand{\ga}{\gamma} \newcommand{\G}{\Gamma} \renewcommand{\l}{\lambda} \newcommand{\n}{\nabla} \newcommand{\p}{\partial} \newcommand{\var}{\varphi} \newcommand{\s}{\sigma} \newcommand{\Sig}{\Sigma} \renewcommand{\t}{\tau} \newcommand{\x}{\xi} \newcommand{\z}{\zeta} \newcommand{\im}{\imath} \newcommand{\jm}{\jmath} \def\mathR{\mbox{$\bf \rm I\makebox [1.2ex] [r] {R}$}} } } } } } \def\dS{\mbox{div}_{\Sig}} \def\wS{\widetilde{\Sig}} \def\nS{\nabla^{\Sig}} \def\iD{\int_{D_{2 \s_0}}} \def\B{\cal B} \def\P{\cal P} \def\Ca{\cal C} \def\div{\operatorname{div}} \def\gr{\operatorname{graph}} \def\lip{\operatorname{Lip}} \def\mass{\operatorname{mass}} \def\supp{\operatorname{supp}} \def\dist{\operatorname{dist}} \def\J{\operatorname{Jac}} \def\grad{\operatorname{grad}} \def\diam{\operatorname{diam}} \def\Area{\operatorname{Area}} \def\per{\operatorname{period}} \def\len{\operatorname{length}} \def\Ric{\operatorname{Ric}} \def\Hess{\operatorname{Hess}} \def\exp{\operatorname{exp}} \def\e{\mathrm{ext}} \def\i{\mathrm{int}} \def\M{\cal M} \def\S{\cal S} \newcommand{\ov}{\overline} \newcommand{\pd}{\partial} \newcommand{\red}[1]{\textcolor{red}{#1}} \newcommand{\dv}{\text{div}} \newcommand{\ee}{{\text{e}}} \newcommand{\hps}{{\text{hypersurface}}} \newcommand{\sk}{{\text{shrinker}}} \newcommand{\sks}{{\text{shrinkers}}} \newcommand{\mf}{\mathbf} \newcommand{\mc}{\mathcal} \newcommand{\mb}{\mathbb} \newcommand{\mfk}{\mathfrak} \newcommand{\oli}{\overline} \newcommand{\eps}{\varepsilon} \newcommand{\wti}{\widetilde} \newcommand{\dt}{\delta} \newcommand{\al}{\alpha} \newcommand{\bn}{\mathbf{n}} \newcommand{\br}{\mathbf{r}} \newcommand{\bN}{\mathbf{N}} \newcommand{\bM}{\mathbf{M}} \newcommand{\bS}{\mathbb{S}} \newcommand{\bT}{\mathbf{T}} \newcommand{\AAa}{\mathbb{A}} \newcommand{\BB}{\mathbb{B}} \newcommand{\CC}{\mathbb{C}} \newcommand{\DD}{\mathbb{D}} \newcommand{\EE}{\mathbb{E}} \newcommand{\FF}{\mathbb{F}} \newcommand{\GG}{\mathbb{G}} \newcommand{\HH}{\mathbb{H}} \newcommand{\II}{\mathbb{I}} \newcommand{\JJ}{\mathbb{J}} \newcommand{\KK}{\mathbb{K}} \newcommand{\LL}{\mathbb{L}} \newcommand{\MM}{\mathbb{M}} \newcommand{\NN}{\mathbb{N}} \newcommand{\OO}{\mathbb{O}} \newcommand{\PP}{\mathbb{P}} \newcommand{\QQ}{\mathbb{Q}} \newcommand{\RR}{\mathbb{R}} \newcommand{\SSp}{\mathbb{S}} \newcommand{\TT}{\mathbb{T}} \newcommand{\UU}{\mathbb{U}} \newcommand{\VV}{\mathbb{V}} \newcommand{\WW}{\mathbb{W}} \newcommand{\XX}{\mathbb{X}} \newcommand{\YY}{\mathbb{Y}} \newcommand{\ZZ}{\mathbb{Z}} \newcommand{\cA}{\mathcal{A}} \newcommand{\cB}{\mathcal{B}} \newcommand{\cC}{\mathcal{C}} \newcommand{\cD}{\mathcal{D}} \newcommand{\cE}{\mathcal{E}} \newcommand{\cF}{\mathcal{F}} \newcommand{\cG}{\mathcal{G}} \newcommand{\cH}{\mathcal{H}} \newcommand{\cI}{\mathcal{I}} \newcommand{\cJ}{\mathcal{J}} \newcommand{\cK}{\mathcal{K}} \newcommand{\cL}{\mathcal{L}} \newcommand{\cM}{\mathcal{M}} \newcommand{\cN}{\mathcal{N}} \newcommand{\cO}{\mathcal{O}} \newcommand{\cP}{\mathcal{P}} \newcommand{\cQ}{\mathcal{Q}} \newcommand{\cR}{\mathcal{R}} \newcommand{\cS}{\mathcal{S}} \newcommand{\cT}{\mathcal{T}} \newcommand{\cU}{\mathcal{U}} \newcommand{\cV}{\mathcal{V}} \newcommand{\cW}{\mathcal{W}} \newcommand{\cX}{\mathcal{X}} \newcommand{\cY}{\mathcal{Y}} \newcommand{\cZ}{\mathcal{Z}} \newcommand{\mbfA}{\mathbf{A}} \newcommand{\mbfB}{\mathbf{B}} \newcommand{\mbfC}{\mathbf{C}} \newcommand{\mbfD}{\mathbf{D}} \newcommand{\mbfE}{\mathbf{E}} \newcommand{\mbfF}{\mathbf{F}} \newcommand{\mbfG}{\mathbf{G}} \newcommand{\mbfH}{\mathbf{H}} \newcommand{\mbfI}{\mathbf{I}} \newcommand{\mbfJ}{\mathbf{J}} \newcommand{\mbfK}{\mathbf{K}} \newcommand{\mbfL}{\mathbf{L}} \newcommand{\mbfM}{\mathbf{M}} \newcommand{\mbfN}{\mathbf{N}} \newcommand{\mbfO}{\mathbf{O}} \newcommand{\mbfP}{\mathbf{P}} \newcommand{\mbfQ}{\mathbf{Q}} \newcommand{\mbfR}{\mathbf{R}} \newcommand{\mbfS}{\mathbf{S}} \newcommand{\mbfT}{\mathbf{T}} \newcommand{\mbfU}{\mathbf{U}} \newcommand{\mbfV}{\mathbf{V}} \newcommand{\mbfW}{\mathbf{W}} \newcommand{\mbfX}{\mathbf{X}} \newcommand{\mbfY}{\mathbf{Y}} \newcommand{\mbfZ}{\mathbf{Z}} \newcommand{\bH}{\mathbf{H}} \newcommand{\sR}{\mathscr{R}} \newcommand{\sL}{\mathscr{L}} \newcommand{\sT}{\mathscr{T}} \newcommand{\sH}{\mathscr{H}} \newcommand{\Si}{\Sigma} \newcommand{\la}{\lambda} \newcommand{\ind}{\mathrm{ind}} \newcommand{\A}{\mathbb{A}} \newcommand{\pr}{\partial} \newcommand{\mbfd}{\mathbf{d}} \newcommand{\mbfx}{\mathbf{x}} \newcommand{\mbfy}{\mathbf{y}} \newcommand{\Euc}{\mathrm{euc}} \newcommand{\orig}{\mathbf{0}} \newcommand{\Div}{\operatorname{div}} \newcommand{\spine}{\operatorname{spine}} \newcommand{\odist}{\overline{\dist}} \DeclareMathOperator{\vol}{Vol} \DeclareMathOperator{\area}{Area} \DeclareMathOperator{\Index}{index} \DeclareMathOperator{\genus}{genus} \DeclareMathOperator{\Clos}{Clos} \DeclareMathOperator{\spt}{spt} \DeclareMathOperator{\Graph}{Graph} \DeclareMathOperator{\rad}{radius} \DeclareMathOperator{\ang}{angle} \DeclareMathOperator{\loc}{loc} \DeclareMathOperator{\tr}{tr} \DeclareMathOperator{\id}{\text{Id}} \DeclareMathOperator{\Ent}{\text{Ent}} \DeclareMathOperator{\Sing}{\text{Sing}} \DeclareMathOperator{\rot}{\text{rot}} \newenvironment{pf}{\paragraph{Proof:}}{\hfill$\square$ \newline} \linespread{1.1} \begin{document} \title[Fate of flow]{On the long-time limit of the mean curvature flow in closed manifolds} \begin{abstract} In this article we show that generally almost regular flows, introduced by Bamler and Kleiner, in closed $3$--manifolds will either go extinct in finite time or flow to a collection of smooth embedded minimal surfaces, possibly with multiplicity. Using a perturbative argument then we construct piecewise almost regular flows which either go extinct in finite time or flow to a stable minimal surface, possibly with multiplicity. We apply these results to construct minimal surfaces in $3$--manifolds in a variety of circumstances, mainly novel from the point of the view that the arguments are via parabolic methods. \end{abstract} \author{Alexander Mramor and Ao Sun} \address{Department of Mathematics, University of Copenhagen, Universitetsparken 5, DK-2100 Copenhagen Ø, Denmark 2300 \newline \newline \indent Department of Mathematics, Lehigh University, Chandler--Ullmann Hall, Bethlehem, PA 18015 \newline} \email{[email protected], [email protected]} \maketitle \section{Introduction} The mean curvature flow, as the gradient flow of the area functional, is a natural tool to find minimal surfaces -- in particular area minimizing and stable ones. The $1$-dimensional mean curvature flow in surfaces, known as the curve shortening flow, has been proven to be a very powerful tool for constructing geodesics. An especially important accomplishment was by Grayson \cite{Grayson89_CSF}, who used the curve shortening flow to settle Lusternik and Schnirelman's proposal to construct $3$ distinct closed embedded geodesics in $(S^2,g)$. A central obstruction to using $n$--dimensional mean curvature flow in $(n+1)$--dimensional ambient manifold to construct minimal surfaces is the more complicated nature of singularity formation in dimensions $n \geq 2$, and weak notions of the flow are necessary. There are a number of reasonable definitions of weak flows to handle this issue; these are discussed in section \ref{prelim}. Very recently, Bamler and Kleiner \cite{BamlerKleiner23_Multiplicity1} introduced the notion of almost regular flow, which are well-behaved Brakke flows whose so--called local scale functions satisfy a certain integrability condition. They proved that in $\R^3$ all generic flows (i.e. flows which only develop mean convex singularities) and hence, by an approximation argument, inner and outermost flows emanating from an embedded closed surface are almost regular. In this framework, they then proved the multiplicity one conjecture of Ilmanen \cite{Ilmanen95_Sing2D}. In this article we use the notions of generic and almost regular flows in general $3$--manifolds and first show the following: \begin{thm}\label{longterm}\label{smoothfate} Let $M$ be a $2$-sided properly embedded surface in a closed Riemannian $3$--manifold $(N,g)$, and let $M_t$ be an almost regular flow from $M$. Then either $M_t$ goes extinct in finite time or there exists a sequence $t_i \to \infty$ for which $M_{t_i}$ converges to $m_1\Sigma_1+\cdots+m_k\Sigma_k$ in the sense of varifolds, where each $\Sigma_j$ is a smoothly embedded minimal surface and each $m_j$ is a positive integer. Moreover, all the $\Sigma_j$'s are disjoint. \end{thm} We point out that some of the $\Sigma_j$ may be nonorientable, even if the initial data is $2$--sided: consider for instance a situation where the boundary of the tubular neighborhood of an embedded projective plane $P$ (say in the 3 manifold $N = \R P^3$), homeomorphic to $S^2$, flows back to $P$ as $t \to \infty$. This can also be interpreted as saying that the topology of the flow may actually "increase" in the limit; we discuss more precisely to what extent the topology of the $\Sigma_i$ can be controlled in terms of the initial data $M$ in Proposition \ref{limit_topology} below. We note that apriori the limit in Theorem \ref{longterm} may not be unique; if the ambient manifold is analytic and the convergence is with multiplicity one we may appeal to the \L{}ojasiewicz-Simon inequality to see this is the case, but we assume/show neither in the statement above. However, we are able to show the limit stable minimal surface is unique whenever it is strictly stable. Recall below that a stable minimal surface is strictly stable if the first eigenvalue of the linearized operator is positive, and as pointed out for example in \cite{Urbano13_one-sided} this notion continues to make sense even for $1$-sided minimal surfaces: \begin{thm}\label{longterm_uniqueness} In the setting of Theorem \ref{longterm}, if the limit stable minimal surfaces are strictly stable, then any varifold converging limit of $M_{t}$ converges to this limit with the same multiplicities as $t\to\infty$. \end{thm} See Theorem \ref{longterm_uniqueness1} below. We would like to point out that the uniqueness of limit in geometric measure theory, in general, is quite hard when the multiplicity of convergence is greater than one. Next, we further study what the possible limit minimal surfaces can be. Because the flow is the gradient of the area functional one expects to typically find area minimizing or at least stable minimal surfaces using it but nevertheless, in general, the long-time limit of mean curvature flow can be unstable. For curve shortening flow, a well-known example is the flow $\gamma_t$ of a closed embedded curve $\gamma \subset (S^2, g_{round})$ which bisects the sphere. It will converge, as $t \to \infty$, smoothly to an equator of the sphere; curves $\gamma$ which do not bisect the sphere instead flow to round points. Most curves $\gamma$ do not bisect the sphere and we can always perturb one that does to one which does not, of course, so most curve shortening flows cannot converge to an unstable geodesic. This observation motivates us to consider to which extent we can show the flow will generically avoid unstable minimal surfaces. The main tool we use to pursue this is the avoidance principle. In $\R^{n+1}$, if two mean curvature flows of closed hypersurfaces are disjoint at time $0$, then they will remain disjoint for all the future time. Moreover, the distance between these two flows is increasing. This basic property motivates the definition of weak set flows by Ilmanen \cite{Ilmanen92_LSF} and White \cite{White95_WSF_Top}. The avoidance principle in a manifold is more subtle, because in general two disjoint mean curvature flows can actually get closer -- consider for instance the flow of small graphs over strictly area minimizing minimal surfaces. In a manifold equipped with a positive Ricci curvature metric, the avoidance principle is very strong though and lets us easily show the following: \begin{thm}\label{Rc_thm} In a Ricci positive manifold $(N^n,g)$, the mean curvature flow of a generic embedded hypersurface $M$ will go extinct in finite time. \end{thm} This can be viewed as a generalization of the generic behavior of the curve shortening flows in $S^2$. As a corollary this yields another proof of the classical fact that $H_{2}(N,\R)$ for $3$--manifolds $N$ of positive Ricci curvature, see Corollary \ref{vanish}. In the case $(N,g)$ is not Ricci positive, there could be locally minimizing surfaces and we cannot expect the flows starting from generic embedded hypersurfaces to go extinct in finite time. However, we can use the avoidance principle to show that after perturbations, the flow can avoid those unstable limits. Our next theorem, inspired by work of Colding and Minicozzi \cite{ColdingMinicozzi12_generic}, says that in closed $3$--manifolds one may construct, from any embedded initial data, piecewise generic mean curvature flows which avoid unstable minimal surfaces as limits: \begin{thm} \label{pw_thm} Let $M^2$ be a $2$--sided properly embedded surface of a closed compact Riemannian $3$--manifold $(N^3,g)$. Then there exists a piecewise almost regular flow emanating from it which either goes extinct in finite time, or there exist $t_i \to \infty$, such that $M_{t_i}$ converges possibly with multiplicity to a (potentially disconnected) stable minimal surface. \end{thm} When the ambient metric is bumpy, which is a generic condition by work of White \cite{White91_Bumpy, White17_Bumpy}, we observe that by Theorem \ref{longterm_uniqueness} the limit in the statement above is unique. Note that in light of \cite{ChenSun24_Multi2inManifold} there are examples where the flow asymptotically converges to a stable limiting surface with multiplicity greater than one so higher multiplicity convergence in the above may actually occur, and in the case the limit is strictly stable seems to be robust under further perturbations. We will give the definition of piecewise almost regular flow in the section below but roughly speaking it is a concatenation of almost regular flows where by--hand isotopies are performed at the jumps; the number of perturbations by isotopy taken will be finite, and the perturbations can be taken as small as we wish. In that sense the statement above is a statement on generic flows following the convention of Colding and Minicozzi \cite{ColdingMinicozzi12_generic}. Of course, it is certainly desirable to only require perturbing the initial data. The obstruction to doing so in the statement above essentially boils down to the possibility of convergence to an unstable minimal surface with multiplicity; in analogy to singularity analysis and the resolution of the multiplicity one conjecture by Bamler and Kleiner \cite{BamlerKleiner23_Multiplicity1} one might expect this to not occur; see the analogous situation in Almgren-Pitts minmax theory \cite{Zhou19_multi1}. We recall the following conjecture in \cite{ChenSun24_Multi2inManifold}: \begin{conj}[Conjecture 1.3 in \cite{ChenSun24_Multi2inManifold}]\label{conj_multi1} Suppose the long-time limit of an almost regular mean curvature flow in $(N^n,g)$ is an unstable minimal surface, then the convergence must be multiplicity $1$. \end{conj} Despite this, we point out that as applications we get flow proofs of the following existence theorems for stable minimal surfaces. In the proofs of these we will often specifically use piecewise generic flows, for instance because the topological change through singularities is easy to understand. We also recall that obtaining stability of the minimal surfaces is helpful because these often have better controlled topology and geometry, for instance in the context of positive scalar curvature. These corollaries are not new and there have been long standing proofs of them using geometric measure theory or harmonic map theory, see \cite{FedererFleming60_Current, SchoenYau79_ExistenceIncompressibleMinSurf, SacksUhlenbeck81_minimal2sphere, MeeksYau80_Top3d,MeeksSimonYau82_MinSurf, FreedmanHassScott83_LeastArea}, among others. On the other hand, the construction of these minimal surfaces via flows, along with a number of fairly explicit perturbations, is quite amenable to numerical approximation with the caveat that one has found appropriate initial data to start with. In the following statements $(N,g)$ is a compact orientable Riemannian $3$--manifold, where in this case oriented surfaces are $2$--sided and vice versa: \begin{cor}\label{existence1} Suppose $\pi_2(N)$ is nontrivial. Then $N$ contains an embedded stable minimal sphere or projective plane. \end{cor} Note that in the statement above we invoke the sphere theorem to find embedded initial data for which to start the flow and so the above does not constitute a new proof of it. As we pointed out already compared to other techniques a pitfall of using the mean curvature flow to find minimal surfaces is that it is best to start with well-prepared (i.e. embedded) initial data. In some cases the flow can produce stable minimal surfaces of nontrivial topology, the most basic of which is the following. \begin{cor}\label{existence2} Suppose $N$ has trivial second homotopy group and contains a homologically nontrivial embedded torus. Then $N$ contains an embedded stable minimal surface homeomorphic either to a torus or a Klein bottle. \end{cor} Recall that hyperbolic manifolds are aspherical, so the space of such $3$--manifolds with $\pi_2$ trivial is quite large and is even generic amongst $3$--manifolds from a number of perspectives, for example see \cite{DunfieldTHurston06_Random3Mfd, Maher10_RandomHeegaard}. For surfaces of more general topology, we can show the following result which is very much in the spirit of the existence result of Freedman, Hass, and Scott \cite{FreedmanHassScott83_LeastArea} -- in fact it implies the result above as we discuss in section \ref{applications}. The surfaces below are known as algebraically incompressible surfaces, and are incompressible in the more geometric sense via Dehn's lemma: \begin{cor}\label{existence3} Suppose $\iota: \Sigma \to N$ is an orientable properly embedded surface of $N$ of genus $g \geq 1$ for which $\iota_{*} \pi_1(\Sigma) \to \pi_1(N)$ is injective, and that $N$ has trivial second homotopy group. Then either $M = \iota(\Sigma)$ is homotopic to an orientable stable minimal surface or a double cover of a stable minimally embedded connect sum of $g+1$ projective planes, where $g$ is the genus of $\Sigma$. \end{cor} By double cover of an embedded surface $P \subset N$ here we mean specifically a surface which is the boundary of a tubular neighborhood of $P$ in $N$. It is a fact (see for instance Lemma 3.6 in \cite{Hatcher07_Notes3Mfd}) that every class of $H_2(N,\Z)$ can be represented by an orientable properly embedded surface $M$. Recalling the Thurston norm $||\sigma||_{T}$ of $\sigma\in H_2(N,\Z)$ is the smallest attained genus of embedded surfaces homologous to $\sigma$, we see any such surface $M$ realizing the norm must be algebraically incompressible. If $||\sigma||_{T} \geq 1$ we can apply the Corollary above then, and clearly this must be the case if we suppose that $\pi_2(N)$ is trivial. We can record the discussion above as follows: \begin{cor}\label{existence0} Suppose $\sigma\in H_2(N, \Z)$, with $\pi_2(N)$ trivial. Then either there exists an embedded oriented stable minimal surface $\Sigma$ in the class of $\sigma$ that realizes the Thurston norm of $\sigma$ or there is an element realizing the Thurston norm which is the double cover of a embedded nonoriented stable minimal surface. \end{cor} Finally, we note that Theorem \ref{longterm} lets us substitute the tightening process in minmax for mean curvature flow, at least in some cases. A major issue in applying the flow to a sweepout is that the flow of a sweepout may not remain a sweepout, but this can be compensated for if the flow of the elements of it will be nonfattening. In particular we show the following: \begin{cor}\label{existence5} Suppose $N$ is diffeomorphic to $S^3$. Then $N$ contains a minimal embedded $S^2$. \end{cor} Originally this result was proved by Simon and Smith using minmax theory, see also \cite{ColdingDeLellis03_minmax}. As to the actual best result known we point out that recently Haslhofer and Ketover in \cite{HaslhoferKetover19_Min2Sphere} showed there are at least two minimal spheres in $S^3$ with a bumpy metric, and very recently Zhichao Wang and Xin Zhou in \cite{WangZhou23_4Sphere} proved that there are at least $4$ embedded minimal spheres in $S^3$ equipped with either a bumpy or Ricci positive metric. We expect that if Conjecture \ref{conj_multi1} is true then the mean curvature flow can also be used to recover their results. We emphasize that in the above we are not applying Theorem \ref{pw_thm} and the minimal surface above is potentially unstable, as must be the case when for instance $N$ is Ricci positive. We conclude the introduction with a remark on the compactness assumptions made throughout. When the ambient manifold is not closed, but merely complete with uniform bounded curvature and injectivity radius lower bound, Theorems \ref{smoothfate} and \ref{pw_thm} remain valid, if we include the possibility that the flow can escape from any compact set. For example if $N$ is $S^2\times\R$ with an infinite long trumpet shape (i.e. a cusp) in one direction the flow of an appropriate $S^2$ section of it will escape to spatial infinity/clear out, neither terminating or converging to a minimal surface. It seems to be a more complicated manner yet to generalize our results to the case $M$ is allowed to be noncompact (say properly embedded in a noncompact $N$), one very basic reason being that the area can be infinite. As before the flow may clear out in the manner of the example above but it seems that in some cases if the flow doesn't clear out or go extinct one could show it must flow to a minimal surface. For a basic example, a well known consequence of Ecker and Huisken \cite{EckerHuisken89_EntireGraph} says that uniformly Lipschitz graphs of bounded height in $\R^n$ must converge to flat, hence minimal, planes. \subsection*{Acknowledgments} A.M. thanks Alec Payne and Felix Schulze for their interest and helpful comments. In the course of preparing the article he was supported by CPH-GEOTOP-DNRF151 from the Danish National Research Foundation and CF21-0680 from the Carlsberg Foundation (via GeoTop and N.M. M{\o}ller respectively) and is grateful for their assistance. A.S. is supported by the AMS-Simons travel grant. He is grateful to the Copenhagen Centre for Geometry \& Topology (GeoTop) at the University of Copenhagen, especially N.M. M{\o}ller, for the invitation to visit in Spring 2024 and participate in the meeting "Masterclass: Recent Progress on Singularity Analysis and Applications of the Mean Curvature Flow". Part of the work was initiated during the visit. \section{Background and preliminary lemmas}\label{prelim} Let $N^{n+1}$ be a Riemannian manifold and $X: M \to N^{n+1}$ be an embedding of a manifold $M^n$ realizing it as a $2$--sided smooth closed hypersurface of $N$, whose image by abuse of notation we also refer to as $M$. Then the mean curvature flow $M_t$ of $M$ is given by (the image of) $X: M \times [0,T) \to N^{n+1}$ satisfying the following: \begin{equation}\label{MCF_equation} \frac{dX}{dt} = \vec{H}, \text{ } X(M, 0) = X(M) \end{equation} An immediate consequence of the first variation formula and the evolution equation above is that the mean curvature flow is the gradient flow of the area functional, in the sense that $\frac{d}{dt}\text{Area}(M_t) = - \int_{M_t} |\vec H|^2 d\mu_t$, where $\mu_t$ is the area measure; from this one sees the mean curvature is well adapted to finding minimal surfaces, which are critical points of the area functional. The first obstruction to the construction of minimal surfaces using mean curvature flow is singularity formation: as a highly nonlinear system, mean curvature flow can develop finite time singularities; then it is even unclear how to continue the flow after the singular time. To overcome this issue, there are three major weak flow approaches: \begin{itemize} \item The Brakke flow \item The weak set flow and level set flow \item The surgery flow \end{itemize} The \textbf{Brakke flow}, introduced by Brakke in his thesis \cite{Brakke78} is a geometric measure-theoretic definition in terms of (typically, integral) varifolds that satisfy the eponymous Brakke inequality. Here we present a definition essentially due to Ilmanen \cite{Ilmanen94_EllipReg}, and the expository is from \cite{BamlerKleiner23_Multiplicity1}. An $n$-dimensional Brakke flow defined on the time-interval $I\subset\R$ in a $(n+1)$-dimensional manifold $(N,g)$ is a family of Radon measures $(\mu_t)_{t\in I}$ such that: \begin{itemize} \item For a.e. $t\in I$ the measure is integer $\cH^n$-rectifiable and the associated varifold has locally bounded first variation with variational vector $\vec H$ in $L^1$. \item For any compact set $K\subset N$, and any $[t_1,t_2]\subset I$, \[ \int_{t_1}^{t_2}\int_K |\vec H|^2 d\mu_t dt<+\infty, \] \item For any $[t_1,t_2]\subset I$ and $u\in C_c^1(N\times[t_1,t_2])$, \begin{equation}\label{eq:BrakkeF} \left.\int_N u(\cdot,t) d\mu_t \right|_{t=t_1}^{t=t_2} \leq \int_{t_1}^{t_2}\int_N (\pr_t u+\nabla u\cdot \vec H-u|\vec H|^2)d\mu_t dt. \end{equation} \end{itemize} One can quickly see that the Radon measures associated to a smooth/classical mean curvature flow, that is a family of smooth manifolds satisfying \eqref{MCF_equation}, is a Brakke flow. A Brakke flow suffers from a serious problem: as \eqref{eq:BrakkeF} is only an inequality, the whole connected components of (the support of) a Brakke flow are allowed to disappear instantaneously! In the same monograph, Ilmanen \cite{Ilmanen94_EllipReg} (see also White \cite{White09_CurrentsVarifolds}) gives a construction via elliptic regularization of a "well-behaved" Brakke flow out of smooth initial data for which these issues don't occur. Such a Brakke flow is known to be: \begin{itemize} \item unit-regular: if the density of the Brakke flow at a spacetime point is $1$, then it is a smooth mean curvature flow in a spacetime neighborhood. \item cyclic mod $2$: suppose the unique associated rectifiable mod-$2$ flat chain of $\mu_t$ is $V(t)$, then $\pr[V(t)]=0$ for a.e. $t\in I$. \end{itemize} Even amongst such well-behaved Brakke flows there happens to be an issue of uniqueness, which brings us to the level set flow which originated from the viscosity method \cite{EvansSpruck91, ChenGigaGoto91_LSF}. Later Ilmanen \cite{Ilmanen92_LSF} and White \cite{White95_WSF_Top} introduced a purely set-theoretic definition using the avoidance principle. The classical avoidance principle says that two disjoint hypersurfaces in a manifold $N$ will stay disjoint, at least as long as one of them is compact. The \textbf{weak set flow} is defined to be a closed subset $\cM$ of $N\times\R$ such that if a smooth mean curvature flow does not intersect $\cM\cap\{t\}$, then it is disjoint from $\cM\cap \{t'\}$ for all $t'>t$. White \cite{White24_Avoidance} proved the following avoidance principle for weak set flows. \begin{thm}\label{ambient_avoidance} Suppose that $N$ is a complete, connected Riemannian manifold with positive injectivity radius such that $|\nabla^k \text{Riem} |$ is bounded for each nonnegative integer $k$. Let $\Lambda$ be a lower bound for the Ricci curvature of $N$, and suppose that $t \in [0, \infty) \to X(t), Y(t)$ are weak set flows in $N$. Then \begin{equation}\label{dist} e^{-\Lambda t} d(X(t), Y(t)) \end{equation} is an increasing function of $t$. \end{thm} In particular, note that for $\Lambda > 0$ two flows don't merely stay disjoint but actively "repel" exponentially quickly -- this will play an important role in the proof of Theorem \ref{Rc_thm} below. Indeed the support of a Brakke flow is a weak set flow, as shown in Ilmanen's monograph. Although weak set flows may not be unique starting from a given hypersurface, there is a canonical weak set flow associated to any initial data called the \textbf{level set flow}: for a given set $A$ is as the envelope of all families of sets initially agreeing with $A$ that satisfy the avoidance principle. As an aside we point out that in fact the level set flow can even be defined this way for closed initial data that is not necessarily a regular hypersurface. While the existence and uniqueness are guaranteed, the level set flow of initial data may develop interior even if it is smooth, as first noted by Ilmanen and White \cite{White02ICM} (and recently appeared in \cite{IlmanenWhite24_Fattening}, see also related works \cite{AngenentIlmanenVelazquez17_fattening, LeeZhao24_MCFconical, Ketover24_Fattening}) -- this fattening can be seen as an indication of nonuniqueness for level set flows. To study the level set flow even in the presence of fattening Hershkovits and White \cite{HershkovitsWhite20_Nonfattening} introduced the notions of innermost/outermost flows. Given a closed hypersurface $\Sigma \subset \R^3$ that is the boundary of a domain $\Omega$, we can define: \begin{itemize} \item The innermost flow $\cM^-$ is the boundary of the level set flow in $N\times\R$ generated by $\Omega\times\{0\}$; \item The outermost flow $\cM^+$ is the boundary of the level set flow in $N\times\R$ generated by $(N\backslash\Omega)\times\{0\}$. \end{itemize} Hershkovits and White noticed that if the level set flow $\cM$ starting from $\Sigma$ fattens, then $\cM^-$ and $\cM^+$ are different. It is expected that if the level set flow fattens, the innermost/outermost flows are natural candidates to serve as the canonical representatives of the weak set flow. It is not immediate to define inner and outermost flows in general $3$--manifolds from a $2$--sided surface $M$ because such a surface need not be separating, as a cross section of $S^2 \times S^1$ illustrates. In the sequel we will define such flows as the following: for a choice of normal vector $\nu$ consider the domains $\cM^-$ and $\cM^+$ bounded by $M$ and $M \pm \epsilon \nu$ respectively, for some small $\epsilon > 0$, where $M \pm \epsilon \nu$ are surfaces that are generated by pushing $M$ in $\epsilon\nu$ direction by the exponential map. Then we define the inner and outermost flows to be the boundary component of the level set flows of $\cM^-$, $\cM^+$ respectively emanating from $M$. The third weak flow is the \textbf{mean curvature flow with surgery}, where one preemptively cuts out regions developing high curvature, classifies them topologically, and then continues the flow in a piecewise manner, see \cite{Head13_MCF2convex, Lauer13_MCFSurgery, BrendleHuisken16_MCFSurgeryR3, HaslhoferKleiner17_MCFsurgery, Daniels-Holgate22_SurgeryApprox}. Locally, the high curvature regions will be modeled on round spheres or cylinders in coordinate patches which will bound solid balls/cylinders respectively. Regions where high curvature transitions into low curvature (i.e. "neckpinches") the surface are modeled on cylinders and at these points one "cuts" the neck at an appropriate spot and places in caps/discs of controlled geometry, in particular so that the remaining low curvature regions have curvature upper bounded by a controlled constant (which is amongst the analytical difficulties in establishing the flow with surgery). We note that the high curvature regions after cap placement will be homeomorphic to either $S^n$ or $S^{n-1} \times S^1$, which makes the flow with surgery a useful tool in topology. This particular consequence won't be so relevant in this work but the well controlled, concretely understandable change of topology across surgeries will be helpful in the sequel. \subsection{Almost regular flow and generic approximation} While these notions of weak flows seem quite different, in $3$--manifolds there is a relatively complete picture to relate all of them. In \cite{BamlerKleiner23_Multiplicity1}, Bamler and Kleiner introduced a new notion of weak flow called \textbf{almost regular flow} which are well behaved Brakke flows in the sense described above along with an extra integrability condition on what they call scale functions, which measure the size of neighborhoods in which the flow is smooth. We refer the readers to \cite{BamlerKleiner23_Multiplicity1} for the definition and properties of almost regular flows. Here we only state properties of almost regular flows that we reference: \begin{enumerate} \item Generic flows, i.e. ones which only encounter mean convex singularities (see discussion and references below) are almost regular. \item The innermost/outermost flows that start from a closed smooth embedded surface can be approximated by generic flows as a consequence are almost regular flows for $t\geq 0$. In particular, if the level set flow from a closed smooth embedded surface is nonfattening then it is almost regular. \item An almost regular flow is regular for a.e. $t\in I$. \item (Lemma 2.7 in \cite{BamlerKleiner23_Multiplicity1}) If $\cM$ is almost regular, then for any test function $u\in C_c^1(N\times[t_1,t_2])$ where $[t_1,t_2]\subset \text{Int} I$, $t\to\int_N u(\cdot,t)d\mu_t$ is continuous and the equality holds in \eqref{eq:BrakkeF}. \end{enumerate} While the almost regular flows have many nice properties, they still possibly develop complicated singularities which obscures the possible topological change of the flow through them. On the other hand, as we have described before, the mean curvature flow with surgery has a clear description of the topological change of the flow. Therefore, we would like to use mean curvature flow with surgery to approximate the almost regular flows we use in the sequel, particularly in showing (some of) the applications mentioned in the introduction. In order to use mean curvature flow with surgery to approximate the almost regular flows we use in practice, we first use a well behaved (unit regular, cyclic) Brakke flow with only cylindrical and spherical singularities to approximate them. Such flows we call generic. It has been conjectured by Huisken that mean curvature flow with generic initial data can only have cylindrical and spherical singularities, and recently there has been various progress on this conjecture, \cite{ColdingMinicozzi12_generic, CCMS20_GenericMCF, chodoshchoischulze2023mean, SunXue2021_initial_closed, SunXue2021_initial_conical}. Using the result of Chodosh, Choi, Mantoulidis and Schulze \cite{CCMS20_GenericMCF} and Chodosh, Choi, and Schulze \cite{chodoshchoischulze2023mean}, Bamler and Kleiner \cite{BamlerKleiner23_Multiplicity1} proved that a mean curvature flow starting from a generic closed embedded surface in a three-manifold can only have cylindrical and spherical singularities. Furthermore, Daniels-Holgate \cite{Daniels-Holgate22_SurgeryApprox} proved that the mean curvature flows with surgeries can approximate a mean curvature flow with only cylindrical and spherical singularities, where the approximation is in the Hausdorff distance. In addition, this approximation is smooth away from the singular set. Combining all the ingredients above gives the following approximation result in $\R^3$: \begin{prop}\label{goodflow_ambient} The inner and outermost flows starting from a closed embedded surface in $\R^3$ can be approximated by mean curvature flows with surgeries in Hausdorff distance, and the approximation is smooth away from the singular set. \end{prop} Note that in the classical case i.e. $N = \R^{3}$ properly embedded hypersurfaces are automatically orientable. If $N$ is more generally simply connected it is not hard to see this true by an intersection number argument, but we point out it does not suffice if $N$ is merely orientable with as the example $\R P^2 \subset \R P^3$ shows. With this in mind, we claim that the following holds true in general closed $3$--manifolds: \begin{prop} Let $M$ be a closed, properly embedded $2$--sided surface $M$ in a closed $3$--manifold $N$. Then the inner and outermost flows of $M$, as defined above, can be approximated by mean curvature flows with surgeries in Hausdorff distance over any fixed time interval and the approximation is smooth away from the singular set. \end{prop} \begin{proof} The main point is to establish the existence of generic mean curvature flow ala \cite{chodoshchoischulze2023mean} and its subsequent approximation by surgery flows, then the statement about inner and outermost flows follows exactly as in the Euclidean case by an approximation argument as discussed in section 7 of \cite{BamlerKleiner23_Multiplicity1}. As is well known, in a blowup limit singularities will still be modeled on solitons in $\R^3$ so we mostly just supply a sketch of this argument. Noting that because $M$ is 2-sided one may meaningfully discuss one-sided perturbations of $M$ and the point made in the previous sentence one readily sees from \cite{CCMS20_GenericMCF, chodoshchoischulze2023mean} that if one encounters a nongeneric singularity of multiplicity one that the initial data can be perturbed to avoid the singularity at that spacetime point. Because $M$ is a compact surface it has finite genus and so by genus monotonicity (see Section 11 of \cite{CCMS20_GenericMCF}) and the classification of genus $0$ shrinkers by Brendle \cite{Brendle16_genus0}, only finitely many (arbitrarily small) perturbations of the initial data are required to produce a generic flow up at least to the first time a high multiplicity tangent flow develops. Now all these generic flows, as shown in section 7 of \cite{BamlerKleiner23_Multiplicity1}, are almost regular flows; note this uses the resolution of the mean convex neighborhood conjecture by Choi, Haslhofer, and Hershkovits \cite{ChoiHaslhoferHershkovits18_MeanConvNeighb} which the authors note carries over in the curved ambient setting. Because under rescaling as in singularity analysis the ambient metric converges smoothly to the Euclidean metric, one can see by contradiction that the resolution of the multiplicity one conjecture applies in the curved ambient setting because all the relevant geometric quantities can be arranged to evolve as they do in the flat case up to arbitrarily small errors -- their argument at a high level is by showing in the case of high multiplicity convergence in a tangent flow that a certain quantity satisfies contradictory bounds so this suffices for our needs. As a result we get that for generic choice of data there will be a flow along which only mean convex singularities may develop for all time; by the resolution of the mean convex neighborhood conjecture \cite{ChoiHaslhoferHershkovits18_MeanConvNeighb} and the barrier argument of Hershkovits and White \cite{HershkovitsWhite20_Nonfattening, HershkovitsWhite23_Avoid}, it is unique among almost regular flows starting from $M$. The approximation theorem of Daniels-Holgate also applies in this setting; most importantly besides the mean convex neighborhood conjecture the required local estimates for surgery also hold in the general ambient setting as shown by Haslhofer and Ketover \cite{HaslhoferKetover19_Min2Sphere}. \end{proof} We next present some properties of such almost regular flows, where $M$ is as usual a properly embedded $2$--sided surface in a given closed $3$--manifold $N$. \begin{lem}\label{lem:PreserveOriented} Suppose that $M_t$ is a generic or inner/outermost flow out of $M$. Then $M_t$ will also be 2--sided for all regular times $t$. \end{lem} \begin{proof} It's obvious if $M_t$ is smooth then it will remain 2--sided, because the flow is an isotopy. In the case that $M_t$ has singularities, we first observe that it is easy to see for the approximating smooth flows $S_t$ to $M_t$ produced in \cite{Daniels-Holgate22_SurgeryApprox} that $S_t$ will remain orientable if $M$ is. The convergence of the approximating surgery flows is smooth during regular times, giving the claim. \end{proof} An important subtlety to remember though, as mentioned in the introduction, is that orientability is not necessarily inherited by the limit we discuss in the next section. In the next statement we write $M\sim \sigma$ to indicate that $M$ is homologous to $\sigma$. \begin{lem}\label{lem:PreserveHomology} Suppose that $M_t$ is a generic or inner/outermost flow out of $M$, with $M\sim \sigma$ where $\sigma\in H_2(N,\Z)$. Then $M_t \sim \sigma$ for all regular times $t\geq 0$. \end{lem} \begin{proof} Again using that the approximation by surgery flows is smooth during regular times, one only needs to note that surgery does not change the $2$nd homology class of a regular surface. \end{proof} \begin{lem}\label{lem:GenusNonincreasing} Suppose that $M_t$ is a generic or inner/outermost flow out of $M$. Then the genus of $M_t$, considered at smooth times, is nonincreasing along the flow. \end{lem} \begin{proof} As in the previous proofs it suffices to note this is the case for approximating surgery flows. Now in the surgery components are either deleted or cylindrical regions are cut and replaced with caps and both of these actions do not increase the genus, giving the claim. \end{proof} We show in the next section that an almost regular flow will either go extinct or flow to a potentially unstable smooth minimal surface, possibly with multiplicity. If the minimal surface is unstable we will afterwards wish to perturb the flow to ensure it doesn't flow to it, but as discussed in the introduction for technical reasons it seems difficult to carry this out by just perturbing the initial data if the multiplicity is high (i.e. greater or equal than $2$). With this in mind we will consider the following piecewise almost regular flows in the sequel: \begin{defn}\label{pwflow_def} Let $M$ be a properly embedded, $2$--sided surface of a $3$--manifold $N$. Then a piecewise almost regular flow $M_t$ emanating from $M$ defined on $[0, \infty)$ is given by an increasing (possibly finite) sequence of times $t_i \to \infty$ for which \begin{enumerate} \item $M_t$ on $[0, t_1]$ is an almost regular flow out of $M$ satisfying the conclusions of Proposition \ref{goodflow_ambient} above and so that $M_{t_1}$ is a smooth surface. \item Denoting by $M_t^1$ on $[0, t_1)$ the flow from above, we let $\tilde{M}^1_{t_1}$ be a surface that is isotopic to $M^1_{t_1}$, and we define $M^2_t$ out of this perturbed surface $\tilde{M}^1_{t_1}$ as above. \item Similarly, we define $M^i_t$ for $i \geq 2$ inductively. \end{enumerate} \end{defn} We end this section with the following technical tool to ensure the limit of the flow we take is smooth: \subsection{Ilmanen's regularity theorem} Because the mean curvature flow is the gradient flow of area functional, the long-time limit is expected to be a critical point to the area functional. From geometric measure theory, the limit is known to be a stationary varifold, and without any other assumptions, the limit can have singularities. In \cite{Ilmanen95_Sing2D}, Ilmanen used Simon's sheeting theorem \cite{Simon94_Willmore} to show that the tangent flow of a first-time singularity of a mean curvature flow of closed embedded surfaces in $\R^3$ must support on a smooth embedded shrinker, possibly with multiplicity. Ilmanen's argument is very general and can be adapted to study the long-time limit of mean curvature flow. For the sake of clarity we precisely state the version that we will use in this paper: \begin{thm}[Ilmanen's regularity for limit surfaces]\label{thm:IlReg} Suppose $\{M_i\}_{i\in\Z_+}$ is a sequence of closed embedded surfaces in a closed manifold $(N^3,g)$, satisfies the following assumptions: \begin{enumerate} \item $\area(M_i)$ and $\text{genus}(M_i)$ are uniformly bounded from above; \item $\int_{M_i}|\vec H|^2 d\mu_{M_i}\to 0$ as $i\to\infty$; \item (Area growth bound) there exists $r_0>0$ such that there exists a constant $C$ such that for any $x\in N$ and $r\in(0,r_0)$, $\area(M_i\cap B_r(x))\leq Cr^2$. Here $B_r(x)$ can either be the intrinsic ball in $(N,g)$ or the extrinsic ball for a fixed isometric embedding $(N,g)\to (\R^m,g_{\text{Euc}})$. \end{enumerate} Then there exists a subsequence of $\{M_i\}_{i\in\Z_+}$, still denoted by $\{M_i\}$, that converges to an integral varifold $V$ in the sense of varifold, such that $V$ is supported on a finitely many disjoint closed embedded minimal surface $\Sigma_1,\cdots,\Sigma_k$, with multiplicity $m_1,\cdots,m_k\geq 1$. Moreover, for any $j=1,2,\cdots,k$, there exists finitely many points $p_{j,1},\cdots,p_{j,\ell_j}$ inside $\Sigma_j$, such that for any $r>0$ that is sufficiently small, for sufficiently large $i$, $M_i\backslash \bigcup_{l=1}^{\ell_j} B_{r}(p_{j,l})$ is the union of $m_k$ number of disjoint connected components, and each component is isotopic to and converges in the varifold sense with either multiplicity $1$ or $2$ to $\Sigma_j \backslash \bigcup_{l=1}^{\ell_j} B_{r}(p_{j,l})$ depending on if the limit is two sided or not. \end{thm} \section{Long-time behavior of almost regular flows: the proof of Theorem \ref{longterm}} In this section we show that in the long term an almost regular flow either goes extinct in finite time or limits to a collection of smooth, but possibly unstable, minimal surfaces. Note that almost regular flows are not necessarily generic, although in the further applications we will often suppose our flow is -- for posterities sake, we work in this more general class as much as possible. We finish with some properties of the minimal surfaces we find should the limit be nonempty. We start with the following total curvature bound: \begin{lem}\label{tabounds} Let $(N^3,g)$ be a closed manifold and $(M_t)_{t\in[0,\infty)}$ is an almost regular flow. For each $\epsilon > 0$, there exists a $C = C(\epsilon, M_0,g)$ such that $\int_{M_t} |A|^2\leq C$ for $t\in[0,\infty)$ away from a set of measure $\epsilon$. \end{lem} \begin{proof} Because $M_t$ is regular for a.e. time $t\geq 0$, we can apply the Gauss-Codazzi equations to get $R_N = K + 2 \Ric_N(\nu,\nu) + |A|^2 - H^2$, where $R_N$ and $\Ric_N$ are the scalar and Ricci curvatures on $N$ respectively. With this in hand, by Gauss--Bonnet we have for any $t > 0$ that is a regular time: \begin{equation} \int_{M_t} |A|^2 = \int_{M_t} H^2 - 4 \pi \chi(M_t) + C\text{Vol}(M_t) \end{equation} Where the constant $C$, independent of $t$, depends on the curvature of $N$. By the property of almost regular flow \cite{BamlerKleiner23_Multiplicity1}, the genus of $M_t$ is bounded by a uniform constant $G$, and hence $\chi(M_t)$ is also bounded by a constant. Of course $\text{Vol}(M_t)$ is bounded in terms of $\text{Vol}(M)$. Since the time derivative of the area is given by $- \int_{M_t} H^2$ (for weak flows this is a consequence of Brakke's inequality) we see measure of the set of times for which $\int H^2$ is greater than a constant $C_1$ is bounded by $C_1/\text{area}(M_0)$. These two observations combined give what we want. \end{proof} Similarly, we have the following: \begin{lem}\label{decay} $\liminf\limits_{t\to\infty}\int_{M_{t}} H^2 = 0$. \end{lem} \begin{proof} Supposing not, then there exists some $\epsilon > 0$ for which $\int_{M_t} H^2 > \epsilon$ for all sufficiently large $t$. This contradicts that $M$ is compact and so of finite area and that $\frac{d}{dt} \text{Area}(M_t) \leq -\int_{M_t} H^2$ by the Brakke inequality. \end{proof} With Lemma \ref{decay} in hand, Allard's compactness theorem implies the following convergence theorem: \begin{cor}\label{lem:VariConv} There exists a sequence of time $t_i\to\infty$ such that $M_{t_i}$ converges to an integral stationary varifold $V$. \end{cor} Of course, from general theory $\supp V$ is not necessarily a regular minimal surface. However, for surfaces in $3$--manifolds, we have more powerful tools to pick a convergence subsequence whose limit is regular, using Ilmanen's regularity theory \cite{Ilmanen95_Sing2D}. We adopt his argument here; Lemma \ref{tabounds} gives us the total curvature bounds we require in the application of Simon's sheeting theorem, but there is a central feature of mean curvature flows in $\R^3$ that does not hold for mean curvature flows in a $3$--manifold which requires us to take some extra care. Namely Huisken's monotonicity formula shows that a mean curvature flow $M_t$ of surfaces in $\R^3$ has a uniform area growth bound $\area(M_t\cap B_R(x))\leq D\pi R^2$ for all $R>0$, $x\in\R^3$ and $t>0$, where $D$ only depends on $M_0$; see Proposition 3.3 in \cite{sun-generic-multi-1}. This area growth bound is crucial in applying Simon's sheeting theorem in \cite{Ilmanen95_Sing2D}; in particular see Theorem 10 and Corollary 11 of \cite{Ilmanen95_Sing2D}. When $M_t$ is a mean curvature flow in a closed manifold $(N,g)$, by isometric embedding $(N,g)$ into $\R^m$, $M_t$ is a mean curvature flow with additional forces $\beta$, where $\beta$ is a tangent vector field on $N$, depending on the second fundamental form of $(N,g)$ in $\R^m$. Because of the forcing term, Huisken's monotonicity formula can not directly provide uniform area growth bound for all future time $t$, but luckily a modified version of it can for short times which will suffice for our purposes. To start, let us fix an isometric embedding of $(N,g)$ into $\R^m$; recalling that we suppose $N$ is compact and denote by $L < \infty$ an upper diameter bound for $N$ under this embedded. Then it happens that the forcing term $\beta$, $\beta(x)\in T_xN$, only depends on the second fundamental form of the isometric embedding of $(N,g)$. Hence $\|\beta\|_{L^\infty}\leq C$ for some fixed constant $C$. Considering \begin{equation} \rho_{x_0,t_0}(x,t)=(4\pi(t_0-t))^{-1}e^{-\frac{|x-x_0|^2}{4(t_0-t)}} \end{equation} which is the backward heat kernel in $2$-dimensions, the monotonicity formula of mean curvature flow with additional forces \cite{White97_Stratif, AM_CMgenericambient} shows that for $0\leq t_1\leq t_2<t_0$, \begin{equation}\label{eq:MonoForce} \int_{M_{t_2}}\rho_{x_0,t_0} (x,t_2) dx \leq e^{C(t_2-t_1)}\int_{M_{t_1}}\rho_{x_0,t_0}(x,t_1)dx. \end{equation} We remark that the monotonicity formula also works for Brakke flows with additional forces, so this inequality holds even $M_t$ is not necessarily smooth for $t\in[t_1,t_2]$. The following lemma shows that, while we may not get a uniform area growth bound for all $t>0$, we can show that for sufficiently large $t$, $M_t$ has a uniform area growth bound. \begin{lem}\label{lem:AreaGrowth} There exists $D>0$ and $\mathbf{t}>0$ such that for $t>\mathbf{t}$, and any $R\in(0,L]$, $\tau\in(1,2)$ and $x_0\in \R^m$ that $\area(B_R(x_0)\cap M_{t_i+\tau})\leq D\pi R^2$ where $L$ is the diameter bound of $N$ as above. \end{lem} \begin{proof} Because $\int_{M_t} \rho_{x_0,t_0}(x,t_0-R^2)\geq (4\pi R^2)^{-1}e^{-\frac{1}{4}}\area(M_t\cap B_R(x_0))$, we have \[ \area(M_t\cap B_R(x_0))\leq 4\pi R^2\int_{M_t} \rho_{x_0,t_0}(x,t_0-R^2)dx. \] By \eqref{eq:MonoForce}, it suffices to show that exists a fixed constant $\mathbf{t}>0$ such that \[ \int_{M_{t-\mathbf{t}}}\rho_{x_0,t+R^2}(x,t-\mathbf{t})dx \] has a uniform upper bound for all $t>\mathbf{t}$, $x_0\in N$ and $R\in(0,L]$. We observe that \[ \begin{split} \int_{M_{t-\mathbf{t}}}\rho_{x_0,t+R^2}(x_0,t-\mathbf{t}) =& \int_{M_{t-\mathbf{t}}} (4\pi(R^2+\mathbf{t}))^{-1} e^{-\frac{|x-x_0|^2}{4(R^2+\mathbf{t})}}dx \\ \leq & (4\pi(R^2+\mathbf{t}))^{-1}\area(M_{t-\mathbf{t}}) \leq (4\pi\mathbf{t})^{-1}\area(M_0). \end{split} \] Therefore, we can choose $\mathbf{t}=1$ to get a uniform upper bound, which yields the uniform area growth bound for $R\in(0,L]$. \end{proof} We are now ready to show the first result discussed in the introduction: \begin{proof}[proof of Theorem \ref{smoothfate}] If the area of $M_t$ is sufficiently small the clearing out lemma \cite[12.2 and 12.5]{Ilmanen94_EllipReg} shows that $M_t$ goes extinct (i.e. is supported on a set of zero measure) a short time later, so we may suppose its area is uniformly bounded below for all finite times. Considering a sequence $t_i \to \infty$ as in Lemma \ref{decay} above, by Allard compactness we may extract a converging subsequence which will be a stationary varifold $V$. By the almost all time regularity of the almost regular flows, we may suppose this set of times is regular as well. We also have the area growth estimate of $M_{t_i}$ from Lemma \ref{lem:AreaGrowth}. With this in hand we may employ Simon's sheeting theorem ala Ilmanen \cite{Ilmanen95_Sing2D} along a sequence of smooth times $t_i \to \infty$ of $M_{t_i}$ to say that $V$ supports on a finite collection of smooth, embedded minimal surfaces $\{\Sigma_1,\cdots,\Sigma_k\}$ and the convergence is $m_j$--sheeted away from a finite number of ``bridge points'' towards $\Sigma_j$. By the regularity of the limit, and the embeddedness of $M_{t_i}$, all the $\Sigma_j$'s are disjoint via the maximum principle. \end{proof} Higher multiplicity convergence can indeed occur, see \cite{ChenSun24_Multi2inManifold}, and in the case one of the $\Sigma_i$ is strictly stable should be a robust phenomenon. Also, it is easy to see the limiting set could be disconnected. Take for instance the initial data given by two disconnected area-minimizing closed embedded (say, strictly stable) minimal surfaces connected by a very thin neck. Then shortly after the flow is started the neck will quickly pinch and retract to each minimizing surface, and the limit of the flow will be the union of these two disconnected minimal surfaces. It is interesting to ask to what extent the limit is unique. If the convergence has multiplicity $1$, and the manifold is analytic, then by the classic \L{}ojasiewicz-Simon inequality, together with the Brakke/White regularity theory, $M_t$ indeed convergence to $\Sigma$ as $t\to\infty$, not only subsequentially. Without this machinery even in the most well-known case, the curve shortening flow on surfaces, this question is not completely understood. In fact in \cite{Grayson89_CSF} Grayson conjectured that there exists an example of curve shortening flow whose limit geodesics are not unique. It is not so hard though to see the limit will be unique if the limiting minimal surfaces are strictly stable, which we show next. To start the following lemma shows that if we carefully choose $t_i$, $M_{t_i}$ will stay in a neighborhood of the limit surfaces. In other words, those necks described in the example above indeed must disappear in a very short period of time. \begin{lem}\label{lem:LimNbhd} In the context of Corollary \ref{smoothfate} given a sequence $s_i \to \infty$ along which the flow converges to a collection of minimal surfaces $\Sigma_j$ for any $\delta>0$ we can choose another sequence of $t_i\to\infty$ such that, when $t_i$ is sufficiently large, $M_{t_i}$ must lie inside the $\delta$-tubular neighborhood of $\cup_{j=1}^k \Sigma_j$. \end{lem} \begin{proof} Denote by $\cN_{\delta}(\Sigma)$ the (open) $\delta$-tubular neighborhood of a closed embedded surface $\Sigma$ in $N$, and consider a sequence of $s_i\to\infty$ as in Corollary \ref{smoothfate} such that $M_{s_i}$ converges to $m_1\Sigma_1+\cdots+m_k\Sigma_k$ in the sense of varifolds. For any $x_0\in N\backslash(\bigcup_{j=1}^k\cN_{\delta}(\Sigma_j))$, $\area(M_{s_i}\cap B_{\delta}(x_0))\to 0$ because $B_{\delta}(x_0)\cap(\bigcup_{j=1}^k\Sigma)=\emptyset$. In particular, this shows that for any $\eps>0$, $x'\in B_{\delta/2}(x_0)$, and $\eta\in(1/2,3/4)$, the Gaussian weight \[ \int_{M_{s_i}}\rho_{x',s_i+(\eta\delta)^2}(x,s_i)dx<\eps, \text{when $s_i$ is sufficiently large.} \] Then by \eqref{eq:MonoForce} again, we see that when $\eps$ is chosen sufficiently small, the Gaussian density of $x'\in B_{\delta/2}(x_0)$ of $M_t$ for $t\in(s_i+\delta/2,s_i+3\delta/4)$ is strictly less than $1$, hence must be $0$. In other words, for $t\in(s_i+\delta/2,s_i+3\delta/4)$, $M_t$ is disjoint from $B_{\delta/2}(x_0)$ for sufficiently large $s_i$. Because $N\backslash \bigcup_{j=1}^k\cN_{\delta}(\Sigma_j)$ is compact, we can cover it by finitely many $B_{\delta/2}(x_0)$'s. This allows us to pick a sequence of $t_i\to\infty$ such that $M_{t_i}\subset \bigcup_{j=1}^k\cN_{\delta}(\Sigma_j)$ that have bounded total curvature and total mean curvature tending to zero. The argument of Corollary \ref{smoothfate} may then be rerun on this sequence to extract a subsequence which converges to a union of smoothly embedded minimal surfaces $\Sigma_j'$. Using these minimal surfaces the conclusion follows by the triangle inequality. \end{proof} We remark that in the above proof conceivably $\Sigma_i \neq \Sigma_i'$, and in particular the further chosen subsequence $M_{t_j}$ may not converge to the same collection of closed embedded minimal surfaces $m_1\Sigma_1+\cdots+m_k\Sigma_k$. But their limit must be very close to them, in the sense that the limit lies in a tubular neighborhood of these minimal surfaces. Now, recall that a stable minimal surface $\Sigma$ is strictly stable if the linearized operator $L_\Sigma$ has only positive eigenvalues. Using a barrier argument and the lemma above then we observe we may show that if the limit stable minimal surfaces are strictly stable, then the limit is unique, even if the convergence is possibly in high multiplicity and in a merely smooth (but not analytic) background manifold $N$: \begin{thm}\label{longterm_uniqueness1} Suppose $M_t$ is an almost regular flow such that there exists $t_i\to\infty$ such that $M_{t_i} \to m_1\Sigma_1+\cdots+m_k\Sigma_k$ as in Theorem \ref{smoothfate}, and each $\Sigma_j$ is strictly stable. Then the limit is unique: namely for any sequence of $s_i\to\infty$ so that $M_{s_i}$ converges as varifolds the limit must be $m_1\Sigma_1+\cdots+m_k\Sigma_k$. \end{thm} \begin{proof} For any $\delta>0$, we can apply Lemma \ref{lem:LimNbhd} to claim that there is another sequence, which we still label $t_i$, such that $M_{t_i}$ lies inside the union of $\delta$-tubular neighborhood of the $\Sigma_j$'s when $t_i$ is sufficiently large. To proceed, we use the neighborhood foliation structure of strictly stable minimal surfaces: because each $\Sigma_j$ is strictly stable, if it is $2$-sided, then there exists a tubular neighborhood $\cN^j$ of $\Sigma_j$ such that $\cN^j$ is foliated by surfaces $\Sigma^t$ for $t\in(-s,s)$, such that when $t\in(-s,0)\cup(0,s)$, $\Sigma^t$ has positive (inward pointing) mean curvature. For $1$-sided $\Sigma_j$, there is a similar structure of the neighborhood. For a proof of this fact, see \cite[Lemma A.1]{StevensSun24_LargeArea} for $2$-sided case, and \cite[Section 6]{StevensSun24_LargeArea} for $1$-sided case. Using this fact, we can find a tubular neighborhood $\cN^j$ of $\Sigma_j$ such that $\pr\cN^j$ has strictly positive mean curvature. In particular, if at the beginning we choose $\delta>0$ sufficiently small so that the $\delta$-tubular neighborhood is contained in $\cN^j$, then $M_t$ will lie within $\bigcup_{j=1}^k\cN^j$ for $t\geq t_i$. Hence we see any limit of $M_t$ has to be supported in $\bigcup_{j=1}^k\cN^j$. If $\delta > 0$ is sufficiently small we may apply Brakke regularity to see the flows of boundary components of $\bigcup_{j=1}^k\cN^j$ are smooth for all $t > 0$ and asymptote back to $\bigcup \Sigma_j$, so by the comparison principle any converging limit of $M_t$ has to be supported on $\bigcup_{j=1}^k\Sigma_j$. The question of convergence in varifold topology to this entire set is more subtle though, because in the above we may only apply Ilmanen's analysis to special sequence of times $s_i \to \infty$. To deal with this, first we point out that for any sequence $s_i \to \infty$ the argument from the proof of Theorem \ref{smoothfate} does apply we get the limit must be supported precisely on $\bigcup_{j=1}^k\Sigma_j$. Moreover by the monotonicity of area along the mean curvature flow the multiplicities may not vary either, so that for any sequence $t_i \to \infty$ for which $M_{t_i}$ converges "well" as in the proof of Theorem \ref{smoothfate}, $M_{t_i}\to m_1\Sigma_1+\cdots+m_k\Sigma_k$ as $t_i\to\infty$. Now, for a given $1 \leq j \leq k$ choose a small $\epsilon > 0$ let $u_{j, \epsilon, D}$ be a smooth function which is supported on the $\epsilon$ tubular neighborhood of a domain $D \subset \Sigma_j$. Because almost regular flows are smooth for almost all times and continuous in the weak topology in time (see Lemma 2.7 in \cite{BamlerKleiner23_Multiplicity1}) Lemmas \ref{tabounds}, \ref{decay} gives us that for any $\delta > 0$ we can approximate any sequence $s_i \to \infty$ by another sequence $s_i' \to \infty$ so that $\int_{M_{s_i}} u_{j, \epsilon, D} - \int_{M_{s_i'}} u_{j, \epsilon, D} < \delta$ and that Ilmanen's analysis applies to the sequence $s_i'$. Because from the above any converging limit of $M_t$ must be contained in $\bigcup \Sigma_j$ if the sequence of $M_{s_i}$ converges as varifolds its support must be contained in $\bigcup_{j=1}^k\Sigma_j$, and so by ranging over $D, j$ and letting $\epsilon, \delta \to 0$ we thus get that if the sequence $M_{s_i}$ converges as varifolds the limit must be precisely $m_1\Sigma_1+\cdots+m_k\Sigma_k$ as claimed. \end{proof} Indeed if one of the minimal surfaces $\Sigma_i$ is merely stable but not strictly stable, the above contracting tubular neighborhood may not exist. An example of such a minimal surface can be found in \cite[Appendix B]{StevensSun24_LargeArea}. To wrap up this section we summarize some topological properties of the limit minimal surfaces in the case $M_t$ can be approximated by generic flows: \begin{prop}\label{limit_topology} Suppose that $M_t$ is an inner or outermost flow of $M$ or is a generic flow whose long term limit is nonempty, and let $m_1\Sigma_1+\cdots+m_k\Sigma_k$ be limiting minimal surfaces as in Theorem \ref{smoothfate}, and that $N$ is oriented implying 2--sidedness is equivalent to orientability. Then the following hold: \begin{enumerate} \item $m_1\Sigma_1+\cdots+m_k\Sigma_k$ in Corollary \ref{smoothfate} is homologous to $M$ in $H_2(N,\Z_2)$. \item If $\Sigma_j$ is $1$--sided, then $m_j$ is even. \item When all the $\Sigma_j$ are orientable $\sum\limits_{j=1}^k m_j\cdot \text{genus}(\Sigma_j)\leq \text{genus}(M)$. More generally \begin{equation} \sum\limits_{j, \text{ }\Sigma_j \text{ 2--sided}} m_i \cdot\text{genus}(\Sigma_j) + \sum\limits_{j, \text{ }\Sigma_j \text{ 1--sided}} \frac{m_j}{2} \cdot (\text{genus}(\Sigma_j) - 1)\leq \text{genus}(M) \end{equation} \end{enumerate} \end{prop} \begin{proof} By Lemma \ref{lem:PreserveHomology}, item (1) holds because this homology group is closed under limits. Now for the other parts we first note that because the convergence to the $\Sigma_j$ is multisheeted away from a discrete set of points, each sheet in the convergence corresponds to a connected component of the boundary of the tubular neighborhood of the limit (or, in other words, a connected component of its normal bundle). If $\Sigma_j$ is $1$-sided there is only one connected component, but about a point locally there are two components giving item (2). For item (3) by genus of nonoriented $\Sigma_j$ we refer to the demigenus and, since this may be somewhat less familiar, first we consider the case all the $\Sigma_j$ are orientable. In this case because all of the limit surfaces $\Sigma_j$ are orientable each sheet over $\Sigma_j$ in the convergence is homeomorphic to $\Sigma_j$ away from a finite set of points following the discussion above; then the claim follows easily from Lemma \ref{lem:GenusNonincreasing}. In case some are nonorientable we note by the classification of surfaces they must be homeomorphic to a connected sum of projective planes, say $k_j$ of them. The demigenus of such a connect sum is $k_j$ and the double cover of such a connect sum, corresponding to a single sheet in the convergence, happens to have genus $k_j - 1$. Because these numbers agree we obtain the formula in item (3). \end{proof} \section{The long--time limit is generically stable or empty: the proofs of Theorems \ref{Rc_thm} and \ref{pw_thm}} From the previous section we see that if the limit of an almost regular flow is nonempty it converges to a collection of smooth minimal surfaces, possibly with multiplicity. We also obtain some rough topological information about the limit. Even so it is still often desirable to specifically produce stable minimal surfaces because they carry more structure; for instance as a basic but important example we recall that stable surfaces in PSC $3$--manifolds must be diffeomorphic to $S^2$ or $\R P ^2$ but just in $S^3$ there are minimal surfaces of arbitrarily large genus. In this section, we show that with appropriately timed small isotopies, we can construct piecewise flows whose long-time limits are either empty or stable minimal surfaces. We consider first the Ricci positive case. As promised in the introduction, arguing in this case turns out to be simple because in this case the avoidance principle says two disjoint flows actually "repel" In fact we don't even need the regularity statements from the previous statement to proceed and indeed our argument works in all dimensions. \begin{proof}[proof of Theorem \ref{Rc_thm}] Where $M$ and $N$ are as in the statement of the theorem, suppose that the flow $M_t$ of $M$ doesn't go extinct in finite time. Since the flow is gradient of the area functional, we may pick a sequence of times $t_i \to \infty$ for which $\int_{M_{t_i}} H^2 \to 0$. After potentially passing to a subsequence the sequence $M_{t_i}$ of surfaces limits to a varifold $\Sigma$ which by the choice of $t_i$ must be stationary. Now consider a one sided perturbation $M'$ of $M$, for which say $d(M, M') > \delta$ for some $\delta > 0$. Since $N$ is compact and Ricci positive, there exists some $\Lambda > 0$ for which the Ricci curvature of $N$ is bounded below by $\Lambda$. Hence by the avoidance principle, Theorem \ref{ambient_avoidance} above, we have that the distance between $M'_t$ and $\Sigma$ is bounded below by $\delta e^{\Lambda t}$ for all times; because $N$ is compact so has bounded diameter for $t$ sufficiently large this gives that the level set flow $M'_t$ of $M'$ must be empty. \end{proof} We point out that as a consequence of the above, that in an orientable $3$--manifold every $2$--cycle can be represented by a properly embedded $2$--sided surface (see, for instance, Lemma 3.6 in \cite{Hatcher07_Notes3Mfd}), and Proposition \ref{limit_topology} we have another proof of the following classical fact: \begin{cor}\label{vanish} $H_{2}(N, \R)$ of a compact orientable $3$--manifold $N$ which admits a metric of positive Ricci curvature vanishes. \end{cor} Indeed, by Hodge theory and the Bochner formula $H^1(N)$ vanishes so the claim follows by Poincare duality. Now in the case $N$ doesn't have Ricci positive curvature the avoidance principle doesn't give as strong information and indeed it is easy to concoct examples where two disjoint flows may converge to each other as $t \to \infty$. For instance let $N$ be a compact manifold for which $H_2(N,\Z)$ is nontrivial (so cannot have positive Ricci curvature by the above). By the direct method in a nontrivial class of $H_2(N,\Z)$ there is an area minimizing surface $\Sigma$, and by perturbing the metric to be bumpy we may even suppose it is strictly area minimizing. By considering disjoint nearby surfaces (say graphical over $\Sigma$) their flows must converge to $\Sigma$ in the long term. With this in mind we now move on to Theorem \ref{pw_thm}. Recall the definition of piecewise generic flow, Definition \ref{pwflow_def}, and note that after potentially perturbing $M$ be a small amount without loss of generality we may suppose we are in the setting of Corollary \ref{smoothfate}. If $M_t$ goes extinct or all the minimal surfaces $\Sigma_j$ it subconverges to, defined as in the previous section, are stable minimal surfaces, then we are done. Otherwise, we apply isotopes to $M_{t}$ at a smooth time far along the flow which ensures the perturbation must not converge to the same surfaces after restarting the flow: \begin{lem}\label{perturb} Let $M$ be a $2$--sided properly embedded surface in a closed compact Riemannian $3$--manifold $(N^3,g)$, and suppose $M_t$ is an almost regular flow out of $M$ which can be approximated by generic flows (if not generic itself, the inner or outermost flow from $M$). Suppose $M_{t_i}$ converges to $m_1\Sigma_1+\cdots+m_k\Sigma_k$ as varifolds, and one of $\Sigma_j$ is unstable. Then for sufficiently large $t_i$ we may isotope $M_{t_i}$ so that any almost regular flow starting from the perturbed surface can not enter the tubular neighborhood of that unstable $\Sigma_j$. \end{lem} \begin{proof} Suppose $\Sigma_1$ is an unstable minimal surface, and in the following, we simply call it $\Sigma$. We first consider the situation that $\Sigma$ is unstable and $2$-sided. Then we can find a unit normal vector field $\nu$ over it, and we may assume $\varphi$ is the first eigenfunction of the linearized operator $L_\Sigma$. Let us use $(x,s)\in\Sigma\times(-\tau,\tau)$ to parametrize a tubular neighborhood $\cN_{\tau}$ of $\Sigma$ by $(x,s)\to\exp_x(s\varphi(x)\nu(x))$. We first define an isotope $\Psi_i$ of the ambient manifold as follows: we define the vector field $\pr_s$ in $\cN_{\tau/2}$ using the coordinate $(x,s)$, then we smoothly extend it to $0$ outside $\cN_{\tau}$ to get a smooth vector field $W$. Then we define $\{\Psi(w)\}_{w\in\R}$ to be the isotopy generated by the vector field $W$. Let us choose a very small $\delta>0$, such that $\cN_\delta(\Sigma)$ is completely contained in the domain given by the parametrization $(x,s)\to\exp_x(s\varphi(x)\nu(x))$ for $s\in(-\tau/10,\tau/10)$, and $\Psi(\tau/2)\cN_\delta(\Sigma)$ will completely leave $\cN_\delta(\Sigma)$. Following Lemma \ref{lem:LimNbhd}, we may assume $M_{t_i}\subset \bigcup_{j=1}^k\cN_{\delta}(\Sigma_j)$ for sufficiently large $t_i$. Then $\tilde M_{t_i}:=\Psi(\tau/2)M_{t_i}$ must be completely disjoint from $\cN_\delta(\Sigma)$, and this is a $2$--sided flow from which we may safely restart the flow from. Recalling that the support of Brakke flows are weak set flows, it's easy to see that any Brakke (and hence almost regular) flow starting from such $M' := \tilde M_{t_i}$ will never enter $\cN_{\delta}(\Sigma)$ again by the avoidance principle. To see this, note that the domain generated by the parametrization $(x,s)\to\exp_x(s\varphi(x)\nu(x))$ for $s\in(-\tau/10,\tau/10)$ has compact, smooth and outwardly mean convex boundary, with two components, which we denote by $\Sigma_{\pm \tau/10}$. By the strict mean convexity and the compactness there exists $\delta, \eta > 0$ so that the smooth mean curvature flow of the boundary exists on $[0, \delta)$ and the distance between $\Sigma_{\pm \tau/10}$ and $(\Sigma'_{\pm \tau/10})_\delta$ is bounded below by $\eta$. The avoidance principle says $(\Sigma_{\pm \tau/10})_t$ and $M'_t$ are disjoint on $[0, \delta)$, and because $\eta > 0$ we can iterate this argument over the intervals $[\delta, 2\delta)$ et. cetera to get the claim. Finally, we briefly describe the situation that $\Sigma$ is unstable and $1$--sided. The argument is similar but now the tubular neighborhood of $\Sigma$ is only parametrized by $\overline{\Sigma}\times(0,\tau)\bigcup \Sigma$, where $\overline{\Sigma}$ is the oriented double cover of $\Sigma$. In this case, we can still construct the isotopy to push the flow outside this tubular neighborhood, and verbatim arguments validate all the previous arguments. \end{proof} With this in hand, we are now ready to prove the main result of this section: \begin{proof}[proof of Theorem \ref{pw_thm}] We may clearly restart the flow after the perturbation in Lemma \ref{perturb}, but apriori the set of times for which we perturb may accumulate to a finite time; we claim that only finitely many perturbations are required though to ensure that afterwards none of the $\Sigma_j$ could be unstable. Suppose $\Sigma$ is an unstable minimal surface that the flow was perturbed to avoid by the lemma; then if the flow $M_t$ starting from this perturbation converges to another unstable minimal surface it must converge to one which is disjoint from $\Sigma$ and hence disjoint from the component of the flow which was just perturbed -- but by the avoidance principle argument above the flow of this connected component may never converge to any such unstable minimal surface. Now because we can take the parameter $\delta$ in the proof above as small as we wish we may suppose the area of the piecewise mean curvature flow is bounded by, say, twice the area of $M$ after the $k$-th perturbation/iteration of Lemma \ref{perturb} for any $k \in \mathbb{N}$. In particular by the clearing out lemma only finitely many components of $M_t$ at a given smooth time might go on to flow to a minimal surface, with the rest must quickly being exhausted; in fact by this we may crudely uniformly upper bound (independent of time) the number of such components. Applying the reasoning above to each such component then gives what we want.\end{proof} \section{Applications to existence questions for minimal surfaces: proofs of corollaries \ref{existence1}, \ref{existence2}, \ref{existence3}, and \ref{existence5}}\label{applications} In this section (aside from the last application, where we use Theorem \ref{longterm}), we consider various circumstances where we can show that the piecewise almost regular flows from the last section will be nonempty as $t \to \infty$ and say something about the topology of the limit. The first is on the existence of stable minimal spheres when $\pi_2(N) \neq 0$. \begin{proof}[proof of Corollary \ref{existence1}] First we note that by the sphere theorem in $3$--manifold topology \cite{Papakyriakopoulos57_Dehn} we can find, using $N$ is orientable, a homotopically nontrivial embedded $2$--sphere $M$. Of course $M$ is itself orientable, and we may use it as initial data in Theorem \ref{pw_thm}. By considering the approximating surgery flows $M_t$ during smooth times will consist of a collection (possibly multiple, if there are nontrivial neckpinches) of embedded $2$--spheres. Now, either the flow from the conclusion will terminate in finite time or converge to a stable minimal surface $\Sigma$ which, by the nature of the convergence discussed in Theorem \ref{smoothfate} and the proof of Proposition \ref{limit_topology}, must be a collection of embedded stable minimal spheres or projective planes because each component for large (smooth) times is covered $k \geq 1$ times by $M_t $ for large $t$, away from mentioned bridge points and each component of this is an embedded $2$--sphere. Picking any of these components then gives the statement. So, it suffices to rule out the flow going extinct in finite time. Supposing this is actually the case, considering the approximating surgery flow one can show that $M$ is isotopic to a "marble" tree as in \cite{BuzanoHaslhoferHershkovits21_Moduli} or similarly \cite{Mramor18_Finiteness}, taking some care because in our setting the flow may not be mean convex even if the high curvature regions are. Because in the mean curvature flow with surgery for large parameters the high curvature regions all bound handlebodies one can show such marble trees bound $3$--balls in $N$ giving that $M$ is in fact homotopically trivial, leading to a contradiction.\end{proof} We remind the reader the next corollary is on the existence of stable minimal tori in the aspherical case. Corollary \ref{existence2} in fact follows from Corollary \ref{existence3}: it isn't hard to see by Dehn's lemma and the trivial $\pi_2$ assumption that if the torus $T$ isn't algebraically incompressible then it bounds a 3 cycle. That being said we give the proof below for the sake of exposition, because it is somewhat simpler than the more general statement. \begin{proof}[proof of Corollary \ref{existence2}] Denote by $M$ the torus in the statement. Because the torus is orientable and $N$ is assumed oriented we have that $M$ is $2$--sided, so that we may apply Theorem \ref{pw_thm} to it. First we argue that the flow must not go extinct. Supposing it does, we note that approximating surgery flows go extinct as well. Considering one of these and denoting it by $S_t$, we note that high curvature regions which are discarded certainly bound $3$--cycles, homeomorphic to either $3$--balls or solid tori $\simeq S^1 \times D^2$, since different time slices along a smooth mean curvature flow are homologous this gives that $M$ is homologous to $S_t$ for all $t > 0$ and hence bounds a $3$--cycle, contradicting that $M$ is assumed to be homologically nontrivial. Next, we note that for any finite smooth time $t$, $M_t$ cannot be given by a union of embedded spheres $P_1, \ldots, P_k$. To see this, because $\pi_2(N)$ is trivial each of the $P_i$ are nullhomotopic and hence are the boundary of a $3$--cycle, so since $M_t$ is homologous to $M$ this gives that $M$ is homologically trivial giving a contradiction. In particular, a connected component of $M_t$ at a smooth time must be homeomorphic to a torus. Now, because the flow doesn't go extinct as before we may produce a sequence of times $t_i \to \infty$ for which $M_{t_i}$ converge to an embedded stable minimal surface $\Sigma$, and we claim one of the connected components of $\Sigma$ is a torus or a Klein bottle (which is the nonorientable surface covered by the torus) -- note this doesn't follow immediately from the case that a component of $M_t$ is a torus. For example, one can take a standardly embedded torus in $\R^3$, tighten the "donut hole" and then push the sides of the inner cylinder along the outer sides of the donut so in this fashion covering the sphere twice, away from bridge points. Of course if one of the components is a torus or Klein bottle we are done, so it suffices to rule out the case none of them are. By Proposition \ref{limit_topology} in this case all the components of $\Sigma$ are spheres or projective planes. Suppose first all the components are spheres: then by assumption they are nullhomotopic and so each of these spheres then could be homotoped to be within coordinate patches homeomorphic to open balls. As these patches are homeomorphic to an open ball in $\R^3$ all surfaces contained within are nullhomologous. Using that homotopic surfaces are homologous as before this implies $M$ itself is nullhomologous, giving a contradiction. If some of the components are projective planes we can argue similarly, because the boundary of their tubular neighborhoods are diffeomorphic to spheres. \end{proof} As the reader recalls from the introduction, the next statement concerns the existence of stable minimal surfaces in the presence of algebraically incompressible initial data, which is to say the inclusion map on fundamental groups injects; by Dehn's lemma, algebraically incompressible surfaces are incompressible in the standard sense i.e. that there are no compressing discs in $N$ along $M$. This topological assumption is arguably most naturally suited to how the topology of a flow may change across singularities, at least generic ones. This is because the effect of flowing through a neckpinch is essentially to glue a thickened disc along the flow of the corresponding domain, and so if the surface is incompressible only spheres can be pinched off. \begin{proof}[proof of Corollary \ref{existence3}] Supposing that $M = \iota(\Sigma)$ is as in the statement, we consider as before the approximating surgery flow $S_t$ to $M_t$. We first claim that at any smooth time a given connected component of it is either homeomorphic to either a sphere or $M$. Supposing not and let $T$ be the smallest time, necessarily right after a surgery, that this isn't the case -- for a fixed choice of approximating surgery flow this time is well defined. Then it's easy to see that the cross section curve $\gamma$ at one of the necks where surgery was performed on $S_T^{-}$ (the presurgery surface) is homotopically nontrivial in $M_T$. The surgery shows that $\gamma$ bounds a disc in $N$ though, so that $S_T^{-}$ is not algebraically incompressible. Since $T$ was the least such time this implies that $M$ itself is not algebraically incompressible, which gives a contradiction. Although it is the case that the flow preserves the topology of $M$ in this sense it could be the case that there is strict genus drop in the limit as $t \to \infty$. In the following we focus only on the connected component $\tilde{M_t}$ of $M_t$ homeomorphic to $M$, and denote its corresponding limiting stable minimal surface $\Sigma$; the reason this suffices for us because, by the $\pi_2$ triviality assumption and that the other components pinched off the flow will be spheres, at any given time $M_t$ will be homotopic to just this component. Now, if the convergence is with multiplicity one then it will be the case $\Sigma$ is orientable and isotopic to $M$. Supposing this is not the case, then from the proof of Theorem \ref{pw_thm} we have that either $\Sigma$ is nonorientable and $M_t$ converges to $\Sigma$ with multiplicity 2, or $M_t \to \Sigma$ with some multiplicity $m$ and there is at least one bridge point $p$, within which for some large time $s$ and some small $r > 0$ we have $M_s \cap B(p,r)$ has less than $m$ components -- our task is to rule out this latter case. Taking $r$ slightly larger if necessary $M \cap (B(p,2r) \setminus B(p, r))$ is comprised of $m$ disjoint annuli, where $m$ is the multiplicity of convergence; after a slight isotopy we may suppose these annuli $A_i$ lay in parallel planes and so may order them from "top" to "bottom" in the obvious sense, with $A_1$ being the top annulus. Without loss of generality the top annulus at the point $p$ is nontrivial in the sense it doesn't border a disc in $M_s \cap B(p,r)$; if not below we can consider the next lowest disc and so on. Considering one of the boundary components $\gamma$ of $A_1$, we may clearly find an embedded disc $D$ in $N$ disjoint from $M_ s$ with boundary $\gamma$. If $\gamma$ is nonseparating or splits $M_s$ into two components of genus greater than zero then $D$ will be a compressing disc because $\gamma$ will be homotopically nontrivial in $M$, which gives a contradiction. If the number of bridge points connected to the top sheet (i.e. where $B(p_i, r_i)$ are the bridge points and neighborhoods, the connected component of $M \setminus \cup_i B(p_i, r_i)$ containing $A_1$) is greater than one then $\gamma$ is nonseparating, so that we are done; let us suppose that $p$ is the only bridge point where the top sheet connects to the lower sheets then. If $\Sigma$ is not an embedded sphere or projective plane then $\gamma$ would disconnect $M$ into two components of nonzero genus, completing the argument in this case. If $\Sigma$ is a sphere or projective plane a bit more arguing is required. As before first suppose that $\Sigma$ is a sphere. Then because $N$ has trivial $\pi_2$, we may homotope $\Sigma$ and hence $M_s$ to lay in a small coordinate chart; of course because $\R^3$ is contractible $M_s$ in this case cannot be incompressible. If it is a projective plane again the boundary of its tubular neighborhood is a sphere, and we may proceed as in the first case. \end{proof} Leaving the world of stable minimal surfaces we now consider the use of our first convergence statement, Theorem \ref{longterm}, to essentially replace the tightening process in minmax. Recalling that in the setting of Corollary \ref{existence5} setting $N \simeq S^3$ we may consider the standard one parameter sweepout $\Sigma_s$ of it by $2$--spheres, where there are two singular values of the sweepout that are single points corresponding to the north and south poles under a fixed choice of diffeomorphism from $N$ to $S^3$. Perhaps the most natural thing to do then is to consider the flow of the sweepout under the flow, by which we mean the family of the flows $(\Sigma_s)_t$ of the individual leaves $\Sigma_s$. Recalling that the width of any sweepout of $S^3$ by spheres is positive by the isoperimetric inequality, we see if we could show that if the flow will remain a sweepout, then Theorem \ref{longterm} says that the flow of at least one leaf will not go extinct, and because $S^3$ is simply connected and embedded hypersurfaces in such spaces are orientable Proposition \ref{limit_topology} gives the limit will be an embedded minimal sphere, as usual potentially with multiplicity. There are some issues with the approach though, unfortunately. The biggest issue in general appears to be that the level set flow of some of the leaves may fatten, but in our particular setting this may be ruled out as we discuss below. There is also an issue with preserving the typical regularity assumptions one make in the definition of sweepout. Recall that a sweepout of $N$ (here, $1$--parameter) is a family of surfaces $\Sigma_s$, $s \in [0,1]$, which for finitely many $s$ is singular in a finite set of points and smooth elsewhere and whose union is all of $N$. Now, we know that for a fixed leaf $\Sigma_s$ the flow $(\Sigma_s)_t$ will be smooth for almost all times $t$, but because we wish to consider the flow of the family, parameterized over the whole unit interval (which is uncountable so Baire category theorem cannot apply), it doesn't seem clear if we can show the flow of the whole family will be smooth for almost all or even many times. The current state of the art also doesn't rule out the size of the singular set of a flow being greater than a finite union of points, either. \begin{proof}[proof of Corollary \ref{existence5}] With respect to the discussion above one may be able to proceed by generalizing the results of minmax theory to more general sweepouts including those "generated" by the flow but instead we will take a somewhat more hands on approach. Denote by $E(s)$ the extinction time of the flow of $\Sigma_s$; if this never occurs of course we set $E(s) = \infty$ so it is a function into the extended reals. We next claim that $E(s)$ is sequentially continuous. To see this, we first note that because each of the $\Sigma_s$ are spheres their singularities under the flow must be mean convex and so each of their flows is nonfattening -- implying for instance that the almost regular flows $(\Sigma_{s_i})_t $ are well defined. Fixing an $s_0 \in [0,1]$, nonfattening also implies that for a sequence $s_i \to s_0$, after potentially passing to a subsequence by Brakke compactness, $\lim\limits_{s_j \to s_0} (\Sigma_{s_i})_t \to (\Sigma_{s_0})_t$ as Brakke flows because $\lim\limits_{s_j \to s_0} (\Sigma_{s_i})_t $ is a weak flow associated to $\Sigma_{s_0}$. With this in mind suppose then that there was a sequence $s_i \to s_0$ and $\epsilon > 0$ for which $E(s_0) < A - \epsilon$, where $A = \lim\limits_{i \to \infty} E(s_i)$. At time $T = E(s_0)$, by Colding and Minicozzi \cite{ColdingMinicozzi16_SingularSet} $(\Sigma_{s_0})_T$ is the union of a finite number of compact embedded Lipschitz curves corresponding to cylindrical singularities along with a countable collection of points corresponding to spherical singularities. This gives that for $i$ sufficiently large we make take the mass of $(\Sigma_{s_i})_T$ to be as small as we like, so in particular the clearing out lemma says these flows must go extinct before $t = A - \epsilon/2$, giving a contradiction to show that $E(s)$ is upper semicontinuous. Similarly $E(s)$ is lower semicontinuous, giving our claim of sequential continuity. Because $E(s)$ is sequentially continuous, by the compactness of the unit interval $\sup\limits_{s \in [0,1]} E(s) = T$ is obtained for some time $s_0$, recycling notation. Now as above we have $(\Sigma_{s_0})_T$ must consist of the union of a finite number of compact embedded Lipschitz curves corresponding to cylindrical singularities along with a countable collection of points corresponding to spherical singularities. Picking some smooth time $t^*$ right before $T$, the resolution of the mean convex neighborhood conjecture gives that $(\Sigma_{s_0})_{t_*}$ is mean convex and after rescaling is locally modeled on bowl solitons, round cylinders, or round spheres. To see why this is useful, first note that $s_0 \in (0,1)$: this is because for $s$ very near $0$ or $1$ that $\Sigma_s$ are approximately small concentric round spheres contained in a coordinate chart so that $\lim\limits_{s \to 0, 1} E(s) = 0$; on the other hand for a fixed smooth $\Sigma_s$ its smooth flow exists for a short but strictly positive time. Secondly, the flows $(\Sigma_{s})_t$ are pairwise disjoint in $s$ because $\Sigma_s$ are and almost regular flows are weak set flows. In particular, for $s - s_0$ sufficiently small $s$ Brakke regularity says that $(\Sigma_s)_{t^*}$ is mean convex and a graph over $(\Sigma_{s_0})_{t_*}$ laying on one side or the other of it, and we can find $s$ so that the flow at time $t^*$ is on either side. Hence we see that for $s$ either slightly greater or smaller than $s_0$ $(\Sigma_{s})_T$ must not be singular, or in other words that their flow lasts at least slightly after time $T$ giving a contradiction to the definition of $T$. \end{proof} \bibliographystyle{alpha} \bibliography{GMT} \end{document} The following corollary directly results from the fact that the varifold convergence can only lead to genus dropping. \begin{cor} Suppose $(N,g)$ is a closed $3$-manifold with bumpy metric. Then for any homology class $\sigma\in H_2(N,\Z)$, there exists a finitely many disjoint closed embedded stable minimal surfaces $\Sigma_1,\cdots,\Sigma_k$ such that $[\Sigma_1]+\cdots+[\Sigma_k]=\sigma$. Moreover, if $\sigma$ can be represented by a closed embedded surface with genus $g$, then $\sum_{j=1}^k \text{genus}(\Sigma_i)\leq g$. \end{cor} In particular, if we run the piecewise flow starting from a closed embedded surface in the homology class with the least genus, the limit surface must have the same number of genus, counting multiplicity. \begin{cor}\label{cor:StableGenus} Suppose $(N,g)$ is a closed $3$-manifold with bumpy metric. Then for any homology class $\sigma\in H_2(N,\Z)$ there exists a finitely many disjoint closed embedded stable minimal surfaces $\Sigma_1,\cdots,\Sigma_k$ such that \[ \sum_{j=1}^k m_i\cdot\text{genus}(\Sigma_i) = \min_{\Sigma\in \sigma} \text{genus}(\Sigma). \] \end{cor} \begin{rem} Corollary \ref{cor:StableGenus} seems to be a feature that can only be detected by flows. In fact, the minimizing surface in a homology class may have genus strictly larger than the minimizing genus representative in the homology class. Consider a manifold is topologically $S^2\times S^1$. We equip it with a $SO(2)$-invariant metric where the $SO(2)$ acting on the $S^2$ factor. Then after taking the quotient, the manifold becomes $[-1,1]\times S^1$, and any $SO(2)$-invariant surface can be represented as a curve in $[-1,1]\times S^1$, and the area of the surface equals a weighted length of the curve. Now we equip $[-1,-1/2]\times S^1$ and $[1/2,1]\times S^1$ with the standard round metrics on $S^2$ product with $S^1$, and we equip the rest part with a metric such that a closed loop $\ell$ has the minimizing weighted length and the weighted length is strictly less than the weighted length of $[-1,-1/2]\times \{*\}$ plus $[1/2,1]\times \{*\}$. Consider the homology class $2[S^2\times\{*\}]$. The minimizing surface $\Sigma$ in this homology class must be $SO(2)$-invariant as well. Otherwise, one can apply $SO(2)$ action to $\Sigma$ to get another surface $\Sigma'$, which is also minimizing, but $\Sigma'$ intersects $\Sigma$. If the rotation is small enough, $\Sigma'$ would be a graph over $\Sigma$, and we can take the smaller area pieces of $\Sigma$ and $\Sigma'$ to get another surface $\Sigma''$ with possibly smaller area, but in the same homology class. If $\Sigma''$ has area strictly smaller than $\Sigma$, then we get a contradiction; if $\Sigma''$ has area equal to $\Sigma$, then it is also an area minimizer in the homology class. However, it is not regular as $\Sigma$ and $\Sigma'$ intersect, contradicting the fact that area-minimizing surfaces are regular in $ 3$ dimensions. Therefore, because $\ell$ is the weighted length minimizing curve in the quotient space, $\ell\times S^1$ is a minimizing surface in the homology class $2[S^2\times\{*\}]$. It has genus $1$. However, $2[S^2\times\{*\}]$ can be realized by two disconnected spheres, which have genus $0$. \end{rem} Perturbation argument in version 1 As discussed before, lemma \ref{lbound} shows that for a given genus $g$ the area bound $m$ is strictly positive. Now suppose $M \subset N$ is as in the statement of the theorem. By an arbitrarily small perturbation as discussed in the preliminaries and used in the section above we may suppose that $M_t$ develops only mean convex singularities. As we showed in the section above, in particular by lemmas \ref{tabounds}, \ref{decay} and corollary \ref{smoothfate} we know that either the weak flow $M_t$ will go extinct in finite time or there is a sequence of times $t_i \to \infty$ for which $M_{t_i}$ converges to a smooth orientable minimal surface $\Sigma$. Because the area of $M$ is less than $2m$ the convergence is with multiplicity one, so for $t_i$ sufficiently large by Brakke regularity \cite{W1} $M_{t_i}$ can be written as the graph of a smooth function $u$ (depending on $i$, but this will be supressed) over $\Sigma$. Of course, if $\Sigma$ is stable then we are finished. Now, imaginably between the times $t_i$ the flow $M_t$ might not be graphical over $\Sigma$ but its a consequence by Brakke regularity that for small forward times from $t_i$ that it is and we write $u(x,t)$ for the corresponding function; since $\Sigma$ is minimal and the convergence is smooth we also see for $i$ large enough $M_t$ can be guaranteed to be written graphically over $\Sigma$ for as long a time as we wish. Because the graph of $u$ moves by the mean curvature flow, it turns out that $u$ solves the following PDE: \begin{equation} \frac{du}{dt} = L_\Sigma u + E(u) \end{equation} where $L_\Sigma = \Delta + |A|^2 + \Ric(\nu,\nu)$ is the Jacobi operator of $\Sigma$, and $E(u)$ is an error term which is $O(||u||_{C^2(\Sigma)})$, proceeding as in section 2.2 of \cite{CHL}. Of course $L_\Sigma$ is a self adjoint operator; denote by $\lambda_1 < \lambda_2 \leq \lambda_3 \leq \cdots \leq \lambda_I < 0 < \lambda_{I + 1} \leq \cdots$ the eigenvalues of $L_\Sigma$ where $I$ is the index of $\Sigma$. Denoting the corresponding eigenfunctions by $f_i(x)$, the functions $e^{-\lambda_i t}f_i(x)$ each solve the linear evolution equation $\frac{du}{dt} = L_\Sigma u$. With this in mind consider the fourier series representation $u(x,t_i + 1) = \sum_{i = 1}^\infty c_i f_i$. The solution $\tilde{u}$ to the linearized flow with this as initial data is given by $\tilde{u}(x,t) = \sum_{i = 1}^\infty c_i e^{-\lambda_i t} f_i$. This well approximates the actual solution $u$ to the flow: \begin{lem} For any $s > 1$, there exists $\epsilon_0, C > 0$ so that when $||u(\cdot, T + 1)||_{C^{1}}$ is sufficiently small \begin{equation} || u( \cdot, T + 1 + t) - \widetilde{u}( \cdot, t)||_{C^{2, \alpha}} \leq C ||u||_{C^{1}}^{1 + \epsilon_0} \end{equation} on the interval $[1,s]$. \end{lem} Because $M_{t_i}$ converges to $\Sigma$ note $||u(\cdot ,t_i )||_{C^1}$ and hence, since $\Sigma$ is minimal, $||u(\cdot ,t_i + 1)||_{C^1} \to 0$ as $t_i \to \infty$. Note in particular this says that in the above $c_i = 0$ for $1 \leq i \leq I$ by the lemma. Recalling that the first eigenfunction of $L_\Sigma$ has a sign, the idea is that a sufficiently small one sided perturbation $M'$ of $M$ will also flow to a graph over $\Sigma$ of a function $w$ at a fixed time $t_i$, but in this case the Fourier expansion of $w(x,t_i + 1)$ will have $c_1 \neq 0$. If the flow is smooth up to time $t_i $ that $M'_{t_i}$ will be a graph over $\Sigma$ is a consequence of smooth dependence on initial conditions but its also true in the presence of singularities by the nonfattening of $M_t$. Now we claim that $M'_t$ cannot converge to $\Sigma$: suppose on the contrary it did. Then we can apply the lemma above with $s$ as large as we wish (using $i$ sufficiently large) but $c_1 e^{-\lambda_1 t}$ grows exponentially quickly. Because $f_1$ is the first eigenfunction of $L_\Sigma$ which corresponds to the second derivative of area, the area of $M_t'$ must be strictly less than the area of $\Sigma$ by similar arguing -- in particular very small perturbations of M' will never flow back to $\Sigma$. By the compactness of minimal surfaces in a bumpy 3 manifold of bounded genus and area and the Lojasiewicz-Simon inequality \cite{LS} there are only finitely many possible areas for minimal surfaces under these conditions so we can repeat the argument above finitely many times to reach the conclusion. Pilgrim process Because there is no Huisken's monotonicity formula in a general closed manifold, this uniform area growth bound is not necessarily true. On the other hand, we can still use Ilmanen's argument, after applying further isotopes, as follows. First, suppose the varifold limit of $M_{t_i}$ is $k\Sigma$, a priori $\Sigma$ is not necessarily a smoothly embedded minimal surface, but it is a stationary varifold. In particular, for any $p\in \supp\Sigma$ and sufficiently small $r>0$, when $t_i$ is sufficiently large, $\area(M_{t_i}\cap B_r(p))\leq (k+1)\theta(\Sigma,p)\pi r^2$. Here $\theta(\Sigma,p)$ is the density of $\Sigma$ at $p$. Next, we show that $\Sigma$ has finite density, namely there exists $m\in\Z_+$ such that $\theta(\Sigma,p)\leq m$ for all $p\in\Sigma$. In fact, by monotonicity formula in Riemannian manifold (\textcolor{violet}{see \href{https://web.math.princeton.edu/~rcabral/pdfs/minimalsurfaces.pdf}{this note, Theorem 7.11}. Is there a better reference?}), there exists $r_0>0$, $A>0$ only depending on $(N,g)$ such that for any $p\in \Sigma$ and $r<r_0$, $|B_r(p)\cap \Sigma|\geq e^{-Ar^2}\theta(\Sigma,p)$. But $\area(B_r\cap M_{t_i})\to k|B_r(p)\cap \Sigma|$ as $t_i\to\infty$, so we must have an uniform upper bound on $\theta(\Sigma,p)$. Suppose $\theta(\Sigma,p)\leq m$ for all $p\in\Sigma$. Then there exists $\eps_0$ depending on $N,m$ and $k$ such that if $\int_{M_{t_i}\cap B_r(p)}|A|^2<\eps<\eps_0$, $M_{t_i}\cap B_{r/2}(p)$ would be a disjoint union of topological disks $D_1,\cdots,D_\ell$, and away from a collection of closed disks $P_1,\cdots,P_N$ (Simon called them ``pimples''), $D_i$'s are graphs over some plane, and the gradient of the graphs are controlled by $\eps^{1/2}$. The sum of the diameters of $P_i$'s are bounded by $C\eps^{1/2}r$. Now we isotope $M_{t_i}$ to ``pilgrim'' the pimples, so that they will only contribute to area by $\pi(C\eps^{1/2}r)^2$. This procedure (and the terminology ``pilgrim'') has been discussed in \cite{??? Almgren-Simon}. In fact, this can be done by finding little round loops on the graph regions of $D_1,\cdots, D_\ell$ ($\ell$ is uniformly bounded by $M_0,m$) with total length bounded by $C'\cdot C\eps^{1/2}r$ where $C'$ only relies on the geometry of $(N,g)$, so that the pimples lying inside each loop. These round loops can be chosen to be the intersection of geodesic balls in $(N,g)$ with $M_{t_i}$. Then we can use isotopes to shrink the pimple to a minimal disk whose boundary is the loop. It is clear that after this process, each pimple will have only area $C'\pi (r')^2$ if the geodesic ball has radius $r'$. Let us call $\tilde{M}_{t_i}$ which is the surface $M_{t_i}$ after this isotopes procedure. Now we can repeat the proof of Ilmanen as we have the desired area growth estimate for each disk $D_1,\cdots, D_\ell$ in the above setting. Then after passing to limit, $\tilde{M}_{t_i}$ converges to an embedded minimal surface $\tilde \Sigma$ except at finitely many points where curvature concentrates. Again by the removability of isolated singularities of minimal surfaces, we can show that $\tilde \Sigma$ is a smoothly embedded minimal surface.
2412.03536v1
http://arxiv.org/abs/2412.03536v1
On unique continuation in measure for fractional heat equations
\documentclass[11pt]{amsart} \usepackage{amssymb,amsmath,epsfig,mathrsfs, enumerate, xparse, mathtools} \usepackage[pagewise]{lineno} \usepackage[nodisplayskipstretch]{setspace} \setstretch{1.5} \usepackage{graphicx}\usepackage[normalem]{ulem} \usepackage{fancyhdr} \pagestyle{fancy} \fancyhead[RO,LE]{\small\thepage} \fancyhead[LO]{\small \emph{\nouppercase{\rightmark}}} \fancyhead[RE]{\small \emph{\nouppercase{\rightmark}}} \fancyfoot[L,R,C]{} \renewcommand{\headrulewidth}{1pt} \usepackage[margin=2.5cm]{geometry} \usepackage[pagebackref,colorlinks=true,linkcolor=blue,citecolor=blue]{hyperref} \hypersetup{ colorlinks = true, urlcolor = blue, linkcolor = blue, citecolor = red , bookmarksopen=true } \theoremstyle{plain} \newtheorem{thrm}{Theorem}[section] \newtheorem{lemma}[thrm]{Lemma} \newtheorem{prop}[thrm]{Proposition} \newtheorem{cor}[thrm]{Corollary} \newtheorem{rmrk}[thrm]{Remark} \newtheorem{dfn}[thrm]{Definition} \newtheorem{prob}[thrm]{Problem} \newtheorem*{hyp}{HYPOTHESIS} \setlength{\textheight}{8.7in} \allowdisplaybreaks \begin{document} \newcommand{\sn}{\mathbb{S}^{n-1}} \newcommand{\SL}{\mathcal L^{1,p}( D)} \newcommand{\Lp}{L^p( Dega)} \newcommand{\py}{ \partial_z^a} \newcommand{\La}{\mathscr{L}_a} \newcommand{\CO}{C^\infty_0( \Omega)} \newcommand{\Rn}{\mathbb R^n} \newcommand{\Rm}{\mathbb R^m} \newcommand{\R}{\mathbb R} \newcommand{\Om}{\Omega} \newcommand{\Hn}{\mathbb H^n} \newcommand{\aB}{\alpha B} \newcommand{\eps}{\ve} \newcommand{\BVX}{BV_X(\Omega)} \newcommand{\p}{\partial} \newcommand{\IO}{\int_\Omega} \newcommand{\bG}{\boldsymbol{G}} \newcommand{\bg}{\mathfrak g} \newcommand{\bz}{\mathfrak z} \newcommand{\bv}{\mathfrak v} \newcommand{\Bux}{\mbox{Box}} \newcommand{\e}{\ve} \newcommand{\X}{\mathcal X} \newcommand{\Y}{\mathcal Y} \newcommand{\W}{\mathcal W} \newcommand{\la}{\lambda} \newcommand{\vf}{\varphi} \newcommand{\rhh}{|\nabla_H \rho|} \newcommand{\Ba}{\mathcal{B}_\beta} \newcommand{\Za}{Z_\beta} \newcommand{\ra}{\rho_\beta} \newcommand{\n}{\nabla} \newcommand{\vt}{\vartheta} \newcommand{\its}{\int_{\{y=0\}}} \numberwithin{equation}{section} \newcommand{\RN} {\mathbb{R}^N} \newcommand{\Sob}{S^{1,p}(\Omega)} \newcommand{\Dxk}{\frac{\partial}{\partial x_k}} \newcommand{\Co}{C^\infty_0(\Omega)} \newcommand{\Je}{J_\ve} \newcommand{\beq}{\begin{equation}} \newcommand{\bea}[1]{\begin{array}{#1} } \newcommand{\eeq}{ \end{equation}} \newcommand{\ea}{ \end{array}} \newcommand{\eh}{\ve h} \newcommand{\Dxi}{\frac{\partial}{\partial x_{i}}} \newcommand{\Dyi}{\frac{\partial}{\partial y_{i}}} \newcommand{\Dt}{\frac{\partial}{\partial t}} \newcommand{\aBa}{(\alpha+1)B} \newcommand{\GF}{\psi^{1+\frac{1}{2\alpha}}} \newcommand{\GS}{\psi^{\frac12}} \newcommand{\HFF}{\frac{\psi}{\rho}} \newcommand{\HSS}{\frac{\psi}{\rho}} \newcommand{\HFS}{\rho\psi^{\frac12-\frac{1}{2\alpha}}} \newcommand{\HSF}{\frac{\psi^{\frac32+\frac{1}{2\alpha}}}{\rho}} \newcommand{\AF}{\rho} \newcommand{\AR}{\rho{\psi}^{\frac{1}{2}+\frac{1}{2\alpha}}} \newcommand{\PF}{\alpha\frac{\psi}{|x|}} \newcommand{\PS}{\alpha\frac{\psi}{\rho}} \newcommand{\ds}{\displaystyle} \newcommand{\Zt}{{\mathcal Z}^{t}} \newcommand{\XPSI}{2\alpha\psi \begin{pmatrix} \frac{x}{\left< x \right>^2}\\ 0 \end{pmatrix} - 2\alpha\frac{{\psi}^2}{\rho^2}\begin{pmatrix} x \\ (\alpha +1)|x|^{-\alpha}y \end{pmatrix}} \newcommand{\Z}{ \begin{pmatrix} x \\ (\alpha + 1)|x|^{-\alpha}y \end{pmatrix} } \newcommand{\ZZ}{ \begin{pmatrix} xx^{t} & (\alpha + 1)|x|^{-\alpha}x y^{t}\\ (\alpha + 1)|x|^{-\alpha}x^{t} y & (\alpha + 1)^2 |x|^{-2\alpha}yy^{t}\end{pmatrix}} \newcommand{\norm}[1]{\lVert#1 \rVert} \newcommand{\ve}{\varepsilon} \newcommand{\D}{\operatorname{div}} \newcommand{\G}{\mathscr{G}} \newcommand{\sa}{\langle} \newcommand{\da}{\rangle} \title[ucp in measure etc]{ On unique continuation in measure for fractional heat equations} \author{Agnid Banerjee} \address{School of Mathematical and Statistical Sciences\\ Arizona State University}\email[Agnid Banerjee]{[email protected]} \author{Nicola Garofalo} \address{School of Mathematical and Statistical Sciences\\ Arizona State University}\email[Nicola Garofalo]{[email protected]} \keywords{} \subjclass{35A02, 35B60, 35K05} \maketitle \begin{abstract} We prove a theorem of unique continuation in measure for nonlocal equations of the type $(\partial_t - \Delta)^s u= V(x,t) u$, for $0<s <1$. Our main result, Theorem \ref{main}, establishes a delicate nonlocal counterpart of the unique continuation in measure for the local case $s=1$. \end{abstract} \section{Introduction and statement of main result} The problem of unique continuation occupies a central position in the analysis of partial differential equations. A fundamental question in the subject is whether the trivial solution is the only one that can vanish to infinite order at one point. When this is the case, one says that the strong unique continuation property holds. In the study of observability inequalities and/or null-controllability of parabolic evolutions over measurable sets, a different type of unique continuation becomes relevant: can a nontrivial solution vanish on a subset of positive measure? If this cannot happen, one says that the relevant differential operator has the unique continuation property in measure, see for instance \cite{EMZ} and the references therein. The primary objective of this paper is to establish a theorem of unique continuation in measure for solutions to the nonlocal parabolic equation \begin{equation}\label{e0} H^s u(x,t) + V(x,t) u(x,t) = 0,\ \ \ \ \ \ \ \ 0<s<1, \end{equation} in the space-time cylinder $B_1 \times (-1, 0]\subset \Rn_x\times \R_t$. In \eqref{e0} we have denoted by $H^s = (\p_t - \Delta_x)^s$ the fractional power of the heat operator $H = \p_t - \Delta_x$ in $\R^{n+1} = \Rn_x \times \R_t$, and on the potential $V$ suitable assumptions will be specified in \eqref{vassump} below. Throughout this paper, for a function $f:\R^{n+1}\to \mathbb C$, we indicate with \[ \hat f(\xi,\sigma) = \int_{\R^{n+1}} e^{-2\pi i (\sa\xi,x\da + \sigma t)} f(x,t) dx dt \] its Fourier transform. Then the action of $H^s$ on a function $f\in \mathscr S(\R^{n+1})$ is defined by the formula \begin{equation}\label{sHft} \widehat{H^s f}(\xi,\sigma) = (4\pi^2 |\xi|^2 + 2\pi i \sigma)^s\ \hat f(\xi,\sigma), \end{equation} with the understanding that we have chosen the principal branch of the complex function $z\to z^s$. As it is well-known, the nonlocal operator $H^s$ is alternatively given by the formula \[ H^{s} f(x,t) = - \frac{s}{\Gamma(1-s)} \int_0^\infty \frac{1}{\tau^{1+s}} \big(P^H_\tau f(x,t) - f(x,t)\big) d\tau, \] where we have let \[ P^H_\tau f(x,t) = (4\pi \tau)^{-\frac n2} \int_{\Rn} e^{-\frac{|x-y|^2}{4\tau}} f(y,t-\tau) dy. \] The domain of the operator $H^s$ will be denoted by \[ \operatorname{Dom}(H^s) = \{u\in L^2(\Rn \times \R)\mid H^s u \in L^2(\Rn \times \R)\}. \] We say that a function $u\in \operatorname{Dom}(H^s)$ solves \eqref{e0} in an open set $\Om\subset \R^{n+1}$ if the equation is satisfied for a.e. point $(x,t)\in \Om$. For a measurable set $E \subset \Rn$, we will indicate by $|E|$ its $n$-dimensional Lebesgue measure. The ball in $\Rn$ centered at the origin and having radius $r>0$ will be denoted by $B_r$. Our main result is the following. \begin{thrm}\label{main} Let $u \in \operatorname{Dom}(H^s)$ solve \eqref{e0} in the cylinder $B_1 \times (-1, 0]$. Assume that $u$ vanishes in $F \times (-1, 0]$, for some measurable set $F \subset B_1$ with $|F|>0$. Then $u \equiv 0$ in $\Rn \times (-1,0]$. \end{thrm} The proof of Theorem \ref{main} will be presented in Section \ref{s:m}. We mention that the main new difficulty with this result is the treatment of the nonlocal operator $H^s$. In the local case, in fact, by an application of the Poincar\'e and energy inequalities, it is easy to show that at any Lebesgue point of its zero set, a solution of the relevant differential operator must vanish to infinite order. This reduces the proof to having the strong unique continuation property for the local operator in question, see for instance \cite{DG, EMZ, Ru2, BZ}. In the framework of the present paper, a corresponding space-like strong unique continuation result has been recently established in \cite{ABDG}, see Theorem \ref{main1} below. However, since the energy inequality for fractional equations has a nonlocal character, a straightforward adaptation of the above mentioned local arguments is all but obvious. This aspect was already mentioned in the introduction of \cite{FF}, where the authors derived the time-independent counterpart of Theorem \ref{main} by analysing the local asymptotic of solutions to the corresponding Caffarelli-Silvestre extension problem for $(-\Delta)^s$. In this note, we present a different approach which, in fact, does reduce the property of unique continuation in measure to that of strong unique continuation. The novel aspects of our proof consist in exploiting in a subtle way some fundamental properties of the solution of the extension problem associated with \eqref{e0}. One of them is Theorem \ref{extmain}, which we use to establish the crucial compactness Lemma \ref{L:pos}. Another important ingredient is the conditional doubling property in Theorem \ref{doub}, which we use in multiple ways in the proof of Theorem \ref{main}. Combining these tools with the trace interpolation inequality in Lemma \ref{L:inter1} (and the Poincar\'e inequality in Lemma \ref{po}), we are able to prove that, if $u$ in Theorem \ref{main} does not vanish identically in the cylinder $B_1 \times (-1, 0]$, then at any Lebesgue point $x_0$ of the set $F$, the following inequality holds for any $\ve>0$ and for all $0<r<r_\ve$ \begin{equation*} \int_{Q_r(x_0, t_0)} u^2 dx dt \leq C \ve^{2/n} \int_{Q_{2r}(x_0, t_0)} u^2 dx dt, \end{equation*} see \eqref{e5} below. This critical information proves that $u$ vanishes to infinite order at $(x_0,t_0)$. We can thus invoke the nonlocal space-like strong unique continuation property in Theorem \ref{main1} to finally infer that it must be $u(\cdot, t_0) \equiv 0$ in $\Rn$. This contradicts the initial assumption that $u\not\equiv 0$ in $B_1 \times (-1, 0]$, thus allowing us to spread the zero set of $u$. The organisation of the paper is as follows. In section \ref{s:n} we introduce some notation and gather the above mentioned results that are used in the proof of Theorem \ref{main} in section \ref{s:m}. In closing, we mention that for the existing literature on strong unique continuation for $(-\Delta)^s$ and its parabolic counterpart $(\partial_t - \Delta)^s$, the reader should see \cite{FF, Ru1, BG, FPS, BGh, AT}. \section{Notations and Preliminaries}\label{s:n} In this section we introduce the relevant notation and gather some auxiliary results that will be useful in the rest of the paper. Generic points in $\Rn \times \R$ will be denoted by $(x_0, t_0), (x,t)$, etc. For an open set $\Omega\subset \Rn_x\times \R_t$, we indicate with $C_0^{\infty}(\Omega)$ the set of compactly supported, smooth functions in $\Om$. We also denote by $H^{\alpha}(\Omega)$ the non-isotropic parabolic H\"older space, see \cite[p. 46]{Li}. Given the nonlocal operator \eqref{sHft}, the parabolic Sobolev space of fractional order $2s$ is \begin{align}\label{dom} \mathscr H^{2s} & = \operatorname{Dom}(H^s) = \{f\in \mathscr S'(\R^{n+1})\mid f, H^s f \in L^2(\R^{n+1})\} \\ & = \{f\in L^2(\R^{n+1})\mid (\xi,\sigma) \to (4\pi^2 |\xi|^2 + 2\pi i \sigma)^s \hat f(\xi,\sigma)\in L^2(\R^{n+1})\}. \notag \end{align} Hereafter in this paper, we assume that $u\in \mathscr H^{2s}$ solves the equation \eqref{e0}, where on the potential $V$ we make the hypothesis that for some $K>0$ one has \begin{equation}\label{vassump} ||V||_{C^1(B_1 \times (-1, 0])} \leq K, \ \text{if}\ s \in [1/2, 1),\ \ \ \ \ ||V||_{C^2(B_1 \times (-1, 0])} \leq K,\ \text{for}\ s \in (0,1/2). \end{equation} Under such assumptions on $u$ and $V$, various basic results hold. In order to state them we next recall the counterpart for the parabolic nonlocal operator $H^s$ of the Caffarelli-Silvestre extension problem in \cite{CS}. When $s=1/2$ such problem was originally introduced and solved by F. Jones in \cite{Jr}, and later independently and more extensively developed for any $s\in (0,1)$ by N\"ystrom and Sande \cite{NS} and Stinga and Torrea in \cite{ST}. Since we need to consider the half-space $\R^{n+1}_{(x,t)} \times \R^+_z$, it will be convenient to combine the ``extension" variable $z>0$ with $x\in \Rn$, and indicate the generic point in the thick space $\Rn_x\times\R_z$ with the letter $X=(x,z)$. Whenever convenient, we will indicate with the short notation $U(X,t)$ the value at the point $((x,t),z)$ of a function $U:\R^{n+1}_{(x,t)} \times \R^+_z\to \R$. This should not cause any confusion in the reader's mind. For $x_0\in \Rn$ and $r>0$ we let $B_r(x_0) = \{x\in \Rn\mid |x-x_0|<r\}$, and denote the upper half-ball by $\mathbb B_r^+(x_0,0)=\{X = (x,z) \in \R^n \times \R^{+}\mid |x-x_0|^2 + z^2 < r^2\}$. The parabolic cylinder in the thin space $Q_r(x_0, t_0) = B_r(x_0) \times [t_0, t_0 + r^2)$. We also will need the upper half-cylinder in thick space $\mathbb Q_r^+((x_0,t_0),0)=\mathbb B_r^+(x_0,0) \times (t_0,t_0+r^2]$. When the center $x_0$ of $B_r(x_0)$ is not explicitly indicated, then we are taking $x_0 = 0$. Similar agreement for the thick half-balls $\mathbb B_r^+(x_0,0)$. \begin{rmrk}\label{R:balls} Since in this paper we make extensive use of the work \cite{ABDG}, it is important that we alert the reader that what we are presently indicating with $\mathbb B_r^+(x_0,0)$ and $\mathbb Q_r^+((x_0,t_0),0)$ were respectively denoted by $\mathbb B_r(x_0,0)$ and $\mathbb Q_r((x_0,t_0),0)$ on p. 6 in \cite{ABDG}. The reader should keep this in mind when comparing the definition of the quantity $\theta$ in \eqref{theta} with the one in (3.3) in that paper. \end{rmrk} For notational ease, $\nabla U$ and $\operatorname{div} U$ will respectively refer to the operators $\nabla_X U$ and $ \operatorname{div}_X U$. The partial derivative in $t$ will be denoted by $\p_t U$, and also at times by $U_t$. The partial derivative $\partial_{x_i} U$ will be denoted by $U_i$. At times, the partial derivative $\partial_{z} U$ will be denoted by $U_{n+1}$. Given a number $a\in (-1,1)$ and a $u:\R^n_x\times \R_t\to \R$, we seek a function $U:\R^n_x\times\R_t\times \R_z^+\to \R$ that satisfies the Dirichlet problem \begin{equation}\label{la} \begin{cases} \La U \overset{def}{=} \partial_t (z^a U) - \operatorname{div}(z^a \nabla U) = 0, \\ U((x,t),0) = u(x,t),\ \ \ \ \ \ \ \ \ \ \ (x,t)\in \R^{n+1}. \end{cases} \end{equation} Denote by $\py$ the weighted normal derivative \begin{equation}\label{nder} \py U((x,t),0)\overset{def}{=} \underset{z \to 0^+}{\lim} z^a \partial_z U((x,t),z). \end{equation} Then, the most basic property of the Dirichlet problem \eqref{la} is that if $s = \frac{1-a}2\in (0,1)$, then one has in $L^2(\R^{n+1})$ \begin{equation}\label{np} 2^{-a}\frac{\Gamma(\frac{1-a}{2})}{\Gamma(\frac{1+a}{2})} \py U((x,t),0)= - H^s u(x,t), \end{equation} where $\Gamma(x) = \int_0^\infty t^{x-1} e^{-t} dt$ is Euler gamma function. In \cite[Corollary 4.6]{BG} we proved that if $u\in \mathscr H^{2s}$, then the function $U$ in \eqref{la} is a weak solution of \begin{equation}\label{wk} \begin{cases} \La U=0 \ \ \ \ \ \ \ \ \ \ \ \ \ \ \ \ \ \ \ \ \ \ \ \ \ \ \ \ \ \ \text{in}\ \R^{n+1}\times \R^+_z, \\ U((x,t),0)= u(x,t)\ \ \ \ \ \ \ \ \ \ \ \ \ \ \ \ \text{for}\ (x,t)\in \R^{n+1}, \\ \py U((x,t),0)= 2^{a} \frac{\Gamma(\frac{1+a}{2})}{\Gamma(\frac{1-a}{2})} V(x,t) u(x,t)\ \ \ \ \text{for}\ (x,t)\in B_1 \times (-1,0]. \end{cases} \end{equation} Note that the third equation in \eqref{wk} is justified by \eqref{e0} and \eqref{np}. For notational purposes, it will be convenient henceforth to work with the following backward version of problem \eqref{wk} \begin{equation}\label{exprob} \begin{cases} z^a \partial_t U + \operatorname{div}(z^a \nabla U)=0\ \ \ \ \ \ \ \ \ \ \ \ \text{in} \ \R^{n+1} \times \R^+_z, \\ U((x,t),0)= u(x,t) \\ \py U((x,t),0)= V(x,t) u(x,t)\ \ \ \ \ \ \ \ \ \text{in}\ B_4 \times [0,16). \end{cases} \end{equation} We note that the former can be transformed into the latter by changing $t \to -t$ and a parabolic rescaling $U_{r_0}(X,t)= U(r_0X, r_0^2t)$ for small enough $r_0$. We also emphasise that, to simplify the notation in \eqref{exprob}, we have incorporated in the potential $V$ the normalising constant $2^{a} \frac{\Gamma(\frac{1+a}{2})}{\Gamma(\frac{1-a}{2})}$ in \eqref{wk}. In the next section we will need the following regularity result established in \cite[Section 5]{BG}. \begin{lemma}\label{reg1} Let $U$ be a weak solution of \eqref{exprob}. Then there exists $\alpha>0$ such that one has up to the thin set $\{z=0\}$ \[ U_i,\ U_t,\ z^a U_z\ \in\ H^{\alpha}(\mathbb B_\frac{1}{2}^+ \times [0, 1/4)),\ \ \ \ i=1,2,..,n. \] Moreover, the relevant H\"older norms are bounded by $\int_{\mathbb B_1^+ \times (0, 1)} U^2 z^a dX dt$. Furthermore, we also have \begin{equation}\label{est} \int_{\mathbb B_{1/2}^+ \times [0, 1/4)} ( |\nabla_x U|^2 + |\nabla \nabla_x U|^2 + U_t^2 ) z^a dX dt \leq C \int_{\mathbb B_1^+ \times (0, 1)} U^2 z^a dX dt. \end{equation} \end{lemma} In the proof of Theorem \ref{main} we will need the following basic conditional doubling property established in \cite[Theorem 3.5]{ABDG}. With $U$ as in \eqref{exprob}, define \begin{equation}\label{theta} \theta \overset{def}{=}\frac{\int_{\mathbb Q_4^+} U(X,t)^2 z^adXdt }{\int_{\mathbb B_1^+} U(X,0)^2 z^adX}. \end{equation} Notice that in \eqref{theta} the notation $U(X,t)$ means $U((x,t),z)$. Similarly, $U(X,0)$ indicates $U((x,0),z)$. Concerning the definition of $\theta$, the reader should also keep Remark \ref{R:balls} in mind. \begin{thrm}\label{doub} Let $U$ be a solution of \eqref{exprob}. There exists $N>2$, depending on $n$, $a$ and the $C^1$-norm of $V$, such that $N\log(N\theta) \ge 1$, and for which: \begin{itemize} \item[(i)] For $r \leq 1/2,$ we have $$\int_{\mathbb B_{2r}^+}U^2(X,0)z^adX \leq (N \theta)^N\int_{\mathbb B_{r}^+}U^2(X,0)z^adX.$$ \end{itemize} Moreover, for $r \leq 1/\sqrt{N \operatorname{log}(N \theta)}$ the following inequality holds: \begin{itemize} \item[(ii)]$$\int_{\mathbb Q_{2r}^+} U^2 z^adXdt \leq \operatorname{exp}(N \operatorname{log}(N \theta) \operatorname{log}(N \operatorname{log}(N \theta)))\int_{\mathbb Q_r^+}U^2z^adXdt.$$ \end{itemize} \end{thrm} We say that a function $u(x,t)$ vanishes to infinite order at $(0,0)$ if for all $k>0$ one has for $r \to 0$ \begin{equation}\label{vp} \int_{B_r \times (-r^2, 0]} u(x,t)^2 dx dt = O(r^k). \end{equation} \begin{rmrk}\label{R:norms} It is worth mentioning here that in the following strong unique continuation results from \cite{ABDG}, Theorems \ref{extmain} and \ref{main1}, which will be needed in the proof of Theorem \ref{main}, the notion of vanishing to infinite order used the $L^\infty$ norm, instead of the $L^2$ norm as in \eqref{vp}. Since the proofs hold unchanged with $L^2$ norms, and since such norms will play an important role in the blowup analysis, we will use \eqref{vp}. \end{rmrk} Theorem \ref{doub}, combined with an appropriate blow-up analysis, was used in \cite{ABDG} to derive the following strong unique continuation property which represents the central result of that work. \begin{thrm}\label{extmain} Let $U$ be a solution of \eqref{exprob} in $\mathbb Q_{2}^+$ with $V$ satisfying the assumption in \eqref{vassump}. If the function $u:Q_2 \to \mathbb R$ defined by $u(x,t) \overset{def}= U((x,t), 0)$ vanishes to infinite order at $(0,0)$, then $U((x,0),z) \equiv 0$ in $\mathbb B_{2}^+$. \end{thrm} \begin{proof} Theorem \ref{extmain} was not explicitly stated in \cite{ABDG}, but its proof is embedded in that of \cite[Theor. 1.1, p. 35-38]{ABDG}. We thus refer the reader to that source. \end{proof} The next result is the just quoted Theorem 1.1 from \cite{ABDG}. Since this result will be used in the proof of Theorem \ref{main}, we need to make a comment here. \begin{thrm}\label{main1} Let $u \in \operatorname{Dom}(H^s)$ solve \eqref{e0} in $B_1 \times (-1, 0]$, and assume that $u$ vanishes to infinite order at $(0,0)$. Then it must be $u(\cdot, 0) \equiv 0$ in $\Rn$. \end{thrm} In Section \ref{s:m} we will also need the following trace interpolation estimate in \cite[Lemma 2.4]{ArB1}, inspired to a related time-independent result in \cite{RS}. \begin{lemma}\label{interpolation} Let $s \in (0,1)$. There exists a constant $C(n,s)>0$ such that, for any $0<\eta <1$ and $f \in C^2_0(\R^n \times \R_+)$, the following holds \begin{align}\label{inte} \int_{\Rn} |\nabla_x f|^2 dx \le C \eta^{2s} \int_{\R^n \times \R_+} ( |\nabla_x f|^2 + |\nabla \nabla_x f|^2) z^a dX dt + C\eta^{-2} ||f||^2_{L^2(\R^n)}. \end{align} \end{lemma} Using Lemmas \ref{reg1} and \ref{interpolation}, by arguing as in \cite[(5.14)-(5.16) on p.195]{ArB1}, we obtain the following estimate. \begin{lemma}\label{L:inter1} Let $U$ be as in \eqref{exprob}. For any $0< \eta <1$, there exists $C>0$, depending on $n, a, K$, such that the following holds \begin{equation*} \int_{Q_1} |\nabla_x u|^2 dxdt \leq C\eta^{2s} \int_{\mathbb Q_4^+} U^2 z^a dXdt + C\eta^{-2} \int_{Q_2} u^2 dxdt. \end{equation*} \end{lemma} Finally, we also need the following well-known form of Poincar\'e inequality, see e.g. \cite[Lemma 3.4, p. 54]{LU}. \begin{lemma}\label{po} Let $v \in W^{1,2}(B_1)$ and denote by $F= \{x\in B_1\mid v(x)=0\}$. If $E\subset B_1$ is any measurable set, one has for some $C=C(n)>0$, \begin{equation}\label{po1} \left(\int_{E} v^2\right)^{1/2} \leq C\ \frac{|E|^{1/n}}{|F|} \left(\int_{B_1} |\nabla_x v|^2\right)^{1/2}. \end{equation} \end{lemma} \vskip 0.3in \section{Proof of Theorem \ref{main}}\label{s:m} We begin by proving a quantitative result that allows to concentrate near the thin space the $L^2$ norm in the thick space of a solution to \eqref{exprob}. In the statement of the next lemma we denote by $\tilde V(x,t)$ a function satisfying the hypothesis \eqref{vassump}. \begin{lemma}\label{L:pos} Let $W$ be a solution to \begin{equation}\label{ex1} \begin{cases} z^a \partial_t W + \operatorname{div}(z^a \nabla W)=0\ \ \ \ \ \ \ \ \ \ \ \ \text{in} \ \mathbb Q_4^+ \\ \py W((x,t),0)= \tilde V(x,t) W((x,t),0)\ \ \ \ \ \ \ \ \ \text{in}\ B_4 \times [0,16), \end{cases} \end{equation} such that for some $C_1>1$ one has \begin{equation}\label{ret} \int_{\mathbb Q_{2}^+} W^2 z^a dXdt \leq C_1. \end{equation} Assume furthermore that \begin{equation}\label{norm} \int_{\mathbb Q_1^+} W^2 z^a dXdt =1. \end{equation} Then there exists $C_0>0$, depending on $n, a, K$ and $C_1$, such that \begin{equation}\label{pos1} \int_{Q_1} W((x,t),0)^2 dxdt \geq C_0. \end{equation} \end{lemma} \begin{proof} We argue by contradiction, and assume that for every $k\in \mathbb N$ there exists $\tilde V_k$ satisfying \eqref{vassump}, and a solution to \eqref{ex1}, $W_k$, which verifies \eqref{ret} and \eqref{norm}, and such that \begin{equation}\label{sm} \int_{Q_1} W_k((x,t),0)^2dxdt \leq \frac{1}{k}. \end{equation} By the theorem of Ascoli-Arzel\`a, we can extract a subsequence $\tilde V_k \to \tilde V_\infty$, uniformly in $\overline{\mathbb Q_1^+}$. Clearly, the limit function $\tilde V_\infty$ will satisfy \eqref{vassump}. Because of \eqref{ret} and the regularity estimate in Lemma \ref{reg1}, it follows that, up to a subsequence, $W_k \to W_\infty$. Furthermore, $W_\infty$ is a weak solution to \begin{equation}\label{Winfty} \begin{cases} z^a \partial_t W_{\infty} + \operatorname{div}(z^a \nabla W_{\infty})=0\ \ \ \ \ \ \ \ \ \ \ \ \ \ \ \ \ \ \ \ \ \ \ \text{in} \ \mathbb Q_2^+, \\ \py W_\infty((x,t),0)= \tilde V_\infty (x,t) W_{\infty} ((x,t),0)\ \ \ \ \ \ \ \ \ \text{in}\ B_2 \times [0,4). \end{cases} \end{equation} Because of uniform convergence and \eqref{sm}, it follows that $w(x,t) = W_{\infty}((x,t),0)=0$ for all $(x,t)$ in $Q_1$. This implies, in particular, that $w(x,t)$ vanishes to infinite order at \emph{every point} $(x_1,t_1)\in Q_1$. Invoking Theorem \ref{extmain} we thus infer that $W_\infty((x,0),z) \equiv 0$ in $\mathbb B_{1}^+$. Consider now any $t_1\in [0,1)$ and denote $\tilde W_\infty((x,t),z) = W_\infty((x,t+t_1),z)$, and $\tilde w(x,t) = \tilde W_\infty((x,t),0)$. From what we have observed above, $\tilde w(x,t)$ vanishes to infinite order at $(0,0)$. Since the extension equation in \eqref{ex1} is translation-invariant in $t$, again by Theorem \ref{extmain} we conclude that $W_\infty((x,t_1),z) = \tilde W_\infty((x,0),z) \equiv 0$ in $\mathbb B_{1}^+$. By the arbitrariness of $t_1\in (0,1)$, we finally conclude that $W_\infty \equiv 0$ in $\mathbb Q_1^+$. On the other hand, again using uniform convergence and \eqref{norm}, we can also assert that the following holds \[ \int_{\mathbb Q_1^+} W_{\infty}^2 z^a dXdt =1, \] thus reaching a contradiction. This proves the lemma. \end{proof} We now turn to the \begin{proof}[Proof of Theorem \ref{main}] In keeping with the presentation in Section \ref{s:n}, we will work with the adjoint nonlocal equation $(\p_t + \Delta)^s u + V(x,t) u = 0$, and the corresponding extension problem \eqref{exprob}. Without loss of generality, we assume that $F \subset B_1$ and that the adjoint version of \eqref{e0} holds in $B_4 \times [0,16)$. We will show that $u \equiv 0$ in $B_1 \times [0,16)$. Using Theorem \ref{main1}, we can then spread the zero set and reach the desired conclusion. We argue by contradiction and suppose that there exists $t_0 \in [0,16)$ such that \begin{equation}\label{assume} u(\cdot,t_0) \not \equiv 0,\ \ \ \ \text{in}\ B_1. \end{equation} Since for the corresponding extension function $U$ in \eqref{exprob} we have $U((x,t_0),0) = u(x,t_0)$, by continuity we must also have $U((x,t_0),z)) \not \equiv 0$ for $(x,z)\in \mathbb B^+_1$, and therefore in particular \[ \int_{\mathbb B^+_1} U((x,t_0),z))^2 z^a dX > 0. \] We claim that this implies that for every $x_0\in B_1$ one has \begin{equation}\label{pos} \int_{\mathbb B_1^+(x_0, 0)} U((x, t_0),z)^2 z^a dX>0. \end{equation} To see \eqref{pos}, consider the function $\tilde U((x,t),z) = U((x,t+t_0),z)$, which also solves \eqref{exprob}. For such function we have \[ \int_{\mathbb B_1^+} \tilde U(X,0)^2 z^adX = \int_{\mathbb B^+_1} U((x,t_0),z))^2 z^a dX > 0, \] and therefore the corresponding $\theta$ in \eqref{theta} is well-defined. We can thus apply the doubling condition (i) in Theorem \ref{doub}, obtaining for any $r>0$ (sufficiently small) \begin{equation}\label{i} \int_{\mathbb B_r^+} \tilde U(X,0)^2 z^a dX > 0. \end{equation} Given now any $x_0\in B_1$, the triangle inequality gives $\mathbb B_{1-|x_0|}^+ \subset \mathbb B^+_1(x_0,0)$. Applying \eqref{i} with $r = 1-|x_0|$, we conclude \[ \int_{\mathbb B_1^+(x_0, 0)} U((x, t_0),z)^2 z^a dX \ge \int_{\mathbb B_r^+} U((x, t_0),z)^2 z^a dX = \int_{\mathbb B_r^+} \tilde U(X,0)^2 z^a dX >0, \] which proves \eqref{pos}. Consider now the function $\bar{U}((x,t),z)= U((x+x_0,t+t_0),z)$, which is also a solution of \eqref{exprob}. Since by \eqref{pos} we have \[ \int_{\mathbb B_1^+} \bar{U}(X,0)^2 z^adX = \int_{\mathbb B_1^+(x_0, 0)} U((x, t_0),z)^2 z^a dX > 0, \] the corresponding \eqref{theta} for $\bar U$ is also well-defined, and from the doubling inequality (ii) in Theorem \ref{doub} we can assert that for some $C_1>1, r_1>0$, the following holds for all $r \leq r_1$ \begin{equation}\label{doub1} \int_{\mathbb Q_{2r}^+((x_0, 0, t_0))} U^2 z^a dX dt \leq C_1 \int_{\mathbb Q_r^+ ((x_0, 0, t_0))} U^2 z^a dX dt. \end{equation} Let now $x_0 \in B_1$ be a Lebesgue point of $F$, i.e. \begin{equation*} \lim_{r \to 0} \frac{|F \cap B_r(x_0)|}{|B_r(x_0)|}=1. \end{equation*} This implies that given any $\ve>0$, there exists $r_\ve>0$ (which without loss of generality we can assume $< \frac{r_1}{100}$) such that for all $r< r_\ve$, one has $|F \cap B_r(x_0)|> (1-\ve)|B_r(x_0)|$. This implies for $r<r_\ve$ \begin{equation}\label{l1} |B_r(x_0)\setminus F| < \ve |B_r(x_0)|. \end{equation} From \eqref{l1} and the rescaled version of the Poincar\'e inequality in Lemma \ref{po} it follows for all $r < r_\ve$ \begin{equation}\label{e1} \int_{Q_r(x_0, t_0)} u^2 dx dt\leq Cr^2 \ve^{2/n} \int_{Q_r(x_0, t_0)}|\nabla_x u|^2 dx dt. \end{equation} Since a change of scale with $r<1$ for the potential $V(x,t)$ decreases the bound $K$ on its $C^{k}$-norm ($k=1$ or $2$) in \eqref{vassump}, from the rescaled version of the interpolation estimate in Lemma \ref{L:inter1} it follows \begin{equation}\label{res1} \int_{Q_r(x_0, t_0)}|\nabla_x u|^2 dx dt \leq \frac{C\eta^{2s}}{r^{3+a}}\int_{\mathbb Q_{4r}^+((x_0, 0, t_0))} U^2 z^a dX dt + \frac{C \eta^{-2}}{r^2} \int_{Q_{2r}(x_0, t_0)} u^2 dx dt. \end{equation} Using \eqref{res1} in \eqref{e1}, we obtain for a new $C>0$ \begin{align}\label{e2} \int_{Q_r(x_0, t_0)} u^2 dx dt & \leq C \eta^{-2} \ve^{2/n} \int_{Q_{2r}(x_0, t_0)} u^2 dx dt+ \frac{C\eta^{2s}\ve^{2/n}}{r^{1+a}} \int_{\mathbb Q_{4r}^+((x_0, 0, t_0))} U^2 z^a dX dt\\ \notag & \leq C \eta^{-2} \ve^{2/n} \int_{Q_{2r}(x_0, t_0)} u^2 dxdt+ \frac{C \eta^{2s} \ve^{2/n}}{r^{1+a}} \int_{\mathbb Q_{r}^+((x_0, 0, t_0))} U^2 z^a dX dt, \notag \end{align} where in the second inequality we have used \eqref{doub1} (which we can, since $r < r_{\ve}< \frac{ r_1}{100}$). Consider now the function \[ W((x,t),z)= \frac{U((x_0 + rx, t_0 + r^2 t),rz)}{\bigg(\frac{1}{r^{n+3+a}}\int_{\mathbb Q_{r}^+((x_0, 0, t_0))} U^2 z^a dX dt\bigg)^{1/2}}. \] Then $W$ solves \eqref{ex1} with a potential given by \[ \tilde V(x,t) = r^{2s} V(x_0 + rx, t_0 + r^2 t). \] Furthermore, by \eqref{doub1} and a change of variable it is seen that $W$ satisfies the bounds \begin{equation}\label{norm1} \begin{cases} \int_{\mathbb Q_1^+} W^2 z^a dX dt =1, \\ \int_{\mathbb Q_2^+} W^2 z^a dX dt \leq C_1. \end{cases} \end{equation} Applying Lemma \ref{L:pos}, we reach the conclusion that $W$ satisfies the inequality \eqref{pos1}. By the change of variable $y= x_0+rx$, $\tau = t_0 + r^2 t$, we infer that $U$ verifies the estimate \begin{equation}\label{pos12} \int_{\mathbb Q_r^+((x_0, 0, t_0))} U^2 z^a dX dt \leq C_0^{-1} r^{1+a} \int_{Q_r(x_0, t_0)} u^2 dx dt. \end{equation} We now insert \eqref{pos12} in \eqref{e2}, obtaining for yet another $C>0$ the following basic inequality \begin{equation}\label{e4} \int_{Q_r(x_0, t_0)} u^2 dx dt \leq C \eta^{-2} \ve^{2/n} \int_{Q_{2r}(x_0, t_0)} u^2 dx dt + C \eta^{2s} \ve^{2/n} \int_{Q_r(x_0, t_0)} u^2 dx dt. \end{equation} Since $\ve<1$, by taking $\eta$ sufficiently small (it suffices to choose $C \eta^{2s}\le 1/2$), we can absorb the second term in the right-hand side of \eqref{e4} in the left-hand side, and finally arrive at the following bound \begin{equation}\label{e5} \int_{Q_r(x_0, t_0)} u^2 dx dt \leq C \ve^{2/n} \int_{Q_{2r}(x_0, t_0)} u^2 dx dt, \end{equation} valid for some $C>1$. Finally, we want to show that \eqref{e5} implies that $u$ vanishes to infinite order at $(x_0,t_0)$. We first observe that, if $f:[0,1]\to [0,\infty)$ is an increasing function such that for every $\delta\in (0,1)$, there exists $r_\delta\in (0,1)$ such that $f(r)\le \delta f(2r)$ for $0<r<r_\delta$, then we have for every $0<r<r_\delta$ \begin{equation}\label{f} f(r) \le \delta \left(\frac r{r_\delta}\right)^{\frac{\log \delta}{\log 2}} f(1). \end{equation} To prove \eqref{f}, we fix $m\in \mathbb N\cup\{0\}$ such that $2^{m} < \frac{r_\delta}{r} \le 2^{m+1}$. We thus find \[ f(1) \ge f(r_\delta) \ge f(2^{m} r) \ge \delta^{-1} f(2^{m} r) \ge\ ...\ge \delta^{-m} f(r). \] This gives for every $0<r<r_\delta$ \begin{equation}\label{fast} f(r) \le f(1) \left(\frac{r}{r_\delta}\right)^{\log\left(\frac{1}{\delta}\right)^{\frac{1}{\log 2}}}. \end{equation} If now $k\in \mathbb N$ is arbitrarily chosen, pick $\delta(k)\in (0,1)$ such that \[ \log\left(\frac{1}{\delta}\right)^{\frac{1}{\log 2}} \cong k. \] Corresponding to such choice, in view of \eqref{fast} there exists $r_k = r_{\delta(k)}\in (0,1)$ such that for every $0<r<r_k$ \[ f(r) \cong f(1) \left(\frac{r}{r_\delta}\right)^{k}. \] This shows that $f(r) = O(r^k)$ as $r\to 0^+$. By the arbitrariness of $k$ we infer that $f$ vanishes to infinite order at $r= 0$. Applying these observations to \[ f(r) = \int_{Q_r(x_0, t_0)} u^2 dx dt, \] by taking $\delta = C\ve^{2/n}$ in \eqref{e5}, we reach the conclusion that $u$ vanishes to infinite order at $(x_0,t_0)$. Invoking Theorem \ref{main1} we finally infer that it must be $u(\cdot, t_0) \equiv 0$ in $\Rn$, which obviously contradicts \eqref{assume}. We have thus proved that it must be $u \equiv 0$ in $B_1 \times [0,16)$. As we have mentioned, now we can spread the zeros of $u$ and show that $u \equiv 0$ in $\Rn \times [0,16)$. \end{proof} \begin{thebibliography}{99} \bibitem{ArB1} V. Arya \& A. Banerjee, \emph{Quantitative uniqueness for fractional heat type operators}, Calc. Var. Partial Differential Equations \textbf{62}~ (2023), no. 7, Paper No. 195, 47 pp. \bibitem{ABDG} V. Arya, A. Banerjee, D. Danielli \& N. Garofalo, \emph{Space-like strong unique continuation for some fractional parabolic equations}, J. Funct. Anal. \textbf{284} (2023), no. 1, Paper No. 109723. \bibitem{AT} A. Audrito \& S. Terracini, \emph{On the nodal set of solutions to a class of nonlocal parabolic reaction-diffusion equations}, arXiv:1807.10135, to appear in Memoirs of AMS. \bibitem{BG} A. Banerjee \& N. Garofalo, \emph{Monotonicity of generalized frequencies and the strong unique continuation property for fractional parabolic equations}, Adv. Math. 336 (2018), 149-241. \bibitem{BGh} A. Banerjee \& A. Ghosh, \emph{Sharp asymptotic of solutions to some nonlocal parabolic equations}, arXiv:2306.00341, to appear in DCDS Series A. \bibitem{BZ} N. Burq \& C. Zuily, \emph{A note on the uniqueness from sets of positive measure for time-dependent parabolic operators}, ArXiv:2010.16161 \bibitem{CS} L. Caffarelli \& L. Silvestre, \emph{An extension problem related to the fractional Laplacian}, Comm. Partial Differential Equations 32~(2007), no. 7-9, 1245-1260. \bibitem{DG} D. G. de Figueiredo \& J. Gossez, \emph{Strict monotonicity of eigenvalues and unique continuation}, Comm. Partial Differential Equations \textbf{17}~ (1992), no. 1-2, 339-346. \bibitem{EF} L. Escauriaza \& F. Fernandez, \emph{Unique continuation for parabolic operators. (English summary)}, Ark. Mat. \textbf{41}~ (2003), no. 1, 35-60. \bibitem{EFV} L. Escauriaza, F. Fernandez \& S. Vessella, \emph{Doubling properties of caloric functions}, Appl. Anal. \textbf{85}~ (2006), no. 1-3, 205-223. \bibitem{EMZ} L. Escauriaza, S. Montaner \& C. Zhang, \emph{Analyticity of solutions to parabolic evolutions and applications}, SIAM J. Math. Anal. \textbf{49}~ (2017), no. 5, 4064-4092. \bibitem{FF} M. Fall \& V. Felli, \emph{Unique continuation property and local asymptotics of solutions to fractional elliptic equations}, Comm. Partial Differential Equations, \textbf{39}~(2014), 354-397. \bibitem{FPS} V. Felli, A. Primo \& G. Siclari, \emph{On fractional parabolic equations with hardy-type potentials}, Commun. Contemp. Math. (2023), Article number 2350062, 1-50. DOI: 10.1142/S0219199723500621. \bibitem{Jr} B. F. Jones, \emph{Lipschitz spaces and the heat equation}. J. Math. Mech. \textbf{18}~(1968/69), 379-409. \bibitem{LU} O. A. Ladyzhenskaya \& N. N. Ural'tseva, \emph{Linear and Quasilinear Elliptic Equations}. Academic Press, 1968. \bibitem{NS} K. Nystr\"om \& O. Sande, \emph{Extension properties and boundary estimates for a fractional heat operator}, Nonlinear Analysis, 140~(2016), 29-37. \bibitem{Li} G. Lieberman, \emph{Second order parabolic differential equations}, World Scientific Publishing Co., Inc., River Edge, NJ, 1996. xii+439 pp. ISBN: 981-02-2883-X. \bibitem{Ru1} A. R\"uland, \emph{Unique continuation for fractional Schr\"odinger equations with rough potentials}, Comm. Partial Differential Equations \textbf{40}~ (2015), no. 1, 77-114. \bibitem{Ru2} A. R\"uland, \emph{Unique continuation for sublinear elliptic equations based on Carleman estimates}, J. Differential Equations \textbf{265}~ (2018), no. 11, 6009-6035. \bibitem{RS} A. R\"uland \& M. Salo, \emph{The fractional Calderón problem: low regularity and stability.} Nonlinear Anal. \textbf{193}~ (2020), 111529, 56 pp. \bibitem{ST} P.~R. Stinga \& J.~L. Torrea, \emph{Regularity theory and extension problem for fractional nonlocal parabolic equations and the master equation}, SIAM J. Math. Anal. 49 (2017), 3893--3924. \end{thebibliography} \end{document}
2412.03834v2
http://arxiv.org/abs/2412.03834v2
Tiling the field $\mathbb{Q}_p$ of $p$-adic numbers by a function
\documentclass[12pt]{amsart} \usepackage{amssymb} \usepackage{geometry} \usepackage{amsmath} \usepackage{amsfonts} \usepackage[mathscr]{eucal} \usepackage{amsthm} \usepackage{bookmark} \usepackage{enumerate} \usepackage{extarrows} \usepackage{changes}\usepackage{color} \usepackage{bbm} \usepackage{verbatim} \usepackage{graphicx} \usepackage{courier} \usepackage{helvet} \usepackage{courier} \usepackage{type1cm} \usepackage{multicol} \usepackage[bottom]{footmisc} \geometry{left=3cm,right=3cm,top=3cm,bottom=3cm} \date{} \theoremstyle{plain} \newtheorem*{thm}{Main Theorem} \newtheorem*{lem}{Main Lemma} \newtheorem{theorem}{Theorem}[section] \newtheorem{proposition}[theorem]{Proposition} \newtheorem{lemma}[theorem]{Lemma} \newtheorem{corollary}[theorem]{Corollary} \newtheorem{question}[theorem]{Question} \newtheorem{conjecture}[theorem]{Conjecture} \theoremstyle{definition} \newtheorem{definition}{Definition}[section] \newtheorem{example}[theorem]{Example} \newtheorem{claim}{Claim} \numberwithin{equation}{section} \theoremstyle{plain} \renewcommand{\theequation}{\arabic{section}.\arabic{equation}} \renewcommand{\thefootnote}{\fnsymbol{footnote}} \author{Shilei Fan} \address{Shilei FAN: School of Mathematics and Statistics, and Key Lab NAA--MOE, Central China Normal University, Wuhan 430079, China} \email{[email protected]} \title{ Tiling the field $\Q_p$ of $p$-adic numbers by a function } \date{\today} \def\Card{{\rm Card}} \def\R{{\textbf{R}}} \def\H{\mathcal{H}} \def\fh{\f{1}{2}} \def\p{\partial} \def\dsum{\displaystyle\sum} \def\f{\frac} \def\R{\mathbb R} \def\N{\mathbb N} \def\1{\mathbbm 1} \def\Z{\mathbb Z} \def\Q{\mathbb Q} \def\C{\mathbb C} \def\df{\displaystyle\frac} \def\e{\mathrm{e}} \def\d{\mathrm{d}} \def\i{\mathrm{i}} \def\v{\mathbf{v}} \def\x{\mathbf{x}} \def\w{\mathbf{w}} \def\g{\mathbf{g}} \def\df{\displaystyle\frac} \def\vsp{\vspace{1mm}} \def\ee{\mathbf{e}} \def\u{\mathbf{u}} \def\pt{\df{\partial}{\partial t}} \def\ff{\mathbf{f}} \def\x{\mathbf{x}} \def\sp{\mathrm{ess\,sup}} \def\al{\alpha} \def\be{\beta} \def\la{\lambda} \def\Om{\Omega} \def\va{\varepsilon} \def\ds{\displaystyle} \def\l{\left} \def\r{\right} \def\v{\varphi} \def\n{\nabla} \def\m{\mathfrak{m}} \thanks{ S. L. FAN was partially supported by NSFC (grants No. 12331004 and No. 12231013). } \begin{document} \maketitle \begin{abstract} This study explores the properties of the function which can tile the field $\Q_p$ of $p$-adic numbers by translation. It is established that functions capable of tiling $\Q_p$ is by translation uniformly locally constancy. As an application, in the field $\Q_p$, we addressed the question posed by H. Leptin and D. M\"uller, providing the necessary and sufficient conditions for a discrete set to correspond to a uniform partition of unity. The study also connects these tiling properties to the Fuglede conjecture, which states that a measurable set is a tile if and only if it is spectral. The paper concludes by characterizing the structure of tiles in \(\mathbb{Q}_p \times \mathbb{Z}/2\mathbb{Z}\), proving that they are spectral sets. \emph{Keywords:} periodic, translation tile, spectral set, $p$-adic field. \end{abstract} \section{Introduction} Consider a locally compact abelian group $G$ equipped with a Haar measure $\mu$. Let $f$ be a function in $L^1(G)$. The function $f$ is said to {\em tile} $G$ at level $w$ by a translation set $T$ if \[ \sum_{t \in T} f(x - t) = w \quad \text{a.e.} \ x\in G. \] Here, the convergence is absolute, the translation set $T$ is required to be locally finite, i.e. for any compact set $K$, the set $T\cap K$ is finite and we allow elements of $T$ to occur with finite multiplicities. We say that $(f, T)$ is a tiling pair at level $w$. If $f = 1_\Omega$ is an indicator function of the measurable set $\Omega$ and $w = 1$, we say that $\Omega$ {\em tiles} the group $G$ by the translation set $T$, and $(\Omega, T)$ is a {\em tiling pair} (at level $1$). The set $\Omega$ is called a {\em tile } of $G$ In the case where \( G = \mathbb{R} \), it was proven in \cite{LW96Invent} that if a bounded measurable set \(\Omega\) with measure-zero boundary tiles the line \(\mathbb{R}\) by a translation set \(T\), then \( T \) is a periodic set, that is, \( T + \tau = T \) for some \( \tau > 0 \). This result was extended in \cite{KL96Duke} (and proved earlier in \cite{LM91}) to tiling \(\mathbb{R}\) by a function \( f \in L^1(\mathbb{R}) \) with compact support. It was shown that if \( (f, T) \) is a tiling pair at some level \( w \) with \( T \) of bounded density and \( f \) is not identically zero, then \( T \) must be a finite union of periodic sets. For more related results and questions, interested readers can refer to \cite{KL21}. For the case \( G = \mathbb{Q}_p \), the field of \( p \)-adic numbers, it was proven in \cite{FFLS} that if \((\Omega, T)\) forms a tiling pair, then \( \Omega \) is a compact open set up to a set of measure zero, which implies \[ 1_\Omega(x) = 1_{\Omega}(x + \tau), \quad \text{a.e.} \ x \in \mathbb{Q}_p \] for some \( \tau \in \mathbb{Q}_p \setminus \{0\} \). This means that \( 1_\Omega(x) \) is a periodic function or the measurable set \( \Omega \) is periodic. Let \( \mathbb{Z}_p \) be the ring of \( p \)-adic integers. A compact open set \( \Omega \) can be expressed as \[ \Omega = \bigsqcup_{c \in C} (c + p^\gamma \mathbb{Z}_p) \] for some finite set \( C \subset \mathbb{Q}_p \) and some integer \( \gamma \in \mathbb{Z} \). Note that \( p^\gamma \mathbb{Z}_p \) is a subgroup of \( \mathbb{Q}_p \) under addition. Hence, a compact open set is a finite union of cosets of some subgroup. Moreover, the structure of the tile \( \Omega \) and the translation set \( T \) are characterized by \( p \)-homogeneous trees in \cite{FFS}. Let $f \in L^{1}(\mathbb{Q}_p)$ be a function. We say that $f$ is \emph{periodic} if \[f(x + \tau) = f(x) \quad \text{a.e. } x \in \mathbb{Q}_p,\] for some $\tau \in \mathbb{Q}_p \setminus \{0\}$. In this note, we show that any function capable of tiling $\mathbb{Q}_p$ is inherently periodic. \begin{theorem}\label{thm-tilingperiodic} Let $f \in L^{1}(\mathbb{Q}_p)$ and $V \subset \mathbb{Z} \setminus \{0\}$ be a finite set of non-zero integers. Suppose that $T$ is a locally finite subset in $\mathbb{Q}_p$, and $v_t \in V$ for $t \in T$ are such that \begin{align} \sum_{t \in T} v_t \cdot f(x-t) = w, \quad \text{a.e. } x \in \mathbb{Q}_p, \end{align} for some $w \in \mathbb{R}$.Then $f$ is uniformly locally constancy, i.e. there exists $n\in \mathbb{Z}$ such that \begin{align}\label{eq-periodic} \forall u\in B(x,p^n), \quad f(x+u)=f(x) \quad \text{a.e. } x\in \Q_p. \end{align} \end{theorem} Remark that Equality \eqref{eq-periodic} implies that $f$ is periodic. Actually, the definition regarding tiling doesn't necessarily require the group to be commutative (abelian). It is proved in \cite{HN79} that in every connected nilpotent group $G$ there exists a discrete subset $T$ and a corresponding non-negative smooth function $\phi$ with compact support in $G$ such that \[\sum_{t\in T} \phi(t x)=1 \quad \text{for all } x\in G,\] i.e., the family $\{\phi(t x): t\in T\}$ forms a partition of unity in $G$, which is called a \emph{uniform partition of unity} in $G$. H. Leptin and D. M\"uller \cite{LM91} proposed two questions: 1. Which locally compact groups do admit uniform partitions of unity? 2. Can one describe the set $T$ which corresponds to a uniform partition of unity?\\ And a positive answer for the first question was obtained by the authors. However, the authors claimed that a comprehensive answer to the second question seems to be beyond the present possibilities. They restricted their considerations to the simplest non-trivial case, namely to the group of reals: $ G = \mathbb{R}$ and provided a complete solution. In the case of $G=\mathbb{Q}_p$, the field of $p$-adic numbers, we have also presented a complete solution. For $x\in \Q_p$ and $r>0$, denote by \[B(x, r)=\{y\in \Q_p: |x-y|_p\leq r\}\] the closed ball of radius $r$ centered at $x$. \begin{theorem} \label{thm:card} A discrete set $T \subset \mathbb{Q}_p$ corresponds to a uniform partition of unity if and only if there exists a sufficiently large integer $n$ such that \[ \#(B(x,p^n)\cap T) =\#(B(y,p^n)\cap T) \] for all $x, y \in \mathbb{Q}_p$. \end{theorem} For other motivation, it is illuminating to observe the inherent connection between tiling by a set and tiling by a function, particularly within the framework of the Fuglede conjecture \cite{MR0470754}. The conjecture stats that a measurable set $\Omega \in \R^d$ of unit measure can tile $\R^d$ by translation if and only if there exists a set $\Lambda \in \R^d$ such that $\{e^{2\pi i \lambda x}, \lambda \in \Lambda\}$ forms an orthonormal basis of $L^2(\Omega)$. Here, the set $\Lambda$ is called a spectrum of $\Omega$ and $(\Omega,\Lambda)$ is called a spectral pair. Denote by $f_\Omega$ the so-called power spectrum of $1_\Omega$ \[f_{\Omega}(\xi)=|\widehat{1_{\Omega}}(\xi)|^2.\] Then $(\Omega,\Lambda)$ is a spectral pair if and only if $(f_{\Omega},\Lambda)$ form a tiling pair at level $1$. Hence, the Fuglede conjecture takes the following symmetric form \begin{align}\label{equ-fug} \Omega\text{ tiles }\R^d \Longleftrightarrow |\widehat{1_{\Omega}}|^2 \text{ tiles } \R^d. \end{align} There are many positive results under different extra assumptions before the work\cite{MR2067470} where Tao gave a counterexample: there exists a spectral subset of ${\R}^{n}$ with $n\ge 5$ which is not a tile. After that, Matolcsi\cite{MR2159781}, Matolcsi and Kolountzakis\cite{MR2264214,MR2237932}, Farkas and Gy \cite{MR2221543}, Farkas, Matolcsi and Mora\cite{MR2267631} gave a series of counterexamples which show that both directions of Fuglede's conjecture fail in ${\R}^{n}\left ( n\ge 3 \right ) $. However, the conjecture is still open in low dimensions $n=1,2$. Generally, for a locally compact abelian group $G$, denote by $\widehat{G} $ its dual group. Consider a Borel measurable subset $\Omega $ in $G$ of unit measure. We say that $\Omega $ is a \textit{spectral set} if there exists a set $\Lambda \subset \widehat{ G} $ which forms an orthonormal basis of the Hilbert space $L^{2} \left ( \Omega \right ) $. Such a set $\Lambda $ is called a \textit{spectrum} of $\Omega $ and $\left ( \Omega ,\Lambda \right ) $ is called a \textit{spectral pair}. The Fuglede conjecture can be generalized to locally compact abelian groups: {\em a Borel measurable set $\Omega \subset G$ of unit measure is a spectral set if and only if it can tile $G$ by translation.} Similarly, the generalized Fuglede conjecture also takes the following symmetric form \begin{align}\label{equ-fugloc} \Omega\text{ tiles }G \Longleftrightarrow |\widehat{1_{\Omega}}|^2 \text{ tiles } G. \end{align} In its generality, this generalized conjecture does not hold true. Instead, we should inquire about the groups for which it holds. This question even arises for finite groups. The counterexamples in $\mathbb{R}^d$, $d\geq 3$, are actually constructed based on counterexamples in finite groups. Substantial work has been done for some finite groups \cite{MR3684890, MR4402630, FFS, MR3649367, MR4085122, MR4493731, MR1772427, MR2237932, MR1895739, MR4422438, MR3695475, MR4186120, MR4604197}. For infinite groups, based on \eqref{equ-fugloc}, Fan et al. \cite{FFLS} proved that Fuglede's conjecture holds in the field $\mathbb{Q}_p$ of $p$-adic numbers. In fact, this is the first example of an infinite abelian group where Fuglede's conjecture holds. It is natural to find other infinite locally compact abelian groups where Fuglede's conjecture holds. As an application of Theorem \ref{thm-tilingperiodic}, we can characterize the structure of the tiles in the infinite group $\Q_p\times \Z/2\Z$. \begin{theorem}\label{thm1.3} Assume that $\Omega= \Omega_0\times\{0\} \cup \Omega_1\times\{1\} $ tiles $\Q_p\times \Z/2\Z$ by translation. \begin{enumerate}[{\rm (1)}] \item If $p>2$, then either $\mu(\Omega_0\cap\Omega_1)=0$ and $ \Omega_0\cup \Omega_1$ can tile $\Q_p$ by translation or $\Omega_0$ and $\Omega_1$ can tiles $\Q_p$ with a common translation set $T_0\subset \Q_p$. \item If $p=2$, then we have three cases: \begin{itemize} \item[(i)] $\mu(\Omega_0\cap \Omega_1)=0$ and $\Omega_0\cup \Omega_1$ tiles $\Q_2$ by translation, \item[(ii)]$\Omega_0$ and $\Omega_1$ can tiles $\Q_2$ with a common translation set $T_0\subset \Q_2$ \item[(iii)] both $\Omega_0$ and $\Omega_1$ are compact open (up to a measure zero set) and there exist $j_0\in \{0,\cdots, n-1\}$ such that $\Omega_0$ and \[\widetilde{\Omega}_1 = \{ x + 2^{n-j_0-1}: x \in \Omega_1\}\] are disjoint (up to a measure zero set) and $\Omega_0\cup \widetilde{\Omega}_1$ tiles $\Q_2$ by translation. \end{itemize} \end{enumerate} \end{theorem} As a consequence, we establish that spectrality of tiles in the infinite group $\Q_p\times \Z/2\Z$. \begin{corollary} Tiles in $\Q_p \times \Z/2\Z$ are spectral sets. \end{corollary} \section{Preliminaries} \subsection{The field $\Q_p$ of $p$-adic numbers }\label{p-adicfield} Let's start with a quick review of \(p\)-adic numbers. Consider the field \(\mathbb{Q}\) of rational numbers and a prime \(p \ge 2\). Any nonzero number \(r \in \mathbb{Q}\) can be expressed as \(r = p^v \frac{a}{b}\), where \(v, a, b \in \mathbb{Z}\) and \((p, a) = 1\) and \((p, b) = 1\) (here \((x, y)\) denotes the greatest common divisor of the integers \(x\) and \(y\)). We define \(|r|_p = p^{-v_p(r)}\) for \(r \neq 0\) and \(|0|_p = 0\). Then \(|\cdot|_p\) is a non-Archimedean absolute value, meaning: \begin{itemize} \item[(i)] \(|r|_p \ge 0\) and equality only when \(r = 0\), \item[(ii)] \(|r s|_p = |r|_p |s|_p\), \item[(iii)] \(|r + s|_p \leq \max\{ |r|_p, |s|_p\}\). \end{itemize} The field \(\mathbb{Q}_p\) of \(p\)-adic numbers is the completion of \(\mathbb{Q}\) under \(|\cdot|_p\). The ring \(\mathbb{Z}_p\) of \(p\)-adic integers is the set of \(p\)-adic numbers with absolute value at most 1. A typical element \(x\) of \(\mathbb{Q}_p\) is written as \begin{equation}\label{HenselExp} x = \sum_{n = v}^\infty a_n p^{n} \quad (v \in \mathbb{Z}, a_n \in \{0,1,\dotsc, p-1\}, \text{ and } a_v \neq 0). \end{equation} Here, \(v_p(x) := v\) is called the \(p\)-{\em valuation} of \(x\). A non-trivial continuous additive character on $\mathbb{Q}_p$ is defined by $$ \chi(x) = e^{2\pi i \{x\}}, $$ where $\{x\} = \sum_{n=v_p(x)}^{-1} a_n p^n$ is the fractional part of $x$ in (\ref{HenselExp}). Remark that \begin{align}\label{one-in-unit-ball} \chi(x) = e^{2\pi i k/p^n}, \quad \text{if } x \in \frac{k}{p^n} + \mathbb{Z}_p \ \ (k, n \in \mathbb{Z}), \end{align} and \begin{align}\label{integral-chi} \int_{p^{-n}\mathbb{Z}_p} \chi(x) \,dx = 0 \ \text{for all } n \geq 1. \end{align} From the character $\xi$, we can obtain all characters $\chi_\xi$ of $\mathbb{Q}_p$ by defining $\chi_\xi(x) = \chi(\xi x)$. The map $\xi \mapsto \chi_{\xi}$ from $\mathbb{Q}_p$ to $\widehat{\mathbb{Q}}_p$ is an isomorphism. We write $\widehat{\mathbb{Q}}_p \simeq \mathbb{Q}_p$ and identify a point $\xi \in \mathbb{Q}_p$ with the point $\chi_\xi \in \widehat{\mathbb{Q}}_p$. For more information on $\mathbb{Q}_p$ and $\widehat{\mathbb{Q}}_p$, the reader is referred to the book \cite{VVZ94}. The following notation will be used in the whole paper. \medskip \\ \noindent {\bf Notation}: \begin{itemize} \item $\mathbb{Z}_p^\times := \mathbb{Z}_p\setminus p\mathbb{Z}_p=\{x\in \mathbb{Q}_p: |x|_p=1\}$, the group of units of $\mathbb{Z}_p$. \item $B(0, p^{n}): = p^{-n} \mathbb{Z}_p$, the (closed) ball centered at $0$ of radius $p^n$. \item $B(x, p^{n}): = x + B(0, p^{n})$. \item $ S(x, p^{n}): = B(x, p^{n})\setminus B(x, p^{n-1})$, a ``sphere". \end{itemize} \subsection{Fourier Transform} The Fourier transform of $f\in L^1(\Q_p)$ is defined to be $$\widehat{f}(\xi)=\int_{\Q_p}f(x)\overline{\chi_\xi(x)} dx \quad (\forall \xi\in \widehat{\Q}_p\simeq \Q_p).$$ A complex function $f$ defined on $\Q_p$ is called \textit{uniformly locally constancy} if there exists $n\in \mathbb{Z}$ such that \[f(x+u)=f(x) \quad \forall x\in \Q_p, \forall u \in B(0, p^n).\] The following proposition shows that for an integrable function $f$, having compact support and being uniformly locally constant are dual properties for $f$ and its Fourier transform. \begin{proposition}[\cite{FFLS}]\label{Prop-compactConstant} Let $f\in L^1(\Q_p)$ be a complex-value integrable function. Then $f$ has compact support if and only if $\widehat{f}$ is uniformly locally constancy. \end{proposition} The Fourier transform of a positive finite measure $\mu $ on $\Q_p$ is defined to be \[\widehat{\mu}(\xi)=\int_{\Q_p}\overline{\chi_\xi(x)} d\mu(x) \quad (\forall \xi\in \widehat{\Q}_p\simeq \Q_p).\] Let $\nu $ be a signed measure on ${\Q}_{p}$, with the finite total variation $\left | \nu \right | $. The measures $$\nu ^{+} =\frac{1}{2} \left ( \left | \nu \right | +\nu \right ) ,\mu ^{-} =\frac{1}{2} \left ( \left | \nu \right | -\nu \right )$$ are called the positive and negative variations of $\nu$. The Fourier transform of $\nu $ is defined to be \[\widehat{\nu}(\xi) =\widehat{\nu^{+} }(\xi)-\widehat{\nu^{-} }(\xi)=\int_{{\Q}_{p} }\overline{\chi_{\xi} (x) } d\nu^{+}(\xi) -\int_{{\Q}_{p} }\overline{\chi_{\xi} (x) } d\nu^{-}(x)=\int_{{\Q}_{p} }\overline{\chi_{\xi} (x) } d\nu(x)\] for all $\xi \in \widehat{{\Q}}_{p}\cong {\Q}_{p}$. \subsection{$\mathbb{Z}$-module generated by $p^n$-th roots of unity} \label{Zmodule} Let $m\ge 2$ be an integer and let $\omega_m = e^{2\pi i/m}$, which is a primitive $m$-th root of unity. Denote by $\mathcal{M}_m$ the set of integral points $(a_0, a_1, \dotsc, a_{m-1}) \in \mathbb{Z}^m$ such that $$ \sum_{j=0} ^{m-1} a_j \omega_m^j =0. $$ The set $\mathcal{M}_m$ is clearly a $\mathbb{Z}$-module. When $m=p^n$ is a power of a prime number, the relation between the coefficients is established. \begin{lemma} [{\cite[Theorem 1]{Sch64}}]\label{SchLemma} If $(a_0,a_1,\dotsc, a_{p^n-1})\in \mathcal{M}_{p^n}$, then for any integer $0\le j\le p^{n-1}-1$ we have $a_j=a_{j+sp^{n-1}}$ for all $s=0,1,\dots, p-1$. \end{lemma} A subset $C\subset \Q_p$ is called a {\em $p$-cycle} in $ \Q_{p}$ is a set of $p$ elements such that \[\sum_{c\in C} \chi(c)=0\] which is equivalent to \begin{align}\label{eq-pcycle} C \equiv \left \{ r,r+\frac{1}{p} ,r+\frac{2}{p} ,\dots ,r+\frac{p-1}{p} \right \} \mod \Z_{p} \end{align} for some $r\in \Q_{p} $. Note that \eqref{eq-pcycle} is also equivalent to that $C$ is a subset of $p$ elements such that for distinct $c, c^{\prime}\in C$, \[ |c-c^{\prime}|_p=p.\] For a $p$-cycle $C$ in $\Q_p$ and $x\in \Z_p^{\times}$, it is easy to see that $x\cdot C$ is also a $p$-cycle in $\Q_p$, which is to say \[\sum_{c\in C} \chi(x c)=0.\] Let $ E\subset \Q_{p}$ be a finite subset. If $\sum_{c\in E} \chi \left ( c \right ) =0$, then by Lemma \ref{SchLemma}, $E$ can be decomposed into some $p$-cycles. Hence, the property $\sum_{c\in E} \chi(c) = 0$ of the set $E\subset \Q_p$ is invariant under rotations. \begin{lemma}[{\cite[ Lemma 2.6]{FFS}}]\label{lem6} Let $\xi _{0}, \xi _{1},\cdots ,\xi _{m-1}$ be $m$ points in $\Q_{p}$. If\: $\sum_{j=0}^{m-1} \chi \left ( \xi _{j} \right ) =0$, then $p\mid m$ and $$\sum_{j=0}^{m-1} \chi \left ( x\xi _{j} \right ) =0$$for all $x\in \Z_{p}^{\times } $. \end{lemma} Actually, by Lemmas \ref{SchLemma} and \ref{lem6}, we have the following corollary. \begin{corollary}\label{corollary2} Let $\xi _{0}, \xi _{1},\cdots ,\xi _{m-1}$ be $m$ points in $\Q_{p}$. If $\sum_{j=0}^{m-1} \alpha _{j} \chi \left ( \xi _{j} \right ) =0$, where $\alpha _{j}\in \Z $, then $$\sum_{j=0}^{m-1}\alpha _{j} \chi \left ( x\xi _{j} \right ) =0$$for all $x\in \Z_{p}^{\times } $. \end{corollary} \subsection{Zeros of the Fourier Transform of a finite discrete signed measure} Let $E$ be a non-empty finite subset of $\Q_p$ and \[\nu=\sum_{x\in E} \alpha_{x}\delta_x,\quad \alpha_{x}\in \Z\setminus\{0\}\] be a finite measure supported in the set $E$. By definition, we have \[ \widehat{\nu} (\xi)= \sum_{x\in E} \alpha_{x}\overline{\chi(\xi x)}= \sum_{x\in E} \alpha_{x}e^ {2\pi i\{-\xi x\}}.\] By Corollary \ref{corollary2}, the zero set of the Fourier transform of a finite discrete measure $\nu$ is invariant under rotations. \begin{lemma}\label{lem-fourierzero} If $\widehat{\nu}(\xi)=0$, then $\widehat{\nu}(\xi \cdot \zeta)=0$ for all $\zeta\in \Z_{p}^{\times }$. \end{lemma} A necessary condition for a point to be a zero of the Fourier transform of $\nu$ is provided. \begin{lemma}\label{lem-necessary} For $\xi\in \Q_p^{\times}$, if $\widehat{\nu}(\xi)=0$, then for any $x\in E$, there exists a distict point $x^{\prime}\in E$ such that \[|x-x^{\prime}|_p\leq \frac{p}{|\xi|_p}. \] \end{lemma} \begin{proof} Since $E$ is a finite subset of $\mathbb{Q}_p$, it is bounded. Specifically, let's assume that for each $x \in E$, the $p$-adic absolute value $|x|_p$ satisfies $|x|_p \leq p^n$ for some integer $n$. For $\xi \in \Q_p^{\times}$, we have $|\xi|_p=p^m$ for some $m\in \Z$. By Lemma \ref{lem-fourierzero}, $\widehat{\nu} (\xi)=0$ if and only if $\widehat{\nu} (-1/p^m)=0$, which is equivalent to \begin{align}\label{eq-Fourierzero} \sum_{x\in E} \alpha_{x}\chi_{1/p^m}(x)=0. \end{align} Note that the character $\chi_{1/p^m}$ is uniformly locally constant on the disks with radius $1/p^m$. Consider the map $\chi_{1/p^m}:E \to \{1, e^{2\pi i /p^{m+n}}, \cdots, e^{2\pi i (p^{m+n}-1)/p^{m+n}}\}$. Denote by $E_j=\{x\in E: \chi_{1/p^m}(x)=e^{2\pi i j/p^{m+n}} \}$. Equality \eqref{eq-Fourierzero} is equivalent to \[ \sum_{j=0} ^{p^{m+n}-1} a_j e^{2\pi i j /p^{m+n}} =0, \] where $a_j=\sum_{x\in E_j} \alpha_x$. For $x\in E$, assume that $\chi_{1/p^m}(x)=e^{2\pi i j /p^{m+n}} $ for some $0\leq j\leq p^{m+n}-1$. By Lemma \ref{SchLemma}, $a_j=a_{j+p^{m+n-1} \mod p^{m+n}}$. If $a_j=0$, then there exists $x^{\prime}\in E_j$ such that $x^{\prime}\neq x$. Noting that $\chi_{1/p^m}(x)= \chi_{1/p^m}(y)$ if and only if $|x-y|_p\leq 1/p^m$. Hence $|x-x^{\prime}|\leq 1/p^m$. On the other hand, if $a_j\neq 0$, then there exists $x^{\prime}\in E$ such that \[\chi_{1/p^m}(x^{\prime})=e^{2\pi i \frac{j+p^{m+n-1}}{p^{m+n}} },\] which implies \[|x-x^{\prime}|_p=\frac{1}{p^{m-1}}.\] \end{proof} \subsection{Bruhat-Schwartz distributions in $\Q_p$}\label{subsec2.4} Here, we provide a concise overview of the theory based on the works of \cite{AKS10, Tai75, VVZ94}. Let $\mathcal{E}$ represent the space of uniformly locally constancy functions. The space $\mathcal{D}$ of \textit{Bruhat-Schwartz test functions} is defined as functions that are uniformly locally constancy and have compact support. A test function $f\in \mathcal{D}$ is a finite linear combination of indicator functions of the form $1_{B(x,p^k)}(\cdot)$, where $k\in \mathbb{Z}$ and $x\in \Q_p$. The largest such number $k$ is denoted by $\ell:= \ell(f)$ and referred to as the \textit{parameter of constancy} of $f$. As $f\in \mathcal{D}$ has compact support, the minimal number $\ell':=\ell'(f)$ such that the support of $f$ is contained in $B(0, p^{\ell'})$ exists and is named the \textit{parameter of compactness} of $f$. Certainly, we have $\mathcal{D}\subset \mathcal{E}$. The space $\mathcal{D}$ is endowed with the topology of a topological vector space as follows: a sequence ${\phi_n }\subset \mathcal{D}$ is termed a \textit{null sequence} if there exist fixed integers $l$ and $l^{\prime}$ such that each $\phi_n$ is constancy on every ball of radius $p^l$, is supported by the ball $B(0,p^{l^{\prime}})$, and the sequence $\phi_n$ uniformly converges to zero. A \textit{Bruhat-Schwartz distribution} $f$ on $\Q_p$ is defined as a continuous linear functional on $\mathcal{D}$. The value of $f$ at $\phi \in \mathcal{D}$ is denoted by $\langle f, \phi \rangle$. Notably, linear functionals on $\mathcal{D}$ are automatically continuous, facilitating the straightforward construction of distributions. Let $\mathcal{D}'$ denote the space of Bruhat-Schwartz distributions, equipped with the weak topology induced by $\mathcal{D}$. A locally integrable function $f$ is treated as a distribution: for any $\phi \in \mathcal{D}$, $$ \langle f,\phi\rangle=\int_{\Q_p} f\phi d\m. $$ Let $E$ be a locally finite discrete subset in $\Q_p$, and let $\nu$ be a discrete signed measure supported in $E$ defined as \begin{align}\label{eq-discretemeasure} \nu = \sum_{x\in E} \alpha_{x}\delta_x, \quad \alpha_{x}\in \Z\setminus\{0\}, \end{align} where $\{\alpha_x: x\in E\}$ is a bounded. The discrete measure $\nu$ defined by \eqref{eq-discretemeasure} is also a distribution: for any $\phi \in \mathcal{D}$, $$ \langle \nu,\phi\rangle= \sum_{x\in E} \alpha_x \phi(x). $$ Here for each $\phi$, the sum is finite because $E$ is locally finite and each ball contains at most a finite number of points in $E$. Since the test functions in $\mathcal{D}$ are uniformly locally constancy and have compact support, the following proposition is a direct consequence of Proposition \ref{Prop-compactConstant} or of the fact (see also \cite[Lemma 4]{Fan15}) that \begin{equation}\label{FB} \widehat{1_{B(c, p^k)}}(\xi) = \chi(-c \xi)\cdot p^{k} \cdot 1_{B(0, p^{-k})}(\xi). \end{equation} \begin{proposition}[\cite{Tai75}, Chapter II 3] The Fourier transformation $f \mapsto \widehat{f}$ is a homeomorphism from $\mathcal{D}$ onto $\mathcal{D}$. \end{proposition} Let $f \in \mathcal{D}'$ be a distribution in $\Q_p$. A point $x\in \Q_p$ is called a {\em zero} of $f$ if there exists an integer $n_0$ such that $$ \langle f, 1_{B(y,p^{n})}\rangle=0, \quad \text {for all } y\in B(x,p^{n_0}) \text{ and all integers } n\leq n_0 \text.$$ Denote by $\mathcal{Z}_f$ the set of all zeros of $f$. Remark that $\mathcal{Z}_f$ is the maximal open set $O$ on which $f$ vanishes, i.e. $\langle f, \phi\rangle=0$ for all $\phi \in \mathcal{D}$ such that the support of $\phi$ is contained in $O$. The {\em support} of a distribution $f$ is defined as the complementary set of $\mathcal{Z}_f$ and is denoted by $\operatorname{supp}(f)$. The Fourier transform of a distribution $f \in \mathcal{D}'$ is a new distribution $\widehat{f}\in \mathcal{D}'$ defined by the duality \[ \langle \widehat{f},\phi\rangle=\langle f,\widehat{\phi}\rangle, \quad \forall \phi \in \mathcal{D}.\] The Fourier transform $f\mapsto \widehat{f}$ is a homeomorphism on $\mathcal{D}'$ under the weak topology \cite[Chapter II 3]{Tai75}. \subsection{Convolution and multiplication of distributions} Denote $$ \Delta_k:=1_{B(0,p^k)}, \quad \theta_k:=\widehat{\Delta}_k=p^k \cdot 1_{B(0,p^{-k})}. $$ Let $f,g\in \mathcal{D}'$ be two distributions. We define the {\em convolution} of $f$ and $g$ by $$ \langle f*g,\phi \rangle =\lim\limits_{k\to \infty} \langle f(x), \langle g(\cdot),\Delta_k(x)\phi(x+\cdot) \rangle \rangle, $$ if the limit exists for all $\phi \in \mathcal{D}$. {\begin{proposition} [\cite{AKS10}, Proposition 4.7.3 ] If $f\in \mathcal{D}^{\prime}$, then $f*\theta_k\in \mathcal{E}$ with the parameter of constancy at least $-k$. \end{proposition} } We define the {\em multiplication} of $f$ and $g$ by $$ \langle f\cdot g,\phi \rangle =\lim\limits_{k\to \infty} \langle g, ( f*\theta_k)\phi \rangle, $$ if the limit exists for all $\phi \in \mathcal{D}$. The above definition of convolution is compatible with the usual convolution of two integrable functions and the definition of multiplication is compatible with the usual multiplication of two locally integrable functions. The following proposition shows that both the convolution and the multiplication are commutative when they are well defined and the convolution of two distributions is well defined if and only if the multiplication of their Fourier transforms is well defined. \begin{proposition} [\cite{VVZ94}, Sections 7.1 and 7.5] \label{Conv-Mul} Let $f, g\in \mathcal{D}'$ be two distributions. Then\\ \indent {\rm (1)} \ If $f*g$ is well defined, so is $g*f$ and $f*g=g*f$.\\ \indent {\rm (2)} \ If $ f\cdot g$ is well defined, so is $g\cdot f$ and $ f\cdot g= g\cdot f$.\\ \indent {\rm (3)} \ $f*g$ is well defined if and only $\widehat{f}\cdot \widehat{g}$ is well defined. In this case, we have $ \widehat{f*g}=\widehat{f}\cdot\widehat{g}$ and $ \widehat{f\cdot g}=\widehat{f}*\widehat{g}. $ \end{proposition} \section{Proof of Theorem \ref{thm-tilingperiodic}} In this section, we will study the following equation \begin{equation}\label{EQ} f*\nu = w \end{equation} where $ f\in L^1(\Q_p)$ with $\int_{\Q_p}fd\m>0$, and $\nu$ is a locally finite signed measure defined by \eqref{eq-discretemeasure} in $\Q_p$. Our discussion is based on the functional equation \[\widehat{f}\cdot\widehat{\nu}=w\delta_0,\] which is implied by $f*\nu=w$ (see Proposition \ref{Conv-Mul}). \subsection{Zeros of the Fourier transform of the measure $\nu$} Let $\nu$ be a discrete signed measure defined by \eqref{eq-discretemeasure}. The support of $\nu$ is the discrete set $E$. Denote by $\nu_n$ the restriction of $\nu$ on the ball $B(0,p^n)$,i.e. \[ \nu_n=\sum_{x\in E\cap B(0,p^n) } \alpha_{x}\delta_x. \] Consider $\nu$ as a distribution on $\Q_p$. For simplicity, denote by $E_n=E\cap B(0,p^n) $ the restriction of $E$ on the ball $B(0, p^n)$. The following proposition characterizes the structure of $ \mathcal{Z}_{\widehat{\nu}}$, the set of zeros of the Fourier transform of $\nu$. It is bounded and is a union of spheres centered at $0$. Recall that the support of $\nu$ is a locally finite subset. Define \begin{align}\label{def-nv} n_{\nu}:=\inf_{\lambda \in E } \max_{\substack {\lambda^{\prime}\in E\\ \lambda\neq \lambda^{\prime}} } v_p(\lambda- \lambda^{\prime}). \end{align} Remark that $n_{\nu}$ can be $-\infty$. \begin{proposition}\label{zeroofE} Let $\nu$ be a discrete measure defined by \eqref{eq-discretemeasure} in $\Q_p$.\\ \indent {\rm (1)} If $\xi\in \mathcal{Z}_{\widehat{\nu}}$, then $S(0,|\xi|_p)\subset \mathcal{Z}_{\widehat{\nu}}$.\\ \indent {\rm (2)} The set $ \mathcal{Z}_{\widehat{\nu}}$ is contained in $B(0,p^{n_\nu+1})$. \end{proposition} \begin{proof} First remark that by using (\ref{FB}) we get \begin{equation}\label{FofMu} \langle \widehat{\nu}, 1_{B(\xi,p^{-n})} \rangle =\langle \nu, \widehat{1_{B(\xi,p^{-n})}} \rangle =p^{-n}\sum_{x\in E_n} \alpha_x \cdot \overline{\chi(\xi x)}=p^{-n} \widehat{\nu_n}(\xi). \end{equation} This expression will be used several times. \indent {\rm (1)} By definition, $\xi\in \mathcal{Z}_{\widehat{\nu}}$ implies that there exists an integer $n_0$ such that $$\langle \widehat{\nu}, 1_{B(\xi,p^{-n})} \rangle=0 , \quad \forall~ n\geq n_0. $$ By (\ref{FofMu}), this is equivalent to \begin{equation}\label{zeross} \sum_{x\in E_n} \alpha_x \cdot \chi(\xi x)=0 ,\quad \forall~ n\geq n_0. \end{equation} For any $\xi^{\prime}\in S(0,|\xi|_p)$, we have $\xi' = u \xi$ for some $u\in \mathbb{Z}_p^\times$. By Corollary \ref{corollary2} and the equality (\ref{zeross}), we obtain $$ \sum_{x\in E_n}\alpha_x \cdot \chi(\xi^{\prime}x )= \sum_{x\in E_n}\alpha_x \cdot \chi(\xi x)=0,\quad \forall~ n\geq n_0. $$ Thus, again by (\ref{FofMu}), $\langle \widehat{\nu}, 1_{B(\xi',p^{-n})} \rangle=0 $ for $ n\geq n_0$. We have thus proved $S(0,|\xi|_p)\subset \mathcal{Z}_{\widehat{\nu}}$. \indent {\rm (2)} We distinguish two cases based on the value of $n_{\nu}$ : $n_\nu=-\infty$ or $n_\nu>-\infty$. { \bf Case 1: $n_\nu=-\infty$.} Fix $\xi \in \Q_p\setminus\{0\}$, since $n_\nu=-\infty$, there exists $x_0\in E$ such that $$\forall x \in E \setminus \{x_0\}, \quad |x-x_0|_p\geq 1/|\xi|_p.$$ We are going to show that $ \mathcal{Z}_{\widehat{\nu}}\subset \{0\}$, which is equivalent to that for each $\xi \in \Q_p\setminus\{0\},$ \[ \langle \widehat{\nu}, 1_{B(\xi,p^{-n})} \rangle \neq 0\] for sufficient large $n$ such that $x_0\in B(0,p^n)$. In fact, if this is not the case, then by (\ref{FofMu}), $\widehat{\nu_n}(\xi)=0.$ Thus by Lemma \ref{lem-necessary}, we can find $x\in E_n$ such that \[ |x-x_0|=p/|\xi|_p<1/|\xi|_p,\] a contradiction. {\bf Case 2: $n_\nu>-\infty$.} Hence, there exists a point $x_0 \in E$ such that \[n_\nu= \max_{\substack {x\in E\\ x\neq x_0} } v_p(x- x_0).\] It follows that $$\forall x \in E \setminus \{x_0\}, \quad |x-x_0|_p\geq p^{-n_\nu}.$$ We are going to show that $ \mathcal{Z}_{\widehat{\nu}}\subset B(0,p^{n_\nu+1})$, which is equivalent to that $\xi \not \in \mathcal{Z}_{\widehat{\nu}}$ when $|\xi|_p\geq p^{n_\nu+2}$. To this end, we will prove that for all integer $n$ large enough such that $x_0 \in B(0, p^n)$, we have $$\langle \widehat{\nu}, 1_{B(\xi,p^{-n})} \rangle \neq 0.$$ In fact, if this is not the case, then by (\ref{FofMu}), $\widehat{\nu_n}(\xi)=0.$ Thus, by Lemma \ref{lem-necessary}, for the given $x_0$, we can find $x \in E_n$, such that \[|x-x_0|_p\leq p/|\xi|_p <p^{-n_{\nu}},\] a contradiction. \end{proof} \subsection{Proof of Theorem \ref{thm-tilingperiodic}} For a function $g: \Q_p \rightarrow \mathbb{R}$, denote $$ \mathcal{N}_g :=\{x \in \Q_p: g(x)=0\}. $$ If $g\in C(\Q_p)$ is a continuous function, then $\mathcal{N}_g $ is a closed set and $ \mathcal{Z}_g$ is the set of interior points of $\mathcal{N}_g $. However, the support of $g$ as a continuous function is equal to the support of $g$ as a distribution. \begin{proposition}[{\cite[Proposition 3.2]{FFLS}}]\label{mainlem} Let $g\in C(\Q_p)$ be a continuous function and let $G\in \mathcal{D}'$ be a distribution. Suppose that the product $H=g\cdot G$ is well defined. Then $$\mathcal{Z}_{H} \subset \mathcal {N}_{g} \cup \mathcal{Z}_{G}.$$ \end{proposition} For $f\in L^1(\Q_p)$, it is evident that $\widehat{f}$ is a continuous function. Consequently, we obtain the following immediate consequence. \begin{corollary}\label{cor:3.3} If $f*\nu=w$ with $f\in L^1(\mathbb{Q}_p)$, then $\Q_p \setminus \{0\} \subset \mathcal {N}_{\widehat{f}} \cup \mathcal{Z}_{\widehat{\nu}},$ which is equivalent to \begin{align}\label{mainprop} \{\xi\in\Q_p: \widehat{f}(\xi) \neq 0 \} \setminus \{0\} \subset \mathcal{Z}_{\widehat{\nu}}. \end{align} \end{corollary} \begin{proof}[Proof of Theorem \ref{thm-tilingperiodic}] By Proposition \ref{zeroofE}, we have $ \mathcal{Z}_{\widehat{\nu}} \subset B(0,p^{n_v+1})$. According to Corollary \ref{cor:3.3}, ${ \rm supp }(\widehat{f}) \subset B(0,p^{n_v+1})$, which is bounded and, therefore, compact. Following Proposition \ref{Prop-compactConstant}, we conclude that $f$ is uniformly locally constancy. \end{proof} \section{Density of $\nu$ and the proof of Theorem \ref{thm:card}} We say that the discrete measure $\nu$ has a {\em bounded density} if the following limit exists for some $x_0\in \Q_p$, $$ D(\nu):=\lim\limits_{k\to \infty}\frac{ \nu(B(x_0,p^k))}{\m (B(x_0,p^k))}, $$ which is called {\em the density} of the discrete measure $\nu$. Actually, if the limit exists for some $x_0 \in \Q_p$, then it exists for all $x\in \Q_p$ and the limit is independent of $x$. In fact, for any $x_0, x_1\in \Q_p$, when $k$ is large enough such that $|x_0-x_1|_p<p^k$, we have $B(x_0, p^k)=B(x_1, p^k)$. The following theorem gets together some properties of the solution $(f, \nu)$ of the equation \eqref{EQ}, which will be proved in this section. \begin{theorem} \label{M1} Let $f\in L^1(\mathbb{Q}_p)$ with $\int_{\mathbb{Q}_p}fd\m>0$, and $\nu$ be a discrete measure defined by \eqref{eq-discretemeasure} with locally finite support in $\Q_p$. Suppose that the equation {\rm (\ref{EQ})} is satisfied by $f$ and $\nu$ for some $w\geq 0$. Then the following statements hold.\\ \indent {\rm (1)} The set $\mathcal{Z}_{\widehat{\nu}}$ is bounded and it is the union of the punctured ball and some spheres with the same center $0$. \\ \indent {\rm (2)}\ The density $D(\nu)$ exists and equals to $1/\int_{\Q_p} f d\m$. Furthermore, there exists an integer $n_f\in \mathbb{Z}$ such that for all integers $n \geq n_f$ we have $$ \forall \xi \in \Q_p, \quad \nu \big(B(\xi, p^{n})\big) = p^{n} D(\nu). $$ \end{theorem} Theorem \ref{M1} (1) and will be proved in $\S 4.1$, the distribution of $\nu$ will be discussed in $\S 4.2$ and the equality $D(\nu)=w/ \int_{\Q_p} f d \m$ will be proved in $\S 4.3$. \subsection{ Structure of $\mathcal{Z}_{\widehat{\nu}}$} Our discussion is based on the functional equation $\widehat{f}\cdot\widehat{\nu}= w \cdot \delta_0$, which is implied by $f*\mu_\nu=w$ (see Proposition \ref{Conv-Mul}). Notice that $\widehat{f}(0)=\int_{\Q_p} f d \m>0$ and that $\widehat{f}$ is a continuous function. It follows that there exists a small ball where $\widehat{f}$ is nonvanishing. Let \begin{align} \label{nf} n_{f}:=\min \{n\in\mathbb{Z}: \widehat{f}(x)\neq 0,\text{ if } x\in B(0,p^{-n}) \}. \end{align} \begin{proposition}\label{prop:4.2} Let $f\in L^1(\mathbb{Q}_p)$ with $\int_{\mathbb{Q}_p}fd\m>0$, and $\nu$ be a discrete measure defined by \eqref{eq-discretemeasure} with locally a finite support in $\Q_p$. Then, \begin{equation} \label{eq:4.2} B(0,p^{-n_f})\setminus \{0\}\subset \mathcal{Z}_{\widehat{\nu}}. \end{equation} \end{proposition} \begin{proof} Actually, \eqref{eq:4.2} is an immediately consequence of \eqref{mainprop}. \end{proof} By Proposition \ref{zeroofE} and \ref{prop:4.2}, the set $\mathcal{Z}_{\widehat{\nu}}$ is bounded and it is the union of the punctured ball $B(0,p^{-n_f})$ and some spheres with the same center $0$. \subsection{Distribution of $\nu$} The discrete measure $\nu$ involved in the equation (\ref{EQ}) shares the following uniform distribution property. \begin{proposition}\label{Structure} The measure $\nu$ of the ball $B(\xi, p^{n_f})$ is independent of $\xi \in \mathbb{Q}_p$. Consequently, the measure $\nu$ admits a bounded density $D(\nu)$. Moreover, for all integers $n \geq n_f$, we have \begin{equation} \label{numberE} \forall \xi \in \mathbb{Q}_p, \quad \nu\big(B(\xi, p^{n})\big) = p^n D(\nu). \end{equation} \end{proposition} \begin{proof} For any given $\xi \in \Q_p$, let $k=-v_p(\xi)$. If $k\leq n_f$, then $B(0,p^{n_f})=B(\xi,p^{n_f})$. So obviously $\nu\big(B(0,p^{n_f})\big)= \nu\big(B(\xi,p^{n_f})\big)$. Now we suppose $k> n_f$. Then, consider any $\eta$ satisfying $$B(\eta,p^{-k})\subset B(0,p^{-n_f})\setminus \{0\}.$$ By \eqref{eq:4.2} in Proposition \ref{prop:4.2}, we have $\langle \widehat{\nu}, 1_{B(\eta,p^{-k})} \rangle =0$, which by (\ref{FofMu}), is equivalent to \begin{equation}\label{eq:4.4} \sum_{x\in E_k} \alpha_x \cdot \chi(\eta x)=0, \end{equation} where $E_k= {\rm supp} \nu \cap B(0,p^k).$ Taking $\eta= p^{k-1}$ in \eqref{eq:4.4}, we have $ \sum_{x\in E_k} \alpha_x \cdot{\chi(p^{k-1}x)}=0. $ Observe that $$B(0,p^k)= \bigsqcup_{i=0}^{p-1} B(ip^{-k},p^{k-1})$$ and that the function $\chi(p^{k-1}\cdot)$ is constant on each ball of radius $p^{k-1}$. So we have $$ 0=\sum_{x\in E_k}\alpha_x \cdot {\chi(p^{k-1}x)}=\sum_{i=0}^{p-1} {\chi\Big(\frac{i}{p}\Big)}\nu\big(B(ip^{-k},p^{k-1})\big). $$ Applying Lemma \ref{SchLemma}, we obtain \begin{equation} \label{ud1} \nu \big(B(ip^{-k},p^{k-1})\big) =\nu\big(B(jp^{-k},p^{k-1})\big), \quad \forall \ 0\le i, j \le p-1. \end{equation} Similarly, taking $\eta=p^{k-2}$ in (\ref{eq:4.4}), we have $$ 0=\sum_{0\le i, j \le p-1}{\chi\Big(\frac{i}{p^2}+\frac{j}{p}\Big)} \nu \big(B(\frac{i}{p^{k}}+\frac{j}{p^{k-1}}, p^{k-2})\big). $$ Again, Lemma \ref{SchLemma} implies \[ \nu\big(B(\frac{i}{p^{k}}+\frac{j}{p^{k-1}}, p^{k-2}) \big)=\nu\big(B(\frac{i}{p^{k}}+\frac{m}{p^{k-1}}, p^{k-2})\big), \quad \forall \ 0\le i, j,m \le p-1. \] Since $$\sum_{j=0}^{p-1} \nu \big(B(\frac{i}{p^{k}}+\frac{j}{p^{k-1}}, p^{k-2})\big)= \nu\big(B(\frac{i}{p^k}, p^{k-1})\big),$$ by (\ref{ud1}), we get $$ \nu\big(B(\frac{i}{p^{k}}+\frac{j}{p^{k-1}}, p^{k-2}) \big)=\nu\big(B(\frac{l}{p^{k}}+\frac{m}{p^{k-1}}, p^{k-2})\big) \quad \forall\ 0\le i, j, l,m \le p-1. $$ We continue these arguments for all $\eta= p^{k-1}, \dotsc, p^{n_f}$. By induction, we have \begin{align}\label{equalnumber} \nu\big( B(\xi_1, p^{n_f})\big)=\nu\big( B(\xi_2, p^{n_f})\big), \quad \forall \xi_1,\xi_2 \in D(0,p^k). \end{align} Thus by (\ref{equalnumber}), $$ \nu \big(B(\xi, p^{n_f})\big)=\nu \big( B(0, p^{n_f} \big).$$ The formula (\ref{numberE}) follows immediately because each ball of radius $p^n$ with $ n\ge n_f$ is a disjoint union of $p^{n-n_f}$ balls of radius $p^{n_f}$ so that $$ \nu \big( B(0,p^n)\big) = p^{n-n_f} \cdot \nu \big( B(0,p^{n_f})\big) . $$ \end{proof} \subsection {Equality $D(\nu)=w/ \int_{\Q_p} f d\m$} \begin{proposition}\label{fisrtprop} The density $D(\nu)$ of $\nu$ satisfies $$D(\nu)=\frac{w}{\int_{\Q_p}f(x)dx}. $$ \end{proposition} \begin{proof} By the integrability of $f$, the quantity $$ \epsilon_n := \int_{\Q_p\setminus B(0, p^n)} f(x) dx $$ tends to zero as the integer $n\to \infty$. Integrating the equality (\ref{EQ}) over the ball $B(0, p^n)$, we have \begin{eqnarray}\label{ee} w \cdot \m (B(0,p^n)) =\sum_{\lambda \in E} \int_{B(0,p^n)}\alpha_{\lambda}\cdot f(x-\lambda)dx. \end{eqnarray} Now we split the sum in (\ref{ee}) into two parts, according to $\lambda \in E\cap B(0, p^n)$ or $\lambda \in E\setminus B(0, p^n)$. Denote $I:=\int_{\Q_p} f d\m$. For $\lambda \in B(0, p^n)$, we have \begin{eqnarray*} \int_{B(0,p^n)} f(x-\lambda)dx = \int_{B(0,p^n)} f(x)dx = I - \epsilon_n. \end{eqnarray*} It follows that \begin{equation}\label{ee1} \sum_{\lambda \in E\cap B(0, p^n)} \int_{B(0,p^n)} \alpha_{\lambda} \cdot f(x-\lambda)dx = \nu\big( B(0, p^n)\big)\cdot (I-\epsilon_n). \end{equation} Notice that \begin{equation}\label{ee2} \int_{B(0,p^n)} f(x-\lambda)dx = \int_{B(-\lambda,p^n)} f(x)dx. \end{equation} For $\lambda \in E\setminus B(0, p^n)$, the ball $B(-\lambda,p^n)$ is contained in $\Q_p\setminus B(0, p^n)$. We partition the support $\Q_p\setminus B(0, p^n)$ into $B_j$'s such that each $B_j$ is a ball of radius $p^n$. Thus the integrals in (\ref{ee2}) for the $\lambda$'s in the same $B_j$ are equal. Let $\lambda_j$ be a representative of $P_j$. Then we have \begin{eqnarray} \sum_{\lambda \in E\setminus B(0,p^n)} \int_{B(0,p^n)} \alpha_{\lambda} \cdot f(x-\lambda)dx = \sum_j \nu(B_j) \cdot \int_{B(-\lambda_j,p^n)} f(x)dx. \nonumber \end{eqnarray} However, by (\ref{numberE}) in Proposition \ref{Structure}, $\nu (B_j) = D(\nu) \cdot \m\big(B(0, p^n)\big)$ if $n \geq n_f$. Thus, for each integer $n\geq n_f$, \begin{align} \sum_{\lambda\in E\setminus B(0,p^n)} \int_{B(0,p^n)}\alpha_{\lambda} \cdot f(x-\lambda)dx & = D(\nu) \cdot \m\big(B(0, p^n)\big) \sum_j \int_{B_j} f(x)dx \nonumber \\ & = D(\nu)\cdot \m\big(B(0, p^n)\big) \int_{\Q_p \setminus B(0, p^n)} f(x) dx \nonumber \\ &= D(\nu)\cdot \m\big(B(0, p^n)\big)\cdot \epsilon_n. \label{ee3} \end{align} Thus from (\ref{ee}), (\ref{ee1}) and (\ref{ee3}), we finally get $$ \left| w\cdot \m\big(B(0, p^n)\big) - \nu\big( B(0, p^n)\big)\cdot I \right| \le 2 D(\nu) \cdot \m(B(0, p^n))\cdot \epsilon_n. $$ We conclude by dividing $\m\big(B(0, p^n)\big)$ and then letting $n\to \infty$. \end{proof} \subsection{Proof of Theorem \ref{thm:card} } \begin{proof}[Proof of Theorem \ref{thm:card}] Consider the convolution equation $f*\nu=1$, where $f\in L^{1}(\Q_p)$ is a non-negative, $\nu=\sum_{t\in T}\delta_t$ and $T$ is a discrete subset in $\Q_p$. By Proposition \ref{Structure}, for integers $n \geq n_f$, we have \[ \#(B(x,p^n)\cap T) =\#(B(y,p^n)\cap T), \quad \forall x, y \in \mathbb{Q}_p.\] On the other hand, if \[ \#(B(x,p^n)\cap T) = \#(B(y,p^n)\cap T), \quad \forall x, y \in \mathbb{Q}_p.\] for some $n\geq 0$. Take \[f=\frac{1_{B(0,p^n)}}{\#(B(x,p^n)\cap T)}. \] Then $f*\nu=1$ with $\nu=\sum_{t\in T}\delta_t$. \end{proof} \section{The structure of tiles on $\mathbbm {\Z}/ p^{n}q\mathbbm {\Z}$ and $\Z/p^n\Z\times\Z/p\Z$} In this section, we primarily utilize the results established in \cite{FFS} and \cite{FKL} to characterize the structure of tiles on $\mathbbm {\Z}/ p^{n}q\mathbbm {\Z}$. The structural properties of tiles in $\mathbb{Z}/p^n\mathbb{Z}$ are characterized by $p$-homogeneity, as demonstrated in \cite{FFS}. \subsection{ $p$-homogeneity of tiles in $\Z/p^n\Z$} Let $nn$ be a positive integer. To any finite sequence $t_0 t_1 \cdots t_{n-1} \in \{0, 1, \dots, p-1\}^{n}$, we associate the integer \[ c = c(t_0 t_1 \cdots t_{n-1}) = \sum_{i=0}^{n-1} t_i p^i \in \{0, 1, \dots, p^{n} - 1\}. \] This establishes a bijection between ${\Z}/p^{n}{\Z}$ and $\{0, 1, \dots, p-1\}^{n}$, which we consider as a finite tree, denoted by ${\mathcal T}^{(n)}$ (see Figure~\ref{fig:1}). The set of vertices of ${\mathcal T}^{(n)}$ is the disjoint union of the sets ${\Z}/p^\gamma{\Z}$, for $0 \le \gamma \le n$. Each vertex, except the root, is identified with a sequence $t_0 t_1 \cdots t_{\gamma-1}$ in ${\Z}/p^\gamma{\Z}$, where $0 \le \gamma \le n$ and $t_i \in \{0, 1, \dots, p-1\}$. The set of edges consists of pairs $(x, y) \in {\Z}/p^\gamma{\Z} \times {\Z}/p^{\gamma+1}{\Z}$ such that $x \equiv y \pmod{p^n}$, where $0 \le \gamma \le n-1$. Each point $c$ of ${\Z}/p^n{\Z}$ is identified with the boundary point $\sum_{i=0}^{n-1} t_i p^i \in \{0, 1, \dots, p^{n} - 1\}$ of the tree. Each subset $C \subset {\Z}/p^n{\Z}$ determines a subtree of ${\mathcal T}^{(n)}$, denoted by ${\mathcal T}_C$, which consists of the paths from the root to the boundary points in $C$. For each $0 \le \gamma \le n$, we denote by \[ C_{\bmod p^\gamma} := \{x \in \{0, 1, \dots, p^\gamma-1\} : \exists y \in C \text{ such that } x \equiv y \pmod{p^\gamma}\} \] the subset of $C$ modulo $p^\gamma$. The set of vertices of ${\mathcal T}_C$ is the disjoint union of the sets $C_{\bmod p^\gamma}$, for $0 \le \gamma \le n$. The set of edges consists of pairs $(x, y) \in C_{\bmod p^\gamma} \times C_{\bmod p^{\gamma+1}}$ such that $x \equiv y \pmod{p^\gamma}$, where $0 \le \gamma \le n-1$. For vertices $u \in C_{\bmod p^{\gamma+1}}$ and $s \in C_{\bmod p^\gamma}$, we call $s$ the \emph{parent} of $u$ or $u$ the \emph{descendant} of $s$ if there exists an edge between $s$ and $u$. Now, we proceed to construct a class of subtrees of ${\mathcal T}^{(n)}$. Let $I$ be a subset of $\{0, 1, \dots, n-1\}$, and let $J$ be its complement. Thus, $I$ and $J$ form a partition of $\{0, 1, \dots, n-1\}$, and either set may be empty. We say a subtree ${\mathcal T}_C$ of ${\mathcal T}^{(n)}$ is of ${\mathcal T}_{I}$-form if its vertices satisfy the following conditions: \begin{enumerate} \item If $i \in I$ and $t_0 t_1\dots t_{i-1} $ is given, then $t_i$ can take any value in $\{0, 1, \dots, p-1\}$. In other words, every vertex in $C_{\bmod p^{i}}$ has $p$ descendants. \item If $i \in J$ and $t_0 t_1 \dots t_{i-1}$ is given, we fix a value in $\{0, 1, \dots, p-1\}$ that $t_i$ must take. That is, $t_i$ takes only one value from $\{0, 1, \dots, p-1\}$, which depends on $t_0 t_1 \dots t_{i-1}$. In other words, every vertex in $C_{\bmod p^{i}}$ has one descendant. \end{enumerate} \begin{figure} \centering \includegraphics[width=0.7\linewidth]{1} \caption{The set ${\Z}/3^4{\Z}=\{0,1,2,\cdots,80\}$ is considered as a tree ${\mathcal T}^{\left(4\right)} $. } \label{fig:1} \end{figure} \begin{figure} \centering \includegraphics[width=0.7\linewidth]{2} \caption{For $p=3$, a ${{\mathcal T}_{I,J }}$-form tree with $n=5$, $I=\{0,2,4\} ,J=\{1,3\}$.} \label{fig:2} \end{figure} Note that such a subtree depends not only on $I$ and $J$ but also on the specific values assigned to $t_i$ for $i \in J$. A ${\mathcal T}_{I}$-form tree is called a finite \emph{$p$-homogeneous tree.} An example of a ${\mathcal T}_{I}$-form tree is shown in Figure~\ref{fig:2}. A set $C \subset {\Z}/p^n{\Z}$ is said to be \emph{$p$-homogeneous} subset of ${\Z}/p^n{\Z}$ with the \emph{branched level set} $I$ if the corresponding tree ${\mathcal T}_C$ is $p$-homogeneous of form ${\mathcal T}_{I}$. If we consider $C \subset \{0, 1, \dots, p^n - 1\}$ as a subset of ${\Z}_p$, then the tree ${\mathcal T}_C$ can be identified with the finite tree determined by the compact open set \[ \Omega = \bigsqcup_{c \in C} (c + p^n{\Z}_p). \] A criterion for a subset $C \subset \Z/p^n\Z$ to be $p$-homogeneous is given in \cite{FFS}. \begin{theorem}[{\cite[Theorem 2.9]{FFS}}] \label{thm5.4} Let $n$ be a positive integer, and let $C \subseteq {\Z}/p^n{\Z}$ be a multiset. Suppose that \begin{enumerate}[{\rm(1)}] \item $\#C \le p^k$ for some integer $k$ with $1 \le k \le n$; \item there exist $k$ integers $1 \le j_1 < j_2 < \dots < j_k \le n$ such that \[ \sum_{c \in C} e^{2\pi i c p^{-j_t}} = 0 \quad \text{for all } 1 \le t \le k. \] \end{enumerate} Then $\#C = p^k$ and $C$ is $p$-homogeneous. Moreover, the tree ${\mathcal T}_C$ is a ${\mathcal T}_{I}$-form tree with $I = \{j_1-1, j_2-1, \dots, j_k-1\}$. \end{theorem} \begin{theorem}[{\cite[Theorem 4.2]{FFS}}] Let $n$ be a positive integer, and let $C \subseteq {\Z}/p^n{\Z}$. Then $C$ tiles ${\Z}/p^n{\Z}$ if and only if $C$ is p-homogeneous. \end{theorem} \subsection{The structure of tiles in $\Z/p^nq\Z$ and $\Z/p^n\Z\times\Z/p\Z$} Write $\Z/p^nq\Z$ as the product form $\mathbb{Z}/p^n\mathbb{Z} \times \mathbb{Z}/q\mathbb{Z}$. The geometrical characterization of tiles in $\mathbb{Z}/p^n\mathbb{Z} \times \mathbb{Z}/q\mathbb{Z}$ was obtained in \cite{FKL}. \begin{theorem}[\cite{FKL}]\label{prop5.7} Let $A$ be a tile of $\mathbb{Z}/p^n\Z \times \Z/ q\Z$. For $0\leq j \leq q-1$, let $A_j=\{x\in \mathbb{Z}/p^n\Z : (x,j)\in A\}$. \begin{enumerate}[{\rm(1)}] \item If $\#A=p^t$, then $A_j$ are disjoint and $\cup_{j=0}^{q-1}A_j$ tiles $\Z/p^n\Z$ by translation. \item If $\#A=p^tq$, then all $A_j$ can tiles $\mathbb{Z}/p^n\Z $ with a common branched level set. \end{enumerate} \end{theorem} Consider the finite abelian group $\mathbb{Z}/p^n\mathbb{Z} \times \mathbb{Z}/p\mathbb{Z}$. Let $ A$ be a tile of $\mathbb{Z}/p^n\mathbb{Z} \times \mathbb{Z}/p\mathbb{Z}$. It is evident that $\#A$ divide $p^{n+1}$. Hence, assume that $\#A=p^i$ for some $1 \leq i\leq n$. Define a map $\pi_1$ from the group $ \mathbb{Z}/p^n\mathbb{Z} \times \mathbb{Z}/p\mathbb{Z}$ to the group $\Z/p^n\mathbb{Z}$ by \[ \pi_1(a, b)=a, \quad \hbox{ for } (a, b)\in \mathbb{Z}/p^n\mathbb{Z} \times \mathbb{Z}/p\mathbb{Z}. \] Let \[\mathcal{Z}_A=\Big\{g\in \mathbb{Z}/p^n\mathbb{Z} \times \mathbb{Z}/p\mathbb{Z}: \widehat {1_ A}(g)=0 \Big\}\] be the set of zeros of the Fourier transform of the function $1_A$. The geometrical characterization of tiles in $\mathbb{Z}/p^n\mathbb{Z} \times \mathbb{Z}/p\mathbb{Z}$ was established in \cite{FKL}. \begin{theorem}[\cite{FKL}]\label{thmfinite} Assume $A$ is a tile of $\mathbb{Z}/p^n\mathbb{Z} \times \mathbb{Z}/p\mathbb{Z}$ with $\#A=p^t$. Let $$ \mathcal{I}_{A}=\big\{i\in \{0,\cdots \ n-1\}: (p^i, 0)\in \mathcal{Z}_ A \big\}. $$ We distinguish three cases. \begin{enumerate}[{\rm(1)}] \item If $\# \mathcal{I}_{A}=t$, then the set $\pi_1(A)$ is a $p$-{\it homogeneous} in $\Z/p^n\Z$ with $\#\pi_1( A)=p^t.$ \item If $\# \mathcal{I}_{A}=t-1$ and $ (p^j, b)\notin \mathcal{Z}_A$ for each $ j\in \{0,1,\cdots, n\}\setminus \mathcal{I}_{A}$ and $b\in \{1,\cdots,p-1\}$, then the sets \[ A_i=\{x\in \Z/{p^n}\Z: (x, i) \in A\}\] are $p$-homogenous in $\Z/p^n\Z$ with $\#A_i=p^{t-1}$. \item If $\#\mathcal{I}_{A}=t-1$ and $(p^{j}, b) \in \mathcal{Z}_{A}$ for some $j\in \{0,1,\cdots, n\}\setminus \mathcal{I}_{A}$ and $b\in\{1,\cdots,p-1\}$, then the set \[\widetilde{A} = \{ x +b_0 y p^{n-j_0-1}: (x, y) \in A\} \] is $p$-homogeneous in $\Z/{p^n}\Z$ with $\# \widetilde{A}=p^t$, where $j_0$ is the minimal number in $\{0,1,\cdots, n\}\setminus\mathcal{I}_{A}$ such that $(p^{j_0},b_0)\in \mathcal{Z}_A $ for some $ b_0\in \{0, \cdots, p-1\}$. \end{enumerate} \end{theorem} \section{Structure of tiles in $\Q_p\times \Z/2\Z$.} Let $\mu$ be the Haar measure $\Q_p$ such that $\mu(\Z_p)=1$. Equip $\Q_p\times \Z/2\Z$ with the Haar measure $\nu$ with $\nu(\Z_p\times\{0\})=1$. \subsection{Structure of tiles} \begin{proof}[Proof of Theorem \ref{thm1.3}] Let $\left ( \Omega ,T \right ) $ be a tiling pair in $\Q_p\times \Z/2\Z$. Write \[\Omega =\left ( \Omega_{0} \times \left \{ 0 \right \} \right ) \sqcup \left ( \Omega_{1}\times \left \{ 1 \right \} \right ),\] and \[T =\left ( T_{0} \times \left \{ 0 \right \} \right ) \sqcup \left ( T_{1}\times \left \{ 1 \right \} \right ).\] Since $\left ( \Omega ,T \right ) $ be a tiling pair in $\Q_p\times \Z/2\Z$, by definition, we have \begin{equation}\label{equ61} 1_{ \Omega_{0} } \ast \mu_{T_{0} }+1_{ \Omega_{1}}\ast \mu_{T_{1} }=1, \quad \mu-a.e. \end{equation} and \begin{equation}\label{equ62} 1_{ \Omega_{1} } \ast \mu_{T_{0} }+1_{ \Omega_{0}}\ast \mu_{T_{1} }=1, \quad \mu-a.e. \end{equation} where $\mu_T=\sum_{t\in T} \delta_t $ is a discrete measure in $\Q_p$. Next, we will transform the tiling problem on $\mathbb{Q}_p \times \mathbb{Z}/2\mathbb{Z}$ by a set into tiling problem on $\mathbb{Q}_p$ by a function. Adding both sides of Equation~\eqref{equ61} and Equation~\eqref{equ62} separately, we get \begin{equation}\label{equ63} \left ( 1_{ \Omega_{0} } +1_{ \Omega_{1}} \right ) \ast \left ( \mu_{T_{0} } +\mu_{T_{1} } \right ) =2. \end{equation} Subtracting Equation \eqref{equ62} from Equation \eqref{equ61}, we get \begin{equation}\label{equ64} \left ( 1_{ \Omega_{0} } -1_{ \Omega_{1}} \right ) \ast \left ( \mu_{T_{0} } -\mu_{T_{1} } \right ) =0. \end{equation} Hence, by Theorem \ref{thm-tilingperiodic}, we deduce from \eqref{equ63} that the function $ f= 1_{ \Omega_{0} } +1_{ \Omega_{1}}$ is uniformly locally constancy. Then, we have $ \Omega_{0} \cap \Omega_{1} $ and $\left ( \Omega_{0} \setminus \Omega_{1} \right ) \cup \left ( \Omega_{1} \setminus \Omega_{0} \right ) $ are almost compact open. Now, we distinguish two cases: $\left ( 1 \right )$ If $\mu_{T_{0}} -\mu_{T_{1}}=0 $; $\left ( 2 \right )$ If $\mu_{T_{0}} -\mu_{T_{1}}\neq 0 $. $\left ( 1 \right )$ If $\mu_{T_{0}} -\mu_{T_{1}}=0 $, then we deduce from (\ref{equ63}) that $$\left ( 1_{ \Omega_{0} } +1_{ \Omega_{1}} \right ) \ast \mu_{T_{0}}=1.$$ Hence $\left ( \Omega_{0}\cup \Omega_{1} ,T_{0} \right ) $ is a tiling pair in ${\Q}_{p} $ and $ \Omega_{0}\cup \Omega_{1}$ is almost compact open. $\left ( 2 \right )$ If $\mu_{T_{0}} -\mu_{T_{1}}\neq 0 $, then by Theorem \ref{thm-tilingperiodic}, we deduce from \eqref{equ64} that the function $ f= 1_{ \Omega_{0} } -1_{ \Omega_{1}}$ is uniformly locally constancy, which implies that $ \Omega_{0} \setminus \Omega_{1}$ and $ \Omega_{1} \setminus \Omega_{0}$ are almost compact open. Hence, both $ \Omega_0$ and $ \Omega_1$ are compact open. Without loss of generality, assume that $ \Omega_0, \Omega_1$ are compact open set in $\Z_p$. Hence, there exist a positive integer $n$ and $C_0,C_1 \in \Z/p^n \Z$ such that \[ \Omega_0=\bigsqcup_{c \in C_0} c + p^n \mathbb{Z}_p, \quad \Omega_1=\bigsqcup_{c \in C_1} c + p^n \mathbb{Z}_p,\] up to measure zero sets. Note that $ \Omega_0\times\{0\} \cup \Omega_1\times \{1\}$ tiles $\Q_p\times\Z/2\Z$ by translation if and only if $C_0\times\{0\} \cup C_1\times \{1\}$ tiles $\Z/p^n \Z \times\Z/2\Z$ by translation. When $p\geq 3$, by Theorem \ref{prop5.7}, either $C_0\cap C_1=\emptyset$ and $C_0\cup C_1$ tiles $\Z/p^n\Z$ by translation or $C_0, C_1$ can tile $\Z/p^n\Z$ by translation with a common translation set. When $p=2$, by Theorem \ref{thmfinite}, we have three cases: (1) $C_0\cap C_1=\emptyset$ and $C_0\cup C_1$ tiles $\Z/2^n\Z$ by translation, (2) $C_0, C_1$ can tile $\Z/2^n\Z$ by translation with a common translation set, (3) there exist $j_0\in \{0,\cdots, n-1\}$ and such that $C_0$ and $\widetilde{C}_1 = \{ x + 2^{n-j_0-1}: x \in C_1\}$ are disjoint and $C_0\cup \widetilde{C}_1$ tiles $\Z/2^n\Z$ by translation. The combination of the two cases led to our final conclusion. \end{proof} \subsection{Tiles are spectral in $\Q_p\times \Z/2\Z$} It is proved in \cite{FFS,FFLS}that Fuglede conjecture holds in $\Q_p$, spectral sets are compact open and are characterized by their $p$-homogeneity. \begin{theorem}[\cite{FFS}]\label{thm6} Let $\Omega $ be a compact open set in $\Q_{p}$. The following statements are equivalent: \begin{enumerate}[{\rm(1)}] \item $\Omega $ is a spectral set; \item $\Omega $ tiles $\Q_{p}$ by translation; \item $\Omega $ is $p$-homogeneous. \end{enumerate} \end{theorem} Let $E$ be a subset of $\Q_{p}$. Define $$I_{E} =\left \{ i\in \Z:\:\exists \: x,y\in E \: such \:that \: v_{p}\left ( x-y \right )=i \right \} $$ as the set of admissible $p$-orders corresponding to the set $E$. The structure of spectra for spectral sets is also characterized. \begin{theorem}[\cite{FFS} Theorem 5.1]\label{thm5} Let $\Omega \subset \Q_{p} $ be a $p$-homogeneous compact open set with the admissible $p$-order set $I_{\Omega } $. \begin{enumerate}[{\rm(1)}] \item The set $\Lambda $ is a spectrum of $\Omega$ if and only if it is $p$-homogeneous discrete set with admissible $p$-order set $I_{\Lambda }=-\left ( I_{\Omega }+1 \right ) $. \item The set $T$ is a tiling complement of $\Omega$ if and only if it is a $p$-homogeneous discrete set with admissible $p$-order set $I_{T} =\Z \setminus I_{\Omega }$. \end{enumerate} \end{theorem} Let $\Omega\subset \Q_p\times \Z/2\Z$ be a tile. Write $\Omega$ as \[\Omega =\left ( \Omega_{0} \times \left \{ 0 \right \} \right ) \sqcup \left ( \Omega_{1}\times \left \{ 1 \right \} \right ).\] When $p>2$, by Theorem \ref{thm1.3}, there are two mutually exclusive cases: {\rm (1)} $\mu(\Omega_0\cap\Omega_1)=0$ and $ \Omega_0\cup \Omega_1$ can tile $\Q_p$ by translation, {\rm (2)} $\Omega_0$ and $\Omega_1$ can tiles $\Q_p$ with a common translation set $T_0\subset \Q_p$. For the first case, $\Omega_0\cup \Omega_1 $ tiles $\Q_p$ by translation. Assume that $(\Omega_0\cup \Omega_1,\Lambda)$ is a spectral pair. Therefore, $(\Omega, \Lambda \times \left \{ 0 \right \})$ is a spectral pair in $\Q_p\times \Z/2\Z$. For the second case, the set $\Omega_0$ tiles $\Q_p$ by translation. It follows that $\Omega_0$ and $\Omega_1$ are spectral sets in $\Q_p$ with a common spectrum $\Lambda$. Therefore, $(\Omega,\left ( \Lambda \times \left \{ 0 \right \} \right ) \sqcup \left ( \Lambda \times \left \{ 1 \right \} \right ))$ is also a spectral pair. When $p=2$, by Theorem \ref{thm1.3}, there are three cases. For the first two cases, it is similar as $p>2$. For the third case, by Theorem \ref{thm1.3}, $\Omega_0$ and $\Omega_1$ are compact open up to measure zero sets. Without loss of generality, assume that $ \Omega_0, \Omega_1$ are compact open set in $\Z_2$. Hence, there exist a positive integer $n$ and $C_0,C_1 \in \Z/2^n \Z$ such that \[ \Omega_0=\bigsqcup_{c \in C_0} c + 2^n \mathbb{Z}_2, \quad \Omega_1=\bigsqcup_{c \in C_1} c + 2^n \mathbb{Z}_2.\] Hence, $C=C_{0}\times\{0\} \cup C_{0}\times\{1\}$ tiles $\Z/2^n \Z \times \Z/2\Z$ by translation. Let $$ \mathcal{I}_{C}=\big\{i\in \{0,\cdots \ n-1\}: (2^i, 0)\in \mathcal{Z}_ C \big\}, $$ and let $j_0\in \{0,1,\cdots, n-1\}\setminus \mathcal{I}_{C}$ such that $(2^{j_0},1)$ in $\mathcal{Z}_C$ and $(2^i, 1)\notin \mathcal{Z}_C$ for $i\in \mathcal{I}_{C}$ with $i<j_0$. Define $$ \Lambda=\Big\{(\sum \limits_{i\in \mathcal{I}_{C}}s_ip^{i-n}, 0)+s_{j_0}(p^{j_0-n},1): s_i, s_j \in \{0, 1, \cdots, p-1\} \Big\}+(\mathbb{L}_{n}\times\{0\}), $$ where \[\mathbb{L}_n=\{\frac{k}{p^{l}}: l> n, 1\leq k\leq p^{l} \hbox{ and } \gcd(k,p)=1\}.\] Hence, $(\Omega, \Lambda)$ forms a spectral pair of $\Q_2\times \Z/2\Z$. \begin{thebibliography}{10} \bibitem{AKS10} S.~Albeverio, A.~Yu. Khrennikov, and V.~M. Shelkovich. \newblock {\em Theory of {$p$}-adic distributions: linear and nonlinear models}, volume 370 of {\em London Mathematical Society Lecture Note Series}. \newblock Cambridge University Press, Cambridge, 2010. \bibitem{MR3684890} C.~Aten, B.~Ayachi, E.~Bau, D.~FitzPatrick, A.~Iosevich, H.~Liu, A.~Lott, I.~MacKinnon, S.~Maimon, S.~Nan, J.~Pakianathan, G.~Petridis, C.~Rojas~Mena, A.~Sheikh, T.~Tribone, J.~Weill, and C.~Yu. \newblock Tiling sets and spectral sets over finite fields. \newblock {\em J. Funct. Anal.}, 273(8):2547--2577, 2017. \bibitem{MR4402630} Thomas Fallon, Gergely Kiss, and G\'{a}bor Somlai. \newblock Spectral sets and tiles in {$\Bbb Z_p^2 \times \Bbb Z_q^2$}. \newblock {\em J. Funct. Anal.}, 282(12):Paper No. 109472, 16, 2022. \bibitem{Fan15} Aihua Fan. \newblock Spectral measures on local fields. \newblock In {\em Difference equations, discrete dynamical systems and applications}, volume 150 of {\em Springer Proc. Math. Stat.}, pages 15--35. Springer, Cham, 2015. \bibitem{FFLS} Aihua Fan, Shilei Fan, Lingmin Liao, and Ruxi Shi. \newblock Fuglede's conjecture holds in {$\Bbb {Q}_p$}. \newblock {\em Math. Ann.}, 375(1-2):315--341, 2019. \bibitem{FFS} Aihua Fan, Shilei Fan, and Ruxi Shi. \newblock Compact open spectral sets in {$\Bbb Q_p$}. \newblock {\em J. Funct. Anal.}, 271(12):3628--3661, 2016. \bibitem{FKL} Shilei Fan, Mamateli Kadir, and Peishan Li. \newblock The structure of tiles in $\mathbb{Z}_{p^n}\times\mathbb{Z}_q$ and $\mathbb{Z}_{p^n}\times\mathbb{Z}_p$. \newblock {\em arXiv:2411.02696}. \bibitem{MR2267631} B\'{a}lint Farkas, M\'{a}t\'{e} Matolcsi, and P\'{e}ter M\'{o}ra. \newblock On {F}uglede's conjecture and the existence of universal spectra. \newblock {\em J. Fourier Anal. Appl.}, 12(5):483--494, 2006. \bibitem{MR2221543} B\'{a}lint Farkas and Szil\'{a}rd~Gy. R\'{e}v\'{e}sz. \newblock Tiles with no spectra in dimension 4. \newblock {\em Math. Scand.}, 98(1):44--52, 2006. \bibitem{MR0470754} Bent Fuglede. \newblock Commuting self-adjoint partial differential operators and a group theoretic problem. \newblock {\em J. Functional Analysis}, 16:101--121, 1974. \bibitem{HN79} B.~Helffer and J.~Nourrigat. \newblock Caracterisation des op\'{e}rateurs hypoelliptiques homog\`enes invariants \`a gauche sur un groupe de {L}ie nilpotent gradu\'{e}. \newblock {\em Comm. Partial Differential Equations}, 4(8):899--958, 1979. \bibitem{MR3649367} Alex Iosevich, Azita Mayeli, and Jonathan Pakianathan. \newblock The {F}uglede conjecture holds in {$\Bbb{Z}_p\times\Bbb{Z}_p$}. \newblock {\em Anal. PDE}, 10(4):757--764, 2017. \bibitem{MR4085122} Gergely Kiss, Romanos~Diogenes Malikiosis, G\'{a}bor Somlai, and M\'{a}t\'{e} Vizer. \newblock On the discrete {F}uglede and {P}ompeiu problems. \newblock {\em Anal. PDE}, 13(3):765--788, 2020. \bibitem{MR4493731} Gergely Kiss, Romanos~Diogenes Malikiosis, G\'{a}bor Somlai, and M\'{a}t\'{e} Vizer. \newblock Fuglede's conjecture holds for cyclic groups of order {$pqrs$}. \newblock {\em J. Fourier Anal. Appl.}, 28(5):Paper No. 79, 23, 2022. \bibitem{MR1772427} Mihail~N. Kolountzakis. \newblock Non-symmetric convex domains have no basis of exponentials. \newblock {\em Illinois J. Math.}, 44(3):542--550, 2000. \bibitem{KL96Duke} Mihail~N. Kolountzakis and Jeffrey~C. Lagarias. \newblock Structure of tilings of the line by a function. \newblock {\em Duke Math. J.}, 82(3):653--678, 1996. \bibitem{KL21} Mihail~N. Kolountzakis and Nir Lev. \newblock Tiling by translates of a function: results and open problems. \newblock {\em Discrete Anal.}, pages Paper No. 12, 24, 2021. \bibitem{MR2264214} Mihail~N. Kolountzakis and M\'{a}t\'{e} Matolcsi. \newblock Complex {H}adamard matrices and the spectral set conjecture. \newblock {\em Collect. Math.}, (Vol. Extra):281--291, 2006. \bibitem{MR2237932} Mihail~N. Kolountzakis and M\'{a}t\'{e} Matolcsi. \newblock Tiles with no spectra. \newblock {\em Forum Math.}, 18(3):519--528, 2006. \bibitem{MR1895739} I.~\L~aba. \newblock The spectral set conjecture and multiplicative properties of roots of polynomials. \newblock {\em J. London Math. Soc. (2)}, 65(3):661--671, 2002. \bibitem{LW96Invent} Jeffrey~C. Lagarias and Yang Wang. \newblock Tiling the line with translates of one tile. \newblock {\em Invent. Math.}, 124(1-3):341--365, 1996. \bibitem{LM91} Horst Leptin and Detlef M\"{u}ller. \newblock Uniform partitions of unity on locally compact groups. \newblock {\em Adv. Math.}, 90(1):1--14, 1991. \bibitem{MR4422438} Romanos~Diogenes Malikiosis. \newblock On the structure of spectral and tiling subsets of cyclic groups. \newblock {\em Forum Math. Sigma}, 10:Paper No. e23, 42, 2022. \bibitem{MR3695475} Romanos~Diogenes Malikiosis and Mihail~N. Kolountzakis. \newblock Fuglede's conjecture on cyclic groups of order {$p^n q$}. \newblock {\em Discrete Anal.}, pages Paper No. 12, 16, 2017. \bibitem{MR2159781} M\'{a}t\'{e} Matolcsi. \newblock Fuglede's conjecture fails in dimension 4. \newblock {\em Proc. Amer. Math. Soc.}, 133(10):3021--3026, 2005. \bibitem{Sch64} I.~J. Schoenberg. \newblock A note on the cyclotomic polynomial. \newblock {\em Mathematika}, 11:131--136, 1964. \bibitem{MR4186120} Ruxi Shi. \newblock Equi-distributed property and spectral set conjecture on {$\Bbb Z_{p^2} \times \Bbb Z_p$}. \newblock {\em J. Lond. Math. Soc. (2)}, 102(3):1030--1046, 2020. \bibitem{Tai75} M.~H. Taibleson. \newblock {\em Fourier analysis on local fields}. \newblock Princeton University Press, Princeton, NJ; University of Tokyo Press, Tokyo, 1975. \bibitem{MR2067470} Terence Tao. \newblock Fuglede's conjecture is false in 5 and higher dimensions. \newblock {\em Math. Res. Lett.}, 11(2-3):251--258, 2004. \bibitem{VVZ94} V.~S. Vladimirov, I.~V. Volovich, and E.~I. Zelenov. \newblock {\em {$p$}-adic analysis and mathematical physics}, volume~1 of {\em Series on Soviet and East European Mathematics}. \newblock World Scientific Publishing Co., Inc., River Edge, NJ, 1994. \bibitem{MR4604197} Tao Zhang. \newblock Fuglede's conjecture holds in {$\Bbb Z_p\times\Bbb Z_{p^n}$}. \newblock {\em SIAM J. Discrete Math.}, 37(2):1180--1197, 2023. \end{thebibliography} \end{document}
2412.03806v1
http://arxiv.org/abs/2412.03806v1
Dynamical Persistent Homology via Wasserstein Gradient Flow
\documentclass[twoside,11pt]{article} \usepackage[abbrvbib, preprint]{jmlr2e} \usepackage{jmlr2e} \newcommand{\dataset}{{\cal D}} \newcommand{\fracpartial}[2]{\frac{\partial #1}{\partial #2}} \usepackage{amsmath} \usepackage{amssymb} \usepackage{algorithm} \usepackage[noend]{algpseudocode} \DeclareMathOperator*{\argmin}{argmin} \usepackage{lastpage} \usepackage{tikz} \usetikzlibrary{matrix, shapes, arrows.meta, calc, decorations.pathreplacing, positioning, quotes, fit} \tikzset{>=latex} rstpageno{1} \begin{document} \title{\scalebox{1.15}{\large Dynamical Persistent Homology via Wasserstein Gradient Flow}} \author{\name Minghua Wang \email [email protected] \\ \addr Department of Computer Science and Engineering\\ State University of New York at buffalo \AND \name Jinhui Xu \email [email protected]\\ \addr Department of Computer Science and Engineering\\ State University of New York at Buffalo} \editor{My editor} \maketitle \begin{abstract} In this study, we introduce novel methodologies designed to adapt original data in response to the dynamics of persistence diagrams along Wasserstein gradient flows. Our research focuses on the development of algorithms that translate variations in persistence diagrams back into the data space. This advancement enables direct manipulation of the data, guided by observed changes in persistence diagrams, offering a powerful tool for data analysis and interpretation in the context of topological data analysis. \end{abstract} \begin{keywords} persistent homology, optimal transport, Wasserstein gradient flow \end{keywords} \section{Introduction} \label{sec:intro} Persistent homology \citep{edelsbrunner2000topological} analyzes the multi-scale topological features of data, and a persistence diagram summarizes these features by keeping their birth and death across a filtration. Persistence diagram is also the most common topological feature that applied in topological data analyais (TDA) machine learning whose typical extraction can be presented as: \[ \text{Data} \rightarrow \text{Simplicial Complex} \xrightarrow{\operatorname{Dgm}_p \circ F} \text{Persistence Diagram}. \] Here, we begin with input data, which can be point clouds, graphs, images, or manifolds, and use these to construct abstract simplicial complexes. A filtration, denoted by \( F \), creates a sequence of nested simplicial complexes from the data. With this structure, we use a matrix reduction algorithm \citep{edelsbrunner2022computational} to compute the persistence diagram, \(\text{Dgm}_p\), for the \( p \)-th homology group. The persistence diagram is a collection of points, each representing the birth and death of a topological feature. Traditionally, the pipeline flows in one direction: from data to persistence diagrams. However, recent research highlights the need to reverse this process, allowing for filtration or even data adjustments through manipulation of persistence diagrams \citep{gameiro2016continuation, hofer2020graph, oudot2020inverse, ballester2024expressivity}. This paper focuses on adapting data based on the dynamics of persistence diagrams along Wasserstein gradient flows. This builds on prior work by using Wasserstein gradient flows to guide the process of modifying data to achieve desired topological characteristics. By developing algorithms that translate changes in persistence diagrams back to the data space, this research enables data analysis and interpretation through TDA. \section{Related Work} As mentioned in Introduction~\ref{sec:intro}, persistence diagrams are a powerful tool for summarizing topological information \citep{edelsbrunner2000topological, edelsbrunner2022computational, chazal2018density}. They have found applications in diverse fields including shape analysis \citep{poulenard2018topological}, image analysis \citep{li2014persistence, singh2007topological}, graph classification \citep{li2012effective, rieck2019persistent, carriere2020perslay, wangpersistent}, link prediction \citep{yan2021link, aktas2019persistence}, and various machine learning tasks \citep{chen2021topological, chen2021z, chen2024topogcl, wang2024multiset}. One key aspect of persistence diagrams is their stability with respect to perturbations in the data they are built upon. The most common metric used to compare persistence diagrams is the bottleneck distance, which corresponds to the $\infty$-Wasserstein distance. The stability of persistence diagrams under the bottleneck distance has been well studied \citep{cohen2005stability,edelsbrunner2006persistence,chazal2009proximity}. However, the bottleneck distance can be sensitive to outliers and often provides overly pessimistic bounds, especially in high-dimensional settings. Recent work has shifted focus to the use of $p$-Wasserstein distances, A central challenge in this area has been establishing the stability of persistence diagrams with respect to the $p$-Wasserstein distance\citep{skraba2020wasserstein}. The differentiability of persistence diagrams is another important consideration for optimization problems. Early work relied on explicitly computing the derivatives of persistence diagrams with respect to function values, often requiring combinatorial updates \citep{poulenard2018topological}. More recently, researchers have leveraged tools from real analytic geometry and o-minimal theory to establish the differentiability of persistence-based functions. \citet{carriere2021optimizing} demonstrated that the persistence map can be viewed as a semi-algebraic map between Euclidean spaces, enabling the definition and computation of gradients for a wide range of persistence-based functions. Moreover, \citet{leygonie2022differential, leygonie2022framework} not only provide a theoretical foundation for the application of gradient-based optimization techniques to persistence-based problems, but also establish a framework for understanding how changes in data parameters impact persistence diagrams. Several previous studies have explored methods for manipulating data through modifications to persistence diagrams. \citet{gameiro2016continuation} developed a method for point cloud continuation by using the Newton-Raphson method to move a persistence diagram towards a target diagram, iteratively updating the point cloud to achieve the desired topological features. \citet{poulenard2018topological} presented an approach for optimizing real-valued functions based on topological criteria, using derivatives of persistence diagrams to guide the modification of the function and achieve desired topological properties. \citet{corcoran2020regularization} focused on regularizing the computation of persistence diagram gradients to improve the optimization process in data science applications. These works demonstrate the potential of using persistence diagrams as a tool for manipulating data to achieve desired topological characteristics. The current work aims to build upon these efforts by developing novel algorithms that translate variations in persistence diagrams back into the data space, with a focus on the $p$-Wasserstein distance and gradient-based optimization techniques. \section{Preliminaries} This study's theoretical foundation rests on two key areas: Wasserstein gradient flows (WGFs) and differentiable persistence diagrams. In this Preliminaries section, we first introduce the concept of gradient flows in Hilbert spaces, providing the necessary mathematical background. We then extend this to gradient flows in Wasserstein spaces, a crucial framework for our analysis. Finally, we present the fundamentals of differentiable persistent homology, an essential tool in topological data analysis that connects persistent homology with differentiable optimization techniques. These concepts form the basis for the methodologies and analyses presented in subsequent chapters. \subsection{Gradient Flow in Hilbert Space} Gradient flow in Hilbert space \citep{ambrosio2008gradient} is a fundamental concept in the analysis of variational problems and partial differential equations. It describes the evolution of a function in a Hilbert space under the influence of its gradient, leading to a minimization of the associated energy functional. Let $\mathcal{H}$ be a Hilbert space with inner product $\langle \cdot, \cdot \rangle$ and norm $\|\cdot\|$. Consider an energy functional $E: \mathcal{H} \to \mathbb{R}$, which is typically assumed to be Fréchet differentiable. The gradient flow of $E$ is the solution to the differential equation \begin{equation} \left\{\begin{array}{l} \dfrac{d u(t)}{dt} = -\nabla E(u(t))\quad \text{for } t>0,\\[0.5em] u(0) = u_0 \end{array}\right. \label{eq:hilbert_gradient_flow} \end{equation} where $u(t) \in \mathcal{H}$ represents the state of the system at time $t$, and $\nabla E(u)$ denotes the gradient of $E$ at $u$. The gradient $\nabla E(u)$ is defined by the property that for all $v \in \mathcal{H}$, \begin{equation} \left. \frac{d}{d\epsilon} E(u + \epsilon v) \right|_{\epsilon=0} = \left\langle \nabla E(u), v \right\rangle. \end{equation} This ensures that the direction of the gradient $\nabla E(u)$ is the direction of the steepest ascent of the functional $E$. Consequently, the negative gradient $-\nabla E(u)$ points in the direction of the steepest descent, which is why it appears in Equation~\eqref{eq:hilbert_gradient_flow}. One of the key properties of gradient flow is that it decreases the energy functional over time. Specifically, if $u(t)$ is a solution to Equation~\eqref{eq:hilbert_gradient_flow}, then \begin{equation} \frac{d}{dt} E(u(t)) = \left\langle \nabla E(u(t)), \frac{d u(t)}{dt} \right\rangle = -\|\nabla E(u(t))\|^2 \leq 0. \label{eq:hilbert_gradient} \end{equation} This implies that $E(u(t))$ is non-increasing along the curve \(u(t)\). Moreover, since \(\frac{d}{d t} E(u(t))=0 \Leftrightarrow\|\nabla E(u(t))\|=0\), if \(E\) has a unique stationary point \(u^\star\), $u(t)$ converges to it as $t \to \infty$ under appropriate conditions. \subsection{Proximal Map in Hilbert Space} From a computational perspective, discretization schemes are essential for approximating gradient flows in Hilbert spaces. The implicit Euler scheme, also known as the minimizing movement scheme or proximal map \citep{bubeck2015convex}, is particularly effective for this purpose. This scheme approximates the continuous-time gradient flow by a sequence of discrete minimization problems: \begin{equation} u_{k+1} = \arg\min_{v \in \mathcal{H}} \left( \frac{1}{2\tau} \|u_k - v\|^2 + E(v) \right) \end{equation} where $\tau > 0$ is the time step, $u_k$ represents the approximation at the $k$-th time step, $E$ is the energy functional, and $\mathcal{H}$ is the Hilbert space. This scheme not only provides a practical method for numerical computations but also offers insights into the theoretical properties of gradient flows, such as their connection to proximal operators and convex analysis. \subsection{Gradient Flow in Wasserstein Space} \label{sec:gfw} Gradient flow in Wasserstein space extends the concept of gradient flow from Hilbert spaces to the space of probability measures. This framework is particularly valuable for studying partial differential equations and variational problems involving mass transport. While this subsection provides a brief introduction, readers are encouraged to consult \citet{ambrosio2008gradient} for a more comprehensive treatment. The Wasserstein space is defined as the space of probability measures with finite second moments, equipped with the Wasserstein distance. More formally, let $(X, d)$ be a metric space. The Wasserstein space of order $p$ over $(X, d)$, denoted by $\mathcal{P}_p(X)$, is the set of all probability measures $\mu$ on $X$ with finite $p$-th moment: \begin{equation} \mathcal{P}_p(X) = \left\{ \mu \in \mathcal{P}(X) \mathrel{\mid} \int_X d(x, x_0)^p \, d\mu(x) < \infty \right\} \label{eq:wass_space} \end{equation} where $x_0$ is a fixed reference point in $X$. This definition ensures that the measures in $\mathcal{P}_p(X)$ have well-behaved moments, which is crucial for various analytical properties. The Wasserstein distance of order $p$ between two probability measures $\mu, \nu \in \mathcal{P}_p(X)$ is defined as: \[W_p(\mu, \nu) = \left( \inf_{\pi \in \Pi(\mu, \nu)} \int_{X \times X} d(x, y)^p \, d\pi(x, y) \right)^{\frac{1}{p}}\] where $\Pi(\mu, \nu)$ represents the set of all joint probability measures on $X \times X$ with marginals $\mu$ and $\nu$. This distance metric quantifies the optimal cost of transporting mass from one probability distribution to another, providing a natural way to compare and analyze probability measures in the context of gradient flows. With a series of works by \citet{benamou2000computational, otto2001geometry, ambrosio2008gradient}, the concept of gradient flow in Wasserstein space has been rigorously established. Consider an energy functional $J: \mathcal{P}_2(\mathbb{R}^d) \to \mathbb{R}$ defined on a Wasserstein space, where $\mathcal{P}_2(\mathbb{R}^d)$ denotes the space of probability measures with finite second moments as in Equation~\eqref{eq:wass_space}. A curve $(\mu_t)_{t \geq 0}$ of probability measures is called a Wasserstein gradient flow for the functional $J$ if it satisfies the following continuity equation in a weak sense: \begin{equation} \frac{\partial \mu_t}{\partial t} = \nabla \cdot \left( \mu_t \nabla \frac{\delta J}{\delta \mu}(\mu_t) \right). \label{eq:wass_gd} \end{equation} Here, $\frac{\delta J}{\delta \mu}: \mathbb{R}^d \rightarrow \mathbb{R}$ represents the first variation of $J$ at $\mu$, defined by \[ \frac{d}{d\varepsilon} J(\mu + \varepsilon \xi) \bigg|_{\varepsilon=0} = \int \frac{\delta J}{\delta \mu} d\xi = \left\langle \frac{\delta J}{\delta \mu}, \xi \right\rangle, \quad \text{for all } \xi \text{ with } \int \mathrm{d} \xi=0. \] This first variation can be interpreted as the ``functional derivative" of $J$ with respect to $\mu$, extending the concept of derivatives to measure spaces. Consequently, $\nabla\frac{\delta J}{\delta \mu} (\mu_t): \mathbb{R}^d \rightarrow \mathbb{R}^d$ for $t \geq 0$ represents a time-dependent family of vector fields. \citet{chewi2020svgd} refer to this as the Wasserstein gradient, denoted as \(\nabla_{W_2} (\mu_t):= \nabla\frac{\delta J}{\delta \mu} (\mu_t)\). This term draws an analogy with the standard gradient in Euclidean spaces due to its similar role in describing the flow of probability measures. Additionally, some literature defines the Wasserstein gradient as \(\nabla_{W_2} (\mu_t):= -\nabla \cdot \left( \mu_t \nabla \frac{\delta J}{\delta \mu}(\mu_t)\right)\) to maintain consistency with Equation~\eqref{eq:hilbert_gradient_flow}. \subsection{JKO Scheme} The JKO scheme, named after Jordan, Kinderlehrer, and Otto, is a time-discretization method for approximating the Wasserstein gradient flow \citep{jordan1998variational}, It involves solving a sequence of minimization problems: \[\mu_{k+1} = \arg \min_{\mu} \left( \frac{1}{2\tau} W_2^2(\mu, \mu_k) + J(\mu) \right),\] where \(\tau\) is the time step. The transition from the continuous Wasserstein gradient flow to the discrete JKO scheme can be understood through the concept of minimizing movement which approximates the continuous flow by discrete steps. The convergence of the JKO scheme to the continuous Wasserstein gradient flow as \(\tau \to 0\) is established through \(\Gamma\)-convergence \citep{ambrosio2008gradient}. The functional \(J(\mu)\) often represents the internal energy or potential energy of the system, and its gradient with respect to the Wasserstein metric drives the evolution of the distribution \citep{villani2009optimal,villani2021topics}. The JKO scheme is particularly useful in numerical simulations of diffusion processes and other phenomena described by partial differential equations (PDEs) in the Wasserstein space \citep{santambrogio2015optimal}. Applications of the JKO scheme include image processing, machine learning, and fluid dynamics, where the evolution of distributions is of interest \citep{peyre2019computational}. \subsection{Differentiability on Persistence Diagrams} \citet{nigmetov2024topological} introduces a novel method that directly uses the cycles and chains involved in the persistence computation to define gradients for larger subsets of the data. This contrasts with prior methods that focused solely on individual PD points. The authors also demonstrate that this approach can significantly reduce the number of steps required for optimization. Our work builds upon this idea, allowing it to adapt data based on variations in PDs along Wasserstein gradient flows. \section{Methodology} We introduce our basic settings as follows. Let a function \( f: \mathcal{K} \to \mathbb{R} \), where \( \mathcal{K} \) denotes a simplicial complex, induce a filtration. This function assigns a real value to each simplex in the complex, thereby defining a sequence of subcomplexes \( \mathcal{K}(t) = \{ \sigma \in \mathcal{K} \mid f(\sigma) \leq t \} \) for \( t \in \mathbb{R} \). Let \(\operatorname{dgm}_p(\mathcal{K}, f)\) be the algorithm to compute \(p\)-th persistence homology based on \citet[Algorithm 1]{nigmetov2024topological}. In this study, we don't take essential features into account. To construct the dynamical persistence diagram, we consider two situations: with or without a target persistence diagram. \subsection{Dynamical Persistent Homology via McCann Interpolation} In this subsection, we assume that the target persistence diagram is known \textit{a priori}. Our objective is to develop a learnable process for generating persistence diagrams guided by a dynamical system. To approach this problem, we draw upon two fundamental theorems from optimal transport theory: the Benamou-Brenier Theorem and the McCann Interpolation Theorem. The Benamou-Brenier Theorem provides a dynamic formulation of optimal transport, allowing us to view the transport problem as a fluid flow minimizing a kinetic energy functional. Meanwhile, the McCann Interpolation Theorem offers a way to construct geodesics in the space of probability measures, which is crucial for understanding the geometry of persistence diagrams. By leveraging these theorems, we can formulate a continuous evolution of persistence diagrams that converges to the target diagram while respecting the underlying topological structure of the data. The Benamou-Brenier theorem shows that the optimal transport cost can be expressed as the infimum of the action integral over all possible time-dependent probability measures \(\mu_t\) and velocity fields \(v_t\) that transport \(\mu_0\) to \(\mu_1\). \begin{theorem}[Benamou-Brenier \citep{benamou2000computational}] \label{thm:bb} Let $\mu_0$ and $\mu_1$ be two probability measures on $\mathbb{R}^d$ with finite second moments. The squared Wasserstein distance $W_2^2(\mu_0, \mu_1)$ between $\mu_0$ and $\mu_1$ can be expressed as: \begin{equation} W_2^2(\mu_0, \mu_1) = \inf_{(\mu_t, v_t)} \int_0^1 \int_{\mathbb{R}^d} \|v_t(x)\|^2 \, d\mu_t(x) \, dt \label{eq:bb_main} \end{equation} subject to the continuity constraints: \begin{equation} \frac{\partial \mu_t}{\partial t} + \nabla \cdot (\mu_t v_t) = 0. \label{eq:bb_continuity} \end{equation} \end{theorem} On the other hand, McCann's interpolation theorem describes the displacement interpolation between two probability measures in the context of optimal transport, showing that the interpolation is a geodesic in the Wasserstein space. \begin{theorem}[McCann Interpolation \citep{villani2021topics}] \label{thm:mccann} Let $\mu_0$ and $\mu_1$ be two probability measures on $\mathbb{R}^d$ with finite second moments, and let $T$ be the optimal transport map pushing $\mu_0$ forward to $\mu_1$. Define the displacement interpolation $\mu_t$ for $t \in [0,1]$ by: \begin{equation} \mu_t = ((1-t) \operatorname{Id} + tT)\# \mu_0, \label{eq:McCann_interplotation} \end{equation} where $\#$ denotes the push-forward operation, $\operatorname{Id}$ is the identity map and $(1-t) \operatorname{Id} + tT$ is the interpolation map. Then: \begin{enumerate} \item $\mu_t$ is a probability measure for each $t \in [0,1]$, \item The curve $\{\mu_t\}_{t \in [0,1]}$ is a geodesic in the Wasserstein space $(\mathcal{P}_2(\mathbb{R}^d), W_2)$, \item The Wasserstein distance between $\mu_0$ and $\mu_1$ can be expressed as: \begin{equation} W_2^2(\mu_0, \mu_1) = \int_{\mathbb{R}^d} \|x - T(x)\|^2 \, d\mu_0(x). \end{equation} \end{enumerate} \end{theorem} Therefore, we have the following remark. \begin{remark}[McCann Interpolation is a WGF~\citep{ambrosio2008gradient}] \label{rem:mccann-bb} Let \(T_t = (1-t)\operatorname{Id} + tT\). Then, \((\mu_t, v_t)\) given by McCann interpolation \begin{align} \mu_t & = T_t\#\mu_0 \\ v_t(x) & = \left\{\begin{array}{ll} T(x_0) - x_0 & \text{if } x=T_t (x_0), \text{where } x_0 \text{ is the initial position to } x.\\ 0 & \text{otherwise} \end{array}\right. \end{align} satisfies Equations~\eqref{eq:bb_main} and \eqref{eq:bb_continuity} in Theorem~\ref{thm:bb}. \end{remark} The McCann interpolation can be viewed as a specific instance of a dynamical process. Based on this insight, we have developed an algorithm that leverages McCann interpolation to guide the optimization within the context of persistence diagrams. \begin{algorithm} \caption{Dynamical Persistent Homology via McCann Interpolation} \label{alg:maccann} \begin{algorithmic}[1] \State \textbf{Input:} $\mathcal K = \{\sigma_i\}_{i=1}^l$ the simplices, $f^{(1)}: \mathcal K \mapsto \mathbb R$ the initial filtration function, $K$ the total number of step for the dynamic process, $Z$ the target persistence diagram, $S$ the number of steps to perform filtration updates. \For{$k \gets 1$ to $K$} \State $t\gets \frac{1}{K-k+1}$ \State $X^{(k)} \gets \operatorname{dgm}_p(\mathcal K, f^{(k)})=\{x_i^{(k)}\}_{i=1}^n$ \Comment{Compute persistence diagram} \State $\pi \gets \operatorname*{OptimalTransportPlan}(X^{(k)}, Z)$ \Comment{Use Sinkhorn Algorithm~\citep{cuturi2013sinkhorn}} \State $X_1 \gets \operatorname*{TargetDgm}(\pi)$ \Comment{Use Equation~\eqref{eq:barycenter}} \State $Y^{(k)} \gets (1-t) X^{(k)} + t X_1$ \Comment{Use McCann Interpolation} \State $f \gets f^{(k)}$ \For{$s\gets 1$ to $S$} \State $\mathcal L \gets \operatorname*{Loss}(\operatorname*{dgm}_p(\mathcal K, f), Y^{(k)})$ \State $\nabla f \gets \operatorname{CriticalSetMethod}(\mathcal L, f)$ \Comment{\citep[Algorithm 3]{nigmetov2024topological}} \State $f \gets f - \eta \nabla f$ \EndFor \State $f^{(k+1)} \gets f$ \EndFor \Return $\{X^{(k)}\}_{k=1}^K, \{Y^{(k)}\}_{k=1}^K, \{f^{(k)}\}_{k=1}^K$ \end{algorithmic} \end{algorithm} In the Algorithm~\ref{alg:maccann}, the $\operatorname*{OptimalTransportPlan}$ can be efficiently computed using the Sinkhorn algorithm~\citep{cuturi2013sinkhorn}, known for its rapid convergence and computational efficiency. Alternatively, conventional linear programming methods can also be employed. The optimal transportation plan \(\pi\) is represented as an \(n \times m\) matrix, where \(n\) and \(m\) denote the cardinalities of the source persistence diagram \(X^{(k)}\) and the target persistence diagram \(Z\), respectively. Each entry \(\pi_{ij}\) in the matrix indicates the amount of mass to be transported from \(x_i \in X^{(k)}\) to \(z_j \in Z\). The mass associated with each point \(x_i \in X^{(k)}\) and \(z_j \in Z\) is \(1/n\) and \(1/m\), respectively. In cases where \(n\) and \(m\) are not equal, the \(\operatorname*{TargetDgm}\) function computes an appropriate target persistence diagram. This is achieved by using the barycenter to determine the target locations, calculated as follows: \begin{equation} x_i = \frac{\sum_{j=1}^{m} \pi_{ij} z_j}{\sum_{j=1}^{m} \pi_{ij}} \quad \text{for all } i. \label{eq:barycenter} \end{equation} The Algorithm~\ref{alg:maccann} demonstrates the use of McCann interpolation to guide optimization on persistence diagrams. The core idea is to obtain the next persistence diagram through McCann interpolation and employ a learnable scheme, specifically the critical set method \citep{nigmetov2024topological}, to determine the subsequent filtration. This approach allows for the construction of the entire dynamical process of the persistence diagram. The results $X^{(k)}, Y^{(k)}, f^{(k)}$ represent the persistence diagram, the next target persistence diagram, and the filtration at step $k$, respectively. We explicitly store $Y^{(k)}$ to maintain the next step's persistence diagram because the learnable scheme cannot guarantee that the persistence diagram is always achievable \citep{nigmetov2024topological}. This is also why we do not compute the full trajectories before learning the filtration. Instead, we compute the McCann interpolation at each step to ensure that the persistence diagram progresses towards the target persistence diagram. In other words, this algorithm optimizes the filtration towards the final target rather than intermediate states. The proposed algorithm is particularly well-suited for applications where a target persistence diagram is explicitly defined. Additionally, in the context of McCann interpolation, our algorithm guarantees that the measure $\mu_t$ evolves along the geodesic in Wasserstein space, provided that the target persistence diagram is achievable at each step. This property ensures that the topological features of the data, as captured by persistence diagrams, change towards the target diagram in the best possible manner. \subsection{Dynamical Persistent Homology via Energy Functional} The previous subsection demonstrated the dynamical persistence diagrams along McCann interpolation trajectories, given a target persistence diagram. In this subsection, we extend our methodology to scenarios without a predefined target diagram. As introduced in Subsection~\ref{sec:gfw}, a dynamical system describes the temporal evolution of a state according to a set of rules or equations. In the context of Wasserstein gradient flow, the probability measure \( \mu_t \) evolves following the gradient flow of an energy functional \( J \) in the Wasserstein space. This evolution follows a path that locally minimizes \( J \) most efficiently with respect to the Wasserstein distance. The choice of functional \(J\) determines the system's ultimate configuration, allowing for diverse outcomes depending on the specific energy landscape. To perform the dynamical persistence diagram through Wasserstein gradient flows, we propose the following Algorithm~\ref{alg:wgf}. \begin{algorithm} \caption{Dynamical Persistent Homology via Energy Functional} \label{alg:wgf} \begin{algorithmic}[1] \State \textbf{Input:} $\mathcal K = \{\sigma_i\}_{i=1}^l$ the simplices, $f^{(1)}: \mathcal K \mapsto \mathbb R$ the initial filtration function, $K$ the total number of step for the dynamic process, $S$ the number of steps to perform filtration updates, $\tau$ the step size in JKO, $J$ the energy functional. \For{$k \gets 1$ to $K$} \State $X^{(k)} \gets \operatorname{dgm}_p(\mathcal{K}, f^{(k)}) = \{x_i^{(k)}\}_{i=1}^n$ \Comment{Compute persistence diagram} \State $\left(y_1^{(k)}, y_2^{(k)}, \ldots, y_n^{(k)}\right) \leftarrow\left(x_1^{(k)}, x_2^{(k)}, \ldots, x_n^{(k)}\right)$ \State $\mu_k \gets \frac{1}{n} \sum_{i=1}^n \delta_{x_i^{(k)}}$ \State $\mu \gets \frac{1}{n} \sum_{i=1}^n \delta_{y_i^{(k)}}$ \Comment{Initialize a new measure} \State $\mu \gets \argmin_{\mu} \frac{1}{2 \tau} W_2^2(\mu_k, \mu)+J(\mu)$ \Comment{Perform JKO} \State $Y^{(k)}\gets \{y_i^{(k)}\}_{i=1}^n$ \Comment{The target at step $k$} \State $f \gets f^{(k)}$ \For{$s\gets 1$ to $S$} \State $\mathcal L \gets \operatorname*{Loss}(\operatorname*{dgm}_p(\mathcal K, f), Y^{(k)})$ \State $\nabla f \gets \operatorname{CriticalSetMethod}(\mathcal L, f)$ \Comment{\citep[Algorithm 3]{nigmetov2024topological}} \State $f \gets f - \eta \nabla f$ \EndFor \State $f^{(k+1)} \gets f$ \EndFor \Return $\{X^{(k)}\}_{k=1}^K, \{Y^{(k)}\}_{k=1}^K, \{f^{(k)}\}_{k=1}^K$ \end{algorithmic} \end{algorithm} Algorithm~\ref{alg:wgf}, akin to Algorithm~\ref{alg:maccann}, outputs $X^{(k)}$, $Y^{(k)}$, and $f^{(k)}$, which represent the current persistence diagram, the target persistence diagram, and the filtration at step $k$, respectively. The key distinction lies in Algorithm~\ref{alg:wgf}'s utilization of the JKO scheme to compute the target persistence diagram at each iteration. Importantly, we initialize the target persistence diagram with the current persistence diagram, ensuring equal cardinalities. This approach enables us to leverage the energy functional to guide the optimization process without prior knowledge of the final target persistence diagram. \section{Case Illustrations} The most beneficial aspect of the proposed methodology is its ability to capture the dynamic evolution of topological features in complex data. To demonstrate the effectiveness of our approach, inspired by the examples in \citet{carriere2021optimizing}, we present two case illustrations: circle denoising and circle emerging. These examples showcase the power of dynamical persistence diagrams in uncovering the underlying topological structures of noisy data and emerging patterns. In both cases, we use Rips filtration. \subsection{Circle Denoising} Our first illustration concerns the denoising of a circle. The input data consists of a set of points in a 2D plane, sampled from a circle with added Gaussian noise. This noisy data forms a perturbed circle, as depicted in Figure~\ref{fig:noised_circle}. The objective is to remove the noise and highlight the primary 1-dimensional topological feature, namely the circle itself. \begin{figure}[H] \centering \includegraphics[width=0.42\textwidth]{img/noised_circle.png} \caption[Noised circle]{The input data is a set of points in a 2D plane, sampled from a circle with added Gaussian noise.} \label{fig:noised_circle} \end{figure} \subsubsection{Repulsion Loss} In this experiment, we need to prevent the points from clustering together. To achieve this, we introduce an auxiliary loss function designed to enforce point separation. Specifically, given the point set \(\mathcal{K}_0 = \{ \sigma_i \}_{i=1}^n\), where each \(\sigma_i \in \mathbb{R}^2\), the repulsion loss is defined as follows: \[ \text{loss} = \sum_{i=1}^{n} \sum_{j=1, j \neq i}^{n} \frac{1}{\|\sigma_i - \sigma_j\|^2 + \epsilon} \] This formula calculates the repulsion loss for the point set \(\mathcal{K}_0\). The parameter \(\epsilon\) is a small positive constant that ensures the denominator does not become zero, thereby avoiding numerical instability. \subsubsection{Circle Denoising via McCann Interpolation on 0th Persistence Diagram} The target persistence diagram represents a 1-dimensional circle, which is the primary topological feature of interest. We apply Algorithm~\ref{alg:maccann} to denoise the 0th persistence diagram, using the target persistence diagram as a reference. More specifically, we set the target of the 0th persistence diagram to $(0, 0.05)$ in the experiment, aiming to give these points a reasonable distance from nearby points. For the 1st persistence diagram, we employ a built-in denoising method. The evolution of the denoising process is illustrated in Figure~\ref{fig:noised_circle_mccann_evolution}, where the denoised circle becomes clearly visible. \begin{figure}[H] \centering \includegraphics[width=.92\textwidth]{img/circle_denoise_mccann.png} \caption[Circle denoising via McCann Interpolation]{ The evolution of the noisy circle data towards the target circle persistence diagram using McCann interpolation on the 0th persistence diagram and a denoising algorithm \citep{nigmetov2024topological} on the 1st persistence diagram.} \label{fig:noised_circle_mccann_evolution} \end{figure} \begin{figure}[H] \centering \includegraphics[width=.92\textwidth]{img/circle_denoise_mccann_0th_dgm.png} \caption[0th PD evolution of circle denoising]{ The evolution of the 0th persistence diagram.} \label{fig:noised_circle_mccann_evolution_0th_dgm} \end{figure} \begin{figure}[H] \centering \includegraphics[width=.92\textwidth]{img/circle_denoise_mccann_1st_dgm.png} \caption[1th PD evolution of circle denoising]{ The evolution of the 1st persistence diagram.} \label{fig:noised_circle_mccann_evolution_1th_dgm} \end{figure} We present the evolution of the 0th and 1st persistence diagrams in Figure~\ref{fig:noised_circle_mccann_evolution_0th_dgm} and Figure~\ref{fig:noised_circle_mccann_evolution_1th_dgm}, respectively. The denoising process effectively eliminates noise from the data, thereby revealing the underlying circular structure. As shown in the figures, the persistence diagrams progressively converge to the target diagram, illustrating the successful denoising of the circle data. However, we observe a gap at the top of the circle in the results. This occurs because the McCann interpolation is applied solely to the 0th persistence diagram, neglecting the 1st persistence diagram. The built-in denoising process lacks control over the upper left 1st persistence pair, which represents the circle. To address this issue, we apply Algorithm~\ref{alg:wgf} to denoise the 1st persistence diagram and let the persistence pair move towards the left. \subsubsection{Circle Denoising via Energy Functional on 1st Persistence Diagram} To achieve the goal we mentioned above, we apply Algorithm~\ref{alg:wgf} with the energy functional \[J(\mu) = \frac{1}{2} \mathbb{E}_{(x, y) \sim \mu} \left[ \min\left(x^2 + (y - 1.2)^2, \frac{(x - y)^2}{2}\right) \right].\] It makes the points near to the diagonal move towards the diagonal, while points far from the diagonal move towards \((0,1.2)\). The results of this process are illustrated in Figure~\ref{fig:noised_circle_wgf_evolution}, Figure~\ref{fig:noised_circle_wgf_evolution_0th_dgm}, and Figure~\ref{fig:noised_circle_wgf_evolution_1st_dgm}. In JKO, we use the sliced Wasserstein implementation from \citet{bonet2022efficient} for efficiency purposes. As shown, the algorithm not only effectively removes noise from the data but also fills gaps at the top of the circle successfully. \begin{figure}[H] \centering \includegraphics[width=.92\textwidth]{img/circle_denoise_all.png} \caption[Circle denoising via McCann Interpolation and Energy Functional]{ The evolution of the noisy circle data towards the target circle persistence diagram using McCann interpolation on the 0th persistence diagram and energy functional on the 1st persistence diagram.} \label{fig:noised_circle_wgf_evolution} \end{figure} \begin{figure}[H] \centering \includegraphics[width=.92\textwidth]{img/circle_denoise_all_0th_dgm.png} \caption[0th PD evolution of circle denoising]{ The evolution of the 0th persistence diagram.} \label{fig:noised_circle_wgf_evolution_0th_dgm} \end{figure} \begin{figure}[H] \centering \includegraphics[width=.92\textwidth]{img/circle_denoise_all_1st_dgm.png} \caption[1st PD evolution of circle denoising]{ The evolution of the 1st persistence diagram.} \label{fig:noised_circle_wgf_evolution_1st_dgm} \end{figure} \subsection{Circle Emerging} Given a set of random 2D points on a plane, we observe that the first persistence diagram is not always empty. In this case, the objective is to enhance the circles represented by the first persistence diagram. In the experiment, we independently and identically sample 250 points from the uniform distribution on the square $[-1,1] \times [-1,1]$. Similar to the previous case, we apply the McCann algorithm with a target of $(0, 0.08)$ on the 0th persistence diagrams. Additionally, we apply Algorithm~\ref{alg:wgf} with the functional \[ J(\mu) = \frac{1}{4} \mathbb{E}_{(x, y) \sim \mu} \left[ (y - (x + 0.15))^2 + x^2 \right] \] to encourage the first persistence pairs above a certain threshold to evolve away from the diagonal line and drift to the left. The results of this process are illustrated in Figure~\ref{fig:emerging_circle_wgf_evolution}. We also present 0th and 1st persistence diagrams in Figure~\ref{fig:emerging_circle_wgf_evolution_0th_dgm} and Figure~\ref{fig:emerging_circle_wgf_evolution_1st_dgm}, respectively. The results demonstrate how the persistence diagrams evolve over time, highlighting the emergence of the circle from the random 2D points. \begin{figure}[H] \centering \includegraphics[width=.92\textwidth]{img/circle_emerging_all.png} \caption[Circle emerging via Wasserstein gradient flow]{ The evolution of the random 2D points towards the target circle persistence diagram using McCann interpolation on the 0th persistence diagram and energy functional on the 1st persistence diagram.} \label{fig:emerging_circle_wgf_evolution} \end{figure} \begin{figure}[H] \centering \includegraphics[width=.92\textwidth]{img/circle_emerging_all_0th_dgm.png} \caption[0th PD evolution of circle emerging]{ The evolution of the 0th persistence diagram.} \label{fig:emerging_circle_wgf_evolution_0th_dgm} \end{figure} \begin{figure}[H] \centering \includegraphics[width=.92\textwidth]{img/circle_emerging_all_1st_dgm.png} \caption[1st PD evolution of circle emerging]{ The evolution of the 1st persistence diagram.} \label{fig:emerging_circle_wgf_evolution_1st_dgm} \end{figure} \section{Limitations and Further Works} First, the current learnable persistence diagram scheme cannot be executed on a GPU, which hinders the scalability of the proposed algorithms when applied to large datasets. This limitation restricts the practical application of these methods in real-world scenarios where computational efficiency is crucial. Second, the proposed methods exclusively consider persistence diagrams. It is worth exploring whether other topological features, such as Zigzag persistence or multiparameter persistence, can be integrated into the proposed framework to enhance its versatility. Third, there is a need to establish statistical theories for the proposed methods when utilizing different energy functionals. For instance, when deriving dynamical persistence diagrams from a time series of real-world data, it is essential to determine the statistical properties of these diagrams. Additionally, identifying an appropriate energy functional that governs the generation of dynamical persistence diagrams from the time series is crucial. Furthermore, guidelines for selecting suitable energy functionals for various tasks should be developed to ensure optimal performance. Finally, a more effective approach is required to address the conflicts arising from singleton losses \citep{nigmetov2024topological} in the proposed methods, as these conflicts can significantly impact the accuracy and reliability of the results. \section{Conclusion} In this study, we propose two methods to integrate Wasserstein gradient flow into the learning process for persistence diagrams. First, by leveraging McCann interpolation, we develop an algorithm that optimizes persistence diagrams along the geodesic in the Wasserstein space. This approach ensures that the optimization path respects the intrinsic geometry of the space, leading to more accurate and meaningful results. Second, we introduce an energy functional-based approach that guides the optimization process without requiring a predefined target persistence diagram. This method allows for greater flexibility and adaptability in various applications. We demonstrate the effectiveness of our methods through case studies, including the denoising of a circle and the emergence of a circle from noisy data. These examples illustrate how our techniques can enhance the temporal evolution of topological features, highlighting the potential of dynamical persistence diagrams in topological data analysis. \newpage \section*{Acknowledgments} We are grateful for the insightful discussion with Dr. Ziyun Huang at the outset of this paper, which played a crucial role in shaping its development. \newpage \vskip 0.2in \bibliography{ref} \end{document}
2412.03863v2
http://arxiv.org/abs/2412.03863v2
Further analysis on the second frequency of union-closed set families
\documentclass[a4paper,12pt]{article} \usepackage[english]{babel} \newcommand*{\arXiv}[1]{\href{http://arxiv.org/abs/#1}{arXiv:#1}} \usepackage[margin=2cm]{geometry} \usepackage{type1cm} \usepackage{titlesec} \usepackage{fancyhdr} \usepackage[dvipsnames]{xcolor} \usepackage{graphicx} \usepackage{titling} \usepackage{tabularx} \usepackage[shortlabels]{enumitem} \usepackage{amsmath, amsthm, amssymb} \usepackage{mathtools} \usepackage{tikz-cd} \usepackage{pgfplots} \pgfplotsset{compat=1.18} \usepackage[breakable]{tcolorbox} \usepackage{standalone} \usetikzlibrary{decorations.pathmorphing} \usetikzlibrary{calc, arrows, matrix} \usetikzlibrary{external} \tikzexternalize[prefix=tikz/] \usepackage[unicode=true, pdfborder={0 0 0}, bookmarksdepth=-1]{hyperref} \usepackage[capitalise,nameinlink]{cleveref} \setlength{\headheight}{15pt} \setlength{\droptitle}{-1.5cm} \parindent=24pt \newtheoremstyle{mystyle} {6pt}{15pt} {\it} {} {\bf} {.} {1em} {} \theoremstyle{mystyle} \renewcommand{\proofname}{\bf Proof:\\} \newtheorem{theorem}{Theorem}[section] \newtheorem{example}[theorem]{Example} \newtheorem{exercise}[theorem]{Exercise} \newtheorem{corollary}[theorem]{Corollary} \newtheorem{property}[theorem]{Property} \newtheorem{proposition}[theorem]{Proposition} \newtheorem{lemma}[theorem]{Lemma} \newtheorem{problem}{Problem} \newtheorem{question}{Question} \newtheorem{fact}{fact}[subsection] \newtheorem*{recall}{Recall} \newtheorem*{remark}{Remark} \newtheorem*{claim}{Claim} \newtheorem*{observation}{Observation} \newtheorem{conjecture}{Conjecture} \theoremstyle{definition} \newtheorem{definition}[theorem]{Definition} \newcommand{\N}{\mathbb{N}} \newcommand{\Z}{\mathbb{Z}} \newcommand{\Q}{\mathbb{Q}} \newcommand{\R}{\mathbb{R}} \newcommand{\C}{\mathbb{C}} \newcommand{\E}{\mathbb{E}} \renewcommand{\Pr}{\mathbb{P}} \newcommand{\6}{\partial} \DeclarePairedDelimiter{\norm}{\lVert}{\rVert} \DeclarePairedDelimiter{\gen}{\langle}{\rangle} \DeclarePairedDelimiter{\floor}{\lfloor}{\rfloor} \DeclarePairedDelimiter{\ceil}{\lceil}{\rceil} \DeclarePairedDelimiter{\innerp}{\langle}{\rangle} \pagestyle{fancy} \lhead{} \chead{} \lfoot{} \cfoot{} \rfoot{\thepage} \renewcommand{\headrulewidth}{0.4pt} \renewcommand{\footrulewidth}{0.4pt} \DeclareMathOperator{\Freq}{Freq} \linespread{1.5} \begin{document} \newcommand{\F}{\mathcal{F}} \newcommand{\fq}{\Freq_{\F}} \title{Further analysis on the second frequency of union-closed set families} \author{Saintan Wu\thanks{ National Taiwan University, [email protected]}} \date{\today} \maketitle \begin{abstract} The Union-Closed Sets Conjecture, also known as Frankl's conjecture, asks whether, for any union-closed set family $\mathcal{F}$ with $m$ sets, there is an element that lies in at least $\frac{1}{2}\cdot m$ sets in $\mathcal{F}$. In 2022, Nagel posed a stronger conjecture that within any union-closed family whose ground set size is at least $k$, there are always $k$ elements in the ground set that appear in at least $\frac{1}{2^{k-1}+1}$ proportion of the sets in the family. Das and Wu showed that this conjecture is true for $k\geq 3$ and $k=2$ if $|\mathcal{F}|$ is outside a particular range. In this companion paper, we analyse further when $\mathcal{F}$ fails Nagel's conjecture for $k=2$ via linear programming. \end{abstract} \section{Introduction} A \textit{union-closed} set family $\mathcal{F}$ is a set family satisfying $A,B\in \mathcal{F}\implies A\cup B\in \mathcal{F}$. Nagel \cite{nagel2023notes} conjectured that: \begin{conjecture}\label{conj:Nagel} For any union-closed set family $|\mathcal{F}|$ with ground set $\geq k$, there are $k$ elements, each of them lies in at least $\frac{1}{2^{k-1}+1}|\mathcal{F}|$ many sets in $\mathcal{F}$. \end{conjecture} Let $F_k(\mathcal{F})$ denote the ratio of sets in $\mathcal{F}$ that contain the $k$-th most frequent element. So the above conjecture states that $F_k(\mathcal{F})\geq \frac{1}{2^{k-1}+1}$ for such families. Notice that when $k=1$, it conjectures the well-known Union-Closed Sets Conjecture, that every nonempty union-closed family has an element appearing in at least $\frac{|\mathcal{F}|}{2}$ sets. \cite{our_main_paper} gives a notion about $k$-good set, which here we will only use $2$-good sets: \begin{definition} Given a union-closed family $\mathcal{F}$, a set $S$ is \textbf{$2$-good} for $\mathcal{F}$ if it does not contain $1$, and for each set $A$ that is not $\varnothing$ nor $\{1\}$, $S\cap A \neq\varnothing$. A $2$-good set is called \textbf{minimal} if its proper subsets are not $2$-good. \end{definition} Then, in the paper, \cite{our_main_paper} proved, via the entropic method for large families and shattering method for small families, that \begin{theorem} \label{thm:main} Let $k \geq 2$, and let $\mathcal{F}$ be a union-closed set family with $|\mathcal{F}| = m$ and $|\cup_{F \in \mathcal{F}} F| \ge k$. Then the $k$th-most frequent element lies in at least $\frac{m}{2^{k-1} + 1}$ sets in $\mathcal{F}$, with equality only if $\mathcal{F}$ is a near-$k$-cube, provided: \begin{itemize} \item[(i)] $k \ge 3$, or \item[(ii)] $k = 2$, and either $m \le 44$ or $m \ge 114$. \end{itemize} \end{theorem} We still did not know if there is a counterexample for $k=2$ when $|\mathcal{F}|\in [45,113]$. We will analyse this case more carefully and narrow the range. Our main result in this paper is the following: \begin{theorem}\label{thm:sidemain} Let $\F$ be union-closed with $|\cup_{F \in \F} F| \ge 2$, and that $\F$ is not a near-$2$-cube. Then, if $f_2(\F) \le \frac13$, we must have \begin{enumerate}[(i)] \item $81 \le |\F| \le 113$, and \item All minimal $2$-good sets $S$ have size $4$. \end{enumerate} \end{theorem} In the following sections, we will focus on the case when $\F$ has $f_2(\F)\leq \frac13$ and is in the case that \cref{thm:main} didn't cover. That is, when $|\F|\in [45,113]$ and any of its minimal $2$-good sets $S$ has size $4$ or $5$. We will further assume that $\varnothing\in \mathcal{F}$, and $1$ is the most abundant element in sets of $\mathcal{F}$. \subsection*{Paper structure and notations} In Section 2, we will briefly explain the idea behind the proof. Then, we will prove several lemmas mentioned in the heuristics. After that, two separate cases $|S|=4$ and $|S|=5$ will be done in \cref{sec:s=4} and \cref{sec:s=5}, respectively. For simplicity, for any set $S$ and element $x$, we use $S+x$ for $S\cup\{x\}$, and $S-x$ for $S\setminus\{x\}$. We will also write, for example, $abd$ for short of $\{a,b,d\}$. \section{Heuristics behind the proof} \subsection{Review of the shattering strategy} Let us first have a brief review of the proof for the case $4\leq |\mathcal{F}|\leq 113$ and $k=2$. We considered a minimal $2$-good set $S$ and showed that by the minimality, for any element $y\in S$, there is $F_y\in \mathcal{F}$ that $F_y\cap S=\{y\}$ because if there was no such $F_y$, $S-y$ became a smaller $2$-good set. By the union-closed property, for any $T\subseteq S$, we find that $F_T:= \bigcup_{y\in T} F_y$ is in $\mathcal{F}$, and $F_T\cap S=T$. This shows that $|S|\leq \log_2|\mathcal{F}|$ because these $2^{|S|}$ sets $F_T,T\subseteq S$, are surely different. Lastly, using $F_T$'s to analyse the \textit{incidence} $\sum_{A\in \mathcal{F}} |A\cap S|$ of $\mathcal{F}$ over $S$ when $f_2(\mathcal{F})\leq \frac13$, we get that either $s=4$ and $m \geq 45$, or $s=5$ and $m\geq 70.5$. \subsection{Linear programming and two ways of winning} We can make a more careful estimate: For each subset $T$ of $S$, we define $q_T$ as the quantity of sets $A$ in $\mathcal{F}$ that $S\cap A=T$. \begin{itemize} \item By $S$ being $2$-good, there are at most $2$ sets that do not intersect $S$, namely $\varnothing$ and $\{1\}$. Thus \[q_\varnothing\leq 2.\] \item By $F_T\cap S=T$, we obtain \[q_T\geq 1 \qquad\text{for each }T\subseteq S.\] \item As $f_2(\mathcal{F})\leq \frac 13$, we obtain \[\sum_{y\in T}q_T\leq \frac m3 \qquad \text{for each }y\in S.\] \end{itemize} It turns out that these are all linear constraints for $q_T$'s and $m$, so we can solve it by setting the following linear programming $L_0$: \begin{description} \item[Variables] $m,q_T:T\subseteq S$ \item[Minimize] $m$ \item[Subject to] \begin{equation*} \begin{array}{r@{\,}ll} q_\varnothing &\leq 2 ,&\\ q_T & \geq 1 ,&T\subseteq S\\ \sum_{T\subseteq S} q_T &= m,&\\ \sum_{y\in T}q_T & \leq \frac m3 ,& y\in S.\\ \end{array} \end{equation*} \end{description} By solving this directly, we get same bounds $m\geq 45$ for $|S|=4$ and $m\geq 70.5$ for $|S|=5$. So what happens when this LP reaches the lower bound? A solution reaching the lower bound of $m$ when $|S|=4$ is {\linespread{1} \[q_{T}=\begin{cases} 2,&T=\varnothing;\\ 8,&|T|=1;\\ 1,&|T|\geq 2. \end{cases}\] } This configuration can cause many dependencies. For example, while there are many pairs of choices of $F_a, F_b$ in the proof, they all result in the unique $F_{ab}:= F_a\cup F_b$! This phenomenon could only happen when each element in $F_{ab}$ is either a common element of $F_a$s or a common element of $F_b$s. In this scenario, elements \textit{outside} $S$ appear in many sets. Hence, we can strengthen $L_0$ by considering another carefully chosen element $x$. This $x$ allows two ways of winning: Either \begin{itemize} \item this $x$ helps us find different sets (by spotting $x\notin F_T, x\in F_T'$) that $F_{T}\cap S=T=F_T'\cap S$ for big $T$. In this way, it gives a greater lower bound for $q_T$. Or, \item $x$ itself lies in many sets. In this way, it gives an extra constraint for $L_0$, as $x$ cannot lie in over $\frac{m}{3}$ sets if $x\neq 1$. \end{itemize} \subsection{The choice of $x$ and three lemmas} We will pick $x\neq 1$ and an $a\in S$ such that there are two sets $F_a,F_a'$ with $F_a\cap (S+x)=a, F_a'\cap (S+x)=ax$. For any choice of $F_y, y\in S$, such that $F_y\cap S=\{y\}$, we associate \[ F_{T}:= \bigcup_{y\in T} F_y, T\subseteq S.\] Now we see how the flexibility of choice of $F_a$ helps us: If we can choose $F_y$ for $y\in S$ such that $x\notin F_T$, then $F_T\cup F_a$ and $F_T\cup F_a'$ are two different sets in $\mathcal{F}$, both of which intersect $S$ at $T+a$, thus $q_{T+a}\geq 2$. We can do much better than this, but we give some definitions before we can precisely state the lemma: \begin{definition}[flexible, covered]\mbox{} For $y\in S$, we say $a$ is $x$-\textit{flexible} if there are two sets $F_y,F_y'$ with $F_y\cap (S+x)=y, F_y'\cap (S+x)=xy$. We say $y$ is \textit{covered} (by $x$) if all choices of $F_y$ contains $x$. Equivalently, $y$ is covered by $x$ iff $S+x-y$ is $2$-good. \end{definition} Let $C$ denote the set of covered elements. \begin{lemma}\label{lemma:smallway} Let $a\in S$ be $x$-flexible. Then $q_T\geq 2$ for each $T\subseteq S$ such that \begin{itemize} \item either $a\in T$ and $T\cap C=\varnothing$, or \item $T$ contains at least $2$ elements in $C$. \end{itemize} \end{lemma} On the other hand, if there is a choice of $\{F_y:y\in S\}$ such that $x\in F_T$, then $x\in F_{T'}$ for any $T\subseteq T'\subseteq S$. we can find many sets in $\mathcal{F}$ that contains $x$ if there are many covered elements. It leads to the second lemma: \begin{lemma}\label{lemma:largeway} Let $a\in S$ be $x$-flexible. and $x$ covers the elements in $C$. Then \[(2^{s}-2^{s-1-|C|}) + \sum_{c\in C} q_c - |C| \leq \frac{m}{3}.\] \end{lemma} These lemmas handle cases where $|C|$ is small and large well. For $|C|=1$, it turns out that we can benefit from assuming $S$ has the best incidence among all minimal $2$-good $s$-sets of $\mathcal{F}$. \begin{lemma}\label{lemma:middleway} Let $a\in S$ be $x$-flexible. Suppose that $S$ is a minimal $2$-good set which maximizes the incidence $\sum_{A\in \mathcal{F}} |A\cap S|$ among all minimal $2$-good $s$-sets of $\mathcal{F}$. Suppose that $x$ only covers $b\in S$. Then \begin{itemize} \item There are $2^{|S|-2}$ sets in $\mathcal{F}$ that intersects $S$ at $\geq 2$ elements, contains $b$ but not $x$. \item There are $2^{|S|-2}-1$ sets in $\mathcal{F}$ that intersects $S$ at $\geq 3$ elements, contains $b$ but not $x$. \item There are $2^{|S|-3}-1$ sets in $\mathcal{F}$ that intersects $S$ at $\geq 4$ elements, contains $b$ but not $x$. \end{itemize} \end{lemma} In the next section, we will justify the choice of $a,x$ and prove these three lemmas. Remind that these lemmas are all under the assumption $f_2(\mathcal{F})\leq \frac13$. To utilise \cref{lemma:middleway}, we will assume that $S$ maximises the incidence among the minimal $2$-good sets with the corresponding size. We will separate the analysis regarding the size of $C$. We obtain a bound for $m$ by first obtaining extra constraints for $q_T$s using \cref{lemma:smallway,lemma:largeway,lemma:middleway}, then solve the linear program $L_0$ with these extra constraints. Detailed LP calculations are available in the companion Git repository\footnote{\url{https://github.com/haur576/k-Union-Closed-Sets-Conjecture}}. A summary of the bounds for $m$ is shown in \cref{fig:summary}. One can find that this is sufficient to reach a contradiction unless $s=4$ and $|C|=0,1$. \begin{figure}[htbp!] \centering \[\begin{array}{c|cccc} \text{Bound for $m$ if $f_2(\mathcal{F})\leq \frac 13$} & |C|=0 &|C|=1 & |C|=2 & |C|\geq 3\\\hline s=4 & \textcolor{red}{m\geq 81} & \textcolor{red}{m\geq 81} & m\geq 114 & \text{infeasible}\\ s=5 & m\geq 118.5 & m\geq 115.5 & m\geq 122 & m\geq 114\\ \text{Using Lemma...} &\text{\labelcref{lemma:smallway}}&\text{\labelcref{lemma:smallway,lemma:middleway}}&\text{\labelcref{lemma:smallway,lemma:largeway}}&\text{\labelcref{lemma:largeway}} \end{array}\] \caption{Summary of bound found in cases} \label{fig:summary} \end{figure} \section{Proofs of the lemmas} First of all, we show that there is always a choice of $a$ and $x$. \begin{lemma} There is $a\in S$ and $x\notin S\cup\{1\}$ such that $a$ is $x$-flexible. \end{lemma} \begin{proof} By the constraints of $L_0$, we find that $\sum_{a\in S}q_a\geq 40$ for $|S|=5$ and $q_y\geq 8$ for each $y\in S, |S|=4$. Therefore, there is $a\in S$ such that $q_a\geq 8$. So $\{F\in \mathcal{F}| F\cap S=\{a\}\}$ has size $\geq 8$. We pick any two sets in this family that differ not only at $1$. Then there is $x \neq 1$ such that $x$ lies in one set but not the other. These two are desired $F_a'$ and $F_a$, respectively. \end{proof} Now, we prove three lemmas introduced in the previous section. We prove \cref{lemma:largeway} first: \begin{proof}[proof of \cref{lemma:largeway}] This lemma is by simple counting: For each $y\in S$, we choose $F_y$ such that $x\in F_y$ if $y=a$ or $y\in C$, and $\notin F_y$ otherwise. By the choice and the definition of covering, $x$ lies in \begin{itemize} \item $F_T$, where $T$ contains $a$ or an element in $C$, and \item any $F\in \mathcal{F}$ such that $F\cap S=\{c\}$ for some $c\in C$. \end{itemize} There are $2^{|S|}-2^{|S|-1-|C|}$ sets from the first case, $\sum_{c\in C}q_c$ sets from the second case and $|C|$ sets from both. As $x$ cannot lie in more than $\frac{m}{3}$ sets, the result follows from the Inclusion-Exclusion Principle. \end{proof} \begin{proof}[proof of \cref{lemma:smallway}] The first case is direct: By the definition of non-covered elements, we can pick $F_y$ for $y\notin C$ such that $x\notin F_y$. If $a\in T$ and $T\cap C=\varnothing$, consider $F_T=\bigcup_{y\in T} F_y$ and $F_T'=F_a'\cup F_T$. These two are different as $x\notin F_T, x\in F_T'$. For the second case, there is always an $F$ that $F\cap S=T$ with $x\in F$, and we will find another which $x\notin F$. We first show that for any $b,c\in C$, $S+x-b-c$ is \textit{not} $2$-good. Suppose conversely that $S+x-b-c$ is $2$-good. If $|S|=4$, then it means that there is a $2$-good set with size $3$, which implies $f_2(\mathcal{F})>\frac 13$. Hence we consider $|S|=5$. Notice that now any set $F\in \mathcal{F}$ that $F\cap S=\{b,c\}$ also contains $x$. By the same counting method of proving \cref{lemma:largeway}, $x$ lies in $q_b+q_c+q_{bc}+28-3$ sets. Consider the LP $L_0$ along with $q_b+q_c+q_{bc}+28-3\leq \frac{m}{3}$, we get $m\geq 129$, contradicting to the assumption. For any $2$ elements $b,c\in C$, since $S+x-b-c$ is not $2$-good, there is a set $G_{bc}\neq \varnothing,\{1\}$ that does not intersect $S+x-b-c$. However, since $S+x-b$ and $S+x-c$ are $2$-good, both $G_{bc}\cap (S+x-b)$ and $G_{bc}\cap (S+x-c)$ are nonempty. This implies $G_{bc}\cap (S+x)=bc$. Now for $T$ that contains $2$ or more elements in $C$, we consider $F_T'$ to be the union of \begin{itemize} \item $G_{bc}$ for each pair $b,c$ of elements in $T\cap C$, and \item $F_y$ for $y\in T\setminus C$ with $x\notin F_y$. \end{itemize} Then $F'_T\cap S=T$ and $x\notin F_T$, as desired. \end{proof} Lastly, we prove \cref{lemma:middleway}. \begin{proof}[proof of \cref{lemma:middleway}] We first show that there are $2^{|S|-2}$ sets in $\mathcal{F}$ containing $b$ but not $x$. As $x$ only covers $b$, we may choose $F_y$ such that $x\in F_a, x\in F_b$ and $x\notin F_y$ for $y\neq a,b$. Then there are $2^{|S|-2}$ resulting $F_T$ in $\mathcal{F}$ that contains $x$ but not $b$, namely those with $a\in T, b\notin T$. Note that $S+x-b$ is a $2$-good set and is minimal. Indeed, for each $y\neq b$ in $S$, $S+x-y$ is not $2$-good, thus $S+x-b-y$ is not $2$-good. (Also, $S+x-b-x=S-b$ is not $2$-good.) As $S$ maximises the incidence among minimal $2$-good $s$-sets, $S+x-b$ has no larger incidence than $S$. Thus, the frequency of $b$ is at least the one of $x$. That is, there are at least $2^{|S|-2}$ sets which contains $b$ but not $x$. In the following we will show that in these $2^{|S|-2}$ sets which contains $b$ but not $x$, \begin{itemize} \item $2^{|S|-2}$ of them intersect $S$ at $\geq 2$ elements, \item $2^{|S|-2}-1$ of them intersect $S$ at $\geq 3$, and \item $2^{|S|-3}-1$ of them intersect $S$ at $\geq 4$. \end{itemize} Let us call the other $|S|-2$ elements $c,d$ and possibly $e$ if $s=5$. for the choice of $\{F_y\}$, we choose $F_y$ such that $x\notin F_y$ for each $y\neq b$. Among these $2^{|S|-2}$ sets, we pick one $F$ with minimised intersection size of intersection with $S$. \begin{itemize} \item If $|F\cap S|=1$, then $F\cap S=\{b\}$ but $F$ does not contain $x$. It contradicts that $b$ is covered. \item If $|F\cap S|=2$, suppose that $F\cap S = ab$ (the case for $bc,bd,be$ are similar). We consider the union \[G_T:= F\cup \bigcup_{y\in T} F_y \in \mathcal{F}\] for any $T$ that does not contain $a$ nor $b$. These $2^{|S|-2}$ sets contain $b$ but not $x$. Also, \begin{itemize} \item $2^{|S|-2}$ of them intersect $S$ at $\geq 2$ elements, \item $2^{|S|-2}-1$ of them intersect $S$ at $\geq 3$ elements, and \item $2^{|S|-2}-1-(|S|-2) \geq 2^{|S|-3}-1$ of them intersect $S$ at $\geq 4$ elements. \end{itemize} Thus, they meet the conditions. \footnote{For the inequality, we used the fact that $2^{|S|-2}-1-(|S|-2) \geq 2^{|S|-3}-1$ when $|S|\geq 4$.} \item $|F\cap S|=3$: Then all these $2^{|S|-2}$ sets intersect $S$ at $\geq 3$ elements, satisfying the first two requirements. Assume that $F\cap S= abc$ (the cases for $abd, bcd, abe, bce, bde$ are similar). We consider the union \[F_T':=F\cup \bigcup_{y\in T} F_y \in \mathcal{F}\] for any $T$ does not contain $a,b,c$. These $2^{|S|-3}$ sets contain $b$ but not $x$, and $2^{|S|-3}-1$ of them intersect $S$ at $\geq 4$ elements. Thus, they meet the conditions. \item $|F\cap S|\geq 4$: Then all $2^{|S|-2}$ sets meet $S$ at $\geq 4$ elements, satisfying these three conditions. \end{itemize} \end{proof} \section{$s=4$}\label{sec:s=4} Now we assume that $S=\{a,b,c,d\}$ and $x,ax\in \mathcal{F}_{S+x}$. \subsection{When $|C|=0$} From \cref{lemma:smallway} we have \[q_T\geq 2 \text{ for any } a\in T\subseteq S.\] Solving the LP $L_0$ along with this additional constraint gives $m\geq 81$. \subsection{When $|C|=1$} Assume that $b$ is covered. From \cref{lemma:smallway} we have \[q_T\geq 2 \text{ for } T =a, ac,ad,acd.\] From \cref{lemma:middleway} we have the following inequalities: \begin{corollary}\mbox{} \begin{equation} \begin{aligned} \sum_{b\in T\subseteq S, |T|\geq 2} q_T &\geq 7+4,\\ \sum_{b\in T\subseteq S, |T|\geq 3} q_T &\geq 4+3, \text{ and }\\ \sum_{b\in T\subseteq S, |T|\geq 4} q_T &\geq 1+1. \end{aligned} \end{equation} \begin{proof} We count $F\in \mathcal{F}$ for those that contain both $b$ and $x$ and those that contain only $b$ but no $x$. By choosing $\{F_y\}$ such that $x\in F_b$ and $x\notin F_y$ for other $y\neq b$, we find that there are $7,4,1$ $F_T$'s with $|F_T\cap S|\geq 2,3,4$, respectively, each of which contains both $b$ and $x$. From \cref{lemma:middleway}, there are $4,3,1$ $F$'s with $|F\cap S|\geq 2,3,4$, respectively, each of which contains $b$ but not $x$. \end{proof} \end{corollary} Solving $L_0$ with these $4+3$ extra constraints gives $m\geq 81$. \subsection{When $|C|=2$} Assume that $b,c$ are covered. From \cref{lemma:smallway} we get \[q_T \geq 2 \text{ for } T=a, ad, bc, bcd, abc, abcd.\] From \cref{lemma:largeway} we get $12 + q_b+q_c \leq \frac{m}{3}$. Solving $L_0$ with these two additional constraints gives $m\geq 114$. \subsection{When $|C|\geq 3$} Assume that $b,c,d$ are covered. From \cref{lemma:largeway} we get $12 + q_b+q_c+q_d \leq \frac m3$. The linear program $L_0$ with this constraint is infeasible. \section{$s=5$}\label{sec:s=5} Now we assume that $S=\{a,b,c,d,e\}$ and $x,ax\in \mathcal{F}_{S+x}$. \subsection{When $|C|=0$} From \cref{lemma:smallway} we get \[q_T\geq 2 \text{ for any } a\in T\subseteq S.\] Solving the LP $L_0$ along with this additional constraint gives $m\geq 118.5$. \subsection{When $|C|=1$} We assume $b\in C$. From \cref{lemma:smallway} we get \[q_T\geq 2 \text{ for any } T\subseteq S \text{ such that }a\in T,b\notin T.\] By \cref{lemma:middleway} we have the following inequalities: \begin{corollary} \begin{equation} \begin{aligned} \sum_{b\in T\subseteq S, |T|\geq 2} q_T &\geq 15+8,\\ \sum_{b\in T\subseteq S, |T|\geq 3} q_T &\geq 11+7, \text{ and }\\ \sum_{b\in T\subseteq S, |T|\geq 4} q_T &\geq 5+3. \end{aligned} \end{equation} \begin{proof} We count $F\in \mathcal{F}$ for those that contain both $b$ and $x$ and those that contain only $b$ but no $x$. By choosing $\{F_y\}$ such that $x\in F_b$ and $x\notin F_y$ for other $y\neq b$, we find that there are $15,11,5$ $F_T$'s with $|F_T\cap S|\geq 2,3,4$, respectively, each of which contains both $b$ and $x$. From \cref{lemma:middleway}, there are $8,7,3$ $F$'s with $|F\cap S|\geq 2,3,4$, respectively, that contains $b$ but not $x$. \end{proof} \end{corollary} Consider $L_0$ with these $8+3$ extra constraints, and we get $m \geq 115.5$. \subsection{When $|C|=2$} We assume $b,c\in C$. From \cref{lemma:smallway} we get \[q_T\geq 2 \text{ for } T=a, ad, ae, ade, bc, bcd, bce, bcde, abc, abcd, abce, abcde.\] From \cref{lemma:largeway} we get $26 + q_b+q_c \leq \frac{m}{3}$. Solving $L_0$ with these two types of additional constraints gives $m\geq 122$. \subsection{When $|C|\geq 3$} Assume $b,c,d\in C$ are covered. From \cref{lemma:largeway} we get $30 + q_b+q_c+q_d-3 \leq \frac{m}{3}$ or $30 + q_b+q_c+q_d+q_e-4 \leq \frac{m}{3}$. Either we get $30 + q_b+q_c+q_d-3 \leq \frac{m}{3}$. Solving $L_0$ with these additional constraints gives us $m\geq 114$. In conclusion, the case $s=5$ is completely solved. \section{Concluding remark} In this paper, we further investigated the potential counterexample for \cref{conj:Nagel} and narrowed down the searching range of their sizes from $[45,113]$ to $[81,113]$ and showed that there is some uniformity to the collection of minimal $2$-good sets. To further improve the bound, a possible approach is to pick $x$ more carefully. In our proof, we can see that there are many choices for extra elements. With such many choices, one may possibly push the bound further. Another perspective is to investigate the uniformity of minimal $2$-good sets of the family. To do this, one could answer this question: \begin{question} What can we say about the minimal $2$-good sets if they all have the same size? \end{question} To first ignore the asymmetry induced by the most common element $1$, we consider a definition slightly different from $k$-good sets. \begin{definition} Let $\mathcal{F}$ be a family, a \textbf{cover} $S$ of $\mathcal{F}$ is a set that intersects every set in $\mathcal{F}$. A \textbf{minimal cover} is a cover whose proper subsets are not covers. Let $\mathcal{MC}(\mathcal{F})$ denote the family of minimal covers of $\mathcal{F}$. \end{definition} Equivalently, a cover is a vertex cover when we see $\mathcal{F}$ as the edge set of a hypergraph on $n$ vertices. In this case, we can characterize when $\mathcal{MC}(F)$ is uniform: \begin{theorem} Let $\mathcal{F}$ be a family, not necessarily be union-closed. \begin{enumerate} \item $\mathcal{MC}(\mathcal{F})$ is an antichain. In other words, any two elements in $\mathcal{MC}(\mathcal{F})$ are incomparable. \item If $\mathcal{F}$ is an antichain, then $\mathcal{MC}(\mathcal{MC}(\F))=\F$. \item Subsequently, let $\mathcal{G}$ be the subfamily of minimal elements in $\mathcal{F}$, then $\mathcal{MC}(\F)$ is $k$-uniform if and only if $\mathcal{G}$ is the minimal covers of a $k$-uniform hypergraph. \end{enumerate} \end{theorem} \begin{proof} We only have to prove the second claim. \begin{description} \item[$\F\subseteq \mathcal{MC}(\mathcal{MC}(\F))$:] For $F\in \F$, it is a cover of $\mathcal{MC}(\F)$ since every $C\in \mathcal{MC}(\F)$ intersects $F$. To show that $F\setminus\{f\}$ is not a cover of $\mathcal{MC}(\F)$ for each element $f\in F$, we need to find a minimal cover $C\in \mathcal{MC}(\F)$ contained in $F^{\complement}\cup\{f\}$. Since $\F$ is an antichain, $F^{\complement}\cup\{f\}$ is indeed a cover and therefore such $C$ exists. \item[$\mathcal{MC}(\mathcal{MC}(\F))\subseteq \F$:] For $C$ a minimal cover of $\mathcal{MC}(F)$, we show that it covers a set in $\F$. If this is true, $C$ must in $\mathcal{F}$ as sets in $\mathcal{F}$ are covers of $\mathcal{MC}(\F)$. Were it not, then for each $F\in \F$, there is an element $a_F\in F$ that $C$ does not have. Notice that $\{a_F:F\}$ is a cover of $\F$, it contains a minimal cover of $\F$. It lies in $\mathcal{MC}(\F)$ but does not intersect $C$, giving a contradiction. \end{description} \end{proof} We hope to understand the potential counterexamples for \cref{conj:Nagel} when $k=2$ better by understanding the families whose minimal $2$-good sets are uniform. \bibliographystyle{amsplain} \providecommand{\bysame}{\leavevmode\hbox to3em{\hrulefill}\thinspace} MR } \providecommand{\MRhref}[2]{ \href{http://www.ams.org/mathscinet-getitem?mr=#1}{#2} } \providecommand{\href}[2]{#2} \begin{thebibliography}{1} \bibitem{our_main_paper} Shagnik Das and Saintan Wu, \emph{Frequent elements in union-closed set families}, 2024, \arXiv{2412.03862}. \bibitem{nagel2023notes} Nicolas Nagel, \emph{Notes on the union closed sets conjecture}, 2023, \arXiv{2208.03803}. \end{thebibliography} \end{document}
2412.04005v1
http://arxiv.org/abs/2412.04005v1
SLE$_κ(ρ)$ processes in the light cone regime on Liouville quantum gravity
\documentclass[11pt]{amsart} \usepackage[leqno]{amsmath} \usepackage{amssymb} \usepackage{enumerate} \usepackage{subfigure} \usepackage{graphicx} \usepackage[all]{xy} \topmargin=0in \oddsidemargin=0in \evensidemargin=0in \textwidth=6.5in \textheight=8.5in \usepackage[usenames,dvipsnames]{xcolor} \usepackage[ colorlinks=true,linkcolor=NavyBlue,urlcolor=RoyalBlue,citecolor=PineGreen,bookmarks=true,bookmarksopen=true,bookmarksopenlevel=2,unicode=true,linktocpage]{hyperref} \usepackage{microtype} \usepackage{centernot} \usepackage{stmaryrd} \usepackage{comment} \usepackage{bm} \usepackage{mathrsfs} \numberwithin{equation}{section} \newtheorem{theorem}{Theorem}[section] \newtheorem{remark}[theorem]{Remark} \newtheorem{lemma}[theorem]{Lemma} \newtheorem{proposition}[theorem]{Proposition} \newtheorem{corollary}[theorem]{Corollary} \newtheorem{problem}[theorem]{Problem} \newtheorem{definition}[theorem]{Definition} \newtheorem{example}[theorem]{Example} \newtheorem{assumption}[theorem]{Assumption} \newcommand{\B}{\mathbf{B}} \newcommand{\C}{\mathbf{C}} \newcommand{\D}{\mathbf{D}} \newcommand{\E}{\mathbf{E}} \newcommand{\F}{\mathbf{F}} \newcommand{\h}{\mathbf{H}} \newcommand{\N}{\mathbf{N}} \newcommand{\Z}{\mathbf{Z}} \newcommand{\p}{\mathbf{P}} \newcommand{\Q}{\mathbf{Q}} \newcommand{\R}{\mathbf{R}} \newcommand{\s}{\mathbf{S}} \newcommand{\Fa}{\mathfrak {a}} \newcommand{\Fd}{\mathfrak {d}} \newcommand{\Fh}{\mathfrak {h}} \newcommand{\Fp}{\mathfrak {p}} \newcommand{\CA}{\mathcal {A}} \newcommand{\CB}{\mathcal {B}} \newcommand{\CC}{\mathcal {C}} \newcommand{\CD}{\mathcal {D}} \newcommand{\CE}{\mathcal {E}} \newcommand{\CF}{\mathcal {F}} \newcommand{\CI}{\mathcal {I}} \newcommand{\CJ}{\mathcal {J}} \newcommand{\CL}{\mathcal {L}} \newcommand{\CM}{\mathcal {M}} \newcommand{\CN}{\mathcal {N}} \newcommand{\CO}{\mathcal {O}} \newcommand{\CP}{\mathcal {P}} \newcommand{\CQ}{\mathcal {Q}} \newcommand{\CR}{\mathcal {R}} \newcommand{\CS}{\mathcal {S}} \newcommand{\CT}{\mathcal {T}} \newcommand{\CU}{\mathcal {U}} \newcommand{\CV}{\mathcal {V}} \newcommand{\CW}{\mathcal {W}} \newcommand{\CX}{\mathcal {X}} \newcommand{\CZ}{\mathcal {Z}} \newcommand{\CG}{\mathcal {G}} \newcommand{\CH}{\mathcal {H}} \newcommand{\GL}{\text{GL}} \newcommand{\GLN}{\mathcal {GL}_n(\R)} \newcommand{\SLN}{\mathcal {SL}_n(\R)} \newcommand{\SLE}{{\rm SLE}} \newcommand{\CLE}{{\rm CLE}} \newcommand{\QLE}{{\rm QLE}} \newcommand{\sgn}{\mathrm{sgn}} \newcommand{\dist}{\mathrm{dist}} \newcommand{\aut}{\mathrm{Aut}} \newcommand{\diam}{\mathrm{diam}} \newcommand{\var}{\mathrm{var}} \newcommand{\im}{\mathrm{Im}} \newcommand{\re}{\mathrm{Re}} \newcommand{\cov}{\mathrm{cov}} \newcommand{\Hom}{\mathrm{Hom}} \newcommand{\End}{\mathrm{End}} \newcommand{\Mat}{\mathrm{Mat}} \newcommand{\CRk}{\mathrm{rank }} \newcommand{\Ran}{\mathrm{Ran}} \newcommand{\loc}{\mathrm{loc}} \newcommand{\supp}{\mathrm{supp }} \newcommand{\diff}{\mathrm{diff}} \newcommand{\diver}{\mathrm{div}} \newcommand{\iso}{\mathrm{iso}} \newcommand{\Ssc}{S_\text{sc}} \newcommand{\Psc}{\Psi_\text{sc}} \newcommand{\confrad}{{\rm CR}} \newcommand{\one}{{\bf 1}} \newcommand{\wt}{\widetilde} \newcommand{\wh}{\widehat} \newcommand{\ol}{\overline} \newcommand{\ul}{\underline} \newcommand{\red}[1]{{\color{red} #1}} \newcommand{\blue}[1]{{\color{blue} #1}} \newcommand{\td}[1]{\textbf{\red{[#1]}}} \newcommand{\note}[1]{\textbf{\blue{[#1]}}} \newcommand{\jm}[1]{\red{#1}} \newcommand{\kk}[1]{\blue{#1}} \newcommand{\NW}{\mathsf{NW}} \newcommand{\NE}{\mathsf{NE}} \newcommand{\SE}{\mathsf{SE}} \newcommand{\SW}{\mathsf{SW}} \newcommand{\RPM}{\mathsf{RPM}} \newcommand{\RPMs}{\mathsf{RPMs}} \newcommand{\BPRPM}{\mathsf{BPRPM}} \newcommand{\BPRPMs}{\mathsf{BPRPMs}} \newcommand{\giv}{\,|\,} \newcommand{\fan}{{\mathbf F}} \newcommand{\Loop}{\CL} \newcommand{\Ito}{Ito} \newcommand{\density}{\rho} \newcommand{\eps}{\epsilon} \renewcommand{\graph}{\CG} \newcommand{\BCLE}{\mathrm{BCLE}} \newcommand{\ccwBCLE}{\BCLE^{\boldsymbol {\circlearrowleft}}} \newcommand{\cwBCLE}{\BCLE^{\boldsymbol {\circlearrowright}}} \newcommand{\BES}{\mathrm{BES}} \newcommand{\BESQ}{\mathrm{BESQ}} \newcommand{\pv}{\mathrm{P.V.}} \newcommand{\cwBoundary}[3]{[#1,#2]_{#3}^{\boldsymbol {\circlearrowright}}} \newcommand{\ccwBoundary}[3]{[#1,#2]_{#3}^{\boldsymbol {\circlearrowleft}}} \newcommand{\ccwBoundaryOpen}[3]{(#1,#2)_{#3}^{\boldsymbol {\circlearrowleft}}} \newcommand{\strip}{{\mathscr{S}}} \newcommand{\cyl}{{\mathscr{C}}} \newcommand{\IG}{\mathrm{IG}} \newcommand{\quant}[2]{{{\mathfrak q}}_{#2}(#1)} \newcommand{\median}[1]{{\mathfrak m}_{#1}} \newcommand{\cp}{\mathrm{cap}} \newcommand{\qcone}[1]{\mu_{\mathrm{QC}}^{W=#1}} \newcommand{\qwedge}[1]{\mu_{\mathrm{QW}}^{W=#1}} \newcommand{\qmeasure}[1]{\mu_{#1}} \newcommand{\qbmeasure}[1]{\nu_{#1}} \newcommand{\qcarpet}[2]{\mu_{#1,#2}} \newcommand{\move}{{\mathfrak m}} \newcommand{\id}{\mathrm{id}} \newcommand{\qnt}{{\mathfrak q}} \newcommand{\lightcone}{{\mathbf L}} \newcommand{\trace}{\text{trace}} \title[$\SLE_\kappa(\rho)$ processes in the light cone regime on Liouville quantum gravity]{$\SLE_{\kappa}(\rho)$ processes in the light cone regime on\\ Liouville quantum gravity} \author{Konstantinos Kavvadias and Jason Miller} \begin{document} \begin{abstract} We study the relationship between certain $\SLE_\kappa(\rho)$ processes, which are variants of the Schramm-Loewner evolution with parameter $\kappa$ in which one keeps track of an extra marked point, and Liouville quantum gravity (LQG). These processes are defined whenever $\rho > -2-\kappa/2$ and in this work we will focus on the light cone regime, meaning that $\kappa \in (0,4)$ and $\max(\kappa/2-4,-2-\kappa/2) < \rho < -2$. Such processes are self-intersecting even though ordinary $\SLE_\kappa$ curves are simple for $\kappa \in (0,4)$. We show that such a process drawn on top of an independent $\sqrt{\kappa}$-LQG surface called a weight $(\rho+4)$-quantum wedge can be represented as a gluing of a pair of trees which are described by the two coordinate functions of a correlated $\alpha$-stable L\'evy process with $\alpha = 1-2(\rho+2)/\kappa$. Combined with another work, this shows that bipolar oriented random planar maps with large faces can be identified in the scaling limit with an $\SLE_\kappa(\kappa-4)$ curve on an independent $\sqrt{\kappa}$-LQG surface for $\kappa \in (4/3,2)$. \end{abstract} \date{\today} \maketitle \setcounter{tocdepth}{1} \tableofcontents \parindent 0 pt \setlength{\parskip}{0.20cm plus1mm minus1mm} \section{Introduction} \label{sec:introduction} \subsection{Overview and setting} The purpose of this work is to study the connection between Liouville quantum gravity (LQG) surfaces and the $\SLE_\kappa(\rho)$ processes, which are variants of the Schramm-Loewner evolution ($\SLE_\kappa$) \cite{schramm2000scaling} in which one keeps track of an extra marked point \cite[Section~8.3]{lsw2003restriction}. These processes are defined whenever $\rho > -2-\kappa/2$ and we will focus on the so-called light cone regime \cite{ms2019lightcone} which means that $\kappa \in (0,4)$ and $\max(\kappa/2-4,-2-\kappa/2) < \rho < -2$. These processes exhibit rather different behavior from ordinary $\SLE_\kappa$. For example, they are self-intersecting even though $\kappa \in (0,4)$. We will begin by giving a brief overview of the objects which will be relevant for this work. Recall that an LQG surface is a random two-dimensional Riemannian manifold which is formally described by the metric tensor \begin{equation} \label{eqn:lqg_form} e^{\gamma h(z)} (dx^2 + dy^2) \end{equation} where $z=x+iy$, $dx^2 + dy^2$ is the Euclidean metric on a domain $D \subseteq \C$, $h$ is (some form of) the Gaussian free field (GFF) on $D$, and $\gamma \in (0,2]$ is a parameter. Since the GFF $h$ and its variants are random variables which live in the space of distributions rather than in a space of functions, some care is required in order to make sense of~\eqref{eqn:lqg_form}. The volume form $\qmeasure{h}$ associated with~\eqref{eqn:lqg_form} was constructed in \cite{ds2011lqgkpz} as the limit as $\epsilon \to 0$ of \begin{equation} \label{eqn:volume_limit} \epsilon^{\gamma^2/2} e^{\gamma h_\epsilon(z)} dx dy \end{equation} where $dx dy$ denotes Lebesgue measure and $h_\epsilon(z)$ denotes the average of $h$ on the circle $\partial B(z,\epsilon)$. The boundary length measure $\qbmeasure{h}$ on a linear segment $L$ was also constructed in \cite{ds2011lqgkpz} as the limit as $\epsilon \to 0$ of \begin{equation} \label{eqn:boundary_limit} \epsilon^{\gamma^2/4} e^{\gamma h_\epsilon(x)/2} dx \end{equation} where $dx$ denotes Lebesgue measure on $L$ and $h_\epsilon(x)$ denotes the average of $h$ on the semi-circle $D \cap \partial B(x,\epsilon)$. The measures $\qmeasure{h}$, $\qbmeasure{h}$ can also be understood as Gaussian multiplicative chaos measures in the framework developed by Kahane \cite{k1985gmc}. The metric (two-point distance function) associated with~\eqref{eqn:lqg_form} was later constructed in the case $\gamma=\sqrt{8/3}$ in \cite{ms2020qle1,ms2021qle2,ms2021qle3} and in \cite{dddf2020tightness,gm2021metric} in the case that $\gamma \in (0,2)$ where the latter works are based on a regularization procedure analogous to~\eqref{eqn:volume_limit} while the former takes a more indirect approach. LQG surfaces are important in statistical mechanics, string theory, and conformal field theory. The regularization procedure~\eqref{eqn:volume_limit}, \eqref{eqn:boundary_limit} implies that the following is true. Suppose that $h$ is an instance of (some form of) the GFF on $D$, $\varphi \colon \wt{D} \to D$ is a conformal map, and \begin{align} \label{eqn:change_of_coordinates} \wt{h} = h \circ \varphi + Q \log |\varphi'| \quad\text{where}\quad Q = \frac{2}{\gamma} + \frac{\gamma}{2}. \end{align} Then $\qmeasure{h}(\varphi(A)) = \qmeasure{\wt{h}}(A)$ for all $A \subseteq \wt{D}$ Borel. We say that two pairs $(D,h)$, $(\wt{D},\wt{h})$ consisting of a planar domain and a field are equivalent as quantum surfaces if they are related as in~\eqref{eqn:change_of_coordinates} and a \emph{quantum surface} is an equivalence class under the equivalence relation defined by~\eqref{eqn:change_of_coordinates}. This definition extends to quantum surfaces which have extra marked points where one requires the conformal map $\varphi$ to take the marked points associated with $\wt{h}$ to those associated with $h$. There has been a substantial amount of work in recent years focused on developing connections between LQG and $\SLE$ and its variants. This started with \cite{she2016zipper} in which it was shown that it is possible to conformally weld together two independent copies $\CW_1,\CW_2$ of a certain type of quantum surface called a (weight $2$) quantum wedge and obtain a quantum wedge $\CW$ (but with weight $4$) where the welding interface $\eta$ is given by an $\SLE_\kappa$ curve which is independent of $\CW$. Roughly, a quantum wedge is an infinite volume surface with two marked points (the ``origin'' and the ``infinity'' point) and two boundary rays which have locally finite quantum length. Here, it is important that the parameters $\kappa$ and $\gamma$ are matched by $\kappa = \gamma^2 \in (0,4)$. A number of other welding results of this type were proved in \cite{dms2014mating} with different types of quantum wedges and depending on the setting one obtains as the welding interface an $\SLE_\kappa(\rho_1;\rho_2)$ process with $\rho_1, \rho_2 > -2$. Recall that the $\SLE_\kappa(\rho)$ processes are variants of $\SLE_\kappa$ in which one keeps track of extra marked points \cite[Section~8]{lsw2003restriction} called force points and the $\rho$'s are the weights, which determine how the force points affect the behavior of the curve. By $\SLE_\kappa(\rho_1;\rho_2)$ we typically mean the case where there is a force point of weight $\rho_1$ (resp.\ $\rho_2$) located at $0_-$ (resp.\ $0_+$). These processes can be boundary intersecting even though ordinary $\SLE_\kappa$ for $\kappa \in (0,4)$ does not intersect the boundary. The threshold $-2$ is important because when $\rho > -2$ an $\SLE_\kappa(\rho)$ process is absolutely continuous with respect to an ordinary $\SLE_\kappa$ process when it is away from the boundary while this is not the case for $\rho \leq -2$. It is also shown in \cite{dms2014mating} that if one welds the two boundary rays of a quantum wedge to each other then one obtains a quantum cone, which is a surface homeomorphic to $\C$, decorated by an independent whole-plane $\SLE_\kappa(\rho)$ process. This ultimately leads to the proof that one can explore and encode an LQG surface by an appropriate space-filling version of $\SLE_{\kappa'}$ for $\kappa' > 4$ (throughout we will use the imaginary geometry \cite{ms2016imag1} convention that $\kappa \in (0,4)$, $\kappa'=16/\kappa > 4$) which can be thought of as the Peano curve which traces the interface between a pair of continuum random trees (CRTs) $\CT_1$, $\CT_2$ drawn in the plane and whose branches are themselves whole-plane $\SLE_\kappa(\rho)$ processes (with $\rho=2-\kappa$) \cite{ms2017imag4}. This yields the so-called ``mating of trees'' representation of LQG, whose importance is that it provides one avenue for making connections between LQG, $\SLE$, and random planar maps ($\RPM$s). The reason for this is that a number of $\RPM$ models can be encoded using so-called tree bijections and the mating of trees representation of LQG can be thought of as a continuous analog of such a bijection. In particular, there have been a number of scaling limit results for $\RPM$s towards $\SLE$ on LQG in which it is shown that the contour functions which encode a pair of discrete trees used to construct a planar map using a tree bijection converge in the scaling limit to the pair of contour functions which encode the continuous trees associated with space-filling $\SLE_{\kappa'}$ on LQG. Such scaling limit results are referred to as being of ``peanosphere'' type and we will discuss this point in more detail later on. As we mentioned earlier, the $\SLE_\kappa(\rho)$ processes with $\rho < -2$ exhibit a rather different character from the $\SLE_\kappa(\rho)$ processes with $\rho > -2$ (we exclude the case $\rho = -2$ because it is in a certain sense degenerate). In particular, they are self-intersecting for $\kappa \in (0,4)$ even though the ordinary $\SLE_\kappa$ processes are not. There are two regimes of $\rho$ values in which these processes are defined: \begin{enumerate}[(i)] \item The loop-making regime: $-2-\kappa/2 < \rho \leq \kappa/2-4$. \item The light cone regime: $\max(\kappa/2-4,-2-\kappa/2) < \rho < -2$. \end{enumerate} The basic properties of the $\SLE_\kappa(\rho)$ processes in the loop-making regime were studied in \cite{msw2017clepercolations} and their relationship with LQG in \cite{msw2020simplecle}. The particular case $\rho=\kappa-6$ in this regime is of special significance because it corresponds to the conformal loop ensembles ($\CLE$) \cite{s2009cle,sw2012cle}. This case is thus motivated because it conjecturally corresponds to the scaling limit of $\RPM$ models decorated by a statistical mechanics model such as the Ising model. In the light cone regime, the basic properties were studied in \cite{ms2019lightcone} and the purpose of the present paper is to study the relationship between these processes and LQG. One application of this regime is that it makes it possible to connect bipolar oriented random planar maps with large faces to LQG \cite{km2022bplargefaces} using a ``peanosphere'' type scaling limit as was briefly described above. \subsection{Main results} \label{subsec:main_results} We now turn to state our main results, which are stated in terms of quantum wedges and $\SLE_\kappa(\rho)$ processes in the light cone regime. We will give the precise definition of the former in Section~\ref{subsec:lqg} and the latter in Section~\ref{subsec:slekapparho}. The theorem statements that we will give concern an $\SLE_\kappa(\rho)$ process $\eta$ with $\max(\kappa/2-4,-2-\kappa/2) < \rho < -2$ drawn on top of an independent quantum wedge $\CW = (\h,h,0,\infty)$ and that the quantum surfaces which are parameterized by the components of $\h \setminus \eta$ together form a quantum wedge. We emphasize that $\CW$ will be homeomorphic to $\h$ while the quantum wedge corresponding to $\h \setminus \eta$ is not homeomorphic to $\h$. As we will describe in further detail in Section~\ref{subsec:lqg}, the starting point for the construction of this latter surface is a Bessel process $Y$ with dimension in $(0,1)$ and therefore has a well-defined local time $\ell$ for the times that it hits $0$. \begin{theorem} \label{thm:quantum_natural_time_cutting} Fix $\kappa \in (0,4) , \rho \in (\kappa/2 - 4, -2) \cap (-2 -\kappa/2 , -2)$, and suppose that $\CW = (\h,h,0,\infty)$ is a quantum wedge of weight $\rho + 4$. Let $\eta$ be an $\SLE_{\kappa}(\rho)$ process in $\h$ from $0$ to $\infty$ with a single force point of weight $\rho$ located at $0_+$ and assume that $\eta$ is independent of $h$. Then the following hold: \begin{enumerate}[(i)] \item The law of the beaded surface consisting of the components of $\h \setminus \eta$ which are to the right of $\eta$ is that of a quantum wedge of weight $\rho + 2$. \item Let $\qnt_{u}$ be the first capacity time $t$ for $\eta$ that the local time at $0$ of the Bessel process $Y$ which encodes the weight $\rho + 2$ wedge is equal to $u$. For each $u > 0$, we have (as path-decorated quantum surfaces) that $(h,\eta)$ and $(h \circ f_{\qnt_{u}}^{-1} + Q\log|(f_{\qnt_{u}}^{-1})'|,f_{\qnt_{u}}(\eta))$ have the same law, where $(f_t)$ is the forward centered Loewner flow corresponding to $\eta$. \end{enumerate} \end{theorem} \begin{definition} \label{def:quantum_natural_time} We call $\qnt_u$ the quantum natural time parameterization of the bubbles which are cut off by $\eta$ from $\infty$. \end{definition} In \cite{dms2014mating}, a similar definition of quantum natural time for $\SLE_{\kappa'}$ processes with $\kappa' \in (4,8)$ is given. In contrast to the case of $\SLE_{\kappa'}$ processes, an $\SLE_\kappa(\rho)$ process as in Theorem~\ref{thm:quantum_natural_time_cutting} parameterized by quantum natural time is not continuous, the reason being that as the process draws the boundary of a large component it does not discover any small components. \begin{figure}[ht!] \begin{center} \includegraphics[scale=0.85]{figures/boundary_length_evolution.pdf} \end{center} \caption{\label{fig:boundary_length_evolution} Shown is an $\SLE_\kappa(\rho)$ process in $\h$ from $0$ to $\infty$ with a single force point located at $0_+$ in the light cone regime and drawn up to a typical quantum natural time $\qnt_u$. The arc which disconnects each component from $\infty$ is drawn in its entirety before $\eta$ ``creeps'' up it disconnects further disks. Its left boundary length $L_u$ is equal to the quantum length of the dark blue arc (counterclockwise from $\eta(\qnt_u)$ to $A_u$) minus the quantum length of the light blue arc ($[A_u,0]$). Similarly, its right boundary length $R_u$ is equal to the quantum length of the dark green arc (clockwise from $\eta(\qnt_u)$ to $B_u$) minus the quantum length of the light green arc ($[0,B_u]$). We prove in Theorem~\ref{thm:boundary_length_evolution} that $(L,R)$ evolves as an $\alpha$-stable L\'evy process where $\alpha$ is determined by $\rho$ as in~\eqref{eqn:alpha_value}.} \end{figure} Theorem~\ref{thm:quantum_natural_time_cutting} allows us to describe the law of the boundary length evolution of $\eta$ when it has the quantum natural time parameterization. More precisely, for each $u > 0$ we let $A_u$ (resp.\ $B_u$) be the leftmost (resp.\ rightmost) point of intersection of $\eta([0,\qnt_u])$ with $\R_-$ (resp.\ $\R_+$). Let also $L_u$ be the difference between the quantum length of the segment of the outer boundary of $\eta([0,\qnt_u])$ connecting $\eta(\qnt_u)$ and $A_u$ with the quantum length of $[A_u,0]$. Similarly, let $R_u$ be the difference between the quantum length of the segment of the outer boundary of $\eta([0,\qnt_u])$ connecting $\eta(\qnt_u)$ and $B_u$ with the quantum length of $[0,B_u]$. Then the following theorem describes the evolution of $(L,R)$. Recall that $(L,R)$ is an $\alpha$-stable L\'evy process if it has stationary and independent increments and for each $c > 0$ we have that $(L_u,R_u)_{u \geq 0}$ and $(c^{-1/\alpha}L_{c u},c^{-1/\alpha}R_{c u})_{u \geq 0}$ have the same law. \begin{theorem} \label{thm:boundary_length_evolution} Suppose that we have the setup described in the statement of Theorem~\ref{thm:quantum_natural_time_cutting} and the process $(L,R)$ is as described just above. Then $(L,R)$ is an $\alpha$-stable L\'evy process with \begin{align}\label{eqn:alpha_value} \alpha = 1 - \frac{2(\rho+2)}{\kappa} \in (1,2). \end{align} Moreover, $L$ (resp.\ $R$) has only upward (resp.\ downward) jumps and the jump times of $L$ coincide with the jump times of $R$. In the case that $\kappa \in (\frac{4}{3},2)$ and $\rho = \kappa - 4$ the sequence of jumps of $(L,-R)$ forms a Poisson point process (p.p.p.) $\Lambda^*$ whose law can be sampled from as follows. \begin{itemize} \item Firstly, we sample a p.p.p.\ $\Lambda = \{(s_j,t_j) : j \in \N \}$ on $\R_+ \times \R_+$ with intensity measure given by $c(du \times t^{-4/\kappa}dt)$, where $c > 0$ is a constant depending only on $\kappa$. \item Next, we sample independently a sequence of i.i.d.\ random variables $(u_j)_{j \in \N}$ which are uniform on $[0,1]$ and consider \begin{align*} \Lambda^* = \{(t_j u_j, (1-u_j)t_j) : (s_j,t_j) \in \Lambda, j \in \N\}. \end{align*} \end{itemize} \end{theorem} Note that by \cite{bertoin1996levy}, the law of a two-dimensional $\alpha$-stable L\'evy process is determined by the law of its jumps. Hence when $\rho = \kappa - 4$, Theorem~\ref{thm:boundary_length_evolution} gives a complete description of the law of the process $(L,R)$. Moreover, combining Theorem~\ref{thm:boundary_length_evolution} with the one-dimensional $\text{KPZ}$ formula for quantum boundary length, we obtain the following formula for the Hausdorff dimension of $\eta \cap \R_+$. \begin{corollary} \label{cor:dim_of_boundary_intersection} Fix $\kappa \in (0,4), \rho \in (\kappa/2-4,-2) \cap (-2 - \kappa/2,-2)$ and let $\eta$ be an $\SLE_{\kappa}(\rho)$ process in $\h$ from $0$ to $\infty$ with its force point located at $0_+$. Then, a.s.\ the Hausdorff dimension of the set $\eta \cap \R_+$ is given by \begin{equation} \label{eqn:dim_of_intersection_with_R_+} -\frac{(2+\rho)(\kappa + 8 + 2\rho)}{2\kappa}. \end{equation} \end{corollary} We note that the analog of Corollary~\ref{cor:dim_of_boundary_intersection} for $\rho > -2$ was given in \cite{mw2017slepaths} and for $-2-\kappa/2 < \rho \leq \kappa/2-4$ follows from the loop-trunk decomposition of such processes given in \cite{msw2017clepercolations} combined with the $\rho > -2$ case given in \cite{mw2017slepaths}. We note that for $\rho = \kappa/2-4$ the value from~\eqref{eqn:dim_of_intersection_with_R_+} is equal to $2-8/\kappa'$ where $\kappa'=16/\kappa$. This, in turn, is equal to the dimension of the intersection of an $\SLE_{\kappa'}$ process in $\h$ with $\partial \h$ determined in \cite{as2011covariant}. This is not a coincidence as it was shown in \cite{ms2019lightcone} that the law of the range of an $\SLE_\kappa(\kappa/2-4)$ process agrees with that of an $\SLE_{\kappa'}(\kappa'/2-4)$ process (but the corresponding curves visit the points in their range in a different order). \subsection{Mating of trees interpretation} \label{subsec:interpretation} We will now describe how Theorems~\ref{thm:quantum_natural_time_cutting} and~\ref{thm:boundary_length_evolution} have the interpretation of giving a mating of trees representation of $\SLE_\kappa(\rho)$ processes in the light cone regime drawn on top of LQG, how it contrasts with the mating of trees representations given in \cite{dms2014mating}, and how this is related to the scaling limits of certain types of $\RPM$s. \begin{figure}[ht!] \begin{center} \includegraphics[scale=0.85]{figures/mot_representation.pdf} \end{center} \caption{\label{fig:mot_representation} Illustration of the ``mating of trees representation'', which describes the topology of an $\SLE_\kappa(\rho)$ process $\eta$ drawn on top of an independent quantum wedge of weight $\rho+4$. {\bf Left:} Points on the graph of $\wh{R}$ are equivalent if they can be connected by a horizontal line which is below the graph of $\wh{R}$ and points on the graph of $3-\wh{L}$ are equivalent if they can be connected by a horizontal line which is above the graph of $3-\wh{L}$. Points on the graphs of $\wh{R}$ and $3-\wh{L}$ are equivalent if they can be connected by a vertical line which does not correspond to a jump time of $(L,R)$. The jump times $(L,R)$ are the yellow regions and in the quotient correspond to doubly marked disks which are cut out by $\eta$. The opening (resp.\ closing) point of such a disk $D$ is given by the projection of the left (resp.\ right) vertical boundary of the corresponding yellow region $Y$ and the left (resp.\ right) boundary of $D$ is given by the projection of the part of $\partial Y$ which is on the graph of $3-\wh{L}$ (resp.\ $\wh{R}$). {\bf Right:} The time $\tau$ is a jump time for $L$, $R$, which corresponds to a time at which $\eta$ disconnects a topological disk from $\infty$ (shown in yellow); $K_{\tau-}$ is the hull cut out by $\eta$ up to just before time $\tau$. The opening point of the disk is $\eta(\tau^-)$ (blue) and the closing point is $\eta(\tau)$ (red). At the time $\tau$, $R$ (resp.\ $L$) makes a downward (resp.\ upward) jump of size equal to the quantum length counterclockwise (resp.\ clockwise) arc of the disk boundary from the opening point to the closing point.} \end{figure} Suppose that $(L,R)$ is an $\alpha$-stable L\'evy process with law as in Theorem~\ref{thm:boundary_length_evolution} so that $L$ (resp.\ $R$) has only upward (resp.\ downward) jumps and that the jump times of $L$ and $R$ coincide. Let $\wt{L}$ (resp.\ $\wt{R}$) be the continuous process obtained by starting with $L$ (resp.\ $R$) and then replacing each jump with a linear segment whose length is equal to square of the jump made by $L-R$ at this time. Since the sum of the squares of the jumps of an $\alpha$-stable L\'evy process with $\alpha \in (1,2)$ on any compact time interval is finite, it follows that $(\wt{L},\wt{R})$ is well-defined. Let $\phi$ be a homeomorphism from $\R$ to $(-1,1)$ which takes $-\infty$ (resp.\ $\infty$) to $-1$ (resp.\ $1$) and fixes $0$. Then the processes $\wh{L}_u = \phi(\wt{L}_{\phi^{-1}(u)})$, $\wh{R}_u = \phi(\wt{R}_{\phi^{-1}(u)})$ are defined on $[0,1)$ and take values in $(-1,1)$. Let $\CI$ be the image under $\phi$ of those intervals where $\wt{L}$, $\wt{R}$ are linear. Suppose that we draw the graph of $\wh{R}$ and the graph of $3-\wh{L}$ in the rectangle $\CR = [0,1] \times [-1,4]$. Let $\CA$ consist of those vertical lines $[(u, \wh{R}_u), (u, 3-\wh{L}_u)]$ where $u \in \CI$. We consider the finest equivalence relation $\sim$ on $\CR$ such that points in $\CR \setminus \CA$ which can be connected by: \begin{itemize} \item a horizontal line which lies below the graph of $\wh{R}$, or \item a horizontal line which lies above the graph of $3-\wh{L}$, or \item a vertical line which connects the graphs of $\wh{R}$ and $3-\wh{L}$ whose $x$-coordinate is not in $\CI$. \end{itemize} Arguing as in \cite{dms2014mating}, it is possible to check using Moore's theorem that the topological space $\CR / \sim$ is homeomorphic to $\h$. Moreover, one can consider the continuous curve $\eta$ which is given by the projection of the curve $u \mapsto 3 - \wh{L}_u$ under the quotient map. Then $\eta$ is homeomorphic to an $\SLE_\kappa(\rho)$ process where $\rho$ is determined by $\alpha$ as in~\eqref{eqn:alpha_value}. If $I$ is a connected component of $\CI$ (i.e., $I = (a,b)$ for some $-1 < a < b < 1$), then the topological disk given by the union of the vertical segments $[(u, \wh{R}_u), (u, 3-\wh{L}_u)]$ for $u \in I$ corresponds to a component which is disconnected from $\infty$ by $\eta$. Recall that a planar map is a graph $G = (V,E)$ together with an embedding into the plane so that no two edges cross, considered up to orientation preserving homeomorphism. A $\RPM$ is a planar map chosen according to some probability measure. Examples of $\RPM$s include planar triangulations and quadrangulations chosen uniformly at random \cite{lg2013bm,m2013bm,lg2019survey,miermontstflour} and weighted by the partition function of various models from statistical mechanics \cite{s2016qginv,kmsw2019bipolar,lsw2017schnyder,gm2021saw,gm2017percolation,gkmw2018active}. One approach to study $\RPM$s is to encode them in terms of random trees and random walks via combinatorial bijections. An important class of such bijections are the so-called \emph{mating of trees bijections} which represent a planar map decorated by a statistical mechanics model as the gluing of a pair of discrete trees. Since discrete trees can be represented in terms of their contour function, such an encoding is equivalent to encoding the map by a two-dimensional walk. The construction of LQG surfaces using matings of trees \cite{dms2014mating} therefore leads to a natural topology on surfaces which is called the \emph{peanosphere topology}. In particular, a surface decorated by a tree can be encoded in terms of a pair of continuous functions $(L,R)$ where $L$ (resp.\ $R$) is given by the contour function of the tree (resp.\ dual tree) on the surface. Then if we have two tree-decorated surfaces with associated pairs of contour functions $(L,R)$ and $(L',R')$, we define the distance between the two surfaces to be the $L^\infty$ distance between $(L,R)$ and $(L',R')$. In other words, the peanosphere topology is the restriction of the $L^\infty$ metric to the space of continuous functions which arise as the contour functions related to tree-decorated surfaces. Since applying a rescaling to a planar map corresponds to applying a rescaling to the discrete pair of trees encoding the map, we say that a sequence of rescaled $\RPM$s converges in distribution to a tree-decorated random quantum surface as the number of vertices (or faces) of the map tends to infinity, if the rescaled random walks encoding the $\RPM$s converge in distribution with respect to the~$L^\infty$ metric to the pair of continuous functions $(L,R)$ encoding the tree-decorated quantum surface. It was shown in \cite{kmsw2019bipolar} that if we pick a bipolar oriented planar map uniformly at random with fixed boundary lengths and whose faces are all triangles then the corresponding rescaled random walk converges in the limit to a correlated two-dimensional Brownian motion. Using the construction related to the peanosphere topology, this result can be interpreted as being a scaling limit result towards the $\SLE$ with parameter $\kappa = 12$ on a $\sqrt{4/3}$-LQG surface. The case of random bipolar oriented planar maps ($\BPRPM$s) with \emph{large faces}, meaning that the law of the face degree is in the domain of attraction of an $\alpha$-stable random variable, are considered in \cite{km2022bplargefaces}. It is shown in \cite[Theorem 1.3]{km2022bplargefaces} that the contour functions for the pair of discrete trees which encode such a $\BPRPM$ converge in the scaling limit to an $\alpha$-stable L\'evy process with the same law as in Theorem~\ref{thm:boundary_length_evolution} in the case that $\rho = \kappa-4$ and $\alpha=4/\kappa-1$. This allows us to interpret $\SLE_\kappa(\kappa-4)$ drawn on top of a weight $\rho+4$ quantum wedge as the scaling limit of $\BPRPM$s with large faces in the peanosphere sense. \subsection*{Outline} The remainder of this article is structured as follows. In Section~\ref{sec:preliminaries}, we will collect a number of preliminaries. Next, in Section~\ref{sec:slegff_couplings} we will collect some results on the $\SLE$/GFF coupling. Finally, we will complete the proofs of our main theorems in Section~\ref{sec:main_theorems_proof}. \subsection*{Acknowledgements} K.K.'s work was supported by the EPSRC grant EP/L016516/1 for the University of Cambridge CDT (CCA) and by ERC starting grant 804166 (SPRS). J.M.'s work was supported by ERC starting grant 804166. \section{Preliminaries} \label{sec:preliminaries} The purpose of this section is collect a number of preliminaries which will be used in the remainder of this article. In Section~\ref{subsec:bessel}, we will give a brief review of Bessel processes. Next, in Section~\ref{subsec:gff} we will give a review of the GFF and in Section~\ref{subsec:lqg} of LQG. In Section~\ref{subsec:slekapparho} we will review the $\SLE_\kappa(\rho)$ processes and in Section~\ref{subsec:ig} the aspects of imaginary geometry which are relevant for this work. Finally, in Section~\ref{subsec:light_cones} we will review the $\SLE_\kappa(\rho)$ processes in the light cone regime in the imaginary geometry framework. \subsection{Bessel processes} \label{subsec:bessel} In this subsection, we recall a few facts about Bessel processes which will play an important role in this paper. We refer the reader to \cite[Chapter XI]{revuz2013continuous} for a more in depth overview of Bessel processes. Fix $\delta \in \R$ and $x \geq 0$. The squared $\delta$-dimensional Bessel process ($\BESQ^\delta$) starting from $x^2$ is given by the unique strong solution to the SDE \begin{equation}\label{eq:square_bessel_sde} dZ_t = \delta dt + 2\sqrt{Z_t} dB_t,\quad Z_0 = x^2 \end{equation} where $B$ is a standard Brownian motion. If we want to emphasize the starting point of a $\BESQ^\delta$ process we will write $\BESQ_{x^2}^\delta$. Standard results for SDEs imply that there is a unique strong solution to~\eqref{eq:square_bessel_sde} up until the first time that $Z$ hits $0$. When $\delta > 0$, there exists a unique strong solution defined for all times which always remains non-negative. Then for $\delta > 0$, the \emph{Bessel process} of dimension $\delta$ ($\BES^{\delta}$), is the process $X_t = \sqrt{Z_t}$, where $Z$ is the unique strong solution to~\eqref{eq:square_bessel_sde}. If we want to emphasize the starting point of a $\BES^\delta$ process we will write $\BES_x^\delta$. If $\delta \geq 2$, then a.s.\ $X_t > 0$ for all $t > 0$ while if $\delta \in (0,2)$, then $X_t$ a.s.\ assumes the value zero on a non-empty random set with zero Lebesgue measure. Also, for all $\delta > 0$, the process $X_t$ is invariant under Brownian scaling, i.e., for every given constant $c > 0$, the processes $(c^{-1/2}X_{ct})$ and $(X_t)$ have the same law. Moreover, when $\delta > 1$, the process $X_t$ is a semimartingale and a strong solution to the SDE \begin{equation} \label{eq:bessel_sde} d X_t = \frac{\delta-1}{2} \cdot \frac{1}{X_t} dt + d B_t, \quad X_0 = x \end{equation} and when $\delta =1$, the process $X_t$ is equal in distribution to $|B|$ and still a semimartingale. However, when $\delta \in (0,1]$, \eqref{eq:bessel_sde} holds up until the first time $t$ such that $X_t = 0$, but it does not hold for larger $t$ since the integral $\int_{0}^{t} X_s^{-1}ds$ is infinite a.s. In order to make sense of a solution to~\eqref{eq:bessel_sde} in integrated form for $\delta \in (0,1)$, we make a principal value correction. More precisely, $X$ satisfies the equation \begin{equation} \label{eq:principal_value_correction_equation} X_t = x + \frac{\delta-1}{2}\text{P.V.}\int_{0}^{t}\frac{1}{X_s} ds + B_t. \end{equation} The principal value correction can be expressed in terms of an integral of the local time processes associated with $X$ (see \cite[Chapter XI]{revuz2013continuous}). Next, we recall the \emph{approximate Bessel processes} for $\delta \in (0,1)$ considered in \cite[Section 6]{s2009cle}. For fixed $\epsilon > 0$, we define an $\epsilon$-$\BES^{\delta}$ process $X_t^{\epsilon}$ to be the Markov process beginning at $x > 0$ that solves~\eqref{eq:bessel_sde} except that each time it hits zero it immediately jumps to $\epsilon$ and continues. Then we have that \begin{equation} \label{eq:approximate_bessel_sde} X_t^{\epsilon} = x + \frac{\delta-1}{2} \int_{0}^{t}\frac{1}{X_s^{\epsilon}}ds + B_t + J_t^{\epsilon} \end{equation} for all times $t \geq 0$, where $J_t^{\epsilon}$ is $\epsilon$ times the number of $\epsilon$-jump discontinuities of $X_t^{\epsilon}$ up to and including time $t$. Then if $t_i$ is the time of the $i$th jump, the following was shown in \cite{s2009cle}: \begin{proposition}\label{prop:epsilon_bessel} As $\epsilon \to 0$, the $X_{t}^{\epsilon}$ converge in law to a $\BES_{x}^{\delta}$ with respect to the $L^{\infty}$ metric on a fixed interval $[0,T]$, with $T>0$. As $\epsilon \to 0$ we have that \begin{enumerate}[(i)] \item $J_{T}^{\epsilon} \to 0$ if $\delta>1$, \item $J_{T}^{\epsilon} \to \infty$ if $0<\delta<1$, and \item\label{it:jtsquared} $J_{T}^{\epsilon^{2}} := \sum_{t_i \leq T} \epsilon^{2} \to 0$ for all $\delta>0$. \end{enumerate} \end{proposition} \subsection{Gaussian free fields} \label{subsec:gff} Let $D \subseteq \C$ be a simply connected domain with harmonically non-trivial boundary and let $H_{0}(D)$ be the Hilbert space closure of $C_{0}^{\infty}(D)$ with respect to the Dirichlet inner product \[ (f,g)_{\nabla} = \frac{1}{2\pi}\int_{D}\nabla f(z) \cdot \nabla g(z) dz.\] Then the zero boundary Gaussian free field (\text{GFF}) $h$ is the random distribution defined by \begin{equation}\label{eq:gff_expression} h = \sum_{n \geq 1}\alpha_n \phi_n \end{equation} where $(\phi_n)_{n \geq 1}$ is an orthonormal basis of $H_{0}(D)$ with respect to $(\cdot,\cdot)_{\nabla}$ and $(\alpha_n)_{n \geq 1}$ is a sequence of i.i.d.\ random variables with distribution $N(0,1)$. Note that a \text{GFF} on $D$ should not be considered as a function but rather as a random variable taking values in the space of distributions. The above construction also applies for other variants of the GFF. In particular, a free boundary \text{GFF} is defined in the same way except that we replace $H_{0}(D)$ by the closure $H(D)$ with respect to $(\cdot,\cdot)_{\nabla}$ of the space of functions $f \in C^{\infty}(D)$ such that $\int_{D}f(z)dz = 0$. Note that in this way, the free boundary \text{GFF} is defined in a space of distributions modulo additive constant. However, we can fix the additive constant for the field by fixing its value when acting on a fixed test function which does not have mean zero. We also note that we can define the free boundary \text{GFF} in the following way. Let $G_{D}^{N}$ be the Green's function with Neumann boundary conditions on $\partial D$ and recall that \begin{equation} \label{eqn:neumann_greens} G_{\h}^{N}(z,w) = -\log |z-w| - \log |z-\overline{w}|. \end{equation} Then, we can define the free boundary \text{GFF} on $\h$ as the centered Gaussian process with covariance kernel~$G_{\h}^{N}$. We can also define the free boundary \text{GFF} in other simply connected domains by conformally mapping a free boundary \text{GFF} on~$\h$. \subsection{Liouville quantum gravity} \label{subsec:lqg} Recall the definition of the \text{LQG} surfaces given in Section~\ref{sec:introduction}. We can also consider \text{LQG} surfaces with marked points $(D,h,x_1,\dots,x_n)$, $(\wt{D},\wt{h},\wt{x}_1,\dots,\wt{x}_n)$ where we consider them to be equivalent if~\eqref{eqn:change_of_coordinates} holds and $x_j = \varphi(\wt{x}_j)$ for all $j=1,\dots,n$. Next we give the definition of a so-called \emph{thick} quantum wedge, which is a type of quantum surface which is homeomorphic to $\h$ and has two marked points: the \emph{origin} and \emph{infinity}. Compact neighborhoods of the former have finite quantum area while any neighborhood of the latter has infinite quantum area. We recall that a quantum surface is an equivalence class under the equivalence relation defined by~\eqref{eqn:change_of_coordinates}. Thus to specify the law of a quantum surface, we are free to choose which domain we will use to parameterize it. In the case of a quantum wedge, the most convenient choice is the infinite strip $\strip = \R \times (0,\pi)$. \begin{definition} \label{def:thick_quantum_wedge} Fix $\gamma \in (0,2)$, $W > \gamma^2 / 2$, and set $\alpha = \frac{\gamma}{2} + Q - \frac{1}{\gamma}W$. A quantum wedge of weight $W$ is the quantum surface together with two marked boundary points at $-\infty$ and $+\infty$ (when parameterized by $\strip$) described by the distribution $h$ on $\strip$ whose law can be sampled from as follows. Let $X_s$ be the process defined for $s \in \R$ such that \begin{itemize} \item For $s \geq 0$, $X_s = B_{2s} + (Q - \alpha)s$ where $B$ is a standard Brownian motion with $B_0 = 0$ conditioned so that $B_{2u} + (Q - \alpha)u > 0$ for all $u > 0$ and \item For $s < 0$, $X_s = \wh{B}_{-2s} + (Q - \alpha)s$ where $\wh{B}$ is a standard Brownian motion independent of $B$ with $\wh{B}_0 = 0$. \end{itemize} Let $\mathcal{H}_1(\strip)$ be the subspace of $H(\strip)$ which consists of those functions which are constant on vertical lines of the form $u + [0,i\pi]$ for $u \in \R$ and let $\mathcal{H}_2(\strip)$ be the subspace of $H(\strip)$ which consists of those functions which have a common mean on all such vertical lines. Then $h$ is the field with projection onto $\mathcal{H}_1(\strip)$ given by the function whose common value on $u + [0,i\pi]$ is $X_u$ for $u \in \R$ and its projection onto $\mathcal{H}_2(\strip)$ is given by the corresponding projection of a free boundary GFF on $\strip$. We fix the additive constant for the projection of $h$ onto $\mathcal{H}_1(\strip)$ (resp.\ $\mathcal{H}_2(\strip)$) so that its average on $[0,i\pi]$ vanishes. \end{definition} \begin{remark} The above embedding of a quantum wedge into $\strip$ is the so-called \emph{circle average embedding}. Note that we can change the embedding $\strip$ while keeping the marked points fixed by applying~\eqref{eqn:change_of_coordinates} and using a conformal transformation $\phi : \strip \rightarrow \strip$ fixing $-\infty$ and $+\infty$, and this corresponds to translating the field $h$ by a constant $r \in \R$. \end{remark} Throughout this paper, we will use the notation $\langle \cdot \rangle$ in order to denote the quadratic variation. \begin{remark} Suppose that $X$ is as in Definition~\ref{def:thick_quantum_wedge} and let $Z_t = \exp( \gamma X_t/2)$. By It\^o's formula, \begin{align*} d\langle Z \rangle_{t} = \frac{\gamma^2}{2} Z_t^2 dt. \end{align*} By \cite[Proposition 3.4]{dms2014mating}, we see that if we reparameterize $Z$ by its quadratic variation then it evolves as a $\BES^{\delta}$ with \begin{align} \label{eqn:bessel_dimension_wrt_weight} \delta = 2 + \frac{2(Q-a)}{\gamma} = 1 + \frac{2}{\gamma^2}W. \end{align} This gives another way of sampling $h$ as follows. Start with a $\BES^{\delta}$ process $Z$ with $\delta$ as in~\eqref{eqn:bessel_dimension_wrt_weight}. Then we sample the projection of $h$ onto $\mathcal{H}_1(\strip)$ to be given by reparameterizing $2\gamma^{-1}\log(Z)$ to have quadratic variation $2dt$ and taking the horizontal translation so that it last hits $0$ at time $0$. The projection onto $\mathcal{H}_2(\strip)$ is sampled as in Definition~\ref{def:thick_quantum_wedge} and independently of $Z$. This is how we define the $a = Q$, $W = \gamma^2/2$ quantum wedge. When $\delta \geq 2$, we call the resulting wedge a \emph{thick} quantum wedge. \end{remark} We can also consider quantum surfaces parameterized by a closed set (not necessarily homeomorphic to a disk) such that each component of its interior together with its prime-end boundary is homeomorphic to the closed unit disk, and $h$ is defined as a distribution on each of these components. \begin{definition} \label{def:thin_quantum_wedge} Fix $\gamma \in (0,2)$ and $W \in (-\gamma^2 / 2, \gamma^2 / 2)$. A quantum wedge of weight $W$ on $\strip$ (with marked points at $-\infty$ and $+\infty$) is the random distribution $h$ on $\strip$ that can be sampled as follows. Let $Y$ be a $\BES^{\delta}$ process where $\delta$ is as in~\eqref{eqn:bessel_dimension_wrt_weight}. Let $\mathcal{H}_1(\strip), \mathcal{H}_2(\strip)$ be as before. We sample a countable collection of distribution valued random variables $h_{e}$ on $\strip$ indexed by the excursions~$e$ of~$Y$ from~$0$ and take the projection of $h_{e}$ onto $\mathcal{H}_1(\strip)$ to be given by reparameterizing $2\gamma^{-1}\log(e)$ to have quadratic variation $2dt$ and the projection of $h_{e}$ onto $\mathcal{H}_2(\strip)$ to be sampled according to the law of the corresponding projection of a free boundary GFF on $\strip$ (with additive constant fixed so that its average on $[0,i\pi]$ vanishes) taken to be independent of the corresponding projection of the other excursions from $0$ of $Y$. Note that $\delta \in (0,2)$ and in that case we call the quantum wedge \emph{thin} quantum wedge. \end{definition} In \cite{dms2014mating}, the thin quantum wedges are defined only when $W \in (0,\gamma^2/2)$. The definition given in Definition~\ref{def:thin_quantum_wedge} is the same as in \cite{dms2014mating} except it is for $W \in (-\gamma^2/2,\gamma^2/2)$. The difference between the regime $W \in (-\gamma^2/2,0]$ and $W \in (0,\gamma^2/2)$ is that for the latter the sum of the boundary lengths of the bubbles which make up the wedge is locally finite while in the former this is not the case. \begin{remark} Let $D \subseteq \C$ be a simply connected domain. Then we can consider a quantum wedge on $D$ with distinct marked points $x,y \in \partial D$ by starting with a quantum wedge on $\strip$ and then applying~\eqref{eqn:change_of_coordinates} for some conformal transformation $\phi : \strip \rightarrow D$ such that $\phi(-\infty) = x$ and $\phi(+\infty) = y$. \end{remark} \subsection{$\SLE_{\kappa}(\rho)$ processes with $\kappa > 0$ and $\rho > -2 - \kappa/2$} \label{subsec:slekapparho} We will now review the definition of the $\SLE_\kappa(\rho)$ processes, first focusing on the forward case and then on the reverse case. In order to differentiate between the former and the latter, we will use tildes for quantities associated with the reverse case. \subsubsection{Forward $\SLE_\kappa(\rho)$} Fix $\kappa > 0$, $\rho > -2 - \kappa/2$, and let $\delta$ be such that \begin{equation} \label{eqn:delta_and_rho} \delta = 1 + \frac{2(\rho + 2)}{\kappa}. \end{equation} Let $X$ be a $\BES^{\delta}$ process and let \begin{align*} V_t = \frac{2}{\sqrt{\kappa}}\text{P.V.}\int_{0}^{t} \frac{1}{X_s} ds\quad \text{and} \quad W = V - \sqrt{\kappa}X. \end{align*} Then an $\SLE_{\kappa}(\rho)$ process is described by the solution to the \text{ODE} \begin{align*} \partial_{t}g_t(z) = \frac{2}{g_t(z) - W_t},\quad g_0(z) = z. \end{align*} For each $t \geq 0$ we let $\h_t$ be the domain of $g_t$ and $K_t = \h \setminus \h_t$ be the hull at time $t$ associated with the $\SLE_\kappa(\rho)$ process. The location of the force point at time $0$ is given by $V_0$ and at time $t$ is given by $g_t^{-1}(V_t)$. When $\rho > -2$, we have that $\delta > 1$ and so the force point at time $t$ is located at the rightmost intersection of the hull at time $t$ with $\R$. The continuity of the process in that case was shown in \cite{ms2016imag1}. When $\rho \in (-2-\kappa/2,-2)$, the force point does not stay in $\R$ because of the principal value correction and since $\delta \in (0,1)$. The continuity of the process in the case that $\rho \in (-2-\kappa/2,\kappa/2-4]$ and $\kappa \in (2,4)$ was shown in \cite{msw2017clepercolations} while the continuity when $\rho \in (-2-\kappa/2,-2) \cap (\kappa/2-4,-2)$ and $\kappa \in (0,4)$ was shown in \cite{ms2019lightcone}. We can more generally consider the $\SLE_\kappa(\rho)$ processes with multiple force points, which we will denote by $\SLE_\kappa(\ul{\rho})$. More specially, suppose that $\ul{x} = (x_1,\ldots,x_n)$ are distinct points in $\partial \h$ (where we consider $0_-$ and $0_+$ to be distinct) and that $\ul{\rho} = (\rho_1,\ldots,\rho_n) \in \R^n$. Suppose that $B$ is a standard Brownian motion and let $W$ be the solution to the SDE \begin{align} \label{eqn:sle_kappa_rho_sde} dW_t = \sqrt{\kappa} dB_t + \sum_{i=1}^n \frac{\rho_i}{W_t - V_t^i} dt, \quad dV_t^i = \frac{2}{V_t^i-W_t} dt, \quad V_0^i = x_i. \end{align} It is shown in \cite{ms2016imag1} that there is a unique solution~\eqref{eqn:sle_kappa_rho_sde} up until the so-called \emph{continuation threshold}, which is the first time $t$ that $\sum_{i : V_t^i = W_t} \rho_i = -2$ and that the corresponding Loewner flow is generated by a continuous curve up until this time. \subsubsection{Reverse $\SLE_\kappa(\wt{\rho})$} \label{subsubsec:reverse_sle_kappa_rho} Fix $\kappa > 0$, $\wt{\rho} > 2-\kappa/2$, and let \begin{equation} \label{eqn:delta_and_rho_reverse} \wt{\delta} = 1 + \frac{2(\wt{\rho}-2)}{\kappa}. \end{equation} Let \[ \wt{V}_{t} = -\frac{2}{\sqrt{\kappa}}\text{P.V.} \int_{0}^{t}\frac{1}{\wt{X}_{s}}ds\quad\text{and}\quad \wt{W} = \wt{V} - \sqrt{\kappa}\wt{X}.\] A reverse $\SLE_{\kappa}(\wt{\rho})$ (with the force point located at $0^{+}$) centered Loewner flow is defined to be the family of conformal maps ($\wt{f_{t}}$) given by $\wt{g_{t}}(z) - \wt{W_{t}}$ where ($\wt{g_{t}}$) solve the reverse Loewner equation \begin{align} \label{eqn:rev_loewner} \partial_{t}{\wt{g}_{t}}(z) = -\frac{2}{\wt{g}_{t}(z) - \wt{W}_{t}},\quad \wt{g}_0(z) = z. \end{align} We note that~\eqref{eqn:rev_loewner} has a unique solution for every fixed $z$ in $\h$ defined for all times $t$. \subsection{Imaginary geometry} \label{subsec:ig} Fix $\kappa \in (0,4)$, $\kappa'=16/\kappa > 4$, and let \begin{equation} \label{eqn:chi_value} \lambda = \frac{\pi}{\sqrt{\kappa}},\quad \lambda' = \frac{\pi}{\sqrt{\kappa'}}, \quad\text{and}\quad \chi = \frac{2}{\sqrt{\kappa}} - \frac{\sqrt{\kappa}}{2}. \end{equation} It was shown in \cite{she2016zipper,ms2016imag1} (see also \cite{dub2009partition}) that the $\SLE_{\kappa}(\ul{\rho})$ processes with $\kappa \in (0,4)$ and $\rho > -2$ can be considered as the flow lines of the vector field $e^{ih / \chi}$, where $h$ is a GFF with fixed boundary data. Let us first explain how this works in the case of the $\SLE_\kappa$ processes. Suppose that~$h$ is a GFF on~$\h$ with boundary conditions given by $-\lambda$ on $\R_-$ and $\lambda$ on $\R_+$. Then it is shown in \cite{ms2016imag1} that there exists a unique coupling of $h$ with an $\SLE_\kappa$ process $\eta$ in $\h$ from $0$ to $\infty$ so that the following is true. Let $(g_t)$ be the Loewner flow for $\eta$, $W$ its driving function, and let $f_t = g_t - W_t$ be its centered Loewner flow. Then for every stopping time $\tau$ for $\eta$ we have that \[ h \circ f_\tau^{-1} - \chi \arg (f_\tau^{-1})' \stackrel{d}{=} h.\] In this coupling, we moreover have that $\eta$ is determined by $h$. More generally, suppose that $\eta$ is an $\SLE_\kappa(\ul{\rho})$ process where $\ul{\rho} = (\ul{\rho}^L; \ul{\rho}^R)$, $\ul{\rho}^L = (\rho_1^L,\ldots,\rho_\ell^L)$, $\ul{\rho}^R = (\rho_1^R,\ldots,\rho_k^R)$ with force points located at $\ul{x} = (\ul{x}^L;\ul{x}^R)$ with $-\infty = x_{\ell+1}^L < x_\ell^L < \cdots < x_1^L \leq x_0^L = 0_-$ and $0_+ = x_0^R \leq x_1^R < \cdots < x_k^R < x_{k+1}^R = +\infty$. Suppose that $h$ is a GFF on $\h$ with boundary conditions given by \begin{align*} -\lambda\left(1+ \sum_{i=1}^j \rho_i^L \right) \quad&\text{in}\quad (x_{i+1}^L,x_i^L] \quad\text{for}\quad 0 \leq i \leq \ell \quad \quad\text{and}\\ \lambda\left( 1+ \sum_{i=1}^j \rho_i^R \right) \quad&\text{in}\quad (x_i^R,x_{i+1}^R] \quad\text{for}\quad 0 \leq i \leq k. \end{align*} Let $(f_t)$ be the centered Loewner flow for $\eta$. Then there exists a unique coupling of $\eta$ with $h$ so that for every stopping time $\tau$ for $\eta$ we have that $h \circ f_\tau^{-1} - \chi \arg (f_\tau^{-1})'$ is a GFF on $\h$ with boundary conditions given by \begin{align*} -\lambda\left(1+ \sum_{i=1}^j \rho_i^L \right) \quad&\text{in}\quad (f_\tau(x_{i+1}^L),f_\tau(x_i^L)] \quad\text{for}\quad 0 \leq i \leq \ell \quad \quad\text{and}\\ \lambda\left( 1+ \sum_{i=1}^j \rho_i^R \right) \quad&\text{in}\quad (f_\tau(x_i^R),f_\tau(x_{i+1}^R)] \quad\text{for}\quad 0 \leq i \leq k \end{align*} where we take the convention that $f_\tau(x_0^L) = 0_-$ and $f_\tau(x_0^R) = 0_+$. In this coupling, we moreover have that $\eta$ is determined by $h$. In both of the above situations, we refer to $\eta$ as the flow line of $h$ from $0$ to $\infty$. We can more generally define the flow line of $h$ from $x$ to $\infty$ by translating $h$. Also, if $\theta \in \R$ then we define the flow line of $h$ with angle $\theta$ to be the flow line of $h + \theta \chi$. The manner in which the flow lines interact was determined in \cite{ms2016imag1}. In particular, suppose that $\eta_{x_i}^{\theta_i}$ for $i=1,2$ are the the flow lines of $h$ starting from $x_1 < x_2$ with angles $\theta_1,\theta_2$. If $\theta_1 > \theta_2$, then $\eta_{x_1}^{\theta_1}$ stays to the left of $\eta_{x_2}^{\theta_2}$. If $\theta_1 = \theta_2$, then $\eta_{x_1}^{\theta_1}$ merges with $\eta_{x_2}^{\theta_2}$ and does not subsequently separate. Finally, if $\theta_1 \in (\theta_2-\pi,\theta_2)$, then $\eta_{x_1}^{\theta_1}$ crosses $\eta_{x_2}^{\theta_2}$ upon intersecting and does not subsequently cross back. The $\SLE_{\kappa'}$ curves are also coupled with the GFF in a similar manner but the interpretation of the coupling is different. Suppose that $h$ is a GFF on $\h$ with boundary conditions given by $\lambda'$ on $\R_-$ and $-\lambda'$ on $\R_+$. Then it is shown in \cite{ms2016imag1} that there exists a unique coupling of $h$ with an $\SLE_{\kappa'}$ process $\eta'$ in $\h$ from $0$ to $\infty$ so that the following is true. Let $(g_t)$ be the Loewner flow for $\eta'$, $W$ its driving function, and let $f_t = g_t - W_t$ be its centered Loewner flow. Then for every stopping time $\tau$ for $\eta'$ we have that \[ h \circ f_\tau^{-1} - \chi \arg (f_\tau^{-1})' \stackrel{d}{=} h.\] In this coupling, we moreover have that $\eta'$ is determined by $h$. More generally, suppose that $\eta$ is an $\SLE_\kappa(\ul{\rho})$ process where $\ul{\rho} = (\ul{\rho}^L; \ul{\rho}^R)$, $\ul{\rho}^L = (\rho_1^L,\ldots,\rho_\ell^L)$, $\ul{\rho}^R = (\rho_1^R,\ldots,\rho_k^R)$ with force points located at $\ul{x} = (\ul{x}^L;\ul{x}^R)$ with $-\infty = x_{\ell+1}^L < x_\ell^L < \cdots < x_1^L \leq x_0^L = 0_-$ and $0_+ = x_0^R \leq x_1^R < \cdots < x_k^R < x_{k+1}^R = +\infty$. Suppose that $h$ is a GFF on $\h$ with boundary conditions given by \begin{align*} \lambda' \left(1+ \sum_{i=1}^j \rho_i^L \right) \quad&\text{in}\quad (x_{i+1}^L,x_i^L] \quad\text{for}\quad 0 \leq i \leq \ell \quad \quad\text{and}\\ -\lambda' \left( 1+ \sum_{i=1}^j \rho_i^R \right) \quad&\text{in}\quad (x_i^R,x_{i+1}^R] \quad\text{for}\quad 0 \leq i \leq k. \end{align*} Let $(f_t)$ be the centered Loewner flow for $\eta'$. Then there exists a unique coupling of $\eta'$ with $h$ so that for every stopping time $\tau$ for $\eta'$ we have that $h \circ f_\tau^{-1} - \chi \arg (f_\tau^{-1})'$ is a GFF on $\h$ with boundary conditions given by \begin{align*} \lambda'\left(1+ \sum_{i=1}^j \rho_i^L \right) \quad&\text{in}\quad (f_\tau(x_{i+1}^L),f_\tau(x_i^L)] \quad\text{for}\quad 0 \leq i \leq \ell \quad \quad\text{and}\\ -\lambda'\left( 1+ \sum_{i=1}^j \rho_i^R \right) \quad&\text{in}\quad (f_\tau(x_i^R),f_\tau(x_{i+1}^R)] \quad\text{for}\quad 0 \leq i \leq k \end{align*} where we take the convention that $f_\tau(x_0^L) = 0_-$ and $f_\tau(x_0^R) = 0_+$. In this coupling, we moreover have that $\eta'$ is determined by $h$. In both of the above situations, we refer to $\eta'$ as the counterflow line of $h$ from $0$ to $\infty$. In \cite{ms2016imag1} the manner in which the flow and counterflow lines of the GFF interact is also described. In particular, suppose that $h$ is a GFF on $\h$ and $\eta'$ is the counterflow line of $h$ from $\infty$ to $0$. Then the left (resp.\ right) boundary of $\eta'$ is equal to the flow line of $h$ from $0$ to $\infty$ with angle $\pi/2$ (resp.\ $-\pi/2$). It is also possible to consider flow lines starting from interior points \cite{ms2017imag4} but we will not need to consider this in the present work. \subsection{Light cones} \label{subsec:light_cones} Now, we review some basic properties of \emph{light cones} and their relation with the $\SLE_{\kappa}(\rho)$ processes. We will first review the definition of the light cone. We will focus on the case that it is defined on~$\h$; the definition on another simply connected domain is given by conformal mapping. Suppose that $h$ is a GFF on $\h$ and $x \in \partial \h$ with piecewise constant boundary conditions that change only finitely many times. Fix angles $\theta_1 \leq \theta_2 \leq \theta_1 + \pi$. The $\SLE_{\kappa}$ \emph{light cone} $\lightcone_x(\theta_1,\theta_2)$ of $h$ starting from $x$ with angle range $[\theta_1,\theta_2]$ is given by the closure of the set of points accessible by the flow lines of $h$ starting from $x$ with angles which are either rational and contained in $[\theta_1,\theta_2]$ or equal to $\theta_1$ or $\theta_2$ and which change angles a finite number of times and only at positive rational times. More generally, if $A$ is a segment of $\partial D$, we let $\lightcone_A(\theta_1,\theta_2)$ be the set of points accessible by flow lines of $h$ starting from a countable dense subset of $A$ with angles which are either rational and contained in $[\theta_1,\theta_2]$ or equal to $\theta_1$ or $\theta_2$ which change angles only a finite number of times and only at positive rational times. It was shown in \cite[Theorem 1.2]{ms2019lightcone} that the range of an $\SLE_{\kappa}(\rho)$ process for $\kappa \in (0,4)$, $\rho \in [\kappa/2-4,-2) \cap (-2 - \kappa/2,-2)$ is equal to $\lightcone_{\R_-}(0,\theta)$ when the boundary of $h$ and~$\theta$ are chosen appropriately. More precisely, the following was shown in \cite{ms2019lightcone}: \begin{theorem} \label{thm:ligthcone_and_sle} Fix $\kappa \in (0,4)$, $\rho \in [\kappa/2-4,-2)$ and $\rho > -2 - \kappa/2$, and suppose that $h$ is a \text{GFF} on $\h$ whose boundary data is given by $-\lambda$ on $\R_-$ and $\lambda (1+\rho)$ on $\R_+$. Let $\eta$ be an $\SLE_{\kappa}(\rho)$ process on $\h$ from $0$ to $\infty$ where its force point is located at $0_+$. For each $t \geq 0$, let $K_t$ denote the closure of the complement of the unbounded connected component of $\h \setminus \eta([0,t])$, let $g_t : \h \setminus K_t \to \h$ be the unique conformal transformation with $g_t(z)-z \to 0$ as $z \to \infty$, and let $(W,V)$ be the Loewner driving pair for $\eta$. There exists a unique coupling of $h$ and $\eta$ such that the following is true. For each $\eta$-stopping time $\tau$, the conditional law of \begin{align*} h \circ g_{\tau}^{-1} - \chi \arg(g_{\tau}^{-1})' \end{align*} given $\eta|_{[0,\tau]}$ is that of a \text{GFF} on $\h$ with boundary conditions given by \begin{align*} h|_{(-\infty,W_{\tau}]} \equiv -\lambda,\quad h|_{( W_{\tau},V_{\tau}]} \equiv \lambda,\quad \text{and} \quad h|_{(V_{\tau},\infty)}\equiv \lambda (1+\rho), \end{align*} where $\lambda = \pi / \sqrt{\kappa}$. Moreover, in the coupling $(h,\eta)$, $\eta$ is a.s.\ determined by $h$. Finally, let \begin{equation}\label{eqn:theta_and_rho} \theta = \theta_{\rho} = \pi \!\left ( \frac{\rho + 2}{\kappa / 2 - 2} \right ). \end{equation} Then the range of $\eta$ is a.s.\ equal to $\lightcone_{\R_-}(0,\theta)$. \end{theorem} Theorem~\ref{thm:ligthcone_and_sle} allows to interpret an $\SLE_{\kappa}(\rho)$ process with the above range of values of $\rho$ as an ordered light cone of flow lines of $e^{ih / \chi}$. \section{SLE/GFF couplings} \label{sec:slegff_couplings} The purpose of this section is to prove two results related to couplings of $\SLE$ with the GFF. In Section~\ref{subsec:reverse_coupling} we will focus on the so-called reverse coupling and will establish a version of it which holds for reverse $\SLE_\kappa(\wt{\rho})$ processes with $\wt{\rho} < 2$. In Section~\ref{subsec:law_of_sle_excursion} we use the forward coupling of $\SLE$ with the GFF in order to describe the law of an excursion of an $\SLE_\kappa(\rho)$ process in the light cone regime. \subsection{Reverse coupling with $\wt{\rho} < 2$} \label{subsec:reverse_coupling} In this subsection we will state and prove a version of the reverse coupling of $\SLE_\kappa(\wt{\rho})$ with the GFF when $\wt{\rho} < 2$. In the case of ordinary reverse $\SLE_\kappa$, this was first proved in \cite{she2016zipper} and it was extended to the case of the reverse $\SLE_\kappa(\wt{\rho})$ processes with $\wt{\rho} > 2$ in \cite{dms2014mating}. In both \cite{she2016zipper} and \cite{dms2014mating}, the process which drives the reverse $\SLE$ is a semimartingale which allows one to use the tools of ordinary stochastic calculus. In contrast, when $\wt{\rho} < 2$ the driving process for a reverse $\SLE_\kappa(\wt{\rho})$ process is not a semimartingale and this introduces some extra complications in the version that we will prove. Throughout, we fix $\kappa >0$, $\wt{\rho} \in (2-\kappa/2 ,2)$, and let \[ \gamma=\min\left( \sqrt{\kappa},\frac{4}{\sqrt{\kappa}} \right) \quad\text{and}\quad Q=\frac{2}{\gamma}+\frac{\gamma}{2}.\] (Recall from Section~\ref{subsubsec:reverse_sle_kappa_rho} that reverse $\SLE_\kappa(\wt{\rho})$ is only defined for $\wt{\rho} > 2-\kappa/2$.) Let $G_\h^N$ be the Neumann Green's function on $\h$ (recall~\eqref{eqn:neumann_greens}). Let $(\wt{f}_t)$ be the centered Loewner flow associated with a reverse $\SLE_\kappa(\wt{\rho})$ process and let \begin{align*} \wt{G}_{t}(y,z) = G_\h^N(\wt{f}_{t}(y),\wt{f}_{t}(z)),\quad \wh{\Fh}_{0}(z) = \frac{2}{\sqrt{\kappa}}\log|z|,\quad \text{and}\quad \wh{\Fh}_{t}(z) = \wh{\Fh}_{0}(\wt{f}_{t}(z)) + Q\log|\wt{f}_{t}'(z)|. \end{align*} The following is the main theorem of this subsection and serves to extend \cite[Theorem~1.2]{she2016zipper} and \cite[Theorem~5.1]{dms2014mating}. \begin{theorem} \label{thm:reverse_coupling} Let $\kappa$, $\wt{\rho}$ be as above, let $\wt{W}$ be the driving function associated with a reverse $\SLE_{\kappa}(\wt{\rho})$ with a single force point located at $0_+$ of weight $\wt{\rho}$, and let $(\wt{f}_{t})$ be the centered reverse Loewner evolution driven by $\wt{W}$. For each $t \geq 0$, let \begin{align*} \wt{\Fh}_{t}(z) = \wh{\Fh}_{t}(z) + \frac{\wt{\rho}}{2\sqrt{\kappa}}\wt{G}_{t}(0_+,z) \end{align*} and let $h$ be an instance of the free boundary $\text{GFF}$ on $\h$ independent of $\wt{W}$. Suppose that $\wt{\tau}$ is an a.s.\ finite stopping time for the filtration generated by the Brownian motion which drives $\wt{W}$. Then \begin{align*} \wt{\Fh}_{0} + h \stackrel{d}{=} \wt{\Fh}_{\wt{\tau}} + h \circ \wt{f}_{\wt{\tau}} \end{align*} where the left and right sides are viewed as distributions modulo additive constant. \end{theorem} Recall from Section~\ref{subsubsec:reverse_sle_kappa_rho} that $\wt{f}_t(0_+)/\sqrt{\kappa}$ is a $\BES^{\wt{\delta}}$ process with \[ \wt{\delta} = 1+ \frac{2(\wt{\rho}-2)}{\kappa}.\] The proofs of \cite[Theorem~1.2]{she2016zipper} and \cite[Theorem~5.1]{dms2014mating} make use of stochastic calculus and require that $\int_0^t \wt{f}_s(0_+)^{-1}ds$ is finite for all $t\geq 0$ a.s. However, when $\wt{\rho} \in (2-\kappa/2, 2)$ we have that $\wt{\delta} \in (0,1)$ so that this integral is infinite a.s.\ and this fact is the key difference in the proof of Theorem~\ref{thm:reverse_coupling} in comparison to that given in \cite{she2016zipper,dms2014mating}. In order to overcome this difficulty, we are going to use Proposition~\ref{prop:epsilon_bessel} to approximate $\wt{f}_t(0_+)/\sqrt{\kappa}$ using an $\epsilon$-$\BES^{\wt{\delta}}$ process and then take a limit as $\epsilon \to 0$. Moreover, we will also make use of the following generalization of It\^o's formula for semimartingales with discontinuities. \begin{theorem} \label{thm:general_Ito} Suppose that $Z_{t} = X_{t} + iY_{t}$ is a semimartingale (not necessarily continuous) with $X_{t} = \re(Z_{t})$ and $Y_{t} = \im(Z_{t})$ and $Z_{t} \in U$ for all $t$ for $U \subseteq \C$ open. Then if $f$ is an analytic function on $U$, it holds that \begin{align*} f(Z_{t}) - f(Z_{0}) &= \int_{0}^{t}f'(Z_{s})dZ_{s} + \frac{1}{2}\int_{0}^{t}f''(Z_{s^{-}})d\langle Z^{c}\rangle_{s}\\ &+\sum_{0 \leq s \leq t}\left( f(Z_{s}) - f(Z_{s^{-}}) - f'(Z_{s^{-}})\Delta Z_{s}\right) \end{align*} \end{theorem} Let us now perform some initial calculations which will be used in the proof of Theorem~\ref{thm:reverse_coupling}. Let~$B$ be the Brownian motion which drives $\wt{X}$ and let $\wt{X}^{\epsilon}$ be the solution of~\eqref{eq:approximate_bessel_sde}. Let also $\wt{J}_t^\epsilon$ be as in~\eqref{eq:approximate_bessel_sde} and $\wt{J}_t^{\epsilon^2}$ be as in part~\eqref{it:jtsquared} of Proposition~\ref{prop:epsilon_bessel} (so that $\wt{J}_t^{\epsilon^2}$ is $\epsilon^2$ times the number of $\epsilon$-jump discontinuities of $X_t^\epsilon$ up to and including time $t$). We consider the processes \begin{align*} \wt{Y}^\epsilon = \frac{2}{\wt{\delta} - 1}(\wt{X}^{\epsilon} - B) \quad\text{and}\quad \wt{W}^{\epsilon} = -\frac{2}{\sqrt{\kappa}}\wt{Y}^{\epsilon} - \sqrt{\kappa}\wt{X}^{\epsilon} \end{align*} By Proposition~\ref{prop:epsilon_bessel}, we can couple $\wt{X}^{\epsilon}$ and $\wt{X}$ such that $\wt{X}^{\epsilon}$ converges to $\wt{X}$ as $\epsilon \to 0$ uniformly on compact intervals a.s. Fix $z \in \h$ and let \begin{align*} \wt{Z}_{t}^{\epsilon} = \wt{g}_{t}(z) + \frac{2}{\sqrt{\kappa}}\wt{Y}_{t}^{\epsilon} + \sqrt{\kappa}\wt{X}_{t}^{\epsilon}. \end{align*} Note that $\wt{Z}^{\epsilon}$ converges to $\wt{f}_{t}(z)$ as $\epsilon \to 0$ uniformly on compact intervals a.s. Since \[ \langle \wt{Z}^{\epsilon}\rangle_{t} = \kappa t + \left(\sqrt{\kappa} + \frac{4}{\sqrt{\kappa}(\wt{\delta} - 1)}\right)^2 \wt{J}_{t}^{\epsilon^{2}},\] by applying Theorem~\ref{thm:general_Ito} we obtain that \begin{align*} \frac{2}{\sqrt{\kappa}}\left( \log \wt{Z}_{t}^{\epsilon} - \log\wt{Z}_{0}^{\epsilon}\right) = \wt{\alpha}_{t}^{\epsilon} + \frac{2}{\sqrt{\kappa}}\int_{0}^{t}\frac{1}{\wt{Z}_{s^{-}}^{\epsilon}}d \wt{Z}_{s}^{\epsilon} - \sqrt{\kappa}\int_{0}^{t}\frac{1}{(\wt{Z}_{s^{-}}^{\epsilon})^{2}}ds \end{align*} where \begin{align*} \wt{\alpha}_{t}^{\epsilon} = \frac{2}{\sqrt{\kappa}}\sum_{0 \leq s \leq t}\left( \log \wt{Z}_{s}^{\epsilon} - \log \wt{Z}_{s^-}^{\epsilon} - \frac{\Delta \wt{Z}_{s}^{\epsilon}}{\wt{Z}_{s^{-}}^{\epsilon}}\right). \end{align*} Similarly we have that \[ \langle \wt{A}^{\epsilon}\rangle_{t} = \frac{16 \wt{J}_{t}^{\epsilon^{2}}}{\kappa (\wt{\delta} -1)^{2}} \quad\text{where}\quad \wt{A}_{t}^{\epsilon} = \wt{g}_t(z) + \frac{2}{\sqrt{\kappa}} \wt{Y}_{t}^{\epsilon}\] and hence by applying Theorem~\ref{thm:general_Ito} again we obtain that \begin{align*} \frac{\wt{\rho}}{\sqrt{\kappa}}\left( \log \wt{A}_{t}^{\epsilon} - \log\wt{A}_{0}^{\epsilon}\right) = \wt{\beta}_{t}^{\epsilon} + \frac{\wt{\rho}}{\sqrt{\kappa}}\int_{0}^{t}\frac{1}{\wt{A}_{s^-}^{\epsilon}}d\wt{A}_{s}^{\epsilon} \end{align*} where \begin{align*} \wt{\beta}_{t}^{\epsilon} = \frac{\wt{\rho}}{\sqrt{\kappa}}\sum_{0 \leq s \leq t}\left(\log \wt{A}_{s}^{\epsilon} - \log \wt{A}_{s^-}^{\epsilon} - \frac{\Delta \wt{A}_{s}^{\epsilon}}{\wt{A}_{s^-}^{\epsilon}}\right). \end{align*} Let $(t_i)$ be as in~\eqref{eq:approximate_bessel_sde}. Since $\wt{X}_{t_{i}^-}^{\epsilon} = 0$ we have that $\wt{A}_{t_{i}^{-}}^{\epsilon} = \wt{Z}_{t_{i}^-}^{\epsilon}$ and so \begin{align} &\frac{2}{\sqrt{\kappa}}\left( \log \wt{Z}_{t}^{\epsilon} - \log \wt{Z}_{0}^{\epsilon} \right) - \frac{\wt{\rho}}{\sqrt{\kappa}} \left( \log\wt{A}_{t}^{\epsilon} - \log \wt{A}_{0}^{\epsilon} \right) \nonumber\\ =&(\wt{\alpha}_{t}^{\epsilon} - \wt{\beta}_{t}^{\epsilon}) +\int_{0}^{t}\frac{2}{\wt{Z}_{s^-}^{\epsilon}}dB_{s} - \frac{2}{\sqrt{\kappa}}\int_{0}^{t}\frac{2}{\wt{Z}_{s^-}^{\epsilon}\wt{f}_{s}(z)}ds\nonumber\\ &-\sqrt{\kappa}\int_{0}^{t}\frac{1}{(\wt{Z}_{s^-}^{\epsilon})^{2}}ds + \frac{2\wt{\rho}}{\sqrt{\kappa}}\int_{0}^{t}\frac{\wt{Z}_{s^-}^{\epsilon} - \wt{f}_{s}(z)}{\wt{A}_{s^-}^{\epsilon}\wt{f}_{s}(z) \wt{Z}_{s^-}^{\epsilon}}ds.\label{eqn:basic_equation} \end{align} Fix $T > 0$. It is easy to see that \[ \log \wt{Z}_{t_i}^{\epsilon} - \log \wt{Z}_{t_{i}^-}^{\epsilon} - \frac{\Delta \wt{Z}_{t_i}^{\epsilon}}{\wt{Z}_{t_{i}^-}^{\epsilon}} = O((\Delta \wt{Z}_{t_i}^{\epsilon})^2) \quad\text{for all}\quad t_i \leq T,\] where the implicit constant is random but independent of $\epsilon$. Since \[ \Delta \wt{Z}_{t_i}^{\epsilon} = \left( \sqrt{\kappa} + \frac{4}{\sqrt{\kappa}(\wt{\delta} - 1)} \right) \epsilon\] we obtain that $\wt{\alpha}_{T}^{\epsilon} = O(\wt{J}_{T}^{\epsilon^2})$, where the implicit constant is random and independent of $\epsilon$. Therefore by applying Proposition~\ref{prop:epsilon_bessel} we obtain that $\wt{\alpha}_{t}^{\epsilon}$ converges to zero as $\epsilon \to 0$ uniformly on compact intervals a.s. A similar analysis holds for $\wt{\beta}_{t}^{\epsilon}$. Moreover, it is easy to see that \[ \int_{0}^{t}\frac{1}{\wt{Z}_{s^-}^{\epsilon}}dB_{s} \to \int_{0}^{t}\frac{1}{\wt{f}_{s}(z)}dB_{s} \quad\text{in}\quad L^{2} \quad\text{as}\quad \epsilon \to 0.\] In particular, the convergence holds in probability. Therefore the above observations imply that the right hand side of~\eqref{eqn:basic_equation} converges in probability as $\epsilon \to 0$ to \begin{align} \label{eq:RHS_convergence} \int_{0}^{t} \frac{2}{\wt{f}_{s}(z)}dB_{s} - Q\int_{0}^{t} \frac{2}{(\wt{f}_{s}(z))^2}ds \end{align} while the left hand side of~\eqref{eqn:basic_equation} converges a.s.\ as $\epsilon \to 0$ to \begin{align} \label{eq:LHS_convergence} \frac{2}{\sqrt{\kappa}}\left(\log \wt{f}_{t}(z) - \log z\right) - \frac{\wt{\rho}}{\sqrt{\kappa}}\left(\log(\wt{f}_{t}(z) - \wt{f}_{t}(0^{+})) - \log z \right) \end{align} Hence the quantities in~\eqref{eq:RHS_convergence} and~\eqref{eq:LHS_convergence} are equal for all $t$ a.s. Next, we follow the same strategy as in \cite[Section~5.1]{dms2014mating} in order to prove Theorem~\ref{thm:reverse_coupling}. We will restate the lemmas used in order to make clear how the equality of the expressions in~\eqref{eq:RHS_convergence} and~\eqref{eq:LHS_convergence} is used. \begin{lemma}[$\text{\cite[Lemma~5.2]{dms2014mating}}$] \label{lem:helping_lemma} For each $z \in \h$, we have that \begin{align*} d\wt{\Fh}_{t}(z) = \re\frac{2}{\wt{f}_{t}(z)}dB_{t} \end{align*} For each $y,z \in \h$ distinct, we have that \begin{align*} d\wt{G}_{t}(y,z) = -\re\left(\frac{2}{\wt{f}_{t}(y)}\right)\re\left(\frac{2}{\wt{f}_{t}(z)}\right)dt \end{align*} \end{lemma} \begin{proof} By~\eqref{eqn:rev_loewner} we obtain that \begin{align*} d\wt{f}_{t}'(z) = \frac{2\wt{f}_{t}'(z)}{\wt{f}_{t}^{2}(z)}dt \end{align*} and thus \begin{align*} \log \wt{f}_{t}'(z) = \int_{0}^{t}\frac{2}{(\wt{f}_{t}(z))^{2}}ds. \end{align*} The first claim then follows by taking real parts in~\eqref{eq:RHS_convergence} and~\eqref{eq:LHS_convergence} and then adding $Q\log|\wt{f}_{t}'(z)|$ to both of them. \newline As for the second claim, we apply the same argument as in \cite[Lemma~5.2]{dms2014mating}. \end{proof} \begin{lemma}[$\text{\cite[Lemma~5.3]{dms2014mating}}$]\label{lem:square_integrable_martingale} For each $ t \geq 0$, $\wt{\Fh}_{t} + h \circ\wt{f}_{t}$ a.s.\ takes values in the space of modulo additive constant distributions. Suppose that $\phi \in C_{0}^{\infty}(\h)$ with $\int_{\h}\phi(z)dz = 0$. Then both $(\wt{\Fh}_{t} + h \circ \wt{f}_{t},\phi)$ and $(\wt{\Fh}_{t},\phi)$ are a.s.\ continuous in $t$ and the latter is a square-integrable martingale. \end{lemma} \begin{proof} It follows from the proof of \cite[Lemma~5.3]{dms2014mating}. \end{proof} For each $\phi \in C_{0}^{\infty}(\h)$ with $\int_{\h}\phi (z)dz = 0$ and $t \geq 0$ we let \begin{align*} \wt{E}_{t}(\phi) = \int_{\h}\int_{\h} \phi(y)\wt{G}_{t}(y,z)\phi(z)dydz \end{align*} be the conditional variance of $(h \circ\wt{f}_{t},\phi)$ given $\wt{f}_{t}$. \begin{lemma}[$\text{\cite[Lemma~5.4]{dms2014mating}}$] \label{lem:variation_formula} For each $\phi \in C_{0}^{\infty}(\h)$ with $\int_{\h} \phi (z)dz = 0$ we have that \begin{align*} d\langle (\wt{\Fh}_{\cdot},\phi)\rangle_t = -d\wt{E}_{t}(\phi) \end{align*} \end{lemma} \begin{proof} It follows from the proof of \cite[Lemma~5.4]{dms2014mating}. \end{proof} We are now ready to prove Theorem~\ref{thm:reverse_coupling}. \begin{proof} It follows by combining Lemmas~\ref{lem:helping_lemma}, \ref{lem:square_integrable_martingale}, and~\ref{lem:variation_formula} as in the proof of \cite[Theorem~5.1]{dms2014mating}. \end{proof} \subsection{Law of an $\SLE_{\kappa}(\rho)$ excursion}\label{subsec:law_of_sle_excursion} In this section, we will give the conditional law of the excursion of an $\SLE_{\kappa}(\rho)$ process in $\h$ with $\kappa \in (0,4)$ and $\rho \in (\kappa/2-4,-2) \cap (-2 -\kappa/2,-2)$ with the force point at $0_+$ which separates a given boundary point $x > 0$ from $\infty$ given the realization of the path before the start of the excursion. \begin{theorem} \label{thm:law_of_sle_excursion} Fix $\kappa \in (0,4)$, $\rho \in (\kappa/2 - 4, -2)$, $\rho > -\kappa/2 - 2$. Let $\eta$ be an $\SLE_{\kappa}(\rho)$ process in $\h$ from $0$ to $\infty$ with the force point located at $0_+$. Fix $x > 0$ and let $T_x$ be the first time that~$\eta$ separates~$x$ from $\infty$. Let $S_x = \sup\{ t < T_x : W_t = V_t\}$ be the most recent time before $T_x$ that~$\eta$ hits its force point. Then the conditional law of $\eta|_{[S_x,T_{x}]}$ given $\eta|_{[0,S_x]}$ and $\eta(T_x)$ is that of an $\SLE_{\kappa}(\rho+2; \kappa-4-\rho)$ process from $\eta(S_x)$ to $\eta(T_x)$ in $\h_{S_x}$ with the weight $\rho+2$ force point located at $\infty$ and the weight $\kappa-4-\rho$ force point located at $(\eta(S_x))^+$. \end{theorem} In order to give the proof of Theorem~\ref{thm:law_of_sle_excursion}, we first need to collect the following lemma. \begin{lemma} \label{lem:sle_separates_points} Fix $\kappa \in (0,4)$, $\rho \in (\kappa/2 - 4, -2)$, $\rho > -\kappa/2 - 2$. Let $\eta$ be an $\SLE_{\kappa}(\rho)$ process in~$\h$ from~$0$ to~$\infty$ with the force point located at $0_+$. Then for each fixed $x>0$, we have that $\eta$ does not hit $x$ and separates $x$ from $\infty$ a.s. \end{lemma} \begin{proof} \noindent{\it Overview.} The proof will be carried out in three steps. First of all, by the scale invariance of the law of $\eta$, it suffices to prove the claim of the lemma when $x=1$. In Step $1$, we show that $\eta$ hits $(1,\infty)$ with positive probability. In Step 2, we use the conformal Markov property of $\SLE_\kappa(\rho)$ in order to show that $\eta$ in fact hits $(1,\infty)$ a.s. Finally, in Step 3 we prove that $\eta$ a.s.\ does not hit $1$. \noindent{\it Step 1. $\eta$ hits $(1,\infty)$ with positive probability.} Let $(V,W)$ be the driving pair encoding $\eta$ as in Section~\ref{subsec:slekapparho} and let $\tau$ be the first time that $\eta$ hits $(1,\infty)$. Let also $(g_t)$ be the family of conformal maps solving the Loewner equation used to define $\eta$. Since $\eta$ is continuous, there exists $\epsilon_1>0$ so that $\p[\eta([0,\epsilon_1]) \subseteq B(0,\frac{1}{2})] > 0$. Moreover, since $\kappa^{-1/2} (V-W)$ is a $\BES^{\delta}$ process starting from $0$ with $\delta$ as in~\eqref{eqn:delta_and_rho}, we obtain that $V_{\epsilon_1} \neq W_{\epsilon_1}$ a.s.\ and so there exist $p,\epsilon_2 \in (0,1)$ such that $\p[A] \geq p$, where $A$ is the event that $\eta([0,\epsilon_1]) \subseteq B(0,\frac{1}{2})$ and $\epsilon_2 \leq V_{\epsilon_1} - W_{\epsilon_1} \leq \epsilon_2^{-1}$. The Markov property of $\eta$ implies that if $\mathcal{F}_t = \sigma(\eta|_{[0,t]})$, then conditioned on $\mathcal{F}_{\epsilon_1}$ and restricted to $A$, the curve $\wt{\eta}(t) = g_{\epsilon_1}(\eta(\epsilon_1 + t)) - W_{\epsilon_1}$ has the law of an $\SLE_{\kappa}(\rho)$ process in $\h$ from $0$ to $\infty$ with the force point located at $V_{\epsilon_1} - W_{\epsilon_1}$. Let $\gamma$ be the concatenation of the segments $[0,i]$, $[i,i+\epsilon_2^{-1}]$ and $[i+\epsilon_2^{-1},\epsilon_2^{-1}]$. By \cite[Lemma~2.5]{mw2017slepaths}, we obtain that there exists a constant $q \in (0,1)$ such that conditionally on $\mathcal{F}_{\epsilon_1}$ and on the event $A$, we have that with probability at least $q$, the curve $\wt{\eta}$ hits $(\epsilon_2^{-1}-\epsilon_2,\epsilon_2^{-1}+\epsilon_2)$ before exiting the $\epsilon_2 / 2$-neighborhood of $\gamma$. The claim then follows possibly by taking $\epsilon_1$ to be smaller since standard estimates for conformal maps imply that $|g_{\epsilon_1}(z)-z|\leq 6 \epsilon_1$ for all $z \in \h_{\epsilon_1}$, where~$\h_{\epsilon_1}$ is the unbounded connected component of~$\h \setminus \eta([0,\epsilon_1])$. \noindent{\it Step 2. $\eta$ hits $(1,\infty)$ a.s.} By Step 1, we have that $\eta$ hits $(1,\infty)$ with positive probability. Let $p$ be the probability that $\eta$ hits $(1,\infty)$ and assume that $p \in (0,1)$. Fix $\epsilon \in (0,p)$. Then there exists $T > 0$ so that the probability that $\eta|_{[0,T]}$ hits $(1,\infty)$ is at least $p-\epsilon$. Let $\tau = \inf\{t \geq T : \eta(t) \in \R_+\}$. On the event that $\eta|_{[0,\tau]}$ has not hit $(1,\infty)$, we let $\wt{\eta} = (g_\tau(\eta) - W_\tau)/(g_\tau(1) - W_\tau)$. Then we have that~$\wt{\eta}$ hits $(1,\infty)$ with probability $p$. Overall, this implies that the probability that $\eta$ hits $(1,\infty)$ is at least $p - \epsilon + (1-p+\epsilon)p$. By taking $\epsilon > 0$ sufficiently small we see that $p - \epsilon + (1-p+\epsilon)p > p$, which is a contradiction. Therefore $p=1$. \noindent{\it Step 3. $\eta$ does not hit $1$ a.s.} Let $p$ be the probability that $\eta$ hits $1$. By the argument given in Step~1, we have that $p < 1$. Assume that $p > 0$ and fix $q \in (0,1)$ so that $p < q < 1$. Fix $\delta > 0$ sufficiently small so that the probability that $\eta$ hits $B(1,\delta)$ is at most $q$. Let $\tau = \inf\{t \geq 0 : \eta(t) \in B(1,\delta)\}$. On the event that $\tau < \infty$, let $\phi_\tau = (g_\tau - W_\tau)/(V_\tau - W_\tau)$ so that $\wt{\eta} = \phi_\tau(\eta)$ is an $\SLE_\kappa(\rho)$ process in $\h$ from $0$ to $\infty$ with its force point located at $1$. Let $\wt{\tau} = \inf\{t \geq 0 : \wt{\eta}(t) \in (1,\infty)\}$. Let $p_1(\delta)$ be the probability that $\wt{\eta}(\wt{\tau}) \neq \phi_{\tau}(1)$ conditionally on the event that $\tau < \infty$. Then, \cite[Lemma~2.5]{mw2017slepaths} implies that there exists $p_2 > 0$ independent of $\delta$ so that $p_1(\delta) \geq p_2$. Let $p_{1,L}(\delta)$ be the probability that $\wt{\eta}(\wt{\tau}) < \phi_\tau(1)$ and $p_{1,R}(\delta)$ be the probability that $\wt{\eta}(\wt{\tau}) > \phi_\tau(1)$, conditionally on the event that $\tau<\infty$, so that $p_1(\delta) = p_{1,L}(\delta) + p_{1,R}(\delta)$. On the event that $\wt{\eta}(\wt{\tau}) < \phi_\tau(1)$, by the conformal Markov property of $\SLE_\kappa(\rho)$ the probability that it subsequently hits $\phi_\tau(1)$ is $p$. Altogether, we have that \[ p \leq q ( (1-p_1(\delta)) + p_{1,L}(\delta) p) \leq q( (1-p_1(\delta)) + p_1(\delta) p) \leq q(1-p_2(1-p)).\] Since $ 1-p_2(1-p)< 1$, this is a contradiction unless $p = 0$ since by taking $\delta > 0$ small we can assume that $p < q < 1$ is arbitrarily close to $p$. \end{proof} \begin{figure}[ht!] \begin{center} \includegraphics[scale=0.85]{figures/sle_excursion_strip.pdf} \end{center} \caption{\label{fig:sle_excursion_strip} Illustration of the setup to prove Theorem~\ref{thm:law_of_sle_excursion}. Shown in orange is the $\SLE_\kappa(\rho)$ process $\eta$, which we view as the light cone path from $0$ to $i\pi$ coupled with the GFF on $\strip$ with the indicated boundary data. The counterflow lines $\eta_{\mathrm{L}}'$ and $\eta_{\mathrm{R}}'$ are shown in red and blue, respectively, and their common outer boundary gives the completion of the excursion that $\eta$ is drawing at the time $\sigma$ (shown in green).} \end{figure} \begin{proof}[Proof of Theorem~\ref{thm:law_of_sle_excursion}] \noindent{\it Step 1. Setup and outline of the proof.} First we note that Lemma~\ref{lem:sle_separates_points} implies that $T_x < \infty$ a.s. We are going to apply arguments which are similar to those presented in the proofs of \cite[Theorem~5.6]{dms2014mating} and \cite[Proposition~5.12]{ms2016imag2}. We will in particular make use of the coupling of $\eta$ with the $\text{GFF}$ light cone introduced in \cite{ms2019lightcone} (see Section~\ref{subsec:light_cones} for a review of this). First, we apply a conformal transformation $\h \to \strip$ taking $0$ to $0$ and $\infty$ to $i \pi$. Then we can view $\eta$ as the light cone exploration path from $0$ to $i \pi$ associated with a $\text{GFF}$ $h$ on $\strip$ and $h$ has boundary values given by $-\lambda$ on $\R_-$, $\lambda (1+\rho)$ on $\R_+$, $-\lambda - \pi \chi$ on $\R_- + i \pi$, and $\lambda (1+\rho) + \pi \chi$ on $\R_+ + i\pi$. (The reason for making this change of coordinates is that it will be easier to read off the law of different objects associated with the GFF on $\strip$ versus $\h$.) Let $\eta_{\text{L}}'$ be the counterflow line of $h+ \pi \chi/2$ starting from $i \pi$ and targeted at $0$. As usual, we let $\kappa'=16/\kappa$. Then we note that $\eta_{\text{L}}'$ has the law of an $\SLE_{\kappa}(\rho_{\text{L}}' ; \rho_{\text{R}}')$ process where $\rho_{\text{L}}' = \kappa'(1+\rho/4)-4$ and $\rho_{\text{R}}' = \kappa'/2-2$ where the force points are located at $(i\pi)_-$ and $(i\pi)_+$. Indeed, the reason for this is that $\rho_{{\mathrm L}}'$, $\rho_{{\mathrm R}}'$ are determined by the equations \[ \lambda'(1+\rho_{{\mathrm L}}') = \lambda(1+\rho)+ \frac{3\pi\chi}{2} \quad\text{and}\quad -\lambda'(1+\rho_{{\mathrm R}}') = -\lambda -\frac{\pi \chi}{2}.\] Hence, a.s.\ $\eta_{\text{L}}'$ does not intersect either $\R_-$ or $\R_- + i \pi$ since $\kappa'/2-2$ is the critical value below which such processes can hit the boundary. The proof is then divided into three subsequent steps. In Step~2, we will describe the interaction between $\eta_{\text{L}}'$ and the pockets formed by $\eta$. In Step~3, we are going to prove that the joint law of $\eta|_{[S_x,T_x]}$ together with certain flow lines satisfies the same resampling property as stated in \cite[Proposition~5.12]{ms2016imag2}. Finally, in Step~4, we complete the proof by observing that if $\eta|_{[S_x,T_x]}$ had the law of an $\SLE_{\kappa}(\rho+2 ; \kappa-4-\rho)$ process, then it would satisfy the same resampling property. \cite[Proposition~5.10]{ms2016imag2} implies that the conditional law of $\eta|_{[S_x,T_x]}$ is uniquely determined by this resampling property and so this completes the proof of the lemma. \noindent{\it Step 2. Interaction between $\eta_{\text{L}}'$ and the pockets of $\eta$.} We now describe the interaction between $\eta_{\text{L}}'$ and the pockets formed by $\eta$ as described in \cite{ms2019lightcone}. Let $P$ be a pocket of formed by $\eta$ with $u$ (resp.\ $v$) its opening (resp.\ closing) point. Let $\eta_1(P)$ be the $0$-angle flow line part of $\partial P$ and $\eta_2(P)$ be the $\theta_{\rho}$-angle flow line part of $\partial P$ with $\theta_{\rho}$ as in~\eqref{eqn:theta_and_rho}. For $z \in \strip$, we let $\eta_z'$ be the counterflow line of $h+\pi \chi/2$ starting from $i \pi$ and targeted at $z$. Then $\eta_{\text{L}}'$ and $\eta_z'$ agree up until they separate $z$ from $0$ a.s., and the right side of $\eta_z'$ stopped upon hitting $z$ is equal to the flow line of $h$ starting from $z$ with angle $0$. It in particular follows that $\eta_{\text{L}}'$ hits the pocket of $\eta$ containing $z$ provided the flow line of $h$ starting from $z$ exits $\strip$ in $\R_+$ or $\R_+ + i \pi$. Assume that we are on this event. Combining with the flow line interaction rules \cite[Theorem~1.7]{ms2017imag4} with \cite[Section~3]{ms2019lightcone}, we obtain that $\eta_{\text{L}}'$ interacts with $P$ in the following way. \begin{enumerate}[(i)] \item $\eta_{\text{L}}'$ enters the interior of $P$ at $u$ after filling the left side of $\eta_1(P)$ in reverse chronological order, where we view $\eta_1(P)$ as an oriented path from $u$ to $v$. \item Upon intersecting $P$, $\eta_{\text{L}}'$ visits some points on the right side of $\eta_1(P)$ as it travels from $u$ to $v$. It does not touch $\eta_2(P)$ up until hitting $v$. \item Upon hitting $v$, it visits all the points of $\eta_2(P)$ in reverse chronological order and, while doing so, $\eta_{\text{L}}'$ makes excursions both into and out of $P$. \end{enumerate} The above interaction rules imply that $\eta_{\text{L}}'$ contains the pockets $P$ of $\eta$ such that the the flow line corresponding to $\eta_1(P)$ exits $\strip$ in $\R_+$ or $\R_+ + i \pi$ a.s. In particular, if $\eta$ makes an excursion away from where it hits its force point and the excursion terminates in $\R_+$ or $\R_+ + i \pi$, then $\eta_{\text{L}}'$ has to hit it at its tip and then fill the path in reverse chronological order a.s. Hence, the excursion that $\eta$ completes at time $T_x$ is contained in $\eta_{\text{L}}'$ a.s.\ and corresponds to an excursion that $\eta_{\text{L}}'$ makes away from $\R_+ \cup (\R_+ + i \pi)$. \noindent{\it Step 3. Resampling argument.} Suppose that $\sigma$ is a stopping time for $\eta$. We assume that we are working on the event that $\eta$ does not hit its force point at time $\sigma$ and that $S_x \leq \sigma \leq T_x$. We then let~$\eta_{\text{R}}'$ be the counterflow line of the restriction of $h-\frac{\pi \chi}{2}$ to the connected component $D$ of $\strip \setminus \eta([0,\sigma])$ whose boundary contains $i\pi$, which starts from the point corresponding to where~$\eta$ has most recently before time $\sigma$ hit its force point, and it is targeted at $\eta(\sigma)$. Then $\eta_{\text{R}}'$ has the law of an $\SLE_{\kappa}(\rho_{1,\text{L}}';\rho_{1,\text{R}}',\rho_{2,\text{R}}')$ process in $D$ with $\rho_{1,\text{L}}' = \kappa'/2-2$, $\rho_{1,\text{R}}' = -(\kappa'/4)(2+\rho)$, and $\rho_{1,\text{R}}'+\rho_{2,\text{R}}'=\kappa'-4$, and the force points are located immediately to the left and right of its starting point and at $i\pi$. Indeed, the reason for this is that $\rho_{1,\text{L}}'$, $\rho_{1,\text{R}}'$, and $\rho_{2,\text{R}}'$ are determined by the equations \[ \lambda'(1+\rho_{1,\text{L}}') = \lambda + \frac{\pi\chi}{2}, \quad -\lambda'(1+\rho_{1,{\mathrm R}}') = \lambda(1+\rho) + \frac{\pi\chi}{2} ,\quad\text{and}\quad -\lambda'(1+\rho_{1,{\mathrm R}}'+\rho_{2,{\mathrm R}}') = -\lambda- \frac{3\pi \chi}{2}.\] Let $\ol{T}$ be a stopping time for the filtration generated by $\eta|_{[0,\sigma]}$ and the time-reversal of $\eta|_{[\sigma,T_x]}$. Note that the segment of $\eta$ from $\eta(\sigma)$ to $\eta(T_x)$ in the clockwise order has boundary conditions $\lambda'+\chi \cdot \text{winding}$ (resp. $-\lambda'+\chi \cdot \text{winding}$) on its right (resp. left) side. Also, the flow line interaction rules imply that the right outer boundary of $\eta_{\text{R}}'$ is given by the flow line of $h|_D$, starting from $\eta(\sigma)$. Conditionally on $\eta|_{[0,\sigma]}$, we let $T_q'$ be the first time that $\eta_q'$ hits $\eta(\ol{T})$ for $q \in \{L,R\}$. Then, $\eta([\ol{T},T_x])$ is equal to the common part of the outer boundaries of $\eta_{\text{L}}'([0,T_L'])$ and $\eta_{\text{R}}'([0,T_R'])$. Let $\eta_{\text{L}}$ (resp.\ $\eta_{\text{R}}$) be the right (resp.\ left) outer boundary of $\eta_{\text{L}}'([0,T_{\text{L}}'])$ (resp.\ $\eta_{\text{R}}'([0,T_{\text{R}}'])$). Then, arguing as in the proof of \cite[Lemma~5.13]{ms2016imag2} gives that the conditional law of $\eta|_{[\sigma,\ol{T}]}$ given $\eta|_{[0,\sigma]}$, $\eta_{\text{L}}$, $\eta_{\text{R}}$, and $\eta|_{[\ol{T},T_x]}$, is that of an $\SLE_{\kappa}(\kappa/2-2;\kappa/2-2)$ process in $D$ starting from $\eta(\sigma)$ and targeted at $\eta(\ol{T})$ with the force points located at $i \pi$ and at the left of the starting point of $\eta_{\text{R}}'$. \noindent{\it Step 4. Conclusion of the proof.} Now we are going to identify the law of $\eta|_{[S_x,T_x]}$ given $\eta|_{[0,S_x]}$ and $\eta(T_x)$. Conditionally on $\eta|_{[0,T_{x}]}$, we sample an $\SLE_{\kappa}(\rho+2 ; \kappa-4-\rho)$ process $\wt{\eta}$ in $D$ from~$\eta(\sigma)$ to~$\eta(T_x)$ where the left (resp.\ right) force point is located at $i \pi$ (resp.\ $(\eta(S_x))^+$). Let~$\wt{T}$ be a stopping time for $\wt{\eta}$ and $\wt{T}'$ be a reverse stopping time for $\wt{\eta}$ given $\wt{\eta}|_{[0,\wt{T}]}$ and assume that we are working on the positive probability event that $\wt{\eta}|_{[0,\wt{T}]}$ and $\wt{\eta}|_{[\wt{T}',\infty)}$ are disjoint. Let $A = \eta([0,\sigma]) \cup \wt{\eta}$ and let $\wt{h}$ be $\text{GFF}$ on $\strip \setminus A$ with the same boundary conditions as $h$ on $\partial{(\strip \setminus A)} \setminus \wt{\eta}$ and with boundary conditions along $\wt{\eta}$ as if $\wt{\eta}$ were the flow line of $\wt{h}$ starting from~$\eta(\sigma)$ and targeted at~$\eta(T_x)$. Let $\wt{\eta}_{L}$ (resp.\ $\wt{\eta}_{R}$) be the flow line of $\wt{h}$ starting from $\wt{\eta}(\wt{T}')$ with angle $\pi$ (resp.\ $-\pi$) and targeted at $i\pi$ (resp. the point corresponding to where $\eta$ has most recently before time $\sigma$ hit its force point). Then by construction we have that the conditional law of $\wt{\eta}_{L}$ and $\wt{\eta}_{R}$ given $A$ is the same as the conditional law of $\eta_{L}$ and $\eta_{R}$ given $\eta|_{[0,T_{u}]}$ in the analogous set up for $\eta$. Moreover, the proof of \cite[Proposition~5.12]{ms2016imag2} implies that the conditional law of $\wt{\eta}|_{[\wt{T},\wt{T}']}$ given $\wt{\eta}_{L}, \wt{\eta}_{R}, \eta|_{[0,\sigma]}, \wt{\eta}|_{[0,\wt{T}]}$ and $\wt{\eta}|_{[\wt{T}',\infty)}$ is that of an $\SLE_{\kappa}(\kappa/2 - 2; \kappa/2 - 2)$ process in the connected component of $\strip \setminus (\eta([0,\sigma]) \cup \wt{\eta}_{L} \cup \wt{\eta}_{R} \cup \wt{\eta}([0,\wt{T}]))$ with $\wt{\eta}(\wt{T})$ on its boundary from $\wt{\eta}(\wt{T})$ to $\wt{\eta}(\wt{T}')$. Since the stopping time $\sigma$ was arbitrary, the proof will be complete if we manage to prove that this resampling property determines the law of the triple $(\eta, \eta_{L}, \eta_{R})$ conditional on $\eta|_{[0,T]}$ and $\eta|_{[T',T_x]}$ where $T,T'$ are the corresponding stopping times for $\eta$. To see this, let $T_{1}, T_{2}$ be two stopping times for $\eta_{L},\eta_{R}$ respectively. Then the law of the triple $(\eta_{1}, \eta_{L}$, $\eta_{R})$ with $\eta_{1}$ = $\eta|_{[T,T']}$ conditional on $\eta|_{[0,T]}$, $\eta|_{[T',T_x]}$ , $\eta_{L}|_{[0,T_{1}]}$ and $\eta_{R}|_{[0,T_{2}]}$ is the same as that of ordinary flow lines of a $\text{GFF}$ conditioned on the positive probability event that $\eta_{L}$ and $\eta_{R}$ do not intersect since this conditioned law satisfies the same resampling properties. In particular, the marginal law of $(\eta_{L},\eta_{R})$ satisfies the hypothesis of \cite[Proposition 5.10]{ms2016imag2} with $\theta_{2} = -\pi$ and $\theta_{1} = \pi$, and so its marginal law is uniquely determined. Since we know the conditional law of $\eta_{1}$ given $(\eta_{L},\eta_{R})$, the law of the triple $(\eta_{1}, \eta_{L}, \eta_{R})$ is uniquely determined and this completes the proof. \end{proof} \section{Poissonian structure} \label{sec:main_theorems_proof} The purpose of this section is to prove Theorems~\ref{thm:quantum_natural_time_cutting} and~\ref{thm:boundary_length_evolution} as well as Corollary~\ref{cor:dim_of_boundary_intersection}. Fix $\kappa \in (0,4)$ and $\rho \in (\kappa/2-4,-2) \cap (-2-\kappa/2,-2)$, and set $\gamma = \sqrt{\kappa} \in (0,2)$. The main step in establishing all of these results is to describe the Poissonian structure of the quantum surfaces which are cut out of a quantum wedge of weight $\rho+4$ by an independent $\SLE_{\kappa}(\rho)$ process. Our strategy will be similar to the one followed in \cite[Section~6]{dms2014mating} for the $\rho \in (-2,\kappa/2-2)$ case. However, as discussed in Section~\ref{subsec:reverse_coupling}, there will be some key differences in the proofs since $\SLE_{\kappa}(\rho)$ processes are not simple for the range of $\rho$ that we consider. As in \cite{dms2014mating}, associated with the quantum surfaces which are cut out by the $\SLE_{\kappa}(\rho)$ process is a local time which comes from the encoding of such surfaces using Bessel processes (see \cite[Section~4]{dms2014mating}). By \cite[Proposition~19.12]{dms2014mating}, we have that this local time should be considered as counting the number of small bubbles which have been cut out by the $\SLE_{\kappa}(\rho)$ process. More precisely, it is the limit as $\epsilon \to 0$ of the number of excised bubbles of quantum mass in $[\epsilon,2\epsilon]$, times an appropriate power of $\epsilon$. If we consider the right-continuous inverse of this local time, then we obtain a different time-parameterization for the $\SLE_{\kappa}(\rho)$ process which is intrinsic to the bubbles that it cuts out viewed as quantum surfaces. Following the terminology of \cite{dms2014mating}, we refer to this notion of time as ``quantum natural time''. We will show that when the $\SLE_{\kappa}(\rho)$ process is drawn on top of a certain independent quantum wedge and it is parameterized by quantum natural time, then the law of the surfaces is invariant under zipping according to a fixed amount of quantum natural time, and hence completing the proof of Theorem~\ref{thm:quantum_natural_time_cutting}. Moreover, by combining the previous results with standard properties of stable processes, we will identify the law of the process associated with the evolution of the changes in boundary lengths, and hence complete the proof of Theorem~\ref{thm:boundary_length_evolution}. Finally, we will give the proof of Corollary~\ref{cor:dim_of_boundary_intersection} as a consequence of Theorem~\ref{thm:boundary_length_evolution}. As before, we let $\strip = \R \times (0,\pi)$ be the the strip. We also let $\strip_+ = \R_+ \times (0,\pi)$ be the positive part of the strip and $\strip_- = \R_- \times (0,\pi)$ be the negative part of the strip. \subsection{Strategy} \label{subsec:strategy} We will give an outline of the strategy used to prove Theorem~\ref{thm:quantum_natural_time_cutting}, as it will take quite a bit of work to complete. Let $h$ be a free boundary GFF on~$\h$ with a log singularity at $0$ and which is coupled with a reverse $\SLE_{\kappa}(\wt{\rho})$ process with $\wt{\rho} = \rho + 4$ as in Theorem~\ref{thm:reverse_coupling}. More precisely, we have that $h = \wh{h} - \frac{\rho + 2}{\gamma}\log |\cdot|$, where $\wh{h}$ is a free boundary $\text{GFF}$ on $\h$. Next, we let $\eta$ be an $\SLE_{\kappa}(\rho)$ process in $\h$ from $0$ to $\infty$ with a single boundary force point at $0_+$ and which is independent of $h$. We will fix the additive constant for $h$ so that the average of $h \circ f_{T_u}^{-1} + Q \log |(f_{T_u}^{-1})'|$ on $\h \cap \partial \D$ is equal to $0$, where $T_u$ is the right-continuous inverse of the local time of $V-W$ at $0$ with $(V,W)$ being the driving pair for~$\eta$, and $(f_t)$ the centered forward Loewner evolution. We will also take $u>0$ to be large but fixed. \begin{enumerate} \item[Step 1.] We will describe the Poissonian structure of the quantum surfaces which are cut out by $\eta|_{[0,T_u]}$ from $\infty$ ordered reverse chronologically in comparison to how they are first visited by $\eta|_{[0,T_u]}$. We will in particular show they have the same structure as that of the bubbles arising from a weight $\rho + 2$ quantum wedge. This step will be carried out in Section~\ref{subsec:lightcone_on_free_boundary_gff} just below (see Theorem~\ref{thm:wedge_cutting}). \item[Step 2.] The weight $\rho+2$ quantum wedge structure of these bubbles leads to a quantum measure which is supported on the union of $\eta([0,T_u]) \cap \partial \h$ and the set of self-intersection points of $\eta$. This measure corresponds to the local time of the Bessel process encoding the weight $\rho+2$ wedge. Using this, we will see that the bubbles cut out by $\eta|_{[0,T_u]}$ (at least near the typical point) ordered according to when they are first visited by $\eta$ have the structure of a weight $\rho+2$ wedge. This will be described in Section~\ref{subsec:natural_time} below. \item[Step 3.] Combining Steps 1 and 2, we will complete the proof of the first part of Theorem~\ref{thm:quantum_natural_time_cutting} by identifying the law of the bubbles which are cut out by $\eta$ when drawn on top of an independent weight $\rho + 4$ quantum wedge. The weight $\rho + 4$ appears because of the combination of the initial singularity at the origin given by $-(2+\rho)/\gamma \log |\cdot|$ and an additional $-\gamma (2-d) \log |\cdot-w|$ singularity with $d = 1 + (2/\gamma^2)(\rho + 2)$ which arises from observing the field at a quantum typical point $w$ (using the measure from Step 2). This will be described in Section~\ref{subsec:natural_time}. \item[Step 4.] As for the second part of Theorem~\ref{thm:quantum_natural_time_cutting}, it will follow from the fact that if we shift the bubbles near the typical point when viewed as quantum surfaces by a fixed amount of local time, then their Poissonian structure remains the same. This will be described in Section~\ref{subsec:natural_time}. \end{enumerate} Once we have proved Theorem~\ref{thm:quantum_natural_time_cutting}, we will use the scaling properties to determine the boundary length evolution and complete the proof of Theorem~\ref{thm:boundary_length_evolution} in Section~\ref{subsec:boundary_length_evolution}. We will also give the proof of Corollary~\ref{cor:dim_of_boundary_intersection} here. \subsection{Poissonian structure of light cone path on a free boundary GFF} \label{subsec:lightcone_on_free_boundary_gff} We will now carry out Step 1 as described in Section~\ref{subsec:strategy}. That is, we will determine the law of the quantum surfaces which are cut out by an $\SLE_{\kappa}(\rho)$ process for $\kappa \in (0,4)$ and $\rho \in (\kappa/2 - 4, - 2) \cap (- 2 - \kappa/2, - 2)$ drawn on top of a free boundary $\text{GFF}$ with an appropriate log singularity and with the additive constant fixed in a certain way. \subsubsection{Statement} \begin{theorem} \label{thm:wedge_cutting} Fix $\kappa \in (0,4)$, $\rho \in (\kappa/2 - 4 , - 2) \cap ( - 2 - \kappa/2 , - 2)$, $\wt{\rho} = \rho + 4$ and let $\gamma = \sqrt{\kappa}$. Let $\wh{h}$ be a free boundary $\text{GFF}$ on $\h$ and let \begin{align*} h = \wh{h} + \frac{(2 - \wt{\rho})}{\gamma}\log|\cdot | = \wh{h} - \frac{2 + \rho}{\gamma}\log| \cdot| \end{align*} Let $\eta$ be an $\SLE_{\kappa}(\rho)$ process starting from $0$ with a single boundary force point of weight $\rho$ located at $0_+$ and assume that $\eta$ is independent of $h$. Let $(f_{t})$ be the centered Loewner flow for $\eta$, $(W , V)$ be its driving process, $L_{t}$ be the local time of $V - W$ at $0$, and let $T_{u} = \inf \{s >0 : L_{s} > u\}$ be the right-continuous inverse of $L_{t}$. Fix $u > 0$ large and assume that the additive constant for $h$ is fixed such that the average of $h \circ f_{T_{u}}^{-1} + Q\log|(f_{T{u}}^{-1})'|$ on $\h \cap \partial \D$ is equal to $0$. Then the law of the quantum surfaces parameterized by the bounded components of $\h \setminus \eta([0,T_{u}])$ ordered in reverse from when they are first visited by $\eta$, is equal to those of a weight $\rho + 2$ wedge up to a time $s_{u}$ which tends to $\infty$ in probability as $u \to \infty$. \end{theorem} Note that in our setup there are some subtleties in the proof of Theorem~\ref{thm:wedge_cutting} since the force point of an $\SLE_{\kappa}(\rho)$ process for the range of the values of $\rho$ that we consider might not lie on the boundary of the domain. However the main idea of the proof of \cite[Theorem~6.1]{dms2014mating} still works in our case. \subsubsection{Setup} \label{sec:setup} First, we recall from~\eqref{eqn:bessel_dimension_wrt_weight} that the dimension of the Bessel process $Y$ associated with a wedge of weight $\rho + 2$ is given by: \begin{align} \label{eq:bessel_dimension} \delta = \delta (\kappa , \rho) = 1 + \frac{2\rho + 4}{\kappa} = 1 + \frac{2\rho + 4}{\gamma^{2}} \end{align} Note that the value of $\delta(\kappa , \rho)$ varies in $(0,1)$ as $\rho$ varies in $(-\kappa/2 -2 , -2)$. Let $(g_{t})$ be the family of conformal maps solving the Loewner equation driven by $W$ and for all $t \geq 0$ let $P_{t}$ be the point of last intersection of $\eta$ either with itself or $\partial \h$ by time $t$. Then we know that $X =\kappa^{-\frac{1}{2}}(V - W)$ has the law of a Bessel process starting from zero of dimension $\delta(\kappa,\rho)$ where $V_{t} = g_{t}(P_{t}^+)$. Let $(T_{u})$ be the family of the right-continuous inverses for the local time of $X$ at $0$. We also consider $\wt{\eta}^0$ to be an $\SLE_{\kappa}(\rho)$ in $\h$ from $0$ to $\infty$ which is independent of $W$ and $h$ a free boundary $\text{GFF}$ on $\h$ independent of $(W , \wt{\eta}^0)$. Fix $u > 0$ and note that $T_{u} \to \infty$ a.s.\ as $u \to \infty$. We consider the process $\wt{X} = (\wt{X}_{t})$ with $\wt{X}_{t} = X_{T_{u} -t}$ for $0 \leq t \leq T_{u}$ and $\wt{X}_{t} = X_{t}$ for $t \geq T_{u}$, and with corresponding family of local times $(\wt{L}_{t}^x)$. We also consider the process $\wt{V} = (\wt{V}_{t})$ with $\wt{V}_{t} = V_{T_{u} - t} - V_{T_{u}}$ for $0 \leq t \leq T_{u}$ and $\wt{V}_{t} = - V_{t}$ for $t \geq T_{u}$. Note that the time-reversal symmetries of Bessel processes (see \cite[Proposition~3.1]{dms2014mating}) imply that $X$ and $\wt{X}$ have the same law. It is also easy to see that \[ \wt{V}_{t} = -\frac{2}{\sqrt{\kappa}}\text{P.V.} \int_{0}^{t}\frac{1}{\wt{X}_{s}}ds.\] Thus $\wt{W}$ has the law of a reverse Loewner driving function, where $\wt{W}_t = W_{T_u - t} - W_{T_u}$ for $0 \leq t \leq T_u$ and $\wt{W}_t = -V_t - \sqrt{\kappa}X_t$ for $ t \geq T_u$. Let $(\wt{g}_t)$ be the family of conformal maps solving the reverse Loewner equation driven by $\wt{W}$ with $\wt{g}_0(z) = z$ and $\wt{f}_t(z) = \wt{g}_t(z)-\wt{W}_t$. Let also $\wt{\eta}^{T_{u}}$ be the random curve such that $\wt{\eta}^{T_{u}}|_{[0,T_{u}]}$ has driving function $W^{T_{u}}$ given by $t \mapsto \wt{W}_{T_{u} - t} - \wt{W}_{T_{u}}$ and $\wt{\eta}^{T_{u}}(s) = \wt{f}_{T_u}(\wt{\eta}^0(s - T_{u}))$ for $s \geq T_{u}$. For $0 \leq s \leq t$ we set $\wt{f}_{s,t} = \wt{f}_{t} \circ (\wt{f}_{s})^{-1}$ and for $0 \leq t \leq T_{u}$ we consider the continuous curve $\wt{\eta}^t$ defined by $\wt{\eta}^t(s) = (\wt{f}_{t,T_{u}})^{-1}(\wt{\eta}^{T_{u}}(T_{u} - t + s))$ for $0 \leq s \leq t$ and $\wt{\eta}^t(s) = \wt{f}_{t}(\wt{\eta}^0(s - t))$ for $s \geq t$. It is easy to check that $W_{s}^t = \wt{W}_{t - s} - \wt{W}_{t}$ and $g_{s}^t(z) = \wt{g}_{t - s}((\wt{f}_{t})^{-1}(z)) - \wt{W}_{t}$ where $W^t$ is the driving function of $\wt{\eta}^t|_{[0,t]}$ and $(g_{s}^t)$ the conformal maps solving the Loewner equation driven by $W^t$. For all $0 \leq s \leq t \leq T_{u}$, let $P_{s}^t$ be the last point of intersection of $\wt{\eta}^t$ before time $s$ either with itself or with $\partial \h$, and set $V_{s}^t = g_{s}^t(P_{s}^t)$. It is again easy to see that $V_{s}^t = V_{T_{u} - t + s}^{T_{u}} - W_{T_{u} - t}^{T_{u}}$ by applying certain uniqueness arguments for the Loewner equation. Also, the above imply that $\wt{U}_{t} = f_{T_{u} - t}^{T_{u}}(\wt{\eta}^{T_{u}}(s))$ where $s = \sup\{0 \leq r \leq T_{u} - t : \wt{\eta}^{T_{u}}(r) \in \partial \h \cup \wt{\eta}^{T_{u}}([0,r))\}$ and $\wt{U}_{t} = \sqrt{\kappa}\wt{X}_{t}$. By construction, $\wt{\eta}^{T_{u}}$ has the law of an $\SLE_{\kappa}(\rho)$ and so if $P$ is a pocket drawn by $\wt{\eta}^{T_{u}}$ and $S_{1}(P)$ is the left side of $P$ with zero angle boundary conditions, then every point on the right side of $S_{1}(P)$ is hit exactly once. Moreover, since the set of times during which $\wt{\eta}^{T_{u}}$ draws $S_{1}(Q)$ for some pocket $Q$, is dense in $\R_+$, we obtain that $\wt{\eta}^{T_{u}}((T_{u} - t,\infty))$ does not intersect the right side of $\wt{\eta}^{T_{u}}([s,T_{u} - t])$. This shows that $\wt{U}_{t}$ lies on the boundary of the connected component of $\h \setminus f_{T_{u} - t}^{T_{u}}(\wt{\eta}^{T_{u}}([T_{u} - t,\infty)))$ whose boundary contains $0_+$. Also, $\wt{\eta}^t([0,t]) = f_{T_{u} - t}^{T_{u}}(\wt{\eta}^{T_{u}}([T_{u} - t,T_{u}]))$ and $\wt{\eta}^t([t,\infty)) = \wt{f}_{t}(\wt{\eta}^0([0,\infty))) = f_{T_{u} - t}^{T_{u}}(\wt{\eta}^{T_{u}}([T_{u},\infty)))$ and hence $\wt{\eta}^t([0,\infty)) = f_{T_{u} - t}^{T_{u}}(\Tilde{\eta}^{T_{u}}([T_{u}-t,\infty)))$. Therefore $\wt{U}_{t}$ lies on the connected component of $\h \setminus \wt{\eta}^t([0,\infty))$ whose boundary contains the origin. We also observe that $\wt{\eta}^t([t,t + r]) = \wt{f}_{t}(\wt{\eta}^0([0,r]))$ for all $0 \leq t \leq T_{u}$ and for all $r > 0$, and that the hull of $\wt{\eta}^t([0,t])$ is equal to the hull of $\wt{f}_{t}(f_{T_{u}}^{T_{u}}(\wt{\eta}^{T_{u}}([0,T_{u}])))$. But $f_{T_{u}}^{T_{u}}(\wt{\eta}^{T_{u}}([0,T_{u}])) \subseteq \partial \h$ and so the hull of $\wt{\eta}^t([0,t + r])$ is equal to the hull of $\wt{f}_{t}(\partial \h \cup \wt{\eta}^0([0,r]))$. Now set \begin{align*} \wt{h}^{T_{u}} = h + \frac{2}{\gamma}\log|\cdot| - \frac{\wt{\rho}}{\gamma}\log|\cdot- \wt{U}_{T_u}| = h + \frac{2}{\gamma}\log|\cdot| - \frac{\wt{\rho}}{\gamma}\log|\cdot| \end{align*} By Theorem~\ref{thm:reverse_coupling}, we obtain that the random distributions \begin{align}\label{eq:law_equivalence} &h + \frac{2 - \wt{\rho}}{\gamma}\log|\cdot|, \nonumber \\ & h \circ \wt{f}_{T_{u}} + \frac{2-\wt{\rho}}{\gamma}\log|\wt{f}_{T_{u}}| +Q\log|(\wt{f}_{T_{u}})'|, \quad\text{and} \nonumber \\ & h \circ \wt{f}_{t} + \frac{2}{\gamma}\log|\wt{f}_{t}| - \frac{\wt{\rho}}{\gamma}\log|\wt{f}_{t} - \wt{U}_{t}| +Q\log|(\wt{f}_{t})'| \end{align} have the same law for all $0 \leq t \leq T$. Set \begin{align*} \wt{h}^t = \wt{h}^{T_{u}} \circ \wt{f}_{t,T_{u}} + Q\log|(\wt{f}_{t,T_{u}})'| \end{align*} Then, the equality in law in~\eqref{eq:law_equivalence} implies that $\wt{h}^t$ and $h + \frac{2}{\gamma}\log|z| - \frac{\wt{\rho}}{\gamma}\log|z - \wt{U}_{t}|$ have the same law for all $0 \leq t \leq T_u$. The above observations show that for every fixed $u \in (0,\infty)$, we can find a random family $(\wt{h}^t,\wt{\eta}^t)_{0\leq t \leq T_u}$, where $\wt{\eta}^t$ is a random continuous curve in $\h$ from $0$ to $\infty$ and $\wt{h}^t$ is a random distribution on $\h$ such that the following hold; \begin{enumerate}[(i)] \item $(\wt{\eta}^t)_{0 \leq t \leq T_u}$ is independent of $\wt{h}^0$ and $\wt{\eta}^0$ has the law of an $\SLE_{\kappa}(\rho)$. \item $(W_{T_u - t}^{T_u} - W_{T_u}^{T_u})_{0 \leq t \leq T_u}$ has the law of a reverse Loewner driving flow restricted to $[0,T_u]$ with dimension $\delta(\kappa,\rho)$ and $(\wt{U}_{t}^t)_{0 \leq t \leq T_u}$ has the law of a Bessel process with dimension $\delta(\kappa,\rho)$ and multiplied by $\sqrt{\kappa}$, restricted to $[0,T_u]$, where $\wt{U}_{t}^{T_u} = V_{T_u - t}^{T_u} - W_{T_u - t}^{T_u} + V_{T_u}^{T_u} - W_{T_u}^{T_u}$. Here, $W^{T_u} |_{[0,T_u]}$ is the Loewner driving function of $\wt{\eta}^{T_u} |_{[0,T_u]}$. \item If $(\wt{f}_{t}^{T_u})_{0 \leq t \leq T_u}$ is the reverse centered Loewner flow associated with $(W_{T_u - t}^{T_u} - W_{T_u}^{T_u})_{0 \leq t \leq T_u}$, then for all $0 \leq t \leq T_u$ and $r >0 $, the hull of $\Tilde{\eta}^{t}([0,t + r])$ is equal to the hull of $\wt{f}_{t}(\partial \h \cup \wt{\eta}^0([0,r]))$. \item $\wt{h}^s = \wt{h}^t \circ \wt{f}_{s,t}^{T_u} + Q\log|(\wt{f}_{s,t}^{T_u})'|$ and $\wt{h}^t$ can be expressed as the sum of a free boundary $\text{GFF}$ on $\h$ plus $\frac{2}{\gamma}\log|z| - \frac{\wt{\rho}}{\gamma}\log|z - \wt{U}_{t}| $ for all $0 \leq s \leq t \leq T_u$. \item $\wt{U}_{t}^{T_u}$ lies on the boundary of the component of $\h \setminus \wt{\eta}^t([0,\infty))$ whose boundary contains 0 for all $t \in [0,T_u]$. \end{enumerate} The following lemma gives us a way to identify the laws of the fields which arise when performing the zipping and the unzipping operations. It is the analogue of \cite[Lemma~6.2]{dms2014mating}. \begin{lemma}[$\text{\cite[Lemma~6.2]{dms2014mating}}$] \label{lem:extension_lemma} There exists a random family $(\wt{h}^t,\wt{\eta}^t)_{t \geq 0}$ where $\wt{h}^t$ is a distribution on $\h$ and $\wt{\eta}^t$ is a continuous curve on $\overline{\h}$ from $0$ to $\infty$ such that the following hold: \begin{enumerate}[(i)] \item $(\wt{\eta}^t)_{t \geq 0}$ is independent of $\wt{h}^0$ and $\wt{\eta}^0$ has the law of an $\SLE_{\kappa}(\rho)$. \item $\wt{W} = (\wt{W}_{t})_{t \geq 0}$ with $\wt{W}_{t} = -W_{t}^t$ has the law of a reverse Loewner driving function with dimension $\delta$ and let $(\wt{f}_{t})_{t \geq 0}$ be the corresponding centered reverse Loewner flow. Also, $\wt{U} = (\wt{U}_{t})_{t \geq 0}$ with $\wt{U}_{t} = \wt{U}_{t}^t$ has the law of a Bessel process with dimension $\delta$ multiplied by $\sqrt{\kappa}$. \item The hull of $\wt{\eta}^t([0,t + r])$ is equal to the hull of $\wt{f}_{t}(\partial \h \cup \wt{\eta}^0([0,r]))$ for all $t \geq 0$ and $r > 0$. \item $\wt{U}_{t}$ lies on the boundary of the component of $\h \setminus \wt{\eta}^t([0,\infty))$ whose boundary contains the origin. \item $\wt{h}^s = \wt{h}^t \circ \wt{f}_{s,t} + Q\log|(\wt{f}_{s,t})'|$ and $\wt{h}^t$ can be expressed as the sum of a free boundary $\text{GFF }$ on $\h$ plus $\frac{2}{\gamma}\log|z| - \frac{\wt{\rho}}{\gamma}\log|z - \wt{U}_{t}|$ for all $0 \leq s \leq t$. \item $\wt{W} = (\wt{W}_{t})_{t \geq 0}$ is the reverse Loewner driving function corresponding to $\wt{U}$. \end{enumerate} \end{lemma} \begin{proof} Noting that Theorem~\ref{thm:reverse_coupling} is an analogue of \cite[Theorem~5.1]{dms2014mating}, the proof follows from the same argument used to prove \cite[Lemma~6.2]{dms2014mating}. \end{proof} We will assume throughout that $(\wt{h}^t)$ and $(\wt{\eta}^t)$ are as in the statement of Lemma~\ref{lem:extension_lemma}. We fix the additive constant for $\wt{h}^0$, hence $\wt{h}^t$ for all $t \geq 0$, by taking its average on $\h \cap \partial \D$ to be equal to $0$. When shifting the above collection of pairs of fields and curves by a fixed amount of local time, it will be important to identify the law of the resulting pair as seen by the description of our strategy for the proof of Theorem~\ref{thm:quantum_natural_time_cutting}. It turns out that the laws of the pairs are invariant when zipping by a fixed amount of local time. However, we note that for general $t \geq 0$, the curve $\wt{\eta}^t$ does not necessarily have the law of a forward $\SLE_{\kappa}(\rho)$ process. \begin{lemma}[$\text{\cite[Lemma~6.3]{dms2014mating}}$] \label{lem:equality_in_law} Suppose that we have the setup as described above. Let $\wt{L}_{t}$ denote the local time of $\wt{U}_{t}$ at $0$ and, for each $u > 0$, we let $\wt{T}_{u}$ be the right-continuous inverse of $\wt{L}_{t}$. For each $u > 0$, we have that $\wt{\eta}^{\wt{T}_{u}}$ has the law of a forward $\SLE_{\kappa}(\rho)$ process. In particular, the laws of the fields $(\wt{h}^{\wt{T}_{u}},\wt{\eta}^{\wt{T}_{u}})$ and $ (\wt{h}^0 , \wt{\eta}^0)$ are the same, where $\wt{h}^{\wt{T}_{u}}$ is viewed as a distribution modulo additive constant. \end{lemma} \begin{proof} It follows from the proof of \cite[Lemma~6.3]{dms2014mating}. \end{proof} We are now going to define the objects which are used in the proof of Theorem~\ref{thm:wedge_cutting}. We will follow the same terminology as in \cite[Section~6.2]{dms2014mating}. For each $t \geq 0$ such that $\wt{U}_{t} \neq 0$, we let $\wt{B}_{t}$ denote the component of $\h \setminus \wt{\eta}^t([0,\infty))$ with $\wt{U}_{t}$ on its boundary. If $\wt{U}_{t} = 0$, we take $\wt{B}_{t} = \emptyset$. $\wt{B}_{t}$ is a Jordan domain as its boundary consists of part of the curve $\wt{\eta}^t$ and an interval in $\partial \h$. For $t\geq 0$ such that $\wt{B}_{t} \neq \emptyset$, we let \begin{enumerate}[(i)] \item $\wt{\zeta}_{t}$ be the closing point of $\partial{\wt{B}_{t}}$ (the opening point is $0$). \item $\wt{\varphi}_{t}: \wt{B}_{t}\rightarrow \strip$ be the unique conformal transformation which takes $\wt{U}_{t}$ to $+\infty$, $0$ to $0$, and $\wt{\zeta}_{t}$ to $-\infty$. \item $\wh{h}^t = \wt{h}^t \circ \wt{\varphi}_{t}^{-1} + Q\log|(\wt{\varphi}_{t}^{-1})'|$. \item $\wh{X}^t$ be the projection of $\wh{h}^t$ onto $\CH_{1}(\strip)$ \end{enumerate} We note that in the setup of Lemma~\ref{lem:extension_lemma}, the transformation from $(\wt{h}^s,\wt{\eta}^s)$ to $(\wt{h}^t,\wt{\eta}^t)$ for $t>s$ corresponds to zipping up for $t-s$ units of capacity time and the transformation from $(\wt{h}^s,\wt{\eta}^s)$ to $(\wt{h}^t,\wt{\eta}^t)$ for $t<s$ corresponds to unzipping for $s-t$ units of capacity time. The reason that we consider the quantum surfaces parameterized by the domains $\wt{B}_t$ is the following. When we perform the forward Loewner flow at times when the $\SLE_{\kappa}(\rho)$ process completes an entire excursion either away from $\partial \h$ or away from itself, we have that a region with finite and positive quantum area parameterized by some $\wt{B}_t$ is cut out from $\infty$. If we apply the reverse Loewner flow, then a region with positive quantum area parameterized by some $\wt{B}_t$ is added all at once. Moreover, we have that every quantum surface parameterized by a bubble lying to the right of the $\SLE_{\kappa}(\rho)$ process $\eta$ will correspond to a newly zipped in quantum surface parameterized by some $\wt{B}_t$ (modulo coordinate change). Furthermore, the latter is determined by the values of the field in a very small region and the conditional law of the field given its values in that region is very close to the unconditioned law. The latter fact will lead to the quantum surfaces which are cut out by $\eta$ to be independent. \subsubsection{Filtration and stopping times} First, we start by constructing the filtration and stopping times that we are going to consider. The definitions and notation are similar to those in \cite[Section~6.2.3]{dms2014mating}. Suppose that we have the setup of Lemma~\ref{lem:extension_lemma}. When performing the zipping and the unzipping operations, we need to keep track of the path and the field as well. However, as we mentioned at the end of Section~\ref{sec:setup}, we will only need to know the values of the field outside of the region that is currently zipped in. Also, it will be convenient to use $\strip$ as the underlying domain. We also note that the dimension of the Bessel process encoding the bubbles which are zipped in is determined by the form of the projection of the field which describes such a bubble onto $\mathcal{H}_1(\strip)$ with fixed additive constant and it does not depend on its projection onto $\mathcal{H}_2(\strip)$. The above observations motivate to consider the following filtration. Consider the process $\wt{V} = (\wt{V}_{s})_{s \geq 0}$ with $\wt{V}_{s} = \wt{U}_{s} + \wt{W}_{s}$ and for $t \geq 0$ we let $\wt{\CF_{t}}$ be the $\sigma$-algebra with respect to which the following are measurable: \begin{enumerate}[(i)] \item Both $\eta = \wt{\eta}^0$ and the driving pair $(\wt{W}_{s},\wt{V}_{s})$ of the reverse Loewner flow for $s \leq t$. \item The field $\wt{h}^t$ restricted to $\h \setminus \wt{B}_{t}$ (i.e., $\wt{h}^t$ restricted to any component of $\h \setminus \wt{\eta}^t$ other than the one currently been generated). \item The restriction of $\wh{h}^t$ to $\strip_-$. \end{enumerate} Note that $\wt{\eta}^t([0,t]) = f_{T_u - t}^{T_u}(\wt{\eta}^{T_u}([T_u - t , T_u]))$ for $t < T_u$ and so $\wt{\eta}^t([0,t])$ is determined by $W_{s+T_u -t}^{T_u}-W_{T_u - t}^{T_u}, s \in [0,t]$. But $\wt{\eta}^{T_u}$ hits either itself or $\R_+$ at time $T_u$ and so $\wt{B}_t$ is a connected component of $\h \setminus \wt{\eta}^t([0,t])$. Also, $W_{s+T_u - t}^{T_u} - W_{T_u}^{T_u} = \wt{W}_{t-s}-\wt{W}_t$ for all $s \in [0,t]$ and so $\wt{B}_t$ is determined by $(\wt{W}_s,\wt{V}_s), 0 \leq s \leq t$. It follows that $\wt{\zeta}_t,\wt{\varphi}_t$, and $\wh{X}^t|_{\R_-}$ are determined by $\wt{\CF}_t$. Moreover, by arguing as in \cite[Section~6.2.3]{dms2014mating}, we obtain that $(\wt{\CF}_{t})$ is a filtration. Next we fix $t_{1},t_{2} \in (0,\infty)$ such that $\wt{U}_{t_{1}} = \wt{U}_{t_{2}} = 0$ and $\wt{U}_{r} > 0 $ for all $r \in (t_{1},t_{2})$. Fix $t_{0} \in (t_{1},t_{2})$. Then the quantum length of $\partial{\wt{B}_{t}}$ with respect to $\wt{h}^t$ remains the same for all $t \in (t_{1},t_{2})$, since $\wt{h}^t|_{\wt{B}_{t}}$ and $\wt{h}^s|_{\wt{B}_{s}}$ remain equivalent as surfaces for all $t,s \in (t_{1},t_{2}) $. Also the same holds for the quantum length of $[\wt{U}_{t},\wt{\zeta}_{t}]$ with respect to $\wt{h}^t$ for $t \in (t_{1},t_{2})$. This shows that the quantum length of $\partial{\wt{B}_{t}} \setminus \partial{\h}$ tends to $0$ as $t \to t_{1}$ and it tends to the quantum length of the clockwise part of $\partial{\wt{B}_{t_{0}}}$ from $\wt{U}_{t_{0}}$ to $\wt{\zeta}_{t_{0}}$ as $t \to t_{2}$. Hence if $\alpha(t)$ is such that $\wh{h}^t$ comes by translating $\wh{h}^{t_{0}}$ horizontally by $\alpha(t)$ units, then $\alpha(t) \to -\infty$ as $t \to t_{1}$ and $\alpha(t) \to +\infty$ as $t \to t_{2}$, and $\alpha$ is continuous in $t$. Therefore if $\sup_{u \in \R}(\wh{X}^{t_{0}}_{u}) \geq r$, then there exists $t \in (t_{1},t_{2})$ such that $\inf\{u \in \R : \wh{X}^{t}_{u} = r\} =0$. Now we are ready to define the stopping times that we are going to consider. We note that our goal is to analyze the structure of the bubbles which are zipped in when viewed as quantum surfaces modulo coordinate change. It will be convenient for the proof of Theorem~\ref{thm:quantum_natural_time_cutting} to analyze the structure of a given bubble which has only been partially zipped in. Also, we will first analyze the collection of ``large bubbles'' in the sense of quantum area and then obtain an asymptotic coupling of these bubbles with the bubbles cut out from the $\SLE_{\kappa}(\rho)$ process as their quantum areas tend to zero. Following again \cite[Section~6.2.3]{dms2014mating}, we define the stopping times as follows. Fix $\epsilon > 0, \overline{\epsilon} > 0$ and $r \in \R$. We define stopping times inductively as follows. We let $\wt{\tau_{1}}^{\epsilon,\overline{\epsilon},r}$ be the first time $t \geq 0$ such that the following hold simultaneously: \begin{enumerate}[(i)] \item The quantum length of $\partial{\wt{B}_{t}} \setminus \partial{\h}$ is at least $\epsilon$ with respect to the $\gamma$-$\text{LQG}$ length measure induced by $\wt{h}^t$, \item $\inf\{u \in \R : \wh{X}^{t}_{u} = r\} =0$, and \item The $\gamma$-$\text{LQG}$ mass of $\strip_-$ associated with the field $\wh{h}^t$ is at least $\overline{\epsilon}$. \end{enumerate} We note that the time $\wt{\tau}_{1}^{\epsilon,\overline{\epsilon},r}$ is a.s.\ finite since our previous observations imply that if $\sup_{u \in \R}\wh{X}^{t}_{u}$ is at least $r$, there will be a time $s$ such that $\wh{X}^s$ first hits $r$ at $u = 0$. We then let $\wt{\sigma}_{1}^{\epsilon,\overline{\epsilon},r}$ be the first time $t$ after time $\wt{\tau}_{1}^{\epsilon,\overline{\epsilon},r}$ that $\wt{U}_{t} = 0$. Given that $\wt{\tau}_{j}^{\epsilon,\overline{\epsilon},r},\wt{\sigma}_{j}^{\epsilon,\overline{\epsilon},r}$ for $1 \leq j \leq k$ have been defined for some $k \in \N$, we let $\wt{\tau}_{k + 1}^{\epsilon,\overline{\epsilon},r}$ be defined in exactly the same way as $\wt{\tau}_{1}^{\epsilon,\overline{\epsilon},r}$ except with $t \geq \wt{\sigma}_{k}^{\epsilon,\overline{\epsilon},r}$. We then let $\wt{\sigma}_{k}^{\epsilon,\overline{\epsilon},r}$ be the first time $t$ after time $\wt{\tau}_{k + 1}^{\epsilon,\overline{\epsilon},r}$ that $\wt{U}_{t} = 0$. For each $j$, we let $\wh{X}^{j,\epsilon,\overline{\epsilon},r}$ be given by $\wh{X}^{\wt{\tau}_{j}^{\epsilon,\overline{\epsilon},r}}$ and $\wh{h}_{j}^{\epsilon,\overline{\epsilon},r}$ be given by $\wh{h}^{\wt{\tau}_{j}^{\epsilon,\overline{\epsilon},r}}$. We emphasize that the process $ \wh{X}_{u}^{j,\epsilon,\overline{\epsilon},r}$ hits $r$ for the first time at time $0$. As we mentioned earlier, the law of the sequences $(\wh{X}^{j,\epsilon,\overline{\epsilon},r})$ and $(\wh{X}^{j,\overline{\epsilon},r}) = (\wh{X}^{j,0,\overline{\epsilon},r})$ will determine the dimension of the Bessel process encoding the quantum surfaces parameterized by the bubbles which are zipped in. In particular, we have that $(\wh{X}^{j,\epsilon,\overline{\epsilon},r})$ and $(\wh{X}^{j,\overline{\epsilon},r})$ are both given by independent Brownian motions with a common downward drift. Following \cite[Section~4]{dms2014mating}, we have that each such Brownian motion corresponds to an excursion of the Bessel process away from $0$. We record the result about the form of the drift in the following proposition. \begin{proposition}[$\text{\cite[Proposition~6.4]{dms2014mating}}$]\label{prop:form_of_drift} Let \begin{align*} \alpha = \frac{\rho + 4}{\gamma} - Q = \frac{\rho + 2}{\gamma} - \frac{\gamma}{2}. \end{align*} Let $\wh{B}_{2t}^{j,\epsilon,\wt{\epsilon},r} = \alpha t - \wh{X}_{t}^{j,\epsilon,\wt{\epsilon},r}$ for $t \geq 0$. Given $\wt{\CF}_{\tau_{j}^{r,\epsilon,\overline{\epsilon}}}$, the process $\wh{B}^{j,\epsilon,\overline{\epsilon},r}$ is a standard Brownian motion with $\wh{B}_{0}^{j,\epsilon,\overline{\epsilon},r} = -r$. \end{proposition} \begin{proof} It follows from the proof of \cite[Proposition~6.4]{dms2014mating}. \end{proof} \subsubsection{Proof of Theorem~\ref{thm:wedge_cutting}} Proposition~\ref{prop:form_of_drift} implies that the projections of the fields $(\wh{h}_j^{\epsilon,\overline{\epsilon},r})_{j \in \N}$ onto $\mathcal{H}_1(\strip)$ and restricted to $\strip_+$ are i.i.d. and they are related to the excursions of a Bessel process from $0$. However, in order to prove Theorem~\ref{thm:wedge_cutting}, we will need to know the dependency between the projections of the fields onto $\mathcal{H}_2(\strip)$ and restricted to $\strip_+$. Note that the Markov property of the $\text{GFF}$ implies that conditionally on $\wt{\CF}_{\wt{\tau}_j^{\epsilon,\overline{\epsilon},r}}$, the field $\wh{h}_j^{\epsilon,\overline{\epsilon},r}|_{\strip_+}$ can be expressed as the sum of a $\text{GFF}$ in $\strip_+$ with Dirichlet boundary conditions on $[0,\pi i]$ and free boundary conditions on $\partial \strip_+ \setminus [0,\pi i]$. Hence, it follows that the way the field $\wh{h}_j^{\epsilon,\overline{\epsilon},r}|_{\strip_+}$ is correlated with the fields $\wh{h}_1^{\epsilon,\overline{\epsilon},r},\cdots,\wh{h}_{j-1}^{\epsilon,\overline{\epsilon},r}$ is encoded by the function which is harmonic in $\strip_+$ with boundary conditions given by the values of $\wh{h}_j^{\epsilon,\overline{\epsilon},r}$ on $[0,\pi i]$ and Neumann boundary conditions on $\partial \strip_+ \setminus [0,\pi i]$. Next we follow the notation and the strategy of \cite[Section~6.2.5]{dms2014mating}. First, we note that the quantum length of $(-\infty,0]$ with respect to $\wh{h}_{j}^{\epsilon,\overline{\epsilon},r}$ is at least $\epsilon > 0$ by the definition of $\wt{\tau}_{j}^{\epsilon,\overline{\epsilon},r}$, and so there exists a unique $\wh{\omega}_{j}^{\epsilon,\overline{\epsilon},r} \in (-\infty,0]$ such that the quantum length of $(-\infty,\wh{\omega}_{j}^{\epsilon,\overline{\epsilon},r}]$ with respect to $\wh{h}_{j}^{\epsilon,\overline{\epsilon},r}$ is equal to $\epsilon > 0$. Also, a.s.\ there exist unique $t_{j,1}^{\epsilon,\overline{\epsilon},r},t_{j,2}^{\epsilon,\overline{\epsilon},r} > 0$ such that $\wt{\tau}_{j}^{\epsilon,\overline{\epsilon},r} \in (t_{j,1}^{\epsilon,\overline{\epsilon},r},t_{j,2}^{\epsilon,\overline{\epsilon},r})$ and $\wt{U}_{t_{j,1}^{\epsilon,\overline{\epsilon},r}} = \wt{U}_{t_{j,2}^{\epsilon,\overline{\epsilon},r}} = 0$. We let $t_{j}^{\epsilon,\overline{\epsilon},r}$ be the first time $t$ in $(t_{j,1}^{\epsilon,\overline{\epsilon},r},\wt{\tau}_{j}^{\epsilon,\overline{\epsilon},r}]$ such that the quantum length of $\partial{\wt{B}_{t}} \setminus \partial{\h}$ with respect to $\wt{h}^t$ is equal to $\epsilon > 0$. Equivalently, $t_{j}^{\epsilon,\overline{\epsilon},r}$ is the first time $t$ in $(t_{j,1}^{\epsilon,\overline{\epsilon},r},\wt{\tau}_{j}^{\epsilon,\overline{\epsilon},r}]$ such that the quantum length of $(-\infty,0]$ with respect to $\wh{h}^t$ is equal to $\epsilon > 0$. Note that $\wh{h}^{t_{j}^{\epsilon,\overline{\epsilon},r}}$ can be derived by translating $\wh{h}_{j}^{\epsilon,\overline{\epsilon},r}$ horizontally by $\wh{\omega}_{j}^{\epsilon,\overline{\epsilon},r}$ units. Let $\wh{\psi}_{j}^{\epsilon,\overline{\epsilon},r}$ be the function which is harmonic in $[\wh{\omega}_{j}^{\epsilon,\overline{\epsilon},r},+\infty) \times [0,i\pi]$ with Neumann boundary conditions on the horizontal parts of the strip boundary and at $+\infty$, and Dirichlet boundary conditions on $\wh{\omega}_{j}^{\epsilon,\overline{\epsilon},r} + [0,i\pi]$ with boundary values given by those of $\wh{h}_{j}^{\epsilon,\overline{\epsilon},r}$. Then $\wh{\psi}_{j}^{\epsilon,\overline{\epsilon},r}$ determines the harmonic part of $\wh{h}_{j}^{\epsilon,\overline{\epsilon},r}$ on $[\wh{\omega}_{j}^{\epsilon,\overline{\epsilon},r},+\infty) \times [0,i\pi]$. In particular, $\wh{\psi}_{j}^{\epsilon,\overline{\epsilon},r}$ determines the harmonic part of $\wh{h}_{j}^{t_{j}^{\epsilon,\overline{\epsilon},r}}$ on $\strip_+$. By applying the Markov property to $\wh{h}_{j}^{t_{j}^{\epsilon,\overline{\epsilon},r}}$, we obtain that conditional on $\wh{\psi}_{j}^{\epsilon,\overline{\epsilon},r}$, the restriction of $\wh{h}_{j}^{t_{j}^{\epsilon,\overline{\epsilon},r}}$ to $\strip_-$ has the law of a zero boundary $\text{GFF}$ on $\strip_-$ plus the harmonic extension to $\strip_-$ of the function whose boundary values coincide with those of $\wh{h}_{j}^{t_{j}^{\epsilon,\overline{\epsilon},r}}$. Since the zero boundary part is independent of of $\wh{h}_{1}^{\epsilon,\overline{\epsilon},r},\cdots,\wh{h}_{j - 1}^{\epsilon,\overline{\epsilon},r}$ and the harmonic part is determined by $\wh{\psi}_{j}^{\epsilon,\overline{\epsilon},r}$, we obtain that conditional on $\wh{\psi}_{j}^{\epsilon,\overline{\epsilon},r}, \wh{h}_{j}^{\epsilon,\overline{\epsilon},r}$ is independent of $\wh{h}_{1}^{\epsilon,\overline{\epsilon},r},\cdots,\wh{h}_{j -1 }^{\epsilon,\overline{\epsilon},r}$. As we explained above, if we show that the functions $\wh{\psi}_j^{\epsilon,\overline{\epsilon},r}$ become trivial as $\epsilon \to 0$, then this will imply that the bubbles which are successively zipped in are independent quantum surfaces. This is the purpose of the next lemma. \begin{lemma}[$\text{\cite[Lemma~6.6]{dms2014mating}}$] Fix $\overline{\epsilon} > 0, j \in \N$ and $r \in \R$. There exists a sequence $(\epsilon_{k})$ of positive numbers decreasing to $0$ such that $\wh{\psi}_{j}^{\epsilon_{k},\overline{\epsilon},r}$ a.s.\ converges to the $0$ function on $\strip_+$ as $k \to +\infty$ with respect to the topology of local uniform convergence modulo a global additive constant. \end{lemma} \begin{proof} The proof follows essentially from the same argument used in the proof of \cite[Lemma~6.6]{dms2014mating}. Nevertheless, we are going to emphasize the parts of the proof where Lemma~\ref{lem:sle_separates_points} and Theorem~\ref{thm:law_of_sle_excursion} are used. Fix $u > 0$ large. For $x > 0$ rational, we let $\CV_{x}$ be the connected component of $\h \setminus \wt{\eta}^{\wt{T}_{u}}$ with $x$ on its boundary. Note that $\wt{\eta}^{\wt{T}_{u}}$ has the law of an $\SLE_{\kappa}(\rho)$ process and so by Lemma~\ref{lem:sle_separates_points} we obtain that a.s.\ such connected component exists for all $x \in \Q_+$. We fix $\epsilon >0$ and we assume that we are working on the event that the quantum length of $\partial{\CV}_x \setminus \partial{\h}$ with respect to $\wt{h}^{\wt{T}_{u}}$ is at least $\epsilon > 0$. Note that the probability of this event tends to 1 as $\epsilon \to 0$ for $x \in \Q_+$ fixed. We let $\xi$(resp.\ $\zeta$) be the first (resp.\ last) time that $\wt{\eta}^{\wt{T}_{u}}$ hits a point on $\partial{\CV_x}$ and let $\varphi^{\epsilon} : \CV_x \rightarrow \strip$ be a conformal transformation with $\varphi^{\epsilon}(\wt{\eta}^{\wt{T}_{u}}(\xi)) = +\infty$ and $\varphi^{\epsilon}(\wt{\eta}^{\wt{T}_{u}}(\zeta)) = -\infty$. Note that $\varphi^{\epsilon}$ is determined up to a horizontal translation, so we can determine $\varphi^{\epsilon}$ by requiring that the $\gamma$-$\text{LQG}$ boundary measure of $(-\infty,0]$ associated with $\wt{h}^{\wt{T}_{u}} \circ (\varphi^{\epsilon})^{-1} + Q\log|((\varphi^{\epsilon})^{-1})'|$ is equal to $\epsilon$. Let $\psi^{\epsilon}$ be the function which is harmonic in $\strip_+$ with Neumann boundary conditions on $\partial{\strip_+} \setminus [0,i\pi]$ and Dirichlet boundary conditions on $[0,i\pi]$ with boundary values given by the values of $\wt{h}^{\wt{T}_{u}} \circ (\varphi^{\epsilon})^{-1} + Q\log|((\varphi^{\epsilon})^{-1})'|$ on $[0,i\pi]$. Then, arguing as in the proof of \cite[Lemma~6.6]{dms2014mating}, we obtain that if we show that as $\epsilon \to 0$, the law of $\psi^{\epsilon}$ converges weakly with respect to the topology of local uniform convergence in $\strip_+$ modulo a global additive constant to a function which is harmonic in $\strip_+$, then we have that $\wh{\psi}_j^{\epsilon,\overline{\epsilon},r}$ will converge to a constant. Therefore, it suffices to show the convergence of $\psi^{\epsilon}$ as $\epsilon \to 0$. We are going to deduce this from Theorem~\ref{thm:law_of_sle_excursion} and the independence of $\wt{\eta}^{\wt{T}_{u}}$ and $\wt{h}^{\wt{T}_{u}}$ (viewed modulo additive constant). For each $\delta > 0$ we let $\eta^{\delta}(t) = \delta^{-1} (\wt{\eta}^{\wt{T}_{u}}(\zeta - t) - \wt{\eta}^{\wt{T}_{u}}(\zeta))$. Then Theorem~\ref{thm:law_of_sle_excursion} combined with the time-reversal symmetries of $\SLE_{\kappa}(\rho_1 ; \rho_2)$ processes (\cite[Theorem 1.1]{ms2016imag2}) imply that the law of $\eta^{\delta}$ converges as $\delta \to 0$ to the law of a continuous curve in $\overline{\h}$. We note that $(\wt{h}^{\wt{T}_{u}},\wt{\eta}^{\wt{T}_{u}})$ and $(\wt{h}^0,\wt{\eta}^0)$ have the same law, where we view the distributions modulo additive constants. Let $h^{\delta}$ be the field which arises by precomposing $\wt{h}^{\wt{T}_{u}}$ with $z \mapsto \delta (z + \wt{\eta}^{\wt{T}_{u}}(\zeta))$. Then $h^{\delta}$ (modulo additive constants) converges as $\delta \to 0$ to a free boundary $\text{GFF}$ on $\h$ by the scale and translation invariance of free boundary $\text{GFF}$s and the independence of $\wt{h}^{\wt{T}_{u}}$ (modulo additive constant) and $\wt{\eta}^{\wt{T}_{u}}$. Therefore $(\eta^{\delta},h^{\delta})$ converges in law as $\delta \to 0$ to the law of a pair consisting of a continuous curve in $\overline{\h}$ and an independent free boundary $\text{GFF}$ on $\h$. For each $t \in (0,\zeta - \xi)$, we let $\wt{\varphi}_t^{\epsilon}$ be the conformal transformation mapping $\h \setminus \wt{\eta}^{\wt{T}_u}([\zeta-t,\zeta])$ onto $\strip$ with $\wt{\eta}^{\wt{T}_u}(\zeta)$ mapped to $-\infty$ and $\wt{\eta}^{\wt{T}_u}(\zeta-t)$ mapped to $+\infty$, and the horizontal translation fixed so that the $\gamma-\text{LQG}$ boundary length of $(-\infty,0]$ with respect to $\wt{h}^{\wt{T}_u} \circ (\wt{\varphi}_t^{\epsilon})^{-1} + Q \log |((\wt{\varphi}_t^{\epsilon})^{-1})'|$ is equal to $\epsilon$. Then, arguing as in the proof of \cite[Lemma~6.6]{dms2014mating}, we obtain that as $t,\epsilon \to 0$ with $\epsilon \to 0$ at a sufficiently fast rate relative to the rate at which $t \to 0$, the law of the function which is harmonic in $\strip_+$ with Neumann boundary conditions on $\partial \strip_+ \setminus [0,\pi i]$ and with Dirichlet boundary conditions on $[0,\pi i]$ given by the values of $\wt{h}^{\wt{T}_u} \circ (\wt{\varphi}_t^{\epsilon})^{-1} + Q \log |((\wt{\varphi}_t^{\epsilon})^{-1})'|$ on $[0,\pi i]$ converges weakly with respect to the topology of local uniform convergence modulo global additive constant. Finally, arguing as in the last paragraph of the proof of \cite[Lemma~6.6]{dms2014mating} gives that the total variation distance between the fields $\wt{h}^{\wt{T}_u} \circ (\wt{\varphi}_t^{\epsilon})^{-1} + Q \log |((\wt{\varphi}_t^{\epsilon})^{-1})'|$ and $\wt{h}^{\wt{T}_u} \circ (\varphi^{\epsilon})^{-1} + Q \log |((\varphi^{\epsilon})^{-1})'|$ tends to $0$ as $t \to 0$ and $\epsilon$ tends to $0$ much faster than $t$. This proves the convergence of $\varphi^{\epsilon}$ and hence the convergence of $\psi^{\epsilon}$. This completes the proof of the lemma. \end{proof} Now we are ready to prove Theorem~\ref{thm:wedge_cutting}. \begin{proof}[Proof of Theorem~\ref{thm:wedge_cutting}] We will be brief since the proof is essentially the same as that of \cite[Theorem~6.1]{dms2014mating}. Fix $u>0$ large and let $Y$ be a Bessel process of dimension $\delta(\kappa,\rho)$ as in~\eqref{eq:bessel_dimension}. As in the proof of \cite[Theorem~6.1]{dms2014mating}, we can use the previous results to obtain an asymptotic coupling where the excursions of $Y$ away from $0$ up until some time $s_u$ each correspond to a connected component of $\h \setminus \wt{\eta}^{\wt{T}_u}$ which is to the left of $X_u$ and this correspondence is bijective, where $X_u$ is the last intersection point of $\wt{\eta}^{\wt{T}_u}$ with $\R_+$ before time $\wt{T}_u$. Note also that $Y$ encodes these bubbles as quantum surfaces from right to left. Moreover, by the construction of the coupling, we have that $\wt{U}_t = V_{T_u-t}^{T_u}-W_{T_u-t}^{T_u}$ for all $t \in [0,T_u]$, where $T_u$ is the right continuous inverse of the local time at $0$ of $V^{T_u} - W^{T_u}$ at $u$. Hence $T_u = \wt{T}_u$ a.s.\ and $\wt{h}^{\wt{T}_u}$ is independent of $\wt{\eta}^{\wt{T}_u}$ with the law of the restriction of $\wt{h}^{\wt{T}_u}$ to the bubbles of $\h \setminus \wt{\eta}^{\wt{T}_u}([0,\wt{T}_u])$ ordered from right to left to be determined by the law of $Y$. This completes the proof of the theorem. \end{proof} \subsection{Zipping according to quantum natural time} \label{subsec:natural_time} Suppose that we have the setup of Theorem~\ref{thm:wedge_cutting}. Then Theorem~\ref{thm:wedge_cutting} implies that if $u>0$ is sufficiently large but fixed, we have that the quantum surfaces which are cut out by $\eta|_{[0,T_u]}$ from $\infty$ and they are to the right of $\eta$ have the same Poissonian structure as those arising from a weight $\rho + 2$ quantum wedge. Let $Y$ be the Bessel process encoding the law of the above bubbles when viewed as quantum surfaces modulo coordinate change. Recall that $Y$ induces a quantum measure $m$ on the union of $\eta([0,T_u]) \cap \R_+$ with the set of self-intersection points of $\eta|_{[0,T_u]}$. In Lemma~\ref{lem:main_lemma}, we will identify the conditional law of the pair $(h,\eta)$ when we sample $w$ from $m$ and condition on it, and then show that when we zoom in near $w$ as in \cite[Proposition~4.7]{dms2014mating}, we have that the beaded quantum surface which consists of the connected components of $\h \setminus \eta$ lying to the right of $w$ converges to that of a quantum wedge of weight $\rho + 2$. However, when we condition on $w$, we add to the field an extra log singularity at $w$ and the law of the curve is weighted by a certain Radon-Nikodym derivative. This leads to the weight $\rho + 4$ that we eventually obtain when we zoom in near $w$. Combining, we obtain part (i) of Theorem~\ref{thm:quantum_natural_time_cutting}. Finally, part (ii) of Theorem~\ref{thm:quantum_natural_time_cutting} will follow by observing that the total variation distance between the laws of the pairs of the fields and curves tends to $0$ as $u \to \infty$ when we zoom in near $w$ and when we zoom in near the point obtained by shifting $w$ according to a fixed amount of quantum measure. As we mentioned in the previous paragraph, we will have to deal with fields perturbed by adding certain smooth functions. Therefore, it will be useful to know how the quantum measure described above changes when we add such a function. This is the content of the next lemma. \begin{lemma}[$\text{\cite[Lemma~6.19]{dms2014mating}}$] \label{lem:smooth_perturbation} Assume that we have the same setup as in Theorem~\ref{thm:wedge_cutting}(with the additive constant for $h$ fixed in the same way) and let $L$ be the local time at $0$ associated with the Bessel process $Y$ which encodes the weight $\rho + 2$ wedge corresponding to the bounded components of $\h \setminus \eta([0,T_{u}])$. For each smooth function $\phi$, let $Y^{\phi}$ be the corresponding process with $h$ replaced by $h + \phi$ (which we assume to be parameterized according to its quadratic variation). Let $\overline{\nu}$ (resp.\ $\overline{\nu}^{\phi}$) denote the empirical measure of the excursions made by $Y$ (resp.\ $Y^{\phi}$) from $0$ which correspond to the bubbles cut off $\infty$ by $\eta([T_{r},T_{u}])$. Let $d$ be the Bessel dimension of $Y$. We a.s.\ have (off a common set of measure $0$) for all $r \in [0,u]$ and all such smooth functions $\phi$ that the limit $m^{\phi}([r,u])$ of $\frac{\overline{\nu}^{\phi}(\CE(\epsilon))}{\nu_{d}^{\BES}(\CE(\epsilon))}$ as $\epsilon \to 0$ exists and (with $m = m^0$) \begin{align*} m^{\phi}([r,u]) = \int_{r}^{u}\exp{ \left(\frac{\gamma(2 - d)}{2}\phi(\eta(T_{v}))\right)dm(v)}. \end{align*} \end{lemma} \begin{proof} It follows from the proof of \cite[Lemma~6.19]{dms2014mating}. \end{proof} We note that Lemma~\ref{lem:smooth_perturbation} implies that the law of $Y^{\phi}$ is mutually absolutely continuous with respect to the law of $Y$ and moreover the local time of $Y^{\phi}$ at $0$ is defined for all smooth $\phi$ simultaneously. Now, we are ready to state and prove Lemma~\ref{lem:main_lemma}. \begin{lemma}[$\text{\cite[Lemma~6.21]{dms2014mating}}$] \label{lem:main_lemma} Assume that we have the setup of Theorem~\ref{thm:wedge_cutting} (with the additive constant for $h$ fixed in the same way) and recall that \begin{align} \label{eq:law_of_the _field} h = \wh{h} - \frac{2 + \rho}{\gamma}\log|\cdot| \end{align} where $\wh{h}$ is a free boundary $\text{GFF}$ on $\h$. For each $0 \leq q < r \leq u$, we let $m_{q,r}$ be the restriction of $m$ as in Lemma~\ref{lem:smooth_perturbation} to subsets of $[q,r]$. Consider the law on $(w,h,\eta)$ triples given by $\CZ^{-1}_{q,r}dm_{q,r}dhd\eta$ where $dh$ denotes the law as in~\eqref{eq:law_of_the _field} and $\CZ_{q,r}^{-1}$ is a normalization constant. (Note that $m_{q,r}$ depends on $h$ and $\eta$.) \begin{enumerate}[(i)] \item Given $w$ and $\eta$, the conditional law of $h$ is equal to the law of $$\wh{h} - \frac{2 + \rho}{\gamma}\log|z| + \frac{\gamma(2 - d)}{2}G(z,\eta(T_w)) + \psi \circ f_{T_{u}}$$ where $\wh{h}$ is a free boundary $\text{GFF}$ on $\h$, $d$ is the dimension of the Bessel process $Y$, $\psi$ is a function which is harmonic outside of $\h \cap \partial{\D}$, and the additive constant is fixed in the same manner as for $h$. \item Given $w$ and $\eta|_{[0,T_{w}]}$, the conditional law of $\eta|_{[T_{w},\infty)}$ is that of an $\SLE_{\kappa}(\rho)$ process in the unbounded component of $\h \setminus \eta([0,T_{w}])$ from $\eta(T_{w})$ to $\infty$ with a single boundary force point of weight $\rho$ located at $(\eta(T_{w}))^{+}$ weighted by the Radon-Nikodym derivative \begin{align*} \CZ^{-1}\exp\!\left(\frac{\gamma^{2}(2-d)^2}{8}\left(\int \int G(f_{T_u}^{-1}(x),f_{T_u}^{-1}(y))dxdy - 2 \int G(\eta(T_w),f_{T_u}^{-1}(x))dx\right)\right) \end{align*} where $G$ denotes the Neumann Green's function on $\h$ and $\CZ^{-1}$ is a normalizing constant and the integrals are all over $\partial \D \cap \partial \h$. \item If one zooms in near $\eta(T_{w})$ as in part (ii) of \cite[Proposition 4.7]{dms2014mating}, then the law of the beaded surface which consists of the components of $\h \setminus \eta$ which are to the right of $\eta|_{[T_{w},\infty)}$ converges to that of a quantum wedge of weight $\rho + 2$. \end{enumerate} \end{lemma} \begin{proof} The proofs of parts (i) and (ii) follow from the proofs of parts (i) and (ii) respectively of \cite[Lemma~6.21]{dms2014mating}. We note that the harmonic function $\psi \circ f_{T_u}$ obtained in part (i) can be defined on $\h$ since the Lebesgue measure of $\eta \cap \h$ is equal to $0$ a.s. The latter follows since the law of $\eta$ on the intervals that it is not colliding with its force point is mutually absolutely continuous with respect to the law of an $\SLE_{\kappa}$ restricted to the corresponding intervals. So the above harmonic function extends to a function defined on $\h$. Now we turn to the proof of part (iii). First of all, we recall that the beaded surface consisting of the bubbles parameterized by the components of $\h \setminus \eta([0,T_r])$ from right to left is given by that of a quantum wedge $\CW$ of weight $\rho + 2$, up to a given amount of local time $M$, which is a random time. Without loss of generality, we can assume that $\CW$ is a bi-infinite wedge (which is just a concatenation of two independent weight $\rho + 2$-wedges). More precisely, let $\CW_1,\CW_2$ be two independent weight $\rho + 2$-wedges with corresponding encoding Bessel processes $(Y_{t}^1)_{t \geq 0}$ and $(Y_{t}^2)_{t \geq 0}$ respectively. Then, their concatenation is the beaded surface $\CW$ encoded by the process $Y = (Y_t)_{t \in \R}$ defined by $Y_t = Y_{t}^1$ for $t \geq 0$ and $Y_t = Y_{-t}^2$ for $t \leq 0$. For $\epsilon \in (0,1)$ we consider the conformal transformation $\varphi_{\epsilon} \colon \h \to \h$ with $\varphi_{\epsilon}(z) = z/\epsilon$ and the $\text{GFF}$ on $\h$, \begin{align*} h^{\epsilon,C} = h \circ f_{T_w}^{-1} \circ \varphi_{\epsilon}^{-1} + Q\log |(f_{T_w}^{-1})' \circ \varphi_{\epsilon}^{-1}| + Q\log (\epsilon) + \frac{C}{\gamma} \end{align*} and set $\epsilon(C) = \sup\{\epsilon \in [0,1] : h_{1}^{\epsilon,C}(0) = 0\}$ and $h^{C} = h^{\epsilon(C),C}$. Here, $h_{1}^{\epsilon,C}(0)$ is the average of the field $h^{\epsilon,C}$ on $\h \cap \partial \D$. We want to show that the beaded surface with respect to $h^{C}$ which consists of the components of $\h \setminus \varphi_{\epsilon}(\eta^w)$ which are to the right of $\varphi_{\epsilon}(\eta^w)$ converges to that of a quantum wedge of weight $\rho + 2$ as $C \to \infty$, where $\eta^w(t) = f_{T_w}(\eta(t+T_w))$ for $t \geq 0$. Next, we fix a large number $N$ and we assume that $X \in [0,N]$ is sampled uniformly at random. If we take $\CW$ and recenter it after shifting by $X$ units of local time, i.e., by considering the wedge encoded by the process $Y^T = (Y_{t}^T)_{t \in \R}$ with $Y_{t}^{T} = Y_{T - t}$ for $t \in \R$ and $T = T_X$, then the bubbles of $\CW$ going from left to right have the law of the bubbles in a weight $\rho + 2$-wedge because the law of the bubbles is invariant under the time-reversal and recentering (simply because the Poisson law has this property). Therefore the law of the beaded surface with respect to $h^C$ which consists of the bubbles of $\h \setminus \varphi_{\epsilon}(\eta^w)$ lying on $B(0,\epsilon(C)^{-1})$ can be sampled as follows. Let $\wt{\CW}$ be a bi-infinite wedge of weight as above coupled with $\eta$ such that the bubbles to the right of $\eta$ (starting from $\eta(T_r)$ and going from right to left) agree with the bubbles of the wedge encoded by $(\wt Y_t)_{t \geq 0}$, where $\wt Y $ is the encoding process for $\wt{\CW}$, and up until $M$ units of local time. We fix $N \in \N$ large and we pick $X \in [0,N]$ uniformly at random. Then we condition on the event that $X \in [0,M]$ and we consider the beaded surface which is part of $\wt{\CW}$ and it is parameterized by the bubbles which are images under $f_{T_X}^{-1} \circ \varphi_{\epsilon}^{-1}$ of the bubbles of $\h \setminus \varphi_{\epsilon}(\eta^X)$ lying on the right of $\varphi_{\epsilon}(\eta^X)$. Then by letting $N \to \infty$, we have that the law of the above surface converges to the law of the surface parameterized by the bubbles of $\h \setminus \varphi_{\epsilon}(\eta^X)$ lying to the right of $\varphi_{\epsilon}(\eta^X)$. Fix $K>0$ and let $\CF_{C,N}$ be the $\sigma$-algebra generated by the bubbles of the previous paragraph. Then the law of these bubbles when conditioning on $\CF_{C,N}$ has Radon-Nikodym derivative with respect to the law without conditioning equal to \begin{align*} \frac{\p[ X \in [0,M]\,|\,\CF_{C,N}]}{\p[ X \in [0,M] ]}. \end{align*} Suppose that we have shown that the $\sigma$-algebra $\CF_{C,N}$ becomes trivial when $C \to \infty$, for every fixed $N \in \N$. Then, by the backwards martingale convergence theorem, we have that the Radon-Nikodym derivative converges to $1$ as $C \to \infty$ a.s. Also, the unconditional law of the bubbles is the same as if we replaced $h^C$ by a wedge of weight $\rho + 2$. By letting $N \to \infty$ we obtain the claim of part (iii). Hence, in order to complete the proof of part (iii), we need to show that $\cap_{C>0} \CF_{C,N}$ is trivial for all $N \in \N$. Let $\CF_{C,N}^1$ (resp.\ $\CF_{C,N}^2$) be the $\sigma$-algebra generated by the restriction of $h_X = h \circ f_{T_X}^{-1} + Q \log |(f_{T_X}^{-1})'|$ to $\h \cap B(0,K \epsilon(C))$ (resp.\ the bubbles cut off $\infty$ by $\eta^X$ which lie on $B(0,K \epsilon(C))$). Set $\CG_{N,C} = \sigma(\CF_{C,N}^1,\CF_{C,N}^2)$ and $\CG_{N} = \cap_{C>0}\CG_{N,C}$. Then it suffices to show that $\CG_N$ is trivial for every fixed $K$. For the latter, it suffices to prove that the law of the pair $((h_X,\phi),\eta^X)$ given $\CG_{C,N}$ converges as $C \to \infty$ to the unconditional law of $((h_X, \phi),\eta^X)$, for every fixed $\phi \in C_{0}^{\infty}(\h)$ with $\int_{\h}\phi(z)dz = 0$. To show this, first we note that the conditional law of the pair consisting of $h_X$ and $\eta^X$ given $X$ is equal to that of an $\SLE_{\kappa}(\rho)$ process $\eta$ in $\h$ with a single force point at $0_+$ weighted by the Radon-Nikodym derivative of part (ii) and $h$ can be expressed as $\wh{h} + \left(\frac{\rho +2 -\gamma^2}{\gamma}\right) \log |\cdot| + \psi \circ f_{T_{N-X}}$, where $\wh{h}$ is a free boundary $\text{GFF}$ and the additive constant is taken so that the average of the field after applying the coordinate change with $f_{T_{N-X}}$ on $\h \cap \partial \D$ is equal to $0$. This form of the conditional law follows by combining part (ii) with the proof of \cite[Theorem~6.16]{dms2014mating}. Set $\wh{\psi} = \left( \frac{\rho + 2 - \gamma^2}{\gamma}\right) \log |\cdot| + \psi \circ f_{T_{N-X}}$ and note that conditional on $X$, $\wh{\psi}$ is determined by $\eta^X$. Note also that the Markov property of the $\text{GFF}$ implies that $\wh{h}$ can be decomposed as $\wh{h} = \wh{h}_C +\wh{ \mathcal{H}}_C$, where $\wh{h}_C$ is a $\text{GFF}$ on $\h \setminus B(0,K \epsilon(C))$ with free boundary conditions on $\h \cap \partial B(0,K\epsilon(C))$ and $\wh{\mathcal{H}}_C$ is a harmonic function on $\h$ with boundary conditions given by those of $\wh{h}$ on $\h \cap \partial B(0,K \epsilon(C))$ and free boundary conditions on $\partial \h \setminus B(0,K\epsilon(C))$. Hence the conditional law of $(h_X,\phi)$ given $X$ and $\eta^X$ is that of a Gaussian random variable with mean $(\wh{\mathcal{H}}_C,\phi)$ and variance $\int_{\h}\int_{\h} \phi(y)G_C(y,z)\phi(z)dydz$, where $G_C$ is the Green's function on $\h \setminus B(0,K \epsilon(C))$ with Dirichlet (resp.\ Neumann) boundary conditions on $\h \cap \partial B(0,K \epsilon(C))$ (resp.\ $\partial \h \setminus B(0,K \epsilon(C)))$. Note that \begin{equation}\label{eq:green_convergence} \int_{\h}\int_{\h} \phi(y) G_C(y,z) \phi(z) dydz \to \int_{\h}\int_{\h} \phi(y) G(y,z) \phi(z) dydz \,\,\,\text{as}\,\,\,C \to \infty, \end{equation} since $G_C \to G$ as $C \to \infty$ locally uniformly. Also, given $X$, $\eta^X$ and $\CG_{C,N}$, the random variable $(\wh{\mathcal{H}}_C,\phi)$ is a Gaussian with mean zero and covariance given by $\int_{\h}\int_{\h} \phi(y) \cov(\wh{\mathcal{H}}_C(y),\wh{\mathcal{H}}_C(z)) \phi(z) dydz$. Then, using the explicit form of the Poisson kernel in $\h \setminus B(0,\epsilon)$, we have that the latter covariance tends to $0$ as $C \to \infty$. Therefore, we obtain that the conditional law of $(h_X,\phi)$ given $(X,\eta^X)$ and $\mathcal{G}_{C,N}$ converges to the conditional law of $(h_X,\phi)$ given $(X,\eta^X)$ as $C \to \infty$. Moreover, $X$ is independent of $\mathcal{G}_{C,N}$ and the conditional law of $\eta^X$ given $X$ and $\mathcal{G}_{C,N}$ converges to the conditional law of $\eta^X$ given $X$ as $C \to \infty$. The claim then follows since $(h_X,\phi)$ and $\eta^X$ are independent given $X$. This completes the proof of part (iii). \end{proof} \begin{proof}[Proof of Theorem~\ref{thm:quantum_natural_time_cutting}.] Now that we have proved Theorem~\ref{thm:wedge_cutting} and Lemma~\ref{lem:main_lemma}, the proof follows the same argument as that of the proof of \cite[Theorem~6.16]{dms2014mating}. \end{proof} \subsection{Quantum boundary length evolution} \label{subsec:boundary_length_evolution} Suppose that we have the setup of Theorem~\ref{thm:quantum_natural_time_cutting}. In this subsection we are going to describe the law of the boundary length evolution of $\eta$ when it has the quantum natural time parameterization as in Theorem~\ref{thm:quantum_natural_time_cutting}, and hence proving Theorem~\ref{thm:boundary_length_evolution}. As an intermediate step in the proof of Theorem~\ref{thm:boundary_length_evolution}, we are going to describe the Poissonian structure of the quantum lengths of the beads of a surface corresponding to a quantum wedge of weight $\rho + 2$ with $\kappa \in (0,4)$ and $\rho \in (\kappa/2-4,-2) \cap (-2-\kappa/2,-2)$. Finally, we are going to prove Corollary~\ref{cor:dim_of_boundary_intersection} as a consequence of Theorem~\ref{thm:boundary_length_evolution}. We start by analyzing the Poissonian structure of the quantum lengths of the beads of a weight $\rho + 2$ quantum wedge. Our strategy will be similar to that used to prove \cite[Proposition~4.18]{dms2014mating}. In particular, we prove the following. \begin{theorem} \label{thm:law_of_quantum_lengths} Fix $\kappa$ and $\rho$ as above. Let $\CW$ be a quantum wedge of weight $\rho + 2$ and let $h$ be the corresponding field. Let $Y$ be the Bessel process encoding $\CW$ and $(e_{j})_{j \in \N}$ be its excursions away from zero. Let $(h_{j})_{j \in \N}$ be the fields encoding the surfaces determined by the above excursions. Let also $t_{j}$ be the boundary quantum length associated with $h_{j}$ for all $j \in \N$. Then $(t_{j})$ has the law of a $\text{p.p.p.}$ with intensity measure given by $c( du \times t^{\alpha}dt)$ where $\alpha = -2 + \frac{2(\rho + 2)}{\kappa}$ and $c > 0$ is a constant depending only on $\alpha$ and $\rho$. \end{theorem} \begin{proof} Note that we can sample $Y$ by first sampling a $\text{p.p.p.}$ $\Lambda^*$ according to $du \times \nu_{\delta}^*(dt)$ with $\nu_{\delta}^*(dt) = c_{\delta}^* t^{\delta - 3}dt$ for some constant $c_{\delta}^*>0$ depending only on $\delta$ and then sampling $Y$ by associating with each $(u,e*) \in \Lambda^*$ a $\BES^{\delta}$ excursion away from zero whose maximum value is given by $e^*$. Here $\delta$ satisfies~\eqref{eq:bessel_dimension}. Note that for fixed $e^*$ the law of the latter can be sampled by joining back to back two independent Bessel processes with dimension $\wt{\delta} = 4 - \delta$ starting from $0$ and up until the first time they hit $e^*$ (see \cite[Remark~3.7]{dms2014mating}). Let $\mu_{\delta}^{x}$ be the law on such paths for $x = e^* \in (0,+\infty)$. Suppose that $X = (X_{t})_{0 \leq t \leq T}$ is sampled from $\mu_{\delta}^c$, where $T$ is the length of the excursion. Since Bessel processes satisfy Brownian scaling, we obtain that the process $\wt{X} = (c^{-1}X_{c^{2}t})_{0 \leq t \leq \wt{T}}$ with $\wt{T} = c^{-2}T$ is sampled from $\mu_{\delta}^{1}$. For $\epsilon \in (0,1)$ we set $T_{\epsilon} = \inf \{t \geq 0 : X_{t} \geq \epsilon \} $, $\wt{T}_{\epsilon} = \inf \{t \geq 0 : \wt{X}_{t} \geq \epsilon \}$ and \begin{align*} \sigma^{\epsilon}(t) = \inf \left\{s \geq 0 : \frac{4}{\gamma^{2}}\int_{0}^{s}\frac{1}{(X^{\epsilon}_{r})^{2}}dr \geq 2t \right \} \\ \wt{\sigma}^{\epsilon}(t) = \inf \left\{s \geq 0 : \frac{4}{\gamma^{2}}\int_{0}^{s}\frac{1}{(\wt{X}^{\epsilon}_{r})^{2}}dr \geq 2t \right \}, \end{align*} where $X^{\epsilon}_{t} = X_{T_{\epsilon} + t}$ for $0 \leq t \leq T^{\epsilon}$ and $\wt{X}^{\epsilon}_{t} = \wt{X}_{\wt{T}_{\epsilon} + t }$ for $0 \leq t \leq \wt{T}^{\epsilon}$, and $T^{\epsilon} = T - T_{\epsilon}$, $\wt{T}^{\epsilon} = \wt{T} - \wt{T}_{\epsilon}$. It follows from \cite[Proposition~3.4]{dms2014mating} that $Z^{\epsilon}$ (resp.\ $\wt{Z}^{\epsilon}$) evolves as a Brownian motion starting from $\frac{2}{\gamma}\log(\epsilon)$ and run twice the speed with drift $\frac{(2-\delta)}{2}\gamma$ up until the first time it hits $\frac{2}{\gamma}\log(c)$ (resp.\ $0$) and then it evolves independently as a Brownian motion starting from $\frac{2}{\gamma}\log(c)$ (resp.\ $0$) and run twice the speed with drift $\frac{ (\delta - 2)}{2}\gamma$, where $Z^{\epsilon} = \frac{2}{\gamma}\log(X_{\sigma^{\epsilon}(t)}^{\epsilon})$ and $\wt{Z}_{t}^{\epsilon} = \frac{2}{\gamma}\log(\wt{X}_{\wt{\sigma}^{\epsilon}(t)}^{\epsilon})$ for $t \geq 0$. Also, we have that $\wt{T}_{\epsilon} = c^{-2}T_{\epsilon c}$ and hence $\wt{X}_{t}^{\epsilon} = c^{-1} X_{c^{2}t}^{\epsilon c}$ and $\wt{\sigma}^{\epsilon}(t) = c^{-2}\sigma^{\epsilon c}(t)$ for all $t \geq 0$. Moreover if $\sigma^{\epsilon} = \inf \{t \geq 0 : Z_{t}^{\epsilon} \geq \frac{2}{\gamma} \log(c) \}$ (resp.\ $\wt{\sigma}^{\epsilon} = \inf\{t \geq 0 : \wt{Z}_{t}^{\epsilon} \geq 0 \}$), then $Z_{\sigma^{\epsilon}+ t}^{\epsilon}$ for $t \geq - \sigma^{\epsilon}$ (resp.\ $\wt{Z}_{\wt{\sigma}^{\epsilon} + t}$ for $t \geq - \wt{\sigma}^{\epsilon}$) converges weakly as $\epsilon \to 0$ with respect to the local uniform topology to a process $Z$ (resp.\ $\wt{Z}$) indexed by $\R$. Note also that $\wt{Z}_t = Z_t - \frac{2}{\gamma}\log(c)$ for all $t \in \R$. The above observations imply that the total quantum length of a surface associated with a given $(u,e^*)$ is given by $e^*U_{e}$ where the $U_{e}$ are i.i.d. random variables indexed by the excursions of $Y$ away from $0$. Also the law of $U_{e}$ is given by the total quantum length of the random surface $\wt{h}$ sampled as follows: \begin{enumerate}[(i)] \item Let $X$ be a sample from $\mu_{\delta}^{1}$. The projection of $\wt{h}$ onto $\CH_{1}(\strip)$ is given by parameterizing $\frac{2}{\gamma}\log(X)$ to have quadratic variation $2dt$. \item The projection of $\wt{h}$ onto $\CH_{2}(\strip)$ is given by taking an independent sample of the law of the corresponding projection onto $\CH_{2}(\strip)$ of a free boundary $\text{GFF}$ on $\strip$. \end{enumerate} Note that $ \alpha = \delta - 3$, where $\alpha$ is as in the statement of the theorem, and if $U$ is sampled from the law of $U_{e}$ then $\E[U^{-\alpha - 1}] = c < \infty$ since $-\alpha -1 = 1 - \frac{2(\rho + 2)}{\kappa} \in (0,\frac{4}{\kappa})$ and so \cite[Lemma~4.20]{dms2014mating} applies. Therefore $\{(u,e^*U_{e}) : (u,e^*) \in \Lambda^* \}$ is a $\text{p.p.p.}$ with intensity measure given by $c(du \times t^{\alpha}dt)$ by applying \cite[Lemma~4.19]{dms2014mating}. We also observe that the above $\text{p.p.p.}$ corresponds to the sequence of jumps of a positive stable subordinator with parameter $-\alpha -1 \in (1,2)$. This completes the proof. \end{proof} Now suppose that we have the same setup as in Theorem~\ref{thm:quantum_natural_time_cutting}. We note that the quantum natural time $(\qnt_{u})_{u \geq 0}$ can be expressed as follows. Let $(x_{j},t_{j})_{j \in \N}$ be the $\text{p.p.p.}$ encoding the local time at $0$ $L$ of $Y$ and let $(e_{j})_{j \in \N}$ be the corresponding excursions of $Y$ away from zero. For all $t \geq 0$ we define $A(t)$ to be the set of $j \in \N$ such that the bubble corresponding to $e_{j}$ has been drawn completely by $\eta$ by capacity time $t$ and $x(t) = \sum_{j \in A(t)}t_{j}$. Then we set \begin{align*} \qnt_{u} = \inf \{t \geq 0 : L_{x(t)} > u \} \end{align*} for all $u \geq 0$. Now we are ready to prove Theorem~\ref{thm:boundary_length_evolution}. Recall the definitions of $A_u, B_u, X_u$ and $Z_u$ in Section~\ref{sec:introduction}. \begin{proof}[Proof of Theorem~\ref{thm:boundary_length_evolution}.] First, we prove that $(X_u,Z_u)$ is an $\alpha$-stable L\'evy process. Note that if we run the process for $u$ units of quantum natural time, the unexplored region (quantum surface parameterized by the unbounded component) is again a weight $\rho + 4$ wedge and the curve is an $\SLE_{\kappa}(\rho)$ (Theorem~\ref{thm:quantum_natural_time_cutting}). Hence the increments of $(X,Z)$ are independent and so it is a Levy process. For all $\epsilon > 0$, let $\CE(\epsilon)$ be the set of excursions of $Y$ with time length at least $\epsilon$. For all $t \geq 0$ let $\eta_{t}(\CE(\epsilon))$ be the number of excursions completed by $Y$ by time $t$ which lie in $\CE(\epsilon)$. Let $\nu_{\delta}$ be the It\^o excursion measure of a $\BES^{\delta}$ process. It follows from \cite[Lemma~6.18]{dms2014mating} that there exists a constant $C_{\delta} > 0$ depending only on $\delta$ such that $\nu_{\delta}(\CE(\epsilon)) = C_{\delta}\epsilon^{\frac{\delta}{2} - 1}$ for all $\epsilon > 0$. We also have from \cite[Proposition~19.12]{kallenberg1997foundations} that $L_{t} = \lim_{\epsilon \to 0}\frac{\eta_{t}(\CE(\epsilon))}{\nu_{\delta}(\CE(\epsilon))}$ for all $t \geq 0$ a.s. Fix $C \in \R$ and let $\wt{Y}$ be the $\BES^{\delta}$ process encoding $h + C$. Then it holds that $\wt{Y}_{t} = e^{\frac{C\gamma}{2}}Y_{e^{-\gamma C}t}$ for all $t \geq 0$. Let $\wt{L},\wt{q},\wt{\eta}_{t}$ be the corresponding quantities for $\wt{Y}$. Then $\wt{\eta}_{t}(\CE(\epsilon)) = \eta_{e^{-\gamma C}t}(\CE(\epsilon e^{-\gamma C}))$ and \begin{align*} \frac{\nu_{\delta}(\CE(\epsilon))}{\nu_{\delta}(\CE(\epsilon e^{-\gamma C}))} = \frac{C_{\delta}\epsilon^{\frac{\delta}{2}-1}}{C_{\delta}\epsilon^{\frac{\delta}{2} - 1}e^{\gamma C (1 - \frac{\delta}{2})}} = e^{\gamma C (\frac{\delta}{2} - 1)} \end{align*} and hence \begin{align*} \frac{\wt{\eta}_{t}(\CE(\epsilon))}{\nu_{\delta}(\CE(\epsilon))} = \frac{\eta_{e^{-\gamma C}t}(\CE(\epsilon e^{-\gamma C}))}{\nu_{\delta}(\CE(\epsilon e^{-\gamma C}))}e^{\gamma C(1 - \frac{\delta}{2})} \end{align*} converges to $e^{\gamma C(1 - \frac{\delta}{2})}L_t$ as $\epsilon \to 0$ for all $t \geq 0$ a.s. Thus $\wt{L}_{t} = e^{\gamma C(1 - \frac{\delta}{2})}L_t$ for all $t \geq 0$ a.s. Moreover $\wt{x}(t) = \sum_{j \in A(t)}\wt{t}_{j} = e^{\gamma C}\sum_{j \in A(t)}t_{j}$ and so $\wt{L}_{\Tilde{x}(t)} = e^{\gamma C(1 - \frac{\delta}{2})}L_{e^{-\gamma C}\wt{x}(t)} = e^{\gamma C(1 - \frac{\delta}{2})}L_{x(t)}$ for all $t \geq 0$ a.s. Therefore we have that $\wt{q}_{u} = \qnt_{e^{\gamma C(\frac{\delta}{2} - 1)}u}$ for all $u \geq 0$ a.s. Let $(\wt{X},\wt{Z})$ be the process describing the change in the boundary length relative to time zero corresponding to the field $h + C$. Since the quantum length scales by $e^{\frac{\gamma C}{2}}$ when we add $C$ to the field, we obtain that \begin{align*} (\wt{X}_{u},\wt{Z}_{u}) = \left(e^{\frac{\gamma C}{2}}X_{e^{\gamma C(\frac{\delta}{2} - 1)}u},e^{\frac{\gamma C}{2}}Z_{e^{\gamma C(\frac{\delta}{2} - 1)}u}\right) \end{align*} for all $u \geq 0$. Set $\mu = e^{\gamma C(\frac{\delta}{2} - 1)}$. Then $\mu^{-\frac{1}{\alpha}} = e^{\frac{\gamma C}{2}}$ and so $(\wt{X}_{u},\wt{Z}_{u}) = (\mu^{-\frac{1}{\alpha}}X_{\mu u},\mu^{-\frac{1}{\alpha}}Z_{\mu u})$ for all $u \geq 0$. The claim of the theorem then follows since $(h,\eta)$ and $(h + C,\eta)$ have the same law for all $C \in \R$ (\cite[Proposition~4.8]{dms2014mating}) and $(\wt{X},\wt{Z})$ is determined by $(h + C,\eta)$. We note that $(X,Z)$ makes a jump at time $u$ if and only if the right continuous inverse $T$ of $L$ makes a jump at time $u$. Note also that there is a correspondence between the discontinuities of $T$ and the excursions $(e_{j})_{j \in \N}$ made by $Y$ away from zero. Let $(h_{j})_{j \in \N}$ be the corresponding sequence of quantum surfaces. We also observe that when $(X,Z)$ makes a jump, both of $X$ and $Z$ make a jump with positive and negative sign respectively. The jump sizes of $X$ and $-Z$ are equal to the quantum lengths of $\R$ and $\R \times \{\pi\}$ respectively under the corresponding surface $(\strip,h_{j},-\infty,+\infty)$, where the opening (resp.\ closing) point of the pocket made by $\eta$ which corresponds to the excursion $e_j$ is mapped to $-\infty$ (resp.\ $+\infty$). We are now ready to describe the law of the boundary length evolution in the case where $\rho = \kappa - 4$ and $\kappa \in (\frac{4}{3},2) $. Since $\rho = \kappa - 4$, we have that the corresponding surfaces have the law of i.i.d. quantum disks. By the resampling property characterizing the two marked points of a unit boundary length quantum disk (\cite[Proposition~A.8]{dms2014mating}) and the Poissonian structure of the sequence of their quantum boundary lengths (Theorem~\ref{thm:law_of_quantum_lengths}), we obtain that the sequence of jumps of $(X,-Z)$ can be sampled as follows: \begin{enumerate}[(i)] \item Firstly, we sample a $\text{p.p.p.}$ $\Lambda = \{(s_{j},t_{j})\}_{j \in \N}$ on $\R_+ \times \R_+$ with intensity measure $c(du \times t^{\alpha}dt)$ where $\alpha$ and $c$ are as in Theorem~\ref{thm:law_of_quantum_lengths}. In our case, we have that $\alpha = -\frac{4}{\kappa}$. \item Then we sample independently a sequence of i.i.d. random variables with the uniform law on $[0,1]$, $\{U_{j}\}_{j \in \N}$ and we consider \begin{align*} \Lambda^* = \{(t_{j}u_{j} , (1 - u_{j})t_{j}) : (s_{j},t_{j}) \in \Lambda \}_{j \in \N} \end{align*} \end{enumerate} Note that $\wt{\Lambda} = \{(s_{j},t_{j},u_{j})\}_{j \in \N}$ is a $\text{p.p.p.}$ on $\R_+ \times \R_+ \times [0,1]$ with intensity measure given by $\nu = c(du \times t^{\alpha}dt) \times \mu$, where $\mu$ is the law of a uniform random variable on $[0,1]$. Consider the function $F \colon \R_+ \times \R_+ \times [0,1] \to \R_+ \times \R_+$ with $F(s,t,u) = (tu,t(1 - u))$. Then $\Lambda^*$ is a $\text{p.p.p.}$ on $ \R_+ \times \R_+$ with intensity measure given by $\nu_{*}F$. Set $\wt{\nu}(dx,dy) = c(x + y)^{\alpha - 1}dxdy$ to be defined on $\R_+ \times \R_+$. It is easy then to check that $\nu_{*}F([x_{1},x_{2}] \times [y_{1},y_{2}]) = \wt{\nu}([x_{1},x_{2}] \times [y_{1},y_{2}])$ for all $0 < x_{1} < x_{2}$ and $0 < y_{1} < y_{2}$. Therefore we have that $\nu_{*}F = \wt{\nu}$ and so we have described the law of $(X,-Z)$ since it is determined by the law of the jumps. This completes the proof of the theorem. \end{proof} Now that we have completed the proof of Theorem~\ref{thm:boundary_length_evolution}, we are ready to prove Corollary~\ref{cor:dim_of_boundary_intersection}. \begin{proof}[Proof of Corollary~\ref{cor:dim_of_boundary_intersection}.] Suppose that we have the setup of Theorems~\ref{thm:quantum_natural_time_cutting} and~\ref{thm:boundary_length_evolution} where the $\SLE_{\kappa}(\rho)$ process $\eta$ is drawn on top of an independent quantum wedge $\CW = (\h,h,0,\infty)$ of weight $\rho + 4$ and we parameterize $\eta$ by quantum natural time with respect to $\CW$. Let $(X,Z)$ be the pair encoding the change of the quantum lengths of the left and right outer boundaries as in Theorem~\ref{thm:boundary_length_evolution}. Then Theorem~\ref{thm:boundary_length_evolution} implies that $(X,-Z)$ has the law of an $\alpha$-stable L{\'e}vy process where $\alpha$ satisfies~\eqref{eqn:alpha_value}. We parameterize $\R_+$ according to quantum length with respect to $h$ and set $I_t = \inf_{s \in [0,t]} Z_s$, $S_t = \sup_{s \in [0,t]}(-Z_s)$ for all $t \geq 0$. Let $L = (L_t)$ be the local time of $Z-I$ at $0$ which is the same as the local time of $S+Z$ at $0$. Let also $L^{-1}$ be the right-continuous inverse of $L$. Then, it follows from \cite[Chapter~VIII]{bertoin1996levy} that $L^{-1}$ is a stable subordinator with index $1 - \frac{1}{\alpha}$. Note that the ladder height process is defined by $H_t = S_{L_t^{-1}}$ for all $t \geq 0$ (see \cite[Chapter~VI]{bertoin1996levy}). Moreover \cite[Lemma~1, Chapter~VIII]{bertoin1996levy} implies that $H$ is a L{\'e}vy stable process with index $\alpha-1 \in (0,1)$ and hence it is an $(\alpha-1)$-stable subordinator. We note that $Z$ achieves a record minimum at time $t$, i.e., $Z_t = I_t$, if and only if $x = \eta(t) \in \R_+$ and then $x = -Z_t$. It follows that $\{\nu_h([0,x]) : x \in \eta \cap \R_+\} = \overline{\{H_t : t \geq 0\}}$. We also note that $H$ has Laplace exponent $\Phi$ given by $\Phi(\lambda) = \lambda^{\alpha-1}$ for all $\lambda > 0$. Therefore, combining with \cite[Theorem~15, Chapter~III]{bertoin1996levy}, we obtain that \begin{equation}\label{eqn:hausdorff_dim_equality} \text{dim}_{\mathcal{H}}(\{H_t : t \geq 0\}) = \text{dim}_{\mathcal{H}}(\{\nu_h([0,x]) : x \in \eta \cap \R_+\}) = \alpha-1 = -\frac{2(\rho+2)}{\kappa}\,\,\text{a.s.} \end{equation} where $\text{dim}_{\mathcal{H}}$ denotes Hausdorff dimension. We note that it follows from \cite[Theorem~4.1]{rhodes2008kpz} that if $\wt{h}$ is a free boundary $\text{GFF}$ on $\h$ and $K \subseteq [0,\infty)$ is a deterministic set, then if $\wh{K} = \{\nu_{\wt{h}}([0,x]) : x \in K\}$, it holds that \begin{equation}\label{eqn:kpz_formula_equation} \text{dim}_{\mathcal{H}}(K) = \left(1+\frac{\gamma^2}{4}\right) \text{dim}_{\mathcal{H}}(\wh{K}) -\frac{\gamma^2}{4}\text{dim}_{\mathcal{H}}(\wh{K})^2\,\,\text{a.s.} \end{equation} Note that if we consider $\CW$ with the circle average embedding, then the law of the restriction of $h$ to any subdomain of $\h$ which is bounded away from $0,\infty$ and $\h \cap \partial \D$ is mutually absolutely continuous with respect to the law of the corresponding restriction of a free boundary $\text{GFF}$ on $\h$ normalized to have average zero on $\h \cap \partial \D$. Since $\eta$ is independent of $h$, we obtain that~\eqref{eqn:kpz_formula_equation} holds a.s.\ when $K$ is replaced by $\eta \cap \R_+$ and $\wt{h}$ is replaced by $h$. Then, \eqref{eqn:dim_of_intersection_with_R_+} follows by combining with~\eqref{eqn:hausdorff_dim_equality}. This completes the proof of the corollary. \end{proof} \bibliographystyle{abbrv} \bibliography{biblio} \end{document}
2412.04148v1
http://arxiv.org/abs/2412.04148v1
Recursively Extended Permutation Codes under Chebyshev Distance
\documentclass[journal,draft,onecolumn]{IEEEtran} \usepackage[dvipdfmx]{graphicx,xcolor} \usepackage[fleqn]{amsmath} \usepackage[percent]{overpic} \usepackage{amsthm} \usepackage{amssymb} \usepackage{amsfonts} \usepackage{newtxtext} \usepackage[varg]{newtxmath} \usepackage{here} \usepackage{bbm} \usepackage{mathtools} \mathtoolsset{showonlyrefs=true} \let\labelindent\relax \usepackage[inline, shortlabels]{enumitem} \usepackage{tablefootnote} \usepackage{mathdots} \usepackage{algorithmic} \usepackage{amssymb} \usepackage{algorithm} \usepackage{xspace} \input{notation} \def\stkeqa{\stackrel{\rm{(a)}}{=}} \def\stkeqb{\stackrel{\rm{(b)}}{=}} \def\stkeqc{\stackrel{\rm{(c)}}{=}} \def\stkeqd{\stackrel{\rm{(d)}}{=}} \def\preqa{\stackrel{\rm{(a)}}{=}} \def\preqb{\stackrel{\rm{(b)}}{=}} \def\preqc{\stackrel{\rm{(c)}}{=}} \def\preqd{\stackrel{\rm{(d)}}{=}} \def\preqe{\stackrel{\rm{(e)}}{=}} \def\eqa{\stackrel{\rm{(a)}}{=}} \def\eqb{\stackrel{\rm{(b)}}{=}} \def\eqc{\stackrel{\rm{(c)}}{=}} \def\eqd{\stackrel{\rm{(d)}}{=}} \def\eqe{\stackrel{\rm{(e)}}{=}} \def\Klove{Kl\o ve } \def\proofname{\bf {Proof}} \def\vector#1{\mbox{\boldmath $#1$}} \theoremstyle{definition}\newtheorem{teiri}{Theorem} \newtheorem{giron}{Discussion} \newtheorem{lem}{Lemma} \newtheorem{example}{Example} \newtheorem{remark}{Remark} \newtheorem{pp}{Proposition} \newtheorem{df}{Definition} \newcommand{\dmin}{d_{\mathrm{min}}} \newcommand{\dinf}{d_{\infty}} \def\Prob{P} \ifCLASSINFOpdf \else \hyphenation{op-tical net-works semi-conduc-tor} \begin{document} \title{Recursively Extended Permutation Codes under Chebyshev Distance} \author{Tomoya~Hirobe,~\IEEEmembership{Non-Member,~IEEE,} and~Kenta~Kasai,~\IEEEmembership{Member,~IEEE}\thanks{T. Hirobe was with Department of Information and Communications Engineering, School of Engineering, Tokyo, 152-8550 Japan.} \thanks{K. Kasai was with Department of Information and Communications Engineering, School of Engineering, Tokyo, 152-8550 Japan.} } \markboth{Journal of \LaTeX\ Class Files,~Vol.~14, No.~8, August~2015}{Shell \MakeLowercase{\textit{et al.}}: Bare Demo of IEEEtran.cls for IEEE Journals} \maketitle \begin{abstract} This paper investigates the construction and analysis of permutation codes under the Chebyshev distance. The direct product group permutation (DPGP) codes, introduced independently by \Klove et al. and Tamo et al., represent the best-known permutation codes in terms of both size and minimum distance. These codes possess algebraic structures that facilitate efficient encoding and decoding algorithms. In particular, this study focuses on recursively extended permutation (REP) codes, which were also introduced by \Klove et al. We examine the properties of REP codes and prove that, in terms of size and minimum distance, the optimal REP code is equivalent to the DPGP codes. Furthermore, we present efficient encoding and decoding algorithms for REP codes. \end{abstract} \begin{IEEEkeywords} permutation codes, Chebyshev distance, $\ell_\infty$ distance, recursively extended permutation codes \end{IEEEkeywords} \IEEEpeerreviewmaketitle \input{body} \ifCLASSOPTIONcaptionsoff \newpage \bibliographystyle{IEEEtran} \bibliography{IEEEabrv,../../../literature/00kasai} \end{document} \newcommand\scalemath[2]{\scalebox{#1}{\mbox{\ensuremath{\displaystyle #2}}}} \def\qed{\hfill $\Box$} \def\QED{\hfill $\Box$} \usepackage{bbm} \usepackage{mathrsfs} \makeatletter \newcommand\RedeclareMathOperator{ \@ifstar{\def\rmo@s{m}\rmo@redeclare}{\def\rmo@s{o}\rmo@redeclare}} \newcommand\rmo@redeclare[2]{ \begingroup \escapechar\m@ne\xdef\@gtempa{{\string#1}}\endgroup \expandafter\@ifundefined\@gtempa {\@latex@error{\noexpand#1undefined}\@ehc} \relax \expandafter\rmo@declmathop\rmo@s{#1}{#2}} \newcommand\rmo@declmathop[3]{ \DeclareRobustCommand{#2}{\qopname\newmcodes@#1{#3}}} \@onlypreamble\RedeclareMathOperator \makeatother \def\white#1{\textcolor{white}{#1}} \def\blue#1{\textcolor{blue}{#1}} \def\murasaki#1{\textcolor{darkpastelpurple}{#1}} \definecolor{grey}{rgb}{0.7, 0.75, 0.71} \def\red#1{\textcolor{red}{#1}} \def\grey#1{\textcolor{grey}{#1}} \def\darkyellow#1{\textcolor[rgb]{0.6,0.6,0.0}{#1}} \def\green#1{\textcolor[rgb]{0.0,0.6,0.0}{#1}} \def\dark-red#1{\textcolor[rgb]{0.7,0.0,0.0}{#1}} \def\black#1{\textcolor[rgb]{0.0,0.0,0.0}{#1}} \definecolor{amber}{rgb}{1.0, 0.75, 0.0} \def\yellow#1{\textcolor{amber}{#1}} \def\needcheck#1{{\underline{#1}{\scriptsize \red{(要チェック)}}}} \def\tsum{{\textstyle \sum}} \def\tprod{{\textstyle \prod}} \def\...{\dotsc} \def\length{\mathrm{length}} \def\RM{\mathrm{RM}} \def\APP{\mathrm{APP}} \def\ymg#1{#1}\def\intRp{\int_{0}^{\infty}} \def\intC{\int_{\Cb}} \def\intR{\int_{-\infty}^{\infty}} \def\intT2{\int_{-T/2}^{T/2}} \def\sumion{\sum_{i=1}^{n}} \def\sumizn{\sum_{i=1}^{n}} \def\sumi1n{\sum_{i=1}^{n}} \def\sumi1N{\sum_{i=1}^{N}} \def\sumi0N--{\sum_{i=0}^{N-1}} \def\sumnZ{\sum_{n=-\infty}^{\infty}} \def\summZ{\sum_{m=-\infty}^{\infty}} \def\sumkZ{\sum_{k=-\infty}^{\infty}} \def\vvv{\vdots} \def\ddd{\ddots} \def\ccc{\cdots} \def\lll{\dotsc} \def\stkge#1{\stackrel{{\scriptsize #1}}{\ge}} \def\stkle#1{\stackrel{{\scriptsize #1}}{\le}} \def\stkeq#1{\stackrel{{\scriptsize #1}}{=}} \def\stksim#1{\stackrel{{\scriptsize #1}}{\sim}} \def\stksimeq#1{\stackrel{{\scriptsize #1}}{\simeq}} \def\stkina{\stackrel{\rm{(a)}}{\in}} \def\stkinb{\stackrel{\rm{(b)}}{\in}} \def\stkinc{\stackrel{\rm{(c)}}{\in}} \def\stkind{\stackrel{\rm{(d)}}{\in}} \def\stkine{\stackrel{\rm{(e)}}{\in}} \def\stkinf{\stackrel{\rm{(f)}}{\in}} \def\stking{\stackrel{\rm{(g)}}{\in}} \def\stklongto#1{\stackrel{{\scriptsize #1}}{\longrightarrow}} \def\stkto#1{\stackrel{{\scriptsize #1}}{\to}} \def\stktoa{\stackrel{\rm{(a)}}{\to}} \def\stktob{\stackrel{\rm{(b)}}{\to}} \def\stktoc{\stackrel{\rm{(c)}}{\to}} \def\stktod{\stackrel{\rm{(d)}}{\to}} \def\stktoe{\stackrel{\rm{(e)}}{\to}} \def\stktof{\stackrel{\rm{(f)}}{\to}} \def\stktog{\stackrel{\rm{(g)}}{\to}} \def\stktoh{\stackrel{\rm{(h)}}{\to}} \def\stkeqa{\stackrel{\rm{(a)}}{=}} \def\stkeqb{\stackrel{\rm{(b)}}{=}} \def\stkeqc{\stackrel{\rm{(c)}}{=}} \def\stkeqd{\stackrel{\rm{(d)}}{=}} \def\stkeqe{\stackrel{\rm{(e)}}{=}} \def\stkeqf{\stackrel{\rm{(f)}}{=}} \def\stkeqg{\stackrel{\rm{(g)}}{=}} \def\stklea{\stackrel{\rm{(a)}}{\le}} \def\stkleb{\stackrel{\rm{(b)}}{\le}} \def\stklec{\stackrel{\rm{(c)}}{\le}} \def\stkled{\stackrel{\rm{(d)}}{\le}} \def\stklee{\stackrel{\rm{(e)}}{\le}} \def\stkgea{\stackrel{\rm{(a)}}{\ge}} \def\stkgeb{\stackrel{\rm{(b)}}{\ge}} \def\stkgec{\stackrel{\rm{(c)}}{\ge}} \def\stkged{\stackrel{\rm{(d)}}{\ge}} \def\stkgee{\stackrel{\rm{(e)}}{\ge}} \def\preqa{\stackrel{\rm{(a)}}{=}} \def\preqb{\stackrel{\rm{(b)}}{=}} \def\preqc{\stackrel{\rm{(c)}}{=}} \def\preqd{\stackrel{\rm{(d)}}{=}} \def\preqe{\stackrel{\rm{(e)}}{=}} \def\eqa{\stackrel{\rm{(a)}}{=}} \def\eqb{\stackrel{\rm{(b)}}{=}} \def\eqc{\stackrel{\rm{(c)}}{=}} \def\eqd{\stackrel{\rm{(d)}}{=}} \def\eqe{\stackrel{\rm{(e)}}{=}} \def\simeqa{\stackrel{\rm{(a)}}{\simeq}} \def\simeqb{\stackrel{\rm{(b)}}{\simeq}} \def\simeqc{\stackrel{\rm{(c)}}{\simeq}} \def\simeqd{\stackrel{\rm{(d)}}{\simeq}} \def\simeqe{\stackrel{\rm{(e)}}{\simeq}} \def\ot{\leftarrow} \def\sMM#1#2#3#4{\textstyle{{\scriptsize \biggl(\begin{array}{@{}r@{}r@{}}#1&#2\\#3&#4\end{array}\biggr)}}} \def\MM#1#2#3#4{\begin{pmatrix}#1&#2\\#3&#4\end{pmatrix}} \def\nd#1#2#3{\frac{1}{\sqrt{2\pi#3}}e^{-\frac{(#1-#2)^2}{2#3}}} \def\ND#1#2#3{\frac{1}{\sqrt{2\pi#3}}\exp\Bigl(-\frac{(#1-#2)^2}{2#3}\Bigr)} \def\e{\epsilon} \def\p#1{\frac{\partial}{\partial #1}} \def\P#1#2{\frac{\partial #1}{\partial #2}} \def\d#1{\frac{d}{d #1}} \def\di#1{\frac{d^i}{d #1^i}} \def\dj#1{\frac{d^j}{d #1^j}} \def\dn#1{\frac{d^n}{d #1^n}} \def\D#1#2{\frac{d #1}{d #2}} \def\Di#1#2{\frac{d^i #1}{d #2^i}} \def\Dj#1#2{\frac{d^j #1}{d #2^j}} \def\Dn#1#2{\frac{d^n #1}{d #2^n}} \def\ul#1{{\underline{#1}}} \def\equivdef{\overset{\text{\tiny def}}{\equiv}} \def\egeq{\overset{\text{\scriptsize {e.g.}}}{=}} \def\=def{\overset{\text{\small def}}{=}} \newcommand{\defeq}{\overset{\text{\small def}}{=}} \def\wstar{☆} \def\pra{\rm{(a)}} \def\prb{\rm{(b)}} \def\prc{\rm{(c)}} \def\prd{\rm{(d)}} \def\pre{\rm{(e)}} \def\ta{\mathrm{ta}} \def\out{\mathrm{out}} \def\Vol{\mathrm{Vol}} \def\grad{\mathrm{grad}} \def\TODO{{\tt \red{TODO:}}} \def\WHY{{\tt \red{WHY?}}} \def\todo#1{{\red{{\tt TODO:}#1}}} \def\SOFTMAX{\operatorname{SOFTMAX}} \def\cov{\operatorname{cov}} \def\Var{\operatorname{Var}} \def\var{\operatorname{var}} \def\rect{\operatorname{rect}} \def\sinc{\operatorname{sinc}} \def\sgn{\operatorname{sgn}} \newcommand{\argmax}{\operatornamewithlimits{argmax}} \newcommand{\Ext}{\operatornamewithlimits{Ext}} \newcommand{\Tr}{\operatornamewithlimits{Tr}} \newcommand{\argmin}{\operatornamewithlimits{argmin}} \DeclareMathOperator{\trace}{trace} \DeclareMathOperator{\tr}{tr} \DeclareMathOperator{\ran}{ran} \DeclareMathOperator*{\esssup}{ess\,sup} \DeclareMathOperator*{\essinf}{ess\,inf} \DeclareMathOperator*{\rank}{rank} \DeclareMathOperator*{\diag}{diag} \DeclarePairedDelimiterX{\inp}[2]{\langle}{\rangle}{#1, #2} \def\GL{\mathrm{GL}} \def\SL{\mathrm{SL}} \def\eq{\mathrm{eq}} \def\Unif{\mathrm{Unif}} \def\CN{\mathcal{CN}} \def\Rayleigh{\mathrm{Rayleigh}} \def\Bayes{\mathrm{Bayes}} \def\MSE{\mathrm{MSE}} \def\MMSE{\mathrm{MMSE}} \def\ZF{\mathrm{ZF}} \def\ML{\mathrm{ML}} \def\LP{\mathrm{LP}} \def\BSC{\mathrm{BSC}} \def\MAP{\mathrm{MAP}} \def\BP{\mathrm{BP}} \def\FFT{\mathrm{FFT}} \def\IDFT{\mathrm{IDFT}} \def\DFT{\mathrm{DFT}} \def\otherwise{{\mathrm{otherwise}}} \def\OTHERWISE{{\mathrm{otherwise}}} \def\tOTHERWISE{{\mathrm{otherwise}}} \def\erfc{\mathrm{erfc}} \def\opt{\mathrm{opt}} \def\SNR{\mathrm{SNR}} \def\op#1{||#1||_\mathrm{op}} \def\Bigfloor#1{\Big\lfloor #1\Big\rfloor} \def\bigfloor#1{\big\lfloor #1\big\rfloor} \def\floor#1{\lfloor #1\rfloor} \def\Bigceil#1{\Big\lceil #1\Big\rceil} \def\bigceil#1{\big\lceil #1\big\rceil} \def\ceil#1{\lceil #1\rceil} \def\af{\mathfrak{A}} \def\Af{\mathfrak{A}} \def\Bf{\mathfrak{B}} \def\Cf{\mathfrak{C}} \def\Df{\mathfrak{D}} \def\Ef{\mathfrak{E}} \def\Ff{\mathfrak{F}} \def\Gf{\mathfrak{G}} \def\Hf{\mathfrak{H}} \def\Jf{\mathfrak{J}} \def\Kf{\mathfrak{K}} \def\Lf{\mathfrak{L}} \def\Mf{\mathfrak{M}} \def\Nf{\mathfrak{N}} \def\Of{\mathfrak{O}} \def\Pf{\mathfrak{P}} \def\Qf{\mathfrak{Q}} \def\Rf{\mathfrak{R}} \def\Sf{\mathfrak{S}} \def\Tf{\mathfrak{T}} \def\Uf{\mathfrak{U}} \def\Vf{\mathfrak{V}} \def\Wf{\mathfrak{W}} \def\Xf{\mathfrak{X}} \def\Yf{\mathfrak{Y}} \def\Zf{\mathfrak{Z}} \def\Ab{\mathbb{A}} \def\Bb{\mathbb{B}} \def\Cb{\mathbb{C}} \def\Db{\mathbb{D}} \def\Eb{\mathbb{E}} \def\Fb{\mathbb{F}} \def\Gb{\mathbb{G}} \def\Hb{\mathbb{H}} \def\Ib{\mathbb{I}} \def\Jb{\mathbb{J}} \def\Kb{\mathbb{K}} \def\Lb{\mathbb{L}} \def\Mb{\mathbb{M}} \def\Nb{\mathbb{N}} \def\Ob{\mathbb{O}} \def\Pb{\mathbb{P}} \def\Qb{\mathbb{Q}} \def\Rb{\mathbb{R}} \def\Sb{\mathbb{S}} \def\Tb{\mathbb{T}} \def\Ub{\mathbb{U}} \def\Vb{\mathbb{V}} \def\Wb{\mathbb{W}} \def\Xb{\mathbb{X}} \def\Yb{\mathbb{Y}} \def\Zb{\mathbb{Z}} \def\Ac{\mathcal{A}} \def\Bc{\mathcal{B}} \def\Cc{\mathcal{C}} \def\Dc{\mathcal{D}} \def\Ec{\mathcal{E}} \def\Fc{\mathcal{F}} \def\Gc{\mathcal{G}} \def\Hc{\mathcal{H}} \def\Ic{\mathcal{I}} \def\Jc{\mathcal{J}} \def\Kc{\mathcal{K}} \def\Lc{\mathcal{L}} \def\Mc{\mathcal{M}} \def\CNc{\mathcal{CN}} \def\Nc{\mathcal{N}} \def\Oc{\mathcal{O}} \def\Pc{\mathcal{P}} \def\Qc{\mathcal{Q}} \def\Rc{\mathcal{R}} \def\Sc{\mathcal{S}} \def\Tc{\mathcal{T}} \def\Uc{\mathcal{U}} \def\Vc{\mathcal{V}} \def\Wc{\mathcal{W}} \def\Xc{\mathcal{X}} \def\Yc{\mathcal{Y}} \def\Zc{\mathcal{Z}} \def\K{\mathbb{K}} \def\<{\langle} \def\>{\rangle} \def\K{\mathbb{K}} \def\spn{\mathrm{span}} \def\SPAN{\mathrm{span}} \def\tatevector#1#2{\left(\begin{array}{c}#1\\\vdots\\#2\end{array}\right)} \def\mat4#1#2#3#4{ \begin{pmatrix} #1&\ccc&#2\\ \vdots&&\vdots\\ #3&\ccc&#4 \end{pmatrix}} \def\Mab#1#2{\left(\begin{array}{cc}#1&#2\end{array}\right)} \def\Mac#1#2{\left(\begin{array}{c}#1\\#2\end{array}\right)} \def\0sf{\mathsf{0}} \def\1sf{\mathsf{1}} \def\asf{\mathsf{a}} \def\bsf{\mathsf{b}} \def\csf{\mathsf{c}} \def\dsf{\mathsf{d}} \def\esf{\mathsf{e}} \def\fsf{\mathsf{f}} \def\gsf{\mathsf{g}} \def\hsf{\mathsf{h}} \def\isf{\mathsf{i}} \def\jsf{\mathsf{j}} \def\ksf{\mathsf{k}} \def\lsf{\mathsf{l}} \def\msf{\mathsf{m}} \def\nsf{\mathsf{n}} \def\osf{\mathsf{o}} \def\psf{\mathsf{p}} \def\qsf{\mathsf{q}} \def\rsf{\mathsf{r}} \def\ssf{\mathsf{s}} \def\tsf{\mathsf{t}} \def\usf{\mathsf{u}} \def\vsf{\mathsf{v}} \def\wsf{\mathsf{w}} \def\xsf{\mathsf{x}} \def\ysf{\mathsf{y}} \def\zsf{\mathsf{z}} \def\Asf{\mathsf{A}} \def\Bsf{\mathsf{B}} \def\Csf{\mathsf{C}} \def\Dsf{\mathsf{D}} \def\Esf{\mathsf{E}} \def\Fsf{\mathsf{F}} \def\Gsf{\mathsf{G}} \def\sfsf{\mathsf{sf}} \def\Isf{\mathsf{I}} \def\Jsf{\mathsf{J}} \def\Ksf{\mathsf{K}} \def\Lsf{\mathsf{L}} \def\Msf{\mathsf{M}} \def\Nsf{\mathsf{N}} \def\Osf{\mathsf{O}} \def\Psf{\mathsf{P}} \def\Qsf{\mathsf{Q}} \def\Rsf{\mathsf{R}} \def\Ssf{\mathsf{S}} \def\Tsf{\mathsf{T}} \def\Usf{\mathsf{U}} \def\Vsf{\mathsf{V}} \def\Wsf{\mathsf{W}} \def\Xsf{\mathsf{X}} \def\Ysf{\mathsf{Y}} \def\Zsf{\mathsf{Z}} \def\arm{\mathrm{a}} \def\brm{\mathrm{b}} \def\crm{\mathrm{c}} \def\drm{\mathrm{d}} \def\erm{\mathrm{e}} \def\frm{\mathrm{f}} \def\grm{\mathrm{g}} \def\hrm{\mathrm{h}} \def\irm{\mathrm{i}} \def\jrm{\mathrm{j}} \def\krm{\mathrm{k}} \def\lrm{\mathrm{l}} \def\mrm{\mathrm{m}} \def\nrm{\mathrm{n}} \def\orm{\mathrm{o}} \def\prm{\mathrm{p}} \def\qrm{\mathrm{q}} \def\rrm{\mathrm{r}} \def\srm{\mathrm{s}} \def\trm{\mathrm{t}} \def\urm{\mathrm{u}} \def\vrm{\mathrm{v}} \def\wrm{\mathrm{w}} \def\xrm{\mathrm{x}} \def\yrm{\mathrm{y}} \def\zrm{\mathrm{z}} \def\Arm{\mathrm{A}} \def\Brm{\mathrm{B}} \def\Crm{\mathrm{C}} \def\Drm{\mathrm{D}} \def\Erm{\mathrm{E}} \def\Frm{\mathrm{F}} \def\Grm{\mathrm{G}} \def\Hrm{\mathrm{H}} \def\Irm{\mathrm{I}} \def\Jrm{\mathrm{J}} \def\Krm{\mathrm{K}} \def\Lrm{\mathrm{L}} \def\Mrm{\mathrm{M}} \def\Nrm{\mathrm{N}} \def\Orm{\mathrm{O}} \def\Prm{\mathrm{P}} \def\Qrm{\mathrm{Q}} \def\Rrm{\mathrm{R}} \def\Srm{\mathrm{S}} \def\Trm{\mathrm{T}} \def\Urm{\mathrm{U}} \def\Vrm{\mathrm{V}} \def\Wrm{\mathrm{W}} \def\Xrm{\mathrm{X}} \def\Yrm{\mathrm{Y}} \def\Zrm{\mathrm{Z}} \def\0BS{\boldsymbol{0}} \def\1BS{\boldsymbol{1}} \def\aBS{\boldsymbol{a}} \def\bBS{\boldsymbol{b}} \def\cBS{\boldsymbol{c}} \def\dBS{\boldsymbol{d}} \def\eBS{\boldsymbol{e}} \def\fBS{\boldsymbol{f}} \def\gBS{\boldsymbol{g}} \def\hBS{\boldsymbol{h}} \def\iBS{\boldsymbol{i}} \def\jBS{\boldsymbol{j}} \def\kBS{\boldsymbol{k}} \def\lBS{\boldsymbol{l}} \def\mBS{\boldsymbol{m}} \def\nBS{\boldsymbol{n}} \def\oBS{\boldsymbol{o}} \def\pBS{\boldsymbol{p}} \def\qBS{\boldsymbol{q}} \def\rBS{\boldsymbol{r}} \def\sBS{\boldsymbol{s}} \def\tBS{\boldsymbol{t}} \def\uBS{\boldsymbol{u}} \def\vBS{\boldsymbol{v}} \def\wBS{\boldsymbol{w}} \def\xBS{\boldsymbol{x}} \def\yBS{\boldsymbol{y}} \def\zBS{\boldsymbol{z}} \def\ABS{\boldsymbol{A}} \def\BBS{\boldsymbol{B}} \def\CBS{\boldsymbol{C}} \def\DBS{\boldsymbol{D}} \def\EBS{\boldsymbol{E}} \def\FBS{\boldsymbol{F}} \def\GBS{\boldsymbol{G}} \def\HBS{\boldsymbol{H}} \def\IBS{\boldsymbol{I}} \def\JBS{\boldsymbol{J}} \def\KBS{\boldsymbol{K}} \def\LBS{\boldsymbol{L}} \def\MBS{\boldsymbol{M}} \def\NBS{\boldsymbol{N}} \def\OBS{\boldsymbol{O}} \def\PBS{\boldsymbol{P}} \def\QBS{\boldsymbol{Q}} \def\RBS{\boldsymbol{R}} \def\SBS{\boldsymbol{S}} \def\TBS{\boldsymbol{T}} \def\UBS{\boldsymbol{U}} \def\VBS{\boldsymbol{V}} \def\WBS{\boldsymbol{W}} \def\XBS{\boldsymbol{X}} \def\YBS{\boldsymbol{Y}} \def\ZBS{\boldsymbol{Z}} \def\0B{\mathbf{0}} \def\1B{\mathbf{1}} \def\aB{\mathbf{a}} \def\bB{\mathbf{b}} \def\cB{\mathbf{c}} \def\dB{\mathbf{d}} \def\eB{\mathbf{e}} \def\fB{\mathbf{f}} \def\gB{\mathbf{g}} \def\hB{\mathbf{h}} \def\iB{\mathbf{i}} \def\jB{\mathbf{j}} \def\kB{\mathbf{k}} \def\lB{\mathbf{l}} \def\mB{\mathbf{m}} \def\nB{\mathbf{n}} \def\oB{\mathbf{o}} \def\pB{\mathbf{p}} \def\qB{\mathbf{q}} \def\rB{\mathbf{r}} \def\sB{\mathbf{s}} \def\tB{\mathbf{t}} \def\uB{\mathbf{u}} \def\vB{\mathbf{v}} \def\wB{\mathbf{w}} \def\xB{\mathbf{x}} \def\yB{\mathbf{y}} \def\zB{\mathbf{z}} \def\AB{\mathbf{A}} \def\BB{\mathbf{B}} \def\CB{\mathbf{C}} \def\DB{\mathbf{D}} \def\EB{\mathbf{E}} \def\FB{\mathbf{F}} \def\GB{\mathbf{G}} \def\HB{\mathbf{H}} \def\IB{\mathbf{I}} \def\JB{\mathbf{J}} \def\KB{\mathbf{K}} \def\LB{\mathbf{L}} \def\MB{\mathbf{M}} \def\NB{\mathbf{N}} \def\OB{\mathbf{O}} \def\PB{\mathbf{P}} \def\QB{\mathbf{Q}} \def\RB{\mathbf{R}} \def\SB{\mathbf{S}} \def\TB{\mathbf{T}} \def\UB{\mathbf{U}} \def\VB{\mathbf{V}} \def\WB{\mathbf{W}} \def\XB{\mathbf{X}} \def\YB{\mathbf{Y}} \def\ZB{\mathbf{Z}} \def\0H{\hat{0}} \def\1H{\hat{1}} \def\aH{\hat{a}} \def\bH{\hat{b}} \def\cH{\hat{c}} \def\dH{\hat{d}} \def\eH{\hat{e}} \def\fH{\hat{f}} \def\gH{\hat{g}} \def\hH{\hat{h}} \def\iH{\hat{i}} \def\jH{\hat{j}} \def\kH{\hat{k}} \def\lH{\hat{l}} \def\mH{\hat{m}} \def\nH{\hat{n}} \def\oH{\hat{o}} \def\pH{\hat{p}} \def\qH{\hat{q}} \def\rH{\hat{r}} \def\sH{\hat{s}} \def\tH{\hat{t}} \def\uH{\hat{u}} \def\vH{\hat{v}} \def\wH{\hat{w}} \def\xH{\hat{x}} \def\yH{\hat{y}} \def\zH{\hat{z}} \def\AH{\hat{A}} \def\BH{\hat{B}} \def\CH{\hat{C}} \def\DH{\hat{D}} \def\EH{\hat{E}} \def\FH{\hat{F}} \def\GH{\hat{G}} \def\HH{\hat{H}} \def\IH{\hat{I}} \def\JH{\hat{J}} \def\KH{\hat{K}} \def\LH{\hat{L}} \def\MH{\hat{M}} \def\NH{\hat{N}} \def\OH{\hat{O}} \def\PH{\hat{P}} \def\QH{\hat{Q}} \def\RH{\hat{R}} \def\SH{\hat{S}} \def\TH{\hat{T}} \def\UH{\hat{U}} \def\VH{\hat{V}} \def\WH{\hat{W}} \def\XH{\hat{X}} \def\YH{\hat{Y}} \def\ZH{\hat{Z}} \def\uaQz{|\uparrow_z\>} \def\daQz{|\downarrow_z\>} \def\uaQx{|\uparrow_x\>} \def\daQx{|\downarrow_x\>} \def\uaQy{|\uparrow_y\>} \def\daQy{|\downarrow_y\>} \def\uaQ{|\uparrow\>} \def\daQ{|\downarrow\>} \def\uaQzKB{|\uparrow_z\> \<\uparrow_z|} \def\uaQxKB{|\uparrow_x\> \<\uparrow_x|} \def\daQzKB{|\downarrow_z\> \<\downarrow_z|} \def\daQxKB{|\downarrow_x\> \<\downarrow_x|} \def\+TT{\texttt{+}} \def\-{\texttt{-}} \def\+KB{|+\> \<+|} \def\-KB{|-\> \<-|} \def\psiKB{|\psi\> \<\psi|} \def\aKB{|a\> \<a|} \def\iKB{|i\> \<i|} \def\aQ{|a\>} \def\q0{|0\>} \def\iQ{|i\>} \def\joQ{|j\>} \def\psiQ{|\psi\>} \def\phiQ{|\phi\>} \def\pheq{\phantom{=}} \def\alphaH{\hat{\alpha}} \def\betaH{\hat{\beta}} \def\gammaH{\hat{\gamma}} \def\deltaH{\hat{\delta}} \def\epsilonH{\hat{\epsilon}} \def\zetaH{\hat{\zeta}} \def\etaH{\hat{\eta}} \def\thetaH{\hat{\theta}} \def\iotaH{\hat{\iota}} \def\kappaH{\hat{\kappa}} \def\lambdaH{\hat{\lambda}} \def\muH{\hat{\mu}} \def\nuH{\hat{\nu}} \def\xiH{\hat{\xi}} \def\piH{\hat{\pi}} \def\rhoH{\hat{\rho}} \def\sigmaH{\hat{\sigma}} \def\tauH{\hat{\tau}} \def\upsilonH{\hat{\upsilon}} \def\phiH{\hat{\phi}} \def\chiH{\hat{\chi}} \def\psiH{\hat{\psi}} \def\omegaH{\hat{\omega}} \def\GammaH{\hat{\Gamma}} \def\DeltaH{\hat{\Delta}} \def\ThetaH{\hat{\Theta}} \def\LambdaH{\hat{\Lambda}} \def\XiH{\hat{\Xi}} \def\PiH{\hat{\Pi}} \def\SigmaH{\hat{\Sigma}} \def\UpsilonH{\hat{\Upsilon}} \def\PhiH{\hat{\Phi}} \def\PsiH{\hat{\Psi}} \def\OmegaH{\hat{\Omega}} \def\varepsilonH{\hat{\varepsilon}} \def\varthetaH{\hat{\vartheta}} \def\varpiH{\hat{\varpi}} \def\varrhoH{\hat{\varrho}} \def\varsigmaH{\hat{\varsigma}} \def\varphiH{\hat{varphi}} \def\0U{\underline{0}} \def\1U{\underline{1}} \def\aU{\underline{a}} \def\bU{\underline{b}} \def\cU{\underline{c}} \def\dU{\underline{d}} \def\eU{\underline{e}} \def\fU{\underline{f}} \def\gU{\underline{g}} \def\hU{\underline{h}} \def\iU{\underline{i}} \def\jU{\underline{j}} \def\kU{\underline{k}} \def\lU{\underline{l}} \def\mU{\underline{m}} \def\nU{\underline{n}} \def\oU{\underline{o}} \def\pU{\underline{p}} \def\qU{\underline{q}} \def\rU{\underline{r}} \def\sU{\underline{s}} \def\tU{\underline{t}} \def\uU{\underline{u}} \def\vU{\underline{v}} \def\wU{\underline{w}} \def\xU{\underline{x}} \def\yU{\underline{y}} \def\zU{\underline{z}} \def\aV{\vec{a}} \def\bV{\vec{b}} \def\cV{\vec{c}} \def\dV{\vec{d}} \def\eV{\vec{e}} \def\fV{\vec{f}} \def\gV{\vec{g}} \def\hV{\vec{h}} \def\iV{\vec{i}} \def\jV{\vec{j}} \def\kV{\vec{k}} \def\lV{\vec{l}} \def\mV{\vec{m}} \def\nV{\vec{n}} \def\oV{\vec{o}} \def\pV{\vec{p}} \def\qV{\vec{q}} \def\rV{\vec{r}} \def\sV{\vec{s}} \def\tV{\vec{t}} \def\uV{\vec{u}} \def\vV{\vec{v}} \def\wV{\vec{w}} \def\xV{\vec{x}} \def\yV{\vec{y}} \def\zV{\vec{z}} \def\AV{\vec{A}} \def\BV{\vec{B}} \def\CV{\vec{C}} \def\DV{\vec{D}} \def\EV{\vec{E}} \def\FV{\vec{F}} \def\GV{\vec{G}} \def\HV{\vec{H}} \def\IV{\vec{I}} \def\JV{\vec{J}} \def\KV{\vec{K}} \def\LV{\vec{L}} \def\MV{\vec{M}} \def\NV{\vec{N}} \def\OV{\vec{O}} \def\PV{\vec{P}} \def\QV{\vec{Q}} \def\RV{\vec{R}} \def\SV{\vec{S}} \def\TV{\vec{T}} \def\UV{\vec{U}} \def\VV{\vec{V}} \def\WV{\vec{W}} \def\XV{\vec{X}} \def\YV{\vec{Y}} \def\ZV{\vec{Z}} \def\aBF{\mathbf{a}} \def\bBF{\mathbf{b}} \def\cBF{\mathbf{c}} \def\dBF{\mathbf{d}} \def\eBF{\mathbf{e}} \def\fBF{\mathbf{f}} \def\gBF{\mathbf{g}} \def\hBF{\mathbf{h}} \def\iBF{\mathbf{i}} \def\jBF{\mathbf{j}} \def\kBF{\mathbf{k}} \def\lBF{\mathbf{l}} \def\mBF{\mathbf{m}} \def\nBF{\mathbf{n}} \def\oBF{\mathbf{o}} \def\pBF{\mathbf{p}} \def\qBF{\mathbf{q}} \def\rBF{\mathbf{r}} \def\sBF{\mathbf{s}} \def\tBF{\mathbf{t}} \def\uBF{\mathbf{u}} \def\vBF{\mathbf{v}} \def\wBF{\mathbf{w}} \def\xBF{\mathbf{x}} \def\yBF{\mathbf{y}} \def\zBF{\mathbf{z}} \def\ABF{\mathbf{A}} \def\BBF{\mathbf{B}} \def\CBF{\mathbf{C}} \def\DBF{\mathbf{D}} \def\EBF{\mathbf{E}} \def\FBF{\mathbf{F}} \def\GBF{\mathbf{G}} \def\HBF{\mathbf{H}} \def\IBF{\mathbf{I}} \def\JBF{\mathbf{J}} \def\KBF{\mathbf{K}} \def\LBF{\mathbf{L}} \def\MBF{\mathbf{M}} \def\NBF{\mathbf{N}} \def\OBF{\mathbf{O}} \def\PBF{\mathbf{P}} \def\QBF{\mathbf{Q}} \def\RBF{\mathbf{R}} \def\SBF{\mathbf{S}} \def\TBF{\mathbf{T}} \def\UBF{\mathbf{U}} \def\VBF{\mathbf{V}} \def\WBF{\mathbf{W}} \def\XBF{\mathbf{X}} \def\YBF{\mathbf{Y}} \def\ZBF{\mathbf{Z}} \def\aT{\tilde{a}} \def\bT{\tilde{b}} \def\cT{\tilde{c}} \def\dT{\tilde{d}} \def\eT{\tilde{e}} \def\fT{\tilde{f}} \def\gT{\tilde{g}} \def\hT{\tilde{h}} \def\iT{\tilde{i}} \def\jT{\tilde{j}} \def\kT{\tilde{k}} \def\lT{\tilde{l}} \def\mT{\tilde{m}} \def\nT{\tilde{n}} \def\oT{\tilde{o}} \def\pT{\tilde{p}} \def\qT{\tilde{q}} \def\rT{\tilde{r}} \def\sT{\tilde{s}} \def\tT{\tilde{t}} \def\uT{\tilde{u}} \def\vT{\tilde{v}} \def\wT{\tilde{w}} \def\xT{\tilde{x}} \def\yT{\tilde{y}} \def\zT{\tilde{z}} \def\AT{\tilde{A}} \def\BT{\tilde{B}} \def\CT{\tilde{C}} \def\DT{\tilde{D}} \def\ET{\tilde{E}} \def\FT{\tilde{F}} \def\GT{\tilde{G}} \def\HT{\tilde{H}} \def\IT{\tilde{I}} \def\JT{\tilde{J}} \def\KT{\tilde{K}} \def\LT{\tilde{L}} \def\MT{\tilde{M}} \def\NT{\tilde{N}} \def\OT{\tilde{O}} \def\PT{\tilde{P}} \def\QT{\tilde{Q}} \def\RT{\tilde{R}} \def\ST{\tilde{S}} \def\TT{\tilde{T}} \def\UT{\tilde{U}} \def\VT{\tilde{V}} \def\WT{\tilde{W}} \def\XT{\tilde{X}} \def\YT{\tilde{Y}} \def\ZT{\tilde{Z}} \def\alphaT{\tilde{\alpha}} \def\betaT{\tilde{\beta}} \def\gammaT{\tilde{\gamma}} \def\deltaT{\tilde{\delta}} \def\epsilonT{\tilde{\epsilon}} \def\zetaT{\tilde{\zeta}} \def\etaT{\tilde{\eta}} \def\thetaT{\tilde{\theta}} \def\iotaT{\tilde{\iota}} \def\kappaT{\tilde{\kappa}} \def\lambdaT{\tilde{\lambda}} \def\muT{\tilde{\mu}} \def\nuT{\tilde{\nu}} \def\xiT{\tilde{\xi}} \def\piT{\tilde{\pi}} \def\rhoT{\tilde{\rho}} \def\sigmaT{\tilde{\sigma}} \def\tauT{\tilde{\tau}} \def\upsilonT{\tilde{\upsilon}} \def\phiT{\tilde{\phi}} \def\chiT{\tilde{\chi}} \def\psiT{\tilde{\psi}} \def\omegaT{\tilde{\omega}} \def\GammaT{\tilde{\Gamma}} \def\DeltaT{\tilde{\Delta}} \def\ThetaT{\tilde{\Theta}} \def\LambdaT{\tilde{\Lambda}} \def\XiT{\tilde{\Xi}} \def\PiT{\tilde{\Pi}} \def\SigmaT{\tilde{\Sigma}} \def\UpsilonT{\tilde{\Upsilon}} \def\PhiT{\tilde{\Phi}} \def\PsiT{\tilde{\Psi}} \def\OmegaT{\tilde{\Omega}} \def\varepsilonT{\tilde{\varepsilon}} \def\varthetaT{\tilde{\vartheta}} \def\varpiT{\tilde{\varpi}} \def\varrhoT{\tilde{\varrho}} \def\varsigmaT{\tilde{\varsigma}} \def\varphiT{\tilde{varphi}} \def\JOc{\overline{{\mathcal{J}}}} \def\aO{\overline{a}} \def\bO{\overline{b}} \def\cO{\overline{c}} \def\dO{\overline{d}} \def\eO{\overline{e}} \def\fO{\overline{f}} \def\gO{\overline{g}} \def\hO{\overline{h}} \def\iO{\overline{i}} \def\jO{\overline{j}} \def\kO{\overline{k}} \def\lO{\overline{l}} \def\mO{\overline{m}} \def\nO{\overline{n}} \def\oO{\overline{o}} \def\pO{\overline{p}} \def\qO{\overline{q}} \def\rO{\overline{r}} \def\sO{\overline{s}} \def\tO{\overline{t}} \def\uO{\overline{u}} \def\vO{\overline{v}} \def\wO{\overline{w}} \def\xO{\overline{x}} \def\yO{\overline{y}} \def\zO{\overline{z}} \def\AO{\overline{A}} \def\BO{\overline{B}} \def\CO{\overline{C}} \def\DO{\overline{D}} \def\EO{\overline{E}} \def\FO{\overline{F}} \def\GO{\overline{G}} \def\HO{\overline{H}} \def\IO{\overline{I}} \def\JO{\overline{J}} \def\KO{\overline{K}} \def\LO{\overline{L}} \def\MO{\overline{M}} \def\NO{\overline{N}} \def\OO{\overline{O}} \def\PO{\overline{P}} \def\QO{\overline{Q}} \def\RO{\overline{R}} \def\SO{\overline{S}} \def\UO{\overline{U}} \def\VO{\overline{V}} \def\WO{\overline{W}} \def\XO{\overline{X}} \def\YO{\overline{Y}} \def\ZO{\overline{Z}} \def\IMIFU{\red{\ovalbox{意味不明}}} \def\alphaO{\overline{\alpha}} \def\thetaO{\overline{\theta}} \def\tauO{\overline{\tau}} \def\betaO{\overline{\beta}} \def\varthetaO{\overline{\vartheta}} \def\piO{\overline{\pi}} \def\upsilonO{\overline{\upsilon}} \def\gammaO{\overline{\gamma}} \def\varpiO{\overline{\varpi}} \def\phiO{\overline{\phi}} \def\deltaO{\overline{\delta}} \def\kappaO{\overline{\kappa}} \def\rhoO{\overline{\rho}} \def\varphiO{\overline{\varphi}} \def\epsilonO{\overline{\epsilon}} \def\lambdaO{\overline{\lambda}} \def\varrhoO{\overline{\varrho}} \def\chiO{\overline{\chi}} \def\varepsilonO{\overline{\varepsilon}} \def\muO{\overline{\mu}} \def\sigmaO{\overline{\sigma}} \def\psiO{\overline{\psi}} \def\zetaO{\overline{\zeta}} \def\nuO{\overline{\nu}} \def\varsigmaO{\overline{\varsigma}} \def\omegaO{\overline{\omega}} \def\etaO{\overline{\eta}} \def\xiO{\overline{\xi}} \def\GammaO{\overline{\Gamma}} \def\LambdaO{\overline{\Lambda}} \def\SigmaO{\overline{\Sigma}} \def\PsiO{\overline{\Psi}} \def\DeltaO{\overline{\Delta}} \def\UpsilonO{\overline{\Upsilon}} \def\OmegaO{\overline{\Omega}} \def\ThetaO{\overline{\Theta}} \def\PiO{\overline{\Pi}} \def\PhiO{\overline{\Phi}} \def\RhoO{\overline{\Rho}} \def\alphaU{\underline{\alpha}} \def\iotaU{\underline{\iota}} \def\thetaU{\underline{\theta}} \def\tauU{\underline{\tau}} \def\betaU{\underline{\beta}} \def\varthetaU{\underline{\vartheta}} \def\piU{\underline{\pi}} \def\upsilonU{\underline{\upsilon}} \def\gammaU{\underline{\gamma}} \def\gammaU{\underline{\gamma}} \def\varpiU{\underline{\varpi}} \def\phiU{\underline{\phi}} \def\deltaU{\underline{\delta}} \def\kappaU{\underline{\kappa}} \def\rhoU{\underline{\rho}} \def\varphiU{\underline{\varphi}} \def\epsilonU{\underline{\epsilon}} \def\lambdaU{\underline{\lambda}} \def\varrhoU{\underline{\varrho}} \def\chiU{\underline{\chi}} \def\varepsilonU{\underline{\varepsilon}} \def\muU{\underline{\mu}} \def\sigmaU{\underline{\sigma}} \def\psiU{\underline{\psi}} \def\zetaU{\underline{\zeta}} \def\nuU{\underline{\nu}} \def\varsigmaU{\underline{\varsigma}} \def\omegaU{\underline{\omega}} \def\etaU{\underline{\eta}} \def\xiU{\underline{\xi}} \def\GammaU{\underline{\Gamma}} \def\LambdaU{\underline{\Lambda}} \def\SigmaU{\underline{\Sigma}} \def\PsiU{\underline{\Psi}} \def\DeltaU{\underline{\Delta}} \def\UpsilonU{\underline{\Upsilon}} \def\OmegaU{\underline{\Omega}} \def\ThetaU{\underline{\Theta}} \def\PiU{\underline{\Pi}} \def\PhiU{\underline{\Phi}} \def\RhoU{\underline{\Rho}} \def\AU{\underline{A}} \def\BU{\underline{B}} \def\CU{\underline{C}} \def\DU{\underline{D}} \def\EU{\underline{E}} \def\FU{\underline{F}} \def\GU{\underline{G}} \def\HU{\underline{H}} \def\IU{\underline{I}} \def\JU{\underline{J}} \def\KU{\underline{K}} \def\LU{\underline{L}} \def\MU{\underline{M}} \def\NU{\underline{N}} \def\OU{\underline{O}} \def\PU{\underline{P}} \def\QU{\underline{Q}} \def\RU{\underline{R}} \def\SU{\underline{S}} \def\TU{\underline{T}} \def\UU{\underline{U}} \def\VU{\underline{V}} \def\WU{\underline{W}} \def\XU{\underline{X}} \def\YU{\underline{Y}} \def\ZU{\underline{Z}} \def\St{\mathtt{S}} \def\ct{\mathtt{c}} \def\vt{\mathtt{v}} \def\ub{\underbrace} \def\ob{\overbrace} \def\tIMPLIES{\mbox{ implies }} \def\tOF{\mbox{ of }} \def\tST{\mbox{ s.t. }} \def\tTRUE{\mbox{true}} \def\tFOR{\mbox{ for }} \def\tMINIMIZE{\mbox{minimize }} \def\tWITH{\mbox{ with }} \def\tFORALL{\mbox{ for all }} \def\tFORANY{\mbox{ for any }} \def\tFORSOME{\mbox{ for some }} \def\tAND{\mbox{ and }} \def\tOR{\mbox{ or }} \def\tIF{\mbox{ if }} \def\tIFF{\mbox{ iff }} \def\tST{\mbox{ subject to }} \def\tSUCHTHAT{\mbox{ such that }} \def\tTHEN{\mbox{ then }} \def\tOUTPUT{\mbox{ output }} \def\ttrue{\rm{true}} \def\OF{\mbox{ of }} \def\ST{\mbox{ s.t. }} \def\MINIMIZE{\mbox{minimize }} \def\WITH{\mbox{ with }} \def\FORANY{\mbox{ for any }} \def\FORSOME{\mbox{ for some }} \def\IFF{\mbox{ iff }} \def\ST{\mbox{ subject to }} \def\SUCHTHAT{\mbox{ such that }} \def\THEN{\mbox{ then }} \def\OUTPUT{\mbox{ output }} \def\true{\rm{true}} \def\pack{\mathrm{pack}} \def\cov{\mathrm{cov}} \def\eff{\mathrm{eff}} \def\LLR{\mathrm{LLR}} \def\RHS{\mathrm{RHS}} \def\LHS{\mathrm{LHS}} \def\Rho{P} \def\v{\mathtt{v}} \def\c{\mathtt{c}} \def\Id{\mathrm{Id}} \def\Aut{\mathrm{Aut}} \def\End{\mathrm{End}} \def\lcm{\mathrm{lcm}} \def\coef{\mathrm{coef}} \def\ord{\mathrm{ord}} \def\Int{\mathrm{Int}} \def\Re{\mathrm{Re}\,} \RedeclareMathOperator{\Im}{Im} \DeclareMathOperator{\Ker}{Ker} \def\cl{\mathrm{Cl}} \def\ann{\mathrm{ann}} \def\dom{\mathrm{Dom}} \def\de{\mathrm{de}} \def\ad{\mathrm{ad}} \def\rank{\mathrm{rank}} \def\diag{\mathrm{diag}} \def\floor#1{\lfloor #1\rfloor} \def\op#1{||#1||_\mathrm{op}} \def\y{\underline{y}} \def\s{\underline{s}} \def\x{\underline{x}} \def\ES{\mathcal{S}} \def\Y{\mathcal{Y}} \def\d{\mathrm{d}} \def\H{\mathcal{H}} \def\O{\mathcal{O}} \def\B{\mathcal{B}} \def\N{\mathbb{N}} \def\R{\mathbb{R}} \def\Q{\mathbb{Q}} \def\Z{\mathbb{Z}} \def\GF{\mathbb{F}} \def\E{\mathbb{E}} \def\C{\mathbb{C}} \def\vx{\vec{x}} \def\vh{\vec{h}} \def\vy{\vec{y}} \def\vz{\vec{z}} \def\vs{\vec{s}} \def\dl{\mathrm{l}} \def\dr{\mathrm{r}} \def\dg{\mathrm{g}} \def\cw{\mathrm{cw}} \def\ss{\boxast} \def\cs{\oast} \def\G{\mathcal{G}} \def\A{\mathcal{A}} \newcommand{\error}{\mathrm{error}} \def\thepage{\arabic{page}} \newcommand{\T}{\mathsf{T}} \newcommand{\I}{\mathbbm{1}} \section{Introduction} In this paper, we explore the subject of {\itshape permutation codes}, which are subsets of all permutations of a fixed length \(n\). The concept of permutation codes originated in the 1960s \cite{1445610}. Vinck et al. later applied permutation codes to power-line communication and \(m\)-ary frequency shift keying (FSK) modulation systems \cite{Vinck2000,866429}, renewing interest in permutation codes \cite{BLAKE19791,1302307,Swart2007}. In \(m\)-ary FSK systems, individual frequencies are assigned to time slots to represent permutation symbols. The use of time and frequency diversity helps reduce the impact of various types of noise, such as background noise, impulse noise, and persistent frequency interference commonly seen in power-line communication systems. For multilevel flash memory applications, the \(\ell_\infty\) norm, known as the Chebyshev distance, is effective for managing issues related to recharging and error correction. Among the distance metrics employed for permutation codes, Chebyshev distance has been thoroughly examined, covering aspects like the Gilbert–Varshamov bound and ball-packing bound \cite{5466536,7342968,7908949}, efficient encoding and decoding algorithms \cite{5466546,5466536}, and systematic code construction methods \cite{8648459,6937135}. \Klove et al. \cite[Sec.~III.A]{5466546} and Tamo et al. \cite[\black{Construction 1}]{5466536} independently introduced a construction of permutation codes based on the Chebyshev distance. In \cite{5466536}, the coordinates are partitioned into \(\mathbb{Z}/d\mathbb{Z}\), and the construction is viewed as a direct product of sub-groups over the symmetric group \(\mathcal{S}_n\), with \(d\) symmetric groups acting as constituent groups. Based on this framework, these codes are termed direct product group permutation (DPGP) codes in this paper. Efficient algebraic encoding and decoding algorithms for DPGP codes have been proposed \cite{5466536,5466546}. DPGP codes demonstrate strong asymptotic normalized minimum distance for permutation codes. As far as the authors are aware, DPGP codes provide the largest code size for a given code length and minimum distance \cite[Fig.~1]{5466536}, except for codes derived using the methods from the Gilbert–Varshamov (GV) bound proof \cite[Thm.~26]{5466536} and short-length codes obtained through greedy algorithms \cite[Sec.~IV.B]{5466546}. DPGP codes form the foundation for various extended code constructions and are thus of significant importance. For example, \cite[Construction 2]{5466536} extends DPGP codes, while \cite{8648459} employs right coset codes of \((n,M,d)\) DPGP codes in \(\mathcal{S}_n\) to construct an alternative structured permutation code distinct from the one proposed in \cite{7435295}. \Klove et al. introduced code extension methods in \cite[Sec.~III.C]{5466546}, referred to here as recursively extended codes (REP). When a code is extended, its size increases by a factor of \( q \), with \( q \) distinct leading elements. For the case \( q=2 \), a simple encoding and decoding method was designed \cite[Sec.~III.C]{5466546}. Because the factor graph connecting the input and output of this encoder forms a tree, MAP decoding becomes feasible using this graph. Kawasumi and Kasai enhanced decoding performance by concatenating this code with LDPC codes \cite{19950201,KawasumiKasai2023}. However, for the general case with \( q>2 \), no specific encoding and decoding scheme has been proposed. The rest of this paper is organized as follows. Section II introduces the necessary notation and fundamental concepts related to the construction of general permutation codes and DPGP codes. Section III describes the properties of extended codes and provides several lemmas that will be used in the proofs in subsequent sections. Section IV discusses REP code properties and presents key theorems regarding optimal REP codes. Section V covers encoding algorithms for REP codes, including both natural and recursive methods, and introduces decoding methods for optimal REP codes. \section{Notation and Preliminaries} For a positive integer $n$, we define $[n]$ as the set $\{0, 1, \ldots, n-1\}$. We denote the set $\{x_0, \ldots, x_{n-1}\}$ by $\{x_j\}_{j=0}^{n-1}$, or simply by $\{x_j\}$ when the context makes the range of $j$ clear. We denote the array $(x_0, \ldots, x_{n-1})$ by $x_0^{n-1}$. Let \( \Sc_n \) be the symmetric group on \([n]\). More precisely, let \( \Sc_{[n]} \), or simply \( \Sc_n \), denote the set of permutations over \([n]\), which can be defined as the set of bijective functions \( f: [n] \to [n] \). To represent a permutation \( f \in \Sc_n \) as an array, we use \( \fU = \left[ f(0), \ldots, f(n-1) \right]\). Let us represent arrays with an underlined variable such as $\xU$. We write the \( j \)-th element of the array \(\xU\) as an array of square brackets: \( x_j \): \(\xU = [x_0, x_1, \ldots, x_{n-1}] \). A subset \( C \subset \Sc_n \) of the symmetric group \( \Sc_n \) is called a {\itshape permutation code} of length \( n \), or simply a code of length \( n \). The elements of \( C \) are called {\itshape codewords}. Let \( C \) be a code of length \( n \) with \( C \subset \Sc_n \), and let \( \cU \) and \( \cU' \) be two codewords in \( C \). The Chebyshev distance between \( \cU \) and \( \cU' \) is defined as \( d_\infty(\cU, \cU') = \max_{j \in [n]} |c_j - c_j'|. \) The minimum distance between different codewords in \( C \) is referred to as the minimum distance of \( C \) and is denoted by \( d_\infty(C) \): \( d_\infty(C) := \min_{\cU, \cU' \in C : \cU \neq \cU'} d_\infty(\cU, \cU'). \) For a code \( C \) containing only one codeword, the minimum distance is defined as infinity. We call a code $\Cc\subset\Sc_n$ and $(n,M,d)$ code if $\Cc$ is of length $n$, of size $M$ and of minimum distance at least $d$. \subsection{Direct Product Group Permutation Codes}\label{212515_27Aug24} In this section, we review a simple permutation code independently discovered by \Klove et al. \cite[Explicit Construction]{5466546} and Tamo et al. \cite[\black{Construction 1}]{5466536}. In this paper, we will refer to the codes as {\it direct product group permutation} (DPGP) codes based on the properties of the fact described below \cite{5466536}. The DPGP code \(G\) of length \(n\) and minimum distance \(d\) is defined as a set of permutations \((\pi_0, \ldots, \pi_{n-1}) \in \Sc_n\) that satisfy the following condition: \( \pi_i \equiv i \pmod{d} \text{ for all } i \in [n]. \) Let \(A_i\) be the set of integers in \([n]\) congruent to \(i\) modulo \(d\). For all \(i \in [d]\), we define \(A_i\) as follows: \( A_i = (d\mathbb{Z} + i) \cap [n] = \{j \in [n] \mid j \equiv i \ (\bmod\ d)\}. \) Then, we can express \(G\) as the direct product of symmetric groups on \(A_i\): \( G = \Sc_{A_0} \times \Sc_{A_1} \times \cdots \times \Sc_{A_{d-1}}. \) The size of \(A_i\) is \(\left\lfloor \frac{n}{d} \right\rfloor \) when \( i \geq (n \bmod\ d)\), and \(\left\lceil \frac{n}{d} \right\rceil \) when \( i < (n \bmod\ d)\). Consequently, the size of the code \(|G| = |A_0| \cdots |A_{d-1}|\) can be expressed as $ |G|=\left(\left\lceil \frac{n}{d} \right\rceil !\right)^{n \bmod\ d} \left(\left\lfloor \frac{n}{d} \right\rfloor !\right)^{d - (n \bmod\ d)}. $ This expression simplifies to \(|G| = \left(\left(\frac{n}{d}!\right)^d\right)\) when \(d\) divides \(n\). We offer an alternative expression for $|G|$. The size of $G$ can be represented as the product of $n$ factors, as shown below: $|G|=\prod_{j=0}^{n-1} {\left( \left\lfloor j/d \right\rfloor + 1 \right)}$ Now, let us proceed with proving this. First, express $n$ in terms of the quotient $q$ and remainder $r$ when divided by $d$, i.e., $n = qd + r$. The product $\prod_{j=0}^{n-1} \left( \left\lfloor \frac{j}{d} \right\rfloor + 1 \right)$ can be rewritten as follows: \begin{align} &\prod_{j=0}^{qd-1} {\left( \left\lfloor j/d \right\rfloor + 1 \right)}\times\prod_{j=qd}^{qd+r-1} {\left( \left\lfloor j/d \right\rfloor + 1 \right)}. \\&= \prod_{p=0}^{q-1} \prod_{s=0}^{d-1} \left( \left\lfloor \frac{pd+s}{d} \right\rfloor + 1 \right) \times \prod_{s=0}^{r-1} \left( \left\lfloor \frac{qd+s}{d} \right\rfloor + 1 \right) \\&= \Bigl(\prod_{p=0}^{q-1}{\left( p + 1 \right)^d}\Bigr)\times(q+1)^r \\ &= (\overbrace{1\cdots 1}^{d \text{ times}})(\overbrace{2\cdots 2}^{d \text{ times}})\cdots(\overbrace{q\cdots q}^{d \text{ times}})\times(q+1)^r \\&= (q!)^d\times(q+1)^r \\&\stkeqa \left( \left\lceil n/d \right\rceil ! \right)^{r}\left( \left\lfloor n/d \right\rfloor ! \right)^{d - r}=|G|. \end{align} The validity of (a) becomes evident upon considering that $ \left\lceil j/d\right\rceil$ is $\left\lfloor n/d\right\rfloor $ if $r=0$ and is $\left\lfloor n/d\right\rfloor+1$, otherwise. \section{Code Extension} In Section \cite[III.~C]{5466546}, \Klove et al. introduced the concept of code extension. In this section, we provide a comprehensive overview of these codes, followed by a discussion of their encoding methods in the subsequent section. The properties of code extension detailed here are either directly derived from or previously established in \cite{5466546}. While the original work in presents several valuable insights regarding code extension, its presentation is somewhat fragmented, making it challenging to cite relevant points clearly. Therefore, the goal of this section is to systematically consolidate the key findings on code extension. By organizing the material in a more cohesive manner, we aim to clarify the relationships and properties associated with code extension, enabling a more straightforward understanding and application of these ideas in further research. \subsection{Definition} The concept of an \textit{extension of a permutation} was introduced in \cite[Section III.C]{5466546}\footnote{Note that the definition provided in \cite[Section III.C]{5466546} contains a minor error, using a strict inequality, specifically defining $\phi_s(x) := x + \mathbbm{1}[x > s]$. This formulation fails to yield a valid permutation for the subsequently defined $\piU^s$.}. Let $\piU = (\pi_0, \dots, \pi_{n-1}) \in \Sc_n$ be a permutation of length $n \geq 1$. The \textit{extended permutation} of $\piU$ with a \textit{head} $s \in [n+1]$ is defined as a permutation of length $n+1$: \begin{align} \piU^s := \bigl[s, \pi_0^s, \pi_1^s, \dots, \pi_{n-1}^s\bigr], \label{001517_13Apr23} \end{align} where $x^s := \phi_s(x) := x + \mathbbm{1}[x \geq s]$. Here, the indicator function $\mathbbm{1}[P]$ equals 1 if the proposition \( P \) is true, and 0 otherwise. Next, we introduce the extension of permutation codes. For $\Cc \subset \Sc_n$ and a set \( S \subset [n+1] \), which we refer to as the \textit{head set}, the \textit{extended code} with head set \( S \) is defined by \[ \Cc^S := \{\piU^s \in \Sc_{n+1} \mid s \in S, \piU \in \Cc\}. \] Since \( \Cc^S \) is empty if \( S \) is empty, we assume throughout this paper, unless otherwise noted, that the head set is non-empty. This code is the set of permutations obtained by extending each codeword \( \piU \in \Cc \) with a head \( s \in S \). To facilitate a more concise definition, we introduce a {formal codeword of length zero}, denoted as \( \varepsilonU \), which satisfies the condition: \[ \varepsilonU^0 = [0]. \] For a subset \( S \subset [n+1] \), we define the \textit{minimum distance} of \( S \) as the smallest difference between distinct elements, formally given by: \[ \dmin(S) \defeq \min \bigl\{ |s - s'| : s, s' \in S, s \neq s' \bigr\}. \] For sets containing only a single distinct element, the minimum distance is defined to be \( \infty \). \begin{example} The extended codeword of $[0123]$ with head $2$ is $[0123]^2=[20134]$. For $\Cc=\{[0123]\}$ and $S=\{0,2,4\}$, we have $\Cc^S=\{[01234],\allowbreak [20134],[40123]\}. $ \end{example} \subsection{Some Properties on Extensions} In this section, we derive several useful properties related to extensions for $n\ge 1$. \begin{lem}\label{144109_3Jun24} For $\pi,\sigma\in [n]$ and $s\in [n+1]$, \begin{enumerate*}[label=\roman*), series = tobecont, itemjoin = \quad] \item \label{145251_3Jun24} $\pi < \sigma$ implies $\pi^s < \sigma^s$. \item \label{145256_3Jun24} $\pi \le \sigma$ implies $\pi^s \le \sigma^s$. \end{enumerate*} \end{lem} \begin{proof}\label{144130_3Jun24} \ref{145251_3Jun24}. In the case where $s \le \pi < \sigma$: $\pi^s = \pi + 1 < \sigma + 1 = \sigma^s$. In the case where $\pi < \sigma < s$: $\pi^s = \pi < \sigma = \sigma^s$. In the case where $\pi < s \le \sigma$: $\pi^s = \pi < \sigma + 1 = \sigma^s$. From \ref{145251_3Jun24} and the fact that $\pi^s = \sigma^s$ when $\pi = \sigma$, \ref{145256_3Jun24} is evident. \end{proof} The following theorem gives a lower bound on $|S|$ in terms of $\dmin(S)$. \begin{teiri}\label{230931_30Oct23} For any subset $S \subset [n+1]$ such that $\dmin(S) \ge d$, the following inequality holds: $d(|S|-1) \le n$. From this, it follows that: $|S| \le \left\lfloor \frac{n}{d} + 1 \right\rfloor$. Conversely, by setting $S = \{0, d, 2d, \ldots, (|S|-1)d\} \subset [n+1]$, we achieve $|S| = \left\lfloor \frac{n}{d} + 1 \right\rfloor$ and $\dmin(S) = d$. \end{teiri} \begin{proof}\label{001824_29Oct23} Consider the set of integers with the following inclusion: \[ \{s_1\} \cup \bigcup_{i=1}^{|S|-1} (s_i, s_{i+1}] \subset [n+1] \] Each constituent set on the left-hand side is disjoint. Considering the sizes of both sides, we have: \[ 1 + \sum_{i=1}^{|S|-1} |(s_i, s_{i+1}]| \le n + 1 \] Moreover, since $d \le |s_i - s_{i+1}| = |(s_i, s_{i+1}]|$, it follows that: \[ 1 + d(|S|-1) \le 1 + \sum_{i=1}^{|S|-1} |(s_i, s_{i+1}]| \le n + 1 \] This concludes the proof. \end{proof} \begin{example}\label{141422_10Jan24} For $n=5$, $S=\{0,3,5\}$, we have $\dmin(S)=2$, $|S|=3$, $\floor{(n+1)/\dmin(S)+1}=\floor{5/2+1}=3$. For $n=6$, $S=\{0,3,6\}$, we have $\dmin(S)=2$, $|S|=3$, $\floor{(n+1)/\dmin(S)+1}=\floor{6/3+1}=3$. \end{example} \begin{lem}\label{010525_27Nov23} For a permutation $\piU \in \Sc_n$ and $s, t \in [n+1]$, we have: \begin{align} \dinf(\piU^s, \piU^t) = |s - t|. \end{align} \end{lem} \begin{proof}\label{012433_27Nov23} The result is clear when $s = t$, as both sides are zero. Now, consider the case when $s \neq t$. We have $\dinf(\piU^s,\piU^t)=\max_{j\in [n+1]}|(\piU^s)_j-(\piU^t)_j|=\max\{|s-t|,|\pi_j^s-\pi_j^t| \tFOR j\in [n]\}=|s-t|$. \end{proof} \begin{lem}\label{lem:phi_kakudai} For \( n > 0 \), \( s \in [n+1] \), and \( \pi, \sigma \in [n] \), we have: \begin{align} \left| \pi^s - \sigma^s \right| = \left| \pi - \sigma \right| + \mathbbm{1}[s \in (\pi, \sigma]]. \label{142940_10Apr23} \end{align} \end{lem} \begin{proof} The following equation provides the proof for the claim. $ |\pi^s-\sigma^s| = \left|\phi_s\left(\pi\right)-\phi_s\left(\sigma\right)\right| =\left|\left(\pi-\sigma\right)+\left(\mathbbm{1}\left\{\pi \geq s\right\}-\mathbbm{1}\left\{\sigma \geq s\right\}\right)\right| = |\pi-\sigma|+\mathbbm{1}[\sigma< s\le \pi \tOR \pi< s\le \sigma] = |\pi-\sigma|+\mathbbm{1}[s\in (\pi, \sigma]]. $ \end{proof} \begin{lem}\label{224431_12Nov23} Let $\Cc$ be a code of length $n$. For distinct codewords $\piU, \sigmaU \in \Cc$ and $s \in [n+1]$, it holds that $\dinf(\piU, \sigmaU) \le d_\infty\bigl(\piU^s, \sigmaU^s\bigr) \le \dinf(\piU, \sigmaU) + 1. $ \end{lem} \begin{proof} The 0-th element of both $\piU^s$ and $\sigmaU^s$ is $s$. From Lem.~\eqref{001517_13Apr23}, we have $d_\infty\bigl(\piU^s, \sigmaU^s\bigr) = \max_{j \in [n+1]}|\pi_j^s - \sigma_j^s|$. From Lem.~\ref{lem:phi_kakudai}, it follows that $|\pi_j - \sigma_j| \le |\pi_j^s - \sigma_j^s| \le |\pi_j - \sigma_j| + 1$. The equality in the second inequality holds if and only if $s \in (\pi_j^s, \sigma_j^s]$. Taking the maximum over all $j \in [n]$, we derive the assertion of the lemma. \end{proof} \begin{lem}[$\phi$ is expansive w.r.t. its second argument]\label{143827_10May23} For a permutation code $\Cc$ of length $n$ and a subset $S \subset [n+1]$, for arbitrary $\piU, \sigmaU \in \Cc$ and $s, t \in S$, the following inequality holds true: $d_\infty\bigl(\piU^s, \sigmaU^t\bigr) \ge |s - t|.$ Equality holds when $\piU = \sigmaU$. \end{lem} \begin{proof} The claim is evident from the following inequality: \begin{align*} d_\infty\left(\piU^s, \sigmaU^t\right) = \max_{j \in [n+1]} \left| (\piU^s)_j - (\sigmaU^t)_j \right| \geq \left| (\piU^s)_0 - (\sigmaU^t)_0 \right| = \left| s - t \right|. \end{align*} From Lem.~\ref{010525_27Nov23}, it is clear that equality holds when $\piU = \sigmaU$. \end{proof} \begin{lem}\label{152042_1Oct23} $\phi:(\Sc_n\times [n+1])\to \Sc_{n+1}$ is a one-to-one mapping. \end{lem} \begin{proof} It is sufficient to show $(\piU,s) \neq (\sigmaU,t)$ implies $\piU^s \neq \sigmaU^t$. First, let's consider the case when $s \neq t$. From Lem.~\ref{143827_10May23}, $s\neq t$ implies $\piU^s \neq \sigmaU^t$. Next, let's consider the case when $\piU \neq \sigmaU$ and $s = t$. There exists $i \in [n]$ such that $\pi_i \neq \sigma_i$. According to Lem.~\ref{lem:phi_kakudai}, we have $|\phi_s(\pi_i) - \phi_s(\sigma_i)| \geq |\pi_i - \sigma_i|$, which in turn implies $\piU^s \neq \sigmaU^t$. \end{proof} From these lemmas, the following theorem is immediately derived. \begin{teiri}\label{205810_6Jul23} For a code $\Cc$ of length $n$ and a subset $S \subset [n+1]$, we have: $|\Cc^S| = |\Cc| \times |S|.$ \end{teiri} \subsection{Lower Bounds on Minimum Distance Through Extension} In this section, we provide several lower bounds on minimum distance through extension. \begin{teiri}\label{153755_6Jan24} For any permutation code $C\subset \Sc_{n}$ and any head set $S \subset [n+1]$, \begin{align} \dmin\left(\Cc^S\right) \geq \min \left(\dmin(S), \dmin(\Cc)\right) \end{align} \end{teiri} \begin{proof}\label{153804_6Jan24} First, consider the case where $|S|=1$, for which $\dmin(S)=\infty$. Let $S=\{s\}$. Any distinct pair of codewords from $\Cc^S$ can be expressed as $(\piU^s, \sigmaU^s)$, with $\piU$ and $\sigmaU$ being distinct elements of $\Cc$. We then have $ \dinf\left(\piU^s, \sigmaU^s\right) \stkgea \dinf\left(\piU, \sigmaU\right) \ge \dmin(\Cc)$, which leads to the inequality $ \dmin(\Cc) = \min \left\{\dmin(S), \dmin(\Cc)\right\}$. The result follows from Lemma~\ref{224431_12Nov23} as used in (a). Now, consider the case where $|S| \ge 2$. For any distinct codewords $\piU^s \neq \sigma^t \in \Cc^S$, we aim to show that $\dinf\left(\piU^s, \sigma^t\right) \geq \min \left(\dmin(S), \dmin(\Cc)\right)$. We examine the following two cases: \begin{itemize} \item If $s \neq t$: From Lemma~\ref{143827_10May23}, we know that $\dinf\left(\piU^s, \sigmaU^t\right) \geq |s-t| \geq \dmin(S)$. \item If $\piU \neq \sigmaU$ and $s = t$: According to Lemma~\ref{224431_12Nov23}, for distinct $\piU$ and $\sigmaU$ in $\Cc$, we have $\dinf\left(\piU^s, \sigmaU^s\right) \geq \dinf\left(\piU, \sigmaU\right) \geq \dmin(\Cc)$. In either case, it follows that $\dinf\left(\piU^s, \sigmaU^t\right) \geq \min \left\{\dmin(S), \dmin(\Cc)\right\}$. \end{itemize} \end{proof} The following theorem provides sufficient conditions on $\Cc$ and $S$ to construct an extended code $\Cc^S$ while ensuring the minimum distance remains at least $d$. \begin{teiri}[{\cite[Theorem 4]{5466546}}]\label{163227_14May23} For a code $\Cc$ of length $n$ and a subset $S \subset [n+1]$, the following holds: $\dmin(S) \ge d$ and $\dmin(\Cc) \ge d$ implies $\dmin(\Cc^S) \ge d$. \end{teiri} \begin{proof}\label{161444_6Jan24} The assumption is equivalent to $\min(\dmin(S), \dmin(\Cc)) \geq d$. By applying Theorem~\ref{153755_6Jan24}, we conclude that $\dmin(\Cc^S) \geq d$. \end{proof} \subsection{Upper Bounds on Minimum Distance Through Extension} The following two theorems give upper bound of the minimum distance of the extended code. \begin{teiri}[Upper bound on $\dmin(\Cc^S)$]\label{003746_1Oct23} Let $\Cc$ be a code of length $n$ and $S \subset [n+1]$ be a head set. \begin{align} \dmin(\Cc^S) \le \dmin(S). \end{align} \end{teiri} \begin{proof} If $|S|=1$, the claim of the theorem would be $d_\infty(\Cc^S) \le \infty$, which renders the claim meaningless. Therefore, we consider the case where $|S| \ge 2$. It suffices to show that there exists a pair of codeword $\Cc^S$, whose distance is $\dmin(S)$. Select $s, t \in S$ such that $|s-t| = \dmin(S)$. For any $\piU \in \Cc$, by Lem.~\ref{010525_27Nov23}, we have $d_\infty\bigl(\piU^s, \piU^t\bigr) = |s-t| = \dmin(S)$. \end{proof} \begin{teiri}\label{161110_1Oct23} Let $\Cc$ be a code of length $n$ and $S \subset [n+1]$ be a head set. Then, it holds that $d_{\infty}(\Cc^S) \le \dmin(\Cc)+1$. \end{teiri} \begin{proof} When $|C| = 1$, $\dmin(\Cc) = \infty$, so the claim is true. Consider the case where $|C| \ge 2$. It is sufficient to show that there exists a pair of codewords in $\Cc^S$ whose distance is less than or equal to $\dmin(\Cc) + 1$. Select distinct $\piU, \sigmaU \in \Cc$ such that $d_\infty(\piU, \sigmaU) = \dmin(\Cc)$. From Lemma~\ref{152042_1Oct23}, we observe that for any $s \in S$, $\piU^s$ and $\sigmaU^s$ are distinct codewords in $\Cc^S$. Hence, it follows that $\dinf\bigl(\piU^s, \sigmaU^s\bigr) \stackrel{\ref{224431_12Nov23}}{\leq} \dinf(\piU, \sigmaU) + 1 = \dmin(\Cc) + 1$, where the inequality is derived using Lemma~\ref{224431_12Nov23}. \end{proof} For $\Cc \subset \Sc_n$ and $S \subset [n+1]$, consider the extension $\Cc \to \Cc^S$. When $|S| = 1$, the size remains unchanged after the extension, i.e., $|\Cc| = |\Cc^S|$, as stated in Theorem~\ref{205810_6Jul23}. Such an extension is referred to as {\itshape size-preserving}. In cases where $|\Cc| < |\Cc^S|$, the extension is called {\itshape size-increasing}. If $\dmin(\Cc) < \dmin(\Cc^S)$, we describe the extension as distance-increasing. We probide an example of an extension that is both size-preserving and distance-increasing. \begin{example}\label{134606_2Oct23} Let $\Cc=\{0123,3012\}$ and $S=\{1\}$. Then, $\Cc^S=\{10234,14023\}$, with $\dmin(S)=\infty$, $\dmin(\Cc)=3$ and $\dmin(\Cc^S)=4$. This a size-preserving and distance-increasing extension. For $\Cc=\{0123,1032\}$ and $S=\{1,3\}$, we have $\Cc^S=\{10234,30124,12043,31042\}$, $\dmin(S)=2$,$\dmin(C)=1$ and $ \dmin(\Cc^S)=2$. This a size-increasing and distance-increasing extension. \end{example} \subsection{Codeword Pairs, Interval Sets, and Maximum Intervals} In this subsection, we derive the lemmas on extensions that are used in the proof of the theorem in Section \ref{164043_17Sep24}. For integers $x, y \ge 0$, we define {\itshape interval} between $y$ and $x$ and denoted it by $(y, x]$ as follows: $(y, x]$ is defined as $\{a \in \Zb \mid y < a \le x\}$ if $y < x$, as $\{a \in \Zb \mid x < a \le y\}$ if $x < y$, and as an empty set if $x = y$. The length of interval $I=(x,y]$ is defined as $|x-y|$ and denoted by $|I|$. For an interval $I=(x,y]\subset [n]$ and $s\in [n+1]$, we define $I^s:=\{a^s:a\in I, s\in S\}=(x^s,y^s]$. \begin{lem}\label{054652_14Jul24} For an interval $I\subset [n+1]$, it holds that $|I^s|=|I|+\I[s\in I]$. \end{lem} \begin{proof} Let $I=(x,y]\subset[n+1]$. The claim is obvious from the following: $|I^s|=|(x^s,y^s]|=|x^s-y^s|\stkeqa|x-y|+\I[s\in (x,y]]=|I|+\I[s\in I]$. In (a), we used \ref{lem:phi_kakudai}. \end{proof} In this section, we define interval sets and maximum intervals for codeword pairs and provide sufficient conditions for increasing the distance when the codeword pairs are extended, using the maximum intervals of the codeword pairs. \begin{lem}\label{153103_22May24} If intervals $I$ and $J$ are disjoint, then $I^s$ and $J^s$ are disjoint. \end{lem} \begin{proof} Without loss of generality, we can write $I = (\pi_1, \sigma_1]$ and $J = (\pi_2, \sigma_2]$ using $\pi_1 < \sigma_1 \le \pi_2 < \sigma_2$. From Lem.~\ref{144109_3Jun24}, we have $\pi_1^s < \sigma_1^s \le \pi_2^s < \sigma_2^s$, so $I^s$ and $J^s$ are disjoint. \end{proof} \begin{lem}\label{195413_3Jun24} If intervals $I$ and $J$ satisfy $I \subset J$, then $I^s \subset J^s$. \end{lem} \begin{proof} Without loss of generality, we can write $I = (\pi_1, \sigma_1]$ and $J = (\pi_2, \sigma_2]$ using $\pi_1 \le \pi_2 < \sigma_2 \le \sigma_1$. From Lem.~\ref{144109_3Jun24}, we have $\pi_1^s \le \pi_2^s < \sigma_2^s \le \sigma_1^s$, so $I^s \subset J^s$ holds. \end{proof} For a pair of permutations $\piU, \sigmaU$ of length $n$, we define the following: \begin{enumerate} \item The set of intervals $(\pi_j, \sigma_j]$ for $j=0, \ldots, n-1$ of non-zero length is called the {\it interval set} between $\piU$ and $\sigmaU$, or simply the interval set, and is denoted by $(\piU, \sigmaU]$. To be precise, $(\piU, \sigmaU] \defeq \{(\pi_j, \sigma_j] \mid \pi_j \neq \sigma_j, j=0, \ldots, n-1\}.$ \item For a pair of codewords $\piU, \sigmaU$, if an interval $(\pi_j, \sigma_j] \in (\piU, \sigmaU]$ contains all other intervals $(\pi_i, \sigma_i] \in (\piU, \sigmaU]$, i.e., $(\pi_j, \sigma_j] \supset (\pi_i, \sigma_i]$, then $(\pi_j, \sigma_j]$ is called the maximum interval of the pair $\piU, \sigmaU$. From the definition, we see that if a maximum interval exists for $\piU, \sigmaU$, it is unique. \end{enumerate} From the definition, the following holds: $(\piU^s, \sigmaU^s] = \{(\pi^s, \sigma^s] \mid (\pi, \sigma] \in (\piU, \sigmaU]\}$. Furthermore, the maximum length of the intervals in the interval set $(\piU, \sigmaU]$ is equal to the distance between $\piU$ and $\sigmaU$: $ \displaystyle \dinf(\piU, \sigmaU) = \max_{J \in (\piU, \sigmaU]} \ell(J).$ For a pair of permutations $\Psf:=(\piU, \sigmaU)$ in $\Sc_n$ and a head $s\in[n+1]$, we denote a pair of permutations $(\piU^s, \sigmaU^s)$ in $\Sc_{n+1}$ by $\Psf^s$. \begin{lem}\label{060127_4Mar24} For a pair of permutations $\Psf:=(\piU, \sigmaU)$ in $\Sc_n$ of length $n$ that has a maximum interval $I$, the following holds: \begin{enumerate*}[label=\roman*), series = tobecont, itemjoin = \quad] \item\label{142848_5Mar24} The permutation pair $\Psf^s$ has maximum interval $I^s$. \item $\dinf(\piU, \sigmaU) = |I|$ \item\label{153314_30Dec23} $\dinf(\piU^s, \sigmaU^s) = \dinf(\piU, \sigmaU) + \I[s \in I]$ \item$|I^s| = |I| + \I[s \in I]$ \end{enumerate*} \end{lem} \begin{proof}\label{162409_30Dec23} First, we prove \ref{142848_5Mar24}. Since $I$ is maximum, we have $J \subset I$ for any interval $J \in (\piU, \sigmaU]$. From this and Lem.~\ref{195413_3Jun24}, it follows that $J^s \subset I^s$. Therefore, $I^s$ is maximum in the permutation pair $\Psf^s$. The fact that $I$ is the unique interval of length $\dinf(\piU, \sigmaU)$ contained in $(\piU^s, \sigmaU^s)$, together with Lem.~\ref{054652_14Jul24}, makes \ref{153314_30Dec23} evident. \end{proof} \begin{lem}\label{000804_8Dec23} Let $\piU$ be a codeword of length $n$, and let $s<t$ for $s,t \in [n+1]$. Then the following holds: \begin{enumerate*}[label=\roman*), series = tobecont, itemjoin = \quad] \item \label{133646_30Dec23} $ (\piU^s,\piU^t]=\{(s,t],(s,s+1],\ldots,(t-1,t]\}$ \item $\#(\piU^s,\piU^t]=|s-t|+1$ \item The codeword pair $(\piU^s,\piU^t)$ has the largest interval $(s,t]$. \end{enumerate*} \end{lem} \begin{proof} We will prove \ref{133646_30Dec23}. Without loss of generality, we can assume that $\piU$ is the identity permutation $\iotaU=[0,1,\ldots,n-1]$. From the definition of extension \eqref{001517_13Apr23}, we have the following: \begin{align} \begin{array}{llll} \iotaU^s=[\black{s},&0,1,\ldots,s-1,\black{s+1},&\black{s+2,\ldots,t}, &t+1,\ldots,n], \\ \iotaU^t=[\black{t},&0,1,\ldots,s-1,\black{s}, &\black{s+1,\ldots,t-1},&t+1,\ldots,n]. \end{array} \end{align} The remaining claims follow from this. \end{proof} \section{Recursively Extended Permutation Codes}\label{164043_17Sep24} In the previous section, we investigated the changes in minimum distance and size resulting from a single code extension. In this section, we investigate permutation codes that repeatedly extended. For each $j=0,\ldots,n-1$, let $S^{(j)}$ be a non-empty subset of $[j+1]$. The construction method for the permutation code $\Cc^{(n)}$ of length $n$ is as follows: First, we define $\Cc^{(0)} := \{\varepsilonU\}$. Next, for $j=1, \ldots, n$, we recursively construct $\Cc^{(j)}$ from $\Cc^{(j-1)}$ using the equation: $\Cc^{(j)} = \phi(\Cc^{(j-1)}; S^{(j-1)}).$ We refer to $\Cc^{(n)}$ constructed in this manner as a {\it recursively extended permutation} (REP) code generated by $\{S^{(j)}\}_{j=0}^{n-1}$. We denote it by $\Cc^{(n)} = \<\{S^{(0)},\ldots,S^{(n-1)}\}\>$. From Thm.~\ref{205810_6Jul23}, we obtain the following: $|\Cc^{(n)}| = \prod_{j=0}^{n-1} |S^{(j)}|.$ \begin{example} In \cite[III. D]{5466546}, a construction of $(n,q^{n-(q-1)d}, d)$ REP code with head sets $S^{(j)}\subset [j+1]$ for $j\in [n]$ is proposed as follows. For integers $n,d,q$ with $q \ge 2$ and $(q-1)d< n$, set $S^{(j)} = \{0\}$ for $0 \le j < (q-1)d$. Set $S^{(j)} = \{\lfloor {j}/{(q-1)}\rfloor x: x = 0, \ldots, q-2\} \cup \{j\}$ for $(q-1)d \le j \le n-1$. We can interpret such $S^{(j)}$ as the positioning of $q$ points within $[j+1]$, ensuring a minimum spacing of $d$ between each point. We observe that $|S^{(j)}|$ is 1 and $\dmin(S^{(j)}) = \infty$ for $0 \le j < (q-1)d$ and $|S^{(j)}| = q$ and $\dmin(S^{(j)}) \ge d$ for $(q-1)d \le j \le n-1$. The size of the code is given by $|\Cc^{(n)}| = \prod_{j=0}^{n-1}|S^{(j)}| = q^{n-(q-1)d}$. Since $\Cc^{(0)} = \{\epsilonU\}$, it follows that $\dmin(\Cc^{(0)}) = \infty$. By repeatedly applying Thm.~\ref{163227_14May23}, it holds that $\dmin(\Cc^{(n)}) \ge d$. \end{example} As seen in the example above, from Thm.~\ref{163227_14May23}, if $\dmin (S^{(j)}) \ge d$ for $j=0,1,\ldots,n-1$, then $\dmin (\Cc^{(n)}) \ge d$. The converse is not true. To achieve $\dmin (\Cc^{(n)}) \ge d$, it is not necessary that $\dmin (S^{(j)}) \ge d$ for $j=0,1,\ldots,n-1$. For instance, consider $S^{(0)} = {0,1}$ and $S^{(1)} = {1}$, where $\dmin(S^{(0)}) = 1$. Then, we have, $\Cc^{(0)} = \{0\}, \ \Cc^{(1)} = \{01, 10\}, \ \Cc^{(2)} = \{102, 120\}, $ and thus $\dmin (\Cc^{(2)}) = 2$. \subsection{The necessary number of size-preserving extensions for increasing minimum distance} A code with a minimum distance of at least $d$ and a length of $n$ is referred to an $[n, d]$ code. In this subsection, we identify the $[n, d]$ code with the largest possible size. From the results of the previous section, it is clear that the minimum distance can increase with extensions. It is difficult to derive a tight upper bound on the size of an $[n, d]$ code from the conventional bounds derived in previous section. We need to evaluate the number of size-preserving extensions required to increase the minimum distance through extensions. Let $\Cc_0$ be a permutation code of some code length. In this subsection, we investigate the number of size-preserving extensions, denoted as $c_1(\Cc_0;d)$, required to increase the minimum distance of extended code of $\Cc_0$ to $d$.We provide both lower and upper bounds on $c_1(\Cc_0;d)$. These bounds will be used in the proof for the optimal REP codes in the next subsection. The code $\Cc_0$ is extended with head set $S_j$ as $\Cc_{j+1} = \Cc_j^{S_j}$ for $j \ge 0$. In this context, we denote the minimum number of size-preserving extension needed for $\Cc_k$ to achieve a minimum distance of $d$ as $c_1^{(k)}(\Cc_0; S_0, \ldots, S_{k-1})$. Formally, this can be written as follows: \begin{align} &c_1(\Cc_0; d) \defeq \min_{k \ge 0} c_1^{(k)}(\Cc_0; d) \label{151909_28Apr24} \\ &c_1^{(k)}(\Cc_0; d) \defeq \min_{S_0, \ldots, S_{k-1} : \dmin(\Cc_k) \ge d} \# \{0 \le l \le k-1 : |S_l| = 1\} \end{align} The following lemma provides an upper bound for $c_1(\Cc^{(n)};d)$. \begin{lem}[Upper bound on $c_1$]\label{165523_23May24} Let $n > d \ge 1$. For any REP code $\Cc^{(n)}$ such that $\dmin(\Cc^{(n)}) \ge d$, for any $1 \le k \le n$, the following holds: \begin{align} c_1(\Cc^{(k)};d) \le n - k \label{031411_8May24} \end{align} \end{lem} \begin{proof} First, since $\dmin(\Cc^{(n)}) \ge d$, we have $c_1(\Cc^{(n)};d) = 0$. Next, we show that $c_1(\Cc^{(n-1)};d) \le 1$. From the assumption: $S^{(n)} = (\Cc^{(n-1)})^{S^{(n-1)}}$, we have $c_1(\Cc^{(n-1)};d) = 1$ if $|S^{(n-1)}| = 1$, and $c_1(\Cc^{(n-1)};d) = 0$, otherwise. Continuing this process, we obtain \eqref{031411_8May24}. \end{proof} In \eqref{151909_28Apr24}, we defined $c_1(\Cc;d)$ for a code $\Cc \subset \Sc_n$. Below, with a slight abuse of notation, we define $c_1(S;d)$ for a head set $S \subset [n+1]$. First, for $S$ with $|S| = 1$, we define $c_1(S;d) = 0$. Next, for $S$ with $|S| \ge 2$, let us write $S = \{s_1, s_2, \ldots\}$ with $s_1 < s_2 < \cdots$. We define $c_1(S;d)$ as the minimum number of increments required to extend the length of each interval $(s_i, s_{i+1}]$ of length less than $d$ to length $d$. More precisely, it is defined as follows: \begin{align} c_1(S;d) \defeq \sum_{j:|s_j-s_{j+1}|<d} (d - |s_j-s_{j+1}|) \label{151901_28Apr24} \end{align} This gives a lower bound for $c_1(\Cc^{S_0};d)$ in Thm.~\ref{231412_20May24}. The following lemma generalizes Thm.~\ref{230931_30Oct23}, which provides an upper bound for $|S|$. By setting $c = 0$, it reduces to Thm.~\ref{230931_30Oct23}. \begin{lem}\label{234753_24Apr24} For $S \subset [n]$, suppose $c \ge c_1(S;d)$. Then, the following holds: \begin{align} |S| \le \frac{n-1+c}{d} + 1\label{132307_12Jul24} \end{align} \end{lem} \begin{proof} Let $J := \{1, \ldots, |S|-1\}$. Define $\JU := \{j \in J : |s_j-s_{j+1}| < d\}$ and $\JO := \{j \in J : |s_j-s_{j+1}| \ge d\}$. We have $|\JU| + |\JO| = |S|-1$. The following holds: \begin{align} n &\stkgea 1 + \sum_{j \in J} |s_j-s_{j+1}| \\&= 1 + \sum_{j \in \JU} |s_j-s_{j+1}| + \sum_{j \in \JO} |s_j-s_{j+1}| \\&\stkgeb 1 + |\JU|d - c + |\JO|d \\&= 1 - c + (|S|-1)d \end{align} In (a), we used the union bound for the inclusion $[n] \supset \{s_1\} \cup \bigcup_{j \in J} (s_j, s_{j+1}]$, we obtain: In (b), we used the assumption: $c \ge c_1(S;d) = \sum_{j \in \JU} (d - |s_j-s_{j+1}|)$ and the fact that $\sum_{j \in \JO} |s_j-s_{j+1}| \ge |\JO|d$. This inequality immediately gives \eqref{132307_12Jul24}. \end{proof} \begin{teiri}\label{153614_27May24} For a code $\Cc_0 \subset \Sc_n$ and a head set $S \subset [n+1]$, let $\Cc_1 = \Cc_0^{S_0}$. For $d \ge 1$, the following holds: \begin{enumerate*}[label=\roman*), series = tobecont, itemjoin = \quad] \item \label{161656_27May24} $c_1(\Cc_0;d) \le c_1(\Cc_1;d) + 1$ \item \label{161706_27May24} $c_1(\Cc_0;d) = c_1(\Cc_1;d) + 1$ implies $|S_0| = 1$. \end{enumerate*} \end{teiri} \begin{proof} \ref{161656_27May24}. Suppose $c_1(\Cc_0;d) > c_1(\Cc_0^{S_0};d) + 1$ and derive a contradiction. Then, there exist $k>0$ and $k-1$ head sets $S_i \subset [n+1+i]$ $(i = 1,2 \ldots, k-1)$ of which at most $c_1(\Cc_0;d) - 2$ head sets are of size one, that satisfy $\dmin(\Cc_k = \Cc_1^{S_1 \cdots S_{k-1}}) \ge d$. This implies $\dmin(\Cc_k = \Cc_0^{S_0 \cdots S_{k-1}}) \ge d$ which contradicts the minimality of $c_1(\Cc_0;d)$. \ref{161706_27May24}. Suppose $|S_0| \neq 1$ and derive a contradiction. There exist head sets $S_i \subset [n+1+i]$ $(i = 1,2 \ldots, k-1)$ of which $c_1(\Cc_1;d)$ head sets are of size one, that satisfy $\dmin(\Cc_k = \Cc_1^{S_1 \cdots S_{k-1}}) \ge d$. From the fact that $\dmin(\Cc_k = \Cc_0^{S_0 \cdots S_{k-1}}) \ge d$ and the assumption $|S_0| \neq 1$, we see that this contradicts the minimality of $c_1(\Cc_0;d)$. \end{proof} For a codeword pair $(\piU, \sigmaU) =: \Psf$, we denote $(\piU^s, \sigmaU^s)$ by $\Psf^s$. We can rewrite Lem.~\ref{060127_4Mar24} as $ \dinf(\Psf^s) = \dinf(\Psf) + \I[s \in J]$. From this, when $(\piU, \sigmaU)$ has maximum interval $I$, it holds that $ |I^s| = |I| + \I[s \in I]$. \begin{lem}\label{152448_18May24} For a code $\Cc$ of length $n$, suppose there are $k$ codeword pairs $\Psf_1, \Psf_2, \ldots, \Psf_k$ each having maximum intervals $I_1, I_2, \ldots, I_k$ that are mutually disjoint. For $S \subset [n+1]$, there exist $k$ codeword pairs $\Qsf_1, \ldots, \Qsf_k$ in the extended code $\Cc^S$ each having mutually disjoint maximum intervals $J_1, J_2, \ldots, J_k$ satisfying the following: when $|S| = 1$, $|J_1| \le |I_1| + 1$, and $|J_i| \le |I_i|$ for $i \neq 1$ and when $|S| \ge 2$, $|J_i| \le |I_i|$ for $1 \le i \le k$. \end{lem} \begin{proof}\label{225701_20May24} First, we prove the case with $|S| = 1$. Let $S = \{s\}$ and $\Qsf_i := \Psf_i^s$ for $1 \le i \le k$. From Lem.~\ref{060127_4Mar24}~\ref{142848_5Mar24} and Lem.~\ref{153103_22May24}, each $\Qsf_i$ has a mutually disjoint maximum interval $J_i := I_i^s$ satisfying $|J_i| = |I_i| + \I[s \in I_i]$. Since $\{I_i\}$ are mutually disjoint, $s$ can be contained in at most one of the intervals $I_i$. Next, we prove the case with $|S| \ge 2$. Let $s$ and $t$ be distinct elements in $S$. Since $I_1, \ldots, I_k$ are disjoint, it suffices to consider the following three cases without loss of generality:\\ i) If $s$ and $t$ are contained in the same interval: let $s, t \in I_1$. For $i \neq 1$, since $I_1$ and $I_i$ are disjoint, $s, t \notin I_i$. For any $\piU \in \Cc$, let $\Qsf_1 = (\piU^s, \piU^t)$. From Lem.~\ref{000804_8Dec23}, $\Qsf_1$ has the maximum interval $J_1 = (s, t] \subsetneq I_1$. This inequality implies $|J_1| < |I_1|$. For $i \neq 1$, let $\Qsf_i = \Psf_i^s$. From Lem.~\ref{153103_22May24} and the fact that $s \notin I_i$, the pairs $\{\Qsf_i\}_{i \neq 1}$ are mutually disjoint. From Lem.~\ref{060127_4Mar24}~\ref{142848_5Mar24}, each $\Qsf_i$ has the maximum interval $J_i = I_i^s = I_i$, hence we have $|J_i|=|I_i|$. Thus, the intervals $\{J_i\}$ are disjoint.\\ ii) If $s$ is not contained in any interval: There is no $i$ such that $s \in I_i$. For $1 \le i \le k$, let $\Qsf_i = \Psf_i^s$. By the same argument as above, $\Qsf_i$ has the maximum interval $J_i$ with $|J_i| = |I_i|$ and these intervals are disjoint.\\ iii) If $s$ and $t$ are contained in different intervals: let $s \in I_1$ and $t \in I_2$. Let $\Qsf_1 = \Psf_2^s$, $\Qsf_2 = \Psf_1^t$, and $\Qsf_i = \Psf_i^s$ for $i \ge 2$. By the same argument as above, $\{\Qsf_i\}$ are mutually disjoint, each of which has the maximum interval $J_i$ with $|J_i| = |I_i|$. \end{proof} \begin{teiri}\label{231412_20May24} For a code $\Cc_0 \subset \Sc_n$ and a head set $S_0 \subset [n+1]$, let $\Cc_1 := \Cc_0^{S_0}$. Then, $c_1(\Cc_1;d) \ge c_1(S_0;d)$ holds. \end{teiri} \begin{proof}\label{231611_20May24} For head sets $S_j \subset [j+1]$ for $j = 1, 1, 2, \ldots$, define $\Cc_{j+1} := \Cc_j^{S_j}$. It is sufficient to show that there are at least $c_1(S_0; d)$ head set of size one among $S_1, \ldots, S_{m-1}$ for any $m\ge 1$ and $S_0,\ldots,S_{m-1}$ such that $\dmin(\Cc_m) \ge d$. Let the elements of $S_0$ be $s_1 < \cdots < s_{k+1}$. Denote $k := |S_0| - 1$. Choose some $\piU \in \Cc_0$ and denote $k$ codeword pairs $(\piU^{s_i}, \piU^{s_{i+1}})$ in $\Cc_1$ by $\Psf_i^0$. Each codeword pair $\Psf_i^0$ has a maximum interval $I_i^0 := (s_i, s_{i+1}]$, and these intervals are mutually disjoint. According to Lem.~\ref{152448_18May24}, there exist $k$ corresponding codeword pairs in $\Cc_1$, each with a mutually disjoint maximum interval. Let these pairs be denoted as $\{\Psf_i^1\}$. Continue this procedure for $\Cc_{i+1}$ for $i = 1, \ldots, m-1$. Consequently, there will be $k$ corresponding codeword pairs in $\Cc_m$, each with a mutually disjoint maximum interval, denoted as $\{\Psf_i^m\}_{i=1}^k$. Since $\dmin(\Cc_m) \ge d$, the length of the intervals for the codeword pairs $\{\Psf_i^m\}_{i=1}^k$ must be at least $d$. From Lem.~\ref{152448_18May24}, it follows that during each extension, at most one corresponding interval increases in length, and the increase is by at most one. Therefore, to increment the size of one of these $k$ disjoint intervals during the $j$-th extension by $S_j$, we need $|S_j| = 1$. By definition, $c_1(S_0; d)$ represents the total number of increments needed to increase the length of each interval $(s_i, s_{i+1}]$ from less than $d$ to $d$. Hence, the number of $j$ such that $|S_j| = 1$ is at least $c_1(S_0; d)$, which completes the proof. \end{proof} \subsection{Optimal REP codes} \label{161538_26Aug24} In this subsection, we prove the following for any $n > d \geq 1$: 1) An upper bound on the size of an $[n,d]$ REP code. 2) There exists an $[n,d]$ REP code whose size achieves the upper bound. 3) The upper bound matches the size of an $[n,d]$ DPGP code. Some readers might conclude from these results that the REP code and DPGP code share the same structure. However, as far as the authors have investigated, no such structure has been found. \begin{teiri}\label{165623_23May24} Let $\Cc^{(n)}$ be an $[n,d]$ REP code. Then it holds that $ |\Cc^{(n)}|\le \prod_{j=0}^{n-1}\floor{j/d+1}$. \end{teiri} \begin{proof} To simplify notation, we write $c^{(k)} := c_1(\Cc^{(k)}; d)$ for $0\le k <n$. We denote the sets of non-decreasing and decreasing points in the sequence $\{c^{(k)}\}$ by $K$ and $K^c$, respectively. Formally, $K \defeq \{0 \le k <n : c^{(k)} \le c^{(k+1)}\}$, $K^c \defeq \{0 \le k <n : c^{(k)} > c^{(k+1)}\}$. For $k \in K^c$, from Thm.~\ref{153614_27May24}, we have $c^{(k)} = c^{(k+1)} + 1$ and $|S^{(k)}| = 1$. Therefore, the following holds: $|\Cc^{(n)}|=\prod_{k=0}^{n-1} |S^{(k)}| = \prod_{k \in K} |S^{(k)}|. $ Furthermore, we can express it as follows: \begin{align} \prod_{k \in K} |S^{(k)}| &\stklea \prod_{k \in K} \left\lfloor \frac{k + c^{(k+1)}}{d} + 1 \right\rfloor \label{020938_27Apr24} \\ &\stkeqb \prod_{i=1}^{|K|} \left\lfloor \frac{k_i + c^{(k_i+1)}}{d} + 1 \right\rfloor \end{align} In (a), we used the fact that from Thm.~\ref{231412_20May24}, $c^{(k+1)} \ge c_1(S^{(k)}; d)$, and from Lem.~\ref{234753_24Apr24}, $|S^{(k)}| \le \left\lfloor \frac{k + c^{(k+1)}}{d} + 1 \right\rfloor$. In (b), we wrote the elements of $K$ in ascending order as $k_1 < k_2 < \cdots < k_{|K|}$. For $|K| = 1$, from Lem.~\ref{165523_23May24}, we have $k_1 + c^{(k_1+1)} \le n-1$, thus proving the theorem. Let us consider the case $|K| \ge 2$. The following holds: \begin{align} \prod_{i=1}^{|K|} \left\lfloor \frac{k_i + c^{(k_i+1)}}{d} + 1 \right\rfloor &\stklec \prod_{i=1}^{|K|} \left\lfloor \frac{n-1-(|K|-i)}{d} + 1 \right\rfloor \end{align} In (c), we used Lem.~\ref{021210_24Jul24}. The proof completes by considering $\prod_{i=1}^{|K|} \left\lfloor \frac{n-1-(|K|-i)}{d} + 1 \right\rfloor = \prod_{j=n-|K|}^{n-1} \left\lfloor \frac{j}{d} + 1 \right\rfloor \le \prod_{j=0}^{n-1} \left\lfloor \frac{j}{d} + 1 \right\rfloor. $ \end{proof} \begin{lem}\label{021210_24Jul24} Denote $m:=|K|$. For $i=1,\ldots,m-1$, it holds that \begin{align} k_i + c^{(k_i+1)} \le n-1 -(m-i) .\label{014925_24Jul24} \end{align} \end{lem} \begin{proof} First, we prove that for $i=1, \ldots, m-1$ \begin{align} k_i + c^{(k_i+1)} < k_{i+1} + c^{(k_{i+1}+1)}. \label{172920_9May24} \end{align} Note that we have $c^{(k_{i+1})} \le c^{(k_{i+1}+1)}$ since $k_{i+1} \in K$. It is sufficient to consider the following two cases. i) The case $k_i$ and $k_{i+1}$ are consecutive, $ k_i + 1=k_{i+1}$, hence $c^{(k_i+1)} = c^{(k_{i+1})}$: Therefore it holds that $k_i + c^{(k_i+1)} \le k_{i+1} + c^{(k_{i+1}+1)} - 1$. ii) The case $k_i$ and $k_{i+1}$ are not consecutive, $k_i+1< k_{i+1}$: For $k_i+1 \le k \le k_{i+1}-1$, it holds $k \in K^c$, then we obtain $c^{(k)} = c^{(k+1)} + 1$ from Thm.~\ref{153614_27May24}. This implies $c^{(k_i+1)} - c^{(k_{i+1})} = k_{i+1} - k_i - 1$. Thus, $k_i + c^{(k_i+1)} \le k_{i+1} + c^{(k_{i+1}+1)} - 1$. From Lem.~\ref{165523_23May24}, we have $k_{m} + c^{(k_{m}+1)} \le n-1$. Applying \eqref{172920_9May24} for $i=m-1$, we get $k_{m-1} + c^{(k_{m-1}+1)} \le k_{m} + c^{(k_{m}+1)} - 1 \le n-2$. Repeating this, we obtain \eqref{014925_24Jul24}. \end{proof} \begin{teiri}\label{223851_4Jun24} For $n > d \ge 1$, there exists an $[n,d]$ REP code $\Cc^{(n)}$ of size $|\Cc^{(n)}| = \prod_{j=0}^{n-1}(\lfloor j/d\rfloor+1)$. \end{teiri} \begin{proof} We construct a REP code $\Cc^{(n)}$ by choosing $S^{(j)} \subset [j+1]$ such that $|S^{(j)}| = \lfloor j/d \rfloor + 1, \quad S^{(j)} := \{0, d, 2d, \ldots, (|S^{(j)}|-1)d\} \quad \text{for} \; j = 0, \ldots, n-1$. We see that $\dmin(S^{(j)}) = \infty$ for $0 \le j < d$, and $\dmin(S^{(j)}) = d$ for $d \le j < n$. From Thm.~\ref{230931_30Oct23}, we understand that such $S^{(j)}$ are the largest possible sets that satisfy $\dmin(S^{(j)}) \ge d$. The subsequent result is obtained by applying Thm.~\ref{205810_6Jul23} and Thm.~\ref{163227_14May23} repeatedly: $|\Cc^{(n)}| = \prod_{j=0}^{n-1} |S^{(j)}| = \prod_{j=0}^{n-1} (\lfloor j/d \rfloor + 1)$, $\dmin(\Cc^{(j)}) = \infty$ for $0 \le j \le d$ and $\dmin(\Cc^{(j)}) \ge d$ for $d < j \le n$. \end{proof} Recall Sec.~\ref{212515_27Aug24}. The size of $[n,d]$ optimal code size is the same as the size of $[n,d]$ DPGP codes whose size is $\prod_{j=0}^{n-1}(\lfloor j/d\rfloor+1)$. \section{Encoding and Decoding Algorithms} In this section, we present several encoding algorithms of REP code $\Cc^{(n)}=\<S^0,\ldots,S^{n-1}\>$. We consider $(s^{(0)},\ldots,s^{(n-1)})\in S^{(0)}\times\cdots\times S^{(n-1)}$ as input to the encoder\footnote{ The size of the code $\Cc^{(n)}$ constructed by $S^{(0)},\ldots,S^{(n-1)}$ is given by $\prod_{j=0}^{n-1}|S^{(j)}|$, as we recall. We represent the message array $\underline{x} = \left(x_0, \ldots, x_{n-1}\right)$, where each $x_j$ is independently chosen from $[|S^{(j)}|]$. We denote the $x_j$-th smallest element in $S^{(j)}$ as $s^{(j)}$. Since, $(x_0, \ldots, x_{n-1})$ and $(s_0, \ldots, s_{n-1})$ correspond one-to-one in this mapping for given $S^{(0)},\ldots,S^{(n-1)}$, we can consider $\sU$ as input. }. \subsection{Natural Encoding Algorithm} \label{182618_25May23} The codewords of $\Cc^{(j+1)}$ are generated by extending the codewords of the $j$-th code $\Cc^{(j)},$ using each element of $S^{(j)}$. By considering the freedom in the selection of each element in $S^{(j)}$ as message, the following {\itshape natural encoding algorithm} is derived. Recall that $\Cc^{(j)}=\phi(\Cc^{(j-1)};S^{(j-1)})$ is defined recursively. Thus, the codeword $\piU^{(j)}$ of $\Cc^{(j)}$ can be expressed as $\piU^{(j)}=\phi(\piU^{(j-1)};s^{(j-1)})$ with $\piU^{(j-1)}\in \Cc^{(j-1)}$ and $s^{(j-1)}\in S^{(j-1)}$. From this observation, it is evident that all codewords of $\Cc^{(n)}$ are exhaustively generated by the naturally defined encoding algorithm. We use $s_j \in S^{(j)}$ for $j \in [n]$ as input to the encoder. Equivalently, we can use $x_j \in [|S^{(j)}|]$ for $j \in [n]$ as the input, where $s_j$ is the $x_j$-th smallest element in $S^{(j)}$. This yields $\piU^{(n)}$ as a codeword of $\Cc^{(n)}$. We denote this encoder, with some abuse of notation, as $\piU^{(n)} := \Cc^{(n)}(\sU)$. \input{natural_encode_figure} \begin{figure}[!t] \begin{algorithm}[H] \caption{Natural Encoding Algorithm of $\Cc^{(n)}$} \begin{algorithmic}[1]\label{natural_encode} \renewcommand{\algorithmicrequire}{\textbf{Input:}} \renewcommand{\algorithmicensure}{\textbf{Output:}} \REQUIRE $ (s^{(0)} , \dotsc, s^{(n-1)})\in S^{(0)}\times\cdots\times S^{(n-1)}$\\ \ENSURE $(\pi_0^{(n)} , \dotsc, \pi_{n-1}^{(n)})\in \Cc^{(n)}$ \FOR {$j:= 1$ to $n$} \STATE $\pi^{(j)} _0 := s^{(j-1)}$ \ENDFOR \FOR {$k := 1$ to $n-1$} \FOR {$j:= k$ to $n$}\label{141220_6Aug23} \STATE $\pi_k^{(j)}:=\phi\left(\pi_{k-1}^{(j-1)} ; s^{(j-1)}\right)$\label{141312_6Aug23} \ENDFOR\label{141314_6Aug23} \ENDFOR \end{algorithmic} \end{algorithm} \end{figure} The formal component-wise description of this encoder is given in Alg.~\ref{natural_encode}. In Fig.~\ref{125759_3Aug23}, we depict the dependencies of each variable that appears in this algorithm for the case of $n=8$. Although natural encoding algorithms are simple, it requires computational complexity of $O(n^2)$. \subsection{Sequential Encoding Algorithm} \label{182606_25May23} For a given encoding algorithm $\xU \mapsto \piU^{(n)}$ for the recursively extended code $\Cc^{(n)}$, the algorithm is said to be sequential if the following condition is met: for each $j\in [n]$ the algorithm determines the $j$-th output $\pi_j^{(n)}$ based on the input $x_j$ and some state variables. The computational order can be rearranged to make natural encoding algorithms sequential. Specifically, the components depicted in Fig.~\ref{125759_3Aug23}, originally calculated from bottom to top, can alternatively be computed from left to right, thereby rendering the algorithm sequential. Despite these modifications, the computational complexity remains $O(n^2)$. In this subsection, we propose an efficient sequential encoding algorithm with computational cost $O(n\log n)$ So far, we have considered $\phi_s(\cdot)$ as a map $\Sc_n\to \Sc_n$ or a map $[n]\to[n+1]$, for head $s\in [n+1]$ We now extend the domain of $\phi_s(\cdot)$ to permutations on $[n]$ without duplicate elements, as follows. For a set $A \subset [n-1]$, define $\phi_s(A) \defeq \{\phi_s(a) \mid a \in A\}\cup\{s\}$ We denote $\min_{r\text{ th}}(A)$ or $\min(A; r)$ denote the $r$-th smallest element in the array $A$, where the smallest element is denoted as $\min(A; 0)$. \begin{lem}\label{181528_2Aug23} Let $\piU^{(n)}\in \Sc_{n}$ and $s,r\in [n]$. Then, the following holds: \begin{align} \phi_s(\min_{r\text{ th}} (\piU^{(n)}))=\min_{r\text{ th}}\bigl(\phi_s' (\piU^{(n)})\bigr), \label{223340_2Aug23} \end{align} where we define $\phi_s'(A) \defeq \{\phi_s(a) \mid a \in A\}$. \end{lem} \begin{proof} Let the elements of \( \piU^{(n)} \) be enumerated in ascending order as \( \sigma_0 < \cdots < \sigma_{n-1} \). Then the LHS of \eqref{223340_2Aug23} is \( \phi_s(\sigma_r)\). Recalling that \( \phi_s(\sigma_r) = \sigma_r + \mathbbm{1}[\sigma_r \ge s] \), it is evident that \( \phi_s(\cdot) \) preserves the order: $\phi_s(x)<\phi_s(y)$ if $x<y$. Since $\phi_s' (\piU^{(n)})= \{\phi_s(\sigma_0),\ldots,\phi_s(\sigma_{n-1})\}$, enumerating the elements of \( \phi_s' (\piU^{(n)}) \) in ascending order yields: $ \phi_s(\sigma_0) < \cdots < \phi_s(\sigma_{n-1})$. Consequently, the RHS of \eqref{223340_2Aug23} is \( \phi_s(\sigma_r) \). \end{proof} Thus far, we have represented a permutation $\fU := [f_0, \ldots, f_{n-1}] \in \Sc_n$ as an array. However, in the following lemma, we will also interpret it as a set of elements for simplicity. To simplify notation, for a set $X \subset [n]$, let $\overline{X}^{[n]} := [n] \setminus X$. \begin{lem}\label{224537_2Aug23} For any $A \subset [n-1]$ and $s\in [n-1]$, it holds that $\phi_s'(\overline{A}^{[n-1]}) = \overline{\phi_s(A)}^{[n]}.$ \end{lem} \begin{proof} For disjoint sets $X$ and $Y$, we write $X \oplus Y$ instead of $X \cup Y$. Since $\phi'_s(\cdot)$ is a bijection from $[n-1]$ to $[n] \setminus \{s\}$, we can partition $[n]$ as follows: $[n] = \phi'_s([n-1]) \oplus \{s\} = \phi'_s(A \oplus \overline{A}^{[n-1]}) = \phi'_s(\overline{A}^{[n-1]}) \oplus \phi'_s(A) \oplus \{s\}. $ From this, the claim immediately follows: $\overline{\phi_s(A)}^{[n]} = \overline{\phi'_s(A) \oplus \{s\}}^{[n]} = \phi'_s(\overline{A}^{[n-1]}).$ \end{proof} Consider Alg.~\ref{214627_21Aug24} for message $\sU$. The following theorem shows that this algorithm functions as the encoder for the code $\Cc^{(n)}$. Specifically, it confirms that the output is identical to that of the natural encoding algorithm. \begin{teiri}\label{153741_1Aug23} Let $\pi_j^{(n)}$ and $\piT_j^{(n)}$ denote the outputs of Alg.~\ref{natural_encode} and Alg.~\ref{214627_21Aug24}, respectively. Then, it holds that $\piT^{(n)}_j = \pi^{(n)}_j$ for any $n > 1$ and $0 \le j < n$. \end{teiri} \begin{proof} For any $n>1$ and $j = 0$, we have $\pi^{(n)}_0 = s^{(n-1)}$, and from Alg.~\ref{214627_21Aug24}, we have $\piT^{(n)}_0 = s_{n-1}$. Therefore, $\piT^{(n)}_j = \pi^{(n)}_j$ for any $n>1$ and $0\le j<n$ holds. We use induction for $j$: we assume that $\piT^{(n-1)}_{j-1} = \pi^{(n-1)}_{j-1}$ for any $n>0$ and derive that $\piT^{(n)}_{j} = \pi^{(n)}_{j}$ for any $n>0$. We have: \begin{align} \phi_{s_{n-1}}(\piU_{[j-1]}^{(n-1)}) &= [s_{n-1}, \phi_{s_{n-1}}(\pi_0^{(n-1)}), \ldots, \phi_{s_{n-1}}(\pi_{j-1}^{(n-1)})] \\ &= [s_{n-1}, \pi_1^{(n)}, \ldots, \pi_j^{(n)}] =: \piU_{[j]}^{(n)} \label{225827_2Aug23}, \end{align} where we denote $\piU_{[j]}^{(n)}$ the array consisting of the first $j$ elements of $\piU^{(n)}$. Since $(n-1)-1-(j-1) = \jO$, we have $\pi_{j-1}^{(n-1)} = \piT_{j-1}^{(n-1)} = \min_{s_{\jO} \text{th}}([n-1] \setminus \tilde{\piU}_{[j-1]}^{(n-1)})$. Applying Lem.~\ref{181528_2Aug23}, we get $\pi_j^{(n)} = \phi_{s_{n-1}}(\pi_{j-1}^{(n-1)}) = \min_{s_{\jO} \text{th}} \Bigl( \phi_{s_{n-1}}'\bigl([n-1] \setminus \tilde{\piU}_{[j-1]}^{(n-1)} \bigr) \Bigr)$. Furthermore, from Lem.~\ref{224537_2Aug23} and \eqref{225827_2Aug23}, we have $\phi'_{s_{n-1}}\bigl([n-1] \setminus \tilde{\piU}_{[j-1]}^{(n-1)} \bigr) = [n] \setminus \phi_{s_{n-1}}(\tilde{\piU}_{[j-1]}^{(n-1)}) = [n] \setminus \tilde{\piU}_{[j]}^{(n)}$. Summarizing the above, we get $\pi_j^{(n)} = \min_{s_{\jO} \text{th}}\bigl([n] \setminus \tilde{\piU}_{[j]}^{(n)} \bigr) = \piT_j^{(n)}$. \end{proof} In Alg.~\ref{214627_21Aug24}, for each index $j$, the process of selecting the $s_j$-th smallest element from the set $[n] \setminus {\pi_0^{(n)}, \ldots, \pi_{j-1}^{(n)}}$ can be performed efficiently. This selection process requires at most $O(\log n)$ steps when implemented with an appropriate search or sorting method. Therefore, the overall computational complexity of the encoding algorithm is $O(n \log n)$. \begin{figure}[!t] \label{002543_29Jun24} \begin{algorithm}[H] \caption{Sequential Encoder of $\Cc^{(n)}=\<S^{(0)},\ldots,S^{(n-1)}\>$} \begin{algorithmic}[1]\label{214627_21Aug24} \renewcommand{\algorithmicrequire}{\textbf{Input:}} \renewcommand{\algorithmicensure}{\textbf{Output:}} \REQUIRE $ (s^{(0)} , \dotsc, s^{(n-1)})\in S^{(0)}\times\cdots\times S^{(n-1)}$\\ \ENSURE $(\piT^{(n)}_0,\ldots,\piT^{(n)}_{n-1})\in \Cc^{(n)}$ \FOR {$j = 0$ to $n-1$} \STATE $\piT^{(n)}_j:=\displaystyle\min_{s_{\jO}\text{ th}}([n]\setminus \{\piT_0^{(n)},\ldots,\piT_{j-1}^{(n)}\})$\label{173310_26Aug24} \ENDFOR \RETURN $(\piT^{(n)}_0,\ldots,\piT^{(n)}_{n-1})$ \end{algorithmic} \end{algorithm} \end{figure} \subsection{Decoding Algorithm of Optimal REP Codes}\label{143813_28Aug23} In Sec.~\ref{161538_26Aug24}, we showed that REP codes $\Cc^{(n)} = \<S^{(0)}, \ldots, S^{(n-1)}\>$ satisfying $\dmin(S^{(j)}) \ge d$ are optimal among $[n, d]$ codes. Let $\sU$ and $\piU$ denote the message and the corresponding codeword. Let $\rhoU$ and $\hat{\sU}$ denote the corresponding received word and the estimated message of the decoder. We propose a decoding algorithm for such codes as described in Alg.~\ref{160732_26Aug24}. The function $\psi_i(\cdot; \cdot)$ defined in line \ref{173558_26Aug24} of this algorithm mirrors the computation described in line \ref{173310_26Aug24} of Alg.~\ref{214627_21Aug24}. It is important to note that $\hat{s}_{\iO} \in S^{(\iO)}$ is chosen so that $\psi_i(\hat{s}_{\iO})$ is closest to $\rho_i$: $|\psi_i(\sH_{\iO})-\rho_{i}|\ge |\psi_i(s_{\iO})-\rho_{i}|$ for any $s_{\iO} \in S^{(\iO)}$. We will show that the decoder can successfully correct any errors as long as $\dinf(\piU, \rhoU) < d/2$, where $\piU$ and $\rhoU$ are the transmitted codeword and the received word, respectively. \begin{figure}[!t] \begin{algorithm}[H] \caption{Sequential Decoder of $\Cc^{(n)}=\<S^{(0)},\ldots,S^{(n-1)}\>$} \begin{algorithmic}[1]\label{160732_26Aug24} \renewcommand{\algorithmicrequire}{\textbf{Input:}} \renewcommand{\algorithmicensure}{\textbf{Output:}} \REQUIRE Received array $(\rho^{(n)}_0,\ldots,\rho^{(n)}_{n-1})$ \ENSURE Estimated message $(\sH_0\in S^{(0)},\ldots,\sH_{n-1}\in S^{(n-1)})$ \FOR {$i = 0$ to $n-1$} \STATE Let $\displaystyle\psi_i(s;\underline{\piH}_{[i]}^{(n)})\defeq\min_{s \text{ th}}[n]\setminus\{\piH_0^{(n)},\ldots,\piH_{i-1}^{(n)}\}$.\label{173558_26Aug24} \STATE $\displaystyle\sH_{\overline{i}}:=\argmin_{s\in S^{(\overline{i})}}|\rho_i^{(n)}-\psi_i(s;\underline{\piH}_{[i]}^{(n)}))|$ \STATE $\piH^{(n)}_i:=\psi_i(\sH_{\overline{i}};\underline{\piH}_{[i]}^{(n)}))$ \ENDFOR \RETURN $(\sH^{(0)},\ldots,\sH^{(n-1)})$ \end{algorithmic} \end{algorithm} \end{figure} \begin{teiri}\label{221904_14Jun24} Consider the setting of optimal REP code and decoder described above. Assume that $|\pi_i - \rho_i| < d/2$ for all $i \in [n]$. Then, it follows that $\hat{\sU} = \sU$. \end{teiri} \begin{proof} Let $s_i$ and $\sH_i$ denote the $i$-th message and its estimate, respectively. We will prove that $\sH_{\overline{i}} = s_{\overline{i}}$ for all $i\in [n]$ by induction. It clear that $\sH_{\overline{0}} = s_{\overline{0}}$. Now, assume that the decoder has correctly estimated up to step $i-1$, specifically: $\sH_{\overline{0}} = s_{\overline{0}}, \ldots, \sH_{\overline{i-1}} = s_{\overline{i-1}}$. We will now derive $\sH_{\overline{i}} = s_{\overline{i}}$. By Alg.~\ref{214627_21Aug24}, we have $\piH_{\overline{0}} = \pi_{\overline{0}}, \ldots, \piH_{\overline{i-1}} = \pi_{\overline{i-1}}$, then, $\pi_{i}=\psi_i(s_{\iO};\hat{\piU}_{[i]}^{(n)}))$. Now, assume for contradiction that $\sH_{\overline{i}} \neq s_{\overline{i}}$. We will derive a contradiction from this assumption. Recall that $\hat{s}_{\iO} \in S^{(\iO)}$ is chosen such that $\psi_i(\hat{s}_{\iO};\underline{\piH}_{[i]}^{(n)})$ is the closest head in $S^{(i)}$ to $\rho_i$. Hence, we have $|\psi_i(\sH_{\iO};\underline{\piH}_{[i]}^{(n)}) - \rho_i| \leq |\psi_i(s_{\iO};\underline{\piH}_{[i]}^{(n)}) - \rho_i| = |\pi_i - \rho_i|$. Since from the premise $|\pi_i - \rho_i| < d/2$, we obtain: $|\psi_i(\sH_{\iO};\underline{\piH}_{[i]}^{(n)}) - \psi_i(s_{\iO};\underline{\piH}_{[i]}^{(n)})| = |\psi_i(\sH_{\iO};\underline{\piH}_{[i]}^{(n)}) - \pi_i| \leq |\psi_i(\sH_{\iO};\underline{\piH}_{[i]}^{(n)}) - \rho_i| + |\pi_i - \rho_i| \leq 2|\pi_i - \rho_i| < d. $ On the other hand, from the premise $\dmin(S^{(i)}) \ge d$ and $\sH_{\iO}, s_{\iO} \in S^{(i)}$, we have $|\sH_{\iO} - s_{\iO}| \geq d$, and from the definition of $\psi(\cdot;\cdot)$, we have $|\psi_i(\sH_{\iO};\underline{\piH}_{[i]}^{(n)}) - \psi_i(s_{\iO};\underline{\piH}_{[i]}^{(n)})| \geq d.$ \end{proof} Since $\dinf(\piU, \rhoU) < d/2$ implies $|\pi_i - \rho_i| < d/2$ for all $i \in [n]$, the condition in Thm.~\ref{221904_14Jun24} can be replaced with $\dinf(\piU, \rhoU) < d/2$. This shows that the performance of this decoder is equivalent to or better than that of the bounded distance decoder. \section{Conclusions} In this paper, we investigate recursively extended permutation (REP) codes. We prove that the optimal REP code matches DPGP codes in terms of size and minimum distance. Moreover, we developed efficient encoding and decoding algorithms for REP codes. \begin{figure}\label{125759_3Aug23} \begin{center} {\scriptsize \begin{tabular}{ @{\hspace{0mm}}l||@{\hspace{2mm}}c@{\hspace{0mm}}c@{\hspace{0mm}}c@{\hspace{0mm}}c@{\hspace{0mm}}c@{\hspace{0mm}}c@{\hspace{0mm}}c@{\hspace{0mm}}c@{\hspace{0mm}}c@{\hspace{0mm}}c@{\hspace{0mm}}c@{\hspace{0mm}}c@{\hspace{0mm}}c@{\hspace{0mm}}c@{\hspace{0mm}}c@{\hspace{0mm}}c@{\hspace{0mm}}c@{\hspace{0mm}}c@{\hspace{0mm}}c@{\hspace{0mm}}c@{\hspace{0mm}}c@{\hspace{0mm}}c@{\hspace{0mm}}c@{\hspace{0mm}}c@{\hspace{0mm}}c@{\hspace{0mm}}c@{\hspace{0mm}}c@{\hspace{0mm}}c@{\hspace{0mm}}c@{\hspace{0mm}} } $\piU^{(8)}$&$s_{(7)}=$&$\pi_0^{(8)}$ && $\pi_1^{(8)}$ && $\pi_2^{(8)}$ && $\pi_3^{(8)}$ && $\pi_4^{(8)}$ && $\pi_5^{(8)}$ && $\pi_6^{(8)}$ && $\pi_7^{(8)}$ \\ $\uparrow_7$&&& $\nearrow\hspace{-3mm}{}^7$&&$\nearrow\hspace{-3mm}{}^7$&&$\nearrow\hspace{-3mm}{}^7$&&$\nearrow\hspace{-3mm}{}^7$&&$\nearrow\hspace{-3mm}{}^7$&&$\nearrow\hspace{-3mm}{}^7$&&$\nearrow\hspace{-3mm}{}^7$&& \\ $\piU^{(7)}$&$s_{(6)}=$&$\pi_0^{(7)}$ && $\pi_1^{(7)}$ && $\pi_2^{(7)}$ && $\pi_3^{(7)}$ && $\pi_4^{(7)}$ && $\pi_5^{(7)}$ && $\pi_6^{(7)}$ && \\ $\uparrow_6$&&& $\nearrow\hspace{-3mm}{}^6$&&$\nearrow\hspace{-3mm}{}^6$&&$\nearrow\hspace{-3mm}{}^6$&&$\nearrow\hspace{-3mm}{}^6$&&$\nearrow\hspace{-3mm}{}^6$&&$\nearrow\hspace{-3mm}{}^6$&& \\ $\piU^{(6)}$&$s_{(5)}=$&$\pi_0^{(6)}$ && $\pi_1^{(6)}$ && $\pi_2^{(6)}$ && $\pi_3^{(6)}$ && $\pi_4^{(6)}$ && $\pi_5^{(6)}$ && \\ $\uparrow_5$&&& $\nearrow\hspace{-3mm}{}^5$&&$\nearrow\hspace{-3mm}{}^5$&&$\nearrow\hspace{-3mm}{}^5$&&$\nearrow\hspace{-3mm}{}^5$&&$\nearrow\hspace{-3mm}{}^5$&& \\ $\piU^{(5)}$&$s_{(4)}=$&$\pi_0^{(5)}$ && $\pi_1^{(5)}$ && $\pi_2^{(5)}$ && $\pi_3^{(5)}$ && $\pi_4^{(5)}$ && \\ $\uparrow_4$&&& $\nearrow\hspace{-3mm}{}^4$&&$\nearrow\hspace{-3mm}{}^4$&&$\nearrow\hspace{-3mm}{}^4$&&$\nearrow\hspace{-3mm}{}^4$&& \\ $\piU^{(4)}$&$s_{(3)}=$&$\pi_0^{(4)}$ && $\pi_1^{(4)}$ && $\pi_2^{(4)}$ && $\pi_3^{(4)}$ && \\ $\uparrow_3$&&& $\nearrow\hspace{-3mm}{}^3$&&$\nearrow\hspace{-3mm}{}^3$&&$\nearrow\hspace{-3mm}{}^3$&& \\ $\piU^{(3)}$&$s_{(2)}=$&$\pi_0^{(3)}$ && $\pi_1^{(3)}$ && $\pi_2^{(3)}$ && \\ $\uparrow_2$&&& $\nearrow\hspace{-3mm}{}^2$&&$\nearrow\hspace{-3mm}{}^2$&& \\ $\piU^{(2)}$&$s_{(1)}=$&$\pi_0^{(2)}$ && $\pi_1^{(2)}$ && \\ $\uparrow_1$&&& $\nearrow\hspace{-3mm}{}^1$&& \\ $\piU^{(1)}$&$s_{(0)}=$&$\pi_0^{(1)}$ && \end{tabular} } \end{center} \caption{Dependency of variables in the natural encoding algorithm for the case of $n=8$. We write $a \xrightarrow{j} b$ when $b = \phi(a; s^{(j)})$ holds.} \end{figure}
2412.04143v1
http://arxiv.org/abs/2412.04143v1
Pin Classes I: Growth Rates and Bounds
\documentclass[11pt,a4paper]{article} \frenchspacing \setlength{\parskip}{9pt plus 3pt minus 1pt} \setlength{\parindent}{0pt} \usepackage[margin=3cm]{geometry} \usepackage{amsmath} \usepackage{amssymb,latexsym} \usepackage{mathabx} \usepackage{amsthm} \usepackage[shortlabels]{enumitem} \usepackage{array} \usepackage{multirow} \usepackage[hang,flushmargin,bottom]{footmisc} \usepackage[margin=0.35in,format=hang,font=small,labelfont=bf]{caption} \usepackage[normalem]{ulem} \usepackage{needspace} \usepackage{graphicx,tikz} \usepackage[colorlinks=true,urlcolor=blue,linkcolor=blue,citecolor=blue]{hyperref} \usepackage{palatino} \usepackage{eulervm} \usetikzlibrary{patterns} \input{tikzperm} \setlength{\textwidth}{6.3in} \setlength{\textheight}{8.7in} \setlength{\topmargin}{0pt} \setlength{\headsep}{0pt} \setlength{\headheight}{0pt} \setlength{\oddsidemargin}{0pt} \setlength{\evensidemargin}{0pt} \newtheorem{thm}{Theorem}[section] \newtheorem*{thm*}{Theorem} \newtheorem{prop}[thm]{Proposition} \newtheorem{cor}[thm]{Corollary} \newtheorem{lemma}[thm]{Lemma} \newtheorem{fact}[thm]{Fact} \newtheorem{obs}[thm]{Observation} \newtheorem{conj}[thm]{Conjecture} \newtheorem*{conj*}{Conjecture} \newtheorem{question}[thm]{Question} \newtheorem{problem}[thm]{Problem} \newtheorem{procedure}[thm]{Procedure} \theoremstyle{definition} \newtheorem{defn}[thm]{Definition} \newtheorem{defn*}{Definition} \newtheorem{example}[thm]{Example} \newtheorem*{example*}{Example} \newtheorem{comment}[thm]{Comment} \newtheorem*{comment*}{Comment} \newcommand{\AAA}{\mathcal{A}} \newcommand{\BBB}{\mathcal{B}} \newcommand{\CCC}{\mathcal{C}} \let\C\CCC \newcommand{\DDD}{\mathcal{D}} \newcommand{\EEE}{\mathcal{E}} \newcommand{\FFF}{\mathcal{F}} \newcommand{\GGG}{\mathcal{G}} \newcommand{\HHH}{\mathcal{H}} \newcommand{\III}{\mathcal{I}} \newcommand{\JJJ}{\mathcal{J}} \newcommand{\KKK}{\mathcal{K}} \newcommand{\LLL}{\mathcal{L}} \newcommand{\MMM}{\mathcal{M}} \newcommand{\NNN}{\mathcal{N}} \newcommand{\OOO}{\mathcal{O}} \newcommand{\PPP}{\mathcal{P}} \newcommand{\QQQ}{\mathcal{Q}} \newcommand{\RRR}{\mathcal{R}} \newcommand{\SSS}{\mathcal{S}} \newcommand{\TTT}{\mathcal{T}} \newcommand{\UUU}{\mathcal{U}} \newcommand{\VVV}{\mathcal{V}} \newcommand{\WWW}{\mathcal{W}} \newcommand{\XXX}{\mathcal{X}} \newcommand{\YYY}{\mathcal{Y}} \newcommand{\ZZZ}{\mathcal{Z}} \newcommand{\Sub}{\text{Sub}} \newcommand{\av}{\mathsf{Av}} \newcommand{\gr}{\mathrm{gr}} \newcommand{\ugr}{\overline{\gr}} \newcommand{\lgr}{\underline{\gr}} \newcommand{\sign}{\mathrm{sgn}} \newcommand{\grup}{\overline{\gr}} \newcommand{\grlow}{\underline{\gr}} \newcommand{\sumclosed}{\raisebox{0.1em}{\scalebox{0.88}{$\bigoplus$}}} \newcommand{\Grid}{\mathsf{Grid}} \newcommand{\Gridhash}{\Grid^\#} \newcommand{\Gridstar}{\Grid^\star} \newcommand{\Geom}{\mathsf{Geom}} \newcommand{\Geomhash}{\Geom^\gridded} \newcommand{\gridded}{\sharp} \newcommand{\sfl}{\mathsf{l}} \newcommand{\sfr}{\mathsf{r}} \newcommand{\sfu}{\mathsf{u}} \newcommand{\sfd}{\mathsf{d}} \newcommand{\tikzcircle}[2][black,fill=black]{\tikz[baseline=-0.5ex]\draw[#1,radius=#2] (0,0) circle ;} \newcommand{\floor}[1]{\left\lfloor #1 \right\rfloor} \newcommand{\rulebreak}{\vspace{24pt}\hrule\vspace{15pt}} \newcommand{\note}[1]{ \marginpar{\raggedright{\footnotesize\it #1}} } \newcommand{\noterb}[1]{\note{\color{red!33!black}\textbf{RB:}~#1}} \newcommand{\commentrb}[1]{{\color{red!33!black}[\emph{\textbf{RB:}~#1}]}} \tikzstyle{vertex}=[circle, draw, fill=black, inner sep=0pt, minimum width=4pt] \newcommand{\smallgrid}{\setplotptradius{2pt}\draw (-.5,0)--++(1,0) (0,-.5)--++(0,1);} \newcommand{\sept}{\tikz[scale=.5,baseline=-3pt]{\smallgrid\node[permpt] at (.25,-.25) {};}} \newcommand{\swpt}{\tikz[scale=.5,baseline=-3pt]{\smallgrid\node[permpt] at (-.25,-.25) {};}} \newcommand{\nept}{\tikz[scale=.5,baseline=-3pt]{\smallgrid\node[permpt] at (.25,.25) {};}} \newcommand{\nwpt}{\tikz[scale=.5,baseline=-3pt]{\smallgrid\node[permpt] at (-.25,.25) {};}} \newcommand{\netwo}{\tikz[scale=.5,baseline=-3pt]{\smallgrid\node[permpt] at (.15,.35) {};\node[permpt] at (.35,.15) {};}} \newcommand{\nwtwo}{\tikz[scale=.5,baseline=-3pt]{\smallgrid\node[permpt] at (-.15,.35) {};\node[permpt] at (-.35,.15) {};}} \newcommand{\swtwo}{\tikz[scale=.5,baseline=-3pt]{\smallgrid\node[permpt] at (-.15,-.35) {};\node[permpt] at (-.35,-.15) {};}} \newcommand{\nthree}{\tikz[scale=.5,baseline=-3pt]{\smallgrid\node[permpt] at (.15,.35) {};\node[permpt] at (.35,.15) {};\node[permpt] at (-.25,.25) {};}} \newcommand{\wthree}{\tikz[scale=.5,baseline=-3pt]{\smallgrid\node[permpt] at (-.15,-.35) {};\node[permpt] at (-.35,-.15) {};\node[permpt] at (-.25,.25) {};}} \newcommand{\nwfive}{\tikz[scale=.5,baseline=-3pt]{\smallgrid\node[permpt] at (.15,.35) {};\node[permpt] at (.35,.15) {};\node[permpt] at (-.15,-.35) {};\node[permpt] at (-.35,-.15) {};\node[permpt] at (-.25,.25) {};}} \newcommand{\plottwocellperm}[2][black]{ \foreach \i [count=\nn] in {#2} {\global\let\n\nn}; \foreach \j [count=\i] in {#2} { \ifnum0=\j { \draw (\i,.5) -- ++(0,\n-1); } \else { \node[permpt,fill=#1,draw=#1] (\j) at (\i,\j) {}; }; } \newcommand{\twocell}[1]{\tikz[scale=.4,baseline=-3pt]{\setplotptradius{2pt} \foreach \i [count=\nn] in {#1} {\global\let\n\nn}; \begin{scope}[yscale=1/(\n-1),xscale=1/3] \foreach \j [count=\i] in {#1} { \ifnum0=\j { \draw (\i,.5-\n/2) -- ++(0,\n-1); } \else { \node[permpt] (\j) at (\i,\j-\n/2) {}; }; \end{scope} }} \title{Pin Classes I: Growth Rates and Bounds} \author{Ben Jarvis\footnote{[email protected]}\\ \small School of Mathematics and Statistics\\ \small The Open University, UK } \begin{document} \maketitle \begin{abstract} Pin sequences play an important role in the structural study of permutation classes. In this paper, we study the permutation classes that comprise all the finite subpermutations contained in an infinite pin sequence. We prove that these permutation classes have proper growth rates and establish a procedure for calculating these growth rates. \end{abstract} \section{Introduction} Since their introduction by Brignall, Huczynska and Vatter~\cite{brignall:decomposing-sim:}, pin sequences have proven to be objects of considerable interest in the study of permutation classes, especially in connection with simple permutations and infinite antichains. Bassino, Bouvel and Rossin~\cite{bassino:enumeration-of-:} showed that the class of all pin sequences has a rational generating function and established its growth rate, and recently, Brignall and Vatter~\cite{bv:wqo-uncountable:} used pin sequences to construct uncountably many well-quasi-ordered permutation classes with distinct enumeration sequences. Given an (infinite) pin sequence, the \emph{pin class} is formed from all finite permutations that are contained in the sequence. In this paper we take up the study of pin classes in a more systematic manner than has previously been attempted. Our main result is that pin classes have \emph{proper} growth rates, not merely upper growth rates as guaranteed by the Marcus-Tardos Theorem~\cite{marcus:excluded-permut:}. We also describe a procedure for finding the growth rate of a given pin class in terms of the recurrent subfactors of the defining pin sequence. Pin classes are most naturally situated in the context of \textbf{centred permutations}, which we define in Section \ref{sec:2}. We shall see in Section \ref{sec:3} that this change of perspective allows us to state a particuarly nice structure theorem for \emph{recurrent} pin classes in terms of the $\boxplus$-sum (an operation specific to centred permutations). This gives us an extraordinarily large family of permutation classes whose generating functions we can find by solving a simple combinatorial problem concerning subfactors of the defining pin sequence. In Section \ref{sec:4} we extend this theory to pin classes defined by non-recurrent pin sequences: in this case we can no longer find generating functions but we can still prove the existence of growth rates and find these by considering a centred class called the $\boxplus$-interior of a pin class. \section{Centred Permutation Classes} \label{sec:2} We refer the reader to Bevan~\cite{bevan2015defs} for background information and basic definitions concerning permutations and permutation classes. \subsection{Centred Permutations} Our main focus in this paper will be a particular method for converting a binary sequence into a permutation class; the resulting permutation classes will be known as \emph{pin classes}. It shall transpire that pin classes are in fact most naturally understood not as classes of permutations in the ordinary sense, but as \emph{centred permutations}. A centred permutation is essentially a permutation with an extra point, designated as the \emph{origin}, which does not contribute to the length of the permutation. A formal definition follows: \begin{defn}[Centred Permutations] A centred permutation of length $n$ is a permutation of length $n+1$ in which one point is designated as the origin, which does not contribute to the length of the centred permutation. \end{defn} \begin{figure}[h] \begin{center} \begin{tikzpicture}[scale=0.35] \node[circle, draw, fill=none, inner sep=0pt, minimum width=\plotptradius] (0) at (0,0) {}; \node[permpt] (1) at (-2,-1) {}; \node[permpt] (2) at (-1,2) {}; \node[permpt] (3) at (1,1) {}; \begin{scope}[shift={(12,0)}] \node[circle, draw, fill=none, inner sep=0pt, minimum width=\plotptradius] (0) at (0,0) {}; \node[permpt] (1) at (-1,-1) {}; \node[permpt] (2) at (1,2) {}; \node[permpt] (3) at (2,1) {}; \end{scope} \end{tikzpicture} \end{center} \caption{Two centred permutations of length $3$. Note that a centred permutation of length $3$ consists of $4$ points (no two of which share either an $x$- or $y$-coordinate) in the plane, of which one is designated as the origin, which does not contribute to the length. Here we denote the origin with an empty circle, whereas the `true' points are represented by solid circles.} \label{fig:centred0} \end{figure} We usually distinguish between centred and uncentred permutations by placing a circle in the superscript: so $\pi$ is a regular permutation and $\pi^{\circ}$ is a centred permutation. (Similarly $\mathcal{C}^{\circ}$ is a class of centred permutations, whereas $\mathcal{C}$ is a class of uncentred permutations.) \begin{figure}[h] \begin{center} \begin{tikzpicture}[scale=0.35] \node[circle, draw, fill=none, inner sep=0pt, minimum width=\plotptradius] (0) at (0,0) {}; \node at (1,1) {1}; \node at (-1,1) {2}; \node at (-1,-1) {3}; \node at (1,-1) {4}; \draw[thick] (0,-2) -- ++ (0,4); \draw[thick] (-2,0) -- ++ (4,0); \end{tikzpicture} \end{center} \caption{The standard quadrant numbering: the origin point of a centred permutation splits the plane into four quadrants, numbered anticlockwise.} \label{fig:quadnumbering} \end{figure} Every centred permutation $\sigma^{\circ}$ of length $n$ is naturally associated with an \emph{underlying} permutation $\sigma$ of length $n$, obtained by removing the origin point from $\sigma^{\circ}$. We also have the \emph{filled-in} permutation $\sigma^{\tikzcircle{1.5pt}}$, the permutation of length $n+1$ obtained by replacing the origin of $\sigma^{\circ}$ with a true point. Though we usually study centred permutation classes in order to understand their associated underlying permutation classes, it is the latter of these notions which provides us with the most convenient notation for a centred permutation: \begin{defn}[One-Line Notation for Centred Permutations] Suppose that $\sigma^{\circ}$ is a centred permutation of length $n$, with corresponding filled-in permutation $\sigma^{\tikzcircle{1.5pt}}$ of length $n+1$. Suppose that the $k$th entry of $\sigma^{\tikzcircle{1.5pt}}$ is the origin of $\sigma^{\circ}$. Then we can write $\sigma^{\circ}$ in one-line notation as follows: \[ \sigma^{\circ} = \sigma^{\tikzcircle{1.5pt}}(1)\sigma^{\tikzcircle{1.5pt}}(2)\dots\underline{\sigma^{\tikzcircle{1.5pt}}(k)}\dots\sigma^{\tikzcircle{1.5pt}}(n)\sigma^{\tikzcircle{1.5pt}}(n+1) \] In other words, this is the one-line notation for $\sigma^{\tikzcircle{1.5pt}}$ with the centre point underlined. See Fig. \ref{fig:centred1} for an example. \end{defn} \begin{figure}[h] \begin{center} \begin{tikzpicture}[scale=0.35] \node[circle, draw, fill=none, inner sep=0pt, minimum width=\plotptradius] (0) at (4,3) {}; \node[permpt] (1) at (1,4) {}; \node[permpt] (2) at (2,2) {}; \node[permpt] (3) at (3,6) {}; \node[permpt] (4) at (5,5) {}; \node[permpt] (5) at (6,1) {}; \draw[thick] (4,0.5) -- ++ (0,6); \draw[thick] (0.5,3) -- ++ (6,0); \end{tikzpicture} \end{center} \caption{The centred permutation $426\underline{3}51$.} \label{fig:centred1} \end{figure} \begin{figure}[h] \begin{center} \begin{tikzpicture}[scale=0.35] \node[circle, draw, fill=none, inner sep=0pt, minimum width=\plotptradius] (0) at (0,0) {}; \node[permpt] (1) at (-2,-1) {}; \node[permpt] (2) at (-1,2) {}; \node[permpt] (3) at (1,1) {}; \draw[thick] (0,-3) -- ++ (0,6); \draw[thick] (-3,0) -- ++ (6,0); \node at (6,0) {$\neq$}; \begin{scope}[shift={(12,0)}] \node[circle, draw, fill=none, inner sep=0pt, minimum width=\plotptradius] (0) at (0,0) {}; \node[permpt] (1) at (-1,-1) {}; \node[permpt] (2) at (1,2) {}; \node[permpt] (3) at (2,1) {}; \draw[thick] (0,-3) -- ++ (0,6); \draw[thick] (-3,0) -- ++ (6,0); \end{scope} \end{tikzpicture} \end{center} \caption{The centred permutations $14\underline{2}3$ and $1\underline{2}43$ are both of length $3$ (the origin point doesn't count) and both have the same underlying (uncentred) permutation $132$, but they are distinct as centred permutations as the origins are in a different place.} \label{fig:centred2} \end{figure} Analogously with the uncentred case we define the notion of centred permutation containment: \begin{defn}[Centred Permutation Containment] Suppose that $\sigma^{\circ}$ is a centred permutation of length $n$ and $\pi^{\circ}$ is a centred permutation of length $m \leq n$. We say that $\pi^{\circ}$ is \emph{contained} in $\sigma^{\circ}$, denoted $\pi^{\circ} \leq \sigma^{\circ}$, if $\pi^{\tikzcircle{1.5pt}}$ can be embedded in $\sigma^{\tikzcircle{1.5pt}}$ in such a way that the origins match up. Explicitly, if \[ \pi^{\circ} = \pi^{\tikzcircle{1.5pt}}(1)\pi^{\tikzcircle{1.5pt}}(2)\dots\underline{\pi^{\tikzcircle{1.5pt}}(k)}\dots\pi^{\tikzcircle{1.5pt}}(m)\pi^{\tikzcircle{1.5pt}}(m+1) \] and \[ \sigma^{\circ} = \sigma^{\tikzcircle{1.5pt}}(1)\sigma^{\tikzcircle{1.5pt}}(2)\dots\underline{\sigma^{\tikzcircle{1.5pt}}(l)}\dots\sigma^{\tikzcircle{1.5pt}}(n)\sigma^{\tikzcircle{1.5pt}}(n+1) \] then $\pi^{\circ} \leq \sigma^{\circ}$ if and only if there is an ascending sequence of $m+1$ indices \begin{equation*} 1 \leq i_{1} < i_{2} < \dots < i_{m+1} \leq n + 1 \end{equation*} such that the sequence $\sigma^{\tikzcircle{1.5pt}}(i_{1})\sigma^{\tikzcircle{1.5pt}}(i_{2})\dots\sigma^{\tikzcircle{1.5pt}}(i_{m+1})$ is order-isomorphic to $\pi^{\tikzcircle{1.5pt}}$ \emph{and} $i_{k} = l$. \end{defn} As with regular permutation containment, centred permutation containment is much easier to understand graphically - see Fig. \ref{fig:centcont} for an example. This is one of the reasons that we generally prefer to draw centered permutations out rather than work with one-line notation. \begin{figure}[h] \begin{center} \begin{tikzpicture}[scale=0.5] \node[circle, draw, fill=none, inner sep=0pt, minimum width=\plotptradius] (0) at (3,2) {}; \node[permpt] (1) at (1,1) {}; \node[permpt] (2) at (2,4) {}; \node[permpt] (3) at (4,3) {}; \draw[thick] (0,2) -- ++ (5,0); \draw[thick] (3,0) -- ++ (0,5); \node at (7,2) {$\leq$}; \begin{scope}[shift={(9,-2)}] \node[circle, red, draw, fill=none, inner sep=0pt, minimum width=\plotptradius] (0) at (5,4) {}; \node[permpt] (1) at (1,7) {}; \node[permpt,red] (2) at (2,1) {}; \node[permpt,red] (3) at (3,8) {}; \node[permpt] (4) at (4,3) {}; \node[permpt,red] (5) at (6,5) {}; \node[permpt] (6) at (7,2) {}; \node[permpt] (7) at (8,6) {}; \draw[thick] (0,4) -- ++ (9,0); \draw[thick] (5,0) -- ++ (0,9); \end{scope} \end{tikzpicture} \end{center} \caption{The centred permutation $14\underline{2}3$ is contained in $7183\underline{4}526$; the relevant embedding is shown by the points marked in red in the diagram of $7183\underline{4}526$. Note that the embedding must make the origins match up.} \label{fig:centcont} \end{figure} Centred permutation containment, of course, immediately allows us to define the centred analogue of a permutation class: \begin{defn}[Centred Permutation Class] A \textbf{centred permutation class} is a non-empty set $\mathcal{C}^{\circ}$ of centred permutations which is downward-closed under centred permutation containment. We call a centred permutation class \textbf{proper} if the set of all underlying (uncentred) permutations of centred permutations in $\mathcal{C}^{\circ}$ does not contain every (uncentred) permutation. \end{defn} Note that $\sigma^{\circ} \leq \pi^{\circ}$ implies that $\sigma \leq \pi$. This means that, for every centred permutation class $\mathcal{C}^{\circ}$, the set $\mathcal{C}$ consisting of the underlying permutations of $\mathcal{C}^{\circ}$ is itself an uncentred permutation class. We call $\mathcal{C}$ the \textbf{underlying} permutation class of $\mathcal{C}^{\circ}$. We wish to use centred permutation classes as a means to study regular permutation classes: there are many interesting permutation classes that are most succinctly described as the underlying class of some given centred permutation class. In particular, we are interested in the growth rates of (centred and uncentred) permutation classes, so we recall the following: \begin{defn}[Growth Rates] Let $(C_{n})$ be a sequence of non-negative integers. The \textbf{upper} and \textbf{lower growth rates} of $(C_{n})$ are defined, respectively, to be \[ \overline{gr}((C_n)) = \limsup_{n\to\infty}\sqrt[n]{C_n}, \quad\text{ and }\quad \underline{gr}((C_n))=\liminf_{n\to\infty}\sqrt[n]{C_n}, \] when these quantities exist. If both quantities exist and $\overline{gr}((C_n))=\underline{gr}((C_n))$, then the sequence has a \textbf{(proper) growth rate}, typically denoted $\gr(C_{n})$. If $\mathcal{C}$ is a (centred or uncentred) permutation class, we define $\mathcal{C}_{n}$ to be the set of permutations of length $n$ in $\mathcal{C}$ and write $C_{n} = |\mathcal{C}_{n}|$. We call $C_{n}$ the \textbf{enumeration sequence} of $\mathcal{C}_{n}$ and say that $\mathcal{C}_{n}$ has (upper, lower, proper) growth rate $\rho$ if $\rho$ is the (upper, lower, proper) growth rate of the enumeration sequence. We write $gr(\mathcal{C})$ for $gr(C_{n})$, and analogously for upper and lower growth rates. \end{defn} We plan to use centred permutation classes to study the growth rates of their underlying (uncentred) permutation classes, so the following is crucial to note: \begin{prop} \label{eqgrs} Suppose that $\mathcal{C}^{\circ}$ is a proper centred permutation class whose underlying permutation class is $\mathcal{C}$. Then: \begin{enumerate} \item The upper growth rate of $\mathcal{C}^{\circ}$ exists and is equal to the upper growth rate of $\mathcal{C}$. \item The lower growth rate of $\mathcal{C}^{\circ}$ exists and is equal to the lower growth rate of $\mathcal{C}$. \item The proper growth rate $gr(\mathcal{C}^{\circ})$ exists if and only if $gr(\mathcal{C})$ exists; if so then $gr(\mathcal{C}^{\circ}) = gr(\mathcal{C})$. \end{enumerate} \end{prop} \begin{proof} As $\mathcal{C}^{\circ}$ is a proper centred permutation class, $\mathcal{C}$ is, by definition, a proper permutation class. Hence by the Marcus-Tardos Theorem~\cite{marcus:excluded-permut:}, $\mathcal{C}$ has an upper growth rate. On noting that a centred permutation can be identified with a $2$-by-$2$ gridded permutation, the fact that $\mathcal{C}^{\circ}$ has the same upper and lower growth rates follows immediately from~{Vatter~\cite[Proposition 2.1]{vatter:small-permutati:}}. (Informally, there are $(n+1)^2$ ways of placing a centre in a permutation of length $n$; given that this factor is polynomial it cannot affect the (exponential) asymptotics of the class.) \end{proof} We also note the following immediate consequence of~{Vatter~\cite[Proposition 2.1]{vatter:small-permutati:}}: \begin{lemma} \label{eqgrs2} Let $\mathcal{C}^{\circ}$ be a proper centred permutation class and let $\mathcal{C}^{\circ+n}$ denote the class consisting of centred permutations in $\mathcal{C}^{\circ}$ with at most $n$ points added anywhere. Then $\overline{gr}(\mathcal{C}^{\circ+n})$ exists and is equal to $\overline{gr}(\mathcal{C}^{\circ})$. Similarly, $\underline{gr}(\mathcal{C}^{\circ+n}) = \underline{gr}(\mathcal{C}^{\circ})$. \end{lemma} We shall also require a centred analogue of the notion of an interval of a permutation: \begin{defn}[$\circ$-intervals of a Centred Permutation] A \textbf{centred-interval} (also written \textbf{$\circ$-interval}) of a centred permutation $\pi^{\circ}$ is an interval containing the origin. We call a centred interval non-trivial if it contains at least one point in addition to the origin. A non-trivial centred interval is \textbf{minimal} if it contains no smaller non-trivial centred interval. \end{defn} \begin{lemma}[Minimal non-trivial $\circ$-intervals in a centred permutation] \label{minint} Let $\pi^{\circ}$ be a centred permutation. Then $\pi^{\circ}$ has either: \begin{itemize} \item one minimal non-trivial $\circ$-interval $\mathcal{I}$; \emph{or} \item two minimal non-trivial $\circ$-intervals $\mathcal{I}_{1}$,$\mathcal{I}_{2}$. In this case, each $\mathcal{I}_{i}$ will contain points in one quadrant only, and from opposite quadrants to each other. \end{itemize} \end{lemma} \begin{proof} Suppose that $\pi^{\circ}$ has two distinct minimal non-trivial $\circ$-intervals $\mathcal{I}_{1}$,$\mathcal{I}_{2}$. Then: \begin{itemize} \item as the intersection of two $\circ$-intervals is a $\circ$-interval, $\mathcal{I}_{1} \cap \mathcal{I}_{2}$ is a $\circ$-interval. \item Hence $\mathcal{I} = \mathcal{I}_{1} \cap \mathcal{I}_{2}$ is a $\circ$-interval contained in both $\mathcal{I}_{1}$ and $\mathcal{I}_{2}$; as these were assumed to be distinct we can deduce that $\mathcal{I}$ is \emph{strictly} contained in at least one of these. Hence, by the assumed minimality of $\mathcal{I}_{1}$ and $\mathcal{I}_{2}$, $\mathcal{I} = \mathcal{I}_{1} \cap \mathcal{I}_{2} = \{\circ\}$ \item Hence $\mathcal{I}_{1}, \mathcal{I}_{2}$ are non-trivial $\circ$-intervals with trivial overlap. Suppose that $\mathcal{I}_{1}, \mathcal{I}_{2}$ both have at least one point in the upper half-plane: say $p_{1} \in \mathcal{I}_{1},p_{2} \in \mathcal{I}_{2}$. Note that $p_{1} \notin \mathcal{I}_{2},p_{2} \notin \mathcal{I}_{1}$ (by $\mathcal{I}_{1} \cap \mathcal{I}_{2} = \{\circ\}$) and that one of $p_{1},p_{2}$ must be lower than the other - without loss of generality, say that $p_{1}$ is lower than $p_{2}$. But then $p_{1}$ is vertically between the origin and $p_{2}$ and hence slices the rectangle $\mathcal{I}_{2}$, contradicting the assumption that $\mathcal{I}_{2}$ is a $\circ$-interval. Hence our supposition is impossible, and $\mathcal{I}_{1}, \mathcal{I}_{2}$ cannot both occupy the upper half-plane. See Fig. \ref{fig:minintervals} for an illustration of this argument. \item By precisely the same logic $\mathcal{I}_{1}, \mathcal{I}_{2}$ cannot both occupy any half-plane, and thus the only possibility remaining is that $\mathcal{I}_{1}$ and $\mathcal{I}_{2}$ are both one-quadrant intervals occupying opposite quadrants, as required. Note that this argument immediately precludes the existence of a third minimal non-trivial $\circ$-interval $\mathcal{I}_{3}$ as this would have to be a one-quadrant interval opposite to both $\mathcal{I}_{1}$ and $\mathcal{I}_{2}$. \end{itemize} \end{proof} \begin{figure}[h] \begin{center} \begin{tikzpicture}[scale=0.35] \node[circle, draw, fill=none, inner sep=0pt, minimum width=\plotptradius] (0) at (0,0) {}; \node[permpt,label={\tiny $p_{1}$}] (1) at (-2,1) {}; \node[permpt,label={\tiny $p_{2}$}] (2) at (1,2) {}; \node[empty,label={\tiny $\pi_{2}^{\circ}$}] (3) at (2.5,0) {}; \draw[thick] (0,-1) -- ++ (0,4); \draw[thick] (-3,0) -- ++ (6,0); \draw[dotted] (-0.5,-0.5) rectangle (1.5,2.5); \end{tikzpicture} \end{center} \caption{The 'interval' $\pi_{2}^{\circ}$ is intersected by the point $p_{1}$ - and hence is not an interval at all.} \label{fig:minintervals} \end{figure} \subsection{The $\boxplus$-decomposition} The following operation on centred permutations will be fundamental in describing the structure of the permutation classes we work with in the next section: \begin{defn}[The Box Sum] Given two centred permutations $\pi^{\circ}$ and $\sigma^{\circ}$ their \textbf{$\boxplus$-sum} (read: \textbf{box-sum}), written $\pi^{\circ} \boxplus \sigma^{\circ}$, is obtained by inflating the origin of $\sigma^{\circ}$ with a copy of $\pi^{\circ}$ - see Fig. \ref{fig:boxsumdef} for an illustration. \end{defn} \begin{figure}[h] \begin{center} \begin{tikzpicture}[scale=0.35] \begin{scope}[shift={(0,4)}] \node[circle, draw, fill=none, inner sep=0pt, minimum width=\plotptradius] (0) at (4,3) {}; \node[permpt] (1) at (5,5) {}; \node[permpt] (2) at (2,4) {}; \node[permpt] (3) at (3,1) {}; \node[permpt] (4) at (1,2) {}; \draw[thin] (4,0.5) -- ++ (0,6); \draw[thin] (0.5,3) -- ++ (6,0); \node at (9,3) {$\boxplus$}; \end{scope} \begin{scope}[shift={(11,2)}] \node[circle, draw, fill=none, inner sep=0pt, minimum width=\plotptradius] (0) at (4,5) {}; \node[permpt] (1) at (3,3) {}; \node[permpt] (2) at (1,4) {}; \node[permpt] (3) at (2,1) {}; \node[permpt] (4) at (5,2) {}; \draw[thin] (4,0.5) -- ++ (0,8); \draw[thin] (0.5,5) -- ++ (7,0); \node at (10,5) {=}; \end{scope} \begin{scope}[shift={(23,0)}] \node[circle, draw, fill=none, inner sep=0pt, minimum width=\plotptradius] (0) at (7,7) {}; \node[permpt] (1) at (8,9) {}; \node[permpt] (2) at (5,8) {}; \node[permpt] (3) at (6,5) {}; \node[permpt] (4) at (4,6) {}; \node[permpt] (1) at (3,3) {}; \node[permpt] (2) at (1,4) {}; \node[permpt] (3) at (2,1) {}; \node[permpt] (4) at (9,2) {}; \draw[thin] (7,0.5) -- ++ (0,12); \draw[thin] (0.5,7) -- ++ (12,0); \draw[dotted] (8.5,0.5) -- ++ (0,12); \draw[dotted] (3.5,0.5) -- ++ (0,12); \draw[dotted] (0.5,9.5) -- ++ (12,0); \draw[dotted] (0.5,4.5) -- ++ (12,0); \end{scope} \end{tikzpicture} \end{center} \caption{The $\boxplus$-sum of the centred permutations $\pi^{\circ} = 241\underline{3}5$ and $\sigma^{\circ} = 413\underline{5}2$ is $413685\underline{7}92$; the origin of $\sigma^{\circ}$ is simply replaced ('inflated') by a copy of $\pi^{\circ}$.} \label{fig:boxsumdef} \end{figure} Fig. \ref{fig:boxsumdef} suggests a close relationship between the $\boxplus$-sum and $\circ$-intervals. Note that if $\mathcal{I}$ is a $\circ$-interval of a centred permutation $\pi^{\circ}$ then $\mathcal{I}$ contains the origin, and so the set of points contained in $\mathcal{I}$ forms a centred permutation $\sigma^{\circ}$. We say that $\mathcal{I}$ \textbf{encloses} the centred permutation $\sigma^{\circ}$, and we immediately observe the following: \begin{obs}[The $\boxplus$-sum and centred intervals] \label{intsum} Let $\pi^{\circ},\sigma^{\circ}$ be (centred) permutations, with $\sigma^{\circ}$ having strictly smaller length than $\pi^{\circ}$. Then: \begin{enumerate} \item $\pi^{\circ} = \sigma^{\circ} \boxplus \tau^{\circ}$ for some centred permutation $\tau^{\circ}$ if and only if $\pi^{\circ}$ contains a (necessarily unique) $\circ$-interval $\mathcal{I}_{\sigma}$ enclosing $\sigma^{\circ}$. \item If $\pi^{\circ} = \sigma^{\circ} \boxplus \tau^{\circ}$ then $\tau^{\circ}$ is the permutation obtained from $\pi^{\circ}$ by deleting all non-origin points contained in the (unique) $\circ$-interval $\mathcal{I}_{\sigma}$. \end{enumerate} \end{obs} We note the following immediate consequence of this result: \begin{cor}[Left-cancellation] Suppose $\sigma^{\circ},\tau^{\circ}_{1},\tau^{\circ}_{2}$ are centred permutations such that \[ \sigma^{\circ} \boxplus \tau^{\circ}_{1} = \sigma^{\circ} \boxplus \tau^{\circ}_{2}. \] Then $\tau^{\circ}_{1} = \tau^{\circ}_{2}$. \end{cor} \begin{proof} Let $\pi^{\circ} = \sigma^{\circ} \boxplus \tau^{\circ}_{1} = \sigma^{\circ} \boxplus \tau^{\circ}_{2}$. Then $\pi^{\circ}$ contains a unique $\circ$-interval $\mathcal{I}_{\sigma}$ enclosing $\sigma^{\circ}$ by Observation \ref{intsum}. Deleting all (non-origin) points in $\mathcal{I}_{\sigma}$ from $\pi^{\circ}$ will result in $\tau^{\circ}_{1}$, but by the same token will also result in $\tau^{\circ}_{2}$; hence $\tau^{\circ}_{1} = \tau^{\circ}_{2}$. \end{proof} Right-cancellation is also easy to prove, but will not be required here. \begin{defn}[$\boxplus$-decomposables and -indecomposables] We say that a centred permutation $\sigma^{\circ}$ is $\boxplus$-\textbf{decomposable} if it can be written as $\pi_{1}^{\circ} \boxplus \pi_{2}^{\circ}$ for two smaller centred permutations $\pi_{1}^{\circ}$ and $\pi_{2}^{\circ}$. Conversely, $\sigma^{\circ}$ is $\boxplus$-\textbf{indecomposable} if it cannot be split up like this. \end{defn} It will be important to note for later that $\sigma^{\circ}$ is $\boxplus$-decomposable if and only if there is a proper, non-trivial $\circ$-interval containing the origin; this is an immediate consequence of Observation \ref{intsum}. For example, the inner rectangle in Fig. \ref{fig:boxsumdef} is an interval because there are no points in the 'shadow' of the rectangle in either the $x$- or $y$-direction. Any $\boxplus$-decomposable centred permutation can be decomposed as a $\boxplus$-sum of $\boxplus$-indecomposables. This decomposition is not necessarily unique, as certain elements commute, as in Fig. \ref{fig:commpair}. \begin{figure}[h] \begin{center} \begin{tikzpicture}[scale=0.35] \node[circle, draw, fill=none, inner sep=0pt, minimum width=\plotptradius] (0) at (2,2) {}; \node[permpt] (2) at (3,4) {}; \node[permpt] (3) at (4,3) {}; \draw[thin] (2,-1) -- ++ (0,6); \draw[thin] (-1,2) -- ++ (6,0); \node at (7,2) {$\boxplus$}; \begin{scope}[shift={(10,0)}] \node[circle, draw, fill=none, inner sep=0pt, minimum width=\plotptradius] (0) at (2,2) {}; \node[permpt] (1) at (1,1) {}; \draw[thin] (2,-1) -- ++ (0,6); \draw[thin] (-1,2) -- ++ (6,0); \node at (7,2) {=}; \end{scope} \begin{scope}[shift={(20,0)}] \node[circle, draw, fill=none, inner sep=0pt, minimum width=\plotptradius] (0) at (2,2) {}; \node[permpt] (1) at (1,1) {}; \node[permpt] (2) at (3,4) {}; \node[permpt] (3) at (4,3) {}; \draw[thin] (2,-1) -- ++ (0,6); \draw[thin] (-1,2) -- ++ (6,0); \node at (7,2) {=}; \end{scope} \begin{scope}[shift={(30,0)}] \node[circle, draw, fill=none, inner sep=0pt, minimum width=\plotptradius] (0) at (2,2) {}; \node[permpt] (1) at (1,1) {}; \draw[thin] (2,-1) -- ++ (0,6); \draw[thin] (-1,2) -- ++ (6,0); \node at (7,2) {$\boxplus$}; \end{scope} \begin{scope}[shift={(40,0)}] \node[circle, draw, fill=none, inner sep=0pt, minimum width=\plotptradius] (0) at (2,2) {}; \node[permpt] (2) at (3,4) {}; \node[permpt] (3) at (4,3) {}; \draw[thin] (2,-1) -- ++ (0,6); \draw[thin] (-1,2) -- ++ (6,0); \end{scope} \end{tikzpicture} \end{center} \caption{The two $\boxplus$-indecomposable centred permutations $\underline{1}32$ and $1\underline{2}$ commute under the $\boxplus$-sum, due to being one-quadrant permutations from opposite quadrants.} \label{fig:commpair} \end{figure} More generally, we have: \begin{lemma}[Commuting elements under the $\boxplus$-sum] \label{commuters}\ \\ Suppose $\pi^{\circ}$ and $\sigma^{\circ}$ are $\boxplus$-indecomposable centred permutations. Then \[ \pi^{\circ} \boxplus \sigma^{\circ} = \sigma^{\circ} \boxplus \pi^{\circ} \] if and only if either $\pi^{\circ} = \sigma^{\circ}$ or $\pi^{\circ}$ and $\sigma^{\circ}$ are one-quadrant permutations from opposite quadrants. \end{lemma} \begin{proof} One direction here is obvious: if either $\pi^{\circ} = \sigma^{\circ}$ or $\pi^{\circ}$ and $\sigma^{\circ}$ are one-quadrant permutations from opposite quadrants then clearly $\pi^{\circ}$ and $\sigma^{\circ}$ commute under the $\boxplus$-sum. For the converse, we assume that the centred permutations $\pi^{\circ}$ and $\sigma^{\circ}$ commute and aim to deduce that they must be of one of these two forms: Consider the centred permutation $\tau^{\circ} = \pi^{\circ} \boxplus \sigma^{\circ} = \sigma^{\circ} \boxplus \pi^{\circ}$. This can be generated in two ways: either by inflating the origin of $\sigma^{\circ}$ by $\pi^{\circ}$ or vice versa. Thus there is a $\circ$-interval $\mathcal{I}_{1}$ of $\tau^{\circ}$ enclosing the permutation $\pi^{\circ}$ and another $\circ$-interval $\mathcal{I}_{2}$ of $\tau^{\circ}$ enclosing the permutation $\sigma^{\circ}$. As $\pi^{\circ}, \sigma^{\circ}$ are both $\boxplus$-indecomposable, $\mathcal{I}_{1}$ and $\mathcal{I}_{2}$ are both minimal non-trivial $\circ$-intervals. Hence, by Lemma \ref{minint}, either $\mathcal{I}_{1} = \mathcal{I}_{2}$, in which case $\pi^{\circ} = \sigma^{\circ}$, or $\mathcal{I}_{1}$ and $\mathcal{I}_{2}$ each contain points in only one quadrant, and from opposite quadrants to each other, in which case $\pi^{\circ},\sigma^{\circ}$ are one-quadrant permutations from opposite quadrants, as required.\end{proof} We shall henceforth call a pair $\{\pi^{\circ},\sigma^{\circ}\}$ of one-quadrant centred permutations from opposite quadrants a \textbf{commuting pair}. In fact, commutativity of these commuting pairs is the \emph{only} way that uniqueness of the $\boxplus$-decomposition can fail, as is demonstrated by the following result: \begin{thm} \label{uniqueness} Let $\pi^{\circ}$ be a centred permutation. Then there exist $\boxplus$-indecomposables $\sigma_{1}^{\circ}, \sigma_{2}^{\circ}, \dots, \sigma_{n}^{\circ}$ such that \[ \pi^{\circ} = \sigma_{1}^{\circ} \boxplus \sigma_{2}^{\circ} \boxplus \dots \boxplus \sigma_{n}^{\circ}. \] Furthermore, this decomposition is unique up to commutativity of adjacent commuting pairs. In other words, if \[ \pi^{\circ} = \tau_{1}^{\circ} \boxplus \tau_{2}^{\circ} \boxplus \dots \boxplus \tau_{k}^{\circ}. \] where $\tau_{1}^{\circ}, \tau_{2}^{\circ}, \dots, \tau_{k}^{\circ}$ are $\boxplus$-indecomposables, then: \begin{enumerate} \item k = n; \item $[\sigma_{1}^{\circ}, \sigma_{2}^{\circ}, \dots, \sigma_{n}^{\circ}] = [\tau_{1}^{\circ}, \tau_{2}^{\circ}, \dots, \tau_{k}^{\circ}]$ as multisets; \item the $n$-tuple $(\sigma_{1}^{\circ}, \sigma_{2}^{\circ}, \dots, \sigma_{n}^{\circ})$ can be transformed into $(\tau_{1}^{\circ}, \tau_{2}^{\circ}, \dots, \tau_{k}^{\circ})$ by a sequence of transpositions of adjacent commuting pairs. \end{enumerate} \end{thm} \begin{proof} On noting that centred permutations are equivalent to $2$-by-$2$-gridded permutations, this follows from~{Bevan, Brignall and Ru\v{s}kuc~\cite[Lemma 3.4]{bbr:unicyclicgrids:}}. We outline a proof here: Existence is clear: just take minimal non-trivial $\circ$-intervals inductively. For uniqueness the key is to prove that if $\sigma_{1}^{\circ} \neq \tau_{1}^{\circ}$ then there is some $l \in \{2,3, \dots, k\}$ such that $\sigma_{1}^{\circ} = \tau_{l}^{\circ}$ and $\sigma_{1}^{\circ} = \tau_{l}^{\circ}$ commutes with $\tau_{j}^{\circ}$ for $j = 1,2, \dots, l-1$. All three claims follow inductively from this. To prove this, note that, as \[ \pi^{\circ} = \tau_{1}^{\circ} \boxplus \tau_{2}^{\circ} \boxplus \dots \boxplus \tau_{k}^{\circ} \] there is, by Lemma \ref{intsum} an increasing sequence of $\circ$-intervals $\mathcal{J}_{1}, \mathcal{J}_{2}, \dots, \mathcal{J}_{k}$ where $\mathcal{J}_{j}$ encloses the centred permutation $\tau_{1}^{\circ} \boxplus \tau_{2}^{\circ} \boxplus \dots \boxplus \tau_{j}^{\circ}$. Also by Lemma \ref{intsum} there is a unique $\circ$-interval $\mathcal{I}$ enclosing $\sigma_{1}^{\circ}$. Note that $\mathcal{J}_{1} \cap \mathcal{I} = \{\circ\}$ (as these are distinct and $\mathcal{I}$ is minimal due to $\boxplus$-indecomposability of $\sigma_{1}^{\circ}$), and so by Lemma \ref{minint} these are both one-quadrant intervals from opposite quadrants - wlog say that $\sigma_{1}^{\circ}$ is entirely contained in quadrant $1$ and $\tau_{1}^{\circ}$ is in quadrant $3$. Now, as $\mathcal{I}$ is minimal $\mathcal{J}_{j} \cap \mathcal{I}$ is either $\{\circ\}$ or $\mathcal{I}$ for all $j$. Further $\mathcal{J}_{k} \cap \mathcal{I} = \mathcal{I}$. As $\mathcal{J}_{j}$ is an increasing sequence of intervals, this implies that there is some $l$ such that $\mathcal{J}_{l} \cap \mathcal{I} = \mathcal{I}$ but $\mathcal{J}_{j} \cap \mathcal{I} = \{\circ\}$ for all $j \leq l-1$. As $\mathcal{J}_{l-1} \cap \mathcal{I} = \{\circ\}$, Lemma \ref{minint} implies that $\tau_{1}^{\circ} \boxplus \tau_{2}^{\circ} \boxplus \dots \boxplus \tau_{l-1}^{\circ}$ is entirely contained in quadrant $3$, so $\sigma_{1}^{\circ}$ commutes with all of $\tau_{j}^{\circ}$ for $j = 1,2, \dots, l-1$. And as $\mathcal{J}_{l} \cap \mathcal{I} = \mathcal{I}$, $\tau_{1}^{\circ} \boxplus \tau_{2}^{\circ} \boxplus \dots \boxplus \tau_{l}^{\circ}$ contains a $\circ$-interval enclosing $\sigma_{1}^{\circ}$. But $\tau_{1}^{\circ} \boxplus \tau_{2}^{\circ} \boxplus \dots \boxplus \tau_{l}^{\circ}$ can be thought of as '$\tau_{l}^{\circ}$ plus some points in the third quadrant', whereas $\sigma_{1}^{\circ}$ is entirely contained in the first quadrant. Hence the interval enclosing $\sigma_{1}^{\circ}$ is in fact entirely contained in $\tau_{l}^{\circ}$, hence $\sigma_{1}^{\circ} \leq \tau_{l}^{\circ}$. But by $\boxplus$-indecomposability of $\tau_{l}^{\circ}$ we conclude that $\sigma_{1}^{\circ} = \tau_{l}^{\circ}$.\end{proof} \subsection{The Generating Function Specification} We say that a centred permutation class $\mathcal{C}^{\circ}$ is \textbf{$\boxplus$-closed} if, for any $\pi^{\circ},\sigma^{\circ} \in \mathcal{C}^{\circ}$, $\pi^{\circ} \boxplus \sigma^{\circ} \in \mathcal{C}^{\circ}$. For any centred permutation class $\mathcal{C}^{\circ}$ we define its \textbf{$\boxplus$-closure}, denoted $\boxplus\mathcal{C}^{\circ}$, to be the smallest $\boxplus$-closed class containing $\mathcal{C}^{\circ}$. Equivalently, $\boxplus\mathcal{C}^{\circ}$ is the set of all centred permutations of the form \[ \sigma^{\circ}_{1} \boxplus \sigma^{\circ}_{2} \boxplus \dots \boxplus \sigma^{\circ}_{n} \] where all $\sigma^{\circ}_{i}$ are $\boxplus$-indecomposables in $\mathcal{C}^{\circ}$. Of course, a centred permutation class is $\boxplus$-closed if and only if it is equal to its own $\boxplus$-closure. We aim in this section to describe a general method for determining the generating function of a $\boxplus$-closed centred permutation class from the enumeration sequence of its $\boxplus$-indecomposables. Suppose we have a $\boxplus$-closed centred permutation class $\mathcal{C}^{\circ}$. Then every element of $\mathcal{C}^{\circ}$ is of the form \[ \pi^{\circ} = \sigma_{1}^{\circ} \boxplus \sigma_{2}^{\circ} \boxplus \dots \boxplus \sigma_{n}^{\circ} \] where each $\sigma_{i}^{\circ}$ is a $\boxplus$-indecomposable in $\mathcal{C}^{\circ}$. Suppose we have the generating function $g(z)$ of the (non-empty) $\boxplus$-indecomposables in $\mathcal{C}^{\circ}$ (crucially, for the class of permutation classes that will be the main focus of this paper, this generating function will be relatively easy to find). \emph{If this decomposition were unique} we could then immediately obtain the generating function of the entire class by applying the \textbf{sequent operator}: \[ \begin{split} f(z) = Seq(f(z)) & = 1 + g(z) + g(z)^{2} + g(z)^{3} + \dots \\ & = \frac{1}{1 - g(z)} \end{split} \] Of course, as was demonstrated in the previous section, this decomposition is \emph{not} in general unique: the issue is that pairs of $\boxplus$-indecomposables from opposite quadrants commute, and so elements of $\mathcal{C}^{\circ}$ cannot be uniquely identified with $n$-tuples of $\boxplus$-indecomposables from the class. As it happens, however, we can easily amend the sequent operator to provide a specification of the generating function of $\mathcal{C}^{\circ}$ in terms of $g(z)$, though, unsurprisingly, we shall also have to use the generating functions $g_{i}(z)$ of the $\boxplus$-indecomposables of $\mathcal{C}^{\circ}$ contained entirely in the $i$th quadrant, for each $i \in \{1,2,3,4\}$, as these are the elements that introduce commutativity into the $\boxplus$-decomposition: \begin{thm}[Generating Function Specification for a $\boxplus$-closed Class]\label{genfuncspec} Let $\mathcal{C}^{\circ}$ be a centred permutation class, with $\boxplus$-closure $\boxplus\mathcal{C}^{\circ}$. Let \[ G(z) = g(z) - g_{1}(z)g_{3}(z) - g_{2}(z)g_{4}(z) \] where $g(z)$ is the generating function of the (non-empty) $\boxplus$-indecomposables of $\mathcal{C}^{\circ}$; and $g_{i}(z)$ is the generating function of the (non-empty) one-quadrant $\boxplus$-indecomposables of $\mathcal{C}^{\circ}$ contained entirely in the $i$th quadrant. Then \[ f(z) = Seq(G(z)) = \frac{1}{1 - G(z)} \] is the generating function of $\boxplus\mathcal{C}^{\circ}$. If $\mathcal{C}^{\circ}$ is itself $\boxplus$-closed then $f(z)$ is in fact the generating function of $\mathcal{C}^{\circ}$. \end{thm} \begin{proof} This follows almost immediately from much more general result of Cartier and Foata~\cite{cartier:problemes-combi:} on trace monoids; see also~\cite[Note V.10]{flajolet:analytic-combin:}. \end{proof} \subsection{The Exponential Growth Theorem} We now have a method for determining the generating function of a $\boxplus$-closed centred permutation class. This may seem initially to be of limited interest, as we are mostly interested in the underlying (uncentred) classes, and these will of course have different generating functions to their centred counterparts. We recall, however, that if $\mathcal{C}^{\circ}$ is a centred permutation class with underlying class $\mathcal{C}$ then $\mathcal{C}^{\circ}$ and $\mathcal{C}$ have the same (upper) growth rate by Proposition \ref{eqgrs}, and by the following consequence of Pringsheim's Theorem we can calculate this from the generating function of $\mathcal{C}^{\circ}$: \begin{thm}[Exponential Growth Theorem, {Flajolet and Sedgewick~\cite{flajolet:analytic-combin:}}]\label{EGT} Suppose that $f(z)$ is the generating function of a proper centred permutation class $\mathcal{C}^{\circ}$. Then the upper growth rate of $\mathcal{C}^{\circ}$ is equal to the reciprocal of the radius of convergence of $f(z)$. \end{thm} \begin{proof} This is an immediate consequence of~\cite[Theorem IV.7]{flajolet:analytic-combin:}, on noting that $f(z)$ certainly has non-negative coefficients. \end{proof} Suppose now that $\mathcal{C}^{\circ}$ is a proper centred permutation class with amended $G$-sequence $G(z)$. Then the generating function of $\boxplus\left(\mathcal{C}^{\circ}\right)$ is given by \[ f(z) = \frac{1}{1 - G(z)} \] By Theorem \ref{EGT} the growth rate of $\boxplus\mathcal{C}^{\circ}$ is then equal to the reciprocal of the radius of convergence of $f(z)$. At first glace it looks like this radius of convergence should be equal to the modulus of the smallest solution of $G(z)=1$ (and we will see that this is true for the classes that we are interested in in this paper) but we need to be careful about $G(z)$ itself possibly having singularities before any solutions of $G(z)=1$. For the time-being we will deal with this issue on a case-by-case basis. \subsection{Examples} Theorems \ref{genfuncspec} and \ref{EGT} are remarkably useful in tandem, suggesting as they do a powerful method for constructing a large number of permutation classes whose growth rates we can easily calculate. For example, the $\boxplus$-closure of a given finite set of $\boxplus$-indecomposables is now easy to calculate, as demonstrated by the following examples: \begin{example}[$\boxplus$-closure of $41\underline{3}52$] $\pi^{\circ} = 41\underline{3}52$ is a $\boxplus$-indecomposable centred permutation. We shall determine the growth rate of its $\boxplus$-closure $\mathcal{C}^{\circ} = \boxplus(\pi^{\circ})$, that is, the smallest $\boxplus$-closed class containing $\pi^{\circ}$. Equivalently, this is the downward closure of the set of centred permutations of the form $\pi^{\circ} \boxplus \pi^{\circ} \boxplus \dots \boxplus \pi^{\circ}$. As $\mathcal{C}^{\circ}$ is (by definition) $\boxplus$-closed we need only enumerate the $\boxplus$-indecomposables in $\mathcal{C}^{\circ}$ in each quadrant. First, the only $\boxplus$-indecomposables contained in $\pi^{\circ}$ (and hence in $\mathcal{C}^{\circ}$) are $\pi^{\circ}$ itself and the four single point centred permutations. Hence the generating function of the $\boxplus$-indecomposables in $\mathcal{C}^{\circ}$ is $g(z) = 4z + z^4$. Of course, as the only single-quadrant $\boxplus$-indecomposables in this list are the four single point permutations, the generating functions $g_{i}(z)$ of the single-quadrant $\boxplus$-indecomposables in $\mathcal{C}^{\circ}$ are given by $g_{1}(z) = g_{2}(z) = g_{3}(z) = g_{4}(z) = z$. Hence the generating function of $\mathcal{C}^{\circ}$ is given by: \[ \begin{split} f(z) & = \frac{1}{1 - [(4z + z^4) - 2z^2]} \\ & = \frac{1}{1 - 4z + 2z^2 - z^4} \end{split} \] And so by Theorem \ref{EGT} we can deduce that the growth rate of $gr(\mathcal{C}^{\circ})$ is the reciprocal of the modulus of the smallest root of the denominator, which is $\approx 3.44372$. \end{example} \begin{example}[The $\mathcal{X}$-class] Next, we turn to a class introduced by Waton~\cite{waton:on-permutation-:} in the context of picture classes: the $\mathcal{X}$-class, consisting of all permutations which can be drawn on an $X$. Waton showed that this class has generating function \[ \frac{z(1 - 2z)}{1 - 4z + 2z^2} \] and hence growth rate $2 + \sqrt{2} \approx 3.41421$ We shall show that this growth rate (but not the generating function) can be derived quickly by instead considering the \emph{centred} permutation class $\mathcal{X}^{\circ}$, defined to be the $\boxplus$-closure of the four single point centred permutations: $\mathcal{X}^{\circ} = \boxplus\{\nept,\nwpt,\swpt,\sept\}$. Clearly $\mathcal{X}^{\circ}$ has $\mathcal{X}$ as its underlying (uncentred) class: we can think of this as simply adding an origin point at the centre of the $X$ diagram used to define $\mathcal{X}$. But then for $\mathcal{X}^{\circ}$ we clearly have $g(z) = 4z$ and $g_{1}(z) = g_{2}(z) = g_{3}(z) = g_{4}(z) = z$; hence $\mathcal{X}^{\circ}$ has generating function: \[ \frac{1}{1 - 4z + 2z^2} \] Note that this has the same denominator as the uncentred class $\mathcal{X}$, demonstrating, as expected, that these two classes have the same growth rate. \end{example} \section{Pin Sequences and Pin Classes} \label{sec:3} \subsection{Pin Sequences} We begin with a definition, following Brignall, Ru\v{s}kuc and Vatter~\cite{brignall:simple-permutat:b}: \begin{defn} A \emph{pin sequence} is a word (finite or infinite) over the language \begin{equation*} \mathcal{L}_{P} = \{1,2,3,4\}(\{\text{l},\text{r}\}\{\text{u},\text{d}\})^{*}\cup \{1,2,3,4\}(\{\text{u},\text{d}\}\{\text{l},\text{r}\})^{*} \end{equation*} \end{defn} We tend to refer to a finite pin sequence as a \emph{pin word} and reserve the term 'pin sequence' for the infinite case. For example, $w=3ldrdrdlurdl$ is a pin word and $4(urul)^{*}$ is a pin sequence (we use the notation $(f)^{*}$ to denote the factor $f$ recurring indefinitely, so $4(urul)^{*} = 4urulurulurulurul\dots$). We think of the letters $u,d,l,r$ as representing up, down, left, right, respectively. We then say that letters $u$ and $d$ have \textbf{vertical alignment} and letters $l$ and $r$ have \textbf{horizontal alignment}. We can then informally descrive a pin sequence as consisting of an inital numeral from $\{1,2,3,4\}$ followed by a sequence of letters from $\{u,l,d,r\}$ which must alternate between horizontal and vertical alignment. Note that this implies that there are $4$ pin words of length $1$ and $2^{n+2}$ pin words of every length $n \geq 2$. We shall also have cause to refer to the language \begin{equation*} \mathcal{L}^{*}_{P} = (\{\text{l},\text{r}\}\{\text{u},\text{d}\})^{*}\cup (\{\text{u},\text{d}\}\{\text{l},\text{r}\})^{*} \end{equation*} Note that a pin word consists of an initial numeral between $1$ and $4$ followed by a (possibly empty) word over $\mathcal{L}^{*}_{P}$. A finite pin word $w$ of length $n$ can be converted into a centred permutation $\pi_{w}^{\circ}$ of length $n$ by the following procedure (see Fig. \ref{fig:pinpermexample} for an illustration of this process): \begin{defn}[The $\pi$-map and pin permutations] Let $n \in \mathbb{N}$; furthermore, let $\Pi_{n}$ denote the set of pin words of length $n$, and $S_{n}^{\circ}$ denote the set of centred permutations of length $n$. The $\pi$-\emph{map} is the map \[ \pi^{\circ}: \Pi_{n} \rightarrow S_{n}^{\circ} \] defined by the following procedure: given $w \in \Pi_{n}$: \begin{enumerate} \item Place the initial origin point $p_{0}$; use this point to split the plane into four numbered quadrants, as in Fig. \ref{fig:quadnumbering}. \item Place the first point $p_{1}$ in the quadrant specified by the initial numeral of the pin word $w$. \item Once the first $k-1$ points have been placed, place the point $p_{k}$ either up, down, left or right (depending on the letter $u$, $d$, $l$, or $r$ appearing in the $k$-th place in the pin sequence) of the bounding rectangle of all previous points $\{p_{0},p_{1}, \dots , p_{k-1} \}$ at the end of a 'pin' which separates the last point $p_{k-1}$ from all previous points. \item Once all $n$ points have been placed, read off the centred permutation $\pi^{\circ}_{w}$ given by the points $\{p_{0}, p_{1}, \dots , p_{n} \}$, with $p_{0}$ as the origin. \end{enumerate} We refer to $\pi^{\circ}_{w}$ as the (centred) pin permutation associated with $w$; similarly, we write $\pi_{w}$ for the underlying (uncentred) permutation of $\pi^{\circ}_{w}$ and refer to this as the (uncentred) pin permutation associated with $w$. We refer to a centred permutation $\sigma^{\circ}$ as a (centred) \textbf{pin permutation} if there is some pin word $w$ such that $\sigma^{\circ} \leq \pi_{w}$. Similarly, an uncentred permutation $\sigma$ is an (uncentred) pin permutation if it is the underlying permutation of a centred pin permutation. Finally, we refer to a centred permutation $\sigma^{\circ}$ as a \textbf{contiguous} pin permutation is there is some pin word $w$ such that $\sigma^{\circ} = \pi_{w}$. \end{defn} \begin{figure}[h] \begin{center} \begin{tikzpicture}[scale=0.5] \node[circle, draw, fill=none, inner sep=0pt, minimum width=\plotptradius] (0) at (6,4) {}; \node[permpt,label={\tiny $p_{1}$}] (1) at (5,6) {}; \node[permpt,label={\tiny $p_{2}$}] (2) at (3,5) {}; \draw[thin] (2) -- ++ (2.5,0); \node[permpt,label={\tiny $p_{3}$}] (3) at (4,8) {}; \draw[thin] (3) -- ++ (0,-3.5); \node[permpt,label={\tiny $p_{4}$}] (4) at (8,7) {}; \draw[thin] (4) -- ++ (-4.5,0); \node[permpt,label={\tiny $p_{5}$}] (5) at (7,2) {}; \draw[thin] (5) -- ++ (0,5.5); \node[permpt,label={\tiny $p_{6}$}] (6) at (1,3) {}; \draw[thin] (6) -- ++ (6.5,0); \node[permpt,label={\tiny $p_{7}$}] (7) at (2,1) {}; \draw[thin] (7) -- ++ (0,2.5); \draw[thick] (6,0) -- ++ (0,9); \draw[thick] (0,4) -- ++ (9,0); \end{tikzpicture} \end{center} \caption{The (centred) pin permutation $31586\underline{4}27$ (and its underlying uncentred permutation $3147526$), constructed from the pin sequence $2lurdld$.} \label{fig:pinpermexample} \end{figure} \begin{obs} \label{slicing} By definition the point $p_{k}$ slices the bounding rectangle $rec(p_{0},p_{1}, \dots , p_{k-1})$ of all previous points, including the origin. In fact, the definition implies that $p_{k}$ is the \textbf{only} point after $p_{k-1}$ that slices this rectangle; this means that if we remove $p_{k}$ from the corresponding centred permutation $rec(p_{0},p_{1}, \dots , p_{k-1})$ becomes a $\circ$-interval, and hence the resulting centred permutation is $\boxplus$-decomposable. This will be important in our later structure theorem for a pin class. \end{obs} We denote the class consisting of \emph{all} centred pin permutations $\mathcal{P}^{\circ}$, and its uncentred counterpart by $\mathcal{P}$. We call this the \textbf{complete pin class}. Bassino, Bouvel and Rossin~\cite{bassino:enumeration-of-:} proved that $\mathcal{P}$ has rational generating function and growth rate $\omega_{\infty} \approx 5.24112$. We shall later find the generating function of $\mathcal{P}^{\circ}$, which of course has the same growth rate as $\mathcal{P}$ by Proposition \ref{eqgrs}. We are primarily interested in this paper in certain subclasses of $\mathcal{P}$, known as \textbf{pin classes}. These are defined somewhat analogously to pin permutations: just as we can convert a (finite) pin word into a permutation, we can also convert an (infinite) pin sequence into a pin class: \begin{defn}[Pin Classes] Suppose that $w$ is an infinite pin sequence. Let $\pi^{\circ}_{n}$ be the pin permutation obtained by the above process from the inital contiguous subsequence of $w$ of length $n$ (which is itself a finite pin word), and consider the set $\Pi = \{ \pi^{\circ}_{1}, \pi^{\circ}_{2}, \pi^{\circ}_{3}, \dots \}$. The downward closure of $\Pi$ under the pattern containment order $\leq$ forms a centred permutation class $\mathcal{C}^{\circ}_{w}$, known as the centred \emph{pin class} of $w$. We similarly define the uncentred pin class of $w$, $\mathcal{C}_{w}$, to be the underlying class of $\mathcal{C}^{\circ}_{w}$. \end{defn} Informally, we can think of this as using the infinite pin sequence $w$ to draw an 'infinite diagram' by the same process as in the finite case; the pin class $\C_{w}$ is then the class of all permutations that can be found somewhere within this infinite diagram (by picking a finite collection of points and 'forgetting' everything else). \begin{example} Taking $w=1(ldrdluru)^{*}$ we generate the pin diagram shown in Fig. \ref{fig:fountain}. We could of course extend this diagram indefinitely. The class $\mathcal{C}_{w}$ then consists of all permutations that can be found somewhere in this diagram (with $\mathcal{C}^{\circ}_{w}$ being the class of all \emph{centred} permutations that can be found). \end{example} \begin{figure}[h] \begin{center} \begin{tikzpicture}[scale=0.35] \node[circle, draw, fill=none, inner sep=0pt, minimum width=\plotptradius] (0) at (9,9) {}; \node[permpt,label={\tiny $p_{1}$}] (1) at (10,11) {}; \node[permpt,label={\tiny $p_{2}$}] (2) at (7,10) {}; \draw[thin] (2) -- ++ (3.5,0); \node[permpt,label={\tiny $p_{3}$}] (3) at (8,7) {}; \draw[thin] (3) -- ++ (0,3.5); \node[permpt,label={\tiny $p_{4}$}] (4) at (12,8) {}; \draw[thin] (4) -- ++ (-4.5,0); \node[permpt,label={\tiny $p_{5}$}] (5) at (11,5) {}; \draw[thin] (5) -- ++ (0,3.5); \node[permpt,label={\tiny $p_{6}$}] (6) at (5,6) {}; \draw[thin] (6) -- ++ (6.5,0); \node[permpt,label={\tiny $p_{7}$}] (7) at (6,13) {}; \draw[thin] (7) -- ++ (0,-7.5); \node[permpt,label={\tiny $p_{8}$}] (8) at (14,12) {}; \draw[thin] (8) -- ++ (-8.5,0); \node[permpt,label={\tiny $p_{9}$}] (9) at (13,15) {}; \draw[thin] (9) -- ++ (0,-3.5); \node[permpt,label={\tiny $p_{10}$}] (10) at (3,14) {}; \draw[thin] (10) -- ++ (10.5,0); \node[permpt,label={\tiny $p_{11}$}] (11) at (4,3) {}; \draw[thin] (11) -- ++ (0,11.5); \node[permpt,label={\tiny $p_{12}$}] (12) at (16,4) {}; \draw[thin] (12) -- ++ (-12.5,0); \node[permpt,label={\tiny $p_{13}$}] (13) at (15,1) {}; \draw[thin] (13) -- ++ (0,3.5); \node[permpt,label={\tiny $p_{14}$}] (14) at (1,2) {}; \draw[thin] (14) -- ++ (14.5,0); \node[permpt,label={\tiny $p_{15}$}] (15) at (2,17) {}; \draw[thin] (15) -- ++ (0,-15.5); \node[permpt,label={\tiny $p_{16}$}] (16) at (17,16) {}; \draw[thin] (16) -- ++ (-15.5,0); \draw[thick] (9,0) -- ++ (0,18); \draw[thick] (0,9) -- ++ (18,0); \end{tikzpicture} \end{center} \caption{The pin class $\mathcal{C}^{\circ}_{w}$ generated by the pin sequence $w = 1(ldrdluru)^{*}$.} \label{fig:fountain} \end{figure} \subsubsection{Oscillations} One particular collection of pin permutations is already well-known and will play a key role in our later theory: \begin{defn}[Oscillations] Let $w$ be a (finite) pin word that stays in its initial quadrant (for example, $1ururur$, $1rururur$, $3ldl$ and $4drdrd$). We call the permutation $\pi^{\circ}_{w}$ generated by $w$ an \textbf{oscillation} (sometimes a \textbf{one-quadrant oscillation} for emphasis). \end{defn} Oscillations have natural `staircase' structures - see Fig. \ref{fig:oscexamples} for examples. Note that there are precisely two oscillations of each length $n\geq3$ in each quadrant (eg. those generated by $1ururu$ and $1rurur$ of length $6$ in the first quadrant), but only one of length $2$ (as $1u$ and $1r$ in fact generate the same (centred) permutation \netwo - see Fig. \ref{fig:2osccollision} for an illustration). \begin{figure}[h] \begin{center} \begin{tikzpicture}[scale=0.3] \node[circle, draw, fill=none, inner sep=0pt, minimum width=\plotptradius] (0) at (0,0) {}; \node[permpt] (1) at (2,1) {}; \node[permpt] (2) at (1,3) {}; \draw[thin] (2) -- ++ (0,-2.5); \node[permpt] (3) at (4,2) {}; \draw[thin] (3) -- ++ (-3.5,0); \node[permpt] (4) at (3,5) {}; \draw[thin] (4) -- ++ (0,-3.5); \node[permpt] (5) at (6,4) {}; \draw[thin] (5) -- ++ (-3.5,0); \node[permpt] (6) at (5,6) {}; \draw[thin] (6) -- ++ (0,-2.5); \draw[thick] (-1,0) -- ++ (8,0); \draw[thick] (0,-1) -- ++ (0,8); \node at (0,-2) {$1ururu$}; \node at (0,-4) {$\underline{1}426375$}; \begin{scope}[shift={(12,0)}] \node[circle, draw, fill=none, inner sep=0pt, minimum width=\plotptradius] (0) at (0,0) {}; \node[permpt] (1) at (1,2) {}; \node[permpt] (2) at (3,1) {}; \draw[thin] (2) -- ++ (-2.5,0); \node[permpt] (3) at (2,4) {}; \draw[thin] (3) -- ++ (0,-3.5); \node[permpt] (4) at (5,3) {}; \draw[thin] (4) -- ++ (-3.5,0); \node[permpt] (5) at (4,6) {}; \draw[thin] (5) -- ++ (0,-3.5); \node[permpt] (6) at (6,5) {}; \draw[thin] (6) -- ++ (-2.5,0); \draw[thick] (-1,0) -- ++ (8,0); \draw[thick] (0,-1) -- ++ (0,8); \node at (0,-2) {$1rurur$}; \node at (0,-4) {$\underline{1}352746$}; \end{scope} \begin{scope}[shift={(29,0)}] \node[circle, draw, fill=none, inner sep=0pt, minimum width=\plotptradius] (0) at (0,0) {}; \node[permpt] (1) at (-1,2) {}; \node[permpt] (2) at (-3,1) {}; \draw[thin] (2) -- ++ (2.5,0); \node[permpt] (3) at (-2,4) {}; \draw[thin] (3) -- ++ (0,-3.5); \node[permpt] (4) at (-5,3) {}; \draw[thin] (4) -- ++ (3.5,0); \node[permpt] (5) at (-4,5) {}; \draw[thin] (5) -- ++ (0,-2.5); \draw[thick] (1,0) -- ++ (-7,0); \draw[thick] (0,-1) -- ++ (0,8); \node at (0,-2) {$2lulu$}; \node at (0,-4) {$46253\underline{1}$}; \end{scope} \end{tikzpicture} \end{center} \caption{Three oscillations and the pin words that generate them. Note the two distinct oscillations of length $6$ in the first quadrant. In general, there are two distinct oscillations of each length $n \geq 2$ in each quadrant.} \label{fig:oscexamples} \end{figure} \begin{figure}[h] \begin{center} \begin{tikzpicture}[scale=0.3] \begin{scope}[shift={(10,0)}] \node[circle, draw, fill=none, inner sep=0pt, minimum width=\plotptradius] (0) at (0,0) {}; \node[permpt] (1) at (1,2) {}; \node[permpt] (2) at (2,1) {}; \draw[thin] (2) -- ++ (-1.5,0); \node[empty] (-1) at (0,4) {}; \draw[thick] (-3,0) -- ++ (6,0); \draw[thick] (0,-3) -- ++ (0,6); \node[] (4) at (0,-5) {$1r$}; \end{scope} \node[circle, draw, fill=none, inner sep=0pt, minimum width=\plotptradius] (0) at (0,0) {}; \node[permpt] (1) at (2,1) {}; \node[permpt] (2) at (1,2) {}; \draw[thin] (2) -- ++ (0,-1.5); \draw[thick] (-3,0) -- ++ (6,0); \draw[thick] (0,-3) -- ++ (0,6); \node[] (3) at (5,0) {=}; \node[] (4) at (0,-5) {$1u$}; \end{tikzpicture} \end{center} \caption{Whilst there are two distinct oscillations in each quadrant for all lengths $n \geq 3$, there is only one at length $2$. This is because the pin words $1u$ and $1r$ in fact generate the \emph{same} centred permutation $\underline{1}32$. This is an example of a \textbf{collision} of pin factors, a phenomenon that we will study in detail later.} \label{fig:2osccollision} \end{figure} We can also consider the pin class $\mathcal{O}^{\circ}$ of \textbf{increasing oscillations}, defined to be $\mathcal{C}^{\circ}_{w}$ for the pin word $1(ru)^{*}$ (see Fig. \ref{fig:oscclass}). This class has been shown to have growth rate $\kappa \approx 2.20557$, a fact which we shall be able to prove later. We will also be able to prove that $\kappa$ is in fact the \emph{smallest} possible growth rate of a pin class, and is only achievable by pin classes which are `essentially' $\mathcal{O}^{\circ}$. \begin{figure}[h] \begin{center} \begin{tikzpicture}[scale=0.25] \node[circle, draw, fill=none, inner sep=0pt, minimum width=\plotptradius] (0) at (0,0) {}; \node[permpt] (1) at (1,2) {}; \node[permpt] (2) at (3,1) {}; \draw[thin] (2) -- ++ (-2.5,0); \node[permpt] (3) at (2,4) {}; \draw[thin] (3) -- ++ (0,-3.5); \node[permpt] (4) at (5,3) {}; \draw[thin] (4) -- ++ (-3.5,0); \node[permpt] (5) at (4,6) {}; \draw[thin] (5) -- ++ (0,-3.5); \node[permpt] (6) at (7,5) {}; \draw[thin] (6) -- ++ (-3.5,0); \node[permpt] (7) at (6,8) {}; \draw[thin] (7) -- ++ (0,-3.5); \node[permpt] (8) at (9,7) {}; \draw[thin] (8) -- ++ (-3.5,0); \node[permpt] (9) at (8,10) {}; \draw[thin] (9) -- ++ (0,-3.5); \node[permpt] (10) at (11,9) {}; \draw[thin] (10) -- ++ (-3.5,0); tellipsis {1a} {1b}; tellipsis {6a} {6b} ; \draw[thick] (0,0) -- ++ (10,0); \draw[thick] (0,0) -- ++ (0,12); \end{tikzpicture} \end{center} \caption{The class $\mathcal{O}^{\circ}$ of \textbf{increasing oscillations}, generated by the pin sequence $1(ru)^{*}$.} \label{fig:oscclass} \end{figure} Oscillations have been studied extensively in the literature. In fact, to a significant extent pin permutations are worth studying because they generalise the oscillations and end up being interesting for much the same reasons. For example, Bevan~\cite{bevan:intervals} demonstrated that an infinite antichain contained in the two-point extension $\mathcal{O}^{+2}$ can be used to construct permutation classes (all of which contain long oscillations) at \emph{every} growth rate $\lambda \geq 2.35526$. This construction in fact generalises to pin classes in general, allowing us to construct genuinely distinct antichains at every real number which is the growth rate of a pin class. We thus have established a method for converting a pin sequence into a (centred or uncentred) permutation class. This allows us to describe an extraordinarily large number of classes which would be awkward to describe by other means (it would, for example, take a lot of work to describe the class in Fig. \ref{fig:fountain} in terms of its basis). There are two main reasons that this construction will prove useful. First, controlling the initial pin sequence allows us to control various properties of the permutation class produced (for example, the aforementioned connection with simple permutations allows us to control the number of simples in a given pin class). Second, and most importantly, we have a process for determining the growth rates of the pin permutation classes produced: this will depend on an in-built structure possessed by pin permutation classes, the \textbf{$\boxplus$-decomposition}. \subsection{Pin Factors} We will need to have a notion of a 'subsequence' of a pin sequence, one that should hopefully preserve pattern containment in both the centred and uncentred case: \begin{defn}[Pin Factor] Suppose that $w$ is a (finite or infinite) pin word. A \textbf{pin factor} of $w$ is either: \begin{itemize} \item an initial contiguous subsequence of $w$; or: \item a non-initial contiguous subsequence of $w$ in which the first letter is replaced by the quadrant in which the corresponding point appears in $\pi_{w}^{\circ}$. \end{itemize} \end{defn} Note that a pin factor of a given pin word is by definition itself a pin word. We write $w_{i,j}$ for the pin factor obtained by taking the contiguous subsequence of $w$ between the $i$th and $j$th place. \begin{example} Consider the pin word $w = 2ruldlurdru$. Then $w_{1,2} = 2r$, $w_{1,4} = 2rul$, $w_{1,9} = 2ruldlurd$ and $w$ itself are all pin factors of $w$ as they are all (finite) initial subsequences. If instead we take out the contiguous subsequence from the fifth to ninth terms we obtain the word $dlurd$. This is a factor in the word sense, but it is not a pin factor as it does not begin with a numeral (and so is not a pin word). However, if we draw out the (centred) permutation $\pi_{w}^{\circ}$ generated by $w$ (see Fig. \ref{fig:pinfactorexample}) we can see that the fifth point placed is in the $3$rd quadrant, so we replace the initial $d$ in $dlurd$ with a $3$ to obtain the pin word $3lurd$, which is now a pin factor of $w$; specifically $w_{5,9} = 3lurd$. For an internal pin word we can always work our what this inital numeral should be by looking at the previous letter: for example, if we wish to work out $w_{9,11}$ we first take out the subword $dru$ (between the $9$th and $11$th places); to work out which numeral we replace the initial $d$ with, note that it was preceded by an $r$ and the $d$ in any $rd$ clearly must be placed in the $4$th quadrant. Hence $w_{9,11} = 4ru$. This suggests a correspondence between pin factors of length $n$ and $\mathcal{L}^{*}$-subwords of length $n+1$, stated below in Lemma \ref{pfsubwcorr}. \begin{figure}[h] \begin{center} \begin{tikzpicture}[scale=0.35] \node[circle, draw, fill=none, inner sep=0pt, minimum width=\plotptradius] (0) at (6,5) {}; \node[permpt] (1) at (5,7) {}; \node[permpt] (2) at (8,6) {}; \draw[thin] (2) -- ++ (-3.5,0); \node[permpt] (3) at (7,9) {}; \draw[thin] (3) -- ++ (0,-3.5); \node[permpt] (4) at (3,8) {}; \draw[thin] (4) -- ++ (4.5,0); \node[permpt,red] (5) at (4,3) {}; \draw[thin] (5) -- ++ (0,5.5); \node[permpt,red] (6) at (1,4) {}; \draw[thin] (6) -- ++ (3.5,0); \node[permpt,red] (7) at (2,11) {}; \draw[thin] (7) -- ++ (0,-7.5); \node[permpt,red] (8) at (10,10) {}; \draw[thin] (8) -- ++ (-8.5,0); \node[permpt,red] (9) at (9,1) {}; \draw[thin] (9) -- ++ (0,9.5); \node[permpt] (10) at (12,2) {}; \draw[thin] (10) -- ++ (-3.5,0); \node[permpt] (11) at (11,12) {}; \draw[thin] (11) -- ++ (0,-10.5); \draw[thick] (6,0.5) -- ++ (0,12); \draw[thick] (0.5,5) -- ++ (12,0); \draw[dotted] (3.5,2.5) rectangle (4.5,3.5); \end{tikzpicture} \end{center} \caption{The construction of the (centred) permutation $\pi_{w}^{\circ}$ from the pin word $w = 2ruldlurdru$. Note that the fifth-placed point, $p_{5}$, corresponding to the first $d$ in the word, is in the $3$rd quadrant, so when we extract the word factor $dlurd$ from $w$ we replace the inital $d$ with a $3$ to obtain the pin factor $\tilde{w} = 3lurd$. Note that the (centred) permutation $\sigma_{\tilde{w}}^{\circ}$ (highlighted in red) is a subpermutation of $\pi_{w}^{\circ}$.} \label{fig:pinfactorexample} \end{figure} \end{example} As Fig. \ref{fig:pinfactorexample} suggests, this notion preserves pattern containment: \begin{obs} \label{pinfactorcontainment} Suppose $w_{1},w_{2}$ are (finite) pin words and that $w_{1}$ is a pin factor of $w_{2}$. Then: \[ \pi^{\circ}_{w_{1}} \leq \pi^{\circ}_{w_{2}} \] \end{obs} It's also worth noting that, while pin factors of $w$ of length $n$ cannot in general be identified with subwords of $w$ of length $n$ (for example, the subword $ulur$ could instantiate either $1lur$ or $2lur$ depending on the context) they can \emph{almost} be identified with subwords of length $n+1$ (with the first letter determining the quadrant numeral): the only issue being initial pin factors, which may not reappear. In practice this will be only a minor inconvenience (which may be completely ignored in the recurrent case) as initial pin factors cannot affect the growth rate of a pin class, at worst only affecting the enumeration sequence. We now have the terminology to define an important category of pin sequences, whose associated pin classes (we shall see) have a structure which will be particularly amenable to the enumeration methods developed in Section 1: \begin{defn}[Recurrent Pin Sequences] We refer to an infinite pin sequence $w$ as \textbf{recurrent} if every pin factor that occurs in $w$ occurs infinitely often; similarly, $w$ is \textbf{eventually recurrent} if every pin factor that occurs after a certain point occurs infinitely often. We will often refer to the pin class $\mathcal{C}^{\circ}_{w}$ generated by a recurrent pin sequence as a \textbf{recurrent pin class}. \end{defn} Note that this definition is subtly different from recurrence in the usual (subword) sense, due to complications introduced by initial subsequences (which begin with a numeral): these initial subwords can never occur as subwords again in $w$ by the definition of a pin sequence, but (in order for $w$ to be recurrent) they \emph{must} appear infinitely often a pin factors. This means it can often be difficult to tell whether a pin sequence is recurrent or not, even in the periodic case, until it is drawn out: for example, $2(ul)^{*}$ is recurrent, but $1(ul)^{*}$ is not, as $w_{1,2}=1u$ never reoccurs as a pin factor. We wish to ignore, as far as possible, this subtle and often irritating distinction between subwords and pin factors. First, we call a pin factor of $w$ \emph{fully internal} if it occurs in $w$ starting after the second place; that is, fully internal pin factors are those of the form $w_{i,j}$ where $3 \leq i < j$. Note that fully internal pin factors can be found somewhere in $w$ with a preceeding letter (as opposed to numeral). This letter is enough to reconstruct the initial numeral of the pin factor, thus leading to the following: \begin{lemma} \label{pfsubwcorr} Let $w$ be a pin sequence. Then the number of fully internal pin factors of $w$ of length $n \geq 2$ is equal to the number of $\mathcal{L}^*$-subwords of $w$ of length $n+1$. \end{lemma} Of course, the fully internal restriction can be removed if we further assume that $w$ is recurrent, as in this case any pin factor beginning in the first or second place can also be found later on; hence all pin factors are fully internal, giving us the following simplification: \begin{cor}\label{recpfsubwcorr} Let $w$ be a recurrent pin sequence. Then the number of pin factors of $w$ of length $n$ is equal to the number of $\mathcal{L}^*$-subwords of $w$ of length $n+1$. \end{cor} \subsubsection{Left-truncations} We shall also require an infinite analogue of a pin factor of $w$, the (infinite) pin \emph{sequence} obtained by starting $w$ at a later point: \begin{defn}[Left-truncation of a pin sequence] Let $w$ be a pin sequence and $n \in \mathbb{N}$. The $n$th \textbf{left-truncation} of $w$, $w_{\geq n}$, is the pin sequence obtained from $w$ by replacing the symbol in the $n$th place with the number of the quadrant in which $p_{n}$ is placed, and then removing all of the previous symbols. \end{defn} Informally, $w_{\geq n}$ is `$w$ but started in the $n$th place'. For example, if $w = 3rurdlurur$ then $w_{\geq 4} = 1dlurur$ and $w_{\geq 7} = 2rur$. Again, we can always work out the numeral to replace the letter in the $n$th place with by looking at the previous symbol. Crucially, left-truncating a pin sequence does not affect the asymptotics of its pin class: \begin{lemma}[Finite Prefix Lemma] \label{fplem} Let $w$ be a pin sequence and $n \in \mathbb{N}$. Then $\overline{gr}(\mathcal{C}^{\circ}_{w}) = \overline{gr}(\mathcal{C}^{\circ}_{w_{\geq n}})$ and $\underline{gr}(\mathcal{C}^{\circ}_{w}) = \underline{gr}(\mathcal{C}^{\circ}_{w_{\geq n}})$. \end{lemma} \begin{proof} Recall that we write $\mathcal{C}^{\circ+k}$ for the $k$-point extenstion of $\mathcal{C}^{\circ}$ (that is, the class of centred permutations from $\mathcal{C}^{\circ}$ with at most $k$ extra points added anywhere). Note that the (infinite) pin diagram generated by $w$ is simply that of $w_{\geq n}$ with $n-1$ extra points, hence: \[ \mathcal{C}^{\circ}_{w_{\geq n}} \subseteq \mathcal{C}^{\circ}_{w} \subseteq \mathcal{C}^{\circ+(n-1)}_{w_{\geq n}} \] and by Lemma \ref{eqgrs2} the classes on the left and right have the same (upper and lower) growth rates. \end{proof} We call this the \textbf{Finite Prefix Lemma}, as it tells us that any finite prefix of a pin sequence cannot affect the growth rate of the associated pin class, a fact that we shall use often. \subsection{The Pin Decomposition} The key idea we will use to enumerate pin classes comes from the following observation: when we take a centred permutation generated by a pin word and remove any interior point, we decompose the resulting permutation as the $\boxplus$-sum of two smaller pin permutations. Fig. \ref{fig:boxdecomposition} illustrates this process. \begin{figure}[h] \begin{center} \begin{tikzpicture}[scale=0.3] \node[circle, draw, fill=none, inner sep=0pt, minimum width=\plotptradius] (0) at (7,6) {}; \node[permpt] (1) at (8,8) {}; \node[permpt] (2) at (5,7) {}; \draw[thin] (2) -- ++ (3.5,0); \node[permpt] (3) at (6,4) {}; \draw[thin] (3) -- ++ (0,3.5); \node[permpt] (4) at (3,5) {}; \draw[thin] (4) -- ++ (3.5,0); \node[permpt] (5) at (4,2) {}; \draw[thin] (5) -- ++ (0,3.5); \node[permpt] (6) at (10,3) {}; \draw[thin] (6) -- ++ (-6.5,0); \node[permpt] (7) at (9,10) {}; \draw[thin] (7) -- ++ (0,-7.5); \node[permpt] (8) at (12,9) {}; \draw[thin] (8) -- ++ (-3.5,0); \node[permpt] (9) at (11,12) {}; \draw[thin] (9) -- ++ (0,-3.5); \node[permpt] (10) at (1,11) {}; \draw[thin] (10) -- ++ (10.5,0); \node[permpt] (11) at (2,1) {}; \draw[thin] (11) -- ++ (0,10.5); \draw[thick] (7,0.5) -- ++ (0,12); \draw[thick] (0.5,6) -- ++ (12,0); \draw[dotted] (9.5,2.5) rectangle (10.5,3.5); \node at (15,6) {$\rightarrow$}; \begin{scope}[shift={(17,0)}] \node[circle, draw, fill=none, inner sep=0pt, minimum width=\plotptradius] (0) at (7,6) {}; \node[permpt] (1) at (8,8) {}; \node[permpt] (2) at (5,7) {}; \draw[thin] (2) -- ++ (3.5,0); \node[permpt] (3) at (6,4) {}; \draw[thin] (3) -- ++ (0,3.5); \node[permpt] (4) at (3,5) {}; \draw[thin] (4) -- ++ (3.5,0); \node[permpt] (5) at (4,2) {}; \draw[thin] (5) -- ++ (0,3.5); \node[permpt] (7) at (9,10) {}; \node[permpt] (8) at (12,9) {}; \draw[thin] (8) -- ++ (-3.5,0); \node[permpt] (9) at (11,12) {}; \draw[thin] (9) -- ++ (0,-3.5); \node[permpt] (10) at (1,11) {}; \draw[thin] (10) -- ++ (10.5,0); \node[permpt] (11) at (2,1) {}; \draw[thin] (11) -- ++ (0,10.5); \draw[thick] (7,0.5) -- ++ (0,12); \draw[thick] (0.5,6) -- ++ (12,0); \draw[pattern=crosshatch,pattern color=black!80,draw=none] (0.5,1.5) rectangle (2.5,8.5); \draw[pattern=crosshatch,pattern color=black!80,draw=none] (8.5,1.5) rectangle (11.5,8.5); \draw[pattern=crosshatch,pattern color=black!80,draw=none] (2.5,8.5) rectangle (8.5,12.5); \draw[pattern=crosshatch,pattern color=black!80,draw=none] (2.5,0.5) rectangle (8.5,1.5); \node at (15,6) {=}; \end{scope} \begin{scope}[shift={(34,2)}] \node[circle, draw, fill=none, inner sep=0pt, minimum width=\plotptradius] (0) at (5,4) {}; \node[permpt] (1) at (6,6) {}; \node[permpt] (2) at (3,5) {}; \draw[thin] (2) -- ++ (3.5,0); \node[permpt] (3) at (4,2) {}; \draw[thin] (3) -- ++ (0,3.5); \node[permpt] (4) at (1,3) {}; \draw[thin] (4) -- ++ (3.5,0); \node[permpt] (5) at (2,1) {}; \draw[thin] (5) -- ++ (0,2.5); \draw[thick] (5,0.5) -- ++ (0,6); \draw[thick] (0.5,4) -- ++ (6,0); \node at (9,4) {$\boxplus$}; \end{scope} \begin{scope}[shift={(45,2)}] \node[circle, draw, fill=none, inner sep=0pt, minimum width=\plotptradius] (0) at (3,2) {}; \node[permpt] (1) at (4,4) {}; \node[permpt] (2) at (6,3) {}; \draw[thin] (2) -- ++ (-2.5,0); \node[permpt] (3) at (5,6) {}; \draw[thin] (3) -- ++ (0,-3.5); \node[permpt] (4) at (1,5) {}; \draw[thin] (4) -- ++ (4.5,0); \node[permpt] (5) at (2,1) {}; \draw[thin] (5) -- ++ (0,4.5); \draw[thick] (3,0.5) -- ++ (0,6); \draw[thick] (0.5,2) -- ++ (6,0); \end{scope} \end{tikzpicture} \end{center} \caption{When we remove the point $p_{6}$ from the pin permutation generated by $1ldldruruld$ we remove the only point that slices the bounding rectangle of the first five points; we can thus contract this rectangle down to a single point and in doing so express the resulting centred permutation as the box sum of $\pi^{\circ}_{1ldld}$ and $\pi^{\circ}_{1ruld}$.} \label{fig:boxdecomposition} \end{figure} We formalise this as follows: \begin{lemma}[Removing an interior point from a pin permutation]\ \\ Suppose that $w$ is a pin word of length $n$, and that $p_{1}, p_{2}, \dots, p_{n}$ are the corresponding points of the centred pin permutation $\pi_{w}^{\circ}$. Let $k \in \{2,3,\dots,n-1\}$. Then: \[ \pi_{w}^{\circ} - \{p_{k}\} = \pi_{w_{1,k-1}}^{\circ} \boxplus \pi_{w_{k+1,n}}^{\circ} \] \end{lemma} \begin{proof} We note that by Observation \ref{slicing}, $p_{k}$ is the \emph{only} point after $p_{k-1}$ that slices the bounding rectangle of $p_{0}, p_{1}, \dots, p_{k-1}$. Hence when $p_{k}$ is removed, $rec(p_{0}, p_{1}, \dots, p_{k-1})$ becomes a $\circ$-interval $\mathcal{I}$ in the resulting permuation, enclosing the permutation $\pi^{\circ}_{w_{1,k-1}}$. Hence, by Observation \ref{intsum}: \[ \pi_{w}^{\circ} - \{p_{k}\} = \pi_{w_{1,k-1}}^{\circ} \boxplus \tau^{\circ} \] for some centred permuation $\tau^{\circ}$. That $\tau^{\circ}$ is $\pi_{w_{k+1,n}}^{\circ}$ follows from Observation \ref{pinfactorcontainment}, as this corresponds to the remaining points. \end{proof} This process equips pin classes with an in-built structure theorem: \begin{thm}[The Pin Decomposition] \label{pindec}\ \\ Suppose that $w$ is an infinite pin sequence and that $\C_{w}^{\circ}$ is the pin class it generates. Then \[ \sigma^{\circ} \in \C_{w}^{\circ} \ \text{iff} \ \sigma^{\circ} = \pi_{w_{1}}^{\circ} \boxplus \pi_{w_{2}}^{\circ} \boxplus \dots \boxplus \pi_{w_{k}}^{\circ} \] where $w_{1}, w_{2}, \dots w_{k}$ is a sequence of pin factors of $w$ that occur \textbf{in that order, in non-overlapping instances and separated from each other by at least one letter} in $w$. \end{thm} \begin{proof} This is almost immediate from the process derived above: $\sigma^{\circ} \in \C_{w}^{\circ}$ must be contained in some centred permutation generated by an inital subsequence $w^{*}$ of $w$, so we can obtain $\sigma^{\circ}$ by deleting letters from $w^{*}$; but every time we do we split the resulting permutation into the $\boxplus$-sum of the pin factors directly before and after. Repeatedly doing this gives the required result. \end{proof} We note one immediate consequence of this theorem: \begin{cor}[$\boxplus$-indecomposables in a pin class] \label{pinfactorsareindecs}\ \\ Let $w$ be a pin sequence and $\C_{w}^{\circ}$ the associated pin class. Suppose $\pi^{\circ} \in \C_{w}^{\circ}$ is a $\boxplus$-indecomposable pin permutation. Then $\pi^{\circ} = \pi_{\tilde{w}}^{\circ}$ for some pin factor $\tilde{w}$ of $w$. \end{cor} The structure theorem \ref{pindec} is often awkward to apply due to the conditions on the pin factors $w_{i}$; it becomes much easier however, if we assume that $w$ is a \emph{recurrent} pin sequence - that is, every pin factor of $w$ occurs infinitely often. The theorem then becomes: \begin{thm}[The Pin Decomposition - Recurrent Case]\ \\ Suppose that $w$ is an recurrent infinite pin sequence and that $\C_{w}^{\circ}$ is the pin class it generates. Then: \[ \sigma^{\circ} \in \C_{w}^{\circ} \ \text{iff} \ \sigma^{\circ} = \pi_{w_{1}}^{\circ} \boxplus \pi_{w_{2}}^{\circ} \boxplus \dots \pi_{w_{k}}^{\circ} \] where $w_{1}, w_{2}, \dots w_{k}$ is a sequence of pin factors of $w$, and $\pi_{w_{i}}^{\circ}$ is the (centred) permutation generated from $w_{i}$. \end{thm} This has the following crucial corollary: \begin{cor}[Recurrent pin classes are $\boxplus$-closed]\label{recbox}\ \\ Suppose that $w$ is a recurrent pin sequence. Then the pin class $\mathcal{C}^{\circ}_{w}$ is $\boxplus$-closed. \end{cor} Corollary \ref{recbox} tells us that to enumerate a \emph{recurrent} pin class $\C_{w}^{\circ}$ it will suffice to enumerate its $\boxplus$-indecomposables and then apply the generating function specification given in Theorem \ref{genfuncspec}. It is thus important for us to know how to enumerate the $\boxplus$-indecomposables in a pin class; Corollary \ref{pinfactorsareindecs} suggests a way of doing this that we will study in a later section. \subsection{Recurrent Pin Classes have Growth Rates} We know that pin classes all have \emph{upper} growth rates by Proposition \ref{eqgrs}. In this section we show that \emph{recurrent} pin classes in fact have \emph{proper} growth rates. In fact, as our eventual aim is to show that \emph{all} pin classes have proper growth rates we will have to be slightly more general here and aim to prove that all $\boxplus$-closed subclasses of $\mathcal{P}^{\circ}$ (the class of all pin permutations) have proper growth rates. Our strategy in proving this will be to follow Arratia~\cite{arratia:on-the-stanley-:} who proved that $\oplus$-closed permutation classes have proper growth rates. Arratia's strategy was to note that, in a $\oplus$-closed class $\mathcal{C}$, the map \[ \left(\sigma, \tau\right) \mapsto \sigma \oplus \tau \] is an injection from $\mathcal{C}_{m} \times \mathcal{C}_{n}$ to $\mathcal{C}_{m+n}$, and so the enumeration sequence of $\mathcal{C}$ satisfies the \textbf{supermultiplicative inequality:} \[ C_{m+n} \geq C_{m}C_{n} \] Finally, Arratia applies the supermultiplicative form of Fekete's Lemma to deduce that $gr(C_{n})$ exists. We aim to apply this method to $\boxplus$-closed centred permutation classes, but we have a problem: as we have seen, the $\boxplus$-decomposition is certainly \emph{not} unique and so the map \begin{equation}\label{pairinjection} \begin{split} \mathcal{C}^{\circ}_{m} \times \mathcal{C}^{\circ}_{n} & \rightarrow \mathcal{C}^{\circ}_{m+n} \\ \left(\sigma^{\circ}, \tau^{\circ}\right) & \mapsto \sigma^{\circ} \boxplus \tau^{\circ} \end{split} \end{equation} is not necessarily an injection and we will not, in general, be able to prove the supermultiplicative inequality for a $\boxplus$-closed subclass of $\mathcal{P}^{\circ}$. We note, however, that even the weaker `supermultiplicative-like' inequality $C_{m+n} \geq kC_{m}C_{n}$ (for some constant $k<1$) would be sufficient to deduce the existence $gr(C_{n})$, on applying Fekete's Lemma to $D_{n}=kC_{n}$ instead. Unfortunately, the enumeration sequence of a $\boxplus$-closed subclass of $\mathcal{P}^{\circ}$ does not in general satisfy even this weaker form of supermultiplicativity: for example, if we take $\mathcal{C}^{\circ}$ to be the $\boxplus$-closure of $\nept$ and $\swpt$ we can easily show that $\lim_{n\rightarrow\infty}|\mathcal{C}^{\circ}_{2n}|/|\mathcal{C}^{\circ}_{n}|^{2} = 0$. The problem in this example is that there is \emph{too much commutativity} due to the class only containing points in the first and third quadrants, which are opposite to each other. As it turns out, this is really the only thing that can go wrong in proving a supermultiplicative-like identity: if we have any points from a pair of adjacent quadrants then we already have enough pairs that don't commute to conclude that $C_{m+n} \geq kC_{m}C_{n}$ for some constant $k$. We thus exclude the bad case in which we have only points from opposite quadrants via the following definition: \begin{defn}[Adjacency Condition] Let $\mu^{\circ}_{i}$ denote the centred permutation consisting of a single point in the $i$th quadrant (so $\mu^{\circ}_{1} = \nept$, etc.). We say that a centred permutation class $\mathcal{C}^{\circ}$ satisfies the \textbf{adjacency condition} if it contains either precisely one of $\mu^{\circ}_{1}, \mu^{\circ}_{2}, \mu^{\circ}_{3}, \mu^{\circ}_{4}$ or it contains a pair $\mu^{\circ}_{i}, \mu^{\circ}_{j}$ from adjacent quadrants. \end{defn} Assuming this condition we may now prove a weak form of supermultiplicativity for the enumeration sequence of a $\boxplus$-closed subclass of $\mathcal{P}^{\circ}$: \begin{prop}\label{supermultgen} Let $\mathcal{C}^{\circ}$ be a $\boxplus$-closed subclass of $\mathcal{P}^{\circ}$ which satisfies the adjacency condition. Let $\mathcal{C}^{\circ}_{n}$ denote the set of centred permutations of length $n$ in $\mathcal{C}^{\circ}$ and write $C_{n} = |\mathcal{C}^{\circ}_{n}|$. Then, for all $m,n \in \mathbb{N}$: \[ C_{m+n} \geq C_{m-1}C_{n-1} \] \end{prop} \begin{proof} Note first that certainly $\mu_{i} \in \mathcal{C}^{\circ}$ for some $i$: now, if $\sigma^{\circ} \in \mathcal{C}^{\circ}_{n-1}$ then $\mu_{i} \boxplus \sigma^{\circ} \in \mathcal{C}^{\circ}_{n}$, and by left-cancellation the implied map from $\mathcal{C}^{\circ}_{n-1}$ to $\mathcal{C}^{\circ}_{n}$ is injective. Hence: \[ C_{n} \geq C_{n-1} \] We now split into cases based on how many of the one-point permutations $\mu^{\circ}_{1}, \mu^{\circ}_{2}, \mu^{\circ}_{3}, \mu^{\circ}_{4}$ are contained in $\mathcal{C}^{\circ}$: \begin{itemize} \item \textbf{Case 1:} Suppose $\mathcal{C}^{\circ}$ contains either one or two of $\mu^{\circ}_{1}, \mu^{\circ}_{2}, \mu^{\circ}_{3}, \mu^{\circ}_{4}$. Then, \emph{by the Adjacency Condition}, $\mathcal{C}^{\circ}$ is contained entirely within one half-plane. Hence there is no commutativity in $\mathcal{C}^{\circ}$ and so (by Theorem \ref{uniqueness}) the map \[ \begin{split} \mathcal{C}^{\circ}_{m} \times \mathcal{C}^{\circ}_{n} & \rightarrow \mathcal{C}^{\circ}_{m+n} \\ \left(\sigma^{\circ}, \tau^{\circ}\right) & \mapsto \sigma^{\circ} \boxplus \tau^{\circ} \end{split} \] is an injection, hence \[ \begin{split} C_{m+n} & \geq C_{m}C_{n} \\ & \geq C_{m-1}C_{n-1} \end{split} \] as required. \item \textbf{Case 2:} Suppose $\mathcal{C}^{\circ}$ contains precisely three of $\mu^{\circ}_{1}, \mu^{\circ}_{2}, \mu^{\circ}_{3}, \mu^{\circ}_{4}$. Without loss of generality, assume that these are $\mu^{\circ}_{1}, \mu^{\circ}_{2}, \mu^{\circ}_{3}$. Note that $\mu^{\circ}_{2}$ commutes with nothing in $\mathcal{C}^{\circ}$. Thus \[ \begin{split} \mathcal{C}^{\circ}_{m} \times \mathcal{C}^{\circ}_{n-1} & \rightarrow \mathcal{C}^{\circ}_{m+n} \\ \left(\sigma^{\circ}, \tau^{\circ}\right) & \mapsto \sigma^{\circ} \boxplus \mu^{\circ}_{2} \boxplus \tau^{\circ} \end{split} \] is an injection. Hence: \[ \begin{split} C_{m+n} & \geq C_{m}C_{n-1} \\ & \geq C_{m-1}C_{n-1} \end{split} \] as required. \item \textbf{Case 3:} Suppose $\mathcal{C}^{\circ}$ contains all four of $\mu^{\circ}_{1}, \mu^{\circ}_{2}, \mu^{\circ}_{3}, \mu^{\circ}_{4}$. We claim that \[ \begin{split} \mathcal{C}^{\circ}_{m-1} \times \mathcal{C}^{\circ}_{n-1} & \rightarrow \mathcal{C}^{\circ}_{m+n} \\ \left(\sigma^{\circ}, \tau^{\circ}\right) & \mapsto \sigma^{\circ} \boxplus \mu^{\circ}_{2} \boxplus \mu^{\circ}_{3} \boxplus \tau^{\circ} \end{split} \] is an injection (as nothing can commute with both of the interior permutations). Hence: \[ C_{m+n} \geq C_{m-1}C_{n-1} \] as required. \end{itemize} \end{proof} This, as it stands, is slightly weaker than a supermultiplicativity result (which should relate $C_{m+n}$ with $C_{m}C_{n}$, not $C_{m-1}C_{n-1}$), but we could easily convert it into such if we could obtain a bound on the ratio between consecutive terms in the enumeration sequence of a $\boxplus$-closed subclass of $\mathcal{P}^{\circ}$. To find such a bound we notice that all pin permutations of length $n+1$ can be obtained as `extensions' of pin permutations of length $n$, motivating the following definition: \begin{defn}[Pin Representations] Let $\sigma^{\circ}$ be a pin permutation. We call a $k$-tuple of pin words $\left(w_{1}, w_{2}, \dots, w_{k}\right)$ a \textbf{pin representation of $\sigma^{\circ}$} if \[ \sigma^{\circ} = \pi^{\circ}_{w_{1}} \boxplus \pi^{\circ}_{w_{2}} \boxplus \dots \boxplus \pi^{\circ}_{w_{k}} \] where each $w_{i}$ is a pin word. \end{defn} Clearly, every pin permutation $\sigma^{\circ}$ has a pin representation (as every $\boxplus$-indecomposable is of the form $\pi^{\circ}_{\tilde{w}}$; but note that we do not require that each $\pi^{\circ}_{w_{i}}$ is $\boxplus$-indecomposable). Note, however, that pin representations are in general highly non-unique (it can, for example, easily be seen that $\left(1, 3, 1ul\right)$, $\left(3, 1, 1ul\right)$ and $\left(1, 1uld\right)$ are all representations of the same centred permutation, namely $51\underline{2}364$). We can now formalise the notion of one pin permutation being an extension of another: \begin{defn}[One-point Extensions] Let $\sigma^{\circ}, \widetilde{\sigma}^{\circ} \in \mathcal{P}^{\circ}$ with $|\widetilde{\sigma}^{\circ}| = |\sigma^{\circ}| + 1$. We say that $\widetilde{\sigma}^{\circ}$ is a \textbf{one-point extension of $\sigma^{\circ}$} if there is some pin representation $\left(w_{1}, w_{2}, \dots, w_{k}\right)$ of $\sigma^{\circ}$ for which there exists \textbf{either:} \begin{itemize} \item some $L \in \{u,d,l,r\}$ such that the concatenation $w_{k}L$ is a valid pin word and \[ \left(w_{1}, w_{2}, \dots, w_{k}L\right) \] is a pin representation of $\widetilde{\sigma}^{\circ}$; \textbf{or:} \item some $Q \in \{1,2,3,4\}$ such that \[ \left(w_{1}, w_{2}, \dots, w_{k}, Q\right) \] is a pin representation of $\widetilde{\sigma}^{\circ}$. \end{itemize} \end{defn} (Informally, $\widetilde{\sigma}^{\circ}$ is a one-point extension of $\sigma^{\circ}$ if there is a pin representation of $\sigma^{\circ}$ which can be extended to a pin representation of $\widetilde{\sigma}^{\circ}$ by the appendment of one final symbol, either as an extension of the final pin word in the representation, or as an additional pin word which consists of a single quadrant numeral.) We note the following crucial facts about one-point extensions: \begin{lemma}\label{onepointfacts} Suppose $\mathcal{C}^{\circ}$ is a subclass of the complete pin class $\mathcal{P}^{\circ}$. Then: \begin{enumerate} \item Every $\widetilde{\sigma}^{\circ} \in \mathcal{C}^{\circ}$ of length $n+1$ is a one-point extension of some $\sigma^{\circ} \in \mathcal{C}^{\circ}$ of length $n$. \item Let $\sigma^{\circ} \in \mathcal{C}^{\circ}$. Then $\sigma^{\circ}$ has at most $12$ one-point extensions in $\mathcal{C}^{\circ}$. \end{enumerate} \end{lemma} \begin{proof} \begin{enumerate} \item Let $\widetilde{\sigma}^{\circ} \in \mathcal{C}^{\circ}$ of length $n+1$ and take a pin representation $\left(w_{1}, w_{2}, \dots, w_{k}\right)$ of $\widetilde{\sigma}^{\circ}$. Simply by deleting the final symbol of $w_{k}$ (which may be all of $w_{k}$ if $w_{k}$ is simply a quadrant numeral) we obtain a pin representation of a permutation $\sigma^{\circ}$ which must also be in $\mathcal{C}^{\circ}$ (by Observation \ref{pinfactorcontainment} and closure of $\mathcal{C}^{\circ}$ under the containment order $\leq$) and which has $\widetilde{\sigma}^{\circ}$ as a one-point extension. \item Let $\sigma^{\circ} \in \mathcal{C}^{\circ}$ and suppose that $\widetilde{\sigma}^{\circ}$ is a one-point extension of $\sigma^{\circ}$ (which may or may not be in $\mathcal{C}^{\circ}$). We claim that \emph{regardless of the pin representation chosen for} $\sigma^{\circ}$, $\widetilde{\sigma}^{\circ}$ must be obtained by adding a point in one of the $12$ positions indicated in Fig. \ref{fig:extensionpossibilities}: if we append a numeral $Q$ to the pin representation then $\widetilde{\sigma}^{\circ} = \sigma^{\circ} \boxplus \mu^{\circ}_{Q}$ and we have added a point in one of the extreme corners; if, on the other hand, we append a letter $L$ to the pin representation then we add a point which is the most extreme in the direction indicated by $L$ and the second-most extreme in another (perpendicular) direction, determined by the final symbol of the pin representation of $\sigma^{\circ}$. \end{enumerate} \end{proof} \begin{figure}[h] \begin{center} \begin{tikzpicture}[scale=0.35] \node[circle, draw, fill=none, inner sep=0pt, minimum width=\plotptradius] (0) at (0,0) {}; \node[permpt] (1) at (5,3) {}; \draw[thin] (1) -- ++ (-2.5,0); \node[permpt] (2) at (3,5) {}; \draw[thin] (2) -- ++ (0,-2.5); \node[permpt] (3) at (-3,5) {}; \draw[thin] (3) -- ++ (0,-2.5); \node[permpt] (4) at (-5,3) {}; \draw[thin] (4) -- ++ (2.5,0); \node[permpt] (5) at (-5,-3) {}; \draw[thin] (5) -- ++ (2.5,0); \node[permpt] (6) at (-3,-5) {}; \draw[thin] (6) -- ++ (0,2.5); \node[permpt] (7) at (3,-5) {}; \draw[thin] (7) -- ++ (0,2.5); \node[permpt] (8) at (5,-3) {}; \draw[thin] (8) -- ++ (-2.5,0); \node[permpt] (9) at (5,5) {}; \node[permpt] (10) at (-5,5) {}; \node[permpt] (11) at (-5,-5) {}; \node[permpt] (11) at (5,-5) {}; \draw[thick] (-7,0) -- ++ (14,0); \draw[thick] (0,-7) -- ++ (0,14); \draw[thick] (-4,-4) rectangle (4,4); \end{tikzpicture} \end{center} \caption{If $\sigma^{\circ}$ is contained in the square then \emph{any} one-point extension of $\sigma^{\circ}$ in $\mathcal{C}^{\circ}$ is formed by adding a point in one of the $12$ positions shown.} \label{fig:extensionpossibilities} \end{figure} Lemma \ref{onepointfacts} immediately implies the following: \begin{prop} \label{upratiobound} Let $\mathcal{C}^{\circ}$ be a subclass of the complete pin class $\mathcal{P}^{\circ}$. Let $\mathcal{C}^{\circ}_{n}$ denote the set of centred permutations of length $n$ in $\mathcal{C}^{\circ}$ and write $C_{n} = |\mathcal{C}^{\circ}_{n}|$. Then, for all $n \in \mathbb{N}$: \[ C_{n} \leq 12C_{n-1} \] \end{prop} Finally, we combine Propositions \ref{supermultgen} and \ref{upratiobound} to deduce a supermultiplicativity-like identity for $\boxplus$-closed subclasses of $\mathcal{P}^{\circ}$ satisfying the adjacency condition: \begin{cor}\label{supermultbound} Let $\mathcal{C}^{\circ}$ be a $\boxplus$-closed subclass of $\mathcal{P}^{\circ}$ which satisfies the adjacency condition and let $C_{n} = |\mathcal{C}^{\circ}_{n}|$. Then, for all $m,n \in \mathbb{N}$: \[ C_{m+n} \geq \frac{1}{144}C_{m}C_{n} \] \end{cor} This is finally enough to prove our main result: \begin{thm} \label{grexist} Suppose that $\mathcal{C}^{\circ}$ is a $\boxplus$-closed subclass of $\mathcal{P}^{\circ}$ which satisfies the adjacency condition. Then $\mathcal{C}^{\circ}$ has a proper growth rate. \end{thm} \begin{proof} Let $C_{n}$ be the enumeration sequence of $\mathcal{C}^{\circ}$ and set $D_{n} = \frac{1}{144}C_{n}$. Then, by Corollary \ref{supermultbound}, \[ D_{m+n} \geq D_{m}D_{n}, \] so we may apply Fekete's Lemma to deduce that $\lim_{n\rightarrow\infty}\sqrt[n]{D_{n}}$ exists and is equal to $\sup_{n\in\mathbb{N}}\sqrt[n]{D_{n}}$, which is finite by Marcus-Tardos. Hence: \[ \begin{split} \lim_{n\rightarrow\infty}\sqrt[n]{C_{n}} & = \lim_{n\rightarrow\infty}(\sqrt[n]{144})(\sqrt[n]{D_{n}}) \\ & = \lim_{n\rightarrow\infty}\sqrt[n]{D_{n}} \\ & < \infty, \end{split} \] which is to say that $gr(\mathcal{C}^{\circ})$ exists. \end{proof} As any recurrent pin class is $\boxplus$-closed and satisfies the adjacency condition, we immediately obtain: \begin{cor}[Recurrent pin classes have proper growth rates] \label{recgrs} Let $w$ be a recurrent pin word. Then the pin class $\mathcal{C}^{\circ}_{w}$ has a proper growth rate. \end{cor} Of course, Corollary \ref{recgrs} (along with Proposition \ref{eqgrs}) immediately implies that \emph{uncentred} pin classes generated by recurrent pin sequences also have growth rates, which will equal that of their centred counterparts: $gr(\mathcal{C}_{w}) = gr(\mathcal{C}^{\circ}_{w})$. \subsection{Classification of Collisions and $\boxplus$-decomposables} We now know that any recurrent pin class $\mathcal{C}^{\circ}_{w}$ has a proper growth rate. In order to find the growth rate of a given recurrent pin class in practice we shall have to enumerate the $\boxplus$-indecomposables contained in $\mathcal{C}^{\circ}_{w}$: if we can do this then Theorem (\ref{genfuncspec}) will allow us to write down the generating function of the class, from which we can deduce the growth rate via the Exponential Growth Theorem. Accordingly, in this section we take up the problem of enumerating the $\boxplus$-indecomposables in a pin class. We begin by noting that Corollary \ref{pinfactorsareindecs} immediately implies the following property of the restriction of the $\pi$-map to pin factors of length $n$: \begin{prop}[Induced $\pi$-map contains all $\boxplus$-indecomposables in its image] Let $w$ be a pin sequence and $\mathcal{C}^{\circ}_{w}$ the associated (centred) pin class. Let $P_{n}$ denote the set of pin factors of $w$ of length $n$ and $\mathcal{B}^{\circ}_{n}$ denote the set of $\boxplus$-indecomposables in $\mathcal{C}^{\circ}_{w}$ of length $n$. Then the induced map \[ \pi_{n}: P_{n} \rightarrow \mathcal{C}^{\circ}_{w} \] defined by \[ \pi_{n}: \tilde{w} \mapsto \pi^{\circ}_{\tilde{w}} \] contains the set $\mathcal{B}^{\circ}_{n}$ in its image. \end{prop} The study of this induced map will be vital for further progress as it suggests a natural method for counting the $\boxplus$-indecomposables in a given pin class $\mathcal{C}^{\circ}_{w}$: if (for some $n$) we had the following two properties: \begin{itemize} \item the image $\pi_{n}(P_{n})$ is equal to $\mathcal{B}^{\circ}_{n}$ (that is, $\pi_{n}(\tilde{w})$ is $\boxplus$-indecomposable for all pin factors $\tilde{w}$ of $w$); \item $\pi_{n}$ is injective \end{itemize} then $\pi_{n}$ would be a bijection between $P_{n}$ and $\mathcal{B}^{\circ}_{n}$, thus reducing the problem of enumerating the $\boxplus$-indecomposables of $\mathcal{C}^{\circ}_{w}$ to the (much simpler) combinatorial problem of counting the pin factors of $w$. We do in fact have some intuitive reasons for suspecting that these two properties should hold, at least for sufficently large $n$: the definition of the $\pi$-map (specifically, the placing of each new point in order to intersect the bounding rectangle of all previous points) seems almost designed to ensure that $\pi_{\tilde{w}}$ can have no proper non-trivial $\circ$-interval, and pin permutations have so much internal structure that it seems unlikely that two distinct pin sequences could conspire to generate the exact same centred permutation. Alas, both turn out not to be true for any $n$: for every length $n \geq 2$ there are pin sequences which generate $\boxplus$-decomposable permutations, as well as pairs of pin sequences which generate the same (centred) permutation. Fortunately, as our intuition suggests, both of these 'bad behaviours' are somewhat pathological and easily classified, so with some minor adjustments we will be able to make our strategy for counting $\boxplus$-indecomposables in a pin class work. We begin by introducing some terminology for these pathological behaviours. First, those pin words whose image under the induced $\pi$-map is not contained in $\mathcal{B}^{\circ}_{n}$: \begin{defn}[$\boxplus$-decomposable pin words] We call a (finite) pin word $w$ a \textbf{$\boxplus$-decomposable pin word} if the centred permution it generates, $\pi^{\circ}_{w}$, is $\boxplus$-decomposable. \end{defn} And those pin words which make injectivity of the induced $\pi$-map fail: \begin{defn}[Collisions] We call a set $\{w_{1}, w_{2}, \dots , w_{k}\}$ of pin words which generate the same centred permutation $\pi^{\circ}$ a \textbf{collision} (sometimes a $k$-\textbf{collision} when we wish to emphasise the size of the set) of pin words. \end{defn} It is fairly easy to find all $\boxplus$-decomposables and collisions of length $n\leq5$ through an exhaustive search, especially on applying symmetries of the square. This search reveals families of $\boxplus$-decomposables and collisions which generalise to all longer lengths. We can then, with considerable effort, prove that \emph{all} examples of lengths greater than $6$ belong to these families, giving us a full classification of these confounding behaviours. This is the content of the following two theorems: the statements will be crucial and will be quoted in full here; the proofs are technical and long and will be relegated to the appendix. \newpage \begin{thm}[Classification of $\boxplus$-decomposables and Collisions] \label{classification} The following two tables form a complete list of $\boxplus$-decomposable pin words and collisions: \end{thm} \begin{tabular}{@{}|>{\centering}m{2em}|>{\centering}m{4em}|>{\centering\arraybackslash}m{10em}|>{\centering}m{5em}|>{\centering\arraybackslash}m{6.5em}|>{\centering\arraybackslash}m{6.5em}|} \hline \multicolumn{6}{|c|}{}\\[0.5pt] \multicolumn{6}{|c|}{\textbf{List of $\boxplus$-decomposable pin words:}}\\[6pt] \hline \multicolumn{2}{|c|}{\textbf{Length}}&\textbf{Representative}&\textbf{Total number}&\multicolumn{2}{|c|}{\textbf{Full List}}\\ \hline \multicolumn{2}{|c|}{$2$} & \begin{tikzpicture}[scale=0.25] \node[circle, draw, fill=none, inner sep=0pt, minimum width=\plotptradius] (0) at (2,1) {}; \node[permpt] (1) at (3,3) {}; \node[permpt] (2) at (1,2) {}; \draw[thin] (2) -- ++ (2.5,0); \node[permpt,white] (3) at (2,5) {}; \draw[thick] (0,1) -- ++ (4,0); \draw[thick] (2,0) -- ++ (0,4); \node[] (4) at (2,-1) {$1l$}; \draw[dashed] (0.5,0.5) rectangle (2.5,2.5); \end{tikzpicture} & 8 & \multicolumn{2}{|c|}{$1l, 1d, 2r, 2d, 3u, 3r, 4l, 4u$} \\ \hline \multicolumn{2}{|c|}{$3$} & \begin{tikzpicture}[scale=0.25] \node[circle, draw, fill=none, inner sep=0pt, minimum width=\plotptradius] (0) at (3,2) {}; \node[permpt] (1) at (4,4) {}; \node[permpt] (2) at (1,3) {}; \draw[thin] (2) -- ++ (3.5,0); \node[permpt] (3) at (2,1) {}; \draw[thin] (3) -- ++ (0,2.5); \node[permpt,white] (4) at (3,6) {}; \draw[thick] (0,2) -- ++ (5,0); \draw[thick] (3,0) -- ++ (0,5); \node[] (4) at (3,-1) {$1ld$}; \draw[dashed] (1.5,0.5) rectangle (3.5,2.5); \end{tikzpicture} & 8 & \multicolumn{2}{|c|}{$1ld, 1dl, 2dr, 2rd, 3ru, 3ur, 4ul, 4lu$} \\ \hline \multirow{2}{*}{$n\geq4$} & \textbf{Type 1:} & \begin{tikzpicture}[scale=0.25] \node[circle, draw, fill=none, inner sep=0pt, minimum width=\plotptradius] (0) at (3,2) {}; \node[permpt] (1) at (6,6) {}; \draw[thin] (1) -- ++ (0,-1.5); \node[permpt] (2) at (8,5) {}; \draw[thin] (2) -- ++ (-2.5,0); \node[permpt] (3) at (7,8) {}; \draw[thin] (3) -- ++ (0,-3.5); \node[permpt] (4) at (1,7) {}; \draw[thin] (4) -- ++ (6.5,0); \node[permpt] (5) at (2,1) {}; \draw[thin] (5) -- ++ (0,6.5); \begin{scope}[shift={(0.5,0.5)}] \node[circle,fill,inner sep=0.5pt] (6) at (3.5,2.5) {}; \node[circle,fill,inner sep=0.5pt] (7) at (4,3) {}; \node[circle,fill,inner sep=0.5pt] (8) at (4.5,3.5) {}; \node[circle,fill,inner sep=0.5pt] (8) at (5,4) {}; \end{scope} \node[empty] (-1) at (3,10) {}; \draw[thick] (0,2) -- ++ (9,0); \draw[thick] (3,0) -- ++ (0,9); \node[] (4) at (4,-1.5) {$1(ur)^{k}uld$ \text{or} $1(ru)^{k}ld$}; \draw[dashed] (1.5,0.5) rectangle (3.5,2.5); \end{tikzpicture} & 8 & \textbf{$n \geq 4$ even:} \[ \begin{split} 1(ur)^{k}uld, \\ 1(ru)^{k}rdl, \\ 2(ul)^{k}urd, \\ 2(lu)^{k}ldr, \\ 3(dl)^{k}dru, \\ 3(ld)^{k}lur, \\ 4(dr)^{k}dlu, \\ 4(rd)^{k}rul \end{split} \] & \textbf{$n \geq 5$ odd:} \[ \begin{split} 1(ru)^{k}ld, \\ 1(ur)^{k}dl, \\ 2(lu)^{k}rd, \\ 2(ul)^{k}dr, \\ 3(ld)^{k}ru, \\ 3(dl)^{k}ur, \\ 4(rd)^{k}lu, \\ 4(dr)^{k}ul \end{split} \] \\ \cline{2-6} & \textbf{Type 2:} & \begin{tikzpicture}[scale=0.25] \node[circle, draw, fill=none, inner sep=0pt, minimum width=\plotptradius] (0) at (5,4) {}; \node[permpt] (1) at (6,6) {}; \node[permpt] (2) at (3,5) {}; \draw[thin] (2) -- ++ (3.5,0); \node[permpt] (3) at (4,2) {}; \draw[thin] (3) -- ++ (0,3.5); \node[permpt] (4) at (1,3) {}; \draw[thin] (4) -- ++ (3.5,0); \node[permpt] (5) at (2,1) {}; \draw[thin] (5) -- ++ (0,2.5); \begin{scope}[shift={(-4,-2.25)}] \node[circle,fill,inner sep=0.5pt] (6) at (3.5,2.5) {}; \node[circle,fill,inner sep=0.5pt] (7) at (4,3) {}; \node[circle,fill,inner sep=0.5pt] (8) at (4.5,3.5) {}; \node[circle,fill,inner sep=0.5pt] (8) at (5,4) {}; \end{scope} \node[empty] (-1) at (5,8) {}; \draw[thick] (-2,4) -- ++ (9,0); \draw[thick] (5,-2) -- ++ (0,9); \node[] (4) at (3,-3.5) {$1l(dl)^{k}$ \text{or} $1l(dl)^{k}d$}; \draw[dashed] (-1,-0.25) rectangle (5.5,5.5); \end{tikzpicture} & 8 & \textbf{$n \geq 4$ even:} \[ \begin{split} 1l(dl)^{k}, \\ 1d(ld)^{k}, \\ 2r(dr)^{k}, \\ 2d(rd)^{k}, \\ 3r(ur)^{k}, \\ 3u(ru)^{k}, \\ 4l(ul)^{k}, \\ 4u(lu)^{k} \end{split} \] & \textbf{$n \geq 5$ odd:} \[ \begin{split} 1(ld)^{k}, \\ 1(dl)^{k}, \\ 2(rd)^{k}, \\ 2(dr)^{k}, \\ 3(ru)^{k}, \\ 3(ur)^{k}, \\ 4(lu)^{k}, \\ 4(ul)^{k} \\ \end{split} \] \\ \hline \end{tabular} \newpage { \centering \begin{tabular}{@{}|>{\centering}m{1.5em}|>{\centering}m{5.5em}|>{\centering\arraybackslash}m{12em}|>{\centering}m{4em}|>{\centering\arraybackslash}m{17.5em}|} \hline \multicolumn{5}{|c|}{}\\[0.4pt] \multicolumn{5}{|c|}{\textbf{List of collisions of pin factors:}}\\[6pt] \hline \multicolumn{2}{|c|}{\textbf{Length}}&\textbf{Representative}&\textbf{Total number}&\textbf{Full List}\\ \hline \multicolumn{2}{|c|}{$2$} & \begin{tikzpicture}[scale=0.2] \begin{scope}[shift={(10,0)}] \node[circle, draw, fill=none, inner sep=0pt, minimum width=\plotptradius] (0) at (0,0) {}; \node[permpt] (1) at (1,2) {}; \node[permpt] (2) at (2,1) {}; \draw[thin] (2) -- ++ (-1.5,0); \node[empty] (-1) at (0,4) {}; \draw[thick] (-3,0) -- ++ (6,0); \draw[thick] (0,-3) -- ++ (0,6); \node[] (4) at (0,-5) {$1r$}; \end{scope} \node[circle, draw, fill=none, inner sep=0pt, minimum width=\plotptradius] (0) at (0,0) {}; \node[permpt] (1) at (2,1) {}; \node[permpt] (2) at (1,2) {}; \draw[thin] (2) -- ++ (0,-1.5); \draw[thick] (-3,0) -- ++ (6,0); \draw[thick] (0,-3) -- ++ (0,6); \node[] (3) at (5,0) {=}; \node[] (4) at (0,-5) {$1u$}; \end{tikzpicture} & 4 \ \text{pairs} & \[ \{1u, 1r\}, \{2l, 2u\}, \{3d, 3l\}, \{4r, 4d\} \] \\ \hline \multicolumn{2}{|c|}{$3$} & \begin{tikzpicture}[scale=0.2] \node[circle, draw, fill=none, inner sep=0pt, minimum width=\plotptradius] (0) at (1,0) {}; \node[permpt] (1) at (3,1) {}; \node[permpt] (2) at (2,3) {}; \draw[thin] (2) -- ++ (0,-2.5); \node[permpt] (3) at (0,2) {}; \draw[thin] (3) -- ++ (2.5,0); \draw[thick] (-2,0) -- ++ (6,0); \draw[thick] (1,-3) -- ++ (0,7); \node[] (3) at (6,0) {=}; \node[] (4) at (1,-4) {$1ul$}; \begin{scope}[shift={(10,0)}] \node[circle, draw, fill=none, inner sep=0pt, minimum width=\plotptradius] (0) at (1,0) {}; \node[permpt] (1) at (0,2) {}; \node[permpt] (2) at (3,1) {}; \draw[thin] (2) -- ++ (-3.5,0); \node[permpt] (3) at (2,3) {}; \draw[thin] (3) -- ++ (0,-2.5); \node[empty] (-1) at (1,5) {}; \draw[thick] (-2,0) -- ++ (6,0); \draw[thick] (1,-3) -- ++ (0,7); \node[] (4) at (1,-4) {$2ru$}; \end{scope} \end{tikzpicture} & 8 \ \text{pairs} & \small \[ \begin{split} \{1ul, 2ru\}, \{1rd, 4ur\}, \{2ur, 1lu\}, \{2ld, 3ul\},\\ \{3lu, 2dl\}, \{3dr, 4ld\}, \{4ru, 1dr\}, \{4dl, 3rd\} \end{split} \] \\ \hline \multicolumn{2}{|c|}{$4$} & \begin{tikzpicture}[scale=0.2] \node[circle, draw, fill=none, inner sep=0pt, minimum width=\plotptradius] (0) at (3,3) {}; \node[permpt] (1) at (4,5) {}; \draw[thin] (1) -- ++ (0,-3.5); \node[permpt] (2) at (1,4) {}; \draw[thin] (2) -- ++ (3.5,0); \node[permpt] (3) at (2,1) {}; \draw[thin] (3) -- ++ (0,3.5); \node[permpt] (4) at (5,2) {}; \draw[thin] (4) -- ++ (-3.5,0); \node[empty] (4) at (3,7) {}; \draw[thick] (0,3) -- ++ (6,0); \draw[thick] (3,0) -- ++ (0,6); \node[] (4) at (3,-1) {$1ldr = 2dru = 3rul = 4uld$}; \end{tikzpicture} & 2 \ \text{quad.s} & \[ \begin{split} \{1ldr, 2dru, 3rul, 4uld\}, \\ \{1dlu, 2rdl, 3urd, 4lur\} \end{split} \] \\ \hline \multirow{2}{*}{$5$} & \textbf{Pathological:} & \begin{tikzpicture}[scale=0.2] \node[circle, draw, fill=none, inner sep=0pt, minimum width=\plotptradius] (0) at (4,3) {}; \node[permpt] (1) at (6,4) {}; \node[permpt] (2) at (5,6) {}; \draw[thin] (2) -- ++ (0,-2.5); \node[permpt] (3) at (2,5) {}; \draw[thin] (3) -- ++ (3.5,0); \node[permpt] (4) at (3,1) {}; \draw[thin] (4) -- ++ (0,4.5); \node[permpt] (5) at (1,2) {}; \draw[thin] (5) -- ++ (2.5,0); \node[empty] (-1) at (4,8) {}; \draw[thick] (0,3) -- ++ (8,0); \draw[thick] (4,0) -- ++ (0,7); \node[] (3) at (10,3) {=}; \node[] (4) at (4,-1) {$1uldl$}; \begin{scope}[shift={(12,0)}] \node[circle, draw, fill=none, inner sep=0pt, minimum width=\plotptradius] (0) at (4,3) {}; \node[permpt] (1) at (6,4) {}; \draw[thin] (1) -- ++ (-4.5,0); \node[permpt] (2) at (5,6) {}; \draw[thin] (2) -- ++ (0,-2.5); \node[permpt] (3) at (2,5) {}; \draw[thin] (3) -- ++ (0,-3.5); \node[permpt] (4) at (3,1) {}; \node[permpt] (5) at (1,2) {}; \draw[thin] (5) -- ++ (2.5,0); \draw[thick] (0,3) -- ++ (8,0); \draw[thick] (4,0) -- ++ (0,7); \node[] (4) at (4,-1) {$3luru$}; \end{scope} \end{tikzpicture} & 4 \ \text{pairs} & \[ \begin{split} \{1uldl, 3luru\}, \{1rdld, 3drur\}, \\ \{2urdr, 4rulu\}, \{2ldrd, 4dlul\} \end{split} \] \\ \cline{2-5} & \textbf{Regular:} & \begin{tikzpicture}[scale=0.2] \node[circle, draw, fill=none, inner sep=0pt, minimum width=\plotptradius] (0) at (5,3) {}; \node[permpt] (1) at (6,5) {}; \node[permpt] (2) at (3,4) {}; \draw[thin] (2) -- ++ (3.5,0); \node[permpt] (3) at (4,1) {}; \draw[thin] (3) -- ++ (0,3.5); \node[permpt] (4) at (1,2) {}; \draw[thin] (4) -- ++ (3.5,0); \node[permpt] (5) at (2,6) {}; \draw[thin] (5) -- ++ (0,-4.5); \draw[thick] (0,3) -- ++ (8,0); \draw[thick] (5,0) -- ++ (0,7); \node[] (3) at (10,3) {=}; \node[] (4) at (5,-1) {$1ldlu$}; \begin{scope}[shift={(12,0)}] \node[circle, draw, fill=none, inner sep=0pt, minimum width=\plotptradius] (0) at (5,3) {}; \node[permpt] (1) at (6,5) {}; \draw[thin] (1) -- ++ (-4.5,0); \node[permpt] (2) at (3,4) {}; \node[permpt] (3) at (4,1) {}; \draw[thin] (3) -- ++ (0,3.5); \node[permpt] (4) at (1,2) {}; \draw[thin] (4) -- ++ (3.5,0); \node[permpt] (5) at (2,6) {}; \draw[thin] (5) -- ++ (0,-4.5); \draw[thick] (0,3) -- ++ (8,0); \draw[thick] (5,0) -- ++ (0,7); \node[] (4) at (5,-1) {$2dlur$}; \end{scope} \end{tikzpicture} & 8 \ \text{pairs} & \small \[ \begin{split} \{1ldlu, 2dlur\}, \{1dldr, 4ldru\}, \\ \{2rdru, 1drul\}, \{2drdl, 3rdlu\}, \\ \{3urul, 2ruld\}, \{3rurd, 4urdl\}, \\ \{4luld, 3uldr\}, \{4ulur, 1lurd\} \end{split} \] \\ \hline \multicolumn{2}{|c|}{$n \geq 6$ \ \textbf{even:}} & \begin{tikzpicture}[scale=0.2] \node[circle, draw, fill=none, inner sep=0pt, minimum width=\plotptradius] (0) at (8,8) {}; \node[permpt] (1) at (9,10) {}; \node[permpt] (2) at (6,9) {}; \draw[thin] (2) -- ++ (3.5,0); \node[permpt] (3) at (7,6) {}; \draw[thin] (3) -- ++ (0,3.5); \node[permpt] (4) at (4,7) {}; \draw[thin] (4) -- ++ (3.5,0); \node[permpt] (5) at (5,5) {}; \draw[thin] (5) -- ++ (0,2.5); \node[permpt] (6) at (1,3) {}; \draw[thin] (6) -- ++ (1.5,0); \node[permpt] (7) at (2,1) {}; \draw[thin] (7) -- ++ (0,2.5); \node[permpt] (8) at (11,2) {}; \draw[thin] (8) -- ++ (-9.5,0); \node[permpt] (9) at (10,11) {}; \draw[thin,dashed] (9) -- ++ (0,-9.5); \node[empty] (-1) at (8,13) {}; \node[circle,fill,inner sep=0.5pt] (10) at (4.5,5.5) {}; \node[circle,fill,inner sep=0.5pt] (11) at (4,5) {}; \node[circle,fill,inner sep=0.5pt] (12) at (3.5,4.5) {}; \node[circle,fill,inner sep=0.5pt] (13) at (3,4) {}; \node[circle,fill,inner sep=0.5pt] (14) at (2.5,3.5) {}; \draw[thick] (8,0) -- ++ (0,12); \draw[thick] (0,8) -- ++ (12,0); \draw[dotted] (9.5,10.5) rectangle (10.5,11.5); \node[] (15) at (6.5,-1) {$1(ld)^{k}r=2(dl)^{k}dru$}; \end{tikzpicture} & 8 \ \text{pairs} & \small \[ \begin{split} \{1(ld)^{k}r, 2(dl)^{k}dru\}, \{1(dl)^{k}u, 4(ld)^{k}lur\}, \\ \{2(dr)^{k}u, 3(rd)^{k}rul\}, \{2(rd)^{k}l, 1(dr)^{k}dlu\}, \\ \{3(ru)^{k}l, 4(ur)^{k}uld\}, \{3(ur)^{k}d, 2(ru)^{k}rdl\}, \\ \{4(ul)^{k}d, 1(lu)^{k}ldr\}, \{4(lu)^{k}r, 3(ul)^{k}urd\} \end{split} \] \\ \hline \multicolumn{2}{|c|}{$n \geq 7$ \ \textbf{odd:}} & \begin{tikzpicture}[scale=0.2] \node[circle, draw, fill=none, inner sep=0pt, minimum width=\plotptradius] (0) at (8,6) {}; \node[permpt] (1) at (9,8) {}; \node[permpt] (2) at (6,7) {}; \draw[thin] (2) -- ++ (3.5,0); \node[permpt] (3) at (7,4) {}; \draw[thin] (3) -- ++ (0,3.5); \node[permpt] (4) at (5,5) {}; \draw[thin] (4) -- ++ (2.5,0); \node[permpt] (5) at (3,1) {}; \draw[thin] (5) -- ++ (0,1.5); \node[permpt] (6) at (1,2) {}; \draw[thin] (6) -- ++ (2.5,0); \node[permpt] (7) at (2,10) {}; \draw[thin] (7) -- ++ (0,-8.5); \node[permpt] (8) at (10,9) {}; \draw[thin,dashed] (8) -- ++ (-8.5,0); \node[empty] (-1) at (8,12) {}; \node[circle,fill,inner sep=0.5pt] (8) at (5.5,4.5) {}; \node[circle,fill,inner sep=0.5pt] (9) at (5,4) {}; \node[circle,fill,inner sep=0.5pt] (10) at (4.5,3.5) {}; \node[circle,fill,inner sep=0.5pt] (11) at (4,3) {}; \node[circle,fill,inner sep=0.5pt] (12) at (3.5,2.5) {}; \draw[thick] (8,0) -- ++ (0,11); \draw[thick] (0,6) -- ++ (11,0); \draw[dotted] (9.5,8.5) rectangle (10.5,9.5); \node[] (13) at (6,-1) {$1(ld)^{k}lu=2(dl)^{k}ur$}; \end{tikzpicture} & 8 \ \text{pairs} & \small \[ \begin{split} \{1(ld)^{k}lu, 2(dl)^{k}ur\}, \{1(dl)^{k}dr, 4(ld)^{k}ru\}, \\ \{2(dr)^{k}dl, 3(rd)^{k}lu\}, \{2(rd)^{k}ru, 1(dr)^{k}ul\}, \\ \{3(ru)^{k}rd, 4(ur)^{k}dl\}, \{3(ur)^{k}ul, 2(ru)^{k}ld\}, \\ \{4(ul)^{k}ur, 1(lu)^{k}rd\}, \{4(lu)^{k}ld, 3(ul)^{k}dr\} \end{split} \] \\ \hline \end{tabular}\par } Note: in the previous table we write $(f)^{k}$ to denote the factor $f$ repeating $k$ times for some $k \geq 0$. \begin{obs} We emphasise some noteworthy features of these lists: \begin{enumerate} \item There is no overlap between the two lists: no pin word which generates a $\boxplus$-decomposable is also involved in a collision. \item All collisions are pairs except for the two quadruples at length $4$. \item There are two distinct families of collisions at length $5$: the \textbf{regular} length $5$ collisions, which generalise to a family of collisions for all $n \geq 6$; and the \textbf{pathological} length $5$ collisions, which are a family unique to length $5$. \item All even collisions of length $n \geq 4$ require all four quadrants, whereas the odd collisions of length $n \geq 5$ require only three. \end{enumerate} \end{obs} \subsection{Enumeration of Recurrent Pin Classes} We now have a general strategy for obtaining the generating function of the $\boxplus$-closure of a pin class: \begin{procedure}[$\boxplus\left(\mathcal{C}^{\circ}_{w}\right)$ Enumeration Procedure]\label{recproc} Suppose that $w$ is a pin word. We can find the generating function of the centred pin class $\boxplus\left(\mathcal{C}^{\circ}_{w}\right)$ by the following procedure: \begin{enumerate} \item Find the generating function $h(z)$ of the pin factors of $w$; \item Enumerate the pin factors of $w$ that generate $\boxplus$-decomposable pin permutations, as well as colliding sets of pin factors of $w$, by comparison with the lists in Theorem \ref{classification}. Subtract these from $h(z)$ to obtain $g(z)$, the generating function of $\boxplus$-indecomposables in $\mathcal{C}^{\circ}_{w}$; \item Amend for commutativity: find the generating function $g_{i}(z)$ of the one-quadrant $\boxplus$-indecomposables of $\mathcal{C}^{\circ}_{w}$ in the $i$th quadrant, for each $i \in \{1,2,3,4\}$. (In other words: enumerate the one-quadrant oscillations.) Set: \[ G(z) = g(z) - g_{1}(z)g_{3}(z) - g_{2}(z)g_{4}(z), \] this is the amended $G$-sequence of $\mathcal{C}^{\circ}_{w}$; \item Then the generating function of $\boxplus\left(\mathcal{C}^{\circ}_{w}\right)$ is given by \[ f(z) = \frac{1}{1 - G(z)} \] and the growth rate of $\boxplus\left(\mathcal{C}^{\circ}_{w}\right)$ is the reciprocal of the radius of convergence of $f(z)$. \end{enumerate} Of course, if $w$ is recurrent then $\mathcal{C}^{\circ}_{w}$ is $\boxplus$-closed and so $f(z)$ is in fact the generating function of $\mathcal{C}^{\circ}_{w}$. \end{procedure} We illustrate this procedure with a sequence of examples, beginning with the class of increasing oscillations $\mathcal{O}^{\circ}$, already defined in Fig. \ref{fig:oscclass}: \begin{example}[The Increasing Oscillations] The growth rate of $\mathcal{O}^{\circ}$ is $\kappa \approx 2.20557$, defined to be the reciprocal of the smallest positive real root of \[ 1 - 2z - z^3 = 0 \] \end{example} \begin{proof} The defining pin sequence of $\mathcal{O}^{\circ}$ is $w=1(ru)^{*}$, which has two pin factors of every length $\geq 2$ (corresponding to the two distinct starting positions within the period) and one pin factor (namely $1$ itself) of length $1$, so \[ h(z) = z + 2z^2 + 2z^3 + 2z^4 + 2z^5 \dots \] By comparison with the lists in Theorem \ref{classification}, there are no $\boxplus$-decomposables amongst the pin factors of $w$ and precisely one colliding pair, namely $\{1u,1r\}$, at length $2$. We thus subtract $1$ at length $2$ to obtain the generating function of the $\boxplus$-indecomposables in $\mathcal{O}^{\circ}$: \[ \begin{split} g(z) & = z + z^2 + 2z^3 + 2z^4 + 2z^5 + \dots \\ & = \frac{z+z^3}{1-z} \end{split} \] Noting that $g_{2}(z) = g_{3}(z) = g_{4}(z) = 0$ we see that $G(z)=g(z)$ and the generating function of $\mathcal{O}^{\circ}$ is \[ \begin{split} f(z) & = \frac{1}{1 - g(z)} \\ & = \frac{1-z}{1 - 2z - z^3} \end{split} \] and we may now deduce the growth rate by the Exponential Growth Theorem \ref{EGT}. \end{proof} We shall see later that $\kappa$ is in fact the smallest possible growth rate of a pin class. For now we give three more classes as examples: \begin{figure}[h] \begin{center} \begin{tikzpicture}[scale=0.35] \draw[very thick] (9,2) -- ++ (0,16); \draw[very thick] (0,2) -- ++ (17,0); \node[circle, draw, fill=none, inner sep=0pt, minimum width=\plotptradius] (0) at (9,2) {}; \node[permpt] (3) at (7,3) {}; \node[permpt] (4) at (8,6) {}; \draw[thin] (4) -- ++ (0,-3.5); \node[permpt] (5) at (13,5) {}; \draw[thin] (5) -- ++ (-5.5,0); \node[permpt] (6) at (12,8) {}; \draw[thin] (6) -- ++ (0,-3.5); \node[permpt] (7) at (5,7) {}; \draw[thin] (7) -- ++ (7.5,0); \node[permpt] (8) at (6,10) {}; \draw[thin] (8) -- ++ (0,-3.5); \node[permpt] (9) at (15,9) {}; \draw[thin] (9) -- ++ (-9.5,0); \node[permpt] (10) at (14,12) {}; \draw[thin] (10) -- ++ (0,-3.5); \node[permpt] (11) at (3,11) {}; \draw[thin] (11) -- ++ (11.5,0); \node[permpt] (12) at (4,14) {}; \draw[thin] (12) -- ++ (0,-3.5); \node[permpt] (13) at (17,13) {}; \draw[thin] (13) -- ++ (-13.5,0); \node[permpt] (14) at (16,16) {}; \draw[thin] (14) -- ++ (0,-3.5); \node[permpt] (15) at (1,15) {}; \draw[thin] (15) -- ++ (15.5,0); \node[permpt] (16) at (2,17) {}; \draw[thin] (16) -- ++ (0,-2.5); \end{tikzpicture} \end{center} \caption{The pin class $\mathcal{V}^{\circ}$, defined by the pin sequence $w = 2(urul)^{*}$} \label{fig:2pinclassV} \end{figure} \begin{example}[Pin Class $\mathcal{V}$] The pin class $\mathcal{V}$ is the underlying (uncentred) permutation class of the centred class $\mathcal{V}^{\circ} = \mathcal{C}^{\circ}_{w}$, where $w = 2(urul)^{*}$ - see Fig. \ref{fig:2pinclassV}. Writing this pin sequence out with letters subscripted by the corresponding quadrant number \[ w = 2u_{2}r_{1}u_{1}l_{2}u_{2}r_{1}u_{1}l_{2}u_{2}r_{1}u_{1}l_{2}u_{2}r_{1}u_{1}l_{2} \dots \] we note immediately that $w$ has $2$ pin factors of length $1$ and $4$ of every greater length. By comparison with our list of $\boxplus$-decomposables and collisions, we note that we have to remove $2$ from this count for lengths $2$ and $3$ (due to the $\boxplus$-decomposables $1l$ and $2r$ at length $2$ and the collisions $\{1ul,2ru\}$ and $\{2ru,1lu\}$ at length $3$) to obtain the generating function of the $\boxplus$-indecomposables of $\mathcal{V}^{\circ}$: \[ \begin{split} g(z) & = 2z + 2z^2 + 2z^3 + 4z^4 + 4z^5 + 4z^6 + 4z^7 + \dots \\ & = \frac{2z + 2z^4}{1-z} \end{split} \] As we are in the upper half-plane, $g_{3}(z)=g_{4}(z)=0$ so the amended $G$-sequence is just $g(z)$ itself, and the generating function of $\mathcal{V}^{\circ}$ is given by \[ \begin{split} f(z) & = \frac{1}{1 - g(z)} \\ & = \frac{1-z}{1 - 3z - 2z^4} \end{split} \] We deduce that $gr(\mathcal{V})$, which we shall call $\nu$, is the reciprocal of the smallest real root of \[ 1 - 3z - 2z^4 = 0; \] explicitly: \[ \nu = gr(\mathcal{V}) = gr(\mathcal{V}^{\circ}) \approx 3.06918 \] \end{example} Next, a class in three quadrants which will force us to confront commutativity: \begin{example}[The class $\mathcal{Y}$] Let $w$ be the pin sequence $1(uldlur)^{*}$. We refer to $\mathcal{C}_{w}$, the associated pin class, as $\mathcal{Y}$ - see Fig. (\ref{fig:Y}). Then the growth rate of $\mathcal{Y}$ is $\gamma$, the reciprocal of the smallest positive real root of \begin{equation*} 1 - 4z + 2z^2 + z^3 - z^4 - 2z^5 - 3z^6 = 0. \end{equation*} Explicitly, $\gamma \approx 3.36637$. \end{example} \begin{figure}[h] \begin{center} \begin{tikzpicture}[scale=0.35] \node[circle, draw, fill=none, inner sep=0pt, minimum width=\plotptradius] (0) at (9,5) {}; \node[permpt] (1) at (11,6) {}; \node[permpt] (2) at (10,8) {}; \draw[thin] (2) -- ++ (0,-2.5); \node[permpt] (3) at (7,7) {}; \draw[thin] (3) -- ++ (3.5,0); \node[permpt] (4) at (8,3) {}; \draw[thin] (4) -- ++ (0,4.5); \node[permpt] (5) at (5,4) {}; \draw[thin] (5) -- ++ (3.5,0); \node[permpt] (6) at (6,10) {}; \draw[thin] (6) -- ++ (0,-6.5); \node[permpt] (7) at (13,9) {}; \draw[thin] (7) -- ++ (-7.5,0); \node[permpt] (8) at (12,12) {}; \draw[thin] (8) -- ++ (0,-3.5); \node[permpt] (9) at (3,11) {}; \draw[thin] (9) -- ++ (9.5,0); \node[permpt] (10) at (4,1) {}; \draw[thin] (10) -- ++ (0,10.5); \node[permpt] (11) at (1,2) {}; \draw[thin] (11) -- ++ (3.5,0); \node[permpt] (12) at (2,13) {}; \draw[thin] (12) -- ++ (0,-11.5); \draw[thick] (9,0) -- ++ (0,14); \draw[thick] (0,5) -- ++ (14,0); \end{tikzpicture} \end{center} \caption{The pin class $\mathcal{Y}^{\circ}$, generated by the pin sequence $w = 1(uldlur)^{*}$.} \label{fig:Y} \end{figure} \begin{proof} We shall find the generating function of the corresponding centred class $\mathcal{Y}^{\circ} = \mathcal{C}^{\circ}_{w}$. We note that $w$, being periodic, is recurrent and we can thus use Procedure \ref{recproc}. First, we enumerate the pin factors of $w=1(uldlur)^{*}$; this will be easier if we first write out $w$ with letters subscripted by the corresponding quadrant number: \[ w = 1u_{1}l_{2}d_{3}l_{3}u_{2}r_{1}u_{1}l_{2}d_{3}l_{3}u_{2}r_{1}u_{1}l_{2}d_{3}l_{3}u_{2}r_{1}u_{1}l_{2}d_{3}l_{3}u_{2}r_{1}\dots \] Clearly there are three pin factors of length $1$, and for $n \geq 2$ the six distinct starting points in the period give six pin factors of length $n$. Hence the generating function of the pin factors of $w$ is \[ h(z) = 3z + 6z^2 + 6z^3 + 6z^4 + 6z^5 + 6z^6 + 6z^7 + \dots \] We now list the collisions and $\boxplus$-decomposables found amongst the pin factors of $w$ by comparison with the lists given in Theorem \ref{classification}: \[ \begin{split} \textbf{Collisions:} \ & \{1ul, 2ru\}, \{2dl, 3lu\} \ (\text{length} \ 3)\\ & \{1uldl, 3luru\}, \{1ldlu, 2dlur\}, \{3urul, 2ruld\} \ (\text{length} \ 5) \\ \textbf{$\boxplus$-dec.s:} \ & 1l, 2d, 3u, 2r \ (\text{length} \ 2) \\ & 1ld, 3ur \ (\text{length} \ 3) \\ & 1uld, 1ldl, 3lur, 3uru \ (\text{length} \ 4) \end{split} \] Hence we subtract these from the generating function of pin factors to obtain the generating function of $\boxplus$-indecomposables of $\mathcal{Y}^{\circ}$: \[ \begin{split} g(z) & = h(z) - (2z^3 + 3z^5) - (4z^2 + 2z^3 + 4z^4) \\ & = 3z + 2z^2 + 2z^3 + 2z^4 + 3z^5 + 6z^6 + 6z^7 + 6z^8 + 6z^9 + \dots \\ \end{split} \] Next we must enumerate the one-quadrant $\boxplus$-indecomposables in each quadrant; this is easily-done by looking at the diagram: $g_{1}(z) = g_{3}(z) = z + z^2$, $g_{2}(z)=z$ and $g_{4}(z)=0$. Hence we can calculate the amended $G$-sequence: \[ \begin{split} G(z) & = g(z) - g_{1}(z)g_{3}(z) - g_{2}(z)g_{4}(z) \\ & = 3z + z^2 + z^4 + 3z^5 + 6z^6 + 6z^7 + 6z^8 + 6z^9 + \dots \\ & = \frac{3z - 2z^2 - z^3 + z^4 + 2z^5 + 3z^6}{1-z} \end{split} \] FInally, by the Generating Function Specification \ref{genfuncspec}, the generating function of $\mathcal{Y}^{\circ}$ is given by: \[ \begin{split} f(z) & = \frac{1}{1 - G(z)} \\ & = \frac{1-z}{1-4z+2z^{2}+z^{3}-z^{4}-2z^{5}-3z^{6}} \end{split} \] This is a rational function, whose singularities are precisely the roots of \[ 1-4z+2z^{2}+z^{3}-z^{4}-2z^{5}-3z^{6} = 0 \] By computation, the smallest root of this equation is positive and real and has reciprocal $\gamma \approx 3.36637$, which by the Exponential Growth Theorem is therefore the growth rate of $\mathcal{Y}$.\end{proof} And finally, a four-quadrant pin class that has in fact been studied before in the literature: the Widdershins Spiral, $\mathcal{W}^{\circ}$, generated by the pin sequence $w = 1(ldru)^{*}$ (see Fig. \ref{fig:widdershins}). This class was first introduced by Murphy~\cite{murphy:restricted-perm:}, who gave the generating function of the uncentred class by an explicit enumeration. Using our specification we can now give an alternative derivation of its growth rate: \begin{figure}[h] \begin{center} \begin{tikzpicture}[scale=0.35] \node[circle, draw, fill=none, inner sep=0pt, minimum width=\plotptradius] (0) at (7,7) {}; \node[permpt] (1) at (8,9) {}; \node[permpt] (2) at (5,8) {}; \draw[thin] (2) -- ++ (3.5,0); \node[permpt] (3) at (6,5) {}; \draw[thin] (3) -- ++ (0,3.5); \node[permpt] (4) at (10,6) {}; \draw[thin] (4) -- ++ (-4.5,0); \node[permpt] (5) at (9,11) {}; \draw[thin] (5) -- ++ (0,-5.5); \node[permpt] (6) at (3,10) {}; \draw[thin] (6) -- ++ (6.5,0); \node[permpt] (7) at (4,3) {}; \draw[thin] (7) -- ++ (0,7.5); \node[permpt] (8) at (12,4) {}; \draw[thin] (8) -- ++ (-8.5,0); \node[permpt] (9) at (11,13) {}; \draw[thin] (9) -- ++ (0,-9.5); \node[permpt] (10) at (1,12) {}; \draw[thin] (10) -- ++ (10.5,0); \node[permpt] (11) at (2,1) {}; \draw[thin] (11) -- ++ (0,11.5); \node[permpt] (12) at (14,2) {}; \draw[thin] (12) -- ++ (-12.5,0); \node[permpt] (13) at (13,14) {}; \draw[thin] (13) -- ++ (0,-12.5); \draw[thick] (0,7) -- ++ (15,0); \draw[thick] (7,0) -- ++ (0,15); \end{tikzpicture} \end{center} \caption{The Widdershins Spiral: this is the smallest pin class in four quadrants.} \label{fig:widdershins} \end{figure} \begin{example}[The Widdershins Spiral] \ \\ Let $\mathcal{W} = \mathcal{C}_{w}$ be the pin class generated by the pin sequence $w = 1(ldru)^{*}$. Then the generating function of the corresponding centred class $\mathcal{W}^{\circ}$ is given by: \begin{equation*} f(z) = \frac{1-z}{1 - 5z + 6z^2 - 2z^3 - z^4 - 3z^5} \end{equation*} Hence $\omega_{0} = gr(\mathcal{W}^{\circ}) \approx 3.48806$. \end{example} \begin{proof} The defining pin sequence $w$ is recurrent so we can enumerate $\mathcal{W}^{\circ}$ using our standard procedure. We first determine the generating function of pin factors of $w$: as $w$ has period $4$ and all four pin factors are already distinct at length $1$ we have: \[ h(z) = 4z + 4z^2 + 4z^3 + 4z^4 + 4z^5 + 4z^6 + \dots \] We must amend this for collisions and $\boxplus$-decomposables to obtain the generating function $g(z)$ of $\boxplus$-indecomposables in $\mathcal{W}^{\circ}$. By comparision with the lists in Theorem \ref{classification} we see that the only collision amongst the pin factors of $w$ is the colliding quadruple $\{1ldr, 2dru, 3rul, 4uld\}$ and that there are precisely four $\boxplus$-decomposables of length $2$ ($1l$, $2d$, $3r$ and $4u$), four $\boxplus$-decomposables of length $3$ ($1ld$, $2dr$, $3ru$ and $4ul$), and none of length $n \geq 4$. Hence the generating function of $\boxplus$-indecomposables in $\mathcal{W}^{\circ}$ is given by: \[ \begin{split} g(z) & = h(z) - 3z^4 - (4z^2 + 4z^3) \\ & = 4z + z^4 + 4z^5 + 4z^6 + 4z^7 + 4z^8 + \dots \\ & = \frac{4z - 4z^2 + z^4 + 3z^5}{1-z} \end{split} \] Next we note that there is precisely $1$ one-quadrant $\boxplus$-indecomposable of length $1$ in each quadrant and none of any greater length: \[ g_{1}(z) = g_{2}(z) = g_{3}(z) = g_{4}(z) = z \] Hence the amended $G$-sequence of $\mathcal{W}^{\circ}$ is given by: \[ \begin{split} G(z) & = g(z) - g_{1}(z)g_{3}(z) - g_{2}(z)g_{4}(z) \\ & = \frac{4z - 4z^2 + z^4 + 3z^5}{1-z} - 2z^2 \\ & = \frac{4z - 6z^2 + 2z^3 + z^4 + 3z^5}{1-z} \end{split} \] And so, by the Generating Function Specification \ref{genfuncspec}, we can derive the generating function of $\mathcal{W}^{\circ}$: \[ \begin{split} f(z) & = \frac{1}{1 - G(z)} \\ & = \frac{1-z}{1 - 5z + 6z^2 - 2z^3 - z^4 - 3z^5} \end{split} \] as required. Finally, we can calculate the growth rate $\omega_{0}$ of $\mathcal{W}^{\circ}$ as the reciprocal of the smallest positive real root of the denominator: $\omega_{0} \approx 3.48806$. \end{proof} \section{Non-recurrent Pin Classes} \label{sec:4} We have now established a general method for determining the growth rate and generating function of a pin class $\mathcal{C}_{w}$ defined by a \emph{recurrent} pin sequence $w$ over the language $\mathcal{L}$. This also allows us to determine the growth rate of an \emph{eventually} recurrent pin class, by the Finite Prefix Lemma \ref{fplem}. This leaves the non-eventually-recurrent case: these classes are not $\boxplus$-closed, so the methods outlined in the previous section will not enable us to determine their generating functions (in fact, the author does not know the generating function of \emph{any} not-eventually-recurrent pin class). Somewhat surprisingly, however, we \emph{can} at least determine the \emph{growth rate} of a general non-recurrent pin class $\mathcal{C}^{\circ}_{w}$. The key idea here is simple: the classes that we know how to enumerate are the $\boxplus$-closed classes, so we enumerate $\mathcal{C}^{\circ}_{w}$ by taking better and better $\boxplus$-closed approximations. The limiting behaviour of these approximations will give us the growth rate, but \emph{not} the generating function, of $\mathcal{C}^{\circ}_{w}$. We begin by noting that we already know how to bound $\mathcal{C}^{\circ}_{w}$ by a $\boxplus$-closed class from above: Procedure \ref{recproc} gives us the generating function of the $\boxplus$-closure of $\mathcal{C}^{\circ}_{w}$; in the non-recurrent case this is not equal to $\mathcal{C}^{\circ}_{w}$ but will function as an upper bound as $\mathcal{C}^{\circ}_{w} \subseteq \boxplus\mathcal{C}^{\circ}_{w}$. We also note that, like pin classes, $\boxplus$-closures of pin classes have proper growth rates: \begin{lemma} Let $w$ be a pin sequence. Then $\boxplus\mathcal{C}^{\circ}_{w}$ has a proper growth rate. \end{lemma} \begin{proof} This is an application of Theorem \ref{grexist}: $\boxplus\mathcal{C}^{\circ}_{w}$ is a $\boxplus$-closed class contained in the complete pin class $\mathcal{P}^{\circ}$ and satisfies that adjacency condition by the same reasoning as $\boxplus\mathcal{C}^{\circ}_{w}$.\end{proof} Hence, for any pin sequence $w$, $gr(\mathcal{C}^{\circ}_{w}) \leq gr(\boxplus\mathcal{C}^{\circ}_{w})$. \subsection{The $\boxplus$-interior of a Pin Class} We now wish to determine a \emph{lower} bound on the growth rate of a pin class, which we again do by comparison with a $\boxplus$-closed class whose generating function we can find. We thus consider the `largest $\boxplus$-closed subclass contained in $\mathcal{C}^{\circ}_{w}$', which we refer to as the $\boxplus$-\emph{interior} of $\mathcal{C}^{\circ}_{w}$. In order to define this rigorously, recall that if $w$ is an infinite pin word over the language $\mathcal{L}$, we use the notation $w_{i,j}$ to denote the pin factor of $w$ taken between the $i$th and $j$th places. Now we can define: \begin{defn}[Recurrent Pin Factors] Let $w$ be a pin sequence and $\widetilde{w}$ a (finite) pin word. We say that $\widetilde{w}$ is a \textbf{recurrent pin factor} of $w$ if for all $n \in \mathbb{N}$ there exist $j \geq i \geq n$ such that $w_{i,j} = \widetilde{w}$. We call a $\boxplus$-indecomposable permutation $\pi^{\circ}_{\tilde{w}}$ generated by a recurrent pin factor $\tilde{w}$ of $w$ a \textbf{recurrent} $\boxplus$-indecomposable in $\mathcal{C}^{\circ}_{w}$. \end{defn} That is, a recurrent pin factor of $w$ is a pin factor that occurs in $w$ infinitely-often. We may now define: \begin{defn}[The $\boxplus$-interior of a pin class] \ \\ Let $w$ be a pin sequence with pin class $\mathcal{C}^{\circ}_{w}$. We define the $\boxplus$-\textbf{interior}, $\mathcal{C}^{\boxplus}_{w}$, of $\mathcal{C}^{\circ}_{w}$ to be the $\boxplus$-closure of the set of pin permutations of the form $\pi^{\circ}_{\tilde{w}}$ where $\tilde{w}$ is a \emph{recurrent} pin factor of $w$. \end{defn} We then have: \begin{obs}[Basic properties of the $\boxplus$-interior] For any pin sequence $w$ over $\mathcal{L}$: \begin{enumerate} \item $\mathcal{C}^{\boxplus}_{w}$ is a (non-empty) $\boxplus$-closed subclass of $\mathcal{C}^{\circ}_{w}$; \item $\mathcal{C}^{\boxplus}_{w}$ is the union of all $\boxplus$-closed subclasses of $\mathcal{C}^{\circ}_{w}$; \item $\mathcal{C}^{\boxplus}_{w} = \mathcal{C}^{\circ}_{w}$ if and only if $\mathcal{C}^{\circ}_{w}$ is $\boxplus$-closed; \item $\sigma^{\circ} \in \mathcal{C}^{\boxplus}_{w}$ if and only if for all $i \in \mathbb{N}$ there exists $j \geq i$ such that $\sigma^{\circ} \leq \pi^{\circ}_{w_{i,j}}$. \item $\mathcal{C}^{\boxplus}_{w}$ is the $\boxplus$-closure of the set of recurrent $\boxplus$-indecomposables in $\mathcal{C}^{\circ}_{w}$. \end{enumerate} \end{obs} Informally, the $\boxplus$-interior of $\mathcal{C}^{\circ}_{w}$ is the set of all centred permutations that can be found in the pin diagram of $w$ in infinitely many (non-overlapping) instances. As the $\boxplus$-interior of a pin class is $\boxplus$-closed, we can determine its generating function in terms of its $G$-sequence, which we can obtain from $w$ as follows: \begin{lemma} Suppose that $g(z)$ is the generating function of the \textbf{recurrent} $\boxplus$-indecomposables in $\mathcal{C}^{\circ}_{w}$ and $g_{1}(z), g_{2}(z), g_{3}(z), g_{4}(z)$ are the generating functions of the \textbf{recurrent} one-quadrant $\boxplus$-indecomposables in quadrants $1,2,3,4$, respectively. Then \[ G(z) = g(z) - g_{1}(z)g_{3}(z) - g_{2}(z)g_{4}(z) \] is the amended $G$-sequence for $\mathcal{C}^{\boxplus}_{w}$, and \[ f(z) = \frac{1}{1 - G(z)} \] is the generating function of $\mathcal{C}^{\boxplus}_{w}$. \end{lemma} \begin{proof} Immediate from the Generating Function Specification on noting that $\mathcal{C}^{\circ}_{w}$ is a $\boxplus$-closed class and the recurrent $\boxplus$-indecomposables of $\mathcal{C}^{\circ}_{w}$ are its $\boxplus$-indecomposables. \end{proof} As with the $\boxplus$-closure we briefly note the following: \begin{lemma} Let $w$ be a pin sequence. Then $\mathcal{C}^{\boxplus}_{w}$ has a proper growth rate. \end{lemma} \begin{proof} If $\mathcal{C}^{\boxplus}_{w}$ contains \nept and \swpt then $w$ contains $1$ and $3$ as recurrent pin factors, which is to say that $w$ visits quadrants $1$ and $3$ infinitely-often. Every time $w$ moves from quadrant $1$ to quadrant $3$ it must pass through either quadrant $2$ or $4$, and so must also visit at least one of these quadrants infinitely-often; hence $\mathcal{C}^{\boxplus}_{w}$ also contains either \nwpt or \sept and so $w$ satisfies the adjacency condition. The same reasoning works if $\mathcal{C}^{\boxplus}_{w}$ contains \nwpt or \sept, and so we conclude that $\mathcal{C}^{\boxplus}_{w}$ satisfies the adjacency condition. As $\mathcal{C}^{\boxplus}_{w}$ is $\boxplus$-closed we can now apply Theorem \ref{grexist} to deduce that $\mathcal{C}^{\boxplus}_{w}$ has a proper growth rate. \end{proof} Recall that $\kappa \approx 2.20557$ is the growth rate of the class $\mathcal{O}^{\circ}$ of increasing oscillations. We shall require the following elementary bound: \begin{lemma} Let $w$ be a pin sequence. Then $gr(\mathcal{C}^{\boxplus}_{w}) \geq \kappa$. \end{lemma} \begin{proof} \begin{itemize} \item If $w$ visits only one quadrant recurrently, which by symmetry we may take to be the first quadrant, then $w=\widetilde{w}(ur)^{*}$, where $\widetilde{w}$ is some finite prefix. Hence $\mathcal{C}^{\boxplus}_{w}$ is just $\mathcal{O}^{\circ}$ with a finite prefix, and so has growth rate $\kappa$. \item Suppose $w$ visits precisely two (necessarily adjacent) quadrants recurrently - by symmetry we may take these to be quadrants $1$ and $2$. Then after some finite prefix, $w$ stays in the upper half-plane, and moves between quadrants $1$ and $2$ infinitely-often. Hence $w$ contains $1l$ and $2r$ as recurrent pin factors, and these must extend to recurrent pin factors $1lu$ and $2ru$ (as $w$ stays in the upper half-plane from this point). Hence $\pi^{\circ}_{1lu}$ and $\pi^{\circ}_{2ru}$ are contained in $\mathcal{C}^{\boxplus}_{w}$ and we note that \nwtwo and \netwo are contained in these two pin permutations, respectively. Hence $\mathcal{C}^{\boxplus}_{w}$ contains the $\boxplus$-closure $\boxplus\{\nwtwo,\netwo\}$, which by calculation has growth rate $\approx 2.73205 > \kappa$. \item Suppose $w$ visits three or four quadrants recurrently - by symmetry assume these include quadrants $1$, $2$ and $3$. Then $\mathcal{C}^{\boxplus}_{w}$ contains $\boxplus\{\nept,\nwpt,\swpt\}$, which by calculation has growth rate $\approx 2.61803 > \kappa$. \end{itemize}\end{proof} We note that this now implies the following as a corollary: \begin{cor} Let $w$ be a pin sequence. Then: \begin{itemize} \item $\overline{gr}(\mathcal{C}_{w}) \geq \kappa$ \item $\overline{gr}(\mathcal{C}_{w}) = \kappa$ if and only if $w = \widetilde{w}(ur)^{*}$ or some symmetry of this form, where $\widetilde{w}$ is a finite prefix. \end{itemize} \end{cor} \subsection{$G$-sequence properties} Let $w$ be a pin sequence, and let $\mathcal{C}^{\circ}$ be either the pin class generated by $w$ or its $\boxplus$-interior. We now know how to obtain the amended $G$-sequence $G(z)$ of $\mathcal{C}^{\circ}$, by enumerating either the pin factors or recurrent pin factors of $w$. Then the generating function of $\boxplus\left(\mathcal{C}^{\circ}\right)$ (note that this is $\mathcal{C}^{\circ}$ itself in the $\boxplus$-interior case) is given by: \[ f(z) = \frac{1}{1 - G(z)} \] We know that the growth rate $\rho$ of $\boxplus\left(\mathcal{C}^{\circ}\right)$ is equal to the reciprocal of the radius of convergence of $f(z)$. We should like to be able to deduce from this that $G(\rho^{-1}) = 1$, as this will give us a strategy for studying growth rates of pin classes in terms of analytic properties of $G(z)$, but this will require a slightly more thorough study of the properties of $G(z)$: \begin{prop}[$G$-sequence of pin classes and $\boxplus$-interiors] \label{Gproperties} Let $w$ be a pin sequence and suppose that $G(z)= \sum_{n=1}^{\infty}a_{n}z^{n}$ ($a_{n} \in \mathbb{Z}$) is the amended $G$-sequence of a class $\mathcal{C}^{\circ}$, which is \textbf{either} the pin class generated by $w$ \textbf{or} its $\boxplus$-interior. Then: \begin{enumerate} \item $a_{1} \in \left\{1, 2, 3, 4\right\}$. \item For all $n \geq 2$: \[ -8n \leq a_{n} < 2^{n+2} \] \item $G(z)$ converges to a smooth function on the interval $\left[0,\frac{1}{2}\right)$, with $G(0) = 0$. \item $G(z) = 1$ has a solution in $\left[0,\frac{1}{\kappa}\right]$. \item Let $\alpha$ be the smallest positive real solution of $G(z)=1$. The growth rate of $\boxplus\mathcal{C}^{\circ}$ is equal to $\alpha^{-1}$. \end{enumerate} \end{prop} \begin{proof} Recall that \begin{equation} \label{amend} G(z) = g(z) - g_{1}(z)g_{3}(z) - g_{2}(z)g_{4}(z) \end{equation} where $g(z)$ counts the $\boxplus$-indecomposables in $\mathcal{C}^{\circ}$ and $g_{i}(z)$ counts the one-quadrant $\boxplus$-indecomposables contained in the $i$th quadrant. \begin{enumerate} \item This is clear from \ref{amend}: the $z$-term of $G(z)$ must agree with the $z$-term of $g(z)$ (as the products $g_{1}(z)g_{3}(z)$ and $g_{2}(z)g_{4}(z)$ have no term of degree lower than $z^2$), and this simply counts the number of quadrants that $\mathcal{C}^{\circ}$ visits (either at all or recurrently), which is clearly either $1$, $2$, $3$ or $4$. \item Let $\preceq$ denote coefficient-wise ordering on formal power series, so $p(z) \preceq q(z)$ means that the $z^n$-coefficient of $p(z)$ is less than or equal to the $z^n$-coefficient of $q(z)$ for all $n \in \mathbb{N}$. Then, from \ref{amend} (and using the fact that the coefficients of $g(z),g_{i}(z)$ are all non-negative): \[ \begin{split} G(z) & = g(z) - g_{1}(z)g_{3}(z) - g_{2}(z)g_{4}(z) \\ & \preceq g(z) \\ & \preceq 4z + \sum_{n=2}^{\infty}2^{n+2}z^n \end{split} \] where the final inequality is simply the number of pin words of length $n$ over $\mathcal{L}$. On the other hand, using the fact that the generating function of \emph{all} one-quadrant $boxplus$-indecomposable pin permutations in the $i$th quadrant is \[ g_{i}(z) = z + z^2 + 2z^3 + 2z^4 + 2z^5 + \dots = \frac{z + z^3}{1-z}, \] we obtain: \[ \begin{split} G(z) & = g(z) - g_{1}(z)g_{3}(z) - g_{2}(z)g_{4}(z) \\ & \succeq - g_{1}(z)g_{3}(z) - g_{2}(z)g_{4}(z) \\ & \succeq -2\frac{(z+z^3)^{2}}{(1-z)^{2}} \\ & = -2z^2 - 4z^3 - 10z^4 - 16z^5 - 24z^6 - 32z^7 - 40z^8 - 48z^9 - \dots \\ & \succeq \sum_{n=1}^{\infty}-8nz^n \end{split} \] Combining these two inequalities gives the desired result. \item This is from the Ratio Test, on noting that the coefficients of $G(z)$ have magnitudes bounded by powers of $2$. \item We know that the all the classes we are dealing with have growth rates greater than or equal to $\kappa$. Hence the generating function of $\boxplus\mathcal{C}^{\circ}$, namely \[ f(z) = \frac{1}{1 - G(z)} , \] has radius of convergence (in $\mathbb{C}$) $R \leq \frac{1}{\kappa} < \frac{1}{2}$. As $f(z)$ has non-negative coefficients (regardless of whether $G(z)$ does), we can apply Pringsheim's Theorem~\cite[Theorem IV.7]{flajolet:analytic-combin:} to deduce that $f(z)$ has a singularity at $z = R$. Singularites of $f(z)$ correspond either to singularities of $G(z)$ or solutions of $G(z)=1$. But $G(z)$ converges on $\left[0,\frac{1}{2}\right)$ which includes $R$, so $G(R)=1$. \item We saw that $R$ solves $G(z)=1$ in the proof of \textit{4.}; there cannot be a smaller solution to this equation as that would give a singularity with magnitude smaller than $R$. Now apply the Exponential Growth Theorem \ref{EGT}. \end{enumerate} \end{proof} We require one more property of $G$-sequences in the $\boxplus$-interior case only: \begin{lemma} \label{G+} Let $G(z)$ be the amended $G$-sequence of the $\boxplus$-interior $\mathcal{C}^{\boxplus}_{w}$ of some pin class $\mathcal{C}^{\circ}_{w}$. Let $\alpha$ be the smallest positive real root of the equation $G(z)=1$, guaranteed to exist by Proposition \ref{Gproperties}. Then $G(z)$ is positive on the interval $(0,\alpha)$. \end{lemma} \begin{proof} We note that this property is obvious if $G(z)$ has non-negative coefficients, which immediately deals with the case in which $w$ visits only two of the quadrants (either recurrently or otherwise). We next suppose that $\mathcal{C}^{\circ}$ has points in precisely three quadrants, which, without loss of generality, we take to be quadrants $1$, $2$ and $3$. Then $\mathcal{C}^{\circ}$ must contain the permutations \nept, \nwpt, \swpt, \netwo and \swtwo (as the defining pin sequence $w$ must turn around in the first and third quadrants infinitely-often), and so: \[ \begin{split} G(z) & = g(z) - g_{1}(z)g_{3}(z) \\ & \succeq (3z + 2z^2 + 2z^3) - \frac{(z+z)^2}{(1-z)^2} \\ & = \frac{3z - 5z^2 + z^3 - 4z^4 + 2z^5 - z^6}{(1-z)^2} \end{split} \] Hence: \[ G(z) = \frac{3z - 5z^2 + z^3 - 4z^4 + 2z^5 - z^6}{(1-z)^2} + F(z) \] where $F(z)$ has non-negative coefficients. By computation the function given here is positive on the desired range, and $F(z)$ is certainly positive here due to non-negativity of its coefficients. Hence $G(z)$ is positive on $(0,\alpha)$. Finally, if $G(z)$ visits all four quadrants then: \[ \begin{split} G(z) & = g(z) - g_{1}(z)g_{3}(z) - g_{2}(z)g_{4}(z)\\ & \succeq 4z - 2\frac{(z+z)^2}{(1-z)^2} \\ & = \frac{4z - 10z^2 + 4z^3 - 4z^4 - 2z^6}{(1-z)^2} \end{split} \] and as in the three quadrant case, this `worst case scenario' function is nevertheless still positive in the desired range. \end{proof} \subsection{$\mathcal{C}^{\circ}_{w}$ has same growth rate as $\mathcal{C}^{\boxplus}_{w}$} The moral of the preceeding sections is that we can sandwich \emph{any} pin class (recurrent or otherwise) between two $\boxplus$-closed (centred) permutation classes that we know how to enumerate. If the pin class happens to be recurrent then these containments are in fact equalities and we have enumerated the pin class. If the pin class is \emph{not} recurrent then we at least have bounds on the growth rate: \begin{equation} \label{conts} gr(\mathcal{C}^{\boxplus}_{w}) \leq gr(\mathcal{C}^{\circ}_{w}) \leq gr(\boxplus\mathcal{C}^{\circ}_{w}) \end{equation} The aim of this section is to prove the remarkable result that the left-hand inequality in (\ref{conts}) is in fact an equality; that is, the growth rate of a pin class is always equal to that of its $\boxplus$-interior. This will enable us to conclude that \emph{any} pin class (recurrent or otherwise) has a proper growth rate, and will enable us to determine this growth rate providing that we can enumerate the recurrent pin factors of the defining pin sequence $w$. The core idea in the proof is relatively simple to understand: if $w$ is a pin sequence we know by the Finite Prefix Theorem that $dfg$ and $cvgbh$ have the same (upper) growth rate and we note that if we take $n$ sufficiently large then the pin factors of $dfg$ are precisely ... \begin{prop} \label{limprop} Let $\mathcal{C}^{\circ}_{w}$ be a pin class generated by a pin sequence $w$. Let $\mathcal{C}^{\boxplus}_{w}$ be the $\boxplus$-interior of $\mathcal{C}^{\circ}_{w}$ and write $w_{\geq n}$ for the left-truncation of $w$ starting in the $n$th position. Then: \[ \lim_{n\rightarrow\infty}gr\left(\boxplus\mathcal{C}^{\circ}_{w_{\geq n}}\right) = gr(\mathcal{C}^{\boxplus}_{w}) \] \end{prop} \begin{proof} Let $g(z)$ be the generating function of the $\boxplus$-indecomposables of $\mathcal{C}^{\boxplus}_{w}$, and $g_{i}(z)$ be the generating function of one-quadrant $\boxplus$-indecomposables of $\mathcal{C}^{\boxplus}_{w}$ in the $i$th quadrant. Then \[ G(z) = g(z) - g_{1}(z)g_{3}(z) - g_{2}(z)g_{4}(z) \] is the amended $G$-sequence of $\mathcal{C}^{\boxplus}_{w}$, and $\alpha = gr(\mathcal{C}^{\boxplus}_{w})^{-1}$ is the smallest positive real root of the equation $G(z) = 1$. Now, let $t \in \mathbb{N}$, and consider the set $\mathcal{W}^{*}_{t}$ of all \textbf{non}-recurrent pin factors of $w$ of length $\leq t$. This is a finite set and each element of it occurs as a pin factor of $w$ only a finite number of times, so there is some $n(t) \in \mathbb{N}$ such that no element of $\mathcal{W}^{*}_{k}$ is contained as a pin factor of $w_{\geq n(t)}$ (for concreteness, we may take $n(t)$ to be the \emph{smallest} positive integer with this property). But of course \emph{all} of the recurrent pin factors of $w$ of length $\leq t$ occur as pin factors of $w_{\geq n(t)}$, so we deduce that the pin factors of $w_{\geq n(t)}$ of length $ \leq t$ are precisely the recurrent pin factors of $w$ of length $\leq t$. Hence if we write $g_{t}(z)$ for the generating function of the $\boxplus$-indecomposables of $\mathcal{C}^{\circ}_{w_{\geq n(t)}}$, and $g_{t, i}(z)$ for generating function of the one-quadrant $\boxplus$-indecomposables of $\mathcal{C}^{\circ}_{w_{\geq n}}$ in the $i$th quadrant, then $g_{t}(z), g_{t,1}(z), g_{t,2}(z), g_{t,3}(z), g_{t,4}(z)$ will agree with $g(z), g_{1}(z), g_{2}(z), g_{3}(z), g_{4}(z)$, respectively, up to and including the $z^{t}$-term. Hence the amended $G$-sequence of $\mathcal{C}^{\circ}_{w_{\geq n(t)}}$, namely \[ G_{t}(z) = g_{t}(z) - g_{t,1}(z)g_{t,3}(z) - g_{t,2}(z)g_{t,4}(z) , \] agrees with $G(z)$ up to and including the $z^{t}$-term. Fix a $t$: we now consider some basic analytic facts about the functions $G(z)$ and $G_{t}(z)$. First, note that by Proposition \ref{Gproperties}, $G(z)$ and $G_{t}(z)$ are smooth functions defined on the interval $\left[0,\frac{1}{\kappa}\right]$. Further, $G(z) = 1$ and $G_{t}(z) = 1$ have solutions in this interval; we call the smallest solution to these equations in this interval $\alpha$ and $\alpha_{t}$, respectively. Then $\mathcal{C}^{\boxplus}_{w}$ and $\boxplus\mathcal{C}^{\circ}_{w_{\geq n(t)}}$ have growth rates $\rho = \alpha^{-1}$ and $\rho_{t} = \alpha^{-1}_{t}$, respectively. Note that we have: \[ \rho = gr(\mathcal{C}^{\boxplus}_{w}) \leq gr(\mathcal{C}^{\circ}_{w}) = gr(\mathcal{C}^{\circ}_{w_{\geq n(t)}}) \leq gr(\boxplus\mathcal{C}^{\circ}_{w_{\geq n(t)}}) = \rho_{t} \] where the two inequalities follow from containment of the corresponding classes, and the middle equality follows from the Finite Prefix Lemma \ref{fplem}. Note that this implies that $\alpha_{t} \leq \alpha$. Now, write $G_{t}(z) = \sum_{n=1}^{\infty}a_{n}z^n$ and $G(z) = \sum_{n=1}^{\infty}b_{n}z^n$: then each $a_{n}, b_{n}$ is an integer (not necessarily positive) with magnitude bounded by $2^{n+2}$ and $a_{n} = b_{n}$ for all $n \leq t$. We combine these facts to deduce a bound on the difference between these two functions: \[ \begin{split} \left|G_{t}(z) - G(z)\right| & = \left|\sum_{n=t+1}^{\infty}(a_{n} - b_{n})z^{n}\right| \\ & \leq \sum_{n=t+1}^{\infty}\left|a_{n} - b_{n}\right|z^n \\ & \leq \sum_{n=t+1}^{\infty}(2^{n+2} + 8n)z^{n} \\ & \leq \sum_{n=t+1}^{\infty}2^{n+3}z^{n} \\ & = \frac{8(2z)^{t+1}}{1-2z} \end{split} \] and so, for $z \in \left[0,\frac{1}{\kappa}\right]$: \[ \begin{split} \left|G_{t}(z) - G(z)\right| & \leq \sup_{z \in \left[0,\frac{1}{\kappa}\right]}\left\{\frac{8(2z)^{t+1}}{1-2z}\right\} \\ & = \frac{8(\frac{2}{\kappa})^{t+1}}{1 - \frac{2}{\kappa}} \\ & = \frac{16}{\kappa - 2} \cdot \left(\frac{2}{\kappa}\right)^{t} \end{split} \] Note, crucially, that (as $\kappa > 2$) this expression approaches $0$ as $t \rightarrow \infty$. We claim that this fact implies that $\alpha_{t} \rightarrow \alpha$: Let $\epsilon > 0$ and consider $G(z)$ on the interval $\left[0, \alpha - \epsilon\right]$. As a continuous function on a closed interval, $G(z)$ achieves a maximum value $M$ on $\left[0, \alpha - \epsilon\right]$. Further, as $G(z)$ is positive on $(0,\alpha]$ (by Lemma \ref{G+}) and $G(z)=1$ does not have a root in $\left[0, \alpha - \epsilon\right]$ (as $\alpha$ is, by definition, the \emph{smallest} positive real root of $G(z)=1$), $M$ must be a positive number smaller than $1$. Now, choose a $K \in \mathbb{N}$ such that: \[ \frac{16}{\kappa - 2} \cdot \left(\frac{2}{\kappa}\right)^{K} < 1 - M \] and take any $t \geq K$. Then, for $z \in \left[0, \alpha - \epsilon\right]$: \[ \begin{split} \left|G_{t}(z)\right| & = \left|(G_{t}(z) - G(z)) + G(z)\right| \\ & \leq \left|G_{t}(z) - G(z)\right| + \left|G(z)\right| \\ & \leq \left|G_{t}(z) - G(z)\right| + G(z) \\ & \leq \frac{16}{\kappa - 2} \cdot \left(\frac{2}{\kappa}\right)^{t} + M \\ & < (1 - M) + M \\ & = 1 \end{split} \] Hence, in particular, $G_{t}(z)=1$ does not have a root in $\left[0, \alpha - \epsilon\right]$. But $G_{t}(z)=1$ certainly does have a root, namely $\alpha_{t}$, which is smaller than $\alpha$. Hence $\alpha_{t} \in \left(\alpha - \epsilon, \alpha\right]$. We have thus proved that for every $\epsilon > 0$ there exists $K \in \mathbb{N}$ such that $\alpha_{t} \in \left(\alpha - \epsilon, \alpha\right]$ for all $t \geq K$; hence $\alpha_{t} \rightarrow \alpha$ as $t \rightarrow \infty$. Hence $gr(\boxplus\mathcal{C}^{\circ}_{\geq n(t)}) \rightarrow gr(\mathcal{C}^{\boxplus}_{w})$ as $t\rightarrow\infty$, as required.\end{proof} We may now finally deduce the main result of this paper: \begin{thm} Let $w$ be a pin sequence. Then the associated pin class $\mathcal{C}^{\circ}_{w}$ (along with its uncentred counterpart $\mathcal{C}_{w}$) has a proper growth rate which is equal to that of its $\boxplus$-interior, $\mathcal{C}^{\boxplus}_{w}$ \end{thm} \begin{proof} Suppose $w$ is a pin sequence. Then $\mathcal{C}^{\boxplus}_{w} \subseteq \mathcal{C}^{\circ}_{w}$ and so \begin{equation}\label{one} gr(\mathcal{C}^{\boxplus}_{w}) = \underline{gr}(\mathcal{C}^{\boxplus}_{w}) \leq \underline{gr}(\mathcal{C}^{\circ}_{w}) \end{equation} where the left-hand equality is due to the fact that $\boxplus$-interiors have (proper) growth rates. Now, letting $n \in \mathbb{N}$ we have \begin{equation}\label{two} \overline{gr}(\mathcal{C}^{\circ}_{w}) = \overline{gr}(\mathcal{C}^{\circ}_{w_{\geq n}}) \end{equation} by the Finite Prefix Lemma \ref{fplem}, and by containment we also have \begin{equation} \label{three} \overline{gr}(\mathcal{C}^{\circ}_{w_{\geq n}}) \leq \overline{gr}(\boxplus\mathcal{C}^{\circ}_{w_{\geq n}}) = gr(\boxplus\mathcal{C}^{\circ}_{w_{\geq n}}) \end{equation} where the right-hand equality follows from existence of proper growth rates of $\boxplus$-closures of pin classes. Combining equations (\ref{one}), (\ref{two}) and (\ref{three}) yields: \[ gr(\mathcal{C}^{\boxplus}_{w}) \leq \underline{gr}(\mathcal{C}^{\circ}_{w}) \leq \overline{gr}(\mathcal{C}^{\circ}_{w}) \leq gr(\boxplus\mathcal{C}^{\circ}_{w_{\geq n}}) \] and taking limits (using Proposition \ref{limprop}) as $n \rightarrow \infty$ forces \[ \underline{gr}(\mathcal{C}^{\circ}_{w}) = \overline{gr}(\mathcal{C}^{\circ}_{w}) \] as required.\end{proof} \subsection{The Complete Class $\mathcal{P}^{\circ}_{c}$} We illustrate the use of this theory by explicitly enumerating the (centred) complete pin class $\mathcal{P}^{\circ}_{c}$. Recall that this is the class of \emph{all} pin permutations. It may not be immediately obvious, but $\mathcal{P}^{\circ}_{c}$ is itself a pin class, and is in fact the \emph{only} pin class that achieves its growth rate: \begin{prop}[Complete Pin Class] \ \begin{enumerate} \item There is a pin sequence $w_{c}$ which contains every finite pin word as a (recurrent) pin factor. \item For any such pin sequence $w_{c}$, $\mathcal{C}^{\circ}_{w} = \mathcal{P}^{\circ}_{c}$. \item The complete pin class has growth rate $\omega_{\infty} \approx 5.24112$, where $\omega_{\infty}$ is defined to be the reciprocal of the smallest positive real root of the equation \[ 1 - 8z + 19z^2 - 26z^3 + 14z^4 - 12z^5 - 8z^6 + 20z^7 - 8z^8 = 0 \] \end{enumerate} \end{prop} \begin{proof} \begin{enumerate} \item Note that $\mathcal{L}^{*}$ is countable, we can thus order all $\mathcal{L}^{*}$-words and concatenate them (after any initial numeral) in this order, possibly placing a letter in between consecutive words to ensure the alignments alternate. This resulting pin sequence $w_{c}$ will contain all $\mathcal{L}^{*}$-words as subword factors, and will hence contain all possible finite pin words as pin factors. \item This is clear by the definition of a pin permutation. \item First, we enumerate the set of pin factors of $w_{c}$, which is to say the set of all finite pin words. Clearly, there are $4$ pin words of length $1$. For length $n \geq 2$, there are $4$ choices for the inital numeral, $4$ choices for the letter in second place, and then $2$ choices for every subsequent letter as the alignments must now alternate. Hence there are $2^{n+2}$ pin words of length $n \geq 2$, and the generating function of the set of all finite pin words is given by: \[ h(z) = 4z + \sum_{n=2}^{\infty}2^{n+2}z^n \] This will be an overcount of the $\boxplus$-indecomposables in $\mathcal{P}^{\circ}$ due to collisions and $\boxplus$-decomposable pin words, but we have already classified all of these in Theorem \ref{classification}. The generating function of the overcount due to collisions is \[ h_{\text{Col}}(z) = 4z^2 + 8z^3 + 6z^4 + 12z^5 + 8z^6 + 8z^7 + 8z^8 + 8z^9 + \dots \] (Note: this is the generating function of the \emph{overcount} due to collisions. At all $n \neq 4$ it is simply equal to the \emph{number} of collisions, as these are all pairs so we need only take away one from the count for each collision. But at $n=4$ there are two colliding \emph{quadruples} so the overcount is $6$.) And the generating function of the $\boxplus$-decomposable pin words is \[ h_{\boxplus\text{-Dec.}}(z) = 8z^2 + 8z^3 + 16z^4 + 16z^5 + 16z^6 + 16z^7 + 16z^8 + 16z^9 + \dots \] Subtracting these from $h(z)$ gives us the generating function of the $\boxplus$-indecomposable pin permutations: \[ \begin{split} g(z) & = h(z) - h_{\text{Col}}(z) - h_{\boxplus\text{-Dec.}}(z) \\ & = 4z + 4z^2 + 16z^3 + 42z^4 + 100z^5 + \sum_{n=6}^{\infty}(2^{n + 2} - 24)z^n \\ & = \frac{4z - 8z^2 + 12z^3 + 2z^4 + 6z^5 + 16z^6 - 8z^7}{(1-z)(1-2z)} \end{split} \] Next, we shall require the generating functions $g_{i}(z)$ ($i \in \{1,2,3,4\}$) of the $\boxplus$-indecomposable pin permutations entirely contained in the $i$th quadrant. Clearly, these are all equal to the generating function of the oscillations; namely: \[ \begin{split} g_{1}(z) = g_{2}(z) = g_{3}(z) = g_{4}(z) & = z + z^2 + 2z^3 + 2z^4 + 2z^5 + 2z^6 + \dots \\ & = \frac{z + z^3}{1-z} \end{split} \] Hence we obtain the amended $G$-sequence for the complete class of pin permutations: \[ \begin{split} G_{\infty}(z) & = g(z) - g_{1}(z)g_{3}(z) - g_{2}(z)g_{4}(z) \\ & = \frac{4z - 14z^2 + 24z^3 - 14z^4 + 12z^5 + 8z^6 - 20z^7 + 8z^8}{(1-z)^{2}(1-2z)} \end{split} \] Thus, we can now apply the Generating Function Specification to obtain the generating function for the class of all (centred) pin permutations: \[ \begin{split} f(z) & = \frac{1}{1 - G_{\infty}(z)} \\ & = \frac{(1-z)^{2}(1-2z)}{1 - 8z + 19z^2 - 26z^3 + 14z^4 - 12z^5 - 8z^6 + 20z^7 - 8z^8} \end{split} \] By the Exponential Growth Rate formula, and the fact that $f(z)$ has positive coefficients, the growth rate of $\mathcal{P}^{\circ}$, and hence also the uncentred class $\mathcal{P}$, is the reciprocal of the smallest positive real root of the denominator, as required. \end{enumerate} \end{proof} \section{Concluding Remarks} \label{sec:5} We have proved that all pin classes have growth rates and established a procedure to determine them. We also have bounds on the possible growth rate of a pin class: for any pin sequence $w$, \[ \kappa \leq gr(\mathcal{C}_{w}) \leq \omega_{\infty} \] where $\kappa \approx 2.20557$ is the reciprocal of the smallest positive real root of \[ 1 - 2z - z^3 = 0 \] and $\omega_{\infty}$ the smallest positive real root of \[ 1 - 8z + 19z^2 - 26z^3 + 14z^4 - 12z^5 - 8z^6 + 20z^7 - 8z^8 = 0 \] A natural further question is what happens within these bounds: what are the possible growth rates of pin classes? We can also ask about bounds on growth rates of pin classes subject to certain characteristics: the number of quadrants visited (recurrently) by a pin class and the length of the longest oscillation contained in $\mathcal{C}^{\circ}_{w}$ are natural characteristics to consider. We can in fact state some answers in the former case already: the pin classes $\mathcal{V}$ and $\mathcal{Y}$ are in fact the smallest pin classes which visit two and three quadrants recurrently, respectively. It is also relatively easy to deduce \emph{upper} bounds on the growth rates of pin classes in two and three quadrants by considering the \textbf{complete pin classes} in these bounded quadrants: for example, the complete class in two quadrants, $\mathcal{V}_{c} = \mathcal{V}_{w_{c}}$ is the pin class generated by a pin sequence $w_{c}$ that contains \emph{all} pin words in quadrants $1$ and $2$ as pin factors. Without too much difficulty (though we omit the proof here) we can calculate the growth rate of $\mathcal{V}_{c}$ to be $\nu_{c} \approx 3.51205$, where $\nu_{c}$ is the reciprocal of the smallest positive real root of \[ 1 - 2z - 4z^2 - 2z^3 - 8z^4 - 4z^5 = 0 \] In the sequel~\cite{brignall-jarvis:pin-classes-ii} we take up the question of what happens within this interval $[\nu,\nu_{c}]$: we move towards a classification of the growth rates of two-quadrant pin classes and observe some interesting structures in this set of growth rates. We show, for example, that there is a point $\nu_{\mathcal{L}} \approx 3.28277$ at which there are uncountably many distinct pin classes and that $\nu_{\mathcal{L}}$ is in fact an accumulation point in the set of pin class growth rates from both above and below. This has potential consequences for the study of well-quasi-ordered permutation classes because $\mathcal{V}$-classes (that is, two-quadrant pin classes) can be used to generate infinite antichains. Potential further directions for study include: \begin{itemize} \item A systematic study of pin class growth rates in three and four quadrants; \item The possibility of conjecturing a classification of `small' antichains (perhaps taking $\nu_{\mathcal{L}}$ as a cut-off) using $\mathcal{V}$-classes; \item The question of whether we can explicitly determine the generating function (not merely the growth rate) or \emph{any} not-eventually-recurrent pin class, such as the Liouville $\mathcal{V}$, introduced in the sequel. \end{itemize} \appendix \section*{Appendix} \renewcommand{\thesubsection}{(\Alph{subsection})} In this appendix we prove Theorem \ref{classification}, that the lists of collisions and $\boxplus$-decomposables given are in fact complete. We begin with collisions: \subsection{Collisions Proof} We can verify that the list of collisions given in the table is complete for lengths $n \geq 5$ by an exhaustive search (which can be done fairly quickly on applying symmetries). We thus aim to prove that the table is complete at lengths $n \geq 6$. We repeat the relevant section of the table for reference: { \centering \begin{tabular}{@{}|>{\centering}m{1.5em}|>{\centering}m{5.5em}|>{\centering\arraybackslash}m{12em}|>{\centering}m{4em}|>{\centering\arraybackslash}m{17.5em}|} \hline \multicolumn{5}{|c|}{}\\[0.4pt] \multicolumn{5}{|c|}{\textbf{List of collisions of pin factors:}}\\[6pt] \hline \multicolumn{2}{|c|}{\textbf{Length}}&\textbf{Representative}&\textbf{Total number}&\textbf{Full List}\\ \hline \multicolumn{2}{|c|}{$n \geq 6$ \ \textbf{even:}} & \begin{tikzpicture}[scale=0.2] \node[circle, draw, fill=none, inner sep=0pt, minimum width=\plotptradius] (0) at (8,8) {}; \node[permpt] (1) at (9,10) {}; \node[permpt] (2) at (6,9) {}; \draw[thin] (2) -- ++ (3.5,0); \node[permpt] (3) at (7,6) {}; \draw[thin] (3) -- ++ (0,3.5); \node[permpt] (4) at (4,7) {}; \draw[thin] (4) -- ++ (3.5,0); \node[permpt] (5) at (5,5) {}; \draw[thin] (5) -- ++ (0,2.5); \node[permpt] (6) at (1,3) {}; \draw[thin] (6) -- ++ (1.5,0); \node[permpt] (7) at (2,1) {}; \draw[thin] (7) -- ++ (0,2.5); \node[permpt] (8) at (11,2) {}; \draw[thin] (8) -- ++ (-9.5,0); \node[permpt] (9) at (10,11) {}; \draw[thin,dashed] (9) -- ++ (0,-9.5); \node[empty] (-1) at (8,13) {}; \node[circle,fill,inner sep=0.5pt] (10) at (4.5,5.5) {}; \node[circle,fill,inner sep=0.5pt] (11) at (4,5) {}; \node[circle,fill,inner sep=0.5pt] (12) at (3.5,4.5) {}; \node[circle,fill,inner sep=0.5pt] (13) at (3,4) {}; \node[circle,fill,inner sep=0.5pt] (14) at (2.5,3.5) {}; \draw[thick] (8,0) -- ++ (0,12); \draw[thick] (0,8) -- ++ (12,0); \draw[dotted] (9.5,10.5) rectangle (10.5,11.5); \node[] (15) at (6.5,-1) {$1(ld)^{k}r=2(dl)^{k}dru$}; \end{tikzpicture} & 8 \ \text{pairs} & \small \[ \begin{split} \{1(ld)^{k}r, 2(dl)^{k}dru\}, \{1(dl)^{k}u, 4(ld)^{k}lur\}, \\ \{2(dr)^{k}u, 3(rd)^{k}rul\}, \{2(rd)^{k}l, 1(dr)^{k}dlu\}, \\ \{3(ru)^{k}l, 4(ur)^{k}uld\}, \{3(ur)^{k}d, 2(ru)^{k}rdl\}, \\ \{4(ul)^{k}d, 1(lu)^{k}ldr\}, \{4(lu)^{k}r, 3(ul)^{k}urd\} \end{split} \] \\ \hline \multicolumn{2}{|c|}{$n \geq 7$ \ \textbf{odd:}} & \begin{tikzpicture}[scale=0.2] \node[circle, draw, fill=none, inner sep=0pt, minimum width=\plotptradius] (0) at (8,6) {}; \node[permpt] (1) at (9,8) {}; \node[permpt] (2) at (6,7) {}; \draw[thin] (2) -- ++ (3.5,0); \node[permpt] (3) at (7,4) {}; \draw[thin] (3) -- ++ (0,3.5); \node[permpt] (4) at (5,5) {}; \draw[thin] (4) -- ++ (2.5,0); \node[permpt] (5) at (3,1) {}; \draw[thin] (5) -- ++ (0,1.5); \node[permpt] (6) at (1,2) {}; \draw[thin] (6) -- ++ (2.5,0); \node[permpt] (7) at (2,10) {}; \draw[thin] (7) -- ++ (0,-8.5); \node[permpt] (8) at (10,9) {}; \draw[thin,dashed] (8) -- ++ (-8.5,0); \node[empty] (-1) at (8,12) {}; \node[circle,fill,inner sep=0.5pt] (8) at (5.5,4.5) {}; \node[circle,fill,inner sep=0.5pt] (9) at (5,4) {}; \node[circle,fill,inner sep=0.5pt] (10) at (4.5,3.5) {}; \node[circle,fill,inner sep=0.5pt] (11) at (4,3) {}; \node[circle,fill,inner sep=0.5pt] (12) at (3.5,2.5) {}; \draw[thick] (8,0) -- ++ (0,11); \draw[thick] (0,6) -- ++ (11,0); \draw[dotted] (9.5,8.5) rectangle (10.5,9.5); \node[] (13) at (6,-1) {$1(ld)^{k}lu=2(dl)^{k}ur$}; \end{tikzpicture} & 8 \ \text{pairs} & \small \[ \begin{split} \{1(ld)^{k}lu, 2(dl)^{k}ur\}, \{1(dl)^{k}dr, 4(ld)^{k}ru\}, \\ \{2(dr)^{k}dl, 3(rd)^{k}lu\}, \{2(rd)^{k}ru, 1(dr)^{k}ul\}, \\ \{3(ru)^{k}rd, 4(ur)^{k}dl\}, \{3(ur)^{k}ul, 2(ru)^{k}ld\}, \\ \{4(ul)^{k}ur, 1(lu)^{k}rd\}, \{4(lu)^{k}ld, 3(ul)^{k}dr\} \end{split} \] \\ \hline \end{tabular}\par } In order to prove this we shall first make the observation that in each of the pairs listed the two pin words end in different letters. We shall call a collision a \emph{minimal collision} if all pin words in the tuple differ in their final letter. We shall first prove that the list above is a complete list of minimal collisions, and then deduce from this that there are no non-minimal collisions. \begin{thm}[Classification of Minimal Collisions] Any minimal collision of length $n \geq 6$ is one of the colliding pairs listed in the table above. \end{thm} \begin{proof} In order to prove this we shall first apply symmetries to this list so that one pin word of each pair ends in $dr$: \begin{figure}[h] \begin{center} \begin{tikzpicture}[scale=0.35] \node[circle, draw, fill=none, inner sep=0pt, minimum width=\plotptradius] (0) at (8,10) {}; \node[permpt] (1) at (10,11) {}; \node[permpt] (2) at (9,8) {}; \draw[thin] (2) -- ++ (0,3.5); \node[permpt] (3) at (6,9) {}; \draw[thin] (3) -- ++ (3.5,0); \node[permpt] (4) at (7,6) {}; \draw[thin] (4) -- ++ (0,3.5); \node[permpt] (5) at (5,7) {}; \draw[thin] (5) -- ++ (2.5,0); \node[permpt] (6) at (3,3) {}; \draw[thin] (6) -- ++ (0,1.5); \node[permpt] (7) at (1,4) {}; \draw[thin] (7) -- ++ (2.5,0); \node[permpt] (8) at (2,1) {}; \draw[thin] (8) -- ++ (0,3.5); \node[permpt] (9) at (12,2) {}; \draw[thin] (9) -- ++ (-10.5,0); \node[permpt] (10) at (11,12) {}; \draw[thin,dashed] (10) -- ++ (0,-10.5); \node[circle,fill,inner sep=0.5pt] (11) at (5.5,6.5) {}; \node[circle,fill,inner sep=0.5pt] (12) at (5,6) {}; \node[circle,fill,inner sep=0.5pt] (13) at (4.5,5.5) {}; \node[circle,fill,inner sep=0.5pt] (14) at (4,5) {}; \node[circle,fill,inner sep=0.5pt] (15) at (3.5,4.5) {}; \draw[thick] (8,0) -- ++ (0,13); \draw[thick] (0,10) -- ++ (13,0); \draw[dotted] (10.5,11.5) rectangle (11.5,12.5); \node[] (16) at (6.5,-1) {$1dldl \dots dldr$}; \node[] (17) at (6.65,-2.5) {$=4ldl \dots dldru$}; \begin{scope}[shift={(18,0)}] \node[circle, draw, fill=none, inner sep=0pt, minimum width=\plotptradius] (0) at (10,5) {}; \node[permpt] (1) at (11,3) {}; \node[permpt] (2) at (8,4) {}; \draw[thin] (2) -- ++ (3.5,0); \node[permpt] (3) at (9,7) {}; \draw[thin] (3) -- ++ (0,-3.5); \node[permpt] (4) at (6,6) {}; \draw[thin] (4) -- ++ (3.5,0); \node[permpt] (5) at (7,8) {}; \draw[thin] (5) -- ++ (0,-2.5); \node[permpt] (6) at (3,10) {}; \draw[thin] (6) -- ++ (1.5,0); \node[permpt] (7) at (4,12) {}; \draw[thin] (7) -- ++ (0,-2.5); \node[permpt] (8) at (1,11) {}; \draw[thin] (8) -- ++ (3.5,0); \node[permpt] (9) at (2,1) {}; \draw[thin] (9) -- ++ (0,10.5); \node[permpt] (10) at (12,2) {}; \draw[thin,dashed] (10) -- ++ (-10.5,0); \node[circle,fill,inner sep=0.5pt] (11) at (6.5,7.5) {}; \node[circle,fill,inner sep=0.5pt] (12) at (6,8) {}; \node[circle,fill,inner sep=0.5pt] (13) at (5.5,8.5) {}; \node[circle,fill,inner sep=0.5pt] (14) at (5,9) {}; \node[circle,fill,inner sep=0.5pt] (15) at (4.5,9.5) {}; \draw[thick] (10,0) -- ++ (0,13); \draw[thick] (0,5) -- ++ (13,0); \draw[dotted] (11.5,1.5) rectangle (12.5,2.5); \node[] (16) at (6.5,-1) {$4lulu \dots luld$}; \node[] (17) at (6.5,-2.5) {$=3ulu \dots luldr$}; \end{scope} \end{tikzpicture} \end{center} \caption{Odd collisions ending in $dr$ of length $\geq 6$} \label{fig:oddcollisionsdr} \end{figure} \begin{figure}[h] \begin{center} \begin{tikzpicture}[scale=0.35] \node[circle, draw, fill=none, inner sep=0pt, minimum width=\plotptradius] (0) at (8,8) {}; \node[permpt] (1) at (9,10) {}; \node[permpt] (2) at (6,9) {}; \draw[thin] (2) -- ++ (3.5,0); \node[permpt] (3) at (7,6) {}; \draw[thin] (3) -- ++ (0,3.5); \node[permpt] (4) at (4,7) {}; \draw[thin] (4) -- ++ (3.5,0); \node[permpt] (5) at (5,5) {}; \draw[thin] (5) -- ++ (0,2.5); \node[permpt] (6) at (1,3) {}; \draw[thin] (6) -- ++ (1.5,0); \node[permpt] (7) at (2,1) {}; \draw[thin] (7) -- ++ (0,2.5); \node[permpt] (8) at (11,2) {}; \draw[thin] (8) -- ++ (-9.5,0); \node[permpt] (9) at (10,11) {}; \draw[thin,dashed] (9) -- ++ (0,-9.5); \node[circle,fill,inner sep=0.5pt] (10) at (4.5,5.5) {}; \node[circle,fill,inner sep=0.5pt] (11) at (4,5) {}; \node[circle,fill,inner sep=0.5pt] (12) at (3.5,4.5) {}; \node[circle,fill,inner sep=0.5pt] (13) at (3,4) {}; \node[circle,fill,inner sep=0.5pt] (14) at (2.5,3.5) {}; \draw[thick] (8,0) -- ++ (0,12); \draw[thick] (0,8) -- ++ (12,0); \draw[dotted] (9.5,10.5) rectangle (10.5,11.5); \node[] (15) at (6.5,-1) {$1ldld \dots ldr$}; \node[] (16) at (6.5,-2.5) {$=2dld \dots ldru$}; \begin{scope}[shift={(18,0)}] \node[circle, draw, fill=none, inner sep=0pt, minimum width=\plotptradius] (0) at (8,4) {}; \node[permpt] (1) at (10,3) {}; \node[permpt] (2) at (9,6) {}; \draw[thin] (2) -- ++ (0,-3.5); \node[permpt] (3) at (6,5) {}; \draw[thin] (3) -- ++ (3.5,0); \node[permpt] (4) at (7,8) {}; \draw[thin] (4) -- ++ (0,-3.5); \node[permpt] (5) at (5,7) {}; \draw[thin] (5) -- ++ (2.5,0); \node[permpt] (6) at (3,11) {}; \draw[thin] (6) -- ++ (0,-1.5); \node[permpt] (7) at (1,10) {}; \draw[thin] (7) -- ++ (2.5,0); \node[permpt] (8) at (2,1) {}; \draw[thin] (8) -- ++ (0,9.5); \node[permpt] (9) at (11,2) {}; \draw[thin,dashed] (9) -- ++ (-9.5,0); \node[circle,fill,inner sep=0.5pt] (10) at (5.5,7.5) {}; \node[circle,fill,inner sep=0.5pt] (11) at (5,8) {}; \node[circle,fill,inner sep=0.5pt] (12) at (4.5,8.5) {}; \node[circle,fill,inner sep=0.5pt] (13) at (4,9) {}; \node[circle,fill,inner sep=0.5pt] (14) at (3.5,9.5) {}; \draw[thick] (8,0) -- ++ (0,12); \draw[thick] (0,4) -- ++ (12,0); \draw[dotted] (10.5,1.5) rectangle (11.5,2.5); \node[] (15) at (6.5,-1) {$4ulul \dots uld$}; \node[] (16) at (6.5,-2.5) {$=1lul \dots uldr$}; \end{scope} \end{tikzpicture} \end{center} \caption{Even collisions ending in $dr$ of length $\geq 6$} \label{fig:evencollisionsdr} \end{figure} We shall now proceed as follows: suppose we have a minimal collision of pin words $w_{1}$ and $w_{2}$ of length $\geq 6$ and that $w_{1}$ ends in $dr$. Then the permutation $\pi^{\circ}$ is of the form given in Fig. \ref{i}. We shall use this to deduce facts about $w_{2}$ in order to show that this collision is in fact one of the pairs listed in Fig.s \ref{fig:oddcollisionsdr} and \ref{fig:evencollisionsdr}. \begin{figure}[h] \begin{center} \begin{tikzpicture}[scale=0.35] \node[] (0) at (2,9) {$w_{1} = \_\_\_\_dr$}; \node[permpt] (1) at (2,0) {}; \draw[thin] (1) -- ++ (0,7); \node[permpt] (2) at (6,1) {}; \draw[thin] (2) -- ++ (-5.5,0); \node[] (3) at (2,-1) {\tiny{(A)}}; \node[] (4) at (6,0) {\tiny{(B)}}; \draw (0.5,2.5) rectangle (3.5,5.5); \end{tikzpicture} \end{center} \caption{The permutation $\pi^{\circ}$ generated by the pin word $w_{1}=\_\_\_\_dr$ has this form. Note that $(A)$ is in the lower half-plane and $(B)$ is in the fourth quadrant. All points other than $(A)$ and $(B)$ (of which there are at least $4$ by the assumption that the length of $\pi^{\circ}$ is $\geq 6$) are in the box, with precisely one point on one side of the pin attached to $(A)$ and all other points on the other.} \label{i} \end{figure} As in Fig \ref{i}, we shall call the lowest and second-lowest points of $\pi^{\circ}$ $(A)$ and $(B)$, respectively. These are generated by the final two letters $dr$ of $w_{1}$. We shall split into four cases based on which position the letter of $w_{2}$ which generates $(A)$ is in: the final letter, the inital numeral, or an internal letter. \subsubsection{Case $1$: $(A)$ is the final point} We begin by deducing various facts about the pin word $w_{2}$, using the fact that the permutation $\pi$ looks like Fig. \ref{ii} (with all points other than $(A)$ and $(B)$ in the box), as well as the assumption that the final letter of $w_{2}$ corresponds to the point $(A)$: \begin{figure}[h] \begin{center} \begin{tikzpicture}[scale=0.35] \node[] (0) at (2,9) {$w_{2} = 4\_\_\_\_d$}; \node[permpt] (1) at (2,0) {}; \draw[thin,dotted] (1) -- ++ (0,7); \node[permpt] (2) at (6,1) {}; \node[] (3) at (2,-1) {\tiny{(A)}}; \node[] (4) at (6,0) {\tiny{(B)}}; \draw (0.5,2.5) rectangle (3.5,5.5); \end{tikzpicture} \end{center} \caption{The fact that the permutation $\pi^{\circ}$ can be generated by the pin word $w_{1}=\_\_\_\_dr$ means that it has this form. Note that $(A)$ is in the lower half-plane and $(B)$ is in the fourth quadrant. All points other than $(A)$ and $(B)$ (of which there are at least $4$ by the assumption that the length of $\pi^{\circ}$ is $\geq 6$) are in the box, with precisely one point on one side of the dotted line attached to $(A)$ and all other points on the other.} \label{ii} \end{figure} \begin{itemize} \item By assumption, the final letter of $w_{2}$ corresponds to the point $(A)$. The point corresponding to the final letter of a pin sequence must be the most extreme point in the direction indicated by that letter. Fig. \ref{ii} shows that $(A)$ is the downmost point but not the most extreme point in any other direction (clearly, $(B)$ is further up and to the right, and the fact that there must be at least one point in the box on each side of the dotted line implies that there is a point further to the left). Hence the final letter of $w_{2}$ \emph{must} be a $d$. \item Consider the pin word $w_{2}^{-}$, formed by removing the final letter of $w_{2}$. This must correspond to a permutation of the shape given in Fig. \ref{iii} (basically Fig. \ref{ii} without the point $(A)$). Note that the point $(B)$ does not separate the bounding rectangle of all other points in the permutation. By the definition of a pin permutation, this can only happen if $(B)$ was the first point placed: so $(B)$ corresponds to the initial numeral of $w_{2}$. But as $(B)$ also corresponds to the final $r$ in the pin word $w_{1} = \_\_\_\_ dr$ it must be in the fourth quadrant. Hence $w_{2}$ must begin with the numeral $4$. \begin{figure}[h] \begin{center} \begin{tikzpicture}[scale=0.35] \node[permpt] (2) at (6,1) {}; \node[] (4) at (6,0) {\tiny{(B)}}; \draw (0.5,2.5) rectangle (3.5,5.5); \end{tikzpicture} \end{center} \caption{The permutation generated by $w_{2}^{-}$ has this form.} \label{iii} \end{figure} \item Hence $w_{2} = 4 \_\_\_\_ d$, with the $4$ corresponding to $(B)$ and the final $d$ corresponding to $(A)$. Suppose that the blank space in the middle contained an $r$. Then this would correspond to a point to the right of all points placed before. But as $(B)$ was the first point placed, this would imply the existence of a point to the right of $(B)$. Fig. \ref{ii} shows that no such point exists, and so $w_{2}$ contains no $r$. \item Similarly, suppose that $w_{2}$ contained another $d$, in addition to the final one. Then this would correspond to a point below $(B)$. But the only point below $(B)$ is $(A)$, already accounted for by the final $d$. Hence $w_{2}$ contains no $d$ apart from its final letter. \item Combining these facts, we see that $w_{2} = 4 \_\_\_\_ d$, with the letters in the blank space alternating between $u$ and $l$. Hence $w_{2} = 4(ul)^{\geq2}d$ or $w_{2} = 4l(ul)^{\geq2}d$, depending of whether the length of $\pi^{\circ}$ is even or odd, respectively. We thus now know what the permutation $\pi$ looks like, as shown in Fig. \ref{iv}. \begin{figure}[h] \begin{center} \begin{tikzpicture}[scale=0.35] \node[circle, draw, fill=none, inner sep=0pt, minimum width=\plotptradius] (0) at (10,2) {}; \node[permpt] (1) at (12,1) {}; \draw[thin,dotted] (1) -- ++ (-10.5,0); \node[permpt] (2) at (11,4) {}; \draw[thin] (2) -- ++ (0,-3.5); \node[permpt] (3) at (8,3) {}; \draw[thin] (3) -- ++ (3.5,0); \node[permpt] (4) at (9,6) {}; \draw[thin] (4) -- ++ (0,-3.5); \node[permpt] (5) at (6,5) {}; \draw[thin] (5) -- ++ (3.5,0); \node[permpt] (6) at (7,7) {}; \draw[thin] (6) -- ++ (0,-2.5); \node[permpt] (7) at (3,8) {}; \draw[thin] (7) -- ++ (1.5,0); \node[permpt] (8) at (4,10) {}; \draw[thin] (8) -- ++ (0,-2.5); \node[permpt] (9) at (1,9) {}; \draw[thin] (9) -- ++ (3.5,0); \node[permpt] (10) at (2,0) {}; \draw[thin] (10) -- ++ (0,9.5); \node[circle,fill,inner sep=0.5pt] (11) at (5,8) {}; \node[circle,fill,inner sep=0.5pt] (12) at (5.5,7.5) {}; \node[circle,fill,inner sep=0.5pt] (13) at (6,7) {}; \node[circle,fill,inner sep=0.5pt] (14) at (6.5,6.5) {}; \node[] (15) at (2,-1) {\tiny{(A)}}; \node[] (16) at (12,0) {\tiny{(B)}}; \draw[thick] (10,-1) -- ++ (0,12); \draw[thick] (0,2) -- ++ (13,0); \begin{scope}[shift={(18,0)}] \node[circle, draw, fill=none, inner sep=0pt, minimum width=\plotptradius] (0) at (10,3) {}; \node[permpt] (1) at (11,1) {}; \draw[thin,dotted] (1) -- ++ (-9.5,0); \node[permpt] (2) at (8,2) {}; \draw[thin] (2) -- ++ (3.5,0); \node[permpt] (3) at (9,5) {}; \draw[thin] (3) -- ++ (0,-3.5); \node[permpt] (4) at (6,4) {}; \draw[thin] (4) -- ++ (3.5,0); \node[permpt] (5) at (7,7) {}; \draw[thin] (5) -- ++ (0,-3.5); \node[permpt] (6) at (5,6) {}; \draw[thin] (6) -- ++ (2.5,0); \node[permpt] (7) at (3,9) {}; \draw[thin] (7) -- ++ (0,-1.5); \node[permpt] (8) at (1,8) {}; \draw[thin] (8) -- ++ (2.5,0); \node[permpt] (9) at (2,0) {}; \draw[thin] (9) -- ++ (0,9.5); \node[circle,fill,inner sep=0.5pt] (10) at (4,8) {}; \node[circle,fill,inner sep=0.5pt] (11) at (4.5,7.5) {}; \node[circle,fill,inner sep=0.5pt] (12) at (5,7) {}; \node[circle,fill,inner sep=0.5pt] (13) at (5.5,6.5) {}; \node[] (14) at (2,-1) {\tiny{(A)}}; \node[] (15) at (11,0) {\tiny{(B)}}; \draw[thick] (10,-1) -- ++ (0,11); \draw[thick] (0,3) -- ++ (12,0); \end{scope} \end{tikzpicture} \end{center} \caption{Even and odd cases for the permutation $\pi^{\circ}$, respectively} \label{iv} \end{figure} \item We now return to $w_{1} = \_\_\_\_dr$; as we now know what the permutation $\pi$ looks like, we can deduce that the blank space here also does not contain a $d$ or $r$: first, if the blank space contained a $d$ then the points corresponding to both this and the next letter would be in the lower half-plane. But Fig. \ref{iv} shows that there is at most one point in the lower half-plane in addition to $(A)$ and $(B)$ (which are already accounted for by the final two letters). Similarly, if the blank space contained an $r$ then the points corresponding to this and the next letter (neither of which can be the point $(B)$ as this is accounted for by the final $r$) would be in the right half-plane. But Fig. \ref{iv} shows that there is at most one point in the right half-plane other than $(B)$. Hence $w_{1} = \_\_\_\_dr$ contains no $d$ or $r$ apart from the final two letters. This is now enough to deduce all of $w_{1}$: $w_{1} = 1(lu)^{\geq1}ldr$ in the even case, and $w_{1} = 3(ul)^{\geq2}dr$ in the odd case. \item This means that there is only one (potential) collision of each length $n \geq 6$ in Case $1$: $w_{1} = 1(lu)^{\geq1}ldr$ paired with $w_{2} = 4(ul)^{\geq2}d$ in the even case, and $w_{1} = 3(ul)^{\geq2}dr$ paired with $w_{2} = 4l(ul)^{\geq2}d$ in the odd case. These are the collisions on the right hand sides of Fig.s \ref{fig:oddcollisionsdr} and \ref{fig:evencollisionsdr}. \end{itemize} \subsubsection{Case $2$: $(A)$ is the first point} We now deal with the case in which $(A)$ is the first point placed according to the pin word $w_{2}$, corresponding to the initial numeral. We will again use the shape of the permutation $\pi$ shown in Fig. \ref{i} to deduce the form of $w_{2}$. \begin{itemize} \item First note that, as $(A)$ corresponds to the letter $d$ in $w_{1}$, $(A)$ must be in the $3$rd or $4$th quadrant. Hence $w_{2} = \{3/4\}\_\_\_\_$ \item Next, note that, as $(B)$ is not the first point placed, it must correspond to a letter. If this letter were $d$ or $l$ then $(B)$ would be below or to the left of $(A)$ (as $(A)$ has already been placed), which it is not (see Fig. \ref{i}). If the letter were $u$, then $(B)$ would be the upmost of all points placed so far, and as Fig. \ref{i} shows that all other points of $\pi$ are above $(B)$, this would mean that $(B)$ would have to be the second point placed, so $w_{2}$ begins with either $3u$ or $4u$: in the first case, both $(A)$ and $(B)$ would be in quadrant $3$ and in the second case $(B)$ would be to the left of $(A)$ - Fig. \ref{i} shows that neither of these is true, so $(B)$ cannot correspond to a $u$ in $w_{2}$. Hence, by elimination, $(B)$ corresponds to an $r$ in $w_{2}$. \item Hence $(B)$ is at the end of a right-pin separating the previously placed point from the bounding rectangle of all other previously placed points and the origin. As the origin is above $(B)$ (as we know $(B)$ is in quadrant $4$), the previously placed point must be the only point below $(B)$, namely $(A)$. Hence $(B)$ corresponds to the first letter of $w_{2}$ after the numeral. \item Hence $w_{2}=\{3/4\}r\_\_\_\_$. We can now easily deduce that the blank space here contains no $r$ or $d$: if it contained an $r$ then this would correspond to a point to the right of $(B)$ (as this has already been placed), and it if contained a $d$ then this would correspond to a point below $(A)$; but Fig. \ref{i} clearly shows that no point of either type exists. \item Hence $w_{2} = \{3/4\}rulul \dots$, and so the permutation $\pi^{\circ}$ looks like one of the permutations in Fig. \ref{v}. \begin{figure}[h] \begin{center} \begin{tikzpicture}[scale=0.35] \node[circle, draw, fill=none, inner sep=0pt, minimum width=\plotptradius] (0) at (5,3) {}; \node[permpt] (1) at (4,1) {}; \node[permpt] (2) at (7,2) {}; \draw[thin] (2) -- ++ (-3.5,0); \node[permpt] (3) at (6,5) {}; \draw[thin] (3) -- ++ (0,-3.5); \node[permpt] (4) at (2,4) {}; \draw[thin] (4) -- ++ (4.5,0); \node[permpt] (5) at (3,7) {}; \draw[thin] (5) -- ++ (0,-3.5); \node[permpt] (6) at (1,6) {}; \draw[thin] (6) -- ++ (2.5,0); \node[circle,fill,inner sep=0.5pt] (7) at (1.5,7) {}; \node[circle,fill,inner sep=0.5pt] (8) at (1,7.5) {}; \node[circle,fill,inner sep=0.5pt] (9) at (0.5,8) {}; \node[circle,fill,inner sep=0.5pt] (10) at (0,8.5) {}; \node[] (11) at (4,0) {\tiny{(A)}}; \node[] (12) at (7,1) {\tiny{(B)}}; \node[] (13) at (7,5) {\tiny{(C)}}; \draw[thick] (5,0) -- ++ (0,8); \draw[thick] (0,3) -- ++ (8,0); \begin{scope}[shift={(13,0)}] \node[circle, draw, fill=none, inner sep=0pt, minimum width=\plotptradius] (0) at (4,3) {}; \node[permpt] (1) at (5,1) {}; \node[permpt] (2) at (7,2) {}; \draw[thin] (2) -- ++ (-2.5,0); \node[permpt] (3) at (6,5) {}; \draw[thin] (3) -- ++ (0,-3.5); \node[permpt] (4) at (2,4) {}; \draw[thin] (4) -- ++ (4.5,0); \node[permpt] (5) at (3,7) {}; \draw[thin] (5) -- ++ (0,-3.5); \node[permpt] (6) at (1,6) {}; \draw[thin] (6) -- ++ (2.5,0); \node[circle,fill,inner sep=0.5pt] (7) at (1.5,7) {}; \node[circle,fill,inner sep=0.5pt] (8) at (1,7.5) {}; \node[circle,fill,inner sep=0.5pt] (9) at (0.5,8) {}; \node[circle,fill,inner sep=0.5pt] (10) at (0,8.5) {}; \node[] (11) at (5,0) {\tiny{(A)}}; \node[] (12) at (7,1) {\tiny{(B)}}; \node[] (13) at (7,5) {\tiny{(C)}}; \draw[thick] (4,0) -- ++ (0,8); \draw[thick] (0,3) -- ++ (8,0); \end{scope} \end{tikzpicture} \end{center} \caption{The two possible cases for the permutation $\pi^{\circ}$, as generated by the pin word $w_{2} = \{3/4\}rulul \dots$. As $n \geq 6$ all the points shown must exist; any further points are a continuation of the oscillation in the second quadrant.} \label{v} \end{figure} \item We now return to $w_{1} = \_\_\_\_dr$ which also generates $\pi^{\circ}$, with the final two letters corresponding to $(A)$ and $(B)$. Note first that all the points in the upper half-plane of Fig. \ref{v} have been placed before $(A)$ in $w_{1}$, and in either case there are at least three points to the left of $(A)$ which have been placed already: this means that the previous letter to the $d$ corresponding to $(A)$ must have been an $r$, and this must correspond to the point marked $(C)$ (the only point other than $(B)$ in the right half-plane). Thus $w_{1} = \_\_\_\_rdr$, with the final three letters corresponding to $(C)$, $(A)$, $(B)$, respectively. Note that the blank space here cannot contain a $d$, as $(A)$ and $(B)$ (already accounted for by the final two letters) are the only points in the lower half-plane. Thus $w_{1} = \_\_\_\_urdr$. But this implies that $(C)$ is the second-upmost point, despite Fig. \ref{v} showing that $(C)$ has at least two points above it. This contradiction implies that there are in fact no collisions in Case $2$. \end{itemize} \subsubsection{Case $3$: $(A)$ is an internal point} We now deal with the case where $(A)$ is generated an internal letter of the pin word $w_{2}$ (ie., neither the initial numeral nor the final letter): \begin{itemize} \item First, we determine which letter represents $(A)$ in $w_{2}$. By looking at Fig. \ref{i}, we see that $(A)$ is in the lower half-plane and is indeed the lowest point in the entire permutation. If $(A)$ were represented by a $u$ in $w_{2}$ it would be in the upper half-plane, so we can exclude this possibility. If $(A)$ were represented by an $l$ or $r$, on the other hand, it would have to follow either the numeral $3$ or $4$ or the letter $d$ (as otherwise $(A)$ would be in the upper half-plane); but in this case $(A)$ would be above the previously-placed point, contradicting the fact that it is the lowest point in the entire permutation. Hence $(A)$ must be represented by the letter $d$ in $w_{2}$. \item Next, note that the $d$ representing $(A)$ must in fact be the \emph{final} $d$ in $w_{2}$: if there were a $d$ after $(A)$ it would place a point below $(A)$, but $(A)$ is the lowest point in the entire permutation. As $(A)$ is internal to $w_{2}$ there is a letter immediately after the $d$ corresponding to $(A)$ in $w_{2}$, either an $l$ or an $r$. The point corresponding to this next letter, which we shall call $(B')$, will be the second-lowest point \emph{so far} when it is placed. But since all points placed after $(B')$ (if there are any) will be in the upper half-plane (as there is no further $d$ in the word), $(B')$ is in fact the second-lowest point in the \emph{entire} permutation. Hence $(B')$ must actually be $(B)$, which must correspond to an $r$ in $w_{2}$ as it is to the right of $(A)$. \item We now know that the points $(A)$ and $(B)$ are generated by a consecutive $dr$ in $w_{2}$, and that there is no letter $d$ in $w_{2}$ after this $dr$ appears; by the same token there is also no further $r$ after this $dr$, as this would generate a point to the right of $(B)$. Hence, after the $dr$ in $w_{2}$ corresponding to $(A),(B)$, the pin word $w_{2}$ alternates between $u$ and $l$ for any remaining points. We now ask how many points are remaining in $w_{2}$ after this final $dr$. \item If there were \emph{no} points after the $dr$ in $w_{2}$ then $w_{1}$ and $w_{2}$ would both end in $dr$, so this would not be a pair of \emph{minimal} collisions. Hence we can assume that there is at least one further letter after the final $dr$, which must be a $u$. \item Now suppose that there were at least two letters after the final $dr$; then $w_{2}$ would have the form $\dots drul \dots$, with any further letters at the end alternating between $u$ and $l$. Thus $\pi^{\circ}$ would look like the permutation shown on the right-hand side of Fig. \ref{vi}. \begin{figure}[h] \begin{center} \begin{tikzpicture}[scale=0.35] \node[] (0) at (2,9) {$w_{1} = \dots dr$}; \node[permpt] (1) at (2,0) {}; \draw[thin] (1) -- ++ (0,5.5); \node[permpt] (2) at (5,1) {}; \draw[thin] (2) -- ++ (-3.5,0); \node[] (3) at (2,-1) {\tiny{(A)}}; \node[] (4) at (5,0) {\tiny{(B)}}; \draw (0.5,2) rectangle (3.5,5); \begin{scope}[shift={(13,2)}] \node[] (0) at (1,7) {$w_{2} = \dots drul \dots$}; \node[permpt] (1) at (1,-2) {}; \draw[thin] (1) -- ++ (0,4.5); \node[permpt] (2) at (4,-1) {}; \draw[thin] (2) -- ++ (-3.5,0); \node[permpt] (3) at (3,4) {}; \draw[thin] (3) -- ++ (0,-5.5); \node[permpt] (4) at (-2,3) {}; \draw[thin] (4) -- ++ (5.5,0); \node[] (6) at (1,-3) {\tiny{(A)}}; \node[] (7) at (4,-2) {\tiny{(B)}}; \node[] (8) at (3,5) {\tiny{(C)}}; \node[] (9) at (-2,2) {\tiny{(D)}}; \node[circle,fill,inner sep=0.5pt] (8) at (-1.5,4) {}; \node[circle,fill,inner sep=0.5pt] (9) at (-2,4.5) {}; \node[circle,fill,inner sep=0.5pt] (10) at (-2.5,5) {}; \node[circle,fill,inner sep=0.5pt] (11) at (-3,5.5) {}; \draw (0,0) rectangle (2,2); \end{scope} \end{tikzpicture} \end{center} \caption{The permutation $\pi^{\circ}$ is (under the assumption that there are at least two letters after the $r$ that generates $(B)$ in $w_{2}$) generated by both the words $w_{1} = \dots dr$ and $w_{2} = \dots drul \dots$, so has both of these forms. The points marked $(A)$ and $(B)$ must match up.} \label{vi} \end{figure} \item Hence the permutation $\pi^{\circ}$ is simultaneously of both forms represented in Fig. \ref{vi}. On closer inspection, however, these forms contradict each other, which we can see by counting the points (other than $(B)$) to the left and right of $(A)$: looking first at the diagram generated by $w_{1}$, the box here is non-empty (as the length of $\pi^{\circ}$ is $\geq 6$), and the pin attached to $(A)$ must separate the previously-placed point from all others (including the origin). Hence, excluding $(B)$, there is precisely one point on one side of $(A)$ and all other points (including the origin) are on the other side. Looking at the diagram generated by $w_{2}$, however, we see that the box here is also non-empty (as $(A)$ is internal there is at least one non-origin point preceeding it), with the previous point on one side and all other points (including the origin) on the other. Including the points marked $(C)$ and $(D)$, we now see that there are at least two points (including the origin but excluding $(B)$) on either side of $(A)$, thus contradicting what the diagram generated by $w_{1}$ told us. We conclude that there cannot in fact be more than one point after the final $dr$ in $w_{2}$. \item Thus there is in fact precisely one point after the final $dr$, corresponding to $(A),(B)$ in $w_{2}$, and so $w_{2} = \dots dru$. Hence our permutation $\pi^{\circ}$ is of both the forms shown in Fig. \ref{vii}. \begin{figure}[h] \begin{center} \begin{tikzpicture}[scale=0.35] \node[] (0) at (2,9) {$w_{1} = \dots dr$}; \node[permpt] (1) at (2,0) {}; \draw[thin] (1) -- ++ (0,5.5); \node[permpt] (2) at (5,1) {}; \draw[thin] (2) -- ++ (-3.5,0); \node[] (3) at (2,-1) {\tiny{(A)}}; \node[] (4) at (5,0) {\tiny{(B)}}; \draw (0.5,2) rectangle (3.5,5); \begin{scope}[shift={(13,2)}] \node[] (0) at (1,7) {$w_{2} = \dots dru$}; \node[permpt] (1) at (1,-2) {}; \draw[thin] (1) -- ++ (0,4.5); \node[permpt] (2) at (4,-1) {}; \draw[thin] (2) -- ++ (-3.5,0); \node[permpt] (3) at (3,4) {}; \draw[thin] (3) -- ++ (0,-5.5); \node[] (6) at (1,-3) {\tiny{(A)}}; \node[] (7) at (4,-2) {\tiny{(B)}}; \node[] (8) at (3,5) {\tiny{(C)}}; \draw (0,0) rectangle (2,2); \end{scope} \end{tikzpicture} \end{center} \caption{The permutation $\pi^{\circ}$ is generated by both the words $w_{1} = \dots dr$ and $w_{2} = \dots dru$, so has both of these forms. The points marked $(A)$ and $(B)$ must match up.} \label{vii} \end{figure} \item But now consider the pin sequence $w_{1}^{-1}$, obtained by removing the final letter from $w_{1}$. This must generate a pin permutation corresponding to the permutations shown in Fig. \ref{vii} with the point $(B)$ removed; in particular, it contains the point $(C)$ which (given the absence of $(B)$) will be both the highest and rightmost point in the corresponding pin permutation $\pi^{\circ}_{w_{1}^{-1}}$. This can only happen if $(C)$ was the \emph{first} point placed in $\pi^{\circ}_{w_{1}^{-1}}$ (otherwise is separates the previously placed point from all others in one direction, making it only the second-most point in that direction); as $(C)$ is clearly in the first quadrant, this means that $w_{1}$ starts with a $1$. Hence $w_{1} = 1 \dots dr$. Further, given that there is no point above $(C)$ in $\pi^{\circ}$ and the only point to the right is $(B)$, accounted for by the final $r$, this means that there is no $u$ in $w_{1}$ and no $r$ except for the final letter. Hence $w_{1}$ is either $1l(dl)^{\geq 1}dr$ or $1(dl)^{\geq 2}dr$, depending on whether the length of $\pi^{\circ}$ is even or odd. Thus $\pi^{\circ}$ is one of the permutations on the left-hand sides of Fig.s \ref{fig:oddcollisionsdr} and \ref{fig:evencollisionsdr}, with $w_{1}$ being the upper word in the pair. Noting that $w_{2} = \dots dru$ cannot contain an $r$ or $u$ apart from the final two letters (all points on the upper and right half-planes are already accounted for) allows us to conclude that $w_{2}$ must be the lower word in the pair. \end{itemize} \end{proof} We can now quickly deduce that \emph{all} collisions are in fact minimal, and thus the list given above is complete: \begin{lemma} Every colliding tuple $\{w_{1},w_{2}, \dots, w_{k}\}$ of pin words is minimal (that is to say, $k \leq 4$ and each pair $w_{i},w_{j}$ differs in the final letter). \end{lemma} \begin{proof} Note that every collision of length $n \leq 5$ (which we have exhaustively listed) is minimal. Now suppose that $\{w_{1},w_{2}\}$ is a non-minimal colliding pair of some length $n \geq 6$. By applying symmetries, we can assume that both $w_{1}$ and $w_{2}$ ends in a $d$. But then this $d$ generates the lowest point in the shared generated pin permutation $\pi^{\circ}$ - in particular, it represents the same point in each pin word. Thus $w_{1}^{-1}$ and $w_{2}^{-1}$ (the pin words obtained by removing the final $d$ from each word in the pair) must also form a collision, indeed a collision of shorter length. Given that $w_{1}$ and $w_{2}$ are not the same word, we can continue this process until we arrive at a minimal collision $w_{1}^{-k},w_{2}^{-k}$. But we have already classified all minimal collisions, and can note that all minimal collisions are of different type: if one of the pair ends in $d$ or $u$ then the other must end in $l$ or $r$, and therefore cannot be extended to a collision of longer length, forming a contradiction. \end{proof} Thus all collisions are minimal and the list given in the table is complete. \subsection{Classification of $\boxplus$-decomposables} We now aim to prove that the list of $\boxplus$-decomposables given in Theorem \ref{classification} is complete: \begin{proof} First, we list the $\boxplus$-decomposables of lengths $2$ and $3$: these are $1l$ and $1ld$ along with their eight symmetries, as can be confirmed by an exhaustive list. We will now construct a $\boxplus$-decomposable pin permutation $\pi^{\circ}$ of length $n \geq 4$, generated by the pin word $w$, and show that it must be one of those listed in the statement of the theorem. We proceed by noting that every $\boxplus$-decomposable pin permutation can be drawn on the diagram in Fig. \ref{fig:boxdec1}, with no points in the shaded region, and at least one point in each of the inner and outer regions. \begin{figure}[h] \begin{center} \begin{tikzpicture}[scale=0.25] \draw[very thick] (0,-11) -- ++ (0,22); \draw[very thick] (-11,0) -- ++ (22,0); \draw[thin] (-4,-11) -- ++ (0,22); \draw[thin] (4,-11) -- ++ (0,22); \draw[thin] (-11,-4) -- ++ (22,0); \draw[thin] (-11,4) -- ++ (22,0); \draw[pattern=crosshatch,pattern color=black!80,draw=none] (-4,4) rectangle (4,11); \draw[pattern=crosshatch,pattern color=black!80,draw=none] (-11,-4) rectangle (-4,4); \draw[pattern=crosshatch,pattern color=black!80,draw=none] (-4,-11) rectangle (4,-4); \draw[pattern=crosshatch,pattern color=black!80,draw=none] (4,-4) rectangle (11,4); \end{tikzpicture} \end{center} \caption{The general shape of a $\boxplus$-decomposable permutation. The shaded region must be empty, whilst both the inner and outer unshaded regions must be non-empty.} \label{fig:boxdec1} \end{figure} By applying symmetries if necessary we can assume that the first point of $\pi^{\circ}$ placed was in the first quadrant. In fact, this point must be placed in the outer region of the first quadrant, as it is impossible to get out of the inner region once a point has been placed in there: if the point $p_{n}$ has been placed in the inner region then the point $p_{n+1}$ will be closer to the origin in the direction specified by the letter that placed $p_{n}$, and hence must also be in the inner region. Hence our permutation $\pi^{\circ}$ starts out like that in Fig. \ref{fig:boxdec2}. \begin{figure}[h] \begin{center} \begin{tikzpicture}[scale=0.25] \draw[very thick] (0,-11) -- ++ (0,22); \draw[very thick] (-11,0) -- ++ (22,0); \draw[thin] (-4,-11) -- ++ (0,22); \draw[thin] (4,-11) -- ++ (0,22); \draw[thin] (-11,-4) -- ++ (22,0); \draw[thin] (-11,4) -- ++ (22,0); \node[permpt,label={\tiny $p_{1}$}] (1) at (5,6) {}; \draw[pattern=crosshatch,pattern color=black!80,draw=none] (-4,4) rectangle (4,11); \draw[pattern=crosshatch,pattern color=black!80,draw=none] (-11,-4) rectangle (-4,4); \draw[pattern=crosshatch,pattern color=black!80,draw=none] (-4,-11) rectangle (4,-4); \draw[pattern=crosshatch,pattern color=black!80,draw=none] (4,-4) rectangle (11,4); \end{tikzpicture} \end{center} \caption{First point of $\pi^{\circ}$ is placed in the outer region of the first quadrant.} \label{fig:boxdec2} \end{figure} Next, note that it is impossible to get into the inner region of the first quadrant now: every point placed in the first quadrant from now on will be either to the right of or above $p_{1}$. Hence in order to construct a $\boxplus$-decomposable, we must at some point move to another quadrant. Again, by symmetry, we can assume that the next quadrant visited by the pin permutation $\pi^{\circ}$ is the second quadrant, and we now split into cases based on whether this first point in the second quadrant is in the inner or outer region: \subsubsection{Case $1$: First point in the second quadrant is in the outer region} In this case, we may (or may not) begin by oscillating in the first quadrant for any length: if there is an oscillation of length $\geq 2$ this will ensure that the first point placed in the second quadrant will be in the outer region (it will be the second-highest point placed so far, and so will be above $p_{1}$, the first point placed, which is in the outer region); if not, we shall simply make this assumption for now. We thus have a permutation that looks like that in Fig. \ref{fig:boxdec3}. \begin{figure}[h] \begin{center} \begin{tikzpicture}[scale=0.25] \draw[very thick] (0,-11) -- ++ (0,22); \draw[very thick] (-11,0) -- ++ (22,0); \draw[thin] (-4,-11) -- ++ (0,22); \draw[thin] (4,-11) -- ++ (0,22); \draw[thin] (-11,-4) -- ++ (22,0); \draw[thin] (-11,4) -- ++ (22,0); \node[permpt,label={\tiny $p_{1}$}] (1) at (5,6) {}; \node[permpt,label={\tiny $p_{2}$}] (2) at (7,5) {}; \draw[thin] (2) -- ++ (-2.5,0); \node[permpt,label={\tiny $p_{3}$}] (3) at (6,8) {}; \draw[thin] (3) -- ++ (0,-3.5); \node[permpt,label={\tiny $p_{4}$}] (4) at (9,7) {}; \draw[thin] (4) -- ++ (-3.5,0); \node[permpt,label={\tiny $p_{5}$}] (5) at (8,10) {}; \draw[thin] (5) -- ++ (0,-3.5); \node[permpt,label={\tiny $p_{6}$}] (6) at (-6,9) {}; \draw[thin] (6) -- ++ (14.5,0); \draw[pattern=crosshatch,pattern color=black!80,draw=none] (-4,4) rectangle (4,11); \draw[pattern=crosshatch,pattern color=black!80,draw=none] (-11,-4) rectangle (-4,4); \draw[pattern=crosshatch,pattern color=black!80,draw=none] (-4,-11) rectangle (4,-4); \draw[pattern=crosshatch,pattern color=black!80,draw=none] (4,-4) rectangle (11,4); \draw[pattern=crosshatch,pattern color=red!80,draw=none] (0,-4) rectangle (4,4); \draw[pattern=crosshatch,pattern color=red!80,draw=none] (-4,0) rectangle (0,4); \end{tikzpicture} \end{center} \caption{The first point of $\pi^{\circ}$ outside the first quadrant can be assumed to be in the second quadrant by symmetry.} \label{fig:boxdec3} \end{figure} We note that the inner regions of the first, second and fourth quadrants (shaded in red in Fig. \ref{fig:boxdec3}) are now inaccesible: there are now at least two points in the outer region of the upper half-plane, and because any new point in the upper half-plane must be either the highest or second-highest placed so far, it is now impossible to place any points below those two, hence the inner region of the upper half-plane will remain empty. A similar argument shows that the inner region of the right-halt plane is now also inaccesible: if we had oscillated in the first quadrant initially then there are now at least two points to the right of the inner region in the right half-plane, and the argument goes through exactly as before. If we went directly to the second quadrant with our second point then there would be only one point to the right of the inner region so far, but in order to place anything in the right half-plane again we would need to take a right step, which would create a second, thus rendering the inner region of the right half-plane again inaccesible. In any case, the only part of the inner region which is now accesible is in the third quadrant, so we must be aiming to end up there. If, after placing our first point in the second quadrant, we took either an upward step or a downward step into the outer region of the third quadrant, then the whole of the inner region would be rendered inaccesbile, by the same argument as in the previous paragraph. Hence we must now take a downstep into the inner region of the third quadrant, as in Fig. \ref{fig:boxdec4}. \begin{figure}[h] \begin{center} \begin{tikzpicture}[scale=0.25] \draw[very thick] (0,-11) -- ++ (0,22); \draw[very thick] (-11,0) -- ++ (22,0); \draw[thin] (-4,-11) -- ++ (0,22); \draw[thin] (4,-11) -- ++ (0,22); \draw[thin] (-11,-4) -- ++ (22,0); \draw[thin] (-11,4) -- ++ (22,0); \node[permpt,label={\tiny $p_{1}$}] (1) at (5,6) {}; \node[permpt,label={\tiny $p_{2}$}] (2) at (7,5) {}; \draw[thin] (2) -- ++ (-2.5,0); \node[permpt,label={\tiny $p_{3}$}] (3) at (6,8) {}; \draw[thin] (3) -- ++ (0,-3.5); \node[permpt,label={\tiny $p_{4}$}] (4) at (9,7) {}; \draw[thin] (4) -- ++ (-3.5,0); \node[permpt,label={\tiny $p_{5}$}] (5) at (8,10) {}; \draw[thin] (5) -- ++ (0,-3.5); \node[permpt,label={\tiny $p_{6}$}] (6) at (-6,9) {}; \draw[thin] (6) -- ++ (14.5,0); \node[permpt,label={\tiny $p_{7}$}] (7) at (-2,-2) {}; \draw[pattern=crosshatch,pattern color=black!80,draw=none] (-4,4) rectangle (4,11); \draw[pattern=crosshatch,pattern color=black!80,draw=none] (-11,-4) rectangle (-4,4); \draw[pattern=crosshatch,pattern color=black!80,draw=none] (-4,-11) rectangle (4,-4); \draw[pattern=crosshatch,pattern color=black!80,draw=none] (4,-4) rectangle (11,4); \draw[pattern=crosshatch,pattern color=red!80,draw=none] (0,-4) rectangle (4,4); \draw[pattern=crosshatch,pattern color=red!80,draw=none] (-4,0) rectangle (0,4); \draw [thin] plot [smooth] coordinates {(-5,9.5) (-5,7) (-4,6) (-2,4) (-2,-2)}; \end{tikzpicture} \end{center} \caption{This is $\boxplus$-indecomposable, but will not be if any right or up step is taken.} \label{fig:boxdec4} \end{figure} Now note that we cannot now place any further points: either a left or right step here will place a point into the shaded region. Hence the only $\boxplus$-decomposable pin permutations in Case $1$ are those shown in Fig. \ref{fig:boxdec4}, which is to say those generated by a pin word of the form $1 \dots ld$, where the only letters in the ellipsis are $u$ and $r$. \subsubsection{Case $2$: First point in the second quadrant is in the inner region} In this case, it is clear that we have to move into the second quadrant immediately after placing the first point in quadrant $1$ (otherwise the first point in quadrant $2$ will be above $p_{1}$ and hence not in the inner region). Hence we start off as in Fig. \ref{fig:boxdec5}. \begin{figure}[h] \begin{center} \begin{tikzpicture}[scale=0.25] \draw[very thick] (0,-11) -- ++ (0,22); \draw[very thick] (-11,0) -- ++ (22,0); \draw[thin] (-7,-11) -- ++ (0,22); \draw[thin] (5,-11) -- ++ (0,22); \draw[thin] (-11,-7) -- ++ (22,0); \draw[thin] (-11,4) -- ++ (22,0); \node[permpt,label={\tiny $p_{1}$}] (1) at (7,6) {}; \node[permpt,label={\tiny $p_{2}$}] (2) at (-2,2) {}; \draw[pattern=crosshatch,pattern color=black!80,draw=none] (-7,4) rectangle (5,11); \draw[pattern=crosshatch,pattern color=black!80,draw=none] (-11,-7) rectangle (-7,4); \draw[pattern=crosshatch,pattern color=black!80,draw=none] (-7,-11) rectangle (5,-7); \draw[pattern=crosshatch,pattern color=black!80,draw=none] (5,-7) rectangle (11,4); \draw [thin] plot [smooth] coordinates {(7.5,5) (5,5) (3,4) (0,2) (-2,2)}; \end{tikzpicture} \end{center} \caption{This is $\boxplus$-indecomposable, but will not be if any further point is added.} \label{fig:boxdec5} \end{figure} Note that we can now no longer take an up- or right-step: doing so would place a point above or to the right of $p_{1}$ and therefore within the shaded region. Conversely, if we only take down- and left-steps we can stay in the inner region indefinitely, as shown in Fig. \ref{fig:boxdec6}. \begin{figure}[h] \begin{center} \begin{tikzpicture}[scale=0.25] \draw[very thick] (0,-11) -- ++ (0,22); \draw[very thick] (-11,0) -- ++ (22,0); \draw[thin] (-7,-11) -- ++ (0,22); \draw[thin] (5,-11) -- ++ (0,22); \draw[thin] (-11,-7) -- ++ (22,0); \draw[thin] (-11,4) -- ++ (22,0); \node[permpt,label={\tiny $p_{1}$}] (1) at (7,6) {}; \node[permpt,label={\tiny $p_{2}$}] (2) at (-2,2) {}; \node[permpt,label={\tiny $p_{3}$}] (3) at (-1,-3) {}; \draw[thin] (3) -- ++ (0,5.5); \node[permpt,label={\tiny $p_{4}$}] (4) at (-4,-2) {}; \draw[thin] (4) -- ++ (3.5,0); \node[permpt,label={\tiny $p_{5}$}] (5) at (-3,-5) {}; \draw[thin] (5) -- ++ (0,3.5); \node[permpt,label={\tiny $p_{6}$}] (6) at (-6,-4) {}; \draw[thin] (6) -- ++ (3.5,0); \node[permpt,label={\tiny $p_{7}$}] (6) at (-5,-6) {}; \draw[thin] (6) -- ++ (0,2.5); \draw[pattern=crosshatch,pattern color=black!80,draw=none] (-7,4) rectangle (5,11); \draw[pattern=crosshatch,pattern color=black!80,draw=none] (-11,-7) rectangle (-7,4); \draw[pattern=crosshatch,pattern color=black!80,draw=none] (-7,-11) rectangle (5,-7); \draw[pattern=crosshatch,pattern color=black!80,draw=none] (5,-7) rectangle (11,4); \draw [thin] plot [smooth] coordinates {(7.5,5) (5,5) (3,4) (0,2) (-2,2)}; \end{tikzpicture} \end{center} \caption{The general form of a $\boxplus$-decomposable pin permutation in Case $2$} \label{fig:boxdec6} \end{figure} Hence we see that we have obtained another family of $\boxplus$-decomposable pin permutations, those generated by the pin words $1ldldl \dots$, and that these are exhaustive in Case $2$. Combining these two cases, we can now see that we have exhaustively obtained all possible $\boxplus$-decomposable pin permutations that begin in the first quadrant and first visit the second. By applying all eight symmetries we thus obtain all $\boxplus$-decomposable pin permutations and can see that these are precisely those described in the statement of the Theorem \ref{classification}. \end{proof} \def\cprime{$'$} \begin{thebibliography}{10} \bibitem{arratia:on-the-stanley-:} Richard Arratia. \newblock On the {S}tanley-{W}ilf conjecture for the number of permutations avoiding a given pattern. \newblock {\em Electron. J. Combin.}, 6:Note 1, 4 pp., 1999. \bibitem{bassino:enumeration-of-:} Fr{\'e}d{\'e}rique Bassino, Mathilde Bouvel, and Dominique Rossin. \newblock Enumeration of pin-permutations. \newblock {\em Electron. J. Combin.}, 18(1):Paper 57, 39, 2011. \bibitem{bbr:unicyclicgrids:} D.~Bevan, R.~Brignall, and N.~Ru\v{s}kuc. \newblock On cycles in monotone grid classes of permutations. \newblock Submitted. \bibitem{bevan2015defs} David Bevan. \newblock Permutation patterns: basic definitions and notation. \newblock arXiv:1506.06673, 2015. \bibitem{bevan:intervals} David Bevan. \newblock Intervals of permutation class growth rates. \newblock {\em Combinatorica}, 38(2):279--303, Apr 2018. \bibitem{brignall-jarvis:pin-classes-ii} R.~Brignall and B.~Jarvis. \newblock Pin classes {II}: {S}mall pin classes. \newblock Submitted. \bibitem{bv:wqo-uncountable:} R.~Brignall and V.~Vatter. \newblock Uncountably many enumerations of well-quasi-ordered permutation classes. \newblock Submitted. \bibitem{brignall:decomposing-sim:} Robert Brignall, Sophie Huczynska, and Vincent Vatter. \newblock Decomposing simple permutations, with enumerative consequences. \newblock {\em Combinatorica}, 28:385--400, 2008. \bibitem{brignall:simple-permutat:b} Robert Brignall, Nik Ru\v{s}kuc, and Vincent Vatter. \newblock Simple permutations: decidability and unavoidable substructures. \newblock {\em Theoret. Comput. Sci.}, 391(1--2):150--163, 2008. \bibitem{cartier:problemes-combi:} P.~Cartier and D.~Foata. \newblock {\em Probl\`emes combinatoires de commutation et r\'earrangements}. \newblock Lecture Notes in Mathematics, No. 85. Springer-Verlag, Berlin, 1969. \bibitem{flajolet:analytic-combin:} Philippe Flajolet and Robert Sedgewick. \newblock {\em Analytic combinatorics}. \newblock Cambridge University Press, Cambridge, 2009. \bibitem{marcus:excluded-permut:} Adam Marcus and G{\'a}bor Tardos. \newblock Excluded permutation matrices and the {S}tanley-{W}ilf conjecture. \newblock {\em J. Combin. Theory Ser. A}, 107(1):153--160, 2004. \bibitem{murphy:restricted-perm:} Maximillian~M. Murphy. \newblock {\em Restricted permutations, antichains, atomic classes, and stack sorting}. \newblock PhD thesis, Univ. of St Andrews, 2002. \bibitem{vatter:small-permutati:} Vincent Vatter. \newblock Small permutation classes. \newblock {\em Proc. Lond. Math. Soc. (3)}, 103:879--921, 2011. \bibitem{waton:on-permutation-:} Steve Waton. \newblock {\em On Permutation Classes Defined by Token Passing Networks, Gridding Matrices and Pictures: Three Flavours of Involvement}. \newblock PhD thesis, Univ. of St Andrews, 2007. \end{thebibliography} \end{document} \usepackage{ifthen} \usetikzlibrary{calc} \usetikzlibrary{decorations.pathreplacing,decorations.markings} \newcommand{\plotptradius}{4pt} \newcommand{\setplotptradius}[1]{\renewcommand{\plotptradius}{#1}} \tikzset{permpt/.style={circle, draw, fill=black, inner sep=0pt, minimum width=\plotptradius}} \tikzset{empty/.style={draw=none, fill=none}} \newcommand\absdot[2]{ \node[permpt] at #1 {}; } \newcommand\absdothollow[2]{ \node[fill=white] at #1 {}; \node[draw=none,fill=none] at #1 [below] {$#2$}; } \newcommand{\plotperm}[2][black]{ \foreach \j [count=\i] in {#2} { \ifnum0=\j {} \else { \node[permpt,fill=#1,draw=#1] (\j) at (\i,\j) {}; }; } \newcommand{\plotpermbox}[4]{ \draw [darkgray, very thick, line cap=round, fill=white] ({#1-0.5}, {#2-0.5}) rectangle ({#3+0.5}, {#4+0.5}); } \newcommand{\plotpermborder}[1]{ \foreach \i [count=\nn] in {#1} {\global\let\n\nn}; \plotpermbox{1}{1}{\n}{\n}; \plotperm{#1}; } \newcommand{\plotpermbordergrid}[1]{ \foreach \i [count=\nn] in {#1} {\global\let\n\nn}; \plotpermbox{1}{1}{\n}{\n}; \draw[step=1cm,gray!50,very thin] (1,1) grid (\n,\n); \plotperm{#1}; } \newcommand{\plotgrid}[1]{ \draw[step=1cm,gray!50,very thin] (0.5,0.5) grid (#1+0.5,#1+0.5); } \newcommand{\plotpermgrid}[1]{ \foreach \i [count=\nn] in {#1} {\global\let\n\nn}; \plotgrid{\n}; \plotperm{#1}; } \newcommand{\plotpinsequence}[1]{ \absdot{(0,0)}{}; \edef\n{0} \edef\s{0} \edef\e{0} \edef\w{0} \edef\x{0} \edef\y{0} \foreach \pin [remember=\pin as \oldpin (initially 1), count=\i] in {#1} { \ifthenelse{\pin=1 \OR \pin=2}{ \ifthenelse{\oldpin=3}{ \xdef\x{\number\numexpr\e-1} }{ \xdef\x{\number\numexpr\w+1} } \ifnum\i=1 \pgfmathparse{\e+1} \xdef\e{\pgfmathresult} }{ \ifthenelse{\oldpin=1}{ \xdef\y{\number\numexpr\n-1} }{ \xdef\y{\number\numexpr\s+1} } \ifnum\i=1 \pgfmathparse{\s-1} \xdef\s{\pgfmathresult} } \ifnum\pin=1 \pgfmathparse{\n+2} \xdef\n{\pgfmathresult} \absdot{(\x,\n)}{}; \ifnum\i>1 \draw (\x,\n) -- (\x,\y-0.5); \else \draw[gray,very thick] (-0.5,-0.5) rectangle (\x+0.5,\n+0.5); \ifnum\pin=2 \pgfmathparse{\s-2} \xdef\s{\pgfmathresult} \absdot{(\x,\s)}{}; \ifnum\i>1 \draw (\x,\s) -- (\x,\y+0.5); \else \draw[gray,very thick] (-0.5,0.5) rectangle (\x+0.5,\s-0.5); \ifnum\pin=3 \pgfmathparse{\e+2} \xdef\e{\pgfmathresult} \absdot{(\e,\y)}{}; \ifnum\i>1 \draw (\e,\y) -- (\x-0.5,\y); \else \draw[gray,very thick] (-0.5,+0.5) rectangle (\e+0.5,\y-0.5); \ifnum\pin=4 \pgfmathparse{\w-2} \xdef\w{\pgfmathresult} \absdot{(\w,\y)}{}; \ifnum\i>1 \draw (\w,\y) -- (\x+0.5,\y); \else \draw[gray,very thick] (0.5,0.5) rectangle (\w-0.5,\y-0.5); }; } \tikzset{ on each segment/.style={ decorate, decoration={ show path construction, moveto code={}, lineto code={ \path [#1] (\tikzinputsegmentfirst) -- (\tikzinputsegmentlast); }, curveto code={ \path [#1] (\tikzinputsegmentfirst) .. controls (\tikzinputsegmentsupporta) and (\tikzinputsegmentsupportb) .. (\tikzinputsegmentlast); }, closepath code={ \path [#1] (\tikzinputsegmentfirst) -- (\tikzinputsegmentlast); }, }, }, mid arrow/.style={postaction={decorate,decoration={ markings, mark=at position .5 with {\arrow[#1]{stealth}} }}}, }
2412.04161v1
http://arxiv.org/abs/2412.04161v1
Geometrically constrained walls in three dimensions
\documentclass[a4paper,reqno]{amsart} \addtolength{\textwidth}{2cm} \addtolength{\textheight}{2cm} \addtolength{\voffset}{-.5cm} \mathsurround=1pt \usepackage{amsmath,amssymb,amsfonts,amsthm} \usepackage{mathtools} \usepackage{fancyhdr,fancyvrb} \usepackage{graphicx} \usepackage{nextpage} \usepackage[dvipsnames,usenames]{color} \usepackage[colorlinks=true, linkcolor=blue, citecolor=red]{hyperref} \usepackage{tikz} \usepackage{tikz-3dplot} \usepackage{mathrsfs} \makeatletter \@namedef{subjclassname@2020}{ \textup{2020} Mathematics Subject Classification} \makeatother \newcommand{\compresslist}{ \setlength{\itemsep}{1.5pt} \setlength{\parskip}{0pt} \setlength{\parsep}{0pt} } \setlength{\parskip}{1ex} \newcommand{\mygraphic}[1]{\includegraphics[height=#1]{env}} \newcommand{\myenv}{(\raisebox{0pt}{\mygraphic{.6em}})} \newcommand{\sfd}{\mathsf d} \newcommand{\ind}{\mathds{1}} \newcommand{\C}{\mathbb{C}} \newcommand{\M}[1]{\mathbb{M}^{#1}} \newcommand{\N}{\mathbb{N}} \newcommand{\R}[1]{\mathbb{R}^{#1}} \renewcommand{\S}[1]{\mathbb{S}^{#1}} \newcommand{\T}{\mathbb{T}} \newcommand{\Z}[1]{\mathbb{Z}^{#1}} \newcommand{\ba}{\mathbf{a}} \newcommand{\bb}{\mathbf{b}} \newcommand{\bc}{\mathbf{c}} \newcommand{\bfx}{\textbf{x}} \newcommand{\be}{\mathbf{e}} \newcommand{\bef}{\mathbf{f}} \newcommand{\bF}{\mathbf{F}} \newcommand{\bM}{\mathbf{M}} \newcommand{\bN}{\mathbf{N}} \newcommand{\bP}{\mathbf{P}} \newcommand{\bT}{\mathbf{T}} \newcommand{\bu}{\mathbf{u}} \newcommand{\bv}{\mathbf{v}} \newcommand{\bx}{\mathbf{x}} \newcommand{\by}{\mathbf{y}} \newcommand{\bz}{\mathbf{z}} \newcommand{\bY}{\mathbf{Y}} \newcommand{\bsigma}{\boldsymbol{\sigma}} \newcommand{\btau}{\boldsymbol{\tau}} \newcommand{\bpsi}{\boldsymbol{\psi}} \newcommand{\bomega}{\boldsymbol{\omega}} \newcommand{\fLie}{\mathfrak{Lie}} \newcommand{\fse}{\mathfrak{se}} \newcommand{\cA}{\mathcal A} \newcommand{\cB}{\mathcal B} \newcommand{\cC}{\mathcal C} \newcommand{\cD}{\mathcal D} \newcommand{\cE}{\mathcal E} \newcommand{\cF}{\mathcal F} \newcommand{\cG}{\mathcal G} \newcommand{\cH}{\mathcal H} \newcommand{\cI}{\mathcal I} \newcommand{\Int}[2]{\cI\left[#1\right](#2)} \newcommand{\cJ}{\mathcal J} \newcommand{\cK}{\mathcal K} \newcommand{\cL}{\mathcal L} \newcommand{\cM}{\mathcal M} \newcommand{\cO}{\mathcal O} \newcommand{\cP}{\mathcal P} \newcommand{\cR}{\mathcal R} \newcommand{\cS}{\mathcal S} \newcommand{\cT}{\mathcal T} \newcommand{\cU}{\mathcal U} \newcommand{\cV}{\mathcal V} \newcommand{\cW}{\mathcal W} \newcommand{\cX}{\mathcal X} \newcommand{\cY}{\mathcal Y} \newcommand{\cZ}{\mathcal Z} \newcommand\bigw{\scalebox{.95}[1]{$\bigwedge$}} \newcommand{\scrL}{\mathscr{L}} \newcommand{\scrR}{\mathscr{R}} \newcommand{\bbeta}{\boldsymbol\beta} \newcommand{\eeta}{\boldsymbol\eta} \newcommand{\fg}{\mathfrak{g}} \newcommand{\fh}{\mathfrak h} \newcommand{\punto}{\boldsymbol\cdot} \newcommand{\inte}[1]{\stackrel{\circ}{#1}} \newcommand{\BL}{\mathrm{BL}} \newcommand{\bulk}{\mathrm{bulk}} \newcommand{\rc}{\mathrm c} \newcommand{\rC}{\mathrm C} \newcommand{\col}{\mathrm{col}} \newcommand{\de}{\mathrm d} \newcommand{\destr}{\mathrm{destr}} \newcommand{\ext}{\mathrm{ext}} \newcommand{\rE}{\mathrm E} g}{\mathrm{fig}} n}{\mathrm{fin}} \newcommand{\form}{\mathrm{form}} \newcommand{\GL}[2]{\mathrm{GL}(#1,#2)} \newcommand{\init}{\mathrm{in}} \newcommand{\itr}{\mathrm{int}} \newcommand{\loc}{\mathrm{loc}} \newcommand{\opt}{\mathrm{opt}} \newcommand{\rr}{\mathrm{r}} \newcommand{\reg}{\mathrm{reg}} \newcommand{\rot}{\mathrm{rot}} \newcommand{\seq}{\mathrm{seq}} \newcommand{\sing}{\mathrm{sing}} \newcommand{\spann}{\operatorname{span}} \newcommand{\sh}{\mathrm{sh}} \newcommand{\SO}[1]{\mathrm{SO}(#1)} \newcommand{\surface}{\mathrm{surface}} \newcommand{\sym}{\mathrm{sym}} \newcommand{\tra}{\mathrm{tr}} \newcommand{\transl}{\mathrm{transl}} \newcommand{\cl}[1]{\overline{#1}} \newcommand{\ti}[1]{\widetilde{#1}} \newcommand{\eps}{\varepsilon} \newcommand{\rhobar}{\bar\rho} \newcommand{\Vrhob}{V_{\rhobar}} \newcommand{\norm}[1]{\left\| #1 \right\|} \newcommand{\norma}[1]{\left\lVert#1\right\rVert} \newcommand{\pdd}[2]{\frac{\partial #1}{\partial #2}} \newcommand{\scp}[2]{\left\langle#1,#2\right\rangle} \newcommand{\pro}[3]{\left(#1\left|\right.#2\right)_{#3}} \newcommand{\bracket}[1]{\left\langle#1\right\rangle} \newcommand{\TV}[2]{\norma{#1}\!(#2)} \newcommand{\abs}[1]{\left|#1\right|} \newcommand{\absb}[1]{\big|#1\big|} \newcommand{\Cl}[2]{C^{#1}_{#2}} \newcommand{\D}[1]{D^{#1}} \newcommand{\Hp}[2]{H^{#1}_{#2}} \newcommand{\Lp}[2]{L^{#1}_{#2}} \newcommand{\W}[3]{W^{#1,#2}_{#3}} \newcommand{\B}[2]{(#1,#2)} \newcommand{\ax}{\operatorname{ax}} \newcommand{\Ax}{\operatorname{Ax}} \newcommand{\ccc}{\subset\subset} \newcommand{\trace}{\operatorname{trace}} \newcommand{\cof}{\operatorname{cof}} \newcommand{\curv}{\operatorname{curv}} \newcommand{\Curv}{\operatorname{Curv}} \newcommand{\diam}{\operatorname{diam}} \newcommand{\dist}[2]{\operatorname{dist}(#1,#2)} \newcommand{\Id}{\operatorname{I}} \newcommand{\Lip}{\operatorname{Lip}} \newcommand{\per}{\operatorname{per}} \newcommand{\supp}{\operatorname{supp}} \newcommand{\spt}{\operatorname{spt}} \newcommand{\tr}{\operatorname{tr}} \newcommand{\sgn}{\operatorname{sgn}} \renewcommand{\O}{\Omega} \renewcommand{\o}{\omega} \newcommand{\e}{\varepsilon} \usepackage{soul} \newcommand{\llb}{\llbracket} \newcommand{\rrb}{\rrbracket} \DeclareMathOperator*{\curl}{curl} \DeclareMathOperator*{\Div}{Div} \DeclareMathOperator*{\supess}{ess\,sup} \DeclareMathOperator*{\ev}{ev} \DeclareMathOperator*{\argmin}{argmin} \newcommand{\QLT}{[0,L]{\times}[0,T]} \newcommand{\note}[1]{{\bf\textcolor[rgb]{1,0,0}{[#1]}}} \newcommand{\red}[1]{\textcolor[rgb]{1,0,0}{#1}} \newcommand{\gre}[1]{\textcolor[rgb]{.1,.6,.1}{#1}} \newcommand{\bla}[1]{\textcolor[rgb]{0,0,0}{#1}} \newcommand{\blu}[1]{\textcolor[rgb]{0,0,1}{#1}} \newcommand{\pur}[1]{\textcolor[rgb]{.75,0,.9}{#1}} \newcommand{\cho}[1]{\textcolor[rgb]{.82,.41,.2}{#1}} \newcommand{\ora}[1]{\textcolor[rgb]{1,.56,0}{#1}} \newcommand{\whi}[1]{\textcolor[rgb]{1,1,1}{#1}} \renewcommand{\div}{\operatorname{div}} \renewcommand{\geq}{\geqslant} \renewcommand{\leq}{\leqslant} \newcommand{\wto}{\rightharpoonup} \newcommand{\wsto}{\stackrel{*}{\rightharpoonup}} \newcommand{\average}{{\mathchoice {\kern1ex\vcenter{\hrule height.4pt width 8pt depth0pt} \kern-11pt} {\kern1ex\vcenter{\hrule height.4pt width 4.3pt depth0pt} \kern-7pt} {} {} }} \newcommand{\ave}{\average\int} \newcommand{\res}{\mathop{\hbox{\vrule height 7pt width .5pt depth 0pt\vrule height .5pt width 6pt depth0pt}}\nolimits} \newcommand{\wk}{\rightharpoonup} \renewcommand{\vec}[2]{\left(\begin{array}{c} #1 \\ #2 \end{array}\right)} \newcommand{\mat}[4]{\left[\begin{array}{cc} #1 & #2 \\ #3 & #4 \end{array}\right]} \newcommand{\matt}[9]{\left[\begin{array}{ccc} #1 & #2& #3 \\ #4 & #5& #6 \\ #7 & #8& #9 \end{array}\right]} \newcommand{\xitilde}{\tilde{\xi}} \mathchardef\emptyset="001F \newcommand{\MM}[1]{{\textcolor[rgb]{0,0,1}{[#1]}}} \newcommand{\RC}[1]{{\textcolor{ForestGreen}{[#1]}}} \newcommand{\GF}[1]{{\textcolor[rgb]{0,0.6,0.8}{[#1]}}} \renewcommand{\theenumi}{(\roman{enumi})} \renewcommand{\labelenumi}{\theenumi} \renewcommand{\arraystretch}{1.2} \numberwithin{equation}{section} \newtheorem{defin}{Definition}[section] \newtheorem{remark}[defin]{Remark} \newtheorem{theorem}[defin]{Theorem} \newtheorem{lemma}[defin]{Lemma} \newtheorem{proposition}[defin]{Proposition} \newtheorem{cor}[defin]{Corollary} \newtheorem{example}[defin]{Example} \title{Geometrically constrained walls in three dimensions} \author[R.~Cristoferi]{Riccardo Cristoferi} \address[R.~Cristoferi]{Department of Mathematics - IMAPP, Radboud University, Nijmegen, The Netherlands} \email{[email protected]} \author[G.~Fissore]{Gabriele Fissore} \address[G.~Fissore]{Department of Mathematics - IMAPP, Radboud University, Nijmegen, The Netherlands} \email{[email protected]} \author[M.~Morandotti]{Marco Morandotti} \address[M.~Morandotti]{Dipartimento di Scienze Matematiche ``G.~L.~Lagrange'', Politecnico di Torino, Corso Duca degli Abruzzi, 24, 10129 Torino, Italy} \email{[email protected]} \date{\today} \subjclass[2020]{} \keywords{} \usepackage{subfiles} \makeindex \raggedbottom \begin{document} \begin{abstract} We study geometrically constrained magnetic walls in a three dimensional geometry where two bulks are connected by a thin neck. Without imposing any symmetry assumption on the domain, we investigate the scaling of the energy as the size of the neck vanishes. We identify five significant scaling regimes, for all of which we characterize the energy scaling; in some cases, we are also able to identify the asymptotic behavior of the domain wall. Finally, we notice the emergence of sub-regimes that are not present int previous works due to restrictive symmetry assumptions. \end{abstract} \maketitle \section{Introduction} \label{sec:intro} \emph{Magnetic domain walls} are regions in which the magnetisation of a material changes from one value to another one. In the presence of extreme geometries, such as that of a dumbbell-shaped domain, the magnetic wall is more likely to be found in or around the neck; in this and similar geometry-driven situations, one usually speaks of \emph{geometrically constrained walls}, to stress the fact that the domain shape plays a pivotal role in the localisation of the transition region of the magnetisation, for instance, when prescribing it in the bulky parts of the dumbbell. The study of the behaviour of the magnetisation in a dumbbell-shaped domain is relevant in micro- and nano-electronics application, where the neck of the dumbbell models magnetic point contacts. If one imposes two different values of the magnetisation, one in each of the two macroscopic components, a transition is expected in the vicinity of the neck, as initially observed by Bruno in \cite{Bru99}: if the neck is small enough, so that the geometry of the material varies drastically when passing from one bulk to the other, it can play a crucial role in determining the location of the magnetic wall, by influencing the mere minimisation of the magnetic energy. Three scenarios are to be considered: the transition may happen either completely inside the neck, or partly inside and partly outside of the neck, or completely outside of the neck. The model features a sufficiently smooth potential which is minimal at the imposed values of the magnetisation in the bulk parts of the dumbbell, and a gradient term panalising transition; the two are competing as soon as the values of the magnetisation in the bulks are not the same. To model the geometries that are of interest in the applications, we will consider an infinitesimally small neck, whose size is determined by three parameters $\eps, \delta, \eta>0$: \begin{equation}\label{eq_neck_ep} N_{\eps}\coloneqq\{(x,y,z)\in\R{3}: |x|\leq \eps,|y|<\delta,|z|<\eta\}, \end{equation} with the understanding that all three of them vanish when $\eps\to0$, that is $\delta=\delta(\eps)\to0$ and $\eta=\eta(\eps)\to0$, as $\eps\to0$. The full domain is described by \begin{equation}\label{eq_domain_eps} \Omega_{\eps} \coloneqq \Omega_{\eps}^{\ell} \cup N_{\eps} \cup \Omega_{\eps}^{\rr}, \end{equation} where $\Omega_{\eps}^{\ell}=\Omega^{\ell}-(\eps,0,0)$ and $\Omega_{\eps}^{\rr}=\Omega^{\rr}+(\eps,0,0)$, for certain sets $\Omega^{\ell}\subset\{x<0\}$ and $\Omega^{\rr}\subset\{x>0\}$ such that $0\in\partial\Omega^{\ell}\cap\partial\Omega^{\rr}$. This geometry makes the $x$ direction the preferred one, whereas the $y$- and $z$-direction can be interchanged upon a change of coordinates; this motivates the fact that we will use, throughout the work, the subscript $\eps$ alone as an indication of the smallness of the neck. We are interested in minimising the non-convex energy $F_\eps\colon H^1(\Omega_\eps)\to[0,+\infty)$ defined by \begin{equation}\label{004} F_\eps(u) \coloneqq \frac{1}{2} \int_{\Omega_\eps} \abs{\nabla u(\textbf{x})}^2\,\de \textbf{x} + \int_{\Omega_\eps} W(u(\textbf{x}))\,\de \textbf{x}, \end{equation} where $W\colon \R{} \to [0,\infty)$ is a $C^1$ function such that $W^{-1}(0)=\{\alpha,\beta\}$ for some $\alpha<\beta$, and $\de \textbf{x} = \de x\de y \de z$. In \eqref{004}, the function $u$ represents a suitable quantity related to the magnetisation field defined on $\Omega_\eps$ and the potential $W$ favours the values $u(\textbf{x})=\alpha$ and $u(\textbf{x})=\beta$ for the magnetisation, corresponding to those to be imposed in the bulks. Here, the competition between the potential and the gradient terms is significantly influenced by the geometry of $\Omega_\eps$\,. \subsection{Related literature} The body of literature on this problem counts many physical contributions stemming from Bruno's work \cite{Bru99} and a few mathematical items, which we are going to briefly review to give a context to our novel results. In \cite{Bru99}, Bruno considers the special geometry of a thin ($0<h\ll1$) three-dimensional wall $\Omega=S\times(-h,h)$, where $S$ is a planar region with a dumbbell shape whose neck is located at the origin and the bulks are in $\{x<0\}$ and $\{x>0\}$. He assumes that the preferred directions of the magnetisation vector are $\mathbf{m}=(0,0,\pm1)$ and makes the Ansatz that it varies only in the $y$-$z$ plane, as a function of the $x$-coordinate, namely $$\mathbf{m}=(0,\cos\theta(x),\sin\theta(x)).$$ The energy that Bruno minimises is the one usually describing Bloch walls and turns out to be a functional of $\theta$ alone, with two emerging length scales when imposing that $\mathbf{m}\approx(0,0,\pm1)$ in $\{\pm x>0\}$: one driven by the shape $S$ of the domain, the other one dictated by the physical parameters entering the expression of the energy. Despite Bruno's insightful intuition, the special form of the magnetisation has some limitations, some of which were removed (for instance, by allowing $\mathbf{m}$ to vary also in the $x$-$z$ plane and considering fully three dimensional geometries) in \cite{MolOsiPon02}. Among the mathematical literature related to this topic, we mention \cite{ArrCar04, ArrCarLC06, ArrCarLC09a, ArrCarLC09b, CDLLMur08, HalVeg84, Jim84, Jim88, Jim04} as far as the PDE aspect is concerned, and \cite{CabFreMorPer01, CDLLMur04, CDLLMur04b, RubSchSte04} for variational approaches. Finally, we discuss two more recent contributions in detail, since the results we obtain are similar to them. In the work by Kohn \& Slastikov \cite{KohSla}, the problem is studied in the full three-dimensional setting, with the assumption that the geometry be axisymmetric: the dumbbell $\Omega_{\epsilon}$ is a rotation body around the $x$ axis, so that the shape parameters of the neck are essentially its length $\eps$ and its radius $\delta$. By taking advantage of a scale-invariant Poincar\'{e} inequality for Sobolev functions and by reducing the problem to a one-dimensional variational one, the authors proved the existence of three possible regimes, according to the value of the limit $\lim_{\eps\to0} \delta/\eps=\lambda\in[0,+\infty]$ and singled out a \emph{thin neck} regime ($\lambda=0$), a \emph{normal neck} regime ($\lambda\in(0,+\infty)$), and a \emph{thick neck} regime ($\lambda=+\infty$). In the first case the transition happens entirely inside the neck and is an affine function of the $x$-coordinate, in the second case the transition happens across the neck, partially inside and partially outside, depending on the value of~$\lambda$, whereas in the third case the transition happens entirely outside of the neck. These behaviours are found by studying the energy of particular competitors (essentially, an affine transition inside the neck and a harmonic transition in a spherical shell just outside of the neck) and then rescaling the minimiser in the vicinity of the neck. \smallskip In the works by Morini \& Slastikov \cite{MorSla12, MorSla15} the same problem was addressed in the case of magnetic thin film, that is when the domain has the shape of a dumbbell, but it is two-dimensional, that is, in Bruno's setting in the limit as $h\to0$. Mathematically speaking, the endeavor is more difficult on two accounts: the scale-invariant Poincar\'{e} inequality is not available in dimension two, and the problem loses its variational character. Methods that are typical from the study of PDE's were employed to construct suitable barriers to estimate the solutions. Moreover, due to the slow decay of the logarithm (the fundamental solution to Laplace's equation in two dimensions), sub-regimes became available in additions to the thin, regular, and thick neck regimes already analyzed by Kohn \& Slastikov: the sub-critical, critical, and super-critical thin neck regimes were found according to the value of the limit $\lim_{\eps\to0} (\delta|\ln\delta|)/\eps\in[0,+\infty]$, displaying a richer variety. In the case of sub-regimes, the rescaling of the minimisers to study their asymptotic behaviour is not trivial; nonetheless, the authors managed to characterise the profiles as the unique solutions to certain PDE's where the boundary conditions track the expected asymptotic behaviour. Both in Kohn \& Slastikov's and in Morini \& Slastikov's papers the technique involves two steps: (i) rescaling the neck to size of order $1$ to estimate the energy of the minimiser inside it (this is achieved by a simple change of variables) and (ii) rescaling the whole domain $\Omega_\eps$ to $\Omega_\infty$ in a way that either a variational problem or a PDE can be studied in $\Omega_\infty$, which brings to the attention that the boundary $\partial\Omega_\infty$ must be a set in which boundary conditions can be prescribed. \subsection{Novelty of the present contribution and main results} In this paper, we study the full three-dimensional case of the problem with no symmetry assumption on the shape of the neck: it will be a parallelepiped as in \eqref{eq_neck_ep} with all three dimensions independent from one another and all vanishing to zero as $\eps\to0$. When considering the mutual rate of convergence to zero of the three parameters $\eps,\delta,\eta$, we single out five regimes that do not emerge in the analysis of Kohn \& Slastikov, and for each of them we study the energy scaling. We notice that in our three-dimensional setting the scale-invariant Poincar\'{e} inequality is not always available. Therefore, only for those regimes for which it is available, we also investigate the behavior of local minimizers and the associated rescaled problem, which possesses a variational structure. As the roles of $\delta$ and $\eta$ can be interchanged, and noticing that the case $\delta\sim\eta$ corresponds, up to homomorphism, to having a cylindrical neck as in \cite{KohSla}, we always consider $\eta\ll\delta$ and we find are the following regimes: \begin{enumerate} \item[$(i)$] \textit{Super thin}: $\varepsilon\gg\delta\gg\eta$, \item[$(ii)$] \textit{Flat thin}: $\eps\approx\delta\gg\eta$, \item[$(iii)$] \textit{Window thick}: $\delta \gg\eta\gg \eps$, \item[$(iv)$] \textit{Narrow thick}: $\delta\gg\eps\approx\eta$, \item[$(v)$] \textit{Letter-box thick}: $\delta\gg\eps\gg\eta$. \end{enumerate} Due to the geometries that some of these regimes describe, in particular when $\eta\ll\delta$ so that the cross section of the junction of the neck with the bulks is a rectangle with a very large aspect ratio, we find an ellipsoidal competitor carrying less energy than the spherical one proposed in the previous works. As it depends on $\ln(\eta/\delta)$, the energy scaling turns out to exhibit sub-regimes in some of these cases, as described in Section~\ref{sec:heuristics}. The main achievements of the paper are the following. \begin{itemize} \item[(A1)] For all of the above mentioned regimes, we identify where the transition happens. More precisely, we find $\lambda_\varepsilon$, with $\lambda_\eps\to+\infty$ as $\eps\to 0$ such that \[ \lim_{\eps\to0} \lambda_\eps F_\eps(u_\eps) = e_0\in (0,+\infty), \] \[ \lim_{\eps\to 0} \lambda_\eps F_\eps(u_\eps,N_\eps) = e_0^N\in [0,e_0]. \] Their interpretation is the following: $e_0$ is the asymptotic energy in the whole domain, and $e_0^N$ that in the neck. Therefore, if $e_0^N=e_0$, we say that the transition happens entirely inside the neck, if $e_0^N=0$, the transition happens entirely outside of the neck, while if $0<e_0^N<e_0$, the transition happens both inside and outside of the neck. We prove that the transition happens entirely inside the neck in the regimes (i) and (ii) (see Theorem ), entirely outside of the regimes (iii) and (iv), and partly inside and partly outside in (v). Moreover, regime (v) is the richest, because it features three sub-regimes, due to the energetic convenience of the ellipsoidal competitor. \item[(A2)] In the regimes (i) and (ii) we fully characterize the asymptotic behavior of the domain wall: it is a harmonic function in the rescaled domain, and the transition from $\alpha$ to $\beta$ is affine inside the neck. \end{itemize} The plan of the paper is the following. In Section \ref{sec:results} we introduce the standing assumptions, present the heuristics for (A1), and introduce the competitors and compute the energy of each of them. In Section \ref{sec:analysis} tackle the regimes one by one, and provide the proofs of the main results. The study of the asymptotic behavior of the domain wall in the regimes (iii), (iv), and (v) will be discussed in a forthcoming paper. \section{Mathematical statement of the problem}\label{sec:results} We study a mathematical model to characterise magnetic domain walls in a three-dimensional dumbbell-shaped domain. The geometry of the domain can be presented as \begin{equation}\label{001} \Omega_\eps = \Omega_\eps^\ell \cup N_\eps \cup \Omega_\eps^\rr, \end{equation} where \begin{equation}\label{002} \Omega_\eps^\ell = \Omega^\ell + (-\eps,0,0) \qquad \text{and} \qquad \Omega_\eps^\rr = \Omega^\rr + (\eps,0,0), \end{equation} \[ \Omega^\ell\subset \{x<0\}, \qquad \Omega^\rr\subset \{x>0\} \] are two given bounded, connected, open subsets of $\R{3}$, and where \begin{equation}\label{003} N_\eps = [-\eps,\eps]\times(-\delta,\delta)\times(-\eta,\eta) \end{equation} is the \emph{neck} connecting the two bulks. We assume that $\Omega_\eps$ is a Lipschitz and that \begin{itemize}\compresslist \item the origin $(0,0,0)$ belongs to $\partial\Omega^\ell \cap \partial\Omega^\rr$; \item $\Omega^\ell \subset \{x<0\}$ and $\Omega^\rr \subset \{x>0\}$; \item there exists $r_0>0$ such that $(\partial\Omega^\ell) \cap B_{r_0}(0,0,0)$ and $(\partial\Omega^\rr) \cap B_{r_0}(0,0,0)$ are contained in the plane $\{x=0\}$, i.e., the bulks are flat and vertical near conjunction with the neck. \end{itemize} We start by giving the definition of isolated local minimizer for the functional $F_\eps$ introduced in \eqref{004}. \begin{defin}[isolated local minimizer for~$F_\eps$] Define $u_{0,\eps}:\Omega_\varepsilon\to\R{}$ as \begin{align*} u_{0,\eps}(\bfx)\coloneqq\begin{cases} \alpha\quad \text{ if } \bfx\in\Omega^\ell_\eps,\\[5pt] \displaystyle 0\quad\text{ if } \bfx\in N_\eps,\\[5pt] \beta\quad \text{ if } \bfx\in\Omega^\rr_\eps. \end{cases} \end{align*} we have that $u$ is an isolated local minimizer for $F_\eps$ if \begin{align}\label{eq:min_pb} F_\eps(u)\leq F_\eps(v), \end{align} for every $v\in B_\eps$, where \begin{equation}\label{Beps} B_\eps\coloneqq\{u\in H^1(\O_\eps):||u-u_{0,\eps}||_{L^2(\O_\eps)}<d,\, \|u\|_{L^\infty(\Omega_\eps)}<\infty \}, \end{equation} with $d<\min\{|\O^\ell|^{1/2},|\O^\rr|^{1/2}\}$. \end{defin} \subsection{Heuristics}\label{sec:heuristics} In this section, we show how to heuristically guess where the main part of the energy concentrates, just by looking at the asymptotic relationships between the three geometric parameters $\eps,\delta,\eta$. First of all, we note that, given the privileged role of the parameter~$\eps$, it is trivial to see that the roles of~$\delta$ and~$\eta$ can be interchanged upon switching the coordinate axes~$y$ and~$z$. The regimes investigated in \cite{KohSla} corresponds to the cases where $\delta\sim\eta$. Therefore, here we limit ourselves to consider the other regimes: The regimes we are going to study are: \begin{enumerate} \item[$(i)$] \textit{Super thin}: $\varepsilon\gg\delta\gg\eta$, \item[$(ii)$] \textit{Flat thin}: $\eps\approx\delta\gg\eta$, \item[$(iii)$] \textit{Window thick}: $\delta \gg\eta\gg \eps$, \item[$(iv)$] \textit{Narrow thick}: $\delta\gg\eps\approx\eta$, \item[$(v)$] \textit{Letter-box thick}: $\delta\gg\eps\gg\eta$. \end{enumerate} We now want to guess where the transition will happen: completely inside, completely outside, or in both regions. To understand this, we reason as follows. First of all, we expect the main part of the energy to be the Dirichlet integral. Therefore, we consider two harmonic functions that play the role of competitors for the minimization problem \eqref{eq:min_pb}: one where the transition from $\alpha$ to $\beta$ happens inside the neck, and the other where it happens only outside (and it is constant inside the neck). We then compare their energies (whose computations will be carried out in Section \ref{sec:competitors}) to get a guess of where the transition will happen. The first harmonic function will be referred to as the \emph{affine competitor}, and has energy of order \[ \text{Energy of the affine competitor } = \frac{\delta\eta}{\varepsilon}. \] The second harmonic function will be referred to as the \emph{elliptical competitor}, and has energy of order \[ \text{Energy of the elliptical competitor } = \frac{|\ln(\eta/\delta)|}{\delta}. \] When one of the two energies is dominant with respect to the other, it is clear where we expect the transition to happen. In the case they are of the same order, we guess that the transition is both inside and outside of the neck. This will be later confirmed by rigorous analysis. The comparison of the energies of the two harmonic competitors leads to the following heuristics: \begin{enumerate} \item[$(i)$] \textit{Super thin neck}: In this case we have \begin{align*} \frac{\delta\eta}{\eps} \frac{|\ln(\eta/\delta)|}{\delta}=\frac{\delta}{\eps}\Big(\frac{\eta}{\delta} \Big|\ln\Big(\frac{\eta}{\delta}\Big)\Big|\Big)\to 0 \end{align*} as $\eps\to0$. We then expect the transition to happen inside of $N_\eps$. \item[$(ii)$] \textit{Flat thin neck}: In that case we obtain \begin{align*} \frac{\delta\eta}{\eps} \frac{|\ln(\eta/\delta)|}{\delta}=\frac{\eta}{\delta}\Big|\ln\Big(\frac{\eta}{\delta}\Big)\Big|\to 0, \end{align*} thus the transition is occurring inside of $N_\eps$. \item[$(iii)$]\textit{Window thick neck}: The comparison of the energies of the harmonic competitors gives \begin{align*} \frac{\delta\eta}{\eps} \frac{|\ln(\eta/\delta)|}{\delta}=\frac{\eta}{\eps} \Big|\ln\Big(\frac{\eta}\delta\Big)\Big|\to+\infty, \end{align*} as $\varepsilon\to0$. The transition is expected to happen entirely outside of the neck \item[$(iv)$] \textit{Narrow thick neck}: In this case we have \begin{align*} \frac{\delta\eta}{\eps} \frac{|\ln(\eta/\delta)|}{\delta}=\Big|\ln\Big(\frac{\eta}\delta\Big)\Big|\to+\infty, \end{align*} as $\eps\to0$. Therefore, we expect the transition to happen outside. \item[$(v)$] \textit{Letter-box neck}: In this case we have the presence of sub-regimes. Indeed, the comparison of the orders of the energies of the harmonic competitors, gives \begin{align*} \frac{\delta\eta}{\eps} \frac{|\ln(\eta/\delta)|}{\delta}=\frac{\eta}{\eps}\Big|\ln\Big(\frac{\eta}{\delta}\Big)\Big|\approx\frac{\eta|\ln\eta|}{\eps}, \end{align*} as $\varepsilon\to0$, whose asymptotic is not clear. Therefore, we have to consider the following sub-regimes: \begin{enumerate} \item[$(a)$] \textit{Sub-critical letter-box neck}, when \begin{align*} \frac{\delta\eta}{\eps} \frac{|\ln(\eta/\delta)|}{\delta}=\frac{\eta|\ln\eta|}{\eps}\to 0, \end{align*} as $\eps\to0$. In such a case, the transition happens outside of the neck. \item[$(b)$] \textit{Critical letter-box neck}, when \begin{align*} \frac{\delta\eta}{\eps} \frac{|\ln(\eta/\delta)|}{\delta}=\frac{\eta|\ln\eta|}{\eps}\to \ell \in(0,\infty), \end{align*} as $\eps\to 0$. In this case the transition happens both inside and outside of the neck. \item[$(c)$] \textit{Super critical letter-box neck}, when \begin{align*} \frac{\delta\eta}{\eps} \frac{|\ln(\eta/\delta)|}{\delta}=\frac{\eta|\ln\eta|}{\eps}\to +\infty, \end{align*} as $\eps\to0$. Here the transition is expected to happen inside of the neck. \end{enumerate} \end{enumerate} \subsection{Competitors}\label{sec:competitors} The goal of this section is to build two harmonic competitors and to compute the order of their energies. For clarity, we present the affine and elliptic competitors separately. However, at the end of the section, they are mixed together in a more general way. \subsubsection{Affine competitor} Here we build the affine competitor inside the neck and we compute its energy. Let $A,B\in\R{}$, and define the affine function $\xi_\varepsilon \colon \R{3}\to\R{}$ as \begin{align}\label{affinecompetitor} \xi_\varepsilon(x,y,z)\coloneqq\begin{cases} A\quad &\Omega^\ell_\eps\,,\\[5pt] \displaystyle\frac{B-A}{2\eps}x+\frac{B+A}{2}\quad &N_\eps\,,\\[5pt] B\quad& \Omega^\rr_\eps\,. \end{cases} \end{align} Then, we have that \begin{align}\label{AffineBound} \int_{N_\varepsilon} |\nabla \xi_\varepsilon|^2 \de \mathbf{x} =\frac{\delta\eta}{\eps}(B-A)^2. \end{align} \subsubsection{Elliptic competitor} In \cite{KohSla}, the authors built an harmonic competitor by imposing boundary condition on \emph{half-spheres} centered at the edges of the neck. The choice of the spherical geometry was dictated by the fact that the authors required $\delta=\eta$. In our case, the geometry will be that of an ellipsoid, driven by the fact that one of the parameters $\delta$ and $\eta$ is larger than the other. In order to define our competitor, we first need to introduce the so called \emph{prolate spheroidal coordinates}. Consider the change of coordinates $(x,y,z) = \Psi(\mu, \nu, \varphi)$, given by \begin{equation}\label{EllipsHConstr} \begin{cases} x = a \sinh \mu \sin \nu \cos \varphi, \\ y = a \cosh \mu \cos \nu, \\ z = a \sinh \mu \sin \nu \sin \varphi, \\ \end{cases} \end{equation} where $(0, \pm a, 0)$ are the coordinates of the foci, $\varphi\in [0,2\pi]$ is the polar angle, $\nu\in[0,\pi]$ is the azimuthal angle and $a,\mu>0$. \\ Define the ellipsoid \begin{equation}\label{ellipsoid} E_M \coloneqq \{\Psi(\mu, \nu, \varphi): \mu < M \}. \end{equation} Moreover, we need consider the left and the right halves of the set $E_M$ translated at the edges of the neck. Namely, we consider the open sets \begin{equation*} E_{M,\eps}^\ell \coloneqq \{\Psi(\mu, \nu, \varphi): \mu < 2M, \, \varphi \in (\pi/2, 3\pi/2) \} - (\eps, 0, 0) \subset \Omega_\eps^\ell \end{equation*} and \begin{equation*} E_{M,\eps}^\rr \coloneqq \{\Psi(\mu, \nu, \varphi): \mu < 2M, \, \varphi \in (-\pi/2, \pi/2) \} + (\eps, 0, 0) \subset \Omega_\eps^\rr. \end{equation*} We are now in position to define our competitor $\xi_\eps:\R{3}\to\R{}$ as \begin{equation}\label{testfootball} \xi_\eps(x,y,z)\coloneqq \begin{cases} \alpha & \text{in $\Omega_\eps^\ell\setminus E_{M,\eps}^\ell$\,,} \\[5pt] \displaystyle \frac{\alpha+\beta}{2}-h(x+\eps,y,z) & \text{in $E_{M,\eps}^\ell$\,,} \\[5pt] \displaystyle \frac{\alpha+\beta}{2} & \text{in $N_\eps$\,,} \\[5pt] \displaystyle \frac{\alpha+\beta}{2}+h_\eps(x-\eps,y,z) & \text{in $E_{M,\eps}^\rr$\,,} \\[5pt] \beta & \text{in $\Omega_\eps^\rr\setminus E_{M,\eps}^\rr$\,,} \end{cases} \end{equation} where $h \colon E_{2M} \backslash E_{2m}\to\R{}$ is the solution of the problem \begin{equation*} \begin{cases} \Delta h=0 & \text{in } E_{2M} \backslash E_{2m}\,, \\[5pt] \displaystyle h = \frac{\beta-\alpha}{2} & \text{on } \partial E_{2M}\,, \\[5pt] h = 0 & \text{on } \partial E_{2m}\,. \end{cases} \end{equation*} Now, our goal is to find explicitly the function $h$ and to estimate, asymptotically, the Dirichlet energy. We look for the solution in the form $h = h(\mu)$. Then the Laplacian in prolate spheroidal coordinates is given by \begin{equation*} \Delta h = \frac{1}{a^2 (\sinh^2 \mu + \sin^2 \nu)} \left(h_{\mu \mu} + (\coth \mu) h_\mu \right) = 0 \end{equation*} or equivalently \begin{equation*} (\sinh \mu h_\mu)_\mu = 0. \end{equation*} It follows that \begin{equation*} h_\mu = \frac{c}{\sinh \mu}, \end{equation*} and thus \begin{equation}\label{hmu} h(\mu) = c \ln |k \tanh(\mu/2)|. \end{equation} Enforcing the boundary conditions \begin{equation*} h(2M) = \frac{\beta-\alpha}{2}\quad\text{and}\quad h(2m) = 0 \end{equation*} yields \begin{equation*} k = \coth m \qquad\text{and}\qquad c = \frac{\beta-\alpha}{2\ln\Big(\displaystyle\frac{\tanh M}{\tanh m}\Big)}. \end{equation*} Hence, we can write \begin{equation*} h(\mu) = \frac{(\beta-\alpha)}{2\ln\Big(\displaystyle\frac{\tanh M}{\tanh m}\Big)}\ln\Big(\displaystyle\frac{\tanh \mu/2}{\tanh m}\Big). \end{equation*} We are now in position to compute the Dirichlet energy of the function $\xi_\varepsilon$. Indeed, by using \eqref{hmu}, we obtain \begin{equation}\label{hUB3} \begin{split} \frac{1}{2} \int_{E_{2M}\setminus E_{2m}} |\nabla \xi_\eps|^2 \, \de \mathbf{x} =&\, \frac{1}{2} \int_{E_{2M}\setminus E_{2m}} |\nabla h|^2 \, \de \mathbf{x} = c^2 a \int_0^\pi \int_0^{2\pi} \int_{2m}^{2M} \frac{\sin \nu}{\sinh \mu} \,\de\nu \de \varphi \de \mu \\ =&\, \frac{\pi a(\beta-\alpha)^2}{\ln \left( \displaystyle\frac{\tanh M}{\tanh m} \right)}. \end{split} \end{equation} In the above expression, there are still two choices we can make: that of the parameter $a$, and of the parameter $m$. We now want to choose $a$ and $m$ in such a way that \begin{equation} \label{fit} (N_\eps \cap \{{x = \pm \eps}\})^\circ \subset \overline{E_{2m} \cap \{x = 0 \}}\pm(\eps,0,0). \end{equation} In order to have \eqref{fit} in force, we note that \eqref{EllipsHConstr} implies that, for all $(x,y,z) \in \partial E_{2m}$ such that $x =0$, it holds \begin{equation}\label{amconditionellipsoid} \frac{y^2}{a^2 \cosh^2 m}+\frac{z^2}{a^2 \sinh^2 m} = 1. \end{equation} Therefore, choosing $a$ and $m$ to satisfy \[ a \sinh m = \sqrt{2} \eta,\quad\quad\quad a \cosh m = \sqrt{2} \delta \] guarantees the validity of \eqref{fit}. We now want to get an asymptotic estimate of \eqref{hUB3}, taking in account the fact that all the regimes in this paper consider the case in which $\eta \ll \delta$. By definition $a^2=\delta^2-\eta^2$, and then $a\approx \delta$ as $\eps\to0$. Moreover \begin{equation*} \tanh m = \frac{\eta}{\delta}. \end{equation*} Observe that in our regiemes, when $\delta \gg \eta$, then $m \ll 1$ and \begin{equation} \ln \left( \frac{\tanh M}{\tanh m} \right) = \ln \tanh M - \ln \tanh m \approx -\ln \tanh m = -\frac{1}{2}\ln \frac{\eta}{\delta}, \end{equation} for $\eps$ small enough. This, together with \eqref{hUB3} implies \begin{equation}\label{footballenergy} \frac{1}{2} \int_{E_{2M}\backslash E_{2m}} |\nabla \xi_\eps|^2 \, \de x \de y \de z \approx \frac{\pi\delta(\beta-\alpha)^2}{|\ln \frac{\eta}{\delta}|}, \end{equation} for $\eps$ small enough. \\ Finally, we note that the elliptic competitor just built gives a better upper bound on the energy of the minimizer $u_\varepsilon$ than the one that could be obtained in \cite{KohSla}, with a spherical harmonic function. Indeed, in the spherical harmoic case, they obtained an upper estimate with a term of order $\delta$. Therefore, we can notice that \[ \frac{\delta}{|\ln(\eta/\delta)|}\ll\delta, \] as $\eps\to0$. Thus, we obtained a competitor whose order of energy is asymptotically lower than the previous one. This is particularly relevant since such a competitor follows the geometry of our problem, in which the shape of the neck presents the $y$ coordinate way smaller than the $z$ coordinate, ruled by $\delta$ and $\eta$ respectively. \subsubsection{Mixed competitor} The idea now, is to mix the affine competitor in the neck, together with the ellipsoidal just built. The purpose of this new competitor, is to describe whenever the transition happens simultaneously inside and outside of the neck. Recalling \eqref{testfootball} and \eqref{EllipsHConstr}, we define the function $\xi_\varepsilon \colon \R{3}\to\R{}$ as \begin{equation}\label{mixedcompetitor} \xi_\eps(x,y,z)\coloneqq \begin{cases} \alpha & \text{in $\Omega_\eps^\ell\setminus E_{M,\eps}^\ell$,} \\[5pt] h_\eps(x+\eps,y,z) & \text{in $E_{M,\eps}^\ell$,} \\[5pt] \displaystyle\frac{B-A}{2\eps}x+\frac{B+A}{2} & \text{in $N_\eps$,} \\[5pt] g_\eps(x-\eps,y,z) & \text{in $E_{M,\eps}^\rr$,} \\[5pt] \beta & \text{in $\Omega_\eps^\rr\setminus E_{M,\eps}^\rr$}, \end{cases} \end{equation} where $h$ is chosen be the solution to \begin{equation*} \begin{cases} \Delta w=0 & \text{in } E_{2M}^\ell \backslash E_{2m}^\ell, \\[5pt] w = \alpha & \text{on } \partial E_{2M}^\ell, \\[5pt] w = A & \text{on } \partial E_{2m}^\ell. \end{cases} \end{equation*} and $g$ solves \begin{equation*} \begin{cases} \Delta w=0 & \text{in } E_{2M}^r \backslash E_{2m}^r, \\[5pt] w= \beta & \text{on } \partial E_{2M}^\rr, \\[5pt] w = B & \text{on } \partial E_{2m}^\rr. \end{cases} \end{equation*} We now want to estimate, asymptotically, its energy. Using the same argument used to obtain \eqref{hmu}, we can write the explicit solution of the problems above as \begin{equation*} h(\mu) = c^\ell \ln |k^\ell \tanh(\mu/2)|,\quad\quad\quad\text{and}\quad g(\mu) = c^\rr \ln |k^\rr \tanh(\mu/2)|. \end{equation*} We can explicitly obtain $c^\ell,k^\ell,c^\rr, k^\rr\in\R{}$ by imposing the boundary conditions and we get \begin{align*} c^\ell=\frac{\alpha-A}{\ln\Big(\displaystyle\frac{\tanh M}{\tanh m}\Big)},\quad c^\rr=\frac{\beta-B}{\ln\Big(\displaystyle\frac{\tanh M}{\tanh m}\Big)} \end{align*} and \begin{align*} k^\ell=\frac{\exp\Big(\displaystyle\ln\abs{\frac{\tanh M}{\tanh m}}\frac{A}{\alpha-A}\Big)}{\tanh M}\,\quad k^\rr=\frac{\exp\Big(\displaystyle\ln\abs{\frac{\tanh M}{\tanh m}}\frac{B}{\beta-B}\Big)}{\tanh M}. \end{align*} Arguing like in \eqref{hUB3}, we get \begin{align}\label{BoundA} \frac{1}{2}\int_{E_{2M}^\ell\setminus E_{2m}^\ell}|\nabla h_\eps|^2=\frac{\pi a(A-\alpha)^2}{|\ln(\delta/\eta)|} \end{align} and \begin{align}\label{BoundB} \frac{1}{2}\int_{E_{2M}^\rr\setminus E_{2m}^\rr}|\nabla g_\eps|^2\ \de \textbf{x}= \frac{\pi a(B-\beta)^2}{|\ln(\delta/\eta)|}, \end{align} as $\eps\to0$. \\ Therefore, from \eqref{AffineBound}, \eqref{BoundA} and \eqref{BoundB}, we obtain \begin{align*} F(\xi_\eps)= \frac{\pi a}{|\ln(\delta/\eta)|}\big[(A-\alpha)^2+(B-\beta)^2\big]+(B-A)^2\frac{\delta \eta}{\eps}. \end{align*} If we notice that since $\eta\ll\delta$m we have $a \approx \delta$, we can write, for $\eps$ small enough \begin{align}\label{lowerbound} F(\xi_\eps)\approx \frac{\pi \delta}{|\ln(\delta/\eta)|}\big[(A-\alpha)^2+(B-\beta)^2\big]+(B-A)^2\frac{\delta \eta}{\eps}. \end{align} if we optimize \eqref{lowerbound} for $A$ and $B$ we get \begin{align}\label{optimalAandB} A=\frac{\displaystyle\frac{\pi\alpha}{\abs{\ln(\eta/\delta)}}+\frac{\eta}{\eps}(\alpha+\beta)}{\displaystyle\frac{\pi}{\abs{\ln(\eta/\delta)}}+\frac{2\eta}{\eps}}\quad\text{and}\quad B=\frac{\displaystyle\frac{\pi\beta}{\abs{\ln(\eta/\delta)}}+\frac{\eta}{\eps}(\alpha+\beta)}{\displaystyle\frac{\pi}{\abs{\ln(\eta/\delta)}}+\frac{2\eta}{\eps}}. \end{align} The choice of $A$ and $B$ in \eqref{optimalAandB} will be crucial in the various regimes when we will need to infer the boundary conditions of the rescaled profile at the edge of the neck. In conclusion, from \eqref{lowerbound}, if $u_\eps$ is a local minimizer, we then have \begin{align}\label{upperboundueps} F(u_\eps)\leq\frac{\pi\delta}{|\ln(\delta/\eta)|}\big[(A-\alpha)^2+(B-\beta)^2\big]+\frac{\delta \eta}{\eps}(B-A)^2. \end{align} Notice that the right-hand side of \eqref{upperboundueps} makes clear the energetic contribution of the competitor inside and outside of the neck, with their respective order of the energy. \section{Analysis of the problem in the several regimes} \label{sec:analysis} In this section we carry out the rigorous analysis of the asymptotic behavior of the solution, obtaining information on its energy and its behavior inside and close to the neck. \subsection{Super-thin neck} In this regime the parameters are ordered as $\eps \gg \delta \gg \eta$. Namely, we have \begin{align*} \lim_{\eps\to0} \frac{\delta}{\eps}=0\quad\text{and}\quad\lim_{\eps\to0}\frac{\eta}{\delta}=0. \end{align*} According to the heuristics in Section \ref{sec:heuristics}, we we expect the transition to happen entirely inside the neck. Thus, in order to understand the transition inside the neck, we rescale it to make it the set $N\coloneqq [-1,1]^3$. If $u_\eps$ is a local minimizer of the functional \eqref{004}, the only rescaling that works is \begin{align*} v_\eps(x,y,z)\coloneqq u_\eps(\eps x,\delta y, \eta z). \end{align*} If we rescale in this way, the limiting domain becomes $$\Omega_\infty = \Omega^\ell_\infty \cup N \cup \Omega^\rr_\infty,$$ where $\Omega^\ell_\infty = \{x<-1\}$ and $\Omega^\rr_\infty = \{x>1\}$. \begin{theorem}\label{Theoremsuperthin} Assume $\eps \gg \delta \gg \eta$. Then, \begin{align*} \lim_{\eps\to0} \frac{\eps}{\delta\eta}F_\eps(u_\eps)=\lim_{\eps\to 0} \frac{\eps}{\delta\eta}F_\eps(u_\eps,N_\eps)= (\beta-\alpha)^2, \end{align*} which means that the main contribution of the energy is given by the transition inside the neck. Moreover, for $\varepsilon>0$ let $v_\varepsilon \colon [-1,1]^3\to\R{}$ be defined as \[ v_\eps(x,y,z)\coloneqq u_\eps(\eps x,\delta y, \eta z). \] Then, $v_\eps$ converges to a function $\hat{v}\in H^1(N)$, of the form $\hat{v}(x,y,z)=v(x)$, where $v\in H^1(-1,1)$ is the unique minimizer of the variational problem \begin{align*} \min\bigg\{ \frac{1}{2} \int_{-1}^1 |v'|^2\ \de x:\ v\in H^1(-1,1),\ v(-1)=\alpha,\ v(1)=\beta\bigg\}. \end{align*} In particular, $v(x)=\dfrac{\beta-\alpha}{2}x+\dfrac{\alpha+\beta}{2}$. \end{theorem} \begin{proof} \emph{Step 1: limit of $v_\varepsilon$.} We claim that \[ \sup_{\varepsilon>0} \|v_\varepsilon\|_{H^1(N)} < \infty. \] First of all, using the fact that $\eps \gg \delta \gg \eta$, we get that \begin{align}\label{eq:est_grad_v_eps} \|\nabla v_\varepsilon\|^2_{L^2(N)} &=\frac{\varepsilon}{2\delta\eta} \int_N \bigg((\partial_x u_\eps)^2 + \frac{\delta^2}{\eps^2}(\partial_y u_\eps)^2 + \frac{\eta^2}{\eps^2}(\partial_z u_\eps)^2\bigg)\,\de\textbf{x} \leq\frac{\varepsilon}{\delta\eta} F_\varepsilon(u_\varepsilon) \leq C <\infty, \end{align} where the first inequality follows by using a change of variable, while the second one from \eqref{AffineBound} with $A=\alpha$ and $B=\beta$. Moreover, from \[ \int_{N} |v_\eps|^2\,\de \mathbf{x}=\frac{1}{\eps\delta\eta}\int_{N_\eps} |u_\eps|^2\,\de\mathbf{x}\leq \frac{\big(\sup_{\mathbf{x}\in N_\eps}|u_\eps|^2\big) |N_\eps|}{\eps\delta\eta}=C. \] Thus, we get that, up to a subsequence, $v_\varepsilon$ converges weakly in $H^1(N)$ to a function $\hat{v}\in H^1(N)$. The independence of the subsequence will follow from Step $2$, where we show that the limit is the \emph{unique} solution to a variational problem. \emph{Step 2: limiting problem} We now want to characterize the function $v$ as the unique solution to a variational problem. We do this in two steps: first, we identify a functional that will be minimized, and then we identify the boundary conditions.\\ We have that \begin{align}\label{eq:est_energy_stn} \liminf_{\eps\to0}\frac{\eps}{\delta\eta} F(u_\eps)&\geq \liminf_{\eps\to0}\frac{\eps}{2\delta\eta}\Big(\int_{N_\eps}|\nabla u_\eps|^2\de \textbf{x} +\int_{N_\eps} W(u_\eps)\ \de\bfx \Big)\nonumber \\[5pt] &=\liminf_{\eps\to0}\frac{1}{2}\int_{N} \bigg((\partial_x v_\eps)^2 + \frac{\eps^2}{\delta^2}(\partial_y v_\eps)^2 + \frac{\eps^2}{\eta^2}(\partial_z v_\eps)^2\bigg)\,\de \textbf{x} +\eps^2\int_N W(v_\eps)\, \de\bfx \nonumber \\[5pt] &\geq\liminf_{\eps\to0}\frac{1}{2}\int_N |\nabla v_\eps|^2 \de \textbf{x} \geq \frac{1}{2}\int_N |\nabla v|^2\, \de \textbf{x} \end{align} Notice that, from the bound \eqref{eq:est_grad_v_eps} and the fact that $\eps/\eta\to\infty$ and $\eps/\delta\to\infty$, we necessarily have that $v$ does not depend on $y$ and $z$. Namely, $v_\eps$ converges to a function $\hat{v}\in H^1(N)$, of the form $\hat{v}(x,y,z)=v(x)$, where $v\in H^1(-1,1)$. Therefore, from \eqref{eq:est_energy_stn}, we can write \begin{align}\label{eq:est_energy_stnpt2} \frac{1}{2}\int_N |\nabla v|^2 \de \textbf{x} = & 2\int_{-1}^1 (\hat{v}')^2\ \de x \geq2\min\Big\{ \int_{-1}^1 (w')^2\ \de x:\ w\in H^1(-1,1),\ w(\pm1)=\hat{v}(\pm1)\Big\} \nonumber \\[5pt] &=(\hat{v}(-1)-\hat{v}(1)\big)^2, \end{align} where last step follows by an explicit minimization. Now we claim that $\hat{v}(-1)=\alpha$ and $\hat{v}(1)=\beta$. We prove the former, since the latter follows by using a similar argument. The idea (introduced in \cite{KohSla}) is to use the scale-invariant Poincaré inequality \begin{align}\label{poincare} \Big(\int_{\Omega_\eps^\ell}|u_\eps-\bar{u}_\eps|^6\ \de \textbf{x}\Big)^{\frac{1}{6}}\leq C \Big(\int_{\Omega_\eps^\ell}|\nabla u_\eps|^2\ \de \textbf{x}\Big)^\frac{1}{2}, \end{align} where $C>0$ and $\bar{u}_\eps$ is the average on $\Omega_\eps^\ell$ of $u_\eps$. Using a change of variable, we estimate (by neglecting the potential term as in \eqref{eq:est_energy_stnpt2}) the right-hand side as \begin{align*} \eps\delta\eta\int_{\Omega_\eps^\ell}|v_\eps-\bar{v}_\eps|^6\ \de \textbf{x} \leq C \Big(\frac{\delta\eta}{\eps} \int_{\Omega_\eps^\ell}(\partial_x v_\eps)^2 +\frac{\eps^2}{\delta^2}(\partial_y v_\eps)^2 +\frac{\eps^2}{\eta^2}(\partial_z v_\eps)^2\ \de \textbf{x} \Big)^3 = C \Big(\frac{\delta\eta}{\eps}\Bigr)^3 \left(\frac{\eps}{\delta\eta}F_\eps(u_\eps)\right)^3, \end{align*} Thus, \begin{align}\label{poincareestimate} \int_{\Omega_\eps^\ell}|v_\eps-\bar{v}_\eps|^6\ \de \textbf{x}\leq C \Big(\frac{\delta\eta}{\eps^2}\Big)^3\left(\frac{\eps}{\delta\eta}F_\eps(u_\eps)\right)^3\to0, \end{align} as $\varepsilon\to0$. Indeed, by assumption $\delta\eta/\eps^2\to0$ as $\eps\to0$ and, thank to \eqref{eq:est_grad_v_eps}, we see that the term $\frac{\eps}{\delta\eta}F_\eps(u_\eps)$ is uniformly bounded in $\varepsilon$ . Therefore, $v_\varepsilon\to \bar{v}_\eps$ in $L^6(\Omega_\eps^\ell)$. We now claim that $\bar{v}_\eps=\alpha$. Indeed, by definition of local minimizer we have that \[ ||u_\eps-\alpha||_{L^1(\Omega_\eps^\ell)}\to0 \] as $\eps\to0$, giving that also the average of $u_\eps$ on $\Omega_\eps^\ell$ tends to $\alpha$ in $L^1$. Therefore, in the rescaled profile $v_\eps$ we also get that $\bar{v}_\eps\to \alpha$ as $\eps\to 0$. This gives that $\hat{v}(-1)=\alpha$. The reasoning for $\O^\rr_\eps$ is analogous, so we get that $\hat{v}(1)=\beta$. \emph{Step 3: asymptotic behavior of the energy.} From \eqref{eq:est_energy_stnpt2} we can conclude that \begin{align}\label{liminfsuperthin} \liminf_{\eps\to0}\frac{\eps}{\delta\eta} F(u_\eps)\geq (\beta-\alpha)^2. \end{align} On the other hand, if $w_\eps$ is the affine competitor in \eqref{affinecompetitor}, we have that \begin{align}\label{limsupsupethin} \limsup_{\eps\to 0}\frac{\eps}{\delta\eta} F(u_\eps)\leq\limsup_{\eps\to 0} \frac{\eps}{\delta\eta}F(w_\eps)=(\beta-\alpha)^2. \end{align} Finally, from \eqref{upperboundueps}, \eqref{liminfsuperthin}, and \eqref{limsupsupethin}, we get \begin{align*} \lim_{\eps\to0} \frac{\eps}{\delta \eta} F(u_\eps)=\lim_{\eps\to 0} \frac{\eps}{\delta \eta} F(u_\eps,N_\eps)=(\beta-\alpha)^2. \end{align*} This concludes the proof. \end{proof} \subsection{Flat-thin neck} In this regime the parameters are ordered as $\eps \approx\delta \gg \eta$. Namely, we have \begin{align*} \lim_{\eps\to0} \frac{\delta}{\eps}=m\in(0,+\infty)\quad\text{and}\quad \lim_{\eps\to 0}\frac{\eta}{\eps}=0. \end{align*} This case is similar to the super-thin regime and techniques used are similar. Therefore, we only highlight the main differences with theorem \ref{Theoremsuperthin}. Since we expect the transition to happen entirely inside the neck, we would like to use a rescaling for which the neck $N_\eps$ transforms in $N\coloneqq [-1,1]^3$. If $u_\eps$ is a local minimizer of the functional \eqref{004}, the only rescaling that works is \begin{align*} v_\eps(x,y,z)\coloneqq u_\eps(\eps x,\eps y, \eta z). \end{align*} If we rescale in this way, the limiting domain becomes $$\Omega_\infty = \Omega^\ell_\infty \cup N \cup \Omega^\rr_\infty,$$ where $\Omega^\ell_\infty = \{x<-1\}$ and $\Omega^\rr_\infty = \{x>1\}$. \begin{theorem} Assume $\eps \approx \delta \gg \eta$. Then, \begin{align*} \lim_{\eps\to0} \frac{1}{\eta}F_\eps(u_\eps)=\lim_{\eps\to 0} \frac{1}{\eta}F_\eps(u_\eps,N_\eps)= (\beta-\alpha)^2, \end{align*} which means that the main contribution of the energy is given by the transition inside the neck. Moreover, for $\varepsilon>0$ let $v_\varepsilon:[-1,1]^3\to\R{}$ be defined as \[ v_\eps(x,y,z)\coloneqq u_\eps(\eps x,\eps y, \eta z). \] Then, $v_\eps$ converges to a function $\hat{v}\in H^1(N)$, of the form $\hat{v}(x,y,z)=v(x)$, where $v\in H^1(-1,1)$ is the unique minimizer of the variational problem \begin{align*} \min\bigg\{ \frac{1}{2} \int_{-1}^1 |v'|^2\ \de x:\ v\in H^1(-1,1),\ v(-1)=\alpha,\ v(1)=\beta\bigg\}. \end{align*} In particular, $v(x)=\dfrac{\beta-\alpha}{2}x+\dfrac{\alpha+\beta}{2}$. \end{theorem} \begin{proof} In the same way as in Theorem \ref{Theoremsuperthin}, we can prove that \begin{align}\label{boundFflatthin} \|\nabla v_\eps\|^2_{L^2(N)}\leq \frac{1}{\eta}F_\eps (u_\eps)\leq C<\infty \end{align} and $||v_\eps||_{L^2(N)}<C$ uniformly in $\varepsilon$. Therefore, by compactness there is $v\in H^1(N)$ such that, up to a subsequence, $v_\eps\wto v$ in $H^1(N)$. The independence of the subsequence will follow from the fact that the limit is the \emph{unique} solution to a variational problem. \\ Thus, we can write \begin{align}\label{liminfflatthin} \nonumber \liminf_{\eps\to0}\frac{1}{\eta} F(u_\eps)&\geq \frac{1}{2} \int_{\O_\infty} \bigg((\partial_x v_\eps)^2 + (\partial_y v_\eps)^2 + \frac{\eps^2}{\eta^2}(\partial_z v_\eps)^2\bigg)\,\de\mathbf{x}+\eps^2\eta\int_{\O_\infty} W(v_\eps)\,\de\mathbf{x}\\[5pt] \nonumber&\geq\int_{-1}^1 (\partial_x\hat{v}')^2+(\partial_y \hat{v})^2\ \de x\de y\\[5pt] \nonumber&\geq\min\bigg\{\int_{[-1,1]^2} (\partial_x w)^2+(\partial_y w)^2\ \de x\de y:\ w\in H^1\big([-1,1]^2\big),\\[5pt] & \phantom{\geq\min \bigg\{\;} w(\pm1,y)=\hat{v}(\pm1,y),\ \forall y\in[-1,1]\bigg\}, \end{align} Where in the previous to last step we used \eqref{boundFflatthin} and the fact that $\eps/\eta\to\infty$. Then we necessarily have that $v$ does not depend on $z$. Namely, $v_\eps$ converges to a function $\hat{v}\in H^1(N)$, of the form $\hat{v}(x,y,z)=v(x,y)$, where $v\in H^1([-1,1]^2)$. \\ Now, would like to show that the boundary conditions $\hat{v}(\pm1,y)$ are independent from $y$. This is done by acting similarly as in \eqref{poincare} and \eqref{poincareestimate}. Indeed by using the scale-invariant Poincarè inequality, we get\\ \begin{align}\label{poincareestimate2} \int_{\Omega_\eps^\ell}|v_\eps-\bar{v}_\eps|^6\ \de \textbf{x}\leq C \Big(\frac{\eta}{\eps}\Big)^2\Big(\frac{1}{\eta}F_\eps(v_\eps)\Big)^3. \end{align} From \eqref{boundFflatthin} and $\eta/\eps\to0$ as $\eps\to0$ we can conclude that $v=\alpha$ on $\Omega^\ell_\infty$ and $v=\beta$ on $\Omega^r_\infty$ independently on $y$. Therefore, from \eqref{liminfflatthin} we get \begin{align*} \liminf_{\eps\to0}\frac{1}{\eta} F(u_\eps,N_\eps) &\geq\min\bigg\{ \int_{[-1,1]^2} (\partial_x w)^2+(\partial_y w)^2\ \de x\de y:\ w\in H^1\big([-1,1]^2\big),\\[5pt] &\phantom{\geq\min\bigg\{} w(-1,y)=\alpha,\ w(1,y)=\beta,\ \forall y\in[-1,1]\bigg\}\\[5pt] &\geq\min\bigg\{ \int_{[-1,1]^2} (\partial_x w)^2\ \de x\de y:\ w\in H^1\big([-1,1]^2\big),\\[5pt] &\phantom{\geq\min\bigg\{} w(-1,y)=\alpha,\ w(1,y)=\beta,\ \forall y\in[-1,1]\bigg\}\\[5pt] &=(\beta-\alpha)^2 \end{align*} On the other hand, if $w_\eps$ is the affine competitor in \eqref{affinecompetitor}, we have that \begin{align}\label{limsupsupethin} \limsup_{\eps\to 0}\frac{1}{\eta} F(u_\eps)\leq\limsup_{\eps\to 0} \frac{1}{\eta}F(w_\eps)\leq(\beta-\alpha)^2. \end{align} Finally, by putting all together and considering \eqref{upperboundueps}, we get \begin{align*} \lim_{\eps\to0} \frac{1}{ \eta} F(u_\eps)=\lim_{\eps\to 0} \frac{1}{ \eta} F(u_\eps,N_\eps)=(\beta-\alpha)^2. \end{align*} This concludes the proof. \end{proof} \subsection{Window thick and narrow thick regimes} Here we consider the scaling of the energy in two regimes: the window thick, where \begin{align*} \lim_{\eps\to0}\frac{\eta}{\delta}=0\quad\text{and}\quad\lim_{\eps\to0} \frac{\eta}{\eps}=+\infty, \end{align*} and the narrow thick, where \begin{align*} \lim_{\eps\to0}\frac{\eta}{\delta}=0\quad\text{and}\quad\lim_{\eps\to0} \frac{\eta}{\eps}=l\in(0,\infty), \end{align*} Since the ellipsoidal competitor outside of the neck provides an energy whose order is lower than the energy of the affine competitor in the neck, we expect the transition happening outside of the neck. Therefore, among all the possible rescalings, we choose to preserve a \textit{window} in the $yz$-plane. More generally, we would like to get a non-trivial domain that connects the two bulks. This is possible if we rescale by $(\delta,\delta,\eta)$ in the window thick regime and by $(\eta,\delta,\eta)$ in the narrow thick one. \begin{theorem}\label{Theoremwindowthick} In the window thick or in the narrow thick regimes, it holds that \begin{equation*} \lim_{\eps\to0} \frac{|\ln(\eta/\delta)|}{\delta}F_\eps(u_\eps)=\lim_{\eps\to 0} \frac{|\ln(\eta/\delta)|}{\delta}F_\eps(u_\eps,\O_\eps\setminus N_\eps)= \pi(\beta-\alpha)^2. \end{equation*} which translates into the fact that the transition happens entirely outside of the neck. \end{theorem} \begin{proof} \emph{Step 1. Window thick regime.} We can find an energy bound by using the competitor built in \eqref{testfootball}. This, together with \eqref{footballenergy}, we get that \begin{align*} \frac{|\ln(\eta/\delta)|}{\delta}F_\eps(u_\eps)\leq \frac{|\ln(\eta/\delta)|}{\delta}F_\eps(\xi_\eps)\leq \pi(\beta-\alpha)^2+C_\eps, \end{align*} for some $C_\eps\geq0$ which is going to $0$ as $\eps\to 0$. We choose the rescaling \begin{align*} \hat{u}_\eps(x,y,z)\coloneqq u_\eps(\delta x,\delta y ,\eta z). \end{align*} In such a way the limiting domain is \begin{align*} \Omega_\infty\coloneqq \{x<0\}\cup\Big[\{0\}\times [-1,1]\times [-1,1]\Big]\cup \{x>0\}. \end{align*} First, we notice that since $u_\eps$ is a critical point of the energy $F_\eps$, we have that $u_\eps$ satisfies the Euler-Lagrange equation \begin{align*} \int_{\Omega_\eps}\nabla u_\eps \cdot \nabla\varphi\ \de\textbf{x}-\int_{\Omega_\eps} W'(u_\eps)\varphi\ \de\textbf{x}=0. \end{align*} For every $\varphi\in H^1(\Omega_\eps)$ Therefore, we get \begin{align}\label{nablatozero} \int_{\Omega_\eps}\abs{\nabla u_\eps}^2\ \de\textbf{x}=\int_{\Omega_\eps} W'(u_\eps)u_\eps\ \de \textbf{x}\to 0, \end{align} since $u_\eps$ is uniformly bounded, and $\chi_{\Omega_\eps} W'(u_\eps)\to 0$ in $L^p(\Omega)$, for every $p\geq 1$, as $\eps\to0$. Therefore, if we set $\hat{u}_\eps(x,y,z)\coloneqq u_\eps(\delta x,\delta y ,\eta z)$, we have that $\hat{u}_\eps$ satisfies weakly \begin{align*} \begin{cases} \Delta \hat{u}_\eps=\delta^2\eta W'(\hat{u}_\eps)\quad&\text{in}\ \Omega_\infty\\[5pt] \displaystyle\frac{\partial \hat{u}_\eps}{\partial \nu}=0\quad&\text{on}\ \partial\Omega_\infty. \end{cases} \end{align*} We would like to apply \cite[Theorem 6.2]{MorSla12}. Since $\delta^2\eta \chi_{\Omega_\eps}W'(\hat{u}_\eps)\to0$ as $\eps\to0$ in $L^p(\Omega)$, for every $p\geq 1$. Therefore, up to a subsequence, there is $u^\ast$, weak solution of the problem \begin{align*} \begin{cases} \Delta u^\ast=0\quad&\text{in}\ \Omega_\infty\\[5pt] \displaystyle\frac{\partial u^\ast}{\partial \nu}=0\quad&\text{on}\ \partial\Omega_\infty, \end{cases} \end{align*} for which $\hat{u}_\eps \to u^\ast$ in $W^{2,p}_{\text{loc}}(\Omega_\infty)$, as $\eps\to0$. It turns out that $u^\ast$ is locally constant. By the Sobolev embedding, we can also state that such a convergence is locally uniform. Consider the prolate ellipsoidal coordinates $\Psi$ defined in \eqref{EllipsHConstr}. Let $M>0$, that will be chosen later, but large enough such that \begin{align}\label{chiuderelafinestra} \Big[\{0\}\times [-1,1]\times [-1,1]\Big]\subset E_M\cap\{x=0\} \end{align} Consider the ellipsoid \begin{align*} E_M\coloneqq\{\Psi(\mu,\nu,\varphi):\mu<2M\}, \end{align*} and the sphere centered at the origin $B_M\coloneqq B_M(0,0,0)$ such that $E_M\subset B_M$. By moving into prolate elliptical coordinates, from the local uniform convergence of $\hat{u}_\eps$ we have \begin{align*} \hat{u}_\eps\big(\Psi(\mu,\nu,\varphi)\big)\to m\quad\text{locally uniformly in}\ B_M\cap \Omega_\infty. \end{align*} In particular, as $\partial B_M\cap \Omega_\infty$ is compact, we get \begin{align*} \hat{u}_\eps\big(\Psi(\mu,\nu,\varphi)\big)\to m\quad\text{ uniformly in}\ \partial B_M\cap \Omega_\infty. \end{align*} From the above fact, we infer that for every $\eps>0$ there is $\gamma>0$ such that \begin{align}\label{firstboundarycondition} m-\gamma < u_\eps\big(\Psi(\mu,\nu,\varphi)\big)< m+\gamma\quad\text{on } \partial B_{M_{\delta\eta}}, \end{align} where $B_{M_{\delta\eta}}$ is the rescaled sphere obtained from $B_M$. The convergence \eqref{firstboundarycondition} is also true on $\partial E_{_{M_{\delta\eta}}}\subset B_{M_{\delta\eta}}$, where $E_{_{M_{\delta\eta}}}$ is the rescaled ellipsoid obtained from $E_M$. \\ Consider $r_0>0$ for which $\partial \Omega_\infty\cap E_{r_0}$is a vertical ellipses. Let $0<\rho<r_0$ and define \begin{align*} E_{\eps,\rho}^\ell &\coloneqq E_\rho\cap \{x<-\eps\}\\[5pt] E_{\eps,\rho}^r&\coloneqq E_\rho\cap \{x>\eps\}. \end{align*} By hypothesis and \eqref{nablatozero}, we have that \begin{align*} \gamma_\eps^\ell&\coloneqq ||u_\eps-\alpha||_{L^\infty(E_{\eps,\rho}^\ell )}\to0,\\[5pt] \gamma_\eps^r&\coloneqq ||u_\eps-\beta||_{L^\infty( E_{\eps,\rho}^r )}\to0, \end{align*} as $\eps\to0$. From the above fact we infer that, \begin{align}\label{secondboundarycondition} \nonumber\alpha-\gamma_\eps^\ell<&u_\eps<\alpha+\gamma_\eps^\ell\quad\text{on }\partial E_{\eps,\rho}^\ell, \\[5pt] \beta-\gamma_\eps^\rr<&u_\eps<\beta+\gamma_\eps^\rr\quad\text{on }\partial E_{\eps,\rho}^\rr. \end{align} Notice that $E_{M_{\delta\eta}}$ and $E_\rho$ are two concentric ellipsoids. Therefore, from \eqref{firstboundarycondition} and \eqref{secondboundarycondition} we can write \begin{align*} \frac{1}{2}\int_{E_{\eps,\rho}^\ell \setminus E_{M_{\delta\eta}}}&\abs{\nabla u}^2\de\textbf{x}\\[5pt] \geq\min\Big\{ \frac{1}{2}\int_{E_{\eps,\rho}^\ell \setminus E_{M_{\delta\eta}}}&\abs{\nabla u_\eps}^2\de\textbf{x}: u=m+\gamma\text{ on } \partial E_{M_{\delta\eta}},\ u=\alpha-\gamma_\eps^\ell\text{ on }\partial E_{\eps,\rho}^\ell \Big\}. \end{align*} By doing similar computations as for \eqref{hUB3}, we get the unique minimizer, in ellipsoidal coordinates, given by \begin{align*} u(\mu)=c \ln\abs{k \tanh(\mu/2)}, \end{align*} where \begin{align*} c=\frac{m+\gamma-\alpha+\gamma_\eps^\ell}{\ln\Big(\displaystyle\frac{\tanh(\rho)}{\tanh(M_{\delta\eta})}\Big)}. \end{align*} By computing its Dirichlet energy, we get \begin{align}\label{loweboundwithgamma} \frac{1}{2a}\ln\Big(\displaystyle\frac{\tanh(\rho)}{\tanh(M_{\delta\eta})}\Big)\int_{E_{\eps,\rho}^\ell \setminus E_{M_{\delta\eta}} }&\abs{\nabla u_\eps}^2\de\textbf{x}\geq \frac{\pi}{2} (m+\gamma-\alpha+\gamma_\eps^\ell)^2. \end{align} Now, we would like to undestand the asymptotic behaviour of the coefficient in front of the left-hand side of \eqref{loweboundwithgamma}. Notice that on $ \partial E_{M_{\delta\eta}}\cap \{x =0\}$ we have \begin{equation}\label{amconditionellipsoid} \frac{y^2}{a^2 \cosh^2 M_{\delta\eta}}+\frac{z^2}{a^2 \sinh^2 M_{\delta\eta}} = 1. \end{equation} We can choose $M_{\delta\eta}$ to satisfy $a \sinh M_{\delta\eta} = \sqrt{2} \eta$ and $a \cosh M_{\delta\eta} = \sqrt{2} \delta$. In such a way, we have that \eqref{chiuderelafinestra} holds. Therefore, \begin{equation*} \tanh M_{\delta\eta} = \frac{\eta}{\delta}. \end{equation*} In particular, $a^2=\delta^2-\eta^2\approx \delta^2$, therefore we can choose $a\approx4\delta$ as the eccentricity of the change of the coordinates. As consequence, we get the following asymptotic estimate \begin{align}\label{lowerboundapprox} \frac{1}{8a}\ln\Big(\displaystyle\frac{\tanh(\rho)}{\tanh(M_{\delta\eta})}\Big)=\frac{1}{8a}\big( \ln \tanh \rho - \ln \tanh M_{\delta\eta}\big) \approx\frac{\abs{\ln(\delta/\eta)}}{8\delta}. \end{align} Therefore, from \eqref{loweboundwithgamma}, \eqref{lowerboundapprox}, since $\gamma_\eps^\ell\to 0$ and by arbitrariness of $\gamma$, we get \begin{align*} \liminf_{\eps\to0}\frac{\abs{\ln(\delta/\eta)}}{2\delta}\int_{E_{\eps,\rho}^\ell \setminus E_{M_{\delta\eta}} }&\abs{\nabla u_\eps}^2\de\textbf{x}\geq 2\pi (m-\alpha)^2. \end{align*} By applying the same idea on $E_{\eps,\rho}^r \setminus E_{M_{\delta\eta}}$ we get \begin{align*} \liminf_{\eps\to0}\frac{\abs{\ln(\delta/\eta)}}{2\delta}\int_{E_\rho \setminus E_{M_{\delta\eta}} }\abs{\nabla u_\eps}^2\de\textbf{x}&\geq 2\pi\big[ (m-\alpha)^2+ (m-\beta)^2\big]\\[5pt] &\geq\pi(\beta-\alpha)^2\\[5pt] &\geq\limsup_{\eps\to0}\frac{\abs{\ln(\delta/\eta)}}{2\delta}\int_{E_\rho \setminus E_{M_{\delta\eta}} }\abs{\nabla u_\eps}^2\de\textbf{x}, \end{align*} where we optimized for $m$ and we used the energy of the competitor built in \eqref{testfootball}. In conclusion we have \begin{align}\label{malphabeta2} m=\frac{\alpha+\beta}{2} \end{align} and \begin{align*} \lim_{\eps\to 0} \frac{\abs{\ln(\delta/\eta)}}{\delta}F_\eps(u_\eps)=\pi(\beta-\alpha)^2. \end{align*} From \eqref{upperboundueps}, we can conclude that \begin{align*} \lim_{\eps\to 0} F_\eps(u_\eps,\Omega_\eps)=\lim_{\eps\to 0}(u_\eps,\Omega_\eps\setminus N_\eps). \end{align*} From \eqref{malphabeta2}, we can infer also that \begin{align*} u_\eps(0,0,0)\to\frac{\alpha+\beta}{2}, \end{align*} as $\eps\to0$. \emph{Step 2. Narrow thick regime.} The proof corresponds to the previous one, with no differences, as we use the same rescaling \begin{align*} \hat{u}_\eps(x,y,z)\coloneqq u_\eps(\delta x,\delta y ,\eta z). \end{align*} This concludes the proof. \end{proof} \subsection{Letter-box regime} In this regime the parameters are ordered as $\delta \gg \eps \gg \eta$. Namely, we have \begin{align*} \lim_{\eps\to0} \frac{\delta}{\eps}=+\infty\quad\text{and}\quad\lim_{\eps\to0}\frac{\eta}{\delta}=0, \end{align*} In this regime, the order of the energy inside and outside of the neck provides two order that are asymptotically the same, namely \begin{align}\label{limitell} \lim_{\eps\to 0}\frac{\eps}{\eta}\abs{\log(\eta/\delta)}=\ell\in(0,\infty). \end{align} In this setting, we expect the transition happening inside and outside of the neck simultaneously. \subsubsection{Critical Letter-box regime} This sub-regime, corresponds to the case in which the limit $\ell$ in \eqref{limitell} is finite and strictly positive. \\ In order to understand the transition inside the neck, we rescale it to make it the set $N\coloneqq [-1,1]^3$. If $u_\eps$ is a local minimizer of the functional \eqref{004}, the only rescaling that works is \begin{align*} v_\eps(x,y,z)\coloneqq u_\eps(\eps x,\delta y, \eta z). \end{align*} If we rescale in this way, the limiting domain becomes $$\Omega_\infty = \Omega^\ell_\infty \cup N \cup \Omega^r_\infty,$$ where $\Omega^\ell_\infty = \{x<-1\}$ and $\Omega^r_\infty = \{x>1\}$. \begin{theorem}\label{Theoremcriticalletterbox} In the Critical Letter-box regime, it holds that \begin{align*} \lim_{\eps\to 0} \frac{\eps}{\delta\eta}F_\eps(u_\eps,N_\eps)= \frac{\pi^2(\beta-\alpha)^2}{(\pi+2\ell)^2}, \end{align*} and that \begin{equation*} \lim_{\eps\to 0} \frac{|\ln(\eta/\delta)|}{\delta}F_\eps(u_\eps,\O_\eps\setminus N_\eps)= 2\pi\frac{\ell^2(\beta-\alpha)^2}{(\pi+2\ell)^2}. \end{equation*} \end{theorem} \begin{proof} \emph{Step $1$. Energy estimate inside the neck.} This part of the proof follows the strategy of Theorem \ref{Theoremsuperthin}. However, we remark that the scale-invariant Poincaré inequality in this regimes does not provide us with any information.\\ Consider the rescaling $v_\varepsilon:[-1,1]^3\to\R{}$ be defined as \[ v_\eps(x,y,z)\coloneqq u_\eps(\eps x,\delta y, \eta z). \] It is possible to prove that $v_\eps$ converges to a function $\hat{v}\in H^1(N)$, of the form $\hat{v}(x,y,z)=v(x)$, where $v\in H^1(-1,1)$. \\ We can write \begin{align}\label{letterbox:lbneck} \nonumber\liminf_{\eps\to0}\frac{\abs{\ln(\eta/\delta)}}{\delta} F(u_\eps)&\geq \liminf_{\eps\to0}2\frac{\abs{\ln(\eta/\delta)}}{\delta}\frac{\delta\eta}{\eps}\int_{-1}^1 (\partial_x v_\eps)^2\ \de x\\[5pt] &\geq 2\ell\big(\hat{v}(-1)+\hat{v}(1)\big)^2. \end{align} \emph{Step $2$. Energy estimate in the bulk.} In order to get both boundary conditions at the edge of the neck, we need to perform two rescalings. The choice is made as \begin{align*} \hat{u}^\pm_\eps(x,y,z)\coloneqq u_\eps(\eta x\pm\eps,\delta y ,\eta z). \end{align*} In such a way the limiting domains are \begin{align*} \Omega_\infty^\ell&\coloneqq \{x<0\}\cup \{-1<y<1,\ -1<z<1,\ x>0\}.\\[5pt] \Omega_\infty^\rr&\coloneqq \{x>0\}\cup \{-1<y<1,\ -1<z<1,\ x<0\}. \end{align*}By using the same strategy of Theorem \ref{Theoremwindowthick}, we can prove that \begin{align}\label{letterbox:lbbulk} \liminf_{\eps\to0}\frac{\abs{\ln(\delta/\eta)}}{2\delta}\int_{(\O_\eps^\ell\cup\O_\eps^\rr)\setminus N_\eps }\abs{\nabla u_\eps}^2\de\textbf{x}&\geq 2\pi\big[ (\hat{v}(-1)-\alpha)^2+ (\hat{v}(1)-\beta)^2\big] \end{align} \emph{Step 3. Limit of the energy.} By putting together \eqref{letterbox:lbneck} and \eqref{letterbox:lbbulk}, we obtain \begin{align}\label{letterboxtooptimize} \nonumber \liminf_{\eps\to0}\frac{\abs{\ln(\delta/\eta)}}{2\delta}\int_{\O_\eps }&\abs{\nabla u_\eps}^2\de\textbf{x}\\[5pt] &\geq2\pi\big[ (\hat{v}(-1)-\alpha)^2+ (\hat{v}(1)-\beta)^2\big]+2\ell\big(\hat{v}(-1)+\hat{v}(1)\big)^2. \end{align} Now, if we optimize \eqref{letterboxtooptimize} for $\hat{v}(-1)$ and $\hat{v}(1)$, we obtain \begin{align*} v(-1)=\frac{\pi\alpha+(\alpha+\beta)\ell}{\pi+2\ell}=\quad\text{and}\quad v(1)=\frac{\pi\beta+(\alpha+\beta)\ell}{\pi+2\ell}. \end{align*} In conclusion, from \eqref{upperboundueps}, we have \begin{align*} \lim_{\eps\to 0} \frac{\eps}{\delta\eta}F_\eps(u_\eps,N_\eps)= \frac{\pi^2(\beta-\alpha)^2}{(\pi+2\ell)^2}, \end{align*} and \begin{equation*} \lim_{\eps\to 0} \frac{|\ln(\eta/\delta)|}{\delta}F_\eps(u_\eps,\O_\eps\setminus N_\eps)= 2\pi\frac{\ell^2(\beta-\alpha)^2}{(\pi+2\ell)^2}. \end{equation*} \end{proof} \begin{remark} From \eqref{letterbox:lbneck}, we can deduce that $\hat{v}$, in the neck, is the unique solution of the variational problem \begin{align*} \min\bigg\{ \frac{1}{2} \int_{-1}^1 |v'|^2 \de x:\ v\in H^1(-1,1),\ v(-1)=\frac{\pi\alpha+(\alpha+\beta)\ell}{\pi+2\ell},\\[5pt] v(1)=\frac{\pi\beta+(\alpha+\beta)\ell}{\pi+2\ell}\bigg\}, \end{align*} whose solution can be explicitly written. \end{remark} The other two critical regimes, obtained when $\ell=0$ or $\ell=\infty$, can easily be obtained by Theorem \ref{Theoremcriticalletterbox}. We list them for completeness, but the proof is omitted. \subsubsection{Super-critical Letter-box regime} In this sub-regime, we have \begin{align*} \lim_{\eps\to 0}\frac{\eps}{\eta}\abs{\log(\eta/\delta)}=+\infty. \end{align*} In this case, we recover the same result as in Theorem \ref{Theoremwindowthick}. \begin{theorem} In the super-critical letter-box regime, it holds \begin{equation*} \lim_{\eps\to0} \frac{|\ln(\eta/\delta)|}{\delta}F_\eps(u_\eps)=\lim_{\eps\to 0} \frac{|\ln(\eta/\delta)|}{\delta}F_\eps(u_\eps,\O_\eps\setminus N_\eps)= \pi(\beta-\alpha)^2. \end{equation*} which translates into the fact that the transition happens entirely outside of the neck. \end{theorem} \subsubsection{Sub-critical Letter-box regime} In this sub-regime, we have \begin{align*} \lim_{\eps\to 0}\frac{\eps}{\eta}\abs{\log(\eta/\delta)}=0. \end{align*} In this case, we recover the same result as in Theorem \ref{Theoremsuperthin}. \begin{theorem} In the super-critical letter-box regime, it holds \begin{align*} \lim_{\eps\to0} \frac{\eps}{\delta\eta}F_\eps(u_\eps)=\lim_{\eps\to 0} \frac{\eps}{\delta\eta}F_\eps(u_\eps,N_\eps)= (\beta-\alpha)^2, \end{align*} which means that the main contribution of the energy is given by the transition inside the neck. Moreover, for $\varepsilon>0$ let $v_\varepsilon:[-1,1]^3\to\R{}$ be defined as \[ v_\eps(x,y,z)\coloneqq u_\eps(\eps x,\delta y, \eta z). \] Then, $v_\eps$ converges to a function $\hat{v}\in H^1(N)$, of the form $\hat{v}(x,y,z)=v(x)$, where $v\in H^1(-1,1)$ is the unique minimizer of the variational problem \begin{align*} \min\bigg\{ \frac{1}{2} \int_{-1}^1 |v'|^2\ \de x:\ v\in H^1(-1,1),\ v(-1)=\alpha,\ v(1)=\beta\bigg\}. \end{align*} In particular, $v(x)=\dfrac{\beta-\alpha}{2}x+\dfrac{\alpha+\beta}{2}$. \end{theorem} \noindent\textbf{Acknowledgements} This research was carried out at the departments of Mathematics of Politecnico di Torino and Radboud University, whose hospitality is gratefully acknowledged. MM and RC are members of GNAMPA (INdAM). This study was carried out within the Geometric-Analytic Methods for PDEs and Applications project (2022SLTHCE), funded by European Union -- Next Generation EU within the PRIN 2022 program (D.D. 104 - 02/02/2022 Ministero dell’Universit\`{a} e della Ricerca). This manuscript reflects only the authors’ views and opinions and the Ministry cannot be considered responsible for them. \bibliographystyle{siam} \def\url#1{} \bibliography{bibliography} \end{document}
2412.04195v1
http://arxiv.org/abs/2412.04195v1
Partial Betti splittings with applications to binomial edge ideals
\documentclass[12pt,twoside]{amsart} \usepackage[english]{babel} \usepackage{amsfonts,amssymb,amsthm,amsmath,mathtools,accents,latexsym} \usepackage[a4paper,top=3cm,bottom=3cm,left=2.5cm,right=2.5cm,marginparwidth=1.75cm]{geometry} \setlength{\parskip}{3pt} \usepackage{xcolor} \usepackage{graphicx,comment,mathtools} \usepackage[colorlinks=true, allcolors=blue]{hyperref} \usepackage{cleveref} \newtheorem{theorem}{Theorem}[section] \newtheorem{lemma}[theorem]{Lemma} \newtheorem{claim}[theorem]{Claim} \newtheorem{proposition}[theorem]{Proposition} \newtheorem{construction}[theorem]{Construction} \newtheorem{corollary}[theorem]{Corollary} \newtheorem{conjecture}[theorem]{Conjecture} \theoremstyle{definition} \newtheorem{definition}[theorem]{Definition} \newtheorem{remark}[theorem]{Remark} \newtheorem{example}[theorem]{Example} \newtheorem{acknowledgement}{Acknowledgement} \newtheorem{notation}[theorem]{Notation} \newtheorem{question}[theorem]{Question} \newcommand{\avj}[1]{\textcolor{purple}{\sffamily ((AVJ: #1))}} \usepackage{tikz} \newcommand*\circled[1]{\tikz[baseline=(char.base)]{ \node[shape=circle,draw,inner sep=2pt] (char) {#1};}} \usepackage{hyperref} \hypersetup{ colorlinks=true, linkcolor=blue, filecolor=magenta, urlcolor=cyan, citecolor=red } \urlstyle{same} \DeclareMathOperator{\tor}{Tor} \DeclareMathOperator{\In}{in} \DeclareMathOperator{\pd}{pd} \DeclareMathOperator{\reg}{reg} \DeclareMathOperator{\comp}{comp} \DeclareMathOperator{\lcm}{lcm} \DeclareMathOperator{\mdeg}{mdeg} \DeclareMathOperator{\rank}{rank} \DeclareMathOperator{\Hom}{Hom} \DeclareMathOperator{\im}{Im} \DeclareMathOperator{\coker}{coker} \DeclareMathOperator{\len}{len} \DeclareMathOperator{\Mon}{Mon} \DeclareMathOperator{\diam}{diam} \DeclareMathOperator{\iv}{iv} \newcommand{\B}{\mathcal{B}} \title{Partial Betti splittings with applications to binomial edge ideals} \date{\today } \author[A.V. Jayanthan]{A.V. Jayanthan} \address[A.V. Jayanthan] {Department of Mathematics, Indian Institute of Technology Madras, Chennai, Tamil Nadu, India - 600036} \email{[email protected] } \author[A. Sivakumar]{Aniketh Sivakumar} \address[A. Sivakumar] {Department of Mathematics, Tulane University, New Oreans, LA, 70118} \email{[email protected]} \author[A. Van Tuyl]{Adam Van Tuyl} \address[A. Van Tuyl] {Department of Mathematics and Statistics\\ McMaster University, Hamilton, ON, L8S 4L8} \email{[email protected]} \keywords{partial Betti splittings, graded Betti numbers, binomial edge ideals, trees} \subjclass[2020]{13D02, 13F65, 05E40} \begin{document} \begin{abstract} We introduce the notion of a partial Betti splitting of a homogeneous ideal, generalizing the notion of a Betti splitting first given by Francisco, H\`a, and Van Tuyl. Given a homogeneous ideal $I$ and two ideals $J$ and $K$ such that $I = J+K$, a partial Betti splitting of $I$ relates {\it some} of the graded Betti of $I$ with those of $J, K$, and $J\cap K$. As an application, we focus on the partial Betti splittings of binomial edge ideals. Using this new technique, we generalize results of Saeedi Madani and Kiani related to binomial edge ideals with cut edges, we describe a partial Betti splitting for all binomial edge ideals, and we compute the total second Betti number of binomial edge ideals of trees. \end{abstract} \maketitle \section{Introduction} Given a homogeneous ideal $I$ of a polynomial ring $R = k[x_1,\ldots,x_n]$ over an arbitrary field $k$, one is often interested in the numbers $\beta_{i,j}(I)$, the graded Betti numbers of $I$, that are encoded into the graded minimal free resolution of $I$. In some situations, we can compute these numbers by ``splitting'' the ideal $I$ into smaller ideals and use the graded Betti numbers of these new ideals to find those of the ideal $I$. More formally, suppose $\mathfrak{G}(L)$ denotes a set of minimal generators of a homogeneous ideal $L$. Given a homogeneous ideal $I$, we can ``split'' this ideal as $I = J+K$ where $\mathfrak{G}(I)$ is the disjoint union of $\mathfrak{G}(J)$ and $\mathfrak{G}(K)$. The ideals $I, J, K$ and $J \cap K$ are then related by the short exact sequence $$0 \longrightarrow J\cap K \longrightarrow J \oplus K \longrightarrow J+K = I \longrightarrow 0.$$ The mapping cone construction then implies that the graded Betti numbers of $I$ satisfy \begin{equation}\label{bettisplit} \beta_{i,j}(I) \leq \beta_{i,j}(J) + \beta_{i,j}(K) + \beta_{i-1,j}(J \cap K) ~~\mbox{for all $i,j \geq 0$}. \end{equation} Francisco, H\`a, and Van Tuyl \cite{francisco_splittings_2008} defined $I = J+K$ to be a {\it Betti splitting} if the above inequality is an equality for all $i,j \geq 0$. Betti splittings of monomial ideals first appeared in work of Eliahou and Kervaire \cite{EK1990}, Fatabbi \cite{fatabbi2001}, and Valla \cite{Valla2005}. In fact, these prototypical results provided the inspiration for Francisco, H\`a, and Van Tuyl's introduction of Betti splittings in \cite{francisco_splittings_2008}. Their paper also provided conditions on when one can find Betti splittings of edge ideals, a monomial ideal associated to a graph (see \cite{francisco_splittings_2008} for more details). Betti splittings have proven to be a useful tool, having been used to study: the graded Betti numbers of weighted edge ideals \cite{kara2022}, the classification of Stanley-Reisner ideals of vertex decomposable ideals \cite{moradi2016}, the linearity defect of an ideal \cite{hop2016}, the depth function \cite{ficarra2023}, componentwise linearity \cite{bolognini2016}, and the Betti numbers of toric ideals \cite{FAVACCHIO2021409,gimenez2024}. In general, an ideal $I$ may not have any Betti splitting. However, it is possible that \Cref{bettisplit} may hold for {\it some} $i,j \geq 0$. In order to quantify this behaviour, we introduce a new concept called a {\it partial Betti splitting} of an ideal $I$. Specifically, if $I = J+K$ with $\mathfrak{G}(I)$ equal to the disjoint union $\mathfrak{G}(J) \cup \mathfrak{G}(K)$, then $I = J+K$ is an {\it $(r,s)$-Betti splitting} if \[\beta_{i,j}(I) = \beta_{i,j}(J)+\beta_{i,j}(K)+\beta_{i-1, j}(J\cap K )\text{\hspace{3mm} for all $(i,j)$ with $i\geq r$ or $j\geq i+s$}.\] Using the language of Betti tables, if $I = J+K$ is an $(r,s)$-Betti splitting, then all the Betti numbers in the $r$-th column and beyond or the $s$-th row and beyond of the Betti table of $I$ satisfy \Cref{bettisplit}. The Betti splittings of \cite{francisco_splittings_2008} will now called {\it complete Betti splittings}. The goal of this paper is two-fold. First, we wish to develop the properties of partial Betti splittings, extending the results of \cite{francisco_splittings_2008}. Note that \cite{francisco_splittings_2008} focused on Betti splittings of monomial ideals; however, as we show, almost all the same arguments work for any homogeneous ideal $I$ of $R = k[x_1,\ldots,x_n]$ when $R$ is graded by a monoid $M$. Among our results, we develop necessary conditions for an $(r,s)$-Betti splitting: \begin{theorem}[\Cref{parcon2}] Let $I$, $J$ and $K$ be homogeneous ideals of $R$ with respect to the standard $\mathbb{N}$-grading such that $\mathfrak{G}(I)$ is the disjoint union of $\mathfrak{G}(J)$ and $\mathfrak{G}(K)$. Suppose that there are integers $r$ and $s$ such that for all $i \geq r$ or $j \geq i+s$, $\beta_{i-1,j}(J \cap K) > 0$ implies that $\beta_{i-1,j}(J) = 0$ and $\beta_{i-1,j}(K) = 0$. Then $I = J + K$ is an $(r,s)$-Betti splitting. \end{theorem} Second, we wish to illustrate (partial) Betti splittings by considering splittings of binomial edge ideals. If $G = (V(G,E(G))$ is a graph on the vertex set $V = [n] :=\{1,\ldots,n\}$ and edge set $E$, the {\it binomial edge ideal of $G$} is the binomial ideal $J_G = \langle x_iy_j - x_jy_i ~|~ \{i,j\} \in E \rangle$ in the polynomial ring $R = k[x_1,\ldots,x_n,y_1,\ldots,y_n]$. Binomial edge ideals, which were first introduced in \cite{herzog_binomial_2010,Ohtani2011}, have connections to algebraic statistics, among other areas. The past decade has seen a flurry of new results about the homological invariants (e.g., Betti numbers, regularity, projective dimension) for this family of ideals (see \cite{ZZ13}, \cite{SZ14}, \cite{deAlba_Hoang_18}, \cite{herzog_extremal_2018}, \cite{KS20}, \cite{jayanthan_almost_2021} for a partial list on the Betti numbers of binomial edge ideals). Interestingly, Betti splittings of binomial edge ideals have not received any attention, providing additional motivation to study this family of ideals. In order to split $J_G$, we wish to partition the generating set $\mathfrak{G}(J_G)$ in such a way that the resulting ideals generated by each partition, say $J$ and $K$, are the binomial edge ideals of some subgraphs of $G$, that is, splittings of the form $J_G = J_{G_1}+J_{G_2}$ where $G_1$ and $G_2$ are subgraphs. We focus on two natural candidates. The first way is to fix an edge $e = \{i,j\} \in E(G)$ and consider the splitting $$J_G = J_{G\setminus e} + \langle x_iy_j- x_jy_i \rangle.$$ where $G\setminus e$ denotes the graph $G$ with the edge $e$ removed. The second way is to fix a vertex $s \in V(G)$ and consider the set $F \subseteq E(G)$ of all edges that contain the vertex $s$. We can then split $J_G$ as follows $$J_G = \langle x_sy_j-x_jy_s ~|~ \{s,j\} \in F \rangle + \langle x_ky_j-x_jy_k ~|~ \{k,l\} \in E(G) \setminus F \rangle.$$ We call such a partition an $s$-partition of $G$. Note that the first ideal is the binomial edge ideal of a star graph, while the second ideal is the binomial edge ideal of the graph $G \setminus \{s\}$, the graph with the vertex $s$ removed. These splittings are reminiscent of the edge splitting of edge ideals and the $x_i$-splittings of monomial ideals introduced in \cite{francisco_splittings_2008}. In general, neither of these splitting will give us a complete Betti splitting. This is not too surprising since the edge ideal analogues are not always complete Betti splittings. So it is natural to ask when we have a partial or complete Betti splitting using either division of $J_G$. Among our results in Section 4, we give a sufficient condition on an edge $e$ of $G$ so that the first partition gives a complete Betti splitting. In the statement below, an edge is a cut-edge if $G \setminus e$ has more connected components than $G$, and a vertex is free if it belongs to a unique maximal clique, a subset of vertices of $G$ such that all the vertices are all adjacent to each other. \begin{theorem}[\Cref{singlefreevertex}]\label{them2} Let $e = \{u,v\} \in E(G)$ be a cut-edge where $v$ is a free vertex in $G\setminus e$. Then $J_G = J_{G\setminus e}+\langle x_uy_v-x_vy_u\rangle$ is a complete Betti splitting. \end{theorem} \noindent Theorem \ref{them2} generalizes previous work of Saeedi Madani and Kiani \cite{kiani_regularity_2013-1}, and it allows us to give new proofs for their results about the Betti numbers, regularity, and projective dimension for some classes of binomial edge ideals (see \Cref{freecutedge}). In the case of $s$-partitions, we again do not always have a complete Betti splitting. However, we can derive a result about the partial Betti splittings for all graphs. \begin{theorem}[\Cref{maintheo2}] Let $J_G$ be the binomial edge ideal of a graph $G$ and let $J_G = J_{G_1}+J_{G_2}$ be an $s$-partition of $G$. Let $c(s)$ be the size of the largest clique that contains $s$. Then $$ \beta_{i,j}(J_G) = \beta_{i,j}(J_{G_1})+\beta_{i,j}(J_{G_2})+\beta_{i-1, j}(J_{G_1}\cap J_{G_2})~~~ \mbox{for all $(i,j)$ with $i\geq c(s)$ or $j\geq i+4$.} $$ In other words, $J_G = J_{G_1}+J_{G_2}$ is a $(c(s), 4)$-Betti splitting. \end{theorem} \noindent Note that if $G$ is a triangle-free graph, then for every vertex $i \in V(G)$ we have $c(i) \leq 2$. We can use the above result to construct a complete Betti splitting for the binomial edge ideals of all triangle-free graphs (see Corollary \ref{trianglefree}). In the final section, we use the complete Betti splitting of \Cref{them2} to explore the (total) graded Betti numbers of binomial edge ideals of trees. In particular, we give formulas for the first and second total Betti numbers for the binomial edge ideal of any tree. Our result extends work of Jayanthan, Kumar, and Sarkar \cite{jayanthan_almost_2021} which computed the first total Betti numbers for these ideals. Our paper is structured as follows. In Section 2 we recall the relevant background. In Section 3 we introduce the notion of a partial Betti splitting and describe some of their basic properties. In Section 4, we consider splittings of $J_G$ using a single edge of $G$, while in Section 5, we consider a splitting of $J_G$ by partitioning the generators on whether or not they contain $x_s$ or $y_s$ for a fixed vertex $s$. In our final section we determine the second total Betti number of binomial edge ideals of trees. \section{Preliminaries} In this section we recall the relevant background on Betti numbers, graph theory, and binomial edge ideals that is required for later results. \subsection{Homological algebra} Throughout this paper $k$ will denote an arbitrary field. Let $R = k[x_1,\ldots,x_n]$ be a polynomial ring over $k$. We will use various gradings of $R$. Recall that if $M$ is a monoid (a set with an addition operation and additive identity), we say a ring $S$ is {\it $M$-graded} if we can write $S = \bigoplus_{j \in M} S_j$, where each $S_j$ is an additive group and $S_{j_1}S_{j_2} \subseteq S_{j_1+j_2}$ for all $j_1,j_2 \in M$. We will primarily use three gradings of $R$ in this paper: (1) $R$ has an $\mathbb{N}$-grading by setting $\deg(x_i) = 1$ for all $i$; (2) $R$ has an $\mathbb{N}^n$-grading by setting $\deg(x_i) = e_i$ for all $i$, where $e_i$ is the standard basis element of $\mathbb{N}^n$; and (3) $R$ has an $\mathbb{N}^2$-grading by setting the degree of some of the $x_i$'s to $(1,0)$, and the degrees of the rest of the $x_i$'s to $(0,1)$. Given an $M$-graded ring $R$, an element $f \in R$ is {\it homogeneous} if $f \in R_j$ for some $j \in M$. We say the {\it degree} of $f$ is $j$ and write $\deg(f) = j$. An ideal $I \subseteq R$ is {\it homogeneous} if it is generated by homogeneous elements. We write $I_j$ to denote all the homogeneous elements of degree $j\in M$ in $I$. We let $\mathfrak{G}(I)$ denote a minimal set of homogeneous generators of $I$. While the choice of elements of $\mathfrak{G}(I)$ may not be unique, the number of generators of a particular degree is an invariant of the ideal. If $I$ is a homogeneous ideal, then the Tor modules ${\rm Tor}_i(k,I)$ are also $M$-graded for all $i \geq 0$. The {\it $(i,j)$-th graded Betti number of $I$} is then defined to be $$\beta_{i,j}(I) := \dim_k {\rm Tor}_i(k,I)_j ~~\mbox{for $i \in \mathbb{N}$ and $j \in M$.}$$ We use the convention that $\beta_{i,j}(I) = 0$ if $i <0$. We are sometimes interested in the (multi)-graded Betti numbers of the quotient $R/I$; we make use of the identity $\beta_{i,j}(R/I) = \beta_{i-1,j}(I)$ for all $i \geq 1$ and $j \in M$. The graded Betti number $\beta_{i,j}(I)$ is also equal to the number of syzygies of degree $j$ in the $i$-th syzygy module of $I$. For further details, see the book of Peeva \cite{P2011}. When $R$ has the standard $\mathbb{N}$-grading, we are also interested in the following two invariants: the {\it (Castelnuovo-Mumford) regularity of $I$}, which is defined as $${\rm reg}(I) = \max\{ j-i ~|~ \beta_{i,i+j}(I) \neq 0\},$$ and the {\it projective dimension of $I$}, which is defined as $${\rm pd}(I) = \max\{i ~|~ \beta_{i,j}(I) \neq 0\}.$$ These invariants measure the ``size'' of the minimal graded free resolution of $I$. \subsection{Graph theory} Throughout this paper, we use $G = (V(G),E(G))$ to represent a finite simple graph where $V(G)$ denotes the vertices and $E(G)$ denotes the edges. Most of our graphs will have the vertex set $[n] = \{1,\dots ,n\}$. A {\it subgraph} of $G$ is a graph $H$ such that $V(H)\subseteq V(G)$ and $E(H)\subseteq E(G)$. An \textit{induced subgraph} on $S\subset V(G)$, denoted by $G[S]$, is a the subgraph with vertex set $S$ and for all $u,v\in S$, if $\{u,v\}\in E(G)$, then $ \{u,v\}\in E(G[S])$. The {\it complement} of a graph, denoted $G^c$, is a graph with $V(G^c) = V(G)$ and $E(G^c) = \{\{u,v\}\mid \{u,v\}\notin E(G)\}$. From a given graph $G = (V(G),E(G))$, if $e \in E(G)$, then we denote by $G\setminus e$ the subgraph of $G$ on the same vertex set, but edge set $E(G\setminus e) = E(G) \setminus \{e\}$. Given any $i \in V(G)$, we let $N_G(i) = \{j ~|~ \{i,j\} \in E(G)\}$ denote the set of {\it neighbours} of the vertex $i$. The {\it degree} of a vertex $i$ is then $\deg_G i = |N_G(i)|$. In the context where there is a fixed underlying graph, we omit the subscript $G$ and write this as $\deg i$. The {\it closed neighbourhood of $i$} is the set $N_G[i] =N_G(i) \cup \{i\}$. If $G = (V(G),E(G))$ is a graph and $e =\{i,j\} \not\in E(G)$, we let $G_e$ denote the graph on $V(G)$, but with edge set $$E(G_e) = E(G) \cup \{\{k,l\} ~|~ k,l \in N_G(i)~~\mbox{or}~~k,l \in N_G(j) \}.$$ So, $G$ is a subgraph $G_e$. We will require a number of special families of graphs. The \textit{$n$-cycle}, denoted $C_n$, is the graph with vertex set $[n]$ with $n \geq 3$ and edge set $\{\{i,i+1\} ~|~ i =1,\ldots,n-1\} \cup \{\{1,n\}\}.$ A \textit{chordal graph} $G$ is a graph where all the induced subgraphs of $G$ that are cycles are 3-cycles, that is, there are no induced $n$-cycles with $n\geq 4$. A \textit{triangle-free graph} is a graph $G$ such that $C_3$ is not an induced subgraph of $G$. A \textit{tree} is a graph which has no induced cycles. A particular example of a tree that we will use is the {\it star graph} on $n$ vertices, denoted $S_n$. Specifically, $S_n$ is the graph on the vertex set $[n]$ and edge set $E(S_n) = \{\{1,k\}\mid 1<k\leq n\}$. A \textit{complete graph} is a graph $G$ where $\{u,v\}\in E(G)$ for all $u,v\in V(G)$. If $G$ is a complete graph on $[n]$, we denote it by $K_n$. A \textit{clique} in a graph $G$ is an induced subgraph $G[S]$ that is a complete graph. A \textit{maximal clique} is a clique that is not contained in any larger clique. A vertex $v$ of $G$ is a \textit{free vertex} if $v$ only belongs to a unique maximal clique in $G$, or equivalently, the induced graph on $N_G(v)$ is a clique. An edge $e = \{u,v\}$ in $G$ is a \textit{cut edge} if its deletion from $G$ yields a graph with more connected components than $G$. Note that a tree is a graph where all of its edges are cut edges. A \textit{free cut edge} is a cut edge $\{u,v\}$ such that both ends, $u$ and $v$, are free vertices in $G \setminus e$. We are also interested in cliques combined with other graphs. A graph $G$ is said to be a \textit{clique-sum} of $G_1$ and $G_2$, denoted by $G = G_1 \cup_{K_r} G_2$, if $V(G_1) \cup V(G_2) = V(G)$, $E(G_1) \cup E(G_2) = E(G)$ and the induced graph on $V(G_1) \cap V(G_2)$ is the clique $K_r$. If $r = 1$, then we write $G = G_1 \cup_v G_2$ for the clique-sum $G_1 \cup _{K_1} G_s$ where $V(K_1) = \{v\}$. A graph $G$ is \textit{decomposable} if there exists subgraphs $G_1$ and $G_2$ such that $G_1\cup_{v}G_2 = G$ and $v$ is a free vertex of $G_1$ and $G_2$. So a decomposable graph is an example of a clique-sum on a $K_1$ where the $K_1$ is a free vertex in both subgraphs. \begin{example} Consider the graph $G$ in \Cref{fig:graph5}, with $V(G) = [7]$ and $$E(G) = \{\{1,2\}, \{2,3\}, \\\{2,4\}, \{4,5\}, \{4,6\}, \{4,7\}, \{6,7\}\}.$$ Here, we can see that $G = T \cup_{\{4\}} K_3$, where $T$ is the tree with $V(T) = \{1,2,3,4,5\}$ and $E(T) = \{\{1,2\}, \{2,3\}, \{2,4\}, \{4,5\}\}$ and $K_3$ is the clique of size $3$, with $V(K_3) = \{4,6,7\}$ and $E(K_3) = \{\{4,6\}, \{4,7\}, \{6,7\}\}$. \begin{figure}[ht] \centering \begin{tikzpicture}[every node/.style={circle, draw, fill=white!60, inner sep=2pt}, node distance=1.5cm] \node (1) at (0, 0) {1}; \node (2) at (1.5, 0) {2}; \node (3) at (3, 0) {3}; \node (4) at (1.5, -1.5) {4}; \node (5) at (0, -1.5) {5}; \node (6) at (0.5, -2.5) {6}; \node (7) at (2.5, -2.5) {7}; \draw (1) -- (2); \draw (2) -- (3); \draw (2) -- (4); \draw (4) -- (5); \draw (4) -- (6); \draw (4) -- (7); \draw (6) -- (7); \end{tikzpicture} \caption{$G = T\cup_{\{4\}}K_3$} \label{fig:graph5} \end{figure} \end{example} \subsection{Binomial edge ideals} Suppose that $G = (V(G),E(G))$ is a finite simple graph with $V(G) = [n]$. The {\it binomial edge ideal} of $G$, denoted $J_G$, is the binomial ideal $$J_G = \langle x_iy_j - x_jy_i ~|~ \{i,j\} \in E(G) \rangle$$ in the polynomial ring $R = k[x_1,\ldots,x_n,y_1,\ldots,y_n]$. In what follows, we will find it convenient to consider different gradings of $R$; we can grade the polynomial ring $R$ either with the standard grading where $\deg x_i=\deg y_i=1$ for all $i$, with an $\mathbb{N}^n$-multigrading where $\deg x_i=\deg y_i=(0,\dots,1,\dots, 0)$, the $i$-th unit vector for all $i$, or with an $\mathbb{N}^2$-grading where $\deg x_i = (1,0)$ for all $i$ and $\deg y_j = (0,1)$ for all $j$. Note that $J_G$ is a homogeneous ideal with respect to all three gradings. We review some useful facts from the literature about the idea $J_G$. Recall that a standard graded ideal $I$ has {\it linear resolution} if $I$ is generated by homogeneous elements of degree $d$ and $\beta_{i,i+j}(I) = 0$ for all $j \neq d$. \begin{theorem}\label{completebetti} Let $G = K_n$ be a complete graph. Then \begin{enumerate} \item The binomial edge ideal $J_G$ has a linear resolution. \item $\beta_{i,i+2}(J_G) = (i+1)\binom{n}{i+2}$ for $i \geq 0$ and $0$ otherwise. \end{enumerate} \end{theorem} \begin{proof} Statement (1) follows from {\cite[Theorem 2.1]{kiani_binomial_2012}}. Statement (2) follows from a more general fact of Herzog, Kiani, and Saaedi Madani \cite[Corollary 4.3]{herzog_linear_2017} on the Betti numbers that appear in the linear strand of a binomial edge ideals applied to $K_n$. \end{proof} The next result is related to a cut edge in a graph. \begin{lemma}[{\cite[Theorem 3.4]{mohammadi_hilbert_2014}}]\label{lemma 3.8} Let $G$ be a simple graph and let $e = \{i,j\}\notin E(G)$ be a cut edge in $G\cup \{e\}$. Let $f_e = x_iy_j-x_jy_i$. Then $J_G:\langle f_e \rangle = J_{G_e}$. \end{lemma} We will require the next result about the Betti polynomials of binomial edge ideals of decomposable graphs. For an $\mathbb{N}$-graded $R$-module $M$, the {\it Betti polynomial of $M$} is $$B_M(s,t) = \sum_{i,j \geq 0} \beta_{i,j}(M)s^it^j.$$ The following result is due to Herzog and Rinaldo, which generalized an earlier result of of Rinaldo and Rauf \cite{rauf_construction_2014}. \begin{theorem}[{\cite[Proposition 3]{herzog_extremal_2018}}]\label{freevertexbetti} Suppose that $G$ is a decomposable graph with decomposition $G = G_1\cup G_2$. Then \[B_{R/J_G}(s, t) = B_{R/J_{G_1}}(s, t)B_{R/J_{G_2}}(s, t).\] \end{theorem} The graded Betti numbers in the linear strand of $J_G$ (all the Betti numbers of the form $\beta_{i,i+2}(J_G))$ were first calculated by Herzog, Kaini, and Saeedi Madani. In the statement below, $\Delta(G)$ is the clique complex of the graph $G$ and $f_{i+1}(\Delta(G))$ is the number of faces in $\Delta(G)$ of dimension $i+1$. \begin{theorem}[{\cite[Corollary 4.3]{herzog_linear_2017}}]\label{linearbinom} Let $G$ be a finite simple graph with binomial edge ideal $J_G$. Then the Betti numbers in the linear strand of $J_G$ are given by \[\beta_{i,i+2}(J_G) = (i+1)f_{i+1}(\Delta(G)) ~~\mbox{for $i\geq 0$.}\] \end{theorem} \begin{example}\label{runningexample} Let $G$ be the finite simple graph on the vertex set $[7]$ with edge set $$E(G) =\{\{1,2\}, \{1,3\}, \{1,4\}, \{1, 5\}, \{1,7\},\{2, 4\}), \{2,5\}, \{2,7\},\{3,7\},\{4,5\},\{6,7\}\}.$$ This graph is drawn in Figure \ref{fig:runningexamp}. \begin{figure}[ht] \centering \begin{tikzpicture}[every node/.style={circle, draw, fill=white!60, inner sep=2pt}, node distance=1.5cm] \node (1) at (1.5, 0) {1}; \node (2) at (1.5, -1.5) {2}; \node (3) at (3, 0) {3}; \node (4) at (0, -1.5) {4}; \node (5) at (0, 0) {5}; \node (6) at (4.5, 0) {6}; \node (7) at (3, -1.5) {7}; \draw (1) -- (2); \draw (1) -- (3); \draw (1) -- (4); \draw (1) -- (5); \draw (1) -- (7); \draw (2) -- (4); \draw (2) -- (5); \draw (2) -- (7); \draw (3) -- (7); \draw (4) -- (5); \draw (6) -- (7); \end{tikzpicture} \caption{Graph $G$} \label{fig:runningexamp} \end{figure} The binomial edge ideal of $G$ is an ideal of $R=k[x_1,\ldots,x_7,y_1,\ldots,y_7]$ with 11 generators. Specifically, \begin{multline*} J_G = \langle x_1y_2-x_2y_1, x_1y_3-x_3y_1, x_1y_4-x_4y_1, x_1y_5-x_5y_1, x_1y_7-x_7y_1, x_2y_4-x_4y_2, \\ x_2y_5-x_5y_2, x_2y_7-x_7y_2, x_3y_7-x_7y_3, x_4y_5-x_5y_4, x_6y_7-x_7x_6 \rangle. \end{multline*} \end{example} \section{Partial Betti splittings} In this section, we define the notion of a partial Betti splitting, generalising the concept of a Betti splitting first established by Francisco, H\`a, and Van Tuyl \cite{francisco_splittings_2008}. While a Betti splitting of an ideal $I$ is a ``splitting" of $I$ into two ideals $I = J+K$ such that {\it all} of the (multi)-graded Betti numbers of $I$ can be related to those of $J, K$ and $J \cap K$, in a partial Betti splitting, we only require that some of these relations to hold. Betti splittings of ideals were originally defined just for monomial ideals, since the original motivation of \cite{francisco_splittings_2008} was to extend Eliahou and Kevaire's splitting of monomial ideals \cite{EK1990}. However, a careful examination of the proofs of \cite{francisco_splittings_2008} reveals that some of the main results hold for all (multi)-graded ideals in a polynomial ring $R = k[x_1,\ldots,x_n]$. We develop partial Betti splittings within this more general context. Assuming that $R$ is $M$-graded, let $I,J$, and $K$ be homogeneous ideals with respect to this grading such that $I = J + K$ and $\mathfrak{G}(I)$ is the disjoint union of $\mathfrak{G}(J)$ and $\mathfrak{G}(K)$. We have a natural short exact sequence $$0 \longrightarrow J \cap K \stackrel{\varphi}{\longrightarrow} J \oplus K \stackrel{\psi}{\longrightarrow} I = J+K \longrightarrow 0,$$ where the maps $\varphi(f) = (f,-f)$ and $\psi(g,h) = g+h$ have degree $0$, i.e., they map elements of degree $j \in M$ to elements of degree $j \in M$. The mapping cone resolution applied to this short exact sequence then implies that $$\beta_{i,j}(I) \leq \beta_{i,j}(J) + \beta_{i,j}(K) + \beta_{i-1,j}(J \cap K) ~~\mbox{for all $i \geq 0$ and $j \in M$}.$$ We are then interested in when we have an equality. The following lemma gives such a condition for a specific $i \in \mathbb{N}$ and $j \in M$. The proof is essentially the same as \cite[Proposition 2.1]{francisco_splittings_2008} which considered only monomial ideals, but for completeness, we have included the details here. \begin{lemma}\label{singlesplit} Let $R$ be a $M$-graded ring, and suppose that $I, J$, and $K$ are homogeneous ideals with respect to this grading such that $I = J+K$ and $\mathfrak{G}(I)$ is the disjoint union of $\mathfrak{G}(J)$ and $\mathfrak{G}(K)$. Let $$0 \longrightarrow J \cap K \stackrel{\varphi}{\longrightarrow} J \oplus K \stackrel{\psi}{\longrightarrow} I = J+K \longrightarrow 0$$ be the natural short exact sequence. Then, for a fixed integer $i > 0$ and $j \in M$, the following two statements are equivalent: \begin{enumerate} \item $\beta_{i,j}(I) = \beta_{i,j}(J)+\beta_{i,j}(K) + \beta_{i-1,j}(J\cap K)$; \item the two maps $$\varphi_i:{\rm Tor}_i(k,J \cap K)_j \rightarrow {\rm Tor}_i(k,J)_j \oplus {\rm Tor}_i(k,K)_j$$ and $$\varphi_{i-1}:{\rm Tor}_{i-1}(k,J \cap K)_j \rightarrow {\rm Tor}_{i-1}(k,J)_j \oplus {\rm Tor}_{i-1}(k,K)_j$$ induced from the long exact sequence of \emph{Tor} using the above short sequence are both the zero map. \end{enumerate} \end{lemma} \begin{proof} Fix an integer $i >0$ and $j \in M$. Using the short exact sequence given in the statement, we can use Tor to create a long exact sequence that satisfies \begin{multline*} \cdots \rightarrow {\rm Tor}_i(k,J \cap K)_j \stackrel{\varphi_i}{\rightarrow} {\rm Tor}_i(k,J)_j \oplus {\rm Tor}_i(k,K)_j \rightarrow {\rm Tor}_i(k,I)_j \rightarrow \\ {\rm Tor}_{i-1}(k,J \cap K)_j \stackrel{\varphi_{i-1}}\rightarrow {\rm Tor}_{i-1}(k,J)_j \oplus {\rm Tor}_{i-1}(k,K)_j \rightarrow \cdots . \end{multline*} Consequently, we have an exact sequence of vector spaces \begin{multline*} 0 \rightarrow {\rm Im}(\varphi_i)_j \rightarrow {\rm Tor}_i(k,J)_j \oplus {\rm Tor}_i(k,K)_j \rightarrow {\rm Tor}_i(k,I)_j \rightarrow \\ {\rm Tor}_{i-1}(k,J \cap K)_j \stackrel{\varphi_{i-1}}\rightarrow A_j \rightarrow 0 \end{multline*} where $$A = {\rm Im}(\varphi_{i-1}) \cong {\rm Tor}(k,J \cap K)/{\ker \varphi_{i-1}}.$$ We thus have $$\beta_{i,j}(I) = \beta_{i,j}(J)+\beta_{i,j}(K) + \beta_{i-1,j}(J\cap K) - \dim_k ({\rm Im}(\varphi_i))_j - \dim_k ({\rm Im}(\varphi_{i-1}))_j.$$ To prove $(1) \Rightarrow (2)$, note that if both $\varphi_i$ and $\varphi_{i-1}$ are the zero map, we have $\beta_{i,j}(I) = \beta_{i,j}(J) + \beta_{i,j}(K) + \beta_{i-1,j}(J \cap K)$. For $(2) \Rightarrow (1)$, if either of $\varphi_i$ or $\varphi_{i-1}$ is not the zero map, either $\dim_k ({\rm Im}(\varphi_i))_j > 0$ or $\dim_k ({\rm Im}(\varphi_{i-1}))_j> 0$, which forces $\beta_{i,j}(I) < \beta_{i,j}(J) + \beta_{i,j}(K) + \beta_{i-1,j}(J \cap K).$ \end{proof} The following corollary, which is \cite[Proposition 3]{francisco_splittings_2008}, immediately follows. \begin{corollary} Let $R$ be a $M$-graded ring, and suppose that $I, J$, and $K$ are homogeneous ideals with respect to this grading such that $I = J+K$ and $\mathfrak{G}(I)$ is the disjoint union of $\mathfrak{G}(J)$ and $\mathfrak{G}(K)$. Let $$0 \longrightarrow J \cap K \stackrel{\varphi}{\longrightarrow} J \oplus K \stackrel{\psi}{\longrightarrow} I = J+K \longrightarrow 0$$ be the natural short exact sequence. Then $\beta_{i,j}(I) = \beta_{i,j}(J)+\beta_{i,j}(K) + \beta_{i-1,j}(J\cap K)$ for all integers $i \geq 0$ and $j \in M$, if and only if the maps $$\varphi_i:{\rm Tor}_i(k,J \cap K)_j \rightarrow {\rm Tor}_i(k,J)_j \oplus {\rm Tor}_i(k,K)_j$$ induced from the long exact sequence of {\rm Tor} using the above short exact sequence are the zero map for all integers $i \geq 0$ and $j \in M$. \end{corollary} Applying \Cref{singlesplit} directly implies that we would need to understand the induced maps between {\rm Tor} modules in order to determine if a specific $(i,j)$-th graded Betti number of $I$ can be determined from those of $J$, $K$, and $J\cap K$. However, we can now modify Theorem 2.3 from \cite{francisco_splittings_2008} to obtain a a specific ``splitting'' of $\beta_{i,j}(I)$ from other graded Betti numbers. \begin{theorem}\label{parcon} Let $R$ be a $M$-graded ring, and suppose that $I, J$, and $K$ are homogeneous ideals with respect to this grading such that $I = J+K$ and $\mathfrak{G}(I)$ is the disjoint union of $\mathfrak{G}(J)$ and $\mathfrak{G}(K)$. Suppose for a fixed integer $i > 0$ and $j \in M$ we have that: \begin{itemize} \item if $\beta_{i,j}(J\cap K)>0$, then $\beta_{i,j}(J) = 0$ and $\beta_{i,j}(K) = 0$, and \item if $\beta_{i-1,j}(J\cap K)>0$, then $\beta_{i-1,j}(J) = 0$ and $\beta_{i-1,j}(K) = 0.$ \end{itemize} Then we have: \begin{equation} \beta_{i,j}(I) = \beta_{i,j}(J)+\beta_{i,j}(K)+\beta_{i-1, j}(J\cap K ). \end{equation} \end{theorem} \begin{proof} Since $I = J+K$, we have the short exact sequence \[0\longrightarrow J\cap K \xlongrightarrow{\varphi} J\oplus K \xlongrightarrow{\psi} J+K = I\longrightarrow 0.\] For all integers $\ell \geq 0$ and $j \in M$, we get the induced maps $$\varphi_\ell:{\rm Tor}_\ell(k,J \cap K)_j \rightarrow {\rm Tor}_\ell(k,J)_j \oplus {\rm Tor}_\ell(k,K)_j$$ from the long exact sequence of {\rm Tor} using the short exact sequence. Let $i > 0$ and $j \in M$ be the fixed $i$ and $j$ as in the statement. There are four cases to consider: (1) $\beta_{i,j}(J \cap K)$ and $\beta_{i-,j}(J \cap K)$ both non-zero, (2) $\beta_{i,j}(J\cap K) = 0$ and $\beta_{i-1,j}(J \cap K) > 0$, (3) $\beta_{i,j}(J\cap K) > 0$ and $\beta_{i-1,j}(J \cap K) = 0$, and (4) both $\beta_{i,j}(J\cap K) = \beta_{i-1,j}(J \cap K) = 0$. In case (1), the maps $\varphi_i$ and $\varphi_{i-1}$ must be the zero map since $0 =\beta_{i,j}(J)$ and $0 = \beta_{i,j}(K)$ imply that ${\rm Tor}_i(k,J)_j \oplus {\rm Tor}_i(k,K)_j = 0$, and similarly, $0 =\beta_{i-1,j}(J)$ and $0 = \beta_{i-1,j}(K)$ imply ${\rm Tor}_{i-i}(k,J)_j \oplus {\rm Tor}_{i-1}(k,K)_j = 0$. The conclusion now follows from \Cref{singlesplit}. For case (2), the map $\varphi_{i-1}$ is the zero map using the same argument as above. On the other hand, $0 = \beta_{i,j}(J \cap K) = \dim_k {\rm Tor}(k, J\cap K)_j$ implies that $\varphi_i$ is the zero map. We now apply \Cref{singlesplit}. Cases (3) and (4) are proved similarly, so we omit the details. \end{proof} We now introduce the notion of a partial Betti splitting, that weakens the conditions of a Betti splitting found in \cite{francisco_splittings_2008}. Note that we assume that $R$ has the standard $\mathbb{N}$-grading. \begin{definition}\label{pardef} Let $I$, $J$ and $K$ be homogeneous ideals of $R$ with respect to the standard $\mathbb{N}$-grading such that $\mathfrak{G}(I)$ is the disjoint union of $\mathfrak{G}(J)$ and $\mathfrak{G}(K)$. Then $I= J + K$ is an {\it $(r,s)$-Betti splitting} if \[\beta_{i,j}(I) = \beta_{i,j}(J)+\beta_{i,j}(K)+\beta_{i-1, j}(J\cap K )\text{\hspace{3mm} for all $(i,j)$ with $i\geq r$ or $j\geq i+s$}.\] If $(r,s) \neq (0,0)$ we call an $(r,s)$-Betti splitting $I=J+K$ a {\it partial Betti splitting}. Otherwise, we say that $I = J+K$ is a {\it complete Betti splitting} if it is a $(0,0)$-Betti splitting, that is, $$\beta_{i,j}(I) = \beta_{i,j}(J) + \beta_{i,,j}(K) + \beta_{i-1,j}(J\cap K) ~~\mbox{for all $i,j \geq 0$}.$$ \end{definition} \begin{remark} A complete Betti splitting is what Francisco, H\`a, and Van Tuyl \cite{francisco_splittings_2008} called a Betti splitting. \end{remark} \begin{remark} We can interpret the above definition with the Betti table of $I$. The {\it Betti table of $I$} is a table whose columns are indexed by the integers $i\geq 0$, and in row $j$ and column $i$, we place $\beta_{i,i+j}(I)$. If $I = J+K$ is an $(r,s)$-Betti splitting, then all the Betti numbers in the Betti table of $I$ in the $r$-th column and beyond or in the $s$-th row and beyond are ``split'', that is, they satisfy $\beta_{i,j}(I) = \beta_{i,j}(J)+\beta_{i,j}(K)+\beta_{i-1, j}(J\cap K ).$ \end{remark} The following observation will be useful. \begin{lemma} Suppose that $I=J+K$ is an $(r,s)$-Betti splitting of $I$. If $r = 0$ or $1$, then $I=J+K$ is a complete Betti splitting. \end{lemma} \begin{proof} Since $I = J+K$ is an $(r,s)$-Betti splitting, we have $\mathfrak{G}(I) = \mathfrak{G}(J) \cup \mathfrak{G}(K)$. Consequently, we always have $$\beta_{0,j}(I) = \beta_{0,j}(J) + \beta_{0,j}(K) + \beta_{-1,j}(J\cap K) = \beta_{0,j}(J)+\beta_{0,j}(K) ~\mbox{for $i=0$ and all $j \geq 0$.}$$ For any $(r,s)$-Betti splitting with $r =0$ or $1$, the definition implies \[\beta_{i,j}(I) = \beta_{i,j}(J)+\beta_{i,j}(K)+\beta_{i-1, j}(J\cap K ) ~\mbox{for all $i > 0$ and all $j \geq 0$}.\] So, for any $i,j \geq 0$, we have $\beta_{i,j}(I) = \beta_{i,j}(J) + \beta_{i,j}(K) + \beta_{i-1,j}(J \cap K)$, that is, we have a complete Betti splitting. \end{proof} We can now use Theorem \ref{parcon} to get a condition on $(r,s)$-Betti splittings. \begin{theorem}\label{parcon2} Let $I$, $J$ and $K$ be homogeneous ideals of $R$ with respect to the standard $\mathbb{N}$-grading such that $\mathfrak{G}(I)$ is the disjoint union of $\mathfrak{G}(J)$ and $\mathfrak{G}(K)$. Suppose that there are integers $r$ and $s$ such that for all $i \geq r$ or $j \geq i+s$, $\beta_{i-1,j}(J \cap K) > 0$ implies that $\beta_{i-1,j}(J) = 0$ and $\beta_{i-1,j}(K) = 0$. Then $I = J + K$ is an $(r,s)$-Betti splitting. \end{theorem} \begin{proof} Let $r$ and $s$ be as in the statement, and suppose that $(i,j)$ is fixed integer tuple that satisfies $i \geq r$ or $j \geq i+s$. But then $(i+1,j)$ also satisfies $i+1 \geq r$ or $j \geq i+s$. Consequently, for this fixed $(i,j)$, the hypotheses imply \begin{enumerate} \item[$\bullet$] if $\beta_{i-1,j}(J\cap K) >0$, then $\beta_{i-1,j}(J) = \beta_{i-1,j}(K) = 0$, and \item[$\bullet$] if $\beta_{i,j}(J\cap K) > 0$, then $\beta_{i,j}(J) = \beta_{i,j}(K) = 0$. \end{enumerate} By Theorem \ref{parcon}, this now implies that $$\beta_{i,j}(I) = \beta_{i,j}(J)+\beta_{i,j}(K) + \beta_{i-1,j}(J\cap K)$$ for this fixed pair $(i,j)$. But since this is true for all $(i,j)$ with either $i \geq r$ or $j \geq i+s$, this means $I=J+K$ is an $(r,s)$-Betti splitting. \end{proof} We end this section with consequences for the regularity and projective dimension of $I$ for a partial Betti splitting. The case for a complete Betti splitting was first shown in \cite[Corollary 2.2]{francisco_splittings_2008}. \begin{theorem}\label{regprojbounds} Suppose that $I=J+K$ is an $(r,s)$-Betti splitting of $I$. Set \begin{eqnarray*} m &= &\max\{ {\rm reg}(J), {\rm reg}(K), {\rm reg}(J\cap K)-1\}, ~~\mbox{and} \\ p &=& \max\{ {\rm pd}(I), {\rm pd}(J), {\rm pd}(J\cap K)+1\}. \end{eqnarray*} Then \begin{enumerate} \item if $m \geq s$, then ${\rm reg}(I) = m$. \item if $p \geq r$, then ${\rm pd}(I) = p$. \end{enumerate} \end{theorem} \begin{proof} By applying the mapping cone construction to the the short exact sequence $$0 \longrightarrow J \cap K \longrightarrow J \oplus K \longrightarrow J+K = I \longrightarrow 0,$$ we always have ${\rm reg}(I) \leq m$ and ${\rm pd}(I) \leq p$. Since $m \geq s$, this means for all $i \geq 0$ $$\beta_{i,i+m}(I)=\beta_{i,i+m}(J)+\beta_{i,i+m}(K) +\beta_{i-1,i+m}(J\cap K)$$ because we have an $(r,s)$-Betti splitting. By our definition of $m$, there is an integer $i$ such that at least one of the three terms on the right hand side must be nonzero. This then forces ${\rm reg}(I) \geq m$, thus completing the proof that ${\rm reg}(I) = m$. Similarly, since $p \geq r$, for all $j \geq 0$ we have $$\beta_{p,j}(I) = \beta_{p,j}(J)+\beta_{p,j}(K) +\beta_{p-1,j}(J\cap K).$$ By our definition of $p$, there is at least one $j$ such that one of the terms on the right hand side is nonzero, thus showing ${\rm pd}(I) \geq p$. Consequently, ${\rm pd}(I) = p$. \end{proof} \begin{example}\label{runningexample2} We illustrate a partial Betti splitting using the binomial edge ideal $J_G$ of \Cref{runningexample}. We ``split'' $J_G$ as $J_G = J + K$ where \begin{eqnarray*} J & = & \langle x_1y_2-x_2y_1, x_1y_3-x_3y_1, x_1y_4-x_4y_1, x_1y_5-x_5y_1, x_1y_7-x_7y_1 \rangle ~~\mbox{and}\\ K& = & \langle x_2y_4-x_4y_2, x_2y_5-x_5y_2, x_2y_7-x_7y_2, x_3y_7-x_7y_3, x_4y_5-x_5y_4, x_6y_7-x_7x_6 \rangle. \end{eqnarray*} We compute the graded Betti tables use in \emph{Macaulay2} \cite{mtwo}. The graded Betti tables of $J$, $K$ and $J \cap K$ are given below. \footnotesize \begin{verbatim} 0 1 2 3 4 0 1 2 3 4 0 1 2 3 4 5 total: 5 20 30 18 4 total: 6 15 20 14 4 total: 15 47 73 62 26 4 2: 5 . . . . 2: 6 2 . . . 2: . . . . . . 3: . 20 30 18 4 3: . 13 8 . . 3: 10 9 2 . . . 4: . . . . . 4: . . 12 14 4 4: 5 26 21 4 . . 5: . . . . . 5: . . . . . 5: . 12 50 58 26 4 Betti Table J Betti Table K Betti Table J intersect K \end{verbatim} \normalsize We compare this to the Betti table of $J_G$: \footnotesize \begin{verbatim} 0 1 2 3 4 5 6 total: 11 44 89 103 70 26 4 2: 11 12 3 . . . . 3: . 32 62 39 8 . . 4: . . 24 64 62 26 4 Betti Table J_G \end{verbatim} \normalsize Then $J_G = J+K$ is {\it not} a complete Betti splitting since $$\beta_{2,4}(J_G) = 3 \neq 0+ 0+ 9 =\beta_{2,4}(J) + \beta_{2,4}(K) + \beta_{1,4}( J\cap K).$$ However, this is an example of a $(4,4)$-Betti splitting since $$\beta_{i,j}(J_G) = \beta_{i,j}(J) + \beta_{i,j}(K) + \beta_{i-1,j}(J\cap K) ~~\mbox{for all $i \geq 4$ and $j \geq i+4$.}$$ \end{example} \section{Betti splittings of binomial edge ideals: cut edge case} In this section and the next, we wish to understand when a binomial edge ideal $J_G$ has a (partial) Betti splitting. A natural candidate to consider is when $G_1$ is a single edge $e = \{u,v\}$ of $G$ and $G_2 = G\setminus e$. More formally, if $f_e = x_uy_v-x_vy_u$ is the binomial associated to $e$, we wish to understand when $$J_G = \langle f_e \rangle + J_{G\setminus e}$$ is either a partial or a complete Betti splitting of $J_G$. As we show in this section, with some extra hypotheses on $e$, this splitting of $J_G$ does indeed give a complete Betti splitting. Since Betti splittings require information about the intersection of the two ideals used in the splitting, the following lemma shall prove useful. \begin{lemma}\label{lemma 2.18} Let $G = (V(G),E(G))$ be a simple graph with $e \in E(G)$. Then, using the standard grading of $R$, we have a graded $R$-module isomorphism $$[J_{G\setminus e} \cap \langle f_e \rangle] \cong [J_{G\setminus e}: \langle f_e \rangle](-2).$$ Furthermore, if $e$ is a cut edge, then $$ \beta_{i,j}(J_{(G\setminus e)}\cap \langle f_e\rangle) = \beta_{i,j-2}(J_{(G\setminus e)_e}) ~\mbox{for all $i \geq 0$}.$$ \end{lemma} \begin{proof} By definition of quotient ideals, we have that $J_{G\setminus e}: \langle f_e \rangle \xrightarrow{\cdot f_e} J_{(G\symbol{92} e)}\cap \langle f_e\rangle$ is an $R$-module isomorphism of degree two. This fact implies the first statement. Now suppose that $e$ is a cut edge. From \Cref{lemma 3.8} we have that $J_{(G\setminus e)_e} = J_{G\setminus e}: \langle f_e \rangle$. Using this fact and the above isomorphisms of modules, we have $$ \tor_i(J_{(G\setminus e)_e},k)_{j-2} = \tor_{i}(J_{G\setminus e}:\langle f_e \rangle, k)_{j-2} \cong \tor_{i}(J_{G\setminus e}\cap \langle f_e\rangle, k)_j. $$ This isomorphism imples that $\beta_{i,j}(J_{(G\setminus e)}\cap \langle f_e\rangle) = \beta_{i,j-2}(J_{(G\setminus e)_e})$ for all $i \geq 0$ for $j \geq 2$. Now, for any $i \geq 0$ and $j=0$, $\beta_{i,0}(J_{(G\setminus e)}\cap \langle f_e\rangle) = \beta_{i,0-2}(J_{(G\setminus e)_e}) =0$. Finally, because $J_{(G\setminus e)_e} = J_{G \setminus e} : \langle f_e \rangle$ is generated by degree two binomials, then $J_{G\setminus e} \cap \langle f_e \rangle$ is generated by degree four elements. Thus $\beta_{i,1}(J_{(G\setminus e)}\cap \langle f_e\rangle) = \beta_{i,1-2}(J_{(G\setminus e)_e}) =0$ for all $i \geq 0$ and $j =1$. \end{proof} With the above lemma, we can study splittings where $e = \{u,v\}$ when $v$ is a pendant vertex, that is, $\deg v = 1$. \begin{theorem}\label{maintheo} Let $e = \{u,v\} \in E(G)$ with $v$ a pendant vertex. Then \begin{enumerate} \item $J_G = J_{G\setminus e}+\langle f_e\rangle$ is a complete Betti splitting, and \item $\beta_{i,j}(J_G) = \beta_{i,j}(J_{G\symbol{92}e}) + \beta_{i-1,j-2}(J_{(G\setminus e)_e})$ for all $i\geq 1$ and $j \geq 0$. \end{enumerate} \end{theorem} \begin{proof} (1). Let $J_G = \langle f_e\rangle+J_{G\setminus e} \subseteq R = k[x_1,\ldots,x_n,y_1,\ldots,y_n]$. We consider the $\mathbb{N}^n$-grading on $R$ given by $\deg x_i = \deg y_i = e_i$, the $i$-th standard basis vector of $\mathbb{N}^n$. Note that $J_G$ is a homogeneous ideal with respect to this grading. Since $\langle f_e\rangle\cap J_{G\setminus e}\subseteq \langle f_e \rangle$, all generators of $\langle f_e\rangle\cap J_{G\setminus e}$ are of the form $rf_e = r(x_uy_v-x_vy_u)$, where $r$ is some polynomial in $R$. Hence, the multidegree of the generators, and thus the multigraded Betti numbers of the ideal $\langle f_e\rangle\cap J_{G\setminus e}$ must occur with multidegrees $\mathbf{a} = (a_1,\ldots,a_n)$ where its $v$-th component $a_v$ is non-zero. Because $v$ is a pendant vertex, $J_{G\setminus e}$ contains no generators having $x_v$ or $y_v$. Thus, $\beta_{i,{\bf a}}(J_{G\symbol{92}e}\cap \langle f_e \rangle )>0$ implies that $\beta_{i,{\bf a}}(J_{G \setminus e}) = 0$ for all $i\in \mathbb{N}$ and all multidegrees ${\bf a} \in \mathbb{N}^n$ as defined above. We have that $\beta_{0,2}(\langle f_e\rangle) = 1$ and $\beta_{i,j}(\langle f_e\rangle) = 0$ for $i\neq 0$ and $j\neq 2$ as $\langle f_e\rangle$ is a principal ideal. Since $J_{G\symbol{92}e}\cap \langle f_e\rangle$ is generated by polynomials of degree three or more, this means that $\beta_{i,j}(J_{G\symbol{92}e}\cap \langle f_e\rangle)>0$ implies $\beta_{i,j}(\langle f_e \rangle) = 0$ for all $i\geq 0$ and degrees $j$. It is clear that since this is true for all degrees $j$, this result also holds for all ${\bf a} \in \mathbb{N}^n$ as well, that is, if $\beta_{i,{\bf a}}(J_{G \setminus e} \cap \langle f_e \rangle) > 0$, then $\beta_{i,{\bf a}}(\langle f_e \rangle) =0$ for all $i \geq 0$ and degrees ${\bf a} \in \mathbb{N}^n$. Therefore \Cref{parcon} implies that $$\beta_{i,{\bf a}}(J_G) = \beta_{i,{\bf a}}(J_{G\setminus e})+ \beta_{i,{\bf a}}(\langle f_e \rangle) + \beta_{i-1,{\bf a}}(J_{G\setminus e} \cap \langle f_e \rangle)$$ for all $i \geq 0$ and ${\bf a} \in \mathbb{N}^n$. Since this true for all multidegrees, we can combine them to obtain the same result with the degrees $j$ in the standard grading. Hence we have: $$\beta_{i,j}(J_G) = \beta_{i,j}(\langle f_e\rangle)+ \beta_{i,j}(J_{G\symbol{92} e}) + \beta_{i-1,j}(J_{G\symbol{92} e}\cap \langle f_e\rangle) ~\text{for all $i,j \geq 0$},$$ that is, $J_G = \langle f_e\rangle+J_{G\setminus e}$ is a complete Betti splitting. An edge with a pendant vertex is a cut edge of $G$. So, to prove (2), we can combine (1) and \Cref{lemma 2.18} to give $$\beta_{i,j}(J_G) = \beta_{i,j}(\langle f_e\rangle)+\beta_{i,j}(J_{G\symbol{92} e}) + \beta_{i-1,j-2}(J_{(G\symbol{92} e)_e})$$ for all integers $i \geq 1$ and $j \geq 0$. On the other hand, $\beta_{i,j}(\langle f_e\rangle) = 0$ for $i\neq 0$ or $j\neq 2$. Hence, $\beta_{i,j}(J_G) = \beta_{i,j}(J_{G\symbol{92}e}) + \beta_{i-1,j-2}(J_{(G\symbol{92}e)_e})$ for all $i\geq 1$ and $j \geq 0$. \end{proof} In \Cref{maintheo}, we have proved that when there is a cut edge $e$ where one end is a pendant vertex, then removing $e$ induces a complete Betti splitting. We can now use this result to derive complete Betti splittings for more general types of edges. \begin{theorem}\label{singlefreevertex} Let $e = \{u,v\} \in E(G)$ be a cut-edge where $v$ is a free vertex in $G\setminus e$. Then \begin{enumerate} \item $J_G = J_{G\setminus e}+\langle f_e\rangle$ is a complete Betti splitting, and \item $\beta_{i,j}(J_G) = \beta_{i,j}(J_{G\symbol{92}e}) + \beta_{i-1,j-2}(J_{(G\setminus e)_e})$ for all $i\geq 1$ and $j \geq 0$. \end{enumerate} \end{theorem} \begin{proof} First note that if we can prove $(2)$, then $(1)$ will follow. To see why, it is immediate that $\beta_{0,j}(J_G) = \beta_{0,j}(J_{G\setminus e}) + \beta_{0,j}(\langle f_e \rangle) +\beta_{-1,j}(J_{G\setminus e} \cap \langle f_e \rangle)$ for all $j \geq 0$. If $i \geq 1$, then \begin{eqnarray*} \beta_{i,j}(J_G) &=& \beta_{i,j}(J_{G\symbol{92}e}) + \beta_{i-1,j-2}(J_{(G\setminus e)_e}) \\ & = & \beta_{i,j}(J_{G\setminus e}) + \beta_{i,j}(\langle f_e \rangle) + \beta_{i-1,j}(J_{G \setminus e} \cap \langle f_e \rangle) \end{eqnarray*} where we are using \Cref{lemma 2.18} and the fact that $\beta_{i,j}(\langle f_e \rangle) = 0$ for all $i \geq 1$. Now note that to prove to $(2)$, we can pass to quotient rings and prove that $$\beta_{i,j}(R/J_G) = \beta_{i,j}(R/J_{G\setminus e}) + \beta_{i-1,j-2}(R/J_{(G\setminus e)_e} ) ~~\mbox{for all $i \geq 2$ and $j \geq 0$}.$$ Let $G$ be a connected graph with cut-edge $e = \{u,v\}$. Let $G_1$ and $G_2$ be the connected components of $G\setminus e$, and suppose $u\in V(G_1)$ and $v\in V(G_2)$. By our hypotheses, the vertex $v$ is a free vertex in $G_2$. Hence, we can see that $G$ is a decomposable graph, with decomposition $G = (G_1\cup \{e\}) \cup_v G_2$ (since pendant vertices are free vertices and $v$ is a pendant vertex of $e$). By \Cref{freevertexbetti} we have \begin{equation}\label{5.21} \beta_{i,j}(R/J_G) = \sum_{\substack{0 \leq i_1\leq i \\ ~0 \leq j_1\leq j}}\beta_{i_1,j_1}(R/J_{G_1\cup \{e\}})\beta_{i-i_1,j-j_1}(R/{J_{G_2}}). \end{equation} Since $e$ is a cut-edge with a pendant vertex in $G_1 \cup \{e\}$, we can now apply \Cref{maintheo} to $R/J_{G_1 \cup \{e_1\}}$. Thus, \begin{multline}\label{1.2} \sum_{\substack{0 \leq i_1\leq i \\0 \leq j_1\leq j}}\beta_{i_1,j_1}(R/{J_{G_1\cup \{e\}}})\beta_{i-i_1,j-j_1}(R/{J_{G_2}}) = \\ \sum_{\substack{2\leq i_1\leq i \\ 0 \leq j_1\leq j}}(\beta_{i_1,j_1}(R/{J_{G_1}}) + \beta_{i_1-1,j_1-2}(R/{J_{(G_1)_e}}))\beta_{i-i_1,j-j_1}(R/{J_{G_2}}) \\ + (\beta_{1,2}(R/{J_{G_1}})+ 1)\beta_{i-1,j-2}(R/{J_{G_2}}) + \beta_{i,j}(R/{J_{G_2}}). \end{multline} Here, we are using the fact that $\beta_{1,j}(R/J_{G_1 \cup \{e\}}) =0$ if $j \neq 2$, and when $j=2$, $J_{G_1 \cup \{e\}}$ has one more generator than $J_{G_1}$, that is, $\beta_{1,2}(R/J_{G_1 \cup \{e\}}) = \beta_{1,2}(R/J_{G_1})+1$. By expanding out and regrouping, we get \footnotesize \begin{align} \label{1.3} \beta_{i,j}(J_G) =& \sum_{ \substack{1\leq i_1\leq i \\ 0\leq j_1\leq j}}\beta_{i_1,j_1}(R/{J_{G_1}})\beta_{i-i_1,j-j_1}(R/{J_{G_2}}) + \beta_{i,j}(R/{J_{G_2}}) \nonumber\\ & + \sum_{\substack{2\leq i_1\leq i \\ 0 \leq j_1\leq j}}\beta_{i_1-1,j_1-2}(R/{J_{(G_1)_e}})\beta_{i-i_1,j-j_1}(R/{J_{G_2}}) +\beta_{i-1,j-2}(R/{J_{G_2}}) \nonumber\\ =& \sum_{ \substack{0 \leq i_1\leq i \\ 0 \leq j_1\leq j}}\beta_{i_1,j_1}(R/{J_{G_1}})\beta_{i-i_1,j-j_1}(R/{J_{G_2}})+ \sum_{\substack{0 \leq i_1\leq i-1 \\ 0 \leq j_1\leq j-2}}\beta_{i_1,j_1}(R/{J_{(G_1)_e}})\beta_{i-1-i_1,j-2-j_1}(R/{J_{G_2}}). \end{align} \normalsize Since $G_1$ and $G_2$ are graphs on disjoint sets of vertices, $J_{G_1}$ and $J_{G_2}$ are ideals on disjoint sets of variables. Hence, \begin{align}\label{1.4} \sum_{\substack{0\leq i_1\leq i \\ 0\leq j_1\leq j}}\beta_{i_1,j_1}(R/{J_{G_1}})\beta_{i-i_1,j-j_1}(R/{J_{G_2}}) & = \beta_{i,j}(R/{J_{G_1}+J_{G_2}}) \nonumber \\ &=\beta_{i,j}(R/{J_{G_1\cup G_2}}) = \beta_{i,j}(R/{J_{(G\setminus e)}}). \end{align} Similarly, the same is true for $(G_1)_e$ and $G_2$. Note, that since $v$ is already a free vertex of $G_2$, we have $(G\setminus e)_e = (G_1)_e \cup G_2$. Hence, \begin{align}\label{1.5} \sum_{\substack{0 \leq i_1\leq i-1 \\ 0 \leq j_1\leq j-2}}\beta_{i_1,j_1}(R/{J_{(G_1)_e}})\beta_{i-1-i_1,j-2-j_1}(R/{J_{G_2}}) & = \beta_{i-1,j-2}(R/{J_{(G_1)_e}+J_{G_2}}) \nonumber\\ & = \beta_{i-1,j-2}(R/{J_{(G_1)_e\cup G_2}}) \nonumber \\ & = \beta_{i-1,j-2}(R/{J_{(G\setminus e)_e}}). \end{align} Thus, substituting \Cref{1.5} with \Cref{1.4} into \Cref{1.3}, we get the desired conclusion. \end{proof} Because we have a complete Betti splitting, \Cref{regprojbounds} implies the collorary. \begin{corollary}\label{singlevertexcor} With the hypotheses as in \Cref{singlefreevertex}, \begin{eqnarray*} {\rm reg}(J_G) &= &\max\{{\rm reg}(J_{G\setminus e}), {\rm reg}((J_{G \setminus e})_e) +1\} ~~\mbox{and} \\ {\rm pd}(J_G) &= &\max\{{\rm pd}(J_{G\setminus e}), {\rm pd}(J_{(G \setminus e)_e}) +1\}. \end{eqnarray*} \end{corollary} \begin{proof} Because $J_G = J_{G\setminus e} + \langle f_e \rangle$ is a complete Betti splitting, \Cref{regprojbounds} gives \begin{eqnarray*} {\rm reg}(J_G) &= &\max\{{\rm reg}(J_{G\setminus e}), {\rm reg}(\langle f_e \rangle), {\rm reg}(J_{G \setminus e} \cap \langle f_e \rangle) -1\} ~~\mbox{and} \\ {\rm pd}(J_G) &= &\max\{{\rm pd}(J_{G\setminus e}), {\rm pd}(\langle f_e \rangle), {\rm pd}(J_{G \setminus e} \cap \langle f_e \rangle) +1\}. \end{eqnarray*} The result now follows since $2 = {\rm reg}(\langle f_e \rangle) \leq {\rm reg}(J_{G \setminus e})$ and $0 = {\rm pd}(\langle f_e \rangle)$ and because \Cref{lemma 2.18} implies ${\rm reg}(J_{G \setminus e} \cap \langle f_e \rangle) = {\rm reg}(J_{(G\setminus e)_e}) +2$ and ${\rm pd}(J_{G \setminus e} \cap \langle f_e \rangle) = {\rm pd}(J_{(G \setminus e)_e})$. \end{proof} Recall that an edge $e = \{u,v\}$ is a free cut-edge of $G$ if both $u$ and $v$ are free vertices of $G \setminus e$. When \Cref{singlefreevertex} is applied to a free cut-edge, we can recover the following results of Saeedi Madani and Kiani \cite{kiani_regularity_2013-1}. \begin{corollary}[{\cite[Proposition 3.4]{kiani_regularity_2013-1}}] \label{freecutedge} Let $e = \{u,v\} \in E(G)$ be a free cut-edge. Then \begin{enumerate} \item $\beta_{i,j}(J_G) = \beta_{i,j}(J_{G\setminus e}) + \beta_{i-1,j-2}(J_{G\setminus e})$, \item \rm pd($J_G$) = pd($J_{G\setminus e}) + 1$, and \item \rm reg($J_G$) = reg($J_{G\setminus e}$) + 1. \end{enumerate} \end{corollary} \begin{proof} When $e$ is a free cut-edge of $G$, then $(G\setminus e)_e = G\setminus e$. The results then follow from \Cref{singlefreevertex} and \Cref{singlevertexcor} by using the equality $J_{(G\setminus e)_e} = J_{G\setminus e}.$ \end{proof} One application of \Cref{maintheo} is finding the Betti numbers of the binomial edge ideals of certain graphs. The corollary below is a new proof of \cite[Proposition 3.8]{jayanthan_almost_2021} for the graded Betti numbers of the binomial edge ideals of any star graph $S_n$. \begin{corollary}\label{star} Let $S_n$ denote the star graph on $n$-vertices. Then we have: \[ \beta_{i}(J_{S_n}) = \beta_{i,i+3}(J_{S_n}) = i\binom{n}{i+2} \text{\hspace{4mm} $i\geq 1$}. \] Furthermore, $\beta_0(J_{S_n}) = \beta_{0,2}(S_n) = n-1$. \end{corollary} \begin{proof} Note that the statement about $0$-th graded Betti numbers just follows from the fact that $S_n$ has $n-1$ edges. Consider the edge $e =\{1,n\}$. Since $S_n\setminus e = S_{n-1} \cup \{n\}$, we have $(S_n\setminus e)_e = K_{n-1} \cup \{n\}$. So from \Cref{maintheo}, we have: \[\beta_{i,j}(J_{S_n}) = \beta_{i,j}(J_{S_{n-1}})+\beta_{k-1,j-2}(J_{K_{n-1}}) ~~\text{ for all $i\geq 1$}.\] We can now use induction to show the above assertion. For $n = 2$, we can see that $S_2$ is just an edge. We know that $\beta_{i,j}(J_{S_2}) = 0$ for all $i\geq 1$. Hence, we can see that it agrees with the above formula as $\binom{2}{r} = 0$ when $r>2$. Now assume the formula holds for $n-1$. We must show that it holds for $n$. From \Cref{completebetti}, we know that $\beta_{i,i+2}(J_{K_{n-1}}) = (i+1)\binom{n-1}{i+2}$ and $\beta_{i,j}(J_{K_{n-1}}) = 0$ if $j\neq i+2$. Hence, using induction and \Cref{maintheo}, we can see that $\beta_{i,j}(J_{S_n}) = \beta_{i,j}(J_{S_{n-1}})+\beta_{i-1,j-2}(J_{K_{n-1}})=0+0$, when $j\neq i+3$. We also have \[\beta_{i,i+3}(J_{S_n}) = \beta_{i,i+3}(J_{S_{n-1}})+\beta_{i-1,i+1}(J_{K_{n-1}}) = i\binom{n-1}{i+2}+i\binom{n-1}{i+1} = i\binom{n}{i+2}.\] This verifies the formula of the statement. \end{proof} \section{Partial Betti splittings of binomial edge ideals: \texorpdfstring{$s$}{s}-partitions} In this section we consider the other natural candidate to study in the context of partial Betti splittings. In this case, we fix a vertex $s \in V)$, and let $G_1$ be the graph with $E(G_1)$ equal to the set of edges of $G$ that contain $s$ (so $G_1$ is isomorphic to a star graph) and $G_2 = G \setminus \{s\}$. We formalize this idea in the next definition. \begin{definition}\label{vpart} For $s\in V(G)$, an {\it $s$-partition} of $J_G$ is the splitting $J_G = J_{G_1}+J_{G_2},$ where $G_1$ is the subgraph of $G$ with $V(G_1) = N_G[s]$ and $E(G_1) = \{\{s,k\}\mid k\in N_G(s)\}$, and $G_2=G\setminus \{s\}$. \end{definition} Note that the graph $G_1$ in an $s$-partition is isomorphic to the star graph $S_{\deg(s)+1}$. We will show that an $s$-partition always gives a partial Betti splitting of $J_G$: \begin{theorem}\label{maintheo2} Let $G$ be a graph on $[n]$ and let $J_G = J_{G_1}+J_{G_2}$ be an $s$-partition of $G$ for some $s\in [n]$. Let $c(s)$ be the size of the largest clique containing $s$. Then, for all $i, j$ with $i \geq c(s)$ or $j \geq i+4$, \begin{equation*} \beta_{i,j}(J_G) = \beta_{i,j}(J_{G_1})+\beta_{i,j}(J_{G_2})+\beta_{i-1, j}(J_{G_1}\cap J_{G_2}). \end{equation*} In other words, $J_G = J_{G_1}+J_{G_2}$ is a $(c(s), 4)$-Betti splitting. \end{theorem} Our proof hinges on a careful examination of $J_{G_2} \cap J_{G_2}$, which is carried out below. \begin{lemma}\label{deg3gen} Let $G$ be a graph on $[n]$ and let $J_G = J_{G_1}+J_{G_2}$ be an $s$-partition of $G$ for some $s\in [n]$. Then the set \[ \mathcal{B} = \{x_sf_{a,b}, y_sf_{a,b}\mid a,b\in N_G(s) \text{ and } \{a,b\}\in E(G)\}.\] is a $k$-basis for $(J_{G_1} \cap J_{G_2})_3$. \end{lemma} \begin{proof} Let $N_G(s) = \{v_1,\dots, v_r\}$. Since $E(G_1) \cap E(G_2) = \emptyset$, the generators of $J_{G_1} \cap J_{G_2}$ are of degree at least $3$. First of all observe that $\B_1 = \{x_if_e, y_if_e\mid e \in E(G_1) \text{ and } i\in \{1, \dots, n\}\}$ and $\B_2 = \{x_if_e, y_if_e\mid e\in E(J_{G_2}) \text{ and } i\in \{1, \dots, n\}\}$ form $k$-bases for the subspaces $(J_{G_1})_3$ and $(J_{G_2})_3$ respectively. Let $P \in (J_{G_1} \cap J_{G_2})_3 = (J_{G_1})_3 \cap (J_{G_2})_3$. Write \begin{equation}\label{eq.P} P = \sum_{g_{i,e}\in \B_1}c_{i,e} g_{i,e}, \end{equation} where $c_{i,e} \in k$. We first claim that the coefficients of $x_if_{a,s}$ and $y_if_{a,s}$ in the linear combination of $P$ are zero if $i \notin \{v_1,\ldots, v_r\}$. We prove this for $x_if_{a,s}$ and the other proof is similar. Let $c$ be the coefficient of $x_if_{a,s}$. Observe that, since $i\notin \{v_1,\dots, v_k\}$, the term $y_sx_ix_a$ in $P$, appears in only one basis element, namely $x_if_{a,s}$. Since $P$ is in $(J_{G_2})_3$ as well, we can write \begin{equation}\label{2.8} P = S+ y_s(c x_ix_a+L) = Q + y_s\left(\sum_{f_e\in \mathfrak{G}(J_{G_2})}c'_e f_e\right), \end{equation} where no terms of $S$ and $Q$ are divisible by $y_s$ and $L$ does not have any monomial terms divisible by $x_ix_a$. Since $y_s$ does not divide any term of $S$ and $Q$, the above equality implies that $c x_ix_a+L = \sum_{f_e\in \mathfrak{G}(J_{G_2})}c'_e f_e$. Now by considering the grading on $R$ given by $\deg x_j = (1,0)$ and $\deg y_j = (0,1)$ for all $j$, we can see that $x_ix_a$ is of degree $(2,0)$ but the degree of each term $f_e$ in $\mathfrak{G}(J_{G_2})$ is $(1,1)$. Hence, for \Cref{2.8} to hold, $c=0$. This completes the proof of the claim. Now consider the case where $i\in \{v_1,\dots, v_k\}$. In this case, it can be seen that the term $y_sx_ix_a$ when written as an element of $(J_{G_1})_3$ appears in the basis elements $x_if_{a,s}$ and $x_af_{i,s}$, and in no other basis element. As before, to make sure that there are no elements of degree $(2,0)$, the coefficients of $x_if_{a,v}$ and $x_af_{i,s}$ in \Cref{eq.P} must be additive inverses of each other. Denote the coefficient of $x_if_{a,s}$ by $c$. Then, $$cx_if_{a,s} - cx_af_{i,s} = cx_s(x_ay_i-x_iy_a) = cx_sf_{a,i}.$$ Similar arguments show that the coefficients of $y_if_{a,s}$ and $y_af_{i,s}$ must be additive inverses of each other, and that the corresponding linear combination in the \Cref{eq.P} appears as $c'y_sf_{a,i}$. Therefore, \Cref{eq.P} becomes \[P = \sum_{a,i\in N_G(s)}c_{i,a} x_sf_{a,i}+c'_{i,a} y_sf_{a,i}.\] Since $P \in (J_{G_2})_3$, it is easily observed that $c_{i,a} = 0$ whenever $\{i,a\} \notin E(G)$. Therefore, $\mathcal{B}$ spans the subspace $(J_{G_1} \cap J_{G_2})_3$. Linear independence is fairly straightforward as $s \neq a, b$ for any $a, b \in N_G(s)$. Hence the assertion of the lemma is proved. \end{proof} \begin{remark}\label{deg4} If $G$ is a triangle-free graph, then there does not exist any $a,b\in N_G(s)$ with $\{a,b\}\in E(G)$ for any $s\in V(G)$. Hence it follows from \Cref{deg3gen} that there are no degree 3 generators of $J_{G_1}\cap J_{G_2}$ for any $s$-partition. Hence, $J_{G_1} \cap J_{G_2}$ is generated by elements of degrees $4$ or higher. \end{remark} Since the generators of $J_{G_1}\cap J_{G_2}$ resemble the generators of a binomial edge ideal, we can calculate its linear strand in terms of the linear strand of some binomial edge ideal. \begin{theorem}\label{thm:Betti-intersection} Let $G$ be a graph on $[n]$ and let $J_G = J_{G_1}+J_{G_2}$ be an $s$-partition of $G$ for some $s\in [n]$. If $G'$ is the induced subgraph of $G$ on $N_G(s)$, then \[\beta_{i,i+3}(J_{G_1}\cap J_{G_2}) = 2\beta_{i,i+2}(J_{G'})+\beta_{i-1,i+1}(J_{G'})\text{\hspace{2mm} for all $i\geq 0$}.\] \end{theorem} \begin{proof} From \Cref{deg3gen}, we have that the minimal degree 3 generators for $J_{G_1}\cap J_{G_2}$ are \[L =\{x_sf_{a,b}, y_sf_{a,b}\mid a,b\in N_G(s) \text{ and } \{a,b\}\in E(G)\}.\] Since, $J_{G_1}\cap J_{G_2}$ is generated in degree 3 or higher, if $I$ is the ideal generated by $L$, then $\beta_{i,i+3}(J_{G_1}\cap J_{G_2}) = \beta_{i,i+3}(I)$ for all $i \geq 0$. Now consider the partition $I = I_x+I_y$, where $$ \mathfrak{G}(I_x) = \{x_sf_{a,b}\mid \text{ $\{a,b\}\in E(G')$}\} ~\mbox{and} ~ \mathfrak{G}(I_y) = \{y_sf_{a,b}\mid \text{$\{a,b\}\in E(G')$}\}. $$ We now claim that \[I_x\cap I_y = \langle\{x_sy_sf_{a,b}\mid \text{$\{a,b\}\in E(G')$}\}\rangle.\] It is clear that each $x_sy_sf_{a,b} \in I_x\cap I_y$. For the other inclusion, consider $g\in I_x\cap I_y$. Since $g$ is in both $I_x$ and $I_y$, we can write $g$ as \[g = x_s\left(\sum k_{a,b}f_{a,b}\right) = y_s\left(\sum k'_{a,b}f_{a,b}\right),\] where $k_{a,b}, k'_{a,b} \in R$. Since, none of the $f_{a,b}$'s involve the variables $x_s$ and $y_s$, some terms of $k_{a,b}$ are divisible by $y_s$, for each $\{a,b\}\in E(G')$. Separating out the terms which are divisible by $y_s$, write: \[g = x_s\left(\sum k_{a,b}f_{a,b}\right) = x_s\left(\sum y_sh_{a,b}f_{a,b}+L\right),\] where no term of $L$ is divisible by $y_s$. Since $g$ is divisible by $y_s$, we have that $y_s|L$. But since no term of $L$ is divisible by $y_s$, this implies that $L=0$. Hence, $$g = x_sy_s\left(\sum h_{a,b}f_{a,b}\right)\in \langle\{x_sy_sf_{a,b}\mid \text{$\{a,b\}\in E(G')$}\}\rangle.$$ It is readily seen that $J_{G'}\xrightarrow{\cdot x_s} I_x$, $J_{G'}\xrightarrow{\cdot y_s} I_y$, and $J_{G'}\xrightarrow{\cdot x_sy_s} I_x\cap I_y$ are isomorphisms of degree 1, 1, and 2 respectively. Now, consider $\mathbb{N}^n$ multigrading on $R$ with $\deg x_i = \deg y_i = e_i$ for all $i=1,\ldots, n$. The above isomorphisms imply that: \[\tor_i(I_x,k)_{\mathbf{a}+e_s}\cong \tor_i(J_{G'},k)_{\mathbf{a}} \cong \tor_i(I_y,k)_{\mathbf{a}+e_s} \] and $$\tor_i(I_x\cap I_y,k)_{\mathbf{a}+2e_s}\cong \tor_i(J_{G'},k)_{\mathbf{a}},$$ where $\mathbf{a} = (a_1,\ldots,a_n) \in \mathbb{N}^n$ with $a_s=0$. Summing up all the multigraded Betti numbers, we get $\beta_{i,j}(I_x) = \beta_{i,j-1}(J_{G'}) = \beta_{i,j}(I_y) $ and $\beta_{i,j}(I_x\cap I_y) = \beta_{i,j-2}(J_{G'})$. Observe that all the non-zero multigraded Betti numbers of $I_x\cap I_y$ occur only on multidegrees $\mathbf{a}+2e_s$ while all Betti numbers of $I_x$ and $I_y$ occur only at $\mathbf{a}+e_s$. Hence, by using \Cref{parcon} and combining all multidegrees, we have $$\beta_{i,j}(I) = \beta_{i,j}(I_x)+\beta_{i,j}(I_y)+\beta_{i-1,j}(I_x\cap I_y) ~~\mbox{for all $i,j \geq 0$}.$$ Therefore, \[\beta_{i,i+3}(J_{G_1}\cap J_{G_2}) = \beta_{i,i+3}(I) = \beta_{i,i+2}(J_{G'})+\beta_{i,i+2}(J_{G'})+\beta_{i-1,i+1}(J_{G'})\] for all $i \geq 0$. \end{proof} We can now prove the main result of this section: \begin{proof}[Proof of \Cref{maintheo2}] We first prove that $\beta_{i,i+3}(J_{G_1}\cap J_{G_2}) = 0$ for all $i\geq c(s)-1$, since we will require this fact later in the proof. It follows from \Cref{thm:Betti-intersection} that for all $i \geq 0$ \[\beta_{i,i+3}(J_{G_1}\cap J_{G_2}) = 2\beta_{i,i+2}(J_{G'})+\beta_{i-1,i+1}(J_{G'}),\] where $G'$ is the induced subgraph of $G$ on $N_G(s)$. From \Cref{linearbinom}, we get $\beta_{i,i+2}(J_{G'}) = (i+1)f_{i+1} (\Delta(G'))$, where $f_k(\Delta(G'))$ is the number of faces of $\Delta(G')$ of dimension $k$. Since the largest clique in $G'$ is of size $c(s)-1$, $\beta_{i,i+2}(J_{G'}) = 0$ for all $i\geq c(s)-2$. Hence $\beta_{i,i+3}(J_{G_1}\cap J_{G_2}) = 0$ for all $i\geq c(s)-1$ by the above formula. Consider the $\mathbb{N}^n$-grading on $R$ given by $\deg x_i = \deg y_i = e_i$, the $i$-th unit vector. Now fix any $i \geq 1$ and let ${\bf a} = (a_1,\ldots,a_n) \in \mathbb{N}^n$ with $\sum_{\ell=1}^n a_\ell \geq i+ 4$. All the generators of $J_{G_1}\cap J_{G_2}$ are of the form $fx_s+gy_s$, so their multigraded Betti numbers occur within multidegrees $\mathbf{a}$ such that its $s$-th component, $a_s$ is non-zero. Since $J_{G_2}$ contains no generators of the form $fx_s+gy_s$, $\beta_{i,{\bf a}}(J_{G_1}\cap J_{G_2})>0$ implies that $\beta_{i,{\bf a}}(J_{G_2}) = 0$ for all $i\in \mathbb{N}$, and similarly, $\beta_{i-1,{\bf a}}(J_{G_1} \cap J_{G_2}) > 0$ implies that $\beta_{i,{\bf a}}(J_{G_2}) = 0$ From \Cref{star}, since $G_1$ is a star graph, \[ \beta_{i}(J_{G_1}) = \beta_{i,i+3}(J_{G_1}) = i\binom{\deg(s)}{i+2} ~\mbox{for all $i\geq 1$}.\] Hence, we can see that for all multidegrees ${\bf a} = (a_1,\dots,a_n)$ with $\sum_{\ell=1}^n a_\ell\geq i+4$, we also have $\beta_{i,{\bf a}}(J_{G_1}\cap J_{G_2})>0$ implies that $\beta_{i,{\bf a}}(J_{G_1})=0$, and $\beta_{i-1,{\bf a}}(J_{G_1}\cap J_{G_2})>0$ implies that $\beta_{i-1,{\bf a}}(J_{G_1})=0$. Therefore, from \Cref{parcon}, we have \[\beta_{i,{\bf a}}(J_G) = \beta_{i,{\bf a}}(J_{G_1})+ \beta_{i,{\bf a}}(J_{G_2})+ \beta_{i-1, {\bf a}}(J_{G_1}\cap J_{G_2}),\] for all $i \geq 0$ and multidegrees ${\bf a}$ with $\sum_{\ell=1}^n a_\ell\geq i+4$. Now fix any $i \geq c(s)$ and ${\bf a} \in \mathbb{N}^n$. As argued above, if $\beta_{i,{\bf a}}(J_{G_1} \cap J_{G_2})>0$, then $\beta_{i,{\bf a}}(J_{G_2}) = 0$ (and a similar statement for $\beta_{i-1,{\bf a}}(J_{G_1} \cap J_{G_2})$). We also know that if $\beta_{i,{\bf a}}(J_{G_1} \cap J_{G_2}) > 0$, with $i \geq c(s)-1$, then $\sum_{\ell=1}^n a_l \geq i+4$ since $J_{G_1} \cap J_{G_2}$ is generated in degree three and $\beta_{i,i+3}(J_{G_1}\cap J_{G_2}) =0$ for all $i \geq c(s)-1$. On the other hand, since ${\rm reg}(J_2) = 3$ by \Cref{star}, we have $\beta_{i,{\bf a}}(J_{G_2}) = 0$ for all $\sum_{\ell=1}^n a_\ell \neq i+3$ if $i \geq 1$. So, we have shown that if $\beta_{i,{\bf a}}(J_{G_1} \cap J_{G_2}) > 0$, then $\beta_{i,{\bf a}}(J_{G_2}) = 0$, and also if $\beta_{i-1,{\bf a}}(J_{G_1} \cap J_{G_2}) > 0$, then $\beta_{i-1,{\bf a}}(J_{G_2}) = 0$. So by using \Cref{parcon}, we have \[\beta_{i,{\bf a}}(J_G) = \beta_{i,{\bf a}}(J_{G_1})+ \beta_{i,{\bf a}}(J_{G_2})+ \beta_{i-1, {\bf a}}(J_{G_1}\cap J_{G_2}),\] for all $i \geq c(s)$ and multidegrees ${\bf a} \in \mathbb{N}^n$. Therefore, by combining these two results we have \[\beta_{i,{\bf a}}(J_G) = \beta_{i,{\bf a}}(J_{G_1})+ \beta_{i,{\bf a}}(J_{G_2})+ \beta_{i-1,{\bf a}}(J_{G_1}\cap J_{G_2}),\] for all $i$ and multidegrees ${\bf a}$ with $i\geq c(s)$ or $\sum_{k=1}^n a_k\geq i+4$. By summing over all multidegrees, we obtain the same result for the standard grading, i.e., $$\beta_{i,j}(J_G) = \beta_{i,j}(J_{G_1})+ \beta_{i,j}(J_{G_2})+ \beta_{i-1, j}(J_{G_1}\cap J_{G_2}),$$ for all $i,j$ with $i\geq c(s)$ or $j\geq i+4$. In other words, we have a $(c(s),4)$-Betti splitting. \end{proof} \begin{example} If $G$ is the graph of \Cref{runningexample}, then we saw in \Cref{runningexample2} that the ideal $J_G$ has a $(4,4)$-Betti splitting. Note that the splitting of \Cref{runningexample2} is an example of an $s$-partition with $s=1$. Furthermore, the largest clique that the vertex $s=1$ belongs to has size four (there is a clique on the vertices $\{1,2,4,5\})$. So, by the previous result $J_G$ will have a $(c(1),4)$-Betti splitting with $c(1)=4$, as shown in this example. \end{example} \begin{corollary}\label{trianglefree} Let $G$ be a graph on $[n]$ and let $J_G = J_{G_1}+J_{G_2}$ be an $s$-partition of $G$ for some $s\in [n]$. If $G$ is a triangle-free graph, then $J_G = J_{G_1}+J_{G_2}$ is a complete Betti splitting. \end{corollary} \begin{proof} Since $G$ is a triangle-free graph, the largest clique containing $s$ is a $K_2$, i.e., $c(s)=2$. Thus \Cref{maintheo2} implies that $J_G = J_{G_1}+J_{G_2}$ is a $(2,4)$-Betti splitting, that is, $$\beta_{i,j}(J_G) = \beta_{i,j}(J_{G_1})+\beta_{i,j}(J_{G_2})+\beta_{i-1, j}(J_{G_1}\cap J_{G_2} )\text{ for all $i\geq 2$ or $j \geq i +4$.}$$ To complete the proof, we just need to show the above formula also holds for the graded Betti numbers $\beta_{i,j}(J_G)$ with $(i,j) \in \{(0,0),(0,1),(0,2),(0,3),(1,1), (1,2),(1,3),(1,4)\}$. We always have $\beta_{0,j}(J_G) = \beta_{0,j}(J_{G_1})+\beta_{0,j}(J_G) + \beta_{-1,j}(J_{G_1}\cap J_{G_2})$ for all $j \geq 0$. Also, since $J_G, J_{G_1}$ and $J_{G_2}$ are generated in degree $2$ and $J_{G_1} \cap J_{G_2}$ generated in degree four (by \Cref{deg4}), we have $$0 = \beta_{1,j}(J_G) = \beta_{1,j}(J_{G_1})+\beta_{1,j}(J_G) + \beta_{0,j}(J_{G_1}\cap J_{G_2}) = 0 + 0 + 0$$ for $j=1,2$. Finally, because $J_{G_1} \cap J_{G_2}$ is generated in degree four, we have $\beta_{1,3}(J_{G_1}\cap J_{G_2}) = \beta_{1,4}(J_{G_1}\cap J_{G_2}) = 0$. Thus, for $(i,j) = (1,3)$ the conditions of \Cref{parcon} are vacuously satisfied (since $\beta_{1,3}(J_{G_1}\cap J_{G_2}) = \beta_{0,3}(J_{G_1}\cap J_{G_2}) = 0$). For $i=1$ and $j=4$, we have $\beta_{1,4}(J_{G_1}\cap J_{G_2}) = 0$ and when $\beta_{0,4}(J_{G_1} \cap J_{G_2}) > 0$, we have $\beta_{0,4}(J_{G_1}) = \beta_{0,4}(J_{G_2}) =0$ since both $J_{G_1}$ and $J_{G_2}$ are generated in degree 2. So again the conditions of \Cref{parcon} are satisfied. Thus $$ \beta_{1,j}(J_G) = \beta_{1,j}(J_{G_1})+\beta_{1,j}(J_{G_2}) + \beta_{1,j}(J_{G_1}\cap J_{G_2}) = \beta_{1,j}(J_{G_1})+\beta_{1,j}(J_G) $$ for $j=3,4$. \end{proof} \begin{corollary} Let $G$ be a graph on $[n]$ and let $J_G = J_{G_1}+J_{G_2}$ be an $s$-partition of $G$ for some $s\in [n]$. \begin{enumerate} \item If $\pd(J_G)\geq c(s)$, then $\pd(J_G) = \max\{ \pd(J_{G_1}), \pd(J_{G_2}), \pd(J_{G_1}\cap J_{G_2})+1\}.$ \item If $\reg(J_G)\geq 4$, then $\reg(J_G) = \max\{\reg(J_{G_2}), \reg(J_{G_1}\cap J_{G_2})-1\}.$ \end{enumerate} \end{corollary} \begin{proof} Given that $\pd(J_G)\geq c(s)$, we know that there is a partial splitting for all $\beta_{i,j}(J_G)$, for all $i\geq c(s)$. Hence, $\pd(J_G) = \max\{ \pd(J_{G_1}), \pd(J_{G_2}), \pd(J_{G_1}\cap J_{G_2})+1\}$. Similarly, if $\reg(J_G)\geq 4$, we know that there is a partial splitting for all $\beta_{i,j}(J_G)$, for all $i\geq c(s)$. Hence, $\reg(J_G) = \max\{ \reg(J_{G_1}), \reg(J_{G_2}), \reg(J_{G_1}\cap J_{G_2})-1\}$. Since $\reg(J_{G_1}) = 3$, we have $\reg(J_G) = \max\{\reg(J_{G_2}), \reg(J_{G_1}\cap J_{G_2})-1\}$. \end{proof} \section{On the total Betti numbers of binomial edge ideals of trees} In this section, we explore an application of \Cref{maintheo} to find certain Betti numbers of trees. In particular, we obtain a precise expression for the second Betti number of $J_T$ for any tree $T$. Note that $\beta_1(J_T)$ was first computed in \cite[ Theorem 3.1]{jayanthan_almost_2021}. We begin with recalling a simple technical result that we require in our main results. \begin{lemma}\label{pendantexist} Let $T$ be a tree which is not an edge with $v\in V(T)$ and let $S_v = \{u\in N_T(v) ~|~ \deg u > 1\}$. Then, there exists $a\in V(T)$ with $\deg a>1$ such that $|S_a|\leq 1.$ \end{lemma} \begin{proof} See \cite[Proposition 4.1]{JK2005}. \end{proof} To compute the second Betti number of $J_T$, we use \Cref{maintheo} to reduce the computation to graphs with a fewer number of vertices. One of the graphs involved in this process becomes a clique sum of a tree and a complete graph. So, we now compute the first Betti number of this class of graphs. \begin{theorem}\label{T+K_m} Let $G=T \cup_{a} K_m$. If $|V(G)| = n$, then \begin{eqnarray*} \beta_1(J_G) &= &\binom{n-1}{2}+2\binom{m}{3}+\sum_{w\notin V(K_m)}\binom{\deg_G w}{3}+\binom{\deg_G a-m+1}{3} \\ & &+(n-m-1)\binom{m-1}{2} +(m-1)\binom{\deg_G a -m+1}{2}. \end{eqnarray*} \end{theorem} \begin{proof} We prove the assertion by induction on $|V(T)|$. If $|V(T)| = 1$, then $G$ is a complete graph and $n = m$. Therefore, by \Cref{completebetti} \[\beta_1(J_G) = 2\binom{n}{3} = \binom{n-1}{2}+2\binom{n}{3}-\binom{n-1}{2}.\] Hence the assertion is true. Assume now that the assertion is true if $|V(T)| \leq n-m$. Let $G = T \cup_a K_m$. Since $E(T)\neq \emptyset$, it follows from \Cref{pendantexist} that there exists $u\in V(T)$ such that $\deg u\neq 1$ and $|S_u|\leq 1$. We now split the remaining proof into two cases. \noindent \textbf{Case 1:} $u\neq a$.\\ Let $e= \{u,v\}$ with $\deg_G v = 1$ and let $G' = G \setminus v$. Then $G' = (T\setminus v) \cup_a K_m$ and $J_{G'} = J_{G\setminus e}$. Note that $\deg_{G'} u = \deg_G u - 1$ and $\deg_{G'} w = \deg_G w$ for all $w \neq u$. From \Cref{maintheo}, we have $\beta_1(J_G) = \beta_1(J_{G\setminus e}) + \beta_{0}(J_{(G\setminus e)_e})$. We now compute the two terms on the right hand side of this equation. It follows by induction that \begin{eqnarray*} \beta_1(J_{G\setminus e}) &= &\binom{n-2}{2}+2\binom{m}{3}+\sum_{w\notin V(K_m), w\neq u}\binom{\deg_{G'} w}{3}+\binom{\deg_G u-1}{3}\\ & &+\binom{\deg_G a-m+1}{3}+ (n-m-2)\binom{m-1}{2} + (m-1)\binom{\deg_G a -m+1}{2}. \end{eqnarray*} Now, $(G\setminus e)_e$ is obtained by adding $\binom{\deg u-1}{2}$ edges to $E(G\setminus e)$. Since $T$ is a tree and $G=T \cup_a K_m$, we have $E(G) = n-m+\binom{m}{2}$. Hence, $G\setminus e$ has $n-m-1 + \binom{m}{2} = n-2+\binom{m-1}{2}$ edges. This means that: \[\beta_0(J_{(G\setminus e)_e}) =|E((G\setminus e)_e)| = n-2 + \binom{m-1}{2} +\binom{\deg_G u-1}{2}.\] Therefore, \begin{eqnarray*} \beta_1(J_{G}) &= & \beta_1(J_{G\setminus e}) + \beta_{0}(J_{(G\setminus e)_e}) \\ & = & \binom{n-2}{2}+2\binom{m}{3}+\sum_{w\notin V(K_m), w\neq u}\binom{\deg_G w}{3}+\binom{\deg_G u-1}{3} \\ & &+ \binom{\deg_G a-m+1}{3} + (n-m-2)\binom{m-1}{2} + (m-1)\binom{\deg_G a -m+1}{2}\\ & &+ n-2 + \binom{m-1}{2} +\binom{\deg_G u-1}{2}\\ &= & \binom{n-1}{2}+2\binom{m}{3}+\sum_{w\notin V(K_m)}\binom{\deg_G w}{3}+\binom{\deg_G a-m+1}{3}\\ & &+(n-m-1)\binom{m-1}{2} +(m-1)\binom{\deg_G a -m+1}{2}. \end{eqnarray*} Therefore, we obtain our desired formula. \noindent \textbf{Case 2:} $u=a$. \noindent Let $e= \{a,v\}$ with $\deg v = 1$. Then, as before, we apply induction to get \begin{eqnarray*} \beta_1(J_{G\setminus e}) &= & \binom{n-2}{2}+2\binom{m}{3}+\sum_{w\notin V(K_m)}\binom{\deg_G w}{3}+ \binom{\deg_G a-m}{3}\\ & &+ (n-m-2)\binom{m-1}{2}+(m-1)\binom{\deg_G a -m}{2}. \end{eqnarray*} There are $\binom{\deg_G a-m}{2}+(m-1)\binom{\deg_G a-m}{1}$ new edges in $(G\setminus e)_e$. Thus \[\beta_0(J_{(G\setminus e)_e}) = |E(G\setminus e)_e| = n-2+\binom{m-1}{2}+\binom{\deg_G a-m}{2} + (m-1)\binom{\deg_G a-m}{1}.\] Using \Cref{maintheo} and the identity $\binom{n}{r} = \binom{n-1}{r}+\binom{n-1}{r-1}$ appropriately, we get: \begin{eqnarray*} \beta_1(J_{G}) & = & \binom{n-2}{2}+2\binom{m}{3}+\sum_{w\notin V(K_m)}\binom{\deg_G w}{3}+ \binom{\deg_G a-m}{3}\\ & &+ (n-m-2)\binom{m-1}{2}+(m-1)\binom{\deg_G a -m}{2}\\ & &+ n-2+\binom{m-1}{2}+\binom{\deg_G a-m}{2} + (m-1)\binom{\deg_G a-m}{1} \\ & = & \binom{n-1}{2}+2\binom{m}{3}+\sum_{w\notin V(K_m)}\binom{\deg_G w}{3}+\binom{\deg_G a-m+1}{3}\\ & & +(n-m-1)\binom{m-1}{2} +(m-1)\binom{\deg_G a -m+1}{2}. \end{eqnarray*} Thus, we get the desired formula. This completes the proof. \end{proof} As an immediate consequence, we recover \cite[ Theorem 3.1]{jayanthan_almost_2021}: \begin{corollary} Let $T$ be a tree on $[n]$. Then \[ \beta_1(J_T) = \binom{n-1}{2}+\sum_{w \in V(T)}\binom{\deg_T w}{3}. \] \end{corollary} \begin{proof} If $G = T$, it can be trivially written as $G = T\cup_a K_1$, where $V(K_1) = \{a\}$. Therefore, taking $m=1$ in \Cref{T+K_m} we get the desired formula. \end{proof} We now compute the second Betti number of a tree using \Cref{T+K_m} and \Cref{maintheo}. This Betti number also depends upon the number of induced subgraphs isomorphic to the following caterpillar tree. We first fix the notation for this graph. \begin{definition} Let $P$ be the graph with $V(P)=[6]$ and $E(P) = \{\{1,2\}, \{2,3\},\\ \{3,4\}, \{2,5\}, \{3,6\} \}$. Given a tree $T$, we define $\mathcal{P}(T)$ to be the collection of all subgraphs of $T$ which are isomorphic to $P$, as shown in \Cref{fig:graph6}. Let $P(T) = |\mathcal{P}(T)|$. \end{definition} \begin{figure}[ht] \centering \begin{tikzpicture}[every node/.style={circle, draw, fill=white!60, inner sep=1.5pt}, node distance=2cm] \node (1) at (0, 0) {1}; \node (2) at (1, 0) {2}; \node (3) at (2, 0) {3}; \node (4) at (3, 0) {4}; \node (5) at (1, -1) {5}; \node (6) at (2, 1) {6}; \draw (1) -- (2); \draw (2) -- (3); \draw (3) -- (4); \draw (2) -- (5); \draw (3) -- (6); \end{tikzpicture} \caption{The graph $P$} \label{fig:graph6} \end{figure} \begin{example}\label{ex:pt} Consider the graph $G$ of \Cref{fig:example of P} with $V(G) = [7]$ and $$E(G) = \{\{1,2\}, \{2,3\}, \{3,4\}, \{2,5\},\\ \{3,6\}, \{3,7\}\}.$$ For this graph, the collection $\mathcal{P}(G)$ will be the induced subgraphs on the following collections of vertices: $\mathcal{P}(G)=\{\{1,2,3,4,5,6\}, \{1,2,3,5,6,7\}, \{1,2,3,4,5,7\}\}$. Hence, $P(G)=3$. \begin{figure}[ht] \centering \begin{tikzpicture}[every node/.style={circle, draw, fill=white!60, inner sep=1.5pt}, node distance=2cm] \node (1) at (0, 0) {1}; \node (2) at (1, 0) {2}; \node (3) at (2, 0) {3}; \node (4) at (3, 0) {4}; \node (5) at (1, -1) {5}; \node (6) at (2, 1) {6}; \node (7) at (2, -1) {7}; \draw (1) -- (2); \draw (2) -- (3); \draw (3) -- (4); \draw (2) -- (5); \draw (3) -- (6); \draw (3) -- (7); \end{tikzpicture} \caption{The graph $G$} \label{fig:example of P} \end{figure} \end{example} \begin{theorem}\label{betti2tree} Let $T$ be a tree on $[n]$, and let $J_T$ be its binomial edge ideal. Then \[\beta_2(J_T) = \binom{n-1}{3}+ 2\sum_{w \in V(T)}\binom{\deg_T w}{4}+\sum_{w \in V(T)}\binom{\deg_T w}{3}(1+|E(T\setminus w)|)+P(T).\] \end{theorem} \begin{proof} We prove the assertion by induction on $n$. If $n=2$, then $T$ is an edge. Since $J_T$ is a principal ideal, we have $\beta_{2}(J_T) = 0$, which agrees with the above formula. Now, assume that $n > 2$ and that the above formula is true for trees with $V(T)\leq n-1$. Let $T$ be a tree with $|V(T)|=n$. We know from \Cref{pendantexist} that there exists a vertex $u$ such that $\deg u>1$ and $|S_u|\leq 1$. Let $e = \{u,v\}$ be an edge such that $v$ is a pendant vertex. If $S_u = \emptyset$, then $T = K_{1,n-1}$. In this situation, the expression in the theorem statement reduces to $\binom{n-1}{3} + 2\binom{n-1}{4} + \binom{n-1}{3}.$ It is an easy verification that this number matches with the formula we obtained in \Cref{star}. We now assume that $|S_u| = 1$. By the choice of $u$, we can see that $(T\setminus e)_e = (T\setminus v)\cup_a K_m \sqcup \{v\}$, where $S_u = \{a\}$ and $m = \deg_T u$. Let $G' = (T\setminus v)\cup_a K_m$. Then $|V(G')| = n-1$ and $J_{G'} = J_{(T\setminus e)_e}$. Observe that $\deg_{(T\setminus e)_e} a = \deg_T a + m-2$. Thus, from \Cref{T+K_m}, we get \begin{eqnarray*} \beta_1\left(J_{(T\setminus e)_e}\right) &= & \binom{n-2}{2} +2\binom{m}{3} + \sum_{w\notin V(K_m)}\binom{\deg_{(T\setminus e)_e} w}{3} +\binom{\deg_{(T\setminus e)_e} a-m+1}{3}\\ & &+(n-m-2)\binom{m-1}{2} + (m-1)\binom{\deg_{(T\setminus e)_e} a -m+1}{2}\\ &= & \binom{n-2}{2} +2\binom{\deg_T u}{3} + \sum_{w\notin V(K_m)}\binom{\deg_T w}{3} +\binom{\deg_T a-1}{3}\\ & &+(n-\deg_T u-2)\binom{\deg_T u-1}{2} + (\deg_T u-1)\binom{\deg_T a-1}{2}. \end{eqnarray*} Let $T' = T\setminus v$. Then $J_{T'} = J_{T\setminus e}$. Note that $|V(T')| = n-1,$ $\deg_{T'} u = \deg_T u-1$, and $\deg_{T'}x = \deg x$ for all $x \in V(T) \setminus\{u\}.$ Additionally $|E(T'\setminus u)| = |E(T \setminus u)|$ and $|E(T' \setminus w)| = |E(T \setminus w) | -1$ for all $w \neq u$. By the induction hypothesis, \begin{eqnarray*} \beta_2(J_{T'}) & = & \binom{n-2}{3} + 2\sum_{w\neq u}\binom{\deg_T w}{4} + 2\binom{\deg_T u-1}{4} \\ & &+\sum_{w\neq u}\binom{\deg_T w}{3}(|E(T\setminus w)|)+\binom{\deg_T u-1}{3}(|E(T \setminus u)|+1)+P(T'). \end{eqnarray*} Thus, it follows from \Cref{maintheo} that \begin{eqnarray*} \beta_2(J_{T}) &= & \binom{n-2}{3}+ 2\sum_{w\neq u}\binom{\deg_T w}{4}+ 2\binom{\deg_T u-1}{4} \\ & &+\sum_{w\neq u}\binom{\deg_T w}{3}(|E(T\setminus w)|)+\binom{\deg_T u-1}{3}(|E(T \setminus u)|+1)+P(T')\\ & &+\binom{n-2}{2}+2\binom{\deg_T u}{3}+\sum_{w\notin V(K_m)}\binom{\deg_T w}{3}+\binom{\deg_T a-1}{3}\\ & &+(n-\deg_T u-2)\binom{\deg_T u-1}{2}+(\deg_T u-1)\binom{\deg_T a-1}{2}. \end{eqnarray*} Note that for all $w \in N_{T'}(u) \setminus \{a\}$, $\deg_{T'}(w) = 1$. Thus $\binom{\deg_{T'} w}{3} = 0$ for all $w\in N_{T'}(u) \setminus \{a\}$. Hence, none of the $w$, $w \neq a$, for which $\binom{\deg_T w}{3} \neq 0$ belong to $V(K_m)$ in $(T\setminus e)_e$. Thus we can write \[\sum_{w\neq u}\binom{\deg_T w}{3}(|E(T\setminus w)|) + \sum_{w\notin V(K_m)}\binom{\deg_T w}{3} = \sum_{w\neq u}\binom{\deg_T w}{3}(|E(T\setminus w)|+1).\] To compare $P(T)$ and $P(T\setminus e)$, observe that the only elements of $\mathcal{P}(T)$ which are not in $\mathcal{P}(T\setminus e)$ are the induced subgraphs which contain the edge $e$. Since $a$ is the only neighbor of $u$ having degree more than one, the total number of such graphs is $(\deg_T u -2)\binom{\deg_T a-1}{2}$. Thus $P(T\setminus e) = P(T) - (\deg_T u -2)\binom{\deg_T a-1}{2}.$ Note also that $|E(T\setminus u)| =n-\deg_T u -1$. Incorporating the above observations in the expression for $\beta_2(J_T)$, and using the identity $\binom{n}{r} = \binom{n-1}{r-1} + \binom{n-1}{r}$, we get \footnotesize \begin{eqnarray*} \beta_2(J_T) &= & \binom{n-1}{3} + 2\sum_{w\neq u}\binom{\deg_T w}{4} + 2\binom{\deg_T u-1}{4}+\sum_{w\neq u,a}\binom{\deg_T w}{3}(|E(T\setminus w)|+1) \\ & &+\binom{\deg_T a}{3}(|E(T\setminus a)|)+\binom{\deg_T u-1}{3}(|E(T\setminus u)|+1)+P(T)+\binom{\deg_T a-1}{2}\\ & &+2\binom{\deg_T u}{3}+\binom{\deg_T a-1}{3}+(|E(T\setminus u)|-1)\binom{\deg_T u-1}{2}\\ &= & \binom{n-1}{3}+ 2\sum_{w\neq u}\binom{\deg_T w}{4} + 2\binom{\deg_T u-1}{4} +\sum_{w\neq u,a}\binom{\deg_T w}{3}(|E(T\setminus w)|+1)\\ & &+\binom{\deg_T a}{3}(|E(T\setminus a)|+1)+\binom{\deg_T u}{3}(|E(T\setminus u)|+1)\\ & &+P(T)+2\binom{\deg_T u}{3}-2\binom{\deg_T u-1}{2}\\ &= & \binom{n-1}{3}+ 2\sum_{w\neq u}\binom{\deg_T w}{4} + 2\binom{\deg_T u-1}{4}+\sum_{w}\binom{\deg_T w}{3}(|E(T\setminus w)|+1)\\ & &+P(T) +2\binom{\deg_T u-1}{3} \\ &= & \binom{n-1}{3} + 2\sum_{w}\binom{\deg_T w}{4} +\sum_{w}\binom{\deg_T w}{3}(1+|E(T\setminus w)|)+P(T). \end{eqnarray*} \normalsize We have now completed the proof. \end{proof} It can be seen that \Cref{betti2tree} builds on \cite[Theorem 3.1]{jayanthan_almost_2021}. We conclude our article by computing certain graded Betti numbers of binomial edge ideals of trees. \begin{theorem}\label{thirdrow} Let $T$ be a tree and $J_T$ be its corresponding binomial edge ideal. Then, \[\beta_{k,k+3}(J_T) = \sum_{w\in V(T)}k\binom{\deg_T w+1}{k+2}\text{ for all k $\geq 2$}.\] \end{theorem} \begin{proof} We prove the assertion by induction on $|V(T)|$. Let $|V(T)|=n=2$. Then $J_T$ is the binomial edge ideal of a single edge. Since this is a principal ideal generated in degree $2$, $\beta_{k,k+3}(J_T)=0$ for all $k\geq 2$, which agrees with the formula. Suppose the assertion is true for all trees with $n-1$ vertices. Let $T$ be a tree with $|V(T)| = n$. Using \Cref{pendantexist}, consider $e=\{u,v\} \in E(T)$, where $u$ is such that $\deg u>1$ and $|S_u|\leq 1$. Then, using \Cref{maintheo}, we get \[\beta_{k,k+3}(J_T) = \beta_{k,k+3}(J_{T\setminus e})+ \beta_{k-1,k+1}(J_{(T\setminus e)_e}).\] Let $T' = T \setminus v$. Then $J_{T'} = J_{T\setminus e}$, $\deg_{T'} u = \deg_T u - 1$ and $\deg_{T'} w = \deg_T w$ for all $w \in V(T') \setminus u$. Also, $(T\setminus e)_e$ is a clique sum of a tree and a complete graph, with the size of the complete graph equal to $\deg u$. Hence using the inductive hypothesis and \Cref{linearbinom} we get: \begin{align*} & \beta_{k,k+3}(J_{T\setminus e}) = \sum_{w\neq u}k\binom{\deg_T w+1}{k+2} + k\binom{\deg_T u}{k+2},~~\mbox{and}\\ & \beta_{k-1,k+1}(J_{(T\setminus e)_e}) = k\binom{\deg_T u}{k+1}. \end{align*} Substituting these values into \Cref{maintheo} we get: \[\beta_{k,k+3}(J_T) = \sum_{w\neq u}k\binom{\deg_T w+1}{k+2} + k\binom{\deg_T u}{k+2}+k\binom{\deg_T u}{k+1} = \sum_{w}k\binom{\deg_T w+1}{k+2}.\] \end{proof} \begin{example} To illustrate some of the results of this section, consider the tree $T$ described in \Cref{ex:pt}. The graded Betti table of $J_T$ is given below: \begin{verbatim} 0 1 2 3 4 5 total: 6 20 41 43 21 4 2: 6 . . . . . 3: . 20 12 3 . . 4: . . 29 40 21 4 \end{verbatim} It follows from \Cref{betti2tree} that for this tree $$\beta_2(J_T) = \binom{6}{3} + 2 \binom{4}{4} + \binom{4}{3}(1+2) + \binom{3}{3}(1+3) + 3 = 41.$$ Additionally, the Betti numbers of the form $\beta_{k,k+3}(J_T)$ with $k \geq 2$ satisfy \Cref{thirdrow} since $\beta_{2,5}(J_T) = 2\left[\binom{4}{4} + \binom{5}{4}\right] = 12$ and $\beta_{3,6}(J_T) = 3 \binom{5}{5} = 3$. \end{example} \noindent {\bf Acknowledgments.} Some of the results in this paper first appeared in the MSc thesis of Sivakumar. Van Tuyl’s research is supported by NSERC Discovery Grant 2024-05299. \bibliographystyle{amsplain} \bibliography{Bibliography} \end{document}
2412.04268v1
http://arxiv.org/abs/2412.04268v1
Solvability of Coupled Forward-Backward Volterra Integral Equations
\documentclass[11pt,a4paper]{article} \usepackage[leqno]{amsmath} \usepackage{graphicx,latexsym,euscript,makeidx,color,bm} \usepackage{epstopdf,subfigure} \usepackage{amsmath,amsfonts,amssymb,amsthm,thmtools,mathtools,mathrsfs,enumerate} \usepackage[colorlinks,linkcolor=blue,citecolor=red]{hyperref} \renewcommand{\baselinestretch}{1.1} \usepackage{geometry} \geometry{left=3cm,right=3cm,top=3.0cm,bottom=3.0cm} \renewcommand{\sectionautorefname}{Section} \sloppy\allowdisplaybreaks[4] \setlength{\jot}{1ex} \renewcommand*{\arraystretch}{1.1} \def\dbA{\mathbb{A}} \def\sA{\mathscr{A}} \def\cA{{\cal A}} \def\BA{{\bm A}} \def\Ba{{\bm a}} \def\dbB{\mathbb{B}} \def\sB{\mathscr{B}} \def\cB{{\cal B}} \def\BB{{\bm B}} \def\Bb{{\bm b}} \def\dbC{\mathbb{C}} \def\sC{\mathscr{C}} \def\cC{{\cal C}} \def\BC{{\bm C}} \def\Bc{{\bm c}} \def\dbD{\mathbb{D}} \def\sD{\mathscr{D}} \def\cD{{\cal D}} \def\BD{{\bm D}} \def\Bd{{\bm d}} \def\dbE{\mathbb{E}} \def\sE{\mathscr{E}} \def\cE{{\cal E}} \def\BE{{\bm E}} \def\Be{{\bm e}} \def\dbF{\mathbb{F}} \def\sF{\mathscr{F}} \def\cF{{\cal F}} \def\BF{{\bm F}} \def\Bf{{\bm f}} \def\dbG{\mathbb{G}} \def\sG{\mathscr{G}} \def\cG{{\cal G}} \def\BG{{\bm G}} \def\Bg{{\bm g}} \def\dbH{\mathbb{H}} \def\sH{\mathscr{H}} \def\cH{{\cal H}} \def\BH{{\bm H}} \def\Bh{{\bm h}} \def\dbI{\mathbb{I}} \def\sI{\mathscr{I}} \def\cI{{\cal I}} \def\BI{{\bm I}} \def\Bi{{\bm i}} \def\dbJ{\mathbb{J}} \def\sJ{\mathscr{J}} \def\cJ{{\cal J}} \def\BJ{{\bm J}} \def\Bj{{\bm j}} \def\dbK{\mathbb{K}} \def\sK{\mathscr{K}} \def\cK{{\cal K}} \def\BK{{\bm K}} \def\Bk{{\bm k}} \def\dbL{\mathbb{L}} \def\sL{\mathscr{L}} \def\cL{{\cal L}} \def\BL{{\bm L}} \def\Bl{{\bm l}} \def\dbM{\mathbb{M}} \def\sM{\mathscr{M}} \def\cM{{\cal M}} \def\BM{{\bm M}} \def\Bm{{\bm m}} \def\dbN{\mathbb{N}} \def\sN{\mathscr{N}} \def\cN{{\cal N}} \def\BN{{\bm N}} \def\Bn{{\bm n}} \def\dbO{\mathbb{O}} \def\sO{\mathscr{O}} \def\cO{{\cal O}} \def\BO{{\bm O}} \def\Bo{{\bm o}} \def\dbP{\mathbb{P}} \def\sP{\mathscr{P}} \def\cP{{\cal P}} \def\BP{{\bm P}} \def\Bp{{\bm p}} \def\dbQ{\mathbb{Q}} \def\sQ{\mathscr{Q}} \def\cQ{{\cal Q}} \def\BQ{{\bm Q}} \def\Bq{{\bm q}} \def\dbR{\mathbb{R}} \def\sR{\mathscr{R}} \def\cR{{\cal R}} \def\BR{{\bm R}} \def\Br{{\bm r}} \def\dbS{\mathbb{S}} \def\sS{\mathscr{S}} \def\cS{{\cal S}} \def\BS{{\bm S}} \def\Bs{{\bm s}} \def\dbT{\mathbb{T}} \def\sT{\mathscr{T}} \def\cT{{\cal T}} \def\BT{{\bm T}} \def\Bt{{\bm t}} \def\dbU{\mathbb{U}} \def\sU{\mathscr{U}} \def\cU{{\cal U}} \def\BU{{\bm U}} \def\Bu{{\bm u}} \def\dbV{\mathbb{V}} \def\sV{\mathscr{V}} \def\cV{{\cal V}} \def\BV{{\bm V}} \def\Bv{{\bm v}} \def\dbW{\mathbb{W}} \def\sW{\mathscr{W}} \def\cW{{\cal W}} \def\BW{{\bm W}} \def\Bw{{\bm w}} \def\dbX{\mathbb{X}} \def\sX{\mathscr{X}} \def\cX{{\cal X}} \def\BX{{\bm X}} \def\Bx{{\bm x}} \def\dbY{\mathbb{Y}} \def\sY{\mathscr{Y}} \def\cY{{\cal Y}} \def\BY{{\bm Y}} \def\By{{\bm y}} \def\dbZ{\mathbb{Z}} \def\sZ{\mathscr{Z}} \def\cZ{{\cal Z}} \def\BZ{{\bm Z}} \def\Bz{{\bm z}} \def\ss{\smallskip} \def\lt{\left} \def\hb{\hbox} \def\ms{\medskip} \def\rt{\right} \def\ae{\hb{a.e.}} \def\bs{\bigskip} \def\lan{\langle} \def\as{\hb{a.s.}} \def\ds{\displaystyle} \def\ran{\rangle} \def\tr{\hb{tr$\,$}} \def\ts{\textstyle} \def\llan{\lt\lan\!} \def\rank{\hb{rank\,}} \def\no{\noindent} \def\rran{\!\rt\ran} \def\var{\hb{var$\,$}} \def\hp{\hphantom} \def\blan{\big\lan} \def\det{\hb{det\,}} \def\nn{\nonumber} \def\bran{\big\ran} \def\range{\hb{Range\,}} \def\rf{\eqref} \def\Blan{\Big\lan} \def\esssup{\hb{esssup}} \def\cd{\cdot} \def\Bran{\Big\ran} \def\essinf{\hb{essinf}} \def\deq{\triangleq} \def\({\Big (} \def\ba{\begin{aligned}} \def\les{\leqslant} \def\){\Big )} \def\ea{\end{aligned}} \def\ges{\geqslant} \def\[{\Big[} \def\bel{\begin{equation}\label} \def\ti{\tilde} \def\]{\Big]} \def\ee{\end{equation}} \def\wt{\widetilde} \def\q{\quad} \def\cl{\overline} \def\h{\widehat} \def\qq{\qquad} \def\tc{\textcolor{red}} \def\tb{\textcolor{blue}} \def\a{\alpha} \def\G{\Gamma} \def\g{\gamma} \def\Om{\Omega} \def\om{\omega} \def\b{\beta} \def\D{\Delta} \def\d{\delta} \def\F{\Phi} \def\p{\phi} \def\z{\zeta} \def\Th{\Theta} \def\th{\theta} \def\Si{\Sigma} \def\si{\sigma} \def\f{\varphi} \def\L{\Lambda} \def\l{\lambda} \def\m{\mu} \def\e{\varepsilon} \def\t{\tau} \def\i{\infty} \def\k{\kappa} \def\n{\nu} \newtheoremstyle{thry}{} {} {\sl} {} {\bf} {.} {.5em} {} \theoremstyle{thry} \newtheorem{thry}{theorem} \newtheorem{theorem}{Theorem}[section] \newtheorem{proposition}[theorem]{Proposition} \newtheorem{corollary}[theorem]{Corollary} \newtheorem{lemma}[theorem]{Lemma} \theoremstyle{definition} \newtheorem{definition}[theorem]{Definition} \newtheorem{example}[theorem]{Example} \newtheorem{problem}[theorem]{Problem} \theoremstyle{remark} \newtheorem{remark}[theorem]{Remark} \newtheorem{notation}[theorem]{Notation} \renewcommand\qedsymbol{$\blacksquare$} \def\punct{} \newtheoremstyle{dotless}{}{}{\rm}{}{\bf}{\punct}{.5em}{} \theoremstyle{dotless} \newtheorem{taggedthmx}{} \newenvironment{taggedthm}[1] {\renewcommand\thetaggedthmx{#1}\taggedthmx} {\endtaggedthmx} \newtheorem{taggedassumptionx}{} \newenvironment{taggedassumption}[1] {\renewcommand\thetaggedassumptionx{#1}\taggedassumptionx} {\endtaggedassumptionx} \makeatletter \renewcommand{\theequation}{ \thesection.\arabic{equation}} \@addtoreset{equation}{section} \newcommand{\setword}[2]{ \phantomsection #1\def\@currentlabel{\unexpanded{#1}}\label{#2} } \makeatother \begin{document} \title{\bf Solvability of Coupled Forward-Backward Volterra Integral Equations} \author{ Wenyang Li\thanks{School of Mathematical Sciences, Shenzhen University, Shenzhen, 518060, China (Email: {\tt [email protected]}). } ~~~ Hanxiao Wang\thanks{Corresponding author. School of Mathematical Sciences, Shenzhen University, Shenzhen, 518060, China (Email: {\tt [email protected]}). This author is supported in part by NSFC Grant 12201424, Guangdong Basic and Applied Basic Research Foundation 2023A1515012104, and the Science and Technology Program of Shenzhen RCBS20231211090537064.} ~~~ Jiongmin Yong\thanks{Department of Mathematics, University of Central Florida, Orlando, FL 32816, USA (Email: {\tt [email protected]}). This author is supported by NSF Grant DMS-2305475.} } \maketitle \no\bf Abstract. \rm Motivated by the optimality system associated with controlled (forward) Volterra integral equations (FVIEs, for short), the well-posedness of coupled forward-backward Voterra integral equations (FBVIEs, for short) is studied. The main feature of FBVIEs is that the unknown $\{(\cX(t,s),\cY(t,s))\}$ has two arguments. By taking $t$ as a parameter and $s$ as a (time) variable, one can regard FBVIE as a system of ordinary differential equations (ODEs, for short), with infinite-dimensional space values $\{(\cX(\cd,s),\cY(\cd,s));\,s\in[0,T]\}$. To establish the well-posedness of such an FBVIE, a new non-local monotonicity condition is introduced, by which a bridge in infinite-dimensional spaces is constructed. Then by generalizing the method of continuation developed by \cite{Hu-Peng1995,Yong1997,Peng-Wu1999} for differential equations, we have established the well-posedness of FBVIEs. The key is to apply the chain rule to the mapping $t\mapsto\big[\int_\cd^T\lan \cY(s,s),\cX(s,\cd)\ran ds+\lan G(\cX(T,T)),\cX(T,\cd)\ran\big](t)$. \ms \no\bf Keywords. \rm forward-backward Volterra integral equation, infinite-dimensional system of ordinary differential equations, two-point boundary problem, method of continuation. \ms \no\bf AMS subject classifications. \rm 45D05, 45J05, 49K21. \section{Introduction} In a number of problems involving biology systems, finance models, and fractional-order differential dynamics, (forward) Volterra integral equations (FVIEs, for short) and the related optimal control theory have attracted strong attention recently. The main feature is that FVIE can be used to describe some dynamics involving memory. Further, if we consider an optimal control problem of FVIE with a Bolza type cost functional, then by applying the Pontryagin maximum principle, one will get a coupled forward-backward Volterra integral equation FBVIE, for short) (see \cite{Angell1976,Carlson1987,Yong2008,Lin2020}). Let us briefly look at such a problem. Consider the following state equation: \bel{state}X(t)=x(t)+\int_0^tb(t,s,X(s),u(s))ds,\qq t\in[0,T],\ee with the following cost functional \bel{cost}J(u(\cd))=h(X(T))+\int_0^Tg(t,X(t),u(t))dt.\ee In the above, $X(\cd)$ is the state valued in $\mathbb{R}^n$, and $u(\cd)$ is the control valued in $\dbR^m$. For simplicity, we assume that $(X,u)\mapsto( b(t,s,X,u),h(X),g(t,X,u))$ is differentiable, and the control domain is the whole space $\dbR^m$. We assume that the state has no constraint (and therefore, the state space is the whole space $\dbR^n$). Our optimal control problem is to minimize \rf{cost}, subject to \rf{state}. Now, let $(\bar X(\cd),\bar u(\cd))$ be an optimal pair of this optimal control problem. For any admissible control $u(\cd)$, let $X(\cd)$ be the solution to the following variational equation: \begin{align*} X(t)&=\int_0^t\(b_x(t,s,\bar X(s),\bar u(s))X(s)+b_u(t,s,\bar X(s),\bar u(s))u(s)\)ds\\ &\equiv\f(t)+\int_0^tA(t,s)X(s)ds,\q t\in[0,T], \end{align*} where $$ \f(t)=\int_0^tb_u(t,s,\bar X(s),\bar u(s))u(s)ds,\qq A(t,s)=b_x(t,s,\bar X(s),\bar u(s)),\q 0\les s\les t\les T. $$ We let $\bar Y(\cd)$ be a continuous function satisfying the following adjoint equation: $$\bar Y(t)=\psi(t)+\int_t^TA(s,t)^\top\bar Y(s)ds,\q t\in[0,T],$$ for some undetermined $\psi(\cd)$. Then \begin{align*} &\int_0^T\bar Y(t)^\top X(t)dt=\int_0^T\[\bar Y(t)^\top\(\f(t)+\int_0^tA(t,s)X(s)ds\)\]dt\\ &\q=\int_0^T\bar Y(t)^\top\f(t)dt+\int_0^T\(\int_s^TA(t,s)^\top \bar Y(t)dt\)^\top X(s)ds\\ &\q=\int_0^T\bar Y(t)^\top\f(t)dt+\int_0^T\(\bar Y(s)-\psi(s)\)^\top X(s)ds. \end{align*} Hence, $$\int_0^T\psi(t)^\top X(t)dt=\int_0^T\bar Y(t)^\top\f(t)dt.$$ On the other hand, by the minimality of the optimal pair $(\bar X(\cd),\bar u(\cd))$, one has \begin{align*} 0&\les \lim_{\e\downarrow 0}{J(\bar u(\cd)+\e u(\cd))-J(\bar u(\cd))\over\e}\\ &=h_x(\bar X(T))X(T)+\int_0^T\(g_x(s,\bar X(s),\bar u(s))X(s)+g_u(s,\bar X(s),\bar u(s))u(s)\)ds\\ &=h_x(\bar X(T))\int_0^T\(b_x(T,s,\bar X(s),\bar u(s))X(s)+b_u(T,s,\bar X(s),\bar u(s))u(s)\)ds\\ &\q+\int_0^T\(g_x(s,\bar X(s),\bar u(s))X(s)+g_u(s,\bar X(s),\bar u(s))u(s)\)ds\\ &=\int_0^T\(h_x(\bar X(T))b_x(T,s,\bar X(s),\bar u(s))+g_x(s,\bar X(s),\bar u(s))\)X(s)ds\\ &\q+\int_0^T\(h_x(\bar X(T))b_u(T,s,\bar X(s),\bar u(s))+g_u(s,\bar X(s),\bar u(s))\)u(s)ds. \end{align*} Now, by taking $$\psi(s)=\(h_x(\bar X(T))b_x(T,s,\bar X(s),\bar u(s))+g_x(s,\bar X(s),\bar u(s))\)^\top,\q s\in[0,T],$$ the above gives \begin{align*} 0&\les\int_0^T\bar Y(t)^\top\int_0^tb_u(t,s,\bar X(s),\bar u(s))u(s)dsdt\\ &\q+\int_0^T\(h_x(\bar X(T))b_u(T,s,\bar X(s),\bar u(s))+g_u(s,\bar X(s),\bar u(s))\)u(s)ds\\ &=\int_0^T\(\int_s^T\bar Y(t)^\top b_u(t,s,\bar X(s),\bar u(s))dt\\ &\q+h_x(\bar X(T))b_u(T,s,\bar X(s),\bar u(s))+g_u(s,\bar X(s),\bar u(s))\)u(s)ds. \end{align*} Then we have the following {\it optimality system:} \bel{OS}\left\{\begin{aligned} &\bar X(t)=x(t)+\int_0^tb(t,s,\bar X(s),\bar u(s))ds, \\ &\bar Y(t)=g_x(t,\bar X(t),\bar u(t))^\top+b_x(T,t,\bar X(t),\bar u(t))^\top h_x(\bar X(T))^\top\\ &\qq\q+\int_t^Tb_x (s,t,\bar X(t),\bar u(t))^\top\bar Y(s)ds,\\ & g_u(t,\bar X(t),\bar u(t))^\top+b_u(T,t,\bar X(t),\bar u(t))^\top h_x(\bar X(T))^\top\\ &\qq\qq+\int_t^Tb_u(s,t,\bar X(t),\bar u(t))^\top\bar Y(s)ds=0,\end{aligned}\right.\qq t\in[0,T]. \ee This is a coupled FBVIE. The coupling is given in the third relation, which is called a {\it stationarity condition} or an {\it optimality condition}. The solution of \rf{OS} will provide a candidate for the optimal trajectory of the corresponding control problems. Therefore, the well-posedness of \rf{OS} is important, at least for the optimal control theory of FVIEs. \ms A careful observation of the above shows that in the case that if we are considering a problem with a linear state equation and the cost being quadratic in control, i.e., \bel{LCONVEX} \begin{aligned} & b(t,s,X,u)=A(t,s)X+B(t,s)u, \q 0\les s\les t\les T,\\ & g(t,X,u)=Q(t,X)+\lan R(t)u,u\ran,\q h(X)=M(X),\q t\in[0,T], \end{aligned} \ee then the optimality condition reads \bel{LCONVEX1} 2R(t)\bar u(t)+ B(T,t)^\top M_x(\bar X(T)) +\int_t^TB(s,t)^\top\bar Y(s)ds=0,\q t\in[0,T].\ee By assuming the existence of $R(\cd)^{-1}$, we end up with \bel{OS-1} \left\{\begin{aligned} \bar X(t)= & x(t)+\int_0^t \[A(t,s)\bar X(s)-{1\over 2}B(t,s) R(s)^{-1}\int_s^T B(r,s)^\top\bar Y(r)dr\\ &\qq-{1\over 2} B(t,s)R(s)^{-1}B(T,s)^\top M_x(\bar X(T))\] ds, \\ \bar Y(t)= & Q_x(t,\bar X(t))+A(T,t)^\top M_x(\bar X(T))+\int_t^T A(s,t)^{\top}\bar Y(s)ds,\end{aligned}\right. \q t\in[0,T]. \ee Motivated by the above, we consider the following FBVIE with general coefficients: \bel{FBVIE-main}\left\{\begin{aligned} &X(t)= x(t)+\int_0^t f\Big(t,s,X(s),Y(s),\int_s^T K(s,r) Y(r)dr,X(T)\Big) ds, \\ &Y(t)=h(t,X(t),X(T))+\int_t^T g(t,s,X(s), Y(s))ds,\end{aligned}\right.\q t\in[0,T],\ee where $x(\cd)$ is a given continuous function, and $f(\cd)$, $g(\cd)$, $h(\cd)$, and $K(\cd)$ are suitable mappings. We will focus on studying the well-posedness of FBVIE \rf{FBVIE-main}. \ms Another motivation for studying FBVIEs is the so-called time-inconsistent optimal control problems, in which the optimality system is also a coupled FBVIE; see \cite{Hu-Jin-Zhou2012,Wang-Yong2021,Hamaguchi2021-1}, for example. In addition, for the feature of involving memory, FBVIE has potential applications in biology models \cite{Gopalsamy1980,Kot2001,AIOmari-Gourley2003}, finance models \cite{Comte1998,ElEuch2018,ElEuch2019}, and infinite-dimensional partial differential equations \cite{Viens-Zhang2019,Wang-Yong-Zhang2022,Wang-Yong-Zhou2023,Bondi2023,Bonesini2023}. For example, we can regard the forward equation and the backward equation as an evolution system and a utility, respectively. When the evolution system is affected by the utility, one will get a coupled FBVIE immediately. Such a phenomenon has appeared in many applications. A typical example is the so-called large investor model in finance (see Cvitani\'{c} and Ma \cite{Cvitanic1996}). \ms To recover the semi-group property of \rf{FBVIE-main}, inspired by \cite{Viens-Zhang2019,Wang-Yong-Zhang2022}, we introduce the following auxiliary system of ODEs with the unknown $(\cX(\cd,\cd),\cY(\cd,\cd))$ having two arguments: \bel{FBVIE-main1}\left\{\begin{aligned} &\cX_s(t,s)= f\Big(t,s,\cX(s,s),\cY(s,s), \int_s^TK(s,r)\cY(r,r)dr,\cX(T,T)\Big),\\ & \qq\qq\qq\qq (t,s)\in\D_*[0,T],\\ &\cY_s(t,s)= -g(t,s,\cX(s,s),\cY(s,s)), \q (t,s)\in\D^*[0,T],\\ &\cX(t,0)=x(t),\q \cY(t,T)=h(t,\cX(t,t),\cX(T,T)),\q t\in[0,T], \end{aligned}\right.\ee where $\D_*[0,T]=\{(t,s)\in[0,T]^2\hbox{ with } t\ges s\}$ and $\D^*[0,T]=\{(t,s)\in[0,T]^2\hbox{ with } t\les s\}$. Indeed, \rf{FBVIE-main1} provides an equivalent representation of \rf{FBVIE-main} with the relationship: \bel{FBVIE-ODEs} X(t)=\cX(t,t),\q Y(t)=\cY(t,t),\q t\in[0,T]. \ee \ms For the forward-backward structure, \rf{FBVIE-main1} is essentially a Fredholm-type integral equation, and thus one cannot use the contraction mapping theorem unless $T>0$ is small enough. When the coefficients do not depend on $t$, \rf{FBVIE-main} reduces to the following forward-backward ODE (FBDE, for short): \bel{FBDE}\left\{\begin{aligned} &p(t)= x+\int_0^t a(s,p(s),q(s)) ds, \\ &q(t)=\psi(p(T))+\int_t^T b(s,p(s), q(s))ds,\end{aligned}\right.\q t\in[0,T].\ee For FBDEs, which can be regarded as a special case of FBSDEs, two types of methods have been developed to prove solvability. The first one is the so-called {\it four-step method} (also called a {\it decoupling method}), which was initiated by Ma, Protter, and Yong \cite{Ma-Protter-Yong1994}. The decoupling method mainly depends on the theory of partial differential equations (PDEs, for short). Since the PDE associated with \rf{FBVIE-main1} is defined on the path space, which is infinite-dimensional, its solvability might be more difficult than that of \rf{FBVIE-main1} itself; see Wang, Yong, and Zhang \cite{Wang-Yong-Zhang2022}, for example. Thus, it seems too difficult to extend the decoupling method of FBDEs to FBVIE \rf{FBVIE-main1}. \ms The second one is called a {\it method of continuation} (or a {\it monotonicity method}), which was introduced by Hu and Peng \cite{Hu-Peng1995}, and then developed by \cite{Yong1997,Peng-Wu1999}. Some further developments on FBSDEs/FBDEs can be found in \cite{Delarue2002,Yong2010,Ma2015,Yu2022}. The main idea of continuation method is to reach the solvability of FBDEs by starting with a known solvable FBDE. The way is to apply the chain rule together with some sort of monotonicity conditions to get certain a priori estimates. More precisely, in Hu and Peng \cite{Hu-Peng1995}, they introduced the following monotonicity condition: \bel{FBDE-MC}\begin{aligned} &\lan a(s,p_1,q_1)-a(s,p_2,q_2),\, q_1-q_2\ran-\lan b(s,p_1,q_1)-b(s,p_2,q_2),\, p_1-p_2\ran\\ &\q\les-\a|p_1-p_2|^2-\b|q_1-q_2|^2,\q \forall p_1,p_2,q_1,q_2\in\dbR^n,\\ &\lan \psi(p_1)-\psi(p_2),\, p_1-p_2\ran\ges \g |p_1-p_2|^2,\q \forall p_1,p_2\in\dbR^n,\end{aligned}\ee where $\a,\b,\g>0$ are given constants, and then applied the chain rule to the following function: \bel{FBDE-Ito} \lan p(t),q(t)\ran,\q t\in[0,T]. \ee In this paper, we will develop this method to prove the solvability of FBVIE \rf{FBVIE-main}. However, it is by no means easy; some comments and discussions can be found in our previous paper \cite[Subsection 5.1]{Wang-Yong-Zhang2022}. \ms We now make a careful analysis of FBVIE \rf{FBVIE-main}. Recall that \rf{FBVIE-main} is equivalent to \rf{FBVIE-main1}. From now on, we will focus on the auxiliary system \rf{FBVIE-main1} rather than \rf{FBVIE-main}. Since at the point $(t,s)$, \rf{FBVIE-main1} involves the values $\cX(s,s)$ and $\cY(s,s)$ at $(s,s)$, it is a non-local system. Alternatively, we can view $t$ as a parameter, and then \rf{FBVIE-main1} can be regarded as a Hamiltonian system of ODEs with the solution $\{(\cX(\cd,s),\cY(\cd,s)\}_{s\in[0,T]}$, and the two-point boundary condition $\cX(\cd,0)=x(\cd),\,\, \cY(\cd,T)=h(\cd,\cX(\cd,\cd),\cX(T,T))$. Thus, \rf{FBVIE-main1} can be regarded as an infinite-dimensional version of FBDE \rf{FBDE}. \ms Note that the monotonicity condition \rf{FBDE-MC} is defined at a local point $(t,x,y)$, but \rf{FBVIE-main1} is a non-local system (or an infinite-dimensional system). Then one immediately realizes that \rf{FBDE-MC} does not work to the system \rf{FBVIE-main1}, for which one should not apply the chain rule to the function $\lan \cX(t,s),\cY(t,s)\ran$ either. So two questions appear naturally: \ms (i) \emph{How can one introduce a proper non-local monotonicity condition for \rf{FBVIE-main1}?} \ms (ii) \emph{Which function should we apply the chain rule?} \ms Our answer is to impose the following {\it non-local monotonicity condition}: \bel{MC-1} \begin{aligned} &\int_t^T \big\lan \h y(s),\, \h f(s,t)\big\ran ds-\Big\lan \h h(t)+\int_t^T\h g(t,s)ds,\, \h x(t) \Big\ran\\ &+\big\lan G(x_1(T))- G(x_2(T)), \,\h f(T,t)\big\ran \les -\gamma|\h x(t)|^2,\q \hb{for some constant }\g>0, \end{aligned}\ee and some smooth function $G(\cd)$ satisfying \bel{MC-3} \begin{aligned} &\lan G(x_1(T))- G(x_2(T)),\, x_1(T)-x_2(T)\ran\ges | G^{1\over 2}_0 [x_1(T)- x_2(T)]|^2,\\ & |G(x_1(T))- G(x_2(T))|\les K| G^{1\over 2}_0 [x_1(T)- x_2(T)]|, \end{aligned} \ee where $G_0\ges 0$ is a positive semi-definite matrix, $K>0$ is a constant, and \bel{MC-2} \begin{aligned} &\h x(t)=x_1(t)-x_2(t),\q \h y(t)=y_1(t)-y_2(t),\\ &\h h(t)=h(t,x_1(t), x_1(T))-h(t,x_2(t),x_2(T)),\\ &\h g(t,s)= g(t,s,x_1(s), y_1(s))- g(t,s,x_2(s), y_2(s)),\\ &\h f(t,s)=f\Big(t,s,x_1(s),y_1(s), \int_s^T K(s,r)y_1(r)dr,x_1(T)\Big)\\ &\qq\qq-f\Big(t,s,x_1(s),y_1(s), \int_s^T K(s,r)y_1(r)dr,x_2(T)\Big), \end{aligned}\ee for any $x_i(\cd),y_i(\cd)\in C([0,T];\dbR^n);\,i=1,2$. Note that \rf{MC-1} is a non-local monotonicity condition (or a monotonicity condition defined on the infinite-dimensional space $C([0,T];\dbR^n)$). Then, with the method of continuation, by applying the chain rule to the following function: \bel{FBVIE-Ito} \int_t^T \lan \cX(s,t),\cY(s,s)\ran ds+\lan G( \cX(T,T)),\cX(T,t)\ran,\q t\in[0,T], \ee we can establish the well-posedness of \rf{FBVIE-main1}. \ms The main difficulty is to find an appropriate monotonicity condition for \rf{FBVIE-main1}. This is achieved based on some new observations for FBDEs. From Subsection \ref{subsec:comparison}, we can see that \rf{MC-1} is essentially an infinite-dimensional version of \rf{FBDE-MC}, and when FBVIE \rf{FBVIE-main1} reduces to an FBDE, it is equivalent to \rf{FBDE-MC}. In Section \ref{sec:application}, we will show the standard condition in LQ optimal control problems for FVIEs is sufficient for \rf{MC-1}. With the non-local monotonicity condition \rf{MC-1}, we still need to find a simple and known solvable FBVIE as the start of applying the method of continuation. Interestingly, we will see that this known solvable FBVIE \rf{linear} is essentially an FBDE. \ms The rest of the paper is organized as follows. In Section \ref{sec:pre}, we introduce some notations and state the main result of our paper. The proof is given in Section \ref{sec:well}. More precisely, we show the idea of finding the non-local monotonicity condition \rf{MC-1} in Subsection \ref{subsec:mc}, compare our non-local monotonicity condition \rf{MC-1} with the local one \rf{FBDE-MC} in Subsection \ref{subsec:comparison}, prove the uniqueness result in Subsection \ref{subsec:unique}, and establish the solvability in Subsection \ref{subsec:existence}. Finally, two simple examples are given in Section \ref{sec:application}. \section{Preliminary and the main result}\label{sec:pre} Let $T>0$ be a given time horizon and denote $$ \D_*[0,T]=\{(t,s)\in[0,T]^2\hbox{ with } t\ges s\} \hbox{ and } \D^*[0,T]=\{(t,s)\in[0,T]^2\hbox{ with } t\les s\}. $$ We introduce the following spaces of functions: \begin{align*} & C([0,T];\dbR^n): \hb{~~the space of $\dbR^n$-valued, continuous functions on $[0,T]$};\\ & L^2([0,T];\dbR^n): \hb{~~the space of $\dbR^n$-valued, measurable, square integrable functions on $[0,T]$}. \end{align*} Similarly, we can define the spaces of continuous functions on $\D_*[0,T]$ and $\D^*[0,T]$ as $C(\D_*[0,T];\dbR^n)$ and $C(\D^*[0,T];\dbR^n)$, respectively. \ms We impose the following assumptions for FBVIE \rf{FBVIE-main1}. \begin{taggedassumption}{(A1)}\label{ass:A1} The non-local monotonicity condition \rf{MC-1}--\rf{MC-3} holds. \end{taggedassumption} \begin{taggedassumption}{(A2)}\label{ass:A2} The coefficients of FBVIE \rf{FBVIE-main1} are continuous functions. Moreover, there exists a constant $L>0$ such that \begin{align*} &|f(t_1,s,x_1,y_1,y_1',x_1')-f( t_2,s, x_2,y_2,y_2',x_2')|\\ &\q\les L\big(|t_1- t_2|^\a+|x_1-x_2|+|y_1-y_2|+|G_0( x_1'-x_2')|+|y_1'-y_2'| \big), \\ &\qq \forall (t_i,s,x_i,y_i,y_i',x_i')\in\D_*[0,T]\times(\dbR^n)^4,\q i=1,2,\\ &|g(t_1,s,x_1,y_1)-g(t_2, s,x_2,y_2)|+|h(t_1,x_1,x_1')-h(t_2,x_2,x_2')|\\ &\q\les L\big(|t_1- t_2|^\a +|x_1- x_2|+|y_1- y_2|+|G_0( x_1'-x_2')|\big), \\ &\qq \forall (t_i,s,x_i,y_i,x_i')\in\D^*[0,T]\times(\dbR^n)^3,\q i=1,2, \end{align*} for some $\a\in(0,1]$, where $G_0\ges 0$ is given by \rf{MC-3}. \end{taggedassumption} The main result of our paper can be stated as follows. \begin{theorem}\label{Thm:well-posedness} Let {\rm \ref{ass:A1}} and {\rm \ref{ass:A2}} hold. Then the FBVIE \rf{FBVIE-main1} admits a unique solution $(\cX(\cd,\cd),\cY(\cd,\cd)) \in C(\D_*[0,T];\dbR^n)\times C(\D^*[0,T];\dbR^n)$. \end{theorem} \begin{remark} The solvability of FBVIEs in a small time horizon was proved by Hamaguchi \cite{Hamaguchi2021} by the classical fixed point theorem. The case with an arbitrarily given $T>0$ is much more challenging. Inspired by \cite{Wang-Yong-Zhang2022}, the first solvability result of FBVIEs in an arbitrarily long time horizon was obtained by Wang, Yong, and Zhou \cite{Wang-Yong-Zhou2023} for linear systems, by introducing the so-called path-dependent Riccati equation. Essentially, this is a four-step method, and heavily depends on the symmetric structure of the linear system. To the best of our knowledge, \autoref{Thm:well-posedness} is the first result on the well-posedness of FBVIEs with general nonlinear coefficients and an arbitrarily given time horizon. \end{remark} \begin{remark} In \cite{Hamaguchi2021,Wang-Yong-Zhang2022,Wang-Yong-Zhou2023}, the authors considered the stochastic case of FBVIEs, which is a generalization of \rf{FBVIE-main1} with an additional It\^{o} integral. We remark that the monotonicity method introduced in the current paper still works for stochastic FBVIEs. Additionally, we need to handle some technical issues. We hope to focus on introducing the basic idea here, and will report the results of stochastic FBVIEs in future publications. \end{remark} \section{Well-posedness}\label{sec:well} \subsection{Find a monotonicity condition in infinite-dimensional space}\label{subsec:mc} It is known that the monotonicity condition plays a central role in establishing the solvability of FBDEs by the method of continuation; see \cite{Hu-Peng1995,Yong1997,Peng-Wu1999}, for example. However, since FBVIE \rf{FBVIE-main1} is a non-local system, the local monotonicity condition \rf{FBDE-MC} defined at a point cannot be applied here. We need to find an infinite-dimensional version of it, and then construct a bridge which can link two infinite-dimensional spaces. \ms The key is to well understand the intuition of calculating $d[\lan p(t),q (t)\ran]$ in Hu and Peng \cite{Hu-Peng1995}. Our idea is to return to the linear-quadratic optimal control theory. \ms \begin{center} \textbf{An insight from value function} \end{center} Consider the controlled linear ODE: \bel{LQ-ODE1} p(t)=x+\int_0^t\[A(s) p(s)+B(s) u(s)\]ds,\q t \in[0,T], \ee and the cost functional of a quadratic form: \bel{LQ-ODE2} J(u(\cd))=\int_0^T\[\lan Q(t)p(t),p(t)\ran+\lan R(t) u(t),u(t)\ran\] d t+\lan G p(T),p(T)\ran. \ee Then the corresponding Hamiltonian system reads \bel{FBDE-LQ}\left\{\begin{aligned} &p(t)= x+\int_0^t \big[A(s)p(s)-B(s)R(s)^{-1}B(s)^\top q(s)\big] ds, \q t\in[0,T],\\ & q(t)=G p(T)+\int_t^T \big[A(s)^\top q(s) +Q(s)p(s)\big] ds,\q t\in[0,T],\end{aligned}\right.\ee with the optimal control $$\bar u(s)=-R(s)^{-1}B(s)^\top q(s),\q s\in[0,T].$$ From Yong and Zhou \cite{Yong-Zhou1999}, one immediately finds that \rf{FBDE-MC} almost coincides with the so-called standard condition, but more importantly, we realize that $\lan p(t),q(t)\ran$ is indeed the function of optimal values; that is $$\lan p(t),q(t)\ran=\int_t^T \[\lan Q(s)p(s),p(s)\ran+\lan R(s)\bar u(s),\bar u(s)\ran\]ds+\lan Gp(T),p(T)\ran.$$ Thus, essentially, Hu and Peng \cite{Hu-Peng1995} calculated the derivative of the optimal value with respect to the time variable, and we know that the derivative is always non-negative under proper conditions. Based on this observation, our approach is beginning to come out: We should calculate the optimal value of optimal control problems for VIEs (i.e., \rf{FBVIE-Ito}), by which we can find a proper monotonicity condition in infinite-dimensional spaces (i.e., \rf{MC-1}). \subsection{Comparison between the non-local monotonicity condition \rf{MC-1} and Hu and Peng's condition \rf{FBDE-MC} }\label{subsec:comparison} In this subsection, we will show that \rf{MC-1} is essentially an infinite-dimensional version of Hu and Peng's monotonicity condition \rf{FBDE-MC} given in \cite{Hu-Peng1995}. When the system \rf{FBVIE-main} reduces to an FBDE, \rf{MC-1} is almost equivalent to \rf{FBDE-MC}. \ms For simplicity, we only consider the Hamiltonian system associated with the LQ optimal control problem \rf{LQ-ODE1}--\rf{LQ-ODE2}. By taking \rf{LQ-ODE1}--\rf{LQ-ODE2} as an LQ optimal control problem for VIEs, the associated FBVIE reads $$ \left\{\begin{aligned} X(t)= & x+\int_0^t \[A(s) X(s)-B(s) R(s)^{-1}\int_s^TB(s)^\top Y(r)dr\\ &\qq-B(s)R(s)^{-1}B(s)^\top G X(T)\] ds, \\ Y(t)= & Q(t)X(t)+A(t)^\top G X(T)+\int_t^T A(t)^{\top} Y(s)ds,\end{aligned}\right. \q t\in [0,T]. $$ Let $$ p(t)=X(t),\q q(t)=\int_t^T Y(s)ds+GX(T),\q t\in[0,T], $$ then we have $$ \left\{\begin{aligned} &p(t)= x+\int_0^t \[A(s) p(s)-B(s) R(s)^{-1}B(s)^\top q(s) \]ds, \\ &q(t)= G p(T)+\int_t^T \[Q(s)p(s)+A(s)^\top q(s)\]ds,\end{aligned}\right.\qq t\in[0,T], $$ which is an FBDE. By \rf{MC-2} and \rf{FBDE-LQ}, the corresponding functions $\h x(\cd),\h y(\cd)$, $\h f(\cd), \h g(\cd), \h h(\cd)$, and $a(\cd), b(\cd), \psi(\cd)$ can be well-defined. Then \begin{align*} &\int_t^T \big\lan \h y(s),\, \h f(s,t)\big\ran ds-\Big\lan \h h(t)+\int_t^T\h g(t,s)ds,\, \h x(t) \Big\ran+\big\lan G\h x(T), \,\h f(T,t)\big\ran \\ &\q=\Big\lan \int_t^T \h y(s)ds+G\h x(T),\, \h f(t) \Big\ran -\Big\lan \h h(t)+\int_t^T\h g(s)ds,\, \h x(t) \Big\ran\\ &\q=\big\lan q_1(t)-q_2(t),\, a(t,p_1(t),q_1(t))-a(t,p_2(t),q_2(t))\big \ran\\ &\qq -\big\lan b(t,p_1(t),q_1(t))-b(t,p_2(t),q_2(t)),\, p_1(t)-p_2(t) \big \ran,\q t\in[0,T].\end{align*} Note that the above only depends on the values at a local point $t$. Then from \rf{MC-1}, we have \bel{MC-MC}\begin{aligned} &\big\lan a(t,p_1,q_1)-a(t,p_2,q_2),\,q_1-q_2\big \ran-\big\lan b(t,p_1,q_1)-b(t,p_2,q_2),\, p_1-p_2 \big \ran\\ &\q \les -\g |p_1-p_2|^2,\q \forall p_1,p_2,q_1,q_2\in\dbR^n,\\ &\lan \psi(p_1)-\psi(p_2),\, p_1-p_2\ran=G|p_1-p_2|^2\ges 0,\q \forall p_1,p_2\in\dbR^n. \end{aligned}\ee The non-local monotonicity condition \rf{MC-1} reduces to a local one. Clearly, the above monotonicity condition is almost equivalent to \rf{FBDE-MC}. Indeed, \rf{MC-MC} is a little bit weaker than \rf{FBDE-MC}, and in \cite{Peng-Wu1999}, Peng and Wu proved the well-posedness of \rf{FBDE} under a condition like this. \subsection{Uniqueness}\label{subsec:unique} \begin{proposition}\label{prop:uniqueness} Let {\rm \ref{ass:A1}} and {\rm \ref{ass:A2}} hold. Then FBVIE \rf{FBVIE-main1} admits at most one solution. \end{proposition} \begin{proof} Suppose that $(\cX_i(\cd,\cd),\cY_i(\cd,\cd))\in C(\D_*[0,T];\dbR^n)\times C(\D^*[0,T];\dbR^n);i=1,2$ satisfy \rf{FBVIE-main1}. Denote \begin{align} &\h \cX(t,s)=\cX_1(t,s)-\cX_2(t,s),\q \h \cY(t,s)=\cY_1(t,s)-\cY_2(t,s), \nonumber\\ &\h h(t)=h(t,\cX_1(t,t),\cX_1(T,T))-h(t,\cX_2(t,t),\cX_2(T,T)),\nn\\ & \h g(t,s)= g(t,s,\cX_1(s,s),\cY_1(s,s))- g(t,s,\cX_2(s,s),\cY_2(s,s)),\nn\\ &\h f(t,s)=f\Big(t,s,\cX_1(s,s),\cY_1(s,s), \int_s^T K(s,r)\cY_1(r,r)dr,\cX_1(T,T)\Big)\nn\\ &\qq\qq-f\Big(t,s,\cX_2(s,s),\cY_2(s,s), \int_s^T K(s,r)\cY_2(r,r)dr,\cX_2(T,T)\Big).\nn \end{align} Then $$ \left\{\begin{aligned} &{d\h\cX(t,s)\over ds}= \h f(t,s), \q (t,s)\in\D_*[0,T],\\ &{d \h\cY(t,s)\over ds}= -\h g(t,s), \q (t,s)\in\D^*[0,T],\\ &\h\cX(t,0)=0,\q \h\cY(t,T)=\h h(t),\q t\in[0,T], \end{aligned}\right. $$ which implies that \begin{align*} &{d\over dt}\[\int_t^T \lan \h\cY(s,s),\h\cX(s,t)\ran ds+\lan G(\cX_1(T,T)) -G(\cX_2(T,T)),\,\h\cX(T,t)\ran\]\\ &\q=-\lan \h\cY(t,t),\h\cX(t,t)\ran+\int_t^T\lan \h\cY(s,s),\h\cX_t(s, t)\ran ds\\ &\qq+\lan G(\cX_1(T,T)) -G(\cX_2(T,T)),\h\cX_t(T, t)\ran\\ &\q=-\Big\lan\h h(t)+\int_t^T \h g(t,s)ds,\, \h\cX(t,t)\Big\ran+\int_t^T \lan \h\cY(s,s),\,\h f(s,t)\ran ds\\ &\qq+\big\lan G(\cX_1(T,T)) -G(\cX_2(T,T)), \,\h f(T,t)\big\ran\\ &\q\les-\g |\h\cX(t,t)|^2,\q t\in[0,T]. \end{align*} Thus, \begin{align*} &\lan G(\cX_1(T,T)) -G(\cX_2(T,T)),\,\h\cX(T,T)\ran+\g\int_0^T |\h\cX(t,t)|^2dt\les0. \end{align*} Note that $$ \lan G(\cX_1(T,T)) -G(\cX_2(T,T)),\,\h\cX(T,T)\ran\ges| G^{1\over 2}_0 [\cX_1(T,T)- \cX_2(T,T)]|^2\ges0. $$ It turns out that $$ \cX_1(t,t)=\cX_2(t,t),\q t\in[0,T]. $$ Then, both $(\cX_1(\cd,\cd),\cY_1(\cd,\cd))$ and $(\cX_2(\cd,\cd),\cY_2(\cd,\cd))$ satisfy \bel{uniqueness-proof1} \left\{\begin{aligned} &{d \cX_i(t,s)\over ds}= f\Big(t,s,\cX(s,s),\cY_i(s,s), \int_s^T K(s,r)\cY_i(r,r)dr,\cX(T,T)\Big), \\ &\qq\qq\qq (t,s)\in\D_*[0,T],\\ &{d \cY_i(t,s)\over ds}= - g(t,s,\cX(s,s),\cY_i(s,s)), \q (t,s)\in\D^*[0,T],\\ &\cX_i(t,0)=x(t),\q \cY_i(t,T)=h(t,\cX(t,t),\cX(T,T)),\q t\in[0,T], \end{aligned}\right. \ee where $$ \cX(t,t):=\cX_1(t,t)=\cX_2(t,t),\q t\in[0,T]. $$ Clearly, FBVIE \rf{uniqueness-proof1} is a decoupled one, and then admits a unique solution. Thus, $(\cX_1(\cd,\cd),\cY_1(\cd,\cd))=(\cX_2(\cd,\cd),\cY_2(\cd,\cd))$. \end{proof} \subsection{Existence}\label{subsec:existence} We now give an existence result of FBVIE \rf{FBVIE-main1}. We begin with a simple FBVIE, whose solvability can be obtained by modifying the result in Peng and Wu \cite{Peng-Wu1999}. \begin{lemma}\label{lemm:alpha=0} For any given continuous functions $f_0(\cd,\cd)$, $g_0(\cd,\cd)$, $h_0(\cd)$, and $x(\cd)$, the following linear FBVIE \bel{linear}\left\{\ba &\cX_s(t,s)=- \int_s^T \cY(r,r)dr-G(\cX(T,T))+f_0(t,s),\q (t,s)\in\D_*[0,T], \\ &\cY_s(t,s)=-g_0(t,s),\q (t,s)\in\D^*[0,T],\\ &\cX(t,0)=x(t),\q \cY(t,T)=\cX(t,t)+h_0(t),\q t\in[0,T],\ea\right.\ee admits a unique solution $(\cX(\cd,\cd),\cY(\cd,\cd)) \in C(\D_*[0,T];\dbR^n)\times C(\D^*[0,T];\dbR^n)$, where $G(\cd)$ is given by the monotonicity condition \rf{MC-3}. \end{lemma} \begin{proof} One can easily check that \rf{linear} satisfies the non-local monotonicity condition \rf{MC-1}. Thus, by \autoref{prop:uniqueness}, it admits at most one solution. For the existence of a solution to \rf{linear}, we divide the proof into two steps. \ms \emph{Step 1.} We first consider the case with the smooth coefficient $f_0(\cd,\cd)$ and smooth initial value $x(\cd)$. By Peng and Wu \cite{Peng-Wu1999}, we know that the following FBDE admits a unique solution $(X(\cd),Y(\cd))$: \bel{lemm:alpha=0-p1}\left\{\ba &dX(t)=\[-Y(t)+f_0(t,t)+x^{\prime}(t)+\int_0^t \frac{\partial f_0(t,r)}{\partial t}dr\]dt, \q t\in[0,T],\\ &-dY(t)=\[X(t)+h_0(t)+\int_t^T g_0(t,r)dr\]dt,\q t\in[0,T],\\ &X(0)=x(0),\q Y(T)=G(X(T)).\ea\right. \ee Let $\cX(t,t)=X(t)$ and $\cY(t,t)=-{d Y(t)\over dt}$, then from the above we have \begin{align*} &\cX(t,t)=x(0)+\int_0^t \[-Y(s)+f_0(s,s)+x^{\prime}(s)+\int_0^s\frac{\partial f_0(s,r)}{\partial s}dr\]ds\\ &\q=x(t)-\int_0^t Y(s)ds+\int_0^t f_0(s,s)ds+\int_0^t\int_r^t\frac{\partial f_0(s,r)}{\partial s}dsdr\\ &\q=x(t)+\int_0^t f_0(t,s)ds-\int_0^t \[ Y(T)-\int _s^T{dY(r)\over dr}dr\] ds\\ &\q=x(t)+\int_0^t \[ f_0(t,s)-G(\cX(T,T))-\int _s^T\cY(r,r)dr\] ds,\q t\in[0,T], \end{align*} and $$\cY(t,t)=\cX(t,t)+h_0(t)+\int_t^T g_0(t,r)dr,\q t\in[0,T].$$ Further, we define \begin{align*} \cX(t,s)&=x(t)+\int_0^s \[ f_0(t,r)-G(\cX(T,T))-\int _r^T\cY(\t,\t)d\t\] dr,\q (t,s)\in\D_*[0,T],\\ \cY(t,s)&=\cX(t,t)+h_0(t)+\int_s^T g_0(t,r)dr,\q (t,s)\in\D^*[0,T], \end{align*} which, clearly, is a solution to \rf{linear}. \ms \emph{Step 2.} We next consider the case with the coefficient $f_0(\cd,\cd)$ and initial value $x(\cd)$ being continuous. By the standard mollification method, we can find a sequence of smooth functions $\{f_{0,n}(\cd,\cd),x_n(\cd)\}_{n>0}$ to uniformly converge to $f_0(\cd,\cd)$ and $x(\cd)$. By the results obtained in Step 1, we know that the following equation admits a unique solution $(\cX_n(\cd,\cd),\cY_n(\cd,\cd))$ for any given $n>0$: \bel{X-m,n}\left\{\ba &{d \cX_n(t,s)\over ds}=- \int_s^T \cY_n(r,r)dr-G(\cX_n(T,T))+f_{0,n}(t,s),\q (t,s)\in\D_*[0,T], \\ &{d\cY_n(t,s)\over ds}=-g_0(t,s),\q (t,s)\in\D^*[0,T],\\ &\cX_n(t,0)=x_n(t),\q \cY_n(t,T)=\cX_n(t,t)+h_0(t),\q t\in[0,T].\ea\right. \ee Denote $\cX_{m,n}(\cd,\cd)=\cX_m(\cd,\cd)-\cX_n(\cd,\cd)$, $\cY_{m,n}(\cd,\cd)=\cY_m(\cd,\cd)-\cY_n(\cd,\cd)$, $x_{m,n}(\cd)=x_m(\cd)-x_n(\cd)$, $f_{0,m,n}(\cd,\cd)=f_{0,m}(\cd,\cd)-f_{0,n}(\cd,\cd)$ and $\h G(\cX_{m,n}(T,T))=G(\cX_{m}(T,T))-G(\cX_{n}(T,T))$ for any given $m,n>0$. Then, $$\left\{\ba &{d \cX_{m,n}(t,s)\over ds}=- \int_s^T \cY_{m,n}(r,r)dr-\h G(\cX_{m,n}(T,T))+f_{0,m,n}(t,s), \q (t,s)\in\D_*[0,T],\\ &{d \cY_{m,n}(t,s)\over ds}=0,\q (t,s)\in\D^*[0,T],\\ &\cX_{m,n}(t,0)=x_{m,n}(t),\q \cY_{m,n}(t,T)=\cX_{m,n}(t,t),\q t\in[0,T].\ea\right.$$ Noting that $\cY_{m,n}(t,t)=\cX_{m,n}(t,t)$ for any $t\in[0,T]$, we get \begin{align*} &{d\over dt}\[\int_t^T \lan \cX_{m,n}(s,t),\cY_{m,n} (s,s)\ran ds+\lan \h G(\cX_{m,n}(T,T)),\cX_{m,n}(T,t)\ran\]\\ &\q=- \lan \cX_{m,n}(t,t),\cX_{m,n} (t,t)\ran+\int_t^T\Big \lan \cY_{m,n} (s,s) , \[f_{0,m,n}(s,t)- \int_t^T \cY_{m,n}(r,r)dr\\ &\qq-\h G(\cX_{m,n}(T,T))\]\Big\ran ds+\Big\lan\h G(\cX_{m,n}(T,T)),\[f_{0,m,n}(T,t)- \int_t^T \cY_{m,n}(r,r)dr\\ &\qq-\h G(\cX_{m,n}(T,T))\]\Big\ran\\ &\q=- \lan \cX_{m,n}(t,t),\cX_{m,n} (t,t)\ran-\Big\lan\h G(\cX_{m,n}(T,T))+\int_t^T \cY_{m,n} (s,s)ds,\,\\ &\qq\q \h G(\cX_{m,n}(T,T))+\int_t^T \cY_{m,n} (s,s)ds\Big\ran+\int_t^T\lan \cX_{m,n}(s,s),\,f_{0,m,n}(s,t)\ran ds\\ &\qq+\lan\h G(\cX_{m,n}(T,T)),f_{0,m,n}(T,t)\ran,\q t\in[0,T]. \end{align*} Further, noting $\cX_{m,n}(t,0)=x_{m,n}(t)$ for any $t\in[0,T]$, we have \begin{align*} &\lan\h G(\cX_{m,n}(T,T)),\cX_{m,n}(T,T)\ran-\int_0^T \lan x_{m,n}(s),\cX_{m,n} (s,s)\ran ds-\lan \h G(\cX_{m,n}(T,T)), x_{m,n}(T)\ran\\ &\q=-\int_0^T\[ \big|\cX_{m,n}(t,t)\big|^2+\Big| \h G(\cX_{m,n}(T,T))+\int_t^T \cY_{m,n} (s,s)ds\Big|^2\]dt\\ &\qq\, +\int_0^T\int_t^T\lan \cX_{m,n}(s,s),\,f_{0,m,n}(s,t)\ran dsdt +\int_0^T\lan \h G(\cX_{m,n}(T,T)),f_{0,m,n}(T,t)\ran dt. \end{align*} By Young's inequality, from \rf{MC-3}, the above implies that \begin{align*} &| G_0^{1\over 2}\cX_{m,n}(T,T)|^2 +\int_0^T\[ \big|\cX_{m,n}(t,t)\big|^2+\Big| \h G(\cX_{m,n}(T,T))+\int_t^T \cY_{m,n} (s,s)ds\Big|^2\]dt\\ &\q\les \e \int_0^T \big|\cX_{m,n}(t,t)\big|^2dt +\e| G_0^{1\over 2}\cX_{m,n}(T,T)|^2 +C_\e \int_0^T \big|x_{m,n}(t)\big|^2dt\\ &\qq +C_\e| x_{m,n}(T)|^2+C_\e \int_0^T\int_t^T \big|f_{0,m,n}(s,t)\big|^2dsdt+C_\e\int_0^T | f_{0,m,n}(T,t)|^2dt, \end{align*} which yields that \begin{align*} &| G_0^{1\over 2}\cX_{m,n}(T,T)|^2 +\int_0^T\[ \big|\cX_{m,n}(t,t)\big|^2+\Big| \h G(\cX_{m,n}(T,T))+\int_t^T \cY_{m,n} (s,s)ds\Big|^2\]dt\\ &\q\les C \[\int_0^T \big|x_{m,n}(t)\big|^2dt+| x_{m,n}(T)|^2+\int_0^T\int_t^T \big|f_{0,m,n}(s,t)\big|^2dsdt\\ &\qq\qq+\int_0^T | f_{0,m,n}(T,t)|^2dt\]. \end{align*} Then noting that $$| \h G(\cX_{m,n}(T,T))|\les K |G_0^{1\over 2}\cX_{m,n}(T,T)|,$$ by the standard results of VIEs, we have \begin{align*} \sup_{t,s}|\cX_{m,n}(t,s)|^2&\les C \[\int_0^T \big|x_{m,n}(t)\big|^2dt+| x_{m,n}(T)|^2+\sup_t\int_0^t \big|f_{0,m,n}(t,s)\big|^2ds\\ &\qq+\int_0^T | f_{0,m,n}(T,t)|^2dt+\sup_t|x_{m,n}(t)|^2\].\end{align*} Thus, $\{\cX_n(\cd,\cd)\}_{n>0}$ is a Cauchy sequence in the space of continuous functions. With the fact that $\cY_{m,n}(t,s)=\cX_{m,n}(t,t)$, we know that $\{\cY_n(\cd,\cd)\}_{n>0}$ is also a Cauchy sequence in the space of continuous functions. Let $(\cX(\cd,\cd),\cY(\cd,\cd))$ be the limit of $\{\cX_n(\cd,\cd),\cY_n(\cd,\cd)\}_{n>0}$ as $n\to\infty$. Taking $n\to\infty$ in \rf{X-m,n}, we know that $(\cX(\cd,\cd),\cY(\cd,\cd))$ is a solution to \rf{linear}. \end{proof} From the proof, we can see that \rf{linear} is essentially an FBDE. Next, we consider the following family of FBVIEs parameterized by $\a \in[0,1]$: \bel{para}\left\{\ba \cX_s(t,s)&=\a f\Big(t,s,\cX(s,s),\cY(s,s), \int_s^T K(s,r)\cY(r,r)dr,\cX(T,T)\Big)\\ &\q+(1-\a)\[- \int_s^T \cY(r,r)dr-G(\cX(T,T))\]+f_0(t,s),\\ & \q\qq (t,s)\in\D_*[0,T],\\ \cY_s(t,s)&=-\big[ \a g(t,s,\cX(s,s),\cY(s,s))+g_0(t,s) \big],\q (t,s)\in\D^*[0,T],\\ \cX(t,0)&=x(t),\q t\in[0,T],\\ \cY(t,T)&=\a h(t,\cX(t,t),\cX(T,T))+(1-\a)\cX(t,t)+h_0(t),\q t\in[0,T],\ea\right.\ee where $f_0(\cd,\cd)$, $g_0(\cd,\cd)$, and $h_0(\cd)$ are continuous functions. Clearly, when $\a=1$, the existence of the solution of \rf{para} implies that of \rf{FBVIE-main1}. \begin{lemma}\label{lem:con} Let {\rm \ref{ass:A1}} and {\rm \ref{ass:A2}} hold. We assume that, for a given $\a_0\in[0,1)$ and for any $f_0(\cd,\cd),g_0(\cd,\cd),h_0(\cd)$, \rf{para} admits a unique solution. Then there exists a constant $\d_0\in(0,1)$, such that for all $\a \in[\a_0,\a_0+\d_0]$, and for any $f_0(\cd,\cd),g_0(\cd, \cd), h_0(\cd)$, \rf{para} admits a unique solution. \end{lemma} \begin{proof} Note that for each $f_0(\cd,\cd)$, $g_0(\cd,\cd)$, $h_0(\cd)$, and $\a_0\in[0,1)$, \rf{para} admits a unique solution. Then, for each pair $(\f(\cd,\cd),\p(\cd,\cd))$, there exists a unique pair $(\cX(\cd,\cd),\cY(\cd,\cd))$ satisfying the following FBVIE: $$ \left\{\ba \cX_s(t,s)&=\a_0 f\Big(t,s,\cX(s,s),\cY(s,s), \int_s^T K(s,r)\cY(r,r)dr,\cX(T,T)\Big)\\ &\q+(1-\a_0)\[- \int_s^T \cY(r,r)dr-G(\cX(T,T))\]\\ &\q+\d f\Big(t,s,\f(s,s),\p(s,s), \int_s^T K(s,r)\p(r,r)dr,\f(T,T)\Big)\\ &\q+\d \[\int_s^T \phi(r,r)dr+G(\f(T,T))\]+f_0(t,s),\q (t,s)\in\D_*[0,T], \\ \cY_s(t,s)&=-\a_0 g(t,s,\cX(s,s),\cY(s,s))-\d g(t,s,\f(s,s),\p(s,s))-g_0(t,s),\\ &\qq\qq\qq (t,s)\in\D^*[0,T],\\ \cX(t,0)&=x(t),\q t\in[0,T],\\ \cY(t,T)&=\a_0 h(t,\cX(t,t),\cX(T,T))+(1-\a_0)\cX(t,t)\\ &\q+\d\[h(t,\f(t,t),\f(T,T))-\f(t,t)\]+h_0(t),\q t\in[0,T].\ea\right. $$ \ms \ms We are going to prove that the mapping $\G_{\a_0+\d}[\cd,\cd]$, defined by \begin{align*} &\G_{\a_0+\d}[\f(\cd,\cd),\p(\cd,\cd)]=(\cX(\cd,\cd),\cY(\cd,\cd)),\\ &\qq \forall (\f(\cd,\cd),\p(\cd,\cd))\in C(\D_*[0, T] ;\dbR^n) \times C(\D^*[0, T] ;\dbR^n), \end{align*} is a contraction. Let $(\f_i(\cd,\cd),\p_i(\cd,\cd))\in C(\D_*[0, T] ;\dbR^n) \times C(\D^*[0, T] ;\dbR^n)$, and $$ (\cX_i(\cd,\cd),\cY_i(\cd,\cd))=\G_{\a_0+\d}[\f_i(\cd,\cd),\p_i(\cd,\cd)],\q \hbox{for } i=1,2. $$ Denote $(\h\f(\cd,\cd), \h\p(\cd,\cd))=(\f_1(\cd,\cd)-\f_2(\cd,\cd),\p_1(\cd,\cd)-\p_2(\cd,\cd))$ and $(\h\cX(\cd,\cd), \h\cY(\cd,\cd))=(\cX_1(\cd,\cd)-\cX_2(\cd,\cd),\cY_1(\cd,\cd)-\cY_2(\cd,\cd))$. Similar to \rf{MC-2}, we can denote \begin{align*} &\h h(t;\f),\q \h f(t,s;\f,\p),\q\h g(t,s;\f,\p), \q\hbox{and}\\ & \h h(t;\cX),\q \h f(t,s;\cX,\cY),\q\h g(t,s;\cX,\cY), \end{align*} with $(x_i(\cd),y_i(\cd))$ replaced by $(\f_i(\cd,\cd),\p_i(\cd,\cd))$ and $(\cX_i(\cd,\cd),\cY_i(\cd,\cd))$, respectively. Next, we denote $\h G(\h\cX(T,T))=G(\cX_1(T,T))-G(\cX_2(T,T))$ and $\h G(\h\f(T,T))=G(\f_1(T,T))-G(\f_2(T,T))$. Then, $$ \left\{\ba {d \h\cX(t,s)\over ds}&=\a_0 \h f(t,s;\cX,\cY)+(1-\a_0)\[- \int_s^T \h\cY(r,r)dr-\h G(\h\cX(T,T))\]\\ &\q+\d \h f(t,s;\f,\p)+\d \[\int_s^T \h\phi(r,r)dr+\h G(\h\f(T,T))\], \q (t,s)\in\D_*[0,T],\\ {d\h\cY(t,s)\over ds}&=-\a_0 \h g(t,s;\cX,\cY)-\d \h g(t,s;\f,\p),\q (t,s)\in\D^*[0,T],\\ \h\cX(t,0)&=0,\q t\in[0,T],\\ \h\cY(t,T)&=\a_0\h h(t;\cX)+(1-\a_0)\h\cX(t,t)+\d[\h h(t;\f)-\h\f(t,t)],\q t\in[0,T].\ea\right. $$ By the same argument as that employed in the proof of \autoref{prop:uniqueness}, we have \begin{align*} &{d\over dt}\[\int_t^T \lan \h\cY(s,s),\h\cX(s,t)\ran ds+\lan\h G(\h\cX(T,T)),\h\cX(T,t)\ran\]\\ &\q=-\Big\lan \a_0\h h(t;\cX)+(1-\a_0)\h\cX(t,t)+\d[\h h(t;\f)-\h\f(t,t)]\\ &\qq\qq\qq+\int_t^T [\a_0 \h g(t,s;\cX,\cY)+\d \h g(t,s;\f,\p)]ds,\, \h\cX(t,t)\Big\ran\\ &\qq+\int_t^T\Big\lan \h\cY(s,s),\,\a_0 \h f(s,t;\cX,\cY)+(1-\a_0)\[- \int_t^T \h\cY(r,r)dr-\h G(\h\cX(T,T))\]\\ &\qq\qq\qq +\d \h f(s,t;\f,\p)+\d \[\int_t^T \h\phi(r,r)dr+\h G(\h\f(T,T))\]\Big\ran ds\\ &\qq+\Big\lan \h G(\h\cX(T,T)),\,\a_0 \h f(T,t;\cX,\cY)+(1-\a_0)\[- \int_t^T \h\cY(r,r)dr-\h G(\h\cX(T,T))\]\\ &\qq\qq\qq +\d \h f(T,t;\f,\p)+\d \[\int_t^T \h\phi(r,r)dr+\h G(\h\f(T,T))\]\Big\ran,\q t\in[0,T]. \end{align*} Then from the non-local monotonicity condition \rf{MC-1}, we have \begin{align*} &{d\over dt}\[\int_t^T \lan \h\cY(s,s),\h\cX(s,t)\ran ds+\lan\h G(\h\cX(T,T)),\h\cX(T,t)\ran\]\\ &\q\les-\d\Big\lan \h h(t;\f)-\h\f(t,t)+\int_t^T \h g(t,s;\f,\p)ds,\, \h\cX(t,t)\Big\ran\\ &\qq+\d \int_t^T\Big\lan \h\cY(s,s),\, \h f(s,t;\f,\p)+\int_t^T \h\phi(r,r)dr+\h G(\h\f(T,T))\Big\ran ds\\ &\qq+\d\Big\lan\h G(\h\cX(T,T)),\,\h f(T,t;\f,\p)+\int_t^T \h\phi(r,r)dr+\h G(\h\f(T,T))\Big\ran\\ &\qq -\a_0 \g|\h\cX(t,t)|^2-(1-\a_0)\[|\h\cX(t,t)|^2+\Big|\int_t^T \h\cY(r,r)dr+\h G(\h\cX(T,T))\Big|^2\],\q t\in[0,T]. \end{align*} Thus, noting $\h\cX(\cd,0)\equiv 0$ and $\g>0$, by \rf{MC-3}, we get \begin{align*} &\big| G_0^{1\over 2}\h\cX(T,T)\big|^2+\int_0^T|\h\cX(t,t)|^2dt\\ &\q\les \d C\int_0^T \bigg\{\Big|\Big\lan \h h(t;\f)-\h\f(t,t)+\int_t^T \h g(t,s;\f,\p)ds,\, \h\cX(t,t)\Big\ran\Big|\\ &\qq\qq+\Big|\int_t^T\Big\lan \h\cY(s,s),\, \h f(s,t;\f,\p)+\int_t^T \h\phi(r,r)dr+\h G(\h\f(T,T))\Big\ran ds\Big|\\ &\qq\qq+\Big|\Big\lan \h G(\h \cX(T,T)),\,\h f(T,t;\f,\p)+\int_t^T \h\phi(r,r)dr+\h G(\h\f(T,T))\Big\ran\Big|\bigg\}dt\\ &\q\les {1\over 2} |G_0^{1\over2}\h\cX(T,T)|^2+{1\over 2}\int_0^T|\h\cX(t,t)|^2dt+\d C\int_0^T|\h\cY(t,t)|^2dt\\ &\qq +\d C|\h\f(T,T)|^2+ \d C\int_0^T \bigg\{| \h h(t;\f)|^2+|\h\phi(t,t)|^2+|\h f(T,t;\f,\p)|^2+|\h\f(t,t)|^2\\ &\qq\qq+\int_t^T\big[ | \h g(t,s;\f,\p)|^2 +|\h f(s,t;\f,\p)|^2\big]ds\bigg\}dt, \end{align*} where the last inequality is obtained by applying the Young's inequality. Thus, \begin{align*} &\big| G_0^{1\over 2}\h\cX(T,T)\big|^2+\int_0^T|\h\cX(t,t)|^2dt\\ &\q\les \d C\int_0^T|\h\cY(t,t)|^2dt +\d C|\h\f(T,T)|^2+ \d C\int_0^T \big[|\h\f(t,t)|^2+|\h\p(t,t)|^2\big]dt. \end{align*} By the standard results of BVIEs, we have \bel{Prop:con-p1} \begin{aligned} &\int_0^T |\h\cY(t,t)|^2dt \les\d C\[\int_0^T\(|\h\f(t,t)|^2+|\h\p(t,t)|^2\)dt+|\h\f(T,T)|^2\]\\ &\qq +C\[\int_0^T|\h\cX(t,t)|^2dt+|G_0\h\cX(T,T)|^2\]\\ &\q\les \d C\[\int_0^T\(|\h\f(t,t)|^2+|\h\p(t,t)|^2\)dt+|\h\f(T,T)|^2\]\\ &\qq +C\[\int_0^T|\h\cX(t,t)|^2dt+|G_0^{1\over 2}\h\cX(T,T)|^2\]. \end{aligned} \ee Combining the above two estimates together, we have \begin{align*} & |G_0^\frac{1}{2}\h\cX(T,T)|^2+\int_0^T|\h\cX(t,t)|^2dt \\ &\q\les \d C\[\int_0^T\(|\h\f(t,t)|^2+|\h\p(t,t)|^2\)dt+\int_0^T|\h\cX(t,t)|^2dt+|G_0^{1\over 2}\h\cX(T,T)|^2+|\h\f(T,T)|^2\], \end{align*} which implies that \begin{align*} & |G_0^\frac{1}{2}\h\cX(T,T)|^2+\int_0^T|\h\cX(t,t)|^2dt \\ &\q\les \d C\[\int_0^T\(|\h\f(t,t)|^2+|\h\p(t,t)|^2\)dt+|\h\f(T,T)|^2\]. \end{align*} Substituting the above into \rf{Prop:con-p1}, we get \begin{align*} &|G_0^\frac{1}{2}\h\cX(T,T)|^2+\int_0^T\(|\h\cX(t,t)|^2+|\h\cY(t,t)|^2\)dt \\ &\q\les \d C\[\int_0^T\(|\h\f(t,t)|^2+|\h\p(t,t)|^2\)dt+|\h\f(T,T)|^2\]. \end{align*} Then by the standard estimates of FVIEs, we have \bel{Prop:con-p2} \begin{aligned} &\sup _{t,s} |\h\cX(t,s)|^2 \les \d C\[\int_0^T\(|\h\f(t,t)|^2+|\h\p(t,t)|^2\)dt+|\h\f(T,T)|^2\]\\ &\qq+C\[ |G_0^\frac{1}{2}\h\cX(T,T)|^2+\int_0^T|\h\cY(t,t)|^2dt\]\\ &\q\les \d C\[\int_0^T\(|\h\f(t,t)|^2+|\h\p(t,t)|^2\)dt+|\h\f(T,T)|^2\]. \end{aligned} \ee On the other hand, the standard estimate of BVIEs implies that \bel{Prop:con-p3} \begin{aligned} &\sup _{t,s} |\h\cY(t,s)|^2 \les \d C\[\int_0^T|\h\p(t,t)|^2dt+\sup _{t} |\h\f(t,t)|^2\]+C\sup _{t} |\h\cX(t,t)|^2\\ &\q\les \d C\[\int_0^T|\h\p(t,t)|^2dt+\sup _{t} |\h\f(t,t)|^2\], \end{aligned} \ee in which the last equality is due to \rf{Prop:con-p2}. Combining the estimates \rf{Prop:con-p2} and \rf{Prop:con-p3} together, we get \begin{align*} \sup_{t,s}\[|\h\cX(t,s)|^2+|\h\cY(t,s)|^2\] \les \d C\sup_{t,s}\[|\h\f(t,s)|^2+|\h\p(t,s)|^2\]. \end{align*} We now choose $\d_0={1\over 2C}$, which is independent of $\a_0$. Clearly, for each fixed $\d\in [0,\d_0]$, the mapping $\G_{\a_0+\d}[\cd,\cd]$ is a contraction. It turns out that this mapping has a unique fixed point $(\cX^{\a_0+\d}(\cd,\cd), \cY^{\a_0+\d}(\cd,\cd))$, which is the unique solution of \rf{para} for $\a=\a_0+\d$. \end{proof} \ms We are ready to give the proof of \autoref{Thm:well-posedness} now. \begin{proof} From \autoref{lemm:alpha=0}, we see immediately that, when $\a=0$, for any $f_0(\cd,\cd)$, $g_0(\cd,\cd)$, and $h_0(\cd)$, FBVIE \rf{para} admits a unique solution. Then by \autoref{lem:con}, for any $f_0(\cd,\cd)$, $g_0(\cd,\cd)$, and $h_0(\cd)$, we can solve the FBVIE \rf{para} successively for the case $\a \in[0, \d_0],[\d_0, 2 \d_0], \ldots, [(N-1)\d_0, 1]$, with $(N-1)\d_0<1\les N\d_0$. It turns out that, when $\a=1$, for any $f_0(\cd,\cd)$, $g_0(\cd,\cd)$, and $h_0(\cd)$, \rf{para} admits a solution, which implies that the solution of \rf{FBVIE-main1} exists. The uniqueness of solutions to \rf{FBVIE-main1} is an immediate consequence of \autoref{prop:uniqueness}. \end{proof} \section{Examples} \label{sec:application} In this section, we shall give two explicit examples of coupled FBVIEs, whose solvability can be obtained from \autoref{Thm:well-posedness}. \begin{example}[Hamiltonian System Derived From Linear-Convex Optimal Control Problems] Consider the controlled Volterra integral equation: $$ X(t)=x(t)+\int_0^t\[A(t, s) X(s)+B(t, s) u(s)\]ds,\q t \in[0,T], $$ and the cost functional: $$ J( u(\cd))=\int_0^T\[Q(t,X(t))+\lan R(t) u(t),u(t)\ran\] d t+M(X(T)). $$ We assume that all the functions involved above are smooth, and \begin{align*} &|Q(t,x)|\les L(1+|x|^2),\q|M(x)|\les L(1+|x|^2),\q \forall x\in\dbR^n, \\ & R(t)\ges \d I_m,\q \lan Q_{x}(t,x_1)-Q_{x}(t,x_2),\,x_1-x_2\ran\ges \d |x_1-x_2|^2,\\ &|Q_{x}(t,x_1)-Q_{x}(t,x_2)|\les L|x_1-x_2|,\\ & \lan M_{x}(x_1)-M_{x}(x_2),\,x_1-x_2\ran\ges |G_0^{1\over 2} (x_1-x_2)|^2,\\ &| M_{x}(x_1)-M_{x}(x_2)|\les L|G_0^{1\over 2} (x_1-x_2)|,\q \forall x_1,x_2\in\dbR^n, \end{align*} for some $\d>0$, $L>0$, and $G_0\ges 0$. A typical example is that \bel{LQ-Form} Q(t,x)=\lan Q(t)x,x\ran, \q \hbox{with } Q(t)\ges \d I_n>0, \q M(x)=\lan G_0 x,x\ran, \ee under which the problem has a linear-quadratic form. In addition, if the state $X(\cd)$ is one-dimensional, we can also take $$ Q(t,x)={x^4-2\over x^2+1}, $$ by which the problem is out of the linear-quadratic framework. The optimal control problem can be stated as follows: Find a control $\bar u(\cd)\in L^2([0,T];\dbR^m)$ such that $$ J(0, x(\cd);\bar u(\cd))=\inf_{u(\cd)\in L^2([0,T];\dbR^m)} J(0, x(\cd);u(\cd)). $$ \ms Clearly, this is the optimal control problem with the form \rf{LCONVEX} given in Introduction. Then from \rf{LCONVEX1} and \rf{OS-1}, the optimal control $\bar u(\cd)$ can be written as: \begin{align}\label{MP-condition} \bar{u}(s)=-{1\over 2}R(s)^{-1}\int_s^T B(r,s)^\top Y(r)dr-{1\over 2}R(s)^{-1}B(T,s)^\top M_x( X(T)), \quad s \in[0, T], \end{align} with the Hamiltonian system \bel{MP-system}\left\{\begin{aligned} X(t)= & x(t)+\int_0^t \[A(t,s) X(s)-{1\over 2}B(t,s) R(s)^{-1}\int_s^T B(r,s)^\top Y(r)dr\\ &\qq-{1\over 2}B(t,s)R(s)^{-1}B(T,s)^\top M_x(X(T))\] ds, \\ Y(t)= & Q_x(t,X(t))+A(T,t)^\top M_x(X(T))+\int_t^T A(s,t)^{\top} Y(s)ds,\end{aligned}\right. \q t\in[0,T]. \ee Then, \begin{align*} &f(t,s,x,y,y^\prime,x^\prime)=A(t,s)x-{1\over 2}B(t,s) R(s)^{-1}y^\prime-{1\over 2}B(t,s)R(s)^{-1}B(T,s)^\top M_x(x^\prime),\\ & K(s,r)=B(r,s)^\top,\q g(t,s,x,y)=A(s,t)^\top y,\q h(t,x,x^\prime)=Q_x(t,x)+A(T,t)^\top M_x(x^\prime). \end{align*} For any $x_i(\cd),y_i(\cd)\in C([0,T];\dbR^n),\,i=1,2$, denote $\h x(\cd)$, $\h y(\cd)$, $\h f(\cd,\cd)$, $\h g(\cd,\cd)$, and $\h h(\cd)$ as that in \rf{MC-2}. Moreover, we take $G(\cd)=M_x(\cd)$, and denote $$ \h G(T)=G (x_1(T))-G (x_2(T)),\q \h Q_x(t)=Q_x(t,x_1(t))-Q_x(t,x_2(t)). $$ Clearly, \rf{MC-3} holds. Then, \begin{align} &\int_t^T \big\lan \h y(s),\, \h f(s,t)\big\ran ds-\Big\lan \h h(t)+\int_t^T\h g(t,s)ds,\, \h x(t) \Big\ran+\big\lan\h G (T), \,\h f(T,t)\big\ran \nn\\ &\q=\int_t^T \bigg\lan \h y(s),\, A(s,t)\h x(t)-{1\over 2}B(s,t) R(t)^{-1}\int_t^TB(r,t)^\top \h y(r)dr\nn\\ &\qq\qq-{1\over 2}B(s,t)R(t)^{-1}B(T,t)^\top \h G (T)\bigg\ran ds\nn\\ &\qq-\Big\lan\h Q_x(t) +A(T,t)^\top \h G (T)+\int_t^T A(s,t)^\top\h y(s) ds,\, \h x(t) \Big\ran\nn\\ &\qq+\bigg\lan \h G (T), \,A(T,t)\h x(t)-{1\over 2}B(T,t) R(t)^{-1}\int_t^TB(r,t)^\top \h y(r)dr\nn\\ &\qq\qq -{1\over 2} B(T,t)R(t)^{-1} B(T,t)^\top\h G (T)\bigg\ran\nn\\ &\q =-\big\lan\h Q_x(t),\, \h x(t)\big\ran-{1\over 2}\Big\lan R(t)^{-1}\[B(T,t)^\top \h G (T)\nn\\ &\qq\,+\int_t^TB(r,t)^\top \h y(r)dr\],\, \[B(T,t)^\top \h G (T)+\int_t^TB(r,t)^\top \h y(r)dr\]\Big\ran\nn\\ &\q\les -\d |\h x(t)|^2,\q t\in[0,T],\nn \end{align} which implies the non-local monotonicity condition \rf{MC-1} holds. Thus, from \autoref{Thm:well-posedness}, FBVIE \rf{MP-system} admits a unique solution. It turns out that the control process $\bar u(\cd)$, given by \rf{MP-condition}, is the unique optimal control. Moreover, if $Q(\cd,\cd)$ and $M(\cd)$ admit the quadratic form \rf{LQ-Form}, then the value function can be given by $$ \begin{aligned} &\int_0^T \lan \cX(s,0),\cY(s,s)\ran ds+\lan G_0\cX(T,0),\cX(T,T)\ran\\ &\q = \int_0^T \big[\lan Q(s)X(s),X(s)\ran +\lan R(s)\bar u(s),\bar u(s)\ran \big]ds+\lan G_0X(T),X(T)\ran, \end{aligned} $$ where $(\cX(\cd,\cd),\cY(\cd,\cd))$ satisfies the following auxiliary system corresponding to \rf{MP-system}: $$ \left\{\begin{aligned} &\cX_s(t,s)= A(t,s)\cX(s,s)-{1\over 2}B(t,s)R(s)^{-1}\int_s^T B(r,s)^\top\cY(r,r)dr\\ &\qq- B(t,s)R(s)^{-1}B(T,s)^\top G_0\cX(T,T),\q (t,s)\in\D_*[0,T], \\ &\cY_s(t,s)= - A(s,t)^{\top}\cY(s,s),\q (t,s)\in\D^*[0,T], \\ & \cX(t,0)=x(t),\q \cY(t,T)=2 Q(t)\cX(t,t)+2A(T,t)^\top G_0\cX(T,T),\q t\in[0,T]. \end{aligned}\right. $$ \end{example} The following is an FBVIE with some general nonlinear terms. We will show that under some proper assumptions, it also satisfies the non-local monotonicity condition \rf{MC-1}--\rf{MC-2}. \begin{example}[Nonlinear FBVIEs] Consider the following nonlinear FBVIE: \bel{FBVIE-Nonlinear}\left\{\begin{aligned} X(t)= & x(t)+\int_0^t \[A(t,s) X(s)+B(t,s)a(s,X(s))\\ &\qq-B(t,s) \int_s^T B(r,s)^\top Y(r)dr\] ds, \\ Y(t)= & b(t,X(t))+\phi\Big(t,\int_t^T B(r,t)^\top Y(r)dr\Big)\\ &\qq+\int_t^T\[ A(s,t)^{\top} Y(s)+\psi(t,s,X(t))\] ds,\end{aligned}\right.\q t\in[0,T]. \ee Assume that all the coefficients of the above are continuous functions. Moreover, there exist constants $\l,L_a,L_b,L_\phi,L_\psi>0$ such that \begin{align*} &|a(s,x_1)-a(s,x_2)|\les L_a |x_1-x_2|,\q |b(s,x_1)-b(s,x_2)|\les L_b |x_1-x_2|,\\ & \lan b(s,x_1)-b(s,x_2) ,x_1-x_2\ran\ges\l |x_1-x_2|^2,\\ &|\psi(t,s,x_1)-\psi(t,s,x_2)|\les L_\psi |x_1-x_2|,\q \forall x_1,x_2\in\dbR^n;\\ &|\phi(s,y^\prime_1)-\phi(s,y^\prime_2)|\les L_\phi |y^\prime_1-y^\prime_2|,\q \forall y^\prime_1,y^\prime_2\in\dbR^n. \end{align*} We now show that under proper conditions, the above satisfies the monotonicity condition \rf{MC-1}. Indeed, \begin{align*} &\int_t^T \big\lan \h y(s),\, \h f(s,t)\big\ran ds-\Big\lan \h h(t)+\int_t^T\h g(t,s)ds,\, \h x(t) \Big\ran\\ &\q =\int_t^T \Big\lan \h y(s),\, \[A(s,t)\h x(t)+B(s,t)[a(t,x_1(t))-a(t,x_2(t))]\\ &\qq -B(s,t) \int_t^T B(r,t)^\top \h y (r)dr\]\Big\ran ds-\Big\lan\h x(t),\, \[b(t,x_1(t))-b(t,x_2(t))\\ &\qq+\phi\Big(t,\int_t^T B(r,t)^\top y_1(r)dr\Big)-\phi\Big(t,\int_t^T B(r,t)^\top y_2(r)dr\Big)\\ &\qq +\int_t^T\( A(s,t)^{\top} \h y(s)+\psi(t,s,x_1(t))-\psi(t,s,x_2(t))\) ds\]\Big\ran\\ &\q = \Big\lan\int_t^T B(s,t)^\top\h y(s)ds,\, \big[a(t,x_1(t))-a(t,x_2(t))\big]\Big\ran\\ &\qq -\Big| \int_t^T B(r,t)^\top \h y (r)dr\Big|^2-\Big\lan\h x(t),\, \[b(t,x_1(t))-b(t,x_2(t))\\ &\qq+\phi\Big(t,\int_t^T B(r,t)^\top y_1(r)dr\Big)-\phi\Big(t,\int_t^T B(r,t)^\top y_2(r)dr\Big)\]\Big\ran\\ &\qq -\Big\lan\h x(t),\,\int_t^T\big[\psi(t,s,x_1(t))-\psi(t,s,x_2(t))\big] ds\Big\ran\\ &\q \les {1\over 2} \Big| \int_t^T B(r,t)^\top \h y (r)dr\Big|^2+ {1\over 2}L_a^2 |\h x(t)|^2-\Big| \int_t^T B(r,t)^\top \h y (r)dr\Big|^2\\ &\qq -\l|\h x(t)|^2+{1\over 2} \Big| \int_t^T B(r,t)^\top \h y (r)dr\Big|^2+ {1\over 2}L_\phi^2 |\h x(t)|^2+L_\psi T|\h x(t)|^2\\ &\q=-\Big(\l-{1\over 2}L_a^2 -{1\over 2}L_\phi^2-L_\psi T\Big) |\h x(t)|^2,\q t\in[0,T]. \end{align*} Let $$ \l-{1\over 2}L_a^2 -{1\over 2}L_\phi^2-L_\psi T>0, $$ and take $G(\cd)\equiv0$. Then the non-local monotonicity condition \rf{MC-1}--\rf{MC-2} holds. One can see that for the case with only one nonlinear term $b(\cd)$, that is $a(\cd),\phi(\cd),\psi(\cd)=0$, we just need to assume that $\l>0$. Thus, by \autoref{Thm:well-posedness}, \rf{FBVIE-Nonlinear} admits a unique solution. \end{example} \section{Conclusion} The main contribution of this paper is that we provide a method for finding a non-local monotonicity condition for coupled forward-backward Volterra integral equations, under which the system admits a unique solution. From this procedure, we can see the celebrated method of continuation developed for establishing the solvability of FBSDEs (see \cite{Hu-Peng1995,Yong1997,Peng-Wu1999}) is also an optimal control approach in some sense. The stochastic version of coupled FBVIEs has more potential in applications; for example, it can be applied in the popular rough Heston model, and it can provide a probabilistic interpretation for non-local PDEs. A natural problem is to extend the current result to stochastic FBVIEs. Although the main methods introduced in the current paper still work, it becomes more technical. In the current paper, we mainly focus on the basic idea of finding a non-local monotonicity condition, and do not hope to touch too many technical issues. We will report the related results of coupled stochastic FBVIEs separately in the near future. \begin{thebibliography}{99} \bibitem{AIOmari-Gourley2003} J. F. M. Al-Omari and S. A. Gourley, \it Stability and traveling fronts in Lotka-Volterra competition models with stage structure, \rm SIAM J. Appl. Math., {\bf 63} (2003), 2063--2086. \bibitem{Angell1976} T. S. Angell, On the optimal control of systems governed by nonlinear Volterra equations, J. Optim. Theory Appl., {\bf19} (1976), 29--45. \bibitem{Bondi2023} A. Bondi and F. Flandoli, \it On the Kolmogorov equation associated with Volterra equations and Fractional Brownian Motion, \rm arxiv:2309.13597, 2023. \bibitem{Bonesini2023} O. Bonesini, A. Jacquier, and A. Pannier, \it Rough volatility, path-dependent PDEs and weak rates of convergence, \rm arxiv:2304.03042, 2023. \bibitem{Carlson1987} D. A. Carlson, \it An elementary proof of the maximum principle for optimal control problems governed by a Volterra integral equation, \rm J. Optim. Theory Appl., {\bf54} (1987), 43--61. \bibitem{Comte1998} F. Comte and E. Renault, \it Long memory in continuous-time stochastic volatility models, \rm Math. Finance, {\bf8} (1998), 291--323. \bibitem{Cvitanic1996} J. Cvitani\'{c} and J. Ma, \it Hedging options for a large investor and forward-backward SDE's, \rm Ann. Appl. Probab., {\bf 6} (1996), 370--398. \bibitem{Delarue2002} F. Delarue, \it On the existence and uniqueness of solutions to FBSDEs in a non-degenerate case, \rm Stochastic Process. Appl., {\bf 99} (2002), 209--286. \bibitem{ElEuch2018} O. El Euch and M. Rosenbaum, \it Perfect hedging in rough Heston models, \rm Ann. Appl. Probab., {\bf28} (2018), 3813--3856. \bibitem{ElEuch2019} O. El Euch and M. Rosenbaum, \it The characteristic function of rough Heston models, \rm Math. Finance, {\bf 29} (2019), 3--38. \bibitem{Gopalsamy1980} K. Gopalsamy, \it Time lags and global stability in two-species competition, \rm Bull. Math. Biol., {\bf 42} (1980), 729--737. \bibitem{Hamaguchi2021} Y. Hamaguchi, \it Small-time solvability of a flow of forward-backward stochastic differential equations, \rm Appl. Math. Optim., {\bf 84} (2021), 567--588. \bibitem{Hamaguchi2021-1} Y.~Hamaguchi, \it Extended backward stochastic Volterra integral equations and their applications to time-inconsistent stochastic recursive control problems, \rm Math. Control Relat. Fields, {\bf11} (2021), 433--478. \bibitem{Hu-Jin-Zhou2012} Y.~Hu, H.~Jin, and X.~Y.~Zhou, \it Time-inconsistent stochastic linear--quadratic control, \rm SIAM J. Control Optim., {\bf 50} (2012), 1548--1572. \bibitem{Hu-Peng1995} Y.~Hu and S.~Peng, \it Solution of forward-backward stochastic differential equations, \rm Probab. Theory Related Fields, {\bf103} (1995), 273--283. \bibitem{Kot2001} M. Kot, \it Elements of mathematical ecology, \rm Cambridge University Press, Cambridge, 2001. \bibitem{Lin2020} P. Lin and J. Yong, \it Controlled singular Volterra integral equations and Pontryagin maximum principle, \rm SIAM J. Control Optim., {\bf 58} (2020), 136--164. \bibitem{Ma-Protter-Yong1994} J.~Ma, P. Protter, and J.~Yong, \it Sovling forward-backward stochastic differential equations explicitly --- a four step scheme, \rm Probab. Theory Related Fields, {\bf 98} (1994), 339--359. \bibitem{Ma2015} J. Ma et al., \it On well-posedness of forward-backward SDEs---a unified approach, \rm Ann. Appl. Probab., {\bf 25} (2015), 2168--2214. \bibitem{Peng-Wu1999} S. Peng and Z. Wu, \it Fully coupled forward-backward stochastic differential equations and applications to optimal control, \rm SIAM J. Control Optim., {\bf37} (1999), 825--843. \bibitem{Viens-Zhang2019} F.~Viens and J.~Zhang, \it A martingale approach for fractional Brownian motions and related path dependent PDEs, \rm Ann. Appl. Probab., {\bf 29} (2019), 3489--3540. \bibitem{Wang-Yong2021} H.~Wang and J.~Yong, \it Time-inconsistent stochastic optimal control problems and backward stochastic Volterra integral equations, \rm ESAIM Control Optim. Calc. Var., {\bf27} (2021), 22. \bibitem{Wang-Yong-Zhang2022} H.~Wang, J.~Yong, and J.~Zhang, \it Path dependent Feynman--Kac formula for forward backward stochastic Volterra integral equations, \rm Ann. Inst. Henri Poincar\'{e} Probab. Stat., {\bf58} (2022), 603--638. \bibitem{Wang-Yong-Zhou2023} H. Wang, J. Yong, and C. Zhou, \it Linear-quadratic optimal controls for stochastic Volterra integral equations: causal state feedback and path-dependent Riccati equations, \rm SIAM J. Control Optim., {\bf61} (2023), 2595--2629. \bibitem{Yong1997} J.~Yong, \it Finding adapted solutions of forward--backward stochastic differential equations: method of continuation, \rm Probab. Theory Related Fields, {\bf107} (1997), 537--572. \bibitem{Yong2008} J.~Yong, \it Well-posedness and regularity of backward stochastic Volterra integral equations, \rm Probab. Theory Related Fields, {\bf142} (2008), 21--77. \bibitem{Yong2010} J.~Yong, \it Forward-backward stochastic differential equations with mixed initial-terminal conditions, \rm Trans. Amer. Math. Soc., {\bf362} (2010), 1047--1096. \bibitem{Yong-Zhou1999} J.~Yong and X.~Y.~Zhou, \it Stochastic Controls: Hamiltonian Systems and HJB Equations, \rm Springer-Verlag, New York, 1999. \bibitem{Yu2022} Z. Yu, \it On forward-backward stochastic differential equations in a domination-monotonicity framework, \rm Appl. Math. Optim., {\bf85} (2022), 5. \end{thebibliography} \end{document}
2412.04527v1
http://arxiv.org/abs/2412.04527v1
Critical Drift for Brownian Bees and a Reflected Brownian Motion Invariance Principle
\documentclass{article} \usepackage[utf8]{inputenc} \usepackage[margin=3cm]{geometry} \usepackage{amsfonts} \usepackage{amsthm} \usepackage{amsmath} \usepackage{mathtools} \usepackage{amssymb} \usepackage{multicol} \usepackage{cite} \usepackage{caption} \usepackage{subcaption} \usepackage{graphicx} \usepackage{tcolorbox} \usepackage{dsfont} \usepackage{xcolor} \usepackage{tikz-cd} \usepackage{subfiles} \usepackage{longtable} \usepackage{hyperref} \usepackage{bbm} \newtheorem{theorem}{Theorem} \newtheorem*{theorem*}{Theorem} \newtheorem{corollary}[theorem]{Corollary} \newtheorem{conj}[theorem]{Conjecture} \newtheorem{lemma}[theorem]{Lemma} \newtheorem{defn}[theorem]{Definition} \newtheorem{propn}[theorem]{Proposition} \newtheorem*{propn-non}{Proposition} \newenvironment{sproof}{ \renewcommand{\proofname}{Sketch Proof}\proof}{\endproof} \makeatletter \def\blfootnote{\gdef\@thefnmark{}\@footnotetext} \makeatother \newcommand{\B}[1]{\mathbb{#1}} \newcommand{\C}[1]{\mathcal{#1}} \newcommand{\oop}{\preccurlyeq} \newcommand{\is}{\mathbbm{1}} \DeclareMathOperator{\erf}{erf} \DeclareMathOperator{\erfc}{erfc} \newcommand{\midL}{\binom{n}{\lfloor n/2\rfloor}} \renewcommand{\arraystretch}{1.5} \linespread{1.25} \usepackage[skip=7.5pt plus1pt, indent=20pt]{parskip} \newcommand{\JB}[1]{\textcolor{blue!70!black}{\textbf{JB:} #1}} \title{Critical Drift for Brownian Bees and a Reflected Brownian Motion Invariance Principle} \author{Jacob Mercer\footnote{\texttt{[email protected]}, Department of Statistics, University of Oxford}} \date{} \begin{document} \maketitle \begin{abstract} $N$-Brownian bees is a branching-selection particle system in $\B{R}^d$ in which $N$ particles behave as independent binary branching Brownian motions, and where at each branching event, we remove the particle furthest from the origin. We study a variant in which $d=1$ and particles have an additional drift $\mu\in\B{R}$. We show that there is a critical value, $\mu_c^N$, and three distinct regimes (sub-critical, critical, and super-critical) and we describe the behaviour of the system in each case. In the sub-critical regime, the system is positive Harris recurrent and has an invariant distribution; in the super-critical regime, the system is transient; and in the critical case, after rescaling, the system behaves like a single reflected Brownian motion. We also show that the critical drift $\mu_c^N$ is in fact the speed of the well-studied $N$-BBM process, and give a rigorous proof for the speed of $N$-BBM, which was missing in the literature. \end{abstract} \section*{Introduction} \blfootnote{This publication is based on work supported by the EPSRC Centre for Doctoral Training in Mathematics of Random Systems: Analysis, Modelling, and Simulation (EP/S023925/1)} \subfile{sections/intro} \section{Some useful couplings of $N$-BBM and Brownian bees} \subfile{sections/couplings} \section{Association of the $N$-BBM} \subfile{sections/appendixB} \section{The Sub-critical Case: $|\mu|<\mu_c^N$} \subfile{sections/subcritical} \section{The Super-critical Case: $|\mu|>\mu_c^N$} \subfile{sections/supercritical} \section{The Critical Case: $|\mu|=\mu_c^N$} \subfile{sections/critical} \section{Proof of Proposition \ref{pathSpaceTfm} and Proposition \ref{asympSmallTC}} \label{techSec} \subfile{sections/technicals} \newpage \section*{Appendix: The Asymptotic Velocity of the $N$-BBM} \subfile{sections/appendixA} \section*{Acknowledgements} The author thanks Julien Berestycki for his supervision of the project and heplful feedback on the paper, as well as helpful conversations with Bastien Mallein, and constructive feedback from Matthias Winkel and Brett Kolesnik. \bibliographystyle{plain} \bibliography{bps} \subfile{} \end{document} Recall that we are trying to prove here that when $|\mu|=\mu_c^N$, the system satisfies an \textit{invariance principle} $$ \left(m^{-1/2}X^{N,\mu_c^N}_i(mt)\right)_{t\geq 0} \xrightarrow[m\to\infty]{d} (\frac{\mu}{|\mu|}\beta^{-1/2}\sigma |B(t)|)_{t\geq 0} $$ in the Skorokhod topology on $\C{D}([0,\infty),\B{R})$ for any particle $X^{N,\mu}_i$, where $B(t)$ is a standard Brownian motion. We want to further prove that the radius of the cloud of particles converges to zero in the scaling limit; that is $$m^{-1/2}\left(X^{N,\mu_c^N}_N(mt) - X^{N,\mu_c^N}_1(mt)\right) \overset{d}{=} 0.$$ Therefore, in the scaling limit, the cloud of particles behaves like a single point mass moving about as reflected Brownian motion. We start with an analogous result for the $N$-BBM system which we will then couple to the $N$-Brownian bees. For ease of notation, for the rest of this work we will denote the $N$-BBM, $Z^{N,\mu_c^N}$, with positive critical drift $\mu_c^N$ and killing on the right simply as $Z$ and the $N$-Brownian bees $X^{N,\mu_c^N}$ with positive critical drift $\mu_c^N$ simply as $X$. Notice that we want to prove a statement of convergence in the space $\C{D}([0,\infty),\B{R})$, the space of cadlag functions $[0,\infty)\to \B{R}$. However, in it's standard formulation, Donsker's theorem, and the machinery of the paper \cite{bbb} which we use here, are for convergence in the space $\C{D}([0,1],\B{R})$. Therefore before proceeding to the following proposition, we state some results from \cite{bills} which will be needed here. Let \begin{align*} g_m(t)=(m-t)\is_{\{m-1\leq t\leq m\}} + \is_{\{t\leq m-1\}} \end{align*} So for any $m>0$, $\psi_m:(X(t))_{t\geq 0} \mapsto (X(t)g_m(t))_{0\leq t\leq m}$ is a map $\C{D}([0,\infty),\B{R})\to \C{D}([0,m],\B{R})$ such that $X(m)g_m(m)=0$. Then \begin{lemma} \label{DinftyConv} (Lemma 3 in Section 16 of \cite{bills}) A necessary and sufficient condition for $(Y_n(t))_{t\geq 0}$ to converge to $(Y(t))_{t\geq 0}$ in $\C{D}([0,\infty),\B{R})$ is that $\psi_m((Y_n(t))_{t\geq 0})$ converges to $\psi_m((Y(t))_{t\geq 0})$ in $\C{D}([0,m],\B{R})$ for every $m>0$ \end{lemma} \begin{propn} \label{critNBBMInvar} Fix some $j$ in $\{1,\ldots,N\}$. Then $$\left(m^{-1/2}Z_j(mt)\right)_{t\geq 0} \xrightarrow[m\to\infty]{d} (\beta^{-1/2}\sigma B(t))_{t\geq 0}$$ in the Skorokhod topology on $\C{D}([0,\infty),\B{R})$, where $B$ is a standard Brownian motion, and $\beta,\sigma$ are constants which we will define in the course of the proof. \end{propn} \begin{proof} The first part of this proof will follow almost exactly as the proof of Proposition 1.4 in \cite{bbb}, which we will give a brief summary of now. Suppose we have a stochastic process $W$ and a sequence of finite stopping times, $\tau_1,\tau_2,\tau_3,\ldots$ satisfying the following properties \begin{enumerate} \item $\tau_{i+1}-\tau_i$ are i.i.d with non-zero and finite mean for $i\geq 1$ \item $W(\tau_{i+1})-W(\tau_i)$ are i.i.d for $i\geq 1$ \item $\sup_{\tau_i \leq t\leq \tau_{i+1}} |W(t)-W(\tau_i)|$ are i.i.d for $i\geq 1$ \item $\B{E}\left[\sup_{\tau_i\leq t\leq \tau_{i+1}}|W(t)-W(\tau_i)|^2\right]<\infty$ \end{enumerate} then $W$ converges to a Brownian motion under the scaling $m^{-1/2}W(mt)$ in distribution in the Skorokhod topology on $\C{D}([0,\infty),\B{R})$. This is a simple corollary of Proposition 1.4 in \cite{bbb}, who go a step further by extending this result to the give a scaling limit for the barycentre of an $N$ particle system, however for our purposes it will be sufficient to show the four properties above. As in Proposition \ref{finExpNBBM}, we consider the $N$-BBM process at some time $t\in [k,k+1]$, for $k\in \B{N}$ as a function of $Z(k)$ and $N$ i.i.d. BBMs. Therefore for $i=1,\ldots, N$, let $$\C{B}^i_k(t)_{0\leq t\leq 1}=\{B^{i,j}_k(t):j=1,\ldots,\C{N}^i_k(t)\}$$ be as in Proposition \ref{finExpNBBM}, but here with drift $\mu=\mu_c^N$. Let the events $A_k$ and the regeneration times $\tau_i$ also be the same as in Proposition \ref{finExpNBBM}. Now in Proposition \ref{finExpNBBM}, we showed that $\{\tau_{i+1}-\tau_i:i\geq 1\}$ are i.i.d. random variables with finite mean, and that $\{Z(\tau_{i+1})-Z(\tau_i):i\geq 1\}$ are i.i.d. random variables. It is also immediate from construction that $\left\{\sup_{\tau_i\leq t \leq \tau_{i+1}}|Z_j(t)-Z_j(\tau_i)|:i\geq 1\right\}$ is an i.i.d family of random variables. Therefore in order to apply the result of Proposition 1.4 in \cite{bbb} it only remains to prove that \begin{align} \label{sqBound} \B{E}\left[\sup_{\tau_i\leq t \leq \tau_{i+1}}|Z_j(t)-Z_j(\tau_i)|^2\right]<\infty. \end{align} Note that $\sup_{\tau_i \leq t\leq \tau_{i+1}}|Z_j(t)-Z_j(\tau_i)|$ is bounded above by $|Z_N(\tau_i)-Z_1(\tau_i)|$ plus $\lceil \tau_{i+1}-\tau_i\rceil$ random variables distributed like $\max_{i=1}^{\C{N}(1)}\sup_{0\leq t\leq 1}|B^i(t)|$, where $\C{B}(t)=\{B^i(t), i=1,\ldots,\C{N}(t)\}$ is a branching Brownian motion. Therefore by the Cauchy-Schwarz inequality, $\sup_{\tau_i \leq t\leq \tau_{i+1}}|Z_j(t)-Z_j(\tau_i)|^2$ is bounded above by \begin{align}\label{boundforSq} (\lceil \tau_{i+1}-\tau_i \rceil + 1)\left(|Z_N(\tau_i) -Z_1(\tau_i)|^2 + \Omega_1^2 + \ldots \Omega_{\lceil \tau_{i+1}-\tau_i \rceil}^2\right), \end{align} where the $\Omega_i$'s are i.i.d random variables distributed like $\max_{i=1}^{\C{N}(1)}\sup_{0\leq t\leq 1}|B^i(t)|$. Since the maximum of $\C{N}(1)$ positive quantities is bounded above by their sum, thus by the many-to-one lemma: \begin{align*} \B{E}\left[\max_{j=1}^{\C{N}(1)} \sup_{0 \leq t\leq 1}|B^i(t)|^2\right] \leq e\B{E}\left[ \sup_{0\leq t\leq 1}|B(t)|^2\right]. \end{align*} Then by the reflection principle: \begin{align*} \B{E}\left[\sup_{0\leq t\leq 1}|B(t)|^2 \right]&=\int_0^\infty \B{P}(\sup_{0\leq t\leq 1}|B(t)|^2 > x)dx = 2\int_0^\infty \B{P}(B(1)>\sqrt{x})dx = 2\int_0^\infty \erfc(\sqrt{x})dx<\infty \end{align*} Therefore since $\B{E}[(\tau_{i+1}-\tau_i)^2]<\infty$ (because $\tau_{i+1}-\tau_i$ is bounded above by a geometric random variable), thus (\ref{boundforSq}) is finite, and hence (\ref{sqBound}) is finite, as required. Therefore our process $Z$ satisfies conditions (1)-(4) of Proposition 1.4 of \cite{bbb}. Now define $\beta=\B{E}[\tau_2-\tau_1]$ and $\sigma^2=Var(Z_j(\tau_2)-Z_j(\tau_1))$. Fix any $T\in (0,\infty)$. Then by combining the Lemmas 5.2, 5.3 and Theorems 5.1, 5.5 of \cite{bbb}, we can conclude that the following invariance principle holds: $$ \left(m^{-1/2}(Z_j(mt)- mt\B{E}[Z_j(\tau_2)-Z_j(\tau_1)])\right)_{0\leq t\leq T} \xrightarrow[m\to\infty]{d} (\beta^{-1/2}\sigma B(t))_{0\leq t\leq T} $$ in the Skorokhod topology on $\C{D}([0,T],\B{R})$, for standard Brownian motion $B$. Whislt \cite{bbb} states the result only for the space $\C{D}([0,1],\B{R})$; all spaces $\C{D}([0,T],\B{R})$ are essentially analogous, in the sense that the choice of finite interval $[0,T]$ is arbitrary. We now show that $\B{E}[Z_j(\tau_2)-Z_j(\tau_1)]=0$. Suppose for contradiction that $\B{E}[Z_j(\tau_2)-Z_j(\tau_1)]=c\neq 0$. Then $\frac{\B{E}[Z_j(\tau_i)]}{\tau_i}\to \frac{c}{\B{E}[\tau_2-\tau_1]}\neq 0$. This is a contradiction, as $Z$ has asymptotic drift $\lim_{t}Z_j(t)/t=\mu_c^N-\mu_c^N=0$. Thus: $$ \left(m^{-1/2}Z_j(mt)\right)_{0\leq t\leq T} \xrightarrow[m\to\infty]{d} (\beta^{-1/2}\sigma B(t))_{0\leq t\leq T}. $$ Since the map $$\psi_T:(m^{-1/2}Z_j(mt))_{0\leq t\leq T} \mapsto (m^{-1/2}Z_j(mt)g_T(t))_{0\leq t\leq T}$$ is continuous (see the proof of Lemma 3 in Section 16 of \cite{bills}), we can apply the mapping theorem (Theorem 2.7 of \cite{bills}), to get that $$ \psi_T\left((m^{-1/2}Z_j(mt))_{t\geq 0}\right) \xrightarrow[m\to\infty]{d} \psi_T\left((\beta^{-1/2}\sigma B(t))_{t\geq 0}\right), $$ and therefore by Lemma \ref{DinftyConv}, since our choice of $T$ was arbitrary, we have $$ \left(m^{-1/2}Z_j(mt)\right)_{t\geq 0} \xrightarrow[m\to\infty]{d} (\beta^{-1/2}\sigma B(t))_{t\geq 0}, $$ as required. \end{proof} \begin{corollary} \label{nbbmInvarCor} $$\left(m^{-1/2}Z(mt)\right)_{t\geq 0} \xrightarrow[m\to\infty]{d} (\beta^{-1/2}\sigma B(t)\underline{1})_{t\geq 0},$$ in the Skorokhod topology, where $\underline{1}=(1,\ldots,1)\in \B{R}^N.$ \end{corollary} \begin{proof} Let $\tau_1,\tau_2,\tau_3,\ldots$ be the regeneration times from Proposition \ref{critNBBMInvar}. Note that since $\tau_i \to \infty$ as $i\to \infty$ and $Z_N(\tau_i)-Z_1(\tau_i), i\geq 0$ are i.i.d with finite mean, thus we have $\tau_i^{-1/2}|Z_N(\tau_i)-Z_1(\tau_i)|\xrightarrow{\B{P}} 0$. And therefore in the scaling limit, the radius of the particle system goes to zero. Then by Slutsky's theorem, we have \begin{align*}\left( m^{-1/2}Z(mt)\right)_{t\geq 0} &=\left( m^{-1/2}Z_1(mt)\underline{1}\right)_{t\geq 0}+\left(0, m^{-1/2}(Z_2(mt)-Z_1(mt)),\ldots,m^{-1/2}(Z_N(mt)-Z_1(mt)\right)_{t\geq 0}\\ &\xrightarrow[m\to\infty]{d}\left(\beta^{-1/2}\sigma B(t)\underline{1}\right)_{t\geq 0}+(0,\ldots,0)=\left(\beta^{-1/2}\sigma B(t)\underline{1}\right)_{t\geq 0} \end{align*} \end{proof} Given that $Z$, in the scaling limit, converges to a Brownian motion with zero drift, we may naturally expect the properties of the Brownian motions `null-recurrence' to also hold for the $N$-BBM with critical drift, $Z$. Thus in the next Proposition we will show that the expected hitting time of $0$ by $Z_1$, uniformly over all initial configurations with $Z_1(0)=1$, is infinite. This will also be helpful for a later result. \begin{propn} \label{infHitNBBM} Let $\mathfrak{X}^+_{N,1}$ be the set of configurations of $N$ particles with the leftmost particle at $1$. Let $\hat{\tau}:=\inf\{t\geq 0: Z_1(t) < 0\}$ be the first hitting time of $0$ by $Z_1$. Then $\B{E}[\hat{\tau}|Z(0)=\rho]=\infty$ for all $\rho \in \mathfrak{X}^+_{N,1}$. \end{propn} \begin{proof} To prove this result we will consider the process $Z_1$ as a \textit{regenerative process} (see Example 1.3 of \cite{fpMomentsPRW}). Note that $Z$ is a Markov process, but $Z_1$ is not. However, if we consider $Z_1$ specifically at the regeneration times $\tau_1,\tau_2,\ldots$ defined in Proposition \ref{finExpNBBM}, then the process $(Z_1(\tau_i))_{i\geq 1}$ is a Markov process. Therefore define for $n\geq 1$ $$\xi_n :=Z_1(\tau_{n+1}) - Z_1(\tau_n)\text{ and }\eta_n:=\inf_{\tau_{n}\leq t< \tau_{n+1}}Z_1(t)-Z_1(\tau_{n}),$$ so that $\inf_{t\geq \tau_1}Z_1(t) = \inf_{n\geq 1}\{Z_1(\tau_1) + \xi_1 + \cdots \xi_{n-1} + \eta_n\}$. The strategy of the proof will be to show that the hitting time of the barrier $n^{3/8}$ by the mean-centred random walk $S_n := \xi_1 + \cdots + \xi_n$ has infinite expectation and that $\eta_n \leq n^{3/8}$ for all $n$ with positive probability. Putting these two results together we will then therefore be able to show that the hitting time of zero by the process $(Z_1(\tau_1)+S_{n-1} + \eta_n)_{n\geq 1}$ has infinite expectation. By the properties described in Proposition \ref{finExpNBBM}, we know that $S_n:=\xi_1 + \xi_2 + \cdots + \xi_n$ is a random walk with $\B{E}\xi_i = 0$ and $\B{E}\xi_i^2 < \infty$. Therefore by Theorem 3.2 of \cite{permantlePeres} we have that \begin{align}\label{permantlePeresIneq} I := \inf_{n\geq 1} \sqrt{n}\B{P}(S_k \geq k^{3/8} \text{ for } 1 \leq k \leq n) >0.\end{align} Now let us turn our attention to the $\eta_i$'s. By the properties which we proved in Proposition \ref{critNBBMInvar}, we know that $\{\eta_i : i\geq 1\}$ is an i.i.d family of positive random variables with $\B{E}[\eta_i^2]<\infty$. In fact, our proof easily extends to showing that $\B{E}[\eta_i^4]<\infty$. In particular, note that $\tau_{i+1}-\tau_i$ is bounded by a geometric random variable which has bounded fourth moment, and $\B{E}[\sup_{0\leq t\leq 1} |B(t)|^4] = 2\int_0^\infty \erfc(x^{1/4})dx < \infty$. Then just as we bound $\eta_i^2$ by (\ref{boundforSq}), we can use the Cauchy-Schwarz inequality to bound $\eta_i^4$ by $$(\lceil \tau_{i+1} - \tau_i \rceil + 1)^3\left(|Z_N(\tau_i) - Z_1(\tau_i)|^4 + \Omega_1^4 + ... + \Omega_{\lceil \tau_{i+1} - \tau_i \rceil}^4 \right) < \infty.$$ Then as $\B{E}[\eta_i^4] < \infty$, we can prove that $\B{P}(\eta_i \geq -i^{3/8} \; \forall i\geq 1)>0 $. In particular, as the $\eta_i$'s are i.i.d negative random variables, then by Markov's inequality \begin{align*} -\log\left(\B{P}(\eta_i \geq - i^{3/8} \; \forall i \geq 1)\right) &= -\log \left(\prod_{i\geq 1}\B{P}(\eta_i \geq -i^{3/8})\right) = - \sum_{i\geq 1}\log\left(\B{P}(\eta_i \geq - i^{3/8})\right) \\ &=-\sum_{i\geq 1}\log \left(1-\B{P}(\eta_i \leq -i^{3/8})\right) = -\sum_{i\geq 1}\log\left(1-\B{P}(\eta_i^4 \geq i^{3/2})\right) \\ &\leq -\sum_{i\geq 1}\log (1-i^{-3/2}\B{E}[\eta_1^4]) \leq -\sum_{i\geq 1} \frac{-i^{-3/2}\B{E}[\eta_1^4]}{1-i^{-3/2}\B{E}[\eta_1^4]} = \sum_{i\geq 1} \frac{\B{E}[\eta_1^4]}{i^{3/2} - \B{E}[\eta_1^4]} < \infty \end{align*} and therefore $\B{P}(\eta_i \geq -i^{3/8} \; \forall i\geq 1) > 0$. Finally it will remain to show that $$\B{P}(S_j > j^{3/8} \; \forall j\leq n, \eta_j \geq -j^{3/8} \; \forall j\leq n+1) \geq \B{P}(S_j > j^{3/8} \; \forall j\leq n)\B{P}(\eta_j \geq - j^{3/8} \; \forall j\leq n+1).$$ Note that the events $A=\{S_j > j^{3/8} \; \forall j\leq n\}$ and $B=\{\eta_j \geq -j^{3/8} \; \forall j\leq n\}$ are both increasing in the sense that if $X(t)\oop X'(t)$ for all $t\geq 0$ and the event $A$ (resp. $B$) occurs for the process $X$, then certainly the event $A$ (resp. $B$) occurs for the process $X'$. We prove in Lemma \ref{nbbmAssoc} that for any $T>0$ the $N$-BBM process $(Z(t))_{0\leq t\leq T}$ is associated and therefore by the definition of association of random variables, increasing events are positively correlated and therefore \begin{align*} \B{P}(S_j > j^{3/8} \; \forall j\leq n, \eta_j \geq -j^{3/8} \; \forall j\leq n+1) &\geq \B{P}(S_j > j^{3/8} \; \forall j\leq n)\B{P}(\eta_j \geq - j^{3/8} \; \forall j\leq n+1) \\ &\geq \frac{I}{\sqrt{n}}\B{P}(\eta_j \geq -j^{3/8}\forall j\geq 1)\end{align*} where the second inequality follows from the fact that $\B{P}(\eta_j \geq -j^{3/8} \forall j\leq n)\geq \B{P}(\eta_j \geq j^{3/8} \forall j\geq 1)$ and (\ref{permantlePeresIneq}). Define $\sigma:=\inf\{n>0: S_{n-1} + \eta_n \leq 0\}$ to be the hitting time of $0$ by the perturbed random walk $(S_n + \eta_n)_{n\geq 1}$. Then: \begin{align*} \B{E}\sigma &= \sum_{n\geq 1}\B{P}(\sigma \geq n) = \sum_{n\geq 1}\B{P}(S_{j-1} + \eta_j >0 \; \forall j\leq n) \\ & \geq \sum_{n\geq 1}\B{P}(S_j > j^{3/8} \; \forall j\leq n-1, \eta_j \geq -j^{3/8} \;\forall j\leq n) \\ & \geq \sum_{n\geq 1}\frac{I}{\sqrt{n}}\B{P}(\eta_j \geq -j^{3/8}\; \forall j\geq 1) = \infty \end{align*} Now suppose that $Z_1(\tau_1) > 0$. Therefore since $\B{E}\sigma = \infty$, $\B{E}[\tau_{i+1}-\tau_i]\in (0,\infty)$, and $\inf_{t\geq \tau_1}Z_1(t) = \inf_{n\geq 1}Z_1(\tau_1)+S_n + \eta_n$, thus it follows that certainly $\B{E}[\inf\{t\geq \tau_1: X_1(t)<0\}]=\infty$. We can therefore conclude that, since for every $\rho\in \mathfrak{X}_{N,1}^+$ we have $Z_1(\tau_1)>0$ with positive probability, we have that $\B{E}[\hat{\tau}|Z(0)=\rho]=\infty$ for every initial condition $\rho\in \mathfrak{X}^+_{N,1}$ \end{proof} Next we prove that an $N$-Brownian bees system with positive critical drift asymptotically spends proportion 1 of its time with all particles in $[0,\infty)$. \begin{lemma} \label{LRExcursions} Let $X$ be $N$-Brownian bees with critical positive drift. Then $$ t^{-1}\int_0^t \is_{X_1(u)<0}du \xrightarrow[t\to\infty]{a.s.}0$$ \end{lemma} \begin{proof} Let $S:=\inf\{s\geq 0: X_1(s)\leq 0\}$ be the first time that $X_1(s)$ exits $(0,\infty)$. Then observe that $$t^{-1}\int_0^t \is_{X_1(u)<0}du \leq t^{-1}\int_S^{S+t} \is_{X_1(u)<0}du$$ so we can assume without loss of generality that $X_1(0)\leq 0$. Now consider the following sequence of stopping times. Fix $\tau_0=0$ and for $i\geq 1$, recursively define $\sigma_{i}:=\inf\{t\geq \tau_{i-1}:X_1(t)\geq 1\}$ and $\tau_{i}:=\inf\{t\geq \sigma_i:X_1(t)\leq 0\}$. Define $N_t:=\max\{n:\tau_n \leq t\}$ to be the index of the most recent stopping time $\tau_i$. Note that all the time that $X_1$ spends in $(-\infty,0]$ is contained in the intervals $[\tau_{i-1},\sigma_i]$ for $i\geq 1$. So then \begin{equation}\label{leftExcBound}t^{-1}\int_0^t \is_{X_1(u)<0}du \leq t^{-1}\sum_{i=1}^{N_t+1} (\sigma_i - \tau_{i-1}) = \frac{N_t+1}{t} \times \frac{1}{N_t +1}\sum_{i=1}^{N_t+1}(\sigma_i - \tau_{i-1}).\end{equation} Therefore the strategy of this proof will be to show that $\B{E}_\xi[\tau_i - \sigma_i]=\infty$ uniformly for all configurations of $X(\sigma_i)$ and that $\B{E}_\xi[\sigma_i - \tau_{i-1}]<\infty$ uniformly for all configurations of $X(\tau_{i-1})$. Then by renewal theory, the first term of the right hand side of (\ref{leftExcBound}) converges to zero and the second term converges to a finite limit, so that that we have the desired convergence to zero by the algebra of limits. Recall that $\mathfrak{X}^+_{N,1}$ is the set of configurations of $N$ particles with the leftmost particle at 1. Now, before $X_1$ hits 0, the process $X$ has the same behaviour as an $N$-BBM. Therefore by Proposition \ref{infHitNBBM}, $\B{E}[\tau_i - \sigma_i |X(\sigma_i)=\xi]=\infty$ for all $\xi \in \mathfrak{X}^+_{N,1}$, which is the set of possible configurations of $X(\sigma_i)$. We now consider the renewal process $N'_t$ with renewal times distributed like $\tau_i-\tau_{i-1}|X(\sigma_i)=\underline{1}$. Since we can couple so that $\{\tau_i - \tau_{i-1} | X(\sigma_i)=\underline{1}\} \leq \{\tau_i - \tau_{i-1} |X(\sigma_i)=\xi\}$ for any $\xi \in \mathfrak{X}^+_{N,1}$, almost surely, therefore we can couple $(N_t)_{t\geq 0}$ and $(N'_t)_{t\geq 0}$ so that $N_t \leq N'_t$ by coupling the inter-renewal times of $N_t, N'_t$. Therefore by standard renewal theory (see Theorem 2.4.7 in \cite{durrett}), $(N_t+1)/t \leq (N'_t+1)/t \overset{\text{a.s.}}{\to} 0$. We now prove that $\frac{1}{N_t+1}\sum_{i=1}^{N_t+1}(\sigma_i - \tau_{i-1}) \to L<\infty$, by showing that $\sup_{\xi\in \mathfrak{X}^+_{N,0}}\B{E}_\xi[\sigma_i - \tau_{i-1}]<\infty$. To do this we will consider the times at which the particle closest to the origin is in $[0,\infty)$. For this part of the proof, it will be preferable to think of the $N$-Brownian bees in terms of their intrinsic labelling; that is, the process $V^N(t)$ instead of the functional $X^N(t)=\Theta^N(V^N(t))$. We will denote by $V$ the $N$-Brownian bees $V^N$ with critical positive drift $\mu_c^N$. Fix $i$ and define the stopping times $\tau'_{i,0},\tau'_{i,1},\tau'_{i,2},\tau'_{i,3},\ldots$ inductively by $\tau_{i,0}' = \tau_i$ and for $j\geq 0$, $$\tau'_{i,j+1}:=\inf\{s > \tau'_{i,j}: V_{\ell_0}(s)\in [0,\infty), s-\tau_i\in \B{N}\},$$ where $\ell_0=\ell_0(s)$ is the index (under the intrinsic labeling) of the particle closest to the origin at time $s$. As in the proof of Propositions \ref{finExpNBBM},\ref{critNBBMInvar}, we will consider our process, here the $N$-Brownian bees, as being embedded in $N$ independent branching Brownian motions. Therefore for $\ell=1,\ldots, N$, let $\C{B}^\ell_j(t)_{0\leq t\leq 1}=\{B^{\ell,m}_j(t):m=1,\ldots,\C{N}^\ell_j(t)\}$ be as in Proposition \ref{finExpNBBM}. $(\C{B}_j^\ell(t))_{0\leq t\leq 1}$ will be the BBM attached, over the interval $[\tau'_{i,j},\tau'_{i,j}+1]$, to the particle which is $\ell$\textsuperscript{th} smallest at time $\tau'_{i,j}$. As before, particles will have 2 types (`alive' and `ghost') and over the interval $[\tau'_{i,j},\tau'_{i,j}+1]$ will behave according to the following dynamics: \begin{itemize} \item At time $\tau'_{i,j}$, all particles have type `alive'. \item When an `alive' particle branches, it branches into 2 `alive' particles, and simultaneously the `alive' particle furthest from the origin (which may be one of the two particles involved in the branching event) is changed from type `alive' to type `ghost'. \item When a `ghost' particle branches, it branches into 2 `ghost' particles. \end{itemize} Then the $N$-Brownian bees systems is described by the set of $N$ `alive' particles. Explicitly, for $t\in [\tau_{i,j}',\tau_{i,j}'+1]$, write $$V(t)=\Psi(V(\tau_{i,j}'),\{(\C{B}^\ell_j(t))_{0\leq t\leq 1}:\ell=1,\ldots,N\})$$ to denote that $V(t)$ is a function of $V(\tau_{i,j}')$ and the $N$ specified and independent BBMs. Recall that at time $\tau'_{i,j}$, the particle closest to zero has index $\ell_0(\tau'_{i,j})$. Then we can now consider the event $E_j=E^{(1)}_j\cap E^{(2)}_j\cap E^{(3)}_j\cap E^{(4)}_j$, where \begin{align*} E^{(1)}_j &= \{\C{N}_j^{\ell_0(\tau'_{i,j})}(1/2)=\C{N}_j^{\ell_0(\tau'_{i,j})}(1)=N, \C{N}_j^{\ell}(1)=1\text{ for }\ell\neq \ell_0(\tau'_{i,j})\}\\ E^{(2)}_j &= \{\text{At each branching time }T_1,\ldots T_{N-1}\text{ of }(\C{B}^{\ell_0(\tau'_{i,j})}_j(t))_{0\leq t\leq 1}\text{ we have }\\ &\quad \quad \quad \text{sign}(V_\ell(\tau'_{i,j}))B^{\ell, 1}_j(T_m - \tau'_{i,j})>1\text{ for }\ell\neq \ell_0(\tau'_{i,j})\text{ and }m=1,\ldots,N-1\}\\ E^{(3)}_j &= \{0>B^{\ell_0(\tau'_{i,j}),\ell}_j(T_m-\tau'_{i,j})>-1\text{ and }0>B_j^{\ell_0(\tau'_{i,j}),\ell}(1/2)>-1\text{ for }m=1,\ldots,N-1\\ &\quad \quad \quad \text{ and }\ell=1,\ldots,\C{N}^{\ell_0(\tau'_{i,j})}_j(T_m - \tau'_{i,j})\}\\ E^{(4)}_j &= \{B^{\ell_0(\tau'_{i,j}),\ell}_j(1))>B^{\ell_0(\tau'_{i,j}),\ell}_j(1/2)+2\text{ for }\ell=1,\ldots,N\} \end{align*} In laymans terms, this is the event that particle $V_{\ell_0(\tau_{i,j}')}(\tau_{i,j}')$ branches $N-1$ times, all in the interval $[\tau'_{i,j},\tau'_{i,j}+1/2]$ whilst no other particle branches (the event $E^{(1)}_j$). During the interval $[\tau'_{i,j},\tau'_{i,j}+1/2]$, the particle $\ell_0(\tau_{i,j}')$ and all of it's descendants stay within $(V_{\ell_0(\tau_{i,j}')}(\tau'_{i,j})-1,V_{\ell_0(\tau_{i,j}')}(\tau'_{i,j}))\subseteq (-1,\infty)$ at branching events (the event $E^{(3)}_j$), whilst all other particles get further from the origin by distance at least $1$ at branching events (the event $E^{(2)}_j$). This means that at each branching time, the `alive' particle furthest from the origin is not among the descendants of particle $V_{\ell_0(\tau_{i,j}')}$, and therefore at time $\tau'_{i,j}+1/2$, the $N$ `alive' particles in the system are exactly the $N$ descendants of $V_{\ell_0(\tau_{i,j}')}$. Subsequently, the event $E^{(4)}_j$ ensures that all of these `alive' particles move at least $2$ units to the right, so that all the `alive' particles are in $[1,\infty)$. As in our construction in Proposition \ref{critNBBMInvar}, the probability of this event occurring is independent on the initial configuration of the system, therefore $\B{P}(E_j)=q_0>0$ uniformly for all $j$. Finally, we consider the time increments $\tau'_{i,j+1}-\tau'_{i,j}$. We want to prove that $$\B{E}[\tau'_{i,j+1}-\tau'_{i,j}]<\infty.$$ To see this, note that if at time $t_0$, we have $V_{\ell_0(t_0)}(t_0)<0$, then whilst $V_{\ell_0(t_0)}(t)<0$, $V_{\ell_0(t_0)}(t)$ is bounded below by a Brownian motion with drift $\mu_c^N$. This is because particles, under the intrinsic labelling, only ever jump closer to the origin. As a Brownian motion with positive drift exits $(-\infty,0]$ in finite expected time, certainly the expected time it takes until $V_{\ell_0(t)}(t)$ exits $(-\infty,0]$ is also finite. Therefore $\B{E}_\xi[\tau'_{i,j+1}-\tau'_{i,j}]<M<\infty$ uniformly for any starting configuration $V(\tau'_{i,j})=\xi\in \mathfrak{X}^+_{N,0}$. Therefore uniformly over initial configurations $X(\tau_{i})=\xi\in \mathfrak{X}^+_{N,0}$, we have $ \B{E}_\xi[\sigma_{i+1} - \tau_{i}] \leq M\B{E}[Geom(q_0)]<\infty$. Therefore by the strong law of large numbers $$\lim_{t\to\infty}\frac{1}{N_t+1}\sum_{i=1}^{N_t+1}(\sigma_{i+1} - \tau_i)\leq M\B{E}[Geom(q_0)] < \infty \quad a.s.,$$ Hence we can conclude that the right hand side of equation (\ref{leftExcBound}) almost surely converges to $0$ as $t\to\infty$, and thus the claim holds. \end{proof} Next we state a technical result about the almost sure continuity (with respect to Brownian motion) of a transformation of path space. In particular, the transformation is one which takes a path $[0,\infty)\to \B{R}$ and transforms it into a path $[0,\infty)\to [0,\infty)$ by removing the excursions in the negative half-line. Specifically, this is the transformation $g$ defined by: $$g:(Z(t))_{t\geq 0}\mapsto \left( Z(\inf\{s\geq 0: \int_0^s \is_{Z(u)\geq 0}du \geq t\})\right)_{t\geq 0}$$ The motivation behind this technical result is that it will allow us to apply the mapping theorem (Theorem 2.7 in \cite{bills}), which states that if $A_n \to A$ in distribution and the transformation $g$ has a discontinuity set $\C{D}_g$ such that $\B{P}(A\in \C{D}_g)=0$, then $g(A_n)\to g(A)$ in distribution. Since our random variables are cadlag paths, we must prove that our transformation is almost surely continuous under the Skorokhod metric on $\C{D}([0,\infty),\B{R})$. The transformation described above is chosen because it transforms a Brownian motion into a reflected Brownian motion; therefore by the mapping theorem and Proposition \ref{critNBBMInvar}, the continuity result will give that: $$g((m^{-1/2}Z_i(mt))_{t\geq 0}) \xrightarrow[m\to\infty]{d} (\beta^{-1/2}\sigma|B(t)|)_{t\geq 0}$$ in the Skorokhod topology on $\C{D}([0,\infty),\B{R})$ for any $i=1,2,\ldots,N$. Before stating the next proposition, we recall the definition of the Skorokhod metric $d_T$ on the space $\C{D}([0,T],\B{R})$ for $T>0$ and the Skorokhod metric $d_\infty$ on $\C{D}([0,\infty),\B{R})$. \begin{defn} Fix $T>0$ and let $\Lambda_T$ be the set of continuous and strictly increasing bijections $[0,T]\to[0,T]$. Then for $X,Y\in \C{D}([0,T],\B{R})$ \begin{align*} d_T(X,Y)&:=\inf_{\lambda \in \Lambda_T}\left\{\sup_{t\in [0,T]}|t-\lambda(t)|\vee \sup_{t\in [0,T]}|X(t)-Y(\lambda(T))|\right\} \end{align*} and for $X,Y\in \C{D}([0,\infty),\B{R})$ \begin{align*} d_\infty(X,Y)&:=\sum_{T\in\B{N}}^\infty 2^{-T}(1\wedge d_T(\psi_T(X),\psi_T(Y))). \end{align*} \end{defn} We can now define precisely our transformation and state the following proposition \begin{propn} \label{pathSpaceTfm} Let $(W(t))_{t\geq 0}$ be a stochastic process whose sample paths are in $\C{D}([0,\infty),\B{R})$, the space of cadlag paths $[0,\infty)\to \B{R}$. Let $g:\C{D}([0,\infty),\B{R})\to\C{D}([0,\infty),\B{R})$ be the map given by: $$ g:(W(t))_{t\geq 0} \mapsto \left(W(\inf\{s\geq 0: \int_0^s \is_{W(u) \geq 0} du \geq t\})\right)_{t\geq 0}. $$ Let $D_g\in \C{D}([0,\infty),\B{R})$ be the discontinuity set of $g$ under the metric $d_\infty$. Then $\B{P}((B_t)_{t\geq 0} \in D_g)=0$, where $B$ is a standard Brownian motion. \end{propn} The final result we need before we can prove Theorem \ref{mainThem}.2 is the following, which essentially states that changing a stochastic process by an asymptotically small time change doesn't change the scaling limit. In particular: \begin{propn} \label{asympSmallTC} Let $(W(t))_{t\geq 0}$ be a stochastic process such that $(m^{-1/2}W(mt))_{t\geq 0}$ converges in distribution as $m\to\infty$ to $(B(t))_{t\geq 0}$ (resp. $(|B(t)|)_{t\geq 0}$) in the Skorokhod topology on $\C{D}([0,\infty),\B{R})$, where $B$ is a standard Brownian motion. Suppose that for every $T>0$ we have $$\sup_{0\leq t\leq T}|m^{-1}\alpha(mt)|\xrightarrow[m\to\infty]{\B{P}} 0.$$ Then $\left(m^{-1/2}W(mt+\alpha(mt))\right)_{t\geq 0}\xrightarrow[m\to\infty]{d}(B(t))_{t\geq 0}$ (resp. $(|B(t)|)_{t\geq 0}$) in the Skorohod topology on $\C{D}([0,\infty),\B{R})$ \end{propn} The proofs of these two results are technical, and so we postpone them until section \ref{techSec}. We are now ready to prove an invariance principle to a reflected Brownian motion, which was the second regime in Theorem \ref{mainThem}. \begin{theorem} Let $X$ be the Brownian bees system with critical drift. Then we have the invariance principle: $$ \left(m^{-1/2}X_1(mt)\right)_{t\geq 0} \xrightarrow[m\to\infty]{d} \left(\beta^{-1/2}\sigma|B(t)|\right)_{t\geq 0}, $$ in the Skorokhod topology on $\C{D}([0,\infty),\B{R})$, for constants $\beta,\sigma$ defined in Proposition \ref{critNBBMInvar}. \end{theorem} \begin{proof} The idea of this proof is the following: since the Brownian bees model with critical drift spends asymptotically zero time to the left of the origin, it can be coupled (up to an asymptotically small time shift) to the image by a transformation similar to $g$ of an $N$-BBM process with critical drift. The invariance principle for the $N$-BBM $Z$, and the mapping theorem with the almost surely continuous map $g$ then yield the conclusion. Recall that $Z$ is the $N$-BBM with killing on the right and critical drift of $\mu_c^N$. Let $\tilde{Z}$ be a time-change of $Z$ in which we remove the excursions where $Z_1<0$; that is $$(\tilde{Z}(t))_{t\geq 0}:=\left(Z(\inf\{s\geq 0: \int_0^s \is_{Z_1(u)\geq 0}du \geq t\})\right)_{t\geq 0}.$$ Let $\tilde{Z}_1$ denote the position of the leftmost particle of $\tilde{Z}$ at time $t$ and note that $\tilde{Z}_1=g(Z_1)$. Therefore by the mapping theorem (Theorem 2.7 in \cite{bills}) and Proposition \ref{pathSpaceTfm} \begin{align}\label{ztildeConv}\lim_{m\to\infty}(m^{-1/2}\tilde{Z}_1(mt))_{t\geq 0} =g\left(\lim_{m\to\infty}(m^{-1/2}Z_1(mt))_{t\geq 0}\right)=g((\beta^{-1/2}\sigma B(t))_{t\geq 0}) \overset{d}{=} (|\beta^{-1/2}\sigma B(t)|)_{t\geq 0}.\end{align} We will now carefully construct regeneration times of $X$, by defining events $C_k$ for $k\in \B{N}$ in which the leftmost particle of $X$ is positive and, in some suitably controlled way, `regenerates' the system by becoming the parent of every particle. As with the proofs of Propositions \ref{finExpNBBM}, \ref{critNBBMInvar} and Lemma \ref{LRExcursions}, we will describe the regeneration event by considering the $N$-Brownian bees system as being embedded in $N$ independent $N$-BBMs. Therefore for $i=1,\ldots,N$, $k=1,2,\ldots$, and $t\geq 0$, let $\C{B}^i_k(t)=\{B^{i,j}_k(t),j=1,\ldots,\C{N}_k^i(t)\}$ be as in Proposition \ref{LRExcursions}, with the same dynamics of `alive' and `ghost' particles describing the behaviour of $X$; that is, defined so that at any time $t\in [k,k+1]$, $X(t)$ is given by the positions of the $N$ `alive' particles in the system and can be explicitly writen as $$X(t)=\Psi(X(k),\{(\C{B}^i_k(t))_{0\leq t\leq 1}:i=1,\ldots,N\}).$$ Again, let the branching Brownian motion $(\C{B}_k^i(t))_{0\leq t\leq 1}$ drive the particle which is the $i$\textsuperscript{th} largest at time $k$. Now we can describe the `regeneration' events, $C_k$, of the process $X$, in terms of the branching Brownian motions and the position of the leftmost particle. Let $C_k:=C_k^{(1)}\cap C_k^{(2)}\cap C_k^{(3)}\cap C_k^{(4)}\cap C_k^{(5)}\cap C_k^{(6)}$, where \begin{align*} C^{(1)}_k &= \{X_1(k-1)\geq 1\}, \\ C^{(2)}_k &= \{\C{N}_{k-1}^1(1)=N\}, \\ C^{(3)}_k &= \{\C{N}_{k-1}^j(1)=1\text{ for }j\neq 1\}, \\ C^{(4)}_k &= \{\text{At each branching time $T_1,\ldots,T_{N-1}$ of $(\C{B}^1_{k-1}(t))_{0\leq t\leq 1}$, we have $B^{1,j}_{k-1}(T_\ell)<0$ for }\\ &\quad \quad \quad \text{for $\ell=1,\ldots,N-1$ and $j=1,\ldots, \C{N}^1_{k-1}(T_\ell)=\ell+1$}\}, \\ C^{(5)}_k &= \{B^{i,1}_{k-1}(T_\ell)>0\text{ for }\ell=1,\ldots,N-1\text{ and }i\neq 1\}, \\ C^{(6)}_k &= \{\min_{1\leq i\leq N}\min_{1\leq j\leq \C{N}^i_{k-1}(1)}\inf_{t\in [0,1]}B^{i,j}_{k-1}(t)>-1/2 \}, \end{align*} and define the regeneration times of $X$ as $\tau_0^X:=0$ and subsequently $$\tau_{i+1}^X:=\inf\{k\in \B{N}:k>\tau_i^X,\text{ and }C_k\text{ occurs}\}.$$ In layman's terms, the event $C_k$ is an event in which the leftmost particle of the system, whilst to the right of the origin, becomes ancestor to every particle in the system over $1$ time unit. Specifically, we start with the leftmost particle in $[1,\infty)$ at time $k-1$ (the event $C^{(1)}_k$) and ask that it branches $N-1$ times in $[k-1,k]$ (the event $C^{(2)}_k$) whilst no other particle branches (the event $C^{(3)}_k$). Then the events $C^{(4)}_k$ and $C^{(6)}_k$ ensure that at each branching time $T_\ell$ of the BBM $\C{B}^1_k$, all descendants of the particle $X_1(k-1)$ are in the interval $[X_1(k-1)-1,X_1(k-1)]\subseteq [1/2,\infty)$, and that at all times \textit{all} particles remain in $[1/2,\infty)$. The event $C_k^{(5)}$ ensures that particles not descended from $X_1(k-1)$, at the branching times, are to the right of their initial position. Together, these conditions ensure that at each branching time, the particle furthest from the origin is not a descendant of the leftmost particle $X_1(k-1)$, and so at time $k$, conditional on the event $C_k$, the $N$ particles alive in the system are all descendants of particle $X_1(k-1)$. We now make the following observation, which demonstrates why we defined the event $C_{k}$ as we did. The events $C^{(2)}_k,\ldots,C^{(6)}_k$ are all independent of the initial condition $X(k-1)$ and depend only on the BBMs driving the system. Therefore the function $\is_{C_k}$ is a function only of $X_1(k-1)$ and the BBMs $\C{B}^1_{k-1},\ldots,\C{B}^N_{k-1}$. Note that, as we see in the proof of Proposition \ref{LRExcursions}, the expected time between successive events such that $\{X_1(t)\geq 1\}$ is finite, say with mean bounded above by $M$ uniformly for all configurations. Furthermore, at each time $k\in \B{N}$ such that $\{X_1(k-1)\geq 1\}$, the event $C_k$ occurs with a non-zero probability $p_0>0$, uniformly for all configurations $X(k-1)$. The number of occasions that $X_1(k-1)\geq 1$ until $C_k$ occurs is therefore a geometric random variable with mean $p_0^{-1}$, therefore certainly we have $\B{E}_\xi[\tau_{i+1}^X-\tau_i^X]\leq M\B{E}[Geom(p_0)]$ for all possible configurations $X(\tau_i^X)=\xi$. Given the process $(X(t))_{t\geq 0}$ we will now construct the process $Z$ (and by extension $\tilde{Z}$) from $X$ so that $Z$ is indeed an $N$-BBM and is coupled to $X$. \textbf{Case 1:} If $X_1(t)\geq 0$ for all $t\in [\tau^X_0,\tau^X_1]$, then $X$ behaves exactly as an $N$-BBM process staying always above zero. Therefore define $(Z(t))_{\tau_0^X \leq t\leq \tau_1^X}:=(X(t))_{\tau_0^X \leq t\leq \tau_1^X}$, and $\tau^Z_1:=\tau_1^X$. Therefore certainly $X(\tau_1^X)=\tilde{Z}(\tau_1^X)$ and $Z$ behaves like an $N$-BBM on $[\tau_0^X,\tau_1^X]$ \textbf{Case 2:} If $X_1$ hits $0$, say for the first time at $\theta_1 \in [\tau^X_0,\tau^X_1]$, then define $(Z(t))_{\tau_0^X \leq t\leq \theta_1}:=(X(t))_{\tau_0^X \leq t\leq \theta_1}$. As all particles are above the origin in this interval, $Z$ certainly behaves like an $N$-BBM. Then on $[\theta_1,\tau_1^X-1]$, we can couple $X$ to an $N$-BBM process $Z$ as in Proposition \ref{generalCouple}, so that $Z(\theta_1)=X(\theta_1)$ and $Z(t)\oop X(t)$ for $t\in [\theta_1,\tau^X_1-1]$. Note that by definition of $\tau_1^X$, $X_1$ must stay positive in $[\tau_1^X-1,\tau_1^X]$, so certainly $\theta_1 < \tau_1^X -1$. Now we consider the position $X_1(\tau_1^X-1)$, the position at which $X_1$ begins the next regeneration event. By our coupling (Proposition \ref{generalCouple}), we certainly have $Z_1(\tau^X_1-1)\oop X_1(\tau^X_1-1)$. So now we continue to run the process $Z$ as an independent $N$-BBM until the time $\theta_2$ at which $Z_1(\theta_2)=X_1(\tau_1^X-1)$. Finally, for $t\in [\theta_2,\theta_2+1]$, we define $Z(t)=\Psi(Z(\theta_2),\{(B^i_{\theta_2}(t))_{0\leq t\leq 1}:i=1,\ldots,N\})$, and $$\tau_1^Z=\inf\left\{s\geq 0: \int_0^s\is_{Z_1(u)\geq 0}du \geq \theta_2+1\right\}.$$ By the construction of our event $C_{\tau_1^X}$, all particles of $X$ stay above $0$ in $[\tau_1^X-1,\tau_1^X]$, and therefore all particles of $Z$ stay above $0$ in $[\theta_2,\theta_2+1]$. Therefore $(Z(t))_{t\in (\theta_2,\theta_2+1]}$ is exactly $(\tilde{Z}(t))_{t\in (\tau_1^Z-1,\tau_1^Z]}$, and $Z$ behaves exactly as an $N$-BBM process. First we claim that $\B{E}_\xi|\tau_1^X - \tau_1^Z|$ is bounded uniformly for all initial conditions $X(\tau_0^X)=\tilde{Z}(\tau_0^Z)=\xi$. As above, we know that $\B{E}_\xi[\tau_{i+1}^X - \tau_i^X]\leq M\B{E}[Geom(p_0)]$. Then the boundedness of $\B{E}_\xi[\tau_1^Z]$ follows because, in case 2, $\tau_1^Z$ is at most $\theta_1$ plus the hitting time of $X_1(\tau_1^X -1)>0$ by $\tilde{Z}_1$, plus $1$. $[0,X_1(\tau_1^X-1)]$ is a compact interval, and therefore the expected hitting time of $X_1(\tau_1^X-1)$ by $\tilde{Z}_1$ is finite. Therefore $\B{E}_\xi|\tau_1^X - \tau_1^Z|$ is uniformly bounded for all $\xi$. Next we claim that $X(\tau_1^X)=\tilde{Z}(\tau_1^Z)$. Since the event $C_{\tau_1^X-1}$ is dependent only on the driving BBMs in $[\tau_1^X-1,\tau_1^X]$ and the position $X_1(\tau_1^X-1)=\tilde{Z}(\tau^Z_1-1)$, therefore our coupling ensures that: $$X(\tau^X_1)=\Psi(X(\tau_1^X-1),\{(\C{B}^i_{\tau_1^X-1}(t))_{0\leq t\leq 1}:i=1,\ldots,N\}) = \Psi(\tilde{Z}(\tau_1^Z-1),\{(\C{B}^i_{\tau_1^Z-1}(t))_{0\leq t\leq 1}:i=1,\ldots,N\}) = \tilde{Z}(\tau^Z_1).$$ Continuing in the same way inductively, we can define stopping times $\tau_i^X$ and $\tau_i^Z$ for $i\geq 2$ so that $i\geq 0$ \begin{align}\label{xzTildeCoupling} X(\tau_i^X)=\tilde{Z}(\tau_i^Z) = \tilde{Z}\left(\tau_i^X + \sum_{j=1}^i ((\tau_j^Z-\tau_{j-1}^Z)-(\tau_j^X-\tau_{j-1}^X))\right). \end{align} and that, whenever $X_1$ exits $(0,\infty)$ in $[\tau_{j-1}^X,\tau_j^X]$, we have $\B{E}_\xi\left[|(\tau_j^Z-\tau_{j-1}^Z)-(\tau_j^X-\tau_{j-1}^X)|\right]<E_{\text{bound}}<\infty$ uniformly for all initial conditions $X(\tau_{j-1}^X)=\tilde{Z}(\tau_{j-1}^Z)=\xi$. Now let $\tau^X(mt)$ be the smallest regeneration time $\tau_i^X$ after $mt$, and let $I(mt)$ denote its index. Let $$\alpha(mt):=\sum_{i=1}^{I(mt)}\left((\tau_i^Z-\tau_{i-1}^Z) - (\tau_i^X-\tau_{i-1}^X)\right),$$ so that we can write $X(\tau^X(mt))=\tilde{Z}(\tau^X(mt)+\alpha(mt))$. Fix $T\in (0,\infty)$. Using the same ideas as in the proof of Lemma \ref{LRExcursions}, we will now show that $\sup_{0\leq t\leq T}|m^{-1}\alpha(mt)|\xrightarrow[m\to\infty]{\B{P}}0.$ Define $N_{mt}:=\sum_{j=1}^{I(mt)}\is_{(\tau_j^Z-\tau_{j-1}^Z)\neq (\tau_j^X-\tau_{j-1}^X)}$. So $N_{mt}$ counts the number of intervals $[\tau_{i-1}^X,\tau_i^X]$ up to $[\tau^X_{I(mt)-1},\tau^X_{I(mt)}]$ in which $X_1$ exits $(0,\infty)$, and hence in which $(\tau^Z_j-\tau^Z_{j-1})$ and $(\tau^X_j-\tau^X_{j-1})$ are not necessarily equal. Note that at each time $\tau_i^X$, we have $X_1(\tau_i^X)>1/2$, so we can bound $N_{mt}$ above by a renewal process whose inter-arrival times are distributed like the hitting time of $0$ started from the initial configuration $(1/2,\ldots,1/2)$. By Proposition \ref{infHitNBBM}, these inter-arrival times have infinite expectation, so by standard renewal theory, $N_{mt}/mt\xrightarrow[t\to\infty]{a.s.} 0$. Combining this with the fact above that $\B{E}_\xi[|(\tau_j^Z - \tau_{j-1}^Z) - (\tau_j^X-\tau_{j-1}^X)|]<E_{\text{bound}}$, the strong law of large numbers gives that $$\lim_{t\to\infty}\frac{1}{N_{mt}}\sum_{i=1}^{I(mt)}\left|(\tau_i^Z-\tau_{i-1}^Z) - (\tau_i^X-\tau_{i-1}^X)\right|<E_{\text{bound}}\quad a.s.,$$ Therefore putting these convergence results together \begin{align*}\sup_{0\leq t\leq T}|m^{-1}\alpha(mt)|\leq \frac{N_{mt}}{m} \times \frac{1}{N_{mt}}\sum_{i=1}^{I(mt)}\left|(\tau_i^Z-\tau_{i-1}^Z) - (\tau_i^X-\tau_{i-1}^X)\right| \xrightarrow[t\to\infty]{a.s.} 0,\end{align*} and thus, since our choice of $T$ was arbitrary, therefore by Proposition \ref{asympSmallTC}, we have \begin{align} \label{removingAlpha} \lim_{m\to\infty}(m^{-1/2}\tilde{Z}_1(\tau^X(mt)+\alpha(mt)))_{t\geq 0}=\lim_{m\to\infty}(m^{-1/2}\tilde{Z}_1(\tau^X(mt))_{t\geq 0}.\end{align} Finally, we note that $\tau^X(mt)-mt$ is, in the language or renewal theory, an `excess lifetime process' or `residual waiting time' process, so it follows that for any $T\in (0,\infty)$, we have $$\sup_{0\leq t\leq T}\left|m^{-1}(\tau^X(mt)-mt)\right|\xrightarrow[m\to\infty]{\B{P}}0$$ (see Example 4.4.8 in \cite{durrett}). Therefore applying Proposition \ref{asympSmallTC} again, we have \begin{align*} \lim_{m\to\infty}(m^{-1/2}X_1(mt))_{t\geq 0}=&\lim_{m\to\infty}(m^{-1/2}X_1(\tau^X(mt)))_{t\geq 0}\quad \quad && \text{(by Proposition \ref{asympSmallTC})}\\ =&\lim_{m\to\infty}(m^{-1/2}\tilde{Z}_1(\tau^X(mt)+\alpha(mt)))_{t\geq 0}\quad \quad && \text{(by (\ref{xzTildeCoupling}))} \\ =&\lim_{m\to\infty}(m^{-1/2}\tilde{Z}_1(\tau^X(mt)))_{t\geq 0}\quad \quad && \text{(by (\ref{removingAlpha}))}\\ =&\lim_{m\to\infty}(m^{-1/2}\tilde{Z}_1(mt))\quad \quad && \text{(by Proposition \ref{asympSmallTC})}\\ \overset{d}{=}&\left(\beta^{-1/2}\sigma |B(t)|\right)_{t\geq 0}\quad \quad && \text{(by (\ref{ztildeConv}))} \end{align*} \end{proof} We can now give our proof of Theorem \ref{mainThem}.2, that is, that $(X(t))_{t\geq 0}$ satisfies the invariance principle that $$\left(m^{-1/2}X(mt)\right)_{t\geq 0}\xrightarrow[m\to\infty]{d}\left(\beta^{-1/2}\sigma|B(t)|\underline{1}\right)_{t\geq 0},$$ where $B$ is a standard Brownian motion and $\underline{1}=(1,\ldots,1)\in \B{R}^N$. \begin{proof} (of Theorem 1.2) Let $\tau_1^X,\tau_2^X,\tau_3^X,\ldots$ be defined as in the above theorem. Note that since $\tau_i^X > i$, thus $\tau_i^X \to \infty$ as $i\to \infty$. Furthermore, the random variables $X_N(\tau^X_i)-X_1(\tau^X_i), i\geq 0$ are i.i.d with finite mean, thus we have $\tau_i^{-1/2}|X_N(\tau^X_i)-X_1(\tau^X_i)|\xrightarrow[i\to\infty]{\B{P}} 0$. And therefore in the scaling limit, the radius of the particle system goes to zero. Then by Slutsky's theorem, we have \begin{align*}\left( m^{-1/2}X(mt)\right)_{t\geq 0} &=\left( m^{-1/2}X_1(mt)\underline{1}\right)_{t\geq 0}+\left(0, m^{-1/2}(X_2(mt)-X_1(mt)),\ldots,m^{-1/2}(X_N(mt)-X_1(mt))\right)_{t\geq 0}\\ &\xrightarrow[m\to\infty]{d}\left(\beta^{-1/2}\sigma B(t)\underline{1}\right)_{t\geq 0}+(0,\ldots,0)=\left(\beta^{-1/2}\sigma B(t)\underline{1}\right)_{t\geq 0} \end{align*} \end{proof} The coupling of Lemma \ref{generalCouple} also immediately gives proof of the transience of the $N$-Brownian bees system. Recall that for Theorem 1.3, we want to prove the following. \begin{theorem*} Let $X^{N, \mu}(t)$ be a $N$-Brownian bees process with drift $|\mu| > \mu_c^N$ and initial condition $X^{N,\mu}(0)=\nu$. Then $$\lim_{t \rightarrow \infty} \frac{X^{N, \mu}_1(t)}{t} = \lim_{t \rightarrow \infty} \frac{X^{N, \mu}_N(t)}{t} = \left(|\mu| - \mu_c^N\right) \text{sign}(\mu) \text{ a.s.}$$ \end{theorem*} The proof of this result is due to Flynn and can be found as Theorem 3.0.2 of \cite{flynn}. We give the argument in full for completeness. \begin{proof} (Due to Flynn, Theorem 3.0.2 of \cite{flynn}) Suppose without loss of generality that $\mu < -\mu_c^N$. Let $T$ be the random time defined by $$T:=\inf\left\{s\geq 0: t\geq s \implies \frac{X_N^{N,\mu}(t)}{t}\leq \frac{\mu + \mu_c^N}{2}<0\right\}$$ This is almost surely finite since, by the coupling of Proposition \ref{generalCouple} $$\liminf_{t \rightarrow \infty} \frac{X^{N, \mu}_N(t)}{t} \leq \lim_{t \rightarrow \infty } \frac{Z^{N, \mu}_N(t)}{t} = \mu + \mu_c^N < 0 .$$ Fix $\epsilon>0$. Since $T$ is almost surely finite, we can choose a $K \in \mathbb{R}$ such that $\mathbb{P}(T \geq K) \leq \epsilon$. Now we construct a process $Y^{N, \mu, K}$ such that $(Y^{N, \mu, K}(t))_{0\leq t\leq K}$ is an $N$-Brownian bees process with drift $\mu$ started from $Y^{N,\mu,K}(0) = X^{N, \mu}(0)$ and $(Y^{N, \mu, K}(t))_{t\geq K}$ is an $N$-BBM process with drift $\mu$ started from $Y^{N,\mu,K}(K)$. $$\lim_{t \rightarrow \infty} \frac{Y^{N, \mu, K}_1(t)}{t} = \lim_{t \rightarrow \infty} \frac{Y^{N, \mu, K}_N(t)}{t} = \mu_c^N + \mu$$ Moreover, by construction of $K$, with probability at least $1- \epsilon$, the processes $Y^{N, \mu, K}$ and $X^{N, \mu}$ will be identical - since if all the particles lie on the left of $0$ then the $N$ Brownian bees moves identically to the $N$-BBM, hence $$\mathbb{P} \left( \lim_{t \rightarrow \infty} \frac{X^{N, \mu}_1(t)}{t} = \lim_{t \rightarrow \infty} \frac{Y^{N, \mu, K}_1(t)}{t}\right) \geq 1 - \epsilon$$ and similarly for $X^{N, \mu}_N(t)$, $Y^{N, \mu, K}_N(t)$. So since $\epsilon$ is arbitrary we deduce that: $$\lim_{t \rightarrow \infty} \frac{X^{N, \mu}_1(t)}{t} = \lim_{t \rightarrow \infty} \frac{X^{N, \mu}_N(t)}{t} = \mu + \mu_c^N \quad a.s.$$ \end{proof} As we noted in the introduction the behaviour of the $N$-Brownian bees system without drift is known when the number of particles tends to $\infty$. In particular, it is known that as $N\to\infty$, the empirical distribution of particles converges to the solution of a free boundary problem given by: \begin{align} \label{beesFBP1} \begin{cases} u_t=\frac{1}{2}u_{xx} + u & x\in(-R_t,R_t), t>0, \\ \int_{-R_t}^{R_t} u(s,t)ds=1 & t>0, \\ u(t,x)=0 & x\notin (R_t,R_t), t>0, \end{cases} \end{align} with an initial condition based on the initial distribution of particles. This result is proven by Berestycki et al. in \cite{afree}, \cite{beesBBNP}. We conjecture that for Brownian bees with sub-critical drift $\mu$ such that $|\mu|<\mu_c^N$, the empirical distribution converges to the solution of the FBP: \begin{align} \begin{cases} u_t=\frac{1}{2}u_{xx} - \mu u_x + u & x\in(-R_t,R_t), t>0, \\ \int_{-R_t}^{R_t} u(s,t)ds=1 & t>0, \\ u(t,x)=0 & x\notin (R_t,R_t), t>0. \end{cases} \end{align} The methods of proof used by Berestycki et al. rely on the symmetry of the system, and view it essentially as a system of reflected Brownian motions on $[0,\infty)$. However these methods no longer work when drift is introduced, since $|B_t + \mu t|$ does not behave like a reflected Brownian motion and in fact is not even Markovian. Therefore this conjecture remains open. Recall Proposition \ref{pathSpaceTfm}: \begin{propn-non} Let $(W(t))_{t\geq 0}$ be a stochastic process whose sample paths are in $\C{D}([0,\infty),\B{R})$, the space of cadlag paths $[0,\infty)\to \B{R}$. Let $g:\C{D}([0,\infty),\B{R})\to\C{D}([0,\infty),\B{R})$ be the map given by: $$ g:(W(t))_{t\geq 0} \mapsto \left(W(\inf\{s\geq 0: \int_0^s \is_{W(u) \geq 0} du \geq t\})\right)_{t\geq 0}. $$ Let $D_g\in \C{D}([0,\infty),\B{R})$ be the discontinuity set of $g$ under the metric $d_\infty$. Then $\B{P}((B_t)_{t\geq 0} \in D_g)=0$, where $B$ is a standard Brownian motion. \end{propn-non} \begin{proof} Let $Y$ be a path in $\C{D}([0,\infty),\B{R})$ such that $\{t:Y(t)=0\}$ is closed and Lebesgue null, and $Y$ is $\alpha$-H\"{o}lder continuous for $\alpha \in (0,1/2)$. Note that Brownian motion satisfies these properties almost surely. Fix $T\in (0,\infty)$ and $\epsilon>0$. We will first show that given $\delta>0$ sufficiently small, $d_\infty(X,Y)<\delta$ implies that $d_T(g(X),g(Y))<\epsilon$. In a slight abuse of notation, we use $d_T(g(X),g(Y))$ to mean the metric $d_T$ on $\C{D}([0,T],\B{R})$ of the paths $g(X)$ and $g(Y)$ \textit{restricted to the interval} $[0,T]$. Let $\tilde{T}$ be the smallest integer time such that $\int_0^{\tilde{T}}\is_{Y(u)\geq 0}du \geq T+1$. Since $d_\infty(X,Y)<\delta$, we have $d_{\tilde{T}}(X,Y)<2^T\delta=:\tilde{\delta}$. So there exists a continuous and strictly increasing bijection $\lambda:[0,\tilde{T}]\to [0,\tilde{T}]$ such that $\sup_{t\in [0,\tilde{T}]}|t-\lambda(t)|<\tilde{\delta}$ and $\sup_{t\in [0,\tilde{T}]}|X(t)-Y(\lambda(t))|<\tilde{\delta}$. We will now show that by choosing $\tilde{\delta}$ sufficiently small, we can make $|\int_0^s \is_{X(u)\geq 0}du-\int_0^s \is_{Y(\lambda(u))\geq 0}du|$ arbitrarily small for all $s\in [0,\tilde{T}]$. Now note that the zero set of $Y$ in $[0,\tilde{T}]$, $S_0:=\{t\in [0,\tilde{T}]:Y(t)=0\}\subseteq [0,\tilde{T}]$ is closed and bounded (and therefore compact) and Lebesgue null. Therefore the zero set of $Y\circ \lambda$, $S^\lambda_0:=\{t\in[0,\tilde{T}]:Y(\lambda(t))=0\}$, which is the image of $S_0$ under the continuous bijection $\lambda$, is also closed. Now let $A:=[0,\tilde{T}]\setminus S^\lambda_0$ be the complement of $S^\lambda_0$, which is thus open, and hence can be written as a countable union of disjoint intervals, $A=\bigcup_{i=1}^\infty A_i$. Now since $\lambda^{-1}$ is a bijection, $\lambda^{-1}(A_i)$ are intervals contained in $[0,\tilde{T}]\setminus S_0$, the complement of $S_0$. And since $\sup_t|t-\lambda(t)| < \tilde{\delta}$, we have that $Leb(\lambda^{-1}(A_i))\in (Leb(A_i)-\tilde{\delta},Leb(A_i)+\tilde{\delta})$. Therefore choosing $k$ sufficiently large, $Leb\left(\cup_{i=1}^k \lambda^{-1}(A_i)\right)>\tilde{T}-\epsilon/4$. Then choosing $\tilde{\delta} < \epsilon/8k$, we have that $Leb\left(\cup_{i=1}^k A_i\right)>Leb\left(\cup_{i=1}^k \lambda^{-1}(A_i)\right)-2k\tilde{\delta} > \tilde{T}-\epsilon/2$. Then given an open interval $A_i=(a_i,b_i)$, define the closed interval $I_i=[a_i+\frac{\epsilon}{4k},b_i-\frac{\epsilon}{4k}]$, so that $\bigcup_{i=1}^k I_i$ is a finite set of closed intervals contained in $A$ such that $Leb\left( \bigcup_{i=1}^k I_i\right)>Leb(\bigcup_{i=1}^k A_i)-2k\frac{\epsilon}{4k} > \tilde{T}-\epsilon$. By the compactness of $\bigcup_{i=1}^k I_i$, the infimum $\inf\{|Y(\lambda(t))|:t\in \cup_{i=1}^k I_i\}$ is attained in $\bigcup_{i=1}^k I_i$ and thus is $>0$. Therefore choosing $\tilde{\delta} < \inf\{|Y(\lambda(t))|:t\in \cup_{i=1}^k I_i\}$, the condition $\sup_{t\leq \tilde{T}}|X(t)-Y(\lambda(t))|<\tilde{\delta}$ ensures that on $\bigcup I_i$ we have either $X(t),Y(\lambda(t))> 0$ or $X(t),Y(\lambda(t))<0$. Therefore for any $s\in [0,\tilde{T}]$ \begin{align}\label{firstLambdaBound} \left|\int_0^s \is_{X(u)\geq 0}du - \int_0^s \is_{Y(\lambda(u))\geq 0}du\right| \leq \int_0^s |\is_{X(t)\geq 0}-\is_{Y(\lambda(t))\geq 0}|du \leq \int_0^{\tilde{T}} \is_{\{u\notin \bigcup I_i\}}du = \tilde{T}-Leb \left(\bigcup I_i\right) < \epsilon.\end{align} Similarly, since $Y$ is $\alpha$-H\"{o}lder continuous, and thus uniformly continuous on $[0,\tilde{T}]$, thus choosing $\tilde{\delta}$ sufficiently small, we can make $|Y(u)-Y(\lambda(u))|$ arbitrarily small, and therefore, as in equation (\ref{firstLambdaBound}), we can make $\int_0^s|\is_{Y(u)\geq 0}-\is_{Y(\lambda(u))\geq 0}|du$ arbitrarily small. It then follows by the triangle inequality that for any $s\in [0,\tilde{T}]$ \begin{align} \label{secondLambdaBound} \left| \int_0^s \is_{X(u)\geq 0}du - \int_0^s \is_{Y(u)\geq 0}du \right| < \epsilon. \end{align} Now we define a function $\tilde{\lambda}:[0,T]\to \B{R}$ as follows: $$s(t):=\inf\left\{u\geq 0:\int_0^u\is_{X(v)\geq 0}dv\geq t\right\}\quad \text{and}\quad \tilde{\lambda}(t):=\int_0^{\lambda(s(t))}\is_{Y(u)\geq 0}du.$$ Since $\int_0^{\tilde{T}}\is_{X(v)\geq 0}du > T$, we have that $s:[0,T]\to [0,\tilde{T}]$, and so the function $\tilde{\lambda}$ is well defined on $[0,T]$. We can also observe that $\tilde{\lambda}$ is by definition non-decreasing. Now by the triangle inequality \begin{align} |t-\tilde{\lambda}(t)| &= \left| t - \int_0^{\lambda(s(t))}\is_{Y\geq 0}\right| = \left| \int_0^{s(t)} \is_{X\geq 0} - \int_0^{s(t)}\is_{Y\geq 0} + \int_0^{s(t)}\is_{Y\geq 0} - \int_0^{\lambda(s(t))}\is_{Y\geq 0} \right| \notag \\ & \label{thirdLineSko}\leq \left|\int_0^{s(t)}\is_{X\geq 0}-\int_0^{s(t)}\is_{Y\geq 0}\right| + \left|\int_0^{s(t)}\is_{Y\geq 0}-\int_0^{\lambda(s(t))}\is_{Y\geq 0}\right| \end{align} where we use the fact that by definition $\int_0^{s(t)} \is_{X \geq 0} = t$. Then first term of (\ref{thirdLineSko}) can be made arbitrarily small by (\ref{secondLambdaBound}) and the second term is bounded by $\tilde{\delta}$, therefore we can choose $\tilde{\delta}$ sufficiently small so that $\sup_{t\in [0,T]}|t-\tilde{\lambda}(t)|<\epsilon$. Next we prove that for all $t\in [0,T]$, we can make $|g(X)(t)-g(Y)(\tilde{\lambda}(t))|$ arbitrarily small by choosing $\delta$ sufficiently small. So fix $t\in [0,T]$, and define $$v(t):=\inf\left\{s\geq 0: \int_0^s \is_{Y\geq 0} \geq \int_0^{\lambda(s(t))} \is_{Y\geq 0}\right\}.$$ Observe that by definition of $g,\lambda, \tilde{\lambda}$ we have $g(X)(t)=X(s(t))$ and $g(Y)(\tilde{\lambda}(t))=Y(v(t))$. We can note also that by definition, $X(s(t))\geq 0$, and if $Y(\lambda(s(t)))\geq 0$, then $v(t)=\lambda(s(t))$. If $Y(\lambda(s(t)))< 0$, then by (\ref{firstLambdaBound}), $|v(t)-\lambda(s(t))|<\epsilon$. Therefore as $Y$ is, by assumption, $1/4$-H\"{o}lder continuous, and by definition of $\lambda$, $|X(s(t))-Y(\lambda(s(t))|<\epsilon$, we have that: \begin{align} \label{gSmall}|g(X)(t) - g(Y)(\tilde{\lambda}(t))| \leq |X(s(t))-Y(\lambda(s(t))| + |Y(\lambda(s(t))-Y(v(t))|\leq \epsilon + C|\lambda(s(t))-v(t)|^{1/4} < \epsilon + C\epsilon^{1/4},\end{align} which can be made arbitrarily small. Hence we can see that $\tilde{\lambda}$ is such that $\sup_{t\in [0,T]}|t-\tilde{\lambda}(t)|$ and $\sup_{t\in [0,T]}|X(t)-Y(\tilde{\lambda}(t))|$ can be made arbitrarily small. However for Skorokhod continuity, we also require $\tilde{\lambda}$ to be increasing and bijective $[0,T]\to [0,T]$, so it remains to show that we can approximate $\tilde{\lambda}$ by an increasing bijection $[0,T]\to [0,T]$. Since $\tilde{\lambda}:[0,T]\to \B{R}$ is increasing, there are at most countably many discontinuities. Call the points of discontinuity $t_1,t_2,\ldots$ and suppose they have jumps of size $J_i:=\tilde{\lambda}(t_i)-\tilde{\lambda}(t_i-)$. Let $N$ be such that $\sum_{i=1}^{N-1}J_i>1-\epsilon$. Then define $\hat{\lambda}_{N,n}$, $n\geq 0$, to be the function $\tilde{\lambda}$ with the discontinuities at $t_{N+1},t_{N+2},\ldots t_{N+n}$ removed: $$ \hat{\lambda}_{N,n}(t):= \tilde{\lambda}(t) - \sum_{i=1}^n J_{N+i}\is_{t\geq t_{N+i}} \leq \hat{\lambda}_{N,n-1}.$$ Then since $(\hat{\lambda}_{N,n})_{n\in \B{N}}$ is a decreasing sequence of non-decreasing functions which is bounded below by $0$, thus $\hat{\lambda}_N:=\lim_{n\to\infty} \hat{\lambda}_{N,n}$ exists and is non-decreasing with $N$ jump discontinuities at $t_1,t_2,\ldots t_N$. Furthermore, by construction we have that $\sup_{t\in [0,T]} |\tilde{\lambda}(t)-\hat{\lambda}_N(t)|\leq \epsilon$. Now consider $\hat{\delta}>0$ taken sufficiently small so that the intervals $[t_i-\hat{\delta},t_i + \hat{\delta}]$, $i=1,\ldots, N$ are pairwise disjoint. Then define $\hat{\lambda}(t)$ to be the function which takes value $\hat{\lambda}_N(t)$ on $[0,T]\setminus\bigcup_{i=1}^N [t_i-\hat{\delta},t_i+\hat{\delta}]$ and which linearly interpolates across each interval. Therefore $\hat{\lambda}$ is continuous, and choosing $\hat{\delta}$ sufficiently small, we can make $\sup_{t\in [0,T]}|\tilde{\lambda}(t)-\hat{\lambda}(t)|$ arbitrarily small. Finally, define $\lambda_{\text{cand}}:=(\hat{\lambda}(t)+\epsilon t)/(\hat{\lambda}(T)+T\epsilon)$. So $\lambda_{\text{cand}}$ is continuous and strictly increasing bijection $\lambda_{\text{cand}}:[0,T]\to [0,T]$, and we can choose $\delta$ sufficiently small so that $\sup_{0\leq t\leq T}|\lambda_{\text{cand}}(t)-\tilde{\lambda}(t)|<\epsilon$. Therefore by the triangle inequality, we can make $\sup_{t\in [0,T]}|\lambda_{\text{cand}}(t)-t|$ arbitrarily small. Moreover, since $g(Y)(t)$ is $1/4$-H\"{o}lder continuous as a function of $t$, thus by the triangle inequality and (\ref{gSmall}): $$\sup_{0\leq t\leq T}|g(X)(t)-g(Y)(\lambda_{\text{cand}}(t))|\leq \sup_{0\leq t\leq T}|g(X)(t)-g(Y)(\tilde{\lambda}(t))| + \sup_{0\leq t\leq T}|g(Y)(\tilde{\lambda}(t))-g(Y)(\lambda_{\text{cand}}(t))|,$$ can be made arbitrarily small. Therefore for $\delta$ sufficiently small, we have that $d_\infty(X,Y)<\delta$ implies $d_T(g(X),g(Y))<\epsilon$. Specifically, say that for each $T\in \B{N}$, $d_\infty(X,Y)<\delta_T \implies d_T(g(X),g(Y))<\epsilon$. Thus for $d_\infty(X,Y)<\min_{T\leq N}\delta_T$, we have: $$ d_\infty(g(X),g(Y))=\sum_{T=1}^\infty 2^{-T}(1\wedge d_T(g(X),g(Y)) < \sum_{T=1}^N 2^{-T}\epsilon + \sum_{T=N+1}^\infty 1 < \epsilon + 2^{-N}. $$ Since $\epsilon,N$ are arbitrary, this can be made arbitrarily small. Hence we can finally conclude that, for all $\epsilon > 0$, there exists $\delta>0$ such that $d_\infty(X,Y)<\delta$ implies that $d_\infty(g(X),g(Y))<\epsilon$. Therefore $g$ is continuous at $Y$. Since Brownian motion almost surely has the properties we asked of $Y$, Brownian motion $B$ is almost surely not in the discontinuity set of the transformation $g$; that is, $\B{P}(B\in D_g)=0$, as required. \end{proof} Recall Proposition \ref{asympSmallTC}: \begin{propn-non} Let $(W(t))_{t\geq 0}$ be a stochastic process such that $(m^{-1/2}W(mt))_{t\geq 0}$ converges in the distribution as $m\to\infty$ to $(B(t))_{t\geq 0}$ (resp. $(|B(t)|)_{t\geq 0}$) in the Skorohod topology on $\C{D}([0,\infty),\B{R})$, where $B$ is a standard Brownian motion. Suppose that for every $T>0$ we have $$\sup_{0\leq t\leq T}|m^{-1}\alpha(mt)|\xrightarrow[m\to\infty]{\B{P}} 0.$$ Then $\left(m^{-1/2}W(mt+\alpha(mt))\right)_{t\geq 0}\xrightarrow[m\to\infty]{d}(B(t))_{t\geq 0}$ (resp. $(|B(t)|)_{t\geq 0}$) in the Skorohod topology on $\C{D}([0,\infty),\B{R})$ \end{propn-non} \begin{proof} We begin by showing that the result holds for convergence in $\C{D}([0,T],\B{R})$. That is, fix $T>0$ and assume that $(m^{-1/2}X(mt))_{0\leq t\leq T} \xrightarrow[m\to\infty]{d}(B(t))_{0\leq t\leq T}$ in the Skorohod topology on $\C{D}([0,T],\B{R})$. In order to prove that $\left(m^{-1/2}X(mt+\alpha(mt))\right)_{0\leq t\leq T}\xrightarrow[m\to\infty]{d}(B(t))_{0\leq t\leq T}$, it suffices to prove that $$(U_m(t))_{0\leq t\leq T}:=\left( m^{-1/2}X(mt+\alpha(mt))-m^{-1/2}X(mt)\right)_{0\leq t\leq T} \xrightarrow[m\to\infty]{\B{P}} 0$$ in the Skorohod topology on $\C{D}([0,T],\B{R})$, since by Slutsky's theorem, if $A_n\xrightarrow{d} A$ and $B_n\xrightarrow{\B{P}}0$, then $A_n + B_n\xrightarrow{d}A$. Therefore we want to prove that $\forall \; \varepsilon > 0$, there exists $M$ such that $m>M$ implies \begin{equation}\label{aimConv}\B{P}\left(\sup_{0\leq t\leq T}|U_m(t)|>\varepsilon\right)< \varepsilon.\end{equation} Now define the modulus of continuity of a process $Y(t)$ as $w(Y,\delta):=\underset{|s-t|\leq \delta}{\sup}|Y(s)-Y(t)|$. Note that $w(Y,\delta)=\underset{|s-t|\leq \delta}{\sup}|Y(s)-Z(s)+Z(s)-Z(t)+Z(t)-Y(t)|$ so using the triangle inequality, we have that $w(Y,\delta)\leq 2\sup_{0\leq t\leq 1}|Y(t)-Z(t)| + w(Z,\delta)$. Therefore by symmetry, it follows that $|w(Y,\delta)-w(Z,\delta)|\leq 2\sup_{0\leq t\leq 1}|Y(t)-Z(t)|$. Hence $w(Y,\delta)$ is continuous in $Y$ with respect to the Skorohod metric. Therefore: $$\lim_{m\to\infty}w\left((m^{-1/2}X(mt))_{0\leq t\leq T},\delta\right)=w\left(\lim_{m\to\infty}(m^{-1/2}X(mt))_{0\leq t\leq T},\delta\right)\overset{d}{=}w((B(t))_{0\leq t\leq T},\delta).$$ The Portmanteau theorem (see Lemma 2.2 of \cite{asympStats}) and Levy's modulus of continuity theorem then imply that $$\underset{m\to\infty}\limsup \,\B{P}\left(w\left((m^{-1/2}X(mt))_{0\leq t\leq T},\delta\right)\geq \varepsilon\right)\leq \B{P}\left(w\left((B(t))_{0\leq t\leq T},\delta\right)\geq \varepsilon\right)\xrightarrow[\delta\to 0]{}0.$$ Now consider the event $E(m,\delta)$ on which $\sup_{0\leq t\leq T}|m^{-1}\alpha(mt)|>\delta$. By definition of modulus of continuity, we have that $\sup_{0\leq t\leq 1}|U_m(t)|\leq w\left((m^{-1/2}X(mt))_{0\leq t\leq T},\delta\right)$ on $E(m,\delta)^c$, and therefore: $$ \limsup_{m\to\infty} \, \B{P}\left( \sup_{0\leq t\leq T} |U_m(t)| \mathbbm{1}_{E(m,\delta)^c}>\varepsilon\right)\xrightarrow[\delta\to 0]{}0 $$ Alternatively, since $\sup_{0\leq t\leq T}|m^{-1}\alpha(mt)|\xrightarrow[m\to\infty]{\B{P}}0$, we have that $\B{P}(E(m,\delta))\to 0$ as $m\to \infty$, and hence $\B{P}\left(\sup_{0\leq t\leq T} |U_m(t)|\mathbbm{1}_{E(m,\delta)}>\varepsilon\right)\leq \B{P}\left(E(m,\delta) \right) \xrightarrow[m\to\infty]{}0$. Therefore $$ \lim_{m\to\infty}\B{P}\left( \sup_{0\leq t\leq T} |U_m(t)|\mathbbm{1}_{E(m\delta)}>\varepsilon\right) = 0 $$ So combining the convergence on $E(m,\delta)$ and $E(m,\delta)^c$, equation (\ref{aimConv}) is proven as required, and hence by Slutsky's theorem, we have $$\left(m^{-1/2}X(mt+\alpha(mt))\right)_{0\leq t\leq T}\xrightarrow[m\to\infty]{d}(B(t))_{0\leq t\leq T}.$$ Finally, since the convergence holds for all $T$, thus by Lemma 3 of section 16 of \cite{bills}, it holds that $$\left(m^{-1/2}X(mt+\alpha(mt))\right)_{t\geq 0}\xrightarrow[m\to\infty]{d}(B(t))_{t\geq 0}$$ in the Skorokhod topology on $\C{D}([0,\infty),\B{R})$. \end{proof} The aim of this section is to prove the following theorem about the velocity of the $N$-BBM system. \begin{theorem} \label{asympVeloc} $$v_N = \sqrt{2} - \frac{\pi^2}{\sqrt{2}(\log N)^2} + o\left( \frac{1}{(\log N)^2}\right).$$ \end{theorem} Our proof will use Theorem 7.6 of \cite{barakErdos} of Mallein and Ramassamy, which we state here for completeness. The theorem gives the asymptotic speed of discrete-time branching random walks with selection ($N$-BRW). In the $N$-BRW, we have $N$ particles and at each discrete time, every particle branches ito a random number of offspring, located at random distances from their parents, and we simultaneously delete all but the $N$ rightmost particles. The offspring is described by a distribution $\C{M}$, which is a random point process. \begin{theorem} \textit{(Theorem 7.6 in \cite{barakErdos})} Let $Z^N$ be an $N$-BRW and let $M\sim \C{M}$ satisfy: \begin{itemize} \item $\kappa(\theta):=\log\B{E}\left(\sum_{m\in M}e^{\theta m}\right) < \infty \quad \forall \theta >0 $ \item $\exists \theta^* > 0$ such that $\theta^* \kappa'(\theta^*)=\kappa(\theta^*)$ \item $\B{E}\left(|\max_{m\in M}m|^2\right)<\infty$ \item $\B{E}\left(\sum_{m\in M}e^{\theta^* m}\left(\log \sum_{m\in M}e^{\theta^* m}\right)^2\right)<\infty$ \end{itemize} Then $v_N:=\lim_{t\to \infty} t^{-1}Z^N_1(t)=\lim_{t\to\infty}t^{-1}Z^N_N(t)$ exists and $v_N=\kappa'(\theta^*) + \frac{\pi^2 \theta^* \kappa''(\theta^*)}{2(\log N)^2} + o\left((\log N)^{-2}\right)$ \end{theorem} We can thus couple $N$-BBM to appropriately chosen $N$-BRWs to determine the speed of $N$-BBM. \begin{lemma} \label{speedUB} $v_N \leq \sqrt{2} - \frac{\pi^2}{\sqrt{2}(\log N)^2} + o\left( \frac{1}{(\log N)^2}\right)$ \end{lemma} \begin{proof} Define $Z^{N,1}(t)$ to be a branching Brownian motion process in which at each integer time we delete all but the $N$ rightmost particles. Therefore $Z^{N,1}$ may have more than $N$ particles at some time $t\in \B{R}\setminus \B{Z}$, but has exactly $N$ particles at each time in $\B{Z}$. By Proposition 3 of \cite{hydroNBBM}, there is a natural coupling between $Z^N$ and $Z^{N,1}$ such that $Z^N(0)\preccurlyeq Z^{N,1}(0) \implies Z^N(t)\preccurlyeq Z^{N,1}(t)$ for all subsequent times $t>0$. Therefore, defining $v_{N,1}:=\lim_k k^{-1}Z_1^{N,1}(k)$, then certainly $v_{N}\leq v_{N,1}$. By construction, taking $Z^{N,1}$ at integer times, $(Z^{N,1}(n))_{n\in \B{N}}$ is an $N$-BRW with reproduction law $\C{M}_1$, say. The law $\C{M}_1$ describes the distribution of particles at time $1$ of a branching Brownian motion process started from a single particle at the origin. Let $M_1$ be a random point process of law $\C{M}_1$. It is easy to check that $\C{M}_1$ satisfies the condition of the above Theorem, and we can calculate $\kappa(\theta) = 1+\theta^2/2$. Therefore the conclusion of the Theorem gives $\theta^*=\sqrt{2}$ and $$ v_N \leq v_{N,1} = \sqrt{2} - \frac{\pi^2}{\sqrt{2}(\log N)^2} + o\left( \frac{1}{(\log N)^2}\right), $$ as required. \end{proof} For the lower bound, we will define another branching random walk $(Y^{N,\delta}(t))_{t\geq 0}$ with selection at discrete times $(k\delta)_{k\in\B{N}}$. Consider a branching Brownian motion process $\hat{Y}^1$ starting with a single particle at the origin in which we permit at most one branching event. Therefore there at most 2 particles in $\hat{Y}^1$ at any time. Let the distribution of the particles of $\hat{Y}^1(\delta)$ be called $\C{\hat{M}}_\delta$. Now define $(Y^{N,\delta}(n\delta))_{n\in\B{N}}$ to be the $N$-BRW starting with $\lfloor \frac{N}{2} \rfloor$ particles with offspring distribution $\C{\hat{M}}_\delta$ and with selection of the right-most $\lfloor \frac{N}{2} \rfloor$ particles. Then since there are always fewer particles in the process $Y^{N,\delta}$ than in $Z^{N}$, Lemma 7.4 of \cite{barakErdos} proves that there exists a coupling such that $Y^{N,\delta}(0)\oop Z^{N}(0) \implies Y^{N,\delta}(t)\oop Z^N(t)$ for all $t\geq 0$. Denoting the speed of the process $Y^{N,\delta}$ by $\hat{v}_{N,\delta}$, it immediately follows that $\hat{v}_{N,\delta}\leq v_N$ Thus it only remains give the velocity $\hat{v}_{N,\delta}$. \begin{lemma} $v_N \geq \sqrt{2} - \frac{\pi^2}{\sqrt{2}(\log N)^2} + o\left( \frac{1}{(\log N)^2}\right)$ \end{lemma} \begin{proof} Let $\hat{M}_\delta$ be a random point process with law $\C{\hat{M}}_\delta$. Conditioning on the branching time, we calculate: \begin{align*} \hat{\kappa}_\delta(\theta)&:=\log\B{E}\left(\sum_{m\in \hat{M}_\delta}e^{\theta m} \right) = \log\left( \int_0^\infty \B{E}\left[\sum_{m\in \hat{M}_\delta}e^{\theta m}\middle| \tau=t\right]e^{-t}dt\right) \\ &=\log \left( \int_0^\delta 2e^{-t}\B{E}[e^{\theta B_\delta}]dt + \int_\delta^\infty e^{-t}\B{E}[e^{\theta B_\delta}]dt\right) \\ &=\frac{\theta^2 \delta}{2} + \log (2-e^{-\delta}) \end{align*} where $(B_t)_{t\geq 0}$ is a standard Brownian motion, and therefore $\theta^* = \sqrt{2\delta^{-1}\log(2-e^{-\delta})}$. So the above Theorem gives $$ v_N \geq \hat{v}_{N,\delta} = \sqrt{2\delta^{-1}\log(2-e^{-\delta})} - \frac{\pi^2 \sqrt{2\delta^{-1}\log(2-e^{-\delta})}}{\sqrt{2}(\log N)^2} + o\left(\frac{1}{(\log N)^2}\right) $$ and by L'H\^{o}pital's rule, $\sqrt{2\delta^{-1}\log(2-e^{-\delta})} \to \sqrt{2}$ as $\delta \downarrow 0$, so $v_N \geq \sqrt{2}-\frac{\pi^2}{2(\log N)^2} + o((\log N)^{-2})$ as required. \end{proof} \begin{proof} \textit{(of Theorem \ref{asympVeloc})} Putting together the preceding two lemmas gives the desired result immediately. \end{proof} Results about the $N$-BBM are often easier to prove than results about the $N$-Brownian bees system due to the fact that the $N$-BBM is translation invariant. A number of our proofs will use properties of the $N$-BBM and then construct a coupling between $N$-BBM and $N$-Brownian bees in order to yield the desired results. Therefore in the first section, we introduce a number of couplings which we will need in the proof of Theorem \ref{mainThem}. We begin by defining the $N$-BBM and $N$-Brownian bees processes exactly. In different proofs, it will make sense to either describe the process by $N$ particles with intrinsic labels which do not change, or to label the particles by order, so we give both definitions here. For an $\B{R}^N$-process, define $\Theta^N:\B{R}^N \to \B{R}^N$ to be the function which ranks the components of the vector, breaking ties arbitrarily. So the $N$-particle processes $(A(t))_{t\geq 0}$ and $(\Theta^N(A(t))_{t\geq 0}$ have the same empirical measure but different labellings of components. \vspace*{1em} \begin{defn} \label{nbbmDef} (\textbf{$N$-BBM}) Let $(W^N(t))_{t\geq 0}=\left((W^N_1(t))_{t\geq 0},\ldots,(W^N_N(t))_{t\geq 0}\right)$ be the trajectories of $N$ particles on the real line. The particles behave as $N$ independent Brownian motions, and at rate $N$ the leftmost particle in the system jumps to the position of a uniformly randomly chosen particle. At any time $t\geq 0$, let $Z^N(t):=\Theta^N(W^N(t))$; so $Z^N$ is the process with ordered components. In the rest of this paper, we will call an $N$-BBM any cadlag stochastic process which has the same empirical distribution as $W^N$ or $Z^N$ \end{defn} \vspace*{1em} \begin{defn} \label{beesDef} (\textbf{$N$-Brownian Bees}) Let $(V^N(t))_{t\geq 0} = \left((V^N_1(t))_{t\geq 0},\ldots,(V^N_N(t))_{t\geq 0}\right)$ be the trajectories of $N$ particles on the real line. The particles behave as $N$ independent Brownian motions, and at rate $N$ the particle furthest from the origin jumps to the position of a uniformly randomly chosen particle. At any time $t\geq 0$, let $X^N(t):=\Theta^N(V^N(t))$; so $X^N$ is the process with ordered components. In the rest of this paper, we will call an $N$-Brownian bees any cadlag stochastic process which has the same empirical distribution as $V^N$ or $X^N$. \end{defn} \paragraph{Remark:} We could equivalently think of the $N$-BBM (resp. $N$-Brownian bees) as being a process in which each particle branches at unit rate and at each branching event the leftmost particle (resp. furthest particle from the origin) is deleted, giving its label to the newly born particle. We also define a notion of `ordering' for particle systems. \vspace*{1em} \begin{defn} Let $A=(a_1,\ldots,a_m)\in \B{R}^m$ and $B=(b_1,\ldots, b_n)\in \B{R}^n$. Then $A\oop B$ (we say that ``$A$ lies to the left of $B$'', or ``$A$ is less than $B$ in the sense of $\oop$'') if and only if $$|\{i:a_i \geq c\}| \leq |\{i: b_i \geq c\}| \; \forall c\in \B{R}.$$ When $m=n$, this means that the empirical cumulative distribution functions are pointwise ordered. \end{defn} Now our first result will prove that both the $N$-BBM and the $N$-Brownian bees can be constructed from i.i.d. Brownian motions, a Poisson point process, and i.i.d. uniform random variables on $\{1,2,\ldots,N\}$. This representation of $N$-BBM and $N$-Brownian bees will then make proving couplings with certain properties very straightforward. The first natural coupling this result gives is that the $N$-BBM process is monotonic as a function of its initial configuration, and the second is that an $N$-Brownian bees process can be coupled to always stay to the left of an $N$-BBM process. \vspace*{1em} \begin{propn} \label{generalCouple} Let $B=((B^{j}(t))_{t\geq 0})_{1\leq j\leq N}$ be a family of $N$ i.i.d Brownian motions, $\nu$ be an initial configuration of $N$ particles, $Q=(Q(t))_{t\geq 0}$ be a Poisson point process of intensity $N$, and $I=(I_i)_{i\in \B{N}}$ be a sequence of i.i.d random variables which are uniform on $\{1,2,\ldots,N\}$. Then there exist functions $\Upsilon$ and $\Xi$ with values in $\mathbb R^N$ such that: $$(X^N(t))_{t\geq 0}=\Theta^N\left( \Upsilon\left(\nu,B, Q, I,t\right)_{t\geq 0}\right)$$ is an $N$-Brownian bees process and $$(Z^N(t))_{t\geq 0}=\Theta^N\left( \Xi\left(\nu,B, Q, I,t\right)_{t\geq 0}\right)$$ is an $N$-BBM process. Moreover, given configurations $\nu$ and $\nu'$ such that $\nu \oop \nu'$, we have $$\Upsilon(\nu, B, Q, I,t) \oop \Xi(\nu, B, Q, I,t)\oop \Xi(\nu', B, Q, I,t) \; \forall t\geq 0$$ \end{propn} \begin{proof} We will construct the processes inductively. Let $\tau_0=0$, and recursively, define $$\tau_{i+1}:=\inf\{t> \tau_i:Q(t)\neq Q(t-)\}$$ to be the discontinuities of the Poisson point process $Q$. These times will be the branching times of the processes. For any initial configuration $\eta$, define $$\Upsilon(\eta, B,Q,I,0) = \Xi(\eta, B,Q,I,0)=\eta .$$ Now suppose for induction that $\Upsilon$ and $\Xi$ are defined up to a time $\tau_i$ so that $\Upsilon\left(\nu,B, Q, I,t\right)_{0\leq t\leq \tau_i}$ is an $N$-Brownian bees process and $\Xi\left(\nu,B, Q, I,t\right)_{0\leq t\leq \tau_i}$ and $\Xi\left(\nu',B,Q,I,t\right)_{0\leq t \leq \tau_i}$ are $N$-BBM processes with $$\Upsilon\left(\nu,B, Q, I,t\right)\oop \Xi\left(\nu,B, Q, I,t\right)\oop \Xi\left(\nu',B,Q,I,t\right) \text{ for } t\in [0,\tau_i].$$ We will then define $\Upsilon$ and $\Xi$ on $[\tau_i,\tau_{i+1}]$. Let $\Upsilon_j(\nu, B,Q,I,\tau_i)$ (resp. $\Xi_j(\nu, B,Q,I,\tau_i)$) denote the $j$\textsuperscript{th} smallest particle of $\Upsilon(\nu, B,Q,I,\tau_i)$ (resp. $\Xi(\nu,B,Q,I,\tau_i)$). Then for $t\in [\tau_i,\tau_{i+1})$, we will drive the particles $\Upsilon_j(\nu,B,Q,I,t)$, $\Xi_j(\nu,B,Q,I,t)$, and $\Xi_j(\nu',B,Q,I,t)$ by the same Brownian motion $(B^{j}(t))_{\tau_i \leq t\leq \tau_{i+1}}$. Since $\Upsilon(\nu,B,Q,I,\tau_i)\oop \Xi(\nu,B,Q,I,\tau_i)\oop \Xi(\nu',B,Q,I,\tau_i)$ implies that $\Upsilon_j(\nu,B,Q,I,\tau_i)\leq \Xi_j(\nu,B,Q,I,\tau_i)\leq \Xi_j(\nu',B,Q,I,\tau_i)$ for all $j\in \{1,2,\ldots,N\}$, then certainly: $$\Upsilon_j(\nu,B,Q,I,\tau_i)+B^{j}(t)-B^{j}(\tau_i)\leq \Xi_j(\nu,B,Q,I,\tau_i)+B^{j}(t)-B^{j}(\tau_i)\leq \Xi_j(\nu',B,Q,I,\tau_i)+B^{j}(t)-B^{j}(\tau_i),$$ for all $j\in \{1,2,\ldots,N\}$, and so we have $\Upsilon(\nu,B,Q,I,t)\oop \Xi(\nu,B,Q,I,t)\oop \Xi(\nu',B,Q,I,t)$ for $t\in [\tau_i,\tau_{i+1})$. Now let us define two operators $k,l:\B{R}^N \times \{1,2,\ldots,N\} \to \B{R}^N$. For a vector $v=(v_1,v_2,\ldots,v_N)$ with ordered components $v_1\leq v_2 \leq \cdots \leq v_N$ we define: $$l(v,i)=(v_2,v_3,\ldots,v_{i-1},v_i,v_i,v_{i+1},\ldots,v_N),$$ and $$k(v,i)=\begin{cases} (v_2,v_3,\ldots,v_{i-1},v_i,v_i,v_{i+1},\ldots,v_N) & \text{ if }|v_1|\geq |v_N| \\ (v_1,v_2,\ldots, v_{i-1},v_i,v_i,v_{i+1},\ldots,v_{N-1}) & \text{ if }|v_N| >|v_1| \end{cases}$$ In words, if $v$ describes the positions of $n$ particles, then $l(v,i)$ duplicates the $i$\textsuperscript{th} smallest particle of $v$ whilst killing the leftmost particle, and $k(v,i)$ duplicates the $i$\textsuperscript{th} smallest particle of $v$ whilst killing the particle of largest magnitude. Note that by definition, we certainly have that $k(v,i)\oop l(v,i)\oop l(v',i)$ for any $v, v' \in \B{R}^N$ such that $v\oop v'$ and $i\in \{1,2,\ldots,N\}$. Finally, we can define $$\Upsilon(\nu,B,Q,I,\tau_{i+1})=k(\Theta^N(\Upsilon(\nu,B,Q,I,\tau_{i+1}-)),I_{i+1}),$$ and $$\Xi(\nu,B,Q,I,\tau_{i+1})=l(\Theta^N(\Xi(\nu,B,Q,I,\tau_{i+1}-)),I_{i+1}),$$ so that it clearly follows that: $$\Upsilon(\nu,B,Q,I,\tau_{i+1})\oop \Xi(\nu,B,Q,I,\tau_{i+1})\oop \Xi(\nu',B,Q,I,\tau_{i+1}).$$ Hence by induction, this ordering holds for all $t\geq 0$. It is obvious from the construction that $\Upsilon(\nu,B,Q,I,t)_{t\geq 0}$ is indeed an $N$-Brownian bees process and $\Xi(\nu,B,Q,I,t)_{t\geq 0}$ is indeed an $N$-BBM process. \end{proof} Now we note that if we want to consider an $N$-BBM or an $N$-Brownian bees process with added drift of $\mu$, we can simply consider it as $\Upsilon(\nu,B^\mu,Q,I,t)_{t\geq 0}$ or $\Xi(\nu,B^\mu,Q,I,t)_{t\geq 0}$ respectively, where $B^\mu$ is a family of i.i.d Brownian motions each with drift $\mu$. It immediately follows that the coupling still holds. From here on, $Z^{N,\mu}$ will be an $N$-BBM process in which each particle has an additional drift of $\mu\in\B{R}$ on each particle, and similarly $X^{N,\mu}$ will be an $N$-Brownian bees process with an additional drift of $\mu \in \B{R}$ on each particle. The next result, Lemma \ref{absCouple}, gives another coupling between an $N$-Brownian bees process and an $N$-BBM. It essentially says that, on time intervals when no particle hits the origin, we can couple the absolute values of $N$-Brownian bees with $N$-BBM. To prove this, we will give an alternative way to represent the $N$-Brownian bees as a function of an initial configuration, a Poisson point process, $N$ i.i.d Brownian motions, and a sequence of uniform random variables. The ordering of this coupling will break down as soon as a particle hits the origin, but this will not be a problem for how we will use the result. \vspace*{1em} \begin{lemma} \label{absCouple} Let $\mu \leq 0$, and let $B$, $\nu$, $Q$, $I$, and $\Xi$ be as in Proposition \ref{generalCouple}. Then there exists a function $\tilde{\Upsilon}$ such that we can write: $$(X^{N,\mu}(t))_{t\geq 0}=\Theta^N\left(\tilde{\Upsilon}(\nu,B^\mu,Q,I,t)_{t\geq 0}\right).$$ so that, if $T:=\inf\{t\geq 0:X_i^{N,\mu}(t)=0 \text{ for some i}\}$ is the first hitting time of the origin by a particle of $X^{N,\mu}$, and $\tilde{\nu}\oop -|\nu|$ (where $|\nu|$ indicates taking absolute value element-wise), then: $$\Xi(\tilde{\nu},B^\mu,Q,I,t) \oop -|\tilde{\Upsilon}(\nu,B^\mu,Q,I,t)|\; \text{ for }t\leq T.$$ \end{lemma} \begin{proof} As with $\Upsilon(\nu,B^\mu,Q,I,t)$, we will define $\tilde{\Upsilon}(\nu,B^\mu,Q,I,t)_{t\geq 0}$ recursively. Let $\tilde{\Upsilon}(\nu,B^\mu,Q,I,0)=\nu$, and suppose for induction that we have constructed $\tilde{\Upsilon}(\nu,B^\mu,Q,I,t)$ up to time $\tau_i$ so that $\Xi(\tilde{\nu},B^\mu,Q,I,t)\oop -|\tilde{\Upsilon}(\nu,B^\mu,Q,I,t)|$ for $t\in [0,\tau_i]$. We will now construct the process up to time $\tau_{i+1}$. Similarly to the previous proof, $\tilde{\Upsilon}_j(\nu,B^\mu,Q,I,t)$ denotes the $j$\textsuperscript{th} smallest particle of $\tilde{\Upsilon}(\nu,B^\mu,Q,I,t)$ and define \[S_{i,j} := \text{sign} \left( \tilde{\Upsilon}_j(\nu,B^\mu,Q,I,\tau_i) \right) \in\{-1,+1\}.\] Let us write the Brownian motions with drift $\mu$, $B^\mu$ as $B^{j}(t)+\mu t$ for each $1\leq j\leq N$. Then for $t\in [\tau_i,\tau_{i+1})$, we will drive the particle $\tilde{\Upsilon}_j(\nu,B^\mu,Q,I,\tau_i)$ by the Brownian motion $$ (-S_{i,j}B^{j}(t)+\mu t)_{\tau_i \leq t< \tau_{i+1}\wedge T};$$ that is, for $t \in [\tau_i, \tau_{i+1}\wedge T)$ we define \[ \tilde \Upsilon_j(t) = \tilde \Upsilon_j(\tau_i) - S_{i,j}(B^j(t)-B^j(\tau_i))+\mu (t-\tau_i) \] Then we observe that before time $T$, $\tilde{\Upsilon}_j(t)$ and $\tilde{\Upsilon}_j(\tau_i)$ have the same sign, and therefore \[ - |\tilde \Upsilon_j(t) | = -S_{i,j}\tilde{\Upsilon}_j(t) = -|\tilde \Upsilon_j(\tau_i)| + (B^{j}(t)-B^j(\tau_i)) - S_{i,j} \mu (t-\tau_i), \quad t \in [\tau_i, \tau_{i+1}\wedge T). \] Since we have $\Xi_j(\tilde{\nu},B^\mu,Q,I,\tau_i)\leq -|\tilde{\Upsilon}_j(\nu,B^\mu,Q,I,\tau_i)|$, we must also have that for $t\in [\tau_i,\tau_{i+1}\wedge T)$: \begin{align*} \Xi_j(\nu,B^\mu,Q,I,t) &=\Xi_j(\nu,B^\mu,Q,I,\tau_i)+B^{j}(t)-B^{j}(\tau_i)+\mu(t-\tau_i) \\ &\leq -|\tilde{\Upsilon}_j(\nu,B^\mu , Q,I,\tau_i)| + B^{j}(t)-B^{j}(\tau_i) - S_{i,j}\mu(t-\tau_i) \\& =-|\tilde{\Upsilon}_j(\nu,B^\mu , Q,I,t)| \end{align*} where we have used that $\mu\leq 0$ and $S_{i,j}\in \{-1,+1\}$. It remains to describe what happens at the branching times. Recall the branching-selection operators $k,l$ from the previous proof, and let $\tilde{I}_{i+1}$ be the index such that $-|\tilde{\Upsilon}_{\tilde{I}_{i+1}}(\nu,B^\mu,Q,I,\tau_{i+1})|$ is the $I_{i+1}$\textsuperscript{th} smallest element of $-|\tilde{\Upsilon}(\nu,B^\mu,Q,I,\tau_{i+1})|$. So then if we define: $$\tilde{\Upsilon}(\nu,B^\mu,Q,I,\tau_{i+1})=k\left(\Theta^N\left(\tilde{\Upsilon}(\nu,B^\mu,Q,I,\tau_{i+1}-)\right),\tilde{I}_{i+1}\right),$$ it immediately follows that if $\tau_{i+1}<T$, then \begin{align*}\Xi(\tilde{\nu},B^\mu,Q,I,\tau_{i+1})=l\left(\Theta^N\left(\Xi(\tilde{\nu},B^\mu,Q,I,\tau_{i+1}-)\right),I_{i+1}\right) \oop l\left(-\left|\Theta^N\left(\tilde{\Upsilon}(\nu,B^\mu,Q,I,\tau_{i+1}-)\right)\right|,I_{i+1}\right)\\ =k\left(\Theta^N\left(\tilde{\Upsilon}(\nu,B^\mu,Q,I,\tau_{i+1}-)\right),\tilde{I}_{i+1}\right)=\tilde{\Upsilon}(\nu,B^\mu,Q,I,\tau_{i+1}). \end{align*} Finally, all that remains is to observe that $\tilde{\Upsilon}(\nu,B^\mu,Q,I,t)_{t\geq 0}$ is a cadlag stochastic process with particles moving like Brownian motions with drift $\mu$. Moreover, at rate $N$, a uniformly randomly chosen particle branches, and by the definition of $k$, the particle furthest from the origin is removed. Therefore $\Theta^N(\tilde{\Upsilon}(\nu,B^\mu,Q,I,t)_{t\geq 0}$ is an $N$-Brownian bees process with drift $\mu$. \end{proof} Recall that for the sub-critical case of Theorem \ref{mainThem}, we want to prove the following: \begin{theorem*} Let $(X^{N,\mu}(t))_{t\geq 0}$ be a one dimensional Brownian bee system with sub-critical drift $\mu<\mu_c^N$. Then $(X^{N,\mu}(nt_0))_{n\in\B{N}}$ is positive Harris recurrent for any $t_0>0$ and has a unique stationary measure $\pi^{N,\mu}$ on $\B{R}^N$ so that for any $\chi\in\B{R}^N$ $$\lim_{t\to\infty}\sup_{C}|\B{P}(X^{N,\mu}(t)\in C|X^{N,\mu}(0)=\chi)-\pi^{N,\mu}(C)|_{TV}=0.$$ \end{theorem*} Therefore let us begin this section by defining positive Harris recurrence. We use Definition (2.2) in Athreya and Ney (\hspace{1sp}\cite{athreyaNey}), which is later used in the context of branching-selection particle systems by Berestycki et al. and Durrett \& Remenik (Proposition 6.5 in \cite{beesBBNP} and Proposition 3.1 in \cite{durrRemenik} respectively). As in \cite{beesBBNP}, this allows us to apply Theorems 4.1 and 6.1 of \cite{athreyaNey} which essentially say that the stochastic process has a unique invariant probability distribution towards which it converges. \begin{defn} A stochastic process $(Y_n)_{n\in\B{N}}$ with state space $\C{Y}$ is called a \textbf{recurrent and strongly aperiodic Harris chain} if $\exists A\subseteq \C{Y}$ such that: \begin{itemize} \item $\exists \varepsilon >0$ and probability measure $q$ on $A$ such that $\B{P}_\xi(Y_1\in C)\geq \varepsilon q(C) \; \; \forall \xi \in A, C\subseteq A$ \item the hitting time of $A$ is almost surely finite for any initial $Y_0 \in \C{Y}$; that is $\B{P}_\xi(\tau_A < \infty)=1 \; \; \forall \xi \in \C{Y}$ where $\tau_A := \inf\{ n\geq 1: Y_n\in A\} $. \end{itemize} Further, we call the proces \textbf{positive Harris recurrent} if $\sup_{\xi \in A}\B{E}_\xi[\tau_A]<\infty $, and \textbf{null Harris recurrent} otherwise. \end{defn} We begin by proving the additional condition for \textit{positive} Harris recurrence. In particular, we will first prove the follow proposition: \begin{propn} \label{finExpNBBM} Let $Z^N$ be an N-BBM system with drift $\mu > -\mu_c^N$. Let $\mathfrak{X}^-_{N,s}$ be the set of initial configurations of $Z^N$ in which the rightmost particle is at position $s\in \B{R}$; that is, $Z_N^N(0)=s$. Let the stopping time $T^\mu_0$ be defined by $$T^\mu_0:=\inf\{t\geq 0:Z^N_N(t)\geq 0\}$$ Then there exist finite constants $a,b$ depending on $\mu$ such that $\sup_{x\in\mathfrak{X}^-_{N,-s}}\B{E}_x[T^\mu_0]\leq a+bs<\infty$. \end{propn} \begin{proof} For the purpose of this proof, it will be helpful to consider, for $k\in \B{N}$, the $N$-BBM process $(Z^N(t))_{k\leq t\leq k+1}$ as a function of $Z^N(k)$ and $N$ independent binary branching Brownian motions (BBMs), $(\C{B}_{k}^1(t))_{0\leq t\leq 1},\ldots$,$(\C{B}_k^N(t))_{0\leq t\leq 1}$. The BBM $(\C{B}_k^i(t))_{0\leq t\leq 1}$ starts from a single particle at the origin and has constant drift $\mu$. We let $\C{N}_k^i(t)$ denote the number of particles in the BBM $\C{B}_k^i$ at time $t$, and label its particles, in the order in which they are born, by $B_k^{i,1}(t),\ldots B^{i,\C{N}_k^i(t)}_k(t)$. We also choose that $(\C{B}_k^i(t))_{0\leq t\leq 1}$ will be the BBM which drives the particle which is $i$\textsuperscript{th} smallest at time $k$, $Z_i^N(k)$ Then given $Z^N(k)$, we can construct $(Z^N(t))_{k\leq t\leq k+1}$ as a subset of $$\bigcup_{i=1}^N \bigcup_{j=1}^{\C{N}_k^i(t)}\{Z_i^N(k)+B^{i,j}_j(t)\}=:U(t)$$ as follows. Particles in $U(t)$ are of two different types, `alive' and `ghost': \begin{itemize} \item At time $k$, all particles are `alive'. \item When an `alive' particle branches, it branches into two `alive' particles, and simultaneously the leftmost `alive' particle (which may be one of the particles involved in the branching event) changes to `ghost' \item When a `ghost' particle branches, it branches into two `ghost' particles. \end{itemize} Then at time $t$, the $N$-BBM $Z^N(t)$ is described by the positions of the $N$ `alive' particles in $U(t)$. For $t\in [k,k+1]$, write \begin{align} Z^N(t)=\Phi\left(Z^N(k),\{(\C{B}^i_k(t))_{0\leq t\leq 1}:i=1,\ldots,N\}\right) \end{align} to see explicitly $Z^N(t)$ as a function of $Z^N(k)$ and $N$ independent BBMs. Now for $k\in \B{N}$, we will consider the following event $A_k := A^{(1)}_k \cap A^{(2)}_k \cap A^{(3)}_k \cap A^{(4)}_k$, where \begin{align*} A^{(1)}_k &=\{\C{N}^N_{k-1}(1)=N\}\\ A^{(2)}_k &=\{\C{N}^1_{k-1}(1)=\C{N}^2_{k-1}(1)=\ldots =\C{N}^{N-1}_{k-1}(1)=1\}\\ A^{(3)}_k &=\{\text{At each branching time }T_1,\ldots,T_{N-1}\text{ of }(\C{B}^N_{k-1}(t))_{0\leq t\leq 1}\text{ we have }B^{i,1}_{k-1}(T_j)<0\\ &\quad \quad \quad\text{for }i,j=1,\ldots,N-1\}\\ A^{(4)}_t &=\{B^{N,i}_{k-1}(T_j)>0\text{ and }B^{N,i}_{k-1}(t)<1\text{ for }i=1,\ldots,\C{N}^{N}_{k-1}(T_j),\;j=1,\ldots,N-1\text{ and }t\in[0,1]\} \end{align*} and define the sequence of stopping times by $\tau_0=0$ and for $i\geq 1$: $$ \tau_i = \inf\{t\in \B{N}: t> \tau_{i-1},\; A_t\text{ occurs}\} $$ In plain English, these are essentially the integer times when the rightmost particle of the system `regenerates' the system by becoming an ancestor of every particle alive in one unit of time. Conditions $A_k^{(3)},A_k^{(4)}$ ensure that at each branching time, the leftmost `alive' particle is one of the particles $Z^N_i(k-1)+B^{i,1}_{k-1}(T_j)$ for $i=1,\ldots, N-1$ and therefore the $N$ `alive' particles at time $k$ are the $N$ particles of the BBM $\C{B}_{k-1}^{N}$. Therefore, the particles alive at time $\tau_i$ are located at \begin{equation}\label{pclPosns}\{Z^N_N(\tau_i-1)+B^{N,j}_{\tau_i-1}(1), j=1,\ldots,\C{N}^N_{\tau_i-1}(1)\}.\end{equation} This construction of the $N$-BBM process then makes it explicitly clear that the event $A_t$ is independent of $Z^N(t-1)$; that is, we can verify whether $A_t$ occurs given only the driving BBMs. Therefore it follows that $\tau_{i+1}-\tau_i$ are i.i.d., and by definition of $\tau_i$, $\B{E}[\tau_{i+1}-\tau_i]\geq 1$. The definition of $A_t$ also makes it clear that the events $A_1,A_2,A_3,\ldots$ are independent and $\B{P}(A_1)=\B{P}(A_2)=\ldots$, therefore $\inf\{k\in \B{N}:A_k\text{ occurs}\}$ is a geometric random variable which bounds $\tau_1$ from above, therefore $\B{E}[\tau_{i+1}-\tau_i]<\infty$. Furthermore, since the processes $(\C{B}_{\tau_i-1}^{N}(t))_{0\leq t\leq 1}$ are i.i.d, we clearly have that $Z^N(\tau_{i+1})-Z^N(\tau_i)$ are i.i.d for $i\geq 1$ (although $Z^N(\tau_1)-Z^N(\tau_0)$ will depend on the initial condition). Then since $\tau_i\xrightarrow[i\to\infty]{a.s.}\infty$, we have $\lim_{i\to\infty}Z(\tau_i)/\tau_i= \lim_{t\to\infty}Z(t)/t = \mu+\mu_c^N$. So by \cite{janson} (equation (1.5)), we have that there exist finite constants $a',b'\geq 0$ such that $$\B{E}\left[\inf\{i\in \B{N}:Z^N_N(\tau_{i+1})-Z^N_N(\tau_1)\geq R\}\right]\leq a'+b'R.$$ Now observe that $Z^N_N(\tau_1)-Z^N_N(0)$ is stochastically dominated by the maximum of a BBM started from $N$ particles. Also, since $\tau_1$ is bounded above by a geometric random variable, $\B{E}e^{\tau_1} < \infty$, therefore by the many-to-one lemma (see for example Lemma 2.1 in \cite{kim}), we have that $$\B{E}[Z_N^N(\tau_1)-Z^N_N(0)]<N\B{E}[e^{\tau_1}]\B{E}\left[\sup_{t<\tau_1}B_t\right]<\infty.$$ Thus it follows that for any $\xi \in \mathfrak{X}^-_{N,-s}$ we have \begin{align*}\B{E}_\xi\left[\inf\{i\in \B{N}:Z_N^N(\tau_{i+1})\geq 0\}\right]&=\B{E}_\xi\left[\inf\{i\in \B{N}:Z^N_N(\tau_{i+1})-Z^N_N(\tau_1)\geq s-Z^N_N(\tau_1)+Z^N_N(0)\}\right]\\ &\leq a' + b'\left(s-\B{E}_\xi[Z^N_N(\tau_1)-Z^N_N(0)]\right).\end{align*} The inequality follows by conditioning on $Z^N_N(\tau_1)$, and noting that $Z^N_N(\tau_{i+1})-Z^N_N(\tau_i)$ is dependent on the distribution of particles $Z^N_N(\tau_i)$ as seen from the rightmost particle $Z^N_N(\tau_i)$, but independent of $Z^N_N(\tau_i)$ itself; therefore independently of $Z^N_N(\tau_1)$, $Z^N_N(\tau_{i+1})-Z^N_N(\tau_i)$ are i.i.d. for $i\geq 1$. Thus $$\B{E}_\xi[T_0^\mu] \leq \B{E}[\tau_1-\tau_0]\B{E}_\xi\left[\inf\{i\in \B{N}:Z_N^N(\tau_{i+1})\geq 0\}\right] \leq \B{E}[\tau_1-\tau_0]\left(a' + b'(s-\B{E}_\xi[Z^N_N(\tau_1)-Z^N_N(0)])\right)=: a+bs,$$ thus completing the proof. \end{proof} Note that the restriction to the set $x\in \mathfrak{X}^-_{N,-s}$ is necessary and the result is not true for general starting configurations, since we could start from a position in which the rightmost particle is arbitrarily far from the origin and therefore has arbitrarily large hitting time. We can start with an initial condition in which all particles except for the rightmost are arbitrarily far from the origin, but the proposition is essentially saying that, since the rightmost particle will eventually have $N$ descendants, the arbitrarily far left positions will not ultimately matter. Note also that the drift $-\mu_c^N$ is the critical drift at which the expected time becomes infinite. We are now ready to prove the positive Harris recurrence of the Brownian bees: \begin{theorem} \label{BeesPosRec} Let $X^{N,\mu}$ be an $N$ Brownian bee system with drift $\mu \in(-\mu_c^N,\mu_c^N)$. Then $(X^{N,\mu}(t_0 n))_{n\in \B{N}}$ is positive Harris recurrent for any $t_0 > 0$ \end{theorem} \begin{proof} Assume without loss of generality $\mu_c^N < \mu < 0$. In the case of $\mu=0$, Theorem \ref{mainThem} is proven in \cite{beesBBNP}. Let $A=[-1,1]^N$ and $\tau_A:=\inf\{n\geq 1:X^{N,\mu}(t_0n)\in A\} $ be the hitting time to $A$ by $(X^{N,\mu}(t_0n))_{n\in\B{N}}$. Define $\tilde{X}(t)$ to be the position of the particle of $X^{N,\mu}(t)$ closest to $0$. Let $\tilde{\sigma}_0:=0$ and recursively define $\tilde{\sigma}_{i+1}:=\inf\{n \in \B{N}: n > \tilde{\sigma}_{i}: \tilde{X}(s)=0\text{ for some }s\in [(n-1)t_0,nt_0]\}$. Define $\mathfrak{X}_{N,M}$ to be the set of configurations of $N$ particles with the particle closest to zero in $[-M,M]$. Now our first observation is that as $\tilde{X}(s)=0$ for some $s \in ((\tilde{\sigma}_i-1)t_0,\tilde{\sigma}_i t_0)$, then $X^{N,\mu}((\tilde{\sigma}_i+1) t_0) \in A$ with non-zero probability bounded below by $p_0$. This can happen by the particle closest to zero branching $N-1$ times in $(\tilde{\sigma}_i t_0,(\tilde{\sigma}_i+1)t_0)$ and the Brownian motions driving the particles having sufficiently small displacements. Next we want to control the `bad' events where $\tilde{X}(s)$ hits $0$ in $s \in ((\tilde{\sigma}_i-1)t_0,\tilde{\sigma}_i t_0)$ but we fail to have $X^{N,\mu}((\tilde{\sigma}_i+1)t_0)\in A$. Using crude bounds on Brownian motion $B$, we can see that \begin{align*}\B{P}\Big(|\tilde{X}((\tilde{\sigma}_1+1)t_0)| &\in (M,M+1]\Big)\leq \B{P}\left(\sup_{t\leq 2t_0}|B_{2t_0}-B_t| > M\right) \\ =&\B{P}\left(\sup_{t\leq 2t_0}B_t + \mu_t > M\right)+\B{P}\left(\inf_{t\leq 2t_0}B_t + \mu_t < -M\right)=2\B{P}(\tilde{B}_{2t_0}>M-\mu t_0)+2\B{P}(\tilde{B}_{2t_0}>M+\mu t_0)\end{align*} where $\tilde{B}$ is a standard Brownian motion and $M \in \B{N}$. This follows from the fact that $|\tilde{X}(t)|$ is bounded above by $|B(t)|$, and therefore $\B{P}(|\tilde{X}|\in (M,M+1])\leq \B{P}(|B(t)|>M)$. The final equality is an immediate application of the reflection principle. Then using the Chernoff bound, we can bound this above by $$ \B{P}\left(|\tilde{X}((\tilde{\sigma}_1+1)t_0)| \in (M,M+1]\right)\leq \bar{C}\exp\left(-\frac{(M-\mu t_0)^2}{4t_0}\right) + \bar{C}\exp\left(-\frac{(M+\mu t_0)^2}{4t_0}\right), $$ for some constant $\bar{C}$. Then by Lemma \ref{absCouple}, there is a coupling between drifted $N$-BBM, $Z^{N,\mu}$, and $X^{N,\mu}$ such that before $\tilde{X}(t)$ hits zero, we have $Z^{N,\mu}(0)=-|X^{N,\mu}(0)| \implies Z^{N,\mu}(t)\oop -|X^{N,\mu}(t)|$. Thus $\tilde{X}(s)$ must hit $0$ before $Z^{N,\mu}_N(s)$ hits $0$, and we can use the coupling of Lemma \ref{absCouple} and Proposition \ref{finExpNBBM} to give the upper bound for $M\in \B{N}$ $$\sup_{x\in \mathfrak{X}_{N,M}}\B{E}_x[\tilde{\sigma}_1]<a + bM \leq C_\mu M,$$ for some constant $C_\mu<\infty$ which depends on $\mu$ and $t_0$. Therefore combining our two bounds, we show that the return time of $\tilde{X}$ to $0$ has finite expectation: $$ \B{E}[\tilde{\sigma}_{i+1}-\tilde{\sigma}_i]\leq \sum_{M=1}^\infty \sup_{x\in\mathfrak{X}_{N,-M-1}} \B{E}_x[\tilde{\sigma}_1]\B{P}(|\tilde{X}(\tilde{\sigma}_i)|\in (M,M+1]) =: C < \infty $$ Then noting that at each time $(\tilde{\sigma}_i+1)t_0$, $\tilde{X}((\tilde{\sigma}_i+1)t_0)\in A$ with probability at least $p_0$, and therefore $\sup_{x\in A}\B{E}_x[\tau_A]\leq C\B{E}[Geo(p_0)]<\infty$, where $Geo(p_0)$ is a geometric random variable with parameter $p_0$. Therefore to prove positive Harris recurrence it remains only to show that $\exists \varepsilon>0$ and measure $q$ such that $$\B{P}_a(X^{N,\mu}(t_0)\in S)\geq \varepsilon q(S)\quad \text{for all }a\in A, S\subseteq A.$$ Fix $a=(a_1,...,a_N)\in A$, $S \subseteq A$. Then conditioning on the event that no branching occurs in $[0,t_0]$ (which happens with probability $e^{-t_0N}$), we have: \begin{align} \label{recurrenceLeb} \B{P}(X^N(t_0)\in S&|X^N(0)=a) \geq e^{-t_0N} \int_S \prod_{i=1}^N(2\pi t_0)^{-1/2}e^{-\frac{(x_i-a_i)^2}{2t_0}}dx_N...dx_1 \\ &\geq e^{-t_0N} (2\pi t_0)^{-N/2}\int_S\prod_{i=1}^Ne^{-1/t_0}dx_N...dx_1 = e^{-t_0N}(2\pi t_0)^{-N/2}e^{-N/t_0}Leb(S) \end{align} So normalising on $A$, which has finite Lebesgue measure, the condition is met. Thus $(X^{N,\mu}(t_0 n))_{n\in\B{N}}$ is positive Harris recurrent for $\mu \in (-\mu_c^N,\mu_c^N)$ \end{proof} \begin{proof} \textit{(Of Theorem \ref{mainThem}.1)} We have proven that for any $t_0>0$ the process $(X^{N,\mu}(nt_0))_{n\in \B{N}}$ is a positive recurrent and strongly aperiodic Harris chain. Therefore by Theorems 4.1 and 6.1 of \cite{athreyaNey}, the conclusion of the Theorem \ref{mainThem}.1 holds. \end{proof} In the early 2000's, the physicists Brunet and Derrida pioneered the study of branching particle systems with selection. In these so called \textit{Brunet-Derrida} or \textit{Branching-Selection} systems, we have a number of particles which can branch and have offspring, and also some \textit{selection mechanism/rule} which removes individuals from the population, keeping the population size constant. The particles removed at branching events are typically the particle of least `fitness' according to some fitness function; for example $f(x)=-\|x\|$, in which case it is always the particle furthest from the origin which is removed. We can also remove particles from the system by measuring fitness with respect to the whole system; for example, removing the particle furthest from the barycentre (as in \cite{bbb}). The selection mechanism therefore introduces a `survival-of-the-fittest' phenomenon into the model. A well known branching-selection particle system is the $N$-branching Brownian motion ($N$-BBM) which is studied in \cite{selecJBOT}, \cite{hydroNBBM}, \cite{maillard}. In the $N$-BBM, $N$ particles behave as independent Brownian motions on the real line, each branching into 2 particles at rate 1, and at each branching event, the leftmost particle is removed in order to keep a fixed-size population of $N$ particles. This mechanism introduces a \textit{selective pressure} which `pushes' the cloud of particles to the right. Writing $Z(t)=(Z_1(t),...,Z_N(t))\in\B{R}^N$ for the positions of the particles at time $t$, then we say that $Z$ has velocity $v_N$ if the following limits exist and are equal: $$ v_N = \lim_{t\to\infty}\min_{1\leq i\leq N} \frac{Z_i(t)}{t} = \lim_{t\to\infty}\max_{1\leq i\leq N} \frac{Z_i(t)}{t} $$ It is well known in the literature (see for example Section 1.3 of \cite{maillard} or Remark 1.6 of \cite{shape}) that the velocity of the $N$-BBM is given by: $v_N = \sqrt{2} - \frac{\pi^2}{\sqrt{2}(\log N)^2} + o\left(\frac{1}{(\log N)^2}\right).$ This is very similar to B\'{e}rard and Gou\'{e}r\'{e} (\hspace{1sp}\cite{berardGouere}), who prove an analogous result for the $N$-branching random walk. A complete proof for the case of the $N$-BBM is given in the appendix. Another branching-selection particle system which has attracted attention lately is called `Brownian bees', and is studied by Berestycki, Brunet, Nolen, and Pennington in \cite{afree}, \cite{beesBBNP}. In the $N$-Brownian bees particle system, $N$ particles behave as Brownian motions, each branching at rate $1$. At each branching event, we delete the particle which is furthest from the origin. The $N$-Brownian bees system is defined in $\B{R}^d$ for any number of dimensions, $d$, but in this paper we specifically study $d=1$. A number of things are known about the $N$-Brownian bees system. Firstly, the system is positive Harris recurrent (Proposition 6.5 in \cite{beesBBNP}). It is also known that in the large population limit, the empirical distribution of particles converges to the solution of a free boundary problem. The aim of this paper is to study the $N$-Brownian bees system in which each particle also has a drift of $\mu$. Intuitively, when the drift is small, the system shouldn't behave much differently to the $N$-Brownian bees system without drift. Essentially, the selection rule which deletes the furthest particle from the origin outweighs the effect of the drift and succeeds in keeping all particles near to the origin. Alternatively, when the drift is sufficiently large, it will win against the selection mechanism and particles will be drifting off to $\pm\infty$. We will therefore prove that there is a critical drift $\mu_c^N$ such that when $|\mu| < \mu_c^N$, the $N$-Brownian bee system is positive Harris recurrent, and when $|\mu| > \mu_c^N$, the system is transient in the sense that each particle diverges to $\frac{\mu}{|\mu|}\infty$ as $t\to\infty$. Now we can observe that when all $N$ particles of a Brownian bees system lie in $(-\infty,0]$, the system has the same behaviour as $N$-BBM with added drift $\mu$, which has asymptotic velocity $\mu + v_N$. Therefore we should expect the critical drift $\mu_c^N$ to be: $$\mu_c^N = v_N = \sqrt{2} - \frac{\pi^2}{\sqrt{2}(\log N)^2} + o\left(\frac{1}{(\log N)^2}\right)$$ In the critical case where $|\mu|=\mu_c^N$, the system becomes null-recurrent and, under a suitable rescaling, the $N$ particles act like a single point mass behaving as a reflected Brownian motion. Our main result is the following theorem: \begin{theorem} \label{mainThem} Let $X^{N,\mu}=(X^{N,\mu}(t))_{t\geq 0}$ be a one dimensional $N$-Brownian bee system with drift $\mu\in \B{R}$, and let $\mu_c^N=v_N$. Then: \begin{enumerate} \item \underline{Sub-Critical Case}: when $|\mu| < \mu_c^N$, then for any $t_0>0$, the process $(X^{N,\mu}(nt_0))_{n\in \B{N}}$ is positive Harris recurrent and therefore has a unique stationary probability measure $\pi^{N,\mu}$ on $\B{R}^N$ so that for any $\chi\in \B{R}^N$ \begin{equation*} \lim_{t\to\infty}\sup_{C}|\B{P}_{\chi}(X^{N,\mu}(t)\in C)-\pi^{N,\mu}(C)|_{TV}=0. \end{equation*} where $|\cdot|_{TV}$ represents the total variation norm, the supremum is taken over all measurable sets, and $\B{P}_\chi(\cdot):=\B{P}(\cdot|X^{N,\mu}(0)=\chi)$. \item \underline{Critical Case}: When $|\mu |=\mu_c^N$, there is an invariance principle $$ \left(m^{-1/2}X^{N,\mu}(mt)\right)_{t\geq 0} \xrightarrow[m\to\infty]{d} \left(\frac{\mu}{|\mu|}\beta^{-1/2}\sigma |B(t)|\underline{1}\right)_{t\geq 0}. $$ in the Skorokhod topology on $\C{D}([0,\infty),\B{R}^N)$, the space of cadlag functions $[0,\infty)\to \B{R}$, where $B$ is a standard Brownian motion, $\underline{1}=(1,\ldots,1)\in \B{R}^N$, and $\beta,\sigma$ are constants which we will define later. \item \underline{Super-Critical Case}: When $|\mu|>\mu_c^N$, $X^{N,\mu}$ is transient in the sense that each particle $X_i^{N,\mu}$ diverges to $\frac{\mu}{|\mu|}\infty$ as $t\to\infty$, and further $$\lim_{t\to\infty}\frac{X^{N,\mu}_1(t)}{t}=\lim_{t\to\infty}\frac{X^{N,\mu}_N(t)}{t} = \left(|\mu| - \mu_c^N\right)\frac{\mu}{|\mu|} \quad \text{a.s.}$$ where $X_i^{N,\mu}$ denotes the $i$\textsuperscript{th} smallest particle of $X^{N,\mu}$. \end{enumerate} \end{theorem} The subcritical and supercritical cases of Theorem \ref{mainThem} were also independently proven in the master's thesis of Flynn (\hspace{1sp}\cite{flynn}). As we noted above, the behaviour of the $N$-Brownian bees system without drift is known when the number of particles tends to $\infty$. In particular for $d=1$, it is known that as $N\to\infty$, the empirical distribution of particles converges to the solution of a free boundary problem given by \begin{align} \label{beesFBP1} \begin{cases} u_t=\frac{1}{2}u_{xx} + u & x\in(-R_t,R_t), t>0, \\ \int_{-R_t}^{R_t} u(s,t)ds=1 & t>0, \\ u(x,t)=0 & x\notin (R_t,R_t), t>0, \\ u \text{ continuous}. \end{cases} \end{align} with an initial condition based on the initial distribution of particles. This result is proven by Berestycki et al. in \cite{afree}, \cite{beesBBNP}. We conjecture here that for Brownian bees with sub-critical drift $\mu$, the empirical distribution converges to the solution of the free boundary problem: \begin{align} \begin{cases} u_t=\frac{1}{2}u_{xx} - \mu u_x + u & x\in(-R_t,R_t), t>0, \\ \int_{-R_t}^{R_t} u(s,t)ds=1 & t>0, \\ u(x,t)=0 & x\notin (R_t,R_t), t>0, \\ u \text{ continuous}. \end{cases} \end{align} The methods of proof used by Berestycki et al. rely on the symmetry of the system, and view it essentially as a system of reflected Brownian motions on $[0,\infty)$. However these methods no longer work when drift is introduced, since $|B_t + \mu t|$ does not behave like a reflected Brownian motion and in fact is not even Markovian. Therefore this conjecture remains unproven. Another property that we will need in the course of our proof is that $N$-BBM is associated. Association is a property of random variables which can essentially be thought of as meaning that `an FKG-type inequality holds' (see definition 1.1 of \cite{assoc}). \begin{defn} Let $X$ be a random variable taking values in a partially ordered space $\C{E}$ with partial ordering $\leq_o$. A function $f:\C{E}\to \B{R}$ is called \textbf{increasing} if for $e_1,e_2\in \C{E}$, we have $e_1 \leq_o e_2 \implies f(e_1)\leq f(e_2)$. An event $A$ is called increasing if the indicator of $A$ is an increasing function. We call the random variable $X$ \textbf{associated} if we have $\B{E}[f(X)g(X)]\geq \B{E}[f(X)]\B{E}[g(X)]$ for all increasing functions $f,g$. \end{defn} An immediate consequence of the definition of association is that if $h:\C{E}\to \C{E}$ is a monotonic increasing function, then $h(X)$ is also associated. This follows because for any increasing functions $f,g$, the monotonicity of $h$ implies that $f\circ h$ and $ g\circ h$ are also increasing functions, so $\B{E}[f(h(X))g(h(X))]\geq \B{E}[f(h(X))]\B{E}[g(h(X))]$. Associated random variables were first studied by Esary, Proschan, and Walkup in \cite{assoc}, and much has been written about them in the literature since then, and we will show here that, by the construction of the $N$-BBM process, this property holds for the $N$-BBM process as well. In particular, we will prove that: \begin{lemma} \label{nbbmAssoc} Fix $T\in (0,\infty)$. Then the $N$-BBM $(Z(t))_{t\in [0,T]}$, viewed as a $\C{D}([0,T],\B{R}^N)$-valued random variable, is associated for all $T$, under the partial order relation $\leq_o$ where $(X_t)_{t\in [0,T]} \leq_o (Y_t)_{t\in [0,T]} \iff X_t \oop Y_t \; \forall t\in [0,T]$. \end{lemma} \begin{proof} We will prove this fact by using the construction: $$(Z^N(t))_{t\geq 0}=\Theta^N\left( \Xi\left(\nu,B, Q, I,t\right)_{t\geq 0}\right)$$ from Proposition \ref{generalCouple}. The space $\C{D}([0,T],\B{R})$ has a natural partial ordering $\leq_T$ where for $x,y\in \C{D}([0,T],\B{R})$, we have $x\leq_T y$ if and only if $x(t)\leq y(t) \; \forall t\in [0,T]$. It is proven by Alexandre Legrand in \cite{levyAssoc} that any $1$-dimensional L\'{e}vy process when viewed as a $\C{D}([0,T],\B{R})$-valued random variable is associated under the partial ordering $\leq_T$ for any $T$. In particular, this includes the Brownian motions and Poisson process used to construct $Z$ above. The association of Brownian motion was already proven by David Barbato in \cite{barbato} under a different partial ordering, but Legrand's work extends this to the more natural partial ordering $\leq_T$. We also know that a uniform random variable on the set $\{1,2,\ldots,N\}$ is associated; Chebyshev's sum inequality states that for increasing functions $f,g:{1,\ldots,N}\to \B{R}$, $\frac1N \sum_{i=1}^N f(i)g(i) \geq \left(\frac1N \sum_{i=1}^N f(i)\right)\left(\frac1N \sum_{j=1}^N g(j)\right)$. Another important property of associated random variables is that association of random variables is preserved under taking products. In particular, if $E_i$ is an $\C{E}_i$-valued associated random variables with partial ordering $\leq_i$ in probability space $(\Omega_i,\C{F}_i,\B{P}_i)$ for $i\in \Gamma$, then $(E_i)_{i\in \Gamma}$ is an $\bigotimes_{i\in \Gamma}\C{E}_i$-valued associated random variable in the product probability space, with the product partial order $\leq_p$ (ie. $(a_i)_{i\in \Gamma} \leq_p (b_i)_{i\in \Gamma} \iff a_i \leq_i b_i$ for all $i\in \Gamma$). This is Lemma 2.5 in \cite{levyAssoc} for finite $\Gamma$, or Theorem 3 in \cite{barbato} for more general $\Gamma$. Therefore, as in the statement of Proposition \ref{generalCouple}, define a family of i.i.d Brownian motions $B=((B^{j}(t)){t\geq 0})_{1\leq j\leq N}$, a Poisson point process with intensity $N$, $Q=(Q(t))_{t\geq 0}$, and a family $I=(I_i)_{i\in \B{N}}$ of uniform random variables. Then $(B,Q,I)_{t\in [0,T]}=((B^{j}(t))_{1\leq j\leq N,t\in [0,T]},(Q(t))_{t\in [0,T]},(I_i)_{i\in \B{N}})$ is an associated random variable under the product partial ordering. Finally it remains to show that the $N$-BBM $(Z^N(t))_{t\in [0,T]}=\Theta^N(\Xi(\nu,B,Q,I,t))_{t\in [0,T]}$ is a monotonic function of the associated random variable $(B,Q,I)_{t\in [0,T]}$ and therefore is also associated. The monotonicity of $Z^N$ as a function of $B$ is obvious; increasing the Brownian motions driving the particles increases $Z^N$. It is also obvious that increasing $Q$ increases $Z^N$, as an increased $Q$ means that positive jumps of particles happen sooner. Finally to see that $\Xi(\nu,B,Q,I,t)_{t\in [0,T]}$ is also monotonic as a function $I$, we note that clearly by definition $l(v,i)\oop l(v,j)$ for $i\leq j$. Hence the $N$-BBM is a monotonic increasing function of the associated random variable $(B,Q,I)_{t\in [0,T]}$ and hence $(Z^N(t)_{t\in [0,T]}=\Theta^N(\Xi(\nu,B,Q,I,t))_{t\in [0,T]}$ is associated. \end{proof}
2412.04316v1
http://arxiv.org/abs/2412.04316v1
Stealthy Optimal Range-Sensor Placement for Target Localization
\documentclass[letterpaper,10pt,conference]{ieeeconf} \IEEEoverridecommandlockouts \overrideIEEEmargins \usepackage[compress]{cite} \usepackage{amsmath,amssymb,amsfonts} \usepackage{algorithmic} \usepackage{graphicx} \usepackage{textcomp} \usepackage[dvipsnames]{xcolor} \usepackage{comment} \usepackage{soul} \usepackage[hidelinks]{hyperref} \usepackage{enumerate} \usepackage{mathtools} \newcommand{\defeq}{\coloneqq} \usepackage[capitalize]{cleveref} \usepackage[font=footnotesize]{caption} \usepackage{subcaption} \usepackage{multirow} \usepackage{gensymb} \usepackage{optidef} \usepackage{booktabs} \usepackage{commath} \usepackage{soul} \usepackage{pgfplots} \usepgfplotslibrary{colormaps} \usepgfplotslibrary{patchplots} \pgfplotsset{compat=1.16} \usepackage{matlab-prettifier} \usepackage{cuted} \usepackage{flushend} \usepackage{accents} \usepackage{lipsum} \newcommand{\unbar}[1]{\underaccent{\bar}{#1}} \DeclareRobustCommand{\hlyellow}[1]{#1} \DeclareRobustCommand{\hlorange}[1]{#1} \DeclareRobustCommand{\hlgreen}[1]{#1} \DeclareRobustCommand{\hlcyan}[1]{#1} \newcommand{\cross}[1]{{#1}^{\times}} \newcommand{\tr}[1]{{#1}^{\top}} \newcommand{\R}{\mathbb{R}} \renewcommand{\norm}[1]{\|#1\|} \newcommand{\normm}[1]{\left\|#1\right\|} \newcommand{\bb}[1]{\mathbf{#1}} \newcommand{\bbb}[1]{\boldsymbol{#1}} \renewcommand{\arraystretch}{1.7} \DeclareMathOperator{\sgn}{sgn} \DeclareMathOperator{\sat}{sat} \DeclareMathOperator{\sig}{sig} \DeclareMathOperator{\diag}{diag} \DeclareMathOperator*{\argmax}{argmax} \DeclareMathOperator*{\argmin}{argmin} \DeclarePairedDelimiter\ceil{\lceil}{\rceil} \DeclarePairedDelimiter\floor{\lfloor}{\rfloor} \newtheorem{theorem}{Theorem} \newtheorem{property}{Property} \newtheorem{lemma}{Lemma} \newtheorem{definition}{Definition} \newtheorem{remark}{Remark} \newtheorem{problem}{Problem} \newtheorem{assumption}{Assumption} \newtheorem{proposition}{Proposition} \newcommand{\LL}[1]{\textcolor{blue}{#1}} \newcommand{\LLL}[1]{\textcolor{red}{#1}} \newcommand{\MHY}[1]{\textcolor{orange}{#1}} \title{\LARGE \bf Stealthy Optimal Range-Sensor Placement for Target Localization } \author{Mohammad Hussein Yoosefian Nooshabadi, Rifat Sipahi, and Laurent Lessard\thanks{Research was sponsored by the Army Research Laboratory and was accomplished under Cooperative Agreement Number W911NF-23-2-0014. The views and conclusions contained in this document are those of the authors and should not be interpreted as representing the official policies, either expressed or implied, of the Army Research Laboratory or the U.S. Government. The U.S. Government is authorized to reproduce and distribute reprints for Government purposes notwithstanding any copyright notation herein.}\thanks{All authors are with the Department of Mechanical and Industrial Engineering, Northeastern University, Boston, MA 02115, USA.\newline {\tt\footnotesize\{yoosefiannooshabad.m, r.sipahi, l.lessard\}\newline @northeastern.edu}} } \begin{document} \maketitle \thispagestyle{empty} \pagestyle{empty} \begin{abstract} We study a stealthy range-sensor placement problem where a set of range sensors are to be placed with respect to targets to effectively localize them while maintaining a degree of stealthiness from the targets. This is an open and challenging problem since two competing objectives must be balanced: (a) optimally placing the sensors to maximize their ability to localize the targets and (b) minimizing the information the targets gather regarding the sensors. We provide analytical solutions in 2D for the case of any number of sensors that localize two targets. \end{abstract} \section{INTRODUCTION}\label{sec: intro} We consider the problem of optimal sensor placement subject to stealthiness constraints. In this problem we have a network of range-only sensors and another network of stationary \emph{targets} (also equipped with range-only sensors). The goal is to obtain spatial configurations of the sensors that maximize their ability to localize the targets while limiting the targets' ability to localize the sensors. This problem concerns two competing objectives, \hlorange{each of which has been studied extensively.} The first competing objective is \emph{target localization} where the goal is to solely optimize the sensors' localization performance \cite{bishop2010optimality, moreno2013optimal, sadeghi2020optimal}. In \cite{moreno2013optimal} the optimal relative sensor-target configuration is derived for multiple sensors and multiple targets in 2D settings. In \cite{sadeghi2020optimal} the scenario with multiple sensors and a single target is considered where the sensors are constrained to lie inside a connected region. Both aforementioned works characterize localization performance using the Fisher Information Matrix (FIM) \cite[\S2]{barfoot2024state}. Target localization is a special case of \textit{optimal sensor placement} where the goal is to find the optimal location of a network of sensors such that some notion of information they obtain is maximized; \hlyellow{this problem has many applications, including structural health monitoring} \cite{ostachowicz2019optimization} \hlyellow{and experiment design} \cite{zayats2010optimal}. \looseness=-1 The second competing objective is \textit{stealthy sensor placement} where the goal is to place the sensors in spatial configurations such that the localization performance of the targets is limited. Various measures of localization performance for the adversary sensors are introduced in the literature, including entropy \cite{molloy2023smoother}, predictability exponent \cite{xu2022predictability} and FIM \cite{farokhi2020privacy}. Stealthiness has also been studied in the context of mobile sensors for \hlyellow{different applications} such as adversarial search-and-rescue \cite{rahman2022adversar}, information acquisition \cite{schlotfeldt2018adversarial}, pursuit-evasion \cite{chung2011search} and covert surveillance \cite{huang2022decentralized}. The work in \cite{karabag2019least} uses the FIM to make the policy of a single mobile sensor difficult to infer for an adversary Bayesian estimator. In \cite{khojasteh2022location} a sensor equipped with a range sensor is allowed to deviate from its prescribed trajectory to enhance its location secrecy encoded via the FIM. The notion of stealthiness is also related to the topics of \textit{privacy} and \textit{security}, which have \hlyellow{applications in} numerical optimization \cite{farokhi2020privacy}, machine learning \cite{zhang2016dynamic}, \hlyellow{smart grids} \cite{li2018information}, and communication \cite{wang2021physical}. In the present paper, \hlorange{we combine the two aforementioned objectives by optimizing localization performance subject to a stealthiness constraint, quantifying each using a min-max formulation of the FIM. To the best of our knowledge, the present study is the first to consider this combination of objectives.} \hlyellow{Possible applications include cooperative distributed sensing and cyber-physical system security; detecting suspicious activities without alerting the attackers.} The paper is organized as follows. In \cref{sec: problem setup}, we describe our min-max FIM formulation. The general case of arbitrarily many sensors and targets leads to a difficult non-convex problem, thus we focus on more tractable special cases. In \cref{sec: Problem formulation 2t2a} we provide a complete solution in the case of two sensors and two targets in 2D. Next, in \cref{sec:Extension}, we treat the case of arbitrarily many sensors and two targets and provide various analytic performance bounds. In \cref{sec: Conclusion and Future Work}, we conclude and discuss future directions. \section{PROBLEM SETUP} \label{sec: problem setup} Consider a 2D arrangement of $m$ sensors $s_1,\dots,s_m$ and $n$ targets $t_1,\dots,t_n$. A possible configuration of these sensors and targets is depicted in \cref{fig: problemDefinition}. We let $\theta_{ij,k}$ denote the angle between $s_i$ and $s_j$ as viewed from $t_k$. Similarly, we let $\beta_{k\ell,j}$ denote the angle between $t_k$ and $t_\ell$ as viewed from $s_j$. \begin{figure}[b!] \centering \includegraphics{fig_problem_defn} \caption{The problem setup in this paper. A set of $m$ sensors (in green) are to be placed such that their ability to localize a set of $n$ targets (in red) is maximized while the targets' ability to localize the sensors is limited.} \label{fig: problemDefinition} \end{figure} We assume sensors and targets use \emph{range-only sensing} and each \hlorange{measurement is subject to additive zero-mean Gaussian noise with variance $\sigma^2$.} Due to the noise, the spatial configuration of the sensors relative to the targets is critical to effectively fuse sensor measurements for localization purposes. The FIM is broadly used to quantify the quality of localization \cite{martinez2006optimal}; it is the inverse of the covariance matrix of the target's position conditioned on the observed measurements. We use the so-called \emph{D-optimality} criterion, which is the determinant of the FIM. Other FIM-based criteria include the A-optimality or E-optimality \cite[\S7]{boyd2004convex}. Since we are using range-only sensing in a 2D setting, the D-optimality, A-optimality, and E-optimality criteria are equivalent \cite{sadeghi2020optimal}. We denote by $\mathcal{I}_k$ the D-optimality criterion for sensors $s_1,\dots,s_m$ localizing a target $t_k$ using their collective range measurements. Similarly we denote by $\mathcal{J}_i$ the D-optimality criterion for targets $t_1,\dots,t_n$ localizing a sensor $s_i$, where \begin{align}\label{eq: det FIM of target k} \mathcal{I}_k & = \frac{1}{\sigma^4}\sum_{1 \leq i < j \leq m}\mkern-18mu\sin^2{\theta_{ij, k}}, & \mathcal{J}_i & = \frac{1}{\sigma^4}\sum_{1\leq k < \ell \leq n}\mkern-18mu\sin^2{\beta_{k\ell, i}}. \end{align} For a detailed derivation of \eqref{eq: det FIM of target k}, see \cite{martinez2006optimal}. \hlgreen{Assuming Gaussian noise is critical in obtaining {\eqref{eq: det FIM of target k}}. Intuitively, information from two range measurements is maximized when the range vectors are perpendicular and minimized when they are parallel.} Motivated by the goal of obtaining the best possible localization of a set of targets while avoiding detection by those same targets, we formulate our problem as follows. \begin{problem}[\textit{min-max optimal stealthy sensor placement}]\label{problem 1} Given target locations $t_1,\dots,t_n$ find angles $\theta_{ij, k}$ and $\beta_{k\ell, i}$ shown in \cref{fig: problemDefinition} corresponding to a feasible arrangement of the sensors $s_1,\dots,s_m$ such that the minimum information that the sensors obtain about the targets is maximized while the maximum information that the targets obtain about the sensors is less than some prescribed level $\gamma^2$. We call $\gamma$ the \textit{information leakage level}. \end{problem} We can formulate \cref{problem 1} as the optimization problem \begin{align}\label{eq: problem def general} \underset{\theta,\, \beta}{\text{maximize}} \quad & \min_{1 \leq k \leq n} \mathcal{I}_k\\ \text{subject to:} \quad & \max_{1\leq i \leq m} \mathcal{J}_i \leq \gamma^2 \notag\\ & (\theta, \beta) \in \mathcal{F}, \notag \end{align} \hlgreen{where $\mathcal{F}$ is the set of all geometrically feasible $(\theta,\beta)$. This constraint ensures that the spatial arrangement of sensors and targets with angles $\theta$ and $\beta$ is realizable. We compute $\mathcal{F}$ for the special case $m=n=2$ in} \cref{prop:cases}. \begin{remark} Instead of constraining the maximum of $\mathcal{J}_i$ (the most information the targets have about any particular sensor), one could constrain the sum or the product of $\mathcal{J}_i$ or some other norm of the vector $(\mathcal{J}_1,\dots,\mathcal{J}_m)$. It is also possible to apply different norms to the expression of $\mathcal{I}_k$. \end{remark} In the next section, we solve \cref{problem 1} for the special case of $m=2$ sensors and $n=2$ targets. \section{TWO SENSORS AND TWO TARGETS}\label{sec: Problem formulation 2t2a} \hlcyan{Substituting {\eqref{eq: det FIM of target k}} into {\eqref{eq: problem def general}} with $m=n=2$ yields} \begin{subequations}\label{eq: problem def for 2s2t} \begin{align} \underset{\theta_1, \theta_2, \beta_1, \beta_2}{\text{maximize}} \quad & \min \bigl(\sin^2\theta_1, \, \sin^2\theta_2\bigr) \label{opt:obj}\\ \text{subject to:} \quad & \max\bigl(\sin^2\beta_1, \, \sin^2\beta_2\bigr) \leq \gamma^2 \label{opt:gamma}\\ & (\theta_1, \theta_2, \beta_1, \beta_2) \in \mathcal{F}, \label{opt:h} \end{align} \end{subequations} where we have used the simplified notation $\theta_k \defeq \theta_{12,k}$ and similarly for $\beta_k$. \hlorange{We also assumed without loss of generality that $\sigma = 1$.} We analyze \eqref{eq: problem def for 2s2t} in three steps: \paragraph{The objective \eqref{opt:obj}} In epigraph form, the level sets $\min\bigl(\sin^2\theta_1, \, \sin^2\theta_2\bigr) \geq \eta^2$ consist of configurations where $\sin\theta_k \geq \eta$ for $k=1,2$. In other words, \begin{equation}\label{eq:thetarange} \theta_k \in [ \arcsin\eta, \pi-\arcsin\eta ] \qquad\text{for }k=1,2. \end{equation} The set of points $t_k$ for which $\theta_k$ is fixed is the arc of a circle passing through $s_1$ and $s_2$. Therefore, given $\eta$, sublevel sets for all $\theta_k$ in \eqref{eq:thetarange} are the exclusive-OR between two congruent discs whose boundaries intersect at $s_1$ and $s_2$ (as shown in \cref{fig: level sets} Left). The objective is maximized at the value 1 when $\theta_1=\theta_2=\tfrac{\pi}{2}$. This corresponds to the configuration when both circles coincide with the dotted circle and $t_k$ lies on this circle. This is logical since localization error is minimized when the range vectors are orthogonal. \begin{figure}[ht] \centering \includegraphics[page=1]{fig_level_sets} \includegraphics[page=2]{fig_level_sets} \caption{\textbf{Left:} Shaded region that must contain $t_1$ and $t_2$ (relative to $s_1$ and $s_2$) so that the objective \eqref{opt:obj} is at least $\eta^2$. The region is formed by two intersecting circles ($\eta=0.7$ shown). The dotted circle shows $\eta=1$. \textbf{Right:} Shaded region that must contain $s_1$ and $s_2$ (relative to $t_1$ and $t_2$) so that information leakage level is at most $\gamma$ ($\gamma=0.7$ shown).} \label{fig: level sets} \end{figure} \paragraph{Stealthiness constraint \eqref{opt:gamma}} Similar to \eqref{eq:thetarange} we have \begin{equation}\label{eq:betarange} \beta_i \in [0,\arcsin\gamma] \cup [\pi-\arcsin\gamma, \pi] \quad\text{for }i=1,2. \end{equation} The set of admissible $\beta_i$ is shown in \cref{fig: level sets} Right. Intuitively, the targets have a large localization error for a sensor when the targets' range vectors are close to being parallel ($\beta_i$ is close to $0$ or to $\pi$). This splits the feasible set into two disjoint regions: \emph{between} $t_1$ and $t_2$ or \emph{outside}. \paragraph{Feasible configurations \eqref{opt:h}} Considering a quadrilateral whose vertices are formed by the sensors and the targets, there are several cases to consider; depending on whether any interior angles are greater than $\pi$ or whether the quadrilateral is self-intersecting. By inspection we have seven distinctive cases, illustrated in \cref{fig: seven cases}. Each case corresponds to a set of constraints given next in \cref{prop:cases}. \begin{figure*} \vspace{2mm} \centering \includegraphics[page=1]{fig_seven_cases}\hfill \includegraphics[page=2]{fig_seven_cases}\hfill \includegraphics[page=3]{fig_seven_cases}\hfill \includegraphics[page=4]{fig_seven_cases}\hfill \includegraphics[page=5]{fig_seven_cases}\hfill \includegraphics[page=6]{fig_seven_cases}\hfill \includegraphics[page=7]{fig_seven_cases} \caption{ The seven possible configurations for two sensors and two targets. Each case is characterized by different constraints relating $\theta_1$, $\theta_2$, $\beta_1$ and $\beta_2$ which are given in \cref{prop:cases}. If sensors $s_1$ and $s_2$ are interchangeable then $\mathcal{C}_4 = \mathcal{C}_5$ and $\mathcal{C}_6 = \mathcal{C}_7$ and there are only five distinct cases. } \vspace{-2mm} \label{fig: seven cases} \end{figure*} \begin{proposition}\label{prop:cases} The feasible configurations $\mathcal{F}$ in \eqref{opt:h} is the union of the seven cases shown in \cref{fig: seven cases}. In other words, \begin{equation*} \mathcal{F} = \biggl\{(\theta_1, \theta_2, \beta_1, \beta_2) \in [0,\pi]^4 \;\bigg|\; \bigcup_{i=1}^7 \mathcal{C}_i\biggr\}, \end{equation*} where $\mathcal{C}_i$ for $i=1,\dots,7$ is the constraint set corresponding to the $i\textsuperscript{th}$ case shown in \cref{fig: seven cases} and expressed below: \begin{subequations} \begin{align*} \mathcal{C}_1 &: &&\theta_1 + \theta_2 + \beta_1 + \beta_2 = 2\pi\\ \mathcal{C}_2 &: &-&\theta_1 + \theta_2 + \beta_1 + \beta_2 = 0,\;\; \theta_2 \leq \theta_1\\ \mathcal{C}_3 &: &&\theta_1 - \theta_2 + \beta_1 + \beta_2 = 0,\;\; \theta_1 \leq \theta_2\\ \mathcal{C}_4 &: &&\theta_1 + \theta_2 - \beta_1 + \beta_2 = 0,\;\; \beta_2 \leq \beta_1\\ \mathcal{C}_5 &: &&\theta_1 + \theta_2 + \beta_1 - \beta_2 = 0,\;\; \beta_1 \leq \beta_2\\ \mathcal{C}_6 &: &&\theta_1 - \theta_2 + \beta_1 - \beta_2 = 0,\;\; \theta_1 + \beta_1 \leq \pi \\ \mathcal{C}_7 &: &&\theta_1 - \theta_2 - \beta_1 + \beta_2 = 0,\;\; \theta_1 + \beta_2 \leq \pi \end{align*} \end{subequations} If the sensors $s_1$ and $s_2$ are interchangeable (e.g., if there are no additional constraints that distinguish $s_1$ from $s_2$) then swapping $\beta_1$ and $\beta_2$ leads to $\mathcal{C}_4 = \mathcal{C}_5$ and $\mathcal{C}_6 = \mathcal{C}_7$. \end{proposition} \medskip The following result is useful in the sequel and is a straightforward consequence of the constraint equations $\mathcal{C}_i$. \medskip \begin{lemma}\label{lem:C1} Suppose $(\theta_1, \theta_2, \beta_1, \beta_2) \in \mathcal{F}$, where $\mathcal{F}$ is defined in \cref{prop:cases}. Then $\theta_1 + \theta_2 + \beta_1 + \beta_2 \leq 2\pi$, where equality is achievable in a non-degenerate configuration (no sensor is placed arbitrarily close to a target) if and only if $(\theta_1, \theta_2, \beta_1, \beta_2) \in \mathcal{C}_1$. \end{lemma} \medskip We provide the analytical results in two theorems. Our first theorem considers the unconstrained case where the sensors may be placed anywhere in relation to the targets. \begin{theorem}\label{theorem 1} Consider the optimization problem \eqref{eq: problem def for 2s2t}. If the sensors $s_1$ and $s_2$ can be freely placed anywhere then a configuration of sensors is globally optimal if and only if: \begin{enumerate}[(i)] \item $s_1$, $s_2$, $t_1$, $t_2$ are \emph{cyclic} (they lie on a common circle), \item $s_1$ and $s_2$ are diametrically opposed on this circle, and \item The common circle has diameter at least $d/\gamma$; where $d$ is the distance between $t_1$ and $t_2$. \end{enumerate} Moreover, any such configuration satisfies $\theta_1 = \theta_2 = \frac{\pi}{2}$ and $\sin\beta_1 = \sin\beta_2$ and has an optimal objective value of $1$. \end{theorem} \begin{proof} The objective \eqref{opt:obj} is upper-bounded by $1$, which is achieved if and only if $\theta_1=\theta_2=\frac{\pi}{2}$. This is possible if and only if $s_1$, $s_2$, $t_1$ and $t_2$ are on a common circle with diameter $\abs{s_1s_2}$. If $s_1$ and $s_2$ lie on alternating sides of $t_1$ and $t_2$, we recover $\mathcal{C}_1$ from \cref{fig: seven cases}. Otherwise, we recover cases $\mathcal{C}_6$ or $\mathcal{C}_7$. Given such a configuration where the common circle has diameter $D$ apply the Law of Sines to $\triangle t_1 t_2 s_i$ and obtain $d/\sin\beta_i = D$. Now \eqref{opt:gamma} is equivalent to $\sin\beta_i \leq \gamma$ and therefore $D \geq d/\gamma$. \end{proof} Examples of optimal configurations proved in Theorem \ref{theorem 1} for the cases $\mathcal{C}_1$ and $\mathcal{C}_6$ or $\mathcal{C}_7$ are illustrated in \cref{fig: 2t2s Theorem1}. \begin{figure}[ht] \centering \includegraphics[page=1]{fig_thm1} \includegraphics[page=2]{fig_thm1} \caption{Examples of optimal sensor configurations when positions are unconstrained (\cref{theorem 1}). The solid circles delineate the feasible set (see \cref{fig: level sets}). The dotted circle is any larger circle passing through $t_1$ and $t_2$. A configuration is optimal if and only if the sensors $s_1$ and $s_2$ lie on this larger circle and are diametrically opposed. This can happen with alternating sensors and targets (left) or with both sensors on the same side (right).} \label{fig: 2t2s Theorem1} \end{figure} Recall that the feasible set is split into two disjoint regions (see \cref{fig: level sets} Right). \cref{theorem 1} confirms that optimal configurations exist with one sensor between $t_1$ and $t_2$ and one outside or with both sensors outside (see \cref{fig: 2t2s Theorem1}). We next investigate the scenarios where \emph{both} sensors are between $t_1$ and $t_2$; that is $\beta_i \in [\pi-\arcsin\gamma,\pi]$ for $i=1,2$. \begin{theorem}\label{theorem 2} Consider \eqref{eq: problem def for 2s2t} with the extra constraint $\beta_i \in [\pi-\arcsin\gamma,\pi]$ for $i=1,2$. Then, a non-degenerate configuration of sensors is globally optimal if and only if: \begin{enumerate}[(i)] \item $s_1$, $t_1$, $s_2$ and $t_2$ are vertices of a parallelogram, and \item $s_1$ and $s_2$ are on different circles through $t_1$ and $t_2$ with diameter $d/\gamma$; where $d$ is the distance between $t_1$, $t_2$. \end{enumerate} Any such configuration satisfies $\theta_1=\theta_2 = \arcsin\gamma$ and $\beta_1=\beta_2 = \pi-\arcsin\gamma$ and has an optimal objective value of $\gamma^2$. \end{theorem} \begin{proof} We prove sufficiency first. Incorporating the constraints on $\beta_1$ and $\beta_2$ and using the fact that $\sin(\cdot)$ is nonnegative on $[0,\pi]$, we can rewrite \eqref{eq: problem def for 2s2t} as \begin{align}\label{eq: problem def for 2s2t simplified} \underset{\theta_1, \theta_2, \beta_1, \beta_2}{\text{maximize}} \quad & \min (\sin\theta_1, \, \sin\theta_2)\\ \textrm{subject to:} \quad &\pi-\arcsin\gamma \leq \beta_i \leq \pi & i&\in\{1,2\}\notag\\ & 0 \leq \theta_k \leq \pi & k&\in\{1,2\}\notag\\ & (\theta_1, \theta_2, \beta_1, \beta_2) \in \mathcal{F}.\notag \end{align} By concavity of $\sin(\cdot)$ on $[0,\pi]$ and Jensen's inequality, \begin{equation}\label{ineq1} \min(\sin\theta_1, \, \sin\theta_2) \leq \frac{\sin\theta_1 \!+\! \sin\theta_2}{2} \leq \sin\biggl( \frac{\theta_1+\theta_2}{2} \biggr). \end{equation} From \cref{lem:C1} we have $\theta_1 + \theta_2 + \beta_1 + \beta_2 \leq 2\pi$ with equality only in case $\mathcal{C}_1$. Therefore, \begin{equation}\label{ineq2} 0 \leq \frac{\theta_1+\theta_2}{2} \leq \frac{2\pi-\beta_1-\beta_2}{2} \leq \arcsin\gamma \leq \frac{\pi}{2}. \end{equation} Since $\sin(\cdot)$ is monotonically increasing on $[0,\tfrac{\pi}{2}]$ we may combine \eqref{ineq1} and \eqref{ineq2} to upper-bound the objective of \eqref{eq: problem def for 2s2t simplified} \[ \min(\sin\theta_1, \, \sin\theta_2) \leq \sin\biggl( \frac{\theta_1+\theta_2}{2} \biggr) \leq \gamma, \] with equality only possible for case $\mathcal{C}_1$. Indeed setting $\theta_1=\theta_2=\arcsin\gamma$ and $\beta_1=\beta_2=\pi-\arcsin\gamma$ renders the bound tight and satisfies the constraint for case $\mathcal{C}_1$ in \cref{prop:cases} and is therefore optimal. This solution is illustrated in \cref{fig: 2t2s Theorem2}. \begin{figure}[ht] \centering \includegraphics{fig_thm2} \caption{Example of an optimal sensor configuration when sensors are constrained to lie in the region between both targets (\cref{theorem 2}). Solid circles delineate the feasible set (see \cref{fig: level sets}). A configuration is optimal if $s_1$ and $s_2$ lie on opposite arcs and $t_1$, $s_1$, $t_2$ and $s_2$ form a parallelogram.} \label{fig: 2t2s Theorem2} \end{figure} Necessity follows from convexity of \eqref{eq: problem def for 2s2t simplified}. Indeed, from the derivation above, any optimal non-degenerate configuration must be of case $\mathcal{C}_1$ (see \cref{fig: seven cases}), so constraint $\mathcal{C}_1$ holds (all constraints are linear). Moreover, the objective of \eqref{eq: problem def for 2s2t simplified} is the minimum of two concave functions, so it is concave. \end{proof} \begin{remark} \hlcyan{Letting $s_1$ and $s_2$ approach $t_1$ and $t_2$, respectively, yields in the limit $\beta_1 = \beta_2 = \pi - \arcsin\gamma$ and $\theta_1 = \theta_2 = \arcsin\gamma$, which is a globally optimal (but degenerate) configuration of cases $\mathcal{C}_1$, $\mathcal{C}_6$, or $\mathcal{C}_7$.} \end{remark} \section{ARBITRARILY MANY SENSORS}\label{sec:Extension} In this section we extend our results to the case with $n=2$ targets and \hlorange{$m\geq 3$} sensors. When the sensors can be freely placed (as considered in \cref{theorem 1}) a globally optimal solution is to place the sensors infinitely far from the targets in a circular arrangement. Under these conditions the stealthiness constraint is automatically satisfied and the problem reduces to optimal sensor placement with a single target \cite{moreno2013optimal, sadeghi2020optimal}. \begin{figure}[ht] \vspace{1mm} \centering \includegraphics{fig_ms2t} \caption{The problem with $n=2$ targets and $m \geq 3$ sensors, where sensors are constrained to be inside the green region ($\gamma = 0.95$ shown). Sensor $s_i$'s position is characterized by heading angles $\alpha_{i,1}$ and $\alpha_{i,2}$ from targets $t_1$ and $t_2$, respectively. Sensors have positive heading angles when right of the center-line and negative when left.} \vspace{-1mm} \label{fig: ms2t} \end{figure} The more practical, yet challenging, scenario is when the sensors are constrained to lie in the region between $t_1$ and $t_2$ (as considered in \cref{theorem 2}). It is convenient to re-parameterize this problem in terms of the \emph{heading angles} $\alpha_{i,k}$ of sensor $s_i$ viewed from target $t_k$ and measured relative to the center-line joining $t_1$ and $t_2$ (as illustrated in \cref{fig: ms2t}). By convention $\alpha_{i,k}>0$ if $s_i$ is to the right of the center-line and $\alpha_{i,k}<0$ if $s_i$ is to the left. Thus we may set $\theta_{ij,k} = |\alpha_{i,k}-\alpha_{j,k}|$ and $\beta_{i} = \pi-|\alpha_{i,1}+\alpha_{i,2}|$ and using \eqref{eq:betarange}, write \eqref{eq: problem def general} in epigraph form as \begin{align}\label{eq: problem def for ms2t} \underset{\alpha, \eta^2}{\text{maximize}} \quad & \eta^2\\ \textrm{subject to:} \quad & \mkern-18mu\sum_{1 \leq i < j \leq m}\mkern-18mu\sin^2(\alpha_{i,k}-\alpha_{j,k}) \geq \eta^2 & k&\in\{1,2\}\notag\\ & \abs{\alpha_{i, 1} + \alpha_{i, 2}} \leq \arcsin \gamma & i&\in\{1,\dots,m\}\notag\\ & \alpha_{i,1}\alpha_{i,2} \geq 0 & i&\in\{1,\dots,m\}.\notag \end{align} The final constraint ensures that $\alpha_{i,1}$ and $\alpha_{i,2}$ have the same sign, so that the $i\textsuperscript{th}$ rays emanating from $t_1$ and $t_2$ are guaranteed to intersect at $s_i$. \cref{eq: problem def for ms2t} is non-convex due to the first and last constraints. A closed-form solution remains elusive, so in the sections that follow, we turn our attention to deriving analytic upper and lower bounds to the optimal $\eta^2$. \subsection{Analytic lower bounds}\label{subsec: lower bound analytical} For the maximization problem \eqref{eq: problem def for ms2t}, any feasible configuration of sensors yields a lower bound on the optimal objective value. Based on the results in \cref{sec: Problem formulation 2t2a}, we focus on two important configurations with associated lower bound \begin{equation}\label{eq: lower bound symmetry} \eta^2 \; = \sum_{1 \leq i < j \leq m}\mkern-18mu\sin^2(\alpha_{i,k}-\alpha_{j,k})\quad\text{for }k\in\{1,2\}. \end{equation} \paragraph{Degenerate configuration}\label{section:degenerate_configuration} Refer to \cref{fig: dgnrt and unfrm} Left and consider the two arcs that form the boundaries of the green region. First, place two sensors on the left arc arbitrarily close to $t_1$ and $t_2$, respectively ($s_1$ and $s_2$). Then similarly place two sensors on the right arc ($s_3$ and $s_4$). Continue placing two sensors at a time on alternating arcs until all sensors are placed. If there is an odd number of sensors then place the last sensor in the center of the arc ($s_5$). Since the sensors are either arbitrarily close to the targets or in the middle of one of the arcs, the angles $\alpha_{i, k}$ are equal to $\pm\psi$ or $\pm\phi$ (see \cref{fig: dgnrt and unfrm}, Left). By straightforward calculations we obtain $\psi = 2\phi = \arcsin\gamma$. As a result, using \eqref{eq: lower bound symmetry}, we can compute the lower bound analytically \begin{equation}\label{eq: analytical lb degenerate} \eta^2_\textup{opt} \geq \left\{ \begin{aligned} &\tfrac{1}{4} m^2 \gamma^2 (2-\gamma^2) \\ &\tfrac{1}{4} (m\!-\!1) \bigl(2-2 (1\!-\!\gamma ^2)^{3/2}+(m\!-\!1)\gamma ^2 (2\!-\!\gamma ^2)\bigr) \\ &\tfrac{1}{4} m^2 \gamma ^2 (2-\gamma ^2) - \gamma ^2 + \gamma^4 \\ &\tfrac{1}{4} (m\!-\!1) \bigl(2-2 (1\!-\!\gamma ^2)^{3/2}+(m\!-\!1)\gamma ^2 (2\!-\!\gamma ^2)\bigr) \\ &\hspace{3.2cm} -\gamma^2+\gamma^4+\gamma ^2\sqrt{1-\gamma ^2}\hspace{-0.3cm} \end{aligned} \right. \end{equation} \noindent where the four lower bounds in \eqref{eq: analytical lb degenerate} correspond to the cases $m\equiv 0,1,2,3 \pmod{4}$, respectively. \begin{figure}[t] \centering \includegraphics[page=1]{fig_dgnrt_unfrm} \includegraphics[page=2]{fig_dgnrt_unfrm} \caption{Two configurations used to lower-bound \eqref{eq: problem def for ms2t}. \textbf{Left:} \emph{Degenerate} configuration. Sensors are placed on the boundary of the green region (arbitrarily close to the targets) forming an angle $\psi$ with the center line. When $m$ is odd the last sensor is placed in the middle of the arc with fewest sensors on it; forming an angle $\phi = \frac{1}{2}\psi$ with the center line. \textbf{Right:} \emph{Uniform} configuration. Sensors are placed such that the lengths of the arcs between neighboring sensors (shown in brown) are equal.} \label{fig: dgnrt and unfrm} \end{figure} \paragraph{Uniform configuration} In this configuration, the sensors are placed on the boundaries of the green region such that the arc length between neighboring sensors is equal to $2\delta$ (see \cref{fig: dgnrt and unfrm} Right). Straightforward computation yields $\delta = \frac{2}{m}\arcsin\gamma$. It can be seen from \cref{fig: dgnrt and unfrm} (Right) that the $(\alpha_{i,k}-\alpha_{j,k})$ are multiples of $\delta$. Direct calculation gives \begin{multline}\label{eq: analytical lb uniform} \eta_\textup{opt}^2 \geq \sum_{1 \leq i < j \leq m}\mkern-18mu\sin^2(\alpha_{i,1}-\alpha_{j,1}) = \mkern-18mu\sum_{1 \leq k \leq m-1}\mkern-18mu(m-k)\sin^2(k\delta)\\ = \frac{1}{8\sin^2\delta}\left( m^2 - 1 + \cos(2m\delta) - m^2\cos(2\delta) \right), \end{multline} \cref{eq: analytical lb uniform} together with $\delta = \frac{2}{m}\arcsin\gamma$ gives the second analytical lower bound to \eqref{eq: problem def for ms2t}. \subsection{Analytic upper bound}\label{subsec: upper bound analytical} We provide two upper-bounds for \eqref{eq: problem def for ms2t}. \paragraph{Constraint approach} \hlorange{Let $\mathcal{S}$ and $\mathcal{D}$ denote the pairs of indices corresponding to agents on the \emph{same} side and \emph{different} side of the centerline between both targets.} \hlorange{Since $|\alpha_{i,k}|\leq\arcsin\gamma$ we can upper-bound $|\alpha_{i,k}-\alpha_{j,k}|$ based on whether $(i,j)$ are on the same side (in $\mathcal{S}$) or on different sides (in $\mathcal{D}$).} As a result, we have for $k\in\{1,2\}$ \begin{equation*} \sin^2(\alpha_{i,k}-\alpha_{j,k}) \leq \begin{cases} \gamma^2 & (i,j) \in \mathcal{S}\\ 4\gamma^2(1-\gamma^2) & (i,j) \in \mathcal{D},\;\gamma \leq \frac{1}{\sqrt{2}} \\ 1 & (i,j) \in \mathcal{D},\;\gamma > \frac{1}{\sqrt{2}}. \end{cases} \end{equation*} Assuming we have $p$ agents on the right side and $m-p$ on the left, we may apply the bound above and obtain: \begin{align}\label{eq: upper bound p dependent} \eta^2 &= \sum_{1\leq i < j \leq m} \mkern-18mu\sin^2(\alpha_{i,k}-\alpha_{j,k}) \leq \biggl(\binom{p}{2} + \binom{m-p}{2}\biggr)\gamma^2 \notag\\ &\hspace{1cm} + m(m-p) \begin{cases} 4\gamma^2(1-\gamma^2) & \gamma \leq \frac{1}{\sqrt{2}} \\ 1 & \gamma > \frac{1}{\sqrt{2}}. \end{cases} \end{align} Maximizing the right-hand side of \eqref{eq: upper bound p dependent} over $p$ gives $p^\star = \frac{m}{2}$ for even and $p^\star = \frac{m-1}{2}$ for odd values of $m$ and \begin{equation}\label{eq: upper bound first method} \eta^2_{\text{opt}} \leq \begin{cases} \frac{1}{4}m\gamma^2\bigl(m(5 -4\gamma^2) - 2\bigr) & \gamma \leq \frac{1}{\sqrt{2}}\\ \frac{1}{4}m\bigl(m - 2\gamma^2 + m \gamma^2 \bigr) & \gamma > \frac{1}{\sqrt{2}}. \end{cases} \end{equation} \cref{eq: upper bound first method} is an analytical upper bound to \eqref{eq: problem def for ms2t}. \paragraph{Jensen approach} Let $g(\theta)$ be a \hlorange{\emph{concave non-decreasing}} upper bound $\sin^2\theta \leq g(\theta)$. Applying Jensen's inequality to \eqref{eq: lower bound symmetry}, we obtain for $k\in\{1,2\}$ \begin{align}\label{eta2 upper bound jensen} \eta^2 &= \sum_{1 \leq i < j \leq m}\mkern-10mu\sin^2(\alpha_{i,k}-\alpha_{j,k}) \notag \\ &\leq \binom{m}{2}g\Biggl(\binom{m}{2}^{-1}\mkern-18mu\sum_{1 \leq i < j \leq m}\mkern-10mu (\alpha_{i,k}-\alpha_{j,k})\Biggr). \end{align} The upper bound is determined by choosing $\alpha_{i,k}$ in order to maximize the minimum of the right-hand side of \eqref{eta2 upper bound jensen} for $k\in\{1,2\}$. We next sketch the derivation. Start by assuming without loss of generality that $\alpha_{1, 1} \geq \dots \geq \alpha_{m, 1}$, then \begin{equation}\label{eq: sum theta 1} \sum_{1 \leq i < j \leq m} \mkern-18mu (\alpha_{i,1}-\alpha_{j,1}) = \sum_{i=1}^m (m - 2i + 1)\alpha_{i, 1}. \end{equation} The steps involve showing that: (1) half of the $\alpha_{i,1}$ must be nonegative and the other half nonpositive, (2) The second constraint of \eqref{eq: problem def for ms2t} is tight leading to $\alpha_{i,2}+\alpha_{i,1} = \pm\arcsin\gamma$ and $\alpha_{1,2}\leq \cdots \leq \alpha_{m,2}$, and (3) The positive-coefficient and negative-coefficient terms of \eqref{eq: sum theta 1} can be separately upper-bounded using $\min(a,b) \leq \tfrac{1}{2}(a+b)$, which has a trivial solution of letting half the $\alpha_{i,k}$ equal to 0 and the other half equal to $\pm \arcsin\gamma$. This solution leads to \begin{equation}\label{eq: upper bound second method} \eta^2_{\text{opt}} \leq \begin{cases} \binom{m}{2}g\Bigl( \frac{3m}{4(m-1)}\arcsin\gamma \Bigr) & m\text{ even}\\ \binom{m}{2}g\Bigl( \frac{3(m+1)}{4m}\arcsin\gamma \Bigr) & m\text{ odd} \end{cases} \end{equation} \cref{eq: upper bound second method} is the second analytical upper bound to \eqref{eq: problem def for ms2t}. For our simulations, we used a tight concave upper-bound: \[ g(\theta) = \begin{cases} 0.724611\cdot \theta & 0\leq \theta\leq 1.16556 \\ \sin^2(\theta) & 1.16556 < \theta < \frac{\pi}{2} \\ 1 & \tfrac{\pi}{2} \leq \theta \leq \pi. \end{cases} \] \subsection{Combining the bounds}\label{sec: results} In \cref{fig: sim results} we plot the lower bounds of \eqref{eq: analytical lb degenerate} and \eqref{eq: analytical lb uniform} and upper bounds of \eqref{eq: upper bound first method} and \eqref{eq: upper bound second method} normalized by $m^2$ for $m=3,6,9$ and $m\to\infty$. Different bounds are tighter for different values of $\gamma$ and $m$. The gap between our best upper and lower bounds (shaded region) is quite small for all $m$. We also solved \eqref{eq: problem def for ms2t} directly with a local optimizer\footnote{\hlyellow{We used MATLAB's {\footnotesize\tt fmincon} local optimizer with the default {\footnotesize\tt interior-point} algorithm and initialized the decision variables with $\alpha_{i,k} \sim \mathrm{Uniform}[-\arcsin\gamma,\arcsin\gamma]$ and $\eta=0$ for each $\gamma,m,i,k$. Using the algorithm {\footnotesize\tt trust-region-reflective} yielded similar results, but {\footnotesize\tt sqp} did not work at all.}}. As seen in \cref{fig: sim results}, the optimizer is often trapped in local minima, leading to solutions that are worse than our lower bounds. Moreover, the optimizer never beats our lower bound, suggesting that our lower bound may be globally optimal. \begin{figure}[!ht] \centering \includegraphics{Figures/fig_bounds.pdf} \caption{Normalized upper and lower bounds provided in \cref{subsec: lower bound analytical} and \cref{subsec: upper bound analytical} for $m=3,6,9$ and $m\to\infty$. In the first three simulations, the exact problem in \eqref{eq: problem def for ms2t} has been solved using {\footnotesize\tt fmincon} in MATLAB. The shaded region indicates the gap between best upper and lower bounds.} \label{fig: sim results} \end{figure} \section{CONCLUSION AND FUTURE RESEARCH}\label{sec: Conclusion and Future Work} We considered the problem of stealthy optimal sensor placement, where a network of range sensors attempts to maximize the localization information obtained about a set of targets, while limiting the information revealed to the targets (which also have range sensors). We provided analytical solutions for $m=2$ sensors and $n=2$ targets. We also found upper and lower bounds for $m\geq 3$ and $n=2$. One direction for future research is to explore different sensor models such as bearing-only sensors or sensors with range-dependent noise. \hlcyan{Another area of exploration is the case of $n\geq 3$ targets.} In \cref{fig: feasible 4 targets}, we show the region satisfying a stealthiness constraint for a particular arrangement of $n=4$ targets. This is far more complex than the $n=2$ case (see \cref{fig: level sets}). Small changes in $\gamma$ or the target locations can cause dramatic changes to the topology and connectedness of the feasible set.\looseness=-1 \begin{figure}[!ht] \centering \includegraphics{Figures/fig_4target_feasible_set.pdf} \caption{Feasible set corresponding to $\sum_{1\leq k < \ell \leq 4}\sin^2{\beta_{k\ell}} \leq \gamma^2$ for different $\gamma$ and $n=4$ targets. Compared to the $n=2$ case (see \cref{fig: level sets}), the feasible set is complicated; small changes in $\gamma$ cause dramatic changes in shape and connectedness.} \label{fig: feasible 4 targets} \end{figure} \section{ACKNOWLEDGMENTS} The authors wish to thank Dr. Christopher M. Kroninger (DEVCOM ARL) for support and insights on this topic. \bibliographystyle{IEEEtran} \bibliography{references} \end{document}
2412.04389v1
http://arxiv.org/abs/2412.04389v1
Hypergraph burning, matchings, and zero forcing
\documentclass[12pt]{amsart} \usepackage{graphicx} \usepackage{amsmath,amsthm,amssymb,epsfig} \usepackage[table]{xcolor} \usepackage{url} \usepackage{tikz} \usepackage{float} \usepackage{mathtools} \usepackage{subcaption} \usepackage[linesnumbered,lined,boxed,commentsnumbered]{algorithm2e} \usepackage{MnSymbol} \usepackage{xcolor} \usepackage{comment} \usepackage{tkz-graph} \usetikzlibrary{fit} \usetikzlibrary{shapes} \usepackage{enumitem} \usepackage[left=3.5cm,top=3.5cm,right=3.5cm,bottom=3.5cm]{geometry} \usepackage{mathrsfs} \newcommand{\calB}{\mathcal{B}} \newcommand{\calN}{\mathcal{N}} \newcommand{\calS}{\mathcal{S}} \newcommand{\IG}{\mathrm{IG}} \newcommand{\core}{\mathrm{core}} \newcommand{\Ind}[1]{\langle #1 \rangle} \newtheorem{theorem}{Theorem} \newtheorem{corollary}[theorem]{Corollary} \newtheorem{lemma}[theorem]{Lemma} \newtheorem{question}{Question} \title{Hypergraph burning, matchings, and zero forcing} \author[A.\ Bonato]{Anthony Bonato} \author[C.\ Jones]{Caleb Jones} \author[T.G.\ Marbach]{Trent G.\ Marbach} \author[T.\ Mishura]{Teddy Mishura} \author[Z.\ Zhang]{Zhiyuan Zhang} \address[A1,A2,A3,A4,A5]{Toronto Metropolitan University, Toronto, Canada} \email[A1]{(A1) [email protected]} \email[A2]{(A2) [email protected]} \email[A3]{(A3) [email protected]} \email[A4]{(A4) [email protected]} \email[A5]{(A5) [email protected]} \keywords{hypergraphs, burning, matchings, zero forcing} \subjclass{05C65,05C70,05C85} \begin{document} \begin{abstract} Lazy burning is a recently introduced variation of burning where only one set of vertices is chosen to burn in the first round. In hypergraphs, lazy burning spreads when all but one vertex in a hyperedge is burned. The lazy burning number is the minimum number of initially burned vertices that eventually burns all vertices. We give several equivalent characterizations of lazy burning on hypergraphs using matchings and zero forcing, and then apply these to give new bounds and complexity results. We prove that the lazy burning number of a hypergraph $H$ equals its order minus the maximum cardinality of a certain matching on its incidence graph. Using this characterization, we give a formula for the lazy burning number of a dual hypergraph and give new bounds on the lazy burning number based on various hypergraph parameters. We show that the lazy burning number of a hypergraph may be characterized by a maximal subhypergraph that results from iteratively deleting vertices in singleton hyperedges. We prove that lazy burning on a hypergraph is equivalent to zero forcing on its incidence graph and show an equivalence between skew zero forcing on a graph and lazy burning on its neighborhood hypergraph. As a result, we show that finding an upper bound on the lazy burning number of a hypergraph is NP-complete, which resolves a conjecture from \cite{BJR}. By applying lazy burning, we show that computing an upper bound on the skew zero forcing number for bipartite graphs is NP-complete. We finish with open problems. \end{abstract} \maketitle \section{Introduction} Graph burning is a simplified model for the spread of influence in a network. Associated with the process is a parameter introduced in \cite{BJR,BJR1}, the burning number, which quantifies the speed at which the influence spreads to every vertex. Given a graph $G$, the burning process on $G$ is a discrete-time process. At the beginning of the first round, all vertices are unburned. In each round, first, all unburned vertices that have a burned neighbor become burned, and then one new unburned vertex is chosen to burn if such a vertex is available. If at the end of round $k$ every vertex of $G$ is burned, then $G$ is $k$-\emph{burnable}. The \emph{burning number} of $G,$ written $b(G),$ is the least $k$ such that $G$ is $k$-burnable. For further background on graph burning, see the survey \cite{survey} and the book \cite{bbook}. \emph{Hypergraph burning} was introduced in \cite{BJP,JT} as a natural variant of graph burning. The rules for hypergraph burning are identical to those of burning graphs, except for how burning propagates within a hyperedge. Recall that in a hypergraph, a \emph{singleton} edge contains exactly one vertex. In hypergraphs, the burning spreads to a vertex $v$ in round $r$ if and only if there is a non-singleton hyperedge $\{v,u_1,\ldots,u_k\}$ such that $v$ was not burned and each of $u_1,u_2,\ldots,u_k$ was burned at the end of round $r-1$. In particular, a vertex becomes burned if it is the only unburned vertex in a hyperedge. A natural variation of burning that is our principal focus here is \emph{lazy hypergraph burning}, where a set of vertices is chosen to burn in the first round only; no other vertices are chosen to burn in later rounds. The \textit{lazy burning number} of $H$, denoted $b_L(H)$, is the minimum cardinality of a set of vertices burned in the first round that eventually burn all vertices. Note that for a hypergraph $H,$ we have that $b_L(H) \le b(H).$ We refer to the set of vertices chosen to burn in the first round as a \emph{lazy burning set}. A lazy burning set $S$ such that $|S|=b_L(H)$ is called \emph{optimal}. If a vertex is burned and not part of a lazy burning set, then we say it is burned by \emph{propagation}; all rounds other than the first where a lazy burning set is chosen are called \emph{propagation rounds}. We note that we consider a slightly modified rule for lazy burning than the one given in \cite{BJP}, which simplifies our discussion. If a hyperedge $h$ contains exactly one unburned vertex $v$, then $v$ burns. The only difference between this rule and the original one is that now singleton hyperedges \emph{spontaneously burn} in the sense that they cause the vertices they contain to burn immediately. Note that this cannot increase the lazy burning number. Further, no optimal lazy burning set will contain a vertex belonging to a singleton edge, and various bounds (such as $|V(H)|-|E(H)|\leq b_L(H)$ from \cite{BJP}) remain unchanged with only minor variations in their proof. In the \emph{zero forcing} coloring process, a vertex subset of a graph $G$ is initially chosen to be black, while the remaining vertices are white (some authors alternatively refer to the colors as blue and white, respectively). Consider the following \emph{zero forcing color change rule}: a black vertex $u$ can change the color of a white vertex $v$ if $v$ is the only white neighbor of $u$; we say that $u$ \emph{forces} $v$ or that $v$ is \emph{forced}, and write $u \rightarrow v.$ A \emph{zero forcing set} of $G$ is a set $Z \subseteq V(G)$ of vertices such that if the vertices of $Z$ are black and the rest are white, then every vertex eventually becomes black after repeated applications of the zero forcing color change rule. The minimum cardinality of a zero forcing set for a graph is known as the \textit{zero forcing number} $z(G)$ of the graph $G$. The {\em skew zero forcing color change rule} requires that if a vertex $u$ has exactly one white neighbor $v$, then $u$ forces $v$. The only difference between the two processes is that in skew zero forcing, a vertex does not need to be black to force one of its neighbors. A {\em skew zero forcing set} and the {\em skew zero forcing number $z_0(G)$} are defined analogously. As shown in \cite{IMA}, we have that $z_0(G)\leq z(G)$. For more background on these coloring processes, see \cite{AIM, FHsurvey, hlsbook, IMA}. The paper is organized as follows. In Section~2, we define chronological lists of lazy burning on a hypergraph, which leads to the notion of a $C$-matching. In Corollary~\ref{cor:Burning_equals_CMatching}, we prove that $b_L(H)=|V(H)|-m(H)$, where $m(H)$ is the maximum cardinality of a $C$-matching on the incidence graph of $H$. This provides a formula for the lazy burning number of the dual hypergraph in Corollary~\ref{cor1} and provides various new lower and upper bounds on the lazy burning number. For example, Corollary~\ref{cor:delta-delta} gives that in a linear hypergraph with minimum degree $\delta$ with at least $\delta$ hyperedges, $b_L(H) \geq |V(H)| -|E(H)| + \binom{\delta}{2}$, while Theorem~\ref{upp} shows that $b_L(H) \leq |V(H)| - \left\lceil \frac{|E(H)|}{\Delta}\right\rceil,$ where $\Delta$ is the maximum vertex degree. In Section~3, we characterize the lazy burning number of a hypergraph using its core, which is the hypergraph that results by iteratively deleting vertices in singleton hyperedges; see Theorem~\ref{thm: empty core}. Section~4 focuses on connections with zero forcing, and Theorem~\ref{zero_set_equivalence} shows that lazy burning is equivalent to zero forcing on incidence graphs. We prove in Theorem~\ref{skew_forcing_equals_lazy_burning} that skew zero forcing on a graph is equivalent to lazy burning on its neighborhood hypergraph. Using this result, we prove that computing an upper bound on the lazy burning number of a hypergraph is NP-complete, which answers Problem 11 from \cite{BJP}. In Theorem~\ref{szf on bpt is npc}, we find applications of lazy burning to skew zero forcing and show the skew zero forcing problem on bipartite graphs is NP-complete. The final section lists open problems. For more background on hypergraphs, see \cite{berge1,berge2,bretto}, and for more background on graph theory, see~\cite{west}. \section{Chronological Lists and $C$-Matchings} A concept gaining in popularity in the literature is a \emph{chronological list of zero forcing}. Given an initial zero forcing set $S_0$, a list of forces $(u_1 \rightarrow v_1, u_2 \rightarrow v_2, \ldots, u_k \rightarrow v_k)$ is defined so that the list of sets $S_0, S_1, S_2, \ldots, S_k$ with $S_{i+1} = S_i \cup \{v_{i+1}\}$ satisfies $N[u_i]\setminus\{v_i\} \subseteq S_{i-1}$. Intuitively, if the set $S_{i-1}$ is a subset of the black vertices at any time during the zero forcing process, then the vertex $u_i$ will force the vertex $v_i$ in the next round if $v_i$ is not already black. It follows that $S_i$ will be a subset of the black vertices at some time in the process. We note that we may have two vertices $v_i$ and $v_j$, with $ v_i$ occurring earlier than $v_j$ in the chronological list, but where vertex $v_j$ would be forced by some black vertex in the zero forcing process in an earlier round than $v_i$. A chronological list of skew zero forces may be defined analogously. If the context is clear, then we abbreviate the lists of either type as a chronological list of forces. We illustrate chronological lists in Figure~\ref{fig:ZF_on_C}, where two adjacent black vertices, $a$ and $b$, are initially chosen as our zero forcing set. In the first zero forcing propagation round, $c$ and $d$ are forced by $a$ and $b,$ respectively. In the second round, $e$ is forced by either $c$ or $d$. However, one possible chronological list of forces is $(b \rightarrow d, d \rightarrow e, e \rightarrow c)$, so $c$ is the last to be forced in the chronological list of forces, even though $c$ is forced before $e$ in the zero forcing process. \begin{figure}[htpb!] \centering \includegraphics[width=\linewidth/2]{pentagon.png} \caption{A zero forcing set for the $5$-cycle, with a chronological list of zero forces given by the vertices labeled 1, 2, and 3.} \label{fig:ZF_on_C} \end{figure} We now introduce an analogous concept for the lazy burning of a hypergraph $H$. A \emph{chronological list of lazy burnings}, $(\{h_1,v_1\},\{h_2,v_2\}, \ldots, \{h_k,v_k\})$ with $v_i\in h_i \in E(H)$, is such that the list of sets $$B_0, B_1=B_0\cup\{v_1\}, B_2=B_1\cup \{v_2\}, \ldots, B_k=B_{k-1}\cup\{v_k\}$$ with $B_{i+1} = B_i\cup \{v_{i+1}\}$ satisfying $h_i\setminus\{v_i\} \subseteq B_{i-1}$. We note that, if $B_k = V(H)$, then $B_0$ is a burning set for $H$ and $k = n - |B_0|$. In the case that we take $B_0$ to be a lazy burning set, if $B_{i-1}$ is burned at any time during the lazy burning process, then each vertex in $h_i\setminus \{v_i\}$ is burned. Hence, the vertex $v_i$ will be burned in the next round if it is not already burned. It follows that the entire hypergraph will be burned at some point and that vertex $v_i$ will be burned by round $i$ within the lazy burning process on $H$. We summarize these observations in the following lemma. \begin{lemma} \label{lem:ChronList_equals_Burning} If $(\{h_1,v_1\},\{h_2,v_2\}, \ldots, \{h_k,v_k\})$ is a chronological list of lazy burnings of $H$, then $V(H) \setminus \{v_1, v_2, \ldots, v_k\}$ is a lazy burning set for $H$, and the vertex $v_i$ will be burned during or before the $i$-th lazy burning propagation round. \end{lemma} A \emph{matching} in a graph is a set of edges such that no two edges in the set share a common vertex. The \emph{incidence graph} $\IG(H)$, also known as a \emph{Levi graph}, of a hypergraph $H$ is the bipartite graph with vertex set $V(H)\cup E(H)$, and with an edge in $\mathrm{IG}(H)$ between vertex $v\in V(H)$ and vertex $h\in E(H)$ whenever $v\in h$. We note that there is an implicit ordering on the parts $V(H)$ and $E(H)$ of the bipartition. We will write an edge of $\IG(H)$ as an ordered pair, say $vh$, where $v\in V(H)$ and $h \in E(H)$. A chronological list of lazy burning on a hypergraph $H$ can be converted to a matching in $\mathrm{IG}(H)$, although we need a more restrictive structure than this for our purposes. We define a \emph{$C$-matching} in $\mathrm{IG}(H)$ to be a list of edges from $\mathrm{IG}(H)$, say $M=(v_1h_1, v_2h_2, \ldots, v_kh_k)$, such that $$ h_i \cap V_{i}=\emptyset,$$ where $V_{i} = \{v_{i+1}, v_{i+2}, \ldots, v_{k}\}$ and $1\leq i\leq k$. Define $m(H)$ to be the maximum cardinality of a $C$-matching on $\mathrm{IG}(H)$. \begin{figure}[h!] \centering \begin{subfigure}{0.40\textwidth} \centering \includegraphics[width=\linewidth]{hypergraph.png} \caption{A hypergraph with hyperedges $\{a,b,c\},\{b,c, d\},\{c,d,e\}$.\ A lazy burning set $\{a,b\}$ is indicated by the filled vertices, and a chronological list of lazy burnings is indicated by the vertices containing numbers.} \label{fig:image1} \end{subfigure} \hfill \begin{subfigure}{0.50\textwidth} \centering \includegraphics[width=\linewidth]{incidence.png} \caption{The corresponding incidence graph of the hypergraph.\ A $C$-matching is indicated by the circled edges, labeled from 1 to 3 based on their appearance in the $C$-matching.} \label{fig:image2} \end{subfigure} \caption{Comparison of a chronological list for lazy burning (A) to a $C$-matching on the corresponding incidence graph (B).} \label{fig:two_images} \end{figure} As the following lemma demonstrates, chronological lists and $C$-matchings are equivalent. \begin{lemma} \label{lem:ChronList_equals_CMatching} The list $(\{h_1,v_1\},\{h_2,v_2\}, \ldots, \{h_k,v_k\})$ is a chronological list of lazy burning on $H$ if and only if $(v_1h_1, v_2h_2, \ldots, v_kh_k)$ is a $C$-matching on $\mathrm{IG}(H)$. \end{lemma} \begin{proof} For the forward implication, suppose that $$(\{h_1,v_1\},\{h_2,v_2\}, \ldots, \{h_k,v_k\})$$ is a chronological list of lazy burning on $H$. For $ 0\leq i\leq k $, we define $V_{i} = \{v_{i+1}, v_{i+2}, \ldots, v_{k}\}$ and $B_{i} = V(H) \setminus V_{i}$. From the definition of a chronological list, for each $1\leq i\leq k$, $h_i\setminus \{v_i\} \subseteq B_{i-1} = V(H) \setminus V_{i-1}$. This is equivalent to $(h_i\setminus \{v_i\})\cap V_{i-1} = \emptyset,$ which itself is equivalent to $h_i \cap V_{i} = \emptyset$. This is what is required to make $(v_1h_1, v_2h_2, \ldots, v_kh_k)$ a $C$-matching of $H$. By reversing the chain of implications in the previous paragraph, the proof of the reverse direction follows. \end{proof} Lemmas~\ref{lem:ChronList_equals_Burning} and \ref{lem:ChronList_equals_CMatching} can then be combined to yield the following equality characterizing the lazy burning number of a hypergraph in terms of its order and a maximum cardinality of a $C$-matching. For a $C$-matching $M = (v_1h_1, v_2h_2,\dots, v_mh_m)$, define $M_V= \{v_1,v_2,\dots, v_m\}$, which is the set of vertices that occur in an edge of $M$. \begin{corollary} \label{cor:Burning_equals_CMatching} Let $H$ be a hypergraph and $M$ be a $C$-matching in $\IG(H)$. We have that $V(H)\setminus M_V$ is a lazy burning set for $H$, and $$m(H) = |V(H)| - b_L(H).$$ \end{corollary} Define the \emph{dual} of a hypergraph $H=(V,E)$ to be the hypergraph $H^*$ with vertex set $E$ and with hyperedge set $\{N(v) : v \in V\}$, where we use $N(v)$ to denote the set of hyperedges in $E$ that contain vertex $v$. The bipartite graph formed by dropping the order of the parts of $\mathrm{IG}(H)$ is the same as that from dropping the order of the parts of $\mathrm{IG}(H^*)$. Given a $C$-matching $M=(v_1h_1, v_2h_2, \ldots, v_kh_k)$ in $\mathrm{IG}(H)$, the \emph{retrograde of $M$}, written $M^R$, is the list of edges of $M$ in reverse order and with the ordering of the vertices in each edge swapped; that is, $(h_kv_k, h_{k-1}v_{k-1}, \ldots, h_1v_1)$. The retrograde of $M$ is an ordered set of ordered edges of $\IG(H^*)$. We will show that this retrograde is a $C$-matching of $\IG(H^*)$. \begin{lemma} \label{lem:retrograde_dual} Let $H=(V,E)$ be a hypergraph. For a $C$-matching $M$ in a bipartite graph $\mathrm{IG}(H)$, the retrograde of $M$ is a $C$-matching in the bipartite graph $\mathrm{IG}(H^*)$. \end{lemma} \begin{proof} Let $M=(v_1h_1, v_2h_2, \ldots, v_mh_m)$ be a $C$-matching of the bipartite graph $(V\cup E,I)$, and define $V_i = \{v_{i+1}, v_{i+2}, \ldots, v_{m}\}$ and $E_i = \{h_1, h_2, \ldots, h_{i-1}\}$. By the definition of incidence graphs and $C$-matchings, we have $h_i \cap V_{i}=\emptyset$. For contradiction, we suppose that the retrograde of $M$, $(h_mv_m, h_{m-1}v_{m-1}, \ldots, h_1v_1)$, is not a $C$-matching of the bipartite graph $\IG(H^*)=(E\cup V,\overline{I})$. Note that this implies that there is some $j$ such that $N(v_j) \cap E_{j} \neq \emptyset$, and so there exists some $i \leq j-1$ such that $h_i \in N(v_j)$. However, this implies that $v_j \in h_i$. Since $j \geq i+1$, we also have that $v_j \in V_{i}$. It then follows that $v_j \in h_i \cap V_{i}$, but this contradicts the fact that $M$ is a $C$-matching. This completes the proof. \end{proof} Lemma~\ref{lem:retrograde_dual} immediately yields the following result. \begin{corollary} \label{cor:CMatching_Duals} For a hypergraph $H$, $m(H) = m(H^*)$. \end{corollary} By combining Corollary~\ref{cor:Burning_equals_CMatching} and Corollary~\ref{cor:CMatching_Duals}, we derive the following. \begin{corollary}\label{cor1} For a hypergraph $H$, $|V(H)| - b_L(H) = |V(H^*)| - b_L(H^*)$. \end{corollary} Alternatively, we may describe the lazy burning number of the dual hypergraph as $ b_L(H^*) = |E(H)| - |V(H)| + b_L(H)$. \subsection{Lower Bounds} Consider a $C$-matching $(v_1h_1, v_2h_2, \ldots, v_k h_k)$ in $\IG(H)$. The vertices in $\bigcup_{j \leq i} h_j$ must contain only vertices that are either in the lazy burning set for the corresponding lazy burning process or in $\{v_1, v_2, \ldots, v_i\}$. As such, describing the cardinalities of the hyperedges and the size of the intersection of hyperedges yields lower bounds on the lazy burning number. \begin{theorem}\label{thm:general_lazy_lowerbound} Let $H$ be a hypergraph such that each pair of hyperedges intersects in at most $\overline{\lambda}$ vertices. For $t$ a positive integer, define $\overline{D}_t$ as the minimum sum of the cardinalities of $t$ distinct hyperedges in $H$. For every $t$ with $1 \leq t \leq m(H)$, we have that \[ b_L(H) \geq \overline{D}_t - \overline{\lambda} \binom{t}{2} - t. \] \end{theorem} \begin{proof} Suppose $B$ is an optimal burning set for $H$. As seen in Corollary~\ref{cor:Burning_equals_CMatching}, we may define a $C$-matching in $\mathrm{IG}(H)$, say $M=(v_1h_1, v_2h_2, \ldots, v_mh_m)$, where $m = m(H)$ and $B = V(H) \setminus M_V$. Define $V_{i} = \{v_{i+1}, v_{i+2}, \dots, v_m\}$ for $1\leq i\leq m$. By the definition of a $C$-matching, we have that $$h_i \cap V_{i}=\emptyset.$$ Fix $t$ for $1\leq t\leq m$. The set $H_t=\left( \bigcup_{1 \leq j \leq t} h_j\right) \setminus\{v_1, v_2,\ldots, v_t\}$ satisfies $H_t\cap \{v_1, v_2,\ldots, v_m\} = \emptyset$ and, therefore, $H_t\subseteq B$. For $1\leq i\leq t$, define the set $h'_i = h_i \setminus \bigcup_{1 \leq j < i} h_{j}$, so that $H'_t=\bigcup_{1 \leq j \leq t} h'_{j}$ is a union of disjoint sets. Since each pair of hyperedges may intersect for at most $\overline{\lambda}$ vertices, we have that $|H'_t|\geq \sum_{j=1}^{t} (|h_{j}| - \overline{\lambda} (j-1))$. Also, $H_t = H'_t\setminus \{v_1, v_2, \ldots, v_m\} \subseteq H'_t\setminus \{v_1, v_2, \ldots, v_t\}$ and, thus, we have \begin{eqnarray*} |H_t| &\geq & \left( \sum_{j=1}^{t} (|h_j| - \overline{\lambda} (j-1)) \right) - t \\ &= & \left( \sum_{j=1}^{t} |h_j|\right) - \overline{\lambda} \binom{t}{2} - t\\ &\geq & \overline{D}_t - \overline{\lambda} \binom{t}{2} - t. \end{eqnarray*} Since $H_t \subseteq B$, the proof is complete. \end{proof} By Corollary~\ref{cor:Burning_equals_CMatching} and Corollary~\ref{cor:CMatching_Duals}, we may consider a dual argument that swaps the roles of the vertices and hyperedges in Theorem~\ref{thm:general_lazy_lowerbound}, which yields the following corollary. \begin{corollary} Let $H$ be a hypergraph such that each pair of vertices occurs together in at most $\lambda$ hyperedges and suppose that the sum of the degrees of the $t$ smallest-degree vertices is $D_t$. For each $t$ with $1 \leq t \leq m(H)$, \[ b_L(H) \geq |V(H)| -|E(H)| + D_t - \lambda \binom{t}{2} - t. \] \end{corollary} If the graph has bounds placed on these parameters, then we may provide a more descriptive outcome. \begin{theorem} \label{thm:BurningLowerUniformity} Let $H$ be a hypergraph such that each pair of hyperedges intersects in at most $\overline{\lambda}$ vertices, each hyperedge has cardinality at least $r$, and there are at least $\lceil \frac{r-1}{\overline{\lambda}} + \frac{1}{2} \rceil$ vertices. We then have that $$b_L(H) \geq \frac{(r-1)^2}{2\overline{\lambda}} + \frac{r -1}{2} - \frac{3\overline{\lambda}}{8}.$$ \end{theorem} \begin{proof} From Theorem~\ref{thm:general_lazy_lowerbound}, we have that $b_L(H) \geq rt - \overline{\lambda}\binom{t}{2} - t$ for $1\leq t\leq m(H)$. We may treat this lower bound as an integer-valued function $f(t)$ and then extend it to a real-valued function $f(x)$, where $x\in\mathbb{R}$. The function $f$ is maximized when the derivative is zero or at the endpoints of its domain. This occurs when $x=\frac{r-1}{\overline{\lambda}} + \frac{1}{2},$ since we have assumed that there are at least $\lceil \frac{r-1}{\overline{\lambda}} + \frac{1}{2} \rceil$ many vertices. As $t\in \mathbb{Z}$, the maximum must then occur at either $\lceil \frac{r-1}{\overline{\lambda}} + \frac{1}{2} \rceil$ or $\lfloor \frac{r-1}{\overline{\lambda}} + \frac{1}{2} \rfloor$. We may therefore assume that $t$ has the form $t = \frac{r-1}{\overline{\lambda}} + \frac{1}{2}+\varepsilon$ for some $-1 < \varepsilon < 1$. Substituting this value back in to $rt - \overline{\lambda}\binom{t}{2} - t$ yields $\frac{(r-1+\overline{\lambda}/2)^2}{2\overline{\lambda}} - \frac{\varepsilon^2\overline{\lambda}}{2}$. We thus find that $b_L(H) \geq \frac{(r-1+\overline{\lambda}/2)^2}{2\overline{\lambda}}-\frac{\overline{\lambda}}{2}$, from which the result follows. \end{proof} We may again make use of Corollary~\ref{cor:Burning_equals_CMatching} to obtain a dual result. \begin{corollary}\label{app} Let $H$ be a hypergraph such that each pair of vertices occurs together in at most $\lambda$ hyperedges. Suppose that the minimum degree of the vertices is $\delta$, and $H$ has at least $\lceil \frac{\delta-1} {\lambda} +\frac{1}{2} \rceil$ hyperedges. We have that \[ b_L(H) \geq |V(H)| -|E(H)| + \frac{\delta-1}{2 \lambda} - \frac{\delta-1}{2} - \frac{3\lambda}{8}. \] \end{corollary} In the case that the hypergraph is linear, meaning $\overline{\lambda}=1$, we may repeat the proof of Theorem~\ref{thm:BurningLowerUniformity}, but where we find that the function is maximized with either $t=r-1$ or $t=r$. This gives the following result. \begin{theorem} If $H$ is a linear hypergraph such that each hyperedge has cardinality at least $r$ and there are at least $r$ hyperedges, then $$b_L(H) \geq \binom{r}{2}.$$ \end{theorem} Corollary~\ref{cor:Burning_equals_CMatching} again can be used when deriving the corresponding dual result. \begin{corollary}\label{cor:delta-delta} If $H$ is a linear hypergraph with minimum degree $\delta$, and there are at least $\delta$ hyperedges, then $$ b_L(H) \geq |V(H)| -|E(H)| + \binom{\delta}{2}.$$ \end{corollary} \subsection{Upper Bounds} By decomposing a $C$-matching into the set of vertices or the set of edges that are contained in the matching, we find that we have a vertex cover or edge cover of $H$, respectively. This also provides us with a number of upper bounds on the lazy burning number of $H$. \begin{lemma} \label{lem:Cmatching_MinEdgeCover} If $H$ is a hypergraph, then we have that any maximum $C$-matching has cardinality at least that of a minimum vertex cover. \end{lemma} \begin{proof} Let $M=(v_1h_1, v_2h_2, \ldots, v_kh_k)$ be a $C$-matching of the maximum possible length $k=|V(H)|-b_L(H)$. We will show that the set of vertices given by $M_V=\{v_1, v_2,\ldots, v_k\}$ forms a vertex cover of $H$. Suppose for contradiction that $M_V$ does not form a vertex cover. There must then be some hyperedge $h$ with none of its vertices in $M_V$. For any vertex $v\in h$, the matched pair $vh$ may be appended to the $C$-matching to form a longer $C$-matching, contradicting maximality. \end{proof} The proof to show that the cardinality of a maximum $C$-matching has at least that of the cardinality of a minimum edge cover of $H$ is analogous with dual terms interchanged. An \emph{isolated vertex} is not in any hyperedge. \begin{lemma} \label{lem:Cmatching_MinVertexCover} If $H$ is a hypergraph with no isolated vertices, then any maximum $C$-matching has a cardinality at least the cardinality of a minimum edge cover of $H$. \end{lemma} There are straightforward upper bounds on the minimum size of a vertex cover and an edge cover in terms of the maximum degree $\Delta(H)$ and maximum hyperedge cardinality $\overline{\Delta}(H)$, respectively. \begin{theorem}\label{upp} If $H$ is a hypergraph with maximum vertex degree $\Delta$, then $$b_L(H) \leq |V(H)| - \left\lceil \frac{|E(H)|}{\Delta}\right\rceil .$$ \end{theorem} \begin{proof} We find a lower bound on the size of a vertex cover of $H$, which combines with Lemma~\ref{lem:Cmatching_MinVertexCover} to complete the proof. If a vertex cover of $H$ contains $t$ vertices, then since each vertex is contained in at most $\Delta$ hyperedges, at most $t\Delta$ hyperedges contain vertices from the vertex cover. Since each hyperedge of $E(H)$ contains a vertex in the cover, we then have $|E(H)| \leq t\Delta$. We then have that $b_L(H) = |V(H)| - m(H) \leq |V(H)| - t$, and the result follows. \end{proof} In an analogous fashion, the hyperedges from a $C$-matching form an edge cover of $H$, from which we derive the following result using an edge cover instead of a vertex cover. \begin{theorem} If $H$ is a hypergraph with maximum hyperedge cardinality $\overline{\Delta}$ that contains no isolated vertices, then $$b_L(H) \leq |V(H)| - \left\lceil \frac{|V(H)|}{\overline{\Delta}}\right\rceil .$$ \end{theorem} In the above two results, we found that the set of vertices (respectively, edges) contained within a $C$-matching formed a vertex (respectively, edge) cover of $H$. Instead, if we consider the ordered list of vertices $(v_1, v_2, \ldots, v_k)$ coming from the restriction of a maximum $C$-matching to just its vertices, then we arrive at a topic previously studied by the present authors in the context of burning Latin square hypergraphs; see \cite{LS}. An $n$-uniform Latin square hypergraph, written $H_L$, has vertex set as the entries of the Latin square $L$ and hyperedges containing all vertices that share a given row, a given column, or a given symbol. A \emph{cover-sequence} of $H_L$ is a sequence of vertices $v_1, v_2,\ldots, v_k$ such that $\{v_1, v_2,\ldots, v_k\}$ is a vertex cover of $H_L$ and each $v_i$ does not share all three of its incident hyperedges with vertices that proceeded it in the sequence. It is straightforward to see that the restriction of a $C$-matching to its vertices satisfies these conditions. Further, if we match a vertex $v_i$ to a hyperedge $h_i$ such that $h_i$ does not contain a vertex earlier in the sequence, then the constructed sequence of vertex-edge pairs forms a $C$-matching on $\mathrm{IG}(H_L)$. As such, $C$-matchings of $\mathrm{IG}(H_L)$ are equivalent to cover-sequences of Latin squares, and so results in this section generalize results on the lazy burning of Latin squares in \cite{LS}. \section{Hypergraph Cores} We introduce another characterization of lazy burning sets in terms of certain induced subhypergraphs. We are interested in the maximum induced subhypergraph so that every hyperedge is of cardinality greater than one. In the literature, this would correspond to the so-called 2-core of the dual hypergraph. For ease of notation, we refer to this simply as the \emph{core} of $H$, written $\core(H)$ (this is not to be confused with the core of a graph as used in graph homomorphism theory). The core can be obtained by continually deleting every hyperedge with cardinality one, along with the vertex it contains, until all remaining hyperedges have cardinality greater than one. Let $H$ be a hypergraph and $U \subseteq V(H)$. The \emph{subhypergraph of $H$ weakly induced by $U$}, denoted $H[U]$, is the hypergraph with $V(H[U]) = U$ and $$ E(H[U]) = \{ h\cap U: h\in E(H)\text{ and } h\cap U\neq\emptyset\}. $$ This is also not to be confused with the strongly induced subhypergraphs; in the following, we mean an {\em induced subhypergraph} as a weakly induced subhypergraph since it is the only type of subhypergraph we are concerned with in this paper. We define the subhypergraph formed by vertex removal as $H\setminus U = H[V(H)\setminus U]$. We now provide an algorithm for determining the core of a hypergraph and a theorem on its correctness. \setlength{\algomargin}{20pt} \begin{algorithm}[!htbp] \SetKwInOut{Input}{input}\SetKwInOut{Output}{output} \Input{A hypergraph $H$.} \Output{The core of $H$, an ordered list of vertex removals $R$.} \BlankLine Set $H' \leftarrow H$;\\ Set $R \leftarrow []$ (an empty list);\\ Set $S \leftarrow \{e\in E(H'): |e| = 1\}$;\\ \While{$S\neq\emptyset$}{ Pick any $\{v\}\in S$; \\ Set $S\leftarrow S\setminus\{\{v\}\}$;\\ Add $v$ to $R$;\\ Set $S \leftarrow S\cup\{\{u\}: \{u,v\}\in E(H')\}$;\\ Set $H' \leftarrow H'\setminus\{v\}$; } \Return{$H'$ and $R$.} \caption{An algorithm returning the core of $H$ and an ordered list of vertex removals.} \label{core_H} \end{algorithm} \begin{theorem} After inputting a hypergraph $H$ to Algorithm~\ref{core_H}, it returns $\core(H)$ and a vertex set $R = V(H)\setminus V(\core(H))$ with an indexing. \end{theorem} \begin{proof} Suppose there are two induced subhypergraphs of $H$, say $H_1$ and $H_2$, such that all their hyperedges are of cardinality greater than one. The subhypergraph $H[V(H_1)\cup V(H_2)]$ also has the cardinality of all its hyperedges greater than one. Therefore, the core of a hypergraph is unique. From Algorithm~\ref{core_H}, we obtain an induced subhypergraph $J$ of $\core(H)$ with no hyperedge of cardinality one. Let $R = V(H)\setminus V(J) = \{v_1,v_2,\dots, v_r\}$ be the removed vertices, indexed by the algorithm. Suppose to the contrary that $J$ is not the core, and hence there is $A \subseteq R$ so that $H' = H[V(J)\cup A] = \core(H)$. Let $m$, with $1\leq m\leq r,$ be the least index so that $v_{m}\in A$. If $R' = \{v_1,v_2,\dots, v_{m-1}\}$, then $R'\subseteq R\setminus A$ and, therefore, $H'$ is an induced subhypergraph of $H\setminus R'$. By Algorithm~\ref{core_H}, we have that $\{v_{m}\}\in E(H\setminus R')$, which implies that $\{v_{m}\}\in E(H')$. This contradicts the assumption that $H'$ contains no singleton hyperedge. \end{proof} We note that Algorithm 1 produces the core and a list of vertex removals in polynomial time. \begin{theorem}\label{thm: core complexity} If $H$ is a hypergraph with order $n$ and $m$ edges, then the complexity of Algorithm~\ref{core_H} is $O(nm)$. \end{theorem} \begin{proof} Line~3 takes $O(m)$-time. The \textbf{while} loop on Line~4 repeats for at most $n$ times, say $O(n)$. For each iteration of the \textbf{while} loop, both Line~8 and 9 can be computed in $O(m)$-time. All other lines take $O(1)$-time. \end{proof} In the following lemma, we collect some properties of cores. The proofs either follow from the definitions or are straightforward, arguing via a chronological list of lazy burning. A hypergraph is {\em degenerate} if its core contains no vertex. \begin{lemma}\label{lemmac} Let $H$ be a hypergraph and $R =\{r_1,r_2,\dots, r_k\} = V(H)\setminus V(\core(H))$ be the vertex set returned by Algorithm~\ref{core_H}. Suppose $R\neq\emptyset$. We have the following. \begin{enumerate} \item For every $1\leq i\leq k$, $\{r_i\}$ is a singleton hyperedge in $H\setminus\{r_1,r_2,\dots, r_{i-1}\}$. In particular, $R\neq\emptyset$ implies $H$ contains a singleton hyperedge. \item If $L\subsetneq L'\subseteq V(H)$ then $\core(H\setminus L')$ is an induced subhypergraph of $\core(H\setminus L)$. If $L' = L\cup\{v\}$ for some $v\in V(H)\setminus L$ such that $\{v\}\in E(H\setminus L)$, then $\core(H\setminus L') = \core(H\setminus L)$, and $H\setminus L'$ is degenerate if and only if $H\setminus L$ is degenerate. \item For all $1\leq i\leq k$, $\core(H) = \core(H\setminus\{r_1,r_2,\dots, r_i\})$. \end{enumerate} \end{lemma} We also need the following lemma. \begin{lemma}\label{lemma: backward closure} Let $H$ be a hypergraph, $h\in E(H)$, and $B\subseteq V(H)$. Assume $h\subseteq B$ and choose any vertex $u\in h$. The vertex set $B$ is a lazy burning set for $H$ if and only if $B\setminus\{u\}$ is a lazy burning set for $H$. \end{lemma} \begin{proof} Fix a chronological list $S$ of $B$ and insert $\{h,u\}$ at the beginning of $S$. One may verify that the new list is a chronological list of a lazy burning set $B\setminus\{u\}$. The reverse implication follows immediately from the fact that $B\setminus \{u\}\subseteq B$. \end{proof} We provide another characterization of lazy burning in the following, which is the main result of the section. \begin{theorem}\label{thm: empty core} A vertex subset $B$ of a hypergraph $H$ is a lazy burning set if and only if the hypergraph $H\setminus B$ is degenerate. \end{theorem} \begin{proof} Let $B\subseteq V(H)$. We proceed by induction on the order of $H\setminus B$, say $k = |V(H\setminus B)|$. The statement holds when $k=0$, where $V(H) = B$. For the induction hypothesis, fix $k\geq 1$, and assume the following holds for any hypergraph $H_0$ and vertex set $B_0\subseteq V(H_0)$ satisfying $|V(H_0\setminus B_0)|< k$: $B_0$ is a lazy burning set for $H_0$ if and only if $H_0\setminus B_0$ is degenerate. Let $H$ be a hypergraph and $B\subseteq V(H)$ such that $|V(H\setminus B)|=k$. We will prove the statement holds for $H$ and $B$. For the forward direction, suppose $B$ is a lazy burning set for $H$. Fix a chronological list of $B$ and let $\{v_1, h_1\}$ be the first element in the list. Let $B_1 = B\cup\{v_1\}$; we have that $B_1$ is a lazy burning set for $H$ since $B\subseteq B_1$. By the induction hypothesis and that $|V(H\setminus B_1)|<k$, we have that $H\setminus B_1$ is degenerate. Finally, since $\{v_1\} = h_1\setminus B$ is a hyperedge in $H\setminus B$, we have that $H\setminus B$ is degenerate by Lemma~\ref{lemmac} (2). For the reverse direction, suppose $H\setminus B$ is degenerate. By Lemma~\ref{lemmac} (1), let $r_1\in R$ be such that $\{r_1\}\in E(H\setminus B)$. This implies that there is a hyperedge $h\in E(H)$ so that $\{r_1\} = h\setminus B$. By Lemma~\ref{lemmac} (2), $H\setminus(B\cup\{r_1\})$ is degenerate. Therefore, by the induction hypothesis, $B\cup \{r_1\}$ is a lazy burning set for $H$. Since $h\subseteq B\cup\{r_1\}$, $B$ is a lazy burning set for $H$ by Lemma~\ref{lemma: backward closure}. \end{proof} The next result reduces the burning number of a hypergraph to the burning number of its core. \begin{theorem}\label{thm: also burns the core} For any hypergraph $H$ and $B\subseteq V(\core(H))$, $B$ is a lazy burning set for $\core(H)$ if and only if it is a lazy burning set for $H$. \end{theorem} \begin{proof} We proceed with the proof by induction on the order of $R = V(H)\setminus V(\core(H))$. When $|R| = 0$, we have $H = \core(H)$, and the statement holds. Let $H$ be a hypergraph so that $|R|>0$ and suppose the statement holds for all hypergraphs $H_0$ such that $|V(H_0)\setminus V(\core(H_0))|<|R|$. Assume an indexing on $R$ by Algorithm~\ref{core_H}. The first element $r_1\in R$ is a singleton hyperedge according to Lemma~\ref{lemmac} (1), so $\core(H) = \core(H\setminus\{r_1\})$ by Lemma~\ref{lemmac} (3). We then have that $$|R\setminus\{r_1\}| = |V(H\setminus\{r_1\})\setminus V(\core(H))|<|R|.$$ By the induction hypothesis, $B$ is a lazy burning set for $\core(H)$ if and only if $B$ is a lazy burning set for $H\setminus\{r_1\}$. We finish the proof by a series of equivalent statements. The set $B$ being a lazy burning set for $\core(H)$ is equivalent to $B$ being a lazy burning set for $\core(H\setminus\{r_1\})$ because they are the same hypergraph according to Lemma~\ref{lemmac} (3). The latter is equivalent to $H\setminus(B\cup \{r_1\})$ being degenerate by Theorem~\ref{thm: empty core}, which is equivalent to $H\setminus B$ being degenerate by applying Lemma~\ref{lemmac} (2) with $L = B$ and $v = r_1$. The latter condition is equivalent to $B$ being a lazy burning set for $H$ by Theorem~\ref{thm: empty core}. The proof follows. \end{proof} As an immediate corollary, we have the following. \begin{corollary} \label{H_equals_core} For any hypergraph $H$, $b_L(H) = b_L(\core(H))$. \end{corollary} By applying hypergraph cores, we show the monotonicity of lazy burning numbers with respect to vertex removal. The following theorem contrasts with zero forcing, which is not monotone. \begin{theorem} The lazy burning number is monotone under vertex removal; that is, for any vertex $v\in V(H)$, $b_L(H)-1\leq b_L(H\setminus\{v\})\leq b_L(H).$ \end{theorem} \begin{proof} For the upper bound, let $B$ be an optimal lazy burning set for $H$. Suppose first that $v\notin B$. Observe that $B\cup\{v\}$ is a lazy burning set for $H$, so $H\setminus(B\cup\{v\})$ is degenerate by Theorem \ref{thm: empty core}. Also note that $H\setminus(B\cup\{v\})=(H\setminus\{v\})\setminus B$ and $B\subseteq V(H\setminus\{v\})$, so we have that $B$ is a lazy burning set for $H\setminus\{v\}$ by Theorem \ref{thm: empty core}. We can therefore conclude that $b_L(H\setminus\{v\})\leq |B|=b_L(H)$ in this case. Now, suppose $v\in B$. Observe that $H\setminus B=(H\setminus\{v\})\setminus (B\setminus\{v\})$, and hence, they have the same core. By Theorem~\ref{thm: empty core}, $H\setminus B$ is degenerate, so $(H\setminus\{v\})\setminus (B\setminus\{v\})$ is also degenerate. This implies $B\setminus\{v\}$ is a lazy burning set for $H\setminus\{v\}$ by Theorem \ref{thm: empty core}. Hence, $b_L(H\setminus\{v\})\leq |B\setminus\{v\}|<|B|=b_L(H)$ in this case. For the lower bound, let $B$ be an optimal lazy burning set for $H\setminus\{v\}$. By Theorem~\ref{thm: empty core}, we have that $(H\setminus\{v\})\setminus B$ is degenerate. Observe that $(H\setminus\{v\})\setminus B = H\setminus(B\cup\{v\})$. Since this hypergraph is degenerate, $B\cup\{v\}$ is a lazy burning set for $H$ by Theorem~\ref{thm: empty core}. We therefore have $b_L(H)\leq |B\cup\{v\}|=|B|+1=b_L(H\setminus\{v\})+1$, and the lower bound follows.\end{proof} \section{Complexity via Zero Forcing} The principal goal of this section is to show that computing an upper bound on the lazy burning number is NP-complete, which answers a conjecture in \cite{BJP}. We first provide another characterization of lazy burning sets in hypergraphs via zero forcing sets in their incidence graphs. \begin{theorem}\label{zero_set_equivalence} For a hypergraph $H$, a subset $B\subseteq V(H)$ is a lazy burning set for $H$ if and only if $B\cup E(H)$ is a zero forcing set for $\mathrm{IG}(H)$. \end{theorem} \begin{proof} For the forward direction, suppose that $B$ is a lazy burning set for $H$. Let $k = |V(H)| - |B|$ and consider a chronological list of lazy burning of $S = (\{v_1,h_1\},\{v_2,h_2\},\dots, \{v_k, h_k\})$ with the corresponding list of burned vertices $B = B_0, B_1,\dots, B_k = V(H)$. For $0\leq i\leq k$, define $B_i' = B_i\cup E(H)\subseteq V(\IG(H))$. For $1\leq i\leq k$, we have that $h_i\setminus\{v_i\}\subseteq B_{i-1}$. It follows that $N_{\IG(H)}(h_i)\setminus\{v_i\}\subseteq B_{i-1} \subseteq B_{i-1}'$ in $\IG(H)$, and hence, the list $(h_1\to v_1, h_1\to v_2, \dots, h_k\to v_k)$ is a chronological list of zero forcing in $\IG(H)$, where $B_0', B_1', \dots, B_k'$ is the corresponding list of black vertices. We may conclude that $B\cup E(H)$ is a zero forcing set for $\IG(H)$. The reverse direction follows analogously by reversing the implications in the forward direction above. \end{proof} We note that Theorem~\ref{zero_set_equivalence} gives the new inequality $$z(\mathrm{IG}(H))\leq b_L(H)+|E(H)|.$$ Denote the lazy burning number as defined in \cite{JT}, where singleton hyperedges do not spontaneously burn, by $b_L^o(H)$. Consider the following decision problems. \medskip \noindent PROBLEM: Lazy burning \\ INSTANCE: A hypergraph $H$ and a positive integer $k \leq |V(H)|$.\\ QUESTION: Is $b_L(H)\leq k$? \medskip \noindent PROBLEM: Lazy burning without spontaneous burning \\ INSTANCE: A hypergraph $H$ and a positive integer $k \leq |V(H)|$.\\ QUESTION: Is $b_L^o(H)\leq k$? \medskip To show the NP-completeness of the two problems, we need the following theorem. \begin{theorem}\label{thm 0} For every hypergraph $H$, if $H$ contains no singleton hyperedge, then $b_L^o(H) = b_L(H)$. Further, $b_L^o(\core(H)) = b_L(H)$. \end{theorem} \begin{proof} For the first statement, if a list is a chronological list of lazy burning in one process, then it is also a chronological list of lazy burning in the other one. For the second statement, we have that $b_L^o(\core(H)) = b_L(\core(H)) = b_L(H)$, where the first equality holds since the core of a hypergraph contains no singleton hyperedge, and the second equality holds by Corollary~\ref{H_equals_core}. \end{proof} The symmetric and skew zero forcing decision problems are as follows. \medskip \noindent PROBLEM: Zero forcing \\ INSTANCE: A graph $G$ and a positive integer $k \leq |V(G)|$.\\ QUESTION: Is $z(G)\leq k$? \medskip \noindent PROBLEM: Skew zero forcing \\ INSTANCE: A graph $G$ and a positive integer $k \leq |V(G)|$.\\ QUESTION: Is $z_0(G)\leq k$? \medskip Let $G$ be a graph and $N(v) = N_G(v)$ be the \textit{neighborhood} of $v\in V(G)$. The {\em open neighborhood hypergraph} ${\calN}(G)$ is a hypergraph with $V({\calN}(G)) = V(G)$ and $E(\calN(G)) = \{N_G(v):v\in V(G)\}$. The {\em closed neighborhood hypergraph ${\calN}[G]$} is defined analogously with the hyperedge set consisting of the {\em closed neighborhood} $N[v] = N_G[v] = N(v)\cup\{v\}$ of each vertex in $G$. The following theorem characterizes skew zero forcing as lazy burning on the open neighborhood hypergraph. \begin{theorem} \label{skew_forcing_equals_lazy_burning} Let $G$ be a graph and $B\subseteq V(G)$. The set $B$ is a skew zero forcing set for $G$ if and only if $B$ is a lazy burning set for $\calN(G)$. In particular, $z_0(G) = b_L(\calN(G))$. \end{theorem} \begin{proof} Let $\calB = (B = B_0, B_1, \dots, B_k = V(G))$ be a list of vertex subsets, where $k = |V(H)| - |B|$, and $V(H)\setminus B = \{v_1, v_2, \dots, v_k\}$ is indexed so that $B_i = B_{i-1}\cup \{v_i\}$ for $1\leq i\leq k$. Suppose that $\calB$ is the corresponding list of black vertices of a chronological list of skew zero forces of $B$, say $S = (u_1\to v_1, u_2\to v_2, \dots, u_k\to v_k)$. For $1\leq i\leq k$, we have that $N(u_i)\setminus\{v_i\}\subseteq B_{i-1}$. Hence, $S$ is a chronological list of skew zero forcing if and only if the list $$(\{v_1, N(u_1)\}, \{v_2, N(u_2)\},\dots, \{v_k, N(u_k)\})$$ is a chronological list of lazy burning of $B$ on $\calN(G)$. \end{proof} An analogous and so omitted argument proves the following. \begin{theorem}\label{thm: closed neighborhood and zero forcing} Let $G$ be a graph and $B\subseteq V(G)$. If $B$ is a zero forcing set for $G$, then $B$ is a lazy burning set for $\calN[G]$. In particular, $b_L(\calN[G])\leq z(G)$. \end{theorem} The converse of Theorem~\ref{thm: closed neighborhood and zero forcing} does not hold. Let $K_{1,j}$ be the star graph, where $V(K_{1,j}) = \{v,u_1, u_2,\dots, u_j\}$ and $v$ is the universal vertex. Note that $N_{K_{1,j}}[v] = V(K_{1,j})$ and $N_{K_{1,j}}[u_i] = \{v, u_i\}$ for each leaf $u_i$. We then have that $$E(\calN[K_{1,j}]) = \{V(K_{1,j}), v u_1, v u_2, \dots, v u_j\}.$$ The set $\{v\}$ is a lazy burning set for ${\calN}[K_{1,j}]$ since burning spreads via each edge $vu_i\in E({\calN}[K_{1,j}])$. Meanwhile, the zero forcing number of a star graph is $j -1 $ by initially forcing all leaves except one. For completeness, we restate results on the complexity of skew zero forcing from \cite{CF}. We first need some terminology introduced there. For a graph $G$, let $T(G)$ denote the graph obtained by ``gluing a triangle'' to each vertex of $G$. More precisely, $V(T(G)) = V(G)\times\{1,2,3\}$ and \begin{equation*} \begin{split} E(T(G)) = &\ \{e\times\{1\}: e\in E(G)\}\ \cup \\ &\ \{(v,i)(v, j): v\in V(G), i,j\in\{1,2,3\}, i\neq j\}. \end{split} \end{equation*} In particular, the induced subgraph $T(G)[V(G)\times\{1\}]$ is isomorphic to $G$ using a canonical map such that $(v,1)\mapsto v$, for $v\in V(G)$. \begin{theorem}[{\cite[Theorem~3.7]{CF}}]\label{red} For a graph $G$ and $B\subseteq V(G)$, $B$ is a zero forcing set if and only if $B\times\{1\}$ is a skew zero forcing set for $T(G)$. Further, $B$ is optimal if and only if $B\times\{1\}$ is optimal; that is $z(G) = z_0(T(G))$. \end{theorem} By Theorem~\ref{red}, the skew zero forcing problem is at least as hard as the zero forcing problem, which is shown to be NP-complete in \cite{A, Y}. We then have the following result. \begin{theorem}[{\cite[Corollary~3.8]{CF}}]\label{szf is NPC} The skew zero forcing problem is NP-complete. \end{theorem} We now state the main result of this section. \begin{theorem} \label{lbsc is npc} The lazy burning problem is NP-complete. \end{theorem} \begin{proof} Given a hypergraph $H$ and a vertex set $B\subseteq V(H)$, determining whether or not $B$ is a lazy burning set for $H$ takes polynomial time by determining whether or not $H\setminus B$ is degenerate, as in Theorem~\ref{thm: empty core}. Theorem~\ref{thm: core complexity} states that Algorithm~\ref{core_H} can verify it in polynomial time. Hence, the lazy burning problem is in NP. By Theorem~\ref{skew_forcing_equals_lazy_burning}, we know that $\calN(\cdot)$ is a transformation from the skew zero forcing problem to the lazy burning problem. That is, if a vertex set $B\subseteq V(G)$ is a skew forcing a set of a graph $G$, then $B\subseteq V(\calN(G)) = V(G)$ is a lazy burning set for the hypergraph $\calN(G)$. Note that $\calN(\cdot)$ is a polynomial-time transformation. Therefore, the lazy burning problem is NP-hard by Theorem~\ref{szf is NPC}, and hence, is NP-complete. \end{proof} We also derive the following. \begin{theorem} The lazy burning problem without spontaneous burning is NP-complete. \end{theorem} \begin{proof} Given a hypergraph $H$ and a vertex subset $B$, we first show that verifying whether or not $B$ is a lazy burning set can take polynomial time. This can be done by considering first taking another hypergraph $H'$ with $V(H') = V(H)$ and $E(H') = E(H)\setminus\{e\in E(H): |e|=1\}$. We know that $B$ is a lazy burning set without spontaneous burning if and only if $ H'\setminus B$ is degenerate. The first step takes $O(m)$-time, and the second step takes $O(nm)$-time by Theorem~\ref{thm: core complexity}. In particular, the lazy burning problem without spontaneous burning is in NP. The function $\core(\cdot)$ is a transformation from the lazy burning problem with spontaneous burning to the lazy burning problem without spontaneous burning, which is proved in Theorem~\ref{thm 0}. Hence, the lazy burning problem without spontaneous burning is NP-hard and, hence, is NP-complete. \end{proof} We may use the complexity results from lazy burning to analyze the complexity of the skew zero forcing numbers of bipartite graphs. Given a bipartite graph $G = (X\cup Y, I)$, we may construct two hypergraphs $H_1$ and $H_2$, where $V(H_1) = X$, $E(H_1) = \{N_G(y): y\in Y\}$, and $H_2 = H_1^*$; that is, we may view every bipartite graph as a hypergraph $H = (V, E)$ by considering $V = X$ and $E = \{N_G(y): y\in Y\}$. Note that the hyperedge set $E=E(H_1)$ may be a multiset, and lazy burning performs identically on hypergraphs whenever $E(H_1)$ is a set or a multiset. \begin{theorem}\label{thm: lb-lbdual is szf} Let $H$ be a hypergraph. We then have that $B$ is a lazy burning set for $H$ and $B^*$ is a lazy burning set for $H^*$ if and only if $B\cup B^*$ is a skew zero forcing set for $\IG(H)$. In particular, $$ z_0(\IG(H)) = b_L(H) + b_L(H^*). $$ \end{theorem} \begin{proof} The proof is straightforward by considering that the neighborhood hypergraph of an incidence graph $G = \IG(H)$ is precisely the disjoint union of $H\cup H^*$. The statement follows by Theorem~\ref{skew_forcing_equals_lazy_burning}. \end{proof} We have the following by the alternative form of Corollary~\ref{cor1}. \begin{corollary}\label{cor: z=2bl} For any hypergraph $H$, $ z_0(\IG(H)) = 2b_L(H) + |E(H)| - |V(H)|$. \end{corollary} \medskip\noindent PROBLEM: Skew zero forcing on bipartite graphs\\ INSTANCE: A bipartite graph $G$ and an integer $k\leq |V(G)|$. \\ QUESTION: Is $z_0(G)\leq k$? \medskip We arrive at the following new result on skew zero forcing. The results in \cite{DeAlba} also give a similar characterization of skew zero forcing on bipartite graphs using special matchings, but our technique is essentially different. \begin{theorem}\label{szf on bpt is npc} The skew zero forcing problem on bipartite graphs is NP-complete. \end{theorem} \begin{proof} For a hypergraph $H$, it takes polynomial time to generate the bipartite graph $\IG(H)$. By Corollary~\ref{cor:Burning_equals_CMatching}, from a lazy burning set $B$ for $H$, we obtain a $C$-matching by performing the lazy burning process and then obtain a lazy burning set for $H^*$. Since verifying if a set of vertices is a skew zero forcing set for a graph takes polynomial time, and the lazy burning problem is NP-complete by Theorem~\ref{lbsc is npc}, we have that the skew zero forcing problem on bipartite graphs is NP-complete. \end{proof} \section{Further Directions} We presented several characterizations of lazy burning on hypergraphs via matchings and zero forcing. It would be interesting to see what new insights can be provided by these connections, especially in the case of zero forcing on graphs. The lazy burning of hypergraphs associated with Steiner Triple Systems was shown to equal their dimension; see \cite{STS}. One question is to determine if there are relationships between the lazy burning number of hypergraphs associated with other designs, such as balanced incomplete block designs, and a natural notion of dimension. A hypergraph is {\em critical $k$-burnable} if $b_L(H) = k$, and removing any vertex results in the decrease of its lazy burning number. It would be interesting to characterize critical $k$-burnable hypergraphs. \section{Acknowledgements} The first author was supported by an NSERC grant, and the second author was supported by an NSERC PGS-D scholarship. \begin{thebibliography}{99} \bibitem{A} A.\ Aazami, \emph{Hardness results and approximation algorithms for some problems on graphs}, PhD thesis, University of Waterloo, 2008. \bibitem{AIM} AIM Minimum Rank\ -\ Special Graphs Work Group, Zero forcing sets and the minimum rank of graphs, \textit{Linear Algebra and its Applications} \textbf{428} (2008) 1628--1648. \bibitem{berge1} C.\ Berge, \emph{Graphs and Hypergraphs}, Elsevier, New York, 1973. \bibitem{berge2} C.\ Berge, \emph{Hypergraphs: The Theory of Finite Sets}, North-Holland, Amsterdam, 1989. \bibitem{survey} A.\ Bonato, A survey of graph burning, \emph{Contributions to Discrete Mathematics} \textbf{16} (2021) 185--197. \bibitem{bbook} A.\ Bonato, \emph{An Invitation to Pursuit-Evasion Games and Graph Theory}, American Mathematical Society, Providence, Rhode Island, 2022. \bibitem{BJR} A.\ Bonato, J.\ Janssen, E.\ Roshanbin, Burning a graph as a model of social contagion, In: \emph{Proceedings of WAW'14}, 2014. \bibitem{BJR1} A.\ Bonato, J.\ Janssen, E.\ Roshanbin, How to burn a graph, \emph{Internet Mathematics} \textbf{1}--\textbf{2} (2016) 85--100. \bibitem{LS} A.\ Bonato, C.\ Jones, T.G.\ Marbach, T.\ Mishura, How to burn a Latin square, Preprint 2024. \bibitem{bretto} A.\ Bretto, \emph{Hypergraph Theory: an Introduction}, Springer, 2013. \bibitem{STS} A.\ Burgess, P.\ Danziger, C.\ Jones, T.G.\ Marbach, D.\ Pike, Burning Steiner Triple Systems, Preprint 2024. \bibitem{BJP} A.\ Burgess, C.\ Jones, D.\ Pike, Extending graph burning to hypergraphs, Preprint 2024. \bibitem{CF} J.\ Cooper, G.\ Fickes, Zero loci of nullvectors and skew zero forcing in graphs and hypergraphs, Preprint 2024. \bibitem{DeAlba} L.M.\ DeAlba, Some results on minimum skew zero forcing sets, and skew zero forcing number, Preprint 2024. \bibitem{FHsurvey} S.M.\ Fallat, L.\ Hogben, The minimum rank of symmetric matrices described by a graph: A survey, \textit{Linear Algebra and its Applications} \textbf{426} (2007) 558--582. \bibitem{hlsbook} L.\ Hogben, J.C.-H.\ Lin, B.L.\ Shader, \textit{Inverse Problems and Zero Forcing for Graphs}, \textbf{270}, American Mathematical Society, 2022. \bibitem{IMA} IMA-ISU Research Group on Minimum Rank, Minimum rank of skew-symmetric matrices described by a graph, \textit{Linear Algebra and its Applications} \textbf{432} (2010) 2457--2472. \bibitem{JT} C.\ Jones, \emph{Hypergraph Burning}, Masters Thesis, Memorial University of Newfoundland, 2023. \bibitem{west} D.B.\ West, \emph{Introduction to Graph Theory}, 2nd edition, Prentice Hall, 2001. \bibitem{Y} B.\ Yang, Fast-mixed searching and related problems on graphs, \emph{Theoretical Computer Science} \textbf{507} (2013) 100--113. \end{thebibliography} \end{document} Given a hypergraph $H$ and $L\subseteq V(H)$, the {\em weak induced subhypergraph} is the hypergraph $H\setminus L$ with vertex set $V(H)\setminus L$ and the edge set consists of $e\setminus L$ for $e\in E(H)$ and $e\not\subseteq L$. \begin{algorithm}[H] \label{core_H} \SetKwInOut{Input}{input}\SetKwInOut{Output}{output} \Input{a hypergraph $H=(V,E)$} \Output{the core of $H$, $C_H$} \BlankLine initialize $C_H=H$ \Repeat{$C_H$ is unchanged}{ \ForAll{$e\in E(C_H)$}{ \If{$|e|=1$}{ Delete both $e$ and the vertex it contains from $C_H$ } } } \Return{$C_H$} \text{(the core of $H$)} \caption{Construct the core of $H$.} \end{algorithm} \begin{lemma} Algorithm \ref{core_H} runs in time at most $\mathcal{O}(nm^2),$ where $|V(H)|=n$ and $|E(H)|=m$. \end{lemma} \begin{proof} Each iteration of the outer loop (lines 2-8) deletes at least one vertex from $C_H$ except for the last iteration. Hence, the outer loop executes at most $n+1$ times. Each instance of the inner loop (lines 3-7) iterates over at most $m$ objects. The operation in line 4 takes constant time. Finally, consider the operation in line 5. Denote the unique vertex in $e$ by $v$. Deleting $v$ from $V(C_H)$ and $e$ from $E(C_H)$ takes constant time. However, the algorithm must also delete $v$ from each other edge in $H$ that contained it, which accounts for at most $m$ more operations. \end{proof} Perhaps unsurprisingly, we can characterize lazy burning sets using the concept of cores. This is interesting because $C_H$ is a rigorously defined set-theoretic object that is independent of the rules of the lazy hypergraph burning game. \begin{theorem} \label{core_equals_lazy_burning} If $H$ is a hypergraph and $L\subseteq V(H)$, then $L$ is a lazy burning set for $H$ if and only if the core of $H\setminus L$ is empty. \end{theorem} \begin{proof} For the forward implication, assume $L$ is a lazy burning set for $H$ and let $u_1,\ldots,u_k$ be an ordering of $V(H)\setminus L$ such that the earlier a vertex burns in the lazy burning process, the earlier it appears in the list (with ties broken arbitrarily). We will prove that none of the vertices $u_i$ are in the core of $H\setminus L$ by induction on $i$. For the base case, $u_1$ burns in the first propagation step because it belongs to some edge $e$ that contains no other unburned vertices. If $e$ was a singleton edge, then $u_1$ gets deleted and hence does not belong to $C_{H\setminus L}$. Otherwise, each other vertex in $e$ belongs to $L$, and hence the corresponding edge in $H\setminus L$ contains only $u_1$. Hence, $u_1$ gets deleted in this case as well. Now, assume each of $u_1,u_2,\ldots,u_i$ gets deleted while constructing $C_{H\setminus L}$. Consider the edge $e$ that causes $u_{i+1}$ to burn in the lazy burning game. By the definition of our ordering, no vertex in $e$ is of the form $u_j$ with $j>i+1$. Now, consider the corresponding edge in $H\setminus L$. It contains only vertices from the set $\{ u_1,u_2,\ldots,u_{i+1} \}$ (that is, no vertices from $L$ or with a higher index than $i+1$). Since each of $u_1,\ldots,u_i$ gets deleted while constructing $C_{H\setminus L}$, $e$ eventually contains only the vertex $u_{i+1}$, so $u_{i+1}$ gets deleted. For the reverse implication, assume the core of $H\setminus L$ is empty and let $u_1,\ldots,u_k$ be an ordering of $V(H)\setminus L$ such that the earlier a vertex gets deleted (via Algorithm \ref{core_H}), the earlier it appears in the list (with ties broken arbitrarily). We will prove that each of the vertices $u_i$ burn in $H$ when $L$ is burned as a lazy burning set by induction on $i$. For the base case, since $u_1$ was deleted from $H\setminus L$ in the first step of the algorithm, it must belong to some singleton edge $e$ in $H\setminus L$. Consider the corresponding edge in $H$. If it is still a singleton (that is, it contains no vertices from $L$), then $u_1$ burns via spontaneous combustion. Otherwise, the corresponding edge in $H$ only contains $u_1$ as well as vertices from $L$, so $u_1$ burns via propagation. Now, assume each of $u_1,u_2,\ldots,u_i$ is burned. Consider the edge $e$ that contains $u_{i+1}$ and causes it to be deleted from $H\setminus L$ (via Algorithm \ref{core_H}). Denote the corresponding edge in $H$ by $e^\prime$. If $e^\prime$ is also a singleton, then $u_{i+1}$ burns via spontaneous combustion. Otherwise, $e^\prime$ becomes the singleton edge $e$ (containing only $u_{i+1}$) after deleting $L$ and a series of vertex deletions via the algorithm. Suppose $e^\prime$ contains vertices of the form $u_j$ with $j>i+1$. For this edge to become a singleton containing only $u_{i+1}$, $u_j$ must be deleted strictly before $u_{i+1}$, which contradicts our ordering of $V(H)\setminus L$. Hence, $e^\prime$ is an edge in $H$ that contains only vertices from the set $L\cup\{ u_1,\ldots, u_{i+1}\}$. Observe that $u_{i+1}\in e^\prime$. Since each of $u_1,\ldots,u_i$ eventually burns, at some point in the lazy burning process $e^\prime$ contains only burned vertices and $u_{i+1}$, and hence $e^\prime$ causes $u_{i+1}$ to burn via propagation. \end{proof} \textcolor{blue}{Use above corollary (whose proof is very similar to the above thm) in complexity result!} Note that the lazy burning number of $C_H$ is the same regardless of whether or not spontaneous combustion is allowed, as $C_H$ contains no singletons. \begin{corollary} \label{no_SC_NP_comp} Let $H$ be a hypergraph. The decision problem \emph{``Is $b_L(H)\leq k$?''} is NP-complete when spontaneous combustion is not allowed. \end{corollary} \begin{proof} Consider an arbitrary hypergraph $H$. If $H$ has no singleton edges, then this decision problem is equivalent to the one from Theorem~\ref{SC_NP_complete}, so assume $H$ has singleton edges. We will show that a solution to the original model of lazy burning yields a solution to lazy burning with spontaneous combustion on $H$, and hence, the original model is ``at least as hard.'' The proof will follow, as the original model of lazy burning is known to be in NP \cite{burgess2024extendinggraphburninghypergraphs}. Given $H$, we first compute $C_H$, which notably only takes polynomial time. Let $L$ be an optimal lazy burning set for $C_H$ in the original model. We then note that $L$ is also an optimal lazy burning set for $C_H$ when spontaneous combustion is allowed. Finally, by Theorem~\ref{core_bl_equality} and its proof, $L$ is an optimal lazy burning set for $H$ when spontaneous combustion is allowed. \end{proof} \textcolor{blue}{Caleb: is the above proof formal enough? Maybe use the following proof instead: Suppose we wish to answer the question ``is $b_L(H)\leq k$?'' (with spontaneous combustion), and we have the answer to the question ``is $b_L(C_H)\leq k$?'' (without spontaneous combustion). If $b_L(C_H)\leq k$, then $b_L(H)\leq k$ (with spontaneous combustion) since any lazy burning set for $C_H$ is a lazy burning set for $H$ (see the proof of Theorem~\ref{core_bl_equality}). Otherwise, $b_L(C_H)> k$ (without spontaneous combustion). Suppose $b_L(H)\leq k$ (with spontaneous combustion). There exists a lazy burning set $L$ for $H$ of size at most $k$. By the same argument as the proof of Theorem~\ref{core_bl_equality}, $L\cap C_H$ is a lazy burning set for $C_H$ (without spontaneous combustion). Since $|L\cap C_H|\leq |L|\leq k$, we have that $b_L(C_H)\leq k$ (without spontaneous combustion), which is a contradiction. Thus, $b_L(H)> k$ (with spontaneous combustion), so a solution to ``is $b_L(C_H)\leq k$?'' yields a solution to the NP-complete decision problem from Theorem~\ref{SC_NP_complete}. Therefore, original lazy burning is NP-complete on the family of hypergraphs that are the cores of other hypergraphs. Since it is in NP \cite{burgess2024extendinggraphburninghypergraphs}, it is NP-complete in general. } \begin{theorem}\label{zero_set_equivalence} If $H$ is a hypergraph and $L\subseteq V(H)$, then $L$ is a lazy burning set for $H$ if and only if $L\cup E(H)$ is a zero forcing set for $\mathrm{IG}(H)$. \end{theorem} \begin{proof} For the forward implication, assume $L$ is a lazy burning set for $H$ and let $u_1,u_2,\ldots,u_k$ be an ordering of $V(H)\setminus L$ such that the earlier a vertex burns in the lazy burning process, the earlier it appears in the list (with ties broken arbitrarily). We claim that $L\cup E(H)$ is a zero forcing set in $\mathrm{IG}(H)$. In particular, we will prove that the vertices corresponding to $u_1,u_2,\ldots,u_k$ in $\mathrm{IG}(H)$ will become forced using induction on $i$. This will prove the claim, seeing as these are the only unforced vertices in $\mathrm{IG}(H)$ at the start of the game. For the base case, $u_1$ burns in the first propagation step because it belongs to some edge $e$ that contains no other unburned vertices. Hence, in $\mathrm{IG}(H)$, $e$ is a forced vertex whose only unforced neighbor is $u_1$, so $e$ forces $u_1$. Now assume each of $u_1,u_2,\ldots,u_i$ becomes forced in $\mathrm{IG}(H)$. Consider the edge $e$ that causes $u_{i+1}$ to burn in the lazy burning game. By the definition of our ordering, no vertex in $e$ is of the form $u_j$ with $j>i+1$. Hence, in $\mathrm{IG}(H)$, $e$ is a forced vertex whose open neighborhood is a subset of $\{ u_1,u_2,\ldots,u_{i+1}\}$ (and $eu_{i+1}\in E(\mathrm{IG}(H))$ is guaranteed). Since $u_{i+1}$ is unforced and each of $u_1,u_2,\ldots,u_i$ is forced, $u_{i+1}$ is the unique unforced neighbor of $e$. Hence, $e$ forces $u_{i+1}$. For the reverse implication, assume $L\cup E(H)$ is a zero forcing set for $\mathrm{IG}(H)$. Let $u_1,\ldots,u_k$ be an ordering of $V(H)\setminus L$ such that the earlier a vertex becomes forced, the earlier it appears in the list (with ties broken arbitrarily). We will prove that each of the vertices $u_i$ burns in $H$ when $L$ is burned as a lazy burning set by induction on $i$. For the base case, since $u_1$ becomes forced in the first time step, it must be the unique unforced neighbor of some vertex $e$, where $e$ belongs to the partite set of vertices that is indexed by $E(H)$. Consider the edge in $H$ corresponding to $e$ (and call it $e$ also for simplicity). We have that $u_1\in e$ in $H$. In $\mathrm{IG}(H)$, each neighbor of $e$ belongs to the zero forcing set except for $u_1$. Thus, in $H$, each vertex in $e$ belongs to the lazy burning set except for $u_1$, so $u_1$ burns via propagation. Now assume each of $u_1,u_2,\ldots,u_i$ is burned in $H$. Consider the vertex $e$ in $\mathrm{IG}(H)$ that forces $u_{i+1}$. Since $e$ is adjacent to $u_{i+1}$, $e$ must belong to the partite set in $\mathrm{IG}(H)$ whose vertices are indexed by $E(H)$. Also, each other neighbor of $e$ must be forced in a time step strictly earlier than $u_{i+1}$, so the open neighborhood of $e$ in $\mathrm{IG}(H)$ is a subset of $\{ u_1,u_2,\ldots,u_{i+1}\}$. Hence, in $H$, $e$ is an edge that is a subset of $\{ u_1,u_2,\ldots,u_{i+1}\}$. Since $u_{i+1}$ is the only unburned vertex in this set and $u_{i+1}\in e$, $u_{i+1}$ burns due to propagation within $e$. \end{proof} \begin{theorem} \label{inf_fam_tight_1} There exists an infinite family of hypergraphs for which the bound in Corollary~\ref{another_zero_bound} is tight. \end{theorem} \begin{proof} Consider the family of generalized stars constructed as follows. Given $k,\ell\geq 2$, define $H_{k,\ell}$ as the hypergraph with $\ell$ edges, each of which contains $k$ vertices of degree one, such that all edges intersect in a central vertex. More formally, $E(H_{k,\ell})=\{e_1,e_2,\ldots,e_\ell\}$ and $V(H_{k,\ell})=\{v,u_1^1,\ldots,u_k^1,\ldots,u_1^\ell,\ldots,u_k^\ell\}$ where $e_i=\{v,u_1^i,\ldots,u_k^i\}$ for each $i$. Note that $H_{k,\ell}$ is $(k+1)$-uniform. First, we show that $b_L(H_{k,\ell})=(k-1)\ell+1$. Any lazy burning set for $H_{k,\ell}$ must contain at least $k-1$ vertices of degree one in each edge. If any edge $e$ contains fewer than $k-1$ burned degree-one vertices at the start of the game, then that edge contains two or more unburned vertices of degree one. These vertices will never burn because $e$ will never contain exactly one unburned vertex. Hence, $b_L(H_{k,\ell})\geq(k-1)\ell$. However, if we choose $k-1$ degree-one vertices in each edge as their lazy burning set, then every edge will contain an unburned vertex of degree one, as well as $v$, which is also unburned, so no burning will spread. Hence, $b_L(H_{k,\ell})>(k-1)\ell$. Indeed, if we initially burn $k-1$ degree-one vertices per edge as well as $v$ then $H_{k,\ell}$ will burn in one round, so $b_L(H_{k,\ell})\leq(k-1)\ell+1$. Since any optimal lazy burning set for $H_{k,\ell}$ contains a degree-one vertex in each edge, the maximum number of non-adjacent vertices one can choose from an optimal lazy burning set is $|E(H_{k,\ell})|=\ell.$ Hence, for this family of hypergraphs, the upper bound from Corollary~\ref{another_zero_bound} is $$b_L(H_{k,\ell})+|E(H_{k,\ell})|-x=b_L(H_{k,\ell})+\ell-\ell=b_L(H_{k,\ell}).$$ We now consider the corresponding incidence graph $I_{H_{k,\ell}}$. Observe that $I_{H_{k,\ell}}$ may be viewed as a rooted tree, with $v$ as the root. The vertices $e_1$ through $e_\ell$ are all incident to $v$ and in the second layer of the tree, and the vertices $u_i^j$ are in the third layer of the tree (and they are all leaves). Now that $e_j$ is adjacent to each vertex $u_1^j,u_2^j,\ldots,u_k^j$. We will prove that $z(I_{H_{k,\ell}})=(k-1)\ell+1$, and the proof will be complete. First, for each vertex $e_j$, at least $k-1$ of the $k$ leaves incident to it must be initially forced. Otherwise, there are two or more unforced leaves incident to $e_j$, and hence $e_j$ will never have a unique unforced neighbor. Thus, the unforced leaves incident to $e_j$ will never become forced, so $z(I_{H_{k,\ell}})\geq (k-1)\ell$. Indeed, if we force exactly $k-1$ leaves per vertex $e_j$ (and force no other vertices), then the vertices $e_j$ will all immediately become forced. However, no more forcing will occur, as each vertex $e_j$ will be incident to two unforced vertices; $v$ and a leaf. Thus, $z(I_{H_{k,\ell}})> (k-1)\ell$. Indeed, if one forces $k-1$ leaves per vertex $e_j$ as well as $v$, then $I_{H_{k,\ell}}$ will become completely forced in two steps, so $z(I_{H_{k,\ell}})\leq (k-1)\ell+1$ and the proof is complete. \end{proof} \begin{proof} Let $L$ be an optimal lazy burning set for $H$, and let $S$ be the corresponding zero forcing set of size $b_L(H)+|E(H)|$ for $\mathrm{IG}(H)$. Denote the chosen vertices in $L$ by $v_1,v_2,\ldots,v_x$ and let $e_1,e_2,\ldots,e_x$ be a list of edges in $H$ such that $v_i\in e_i$ for each $i$. Observe that the neighborhoods of the vertices $v_i$ in $\mathrm{IG}(H)$ are disjoint. Hence, in $\mathrm{IG}(H)$, the vertices $v_i$ and $e_i$ are adjacent, and if $i\neq j$, then the vertices $v_i$ and $e_j$ are not adjacent. We claim that $S\setminus\{e_1,e_2,\ldots,e_k\}$ is a zero forcing set in $\mathrm{IG}(H)$. Each $v_i$ is part of the initially forced set, and $e_i$ is the only neighbor of $v_i$ that has been removed from the forcing set. As every neighbor of $v_i$ in $\mathrm{IG}(H)$ is forced except for $e_i$, $v_i$ will immediately force $e_i$. The proof follows, as $S$ is a forcing set for $\mathrm{IG}(H)$. \end{proof}
2412.04579v1
http://arxiv.org/abs/2412.04579v1
Solvable families of random block tridiagonal matrices
\documentclass[12pt]{article} \title{Solvable families of random block tridiagonal matrices} \date{} \author{Brian Rider and Benedek Valk\'o} \oddsidemargin 0in \topmargin 0in \headheight 0in \headsep 0in \textheight 9in \textwidth 6.7in \renewcommand{\baselinestretch}{1.3} \usepackage{amsfonts,color} \usepackage{graphicx} \usepackage{amsmath} \usepackage{amsthm} \usepackage{amssymb, url} \usepackage{hyperref} \newtheorem{theorem}{Theorem} \newtheorem{lemma}[theorem]{Lemma} \newtheorem{proposition}[theorem]{Proposition} \newtheorem{corollary}[theorem]{Corollary} \newtheorem{claim}[theorem]{Claim} \newtheorem{fact}[theorem]{Fact} \newtheorem{conjecture}[theorem]{Conjecture} \theoremstyle{definition} \newtheorem{definition}[theorem]{Definition} \newtheorem{remark}[theorem]{Remark} \newtheorem{example}[theorem]{Example} \newtheorem{algorithm}[theorem]{Algorithm} \newtheorem{examples}[theorem]{Examples} \newcommand{\eps}{\varepsilon} \newcommand{\Z}{{\mathbb Z}} \newcommand{\ZZ}{{\mathbb Z}} \newcommand{\FF}{{\mathbb{F}}} \newcommand{\UU}{{\mathbb U}} \newcommand{\R}{{\mathbb R}} \newcommand{\CC}{{\mathbb C}} \newcommand{\ud}{{\mathbb U}} \newcommand{\Rnn}{{\R_{\geq 0}}} \newcommand{\N}{{\mathbb N}} \newcommand{\cP}{{\mathcal P}} \newcommand{\cC}{{\mathcal C}} \newcommand{\ev}{{\rm E}} \newcommand{\pr}{\mbox{\rm P}} \newcommand{\lstar}{{\raise-0.15ex\hbox{$\scriptstyle \ast$}}} \newcommand{\ldot}{.} \newcommand{\vfi}{\varphi} \newcommand{\cN}{\mathcal{N}} \newcommand{\var}{\text{Var }} \newcommand{\mat}[4]{\left( \begin{array}{cc} #1 & #2 \\ #3 & #4 \\ \end{array} \right)} \theoremstyle{remark} \newcommand{\Balpha}{\underline{\alpha}} \newcommand{\Btheta}{\underline{\theta}} \newcommand{\Blambda}{\underline{\lambda}} \newcommand{\Bq}{\underline{q}} \newcommand{\Bx}{\underline{x}} \newcommand{\By}{\underline{y}} \newcommand{\Ba}{\underline{a}} \newcommand{\Bb}{\underline{b}} \newcommand{\zz}{\mathbb{Z}} \newcommand{\cc}{\mathbb{C}} \newcommand{\rr}{\mathbb{R}} \newcommand{\ind}{{\bf{1}}} \newcommand{\cB}{\mathcal{B}} \newcommand{\cZ}{\mathcal{Z}} \newcommand{\cF}{\mathcal{F}} \newcommand{\cW}{\mathcal{W}} \newcommand{\cS}{\mathcal{S}} \newcommand{\cT}{\mathcal{T}} \newcommand{\cM}{\mathcal{M}} \newcommand{\cFF}{\widetilde {\mathcal{F}}} \newcommand{\cL}{\mathcal{L}} \newcommand{\qq}{\mathbb{Q}} \newcommand{\hh}{\mathbb{H}} \newcommand{\oo}{\mathbb{O}} \newcommand{\cX}{\mathcal{X}} \newcommand{\re}{\text{Re}} \newcommand{\sech}{\text{ sech }} \newcommand{\Tr}{\textup{Tr}} \def\eqd{\stackrel{d}{=}} \newcommand{\la}{\langle} \newcommand{\ra}{\rangle} \newcommand{\sgn}{\operatorname{sgn}} \newcommand{\Pf}{\operatorname{Pf}} \newcommand{\Hf}{\operatorname{Hf}} \newcommand{\ww}{\boldsymbol\omega} \newcommand{\nn}{\boldsymbol\eta} \newcommand{\cA}{\mathcal{A}} \newcommand{\cG}{\mathcal{G}} \newcommand{\cD}{\mathcal{D}} \newcommand{\dd}{\Theta} \newcommand{\T}{\dag} \newcommand{\lst}[1]{[\![#1 ]\!]} \newcommand{\nint}[2]{\lfloor #1 \rfloor_{#2}} \newcommand{\nfr}[2]{\left\{ #1 \right\}_{#2}} \newcommand{\mbf}[1]{\mathbf{#1}} \newcommand{\wt}[1]{\widetilde{#1}} \newcommand{\HH}{\mathtt{H}_{\beta, n}} \newcommand{\WW}{\mathtt{W}_{\beta, n,m}} \newcommand{\SQW}{\mathtt{SqW}_\beta} \newcommand{\benedek}[1]{\textcolor{red}{#1}} \newcommand{\brian}[1]{\textcolor{blue}{#1}} \bibliographystyle{plain} \begin{document} \maketitle \abstract{We introduce two families of random tridiagonal block matrices for which the joint eigenvalue distributions can be computed explicitly. These distributions are novel within random matrix theory, and exhibit interactions among eigenvalue coordinates beyond the typical mean-field log-gas type. Leveraging the matrix models, we go on to describe the point process limits at the edges of the spectrum in two ways: through certain random differential operators, and also in terms of coupled systems of diffusions. Along the way we establish several algebraic identities involving sums of Vandermonde determinant products. } \section{Introduction} Trotter observed that if one applies the Householder tridiagonalization process to a GOE or GUE random matrix then the resulting real symmetric tridiagonal matrix will have independent entries (up to symmetry) with normal and chi distributions \cite{Trotter}. In \cite{DE} Dumitriu and Edelman presented a far reaching generalization of this result. They show that, for any $\beta > 0$, the $ n \times n$ random Jacobi matrix with independent $N(0,\frac{2}{\beta})$ random variables along the diagonal, and independent $ \frac{1}{\sqrt{\beta}} \chi_{\beta(n-1)}, \frac{1}{\sqrt{\beta}} \chi_{\beta(n-2)}, \dots, \frac{1}{\beta} \chi_\beta$ random variables along the off-diagonals, has joint eigenvalue density proportional to: \begin{equation} \label{eig_DE} \left|\Delta(\lambda)\right|^\beta e^{-\frac{\beta}{4} \sum_{j=1}^n \lambda_j^2}. \end{equation} Here $\Delta(\lambda)$ denotes the usual Vandermonde determinant of the eigenvalues. This includes Trotter's result for GOE or GUE upon setting $\beta=1$ or $2$. The Dumitriu-Edelman model for the Gaussian, or ``Hermite", beta ensemble, along with their Laguerre counterparts, initiated an immense amount of activity in the study of the scaling limits of beta ensembles. See for instance, \cite{ES}, \cite{KillipNenciu}, \cite{RRV}, \cite{RR}, \cite{KS}, \cite{BVBV}, \cite{KRV}, and \cite{BVBV_sbo}. Motivated both by the original construction of \cite{DE} along with its ensuing impact, here we establish two families of similarly solvable block-tridiagonal matrix models. Let $\HH (r,s)$ denote the distribution of the $rn \times rn$ symmetric or Hermitian block tridiagonal matrix with $r \times r$ diagonal blocks distributed as independent copies of G(O/U)E, and descending upper diagonal blocks distributed as independent copies of the (lower triangular) positive square root of a real/complex Wishart with parameters $(r, (r+s)(n-i))$. Here $i$ is the index of the offdiagonal block entry, and $\beta=1$ and 2 corresponds to the real and complex case, respectively. As in the $r=1$ case, the diagonal and and offdiagonal variables are also independent of each other. A more detailed description of these ensembles is provided in Section \ref{subs:matrix_distr}. Note of course that the Wishart distribution is the natural multivariate analog of the $\chi^2$ distribution, and that $\HH(1,s)$ is just the original Dumitriu-Edelman model, after a reparameterization. Further, when $s=0$, our model may in fact be arrived by a suitable block tridiagonalization procedure of the corresponding $rn \times rn$ G(O/U)E, {\`a} la Trotter. This has already been noticed in \cite{Spike2} in the context of eigenvalue spiking. Finding a suitable general beta version of the spiked Tracy-Widom laws introduced in that paper was another motivation for our work. Our main result is: \begin{theorem} \label{thm:main} For $\beta =1$ and $2$, the symmetrized joint eigenvalue density of $\HH(r,s)$ can be computed explicitly in the following cases: \begin{align} \label{density1} \frac{1}{Z_{n, \beta, r, 2}} |\Delta({\lambda})|^{\beta} \left( \sum_{(\mathcal{A}_1,\dots,\mathcal{A}_r)\in \cP_{r,n}} \prod_{j=1}^r \Delta(\cA_j)^2 \right) e^{- \frac{\beta}{4}\sum_{i=1}^{rn} \lambda_i^2}, \quad \mbox{ for } r \ge 2, \ \beta s=2, \end{align} and \begin{align} \label{density2} \frac{2^n}{Z_{n, \beta, 2, \beta s}} \Delta({\lambda})^{\beta+\frac{\beta s}{2}} \left|\Pf \left(\frac{{\bf{1}}_{i \neq j}}{\lambda_i -\lambda_j} \right)\right|^{\frac{\beta s}{2}} e^{- \frac{\beta}{4}\sum_{i=1}^{2n}\lambda_i^2} \quad \mbox{ for } r = 2, \ \beta s = 2,4. \end{align} It further holds that \begin{align*} &Z_{n, \beta, r, \beta s} =(n r)! (2\pi)^{\frac{nr}{2}} \left(\tfrac{\beta}{2}\right)^{a_{n,\beta,r,s}} \Gamma\left(\tfrac{\beta}{2}\right)^{-nr} \prod_{k=1}^{nr} \Gamma\left(\tfrac{\beta}{2}\left(k+s \lceil\tfrac{k}{r}\rceil\right)\right) \times \begin{cases} 1, \quad &\beta s=2,\\ (\beta/12)^n, \quad &\beta s=4, \end{cases} \end{align*} with $a_{n,\beta,r,s}=%-\frac{\beta}{4} n r (n (r+s)+s-1)-\frac{nr}{2}= -\frac{\beta}{4} n r (n (r+s)+s)+\left(\tfrac{\beta}{4}-\tfrac{1}{2}\right){nr}$ for all $n$, $\beta = 1$ and $2$, and combinations of $r$ and $s$ in \eqref{density1} and \eqref{density2}. \end{theorem} Here for $r\ge 2$ and $n\ge 1$, $\cP_{r,n}$ denotes the set of size $r$ equipartitions of $\lst{rn} := \{ 1,2, \dots rn\}$. That is, $\{\cA_1, \dots\cA_r\}\in \cP_{r,n}$ if $|\cA_i|=n$ for all $i$ and the $\cA_i$ form a partition of $\lst{rn}$. With that, for any $\cA \subset \lst{rn}$, we write $\Delta(\cA)$ as shorthand for the Vandermonde determinant in the $|\cA|$ ordered eigenvalue variables with indices drawn from $\cA$ (suppressing the explicit dependence on $\lambda_i, i \in \cA$). Finally, $\Pf(M)$ denotes the Pfaffian of $M$. In both \eqref{density1} and \eqref{density2} we see novel types of interactions among the points beyond the usual $|\Delta({\lambda})|$ to some power. The formulas for the overlapping $r=2$, $\beta s = 2$ cases are shown to agree by a Pfaffian/Vandermonde identity, see Lemma \ref{lem:det4_identities} below. This is one of several identities involving sums of powers of Vandermonde determinants that we prove in Section \ref{sec:det_identities}. We also note that \eqref{density1} is consistent with \eqref{eig_DE} upon taking $r=1$, as then the sum over equipartitions reduces to $\Delta(\lambda)^2 = \Delta(\lambda)^{\beta s}$. One might anticipate that the form of the $r=2$ family should generalize to all even integer $\beta s$. However, computer assisted calculations for small $n$ values indicate that the Pffafian structure in \eqref{density2} breaks down for $\beta s=6$. Understanding what happens for larger block size $r$ beyond $\beta s=2$ also remains open. Our difficulty in extending exact formulas to either parameter regime is tied to our approach to proving Theorem \ref{thm:main}. This rests on computing the absolute $\beta s$-moment of a certain structured determinant over the Haar distributed Orthogonal or Unitary group (in dimension $rn$). We do this by expansion and re-summation, the underlying complexity of which grows in both $r$ and $\beta s$. In another direction, our block model could certainly be constructed using quaternion ingredients, leading to $\HH(r,s)$ with $\beta=4$. The non-commutativity of the quaternion variables poses additional technical challenges in extending Theorem \ref{thm:main} to that setting, though we expect these are not insurmountable. Next, a natural question is whether densities of the form \eqref{density1} or \eqref{density2} appear ``in the wild". In fact, the $r=2$ family bears close resemblance to what is known as the Moore-Read, or Pfaffian, state for the fractional quantum Hall effect, see \cite{MR_1991}. In that theory the points lie in the complex plane, so \eqref{density2} might be viewed as a one-dimensional caricature of these states in the same way that the Gaussian (and other) beta ensembles are one-dimensional caricatures of a true coulomb gas. The eigenvalues of random block matrices have of course been studied in a number of capacities, most notably perhaps as structured band matrices connected to the Anderson or Wegner orbital models, see for example \cite{SchSch} and the references therein. Motivated by the theory of matrix orthogonal polynomials, \cite{Dette1} and \cite{Dette2} introduce families of ``block beta" Hermite, Laguerre and Jacobi ensembles built out of Gaussian and/or $\chi$ variables, and study their limiting density of states. The large deviations of related ensembles have been considered in \cite{Rouault1} and \cite{Rouault2}. Our work though is the first to provide a systematic approach to finding solvable block models. We close the introduction with descriptions of: (i) the soft edge asymptotics for $\HH(r,s)$, and (ii), how the results stated through that point, including the associated asymptotics, extend to a family of block Wishart (or Laguerre) ensembles. After this, Section 2 lays out some basic facts on the spectral theory of block tridiagonal matrices along with the detailed definitions of our various matrix models. Section 3 provides an overview of the eigenvalue density derivations, identifying a certain moment calculation as fundamental (see Theorem \ref{thm:moment}). That calculation is spread over Sections 4 and 5, for moments $\beta s =2$ and $\beta s = 4$ respectively. Section 6 establishes a number of identities (and presents a conjecture in a related spirit) involving sums of Vandermonde determinant powers required in the preceding. Finally, Section 7 is devoted to asymptotics. \subsection{Soft edge asymptotics of $\HH(r,s)$} While it does not appear possible to compute correlations directly from the formulas \eqref{density1} or \eqref{density2}, the random operator approach is available. In the block setting this was developed by Bloemendal and Vir\'ag for the soft edge in \cite{Spike2}, and their approach applies to our case for any values of $r$ and $s$. In fact, it even applies in the $\beta=4$ case where we do not have statements about the joint eigenvalue densities. Introduce the $\beta =1,2,$ or $4$ matrix Brownian motion $B_x$ in dimension $r$: the independent, stationary increment process for which $B_y- B_x \sim B_{y-x}$ is distributed as $\sqrt{y-x}$ times a copy of $r \times r$ G(O/U/S)E. Next, for $\gamma > 0$, bring in the differential operator acting on $r$-dimensional vector valued functions on $\R_{+}$, \begin{equation}\label{eq:H_op} \mathcal{H}_{\beta, \gamma} = - \frac{d^2}{dx^2} + rx + \sqrt{\frac{2}{\gamma}} B'_x. \end{equation} When $\gamma=1$ this is the multivariate Stochastic Airy Operator of \cite{Spike2}. In particular, with a Dirichlet boundary condition at the origin, the spectrum of $-\mathcal{H}_{\beta} = -\mathcal{H}_{\beta, 1}$ is given by the $\operatorname{Airy}_\beta$ process, the edge scaling limit of the Gaussian beta ensemble. The largest value of this process (which is minus the ground state eigenvalue of $\mathcal{H}_{\beta}$), has classical Tracy-Widom distribution $TW_\beta$ with $\beta =1,2, 4$. \begin{theorem} \label{thm:limit_op} For any $r, s$ and $\beta=1,2,4$, let $\mathbf{T}_n \sim \HH(r,s)$. Denote by $\lambda_0^{(n)} < \lambda_1^{(n)} < \cdots $ the eigenvalues of the renormalized \begin{equation*} \mathbf{H}_n = \gamma^{-1/2} (rn)^{1/6} \Bigl(2 \sqrt{(r+s)n} {I}_{rn} - \mathbf{T}_n \Bigr), \end{equation*} and by $\Lambda_0 < \Lambda_1 < \cdots$ the Dirichlet eigenvalues of $ \mathcal{H}_{\beta, \gamma}$ with the choice $\gamma = \frac{r+s}{r}$ . Then the point process $\{ \lambda_0^{(n)} ,\lambda_1^{(n)} , \dots\}$ converges in law to $\{\Lambda_0, \Lambda_1, \dots \} $ as $n\to \infty$. \end{theorem} The proof of Theorem \ref{thm:limit_op} follows that of the main result of \cite{Spike2}, though we sketch an overview of the ideas in Section \ref {sec:asymptotics}. Similarly, Theorem 1.5 of \cite{Spike2} provides a second description of the limiting point process $\{ \Lambda_i \}_{i \ge 0}$ via matrix oscillation theory. Applying the same here yields: \begin{corollary} \label{cor:osc} Define the measure $\mathbb{P}$ on paths $\mbf{p}=(p_1, \dots p_r):[0,\infty) \mapsto ( -\infty, \infty]$ induced by the stochastic differential equation system \begin{equation} \label{mult_sde} dp_i = \frac{2}{\sqrt{\beta \gamma}} db_i + \left(\lambda + rx - p_i^2 + \sum_{j \neq i} \frac{2}{p_i - p_j} \right)dx,\qquad 1\le i \le r, \end{equation} starting from $(p_1(0), \cdots , p_r(0)) = \{\infty\}^r$ and entering $\{ p_1 < \cdots < p_r\}$ at $x>0$. Here $(b_1, \cdots b_k)$ is a standard real $r$-dimensional Brownian motion; $p_1$ can hit $-\infty$ in finite time, whereupon it is placed at $+\infty$ and the re-indexed process starts afresh. Then with $\Lambda_0< \Lambda_1< \cdots $ defined as in Theorem \ref{thm:limit_op}, it holds that \begin{align} P( \Lambda_k \le \lambda ) = \mathbb{P} ( x\mapsto \mbf{p}(x) \mbox{ explodes at most } {k} \mbox{ times } ) \end{align} for all $k \ge 0$. \end{corollary} The above corollary immediately implies that, whenever $\beta \gamma$ equals a classical value, {\em{i.e.}} $1,2,$ or $4$, we can deduce that the limiting edge point process corresponds to that of the G(O/U/S)E. In particular, in this case $\Lambda_0$ will have $TW_{\beta \gamma}$ distribution. This again is one of the primary take-aways of \cite{Spike2}. Due to the equivalence of the pre-limit models across different values of $r$, it is known that, again when the diffusion parameter is classical, the explosion times of \eqref{mult_sde} are equal in law for all $r\ge 1$. No direct proof of this striking fact is known. Specifying to the cases for which we have novel explicit joint eigenvalue densities, this implies: \begin{corollary} \label{cor:betalimit} Consider the random point process defined by the $r=2$, $\beta s = 2$ joint density \eqref{density1} in Theorem \ref{thm:main}. When $\beta=1$, the appropriately rescaled point process converges in law to the $\operatorname{Airy}_2$ point process. In the case of $r=2$ and $\beta s= 4$ the appropriately scaled process determined by \eqref{density2} in Theorem \ref{thm:main}converges in law to the $\operatorname{Airy}_4$ point process when $\beta=2$. In particular, in these cases the largest eigenvalues (after rescaling) converge to the classical $TW_2$ and $TW_4$ distributions, respectively. \end{corollary} Conjecturing that the $r$-fold diffusion characterization of Corollary \ref{cor:osc} provides the description of the $\operatorname{Airy}_{\beta \gamma}$ process for any $\beta \gamma>0$ we arrive to the following. \begin{conjecture} \label{con:betalimit} More generally, the point process scaling limit of \eqref{density1} is distributed as $\operatorname{Airy}_{\beta+2/r}$ for all $r \ge 2$ and $\beta =1$ or $2$. In the case of \eqref{density2} with $\beta s = 4$ and $\beta=1$, the point process scaling limit is $\operatorname{Airy}_{3}$. \end{conjecture} \subsection{Block Laguerre ensembles} In \cite{DE} the authors also produce $\beta$ generalizations of the classical Laguerre (Wishart) ensemble, showing that there is an $n\times n$ tridiagonal matrix model built out of independent $\chi$ variables for which the eigenvalue density is proportional to \begin{equation} \label{eig_DE1} \left|\Delta(\lambda)\right|^\beta \prod_{i=1}^n \lambda_i^{\frac{\beta}{2}(m-n+1) -1} e^{-\frac{\beta}{2} \sum_{i=1}^n \lambda_i} \mathbf{1}_{\R_+^n}. \end{equation} When $\beta =1$ or $2$ this coincides with that of the law of a sample covariance matrix for $m\ge n$ independent real or complex normal samples in dimension $n$. Along with $\beta$ now taking any positive value, the model behind \eqref{eig_DE1} allows $m$ to be generalized to any real number greater than $n-1$. We define the distribution $\mathtt{W}_{n,m, \beta}(r, s)$ on nonnegative definite block tridiagonals as follows. Let $\mathbf{L}_n$ be an $rn \times rn$ block bidiagonal matrix with independent $r\times r$ diagonal and upper offdiagonal blocks denoted by $\{\mbf{D}_i\}_{i=1,n}$ and $\{\mbf{O}_i\}_{i=1, n-1}$, that are lower and upper triangular matrices, respectively. Distribute these according to square-root Wishart matrices with parameters $(r, (r+s)(m+1 -i))$ and $(r, (r+s)(n-i))$, respectively. Then $\mathtt{W}_{n, ,m, \beta}(r, s)$ has the law $\mbf{L}_n \mbf{L}_n^\dagger$. Full details are provided in Definition \ref{def:BlockW}. Again, when $s=0$ this model has been considered previously in \cite{Spike2} and \cite{RR} in connection to eigenvalue spiking. In that case the underlying random matrix $\mbf{L}_n$ arises from an explicit block bi-diagonalization of an $rn \times rm$ matrix of independent Gaussians. Effectively the same considerations behind Theorem \ref{thm:main} imply the following. \begin{theorem}\label{thm:main_W} The joint eigenvalue density of $\mathtt{W}_{n, m, \beta}(r, s)$ for $\beta=1$ or $2$ has the form \eqref{density1} for general $r\ge 2$ and $\beta s=2$ and \eqref{density2} for $r =2$ and $\beta s =2$ or $4$ with an explicitly computable normalizing constant, the only change being that the Gaussian weight $ e^{-\frac{\beta}{4} \sum_{i=1}^{rn} \lambda_i^2}$ is replaced by $ \prod_{i=1}^{rn} \lambda_i^{\frac{\beta}{2}( (r+s)(m-n)+1)-1} e^{-\frac{\beta}{2} \lambda_i}$, restricted to $\R_{+}^{rn}$. \end{theorem} In terms of asymptotics, we focus on the choice $m = n +a $ for fixed $a > -1/(r+s)$ as $n \rightarrow \infty$ and look at the scaling limit of the smallest eigenvalues, which end up being in the vicinity of the origin. This is the random matrix hard edge, and introduces novel limiting phenomena beyond what we have seen for $\mathtt{H}_{n, \beta}(r, s)$. Note that it may proved along the same lines to Theorem \ref{thm:limit_op} that the suitably centered and scaled largest eigenvalues under $\mathtt{W}_{n, m, \beta}(r, s)$ will converge to those of $\mathcal{H}_{\beta, \gamma}$, for an appropriate $\gamma$, and the same is in fact true for the smallest eigenvalues when $\liminf_{n\to \infty} m/n>1$. For the hard edge, the characterizing limit operator is now of Sturm-Liouville type: again acting on $r$-dimensional vector valued functions, \begin{equation} \label{matrixgenerator} \mathcal{G}_{\beta, \gamma} = - e^{rx} \, {\bf{Z}_x} \frac{d}{dx} {\mbf{Z}_x^{-1} } \frac{d}{dx}. \end{equation} Here $x \mapsto {{\mbf{Z}}_x} $ is a symmetrized version of drifted Brownian on the general real or complex linear group dimension $r$, the parameters $\gamma$ and $a$ coefficients of the defining stochastic differential equation (see \eqref{WandA} below). Similar to $\mathcal{H}_{\beta, \gamma}$, the operator $\mathcal{G}_{\beta, \gamma}$ for $\gamma =1$ has previously been shown to characterize multi-spiked hard edge laws \cite{RR2} for $\beta =1,2,4$. For $\gamma=1$ and $r=1$ this is the Stochastic Bessel Operator introduced by Ram\'{\i}rez and Rider in \cite{RR}. In analogy with Theorem \ref{thm:limit_op} and Corollary \ref{cor:osc}, we have: \begin{theorem} \label{thm:limit_op1} For $\mbf{W}_n \sim \mathtt{W}_{ n, n+a, n}(r, s)$ denote by $0 < {\lambda}_0^{(n)} < {\lambda}_1^{(n)} < \cdots $ the point process of eigenvalues of $ \frac{rn}{\gamma} \, \mbf{W}_n$. As $n \rightarrow \infty$ this converges in law to the point process $0 < \hat{\Lambda}_0< \hat{\Lambda}_1 <\cdots $ of Dirichlet eigenvalues of $ \mathcal{G}_{\beta, \gamma}$ with $\gamma = \frac{r+s}{r}$. \end{theorem} The dependence on the many underlying parameters is made more explicit in the Riccati picture. \begin{corollary} \label{cor:osc1} Let $\mathbb{P}$ be the measure on (non-intersecting) paths ${\mathbf{q}}: [\mu, \infty) \mapsto [-\infty, \infty]^r$ defined by \begin{equation} \label{rrq} d q_{i} = \frac{2}{\sqrt{\beta \gamma}} q_{i} db_i + \left( \left(\frac{a}{\gamma} + \frac{2}{\beta \gamma}\right) q_{i} - q_{i}^2 - e^{-r x} + q_{i} \sum_{j \neq i} \frac{ q_{i} + q_{j}}{ q_{i}- q_{j} } \right) dx, \end{equation} started from $\{ \infty\}^r$ with the same ordering and re-indexing conventions upon possible passages to $-\infty$ described in Corollary \ref{cor:osc}. With $0 < \hat{\Lambda}_0< \hat{\Lambda}_1 <\cdots $ defined in Theorem \ref{thm:limit_op1} it holds \begin{equation} \label{HardEdge_zeros} P (\hat{\Lambda}_k > \lambda) = \mathbb{P} ( x \mapsto \mbf{q}(x) \mbox{ vanishes at most } k \mbox{ times } ) \end{equation} for any given $k = 0,1,\dots$. \end{corollary} And again, whenever $\beta \gamma = 1, 2$ or $4$ we conclude that the point process scaling limit of the smallest eigenvalues of ${\mathtt{W}}_{n, \beta} (r, s)$ is the classical hard edge, or Bessel, point process. More generally, we conjecture that these limits are given by the general $\beta \gamma$ hard edge process defined in \cite{RR}. In particular, versions of Corollary \ref{cor:betalimit} and Conjecture \ref{con:betalimit} are readily formulated. We record these at the end of Section 7. Having dealt with the soft and hard edge scaling limit of our models, it is natural to ask if the same can be done in the bulk case. The analogous results to \cite{Spike2} and \cite{RR2} for the bulk have not though yet been developed. Another natural future direction is to extend our results to circular ensembles using the results of \cite{KillipNenciu} as a starting point. \medskip \noindent\textbf{Acknowledgements.} The authors thank Philippe Di Francesco for pointing out reference \cite{DSZ}. B.V.~was partially supported by the University of Wisconsin – Madison Office of the Vice Chancellor for Research and Graduate Education with funding from the Wisconsin Alumni Research Foundation and by the National Science Foundation award DMS-2246435. \section{Preliminaries} We start by outlining some basic facts on the spectral theory of block Jacobi matrices, then introduce the various distributions which we will work with. Throughout the paper we will use $\FF$ to denote $\R$ ($\beta=1$) or $\CC$ ($\beta=2$). In particular, we use $\FF$-hermitian and $\FF$-unitary for real symmetric/hermitian and orthogonal/unitary matrices. We use $\mbf{X}^\T$ to denote the transpose/conjugate transpose of an $\FF$-matrix $\mbf{X}$. \subsection{Block Jacobi matrices} We work with the following block generalization of tridiagonal Jacobi matrices. \begin{definition} Let $r, n\ge 1$. An $(rn)\times(rn)$ matrix $\mbf{T}$ is called an $\FF$-valued $r$-block Jacobi matrix if it is a $\FF$-hermitian block tridiagonal matrix built from $r\times r$ blocks satisfying the following conditions. The diagonal blocks $\mbf{A}_1, \dots, \mbf{A}_n$ are $r\times r$ $\FF$-hermitian matrices. The off-diagonal blocks $\mbf{B}_1, \dots, \mbf{B}_{n-1}$ above the diagonal are lower triangular with positive diagonal entries, see \eqref{eq:T}. We denote the set of such matrices by $\mathfrak{M}_{n,\beta, r}$. \begin{align}\label{eq:T} \mbf{T}= \left[\begin{array}{ccccc} \mbf{A}_1& \mbf{B}_1 & 0 &\dots & \\ \mbf{B}_1^{\dag} & \mbf{A}_2 &\mbf{B}_2 &\dots \\ 0&\ddots & \ddots & \ddots &0 \\ & 0 & \mbf{B}_{n-2}^\dag &\mbf{A}_{n-1} &\mbf{B}_{n-1} \\ & & 0 & \mbf{B}_{n-1}^\dag & \mbf{A}_n\\ \end{array} \right] \end{align} \end{definition} Note that an $r$-block Jacobi matrix can be viewed $(2r+1)$-diagonal band matrix with positive entries at the boundaries of the band. Let $\mbf{e}_{\lst{r}}=[\mbf{I}_r,\mbf{0}_{r\times (n-1)r}]^{\T}$ denote $(rn)\times r$ matrix built from the first $r$ coordinate vectors. (We do not explicitly denote the $n$-dependence.) The proof of the following theorem can be found for example in \cite{Spike2}, it relies on the Householder tridiagonalization algorithm in a block setting. \begin{theorem}[\cite{Spike2}]\label{thm:block_basic_1} Suppose that $\mbf{M}$ is an $\FF$-hermitian $rn\times rn$ matrix for which the matrix \begin{align}\label{eq:S1234} \mbf{S}=[\mbf{e}_{\lst{r}}, \mbf{M}\mbf{e}_{\lst{r}},\dots, \mbf{M}^{n-1}\mbf{e}_{\lst{r}}] \end{align} is invertible. Then there is an $\FF$-unitary matrix $\mbf{O}$ of the form $\mbf{I}_r\oplus \widetilde{\mbf{O}}$ and a unique $\mbf{T}\in \mathfrak{M}_{n,\beta, r}$, so that $\mbf{T}=\mbf{O}^{\T} \mbf{M} \mbf{O}$. The matrix $\mbf{O}$ can be chosen as the $\mbf{Q}$ in the unique QR decomposition $\mbf{S}=\mbf{Q}\mbf{R}$ for which $\mbf{R}$ has positive diagonal entries. \end{theorem} For $r=1$ the spectral measure of an $n\times n$ tridiagonal hermitian matrix $\mbf{T}$ with respect to the first coordinate vector $\mbf{e}_1$ is defined as the probability measure \begin{align}\label{eq:spec_m} \mu=\sum_{j=1}^n |\mbf{v}_{j,1}|^2 \delta_{\lambda_j}. \end{align} Here $\mbf{v}_{j,1}$ is the first coordinate of the normalized eigenvector corresponding to $\lambda_j$. Our next definition provides a natural extension of the spectral measure for $r$-block Jacobi matrices. \begin{definition} Suppose that $\mbf{M}$ is an $\FF$-hermitian $rn\times rn$ matrix. We define the spectral measure of $\mbf{M}$ with respect to $\mbf{e}_{\lst{r}}$ as the $r\times r$ matrix-valued measure \begin{align} \mu_{\lst{r}}=\sum_{j=1}^{rn} \mbf{v}_{j,\lst{r}} \cdot \mbf{v}_{j,\lst{r}}^{\T} \,\delta_{\lambda_j}. \end{align} Here $\mbf{v}_{j}$ is the normalized eigenvector corresponding to $\lambda_j$, and $\mbf{v}_{j,\lst{r}}\in \FF^r$ is the projection of $\mbf{v}_j$ to the first $r$ coordinates. \end{definition} Note that $\mu_{\lst{r}}$ only depends on the eigenspaces, so it is well-defined even though the choice of $\mbf{v}$ is not unique. If $\mbf{T}$ is the $r$-block Jacobi matrix obtained from an $\FF$-hermitian $\mbf{M}$ via Theorem \ref{thm:block_basic_1} then we have \begin{align} \int x^j d\mu_{\lst{r}}=\mbf{e}_{\lst{r}}^{\T} \mbf{M}^j \mbf{e}_{\lst{r}}= \mbf{e}_{\lst{r}}^{\T} \mbf{T}^j \mbf{e}_{\lst{r}}. \end{align} It can be shown that there is a one-to-one correspondence between the $r$-block Jacobi matrices and possible $r\times r$ matrix valued `probability' measures, see Section 2 of \cite{MOPUC}. \subsection{Random block matrices}\label{subs:matrix_distr} We start with an overview of the various distributions that serve as building blocks for our models, and then provide a precise definition of the $\HH(r,s)$ and $\WW(r,s)$ distributions. \begin{definition} The $\FF$-valued standard normal is denoted by $\FF N(0,1)$. The components are independent mean zero normals with variance $\frac{1}{\beta}$. The probability density function is proportional to $e^{-\frac{\beta}{2} |x|^2}$. \end{definition} We record the fact that if $\mbf{x}$ is a $d$-dimensional random vector with i.i.d.~$\FF N(0,1)$ entries then the distribution of $|\mbf{x}|$ is $\frac{1}{\sqrt{\beta}}\chi_{\beta d}$. The probability density function of $|\mbf{x}|$ is \[ 2\, \frac{ (\beta/2)^{\frac{\beta d}{2}}}{\Gamma(\beta d/2)} x^{\beta d-1} e^{-\frac{\beta}{2} x^2}. \] \begin{definition} Let $\mbf{Y}$ be an $n\times n$ matrix with i.i.d.~$\FF N(0,1)$ entries, and set $\mbf{X}=\frac1{\sqrt{2}} (\mbf{Y}+\mbf{Y}^{\T})$. The distribution of $\mbf{X}$ is called the $\FF$-valued Gaussian ensemble, or G$\FF$E$(n)$. For $\beta=1$ this is the Gaussian Orthogonal Ensemble (GOE), and for $\beta=2$ this is the Gaussian Unitary Ensemble (GOE). \end{definition} The diagonal entries of G$\FF$E are $N(0,\tfrac{2}{\beta})$ distributed, while the off-diagonal entries are i.i.d.~$\FF N(0,1)$. The entries are independent up to the real/hermitian symmetry. In the matrix variables the probability density function of G$\FF$E is proportional to $ e^{-\frac{\beta}{4} \Tr \mbf{X}\mbf{X}^{\T}}$. \begin{definition} Let $\mbf{Y}$ be an $n\times m$ (with $n\le m$) matrix with i.i.d.~$\FF N(0,1)$ entries. The distribution of the matrix $\mbf{X}=\mbf{Y}\mbf{Y}^T$ is called the $\FF$-valued Wishart distribution with parameters $(n,m)$. \end{definition} The following is a classical result in random matrix theory. \begin{theorem} The joint eigenvalue density of the $\FF$-valued $n\times n$ Gaussian ensemble is given by \eqref{eig_DE}. The distribution is called the Gaussian beta ensemble, and it is denoted by $G{\beta}E(n)$. The joint eigenvalue density of the $\FF$-valued Wishart distribution with parameters $(n,m)$ is given by \eqref{eig_DE1}. The distribution is called the Laguerre beta ensemble, and it is denoted by $L{\beta}E(n,m)$. In both cases the normalized eigenvectors can be chosen in a way so that the eigenvector matrix is Haar-distributed on the $n\times n$ $\FF$-unitary matrices while being independent of the eigenvalues. \end{theorem} \begin{definition} The $\FF$-valued square root Wishart matrix with parameters $n\le m$ is the distribution of the $n\times n$ lower triangular matrix $\mbf{X}$ with the following independent entries: \begin{align} x_{i,j}\sim \begin{cases} \FF N(0,1),& \qquad \text{if $i>j$},\\ \frac{1}{\sqrt{\beta}} \chi_{\beta (m+1-i)},& \qquad \text{if $i=j$},\\ 0,& \qquad \text{if $i<j$}. \end{cases} \end{align} We denote this distribution by $\SQW(n,m)$. \end{definition} We note that the joint probability density function of the non-zero entries of $\SQW(n,m)$ is proportional to \begin{align}\label{eq:SqW_pdf} \prod_{i>j} e^{-\frac{\beta}{2} |x_{i,j}|^2} \prod_{i=1}^n x_{i,i}^{\beta (m+1-i)-1} e^{-\frac{\beta}{2} x_{i,i}^2}=e^{-\frac{\beta}{2} \Tr \mbf{X}\mbf{X}^\T} \det(\mbf{X})^{\beta (m+1)-1} \prod_{i=1}^n x_{i,i}^{-\beta i}. \end{align} As the following classical result due to Bartlett \cite{Bartlett1933} shows, $\SQW(n,m)$ gives the distribution of the Cholesky factor of the Wishart distribution. \begin{theorem}[\cite{Bartlett1933}]\label{thm:bartlett} Suppose that the matrix $\mbf{X}$ has $\FF$-valued Wishart distribution with parameters $(n,m)$. Let $\mbf{R}$ be the lower triangular square root of $\mbf{X}$ with almost surely positive diagonal entries: $\mbf{X}=\mbf{R} \mbf{R}^{\T}$. Then $\mbf{R}$ has $\SQW(n,m)$ distribution. \end{theorem} We can now introduce the family of random block matrices that we study. \begin{definition} \label{def:BlockH} Let $r,n\ge 1$ and $s\ge 0$. We denote by $\HH(r,s)$ the distribution of the $\FF$-valued random $r$-block Jacobi matrix of size $(rn)\times(rn)$ with independent blocks $\mbf{A}_k, \mbf{B}_k$ where $\mbf{A}_k\sim$ G$\FF$E$(r)$ and $\mbf{B}_k\sim \SQW(r,(r+s)(n-k))$. \end{definition} Note that $\HH(1,0)$ is just the distribution of the tridiagonal matrix of Dumitriu and Edelman (and Trotter) given for the Gaussian beta ensemble. As the following theorem shows, for $r\ge 1$ the $\HH(r,0)$ distribution is the result of the $r$-block Householder process applied to G$\FF$E$(rn)$. \begin{theorem}[\cite{Spike2}]\label{thm:GFE_block} Let $\mbf{M}$ have G$\FF$E$(rn)$ distribution, and consider the matrix $\mbf{S}$ defined via \eqref{eq:S1234}. Then $\mbf{S}$ is a.s.~invertible, and the $r$-block Jacobi matrix $\mbf{T}$ produced by Theorem \ref{thm:block_basic_1} has $\HH(r,0)$ distribution. The eigenvalues of $\mbf{T}$ are distributed as $G\beta E(rn)$, and the normalized eigenvector matrix $\mbf{V}=[\mbf{v}_{i,j}]_{i,j\in \lst{rn}}$ can be chosen in a way so that the first $r$ rows of $\mbf{V}$ are independent of the eigenvalues and have the same distribution as the first $r$ rows of an $rn\times rn$ Haar $\FF$-unitary matrix. \end{theorem} Theorem \ref{thm:GFE_block} fully describes the distribution of the matrix valued spectral measure $\mu_{\lst{r}}$ of $\mbf{T}$. In particular, it shows that the weights and the support are independent of each other, and the weights can be obtained from a Haar $\FF$-unitary matrix. \begin{definition}\label{def:BlockW} Let $r,n\ge 1$, $m>-1/r$, and $s\ge 0$. Let $\mathbf{L}$ be an $rn \times rn$ block bidiagonal matrix with independent $r\times r$ diagonal and upper offdiagonal blocks denoted by $\{\mbf{D}_i\}_{i=1,n}$ and $\{\mbf{O}_i\}_{i=1, n-1}$ with $\mbf{D}_i^{\T}\sim \SQW(r,(r+s)(m+1-i))$ and $\mbf{O}_i\sim \SQW(r,(r+s)(n-i))$. We denote the distribution of $\mbf{W}=\mbf{L}\mbf{L}^{\T}$ by $\WW(r,s)$. \end{definition} Again, $\WW(1,0)$ is just the tridiagonal model given by Dumitriu and Edelman for the Laguerre beta ensemble. The analogue of Theorem \ref{thm:GFE_block} holds. \begin{theorem}[\cite{Spike2}]\label{thm:W_block} Let $\mbf{M}$ have $\FF$-valued Wishart distribution with parameters $(rn,rm)$, and consider the matrix $\mbf{S}$ defined via \eqref{eq:S1234}. Then $\mbf{S}$ is a.s.~invertible, and the $r$-block Jacobi matrix $\mbf{T}$ produced by Theorem \ref{thm:block_basic_1} has $\WW(r,0)$ distribution. The eigenvalues of $\mbf{T}$ are distributed as $L\beta E(rn,rm)$, and the normalized eigenvectors can be chosen in a way that the first $r$ rows are independent of the eigenvalues and have the same distribution as the first $r$ rows of an $rn\times rn$ Haar $\FF$-unitary matrix. \end{theorem} \section{New distributions via biasing} We start this section with a brief review of the Dumitriu-Edelman result \cite{DE}. We introduce the key tools for our block generalization and provide the proofs of our main theorems modulo a certain moment computation that is delayed to the subsequent sections. \subsection{Revisiting the Hermite beta ensemble} For completeness, we state the Dumitriu-Edelman result in full and provide a proof which foreshadows the techniques used to prove Theorem \ref{thm:main}. \begin{theorem}[\cite{DE}]\label{thm:DE} Fix $\beta>0$ and an integer $n\ge 1$. Let $a_1,\dots, a_n, b_1, \dots, b_{n-1}$ be independent random variables with $a_j\sim N(0,\tfrac{2}{\beta})$, $b_j\sim \frac{1}{\sqrt{\beta}}\chi_{\beta (n-j)}$. Then the symmetric tridiagonal matrix $\mbf{T}$ with diagonal $a_1,a_2,\dots$ and off-diagonal $b_1,b_2, \dots$ has a joint symmetrized eigenvalue density on $\R^n$ given by \ \begin{align}\label{eq:GbE} \frac{1}{Z_{n,\beta}} \left|\Delta(\lambda)\right|^\beta e^{-\frac{\beta}{4} \sum_{j=1}^n \lambda_j^2}, \end{align} with \begin{align}\label{eq:GbE_constant} Z_{n,\beta}={n!} (2\pi)^{n/2} (\beta/2)^{-\frac{\beta}{4}n(n-1)-\frac{n}{2}} \,\Gamma(\beta/2)^{-n} \prod_{j=1}^n \Gamma(\beta j/2). \end{align} Moreover, the spectral weights of $\mbf{T}$ corresponding to the first coordinate vector have Dirichlet$(\beta/2,\dots, \beta/2)$ joint distribution, and this weight vector is independent of the eigenvalues. \end{theorem} \begin{proof} Consider an $n\times n$ Jacobi matrix $\mbf{T}$ with diagonal entries $a_1,\dots, a_n$ and off-diagonal positive entries $b_1, \dots, b_{n-1}$. Denote by $p_j$ the spectral weight of $\lambda_j$ in the spectral measure \eqref{eq:spec_m}. It is well known that \begin{align}\label{eq:magic_Delta_p} |\Delta({\lambda})|= \prod_{k=1}^n p_k^{-1/2} \prod_{k=1}^{n-1} b_k^{(n-k)}, \end{align} see for instance eq.~1.148 of \cite{ForBook}. We also take as given that the theorem holds for $\beta=1$ due to \cite{Trotter}, and the fact that the Householder tridiagonalization process does not change the spectral measure with respect to the first coordinate. Next, for $\mbf{T}$ be a random tridiagonal matrix defined in the statement with $\beta=1$, introduce a biased version of the distribution of $\mbf{T}$ with the biasing function \[ g_\beta(\mbf{b})=\prod_{k=1}^{n-1} b_k^{(\beta-1)(n-k)}. \] The biasing produces a random tridiagonal matrix $\mbf{\wt{T}}$ where the diagonal and off-diagonal entries are still independent, the distribution of the diagonal entries is still $N(0,2)$, but the distribution of the $k$th off-diagonal entry has changed from $\chi_{n-k}$ to $\chi_{\beta(n-k)}$. By \eqref{eq:magic_Delta_p} we have \begin{align}\label{eq:bias_DE} g_\beta(\mbf{b})=|\Delta({\lambda})|^{\beta-1} \prod_{k=1}^n p_k^{-\frac{\beta-1}{2}}, \end{align} hence biasing the entries of $\mbf{T}$ with $g_\beta(\mbf{b})$ is the same as biasing the spectral variables $\lambda, \mbf{p}$ with the appropriate product on the right hand side of \eqref{eq:bias_DE}. This immediately implies that the eigenvalues and spectral weights of $\mbf{\wt{T}}$ are still independent of each other, that the joint eigenvalue density of $\mbf{\wt{T}}$ is proportional to $|\Delta(\lambda)|^\beta e^{-\frac{1}{4}\sum_{k=1}^n \lambda_k^2}$, and that its spectral weights have Dirichlet$(\beta/2,\dots,\beta/2)$ distribution. The complete statement of the theorem now follows after scaling $\mbf{\wt{T}}$ by $ \frac{1}{\sqrt{\beta}}$. The value of the normalizing constant $Z_{n,\beta}$ follows from the known $\beta=1$ factor (see eq.~1.160 of \cite{ForBook}) along with an evaluation of $E[g_\beta(\mbf{b})]$. \end{proof} \begin{remark} The original proof of Theorem \ref{thm:DE} given in \cite{DE} is slightly different. It uses the $\beta=1$ case as a starting point to derive an expression for the Jacobian of the one-to-one transformation $(\mbf{a},\mbf{b}) \mapsto ({\lambda}, \mbf{p})$. It then uses the identity \eqref{eq:magic_Delta_p} to compute the joint density of the spectral variables $({\lambda}, \mbf{p})$ for the tridiagonal matrix $\mbf{T}$. As it was already remarked in \cite{DE}, the expression for the Jacobian of the transformation $(\mbf{a},\mbf{b}) \mapsto ({\lambda}, \mbf{p})$ can also be computed directly, without relying on Theorem \ref{thm:DE} being true in the $\beta=1$ case. \end{remark} \subsection{Key spectral identity} We establish a block Jacobi matrix equivalent of the identity \eqref{eq:magic_Delta_p} which relates products of powers of the determinants of the off-diagonal blocks to eigenvalues and eigenvector entries. First we need the following definition. \begin{definition} For $\lambda = \{ \lambda_j\}_{j \in \lst{rn} }$ and an $r\times rn $ matrix $\mbf{X}$ define the $rn \times rn$ matrix $\mbf{M}=\mbf{M}({\lambda}, \mbf{X})$ entry-wise as \begin{align}\label{eq:MagicM_ij} M_{i,j}=\lambda_i^{\nint{j}{r}} x_{ \nfr{j}{r},i}, \end{align} in which \begin{align*} \nint{j}{r} :=\left\lfloor \frac{j-1}{r} \right\rfloor, \qquad \nfr{j}{r}:=j-r \nint{j}{r}. \end{align*} Note that the shift in $ \nint{j}{r}$ means that $\lfloor r \rfloor_r = 0$ and hence $\nfr{j}{r}\in \lst{r}$. For an $rn \times rn$ matrix $\mbf{X}$ we define $\mbf{M}({\lambda}, \mbf{X})$ using the $r \times rn$ submatrix of the first $r$ rows of $\mbf{X}$. \end{definition} The operation $({\lambda}, \mbf{X})\mapsto \mbf{M}$ can be realized as a block-matrix built from the row-by-row Kronecker products of the $rn\times r$ matrix $\mbf{X}^T$ and the $rn\times n$ Vandermonde type matrix $\mbf{\Lambda}=(\lambda_{i}^{j-1})_{i\in \lst{rn},j\in \lst{n}}$. Similar constructions show up in the literature as the ``face-splitting product'' or Khatri–Rao product \cite{KR_1968}. As an example, \begin{align}\label{eq:M6} \mbf{M}({\lambda}, \mbf{X})= \left( \begin{array}{cccccc} x_{1,1} & x_{2,1} & \lambda_1 x_{1,1} & \lambda_1 x_{2,1} & \lambda_1^2 x_{1,1} & \lambda_1^2 x_{2,1} \\ x_{1,2} & x_{2,2} & \lambda_2 x_{1,2} & \lambda_2 x_{2,2} & \lambda_2^2 x_{1,2} & \lambda_2^2 x_{2,2} \\ x_{1,3} & x_{2,3} & \lambda_3 x_{1,3} & \lambda_3 x_{2,3} & \lambda_3^2 x_{1,3} & \lambda_3^2 x_{2,3} \\ x_{1,4} & x_{2,4} & \lambda_4 x_{1,4} & \lambda_4 x_{2,4} & \lambda_4^2 x_{1,4} & \lambda_4^2 x_{2,4} \\ x_{1,5} & x_{2,5} & \lambda_5 x_{1,5} & \lambda_5 x_{2,5} & \lambda_5^2 x_{1,5} & \lambda_5^2 x_{2,5} \\ x_{1,6} & x_{2,6} & \lambda_6 x_{1,6} & \lambda_6 x_{2,6} & \lambda_6^2 x_{1,6} & \lambda_6^2 x_{2,6} \\ \end{array} \right), \end{align} illustrates the $n=3$, $r=2$ case. The advertised formula, recorded next, involves the determinant of $\mbf{M}$. \begin{proposition}\label{prop:magic} Suppose that $\mbf{T}\in \mathfrak{M}_{n,\beta, r}$ with blocks $\mbf{A}_j, \mbf{B}_j$. Then \begin{align}\label{eq:magic11} \prod_{j=1}^{n-1} \det(\mbf{B}_j)^{n-j}=|\det \mbf{M}({\lambda}, \mbf{Q})|, \end{align} where $\mbf{\lambda}$ are the eigenvalues of $\mbf{T}$, and $\mbf{Q}$ is the matrix of the normalized eigenvectors of $\mbf{T}$ ordered according to $\lambda$. \end{proposition} When $r=1$ one can easily see from \eqref{eq:M6} that $\det {\mbf{M}(\lambda, \mbf{Q})} = \Delta(\lambda) \prod_{i=1}^n q_{1,i}$ upon factoring out the eigenvector coordinates from each row, recovering the identity \eqref{eq:magic_Delta_p}. We remark that for a particular $\mbf{T}$ the choice of $\mbf{Q}$ is not unique: eigenvectors could be multiplied by a unit phase, and there is even more freedom if $\mbf{T}$ has eigenvalues with multiplicity higher than one. The proof below shows that the expression on the right hand side of \eqref{eq:magic11} does not depend on the particular choice of $\mbf{Q}$. \begin{proof}[Proof of Proposition \ref{prop:magic}] Consider the $rn \times rn $ matrix $\mbf{S}$ from \eqref{eq:S1234} built from $\mbf{T}$. This is an upper triangular block matrix where the $j$th diagonal block is $\mbf{B}_{j-1}^{\T}\cdots \mbf{B}_1^{\T}$, the first diagonal block being $\mbf{I}_r$. The determinant of $\mbf{S}$ is the product of the determinants of its diagonal blocks: \[ \det \mbf{S}=\prod_{j=2}^n \det(\mbf{B}_{j-1}^{\T}\cdots \mbf{B}_1^{\T})=\prod_{j=1}^{n-1} \det(\mbf{B}_j)^{n-j}. \] Denote by $\boldsymbol{\Lambda}$ the diagonal matrix built from $\lambda$. Then we have $\mbf{T}=\mbf{Q}\boldsymbol{\Lambda} \mbf{Q}^{\T}$ and \begin{align} \mbf{T}^{j} \mbf{e}_{\lst{r}}= \mbf{Q} \boldsymbol{\Lambda} \mbf{Q}^{\T}\mbf{e}_{\lst{r}}. \end{align} Hence $\mbf{S}= {\mbf{Q}} \widetilde{\mbf{S}}$, and $|\det \mbf{S}|=|\det \wt{\mbf{S}}|$ with \begin{equation*} \widetilde{\mbf{S}}=[\mbf{Q}^{\T}\mbf{e}_{\lst{r}}, \boldsymbol{\Lambda}\mbf{Q}^{\T}\mbf{e}_{\lst{r}}, \dots, \boldsymbol{\Lambda}^{n-1}\mbf{Q}^{\T}\mbf{e}_{\lst{r}}]=\mbf{M}(\mbf{\lambda}, \overline{\mbf{Q}}). \end{equation*} Since $|\det \mbf{M}(\mbf{\lambda}, \overline{\mbf{Q}})|=|\det \mbf{M}(\mbf{\lambda}, {\overline{}\mbf{Q}})|$, the proposition follows. \end{proof} We remark that for $r=2$ and $\lambda_j>0$ the following remarkable identity holds for $\det \mbf{M}({\lambda}, \mbf{Q})$ (see equation (52) in \cite{LT2006}): \begin{align}\label{eq:Pfaffian_detM} \det \mbf{M}({\lambda}, \mbf{Q})=\prod_{1\le i<j\le 2n}(\sqrt{\lambda_i}+\sqrt{\lambda_j}) \, \operatorname{Pf}\left(\frac{q_{1,i}q_{2,j}-q_{1,j}q_{2,i}}{\sqrt{\lambda_i}+\sqrt{\lambda_j}}\right). \end{align} We are not aware of extensions of this result for $r>2$. \subsection{Proofs of Theorem \ref{thm:main} and \ref{thm:main_W}} We are now ready to present the proofs of our main theorems, modulo one key ingredient. \begin{proof}[Proof of Theorem \ref{thm:main} -- first steps] The joint density of the entries of a matrix $\mbf{T}$ distributed as $\HH(r,s)$ is given by \begin{align} f(\mbf{A},\mbf{B})=C_{\beta,n,r,s} \exp(-\frac{\beta}{4} \Tr \mbf{T} \mbf{T}^{\T}) \prod_{m=1}^{n-1} (\det \mbf{B}_m)^{\beta(r+s)(n-m)+\beta-1} \prod_{m=1}^{n-1} \prod_{j=1}^r (b_{j,j}^{(m)})^{-\beta j}, \end{align} where $C_{\beta, n, r, s}$ is an explicitly computable normalizing constant. Note that the distribution $\HH(r,s)$ can be realized as the biased version of the distribution $\HH(r,0)$ with the biasing function $ \prod_{m=1}^{n-1} (\det \mbf{B}_m)^{\beta s(n-m)}$. Using $E_{s}$ for the expectation with respect to $\HH(r,s)$ (suppressing the dependence on $\beta, n, r$) we have, for any (bounded) test function $h$, \begin{align} E_{s}[h(\mbf{T})] & =\frac{E_{0}\left[h(\mbf{T})\prod_{m=1}^{n-1} (\det \mbf{B}_m)^{\beta s(n-m)} \right]} {E_{0}\left[\prod_{m=1}^{n-1} (\det \mbf{B}_m)^{\beta s(n-m)} \right]} \\ & := \frac{1}{Z_n} E_{0}\left[h(\mbf{T})\prod_{m=1}^{n-1} (\det \mbf{B}_m)^{\beta s(n-m)} \right]. \nonumber \end{align} In particular, this is true if $h(\mbf{T})=g({\lambda})$ with a (bounded) test function $g$. By Proposition \ref{prop:magic} we have \begin{align}\label{eq:magic_identity} \prod_{m=1}^{n-1} (\det \mbf{B}_m)^{\beta s(n-m)}=|\det \mbf{M}({\lambda}, \mbf{Q})|^{\beta s} \end{align} where $\mbf{Q}$ is the $r\times (rn)$ matrix of the first $r$ coordinates of the normalized eigenvectors of $\mbf{T}$. By Theorem \ref{thm:GFE_block}, $\mbf{Q}$ and ${\lambda}$ are independent under the distribution $\HH(r,0)$, and $\mbf{Q}$ is distributed as the first $r$ rows of a Haar $\FF$-unitary matrix. This implies that \begin{align*} E_{s}[g({\lambda})] & = \frac{1}{Z_n} E_{0}\left[g({\lambda}) F_{\beta,r,n, \beta s}({\lambda})\right] \end{align*} where \begin{align} \label{Reweight} F_{\beta, r,n, \beta s}({\lambda}) & = E_{\boldsymbol{Q}} \left[|\det \mbf{M}({\lambda}, \mbf{Q})|^{\beta s}\right]. \end{align} In other words, the distribution of ${\lambda}$ is just G$\beta$E biased by the functional $ F_{\beta, r,n,\beta s}({\lambda})$. As for the normalizer, the left hand side of \eqref{eq:magic_identity} readily yields \begin{align}\nonumber Z_n &= E_{0} \left[ \prod_{m=1}^{n-1}(\det \mbf{B}_m)^{(s)(n-m)} \right] = \prod_{m=1}^{n-1} \prod_{i=1}^r E (b^{(m)}_{i,i})^{\beta s(n-m)} \\ \nonumber & = \prod_{m=1}^{n-1} \prod_{i=1}^r (\beta/2) ^{-\frac{\beta s}{2}(n-m)}\frac{ \Gamma \left(\frac{\beta}{2}((r+s)(n-m)-i+1 ))\right)}{\Gamma \left(\frac{\beta}{2}{((r(n-m)-i+1) }\right)}\\&=(\beta/2)^{-\frac{\beta }{4} r s n(n-1)} \prod_{m=1}^{n-1} \prod_{i=1}^r \frac{ \Gamma \left(\frac{\beta}{2}((r+s)(n-m)-i+1 )\right)}{\Gamma \left(\frac{\beta}{2}(r(n-m)-i+1) \right)}.\label{eq:bias_const} \end{align} since the $b^{(m)}_{i,i}$ are independent $ \frac{1}{\sqrt{\beta}} \times \chi_{\beta (r(n-m)+1-i)}$ variables. \end{proof} Identifying the joint eigenvalue density for any $\HH(r,s)$ model then comes down to computing the determinant moments indicated in \eqref{Reweight}. We have been able to do this only for general $r\ge 2$ and for $\beta s=2$, and $r=2, \beta s=4$. In particular, the proof of Theorem \ref{thm:main} is finished by establishing the following. \begin{theorem} \label{thm:moment} With $\boldsymbol{Q}$ Haar distributed $\FF$-unitary matrix we have \begin{align*} E_{\mbf{Q}} | \det \mbf{M}({\lambda}, \mbf{Q}) |^{\beta s} = c_{n, \beta, r,\beta s} \times \begin{cases} \sum\limits_{(\cA_1,\dots, \cA_r)\in \cP_{r,n}} \prod_{j=1}^r \Delta(\cA_j)^2, & \mbox{ for } r \ge2, \beta s=2,\\[10pt] \sum\limits_{(\cA,\cA')\in \cP_{2,n}} \Delta(\cA)^4 \Delta(\cA')^4, & \mbox{ for } r=2, \beta s = 4, \end{cases} \end{align*} where \begin{align*} c_{n,\beta, r, \beta s} =(\beta/2) ^{\frac{\beta}{2}rsn} \prod_{i=1}^{r} \frac{\Gamma \left(\frac{\beta }{2}(rn+1-i)\right)}{ \Gamma \left(\frac{\beta}{2}((r+s)n+1-i) \right)} \times \begin{cases} 1, \qquad &r\ge2, \beta s=2,\\ (12/\beta)^n, \qquad &r=2, \beta s=4. \end{cases} \end{align*} \end{theorem} One immediately recognizes the interaction term reported in \eqref{density1}. To arrive at the form of $r=2$, $s=2,4$ densities appearing in \eqref{density2} we will also prove: \begin{equation} \label{id_quad_to_square} \sum_{ (\cA,\cA')\in \cP_{2,n} } \Delta (\cA)^4 \Delta (\cA')^4 = 2^{-n} \left(\sum_{ (\cA,\cA')\in \cP_{2,n} } \Delta (\cA)^2 \Delta (\cA')^2\right)^2, \end{equation} and \begin{align} \label{id_pfaff} \sum_{ (\cA,\cA')\in \cP_{2,n} } \Delta(\cA)^2 \Delta (\cA')^2&=(-2)^n \Delta (\lambda) \Pf\left(\frac{\ind_{i\neq j}}{\lambda_i-\lambda_j}\right). \end{align} The proof of Theorem \ref{thm:moment} is divided between Sections \ref{sec:2moment} and \ref{sec:4moment}, for $s=2$ and $s=4$ respectively. The identities \eqref{id_quad_to_square} and \eqref{id_pfaff} are proven in Section \ref{sec:det_identities} along with a number of related determinantal formulas used throughout. \begin{proof}[Proof of Theorem \ref{thm:main} -- final steps] We have shown that the joint eigenvalue distribution of $\HH(r,s)$ is just $G\beta{E}(rn)$ biased by the functional $ F_{\beta, r,n,\beta s}({\lambda})$ given in \eqref{Reweight}. Theorem \ref{thm:moment} provides $ F_{\beta, r,n,\beta s}({\lambda})$ for $r\ge 2$ and $\beta s=2$ and for $r=2$, $\beta s=4$. When $r\ge 2$, $\beta s=2$ this gives the joint density function \eqref{density1}. The normalizing constant $Z_{n,\beta, r, \beta s}$ satisfies \begin{align}\label{eq:norm_const} Z_{n,\beta,r,\beta s}=Z_{nr, \beta} Z_n c_{n,\beta, r, \beta s}^{-1}, \end{align} where $Z_{nr,\beta}$ is the normalizing constant in \eqref{eq:GbE_constant}, $Z_n$ is given in \eqref{eq:bias_const} and $c_{n,\beta, r, \beta s}$ is given in Theorem \ref{thm:moment}. When $r=2$ and $\beta s=2$ or 4 then we also use \eqref{id_quad_to_square} and \eqref{id_quad_to_square} to rewrite expression in Theorem \ref{thm:moment} with a Pfaffian to obtain the joint density function \eqref{density2}. The normalizing constant is again given by \eqref{eq:norm_const}, note that we needed the additional $2^n$ factor in \eqref{density2} to match the two forms of the $r=2$, $\beta s=2$ case. The reported constant in Theorem \ref{thm:main} now follows after some algebra, noting that \begin{align*} \prod_{m=1}^{n-1}\prod_{i=1}^r \frac{ \Gamma \left(\frac{\beta}{2}((r+s)(n-m)-i+1 )\right)}{\Gamma \left(\frac{\beta}{2}(r(n-m)-i+1) \right)} \prod_{i=1}^{r} \frac{ \Gamma \left(\frac{\beta}{2}((r+s)n+1-i) \right)}{\Gamma \left(\frac{\beta }{2}(rn+1-i)\right)} = \prod_{k=1}^{rn} \frac{\Gamma\left(\frac{\beta}{2} \left(k+s \lceil\frac{k}{r} \rceil\right)\right)}{\Gamma\left(\frac{\beta}{2} k\right)}. \end{align*} \end{proof} The proof of Theorem \ref{thm:main_W} follows the same biasing idea as the proof of Theorem \ref{thm:main}. \begin{proof}[Proof of Theorem \ref{thm:main_W}] Let $\mbf{T}=\mbf{L}\mbf{L}^{\T}\sim \WW(r,s)$, with $\mbf{L}$ as in Definition \ref{def:BlockW}. (Note that $\mbf{L}$ is a function of $\mbf{T}$.) The joint density of the entries $\mbf{O}_m, \mbf{D}_m$ of $\mbf{L}$ are given by \begin{align*} f(\mbf{D}, \mbf{O})=&C_{\beta, n,m,r,s} \exp(-\frac{\beta}{2}\Tr \mbf{T} \mbf{T}^{\T}))\prod_{j=1}^n (\det \mbf{D}_j)^{\beta (r+s)(m-j)+\beta-1} \\ & \times \prod_{j=1}^{n-1} (\det \mbf{O}_j)^{\beta (r+s)(n-j)+\beta-1}\prod_{j=1}^n \prod_{k=1}^r (d^{(j)}_{k, k})^{-\beta k} \prod_{j=1}^{n-1} \prod_{k=1}^r (o^{(j)}_{k, k})^{-\beta k}, \end{align*} where $C_{\beta,n,m,r,s}$ is an explicitly computable normalizing constant. As before, we can realize $\WW(r,s)$ using a biased version of $\WW(r,0)$. Using $E_s$ for the expectation with respect to $\WW(r,s)$ we have, for any bounded test function $h$, \begin{align} E_{s}[h(\mbf{T})] & =\frac{E_{0}\left[h(\mbf{T})\prod_{j=1}^n (\det \mbf{D}_j)^{\beta s(m-j)} \prod_{j=1}^{n-1} (\det \mbf{O}_j)^{\beta s(n-j)} \right]} {E_{0}\left[\prod_{j=1}^n (\det \mbf{D}_j)^{\beta s(m-j)} \prod_{j=1}^{n-1} (\det \mbf{O}_j)^{\beta s(n-j)} \right]} \\ & := \frac{1}{Z_{n,m}} E_{0}\left[h(\mbf{T})\prod_{j=1}^n (\det \mbf{D}_j)^{\beta s(m-j)} \prod_{j=1}^{n-1} (\det \mbf{O}_j)^{\beta s(n-j)} \right]. \nonumber \end{align} The matrix $\mbf{T}=\mbf{L} \mbf{L}^{\T}$ has off-diagonal blocks $\mbf{B}_j=\mbf{D}_j \mbf{O}_j$, hence by Proposition \ref{prop:magic} we have \begin{align} \prod_{j=1}^{n-1} \det(\mbf{D}_j \mbf{O}_j)^{n-j}=\prod_{j=1}^{n} \det(\mbf{D}_j)^{n-j} \prod_{j=1}^{n-1} (\det \mbf{O}_j)^{n-j}=|\det \mbf{M}({\lambda}, \mbf{Q})|, \end{align} where $\mbf{Q}$ is the matrix of the normalized eigenvalue matrix of $\mbf{T}$. We also have \begin{align} \prod_{j=1}^{rn} \lambda_j=\det \mbf{T}=(\det \mbf{L})^2=\prod_{j=1}^n (\det \mbf{D}_j)^2. \end{align} Using $h(\mbf{T})=g({\lambda})$ with a (bounded) test function $g$ we now get \begin{align*} E_{s}[g({\lambda})] & = \frac{1}{Z_n} E_{0}\left[g({\lambda}) F_{\beta,r,n,m, s}({\lambda})\right] \end{align*} where \begin{align} \label{Reweight1} F_{\beta, r,n, m,s}({\lambda}) & =\prod_{j=1}^{rn} \lambda_j^{\frac{\beta}{2}s(m-n)} \times E_{\boldsymbol{Q}} \left[|\det \mbf{M}({\lambda}, \mbf{Q})|^{\beta s} \right] \end{align} Hence the distribution of $\lambda$ under $\WW(r,s)$ is just L$\beta$E$(rn,rm)$ biased by $F_{\beta, r,n, m,s}({\lambda})$. The statement of the theorem now follows from Theorem \ref{thm:moment} and the identities \eqref{id_quad_to_square} and \eqref{id_pfaff}. The normalizing constant can be explicitly evaluated using a similar argument as in Theorem \ref{thm:main}. \end{proof} \subsection{A reduction and a representation} We end this section with two lemmas that play significant roles in our computations of the moments of $\det \mbf{M}({\lambda}, \mbf{Q})$. The first observation is that in computing moments of $\det \mbf{M}({\lambda}, \mbf{Q})$ we may replace the Haar distributed $\mbf{Q}$ with a matrix of independent Gaussians. \begin{lemma}[Gaussian reduction lemma]\label{lem:Gauss_red} Let $\mbf{X}$ be an $rn \times rn$ matrix of $i.i.d.$ $\mathbb{F} N(0,1)$ entries, and $\mbf{Q}$ a Haar distributed $\mathbb{F}$-unitary matrix. Let $R_{i,i}\sim \frac{1}{\sqrt{\beta}} \chi_{\beta(rn+1-i)}, 1\le i\le r$ be independent of each other and of $\mbf{Q}$. Then for any fixed collection $\lambda_i\in \R, i \in \lst{rn}$ we have the following distributional identity: \begin{align}\det \mbf{M}({\lambda}, \mbf{X})\eqd \prod_{i=1}^{r} R_{i,i}^n \times \det \mbf{M}({\lambda}, \mbf{Q}). \end{align} It follows that for any $s>0$ we have \begin{align} E |\det \mbf{M}({\lambda}, \mbf{Q})|^{\beta s}=\kappa_{n,\beta, n,r, \beta s} \cdot E |\det \mbf{M}({\lambda}, \mbf{X})|^{\beta s}, \end{align} with $\kappa_{n,\beta, n,r, \beta s} =(\beta/2) ^{\frac{1}{2}rsn}\times \prod_{i=1}^{r} \frac{\Gamma \left(\frac{\beta }{2}(rn+1-i)\right)}{ \Gamma \left(\frac{\beta}{2}((r+s)n+1-i) \right)}$. \end{lemma} \begin{proof} Denote the columns of $\mbf{X}$ by $\mbf{X}_i, i\in \lst{rn}$. Consider the QR decomposition $\mathbf{X}=\mathbf{Q}\mathbf{R}$ of the matrix $\mathbf{X}=[\mathbf{X}_1, \dots, \mathbf{X}_{nr}]$ where the diagonal elements of $\mbf{R}$ are chosen to be positive real numbers. From the $\FF$-unitary invariance of $\mathbf{X}$ it follows that $\mathbf{Q}$ and $\mathbf{R}$ are independent and $\mathbf{Q}$ is a Haar distributed $\mathbb{F}$-unitary matrix. (See e.g.~\cite{Meckes2019}.) By Theorem \ref{thm:bartlett} we also know that the diagonal entries $R_{i,i}, i\in \lst{rn}$ of $\mathbf{R}$ are independent with ${R}_{i,i}\sim \frac{1}{\sqrt{\beta}}\chi_{\beta(rn+1-i)}$. From $\mbf{X}=\mbf{Q}\mbf{R}$ we have \[ \mbf{X}_i=R_{i,i} \mbf{Q}_i+\sum_{j=1}^{i-1} R_{j,i} \mbf{Q}_j, \] and using elementary row operations we get \begin{align}\label{eq:magic_dist} \det\mbf{M}({\lambda}, \mbf{X})=\prod_{a=1}^r R_{a,a}^n \cdot \det\mbf{M}({\lambda}, \mbf{Q}). \end{align} Using the independence of $\mbf{R}$ and $\mbf{Q}$ the statement of the lemma now follows. \end{proof} Our second observation is that, by regrouping terms in its Laplace expansion, the determinant of $\mbf{M}(\lambda, \mbf{Q})$ can be written as a weighted sum of Vandermonde determinant products. We will index the permutations in this expansion in a particular way, and this appears frequently enough that we isolate its definition. \begin{definition} For $\underline{\cA}=(\cA_1,\dots,\cA_r)\in \cP_{r,n}$ let $\sigma_{\underline{\cA}}\in S_{rn}$ denote the following permutation of the set $\lst{rn}$: \begin{align*} \sigma_{\underline{\cA}}(a+(m-1)n)= \text{the } a^{th} \text{ largest element of } \cA_m, \qquad \text{for}\, a\in \lst{n}, m\in \lst{r}. \end{align*} Now when $r=2$ the elements of $(\cA_1, \cA_2)$ of $\cP_{2,n}$ can be identified just with the subsets $\cA\subset \lst{2n}$ of cardinality $n$. Using $\cA'$ for the complement of $\cA$ in $\lst{2n}$ throughout, we further denote: \begin{equation} \label{shorthand} \sigma_{{\cA}} := \sigma_{\cA, \cA'} \in S_{2n}. \end{equation} \end{definition} With this set, we have the following representation. \begin{lemma}\label{prop:M_expansion} Let $\lambda_i, i \in \lst{rn}$, and $Q_{a,b}, a\in \lst{r}, b\in \lst{rn}$ be fixed numbers. For $m\in \lst{r}$, $\cA\subset \lst{rn}$ let $Q_m(\cA)=\prod_{a\in \cA} Q_{m,a}$. Then \begin{align}\label{eq:detM} \det \mbf{M}({\lambda}, \mbf{Q})&= \sum_{(\cA_1,\dots,\cA_r)\in \cP_{r,n}} \!\!\sgn(\sigma_{\underline{\cA}}) \prod_{m=1}^r Q_{m}(\cA_m) \Delta(\cA_m). \end{align} \end{lemma} \begin{proof} First, any permutation $\sigma\in S_{rn}$ of $\lst{rn}$ can be mapped in a one-to-one way to a pair consisting of an element of $\cP_{r,n}$ and a vector $(\sigma_1, \dots, \sigma_r)$ of $r$ permutations of $S_n$. For $\sigma\in S_{rn}, m\in \lst{r}$ let \begin{align*} \cA_m=\{\sigma(i); i\in \lst{rn}, \nfr{i}{r}=m\} =\{\sigma(m), \sigma(m+r), \dots, \sigma(m+r(n-1))\}, \end{align*} and let $\sigma_m$ be the permutation taking the ordered version of $\cA_m$ to the sequence \[(\sigma(m), \sigma(m+r), \dots, \sigma(m+r(n-1))). \] (Strictly speaking, $\sigma_m$ is the permutation that is generated by this mapping on the relative ranks of the elements of $\cA_m$ within the set.) Note that $\sigma$ can be reconstructed from the pair $(\cA_1,\dots, \cA_r), (\sigma_1, \dots, \sigma_r)$, and we have that \begin{align} \sgn(\sigma)=\sgn(\sigma_{\underline{\cA}}) \prod_{m=1}^r \sgn(\sigma_m). \end{align} Next, recall the expression of the entries $M_{i,j}$ of $\mbf{M}=\mbf{M}({\lambda}, \mbf{Q})$ from \eqref{eq:MagicM_ij}, and write out the determinant expansion: \begin{align*} \det \mbf{M}&= \det \mbf{M}^{\dagger}=\sum_{\sigma\in S_{rn}} \sgn(\sigma) \prod_{i=1}^{rn} M_{\sigma(j),i}\\ &=\sum_{\sigma\in S_{rn}} \sgn(\sigma) \prod_{m=1}^{r} \prod_{a=0}^{n-1} \lambda_{\sigma(m+ar)}^{a} Q_{m,\sigma(m+ar)} \\ &=\sum_{(\cA_1,\dots,\cA_r)\in \cP_{r,n}}\!\!\sgn(\sigma_{\underline{\cA}}) \prod_{m=1}^r Q_{m}(\cA_m) \sum_{\sigma_1, \dots, \sigma_r} \prod_{m=1}^r \sgn(\sigma_m) \prod_{a=0}^{n-1} \lambda_{\sigma_m(m+a r)}^{a}. \end{align*} Note that the second summation in the last line is over the possible vector of permutations $(\sigma_1,\dots,\sigma_r)\in S_n^r$ corresponding to a particular partition $(\cA_1,\dots,\cA_r)\in \cP_{r,n}$. For a fixed $m\in \lst{r}$ and $\cA_m$ the sum $\sum_{\sigma_m} \sgn(\sigma_m) \prod_{a=0}^{n-1} \lambda_{\sigma_m(m+a r)}^{a}$ is just the Vandermonde determinant $\Delta(\cA_m)$ which concludes the proof. \end{proof} \section{Evaluating $E_{\boldsymbol{Q}} \left[|\det \mbf{M}({\lambda}, \mbf{Q})|^2\right]$} \label{sec:2moment} The next proposition gives the proof of the first statement of Theorem \ref{thm:moment}. \begin{proposition}\label{prop:det_l=2} Fix $r\ge 2, n\ge 2$. Let $\mbf{Q}$ be an $rn\times rn$ Haar distributed $\mathbb{F}$-unitary matrix. Then, for all ${\lambda}\in \R^{rn}$ we have \begin{align}\label{eq:det2_general} E_{\boldsymbol{Q}} \left[|\det \mbf{M}( {\lambda}, \mbf{Q})|^2\right] = \kappa_{n,\beta, r,2} \sum_{(\cA_1,\dots, \cA_r)\in \cP_{r,n}} \prod_{j=1}^r \Delta (\cA_j)^2. \end{align} Here $\kappa_{n,\beta,r,2}$ is as defined in Lemma \ref{lem:Gauss_red}. \end{proposition} We provide two proofs. The first relies on the Gaussian reduction of Lemma \ref{lem:Gauss_red}. The second is provided for historical interest: it only uses the exchangeability properties of the Haar distribution, together with a determinantal identity from 1851 due to Sylvester \cite{Sylvester1851}. As the notation involved in the latter approach gets fairly heavy, we only present this second proof for $r=2$, $\beta =1$. We also note that in the $r=2$ case one can use the identity \eqref{eq:Pfaffian_detM} to obtain yet another independent proof. \begin{proof}[Proof of Proposition \ref{prop:det_l=2} using Gaussian reduction] Given Lemma \ref{lem:Gauss_red} the statement is equivalent to the following. Let $\mbf{X}$ be an $r\times (rn)$ matrix with i.i.d.~standard real or complex Gaussian entries $X_{a,b}, a\in\lst{r}, n\in \lst{rn}$, then \begin{align}\label{eq:detM_2_Gauss} E \left[|\det \mbf{M}({\lambda}, \mbf{X})|^2\right] = \sum_{(\cA_1,\dots, \cA_r)\in \cP_{r,n}} \prod_{j=1}^r \Delta(\cA_j)^2. \end{align} For a subset $\cA\subset \lst{rn}$ and $m\in \lst{r}$ we denote by $X_{m}(\cA)$ the product $\prod_{j\in \cA} X_{m,j}$. From Proposition \ref{prop:M_expansion} we get \begin{align} E[|\det \mbf{M}|^2]=\sum_{\underline{\cA}\in \cP_{r,n}}\sum_{\underline{\cB}\in \cP_{r,n}} \sgn(\sigma_{\underline{\cA}})\sgn(\sigma_{\underline{\cB}}) E[\prod_{m=1}^r X_{m}(\cA_m) \overline{X}_{m}(\cB_m)] \prod_{m=1}^r \Delta(\cA_m) \Delta(\cB_m) \end{align} Since $X_{a,b}$ are i.i.d.~$\FF N(0,1)$ random variables, we have \begin{align*} E[\prod_{m=1}^r X_{m}(\cA_m) \overline{X}_{m}(\cB_m)] =\begin{cases} 1, \qquad &\text{if $\cA_m=\cB_m$ for all $m\in \lst{r}$,}\\ 0, \qquad &\text{otherwise.} \end{cases} \end{align*} From this the identity \eqref{eq:detM_2_Gauss} and the statement of the proposition follows. \end{proof} \begin{proof}[Proof of Proposition \ref{prop:det_l=2} for $r=2$ and $\beta=1$ using exchangeability] Using again Proposition \ref{prop:M_expansion}, we have that \begin{align*} E[\det \mbf{M}^2]=&\sum_{|\cA|=n, \cA\subset\lst{2n}}\sum_{|\cB|=n, \cB\subset\lst{2n}} \sgn(\sigma_{\cA}) \sgn(\sigma_{\cB}) E[ Q_{1}(\cA) Q_{2}(\cA')Q_{1}(\cB) Q_{2}(\cB')] \\ &\hskip 130pt \times \Delta(\cA) \Delta(\cA')\Delta(\cB) \Delta(\cB'). \end{align*} Recall the shorthand \eqref{shorthand}. The expression $Q_{1}(\cA) Q_{2}(\cA')Q_{1}(\cB) Q_{2}(\cB')$ is a monomial of the form $\prod_{j=1}^{2n} q_{1,j}^{a_j} q_{2,j}^{2-a_j}$ where for each $j$ we have $a_j\in \{0,1,2\}$. By the exchangeability of the columns of a Haar orthogonal matrix we have \begin{align}\label{eq:def_g} E[\prod_{j=1}^{2n} q_{1,j}^{a_j} q_{2,j}^{2-a_j}]=g(c_0,c_1,c_2) \end{align} for some function $g$ where $c_i=|\{a_j=i: j\in \lst{2n}\}|$. Moreover, \begin{align*} E[Q_{1}(\cA) Q_{2}(\cA')Q_{1}(\cB) Q_{2}(\cB')]=g(a,a,2n-2a), \qquad a=|\cA\cap \cB|=|\cA' \cap \cB'|. \end{align*} Thus \begin{align*} E[\det \mbf{M}^2]=&\sum_{|\cA|=n, \cA\subset\lst{2n}}\sgn(\sigma_{\cA}) \Delta(\cA) \Delta(\cA')\\ &\hskip 80pt \times \sum_{a=0}^n g(a,a,2n-a) \sum_{|\cB|=n, |\cA\cap \cB|=a} \sgn(\sigma_{\cB}) \Delta(\cB) \Delta(\cB'). \end{align*} We will show that for any fixed $0\le a\le n$, $\cA\subset \lst{2n}$ with $|\cA|=n$, and $\cS\subset \cA$ with $|\cS|=a$ we have \begin{align}\label{eq:det_sylv} (-1)^{n-a} \Delta(\cA) \Delta(\cA') = \sgn(\sigma_{\cA}) \sum_{|\cB|=n, \cA\cap \cB=\cS} \sgn(\sigma_{\cB}) \Delta(\cB) \Delta(\cB'). \end{align} This will imply \begin{align} E[\det \mbf{M}^2]=\left(\sum_{a=0}^{n}(-1)^{n-a} \binom{n}{a}g(a,a,2n-2a)\right) \sum_{|\cA|=n, \cA\subset\lst{2n}} \Delta(\cA)^2 \Delta(\cA')^2, \end{align} which proves the proposition for $r=2$ with the constant left in the form \begin{align*} c_{n,2,2}&=\sum_{a=0}^{n}(-1)^{n-a} \binom{n}{a}g(a,a,2n-2a)=E\left[\prod_{j=1}^n \left(q_{1,2j-1}q_{2,2j-1}q_{1,2j}q_{2,2j}-q_{1,2j-1}^2 q_{2,2j}^2\right)\right], \end{align*} by \eqref{eq:def_g}. Returning to \eqref{eq:det_sylv}, when $a=n$ the sum on the right side of that identity only contains the term corresponding to $\cB=\cA$, so the two sides are equal. When $a=0$ then again the right side only contains one term, the one corresponding to $\cB=\cA'$. Since $\sigma_{\cA'}$ can be obtained from $\sigma_{\cA}$ using $n$ transpositions, \eqref{eq:det_sylv} holds in this case as well. To prove \eqref{eq:det_sylv} in the $1\le a\le n-1$ case we use the following identity due to Sylvester \cite{Sylvester1851}. Suppose that $\mbf{R}_1$, $\mbf{R}_2$ are $n\times n$ matrices and $1\le m\le n$. Let $I$ be a fixed subset of column indices of $\mbf{R}_1$ with $|I|=m$. Then \begin{align}\label{eq:Sylvester} \det(\mbf{R}_1) \det(\mbf{R}_2)=\sum_{(\widehat{\mbf{R}}_1, \widehat{\mbf{R}}_2)} \det(\widehat{\mbf{R}}_1) \det( \widehat{\mbf{R}}_2) \end{align} where the sum is over all pairs of $n\times n$ matrices $(\widehat{\mbf{R}}_1, \widehat{\mbf{R}}_2)$ which can be obtained from $(\mbf{R}_1, \mbf{R}_2)$ by interchanging the $m$ columns of $\mbf{R}_1$ corresponding to $I$ with $m$ columns of $\mbf{R}_2$, while preserving the ordering of the columns. Fix $1\le a\le n-1$, $\cA\subset \lst{2n}$ with $|\cA|=n$, and $\cS\subset \cA$ with $|\cS|=a$. Let $\mbf{R}_1$ be the $n\times n$ Vandermonde matrix corresponding to $\cA$, $\mbf{R}_2$ the Vandermonde matrix corresponding to $\cA'$, and consider Sylvester's identity applied for $m=n-a$ where the fixed set of coordinates of $\mbf{R}_1$ correspond to $\widetilde{\cS}:=\cA\setminus \cS$. For each $(\widehat{\mbf{R}}_1,\widehat{\mbf{R}}_2)$ in the sum, let $\cB$ be the set of indices corresponding to the columns of $\widehat{\mbf{R}}_1$, then $|\cB|=n, \cA\cap \cB=\cS$. We set $\cT=\cA' \cap \cB'$ and $\widetilde{\cT}=\cA'\setminus \cT$. The left side of \eqref{eq:Sylvester} is $\Delta(\cA)\Delta(\cA')$, while the sum on the right side is \begin{align} \sum_{|\cB|=n, \cA\cap \cB=\cS}\sgn(\sigma_{\cA,\widetilde \cS\to \widetilde \cT}) \sgn(\sigma_{\cA',\widetilde \cT\to \widetilde \cS}) \Delta(\cB)\Delta(\cB'). \end{align} Here $\sigma_{\cA,\widetilde \cS\to \widetilde \cT}$ is the permutation of the columns of $\cA$ that we get after replacing the columns corresponding to $\wt \cS$ with the columns corresponding to $\wt \cT$ from $\cA' $ (keeping their order), and then reordering them based on their indices. The permutation $\sigma_{\cA',\widetilde \cT\to \widetilde \cS}$ is defined similarly. The identity \eqref{eq:det_sylv} now follows if we show that \begin{align}\label{eq:perm_sgn} \sgn(\sigma_{\cA,\widetilde \cS\to \widetilde \cT}) \sgn(\sigma_{\cA',\widetilde \cT\to \widetilde \cS})=(-1)^{n-a} \sgn(\sigma_{\cA}) \sgn(\sigma_{\cB}). \end{align} We will do this by transforming the permutation $\sigma_{\cA}$ into $\sigma_{\cB}$ while keeping track of the signature of the required steps. Consider the permutation $\sigma_{\cA}$. Interchange the indices in $\wt \cS$ with $\wt \cT$ (keeping their respective order), since $|\wt \cS|=|\wt \cT|=n-a$, we can achieve this with $n-a$ transpositions. Then reorder the elements in positions $1,3,\dots, 2n-1$, and then in the positions $2,4, \dots, 2n$. We can do this using the permutations $\sigma_{\cA,\widetilde \cS\to \widetilde \cT}$ and $\sigma_{\cA',\widetilde \cT\to \widetilde \cS}$ applied to these collections of indices, so the signature of this step is exactly $\sgn(\sigma_{\cA,\widetilde \cS\to \widetilde \cT}) \sgn(\sigma_{\cA',\widetilde \cT\to \widetilde \cS})$. The resulting permutation is exactly $\sigma_{\cB}$, and the signature of the steps we have taken is $(-1)^{n-a}\sgn(\sigma_{\cA,\widetilde \cS\to \widetilde \cT}) \sgn(\sigma_{\cA',\widetilde \cT\to \widetilde \cS})$, which proves \eqref{eq:perm_sgn}. \end{proof} \section{Evaluating $E_{\boldsymbol{Q}} \left[|\det \mbf{M}({\lambda}, \mbf{Q})|^4\right]$ for $r=2$} \label{sec:4moment} We show: \begin{proposition}\label{prop:det_l=4} Fix $n\ge 2$. Let $\mbf{X}$ be a $2\times (2n)$ matrix of independent $\FF N(0,1)$ random variables. For all ${\lambda}\in \R^{2n}$ it holds that \begin{align}\label{eq:det4} E |\det \mbf{M}({\lambda}, \mbf{X})|^4 = (12/\beta)^n \sum_{\cA\subset \lst{2n}, |\cA|=n} \Delta (\cA)^4 \Delta(\cA')^4. \end{align} \end{proposition} This establishes the second statement of Theorem \ref{thm:moment} via the Gaussian reduction Lemma \ref{lem:Gauss_red}. \begin{proof} We carry out the proof in the $\beta=1$ case, when the entries of $\mbf{X}$ are real standard Gaussians, commenting on the necessary modifications when $\beta =2$ at the end. As both sides of the claimed identity are continuous in $\lambda_{i}, i\in \lst{2n}$ , we can assume that those variables are distinct. Next, by Proposition \ref{prop:M_expansion} we have \[ \det \mbf{M}({\lambda},\mbf{X})=\sum_{\cA\subset \lst{2n}, |\cA|=n} \sgn(\sigma_{\cA}) X_1(\cA)X_2(\cA') \Delta(\cA)\Delta(\cA') \] and so also \begin{align}\label{eq:det^4} E[ \det \mbf{M}^4]&=\sum_{\substack{\cA_i\subset \lst{2n}, |\cA_i|=n\\ 1\le i\le 4}} \prod_{i=1}^4 \sgn(\sigma_{\cA_i}) E \left [\prod_{i=1}^4 X_1(\cA_i) X_2(\cA_i') \right] \prod_{i=1}^4 \Delta(\cA_i)\Delta(\cA_i'). \end{align} For a given choice of $\cA_1,\cA_2, \cA_3, \cA_4$ and $(j_1,j_2,j_3,j_4)\in \{0,1\}^4$ let \begin{equation} \cB_{(j_1,j_2,j_3,j_4)}=\cA_1^* \cA_2^* \cA_3^* \cA_4^*, \quad \text{with} \quad \cA_i^*=\begin{cases} \cA_i,& \qquad \text{if } j_i=1,\\ \cA_i',& \qquad \text{if } j_i=0. \label{overlaps} \end{cases} \end{equation} Also set $b_{(j_1,j_2,j_3,j_4)}=|\cB_{(j_1,j_2,j_3,j_4)}|$. Because the entries of $\mbf{X}$ are i.i.d.~standard normals, the expected value of $\prod_{i=1}^4 X_1(\cA_i) X_2(\cA_i')$ is zero if any entry of $\mbf{X}$ appears an odd number of times. Since the fourth moment of a standard normal is 3, this yields \begin{align}\label{eq:det4_111} E\left[\prod_{i=1}^4 X_1(\cA_i) X_2(\cA_i')\right]=3^{b_{(1,1,1,1)}+b_{(0,0,0,0)}} \ind(b_{(j_1,j_2,j_3,j_4)}=0 \text{ if $\sum_{i=1}^4 j_i$ is odd}). \end{align} In other words, after we take expected value in \eqref{eq:det^4}, only those quadruples $(\cA_1,\dots, \cA_4)$ contribute for which intersections of the form $\cA_{i_1}' \cA_{i_2} \cA_{i_3} \cA_{i_4}$ and $\cA_{i_1} \cA_{i_2}' \cA_{i_3}' \cA_{i_4}'$ are empty (for distinct $i_1,\dots, i_4$). Let $\cG_n$ denote the set of quadruples $(\cA_1,\dots, \cA_4)$ that satisfy this compatibility condition together with $|\cA_i|=n$, $\cA_i\subset \lst{2n}$. To simplify notation, we encode the sequences $(j_1,j_2,j_3,j_4)$ where $\sum_{i=1}^4 j_i$ is even with the integers 1, \dots, 8 as follows. \begin{align} \label{setlist} \begin{array}{c|c|c|c|c|c|c|c} 1111 & 1100 & 1010 & 1001 & 0110 & 0101 & 0011 & 0000\\ \hline 1 & 2 & 3 & 4 & 5 & 6 & 7 & 8\\ \end{array} \end{align} Hence $\cB_{(1,0,1,0)}$ and $\cB_3$ both denote the set $\cA_1 \cA_2' \cA_3 \cA_4'$, and $b_3=b_{(1,0,1,0)}$. At the same time, if $(\cA_1,\dots, \cA_4)\in \cG_n$, then each $\cA_i$ and $\cA_i'$ can be written as the disjoint union of four of the sets $\cB_j, 1\le j\le 8$. For example, $\cA_1=\cB_1\cup \cB_2\cup \cB_3\cup \cB_4$. From $|\cA_i|=|\cA_i'|$, $1\le i\le 4$, we get four equations with the sum of four terms on both sides. For instance, $|\cA_1|=|\cA_1'|$ yields \[ b_1+b_2+b_3+b_4=b_5+b_6+b_7+b_8, \] and so on. These four equations can be readily reduced to the simpler identities \begin{align} b_j=b_{9-j}, \qquad 1\le j\le 4. \end{align} Equations \eqref{eq:det4} and \eqref{eq:det4_111} together with $b_1=b_8$ lead to \begin{align}\label{eq:det_4_2} E\left[ \det \mbf{M}^4\right]=\sum_{(\cA_1,\dots,\cA_4)\in \cG_n} 9^{ b_1} \prod_{i=1}^4 \sgn(\sigma_{\cA_i}) \prod_{i=1}^4 \Delta(\cA_i)\Delta(\cA_i'). \end{align} Next, for a particular $(\cA_1,\dots, \cA_4)\in \cG_n$ consider the following ``special'' ordering of the elements of $\lst{2n}$: $x\prec y$ if $x\in \cB_i, y\in \cB_j$ with $i<j$ or if $x<y$ with $x,y\in \cB_i$. Denote by $\wt \Delta(\cA)$ the Vandermonde determinant corresponding to the elements $\lambda_i, i\in \cA$ using our special ordering, and let $\sigma_i$ be the permutation that maps $1,3,\dots, 2n-1$ and $2,4,\dots, 2n$ into the elements of $\cA_i$ and $\cA_i'$ according to the special order. These permutations can be visualized using the following table: \begin{align} \begin{array}{r|c|c} &1,3,\dots,2n-1&2,4,\dots,2n \\ \hline \sigma_1&\cB_1, \cB_2,\cB_3,\cB_4&\cB_5, \cB_6,\cB_7,\cB_8\\ \hline \sigma_2&\cB_1, \cB_2,\cB_5,\cB_7&\cB_3, \cB_4,\cB_7,\cB_8\\ \hline \sigma_3&\cB_1, \cB_3,\cB_5,\cB_7&\cB_2, \cB_4,\cB_6,\cB_8\\ \hline \sigma_4&\cB_1, \cB_4,\cB_6,\cB_7&\cB_2, \cB_3,\cB_5,\cB_8 \end{array} \end{align} Note that for each $1\le i\le 4$ the permutation $\sigma_{\cA_i}$ can be obtained from $\sigma_{i}$ by reordering the elements $\sigma(2i-1),1\le i\le n$ and $\sigma(2i),1\le i\le n$, respectively. These two permutations correspond to the column permutations that take the Vandermonde matrices corresponding to $\wt \Delta(\cA_i)$ and $\wt \Delta(\cA_i')$ into the Vandermonde matrices of $\Delta(\cA_i)$ and $\Delta(\cA_i')$. From this observation we get \begin{align} \sgn(\sigma_{\cA_i}) \Delta(\cA_i)\Delta(\cA_i')= \sgn(\sigma_i) \wt \Delta(\cA_i)\wt \Delta(\cA_i')\qquad 1\le i\le 4, \end{align} and hence \begin{align}\label{eq:det_4_3} E\left[ \det \mbf{M}^4\right]=\sum_{(\cA_1,\dots,\cA_4)\in \cG_n} 9^{ b_1} \prod_{i=1}^4 \sgn(\sigma_i) \prod_{i=1}^4 \wt\Delta(\cA_i)\wt\Delta(\cA_i'), \end{align} as an equivalent form of \eqref{eq:det_4_2}. One reason for introducing the above ordering is that: for every $(\cA_1,\dots,\cA_4)\in \cG_n$, \[ \prod_{i=1}^4 \sgn(\sigma_i)=1. \] To see this, first note that since $|\cB_3|=|\cB_6|=b_3$ and $|\cB_4|=|\cB_5|=b_4$, we can obtain $\sigma_2$ from $\sigma_1$ using the following steps: switch the respective elements of $\cB_3$ with $\cB_6$, and then the elements of $\cB_4$ with those of $\cB_5$ (this takes $b_3+b_4$ transpositions). Then switch the order of the `blocks' $\cB_6$ and $\cB_5$, and the blocks $\cB_4$ and $\cB_3$ (these two steps can be done with the same number of transpositions). We can visualize these steps as follows: \[ \left(\cB_1, \cB_2,\cB_3,\cB_4|\cB_5, \cB_6,\cB_7,\cB_8\right)\to \left(\cB_1, \cB_2,\cB_6,\cB_5|\cB_4, \cB_3,\cB_7,\cB_8\right) \to \left(\cB_1, \cB_2,\cB_5,\cB_6|\cB_3, \cB_4,\cB_7,\cB_8\right). \] This shows that \begin{align}\label{eq:sign_1} \sgn(\sigma_1)=\sgn(\sigma_2) (-1)^{b_3+b_4}. \end{align} Similarly, we can obtain $\sigma_4$ from $\sigma_3$ by switching the respecting elements of $\cB_i$ with $\cB_{9-i}$ for $i=3,4$, and then switch the order of $\cB_6, \cB_4$ and $\cB_5, \cB_3$: \[ \left(\cB_1, \cB_3,\cB_5,\cB_7|\cB_2, \cB_4,\cB_6,\cB_8\right)\to \left(\cB_1, \cB_6,\cB_4,\cB_7|\cB_2, \cB_5,\cB_3,\cB_8\right) \to \left(\cB_1, \cB_4,\cB_6,\cB_7|\cB_2, \cB_3,\cB_5,\cB_8\right). \] This yields \begin{align}\label{eq:sign_2} \sgn(\sigma_3)=\sgn(\sigma_4) (-1)^{b_3+b_4}. \end{align} Combining \eqref{eq:sign_1} and \eqref{eq:sign_2} produces the claimed $\prod_{i=1}^4 \sgn(\sigma_i)=1$. At this point then we have the identity, \begin{align} \label{eq:det_4_4} E\left[ \det \mbf{M}^4\right]=\sum_{(\cA_1,\dots,\cA_4)\in \cG_n} 9^{b_1} \prod_{i=1}^4 \wt\Delta(\cA_i)\wt\Delta(\cA_i'), \end{align} and our next step is to rewrite the right hand side of \eqref{eq:det_4_4} as a product of Cauchy determinants. Towards this, for two disjoint sets $\cS_1, \cS_2\subset \lst{2n}$ we introduce the notation \begin{align}\label{eq:defdelta} \dd(\cS_1,\cS_2):= \prod_{s_1\in \cS_1} \prod_{s_2\in \cS_2}(\lambda_{s_1}-\lambda_{s_2}), \end{align} with the empty product defined as 1. For a particular $(\cA_1,\dots, \cA_4)\in \cG_n$ for each $1\le i\le 4$ we can group the variables into the sets $\cB_j$ and write \begin{align}\label{eq:prod_det_0} \wt\Delta(\cA_i)\wt\Delta(\cA_i')=\prod_{j=1}^8 \Delta(\cB_j)\prod_{a_1>a_2} \dd(\cB_{a_1}, \cB_{a_2})\prod_{b_1>b_2} \dd(\cB_{b_1}, \cB_{b_2}). \end{align} Here $a_1,a_2$ are indices from the ``$\cB$-partition'' of $\cA_i$ and $b_1,b_2$ are indices from the the ``$\cB$-partition'' of $\cA_i'$. Multiplying these equations for $1\le i\le 4$ and some bookkeeping yields \begin{align}\label{eq:prod_det_44} \prod_{i=1}^4\wt\Delta(\cA_i)\wt\Delta(\cA_i')=\prod_{j=1}^8 \Delta(\cB_j)^4 \prod_{\substack{9- j_1\neq j_2\\1\le j_2< j_1\le 8}} \dd(\cB_{j_1}, \cB_{j_2})^2. \end{align} A similar argument gives \begin{align}\label{eq:Delta_prod} \Delta(\lambda)^2=\prod_{j=1}^8 \Delta(\cB_j)^2 \prod_{1\le a<b\le 8} \dd(\cB_b, \cB_a)^2. \end{align} Recall next the Cauchy determinant formula. For two disjoint sets $\cS_1, \cS_2\subset \lst{2n}$ with $|\cS_1|=|\cS_2|$ we denote \begin{align}\label{eq:Cauchy_det} C_{\cS_1,\cS_2}=\det \left(\frac{1}{\lambda_{s_1}-\lambda_{s_2}}\right)_{s_1\in \cS_1, s_2, \cS_2} = (-1)^{\binom{|\cS_1|}{2}} \cdot \frac{\Delta(\cS_1)\Delta(\cS_2)}{\dd(\cS_1,\cS_2)}. \end{align} (If $\cS_1, \cS_2$ are empty we set $C_{\cS_1,\cS_2}=1$). From \eqref{eq:prod_det_44} and \eqref{eq:Delta_prod} we may deduce that \begin{align}\label{eq:prod_det_4} \Delta(\lambda)^{-2} \prod_{i=1}^4\wt\Delta(\cA_i)\wt\Delta(\cA_i')=\prod_{j=1}^4 C_{\cB_j,\cB_{9-j}}^2, \end{align} for again any $(\cA_1,\dots, \cA_4)\in \cG_n$ The point is that by Lemma \ref{lem:Cauchy_cycle} below we have the following. Suppose that $b_i, 1\le i\le 8$ are fixed nonnegative integers with $b_i=b_{9-i}$ and $\sum_{i=1}^4 b_i=n$. Let $\cP_{\underline{b}}$ be the collection of partitions $\underline{\cB}=(\cB_1,\dots, \cB_8)$ of $\lst{2n}$ with $|\cB_i|=b_i, 1\le i\le 8$. Then \begin{align}\label{eq:Cauchy_sum} \sum_{\underline{\cB}\in \cP_{\underline{b}}} \prod_{i=1}^4 C_{\cB_i,\cB_{9-i}}^2 =\binom{n}{b_1,b_2,b_3,b_4} \sum_{\cA\subset \lst{2n}, |\cA|=n} C_{\cA,\cA'}^2. \end{align} Taking \eqref{eq:Cauchy_sum} for granted, \eqref{eq:prod_det_4} allows us to rewrite \eqref{eq:det_4_4} as in \begin{align}\notag E\left[ \det \mbf{M}^4\right]&=\Delta(\lambda)^2\sum_{b_1+\cdots+b_4=n, b_i\ge 0} 9^{b_1} \binom{n}{b_1,b_2,b_3,b_4} \sum_{\cA\subset \lst{2n}, |\cA|=n} C_{\cA,\cA'}^2\\ &=12^n \times \Delta(\lambda)^2 \sum_{\cA\subset \lst{2n}, |\cA|=n} C_{\cA,\cA'}^2\label{eq:det4_9}, \end{align} having used the multinomial theorem in the second line. We can now finish the proof. For any $\cA\subset \lst{2n}$ with $|\cA|=n$, \begin{align}\label{eq:det2n_C} \Delta(\lambda)^2= \Delta(\cA)^2 \Delta(\cA')^2 \dd(\cA,\cA')^2, \end{align} which by the Cauchy determinant formula implies \[ \Delta(\lambda)^2 C_{\cA,\cA'}^2= \Delta(\cA)^4 \Delta(\cA')^4. \] Substituting the above into \eqref{eq:det4_9} produces \[ E\left[ \det \mbf{M}^4\right]=12^n \sum_{\cA\subset \lst{2n}, |\cA|=n} \Delta(\cA)^4 \Delta(\cA')^4, \] which is the claimed statement for $\beta=1$. For $\beta =2$, the key change comes in equation \eqref{eq:det4_111}, where one now gets \begin{align}\label{eq:new_det4} &E \left [\prod_{i=1}^2 X_1(\cA_i) X_2(\cA_i') \prod_{i=3}^4 \overline{X}_1(\cA_i) \overline{X}_2(\cA_i')\right]\\ &\hskip100pt \notag=2^{b_{(1,1,1,1)}+b_{(0,0,0,0)}} \ind(b_{(j_1,j_2,j_3,j_4)}=0 \text{ if $j_1+j_2\neq j_3+j_4$}). \end{align} The coefficient $2$ and the updated condition in the indicator function result from the fourth absolute moment and the rotational invariance of the standard complex Gaussian distribution, respectively. Equation \eqref{eq:new_det4} now implies that if the sets $\cB_{(1,0,1,0)}$ and $\cB_{(0,1,0,1)}$ are not empty then the corresponding quadruple $(\cA_1,\dots,\cA_4)$ does not contribute in the sum analogous to \eqref{eq:det^4}. This translates to having $b_3=b_6=0$ in all subsequent calculations. In particular, the multinomial factor in \eqref{eq:det4_9} is replaced by $ 4^{b_1} \binom{n}{b_1,b_2,b_4}$, which explains the reported constant of $6^n$ for $\beta=2$. \end{proof} \section{Determinantal identities} \label{sec:det_identities} The main goal of the section is to prove the following identity which is crucial in the proof of Proposition \ref{prop:det_l=4}. \begin{lemma}\label{lem:Cauchy_cycle} Let $n\ge 2$, and suppose that $b_i, 1\le i\le 8$ are fixed nonnegative integers with $b_i=b_{9-i}$ and $\sum_{i=1}^4 b_i=n$. Let $\cP_{\underline{b}}$ be the set of partitions $\underline{\cB}=(\cB_1,\dots, \cB_8)$ of $\lst{2n}$ with $|\cB_i|=b_i, 1\le i\le 8$. Then \begin{align}\label{eq:Cauchy_sum_1} \sum_{\underline{\cB}\in \cP_{\underline{b}}} \prod_{j=1}^4 C_{\cB_j,\cB_{9-j}}^2 =\binom{n}{b_1,b_2,b_3,b_4} \sum_{\cA\subset \lst{2n}, |\cA|=n} C_{\cA,\cA'}^2. \end{align} \end{lemma} We also establish the following which unifies the various descriptions of our $r=2$ eigenvalue densities. \begin{lemma}\label{lem:det4_identities} Fix $n\ge 2$, and let ${\lambda}\in \R^{2n}$. Then the following identities hold: \begin{align}\label{eq:det4_id_1} \sum_{\cA\subset \lst{2n}, |\cA|=n} \Delta (\cA)^4 \Delta (\cA')^4 & =2^{-n} \left(\sum_{\cA\subset \lst{2n}, |\cA|=n} \Delta (\cA)^2 \Delta (\cA')^2\right)^2 \\ & =2^n \Delta(\lambda)^2 \left|\Pf\left(\frac{\ind_{i\neq j}}{\lambda_i-\lambda_j}\right)\right|^2. \label{eq:det4_id_2} \end{align} \end{lemma} First though we present a conjectured generalization of Lemma \ref{lem:Cauchy_cycle} motivated by the proof of Proposition \ref{prop:det_l=4}. \subsection{A conjecture on certain Vandermonde sums} The Gaussian reduction was key to computing $E_{\boldsymbol{Q}} \left[|\det \mbf{M}({\lambda}, \mbf{Q})|^4\right]$ as it vastly cuts down the number of terms on the right hand side of the basic formula \eqref{eq:det^4}. Assuming however that the end result is only a function of the exchangeability of the rows and columns of $\mbf{Q}$, leads to an interesting set of conjectures involving sums of squared Vandermonde determinants. To explain, recall the enumeration of ``overlaps" $\cB_{(j_1,j_2, j_3, j_4)}$ defined in \eqref{overlaps}. Here $j_i \in \{0,1\}$, and previously all overlaps for which $\sum_{i=1}^4 j_i$ is odd immediately dropped from consideration. Restoring these, we have sixteen possible overlaps $(j_1, j_2, j_3, j_4)$ which we again list in reverse lexicographic order, and now denote their sizes by $x_i$, $i \in \{1, \dots, 16\}$. (Compare with \eqref{setlist}.) In particular, we now have present for example $\cB_{(1,1,0,1)}$ as the third element on the list for which we set $x_3 = |B_{(1,1,0,1)}| $. With these conventions set we can state: \begin{conjecture} Fix nonnegative integers $x_1, \dots, x_{16}$ with $x_1+\dots+x_8=n$. Then \begin{align} \label{conjecture} &\sum_{(\cA_1,\dots,\cA_4)\subset \mathcal{S}_{\mbf{x}}} \prod_{i=1}^4 \sgn(\sigma_{\cA_i}) \Delta_{\cA_i}^2 \Delta_{\cA_i'}^2 \\ & \qquad\qquad = \ind(x_i=x_{17-i}) \, (-1)^{x_2+x_3+x_5+x_7} { n \choose x_1, \dots , x_8 } \sum_{\cA \subset \lst{2n}, |\cA|=n} \Delta_{\cA}^4 \Delta_{\cA'}^4. \nonumber \end{align} Here $\mathcal{S}_{\mbf{x}}$ is the collection of all quadruples $(\cA_1,\dots,\cA_4)$ where the overlap sizes are given by $x_1,\dots, x_{16}$. \end{conjecture} The identity \eqref{eq:Cauchy_sum_1} of Lemma \ref{lem:Cauchy_cycle}, which can be recast in terms of Vandermonde instead of Cauchy determinants, is in fact the special case of \eqref{conjecture} where one restricts the sum on the left hand side to ``even" symmetric overlaps (for which $x_2 = x_3 = x_5=x_8 = 0$). \subsection{Proofs of Lemmas \ref{lem:Cauchy_cycle} and \ref{lem:det4_identities}} We start by recalling the definitions of the Pfaffian and Hafnian of a matrix. \begin{definition}\label{def:PfaffHaf} For a $2n \times 2n$ skew-symmetric matrix $M$, the Pfaffian of $M$ is defined by \begin{align}\label{eq:Pfaffiandef} \Pf(M)=\frac{1}{2^n n!} \sum_{\sigma\in S_{2n}} \sgn(\sigma) \prod_{i=1}^n M_{{\sigma(2i-1)},{\sigma(2i)}} = \sum_{\alpha\in \Pi_{2n}} \sgn(\pi_\alpha) \prod_{i=1}^n M_{\alpha_{i,1},\alpha_{i,2}}, \end{align} while for a $2n \times 2n$ symmetric matrix $M$, the Hafnian of $M$ is defined by \begin{align}\label{eq:Hafniandef} \Hf(M)=\frac{1}{2^n n!} \sum_{\sigma\in S_{2n}} \prod_{i=1}^n M_{{\sigma(2i-1)},{\sigma(2i)}} =\sum_{\alpha\in \Pi_{2n}} \prod_{i=1}^n M_{\alpha_{i,1},\alpha_{i,2}}. \end{align} Here $\Pi_{2n}$ refers to the set of matchings on $\lst{2n}$, {\em{i.e}}, partitions $(\alpha_{1,1},\alpha_{1,2}), \dots (\alpha_{n,1},\alpha_{n,2})$ of $\lst{2n}$ with $\alpha_{i,1}<\alpha_{i,2}$ and $\alpha_{1,1}<\cdots<\alpha_{n,1}$. In the Pfaffian formula, $\pi_\alpha$ denotes the corresponding permutation $(\alpha_{1,1},\alpha_{1,2}, \alpha_{2,1},\alpha_{2,2}\dots \alpha_{n,1},\alpha_{n,2})$. \end{definition} Breaking the proof of Lemma \ref{lem:det4_identities} into two parts,, we start with: \begin{proposition}\label{prop:det2+Pf} Let ${\lambda}\in \R^{2n}$. Then we have \begin{align}\label{eq:det2+Pf} \sum_{|\cA|=n, \cA\subset\lst{2n}} \Delta(\cA)^2 \Delta (\cA')^2&=(-1)^n 2^n \Delta (\lambda) \Pf\left(\frac{\ind_{i\neq j}}{\lambda_i-\lambda_j}\right). \end{align} The right side is defined as the appropriate limit if the $\lambda_i$'s are not all distinct. \end{proposition} \begin{proof} Assuming that all the $\lambda_i$'s are distinct, the definition \eqref{eq:Pfaffiandef} gives \begin{align} \Pf\left(\frac{\ind_{i\neq j}}{\lambda_i-\lambda_j}\right)& =\sum_{\alpha\in \Pi_{2n}} \sgn(\pi_\alpha) \prod_{i=1}^n \frac{1}{\lambda_{\alpha_{i,1}}-\lambda_{\alpha_{i,2}}}. \end{align} A matching $\alpha \in \Pi_{2n}$ can be encoded with a pair $(\cA, \tau)$ where $|\cA|=n, \cA\subset \lst{2n}$ and $\tau\in S_n$. Indeed, $\cA=\{\alpha_{1,1}, \dots, \alpha_{n,1}\}$ and the permutation $\tau\in S_n$ that takes the ordered version of $\cA'$ into $(\alpha_{1,2}, \dots, \alpha_{n,2})$ identifies the matching $\alpha$. Note that $\sgn(\pi_\alpha)$ is equal to $\sgn(\sigma_{\cA} )\sgn(\eta_n)$ where $\sigma_{\cA}$ is defined in \eqref{shorthand}, and $\eta_n$ is the permutation $(1,n+1,2, n+2, \dots, n, 2n)$. This leads to \begin{align} \label{eq:Pf4_2} \Pf\left(\frac{\ind_{i\neq j}}{\lambda_i-\lambda_j}\right)=\sgn(\eta_n) \sum_{|\cA|=n, \cA\subset \lst{2n}} \sgn(\sigma_{\cA}) \sum_{\tau\in S_n} \sgn(\tau) \prod_{i=1}^n \frac{1}{\lambda_{a'_{\tau(i)}}-\lambda_{a_i}}, \end{align} where we used $a_i, a_i', i\in \lst{n}$ to denote the (ordered) elements of $\cA$ and $ \cA'$, respectively. For a given $|\cA|=n, \cA\subset \lst{2n}$ we have \[ \Delta(\lambda)=\Delta(\cA)\Delta(\cA') \prod_{\substack{1\le i<j\le 2n\\ \left|\{i,j\}\cap \cA\right|=1}} (\lambda_j-\lambda_i). \] By \eqref{eq:defdelta} we have \[ \prod_{\substack{1\le i<j\le 2n\\ \left|\{i,j\}\cap \cA\right|=1}} (\lambda_j-\lambda_i)=(-1)^{k_\cA} \dd(\cA, \cA') \] with $k_{\cA}$ the number of pairs $(i,j)$ with $1\le i<j\le 2n$ and $i\in \cA, j\in \cA'$. Observe then that $n^2-k_{\cA}$ is exactly the number of inversions of the permutation $\sigma_{\cA}$. Hence, \begin{align} \label{eq:det_exp_A_Ac} \Delta (\lambda)=(-1)^n \sgn(\sigma_{\cA}) \Delta (\cA)\Delta (\cA') \dd(\cA, \cA'). \end{align} Using this identity we get \begin{align}\notag \sum_{|\cA|=n, \cA\subset\lst{2n}} \frac{\Delta (\cA)^2 \Delta (\cA')^2}{ \Delta(\lambda)}&=(-1)^n \sum_{|\cA|=n, \cA\subset\lst{2n}} \sgn(\sigma_{\cA}) \frac{\Delta(\cA) \Delta(\cA')}{\dd(\cA, \cA')} \\ &=(-1)^{n+\binom{n}{2}}\sum_{|\cA|=n, \cA\subset\lst{2n}} \sgn(\sigma_{\cA}) C_{\cA, \cA'} \, \label{eq:det4_123} \\ &=(-1)^{n+\binom{n}{2}}\sum_{|\cA|=n, \cA\subset\lst{2n}} \sgn(\sigma_{\cA}) \sum_{\tau\in S_n} \sgn(\tau) \frac{1}{\lambda_{a_i}-\lambda_{a'_{\tau(i)}}}, \notag \end{align} where in the last line we expanded the Cauchy determinant, and used that $a_i, a_i', i\in \lst{n}$ for the ordered elements of $\cA, \cA'$, respectively. Note that $\sgn(\eta_n)=(-1)^{\binom{n}{2}}$, hence by comparing \eqref{eq:det4_123} with \eqref{eq:Pf4_2} the statement follows. \end{proof} \begin{proposition}\label{prop:det4+det2} Let ${\lambda}\in \R^{2n}$. Then we have \begin{align}\label{eq:det4+det2} \sum_{\cA\subset \lst{2n}, |\cA|=n} \Delta (\cA)^4 \Delta (\cA')^4&=2^{n} \Delta(\lambda)^2 \Hf\left(\frac{\ind_{i\neq j}}{(\lambda_i-\lambda_j)^2}\right). \end{align} Again, the right side is defined as the appropriate limit if the $\lambda_i$'s are not all distinct. \end{proposition} To prove Proposition \ref{prop:det4+det2} we will use the following lemma, which is well-known in the statistical physics community. (See e.g.~(D.29) in \cite{FGM_1995}.) We include the proof for completeness. \begin{lemma}[Cycle cancellation]\label{lem:cycle} Suppose that $k\ge 3$, and $z_1, \dots, z_k$ are distinct numbers. Then \begin{align}\label{eq:cycle} \sum^*_{\sigma} \prod_{i=1}^k \frac{1}{z_i-z_{\sigma(i)}}=0, \end{align} where the sum is over all cycles $\sigma$ supported on $\{1,2,\dots,k\}$. We also have \begin{align}\label{eq:cycle_full} \sum_{\sigma\in S_k} \prod_{i=1}^k \frac{1}{z_{\sigma(i)}-z_{\sigma(i+1)}}=0, \end{align} where $\sigma(k+1)=\sigma(1)$. \end{lemma} \begin{proof} We have \begin{align}\label{eq:cycle_3} \frac{1}{(z_1-z_a)(z_b-z_1)}=\frac{1}{z_b-z_a}\cdot \left(\frac{1}{z_b-z_1}-\frac{1}{z_a-z_1}\right). \end{align} Fix a length $k-1$ cycle $(b_1, \dots, b_{k-1})$ on $\{1,\dots, k-1\}$, and consider all length $k$ cycles that we get by inserting $k$ at some point. Then by \eqref{eq:cycle_3} the contribution of all of these cycles in the sum in \eqref{eq:cycle} (using $b_k=b_1$) is \[ \left(\sum_{j=1}^{k-1} \frac{1}{z_{b_j}-z_k}-\frac{1}{z_{b_{j+1}}-z_k}\right) \prod_{j=1}^{k-1} \frac{1}{z_{b_j}-z_{b_{j+1}}}=0. \] This proves \eqref{eq:cycle}. The identity \eqref{eq:cycle_full} now also follows by observing that the sum in \eqref{eq:cycle_full} is exactly $k$ times the sum in \eqref{eq:cycle}. \end{proof} \begin{proof}[Proof of Proposition \ref{prop:det4+det2}] It is sufficient to prove the statement in the case where all the $\lambda_i$'s are distinct. We first write \begin{align}\label{eq:det4det2_0} \sum_{|\cA|=n, \cA\subset\lst{2n}} \frac{\Delta(\cA)^4 \Delta(\cA')^4}{ \Delta(\lambda)^2}& = \sum_{|\cA|=n, \cA\subset\lst{2n}} C_{\cA, \cA'}^2 \\ &=\frac{1}{(n!)^2} \sum_{\substack{a_1,\dots, a_n, a_1', \dots, a_n'\\\{a_1,\dots, a_n, a_1', \dots, a_n'\}=\lst{2n}}} C_{(a_1,\dots,a_n), (a_1', \dots, a_n')}^2.\label{eq:det4det2_1} \end{align} Here we used \eqref{eq:det_exp_A_Ac}, and in the second step we reordered the rows and columns of the Cauchy determinants in all possible ways. Next we expand the Cauchy determinants to get \begin{align}\nonumber & \sum_{|\cA|=n, \cA\subset\lst{2n}} \frac{ \Delta (\cA)^4 \Delta (\cA')^4}{ \Delta (\lambda)^2}=\\ &\hskip100pt\frac{1}{(n!)^2}\sum_{a_i, a_i'} \sum_{\sigma, \tau\in S_n} \sgn(\sigma\circ \tau^{-1}) \prod_{i=1}^n \frac{1}{(\lambda_{a_i} - \lambda_{a'_{\sigma(i)}})(\lambda_{a_i} - \lambda_{a'_{\tau(i)}})}. \label{eq:det4_sum} \end{align} (The restrictions on $a_i, a_i'$ are the same as in \eqref{eq:det4det2_1}.) We first evaluate the diagonal part of the double sum in \eqref{eq:det4_sum} by showing that \begin{align}\label{eq:det4_diag} \frac{1}{(n!)^2}\sum_{a_i, a_i'} \sum_{\sigma\in S_{n}} \prod_{i=1}^n \frac{1}{(\lambda_{a_i} - \lambda_{a'_{\sigma(i)}})^2}=2^n \Hf \left( \frac{\ind_{i \neq j }}{(\lambda_i -\lambda_j)^{2}} \right). \end{align} Consider the mapping that takes a particular choice of $a_1,\dots, a_n, a_1', \dots, a_n'$ and $\sigma\in S_n$ into the matching of $\lst{2n}$ that matches $a_i$ with $a_{\sigma(i)}'$ for $1\le i\le n$. Each particular matching $\alpha\in \Pi_{2n}$ shows up exactly $2^n (n!)^2$ times as the result of this mapping, which proves \eqref{eq:det4_diag}. To complete the proof of our proposition we just need to show that the `non-diagonal' terms in the sum \eqref{eq:det4_sum} cancel out. For this it is sufficient to show that if $\sigma\neq \tau$ are fixed elements of $S_n$ then \begin{equation} \label{eq:off_diag_vanish} \sum_{\substack{a_1,\dots, a_n, a_1', \dots, a_n'\\\{a_1,\dots, a_n, a_1', \dots, a_n'\}=\lst{2n}}} \prod_{i=1}^n \frac{1}{(\lambda_{a_i} - \lambda_{a'_{\sigma(i)}})(\lambda_{a_i} - \lambda_{a'_{\tau(i)}})}=0. \end{equation} For a particular pair $\sigma, \tau$ consider the permutation on $\lst{2n}=\{a_1,\dots, a_n, a_1', \dots, a_n'\}$ that takes $a_i$ to $a_{\sigma_i}'$ and $a_i'$ to $a_{\tau^{-1}(i)}$. This permutation has even cycles, and because $\sigma\neq \tau$ it must have at least one cycle of length at least 4. Let one of these cycles be \begin{align}\label{eq:cycle_123} a_{i_1}\to a'_{j_1} \to a_{i_2} \to a'_{j_2} \to \dots a'_{j_k}\to a_{i_1}. \end{align} Here $1\le i_1,\dots,i_k\le n$ are distinct, and the same holds for $j_1, \dots, j_k$. Let $\cB\subset \lst{2n}$ with $|\cB|=2k$. We claim that \begin{align}\label{eq:off_diag_vanish_1} \sum_{\substack{a_1,\dots, a_n, a_1', \dots, a_n'\\\{a_1,\dots, a_n, a_1', \dots, a_n'\}=\lst{2n}\\ \{a_{i_1},a'_{j_1}, a_{i_2}, a'_{j_2} \dots, a_{i_k}, a'_{j_k}\}=\cB }} \prod_{i=1}^n \frac{1}{(\lambda_{a_i} - \lambda_{a'_{\sigma(i)}})(\lambda_{a_i} - \lambda_{a'_{\tau(i)}})}=0. \end{align} The sum on the left can be rewritten as a double sum where we first sum over all possible assignments of the values of $a_{i_1},a'_{j_1}, a_{i_2}, a'_{j_2} \dots, a_{i_k}, a'_{j_k}$, and then in the second sum we sum over the remaining $2n-2k$ variables. Factoring out the terms corresponding to the indices not in $\cB$, the inner sum becomes \[ \sum_{\{a_{i_1},a'_{j_1}, a_{i_2}, a'_{j_2} \dots, a_{i_k}, a'_{j_k}\}=\cB} \prod_{\ell=1}^k \frac{1}{\lambda_{a_{i_\ell}}-\lambda_{a'_{j_\ell}}}\cdot \frac{1}{\lambda_{a'_{j_\ell}}-\lambda_{a_{i_{\ell+1}}}} \] with $i_{k+1}=i_1$. But this is equal to 0 by \eqref{eq:cycle_full} of Lemma \ref{lem:cycle}, as applied to the sum over the permutations of elements of $\cB$ and the values $\lambda_{a_{i_1}}, \lambda_{a'_{j_1}}, \dots, \lambda_{a_{i_k}}, \lambda_{a'_{j_k}}$. This proves \eqref{eq:off_diag_vanish_1}, which gives \eqref{eq:off_diag_vanish} and the statement of the proposition. \end{proof} We now have all the components to prove Lemma \ref{lem:det4_identities}. \begin{proof}[Proof of Lemma \ref{lem:det4_identities}] The proof follows from the statements of Proposition \ref{prop:det2+Pf} and \ref{prop:det4+det2} once we establish that \begin{align*}\label{eq:Pf2=Hf} \Pf\left(\frac{\ind_{i\neq j}}{\lambda_i-\lambda_j}\right)^2= \det\left(\frac{\ind_{i\neq j}}{\lambda_i-\lambda_j}\right) = \Hf\left(\frac{\ind_{i\neq j}}{(\lambda_i-\lambda_j)^2}\right). \end{align*} The first equality here is due the standard fact that square of the Pfaffian of a skew-symmetric matrix is equal to its determinant. The second has in fact been noticed before in \cite{DSZ}, and can be seen by expansion: \[ \det\left(\frac{\ind_{i\neq j}}{\lambda_i-\lambda_j}\right)=\sum_{\sigma\in S_{2n}, \sigma(j)\neq j} \sgn(\sigma) \prod_{j=1}^{2n} \frac{1}{\lambda_j-\lambda_{\sigma(j)}}, \] where the sum is over all the permutations of $\lst{2n}$ that have no fixed elements. Because of Lemma \ref{lem:cycle}, the contribution of the permutations that have a cycle of length at least 3 cancels out. The remaining terms correspond to the permutations that only have 2-cycles, that is, the permutations that corresponding to perfect matchings of $\lst{2n}$. For such a permutation $\sigma$ we have \[ \sgn(\sigma) \prod_{j=1}^{2n} \frac{1}{\lambda_j-\lambda_{\sigma(j)}}=\prod_{i\in \cA} \frac{1}{(\lambda_{i}-\lambda_{\sigma(i)})^2} \] where $\cA$ is a set containing an element from each 2-cycle. By \eqref{eq:Hafniandef} the sum of these terms is exactly the Hafnian of the $n\times n$ matrix with $(i,j)$ entry $\frac{\ind_{i\neq j}}{(\lambda_i-\lambda_j)^2}$. \end{proof} We are now ready to prove Lemma \ref{lem:Cauchy_cycle}. \begin{proof}[Proof of Lemma \ref{lem:Cauchy_cycle}] We first note that by \eqref{eq:det4det2_0} and Proposition \ref{prop:det4+det2} we have \begin{align}\label{eq:C_Hf} \sum_{\cA\subset \lst{2n}, |\cA|=n} C_{\cA,\cA'}^2=2^n \Hf\left(\frac{\ind_{i\neq j}}{(\lambda_i-\lambda_j)^2}\right). \end{align} Fix $b_1, \dots, b_4$ with $\sum_{i=1}^4 b_i=n$ and set $b_i=b_{9-i}$ for $5\le j\le 8$. For a particular $\sigma\in S_{2n}$ denote by $\cB_{\sigma,1}, \dots, \cB_{\sigma,4},\cB'_{\sigma,1}, \dots, \cB'_{\sigma,4}$ the ordered lists we obtain once we partition $(\sigma(1),\dots, \sigma(2n))$ into parts of lengths $b_1,b_2, b_3, b_4, b_8, b_7, b_6, b_5$. Then we have \begin{align}\label{eq:Cauchy_B4_1} \sum_{\underline{\cB}\in \cP_{\underline{b}}} \prod_{i=1}^4 C_{\cB_i,\cB_{9-i}^2} =\frac{1}{(b_1! b_2! b_3! b_4!)^2}\sum_{\sigma\in S_{2n}} \prod_{j=1}^4 C^2_{\cB_{\sigma,j},\cB'_{\sigma,j}}. \end{align} We introduce the temporary notation $z_i=\lambda_{\sigma(i)}$ and $z'_i=\lambda_{\sigma(n+i)}$ for $1\le i\le n$, and also $\cD_1=\{1,\dots, b_1\}$ $\cD_2=\{b_1+1,\dots, b_1+b_2\}$, $\cD_3=\{b_1+b_2+1, \dots, b_1+b_2+b_3\}$, $\cD_4=\{b_1+b_2+b_3+1,\dots,n\}$. For a given $1\le j\le 4$ we can expand the square of the appropriate Cauchy determinant as \begin{align} C^2_{\cB_{\sigma,j},\cB'_{\sigma,j}}=\sum_{\eta_j, \tilde \eta_j\in S(\cD_j)} \sgn(\eta_j)\sgn(\tilde \eta_j) \prod_{i\in \cD_j} \frac{1}{z_i-z'_{\eta_j(i)}} \cdot \frac{1}{z_i-z'_{\tilde \eta_j(i)}}, \end{align} where $S(\cD_j)$ is the set of permutations of $\cD_j$. We can represent $\eta_1,\dots, \eta_4$ as a single permutation of $\lst{n}$ that preserves $\cD_1,\cD_2,\cD_3,\cD_4$. Denoting the set of these permutations $S_{b_1,b_2,b_3,b_4}$ we get \begin{align}\label{eq:Cauchy_B4_1_2} \sum_{\underline{\cB}\in \cP_{\underline{b}}} \prod_{j=1}^4 C_{\cB_j,\cB_{9-j}}^2 =\frac{1}{(b_1! b_2! b_3! b_4!)^2}\sum_{\sigma\in S_{2n}} \sum_{\eta,\tilde \eta \in S_{b_1,b_2,b_3,b_4}} \sgn(\eta) \sgn(\tilde \eta) \prod_{i=1}^n \frac{1}{z_i-z'_{\eta(i)}}\cdot \frac{1}{z_i-z'_{\tilde \eta(i)}}. \end{align} Just as in previous computations, we consider the diagonal and off-diagonal terms of the resulting sum separately, and show that the off-diagonal terms cancel. The diagonal terms correspond to the cases $\eta=\tilde \eta$, this gives \begin{align}\label{eq:Cauchy_B4_2} \frac{1}{(b_1! b_2! b_3! b_4!)^2}\sum_{\sigma\in S_{2n}} \sum_{\eta \in S_{b_1,b_2,b_3,b_4}} \prod_{i=1}^n \frac{1}{(z_i-z'_{\eta(i)})^2}. \end{align} Note that \[ \prod_{i=1}^n \frac{1}{(z_i-z'_{\eta(i)})^2}=\prod_{\ell=1}^n \frac{1}{(\lambda_{\alpha_{i,1}}-\lambda_{\alpha_{i,2}})^2} \] where $(\alpha_{1,1},\alpha_{1,2}), \dots (\alpha_{n,1},\alpha_{n,2})$ is a matching of $\lst{2n}$, i.e.~an element of $\Pi_{2n}$. The term corresponding to a particular matching $\alpha\in \Pi_{2n}$ shows up in the sum in \eqref{eq:Cauchy_B4_2} exactly $2^n n! b_1! b_2! b_3! b_4!$ times. Indeed:\\ -- for each pair $(\alpha_{i,1},\alpha_{i,2})$ in the matching we can choose which element will be a $z_i$ ($2^n$ choices), this identifies the index sets $\{\sigma(1),\dots,\sigma(n)\}$ and $\{\sigma(n+1),\dots,\sigma(2n)\}$,\\ -- we can choose the values $\sigma(1), \dots, \sigma(n)$ by choosing one of the $n!$ permutations of the corresponding set, this will identify the lists $\cB_{\sigma,j}, 1\le j\le 4$, and the \emph{sets} corresponding to the lists $\cB'_{\sigma,j}, 1\le j\le 4$,\\ -- we can choose the ordering of elements within the sets corresponding to $\cB'_{\sigma,j}, 1\le j\le 4$, this can be done $b_1!b_2!b_3!b_4!$ ways, this completely identifies $\sigma$ and $\eta$. Our calculations show that the sum in \eqref{eq:Cauchy_B4_2} is equal to \[ \frac{n!}{b_1!b_2!b_3!b_4!} 2^n \sum_{\alpha\in \Pi_{2n}} \prod_{i=1}^n \frac{1}{(\lambda_{\alpha_{i,1}}-\lambda_{\alpha_{i,2}})^2}=\binom{n}{b_1,b_2,b_3,b_4} 2^n \Hf\left(\frac{\ind_{i\neq j}}{(\lambda_i-\lambda_j)^2}\right). \] This, together with \eqref{eq:C_Hf}, shows that the contribution of the diagonal terms is indeed equal to the expression on the right side of \eqref{eq:Cauchy_sum_1}. To finish the proof of our lemma we need to show that the off-diagonal terms in \eqref{eq:Cauchy_B4_1_2} cancel out. For this it is enough to show that if $\eta\neq \tilde \eta$ are fixed permutations in $S_{b_1,b_2,b_3,b_4}$ then the following sum is equal to zero: \begin{align}\label{eq:Cauchy_B4_3} \sum_{\sigma\in S_{2n}} \prod_{i=1}^n \frac{1}{z_i-z'_{\eta(i)}}\cdot \frac{1}{z_i-z'_{\tilde \eta(i)}}. \end{align} To prove this statement, we first note that the pair $\eta, \tilde \eta$ generates a permutation of $\lst{2n}$ with $i\to n+\eta(i)$, $n+i\to \tilde \eta^{-1}(i)$, $1\le i\le n$. This permutation has only even length cycles, and since $\eta\neq \tilde \eta$, it has at least one cycle with length $2\ell\ge 4$. Let one of these cycles be \[ i_1\to n+j_1\to i_2\to n+j_2\to \dots n+j_\ell\to i_1. \] The contribution of this cycle to the term corresponding to a particular $\sigma\in S_{2n}$ in \eqref{eq:Cauchy_B4_3} is \begin{align}\label{eq:Cauchy_B4_4} (-1)^k \prod_{k=1}^\ell \frac{1}{z_{i_k}-z'_{j_k}}\cdot \frac{1}{z'_{j_k}-z_{z_{i_{k+1}}}}. \end{align} (With $i_{n+1}=i_1$.) The proof now follows along the line of the proof of \eqref{eq:off_diag_vanish_1}. Let \[ \cF=\{i_1, \dots, i_\ell, n+j_1, \dots, n+j_\ell\}. \] We can evaluate the sum in \eqref{eq:Cauchy_B4_3} in two steps by first fixing the values of $\sigma$ outside the index set $\cF$, and then assigning the values of \[ \sigma(i_1), \dots, \sigma(i_\ell), \sigma(n+j_1), \dots, \sigma(n+j_\ell) \] out of the remaining $2\ell$ indices in all possible ways. By Lemma \ref{lem:cycle} the sum of the terms \eqref{eq:Cauchy_B4_4} in this second step will be 0, which proves that the sum in \eqref{eq:Cauchy_B4_3} is also 0. This shows that the contribution of the off-diagonal terms in \eqref{eq:Cauchy_B4_1_2} vanishes, proving our lemma. \end{proof} \section{Asymptotics} \label{sec:asymptotics} We provide sketches of the proofs of Theorem \ref{thm:limit_op} and Corollary \ref{cor:osc}, along with the complementary Theorem \ref{thm:limit_op1} and Corollary \ref{cor:osc1}. \subsection{The ${\tt{H}}_{\beta, n}(r,s)$ soft edge} The edges of eigenvalue support for ${\tt{H}}_{\beta,n} (r,s)$ can be tracked through its limiting density of states. The method of Trotter \cite{Trotter} gives the following (which we state without proof): \begin{proposition} Denote by $\mu_\gamma$ the semi-circle distribution with density $ \frac{1}{\pi \gamma} \sqrt{(4 \gamma - \lambda^2)_+}$. For any integer $r$ any $s > 1-r$, the empirical spectral measure $\frac{1}{rn} \sum_{i=1}^{rn} \delta_{\lambda_i}$ of eigenvalues of $\HH(r,s)$ converges weakly in law to $\mu_\gamma$ with $\gamma = \frac{r+s}{r}$. \end{proposition} To study the scaling limit of the maximal eigenvalues let $\mbf{T}_n$ be distributed as ${\tt{H}}_{\beta, n}(r,s)$ and consider the centered and scaled matrix model \begin{align} \label{eq:Hn} \mathbf{H}_n & = \gamma^{-1/2} (rn)^{1/6} (2 \sqrt{(r+s)n} {I}_{rn} - \mathbf{T}_n ) \\ & = 2 m_n^{2} \mbf{I}_{rn} - \sqrt{\frac{m_n}{{\gamma}}} \mbf{T}_n, \quad \quad \quad \quad m_n = (rn)^{1/3}. \nonumber \end{align} We identify $m_n$ as a continuum scale and view $v \in \mathbb{F}^{nr}$, on which $\mathbf{H}_n$ acts, as a vector-valued step function: \begin{equation} \label{embed} (v_0, v_1, \dots) \in \ell_{nr}^2 \mapsto v(x) = v_{\lfloor m_n x \rfloor} \in L^2(\R_+, \mathbb{F}^r). \end{equation} Said another way, for integers $i\in [0, r-1] $ and $k \in [0,n]$, $v_{i + k r}$ is identified with $v_i( k/m_{n})$ With this embedding in mind, write $ \mathbf{H}_n = m_n^2 \Delta_n + m_n {\mathbf{V}}_n$. Here $\Delta_n$ denotes discrete Laplacian in the underlying basis: it has diagonal blocks $2{I}_r$ and off-diagonal blocks $- {I}_r$. The $\mbf{V}_n$ is then considered to define a matrix-valued potential. The main technical result of \cite{Spike2} provides criteria for the convergence of the spectrum of $\mathbf{H}_n$ (in law) in terms of the convergence of the integrated components of $\mathbf{V}_n$. A version of this, tailored to the particular setting considered here, is the following. \begin{proposition} \label{Model_SoftEdgeConv} Suppose that $\mbf{T}_n$ is an $rn\times rn$ Jacobi $r$-block matrix with blocks $\mbf{A}_{n,i}, \mbf{B}_{n,i}$. Let $\mbf{H}_n$ and $m_n$ be defined as in \eqref{eq:Hn}. Expressing $\mathbf{H}_n$ as $ m_n^2 \Delta_n + m_n {\mathbf{V}}_n$, denote the running sums of the diagonal and upper diagonal block components of the potential $\mbf{V}_n$ by: for $0 \le x \le (rn)^{2/3}$, \begin{align} \label{eq:potential_int} {\bf{Y}}_{n,1}(x) = \sum_{i=1}^{[m_n x]} \left( - \frac{1}{\sqrt{\gamma m_n}} \mathbf{A}_{n,i} \right) \quad {\bf{Y}}_{n,2}(x) = \sum_{i=1}^{[m_n x]} \left( m_n^2 \mbf{I}_r - \frac{1}{\sqrt{\gamma m_n}} \mathbf{B}_{n,i} \right). \end{align} Now assume that: \medskip \noindent {(i)} Both $\{ {\bf{Y}}_{n,1}(x)\}_{x \ge 0}$ and $\{ {\bf{Y}}_{n,2}(x)\}_{x \ge 0}$ are tight and \begin{equation} \label{potential_convergence} {\bf{Y}}_{n,1}(x) + ({\bf{Y}}_{n,2}(x) + {\bf{Y}}_{n,2}^{\dagger}(x) ) \Rightarrow \frac{1}{2} r x^2 \mbf{I}_r+ \sqrt{\frac{2}{\gamma}} B(x) \end{equation} in law in the uniform-on-compacts topology; \medskip \noindent {(ii)} For $j=1,2$ there is a decomposition of the increments, \begin{equation} \label{potential_decomp} (\Delta {\bf{Y}})_{n, j} = \nn_{n,j} + (\Delta {\ww})_{n,j}, \end{equation} where: (1) the $\nn_{n,j, i} $ are diagonal and are bounded entry-wise as in \begin{align} \kappa^{-1} x - \kappa \le \nn_{n,1}(x) + \nn_{n,2}(x) \le \kappa x + \kappa, & \ \ \ \nn_{n,2}(x) \le 2 m_n, \label{compact1} \end{align} for some $\kappa \ge 1$, and (2), with $\|\cdot\|$ the spectral norm, there exists an $\epsilon > 0$ so that \begin{align} \| \ww_{n,1}(x) - \ww_{n,1}(y)\|^2 + \| \ww_{n,2}(x) - \ww_{n,2}(y)\|^2 & \le \kappa_n x^{1-\epsilon} + \kappa_n, \label{compact3} \end{align} for all $|x-y| \le 1$ and $\kappa_n \ge 1$ a sequence of tight random constants. \medskip Then, with $\mathcal{H}_{\beta, \gamma}$ defined in \eqref{eq:H_op}, we have that any finite collection of ordered eigenvalues of $\mathbf{H}_n$, along with their associated eigenfunctions as elements in $L^2 (\mathbb{F}^r)$, converge jointly in law to the corresponding eigenvalues/eigenfunctions of $ \mathcal{H}_{\beta, \gamma}$. \end{proposition} Condition (i) simply identifies the correct limit potential. Condition (ii) provides an almost sure lower bound on $\langle v, \mathbf{H}_n v \rangle$ independent of $n$ which is essential for extracting eigenvalue limits. The bound \eqref{compact3} controls the random oscillations of the potential by the growth of $\nn_{n,j}$, which by \eqref{compact1} is well-controlled in terms of its natural limit. Both conditions are readily checked in our case. The independence of the entries of ${\bf{Y}}_{n,1}$ and ${\bf{Y}}_{n,2}$ allows one to establish the functional limit theorem in (i) component-wise. For (ii) one can take $\nn_{n,0} =0$ and $\nn_{n,2}$ the vector of expectations of centered $\chi$ variables appearing on the diagonal of ${\bf{Y}}_{n,2}$. In both (i) and (ii) that the sequence $\chi_p -\sqrt{p}$ converges in law to a $N(0,\frac{1}{2})$ random variable as $p \uparrow \infty$ and satisfies a uniform in $p$ subgaussian bound plays a fundamental role. With the matrix models considered in \cite{Spike2} so similar to $\mathbf{H}_n$, the necessary details are effectively identical to what has already been done there. The proof of Corollary \ref{cor:osc} is also similar to that of the corresponding statement in \cite{Spike2}, though here is a sketch of the main idea. Fix $\lambda\in \R$ and consider the system: \begin{align} \label{Eig:system} dF(x) & = F'(x) dx, \\ dF'(x) & = (\lambda - r x) F(x) + \sqrt{\frac{2}{\gamma}} F (x) dB(x), \nonumber \end{align} with initial condition $F$ satisfying $(F(0), F'(0)) = (0, \mbf{I}_r)$. Here $F$ is an $r\times r$ matrix valued function. It can be shown that the number of eigenvalues of the Dirichlet problem for $\mathcal{H}_{\beta, \gamma}$ less than $\lambda$ coincides with the number of zeros of $\det F$ on $\R_+$ Next define $P(x) = F'(x) F(x)^{-1} $, the matrix Riccati substitution. This satisfies \begin{equation} \label{matrix_ricatti} d P(x) = ((\lambda - r x)\mbf{I}_r - P^2(x))dx + \sqrt{\frac{2}{\gamma}} dB(x). \end{equation} One may then verify that points $x'$ where $\det F$ vanishes correspond to $P(x)$ possessing an eigenvalue $p(x)$ that explodes to $-\infty$ as $x \rightarrow x'$. This is exactly the content of Theorem \ref{cor:osc}: the stochastic differential equation \eqref{mult_sde}, as can be derived from \eqref{matrix_ricatti} by an application of It\^{o}'s Lemma, describes the evolution of the eigenvalues $(p_1(x), \dots, p_r(x))$ of $P(x)$, continued through explosion times to $-\infty$. (Again, see \cite{Spike2} for additional details.) \subsection{Hard edge for ${\tt{W}}_{\beta, n, n+a}(r,s)$} To give a precise definition of $\mathcal{G}_{\beta, \gamma}$ from \eqref{matrixgenerator} we first have to define $\mbf{Z}_x$. We introduce a new type of $r \times r$ matrix Brownian motion $x \mapsto B_x$ in which all entries are independent, the off-diagonal entries are standard ${\mathbb{F}}$-Brownian motions and the diagonal entries are real Brownian motions with common diffusion coefficient $\frac{1}{\beta}$. Then the coefficient matrix $\mbf{Z}_x$ is given by \begin{equation} \label{WandA} \mbf{Z}_x = \mbf{Y}_x \mbf{Y}_x^{\dagger}, \quad \mbf{Y}_x^{-1} d \mbf{Y}_x = \frac{1}{\sqrt{\gamma}} dB_x + \left( \frac{a}{2 \gamma} - \frac{1}{2\beta \gamma} \right) \mbf{I}_r dx, \end{equation} in which $\mbf{Y}_0 = 0.$ Notice that in the $r=1$ and $\gamma =1$ setting in which the Stochastic Bessel Operator was first introduced, $Z_x = e^{ \frac{2}{\sqrt{\beta}} b(x) + a x}$ with a standard one-dimensional Brownian motion $b(x)$. While \eqref{matrixgenerator} is a nice format in which to package the limiting operator $\mathcal{G}_{\beta, \gamma}$, we will actually identify this operator via its inverse. An exercise shows that, specifying a Dirichlet condition at the origin, $\mathcal{G}_{\beta, \gamma} = \mathcal{L}_{\beta, \gamma} \mathcal{L}_{\beta, \gamma}^{\dagger} $ where \begin{equation} \label{kernel_ops} \mathcal{L}_{\beta, \gamma}^{-1} f (x) = \int_x^\infty {\ell}(x,y) f(y) dy, \quad {\ell}(x,y) = e^{-rx/2} \mbf{Y}_x^{-1} \mbf{Y}_y. \end{equation} This structure is natural as $\mathtt{W}_{n,m, \beta}(r,s)$ is realized as a matrix model via $\mbf{L}_n \mbf{L}_n^{\dagger}$, where $\mbf{L}_n$ is block bi-diagonal, and hence explicitly invertible. The results of \cite{RR2} imply that $\mathcal{L}_{\beta, \gamma}^{-1}$ is Hilbert-Schmidt for any $\gamma >0$ and $a>-1$, and the main convergence result there can be summarized, in the spirit of Proposition \ref{Model_SoftEdgeConv}, as follows. \begin{proposition}\label{prop:hard_lim} Given a block bidiagonal matrix $\mbf{L}_n$ with independent diagonal and upper diagonal $r \times r$ blocks $\mbf{D}_k=\mbf{D}_{n,k}$ and $\mbf{O}_k=\mbf{O}_{n,k}$, embed $ \mbf{ L}_n^{-1}$ into $L^2([0,1], \mathbb{F}^r)$ in the manner of \eqref{embed} with now $m_n = n$. In particular, define the piecewise step kernel \begin{equation} \label{explicit_inv} \ell_{n}(x,y) = \sqrt{ \frac{ {n} \gamma}{r}} {\mbf{D}}_{\lfloor nx \rfloor}^{-1} {\mbf{X}}_n(x)^{-1} \mbf{X}_n(y) \mathbf{1}_{0 \le x < y \le 1}, \quad {\mbf{X}}_n(x) = \prod_{k= 1}^{ \lfloor nx \rfloor} {\mbf{O}}_k {\mbf{D}}_{k+1}^{-1}. \end{equation} Assume that: \medskip (i) As $n \rightarrow \infty$, in the uniform-on-compacts topology $$ \Bigl( \sqrt{\frac{{n} \gamma}{r}} \, \mbf{D}_{\lfloor nx \rfloor}^{-1}, \ \mbf{X}_n(x) \Bigr) \Rightarrow \Bigl( \frac{1}{r \sqrt{ 1-x}} \mbf{I}_r, \, \mbf{X}_x \Bigr), \qquad \text{for } x\in [0,1), $$ where $\mbf{X}_x$ satisfies the matrix stochastic differential equation $$ \, \mbf{X}_x^{-1} d \mbf{X}_x = \frac{1}{\sqrt{r \gamma (1-x)}} d B_x - \frac{a -\frac{1}{ \beta}}{2 r\gamma (1- x)} {I}_r dx, \qquad \mbf{X}_0=0. $$ (ii) There is the bound $$ \int_0^1 \int_0^y | \ell_n(x,y) |^2 dx dy \le \kappa_n $$ with $\kappa_n$ a sequence of tight random constants. \medskip Then $spec( \frac{rn}{ \gamma} \mbf{L}_n \mbf{L}_n^\dagger) \rightarrow spec(\mathcal{G}_{\beta, \gamma})$ in the manner described in Proposition \ref{Model_SoftEdgeConv}. \end{proposition} The point is that the integral kernel $\ell_n(x,y)$ is an exact representation of $(\sqrt{rn / \gamma} \, \mbf{L}_n)^{-1}$. The convergence in (i) identifies the pointwise limit of $\ell_n$ while (ii) implies (subsequential) convergence of the corresponding integral operator in Hilbert-Schmidt norm. Combined, we have that $ (\frac{rn}{ \gamma} \mbf{L}_n \mbf{L}_n^{\dagger})^{-1}$ converges in norm in the same subsequential sense; convergence of the finite parts of the spectrum follows. Checking the conditions of Proposition \ref{prop:hard_lim} for ${\tt{W}}_{\beta, n, n+a}(r,s)$ follows the arguments of \cite{RR2}. For (i), by an explicit expansion of the inverse of the diagonal blocks, the increments of $\mbf{X}_n$ have the form $$ {\mbf{O}}_k {\mbf{D}}_{k+1}^{-1} = \mbf{I}_r + \frac{1}{\sqrt{ (r+s)(n-k)}} \mbf{G}_k + \frac{-a+ \frac{1}{\beta} }{2(r+s) (n-k)} \mbf{I}_r + \boldsymbol{\varepsilon}_k. $$ Here the $\mbf{G}_k$ are independent and have independent entries which are $\FF$-normals off diagonal and centered $\chi$ variables on the diagonal. As such, the $\mbf{G}_k$ form the increments of the limiting matrix Brownian motion $B_x$. The $\boldsymbol{\varepsilon}_k$ matrices are error terms for which one can derive the estimate $E \|\boldsymbol{\eps}_k \|^p = O( (n-k)^{-\frac{3}{2} p})$. The proof of (ii) is far more technical, as it requires sharp control of the paths of $x \mapsto \mbf{X}_n(x)$ in a vicinity of $x=1$. Note that while the embedding of the matrix $\mbf{L}_n^{-1}$ naturally takes place as an operator on $[0,1]$, the advertised limit $\mathcal{L}^{-1}_{\beta, \gamma}$ lives on $[0, \infty)$. This is just more convenient for eventual comparison to the soft edge operator $\mathcal{H}_{\beta, \gamma}$, and the kernel $\ell(x,y)$ is related to the limit of $\ell_n$, which is constructed from the limit objects defined in (i) above, by the simple change of variables $x \mapsto 1 - e^{-rx} =\varphi(x)$. As one can readily check, $ \mbf{X}_{\varphi(x)} = \mbf{Y}_x$ and $ \frac{1}{r\sqrt{1-\varphi(x)}} d\varphi(x) = e^{-rx/2} dx$. This establishes Theorem \ref{thm:limit_op1} From here the proof of Corollary \ref{cor:osc1} amounts to writing out the eigenvalue problem for $\mathcal{G}_{\beta, \gamma}$ as a system, then invoking the matrix Riccati correspondence, in analogy with equations \ref{Eig:system} and \ref{matrix_ricatti}. Section 4 of \cite{RR2} provides the details. We conclude by recording the hard edge versions of Corollary \ref{cor:betalimit} and Conjecture \ref{con:betalimit}. Denote by $\operatorname{Bessel}_{\beta, a}$ the random point process defined by the $r=1$ and $\gamma =1$ case of Theorem \ref{thm:limit_op1} (and Corollary \ref{cor:osc1}). To be totally concrete, the points of $\operatorname{Bessel}_{\beta, a}$ are the Dirichlet eigenvalues of the one-dimensional operator \begin{align*} -e^{(a+1)x +\frac{2}{\sqrt{\beta}} b(x)} \frac{d}{dx} e^{-ax -\frac{2}{\sqrt{\beta}} b(x)} \frac{d}{dx} \cdot \end{align*} acting on the positive half-line. This is well defined for all $\beta > 0$ and $a> -1$. \begin{corollary} Consider the point processes of eigenvalues for our solvable instances of ${\tt{W}}_{\beta, n, n+a}(r,s)$. These have explicit joint densities proportional to \begin{align} \label{density3} |\Delta({\lambda})|^{\beta} \left( \sum_{(\mathcal{A}_1,\dots,\mathcal{A}_r)\in \cP_{r,n}} \prod_{j=1}^r \Delta(\cA_j)^2 \right) \prod_{i=1}^{rn} \lambda_i^{\frac{\beta}{2}( (r+s)a+1)-1} e^{-\frac{\beta}{2} \lambda_i} \ind_{\R_n^+} \end{align} for $r \ge 2$ and $\beta s=2$, and to \begin{align} \label{density4} \Delta({\lambda})^{\beta+\frac{\beta s}{2}} \left|\Pf \left(\frac{{\bf{1}}_{i \neq j}}{\lambda_i -\lambda_j} \right)\right|^{\frac{\beta s}{2}} \prod_{i=1}^{rn} \lambda_i^{\frac{\beta}{2}( (r+s) a+1)-1} e^{-\frac{\beta}{2} \lambda_i} \ind_{\R_n^+} \end{align} for $r=2$ and $\beta s =2$ or $4$. When $r=2$, $\beta s =2 $ and $\beta=1$, the scaling limit of the minimal points is given by ${\operatorname{Bessel}}_{2, a/2}$. When $r=2$, $\beta s = 4$, and $\beta = 2$, the scaling limit is $\operatorname{Bessel}_{4, a/2}$. \end{corollary} \begin{conjecture} More generally, the minimal points under \eqref{density3} have scaling limit given by ${\operatorname{Bessel}}_{\beta +\frac{2}{r}, a/(1+\frac{2}{\beta r})}$ for $r \ge 2$ and $\beta =1$ or $2$. For the minimal points under \eqref{density4} with $\beta s =4$ and $\beta =1$ the scaling limit is instead ${\operatorname{Bessel}}_{3, a/3}$. \end{conjecture} \def\cprime{$'$} \begin{thebibliography}{10} \bibitem{Bartlett1933} M.~S. Bartlett. \newblock On the theory of statistical regression. \newblock {\em Proc. R. Soc. Edinburgh}, 53:260--283, 1933. \bibitem{Spike2} A.~Bloemendal and B.~Vir{\'a}g. \newblock Limits of spiked random matrices {II}. \newblock {\em Ann.~Probab.}, 44:2726--2769, 2016. \bibitem{MOPUC} D.~Damanik, A.~Pushnitski, and B.~Simon. \newblock The analytic theory of matrix orthogonal polynomials. \newblock {\em Surveys in Approximation Theory}, 8:1--85, 2008. \bibitem{Dette1} H.~Dette and B.~Reuther. \newblock Random block matrices and matrix orthogonal polynomials. \newblock {\em J.~Theoretical Probab.}, 23:378--400, 2008. \bibitem{DSZ} P.~Di~Francesco, H.~Saluer, and J.B. Zuber. \newblock Critical {I}sing correlation functions in the plane and on the torus. \newblock {\em Nuclear Phyics B}, B290:527--581, 1987. \bibitem{DE} I.~Dumitriu and A.~Edelman. \newblock Matrix models for beta ensembles. \newblock {\em J. Math. Phys.}, 43(11):5830--5847, 2002. \bibitem{ES} A.~Edelman and B.~D. Sutton. \newblock From random matrices to stochastic operators. \newblock {\em J. Stat. Phys.}, 127(6):1121--1165, 2007. \bibitem{ForBook} P.~J. Forrester. \newblock {\em Log-gases and random matrices}, volume~34 of {\em London Mathematical Society Monographs Series}. \newblock Princeton University Press, Princeton, NJ, 2010. \bibitem{FGM_1995} J.~Fr{\"o}hlich, R.~G{\"o}tschmann, and P.A. Marchetti. \newblock The effective gauge field action of a system of non-relativistic electrons. \newblock {\em Comm.~Math.~Physics}, 173:417--452, 1995. \bibitem{Rouault1} F.~Gamboa, J.~Nagel, and A.~Rouault. \newblock Sum rules and large deviations for spectral matrix measures. \newblock {\em Bernoulli}, 25(1):712--741, 2019. \bibitem{Rouault2} F.~Gamboa, J.~Nagel, and A.~Rouault. \newblock Large deviations and a new sum rule for spectral matrix measures of the {J}acobi ensemble. \newblock {\em Random Matrices Theory Appl.}, 10(1):Paper No. 2150008, 36, 2021. \bibitem{Dette2} M.~Guhlich, J.~Nagel, and H.~Dette. \newblock Random block matrices generalizing the classical {J}acobi and {L}aguerre ensembles. \newblock {\em J.~Multivariate Anal.}, 101(8):1884--1897, 2010. \bibitem{KR_1968} C.G. Khatri and C.G. Rao. \newblock Solutions to some functional equations and their applications to characterization of probability distributions. \newblock {\em Sankhy{\=a}: the Indian journal of statistics, series A}, pages 167--180, 1968. \bibitem{KillipNenciu} R.~Killip and I.~Nenciu. \newblock Matrix models for circular ensembles. \newblock {\em International Mathematics Research Notices}, 2004(50):2665--2701, 2004. \bibitem{KS} R.~Killip and M.~Stoiciu. \newblock Eigenvalue statistics for {CMV} matrices: from {P}oisson to clock via random matrix ensembles. \newblock {\em Duke Math. J.}, 146(3):361--399, 2009. \bibitem{KRV} M.~Krishnapur, B.~Rider, and B.~Vir{\'a}g. \newblock Universality of the stochastic {Airy} operator. \newblock {\em Comm.~Pure and Appl.~Math.}, 2016. \bibitem{LT2006} J-G. Luque and J-Y. Thibon. \newblock Pfaffian and {H}afnian identities in shuffle algebras. \newblock {\em Adv.~Appl.~Math.}, 29(4):620--646, 2002. \bibitem{Meckes2019} E.~S. Meckes. \newblock {\em The Random Matrix Theory of the Classical Compact Groups}. \newblock Cambridge Tracts in Mathematics. Cambridge University Press, 2019. \bibitem{MR_1991} G.~Moore and N.~Read. \newblock Nonabelians in the fractional quantum {Hall} effect. \newblock {\em Nuclear Phyics B}, B360:527--581, 1991. \bibitem{RR} J.~A. Ram{\'{\i}}rez and B.~Rider. \newblock Diffusion at the random matrix hard edge. \newblock {\em Comm. Math. Phys.}, 288(3):887--906, 2009. \bibitem{RR2} J.~A. Ram{\'{\i}}rez and B.~Rider. \newblock Spiking the random matrix hard edge. \newblock {\em Probab.~Theory Rel.~Fields}, 169(1):425--467, 2017. \bibitem{RRV} J.~A. Ram{\'{\i}}rez, B.~Rider, and B.~Vir{{\'a}}g. \newblock Beta ensembles, stochastic {A}iry spectrum, and a diffusion. \newblock {\em J. Amer. Math. Soc.}, 24(4):919--944, 2011. \bibitem{SchSch} M.~Scherbina and T.~Scherbina. \newblock Universality for 1d random band matrices. \newblock {\em Comm.~Math.~Phys.}, 385(2):667--716, 2021. \bibitem{Sylvester1851} J.~Sylvester. \newblock On a certain fundamental theorem of determinants. \newblock {\em The London, Edinburgh, and Dublin Philosophical Magazine and Journal of Science}, 2(9):142--145, 1851. \bibitem{Trotter} H.~F. Trotter. \newblock Eigenvalue distributions of large {H}ermitian matrices; {W}igner's semicircle law and a theorem of {K}ac, {M}urdock, and {S}zeg{\H o}. \newblock {\em Adv. in Math.}, 54(1):67--82, 1984. \bibitem{BVBV} B.~Valk\'o and B.~Vir\'ag. \newblock Continuum limits of random matrices and the {B}rownian carousel. \newblock {\em Inventiones Math.}, 177:463--508, 2009. \bibitem{BVBV_sbo} B.~Valk{\'o} and B.~Vir{\'a}g. \newblock The {Sine$_\beta$} operator. \newblock {\em Inventiones Math.}, 209(1):275--327, 2017. \end{thebibliography} {{ \bigskip \bigskip \noindent \textsc{Brian Rider}, \textsc{Department of Mathematics, Temple University, 1805 North Broad St, Philadelphia, PA 19122}, \texttt{[email protected]} \medskip \noindent \textsc{Benedek Valk\'o}, \textsc{Department of Mathematics, University of Wisconsin -- Madison, 480 Lincoln Dr, Madison, WI 53706}, \texttt{[email protected]} }} \end{document}
2412.12130v1
http://arxiv.org/abs/2412.12130v1
On a nonlinear Diophantine equation with powers of three consecutive $k$--Lucas Numbers
\documentclass[11pt,a4paper]{article} \usepackage[latin1]{inputenc} \usepackage{amsmath} \usepackage{amsthm} \usepackage{amsfonts} \usepackage{amsfonts,amsthm,latexsym,amsmath,amssymb,amscd,epsfig,psfrag,enumerate} \usepackage{graphics,graphicx, bezier, float, color, hyperref} \usepackage{amssymb,url} \usepackage{multienum} \usepackage[table]{xcolor} \usepackage{multicol,multirow} \usepackage{graphicx} \usepackage{fancyvrb} \usepackage{parskip} \usepackage[toc,page]{appendix} \sloppy \setlength{\parindent}{0pt} \setlength\parskip{0.1in} \usepackage[top=2.7cm, bottom=2.7cm, left=1.5cm, right=1.5cm]{geometry} \usepackage{xcolor} \usepackage{blkarray} \newtheorem{theorem}{Theorem}[section] \newtheorem{lemma}{Lemma}[section] \newtheorem{cor}{Corollary}[section] \newtheorem{remark}{Remark}[section] \usepackage[none]{hyphenat}[section] \newtheorem{definition}{Definition}[section] \newcommand\numberthis{\addtocounter{equation}{1}\tag{\theequation}} \numberwithin{equation}{section} \numberwithin{table}{section} \numberwithin{figure}{section} \title{On a nonlinear Diophantine equation with powers of three consecutive $k$--Lucas Numbers} \author{Herbert Batte$^{1,*} $ and Florian Luca$^{1,2}$} \date{} \begin{document} \maketitle \abstract{ Let $(L_n^{(k)})_{n\geq 2-k}$ be the sequence of $k$--generalized Lucas numbers for some fixed integer $k\ge 2$ whose first $k$ terms are $0,\ldots,0,2,1$ and each term afterwards is the sum of the preceding $k$ terms. In this paper, we completely solve the nonlinear Diophantine equation $\left(L_{n+1}^{(k)}\right)^x+\left(L_{n}^{(k)}\right)^x-\left(L_{n-1}^{(k)}\right)^x=L_m^{(k)}$, in nonnegative integers $n$, $m$, $k$, $x$, with $k\ge 2$.} {\bf Keywords and phrases}: $k$--generalized Lucas numbers; linear forms in logarithms; Baker--Davenport reduction method, LLL--algorithm. {\bf 2020 Mathematics Subject Classification}: 11B39, 11D61, 11D45. \thanks{$ ^{*} $ Corresponding author} \section*{Statements and Declarations} \textbf{Competing Interests:} The authors declare that they have no conflict of interest. \\ \\ \textbf{Ethical Approval:} This article does not contain any studies involving human participants or animals performed by any of the authors. \\ \\ \textbf{Data Availability Statement:} Data sharing is not applicable to this article as no datasets were generated or analyzed during the current study. \\ \\ \textbf{Author Contributions:} The authors contributed equally to the conceptualization, analysis, and writing of the manuscript. \section{Introduction}\label{intro} \subsection{Background}\label{sec:1.1} For any integer \( k \ge 2 \), the sequence of $k$--generalized Lucas numbers is defined by the recurrence relation: \[ L_n^{(k)} = L_{n-1}^{(k)} + \cdots + L_{n-k}^{(k)}, \quad \text{for all} \ n \ge 2, \] with the initial terms set as \( L_0^{(k)} = 2 \) and \( L_1^{(k)} = 1 \) for all \( k \ge 2 \), and \( L_{2-k}^{(k)} = \cdots = L_{-1}^{(k)} = 0 \) for \( k \ge 3 \). For \( k = 2 \), this reduces to the classical sequence of Lucas numbers, and the superscript ${}^{(k)}$ is typically omitted. The study of sequences such as the Fibonacci and Lucas numbers, and their generalizations, has led to the discovery of numerous interesting identities and Diophantine equations. Similar to how various authors have explored identities involving the $k$--Fibonacci numbers, the $k$--Lucas numbers also exhibit a rich structure that can be investigated through the lens of number theory. In \cite{GGL}, Gmez et al. studied the Diophantine equation \begin{align}\label{eqGGL} \left(F_{n+1}^{(k)}\right)^x+\left(F_{n}^{(k)}\right)^x-\left(F_{n-1}^{(k)}\right)^x=F_m^{(k)}, \end{align} in nonnegative integers $n$, $m$, $k$, $x$, with $n\ge 1$ and $k\ge 2$, where $F_r^{(k)}$ is the $r^{\text{th}}$ term of the $k$--Fibonacci sequence. They showed that the Diophantine eq. \eqref{eqGGL} does not have non--trivial solutions $(k, n, m, x)$ with $k \ge 2$, $n \ge 1$, $m \ge 2$ and $x \ge 1$. Motivated by the work in \cite{GGL}, we study the same Diophantine equation but with $k$--Lucas numbers. Specifically, we solve the nonlinear Diophantine equation \begin{align}\label{eq:main} \left(L_{n+1}^{(k)}\right)^x+\left(L_{n}^{(k)}\right)^x-\left(L_{n-1}^{(k)}\right)^x=L_m^{(k)}, \end{align} in nonnegative integers $n$, $m$, $k$, $x$, with $k\ge 2$. We prove the following result. \subsection{Main Result}\label{sec:1.2} \begin{theorem}\label{thm1.1} Let $(L_n^{(k)})_{n\geq 2-k}$ be the sequence of $k$--generalized Lucas numbers. Then, the only integer solutions $(n,m,k,x)$ to Eq. \eqref{eq:main} with $n\ge 0,~m\ge 0,~k\ge 2$ and $x\ge 0$ (with the convention that $0^0:=1$ in case $(n,x)=(0,0)$ and $k\ge 3$) are \begin{align*} (n,m,k,x)\in \{(n,1,k,0),(1,0,k,1):k\ge 2\}\cup\{(0,2,k,1),(1,3,k,2):k\ge 3\}\cup \{(0,3,2,1),(0,3,2,2)\}. \end{align*} \end{theorem} \section{Methods} \subsection{Preliminaries} It is known that \begin{align}\label{eq2.2} L_n^{(k)} = 3 \cdot 2^{n-2},\qquad \text{for all}\qquad 2 \le n \le k. \end{align} Additionally, $L_{k+1}^{(k)}=3\cdot 2^{k-1}-2$ and by induction one proves that \begin{equation} \label{eq:32} L_n^{(k)}<3\cdot 2^{n-2}\qquad {\text{\rm holds for all}}\qquad n\ge k+1. \end{equation} Next, we revisit some properties of the $k$--generalized Lucas numbers. They form a linearly recurrent sequence of characteristic polynomial \[ \Psi_k(x) = x^k - x^{k-1} - \cdots - x - 1, \] which is irreducible over $\mathbb{Q}[x]$. The polynomial $\Psi_k(x)$ possesses a unique real root $\alpha(k)>1$ and all the other roots are inside the unit circle, see \cite{MIL}. The root $\alpha(k):=\alpha$ is in the interval \begin{align}\label{eq2.3} 2(1 - 2^{-k} ) < \alpha < 2 \end{align} as noted in \cite{WOL}. As in the classical case when $k=2$, it was shown in \cite{BRL} that \begin{align}\label{eq2.4} \alpha^{n-1} \le L_n^{(k)}\le2\alpha^n, \quad \text{holds for all} \quad n\ge0, ~k\ge 2. \end{align} Also, $L_{-1}^{(k)}=-1$ if $k=2$ and $0$ otherwise. In particular, combining \eqref{eq2.4} and \eqref{eq:main}, we have, if $n\ge 1$, \begin{align*} \alpha^{m-1} \le L_m^{(k)}=\left(L_{n+1}^{(k)}\right)^x+\left(L_{n}^{(k)}\right)^x-\left(L_{n-1}^{(k)}\right)^x<2\left(L_{n+1}^{(k)}\right)^x\le \alpha^{(n+3)x+2}, \end{align*} where we have used the fact that $2<\alpha^2$, for all $k\ge 2$. The above inequality also holds for $n=0$ since in this case $$ (L_{n+1}^{(k)})^x+(L_n^{(k)})^x-(L_{n-1}^{(k)})^x\le 1+2^x+1\le 2^{x+1}<\alpha^{2x+2}<\alpha^{(n+3)x+2}. $$ This gives $m<(n+3)x+3$. On the other hand, if $n\ge 1$, and $x\ge 2$, we have \begin{align*} \alpha^{m+2}>2\alpha^{m} \ge L_m^{(k)}=\left(L_{n+1}^{(k)}\right)^x+\left(L_{n}^{(k)}\right)^x-\left(L_{n-1}^{(k)}\right)^x>\alpha^{nx}+\alpha^{(n-1)x}-2^x\alpha^{(n-1)x}\ge \alpha^{nx-1}, \end{align*} from which we have $m>nx-3$. The above inequality also holds for $x=1$ since in this case $$ \alpha^{m+2}>2\alpha^m\ge L_m^{(k)}=L_{n+1}^{(k)}+L_n^{(k)}-L_{n-1}^{(k)}\ge 2L_{n}^{(k)}> \alpha^{n},\quad {\text{\rm so}}\qquad m>n-3. $$ Finally, the above inequality holds for $n=0$ as well since we assume that $m\ge 0$. Therefore, we have \begin{align}\label{m_b} nx-3 <m<(n+3)x+3. \end{align} Let $k\ge 2$ and define \begin{equation} \label{eq:fk} f_k(x):=\dfrac{x-1}{2+(k+1)(x-2)}. \end{equation} We have $$ \frac{df_k(x)}{dx}=-\frac{(k-1)}{(2+(k+1)(x-2))^2}<0\qquad {\text{\rm for~all}}\qquad x>0. $$ In particular, inequality \eqref{eq2.3} implies that \begin{align}\label{eq2.5} \dfrac{1}{2}=f_k(2)<f_k(\alpha)<f_k(2(1 - 2^{-k} ))\le \dfrac{3}{4}, \end{align} for all $k\ge 3$. It is easy to check that the above inequality holds for $k=2$ as well. Further, it is easy to verify that $|f_k(\alpha_i)|<1$, for all $2\le i\le k$, where $\alpha_i$ are the remaining roots of $\Psi_k(x)$ for $i=2,\ldots,k$. Moreover, it was shown in \cite{BRL} that \begin{align}\label{eq2.6} L_n^{(k)}=\displaystyle\sum_{i=1}^{k}(2\alpha_i-1)f_k(\alpha_i)\alpha_i^{n-1}~~\text{and}~~\left|L_n^{(k)}-f_k(\alpha)(2\alpha-1)\alpha^{n-1}\right|<\dfrac{3}{2}, \end{align} for all $k\ge 2$ and $n\ge 2-k$. This means that \begin{equation} \label{eq:Lnwitherror} L_n^{(k)}=f_k(\alpha)(2\alpha-1)\alpha^{n-1}+e_k(n), \qquad {\text{\rm where}}\qquad |e_k(n)|<1.5. \end{equation} The left expression in \eqref{eq2.6} is known as the Binet formula for $L_n^{(k)}$. Furthermore, the right inequality in \eqref{eq2.6} shows that the contribution of the zeros that are inside the unit circle to $L_n^{(k)}$ is small. Next, we state the following result which we shall use later in the proof of the main result. It is stated as Lemma 3 in \cite{Fay}. \begin{lemma}[Lemma 3 in \cite{Fay}]\label{fay} Let $k \ge 2$, $c \in (0,1)$ and $2\le n < 2^{ck}$. Then \[ L_n^{(k)} = 3 \cdot 2^{n-2}(1 + \zeta'_n), \quad \text{with} \quad |\zeta'_n| < \begin{cases} \dfrac{4}{2^{(1-c)k}}, & \text{if} \ c \le 0.693, \\ \dfrac{8.1}{2^{(1-c)k}}, & \text{otherwise}. \end{cases} \] \end{lemma} Next, we prove the following result. \begin{lemma}\label{lem:Lnx} Let \( k \ge 2 \), \( x \), \( n \) be positive integers. Then \[ \left(L_n^{(k)}\right)^x = f_k(\alpha)^x (2\alpha-1)^x\alpha^{(n-1)x}(1 + \eta_n), \] with \[ |\eta_n| < \dfrac{1.5xe^{1.5x/\alpha^{n-1}}}{\alpha^{n-1}}. \] \end{lemma} \begin{proof} By the Binet formula in \eqref{eq:Lnwitherror} and inequalities \eqref{eq2.5}, it follows that \[ \left(L_n^{(k)}\right)^x= f_k(\alpha)^x (2\alpha-1)^x\alpha^{(n-1)x} \left( 1 + \dfrac{e_k(n)}{f_k(\alpha)(2\alpha-1) \alpha^{n-1}} \right)^x. \] So, if we define \[ \eta_n := \left( 1 + \dfrac{e_k(n)}{f_k(\alpha)(2\alpha-1) \alpha^{n-1}} \right)^x - 1 = \sum_{j=1}^x \binom{x}{j} \left( \frac{e_k(n)}{f_k(\alpha)(2\alpha-1) \alpha^{n-1}} \right)^j; \] we get \[ |\eta_n| < \sum_{j=1}^x \dfrac{(1.5x/\alpha^{n-1})^j}{j!} < \dfrac{1.5x}{\alpha^{n-1}} \sum_{j=1}^x \dfrac{(1.5x/\alpha^{n-1})^{j-1}}{(j-1)!} \le \dfrac{1.5xe^{1.5x/\alpha^{n-1}}}{\alpha^{n-1}}, \] where we have used the fact $ |e_k(n)| < 1.5$ and $ (2\alpha-1)f_k(\alpha)>1$. \end{proof} Lastly here, we prove the following. \begin{lemma}\label{Ln:x} Let $x \geq 1$, $k \geq 2$, $i \in \{-1,0, 1\}$, $n + i \geq k + 2$ and $\max\{n + i, 16x\} < 2^{ck}$ for some $c \in (0, 0.25)$. Then, the estimate \[ \left(L_{n+i}^{(k)}\right)^x = 3^x\cdot 2^{(n+i-2)x} \left(1 + \xi_{n+i} \right) \] holds with \[ |\xi_{n+i}| <\dfrac{2}{2^{(1-2c)k}}. \] \end{lemma} \begin{proof} By Lemma \ref{fay} with $c \in (0, 0.25)$, we have \[ L_{n+i}^{(k)} = 3 \cdot 2^{n+i-2} \left( 1 + \zeta'_{n+i} \right), \quad \text{with} \quad |\zeta'_{n+i}| < \dfrac{4}{2^{(1-c)k}}. \] Hence, \begin{eqnarray*} \left(L_{n+i}^{(k)}\right)^x & = & 3^x\cdot 2^{(n+i-2)x} \left(1 +\zeta_{n+i}' \right)^x\\ & = & 3^x 2^{(n+i-2)x} \exp(x\log(1+\zeta_{n+i}'))\\ & = & 3^x 2^{(n+i-2)x} \exp(\eta_{n+i}')\qquad |\eta_{n+i}'|<2x|\zeta_{n+i}'|\\ & = & 3^x2^{(n+i-2)x}(1+\xi_{n+i}')\qquad |\xi_{n+i}'|<2|\eta_{n+i}'|<4x|\zeta_{n+i}'|<\frac{16x}{2^{(1-c)k}}<\frac{1}{2^{(1-2c)k}}\left(<\frac{1}{2}\right) \end{eqnarray*} The above calculations are justified since both inequalities $|\log(1+y)|<2|y|$ and $|\exp y-1|<2|y|$ hold for $y\in (-1/2,1/2)$, and the fact that all the intermediate quantities $\zeta_{n+i}',~\eta_{n+i}',\xi_{n+i'}$ are in the range $(-1/2,1/2)$ follows from the very last inequality above. \end{proof} \subsection{Linear forms in logarithms} We use Baker--type lower bounds for nonzero linear forms in logarithms of algebraic numbers. There are many such bounds mentioned in the literature but we use one of Matveev from \cite{MAT}. Before we can formulate such inequalities, we need the notion of height of an algebraic number recalled below. \begin{definition}\label{def2.1} Let $ \gamma $ be an algebraic number of degree $ d $ with minimal primitive polynomial over the integers $$ a_{0}x^{d}+a_{1}x^{d-1}+\cdots+a_{d}=a_{0}\prod_{i=1}^{d}(x-\gamma^{(i)}), $$ where the leading coefficient $ a_{0} $ is positive. Then, the logarithmic height of $ \gamma$ is given by $$ h(\gamma):= \dfrac{1}{d}\Big(\log a_{0}+\sum_{i=1}^{d}\log \max\{|\gamma^{(i)}|,1\} \Big). $$ \end{definition} In particular, if $ \gamma$ is a rational number represented as $\gamma=p/q$ with coprime integers $p$ and $ q\ge 1$, then $ h(\gamma ) = \log \max\{|p|, q\} $. The following properties of the logarithmic height function $ h(\cdot) $ will be used in the rest of the paper without further reference: \begin{equation}\nonumber \begin{aligned} h(\gamma_{1}\pm\gamma_{2}) &\leq h(\gamma_{1})+h(\gamma_{2})+\log 2;\\ h(\gamma_{1}\gamma_{2}^{\pm 1} ) &\leq h(\gamma_{1})+h(\gamma_{2});\\ h(\gamma^{s}) &= |s|h(\gamma) \quad {\text{\rm valid for}}\quad s\in \mathbb{Z}. \end{aligned} \end{equation} In Section 3, equation (12) of \cite{Brl} these properties were used to show the following inequality: \begin{align}\label{eq2.9} h\left(f_k(\alpha)\right)<3\log k, ~~\text{for all}~~k\ge 2. \end{align} A linear form in logarithms is an expression \begin{equation} \label{eq:Lambda} \Lambda:=b_1\log \gamma_1+\cdots+b_t\log \gamma_t, \end{equation} where $\gamma_1,\ldots,\gamma_t$ are positive real algebraic numbers and $b_1,\ldots,b_t$ are integers. We assume, $\Lambda\ne 0$. We need lower bounds for $|\Lambda|$. We write ${\mathbb K}:={\mathbb Q}(\gamma_1,\ldots,\gamma_t)$ and $D$ for the degree of ${\mathbb K}$ over ${\mathbb Q}$. We give a direct consequence of Matveev's inequality from \cite{MAT}. That is, we quote it in a form which we shall use. \begin{theorem}[Matveev, \cite{MAT}] \label{thm:Mat} Put $\Gamma:=\gamma_1^{b_1}\cdots \gamma_t^{b_t}-1=e^{\Lambda}-1$. Then $$ \log |\Gamma|>-1.4\cdot 30^{t+3}\cdot t^{4.5} \cdot D^2 (1+\log D)(1+\log B)A_1\cdots A_t, $$ where $B\ge \max\{|b_1|,\ldots,|b_t|\}$ and $A_i\ge \max\{Dh(\gamma_i),|\log \gamma_i|,0.16\}$ for $i=1,\ldots,t$. \end{theorem} In our application of Matveev's result (Theorem \ref{thm:Mat}), we need to ensure that the linear forms in logarithms are indeed nonzero. To ensure this, we shall need the following result given as Lemma 2.8 in \cite{GGL1}. \begin{lemma}[Lemma 2.8 in \cite{GGL1}]\label{lemGLm} Let \( N := N_{\mathbb{K}/\mathbb{Q}}, \) where \( \mathbb{K} = \mathbb{Q}(\alpha) \). Then \begin{enumerate}[(i)] \item For \( n, m \geq 1 \) and \( k \geq 2 \), \( |N(\alpha)| = 1 \). \item \( N(2\alpha - 1) = 2^{k+1} - 3 \) and \( N(f_k(\alpha)) = (k - 1)^2 / (2^{k+1}k^k - (k + 1)^{k+1}) \). \item For \( k \geq 2 \), \( N((2\alpha - 1)f_k(\alpha)) \le 1 \). This inequality is strict for $k\ge 3$ (and is an equality for $k=2$). \end{enumerate} \end{lemma} At some point, we treat cases with $t=2$, that is, linear forms in two logarithms. Let $A_1,~A_2>1$ be real numbers such that \begin{equation} \label{eq:Aih} \log A_i\ge \max\left\{h(\gamma_i),\frac{|\log \gamma_i|}{D},\frac{1}{D}\right\}\quad {\text{\rm for}}\quad i=1,2. \end{equation} Put $$ b':=\frac{|b_1|}{D\log A_2}+\frac{|b_2|}{D\log A_1}. $$ The following result is Corollary 2 in \cite{LMN}. \begin{theorem}[Laurent et al., \cite{LMN}] \label{thm:LMNh} In case $t=2$, we put $$\Lambda:=b_1\log \gamma_1-b_2\log \gamma_2, $$ where $|\gamma_{1}|, |\gamma_{2}|\ge 1$ are two multiplicatively independent real algebraic numbers and $b_1, b_2$ are positive integers. Then, we have $$ \log |\Lambda|\ge -24.34 D^4\left(\max\left\{\log b'+0.14,\frac{21}{D},\frac{1}{2}\right\}\right)^2\log A_1\log A_2. $$ \end{theorem} During the calculations, upper bounds on the variables are obtained which are too large, thus there is need to reduce them. To do so, we use some results from approximation lattices and the so--called LLL--reduction method from \cite{LLL}. We explain this in the following subsection. \subsection{Reduced Bases for Lattices and LLL--reduction methods}\label{sec2.3} Let $k$ be a positive integer. A subset $\mathcal{L}$ of the $k$--dimensional real vector space ${ \mathbb{R}^k}$ is called a lattice if there exists a basis $\{b_1, b_2, \ldots, b_k \}$ of $\mathbb{R}^k$ such that \begin{align*} \mathcal{L} = \sum_{i=1}^{k} \mathbb{Z} b_i = \left\{ \sum_{i=1}^{k} r_i b_i \mid r_i \in \mathbb{Z} \right\}. \end{align*} We say that $b_1, b_2, \ldots, b_k$ form a basis for $\mathcal{L}$, or that they span $\mathcal{L}$. We call $k$ the rank of $ \mathcal{L}$. The determinant $\text{det}(\mathcal{L})$, of $\mathcal{L}$ is defined by \begin{align*} \text{det}(\mathcal{L}) = | \det(b_1, b_2, \ldots, b_k) |, \end{align*} with the $b_i$'s being written as column vectors. This is a positive real number that does not depend on the choice of the basis (see \cite{Cas}, Section 1.2). Given linearly independent vectors $b_1, b_2, \ldots, b_k$ in $ \mathbb{R}^k$, we refer back to the Gram--Schmidt orthogonalization technique. This method allows us to inductively define vectors $b^*_i$ (with $1 \leq i \leq k$) and real coefficients $\mu_{i,j}$ (for $1 \leq j \leq i \leq k$). Specifically, \begin{align*} b^*_i &= b_i - \sum_{j=1}^{i-1} \mu_{i,j} b^*_j,~~~ \mu_{i,j} = \dfrac{\langle b_i, b^*_j\rangle }{\langle b^*_j, b^*_j\rangle}, \end{align*} where \( \langle \cdot , \cdot \rangle \) denotes the ordinary inner product on \( \mathbb{R}^k \). Notice that \( b^*_i \) is the orthogonal projection of \( b_i \) on the orthogonal complement of the span of \( b_1, \ldots, b_{i-1} \), and that \( \mathbb{R}b_i \) is orthogonal to the span of \( b^*_1, \ldots, b^*_{i-1} \) for \( 1 \leq i \leq k \). It follows that \( b^*_1, b^*_2, \ldots, b^*_k \) is an orthogonal basis of \( \mathbb{R}^k \). \begin{definition} The basis $b_1, b_2, \ldots, b_n$ for the lattice $\mathcal{L}$ is called reduced if \begin{align*} \| \mu_{i,j} \| &\leq \frac{1}{2}, \quad \text{for} \quad 1 \leq j < i \leq n,~~ \text{and}\\ \|b^*_{i}+\mu_{i,i-1} b^*_{i-1}\|^2 &\geq \frac{3}{4}\|b^*_{i-1}\|^2, \quad \text{for} \quad 1 < i \leq n, \end{align*} where $ \| \cdot \| $ denotes the ordinary Euclidean length. The constant $ {3}/{4}$ above is arbitrarily chosen, and may be replaced by any fixed real number $ y $ in the interval ${1}/{4} < y < 1$ (see \cite{LLL}, Section 1). \end{definition} Let $\mathcal{L}\subseteq\mathbb{R}^k$ be a $k-$dimensional lattice with reduced basis $b_1,\ldots,b_k$ and denote by $B$ the matrix with columns $b_1,\ldots,b_k$. We define \[ l\left( \mathcal{L},y\right):= \left\{ \begin{array}{c} \min_{x\in \mathcal{L}}||x-y|| \quad ;~~ y\not\in \mathcal{L}\\ \min_{0\ne x\in \mathcal{L}}||x|| \quad ;~~ y\in \mathcal{L} \end{array} \right., \] where $||\cdot||$ denotes the Euclidean norm on $\mathbb{R}^k$. It is well known that, by applying the LLL--algorithm, it is possible to give in polynomial time a lower bound for $l\left( \mathcal{L},y\right)$, namely a positive constant $c_1$ such that $l\left(\mathcal{L},y\right)\ge c_1$ holds (see \cite{SMA}, Section V.4). \begin{lemma}\label{lem2.5} Let $y\in\mathbb{R}^k$ and $z=B^{-1}y$ with $z=(z_1,\ldots,z_k)^T$. Furthermore, \begin{enumerate}[(i)] \item if $y\not \in \mathcal{L}$, let $i_0$ be the largest index such that $z_{i_0}\ne 0$ and put $\sigma:=\{z_{i_0}\}$, where $\{\cdot\}$ denotes the distance to the nearest integer. \item if $y\in \mathcal{L}$, put $\sigma:=1$. \end{enumerate} \noindent Finally, let \[ c_2:=\max\limits_{1\le j\le k}\left\{\dfrac{||b_1||^2}{||b_j^*||^2}\right\}. \] Then, \[ l\left( \mathcal{L},y\right)^2\ge c_2^{-1}\sigma^2||b_1||^2:=c_1^2. \] \end{lemma} In our application, we are given real numbers $\eta_0,\eta_1,\ldots,\eta_k$ which are linearly independent over $\mathbb{Q}$ and two positive constants $c_3$ and $c_4$ such that \begin{align}\label{2.9} |\eta_0+a_1\eta_1+\cdots +a_k \eta_k|\le c_3 \exp(-c_4 H), \end{align} where the integers $a_i$ are bounded as $|a_i|\le A_i$ with $A_i$ given upper bounds for $1\le i\le k$. We write $A_0:=\max\limits_{1\le i\le k}\{A_i\}$. The basic idea in such a situation, from \cite{Weg}, is to approximate the linear form \eqref{2.9} by an approximation lattice. So, we consider the lattice $\mathcal{L}$ generated by the columns of the matrix $$ \mathcal{A}=\begin{pmatrix} 1 & 0 &\ldots& 0 & 0 \\ 0 & 1 &\ldots& 0 & 0 \\ \vdots & \vdots &\vdots& \vdots & \vdots \\ 0 & 0 &\ldots& 1 & 0 \\ \lfloor C\eta_1\rfloor & \lfloor C\eta_2\rfloor&\ldots & \lfloor C\eta_{k-1}\rfloor& \lfloor C\eta_{k} \rfloor \end{pmatrix} ,$$ where $C$ is a large constant usually of the size of about $A_0^k$ . Let us assume that we have an LLL--reduced basis $b_1,\ldots, b_k$ of $\mathcal{L}$ and that we have a lower bound $l\left(\mathcal{L},y\right)\ge c_1$ with $y:=(0,0,\ldots,-\lfloor C\eta_0\rfloor)$. Note that $ c_1$ can be computed by using the results of Lemma \ref{lem2.5}. Then, with these notations the following result is Lemma VI.1 in \cite{SMA}. \begin{lemma}[Lemma VI.1 in \cite{SMA}]\label{lem2.6} Let $S:=\displaystyle\sum_{i=1}^{k-1}A_i^2$ and $T:=\dfrac{1+\sum_{i=1}^{k}A_i}{2}$. If $c_1^2\ge T^2+S$, then inequality \eqref{2.9} implies that we either have $a_1=a_2=\cdots=a_{k-1}=0$ and $a_k=-\dfrac{\lfloor C\eta_0 \rfloor}{\lfloor C\eta_k \rfloor}$, or \[ H\le \dfrac{1}{c_4}\left(\log(Cc_3)-\log\left(\sqrt{c_1^2-S}-T\right)\right). \] \end{lemma} Finally, we present an analytic argument which is Lemma 7 in \cite{GL}. \begin{lemma}[Lemma 7 in \cite{GL}]\label{Guz} If $ r \geq 1 $, $T > (4r^2)^r$ and $T > \dfrac{p}{(\log p)^r}$, then $$p < 2^r T (\log T)^r.$$ \end{lemma} SageMath 9.5 is used to perform all computations in this work. \section{Proof of Theorem \ref{thm1.1}.}\label{Sec3} In this section, we prove Theorem \ref{thm1.1}. To do this, we first find some trivial solutions. \subsection{Trivial solutions} Here, we study equation \eqref{eq:main} in the following trivial scenarios. \begin{enumerate}[(a)] \item If $x=0$, then (assuming $0^0:=1$ when $n=0$ and $k\ge 3$) \eqref{eq:main} becomes $L_m^{(k)}=1$, for which $m=1$. We therefore get $(n,m,k,x)=(n,1,k,0)$ with $n\ge 1$ for all $k\ge 2$. \item If $x=1$, then the Diophantine equation \eqref{eq:main} becomes \begin{align}\label{x1} L_{n+1}^{(k)}+L_{n}^{(k)}-L_{n-1}^{(k)}=L_m^{(k)}. \end{align} Moreover, the inequalities in \eqref{m_b} tells us that for $x=1$, we have $n-3 <m<n+6$, for all $k\ge 2$. This implies that $ m\in\{n-2, n-1,n,n+1,\ldots,n+5\}$. A straightforward verification shows that the only solutions to \eqref{x1} with these options are $(n,m,k,x)=(0,2,k,1)$ with $k\ge 3$, $(n,m,k,x)=(1,0,k,1)$ for all $k\ge 2$ and also the sporadic solution $(n,m,k,x)=(0,3,2,1)$. \end{enumerate} Assume from now on that $x\ge 2$. Suppose next that $m\le 2$. Since $m\ge nx-2$, we get $nx\le 4$ and since $x\ge 2$, we get $n\in \{0,1,2\}$. Thus, $L_m^{(k)}\in \{2,1,3\}$, while $$ \left(L_{n+1}^{(k)}\right)^x+\left(L_n^{(k)}\right)^x-\left(L_{n-1}^{(k)}\right)^x\ge 1+2^x-1=2^x\ge 4>L_m^{(k)}, $$ so there are no solutions in this range. For the remaining part of the proof, we assume $k\ge 2$, $x\ge 2$, $m\ge 3$. \subsection{The case $k=2$ and $n=0,2$} Assume $k=2$. In the case $n=0$ we get the equation $$ 1+2^x-(-1)^x=L_m. $$ When $x$ is even, we get $L_m=2^x$ and it is known (for example, by Carmichael's primitive divisor theorem \cite{Car}) that the only powers of $2$ in the Lucas sequence are $m=0,~3$ for which $L_0=2,~L_3=2^2$ and the only one with even exponent $x$ is $L_3=2^2$. We get the solution $(n,m,k,x)=(0,3,2,2)$. When $x$ is odd, we get $L_m=2^x+2$. When $x=1$, we get the solution $m=3$ which leads to $(n,m,k,x)=(0,3,2,1)$. When $x=2$, we get $L_m=6$, which is false since $6$ is not a member of the sequence of Lucas numbers. When $x\ge 3$, we get that $2\| L_m$ so $m$ is even (and multiple of $3$). In particular, $L_m=L_{m/2}^2\pm 2$. We thus get $$ L_{m/2}^2\pm 2=2^x+2,\qquad {\text{\rm so}}\qquad L_{m/2}^2\in \{2^x,2^x+4\}. $$ Since $x$ is odd, the equation $L_{m/2}^2=2^x$ is not possible. Thus, $L_{m/2}^2=2^x+4$ leading to $2\| L_{m/2}$ and $(L_{m/2}^2/2)^2=2^{x-2}+1$. This is a particular instance of the Catalan equation and its only solution is $x-2=3$ and $L_{m/2}/2=3$, leading to $L_{m/2}=6$, but this is wrong since $6$ is not a member of the sequence of Lucas numbers. Assume $n=2$. In this case we get the equation $$ 4^x+3^x-1=L_m. $$ The left--hand side is a multiple of $3$, so $m$ is even. Considering the values of $x$ modulo $4$, we get that the left--hand side is congruent to $1$ modulo $5$ (if $x\equiv 0,1\pmod 5$), to $4\pmod 5$ (if $x\equiv 2\pmod 4$) and to $0\pmod 5$ (if $x\equiv 3\pmod 4$). Since the Lucas sequence is periodic modulo $5$ with period $4$ and achieves the values $2,1,3,4,2,1,3,4,\ldots$, we get that only the cases $L_m\equiv 1,4\pmod 5$ are possible and this implies $m\equiv 1,3\pmod 4$. This contradicts the fact that $m$ is even. \subsection{The case $n\le k$}\label{subsec3.2} \subsubsection{Bounding $x$ and $m$ in terms of $k$} We start by revisiting \eqref{eq2.2}. Assume $n\ge 3$. Then we rewrite \eqref{eq:main} as \begin{align*} L_m^{(k)}&= \left(L_{n+1}^{(k)}\right)^x+\left(L_{n}^{(k)}\right)^x-\left(L_{n-1}^{(k)}\right)^x\\ &=\left(3\cdot 2^{n-1}\right)^x+\left(3\cdot 2^{n-2}\right)^x-\left(3\cdot 2^{n-3}\right)^x. \end{align*} Assume $m\le k$. Then, since $m\ge 3$, we have $L_m^{(k)}=3\cdot 2^{m-2}$. If $n\ge 3$, then the equation above is \begin{align*} 3\cdot2^{m-2}=\left(3\cdot 2^{n-1}\right)^x+\left(3\cdot 2^{n-2}\right)^x-\left(3\cdot 2^{n-3}\right)^x. \end{align*} The right--hand side above is divisible by $3^x$. So, $3^x$ must divide $3\cdot2^{m-2}$, which is false for $x\ge 2$. This was in case $n\ge 3$. We must consider the cases of small $n$ as well. If $n=0,1,2$, then since $k\ne 2$ when $n=0,2$, we have $$ 3\cdot 2^{m-2}=L_m^{(k)}=(L_{n+1}^{(k)})^x+(L_n^{(k)})^x-(L_{n-1}^{(k)})^x\in \{1+2^x,3^x+1-2^x,6^x+3^x-1\} $$ (the first element above corresponds to $n=0$ and $k\ge 3$, the next element corresponds to $n=1$ and the third element corresponds to $n=2$ and $k\ge 3$, respectively). We need to find out when the elements of the above set are of the form $3\cdot 2^{m-2}$. The first is odd and the third is not a multiple of $3$, so they cannot be of the form $3\cdot 2^{m-2}$. Finally, for the second element, we get the equation $3\cdot 2^{m-2}=3^x+1-2^x$. This has the solution $(x,m)=(2,3)$ and no other solution since for $x\ge 3$, the right--hand side is at least $20$ and not a multiple of $8$, so it cannot be of the form $3\cdot 2^{m-2}$. Thus, we get the additional solution $(n,m,k,x)=(1,3,k,2)$. Note that $k\ge 3$ since $3=m\le k$. For the rest of this section, we assume $k\ge 2$, $x\ge 2$, $m\ge \max\{k+1,nx-2\}$. To proceed, we prove the following result. \begin{lemma}\label{lem3.1} Let $n$, $m$, $k$, $x$ be integer solutions to Eq. \eqref{eq:main} with $k\ge 2$, $0\le n\le k$, $k\ge 2$ and $m\ge \max\{k+1,nx-2\}$, then \begin{align*} x< 3.1\cdot 10^{14} k^5(\log k)^2 \log m. \end{align*} \end{lemma} \begin{proof} Assume first that $n\ge 3$. We go back to \eqref{eq:main} and rewrite it as \begin{align*} L_m^{(k)}&=\left(3\cdot 2^{n-1}\right)^x+\left(3\cdot 2^{n-2}\right)^x-\left(3\cdot 2^{n-3}\right)^x. \end{align*} In case $n\in \{0,1,2\}$, then again using the fact that $k\ge 3$ when $n\in \{0,2\}$, the right--hand side above is \begin{equation} \label{eq:r} 2^x+1,\quad 3^x+1-2^x, \quad 6^x+3^x-1. \end{equation} Now, using the Binet formula in \eqref{eq:Lnwitherror}, we have $$ L_m^{(k)}-f_k(\alpha)(2\alpha-1)\alpha^{m-1}=e_k(m). $$ Thus, assuming $n\ge 3$, we get \begin{align*} L_m^{(k)}-f_k(\alpha)(2\alpha-1)\alpha^{m-1}&=e_k(m)\\ \left(3\cdot 2^{n-1}\right)^x+\left(3\cdot 2^{n-2}\right)^x-\left(3\cdot 2^{n-3}\right)^x-f_k(\alpha)(2\alpha-1)\alpha^{m-1}&=e_k(m)\\ \left(3\cdot 2^{n-1}\right)^x-f_k(\alpha)(2\alpha-1)\alpha^{m-1}&=-\left(3\cdot 2^{n-2}\right)^x+\left(3\cdot 2^{n-3}\right)^x+e_k(m)\\ 1-3^{-x}2^{-(n-1)x}f_k(\alpha)(2\alpha-1)\alpha^{m-1}&=-\dfrac{1}{2^x}+\dfrac{1}{2^{2x}}+\dfrac{e_k(m)}{\left(3\cdot 2^{n-1}\right)^x}. \end{align*} Taking absolute values and simplifying, we get \begin{align} \label{eq:gen} |\Gamma_1|:= \left|3^{-x}2^{-(n-1)x}f_k(\alpha)(2\alpha-1)\alpha^{m-1}-1\right|<\dfrac{3}{2^x}. \end{align} This was for the case $n\ge 3$. For the cases when $n=0,1,2$, the amount $3\cdot 2^{n-1}$ above gets replaced by $2,3,6$, respectively and the right--hand sides get replaced by $$ \frac{1}{2^x}+\frac{|e_k(m)|}{2^x},\quad \left(\frac{2}{3}\right)^x+\frac{1}{3^x}+\frac{|e_k(m)|}{3^x},\quad \left(\frac{3}{6}\right)^x+\left(\frac{1}{6}\right)^x+\frac{|e_k(m)|}{6^x}, $$ respectively. All these expressions are bounded by $3(2/3)^x$. Thus, we get \begin{equation} \label{eq:gen1} |\Gamma_1|<3\left(\frac{2}{3}\right)^x, \end{equation} where now \begin{equation} \label{g1} \Gamma_1:=\delta^{-x} f_k(\alpha)(2\alpha-1)\alpha^{m-1}-1,\qquad \delta\in \{2,3,6\}. \end{equation} Note that $\Gamma_1\ne 0$ in all cases, otherwise we would have $$ f_k(\alpha)(2\alpha-1)\alpha^{m-1}\in \{(3\cdot 2^{n-1})^x,2^x,3^x,6^x\}. $$ Applying norms in ${\mathbb K}={\mathbb Q}(\alpha)$ and using $|N(\alpha)|=1$, and item $(ii)$ and $(iii)$ of Lemma \ref{lemGLm}, the above equation becomes $$ N\left(f_k(\alpha)\right)\cdot N(2\alpha-1)\in \{(3\cdot 2^{n-1})^{kx},2^{kx}, 3^{kx},6^{kx}\}. $$ which implies that \begin{align*} 1\ge \dfrac{(k - 1)^2 }{ 2^{k+1}k^k - (k + 1)^{k+1}}\cdot \left(2^{k+1}-3\right)=\left(2\right)^{kx}\ge \left(2\right)^{2\cdot 2}=16, \end{align*} a contradiction, for $x\ge 2$ and $k\ge 2$. Thus, $\Gamma_1\ne 0$. The algebraic number field containing the following $\gamma_i$'s is $\mathbb{K} := \mathbb{Q}(\alpha)$. When $n\ge 3$, we have $D = k$, $t :=4$, \begin{equation}\nonumber \begin{aligned} \gamma_{1}&:=(2\alpha-1)f_k(\alpha),\qquad \gamma_{2}:=\alpha, \qquad\gamma_{3}:=3,\qquad \gamma_{4}:=2,\\ b_{1}&:=1,\qquad b_{2}:=m-1,\qquad b_{3}:=-x,\qquad b_{4}:=-(n-1)x. \end{aligned} \end{equation} We can take $A_3:=k \log 3$ and $A_4:=k \log 2$. Additionally, $h(\gamma_{2})=(\log \alpha)/k <0.7/k$, so we take $A_{2}:=0.7$. For $A_1$, we first compute \begin{align*} h(\gamma_{1}):=h\left((2\alpha-1)f_k(\alpha)\right)\le h\left((2\alpha-1)\right)+h\left(f_k(\alpha)\right)<\log 3+3\log k<6\log k, \end{align*} for all $k\ge 2$. So, we can take $A_1:=6k\log k$. Next, $B \geq \max\{|b_i|:i=1,2,3, 4\}$, and by relation \eqref{m_b}, we can take $B:=m$. Now, by Theorem \ref{thm:Mat}, \begin{align}\label{eq3.4} \log |\Gamma_1| &> -1.4\cdot 30^{7} \cdot 4^{4.5}\cdot k^2 (1+\log k)(1+\log m)\cdot 6k\log k \cdot (0.7) \cdot (k\log 3)\cdot (k\log 3)\nonumber\\ &> -2.1\cdot 10^{14} k^5(\log k)^2 \log m. \end{align} For the cases $n=0,1,2$, we take $t:=3$ instead, keep the same $\gamma_1,~\gamma_2$ but $\gamma_3:= \delta$. Thus, we can take $A_3:=k\log 6$. We then get \begin{align}\label{eq3.41} \log |\Gamma_1| &> -1.4\cdot 30^{6} \cdot 4^{3.5}\cdot k^2 (1+\log k)(1+\log m)\cdot 6k\log k \cdot (0.7) \cdot (k\log 6))\nonumber\\ &> -2.7\cdot 10^{12} k^4(\log k)^2 \log m. \end{align} When $n\ge 3$, comparing \eqref{eq:gen} and \eqref{eq3.4}, we get \begin{align*} x\log 2-\log 3&<2.1\cdot 10^{14} k^5(\log k)^2 \log m, \end{align*} which leads to $x<3.1\cdot 10^{14} k^5(\log k)^2 \log m$. When $n=0,1,2$, applying instead \eqref{eq:gen1} we get $$ x\log(3/2)-\log 3<2.7\cdot 10^{12} k^4(\log k)^2 \log m, $$ and this gives a smaller bound on $x$. \end{proof} Next, we prove the following. \begin{lemma}\label{lem3.2} Let $n$, $m$, $k$, $x$ be integer solutions to Eq. \eqref{eq:main} with $k\ge 2$, $0\le n\le k$, $k\ge 2$ and $m\ge \max\{k+1,5\}$, then \begin{align*} m< 6.3\cdot 10^{32}k^{10}(\log k)^5 . \end{align*} \end{lemma} \begin{proof} If $n=0,1,2$, then $$ m< (n+3)x+3\le 5x+3<5\cdot 3.1\cdot 10^{14} k^5 (\log k)^2\log m+3<2\cdot 10^{15} k^5 (\log k)^2 \log m. $$ Thus, $$ \frac{m}{\log m}<2\times 10^{15} k^5 (\log k)^2. $$ Applying Lemma \ref{Guz} with $p:=m$, $r:=2$ and $T:=2\times 10^{15} k^5 (\log k)^2$ we get a better bound than the one from the statement of the lemma. From now on, we assume $n\ge 3$. We go back to \eqref{eq:main} and rewrite it as \begin{align*} L_m^{(k)}&=\left(3\cdot 2^{n-1}\right)^x+\left(3\cdot 2^{n-2}\right)^x-\left(3\cdot 2^{n-3}\right)^x=\left(3\cdot 2^{n-2}\right)^x\left(2^x+1-2^{-x}\right). \end{align*} As before, we use the Binet formula in \eqref{eq:Lnwitherror} and have \begin{align*} L_m^{(k)}-f_k(\alpha)(2\alpha-1)\alpha^{m-1}&=e_k(m)\\ \left(3\cdot 2^{n-2}\right)^x\left(2^x+1-2^{-x}\right)-f_k(\alpha)(2\alpha-1)\alpha^{m-1}&=e_k(m)\\ \left(3\cdot 2^{n-2}\right)^{x}\left(2^x+1-2^{-x}\right)\left(f_k(\alpha)\right)^{-1}(2\alpha-1)^{-1}\alpha^{-(m-1)}-1&=\dfrac{e_k(m)}{f_k(\alpha)(2\alpha-1)\alpha^{m-1}}. \end{align*} Taking absolute values and simplifying, we get \begin{align}\label{g2} |\Gamma_2|&:=\left|\left(3\cdot 2^{n-2}\right)^{x}\left(2^x+1-2^{-x}\right)\left(f_k(\alpha)\right)^{-1}(2\alpha-1)^{-1}\alpha^{-(m-1)}-1\right| <\dfrac{1.5}{0.5\cdot1.5\alpha^{m-1}} =\dfrac{2}{\alpha^{m-1}}, \end{align} where we have used the fact that $f_k(\alpha)>0.5$ and $2\alpha-1>1.5$, for all $k\ge 2$. Clearly, $\Gamma_2\ne 0$, otherwise we would have \begin{align*} f_k(\alpha)= \dfrac{\left(3\cdot 2^{n-2}\right)^x\left(2^x+1-2^{-x}\right)}{(2\alpha-1)\alpha^{m-1}}. \end{align*} Taking norms in ${\mathbb K}={\mathbb Q}(\alpha)$ as before, the above equation becomes \begin{align*} N\left(f_k(\alpha)\right)\cdot N(2\alpha-1)=\left(3\cdot 2^{n-2}\right)^{kx}\left(2^x+1-2^{-x}\right)^k. \end{align*} We have already shown before that the left--hand side above is $\le 1$. Therefore, \begin{align*} 1\ge N\left(f_k(\alpha)\right)\cdot N(2\alpha-1)=\left(3\cdot 2^{n-2}\right)^{kx}\left(2^x+1-2^{-x}\right)^k>500, \end{align*} a contradiction, for all $n\ge 3$, $x\ge 2$ and $k\ge 2$. Note that in the deduction above, we have used the fact that the function $\left(2^x+1-2^{-x}\right)$ is increasing and it is at least $ 4$ for all $x\ge 2$. Thus, $\Gamma_2\ne 0$. The algebraic number field containing the following $\gamma_i$'s is $\mathbb{K} := \mathbb{Q}(\alpha)$, so $D = k$, $t :=4$, \begin{equation}\nonumber \begin{aligned} \gamma_{1}&:=3,\qquad \gamma_{2}:=2, \qquad\gamma_{3}:=\left(2^x+1-2^{-x}\right)/\left((2\alpha-1)f_k(\alpha)\right),\qquad \gamma_{4}:=\alpha,\\ b_{1}&:=x\qquad b_{2}:=(n-2)x,\qquad b_{3}:=1,\qquad b_{4}:=-(m-1). \end{aligned} \end{equation} As before, $A_1:=k \log 3$, $A_2:=k \log 2$ and $A_{4}:=0.7$. To determine what $A_3$ could be, we first compute \begin{align*} h(\gamma_{3})&=h\left(\left(2^x+1-2^{-x}\right)/\left((2\alpha-1)f_k(\alpha)\right)\right)\le h\left(2^x+1-2^{-x}\right)+h(2\alpha-1)+h\left(f_k(\alpha)\right)\\ &<2x\log 2+2\log 2+\log 3+3\log k\\ &<2\left(3.1\cdot 10^{14} k^5(\log k)^2 \log m\right)\log 2+2\log 2+6\log k\\ &<4.5\cdot 10^{14} k^5(\log k)^2 \log m, \end{align*} where we have used Lemma \ref{lem3.1}. Thus, we can take $A_3:=4.5\cdot 10^{14} k^6(\log k)^2 \log m$. Next, $B \geq \max\{|b_i|:i=1,2,3,4\}$, and by relation \eqref{m_b}, we can take $B:=m$. Now, by Theorem \ref{thm:Mat}, \begin{align}\label{eq3.8} \log |\Gamma_2| &> -1.4\cdot 30^{7} \cdot 4^{4.5}\cdot k^2 (1+\log k)(1+\log m)\cdot k \log 3\cdot k \log 2\cdot 0.7\cdot 4.5\cdot 10^{14} k^6(\log k)^2 \log m\nonumber\\ &> -6.8\cdot 10^{27} k^{10}(\log k)^3 (\log m)^2. \end{align} Comparing \eqref{g2} and \eqref{eq3.8}, we get \begin{align*} (m-1)\log\alpha-\log 2&<6.8\cdot 10^{27} k^{10}(\log k)^3 (\log m)^2, \end{align*} which leads to $m<1.5\cdot 10^{28} k^{10}(\log k)^3 (\log m)^2$. We now apply Lemma \ref{Guz} with $p:=m$, $r:=2$, $T:=1.5\cdot 10^{28} k^{10}(\log k)^3 >(4r^2)^r=256$ for all $k\ge 2$. We get \begin{align*} m&<2^2\cdot 1.5\cdot 10^{28} k^{10}(\log k)^3\left(\log 1.5\cdot 10^{28} k^{10}(\log k)^3\right)^2\\ &=6\cdot 10^{28} k^{10}(\log k)^3 \left(\log (1.5\cdot 10^{28})+10\log k+3\log\log k\right)^2\\ &<6\cdot 10^{28} k^{10}(\log k)^5 \left(\dfrac{65}{\log k}+10+\dfrac{3\log\log k}{\log k}\right)^2, \end{align*} so $m<6.3\cdot 10^{32}k^{10}(\log k)^5 $. \end{proof} \subsubsection{The case $k>800$}\label{sub322} Here, we proceed by assuming for a moment that $k>800$. We get $m<6.3\cdot 10^{32}k^{10}(\log k)^5 <2^{0.28k}$, for all $k>800$. Assume $n\ge 3$. We can rewrite \eqref{eq:main} as \begin{align*} L_m^{(k)} &=\left(3\cdot 2^{n-1}\right)^x+\left(3\cdot 2^{n-2}\right)^x-\left(3\cdot 2^{n-3}\right)^x. \end{align*} By part $(i)$ of Lemma \ref{fay} with $c:=0.28$, we have \begin{align*} \left|\left(3\cdot 2^{n-1}\right)^x+\left(3\cdot 2^{n-2}\right)^x-\left(3\cdot 2^{n-3}\right)^x-3\cdot 2^{m-2}\right|<3\cdot 2^{m-2}\cdot \dfrac{4}{2^{0.72k}}, \end{align*} which can be rewritten as \begin{align}\label{eqc} \left|3^{x-1}(2^{2x}+2^x-1)-2^{m-2-(n-3)x}\right|<2^{m-2-(n-3)x} \cdot\frac{ 4}{2^{0.72 k}}. \end{align} Now, putting $y:=m-2-(n-3)x$ in \eqref{eqc} and dividing through by $2^y$, we get \begin{align}\label{eq:0} \left|\frac{3^{x-1} (2^{2x}+2^x-1)}{2^y}-1\right|<\frac{4}{2^{0.72 k}}. \end{align} This was for $n\ge 3$. For $n=0,1,2$ (since $k\ge 3$ for $n=0,2$), the same argument leads to the following inequalities \begin{eqnarray*} \left|\frac{2^{x-(m-2)} (1+2^{-x})}{3}-1\right| & < & \frac{4}{2^{0.72k}},\qquad n=0;\\ \left|\frac{3^{x-1}(1-(2^x-1)/3^x)}{2^{m-2}}-1\right| & < & \frac{4}{2^{0.72k}},\qquad n=1;\\ \left|\frac{3^{x-1}(1+(3^x-1)/6^x)}{2^{m-2-x}}-1\right| & < & \frac{4}{2^{0.72k}}, \qquad n=2. \end{eqnarray*} The first one is false which shows that the case $n=0$ cannot hold for $k>800$. Indeed, either $x-(m-2)\le 1$ or $x-(m-2)\ge 2$. If $x-(m-2)\le 1$, then $$ \frac{2^{x-(m-2)} (1+2^{-x})}{3}\le \frac{2(1+1/2^2)}{3}=\frac{5}{6},\qquad {\text{\rm so}}\qquad \left|\frac{2^{x-(m-2)}(1+2^{-x})}{3}-1\right|\ge \frac{1}{6}, $$ while if $x-(m-2)\ge 2$, then $$ \frac{2^{x-(m-2)} (1+2^{-x})}{3}\ge \frac{2^2}{3}=\frac{4}{3},\qquad {\text{\rm so}}\qquad \left|\frac{2^{x-(m-2)}(1+2^{-x})}{3}-1\right|\ge \frac{1}{3}. $$ Thus, in all cases, when $n=0$, we get $1/6<4/2^{0.72k}$, so $2^{0.72k}<24$, which is false for $k>800$. Going back to the case $n\ge 3$, the left--hand side in \eqref{eq:0} is \begin{align*} \left|e^{(x-1)\log 3+\log(2^{2x}+2^x+1)-y\log 2}-1\right|, \end{align*} and the right--hand side is $<1/2$. So, we get by a simple argument in calculus that \begin{align*} \left|(x-1)\log 3+\log(2^{2x}+2^x-1)-y\log 2\right|<\frac{8}{2^{0.72 k}}. \end{align*} This can be rewritten as \begin{align*} \left|(x-1)\log 3+2x\log 2+\log\left(1+\frac{2^x-1}{2^{2x}}\right)-y\log 2\right|<\frac{8}{2^{0.72k}}, \end{align*} or \begin{equation}\label{eq:1} |(x-1)\log 3-z\log 2|<\frac{8}{2^{0.72k}}+\log\left(1+\frac{2^x-1}{2^{2x}}\right)<\frac{8}{2^{0.72k}}+\frac{2^x-1}{2^{2x}}, \end{equation} with $z:=y-2x$. Since $x\ge 2$, the right--hand side in \eqref{eq:1} is $<6/16$, so $z>0$. The cases $n=1,~2$ lead, by similar arguments, to \begin{equation}\label{eq:1prime} |(x-1)\log 3-z\log 2|<\frac{8}{2^{0.72k}}+\delta(x),\qquad \delta(x):=\begin{cases} -\log\left(1-\dfrac{2^x-1}{3^x}\right), & \text{if} \ n=1, \\ \log\left(1+\dfrac{3^x-1}{6^x}\right), & \text{if} \ n=2,\end{cases} \end{equation} and $z:=m-2$ when $n=1$ and $z:=m-2-x$, when $n=2$. In both cases above the right--hand sides are $<3/4$ so $z>0$. We get that $$ \delta(x)\le \frac{2(2^x-1)}{3^x}\quad (n=1)\qquad {\text{\rm and}}\qquad \delta(x)<\frac{3^x-1}{6^x}\qquad (n=2). $$ Further, $$ z\log 2\le (x-1)\log 3+|(x-1)\log 3-z\log 2|<(x-1)\log 3+\frac{3}{4}, $$ so \begin{equation} \label{eq:2} \frac{z}{\log 3}<\frac{x-1}{\log 2} +\frac{3}{4(\log 2\log 3)}<\frac{x-1}{\log 2}+1<2x. \end{equation} The left--hand side in \eqref{eq:1} (or \eqref{eq:1prime}) is of the form \begin{align*} |b_1\log \gamma_1-b_2\log \gamma_2|, \end{align*} where $\gamma_1:=3,~\gamma_2:=2,~b_1:=x-1$ and $b_2:=z$. The numbers $\gamma_1$, $\gamma_2$ are rational so $D=1$. We take \begin{align*} \log A_1\ge \max\left\{h(\gamma_{1}), |\log \gamma_1|, 1\right\},\quad \log A_2\ge \max\left\{h(\gamma_{2}), |\log \gamma_2|, 1\right\}, \end{align*} so that $\log A_1=\log 3$ and $\log A_2=1$. Further, \begin{align*} b'=\frac{b_1}{D\log A_2}+\frac{b_2}{D\log A_1}=x-1+\frac{z}{\log 3}<x-1+2x<3x, \end{align*} by \eqref{eq:2}. Since $\gamma_1,~\gamma_2$ are real, positive and multiplicatively independent, Theorem \ref{thm:LMNh} shows that \begin{align}\label{eq:b} |(x-1)\log 3-z\log 2|>\exp\left(-24.34 \left(\max\left\{21, \log b'+0.14\right\}\right)^2\log 3\right). \end{align} Since \begin{align*} \frac{8}{2^{0.72k}}+\delta(x)\le \frac{8}{2^{0.72k}}+\frac{2}{2^{x/2}}\le \frac{10}{2^{\min\{0.72k,rx\}}}, \end{align*} where \begin{equation} \label{eq:delta} \delta(x)\in\left\{\frac{2^x-1}{2^{2x}},\frac{2(2^x-1)}{3^x},\frac{3^x-1}{6^x}\right\}, \end{equation} and we take $r=1$ except if $n=1$ when $r=1/2$, we compare \eqref{eq:1} and \eqref{eq:b} to conclude that \begin{equation} \label{eq:3} \min \{0.72 k,rx\} \log 2-\log 10<24.34 (\max\{21,\log(3x)+0.14\})^2\log 3. \end{equation} We proceed in two cases, when $\min \{0.72 k,x\}$ is $0.72k$ or $rx$. \medskip \begin{enumerate}[(i)] \item Assume first that $\min \{0.72 k,rx\}=0.72k$, then $$ (0.72 \log 2)k<24.34( \max\{21, \log(3x)+0.14\})^2\log 3+\log 10. $$ If $\max\{21, \log(3x)+0.14\}=21$, we get $ (0.72\log 2)k<24.34\cdot (21)^2\log 3+\log 10$, so $k<24000$. If $\max\{21, \log(3x)+0.14\}= \log(3x)+0.14$, we get $$ (0.72\log 2)k <24.34\log 3 (\log(3x)+0.14)^2+\log 10, $$ so that $$ k<54 (\log(3x)+0.14)^2+5<59(\log(3x)+0.14))^2, $$ for all $x\ge 2$. By Lemma \ref{lem3.2}, we have $$ \log m<\log 6.3+32\log 10+10(\log k)+5\log\log k, $$ and by Lemma \ref{lem3.1}, $$ x<3.1\cdot 10^{14} k^5 (\log k)^2 \log m. $$ Thus, \begin{align*} x &< 3.1\cdot 10^{14} (59(\log(3x)+0.14)^2)^{5} (\log 59+2\log(\log(3x)+0.14))^2\\ &\times (\log 6.3+32\log 10+10(\log(59(\log(3x)+0.14)^2))+5\log(\log 59+2\log(\log(3x)+0.14))). \end{align*} We get $x<10^{47}$. \medskip \item Next, if $\min \{0.72 k,rx\}=rx$, we then get that $$ rx\log 2-\log 10<24.34(\max\{21,\log(3x)+0.10\})^2\log 3. $$ If the maximum in the right above is in $21$, we then get $rx<18000$, while if the maximum is in $\log(3x)+0.14$, we then get $$ rx\log 2<24.34\log 3 (\log(3x)+0.14)^2+\log 10\le 24.34\log 3(\log(3rx)+0.14+\log 2)^2+\log 10, $$ so that $rx<4000$. \end{enumerate} \medskip So, in all cases $x<10^{47}$. We now return to \eqref{eq:1} which we write as \begin{align*} \left|\frac{\log 3}{\log 2}-\frac{z}{x-1}\right|<\frac{8/2^{0.72k}+\delta(x)}{(x-1)\log 2}<\frac{12/2^{0.72k}+2\delta(x)}{x-1}, \end{align*} where $\delta(x)$ is one of the three functions appearing in \eqref{eq:delta} (and $\delta(x)=(2^x-1)/2^{2x}$ if $n\ge 3$). We generated the $97$th convergent $p_{97}/q_{97}=[a_0,a_1,\ldots,a_{97}]=[1,1,\ldots,3]$ of $\log 3/\log 2$ and we got that $q_{97}>10^{47}$ and that $\max\{a_k: 0\le k\le 97\}=55$. By well--known properties of continued fractions, we get \begin{align*} \frac{1}{57(x-1)^2}<\left|\frac{\log 3}{\log 2}-\frac{y-2x}{x-1}\right|<\frac{12/2^{0.72k}+2\delta(x)}{x-1}. \end{align*} Therefore, we have \begin{align*} \frac{1}{57(x-1)}<\frac{12}{2^{0.72k}}+2\delta(x). \end{align*} The left--hand side is at least $1/(57\cdot 10^{47})$, while $12/2^{0.72k}<10^{-170}$ since $k>800$. So, we get $$ \frac{1}{58 (x-1)}<2\delta(x), $$ which gives $x\le 10$ except if $n=1$ for which $x\le 20$. The cases $n=1,2$ lead to $m<(n+3)x+3<100<k$, a contradiction. Thus, $x\le 10$, $n\ge 3$, $z=y-2x$ and \begin{equation} \label{eq:4} |(x-1)\log 3-(y-2x)\log 2|<\frac{8}{2^{0.72k}}+\frac{2^x-1}{2^{2x}}. \end{equation} We used a simple code to compute the minimum values of the expression \[ |(x-1)\log 3 - (y-2x)\log 2|, \] for integer values of \( y - 2x \). By assigning values to \( x \) ranging from 2 to 10, the code found the \( y \) that minimized the expression for each \( x \). The results showed that the left--hand side of \eqref{eq:4} was at least \[ 0.28,~0.11,~0.16,~0.23,~0.05,~0.33,~0.06,~0.22,~0.18, \] for \( x = 2, 3, \ldots, 10 \), respectively. It turns out that for no $x\in \{2,3,\ldots,10\}$ inequality \eqref{eq:4} is satisfied. This finishes the argument that the case $k>800$ is not possible. \subsubsection{The case $k\le800$} To proceed, we go back to \eqref{eq:gen} with the assumption that $3\le n \le k$ and write \begin{align*} |\Lambda_1|:=|\log(\Gamma_1+1)|=\left|\log f_k(\alpha)+\log(2\alpha-1)+(m-1)\log\alpha-x\log 3-(n-1)x\log 2\right|<\dfrac{4.5}{2^x}, \end{align*} where $a_1 := 1$, $a_2 := 1$, $a_3 := m-1$, $a_4 := -x$, $a_5 := -(n-1)x$ are integers with \[ \max\{|a_i| : 1 \leq i \leq 5\} < m<6.3 \cdot 10^{32} k^{10} (\log k)^5< 10^{66}, \] where we used Lemma \ref{lem3.2}, for all $k\le 800$. For each $k \in [2, 800]$, we used the LLL--algorithm to compute a lower bound for the smallest nonzero number of the form $|\Lambda_1|$, with integer coefficients $a_i$ not exceeding $6.3 \cdot 10^{32} k^{10} (\log k)^5$ in absolute value. Specifically, we consider the approximation lattice $$ \mathcal{A}=\begin{pmatrix} 1 & 0 & 0 & 0 & 0 \\ 0 & 1 & 0 & 0 & 0 \\ 0 & 0 & 1 & 0 & 0 \\ 0 & 0 & 0 & 1 & 0 \\ \lfloor C\log f_k(\alpha)\rfloor & \lfloor C\log (2\alpha-1) \rfloor& \lfloor C\log \alpha \rfloor& \lfloor C\log 3 \rfloor & \lfloor C\log2 \rfloor \end{pmatrix} ,$$ with $C:= 10^{331}$ and choose $y:=\left(0,0,0,0,0\right)$. Now, by Lemma \ref{lem2.5}, we get $$c_2=10^{-69}\qquad\text{and}\qquad l\left(\mathcal{L},y\right)\ge c_1:=3.13\cdot10^{68}.$$ So, Lemma \ref{lem2.6} gives $S=5\cdot 10^{132}$ and $T=2.5\cdot 10^{66}$. Since $c_1^2\ge T^2+S$, then choosing $c_3:=4.5$ and $c_4:=\log2$, we get $x\le 872$. Next, if $n\in\{0,1,2\}$, we instead go back to \eqref{g1} together with \eqref{eq:gen1} and write \begin{align*} \left|\log f_k(\alpha)+\log(2\alpha-1)+(m-1)\log\alpha-x\log \delta\right|<\dfrac{4.5}{1.5^x}, \end{align*} where $a_1 := 1$, $a_2 := 1$, $a_3 := m-1$, $a_4 := -x$ are integers with $\max\{|a_i| : 1 \leq i \leq 5\} < 10^{66}$, as before. Again, for each $k \in [2, 800]$ and each $\delta\in\{2,3,6\}$, we used the LLL--algorithm to compute a lower bound for the smallest nonzero number of the form above and consider the approximation lattice $$ \mathcal{A}=\begin{pmatrix} 1 & 0 & 0 & 0 \\ 0 & 1 & 0 & 0 \\ 0 & 0 & 1 & 0 \\ \lfloor C\log f_k(\alpha)\rfloor & \lfloor C\log (2\alpha-1) \rfloor& \lfloor C\log \alpha \rfloor& \lfloor C\log \delta \rfloor \end{pmatrix} ,$$ with $C:= 10^{265}$ and choose $y:=\left(0,0,0,0\right)$. So, Lemma \ref{lem2.5} gives $c_2=10^{-70}$ and $l\left(\mathcal{L},y\right)\ge c_1:=2.94\cdot10^{68}$. Moreover, Lemma \ref{lem2.6} gives $S=4\cdot 10^{132}$ and $T=2\cdot 10^{66}$. Since $c_1^2\ge T^2+S$, then choosing $c_3:=4.5$ and $c_4:=\log1.5$, we get $x\le 1119$. Thus, $x\le 1119$ for all cases when $0\le n\le k\le 800$. Finally in this subsection, we go back to \eqref{g2} and write $|\Lambda_2|:=|\log(\Gamma_2+1)|$ as \begin{align*} \left|\log f_k(\alpha)+\log(2\alpha-1)+(m-1)\log\alpha-x\log 3-(n-2)x\log 2-\log \left(2^x+1-2^{-x}\right)\right|<\dfrac{3}{\alpha^{m-1}} \end{align*} where $a_1 := 1$, $a_2 := 1$, $a_3 := m-1$, $a_4 := -x$, $a_5 := -(n-2)x$ and $a_6:=-1$ (in case $n\ge 3$) are integers with \[ \max\{|a_i| : 1 \leq i \leq 5\} < m<6.3 \cdot 10^{32} k^{10} (\log k)^5< 10^{66}, \] where we used Lemma \ref{lem3.2}, for all $k\le 800$. So, for each $k \in [2, 800]$ and each $x\in [1,1119]$ we used the LLL--algorithm to compute a lower bound for the smallest nonzero number of the form $|\Lambda_2|$, with integer coefficients $a_i$ not exceeding $6.3 \cdot 10^{32} k^{10} (\log k)^5$ in absolute value. That is, we instead consider the approximation lattice $$ \mathcal{A}=\begin{pmatrix} 1 & 0 & 0 & 0 & 0 & 0 \\ 0 & 1 & 0 & 0 & 0 & 0\\ 0 & 0 & 1 & 0 & 0 & 0\\ 0 & 0 & 0 & 1 & 0 & 0\\ 0 & 0 & 0 & 0 & 1 & 0\\ \lfloor C\log f_k(\alpha)\rfloor & \lfloor C\log (2\alpha-1) \rfloor& \lfloor C\log \alpha \rfloor& \lfloor C\log 3 \rfloor & \lfloor C\log2 \rfloor & \lfloor C\log\left(2^x+1-2^{-x}\right) \rfloor \end{pmatrix} ,$$ with $C:= 10^{396}$ and choose $y:=\left(0,0,0,0,0,0\right)$. Now, by Lemma \ref{lem2.5}, we get $$c_2:=10^{-69}\qquad\text{and}\qquad l\left(\mathcal{L},y\right)\ge c_1:=3.87\cdot10^{68}.$$ So, Lemma \ref{lem2.6} gives $S=6\cdot 10^{132}$ and $T=3\cdot 10^{66}$. Since $c_1^2\ge T^2+S$, then choosing $c_3:=3$ and $c_4:=\log\alpha$, we get $m\le 1474$. This was for $n\ge 3$. If $n\in \{0,1,2\}$, then $m<(n+3)x+3\le 5x+3\le 5598$. Therefore, for the case when $0\le n\le k$, we have that all solutions $n$, $m$, $k$, $x$ to \eqref{eq:main} satisfy $k\in [2,800]$, $m\in [3,5597]$, $n\in [0,\min\{k,\lfloor(m+3)/x\rfloor\}]$ and $x\in[1,1119]$. A computational search in SageMath within these specified ranges for solutions to the Diophantine equation \eqref{eq:main} yielded only the solutions presented in the Theorem \ref{thm1.1}. To efficiently handle large \( L_n^{(k)} \) values, our SageMath 9.5 code utilized batch processing, iterating through all \( (n, m,k, x) \) combinations within these ranges. \subsection{The case $n> k$} \subsubsection{Bounding $x$, $n$ and $m$ in terms of $k$} Again, we proceed as in Subsection \ref{subsec3.2}. We start by proving a series of results. \begin{lemma}\label{lem3.3} Let $n$, $m$, $k$, $x$ be integer solutions to Eq. \eqref{eq:main} with $n>k\ge 2$, $x\ge 2$ and $m\ge 3$, then \begin{align*} x<2.6\cdot 10^{15} n k^4(\log k)^3\log n . \end{align*} \end{lemma} \begin{proof} We go back to \eqref{eq:main} and rewrite it using the Binet formula in \eqref{eq:Lnwitherror} as \begin{align*} L_m^{(k)}-f_k(\alpha)(2\alpha-1)\alpha^{m-1}&=e_k(m)\\ f_k(\alpha)(2\alpha-1)\alpha^{m-1}-\left(L_{n+1}^{(k)}\right)^{x}&=\left(L_{n}^{(k)}\right)^x-\left(L_{n-1}^{(k)}\right)^x-e_k(m)\\ 0<f_k(\alpha)(2\alpha-1)\alpha^{m-1}\left(L_{n+1}^{(k)}\right)^{-x}-1&=\left(\dfrac{L_{n}^{(k)}}{L_{n+1}^{(k)}}\right)^x-\left(\dfrac{L_{n-1}^{(k)}}{L_{n+1}^{(k)}}\right)^x-\dfrac{e_k(m)}{\left(L_{n+1}^{(k)}\right)^x}\\ &<\left(\dfrac{L_{n}^{(k)}}{L_{n+1}^{(k)}}\right)^x<0.7^x<\dfrac{1}{1.4^x}, \end{align*} The above calculation is justified for $n>k\ge 2$ and $x\ge 2$, because then $n\ge 3$ so $L_n^{(k)}>L_{n-1}^{(k)}> 1$, so since $x\ge 2$, $$ \left(L_{n}^{(k)}\right)^x\ge \left(L_{n-1}^{(k)}+1\right)^x>\left(L_{n-1}^{(k)}\right)^x+x\ge \left(L_{n-1}^{(k)}\right)^x+2>\left(L_{n-1}^{(k)}\right)^x+e_k(m) $$ and $$ \left(L_{n-1}^{(k)}\right)^x\ge 2^x\ge 4>1.5>-e_{k}(m),\qquad {\text{\rm so}}\qquad (L_{n-1}^{(k)})^x+e_k(m)>0. $$ We thus get \begin{align}\label{g3} |\Gamma_3|:=\left|f_k(\alpha)(2\alpha-1)\alpha^{m-1}\left(L_{n+1}^{(k)}\right)^{-x}-1\right|<\dfrac{1}{1.4^x}. \end{align} Note that $\Gamma_3\ne 0$, otherwise we would have \begin{align*} f_k(\alpha)= \dfrac{\left(L_{n+1}^{(k)}\right)^x}{(2\alpha-1)\alpha^{m-1}}. \end{align*} Taking norms in ${\mathbb K}={\mathbb Q}(\alpha)$ as before, the above equation becomes \begin{align*} N\left(f_k(\alpha)\right)\cdot N(2\alpha-1)=\left(L_{n+1}^{(k)}\right)^{kx}. \end{align*} We have already shown before that the left--hand side above is $\le 1$. Therefore, \begin{align*} 1\ge N\left(f_k(\alpha)\right)\cdot N(2\alpha-1)=\left(L_{n+1}^{(k)}\right)^{kx}\ge \left(L_{4}^{(2)}\right)^4=2401, \end{align*} a contradiction, for all $n>k\ge 2$ and $x\ge 2$. The algebraic number field containing the following $\gamma_i$'s is $\mathbb{K} := \mathbb{Q}(\alpha)$. We have $D = k$, $t :=3$, \begin{equation}\nonumber \begin{aligned} \gamma_{1}&:=L_{n+1}^{(k)}, \qquad\gamma_{2}:=(2\alpha-1)f_k(\alpha),\qquad \gamma_{3}:=\alpha,\\ b_{1}&:=-x,\qquad b_{2}:=1,\qquad b_{3}:=m-1. \end{aligned} \end{equation} Since $h\left(L_{n+1}^{(k)}\right)\le h(2\alpha^n)<n\log2\alpha<n\log 4<1.4n$, we can take $A_1:=1.4kn$. As before, we take $A_{2}:=6k\log k$, $A_{3}:=0.7$ and $B:=m$. Now, by Theorem \ref{thm:Mat}, \begin{align}\label{eq3.12} \log |\Gamma_3| &> -1.4\cdot 30^{6} \cdot 3^{4.5}\cdot k^2 (1+\log k)(1+\log m)\cdot 1.4kn\cdot 0.7\cdot 6k\log k\nonumber\\ &> -4.0\cdot 10^{12} n k^4(\log k)^2 \log m. \end{align} Comparing \eqref{g3} and \eqref{eq3.12}, we get \begin{align*} x\log 1.4&<4.0\cdot 10^{12} n k^4(\log k)^2 \log m,\\ x&<1.2\cdot 10^{13} n k^4(\log k)^2 \log m. \end{align*} Now, by inequality \eqref{m_b}, $m<(n+3)x+3$, hence \begin{align*} x&<1.2\cdot 10^{13} n k^4(\log k)^2 \log ((n+3)x+3)\\ &<2.7\cdot 10^{13} n k^4(\log k)^2 \log (nx). \end{align*} Now, multiplying $n$ on both sides of the above inequality, we get $nx<2.7\cdot 10^{13} n^2 k^4(\log k)^2 \log (nx)$. We now apply Lemma \ref{Guz} with $p:=nx$, $r:=1$, $T:=2.7\cdot 10^{13} n^2 k^4(\log k)^2$ and have \begin{align*} nx&<2\cdot 2.7\cdot 10^{13} n^2 k^4(\log k)^2\log 2.7\cdot 10^{13} n^2 k^4(\log k)^2\\ &=5.4\cdot 10^{13} n^2 k^4(\log k)^2 \left(\log (2.7\cdot 10^{13})+2\log n+4\log k+2\log\log k\right)\\ &<5.4\cdot 10^{13} n^2 k^4(\log k)^3\log n \left(\dfrac{31}{\log n\log k}+\dfrac{2}{\log k}+\dfrac{4}{\log n}+\dfrac{2\log\log k}{\log n\log k}\right), \end{align*} so that $nx<2.6\cdot 10^{15} n^2 k^4(\log k)^3\log n $. Dividing both sides by $n$, we get $x<2.6\cdot 10^{15} n k^4(\log k)^3\log n $. \end{proof} To proceed, we now work under the assumption that $n > 800$, so that we can explicitly determine an upper bound for $n$, $m$ and $x$ in terms of $k$ only. We prove the following result. \begin{lemma}\label{lem3.4} Let $n$, $m$, $k$, $x$ be integer solutions to Eq. \eqref{eq:main} with $n>\max\{800,k\}$, $x\ge 2$ and $m\ge 3$, then \begin{align*} n<5.5\cdot 10^{27} k^6(\log k)^6, \qquad x< 8.7\cdot 10^{30}k^7(\log k)^8\qquad\text{and}\qquad m<5.6\cdot 10^{44}k^{10}(\log k)^{12}. \end{align*} \end{lemma} \begin{proof} Since $n>k$, then Lemma \ref{lem3.3} tells us that \begin{align}\label{x_bound1} x<2.6\cdot 10^{15} n^5(\log n)^4. \end{align} So, for each $i\in\{-1,0,1\}$, the inequality \begin{align*} \dfrac{x}{\alpha^{n+i-1}}<\dfrac{2.6\cdot 10^{15} n^5(\log n)^4}{\alpha^{n-2}}<\dfrac{1}{\alpha^{0.7n}}, \end{align*} holds for all $n>800$. Now, Lemma \ref{lem:Lnx} tells us that \begin{align}\label{eq3.14} \left(L_{n+i}^{(k)}\right)^x = f_k(\alpha)^x (2\alpha-1)^x\alpha^{(n+i-1)x}(1 + \eta_{n}),\qquad\text{with}\qquad |\eta_{n}| < \dfrac{1.5xe^{1.5x/\alpha^{n+i-1}}}{\alpha^{n+i-1}}<\dfrac{10}{\alpha^{0.7n}}. \end{align} So, we rewrite \eqref{eq:main} using \eqref{eq3.14} as \begin{align*} f_k(\alpha)(2\alpha-1)\alpha^{m-1}&-f_k(\alpha)^x(2\alpha-1)^x\alpha^{(n-1)x}\left(1+\alpha^{-x}-\alpha^{-2x}\right)\\ &=f_k(\alpha)^x(2\alpha-1)^x\alpha^{(n-1)x}\left(1+\alpha^{-x}-\alpha^{-2x}\right)\eta_n+e_k(m). \end{align*} Dividing both sides of the above equality by $f_k(\alpha)^x(2\alpha-1)^x\alpha^{(n-1)x}$ and taking absolute values, we get \begin{align*} &\left|f_k(\alpha)(2\alpha-1)\alpha^{m-1}f_k(\alpha)^{-x}(2\alpha-1)^{-x}\alpha^{-(n-1)x}-\left(1+\alpha^{-x}-\alpha^{-2x}\right)\right|\\ &<|\eta_n|\left(1+\alpha^{-x}-\alpha^{-2x}\right)+\dfrac{1.5}{f_k(\alpha)^x(2\alpha-1)^x\alpha^{(n-1)x}}\\ &<\dfrac{15}{\alpha^{0.7n}}+\dfrac{1.5}{\alpha^{(n-1)x}}<\dfrac{18}{\alpha^{0.7n}}, \end{align*} where we have used the fact that $0<1+\alpha^{-x}-\alpha^{-2x}<1.5$, $f_k(\alpha)(2\alpha-1)\alpha^{n-1}>\alpha^{n-1}$ and $(n-1)x>0.7n$, for all $n>800$, $x\ge 2$ and $k\ge 2$. This means that \begin{align}\label{g4} |\Gamma_4|:= \left|f_k(\alpha)^{1-x}(2\alpha-1)^{1-x}\alpha^{m-1-(n-1)x}-1\right|<\dfrac{18}{\alpha^{0.7n}}+\dfrac{1}{\alpha^x}+\dfrac{1}{\alpha^{2x}}<\dfrac{27}{\alpha^{\min\{0.7n,x\}}}. \end{align} Note that $\Gamma_4\ne 0$, otherwise we would have \begin{align}\label{fk} \left((2\alpha-1)f_k(\alpha)\right)^{x-1}=\alpha^{m-1-(n-1)x}. \end{align} Applying norms in ${\mathbb K}={\mathbb Q}(\alpha)$ and using $|N(\alpha)|=1$, equation \eqref{fk} becomes \begin{align*} |N\left((2\alpha-1)f_k(\alpha)\right)|^{x-1}=1, \end{align*} which implies that $x=1$, for $k\ge 3$, contradicting the working assumption that $x\ge 2$. For the special case when $k=2$, we have that $(2\alpha-1)f_k(\alpha)=\alpha$, so that \eqref{fk} becomes $\alpha^{x-1}=\alpha^{m-1-(n-1)x}$, giving $m=nx$. At this point, we go back and rewrite \eqref{eq:main} with $k=2$ and $m=nx$ as \begin{align*} \left(L_{n+1}\right)^x+\left(L_{n}\right)^x-\left(L_{n-1}\right)^x=L_{nx}. \end{align*} Now, since $\left(L_{n}\right)^x-\left(L_{n-1}\right)^x>0$ for all $x\ge 2$ and $n\ge 4$, we have by the Binet formula $L_n=\alpha^n+\beta^n$, that \begin{align*} \left(\alpha^{n+1}-1\right)^x<\left(\alpha^{n+1}+\beta^{n+1}\right)^x= \left(L_{n+1}\right)^x\le \left(L_{n+1}\right)^x+\left(L_{n}\right)^x-\left(L_{n-1}\right)^x=L_{nx}< \alpha^{nx}+1. \end{align*} This leads to \begin{align*} \alpha^x\left(1-\frac{1}{\alpha^{n+1}}\right)^x< 1+\frac{1}{\alpha^{nx}}, \end{align*} so that \begin{align*} 2< \alpha^2\left(1-\frac{1}{\alpha^{4+1}}\right)^2\le \alpha^x\left(1-\frac{1}{\alpha^{n+1}}\right)^x< 1+\frac{1}{\alpha^{nx}}\le 1+\frac{1}{\alpha^{4\cdot 2}}<1.1, \end{align*} which is also a contradiction. Thus, $\Gamma_4\ne 0$. The algebraic number field containing the following $\gamma_i$'s is $\mathbb{K} := \mathbb{Q}(\alpha)$. We have $D = k$, $t :=2$, \begin{equation}\nonumber \begin{aligned} \gamma_{1}&:=(2\alpha-1)f_k(\alpha),\qquad \gamma_{2}:=\alpha,\\ b_{1}&:=1-x,\qquad b_{2}:=m-1-(n-1)x. \end{aligned} \end{equation} As before, we take $A_{1}:=6k\log k$ and $A_{2}:=0.7$. By the second inequality in \eqref{m_b}, we can take $B:=x$. So, Theorem \ref{thm:Mat} gives \begin{align}\label{eq3.16} \log |\Gamma_4| &> -1.4\cdot 30^{5} \cdot 2^{4.5}\cdot k^2 (1+\log k)(1+\log x)\cdot 0.7\cdot 6k\log k\nonumber\\ &> -2.0\cdot 10^{10} k^3(\log k)^2 \log x. \end{align} Comparing \eqref{g4} and \eqref{eq3.16}, we get \begin{align*} \min\{0.7n,x\}\log\alpha-\log 27&<2.0\cdot 10^{10} k^3(\log k)^2 \log x,\\ \min\{0.7n,x\}&<5.0\cdot 10^{10} k^3(\log k)^2 \log x. \end{align*} We distinguish between two cases. \textbf{Case I}: If the $\min\{0.7n,x\}=0.7n$, then $n<7.2\cdot 10^{10} k^3(\log k)^2 \log x$, and using Lemma \ref{lem3.3}, we get \begin{align*} n&<7.2\cdot 10^{10} k^3(\log k)^2 \log \left( 2.6\cdot 10^{15} n k^4(\log k)^3\log n \right) \\ &=7.2\cdot 10^{10} k^3(\log k)^2 \left(\log 2.6\cdot 10^{15} +\log n +4\log k +3\log\log k+\log\log n \right)\\ &<7.2\cdot 10^{10} k^3(\log k)^3\log n \left(\dfrac{36}{\log n\log k} +\dfrac{1}{\log k} +\dfrac{4}{\log n} +3\dfrac{\log\log k}{\log n\log k}+\dfrac{\log\log n}{\log n\log k} \right)\\ &<7.3\cdot 10^{11} k^3(\log k)^3\log n. \end{align*} This tells us that $n/\log n<7.3\cdot 10^{11} k^3(\log k)^3$. Applying Lemma \ref{Guz} with $$ p:=n, \quad r:=1, \quad T:=7.3\cdot 10^{11} k^3(\log k)^3, $$ we get \begin{align*} n&<2\cdot7.3\cdot 10^{11} k^3(\log k)^3\log \left(7.3\cdot 10^{11} k^3(\log k)^3\right)\\ &<1.5\cdot 10^{12} k^3(\log k)^3 \left(\log (7.3\cdot 10^{11})+3\log k+3\log\log k\right)\\ &<1.5\cdot 10^{12} k^3(\log k)^4 \left(\dfrac{28}{\log k}+3+\dfrac{3\log\log k}{\log k}\right), \end{align*} so that $n<6.3\cdot 10^{13} k^3(\log k)^4$. Inserting this upper bound of $n$ in Lemma \ref{lem3.3}, we have \begin{align*} x&<2.6\cdot 10^{15} \left(6.3\cdot 10^{13} k^3(\log k)^4\right) k^4(\log k)^3\log \left(6.3\cdot 10^{13} k^3(\log k)^4\right)\\ &< 1.8\cdot 10^{29}k^7(\log k)^8\left(\dfrac{32}{\log k}+3+\dfrac{4\log\log k}{\log k}\right)\\ &< 8.7\cdot 10^{30}k^7(\log k)^8. \end{align*} By inequality \eqref{m_b}, $m<(n+3)x+3<5.6\cdot 10^{44}k^{10}(\log k)^{12}$. \medskip \textbf{Case II}: If the $\min\{0.7n,x\}=x$, then $x<5.0\cdot 10^{10} k^3(\log k)^2 \log x$. Applying Lemma \ref{Guz} with $p:=x$, $r:=1$, $T:=5.0\cdot 10^{10} k^3(\log k)^2$, we get \begin{align*} x&<2\cdot 5.0\cdot 10^{10} k^3(\log k)^2\log \left(5.0\cdot 10^{10} k^3(\log k)^2\right)\\ &=1.0\cdot 10^{11} k^3(\log k)^2 \left(\log (5.0\cdot 10^{10})+3\log k+2\log\log k\right)\\ &< 10^{11} k^3(\log k)^3 \left(\dfrac{25}{\log k}+3+\dfrac{2\log\log k}{\log k}\right), \end{align*} so that \begin{align}\label{xc} x<4.0\cdot 10^{12} k^3(\log k)^3. \end{align} Now, since $x\le 0.7n$, then for each $i\in\{-1,0,1\}$, the inequality \begin{align*} \dfrac{x}{\alpha^{n+i-1}}<\dfrac{0.7n}{\alpha^{n-2}}<\dfrac{1}{\alpha^{0.97n}}, \end{align*} holds for all $n>800$. Now, Lemma \ref{lem:Lnx} tells us that \begin{align}\label{eq3.17} \left(L_{n+i}^{(k)}\right)^x = f_k(\alpha)^x (2\alpha-1)^x\alpha^{(n+i-1)x}(1 + \eta_{n}),\qquad\text{with}\qquad |\eta_{n}| < \dfrac{1.5xe^{1.5x/\alpha^{n+i-1}}}{\alpha^{n+i-1}}<\dfrac{2}{\alpha^{0.97n}}. \end{align} So, we rewrite \eqref{eq:main} using \eqref{eq3.17} as \begin{align*} f_k(\alpha)(2\alpha-1)\alpha^{m-1}&-f_k(\alpha)^x(2\alpha-1)^x\alpha^{(n-1)x}\left(1+\alpha^{-x}-\alpha^{-2x}\right)\\ &=f_k(\alpha)^x(2\alpha-1)^x\alpha^{(n-1)x}\left(1+\alpha^{-x}-\alpha^{-2x}\right)\eta_n+e_k(m). \end{align*} Dividing both sides of the above equality by $ f_k(\alpha)(2\alpha-1)\alpha^{m-1}$ and taking absolute values, we get \begin{align*} &\left|f_k(\alpha)^{x-1}(2\alpha-1)^{x-1}\alpha^{(n-1)x-(m-1)}\left(1+\alpha^{-x}-\alpha^{-2x}\right)-1\right|\\ &<|\eta_n|f_k(\alpha)^{x-1}(2\alpha-1)^{x-1}\alpha^{(n-1)x-(m-1)}\left(1+\alpha^{-x}-\alpha^{-2x}\right)+\dfrac{1.5}{f_k(\alpha)(2\alpha-1)\alpha^{m-1}}\\ &<\dfrac{2}{\alpha^{0.97n}}\cdot \dfrac{1}{\left(f_k(\alpha)(2\alpha-1)\alpha\right)^{1-x}}\cdot \dfrac{1}{\alpha^{(m-2)-(n-2)x}} +\dfrac{1.5}{\alpha^{m-1}}\\ &<\dfrac{2}{\alpha^{0.97n}}\cdot \dfrac{1/\left(0.5\cdot 2\cdot1\right)^{1-x}}{\alpha^{-5}} +\dfrac{1.5}{\alpha^{n-4}}<\dfrac{2}{\alpha^{0.97n-5}} +\dfrac{1.5}{\alpha^{n-4}}\\ & <\dfrac{3}{\alpha^{0.97n-5}}, \end{align*} for $n>800$, $x\ge 2$ and $k\ge 2$. This means that \begin{align}\label{g5} |\Gamma_5|:= \left|f_k(\alpha)^{x-1}(2\alpha-1)^{x-1}\alpha^{(n-1)x-(m-1)}\left(1+\alpha^{-x}-\alpha^{-2x}\right)-1\right|<\dfrac{3}{\alpha^{0.97n-5}}. \end{align} Note that $\Gamma_5\ne 0$, otherwise we would have \begin{align*} \left((2\alpha-1)f_k(\alpha)\right)^{x-1}\alpha^{(n-1)x-(m-1)}\left(1+\alpha^{-x}-\alpha^{-2x}\right)=1. \end{align*} Taking norms in ${\mathbb K}={\mathbb Q}(\alpha)$ as before and using parts $(i)$ and $(ii)$ of Lemma \ref{lemGLm}, the above equation becomes \begin{align}\label{prod} \left(\dfrac{\left( 2^{k+1} - 3\right) (k - 1)^2}{ 2^{k+1}k^k - (k + 1)^{k+1}}\right)^{x-1}\cdot N\left(1+\alpha^{-x}-\alpha^{-2x}\right)&=1,\nonumber\\ \left(\dfrac{\left( 2^{k+1} - 3\right) (k - 1)^2}{ 2^{k+1}k^k - (k + 1)^{k+1}}\right)^{x-1}\cdot \left(1+\alpha^{-x}-\alpha^{-2x}\right)\prod_{j= 2}^k\left|1+(\alpha^{(j)})^{-x}-(\alpha^{(j)})^{-2x}\right|&=1. \end{align} Since $1+\alpha^{-x}-\alpha^{-2x}<2$, for all $x\ge 2$, we have from \eqref{prod} that \begin{align*} 1&= \left(\dfrac{\left( 2^{k+1} - 3\right) (k - 1)^2}{ 2^{k+1}k^k - (k + 1)^{k+1}}\right)^{x-1}\cdot \left(1+\alpha^{-x}-\alpha^{-2x}\right)\prod_{j= 2}^k\left|1+(\alpha^{(j)})^{-x}-(\alpha^{(j)})^{-2x}\right|\\ &<\left(\dfrac{\left( 2^{k+1} - 3\right) (k - 1)^2}{ 2^{k+1}k^k - (k + 1)^{k+1}}\right)^{x-1}\cdot 2\cdot \prod_{j= 2}^k\left|(\alpha^{(j)})^{-2x}-(\alpha^{(j)})^{-x}-1\right|\\ &<\left(\dfrac{\left( 2^{k+1} - 3\right) (k - 1)^2}{ 2^{k+1}k^k - (k + 1)^{k+1}}\right)^{x-1}\cdot 2\prod_{j= 2}^k3\left|(\alpha^{(j)})^{-2x}\right| <\left(\dfrac{\left( 2^{k+1} - 3\right) (k - 1)^2}{ 2^{k+1}k^k - (k + 1)^{k+1}}\right)^{x-1}\cdot 2\cdot 3^{k-1}\alpha^{2x}\\ &<\left(8^{x/(x-1)}\cdot3^{(k-1)/(x-1)} \dfrac{\left( 2^{k+1} - 3\right) (k - 1)^2}{ 2^{k+1}k^k - (k + 1)^{k+1}}\right)^{x-1} \\ &<\left(8^{2}\cdot3^{k-1} \dfrac{\left( 2^{k+1} - 3\right) (k - 1)^2}{ 2^{k+1}k^k - (k + 1)^{k+1}}\right)^{x-1} <1, \end{align*} for all $k\ge 8$ and $x\ge 2$, a contradiction. When $k=3,4,5,6,7$, we have that the expression \begin{align*} \dfrac{\left( 2^{k+1} - 3\right) (k - 1)^2}{ 2^{k+1}k^k - (k + 1)^{k+1}}, \end{align*} in \eqref{prod} is equal to $$ 13/44,\quad 29/563, \qquad 61/9584, \quad 125/205937\quad 253/5390272, $$ respectively. Since $13\nmid 44$, $29\nmid 563$, $61\nmid 9584$, $125\nmid 205937$ and $253\nmid 5390272$, equation \eqref{prod} can not hold when $k=3,4,5,6,7$. When $k=2$, we have that \begin{align*} \dfrac{\left( 2^{k+1} - 3\right) (k - 1)^2}{ 2^{k+1}k^k - (k + 1)^{k+1}}=1. \end{align*} In this case, equation \eqref{prod} becomes \begin{align*} \left(1+\alpha^{-x}-\alpha^{-2x}\right)\left(1+\beta^{-x}-\beta^{-2x}\right)=\pm 1, \end{align*} where $\alpha=\left(1+\sqrt5\right)/2$ and $\beta=\left(1-\sqrt5\right)/2 $. We multiply both sides of the above equation by $(\alpha\beta)^{2x}=1$ and get \begin{align}\label{lx} \left(\alpha^{2x}+\alpha^{x}-1\right)\left(\beta^{2x}+\beta^{x}-1\right)&=\pm 1,\nonumber\\ 1+ (-1)^x(\alpha^x+\beta^x)-\left(\alpha^{2x}+\beta^{2x}\right)+(-1)^x+1-(\alpha^x+\beta^x) &=\pm 1,\nonumber\\ 2+ ((-1)^x-1)L_x-L_{2x}+(-1)^x &=\pm 1. \end{align} Now, if $x$ is even in \eqref{lx}, then we have $L_{2x} =2,4$ leading to $x=0$ which contradicts the working assumption that $x\ge 2$. On the other hand, if $x$ is odd in \eqref{lx}, then $L_{2x}+L_x =0,2$, therefore we get that $L_{2x} =-L_x,~2-L_x$, which are both negative numbers for $x\ge 2$, another contradiction. Thus, in all cases, $\Gamma_5\ne 0$. The algebraic number field containing the following $\gamma_i$'s is $\mathbb{K} := \mathbb{Q}(\alpha)$. We have $D = k$, $t :=3$, \begin{equation}\nonumber \begin{aligned} \gamma_{1}&:=(2\alpha-1)f_k(\alpha),\qquad \gamma_{2}:=\alpha,\qquad\gamma_{3}:=\left(1+\alpha^{-x}-\alpha^{-2x}\right),\\ b_{1}&:=x-1,\qquad b_{2}:=(n-1)x-(m-1),\qquad b_3:=1. \end{aligned} \end{equation} As before, we take $A_{1}:=6k\log k$ and $A_{2}:=0.7$. For $A_3$, we first compute \begin{align*} Dh(\gamma_3)=kh\left(1+\alpha^{-x}-\alpha^{-2x}\right)<3kx(\log\alpha)/k, \end{align*} so that we can take $A_3:=3x$. By the second inequality in \eqref{m_b}, we can take $B:=x$. So, Theorem \ref{thm:Mat} gives \begin{align}\label{eq3.19} \log |\Gamma_5| &> -1.4\cdot 30^{6} \cdot 3^{4.5}\cdot k^2 (1+\log k)(1+\log x)\cdot 0.7\cdot 6k\log k\cdot 3x\nonumber\\ &> -1.1\cdot 10^{13} x k^3(\log k)^2 \log x. \end{align} Comparing \eqref{g5} and \eqref{eq3.19}, we get \begin{align*} (0.97n-5)\log\alpha-\log 3&<1.1\cdot 10^{13} x k^3(\log k)^2 \log x,\\ n&<2.8\cdot 10^{13} k^3(\log k)^2 x\log x. \end{align*} Moreover, by \eqref{xc}, $x<4.0\cdot 10^{12} k^3(\log k)^3$, so we have \begin{align*} n&<2.8\cdot 10^{13} k^3(\log k)^2 \cdot 4.0\cdot 10^{12} k^3(\log k)^3\cdot \log \left(4.0\cdot 10^{12} k^3(\log k)^3\right)\\ &<1.2\cdot 10^{26} k^6(\log k)^6\left(\dfrac{30}{\log k}+3+3\dfrac{\log\log k}{\log k}\right)\\ &<5.5\cdot 10^{27} k^6(\log k)^6. \end{align*} Lastly, by inequality \eqref{m_b}, $m<(n+3)x+3<2.3\cdot 10^{40}k^9(\log k)^{9}$. Comparing all inequalities of $n$, $m$ and $x$ in both cases, we have the inequalities in Lemma \ref{lem3.4}. \end{proof} \subsubsection{The case $k>800$} The inequalities in Lemma \ref{lem3.4} were obtained under the assumptions that $n > 800$. However, when $n \le 800$, the inequalities \eqref{m_b} and \eqref{x_bound1} yield even smaller upper bounds for $x$ and $m$ in terms of $k$. From now on, let us assume that $k > 800$. By Lemma \ref{lem3.4}, we get \begin{align*} n+i<5.51\cdot 10^{27} k^6(\log k)^6<2^{0.24k} \qquad\text{and}\qquad m<5.6\cdot 10^{44}k^{10}(\log k)^{12} <2^{0.39k}, \end{align*} for all $k>800$ and $i\in\{-1,0,1\}$. By Lemma \ref{Ln:x} with $c=0.39$ and $x=1$, we have \begin{align*} L_m^{(k)} = 3 \cdot 2^{m-2} \left( 1 + \xi_m \right), \quad \text{with} \quad |\xi_m| < \dfrac{2}{2^{0.22k}}. \end{align*} Again, by Lemma \ref{Ln:x} with $c=0.24$ and $x\ge 2$, we have \begin{align*} \left(L_{n+i}^{(k)}\right)^x = 3^x\cdot 2^{(n+i-2)x} \left(1 +\xi_{n+i} \right), \quad \text{with} \quad |\xi_{n+i}| <\dfrac{2}{2^{0.52k}}. \end{align*} Now, for all $n>k\ge 2$, we rewrite \eqref{eq:main} as \begin{align*} \left|3^x\cdot 2^{(n-1)x}\left(1+2^{-x}-2^{-2x}\right) - 3 \cdot 2^{m-2} \right|&\le\left( 3^x\cdot 2^{(n-1)x} \left(1+2^{-x}+2^{-2x}\right)+3 \cdot 2^{m-2} \right)\cdot \dfrac{2}{2^{0.22k}}, \end{align*} where we chose $2/2^{0.22k}$ over $2/2^{0.52k}$ since $2/2^{0.52k}<2/2^{0.22k}$. In the above, we divide through by the $\max\{3^x\cdot 2^{(n-1)x}\left(1+2^{-x}-2^{-2x}\right),~ 3 \cdot 2^{m-2}\}$. In the left--hand side, we have $$\left|3^{x-1}\cdot2^{-(m-2)}\cdot 2^{(n-1)x}\left(1+2^{-x}-2^{-2x}\right) -1\right|\quad\text{or}\quad \left|3^{1-x}\cdot2^{(m-2)}\cdot 2^{-(n-1)x}\left(1+2^{-x}-2^{-2x}\right)^{-1} -1\right|,$$ while on the right--hand side, we have $$ \left(\dfrac{1+2^{-x}+2^{-2x}}{1+2^{-x}-2^{-2x}}+\dfrac{3 \cdot 2^{m-2}}{3^x\cdot 2^{(n-1)x}\left(1+2^{-x}-2^{-2x}\right)} \right)\cdot \dfrac{2}{2^{0.22k}}<(1.25+1)\cdot \dfrac{2}{2^{0.22k}}= \dfrac{4.5}{2^{0.22k}}, $$ or $$ \left(\dfrac{3^x\cdot 2^{(n-1)x}(1+2^{-x}+2^{-2x})}{3 \cdot 2^{m-2}}+1 \right)\cdot \dfrac{2}{2^{0.22k}}\le(1+1)\cdot \dfrac{2}{2^{0.22k}}= \dfrac{4}{2^{0.22k}}, $$ respectively. Thus, in both cases, we have \begin{align}\label{eqc1} \left|3^{x-1}\cdot2^{-(m-2)}\cdot 2^{(n-1)x}\left(1+2^{-x}-2^{-2x}\right) -1\right|< \dfrac{4.5}{2^{0.22k}}. \end{align} Again, putting $y:=m-2-(n-3)x$ in \eqref{eqc1} and dividing through by $2^y$, we get \begin{align}\label{eq:01} \left|\frac{3^{x-1} (2^{2x}+2^x-1)}{2^y}-1\right|<\dfrac{4.5}{2^{0.22k}}. \end{align} Again, the left--hand side in \eqref{eq:01} is \begin{align*} \left|e^{(x-1)\log 3+\log(2^{2x}+2^x+1)-y\log 2}-1\right|, \end{align*} and the right--hand side is $<1/2$. So, we get by a simple argument in calculus that \begin{align*} \left|(x-1)\log 3+\log(2^{2x}+2^x-1)-y\log 2\right|<\dfrac{9}{2^{0.22k}}. \end{align*} This can be rewritten as \begin{align*} \left|(x-1)\log 3+2x\log 2+\log\left(1+\frac{2^x-1}{2^{2x}}\right)-y\log 2\right|<\dfrac{9}{2^{0.22k}}, \end{align*} or \begin{equation}\label{eq:11} |(x-1)\log 3-(y-2x)\log 2|<\dfrac{9}{2^{0.22k}}+\log\left(1+\frac{2^x+1}{2^{2x}}\right)<\dfrac{9}{2^{0.22k}}+\frac{2^x-1}{2^{2x}}. \end{equation} Since $x\ge 2$, the right--hand side in \eqref{eq:11} is still $<6/16$, so $z:=y-2x>0$ as before. This means that the conclusion in \eqref{eq:2} still holds even in this case. The left--hand side in \eqref{eq:11} is of the form \begin{align*} |b_1\log \gamma_1-b_2\log \gamma_2|, \end{align*} where $\gamma_1:=3,~\gamma_2:=2,~b_1:=x-1$ and $b_2:=z$. So, we take as before that $D=1$, $\log A_1=\log 3$, $\log A_2=1$ and $b'<3x$, by \eqref{eq:2}. Now, Theorem \ref{thm:LMNh} shows that \begin{align}\label{eq:b1} |(x-1)\log 3-(y-2x)\log 2|>\exp\left(-24.34 \left(\max\left\{21, \log b'+0.14\right\}\right)^2\log 3\right). \end{align} Since \begin{align*} \frac{9}{2^{0.22k}}+\frac{2^x-1}{2^{2x}}\le \frac{9}{2^{0.22k}}+\frac{1}{2^x}\le \frac{10}{2^{\min\{0.22k,x\}}}, \end{align*} we compare \eqref{eq:11} and \eqref{eq:b1} to conclude that \begin{equation} \label{eq:31} \min \{0.22 k,x\} \log 2-\log 10<24.34 (\max\{21,\log(3x)+0.14\})^2\log 3. \end{equation} We proceed in two cases, when $\min \{0.22 k,x\}$ is $0.22k$ or $x$. \begin{enumerate}[(i)] \item Assume first that $\min \{0.22 k,x\}=0.22k$, then $$ (0.22 \log 2)k<24.34( \max\{21, \log(3x)+0.14\})^2\log 3+\log 10. $$ If $\max\{21, \log(3x)+0.14\}=21$, we get $ (0.22\log 2)k<24.34\cdot (21)^2\log 3+\log 260$, so $k<77350$. If $\max\{21, \log(3x)+0.14\}= \log(3x)+0.14$, we get $$ (0.22\log 2)k <24.34\log 3 (\log(3x)+0.14)^2+\log 10, $$ so that $$ k<176 (\log(3x)+0.14)^2+16<187(\log(3x)+0.14))^2, $$ for all $x\ge 1$. By Lemma \ref{lem3.4}, we have $$ x<8.7\cdot 10^{30} k^7 (\log k)^8. $$ Thus, \begin{align*} x &< 8.7\cdot 10^{30} (187(\log(3x)+0.14)^2)^{7} (\log 187+2\log(\log(3x)+0.14))^8. \end{align*} We get $x<10^{89}$. \item Next, if $\min \{0.22 k,x\}=x$, we then get $$ x\log 2-\log 10<24.34(\max\{21,\log(3x)+0.10\})^2\log 3. $$ If the maximum in the right above is in $21$, we then get $x<18000$, while if the maximum is in $\log(3x)+0.14$, we then get $$ x\log 2<24.34\log 3 (\log(3x)+0.14)^2+\log 10, $$ so that $x<4000$. \end{enumerate} So, in all cases $x<10^{89}$. We now return to \eqref{eq:11} which we write as \begin{align*} \left|\frac{\log 3}{\log 2}-\frac{y-2x}{x-1}\right|<\frac{9/2^{0.22k}+(2^x-1)/(2^{2x})}{(x-1)\log 2}<\frac{13/2^{0.22k}+2(2^x-1)/(2^{2x})}{x-1}. \end{align*} Now, we generated the $187$th convergent $p_{187}/q_{187}=[a_0,a_1,\ldots,a_{187}]=[1,1,\ldots,2]$ of $\log 3/\log 2$ and we got that $q_{187}>10^{89}$ and that $\max\{a_k: 0\le k\le 187\}=55$. By well--known properties of continued fractions, we get \begin{align*} \frac{1}{57(x-1)^2}<\left|\frac{\log 3}{\log 2}-\frac{y-2x}{x-1}\right|<\frac{13/2^{0.22k}+2(2^x-1)/(2^{2x})}{x-1}. \end{align*} Therefore, we have \begin{align*} \frac{1}{57(x-1)}<\frac{13}{2^{0.22k}}+\frac{2^x-1}{2^{2x-1}}. \end{align*} The left--hand side is at least $1/(57\cdot 10^{89})$, while $13/2^{0.22k}<10^{-50}$ since $k>800$. So, we get $$ \frac{1}{58 (x-1)}<\frac{2^x-1}{2^{2x-1}}, $$ which gives $x\le 10$, as before in Subsection \ref{sub322}. Thus, \begin{equation} \label{eq:41} |(x-1)\log 3-(y-2x)\log 2|<\frac{9}{2^{0.22k}}+\frac{2^x-1}{2^{2x}}. \end{equation} Again, we used a simple code to compute the minimum values of the expression \[ |(x-1)\log 3 - (y-2x)\log 2|, \] for integer values of \( y - 2x \). As before, by assigning values to \( x \) ranging from 2 to 10, the code found the \( y \) that minimized the expression for each \( x \). The results still showed that the left--hand side of \eqref{eq:41} was at least \[ 0.28,~0.11,~0.16,~0.23,~0.05,~0.33,~0.06,~0.22,~0.18, \] for \( x = 2, 3, \ldots, 10 \), respectively. Again, it turned out that for no $x\in \{2,3,\ldots,10\}$, inequality \eqref{eq:41} is satisfied. This finishes the argument that the case $k>800$ is not possible here. \subsubsection{The case $k\le800$} To proceed, let us assume that $n > 800$, so, we can use Lemma \ref{lem3.4} to obtain upper bounds on $n$, $x$, and $m$. We assume that $x \leq 10$. Using inequality \eqref{g5}, we write $|\Lambda_5|:=|\log (\Gamma_5+1)|$ as \begin{align*} |\Lambda_5|&:= \left|(x-1)\log f_k(\alpha)+(x-1)\log(2\alpha-1)+\left((n-1)x-(m-1)\right)\log\alpha+\log\left(1+\alpha^{-x}-\alpha^{-2x}\right)\right|\\ &<\dfrac{4.5}{\alpha^{0.97n-5}}, \end{align*} where $a_1 := x-1$, $a_2 := x-1$, $a_3 := (n-1)x-(m-1)$, $a_4 := 1$ are integers with \[ \max\{|a_i| : 1 \leq i \leq 4\} < m<5.6\cdot 10^{44}k^{10}(\log k)^{12}<4.8\cdot 10^{83}, \] where we used Lemma \ref{lem3.4}, for all $k\le 800$. For each $k \in [2, 800]$ and $x\in [2,10]$, we used the LLL--algorithm to compute a lower bound for the smallest nonzero number of the form $|\Lambda_5|$, with integer coefficients $a_i$ not exceeding $5.6\cdot 10^{44}k^{10}(\log k)^{12}$ in absolute value. Specifically, we consider the approximation lattice $$ \mathcal{A}=\begin{pmatrix} 1 & 0 & 0 & 0 \\ 0 & 1 & 0 & 0 \\ 0 & 0 & 1 & 0 \\ \lfloor C\log f_k(\alpha)\rfloor & \lfloor C\log (2\alpha-1) \rfloor& \lfloor C\log \alpha \rfloor& \lfloor C\log \left(1+\alpha^{-x}-\alpha^{-2x}\right) \rfloor \end{pmatrix} ,$$ with $C:= 10^{335}$ and choose $y:=\left(0,0,0,0\right)$. Now, by Lemma \ref{lem2.5}, we get $$c_2:=10^{-85}\qquad\text{and}\qquad l\left(\mathcal{L},y\right)\ge c_1:=1.56\cdot10^{84}.$$ So, Lemma \ref{lem2.6} gives $S=9.3\cdot 10^{167}$ and $T=9.6\cdot 10^{83}$. Since $c_1^2\ge T^2+S$, then choosing $c_3:=4.5$ and $c_4:=\log\alpha$, we get $0.97n-5\le 1235$, or $n\le 1278$. Now, we work with $x>10$. We go to inequality \eqref{g4} and write $|\Lambda_4|:=|\log (\Gamma_4+1)|$ as \begin{align*} |\Lambda_4|:= \left|\left(m-1-(n-1)x\right)\log\alpha-(x-1)\log \left((2\alpha-1)f_k(\alpha)\right)\right|<\dfrac{40.5}{\alpha^{\min\{0.7n,x\}}}. \end{align*} where we have used the fact that $\min\{0.7n,x\} \geq 10$. Dividing both sides of the above inequality by $(x - 1) \log \alpha$, we obtain \begin{align}\label{eqr2} \left| \frac{\log \left((2\alpha-1)f_k(\alpha)\right)}{\log \alpha} - \frac{m-1-(n-1)x}{x - 1} \right| < \frac{40.5}{\alpha^{\min\{0.7n,x\}}(x - 1) \log \alpha} < \frac{10}{\alpha^{\min\{0.7n,x\}}}, \end{align} for all $x>10$. We proceed by examining two cases from \eqref{eqr2}. \textbf{Case I}: If $m-1 = (n-1)x$, then inequality \eqref{eqr2} becomes \[ \left| \frac{\log \left((2\alpha-1)f_k(\alpha)\right)}{\log \alpha} \right| < \frac{10}{\alpha^{\min\{0.7n,x\}}}. \] We get \begin{align*} \frac{10}{\alpha^{\min\{0.7n,x\}}}> \frac{\log \left((2\alpha-1)f_k(\alpha)\right)}{\log \alpha}>\frac{\log (2.2\cdot 0.5)}{\log 2}>0.1 \end{align*} for all $k \in [2, 800]$. Thus, $\alpha^{\min\{0.7n,x\}}<100$, or equivalently, $\min\{0.7n,x\}<10$. This contradicts our assumptions that $n > 800$ and $x > 10$. So this case is not possible. \textbf{Case II}: If $m-1 \ne (n-1)x$, we have \begin{align*} |\Lambda_4|:= \left|(x-1)\log f_k(\alpha)+(x-1)\log(2\alpha-1)+\left((n-1)x-(m-1)\right)\log\alpha\right|<\dfrac{40.5}{\alpha^{\min\{0.7n,x\}}}, \end{align*} where $a_1 := x-1$, $a_2 := x-1$, $a_3 := (n-1)x-(m-1)$ are integers with \[ \max\{|a_i| : 1 \leq i \leq 3\} < m<5.6\cdot 10^{44}k^{10}(\log k)^{12}<4.8\cdot 10^{83}, \] For each $k \in [2, 800]$, we again used the LLL--algorithm to compute a lower bound for the smallest nonzero number of the form $|\Lambda_4|$, with integer coefficients $a_i$ not exceeding $5.6\cdot 10^{44}k^{10}(\log k)^{12}$ in absolute value. We consider the approximation lattice $$ \mathcal{A}=\begin{pmatrix} 1 & 0 & 0 \\ 0 & 1 & 0 \\ \lfloor C\log f_k(\alpha)\rfloor & \lfloor C\log (2\alpha-1) \rfloor& \lfloor C\log \alpha \rfloor \end{pmatrix} ,$$ with $C:= 10^{252}$ and choose $y:=\left(0,0,0\right)$. So, by Lemma \ref{lem2.5}, we get $$c_2=10^{-87}\qquad\text{and}\qquad l\left(\mathcal{L},y\right)\ge c_1:=1.2\cdot10^{85}.$$ So, Lemma \ref{lem2.6} gives $S=7.0\cdot 10^{167}$ and $T=7.2\cdot 10^{83}$. Since $c_1^2\ge T^2+S$, then choosing $c_3:=40.5$ and $c_4:=\log\alpha$, we get $\min\{0.7n,x\}\le 825$. This implies that $n\le 1178$ or $x\le 825$. Let us first assume that $n\le 1178$ holds, that is $\min\{0.7n,x\}=0.7n$. We go to inequality \eqref{g3} and write $|\Lambda_3|:=|\log (\Gamma_3+1)|$ as \begin{align*} |\Lambda_3|:=\left|\log f_k(\alpha)+\log (2\alpha-1)+(m-1)\log\alpha-x\log\left(L_{n+1}^{(k)}\right)\right|<\dfrac{1.5}{1.4^x}, \end{align*} where $a_1 := 1$, $a_2 := 1$, $a_3 := m-1$, $a_4 := -x$ are integers with \[ \max\{|a_i| : 1 \leq i \leq 4\} < m<5.6\cdot 10^{44}k^{10}(\log k)^{12}<4.8\cdot 10^{83}, \] where we used Lemma \ref{lem3.4}, for all $k\le 800$. For each $k \in [2, 800]$ and $n\in [801,1178]$, we used the LLL--algorithm to compute a lower bound for the smallest nonzero number of the form $|\Lambda_3|$, with integer coefficients $a_i$ not exceeding $5.6\cdot 10^{44}k^{10}(\log k)^{12}$ in absolute value. Specifically, we consider the approximation lattice $$ \mathcal{A}=\begin{pmatrix} 1 & 0 & 0 & 0 \\ 0 & 1 & 0 & 0 \\ 0 & 0 & 1 & 0 \\ \lfloor C\log f_k(\alpha)\rfloor & \lfloor C\log (2\alpha-1) \rfloor& \lfloor C\log \alpha \rfloor& \left\lfloor C\log \left(L_{n+1}^{(k)}\right) \right\rfloor \end{pmatrix} ,$$ with $C:= 10^{335}$, $y:=\left(0,0,0,0\right)$, $c_2:=10^{-85}$ and $l\left(\mathcal{L},y\right)\ge c_1:=1.56\cdot10^{84}$. As before with $\Lambda_5$, Lemma \ref{lem2.6} gives $S=9.3\cdot 10^{167}$ and $T=9.6\cdot 10^{83}$. Choosing $c_3:=1.5$ and $c_4:=\log 1.4$, we get $x\le 1722$. Next, if $x\le 825$ holds, that is $\min\{0.7n,x\}=x$, we go back to $|\Lambda_5|$ and apply LLL--algorithm. We obtain a similar conclusion as before that $n\le 1278$. To sum up, we have that the integer solutions $n$, $m$, $k$, $x$ to \eqref{eq:main} with $n > k \ge 2$, $k \le 800$, $n > 800$ and $x \ge 2$, satisfy $n \le 1278$ and $x \le 1722$. Finally, we consider the case when $n \le 800$ and since we are working with $k \le 800$ and $n > k$, we get $k \in [2,799]$. Now, we use $\Lambda_3$ as defined before. We apply the LLL-- algorithm with \[ \max\{|a_i| : 1 \leq i \leq 4\} < m<(n+3)x+3<803\cdot 2.6\cdot 10^{15} n k^4(\log k)^3\log n+3<1.4\cdot 10^{36}, \] where we have used Lemma \ref{lem3.3}. We consider the same approximation lattice as for $\Lambda_3$, but with $C:= 10^{145}$. Choosing $y:=\left(0,0,0,0\right)$, we get $c_2:=10^{-39}$ and $l\left(\mathcal{L},y\right)\ge c_1:=4.0\cdot10^{37}$. Lemma \ref{lem2.6} gives $S=7.84\cdot 10^{72}$ and $T=2.8\cdot 10^{36}$. Choosing $c_3:=1.5$ and $c_4:=\log 1.4$, we get $x\le 736$. To conclude the case $n>k$, we look for solutions to \eqref{eq:main} in the ranges $k\in [2, 800]$, $n \in [k + 1, 800]$ and $x \in [2, 736]$ or $n \in [801, 1278]$ and $x \in [2, 1722]$, all with $nx-3 <m<(n+3)x+3$. A computational search in SageMath 9.5 allows us to conclude that there are no other integral solutions to \eqref{eq:main} apart from those given in Theorem \ref{thm1.1}. To efficiently handle large \( L_n^{(k)} \) values, our SageMath 9.5 code utilized batch processing, iterating through all \( (n, m,k, x) \) combinations within these ranges. \qed \section*{Acknowledgments} We thank the school of mathematics at Stellenbosch university for providing the necessary resources and support during this research. Additionally, we are grateful for the use of virtual SageMath 9.5 libraries that made computations in this work possible. \begin{thebibliography}{99} \bibitem{Brl} Bravo, J. J., \& Luca, F. (2013). On the largest prime factor of the $k$--Fibonacci numbers, {\it International Journal of Number Theory\/} {\bf 9}, 1351--1366. \bibitem{BRL} Bravo, J. J., \& Luca, F. (2014). Repdigits in $k$--Lucas sequences. {\it Proceedings--Mathematical Sciences\/} {\bf 124}, 141--154. \bibitem{Car} Carmichael, R. D. (1913). On the numerical factors of the arithmetic forms $\alpha^n\pm \beta^n$, Ann. Math. (2) {\bf 15} (1913), 30--70. \bibitem{Cas} Cassels, J. W. S. An introduction to the geometry of numbers. Springer Science \& Business Media, 2012. \bibitem{Weg} de Weger, B. M. (1987). Solving exponential Diophantine equations using lattice basis reduction algorithms, {\it Journal of Number Theory\/} {\bf 26}, 325--367. \bibitem{Fay} Faye, B., Garca, J., \& Gomez, C. A. (2024). $k$--Generalized Lucas numbers, perfect powers and the problem of Pillai. \textit{Monatshefte fr Mathematik}, 1--47. \bibitem{GGL1} Gmez, C. A., Gmez, J. C., \& Luca, F. (2020). Multiplicative dependence between $k-$Fibonacci and $k-$Lucas numbers. Periodica Mathematica Hungarica, {\bf 81}(2), 217--233. \bibitem{GGL} Gmez, C. A., Gmez, J. C., \& Luca, F. (2024). A Diophantine Equation With Powers of Three Consecutive $k$--Fibonacci Numbers. \textit{Results in Mathematics}, 79(4), 136. \bibitem{GL} G{\'u}zman--Sanchez, S., \& Luca, F. (2014). Linear combinations of factorials and S-units in a binary recurrence sequence. {\it Annales Math{\'e}matiques du Qu{\'e}bec\/} {\bf 38}, 169--188. \bibitem{LMN} Laurent, M., Mignotte, M., \& Nesterenko, Y. (1995). Formes linaires en deux logarithmes et dterminants d'interpolation. Journal of number theory, 55(2), 285--321. \bibitem{LLL} Lenstra, A. K., Lenstra, H. W., \& Lovsz, L. (1982). Factoring polynomials with rational coefficients. {\it Mathematisches Annalen\/} {\bf 261}, 515--534. \bibitem{MAT} Matveev, E. M. (2000). An explicit lower bound for a homogeneous rational linear form in the logarithms of algebraic numbers. II. {\it Izvestiya: Mathematics\/} {\bf 64}, 1217. \bibitem{MIL} Miles, E. P. (1960). Generalized Fibonacci numbers and associated matrices. {\it The American Mathematical Monthly} {\bf 67}, 745--752. \bibitem{SMA} Smart, N. P. (1998). The algorithmic resolution of Diophantine equations: a computational cookbook (Vol. 41). Cambridge University Press. \bibitem{WOL} Wolfram, D. A. (1998). Solving generalized Fibonacci recurrences. {\it The Fibonacci Quarterly} {\bf 36}, 129--145. \end{thebibliography} \section*{Addresses} $ ^{1} $ Mathematics Division, Stellenbosch University, Stellenbosch, South Africa. Email: \url{[email protected]} \qquad\url{https://orcid.org/0000-0003-3882-0189} \vspace{0.3cm}\\ \noindent $ ^{2} $ Centro de Ciencias Matem\'aticas UNAM, Morelia, Mexico. Email: \url{[email protected]}\qquad \url{https://orcid.org/0000-0003-1321-4422} \end{document}
2412.04740v1
http://arxiv.org/abs/2412.04740v1
Estimates for the first eigenvalue of the one-dimensional $p$-Laplacian
\documentclass[a4paper,12pt]{article} \usepackage{latexsym} \usepackage{amssymb} \usepackage{amsthm} \usepackage{amsmath} \usepackage{color} \newtheorem{theorem}{Theorem}[section] \newtheorem{lemma}[theorem]{Lemma} \newtheorem{proposition}[theorem]{Proposition} \newtheorem{corollary}[theorem]{Corollary} \newtheorem*{conjecture}{Conjecture} \theoremstyle{definition} \newtheorem{definition}[theorem]{Definition} \newtheorem{example}[theorem]{Example} \newtheorem{remark}[theorem]{Remark} \newtheorem{assumption}[theorem]{Assumption} \newtheorem{problem}[theorem]{Problem} \renewcommand{\theequation}{{\rm \thesection.\arabic{equation}}} \renewcommand{\labelenumi}{\rm (\roman{enumi})} \begin{document} \title{Estimates for the first eigenvalue of the one-dimensional $p$-Laplacian} \author{ Ryuji \ Kajikiya${}^{*1}$ and \ Shingo \ Takeuchi${}^{**2}$ \\[1ex] {\small\itshape ${}^*$ Center for Physics and Mathematics,}\\ {\small\itshape Osaka Electro-Communication University,}\\ {\small\itshape Neyagawa, Osaka 572-8530, Japan} \\ {\small\upshape E-mail: [email protected]} \\[1ex] {\small\itshape ${}^{**}$ Department of Mathematical Sciences,}\\ {\small\itshape Shibaura Institute of Technology,}\\ {\small\itshape 307 Fukasaku, Minuma-ku, Saitama-shi,}\\ {\small\itshape Saitama 337-8570, Japan} \\ {\small\upshape E-mail: [email protected]} } \footnotetext[1]{The first author was supported by JSPS KAKENHI Grant Number 20K03686.} \footnotetext[2]{The second author was supported by JSPS KAKENHI Grant Number 22K03392.} \date{} \maketitle \begin{abstract} In the present paper, we study the first eigenvalue $\lambda(p)$ of the one-dimensional $p$-Laplacian in the interval $(-1,1)$. We give an upper and lower estimate of $\lambda(p)$ and study its asymptotic behavior as $p\to 1+0$ or $p\to\infty$. \end{abstract} {\itshape Key words and phrases.} $p$-Laplacian, first eigenvalue, estimate. \newline 2020 {\itshape Mathematical Subject Classification.} 34B09, 34L30, 26D05, 33B10. \section{Introduction and main result}\label{section-1} \setcounter{equation}{0} We study the first eigenvalue $\lambda(p)$ of the one-dimensional $p$-Laplacian, \begin{equation}\label{eq:1.1} (|u'|^{p-2}u')' + \lambda(p)|u|^{p-2}u =0 \quad \mbox{in } (-1,1), \quad u(-1)=u(1)=0, \end{equation} where $1<p<\infty$. Then $\lambda(p)$ is represented as \begin{equation}\label{eq:1.2} \lambda(p)=(p-1)\left(\dfrac{\pi}{p\sin(\pi/p)} \right)^p. \end{equation} For the proof of \eqref{eq:1.2}, we refer the readers to \cite{DM, KTT} or \cite[pp.4--5]{DR}. In the problem \eqref{eq:1.1}, if the interval $(-1,1)$ is replaced by $(-L,L)$ with $L>0$, then the first eigenvalue is written as $\lambda(p,L)=\lambda(p)/L^p$. Kajikiya, Tanaka and Tanaka~\cite{KTT} proved the next theorem. \begin{theorem}[\cite{KTT}]\label{th:1.1} \begin{enumerate} \item If $0<L\leq 1$, $\lambda_p(p,L)>0$ for $1<p<\infty$, where $\lambda_p(p,L)$ denotes the partial derivative with respect to $p$. Therefore $\lambda(p,L)$ is strictly increasing with respect to $p$. Moreover, $\lambda(p,L)$ diverges to infinity as $p\to\infty$. \item If $L>1$, then there exists a unique $p_*(L)>0$ such that $\lambda_p(p,L)>0$ for $p\in (1,p_*(L))$ and $\lambda_p(p,L)<0$ for $p\in(p_*(L),\infty)$ and $\lambda(p,L)$ converges to zero as $p \to\infty$. \end{enumerate} \end{theorem} The theorem above gives an information on the monotonicity or non-monotonicity of the eigenvalue. In the present paper, we concentrate on $\lambda(p)$ because the properties of $\lambda(p,L)$ follow from those of $\lambda(p)$ by the relation $\lambda(p,L)=\lambda(p)/L^p$. The eigenvalue $\lambda(p)$ in \eqref{eq:1.2} seems complicated and difficult to understand. Therefore we shall give a simple and easy estimate for it. This is our purpose of the present paper. Our another interest is to investigate how $\lambda(p)$ and its derivative behave as $p\to 1+0$ or $p \to \infty$. In the present paper, we give the estimate and the asymptotic behavior of the first eigenvalue $\lambda(p)$. Our main result is as follows. \begin{theorem}\label{th:1.2} The first eigenvalue $\lambda(p)$ is estimated as \begin{equation}\label{eq:1.3} p<\lambda(p)<p+\frac{\pi^2}{6}-1 \quad \mbox{for } 2\leq p<\infty, \end{equation} \begin{equation}\label{eq:1.4} \left(\frac{p}{p-1}\right)^{p-1}<\lambda(p)<(p-1)^{1-p}\left(1+\frac{\pi^2}{6}(p-1)\right)^{p-1} \quad \mbox{for } 1<p<2. \end{equation} \end{theorem} In the theorem above, we give the lower and upper estimates of $\lambda(p)$. These terms satisfy the following inequalities. \begin{lemma}\label{le:1.3} For $1<p<2$, it holds that \begin{equation}\label{eq:1.5} p<\left(\frac{p}{p-1}\right)^{p-1}, \end{equation} \begin{equation}\label{eq:1.6} (p-1)^{1-p}\left(1+\frac{\pi^2}{6}(p-1)\right)^{p-1} <p+\frac{\pi^2}{6}-1. \end{equation} \end{lemma} Observing Theorem \ref{th:1.2} and Lemma \ref{le:1.3}, we have the next result, which is an easy and simple estimate for $\lambda(p)$. \begin{corollary}\label{co:1.4} The first eigenvalue $\lambda(p)$ satisfies \eqref{eq:1.3} for all $1<p<\infty$. \end{corollary} We shall show that $\lambda(p)$ is analytic for $p \in (1,\infty)$. We put $$ p:=\pi/x, \quad y:=\left(\dfrac{\pi}{p\sin(\pi/p)} \right)^p= \left(\dfrac{x}{\sin x} \right)^{\pi/x}. $$ Then $\lambda(p)=(p-1)y$. We compute $\log y$ as $$ \log y=-\frac{\pi}{x}\log\left(\frac{\sin x}{x}\right). $$ Since $\sin x/x$ is positive and analytic in $(0,\pi)$, the function $\log y$ is analytic with respect to $x \in (0,\pi)$, and so is $y=e^{\log y}$. Accordingly, $y$ (hence $\lambda(p)$) is analytic with respect to $p$ because $p=\pi/x$. We observe that $$ \log \lambda(p)=\log\left(\frac{\pi-x}{x}\right) -\frac{\pi}{x}\log\left(\frac{\sin x}{x}\right). $$ The function above is not well defined at $x=0$. However, we shall show that $\lambda(p)-p=\lambda(\pi/x)-\pi/x$ is analytic for $x\in (-\pi,\pi)$. Moreover we shall give its Maclaurin expansion, from which we derive the behavior of $\lambda(p)$ near $p=\infty$. \begin{theorem}\label{th:1.5} The function $\lambda(\pi/x)-\pi/x$ is analytic in $(-\pi,\pi)$ and its Maclaurin series is written as \begin{equation}\label{eq:1.7} \lambda\left(\frac{\pi}{x}\right)-\frac{\pi}{x} = \frac{\pi^2}{6}-1 +\left(\frac{\pi^3}{72}-\frac{\pi}{6}\right)x +\left(\frac{\pi^4}{1296}-\frac{\pi^2}{120}\right)x^2+\cdots. \end{equation} The expansion above is rewritten as, for $1<p<\infty$, \begin{equation}\label{eq:1.8} \lambda(p) = p+ \frac{\pi^2}{6}-1 +\left(\frac{\pi^3}{72}-\frac{\pi}{6}\right)\frac{\pi}{p} +\left(\frac{\pi^4}{1296}-\frac{\pi^2}{120}\right)\left(\frac{\pi}{p}\right)^2 +\cdots. \end{equation} \end{theorem} Denote the derivative of the first eigenvalue $\lambda(p)$ by $\lambda'(p)$. We shall compute their limits as $p\to 1+0$ or $p\to \infty$ by using the theorem above. \begin{theorem}\label{th:1.6} The first eigenvalue $\lambda(p)$ and its derivative $\lambda'(p)$ satisfy \begin{equation}\label{eq:1.9} \lim_{p\to 1+0}\lambda(p)=1, \quad \lim_{p\to 1+0}\lambda'(p)=\infty, \end{equation} \begin{equation}\label{eq:1.10} \lim_{p\to\infty}(\lambda(p)-p)=\frac{\pi^2}{6}-1, \quad \lim_{p\to\infty}\lambda'(p)=1. \end{equation} \end{theorem} As a byproduct of \eqref{eq:1.3}, we immediately obtain the following inequality for the sinc function $\sin{(\pi x)}/(\pi x)$. This inequality may already be known, but at least we could not find any literature with its proof. \begin{corollary}\label{co:1.7} It holds that $$ \left(\frac{1-x}{1+((\pi^2/6)-1)x}\right)^x <\frac{\sin{(\pi x)}}{\pi x} <(1-x)^x \quad \mbox{for } 0<x<1. $$ \end{corollary} \section{Proof of the theorem} \setcounter{equation}{0} We shall prove Theorem \ref{th:1.2}. We first prove the next proposition, the lower estimate of $\lambda(p)$ for $2\leq p<\infty$. \begin{proposition}\label{pr:2.1} It holds that \begin{equation}\label{eq:2.1} p<(p-1)\left(\dfrac{\pi}{p\sin(\pi/p)} \right)^p \quad \mbox{for } 2 \leq p<\infty. \end{equation} \end{proposition} \begin{proof} We shall use the inequality below, which is proved by Zhu~\cite{L}, $$ \dfrac{\sin x}{x}\leq \dfrac{2}{\pi}+\dfrac{\pi-2}{\pi^3}(\pi^2 - 4x^2) \quad \mbox{for } 0<x\leq \pi/2. $$ Let $p\geq 2$. Substituting $x=\pi/p$ in the inequality above, we have $$ \dfrac{\sin (\pi/p)}{\pi/p}\leq \dfrac{2}{\pi}+\dfrac{\pi-2}{\pi^3}(\pi^2 - 4\pi^2/p^2) =1-\dfrac{4(\pi-2)}{\pi p^2}. $$ Put $A:=4(\pi-2)/(\pi p^2)$. We note that $0<A<1$ because $p\geq 2$. We rewrite the inequality above as $$ \dfrac{\pi}{p\sin(\pi/p)} \geq (1-A)^{-1}. $$ It is enough to prove the inequality below, \begin{equation}\label{eq:2.2} p<(p-1)(1-A)^{-p}. \end{equation} Indeed, combining the two inequalities above, we obtain \eqref{eq:2.1}. Taking the logarithm of \eqref{eq:2.2}, we have \begin{equation}\label{eq:2.3} \log p < \log(p-1) - p\log (1-A). \end{equation} We shall show the inequality above. The Maclaurin series shows that $$ \log(1-x)=-x-\dfrac{x^2}{2}-\dfrac{x^3}{3}-\cdots < -x \quad \mbox{for } 0<x<1. $$ Substituting $x=A$ in the inequality above, we obtain $$ -\log(1-A) > A. $$ Therefore, to show \eqref{eq:2.3}, we have only to prove that $$ \log p < \log(p-1) + pA, $$ i.e., \begin{equation}\label{eq:2.4} \log p < \log(p-1) + \dfrac{4(\pi-2)}{\pi p}. \end{equation} We define $$ f(p):=\log(p-1)- \log p + \dfrac{4(\pi-2)}{\pi p}. $$ We shall show that $f(p)>0$ for $p\geq2$. Differentiating it, we obtain $$ f'(p)=\dfrac{1}{p-1} - \dfrac{1}{p} - \dfrac{4(\pi-2)}{\pi p^2}, $$ i.e., $$ \pi p^2(p-1)f'(p)=- (3\pi - 8)p + 4(\pi-2). $$ Thus $f(p)$ achieves its maximum at $p^*:=4(\pi-2)/(3\pi-8)$. Observe that $p^*>2$. Hence $f(p)$ is increasing for $p\in(2,p^*)$ and decreasing for $p\in(p^*,\infty)$. Moreover, we have $$ f(2)=\dfrac{2(\pi-2)}{\pi} -\log 2=0.033613\cdots >0, \quad \lim_{p\to\infty}f(p)=0, $$ which shows that $f(p)>0$ for $p\in[2,\infty)$. Therefore \eqref{eq:2.4} holds for $p\in[2,\infty)$. The proof is complete. \end{proof} We next prove the upper estimate of the eigenvalue $\lambda(p)$ for $p\geq 2$, \begin{equation}\label{eq:2.5} \lambda(p)=(p-1)\left(\dfrac{\pi}{p\sin(\pi/p)}\right)^p < p+\dfrac{\pi^2}{6}-1. \end{equation} Taking the logarithm of \eqref{eq:2.5}, we have $$ \log(p-1)+p\log(\pi/(p\sin (\pi/p)))<\log(p+(\pi^2/6)-1). $$ Putting \begin{equation}\label{eq:2.6} p:=\pi/x, \quad a:=(\pi^2/6)-1, \end{equation} we obtain $$ \log((\pi/x)-1)+(\pi/x)\log(x/\sin x) <\log((\pi/x)+a), $$ which is rewritten as \begin{equation}\label{eq:2.7} x\log\left(\dfrac{\pi+ax}{\pi-x}\right) + \pi\log\left(\dfrac{\sin x}{x}\right)>0. \end{equation} Note that \eqref{eq:2.5} is equivalent to \eqref{eq:2.7}. To prove \eqref{eq:2.7}, we estimate its first term from below in the next lemma. \begin{lemma}\label{le:2.2} Let $a$ be defined by \eqref{eq:2.6}. For $0<x<\pi$, it holds that $$ \log\left(\dfrac{\pi+ax}{\pi-x}\right) >\dfrac{\pi}{6}x + \dfrac{1}{72}(12-\pi^2)x^2 +\dfrac{108-18\pi^2+\pi^4}{648\pi}x^3. $$ \end{lemma} \begin{proof} The right hand side is the first three terms in the Maclaurin series of the left hand side. We put $$ g(x):=\log\left(\dfrac{\pi+ax}{\pi-x}\right) -\dfrac{\pi}{6}x - \dfrac{1}{72}(12-\pi^2)x^2 -\dfrac{108-18\pi^2+\pi^4}{648\pi}x^3. $$ We shall show that $g(x)>0$ in $(0,\pi)$. The derivative of $g(x)$ is computed as $$ g'(x) =\dfrac{(a+1)\pi}{(\pi+ax)(\pi-x)} -\dfrac{\pi}{6}-\dfrac{1}{36}(12-\pi^2)x -\dfrac{108-18\pi^2+\pi^4}{216\pi}x^2. $$ Putting $G(x):=6(\pi+ax)(\pi-x)g'(x)$ and using $a=(\pi^2/6)-1$, we obtain \begin{align*} G(x) & =\dfrac{1}{216}(864-216\pi^2+24\pi^4-\pi^6)x^3 \\ & \quad + \dfrac{1}{216\pi}(-648+216\pi^2-24\pi^4+\pi^6)x^4. \end{align*} We compute that \begin{align*} & 864-216\pi^2+24\pi^4-\pi^6=108.594\cdots>0, \\ & -648+216\pi^2-24\pi^4+\pi^6 =107.405\cdots>0. \end{align*} Hence $G(x)>0$ for $x>0$. Thus $g'(x)>0$ for $0<x<\pi$ and therefore $g(x)$ is increasing in $(0,\pi)$. Since $g(0)=0$, $g(x)$ is positive for $0<x<\pi$. The proof is complete. \end{proof} The first term in \eqref{eq:2.7} is estimated from below by Lemma~\ref{le:2.2}. We shall evaluate the second term in the next lemma. \begin{lemma}\label{le:2.3} For $0<x\leq \pi/2$, it holds that $$ \log\left(\dfrac{\sin x}{x}\right) > -\dfrac{1}{6}x^2 -\dfrac{1}{180}x^4 -\dfrac{17}{15120}x^6 -\frac{41}{604800}x^8. $$ \end{lemma} \begin{proof} We first show that \begin{equation}\label{eq:2.8} \log(1-t)+t+\dfrac{1}{2}t^2+ \dfrac{1}{2}t^3>0 \quad \mbox{for } 0<t \leq 2/5. \end{equation} We denote the left hand side by $h(t)$, that is, $$ h(t):=\log(1-t)+t+\dfrac{1}{2}t^2+ \dfrac{1}{2}t^3. $$ Differentiating it, we obtain $$ h'(t)=-\frac{1}{1-t}+1+t+ \frac{3}{2}t^2 =\frac{t^2(1-3t)}{2(1-t)}. $$ Therefore $h(t)$ is increasing in $(0,1/3)$ and decreasing in $(1/3,1)$. Since $h(0)=0$, it is positive in $(0,1/3]$. For $1/3<t<2/5$, we get $$ h(t)> h(2/5)=\log(3/5)+\frac{64}{125}=0.0011743\cdots>0. $$ Thus $h(t)$ is positive for $0<t\leq 2/5$, i.e., \eqref{eq:2.8} holds. We rewrite \eqref{eq:2.8} as \begin{equation}\label{eq:2.9} \log(1-t)>-t-\dfrac{1}{2}t^2- \dfrac{1}{2}t^3 \quad \mbox{for } 0<t \leq 2/5. \end{equation} Put $k(x):=(x-\sin x)/x$. Then it is increasing for $x\in(0,\pi)$. We have $$ k(x)\leq k(\pi/2)=\frac{\pi-2}{\pi}=0.36338\cdots<2/5 \quad \mbox{for } 0<x\leq \pi/2. $$ Accordingly, we obtain \begin{equation}\label{eq:2.10} 0<\frac{x-\sin x}{x}<\frac{2}{5} \quad \mbox{if } 0<x\leq \frac{\pi}{2}. \end{equation} Computing the derivatives, we easily prove that $$ \sin x >x -\dfrac{1}{6} x^3 \quad \mbox{for } x>0, $$ $$ \sin x >x -\dfrac{1}{3!} x^3 + \frac{1}{5!}x^5 - \frac{1}{7!}x^7 \quad \mbox{for } x>0. $$ From the inequalities above, it follows that \begin{equation}\label{eq:2.11} 0<\dfrac{x-\sin x}{x}< \frac{1}{6}x^2 \quad \mbox{for } x>0, \end{equation} \begin{equation}\label{eq:2.12} 0<\dfrac{x-\sin x}{x}< \frac{1}{6}x^2 - \frac{1}{5!}x^4 + \frac{1}{7!}x^6 \quad \mbox{for } x>0. \end{equation} By \eqref{eq:2.12}, we get \begin{align}\label{eq:2.13} -\frac{1}{2}\left(\dfrac{x-\sin x}{x}\right)^2 & >-\frac{1}{2}\left(\frac{1}{6}x^2 - \frac{1}{5!}x^4 + \frac{1}{7!}x^6\right)^2 \nonumber \\ & =-\frac{1}{72}x^4-\frac{1}{2}\left(\frac{1}{5!}\right)^2x^8-\frac{1}{2}\left(\frac{1}{7!}\right)^2x^{12} \nonumber \\ &\quad +\frac{1}{6\cdot 5!}x^6+\frac{1}{5!\cdot 7!}x^{10}-\frac{1}{6\cdot 7!}x^8. \end{align} Observe that \begin{equation}\label{eq:2.14} -\frac{1}{2}\left(\frac{1}{7!}\right)^2x^{12}+ \frac{1}{5!\cdot 7!}x^{10}>0 \quad \mbox{for } 0<x<\pi/2. \end{equation} By \eqref{eq:2.13} and \eqref{eq:2.14}, we obtain \begin{equation}\label{eq:2.15} -\frac{1}{2}\left(\dfrac{x-\sin x}{x}\right)^2 >-\frac{1}{72}x^4+\frac{1}{720}x^6 -\frac{41}{604800}x^8. \end{equation} It follows from \eqref{eq:2.11} that \begin{equation}\label{eq:2.16} -\frac{1}{2}\left(\dfrac{x-\sin x}{x}\right)^3>-\frac{1}{2}\left(\frac{1}{6}x^2\right)^3. \end{equation} By \eqref{eq:2.12}, \eqref{eq:2.15} and \eqref{eq:2.16}, we obtain \begin{align}\label{eq:2.17} & -\dfrac{x-\sin x}{x} -\frac{1}{2}\left(\dfrac{x-\sin x}{x}\right)^2 -\frac{1}{2}\left(\dfrac{x-\sin x}{x}\right)^3 \nonumber \\ & >-\frac{1}{6}x^2 + \frac{1}{5!}x^4 - \frac{1}{7!}x^6 -\frac{1}{72}x^4 + \frac{1}{720}x^6 -\frac{41}{604800}x^8 -\dfrac{1}{432}x^6 \nonumber \\ & = -\frac{1}{6}x^2 -\frac{1}{180}x^4 - \frac{17}{15120}x^6 -\frac{41}{604800}x^8. \end{align} Observe \eqref{eq:2.9} and \eqref{eq:2.10}. Put $t:=(x-\sin x)/x$. Since $\log(\sin x/x)=\log(1-t)$, we substitute $t=(x-\sin x)/x$ in \eqref{eq:2.9} and use \eqref{eq:2.17}. Then we obtain $$ \log(\sin x/x)=\log(1-t)> -\frac{1}{6}x^2 -\frac{1}{180}x^4 - \frac{17}{15120}x^6 -\frac{41}{604800}x^8. $$ The proof is complete. \end{proof} Combining Lemmas \ref{le:2.2} and \ref{le:2.3}, we shall prove \eqref{eq:2.7}, which is equivalent to \eqref{eq:2.5}. \begin{proposition}\label{pr:2.4} The inequality \eqref{eq:2.7} holds for $0<x\leq \pi/2$, that is, \eqref{eq:2.5} remains valid for $2\leq p <\infty$. \end{proposition} \begin{proof} By Lemmas \ref{le:2.2} and \ref{le:2.3}, we have for $0<x\leq \pi/2$, \begin{align*} & x\log\left(\dfrac{\pi+ax}{\pi-x}\right) + \pi\log\left(\dfrac{\sin x}{x}\right) \\ & >\dfrac{\pi}{6}x^2 + \dfrac{1}{72}(12-\pi^2)x^3 +\dfrac{108-18\pi^2+\pi^4}{648\pi}x^4 -\dfrac{\pi}{6}x^2 -\dfrac{\pi}{180}x^4 \\ & \quad -\dfrac{17\pi}{15120}x^6 - \frac{41\pi}{604800}x^8\\ & = \left[\dfrac{1}{72}(12-\pi^2) -\frac{108\pi^2-5\pi^4-540}{3240\pi}x -\dfrac{17\pi}{15120}x^3 - \frac{41\pi}{604800}x^5 \right]x^3. \end{align*} Here we note that $$ 108\pi^2-5\pi^4-540=38.8718\cdots>0. $$ Therefore $$ \phi(x):=\dfrac{1}{72}(12-\pi^2) -\frac{108\pi^2-5\pi^4-540}{3240\pi}x -\dfrac{17\pi}{15120}x^3 - \frac{41\pi}{604800}x^5, $$ is a decreasing function of $x$. For $0<x\leq \pi/2$, it holds that \begin{align*} \phi(x) & \geq \phi(\pi/2)=\dfrac{1}{72}(12-\pi^2) -\frac{108\pi^2-5\pi^4-540}{3240\pi}\frac{\pi}{2} -\dfrac{17\pi}{15120}(\pi/2)^3 \\ & \quad - \frac{41\pi}{604800}(\pi/2)^5 \\ & = \frac{1}{4} -\frac{11\pi^2}{360} + \frac{229\pi^4}{362880} - \frac{41\pi^6}{19353600} =0.0078633\cdots>0. \end{align*} Therefore, for $0<x \leq \pi/2$, we have $$ x\log\left(\dfrac{\pi+ax}{\pi-x}\right) + \pi\log\left(\dfrac{\sin x}{x}\right) >0. $$ \end{proof} Using Propositions \ref{pr:2.1} and \ref{pr:2.4}, we shall show Theorem \ref{th:1.2}. \begin{proof}[Proof of Theorem \ref{th:1.2}] In Propositions \ref{pr:2.1} and \ref{pr:2.4}, we have already proved \eqref{eq:1.3}. By using the duality $1/p+1/q=1$, we shall derive \eqref{eq:1.4} from \eqref{eq:1.3}. Let $q\in (1,2)$ be any number. Put $p:=q/(q-1)$. Then $2<p<\infty$. Substituting $p=q/(q-1)$ in the first inequality in \eqref{eq:1.3}, we have \begin{equation}\label{eq:2.18} \dfrac{q}{q-1}<\dfrac{1}{q-1}\left(\dfrac{(q-1)\pi}{q\sin((q-1)\pi/q)}\right)^{q/(q-1)}. \end{equation} Using the relation \begin{equation}\label{eq:2.19} \sin((q-1)\pi/q)=\sin(\pi - \pi/q)=\sin(\pi/q), \end{equation} we reduce \eqref{eq:2.18} to $$ q<\left(\dfrac{(q-1)\pi}{q\sin(\pi/q)}\right)^{q/(q-1)}. $$ or equivalently, $$ \left(\frac{q}{q-1}\right)^{q-1} < (q-1)\left(\dfrac{\pi}{q\sin(\pi/q)}\right)^q=\lambda(q). $$ This is the lower estimate of $\lambda(p)$ in \eqref{eq:1.4} with $p$ replaced by $q$. We shall show the upper estimate in \eqref{eq:1.4}. Let $q\in (1,2)$ be any number. Put $p:=q/(q-1)$. Then $p\in(2,\infty)$. Substituting $p=q/(q-1)$ in the second inequality in \eqref{eq:1.3}, we have $$ \dfrac{1}{q-1}\left(\frac{(q-1)\pi}{q\sin((q-1)\pi/q)}\right)^{q/(q-1)} < \frac{1}{q-1}+\frac{\pi^2}{6}. $$ By \eqref{eq:2.19}, the inequality above is rewritten as $$ (q-1) \left(\frac{\pi}{q\sin(\pi/q)}\right)^q < (q-1)^{1-q}\left(1+ \frac{\pi^2}{6}(q-1)\right)^{q-1}. $$ Consequently, we have the upper estimate of $\lambda(p)$ in \eqref{eq:1.4}. The proof is complete. \end{proof} \begin{proof}[Proof of Lemma \ref{le:1.3}] We rewrite \eqref{eq:1.5} as $$ (p-1)^{p-1}<p^{p-2} \quad \mbox{for } 1<p<2. $$ Taking the logarithm of the inequality above, we have $$ (p-1)\log(p-1)<(p-2)\log p. $$ We shall show the above inequality. Put $$ f(p):=(p-2)\log p -(p-1) \log(p-1). $$ It is enough to show that $f(p)$ is positive. Differentiating $f(p)$, we have $$ f'(p)=\log p -\log (p-1) - \frac{2}{p}, $$ $$ f''(p)=\frac{1}{p} - \frac{1}{p-1} +\frac{2}{p^2}=-\dfrac{2-p}{p^2(p-1)}<0 \quad \mbox{for } p\in(1,2). $$ Hence $f(p)$ is concave. Furthermore, we have $$ \lim_{p\to 1+0}f(p)=0, \quad f(2)=0, $$ which means that $f(p)>0$ for $1<p<2$. Accordingly, \eqref{eq:1.5} holds. We shall prove \eqref{eq:1.6}. Let $1<p<2$. Put $x:=p-1$. Since $1<p<2$, we have $0<x<1$. Then \eqref{eq:1.6} is rewritten as $$ x^{-x}(1+(\pi^2/6)x)^x < x+ \pi^2/6 \quad \mbox{for } 0<x<1. $$ Taking the logarithm of the inequality above, we have $$ -x\log x +x\log(1+(\pi^2/6)x)<\log(x+\pi^2/6), $$ or equivalently, \begin{equation}\label{eq:2.20} \log(x+\pi^2/6) + x\log x -x\log(1+(\pi^2/6)x)>0. \end{equation} Observe that \eqref{eq:2.20} is equivalent to \eqref{eq:1.6}. We define $$ g(x):=\log(x+\pi^2/6) + x\log x -x\log(1+(\pi^2/6)x). $$ We have only to prove that $g(x)$ is positive in $(0,1)$. The first and second derivatives of $g(x)$ are computed as $$ g'(x)=\dfrac{1}{x+(\pi^2/6)} + \log x +1 -\log(1+(\pi^2/6)x) -\frac{\pi^2 x}{\pi^2 x + 6}. $$ $$ g''(x)=-\frac{36}{(6x+\pi^2)^2} + \frac{1}{x} - \frac{\pi^2}{\pi^2 x + 6} -\frac{6\pi^2}{(\pi^2 x +6)^2}. $$ Then we have \begin{align*} & x(6x+\pi^2)^2(\pi^2 x +6)^2g''(x) \\ & = 36(-\pi^4x^3+(-12\pi^2+36)x^2+(12\pi^2-36)x+\pi^4) \\ & =36[\pi^4(1-x^3)+(12\pi^2-36)x(1-x)]>0. \end{align*} Consequently, $g''(x)>0$ for $0<x<1$, and therefore $g(x)$ is convex in $(0,1)$. Moreover, \begin{align*} g'(1) & =\frac{6}{\pi^2+6} +1 -\log(1+(\pi^2/6))-\frac{\pi^2}{\pi^2+6} \\ & =\frac{12}{\pi^2+6} -\log(1+(\pi^2/6))=-0.21648\cdots <0. \end{align*} Thus $g'(x)<0$ for $0<x<1$ and hence $g(x)$ is decreasing. Since $g(1)=0$, $g(x)$ is positive for $0<x<1$. The proof is complete. \end{proof} We shall prove Theorem \ref{th:1.5}, the analyticity of $\lambda(\pi/x)-\pi/x$. \begin{proof}[Proof of Theorem \ref{th:1.5}] Putting $p:=\pi/x$, we write $\lambda(p)-p$ as \begin{align}\label{eq:2.21} \lambda(p)-p & =p\left[\left(\frac{\pi}{p\sin(\pi/p)}\right)^p-1\right]- \left(\frac{\pi}{p\sin(\pi/p)}\right)^p \nonumber \\ & =\frac{\pi(y-1)}{x}- y, \end{align} where $y$ is defined by $$ y:=\left(\frac{\pi}{p\sin(\pi/p)}\right)^p=\left(\frac{x}{\sin x}\right)^{\pi/x}. $$ Then we have \begin{equation}\label{eq:2.22} \log y=-\frac{\pi}{x}\log(\sin x/x)=-\frac{\pi}{x}\log(1-(x-\sin x)/x). \end{equation} The function $\sin x/x$ is analytic in $\mathbb{R}$. More precisely, the point $x=0$ is a removable singularity. After defining $\sin x/x =1$ at $x=0$, it is analytic in $\mathbb{R}$. Since $0<\sin x/x\leq 1$ for $x \in (-\pi,\pi)$, \eqref{eq:2.22} shows that $\log y$ is analytic in $(-\pi,\pi)$, and so is $y=e^{\log y}$. We shall use the Maclaurin series: \begin{equation}\label{eq:2.23} \frac{x-\sin x}{x}=\frac{1}{3!}x^2-\frac{1}{5!}x^4+\frac{1}{7!}x^6 - \cdots, \end{equation} \begin{equation}\label{eq:2.24} \log(1-t)=-t-\frac{1}{2}t^2-\frac{1}{3}t^3+\cdots. \end{equation} Putting $t=(x-\sin x)/x$ and using \eqref{eq:2.23} with \eqref{eq:2.24}, we obtain \begin{align}\label{eq:2.25} & \log(1-(x-\sin x)/x) \nonumber \\ & =-\frac{x-\sin x}{x} -\frac{1}{2}\left(\frac{x-\sin x}{x}\right)^2 - \frac{1}{3}\left(\frac{x-\sin x}{x}\right)^3 - \cdots \nonumber \\ & =-\frac{1}{6}x^2 - \frac{1}{180}x^4 + \cdots. \end{align} Substituting \eqref{eq:2.25} in \eqref{eq:2.22}, we have \begin{align*} \log y & =-\frac{\pi}{x}\left(-\frac{1}{6}x^2 - \frac{1}{180}x^4 + \cdots \right) \\ & =\frac{\pi}{6}x + \frac{\pi}{180}x^3 + \cdots. \end{align*} Using the Maclaurin expansion of $e^t$ with the formula above, we have \begin{align}\label{eq:2.26} y & =\exp\left(\frac{\pi}{6}x + \frac{\pi}{180}x^3 + \cdots\right) \nonumber \\ & =1+ \left(\frac{\pi}{6}x + \frac{\pi}{180}x^3 + \cdots\right) + \frac{1}{2!}\left(\frac{\pi}{6}x + \frac{\pi}{180}x^3 + \cdots\right)^2 \nonumber \\ & \quad + \frac{1}{3!}\left(\frac{\pi}{6}x + \frac{\pi}{180}x^3 + \cdots\right)^3 +\cdots \nonumber \\ & =1+ \frac{\pi}{6}x + \frac{\pi^2}{72}x^2 + \left(\frac{\pi}{180} + \frac{\pi^3}{1296}\right)x^3+ \cdots. \end{align} Accordingly, $(\pi/x)(y-1)$ is analytic for $x\in (-\pi,\pi)$. Using \eqref{eq:2.26} in \eqref{eq:2.21}, we obtain \begin{align*} \lambda(p)-p & =\frac{\pi}{x}(y-1)-y \\ & = \frac{\pi^2}{6} + \frac{\pi^3}{72}x + \left(\frac{\pi^2}{180} + \frac{\pi^4}{1296}\right)x^2+\cdots \\ & \quad -\left[1+ \frac{\pi}{6}x + \frac{\pi^2}{72}x^2 + \left(\frac{\pi}{180} + \frac{\pi^3}{1296}\right)x^3 + \cdots\right] \\ & =\frac{\pi^2}{6}-1 + \left(\frac{\pi^3}{72}-\frac{\pi}{6}\right)x + \left(\frac{\pi^4}{1296}-\frac{\pi^2}{120}\right)x^2 +\cdots. \end{align*} Consequently, $\lambda(\pi/x)-\pi/x$ is analytic and \eqref{eq:1.7} holds true. The proof is complete. \end{proof} We investigate the limits of $\lambda(p)$ and $\lambda'(p)$ as $p\to 1$. \begin{proposition}\label{pr:2.5} The assertion \eqref{eq:1.9} holds. \end{proposition} \begin{proof} We shall show the first claim in \eqref{eq:1.9}. Put $p:=\pi/x$. Then $p\to 1+0$ if and only if $x\to \pi-0$. The eigenvalue $\lambda(p)$ is rewritten as \begin{align*} \lambda(p) & =(p-1)\left(\frac{\pi}{p\sin(\pi/p)}\right)^p =(p-1)^{1-p}\left(\frac{(p-1)\pi}{p\sin(\pi/p)}\right)^p \\ & =\left(\frac{\pi-x}{x}\right)^{(x-\pi)/x}\left(\frac{\pi-x}{\sin x}\right)^{\pi/x}. \end{align*} Then the first limit in \eqref{eq:1.9} follows from the facts that $$ \lim_{x\to \pi-0} \left(\frac{\pi-x}{x}\right)^{(x-\pi)/x} =\lim_{x\to\pi-0}\left(\frac{\pi-x}{\sin x}\right)^{\pi/x}=1. $$ To compute the limit of $\lambda'(p)$ as $p \to 1$, we shall prove the formula, \begin{equation}\label{eq:2.27} \lambda'(p)=\lambda(p) \left[\frac{x}{\pi-x} - \log(\sin x/x) + \frac{x\cos x-\sin x}{\sin x}\right], \end{equation} with $p:=\pi/x$. Taking the logarithm of $\lambda(p)$ in \eqref{eq:1.2}, we obtain $$ \log \lambda(p)=\log(p-1)+ p\log(\pi/(p\sin (\pi/p))). $$ Putting $p:=\pi/x$, we have \begin{equation}\label{eq:2.28} \log \lambda(p)=\log((\pi-x)/x) - \frac{\pi}{x}\log(\sin x/x). \end{equation} Differentiating $\log \lambda(p)$ with respect to $x$, we get $$ \frac{d \log \lambda(p)}{dx} = \frac{\lambda'(p)}{\lambda(p)}\frac{dp}{dx}=-\frac{\pi}{x^2}\frac{\lambda'(p)}{\lambda(p)}. $$ Differentiating both sides of \eqref{eq:2.28}, we have $$ -\frac{\pi}{x^2}\frac{\lambda'(p)}{\lambda(p)} =-\frac{\pi}{x(\pi-x)} + \frac{\pi}{x^2}\log(\sin x/x) -\frac{\pi(x\cos x-\sin x)}{x^2\sin x}. $$ Multiplying both sides by $-(x^2/\pi)\lambda(p)$, we obtain \eqref{eq:2.27}. Rewrite \eqref{eq:2.27} as \begin{equation}\label{eq:2.29} \lambda'(p)=\lambda(p) \left[ -\log(\sin x/x) + x\left(\frac{1}{\pi-x} + \frac{\cos x}{\sin x}\right)-1\right]. \end{equation} Note that $p\to 1+0$ is equivalent to $x\to \pi-0$. We shall prove that \begin{equation}\label{eq:2.30} \lim_{x\to\pi-0}\left(\frac{1}{\pi-x} + \frac{\cos x}{\sin x}\right)=0. \end{equation} We easily find that $$ \frac{1}{\pi-x} + \frac{\cos x}{\sin x}= \frac{\sin x + (\pi-x)\cos x}{(\pi-x)\sin x}. $$ Using L'Hospital's rule twice, we get \begin{align*} \lim_{x\to\pi-0}\frac{\sin x + (\pi-x)\cos x}{(\pi-x)\sin x} & =\lim_{x\to\pi-0}\frac{-(\pi-x)\sin x}{-\sin x + (\pi-x)\cos x} \\ & =\lim_{x\to\pi-0}\frac{\sin x - (\pi-x)\cos x}{-2\cos x - (\pi-x)\sin x}=0. \end{align*} Therefore we obtain \eqref{eq:2.30}. Observe that $\log(\sin x/x)$ diverges to $-\infty$ as $x\to \pi-0$. Using \eqref{eq:2.30} in \eqref{eq:2.29} and noting that $\lambda(p)$ converges to $1$ as $p\to 1+0$, we find that $\lambda'(p)\to \infty$ as $p\to 1+0$. Consequently, we obtain the second claim in \eqref{eq:1.9}. \end{proof} Using Proposition \ref{pr:2.5} and Theorem \ref{th:1.5}, we shall prove Theorem \ref{th:1.6}. \begin{proof}[Proof of Theorem \ref{th:1.6}] The assertion \eqref{eq:1.9} has already been proved in Proposition \ref{pr:2.5}. We shall show \eqref{eq:1.10}. Letting $p\to\infty$ in \eqref{eq:1.8}, we have the first limit in \eqref{eq:1.10}. Denote the right hand side in \eqref{eq:1.7} by $f(x)$. Then we have $$ \lambda(\pi/x) -\pi/x=f(x). $$ Differentiating it, we have $$ -\frac{\pi}{x^2}(\lambda'(\pi/x)-1)=f'(x), $$ which is rewritten as $$ \lambda'(\pi/x) -1 =-\frac{x^2}{\pi}f'(x). $$ Letting $x\to +0$ (or $p\to \infty$), we have $\lim_{p\to\infty}(\lambda'(p)-1)=0$. The proof is complete. \end{proof} We conclude the present paper by proving Corollary~\ref{co:1.7}. \begin{proof}[Proof of Corollary~\ref{co:1.7}] By \eqref{eq:1.2} and \eqref{eq:1.3}, we obtain $$ p<(p-1)\left(\frac{\pi}{p\sin(\pi/p)}\right)^p <p+a, \quad \mbox{with } a:=\frac{\pi^2}{6}-1. $$ We transform the inequality above to $$ \left(\frac{p}{p-1}\right)^{1/p} < \frac{\pi}{p\sin(\pi/p)}< \left(\frac{p+a}{p-1}\right)^{1/p}. $$ Putting $p:=1/x$, we have $$ \left(\frac{1}{1-x}\right)^x < \frac{\pi x}{\sin(\pi x)}< \left(\frac{1+ax}{1-x}\right)^x. $$ Taking the reciprocal of both sides, we obtain the conclusion. \end{proof} \begin{thebibliography}{99} \bibitem{DR} O. Do\v{s}l\'{y} and P. \v{R}eh\'{a}k, Half-linear differential equations, North-Holland Mathematics Studies 202, Elsevier, Amsterdam, 2005. \bibitem{DM} P. Dr\'{a}bek and R. Man\'{a}sevich, On the closed solution to some nonhomogeneous eigenvalue problems with $p$-Laplacian, Differential Integral Equations 12, 773--788, (1999). \bibitem{KTT} R. Kajikiya, M. Tanaka and S. Tanaka, Bifurcation of positive solutions for the one-dimensional $(p,q)$-Laplace equation. Electronic Journal of Differential Equations, 2017, Paper No. 107, (2017). \bibitem{L} L. Zhu, Sharpening Jordan's inequality and the Yang Le inequality, Appl. Math. Lett. (3) 19, 240--243, (2006). \end{thebibliography} \end{document}
2412.04848v2
http://arxiv.org/abs/2412.04848v2
Quadratic Modelings of Syndrome Decoding
\documentclass[runningheads]{llncs} \usepackage[utf8]{inputenc} \usepackage{amssymb} \usepackage{listings} \usepackage{amsfonts} \usepackage{float} \usepackage{amsmath,latexsym} \usepackage{graphicx} \usepackage{fancyvrb} \usepackage{authblk} \usepackage{paralist} \usepackage{makecell} \usepackage{comment} \usepackage{cite} \DeclareMathOperator{\lcm}{lcm} \usepackage[table,xcdraw]{xcolor} \newif\ifanonymous \anonymousfalse \usepackage{xcolor} \usepackage{tikz-cd} \usepackage{xcolor} \definecolor{linkcolor}{rgb}{0.65,0,0} \definecolor{citecolor}{rgb}{0,0.4,0} \definecolor{urlcolor}{rgb}{0,0,0.65} \usepackage[colorlinks=true, linkcolor=linkcolor, urlcolor=urlcolor, citecolor=citecolor]{hyperref} \definecolor{darkblue}{RGB}{0,0,160} \definecolor{darkdarkred}{RGB}{180,0,0} \definecolor{darkgreen}{RGB}{0,140,0} \newcommand{\FF}{\mathbb{F}} \newcommand{\FFt}{\mathbb{F}_2} \newcommand{\FFq}{\mathbb{F}_q} \newcommand{\FFqm}{\mathbb{F}_{q^m}} \newcommand{\K}{\mathbb{K}} \newcommand{\vh}{\mathbf{h}} \newcommand{\vs}{\mathbf{s}} \newcommand{\vb}{\mathbf{b}} \newcommand{\vc}{\mathbf{c}} \newcommand{\ve}{\mathbf{e}} \newcommand{\vu}{\mathbf{u}} \newcommand{\vv}{\mathbf{v}} \newcommand{\vw}{\mathbf{w}} \newcommand{\vx}{\mathbf{x}} \newcommand{\vy}{\mathbf{y}} \newcommand{\vt}{\mathbf{t}} \newcommand{\vz}{\mathbf{z}} \newcommand{\vH}{\mathbf{H}} \newcommand{\parts}[2]{\left\{{#1 \atop #2}\right\}} \newcommand{\htop}{{\mathrm{top}}} \newtheorem{algorithm}{Algorithm} \newtheorem{modeling}{Modeling} \newtheorem{notation}{Notation} \newcommand{\Cf}{\mathbf{C}_f} \newcommand{\HH}{\mathbf{H}} \newcommand{\X}{\mathcal{X}} \newcommand{\CC}{\mathcal{C}} \newcommand{\OO}{\mathcal{O}} \newcommand{\GG}{\mathcal{G}} \newcommand{\LL}{\mathcal{L}} \newcommand{\Fqm}{\mathbb{F}_{q^m}} \newcommand{\Fq}{\mathbb{F}_2} \newcommand{\supp}{\mathsf{supp}} \newcommand{\Span}{\mathsf{span}} \newcommand{\rk}{\mathsf{rk}} \newcommand{\hash}{\mathsf{hash}} \newcommand{\wt}{\mathsf{wt}} \newcommand{\lm}{\mathsf{lm}} \newcommand{\Mat}{\mathsf{Mat}} \newcommand{\pk}{\mathsf{pk}} \newcommand{\sk}{\mathsf{sk}} \newcommand{\fail}{\mathsf{fail}} \newcommand{\init}{\mathsf{init}} \newcommand{\GL}{{\sf GL}} \newcommand{\ireg}[1]{i_{\mathrm{reg}}(#1)} \newcommand{\dreg}[1]{d_{\mathrm{reg}}(#1)} \newcommand{\pr}{{\mathbb{P}}} \newcommand{\ord}{\mathsf{ord}} \newcommand{\alec}[1]{{\color{red} $\clubsuit\clubsuit\clubsuit$ Alessio C.: [#1]}} \newcommand{\alem}[1]{{\color{blue} $\clubsuit\clubsuit\clubsuit$ Alessio M.: [#1]}} \newcommand{\alex}[1]{{\color{orange} $\clubsuit\clubsuit\clubsuit$ Alex: [#1]}} \newcommand{\rocco}[1]{{\color{purple} $\clubsuit\clubsuit\clubsuit$ Rocco: [#1]}} \newcommand{\ryann}[1]{{\color{darkgreen} $\clubsuit\clubsuit\clubsuit$ Ryann: [#1]}} \newcommand{\todo}[1]{{\color{magenta} $\star$ \underline{To do:} [#1]}} \begin{document} \title{Quadratic Modelings of Syndrome Decoding} \author{Alessio Caminata \inst{1} \and Ryann Cartor \inst{2}\and Alessio Meneghetti \inst{3}\and Rocco Mora \inst{4} \and Alex Pellegrini \inst{5}} \authorrunning{A. Caminata et al.} \institute{Universit\`a di Genova \and Clemson University \and Universit\`a di Trento \and CISPA Helmholtz Center for Information Security \and Eindhoven University of Technology } \maketitle \begin{abstract} This paper presents enhanced reductions of the bounded-weight and exact-weight Syndrome Decoding Problem (SDP) to a system of quadratic equations. Over $\FFt$, we improve on a previous work and study the degree of regularity of the modeling of the exact weight SDP. Additionally, we introduce a novel technique that transforms SDP instances over $\FF_q$ into systems of polynomial equations and thoroughly investigate the dimension of their varieties. Experimental results are provided to evaluate the complexity of solving SDP instances using our models through Gr\"obner bases techniques. \keywords{Syndrome Decoding \and Gr\"obner Basis \and Cryptanalysis \and Code-Based Cryptography \and Multivariate Cryptography} \end{abstract} \section{Introduction}\label{sec:intro} As widespread quantum computing becomes closer to reality, accurate cryptanalysis of post-quantum cryptosystems is of the utmost importance. Code-based cryptography is one of the main areas of focus in the search for quantum-secure cryptosystems. This is well represented by the NIST Post-Quantum Standardization Process, where as many as three finalists, namely Classic McEliece \cite{bernstein2017classic} (an IND-CCA2 secure variation of McEliece's very first code-based scheme \cite{mceliece1978public}), HQC \cite{melchor2018hamming} and BIKE \cite{aragon2022bike}, belong to this family. Similarly, NIST's additional call for digital signatures has numerous proposals that make use of linear codes. Many of the proposed schemes are based on the hardness of (sometimes structured variants of) the syndrome decoding problem. The parameters of many code-based schemes are carefully chosen to align with the latest advancements with respect to this computational problem. Despite decades of intensive research in this direction, all the algorithms developed so far exhibit exponential complexity. This is not surprising, since the problem has been shown to be NP-hard \cite{berlekamp1978inherent}. In particular, after more than 60 years of investigation since the groundbreaking paper of Prange \cite{DBLP:journals/tit/Prange62}, the reduction in the exponent for most parameters of interest has been minimal \cite{stern1989method, D89, finiasz2009security, bernstein2011smaller, may2011decoding, becker2012decoding, may2015computing, both2018decoding}. All the works mentioned fall into the family of Information Set Decoding (ISD) algorithms, whose basic observation is that it is easier to guess error-free positions, and guessing enough of them is sufficient to decode. This resistance to ISD algorithms makes the syndrome decoding problem a reliable foundation for code-based cryptosystems. To comprehensively assess security, it is imperative to consider attacks stemming from various other realms of post-quantum cryptography. For instance, attacks typically associated with multivariate or lattice-based schemes should also be taken into account for code-based schemes, when applicable. A remarkable example is offered by dual attacks, originally introduced in lattice-based cryptography, where, however, they have been strongly questioned. In contrast, their code-based counterpart \cite{carrier2022statistical, carrier2024reduction} has recently outperformed ISD techniques for a non-negligible regime of parameters, by reducing the decoding problem to the closely related Learning Parity with Noise problem. Concerning polynomial system solving strategies, another notable illustration of this is the algebraic MinRank attack, which broke the rank-metric code-based schemes RQC and Rollo \cite{bardet2020algebraic, DBLP:conf/asiacrypt/BardetBCGPSTV20} and now represents the state-of-the-art for MinRank cryptanalysis, beating combinatorial approaches. In the Hamming metric, a reduction that transforms an instance of the syndrome decoding problem into a system of quadratic equations over $\mathbb{F}_2$ was introduced in \cite{2021/meneghetti}. The most expensive step of the transformation, in terms of numbers of new variables and new equations introduced, is the so-called \textit{Hamming-weight computation encoding}. Indeed, for a binary linear code of length $n$, the procedure dominates the overall complexity of the reduction with a complexity of $\mathcal{O}(n\log_2(n)^2)$. Despite the considerable theoretical interest in this transformation, the latter is too inefficient to be of practical interest in solving the syndrome decoding problem. Thus, the problem of improving the reduction in order to obtain a more effectively solvable system remains open. Moreover, \cite{2021/meneghetti} covers only the binary case, leaving unanswered the challenge of modeling through algebraic equations the decoding problem for codes defined over finite fields with more than two elements. \paragraph{Our contribution.} In this work, we improve on the reduction presented in \cite{2021/meneghetti} by a factor of \(\log_2(n)\), thereby reducing the number of introduced variables and equations and achieving an overall reduction cost of \(\mathcal{O}(n\log_2(n))\). This improvement is achieved by leveraging the recursive structure of the equations generated by the Hamming-weight computation encoding and by transforming the equations similarly to the reduction procedure in Buchberger's algorithm \cite{1965/buchberger} for Gröbner basis computation. When considering a version of the syndrome decoding problem that requires an error vector with a specified Hamming weight, we derive a further improved modeling, for which we study the degree of regularity. As a second contribution, we present a novel approach that transforms an instance of the syndrome decoding problem over \(\mathbb{F}_{q}\) for \(q \geq 2\) into a system of polynomial equations. This significantly broadens the applicability of our methods to a wider range of code-based cryptosystems. A common feature of our algebraic modelings is that if the decoding problem admits multiple solutions, the Gröbner basis naturally determines all of them. We also provide theoretical and experimental data to analyze the complexity of solving syndrome decoding instances using our modelings, demonstrating that, at least for small parameters, our new strategy is practical and successful. Software (MAGMA scripts) supporting this work can be found \href{https://github.com/rexos/phd-cryptography-code/tree/main/modelings}{here}. \paragraph{Structure of the paper.} The next section recalls the background and notions necessary for this work. In Section~\ref{sec:mps}, we review the reduction described in \cite{2021/meneghetti} from the syndrome decoding problem to that of finding the zeroes of a set of polynomials. In Section~\ref{sec:EWM}, we describe two modelings that improve upon \cite{2021/meneghetti}. We study the degree of regularity of the modeling for the exact weight syndrome decoding problem, along with experimental results, in Section~\ref{sec:complexity-analysis}. Finally, in Section~\ref{sec:Fq}, we present a novel modeling of the syndrome decoding problem over $\mathbb{F}_{q}$ with $q \geq 2$, for which we provide a theoretical study of the variety and experimental analysis of the solving complexity with Gr\"obner bases techniques. \section{Preliminaries} \label{sec:prelim} This paper investigates the reduction of the Syndrome Decoding Problem (SDP) into a Polynomial System Solving Problem (PoSSo). In this section, we briefly recall the definitions of both problems, as well as the notions of solving degree and degree of regularity, which are commonly used to estimate the computational complexity of the PoSSo problem. \subsection{The Syndrome Decoding Problem} An $[n,k]$-linear code $\mathcal{C}$ is a $k$-dimensional subspace of $\FF_q^n$. We call $n$ the length of the code, and $k$ its dimension. An element $\mathbf{x}\in\FF_q^n$ is called a codeword if $\mathbf{x}\in\mathcal{C}$. The number of nonzero entries in $\mathbf{x}$ is called the Hamming weight of $\mathbf{x}$ and we denote it as $\wt(\mathbf{x})$. Given a code $\mathcal{C}$ we define a parity check matrix of $\mathcal{C}$ as $\mathbf{H}\in\FF_q^{(n-k)\times n}$ such that the right kernel of $\mathbf{H}$ is the code $\mathcal{C}$. The subspace spanned by the rows of $\HH$ is called the dual code of $\mathcal{C}$. Many code-based cryptosystems rely on the hardness of solving the Syndrome Decoding Problem (SDP), see Problems~\ref{BSDP} and~\ref{EWSDP} described below. \begin{problem}[SDP: Syndrome Decoding Problem]\label{BSDP} Given integers $n,k,t$ such that $k\leq n$ and $t\leq n$, an instance of the problem SD$(\HH,\mathbf{s},t)$ consists of a parity check matrix $\mathbf{H}\in\FF_q^{(n-k)\times n}$ and a vector $\mathbf{s}\in\FF_q^{n-k}$ (called the syndrome). A solution to the problem is a vector $\mathbf{e}\in \mathbb{F}_q^n$ such that $\mathbf{He}^\top=\mathbf{s}^\top$ and $\wt(\mathbf{e})\leq t$. \end{problem} \noindent In later sections, we will also refer to Problem~\ref{BSDP} as the ``Bounded Syndrome Decoding" Problem. We will also consider the following variant of SDP. \begin{problem}[ESDP: Exact Weight Syndrome Decoding Problem]\label{EWSDP} Given integers $n,k,t$ such that $k\leq n$ and $t\leq n$, an instance of the problem ESD$(\HH,\mathbf{s},t)$ consists of a parity check matrix $\mathbf{H}\in\FF_q^{(n-k)\times n}$ and a vector $\mathbf{s}\in\FF_q^{n-k}$ (called the syndrome). A solution to the problem is a vector $\mathbf{e}\in \mathbb{F}_q^n$ such that $\mathbf{He}^\top=\mathbf{s}^\top$ and $\wt(\mathbf{e})= t$. \end{problem} Additionally, a close variant of the Syndrome Decoding Problem is the \textit{Codeword Finding Problem}, where the syndrome $\vs$ is the zero vector ${\mathbf{0}}$. Since the null vector is always a solution of the parity-check equations $\mathbf{He}^\top=\mathbf{0}^\top$, a nonzero $\ve$ of weight at most (or exactly) $t$ is sought. The name of the problem refers to the fact that any element in the right kernel of $\mathbf{H}$ belongs to the code $\mathcal{C}$ having $\HH$ as parity-check matrix. We will later need to distinguish this variant in the analysis of one of our modelings. In addition to length and dimension, a fundamental notion in coding theory and consequently in code-based cryptography is the minimum distance $d$ of an $\FF_q$-linear code, i.e. the Hamming weight of the smallest nonzero codeword in the code. Such a quantity is strictly related to the number of solutions to the syndrome decoding problem. Knowing the expected number of solutions from given parameters is extremely important in cryptography, in order to assess the security correctly. It is guaranteed that the problem does not admit more than one solution as long as the number of errors is upper bounded by $\frac{d-1}{2}$. However, in practice, much better can be done for randomly generated codes. Indeed, it turns out that random codes achieve the so-called Gilbert-Varshamov (GV) distance $d_{GV}$, defined as the largest integer such that \[ \sum_{i=0}^{d_{GV}-1} \binom{n}{i}(q-1)^i \le q^{n-k}. \] It can be shown that, as long as the number of errors is below the Gilbert-Varshamov distance, the Syndrome Decoding problem \textit{typically} has a unique solution. Moreover, the instances where the number of errors attains the GV distance are those supposed to be the most difficult. \subsection{The Polynomial System Solving Problem} The Polynomial System Solving Problem (PoSSo) is the following. We define it over a finite field $\FF_q$, athough it can be more generally considered over any field. \begin{problem}[PoSSo: Polynomial System Solving]\label{PoSSo} Given integers $N,r\geq2$, an instance of the PoSSo problem consists of a system of polynomials $\mathcal{F}=\{f_1,\dots,f_r\}$ in $R=\FF_q[x_1,\dots,x_N]$ with $N$ variables and coefficients in $\FF_q$. A solution to the problem is a vector $\mathbf{a}\in\FF_q^N$ such that $f_1(\mathbf{a})=\cdots=f_r(\mathbf{a})=0$. \end{problem} \begin{remark}A special case of PoSSo when $\deg(f_i)=2$ for $1\leq i\leq r$ is called MQ (Multivariate Quadratic) and is the basis for multivaritate cryptography. \end{remark} The following outlines a standard strategy for finding the solutions of a polynomial system $\mathcal{F}$ by means of Gr\"obner bases. \begin{compactenum} \item Find a degree reverse lexicographic ($\mathsf{degrevlex}$) Gr\"obner basis of the ideal $\langle\mathcal{F}\rangle$; \item Convert the obtained $\mathsf{degrevlex}$ Gr\"obner basis into a lexicographic ($\mathsf{lex}$) Gr\"obner basis, where the solutions of the system can be easily read from the ideal in this form. \end{compactenum} The second step can be done by FGLM \cite{FGLM93}, or a similar algorithm, whose complexity depends on the degree of the ideal. This is usually faster than the first step, especially when the system $\mathcal{F}$ has few solutions. Therefore, we focus on the first step. The fastest known algorithms to compute a $\mathsf{degrevlex}$ Gr\"obner basis are the linear algebra based algorithms such as F4 \cite{faugereF4}, F5 \cite{F5paper}, or XL \cite{XL00}. These transform the problem of computing a Gr\"obner basis into one or more instances of Gaussian elimination of the Macaulay matrices. The complexity of these algorithms is dominated by the Gaussian elimination on the largest Macaulay matrix encountered during the process. The size of a Macaulay matrix depends on the degrees of the input polynomials $f_1,\dots,f_r$, on the number of variables $N$, and on a degree $d$. In a nutshell, the \emph{Macaulay matrix} $M_{\leq d}$ of degree $d$ of $\mathcal{F}$ has columns indexed by the monic monomials of degree $\leq d$, sorted in decreasing order from left to right (with respect to the chosen $\mathsf{degrevlex}$ term order). The rows of $M_{\leq d}$ are indexed by the polynomials $m_{i,j}f_j$, where $m_{i,j}$ is a monic monomial such that $\deg(m_{i,j}f_j)\leq d$. The entry $(i,j)$ of $M_{\leq d}$ is the coefficient of the monomial of column $j$ in the polynomial corresponding to the $i$-th row. The \emph{solving degree} of $\mathcal{F}$ is defined as the least degree $d$ such that Gaussian elimination on the Macaulay matrix $M_{\leq d}$ produces a $\mathsf{degrevlex}$ Gr\"obner basis of $\mathcal{F}$. We denote the solving degree of $\mathcal{F}$ by $d_{\mathrm{sol}}(\mathcal{F})$. We have to compute Macaulay matrices up to degree $d_{\mathrm{sol}}=d_{\mathrm{sol}}(\mathcal{F})$, and the largest one we encounter has $a=\sum_{i=1}^r{{N+d_{\mathrm{sol}}-d_i}\choose{d_{\mathrm{sol}}-d_i}}$ many rows and $b={{N+d_{\mathrm{sol}}}\choose{d_{\mathrm{sol}}}}$ many columns, where $d_i=\deg f_i$. Therefore, taking into account the complexity of Gaussian elimination of this matrix, an upper bound on the complexity of solving the system $\mathcal{F}$ with this method is \begin{equation}\label{eq:GBcomplexity} \OO\left({{N+d_{\mathrm{sol}}}\choose{d_{\mathrm{sol}}}}^\omega\right), \end{equation} with $2\leq\omega\leq3$. \begin{remark} If $\mathcal{F}$ is not homogeneous, Gaussian elimination on $M_{\leq d}$ may produce a row corresponding to a polynomial $f$ with $\deg f<d$, where the leading term of $f$ was not the leading term of any row in $M_{\leq d}$. Some algorithms, for example $F4$, address this by adding rows for polynomials $mf$ ($\deg(mf)\leq d$) for some monomial $m$ and recomputing the reduced row echelon form. If no Gr\"obner basis is found in degree $\leq d$, they proceed to higher degrees, potentially enlarging the span of $M_{\leq d}$ and reducing the solving degree. Throughout this paper, we consider only the case where no extra rows are added. Note that the solving degree as defined above is an upper bound on the degree at which algorithms using this variation terminate. \end{remark} Since the solving degree of a polynomial system may be difficult to estimate, several invariants related to the solving degree (that are hopefully easier to compute) have been introduced. One of the most important is the \emph{degree of regularity} introduced by Bardet, Faug\`ere, and Salvy \cite{bardet2004complexity}. We briefly recall its definition and connection with the solving degree. Let $\langle\mathcal{F}^{\mathrm{top}}\rangle=\langle f_1^{\mathrm{top}},\dots,f_r^{\mathrm{top}}\rangle$ be the ideal of the polynomial ring $R$ generated by the homogeneous part of highest degree of the polynomial system $\mathcal{F}$. Assume that $\langle\mathcal{F}^{\mathrm{top}}\rangle_d=R_d$ for $d\gg0$. The \emph{degree of regularity} of $\mathcal{F}$ is \begin{equation*} \dreg{\mathcal{F}}=\min\{d\in\mathbb{N}\mid \langle\mathcal{F}^{\mathrm{top}}\rangle_e=R_e \ \forall e\geq d\}. \end{equation*} The degree of regularity can be read off from the Hilbert series of $\langle\mathcal{F}^{\mathrm{top}}\rangle$. Let $I$ be a homogeneous ideal of $R$, and let $A=R/I$. For an integer $d\geq 0$, we denote by $A_d$ the homogeneous component of degree $d$ of $A$. The function $\mathrm{HF}_A(-):\mathbb{N}\rightarrow\mathbb{N}$, $\mathrm{HF}_A(d)=\dim_{\FF_q}A_d$ is called \emph{Hilbert function} of $A$. The generating series of $\mathrm{HF}_A$ is called \emph{Hilbert series} of $A$. We denote it by $\mathrm{HS}_A(z)=\sum_{d\in\mathbb{N}}\mathrm{HF}_A(d)z^d$. \begin{remark}\label{rem:polyHS} Under the assumption that $\langle\mathcal{F}^{\mathrm{top}}\rangle_d=R_d$ for $d\gg0$, the Hilbert series of $A=R/\langle\mathcal{F}^{\mathrm{top}}\rangle$ is a polynomial. Then, the degree of regularity of $\mathcal{F}$ is given by $\dreg{\mathcal{F}}=\deg \mathrm{HS}_A(z)+1$ (see \cite[Theorem~12]{2021/caminatagorla}). \end{remark} \noindent Under suitable assumptions, the degree of regularity provides an upper bound for the solving degree \cite{CaminataG23, 2023/salizzoni, Semaev2021651}. Moreover, it is often assumed that the two values are close. Although this occurs in many relevant situations, there are examples where these two invariants can be arbitrarily far apart (see \cite{2021/caminatagorla, 2013/dingschmidt, Bigdeli202175}). We will see in Section~\ref{sec:dreg-EWM} that the degree of regularity of the system presented in Section~\ref{subsec:f2ESD} seems to yield a much higher value than the solving degree achieved during the Gr\"obner basis algorithm. \section{The MPS Modeling}\label{sec:mps} This section is devoted to an overview of the algebraic modeling of the syndrome decoding problem proposed in~\cite{2021/meneghetti} (referred to as the MPS modeling). We fix the following notation for this section. \begin{notation}\label{MPSnotation} Let $n\ge 2$ and let $\CC \subseteq \FF_2^n$ be a $[n,k,d]$-linear code having a parity check matrix $\HH \in \FF_2^{(n-k) \times n}$. We define $\ell = \lfloor \log_2(n) \rfloor + 1$. Let $\vs \in \FF_2^{n-k}$ play the role of the syndrome and let $0\le t \le \lfloor (d-1)/2 \rfloor$ be the target error weight. Let $X = \left(x_1,\ldots,x_n\right)$ and $Y=(Y_1,\dots,Y_n)$ with $Y_j=(y_{j,1}, \dots, y_{j,\ell})$ be two sets of variables and we consider the polynomial ring $\FF_2[X,Y]$. \end{notation} We define the following maps $\pi_i$ for $i=1,\ldots,n$, \begin{align*} \pi_i : \FFt^{n} &\rightarrow \FFt^i \\ (v_1,\ldots,v_n) &\mapsto (v_1,\ldots,v_i). \end{align*} The construction of the proposed algebraic modeling consists of four steps and uses the variables contained in $X$ and $Y$ to express relations and dependencies. Each of these steps produces a set of polynomials in $\FF_2[X,Y]$. An extra step of the construction reduces the aforementioned polynomials to quadratic polynomials. The idea is to construct an algebraic system having a variety containing elements $(\vx \mid \vy_1 \mid \cdots \mid \vy_n)\in \FFt^{n(\ell + 1)}$ whose first $n$ entries represent an element $\vx$ of $\FFt^n$ such that $\HH\vx^\top = \vs^\top$. The remaining $n\ell$ entries are considered to be the concatenation of $n$ elements $\vy_i \in \FFt^{\ell}$ where the elements of $\vy_i$ represent the binary expansion of $\wt(\pi_i(\vx))$ for every $i=1,\ldots,n$, with $\pi_i(\vx)=(x_1,\dots,x_i)$. By this definition, the list $\vy_n$ represents the binary expansion of $\wt(\vx)$. The system finally enforces that $\vy_n$ represents the binary expansion of an integer $t^\prime$ such that $t^\prime \le t$. The elements of the variety of solutions of this algebraic modeling are finally projected onto their first $n$ coordinates, revealing the solutions to the original syndrome decoding problem. Here is a description of the four steps of reduction of the MPS modeling. We describe the set obtained in each step as a set of polynomials in $\FFt[X,Y]$. \begin{itemize} \item \textit{Parity check encoding.} This step ensures that the solution of the algebraic system satisfies the parity check equations imposed by the parity check matrix $\HH$ and the syndrome vector $\vs$. Here, we compute the set of $n-k$ linear polynomials \begin{equation}\label{eq:pce} \left\{\sum_{i=1}^n h_{i,j}x_i + s_j \mid j\in\{1,\ldots,n-k\}\right\}. \end{equation} \item \textit{Hamming weight computation encoding.} This part of the modeling provides a set of polynomials that describes the binary encoding of $\wt(\pi_i(\vx))$ for every $i=1,\ldots,n$ described above. The set of polynomials achieving this goal, is given by the union of the three following sets consisting of the $\ell+n-1$ polynomials in the sets \begin{equation} \begin{split}\label{eq:lineareqs} &\left\{ f_{1,1}=x_1 + y_{1,1}, f_{1,2}=y_{1,2}, \ldots, f_{1,\ell}=y_{1,\ell} \right\},\\ &\left\{f_{i,1}=x_i + y_{i, 1} + y_{i-1,1} \mid i=2,\ldots,n \right\} \end{split} \end{equation} and the $(n-1)(\ell -1)$ polynomials \begin{equation}\label{eq:othereqs} \left\{ f_{i,j}=\left(\prod_{h=1}^{j-1}y_{i-1, h}\right)x_i + y_{i,j} + y_{i-1,j} \mid i=2,\ldots,n,\ j=2,\ldots,\ell \right\}. \end{equation} We labeled the polynomials of the sets in~\eqref{eq:lineareqs} and in~\eqref{eq:othereqs} because the improvements in the next sections will mainly involve them. \item \textit{Weight constraint encoding.} This part produces a set consisting of a single polynomial that enforces the constraint $\wt(\vx) \le t$ by dealing with the variables in $Y_n$. Let $\vv \in \FFt^\ell$ represent the binary expansion of $t$. Consider the $\ell$ polynomials in $\FFt[X,Y]$ defined as $$f_j = (y_{n, j} +v_j)\prod_{h=j+1}^\ell (y_{n, h} + v_h + 1) $$ for $j=1,\ldots,\ell$. The set is the singleton \begin{equation}\label{eq:MPSwce} \left\{ \sum_{j=1}^\ell (v_j + 1)f_j \right\}. \end{equation} \item \textit{Finite field equations.} The set of $n + n\ell$ finite field polynomials of $\FFt[X,Y]$ is \begin{equation} \label{eq:ffe} \left\{x_i^2- x_i \mid i=1,\ldots,n\right\} \cup \left\{y_{i,j}^2- y_{i,j} \mid i=1,\ldots,n,\ j=1,\ldots,\ell\right\}, \end{equation} and ensures that the elements of the variety are restricted to elements of $\FFt^{n(\ell + 1)}$. \end{itemize} The algebraic system corresponding to an instance of the syndrome decoding problem is then the union of the four sets described above. Clearly, this is not a quadratic system; thus the authors apply a linearization strategy that introduces a number of auxiliary variables used to label monomials of degree $2$. This eventually results in a large quadratic system in many more than just $n(\ell + 1)$ variables. In fact, the final quadratic system ends up having equations and variables bounded by $\OO(n\log_2(n)^2)$. \section{Improving the MPS Modeling}\label{sec:EWM} In this section, we provide improvements of the MPS modeling that reduce the number of equations and variables in the final algebraic system. We keep the same notation as in Notation~\ref{MPSnotation}. First, we consider the case of the syndrome decoding problem, i.e. with a bounded weight error. We then consider the case of the exact weight syndrome decoding problem. We observe that one can avoid the linearization step as the resulting system is already quadratic. \subsection{Improved Modeling for the Case of SDP}\label{subsec:f2SD} We consider the $\mathsf{degrevlex}$ monomial ordering on $\FFt[X,Y]$ with the $X$ variables greater than the $Y$ variables, and denote by $\lm(p)$ the leading monomial of a polynomial $p$. Notice that since we are in the binary case, the notions of leading monomial and that of leading term coincide. Denote by $F = \{f_{i,j} \mid i=1,\ldots,n,\ j=1,\ldots,\ell\} \subset \FFt[X,Y]$ the set of polynomials of cardinality $n\ell$ given by \eqref{eq:lineareqs} and \eqref{eq:othereqs} for a code of length $n$. We aim at building a set $G=\{g_{i,j} \mid i=1,\ldots,n,\ j=1,\ldots,\ell\}\subset \FFt[X,Y]$ consisting of polynomials of degree at most $2$ such that $\langle G \rangle = \langle F \rangle$. Denote with $F[i,j]$ the polynomial $f_{i,j}$, similarly for $G$. We first give a description of the set $G$ and then formally describe the new modeling. Construct $G$ as follows: \begin{itemize} \item Put $G[1,1] = x_1 + y_{1,1}$ and $G[1,h] = y_{1,h}$ for $h = 2,\ldots, \ell$; \item Set $G[i,1] = F[i,1] = x_i + y_{i, 1} + y_{i-1,1}$ for every $i = 2,\ldots,n$; \item Compute \begin{align*} G[i,j] &= F[i,j] + y_{i-1, j-1}F[i,j-1]\\ &= F[i,j] + \lm(F[i,j]) + y_{i-1, j-1}(y_{i,j-1} + y_{i-1,j-1})\\ &= y_{i,j} + y_{i-1,j} + y_{i-1,j-1}^2 + y_{i,j-1}y_{i-1,j-1}. \end{align*} for every $i=2,\ldots,n$ and $j = 2,\ldots,\ell$, where equality holds because $\lm(F[i,j]) = y_{i-1,j-1}\lm(F[i,j-1])$. \end{itemize} \begin{remark} The algebraic system we are going to construct contains the field polynomials $x_i^2- x_i$ for each $i=1,\ldots,n$ and $y_{i,j}^2- y_{i,j}$ for every $i=1,\ldots,n$ and $j=1,\ldots,\ell$. Therefore, in terms of generating elements of the ideal, any squared term in $G[i,j]$ can be reduced to a linear term. \end{remark} The set $G \subset \FFt[X,Y] $ contains $n\ell$ polynomials of degree at most two. The following proposition proves that the set $G \subset \FFt[X,Y]$ computed as above and $F$ generate the same ideal of $\FFt[X,Y]$. \begin{proposition} We have $\langle G \rangle = \langle F \rangle$. \end{proposition} \begin{proof} The inclusion $\langle G \rangle \subseteq\langle F \rangle$ is trivial. To prove the other inclusion, we show that we can write any element of the basis $F$ as an $\FFt[X,Y]$-linear combination of elements of the basis $G$. By construction, $G[1,j] = F[1,j]$ for every $j=1,\ldots,\ell$. For every $i = 2,\ldots,n$ we prove $F[i,j]\in \langle G \rangle$ by induction on $j$.\\ For $j=1$ we have $F[i,1] = G[i,1]$.\\ Assume that $F[i,j] = \sum_{h=1}^j p_{i,j,h} G[i,h]$ with $p_{i,j,h}\in \FFt[X,Y]$. Then by construction we have \begin{align*} F[i,j+1] &= G[i,j+1] - y_{i-1, j}F[i,j]\\ &= G[i,j+1] - y_{i-1, j} \sum_{h=1}^j p_{i,j,h} G[i,h] \end{align*} proving the claim. \qed \end{proof} We thus redefine the Hamming weight computation encoding as follows: \begin{itemize} \item \textit{Hamming weight computation encoding.} Compute the following union of subsets of $\FFt[X,Y]$: \begin{align*} &\left\{ x_1 + y_{1,1}, y_{1,2}, \ldots, y_{1,\ell} \right\} \cup \left\{x_i + y_{i, 1} + y_{i-1,1} \mid i=2,\ldots,n \right\}\\ &\cup \big\{ y_{i,j-1}y_{i-1,j-1} + y_{i,j} + y_{i-1,j-1} + y_{i-1,j} \\ & \ \ \ \mid i=2,\ldots,n,\ j=2,\ldots,\ell \big\}, \end{align*} \end{itemize} \subsubsection{Further improvement.} Set now $\ell_t = \lfloor \log_2 (t) \rfloor + 1$. A further improvement to the MPS modeling (described in Equation~\eqref{eq:SDhwce}) follows by observing that in the non-trivial case where $t < n$, we can impose that the last $\ell-\ell_t$ entries of $\vy_i$ must be $0$ for every $i=1,\ldots,n$. This means that we can add the linear equations $y_{i, j} = 0$ for every $i=1,\ldots,n$ and $j=\ell_t+1,\ldots,\ell$. By inspection, setting the aforementioned variables to $0$ will make part of the equations of the Hamming weight computation encoding vanish. We can equivalently simply consider the equations that remain, and get rid of the variables which have been set to $0$. Consider the following updated notation. \begin{notation}\label{ImprovedMPSnotation} Let $n\ge 2$ and let $\CC \subseteq \FF_2^n$ be a $[n,k,d]$-linear code having a parity check matrix $\HH \in \FF_2^{(n-k) \times n}$. Let $\vs \in \FF_2^{n-k}$ play the role of the syndrome and let $0\le t \le \lfloor (d-1)/2 \rfloor$ be the target error weight. We define $\ell_t = \lfloor \log_2(t) \rfloor + 1$. Let $X = \left(x_1,\ldots,x_n\right)$ and $Y=(Y_1,\dots,Y_n)$ with $Y_j=(y_{j,1}, \dots, y_{j,\ell_t})$ be two sets of variables and consider the polynomial ring $\FF_2[X,Y]$. \end{notation} Under Notation~\ref{ImprovedMPSnotation}, the effect of our improvement on the set of polynomials produced by the Hamming weight computation encoding is the following. \begin{itemize} \item \textit{Hamming weight computation encoding.} Compute the following union of subsets of $\FFt[X,Y]$: \begin{equation}\label{eq:SDhwce} \begin{split} &\left\{ x_1 + y_{1,1}, y_{1,2}, \ldots, y_{1,\ell_t} \right\} \cup \left\{x_i + y_{i, 1} + y_{i-1,1} \mid i=2,\ldots,n \right\}\\ &\cup \big\{ y_{i,j-1}y_{i-1,j-1} + y_{i,j} + y_{i-1,j-1} + y_{i-1,j} \\ & \ \ \ \mid i=2,\ldots,n,\ j=2,\ldots,\ell_t \big\} \cup \left\{ y_{i,\ell_t}y_{i-1,\ell_t} + y_{i-1,\ell_t} \mid i=2,\ldots,n\right\}. \end{split} \end{equation} \end{itemize} The effect on the weight constraint encoding is simply the decrease in the degree from $\ell$ to $\ell_t$ of the produced polynomial. This is the only non-quadratic polynomial left in the modeling. We can turn this polynomial into a set of $\OO(t\ell_t)$ polynomials of degree up to $2$ in $\OO(t\ell_t)$ variables with the same linearization techniques described in~\cite[Fact 1 and Lemma 11]{2021/meneghetti}. To summarize, our modeling is defined in the following way. \begin{modeling}[Improved Modeling for the SDP over $\FF_2$] \label{modeling: improvedSD_F2} Given an instance $(\HH,\mathbf{s},t)$ of Problem~\ref{BSDP} over $\FF_2$, Modeling~\ref{modeling: improvedSD_F2} is the union of the sets of polynomials \eqref{eq:pce},\eqref{eq:MPSwce}, \eqref{eq:ffe} and \eqref{eq:SDhwce}. \end{modeling} The improved modeling is an algebraic system of $\OO(n(\ell_t+2) -k + t\ell_t)$ polynomials of degree at most $2$ in $\OO(n(\ell_t+1) + t\ell_t)$ variables. Note that most applications of the SDP to code-based cryptography, for instance in the McEliece scheme, choose $t \ll n$, hence the asymptotic bounds on the number of polynomials and variables in the improved modeling are both $\OO(n\ell_t)$. As shown in Table \ref{table: improvement}, our modeling improves over MPS by a factor of $\log_2(n) \log_t(n)$. \begin{table}[H] \centering \begin{tabular}{|c|c|c|} \hline & \# Polynomials & \# Variables\\ \hline \cite{2021/meneghetti} & $\mathcal{O}( n \log_2(n)^2)$ & $\mathcal{O}( n \log_2(n)^2)$ \\ \hline Modeling~\ref{modeling: improvedSD_F2} & $\OO(n\log_2(t))$ & $\OO(n\log_2(t))$\\ \hline \end{tabular} \vspace{2mm} \caption{Comparison with the asymptotic size of the polynomial system in \cite[Theorem 13]{2021/meneghetti}, where $n$ is the length of the code and $t$ the bound on the weight of the target vector, that is $\wt(\ve)\leq t$.} \label{table: improvement} \end{table} \subsection{Improved Modeling for the Case of ESDP}\label{subsec:f2ESD} It is possible to obtain an algebraic modeling for the ESDP by tweaking the modeling described in the previous section. In fact, it is enough to redefine the weight constraint encoding to enforce that $\vy_n$ represents the binary expansion of an integer $t^\prime$ such that $t^\prime=t$ exactly. To this end, let $\vv \in \FFt^{\ell_t}$ represent the binary expansion of an integer $t$. Under the same notation as in Notation~\ref{ImprovedMPSnotation}, the following version of the weight constraint encoding describes the ESDP modeling with $\wt(\ve) = t$. \begin{itemize} \item \textit{Weight constraint encoding.} Compute the following set of linear polynomials: \begin{equation}\label{eq:ESDwce} \left\{ y_{n, j} + v_j \mid j=1,\ldots,\ell_t \right\}. \end{equation} \end{itemize} Using these polynomials leads to Modeling \begin{modeling}[Improved Modeling for the ESDP over $\FF_2$] \label{modeling: improvedESD_F2} Given an instance $(\HH,\mathbf{s},t)$ of Problem~\ref{EWSDP} over $\FF_2$, Modeling~\ref{modeling: improvedESD_F2} is the union of the sets of polynomials \eqref{eq:pce}, \eqref{eq:ffe}, \eqref{eq:SDhwce} and \eqref{eq:ESDwce}. \end{modeling} Observe that, replacing the original Hamming weight computation encoding with that in~\eqref{eq:SDhwce} and the weight constraint encoding with that in~\eqref{eq:ESDwce}, we obtain an algebraic system of polynomials of degree at most $2$ for ESDP. Hence, linearization is not needed, moreover, we can give the exact number of equations and variables of this system. We report these values in Table~\ref{table:esd-model-sizes}. \begin{table}[H] \centering \begin{tabular}{|c|c|c|} \hline & \# Polynomials & \# Variables\\ \hline Modeling~\ref{modeling: improvedESD_F2} & $2n\ell_t + 3n + \ell_t - k - 1$ & $n(\ell_t + 1)$\\ \hline \end{tabular} \vspace{2mm} \caption{Number of equations and variables of the algebraic modeling of ESDP with $\wt(\ve)=t$. The value of $\ell_t$ is $\lfloor \log_2(t) \rfloor + 1$.} \label{table:esd-model-sizes} \end{table} \section{Complexity Analysis of Modeling~\ref{modeling: improvedESD_F2}}\label{sec:complexity-analysis} \label{sec:dreg-EWM} In this section, we investigate the complexity of solving the algebraic system for the ESDP given in Modeling~\ref{modeling: improvedESD_F2} using standard Gröbner basis methods. An upper bound on the complexity is given by the formula \eqref{eq:GBcomplexity} which depends on both the number of variables and the solving degree. Typically, the solving degree of the system is estimated by assessing its degree of regularity. However, in our analysis, we experimentally show that the degree of regularity often significantly exceeds the solving degree for systems given in Section~\ref{subsec:f2ESD} (see the results in Table~\ref{Tab:q2-SolveDeg}). This distinction is crucial in cryptography, where these concepts are frequently used interchangeably. Our findings underscore the importance of thoroughly verifying such claims to ensure accurate security assessments and parameter selection. \begin{remark} We point out that the study in \cite{2023/briaud} investigates a particular case of the problem that this paper deals with, that is the \emph{regular} syndrome decoding problem. The regular syndrome decoding problem considers error vectors having a regular distribution of non-zero entries. The algebraic modeling proposed in~\cite{2023/briaud} is conjectured to exhibit semi-regular behavior when the linear parity-check constraints and the fixed, structured quadratic polynomials are considered separately. This suggests that, to some extent, their model behaves like a random polynomial system. Despite the fact that the problem tackled in~\cite{2023/briaud} is a particular case of the problem we consider, our modeling has not been devised as a generalization of their modeling. Furthermore, we show that for the more general case, our modeling yields different results. \end{remark} For the rest of this section, we retain the notation defined in Notation~\ref{ImprovedMPSnotation}. We consider the polynomial ring $\FFt[X,Y]$ with the $\mathsf{degrevlex}$ term order with the $X$ variables greater than the $Y$ variables. Let $S \subset \FFt[X,Y]$ be the set of polynomials of Modeling~\ref{modeling: improvedESD_F2} as described in Section~\ref{subsec:f2ESD}. Let $L$ and $Q$ denote the sets of linear and quadratic polynomials, respectively. Clearly $S = L \cup Q$. Write also $L = L_\vH \cup P$, where $L_\vH$ denotes the set of linear polynomials in~\eqref{eq:pce} introduced with the parity check matrix $\vH$, and $P$ denotes the remaining linear polynomials in $S$. In other words, $P$ is the following set \[\begin{split} P = &\left\{ x_1 + y_{1,1}, y_{1,2}, \ldots, y_{1,\ell_t} \right\} \cup \left\{x_i + y_{i, 1} + y_{i-1,1} \mid i=2,\ldots,n \right\} \\ \cup &\left\{ y_{n, j} + v_j \mid j=1,\ldots,\ell_t \right\}. \end{split} \] We want to estimate the degree of regularity of $S$. Since we do not know $L_\vH$ a priori, we consider the set $S\setminus L_\vH = Q \cup P$ and compute its degree of regularity. Indeed, we found that analyzing the degree of regularity or solving degree of the system with the linear equations \eqref{eq:pce} of $L_\vH$ included was too challenging and unpredictable, as it heavily depends on the specific instance of the parity check matrix $\vH$. For this reason, we chose to establish mathematical results for the system without $L_{\vH}$, with the aim of providing a clearer foundation. Notice that the degree of regularity of $S\setminus L_\vH = Q \cup P$ gives an upper bound to the degree of regularity of the whole system $S$ (see Remark~\ref{rem:range fordregS}). We break down the problem by first computing the degree of regularity of $Q$ and then that of $Q \cup P$. We take advantage of the fact that the Hilbert series of $Q$ and of $Q \cup P$ are polynomials and compute their degree, i.e. for instance, $\dreg{Q}=\deg \mathrm{HS}_{\FFt[X,Y]/\langle Q^\htop\rangle}(z)+1$ as per Remark~\ref{rem:polyHS}, similarly for $Q\cup P$. To this end, we are going to compute the maximum degree of a monomial in $\FFt[X,Y]/\langle Q^\htop\rangle$, similarly we do for $Q \cup P$. \subsubsection{The quadratic polynomials.}\label{subsec:quad-polys} We begin by studying the degree of regularity of the quadratic part $Q$ of the system $S$ of Modeling~\ref{modeling: improvedESD_F2}. The highest degree part of $Q$ has a very nice structure, as explained in the following remark. \begin{remark}\label{rem:qtopdef} The set $Q^\htop$ is the union of the following three sets $$\left\{x_i^2 \mid i=1,\ldots,n\right\}, \left\{y_{i,j}^2 \mid i=1,\ldots,n,\ j=1,\ldots,\ell_t\right\}$$ and $$\left\{ y_{i-1,j}y_{i,j} \mid i=2,\ldots,n,\ j=1,\ldots,\ell_t \right\}.$$ The ideal $\langle Q^\htop \rangle \subseteq \FFt[X,Y]$ is thus a monomial ideal. \end{remark} The following lemma gives the structure of the quotient ring $\FFt[X,Y]/\langle Q^\htop \rangle$. \begin{lemma}\label{lem:groebnerQh} The set $Q^\htop$ is a Gr\"obner basis of the ideal $\langle Q^\htop\rangle$. \end{lemma} \begin{proof} As observed in Remark~\ref{rem:qtopdef}, $Q^\htop$ is a monomial ideal. Given any two elements of $m_1,m_2 \in Q^\htop$ it is clear that for $a = \lcm (m_1,m_2)/m_1 \in \FFt[X,Y]$ and $b = \lcm (m_1,m_2)/m_2 \in \FFt[X,Y]$ we have that $am_1 - bm_2 = 0$. \qed \end{proof} \ifodd0 We can exploit the knowledge of the Gr\"obner basis of $\langle Q^\htop \rangle$ given in Lemma \ref{lem:groebnerQh} to compute the coefficients of the Hilbert series $\mathcal{H}_R$. The $(k+1)$-th coefficient of $\mathcal{H}_R$ is given by $\dim_{\FFq}(\FFt[X,Y]_k/I_k)$, in other words, the number of monomials of degree $k$ in $R$. This coincides with the number of monomials of $\FFt[X,Y]$ of degree $k$ that are not a multiple of any monomial in $\GG$. We can model this problem in terms of subsets of $[n(l+1)]$, or equivalently, elements of $2^{[n(l+1)]}$. Let $B_1,\ldots B_{n\ell -n-\ell +1}$ be the sets of two elements indexing the variables of each mixed monomial in $\GG$ (monomials in the third set). Counting monomials of degree $k$ in $R$ boils down to counting the number of subsets of $[n(l+1)]$ of cardinality $k$ not containing any $B_i$. \begin{example}\label{ex:n4} Let $n=4$ be the length of a code, then $\ell_t = 2$. A Gr\"obner basis of $\langle Q^\htop \rangle$ is the union of \begin{equation*} \left\{ y_{1,1}y_{2,1}, y_{1,2}y_{2,2}, y_{2,1}y_{3,1}, y_{2,2}y_{3,2}, y_{3,1}y_{4,1}, y_{3,2}y_{4,2}\right\} \end{equation*} and \begin{equation*} \left\{ x_{1}^2, x_{2}^2, x_{3}^2, x_{4}^2, y_{1,1}^2, y_{1,2}^2, y_{2,1}^2, y_{2,2}^2, y_{3,1}^2, y_{3,2}^2, y_{4,1}^2, y_{4,2}^2 \right\}. \end{equation*} \ifodd0 Following our argument we obtain the $(n-1)\cdot(l-1) = n\ell -n-\ell+1 = 6$ sets $B_i$, indexing mixed monomials, are \begin{align*} B_1 = \{1,4\},&B_2 = \{4,7\},B_3 = \{7,11\},\\ B_4 = \{2,5\},&B_5 = \{5,8\},B_6 = \{8,11\}. \end{align*} \end{example} \noindent The following simple lemma is crucial for computing the degree of regularity of $Q$. For the sake of simplicity, we state it in terms of sets, and it ultimately provides a method to construct maximal monomials in the quotient ring $\FFt[X,Y]/\langle Q^\htop \rangle$. \begin{lemma}\label{lem:maximalset} Let $ \mathcal{N} = \{1, 2, 3, \dots, n\} $ and $ \mathcal{P} = \{\{1,2\}, \{2,3\}, \dots, \{n-1, n\}\} $, where $ \mathcal{P} $ consists of consecutive pairs of elements from $ \mathcal{N} $. Then: \begin{itemize} \item If $ n $ is even, there are exactly two sets of maximal cardinality $ \mathcal{S}_1, \mathcal{S}_2 \subseteq \mathcal{N} $ such that no set in $ \mathcal{P} $ is a subset of $ \mathcal{S} $. \item If $ n $ is odd, there is exactly one set of maximal cardinality $ \mathcal{S} \subseteq \mathcal{N} $ such that no set in $ \mathcal{P} $ is a subset of $ \mathcal{S} $. \end{itemize} \end{lemma} \begin{proof} We aim to find the number of sets of maximal cardinality $ \mathcal{S} \subseteq \mathcal{N} $ such that no pair from $ \mathcal{P} $ (i.e., no two consecutive elements) appears in $ \mathcal{S} $. In order to avoid pairs of consecutive elements, we can only select non-consecutive elements from $ \mathcal{N} $. To maximize the size of $ \mathcal{S} $, we select every other element from $ \mathcal{N} $. The size of such a set of maximal cardinality $ \mathcal{S} $ is: $\left\lceil \frac{n}{2} \right\rceil$. Thus: \begin{itemize} \item If $ n $ is even, a set of maximal cardinality contains $ \frac{n}{2} $ elements. \item If $ n $ is odd, a set of maximal cardinality contains $ \frac{n+1}{2} $ elements. \end{itemize} \textbf{Case 1: $ n $ is even.} Let $ n = 2k $. The largest possible set $ \mathcal{S} $ will contain $ k = \frac{n}{2} $ elements. There are exactly two ways to construct such a set: \begin{enumerate} \item Start with 1 and select every other element: $\mathcal{S}_1 = \{1, 3, 5, \dots, n-1\}.$ This set contains all the odd-numbered elements of $ \mathcal{N} $, and its size is $ k $. \item Start with 2 and select every other element: $\mathcal{S}_2 = \{2, 4, 6, \dots, n\}.$ This set contains all the even-numbered elements of $ \mathcal{N} $, and its size is also $ k $. \end{enumerate} Since there are no other ways to select $ k $ elements without picking consecutive elements, these are the only two sets of maximal cardinality for $ n $ even.\\ \textbf{Case 2: $ n $ is odd.} Let $ n = 2k + 1 $. The largest possible set $ \mathcal{S} $ contains $ k + 1 = \frac{n+1}{2} $ elements. In this case, there is only one way to construct a set of size $ k + 1 $ that avoids consecutive elements, i.e. start with 1 and select every other element: $\mathcal{S}_1 = \{1, 3, 5, \dots, n\}.$ This set contains $ k + 1 $ elements and avoids consecutive pairs. If we were to start with 2 and select every other element, we would only get $ k $ elements: $\mathcal{S}_2 = \{2, 4, 6, \dots, n-1\}.$ This is not maximal, as it contains fewer than $ k + 1 $ elements. Thus, for $ n $ odd, there is exactly one maximal set. \qed \end{proof} Lemma~\ref{lem:maximalset} can be used to prove the following corollary, which we will use to construct a maximal degree monomial in $\FFt[X,Y]/\langle Q^\htop \rangle$. The idea behind the construction lies in the observation that a Gr\"obner basis of $Q^\htop$ can be written as the union of disjoint subsets $Q^\htop_{j,n}$ for $j=1,\ldots,\ell_t$, see Theorem~\ref{Thm:Dreg-of-Qtop}, which we describe in the next corollary. Also, the next corollary computes a maximal degree monomial with respect to $Q^\htop_{j,n}$ for every $j=1,\ldots,\ell_t$. Given these monomials, computing a maximal degree monomial in $\FFt[X,Y]/\langle Q^\htop \cup P^\htop\rangle$, or equivalently, the degree of its Hilbert series, becomes feasible with a slight modification of the subsets due to the presence of linear polynomials in $P^\htop$. \begin{corollary}\label{cor:maximalmonomial} Let $n\in \mathbb{N}$ with $n\ge 2$, and define $$Q^\htop_{j,n} := \left\{ y_{1,j}y_{2,j}, y_{2,j}y_{3,j}, \ldots, y_{n-1,j}y_{n,j}\right\} \cup \left\{y_{i,j}^2 \mid i=1,\ldots,n\right\} \subset \FFt[y_{1,j},\ldots,y_{n,j}],$$ for some $j\in \mathbb{N}$. If $n$ is even then there exists two monomials of maximal degree $\left\lceil\frac{n}{2} \right\rceil$ in $\FFt[y_{1,j},\ldots,y_{n,j}]/\langle Q^\htop_{j,n} \rangle$, namely \[ m_1 = \prod_{\substack{i=1,\ldots,n-1,\\ i\ \text{odd}}}y_{i,j} \quad \textnormal{and}\quad m_2 =\prod_{\substack{i=2,\ldots,n,\\ i\ \text{even}}}y_{i,j}. \] If $n$ is odd, then there exists a unique monomial of maximal degree $\left\lceil\frac{n}{2} \right\rceil$ in $\FFt[y_{1,j},\ldots,y_{n,j}]/\langle Q^\htop_{j,n} \rangle$, namely \[ m = \prod_{\substack{i=1,\ldots,n,\\ i\ \text{odd}}}y_{i,j}. \] \end{corollary} \noindent We are ready to prove the following theorem, which provides the degree of regularity of $Q$. \begin{theorem}\label{Thm:Dreg-of-Qtop} $$\dreg{Q}= \begin{cases} n + \ell_t n/2 + 1 \quad &\text{ if } n \equiv 0 \bmod 2\\ n + \ell_t(n+1)/2 + 1 \quad &\text{ if } n \equiv 1 \bmod 2 \end{cases}.$$ Equivalently, $$\dreg{Q} = n + \ell_t\lceil n/2 \rceil + 1.$$ \end{theorem} \begin{proof} Let $Q^\htop_{j,n} \subset \FFt[y_{1,j},\ldots,y_{n,j}]$ as in Corollary~\ref{cor:maximalmonomial}, for every $j=1,\ldots,\ell_t$. Observe that \begin{equation}\label{eq:qtopasunion} Q^\htop = \bigcup_{j=1}^{\ell_t} Q^\htop_{j,n} \cup \left\{x_i^2 \mid i=1,\ldots,n\right\}. \end{equation} Corollary~\ref{cor:maximalmonomial} computes a monomial $m_j \in \FFt[y_{1,j},\ldots,y_{n,j}]$ of maximal degree $\lceil n/2 \rceil$ such that $m_j \not \in \langle Q^\htop_h\rangle$ for every $j=1,\ldots,\ell_t$ and every $h=1,\ldots,\ell_t$. This implies that $m_j \not \in \langle Q^\htop \rangle$ for every $j$. It is now clear that the monomial \[ m:= \prod_{i=1}^n x_i \prod_{j=1}^{\ell_t}m_j \in \FFt[X,Y] \] is such that $m \not \in \langle Q^\htop \rangle$. Note that the the set $\left\{x_i^2 \mid i=1,\ldots,n\right\}$ in \eqref{eq:qtopasunion} enforces that $m$ must be squarefree in the variables $x_1,\ldots,x_n$. By the maximality of each $m_j$ and that of $\prod_{i=1}^n x_i$, any multiple of $m$ by a non-constant term would trivially be in $\langle Q^\htop \rangle$. Since $$d:=\deg m = n + \ell_t\lceil n/2 \rceil,$$ we have that the $(d+1)$-th coefficient of the Hilbert series of $\FFt[X,Y]/\langle Q^\htop \rangle$ is $0$. The result on the degree of regularity $\dreg{Q}$ follows. \qed \end{proof} \ifodd0 \begin{theorem}\label{Thm:Dreg-of-Qtop} $$\dreg{Q}= \begin{cases} 2n + (\ell-1)n/2 + 1 \quad &\text{ if } n \equiv 0 \bmod 2\\ 2n + (\ell-1)(n+1)/2 + 1 \quad &\text{ if } n \equiv 1 \bmod 2 \end{cases}.$$ Equivalently, $$\dreg{Q} = 2n + (\ell-1)\lceil n/2 \rceil + 1.$$ \end{theorem} \begin{proof} We set $R=\FFt[X,Y]/\langle Q^\htop \rangle$. We show that the maximum degree of a monomial of $R$ is $d := 2n + (l-1)\lceil n/2 \rceil$ via a constructive argument and show that any monomial of degree $d+1$ lives in $\langle Q^\htop \rangle$. From Lemma~\ref{lem:groebnerQh} and Remark~\ref{rem:qtopdef} a Gr\"obner basis of $\langle Q^\htop \rangle$ is $Q^\htop$ itself, i.e. the union of three sets of quadratic monomials, the first two being the squares of all the variables $\vx$ and $\vy$ and the third being the set of mixed monomials $$\left\{ y_{(i-1)\ell+j-1}y_{(i-2)\ell+j-1} \mid i=2,\ldots,n,\ j=2,\ldots,\ell\right\}.$$ According to the first two sets, to construct a monomial of $R$ we need to build a square free monomial, i.e. which is at most linear in each variable. The third set imposes restrictions on the variables that can be chosen to build the monomial. Assume that $n \equiv 0 \bmod 2$, the case with $n$ odd can be proven similarly, by a different choice of variables. Consider the set of variables $$V_1 := \left\{y_{(h(\ell-1)-2)\ell +j - 1} \mid h=1,\ldots,n/2,\ j=2,\ldots,\ell\right\}$$ and let $M_1:= \prod_{v\in V_1}v$. Furthermore, none of the variables in the set $$V_2 := \left\{x_i,y_{i\ell} \mid i=1,\ldots,n\right\}$$ appears in a mixed monomial. Let $M_2 := \prod_{v\in V_2}v$. Then $M := M_1M_2$ is a square-free monomial in $R$ of degree $d$. It is easy to verify that $M$ is not a multiple of any of the mixed monomials in the Gr\"obner basis of $\langle Q^\htop \rangle$ and that $d$ is maximal. In addition, any monomial of maximal degree can be constructed similarly. Indeed, any multiple of $M$ is also the multiple of a monomial in the Gr\"obner basis of $\langle Q^\htop \rangle$. To see this, note that, on the one hand, if we take a multiple of $yM$ with $v| M$, then $M$ is no longer square-free and thus $v^2 | M$ with $v^2$ lying on the Gr\"obner basis of $\langle Q^\htop \rangle$. On the other hand, the variables that do not appear in $M$ are exactly those in the set $$V_3 := \left\{y_{(h(\ell-1)-1)\ell +j - 1} \mid h=1,\ldots,n/2,\ j=2,\ldots,\ell\right\}.$$ Let $v \in V_3$, and consider the monomial $vM$. We can write $v = y_{(h(\ell-1)-1)\ell +j - 1}$ for some $h\in [1,n/2]$ and $j=2,\ldots,\ell$. By construction of $M$, we have that $vw | vM$ where $w = y_{(h(\ell-1)-2)\ell +j - 1}$. Setting $i = h(\ell -1)$ we find that the product $vw = y_{(i-1)\ell+j-1}y_{(i-2)\ell+j-1}$ is a mixed monomial on the Gr"obner basis of $\langle Q^\htop \rangle$ meaning that $vM \equiv 0$ in $R$. Finally, notice that any divisor of degree $d' < d$ of $M$ is a nonzero monomial of $R$. This proves that the $d$-th coefficient of the Hilbert series of $R$ is positive and that, by maximality of $d$, the $(d+1)$-th coefficient is the first $0$ coefficient of the series, proving our claim. \qed \end{proof} \begin{example} Let $n=8$ and $\ell_t=3$. According to Theorem~\ref{Thm:Dreg-of-Qtop} the degree of regularity of $ Q$ is $21$. \noindent Using MAGMA, we compute and report the Hilbert series of the quotient ring $\FFt[X,Y]/\langle Q^\htop\rangle$, i.e. \begin{align*} \mathrm{HS}_{\FFt[X,Y]/\langle Q^\htop\rangle} (z) =&\ 125z^{20} + 2500z^{19} + 23075z^{18} + 130800z^{17} + \\ &\ 511140z^{16} + 1465020z^{15} + 3198081z^{14} +\\ &\ 5448312z^{13} + 7360635z^{12} + 7966528z^{11} + \\ &\ 6946904z^{10} + 4889800z^9 + 2773415z^8 + \\ &\ 1260580z^7 + 454625z^6 + 128080z^5 + 27524z^4 + \\ &\ 4348z^3 + 475z^2 + 32z + 1, \end{align*} thus $\dreg{Q}=\deg \mathrm{HS}_{\FFt[X,Y]/\langle Q^\htop\rangle}+1=21$, matching our results. \end{example} \subsubsection{The linear polynomials.} In this section, we study how the degree of regularity computed in Theorem~\ref{Thm:Dreg-of-Qtop} changes when we add to the quadratic equations $Q$ also the fixed linear equations of $P$, which do not depend on the specific instance of the problem. Specifically, we compute the degree of regularity of $Q \cup P$. For this, we need to consider the ideal $\langle Q^\htop \cup P^\htop\rangle$. Note that this ideal contains $\langle Q^\htop \rangle$, which means that the variety of the former is a subset of the variety of the latter. In particular, the ideal $\langle Q^\htop \cup P^\htop\rangle$ is also zero-dimensional, so its degree of regularity is well-defined. We will use similar arguments to those applied to $\langle Q^\htop \rangle$ to study it. \begin{remark}\label{rem:qtopptopdef} The set $Q^\htop \cup P^\htop$ is the union of the following sets $$\left\{x_i^2 \mid i=1,\ldots,n\right\},\left\{x_i \mid i=1,\ldots,n\right\}, \left\{y_{i,j}^2 \mid i=1,\ldots,n,\ j=1,\ldots,\ell_t\right\},$$ $$\left\{y_{1,j} \mid j=2,\ldots,\ell_t\right\},\left\{y_{n,j} \mid j=1,\ldots,\ell_t\right\}$$ and $$\left\{ y_{i-1,j}y_{i,j} \mid i=2,\ldots,n,\ j=1,\ldots,\ell_t \right\}.$$ and the ideal $\langle Q^\htop \cup P^\htop \rangle \subseteq \FFt[X,Y]$ is thus a monomial ideal. \end{remark} Next lemma provides a Gr\"obner basis of the ideal $\langle Q^\htop \cup P^\htop \rangle \subseteq \FFt[X,Y]$. \begin{lemma}\label{lem:gbqtopptop} A Gr\"obner basis $G$ for $\langle Q^\htop \cup P^\htop \rangle \subseteq \FFt[X,Y]$ is $$G = \left\{x_i \mid i=1,\ldots,n\right\} \cup \left\{ y_{i-1,j}y_{i,j} \mid i=3,\ldots,n-1,\ j=1,\ldots,\ell_t \right\}\cup $$ $$\left\{ y_{1,1}y_{2,1}\right\}\cup\left\{y_{1,j} \mid j=2,\ldots,\ell_t\right\}\cup\left\{y_{n,j} \mid j=1,\ldots,\ell_t\right\}\cup$$ $$\left\{y_{i,j}^2 \mid i=2,\ldots,n-1,\ j=1,\ldots,\ell_t\right\} \cup \left\{y_{1,1}^2\right\}. $$ \end{lemma} \begin{proof} The proof of this statements follows directly from inspecting Remark~\ref{rem:qtopptopdef} and the same observations as in proof of Lemma~\ref{lem:groebnerQh}. \qed \end{proof} The next theorem gives the exact value of the degree of regularity of the system $Q \cup P$. The proof uses similar arguments to those used for the proof of Theorem~\ref{Thm:Dreg-of-Qtop}. \begin{theorem}\label{thm:dregQtopPtop} The degree of regularity of $Q \cup P$ is $$\dreg{Q \cup P} = \left\lceil\frac{n-1}{2} \right\rceil + (\ell_t - 1)\left\lceil\frac{n-2}{2} \right\rceil + 1.$$ \end{theorem} \begin{proof} Define the set \[ \tilde{Q}^\htop_{j,n} := Q^\htop_{j,n-1} \setminus \{y_{1,j}y_{2,j}\} \subset \FFt[y_{2,j},\ldots,y_{n-1,j}], \] for every $j=1,\ldots,\ell_t$. Let $G$ be a Gr\"obner basis of $\langle Q^\htop \cup P^\htop\rangle$ as in Lemma~\ref{lem:gbqtopptop}. Due to the presence of the linear monomials contributed by $P^\htop$, we observe \begin{equation}\label{eq:qtopptopasunion} G = Q^\htop_{1,n-1} \cup \bigcup_{j=2}^{\ell_t} \tilde{Q}^\htop_{j,n-1} \cup \left\{x_i^2 \mid i=1,\ldots,n\right\}. \end{equation} Applying Corollary~\ref{cor:maximalmonomial}, we can get a monomial $m_1 \in \FFt[y_{1,1},\ldots,y_{n-1,1}]$ of maximal degree $\deg m_1 = \lceil (n-1)/2 \rceil$ such that $m_1 \not \in \FFt[y_{1,1},\ldots,y_{n-1,1}]/\langle Q^\htop_{1,n-1} \rangle$. We can obtain other $\ell_t - 1$ monomials $m_j$ of maximal degree $d = \lceil (n-2)/2 \rceil$, such that $m_j \not \in \langle \tilde{Q}^\htop_{h,n-1} \rangle$ for every $h=1,\ldots,\ell_t$ and every $j = 2,\ldots,\ell_t$. Let now $$m := \prod_{j=1}^{\ell_t}m_j \in \FFt[X,Y]/\langle G\rangle$$ then $$d:=\deg m = \left\lceil\frac{n-1}{2} \right\rceil + (\ell_t - 1)\left\lceil\frac{n-2}{2} \right\rceil,$$ meaning that the $(d+1)$-th coefficient of the Hilbert series of $\FFt[X,Y]/\langle G \rangle$ is $0$. The result on the degree of regularity $\dreg{Q\cup P}$ follows. \qed \end{proof} \begin{example} Let $n=8$ and $\ell_t=3$. According to Theorem~\ref{thm:dregQtopPtop} the degree of regularity of $ Q \cup P$ is $11$. \noindent Using MAGMA, we compute and report the Hilbert series of the quotient ring $\FFt[X,Y]/\langle Q^\htop \cup P^\htop\rangle$, i.e. \begin{align*} \mathrm{HS}_{\FFt[X,Y]/\langle Q^\htop \cup P^\htop\rangle} (z) =&\ 16z^{10} + 240z^9 + 1188z^8 + 2920z^7 + 4132z^6 + 3608z^5 +\\ &\ 2005z^4 + 710z^3 + 155z^2 + 19z + 1, \end{align*} thus $\dreg{Q\cup P}=\deg \mathrm{HS}_{\FFt[X,Y]/\langle Q^\htop \cup P^\htop\rangle}+1=11$, matching our results. \end{example} \begin{remark}\label{rem:range fordregS} Since Theorem~\ref{thm:dregQtopPtop} considers the set $Q^\htop \cup P^\htop = S^\htop \setminus L_\vH^\htop$, it only gives an upper bound to the degree of regularity of $S$, that is $$\dreg{S}\leq \left\lceil\frac{n-1}{2} \right\rceil + (\ell_t - 1)\left\lceil\frac{n-2}{2} \right\rceil + 1.$$ \end{remark} In the next section, we provide some experimental data showing the gap between the value computed in Theorem~\ref{thm:dregQtopPtop} and that of the actual solving degree. \subsection{Experimental results} We performed several experiments for Modeling~\ref{modeling: improvedESD_F2} taking as input both random and Goppa codes, and we obtained a solving degree which is much smaller than the upper bound for the degree of regularity computed in Theorem~\ref{thm:dregQtopPtop}. This results in a much lower complexity estimate. We provide a selection of our experiments in Table~\ref{Tab:q2-SolveDeg}. The MAGMA code used for our experiments can be found \href{https://github.com/rexos/phd-cryptography-code/blob/main/modelings/Modeling2.txt}{here}. \vspace{-2mm} \input{q2-SD-examples} \subsubsection{Comparison with combinatorial attacks.} We also compare the complexity of solving ESDP using Gr\"obner basis techniques with the more traditional combinatorial attack proposed by Prange. The latter is widely regarded as a reference algorithm for decoding linear codes in the Hamming metric, and forms the basis of many state-of-the-art attacks on code-based cryptosystems. However, Gröbner basis computations are known to be computationally expensive, particularly as the number of variables and the degree of the polynomials grow. In contrast, combinatorial methods like Prange exploit structured randomization and search to solve the problem more efficiently, albeit with large memory requirements. In Table~\ref{Tab:q2-SolveDeg}, the complexity of the Prange algorithm has been computed using the Esser-Bellini cryptographic estimator~\cite{PKC:EssBel22}, see also this~\href{https://estimators.crypto.tii.ae/configuration?id=SDEstimator}{link}. On the other hand, the complexity of solving the polynomial system of Modeling~\ref{modeling: improvedESD_F2} with Gr\"obner bases methods has been computed using formula~\eqref{eq:GBcomplexity} with $N = n(\lfloor\log_2(t)\rfloor + 2)$, the highest step degree $d_M$ achieved in the MAGMA experiments as a proxy for the solving degree $d_{\mathrm{sol}}$ and the conservative matrix multiplication constant $\omega = 2.807$. These results confirm that direct combinatorial attacks like Prange outperform algebraic methods when solving the syndrome decoding problem, particularly for parameters of practical interest in code-based cryptography. Despite this, algebraic approaches remain valuable from a theoretical perspective and may offer insights into alternative solution strategies that could be leveraged in other contexts. We include this comparison to emphasize that the purpose of our Gröbner basis approach is not to compete with combinatorial attacks in efficiency, but rather to provide an alternative algebraic perspective on the syndrome decoding problem. Such perspectives can contribute to understanding the hardness of related problems in post-quantum cryptography. \input{ExtensionFields.tex} \subsection{Experimental results.} We solved the quadratic system associated with Modeling~\ref{modeling: quadBSD_Fq} for several random codes. We show in Table~\ref{Tab:qNEQ2-SolveDeg} that, similarly to the case over $\FFt$, the solving degree is surprisingly small. MAGMA code used for our experiments with Modeling~\ref{modeling: quadBSD_Fq} can be found at \href{https://github.com/rexos/phd-cryptography-code/blob/main/modelings/Modeling5.magma}{this link}. \input{qNOT2-SD-examples} \section{Conclusion and Future Directions} We have presented a new algebraic cryptanalysis for both the bounded and the exact versions of the Syndrome Decoding problem. In the binary case, our modelings significantly improved the previous attempt of \cite{2021/meneghetti}, by capturing the weight condition on the solution vector with quadratic polynomials. We have also experimentally shown that the behavior of the associated Gr\"obner basis is very different from that of a random system with the same number of variables and equations, \textit{leading to a much better complexity}. We have thus taken an important step towards making algebraic algorithms potentially competitive for the decoding problem. We introduced algebraic modelings for the first time in the case of the general syndrome decoding problem over larger finite fields. Notably, one of them is quadratic with a number of variables and equations that is linear or quasi-linear in the code length, \textit{independently from the field size}. We have analyzed that, despite the constant degree of the equations involved, the system correctly solves the decoding problem and with high probability does not have spurious solutions for all parameters that are relevant to the problem. \vskip 0.5cm An open question to this work is to understand more clearly the behavior of the Gr\"obner basis computation both in the binary and in the general finite field cases and to get a theoretical estimate of the complexity that better matches with the one obtained from the experiments. This is a difficult task, as it is often the case for very structured algebraic systems, and probably requires to develop dedicated tools to analyze such behavior. Another interesting and natural follow-up to this work can be to analyze the impact of hybrid strategies on solving the proposed systems. Since the weight of the solution sought is relatively low, a convenient choice is to set most of the variables of $X$ to 0. It is not difficult to see that this approach is reminiscent of the guess part in Prange or later ISD algorithms. In the case of binary systems, we have verified experimentally that the best hybrid trade-off actually boils down to the Prange algorithm, the best complexity being indeed obtained when enough zeros to linearize the system are guessed. However, the system hides a lot of structure and offers many different ways to specialize variables. For example, the auxiliary variables from the vector $\vy$ can also be fixed and they too have different probabilities of taking a value equal to 0 or 1. It is therefore not at all unrealistic to speculate that an ad-hoc and smart hybridization technique may lead to a better trade-off than a fully combinatorial approach. \subsection*{Acknowledgments.} We would like to thank the reviewers for their detailed and valuable feedback. Additionally, special thanks to Magali Bardet, Tanja Lange and Alberto Ravagnani for the fruitful discussions and insights. This publication was created with the co-financing of the European Union FSE-REACT-EU, PON Research and Innovation 2014-2020 DM1062/2021. A. Caminata is supported by the PRIN 2020 grant 2020355B8Y ``Squarefree Gr\"obner degenerations, special varieties and related topics'', by the PRIN PNRR 2022 grant P2022J4HRR ``Mathematical Primitives for Post Quantum Digital Signatures'', by the MUR Excellence Department Project awarded to Dipartimento di Matematica, Università di Genova, CUP D33C23001110001, and by the European Union within the program NextGenerationEU. A. Meneghetti acknowledges support from Ripple's University Blockchain Research Initiative. A. Caminata and A. Meneghetti are members of the INdAM Research Group GNSAGA. \bibliography{biblio.bib,crypto,abbrev0} \bibliographystyle{splncs04} \appendix \section{Section~\ref{Sec:Dim_of_Var} Bounds on the zero-dimensionality}\label{app:Dim_Exp} \begin{table}[H] \centering \resizebox{0.9\textwidth}{!}{\begin{tabular}{|c ||c c c c||} \hline $[n,k]_q$& $t=\lfloor (d_{GV}-1)/2\rfloor$ & $t=\lfloor (n-k)/2\rfloor$ & $t=d_{GV}$ & $t=d_{GV}+1$ \\ \hline $[100,50]_{2}$ & \makecell{$t=5,$ \\ $\pr\le 2.32\cdot 10^{-20} $} & \makecell{$t= 25,$ \\ $\pr\le 1 $} & \makecell{$t=12 ,$ \\ $\pr\le 6.73\cdot 10^{-9} $} & \makecell{$t=13 ,$ \\ $\pr\le 1.84\cdot 10^{-7} $} \\ \hline $[100,50]_{7}$ & \makecell{$t= 12 , $ \\ $ \pr\le 8.44\cdot 10^{-51} $} & \makecell{$t= 25 , $ \\ $ \pr\le 1.89 \cdot 10^{-20} $} & \makecell{$t= 25 , $ \\ $ \pr\le 1.89 \cdot 10^{-20} $} & \makecell{$t= 26 , $ \\ $ \pr\le 2.68 \cdot 10^{-18}$} \\ \hline $[100,50]_{127}$ & \makecell{$t= 18 , $ \\ $ \pr\le 5.48 \cdot 10^{-118} $} & \makecell{$t= 25 , $ \\ $ \pr\le 1.23 \cdot 10^{-84} $} & \makecell{$t= 37 , $ \\ $ \pr\le 5.38 \cdot 10^{-30} $} & \makecell{$t= 38 , $ \\ $ \pr\le 1.44 \cdot 10^{-25} $} \\ \hline $[100,80]_{2}$ & \makecell{$t= 1 , $ \\ $ \pr\le 9.09 \cdot 10^{-11} $} & \makecell{$t= 10 , $ \\ $ \pr\le 1 $} & \makecell{$t= 4 , $ \\ $ \pr\le 3.14\cdot 10^{-4} $} & \makecell{$t= 5 , $ \\ $ \pr\le 0.0268$} \\ \hline $[100,80]_{7}$ & \makecell{$t= 3 , $ \\ $ \pr\le 4.06 \cdot 10^{-25} $} & \makecell{$t= 10 , $ \\ $ \pr\le 2.92 \cdot 10^{-5} $} & \makecell{$t= 8 , $ \\ $ \pr\le 1.30 \cdot 10^{-10} $} & \makecell{$t= 9 , $ \\ $ \pr\le 6.55 \cdot 10^{-8} $} \\ \hline $[100,80]_{127}$ & \makecell{$t= 6 , $ \\ $ \pr\le 1.16 \cdot 10^{-52} $} & \makecell{$t= 10 , $ \\ $ \pr\le 1.14 \cdot 10^{-31} $} & \makecell{$t= 13 , $ \\ $ \pr\le 1.97 \cdot 10^{-16} $} & \makecell{$t= 14 , $ \\ $ \pr\le 1.97 \cdot 10^{-11} $} \\ \hline \end{tabular} } \vspace{2mm} \caption{Bound on the probability $\pr$ that the ideal associated with the system has a strictly positive dimension for the decoding problem with a randomly sampled syndrome.} \label{table: bound_SD} \end{table} \vspace{-.5in} \begin{table}[H] \centering \resizebox{0.8\textwidth}{!}{\begin{tabular}{|c ||c c c c c||} \hline $[n,k]_q$& $t=\lfloor (d_{GV}-1)/2\rfloor$ & $t=\lfloor (n-k)/2\rfloor$ & $t=d_{GV}-2$ & $t=d_{GV}-1$ & $t=d_{GV}$\\ \hline $[ 100 , 50 ]_{ 2 }$ & \makecell{$t= 5 ,$ \\ $ \pr\le 2.07\cdot 10^{-6} $} & \makecell{$t= 25 ,$ \\ $ \pr\le 1 $} & \makecell{$t= 10 ,$ \\ $ \pr\le 1 $} & \makecell{$t= 11 ,$ \\ $ \pr\le 1 $} & \makecell{$t= 12 ,$ \\ $ \pr\le 1 $} \\ \hline $[ 100 , 50 ]_{ 7 }$ & \makecell{$t= 12 ,$ \\ $ \pr\le 8.08\cdot 10^{-18} $} & \makecell{$t= 25 ,$ \\ $ \pr\le 1 $} & \makecell{$t= 23 ,$ \\ $ \pr\le 0.378 $} & \makecell{$t= 24 ,$ \\ $ \pr\le 1 $} & \makecell{$t= 25 ,$ \\ $ \pr\le 1 $} \\ \hline $[ 100 , 50 ]_{ 127 }$ & \makecell{$t= 18 ,$ \\ $ \pr\le 1.46\cdot 10^{-48} $} & \makecell{$t= 25 ,$ \\ $ \pr\le 6.16\cdot 10^{-30} $} & \makecell{$t= 35 ,$ \\ $ \pr\le 3.04\cdot 10^{-5} $} & \makecell{$t= 36 ,$ \\ $ \pr\le 0.00696 $} & \makecell{$t= 37 ,$ \\ $ \pr\le 1 $} \\ \hline $[ 100 , 80 ]_{ 2 }$ & \makecell{$t= 1 ,$ \\ $ \pr\le 9.54\cdot 10^{-5} $} & \makecell{$t= 10 ,$ \\ $ \pr\le 1 $} & \makecell{$t= 2 ,$ \\ $ \pr\le 0.0142 $} & \makecell{$t= 3 ,$ \\ $ \pr\le 1 $} & \makecell{$t= 4 ,$ \\ $ \pr\le 1 $} \\ \hline $[ 100 , 80 ]_{ 7 }$ & \makecell{$t= 3 ,$ \\ $ \pr\le 6.93\cdot 10^{-10} $} & \makecell{$t= 10 ,$ \\ $ \pr\le 1 $} & \makecell{$t= 6 ,$ \\ $ \pr\le 0.00176 $} & \makecell{$t= 7 ,$ \\ $ \pr\le 0.165 $} & \makecell{$t= 8 ,$ \\ $ \pr\le 1 $} \\ \hline $[ 100 , 80 ]_{ 127 }$ & \makecell{$t= 6 ,$ \\ $ \pr\le 4.20\cdot 10^{-21} $} & \makecell{$t= 10 ,$ \\ $ \pr\le 1.59\cdot 10^{-8} $} & \makecell{$t= 11 ,$ \\ $ \pr\le 1.65\cdot 10^{-5} $} & \makecell{$t= 12 ,$ \\ $ \pr\le 0.0155$} & \makecell{$t= 13 ,$ \\ $ \pr\le 1 $} \\ \hline \end{tabular} } \vspace{2mm} \caption{Bound on the probability $\pr$ that the ideal associated with the system has strictly positive dimension for the codeword finding problem (i.e. with null syndrome).} \label{table: bound_CF} \end{table} \end{document} \begin{table}[H] \centering \begin{tabular}{|c|c|c||c|c|c|c|c|c|c|c|} \hline \cellcolor{gray!20!}$n$& \cellcolor{gray!20!}$k$&\cellcolor{gray!20!}$t$& \cellcolor{gray!20!}Code Type&\cellcolor{gray!20!} \# Eqs &\cellcolor{gray!20!} \# Vars& \cellcolor{gray!20!}$d_{\mathrm{reg}}$ &\cellcolor{gray!20!} SR $d_{\mathrm{reg}}$&\cellcolor{gray!20!} $d_{\mathrm{M}}$&\cellcolor{gray!20!}Prange&\cellcolor{gray!20!} Modeling~\ref{modeling: improvedESD_F2} \\ \hline 8 & 2 & 2 & Goppa & (17,38) & 24 & $\leq8$ & 3 & 2 & 9 & 23 \\ \hline 10 & 5 & 4 & Random* & (20,67) & 40 & $\leq14$ & 5 & 3 & 10 & 38 \\ \hline 16 & 8 & 2 & Goppa & (27,78) & 48 & $\leq16$ & 5 & 3 & 12 & 40 \\ \hline 20 & 10 & 5 & Random* & (35,137) & 80 & $\leq29$ & 7 & 3 & 12 & 46 \\ \hline 30 & 15 & 7 & Random* & (50,207) & 120 & $\leq44$ & 10 & 4 & 15 & 65 \\ \hline 32 & 12 & 4 & Goppa & (57,221) & 128 & $\leq 47$ & 10 & 3 & 16 & 52 \\ \hline 32 & 17 & 3 & Goppa & (50,158) & 96 & $\leq32$ & 5 & 4 & 16 & 61 \\ \hline 32 & 22 & 2 & Goppa & (45,158) & 96 & $\leq 32$ & 7 & 3 & 15 & 48 \\ \hline 40 & 20 & 8 & Random* & (67,356) & 160 & $\leq78$ & 16 & 5 & 17 & 88 \\ \hline 50 & 30 & 5 & Random & (75,347) & 200 & $\leq74$ & 15& 4 & 20 & 73\\ \hline 50 & 40 & 4 & Random & (65,347) & 200 & $\leq74$ & 17& 4 & 14 & 73\\ \hline 64 & 52 & 2 & Goppa & (79,318) & 192 & $\leq64$ & 14 & 4 & 18 & 72\\ \hline 64 & 40 & 4 & Goppa & (93,445) & 256 & $\leq95$ & 19 & 4 & 21 & 77\\ \hline 64 & 16 & 8 & Goppa & (119,572) & 320 & $\leq126$ & 21 & 4 & 19 & 81\\ \hline \end{tabular} \vspace{2mm} \caption{This table shows experimental results for $\mathbb{F}_2$-linear codes using Modeling~\ref{modeling: improvedESD_F2} for the ESDP. The number of equations is given in the format (\#linear equations, \#quadratic equations). The values in the $d_{\mathrm{M}}$ column represent the smallest degree D such that MAGMA function GroebnerBasis(F,D) gives the Gr\"obner basis, i.e. highest step degree achieved when directly computing the Gröbner basis of the system in MAGMA, but ignoring the unnecessary steps in high degree that MAGMA F4 algorithm may do to insure termination. The SR $d_{reg}$ column gives the degree of regularity of a semi-regular system of equations with the associated number of linear equations, quadratic equations, and variables, using \cite[Corollary 3.3.8]{B04}. The values in $d_{\mathrm{reg}}$ column are upper bounds for the degree of regularity of the system as provided by Theorem~\ref{thm:dregQtopPtop} and Remark~\ref{rem:range fordregS}. Random codes with ``*'' are decoding challenges from \url{https://decodingchallenge.org/syndrome}, with a number of errors slightly above Gilbert-Varshamov distance. The other random code instances are below GV distance instead. Instances with ``Goppa'' Code Type are random full-length binary Goppa codes with a number of errors equal to the Goppa polynomial degree. The Prange and Modeling 2 columns state the $\log_2$ of the complexities of the Prange algorithm (Esser-Bellini estimator~\cite{PKC:EssBel22}) and Gr\"obner basis computations \eqref{eq:GBcomplexity} with $d_{\mathrm{M}}$, respectively. } \label{Tab:q2-SolveDeg} \end{table}\vspace{-.2in} \section{Modelings over $\FF_q$} \label{sec:Fq} Each of the modelings we have discussed thus far (MPS, Modeling \ref{modeling: improvedSD_F2}, and Modeling \ref{modeling: improvedESD_F2}) are limited to the binary case. To the best of our knowledge, there is no modeling of the general syndrome decoding problem over $\FF_q$ for $q>2$ in the literature. In this section, we adapt the previous modelings to a generic finite field $\FF_q$, for some prime power $q\ge 2$, and explain how to efficiently (i.e. in polynomial time) obtain a polynomial system encoding an instance of the Syndrome Decoding Problem. We will adopt the following notation throughout this section. \begin{notation}\label{FQnotation} Let $n\geq2$ and let $\CC \subseteq \FF_q^n$ be a $[n,k,d]$-linear code having a parity check matrix $\HH \in \FF_q^{(n-k) \times n}$. The vector $\vs \in \FF_q^{n-k}$ denotes the syndrome and $0\le t \le \lfloor (d-1)/2 \rfloor$ is the target error weight. Let $r_1,r_2>0$ be two integers. We will work over the polynomial ring $\FF_{q^{r_1}}[X,Y,Z]$, where $X=(x_1,\dots,x_n)$, $Y=(Y_1,\dots,Y_n)$, $Y_j=(y_{j,1}, \dots, y_{j,r_2})$, and $Z=(z_1,\dots,z_n)$ are variables. \end{notation} As in the previous sections, $\vx=(x_1,\dots,x_n)$ is the vector of variables corresponding to the solution of the syndrome decoding problem. On the other hand, the role of the integers $r_1,r_2$ and of the variables $Y$ and $Z$ will be illustrated later. We separately describe and explain the sets of polynomials that, together, model Problems~\ref{BSDP} and \ref{EWSDP}. Then we provide an analysis of the correctness of our modelings. \subsection{Construction of the Equations} \subsubsection{Identifying the support of a vector of $\mathbb{F}_q^n$.} Unlike the Boolean case, the value of an element in the support of $\vx$ is not uniquely determined when $q\ge 3$. In order to count the number of nonzero coordinates with algebraic equations, we first need to map all nonzero elements to a unique element of $\FFq^*$, say 1. Thus, in addition to the $Y$'s variables encoding the partial Hamming weights, we introduce here another length-$n$ vector of variables $Z=(z_1,\dots,z_n)$, each of which can only assume two values, 0 or 1, depending on whether the corresponding $X$ coordinate is nonzero. First, we tackle the problem of describing the relation between $X$ and $Z$ through algebraic equations. We distinguish two cases, depending on the target version of the problem, and then prove the sets of polynomials correctly describe our target. \begin{itemize} \item \textit{Support constraint encoding for Problem~\ref{BSDP}.} Compute the following set of $2n$ quadratic polynomials \begin{equation} \label{eq:sceBSD} \{ x_j(z_j-1) \mid j=1,\dots,n\} \cup \{ z_j^2-z_j \mid j=1,\dots,n\}. \end{equation} \item \textit{Support constraint encoding for Problem~\ref{EWSDP}.} Compute the following set of $n$ polynomials of degree $q-1$ \begin{equation} \label{eq:sceEWSD} \{z_j-x_j^{q-1}\mid j=1,\dots,n\}. \end{equation} \end{itemize} In the first case, the condition $z_j=1$ if $x_j\ne 0$ is given from the first set of polynomials. Otherwise, the second set implies $z_j\in\{0,1\}$. Therefore, the support of $(z_1,\ldots,z_n)$ contains the support of $(x_1,\ldots,x_n)$ and thus $\wt((z_1,\ldots,z_n))\ge \wt((x_1,\ldots,x_n))$. In the second case, in order for the corresponding equations to be satisfied, $z_j=1$ if and only if $x_j\ne 0$, and $z_j=0$ otherwise. Hence $\wt((z_1,\ldots,z_n))=\wt((x_1,\ldots,x_n))$. From a computational point of view, the support constraint encoding for Problem~\ref{EWSDP} has a strong limitation, that is the high degree of the polynomials. A Gr\"obner basis computation would need to reach at least degree $q-1$ before taking into account such polynomials, leading to infeasible calculations unless $q$ is very small. This is reminiscent of the problem of including field equations in modelings over large fields. Yet, this issue does not appear in the support constraint encoding for Problem~\ref{BSDP}: the polynomials have constant degree 2 regardless of the field size $q$, making a modeling for Problem~\ref{BSDP} more realistic and valuable for effective computations. \subsubsection{Hamming weight computation encoding.} A difficulty arising from a direct generalization to large fields of the previous approach is the update of the weight registers, i.e. of the vectors $\vy_i$'s. In order to overcome this limitation, we introduce a different strategy for encoding the partial weights. More precisely, we substitute their binary expansion with vectors from a linear recurring sequence over an extension of $\FF_q$. As we will see, this approach naturally allows for the choice of different trade-offs between the number of variables and finite field size. We first recall some known results about (univariate) polynomials over finite fields, companion matrices, and linear recurring sequences. We mainly refer to \cite{LN94} for this part. \begin{definition}[Companion matrix, Chapter 2, \S5 \cite{LN94}] Let $f(x)=x^d+f_{d-1}x^{d-1}+\dots+f_0\in \FF_q[x]$ be a monic polynomial. Its companion matrix is \begin{equation} \label{eq: companion} \Cf = \begin{bmatrix} 0 & 0 & \cdots & 0 & -f_0\\ 1 & 0 & \cdots & 0 & -f_1\\ 0 & 1 & \cdots & 0 & -f_2\\ \vdots & \vdots& \ddots &\vdots &\vdots\\ 0 & 0 & \cdots & 1 & -f_{d-1} \end{bmatrix}. \end{equation} \end{definition} It is well known that the equation $f(\Cf)=0$ is satisfied, hence, if $f$ is a monic irreducible polynomial over $\FF_q$, then its companion matrix $\Cf$ plays the role of a root of $f$. It follows that the elements of the extension field $\FF_{q^d}$ can be written, according to this representation, as polynomials in $\Cf$ of degree strictly less than $d$. We also recall that the order of a non-constant polynomial $f$ with $f_0\neq 0$ is the least positive integer $e$ such that $f(x) \mid x^e-1$. A polynomial in $\FF_q[x]$ of degree $d$ is said primitive if it is monic, $f(0)\ne 0$, and $\ord(f)=q^d-1$. Such polynomials can be found from the factorization of $x^{q^d-1}-1$. The theory of linear recurring sequences says that the sequence of vectors $\vy_0,\Cf \vy_0,\Cf^2 \vy_0,\dots$, for some nonzero $\vy_0$ and companion matrix with $f_0\ne 0$, is periodic with least period equal to the order of $f$, when the latter is irreducible (cf. \cite[Theorem 6.28]{LN94}). Therefore, by choosing $f$ primitive, we obtain a sequence of vectors $\vy_0,\Cf \vy_0,\Cf^2 \vy_0,\dots$ of maximal order $q^d-1$. On the other hand, the choice of $\vy_0$ does not seem to affect any property of our modeling. Without loss of generality, from now on we fix the initial state vector \begin{equation} \vy_0=\begin{pmatrix} 1&0& \cdots &0 \end{pmatrix}^\top. \end{equation} By tuning the values $r_1$ and $r_2$ we can choose the number of variables used for the Hamming weight computation encoding, at the cost of working over more or less large field extensions. More precisely, take \[ m:= \min \{i \in \mathbb{N} \mid q^i > \max(t, n-t)+1 \}, \] and let $r_1,r_2$ be two positive integers such that $m\le r_1 r_2$. Then, let $f\in \FF_{q^{r_1}}[x]$ be a primitive polynomial of degree $r_2$ and $\Cf \in \FF_{q^{r_1}}^{r_2\times r_2}$ its companion matrix. For convenience sake, we will use the column vector notation for the $Y_j$'s blocks of variables, i.e. $$Y_j= \begin{pmatrix} y_{j,1}& \cdots& y_{j,r_2} \end{pmatrix}^\top .$$ The polynomial encoding the partial Hamming weight is the following. \begin{itemize} \item \textit{Hamming weight computation encoding.} Compute the $n r_2$ affine bilinear (in $Y$ and $Z$) polynomials from the expansion of \begin{equation} \label{eq: hwceFqinit} Y_1-(1-z_1)\cdot \vy_0-z_1\cdot\Cf\cdot \vy_0 \end{equation} and \begin{equation} \label{eq: hwceFq} \{Y_j-(1-z_j)\cdot Y_{j-1}-z_j\cdot\Cf\cdot Y_{j-1} \;,\qquad \mathrm{for}\;j\in\{2,\ldots,n\}\}. \end{equation} \end{itemize} \begin{remark} Using this approach, $r_1$ determines the finite field over which we define the resulting polynomial system (and thus the Multivariate Quadratic Problem instance), while $r_2$ determines the number of variables required for the weight computation encoding (which is strictly linked to the computational complexity). Observe that, together, equations \eqref{eq: hwceFqinit} and \eqref{eq: hwceFq} correspond to $nr_2$ polynomial equations over $\FF_{q^{r_1}}$ in $nr_2+n$ variables. In several cases, working with a small amount of variables (namely, $r_1$ large and $r_2$ small) is preferable, but there are some instances in which working over small finite fields with a large number of variables (i.e. $r_1$ small and $r_2$ large) is advantageous. \end{remark} The next result shows that the Hamming weight of $\vz$ is correctly computed. \begin{proposition}\label{prop: correctness} Consider the system given by \eqref{eq: hwceFqinit} and \eqref{eq: hwceFq} over $\FF_{q^{r_1}}[Y, Z]$. Any solution $(\vy, \vz)=(\vy_1,\dots,\vy_n,\vz)\in \FF_{q^{r_1}}^{r_2 n}\times \{0,1\}^n$ of the system satisfies \[\vy_j=\Cf^{\wt(\pi_j(\vz))}\vy_0.\] In particular, $\vy_n=\Cf^{\wt(\vz)}\vy_0.$ \end{proposition} \begin{proof} It follows directly from the hypotheses by an inductive argument. \\ The first step is considering $\vy_1:=(y_{1,1},\ldots,y_{1,r_2})^{\top}$, which by definition is $$ \vy_1=(1-z_1)\vy_0 + z_1 \Cf \vy_0= \left\{ \begin{array}{rl} \vy_0&\mathrm{if}\; z_1=0\\ \Cf \vy_0&\mathrm{if}\; z_1\neq 0 \end{array}\;, \right. $$ namely, $\vy_1=\Cf^{\wt(z_1)}\vy_0=\Cf^{\wt(\pi_1(\vz))}\vy_0$. \\ For the inductive step, we consider $\vy_{j-1}:=(y_{j-1,1},\ldots,y_{j-1,r_2})^{\top}$ to be equal to $\Cf^{\wt(\pi_{j-1}(\vz))}\vy_0$, and we look at the definition of $\vy_j$. We have $$ \vy_j=(1-z_j)\vy_{j-1}+z_j \Cf \vy_{j-1}=\left\{ \begin{array}{cl} \vy_{j-1}&\mathrm{if}\; z_j=0\\ \Cf \vy_{j-1}&\mathrm{if}\; z_j\neq 0 \end{array}\;, \right. $$ and either way, we obtain $$ \vy_j=\Cf^{z_j}\cdot \vy_{j-1}=\Cf^{z_j}\Cf^{\wt(\pi_{j-1}(\vz))}\vy_0=\Cf^{\wt(z_j)+\wt(\pi_{j-1}(\vz))}\vy_0=\Cf^{\wt(\pi_{j}(\vz))}\vy_0.$$ \qed \end{proof} \subsubsection{Weight constraint encoding.} As for the previous modelings, the weight constraint encoding simply ensures that the representation of the last partial Hamming weight coincides with the representation of the total Hamming weight. \begin{itemize} \item \textit{Weight constraint encoding.} Compute the $r_2$ affine linear polynomials in $Y$ from the expansion of \begin{equation} \label{eq: wceFq} \vy_n-\Cf^{t} \vy_0 . \end{equation} \end{itemize} \begin{corollary} \label{cor: Z} Consider the system given by \eqref{eq: hwceFqinit}, \eqref{eq: hwceFq} and \eqref{eq: wceFq} over $\FF_{q^{r_1}}[Y, Z]$ and let $z_1,\ldots,z_n$ be either $0$ or $1$. Then \begin{enumerate} \item The number of solutions $(\vy, \vz)\in \FF_{q^{r_1}}^{r_2 n}\times \{0,1\}^n$ of the system is equal to the number of binary vectors of Hamming weight equal to $t$, i.e. $\binom{n}{t}$; \item If $(\bar{\vy},\bar{\vz})$ and $(\tilde{\vy},\tilde{\vz})$ are two distinct solutions, then $\bar{\vz}\ne \tilde{\vz}$. \end{enumerate} \end{corollary} \begin{proof} From Proposition~\ref{prop: correctness} and \eqref{eq: wceFq}, it follows that any solution $(\vy, \vz)$ satisfies \[ \Cf^t\vy_0= \begin{pmatrix} y_{n,1}\\ \vdots\\ y_{n,r_2} \end{pmatrix} =\Cf^{\wt(\vz)}\vy_0, \] which implies $\ord(f) \mid \lvert t-\wt{(\vz)}\rvert$. Since $f$ is primitive, we obtain \[ \ord(f)=(q^{r_1})^{r_2} -1 \ge q^m - 1> \max(t,n-t). \] On the other hand, $0\le \wt(\vz)\le n$, hence $\lvert t-\wt{(\vz)}\rvert \le \min(t,n-t)$. Therefore, the only way $\ord(f)$ can divide $\lvert t-\wt{(\vz)}\rvert$, is that $\lvert t-\wt{(\vz)}\rvert=0$, i.e. $\wt(\vz)=t$. Substituting $\vz$ in \eqref{eq: hwceFqinit} uniquely determines all the values $y_{1,1},\dots,y_{1,r_2}$ by linear equations of the form $y_{1,i}=c_i$. Moreover, substituting $y_{j-1,1},\dots,y_{j-1,r_2}$ in \eqref{eq: hwceFq} recursively determines all the values $y_{j,1},\dots,y_{j,r_2}$ in a similar manner. Therefore, if $\bar{\vz}= \tilde{\vz}$ are the projections over the last $n$ coordinates of two solutions, then also $(\bar{\vy},\bar{\vz})=(\tilde{\vy},\tilde{\vz})$, which concludes the proof. \qed \end{proof} \subsubsection{Field equations.} The field equations concern only the $X$ part of the variables. \begin{itemize} \item \textit{Field equations.} The equations are obtained from the $n$ polynomials \begin{equation} \label{eq: ffe_Fq} \{x_j^q-x_j \mid j =1,\dots,n\}. \end{equation} \end{itemize} Indeed, $\vz$ already lies over $\{0,1\}^n$ because of the support constraint equations (and \eqref{eq: ffe_Fq}), while \eqref{eq: hwceFqinit} and \eqref{eq: hwceFq} force $Y$ to lie over $\FF_{q^{r_1}}$. \subsection{The Modelings} We are finally ready to describe the algebraic systems over $\FF_{q^{r_1}}[X,Y,Z]$ for Problems~\ref{EWSDP} and \ref{BSDP} and prove their correctness. \begin{modeling}[Modeling for the SDP over $\FF_q$] \label{modeling: BSD_Fq} Given an instance $(\HH,\mathbf{s},t)$ of Problem~\ref{BSDP} over $\FF_q$, Modeling~\ref{modeling: BSD_Fq} is the union of the sets of polynomials \eqref{eq:pce}, \eqref{eq:sceBSD}, \eqref{eq: hwceFqinit}, \eqref{eq: hwceFq}, \eqref{eq: wceFq} and \eqref{eq: ffe_Fq}. \end{modeling} \begin{modeling}[Modeling for the ESDP over $\FF_q$] \label{modeling: EWSD_Fq} Given an instance $(\HH,\mathbf{s},t)$ of Problem~\ref{EWSDP} over $\FF_q$, Modeling~\ref{modeling: EWSD_Fq} is the union of the sets of polynomials \eqref{eq:pce}, \eqref{eq:sceEWSD}, \eqref{eq: hwceFqinit}, \eqref{eq: hwceFq}, \eqref{eq: wceFq} and \eqref{eq: ffe_Fq}. \end{modeling} As already said, finite field equations cannot be efficiently taken into account when dealing with large fields. In the exact weight syndrome decoding modeling, the support constraint equations are high-degree as well, so the problem would persist. On the other hand, it becomes convenient to remove the field equations in the bounded syndrome decoding problem. This leads to a new quadratic modeling. \begin{modeling}[Quadratic Modeling for the SDP over $\FF_q$] \label{modeling: quadBSD_Fq} Given an instance $(\HH,\mathbf{s},t)$ of Problem~\ref{BSDP} over $\FF_q$, Modeling~\ref{modeling: quadBSD_Fq} is the union of the sets of polynomials \eqref{eq:pce}, \eqref{eq:sceBSD}, \eqref{eq: hwceFqinit}, \eqref{eq: hwceFq}, \eqref{eq: wceFq}. \end{modeling} In the next section we thoroughly investigate the effect of removing the field equations from Modeling~\ref{modeling: EWSD_Fq}. We find, that at least for the parameter choices interesting for cryptography, the solutions of our modeling without field equations still lie over $\FFq$ with high probability. Table~\ref{table:Fq-model-sizes} provides the number of variables and equations for the three modelings over $\FF_q$. \begin{table}[H] \centering \begin{tabular}{|c|c|c|c|} \hline & \# Polynomials & \# Variables & Degree\\ \hline Modeling~\ref{modeling: BSD_Fq} & $4n-k+n r_2+r_2$ & $n(r_2 + 2)$ & $q$\\ \hline Modeling~\ref{modeling: EWSD_Fq} & $3n-k+n r_2+r_2$ & $n(r_2 + 2)$ & $q$\\ \hline Modeling~\ref{modeling: quadBSD_Fq} & $3n-k+n r_2+r_2$ & $n(r_2 + 2)$ & $2$\\ \hline \end{tabular} \vspace{2mm} \caption{Number of equations, number of variables and maximum degree of the algebraic modelings over $\FF_{q^{r_1}}$.} \label{table:Fq-model-sizes} \end{table} \begin{remark} Since $r_2$ can be chosen as at most $m=\OO(\log_q (n))$, both the number of polynomials and variables are quasi-linear in the code-length $n$ in all the three modelings, namely they are $\OO(\log_2 (n))$. At the cost of defining the system over $\FF_{q^{r_1}}=\FF_{q^{m}}$, these quantities become linear in $n$, as the choice $r_2=1$ is possible. \end{remark} The modelings above capture exactly the corresponding syndrome decoding problem variants. \begin{theorem} \label{prop: sol iff} Given an instance $(\HH,\vs,t)$, \begin{enumerate} \item The vector $(\vx, \vy, \vz)$ is a solution of Modeling~\ref{modeling: BSD_Fq} if and only if $\vx$ is a solution of Problem~\ref{BSDP} and $\vx\in\FF_q$; \item The vector $(\vx, \vy, \vz)$ is a solution of Modeling~\ref{modeling: EWSD_Fq} if and only if $\vx$ is a solution of Problem~\ref{EWSDP} and $\vx\in\FF_q$; \item The vector $(\vx, \vy, \vz)$ is a solution of Modeling~\ref{modeling: quadBSD_Fq} if and only if $\vx$ is a solution of Problem~\ref{BSDP} and $\vx\in\overline{\FF_q}$, where $\overline{\FF_q}$ denotes the algebraic closure of $\FF_q$. \end{enumerate} \end{theorem} \begin{proof} Since the parity-check equations \eqref{eq:pce} belong to all three modelings, it remains to prove the conditions on the weight of $\vx$ and to determine the base field over the vector can lie.\\ \textit{Proof of 1.} Modeling~\ref{modeling: BSD_Fq} contains the field equations, therefore $\vx\in \FF_q^n$. It has already been proved that \eqref{eq:sceBSD} implies $\wt(\vx)\le\wt(\vz)$. By Corollary~\ref{cor: Z}, \eqref{eq: hwceFqinit}, \eqref{eq: hwceFq} and \eqref{eq: wceFq} imply that $\wt(\vz)=t$, hence $\wt(\vx)\le t$.\\ \textit{Proof of 2.} Modeling~\ref{modeling: EWSD_Fq} contains the field equations, therefore $\vx\in \FF_q^n$. It has already been proved that \eqref{eq:sceEWSD} implies $\wt(\vx)=\wt(\vz)$. By Corollary~\ref{cor: Z}, \eqref{eq: hwceFqinit}, \eqref{eq: hwceFq} and \eqref{eq: wceFq} imply that $\wt(\vz)=t$, hence $\wt(\vx)= t$.\\ \textit{Proof of 3.} The proof is analogous to the proof of \textit{1.}, with the only exception that Modeling~\ref{modeling: quadBSD_Fq} does not contain the field equations. Hence, the solutions of the system are all the vectors defined over the algebraic closure $\overline{\FF_q}$ that satisfy the parity-check equations and have weight at most $t$. \qed \end{proof} \subsection{The Dimension of the Variety Associated with Modeling~\ref{modeling: quadBSD_Fq}}\label{Sec:Dim_of_Var} Unlike the modelings that include the field equations, Modeling~\ref{modeling: quadBSD_Fq} is not a priori associated with a zero-dimensional ideal. This represents the main drawback of Modeling~\ref{modeling: quadBSD_Fq} compared to Modeling~\ref{modeling: BSD_Fq}. The zero-dimensional property is desirable because it is necessary for defining the degree of regularity and for applying the FGLM algorithm to convert the $\mathsf{degrevlex}$ Gröbner basis into a $\mathsf{lex}$ basis. While it is possible to convert non-zero dimensional ideals using methods such as Gr\"obner walk or others, the process may not be as straightforward \cite{GBwalk1, GBwalk2}. In this subsection, we analyze the dimension of the variety associated with the ideal corresponding to Modeling~\ref{modeling: quadBSD_Fq}. We will explore the conditions under which the variety is zero-dimensional, as well as the probability of this occurring. We begin with the following reduction. \begin{proposition}\label{prop:reductiontoProblem} Let $\vx\in\FF_q^n$ be a vector which is a solution of Problem~\ref{BSDP} for a given instance $(\HH,\vs,t)$. In other words, $\vx$ satisfies $\mathbf{H}\vx^\top=\mathbf{s}^\top$ and $\wt(\vx)\leq t$. Then, there exist finitely many $(\vy,\vz)$ such that $(\vx, \vy, \vz)$ is a solution of Modeling~\ref{modeling: quadBSD_Fq}. \end{proposition} \begin{proof} Let us first consider a vector $\vx$ satisfying the parity-check equations and such that $\wt(\vx)=t$. Then, in Modeling~\ref{modeling: quadBSD_Fq}, the $z_i$ must detect exactly the support of $\vx$, and $\vx$ uniquely determines a solution $(\vx,\vy,\vz)$ of the system. If instead $\wt(\vx)=\bar{t}<t$, then there exist $t-\bar{t}$ indexes where the $Z$ variables can have value 1 while $\vx_i=0$. Thus, for any solution $\bar{\vx}\in\FF_q^n$ of the parity check matrix of weight $\bar{t}$, there exist \[ \binom{n-\bar{t}}{t-\bar{t}} \] different solutions $(\bar{\vx}, \vy, \vz)$. A special case is given by the codeword finding problem, i.e. where the syndrome $\vs$ is the zero vector. Here the null vector is a solution of Problem~\ref{BSDP} and leads to $\binom{n}{t}$ solutions, thus likely increasing a lot the solving degree and the cost of a Gr\"obner basis computation. We can get rid of all these solutions by fixing one variable $z_i=1$, thus forcing any solution to have weight at least 1 and removing the null vector. If the target solution has weight $t$, then the guess has success with probability $t/n$. We will discuss this strategy in more detail at the end of this subsection. \qed \end{proof} Proposition~\ref{prop:reductiontoProblem} implies that each solution to the decoding problem (Problem~\ref{BSDP}) corresponds to a finite number of solutions for Modeling~\ref{modeling: quadBSD_Fq}. This allows us to conduct an analysis that is independent of the specific modeling, as long as it accurately encodes the decoding problem in the sense of Theorem~\ref{prop: sol iff}. Therefore, we will focus on the Krull dimension of the solution set of Problem~\ref{BSDP} and provide a probability estimate for this dimension being zero. First, in the following remark we briefly collect the definition of Krull dimension and some important properties we will use in the sequel. Expanded details and proofs can be found e.g. in \cite[\S4, Chapter~9]{cox1997ideals} or other standard references in commutative algebra and algebraic geometry. \begin{remark}[Krull dimension]\label{prop: Krull=dim}\label{prop: max_dim} Let $\K$ be an algebraically closed field (we will apply the following definitions and results to $\K=\overline{\FF_q}$). An affine variety $V$ is the zero locus in $\K^m$ of a proper ideal $I$ of the polynomial ring $\K[x_1,\dots,x_m]$. We say that $V$ is irreducible if it is not possible to write $V=V_1\cup V_2$ where $V_1,V_2\subsetneq V$ are two proper subvarieties. Irreducibility of $V$ is equivalent to the corresponding ideal $I$ being prime. The \emph{Krull dimension} or simply the dimension of a variety $V$ is defined as the maximal length $d$ of the chains $V_0\subsetneq V_1\subsetneq \cdots \subsetneq V_d$, of distinct nonempty irreducible subvarieties of $V$. This is also equivalent to the supremum of the lengths of all chains of prime ideals containing the defining ideal $I$ of $V$. For example, the Krull dimension of an affine linear space $\mathcal{L}$ generated by $a$ linearly independent affine linear polynomials $L_1,\dots,L_a$ is precisely $m-a$, that is its dimension as an affine space. This can be seen by completing the polynomials to a maximal linearly independent system of $m$ equations (in $m$ variables) $L_1,\dots,L_a,L_{a+1},\dots,L_m$ and then considering the following maximal chain of prime ideals \[ \mathcal{L}=\langle L_1,\dots L_a\rangle\subsetneq\langle L_1,\dots L_a,L_{a+1}\rangle\subsetneq\cdots\subsetneq \langle L_1,\dots L_m\rangle. \] Notice that each ideal in this chain is prime, being generated by linearly independent polynomials of degree $1$. Finally, we mention that, thanks to the Noetherian property of the polynomial ring, a variety $V$ can be written uniquely as the union of irreducible varieties, which are called the irreducible components of $V$. Thus, the dimension of $V$ coincides with the largest of the dimensions of its irreducible components (see \cite[\S 4,Chapter 9, Corollary 9]{cox1997ideals}). \end{remark} Let $S\subseteq [n]$. Given a matrix $\mathbf{H}$ with $n$ columns and a vector $\vx \in \FF_q^n$, we denote by $\mathbf{H}_S$ the submatrix of $\mathbf{H}$ of columns indexed by $S$ and by $\vx_S \in \FF_q^{|S|}$ the vector obtained by deleting the coordinates corresponding to $[n]\setminus S$ from $\vx\in\FF_q^n$. On the contrary, let $\mathsf{pad}_S(\vx)\in\FF_q^n$ be the vector obtained from $\vx\in \FF_q^{|S|}$ by padding with 0's the positions corresponding to $[n]\setminus S$. \begin{proposition} Let $\mathcal{C}$ be an $[n,k]$ code with parity-check matrix $\mathbf{H}\in \FF_q^{(n-k)\times n}$. Then the set of solutions of Problem~\ref{BSDP} with target weight $t$ and syndrome $\vs$ for the code $\mathcal{C}$ is the finite union of irreducible components, namely \[ \bigcup_{S \subset [n], |S|=t} \{ \mathsf{pad}_S(\vx) \in \FF_q^n \mid \mathbf{H}_S \vx= \vs\}. \] \end{proposition} \begin{proof} For any $S$ of cardinality $t$, $\wt(\mathsf{pad}_S(\vx))=\wt(\vx)\le t$ and $\mathbf{H}\cdot \mathsf{pad}_S({\vx}) =\mathbf{H}_S \vx=\vs$. Thus, the set of solutions of the decoding problem contains $$ \bigcup_{S \subset [n], |S|=t} \{ \mathsf{pad}_S(\vx) \in \FF_q^n \mid \mathbf{H}_S \vx= \vs\}.$$ On the other hand, for any solution $\vx \in \FF_q^n$ to the decoding problem, let $S$ be a set of cardinality $t$ containing the support of $\vx$. Then, $\vx\in \{ \mathsf{pad}_S(\vx) \in \FF_q^n \mid \mathbf{H}_S \vx= \vs\}$. Finally, all the sets $ \{ \mathsf{pad}_S(\vx) \in \FF_q^n \mid \mathbf{H}_S \vx= \vs\}$ are irreducible being affine linear spaces. \qed \end{proof} The next proposition characterizes the dimension of the irreducible components of the set of solutions of the decoding problem and the finite field extension over which solutions are defined. \begin{proposition} \label{prop: variety} The Krull dimension of the solution set of Problem~\ref{BSDP} with target weight $t$ and syndrome $\vs$ for the linear code with parity-check matrix $\mathbf{H}$ is \begin{equation} \label{eq: dimension_Ideal} t-\min\{\rk(\mathbf{H}_S) \mid S\subseteq [n], |S|=t, \rk((\mathbf{H}_S\mid \vs))=\rk(\mathbf{H}_S)\}. \end{equation} Moreover, if the dimension is 0, then all solutions lie over $\FF_q$. \end{proposition} \begin{proof} Let us fix the support $S$ of $t$ possible error positions. By Remark~\ref{prop: Krull=dim}, the Krull dimension of the irreducible components coincides with their dimensions as affine linear spaces. The case study of the set of solutions of \[ \mathbf{H}_S \vx = \vs, \] seen as a variety, thus becomes the following: \begin{itemize} \item if $\rk((\mathbf{H}_S\mid \vs))>\rk(\mathbf{H}_S) \Rightarrow$ the variety is empty; \item if $\rk((\mathbf{H}_S\mid \vs))=\rk(\mathbf{H}_S) \Rightarrow$ the variety has dimension $t-\rk(\mathbf{H}_S)$. In particular, if the variety has dimension 0, i.e. $\rk(\mathbf{H}_S)=t$, then it has a unique element, which belongs to $\FF_q$. \end{itemize} By Remark~\ref{prop: max_dim}, the dimension of the solutions set is obtained as the maximum dimension over all the irreducible components corresponding to some $S$ for which $\rk((\mathbf{H}_S\mid \vs))=\rk(\mathbf{H}_S)$: \begin{align*} &\max \{ t - \rk(\mathbf{H}_S) \mid S\subseteq [n], |S|=t, \rk((\mathbf{H}_S\mid \vs))=\rk(\mathbf{H}_S)\}\\=& t-\min\{\rk(\mathbf{H}_S) \mid S\subseteq [n], |S|=t, \rk((\mathbf{H}_S\mid \vs))=\rk(\mathbf{H}_S)\}. \end{align*} Let us now consider the case of a zero-dimensional variety. Then for any choice of $S$, there is at most a solution and it must belong to $\FF_q^n$. Hence, all solutions belong to $\FF_q^n$. \qed \end{proof} \begin{corollary} \label{cor: dim_variety} The dimension of the variety associated with Modeling~\ref{modeling: quadBSD_Fq} is \begin{equation} t-\min\{\rk(\mathbf{H}_S) \mid S\subseteq [n], |S|=t, \rk((\mathbf{H}_S\mid \vs))=\rk(\mathbf{H}_S)\}. \end{equation} \end{corollary} \begin{proof} It readily follows from Propositions \ref{prop: max_dim} and \ref{prop: variety} and the fact that each solution of Problem~\ref{BSDP} corresponds to a finite number of solutions of Modeling~\ref{modeling: quadBSD_Fq}, thus it does not increase the dimension of the variety. \qed \end{proof} For relevant and not too-small parameters, we usually have $t\ll n-k$. Assuming the weight distribution of a linear code follows closely the Bernoulli one, we can estimate the probability that the ideal is zero-dimensional, and thus, by exploiting the proof of Proposition \ref{prop: variety}, that all the solutions $\vx$ lie over $\FF_q$. \begin{proposition} \label{prop: bound} Let $\mathcal{C}$ be an $\mathbb{F}_q$-linear code and let $W_i(\mathcal{C})$ the number of codewords of weight exactly $i$ in $\mathcal{C}$. Then the probability that Modeling~\ref{modeling: quadBSD_Fq} provides a variety of strictly positive dimension when $t<n-k$ is upper bounded by \[ \sum_{i=1}^t W_i(\mathcal{C})\left(\frac{1}{q^{n-k-i+1}}+\binom{n-i}{t-i}\left(\frac{1}{q^{n-k-t+1}}-\frac{1}{q^{n-k-i+1}}\right)\right) \] for a randomly sampled syndrome. For the codeword finding problem, the same probability is upper bounded by \[ \sum_{i=1}^t W_i(\mathcal{C})\binom{n-i}{t-i}. \] \end{proposition} \begin{proof} It follows from Corollary~\ref{cor: dim_variety} that the variety has positive dimension if and only if there exists a set $S\subseteq [n]$, $|S|=t$, such that $\mathbf{H}_S$ is not full-rank and the syndrome $\vs$ belongs to the column space of $\mathbf{H}_S$. The parity-check matrix $\mathbf{H}$ has $i$ linearly dependent columns indexed by the set $S$ if and only if the corresponding code $\mathcal{C}$ has a codeword of weight $\le i$ with support contained in $S$. Hence any codeword of positive weight $i\le t$ with support $S'$ identifies a set of $\binom{n-i}{t-i}$ supersets $S\supseteq S'$. On the other hand, each set $S$ of $t$ dependent columns is associated with \textit{at least} one codeword of weight $\le t$, hence iterating over such codewords is enough to guarantee an upper bound. Let $W_i(\mathcal{C})$ be the number of codewords in $\mathcal{C}$ of weight exactly $i$. By splitting the event $\{\vs \in \mathsf{ColSpace}(\mathbf{H}_{S})\}$ into the union of the two disjoint events $\{\vs \in \mathsf{ColSpace}(\mathbf{H}_{\supp(\vc)})\}$ and $\{\vs \in \mathsf{ColSpace}(\mathbf{H}_{S})\setminus \mathsf{ColSpace}(\mathbf{H}_{\supp(\vc)})\}$ and using that $\rk(\mathbf{H}_{\supp(\vc)})\le i-1$ and $\rk(\mathbf{H}_{S})\le \rk(\mathbf{H}_{\supp(\vc)})+(t-i)$, we thus obtain an upper bound on the sought probability: \begin{align*} &\mathbb{P}\left(\bigcup_{\substack{S\subseteq [n] \\ |S|=t \land \rk(\mathbf{H}_S)<t}} \{\vs\in \mathsf{ColSpace}(\mathbf{H}_S)\}\right)\\ \le &\sum_{\substack{S\subseteq [n] \\ |S|=t \land \rk(\mathbf{H}_S)<t}}\mathbb{P}(\{\vs\in \mathsf{ColSpace}(\mathbf{H}_S)\})\\ \le & \sum_{\substack{\vc \in \mathcal{C} \\ \wt(\vc)\le t} } \left( \mathbb{P}(\{\vs\in \mathsf{ColSpace}(\mathbf{H}_{\supp(\vc)})\}) + \sum_{\substack{\supp(\vc)\subseteq S\subseteq [n] \\ |S|=t }} \mathbb{P}(\{\vs\in \mathsf{ColSpace}(\mathbf{H}_S)\setminus \mathsf{ColSpace}(\mathbf{H}_{\supp(\vc)})\})\right)\\ = & \sum_{i=1}^t W_i(\mathcal{C}) \left( \frac{q^{\dim_{\FF_q} \mathsf{ColSpace}(\mathbf{H}_{\supp(\vc)})}}{q^{n-k}}+ \binom{n-i}{t-i}\frac{q^{\dim_{\FF_q} \mathsf{ColSpace}(\mathbf{H}_{S})}-q^{\dim_{\FF_q} \mathsf{ColSpace}(\mathbf{H}_{\supp(\vc)})}}{q^{n-k}} \right)\\ = & \sum_{i=1}^t W_i(\mathcal{C}) \left( \frac{q^{\rk(\mathbf{H}_{\supp(\vc)})}}{q^{n-k}}+ \binom{n-i}{t-i}\frac{q^{\rk(\mathbf{H}_{S})}-q^{\rk(\mathbf{H}_{\supp(\vc)})}}{q^{n-k}} \right)\\ \le & \sum_{i=1}^t W_i(\mathcal{C}) \left( \frac{q^{\rk(\mathbf{H}_{\supp(\vc)})}}{q^{n-k}}+ \binom{n-i}{t-i}\frac{q^{\rk(\mathbf{H}_{\supp(\vc)})}}{q^{n-k}}(q^{t-i}-1) \right)\\ \le & \sum_{i=1}^t W_i(\mathcal{C}) \left( \frac{1}{q^{n-k-i+1}}+ \binom{n-i}{t-i} \left(\frac{1}{q^{n-k-t+1}}-\frac{1}{q^{n-k-i+1}}\right) \right). \end{align*} Finally, in the case of the codeword finding problem, i.e. if the syndrome is the zero vector, the condition on the positive dimension of the variety boils down to the existence of the set $S$, $|S|=t$, such that the rank $\mathbf{H}_S$ is defective, as the zero vector belongs to any linear subspace. Therefore, in this case, the calculations are simplified into: \[ \mathbb{P}\left(\bigcup_{\substack{S\subseteq [n] \\ |S|=t \land \rk(\mathbf{H}_S)<t}} \{0\in \mathsf{ColSpace}(\mathbf{H}_S)\}\right) \le\sum_{\substack{S\subseteq [n] \\ |S|=t \land \rk(\mathbf{H}_S)<t}} 1 \le \sum_{i=1}^t W_i(\mathcal{C}) \binom{n-i}{t-i} . \] \qed\end{proof} \begin{remark} For random codes, the weight distribution follows closely the Bernoulli one, i.e. $W_i(\mathcal{C})\approx \frac{\binom{n}{i}(q-1)^i}{q^{n-k}}$. Under this assumption, the probability of having a zero-dimensional ideal for the decoding modeling with a random syndrome can be estimated as \[ \frac{1}{q^{n-k}}\sum_{i=1}^t \binom{n}{i}(q-1)^i\left(\frac{1}{q^{n-k-i+1}}+\binom{n-i}{t-i}\left(\frac{1}{q^{n-k-t+1}}-\frac{1}{q^{n-k-i+1}}\right)\right), \] while for the codeword finding problem as \[ \frac{1}{q^{n-k}}\sum_{i=1}^t \binom{n}{i}\binom{n-i}{t-i}(q-1)^i. \] We remark in particular that the bound is independent from the choice of $(r_1,r_2)$. Different admissible pairs provide of course different solutions, but the projections of the varieties with respect to the $x_i$'s variables are the same over the field closure, which is what determines the dimension of the associated ideals. \end{remark} In Appendix~\ref{app:Dim_Exp}, Tables \ref{table: bound_SD} and \ref{table: bound_CF}, we provide examples of such bounds for concrete parameters. In the case of syndrome decoding, these probabilities are very small even at Gilbert-Varshamov distance and the issue of having ideals of positive dimension is thus absolutely negligible for the purpose of cryptanalysis. On the opposite, the bound on the probability of the same event for the codeword finding problem becomes completely useless when approaching the Gilbert-Varshamov distance. Indeed, the trivial upper bound ``$\pr \le 1$'' entries from the two tables mean that the bound given by Proposition \ref{prop: bound} gives a number larger than 1. A possible workaround for the described issue with the codeword finding version is to make use of hybrid methods. Indeed, it is enough to guess a number of nonzero positions equal or greater than the ideal dimension to decrease the latter to 0 with high probability. Recalling that the solution space is projective and one the value of one nonzero entry can be chosen arbitrarily, specializing $l$ coordinates has a success probability of $\frac{\binom{n-l}{t-l}}{\binom{n}{t} (q-1)^{l-1}}$. In cryptanalysis, however, it is usually assumed to know the minimal weight of a (nonzero) solution. This is because, if there exists a solution of weight smaller than the target, then the challenge is actually easier. A simple strategy to obtain a zero-dimensional ideal in this setting is thus the following. If we suppose to know a lower bound $d'$ the minimum distance $d(C)$ of the code, then it means that any $d'-1$ columns of the parity-check matrix $\mathbf{H}$ are linearly independent. Therefore, Equation~\eqref{eq: dimension_Ideal} implies that the dimension of the ideal for the bounded weight modeling with target weight $d'$ is exactly 1 and thus it is enough to specialize one variable $x_i$ to any element in $\FF_q$ to obtain a zero-dimensional ideal. \begin{table}[h] \resizebox{\textwidth}{!}{\begin{tabular}{|c|c|c|c|c|c|c|c|c|c|c|c|c|c|c|c|c|c|c|} \hline \cellcolor{gray!20!} & \cellcolor{gray!20!} &\cellcolor{gray!20!} & \cellcolor{gray!20!} & \multicolumn{5}{|c|}{\cellcolor{gray!20!}$q=7$} & \multicolumn{5}{|c|}{\cellcolor{gray!20!}$q=16,17$}& \multicolumn{5}{|c|}{\cellcolor{gray!20!}$q=127$}\\ \hline \cellcolor{gray!20!}$n$ & \cellcolor{gray!20!}$k$ &\cellcolor{gray!20!} $t$ & \cellcolor{gray!20!}\#lin & \cellcolor{gray!20!}$r_1$ & \cellcolor{gray!20!}$r_2$ & \cellcolor{gray!20!}\#quad & \cellcolor{gray!20!}\#vars & \cellcolor{gray!20!}$d_{\mathrm{M}}$ & \cellcolor{gray!20!}$r_1$ & \cellcolor{gray!20!}$r_2$ & \cellcolor{gray!20!}\#quad & \cellcolor{gray!20!}\#vars & \cellcolor{gray!20!}$d_{\mathrm{M}}$ & \cellcolor{gray!20!}$r_1$ & \cellcolor{gray!20!}$r_2$ & \cellcolor{gray!20!}\#quad & \cellcolor{gray!20!}\#vars & \cellcolor{gray!20!}$d_{\mathrm{M}}$ \\ \hline 10 & 5 & 2 & 5 & \multicolumn{5}{|c|}{$m=2$} & \multicolumn{5}{|c|}{$m=1$} & \multicolumn{5}{|c|}{$m=1$} \\ \cline{5-19} &&&& 2 & 1 & 30& 30& 4 & 1 & 1 & 30 & 30 & 4 & 1 & 1 & 30 & 30 & 4 \\ &&&& 1 & 2 & 40& 40& 3 &&&&& &&&&& \\ \hline 15 & 9 & 3 & 6 & \multicolumn{5}{|c|}{$m=2$} & \multicolumn{5}{|c|}{$m=1$} & \multicolumn{5}{|c|}{$m=1$} \\ \cline{5-19} &&&& 2 & 1 & 45& 45& 4 & 1 & 1 & 45 & 45 & 4 & 1 & 1 & 45 & 45 & 4 \\ &&&& 1 & 2 & 60& 60& 4 &&&&& &&&&& \\ \hline 19 & 10 & 5 & 9 & \multicolumn{5}{|c|}{$m=2$} & \multicolumn{5}{|c|}{$m=1$} & \multicolumn{5}{|c|}{$m=1$} \\ \cline{5-19} &&&& 2 & 1 & 57& 57& 5 & 1 & 1 & 57 & 57 & 5 & 1 & 1 & 57 & 57 & 5 \\ &&&& 1 & 2 & 76& 76& 4 &&&&& &&&&& \\ \hline 22 & 14 & 4 & 8 & \multicolumn{5}{|c|}{$m=2$} & \multicolumn{5}{|c|}{$m=2$} & \multicolumn{5}{|c|}{$m=1$} \\ \cline{5-19} &&&& 2 & 1 & 66& 66& 5 & 2 & 1 & 66& 66& 5 & 1 & 1 & 66 & 66 & 4 \\ &&&& 1 & 2 & 88& 88 & 4 & 1 & 2 & 88& 88 & 4 &&&&& \\ \hline 30 & 20 & 4 & 10 & \multicolumn{5}{|c|}{$m=3$} & \multicolumn{5}{|c|}{$m=2$} & \multicolumn{5}{|c|}{$m=1$} \\ \cline{5-19} &&&& 3 & 1 & 90& 90& $\ge 6$ & 2 & 1 & 90& 90& $\ge 6$ & 1 & 1 & 90 & 90 & $\ge 6$ \\ &&&& 2 & 2 & 120& 120 & 4 & 1 & 2 & 120& 120 & 5 &&&&& \\ &&&& 1 & 3 & 150& 150& 4 &&&&&& &&&&\\ \hline \end{tabular} } \vspace{2mm} \caption{This table gives information from experiments using random $\FF_q$-linear codes using Modeling~\ref{modeling: quadBSD_Fq}. The values in the $d_{\mathrm{M}}$ column represent the highest step degree achieved when directly computing the Gröbner basis of the system in MAGMA. The column ``\#lin'' denotes the number of linear equations, i.e. of parity-check equations, which is independent from the field size. The columns ``\#quad'' and ``\#vars'' stand for the number of quadratic equations and the number of variables, which depend on the value $r_2$ instead. The integer $r_1$ is the extension field degree over which the equations are defined. We recall that the value $m$ leads to different possible choices of $(r_1,r_2)$ and we give all minima with respect to the standard partial order on pairs.} \label{Tab:qNEQ2-SolveDeg} \end{table} \begin{table}[] \centering \begin{tabular}{|c|c|c|c|c|c|c|c|c|} \multicolumn{9}{c}{$q=7$} \\ \hline $n$ & $k$ & $t$ & \# Lin Eqs & \# Quad Eqs & \# Vars & SR $d_{reg}$ & SD & \# Solutions \\ \hline 8 & 2 & 2 & 9 & 32 & 33 & 8 & 3 & 1 \\ \hline 8 & 4 & 1 & 7 & 32 & 33 & 10 & 3 & 6 \\ \hline 10 & 5 & 1 & 8 & 40 & 41 & 12 & 3 & 2 \\ \hline 16 & 2 & 6 & 17 & 64 & 65 & 13 & 3 & 1 \\ \hline 16 & 3 & 4 & 16 & 64 & 65 & 14 & 3 & 1 \\ \hline 16 & 4 & 4 & 15 & 64 & 65 & 14 & 3 & 1 \\ \hline 16 & 8 & 2 & 11 & 64 & 65 & 18 & 4 & 1 \\ \hline 16 & 8 & 1 & 11 & 64 & 65 & 18 & 4 & 1 \\ \hline 20 & 10 & 2 & 13 & 80 & 81 & 21 & 4 & 1 \\ \hline \end{tabular} \caption{This table gives information from experiments using Random $\mathbb{F}_7$-linear codes using the {\color{red} do we have a name?} modeling. The value $\mathrm{sd}(S)$ is the solving degree of $S$, computed as the highest step degree achieved when directly computing the Gr\"obner basis of the system in MAGMA. The values in the SD column give the highest step degree achieved when directly computing the Gr\"obner basis of the system in MAGMA. The SR $d_{reg}$ column gives the degree of regularity of a Semi-Regular system of equations with the associated number of linear equations, quadratic equations, and variables. }\label{Tab:q2-SolveDeg} \end{table}
2412.04932v1
http://arxiv.org/abs/2412.04932v1
Trickle groups
\pdfsuppresswarningpagegroup=1 \documentclass{louloupart1} \usepackage{pinlabel} \usepackage{graphicx} \usepackage[all]{xy} \usepackage{tikz} \usetikzlibrary{decorations.markings,arrows,arrows.meta,positioning} \tikzset{->-/.style={decoration={ markings, mark=at position #1 with {\arrow{Computer Modern Rightarrow[length=5pt,width=5pt]}}},postaction={decorate}}} \tikzset{->-rev/.style={decoration={ markings, mark=at position #1 with {\arrow{Computer Modern Rightarrow[length=5pt,width=5pt,reversed]}}},postaction={decorate}}} \title[Trickle groups]{Trickle groups} \author[P Bellingeri]{Paolo Bellingeri} \givenname{Paolo} \surname{Bellingeri} \address{Paolo Bellingeri, Laboratoire Nicolas Oresme, UMR 6139, CNRS, Université de Caen-Normandie, 14032 Caen Cedex, France \vskip 3 pt Eddy Godelle, Laboratoire Nicolas Oresme, UMR 6139, CNRS, Université de Caen-Normandie, 14032 Caen Cedex, France \vskip 3 pt Luis Paris, Institut de Mathématiques de Bourgogne, UMR 5584, CNRS, Université de Bourgogne, 21000 Dijon, France} \email{[email protected]} \author[E Godelle]{Eddy Godelle} \givenname{Eddy} \surname{Godelle} \email{[email protected]} \author[L Paris]{Luis Paris} \givenname{Luis} \surname{Paris} \email{[email protected]} \newtheorem{thm}{Theorem}[section] \newtheorem{lem}[thm]{Lemma} \newtheorem{prop}[thm]{Proposition} \newtheorem{corl}[thm]{Corollary} \theoremstyle{definition} \newtheorem*{defin}{Definition} \newtheorem*{rem}{Remark} \newtheorem*{expl}{Example} \newtheorem*{acknow}{Acknowledgments} \newtheorem*{expl1}{Example 1} \newtheorem*{expl2}{Example 2} \newtheorem*{expl3}{Example 3} \numberwithin{equation}{section} \makeatletter \@arabic\c@figure} \@addtoreset{figure}{section} \makeatother \begin{document} \def\N{\mathbb N} \def\R{\mathbb R} \def\SSS{\mathfrak S} \def\SS{\mathcal S} \def\id{{\rm id}} \def\VJ{{\rm VJ}} \def\KJ{{\rm KJ}} \def\link{{\rm link}} \def\starE{{\rm star}} \def\Tr{{\rm Tr}} \def\Z{\mathbb Z} \def\supp{{\rm supp}} \def\st{{\rm st}} \def\pil{{\rm pil}} \def\RR{\mathcal R} \def\LL{\mathcal L} \def\nf{{\rm nf}} \def\syl{{\rm syl}} \def\FF{\mathcal F} \def\AC{{\rm AC}} \def\Div{{\rm Div}} \def\Ker{{\rm Ker}} \def\KVJ{{\rm KVJ}} \def\PVJ{{\rm PVJ}} n{{\rm fi}} \def\linkE{{\rm link}} \def\Aut{{\rm Aut}} \def\AA{\mathcal A} \begin{abstract} A new family of groups, called trickle groups, is presented. These groups generalize right-angled Artin and Coxeter groups, as well as cactus groups. A trickle group is defined by a presentation with relations of the form $xy = zx$ and $x^\mu = 1$, that are governed by a simplicial graph, called a trickle graph, endowed with a partial ordering on the vertices, a vertex labeling, and an automorphism of the star of each vertex. We show several examples of trickle groups, including extended cactus groups, certain finite-index subgroups of virtual cactus groups, Thompson group F, and ordered quandle groups. A terminating and confluent rewriting system is established for trickle groups, enabling the definition of normal forms and a solution to the word problem. An alternative solution to the word problem is also presented, offering a simpler formulation akin to Tits' approach for Coxeter groups and Green's for graph products of cyclic groups. A natural notion of a parabolic subgraph of a trickle graph is introduced. The subgroup generated by the vertices of such a subgraph is called a standard parabolic subgroup and it is shown to be the trickle group associated with the subgraph itself. The intersection of two standard parabolic subgroups is also proven to be a standard parabolic subgroup. If only relations of the form $xy = zx$ are retained in the definition of a trickle group, then the resulting group is called a preGarside trickle group. Such a group is proved to be a preGarside group, a torsion-free group, and a Garside group if and only if its associated trickle graph is finite and complete. \smallskip\noindent {\bf AMS Subject Classification\ \ } Primary: 20F10, Secondary: 20F05, 20F36, 20F55, 20F65. \smallskip\noindent {\bf Keywords\ \ } Trickle groups, right-angled Coxeter groups, right-angled Artin groups, cactus groups, virtual cactus groups, Thompson group F, word problem, rewriting systems, preGarside groups, Garside groups. \end{abstract} \maketitle \section{Introduction}\label{sec1} There are numerous groups in the literature defined by relations of the form $xy = zx$, often with additional constraints on the orders of generators. Prominent examples include right-angled Artin groups, right-angled Coxeter groups, and more generally, graph products of cyclic groups. The aim of the present paper is to study a specific family of such groups that we call \emph{trickle groups}. Cactus groups are emblematic examples of trickle groups. These groups first appeared as quasi-braid groups in the study of the mosaic operad \cite{Devad1,EHKR1,KhWi1} and they were subsequently generalized to all Coxeter groups \cite{DaJaSc1}. Their significance was further highlighted in their connection to coboundary categories \cite{HenKam1}, mirroring the role of braid groups in braided categories. Note that the term ``cactus groups'' was coined in \cite{HenKam1}. Additionally, cactus groups and their generalizations to Coxeter groups have found applications in representation theory under various guises \cite{KnTaWo1,Bonna1,Losev1,ChGlPy1,RouWhi1}. For $n \in \N_{\ge 2}$, the \emph{cactus group} $J_n$ is defined by the presentation with generators $x_{p,q}$, $1 \le p < q \le n$, and relations: \begin{itemize} \item[(j1)] $x_{p,q}^2 = 1$, for $1 \le p <q \le n$, \item[(j2)] $x_{p,q} x_{m,r} = x_{m,r} x_{p,q}$, for $[p,q] \cap [m,r] = \emptyset$, \item[(j3)] $x_{p,q} x_{m,r} = x_{p+q-r,p+q-m} x_{p,q}$, for $[m,r]\subset[p,q]$. \end{itemize} The elements of $J_n$ are often depicted using planar diagrams. More precisely, an element $g \in J_n$ is represented by an $n$-tuple of smooth paths in the plane, $b=(b_1,\dots,b_n)$, $b_i : [0,1] \to \R^2$, satisfying the following conditions. \begin{itemize} \item There exists a permutation $\sigma \in \SSS_n$ such that $b_i(0)=(0,i)$ and $b_i(1)=(1,\sigma(i))$, for all $i\in \{1,\dots, n\}$. \item For all $t\in [0,1]$ and all $i \in \{1, \dots, n\}$ we have $\pi_1(b_i(t))=t$, where $\pi_1 : \R^2 \to \R$ denotes the projection onto the first coordinate. \item Crossings between the $b_i$'s may be multiple but are always transversal. \end{itemize} The generator $x_{p,q}$ is represented in Figure \ref{fig1_1} and relation (j3) is illustrated in Figure \ref{fig1_2}. \begin{figure}[ht!] \begin{center} \includegraphics[width=2.8cm]{BeGoPaFig1_1.pdf} \caption{Generator of $J_n$}\label{fig1_1} \end{center} \end{figure} \begin{figure}[ht!] \begin{center} \includegraphics[width=8.4cm]{BeGoPaFig1_2.pdf} \caption{Relation (j3) in the presentation of $J_n$}\label{fig1_2} \end{center} \end{figure} The extension of this definition to Coxeter groups is straightforward. Let $(W,S)$ be a Coxeter system associated with a Coxeter graph $\Upsilon$. For $X \subseteq S$, the subgroup of $W$ generated by $X$ is called a \emph{standard parabolic subgroup} and is denoted by $W_X$, and the full Coxeter subgraph of $\Upsilon$ spanned by $X$ is denoted by $\Upsilon_X$. We say that $X \subseteq S$ is \emph{irreducible} if $\Upsilon_X$ is connected, and we say that $X$ is of \emph{spherical type} if $W_X$ is finite. In the latter case $W_X$ contains a unique element of maximal length (with respect to $S$), denoted by $w_X$, and this element satisfies $w_X X w_X^{-1}=X$ and $w_X^2=\id$ (see \cite{Bourb1}). n}$. n}$, and relations: \begin{itemize} \item[(j1)] n}$, \item[(j2)] $x_X x_Y = x_Y x_X$, if $\Upsilon_{X\cup Y}$ is the disjoint union of $\Upsilon_X$ and $\Upsilon_Y$, meaning that $X \cap Y = \emptyset$ and $st = ts$ for all $s \in X$ and $t \in Y$, \item[(j3)] $x_X x_Y = x_{w_X(Y)} x_X$, for $Y \subset X$. \end{itemize} Our aim is to investigate the combinatorial properties of these groups within a broader framework that encompasses various more or less natural generalizations of cactus groups. Let $\Gamma$ be a simplicial graph. We denote the vertex set of $\Gamma$ by $V (\Gamma)$ and the edge set by $E (\Gamma)$. The \emph{link} of a vertex $x \in V(\Gamma)$, denoted by $\linkE_x (\Gamma)$, is the full subgraph of $\Gamma$ spanned by $\{y \in V (\Gamma) \mid \{x, y\} \in E(\Gamma)\}$. The \emph{star} of $x$, denoted by $\starE_x (\Gamma)$, is the full subgraph of $\Gamma$ spanned by $\{y\in V (\Gamma) \mid \{x, y\} \in E(\Gamma)\}\cup \{x\}$. A \emph{trickle graph} is a quadruple $(\Gamma, \le, \mu, (\varphi_x)_{x\in V(\Gamma)})$, where: \begin{itemize} \item $\Gamma$ is a simplicial graph, \item $\le$ is a (partial) order on $V (\Gamma)$, \item $\mu: V(\Gamma) \to \N_{\ge 2} \cup \{\infty\}$ is a vertex labeling, \item $\varphi_x: \starE_x (\Gamma) \to \starE_x (\Gamma)$ is an automorphism of $\starE_x (\Gamma)$ for all $x \in V(\Gamma)$, \end{itemize} that must satisfy certain conditions defined in Section \ref{sec2}. The \emph{trickle group} $\Tr (\Gamma)$ associated with a trickle graph $\Gamma = (\Gamma, \le, \mu,(\varphi_x)_{x\in V(\Gamma)})$ is the group defined by the following presentation. \begin{gather*} \Tr (\Gamma) = \langle V(\Gamma) \mid x^{\mu(x)} = 1 \text{ for all } x \in V (\Gamma) \text{ such that } \mu (x) \neq \infty\,,\ \varphi_x(y)\,x = \varphi_y(x)\,y \\ \text{ for all } \{x, y\} \in E(\Gamma) \rangle\,. \end{gather*} One of the conditions in the definition of a trickle graph in Section \ref{sec2} entails that, for any edge $\{x,y\} \in E (\Gamma)$, either $\varphi_x (y) = y$ or $\varphi_y (x) = x$. Consequently, the above presentation is indeed a presentation with relations of the form $xy=zx$ and $x^\mu=1$. Another immediate consequence of these conditions is that the trivial order is admissible, and therefore right-angled Coxeter groups, right-angled Artin groups and, more generally, graph products of cyclic groups are trickle groups. As mentioned above, cactus groups are trickle groups, where here $\mu(x) = 2$ for all $x \in V (\Gamma)$. Trickle groups include other groups naturally related to cactus groups, such as the ``Artin'' versions of cactus groups (associated with Coxeter groups), where we keep relations (j2) and (j3) and ignore relations (j1). In this case, such a group is also a preGarside group in the sense of \cite{GodPar2} (see Section \ref{sec7} and Subsection \ref{subsec2_5}). More generally, the example of cactus groups associated with Coxeter systems can be naturally extended to families of subgroups of a given group $G$ satisfying certain properties that are presented in Subsection \ref{subsec3_1}. Our study is also an opportunity to investigate \emph{virtual cactus groups} since, as we will see in Subsection \ref{subsec3_2}, they contain finite index subgroups that are trickle groups. Let $S_1, \dots, S_\ell$ be a collection of $\ell$ circles immersed in the plane having only double transverse crossings. We assign to each crossing a ``positive'', ``negative'' or ``virtual'' value that we indicate on the graphical representation of $S_1 \cup \cdots \cup S_\ell$ as in Figure \ref{fig1_3}. Such a figure is called a \emph{virtual link diagram}. We consider the equivalence relation on the set of virtual link diagrams generated by the isotopy and the virtual Reidemeister moves as defined by Kauffman \cite{Kauff1,Kauff2}. An equivalence class of virtual link diagrams is a \emph{virtual link}. \begin{figure}[ht!] \begin{center} \includegraphics[width=4.4cm]{BeGoPaFig1_3.pdf} \caption{Crossings in a virtual link diagram}\label{fig1_3} \end{center} \end{figure} Since the publication of Kauffman's seminal paper \cite{Kauff1}, the theory of virtual knots and links has grown significantly, and this notion has been extended to other combinatorial and/or topological objects represented by planar diagrams. The leitmotif underlying these theories is that two arcs connecting the same points and passing only through virtual crossings are equivalent. Since the elements of the cactus group $J_n$ are represented by planar diagrams, it is natural to extend $J_n$ by adding virtual crossings to the cactus crossings while keeping the principle that two arcs connecting the same points and passing only through virtual crossings are equivalent. Then we obtain the \emph{virtual cactus group}, $\VJ_n$, which will be studied in detail in Subsection \ref{subsec3_2}. Note that these groups are not new, having been introduced in \cite{IKLPR1}, where it is shown that $\VJ_n$ is the $\SSS_n$-equivariant fundamental group of the real form of the ``cactus flower moduli space'' $\bar F_n$. The connection between virtual cactus groups and trickle groups mirrors that between virtual braid groups and Artin groups \cite{GodPar1, BeCiPa1, BelPar1, BePaTh1}. We prove that $\VJ_n$ can be decomposed as a semi-direct product $\VJ_n = \KJ_n \rtimes \SSS_n$, where $\SSS_n$ is the symmetric group on $\{1,\dots, n\}$, and $\KJ_n$ is a trickle group (see Proposition \ref{prop3_7}). This decomposition enables us to address the word problem in $\VJ_n$, to define normal forms for the elements of $\VJ_n$, and to show that $J_n$ embeds into $\VJ_n$. It is probable that numerous other trickle groups exist within the literature. Among these, we have pinpointed two specific examples: one originating from dynamical systems and the other from knot theory. Thompson group $F$, introduced by Richard Thompson in 1965 in an unpublished manuscript, is a group of homeomorphisms of the real line with many unusual properties. For instance, its derived group $F'$ is a simple group, the quotient $F/F'$ is a free abelian group of rank $2$, and it contains no subgroup isomorphic to the rank $2$ free group. We refer to \cite{CaFlPa1} for a general overview on this group. A well-known presentation for $F$ is as follows: \[ \langle x_n\,,\ n\in \N \mid x_k x_n = x_{n+1} x_k \text{ for } k < n \rangle\,, \] which, while featuring relations of the form $xy=zx$, does not fulfill all the requirements of a trickle group presentation. However, in Subsection \ref{subsec3_3} we show another presentation for $F$ that is indeed a trickle presentation (see Theorem \ref{thm3_15}). So, Thompson group $F$ is a trickle group. This structure is particularly noteworthy as any ``natural standard parabolic subgroup'' of $F$ is a copy of $F$ itself (see Proposition \ref{prop3_16}). Moreover, Theorem \ref{thm2_14} will imply that $F$ is a preGarside group. Quandles are algebraic structures whose axioms reproduce the Reidemeister moves in knot theory. Independently introduced by Joyce \cite{Joyce1} and Matveev \cite{Matve1}, they have been frequently used to construct knot or link invariants. As noted by Joyce \cite{Joyce1} and Matveev \cite{Matve1}, a classification of quandles would de facto entail a classification of knots. This explains the difficulty of studying the set of all quandles, leading researchers to focus on specific families of quandles. Ordered quandles were introduced recently in this perspective \cite{BaPaSi1,DDHPV1}, and it turns out that they are a disguised form of trickle graphs. The details of this construction are given in Subsection \ref{subsec3_4}. As previously mentioned, the objective of this paper is a combinatorial study of trickle groups. In Section \ref{sec4} we determine a confluent and terminating rewriting system for these groups (see Theorem \ref{thm2_4}). According to Newman \cite{Newma1}, this enables the definition of normal forms for their elements and, consequently, a solution to the word problem (see Corollary \ref{corl2_5}). Moreover, it yields other immediate results, such as a characterization of finite trickle groups (see Corollary \ref{corl2_7}). Notice that our rewriting system and its associated normal forms are not novel for graph products of cyclic groups \cite{Wyk1,HerMei1,CrGoWi1}, and a similar rewriting system for classical cactus groups was considered in \cite{Genev1}. Note also that the term ``trickle'', used to designate trickle groups, originates from this algorithm because it metaphorically involves pushing down as many syllables as possible using only relations of the form $xy = zx$. The remaining sections of the paper explore how trickle groups behave in a manner analogous to groups studied in the theory of Coxeter, Artin, and Garside groups. Building upon the algorithms and methods introduced in Section \ref{sec4}, we present in Section \ref{sec5} an alternative algorithm for solving the word problem in a trickle group. This new algorithm offers a simpler formulation compared to the one presented in Section \ref{sec4}. Furthermore, it aligns more closely with the algorithms described in \cite{Tits1} for Coxeter groups, in \cite{Green1} for graph products of cyclic groups, and more generally, in \cite{ParSoe1} for Dyer groups. A natural notion of a \emph{parabolic subgraph} emerges in the context of trickle graphs. This leads to the natural question of studying the trickle groups defined by such subgraphs. Drawing a parallel with the theory of Coxeter and Artin groups, we refer to these as \emph{standard parabolic subgroups}. As the name suggests, we establish in Section \ref{sec6} that a standard parabolic subgroup is indeed a subgroup of the trickle group associated with the original graph (see Theorem \ref{thm2_10}). Moreover, we prove that the intersection of two standard parabolic subgroups is a standard parabolic subgroup (see Corollary \ref{corl2_12}). Section \ref{sec7} establishes a connection between trickle groups and Garside theory. A \emph{preGarside trickle graph} is defined as a trickle graph $\Gamma = (\Gamma, \le, \mu, (\varphi_x)_{x \in V (\Gamma)})$ for which $\mu (x) = \infty$ for all $x \in V (\Gamma)$. A \emph{preGarside trickle group} is a trickle group associated with a preGarside trickle graph. Additionally, one can define an associated \emph{preGarside trickle monoid} using the same presentation, but interpreted as a monoid presentation. The term ``preGarside'' originates from \cite{GodPar2}, where the authors investigate monoids and groups that they call preGarside monoids and preGarside groups. Notable examples of such monoids and groups include all Artin monoids and all Artin groups. Garside monoids and Garside groups are also preGarside monoids and preGarside groups. In Section \ref{sec7}, we prove that a preGarside trickle monoid is indeed a preGarside monoid and that a preGarside trickle group is a preGarside group (see Theorem \ref{thm2_14}). Furthermore, the monoid and the group are respectively a Garside monoid and a Garside group if and only if $\Gamma$ is finite and complete (see Theorem \ref{thm2_15}). This introduces new examples of Garside groups to which we can associate Coxeter-style quotients. Another objective of Section \ref{sec7} is to address certain questions that arise for preGarside monoids and groups within the context of preGarside trickle monoids and groups. First, we already know from Section \ref{sec4} that a preGarisde trickle group has a solution to the word problem. Then, we prove in Section \ref{sec7} that, if the vertex set of the graph is finite, then the preGarside trickle group is torsion-free (see Theorem \ref{thm2_16}). The remaining questions concern the relationship between monoids and groups. These inquiries, posed in \cite{GodPar2}, are specifically focused on preGarside monoids and groups. We prove that a preGarside trickle monoid embeds into its enveloping group (see Theorem \ref{thm2_17}) and we prove several results concerning its parabolic submonoids and subgroups (see Theorem \ref{thm2_18}). Trickle groups appear to be a reasonable generalization of graph products of cyclic groups, and consequently, of right-angled Artin groups and right-angled Coxeter groups. Therefore, it is natural to explore whether results known for some or all graph products of cyclic groups can be extended to some or all trickle groups. Relevant questions in this direction include: \begin{itemize} \item[(1)] Do the normal forms described in Section \ref{sec4} form a regular language? Are trickle groups automatic or bi-automatic? \item[(2)] Which trickle groups admit geometric actions on CAT(0) cube complexes? \item[(3)] Are trickle groups residually finite? Are preGarside trickle groups residually nilpotent without torsion? Can we determine the Lie algebra associated with their lower central series, as done for right-angled Artin groups (see \cite{DucKro1})? \item[(4)] Which preGarside trickle groups are orderable, bi-orderable, or admit isolated orders? \end{itemize} Additionally, it would be valuable to discover new examples of trickle groups, particularly those arising from areas of mathematics beyond group theory. The paper is organized as follows. Section \ref{sec2} presents the fundamental definitions and precise statements of our main results. It is divided into five subsections. In the first subsection we introduce the concepts of trickle graphs and trickle groups, along with illustrative examples like graph products of cyclic groups. In Subsection \ref{subsec2_2} we state the main results of Section \ref{sec4}, which focuses on the trickle algorithm. In Subsection \ref{subsec2_3} we state the main results of Section \ref{sec5}, which focuses on the Tits-style algorithm. In Subsection \ref{subsec2_4} we state the main results of Section \ref{sec6}, which focuses on parabolic subgroups. In Subsection \ref{subsec2_5} we state the main results of Section \ref{sec7}, which focuses on preGarside trickle groups. Section \ref{sec3} is devoted to examples and it is divided into four subsections: Subsection \ref{subsec3_1} for cactus groups in their generalized version, Subsection \ref{subsec3_2} for virtual cactus groups, Subsection \ref{subsec3_3} for Thompson group F, and Subsection \ref{subsec3_4} for ordered quandle groups. As mentioned earlier, Sections \ref{sec4}, \ref{sec5}, \ref{sec6} and \ref{sec7} contain the proofs: those concerning the trickle algorithm in Section \ref{sec4}, those concerning the Tits-style algorithm in Section \ref{sec5}, those concerning parabolic subgroups in Section \ref{sec6}, and those concerning preGarside trickle groups in Section \ref{sec7}. \begin{acknow} This work originated during a research residency program titled ``Cactus and Posets'' at CIRM (Luminy, Marseille, France) from November 7th to 11th, 2022. The three authors extend their sincere gratitude to CIRM for the generous support and resources provided (funding, dedicated workspaces, library access, etc.), without which this project would not have been possible. \end{acknow} \section{Definitions and statements}\label{sec2} \subsection{Definitions and first examples}\label{subsec2_1} The set of vertices of a simplicial graph $\Gamma$ is denoted by $V(\Gamma)$ and the set of its edges is denoted by $E(\Gamma)$. The \emph{link} of a vertex $x \in V(\Gamma)$, denoted by $\link_x(\Gamma)$, is the full subgraph of $\Gamma$ spanned by $\{y \in V (\Gamma) \mid \{x, y\} \in E (\Gamma) \}$, and the \emph{star} of $x$, denoted by $\starE_x (\Gamma)$, is the full subgraph of $\Gamma$ spanned by $\{y \in V(\Gamma) \mid \{x, y\} \in E (\Gamma)\} \cup \{x\}$. \begin{defin} A \emph{trickle graph} is a quadruple $(\Gamma, \le, \mu, (\varphi_x)_{x \in V (\Gamma)})$, where \begin{itemize} \item $\Gamma$ is a simplicial graph, \item $\le$ is a (partial) order on $V (\Gamma)$, \item $\mu$ is a labeling $\mu: V(\Gamma) \to \N_{\ge 2} \cup \{\infty\}$ of the vertices, \item $\varphi_x: \starE_x (\Gamma) \to \starE_x (\Gamma)$ is an automorphism of $\starE_x (\Gamma)$ for all $x \in V (\Gamma)$. \end{itemize} For $x, y \in V (\Gamma)$ the notation $x||y$ means that $x$ and $y$ are not comparable in the sense that $x \not \le y$ and $y \not \le x$. We set $E_{||} (\Gamma) = \{\{x, y\} \in E(\Gamma) \mid x||y \}$. The quadruple $(\Gamma, \le, \mu, (\varphi_x)_{x \in V (\Gamma)})$ must satisfy the following conditions. \begin{itemize} \item[(a)] For all $x, y \in V (\Gamma)$, if $x < y$, then $\{x, y\} \in E(\Gamma)$. \item[(b)] For all $x, y, z \in V (\Gamma)$, if $\{x, y\} \in E_{||} (\Gamma)$ and $z \le y$, then $\{ x, z\} \in E_{||} (\Gamma)$. \item[(c)] For all $x \in V(\Gamma)$ and all $y, z \in \starE_x (\Gamma)$, we have $z\le y$ if and only if $\varphi_x(z) \le \varphi_x (y)$. \item[(d)] For all $x \in V(\Gamma)$ and all $ y \in \starE_x (\Gamma)$, if $\varphi_x (y) \neq y$, then $y < x$. \item[(e)] For all $x \in V (\Gamma)$, if $\mu (x)$ is finite, then $\varphi_x$ has finite order and its order divides $\mu (x)$. \item[(f)] For all $x \in V(\Gamma)$ and all $y \in \starE_x (\Gamma)$, $\mu (\varphi_x (y)) = \mu (y)$. \item[(g)] For all $x, y, z \in V (\Gamma)$, if $z < y < x$, then $(\varphi_x \circ \varphi_y) (z) = (\varphi_{y'} \circ \varphi_x) (z)$, where $y' = \varphi_x (y)$. \end{itemize} We will often say that $\Gamma$ is a trickle graph meaning that, implicitly, $\le$, $\mu$ and $(\varphi_x)_{x \in V (\Gamma)}$ are also given. \end{defin} \begin{rem} Let $x, y, z \in V(\Gamma)$ be such that $z \le y \le x$. Then it is easily seen that $(\varphi_x \circ \varphi_y) (z) = (\varphi_{y'} \circ \varphi_x) (z)$, where $ y' = \varphi_x (y)$, if either $z=y$ or $y=x$. So, Condition (g) in the definition of a trickle graph also holds if at least two of the three vertices are equal. \end{rem} \begin{defin} The \emph{trickle group} $\Tr (\Gamma)$ associated with a trickle graph $\Gamma = (\Gamma, \le, \mu, (\varphi_x)_{x \in V(\Gamma) })$ is the group defined by the following presentation. \[ \Tr (\Gamma) = \langle V(\Gamma) \mid x^{\mu (x)} = 1 \text{ for } x \in V (\Gamma) \text{ such that } \mu ( x) \neq \infty \,, \ \varphi_x (y) \, x = \varphi_y (x) \,y \text{ for } \{x, y\} \in E (\Gamma) \rangle\,. \] \end{defin} \begin{rem} By Condition (d) in the definition of a trickle graph, if $\{x, y\} \in E_{||} (\Gamma)$, then the relation $\varphi_x (y)\, x = \varphi_y(x) \,y$ becomes $y x = x y$. If $x < y$, then this relation becomes $y x = \varphi_y (x) \, y$. So, $\Tr (\Gamma)$ has a presentation with relations of the form $x y = z x$ and $x^\mu=1$. \end{rem} \begin{expl1} Let $\Gamma$ be a simplicial graph and let $\mu : V(\Gamma) \to \N_{\ge 2} \cup \{ \infty\}$ be a labeling of the vertices. To the pair $(\Gamma, \mu)$ we associate the \emph{graph product of cyclic groups} $G(\Gamma, \mu)$ defined by the following presentation. \[ G (\Gamma, \mu) = \langle V (\Gamma) \mid x^{\mu (x)} = 1 \text{ for } x \in V (\Gamma) \text{ such that } \mu (x) \neq \infty\,,\ x y = y x \text{ for } \{x, y\} \in E(\Gamma) \rangle\,. \] If $\mu (x) = \infty$ for all $x \in V(\Gamma)$, then $G (\Gamma, \mu)$ is a \emph{right-angled Artin group}, and if $\mu (x) = 2$ for all $x \in V(\Gamma)$, then $G (\Gamma, \mu)$ is a \emph{right-angled Coxeter group}. It is easily seen that $G (\Gamma, \mu)$ is a trickle group, where $\le$ is the trivial order defined by $x \le y$ if and only if $x = y$, and $ \varphi_x$ is the identity of $\starE_x (\Gamma)$ for all $x \in V(\Gamma)$. \end{expl1} \begin{expl2} Le $\Upsilon$ be a Coxeter graph and let $(W,S)$ be its associated Coxeter system. As in the introduction, for $X \subseteq S$, we denote by $W_X$ the standard parabolic subgroup generated by $X$ and by $\Upsilon_X$ the full Coxeter subgraph of $\Upsilon$ spanned by $X$. Recall that $X \subseteq S$ is called \emph{irreducible} and of \emph{spherical type} if $\Upsilon_X$ is connected and $W_X$ is finite. Recall also that, in this case, $W_X$ contains a unique element of maximal length (with respect to $S$), denoted by $w_X$, and this element satisfies $w_X^2 = 1$ and $w_X X w_X = X$ (see \cite{Bourb1}). n}$ the set of non-empty subsets $X \subseteq S$ that are irreducible and of spherical type. We define $\Gamma = (\Gamma, \le, \mu, (\varphi_x)_{x \in V (\Gamma)})$ as follows. n$. Two vertices $x_X$ and $x_Y$ are connected by an edge if either $X \subset Y$, or $Y \subset X$, or $\Upsilon_{X \cup Y}$ is the disjoint union of $\Upsilon_X$ and $\Upsilon_Y$ in the sense that $X \cap Y = \emptyset$ and $\{s, t\} \not \in E (\Upsilon)$ for all $s \in X$ and $t \in Y$. We set $x_X \le x_Y$ if $X \subseteq Y$. n}$. Let $x_X \in V (\Gamma)$ and $x_Y \in V (\starE_{x_X} (\Gamma))$. We set $\varphi_{x_X} (x_Y) = x_{w_X(Y)}$ if $Y \subset X$ and $\varphi_{x_X} (x_Y) = x_Y$ otherwise. It is easily verified that $\le$ is a (partial) order on $V (\Gamma)$ and that, for all $x_X \in V (\Gamma)$, $\varphi_{x_X}$ is an automorphism of $\starE_{x_X} (\Gamma)$. Now, we prove that $\Gamma = (\Gamma, \le, \mu, (\varphi_x)_{x \in V(\Gamma)})$ satisfies Conditions (a) to (g) of the definition of a trickle graph. \begin{lem}\label{lem2_1} The quadruple $\Gamma = (\Gamma, \le, \mu, (\varphi_x)_{x \in V(\Gamma)})$ above defined is a trickle graph. \end{lem} \begin{proof} Conditions (a), (d) and (f) are satisfied by definition of $\Gamma$. We show that $\Gamma$ satisfies Condition (b). n}$ be such that $\{x_X, x_Y\} \in E_{||} (\Gamma)$ and $x_Z \le x_Y$. Then $\Upsilon_{X \cup Y}$ is the disjoint union of $\Upsilon_X$ and $\Upsilon_Y$ and $Z \subset Y$. Thus, $\Upsilon_{X \cup Z}$ is the disjoint union of $\Upsilon_X$ and $\Upsilon_Z$, hence $\{x_X, x_Z\} \in E_{||} (\Gamma)$. We show that $\Gamma$ satisfies Condition (e). n}$. Since $w_X$ has order $2$, we have $\varphi_{x_X}^2 (x_Y) = x_{w_X^2(Y)} = x_Y$ if $Y \subset X$. We have $\varphi_{x_X}^2 (x_Y) = \varphi_{x_X} (x_Y) = x_Y$ for every other vertex of $\starE_{x_X} (\Gamma)$, hence the order of $\varphi_{x_X}$ divides $\mu(x_X) =2$. We show that $\Gamma$ satisfies Condition (c). n}$ be such that $x_Z, x_Y \in \starE_{x_X} (\Gamma)$. Since, by Condition (e) already proved, $\varphi_{x_X}^2 = \id$, to show the equivalence $x_Z \le x_Y\ \Leftrightarrow\ \varphi_{x_X} (x_Z) \le \varphi_{x_X} (x_Y)$, it suffices to show the implication $x_Z \le x_Y\ \Rightarrow\ \varphi_{x_X} (x_Z) \le \varphi_{x_X} (x_Y)$. Suppose $x_Z \le x_Y$, that is, $Z \subseteq Y$. If $\Upsilon_{X \cup Y}$ is the disjoint union of $\Upsilon_X$ and $\Upsilon_Y$, then $\Upsilon_{X \cup Z}$ is the disjoint union of $\Upsilon_X$ and $\Upsilon_Z $, hence $\varphi_{x_X} (x_Z) = x_Z \le x_Y = \varphi_{x_X} (x_Y)$. If $x_X \le x_Z \le x_Y$, then $X \subseteq Z \subseteq Y$, hence $\varphi_{x_X} (x_Z) = x_Z \le x_Y = \varphi_{x_X} (x_Y)$. If $x_Z \le x_X \le x_Y$, then $Z \subseteq X \subseteq Y$ hence $w_X (Z) \subseteq X \subseteq Y$, and therefore $\varphi_{x_X} (x_Z) = x_{w_X(Z)} \le x_X \le x_Y = \varphi_{x_X} (x_Y)$. If $x_X \le x_Y$ and $x_Z || x_X$, then $X \subseteq Y$, $Z \subseteq Y$ and $\Upsilon_{X \cup Z}$ is the disjoint union of $\Upsilon_X$ and $\Upsilon_Z$, hence $\varphi_{x_X} (x_Z) = x_Z \le \varphi_{x_X} (x_Y) = x_Y$. If $x_Z \le x_Y \le x_X$, then $Z \subseteq Y \subseteq X$, hence $w_X(Z) \subseteq w_X(Y) \subseteq X$, and therefore $\varphi_{x_X} (x_Z) = x_{w_X(Z)} \le x_{w_X(Y)} = \varphi_{x_X} (x_Y)$. Finally we show that $\Gamma$ satisfies Condition (g). n}$ be such that $Z \subseteq Y \subseteq X$. Let $Y' = w_X (Y)$. We have \[ (w_X w_Y) (Z) = (w_X w_Y w_X^{-1} w_X) (Z) = (w_{w_X(Y)} w_X) (Z) = (w_{Y'} w_X) (Z)\,, \] hence $(\varphi_{x_X} \circ \varphi_{x_Y}) (x_Z) = (\varphi_{x_{Y'}} \circ \varphi_{x_X}) (x_Z)$. \end{proof} It is obvious that the cactus group $C(W,S)$ is equal to the trickle group $\Tr (\Gamma)$. \end{expl2} \begin{expl3} Let $\Gamma = (\Gamma, \le, \mu, (\varphi_x)_{x \in V(\Gamma)})$ be a trickle graph. Notice that $\widetilde{\Gamma} = (\Gamma, \le, \mu, (\varphi_x^{-1})_{x \in V(\Gamma)})$ is also a trickle graph which we call the \emph{dual trickle graph} of $\Gamma$. On the other hand, we can define the \emph{dual trickle group} $\widetilde{\Tr} (\Gamma)$ by the following presentation. \[ \widetilde{\Tr} (\Gamma) = \langle V (\Gamma) \mid x^{\mu(x)} = 1 \text{ for } x \in V (\Gamma) \text{ such that } \mu (x) \neq \infty\,,\ x\, \varphi_x (y) = y\, \varphi_y (x) \text{ for } \{ x, y \} \in E (\Gamma) \rangle\,. \] Trickle groups and dual trickle groups are related by the following. \begin{prop}\label{prop2_2} Let $\Gamma = (\Gamma, \le, \mu, (\varphi_x)_{x \in V(\Gamma)})$ be a trickle graph. Then $\widetilde{\Tr} (\widetilde{\Gamma})=\Tr (\Gamma)$. \end{prop} \begin{proof} The group $\widetilde{\Tr} (\widetilde{\Gamma})$ has the following presentation. \begin{gather*} \widetilde{\Tr} (\widetilde{\Gamma}) = \langle V (\Gamma) \mid x^{\mu (x)} = 1 \text{ for } x \in V (\Gamma) \text{ such that } \mu (x) \neq \infty\,,\ x\, \varphi_x^{-1} (y) = y \, \varphi_y^{-1} (x)\\ \text{ for } \{x, y\} \in E(\Gamma) \rangle\,. \end{gather*} Let $f : V(\Gamma) \to \widetilde{\Tr} (\widetilde{\Gamma})$ be the map defined by $f(x) = x$ for all $x \in V(\Gamma)$. Let $x \in V(\Gamma)$ be such that $\mu (x) \neq \infty$. Then $f(x)^{\mu (x)} = x^{\mu (x)} = 1$. Let $e = \{x, y\} \in E (\Gamma)$. If $x || y$, then with $e$ we associate the relation $xy = yx$ in the presentation of $\Tr (\Gamma)$ as well as in that of $\widetilde{\Tr} (\widetilde{\Gamma})$. So, $f(x) \, f(y) = f(y) \, f(x)$. Suppose $x < y$. The case $y < x$ is treated in the same way. With $e$ we associate the relation $y x = \varphi_y(x)\,y$ in the presentation of $\Tr (\Gamma)$. Let $x' = \varphi_y (x)$. By definition of a trickle graph we have $e' =\{ x' ,y \} \in E(\Gamma)$ and $x' < y$. With $e'$ we associate the relation $y \, \varphi_y^{-1} (x') = x' y$ in the presentation of $\widetilde{\Tr} (\widetilde{\Gamma})$. But, since $x' = \varphi_y (x)$, this relation is also read $y x = \varphi_y(x)\, y$. Thus, $f(y) \, f(x) = f(\varphi_y(x)) \, f(y)$. This shows that $f$ induces a homomorphism $f : \Tr (\Gamma) \to \widetilde{\Tr} (\widetilde{\Gamma})$. We show in the same way that we have a homomorphism $f': \widetilde{\Tr} (\widetilde{\Gamma}) \to \Tr (\Gamma)$ which sends $x$ to $x$ for all $x \in V(\Gamma)$. It is clear that $f'$ is the inverse of $f$, hence $f$ is an isomorphism. \end{proof} \begin{rem} The proof of Proposition \ref{prop2_2} proves more than what is stated: it actually proves that the presentation of $\widetilde{\Tr} (\widetilde{\Gamma})$ is equal to that of $\Tr (\Gamma)$. Nevertheless, even if the two presentations coincide, it will be useful subsequently to call the presentation \[ \langle V (\Gamma) \mid x^{\mu (x)} = 1 \text{ for } x \in V (\Gamma) \text{ such that } \mu (x) \neq \infty\,,\ x\,\varphi_x^{-1} (y) = y \, \varphi_y^{-1} (x) \text{ for } \{x, y\} \in E(\Gamma) \rangle \] the \emph{dual presentation} of $\Tr (\Gamma)$. \end{rem} \end{expl3} \subsection{The trickle algorithm -- Results of Section \ref{sec4}}\label{subsec2_2} Let $A$ be a set, which we call an \emph{alphabet}, and let $A^*$ be the free monoid on $A$. The elements of $A^*$ are called \emph{words} and they are written as finite sequences. The empty word is denoted by $\epsilon$ and the concatenation of two words $w_1, w_2 \in A^*$ is denoted by $w_1 \cdot w_2$. A \emph{rewriting system} on $A^*$ is defined to be a subset $R \subseteq A^* \times A^*$. Let $w, w' \in A^*$. We set $w \stackrel{R}{\to} w'$ or simply $w \to w'$ if there exist $w_1, w_2 \in A^*$ and $(u,v) \in R$ such that $w = w_1 \cdot u \cdot w_2$ and $w' = w_1 \cdot v \cdot w_2$. More generally, we set $w \stackrel{R\ *}{\to} w'$ or simply $w \to^* w'$ if either $w = w'$ or there exists a finite sequence $w = w_0, w_1, \dots, w_p = w'$ in $A^*$ such that $w_{i-1} \to w_i$ for all $i \in \{1, \dots, p\}$. A word $w \in A^*$ is said to be \emph{$R$-reducible} if there exists $w' \in A^*$ such that $w \to w'$. Otherwise we say that $w$ is \emph{$R$-irreducible}. The pair $(A, R)$ is a \emph{rewriting system for a monoid} $M$ if $\langle A \mid u = v \text{ for } (u,v) \in R \rangle^+$ is a monoid presentation for $M$. A \emph{rewriting system for a group} $G$ is a rewriting system for $G$ viewed as a monoid. In particular, in this case $A$ generates $G$ as a monoid. If $(A, R)$ is a rewriting system for a monoid $M$ and $w=(\alpha_1, \alpha_2, \dots,\alpha_\ell) \in A^*$, then we denote by $\overline{w} = \alpha_1 \alpha_2 \dots \alpha_\ell$ the element of $M$ represented by $w$. Let $R$ be a rewriting system on $A^*$. We say that $R$ is \emph{terminating} if there is no infinite sequence $\{w_k\}_{k=0}^\infty$ in $A^*$ such that $w_{k-1} \to w_k$ for all $k \in \N_{\ge 1}$. We say that $R$ is \emph{confluent} if, for all $u, v_1, v_2 \in A^*$ such that $u \to^* v_1$ and $u \to^* v_2$, there exists $w \in A^*$ such that $v_1 \to^* w$ and $v_2 \to^* w$. The importance of terminating and confluent rewriting systems comes from the following. \begin{thm}[Newman \cite{Newma1}]\label{thm2_3} Let $(A,R)$ be a terminating and confluent rewriting system for a monoid $M$. \begin{itemize} \item[(1)] For all $w'\in A^*$ there exists a unique $R$-irreducible word $w \in A^*$ such that $w' \to^* w$. \item[(2)] For all $g \in M$ there exists a unique $R$-irreducible word $w \in A^*$ such that $g = \overline{w}$. \end{itemize} \end{thm} Now, we fix a trickle graph $\Gamma = (\Gamma, \le, \mu, (\varphi_x)_{x \in V (\Gamma)})$, and we turn to describe a rewriting system for $\Tr (\Gamma)$. The alphabet of our rewriting system is not $V(\Gamma) \sqcup V (\Gamma)^{-1}$, as one may expect, but a more complicated set $\Omega = \Omega (\Gamma)$, which is generally infinite, and which is described as follows. Throughout the paper we use the following notations. For $\mu \in \N_{\ge 2} \cup \{\infty\}$ we set $\Z_\mu = \Z / \mu \Z$ if $\mu \neq \infty$ and $\Z_\mu = \Z$ if $\mu = \infty$. The set of \emph{syllables} of $\Gamma$ is the abstract set \[ S (\Gamma) = \{x^a \mid x \in V (\Gamma) \text{ and } a \in \Z_{\mu (x)} \setminus \{0\} \}\,. \] A \emph{stratum} of $\Gamma$ is a finite subset $U = \{x_1^{a_1}, x_2^{a_2}, \dots, x_p^{a_p} \} \subseteq S (\Gamma)$ such that $x_i \neq x_j$ and $\{x_i, x_j\} \in E (\Gamma)$ for all $i, j \in \{1, \dots, p\}$ such that $i \neq j$. The \emph{support} of $U$ is $\supp (U) = \{x_1, x_2, \dots, x_p\} \subseteq V (\Gamma)$ and its \emph{length} is the integer $p$, which is denoted by $\lg_\st (U)$. The empty set $\emptyset$ is assumed to be a stratum whose support is $\emptyset$. The set of strata is denoted by $\Omega = \Omega (\Gamma)$. The set $\Omega$ is the alphabet of our rewriting system. The elements of $\Omega^*$ are called \emph{pilings}. If $u = (U_1, \dots, U_p)$ is a piling, then $p$ is the \emph{length} of $u$, which is denoted by $\lg_\pil (u)$. Now, we define three operations on the strata which will be used to define our rewriting system. The first operation consists in removing an element from a stratum. If $U = \{x_1^{a_1}, x_2^{a_2}, \dots, x_p^{a_p}\}$ is a non-empty stratum and $x_i^{a_i} \in U$, then we set \[ L (U, x_i^{a_i}) = U \setminus \{ x_i^{a_i} \} = \{ x_1^{a_1}, \dots, x_{i-1}^{a_{i-1}} , x_{i+1}^{a_{i+1}}, \dots, x_p^{a_p}\}\,. \] Note that $L (U, x_i^{a_i})$ is a stratum. The second operation consists in ``extracting'' a syllable from a stratum. Let $U = \{x_1^{a_1}, x_2^{a_2}, \dots, x_p^{a_p}\}$ be a non-empty stratum and let $x_i^{a_i} \in U$. We number the elements of $U$ such that, if $x_j > x_k$, then $j < k$. Then we set \[ \gamma (U, x_i^{a_i}) = \big( (\varphi_{x_1}^{a_1} \circ \varphi_{x_2}^{a_2} \circ \cdots \circ \varphi_{x_{i-1}}^{a_{i-1}}) (x_i)\big)^{a_i}\,. \] It is easily seen that $\gamma(U, x_i^{a_i})$ is well-defined and belongs to $S(\Gamma)$. Moreover, it will be proved in Section \ref{sec4} (see Lemma \ref{lem4_2}) that the definition of $\gamma (U, x_i^{a_i})$ does not depend on the choice of the numbering of the elements of $U$. The third operation consists in ``adding'' a syllable to a stratum. Let $U = \{x_1^{a_1}, x_2^{a_2}, \dots, x_p^{a_p} \} \in \Omega$ and let $y^b \in S(\Gamma)$. We say that $y^b$ can be \emph{added} to $U$ if either $y \in \supp (U)$ or $\{y, x_i\} \in E(\Gamma)$ for all $i \in \{1, \dots, p\}$. Suppose $y^b$ can be added to $U$. If $y \not \in \supp (U)$, then we set \[ R (U, y^b) = \{ \varphi_y^{-b} (x_1)^{a_1}, \dots, \varphi_y^{-b} (x_p)^{a_p}, y^b\}\,. \] If $y = x_i \in \supp (U)$ and $b + a_i = 0$ (in $\Z_{\mu (y)}$), then we set \[ R(U, y^b) = \{\varphi_y^{-b} (x_1)^{a_1}, \dots, \varphi_y^{-b} (x_{i-1})^{a_{i-1}}, \varphi_y^{-b} (x_{i+1})^{a_{i+1}}, \dots, \varphi_y^{-b} (x_p)^{a_p}\}\,. \] If $y = x_i \in \supp (U)$ and $b + a_i \neq 0$ (in $\Z_{\mu (y)}$), then we set \[ R (U, y^b) = \{ \varphi_y^{-b} (x_1)^{a_1}, \dots, \varphi_y^{-b} (x_{i-1})^{a_{i-1}}, y^{a_i+b}, \varphi_y^{-b} (x_{i+1})^{a_{i+1}}, \dots, \varphi_y^{-b} (x_p) ^{a_p}\}\,. \] Note that, in the third case, since $y = x_i = \varphi_y^{-b}(x_i)$, $y^{a_i+b}$ can be replaced by $\varphi_y^{-b} (x_i)^{a_i+b}$. Note also that $R (U, y^b)$ is always a stratum. \begin{defin} Let $(U, V)$ be a pair of strata with $V \neq \emptyset$ and let $x^a \in V$. We set $V' = L(V, x^a)$ and $y^a = \gamma (V, x^a)$. We assume that $y^a$ can be added to $U$ and we set $U' = R(U, y^a)$. Then we say that $r = ((U, V), (U', V')) \in \Omega^* \times \Omega^*$ is a \emph{T-transformation}. In this case we write $T (U, V, x^a) = (U', V')$. We denote by $\RR_1$ the set of T-transformations. On the other hand, we set $\RR_0 = \{ ((\emptyset), \epsilon)\} \subset \Omega^* \times \Omega^*$, where $(\emptyset)$ is the piling of length $1$ whose only entry is $\emptyset$ and $\epsilon$ is the empty piling of length $0$. Finally, we set $\RR = \RR (\Gamma)= \RR_0 \cup \RR_1$. \end{defin} The following will be proved in Section \ref{sec4}. \begin{thm}\label{thm2_4} Let $\Gamma = (\Gamma, \le, \mu, (\varphi_x)_{x \in V(\Gamma)})$ be a trickle graph, let $\Omega = \Omega (\Gamma)$ be the set of strata of $\Gamma$, and let $\RR = \RR (\Gamma) \subseteq \Omega^* \times \Omega^*$ be as defined above. Then $\RR$ is a rewriting system for $\Tr (\Gamma)$, and it is terminating and confluent. \end{thm} As a consequence of Theorem \ref{thm2_4} we get that $\langle \Omega \mid u = v \text{ for } (u,v) \in \RR \rangle^+$ is a monoid presentation for $\Tr (\Gamma)$. We explain how to go from this presentation to the standard one and vice versa. Set $M (\Gamma) = \langle \Omega \mid u = v \text{ for } (u,v) \in \RR \rangle^+$. Let $U \in \Omega$. We write $U = \{ x_1^{a_1}, x_2^{a_2}, \dots, x_p^{a_p} \}$ so that, if $x_i > x_j$, then $i < j$, and we set $\omega (U) = x_1^{a_1} x_2^{a_2} \dots x_p^{a_p} \in \Tr (\Gamma)$. We will show in Lemma \ref{lem4_13} that the definition of $\omega (U)$ does not depend on the choice of the numbering of the elements of $U$. Then the isomorphism $\Phi: M (\Gamma) \to \Tr (\Gamma)$ sends $U$ to $\omega (U)$ for all $U \in \Omega$. The reverse isomorphism $\Psi: \Tr (\Gamma) \to M (\Gamma)$ sends $x$ (resp. $x^{-1}$) to the piling $(\{x\})$ (resp. $(\{ x^{-1} \})$) of length $1$ for all $x \in V (\Gamma)$. Details and proofs about these isomorphisms will be given in Section \ref{sec4}. Recall that, if $G$ is a group and $V$ is a generating set for $G$, then a \emph{set of normal forms} for $G$ based on $V \sqcup V^{-1}$ is a language $\LL \subseteq (V \sqcup V^{-1})^*$ such that, for all $g \in G$, there exists a unique $w \in \LL$ such that $\overline{w} = g$. The above constructions provide normal forms for $\Tr (\Gamma)$ based on $V (\Gamma) \sqcup V(\Gamma)^{-1}$ as well as an algorithm to calculate them when $V (\Gamma)$ is finite, as follows. Let $\mu \in \N_{\ge 2} \cup \{ \infty \}$ and $a \in \Z_\mu \setminus \{ 0\}$. If $\mu = \infty$, then we set $\rho (a) = a$. If $\mu \neq \infty$, then we denote by $\rho (a)$ the unique representative of $a$ sitting inside $\{1, \dots, \mu-1\}$. We fix a total order $\preceq$ on $V (\Gamma)$ which extends the partial order $\le$ in the sense that, if $x \le y$, then $x \preceq y$. Such a total order always exists but it is not unique in general. Let $U$ be a non-empty stratum that we write $U = \{ x_1^{a_1}, x_2^{a_2}, \dots, x_p^{a_p} \}$ with $x_1 \succ x_2 \succ \cdots \succ x_p$. Then we set $\hat \omega (U) = x_1^{\rho (a_1)} \cdot x_2^{\rho (a_2)} \cdots x_p^{\rho (a_p)} \in (V (\Gamma) \sqcup V (\Gamma)^{-1})^*$. Note that $\hat \omega (U)$ is a representative of $\omega (U)$. Let $g \in \Tr (\Gamma)$. By Theorems \ref{thm2_3} and \ref{thm2_4} there exists a unique piling $w = (U_1, U_2, \dots, U_p) \in \Omega^*$ such that $w$ is $\RR$-irreducible and $\overline{w} = \Psi (g)$. Then we set \[ \nf (g) = \hat \omega (U_1) \cdot \hat \omega (U_2) \cdots \hat \omega (U_p) \in (V (\Gamma) \sqcup V (\Gamma)^{-1})^*\,. \] The following is a direct consequence of the previous constructions. \begin{corl}\label{corl2_5} Let $\Gamma = (\Gamma, \le, \mu, (\varphi_x)_{x \in V(\Gamma)})$ be a trickle graph. \begin{itemize} \item[(1)] The set $\LL = \LL (\Gamma) = \{ \nf (g) \mid g \in \Tr (\Gamma) \}$ is a set of normal forms for $\Tr (\Gamma)$ based on $V (\Gamma) \sqcup V (\Gamma)^{-1}$. \item[(2)] Suppose that $V( \Gamma)$ is finite. Then there exists an algorithm which, given a word $w \in (V(\Gamma) \sqcup V(\Gamma)^{-1})^*$, calculates $\nf (\overline{w})$. In particular, this algorithm is a solution to the word problem in $\Tr (\Gamma)$. \end{itemize} \end{corl} \begin{rem} Corollary \ref{corl2_5}\,(2) can be extended to a trickle graph with infinite but countable vertex set $V (\Gamma)$ provided that there exist an algorithm to manipulate the elements of $V (\Gamma)$, an algorithm which, given $x, y \in V (\Gamma)$, decides whether $\{x, y \} \in E (\Gamma)$ or not, an algorithm which, given $x,y \in V (\Gamma)$, decides whether $x \le y$ or not, and an algorithm which, given $x \in V (\Gamma)$ and $y \in \starE_x (\Gamma)$, determines $\varphi_x (y)$. \end{rem} Another straightforward consequence of Theorems \ref{thm2_3} and \ref{thm2_4} is the following. \begin{corl}\label{corl2_6} Let $\Gamma = (\Gamma, \le, \mu, (\varphi_x)_{x \in V(\Gamma)})$ be a trickle graph. Then the map $S (\Gamma) \to \Tr (\Gamma)$, $x^a \mapsto x^a$, is injective. In particular, $V(\Gamma)$ is a subset of $\Tr (\Gamma)$. \end{corl} \begin{proof} The result follows from the fact that, for $x^a \in S(\Gamma)$, $x^{\rho (a)} \in (V(\Gamma) \sqcup V (\Gamma)^{-1})^*$ is the normal form of $x^a \in \Tr (\Gamma)$. \end{proof} Another more unexpected consequence is the following. \begin{corl}\label{corl2_7} Let $\Gamma = (\Gamma, \le, \mu, (\varphi_x)_{x \in V(\Gamma)})$ be a trickle graph. Then $\Tr (\Gamma)$ is finite if and only if $V(\Gamma)$ is finite, $\Gamma$ is complete, and $\mu (x) \neq \infty$ for all $x \in V(\Gamma)$. \end{corl} \begin{proof} Suppose $V (\Gamma)$ is infinite. Then $\Tr (\Gamma)$ is infinite because $V(\Gamma) \subseteq \Tr (\Gamma)$. Suppose there exists $x \in V(\Gamma)$ such that $\mu (x) = \infty$. Then, by Corollary \ref{corl2_6}, $\{ x^a \mid a \in \Z \setminus \{0\} \} \subseteq S (\Gamma) \subseteq \Tr (\Gamma)$, hence $\Tr (\Gamma)$ is infinite. Suppose $\Gamma$ is not complete. Let $x,y \in V(\Gamma)$ be such that $\{x, y\} \not \in E (\Gamma)$. Let $u=(\{x\}, \{y\}) \in \Omega^*$. Then, for all $n \in \N$, $u^n$ is an $\RR$-irreducible piling of length $2n$, hence, by Theorems \ref{thm2_3} and \ref{thm2_4}, the set $\{\overline{u ^n} \mid n \in \N\}$ is an infinite subset of $\Tr (\Gamma)$, and therefore $\Tr (\Gamma)$ is infinite. Suppose $V(\Gamma)$ is finite, $\Gamma$ is complete, and $\mu (x) \neq \infty$ for all $x \in V(\Gamma)$. We fix a total order $\preceq$ on $V (\Gamma)$ which extends the partial order $\le$, and we write $V (\Gamma) = \{ x_1, x_2, \dots, x_p \} $ with $x_1 \succ x_2 \succ \cdots \succ x_p$. Then the set of normal forms for $\Tr (\Gamma)$ is \[ \{ x_{i_1}^{a_1} \cdot x_{i_2}^{a_2} \cdots x_{i_q}^{a_q} \mid 1 \le i_1 < i_2 < \cdots < i_q \le p\,,\ 1 \le a_j \le \mu (x_{i_j})-1 \text{ for } 1 \le j \le q\}\,. \] This set is finite and it is in one-to-one correspondence with $\Tr (\Gamma)$, hence $\Tr (\Gamma)$ is finite. \end{proof} \subsection{The Tits-style algorithm -- Results of Section \ref{sec5}}\label{subsec2_3} Let $\Gamma = (\Gamma, \le, \mu, (\varphi_x)_{x\in V(\Gamma)})$ be a trickle graph. Recall that the set of \emph{syllables} of $\Tr (\Gamma)$ is $S (\Gamma) = \{x^a \mid x \in V(\Gamma) \text{ and } a \in \Z_{\mu (x)} \setminus\{0\} \}$. Recall also that, by Corollary \ref{corl2_6}, $S (\Gamma)$ is a subset of $\Tr (\Gamma)$. The elements of $S(\Gamma)^*$ are called \emph{syllabic words}. For $w=(x_1^{a_1}, x_2^{a_2}, \dots, x_p^{a_p}) \in S(\Gamma)^*$ we denote by $\overline{w} = x_1^{a_1} x_2^{a_2} \dots x_p^{a_p}$ the element of $\Tr (\Gamma)$ represented by $w$. The integer $p$ is called the \emph{length} of $w$ and it is denoted by $\lg (w)$. The smallest length of a syllabic word representing an element $g \in \Tr (\Gamma)$ is called the \emph{syllabic length} of $g$ and it is denoted by $\lg_{\syl} (g)$. A syllabic word $w = (x_1^{a_1}, x_2^{a_2}, \dots, x_p^{a_p})$ is said to be \emph{syllabically reduced} if $\lg (w) = \lg_\syl (\overline{w})$. Our Tits-style algorithm uses a rewriting system on $S (\Gamma)^*$, but it does not use Newman's results \cite{Newma1} stated in Theorem \ref{thm2_3}. This rewriting system is defined as follows. We set \begin{gather*} \RR_{I} = \{ ((x^a, x^{-a}), \epsilon) \mid x \in V (\Gamma)\,,\ a \in \Z_{\mu (x)} \setminus \{0\} \}\ \cup\\ \{ ((x^a, x^b),(x^{a+b})) \mid x \in V (\Gamma)\,, \ a, b \in \Z_{\mu (x)} \setminus \{0\} \text{ and } a + b \neq 0\}\,,\\ \RR_{II} = \{ ((x^a, y^b), (\varphi_x^a (y)^b, \varphi_y^{-b}(x)^a)) \mid x^a, y^b \in S (\Gamma) \text{ and } \{x,y\} \in E (\Gamma) \}\,,\\ \RR_M = \RR_I \cup \RR_{II}\,. \end{gather*} Let $x^a, y^b \in S (\Gamma)$ be such that $\{x,y\} \in E (\Gamma)$. Note that, if $x || y$, then $(\varphi_x^a (y)^b, \varphi_y^{-b}(x)^a) = (y^b, x^a)$, if $x > y$, then $ (\varphi_x^a (y)^b, \varphi_y^{-b}(x)^a) = (\varphi_x^a (y)^b, x^a)$, and, if $x < y$, then $(\varphi_x^a (y)^b, \varphi_y^{-b}(x)^a) = (y^b, \varphi_y^{-b}(x)^a)$. Note also that in all three cases we have the equality $x^a y^b = \varphi_x^a (y)^b\, \varphi_y^{-b}(x)^a$ in $\Tr (\Gamma)$. For $X \in \{I, II, M\}$ and $w, w' \in S (\Gamma)^*$, we write $w \stackrel{X}{\to} w'$ in place of $w \stackrel{\RR_X}{\to} w'$ and $w \stackrel{X\ *}{\to} w'$ in place of $w \stackrel{\RR_X\, *}{\longrightarrow} w'$. Let $w, w' \in S (\Gamma)^*$. Notice that, if $w \stackrel{I}{\to} w'$, then $\lg (w) > \lg (w')$. In particular, the operation $\stackrel{I}{\to}$ is not reversible. On the other hand the operation $\stackrel{II}{\to}$ is reversible in the sense that, if $w \stackrel{II}{\to} w'$, then $\lg (w) = \lg (w')$ and $w' \stackrel{II}{\to} w$. We say that a syllabic word $w \in S (\Gamma)^*$ is \emph{syllabically $M$-reduced} if there is no word $w' \in S (\Gamma)^*$ such that $w \stackrel{M\ *}{\to} w'$ and $\lg (w') < \lg (w)$. The following theorem is similar to the classical solution to the word problem for Coxeter groups \cite{Tits1}, for graph products of cyclic groups \cite{Green1}, and, more generally, for Dyer groups \cite{ ParSoe1}. It will be proved in Section \ref{sec5}. \begin{thm}\label{thm2_8} Let $\Gamma = (\Gamma, \le, \mu, (\varphi_x)_{x\in V(\Gamma)})$ be a trickle graph. \begin{itemize} \item[(1)] For every $w \in S (\Gamma)^*$, $w$ is syllabically reduced if and only if $w$ is syllabically $M$-reduced. \item[(2)] For every $w, v \in S (\Gamma)^*$, if $w$ and $v$ are syllabically reduced and $\overline{w} = \overline{v}$, then $w \stackrel{II\, *}{\to} v$. \end{itemize} \end{thm} \subsection{Parabolic subgroups -- Results of Section \ref{sec6}}\label{subsec2_4} \begin{defin} Let $\Gamma = (\Gamma, \le, \mu, (\varphi_x)_{x\in V(\Gamma)})$ be a trickle graph. A full subgraph $\Gamma_1$ of $\Gamma$ is called \emph{parabolic} if, for every $x \in V(\Gamma_1)$, the map $\varphi_x$ stabilizes $\starE_x (\Gamma_1)$. \end{defin} \begin{expl} Let $\Gamma = (\Gamma, \le, \mu, (\varphi_x)_{x\in V(\Gamma)})$ be a trickle graph. \begin{itemize} \item[(1)] If $X$ is a subset of $V(\Gamma)$ whose elements are pairwise incomparable, then the full subgraph of $\Gamma$ spanned by $X$ is parabolic. \item[(2)] If $X$ is a subset of $V(\Gamma)$ and $X_\downarrow = \{y \in V(\Gamma) \mid \exists x \in X, y \leq x\}$, then the full subgraph of $\Gamma$ spanned by $X_\downarrow$ is parabolic. \end{itemize} \end{expl} The following can be easily checked. \begin{lem}\label{lem2_9} Let $\Gamma = (\Gamma, \le, \mu, (\varphi_x)_{x\in V(\Gamma)})$ be a trickle graph and let $\Gamma_1$ be a parabolic subgraph of $\Gamma$. We denote by $\le_1$ and by $\mu_1$ the restrictions of $\le$ and $\mu$ to $V(\Gamma_1)$, respectively, and, for $x \in V (\Gamma_1)$, we denote by $\varphi_{1,x}$ the restriction of $\varphi_x$ to $\starE_x (\Gamma_1)$. \begin{itemize} \item[(1)] The quadruple $\Gamma_1 = (\Gamma_1, \le_1, \mu_1, (\varphi_{1,x})_{x\in V(\Gamma_1)})$ is a trickle graph. \item[(2)] The embedding of $V(\Gamma_1)$ into $V(\Gamma)$ induces a group homomorphism $\iota_1: \Tr (\Gamma_1) \to \Tr (\Gamma)$. \end{itemize} \end{lem} In our study in Section \ref{sec6} we will use the normal forms for $\Tr (\Gamma)$ as they are defined in Subsection \ref{subsec2_2}. We recall them. We fix a total order $\preceq$ on $V(\Gamma)$ which extends the partial order $\le$ in the sense that, if $x \le y$, then $x \preceq y$. If $U$ is a non-empty stratum that we write $U = \{ x_1^{a_1}, x_2^{a_2}, \dots, x_p^{a_p} \}$ with $x_1 \succ x_2 \succ \cdots \succ x_p$, then we set $\hat \omega (U) = x_1^{\rho (a_1)} \cdot x_2^{\rho (a_2)} \cdots x_p^{\rho (a_p)} \in (V (\Gamma) \sqcup V (\Gamma)^{-1})^*$. Let $g \in \Tr (\Gamma)$. By Theorems \ref{thm2_3} and \ref{thm2_4} there exists a unique piling $w = (U_1, U_2, \dots, U_p) \in \Omega^*$ such that $w$ is $\RR$-irreducible and $\overline{w} = g$. Then the \emph{normal form} of $g$ is \[ \nf_\Gamma (g) = \nf (g) = \hat \omega (U_1) \cdot \hat \omega (U_2) \cdots \hat \omega (U_p) \in (V (\Gamma) \sqcup V (\Gamma)^{-1})^*\,. \] Let $\Gamma_1$ be a parabolic subgraph of $\Gamma$. As before, we assume that $V (\Gamma_1)$ is endowed with the restriction $\le_1$ of $\le$ to $V (\Gamma_1)$ and with the restriction $\mu_1: V (\Gamma_1) \to \N_{\ge 2} \cup \{ \infty\}$ of $\mu$ to $V (\Gamma_1)$. Also, for each $x \in V (\Gamma_1)$, we denote by $\varphi_{1,x}$ the restriction of $\varphi_x$ to $\starE_x (\Gamma_1)$. Thus, as pointed out in Lemma \ref{lem2_9}, $\Gamma_1 = (\Gamma_1, \le_1, \mu_1, (\varphi_{1,x})_{x \in V (\Gamma_1)})$ is a trickle graph. Now, we denote by $\preceq_1$ the restriction of $\preceq$ to $V (\Gamma_1)$ and we observe that $\preceq_1$ extends the partial order $\le_1$, hence it can be used to define normal forms for $\Tr (\Gamma_1)$. Unless otherwise stated, we will always use this total order to define normal forms for $\Tr (\Gamma_1)$. The main result of Section \ref{sec6} is the following. \begin{thm}\label{thm2_10} Let $\Gamma = (\Gamma, \le, \mu, (\varphi_x)_{x\in V(\Gamma)})$ be a trickle graph and let $\Gamma_1$ be a parabolic subgraph of $\Gamma$. \begin{itemize} \item[(1)] The homomorphism $\iota_1: \Tr (\Gamma_1) \to \Tr (\Gamma)$ induced by the embedding of $V (\Gamma_1)$ into $V (\Gamma)$ is injective. In particular, we can identify $\Tr (\Gamma_1)$ with its image in $\Tr (\Gamma)$ under $\iota_1$. \item[(2)] Let $g \in \Tr (\Gamma)$. We have $g \in \Tr (\Gamma_1)$ if and only if $\nf (g) \in (V (\Gamma_1) \sqcup V(\Gamma_1)^{-1})^*$. Moreover, in this case, $g$ has the same normal form in $\Tr (\Gamma)$ as in $\Tr (\Gamma_1)$, that is, $\nf_\Gamma (g) = \nf_{\Gamma_1} (g)$. \end{itemize} \end{thm} \begin{defin} Let $\Gamma = (\Gamma, \le, \mu, (\varphi_x)_{x \in V (\Gamma)})$ be a trickle graph and let $\Gamma_1$ be a parabolic subgraph of $\Gamma$. Then $\Tr (\Gamma_1)$, viewed as a subgroup of $\Tr (\Gamma)$, is called a \emph{standard parabolic subgroup} of $\Tr (\Gamma)$. \end{defin} \begin{expl} Let $(W,S)$ be a Coxeter system. Let $\Gamma = (\Gamma, \le, \mu, (\varphi_x)_{x \in V (\Gamma)})$ be the trickle graph such that $\Tr (\Gamma) = C(W, S)$, as defined in Subsection \ref{subsec2_1}. n} \}$. Let $Y \subseteq S$. n} \mid X \subseteq Y \}$ and we denote by $\Gamma_Y$ the full subgraph of $\Gamma$ spanned by $\{ x_X \mid X \in \SS_{Y,\fin}\}$ (Caution: do not confuse $\Gamma_Y$ with the full Coxeter subgraph spanned by $Y$). The proof of the following is left to the reader. \begin{lem}\label{lem2_11} Under the above assumptions, $\Gamma_Y$ is a parabolic subgraph of $\Gamma$ and $\Tr (\Gamma_Y)$ is naturally isomorphic to the cactus group $C (W_Y, Y)$. In particular, $C(W_Y, Y)$ is a standard parabolic subgroup of $C(W, S)$. \end{lem} \end{expl} If $\Gamma_1$ and $\Gamma_2$ are two full subgraphs of a trickle graph $\Gamma$, then we denote by $\Gamma_1 \cap \Gamma_2$ the full subgraph of $\Gamma$ spanned by $V (\Gamma_1) \cap V (\Gamma_2)$. It is easily seen that, if $\Gamma_1$ and $\Gamma_2$ are both parabolic subgraphs, then $\Gamma_1 \cap \Gamma_2$ is also a parabolic subgraph. Now, from Theorem \ref{thm2_10} we deduce the following. \begin{corl}\label{corl2_12} Let $\Gamma = (\Gamma, \le, \mu, (\varphi_x)_{x\in V(\Gamma)})$ be a trickle graph and let $\Gamma_1, \Gamma_2$ be two parabolic subgraphs of $\Gamma$. Then $\Tr (\Gamma_1) \cap \Tr (\Gamma_2) = \Tr (\Gamma_1 \cap \Gamma_2)$. In particular, the intersection of two standard parabolic subgroups of $\Tr (\Gamma)$ is a standard parabolic subgroup. \end{corl} \begin{proof} The inclusion $\Tr (\Gamma_1 \cap \Gamma_2) \subseteq \Tr (\Gamma_1) \cap \Tr (\Gamma_2)$ is obvious. We prove the reverse inclusion. Let $g \in \Tr (\Gamma_1) \cap \Tr (\Gamma_2)$. By Theorem \ref{thm2_10}, $\nf (g) \in (V (\Gamma_1) \sqcup V (\Gamma_1)^{-1})^*$ and $\nf (g) \in (V (\Gamma_2 ) \sqcup V (\Gamma_2)^{-1})^*$. But \[ (V (\Gamma_1) \sqcup V (\Gamma_1)^{-1})^* \cap (V (\Gamma_2) \sqcup V (\Gamma_2)^{-1})^* = (V (\Gamma_1 \cap \Gamma_2) \sqcup V (\Gamma_1 \cap \Gamma_2)^{-1})^*\,, \] hence $\nf (g) \in (V (\Gamma_1 \cap \Gamma_2) \sqcup V (\Gamma_1 \cap \Gamma_2)^{-1})^*$, and therefore $g \in \Tr (\Gamma_1 \cap \Gamma_2)$. \end{proof} Another straightforward consequence of Theorem \ref{thm2_10} is the following. \begin{corl}\label{corl2_13} Let $\Gamma = (\Gamma, \le, \mu, (\varphi_x)_{x \in V(\Gamma)})$ be a trickle graph with $V (\Gamma)$ finite, and let $\Gamma_1$ be a parabolic subgraph of $\Gamma$. There exists an algorithm which, given $g \in \Tr (\Gamma)$, decides whether $g$ belongs to $\Tr (\Gamma_1)$ or not. \end{corl} \begin{proof} Let $g \in \Tr (\Gamma)$. Then, by Theorem \ref{thm2_10}, $g \in \Tr (\Gamma_1)$ if and only if $\nf (g) \in (V (\Gamma_1) \sqcup V (\Gamma_1)^{-1})^*$. \end{proof} \begin{rem} As with Corollary \ref{corl2_5}\,(2), Corollary \ref{corl2_13} can be extended to the case where $V (\Gamma)$ is infinite but countable, provided that one has an algorithm for manipulating the elements of $V (\Gamma)$, an algorithm which, given $x,y \in V (\Gamma)$, decides whether $\{ x,y\}$ belongs to $E (\Gamma)$ or not, an algorithm which, given $x,y \in V (\Gamma)$, decides whether $x < y$ or not, an algorithm which, given $x \in V (\Gamma)$ and $y \in \starE_x (\Gamma)$, calculates $\varphi_x (y)$, and an algorithm which, given $x \in V (\Gamma)$, decides whether $x \in V (\Gamma_1)$ or not. \end{rem} \subsection{PreGarside trickle groups -- Results of Section \ref{sec7}}\label{subsec2_5} We start by recalling some definitions and notations on monoids. We say that a monoid $M$ is \emph{cancellative} if, for all $a, b, c, d \in M$, if $c a d = c b d$, then $a = b$. We say that $M$ is \emph{atomic} if there exists a map $\nu : M \to \N$, called a \emph{norm}, such that $\nu (a) = 0$ if and only if $a=1$, and $\nu (a b) \ge \nu (a) + \nu (b)$ for all $a, b \in M$. If $M$ is atomic, then we can define two partial orders $\le_L$ and $\le_R$ on $M$ as follows. Let $a, b \in M$. We set $a \le_L b$ if there exists $c \in M$ such that $a c = b$, and we set $a \le_R b$ if there exists $c' \in M$ such that $c' a = b$. For $b \in M$ we set $\Div_L (b) = \{a \in M \mid a \le_L b \}$ and $\Div_R (b) = \{ a \in M \mid a \le_R b\}$. We say that $b$ is \emph{balanced} if $\Div_L (b) = \Div_R (b)$, and, in this case, we set $\Div (b) = \Div_L (b) = \Div_R (b)$. The enveloping group of a monoid $M$ is denoted by $G(M)$ and the natural homomorphism from $M$ to $G (M)$ is denoted by $\iota_M : M \to G(M)$. \begin{defin} A monoid $M$ is called a \emph{preGarside monoid} if it satisfies the following three properties. \begin{itemize} \item[(a)] $M$ is cancellative and atomic. \item[(b)] For all $a,b \in M$, if the set $\{ c \in M \mid a \le_L c \text{ and } b \le_L c \}$ is non-empty, then it contains a least element that we denote by $a \vee_L b$. \item[(c)] For all $a,b \in M$, if the set $\{ c \in M \mid a \le_R c \text{ and } b \le_R c \}$ is non-empty, then it contains a least element that we denote by $a \vee_R b$. \end{itemize} A \emph{preGarside group} is the enveloping group of a preGarside monoid. \end{defin} \begin{defin} Let $M$ be a preGarside monoid. An element $\Delta \in M$ is called a \emph{Garside element} if $\Delta$ is balanced and $\Div (\Delta)$ generates $M$. If $M$ contains a Garside element, then $M$ is called a \emph{Garside monoid} and $G (M)$ is called a \emph{Garside group}. \end{defin} The notions of Garside monoids and Garside groups were introduced by Dehornoy and the third author in \cite{DehPar1,Dehor1}, where they show that these monoids and groups share many properties with spherical type Artin monoids and groups such as solutions to the word problem and to the conjugacy problem. The theory of Garside groups is experiencing a significant growth and many examples and applications have appeared since the publication of \cite{DehPar1,Dehor1}. On the other hand, our definitions of preGarside monoid and group are taken from \cite{GodPar2}. The motivation for their study is that, by \cite{BriSai1}, every Artin monoid is a preGarside monoid and every Artin group is a preGarside group. So, preGarside groups are to Garside groups what Artin groups are to spherical type Artin groups. \begin{defin} We say that a trickle graph $\Gamma = (\Gamma, \le, \mu, (\varphi_x)_{x \in V (\Gamma)})$ is a \emph{preGarside trickle graph} if $\mu (x) = \infty$ for all $x \in V (\Gamma)$. In this case, in addition to the trickle group $\Tr (\Gamma)$, we associate with $\Gamma$ a \emph{preGarside trickle monoid}, $\Tr^+ (\Gamma)$, which is defined by the following monoid presentation. \[ \Tr^+ (\Gamma) = \langle V (\Gamma) \mid \varphi_x (y)\, x = \varphi_y (x)\, y \text{ for all } \{ x, y \} \in E (\Gamma) \rangle^+ \,. \] It is clear that $\Tr (\Gamma)$ is the enveloping group of $\Tr^+ (\Gamma)$. \end{defin} The above definition is motivated by the following theorem which will be proved in Section \ref{sec7}. \begin{thm}\label{thm2_14} Let $\Gamma$ be a preGarside trickle graph. Then $\Tr^+ (\Gamma)$ is a preGarside monoid. \end{thm} The next question is: when is $\Tr^+ (\Gamma)$ a Garside monoid? The answer is given by the following theorem which will be also proved in Section \ref{sec7}. \begin{thm}\label{thm2_15} Let $\Gamma$ be a preGarside trickle graph. Then $\Tr^+ (\Gamma)$ is a Garside monoid if and only if $V(\Gamma)$ is finite and $\Gamma$ is a complete graph. \end{thm} \begin{rem} Any spherical type Artin group $A$ has a natural finite quotient $W$, which is its associated Coxeter group, and the projection $A \to W$ is an important tool in the study of the group. Such a quotient does not exist in general for a given Garside group, and determining which Garside groups have such quotients is a standard question in the theory (see for instance \cite{Gobet1}). Now, notice that Theorem \ref{thm2_15} provides new examples of such Garside groups. Indeed, if $\Gamma = (\Gamma, \le, \mu, (\varphi_x)_{x \in V (\Gamma)})$ is a preGarisde trickle graph with finite $V (\Gamma)$ and $\Gamma$ complete, then, by Theorem \ref{thm2_15}, $\Tr (\Gamma)$ is a Garside group and, by Corollary \ref{corl2_7}, the quotient of $\Tr (\Gamma)$ by the relations $x^{\bar \mu(x)} = 1$, $x \in V (\Gamma)$, where $\bar \mu (x)$ is the order of $\varphi_x$ if $\varphi_x$ is different from the identity and $\bar \mu (x) = 2$ if $\varphi_x$ is the identity, is a finite trickle group. \end{rem} In \cite{GodPar3} four elementary questions are asked on Artin groups, questions which are unsolved to this date, and which also arise naturally for preGarside groups. Two of them are of particular interest in this paper: \begin{itemize} \item[(1)] Is any preGarside group torsion-free? \item[(2)] Does any preGarside group has a solution to the word problem? \end{itemize} Concerning preGarside trickle groups, a positive answer to Question (2) when $V (\Gamma)$ is finite is given by Corollary \ref{corl2_5}. A positive answer to Question (1) in the case where $V (\Gamma)$ is finite is given by the following theorem which will be proved in Section \ref{sec7}. \begin{thm}\label{thm2_16} Let $\Gamma$ be a preGarside trickle graph with $V (\Gamma)$ finite. Then $\Tr (\Gamma)$ is torsion-free. \end{thm} We do not know whether the conclusion of Theorem \ref{thm2_16} holds if $V (\Gamma)$ is infinite. Another question more specific to preGarside groups is the following. \begin{itemize} \item[(3)] Let $M$ be a preGarside monoid. Is the natural homomorphism $\iota_M : M \to G(M)$ injective? \end{itemize} We know from \cite{Paris1} that the answer is yes if the monoid is an Artin monoid. The answer is also yes if the monoid is a preGarside trickle monoid as stated in the following theorem which will be proved in Section \ref{sec7}. \begin{thm}\label{thm2_17} Let $\Gamma$ be a preGarside trickle graph. Then the natural homomorphism $\iota_{\Tr^+ (\Gamma)}: \Tr^+ (\Gamma) \to \Tr (\Gamma)$ is injective. \end{thm} \begin{defin} Let $M$ be a preGarside monoid and let $N$ be a submonoid of $M$. We say that $N$ is \emph{special} if, for all $a,b \in M$, if $ab \in N$, then $a,b \in N$. We say that $N$ is a \emph{parabolic submonoid} of $M$ if it satisfies the following three properties. \begin{itemize} \item[(a)] $N$ is special. \item[(b)] For all $a,b \in N$, if $a \vee_L b$ exists, then $a\vee_L b \in N$. \item[(c)] For all $a,b \in N$, if $a \vee_R b$ exists, then $a\vee_R b \in N$. \end{itemize} \end{defin} In the case of PreGarside trickle monoids, this definition, taken from \cite{GodPar2}, coincides with that given in Subsection \ref{subsec2_4} (see Proposition \ref{prop7_8}). On the other hand, in the case of Garside monoids, it coincides with the definition of \cite{Godel1,Godel2}. In particular, a parabolic submonoid of a Garside monoid is itself a Garside monoid. Now, a consequence of Proposition \ref{prop7_8} combined with the results of Subsection \ref{subsec2_4} is a positive answer in the particular case of preGarside trickle monoids to three questions asked in \cite{GodPar2} for parabolic submonoids of preGarside monoids. \begin{thm}\label{thm2_18} Let $M$ be a preGarside trickle monoid. \begin{itemize} \item[(1)] Let $N$ be a parabolic submonoid of $M$. Then the embedding $N \hookrightarrow M$ induces an embedding $G (N) \hookrightarrow G (M)$. So, we can consider $G(N)$ as a subgroup of $G(M)$. \item[(2)] Let $N$ be a parabolic submonoid of $M$. Then $M \cap G(N) = N$. \item[(3)] Let $N_1$ and $N_2$ be two parabolic submonoids of $M$. Then $N_1 \cap N_2$ is a parabolic submonoid of $M$ and $G(N_1) \cap G (N_2) = G (N_1 \cap N_2)$. \end{itemize} \end{thm} Again, a detailed proof of this theorem will be given in Section \ref{sec7}. \section{Examples}\label{sec3} \subsection{Generalized cactus groups}\label{subsec3_1} Let $G$ be a group. A \emph{cactus basis} based on $G$ is a non-empty family $\FF$ of pairs $X = (G_X, \Delta_X)$, where $G_X$ is a subgroup of $G$ and $\Delta_X$ is an element of $G_X$, such that, \begin{itemize} \item[(a)] $G_X \neq G_Y$ for $X,Y \in \FF$ distinct; \item[(b)] for $X, Y \in \FF$, if $G_Y \subset G_X$, then $(\Delta_X G_Y \Delta_X^{-1}, \Delta_X \Delta_Y \Delta_X^{-1}), (\Delta_X^{-1} G_Y \Delta_X, \Delta_X^{-1} \Delta_Y \Delta_X) \in \FF$. \end{itemize} Recall that, if $H_1, H_2$ are two subgroups of a group $G$, then $[H_1, H_2]$ denotes the subgroup of $G$ generated by $\{h_1 h_2 h_1^{-1} h_2^{-1} \mid h_1 \in H_1\,,\ h_2 \in H_2\}$. In particular, the equality $[H_1, H_2]=\{ 1 \}$ means that $h_1 h_2 = h_2 h_1$ for all $h_1 \in H_1$ and $h_2 \in H_2$. Let $\FF$ be a cactus basis based on $G$. Let $\mu : \FF \to \N \cup \{\infty\}$ be a map which satisfies the following conditions: \begin{itemize} \item for all $X \in \FF$, if $\mu (X)$ is finite, then the order of $\Delta_X$ is finite and divides $\mu (X)$; \item for all $X = (G_X, \Delta_X)$ and $Y = (G_Y, \Delta_Y)$ in $\FF$, if $G_Y \subset G_X$, then $\mu (Y) = \mu (Y')$, where $Y' = (\Delta_X G_Y \Delta_X^{-1}, \Delta_X \Delta_Y \Delta_X^{-1})$. \end{itemize} Then we define a quadruple $\Gamma_{\FF,\mu} = \Gamma = (\Gamma, \le, \mu, (\varphi_x)_{x \in V (\Gamma)})$ as follows. The set of vertices of $\Gamma$ is a set $V(\Gamma) = \{ x_X \mid X \in \FF\}$ in one-to-one correspondence with $\FF$. Two vertices $x_X$ and $x_Y$ are connected by an edge if either $G_X \subset G_Y$, or $G_Y \subset G_X$, or $G_X \cap G_Y = \{1\}$ and $[G_X, G_Y]= \{1\}$. We set $x_X \le x_Y$ if $G_X \subseteq G_Y$. The map $\mu : V (\Gamma) \to \N_{\ge 2} \cup \{ \infty\}$ is defined by $\mu (x_X) = \mu (X)$ for all $x_X \in V (\Gamma)$. Let $x_X \in V (\Gamma)$ and $x_Y \in V (\starE_{x_X} (\Gamma))$. If $x_Y \le x_X$, then we set $\varphi_{x_X} (x_Y) = x_{Y'}$, where $X=(G_X,\Delta_X)$, $Y=(G_Y, \Delta_Y)$ and $Y'= (\Delta_X G_Y \Delta_X^{-1}, \Delta_X \Delta_Y \Delta_X^{-1})$. If $x_Y \not \le x_X$, then we set $\varphi_{x_X} (x_Y) = x_Y$. The proof of the following is identical to that of Lemma \ref{lem2_1}, hence it is left to the reader. \begin{lem}\label{lem3_1} Let $\FF$ be a cactus basis based on a group $G$ and let $\mu : \FF \to \N_{\ge 2} \cup \{ \infty \}$ be a map as above. Then $\Gamma_{\FF,\mu} = (\Gamma, \le, \mu, (\varphi_x)_{x \in V(\Gamma)})$ is a trickle graph. \end{lem} \begin{rem} Let $\FF$ be a cactus basis based on a group $G$ and let $\mu : \FF \to \N_{\ge 2} \cup \{ \infty \}$ be a map as above. Then there is a homomorphism $\pi: \Tr (\Gamma_{\FF,\mu}) \to G$ which sends $x_X$ to $\Delta_X$ for all $X \in \FF$. This homomorphism plays an important role in the study of cactus groups associated to Coxeter systems. In that case it is known to be surjective and its kernel is known to embed into a right-angled Coxeter group (see \cite{Mosto1, Yu1}). \end{rem} It is not difficult to produce examples of cactus basis. Below we give three examples which seem particularly interesting to us because they highlight the link between trickle groups and the theory of Coxeter, Artin and Garside groups. \begin{expl1} Let $(W,S)$ be a Coxeter system. n}$, $w_X$ denotes the longest element in $W_X$. n} \}$ is a cactus basis based on $W$. n}$, then $\Tr(\Gamma_{\FF,\mu})$ is the cactus group $C(W, S)$. n}$, then $\Tr (\Gamma_{\FF,\mu})$ is the ``Artin'' version of $C(W,S)$. We call this group the \emph{Artin-cactus group} associated with $(W,S)$ and we denote it by $\AC (W,S)$. Like Artin groups, these groups are preGarside groups (see Theorem \ref{thm2_14}). We also know that, when $S$ is finite, they are torsion-free and they have a solution to the word problem (see Theorem \ref{thm2_16} and Corollary \ref{corl2_5}). \end{expl1} \begin{expl2} Dual structures for Artin groups were introduced by Birman--Ko--Lee \cite{BiKoLe1} for the braid groups and by Bessis \cite{Bessi1} for all Artin groups. They are quite mysterious but they can be extremely useful for understanding some Artin groups. For example, they are the main tool in the solutions to the word problem and to the $K (\pi, 1)$ conjecture for Artin groups of affine type \cite{McCSul1,PaoSal1}. Let $(W,S)$ be a Coxeter system of spherical type (i.e. such that $W$ is finite). Then another cactus basis based on $W$ can be defined as follows. We denote by $T = \{ w s w^{-1} \mid w \in W\,,\ s \in S\}$ the set of reflections of $W$ and by $\lg_T$ the word length on $W$ with respect to $T$. Fix a Coxeter element $c$. Then the triple $(W,T,c)$ is called a \emph{Coxeter dual system} (see \cite[Definition 1.3.2]{Bessi1}). Let $\delta$ be a \emph{standard parabolic element} (relative to $c$), that is, an element of $W$ such that $\lg_T (\delta) + \lg_T (\delta^{-1} c) = \lg_T (c)$. Let $t_1 t_2\dots t_k = \delta$ be a decomposition of $\delta$ on $T$ with $k = \lg_T (\delta)$. Then the \emph{standard (dual) parabolic subgroup} $W_\delta$ associated with $\delta$ is the subgroup generated by $\{t_1, t_2, \dots, t_k\}$. We know from \cite[Corollary 1.6.2, Definition 1.6.3]{Bessi1} that this subgroup depends only on $\delta$ and not on the chosen decomposition and that $(W_\delta, T \cap W_\delta, \delta)$ is a dual Coxeter system. Moreover, by \cite[Lemma 2.5]{BraWat1}, if $\delta, \delta'$ are two standard parabolic elements, then $\delta = \delta'$ if and only if $W_\delta = W_{\delta'}$. We say that a standard parabolic subgroup $W_\delta$ (or $\delta$) is \emph{irreducible} if we cannot decompose $T \cap W_{\delta}$ into a non-trivial partition $ T_1\sqcup T_2$ such that $xy = yx$ for all $x \in T_1$ and $y \in T_2$. Let $(W,T,c)$ be a dual Coxeter system with $W$ finite. Then the family $\FF$ consisting of the pairs $X_\delta = (W_\delta, \delta)$, where $\delta$ is a standard (non-trivial) irreducible parabolic element, is a cactus basis based on $W$. If we denote by $\mu (X_\delta)$ the order of $\delta$ for all $X_\delta \in \FF$, then we call $\Tr(\Gamma_{\FF, \mu})$ the \emph{dual cactus group} associated with $(W,T,c)$ and we denote it by $C^* (W,T,c)$. If we set $\mu (X_\delta) = \infty$ for all $X_\delta \in \FF$, then we call $\Tr (\Gamma_{\FF,\mu})$ the \emph{dual Artin-cactus group} associated with $(W,T,c)$ and we denote it by $\AC^*(W,T,c)$. We do not know when $C^* (W, T, \delta)$ is isomorphic to $C (W, S)$ and when $\AC^* (W, T, \delta)$ is isomorphic to $\AC (W, S)$. \begin{expl} Let $W$ be the symmetric group on three letters endowed with its dual presentation \[ W = \langle x ,y ,z \mid x^2 = y^2 = z^2 = 1\,,\ xy = yz = zx \rangle\,. \] For the Coxeter element $c = xy$, there are four standard non-trivial parabolic elements, $x, y, z, u=xy$, and the dual cactus group and dual Artin-cactus group have the following presentations: \begin{gather*} C^* = \langle x, y, z, u \mid x^2 = y^2 = z^2 = u^3 = 1\,,\ x u = u z\,,\ y u = u x\,, z u = u y \rangle\,,\\ \AC^* = \langle x, y, z, u \mid x u = u z\,,\ y u = u x\,, z u = u y \rangle\,. \end{gather*} \end{expl} \end{expl2} \begin{expl3} Examples 1 and 2 extend naturally to preGarside monoids (or groups) as follows. Let $M$ be a preGarside monoid. An \emph{atom} of $M$ is an element $a$ in $M \setminus \{1\}$ such that, for all $a_1, a_2 \in M$, if $a = a_1 a_2$, then $a_1 = 1$ or $a_2 = 1$. We denote by $\AA$ the set of atoms of $M$. It is easily shown that $\AA$ generates $M$ and, if $N$ is a parabolic submonoid, then $\AA \cap N$ is the set of atoms of $N$ and it generates $N$. Let $N$ be a Garside parabolic submonoid of $M$. By \cite{Dehor1} the submonoid $N$ admits a least Garside element, which we denote by $\Delta_N$, in the sense that, if $\Delta'_N$ is another Garside element of $N$, then $\Delta_N \in \Div (\Delta'_N)$. We know that $N$ embeds into $G(N)$ and that the conjugation by $\Delta_N$ leaves $N$ invariant, that is, the map $N \to N$, $ g \mapsto \Delta_N g \Delta_N^{-1}$, is well-defined and is an automorphism. Moreover, if $N_1$ is a parabolic submonoid of $N$, then $N_1$ and $\Delta_N N_1 \Delta_N^{-1}$ are both Garside parabolic submonoids of $N$ and $\Delta_N \Delta_{N_1} \Delta_N = \Delta_{\Delta_N N_1 \Delta_N^{-1}}$. Let $M$ be a preGarside monoid and let $\AA$ be the set of its atoms. We say that a parabolic submonoid $N$ of $M$ is \emph{irreducible} if we cannot decompose $\AA \cap N$ into a non-trivial partition $\AA_1 \sqcup \AA_2$ such that $ab = ba$ for all $a \in \AA_1$ and $b \in \AA_2$. Then the family $\FF$ consisting of the pairs $X_N = (G(N), \Delta_N)$, with $N$ an irreducible Garside parabolic submonoid of $M$, is a cactus basis. Note that we do not specify that $\FF$ is a cactus basis based on a given group, and actually we do not know if $\FF$ is a cactus basis based on a group ($G(M)$, for example), but the reader will easily extend the definition in this context. To complete the data we also need to define the labeling $\mu: \FF \to \N_{\ge 2} \cup \{\infty\}$. There are two natural choices. The first consists in taking $\mu(X_N)$ the order of the conjugation by $\Delta_N$ in $\Aut (G(N))$ if different from $1$ and $2$ if the conjugation by $\Delta_N$ is the identity. Note that $\Delta_N$ has always infinite order. The second consists in setting $\mu (X_N) = \infty$ for all $X_N \in \FF$. \end{expl3} \subsection{Virtual cactus groups}\label{subsec3_2} \begin{defin} Let $n \ge 2$. The \emph{virtual cactus group} on $n$ strands, denoted by $\VJ_n$, is the group defined by the presentation with generators \[ x_{p,q}\,,\ 1\le p < q \le n \quad \text{and} \quad \rho_i\,,\ 1 \le i \le n-1\,, \] and relations: \begin{itemize} \item[(j1)] $x_{p,q}^2 = 1$ for $1 \le p < q \le n$, \item[(j2)] $x_{p,q} x_{m,r} = x_{m,r} x_{p,q}$ for $[p,q] \cap [m,r] = \emptyset$, \item[(j3)] $x_{p,q} x_{m,r} = x_{p+q-r,p+q-m} x_{p,q}$ for $[m,r] \subset [p,q]$, \item[(s1)] $\rho_i^2 = 1$ for $1 \le i \le n-1$, \item[(s2)] $\rho_i \rho_j \rho_i = \rho_j \rho_i \rho_j$ for $ |i-j| = 1$, \item[(s3)] $\rho_i \rho_j = \rho_j \rho_i$ for $|i-j| > 1$, \item[(m1)] $\rho_i x_{p,q} = x_{p,q} \rho_i$ for $i < p-1$ and for $i \ge q+1$, \item[(m2)] $x_{p,q} \rho_{q} \rho_{q-1} \dots \rho_p = \rho_{q} \rho_{q-1} \dots \rho_p x_{p+1,q+1} $ for $1 \le p < q <n$. \end{itemize} \end{defin} As pointed out in the introduction, the elements of the virtual cactus group are represented by planar diagrams. To the usual diagrams which represent the elements of $J_n$ we add virtual crossings keeping the principle that two arcs connecting the same points and passing only through virtual crossings are equivalent. For instance, the generators $x_{p,q}$ and $\rho_i$ are depicted in Figure \ref{fig3_1} and the relation (m2) is illustrated in Figure \ref{fig3_2}. \begin{figure}[ht!] \begin{center} \includegraphics[width=6cm]{BeGoPaFig3_1.pdf} \caption{Generators of $\VJ_n$}\label{fig3_1} \end{center} \end{figure} \begin{figure}[ht!] \begin{center} \includegraphics[width=10.2cm]{BeGoPaFig3_2.pdf} \caption{Relation (m2) in the presentation of $\VJ_n$}\label{fig3_2} \end{center} \end{figure} Relations (j1--j3) are the relations which define the cactus group $J_n$, and they induce a homomorphism $\iota_J : J_n \to \VJ_n$ which sends $x_{p,q}$ to $x_{p,q}$ for all $1 \le p < q \le n$. Relations (s1--s3) are the relations in the standard presentation of the symmetric group $\SSS_n$, and they induce a homomorphism $\iota_S: \SSS_n \to \VJ_n$ which sends $s_i=(i,i+1)$ to $\rho_i$ for all $1 \le i \le n-1$. Relations (m1--m2), called the \emph{mixed relations}, make that two arcs connecting the same points and passing through only virtual crossings are equivalent. There is a homomorphism $\pi_K: \VJ_n \to \SSS_n$ defined by \[ \pi_K (x_{p,q}) = 1 \text{ for } 1 \le p < q \le n \text{ and } \pi_K (\rho_i) = s_i \text{ for } 1 \le i \le n-1\,. \] Note that $\pi_K \circ \iota_S = \id_{\SSS_n}$, hence $\pi_K$ is surjective, $\iota_S$ is injective, and there is a semi-direct product decomposition $\VJ_n = \KVJ_n \rtimes \SSS_n$, where $\KVJ_n = \Ker (\pi_K)$. The group $\VJ_n$ is not a trickle group, but $\KVJ_n$ is (see Proposition \ref{prop3_7}). This enables to solve the word problem in $\VJ_n$ but also to prove other results on $\VJ_n$ such as the fact that $\iota_J : J_n \to \VJ_n$ is injective (see Corollary \ref{corl3_9}). A different proof of this fact can be found in \cite[Corollary 11.4]{IKLPR1}. \begin{rem} For $1 \le p < q \le n$ we denote by $t(p,q)$ the permutation which sends $p$ to $q$ and $q$ to $p$, $p+1$ to $q-1$ and $q-1$ to $p+1$, and so on. More precisely, for all $0 \le i \le q-p$, $t(p,q)$ sends $p+i$ to $q-i$, and $t(p,q)(k) =k$ if $k \not \in [p,q]$. There exists another natural epimorphism from $\VJ_n$ to $\SSS_n$, denoted by $\pi_P: \VJ_n \to \SSS_n$, defined by \[ \pi_P (x_{p,q}) = t(p,q) \text{ for } 1 \le p < q \le n \text{ and } \pi_P (\rho_i) = s_i \text{ for } 1 \le i \le n-1\,. \] The kernel of $\pi_P$ is the \emph{pure virtual cactus group} $\PVJ_n$ studied in \cite{IKLPR1}, and it is different from the group $\KVJ_n$ that we study. A presentation for $\PVJ_n$ is given in \cite[Lemma 10.12]{IKLPR1}. \end{rem} For $2 \le \ell \le n$ we set $V_\ell = \{ (t_1, \dots, t_\ell) \in \{1, \dots, n\}^\ell \mid t_i \neq t_j \text{ for } i \neq j\}$. Let $y = (t_1, \dots, t_\ell) \in V_\ell$. We choose $w \in \SSS_n$ such that $w(i) = t_i$ for all $1 \le i \le \ell$ and we set $\delta_y = \iota_S(w) \,x_{1,\ell}\, \iota_S(w)^{-1}$. Note that $\delta_y \in \KVJ_n$. \begin{lem}\label{lem3_2} Let $2 \le \ell \le n$ and let $y \in V_\ell$. Then the above definition of $\delta_y$ does not depend on the choice of $w \in \SSS_n$. \end{lem} \begin{proof} We set $y = (t_1, \dots, t_\ell)$ and we take $w, w' \in \SSS_n$ such that $w(i) = w'(i) = t_i$ for all $1 \le i \le \ell$. We have $w^{-1} w'(i) = i$ for all $i \in \{1, \dots, \ell\}$, hence $w^{-1} w'$ lies in the subgroup of $\SSS_n$ generated by $s_{\ell+1}, \dots, s_{n-1}$, and therefore $\iota_S (w^{-1} w')$ lies in the subgroup of $\VJ_n$ generated by $\rho_{\ell+1}, \dots, \rho_{n-1}$. Relations (m1) imply that $\rho_i x_{1,\ell} = x_{1,\ell} \rho_i$ for all $i \in \{\ell+1, \dots, n-1\}$, hence $\iota_S (w^{-1} w') \, x_{1,\ell}\, \iota_S (w'^{-1} w) = x_{1,\ell}$, and therefore $\iota_S(w) \,x_{1,\ell}\, \iota_S (w)^{-1} = \iota_S (w') \,x_{1,\ell} \, \iota_S (w')^{-1}$. \end{proof} We set \[ V = \bigcup_{\ell=2}^n V_\ell\,. \] We consider the action of $\SSS_n$ on $V$ defined by, for $w \in \SSS_n$ and $y = (t_1, \dots, t_\ell) \in V$, \[ w \cdot y = (w (t_1) , \dots, w (t_\ell))\,. \] \begin{lem}\label{lem3_3} \begin{itemize} \item[(1)] Let $w \in \SSS_n$ and $y \in V$. Then $\iota_S (w) \, \delta_y \, \iota_S (w)^{-1} = \delta_{w \cdot y}$. \item[(2)] Let $1 \le p < q \le n$. Then $\delta_{(p,p+1,\dots,q)} = x_{p,q}$. \end{itemize} \end{lem} \begin{proof} We set $y = (t_1, \dots, t_\ell)$ and we choose $w' \in \SSS_n$ such that $w' \cdot (1, \dots, \ell) = (t_1, \dots, t_\ell)$. Notice that $w \cdot y = (ww'(1), \dots, ww'(\ell))= ww' \cdot (1, \dots, \ell)$. Then \[ \iota_S (w) \, \delta_y \, \iota_S (w)^{-1} = \iota_S (w) \, \iota_S (w') \, x_{1,\ell} \, \iota_S (w')^{-1} \, \iota_S (w)^{-1} = \iota_S (w w') \, x_{1,\ell} \, \iota_S (w w')^{-1} = \delta_{w \cdot y}\,. \] Let $u = s_1 s_2 \dots s_{n-1} = (1, 2, \dots, n)$. We easily see using Relations (m1) and (m2) that, for all $1 \le m < r < n$, \[ \iota_S (u) \, x_{m,r}\, \iota_S (u)^{-1} = x_{m+1,r+1}\,. \] On the other hand, $u \cdot (m, m+1, \dots, r) = (u(m), u(m+1), \dots, u(r)) = (m+1, m+2, \dots, r+1)$. So, \[ \delta_{(p, p+1, \dots, q)} = \iota_S (u)^{p-1} \, x_{1,q-p+1} \, \iota_S(u)^{-p+1} = x_{p,q}\,. \proved \] \end{proof} \begin{lem}\label{lem3_4} The set $\{ \delta_y \mid y \in V\}$ generates $\KVJ_n$. \end{lem} \begin{proof} Let $g \in \KVJ_n$. Since $g$ belongs to $\VJ_n$, there exist $ k \ge 0$, $p_1, \dots, p_k, q_1, \dots, q_k \in \N$, $w_0, w_1, \dots, w_k \in \SSS_n$ such that $1 \le p_i <q_i \le n$ for all $1 \le i \le k$, and \[ g = \iota_S (w_0) \, x_{p_1,q_1} \, \iota_S (w_1) \dots x_{p_k,q_k} \, \iota_S(w_k)\,. \] For $1 \le i \le k+1$ we set $v_i = w_0 w_1 \dots w_{i-1}$. Then \[ g = \iota_S (v_1) \, x_{p_1,q_1} \, \iota_S (v_1)^{-1} \, \iota_S (v_2) \, x_{p_2,q_2} \, \iota_S (v_2)^{-1} \dots \iota_S(v_k) \, x_{p_k,q_k} \, \iota_S (v_k)^{-1} \, \iota_S (v_{k+1})\,. \] Since $g \in \KVJ_n$, we have $1 = \pi_K (g)= v_{k+1}$, hence \[ g = \iota_S (v_1) \, x_{p_1,q_1} \, \iota_S (v_1)^{-1} \, \iota_S (v_2) \, x_{p_2,q_2} \, \iota_S (v_2)^{-1} \dots \iota_S(v_k) \, x_{p_k,q_k} \, \iota_S (v_k)^{-1} = \delta_{y_1} \delta_{y_2} \dots \delta_{y_k}\,, \] where $y_i = v_i \cdot (p_i, p_i + 1, \dots, q_i)$ for all $1 \le i \le k$. \end{proof} Now we define a quadruple $\Gamma = (\Gamma, \le, \mu, (\varphi_x)_{x \in V(\Gamma)})$ as follows. The set of vertices of $\Gamma$ is $V = \bigcup_{\ell=2}^n V_\ell$. We set $\mu (y) = 2$ for all $y \in V$. For $y = (t_1, \dots, t_\ell)$ and $y' = (t_1', \dots, t_k')$ in $V$, we set $y' \le y$ if there exist $1 \le p < q \le \ell$ such that $y'= (t_p, t_{p+1}, \dots, t_q)$. Let $y = (t_1, \dots, t_\ell)$ and $y' = (t_1', \dots, t_k')$ in $V$ such that $y \neq y'$. Then $\{y, y'\} \in E (\Gamma)$ if and only if either $y \le y'$ or $y' \le y$ or $\{t_1, \dots, t_\ell\} \cap \{t_1', \dots, t_k' \} = \emptyset$. Let $y = (t_1, \dots, t_\ell) \in V$ and $y' \in \starE_y (\Gamma)$. If $y' \not \le y$, then we set $\varphi_y (y') = y'$. Suppose $y' \le y$. We know that there exist $1 \le p < q \le \ell$ such that $y' = (t_p, \dots, t_q)$. Then we set $\varphi_y (y') = (t_{1 + \ell - q}, \dots, t_{1 + \ell - p})$. \begin{lem}\label{lem3_5} The above defined quadruple $\Gamma = (\Gamma, \le, \mu, (\varphi_x)_{x\in V(\Gamma)})$ is a trickle graph. \end{lem} \begin{proof} Conditions (a) and (d) are satisfied by definition, and Conditions (b) and (f) are obviously satisfied. It is easily seen that, for all $x \in V$, $\varphi_x^2=1$, hence Condition (e) is also satisfied. We show that $\Gamma$ satisfies Condition (c). Let $x \in V$ and $y, z \in \starE_x (\Gamma)$ be such that $z \le y$. If $y || x$ then, by Condition (b), $z || x$, hence $\varphi_x (z) = z \le y = \varphi_x (y)$. Suppose $z \le y < x$. Set $x = (t_1, \dots, t_\ell)$. There exist $1 \le p \le k < m \le q \le \ell$ such that $y = (t_p, \dots, t_q)$ and $z = (t_k, \dots, t_m)$. We have $\varphi_x (y) = (t_{1+\ell-q}, \dots, t_{1+\ell-p})$, $\varphi_x (z) = (t_{1+\ell-m}, \dots, t_{1+\ell -k})$ and $1 \le 1+\ell-p \le 1+\ell-m < 1+\ell-k \le 1 + \ell -p \le \ell$, hence $\varphi_x (z) \le \varphi_x (y)$. This shows that, if $z \le y$, then $\varphi_x (z) \le \varphi_x (y)$. But $\varphi_x^2 = 1$, hence we actually have $z \le y$ if and only if $\varphi_x (z) \le \varphi_x (y)$. Now, we show that $\Gamma$ satisfies Condition (g). Let $x, y, z \in V$ be such that $z \le y \le x$. Set $x = (t_1, \dots, t_\ell)$. There exist $1 \le p \le k < m \le q \le \ell$ such that $y = (t_p, \dots, t_q)$ and $z = (t_k, \dots, t_m)$. Let $y' = \varphi_x (y) = (t_{1+\ell-q}, \dots, t_{1+\ell-p})$. Then a direct calculation shows that \[ (\varphi_x \circ \varphi_y) (z) = (t_{1 + \ell - p - q + k}, \dots, t_{1 + \ell - p - q + m}) = (\varphi_{y'} \circ \varphi_x) (z)\,. \proved \] \end{proof} \begin{lem}\label{lem3_6} There is a surjective homomorphism $\Phi: \Tr (\Gamma) \to \KVJ_n$ which sends $x$ to $\delta_x$ for all $x \in V$. \end{lem} \begin{proof} Let $x = (t_1, \dots, t_\ell) \in V$. Let $w \in \SSS_n$ be such that $w(i) = t_i$ for all $1 \le i \le n$. Then \[ \delta_x^2 = \iota_S (w)\, x_{1,\ell}^2\, \iota_S (w)^{-1} = \iota_S (w) \, \iota_S (w)^{-1} = 1\,. \] Let $x, y \in V$ be such that $\{x, y\} \in E_{||} (\Gamma)$. We set $x=(t_1, \dots, t_\ell)$ and $y = (t_1', \dots, t_k')$. By definition we have $\{t_1, \dots, t_\ell\} \cap \{t_1', \dots, t_k'\} = \emptyset$, hence we can choose $w \in \SSS_n$ such that $ w(i)=t_i$ for all $1 \le i \le \ell$ and $w (\ell + j) = t_j'$ for all $1 \le j\le k$. Then, by Lemma \ref{lem3_3}, \[ \delta_x \delta_y = \iota_S(w) \, x_{1,\ell} x_{\ell+1,\ell+k}\, \iota_S(w)^{-1} = \iota_S(w) \, x_{\ell+1,\ell+k} x_{1,\ell} \, \iota_S(w)^{-1} = \delta_y \delta_x\,. \] Let $x, y \in V$ be such that $y \le x$. We set $x = (t_1, \dots, t_\ell)$ and $y = (t_p, \dots, t_q)$, where $1 \le p < q \le \ell$. We choose $w \in \SSS_n$ such that $w (i) = t_i$ for all $1 \le i \le \ell$. Then, by Lemma \ref{lem3_3}, \[ \delta_x \delta_y = \iota_S(w) \, x_{1,\ell} x_{p,q} \, \iota_S(w)^{-1} = \iota_S(w) \, x_{1+\ell-q,1+\ell-p} x_{1,\ell} \, \iota_S(w)^{-1} = \delta_{\varphi_x(y)} \delta_x\,. \] This shows that there is a homomorphism $\Phi: \Tr (\Gamma) \to \KVJ_n$ which sends $x$ to $\delta_x$ for all $x \in V$. This homomorphism is surjective because, by Lemma \ref{lem3_4}, $\{\delta_x \mid x \in V\}$ generates $\KVJ_n$. \end{proof} \begin{prop}\label{prop3_7} The homomorphism $\Phi: \Tr (\Gamma) \to \KVJ_n$ of Lemma \ref{lem3_6} is an isomorphism. \end{prop} \begin{proof} The action of $\SSS_n$ on $V$ extends to an action of $\SSS_n$ on $\Tr (\Gamma)$, hence we can consider the semi-direct product $G = \Tr (\Gamma ) \rtimes \SSS_n$. Furthermore, by Lemma \ref{lem3_3}, the map $\hat \Phi: G \to \VJ_n$ defined by $\hat \Phi (g,w) = \Phi(g) \, \iota_S (w)$ is a group homomorphism. We show that there is a homomorphism $\Psi: \VJ_n \to G$ which sends $x_{p,q}$ to $x_{p,q}' = (p, p+1, \dots, q ) \in V \subseteq \Tr (\Gamma)$ for $1 \le p < q \le n$, and which sends $\rho_i$ to $s_i = (i, i + 1) \in \SSS_n$ for $1 \le i \le n-1$. The relations (j1) $x_{p,q}'^2 = 1$ for $1 \le p < q \le n$, (j2) $x_{p,q}' x_{m,r}' = x_ {m,r}' x_{p,q}'$ for $[p,q] \cap [m,r] = \emptyset$, and (j3) $x_{p,q}' x_{m,r}' = x_{p+q-r,p+q-m}' x_{p,q}'$ for $[m,r] \subset [p,q]$, follow from the definitions of the trickle graph $\Gamma$ and of the associated presentation of $\Tr (\Gamma)$. The relations (s1) $s_i^2 = 1$ for $1 \le i \le n-1$, (s2) $s_i s_j s_i = s_j s_i s_j$ for $|i - j| = 1$, and (s3) $s_i s_j = s_j s_i$ for $|i - j| > 1$, are the relations of the standard presentation of the symmetric group $\SSS_n$. Finally, the relations (m1) $s_i x_{p,q}' = x_{p,q}' s_i$ for $i < p-1$ and for $i \ge q+1$, and (m2) $ x_{p,q}'s_{q} s_{q-1} \dots s_p=s_{q} s_{q-1} \dots s_p x_{p+1,q+1}'$ for $1 \le p < q <n$, follow from the action of $\SSS_n$ on $V$. For every $w \in \SSS_n$ we have $(\Psi \circ \hat \Phi) (w) = \Psi (\iota_S (w)) = w$. On the other hand, let $x = (t_1, \dots, t_\ell) \in V$. We choose $w \in \SSS_n$ such that $w \cdot (1, \dots, \ell) = x = (t_1, \dots, t_\ell)$. Then \[ (\Psi \circ \hat \Phi) (x) = \Psi (\delta_x) = \Psi (\iota_S (w)\, x_{1,\ell}\, \iota_S (w)^{-1}) = w \, x_{1,\ell}'\, w^{-1} = w \cdot x_{1,\ell}' = x\,. \] This shows that $\Psi \circ \hat \Phi = \id_G$, hence $\hat \Phi$ is injective, and therefore $\Phi$ is injective. Since we already know that $\Phi$ is surjective, this completes the proof of the proposition. \end{proof} Since $\KVJ_n$ is a finite index subgroup of $\VJ_n$, the following follows from Proposition \ref{prop3_7} and Corollary \ref{corl2_5}. \begin{corl}\label{corl3_8} Let $n \ge 2$. Then $\VJ_n$ has a solution to the word problem. \end{corl} Another consequence of Proposition \ref{prop3_7} is the following. \begin{corl}[Ilin--Kamnitzer--Li--Przytycki--Rybnikov \cite{IKLPR1}]\label{corl3_9} Let $n \ge 2$. Then the natural homomorphism $\iota_J: J_n \to \VJ_n$ is injective. \end{corl} \begin{proof} Recall that, for $1 \le p < q \le n$, we set $x_{p,q}' = (p,p+1,\dots,q) \in V$. Set $U = \{x_{p,q}' \mid 1 \le p < q \le n\}$ and denote by $\Gamma_1$ the full subgraph of $\Gamma$ spanned by $U$. It is easily seen that $\Gamma_1$ is a parabolic subgraph of $\Gamma$. We denote by $\le_1$ and $\mu_1$ the restrictions of $\le$ and $\mu$ to $V(\Gamma_1)$, respectively, and, for every $x \in V (\Gamma_1)$, we denote by $\varphi_{1,x}$ the restriction of $\varphi_x$ to $\starE_x (\Gamma_1)$. Then $\Gamma_1 = (\Gamma_1, \le_1 ,\mu_1, (\varphi_{1,x})_{x\in U})$ is a trickle graph and, by Theorem \ref{thm2_10}, the inclusion $U \hookrightarrow V$ induces an injective homomorphism $\iota: \Tr (\Gamma_1) \to \Tr (\Gamma)$. We easily see that the map $\{x_{p,q} \mid 1 \le p < q \le n\} \to U$, $x_{p,q} \mapsto x_{p,q}'$, induces an isomorphism $\phi: J_n \to \Tr (\Gamma_1)$ and that the composition $\iota \circ \phi: J_n \to \Tr (\Gamma) = \KVJ_n \subseteq \VJ_n$ coincides with $\iota_J: J_n \to \VJ_n$. So, $\iota_J: J_n \to \VJ_n$ is injective. \end{proof} \subsection{Thompson group F}\label{subsec3_3} There are several equivalent definitions of Thompson group $F$. In this paper we consider the one as a subgroup of the group of orientation preserving piecewise linear homeomorphisms of the real line (see \cite[Theorem 1.4.1]{Buril1}). \begin{defin} Let $f: \R \to \R$ be an orientation preserving piecewise linear homeomorphism of the real line. We say that $f$ belongs to \emph{Thompson group $F$} if there exist two finite sequences with the same cardinality $u_0, u_1, \dots, u_n$ and $v_0, v_1, \dots, v_n$ in $\Z [\frac{1}{2}]$ such that: \begin{itemize} \item $u_0 < u_1 < \cdots < u_n$ and $v_0 <v_1 < \cdots <v_n$, \item $f$ sends linearly the interval $[u_{i-1}, u_i]$ onto $[v_{i-1}, v_i]$ for all $i \in \{1, \dots, n\}$, \item $f$ sends the interval $(-\infty, u_0]$ onto $(-\infty, v_0]$ via the translation $t \mapsto t + v_0 - u_0$, and $f$ sends the interval $[u_n, +\infty)$ onto $[v_n, +\infty)$ via the translation $t \mapsto t + v_n - u_n$. \end{itemize} \end{defin} Our goal in this subsection is to provide a trickle graph $\Gamma = (\Gamma, \le, \mu,(\varphi_x)_{x \in V(\Gamma)})$ such that $F \simeq \Tr (\Gamma)$. We start by defining the vertex set $V = V (\Gamma)$. For $p \in \N$ we define a subset $V_p \subseteq \Z [\frac{1}{2}]$ and a map $s_p: V_p \to V_p$ which satisfies $x <s_p (x)$ and $[x, s_p(x)] \cap V_p = \{x, s_p(x)\}$ for all $x \in V_p$ by induction on $p$. We set $V_0 = \Z$ and we set $s_0 (x) = x + 1$ for all $x \in V_0$. Suppose $V_p$ and $s_p: V_p \to V_p$ are defined. Let $x \in V_p$. Let $x' = s_p(x)$ and, for $k \in \N$, let $u_k = u_{p+1,k} (x) = x' - \frac{x'-x}{2^k}$ (see Figure \ref{fig3_3}). We set $V_{p+1} (x) = \{u_{p+1,k} (x) \mid k \in \N\}$. Then \[ V_{p+1} = \bigsqcup_{x \in V_p} V_{p+1} (x)\,. \] If $x \in V_p$ and $k \in \N$, then we set $s_{p+1} (u_{p+1, k} (x)) = u_{p+1, k+1} (x)$. So, in Figure \ref{fig3_3}, $s_{p+1}$ sends $u_k$ to $u_{k+1}$. \begin{figure}[ht!] \begin{center} \includegraphics[width=8cm]{BeGoPaFig3_3.pdf} \caption{Definition of $V_{p+1}$}\label{fig3_3} \end{center} \end{figure} Observe that $V_p\subset V_{p+1}$ for all $p \in \N$ and that $\bigcup_{p \in \N} V_p = \Z [\frac{1}{2}]$. So, $\{ V_p \mid p \in \N \}$ is a filtration of $\Z [\frac{1}{2}]$. Now, we set $V = \Z [\frac{1}{2}] \sqcup \{\infty\}$. We endow $\Z [\frac{1}{2}]$ with the order induced by the one of $\R$, and we set $x < \infty$ for all $x \in \Z [\frac{1}{2}]$. So, the order $\le$ on $V$ is a total order and $\Gamma$ is defined to be the complete graph on $V$. We set $\mu (x) = \infty$ for all $x \in V$. It remains to define $\varphi_x$ for all $x \in V$, but for this we first need to define the generating system $\{h_x \mid x\in V\}$ for $F$ corresponding to the trickle structure. In order to define the generating system $\{h_x \mid x\in V\}$, in addition to the sets $V_p$ and to the maps $s_p: V_p \to V_p$, we need other maps $t_p: V_p \to V_p$, $p \in \N$, defined by induction on $p$ as follows. We set $t_0 (x) = x - 1$ for all $x \in V_0 = \Z$. Let $p \ge 1$ and let $x \in V_p$. If $x \in V_{p-1}$, then we set $t_p (x) = t_{p-1} (x)$. If $x \not \in V_{p-1}$, then there exists $y \in V_{p-1}$ and $k \ge 1$ such that $x = u_{p, k} (y)$, and we set $t_p (x) = u_{p, k-1} (y)$. Note that, for all $x \in V_p$, there exists $N \ge 0$ such that $t_p^n (x) \in V_0 = \Z$ for all $n\ge N$. The map $h_\infty$ is defined to be the translation $h_\infty: \R \to \R$, $t \mapsto t-1$. We define $h_x$ for $x \in \Z [\frac{1}{2}]$ as follows. Let $p$ be the least integer $\ge 0$ such that $x \in V_p$. We set $x' = t_p (x)$ (see Figure \ref{fig3_4}). For all $k \ge 0$ we set $v_k = s_{p+1}^k (x')$ and $v_{-k} = t_p^k (x')$. We have \begin{gather*} \cdots < v_{-k} < \cdots < v_{-1} < v_0 = x' < v_1 < \cdots < v_k < \cdots\,,\\ \lim\limits_{k \to -\infty} v_k = -\infty \text{ and } \lim\limits_{k \to +\infty} v_k =x\,. \end{gather*} Then $h_x: \R \to \R$ sends linearly the interval $[v_k, v_{k+1}]$ onto $[v_{k-1}, v_k]$ (preserving the orientation) for all $k \in \Z$ and it is the identity on $[x, +\infty)$. \begin{figure}[ht!] \begin{center} \includegraphics[width=8.8cm]{BeGoPaFig3_4.pdf} \caption{Definition of $h_x$}\label{fig3_4} \end{center} \end{figure} The proof of the following is left to the reader. \begin{lem}\label{lem3_10} Let $x \in \Z [\frac{1}{2}]$. Let $(v_k)_{k\in \Z}$ be as defined above. \begin{itemize} \item[(1)] $h_x$ sends linearly the interval $[v_1,x]$ onto $[v_0,x]$. \item[(2)] $h_x$ sends the interval $[x,+\infty)$ onto $[x,+\infty)$ via the identity. \item[(3)] Let $N$ be the least integer $\ge 0$ such that $v_{-N} \in V_0 = \Z$. Then $h_x$ sends $(-\infty,v_{-N}]$ onto $(-\infty,v_{-N-1}]=(-\infty,v_{-N}-1]$ via the translation $t \mapsto t-1$. \end{itemize} In particular, $h_x\in F$. \end{lem} Note that we also have $h_\infty \in F$. Furthermore, we know that $F$ is generated by $\{h_0,h_\infty\}$ (see \cite[Corollary 2.6]{CaFlPa1}), hence: \begin{lem}\label{lem3_11} The set $\{h_x \mid x \in V\}$ generates $F$. \end{lem} The next result is an observation. \begin{lem}\label{lem3_12} Let $x \in V$. Then $h_x (\Z [\frac{1}{2}]) = \Z [\frac{1}{2}]$, $h_x$ preserves the order of $\Z [\frac{1}{2}]$, and $h_x(y)=y$ for all $y \in \Z [\frac{1}{2}]$ such that $y \ge x$. \end{lem} For $x \in V$, we set $\varphi_x (\infty) = \infty$ and $\varphi_x (y) = h_x(y)$ for all $y \in \Z [\frac{1}{2}]$. Now we need to show that $\Gamma = (\Gamma, \le, \mu, (\varphi_x)_{x \in V})$ is a trickle graph and that the map $V \to F$, $ x \mapsto h_x$, induces an isomorphism $\Tr (\Gamma) \to F$. We begin by proving the following lemma which is a key point in the proofs that will follow. \begin{lem}\label{lem3_13} Let $x \in V$ and let $y \in \Z [\frac{1}{2}]$ be such that $y < x$. Let $y' = h_x (y)$. Then $h_x \circ h_y \circ h_x^{-1} = h_{y'}$. \end{lem} \begin{proof} First assume $x=\infty$. Then $y'=y-1$ and by construction $h_{y'} = h_{y-1} = h_\infty \circ h_y \circ h_\infty^{-1}$. Now assume $x \in \Z [\frac{1}{2}]$. Let $p$ be the least integer $\ge 0$ such that $x \in V_p$. Set $x' = t_p (x)$. For all $k \ge 0$ we set $v_k = s_{p+1}^k (x')$ and $v_{-k} = t_p^k (x')$. Then $h_x: \R \to \R$ sends linearly the interval $[v_k, v_{k+1}]$ onto $[v_{k-1},v_k]$ for all $k \in \Z$ and it is the identity on $[x, +\infty)$. We denote by $\alpha: (-\infty, x) \to \R$ the orientation preserving piecewise linear homeomorphism which sends linearly $[v_k, v_{k+1}]$ onto $[k, k+1]$ for all $k \in \Z$ (see Figure \ref{fig3_5}). Since $h_x$, $h_y$ and $h_{y'}$ induce homeomorphisms of $(-\infty, x)$, we can consider the homeomorphisms of the real line $g_x = \alpha \circ h_x|_{ (-\infty, x)} \circ \alpha^{-1}$, $g_y = \alpha \circ h_y|_{(-\infty, x)} \circ \alpha^{-1}$ and $ g_{y'} = \alpha \circ h_{y'}|_{(-\infty, x)}\circ \alpha^{-1}$. The following properties are easily observed: \begin{itemize} \item $\alpha(\Z [\frac{1}{2}] \cap (-\infty, x)) = \Z[\frac{1}{2}]$, \item $g_x = h_\infty$, $g_y = h_{\alpha(y)}$ and $g_{y'} = h_{\alpha(y')}$. \end{itemize} We have $\alpha (y') = (\alpha \circ h_x) (y) = g_x (\alpha (y)) = h_\infty (\alpha (y))$, hence, by the case $x =\infty$ already treated, \[ g_{y'} = h_{\alpha(y')} = h_\infty \circ h_{\alpha(y)} \circ h_\infty^{-1} = g_x \circ g_y \circ g_x^{-1}\,. \] Since $h_x|_{[x, +\infty)} = h_y|_{[x, +\infty)} = h_{y'}|_{[x, +\infty)} = \id_{[x ,+\infty)}$, we conclude that $h_{y'} =h_x \circ h_y \circ h_x^{-1}$. \end{proof} \begin{figure}[ht!] \begin{center} \includegraphics[width=8.8cm]{BeGoPaFig3_5.pdf} \caption{Definition of $\alpha$}\label{fig3_5} \end{center} \end{figure} Now we prove that $\Gamma = (\Gamma, \le, \mu, (\varphi_x)_{x\in V(\Gamma)})$ is a trickle graph. \begin{lem}\label{lem3_14} The above defined quadruple $\Gamma = (\Gamma, \le, \mu, (\varphi_x)_{x\in V})$ is a trickle graph. \end{lem} \begin{proof} We should prove that $(\Gamma, \le, \mu, (\varphi_x)_{x\in V(\Gamma)})$ satisfies Conditions (a) to (g) of the definition of a trickle graph. Condition (a) is obvious because $\Gamma$ is a complete graph. Condition (b) is empty because $E_{||}(\Gamma)=\emptyset$. Condition (c) holds because $\varphi_x: V \to V$ preserves the order of $V$ for all $x \in V$. Condition (d) holds by Lemma \ref{lem3_12}. Conditions (e) and (f) are obvious because $\mu (x) = \infty$ for all $x \in V$. Condition (g) is a direct consequence of Lemma \ref{lem3_13}. \end{proof} Now, we prove that $\Tr (\Gamma)$ is isomorphic to $F$. \begin{thm}\label{thm3_15} Let $\Gamma = (\Gamma, \le, \mu, (\varphi_x)_{x\in V})$ be the trickle graph of Lemma \ref{lem3_14}. Then $\Tr (\Gamma)$ is isomorphic to $F$. \end{thm} \begin{proof} By Lemma \ref{lem3_13} there exists a homomorphism $\Phi: \Tr (\Gamma) \to F$ which sends $x$ to $h_x$ for all $x \in V$. This homomorphism is surjective because $\{h_x \mid x \in V\}$ generates $F$ (see Lemma \ref{lem3_11}). So, it remains to prove that $\Phi$ is injective. Let $g \in \Tr (\Gamma) \setminus \{1\}$. Consider the rewriting system $\RR = \RR (\Gamma)$ of Theorem \ref{thm2_4}. Since $\Gamma$ is a complete graph, the $\RR$-irreducible pilings are all of length $1$ or $0$. Moreover, by Theorems \ref{thm2_3} and \ref{thm2_4}, there exists a unique $\RR$-irreducible piling which represents $g$. This piling is different from the empty one, because $g \neq 1$, hence it is of the form $(U)$, where $U=\{x_1^{a_1}, \dots, x_p^{a_p}\}$ and $x_1 > x_2 > \cdots >x_p$, where $p \ge 1$. Then $g = \omega (U) = x_1^{a_1} x_2^{a_2} \dots x_p^{a_p}$, hence \[ \Phi(g) = h_{x_1}^{a_1} \circ h_{x_2}^{a_2} \circ \cdots \circ h_{x_p}^{a_p}\,. \] The homeomorphism $h_{x_1}$ is of infinite order, hence, if $p=1$, then $\Phi (g) = h_{x_1}^{a_1} \neq \id$. Suppose $p \ge 2$. Choose $t \in (x_2,x_1)$. Since $h_{x_i} (t) = t$ for all $2 \le i \le p$ and $h_{x_1}$ is strictly decreasing on $(-\infty, x_1)$, \[ \Phi(g)(t) = h_{x_1}^{a_1}(t) \neq t\,, \] hence $\Phi(g) \neq \id$. This shows that $\Ker (\Phi) = \{1\}$, hence $\Phi$ is injective. \end{proof} To finish this subsection we show that some natural parabolic subgroups of $\Tr (\Gamma)$ are isomorphic to $F$ itself. More precisely, let $x\in \Z [\frac{1}{2}]$, let $V_x = \{x\}_\downarrow = \{ y \in V \mid y \le x\}$, and let $\Gamma_x$ be the full subgraph of $\Gamma$ spanned by $V_x$. As observed in Subsection \ref{subsec2_4}, $\Gamma_x$ is a parabolic subgraph of $\Gamma$. \begin{prop}\label{prop3_16} Let $x \in \Z [\frac{1}{2}]$. There exists a bijection $\beta: V_x \to V$ which induces an isomorphism $\beta: \Tr(\Gamma_x) \to \Tr (\Gamma)$. In particular, $\Tr (\Gamma_x)$ is isomorphic to $F$. \end{prop} \begin{proof} We consider the map $\alpha$ defined in the proof of Lemma \ref{lem3_13}. Let $p$ be the least integer $\ge 0$ such that $x \in V_p$. Set $x' = t_p (x)$. For all $k \ge 0$ we set $v_k = s_{p+1}^k (x')$ and $v_{-k} = t_p^k (x')$. Then $\alpha: (-\infty, x) \to \R$ denotes the orientation preserving piecewise linear homeomorphism which sends linearly $[v_k, v_{k+1}]$ onto $[k, k+1]$ for all $k \in \Z$ (see Figure \ref{fig3_5}). We have $\alpha( (-\infty, x) \cap \Z [\frac{1}{2}]) = \alpha (V_x \setminus \{x\}) = \Z [\frac{1} {2}]$. So, there is a bijection $\beta: V_x \to V$ which sends $y$ to $\alpha (y)$ for all $y \in V_x \setminus \{x\}$ and sends $x$ to $\infty$. It is easily checked that, for all $y,y' \in V_x$, we have $y \le y'$ if and only if $\beta(y) \le \beta (y')$, and that, for all $y \in V_x$, $h_{\beta (y)} = \alpha \circ h_y|_{(-\infty,x)} \circ \alpha^{-1}$. Thus, $\beta$ induces an isomorphism between $\Gamma_x = (\Gamma_x, \le_x, \mu_x, (\varphi_{x,y})_{y \in V_x})$ and $\Gamma = (\Gamma, \le, \mu, (\varphi_y)_{y \in V})$, where $\le_x$ and $\mu_x$ are the restrictions of $\le$ and $\mu$ to $V_x$, respectively, and, for all $y \in V_x$, $\varphi_{x,y}$ is the restriction of $\varphi_y$ to $V_x = \starE_y (\Gamma_x)$. This $\beta$ obviously defines an isomorphism $\beta : \Tr (\Gamma_x) \to \Tr (\Gamma)$. \end{proof} \begin{rem} By identifying $\Tr (\Gamma)$ with $F$ it is easily seen that $\Tr (\Gamma_x)$ is the subgroup of $F$ consisting of the elements whose support is contained in $(-\infty, x)$. It is well-known and easy to prove that this subgroup is a copy $F$. \end{rem} \subsection{Ordered quandle groups}\label{subsec3_4} A \emph{quandle} is a non-empty set $Q$ endowed with an operation $*$ which satisfies the following properties. \begin{itemize} \item[(1)] For all $x\in Q$, $x * x = x$. \item[(2)] For all $x, y, z \in Q$, $(z * x) * (y * x) = (z * y) * x$. \item[(3)] For all $x \in Q$, the map $Q \to Q$, $y \mapsto y * x$, is a bijection. \end{itemize} A quandle $Q$ is said to be \emph{ordered} (on the right) if there exists a total order $\le$ on $Q$ such that, for all $x, y, z \in Q$, we have $y \le z$ if and only if $y * x \le z * x$. \begin{expl1} A group $G$ is called \emph{bi-orderable} if there exists a total order $\le$ on $G$ which is invariant under left and right multiplication. Examples of bi-orderable groups are free abelian groups, free groups and, more generally, right-angled Artin groups. Other important examples are the pure braid groups. Let $G$ be a bi-orderable group, let $\le$ be a total order on $G$ invariant under left and right multiplication, and let $*$ be the operation on $G$ defined by $y * x = xyx^{-1}$. Then $(G, *, \le)$ is an ordered quandle. \end{expl1} \begin{expl2} Consider the field $\R$ of real numbers endowed with its natural total order. Then, for a fixed $\alpha>0$, the binary operation $x*y = \alpha x + (1-\alpha) y$ endows $\R$ with an ordered quandle structure (see \cite{BaElh1}). \end{expl2} Let $(Q,*,\le)$ be an ordered quandle. Let $\Gamma$ be the full graph whose set of vertices is $Q$. For $x \in Q$ we define $\varphi_x: Q \to Q$ by $\varphi_x (y) = y$ if $x \le y$ and $\varphi_x (y) = y * x$ if $y \le x$. We set $\mu (x) = \infty$ for all $x \in Q$. \begin{prop}\label{prop3_17} The above defined quadruple $\Gamma = (\Gamma, \le, \mu, (\varphi_x)_{x\in Q})$ is a trickle graph. \end{prop} \begin{proof} Since $\Gamma$ is a full graph and $y\mapsto y * x$ is an order-preserving bijection, the map $\varphi_x$ is an order-preserving automorphism of $\Gamma$ for all $x \in Q$. Since $\Gamma$ is a full graph and $\le$ is a total order, Conditions (a) and (b) are trivially satisfied. Conditions (e) and (f) hold because $\mu (x) = \infty$ for all $x \in Q$. Condition (c) follows from the fact that $Q$ is ordered, and Condition (d) follows from the definition of $\varphi_x$. Finally, Condition (g) is satisfied because $Q$ is a quandle. \end{proof} \section{The trickle algorithm}\label{sec4} In this section we fix a trickle graph $\Gamma = (\Gamma, \le, \mu, (\varphi_x)_{x\in V(\Gamma)})$ and we keep the notations of Subsection \ref{subsec2_2}. Our aim is to prove Theorem \ref{thm2_4}. Let $A$ be an alphabet and let $R$ be a rewriting system on $A^*$. A \emph{critical pair} for $R$ is a quintuple $(u_1, u_2, u_3, v_1, v_2)$ of elements of $A^*$ satisfying one of the following two conditions. \begin{itemize} \item[(a)] $(u_1 \cdot u_2, v_1) \in R$, $(u_2 \cdot u_3, v_2) \in R$ and $u_2 \neq \epsilon$; \item[(b)] $(u_1 \cdot u_2 \cdot u_3, v_1) \in R$ and $(u_2, v_2)\in R$. \end{itemize} We say that the critical pair $(u_1, u_2, u_3, v_1, v_2)$ is \emph{resolved} if there exists $w \in A^*$ such that \begin{itemize} \item $v_1 \cdot u_3 \to^* w$ and $u_1 \cdot v_2 \to^* w$ in case (a), \item $v_1 \to^* w$ and $u_1 \cdot v_2 \cdot u_3 \to^* w$ in case (b). \end{itemize} We will say that the critical pair is of \emph{type (a)} or of \emph{type (b)} depending on which of the above conditions (a) or (b) it satisfies. The following will be used to show that our rewriting system is confluent. \begin{thm}[Newman \cite{Newma1}]\label{thm4_1} Let $A$ be an alphabet and let $R$ be a terminating rewriting system on $A^*$. If all critical pairs of $R$ are resolved, then $R$ is confluent. \end{thm} Now, we move on to the proof of Theorem \ref{thm2_4}. We start by showing that the extraction operation is well-defined. \begin{lem}\label{lem4_2} Let $U = \{x_1^{a_1}, x_2^{a_2}, \dots, x_p^{a_p}\}$ be a non-empty stratum and let $x_i^{a_i} \in U$. Set $U (x_i^{a_i}) = \{ x_j^{a_j} \mid x_j \ge x_i \}$. Then $\supp (U (x_i^{a_i}))$ is totally ordered and $\gamma (U,x_i^{a_i}) = \gamma (U (x_i^{a_i}), x_i^{a_i})$. In particular, the definition of $\gamma (U, x_i^{a_i})$ does not depend on the choice of the numbering of the elements of $U$. \end{lem} \begin{proof} As always we assume that, if $x_j > x_k$, then $j <k$. Recall that \[ \gamma (U, x_i^{a_i}) = \big( (\varphi_{x_1}^{a_1} \circ \varphi_{x_2}^{a_2} \circ \cdots \circ \varphi_{x_{i-1 }}^{a_{i-1}}) (x_i) \big)^{a_i}\,. \] Let $x_j^{a_j}, x_k^{a_k} \in U (x_i^{a_i})$. Since $x_i \le x_j$, $x_i \le x_k$, and $\{x_i, x_k\} \in E (\Gamma)$, Condition (b) in the definition of a trickle graph implies that either $x_j \le x_k$, or $x_k \le x_j$. This shows that $\supp (U (x_i^{a_i}))$ is totally ordered. Then, since $\supp (U (x_i^{a_i}))$ is totally ordered, the definition of the extraction operation in $U (x_i^{a_i})$ is unique, that is, $\gamma (U (x_i^{a_i}), x_i^{a_i})$ is well-defined. It remains to show that $\gamma (U, x_i^{a_i}) = \gamma (U (x_i^{a_i}), x_i^{a_i})$. Notice that this implies that the definition of $\gamma (U, x_i^{a_i})$ does not depend on the choice of the numbering of the elements of $U$. It suffices to show that, if $x_j^{a_j} \not \in U(x_i^{a_i})$, then $\gamma(U, x_i^{a_i}) =\gamma (L(U, x_j^{a_j}), x_i^{a_i})$. If $j > i$, then there is nothing to prove. Suppose $j < i$. Since $x_i \not \le x_j$, according to Condition (c) in the definition of a trickle graph, \begin{gather*} \varphi_{x_{j+1}}^{a_{j+1}} \circ \cdots \circ \varphi_{x_{i-1}}^{a_{i-1}} (x_i) \not \le \varphi_{x_{j+1}}^{a_{j+1}} \circ \cdots \circ \varphi_{x_{i-1}}^{a_{i-1}} (x_j) = x_j\ \ \Rightarrow\\ \varphi_{x_{j}}^{a_j} \circ \varphi_{x_{j+1}}^{a_{j+1}} \circ \cdots \circ \varphi_{x_{i-1}}^{a_{i-1}} (x_i) = \varphi_{x_{j+1}}^{a_{j+1}} \circ \cdots \circ \varphi_{x_{i-1}}^{a_{i-1}} (x_i)\ \ \Rightarrow\\ \varphi_{x_1}^{a_1} \circ \varphi_{x_2}^{a_2} \circ \cdots \circ \varphi_{x_{i-1}}^{a_{i-1}} (x_i) = \varphi_{x_{1}}^{a_{1}} \circ \cdots \circ \varphi_{x_{j-1}}^{a_{j-1}} \circ \varphi_{x_{j+1}}^{a_{j+1}} \circ \cdots \circ \varphi_{x_{i-1}}^{a_{i-1}} (x_i)\ \ \Rightarrow\\ \gamma (U, x_i^{a_i}) = \gamma (L (U, x_j^{a_j}), x_i^{a_i})\,. \end{gather*} \end{proof} \begin{corl}\label{corl4_3} Let $U = \{x_1^{a_1}, x_2^{a_2}, \dots, x_p^{a_p}\}$ be a non-empty stratum and let $x_i^{a_i} \in U$. Suppose that, if $x_j > x_k$, then $j <k$. \begin{itemize} \item[(1)] Let $j \in \{1, \dots, p\}$ be such that $x_i \not \le x_j$. Then $\gamma (L(U, x_j^{a_j}), x_i^{a_i}) = \gamma (U, x_i^{a_i})$. \item[(2)] Let $j \in \{i-1, i, i+1, \dots, p\}$. Then $\gamma (U, x_i^{a_i}) = \left( \varphi_{x_1}^{a_1} \circ \varphi_{x_2}^{a_2} \circ \cdots \circ \varphi_{x_j}^{a_j}(x_i) \right)^{a_i}$. \end{itemize} \end{corl} \begin{proof} Part (1) is a straightforward consequence of Lemma \ref{lem4_2}. We prove Part (2). For $j \ge i$, we have $x_i \not \le x_j$, hence $\varphi_{x_j} (x_i) = x_i$. So, if $j \ge i$, then \[ \left( \varphi_{x_1}^{a_1} \circ \varphi_{x_2}^{a_2} \circ \cdots \circ \varphi_{x_j}^{a_j}(x_i) \right)^{a_i} = \left( \varphi_{x_1}^{a_1} \circ \varphi_{x_2}^{a_2} \circ \cdots \circ \varphi_{x_{i-1}}^{a_{i-1}}(x_i) \right)^{a_i} = \gamma (U, x_i^{a_i})\,. \proved \] \end{proof} \begin{lem}\label{lem4_4} The rewriting system $\RR$ is terminating. \end{lem} \begin{proof} Define the weight of a piling $u = (U_1, \dots, U_r) \in \Omega^*$ as $r + \sum_{i = 1}^r i \cdot \lg_\st (U_i)$. It is easily seen that each elementary rewriting step strictly reduces the weight. So, there is no infinite rewriting sequence, and therefore $\RR$ is terminating. \end{proof} \begin{prop}\label{prop4_5} The rewriting system $\RR$ is confluent. \end{prop} We know from Lemma \ref{lem4_4} that $\RR$ is terminating, hence, by Theorem \ref{thm4_1}, in order to prove Proposition \ref{prop4_5} it suffices to show that every critical pair is resolved. Let $(u_1, u_2, u_3, v_1, v_2)$ be a critical pair. Then we have one of the following three situations. \begin{itemize} \item[(C1)] $u_1 = \epsilon$, $u_2 = (\emptyset)$, $u_3 = (W)$, $v_1 = T (\emptyset, W, z^c)$ and $v_2 = \epsilon$, where $W \in \Omega$ and $z^c \in W$. This is a critical pair of type (b). \item[(C2)] $u_1 = (U)$, $u_2 = (V)$, $u_3 = (W)$, $v_1 = T (U, V, y^b)$ and $v_2 = T (V, W, z^c)$, where $U, V, W \in \Omega$, $y^b \in V$ and $z^c \in W$. This is a critical pair of type (a). \item[(C3)] $u_1 = u_3 = \epsilon$, $u_2 = (U, V)$, $v_1 = T (U, V, y^b)$ and $v_2 = T (U, V, y'^{b'})$, where $U, V \in \Omega$, $y^b, y'^{b'} \in V$ and $y^b \neq y'^{b'}$. This is a critical pair of type (b). \end{itemize} We show in each case that the critical pair is resolved. This requires a case-by-case proof through several lemmas. \begin{lem}\label{lem4_6} Let $x, y, z \in V (\Gamma)$ be such that $z \le y \le x$. Then: \begin{itemize} \item[(1)] $(\varphi_x \circ \varphi_y) (z) = (\varphi_{\varphi_x (y)} \circ \varphi_x) (z)$, \item[(2)] $(\varphi_x^{-1} \circ \varphi_y) (z) = (\varphi_{\varphi_x^{-1} (y)}\circ \varphi_x^{-1}) (z)$, \item[(3)] $(\varphi_x \circ \varphi_y^{-1}) (z) = (\varphi_{\varphi_x(y)}^{-1} \circ \varphi_x) (z)$, \item[(4)] $(\varphi_x^{-1} \circ \varphi_y^{-1}) (z) = (\varphi_{\varphi_x^{-1}(y)}^{-1} \circ \varphi_x^{-1}) (z)$. \end{itemize} \end{lem} \begin{proof} Part (1) is Condition (g) in the definition of a trickle graph. For Part (2) we apply Part (1) to $z_1 = \varphi_x^{-1} (z) \le y_1 = \varphi_x^{-1} (y) \le x$. For Part (3) we apply Part (1) to $z_1 = \varphi_y^{-1} (z) \le y \le x$. Finally, for Part (4) we apply Part (2) to $z_1 = \varphi_y^{-1} (z) \le y \le x$. \end{proof} \begin{lem}\label{lem4_7} Let $x, y, z \in V (\Gamma)$ be such that $z \le y \le x$ and let $a, b \in \Z$. Then \[ (\varphi_x^a \circ \varphi_y^b) (z) = (\varphi_{\varphi_x^a (y)}^b \circ \varphi_x^a) (z)\,. \] \end{lem} \begin{proof} If either $a=0$ or $b=0$ then the result is obvious. So, we can assume that $a \neq 0$ and $b \neq 0$. From here the proof is divided into four cases, depending on whether $a > 0$ and $b > 0$, or $a < 0$ and $b > 0$, or $a > 0$ and $b < 0$, or $a < 0$ and $b < 0$. We treat the case $a > 0$ and $b >0$. The other three cases can be treated in the same way. So, we take two integers $a > 0$ and $b >0$. First, we prove by induction on $a$ that \[ (\varphi_x^a \circ \varphi_y) (z) = (\varphi_{\varphi_x^a (y)} \circ \varphi_x^a) (z)\,. \] The case $a = 1$ is Lemma \ref{lem4_6}\,(1). Assume $a \ge 2$ and that the induction hypothesis holds. Let $y_1 = \varphi_x (y)$ and $z_1 = \varphi_x (z)$. By applying the induction hypothesis to $z_1 \le y_1 \le x$ and Lemma \ref{lem4_6}\,(1) we get: \begin{gather*} (\varphi_{\varphi_x^a (y)} \circ \varphi_x^a) (z) = (\varphi_{\varphi_x^{a-1} (y_1)} \circ \varphi_x^{a-1}) (z_1) = (\varphi_x^{a-1} \circ \varphi_{y_1}) (z_1) = (\varphi_x^{a-1} \circ \varphi_{\varphi_x (y)} \circ \varphi_x) (z) =\\ (\varphi_x^{a-1} \circ \varphi_x \circ \varphi_y) (z) = (\varphi_x^a \circ \varphi_y) (z)\,. \end{gather*} Now we fix $a \ge 1$ and we prove by induction on $b$ that \[ (\varphi_x^a \circ \varphi_y^b) (z) = (\varphi_{\varphi_x^a (y)}^b \circ \varphi_x^a) (z)\,. \] The case $b = 1$ is proved in the previous paragraph. Assume $b \ge 2$ and that the induction hypothesis holds. Let $z_1 = \varphi_y^{b-1} (z)$. By applying the induction hypothesis and the case $b = 1$ to $z_1 \le y \le x$ we get: \begin{gather*} (\varphi_{\varphi_x^a (y)}^b \circ \varphi_x^a) (z) = (\varphi_{\varphi_x^a (y)} \circ \varphi_{\varphi_x^a (y)}^{b-1} \circ \varphi_x^a) (z) = (\varphi_{\varphi_x^a (y)} \circ \varphi_x^a \circ \varphi_y^{b-1}) (z) =\\ (\varphi_{\varphi_x^a (y)} \circ \varphi_x^a) (z_1) = (\varphi_x^a \circ \varphi_y) (z_1) = (\varphi_x^a \circ \varphi_y^b) (z)\,. \end{gather*} \end{proof} \begin{lem}\label{lem4_8} In Case (C1) every critical pair $(u_1, u_2, u_3, v_1, v_2)$ is resolved. \end{lem} \begin{proof} We have $u_1 = \epsilon$, $u_2 = (\emptyset)$, $u_3 = (W)$, $v_1 = T (\emptyset, W, z^c)$ and $v_2 = \epsilon$, where $W \in \Omega$ and $z^c \in W$. We must show that there exists $w \in \Omega^*$ such that $v_1 \to^* w$ and $u_1 \cdot v_2 \cdot u_3 \to^* w$. In fact, we set $w = u_1 \cdot v_2 \cdot u_3 = u_3 = (W)$ and we show that $v_1 \to^* w$. We set $W = \{z_1^{c_1}, \dots, z_p^{c_p}\}$ so that, if $z_j > z_k$, then $j < k$. Let $i \in \{1, \dots, p\}$ be such that $z^c = z_i^{c_i}$. Let $y = (\varphi_{z_1}^{c_1} \circ \cdots \circ \varphi_{z_{i-1}}^{c_{i-1}}) (z_i)$. We have $\gamma (W, z^c) = y^c$, and $v_1 = T (\emptyset, W, z^c) = (\{y^c\}, L(W, z^c))$. We define $(V_j, W_j) \in \Omega^*$ for $j \in \{0,1, \dots, p-1\}$ as follows. We set $V_0 = \{y^c\}$ and $W_0 = L(W, z^c)$. For $1 \le j< i$ we set \[ V_j = \{z_1^{c_1}, \dots, z_j^{c_j}, u_j^c \} \text{ and } W_j = \{z_{j+1}^{c_{j+1}}, \dots, z_{i-1}^{c_{i-1}}, z_{i+1}^{c_{i+1}}, \dots, z_p^{c_p}\}\,, \] where \[ u_j = (\varphi_{z_{j+1}}^{c_{j+1}} \circ \cdots \circ \varphi_{z_{i-1}}^{c_{i-1}}) (z_i)\,. \] For $i \le j \le p-1$ we set \[ V_j = \{z_1^{c_1}, \dots, z_{j+1}^{c_{j+1}}\} \text{ and } W_j = \{z_{j+2}^{c_{j+2}}, \dots, z_p^{c_p}\}\,. \] We show that $(V_j, W_j) \to (V_{j+1}, W_{j+1})$ for all $j \in \{0, \dots, p-2\}$. Since $(V_0, W_0) = v_1$ and $(V_{p-1}, W_{p-1}) = (W, \emptyset) \to (W) =w$, this implies that $v_1 \to^*w$. For $1 \le j < i$ we have $\gamma (W_{j-1}, z_j^{c_j}) = z_j^{c_j}$, and for $i \le j \le p-2$ we have $\gamma (W_{j-1}, z_{j+1}^{c_{j+1}}) = z_{j+1}^{c_{j+1}}$. For $1 \le j < i$ we have $L (W_{j-1} ,z_j^{c_j}) = W_j$, and for $i \le j \le p-2$ we have $L (W_{j-1}, z_{j+1}^{c_{j+1}}) = W_j$. For $1 \le k < j \le p$ we have $\{z_k, z_j\} \in E(\Gamma)$, because $W$ is a stratum, and for $j < i$ we have $\{u_{j-1}, z_j\} \in E(\Gamma)$, because, by the choice of the numbering of the indices, \[ u_{j-1} = (\varphi_{z_j}^{c_j} \circ \cdots \circ \varphi_{z_{i-1}}^{c_{i-1}}) (z_i) \text{ and } z_j = (\varphi_{z_j}^{c_j} \circ \cdots \circ \varphi_{z_{i-1}}^{c_{i-1}}) (z_j)\,. \] So, for $1 \le j \le i-1$ the syllable $z_j^{c_j}$ can be added to $V_{j-1}$ and $R (V_{j-1}, z_j^{c_j}) = V_j$. Similarly, for $i \le j \le p-2$ the syllable $z_{j+1}^{c_{j+1}}$ can be added to $V_{j-1}$ and $R (V_{j-1}, z_{j+1}^{c_{j+1}}) = V_j$. We deduce that, for every $j \in \{1, \dots, i-1\}$, we have $(V_j, W_j) = T(V_{j-1}, W_{j-1}, z_j^{c_j})$, and, for every $j \in \{i, \dots, p-2\}$, we have $(V_j, W_j) = T (V_{j-1}, W_{j-1}, z_{j+1}^{c_{j+1}})$. So, $(V_j, W_j) \to (V_{j+1}, W_{j+1})$ for all $j \in \{0, \dots, p-2\}$. \end{proof} \begin{lem}\label{lem4_9} Let $U$ be a non-empty stratum and let $x^a \in U$. Let $y^b \in S (\Gamma)$ be a syllable such that $x \neq y$ and $y^b$ can be added to $U$. Then \[ \gamma (R (U, y^b), \varphi_y^{-b} (x)^a) = \gamma (U, x^a)\,. \] \end{lem} \begin{proof} We set $U = \{ x_1^{a_1}, \dots, x_p^{a_p}\}$ and we assume as always that, if $x_j > x_k$, then $j<k$. Let $i \in \{1, \dots, p\}$ be such that $x^a =x_i^{a_i}$. By Lemma \ref{lem4_2} we can assume that $U (x^a) = \{x_1^{a_1}, \dots, x_{i-1}^{a_{i-1}}, x_i^{a_i} \}$ and $x_1 > \cdots > x_{i-1} > x_i$. From here the proof of the lemma is divided into two cases depending on whether $x \not < y$ or $x < y$. {\it Case 1: $x \not < y$.} Let $j \in \{1, \dots, p\}$. Note that \[ \varphi_y^{-b} (x_j) \ge \varphi_y^{-b} (x_i)\ \Leftrightarrow\ x_j \ge x_i \Leftrightarrow\ 1\le j \le i\,. \] Suppose $1\le j \le i$. If $x > y$, then $x_j > y$, hence $\varphi_y^{-b} (x_j) = x_j$. Suppose $y || x$. If we had $y > x_j$, then by Condition (b) in the definition of a trickle graph we would have $x_i || x_j$, which would contradict the hypothesis $x_j \ge x_i$. Thus, $y || x_j$ or $y< x_j$, hence $\varphi_y^{-b} (x_j) = x_j$. So, $\varphi_y^{-b} (x_j) = x_j$ for all $j \in \{1, \dots, i-1,i\}$, hence $R (U, y^b) (x^a) = U (x^a)$. It follows that \[ \gamma (R (U, y^b), \varphi_y^{-b} (x)^a) = \gamma (U (x^a), x^a) = \gamma (U, x^a) \,. \] {\it Case 2: $x < y$.} By Lemma \ref{lem4_2} there exists $k \in \{1, \dots, i-1 \}$ such that $x_{k-1} > y > x_k$ if $y \not \in \supp (U)$ (which simply means $y > x_1$ if $k=1$) and $y = x_k$ if $y \in \supp (U)$. For each $j \in \{k, \dots, i-1, i \}$ we set $z_j = \varphi_y^{-b} (x_j)$. In particular, $\varphi_y^{-b} (x) = z_i$. We have \begin{gather*} R(U, y^b) (z_i^a) = \{ x_1^{a_1}, \dots, x_{k-1}^{a_{k-1}}, y^b, z_k^{a_k}, z_{k+1}^{a_{k+1}}, \dots, z_{i-1}^{a_{i-1}}, z_i^{a_i} \} \text{ if } y \not \in \supp (U)\,,\\ R(U, y^b) (z_i^a) = \{ x_1^{a_1}, \dots, x_{k-1}^{a_{k-1}}, x_k^{b+a_k}, z_{k+1}^{a_{k+1}}, \dots, z_{i-1}^{a_{i-1}}, z_i^{a_i} \} \text{ if } y = x_k \in \supp (U) \text{ and } b + a_k \neq 0\,,\\ R(U, y^b) (z_i^a) = \{ x_1^{a_1}, \dots, x_{k-1}^{a_{k-1}}, z_{k+1}^{a_{k+1}}, \dots, z_{i-1}^{a_{i-1}}, z_i^{a_i} \} \text{ if } y= x_k \in \supp (U) \text{ and } b + a_k = 0\,. \end{gather*} In the three cases we have \[ \gamma (R (U, y^b), \varphi_y^{-b} (x)^a) = \gamma (R (U, y^b) (z_i^{a_i}), z_i^{a_i}) = (\varphi_{x_1}^{a_1} \circ \cdots \circ \varphi_{x_{k-1}}^{a_{k-1}} \circ \varphi_y^b \circ \varphi_{z_k}^{a_k} \circ \cdots \circ \varphi_{z_{i-1}}^{a_{i-1}}) (z_i)^a\,. \] By applying Lemma \ref{lem4_7} several times we get \begin{gather*} (\varphi_{z_k}^{a_k} \circ \cdots \circ \varphi_{z_{i-1}}^{a_{i-1}}) (z_i) =\\ (\varphi_{z_k}^{a_k} \circ \cdots \circ \varphi_{z_{i-2}}^{a_{i-2}} \circ \varphi_{\varphi_y^{-b} (x_{i-1})}^{a_{i-1}} \circ \varphi_y^{-b}) (x_i) =\\ (\varphi_{z_k}^{a_k} \circ \cdots \circ \varphi_{\varphi_y^{-b} (x_{i-2})}^{a_{i-2}} \circ \varphi_y^{-b} \circ \varphi_{x_{i-1}}^{a_{i-1}}) (x_i) = \cdots =\\ (\varphi_y^{-b} \circ \varphi_{x_k}^{a_k} \circ \cdots \circ \varphi_{x_{i-1}}^{a_{i-1}}) (x_i)\,, \end{gather*} hence \[ \gamma (R (U, y^b), \varphi_y^{-b} (x)^a) = (\varphi_{x_1}^{a_1} \circ \cdots \circ \varphi_{x_{k-1}}^{a_{k-1}} \circ \varphi_{x_k}^{a_k} \circ \cdots \circ \varphi_{x_{i-1}}^{a_{i-1}}) (x_i)^a = \gamma (U,x^a)\,. \proved \] \end{proof} \begin{lem}\label{lem4_10} In Case (C2) every critical pair $(u_1,u_2,u_3,v_1,v_2)$ is resolved. \end{lem} \begin{proof} There exist $U, V, W \in \Omega$, $y^b \in V$ and $z^c \in W$ such that $u_1 = (U)$, $u_2 = (V)$, $ u_3 = (W)$, $v_1 = T(U, V, y^b)$ and $v_2 = T(V, W, z^c)$. Let $y'^b = \gamma (V, y^b)$. Then $y'^b$ can be added to $U$ and $T(U,V,y^b) = (U_1, V_1)$, where $U_1 = R (U, y'^b)$ and $V_1 = L (V, y^b)$. Let $z'^c = \gamma (W, z^c)$. Then $z'^c$ can be added to $V$ and $T(V, W, z^c) = (V_2, W_2)$, where $V_2 = R(V, z'^c)$ and $W_2 = L(W, z^c)$. We must show that there exists $w \in \Omega^*$ such that $(U_1, V_1, W) \to^* w$ and $(U, V_2, W_2) \to^* w$. We set $V=\{ y_1^{b_1}, \dots, y_p^{b_p} \}$ and we assume as always that, if $y_j > y_k$, then $j < k$. Let $i \in \{1, \dots, p\}$ be such that $y^b = y_i^{b_i}$. From here the proof is divided into three cases depending on whether $z' \neq y$, or $z'=y$ and $b+c \neq 0$, or $z'=y$ and $b +c=0$. {\it Case 1: $z' \neq y$.} We set \begin{gather*} V_3 = \{ \varphi_{z'}^{-c} (y_k)^{b_k} \mid 1 \le k \le p \text{ and } k \neq i \} \cup \{ z'^c \} \text{ if } z' \not \in \supp (V)\,,\\ V_3 = \{ \varphi_{z'}^{-c} (y_k)^{b_k} \mid 1 \le k \le p \text{ and } k \not\in \{i, j\} \} \cup \{ z'^{b_j +c} \} \text{ if } z' =y_j \in \supp (V) \text{ and } b_j + c \neq 0\,,\\ V_3 = \{ \varphi_{z'}^{-c} (y_k)^{b_k} \mid 1 \le k \le p \text{ and } k \not\in \{i, j\} \} \text{ if } z' =y_j \in \supp (V) \text{ and } b_j + c = 0\,, \end{gather*} and we set $w= (U_1, V_3, W_2)$. Observe that $z'^c$ can be added to $V_1$ and $R(V_1,z'^c) = V_3$, hence $T(V_1,W,z^c) = (V_3, W_2)$, and therefore $(U_1, V_1, W) \to (U_1, V_3, W_2) = w$. By Lemma \ref{lem4_9} we have $\gamma (V_2, \varphi_{z'}^{-c} (y)^b) = \gamma (V, y^b) = y'^b$. We also know that $y'^b$ can be added to $U$ and $R(U,y'^b) = U_1$. Moreover, $L (V_2, \varphi_{z'}^{-c} (y)^b) = V_3$, hence $T(U, V_2, \varphi_{z'}^{-c} (y)^b) = (U_1, V_3)$, and therefore $(U, V_2, W_2) \to (U_1, V_3, W_2) = w$. {\it Case 2: $z' = y$ and $b+c \neq 0$.} We set \[ V_4 = \{ y_1^{b_1}, \dots, y_{i-1}^{b_{i-1}}, \varphi_y^{-c} (y_{i+1})^{b_{i+1}}, \dots, \varphi_y^{-c} (y_p)^{b_p} \}\,, \] $U_4= R (U, y'^{b+c})$, and $w = (U_4, V_4, W_2)$. The syllable $z'^c = y^c$ can be added to $V_1$ and $R(V_1, z'^c) = V_3$, where \[ V_3 = \{ y_1^{b_1}, \dots, y_{i-1}^{b_{i-1}}, y^c, \varphi_y^{-c} (y_{i+1})^{ b_{i+1}}, \dots, \varphi_y^{-c} (y_p)^{b_p} \}\,, \] hence $T(V_1,W, z^c) = (V_3,W_2)$, and therefore $(U_1, V_1, W) \to (U_1, V_3, W_2)$. We have $\gamma (V_3, y^c) = y'^c$, which can be added to $U_1$, $R(U_1, y'^c) = R (U, y'^{b+c}) = U_4$, and $L(V_3, y^c) = V_4$, hence $T (U_1, V_3, y^c) = (U_4, V_4)$, and therefore $(U_1, V_3, W_2) \to (U_4, V_4, W_2) = w$. So, $(U_1, V_1, W) \to^* w$. Note that \[ V_2 = \{ y_1^{b_1}, \dots, y_{i-1}^{b_{i-1}}, y^{b+c}, \varphi_y^{-c} (y_{i+1})^{b_{i+1}}, \dots, \varphi_y^{-c} (y_p)^{b_p} \}\,. \] We have $\gamma (V_2, y^{b+c}) = y'^{b+c}$, which can be added to $U$. Moreover, $R (U, y'^{b+c}) = U_4$ and $L (V_2, y^{b+c}) = V_4$, hence $T (U, V_2, y^{b+c}) = (U_4, V_4)$, and therefore $(U,V_2, W_2) \to (U_4, V_4, W_2)=w$. {\it Case 3: $z'=y$ and $b+c=0$.} We set $w = (U, V_2, W_2)$ and we show that $(U_1, V_1, W) \to^* w$. The syllable $z'^c = y^c$ can be added to $V_1$ and $R(V_1,z'^c) = V_3$, where \[ V_3 = \{ y_1^{b_1}, \dots, y_{i-1}^{b_{i-1}}, y^c, \varphi_y^{-c} (y_{i+1})^{b_{i+1}}, \dots, \varphi_y^{-c} (y_p)^{b_p} \}\,, \] hence $T (V_1, W, z^c) = (V_3, W_2)$, and therefore $(U_1, V_1, W) \to (U_1, V_3, W_2)$. We have $\gamma (V_3, y^c) = y'^c$, which can be added to $U_1$. We have $R(U_1, y'^c) = U$, because $c=-b$, and we have $L(V_1,y^c) = V_2$, since \[ V_2 = \{ y_1^{b_1}, \dots, y_{i-1}^{b_{i-1}}, \varphi_y^{-c} (y_{i+1})^{b_{i+1}}, \dots, \varphi_y^{-c} (y_p)^{b_p} \}\,, \] hence $T (U_1, V_3, y^c) = (U, V_2)$, and therefore $(U_1, V_3, W_2) \to (U, V_2, W_2) = w$. So, $(U_1, V_1, W) \to^* w$. \end{proof} \begin{lem}\label{lem4_11} Let $U$ be a non-empty stratum and let $x^a, y^b \in U$ be such that $x \neq y$. Let $x'^a = \gamma (U, x^a)$, $y'^b = \gamma (U, y^b)$, $x''^a = \gamma (L (U, y ^b), x^a)$ and $y''^b = \gamma (L(U, x^a), y^b)$. \begin{itemize} \item[(1)] We have $\{x',y'\}, \{x'',y'\} \in E(\Gamma)$. \item[(2)] We have $x || y$ if and only if $x' || y'$, and we have $x \le y$ if and only if $x' \le y'$. \item[(3)] We have $\varphi_{x'}^{-a} (y') = \varphi_{x''}^{-a} (y') = y''$. \item[(4)] Let $V$ be another stratum. If both $x'^a$ and $y'^b$ can be added to $V$, then $y''^b$ can be added to $R(V,x'^a)$. \end{itemize} \end{lem} \begin{proof} We set $U = \{ x_1^{a_1}, \dots, x_p^{a_p} \}$ and we suppose that, if $x_k > x_\ell$, then $k < \ell$. Let $i,j \in \{1, \dots, p\}$ be such that $x=x_i$ and $y = x_j$. By Lemma \ref{lem4_2} we can assume that $U (y) = \{x_1^{a_1}, \dots, x_{j-1}^{a_{j-1}}, x_j^{a_j} \}$ and $x_1 > \cdots > x_{j-1} > x_j= y$. From here the proof is divided into two cases depending on whether $y \not < x$, or $y < x$. {\it Case 1: $y \not < x$.} Since $U (y) = \{x_1^{a_1}, \dots, x_{j-1}^{a_{j-1}}, x_j^{a_j} \}$, we have $i > j$. By definition, $x' = (\varphi_{x_1}^{a_1} \circ \cdots \circ \varphi_{x_{i-1}}^{a_{i-1}}) (x_i)$ and, by Corollary \ref{corl4_3}\,(2), $y' = (\varphi_{x_1}^{a_1} \circ \cdots \circ \varphi_{x_{i-1}}^{a_{i-1}}) (x_j)$, hence $\{x',y'\} \in E (\Gamma)$, we have $x|| y$ if and only if $x' || y'$, and we have $x < y$ if and only if $x' < y'$. Moreover, since $y \not < x$, we have $y' \not < x'$, hence $\varphi_{x'}^{-a} (y') = y'$. We have $L (U, y^b) = \{ x_1^{a_1}, \dots, x_{j-1}^{a_{j-1}}, x_{j+1}^{a_{j+1}}, \dots, x_p^{a_p} \}$, hence $x'' = (\varphi_{x_1}^{a_1} \circ \cdots \circ \varphi_{x_{j-1}}^{a_{j-1}} \circ \varphi_{x_{j+1}}^{a_{j+1}} \circ \cdots \circ \varphi_{x_{i-1}}^{a_{i-1}}) (x_i)$. On the other hand, if $k > j$, then $x_k \not > x_j$, hence $\varphi_{x_k}^{a_k} (x_j) = x_j$. So, \[ y' = (\varphi_{x_1}^{a_1} \circ \cdots \circ \varphi_{x_{j-1}}^{a_{j-1}}) (x_j) = (\varphi_{x_1}^{a_1} \circ \cdots \circ \varphi_{x_{j-1}}^{a_{j-1}} \circ \varphi_{x_{j+1}}^{a_{j+1}} \circ \cdots \circ \varphi_{x_{i-1}}^{a_{i-1}}) (x_j)\,. \] It follows that $\{x'', y' \} \in E (\Gamma)$ and $y' \not < x''$. In particular, $\varphi_{x''}^{-a} (y') = y'$. We have $L (U, x^a) = \{ x_1^{a_1}, \dots, x_{i-1}^{a_{i-1}}, x_{i+1}^{a_{i+1}}, \dots, x_p^{a_p} \}$, hence $y'' = (\varphi_{x_1}^{a_1} \circ \cdots \circ \varphi_{x_{j-1}}^{a_{j-1}}) (x_j) = y'$. So, $\varphi_{x'}^{-a} (y') = \varphi_{x''}^{-a} (y') = y'' = y'$. Let $V$ be another stratum such that both $x'^a$ and $y'^b$ can be added to $V$. Set $V = \{z_1^{c_1}, \dots, z_q^{c_q} \}$. The support of $R (V, x'^a)$ is contained in $\{\varphi_{x'}^{-a} (z_1), \dots, \varphi_{x'}^{-a} (z_q)\} \cup \{x'\}$. Since $y'^b$ can be added to $V$, $y'' = \varphi_{x'}^{-a}(y')$, and $\{x', y'\} \in E(\Gamma)$, it follows that $y''^b$ can be added to $R (V, x'^a)$. {\it Case 2: $y < x$.} Since $U (y) = \{x_1^{a_1}, \dots, x_{j-1}^{a_{j-1}}, x_j^{a_j} \}$, we have $i < j$. We have $y' = (\varphi_{x_1}^{a_1} \circ \cdots \circ \varphi_{x_{j-1}}^{a_{j-1}}) (x_j)$ and, by Corollary \ref{corl4_3}\,(2), $x' = (\varphi_{x_1}^{a_1} \circ \cdots \circ \varphi_{x_{j-1}}^{a_{j-1}}) (x_i)$, hence $\{x', y' \} \in E (\Gamma)$ and $y' < x'$. Furthermore, $L(U, y^b) = \{x_1^{a_1}, \dots, x_{j-1}^{a_{j-1}}, x_{j+1}^{a_{j+1}}, \dots, x_p^{a_p}\}$, hence, $x'' = (\varphi_{x_1}^{a_1} \circ \cdots \circ \varphi_{x_{i-1}}^{a_{i-1}}) (x_i) = x'$. In particular, $\{x'', y'\} \in E (\Gamma)$ and $y' < x''$. Since $L (U, x^a) = \{x_1^{a_1}, \dots, x_{i-1}^{a_{i-1}}, x_{i+1}^{a_{i+ 1}}, \dots, x_p^{a_p} \}$, we have $y'' = (\varphi_{x_1}^{a_1} \circ \cdots \circ \varphi_{x_{i-1}}^{a_{i-1}} \circ \varphi_{x_{i+1}}^{a_{i+1}} \circ \cdots \circ \varphi_{x_{j-1}}^{a_{j-1}}) (x_j)$. On the other hand, by applying Lemma \ref{lem4_7} several times we get \begin{gather*} \varphi_{x''}^{-a} (y') = \varphi_{x'}^{-a} (y') = \big( \varphi_{(\varphi_{x_1}^{a_1} \circ \cdots \circ \varphi_{x_{i-1}}^{a_{i-1}}) (x_i)}^{-a} \circ \varphi_{x_1}^{a_1} \circ \cdots \circ \varphi_{x_{j-1}}^{a_{j-1}} \big) (x_j) =\\ \big( \varphi_{x_1}^{a_1} \circ \varphi_{(\varphi_{x_2}^{a_2} \circ \cdots \circ \varphi_{x_{i-1}}^{a_{i-1}}) (x_i)}^{-a} \circ \varphi_{x_2}^{a_2} \circ \cdots \circ \varphi_{x_{j-1}}^{a_{j-1}} \big) (x_j) = \cdots =\\ (\varphi_{x_1}^{a_1} \circ \cdots \circ \varphi_{x_{i-1}}^{a_{i-1}} \circ \varphi_{x_i}^{-a} \circ \varphi_{x_i}^a \circ \varphi_{x_{i+1}}^{a_{i+1}} \circ \cdots \circ \varphi_{x_{j-1}}^{a_{j-1}}) (x_j) = \\ (\varphi_{x_1}^{a_1} \circ \cdots \circ \varphi_{x_{i-1}}^{a_{i-1}} \circ \varphi_{x_{i+1}}^{a_{i+1}} \circ \cdots \circ \varphi_{x_{j-1}}^{a_{j-1}}) (x_j) = y''\,. \end{gather*} Let $V$ be another stratum such that both $x'^a$ and $y'^b$ can be added to $V$. Then we show that $y''^b$ can be added to $R (V, x'^a)$ in the same way as in Case 1. \end{proof} \begin{lem}\label{lem4_12} In Case (C3) every critical pair $(u_1, u_2, u_3, v_1, v_2)$ is resolved. \end{lem} \begin{proof} There exist $U, V \in \Omega$ and $y^b, z^c \in V$ such that $y \neq z$, $u_1 = u_3 = \epsilon$, $u_2 = (U,V) $, $v_1 = T (U, V, y^b)$ and $v_2 = T (U, V, z^c)$. Let $y'^b = \gamma (V, y^b)$. Then $y'^b$ can be added to $U$ and $T (U, V, y^b) = (U_1, V_1)$, where $U_1 = R(U, y'^b)$ and $V_1 = L (V, y^b)$. Let $z'^c = \gamma (V, z^c)$. Then $z'^c$ can be added to $U$ and $T (U, V, z^c) = (U_2, V_2)$, where $U_2 = R (U, z'^c)$ and $V_2 = L(V, z^c)$. We must show that there exists $w \in \Omega^*$ such that $(U_1, V_1) \to^* w$ and $(U_2, V_2) \to^* w$. Let $z''^c = \gamma (V_1, z^c)$ and $y''^b = \gamma (V_2, y^b)$. We know from Lemma \ref{lem4_11}\,(4) that $z''^c$ can be added to $U_1$ and $y''^b$ can be added to $U_2$. Let $(U_3, V_3) = T (U_1, V_1, z^c)$ and $(U_4, V_4) = T (U_2, V_2, y^b)$, where $U_3 = R (U_1, z''^c)$, $V_3 = L (V_1, z^c)$, $U_4 = R (U_2, y''^b)$ and $V_4 = L(V_2, y^b)$. We will prove that $(U_3, V_3) = (U_4, V_4)$. Then, by setting $w = (U_3, V_3) = (U_4, V_4)$, we will have $(U_1, V_1) \to w$ and $(U_2, V_2) \to w$. Clearly, $V_3 = V_4 = V \setminus \{ y^b, z^c\}$, hence we only need to prove that $U_3 = U_4$. We set $U= \{x_1^{a_1}, \dots, x_p^{a_p} \}$ and we assume as always that, if $x_i > x_j$, then $i < j$. The rest of the proof is divided into two cases depending on whether $y || z$, or $y < z$. The case $z < y$ is treated in the same way as the case $y < z$ by interchanging the roles of $y$ and $z$. {\it Case 1: $y || z$.} By Lemma \ref{lem4_11} we have $y' || z'$, $y'' = \varphi_{z'}^{-c} (y') = y'$, and $z'' = \varphi_{y'}^{-b} (z') = z'$. Let \begin{gather*} W_1 = \{ x_i^{a_i} \mid 1 \le i \le p \text{ and } x_i \le y'\}\,, \ W_2 = \{ x_i^{a_i} \mid 1 \le i \le p \text{ and } x_i \le z'\}\,, \\ W_3 = U \setminus (W_1 \cup W_2)\,. \end{gather*} Condition (b) in the definition of a trickle graph implies that $W_1 \cap W_2 = \emptyset$, hence we have the disjoint union $U = W_1 \sqcup W_2 \sqcup W_3$. We have $U_1 = R(U, y'^b) = W_1' \sqcup W_2 \sqcup W_3$, where \begin{gather*} W_1' = \{ \varphi_{y'}^{-b} (x_i)^{a_i} \mid 1 \le i \le p \text{ and } x_i \le y' \} \cup \{y'^b \} \text{ if } y' \not \in \supp (W_1)\,,\\ W_1' = \{ \varphi_{y'}^{-b} (x_i)^{a_i} \mid 1 \le i \le p\,,\ x_i \le y' \text{ and } i \neq k \} \cup \{y'^{a_k+b} \} \text{ if } y' = x_k \in \supp (W_1)\\ \hskip 5 truecm \text{ and } a_k + b \neq 0\,,\\ W_1' = \{ \varphi_{y'}^{-b} (x_i)^{a_i} \mid 1 \le i \le p\,,\ x_i \le y' \text{ and } i \neq k \} \text{ if } y' = x_k \in \supp (W_1) \text{ and } a_k + b = 0\,. \end{gather*} Then $U_3 = R (U_1, z''^c) = R(U_1, z'^c) = W_1' \sqcup W_2' \sqcup W_3$, where \begin{gather*} W_2' = \{ \varphi_{z'}^{-c} (x_i)^{a_i} \mid 1 \le i \le p \text{ and } x_i \le z' \} \cup \{z'^c \} \text{ if } z' \not \in \supp (W_2)\,,\\ W_2' = \{ \varphi_{z'}^{-c} (x_i)^{a_i} \mid 1 \le i \le p\,,\ x_i \le z' \text{ and } i \neq \ell \} \cup \{z'^{a_\ell+c} \} \text{ if } z' = x_\ell \in \supp (W_2)\\ \hskip 5 truecm \text{ and } a_\ell + c \neq 0\,,\\ W_2' = \{ \varphi_{z'}^{-c} (x_i)^{a_i} \mid 1 \le i \le p\,,\ x_i \le z' \text{ and } i \neq \ell \} \text{ if } z' = x_\ell \in \supp (W_2) \text{ and } a_\ell + c = 0\,. \end{gather*} By using the same argument we prove that $U_2= R(U, z'^c) = W_1 \sqcup W_2' \sqcup W_3$, and then that $U_4 = R(U_2, y''^b) = R (U_2, y'^b) = W_1' \sqcup W_2' \sqcup W_3$, hence $U_3 = U_4$. {\it Case 2: $y < z$.} By Lemma \ref{lem4_11} we have $y' < z'$, $z'' = \varphi_{y'}^{-b} (z') = z'$, $y''=\varphi_{z'} ^{-c} (y')$ and $y'' < z'$. Let \begin{gather*} W_1 = \{ x_i^{a_i} \mid 1 \le i \le p \text{ and } x_i \le y' \}\,,\ W_2 = \{ x_i^{a_i} \mid 1 \le i \le p\,,\ x_i \le z' \text{ and } x_i \not \le y' \}\,,\\ W_3 = U \setminus (W_1 \cup W_2)\,. \end{gather*} By construction we have the disjoint union $U = W_1 \sqcup W_2 \sqcup W_3$. We have $U_1 = R (U, y'^b) = W_1^{(1)} \sqcup W_2 \sqcup W_3$, where \begin{gather*} W_1^{(1)} = \{ \varphi_{y'}^{-b} (x_i)^{a_i} \mid 1 \le i \le p \text{ and } x_i \le y' \} \cup \{y'^b \} \text{ if } y' \not \in \supp (W_1)\,,\\ W_1^{(1)} = \{ \varphi_{y'}^{-b} (x_i)^{a_i} \mid 1 \le i \le p\,,\ x_i \le y' \text{ and } i \neq k \} \cup \{y'^{a_k+b} \} \text{ if } y' = x_k \in \supp (W_1)\\ \hskip 4 truecm \text{ and } a_k + b \neq 0\,,\\ W_1^{(1)} = \{ \varphi_{y'}^{-b} (x_i)^{a_i} \mid 1 \le i \le p\,,\ x_i \le y' \text{ and } i \neq k \} \text{ if } y' = x_k \in \supp (W_1) \text{ and } a_k + b = 0\,. \end{gather*} Then $U_3 = R (U_1, z''^c) = R(U_1, z'^c) = W_1^{(2)} \sqcup W_2' \sqcup W_3$, where \begin{gather*} W_1^{(2)} = \{(\varphi_{z'}^{-c} \circ \varphi_{y'}^{-b}) (x_i)^{a_i} \mid 1 \le i \le p \text{ and } x_i \le y' \} \cup \{y''^b \} \text{ if } y' \not \in \supp (W_1)\,,\\ W_1^{(2)} = \{ (\varphi_{z'}^{-c} \circ \varphi_{y'}^{-b}) (x_i)^{a_i} \mid 1 \le i \le p\,,\ x_i \le y' \text{ and } i \neq k \} \cup \{y''^{a_k+b} \}\\ \hskip 4 truecm \text{ if } y' = x_k \in \supp (W_1) \text{ and } a_k + b \neq 0\,,\\ W_1^{(2)} = \{ (\varphi_{z'}^{-c} \circ \varphi_{y'}^{-b}) (x_i)^{a_i} \mid 1 \le i \le p\,,\ x_i \le y' \text{ and } i \neq k \} \text{ if } y' = x_k \in \supp (W_1)\\ \hskip 4 truecm \text{ and } a_k + b = 0\,, \end{gather*} and \begin{gather*} W_2' = \{ \varphi_{z'}^{-c} (x_i)^{a_i} \mid 1 \le i \le p\,,\ x_i \le z' \text{ and } x_i \not \le y' \} \cup \{z'^c \} \text{ if } z' \not \in \supp (W_2)\,,\\ W_2' = \{ \varphi_{z'}^{-c} (x_i)^{a_i} \mid 1 \le i \le p\,,\ x_i \le z'\,,\ x_i \not \le y' \text{ and } i \neq \ell \} \cup \{z'^{a_\ell+c} \}\\ \hskip 4 truecm \text{ if } z' = x_\ell \in \supp (W_2) \text{ and } a_\ell + c \neq 0\,,\\ W_2' = \{ \varphi_{z'}^{-c} (x_i)^{a_i} \mid 1 \le i \le p\,,\ x_i \le z'\,,\ x_i \not \le y' \text{ and } i \neq \ell \} \text{ if } z' = x_\ell \in \supp (W_2)\\ \hskip 4 truecm \text{ and } a_\ell + c = 0\,. \end{gather*} On the other hand, $U_2 = R (U, z'^c) = W_1^{(3)} \sqcup W_2' \sqcup W_3$, where \[ W_1^{(3)} = \{ \varphi_{z'}^{-c} (x_i)^{a_i} \mid 1 \le i \le p \text{ and } x_i \le y'\}\,. \] Then $U_4 = R (U_2, y''^b) = W_1^{(4)} \sqcup W_2' \sqcup W_3$, where \begin{gather*} W_1^{(4)} = \{(\varphi_{y''}^{-b} \circ \varphi_{z'}^{-c}) (x_i)^{a_i} \mid 1 \le i \le p \text{ and } x_i \le y' \} \cup \{y''^b \} \text{ if } y' \not \in \supp (W_1)\,,\\ W_1^{(4)} = \{ (\varphi_{y''}^{-b} \circ \varphi_{z'}^{-c}) (x_i)^{a_i} \mid 1 \le i \le p\,,\ x_i \le y' \text{ and } i \neq k \} \cup \{y''^{a_k+b} \}\\ \hskip 4 truecm \text{ if } y' = x_k \in \supp (W_1) \text{ and } a_k + b \neq 0\,,\\ W_1^{(4)} = \{ (\varphi_{y''}^{-b} \circ \varphi_{z'}^{-c}) (x_i)^{a_i} \mid 1 \le i \le p\,,\ x_i \le y' \text{ and } i \neq k \}\text{ if } y' = x_k \in \supp (W_1)\\ \hskip 4 truecm \text{ and } a_k + b = 0\,. \end{gather*} Since $y'' = \varphi_{z'}^{-c} (y')$, by Lemma \ref{lem4_7} we have $(\varphi_{y''}^{-b} \circ \varphi_{z'}^{-c}) (x_i) = (\varphi_{z'}^{-c} \circ \varphi_{y'}^{-b}) (x_i)$ for all $i \in \{1, \dots, p\}$ such that $x_i \le y'$, hence $W_1^{(4)} = W_1^{(2)}$, and therefore $U_3 = U_4$. \end{proof} \begin{proof}[Proof of Proposition \ref{prop4_5}] We know from Lemma \ref{lem4_4} that $\RR$ is terminating, hence, by Theorem \ref{thm4_1}, it suffices to prove that every critical pair is resolved. We have seen that there are three types of critical pairs. In Case (1) we know that every critical pair is resolved by Lemma \ref{lem4_8}, in Case (2) we know that every critical pair is resolved by Lemma \ref{lem4_10}, and in Case (3) we know that every critical pair is resolved by Lemma \ref{lem4_12}. \end{proof} Now we know that $\RR$ is terminating and confluent (see Lemma \ref{lem4_4} and Proposition \ref{prop4_5}), hence, in order to prove Theorem \ref{thm2_4} it remains to show that $\RR$ is a rewriting system for $\Tr (\Gamma)$, that is, to prove that $\Tr (\Gamma) \simeq \langle \Omega \mid u=v \text{ for } (u,v) \in \RR \rangle^+$. We set $M (\Gamma) = \langle \Omega \mid u=v \text{ for } (u,v) \in \RR \rangle^+$. We will construct homomorphisms $\Phi: M (\Gamma) \to \Tr (\Gamma)$ and $\Psi: \Tr (\Gamma) \to M (\Gamma)$, and then we will prove that $\Psi \circ \Phi = \id_{M (\Gamma)}$ and $\Phi \circ \Psi = \id_{\Tr (\Gamma)}$. Let $U$ be a non-empty stratum. We write $U = \{ x_1^{a_1}, \dots, x_p^{a_p} \}$ so that, if $x_i > x_j$, then $i < j$, and we set \[ \omega (U) = x_1^{a_1} x_2^{a_2} \dots x_p^{a_p} \in \Tr (\Gamma)\,. \] If $U = \emptyset$, then we set $\omega (U) = 1$. \begin{lem}\label{lem4_13} Let $U \in \Omega$. Then the definition of $\omega (U)$ does not depend on the choice of the numbering of the elements of $U$. \end{lem} \begin{proof} Suppose we have $U = \{ x_1^{a_1}, \dots, x_p^{a_p} \} = \{ {x_1'}^{a_1'}, \dots, {x_p'}^{a_p'} \} $ so that, if $x_i > x_j$, then $i < j$, and if $x_i' > x_j'$, then $i < j$. We prove that $x_1^{a_1} \dots x_p^{a_p} = {x_1'}^{a_1'} \dots {x_p'}^{a_p'}$ by induction on $p = \lg_\st (U )$. If $p=0$, then $U = \emptyset$, and there is nothing to prove. We assume that $p \ge 1$ and that the induction hypothesis holds. Let $i \in \{1, \dots, p \}$ be such that $x_1^{a_1} = {x_i'}^{a_i'}$. Let $j \in \{1, \dots, p\}$. If $j < i$, then either $x_j' > x_i'$ or $x_j' || x_i'$. But, since ${x_i'}^{a_i'} = x_1^{a_1}$, there is no ${x_j'}^{a_j'} \in U$ such that $x_j' > x_i'$, hence, if $j < i$, then $x_j' || x_i'$. It follows that \begin{gather*} {x_1'}^{a_1'} \dots {x_{i-1}'}^{a_{i-1}'} {x_i'}^{a_i'} {x_{i+1}'}^{a_{i+1}'} \dots {x_p'}^{a_p'} = {x_i'}^{a_i'} {x_1'}^{a_1'} \dots {x_{i-1}'}^{a_{i-1}'} {x_{i+1}'}^{a_{i+1}'} \dots {x_p'}^{a_p'} =\\ x_1^{a_1} {x_1'}^{a_1'} \dots {x_{i-1}'}^{a_{i-1}'} {x_{i+1}'}^{a_{i+1}'} \dots {x_p'}^{a_p'}\,. \end{gather*} By the induction hypothesis applied to $U \setminus \{ x_1^{a_1} \}$, \[ x_2^{a_2} \dots x_p^{a_p} = {x_1'}^{a_1'} \dots {x_{i-1}'}^{a_{i-1}'} {x_{i+1}'}^{a_{i+1}'} \dots {x_p'}^{a_p'}\,, \] hence $x_1^{a_1} \dots x_p^{a_p} = {x_1'}^{a_1'} \dots {x_p'}^{a_p'}$. \end{proof} Let $\Phi^* : \Omega^* \to \Tr (\Gamma)$ be the homomorphism which sends $U$ to $\omega (U)$ for all $U \in \Omega$. It is easily verified that, for all $(u,v) \in \RR$, $\Phi^* (u) = \Phi^* (v)$, hence $\Phi^*$ induces a homomorphism $\Phi : M (\Gamma) \to \Tr (\Gamma)$ which sends $U$ to $\omega (U)$ for all $U \in \Omega$. For the construction of $\Psi : \Tr (\Gamma) \to M (\Gamma)$ we use the following monoid presentation for $\Tr (\Gamma)$: \[ \Tr (\Gamma) = \left\langle S (\Gamma) \left| \begin{array}{ll} x^a \cdot x^b = x^{a+b} &\text{for } x \in V(\Gamma),\ a, b \in \Z_{\mu (x)} \setminus \{0\} \text{ and } a+b \neq 0\,,\\ x^a \cdot x^b = 1 &\text{for } x \in V(\Gamma),\ a, b \in \Z_{\mu (x)} \setminus \{0\} \text{ and } a+b = 0\,,\\ \varphi_x^a(y)^b \cdot x^a = \varphi_y^b (x)^a \cdot y^b &\text{for } \{x, y\} \in E (\Gamma),\ a \in \Z_{\mu (x)} \setminus \{ 0 \},\ b \in \Z_{\mu (y)} \setminus \{ 0 \} \end{array} \right. \right\rangle^+\,. \] \begin{lem}\label{lem4_14} The homomorphism $\Psi^*: S (\Gamma)^* \to M (\Gamma)$ which sends $x^a$ to $(\{x^a\})$ induces a homomorphism $ \Psi: \Tr (\Gamma) \to M (\Gamma)$. \end{lem} \begin{proof} Let $x \in V (\Gamma)$ and $a,b \in \Z_{\mu (x)} \setminus \{ 0 \}$ be such that $a + b \neq 0$. We have the following sequence of rewriting transformations \[ (\{x^a\}, \{x^b\}) \stackrel{\RR}{\to} (\{x^{a+b} \}, \emptyset) \stackrel{\RR}{ \to} (\{ x^{a+b} \})\,, \] hence $\Psi^* (x^a \cdot x^b) = \Psi^* (x^{a+b})$. Let $x \in V (\Gamma)$ and $a,b \in \Z_{\mu (x)} \setminus \{ 0 \}$ be such that $a + b = 0$. We have the following sequence of rewriting transformations \[ (\{x^a\}, \{x^b\}) \stackrel{\RR}{\to} (\emptyset, \emptyset) \stackrel{\RR}{\to} (\emptyset) \stackrel {\RR}{\to} \epsilon\,, \] hence $\Psi^* (x^a \cdot x^b) = \Psi^* (1)$. Let $x^a, y^b \in S (\Gamma)$ be such that $\{x, y\} \in E (\Gamma)$ and $x || y$. We have the following sequences of rewriting transformations \begin{gather*} ( \{\varphi_x^a(y)^b\}, \{x^a\} ) = ( \{y^b\}, \{x^a\} ) \stackrel{\RR}{\to} ( \{x^a, y^b\}, \emptyset) \stackrel{\RR}{\to} ( \{x^a, y^b\})\,, \\ ( \{ \varphi_y^b (x)^a\}, \{y^b\} ) = ( \{ x^a\}, \{y^b\} ) \stackrel{\RR}{\to} (\{ x^a, y^b\}, \emptyset) \stackrel{\RR}{\to} ( \{x^a, y^b\})\,, \end{gather*} hence $\Psi^* (\varphi_x^a(y)^b \cdot x^a) = \Psi^*(\varphi_y^b (x)^a \cdot y^b)$. Let $x^a, y^b \in S (\Gamma)$ be such that $\{x, y\} \in E (\Gamma)$ and $x < y$. We have the following sequences of rewriting transformations \begin{gather*} ( \{\varphi_x^a(y)^b\}, \{x^a\} ) = ( \{y^b\}, \{x^a\} ) \stackrel{\RR}{\to} ( \{ y^b, x^a\}, \emptyset ) \stackrel{\RR}{\to} ( \{ y^b, x^a\})\,, \\ ( \{ \varphi_y^b (x)^a\}, \{y^b\} ) \stackrel{\RR}{\to} ( \{ y^b, (\varphi_y^{-b} \circ \varphi_y^b)(x)^a \}, \emptyset ) = ( \{ y^b, x^a\}, \emptyset ) \stackrel{\RR}{\to} ( \{ y^b, x^a\})\,, \end{gather*} hence $\Psi^* (\varphi_x^a(y)^b \cdot x^a) = \Psi^*(\varphi_y^b (x)^a \cdot y^b)$. \end{proof} \begin{lem}\label{lem4_15} We have $\Psi \circ \Phi = \id_{M (\Gamma)}$ and $\Phi \circ \Psi = \id_{\Tr (\Gamma)}$. \end{lem} \begin{proof} Let $U \in \Omega$. We write $U = \{ x_1^{a_1}, x_2^{a_2}, \dots, x_p^{a_p} \}$ so that, if $x_i > x_j$, then $i < j$. We have the following sequence of rewriting transformations in $\Omega^*$: \begin{gather*} ( \{ x_1^{a_1}\}, \{x_2^{a_2}\}, \dots, \{x_p^{a_p} \}) \to ( \{ x_1^{a_1}, x_2^{a_2}\}, \emptyset, \{x_3^{a_3}\}, \dots, \{x_p^{a_p} \}) \to ( \{ x_1^{a_1}, x_2^{a_2}\}, \{x_3^{a_3}\}, \dots, \{x_p^{a_p} \})\\ \to \cdots \to (\{x_1^{a_1}, \dots, x_{p-1}^{a_{p-1}} \}, \{x_p^{a_p}\} ) \to (\{x_1^{a_1}, \dots, x_{p-1}^{a_{p-1}}, x_p^{a_p} \}, \emptyset ) \to (\{x_1^{a_1}, \dots, x_{p-1}^{a_{p-1}}, x_p^{a_p} \}) = (U)\,. \end{gather*} So, \[ (\Psi \circ \Phi) ((U)) = \Psi ( x_1^{a_1} x_2 ^{a_2} \dots x_p^{a_p} ) = ( \{ x_1^{a_1}\}, \{x_2^{a_2}\}, \dots, \{x_p^{a_p} \} ) = (U)\,. \] Since $M (\Gamma)$ is generated by $\{ (U) \mid U \in \Omega \}$, this shows that $\Psi \circ \Phi = \id_{M (\Gamma)}$. For $x^a \in S (\Gamma)$ we have $(\Phi \circ \Psi) (x^a) = \Phi (\{x^a\}) = x^a$. Since $\Tr (\Gamma)$ is generated by $S (\Gamma)$, we conclude that $\Phi \circ \Psi = \id_{\Tr (\Gamma)}$. \end{proof} The following is a straightforward consequence of Lemma \ref{lem4_15}, and it ends the proof of Theorem \ref{thm2_4}. \begin{prop}\label{prop4_16} The pair $(\Omega,\RR)$ is a rewriting system for $\Tr (\Gamma)$. \end{prop} \section{The Tits-style algorithm}\label{sec5} In this section we fix a trickle graph $\Gamma = (\Gamma, \le, \mu, (\varphi_x)_{x \in V(\Gamma)})$ and we keep the notations of Subsection \ref{subsec2_3}. Our goal is to prove Theorem \ref{thm2_8}. We fix a total order $\preceq$ on $V(\Gamma)$ which extends the partial order $\le$ in the sense that, if $x \le y$, then $x \preceq y$. If $U$ is a non-empty stratum written $U = \{x_1^{a_1}, x_2^{a_2}, \dots, x_p^{a_p}\}$ with $x_1 \succ x_2 \succ \cdots \succ x_p$, then we set $\hat \omega^S (U) = (x_1^{a_1}, x_2^{a_2}, \dots, x_p^{a_p}) \in S (\Gamma)^*$. On the other hand we set $\hat \omega^S (U) = \epsilon$ if $U = \emptyset$. Notice that $\hat \omega^S (U)$ is a representative of $\omega (U)$. The following three lemmas are preliminaries to the proof of Theorem \ref{thm2_8}. \begin{lem}\label{lem5_1} Let $U = \{x_1^{a_1}, \dots, x_p^{a_p} \}$ be a stratum. We suppose that the numbering of the elements of $U$ is chosen so that, if $x_i > x_j$, then $i < j$. Let $v = (x_1^{a_1}, \dots, x_p^{a_p}) \in S (\Gamma)^*$. Then $v \stackrel{II\ *}{\to} \hat \omega^S (U)$. \end{lem} \begin{proof} We argue by induction on $p = \lg_\st (U)$. If $p = 0$, then $U = \emptyset$ and $v = \hat \omega^S (U) = \epsilon$. So, we can assume that $p \ge 1$ and that the induction hypothesis holds. We write $U=\{ {x_1'}^{a_1'}, \dots, {x_p'}^{a_p'} \}$ with $x_1' \succ \cdots \succ x_p'$. Let $i \in \{1, \dots, p \}$ be such that ${x_1'}^{a_1'} = x_i^{a_i}$. Let $j \in \{1, \dots, p\}$. If $j < i$, then either $x_j > x_i$ or $x_j || x_i$. But, since $x_i^{a_i} = {x_1'}^{a_1'}$, there is no $x_j^{a_j} \in U$ such that $x_j > x_i=x_1'$, hence, if $j < i$, then $ x_j || x_i$. It follows that \begin{gather*} v= (x_1^{a_1}, \dots, x_{i-1}^{a_{i-1}}, x_i^{a_i}, x_{i+1}^{a_{i+1}}, \dots, x_p^{a_p}) \stackrel{II}{\to} (x_1^{a_1}, \dots, x_{i-2}^{a_{i-2}}, x_i^{a_i}, x_{i-1}^{a_{i-1}}, x_{i+1}^{a_{i+1}}, \dots, x_p^{a_p}) \stackrel{II}{\to} \cdots \stackrel{II}{\to}\\ (x_i^{a_i}, x_1^{a_1}, \dots, x_{i-1}^{a_{i-1}}, x_{i+1}^{a_{i+1}}, \dots, x_p^{a_p}) = ({x_1'}^{a_1'}, x_1^{a_1}, \dots, x_{i-1}^{a_{i-1}}, x_{i+1}^{a_{i+1}}, \dots, x_p^{a_p})\,. \end{gather*} By the induction hypothesis applied to $U \setminus \{ x_i^{a_i} \}$ we have \[ (x_1^{a_1}, \dots, x_{i-1}^{a_{i-1}}, x_{i+1}^{a_{i+1}}, \dots, x_p^{a_p}) \stackrel{II\ *}{\to} ({x_2'}^{a_2'}, \dots, {x_p'}^{a_p'})\,. \] So, \[ v = (x_1^{a_1}, \dots, x_p^{a_p}) \stackrel{II\ *}{\to} ({x_1'}^{a_1'}, \dots, {x_p'}^{a_p'}) = \hat \omega^S (U)\,. \proved \] \end{proof} \begin{lem}\label{lem5_2} Let $U$ be a non-empty stratum and let $x^a \in U$. Let $y^a = \gamma (x^a, U)$. Then $\hat \omega^S (U) \stackrel{II\ *}{\to} (y^a) \cdot \hat \omega^S (L (U, x^a))$. \end{lem} \begin{proof} We write $U = \{ x_1^{a_1}, \dots, x_p^{a_p} \}$ with $x_1 \succ x_2 \succ \cdots \succ x_p$. Let $i \in \{1, \dots, p \}$ be such that $x^a = x_i^{a_i}$. Note that, if $j < i$, then $x_j || x_i$ or $x_j > x_i$. This implies that, if $j < i$, then $x_j || (\varphi_{x_{j+1}}^{a_{j+1}} \circ \cdots \circ \varphi_{x_{i-1}}^{a_{i-1}}) (x_i)$ or $x_j > (\varphi_{x_{j+1}}^{a_{j+1}} \circ \cdots \circ \varphi_{x_{i-1}}^{a_{i-1}}) (x_i)$. Then we have the following sequence of rewriting transformations \begin{gather*} \hat \omega^S (U) = (x_1^{a_1}, \dots, x_{i-1}^{a_{i-1}}, x_i^{a_i}, x_{i+1}^{a_{i+1}}, \dots, x_p^{a_p}) \stackrel{II}{\to}\\ (x_1^{a_1}, \dots, x_{i-2}^{a_{i-2}}, \varphi_{x_{i-1}}^{a_{i-1}} (x_i)^{a_i}, x_{i-1}^{a_{i-1}}, x_{i+1}^{a_{i+1}}, \dots, x_p^{a_p}) \stackrel{II}{\to} \cdots \stackrel{II}{\to}\\ (x_1^{a_1}, (\varphi_{x_2}^{a_2} \circ \cdots \circ \varphi_{x_{i-1}}^{a_{i-1}}) (x_i)^{a_i}, x_2^{a_2}, \dots, x_{i-1}^{a_{i-1}}, x_{i+1}^{a_{i+1}}, \dots, x_p^{a_p}) \stackrel{II}{\to}\\ ( (\varphi_{x_1}^{a_1} \circ \cdots \circ \varphi_{x_{i-1}}^{a_{i-1}}) (x_i)^{a_i}, x_1^{a_1}, \dots, x_{i-1}^{a_{i-1}}, x_{i+1}^{a_{i+1}}, \dots, x_p^{a_p}) = (y^a) \cdot \hat \omega^S (L (U, x^a))\,. \end{gather*} \end{proof} \begin{lem}\label{lem5_3} Let $U \in \Omega$ be a stratum and let $y^b \in S (\Gamma)$ be a syllable which can be added to $U$. Then \[ \hat \omega^S (U) \cdot (y^b) \stackrel{M\ *}{\to} \hat \omega^S (R (U, y^b))\,. \] \end{lem} \begin{proof} We write $U = \{x_1^{a_1}, \dots, x_p^{a_p}\}$ with $x_1 \succ \cdots \succ x_p$. Let $i \in \{0, 1, \dots, p\}$ be such that $x_i \succeq y \succ x_{i+1}$. If $i=0$ this means that $y \not \in \supp (U)$ and $y \succ x_1$, and if $i=p$ this means that $x_p \succeq y$. If $y \in \supp (U)$, then $y=x_i$, otherwise $x_i \succ y \succ x_{i+1}$. Let $j \in \{i+1, \dots, p\}$. Then either $y > x_j$ or $y || x_j$. In both cases we have \[ ((x_j^{a_j}, y^b) , (y^b,\varphi_y^{-b} (x_j)^{a_j})) \in \RR_{II}\,. \] It follows that \[ \hat \omega^S (U) \cdot (y^a) = (x_1^{a_1}, \dots, x_p^{a_p}, y^a) \stackrel{II \ *}{\to} (x_1^{a_1}, \dots, x_i^{a_i}, y^b, \varphi_y^{-b} (x_{i+1})^{a_{i+1}}, \dots, \varphi_y^{-b} (x_p)^{a_p})\,. \] If $y \not \in \supp (U)$, then \[ R(U, y^b) = \{x_1^{a_1}, \dots, x_i^{a_i}, y^b, \varphi_y^{-b} (x_{i+1})^{a_{i+1}}, \dots, \varphi_y^{-b} (x_p)^{a_p}\}\,, \] hence, by Lemma \ref{lem5_1}, \[ (x_1^{a_1}, \dots, x_i^{a_i}, y^b, \varphi_y^{-b} (x_{i+1})^{a_{i+1}}, \dots, \varphi_y^{-b} (x_p)^{a_p}) \stackrel{II \ *}{\to} \hat \omega^S (R (U, y^b))\,. \] If $y = x_i \in \supp (U)$ and $b+a_i \neq 0$, then \[ R(U, y^b) = \{x_1^{a_1}, \dots, x_{i-1}^{a_{i-1}}, y^{a_i+b}, \varphi_y^{-b} (x_{i+1})^{a_{i+1}}, \dots, \varphi_y^{-b} (x_p)^{a_p}\}\,, \] hence, by Lemma \ref{lem5_1}, \begin{gather*} (x_1^{a_1}, \dots, x_{i-1}^{a_{i-1}}, x_i^{a_i}, y^b, \varphi_y^{-b} (x_{i+1})^{a_{i+1}}, \dots, \varphi_y^{-b} (x_p)^{a_p}) \stackrel{I }{\to}\\ (x_1^{a_1}, \dots, x_{i-1}^{a_{i-1}}, y^{a_i+b}, \varphi_y^{-b} (x_{i+1})^{a_{i+1}}, \dots, \varphi_y^{-b} (x_p)^{a_p}) \stackrel{II \ *}{\to} \hat \omega^S (R (U, y^b))\,. \end{gather*} If $y = x_i \in \supp (U)$ and $b+a_i = 0$, then \[ R(U, y^b) = \{x_1^{a_1}, \dots, x_{i-1}^{a_{i-1}}, \varphi_y^{-b} (x_{i+1})^{a_{i+1}}, \dots, \varphi_y^{-b} (x_p)^{a_p}\}\,, \] hence, by Lemma \ref{lem5_1}, \begin{gather*} (x_1^{a_1}, \dots, x_{i-1}^{a_{i-1}}, x_i^{a_i}, y^b, \varphi_y^{-b} (x_{i+1})^{a_{i+1}}, \dots, \varphi_y^{-b} (x_p)^{a_p}) \stackrel{I}{\to}\\ (x_1^{a_1}, \dots, x_{i-1}^{a_{i-1}}, \varphi_y^{-b} (x_{i+1})^{a_{i+1}}, \dots, \varphi_y^{-b} (x_p)^{a_p}) \stackrel{II \ *}{\to} \hat \omega^S (R (U, y^b))\,. \end{gather*} \end{proof} The following definition is a slight modification of the definition of normal form given in Subsection \ref{subsec2_2}. Let $g \in \Tr (\Gamma)$. By Theorems \ref{thm2_3} and \ref{thm2_4} there exists a unique piling $w = (U_1,U_2, \dots, U_p) \in \Omega^*$ such that $w$ is $\RR$-irreducible and $w = \Psi (g)$. Then we define the \emph{$S$-normal form} of $g$ to be \[ \nf^S (g) = \hat \omega^S (U_1) \cdot \hat \omega^S (U_2) \cdots \hat \omega^S (U_p) \in S(\Gamma)^*\,. \] Theorem \ref{thm2_8} will be a consequence of the following. \begin{prop}\label{prop5_4} Let $v \in S (\Gamma)^*$ and let $g = \bar v \in \Tr (\Gamma)$. Then $v \stackrel{M\ *}{\to} \nf^S (g)$. \end{prop} \begin{proof} For a piling $w = (U_1, \dots, U_p) \in \Omega^*$ we set \[ \hat \omega^S (w) = \hat \omega^S (U_1) \cdot \hat \omega^S (U_2) \cdots \hat \omega^S (U_p) \in S(\Gamma)^*\,. \] Let $w, w' \in \Omega^*$. It follows from Lemmas \ref{lem5_2} and \ref{lem5_3} that, if $w \stackrel{\RR\ *}{\to} w'$, then $\hat \omega^S (w) \stackrel{M\ *}{\to} \hat \omega^S (w')$. Let $v = (x_1^{a_1}, \dots, x_p^{a_p}) \in S (\Gamma)^*$ and let $g = \bar v \in \Tr (\Gamma)$. Set $w = ( \{ x_1^{a_1} \}, \{ x_2^{a_2} \}, \dots, \{x_p^{a_p} \}) \in \Omega^*$ and denote by $w'$ the unique piling in $\Omega^*$ such that $w'$ is $\RR$-irreducible and $\overline{w'} = g$. Then, by Theorems \ref{thm2_3} and \ref{thm2_4}, we have $w \stackrel{\RR\ *}{\to} w'$, hence, by the above, \[ v = \hat \omega^S (w) \stackrel{M \ *}{\to} \hat \omega^S (w') = \nf^S (g)\,. \proved \] \end{proof} \begin{proof}[Proof of Theorem \ref{thm2_8}] To prove Theorem \ref{thm2_8} it suffices to show that, if $w, w' \in S (\Gamma)^*$ are syllabically $M$-reduced and $\overline{w} = \overline{w'}$, then $w \stackrel{II\ *}{\to} w'$. Let $w, w' \in S (\Gamma)^*$ be two syllabically $M$-reduced words and let $g \in \Tr (\Gamma)$ be such that $\overline{w} = \overline{w'} = g$. By Proposition \ref{prop5_4} we have $w \stackrel{M\ *}{\to} \nf^S (g)$ and $w' \stackrel{M\ *}{\to} \nf^S (g)$. Since $w$ and $w'$ are both syllabically $M$-reduced, we actually must have $w \stackrel{II\ *}{\to} \nf^S (g)$ and $w' \stackrel {II\ *}{\to} \nf^S (g)$. Finally, since the operation $\stackrel{II}{\to}$ is reversible, we conclude $w \stackrel{II\ *}{\to} \nf^S (g) \stackrel{II\ *}{\to} w'$. \end{proof} \section{Parabolic subgroups}\label{sec6} \begin{proof}[Proof of Theorem \ref{thm2_10}] Note that $\Omega (\Gamma_1) \subseteq \Omega (\Gamma)$, hence $\Omega (\Gamma_1)^* \subseteq \Omega (\Gamma)^*$. It is easily seen that, if $U \in \Omega (\Gamma_1)$ and $x^a \in U$, then $\gamma (U, x^a) \in S (\Gamma_1)$. It is also easily seen that, if $U \in \Omega (\Gamma_1)$ and $x^a \in S (\Gamma_1)$ can be added to $U$, then $R (U, x^a) \in \Omega (\Gamma_1)$. Let $w \in \Omega (\Gamma_1)^*$ and $w' \in \Omega (\Gamma)^*$. Then from the above observations it follows that, if $w \stackrel{\RR (\Gamma)\ *}{\longrightarrow} w'$, then $w' \in \Omega (\Gamma_1)^*$ and $w \stackrel{\RR (\Gamma_1)\ *}{\longrightarrow} w'$. Let $g \in \Tr (\Gamma_1)$. Write $g = x_1^{a_1} \dots x_p^{a_p}$ with $x_1^{a_1}, \dots, x_p^{a_p} \in S (\Gamma_1)$, and set \[ v = (\{x_1^{a_1}\}, \{x_2^{a_2}\}, \dots, \{x_p^{a_p}\}) \in \Omega (\Gamma_1)^*\,. \] Let $w \in \Omega (\Gamma)^*$ be the unique $\RR (\Gamma)$-irreducible piling such that $v \stackrel{\RR (\Gamma)\ *}{\longrightarrow} w$. From the above we get $w \in \Omega (\Gamma_1)^*$ and $v \stackrel{\RR (\Gamma_1)\ *}{\longrightarrow} w$. Moreover, $w$ is $\RR (\Gamma_1)$-irreducible because it is $\RR (\Gamma)$-irreducible. Write $w = (U_1, \dots, U_q)$ with $U_1, \dots, U_q \in \Omega (\Gamma_1)$. Then \[ \nf_\Gamma (\iota_1 (g)) =\nf_{\Gamma_1} (g) = \hat \omega (U_1) \cdot \hat \omega (U_2) \cdots \hat \omega (U_q)\,. \] Let $g \in \Ker (\iota_1)$. From the above we get $\nf_{\Gamma_1} (g) = \nf_\Gamma (\iota_1 (g)) = \epsilon$, hence $g=1$. This shows that $\iota_1$ is injective. Let $g \in \Tr (\Gamma)$. We already know that, if $g \in \Tr (\Gamma_1)$, then $\nf_\Gamma (g) = \nf_{\Gamma_1} (g)$, hence $\nf_\Gamma (g) \in (V (\Gamma_1) \sqcup V (\Gamma_1)^{-1})^*$. Conversely, it is obvious that, if $\nf_\Gamma (g) \in (V (\Gamma_1) \sqcup V (\Gamma_1)^{-1})^*$, then $g = \overline{\nf_\Gamma (g)} \in \Tr (\Gamma_1)$. \end{proof} \section{PreGarside trickle groups}\label{sec7} In this section $\Gamma = (\Gamma, \le, \mu, (\varphi_x)_{x \in V (\Gamma)})$ denotes a fixed preGarside trickle graph. We also fix a total order $\preceq$ on $V (\Gamma)$ which extends the partial order $\le$ in the sense that, for all $x, y \in V (\Gamma)$, if $x \le y$, then $x \preceq y$. The proof of Theorem \ref{thm2_14} is based on the existence of a complemented presentation for $\Tr^+ (\Gamma)$ that satisfies the sharp $\theta$-cube condition. So, we start by recalling these notions and explaining their link with Theorem \ref{thm2_14}. \begin{defin} Let $\Lambda$ be a simplicial graph. As always we denote by $V (\Lambda)$ its vertex set and by $E (\Lambda)$ its edge set. Let $\hat E (\Lambda) = \{ (x,y) \in V (\Lambda) \times V (\Lambda) \mid \{ x, y \} \in E (\Lambda) \}$. A \emph{partial complement} on $V (\Lambda)$ based on $\Lambda$ is a map $f : \hat E (\Lambda) \to V (\Lambda)^*$. A presentation $M = \langle V \mid R \rangle^+$ for a monoid $M$ is called \emph{complemented} if there exist two partial complements $f$ on a graph $\Lambda$ and $g$ on a graph $\Lambda'$ such that $V (\Lambda) = V (\Lambda') = V$ and \[ R = \{ x\, f(x,y) = y \, f(y,x) \mid \{x,y\} \in E(\Lambda) \} = \{ g(y,x)\,x = g(x,y)\,y \mid \{x,y\} \in E(\Lambda') \}\,. \] Such a presentation is called \emph{short} if $f(x,y) \in V$ for all $(x,y) \in \hat E (\Lambda)$, that is, if all relations in $R$ are of the form $xy' = yx'$ with $x,y,x',y' \in V$. \end{defin} \begin{rem} The definition of short presentation given here is more restrictive than the one given in \cite{DDGKM1}, but it allows to consider only homogeneous monoid presentations. In particular, this implies that the considered monoids are all atomic (see the remark below). \end{rem} \begin{expl} Let $\Gamma = (\Gamma, \le, \mu, (\varphi_x)_{x \in V (\Gamma)})$ be our preGarside trickle graph. For all $(x, y) \in \hat E (\Gamma)$ we set $f (x, y) = \varphi_x^{-1} (y)$ and $g (x, y) = \varphi_y (x)$. Then, by definition and (the proof of) Proposition \ref{prop2_2}, $\Tr^+ (\Gamma)$ has the presentation $\Tr^+ (\Gamma) = \langle V \mid R \rangle$, where $V = V(\Gamma)$ and \[ R = \{ x\, f (x, y) = y \, f (y, x) \mid \{ x, y \} \in E (\Gamma) \} = \{ g(y, x) \, x = g (x, y) \, y \mid \{ x, y \} \in E (\Gamma) \} \,. \] In particular, this presentation is a complemented short presentation. \end{expl} \begin{rem} Let $M$ be a monoid with a complemented short presentation $M = \langle V \mid R \rangle$. Define a map $\nu : M \to \N$ as follows. If $a \in M$ is written $a=x_1 \dots x_p$ with $x_1, \dots, x_p \in V$, then set $\nu (a) =p$. The map $\nu$ is well-defined because the relations in $R$ are homogeneous, hence the length $p$ does not depend on the choice of the expression of $a$. It is clear that $\nu (a) = 0$ if and only if $a = 1$ and $\nu (ab) = \nu (a) + \nu (b)$ for all $a, b \in M$, hence $\nu$ is a norm and $M$ is atomic. In particular, $\Tr^+ (\Gamma)$ is atomic. \end{rem} The sharp $\theta$-cube condition introduced in \cite[Definition II.4.14]{DDGKM1} is technical, but in our case where the presentations are short, its definition is simpler as, in particular, there is no need of extending the partial map $\star$ to $V^*$ as in the general case. \begin{defin} Let $\langle V \mid R \rangle^+$ be a complemented short presentation of a monoid $M$. Recall that there exist a graph $\Lambda$ with $V = V (\Lambda)$ and a partial complement $g : \hat E (\Lambda) \to V$ such that \[ R = \{ g(y,x)\, x = g(x, y)\, y \mid \{ x, y \} \in E (\Lambda) \}\,. \] We define a partial map $(x ,y) \mapsto x \star y$ from $(V \cup\{ \epsilon\}) \times (V \cup\{ \epsilon\})$ to $V \cup \{ \epsilon \}$ as follows. Let $x, y \in V$ be such that $x \neq y$. If $\{x, y\} \in E (\Lambda)$, then we set $x \star y = g (x, y)$. If $\{x, y\} \not \in E (\Lambda)$, then neither $x \star y$ nor $y \star x$ is defined. On the other hand, for $x \in V \cup \{ \epsilon \}$ we set $x \star x = \epsilon \star x = \epsilon$ and $x \star \epsilon = x$. We say that the presentation $\langle V \mid R \rangle^+$ satisfies the \emph{left sharp $\theta$-cube condition} if, for all $x, y, z \in V$ pairwise distinct, we have the following alternative: \begin{itemize} \item either $(z \star x) \star (y \star x)$ and $(z \star y) \star (x \star y)$ are both defined and are equal, \item or neither expression is defined. \end{itemize} We define similarly the \emph{right sharp $\theta$-cube condition}. We say that the presentation satisfies the \emph{sharp $\theta$-cube condition} if it satisfies both, the left and the right sharp $\theta$-cube conditions. \end{defin} The main tool in the proof of Theorem \ref{thm2_14} comes from \cite[Proposition II.4.16]{DDGKM1}. \begin{prop}[Dehornoy--Digne--Godelle--Krammer--Michel \cite{DDGKM1}]\label{prop7_1} If a monoid $M$ admits a short complemented presentation that satisfies the sharp $\theta$-cube condition, then $M$ is a preGarside monoid. \end{prop} \begin{proof}[Proof of Theorem \ref{thm2_14}] We already know that $\Tr^+ (\Gamma)$ has a short complemented presentation. So, in order to prove Theorem \ref{thm2_14}, it suffices to show that the standard presentation of $\Tr^+ (\Gamma)$ satisfies the sharp $\theta$-cube condition. By symmetry and thanks to Proposition \ref{prop2_2}, it suffices to show that this presentation satisfies the left sharp $\theta$-cube condition. Let $x, y \in V (\Gamma)$ be such that $x \neq y$. If $\{x, y\} \in E (\Gamma)$, then we set $x \star y = \varphi_{y} (x)$, and, if $\{x, y\} \not \in E(\Gamma)$, then $x \star y$ is not defined. On the other hand, for $x \in V (\Gamma) \cup \{\epsilon\}$ we set $\epsilon \star x = x \star x = \epsilon$ and $x \star \epsilon = x$. The fact that the standard presentation of $\Tr^+ (\Gamma)$ satisfies the left sharp $\theta$-cube condition is a straightforward consequence of the following two claims. {\it Claim 1.} For all $x, y, z \in V$ pairwise distinct we have the following alternative: \begin{itemize} \item either $( z \star x) \star ( y \star x)$ and $(z \star y) \star (x \star y)$ are both defined, \item or neither expression is defined. \end{itemize} Furthermore, $(z \star x) \star (y \star x)$ and $(z \star y) \star (x \star y)$ are both defined if and only if $\{x, y\}$, $\{x, z\}$, and $\{y, z\}$ belong to $E (\Gamma)$. {\it Proof of Claim 1.} By symmetry between $x$ and $y$, it suffices to show that $(z \star x) \star (y \star x)$ is defined if and only if $\{x, y\}$, $\{x ,z\}$ and $\{y, z\}$ belong to $E (\Gamma)$. Note that, if $y \star x$ is defined, then $y \star x = \varphi_x (y)$ lies in $V (\Gamma)$ (and it is different from $\epsilon$), and, if $y \star x = \varphi_x (y)$ and $z \star x = \varphi_x (z)$ are both defined, then they are different. So, saying that $(z \star x) \star (y \star x)$ is defined is equivalent of saying that $\{x, y\}$, $\{x, z\}$ and $\{(y \star x), (z \star x) \} = \{\varphi_x (y), \varphi_x (z) \}$ belong to $E (\Gamma)$. But $\{\varphi_x (y), \varphi_x (z)\}$ belongs to $E (\Gamma)$ if and only if $\{y, z\}$ belongs to $E (\Gamma)$, hence $(z \star x) \star (y \star x)$ is defined if and only if $\{x, y\}$, $\{x, z\}$ and $\{ y, z \}$ belong to $E (\Gamma)$. This completes the proof of Claim 1. {\it Claim 2.} Let $x, y, z \in V(\Gamma)$ pairwise distinct be such that $\{x, y\}, \{x, z\}, \{y, z\} \in E(\Gamma)$. Then \[ (z \star x) \star (y \star x) = (z \star y) \star (x \star y)\,. \] {\it Proof of Claim 2.} The proof of Claim 2 is divided into two cases depending on whether $x || y$ or $y < x$. By symmetry, the case $x < y$ is treated in the same way as the case $y < x$. {\it Case 1:} $x || y$. By Condition (b) in the definition of a trickle graph, we cannot have together $z < x$ and $z < y$. Thus, we have two subcases: $z<x$ and $z \not < y$ for the first, and $z \not < x$ and $z \not < y$ for the second. The subcase $z \not < x$ and $z < y$ is treated in the same way as the subcase $z<x$ and $z \not < y$. Suppose $z < x$ and $z \not < y$. Then $\varphi_x (z) \not < \varphi_x (y) = y$, hence \[ (z \star x) \star (y \star x) = \varphi_x (z) \star y = \varphi_x (z) = z \star x = (z \star y) \star (x \star y)\,. \] Suppose $z \not < x$ and $z \not < y$. Then \[ (z \star x) \star (y \star x) = z \star y = z = z \star x = (z \star y) \star (x \star y)\,. \] {\it Case 2:} $y < x$. Here we have three subcases: $z \not < x$ (which implies $z \not < y$) for the first, $z < x$ and $z \not < y$ for the second, and $z < y$ (which implies $z < y < x$) for the third. Suppose $z \not < x$. Then $z = \varphi_x (z) \not < \varphi_x (y)$, hence \[ (z \star x) \star (y \star x) = z \star \varphi_x (y) = z = z \star x = (z \star y) \star (x \star y)\,. \] Suppose $z < x$ and $z \not < y$. Then $\varphi_x (z) \not < \varphi_x (y)$, hence \[ (z \star x) \star (y \star x) = \varphi_x (z) \star \varphi_x (y) = \varphi_x (z) = z \star x = (z \star y) \star (x \star y)\,. \] Suppose $z < y$. Then, by Condition (g) in the definition of a trickle graph, \[ (z \star x) \star (y \star x) = \varphi_x (z) \star \varphi_x (y) = (\varphi_{\varphi_x (y)} \circ \varphi_x) (z) = (\varphi_x \circ \varphi_y) (z) = \varphi_y (z) \star x = (z \star y) \star (x \star y)\,. \] This completes the proof of Claim 2 and therefore of Theorem \ref{thm2_14}. \end{proof} For the remaining proofs in this section we need to understand how the rewriting system $\RR$ of Section \ref{sec4} can be restricted to positive words. Then Theorem \ref{thm2_17} will be a straightforward consequence of this analysis, and therefore it will be proved before Theorem \ref{thm2_15} and Theorem \ref{thm2_16}. Set $S^+ (\Gamma) = \{ x^a \mid x \in V (\Gamma) \text{ and } a \in \N_{\ge 1} \}$. A \emph{positive stratum} of $\Gamma$ is a finite subset $U = \{x_1^{a_1}, x_2^{a_2}, \dots, x_p^{a_p}\} \subseteq S^+ (\Gamma)$ such that $x_i \neq x_j$ and $\{x_i, x_j\} \in E (\Gamma)$ for all $i, j \in \{1, \dots, p\}$, $i \neq j$. The set of positive strata is denoted by $\Omega^+ = \Omega^+ (\Gamma)$. Note that $\Omega^+ \subset \Omega$, hence $(\Omega^+)^* \subset \Omega^*$. It is easily seen that, if $U \in \Omega^+$ and $x^a \in U$, then $\gamma (U, x^a) \in S^+ (\Gamma)$. It is also easily seen that, if $U \in \Omega^+$ and $x^a \in S^+ (\Gamma)$ can be added to $U$, then $R (U, x^a) \in \Omega^+$. Let $\RR^+ = \RR \cap ((\Omega^+)^* \times (\Omega^+)^*)$. Then the following is a direct consequence of the above observations combined with Theorem \ref{thm2_4}. \begin{lem}\label{lem7_2} \begin{itemize} \item[(1)] $\RR^+$ is a terminating and confluent rewriting system. \item[(2)] Let $w \in (\Omega^+)^*$ and $w' \in \Omega^*$. If $w \stackrel{\RR\ *}{\longrightarrow} w'$, then $w' \in (\Omega^+)^*$ and $w \stackrel{\RR^+\ *}{\longrightarrow} w'$. \end{itemize} \end{lem} Set $M^+ (\Gamma) = \langle \Omega^+ \mid u = v \text{ for } (u, v) \in \RR^+ \rangle^+$. Let $U\in \Omega^+$, $U \neq \emptyset$. Write $U = \{ x_1^{a_1}, \dots, x_p^{a_p} \}$ with $x_1 \succ x_2 \succ \cdots \succ x_p$, and set $\omega^+ (U) = x_1^{a_1} \dots x_p^{a_p} \in \Tr^+ (\Gamma)$. If $U = \emptyset$, then set $\omega^+ (U) = 1$. The same proofs as the ones of Lemmas \ref{lem4_14} and \ref{lem4_15} applied to $M^+ (\Gamma)$ and $\Tr^+ (\Gamma)$ prove the following. \begin{lem}\label{lem7_3} We have an isomorphism $\Phi^+ : M^+ (\Gamma) \to \Tr^+ (\Gamma)$ which sends $U$ to $\omega^+ (U)$ for all $U \in \Omega^+$. \end{lem} If $U$ is a non-empty positive stratum that we write $U = \{ x_1^{a_1}, x_2^{a_2}, \dots, x_p^{a_p} \}$ with $x_1 \succ x_2 \succ \cdots \succ x_p$, then we set $\hat \omega^+ (U) = x_1^{a_1} \cdot x_2^{a_2} \cdots x_p^{a_p} \in V (\Gamma)^*$. Note that $\hat \omega^+ (U)$ is a representative of $\omega^+(U)$. Let $g \in \Tr^+ (\Gamma)$. By the above there exists a unique piling $w = (U_1, U_2, \dots, U_p) \in (\Omega^+)^*$ such that $w$ is $\RR^+$-irreducible and $w$ represents $g$. Then we set \[ \nf^+ (g) = \hat \omega^+ (U_1) \cdot \hat \omega^+ (U_2) \cdots \hat \omega^+ (U_p) \in V (\Gamma)^*\,. \] The following is a direct consequence of the above observations. \begin{lem}\label{lem7_4} \begin{itemize} \item[(1)] Let $g \in \Tr^+ (\Gamma)$. Then $\nf^+ (g) = \nf (\iota_{\Tr^+ (\Gamma)} (g))$. \item[(2)] Let $g \in \Tr (\Gamma)$. We have $g \in \iota_{\Tr^+ (\Gamma)} (\Tr^+ (\Gamma))$ if and only if $\nf (g) \in V(\Gamma)^*$. \end{itemize} \end{lem} \begin{proof}[Proof of Theorem \ref{thm2_17}] Let $g, h \in \Tr^+ (\Gamma)$. If $\iota_{\Tr^+ (\Gamma)} (g) = \iota_{\Tr^+ (\Gamma)} (h)$, then we have $\nf ( \iota_{\Tr^+ (\Gamma)} (g)) = \nf ( \iota_{\Tr^+ (\Gamma)} (h))$, hence, by Lemma \ref{lem7_4}, $\nf^+ (g) = \nf^+ (h)$, and therefore $g=h$. \end{proof} Now, we turn to the proof of Theorem \ref{thm2_15}. The following two lemmas are preliminaries of it. \begin{lem}\label{lem7_5} Let $g \in \Tr^+ (\Gamma)$. Let $w = (U_1, \dots, U_p)$ be the unique piling in $(\Omega^+)^*$ such that $\Phi^+ (\bar w) = g$ and $w$ is $\RR^+$-irreducible. Write $U_1=\{ x_1^{a_1}, \dots, x_q^{a_q} \}$ with $x_1 \succ x_2 \succ \cdots \succ x_q$. Let $\psi = \varphi_{x_1}^{a_1} \circ \varphi_{x_2}^{a_2} \circ \cdots \circ \varphi_{x_q}^{a_q}$. Then $\Div_L (g) \cap V (\Gamma) = \psi (\{x_1, x_2, \dots, x_q\})$. \end{lem} \begin{proof} Let $y \in \psi (\{x_1, x_2, \dots, x_q\})$. There exists $i \in \{1, \dots, q\}$ such that $y = \psi (x_i)$. Since $x_j \not > x_i$ for $j \ge i$, we have $y = \psi(x_i) =(\varphi_{x_1}^{a_1} \circ \cdots \circ \varphi_{x_{i-1}}^{a_{i-1}}) (x_i)$. Then \begin{gather*} g = x_1^{a_1} \dots x_{i-1}^{a_{i-1}} x_i^{a_i} x_{i+1}^{a_{i+1}} \dots x_q^{a_q} \, \omega (U_2) \dots \omega (U_p) =\\ x_1^{a_1} \dots x_{i-2}^{a_{i-2}} \varphi_{x_{i-1}}^{a_{i-1}} (x_i)^{a_i} x_{i-1}^{a_{i-1}} x_{i+1}^{a_{i+1}} \dots x_q^{a_q} \, \omega (U_2) \dots \omega (U_p) = \cdots =\\ (\varphi_{x_1}^{a_1} \circ \cdots \circ \varphi_{x_{i-1}}^{a_{i-1}}) (x_i)^{a_i}\, x_1^{a_1} \dots x_{i-1}^{a_{i-1}} x_{i+1}^{a_{i+1}} \dots x_q^{a_q} \, \omega (U_2) \dots \omega (U_p) =\\ y\,y^{a_i-1} x_1^{a_1} \dots x_{i-1}^{a_{i-1}} x_{i+1}^{a_{i+1}} \dots x_q^{a_q} \, \omega (U_2) \dots \omega (U_p)\,, \end{gather*} hence $y \in \Div_L (g) \cap V (\Gamma)$. Let $y \in \Div_L (g) \cap V (\Gamma)$. Let $g' \in \Tr^+ (\Gamma)$ be such that $g = yg'$. Let $w'$ be the unique piling in $(\Omega^+)^*$ such that $\Phi^+ (\overline{w'}) = g'$ and $w'$ is $\RR^+$-irreducible. Let $u = \{y\} \cdot w' \in (\Omega^+)^*$. We have $\Phi^+ (\bar u) = g$, hence $u \stackrel{\RR^+\ *}{\longrightarrow} w$. In other words, there exists a finite sequence $u_0=u, u_1, \dots, u_\ell=w$ in $(\Omega^+)^*$ such that $u_{j-1} \stackrel{\RR^+}{\to} u_j$ for all $j \in \{1, \dots, \ell \}$. For each $j \in \{0,1, \dots, \ell \}$ we denote by $V_j$ the first stratum in the piling $u_j$ and we set $V_j = \{z_{1,j}^{c_{1,j}}, \dots, z_{r_j,j}^{c_{r_j,j}} \}$ with $z_{1,j} \succ \cdots \succ z_{r_j,j}$. {\it Claim.} Let $j \in \{ 0, 1, \dots, \ell\}$. Then there exists $i \in \{1, \dots, r_j \}$ such that $y = (\varphi_{z_{1,j}}^{c_{1,j}} \circ \cdots \circ \varphi_{z_{i-1,j}}^{c_{i-1,j}}) (z_{i,j})$. {\it Proof of the claim.} We argue by induction on $j$. Suppose $j = 0$. Then $V_0 = \{y\}$ and the claim is trivial. Suppose $j > 0$ and that the induction hypothesis holds. By the induction hypothesis there exists $i \in \{1, \dots, r_{j-1} \}$ such that $y = (\varphi_{z_{1,j-1}}^{c_{1,j-1}} \circ \cdots \circ \varphi_{z_{i-1,j-1}}^{c_{i-1,j-1}}) (z_{i,j-1})$. If $V_j = V_{j-1}$, then there is nothing to prove. So, we can assume that $V_j \neq V_{j-1}$. Then there exists a syllable $z^c$ that can be added to $V_{j-1}$ such that $V_j = R(V_{j-1},z^c)$. Let $k \in \{0, \dots, r_{j-1} \}$ be such that $z_{k,j-1} \succeq z \succ z_{k+1,j-1}$. If $k=0$, then this means that $z \succ z_{1,j-1}$. If $k=r_{j-1}$, then this means that $z_{r_{j-1},j-1} \succeq z$. Also, note that, if $z = z_{k,j-1}$, then $c + c_{k,j-1}>0$, hence $c + c_{k,j-1} \neq 0$. If $k \ge i$, then \[ (\varphi_{z_{1,j}}^{c_{1,j}} \circ \cdots \circ \varphi_{z_{i-1,j}}^{c_{i-1,j}}) (z_{i,j}) = (\varphi_{z_{1,j-1}}^{c_{1,j-1}} \circ \cdots \circ \varphi_{z_{i-1,j-1}}^{c_{i-1,j-1}}) (z_{i,j-1}) = y\,. \] If $k < i$ and $z \neq z_{k,j-1}$, then, by applying Lemma \ref{lem4_7} several times, \begin{gather*} (\varphi_{z_{1,j}}^{c_{1,j}} \circ \cdots \circ \varphi_{z_{i,j}}^{c_{i,j}}) (z_{i+1,j}) =\\ (\varphi_{z_{1,j-1}}^{c_{1,j-1}} \circ \cdots \circ \varphi_{z_{k,j-1}}^{c_{k,j-1}} \circ \varphi_z^c \circ \varphi_{\varphi_z^{-c} (z_{k+1,j-1})}^{c_{k+1,j-1}} \circ \cdots \circ \varphi_{\varphi_z^{-c} (z_{i-1,j-1})}^{c_{i-1,j-1}}) (\varphi_z^{-c} (z_{i,j-1})) =\\ (\varphi_{z_{1,j-1}}^{c_{1,j-1}} \circ \cdots \circ \varphi_{z_{k,j-1}}^{c_{k,j-1}} \circ \varphi_z^c \circ \varphi_z^{-c} \circ \varphi_{z_{k+1,j-1}}^{c_{k+1,j-1}} \circ \cdots \circ \varphi_{z_{i-1,j-1}}^{c_{i-1,j-1}}) (z_{i,j-1}) =\\ (\varphi_{z_{1,j-1}}^{c_{1,j-1}} \circ \cdots \circ \varphi_{z_{i-1,j-1}}^{c_{i-1,j-1}}) (z_{i,j-1}) = y\,. \end{gather*} If $k < i$ and $z= z_{k,j-1}$, then we prove in the same way that \[ (\varphi_{z_{1,j}}^{c_{1,j}} \circ \cdots \circ \varphi_{z_{i-1,j}}^{c_{i-1,j}}) (z_{i,j}) = y\,. \] This completes the proof of the claim. Applying the claim to $j=\ell$ we see that there exists $i \in \{1, \dots, q \}$ such that $y = (\varphi_{x_1}^{a_1} \circ \cdots \circ \varphi_{x_{i-1}}^{a_{i-1}}) (x_i) = \psi (x_i)$. \end{proof} An element $g \in \Tr^+ (\Gamma)$ is called \emph{square-free} if it can be written in the form $g = x_1 x_2 \dots x_p$ with $x_1 \succ x_2 \succ \cdots \succ x_p$. We denote by $\SF (\Gamma)$ the set of square-free elements. \begin{lem}\label{lem7_6} Suppose $V (\Gamma)$ is finite and $\Gamma$ is complete. Write $V (\Gamma) = \{x_1, \dots, x_n\}$ with $x_1 \succ x_2 \succ \cdots \succ x_n$, and set $\Delta = x_1 x_2 \dots x_n$. Then $\Delta = \vee_L V (\Gamma) = \vee_R V (\Gamma)$ and $\Div_L (\Delta) = \Div_R (\Delta) = \SF (\Gamma)$. In particular, $\Delta$ is balanced and $\Div (\Delta)$ generates $\Tr^+ (\Gamma)$, that is, $\Delta$ is a Garside element. \end{lem} \begin{proof} Let $\nu : \Tr^+ (\Gamma) \to \N$ be the length homomorphism which sends $x_i$ to $1$ for all $i \in \{1, \dots, n\}$. For $p \in \{0,1, \dots, n\}$ we denote by $\SF_p (\Gamma)$ the set of square-free elements of length $p$. Let $g \in \SF_p (\Gamma)$. We write $g = x_{i_1} x_{i_2} \dots x_{i_p}$ with $i_1 < i_2 < \cdots < i_p$, we set $\psi_g = \varphi_{x_{i_1}} \circ \cdots \circ \varphi_{x_{i_p}}$ and $X_g = \psi_g (\{x_{i_1}, \dots, x_{i_p} \})$. We know by Lemma \ref{lem7_5} that $\Div_L (g) \cap V (\Gamma) = X_g$. Furthermore, again by Lemma \ref{lem7_5}, if $h \in \Tr^+ (\Gamma)$ satisfies $\Div_L (h) \cap V (\Gamma) \supseteq X_g$, then $\nu (h) \ge p$. This implies that $\vee_L X_g$ exists and $\vee_L X_g = g$. Denote by $\PP_p (V (\Gamma))$ the set of subsets of $V (\Gamma)$ with cardinality $p$. We have an injective map $\SF_p (\Gamma) \to \PP_p (V (\Gamma))$, $g \mapsto X_g$, and $|\SF_p (\Gamma)| = |\PP_p (V (\Gamma))| = \binom{n}{p}$, hence this map is a bijection. In particular, since $\SF_n (\Gamma) = \{\Delta\}$ and $\PP_n (V (\Gamma)) = \{ V(\Gamma)\}$, we have $\vee_L V(\Gamma) = \Delta$. Let $g \in \Tr^+ (\Gamma)$. If $g \le_L \Delta$, then $g$ is square-free, otherwise by applying the trickle algorithm we see that $\Delta$ would not be square-free. Suppose that $g$ is square-free. Then, since $X_g \subseteq V (\Gamma)$, we have $g = \vee_L X_g \le_L \vee_L V (\Gamma) = \Delta$. This shows that $\Div_L (\Delta) = \SF (\Gamma)$. For $p \in \{0, 1, \dots, n\}$ we denote by $\widetilde{\SF}_p (\Gamma)$ the set of elements of $\Tr^+ (\Gamma)$ that can be written in the form $x_{i_p} \dots x_{i_2} x_{i_1}$ with $i_1 < i_2 < \cdots < i_p$. We show by induction on $p$ that $\widetilde{\SF}_p (\Gamma) = \SF_p (\Gamma)$. The cases $p=0$ and $p=1$ are obvious, hence we can assume that $p \ge 2$ and that the induction hypothesis holds. Let $g \in \SF_p (\Gamma)$. Write $g$ in the form $g = x_{i_1} x_{i_2} \dots x_{i_p}$ with $i_1 < i_2 < \cdots < i_p$. Since, for all $j \in \{2, \dots, p\}$, we have $x_{i_1} > x_{i_j}$ or $x_{i_1} || x_{i_j}$, \[ g= \varphi_{x_{i_1}}(x_{i_2}) \dots \varphi_{x_{i_1}}(x_{i_p}) \, x_{i_1}\,. \] By Lemma \ref{lem4_13}, $\varphi_{x_{i_1}}(x_{i_2}) \dots \varphi_{x_{i_1}}(x_{i_p}) \in \SF_{p-1} (\Gamma)$. Let $V_1 = \{ x \in V (\Gamma) \mid x \not \ge x_{i_1} \}$ and let $\Gamma_1$ be the full subgraph of $\Gamma$ spanned by $V_1$. Observe that $\Gamma_1$ is a parabolic subgraph of $\Gamma$ and $\varphi_{x_{i_1}}(x_{i_2}) \dots \varphi_{x_{i_1}}(x_{i_p}) \in \SF_{p-1} (\Gamma_1)$. By the induction hypothesis, $\SF_{p-1} (\Gamma_1) = \widetilde{\SF}_{p-1} (\Gamma_1)$, hence $\varphi_{x_{i_1}}(x_{i_2}) \dots \varphi_{x_{i_1}}(x_{i_p})$ can be written in the form $x_{k_p} \dots x_{k_3} x_{k_2}$ with $k_2 < k_3 < \cdots < k_p$ and $x_{k_j} \in V_1$ for all $j \in \{2, \dots, p\}$. Note that, for $j \in \{2, \dots, p\}$, as $x_{k_j} \in V_1$, either $x_{i_1} > x_{k_j}$ or $x_{i_1} || x_{k_j}$. Applying Lemma \ref{lem4_13} to the dual presentation of $\Tr^+ (\Gamma)$ we deduce that $g = x_{k_p} \dots x_{k_2} x_{i_1} \in \widetilde{\SF}_p (\Gamma)$. This shows that $\SF_p (\Gamma) \subseteq \widetilde{\SF}_p (\Gamma)$. We show in the same way that $\widetilde{\SF}_p (\Gamma) \subseteq \SF_p (\Gamma)$, hence $\widetilde{\SF}_p (\Gamma) = \SF_p (\Gamma)$. Let $\widetilde{\SF} (\Gamma) = \bigcup_{p=0}^n \widetilde{\SF}_p (\Gamma)$. From the equality $\SF_n (\Gamma) = \widetilde{\SF}_n (\Gamma)$ it follows that $\Delta = x_n \dots x_2 x_1$. Moreover, by applying the previous reasoning to the dual presentation of $\Tr^+ (\Gamma)$ we get $\Div_R (\Delta) = \widetilde{\SF} (\Gamma) = \SF (\Gamma)$. \end{proof} \begin{proof}[Proof of Theorem \ref{thm2_15}] Suppose $\Tr^+ (\Gamma)$ is a Garside monoid. Let $\Delta$ be a Garside element of $\Tr^+ (\Gamma)$. By definition, $\Div (\Delta) = \Div_L (\Delta)$ generates $\Tr^+ (\Gamma)$, hence $V (\Gamma) \subseteq \Div_L (\Delta)$, that is, $\Div_L (\Delta) \cap V (\Gamma) = V (\Gamma)$. Now, Lemma \ref{lem7_5} implies that $\Div_L (\Delta) \cap V(\Gamma) = V(\Gamma)$ is a finite stratum, hence $V (\Gamma)$ is finite and $\Gamma$ is complete. Suppose $V (\Gamma)$ is finite and $\Gamma$ is complete. We write $V (\Gamma) = \{ x_1, \dots, x_n \}$ with $x_1 \succ x_2 \succ \cdots \succ x_n$, and we set $\Delta = x_1 x_2 \dots x_n$. Then, by Lemma \ref{lem7_6}, $\Delta$ is a Garside element, hence $\Tr^+ (\Gamma)$ is a Garside monoid. \end{proof} \begin{proof}[Proof of Theorem \ref{thm2_16}] We argue by induction on $|V (\Gamma)|$. If $|V (\Gamma)| = 1$, then $\Tr (\Gamma) \simeq \Z$, hence $\Tr (\Gamma)$ is torsion-free. So, we can assume that $|V (\Gamma)| \ge 2$ and that the induction hypothesis holds. Let $Y$ be the set of maximal elements of $V (\Gamma)$ for the (partial) order $\le$. First, suppose that there exist $y_1, y_2 \in Y$ such that $y_1 \neq y_2$ and $\{ y_1, y_2 \} \not \in E (\Gamma)$. Let $X_1 = V (\Gamma) \setminus \{ y_1 \}$, $X_2 = V (\Gamma) \setminus \{ y_2 \}$ and $X_{12} = V (\Gamma) \setminus \{ y_1, y_2 \}$. We denote by $\Gamma_1$ the full subgraph of $\Gamma$ spanned by $X_1$, by $\Gamma_2$ the full subgraph of $\Gamma$ spanned by $X_2$, and by $\Gamma_{12}$ the full subgraph of $\Gamma$ spanned by $X_{12}$. It is easily seen that $\Gamma_1$, $\Gamma_2$, and $\Gamma_{12}$ are parabolic subgraphs of $\Gamma$. Moreover, by Theorem \ref{thm2_10}, the embedding of $\Gamma_{12}$ into $\Gamma_k$ induces an injective homomorphism $\Tr (\Gamma_{12}) \hookrightarrow \Tr (\Gamma_k)$ and, by the induction hypothesis, $\Tr (\Gamma_k)$ is torsion-free, for $k \in \{1, 2\}$. From the standard presentation of $\Tr (\Gamma)$ we see that $\Tr (\Gamma) = \Tr (\Gamma_1) *_{\Tr (\Gamma_{12})} \Tr (\Gamma_2)$, hence, by \cite[Chapter I, Corollary 1]{Serre1}, $\Tr (\Gamma)$ is torsion-free. Suppose $\{y_1, y_2 \} \in E (\Gamma)$ for all $y_1, y_2 \in Y$, $y_1 \neq y_2$. Note that, since all the elements of $Y$ are maximal for the order $\le$, we have $y_1 || y_2$ for all $y_1, y_2 \in Y$, $y_1 \neq y_2$. Let $X = V(\Gamma) \setminus Y$. Denote by $\Gamma_Y$ the full subgraph of $\Gamma$ spanned by $Y$ and by $\Gamma_X$ the full subgraph of $\Gamma$ spanned by $X$. It is easily observed that $\Gamma_X$ and $\Gamma_Y$ are both parabolic subgraphs of $\Gamma$ and that $\Tr (\Gamma_Y) \simeq \Z^{|Y|}$. Let $y \in Y$. Let $x \in X$ be such that $x \not < y$. By definition, there exists $y' \in Y$ such that $x < y'$. Since $y || y'$, Condition (b) in the definition of a trickle graph implies that $x|| y$. Thus, $X \subseteq \starE_y (\Gamma)$ and $\varphi_y (X) = X$. Let $y,y' \in Y$, $y \neq y'$. Let $x \in X$. If $x \not < y$ and $x \not < y'$, then \[ (\varphi_y \circ \varphi_{y'}) (x) = \varphi_y (x) = x = \varphi_{y'} (x) = (\varphi_{y'} \circ \varphi_y) (x)\,. \] If $x < y$, then, by Condition (b) in the definition of a trickle graph, $x || y'$. Since $\varphi_y$ is an automorphism of $\starE_y (\Gamma)$, we also have $\varphi_y(x) || y'$. So, \[ (\varphi_y \circ \varphi_{y'}) (x) = \varphi_y (x) = \varphi_{y'}(\varphi_y (x)) = (\varphi_{y'} \circ \varphi_y) (x). \] If $x < y'$, then we prove in the same way that \[ (\varphi_y \circ \varphi_{y'}) (x) = (\varphi_{y'} \circ \varphi_y) (x). \] This shows that we have an action of $\Tr (\Gamma_Y) \simeq \Z^{|Y|}$ on $\Tr (\Gamma_X)$ defined by $y \cdot x = \varphi_y (x)$ for all $y \in Y$ and $x \in X$, and that $\Tr (\Gamma) = \Tr (\Gamma_X) \rtimes \Tr (\Gamma_Y)$. Since $\Tr (\Gamma_Y) \simeq \Z^{|Y|}$ is torsion-free and, by the induction hypothesis, $\Tr (\Gamma_X)$ is torsion-free, it follows that $\Tr (\Gamma)$ is also torsion-free. \end{proof} Now, we turn to the study of parabolic submonoids and subgroups of preGarside trickle monoids and groups. We first observe that, by repeating mutatis mutandis the proof of Theorem \ref{thm2_10}, we get the following. \begin{lem}\label{lem7_7} Let $\Gamma_1$ be a parabolic subgraph of $\Gamma$. \begin{itemize} \item[(1)] The canonical homomorphism $\iota_{\Tr^+ (\Gamma_1)}: \Tr^+ (\Gamma_1) \to \Tr^+ (\Gamma)$ is injective. So, we can identify $\Tr^+ (\Gamma_1)$ with $\iota_{\Tr^+ (\Gamma_1)} (\Tr^+ (\Gamma_1))$. \item[(2)] Let $g \in \Tr^+ (\Gamma)$. We have $g \in \Tr^+ (\Gamma_1)$ if and only if $\nf^+ (g) \in V (\Gamma_1)^*$. Moreover, in this case, $g$ has the same normal form in $\Tr^+ (\Gamma)$ as in $\Tr^+ (\Gamma_1)$, that is, $\nf_\Gamma^+ (g) = \nf_{\Gamma_1}^+ (g)$. \end{itemize} \end{lem} As mentioned in Section \ref{sec2}, we have two different definitions of parabolicity, the one coming from the trickle groups and the one coming from the preGarside monoids. Now, we show that these two definitions coincide. \begin{prop}\label{prop7_8} Let $N$ be a submonoid of $\Tr^+ (\Gamma)$. Then $N$ is a parabolic submonoid of $\Tr^+ (\Gamma)$ if and only if there exists a parabolic subgraph $\Gamma_1$ of $\Gamma$ such that $N = \Tr^+ (\Gamma_1)$. \end{prop} \begin{proof} Suppose $N$ is a parabolic submonoid of $\Tr^+ (\Gamma)$. Set $X = V (\Gamma) \cap N$ and denote by $\Gamma_X$ the full subgraph of $\Gamma$ spanned by $X$. Let $g \in N$. Since $V (\Gamma)$ generates $\Tr^+ (\Gamma)$, the element $g$ can be written in the form $g = x_1 x_2 \dots x_p$ with $x_1, \dots, x_p \in V (\Gamma)$. Since $N$ is a parabolic submonoid, $N$ is a special submonoid, hence $x_1, \dots, x_p \in N$, that is, $x_1, \dots, x_p \in X$. This shows that $N$ is the submonoid of $\Tr^+ (\Gamma)$ generated by $X$. Let $x, y \in X$ be such that $y < x$. Since $\varphi_x(y) \, x = x y \in N$ and $N$ is special, we have $\varphi_x(y) \in N \cap V (\Gamma) = X$. This implies that, for all $x \in X$, $\varphi_x$ restricts to an automorphism of $\starE_x (\Gamma_X)$. So, $\Gamma_X$ is a parabolic subgraph of $\Gamma$ and $N = \Tr^+ (\Gamma_X)$. Now, we take a parabolic subgraph $\Gamma_1$ of $\Gamma$ and we show that $\Tr^+ (\Gamma_1)$ is a parabolic submonoid of $\Tr^+ (\Gamma)$. {\it Claim 1.} $\Tr^+ (\Gamma_1)$ is special. {\it Proof of Claim 1.} We prove that, if $x \in V (\Gamma)$ and $h \in \Tr^+ (\Gamma)$ are such that $g= xh \in \Tr^+ (\Gamma_1)$, then $x \in V (\Gamma_1)$ and $h \in \Tr^+ (\Gamma_1)$. Let $w = (U_1, \dots, U_p)$ be the unique piling in $(\Omega^+)^*$ such that $\Phi^+ (\bar w) = g$ and $w$ is $\RR^+$-irreducible. Write $U_1=\{ x_1^{a_1}, \dots, x_q^{a_q} \}$ with $x_1 \succ x_2 \succ \cdots \succ x_q$. Let $\psi = \varphi_{x_1}^{a_1} \circ \varphi_{x_2}^{a_2} \circ \cdots \circ \varphi_{x_q}^{a_q}$. We know from Lemma \ref{lem7_5} that $x \in \psi (\{x_1, x_2, \dots, x_q\})$, hence there exists $i \in \{1, \dots, q \}$ such that $x = (\varphi_{x_1}^{a_1} \circ \cdots \circ \varphi_{x_{i-1}}^{a_{i-1}}) (x_i)$. We have \[ g = x x^{a_i-1} x_1^{a_1} \dots x_{i-1}^{a_{i-1}} x_{i+1}^{a_{i+1}} \dots x_q^{a_q} \omega (U_2) \dots \omega (U_p)\,. \] By Lemma \ref{lem7_7}, $\nf^+ (g) \in V (\Gamma_1)^*$, hence $x_1, \dots, x_q \in V (\Gamma_1)$, and therefore $x \in V (\Gamma_1)$. Moreover, the inclusion $\nf^+ (g) \in V (\Gamma_1)^*$ also implies that $\omega (U_2), \dots, \omega (U_p) \in \Tr^+ (\Gamma_1)$, hence \[ h = x^{a_i-1} x_1^{a_1} \dots x_{i-1}^{a_{i-1}} x_{i+1}^{a_{i+1}} \dots x_q^{a_q} \omega (U_2) \dots \omega (U_p) \in \Tr^+ (\Gamma_1)\,. \] This completes the proof of Claim 1. {\it Claim 2.} Let $g_1, g_2 \in \Tr^+ (\Gamma_1)$. Suppose there exists $g \in \Tr^+ (\Gamma)$ such that $g_1, g_2 \le_L g$. Then there exists $g' \in \Tr^+ (\Gamma_1)$ such that $g_1, g_2 \le_L g' \le_L g$. {\it Proof of Claim 2.} We argue by induction on the word length $\nu (g)$ of $g$. If $\nu (g_1) = 0$, then $g_1 = 1$ and $g' = g_2$ satisfies $g' \in \Tr^+ (\Gamma_1)$ and $g_1, g_2 \le_L g' \le_L g$. Similarly, if $\nu (g_2) = 0$, then $g_2 = 1$ and $g' = g_1$ satisfies $g' \in \Tr^+ (\Gamma_1)$ and $g_1, g_2 \le_L g' \le_L g$. The case $\nu (g_1)= 0$ contains the case $\nu (g) = 0$, hence we can assume that $\nu (g_1) \ge 1$, $\nu (g_2) \ge 1$, and that the induction hypothesis holds. Let $y_1, y_2 \in V (\Gamma_1)$ be such that $y_1 \le_L g_1$ and $y_2 \le_L g_2$, and let $h_1, h_2 \in \Tr^+ (\Gamma_1)$ be such that $g_1 = y_1 h_1$ and $g_2 = y_2 h_2$. First, assume that $y_1 = y_2$. Let $h \in \Tr^+ (\Gamma)$ be such that $y_1 h = g$. Since $\Tr^+ (\Gamma)$ is cancellative, $h_1, h_2 \le_L h$, hence, by the induction hypothesis, there exists $h' \in \Tr^+ (\Gamma_1)$ such that $h_1, h_2 \le_L h' \le_L h$. Let $g' = y_1 h'$. Then $g' \in \Tr^+ (\Gamma_1)$ and $g_1, g_2 \le_L g' \le_L g$. Now, assume $y_1 \neq y_2$. Let $w = (U_1, \dots, U_p)$ be the unique piling in $(\Omega^+)^*$ such that $\Phi^+ (\bar w) = g$ and $w$ is $\RR^+$-irreducible. Write $U_1=\{ x_1^{a_1}, \dots, x_q^{a_q} \}$ with $x_1 \succ x_2 \succ \cdots \succ x_q$. Let $\psi = \varphi_{x_1}^{a_1} \circ \cdots \circ \varphi_{x_q}^{a_q}$. Since $y_1, y_2 \le_L g$, by Lemma \ref{lem7_5} we have $y_1, y_2 \in \psi (\{x_1, x_2, \dots, x_q\})$. Let $i, j \in \{1, \dots, q\}$ be such that $y_1 = \psi(x_i)$ and $y_2 = \psi(x_j)$. We can assume without loss of generality that $i < j$. Then, since $y_1 = \psi (x_i)$ and $y_2 = \psi (x_j)$, we have $\{y_1, y_2\} \in E (\Gamma)$ and $y_1 \not < y_2$. Moreover, \begin{gather*} g = x_1^{a_1} \dots x_q^{a_q} \omega(U_2) \dots \omega (U_p) = y_2^{a_j} x_1^{a_1} \dots x_{j-1}^{a_{j-1}} x_{j+1}^{a_{j+1}} \dots x_q^{a_q} \omega(U_2) \dots \omega (U_p) =\\ y_2^{a_j} y_1^{a_i} x_1^{a_1} \dots x_{i-1}^{a_{i-1}} x_{i+1}^{a_{i+1}} \dots x_{j-1}^{a_{j-1}} x_{j+1}^{a_{j+1}} \dots x_q^{a_q} \omega(U_2) \dots \omega (U_p) =\\ y_2 y_1 (\varphi_{y_1}^{-1}(y_2))^{a_j-1} y_1^{a_i-1} x_1^{a_1} \dots x_{i-1}^{a_{i-1}} x_{i+1}^{a_{i+1}} \dots x_{j-1}^{a_{j-1}} x_{j+1}^{a_{j+1}} \dots x_q^{a_q} \omega(U_2) \dots \omega (U_p)\,, \end{gather*} hence $y_2 y_1 = y_1\, \varphi_{y_1}^{-1} (y_2) \le_L g$. Let $h \in \Tr^+ (\Gamma)$ be such that $y_2 y_1 h = y_1\, \varphi_{y_1}^{-1} (y_2)\, h = g$. We have $h_2, y_1 \le_L y_1 h$, hence, by the induction hypothesis, there exists $k_2 \in \Tr^+ (\Gamma_1)$ such that $h_2, y_1 \le_L k_2 \le_L y_1 h$. Similarly, there exists $k_1 \in \Tr^+ (\Gamma_1)$ such that $h_1, \varphi_{y_1}^{-1} (y_2) \le_L k_1 \le_L \varphi_{y_1}^{-1}(y_2)\, h$. Let $\ell_1, \ell_2 \in \Tr^+ (\Gamma_1)$ be such that $k_2 = y_1 \ell_2$ and $k_1 = \varphi_{y_1}^{-1} (y_2)\, \ell_1$. We have $\ell_1, \ell_2 \le_L h$, hence, by the induction hypothesis, there exists $h' \in \Tr^+ (\Gamma_1)$ such that $\ell_1, \ell_2 \le_L h' \le_L h$. Let $g' = y_2 y_1 h' = y_1\, \varphi_{y_1}^{-1} (y_2)\, h'$. Then $g' \in \Tr^+ (\Gamma_1)$ and $g_1, g_2 \le_L g' \le_L g$. This completes the proof of Claim 2. Recall that $\Gamma_1$ is a parabolic subgraph of $\Gamma$ and that we want to prove that $\Tr^+ (\Gamma_1)$ is a parabolic submonoid of $\Tr^+ (\Gamma)$. We know from Claim 1 that $\Tr^+ (\Gamma_1)$ is special. Let $g_1, g_2 \in \Tr^+ (\Gamma_1)$ be such that $g_1 \vee_L g_2$ exists in $\Tr^+ (\Gamma)$. By Claim 2 there exists $g' \in \Tr^+ (\Gamma_1)$ such that $g_1, g_2 \le_L g' \le_L g_1 \vee_L g_2$. Then, by definition, $g' = g_1 \vee_L g_2 \in \Tr^+ (\Gamma_1)$. Let $g_1, g_2 \in \Tr^+ (\Gamma_1)$ be such that $g_1 \vee_R g_2$ exists in $\Tr^+ (\Gamma)$. By applying the above argument to the dual presentation of $\Tr^+ (\Gamma)$ we get that $g_1 \vee_R g_2 \in \Tr^+ (\Gamma_1)$. \end{proof} \begin{proof}[Proof of Theorem \ref{thm2_18}] Let $N_1$ be a parabolic submonoid of $\Tr^+ (\Gamma)$. By Proposition \ref{prop7_8} there exists a parabolic subgraph $\Gamma_1$ of $\Gamma$ such that $N_1 = \Tr^+ (\Gamma_1)$. The homomorphism $G(N_1) \to \Tr (\Gamma)$ induced by the embedding $N_1 \hookrightarrow \Tr^+ (\Gamma)$ is the homomorphism $\iota_1 : \Tr (\Gamma_1) \to \Tr (\Gamma)$ induced by the embedding of $\Gamma_1$ into $\Gamma$, and we know from Theorem \ref{thm2_10} that this homomorphism is injective. Let $g \in G(N_1) \cap \Tr^+ (\Gamma) = \Tr (\Gamma_1) \cap \Tr^+ (\Gamma)$. By Theorem \ref{thm2_10} we have $\nf (g) \in (V (\Gamma_1) \cup V (\Gamma_1)^{-1})^*$ and by Lemma \ref{lem7_4} we have $\nf (g) \in V(\Gamma)^*$, hence $\nf (g) \in V (\Gamma_1)^*$, and therefore $g \in \Tr^+ (\Gamma_1)$. This shows that $N_1 = \Tr^+ (\Gamma_1) = G (N_1) \cap \Tr^+ (\Gamma)$. Let $N_2$ be another parabolic submonoid of $\Tr^+ (\Gamma)$. As for $N_1$, there exists a parabolic subgraph $\Gamma_2$ of $\Gamma$ such that $N_2 = \Tr^+ (\Gamma_2)$. Let $g \in N_1 \cap N_2 = \Tr^+ (\Gamma_1) \cap \Tr^+ (\Gamma_2)$. By Lemma \ref{lem7_7} we have $\nf^+ (g) \in V(\Gamma_1)^* \cap V (\Gamma_2)^* = V (\Gamma_1 \cap \Gamma_2) ^*$, hence $g \in \Tr^+ (\Gamma_1 \cap \Gamma_2)$. This shows that $N_1 \cap N_2 \subseteq \Tr^+ (\Gamma_1 \cap \Gamma_2)$. Since the inclusion $\Tr^+ (\Gamma_1 \cap \Gamma_2) \subseteq N_1 \cap N_2$ is obvious, it follows that $N_1 \cap N_2 = \Tr^+ (\Gamma_1 \cap \Gamma_2)$. In particular, by Proposition \ref{prop7_8}, $N_1 \cap N_2$ is a parabolic submonoid of $\Tr^+ (\Gamma)$. Finally, applying Corollary \ref{corl2_12} we get \[ G (N_1) \cap G (N_2) = \Tr (\Gamma_1) \cap \Tr (\Gamma_2) = \Tr (\Gamma_1 \cap \Gamma_2) = G (N_1 \cap N_2)\,. \proved \] \end{proof} \frenchspacing \begin{thebibliography}{DDH$\phantom{}^+$07} \bibitem[BE22]{BaElh1} {\bf I Ba, M Elhamdadi,} {\it Circular orderability and quandles,} Preprint, arXiv:2204.09458, 2022. \bibitem[BPS22]{BaPaSi1} {\bf V\,G Bardakov, I\,B\,S Passi, M Singh,} {\it Zero-divisors and idempotents in quandle rings,} Osaka J. Math. 59 (2022), no. 3, 611--637. \bibitem[BCP16]{BeCiPa1} {\bf P Bellingeri, B\,A Cisneros de la Cruz, L Paris,} {\it A simple solution to the word problem for virtual braid groups,} Pacific J. Math. 283 (2016), no. 2, 271--287. \bibitem[BP20]{BelPar1} {\bf P Bellingeri, L Paris,} {\it Virtual braids and permutations,} Ann. Inst. Fourier (Grenoble) 70 (2020), no. 3, 1341--1362. \bibitem[BPT23]{BePaTh1} {\bf P Bellingeri, L Paris, A-L Thiel,} {\it Virtual Artin groups,} Proc. London Math. Soc. (3) 126 (2023), no. 1, 192--215. \bibitem[Bes03]{Bessi1} {\bf D Bessis,} {\it The dual braid monoid,} Ann. Sci. École Norm. Sup. (4) 36 (2003), no. 5, 647--683. \bibitem[BKL98]{BiKoLe1} {\bf J Birman, K\,H Ko, S\,J Lee,} {\it A new approach to the word and conjugacy problems in the braid groups,} Adv. Math. 139 (1998), no. 2, 322--353. \bibitem[Bon16]{Bonna1} {\bf C Bonnaf\'e,} {\it Cells and cacti,} Int. Math. Res. Not. IMRN (2016), no. 19, 5775--5800. \bibitem[Bou68]{Bourb1} {\bf N Bourbaki,} {\it \'El\'ements de math\'ematique. Fasc. XXXIV. Groupes et alg\`ebres de Lie. Chapitre IV: Groupes de Coxeter et syst\`emes de Tits. Chapitre V: Groupes engendr\'es par des r\'eflexions. Chapitre VI: syst\`emes de racines,} Actualit\'es Scientifiques et Industrielles No 1337. Hermann, Paris, 1968. \bibitem[BW08]{BraWat1} {\bf T Brady, C Watt,} {\it Non-crossing partition lattices in finite real reflection groups,} Trans. Amer. Math. Soc. 360 (2008), no. 4, 1983--2005. \bibitem[BS72]{BriSai1} {\bf E Brieskorn, K Saito,} {\it Artin-Gruppen und Coxeter-Gruppen,} Invent. Math. 17 (1972), 245--271. \bibitem[Bur24]{Buril1} {\bf J. Burillo,} {\it Introduction to Thompson's group $F$,} Book currently being written, available at https://web.mat.upc.edu/pep.burillo. \bibitem[CFP96]{CaFlPa1} {\bf J\,W Cannon, W\,J Floyd, W\,R Parry,} {\it Introductory notes on Richard Thompson's groups,} Enseign. Math. (2) 42 (1996), no. 3--4, 215--256. \bibitem[CGP20]{ChGlPy1} {\bf M Chmutov, M Glick, P Pylyavskyy,} {\it The Berenstein-Kirillov group and cactus groups,} J. Comb. Algebra 4 (2020), no. 2, 111--140. \bibitem[CGW09]{CrGoWi1} {\bf J Crisp, E Godelle, B Wiest,} {\it The conjugacy problem in subgroups of right-angled Artin groups,} J. Topol. 2 (2009), no. 3, 442--460. \bibitem[DDH$\phantom{}^+$07]{DDHPV1} {\bf M\,A Dabkowska, M\,K Dabkowski, V\,S Harizanov, J\,H Przytycki, M\,A Veve,} {\it Compactness of the space of left orders,} J. Knot Theory Ramifications 16 (2007), no. 3, 257--266. \bibitem[DJS03]{DaJaSc1} {\bf M Davis, T Januszkiewicz, R Scott,} {\it Fundamental groups of blow-ups,} Adv. Math. 177 (2003), no. 1, 115--179. \bibitem[Deh02]{Dehor1} {\bf P Dehornoy,} {\it Groupes de Garside,} Ann. Sci. \'Ecole Norm. Sup. (4) 35 (2002), no. 2, 267--306. \bibitem[DDG$\phantom{}^+$15]{DDGKM1} {\bf P Dehornoy, F Digne, E Godelle, D Krammer, J Michel,} {\it Foundations of Garside theory,} EMS Tracts Math., 22, European Mathematical Society (EMS), Zürich, 2015. \bibitem[DP99]{DehPar1} {\bf P Dehornoy, L Paris,} {\it Gaussian groups and Garside groups, two generalisations of Artin groups,} Proc. London Math. Soc. (3) 79 (1999), no. 3, 569--604. \bibitem[Dev99]{Devad1} {\bf S\,L Devadoss,} {\it Tessellations of moduli spaces and the mosaic operad,} Contemp. Math., 239, American Mathematical Society, Providence, RI, 1999, 91--114. \bibitem[DK92]{DucKro1} {\bf G Duchamp, D Krob,} {\it The lower central series of the free partially commutative group,} Semigroup Forum 45 (1992), no. 3, 385--394. \bibitem[EHK$\phantom{}^+$10]{EHKR1} {\bf P Etingof, A Henriques, J Kamnitzer, E\,M Rains,} {\it The cohomology ring of the real locus of the moduli space of stable curves of genus 0 with marked points,} Ann. of Math. (2) 171 (2010), no. 2, 731--777. \bibitem[Gen22]{Genev1} {\bf A Genevois,} {\it Cactus groups from the viewpoint of geometric group theory,} Topology Proceedings, to appear, arXiv: 2212.03494, 2022. \bibitem[Gob24]{Gobet1} {\bf T Gobet,} {\it Toric reflection groups,} J. Aust. Math. Soc. 116 (2024), no. 2, 171--199. \bibitem[God07]{Godel1} {\bf E Godelle,} {\it Parabolic subgroups of Garside groups,} J. Algebra 317 (2007), no. 1, 1--16. \bibitem[God10]{Godel2} {\bf E Godelle,} {\it Parabolic subgroups of Garside groups II: ribbons,} J. Pure Appl. Algebra 214 (2010), no. 11, 2044--2062. \bibitem[GP12a]{GodPar1} {\bf E Godelle, L Paris,} {\it $K(\pi,1)$ and word problems for infinite type Artin-Tits groups, and applications to virtual braid groups,} Math. Z. 272 (2012), no. 3--4, 1339--1364. \bibitem[GP12b]{GodPar3} {\bf E Godelle, L Paris,} {\it Basic questions on Artin-Tits groups,} Configuration spaces, 299--311. CRM Series, 14, Edizioni della Normale, Pisa, 2012. \bibitem[GP13]{GodPar2} {\bf E Godelle, L Paris,} {\it PreGarside monoids and groups, parabolicity, amalgamation, and FC property,} Internat. J. Algebra Comput. 23 (2013), no. 6, 1431--1467. \bibitem[Gre90]{Green1} {\bf E\,R Green,} {\it Graph products of groups,} Ph. D. Thesis, University of Leeds, 1990. \bibitem[HK06]{HenKam1} {\bf A Henriques, J Kamnitzer,} {\it Crystals and coboundary categories,} Duke Math. J. 132 (2006), no. 2, 191--216. \bibitem[HM95]{HerMei1} {\bf S Hermiller, J Meier,} {\it Algorithms and geometry for graph products of groups,} J. Algebra 171 (1995), no. 1, 230--257. \bibitem[${\rm IKL}^+$23]{IKLPR1} {\bf A Ilin, J Kamnitzer, Y Li, P Przytycki, L Rybnikov,} {\it The moduli space of cactus flower curves and the virtual cactus group,} Preprint, arXiv:2308.06880, 2023. \bibitem[Joy99]{Joyce1} {\bf D Joyce,} {\it A classifying invariant of knots, the knot quandle,} J. Pure Appl. Algebra 23 (1982), no. 1, 37--65. \bibitem[Kau99]{Kauff1} {\bf L\,H Kauffman,} {\it Virtual knot theory,} European J. Combin. 20 (1999), no. 7, 663--690. \bibitem[Kau00]{Kauff2} {\bf L\,H Kauffman,} {\it A survey of virtual knot theory,} Knots in Hellas '98 (Delphi), 143--202, Ser. Knots Everything, 24, World Sci. Publ., River Edge, NJ, 2000. \bibitem[KW19]{KhWi1} {\bf A Khoroshkin, T Willwacher,} {\it Real moduli space of stable rational curves revisted,} preprint, arXiv:1905.04499, 2019, to appear in J. Eur. Math. Soc.. \bibitem[KTW04]{KnTaWo1} {\bf A Knutson, T Tao, C Woodward,} {\it A positive proof of the Littlewood-Richardson rule using the octahedron recurrence,} Electron. J. Combin. 11 (2004), no. 1, Research Paper 61, 18 pp. \bibitem[Los19]{Losev1} {\bf I Losev,} {\it Cacti and cells,} J. Eur. Math. Soc. (JEMS) 21 (2019), no. 6, 1729--1750. \bibitem[MS17]{McCSul1} {\bf J McCammond, R Sulway,} {\it Artin groups of Euclidean type,} Invent. Math. 210 (2017), no. 1, 231--282. \bibitem[Mat82]{Matve1} {\bf S\,V Matveev,} {\it Distributive groupoids in knot theory,} Mat. Sb. (N.S.) 119 (161) (1982), no. 1, 78--88, 160. \bibitem[Mos19]{Mosto1} {\bf J Mostovoy,} {\it The pure cactus group is residually nilpotent,} Arch. Math. (Basel) 113 (2019), no. 3, 229--235. \bibitem[New42]{Newma1} {\bf M\,H\,A Newman,} {\it On theories with a combinatorial definition of ``equivalence'',} Ann. of Math. (2) 43 (1942), 223--243. \bibitem[PS21]{PaoSal1} {\bf G Paolini, M Salvetti,} {\it Proof of the $K (\pi, 1)$ conjecture for affine Artin groups,} Invent. Math. 224 (2021), no. 2, 487--572. \bibitem[Par02]{Paris1} {\bf L Paris,} {\it Artin monoids inject in their groups,} Comment. Math. Helv. 77 (2002), no. 3, 609--637. \bibitem[PS23]{ParSoe1} {\bf L Paris, M Soergel,} {\it Word problem and parabolic subgroups in Dyer groups,} Bull. Lond. Math. Soc. 55 (2023), no. 6, 2928--2947. \bibitem[RW24]{RouWhi1} {\bf R Rouquier, N White,} {\it Cactus groups and Lusztig's asymptotic algebra,} preprint, arXiv:2408.16922, 2024. \bibitem[Ser77]{Serre1} {\bf J\,P Serre,} {\it Arbres, amalgames, SL2,} Astérisque, No. 46, Société Mathématique de France, Paris, 1977. \bibitem[Tit69]{Tits1} {\bf J Tits,} {\it Le probl\`eme des mots dans les groupes de Coxeter,} 1969 Symposia Mathema\-ti\-ca (INDAM, Rome, 1967/68), Vol. 1, pp. 175--185, Academic Press, London. \bibitem[Wyk94]{Wyk1} {\bf L Van Wyk,} {\it Graph groups are biautomatic,} J. Pure Appl. Algebra 94 (1994), no. 3, 341--352. \bibitem[Yu23]{Yu1} {\bf R Yu,} {\it Linearity of generalized cactus groups,} J. Algebra 635 (2023), 256--270. \end{thebibliography} \end{document}
2412.04973v1
http://arxiv.org/abs/2412.04973v1
On the Laplace equation with non-local dynamical boundary conditions
\documentclass[10pt,a4paper]{article} \usepackage[utf8]{inputenc} \usepackage[english]{babel} \usepackage{amsmath} \usepackage{amsfonts} \usepackage{amsthm} \usepackage{amssymb} \usepackage{color} \usepackage[margin=2cm]{geometry} \usepackage[colorlinks=false]{hyperref} \newtheorem{theorem}{Theorem}[section] \newtheorem{definition}{Definition} \newtheorem{lemma}{Lemma} \newtheorem{prop}{Proposition}[section] \newtheorem{remark}{Remark}[section] \newtheorem{assumption}{Assumption} \numberwithin{equation}{section} \allowdisplaybreaks \newcommand{\R}{{\mathbb R}} \newcommand{\RR}{{\mathbb R}} \newcommand{\N}{{\mathbb N}} \newcommand{\C}{{\mathbb C}} \newcommand{\D}{{\mathbb D}} \def\D{\mathbb D} \def\const{\text{\upshape Const.}} \def\loc{{\text{\upshape loc}}} \def\negquad{\!\!\!\!} \def\negqquad{\!\!\!\!\!\!\!\!} \def\eps{\varepsilon} \def\om{{\overline{m}}} \begin{document} \title{On the Laplace equation with non-local dynamical boundary conditions} \author{\textsc{Raffaela Capitanelli \& Mirko D'Ovidio}\\ [email protected], [email protected]\\ \textit{Sapienza University of Rome, Rome, Italy}} \maketitle \begin{abstract} Aim of the paper is to study non-local dynamic boundary conditions of reactive-diffusive type for the Laplace equation from analytic and probabilistic point of view. In particular, we provide compact and probabilistic representation of the solution together with an interpretation in term of boundary processes. \end{abstract} {\bf MSC 2020:} 60J50; 35J25 {\bf KEYWORDS:} Brownian motions; Dynamic boundary conditions; Non-Local operators. \section{Introduction} Aim of the paper is to study non-local dynamic boundary conditions of reactive-diffusive type for the Laplace equation. More precisely, for $k \in \mathbb{R}$ and $l >0$, we consider the following problem \begin{align} \label{E} \left\lbrace \begin{array}{ll} \displaystyle \Delta u=0 \qquad &\text{in $(0,\infty)\times\Omega$,}\\ D_t^{\alpha} u = k \partial_{\bf n} u +l\Delta_\Gamma u \qquad &\text{on $(0,\infty)\times \Gamma$,}\\ u(0,x)=u_0(x) & \text{on $ \Gamma$,} \end{array} \right . \end{align} where $u=u(t,x)$, $t\in [0, \infty)$, $x\in\Omega$ and $\Gamma=\partial\Omega$. The symbol $\Delta_\Gamma$ denotes the Laplace--Beltrami operator on $\Gamma$ and $\partial_{\bf n} u$ is the normal derivative being ${\bf n}$ the outward normal to $\Omega.$ We assume that $\Omega$ is a $C^\infty$ bounded domain of $\R^N$ ($N\ge 2$) and $\Gamma$ is a Riemannian manifold endowed with the natural inherited metric. The non-local operator \begin{align} \label{dc} D^\alpha_t u(t) = \frac{1}{\Gamma(1-\alpha)} \int_0^t u^\prime(s) (t-s)^{-\alpha} ds, \quad \alpha \in (0,1) \end{align} is the Caputo-D\v{z}rba\v{s}jan derivative. We recall that, for $z>0$, $\Gamma(z) = \int_0^\infty s^{z-1} e^{-s}ds$ is the absolutely convergent Euler integral also termed gamma function.\\ We provide a compact representation for the solution to the problem \eqref{E}. We exploit the result in \cite{VV1} for which adding the Laplace--Beltrami operator on the boundary ($l > 0$) the corresponding problem is well-posed for any value of $k$. Moreover, under suitable spectral conditions, we show that such solution has the probabilistic representation \begin{align} \mathbf{E}_{\mu(x)}[u_0(X^\Gamma \circ L_t)] = \int_\Gamma \mathbf{E}_y[u_0(X^\Gamma \circ L_t)] \mu(x,dy), \quad x \in \Omega \cup \Gamma \label{repIniDist} \end{align} where the initial distribution $\mu(x,dy)$ is an harmonic measure and $X^\Gamma$ is a boundary Markov process with random time $L$ explaining the non-local dynamic. Thus, our problem can be treated in terms of a fractional Cauchy problem on compact manifold without boundary for which \begin{align*} \mathbf{E}_{\mu(x)}[u_0(X^\Gamma \circ L_t)] = \mathbf{E}_x[u_0(X^\Gamma \circ L_t)], \quad x \in \Gamma \end{align*} gives the solution on $\Gamma$ according with the Poisson integral \eqref{repIniDist}.\\ The operator \eqref{dc} has been introduced in \cite{caputoBook,CapMai71,CapMai71b} by the first author and separately in a series of works starting from \cite{Dzh66,DzhNers68} by the second author. For $\alpha=1$ the operator $D^\alpha_t$ becomes the ordinary derivative for which we get the dynamical boundary condition of reactive-diffusive type \begin{align} \dot{u} = k \partial_{\bf n} u + l \Delta_\Gamma u \quad \textrm{on } (0,\infty) \times \Gamma. \label{RDtypeC} \end{align} From an analytical point of view, such a \textit{non fractional} dynamical boundary condition has been studied in \cite{VV1} for the Laplace equation. In particular, such a condition describes a heat conduction process in $\Omega$ with a heat source on the boundary which can depend on the heat flux around the boundary and on the heat flux across it (see for example \cite{GM} and the reference therein). We remark that in the case $l=0$, the problem has been studied by many authors (see for example \cite{E}, \cite{H}, \cite{JL}), in this case the corresponding problem is well-posed if and only if $k\leq 0$. For the parabolic case associated with \eqref{RDtypeC} we mainly refer to \cite{VV2} where the authors considered the heat equation in place of the Laplace equation. We refer to \cite{Dov24II} for the parabolic case associated with the non-local condition in \eqref{E} which has been investigated only in case $l=0$ and $k<0$. For the fractional Cauchy problem we may refer to a very large literature. Such problems are well-know and they have been investigated by many researchers on both regular domains (see for example \cite{Chen17, GLY15, Koc2011, MLP2001, MNV09, Toaldo2015}) or irregular domains (see for example \cite{CCL, CapDovFEforms19, CapDovFCP21}). Recently the non-local dynamic boundary conditions for the heat equation have been investigated by \cite{Dov22fcaa, Dov24fcaa} together with the associated probabilistic representation in terms of sticky Brownian motions. The plan of the paper is the following. In the second section we study the Laplace equation with fractional dynamical boundary conditions from the analytical point of view and in particular we obtain a representation formula for the solution. In the third section we deal with the same problem from the probabilistic point of view and in particular we give a interpretation for the solution in term of boundary processes. In the last section we consider the Laplace equation with a general non local boundary condition. \section{Compact representation} In this section we study the Laplace equation on a bounded regular domain $\Omega$ of $\R^N$ ($N\ge 2$) with time-fractional dynamical boundary condition of reactive--diffusive type described in \eqref{E}. We denote by $\mathcal{D}(\Gamma )$ the space $C^\infty(\Gamma )$ and by $\mathcal{D}'(\Gamma)$ its dual space, that is the space of distributions on $\Gamma$. Moreover for any $s \in \R$ we denote by the symbols $H^s(\Omega)$ and $H^s(\Gamma)$ the Sobolev spaces of distributions respectively on $\Omega$ and on its boundary $ \Gamma$ (see \cite{LM}). We recall that the Laplace-Beltrami operator $\Delta_\Gamma$ can be defined on $C^\infty(\Gamma)$ by the formula \begin{equation}\label{M1} -\int_\Gamma (\Delta_\Gamma u) v \,ds=\int _\Gamma \nabla_\Gamma u\, \nabla_\Gamma v \,ds \end{equation} for any $u,v\in C^\infty(\Gamma)$ where we denote by $\nabla_\Gamma$ the Riemannian gradient and by $ds$ the natural volume element on $\Gamma.$ We recall that, for any $s\in \R $, the operator $ \Delta_\Gamma : D(\Delta_\Gamma) = H^{s+2}(\Gamma)\to H^s(\Gamma)$ generates an analytic semigroup on $H^{s+2}(\Gamma)$ (see, for example, Appendix A in \cite{VV1}). Before stating our result, we recall some properties of the Dirichlet--to--Neumann operator $A_{DN}$ (see \cite{LM}). For any $g\in H^s(\Gamma)$, $s>0$, the non--homogeneous Dirichlet problem \begin{equation}\label{NH} \begin{cases} \Delta v=0,& \text{in $\Omega$}, \\ v =g & \text{on $\Gamma$,} \end{cases} \end{equation} has a unique solution $v\in H^{s+1/2}(\Omega).$ Moreover, for any datum $g\in H^s(\Gamma)$ of the non--homogeneous Dirichlet problem \eqref{NH}, the operator $\D $ that associates the datum $g$ to the solution $v= \D g $ is linear and bounded from $H^s(\Gamma)$ to $H^{s+\frac12}(\Omega).$ As $v$ has normal derivative $\partial_{\bf n} v \in H^{s-1}(\Gamma),$ the Dirichlet--to--Neumann operator $A_{DN}:$ $g \mapsto \partial_{\bf n} v$ is bounded from $H^s(\Gamma)$ to $H^{s-1}(\Gamma).$ We point out that, for all $u,v\in C^\infty(\Gamma)$, by integrating by parts, we obtain \begin{equation}\label{M2} \int_\Gamma A_{DN} u\, v ds=\int_\Omega\nabla (\D u)\nabla (\D v) d \mu \end{equation} (we denote by $d\mu$ the natural volume element on $\Omega)$. We have that $A_{DN}$ generates an analytic semigroup. Furthermore it is possible to prove that for $l>0$ the operator $k A_{DN} + l \Delta_\Gamma $ with domain $D(k A_{DN} + l \Delta_\Gamma) = H^{s+2}(\Gamma)$ generates an analytic semigroup in $H^s$ (see the proof of Theorem 1 in \cite{VV1}). \bigskip Now our aim is to prove a representation formula for the solution of problem \eqref{E} following the same approach of \cite{VV1}. We start by recalling Theorem 2 from \cite{VV1}. \begin{theorem}\label{th1} Let $l > 0.$ Then there is an Hilbert basis $\{\varphi_n, n\in \N\}$ of $L^2(\Gamma)$, $\varphi_n\in \mathcal{D}(\Gamma),$ $\varphi_n$ real valued, and an increasing real sequence $\{\lambda_n, n\in \N\}$, $\lambda_n\to\infty $ as $ n\to\infty,$ each $\lambda_n$ having finite multiplicity, such that for any $n\in \N$ a unique solution $\psi_n$ (which belongs to $C^\infty(\bar\Omega )$ of the Dirichlet non-homogeneous problem \begin{align} \label{E1} \left\lbrace \begin{array}{ll} \displaystyle \Delta \psi_n=0 \qquad &\text{in $\Omega$,}\\ \psi_n(x)=\varphi_n(x)&\text{on $\Gamma$} \end{array} \right. \end{align} solves the eigenvalue problem \begin{align} \label{E2} \left\lbrace \begin{array}{ll} \displaystyle \Delta \psi_n=0 \qquad &\text{in $\Omega$,}\\ \lambda_n\psi_n =-k \partial_{\bf n} \psi_n - l \Delta_\Gamma \psi_n \qquad &\text{on $ \Gamma$.} \end{array} \right . \end{align} \end{theorem} We now consider, for a real parameter $\Lambda,$ the operator $$A_{\Lambda} := - k A_{DN} - l \Delta_\Gamma + \Lambda I$$ in $H^s(\Gamma)$. It is possible to prove that there exists $\bar\Lambda\geq 0$ such that for any $\Lambda\geq \bar\Lambda$ the following problem $$- k A_{DN} v- l \Delta_\Gamma v + \Lambda v=h$$ with $h$ an arbitrary element of $H^s(\Gamma)$ has a unique solution $v\in H^{s+2}(\Gamma)$ (see Lemma 1 in \cite{VV1}). Then from now on we put $\Lambda=\bar\Lambda$ without loss of generality. Denote by $ <\cdot ,\cdot >$ the norm $ (\cdot ,\cdot )_{L^2(\Gamma)}$. For the operator $$A_{\Lambda}:D(A_{\Lambda})=H^2(\Gamma)\subset L^2(\Gamma)\to L^2(\Gamma)$$ we consider nonnegative real powers $$A^{s/2}_{\Lambda}:D(A^{s/2}_{\Lambda})\subset L^2(\Gamma)\to L^2(\Gamma)$$ where $$D(A^{s/2}_{\Lambda})=\{ u\in L^2(\Gamma) : \sum_{n=1}^\infty | <u, \varphi_n>|^2 (\lambda_n+\Lambda)^s<\infty)\}$$ and $$A^{s/2}_{\Lambda}=\sum_{n=1}^\infty <u, \varphi_n> (\lambda_n+\Lambda)^{\frac{s}2} \varphi_n$$ for $u\in D(A^{s/2}_{\Lambda}).$ It is possible to prove that $H^s(\Gamma)=D(A^{s/2}_{\Lambda})$ (see the proof of Theorem 2 in \cite{VV1}). Then we equip $H^s(\Gamma),$ $s\in \R,$ with the scalar product $$((u,v))_{H^s(\Gamma)}=(A_\Lambda^{s/2} u, A_\Lambda^{s/2} v)_{L^2(\Gamma)}$$ and norm $$||| \cdot|||_{H^s(\Gamma)} =||A_\Lambda^{s/2} u||_{L^2(\Gamma)}$$ which is equivalent to the standard one. Then $ \{\varphi_n/|||\varphi_n|||_{H^s(\Gamma)}, n \in \N\}$ is an Hilbert basis of $H^s(\Gamma)$ for all $s\in \R,$ provided this space is endowed with the scalar product $((\cdot ,\cdot ))_{H^s(\Gamma)} $ and norm $|||\cdot|||_{H^s( \Gamma)}$ We use the following characterization of Sobolev space $$H^s(\Gamma)=\{ u\in \mathcal{D}': \sum_{n=1}^\infty | <u, \varphi_n>|^2 (\lambda_n+\Lambda)^s <\infty \}$$ (for the proof, see, for example, Appendix B in \cite{VV1}). Further on we consider the problem \begin{align} \label{ELambda} \left\lbrace \begin{array}{ll} \displaystyle \Delta u=0 \qquad &\text{in $(0,\infty)\times\Omega$,}\\ D_t^{\alpha} u = -A_\Lambda u \qquad &\text{on $(0,\infty)\times \Gamma$,}\\ u(0,x)=u_0(x) & \text{on $ \Gamma$,} \end{array} \right . \end{align} which coincides with \eqref{E} for $\Lambda=0$. We say that $u$ is solution of problem \eqref{ELambda} if $$u= \D v$$ $$v\in C([0,\infty);H^{s}(\Gamma))$$ $$ D^\alpha_t v \in C((0,\infty);H^{s}(\Gamma)) $$ such that \begin{align*} D_t^{\alpha} v = - A_\Lambda v \; \textrm{ on $H^s(\Gamma)$ for all $t>0$ with $v(0,x)=u_0(x)$ on $\Gamma$.} \end{align*} We point out that as $u= \D v$ then $u\in C^{\infty}(\Gamma)$ and $ \Delta u=0$ in classical sense.\\ We show below that the solution to $D_t^{\alpha} v = k \partial_{\bf n} v +l\Delta_\Gamma v - \Lambda v$ on $H^s(\Gamma)$ with initial datum $u_0 \in H^s(\Gamma)$ can be obtained, under suitable conditions, via time change according with the well-established theory associated with fractional Cauchy problems on compact manifold without boundary. \\ Thus, we provide spectral conditions for which, given any $u_0\in H^s(\Gamma)$ there is a unique solution $v$ satisfying the time fractional equation $D_t^{\alpha} v = k \partial_{\bf n} v +l\Delta_\Gamma v -\Lambda v$ on the boundary $\Gamma$ with the initial condition $v(0,x)=u_0(x)$ on $ \Gamma$. Existence and uniqueness of the solution to problem \eqref{E} is therefore obtained by noticing that $u = \D v$. By using the previous arguments, we prove the following representation formula (see also Remark \ref{rmk:IMP}). First we recall that $E_{\alpha, \beta}$ with $\alpha,\beta>0$ is the well-known Mittag-Leffler function (see \cite{GorKilMaiRog} for definition and properties).\\ \begin{theorem} \label{th2} Let $\Lambda=0$, $l > 0$, $k \in \mathbb{R}$ and $\alpha \in (0,1]$. Let assume $\lambda_1\geq 0.$ Then for any $u_0 \in H^s( \Gamma)$ the solution $u$ of problem \eqref{ELambda} is given by \begin{align} u(t, x) = \sum_{n=1}^\infty <u_0,\varphi_n>E_{\alpha,1}(-t^{\alpha}\lambda_n)\psi_n(x) \label{uAlfa} \end{align} the series being convergent in $C((0,\infty) \times \bar\Omega).$ \end{theorem} \begin{proof} Let us consider $\alpha \in (0,1)$. By using the method of separation of variables, we put $u(x, t) = G(t)F(x).$ Then substituting into the boundary condition in \eqref{E}, we obtain on $\Gamma$ $$ F(x) D_t^{\alpha} G(t)= G(t)( k \partial_{\bf n} F(x) +l\Delta_\Gamma F(x)) $$ and therefore $$ \frac{ D_t^{\alpha} G(t)}{G(t)}= \frac{ k \partial_{\bf n} F(x) +l\Delta_\Gamma F(x)}{F(x)}=-\lambda$$ for some $\lambda>0.$ Then we obtain \begin{equation} \label{1} k \partial_{\bf n} F(x) + l\Delta_\Gamma F(x)=-\lambda F(x) \end{equation} and \begin{equation}\label{2}D_t^{\alpha} G(t)= -\lambda G(t).\end{equation} From Theorem \ref{th1}, we have that there is an Hilbert basis $\{\varphi_n, n\in \N\}$ of $L^2(\Gamma)$, $\varphi_n\in \mathcal{D}(\Gamma),$ $\varphi_n$ real valued, and an increasing real sequence $\{\lambda_n, n\in \N\}$, $\lambda_n\to\infty $ as $ n\to\infty,$ each $\lambda_n$ having finite multiplicity, such that for any $n\in \N$ the first eigenvalue problem \eqref{1} is solved by $$k \partial_{\bf n} \psi_n + l \Delta_\Gamma \psi_n = - \lambda_n\psi_n .$$ It is well-known that the solution to \begin{align} D^\alpha_t \varphi = - c\, \varphi, \quad \varphi(0)=1, \quad c>0, \label{eqML} \end{align} is given by the Mittag-Leffler function $\varphi(t)=E_{\alpha,1}(-c t^\alpha)$. Then second eigenvalue problem \eqref{2} with $\lambda=\lambda_n$ is solved by the function $ E_{\alpha,1}(-t^{\alpha}\lambda_n).$ By considering the initial datum $$u_0=\sum_{n=1}^\infty <u_0,\varphi_n>\varphi_n$$ in $H^s( \Gamma)$ we obtain that for $x\in \Gamma$ $$u(t, x) =\sum_{n=1}^\infty <u_0,\varphi_n>E_{\alpha,1}(-t^{\alpha}\lambda_n)\varphi_n(x)$$ in $H^s( \Gamma)$ for all $t\geq 0$ where the convergence is in the $C([0,\infty); H^s(\Gamma )).$ In fact we have that the serie converges in $C([0,T); H^s(\Gamma ))$ for all $T > 0$ as $$|||\varphi_n|||^2_{H^s(\Gamma)}=(\lambda_n+\Lambda)^s$$ for all $s\in \R$ and $n\in \N$ and so $$||| <u_0,\varphi_n>E_{\alpha,1}(-t^{\alpha}\lambda_n)\varphi_n(x)|||^2_{H^s(\Gamma)}\leq | <u_0,\varphi_n> |^2 (E_{\alpha,1}(-t^{\alpha}\lambda_1))^2 (\lambda_n+\Lambda)^s$$ by using the fact that $E_{\alpha,1}(-t^{\alpha}\lambda_n)$ is completely monotone for $0\leq t \leq T$ and $0 < \alpha < 1$ and $\lambda_n\geq \lambda_1.$ Therefore as $u_0\in H^s(\Gamma )$ and then $\sum_{n=1}^{\infty} | <u_0,\varphi_n> |^2(\lambda+\Lambda)^s<\infty$ we obtain that $$\sum_{n=1}^{\infty} || <u_0,\varphi_n>E_{\alpha,1}(-t^{\alpha}\lambda_n)\varphi_n(x)||^2_{C([0,T], H^s(\Gamma))}< \infty$$ and then the serie converges in $C([0,T); H^s(\Gamma ))$ for all $T > 0$ and therefore in $C([0,\infty); H^s(\Gamma)).$ Because, for any datum $g\in H^s(\Gamma)$ of the non--homogeneous Dirichlet problem \eqref{NH}, the operator $ \D$ that associates the datum $g$ to the solution $v= \D g$ is linear and bounded from $H^s(\Gamma)$ to $H^{s+\frac12}(\Omega),$ by considering $\D\varphi_n=\psi_n,$ we obtain that $$u(t, x) =\sum_{n=1}^\infty <u_0,\varphi_n>E_{\alpha,1}(-t^{\alpha}\lambda_n)\psi_n(x)$$ converges in $H^{s+1/2}( \Omega)$ for all $t\geq 0$ where the convergence is in the $C([0,\infty); H^{s+1/2}(\Omega )) $ topology. Now we consider the time fractional derivative of $$u(t, x) =\sum_{n=1}^\infty <u_0,\varphi_n>E_{\alpha,1}(-t^{\alpha}\lambda_n)\varphi_n(x). $$ in $C([\varepsilon,T); H^s(\Gamma))$ for all $0<\varepsilon<T<\infty.$ We prove the convergence of $$\sum_{n=1}^\infty <u_0,\varphi_n> (-\lambda_n)E_{\alpha,1}(-t^{\alpha}\lambda_n)\varphi_n(x) $$ by considering $$||| <u_0,\varphi_n> (-\lambda_n) E_{\alpha,1}(-t^{\alpha}\lambda_n)\varphi_n(x)|||^2_{H^s(\Gamma)}\leq | <u_0,\varphi_n> |^2 (E_{\alpha,1}(-t^{\alpha}\lambda_n))^2 |\lambda_n|^2 (\lambda_n+\Lambda)^s.$$ By using again the property of $E_{\alpha,1}$ with $0 < \alpha < 1,$ (see Theorem 1.6 in \cite{POD}) we have that $$|(E_{\alpha,1}(-t^{\alpha}\lambda_n))^2 |\lambda_n|^2\leq (\frac 1{1+ t^{\alpha}\lambda_n})^2 |\lambda_n|^2.$$ Then there exist $n_0$ such that, for any $n>n_0$ for $0<\varepsilon<t,$ we have $$||| <u_0,\varphi_n> (-\lambda_n) E_{\alpha,1}(-t^{\alpha}\lambda_n)\varphi_n(x)|||^2_{H^s(\Gamma)}\leq C_\varepsilon | <u_0,\varphi_n> |^2 (\lambda_n+\Lambda)^s$$ and then the serie converges in $C([\varepsilon,T); H^s(\Gamma))$ for all $0<\varepsilon<T<\infty.$ The Laplace equation on the bulk is trivially satisfied for the construction of $ \psi_n.$ For $\alpha=1$, it is well-known that $E_{1,1}(z) = e^{-z}$ and then the solution $u$ takes the form \begin{align} u(t, x) =\sum_{n=1}^\infty <u_0,\varphi_n> e^{-t \lambda_n}\psi_n(x) \label{uUno} \end{align} which is the solution as it has been proved in Theorem 3 of \cite{VV1}. \end{proof} \begin{remark} Observe that the spectral condition is not trivial. We point out that, for example, the condition $\lambda_1\geq 0$ is satisfied when $\Omega$ is an Euclidean ball with radius $R$ in $\R^N$ if $k \leq l R (N-1)$ (see Theorem 6 in \cite{VV1}). \label{rmk:IMP} \end{remark} Recall that \begin{align*} -A_\Lambda = k A_{DN} + l \Delta_\Gamma - \Lambda I, \quad k \in \mathbb{R},\, l> 0,\, \Lambda > 0 \end{align*} is self-adjoint, positive operator (see \cite[proof of Theorem 2]{VV1}). By using the one to one correspondence between the non-positive self-adjoint operator $-A_\Lambda$ and the family of closed symmetric form (see Theorem 1.3.1 in \cite{FUK}) we consider the closed symmetric form $\mathcal{E} $ defined as \begin{align} \mathcal{E}(u,v)=(\sqrt{A_\Lambda u}, \sqrt{A_\Lambda v})\end{align} with domain $D(\mathcal{E}) =D(\sqrt{A_\Lambda u})$. In particular, by using \eqref{M1} and \eqref{M2} we obtain \begin{align} \mathcal{E}(u,v) = - k \int_\Omega\nabla (\D u)\nabla (\D v) \,d\mu+ l \int _\Gamma \nabla_\Gamma u\, \nabla_\Gamma v \,ds + \Lambda \int _\Gamma uv\,ds \label{DFprocess}. \end{align} For $k \leq 0$ we get a Dirichlet form with Markovian semigroups and resolvents (see, for definitions and properties, see \cite{FUK}). We remark that the previous form can be associated to the so-called Venttsel' or Wentzell boundary condition (see for example pag 26 in \cite{{GWbook}}). We point out that recently fractional-in-time Venttsel' problems in irregular domains has been studied in \cite{CCL} by proving well-posedness, regularity and asymptotic results. \\ In the next section we consider the probabilistic representation of the solution. \section{Probabilistic representation} We introduce briefly the processes we deal with further on. As usual, we denote by $\mathbf{E}_x$ the mean operator under the measure $\mathbf{P}_x$ where $x$ is a starting point: \begin{itemize} \item[i)] $X^+= \{X^+_t\}_{t\geq 0}$ is a reflecting Brownian motion on $\overline{\Omega}$ with boundary local time $\gamma^+ = \{\gamma^+_t\}_{t\geq 0}$. The process $X^+$ has generator $(G^+, D(G^+))$ where $G^+=\Delta$ is the Neumann Laplacian with \begin{align*} D(G^+)= \{\varphi, \Delta \varphi \in C(\overline{\Omega}),\, \varphi \in H^1(\Omega)\,:\, \partial_{\bf n} \varphi = 0\}; \end{align*} \item[ii)] $H=\{H_t\}_{t\geq 0}$ is a subordinator with $\mathbf{E}_0[\exp(-\lambda H_t)] = \exp(-t \Phi(\lambda))$, $\lambda >0$; \item[iii)] $L=\{L_t\}_{t\geq 0}$ is the inverse $L_t= \inf\{s \geq 0\,:\, H_s >t\}$ to $H$. \end{itemize} Moreover, we introduce the first hitting time \begin{align*} \tau := \inf\{t\,:\, X^+_t \in \Gamma\}. \end{align*} Notice that we may equivalently consider the first hitting time of a Brownian motion on $\mathbb{R}^d$ started at $x \in \Omega$. We underline that $X^+$ will be a reference (base) process to start with all over the paper. The symbol \begin{align} \label{symbPhi} \Phi(\lambda) = \int_0^\infty \left( 1 - e^{-\lambda t} \right)\Pi(dt), \quad \lambda > 0 \end{align} is the Bernstein function (or L\'{e}vy symbol) characterizing the subordinator $H$. We also introduce the Caputo-D\v{z}rba\v{s}jan type derivative defined as the convolution operator \begin{align*} D^\Phi_t \varphi(t) = (\varphi^\prime * \kappa)(t), \quad \alpha \in (0,1) \end{align*} involving the singular kernel $\kappa$ for which \begin{align*} \int_0^\infty e^{-\lambda t} \kappa(t)dt = \frac{\Phi(\lambda)}{\lambda}, \quad \lambda>0. \end{align*} It is well-known that $\kappa$ can be identified as the tail of a L\'{e}vy measure $\Pi$ in \eqref{symbPhi}. In particular, $\kappa(t) = \Pi(t, \infty)$. We point out that for $\Phi(\lambda)=\lambda^\alpha$ with $\alpha \in (0,1)$, that is the case of stable subordinators, $D^\Phi_t = D_t^{\alpha}$ introduced in \eqref{dc}. For a discussion on $H$ and $L$ we refer to the book \cite{Ber99}. Here we only provide the result in Theorem \ref{thm:eqM} below for the sake of completeness. Our discussion in this section will focus on the following facts. The well-known Dynkin's formula \begin{align*} \mathbf{E}_x[u(X^+_\tau)] = u(x) + \mathbf{E}_x[\int_0^\tau \Delta u(X^+_s) ds] \end{align*} holds in case $\Omega$ is a transient domain (that is $\mathbf{P}_x(\tau < \infty) = 1$ for every $x \in \Omega$) with $\mathbf{E}_x[\tau] < \infty$ for every $x \in \Omega$. Moreover, we ask for $u$ to be continuous and bounded on $\overline{\Omega}$. If $u$ is harmonic, that is $\Delta u=0$ in $\Omega$, then (also neglecting the condition $\mathbf{E}_x[\tau] < \infty$) the Dynkin's formula writes $\mathbf{E}_x[u(X^+_\tau)] = u(x)$ where $\mathbf{P}_x(X^+_\tau \in dy)$ plays the role of harmonic measure (or Poisson kernel). In particular, $u$ must satisfy the mean value property in $\Omega$ and this implies that $u \in C^\infty(\overline{\Omega})$. If $\Omega$ is a regular (that is $\mathbf{P}_x(\tau =0)=1$ for every $x \in \Gamma$) and transient domain, then every bounded and continuous function on $\Gamma$ has a unique, bounded, harmonic extension. We recall that every bounded domain is transient. In our case, in which $\Gamma$ is a smooth boundary, the harmonic measure is absolutely continuous with respect to the surface measure and the Poisson kernel can be explicitly calculated depending on the dimension $N$. It worth noticing what follows. For $N\geq 3$ the Brownian motion is transient. For $N=2$ the Brownian motion visits a given point with probability zero whereas it visits any neighbourhood of that point with probability one. For $N=1$ the Brownian motion visits a point infinitely many times and the corresponding set of hitting times is a perfect set (a large and uncountable set of zero Lebesgue measure, there are no isolated points). We only consider $\Omega \subset \mathbb{R}^N$ with $N\geq 2$. \begin{theorem} Let us write $M_\Phi(t, \theta) = \mathbf{E}_0[\exp(-\theta L_t)]$, $t>0$, $\theta>0$. Then, $\varphi(t) = M^0_\Phi(t, \theta)$ is the unique solution to \begin{equation*} \left\lbrace \begin{array}{ll} \displaystyle D^\Phi_t \, \varphi(t) = - \theta\, \varphi(t), & t>0,\\ \displaystyle \varphi(0)=1. \end{array} \right. \end{equation*} \label{thm:eqM} \end{theorem} \begin{proof} The proof follows by standard arguments. Since $D^\Phi_t$ is a convolution-type operator, we get \begin{align*} \int_0^\infty e^{-\lambda t} D^\Phi_t \varphi\, dt = & \left( \int_0^\infty e^{-\lambda t} \kappa(t)dt \right) \left( \int_0^\infty e^{-\lambda t} \varphi^\prime(t)\, dt \right)\\ = & \frac{\Phi(\lambda)}{\lambda}(\lambda \tilde{\varphi}(\lambda) - 1), \quad \lambda>0. \end{align*} Our problem leads to \begin{align*} \Phi(\lambda)\, \tilde{\varphi}(\lambda) + \theta \tilde{\varphi}(\lambda) = \frac{\Phi(\lambda)}{\lambda} \quad \textrm{from which} \quad \tilde{\varphi}(\lambda) = \frac{\Phi(\lambda)}{\lambda} \frac{1}{\theta + \Phi(\lambda)}. \end{align*} We immediately see that \begin{align*} \tilde{\varphi}(\lambda) = \int_0^\infty e^{-\theta s} \frac{\Phi(\lambda)}{\lambda} e^{-s \Phi(\lambda)} ds = \int_0^\infty e^{-\lambda t} \int_0^\infty e^{-\theta s} \mathbf{P}_0(L_t \in ds) dt \end{align*} where the last identity follows from $\mathbf{P}_0(L_t < s) = \mathbf{P}_0(H_s > t)$. Thus, $\varphi(t) = \mathbf{E}_0 [\exp(-\theta L_t)]$ which is continuous on $[0, \infty)$ and this implies uniqueness of the inverse Laplace transform. \end{proof} Further on we consider $\Phi(\lambda)=\lambda^\alpha$ with $\alpha \in (0,1).$ Only our last result is concerned with a general $\Phi$. Moreover, we underline that \begin{align} -A_\Lambda = kA_{DN} + l\Delta_\Gamma - \Lambda I \quad \textrm{in case} \quad k \leq 0, \; l \geq 0, \; \Lambda\geq 0 \label{Agen} \end{align} with \begin{align*} D(-A_\Lambda) \subset C(\Gamma) \cap D(A^{s/2}_\Lambda) \end{align*} plays a special role. In particular, $(-A_0, D(-A_0))$ where \begin{align*} -A_0 = kA_{DN} + l\Delta_\Gamma \quad \textrm{with} \quad k<0,\; l \geq 0 \end{align*} generates a Markov process on $\Gamma$ with right-continuous paths. The negative Dirichlet-Neumann operator $-A_{DN}$ is a non-local operator introducing jumps and the Laplace-Beltrami operator $\Delta_\Gamma$ gives the diffusive part. \\ Let us denote by $X^\Gamma=\{X^\Gamma_t\}_{t\geq 0}$ the process on $\Gamma$ with generator $(-A_\Lambda, D(-A_\Lambda))$. Since $\Omega$ is smooth, then there exist a unique linear and bounded trace operator $T: H^1(\Omega) \to L^2(\Gamma)$ such that $Tu= u|_\Gamma$. Consider the Dirichlet form \begin{align} \mathcal{E}(Tu,Tv) := \int_\Omega \nabla u \nabla v\, dx, \quad u,v \in F := \{\varphi \in H^1(\Omega)\,:\, \Delta \varphi = 0\} \label{DFDN} \end{align} with $D(\mathcal{E}) := \{ T\varphi\,:\, \varphi \in H^1(\Omega) \}$. Thus, $D(\mathcal{E}) = H^{1/2}(\Gamma)$ is the trace space. There exists a Markov process on $\Gamma$ associated with $(\mathcal{E}, D(\mathcal{E}))$, see \cite[Proposition 3.1]{BS}. In particular, for $l=0$ and $\Lambda=0$, we say that $X^\Gamma$ is the process on $\Gamma$ associated with the Dirichlet form $(\mathcal{E}, D(\mathcal{E}))$ given in \eqref{DFDN} for $k<0$. It is therefore a pure jump process with generator $(-A_{DN}, D(-A_{DN}))$ up to some scaling introduced by $|k|$. The operator $\Delta_\Gamma$ is associated with a Brownian diffusion on the compact manifold $\Gamma$. It is well-known that we have a Markov semigroup with compact representation. The conservative part introduced by $\Lambda>0$ can be treated in a standard way. For $k, l, \Lambda$ as specified in \eqref{Agen}, the Dirichlet form \eqref{DFprocess} ensures existence of a stochastic process associated with $(-A_\Lambda, D(-A_\Lambda))$. Moreover, from the theory of Dirichlet form, we have a Markov semigroup. We now underline a connection with time-changed semigroups. We recall that, for $k \in \mathbb{R}$ and $l>0$, the problem \begin{align} \left\lbrace \begin{array}{ll} \displaystyle \Delta w =0 \qquad &\text{in $(0,\infty)\times\Omega$,}\\ \dot{w} = k \partial_{\bf n} w +l\Delta_\Gamma w \qquad &\text{on $(0,\infty)\times \Gamma$,}\\ w(0,x)=u_0(x) & \text{on $ \Gamma$,} \end{array} \right . \label{EsolVVw} \end{align} has been investigated in \cite{VV1} where they obtained an analytic semigroup with compact representation. We restrict our analysis in case of non negative eigenvalues of $-A_0$. \begin{theorem} Let $\Lambda=0$, $l > 0$, $k \in \mathbb{R}$ and $\alpha \in (0,1]$. Let assume $\lambda_1\geq 0.$ Let $w$ be the solution to \eqref{EsolVVw}. Then, the solution \eqref{uAlfa} can be written as \begin{align*} u(t,x) = \int_0^\infty w(s,x) \mathbf{P}_0(L_t \in ds) \end{align*} where \begin{align} \int_0^\infty e^{-\lambda t} \mathbf{P}_0(L_t \in ds) = E_\alpha(-\lambda t^\alpha), \quad t \geq 0,\; \lambda \geq 0. \label{ContDensL} \end{align} \label{thm:SOLw} \end{theorem} \begin{proof} It is well-known that \eqref{ContDensL} hold true (see \cite{Bingham71}). Indeed, $L$ is an inverse to a stable subordinator. Moreover, \begin{align*} E_\alpha(-\lambda t^\alpha) = \sum_{k \geq 0} \frac{(-\lambda t^\alpha)^k}{\Gamma(\alpha k + 1)} \quad \textrm{becomes} \quad E_1(-\lambda t) = e^{-\lambda t} \end{align*} for $\alpha=1$. From Theorem \ref{th2}, for $\alpha=1$, the solution $u$ takes the form \begin{align*} u(t, x) = \sum_{n=1}^\infty <u_0,\varphi_n> e^{-t \lambda_n}\psi_n(x) \end{align*} which can be associated with the fact that $L_t \to t$ a.s. as $\alpha \to 1$ (see \cite{Ber99}). Thus, we conclude that, $\forall\, T>0$, pointwise on $(0, T) \times \overline{\Omega}$, \begin{align*} u = w \quad \textrm{for } \alpha=1 \end{align*} Now we consider $\alpha \in (0,1)$.\\ The previous result says that \begin{align*} u(t, x) = & \sum_{n=1}^\infty <u_0,\varphi_n> E_{\alpha,1}(-t^{\alpha}\lambda_n)\psi_n(x)\\ = & \sum_{n=1}^\infty <u_0,\varphi_n> \left( \int_0^\infty e^{- s \lambda_n} \mathbf{P}_0(L_t \in ds) \right) \psi_n(x)\\ = & \int_0^\infty \left( \sum_{n=1}^\infty <u_0,\varphi_n> e^{- s \lambda_n} \psi_n(x) \right) \mathbf{P}_0(L_t \in ds)\\ = & \int_0^\infty w(s,x) \mathbf{P}_0(L_t \in ds) \end{align*} which is the claim. \end{proof} The last result says that the solution $u$ is written in terms of a time-changed process. This is not obvious and will be associated below with the fact that the behaviour of $X^\Gamma$ on the boundary is not affected from the behaviour on the interior. More precisely, we now provide the probabilistic interpretation of the Laplace equation with fractional dynamical boundary condition. We recall that, given a random time $T$ (or a stochastic process $T=\{T_t\}_{t\geq 0}$) and a process $Z = \{Z_t\}_{t\geq 0}$, we analogously write $Z_T$ or $Z \circ T$. Moreover we introduce the process $X^{0, \Gamma} = \{X^{0, \Gamma}_t\}_{t\geq 0}$ with generator $(-A_0, D(-A_0))$. In particular, we underline that $X^\Gamma = X^{0, \Gamma}$ in case $\Lambda=0$. \begin{theorem} \label{th3} Let $\Lambda=0$, $l \geq 0$, $k <0$ and $\alpha \in (0,1]$. The solution to \eqref{ELambda} has the probabilistic representation \begin{align} u(t, x) = \mathbf{E}_x[\mathbf{E}_{X^+_\tau}[u_0(X^{0, \Gamma}\circ L_t)]], \quad t>0, \; x \in \overline{\Omega}. \label{ProbRepSol} \end{align} \end{theorem} \begin{proof} Let us consider the solution $u: (0, \infty)\times \overline{\Omega} \to \mathbb{R}$ to the problem \begin{equation} \left\lbrace \begin{array}{ll} \displaystyle \Delta u = 0, & \textrm{in } (0, \infty) \times \Omega,\\ \displaystyle Tu = v, & \textrm{on } (0, \infty) \times \Gamma. \end{array} \right . \label{eqProofHarm} \end{equation} Then $\forall\, t \in (0, \infty)$, the solution $u(t, \cdot)$ has the probabilistic representation \begin{align} u(t,x) = \mathbf{E}_x[v(t, X^+_{\tau})], \quad t\geq 0, \; x \in \overline{\Omega}. \label{eq0} \end{align} Now, for $l=0$, we consider the datum $v: (0, \infty)\times \Gamma \to \mathbb{R}$ as the solution to the problem \begin{equation} \left\lbrace \begin{array}{ll} \displaystyle D^\alpha_t v = k A_{DN} v, & (0, \infty) \times \Gamma\\ \displaystyle v_0 = u_0 \in H^s(\Gamma) \end{array} \right. \label{fcpDN} \end{equation} where $-A_{DN}$ is the generator of the right-continuous Markov process $X^{0, \Gamma}$ on $\Gamma$. Thus, for the fractional Cauchy problem \eqref{fcpDN} we have (see for example \cite{CapDovFEforms19}) \begin{align} v(t,x) = \mathbf{E}_x[u_0(X^{0, \Gamma} \circ L_t)], \quad t\geq 0,\; x \in \Gamma. \label{eq2} \end{align} Thus, \begin{align*} u(t,x) \stackrel{\eqref{eq0}}{=} \mathbf{E}_x[v(t, X^+_\tau)] \stackrel{\eqref{eq2}}{=} \mathbf{E}_x\left[\mathbf{E}_{X^+_\tau}[u_0(X^{0, \Gamma}\circ L_t)]\right], \quad t\geq 0,\; x \in \overline{\Omega}. \end{align*} In case $l>0$, by considering the datum $v: (0, \infty)\times \Gamma \to \mathbb{R}$ as the solution to the problem \begin{equation} \left\lbrace \begin{array}{ll} \displaystyle D^\alpha_t v = k A_{DN} v + l\Delta_\Gamma v, \quad (0, \infty) \times \Gamma\\ \displaystyle v_0 = u_0 \in H^s(\Gamma) \end{array} \right. \end{equation} we include also the diffusive part, that is the Brownian motion on $\Gamma$ with generator $(\Delta_\Gamma, D(\Delta_\Gamma))$. Since $k<0$, the solution $u$ to the problem \eqref{E} has the probabilistic representation \begin{align*} u(t,x) = & \mathbf{E}_x\left[\mathbf{E}_{X^+_\tau}[u_0(X^{0, \Gamma} \circ L_t)]\right], \quad t\geq 0,\; x \in \overline{\Omega} \end{align*} where $X^{0, \Gamma}$ on $\Gamma$ is generated by $(-A_0, D(-A_0))$. \end{proof} We now consider the case $\Lambda>0$.\begin{theorem} \label{thm:timeChange} Let $\Lambda>0$, $l \geq 0$, $k <0$ and $\alpha \in (0,1]$. The solution to \eqref{ELambda} has the probabilistic representation \begin{align*} u(t,x) = & \mathbf{E}_x\left[\mathbf{E}_{X^+_\tau}[u_0(X^\Gamma \circ L_t)]\right]\\ = & \mathbf{E}_x\left[e^{-\Lambda L_t} \mathbf{E}_{X^+_\tau}[u_0(X^{0, \Gamma} \circ L_t)]\right], \quad t\geq 0,\; x \in \overline{\Omega}. \end{align*} In particular, \begin{align*} u(t,x) = & \int_0^\infty e^{-\Lambda s} \mathbf{E}_x\left[\mathbf{E}_{X^+_\tau}[u_0(X^{0, \Gamma}_s)]\right] \mathbf{P}_0(L_t \in ds)\\ = & \int_0^\infty e^{-\Lambda s} w(s,x) \mathbf{P}_0(L_t \in ds) \end{align*} where $w$ is the solution to \eqref{EsolVVw}. \end{theorem} \begin{proof} The process $X^\Gamma$ on $\Gamma$ is generated by $(-A_\Lambda, D(-A_\Lambda))$ as defined above and $X^{0,\Gamma}$ on $\Gamma$ is right-continuous and generated by $(-A_0, D(-A_0))$ where $-A_0$ is given by $-A_{DN}$ and $\Delta_\Gamma$ without perturbation. Thus, $X^{0, \Gamma}$ is a Markov process with right-continuous paths for which the Feynmann-Kac formula \begin{align*} v(t,x) = \mathbf{E}_x \left[ e^{-\Lambda t} u_0(X^{0, \Lambda}_t) \right], \quad t >0,\, x \in \Gamma \end{align*} solves the problem to find $v \in C((0, \infty), \Gamma)$ such that \begin{align} \dot{v} = -A_0 v - \Lambda v, \quad v_0 = u_0 \in D(-A_0). \label{CPproof} \end{align} That is, we solve $\dot{v} = -A_\Lambda v$ on $(0, \infty) \times \Gamma$. Thus, as in \eqref{eqProofHarm}, we proceed by considering \begin{align} \mathbf{E}_x[v(t,X^+_\tau)] = e^{-\Lambda t} \mathbf{E}_x\left[\mathbf{E}_{X^+_\tau}[u_0(X^{0, \Gamma}_t)]\right] = e^{-\Lambda t} w(t,x), \quad t\geq 0,\; x \in \overline{\Omega} \label{TEMPw} \end{align} in order to obtain a solution to \eqref{ELambda} with $\alpha=1$. The last equality is justified by the fact that $A_\Lambda$ is positive definite, then Theorem \ref{thm:SOLw} applies. The solution \eqref{TEMPw} coincides with $w$ as $\Lambda=0$. For $\alpha \in (0,1)$, the fractional Cauchy problem associated with \eqref{CPproof} has a solution obtained via time-changed semigroup. According with the proof of Theorem \ref{th3} we can write \begin{align*} u(t,x) = & \mathbf{E}_x[v(L_t, X^+_\tau)]\\ = & \int_0^\infty e^{-\Lambda s} w(s,x) \mathbf{P}_0(L_t \in ds)\\ = & \mathbf{E}_x\left[e^{-\Lambda L_t} \mathbf{E}_{X^+_\tau}[u_0(X^{0,\Gamma} \circ L_t)]\right]\\ = & \mathbf{E}_x\left[\mathbf{E}_{X^+_\tau}[u_0(X^\Gamma \circ L_t)]\right] \quad t\geq 0,\; x \in \overline{\Omega}. \end{align*} This concludes the proof. \end{proof} \section{General non local conditions } The fractional problem \eqref{E} can be regarded as special case of the non-local problem \begin{align} \label{ENL} \left\lbrace \begin{array}{ll} \displaystyle \Delta u=0 \qquad &\text{in $(0,\infty)\times\Omega$,}\\ D_t^{\Phi} u = k \partial_{\bf n} u +l\Delta_\Gamma u - \Lambda u \qquad &\text{on $(0,\infty)\times \Gamma$,}\\ u(0,x)=u_0(x) & \text{on $ \Gamma$.} \end{array} \right . \end{align} For the process $L$ with symbol $\Phi$ we introduce the function \begin{align*} M^\Lambda_\Phi(t, \lambda) : = \mathbf{E}_0[\exp - (\Lambda + \lambda) L_t], \quad (\lambda + \Lambda) \geq 0,\; \Lambda > 0,\; t>0. \end{align*} Also for this problem we provide the compact representation of the solution to \eqref{ENL} which is strictly related with Theorem \ref{th2}. \begin{theorem} \label{th2NL} Let $\Lambda=0$, $l > 0$, $k \in \mathbb{R}$ and $\alpha \in (0,1]$. Let assume $\lambda_1 + \Lambda \geq 0.$ Then for any $u_0 \in H^{s+2}( \Gamma)$ the solution $u$ of problem \eqref{ENL} is given by \begin{align} u(t, x) =\sum_{n=1}^\infty <u_0,\varphi_n> M^\Lambda_\Phi(t, \lambda_n)\psi_n(x), \quad t>0, \; x \in \overline{\Omega} \label{uPhi} \end{align} the series being convergent in $C((0,\infty) \times \bar\Omega)$. \end{theorem} \begin{proof} First we observe that $\lambda_n + \Lambda \geq 0$ implies that the \eqref{uPhi} writes \begin{align} \label{eqPROOFuLambda} u(t,x) = \sum_{n=1}^\infty <u_0,\varphi_n> \left( \int_0^\infty e^{-(\lambda_n + \Lambda) s} \mathbf{P}_0(L_t \in ds) \right) \psi_n(x), \quad t>0, \; x \in \overline{\Omega} \end{align} and, for a given $\Lambda>0$, $M^\Lambda_\Phi(t, \lambda_1) \leq M^\Lambda_\Phi(t, \lambda_n)$ $\forall\, n$. We can use the same arguments as in the proof of Theorem \ref{th2} together with the following facts: as for $u_0 \in H^{s+2}(\Omega)$ \begin{align} ||| D^\Phi_t u(t,x) |||_{H^s(\Gamma)} \leq \sqrt{\sum_{n=1}^\infty |<u_0,\varphi_n>|^2 |\lambda_n + \Lambda|^{2+s} } < \infty \label{fact1} \end{align} and \begin{align} ||| -A_\Lambda u |||_{H^s(\Gamma)} \leq ||| A_0 u + \Lambda u |||_{H^s(\Gamma)} \leq \sqrt{\sum_{n=1}^\infty |<u_0,\varphi_n>|^2 |\lambda_n + \Lambda|^{2+s} }< \infty \label{fact2} \end{align} By replacing \eqref{eqML} with \begin{align} D^\Phi_t M^\Lambda_\Phi(t, \lambda) = - (\lambda + \Lambda) M^\Lambda_\Phi(t, \lambda), \quad M^\Lambda_\Phi(0, \lambda) = 1 \label{eqM} \end{align} we can show that $u$ solves \eqref{ENL}. The equation \eqref{eqM} can be obtained from Theorem \ref{thm:eqM}. Moreover, the series \eqref{uPhi} converges in $L^2(\overline{\Omega})$ uniformly in $(0, \infty)$ and, from the monotonicity of $M^\Lambda_\Phi (t,\lambda)$, \begin{align*} \sum_{n=1}^{\infty} || <u_0,\varphi_n> M^\Lambda_\Phi(t, \lambda_n) \varphi_n(x)||^2_{C([0,\infty), H^s(\Gamma))}< \infty \end{align*} implies that \eqref{uPhi} converges in $C((0, \infty)\times \overline{\Omega})$. \end{proof} We point out that the probabilistic representation of \eqref{uPhi} can be obtained by following the same arguments as in Section 3.\\ We conclude by noting that this result can be useful to model delayed and rushed motions through time change (see, for definitions and examples, \cite{CapDovDelRus17}). \vspace{1cm} {\bf Acknowledgments} The authors are supported by INdAM-GNAMPA and Sapienza University of Rome (Ateneo 2021 and Ateneo 2022). The authors thank MUR for the support under the project PRIN 2022 - 2022XZSAFN: Anomalous Phenomena on Regular and Irregular Domains: Approximating Complexity for the Applied Sciences - CUP B53D23009540006 (\url{https://www.sbai.uniroma1.it/~mirko.dovidio/prinSite/index.html}). \begin{thebibliography}{99} \bibitem{Ber99} J. Bertoin, Subordinators: Examples and Applications. In: Bernard, P. (eds) Lectures on Probability Theory and Statistics. Lecture Notes in Mathematics, vol 1717. Springer, Berlin, Heidelberg (1999). \bibitem{BS} L. Beznea, S. Vl\u{a}doiu. Markov processes on the Lipschitz boundary for the Neumann and Robin problems. \textit{J. Math. Anal. Appl.}, 455(1), (2017), 292--311. \bibitem{Bingham71} N. H. Bingham, Limit theorems for occupation times of markov processes. \textit{Zeitschrift fur Wahrscheinlichkeitstheorie und verwandte Gebiete} {\bf 17} (1071), 1--22. \bibitem{CCL} R. Capitanelli, S. Creo, M. R. Lancia, Asymptotics for time-fractional Venttsel' problems in fractal domains. \textit{Fractal Fract. }(2023), 7(6), 479. \bibitem{CapDovFEforms19} R. Capitanelli, M. D'Ovidio, Fractional equations via convergence of forms. \textit{Fract. Calc. Appl. Anal.} {\bf 22} (2019), 844 - 870. \bibitem{CapDovDelRus17} R. Capitanelli, M. D'Ovidio, Delayed and Rushed motions through time change. \textit{ALEA, Lat. Am. J. Probab. Math. Stat.} {\bf 17} (2020), 183--204. \bibitem{CapDovFCP21} R. Capitanelli, M. D'Ovidio, Fractional Cauchy problem on random snowflakes. \textit{Journal of Evolution Equations} {\bf 21} (2021), 2123--2140. \bibitem{caputoBook} M. Caputo., Elasticit\`{a} e dissipazione. Zanichelli, Bologna (1969). \bibitem{CapMai71} M. Caputo, F. Mainardi, Linear models of dissipation in anelastic solids. La Rivista del Nuovo Cimento \textbf{1}, 161--198 (1971). \bibitem{CapMai71b} M. Caputo, F. Mainardi, A new dissipation model based on memory mechanism. PAGEOPH \textbf{91}, 134--147 (1971). \bibitem{Chen17} {Z.-Q. Chen,} {Time fractional equations and probabilistic representation,} \textit{Chaos, Solitons \& Fractals} {\bf 102} (2017) 168-174. \bibitem{CF} Z.-Q. Chen, M. Fukushima. Symmetric Markov processes, time change, and boundary theory. London Mathematical Society Monographs Series. Princeton University Press, Princeton, NJ, 2012. \bibitem{Dov22fcaa} M. D'Ovidio, Fractional Boundary Value Problems. \textit{Fract. Calc. Appl. Anal.} {\bf 1} 25, (2022), 29 -- 59. \bibitem{Dov24fcaa} M. D'Ovidio, Fractional Boundary Value Problems and Elastic Sticky Brownian Motions. \textit{Fract. Calc. Appl. Anal.} {\bf 27} (2024) 2162--2202. \bibitem{Dov24II} M. D'Ovidio, Fractional Boundary Value Problems and Elastic Sticky Brownian Motions, II: The bounded domain. \textit{Submitted} (2024) arXiv:2205.04162. \bibitem{Dzh66} M. M. D\v{z}rba\v{s}jan, M. M., Integral Transforms and Representations of Functions in the Complex Plane. Nauka, Moscow (1966) (in Russian). \bibitem{DzhNers68} M. M. D\v{z}rba\v{s}jan, A. B. Nersessian, A. B. Fractional derivatives and the Cauchy problem for differential equations of fractional order. Izv. Akad. Nauk Armjan. SSR. Ser. Mat. \textbf{3}(1),(1968) (in Russian) 1--29. \bibitem{D} J. Douglas. Solution of the problem of Plateau. \textit{Trans. Amer. Math. Soc}., 33(1) (1931), 263--321. \bibitem{E} J. Escher, The Dirichlet-Neumann operator on continuous functions. \textit{ Ann. Sc. Norm. Super. Pisa Cl. Sci.} (4) 21 (2) (1994), 235--266. \bibitem{FUK} M Fukushima, Y. Oshima, M. Takeda, \emph{Dirichlet forms and symmetric Markov processes}. W. de Gruyter: Berlin, 1994. \bibitem{GM} {C. G. Gal, M. Meyries } Nonlinear elliptic problems with dynamical boundary conditions of reactive and reactive--diffusive type. \textit{Proc. London Math. Soc.} (3) 108 (2014) 1351--1380. \bibitem{GLY15} R. Gorenflo, Y. Luchko, M. Yamamoto, Time-fractional diffusion equation in the fractional Sobolev spaces. \textit{Fract. Calc. Appl. Anal.} \textbf{18}, No 3 (2015), 799--820. \bibitem{GorKilMaiRog} R. Gorenflo, A. A. Kilbas, F. Mainardi, S. V. Rogosin, Mittag-Leffler Functions, Related Topics and Applications, Springer-Verlag Berlin Heidelberg 2014. \bibitem{H} T. Hintermann, Evolution equations with dynamic boundary conditions. \textit{Proc. Roy. Soc. Edinburgh Sect. A} 113 (1-2) (1989) 43--60. \bibitem{Koc2011} {A. N. Kochubei,} {General fractional calculus, evolution equations, and renewal processes.} \textit{Integral Equations and Operator Theory }\textbf{71} (2011), 583--600. \bibitem{JL} J.L. Lions, Quelques m\'ethodes de r\'esolutions des probl\`emes aux limites non lin\'eaires, Dunod, Paris, 1969. \bibitem{LM} J.-L. Lions and E.~Magenes, \emph{Probl\`emes aux limites non homog\`enes et applications. {V}ol. 1}, Travaux et Recherches Math\'ematiques, No. 17, Dunod, Paris, 1968. \bibitem{MLP2001} F. Mainardi, Y. Luchko, G. Pagnini, The fundamental solution of the space-time fractional diffusion equation. \textit{Fract. Calc. Appl. Anal.} \textbf{4}, No 2 (2001), 153--192. \bibitem{MNV09} M. M. Meerschaert, E. Nane, P. Vellaisamy, Fractional Cauchy problems on bounded domains. \textit{Ann. Probab.} \textbf{37}, No 3 (2009), 979--1007. \bibitem{POD} I. Podlubny, Fractional differential equations, Mathematics in Science and Engineering, 198, An introduction to fractional derivatives, fractional differential equations, to methods of their solution and some of their applications, Academic Press, 1999. \bibitem{Toaldo2015} {B. Toaldo.} Convolution-type derivatives, hitting-times of subordinators and time-changed $C_0$-semigroups. \textit{Potential Analysis} \textbf{42} (2015), 115--140. \bibitem{VV1} J.L. Vazquez, E.Vitillaro, On the Laplace equation with dynamical boundary conditions of reactive-diffusive type. \textit{J. Math. Anal. Appl. }{354} (2009), no. 2, 674--688. \bibitem{VV2} J.L. Vazquez, E.Vitillaro, Heat equation with dynamical boundary conditions of reactive-diffusive type. \textit{Journal of Differential Equations} 250 (2011), 2143--2161. \end{thebibliography} \end{document}
2412.04962v1
http://arxiv.org/abs/2412.04962v1
Divisible design graphs from symplectic graphs over rings with precisely three ideals
\documentclass[10pt]{elsarticle} \usepackage[cp1251]{inputenc} \usepackage[english]{babel} \usepackage{comment} \usepackage{amsmath} \usepackage{amssymb} \usepackage{amsfonts} \usepackage{amsthm} \usepackage{mathtools} \usepackage{dsfont} \usepackage{rotating} \usepackage{graphicx} \usepackage{floatflt,epsfig} \usepackage{lineno,hyperref} \usepackage{enumerate} \usepackage{colortbl} \usepackage{enumitem} \usepackage{array,tabularx,tabulary,booktabs} \usepackage{longtable} \usepackage{multirow} \usepackage{wrapfig} \usepackage{subcaption} \usepackage{pdflscape} \usepackage[table]{xcolor} \usepackage[left=1in,right=1in,top=1in,bottom=1in]{geometry} \newcolumntype{^}{>{\currentrowstyle}} \newcommand{\rowstyle}[1]{ \gdef\currentrowstyle{#1} #1\ignorespaces } \journal{Arxiv} \setcounter{page}{1} \newtheorem{lemma}{Lemma} \newtheorem{mlemma}{Main Lemma} \newtheorem{theorem}{Theorem} \newtheorem{corollary}{Corollary} \newtheorem{proposition}{Proposition} \newtheorem{construction}{Construction} \newtheorem{remark}{Remark} \newtheorem{problem}{Problem} \newtheorem{example}{Example} \newtheorem{conjecture}{Conjecture} \bibliographystyle{elsarticle-num} \DeclareMathOperator{\rank}{rank} \DeclareMathOperator{\textif}{if} \DeclareMathOperator{\otherwise}{otherwise} \newcommand{\F}{\mathbb{F}} \DeclarePairedDelimiter{\lrangle}{\langle}{\rangle} \newcommand{\PG}{\operatorname{PG}} \begin{document} \renewcommand{\abstractname}{Abstract} \renewcommand{\refname}{References} \renewcommand{\tablename}{Table} \renewcommand{\arraystretch}{0.9} \newcommand\qbin[3]{\left[\begin{matrix} #1 \\ #2 \end{matrix} \right]_{#3}} \thispagestyle{empty} \sloppy \begin{frontmatter} \title{Divisible design graphs from symplectic graphs over rings with precisely three ideals} \author[01]{Anwita Bhowmik} \ead{[email protected]} \author[02]{Sergey Goryainov} \ead{[email protected]} \address[01] {Postdoctoral Research Station of Mathematics, School of Mathematical Sciences, Hebei Normal University, Shijiazhuang 050024, P.R. China} \address[02] {School of Mathematical Sciences, Hebei International Joint Research Center for Mathematics and Interdisciplinary Science, Hebei Key Laboratory of Computational Mathematics and Applications, Hebei Workstation for Foreign Academicians,\\Hebei Normal University, Shijiazhuang 050024, P.R. China} \begin{abstract} In this paper we construct two new infinite families of divisible design graphs based on symplectic graphs over rings with precisely three ideals. \end{abstract} \begin{keyword} divisible design graph; symplectic graph; group divisible design; ring; ideal \vspace{\baselineskip} \MSC[2020] 05B05 \sep 51E30 \sep 13H99 \end{keyword} \end{frontmatter} \section{Introduction} A \emph{divisible design graph} (a DDG for short) with parameters $(v,k,\lambda_1,\lambda_2,m,n)$ is a $k$-regular graph on $v$ vertices such that its vertex set can be partitioned into $m$ classes of size $n$ with the following two properties: any two distinct vertices from the same class have precisely $\lambda_1$ common neighbours and any two vertices from different classes have precisely $\lambda_2$ common neighbours. The partition from the definition of a DDG is called the \emph{canonical partition}. DDGs were introduced in \cite{M08} and \cite{HKM11} as a bridge between graph theory and design theory (the adjacency matrix of a divisible design graph can be regarded as the incidence matrix of a group divisible design \cite{B77}) and have been studied in \cite{CH14}, \cite{GHKS19}, \cite{KS21}, \cite{S21}, \cite{K22}, \cite{CS22}, \cite{PS22}, \cite{P22}, \cite{T22} \cite{K23}, \cite{GK24} and \cite{DGHS24}. DDGs can be viewed as an important subclass of Deza graphs \cite{EFHHH99}. The bibliography on strictly Deza graphs and DDGs, and databases of small examples can be found in \cite{P}. The family of symplectic strongly regular graphs \cite[Section 2.5]{BV22} is an important example of strongly regular polar graphs. Note that the vertex set of a symplectic graph is the set of 1-dimensional subspaces of a $2e$-dimensional vector space over a finite field for some integer $e \ge 2$. In the last two decades, there have been studies of the analogues of the strongly regular polar graphs (including the analogues of symplectic polar graphs) with the vector space to be replaced by a module over a finite ring (see \cite{MP11}, \cite{LWG13}, \cite{MP13}, \cite{G13}, \cite{GWZ13}, \cite{GLW14}, \cite{LGW14} for Deza graphs that can be constructed in this way). We also refer to \cite{LW08} and \cite{GLW10} for constructions of Deza graphs (the latter one generalises the former one) based on symplectic graphs over finite fields. In this paper, we work with the same setting of a symplectic space over a ring as given in \cite{MP13} and consider a certain class of rings. Throughout this paper, let $K$ be a finite commutative ring with identity, having precisely three ideals: the zero ideal, $K$ itself and $J=\langle r\rangle$. Without loss of generality we may assume that $|K|=q^2$, $|J|=q$ and $K/J \cong \mathbb{F}_q$, where $q$ is a prime power and $\mathbb{F}_q$ is the finite field of order $q$ (see Section \ref{sec:Prelim} for a background on rings). Let $K^\times$ denote the set of units in $K$. Let $e\geq 2$ be an integer. Let $V'=\{(a_1,a_2,\ldots, a_{2e}): a_1,a_2,\ldots,a_{2e}\in K \text{ and } a_j\in K^\times\text{ for some }j\in \{1,2,\ldots, 2e\}\}$. We define an equivalence relation $\sim$ on $V'$ as: $(a_1,a_2,\ldots,a_{2e})\sim (b_1,b_2,\ldots,b_{2e})$ if and only if there exists $\lambda\in K^\times$ such that $a_j=\lambda b_j, 1\leq j\leq 2e$. Let $V$ denote the set of equivalence classes on $V'$ corresponding to the equivalence relation $\sim$. We denote by $[a_1,a_2,\ldots, a_{2e}]$ the equivalence class of $(a_1,a_2,\ldots,a_{2e})$. Let $M$ be the $2e\times 2e$ non-singular matrix with entries in $K$, given by \begin{align*} M=\begin{bmatrix} 0 & I_e\\ -I_e & 0\\ \end{bmatrix} \end{align*} where $I_e$ is the $e\times e$ identity matrix. We define two graphs $X(2e,K)$ and $Y(2e,K)$, both with vertex set $V$: \begin{itemize} \item in the graph $X(2e,K)$, a vertex $[a_1,a_2,\ldots,a_{2e}]$ is adjacent to a vertex $[b_1,b_2,\ldots,b_{2e}]$ if and only if $(a_1,a_2,\ldots,a_{2e})M(b_1,b_2,\ldots,b_{2e})^t$ belongs to $K \setminus \{0\}$; \item in the graph $Y(2e,K)$, a vertex $[a_1,a_2,\ldots,a_{2e}]$ is adjacent to a vertex $[b_1,b_2,\ldots,b_{2e}]$ if and only if $(a_1,a_2,\ldots,a_{2e})M(b_1,b_2,\ldots,b_{2e})^t$ belongs to $J \setminus \{0\}$. \end{itemize} Clearly, $Y(2e,K)$ is a spanning subgraph of $X(2e,K)$. \medskip The main results of this paper are the following two theorems. \begin{theorem}\label{thm:Main1}Let $e\geq 2$ be an integer. Let $K$ be a finite commutative ring with identity having precisely three ideals: $\langle 0\rangle$, $J$ and $K$. Let $K/J\cong \mathbb{F}_q$, where $\mathbb{F}_q$ is a finite field of order $q$ for a prime power $q$. Then the graph $X(2e,K)$ is a divisible design graph with parameters $(v,k,\lambda_1,\lambda_2,m,n)$ where \begin{align*} v&=\dfrac{q^{2e-1}(q^{2e}-1)}{q-1}, ~~k=q^{4e-2}+q^{4e-3}-q^{2e-2}, ~~\lambda_1=q^{4e-2}+q^{4e-3}-q^{4e-4}-q^{2e-2},\\ \lambda_2&=q^{4e-2}+q^{4e-3}-q^{4e-4}-q^{4e-5}-q^{2e-2}+q^{2e-3},~~ m=\dfrac{q^{2e}-1}{q-1},~~ n=q^{2e-1}. \end{align*} \end{theorem} \begin{theorem}\label{thm:Main2} Let $e\geq 2$ be an integer. Let $K$ be a finite commutative ring with identity having precisely three ideals: $\langle 0\rangle$, $J$ and $K$. Let $K/J\cong \mathbb{F}_q$, where $\mathbb{F}_q$ is a finite field of order $q$ for a prime power $q$. Then the graph $Y(2e,K)$ is a divisible design graph with parameters $(v,k,\lambda_1,\lambda_2,m,n)$ where \begin{align*} v&=\dfrac{q^{2e-1}(q^{2e}-1)}{q-1}, ~~k=q^{4e-3}-q^{2e-2}, ~~\lambda_1=q^{4e-3}-q^{4e-4}-q^{2e-2},\\ \lambda_2&=q^{4e-4}-q^{4e-5}-q^{2e-2}+q^{2e-3}, ~~m=\dfrac{q^{2e}-1}{q-1}, ~~n=q^{2e-1}. \end{align*} \end{theorem} \begin{example} Let $p$ be a positive integer that is a prime. Then the ring $K_1 = \mathbb{Z}/p^2\mathbb{Z}$ has precisely three ideals, where $|K_1| = p^2$ and the non-trivial ideal $J$ is given by the elements divisible by $p$ (if we represent $K_1$ as the set of remainders modulo $p^2$). \end{example} \begin{example} Let $q$ be a positive integer that is a prime power and $\mathbb{F}_q[x]$ be the ring of polynomials of finite degree over the finite field $\mathbb{F}_q$. Then the quotient ring $K_2 = \mathbb{F}_q[x]/ \langle x^2\rangle$ has precisely three ideals, where $|K_2| = q^2$ and the non-trivial ideal $J$ is given by all the elements divisible by $x$ (if we represent $K_2$ as the set of remainders modulo $x^2$). \end{example} Note that if $q = p$, then the rings $K_1$ and $K_2$ have the same order, but are not isomorphic (as they have different characteristic); in this case the graphs $X(2e,K_1)$ and $X(2e,K_2)$ have the same parameters and the graphs $Y(2e,K_1)$ and $Y(2e,K_2)$ also have the same parameters. We have verified computationally that for $q = p = 2$ the graphs $X(4,K_1)$ and $X(4,K_2)$ are not isomorphic and the graphs $Y(4,K_1)$ and $Y(4,K_2)$ are also not isomorphic. We do not know any example of a finite commutative ring with identity, having precisely three ideals and being not isomorphic to $K_1$ or $K_2$. In particular, the classification of small local rings \cite{N18} does not give such examples. The paper is organised as follows. In Section \ref{sec:Prelim} we give some preliminary definitions and results. In section \ref{sec:Proofs}, we give proofs of Theorem \ref{thm:Main1} and Theorem \ref{thm:Main2} in a series of lemmas and propositions. In Section \ref{sec:Remarks}, we give some concluding remarks. \section{Preliminaries}\label{sec:Prelim} In this section, we give preliminary definitions and results from ring theory. A \emph{ring} $R$ is a set together with two binary operations $+$ and $\cdot$ (called \emph{addition} and \emph{multiplication}) satisfying the following axioms: \begin{itemize} \item $(R,+)$ is an abelian group (with additive identity denoted by $0$), \item the operation $\cdot$ is associative: $(a\cdot b)\cdot c=a\cdot (b\cdot c)$ for all $a,b,c\in R$, and \item the distributive laws hold in $R$: for all $a,b,c\in R$, \begin{align*} (a+b)\cdot c=a\cdot c+b\cdot c\text{ and }a\cdot (b+c)=a\cdot b+a\cdot c. \end{align*} \end{itemize} We write $ab$ instead of $a\cdot b$ as shorthand. A ring $R$ is called \emph{commutative} if multiplication is commutative, that is, $ab=ba$ for all $a,b\in R$. A ring $R$ is said to have an \emph{identity} if there is an element $1\in R$ with $1\cdot a=a\cdot 1=a$ for all $a\in R$. Let $R$ be a ring with identity $1\neq 0$. An element $a$ of $R$ is called a \emph{unit} (or $a$ is said to be \emph{invertible}) in $R$ if there is some $b$ in $R$ such that $ab=ba=1$. The set of units in $R$ is denoted by $R^\times$. A nonzero element $a\in R$ is called a \emph{zero divisor} if there is a nonzero element $b\in R$ such that either $ab=0$ or $ba=0$. A \emph{subring} of the ring $R$ is a subset of $R$ that is itself a ring when binary operations of addition and multiplication on $R$ are restricted to the subset. Let $R$ and $S$ be rings. A \emph{ring homomorphism} is a map $\phi:R\rightarrow S$ satisfying \begin{itemize} \item $\phi(a+b)=\phi(a)+\phi(b)$ and \item $\phi(ab)=\phi(a)\phi(b)$ \end{itemize} for all $a,b\in R$. A bijective ring homomorphism is called an \emph{isomorphism}. \begin{proposition}[{\cite[Exercise 17, p. 231]{DF04}}] Let $R$ and $S$ be rings. Then the direct product $R\times S$ is a ring under componentwise addition and multiplication. \end{proposition} Now, let $R$ be a commutative ring with identity. The ring $R$ is called an \emph{integral domain} if $1\neq 0$ and $R$ has no zero divisors. A subset $I$ of $R$ is called an \emph{ideal} of $R$ if $I$ is a subring of $R$ and $I$ is closed under multiplication by elements of $R$, that is, $aI\subseteq I$ for all $a\in R$. Let $A$ be a subset of $R$. Let $\langle A\rangle$ denote the smallest ideal of $R$ containing $A$; this ideal is called the \emph{ideal generated by $A$}. An ideal $I$ of $R$ is called a \emph{principal ideal} if $I$ is generated by a single element, that is, if there exists $a\in R$ such that $I=\langle a\rangle=\{ab:b\in R\}$. Let $a,b\in R$. We say that \emph{$a$ divides $b$} if there exists $c\in R$ such that $b=ac$. Note that $a$ divides $b$ if and only if $\langle b\rangle\subseteq\langle a\rangle$. Let $I$ be an ideal of $R$. Then the (additive) \emph{quotient ring of $R$ by $I$}, denoted by $R/I$, is the ring under the binary operations: $$(a+I)+(b+I):=(a+b)+I \text{ and } (a+I)\cdot (b+I) :=ab+I$$ for all $a,b\in R$. \begin{theorem}[{\cite[Theorem 8, p. 246]{DF04}}] Let $R$ be a commutative ring with identity, and let $I$ be an ideal of $R$. The correspondence $A\xleftrightarrow{}A/I$ is an inclusion preserving bijection between the set of subrings $A$ of $R$ that contain $I$ and the set of subrings of $R/I$. Furthermore, $A$ (a subring containing $I$) is an ideal of $R$ if and only if $A/I$ is an ideal of $R/I$. \end{theorem} An ideal $\mathfrak{m}$ in $R$ is called a \emph{maximal ideal} if $\mathfrak{m}\neq R$ and the only ideals containing $\mathfrak{m}$ are $\mathfrak{m}$ and $R$. \begin{proposition}[{\cite[Proposition 12, p. 254]{DF04}}] Let $R$ be a commutative ring with identity. The ideal $\mathfrak{m}$ is a maximal ideal if and only if the quotient ring $R/\mathfrak{m}$ is a field. \end{proposition} The ring $R$ is called a \emph{local ring} if it has a unique maximal ideal. \begin{proposition}[{\cite[Exercise 37, p. 259]{DF04}}] Let $R$ be a commutative ring with identity. If $R$ is a local ring with maximal ideal $\mathfrak{m}$, then every element of $R\setminus\mathfrak{m}$ is a unit. Thus, $R\setminus\mathfrak{m}$ equals the set of units in $R$. \end{proposition} The ring $R$ is called a \emph{principal ideal ring} if every ideal of $R$ is principal. The ring $R$ is called a \emph{principal ideal domain (PID)} if it is an integral domain which is a principal ideal ring. An ideal $P$ of $R$ is called a \emph{prime ideal} if $P\neq R$ and whenever the product $ab$ of two elements $a,b\in R$ is an element of $P$, then at least one of $a$ and $b$ is an element of $P$. Let $R$ be an integral domain. For an element $a\in R$ such that $a$ is nonzero and a non-unit, \begin{itemize} \item $a$ is called \emph{irreducible} in $R$ if $a=bc$ with $b,c\in R$ implies $a\in R^\times$ or $b\in R^\times$, and \item $a$ is called \emph{prime} in $R$ if the ideal $\langle a\rangle$ is a prime ideal. \end{itemize} Two elements $a$ and $b$ of $R$ are said to be \emph{associate} in $R$ if $a=bc$ for some unit $c$ in $R$ (note that in this case, $\langle a\rangle=\langle b\rangle$). A \emph{unique factorisation domain (UFD)} is an integral domain $R$ in which every nonzero element $a\in R$ which is not a unit has the following two properties: \begin{itemize} \item $a$ can be written as a finite product of irreducibles in $R$: $a=p_1\cdots p_k$ where the factors are irreducibles in $R$, and \item the decomposition is unique up to associates: that is, if $a=p_1'\cdots p_{\ell}'$ is another factorisation of $a$ into irreducibles, then $k=\ell$ and there is some renumbering of the factors so that $p_j$ is associate to $p_j'$ for $j=1,\ldots,k$. \end{itemize} \begin{proposition}[{\cite[Proposition 12, p. 286]{DF04}}] In a UFD a nonzero non-unit element is a prime if and only if it is irreducible. \end{proposition} \begin{theorem}[{\cite[Theorem 14, p. 287]{DF04}}] Every PID is a UFD. \end{theorem} \begin{proposition} Let $R$ be a finite commutative ring with identity. Then every element of $R$ is either a unit or a zero divisor. \end{proposition} \begin{proof} Let $a$ be a nonzero non-unit element of $R$. Then $\langle a\rangle\neq R$, and the function \begin{align*} \phi&: R\rightarrow\langle a\rangle\\ &x\mapsto ax \end{align*} cannot be injective. So, there exists $x\neq y$ such that $ax=ay$. Thus, we find that $a(x-y)=0$ where $x-y\neq 0$, wherby $a$ is a zero divisor. \end{proof} Let $I$ be an ideal of $R$. Let $a,b\in R$. We write $a\equiv b\pmod J$ if $a-b\in J$. For the ring $R$, we call an ideal $J$ \emph{non-trivial} if it is a nonzero ideal which is not equal to $R$. \begin{lemma} Let $K$ be a finite commutative ring with identity having precisely three ideals. Then $K\cong P/\langle p^2\rangle$ for some principal ideal domain $P$ and prime $p\in P$. \end{lemma} \begin{proof} $K$ is a principal ideal ring, so by \cite[Theorem 1]{H68}, it is a finite direct product of quotients of PIDs. Let $K\cong P_1/Q_1\times \cdots\times P_i/Q_i$ where $i\geq 1$, and for $j\in\{1,\ldots,i\}$, $P_j$ is a PID and $Q_j$ is an ideal of $P_j$. But $K$ has precisely three ideals, so $i=1$ and therefore $K\cong P/Q$ for some PID $P$ and some ideal $Q$ of $P$. Let $Q=\langle x\rangle$ for some $x\in Q$. Then, ideals of $P/Q$ are of the form $Q_1/Q$ where $Q_1$ is an ideal containing $Q$. If $Q_1=\langle y\rangle$, then $Q_1$ contains $Q$ if and only if $y$ divides $x$. So, for $P/Q$ to have precisely three ideals, $x$ must have exactly three divisors in $P$, unique up to units. If $x=0$, then $K$ must have exactly three elements, unique up to units, which is not possible. Therefore $x\neq 0$; let $x=p_1^{h_1}\cdots p_i^{h_i}$ (unique up to units), $i\geq 1$. Then $x$ has $(h_1+1)\cdots (h_i+1)$ divisors, so $(h_1+1)\cdots (h_i+1)=3$ implies $i=1, h_1=2$. This completes the proof. \end{proof} \begin{lemma} Let $K$ be a finite commutative ring having identity and a unique non-trivial ideal $J$. Then $K/J\cong \mathbb{F}_q$, where $\mathbb{F}_q$ is a finite field of order $q$ for a prime power $q$. \end{lemma} \begin{proof} The proof follows from the fact that the ideals of $K$ are $\langle 0\rangle \subsetneq J\subsetneq K$, whereby $K$ is a local ring with unique maximal ideal $J$. \end{proof} \begin{lemma}\label{lemij=0} Let $K$ be a finite commutative ring having identity and a unique non-trivial ideal $J$. Then, for any $x,y\in J$, the equality $xy=0$ holds. \end{lemma} \begin{proof} The result holds if $x=0$ or $y=0$, so assume that $x\neq 0, y\neq 0$. Suppose $xy\neq 0$. Then $\langle xy\rangle=J$ or $\langle xy\rangle=K$. If $\langle xy\rangle=K$ then $1\in K=\langle xy\rangle$, so $xyw=1$ for some $w\in K$, which implies that $x$ is a unit, so $x\notin J$, a contradiction. Therefore, $\langle xy\rangle=J$. Then, $xyz=x$ for some $z\in K$, that is, $x(yz-1)=0$. But $y\in J$ implies $yz\in J$. Since $J$ is a non-trivial ideal of $K$, $1$ does not belong to $J$. Then $(yz-1)\notin J$, so $(yz-1)$ is a unit. which implies $x=0$, a contradiction. This completes the proof. \end{proof} \begin{lemma}\label{lemjr} Let $K$ be a finite commutative ring with identity having a unique non-trivial ideal $J=\langle r\rangle$, such that $K/J\cong \mathbb{F}_q$, where $\mathbb{F}_q$ is a finite field of order $q$ for a prime power $q$. Let $K/J=\{z_1+J,z_2+J,\ldots, z_q+J\}$ where $\{z_1,z_2,\ldots,z_q\}$ is a set of coset representatives. Then $J=\{z_1r,z_2r,\ldots, z_qr\}$ and $K=\{z_{j_1}+z_{j_2}r: j_1,j_2\in\{1,2,\ldots,q\}\}$, where $|J|=q$ and $|K|=q^2$. \end{lemma} \begin{proof} Evidently, $\{z_1r,z_2r,\ldots, z_qr\}\subseteq J$. Let $1\leq j_1<j_2\leq q$, and suppose $z_{j_1}r=z_{j_2}r$. This implies $(z_{j_1}-z_{j_2})r=0$. Since $z_{j_1}+J$ and $z_{j_2}+J$ are distinct elements of $K/J$, $(z_{j_1}-z_{j_2})$ does not belong to $J$, and so $(z_{j_1}-z_{j_2})$ is a unit. This implies $r=0$ which is not possible. Therefore, $J=\{z_1r,z_2r,\ldots, z_qr\}$. Next, we observe that $|K|=|K/J|\cdot|J|=q^2$, so it suffices to show that the elements of $\{z_{j_1}+z_{j_2}r: j_1,j_2\in\{1,2,\ldots,q\}\}$ are distinct. Let $z_{j_1}+z_{j_2}r=z_{j_3}+z_{j_4}r$ for some $j_1,j_2,j_3,j_4\in\{1,2,\ldots,q\}$. Then $z_{j_1}-z_{j_3}=(z_{j_2}-z_{j_4})r$, and since $(z_{j_2}-z_{j_4})r\in J$, we conclude $z_{j_1}=z_{j_3}$. So, $(z_{j_2}-z_{j_4})r=0$. If $z_{j_2}\neq z_{j_4}$ then $z_{j_4}+J\neq z_{j_2}+J$. So, $(z_{j_2}-z_{j_4})$ is a unit, and, consequently, $r=0$ which is not possible. This completes the proof. \end{proof} Note that if $K/J=\{z_1+J,z_2+J,\ldots, z_q+J\}$ where $\{z_1,z_2,\ldots,z_q\}$ is a set of coset representatives, then for $j_1,j_2\in\{1,2,\ldots,q\}$, $(z_{j_1}-z_{j_2})$ is invertible in $K$ if and only if $j_1\neq j_2$. \begin{lemma} Let $K$ be a finite commutative ring with identity having a unique non-trivial ideal $J=\langle r\rangle$, such that $K/J\cong \mathbb{F}_q$, where $\mathbb{F}_q$ is a finite field of order $q$ for a prime power $q$. Let $K/J=\{z_1+J,z_2+J,\ldots, z_q+J\}$ where $\{z_1,z_2,\ldots,z_q\}$ is a set of coset representatives with $z_{1}=0$. Let $j_1,j_2\in\{1,2,\ldots,q\}$. Then the following hold. \begin{enumerate}[label=\textnormal{(\arabic*)},ref=\thelemma (\arabic*)] \item \label{lempart1}$z_{j_1}+z_{j_2}r\in J$ if and only if $z_{j_1}=0$. \item \label{lempart2} $z_{j_1}+z_{j_2}r=0$ if and only if $z_{j_1}=z_{j_2}=0$. \end{enumerate} \end{lemma} \begin{proof} The proofs readily follow by noting that for $j\in \{1,2,\ldots,q\}$, $z_{j}$ belongs to $J$ if and only if $z_{j}=0$. \end{proof} \section{Proofs of Theorem \ref{thm:Main1} and Theorem \ref{thm:Main2}}\label{sec:Proofs} In this section, we give proofs of Theorem \ref{thm:Main1} and Theorem \ref{thm:Main2} in a series of lemmas and propositions. For a graph $G$, we denote the complementary graph of $G$ by $\overline{G}$. \subsection{The number of vertices and the degrees of the graphs $X(2e,K)$ and $Y(2e,K)$} In this section, we determine the number of vertices and the degrees of the graphs $X(2e,K)$ and $Y(2e,K)$. \begin{proposition}\label{prop1} Let $e\geq 2$ be an integer. Let $K$ be a finite commutative ring with identity having a unique non-trivial ideal $J$ such that $K/J\cong \mathbb{F}_q$, where $q$ is a prime power. The number of vertices in each of the graphs $X(2e,K)$ and $Y(2e,K)$ is $\dfrac{q^{2e-1}(q^{2e}-1)}{q-1}$. \end{proposition} \begin{proof} The number of elements in $V'$ is $(q^2)^{2e}-q^{2e}$. We find that the number of elements in an equivalence class corresponding to the relation $\sim$ is the number of elements in $K^\times$, by noting that $\lambda_1 (x_1,x_2,\ldots, x_{2e})=\lambda_2 (x_1,x_2,\ldots, x_{2e})$ for some $\lambda_1,\lambda_2\in K^\times$ and $(x_1,x_2,\ldots,x_{2e})\in V'$ implies that $(\lambda_1-\lambda_2)x_j=0$ for $1\leq j\leq 2e$, so if $x_i\in K^\times$ for some $i\in\{1,2,\ldots,2e\}$ then $(\lambda_1-\lambda_2)x_i=0$ implies that $\lambda_1=\lambda_2$. So, the number of vertices in the graph equals the number of equivalence classes corresponding to $\sim$, that is, $$\dfrac{(q^2)^{2e}-q^{2e}}{q^2-q} = \dfrac{q^{2e-1}(q^{2e}-1)}{q-1}. $$ \end{proof} \begin{proposition}\label{prop2} Let $e\geq 2$ be an integer. Let $K$ be a finite commutative ring with identity having a unique non-trivial ideal $J$ such that $K/J\cong \mathbb{F}_q$, where $q$ is a prime power. Then the graph $\overline{X(2e,K)}$ is regular of degree $\dfrac{q^{4e-3}-q^{2e-2}-q+1}{q-1}$, and the graph $X(2e,K)$ is regular of degree $q^{4e-2}+q^{4e-3}-q^{2e-2}$. \end{proposition} \begin{proof} Let us fix a vertex $[a_1,a_2,\ldots,a_{2e}]$ of the graph $\overline{X(2e,K)}$. We find the number of possibilities for the vertex $[b_1,b_2,\ldots,b_{2e}]$ such that $[b_1,b_2,\ldots,b_{2e}]$ is adjacent to $[a_1,a_2,\ldots,a_{2e}]$ in $\overline{X(2e,K)}$. To begin with, we find the number of solutions $(b_1,b_2,\ldots, b_{2e})$ such that \begin{align}\label{eq0} &(a_1b_{e+1}-a_{e+1}b_1) + (a_2b_{e+2}-a_{e+2}b_2) + \cdots + (a_eb_{2e}-a_{2e}b_e )=0. \end{align} Since $(a_1,a_2,\ldots,a_{2e})\in V'$, there exists $i\in\{1,2,\ldots,2e\}$ such that $a_i\in K^\times$. If $i\leq e$, then \begin{align}\label{eqless} b_{e+i} &= a_i^{-1}((a_{e+1}b_1-a_1b_{e+1}) + (a_{e+2}b_2-a_2b_{e+2}) \notag\\ &+\cdots+ (a_{e+i-1}b_{i-1}-a_{i-1}b_{e+i-1}) +a_{e+i}b_i + (a_{e+i+1}b_{i+1}- a_{i+1}b_{e+i+1})\notag \\ &+ \cdots + (a_{2e}b_e- a_eb_{2e} )), \end{align} and if $i\geq e+1$, then \begin{align}\label{eqmore} b_{i-e} &= a_i^{-1}((a_{1}b_{e+1}-a_{e+1}b_{1}) + (a_{2}b_{e+2}-a_{e+2}b_{2}) \notag\\ &+\cdots+ (a_{i-1-e}b_{i-1}-a_{i-1}b_{i-1-e}) +a_{i-e}b_i + (a_{i+1-e}b_{i+1}- a_{i+1}b_{i+1-e})\notag \\ &+ \cdots + (a_{e}b_{2e}- a_{2e}b_{e} )). \end{align} This shows that there are $2e-1$ choices of $b_j$'s possible. Since $(b_1,b_2,\ldots,b_{2e})\in V'$, it must have an invertible entry. When $i\leq e$ (resp. $i\geq e+1$), if all $b_j$'s to the right hand side of Equation \eqref{eqless} (resp. \eqref{eqmore}) are elements in $J$, then $b_{e+i}$ (resp. $b_{i-e}$) is also an element in $J$, so $(b_1,b_2,\ldots,b_{2e})$ does not belong to $ V'$. On the other hand, when $i\leq e$ (resp. $i\geq e+1$), if there exists $j\neq e+i$ (resp. $j\neq i-e$) such that $b_j\in K^\times$, then $(b_1,b_2,\ldots,b_{2e})$ belongs to $ V'$. So, to count the number of tuples $(b_1,b_2,\ldots,b_{2e})\in V'$ satisfying Equation \eqref{eq0}, it is enough to count the number of tuples $(b_1,b_2,\ldots,b_{2e})\in \underbrace{K\times K\times \cdots\times K}_{2e\text{ times}}$ satisfying Equation \eqref{eq0} such that for all $j\neq e+i$ when $i\leq e$ (resp. $j\neq i-e$ when $i\geq e+1$), $b_j$ does not belong to $J$. Moreover, Equations \eqref{eqless} and \eqref{eqmore} are satisfied by putting $b_j=a_j$ for all $j\in\{1,2,\ldots,2e\}$. So, the number of possibilities for the vertex $[b_1,b_2,\ldots,b_{2e}]$ being adjacent to $[a_1,a_2,\ldots,a_{2e}]$ such that Equation \eqref{eq0} holds is given by \begin{align*} &\dfrac{\text{the number of tuples }(b_1,b_2,\ldots,b_{2e})\in V'\text{ satisfying Equation }\eqref{eq0} }{\text{size of an equivalence class corresponding to }\sim}\\ &-\text{ the possibility that }[b_1,b_2,\ldots,b_{2e}]=[a_1,a_2,\ldots,a_{2e}]\\ &=\dfrac{(q^2)^{2e-1}-q^{2e-1}}{q^2-q}-1\\ &= \dfrac{q^{4e-3}-q^{2e-2}-q+1}{q-1}, \end{align*} which equals the degree of the graph $\overline{X(2e,K)}$. Then, $X(2e,K)$ is regular of degree \begin{align*} &\dfrac{q^{2e-1}(q^{2e}-1)}{q-1}-1-\dfrac{q^{4e-3}-q^{2e-2}-q+1}{q-1}\\ =&\dfrac{q^{4e-1}-q^{2e-1}-q^{4e-3}+q^{2e-2}+q-1}{q-1} - 1 \\ =& q^{4e-2}+q^{4e-3}-q^{2e-2}, \end{align*} and the proof is complete. \end{proof} \begin{proposition}\label{prop3} Let $e\geq 2$ be an integer. Let $K$ be a finite commutative ring with identity having a unique non-trivial ideal $J=\langle r\rangle$ such that $K/J\cong \mathbb{F}_q$ for a prime power $q$. Let $e\geq 1$ be an integer. Then $Y(2e,K)$ is regular of degree $q^{4e-3}-q^{2e-2}$. \end{proposition} \begin{proof} Let $K/J=\{z_1+J,z_2+J,\ldots, z_q+J\}$ where $\{z_1,z_2,\ldots,z_q\}$ is a set of coset representatives with $z_1=0$. Then $J=\{z_1r,z_2r,\ldots, z_qr\}$. Now, let us fix a vertex $[a_1,a_2,\ldots,a_{2e}]$ of the graph $Y(2e,K)$. We find the number of possibilities of the vertex $[b_1,b_2,\ldots,b_{2e}]$ such that $[b_1,b_2,\ldots,b_{2e}]$ is adjacent to $[a_1,a_2,\ldots,a_{2e}]$ in $Y(2e,K)$, that is, \begin{align}\label{eq01} &(a_1b_{e+1}-a_{e+1}b_1) + (a_2b_{e+2}-a_{e+2}b_2) + \cdots + (a_eb_{2e}-a_{2e}b_e )=z_jr \end{align} for some $j \in \{2,\ldots, q\}$. Since $(a_1,a_2,\ldots,a_{2e})\in V'$, there exists $i\in\{1,2,\ldots,2e\}$ such that $a_i\in K^\times$. If $i\leq e$, then \begin{align}\label{eqless1} b_{e+i} &= a_i^{-1}(z_jr+(a_{e+1}b_1-a_1b_{e+1}) + (a_{e+2}b_2-a_2b_{e+2}) \notag\\ &+\cdots+ (a_{e+i-1}b_{i-1}-a_{i-1}b_{e+i-1}) +a_{e+i}b_i + (a_{e+i+1}b_{i+1}- a_{i+1}b_{e+i+1})\notag \\ &+ \cdots + (a_{2e}b_e- a_eb_{2e} )), \end{align} and if $i\geq e+1$, then \begin{align}\label{eqmore1} b_{i-e} &= a_i^{-1}((a_{1}b_{e+1}-a_{e+1}b_{1}) + (a_{2}b_{e+2}-a_{e+2}b_{2}) \notag\\ &+\cdots+ (a_{i-1-e}b_{i-1}-a_{i-1}b_{i-1-e}) +a_{i-e}b_i + (a_{i+1-e}b_{i+1}- a_{i+1}b_{i+1-e})\notag \\ &+ \cdots + (a_{e}b_{2e}- a_{2e}b_{e} )-z_jr). \end{align} This shows that there are $2e-1$ choices of $b_j$'s possible. Since $(b_1,b_2,\ldots,b_{2e})\in V'$, it must have an invertible entry. When $i\leq e$ (resp. $i\geq e+1$), if all $b_j$'s to the right hand side of Equation \eqref{eqless1} (resp. \eqref{eqmore1}) are elements in $J$, then $b_{e+i}$ (resp. $b_{i-e}$) is also an element in $J$, so $(b_1,b_2,\ldots,b_{2e})\notin V'$. On the other hand, when $i\leq e$ (resp. $i\geq e+1$), if there exists $j\neq e+i$ (resp. $j\neq i-e$) such that $b_j\in K^\times$, then $(b_1,b_2,\ldots,b_{2e})\in V'$. So, to count the number of tuples $(b_1,b_2,\ldots,b_{2e})\in V'$ satisfying Equation \eqref{eq01}, it is enough to count the number of tuples $(b_1,b_2,\ldots,b_{2e})\in \underbrace{K\times K\times \cdots\times K}_{2e\text{ times}}$ satisfying Equation \eqref{eq01} such that for all $j\neq e+i$ when $i\leq e$ (resp. $j\neq i-e$ when $i\geq e+1$), $b_j$ does not belong to $J$. Therefore, the number of possibilities for the vertex $[b_1,b_2,\ldots,b_{2e}]$ being adjacent to $[a_1,a_2,\ldots,a_{2e}]$ such that Equation \eqref{eq01} holds is given by \begin{align*} &\dfrac{(\text{the number of choices of }k)\times ( \text{the number of tuples }(b_1,b_2,\ldots,b_{2e})\in V'\text{ satisfying Equation }\eqref{eq01})}{\text{size of an equivalence class corresponding to }\sim}\\ &=\dfrac{(q-1)((q^2)^{2e-1}-q^{2e-1})}{q^2-q}\\ &= q^{4e-3}-q^{2e-2}. \end{align*} This completes the proof. \end{proof} \subsection{The canonical partition of the graphs $X(2e,K)$ and $Y(2e,K)$} In this section, we describe the canonical partition of the graphs $X(2e,K)$ and $Y(2e,K)$. We recall that $J=\langle r\rangle$ is the unique non-trivial ideal of $K$ such that $|J|=q$ and $|K|=q^2$ for some prime power $q$, where $K/J=\{z_1+J,z_2+J,\ldots, z_q+J\}$ and $\{z_1,z_2,\ldots,z_q\}$ is a set of coset representatives with $z_1=0$. Put $T:=\underbrace{J \times J \times \ldots \times J}_{2e \text{ times}}$. Then $T$ is a vector space over $K/J$ of dimension $2e$, with the scalar multiplication defined as, $$(z_{j}+J).(x_1,x_2,\ldots,x_{2e})=(z_jx_1,z_j x_2,\ldots,z_jx_{2e}),$$ where $j\in\{1,2,\ldots,q\}$ and $x_1,x_2,\ldots,x_{2e}\in J$ (the operation is well defined due to Lemma \ref{lemij=0}). We say that a $(2e-1)$-dimensional subspace of $T$ is a \emph{hyperplane}. Let $x$ and $y$ be elements of $J$ such that $x\neq 0, y\neq 0$. We observe that by Lemma \ref{lemjr}, $xu\in H$ if and only if $yu\in H$. This means that the statement of the following lemma does not depend on the choice of the generator $r$ of the ideal $J$. \begin{lemma}\label{lem:pu} Let $H$ be a hyperplane of $T$ and let $u\in V'$. Then the following statements hold.\\ {\rm (1)} If $ru \in H$, then $|\{[u+h] : h \in H\}| = |H|/q$.\\ {\rm (2)} If $ru \notin H$, then $|\{[u+h] : h \in H\}| = |H|$.\\ \end{lemma} \begin{proof} It suffices to show that, given $h \in H$, the equation \begin{equation}\label{eq:cond1} u+h = \sigma(u+f) \end{equation} has exactly $q$ solutions (resp. a unique solution) in variables $\sigma \in K^\times$ and $f \in H$ whenever $ru \in H$ (resp. $ru \notin H$). Equation (\ref{eq:cond1}) is equivalent to \begin{equation}\label{eq:cond2} (\sigma-1)u = h-\sigma f. \end{equation} The right-hand side of Equation (\ref{eq:cond2}) is a $2e$-tuple with entries in $J$ and $u$ is a $2e$-tuple having at least one invertible entry. This implies that $\sigma-1$ belongs to $J$, so $\sigma=1+z_jr$ for some $j\in\{1,2,\ldots,q\}$. Using Lemma \ref{lemij=0}, Equation (\ref{eq:cond2}) can be rewritten as $$ z_jru = h - f $$ for some $j\in\{1,2,\ldots,q\}$ which is equivalent to \begin{equation}\label{eq:cond3} f = h - z_jru. \end{equation} Thus, if $ru \in H$, then the set of solutions $(\sigma,f)$ of Equation (\ref{eq:cond1}) is $$ \{(1+z_jr,h-z_jru) : j \in \{1,2,\ldots,q\}\}. $$ Moreover, if $ru \notin H$, then $z_jru$ and, consequently, $h - z_jru$ do not belong to $H$ unless $j=1$, which means that $j=1$, so Equations (\ref{eq:cond3}) and (\ref{eq:cond1}) have a unique solution $(\sigma,f)$ given by $(1,h)$. This completes the proof. \end{proof} \begin{lemma}\label{lem:class} Let $H_1, H_2$ be hyperplanes of $T$ and let $u\in V'$ such that $ru \notin H_1 \cup H_2$. Then $$\{[u+h] : h \in H_1\} = \{[u+f] : f \in H_2\}$$ holds. \end{lemma} \begin{proof} In view of Lemma \ref{lem:pu}(2), it suffices to show that for any $g \in T \setminus H_1$ there exist $\sigma \in K^\times$ and $h \in H_1$ such that $$ u+h = \sigma(u+g). $$ Note that if $j$ runs over $\{2,\ldots,q\}$ and $h$ runs over $H_1$, then $(h-z_jru)$ runs over $T \setminus H_1$. Thus, we have $$ g = h - z_jru $$ for some uniquely determined $j \in \{2,\ldots,q\}$ and $h \in H_1$. Put $\sigma = 1+z_jr$. Then, using Lemma \ref{lemij=0}, we have $$ \sigma(u+g) = (1+z_jr)(u+h-z_jru) = u+h, $$ which completes the proof. \end{proof} Let $H$ be a hyperplane and $u$ be a $2e$-tuple over $K$ having an invertible entry, such that $ru \notin H$. Denote by $C(H,u)$ the set $\{[u+h] : h \in H\}$. We note that, for any $u_1\in V'$ such that $ru_1\notin H$ and $[u_1]\in C(H,u)$, the equality $C(H,u_1)=C(H,u)$ holds. We also use the \emph{Gaussian coefficient} $\qbin{n}{i}{q}$ to denote the number of $i$-dimensional subspaces of an $n$-dimensional vector space over $\F_q$. \begin{proposition}\label{prop:CHu} Let $H_1, H_2$ be hyperplanes of $T$ and let $u_1\in V'$, $u_2\in V'$ such that $ru_1\notin H_1, ru_2\notin H_2$. If the sets $C(H_1,u_1)$ and $C(H_2,u_2)$ have a vertex in common, then they coincide. \end{proposition} \begin{proof} Let the two sets $C(H_1,u_1)$ and $C(H_2,u_2)$ contain a common vertex $[u]$. This means there exist $h_1 \in H_1$ and $h_2 \in H_2$ such that $[u] = [u_1+h_1] = [u_2+h_2]$. That is, there exist $\sigma_1,\sigma_2 \in K^\times$ such that $\sigma_1u = u_1+h_1$ and $\sigma_2u = u_2+h_2$. This implies $u_1 = \sigma_1u - h_1$ and $u_2 = \sigma_2u - h_2$. Thus, we have \begin{align*} C(H_1,u_1) &= \{[u_1+h] : h \in H_1\}\\ &= \{[\sigma_1u - h_1+h] : h \in H_1\}\\ &= \{[\sigma_1u + h] : h \in H_1\}\\ &= \{[u + \sigma_1^{-1}h] : h \in H_1\}\\ &= \{[u + h] : h \in \sigma_1^{-1}H_1\}. \end{align*} Note that $\sigma_1^{-1}H_1$ is a hyperplane as it is a subgroup of order $q^{2e-1}$ in $T$. Also, $ru$ does not belong to $\sigma_1^{-1}H_1$. Indeed, suppose $ru$ belongs to $\sigma_1^{-1}H_1$. Then there exists $f \in H_1$ such that $ru = \sigma_1^{-1}f$ and $f = \sigma_1ru$. The condition $u_1 = \sigma_1u - h_1$ then implies $ru_1 = \sigma_1ru = f$, that is, $ru_1$ belongs to $H_1$, a contradiction. Thus, we conclude that $$ C(H_1,u_1) = C(\sigma_1^{-1}H_1,u). $$ In a similar way, we have that $\sigma_2^{-1}H_2$ is a hyperplane, $ru$ does not belong to $\sigma_2^{-1}H_2$ and $$ C(H_2,u_2) = C(\sigma_2^{-1}H_2,u). $$ By Lemma \ref{lem:class}, we have $C(\sigma_1^{-1}H_1,u) = C(\sigma_2^{-1}H_2,u)$, which finally means $C(H_1,u_1) = C(H_2,u_2)$. This completes the proof. \end{proof} \begin{corollary}\label{cor:Partition} There exists a uniquely determined partition of the vertex sets of $X(2e,K)$ and $Y(2e,K)$ into classes $C(H,u)$. Moreover, the size of each class is $q^{2e-1}$ and the number of such classes in the partition is the Gaussian coefficient $\qbin{2e}{2e-1}{q} = \frac{q^{2e}-1}{q-1}$. \end{corollary} \begin{proof} This follows from Proposition \ref{prop:CHu} and Proposition \ref{prop1}. \end{proof} We denote the partition from Corollary \ref{cor:Partition} by $\Pi(2e,K)$. \subsection{The numbers of common neighbours of two vertices in the graph $X(2e,K)$} In this section, we determine the numbers of common neighbours of two vertices in the graph $X(2e,K)$. \begin{proposition}\label{propmain1} Let $C(H,u)$ be a class of the partition $\Pi(2e,K)$, where $H$ is a hyperplane of $T$ and $u\in V'$ such that $ru\notin H$. Then any two distinct vertices from $C(H,u)$ non-adjacent in $\overline{X(2e,K)}$ have $\dfrac{(q^{2e-2}-1)q^{2e-2}}{q-1}$ common neighbours in $\overline{X(2e,K)}$, and any two vertices from $C(H,u)$ adjacent in $\overline{X(2e,K)}$ have $\dfrac{(q^{2e-2}-1)q^{2e-2}}{q-1} - 2$ common neighbours in $\overline{X(2e,K)}$. \end{proposition} \begin{proof} Let $[a],[b]$ be two distinct vertices from $C(H,u)$. Without loss of generality, we may assume $a = u$ and $b = u+h$ for some $h \in H$ where $h$ is not identically $0$. Let $[x]$ be a vertex of $\overline{X(2e,K)}$ that is a common neigbour of $[a]$ and $[b]$. Then we have \begin{equation}\label{eq:condition1} (a_1x_{e+1}-a_{e+1}x_1) + (a_2x_{e+2}-a_{e+2}x_2) + \dots + (a_ex_{2e}-a_{2e}x_e) = 0 \end{equation} and \begin{equation}\label{eq:condition2} (b_1x_{e+1}-b_{e+1}x_1) + (b_2x_{e+2}-b_{e+2}x_2) + \dots + (b_ex_{2e}-b_{2e}x_e) = 0. \end{equation} Since $b = a + h$, Equation (\ref{eq:condition2}) implies \begin{align}\label{eq:condition7} &(a_1x_{e+1}-a_{e+1}x_1) + (a_2x_{e+2}-a_{e+2}x_2) + \dots + (a_ex_{2e}-a_{2e}x_e) + \notag\\ & (h_1x_{e+1}-h_{e+1}x_1) + (h_2x_{e+2}-h_{e+2}x_2) + \dots + (h_ex_{2e}-h_{2e}x_e) = 0 \end{align} and, consequently, \begin{align}\label{eq:condition8} (h_1x_{e+1}-h_{e+1}x_1) + (h_2x_{e+2}-h_{e+2}x_2) + \dots + (h_ex_{2e}-h_{2e}x_e) = 0 \end{align} A vertex $[x]$ is a common neighbour of $[a]$ and $[b]$ if and only if $x$ satisfies the system of Equations (\ref{eq:condition1}) and (\ref{eq:condition8}). Thus, it suffices to count such $2e$-tuples $(x_1,x_2,\ldots,x_{2e})$. Using Lemma \ref{lemjr}, we assume that for every $j \in \{1,2,\ldots,2e\}$, $a_j = c_j+d_jr$ and $x_j = s_j+t_jr$, where $c_j,d_j,s_j,t_j$ are from $\{z_1,z_2,\ldots,z_q\}$, $c_j,d_j$ are constants and $s_j,t_j$ are variables. Equation (\ref{eq:condition1}) is then equivalent to \begin{align}\label{eq:condition9} & (c_1+d_1r)(s_{e+1}+t_{e+1}r) - (c_{e+1}+d_{e+1}r)(s_1+d_1r) +\notag\\ & (c_2+d_2r)(s_{e+2}+t_{e+2}r) - (c_{e+2}+d_{e+2}r)(s_2+d_2r) + \dots +\notag\\ & (c_e+d_er)(s_{2e}+t_{2e}r) - (c_{2e}+d_{2e}r)(s_e+d_er) = 0, \end{align} and using Lemma \ref{lemij=0}, Equation \eqref{eq:condition9} is equivalent to \begin{align}\label{eq:condition10} & (c_1s_{e+1} + (c_1t_{e+1}+d_1s_{e+1})r) - (c_{e+1}s_1 + (c_{e+1}t_1+d_{e+1}s_1)r) +\notag\\ & (c_2s_{e+2} + (c_2t_{e+2}+d_2s_{e+2})r) - (c_{e+2}s_2 + (c_{e+2}t_2+d_{e+2}s_2)r) + \dots +\notag\\ & (c_es_{2e} + (c_et_{2e}+d_es_{2e})r) - (c_{2e}s_e + (c_{2e}t_e+d_{2e}s_e)r) = 0, \end{align} which we rewrite as \begin{align}\label{eq:group} & (c_1s_{e+1} - c_{e+1}s_1) + (c_2s_{e+2}-c_{e+2}s_2) + \cdots + (c_es_{2e}-c_{2e}s_e) + \notag \\ &\big((c_1t_{e+1}+d_1s_{e+1}) - (c_{e+1}t_1+d_{e+1}s_1) + (c_2t_{e+2}+d_2s_{e+2})-(c_{e+2}t_2+d_{e+2}s_2) + \dots + \notag\\ & (c_et_{2e}+d_es_{2e})-(c_{2e}t_e+d_{2e}s_e)\big)r = 0. \end{align} Let \begin{align}\label{eqfirst} (c_1s_{e+1} - c_{e+1}s_1) + (c_2s_{e+2}-c_{e+2}s_2) + \cdots + (c_es_{2e}-c_{2e}s_e) = z_{j_1}+z_{j_2}r \end{align} for some $j_1,j_2\in\{1,2,\ldots,q\}$. Then, we deduce that Equation \ref{eq:group} is equivalent to the system of the following two linear equations modulo $J$: \begin{align}\label{eq:condition11} (c_1s_{e+1} - c_{e+1}s_1) + (c_2s_{e+2}-c_{e+2}s_2) + \cdots + (c_es_{2e}-c_{2e}s_e) \equiv 0 \pmod J \end{align} and \begin{align}\label{eq:condition12} z_{j_2} + \bigg(\big((c_1t_{e+1}+d_1s_{e+1}) - (c_{e+1}t_1+d_{e+1}s_1) \big) + & \big((c_2t_{e+2}+d_2s_{e+2})-(c_{e+2}t_2+d_{e+2}s_2)\big) + \dots + \notag\\ & \big((c_et_{2e}+d_es_{2e})-(c_{2e}t_e+d_{2e}s_e)\big)\bigg) \equiv 0 \pmod J. \end{align} Indeed, let \begin{align*} E:=\big((c_1t_{e+1}+d_1s_{e+1}) - (c_{e+1}t_1+d_{e+1}s_1)\big) + & \big((c_2t_{e+2}+d_2s_{e+2})-(c_{e+2}t_2+d_{e+2}s_2) \big)+ \dots + \notag\\ & \big((c_et_{2e}+d_es_{2e})-(c_{2e}t_e+d_{2e}s_e)\big), \end{align*} and let \begin{align}\label{eqE} z_{j_2}+E=z_{j_3}+z_{j_4}r \end{align} for some $j_3,j_4\in\{1,2,\ldots,q\}$. Let Equation \eqref{eq:group} hold. Using Equation \eqref{eqfirst}, we obtain that $(z_{j_1}+z_{j_2}r)+Er=0$, and using Lemma \ref{lemij=0} and Equation \eqref{eqE}, this implies that $z_{j_1}+z_{j_3}r=0$. So, Lemma \ref{lempart2} implies $z_{j_1}=z_{j_3}=0$, and this implies Equations \eqref{eq:condition11} and \eqref{eq:condition12}. Conversely, Equations \eqref{eq:condition11} and \eqref{eq:condition12} imply $z_{j_1}=z_{j_3}=0$, so the left hand side of Equation \eqref{eq:group} yields $(z_{j_1}+z_{j_2}r)+Er=(z_{j_2}+E)r=z_{j_3}r=0$. Now, we find the number of solutions of Equations \eqref{eq:condition8}, \eqref{eq:condition11} and \eqref{eq:condition12}. Since the $2e$-tuple $a$ has an invertible entry, there exists $i \in \{1,2,\ldots,2e\}$ such that $c_i \notin J$. Assume that $i$ belongs to $\{1,2,\ldots,e\}$ (otherwise, the proof is similar). Then Equation (\ref{eq:condition12}) is equivalent to \begin{align}\label{eq:condition13} t_{e+i} \equiv &~c_{i}^{-1}\bigg(-z_{j_2}+ \big((c_{e+1}t_1+d_{e+1}s_1) - (c_1t_{e+1}+d_1s_{e+1})\big) +\\ &~\big((c_{e+2}t_2+d_{e+2}s_2) - (c_2t_{e+2}+d_2s_{e+2})\big)+ \dots + \notag\\ &~\big((c_{e+i-1}t_{i-1}+d_{e+i-1}s_{i-1}) - (c_{i-1}t_{e+i-1}+d_{i-1}s_{e+i-1})\big) + \notag\\ &~\big((c_{e+i}t_i+d_{e+i}s_i) - d_is_{e+i}\big) + \notag\\ &~\big((c_{e+i+1}t_{i+1}+d_{e+i+1}s_{i+1}) - (c_{i+1}t_{e+i+1}+d_{i+1}s_{e+i+1})\big) + \dots + \notag\\ &~\big((c_{2e}t_e+d_{2e}s_e) - (c_et_{2e}+d_es_{2e})\big)\bigg) \pmod J \end{align} On the other hand, for every $j \in \{1,2,\ldots,2e\}$, let $h_j = w_jr$, where $w_j$ belongs to $\{z_1,z_2,\ldots,z_q\}$ (this is due to Lemma \ref{lempart1}). Then Equation (\ref{eq:condition8}) is equivalent to \begin{align}\label{eq:condition14} \big(w_1r(s_{e+1}+t_{e+1}r)-w_{e+1}r(s_1+t_1r)\big) + & \big(w_2r(s_{e+2}+t_{e+2}r)-w_{e+2}r(s_2+t_2r)\big) + \dots + \notag\\ & \big(w_er(s_{2e}+t_{2e}r)-w_{2e}r(s_e+t_er)\big) = 0, \end{align} which is equivalent to \begin{align}\label{eq:condition15} (w_1s_{e+1}-w_{e+1}s_1) + (w_2s_{e+2}-w_{e+2}s_2) + \dots + (w_es_{2e}-w_{2e}s_e) \equiv 0 \pmod J. \end{align} Thus, the system of Equations (\ref{eq:condition1}) and (\ref{eq:condition8}) we need to solve is equivalent to the system of Equations (\ref{eq:condition11}), (\ref{eq:condition13}) and (\ref{eq:condition15}). Let us show that the system of Equations (\ref{eq:condition11}) and (\ref{eq:condition15}) has rank $2e-2$. Since the $2e$-tuple $a$ has an invertible entry and the $2e$-tuple $h$ is not identically $0$, each of Equations (\ref{eq:condition11}) and (\ref{eq:condition15}) is non-trivial. Let us show that Equations (\ref{eq:condition11}) and (\ref{eq:condition15}) are linearly independent. Suppose to the contrary they are dependent, that is, there exists $\delta \in \{z_2,\ldots,z_q\}$ such that, for every $j \in \{1,2,\ldots,2e\}$, we have \begin{equation}\label{eq:condition16} c_j \equiv \delta w_j \pmod J. \end{equation} This implies $$ra = (c_1r,c_2r,\ldots,c_{2e}r) = \delta(w_1r,w_2r,\ldots,w_{2e}r) = \delta h$$ belongs to $H$, a contradiction. Thus, there exist $\ell, k$ such that $1 \le \ell < k \le 2e$ holds and the system of Equations (\ref{eq:condition11}) and (\ref{eq:condition15}) is equivalent to the system of equations \begin{equation}\label{eq:condition17} s_{\ell} \equiv M_{\ell}(s_1,s_2,\ldots,s_{\ell-1},s_{\ell+1},\ldots,s_{k-1},s_{k+1},\ldots,s_{2e}) \pmod J \end{equation} and \begin{equation}\label{eq:condition18} s_{k} \equiv M_k(s_1,s_2,\ldots,s_{\ell-1},s_{\ell+1},\ldots,s_{k-1},s_{k+1},\ldots,s_{2e}) \pmod J, \end{equation} where $M_{\ell}(s_1,s_2,\ldots,s_{\ell-1},s_{\ell+1},\ldots,s_{k-1},s_{k+1},\ldots,s_{2e})$ and $M_k(s_1,s_2,\ldots,s_{\ell-1},s_{\ell+1},\ldots,s_{k-1},s_{k+1},\ldots,s_{2e})$ are linear combinations of the $2e-2$ variables $s_1,s_2,\ldots,s_{\ell-1},s_{\ell+1},\ldots,s_{k-1},s_{k+1},\ldots,s_{2e}$. Thus, the system of Equations (\ref{eq:condition11}), (\ref{eq:condition13}) and (\ref{eq:condition15}) is equivalent to the system of Equations (\ref{eq:condition17}), (\ref{eq:condition18}) and (\ref{eq:condition13}). Note that once the values of $s_1,s_2,\ldots,s_{2e}$ (and therefore the value of $z_{j_2}$ by Equation \eqref{eqfirst}) and $t_1,t_2,\ldots,t_{e+i-1},t_{e+i+1},\ldots,t_{2e}$ are determined, then the value of $t_{e+i}$ is also determined according to Equation (\ref{eq:condition13}). Now we are ready to count the number of solutions of the system of Equations (\ref{eq:condition17}), (\ref{eq:condition18}) and (\ref{eq:condition13}). The system of Equations (\ref{eq:condition17}), (\ref{eq:condition18}) has $q^{2e-2}$ solutions, but the zero solution must be excluded as the $2e$-tuple $x$ has an invertible entry. Thus, $q^{2e-2}-1$ solutions of Equations (\ref{eq:condition17}) and (\ref{eq:condition18}) satisfy our requirements. Moreover, for every such a solution there are $q^{2e-1}$ solutions of Equation (\ref{eq:condition13}) as the variables $t_1,t_2,\ldots,t_{e+i-1},t_{e+i+1},\ldots,t_{2e}$ are independent. Thus, the total number of solutions of the system of Equations (\ref{eq:condition17}), (\ref{eq:condition18}) and (\ref{eq:condition13}) is $$ (q^{2e-2}-1)q^{2e-1}. $$ Finally, we conclude that if the vertices $[a]$ and $[a+h]$ are non-adjacent, then they have $$ \dfrac{(q^{2e-2}-1)q^{2e-1}}{q^2-q} = \dfrac{(q^{2e-2}-1)q^{2e-2}}{q-1} $$ common neighbours in $\overline{X(2e,K)}$. Also, if the vertices $[a]$ and $[a+h]$ are adjacent, then the $2e$-tuples from $[a]$ and $[a+h]$ were counted among the $$ (q^{2e-2}-1)q^{2e-1} $$ solutions of the system of Equations (\ref{eq:condition17}), (\ref{eq:condition18}) and (\ref{eq:condition13}). Thus, the number of common neighbours of the vertices $[a]$ and $[a+h]$ in $\overline{X(2e,K)}$ is $$ \dfrac{(q^{2e-2}-1)q^{2e-2}}{q-1} - 2, $$ which completes the proof. \end{proof} \begin{corollary}\label{cor:XTwoVerticesFromTheSameClass} Let $C(H,u)$ be a class of the partition $\Pi(2e,K)$, where $H$ is a hyperplane of $T$ and $u\in V'$ such that $ru\notin H$. Then any two distinct vertices from $C(H,u)$ have $q^{4e-2}+q^{4e-3}-q^{4e-4}-q^{2e-2}$ common neighbours in $X(2e,K)$. \end{corollary} A class $C(H,u)$ of the partition $\Pi(2e,K)$, being the set of vertices $\{[u+h] : h \in H\}$ can be naturally identified with the set of $2e$-tuples $\{\sigma(u+h) : \sigma \in K^\times, h \in H\}$ of size $(q^2-q)q^{2e-1} = (q-1)q^{2e}$. \begin{lemma}\label{lem:cosetsOfT} Let $C(H,u)$ be a class of the partition $\Pi(2e,K)$, where $H$ is a hyperplane of $T$ and $u\in V'$ such that $ru\notin H$. Then the corresponding set of $2e$-tuples $\{\sigma(u+h) : \sigma \in K^\times, h \in H\}$ includes the coset $u + T$ and can moreover be partitioned into $q-1$ cosets of $T$, where a set of representatives of these cosets is $\{z_ju : j \in \{2,\ldots,q\}\}$. \end{lemma} \begin{proof} We recall that $J=\langle r\rangle$. Let $\sigma\in K^\times$ and $h\in H$. We consider the difference $\sigma(u+h) - z_ju$ for some $j\in\{2,\ldots,q\}$. Using Lemma \ref{lemjr}, we assume that $\sigma = s + tr$ where $s,t \in \{z_1,z_2,\ldots,z_q\}$ and $s\neq z_1$ by Lemma \ref{lempart1}. Then, using Lemma \ref{lemij=0} we have \begin{equation}\label{eq:condition19} \sigma(u+h) - z_ju = (s+tr)(u+h) - z_ju = su + tru + sh - z_ju = (s-z_j)u + \underbrace{tru}_{\substack{\notin H\\\text{ if }t\neq 0}}+\underbrace{sh}_{\in H}. \end{equation} Put $s = z_j$ in Equation (\ref{eq:condition19}). Then we get \begin{equation}\label{eq:condition20} \sigma(u+h) - z_ju= \underbrace{tru}_{\substack{\notin H\\\text{ if }t\neq 0}}+\underbrace{z_jh}_{\in H}. \end{equation} In Equation (\ref{eq:condition20}), if $t$ runs over $ \{z_1,z_2,\ldots,z_q\}$ and $h$ runs over $H$, then $(tru + z_jh)$ runs over $T$. Thus, the set of $2e$-tuples $\{\sigma(u+h) : \sigma \in K^\times, h \in H\}$ includes the coset $z_ju + T$, and if $1\in z_i+J$ for some $i\in\{2,\ldots,q\}$, then $z_iu+T=u+T$. Moreover, this set of $2e$-tuples can be partitioned into $q-1$ cosets of $T$, where a set of representatives of these cosets is $\{z_ju : j \in \{2,\ldots,q\}\}$. This completes the proof. \end{proof} \begin{proposition}\label{propmain2} Let $H_1,H_2$ be hyperplanes of $T$, and let $a\in V'$, $b\in V'$ with $ra\notin H_1,rb\notin H_2$, such that $C(H_1,a)$ and $C(H_2,b)$ are distinct classes of the partition $\Pi(2e,K)$. Then if the vertices $[a]$ and $[b]$ are non-adjacent in $\overline{X(2e,K)}$, then they have $\dfrac{(q^{2e-2}-1)q^{2e-3}}{q-1}$ common neighbours in $\overline{X(2e,K)}$, and if the vertices $[a]$ and $[b]$ are adjacent in $\overline{X(2e,K)}$, then they have $\dfrac{(q^{2e-2}-1)q^{2e-3}}{q-1} - 2$ common neighbours in $\overline{X(2e,K)}$. \end{proposition} \begin{proof} Since $[b]$ does not belong to $C(H_1,a)$, we conclude that there exists no $\sigma \in K^\times$ and $h \in H_1$ such that $b = \sigma(a+h)$ holds. Let $[x]$ be a common neighbour of $[a]$ and $[b]$. Then we have a system of the equations \begin{equation}\label{eq:condition21} (a_1x_{e+1}-a_{e+1}x_1) + (a_2x_{e+2}-a_{e+2}x_2) + \dots + (a_ex_{2e}-a_{2e}x_e) = 0 \end{equation} and \begin{equation}\label{eq:condition22} (b_1x_{e+1}-b_{e+1}x_1) + (b_2x_{e+2}-b_{e+2}x_2) + \dots + (b_ex_{2e}-b_{2e}x_e) = 0. \end{equation} We consider the ordering of variables $x_{e+1},x_{e+2},\ldots,x_{2e},x_1,\ldots,x_e$ and write down the matrix of coefficients: $$ \left( \begin{matrix} a_1 & a_2 & \dots & a_{2e}\\ b_1 & b_2 & \dots & b_{2e} \end{matrix} \right) $$ Note that there exist $i \in \{1,2,\ldots,2e\}$ such that $a_i$ is invertible. Then the matrix of coefficients can be written as \begin{align*} \left( \begin{matrix} a_1 & \dots & a_i & \dots & a_{2e}\\ b_1 & \dots & b_i & \dots & b_{2e} \end{matrix} \right), \end{align*} which is equivalent to \begin{align*} \left( \begin{matrix} a_1a_i^{-1} & \dots & 1 & \dots & a_{2e}a_i^{-1}\\ b_1 & \dots & b_i & \dots & b_{2e} \end{matrix} \right) \end{align*} and, consequently, \begin{align}\label{mat} \left( \begin{matrix} a_1a_i^{-1} & \dots & 1 & \dots & a_{2e}a_i^{-1}\\ b_1-a_1a_i^{-1}b_i & \dots & 0 & \dots & b_{2e}-a_{2e}a_i^{-1}b_i \end{matrix} \right). \end{align} Now, let us show that there exists $\gamma\in \{1,2,\ldots,2e\}\setminus\{i\}$ such that $b_{\gamma}-a_{\gamma}a_i^{-1}b_i$ is invertible. To begin with, we find that there exists $j \in \{1,2,\ldots,2e\} \setminus \{i\}$ such that $b_j$ is invertible (otherwise, $a_i$ and $b_i$ are the only invertible entries of $a$ and $b$ and there exist $\sigma \in K^\times$ and $h \in T$ such that $b = \sigma(a+h)$, a contradiction by Lemma \ref{lem:cosetsOfT}). Suppose to the contrary that $b_{\gamma}-a_{\gamma}a_i^{-1}b_i$ is a zero divisor for all $\gamma \in \{1,2,\ldots,2e\}\setminus\{i\}$, that is, $$b_{\gamma}-a_{\gamma}a_i^{-1}b_i = t_{\gamma}r$$ for some $t_\gamma\in\{z_1,z_2,\ldots,z_q\}$. In particular, we have \begin{equation}\label{eq:condition23} b_j-a_ja_i^{-1}b_i = t_jr, \end{equation} which implies that $a_j$ and $b_i$ are also invertible (otherwise, the element on left hand side of the equality in Equation (\ref{eq:condition23}) is invertible and equal to a zero divisor, a contradiction). Thus, without loss of generality, we may assume $a_i = b_i = 1$ (we can multiply the original two equations by $a_i^{-1}$ and $b_i^{-1}$). We also deduce that for any $\gamma \in \{1,2,\ldots,2e\}$ the elements $a_{\gamma}$ and $b_{\gamma}$ are both invertible or both zero divisors. Equation (\ref{eq:condition23}) then becomes \begin{equation}\label{eq:condition24} b_j-a_j = t_jr. \end{equation} We observe that as $j$ varies over $\{1,2,\ldots,2e\}\setminus\{i\}$ such that $b_j$ is invertible, Equation \eqref{eq:condition24} holds. This means that $b-a \in T$, that is, $a$ and $b$ belong to the same coset of $T$. Then by Lemma \ref{lem:cosetsOfT}, $[a]$ and $[b]$ belong to the same class of the partition $\Pi(2e,K)$, which contradicts the choice of $[a]$ and $[b]$. Thus there exist $\ell, k \in \{1,2,\ldots,2e\}$ such that $1 \le \ell < k \le 2e$ and the system of Equations (\ref{eq:condition21}) and (\ref{eq:condition22}) is equivalent to the system of equations \begin{equation}\label{eq:condition25} x_{\ell} = M_{\ell}(x_1,x_2,\ldots,x_{\ell-1},x_{\ell+1},\ldots,x_{k-1},x_{k+1},\ldots,x_{2e}) \end{equation} and \begin{equation}\label{eq:condition26} x_{k} = M_k(x_1,x_2,\ldots,x_{\ell-1},x_{\ell+1},\ldots,x_{k-1},x_{k+1},\ldots,x_{2e}), \end{equation} where $M_{\ell}(x_1,x_2,\ldots,x_{\ell-1},x_{\ell+1},\ldots,x_{k-1},x_{k+1},\ldots,x_{2e})$ and $M_k(x_1,x_2,\ldots,x_{\ell-1},x_{\ell+1},\ldots,x_{k-1},x_{k+1},\ldots,$\\$x_{2e})$ are linear combinations of the $2e-2$ variables $x_1,x_2,\ldots,x_{\ell-1},x_{\ell+1},\ldots,x_{k-1},x_{k+1},\ldots,x_{2e}$. Now we are ready to count the number of solutions of the system of Equations (\ref{eq:condition25}) and (\ref{eq:condition26}) in the variables $x_1,x_2,\ldots,x_{2e},y$ and $z$. It has $(q^2)^{2e-2}$ solutions, but the solutions $x$ such that all the entries are zero divisors must be excluded as the $2e$-tuple $x$ has an invertible entry. Thus, $(q^2)^{2e-2}-q^{2e-2}$ solutions of Equations (\ref{eq:condition25}) and (\ref{eq:condition26}) satisfy our requirements. Finally, we conclude that if the vertices $[a]$ and $[b]$ are non-adjacent, then they have $$ \dfrac{(q^{2e-2}-1)q^{2e-2}}{q^2-q} = \dfrac{(q^{2e-2}-1)q^{2e-3}}{q-1} $$ common neighbours in $\overline{X(2e,K)}$. Also, if the vertices $[a]$ and $[b]$ are adjacent, then the $2e$-tuples from $[a]$ and $[b]$ were counted among the $$ (q^{2e-2}-1)q^{2e-2} $$ solutions of the system of Equations (\ref{eq:condition25}) and (\ref{eq:condition26}). Thus, the number of common neighbours of the vertices $[a]$ and $[b]$ in $\overline{X(2e,K)}$ is $$ \dfrac{(q^{2e-2}-1)q^{2e-3}}{q-1} - 2, $$ and the proof is complete. \end{proof} \begin{corollary}\label{cor:XTwoVertices FromDifferentClasses} Let $H_1,H_2$ be hyperplanes of $T$, and let $a,b\in V'$ with $ra\notin H_1,rb\notin H_2$, such that $C(H_1,a)$ and $C(H_2,b)$ are distinct classes of the partition $\Pi(2e,K)$. Then the vertices $[a]$ and $[b]$ have $q^{4e-2}+q^{4e-3}-q^{4e-4}-q^{4e-5}-q^{2e-2}+q^{2e-3}$ common neighbours in $X(2e,K)$. \end{corollary} \subsection{The numbers of common neighbours of two vertices in the graph $Y(2e,K)$} In this section, we determine the numbers of common neighbours of two vertices in the graph $Y(2e,K)$. \begin{proposition}\label{prop:YTwoVerticesFromTheSameClass} Let $C(H,u)$ be a class of the partition $\Pi(2e,K)$, where $H$ is a hyperplane of $T$ and $u\in V'$ such that $ru\notin H$. Then any two distinct vertices from $C(H,u)$ have $q^{4e-3}-q^{4e-4}-q^{2e-2}$ common neighbours in $Y(2e,K)$. \end{proposition} \begin{proof} We proceed similarly as in the proof of Proposition \ref{propmain1}. Let $[a],[b]$ be two distinct vertices from $C(H,u)$. Without loss of generality, we may assume $a = u$ and $b = u+h$ for some $h \in H$ where $h$ is not identically $0$. Let $[x]$ be a vertex of $\overline{X(2e,K)}$ that is a common neigbour of $[a]$ and $[b]$. Analogous to Equations \eqref{eq:condition1} and \eqref{eq:condition7}, we have \begin{equation}\label{yconda} (a_1x_{e+1}-a_{e+1}x_1) + (a_2x_{e+2}-a_{e+2}x_2) + \dots + (a_ex_{2e}-a_{2e}x_e) = yr\text{ for some }y\in\{z_2,\ldots,z_q\} \end{equation} and \begin{align}\label{eqcond1} &(a_1x_{e+1}-a_{e+1}x_1) + (a_2x_{e+2}-a_{e+2}x_2) + \dots + (a_ex_{2e}-a_{2e}x_e) + \notag\\ & (h_1x_{e+1}-h_{e+1}x_1) + (h_2x_{e+2}-h_{e+2}x_2) + \dots + (h_ex_{2e}-h_{2e}x_ e) = zr\text{ for some }y\in\{z_2,\ldots,z_q\} , \end{align} and Equation \eqref{eqcond1} implies that \begin{align}\label{ycondb} (h_1x_{e+1}-h_{e+1}x_1) + (h_2x_{e+2}-h_{e+2}x_2) + \dots + (h_ex_{2e}-h_{2e}x_e) = (z-y)r \end{align} Our task is to solve Equations \eqref{yconda} and \eqref{ycondb}. Using Lemma \ref{lemjr}, we assume that for every $j \in \{1,2,\ldots,2e\}$, $a_j = c_j+d_jr$ and $x_j = s_j+t_jr$, where $c_j,d_j,s_j,t_j$ are from $\{z_1,z_2,\ldots,z_q\}$, $c_j,d_j$ are constants and $s_j,t_j$ are variables. Then Equation \eqref{yconda} yields \begin{align}\label{sud} & (c_1+d_1r)(s_{e+1}+t_{e+1}r) - (c_{e+1}+d_{e+1}r)(s_1+d_1r) +\notag\\ & (c_2+d_2r)(s_{e+2}+t_{e+2}r) - (c_{e+2}+d_{e+2}r)(s_2+d_2r) + \dots +\notag\\ & (c_e+d_er)(s_{2e}+t_{2e}r) - (c_{2e}+d_{2e}r)(s_e+d_er) = yr, \end{align} and using Lemma \ref{lemij=0}, Equation \eqref{sud} yields \begin{align}\label{ycondaa} & (c_1s_{e+1} + (c_1t_{e+1}+d_1s_{e+1})r) - (c_{e+1}s_1 + (c_{e+1}t_1+d_{e+1}s_1)r) +\notag\\ & (c_2s_{e+2} + (c_2t_{e+2}+d_2s_{e+2})r) - (c_{e+2}s_2 + (c_{e+2}t_2+d_{e+2}s_2)r) + \dots +\notag\\ & (c_es_{2e} + (c_et_{2e}+d_es_{2e})r) - (c_{2e}s_e + (c_{2e}t_e+d_{2e}s_e)r) = yr. \end{align} Now, let \begin{align}\label{eqcond5} (c_1s_{e+1} - c_{e+1}s_1) + (c_2s_{e+2}-c_{e+2}s_2) + \cdots + (c_es_{2e}-c_{2e}s_e) = z_{j_1}+z_{j_2}r \end{align} for some $j_1,j_2\in\{1,2,\ldots,q\}$. We proceed as in the proof of Proposition \ref{propmain1}, where we concluded that Equation \eqref{eq:group} is equivalent to Equations \eqref{eq:condition11} and \eqref{eq:condition12}. Here, we deduce that Equation \eqref{ycondaa} is equivalent to the system of the following two linear equations modulo $J$: \begin{align}\label{ycondabreak1} (c_1s_{e+1} - c_{e+1}s_1) + (c_2s_{e+2}-c_{e+2}s_2) + \cdots + (c_es_{2e}-c_{2e}s_e) \equiv 0 \pmod J \end{align} and \begin{align}\label{ycondabreak2} z_{j_2}+ \big((c_1t_{e+1}+d_1s_{e+1}) - (c_{e+1}t_1+d_{e+1}s_1)\big) + & \big((c_2t_{e+2}+d_2s_{e+2}-c_{e+2}t_2+d_{e+2}s_2)\big) + \dots + \notag\\ & \big((c_et_{2e}+d_es_{2e})-(c_{2e}t_e+d_{2e}s_e)\big) \equiv y \pmod J. \end{align} Now, we find the number of solutions of Equations \eqref{ycondb}, \eqref{ycondabreak1} and \eqref{ycondabreak2}. Since the $2e$-tuple $a$ has an invertible entry, there exists $i \in \{1,2,\ldots,2e\}$ such that $c_i \notin J$. Assume that $i$ belongs to $\{1,2,\ldots,e\}$ (otherwise, the proof is similar). Then Equation (\ref{ycondabreak2}) is equivalent to \begin{align}\label{ycondabreak2equiv} t_{e+i} \equiv &~c_{i}^{-1}\bigg(y-z_{j_2}+\big((c_{e+1}t_1+d_{e+1}s_1) - (c_1t_{e+1}+d_1s_{e+1})\big) +\\ &~\big((c_{e+2}t_2+d_{e+2}s_2) - (c_2t_{e+2}+d_2s_{e+2})\big)+ \dots + \notag\\ &~\big((c_{e+i-1}t_{i-1}+d_{e+i-1}s_{i-1}) - (c_{i-1}t_{e+i-1}+d_{i-1}s_{e+i-1})\big) + \notag\\ &~ \big((c_{e+i}t_i+d_{e+i}s_i) - d_is_{e+i}\big) + \notag\\ &~\big((c_{e+i+1}t_{i+1}+d_{e+i+1}s_{i+1}) - (c_{i+1}t_{e+i+1}+d_{i+1}s_{e+i+1})\big) + \dots + \notag\\ &~\big((c_{2e}t_e+d_{2e}s_e) - (c_et_{2e}+d_es_{2e})\big)\bigg) \pmod J \end{align} On the other hand, for every $j \in \{1,2,\ldots,2e\}$, let $h_j = w_jr$, where $w_j$ belongs to $\{z_1,z_2,\ldots,z_q\}$ (this is due to Lemma \ref{lempart1}). Then Equation \eqref{ycondb} yields \begin{align} \big(w_1r(s_{e+1}+t_{e+1}r)-w_{e+1}r(s_1+t_1r)\big) + & \big(w_2r(s_{e+2}+t_{e+2}r)-w_{e+2}r(s_2+t_2r)\big) + \dots + \notag\\ & \big(w_er(s_{2e}+t_{2e}r)-w_{2e}r(s_e+t_er)\big) = (z-y)r, \end{align} which is equivalent to \begin{align}\label{ycondbbreak} (w_1s_{e+1}-w_{e+1}s_1) + (w_2s_{e+2}-w_{e+2}s_2) + \dots + (w_es_{2e}-w_{2e}s_e) \equiv z-y \pmod J. \end{align} Thus, the system of equations we need to solve is equivalent to the system of Equations \eqref{ycondabreak1}, \eqref{ycondabreak2equiv} and \eqref{ycondbbreak}. Similar to the proof of Proposition \ref{propmain1}, we find that the system of Equations (\ref{ycondabreak1}) and (\ref{ycondbbreak}) has rank $2e-2$, as follows. Since the $2e$-tuple $a$ has an invertible entry and the $2e$-tuple $h$ is not identically $0$, each of Equations (\ref{ycondabreak1}) and (\ref{ycondbbreak}) is non-trivial. Let us show that Equations (\ref{ycondabreak1}) and (\ref{ycondbbreak}) are linearly independent. Suppose to the contrary they are dependent, that is, there exists $\delta \in \{z_2,\ldots,z_q\}$ such that, for every $j \in \{1,2,\ldots,2e\}$, we have \begin{equation}\label{eqcond2} c_j \equiv \delta w_j \pmod J. \end{equation} This implies that $ra$ belongs to $H$, a contradiction. Thus, there exist $\ell, k$ such that $1 \le \ell < k \le 2e$ holds and the system of Equations (\ref{ycondabreak1}) and (\ref{ycondbbreak}) is equivalent to the system of equations \begin{equation}\label{eqcond3} s_{\ell} \equiv M_{\ell}(s_1,s_2,\ldots,s_{\ell-1},s_{\ell+1},\ldots,s_{k-1},s_{k+1},\ldots,s_{2e},y,z) \pmod J \end{equation} and \begin{equation}\label{eqcond4} s_{k} \equiv M_k(s_1,s_2\ldots,s_{\ell-1},s_{\ell+1},\ldots,s_{k-1},s_{k+1},\ldots,s_{2e},y,z) \pmod J, \end{equation} where $M_{\ell}(s_1,s_2,\ldots,s_{\ell-1},s_{\ell+1},\ldots,s_{k-1},s_{k+1},\ldots,s_{2e},y,z)$ and $M_k(s_1,s_2,\ldots,s_{\ell-1},s_{\ell+1},\ldots,s_{k-1},s_{k+1},\ldots,$\\$s_{2e},y,z)$ are linear combinations of the $2e$ variables $s_1,s_2,\ldots,s_{\ell-1},s_{\ell+1},\ldots,s_{k-1},s_{k+1},\ldots,s_{2e},y,z$. Thus, the system of Equations (\ref{ycondabreak1}), (\ref{ycondabreak2equiv}) and (\ref{ycondbbreak}) is equivalent to the system of Equations (\ref{ycondabreak2equiv}), (\ref{eqcond3}) and (\ref{eqcond4}). Note that once the values of $s_1,s_2,\ldots,s_{2e}$ (and therefore the value of $z_{j_2}$ by Equation \eqref{eqcond5}) and $t_1,t_2,\ldots,t_{e+i-1},t_{e+i+1},\ldots,t_{2e}$ are determined, then the value of $t_{e+i}$ is also determined according to Equation (\ref{ycondabreak2equiv}). Now we are ready to count the number of solutions of the system of Equations (\ref{ycondabreak2equiv}), (\ref{eqcond3}) and (\ref{eqcond4}). Depending on the values of $y$ and $z$, we have the following two cases: \begin{itemize} \item Case 1: $y=z$, and \item Case 2: $y\neq z$. \end{itemize} Corresponding to Case 1, the system of Equations \eqref{eqcond3} and \eqref{eqcond4} in the variables $s_1,s_2,\ldots,s_{2e}$ has $q^{2e-2}$ solutions, but the zero solution must be excluded as the $2e$-tuple $x$ has an invertible entry. Thus, $q^{2e-2}-1$ solutions of Equations \eqref{eqcond3} and \eqref{eqcond4} satisfy our requirements. For every such a solution there are $q^{2e-1}$ solutions of Equation \eqref{ycondabreak2equiv} as the variables $t_1,t_2,\ldots,t_{e+i-1},t_{e+i+1},\ldots,t_{2e}$ are independent. Moreover, since $y,z\in\{z_2,\ldots,z_q\}$, there are $(q-1)$ possibilities of $y=z$. Thus, the total number of solutions of the system of Equations \eqref{ycondabreak2equiv}, \eqref{eqcond3} and \eqref{eqcond4} for Case 1 is $$ (q^{2e-2}-1)q^{2e-1}(q-1). $$ Corresponding to Case 2, the system of Equations \eqref{eqcond3} and \eqref{eqcond4} in the variables $s_1,s_2,\ldots,s_{2e}$ has $q^{2e-2}$ solutions. For every such a solution there are $q^{2e-1}$ solutions of Equation \eqref{ycondabreak2equiv}. Moreover, there are $(q-1)^2-(q-1)=(q-1)(q-2)$ possibilities of $y\neq z$. Thus, the total number of solutions of the system of Equations \eqref{ycondabreak2equiv}, \eqref{eqcond3} and \eqref{eqcond4} for Case 2 is $$ q^{2e-2}q^{2e-1}(q-1)(q-2). $$ So, we have that corresponding to Case 1, the vertices $[a]$ and $[a+h]$ have $$ \dfrac{(q^{2e-2}-1)q^{2e-1}(q-1)}{q^2-q} = (q^{2e-2}-1)q^{2e-2} $$ common neighbours in ${Y(2e,K)}$, and corresponding to Case 2, the vertices $[a]$ and $[a+h]$ have $$ \dfrac{q^{2e-2}q^{2e-1}(q-1)(q-2)}{q^2-q} = q^{2(2e-2)}(q-2) $$ common neighbours in ${Y(2e,K)}$. Finally, we combine Cases 1 and 2 and conclude that the vertices $[a]$ and $[a+h]$ have $$(q^{2e-2}-1)q^{2e-2}+q^{2(2e-2)}(q-2)=q^{4e-3}-q^{4e-4}-q^{2e-2}$$ common neighbours in $Y(2e,K)$, and the proof is complete. \end{proof} \begin{proposition}\label{prop:YTwoVerticesFromDifferentClasses} Let $H_1,H_2$ be hyperplanes of $T$, and let $a\in V'$, $b\in V'$ with $ra\notin H_1,rb\notin H_2$, such that $C(H_1,a)$ and $C(H_2,b)$ are distinct classes of the partition $\Pi(2e,K)$. Then $[a]$ and $[b]$ have $q^{2e-3}(q-1)(q^{2e-2}-1)$ common neighbours in $Y(2e,K)$. \end{proposition} \begin{proof} The proof goes along similar lines as that of Proposition \ref{propmain2}. Since $[b]$ does not belong to $C(H_1,a)$, we conclude that there exists no $\sigma \in K^\times$ and $h \in H_1$ such that $b = \sigma(a+h)$ holds. Let $[x]$ be a common neighbour of $[a]$ and $[b]$. Then we have a system of the equations \begin{equation}\label{eq:condition21y} (a_1x_{e+1}-a_{e+1}x_1) + (a_2x_{e+2}-a_{e+2}x_2) + \dots + (a_ex_{2e}-a_{2e}x_e) = yr \end{equation} and \begin{equation}\label{eq:condition22y} (b_1x_{e+1}-b_{e+1}x_1) + (b_2x_{e+2}-b_{e+2}x_2) + \dots + (b_ex_{2e}-b_{2e}x_e) = zr \end{equation} for some $y,z\in\{z_2,\ldots,z_q\}$. We consider the ordering of variables $x_{e+1},x_{e+2},\ldots,x_{2e},x_1,\ldots,x_e$ and write down the augmented matrix: $$ \left( \begin{array}{cccc|c} a_1 & a_2 & \dots & a_{2e}& yr\\ b_1 & b_2 & \dots & b_{2e}& zr \end{array} \right) $$ Note that there exist $i \in \{1,2,\ldots,2e\}$ such that $a_i$ is invertible. Then the matrix of coefficients can be written as \begin{align*} \left( \begin{array}{ccccc|c} a_1 & \dots & a_i & \dots & a_{2e}& yr\\ b_1 & \dots & b_i & \dots & b_{2e}& zr \end{array} \right), \end{align*} which is equivalent to \begin{align*} \left( \begin{array}{ccccc|c} a_1a_i^{-1} & \dots & 1 & \dots & a_{2e}a_i^{-1} & y ra_i^{-1}\\ b_1 & \dots & b_i & \dots & b_{2e} & zr \end{array} \right), \end{align*} and, consequently, \begin{align}\label{matt} \left( \begin{array}{ccccc|c} a_1a_i^{-1} & \dots & 1 & \dots & a_{2e}a_i^{-1}& yr a_i^{-1}\\ b_1-a_1a_i^{-1}b_i & \dots & 0 & \dots & b_{2e}-a_{2e}a_i^{-1}b_i &zr-yra_i^{-1}b_i \end{array} \right). \end{align} Since the matrix of coefficients in \eqref{matt} equals the matrix in \eqref{mat} with the same conditions on the entries in $a$ and $b$, we conclude (as in the proof of Proposition \ref{propmain1}) that there exists $\gamma\in \{1,2,\ldots,2e\}\setminus\{i\}$ such that $b_{\gamma}-a_{\gamma}a_i^{-1}b_i$ is invertible. Thus there exist $\ell, k \in \{1,2,\ldots,2e\}$ such that $1 \le \ell < k \le 2e$ and the system of Equations (\ref{eq:condition21y}) and (\ref{eq:condition22y}) is equivalent to the system of equations \begin{equation}\label{eq:condition25y} x_{\ell} = M_{\ell}(x_1,x_2,\ldots,x_{\ell-1},x_{\ell+1},\ldots,x_{k-1},x_{k+1},\ldots,x_{2e},y,z) \end{equation} and \begin{equation}\label{eq:condition26y} x_{k} = M_k(x_1,x_2,\ldots,x_{\ell-1},x_{\ell+1},\ldots,x_{k-1},x_{k+1},\ldots,x_{2e},y,z), \end{equation} where $M_{\ell}(x_1,x_2,\ldots,x_{\ell-1},x_{\ell+1},\ldots,x_{k-1},x_{k+1},\ldots,x_{2e},y,z)$ and $M_k(x_1,x_2,\ldots,x_{\ell-1},x_{\ell+1},\ldots,x_{k-1},x_{k+1},$\\$\ldots,x_{2e},y,z)$ are linear combinations of the $2e$ variables $x_1,x_2,\ldots,x_{\ell-1},x_{\ell+1},\ldots,x_{k-1},x_{k+1},\ldots,x_{2e},y$, and $z$. Now we are ready to count the number of solutions of the system of Equations (\ref{eq:condition25y}) and (\ref{eq:condition26y}) in the variables $x_1,x_2,\ldots,x_{2e},y$ and $z$. It has $(q^2)^{2e-2}(q-1)^2$ solutions, but the solutions $x$ such that all the entries are zero divisors must be excluded as the $2e$-tuple $x$ has an invertible entry. Thus, $((q^2)^{2e-2}-q^{2e-2})(q-1)^2$ solutions of Equations (\ref{eq:condition25y}) and (\ref{eq:condition26y}) satisfy our requirements, and we conclude that the vertices $[a]$ and $[b]$ have $$ \dfrac{(q^{2e-2}-1)q^{2e-2} (q-1)^2}{q^2-q}= q^{2e-3}(q-1)(q^{2e-2}-1) $$ common neighbours in ${Y(2e,K)}$, which completes the proof. \end{proof} Now Theorem \ref{thm:Main1} follows from Proposition \ref{prop1}, Proposition \ref{prop2}, Corollary \ref{cor:Partition}, Corollary \ref{cor:XTwoVerticesFromTheSameClass} and Corollary \ref{cor:XTwoVertices FromDifferentClasses}, and Theorem \ref{thm:Main2} follows from Proposition \ref{prop1}, Proposition \ref{prop3}, Corollary \ref{cor:Partition}, Proposition \ref{prop:YTwoVerticesFromTheSameClass} and Proposition \ref{prop:YTwoVerticesFromDifferentClasses}. \section{Concluding remarks}\label{sec:Remarks} In Corollary \ref{cor:Partition}, we showed the existence of a partition into classes $C(H,u)$, which is sufficient to prove Theorem \ref{thm:Main1} and Theorem \ref{thm:Main2}. However, the disadvantage of our results is the the canonical partition is given implicitly. The following problem naturally arises. \begin{problem} Let $m$ be the number of classes of the canonical partition of the graphs $X(2e,K)$ and $Y(2e,K)$ from Theorem \ref{thm:Main1} and Theorem \ref{thm:Main2}, which is equal to the number of $2e-1$-dimensional subspaces in a $2e$-dimensional vector space over a finite field $\mathbb{F}_q$. How can we explicitly choose hyperplanes $H_1,H_2,\ldots,H_m$ and tuples $u_1,u_2,\ldots,u_m$ such that $C(H_1,u_1), C(H_2,u_2), \ldots, C(H_m,u_m)$ is the canonical partition of the graphs $X(2e,K)$ and $Y(2e,K)$? \end{problem} \section*{Acknowledgements} \label{Ack} This work is supported by Natural Science Fundation of Hebei Province (A2023205045). \begin{thebibliography}{00} \bibitem{B77} R.C. Bose, \emph{Symmetric group divisible designs with the dual property}, J. Statist. Plann. Inference 1 (1977) 87--101. \url{https://doi.org/10.1016/0378-3758(77)90008-8} \bibitem{BV22} A. E. Brouwer, H. Van Maldeghem, \emph{Strongly regular graphs}, Cambridge University Press, Cambridge (2022). \bibitem{CH14} D. Crnkovi\'c, W. H. Haemers, \emph{Walk-regular divisible design graphs}, Designs, Codes and Cryptography, 72 (2014) 165--175. \url{https://doi.org/10.1007/s10623-013-9861-0} \bibitem{CS22} D. Crnkovi\'c, A. \v{S}vob, \emph{New constructions of divisible design Cayley graphs}, Graphs and Combinatorics, 38 (2022) Article number 17. \url{https://doi.org/10.1007/s00373-021-02440-4} \bibitem{DGHS24} B. De Bruyn, S. Goryainov, W. Haemers, and L. Shalaginov, \emph{Divisible design graphs from the symplectic graph}, arXiv:2404.09902, \url{https://arxiv.org/abs/2404.09902} \bibitem{DF04} D. S. Dummit, R. M. Foote, {\it Abstract Algebra}, Third Edition, John Wiley \& Sons, Inc. (2004). \bibitem{EFHHH99} M. Erickson, S. Fernando, W.H. Haemers, D. Hardy, J. Hemmeter, \emph{Deza graphs: A generalization of strongly regular graphs}, Journal of Combinatorial Designs, 7 (1999) 395--405. \url{http://doi.org/10.1002/(SICI)1520-6610(1999)7:6 \bibitem{LW08} F. Li, Y. Wang, \emph{Subconstituents of symplectic graphs}, European Journal of Combinatorics, 29 (2008) 1092--1103. \url{http://doi.org/10.1016/j.ejc.2007.08.001} \bibitem{GK24} A.L. Gavrilyuk, V.V. Kabanov, \emph{Strongly regular graphs decomposable into a divisible design graph and a Hoffman coclique}, Designs, Codes and Cryptography, 92, 1379--1391 (2024). \url{https://doi.org/10.1007/s10623-023-01348-9} \bibitem{GHKS19} S. Goryainov, W. H. Haemers, V. V. Kabanov, L. Shalaginov, \emph{Deza graphs with parameters $(n, k, k-1, a)$ and $\beta = 1$}, Journal of Combinatorial Designs, 27 (2019) 188--202. \url{https://doi.org/10.1002/jcd.21644} \bibitem{G13} Z. Gu, \emph{Subconstituents of symplectic graphs modulo $p^n$}, Linear Algebra and its Applications, 439 (2013) 1321--1329. \url{https://doi.org/10.1016/j.laa.2013.04.015} \bibitem{GWZ13} Z. Gu, Z. Wan, K. Zhou, \emph{Subconstituents of orthogonal graphs of odd characteristic~--~continued}, Linear Algebra and its Applications 439 (2013) 2861--2877. \url{https://doi.org/10.1016/j.laa.2013.08.010} \bibitem{GLW10} J. Guo, F. Li, K. Wang, \emph{Deza graphs based on symplectic spaces}, European Journal of Combinatorics, 31 (2010) 1969--1980. \url{http://doi.org/10.1016/j.ejc.2010.05.006} \bibitem{GLW14} J. Guo, F. Li, K. Wang, \emph{Orthogonal graphs over Galois rings of odd characteristic}, European Journal of Combinatorics, 39 (2014) 113--121. \url{https://doi.org/10.1016/j.ejc.2014.01.002} \bibitem{HKM11} W. H. Haemers, H. Kharaghani, M. A. Meulenberg, \emph{Divisible design graphs}, Journal of Combinatorial Theory, Series A, 118 (2011) 978--992. \url{https://doi.org/10.1016/j.jcta.2010.10.003} \bibitem{H68} T. Hungerford, {\it On the structure of principal ideal rings}, Pacific Journal of Mathematics 25 (1968), no. 3, 543--547. \bibitem{K22} V. V. Kabanov, \emph{New versions of the Wallis-Fon-Der-Flaass construction to create divisible design graphs}, Discrete Mathematics, 345(11) (2022) 113054. \url{https://doi.org/10.1016/j.disc.2022.113054} \bibitem{K23} V. V. Kabanov, \emph{A new construction of strongly regular graphs with parameters of the complement symplectic graph}, The Electronic Journal of Combinatorics, 30(1) (2023) \#P1.25. \url{https://doi.org/10.37236/11343} \bibitem{KS21} V. V. Kabanov, L. Shalaginov, \emph{On divisible design Cayley graphs}, The Art of Discrete and Applied Mathematics, 4(2) (2021) \#P2.02. \url{https://doi.org/10.26493/2590-9770.1340.364} \bibitem{LGW14} F. Li, J. Guo, K. Wang, \emph{Orthogonal graphs over Galois rings of odd characteristic}, European Journal of Combinatorics 39 (2014) 113--121. \url{https://doi.org/10.1016/j.ejc.2014.01.002} \bibitem{LWG13} F. Li, K. Wang, J. Guo, \emph{More on symplectic graphs modulo $p^n$}, Linear Algebra and its Applications, 438 (2013) 2651--2660. \url{https://doi.org/10.1016/j.laa.2012.06.026} \bibitem{MP11} Y. Meemark, T. Prinyasart, {\it On symplectic graphs modulo $p^n$}, Discrete Mathematics 311 (2011), no. 17, 1874--1878. \url{https://doi.org/10.1016/j.disc.2011.05.005} \bibitem{MP13} Y. Meemark, T. Puirod, \emph{Symplectic graphs over finite local rings}, European Journal of Combinatorics, 34 (2013) 1114--1124. \url{https://doi.org/10.1016/j.ejc.2013.03.003} \bibitem{M08} M.A. Meulenberg, \emph{Divisible design graphs}, Master's thesis, Tilburg University (2008). \url{http://alg.imm.uran.ru/dezagraphs/Divisible_design_graphs_M.A._Meulenberg.pdf} \bibitem{N18} A. Nowicki, \emph{Tables of finite commutative local rings of small orders}, 2018, \url{https://www.researchgate.net/profile/Andrzej-Nowicki-2/publication/328319576} \bibitem{P} D. Panasenko, \emph{Strictly Deza graphs}, Website, \url{http://alg.imm.uran.ru/dezagraphs/main.html} \bibitem{P22} D. Panasenko, \emph{The vertex connectivity of some classes of divisible design graphs}, Siberian Electronic Mathematical Reports, 19(2) (2022) 426--438. \url{http://semr.math.nsc.ru/v19/n2/p426-438.pdf} \bibitem{PS22} D. Panasenko, L. Shalaginov, \emph{Classification of divisible design graphs with at most $39$ vertices}, Journal of Combinatorial Designs, 30(4) (2022) 205--219. \url{https://doi.org/10.1002/jcd.21818} \bibitem{S21} L. Shalaginov, \emph{Divisible design graphs with parameters $(4n, n+2, n-2, 2, 4, n)$ and $(4n, 3n-2, 3n-6, 2n-2, 4, n)$}, Siberian Electronic Mathematical Reports, 18(2) (2021) 1742--1756. \url{https://doi.org/10.33048/semi.2021.18.134} \bibitem{T22} L. Tsiovkina, \emph{Covers of complete graphs and related association schemes}, Journal of Combinatorial Theory, Series A, 191 (2022) Article ID 105646. \url{https://doi.org/10.1016/j.jcta.2022.105646} \end{thebibliography} \end{document}
2412.04952v1
http://arxiv.org/abs/2412.04952v1
Non-isomorphic maximal function fields of genus $q-1$
\documentclass[12pt,a4]{amsart} \setlength{\textwidth}{\paperwidth} \addtolength{\textwidth}{-2in} \calclayout \usepackage{xcolor} \usepackage{amsmath, amssymb, amsthm, amscd,color,comment} \usepackage[all,cmtip]{xy} \let\objectstyle=\displaystyle \usepackage{mathrsfs} \usepackage{tabularx} \usepackage{booktabs} \usepackage{enumitem} \usepackage{comment} \usepackage{tikz} \usetikzlibrary{calc} \usepackage[labelfont=bf,format=plain,justification=raggedright,singlelinecheck=false]{caption} \newcommand{\cc}{\mathfrak{c}} \newcommand{\al}{\alpha} \newcommand{\T}{\tilde{T}} \newcommand{\PP}{\mathcal{P}} \newcommand{\QQ}{\mathcal{Q}} \newcommand{\F}{\mathbb{F}} \newcommand{\Z}{\mathbb{Z}} \newcommand{\X}{\mathcal{X}} \newcommand{\Div}{\mathrm{Div}} \newcommand{\PGU}{\mathrm{PGU}} \newcommand{\aut}{\mathrm{Aut}} \newcommand{\Fq}{\mathbb{F}_q} \newcommand{\Fqq}{\mathbb{F}_{q^2}} \renewcommand{\vec}[1]{{\bf #1}} \newcommand{\Fr}{\mathrm{Fr}} \newcommand{\wt}{\mathrm{wt}} \newcommand{\ev}{\mathrm{ev}} \newcommand{\im}{\mathrm{im}} \newcommand{\Pinf}{P_{\infty}} \newcommand{\val}{v_{\Pinf}} \newcommand{\MGS}{\mathcal M_{s,\ell}(D,G,A)} \newcommand{\mR}{\mathbb{R}} \newcommand{\mZ}{\mathbb{Z}} \newcommand{\mN}{\mathbb{N}} \newcommand{\mQ}{\mathbb{Q}} \newcommand{\mC}{\mathbb{C}} \newcommand{\mB}{\mathbb{B}} \newcommand{\mP}{\mathbb{P}} \newcommand{\mL}{\mathcal{L}} \newcommand{\mF}{\mathbb{F}} \newcommand{\mO}{\mathcal{O}} \newcommand{\mT}{\mathcal{T}} \newcommand{\mG}{\mathbb{G}} \newcommand{\cF}{\mathcal{F}} \newcommand{\cH}{\mathcal{H}} \newcommand{\tcF}{\tilde{\mathcal{F}}} \DeclareMathOperator{\Tr}{Tr} \DeclareMathOperator{\N}{N} \numberwithin{equation}{section} \theoremstyle{plain} \newtheorem{theorem}[equation]{Theorem} \newtheorem{corollary}[equation]{Corollary} \newtheorem{lemma}[equation]{Lemma} \newtheorem{proposition}[equation]{Proposition} \theoremstyle{definition} \newtheorem{defn}[equation]{Definition} \theoremstyle{remark} \newtheorem{remark}[equation]{Remark} \newtheorem{example}[equation]{Example} \newtheorem{notation}[equation]{Notation} \newtheorem{terminology}[equation]{Terminology} \newtheorem{fact}[equation]{Fact} \usepackage{bookmark} \usepackage{hyperref} \begin{document} \title{Non-isomorphic maximal function fields of genus $q-1$} \thanks{$^1$ Technical University of Denmark, Kgs. Lyngby, Denmark, [email protected]} \thanks{{\bf Keywords}: Hermitian function field; Maximal function field; Isomorphism classes; Automorphism group} \thanks{{\bf Mathematics Subject Classification (2010)}: 11G, 14G} \author{Jonathan Niemann$^1$} \begin{abstract} The classification of maximal function fields over a finite field is a difficult open problem, and even determining isomorphism classes among known function fields is challenging in general. We study a particular family of maximal function fields defined over a finite field with $q^2$ elements, where $q$ is the power of an odd prime. When $d := (q+1)/2$ is a prime, this family is known to contain a large number of non-isomorphic function fields of the same genus and with the same automorphism group. We compute the automorphism group and isomorphism classes also in the case where $d$ is not a prime. \end{abstract} \maketitle \section{Introduction} Function fields over finite fields with many rational places have been studied extensively in the past decades, partly due to the role they play in constructing error-correcting codes with good parameters. The number of rational places of such a function field is bounded from above by the Hasse-Weil bound. In fact, if $\cF$ is a function field defined over $\Fqq$, then $$ N(\cF) \leq q^2 + 1 + 2g(\cF)q, $$ where $g(\cF)$ is the genus of $\cF$ and $N(\cF)$ is the number of places of degree one over $\Fqq$. A function field attaining this bound is called $\Fqq$-maximal, and the classification of all $\Fqq$-maximal function fields is a big open problem. Perhaps the most studied example of a $\Fqq$-maximal function field is the Hermitian function field $$ \cH := \Fqq(x,y) \ \text{ with } y^{q+1} = x^q + x. $$ It has genus $g(\cH) = q(q-1)/2$, which is largest possible for a maximal function field over $\Fqq$, and any other maximal function field with the same genus is isomorphic to $\cH$ (see \cite{ihara_some_remarks_1982} and \cite{ruck_characterization_1994}). Moreover, the automorphism group of $\cH$ is exceptionally large; it is isomorphic to $\mathrm{PGU}(3,q)$ which has order $q^3(q^2-1)(q^3+1)$. Any subfield of a maximal function field is again maximal (see \cite{Serre}), and the subfields of $\cH$ corresponding to subgroups of $\mathrm{PGU}(3,q)$ have turned out to be a rich source of examples of maximal function fields (see e.g. \cite{garcia_subfields_2000}). In many cases, these examples come in families of function fields with the same genus, and it is natural to ask for a description of the isomorphism classes within such families. It is, in general, a difficult task to determine whether two function fields of the same genus are isomorphic or not. Knowing also the automorphism group is sometimes sufficient, but there are examples of non-isomorphic maximal function fields of the same genus with isomorphic automorphism groups. One such example is given by the family of function fields corresponding to the curves studied in \cite{giulietti_m=2_curves_2006}. Let $q$ be a power of an odd prime such that $d = (q+1)/2 > 3$ is prime. Then these function fields are of the form $$ \cF_i := \Fqq(x,y) \ \text{ with } y^{q+1} = x^{2i}(x^2 + 1), $$ for $1 \leq i \leq d-2$. They are subfields of the Hermitian (see \cite[Example 6.4]{garcia_subfields_2000}), and the isomorphism classes and automorphism groups were determined in \cite{giulietti_m=2_curves_2006}. Another example of non-isomorphic maximal function fields of the same genus was given in \cite{beelen_families_2024}. For $q$ a power of an odd prime and $d = (q+1)/2$, not necessarily prime, these function fields are of the form $$ F_j := \Fqq(x,y) \ \text{ with } y^d = x^j(x^2 + 1), $$ for $j \in \mZ$ with $\gcd(j(j+2),d) =1$. They are subfields of the Hermitian (see \cite[Example 6.4]{garcia_subfields_2000}, and the isomorphism classes as well as the automorphism groups were described in \cite{beelen_families_2024}, except for the automorphism group of $F_{(d-2)/2}$ which is still unknown. In this paper, we will extend most of the results of \cite{giulietti_m=2_curves_2006} to also include the case where $d = (q+1)/2$ is not a prime. In particular, we will determine the isomorphism classes and the full automorphism group for the function fields $\{\cF_i\}_i$, and count the number of isomorphism classes. The structure of the automorphism group is given in Theorem \ref{thm:aut}, while the results regarding the isomorphism classes are collected in Theorem \ref{thm:main_iso_classes} and Theorem \ref{thm:number_iso_classes}. The core idea is to consider the degree two subfields of $\cF_i$. It turns out that these subfields are, in many cases, isomorphic to fields of the form $F_{j}$, as defined above. Our results then follow from combining the findings of \cite{beelen_families_2024} with a careful study of the automorphism group of $\cF_i$. The paper is organized as follows: In Section \ref{sec:the_function_fields} we recall some initial observations regarding the function fields $\{\cF_i\}_i$. This includes a description of some divisors, automorphisms and in particular a number of explicit isomorphisms among the function fields. The rest of the paper is then concerned with showing that no other isomorphisms exist. In Section \ref{sec:subext} we describe the degree two subfields mentioned above, and in Section \ref{sec:weierstrass} we obtain partial results regarding the the Weierstrass semigroups at some special rational places. The automorphism group of $\cF_i$ is completely determined in Section \ref{sec:aut}, and finally the isomorphism classes are described and counted in Section \ref{sec:iso}. \section{The function fields $\cF_i$}\label{sec:the_function_fields} Let $q$ be the power of an odd prime and define $d = (q+1)/2$. We study the family of function fields of the form $\mathcal{F}_i := \F_{q^2}(x,y)$ where \begin{equation}\label{eq:Fi} y^{q+1} = x^{2i}(x^2 + 1), \end{equation} for $i\in\mZ$ with $\gcd(i(i+1),d) = 1$. By making the change of variables $y' := ay$, for some $a\in \Fqq$ satisfying $a^{q+1}=-1$, we see that $\cF_i$ belongs to the class the function fields considered in \cite[Example 6.4, Case 2]{garcia_subfields_2000}. It follows that $\cF_i$ is a subfield of the Hermitian function field, and hence $\Fqq$-maximal with $p$-rank zero (see \cite[Lemma 9.73]{hirschfeld_algebraic_2008}). Moreover, the genus of $\cF_i$ is $q-1$, since we are assuming $\gcd(i(i+1),d)=1$. In \cite{giulietti_m=2_curves_2006} these function fields were studied in the case where $d$ is prime. In this section, we recall some properties of $\cF_i$ that hold for any $d$. \subsection{Some divisors and special places}\label{sec:divisors_and_omega} Let $\alpha \in \Fqq$ be some element satisfying $\alpha^2 = -1$. By considering $\cF_i$ as a Kummer extension of $\Fqq(x)$ (see \cite[Proposition 3.7.3]{Sti}), we determine the following divisors in $\cF_i$: \begin{align}\label{eq:divisors} \begin{split} (x) &= d(P_0^1 + P_0^2) - d(P_\infty^1 + P_\infty^2) \\ (y) &= i (P_0^1 + P_0^2) + (P_\alpha + P_{-\alpha}) - (i+1)(P_\infty^1 + P_\infty^2), \text{ and } \\ (dx) &= (d-1) (P_0^1 + P_0^2) + Q (P_\alpha + P_{-\alpha}) - (d+1)(P_\infty^1 + P_\infty^2), \end{split} \end{align} where $P_0^1$ and $P_0^2$ (respectively $P_\infty^1$ and $P_\infty^2$) are the places lying above the zero (respectively pole) of $x$ in $\Fqq(x)$, and $P_\alpha$ (respectively $P_{-\alpha}$) is the place lying above the zero of $(x-\alpha)$ (respectively $(x+\alpha)$). We denote the set of these six places by $\Omega$. \newline In Section \ref{sec:weierstrass} we will describe the gapsequences of the places of $\Omega$. The key to obtaining this description is the connection between gaps and regular differentials given by the following result: \begin{proposition}\cite[Corollary 14.2.5]{villa_salvador_topics_2006}\label{prop:reg_diff_gap} Let $F$ be an algebraic function field of genus $g$ over some field $K$. Let $P$ be a place of $F$ and $\omega$ a regular differential on $F$. Then $v_P(\omega) + 1$ is a gap at $P$. \end{proposition} In the special case $i = 1$ we will use the information on the semigroups to determine the automorphism group of $\cF_1$. \subsection{First observations regarding the automorphism groups} We write $\aut(\cF_i)$ for the $\overline{\F}_{q^2}$-automorphism group of $\overline{\F}_{q^2}\cF_i$. Note that this is the same as the $\F_{q^2}$-automorphism group since $\cF_i$ is $\F_{q^2}$-maximal. We immediately find that $\aut(\cF_i)$ contains a subgroup isomorphic to $\mZ_2 \times \mZ_{q+1}$. Indeed, we have $$ H_i := \{ \sigma : (x,y) \mapsto (ax,by) \mid a,b\in \F_{q^2}, a^2 = b^{q+1} = 1\} \subseteq \aut(\cF_i). $$ Note that $d$ is odd since $\gcd(i(i+1),d) = 1$, so $q+1 \equiv 2 \pmod 4$. This means that the unique Sylow $2$-group of $H_i$ is isomorphic to $\mZ_2 \times \mZ_2$. In particular, $H_i$ contains three involutions that give rise to three subfields, $F$, $F'$, and $F''$, of $\cF_i$ satisfying $[\cF_i : F] =[\cF_i : F'] =[\cF_i : F''] = 2$. We will study these subfields further in Section \ref{sec:subext}. In fact, it turns out that they are isomorphic to fields of the type studied in \cite{beelen_families_2024}, and this will be the key to understanding the isomorphism classes of $\{\cF_i\}_i$. \subsection{Explicit isomorphisms}\label{sec:explicit_iso} We will determine the isomorphism classes in $\{\cF_i\}_{i}$ by pointing out a number of explicit isomorphisms and then showing that no more isomorphisms exist. The explicit isomorphisms are similar to those described in \cite[Section 7]{giulietti_m=2_curves_2006}:\newline If $i \equiv j \pmod d$ then $\cF_i$ is isomorphic to $\cF_j$. Indeed, write $j = md + i$ for some $m\in \mZ$, then $\varphi: \cF_i \to \cF_j$ given by $(x,y) \mapsto (x,y/x^m)$ is an isomorphism. Similarly, if $i \equiv -j - 1 \mod d$ then $\cF_i$ is isomorphic to $\cF_j$. The isomorphism is given by $(x,y) \mapsto (1/x, y/x^m)$ where $m\in \mZ$ is chosen such that $i = md - j - 1$. This means that we can limit ourselves to studying the function fields corresponding to $i = 1, \dots, \frac{d-1}{2}$ where $\gcd(i(i+1),d)=1$. \newline Now choose $a\in \Fqq$ such that $a^{q+1} = -1$. We describe the rest of the explicit isomorphisms at the same time: \newline If $i,j \in \mZ$ with $\gcd(i(i+1),d) = \gcd(j(j+1),d) = 1$ satisfy either \begin{alignat*}{2} &(1)& \quad ij &\equiv 1 \pmod d, \\ &(2)& \quad ij + i + 1 &\equiv 0 \pmod d, \\ &(3)& \quad ij + i + j &\equiv 0 \pmod d, \text{ or } \\ &(4)& \quad ij + j + 1 &\equiv 0 \pmod d, \end{alignat*} then $\cF_i$ and $\cF_j$ are isomorphic and an isomorphism from $\cF_i$ to $\cF_j$ is given by respectively \begin{alignat*}{3} &(1)& \quad(x,y) \mapsto \left(\frac{a^dy^d}{x^{j}}, \frac{a^{i+1}y^{i}}{x^r}\right), \ & \text{ with } r := (ij - 1)/d, \\ &(2)& \quad (x,y) \mapsto \left(\frac{x^j}{a^dy^d}, \frac{x^r}{a^iy^{i+1}}\right), \ & \text{ with } r := (ij + i + 1)/d, \\ &(3)& \quad(x,y) \mapsto \left(\frac{x^{j+1}}{a^dy^d}, \frac{x^r}{a^iy^{i+1}}\right), \ & \text{ with } r := (ij + i + j)/d, \text{ and }\\ &(4)& \quad (x,y) \mapsto \left(\frac{a^dy^d}{x^{j+1}}, \frac{a^{i+1}y^{i}}{x^r}\right), \ & \text{ with } r := (ij + j + 1)/d. \end{alignat*} In Section \ref{sec:iso} we will show that there are no other isomorphisms. For now, note that $(3)$ gives rise to an isomorphism between $\cF_1$ and $\cF_{(d-1)/2}$, so we can limit our considerations to $i = 1, \dots, (d-3)/2$, satisfying $\gcd(i(i+1),2) =1$. We will continue with this simplification throughout the rest of the paper, except in the case $q=5$ where $(d-1)/2 = 1$. We will treat this case separately in the next section, after making some remarks regarding other special cases. \subsection{The special cases}\label{sec:special} There are two cases where the isomorphisms described above immediately give rise to extra automorphisms. \newline If $i^2 + i + 1 \equiv 0 \pmod d$ then the isomorphism from $(2)$ gives rise to an extra automorphism of the form $$ \omega: (x,y) \mapsto \left( \frac{x^i}{a^d y^d}, \frac{x^r}{a^iy^{i+1}} \right), $$ where $r := (i^2 + i + 1)/d$ and $a$ is as above. It can be checked directly that this automorphism has order three, and that it acts as a 3-cycle on the subfields $F$, $F'$, and $F''$. Similarly, if $i = 1$ then the isomorphism from $(1)$ gives rise to an extra automorphism $$ \omega_1: (x,y) \mapsto \left( \frac{a^dy^d}{x}, a^2y\right). $$ By pre-composing with the automorphism $(x,y) \mapsto (\pm x, 1/a^2 y)$ from $H_1$, we obtain two extra involutions in $\aut(\cF_1)$, namely $$ \pi : (x,y) \mapsto \left( \frac{a^dy^d}{x},y\right), $$ and $$ \pi' : (x,y) \mapsto \left( -\frac{a^dy^d}{x},y\right). $$ The case $q=5$ is extra special; we have $d = 3$, so for $i=1$ we get additional automorphisms from both $(2)$ and $(1)$. The genus is $q-1 = 4$, which is equal to second largest possible genus for a maximal curve over $\mathbb{F}_{5^2}$, so $\cF_1$ is isomorphic to the function field $\mathbb{F}_{5^2}(s,t)$ defined by $t^3 = s^5 + s$ (see \cite[Theorem 3.1]{fuhrmann_maximal_1997}). The automorphism group of this function field is known to be a group of order $360 = 60(q+1)$, and it is isomorphic to the semidirect product of a cyclic group of order $3$ and $\mathrm{PGL}(2,5)$ (see \cite[Theorem 12.11]{hirschfeld_algebraic_2008}). The number of isomorphism classes in $\{\cF_i\}_i$ is just one for $q=5$. Since this case is now completely settled, we will often assume $q > 5$ in the following to simplify matters. \section{Three subfields of $\cF_i$ of degree two}\label{sec:subext} Assume for the rest of this section that $q > 5$. For a fixed index $i$, satisfying $1\leq i \leq \frac{d-3}{2}$ and $\gcd(i(i+1),d)=1$, we describe the three subfields associated to the involutions of $H_i$. We claim that each of them is isomorphic to a function field of the form $F_j := \F_{q^2}(z,t)$ with $$ z^d = t^j(t^2+1), $$ where $1 \leq j \leq \frac{d-3}{2}$ or $j = d-1$ and $\gcd(j(j+2),d)=1$. These are function fields of the type studied in \cite{beelen_families_2024}. \newline First, we find a degree two subfield fixed by the involution $\sigma_0:(x,y) \mapsto (x,-y)$. Let $t_0 := y^2$ and note that $$ t_0^d = x^{2i}(x^2+1). $$ This shows that the subfield $\F_{q^2}(x,t_0) \subseteq \cF_i$ is isomorphic to $F_{2i}$. If $1\leq 2i \leq \frac{d-3}{2}$ we are done since the $\gcd$-condition follows from the $\gcd$-assumption on $i$. Otherwise, we use the isomorphism from \cite[Lemma 3.2]{beelen_families_2024}: Define $\tilde{x} := 1/x$ and $\tilde{t}_0 := t_0/x$ and note that $$ \tilde{t}_0^d = \tilde{x}^{d-2i-2}(\tilde{x}^2+1). $$ This shows that $\F_{q^2}(x,t_0) = \F_{q^2}(\tilde{x},\tilde{t}_0) \subseteq \cF_i$ is isomorphic to $F_{d-2i-2}$. Since $\frac{d-1}{2} \leq 2i \leq d-3$ (using that $d$ is odd), we have $$ d-2-(d-3) \leq d-2i-2 \leq d-2-\frac{d-1}{2}, $$ i.e. $$ 1 \leq d-2i-2 \leq \frac{d-3}{2}. $$ Moreover, $$ \gcd\left((d-2i-2)(d-2i),d\right) = \gcd\left(2i(2i+2),d\right) = \gcd\left(i(i+1),d\right) = 1, $$ since $d$ is odd. This finishes the proof of the claim for $\sigma_0$. \newline For the two other involutions of $H_i$ we need to consider several different cases. Since $\gcd(i(i+1),d)=1$, there is a unique $j \in \{1, \dots, d-1\}$ such that $j$ is an inverse of $i$ modulo $d$. The first two cases depend on whether $j$ is in $\{1, \dots, \frac{d-1}{2}\}$ or in $\{\frac{d+1}{2}, \dots, d-1\}$. Case 3 and 4 depend instead on the inverse of $i+1$ modulo $d$. In each case, the last part of the argument above is needed, but we will not repeat it. \newline \textbf{Case 1:} Suppose there exists $j\in \mZ$ such that $1\leq j \leq \frac{d-1}{2}$ and $ij \equiv 1 \pmod d$. If $j = \frac{d-1}{2}$, then $i \equiv 2 \pmod d$, but this is in contradiction with our assumption on $i$, so we may assume $1 \leq j \leq \frac{d-3}{2}$. We now use the isomorphism $(1)$ given in Section \ref{sec:explicit_iso}. Define $r := \frac{ij-1}{d}$ and pick $a \in \F_{q^2}$ such that $a^{q+1} = -1$. Further, define $x_1 := \frac{a^d y^d}{x^i}$ and $y_1 := \frac{a^{j+1} y^j}{x^r}$. Then, one can check directly that $$ y_1^{q+1} = x_1^{2j}(x_1^2 + 1). $$ Proceeding like above, we define $t_1 := y_1^2$ and obtain a subfield isomorphic to $F_{2j}$. Note that the $\gcd$-condition is satisfied for $2j$ and $2j+2$: \newline It follows from $ij \equiv 1 \pmod d$ that $\gcd(2j,d)=1$. Since $(j+1)(i+1) \equiv (i + 1) + (j + 1) \pmod d$ and $\gcd((i+1),d)=1$ we also get $\gcd(2j+2,d)=\gcd(j+1,d)=1$. \newline This means we can copy the argument above and finish the proof of the claim in this case. From the explicit description we see that this subfield is fixed by $\sigma_1:(x,y) \mapsto (-x,y)$ if $i$ is even and $\sigma_2:(x,y) \mapsto (-x,-y)$ if $i$ is odd. \newline \textbf{Case 2:} Suppose there exists $j_0 \in \mZ$ such that $\frac{d+1}{2} \leq j_0 \leq d-1$ and $ij_0 \equiv 1 \pmod d$. Note that $j_0 = d-1$ would imply $i\equiv -1 \pmod d$ which is impossible since we assume $1\leq i \leq \frac{d-3}{2}$. Using this, we get that $j := d-(j_0+1)$ satisfies $$ 1\leq j \leq \frac{d-3}{2}, $$ and $$ ij + i + 1 \equiv -ij_0 - i + i + 1 \equiv 0 \mod d. $$ We now use the isomorphism $(2)$ given in Section \ref{sec:explicit_iso}. Define $r := (ij + i + 1)/d$, $a$ like above, $x_2 := \frac{x^i}{a^d y^d}$, and $y_2 := \frac{x^r}{a^j y^{j+1}}$. Then, we have $$ y_2^{q+1} = x_2^{2j}(x_2^2 + 1). $$ Proceeding as before we define $t_2 := y_2^2$ and obtain a subfield isomorphic to $F_{2j}$. The $\gcd$-condition is satisfied since $$ \gcd(2j(2j+2),d) = \gcd(j(j+1),d) = \gcd(j_0(j_0+1),d) = 1, $$ and we finish with the same argument as previously. Note that this subfield is also fixed by $\sigma_1:(x,y) \mapsto (-x,y)$ if $i$ is even and $\sigma_2:(x,y) \mapsto (-x,-y)$ if $i$ is odd. \newline \textbf{Case 3:} Suppose there exists $j_0 \in \mZ$ such that $1 \leq j_0 \leq \frac{d-1}{2}$ and $(i+1)j_0 \equiv 1 \pmod d$. Note that $j_0 = 1$ would imply $i \equiv 0 \pmod d$ which is impossible. Using this, we get that $j := j_0-1 $ satisfies $$ 1\leq j \leq \frac{d-3}{2}, $$ and $$ ij + i + j \equiv ij_0 - i + i + j_0 - 1 \equiv 0 \mod d. $$ We now use the isomorphism $(3)$ given in Section \ref{sec:explicit_iso}. Define $r := (ij + i + j)/d$, $a$ like above, $x_3 := \frac{x^{i+1}}{a^d y^d}$, and $y_3 := \frac{x^r}{a^j y^{j+1}}$. Then, we have $$ y_3^{q+1} = x_3^{2j}(x_3^2 + 1). $$ Proceeding like above we define $t_3 := y_3^2$ and obtain a subfield isomorphic to $F_{2j}$. The $\gcd$-condition is satisfied since $$ \gcd(2j(2j+2),d) = \gcd(j(j+1),d) = \gcd((j_0-1)j_0,d) = \gcd(ij_0^2,d) = 1, $$ and we are again in a situation where we can easily finish the argument. This subfield is fixed by $\sigma_1:(x,y) \mapsto (-x,y)$ if $i$ is odd and $\sigma_2:(x,y) \mapsto (-x,-y)$ if $i$ is even. \newline \textbf{Case 4:} Suppose there exists $j_0 \in \mZ$ such that $\frac{d+1}{2} \leq j_0 \leq d-1$ and $(i+1)j_0 \equiv 1 \pmod d$. Now, $j := -j_0+d $ satisfies $$ 1\leq j \leq \frac{d-1}{2}, $$ and $$ ij + j + 1 \equiv -ij_0 - j_0 + 1 \equiv 0 \mod d. $$ We now use the isomorphism $(4)$ given in Section \ref{sec:explicit_iso}. Define $r := (ij + j+1)/d$, $a$ like above, $x_4 := \frac{a^d y^d}{x^{i+1}}$, and $y_4 := \frac{a^{j+1} y^j}{x^r}$. Then, we have $$ y_4^{q+1} = x_4^{2j}(x_4^2 + 1). $$ Proceeding like before, we define $t_4 := y_4^2$ and obtain a subfield isomorphic to $F_{2j}$. The $\gcd$-condition is satisfied since $$ \gcd(2j(2j+2),d) = \gcd(j(j+1),d) = \gcd(j_0(1-j_0),d) = \gcd(ij_0^2,d) = 1. $$ If $\1 \leq 2j \leq \frac{d-3}{2}$ or $2j = d-1$ we are done. Otherwise we copy the argument from previously. Note that this subfield is also fixed by $\sigma_1:(x,y) \mapsto (-x,y)$ if $i$ is odd and $\sigma_2:(x,y) \mapsto (-x,-y)$ if $i$ is even. \newline By combining all of the above we have proven our claim; each of the three subfields corresponding to the involutions of $H_i$ are isomorphic to a function field of the form $F_j$ where $1 \leq j \leq \frac{d-3}{2}$ or $j = d-1$ and, in both cases, $\gcd(j(j+2),d)=1$. \\ The isomorphism classes in the family $\{F_i\}_i$ were described in \cite{beelen_families_2024}, and we use these results to obtain two useful lemmas: \begin{lemma} \label{lemma:iso_subfields_onlyif} Assume $i_1$ and $i_2$ satisfy $1\leq i_1,i_2 \leq \frac{d-3}{2}$ and $\gcd(i_1(i_1+1),d)=\gcd(i_2(i_2+1),d)=1$. Let $F'$ be a subfield of $\cF_{i_1}$ associated to an involution of $H_{i_1}$ and let $F''$ be a subfield of $\cF_{i_2}$ associated to an involution of $H_{i_2}$. If $F'$ is isomorphic to $F''$ then either \begin{align*} i_1i_2 \equiv 0 &\pmod d,\\ i_1i_2 + i_1 + i_2 \equiv 0 &\pmod d,\\ i_1i_2 + i_1 + 1 \equiv 0 &\pmod d,\\ i_1i_2 + i_2 + 1 \equiv 0 &\pmod d, \end{align*} or we have $i_1 = i_2$. \end{lemma} \begin{proof} For each of $F'$ and $F''$ we can go through the cases mentioned in the above discussion, in combination with Theorem 5.1 and 5.2 from \cite{beelen_families_2024}. This leaves us with only a finite number of cases to check: \newline We know that $F'$ is isomorphic to either $F_{2j_1}$ or $F_{d-2j_1-2}$ where either $j_1 = i_1$ or $j_1$ is equal to the $j$ that appeared in one of the four cases discussed above. Similarly, $F''$ is isomorphic to either $F_{2j_2}$ or $F_{d-2j_2-2}$, with $j_2$ equal to $j$ as in one of the four cases or $j_2=i_2$. In any case, the results of \cite{beelen_families_2024} imply that the indices, $2j_1$ or $d-2j_1-2$, and, $2j_2$ or $d-2j_2-2$, must be equal modulo $d$. This amounts to four cases, but in the end it means that either \begin{align*} j_2 \equiv j_1 &\pmod d, \text{ or }\\ -j_2-1 \equiv j_1 &\pmod d.\\ \end{align*} On the other hand, if we go through the cases above, we see that either \begin{align*} i_1 \equiv j_1 &\pmod d, &(\text{the } \sigma_0 \text{ case)}\\ i_1^{-1} \equiv j_1 &\pmod d, &(\text{Case 1})\\ -i_1^{-1}-1 \equiv j_1 &\pmod d, &(\text{Case 2})\\ (i_1+1)^{-1} - 1\equiv j_1 &\pmod d,\text{ or } &(\text{Case 3}) \\ -(i_1+1)^{-1} \equiv j_1 &\pmod d. &(\text{Case 4})\\ \end{align*} We have something similar for $j_2$ (replacing $i_1$ by $i_2$). To finish the proof, one now has to go through all the cases and check that we arrive at one of the equivalences from the statement of the theorem, or $i_1 = i_2$. We give a few examples: \newline \begin{itemize} \item If $i_1 \equiv i_2 \pmod d$ then $i_1 = i_2$, since $1 \leq i_1,i_2 \leq \frac{d-1}{2}$. \\ \item If $i_1 \equiv i_2^{-1} \pmod d$ then $i_1 i_2 \equiv 1 \pmod d$.\\ \item If $i_1 \equiv -i_2^{-1} - 1 \pmod d$ then $i_1i_2 + i_2 + 1 \equiv 0 \pmod d$.\\ \item If $i_1 \equiv (i_2 + 1)^{-1} - 1 \pmod d$ then $i_1i_2 + i_1 + i_2 \equiv 0 \pmod d$.\\ \item If $i_1 \equiv -(i_2+1)^{-1} \pmod d$ then $i_1i_2 + i_1 + 1 \equiv 0 \pmod d$. \\ \item If $i_1^{-1} \equiv -i_2^{-1} - 1 \pmod d$ then $i_1i_2 + i_1 + i_2 \equiv 0 \pmod d$.\\ \item If $i_1^{-1} \equiv (i_2 + 1)^{-1} - 1 \pmod d$ then $i_1i_2 + i_2 + 1 \equiv 0 \pmod d$.\\ \item If $i_1^{-1} \equiv -(i_2+1)^{-1} \pmod d$ then $i_1 + i_2 + 1 \equiv 0 \pmod d$, but this cannot happen since $1 \leq i_1,i_2 \leq \frac{d-3}{2}$.\\ \end{itemize} The rest of the cases can be treated in a similar way. \end{proof} \begin{lemma}\label{lemma:non_iso_conditions} Assume $1\leq i \leq \frac{d-3}{2}$ and $\gcd(i(i+1),d)=1$. In $\cF_i$, the three subfields $F$, $F'$, and $F''$, corresponding to the involutions of $H_i$, are pairwise non-isomorphic unless either \begin{enumerate}[label=(\alph*)] \item $i = 1$, or \item $i^2 + i + 1 \equiv 0 \pmod d$. \end{enumerate} In the first case, exactly two of the subfields are isomorphic and in the second case all three are isomorphic. Moreover, $F_{d-1}$ is isomorphic to one of the three fields if and only if (a) holds. \end{lemma} \begin{proof} This follows from considerations very similar to those in the proof of the previous lemma. We show only a few details regarding the special cases: \newline \begin{itemize} \item If $i = 1$ then $\sigma_0$ fixes a field isomorphic to $F_2$, $\sigma_1$ fixes a field isomorphic to $F_{d-1}$ (this is Case 4 with $j_0 = (d+1)/2$), and $\sigma_2$ fixes a field isomorphic to $F_2$ (this is Case 1 with $j=1$). \newline \item If $i^2 + i + 1 \equiv 0 \pmod d$ then there are two cases. If $1 \leq 2i \leq \frac{d-3}{2}$ then $\sigma_0$ fixes $F_{2i}$, we get a field isomorphic to $F_{2i}$ from Case 2 (with $j_0 = d - (i+1)$, and we get another field isomorphic to $F_{2i}$ from Case 4 (here $j_0 = d-i$). Similarly, if $\frac{d-1}{2} \leq 2i \leq d-3$ we get that the three fields are all isomorphic to $F_{d-2i-2}$. \newline \end{itemize} The fact that $F_{d-1}$ does not occur except in case $(a)$ can also be checked by going through the cases: We must have $j = \frac{d-1}{2}$, and this means that we are in Case $4$ with $i=1$. \end{proof} These two lemmas will be important for determining both the isomorphism classes in $\{\cF_i\}_i$, as well as the automorphism group of each $\cF_i$. We will consider the automorphism groups in Section \ref{sec:aut} and then return to the isomorphism classes in Section \ref{sec:iso}, but first we will need some results on the Weierstrass semigroups at the places of $\Omega$. \newline \section{The semigroups at the places of $\Omega$}\label{sec:weierstrass} Instead of considering the Weierstrass semigroups directly, we describe the gapnumbers at the places of $\Omega$. For $i=1$ we show that the gapsequences at $Q_\infty^1$ and $Q_\infty^2$, and hence the semigroups, are distinct from those at the the other places of $\Omega$. This will be useful for determining $\aut(F_1)$ later. First consider $\cF_i = \Fqq(x,y)$, for any $i$ satisfying $\gcd(i(i+1),d) = 1$.\newline For $k,l \in \mZ$ define the differential $\omega_{k,l} := x^{k-1}y^{l-q-1}dx$. From Equation \ref{eq:divisors} we get \begin{align*} (\omega_{k,l}) = \ &\left( k d + (l-q-1) i - 1 \right) \left(Q_0^1 + Q_0^2\right) + \left(l-1 \right) \left(Q_\alpha + Q_{-\alpha}\right)\\ &- \left(kd + (l-q-1)(i+1) + 1 \right) \left(Q_\infty^1 + Q_\infty^2\right). \end{align*} This means that $\omega_{k,l}$ is regular if and only if \begin{align*} l &>0, \\ kd + li &> i(q+1), \ \text{ and }\\ kd + (i+1)l &< (i+1)(q+1). \end{align*} In other words, $\omega_{k,l}$ is regular exactly if $(k,l)$ is an (integral) interior point of the triangle $\Delta$ with vertices $(0,q+1)$, $(2i,0)$ and $(2(i+1),0)$. Using Pick's theorem and $\gcd((i+1)i,d) = 1$, we find the number of interior integral points of this triangle to be $q-1$, i.e., equal to the genus of $\cF_i$ (as predicted also by well-known results on Newton polygons). \newline By Proposition \ref{prop:reg_diff_gap}, the regular differentials described above give rise to gap numbers for the places of $\Omega$. The number of distinct differentials equals the number of gaps, i.e., $g(\cF_i) = q-1$, but in some cases two distinct differentials give rise to the same gap number. We will describe the gapsequences completely by considering linear combinations of the $\omega_{k,l}$'s. \newline Denote by $G_\infty$, $G_0$ and $G_\alpha$ the gapsequences at $Q_\infty^1$, $Q_0^1$ and $Q_\alpha$ respectively. Note that they also equal the gapsequences at $Q_\infty^2$, $Q_0^2$ and $Q_{-\alpha}$, since these pairs of places form orbits under $H_i$. Moreover, denote by $\Delta_1$ the triangle with vertices $(i+1,d)$, $(2i+1,0)$ and $(2(i+1),0)$, and by $\Delta_2$ the triangle with vertices $(i,d)$, $(2i,0)$ and $(2i+1,0)$ (see Figure \ref{fig:1_delta}). We write $\Delta^\circ$ (respectively $\Delta_1^\circ$, $\Delta_2^\circ$) for the interior points of $\Delta$ (respectively $\Delta_1$, $\Delta_2$). \input{figure1} \begin{proposition}\label{prop:semigroups} With notation as above, we have \begin{align*} G_\infty = \ &\{-kd - (l-q-1)(i+1) \ \mid \ (k,l) \in \Delta^\circ, l < d \} \\ &\cup \ \{-kd-(l-q-1)(i+1) + q+1 \ \mid \ (k,l) \in \Delta_1^\circ \}, \\ \\ G_0 = \ &\{kd + (l-q-1)i \ \mid \ (k,l) \in \Delta^\circ, l < d \} \\ &\cup \ \{kd + (l-q-1)i + q+1 \ \mid \ (k,l) \in \Delta_2^\circ \}, \text{ and } \\ \\ G_\alpha = \ &\{ l \ \mid \ (k,l) \in \Delta^\circ \setminus \Delta_1^\circ \} \ \cup \ \{l + q+1 \mid (k,l) \in \Delta_1^\circ \}. \\ \end{align*} \end{proposition} \begin{proof} We will show details only for the description of $G_\infty$. The results regarding $G_0$ and $G_\alpha$ are obtained in a similar way. \\ Let $G_1$ be the first set in the union above and $G_2$ the second set. The claim is then that $G_\infty = G_1 \cup G_2$. It follows from Proposition \ref{prop:reg_diff_gap} and the discussion above that the elements of $G_1$ are gap numbers. To see that distinct pairs $(k,l), (k',l') \in \Delta^\circ$, with $l,l'<d$, give rise to distinct gap numbers assume that $$ -kd - (l-q-1)(i+1) = -k'd - (l'-q-1)(i+1). $$ Then $kd + l(i+1) = k'd + l'(i+1)$, and working modulo $d$ yields $l = l'$, since $\gcd(i+1,d)=1$ and $l,l' < d$. This implies also $k = k'$, so in fact $(k,l) = (k',l')$. This shows that $$ |G_1| = |\{(k,l) \in \Delta^\circ \ \mid \ l<d\}| = q-1 - \frac{q-1}{4}, $$ and all these elements are gap numbers at $Q_\infty^1$. \newline Now consider instead $G_2$. For $(k,l) \in \Delta_1^\circ$ a direct check shows that $(k-(i+1), l+d)\in \Delta^\circ$. This means that both $\omega_{k,l}$ and $\omega_{k-(i+1), l+d}$ are regular differentials, and so is $\omega := \omega_{k,l}-\omega_{k -(i+1), l + d}$. We determine $v_{Q_\infty^1}(\omega)$ by rewriting \begin{align*} \omega &= \left(x^{k-1}y^{l-q-1} - x^{k-(i+1)-1}y^{l+d-1}\right) dx \\ &= \left(1-x^{-(i+1)}y^d\right) x^{k-1}y^{l-1} dx \\ &= x^{-(i+1)}\left(y^d - x^{i+1}\right) \omega_{k,l} \\ &= \frac{x^{i-1}}{y^d + x^{i+1}} \omega_{k,l}, \end{align*} where the last equality follows from the defining equation of $\cF_i$. This means that \begin{align*} v_{Q_\infty^1}(\omega) &= v_{Q_\infty^1}(\omega_{k,l}) + v_{Q_\infty^1}\left(\frac{x^{i-1}}{y^d + x^{i+1}}\right) \\ &= v_{Q_\infty^1}(\omega_{k,l}) + d(i-1) - d(i+1) \\ &= -kd-(l-q-1)(i+1)-1 + q+1, \end{align*} so Proposition \ref{prop:reg_diff_gap} shows that the elements of $G_2$ are in fact gap numbers. A similar argument as for $G_1$ shows that distinct integral points in $\Omega_1^\circ$ give rise to distinct gap numbers, so we have $$ |G_2| = |\{(k,l) \in \Delta_1^\circ \}| = \frac{q-1}{4}. $$ The total number of gaps is known to be $g(\cF_i) = |G_1| + |G_2|$, so we are done if we can show $G_1 \cap G_2 = \emptyset$. To see that this is true, assume that $$ -kd - (l-q-1)(i+1) = -k'd - (l'-q-1)(i+1) + q+1, $$ for some $(k,l) \in \Delta^\circ$, with $l<d$, and $(k',l') \in \Delta_1^\circ$. Then working modulo $d$ yields $l = l'$ and it follows that $d(k'-k) = q+1$, i.e., $k'-k = 2$. The width of $\Delta^\circ$ is strictly smaller than 2, so this is a contradiction. We conclude that $G_\infty = G_1 \cup G_2$ as desired. \newline The results on $G_0$ and $G_\alpha$ are obtained analogously, using differentials of the form $\omega_{k,l} - \omega_{k-i,l+d}$ and $\omega_{k,l}-\alpha \omega_{k-1,l}$ respectively (where as usual $\alpha$ is an element of $\Fqq$ satisfying $\alpha^2 = -1$). \end{proof} Even with this rather explicit description it seems difficult to distinguish the gapsequences, or semigroups, at the places of $\Omega$ in general. However, in the special case $i=1$ we are able to do so: \begin{corollary}\label{cor:semigrous_i=1} For $i=1$ and $q > 5$, the gapsequence $G_\infty$ is different from both $G_0$ and $G_\alpha$. \end{corollary} \begin{proof} We show that $d+2$ is in $G_0$ and $G_\alpha$ but not in $G_\infty$. To see that $d+2 \in G_0$ we check that $(3,2) \in \Delta^0$. Indeed, we have $2 > 0$, $3\cdot d + 2 > q+1$ and $3d + 4 < 2(q+1)$ since $q>5$. Note that also $2 < d$, so it follows from Proposition \ref{prop:semigroups} that $G_0$ contains $3d + (2-q-1) = d + 2$. Similarly, it can be checked that $(1,d+2) \in \Delta^\circ \setminus \Delta_1^\circ$ and this implies $d+2 \in G_\alpha$. \newline On the other hand, if $d+2 \in G_\infty$ then, since $d+2 < q+1$, there exists $(k,l)\in \Delta^\circ$ with $l<d$, such that $$ -kd -2(l-q-1) = d + 2. $$ Working modulo $d$ implies $l = d-1$ and inserting this back into the equation yields $k=1$ as the only option. This is a contradiction since $kd + l = 2d-1 = q$, which shows that $(k,l)=(d-1,1)$ is not an interior point of $\Delta$. The desired result follows. \end{proof} In particular, the $\aut(\cF_1)$-orbit containing $Q_\infty^1$ and $Q_\infty^2$ does not contain any other places from $\Omega$. We will use this observation to determine $\aut(\cF_1)$ in the end of the following section. \newline \section{The automorphism group of $\cF_i$}\label{sec:aut} We determine the the structure of the automorphism group of $\cF_i$. For convenience, we still assume $1 \leq i \leq \frac{d-3}{2}$, as well as $\gcd(i(i+1),d)=1$ and $q > 5$. As mentioned in the introduction, we already know a subgroup $H_i\subseteq \aut(\cF_i)$, which is isomorphic to $\mZ_2 \times \mZ_{q+1}$. This means that $H_i$ has a unique Sylow $2$-group, $S$, which is isomorphic to $\mZ_2 \times \mZ_2$. For $i \neq 1$, we will show that $S$ is also the unique Sylow $2$-group of $G$, and use this fact to determine the full automorphism group of $\cF_i$. To complete also the case $i =1 $, we will need the results on the Weierstrass semigroups at the places of $\Omega$. In most cases, we will conclude that there are no more automorphisms than those in $H_i$. \newline \subsection{The case $i \neq 1$} In the rest of this section we assume $i \in \{ 2, \dots, (d-3)/2\}$ with $\gcd(i(i+1),d) = 1$. Note that this also implies $q>5$. First, we show that any involution of $\aut(\cF_i)$ is conjugate to one of the three involutions of $H_i$. This will be useful both for determining the full automorphism group of $\cF_i$ and for describing the isomorphism classes, since it implies that any degree two subfield of $\cF_i$ is isomorphic to one of the three described in Section \ref{sec:subext}. \begin{theorem}\label{thm:2sylow_is_klein} For $i = 2, \dots, (d-3)/2$ with $\gcd(i(i+1),d) = 1$, any involution of $\aut(\cF_i)$ is conjugate to one of the three involutions of $H_i$. \end{theorem} \begin{proof} Assume $i \neq 1$. Denote by $S$ the Sylow $2$-group of $H_i$ and by $S_2$ be the Sylow $2$-group of $\aut(\cF_i)$ that contains $S$. Recall that $S$ is isomorphic to $\mZ_2 \times \mZ_2$. Since $g(\cF_i) = q-1$ is even we can apply \cite[Lemma 6.2]{giulietti_algebraic_many_aut_2019} to obtain a cyclic subgroup of $S_2$ of index 2. \newline \textbf{Claim 1:} There exists $\varphi \in S$ such that $\varphi$ is central in $S_2$. \newline In fact, since $S_2$ is a $2$-group its center is non-trivial and hence contains an element of order $2$, say $\alpha$. Now, if $\alpha \not\in S$ then $\langle \alpha, S\rangle$ is isomorphic to $\mZ_2\times \mZ_2\times \mZ_2$, but this is in contradiction with \cite[Lemma 6.1]{giulietti_algebraic_many_aut_2019} since this $2$-group does not contain a cyclic group of index two. \newline \textbf{Claim 2:} $S_2/\langle \varphi \rangle$ has order two. \newline Let $F$ denote the fixed field of $\langle \varphi \rangle$. It is a consequence of Galois theory (see \cite[Theorem 11.36]{hirschfeld_algebraic_2008}) that $S_2/\langle \varphi \rangle$ is isomorphic to a subgroup of $\aut(F)$. Now, the automorphism group of $F$ is well understood: From the discussion in Section \ref{sec:subext} we know that $F$ is isomorphic to $F_j$ for some $j \in \mZ$ with $1 \leq j \leq \frac{d-3}{2}$ or $j=d-1$, and $\gcd(j(j+2),d) = 1$. In fact, by Lemma \ref{lemma:non_iso_conditions}, our assumption on $i$ ensures $j\neq d-1$. It follows then, from \cite[Theorem 4.8]{beelen_families_2024} that $\aut(F_j)$ is either cyclic of order $q+1$ or the semidirect product of a cyclic group of order $q+1$ and another cyclic group of order $3$. In any case, since $q \equiv 1 \pmod 4$, this implies the claim. \newline It follows from the above that $S_2$ is a group of order four containing (an isomorphic copy of) $\mZ_2\times \mZ_2$, that is $S_2 = S \simeq \mZ_2 \times \mZ_2$. Any other involution $\psi \in \aut(\cF_i)$ is contained in a Sylow 2-group and hence conjugate to an element of $S_2$. This finishes the proof. \end{proof} As an easy consequence we obtain the following: \begin{corollary}\label{cor:iso_subext} For $i = 2, \dots, (d-3)/2$ with $\gcd(i(i+1),d) = 1$, any degree two subfield of $\cF_i$ is isomorphic to one of the three fixed fields of the involutions of $H_i$. \end{corollary} We will now distinguish between two different cases. The first case is that in which the three degree two subfields described in Section \ref{sec:subext} are pairwise non-isomorphic. Then, for each Sylow 2-group there are exactly three, pairwise non-isomorphic, degree two subfields arising as fixed fields of the involutions of that group. We will often make use of this, as well as the fact that these three subfields are isomorphic to $F$, $F'$, and $F''$ respectively. In the second case, in which $i^2 + i + 1 \equiv 0 \pmod d$, all three degree two subfields are isomorphic, and we have an extra automorphism $\gamma$ of order three as defined in Section \ref{sec:special}. By Lemma \ref{lemma:non_iso_conditions} this covers everything except $i=1$, which we will deal with separately. For $i^2 + i + 1 \equiv 0 \pmod d$, we will need the fact that $\omega$ normalizes $H_i$, i.e., that $\langle \omega, H_i\rangle = H_i \rtimes \langle \omega \rangle$. To see this, denote by $F$ a subfield of $\cF_i$ corresponding to an involution of $H_i$. We know from \cite[Theorem 4.8]{beelen_families_2024} that $|\aut(F)| = q+1$, since the characteristic three case does not occur when $i^2 + i + 1 \equiv 0 \pmod d$ (see the comment after Lemma \ref{lemma:number_i^2+i+1_pi(d)}). The degrees match, so the fixed field of $\aut(F)$ is equal to the fixed field of $H_i$ in $\cF_i$. For $h \in H_i$ we have $$ \omega^{-1} h \omega \vert_F \in \aut(F). $$ so $\omega^{-1}h\omega$ fixes the fixed field of $\aut(F)$, which is equal to the fixed field of $H_i$. This means that $\omega^{-1}h\omega \in H_i$, and we conclude that $\langle \omega, H_i \rangle = \langle\omega\rangle \rtimes H_i$ as desired. In particular, $\langle \omega, H_i \rangle$ is a subgroup of $G$ of order $3(q+1)$, and it contains no more involutions than those coming from $H_i$. Now, we give some further results regarding the involutions and Sylow 2-subgroups of $G$. We know that the involutions of $S$, and hence all the involutions of $G$, fix exactly two places. It turns out that knowing these places is enough to know the involution: \begin{lemma}\label{lemma:inv_by_fixed_places} For $i = 2, \dots, (d-3)/2$ with $\gcd(i(i+1),d) = 1$, any involution of $G$ is completely determined by the two places it fixes. \end{lemma} \begin{proof} Suppose that $\sigma_1,\sigma_2\in G$ are involutions fixing the same places $P$ and $P'$. We claim that $\sigma_1 = \sigma_2$. To show this, first note that both $\sigma_1$ and $\sigma_2$ are in the stabilizer, $G_P$, of $P$. From \cite[Theorem 11.49]{hirschfeld_algebraic_2008} we know that $G_P = S_p \rtimes C$ where $S_p$ is a $p$-Sylow subgroup of $G_P$ and $C$ is a cyclic subgroup of $G_P$. The characteristic, $p$, is odd by assumption, so $S_p$ has no involutions. Moreover, a cyclic subgroup has at most one involution, so the image of $\sigma_1$ and $\sigma_2$ in $G_P/S_p \simeq C$ must be equal. This means that $$ \sigma_1 \circ \sigma_2 = \sigma_1 \circ \sigma_2^{-1} \in S_p, $$ i.e., $\varphi := \sigma_1 \circ \sigma_2 \in S_p\subseteq G$ is either the identity or has order $p$. Recall that the $p$-rank of $\cF_i$ is zero, since $\cF_i$ is $\Fqq$-maximal, so any element of order $p$ has exactly one fixed place (see \cite[Lemma 11.129]{hirschfeld_algebraic_2008}). We know that $\varphi$ fixes both $P$ and $P'$, so it cannot be an element of order $p$. Then, $\varphi$ must be the identity, and we conclude that $\sigma_1 = \sigma_2$, as wished. \end{proof} Another important observation is the following: \begin{lemma}\label{lemma:2syl_trivial_intersection} For $i = 2, \dots, (d-3)/2$ with $\gcd(i(i+1),d) = 1$, the intersection of two distinct Sylow $2$-subgroups of $G$ is trivial. \end{lemma} \begin{proof} Suppose there exists two different Sylow $2$-subgroups with non-trivial intersection. By conjugating with a suitable automorphism we get that $S \subseteq H_i$ has non-trivial intersection with some other Sylow $2$-subgroup $S'$. Pick $\gamma \in G$ such that $$ S' = \gamma^{-1} S \gamma, $$ and consider some $\sigma \in S \cap S'$ different from the identity. Then, find $\sigma_1 \in S$ such that $$ \sigma = \gamma^{-1} \sigma_1 \gamma, $$ and note that the fixed field of $\sigma_1$ must be a degree two subfield of $\cF_i$. Denote this subfield by $F$, and let $F'$ and $F''$ be the two other degree two subfields fixed by elements of $S$. The fixed field of $\sigma$ must also be among these three, since $\sigma \in S$. Now, consider the degree two subfield $\gamma^{-1}(F)$. It is easy to check that $\sigma = \gamma^{-1} \sigma_1 \gamma$ fixes all elements of $\gamma^{-1}(F)$. Moreover, the degrees fit so this must be the fixed field of $\sigma$, and hence equal to either $F$, $F'$ or $F''$. If the three degree two subfields are pairwise non-isomorphic, the only option is $$ \gamma^{-1}(F) = F. $$ This means that $\gamma$ restricts to an automorphism on $F$, so $\gamma \in H_i$ and hence $$ S' = \gamma^{-1} S_1 \gamma \subseteq H_i. $$ We conclude that $S = S'$, which is a contradiction. \newline If instead all three degree two subfields are isomorphic, we have $i^2 + i + 1 \equiv 0 \pmod d$, and there is an automorphism $\omega \in G$, as described previously, which acts as a $3$-cycle on $F$, $F'$ and $F''$. This means that $$ \omega^{k} \gamma^{-1} \vert_F \in \aut(F) $$ for some $k \in \{0,1,2\}$, and hence $\omega^k \gamma^{-1} \in H_i$, so $\gamma \in \langle \omega, H_i \rangle = H_i \rtimes \langle \omega \rangle$, which implies $S = S'$. We conclude that distinc Sylow 2-subgroups of $G$ have trivial intersection. \end{proof} Finite groups of even order satisfying that different Sylow 2-groups intersect trivially were characterized by M. Suzuki in \cite{suzuki_finite_1964}. Using this, as well as the characterization of certain 2-transitive groups by Kantor, O'Nan and Seitz in \cite{kantor_2-transitive_1972}, we are now able to show a key result regarding the structure of $G$: \begin{theorem}\label{thm:syl2_is_normal} For $i = 2, \dots, (d-3)/2$ with $\gcd(i(i+1),d) = 1$, $S$ is the unique Sylow $2$-subgroup in $G$. \end{theorem} \begin{proof} If the three degree two subfields are pairwise non-isomorphic then the involutions in $S$ must belong to distinct conjugacy classes. By Lemma \ref{lemma:2syl_trivial_intersection} above we can apply \cite[Lemma 6]{suzuki_finite_1964}, which then implies that $S$ is the unique Sylow $2$-subgroup. \newline Otherwise, all three degree two subfields are isomorphic, so assume from now on that $i^2 + i + 1 \equiv 0 \pmod d$, and that there is more than one Sylow $2$-subgroup of $G$. \newline From \cite[Lemma 6]{suzuki_finite_1964} we conclude that all involutions of $G$ are conjugate. By applying Suzuki's classification \cite[Theorem 2]{suzuki_finite_1964} and using $S \simeq \mZ_2\times \mZ_2$ we get that $G$ contains a normal subgroup $G_1$ and $G_2$ such that $$ \{\text{id}\} \subseteq G_2 \subsetneq G_1 \subseteq G, $$ where both $|G/G_1|$ and $|G_2|$ are odd and $G_1/G_2$ is isomorphic to $A_5$ (the alternating group on five elements). From this we deduce some further results regarding the structure of $G$, which will eventually lead to the contradiction we are searching for. \newline \textbf{Claim 1:} The number of Sylow $2$-subgroups of $G$ is five. \newline Let $n_2$ be the number of Sylow $2$-subgroups. From the discussion following Theorem 2 in \cite{suzuki_finite_1964} we see that $G_1/G_2 \simeq A_5$ acts 2-transitively on the set of Sylow $2$-groups of $G$. This immediately implies that $n_2 \leq 6$, since the order of $A_5$ has to be divisible by $n_2(n_2-1)$. On the other hand $A_5$ has five different Sylow 2-subgroups, so we obtain $$ 5 \leq n_2 \leq 6 $$ by using that $|G/G_1|$ is odd. By Sylow's theorem $n_2$ is odd, so we conclude that $n_2 = 5$. \newline \textbf{Claim 2:} The set $\Omega$ is a $G$-orbit. \newline Fix some place $P \in \Omega$. We consider the connection between the number of Sylow 2-subgroups and the size of the $G$-orbit of $P$. Let $\sigma \in H$ be some involution fixing $P$ and another place $P'\in \Omega$, and denote by $O_P$ the $G$-orbit of $P$. For any $\gamma \in \aut(\cF_i)$, we have an involution fixing the places $\gamma(P)$ and $\gamma(P')$, namely $$ \sigma_\gamma := \gamma \circ \sigma \circ \gamma^{-1}. $$ If, for $\gamma_1,\gamma_2 \in G$, we have $$ \{ \gamma_1(P), \gamma_1(P')\} \neq \{\gamma_2(P), \gamma_2(P')\}, $$ then Lemma \ref{lemma:inv_by_fixed_places} implies that $\sigma_{\gamma_1}$ and $\sigma_{\gamma_2}$ are different involutions. The number of involutions of $G$ is $3\cdot n_2 = 15$, so this means that $$ 15 \geq |O_P|/2. $$ Recall that $H_i$ acts with long orbits outside of $\Omega$, so $$ |O_P| = 6 + 2k (q+1) \leq 30, $$ which is true only if $k=0$ or $q \leq 11$. Now, the only options for $q \leq 11$ are $q = 5$ and $q=9$. In the first case we must have $i = 1$, so this option is not valid, and in the second case the equation $i^2 + i + 1 \equiv 0 \pmod d$ has no solutions, so this case does not occur. We conclude that $k = 0$, so in fact $O_P = \Omega$. \newline \textbf{Claim 3:} $G$ acts 2-transitively on $\Omega$. \newline The number of involutions is $15 = \binom{6}{2}$, they are all in the same conjugacy class and any involution fixes exactly two places in $\Omega$. This means there is a 1-to-1 correspondence between pairs of places of $\Omega$ and involutions of $G$. Now fix some $P \in \Omega$ and choose $P' \in \Omega$ such that $\{P,P'\}$ forms an $H_i$-orbit. Let $\pi \in H_i$ be some automorphism switching $P$ and $P'$, and let $\sigma$ be the involution that fixes $P$ and $P'$. For a place $Q \in \Omega \setminus \{P,P'\}$ denote by $\sigma'$ the involution fixing $P$ and $Q$, and determine $\gamma \in G$ such that $$ \sigma' = \gamma \sigma \gamma^{-1}. $$ Then $\gamma$ maps $\{P, P'\}$ to $\{ P, Q\}$, so either $\gamma$ fixes $P$ and maps $P'$ to $Q$ or $\gamma \circ \pi$ fixes $P$ and maps $P'$ to $Q$. This shows that the stabilizer of $P$ acts transitively on $\Omega \setminus \{P\}$, so we conclude that $G$ acts 2-transitively on $G$. \newline Finally, we will use the classification by Kantor, O'Nan and Seitz in \cite{kantor_2-transitive_1972} to obtain a contradiction. Note that the stabilizer of two different places in $\Omega$ is cyclic by \cite[Theorem 11.49]{hirschfeld_algebraic_2008} and \cite[Lemma 11.129]{hirschfeld_algebraic_2008}, since the $p$-rank of $\cF_i$ is zero. This means we can apply the classification result \cite[Theorem 1.1]{kantor_2-transitive_1972}. Since the order of $\Omega$ is not a prime power, $G$ cannot have a regular normal subgroup (see e.g. \cite[Theorem 1.7.5]{biggs_permutation_1979}), so $G$ must be one of the groups $$ \mathrm{PSL}(2,q_0), \ \mathrm{PGL}(2,q_0), \ \mathrm{PSU}(3,q_0), \ \mathrm{PGU}(3,q_0), \ \mathrm{Sz}(q_0), \text{ or } \mathrm{Ree}(q_0), $$ where $q_0$ is a prime power. We know $|G|$ is divisible by four but not eight, and this is enough to exclude $\mathrm{PSU}(3,q_0)$, $\mathrm{PGU}(3,q_0)$ and $\mathrm{Ree}(q_0)$. Also, the only option for $\mathrm{Sz}(q_0)$ is $q_0 = 2$, but in this case three does not divide the order. The group $\mathrm{PGL}(2,q_0)$ has order divisible by eight except for $q_0 = 2$ and $q_0 = 4$, but $G \simeq \mathrm{PGL}(2,2)$ or $G \simeq \mathrm{PGL}(2,4)$ would imply $$ 6(q+1) \leq |G| \leq 60, $$ which only happens for $q \leq 9$, and we already saw that $q = 5$ and $q = 9$ does not occur. A similar argument shows that $G \simeq \mathrm{PSL}(2,q_0)$ cannot happen for $q_0$ even. If $q_0$ is odd, then the number of involutions of $\mathrm{PSL}(2,q_0)$ is known to be $q_0(q_0-1)/2$ (see, e.g., \cite[Section 13, Theorem 1.4 and the beginning of Subsection 13.3]{gorenstein1980finite}), and this is not equal to $15$ for any valid choice of $q_0$. There are no more remaining options, so we have arrived at a contradiction. We conclude that $S$ is the unique Sylow $2$-subgroup of $G$ as desired. \end{proof} The description of the full automorphism group now follows easily: \begin{corollary} For $i = 2, \dots, (d-3)/2$ with $\gcd(i(i+1),d) = 1$ we have $$ \aut(\cF_i) = \begin{cases} H_i \rtimes \langle \omega \rangle &\text{ if } \ i^2 + i + 1 \equiv 0 \pmod d, \text{ and } \\ \hfil H_i &\text{ otherwise.} \end{cases} $$ \end{corollary} \begin{proof} For $\sigma \in G$, it follows from Theorem \ref{thm:syl2_is_normal} that $\sigma(F)\in \{F, F', F''\}$. We consider the two different cases. \newline Assume first that $i^2 + i + 1 \not\equiv 0 \pmod d$. Then $F$, $F'$ and $F''$ are pairwise non-isomorphic, so the only option is $\sigma(F) = F$. This means that $\sigma\vert_F \in \aut(F)$. From \cite[Theorem 4.8]{beelen_families_2024} we know $|\aut(F)| = q+1$ unless $F \simeq F_1$ and $q$ is a power of three. In this case, replace $F$ by $F'$, and note that $F' \not\simeq F_1$. Since the degrees match, the fixed field of $\aut(F)$, or $\aut(F')$, must be equal to the fixed field of $H_i$ in $\cF_i$. In particular, $\sigma$ fixes the fixed field of $H_i$, and hence $\sigma \in H_i$. Since $\sigma$ was arbitrary this shows $G = H_i$. \newline If instead $i^2 + i + 1 \equiv 0 \pmod d$ then $F$, $F'$ and $F''$ are all isomorphic, and $\aut(\cF_i)$ contains an automorphism, $\omega$, which acts as a $3$-cycle on $\{F,F',F''\}$. In particular, $$ \omega^k \sigma \vert_F \in \aut(F), $$ for some $k \in \{0,1,2\}$. From \cite[Theorem 4.8]{beelen_families_2024} we know $|\aut(F)| = q+1$, so again the fixed field of $\aut(F)$ is equal to the fixed field of $H_i$. This implies that $\omega^k \sigma \in H_i$, so $\sigma \in \langle \omega, H_i \rangle = H_i \rtimes \langle \omega \rangle$, and this finishes the proof. \end{proof} \subsection{The case $i=1$}\label{sec:special_i=1} The previously used methods appear to be inadequate in this case. One reason is that the automorphism group now contains more involutions. Another, is that one of the subfields arising from the involutions of $H_1$ is $F_{d-1}$, which is isomorphic to the Roquette curve and hence has a large automorphism group. Instead, we will rely on information regarding the Weierstrass semigroups at the places of $\Omega$, and use a method similar to what was done in \cite{beelen_families_2024}. \newline We claim that $\aut(\cF_1)$ is generated by $\pi$ and $H_1$, where $\pi$ is the involution defined in Section \ref{sec:special}. In fact, we have the following theorem: \begin{theorem} For $q > 5$ and $i=1$, the automorphism group of $\cF_i$ is the semidirect product of $H_i$ and a group of order two. In particular, we have $|\aut(\cF_i)| = 4(q+1)$. \end{theorem} \begin{proof} Define $G := \aut(\cF_1)$ and $g := g(\cF_1) = q-1$. Direct calculations show that $\langle H_1, \pi \rangle = H_1 \rtimes \langle \pi \rangle$, so $|G| \geq 4(q+1)$, and the theorem follows if we can show $|G| \leq 4(q+1)$. We check the result directly with a computer for $q < 37$, and for $q \geq 37$ we proceed by considering the orbit of $Q_\infty^1$: \newline Assume from now on that $q\geq 37$, and denote by $O_\infty$ the $G$-orbit containing both $Q_\infty^1$ and $Q_\infty^2$. By Corollary \ref{cor:semigrous_i=1} it cannot contain any other places from $\Omega$. If the orbit is of length more than two then, since $H_1$ acts with long orbits outside of $\Omega$, the orbit-stabilizer theorem yields \begin{align*} |G| = |O_\infty| \cdot |\aut(\cF_1)_{Q_\infty^1}| \geq (2 + 2(q+1)) (q+1) = (2g + 6)(g+2) > 84(g-1), \end{align*} because $q \geq 37$. Hence \cite[Theorem 11.56]{hirschfeld_algebraic_2008} applies, so $|G|$ is divisible by the characteristic $p$, and one of the following cases holds: \begin{enumerate} \item $G$ has exactly one short orbit, \item $G$ has exactly three short orbits, of which two have cardinality $|G|/2$, or \item $G$ has exactly two short orbits, of which at least one is non-tame, i.e., the order of the stabilizer of a place in the orbit is divisible by $p$. \end{enumerate} All places of $\Omega$ have a non-trivial stabilizer (they each contain a cyclic subgroup of $H_1$ of order $(q+1)$), so they must be contained in short orbits of $G$. This immediately excludes the first case because of Corollary \ref{cor:semigrous_i=1}. The second case also cannot occur; the stabilizers of each place in $\Omega$ is of order at least $q+1$, so this would again imply that all places of $\Omega$ are in the same orbit. We are left with Case (3): \newline Assume that $G$ gives rise to exactly two short orbits, $O_1$ and $O_2$, and that at least one of them, say $O_1$, is non-tame. The places of $\Omega$ cannot all be in the same orbit, again by Corollary \ref{cor:semigrous_i=1}, so there exists some $P \in \Omega \cup O_1$. By \cite[Theorem 11.49]{hirschfeld_algebraic_2008} we may write $$ \aut(\cF_1)_{P} = S_p \rtimes C, $$ where $S_p$ is a Sylow $p$-subgroup of $\aut(\cF_1)_{P}$ and $C$ is cyclic or order not divisible by $p$. Note that the cyclic subgroup of $H_i$ which fixes $P$ is contained in $C$, so the order of $C$ is a multiple of $q+1$. Now, define $E_P$ to be the fixed field of $S_P$ in $\cF_1$, so that $\overline{C} := \aut(\cF_1)/S_p \simeq C$ is a cyclic subgroup of $\aut(E_P)$. We consider three different cases, depending on the genus of $E_P$: \newline \textbf{Case 1:} Assume $g(E_P) \geq 2$. Then we can apply \cite[Theorem 11.79]{hirschfeld_algebraic_2008} to obtain $$ q+1 \leq |C| \leq 4g(E_P) + 4. $$ On the other hand, the Riemann-Hurwitz formula applied to the extension $\cF_1/E_P$ yields $$ 2g - 2 \geq |S_P| (2g(E_P)-2) + (|S_P|-1). $$ From combining the above we get $$ q+1 \leq |C| \leq \frac{4q - 6}{|S_P|} + 6, $$ which in turn implies $|S_P| < 5$, since $q \geq 37$. Hence, only the case $|S_P| = p = 3$ remains, and in this case we have $|C| < \frac{4q-6}{3} -2 < 2(q+1)$. Since $|C|$ is a multiple of $q+1$, this implies $|C| = q+1$ so that $C\subseteq H_1$. Now, consider a generator $\tau$ of $S_3$. By definition $\tau$ fixes $P$, and since the $p$-rank of $\cF_1$ is zero it fixes no other places by \cite[Lemma 11.129]{hirschfeld_algebraic_2008}. In particular, $\tau$ acts with orbits of length three on the remaining five places of $\Omega$, so there must be a $\tau$-orbit containing both a place from $\Omega$ and a place not in $\Omega$. This is a contradiction since $C$ acts on the $S_P$-orbits, and $C$ acts with orbits of length at most two on places of $\Omega$ and orbits of length $q+1$ everywhere else. \newline \textbf{Case 2:} Assume $g(E_P) = 1$. Then \cite[Remark 11.95]{hirschfeld_algebraic_2008} implies that $q < 13$, but we are assuming $q \geq 37$. \newline \textbf{Case 3:} Assume $g(E_P) = 0$. Then \cite[Theorem 11.91]{hirschfeld_algebraic_2008} implies that $\overline{C}$ fixes exactly two places of $E_P$ and acts with long orbits everywhere else. This means that the cyclic group $H':= H_1 \cap C$ fixes exactly two $S_P$-orbits. One of them is $\{P\}$ and the other one must contain anything with a nontrivial $H'$-stabilizer. In particular, all the remaining places of $\Omega$ must be in the same $S_P$-orbit, and hence all of $\Omega$ is in the same $G$-orbit, but this is in contradiction with Corollary \ref{cor:semigrous_i=1}. \newline We obtain a contradiction in all cases, so we conclude that $O_\infty = \{Q_\infty^1, Q_\infty^2\}$. By the orbit-stabilizer theorem this implies $$ |G| = 2 |S|, $$ where $S := \aut (\cF_1)_{Q_\infty^1}$. We know that $S$ contains a cyclic subgroup $H' := H_i \cap S$ of order $q+1$, and we will finish the proof by showing $|S| \leq 2|H'| = 2(q+1)$. \newline First note that the elements of $S$ fix both places in $O_\infty = \{Q_\infty^1, Q_\infty^2\}$. From \cite[Lemma 11.129]{hirschfeld_algebraic_2008} we therefore get that $S$ contains no element of order $p$, and it follows both that $G$ is tame and that $S_P$ is cyclic (by \cite[Theorem 11.49]{hirschfeld_algebraic_2008}). Now, consider a generator $\beta$ of $S$. Since $S$ is cyclic $H'$ is normal in $S$, so $S$ acts on the orbits of $H'$. In particular, $S$ acts on the set of short $H'$-orbits $\left\{ \{Q_0^1,Q_0^2\},\{Q_\alpha, Q_{-\alpha}\}\right\}$. It follows that $\beta^2$ fixes the divisor of both $x$ and $y$, so we must have $$ \beta(x) = \lambda x \ \text{ and } \ \beta(y) = \mu y, $$ for some $\lambda, \mu \in \Fqq$. From the defining equation of $\cF_1$ we obtain $$ \mu^{q+1} y^{q+1} = \mu^{q+1} x^2(x^2 + 1) = \lambda^2 x^2(\lambda^2 x^2 + 1), $$ which is only possible if $\mu^{q+1} = \lambda^2 = 1$. We conclude that $\beta^2 \in H_1$, and since $\beta^2 \in S$ by definition, this shows $\beta^2 \in H'$. Finally, this implies $$ |G| = 2\cdot|S| \leq 2\cdot (2\cdot|H'|) = 4(q+1), $$ as desired. We conclude that $|G| = 4(q+1)$ which means $G = \langle H_1, \pi\rangle = H_1 \rtimes \langle \pi \rangle$, and this finishes the proof. \end{proof} We sum up the results regarding automorphism groups in the following theorem: \begin{theorem}\label{thm:aut} Let $q$ be the power of an odd prime with $q > 5$, and suppose $1 \leq i \leq (d-3)/2$ with $\gcd(i(i+1),d)=1$. Then, keeping the notation from previously, the automorphism group of $\cF_i$ is given by $$ \aut(\cF_i) = \begin{cases} H_i \rtimes \langle \pi \rangle & \text{ if } \ i=1, \\ \hfil H_i \rtimes \langle \omega \rangle &\text{ if } \ i^2 + i + 1 \equiv 0 \pmod d, \text{ and } \\ \hfil H_i &\text{ otherwise.} \end{cases} $$ In particular, the order of the automorphism group is $4(q+1)$ if $i=1$, $3(q+1)$ if $i^2 + i + 1 \equiv 0 \pmod d$ and $q+1$ otherwise. \end{theorem} \section{Isomorphism classes}\label{sec:iso} We determine the isomorphism classes among $\{\cF_i\}_i$ and calculate the number of distinct isomorphism classes. Note that the results are in accordance with the findings of \cite{giulietti_m=2_curves_2006} when $d$ is a prime. The main result is the following: \begin{theorem}\label{thm:main_iso_classes} For $1 \leq i_1 < i_2 \leq \frac{d-1}{2}$ with $\gcd(i_1(i_1+1),d)=\gcd(i_2(i_2+1),d) = 1$, the function fields $\cF_{i_1}$ and $\cF_{i_2}$ are isomorphic if and only if \begin{align*} i_1i_2 \equiv 0 &\pmod d,\\ i_1i_2 + i_1 + i_2 \equiv 0 &\pmod d,\\ i_1i_2 + i_1 + 1 \equiv 0 &\pmod d, \text{ or }\\ i_1i_2 + i_2 + 1 \equiv 0 &\pmod d.\\ \end{align*} \end{theorem} \begin{proof} For $q=5$ there is nothing to show, so assume from now on that $q>5$. The ``if'' part is covered by the explicit isomorphisms given in Section \ref{sec:explicit_iso}. The ``only if'' part follows from combining Theorem \ref{thm:aut} and Lemma \ref{lemma:iso_subfields_onlyif}. In fact, suppose that $\cF_{i_1}$ and $\cF_{i_2}$ are isomorphic. We consider three different cases: \newline \textbf{Case 1:} If $i_1 = 1$, then it follows from Theorem \ref{thm:aut} that $i_2 = \frac{d-1}{2}$, and we have $i_1i_2+i_1+i_2 \equiv 0 \pmod d$. \newline \textbf{Case 2:} If $i_1^2 + i_1 + 1 \equiv 0 \pmod d$, then it follows from Theorem \ref{thm:aut} that also $i_2^2 + i_2 + 1 \equiv 0 \pmod d$, and hence that the only involutions in $\aut(\cF_{i_1})$ and $\aut(\cF_{i_2})$ are those coming from $H_{i_1}$, respectively $H_{i_2}$. Applying Lemma \ref{lemma:iso_subfields_onlyif} now gives the desired result. In fact, it follows from the discussion in the proof of Lemma \ref{lemma:non_iso_conditions} that $i_1 = i_2$. \newline \textbf{Case 3:} Otherwise, it follows from Theorem \ref{thm:aut} that $\aut(\cF_{i_1}) = H_{i_1}$, and hence also $\aut(\cF_{i_2}) = H_{i_2}$. Applying Lemma \ref{lemma:iso_subfields_onlyif} now gives the desired result. \end{proof} The number of isomorphism classes in $\{\cF_i\}_i$ hence depends on the number of distinct solutions to $i^2 + i + 1 \equiv 0 \pmod d$. We determine this number in terms of the prime facotization of $d$. \begin{lemma}\label{lemma:number_i^2+i+1_pi(d)} Assume $q>5$. Write $d = p_1^{\alpha_1}\cdots p_n^{\alpha_n}$ for distinct odd primes $p_1, \dots , p_n$ and $\alpha_1, \dots, \alpha_n \in \mZ_{\geq 0}$. Let $m_1$ (respectively $m_2$) be the number of primes among $p_1, \dots, p_n$ congruent to one (respectively two) modulo three. Then, the number of distinct solutions to $i^2 + i + 1 \equiv 0 \pmod d$ in $\{1, \dots, \frac{d-3}{2}\}$ is $$ \pi(d) = \begin{cases} 0 &\text{if } 9\mid d \text{ or } m_2 \geq 1, \\ 2^{m_1 - 1} &\text{otherwise.} \end{cases} $$ \end{lemma} \begin{proof} We first count solutions for $i\in \{0, \dots, d-1\}$. By the Chinese Remainder Theorem this can be reduced to counting solutions of $i^2 + i + 1 \equiv 0 \pmod{p^k}$ in $\{0,\dots, p^k-1\}$, for $p$ in $\{p_1, \dots, p_n\}$. If $p = 3$ and $k=1$ there is exactly one solution, namely $i=1$. A direct check shows that $i^2 + i + 1 \equiv 0 \pmod 9$ never holds, so if $p = 3$ and $k \geq 2$ there are no solutions. Suppose $p>3$, and note that then $i \equiv 1 \pmod p$ is never a solution. Since $(i^3-1) = (i-1)(i^2+i+1)$ this means that the solutions of $i^2 + i + 1 \equiv 0 \pmod{p^k}$ in $\{0,\dots, p^k-1\}$ correspond to elements of order three in $\left(\mZ/p^k\mZ\right)^\times$. This group is cyclic of order $p^{k-1}(p-1)$, so there are no elements of order three if $p \equiv 2 \pmod 3$, and exactly two elements of order three if $p \equiv 1 \pmod 3$. We conclude that the number of solutions to $i^2 + i + 1 \equiv 0 \pmod d$ in $\{0, \dots, d-1\}$ is zero if $9\mid d$ or $m_2 \geq 1$, and $2^{m_1}$ otherwise. To finish the proof, note that if $i^2 + i + 1 \equiv 0 \pmod d$ then $d-(i+1)$ is another solution. We assume $q > 5$, so this means that the solutions to $i^2 + i + 1 \equiv 0 \pmod d$ among $\{1, \dots, d-1\}$ come in pairs, with exactly one member of each pair being in $\{1, \dots, \frac{d-3}{2}\}$. The desired result now follows. \end{proof} As an easy consequence, we note that if $q$ is a power of $3$ then $d \equiv 2 \pmod 3$, so it is divisible by at least one prime congruent to $2$ modulo $3$, and hence $i^2 + i + 1 \equiv 0 \pmod d$ has no solutions. \newline The number of isomorphism classes can now be determined: \begin{theorem}\label{thm:number_iso_classes} Let $q > 5$ be the power of a prime with $q \equiv 1 \pmod 4$, $d := (q+1)/2$ odd, and $\{\cF_i\}_i$ as defined in Equation \ref{eq:Fi}. Write $d = p_1^{\alpha_1}\cdots p_n^{\alpha_n}$ for distinct odd primes $p_1, \dots , p_n$ and $\alpha_1, \dots, \alpha_n \in \mZ_{\geq 0}$. The number of isomorphism classes among the function fields $\{\cF_i\}_{i}$ is $$ N(d) = \frac{\varphi_2(d) + 4\pi(d) + 3}{6}, $$ where $\pi(d)$ is as defined in Lemma \ref{lemma:number_i^2+i+1_pi(d)} and $$ \varphi_2(d) = p_1^{\alpha_1-1}(p_2-2) \cdots p_n^{\alpha_n - 1}(p_n - 2). $$ \end{theorem} \begin{proof} We follow the same strategy as in \cite[Theorem 5.3]{beelen_families_2024}. Using the first explicit isomorphism mentioned in Section \ref{sec:explicit_iso} we reduce the problem to counting isomorphism classes for $i \in \{0,1, \dots, d-1\}$. Among these numbers, there are exactly $\varphi_2(d)$ choices for $i$ such that $\gcd(i(i+1),d)=1$. By the second isomorphism mentioned in Section \ref{sec:explicit_iso}, we can reduce this to $\frac{\varphi_2(d)+1}{2}$ valid choices for $i \in \{0, \dots, \frac{d-1}{2}\}$. Now consider the function fields arising from these choices of $i$ only. Then, as noted in the proof of the above theorem, the set $\{\cF_1,\cF_{(d-1)/2}\}$ is an isomorphism class, and so is $\{\cF_i\}$ for any $i$ satisfying $i^2 + i + 1 \equiv 0 \pmod d$. We claim that all other isomorphism classes will have size three: In fact, suppose $i\in \{2, \dots, \frac{d-3}{2}\}$ satisfies $\gcd(i(i+1),d) = 1$ and $i^2 + i + 1 \not\equiv 0 \pmod d$. Then Theorem \ref{thm:main_iso_classes} shows $\cF_i$ is isomorphic to $\cF_{i'}$ and $\cF_{i''}$, where $i'$ is equal to $j$ as defined in either Case 1 or Case 2 from Section \ref{sec:subext} (depending on the inverse of $i \pmod d$), and $i''$ is equal to $j$ as defined in either Case 3 or Case 4 (depending on the inverse of $i+1 \pmod d$). The assumptions on $i$ guarantee that $i$, $i'$ and $i''$ are distinct. \newline From the above observations we conclude that the number of isomorphism classes is $$ 1 + \pi(d) + \frac{1}{3}\left(\frac{\varphi_2(d)+1}{2} - 2 - \pi(d) \right) = \frac{\varphi_2(d) + 4\pi(d) + 3}{6}, $$ where $\pi(d)$ is as defined in Lemma \ref{lemma:number_i^2+i+1_pi(d)}. \end{proof} Note that $\varphi_2(d) = d-2$ when $d$ is prime, so in this case we recover the formula for $N(d)$ given in \cite[Theorem 1.1]{giulietti_m=2_curves_2006}. \section*{Acknowledgements} The author would like to thank Maria Montanucci and Peter Beelen for helpful discussions and insightful suggestions throughout the work that led to this paper. \bibliographystyle{abbrv} \bibliography{ref} \end{document} \begin{figure}[h!] \begin{tikzpicture}[scale = 1.1] \draw[->] (-0.5,0) -- (6.5,0) node[right] {$k$}; \draw[->] (0,-0.5) -- (0,6.5) node[above] {$l$}; \def\i{1} \def\q{5} \def\d{3} \coordinate (A) at (0, {\q+1}); \coordinate (B) at ({2*\i}, 0); \coordinate (C) at ({2*(\i+1)}, 0); \coordinate (D1) at ({\i+1}, {\d}); \coordinate (E1) at ({2*\i+1}, 0); \coordinate (F1) at ({2*(\i+1)}, 0); \coordinate (D2) at ({\i}, {\d}); \coordinate (E2) at ({2*\i}, 0); \coordinate (F2) at ({2*\i+1}, 0); \draw[thick] (A) -- (B) -- (C) -- cycle; ll (A) circle (2pt); ll (B) circle (2pt); ll (C) circle (2pt); \node[left] at (A) {$(0, q+1)$}; \node[below left] at (B) {$({2}i, 0)$}; \node[below right] at (C) {$({2(i+1)}, 0)$}; \draw[thick, dashed] (D1) -- (E1) -- (F1) -- cycle; ll (D1) circle (2pt); ll (E1) circle (2pt); ll (F1) circle (2pt); \node[right] at (D1) {$({i+1}, d)$}; \node[below] at (E1) {$({2i+1}, 0)$}; \node at ($(D1)!0.45!(E1)!0.40!(F1)$) {$\Delta_1$}; \draw[thick, dashed] (D2) -- (E2) -- (F2) -- cycle; ll (D2) circle (2pt); ll (E2) circle (2pt); ll (F2) circle (2pt); \node[left] at (D2) {$({i}, d)$}; \node at ($(D2)!0.45!(E2)!0.40!(F2)$) {$\Delta_2$}; \end{tikzpicture} \captionsetup{justification=centering} \caption{The triangles $\Delta$, $\Delta_1$ and $\Delta_2$.} \label{fig:1_delta} \end{figure}
2412.05068v1
http://arxiv.org/abs/2412.05068v1
On constant mean curvature surfaces satisfying integrable boundary conditions
\documentclass[12pt]{amsart} \setlength{\topmargin}{-.8in} \setlength{\textheight}{9.15in} \setlength{\textwidth}{6in} \setlength{\oddsidemargin}{3.5ex} \setlength{\evensidemargin}{0pt} \setlength{\headsep}{.5in} \setlength{\footskip}{.5in} \setlength{\parindent}{0pt} \setlength{\parskip}{12pt} \usepackage[draft]{graphicx} \usepackage{amssymb} \usepackage{amsmath} \usepackage{amstext} \usepackage{amsbsy} \usepackage{amsxtra} \usepackage{amsfonts} \usepackage{latexsym} \usepackage{alltt} \usepackage{bbm} \usepackage{url} \usepackage{verbatim} \usepackage{amscd} \usepackage[all]{xy} \renewcommand{\theenumi}{\roman{enumi}} \renewcommand{\labelenumi}{(\theenumi)} \renewcommand{\theenumii}{\roman{enumii}} \renewcommand{\labelenumii}{(\theenumii)} \theoremstyle{plain} \newtheorem{theorem}{Theorem}[section] \newtheorem{corollary}[theorem]{Corollary} \newtheorem{lemma}[theorem]{Lemma} \newtheorem{proposition}[theorem]{Proposition} \newtheorem{remark}[theorem]{Remark} \newtheorem{example}[theorem]{Example} \newtheorem{definition}[theorem]{Definition} \newcommand{\mi}{\mathbbm i} \newcommand{\C}{\mathbb C} \newcommand{\R}{\mathbb R} \newcommand{\Q}{\mathbb Q} \newcommand{\Z}{\mathbb Z} \newcommand{\N}{\mathbb N} \newcommand{\T}{\mathbb T} \newcommand{\s}{\mbox{\boldmath $\sigma$}} \newcommand{\CP}{\C \mathbb{P}^1} \newcommand{\MT}{\mbox{\small{$\widetilde{M}$}}} \newcommand{\M}{\mathcal{M}} \newcommand{\tr}{\mathrm{tr}} \newcommand{\Id}{\mathrm{Id}} \newcommand{\Order}{{\rm O}} \newcommand{\REs}{\xi_{\mbox{\tiny{$-1$}}}} \newcommand{\dirac}{\not\hspace{-.6mm}\partial} \DeclareMathOperator{\arccot}{arccot} \newcommand{\ind}[1]{_{\mbox{\rm\scriptsize{#1}}}} \newcommand{\SL}{\mathrm{SL}_{\mbox{\tiny{$2$}}}} \newcommand{\Sl}{\mathfrak{sl}_{\mbox{\tiny{$2$}}}} \newcommand{\SU}{\mathrm{SU}_{\mbox{\tiny{$2$}}}} \newcommand{\su}{\mathfrak{su}_{\mbox{\tiny{2}}}} \newcommand{\SO}{\mathrm{SO}(3)} \newcommand{\so}{\mathfrak{so}_{\mbox{\tiny{$3$}}}} \newcommand{\GL}{\mathrm{GL}(2,\C)} \newcommand{\gl}{\mathfrak{gl}(2,\C)} \numberwithin{equation}{section} \DeclareMathOperator*{\ord}{ord} \DeclareMathOperator{\RE}{Re} \DeclareMathOperator{\IM}{Im} \title[CMC surfaces with integrable boundary conditions]{On constant mean curvature surfaces satisfying integrable boundary conditions} \thanks{{\it Mathematics Subject Classification.} 53A10, 53C42, 37K10. \today} \begin{document} \author{Martin Kilian} \address{Martin Kilian, University College Cork, Ireland.} \begin{abstract} We consider the local theory of constant mean curvature surfaces that satisfy one or two integrable boundary conditions and determine the corresponding potentials for the generalized Weierstrass representation. \end{abstract} \maketitle \section*{Introduction} Imposing additional properties, like symmetries, embeddedness, simply-connectedness or compactness on a constant mean curvature surface often severely limits the classes of examples. Historically, the first reduction by Delaunay \cite{Del} resulted in classifying the rotationally symmetric examples. A few decades later, Enneper \cite{Enn, Enn1} and Dobriner \cite{Dob, Dob1} considered surfaces foliated by planar or spherical curvature lines. Modern accounts and further developments can be found in the works of Abresch \cite{Abr} and Wente \cite{Wen, Wen:enn} amongst many others, where the structure equations are reduced to elliptic ordinary differential equations. Here we consider the local theory of constant mean curvature surfaces that satisfy one or two integrable boundary conditions. The integrable boundary condition reflects the geometric property that the surface meets a sphere at a constant angle along the curve of intersection. From our integrable systems point of view this means studying the corresponding 'potentials' in the generalized Weierstrass representation of Dorfmeister, Pedit and Wu \cite{DorPW}. By a result of \cite{KilS2021} all such surfaces are of finite type, so their potentials are polynomial, and the purpose here is to determine all such potentials. We first study the additional so called 'K-symmetry' on the space of potentials that comes from the integrable boundary condition. In Theorem \ref{th:imaginary} we show that K-symmetry halves the dimension of the space of potentials of arbitrary fixed degree. We show in Corollary \ref{th:off-diagonalKsymmetry} that all off-diagonal K-symmetric potentials of arbitrary high degree yield degree one surfaces, that is, associated family members of Delaunay surfaces. In the second part we consider potentials that give solutions of the elliptic $\sinh$-Gordon equation satisfying two integrable boundary conditions, that is free boundary constant mean curvature surfaces with two boundary components. We do not consider period problems here, but focus on the algebraic conditions for the local theory of such surfaces. \section{Integrable boundary conditions} Let $U \subset \R^2$ be an open, connected and simply-connected set containing the origin. An associated family $\mathbf{f}_\lambda:U \to \su \cong \R^3,\,\lambda \in \mathbb{S}^1$ of conformaly immersed surfaces with constant mean curvature $H \neq 0$ can be framed by an $\mathrm{SU}_2$--valued \emph{extended frame} $\mathbf{F}_\lambda$ . It satisfies a linear partial differential system $\mathbf{F}_\lambda^{-1} d\mathbf{F}_\lambda = \mathbf{\Omega}_\lambda$. Here $\mathbf{\Omega}_\lambda = \mathbf{U}_\lambda dx + \mathbf{V}_\lambda dy$ is a $2 \times 2$ matrix-valued 1-form which encodes the surface invariants. For the induced metric $\exp [2\omega] \,(dx^2 + dy^2)$ the $\sinh$-Gordon equation \[ \Delta \,\omega + \sinh \omega = 0 \] is the integrability condition \[ 2d\mathbf{\Omega}_\lambda + [\mathbf{\Omega}_\lambda \wedge \mathbf{\Omega}_\lambda ] = 0 \] for $\mathbf{\Omega}_\lambda$. There is a loop group method to obtain integrable connections via a generalized Weierstrass representation \cite{DorPW}. For us it suffices, due to a result in \cite{KilS2021}, to restrict to the finite gap setting, for which we invoke the Symes method \cite{BurP_adl}: extended frames can be obtained from polynomial Killing fields, and these in turn are determined by their intial conditions, the finite gap potentials. A polynomial Killing field is a solution of a Lax equation \[ d\eta = [\eta,\,\Omega (\eta)] \qquad \eta(0) = \xi_\lambda\,. \] The initial condition (or potential) $\xi_\lambda$ is a $\mathrm{sl}_2 (\C)$--valued Laurent-polynomial $\lambda \mapsto \xi_\lambda$ and $\eta \mapsto \Omega (\eta)$ is a projection, and $\Omega = \Omega(\eta)$ integrates to an extended frame $\mathbf{F}_\lambda^{-1} d\mathbf{F}_\lambda = \Omega$ of a constant mean curvature surface. The relationship between extended frame and polynomial Killing field is $\eta_\lambda = \mathbf{F}_\lambda^{-1} \xi_\lambda \mathbf{F}_\lambda$. Writing $\partial_\lambda$ for differentiation with respect to $\lambda$, the corresponding associated family is given by \begin{equation} \label{eq:SymBobenko} \mathbf{f}_\lambda = - 2\,\mi \lambda H^{-1} (\partial_\lambda \mathbf{F}_\lambda) \,\mathbf{F}_\lambda^{-1}\,. \end{equation} After a choice of normalizations as in \cite{HKS1} for the surface invariants, $\mathbf{U}_\lambda$ takes the form \[ \mathbf{U}_\lambda= \frac{\mathbbm{i}}{8} \left( \begin{array}{cc} -2 \omega_y & e^\omega \lambda^{-1} + e^{-\omega } \\ e^{-\omega } + e^\omega \lambda & 2 \omega_y \\ \end{array} \right)\,. \] Note that \begin{equation*} \overline{\mathbf{U}_{\bar\lambda}}^t = - \mathbf{U}_{\lambda^{-1}} \qquad \mbox{ and} \qquad \overline{\mathbf{F}_{1/\bar\lambda}}^t = \mathbf{F}_\lambda ^{-1}\,. \end{equation*} We study solutions $\omega: U \to \mathbb{R}$ of the sinh-Gordon equation that satisfy an additional \emph{integrable boundary condition} \begin{equation} \label{eq:boundary} \omega_y = e^\omega A + e^{-\omega} B \qquad \mbox{ along \quad $y=0$}\,. \end{equation} Here $A$ and $B$ are arbitrary real numbers. Also, when we write 'along $y=0$' we mean its restriction to the domain, so $\{ y=0 \} \cap U$. \subsection{K-matrix} Define for $A,\,B \in \mathbb{R}$ and $\lambda \in \mathbb{C}^\ast$ the K-matrix \begin{equation} \label{eq:K} K(\lambda) = \begin{pmatrix} 4 A -4 B \lambda & \lambda - \lambda^{-1} \\ \lambda - \lambda^{-1} & 4 A - 4 B \lambda^{-1} \end{pmatrix} \end{equation} Note that $K(\pm 1) = 4(A \mp B)\,\mathbbm{1}$, as well as \begin{equation} \label{eq:Kproperties} K(\lambda) = K(\lambda)^{\,t} \quad \mbox{ and } \quad \overline{K(\bar\lambda)} = K(\lambda) \quad \mbox{ and } \quad K(\lambda^{-1}) = \mathrm{adj}\,K(\lambda)\,. \end{equation} \begin{lemma} \label{lem:Ucomm1} For $\lambda \neq \pm 1$ we have $\omega_y = e^\omega A + e^{-\omega} B$ along $y=0$ if and only if \begin{equation} \label{eq:symU} K\,\mathbf{U}_\lambda = \mathbf{U}_{\lambda^{-1}} \,K \,. \end{equation} \end{lemma} \begin{proof} Compute $K \,\mathbf{U}_\lambda - \mathbf{U}_{\lambda^{-1}} \,K = \tfrac{\mathbbm{i}}{2} (\lambda^{-1} -\lambda ) \left( \omega_y - \left(e^\omega A + e^{-\omega} B \right)\right) \bigl( \begin{smallmatrix} 0 & -1\\ 1 & 0 \\ \end{smallmatrix} \bigr)$. \end{proof} \begin{lemma} \label{lem:Fswitch2} Suppose $\tfrac{d}{dx} \mathbf{F}_\lambda = \mathbf{F}_\lambda \mathbf{U}_\lambda,\,\mathbf{F}_\lambda(0) = \mathbbm{1}$. Then along $y=0$ the condition \eqref{eq:symU} holds if and only if \begin{equation} \label{eq:KF} K\,\mathbf{F}_\lambda = \mathbf{F}_{\lambda^{-1}}\,K \qquad \mbox{ along \quad $y=0$}\,. \end{equation} \end{lemma} \begin{proof} Suppose along $y=0$ the condition \eqref{eq:symU} holds. Consider $\mathbf{G}_\lambda = K\,\mathbf{F}_\lambda\,K^{-1}$. Then $\tfrac{d}{dx} \mathbf{G}_\lambda = \mathbf{G}_\lambda \mathbf{U}_{\lambda^{-1}},\,\mathbf{G}_\lambda (0) = \mathbbm{1}$. By uniqueness of solutions $\mathbf{G}_\lambda = \mathbf{F}_{\lambda^{-1}}$, and thus $K\,\mathbf{F}_\lambda\,K^{-1} = \mathbf{F}_{\lambda^{-1}}$. Conversely, suppose \eqref{eq:KF} holds. Differentiating with respect to $x$ gives $K\,\mathbf{F}_\lambda' = \mathbf{F}_{\lambda^{-1}}'\,K$ and hence $K\,\mathbf{F}_\lambda \mathbf{U}_\lambda = \mathbf{F}_{\lambda^{-1}}\mathbf{U}_{\lambda^{-1}} \,K$. Using \eqref{eq:KF} again gives the claim. \end{proof} \begin{corollary} Let $\mathbf{F}_\lambda$ be an extended frame satisfying \eqref{eq:KF} along $y=0$, and $\zeta_\lambda = \mathbf{F}_\lambda^{-1} \xi_\lambda \mathbf{F}_\lambda$ a polynomial Killing field. Then along $y=0$ we have \begin{equation} \label{eq:PfKsym1} K \zeta_\lambda + \overline{\zeta_{\bar\lambda}}^t K = \mathbf{F}_{\lambda^{-1}}^{-1} \bigl( \, K\,\xi_\lambda + \overline{\xi_{\bar{\lambda}}}^t K \,\bigr) \,\mathbf{F}_\lambda\,. \end{equation} \end{corollary} A polynomial Killing field $\zeta_\lambda$ is K-symmetric (along $y=0$) if \begin{equation}\label{eq:PfKsym2} K \zeta_\lambda + \overline{\zeta_{\bar\lambda}}^t K = 0\,. \end{equation} By \eqref{eq:PfKsym1} this holds if and only if its potential $\xi_\lambda$ is K-symmetric \begin{equation} \label{eq:Sklyanin-potentials} K\,\xi_\lambda + \overline{\xi_{\bar{\lambda}}}^t K = 0\,. \end{equation} \subsection{K-symmetry and the Symes map} The Symes map $\xi_\lambda \mapsto \mathbf{F}_\lambda$ utilizes the Iwasawa factorization \cite{PreS} at each $z \in U$, and we write \begin{equation} \label{eq:Iwasawa} \mathbf{\Phi}_\lambda (z) = \exp [z\,\xi_\lambda ] = \mathbf{F}_\lambda (z,\bar z) \,\mathbf{B}_\lambda (z,\bar z)\,. \end{equation} With $\mathbf{F}_\lambda^{-1} d\,\mathbf{F}_\lambda = \mathbf{\Omega}_\lambda$ we have the gauge relation \begin{equation} \label{eq:gauge} \xi_\lambda = \mathbf{B}_\lambda^{-1} \mathbf{\Omega}_\lambda \mathbf{B}_\lambda + \mathbf{B}_\lambda^{-1} d\,\mathbf{B}_\lambda\,. \end{equation} Next, we determine how \rm{K}-symmetry \eqref{eq:Sklyanin-potentials} on a potential descends to the maps $\mathbf{\Phi}_\lambda$ and $\mathbf{B}_\lambda$. Lemma \ref{lem:Fswitch2} already dealt with the extended frame $\mathbf{F}_\lambda$, as its \rm{K}-symmetry \eqref{eq:KF} came directly from its differential equation. \begin{lemma} \label{th:KPhi} Let $\xi_\lambda$ be a K-symmetric potential. Then \begin{equation} \label{eq:PhiK-symmetry} K\,\mathbf{\Phi}_\lambda (z) = \overline{\mathbf{\Phi}_{\bar\lambda} (\bar z)} ^{t^{-1}} K \qquad \mbox{for all $z \in U$}\,. \end{equation} \end{lemma} \begin{proof} We compute $\overline{\mathbf{\Phi}_{\bar\lambda}(\bar z) }^{\,t} = \exp[z \,\overline{\xi_{\bar\lambda}}^{\,t}] = \exp[ - z\,K \,\xi_\lambda K^{-1} ] = K \,\mathbf{\Phi}_{\lambda}^{-1}(z)\, K^{-1}$. \end{proof} \begin{lemma} Let $\xi_\lambda$ be a {\rm{K}}-symmetric potential, and $ \exp [z\,\xi_\lambda ] = \mathbf{F}_\lambda \,\mathbf{B}_\lambda$ the pointwise Iwasawa factorization for $z \in U$. Assume that $K\,\mathbf{F}_\lambda = \mathbf{F}_{\lambda^{-1}}\,K$ along $y=0$. Then \begin{equation}\label{eq:KB} K\,\mathbf{B}_\lambda = \overline{\mathbf{B}_{\bar\lambda}} ^{t^{-1}} K \qquad \mbox{ along \quad $y=0$}\,. \end{equation} \end{lemma} \begin{proof} By Lemma \ref{th:KPhi} we have $K\,\mathbf{\Phi}_\lambda = \overline{\mathbf{\Phi}_{\bar\lambda}} ^{t^{-1}} K$ and thus \begin{equation*} K\,\mathbf{F}_\lambda \,\mathbf{B}_\lambda = \overline{\mathbf{F}_{\bar\lambda}} ^{t^{-1}} \overline{\mathbf{B}_{\bar\lambda}} ^{t^{-1}} K \,. \end{equation*} Using the assumption on $\mathbf{F}_\lambda$ along $y=0$, and that $\overline{\mathbf{F}_{\bar{\lambda}}}^t = \mathbf{F}_{\lambda^{-1}}^{-1}$ we obtain \eqref{eq:KB}. \end{proof} \subsection{K-symmetry and the associated family} Finally, we look at K-symmetry of the associated family. Let $\partial_\lambda$ denote differentiation with respect to $\lambda$. \begin{lemma} \label{th:KSymmetrySB} Assume that $K\,\mathbf{F}_\lambda = \mathbf{F}_{\lambda^{-1}}\,K$ along $y=0$. Then for the associated family \eqref{eq:SymBobenko} we have along $y=0$ that \begin{equation} \label{eq:Ksym-f} K\,\mathbf{f}_\lambda + \overline{\mathbf{f}_\lambda}^t K = -2\mi\lambda H^{-1} \left( K( \partial_\lambda \mathbf{F}_\lambda ) - (\partial_\lambda \mathbf{F}_{\lambda^{-1}}) K \right) \mathbf{F}_\lambda^{-1} \,. \end{equation} \end{lemma} \begin{proof} Differentiating $K\,\mathbf{F}_\lambda = \mathbf{F}_{\lambda^{-1}}\,K$ with respect to $\lambda$ gives \begin{equation} \label{eq:DiffKF} \mathbf{F}_{\lambda^{-1}}\,\partial_\lambda K - \partial_\lambda K\,\mathbf{F}_\lambda = K\partial_\lambda \mathbf{F}_\lambda - \partial_\lambda \mathbf{F}_{\lambda^{-1}}\,K\,. \end{equation} Inserting $\mathbf{F}_\lambda = K^{-1} \mathbf{F}_{\lambda^{-1}} K$ into \eqref{eq:SymBobenko} yields \[ \mathbf{f}_\lambda = K^{-1} \mathbf{f}_{\lambda^{-1}} K - 2\mi\lambda H^{-1} \left( K^{-1} \mathbf{F}_{\lambda^{-1}} \partial_\lambda K \mathbf{F}_\lambda^{-1} - K^{-1} \partial_\lambda K \right) \] and thus \[ K \mathbf{f}_\lambda - \mathbf{f}_{\lambda^{-1}} K = - 2\mi\lambda H^{-1} \left( \mathbf{F}_{\lambda^{-1}} \partial_\lambda K - \partial_\lambda K \mathbf{F}_\lambda \right) \mathbf{F}_\lambda^{-1}\,. \] Using \eqref{eq:DiffKF} and $\mathbf{f}_{\lambda^{-1}} = - \overline{\mathbf{f}_\lambda}^t$ proves the claim. \end{proof} We say that an associated family is K-symmetric along $y=0$ if \begin{equation*} K\,\mathbf{f}_\lambda + \overline{\mathbf{f}_\lambda}^t K = 0\,. \end{equation*} By equation \eqref{eq:Ksym-f} this holds if and only if $K( \partial_\lambda \mathbf{F}_\lambda ) = (\partial_\lambda \mathbf{F}_{\lambda^{-1}}) K$. \section{K-symmetric potentials} Expanding the matrix of a finite-gap potential as \[ \xi_\lambda = \sum_{k=-1}^d \hat{\xi}_k \lambda^k\,, \] then its coefficient matrices $\hat{\xi}_k \in \mathrm{sl}_2 (\C)$ satisfy the \emph{reality condition} \begin{equation} \label{eq:potential-reality} \hat{\xi}_k = - \overline{\hat{\xi\,}}_{d-k-1}^t \quad -1 \leq k \leq d\,. \end{equation} To match up with the structure of $\mathbf{U}_\lambda$ we have \begin{equation} \label{eq:residue} \hat{\xi}_{-1} \in \begin{pmatrix} 0 & \mi\R_+ \\ 0 & 0 \end{pmatrix}\,, \end{equation} which ensures that also $\hat{\xi}_d \neq 0$. We say that $\xi_\lambda$ has \emph{degree} $d$. Let us write \begin{equation*} \xi_\lambda = \begin{pmatrix} \alpha_\lambda & \beta_\lambda \\ \gamma_\lambda & -\alpha_\lambda \end{pmatrix} \end{equation*} with entries \begin{equation*} \alpha_\lambda = \sum_{k=0}^{d-1} \alpha_k \lambda^k \,,\qquad \beta_\lambda = \sum_{k=-1}^{d-1} \beta_k \lambda^k \,\,,\qquad \gamma_\lambda = \sum_{k=0}^d \gamma_k \lambda^k \qquad \mathrm{ for } \quad \alpha_k,\, \beta_k,\,\gamma_k \in \C\,. \end{equation*} The reality condition \eqref{eq:potential-reality} on the coefficients read \begin{equation}\label{eq:reality} \gamma_k =-\overline{\beta}_{d-k-1} \qquad \mathrm{and} \qquad \alpha_k = -\overline{\alpha}_{d-k-1} \end{equation} The latter splits into real and imaginary parts \begin{equation*} \RE [ \alpha_k + \alpha_{d-k-1}] = 0 = \IM [ \alpha_k - \alpha_{d-k-1}] \,. \end{equation*} When degree $d$ is odd, then for $k=(d-1)/2$ also $d-k-1= (d-1)/2$ and hence \begin{equation*} \RE \alpha_{(d-1)/2} = 0 \,. \end{equation*} Note that there is no freedom in the choice of $\gamma_\lambda$, as all its coefficients are determined by the coefficients of $\beta_\lambda$. There are $2d+1$ real degrees of freedom for $\beta_\lambda$, and $d$ real degrees of freedom for $\alpha_\lambda$ when $d$ is even, and $d-1$ when $d$ id odd. Hence the space of potentials of degree $d$ is an open subset of a $3d+1$ dimensional real vectorspace when $d$ is even, and an open subset of a $3d$ dimensional real vectorspace when $d$ is odd. In terms of the four matrix entries [ij] in the order [11], [12], [21] respectively [22], equation \eqref{eq:Sklyanin-potentials} reads \begin{equation}\label{eq:Sklyanin-potential-symmetry} \begin{split} (\lambda - \lambda^{-1})\left( \gamma_\lambda + \overline{\gamma_{\bar{\lambda}}} \right) +4( A - B\lambda ) \left( \alpha _\lambda + \overline{\alpha_{\bar{\lambda}}} \right) &\equiv 0 \,, \\ -(\lambda - \lambda^{-1})\left( \alpha_\lambda - \overline{\alpha_{\bar{\lambda}}} \right) + 4( A - B\lambda )\beta_\lambda + 4(A-B\lambda^{-1}) \overline{\gamma_{\bar{\lambda}}} &\equiv 0 \,, \\ (\lambda - \lambda^{-1})\left( \alpha_\lambda - \overline{\alpha_{\bar{\lambda}}} \right) + 4( A - B\lambda )\overline{\beta_{\bar{\lambda}}} + 4(A-B\lambda^{-1}) \gamma_\lambda &\equiv 0 \,, \\ (\lambda - \lambda^{-1})\left( \beta_\lambda + \overline{\beta_{\bar{\lambda}}} \right) - 4( A - B\lambda^{-1} ) \left( \alpha _\lambda + \overline{\alpha_{\bar{\lambda}}} \right) &\equiv 0 \,. \end{split} \end{equation} Inspection of the two diagonal conditions in \eqref{eq:Sklyanin-potential-symmetry} proves \begin{proposition} \label{th:1st} For the entries of a \rm{K}-symmetric potential we have \begin{enumerate} \item $\displaystyle{\alpha_\lambda + \overline{\alpha_{\bar{\lambda}}} \equiv 0 \Longleftrightarrow \beta_\lambda + \overline{\beta_{\bar{\lambda}}} \equiv 0} \Longleftrightarrow \gamma_\lambda + \overline{\gamma_{\bar{\lambda}}} \equiv 0$. \item $\displaystyle{\alpha_\lambda \equiv 0 \Longrightarrow \beta_\lambda + \overline{\beta_{\bar{\lambda}}} \equiv 0}$. \end{enumerate} \end{proposition} In each of the four equations \eqref{eq:Sklyanin-potential-symmetry} we have powers of $\lambda$ ranging from $\lambda^{-1}$ to $\lambda^{d+1}$. Comparing coefficients gives for $k=-1,\,\ldots,\,d+1$ the system \begin{equation}\label{eq:Sklyanin-polynomial} \begin{split} -\left( \overline{\gamma}_{k+1} + \gamma_{k+1} \right) + 4 A \left(\overline{\alpha}_k + \alpha _k \right) + \overline{\gamma}_{k-1} + \gamma_{k-1} - 4 B \left( \overline{\alpha}_{k-1} + \alpha_{k-1} \right) &= 0 \\ \alpha_{k+1} - \overline{\alpha}_{k+1} - 4 B \overline{\gamma}_{k+1} + 4 A (\overline{\gamma}_k + \beta_k ) + \overline{\alpha}_{k-1} - \alpha_{k-1} - 4 B \beta_{k-1} &= 0 \\ \overline{\alpha}_{k+1} - \alpha_{k+1} - 4 B \gamma_{k+1} + 4 A (\overline{\beta}_k + \gamma_k ) + \alpha_{k-1} - \overline{\alpha}_{k-1} - 4 B \overline{\beta}_{k-1} &= 0 \\ 4 B ( \overline{\alpha}_{k+1} + \alpha_{k+1} ) - \overline{\beta}_{k+1} - \beta_{k+1} - 4 A (\overline{\alpha}_k + \alpha_k ) + \beta_{k-1} + \overline{\beta}_{k-1} &= 0\,. \end{split} \end{equation} Splitting into real part $\RE$ and imaginary part $\IM$ gives \begin{equation} \label{eq:split_equations} \begin{split} - \RE\gamma_{k+1} + 4 A \RE \alpha _k + \RE\gamma_{k-1} - 4 B \RE \alpha_{k-1} &= 0 \\ \mi\IM \alpha_{k+1} - 2 B \overline{\gamma}_{k+1} + 2 A (\overline{\gamma}_k + \beta_k ) - \mi\IM\alpha_{k-1} - 2 B \beta_{k-1} &= 0 \\ -\mi\IM \alpha_{k+1} - 2 B \gamma_{k+1} + 2 A (\overline{\beta}_k + \gamma_k ) + \mi\IM \alpha_{k-1} - 2 B \overline{\beta}_{k-1} &= 0 \\ 4 B \RE\alpha_{k+1} - \RE\beta_{k+1} - 4 A \RE \alpha_k + \RE \beta_{k-1} &= 0 \,.\end{split} \end{equation} \subsection{Real Parts.} Adding the first and last equations in \eqref{eq:split_equations} gives the recursion \begin{equation} \label{eq:alpha_recursion} 4B\RE [\alpha_{k+1} - \alpha_{k-1} ] -\RE [\beta_{d-k} - \beta_{d-k-2} ] + \RE [ \beta_{k-1} - \beta_{k+1} ] = 0\,. \end{equation} Plugging in $k=-1$ gives \[ 4B \RE \alpha_0 = \RE \beta_0 \] Plugging in $k=0$ gives \[ 4B \RE \alpha_1 + \RE \beta_{d-2} + \RE [\beta_{-1} - \beta_1 ] = 0 \] Hence $\RE\alpha_0,\,\RE\alpha_1$ are uniquely determined by $\RE\beta_0,\,\RE\beta_1$ and $\RE\beta_{d-2}$. Now the recursion \eqref{eq:alpha_recursion} uniquely determines the remaining $\RE\alpha_k$. We summarize in the following \begin{proposition} The real parts of the coefficients of $\beta_\lambda$ uniquely determine the real parts of the coefficients of $\alpha_\lambda$. If the coefficients of $\beta_\lambda$ are all imaginary, then also all the coefficients of $\alpha_\lambda$ are imaginary. \end{proposition} \subsection{Imaginary Parts.} Adding and subtracting the two middle equations in \eqref{eq:split_equations} and using \eqref{eq:reality} that $\gamma_k =-\overline{\beta}_{d-k-1}$ we get recursions \begin{equation} \label{eq:beta_recursion} \begin{split} A\,\RE [\beta_k - \beta_{d-k-1} ] + B\,\RE [ \beta_{d-k-2} - \beta_{k-1} ] &= 0\,,\\ \IM[\alpha_{k+1} - \alpha_{k-1}]+2A\IM[\beta_k-\beta_{d-k-1}] + 2 B \IM [\beta_{d-k-2}-\beta_{k-1}] &=0\,. \end{split} \end{equation} and combining gives \begin{equation} \label{eq:beta_recursion1} \mi\IM[\alpha_{k+1} - \alpha_{k-1}]+2A(\beta_k-\beta_{d-k-1}) + 2 B (\beta_{d-k-2}-\beta_{k-1}) =0\,. \end{equation} For $k=-1$ we obtain \begin{equation} \label{eq:1stbetarecursion} \begin{split} A\,\RE \beta_{-1} + B\,\RE \beta_{d-1} &= 0\,, \\ \mi\IM \alpha_0 + 2A \beta_{-1} + 2B \beta_{d-1} &= 0\,, \end{split} \end{equation} and since $\RE\beta_{-1} = 0$ by \eqref{eq:residue} it follows that \begin{equation} \label{eq:re-top} \RE \beta_{d-1} = 0\,, \end{equation} and the two imaginary numbers $\beta_{-1},\,\beta_{d-1}$ determine the imaginary part of $\alpha_0$. For $k=0$ we obtain \begin{equation*} \begin{split} A\RE \beta_0 + B \RE \beta_{d-2} &= 0\,, \\ \mi \IM \alpha_1 + 2A (\beta_0 - \beta_{d-1}) + 2B(\beta_{d-2} - \beta_{-1}) &= 0\,, \end{split} \end{equation*} Hence $\RE\beta_0$ determines $\RE\beta_{d-2}$, and the imaginary number $A\beta_0 + B \beta_{d-2}$ determines $\IM\alpha_1$. Next for $k=1$ we get \begin{equation*}\begin{split} A\RE[\beta_1 - \beta_{d-2}] + B \RE [\beta_{d-3} - \beta_0] &= 0\,,\\ \mi \IM [\alpha_2 - \alpha_0] + 2A (\beta_1 - \beta_{d-2}) + 2B(\beta_{d-3} - \beta_0) &= 0\,, \end{split} \end{equation*} and therefore $\RE\beta_1$ determines $\RE\beta_{d-3}$. Likewise, $\RE\beta_2$ determines $\RE\beta_{d-4}$. Thus $\RE\beta_k$ determines $\RE\beta_{d-k-2}$. When the degree is even $d=2N$, then the last degree of freedom is given by $\RE\beta_{N-2}$ which determines $\RE\beta_N$. Hence choices for $\RE\beta_0, \RE\beta_1,\,\ldots ,\, \RE\beta_{N-2}$ determine the remaining $\RE\beta_{N-1},\,\ldots ,\, \RE_{d-2}$. Thus for even degree there are $(d-2)/2$ choices. Likewise, when the degree is odd, there are $(d-1)/2$ choices. When $d$ is odd, then for $k=(d+1)/2$ we have $d-k-1 = (d-3)/2$ and hence \begin{equation*}\begin{split} \RE [ \alpha_{(d+1)/2} + \alpha_{(d-3)/2}] &= 0\,,\\ \IM [ \alpha_{(d+1)/2} - \alpha_{(d-3)/2}] &= 0\,. \end{split} \end{equation*} Now $(d+1)/2 - (d-3)/2 = 2$, so these two indices are two apart, but the recursion \eqref{eq:beta_recursion} gives no additional condition, as it reads \begin{equation*} \IM [\alpha_{(d+1)/2} - \alpha_{(d-3)/2} ] + 2A (\beta_{(d-1)/2} - \beta_{(d-1)/2}) + 2B (\beta_{(d-3)/2} - \beta_{(d-3)/2}) = 0 \,. \end{equation*} Therefore there is no additional constraint on the imaginary parts of the coefficients of $\beta_\lambda$ for the case $d$ odd. \\ If $d$ is even then the indices $k$ and $d-k-1$ are never two apart, so the recursion \eqref{eq:beta_recursion} in conjunction with the reality condition gives no additional constraint in this case. There is a further one-dimensional reduction on the coefficients of $\beta_\lambda$ when the degree is even due to the following \begin{proposition} Let $\xi_\lambda$ be a \rm{K}-symmetric potential with non-zero diagonal. If the degree $d$ is even, then \begin{equation*} \sum_{k=-1}^{d-1} (-1)^{k+1} \beta_k = 0\,. \end{equation*} \end{proposition} \begin{proof} When $d$ is even, the recursion \eqref{eq:beta_recursion1} gives \begin{equation*} \begin{split} \mi\IM\alpha_0 &= -2A\,\beta_{-1} - 2B\,\beta_{d-1} \\ \mi\IM\alpha_1 &= - 2 A (\beta_0 -\beta_{d-1}) - 2 B(-\beta_{-1} + \beta_{d-2}) \\ \vdots & \\ \mi\IM\alpha_{d-2} &= -2A(\beta_{-1} + \beta_1 - \beta_2 + \ldots -\beta_{d-2} ) - 2B( - \beta_0 + \beta_1 - \ldots+ \beta_{d-3} + \beta_{d-1}) \\ \mi\IM\alpha_{d-1} &= -2A(\beta_0 - \beta_1 + \beta_2 - \ldots - \beta_{d-1}) - 2B (-\beta_{-1} + \beta_0 - \beta_1 + \ldots + \beta_{d-2})\,. \end{split} \end{equation*} The symmetry $\IM\alpha_k = \IM\alpha_{d-k-1}$ for all $k=0,\,\ldots,\,d-1$ gives the claim. \end{proof} In conclusion, when $d$ is even, there are $d$ choices, and when $d$ is odd there are $d+1$ real choices for $\IM\beta_{-1},\,\ldots,\,\IM \beta_{d-1}$ that uniquely determine $\IM\alpha_0,\,\ldots,\,\IM\alpha_{d-1}$. This proves \begin{proposition}\label{th:beta_count} Let $\xi_\lambda$ be a \rm{K}-symmetric potential of degree $d$. \rm{(i)} If $d$ is even, there are $(d-2)/2$ real degrees of freedom for the real parts of the coefficients of $\beta_\lambda$. If $d$ is odd, there are $(d-1)/2$ real degrees of freedom for the real parts of the coefficients of $\beta_\lambda$. \rm{(ii)} The imaginary parts of the coefficients of $\beta_\lambda$ uniquely determine the imaginary parts of the coefficients of $\alpha_\lambda$, and there are $d+1$ many real choices when $d$ is odd, and $d$ many when $d$ is even. \end{proposition} Adding up parameter counts from Proposition \ref{th:beta_count} shows that K-symmetry halves the dimension of the parameter space. More precisely, we have proven \begin{theorem} \label{th:imaginary} The space of \rm{K}-symmetric potentials of degree $d$ is real $(3d-2)/2$--dimensional for even $d$, and $(3d+1)/2$--dimensional for odd $d$. \end{theorem} \begin{example} \label{ex:im} (i) For $d=1$ we have $\RE \beta_0 = 0,\,\RE\alpha_0 = 0$ and $\IM \alpha_0 = -2 (A\beta_{-1} + B\beta_0)$. Hence degree one potentials all have purely imaginary entries and have the form \begin{equation*} \xi_\lambda = \mi \begin{pmatrix} -2(A\,\beta_{-1} + B\,\beta_0) & \beta_{-1} \lambda^{-1} + \beta_0 \\ \beta_0 + \beta_{-1} \lambda & 2(A\,\beta_{-1} + B\,\beta_0) \end{pmatrix} \end{equation*} with $\beta_{-1}, \,\beta_0 \in \R$ free, $\beta_{-1} > 0$. (ii) For $d=2$ we have $\RE\beta_1 = 0$ and $(A+B)\RE\beta_0 = 0$ so either $\RE\beta_0 = 0$ or $B=-A$. Further, $\RE\alpha_0 = -\RE\alpha_1 = \RE\beta_0/(4B)$ and $\mi\IM\alpha_0 = -2(A\beta_{-1} +B\beta_1)$ as well as $\mi\IM\alpha_1 = -2(A(\beta_0 - \beta_1) + B(\beta_0 - \beta_{-1}))$. Thus reality $\overline{\alpha_0} = -\alpha_1$ holds if and only if $$\beta_{-1} - \beta_0 + \beta_1 = 0\,,$$ and again all entries are purely imaginary and given by \begin{equation*} \begin{split} \alpha_\lambda &= -2(A\beta_{-1} + B\beta_1) (1+\lambda) \,, \\ \beta_\lambda &= \beta_{-1}\lambda^{-1} +(\beta_{-1} + \beta_1) + \beta_1\lambda \,. \end{split} \end{equation*} The general form of degree two \rm{K}-symmetric potentials is \begin{equation*} \xi_\lambda = \mi \begin{pmatrix} -2 (A\beta_{-1} + B \beta_1)(1+\lambda) & \beta_{-1}( \lambda^{-1} + 1) + \beta_1 (1+ \lambda) \\ \beta_1(1+\lambda) + \beta_{-1}(1+ \lambda^2) & 2(A \beta_{-1} + B\beta_1)(1+\lambda) \end{pmatrix}\,. \end{equation*} (iii) For $d=3$ we have $\RE\beta_2 = 0$ and $A\RE\beta_0 + B\RE\beta_1 = 0$. Hence $\RE\beta_1 = -(A/B) \RE\beta_0$. Further, $\RE\alpha_0 = -\RE\alpha_2 = (1/4B)\RE\beta_0$ and $\RE\alpha_1 = 0$. Hence $\RE\beta_0 \neq 0$ guarantees that not all coefficients are purely imaginary. Continuing, we get $\mi\IM\alpha_0 = \mi\IM\alpha_2 = -2(A\beta_{-1} + B\beta_2),\,\mi\IM\alpha_1 = 2(A\beta_2 + B\beta_{-1})$. Thus in the degree 3 case we see that it is possible to get K-symmetric potentials which do not have purely imaginary entries, and \begin{equation*} \begin{split} \alpha_0 &= 1/(4B)\RE\beta_0 -2(A\beta_{-1} + B\beta_2) \,,\\ \alpha_1 &= 2(A\beta_2 + B\beta_{-1}) \,,\\ \alpha_2 &= -1/(4B)\RE\beta_0 - 2(A\beta_{-1} + B \beta_2 )\,. \end{split} \end{equation*} Here $\beta_{-1}, \beta_2 \in \mi\R$, so also $\alpha_1 \in \mi\R$. But $\beta_0,\,\beta_1 \in \C$ in general, subject only to $A\RE\beta_0 + B\RE\beta_1 = 0$, and thus also $\alpha_0,\,\alpha_2 \in \C$ in general. \end{example} \section{Imaginary potentials} By Proposition \ref{th:1st} \rm{(i)}, a K-symmetric potential satisfying $\alpha_\lambda + \overline{\alpha_{\bar{\lambda}}} \equiv 0$ has purely imaginary coefficients so that \begin{equation}\label{eq:Sklyanin-reality} \alpha_\lambda + \overline{\alpha_{\bar{\lambda}}} \equiv \beta_\lambda + \overline{\beta_{\bar{\lambda}}} \equiv \gamma_\lambda + \overline{\gamma_{\bar{\lambda}}} \equiv 0\,. \end{equation} Hence we can write such an \emph{imaginary potential} as \[ \xi_\lambda = \mi \begin{pmatrix} \alpha_\lambda & \beta_\lambda \\ \gamma_\lambda & -\alpha_\lambda \end{pmatrix} \] where $\alpha_\lambda,\,\beta_\lambda,\,\gamma_\lambda$ all have real coefficients. Now the recursion \eqref{eq:beta_recursion1} reads \begin{equation} \label{eq:beta_recursion2} \alpha_{k+1} - \alpha_{k-1} + 2A(\beta_k-\beta_{d-k-1}) + 2 B (\beta_{d-k-2}-\beta_{k-1}) =0\,. \end{equation} For $k=-1$ we obtain \begin{equation}\label{eq:alpha0} \alpha_0 = -2A\,\beta_{-1} - 2B\,\beta_{d-1}\,. \end{equation} \begin{example} Since \rm{K}-symmetric potentials of degree one and two are imaginary and computed in example \ref{ex:im}, the first notable difference occurs for $d=3$: \\ {\rm{(i)}} For $d=3$ the recursion gives \begin{equation*} \begin{split} \alpha_0 &= -2A\beta_{-1} - 2 B \beta_2 \\ \alpha_1 &= - 2A(\beta_0 - \beta_2) - 2B(-\beta_{-1} + \beta_1) \\ \alpha_2 &= -2A\beta_{-1} - 2 B \beta_2 \,. \end{split} \end{equation*} Hence reality $\alpha_0 = \alpha_2$ is satisfied without a further condition. \\ {\rm{(ii)}} For $d=4$ the recursion gives \begin{equation*} \begin{split} \alpha_0 &= -2A\beta_{-1} - 2 B \beta_3 \\ \alpha_1 &= - 2A ( \beta_0 - \beta_3) -2B (- \beta_{-1} + \beta_2) \\ \alpha_2 &= -2 A (\beta _{-1}+\beta _1 - \beta _2) -2 B (-\beta_0 + \beta_1 + \beta_3) \\ \alpha_3 &= -2 A (\beta_0 - \beta_1 + \beta_2 - \beta_3 ) -2 B (-\beta_{-1} + \beta_0 - \beta_1 + \beta_2 ) \end{split} \end{equation*} The reality conditions $\alpha_0 = \alpha_3,\,\alpha_1 = \alpha_2$ hold for all $A,\,B$ if and only if \begin{equation} \label{eq:beta_sum2} \beta_{-1} -\beta_0 + \beta_1 - \beta_2 + \beta_3 = 0\,. \end{equation} \end{example} \subsection{Off-diagonal K-symmetric potentials.} By Proposition \ref{th:1st} \rm{(ii)}, an off-diagonal K-symmetric potential is imaginary, so of the form \begin{equation} \label{eq:offpot} \xi_\lambda = \mi\, \begin{pmatrix} 0 & \beta_\lambda \\ \gamma_\lambda & 0 \end{pmatrix} \,. \end{equation} where $\beta_\lambda,\,\gamma_\lambda$ have only real coefficients. Now the recursion \eqref{eq:beta_recursion1} reads \begin{equation} \label{eq:beta_recursion3} A(\beta_k-\beta_{d-k-1}) + B (\beta_{d-k-2}-\beta_{k-1}) =0 \end{equation} for $k=-1,\,\ldots,\,d+1$. Amongst the $d+1$ free paramters for off-diagonal, and hence imaginary $\beta_\lambda$, this recursion reduces the freedom by a further $d/2$ when $d$ is even, and $(d-1)/2$ for $d$ odd. The first equations for $k=-1,\,\ldots,\,2$ read \begin{equation*} \begin{split} A\beta_{-1} + B \beta_{d-1} &= 0 \\ A(\beta_0 - \beta_{d-1}) + B( \beta_{d-2} - \beta_{-1} ) &= 0 \\ A(\beta_1 - \beta_{d-2}) + B( \beta_{d-3} - \beta_0 ) &= 0 \\ A(\beta_2 - \beta_{d-3}) + B( \beta_{d-4} - \beta_1 ) &= 0 \end{split} \end{equation*} \begin{example} We compute some low degree examples:\\ 1. For $d=1$ we get $\beta_0 = -\beta_{-1} A/B$, so degree one off-diagonal \rm{K}-symmetric potentials look like \begin{equation} \label{eq:deg1potential} \xi_\lambda = \mi \beta_{-1} \begin{pmatrix} 0 & \lambda^{-1} -A/B \\ -A/B + \lambda & 0 \end{pmatrix} \end{equation} which drops rank at $\lambda = A/B,\,B/A$. \\ 2. For $d=2$ we get $A\beta_{-1} + B \beta_1 = 0$ and $A(\beta_0 - \beta_1) + B( \beta_0 - \beta_{-1} ) = 0$, so degree two potentials are of the form \begin{equation} \label{eq:deg2potential} \begin{split} \xi_\lambda &= \mi \beta_{-1} \begin{pmatrix} 0 & \lambda^{-1} + (1-A/B) - A/B \lambda \\ -A/B + (1-A/B) \lambda +\lambda^2 & 0 \end{pmatrix} \\ &= \mi \beta_{-1} (\lambda + 1) \begin{pmatrix} 0 & \lambda^{-1} - A/B \\\ -A/B + \lambda & 0 \end{pmatrix} \end{split} \end{equation} Again $\det \xi_\lambda$ has simple roots at $\lambda = A/B,\,B/A$. In addition $\xi_\lambda \equiv 0$ at $\lambda = -1$. \\ 3. For $d=3$ we can write $\beta_1,\,\beta_2$ in terms of $\beta_{-1},\,\beta_0$, in fact \[ \beta_1 = (1-\tfrac{A^2}{B^2}) \beta_{-1} - \tfrac{A}{B} \beta_0\,\,,\qquad \beta_2 = - \tfrac{A}{B} \beta_{-1} \,. \] Inserting these expressions into $\beta_\lambda,\, \gamma_\lambda$, we observe that they have a common factor of degree 2, more specifically \begin{equation*} \beta_\lambda / (\lambda^{-1} - A/B) = \gamma_\lambda /(\lambda - A/B) = \beta_{-1} \left(\lambda^2 + \tfrac{A}{B}\lambda + 1\right)+\lambda \beta_0 \,. \end{equation*} Hence degree 3 off-diagonal \rm{K}-symmetric potentials are all of the form \begin{equation} \label{eq:deg3potential} \xi_\lambda = \mi (\beta_{-1} \left(\lambda^2 + \tfrac{A}{B}\lambda + 1\right)+\lambda \beta_0) \begin{pmatrix} 0 & \lambda^{-1} - A/B \\\ -A/B + \lambda & 0 \end{pmatrix} \,. \end{equation} This is the general behaviour of off-diagonal finite gap K-symmetric potentials, and the content of the next theorem. \end{example} \begin{theorem} Let $\xi_\lambda$ be an off-diagonal \rm{K}-symmetric potential of degree $d$. Then there exists a unique real polynomial $\lambda \mapsto p(\lambda)$ of degree $d-1$ such that \[ \xi_\lambda = \mi\,p(\lambda) \begin{pmatrix} 0 & \lambda^{-1} - A/B \\\ -A/B + \lambda & 0 \end{pmatrix} \,. \] \end{theorem} \begin{proof} Write $\xi_\lambda$ as in \eqref{eq:offpot} with entries $\beta_\lambda,\,\gamma_\lambda$ with $\deg \gamma_\lambda = d$. Long division shows that the unique polynomial $\sum_{k=0}^{d-1} p_k \lambda^k$ of degree $d-1$ with coefficients recursivley defined by \begin{equation*} p_0 = \beta_{-1}\,, \qquad p_k = \beta_{k-1} +\frac{A}{B}\, p_{k-1} \quad \mathrm{for} \quad k=1,\,\ldots ,\,d-1 \end{equation*} divides both $\beta_\lambda,\,\gamma_\lambda$ with quotients $\lambda^{-1} - A/B$ respectively $\lambda - A/B$. Since all coefficients $p_k$ are real, this concludes the proof. \end{proof} The spectral curve $\Sigma$ is the 2-point compactification of \[ \Sigma^* = \left\{ (\nu,\, \lambda ) \in \C^2 \mid \nu^2 = - \det \xi_\lambda \right\}\,. \] When $\xi = p \,\tilde{\xi}$ for a polynomial $p$, then after renormalization $\xi$ and $\tilde{\xi}$ give the same spectral curve, since the roots of $p^2$ all have even order. Any spectral curve comes from an isospectral family of potentials that all share the same determinant. In each isospectral family there are off-diagonal potentials, and by Proposition \ref{th:1st} \rm{(ii)} they are automatically imaginary. \begin{corollary} \label{th:off-diagonalKsymmetry} All spectral curves of off-diagonal {\rm{K}}-symmetric potentials have genus one. Higher spectral genus examples must come from potentials with non-zero diagonal. \end{corollary} \section{Spectral Theory of K-matrices} We compute the values of $\lambda$ for which a K-matrix fails to be invertible, and determine the corresponding kernels. We show that the eigenvectors of $K$ are $\lambda$-independent, and we compute the residues at the simple poles of $K^{-1}$. \subsection{Kernels.} Computing \[ -\lambda^2 \det K (\lambda) = \lambda ^4 + 16 A B \lambda ^3 - 2 \lambda ^2 \left(8 A^2+8 B^2+1\right)+16 A B \lambda +1 \] and $\lambda \mapsto \det K(\lambda)$ has four simple roots \begin{equation} \begin{split} \label{eq:roots} \varrho &= \frac{2A - \sqrt{4 A^2+1}}{2 B + \sqrt{4 B^2+1}} \quad \mbox{ and } \quad r = \frac{2A + \sqrt{4 A^2+1}}{2 B + \sqrt{4 B^2+1}}\,, \\ \varrho^{-1} &=\frac{2A + \sqrt{4 A^2+1}}{2 B-\sqrt{4 B^2+1}} \quad \mbox{ and } \quad r^{-1} = \frac{2A - \sqrt{4 A^2+1}}{2 B-\sqrt{4 B^2+1}} \,. \end{split} \end{equation} The null-set of $\det K$, that is \begin{equation}\label{eq:nullset} \mathcal{N} = \{ \varrho,\,r,\,\varrho^{-1},\,r^{-1} \}\,, \end{equation} is invariant under inversion as well as under the map $(A,\,B) \mapsto (-A,\,-B)$. The matrices $K(\lambda),\,\lambda \in \mathcal{N}$ drop rank, and kernels compute to \begin{equation} \label{eq:kernels} \begin{split} &\ker K (\varrho) = \ker K (r) =\ker \begin{pmatrix} 1 & 2 B + \sqrt{1 + 4 B^2} \\ 0 & 0 \end{pmatrix} \,,\\ &\ker K (\varrho^{-1}) = \ker K (r^{-1}) =\ker \begin{pmatrix} 1 & 2 B - \sqrt{1 + 4 B^2} \\ 0 & 0 \end{pmatrix} \,. \end{split} \end{equation} Note that \begin{equation*} \ker K (\varrho) \perp \ker K (\varrho^{-1}) \end{equation*} and that both kernels are independent of the value of $A$ in the $K$-matrix. \subsection{Spectrum.} $K$ has eigenvalues $\mu_\pm = \tfrac{1}{2} ( \tr K \pm \sqrt{\tr^2 K - 4 \det K})$ and compute to \begin{equation} \label{eq:mu_pm} \begin{split} \mu_- (\lambda) &= 4 A - 2 B \left(\lambda + \lambda^{-1} \right) - \sqrt{4 B^2+1} \left(\lambda - \lambda^{-1} \right) \\ \mu_+ (\lambda) &= 4 A - 2 B \left(\lambda + \lambda^{-1} \right) + \sqrt{4 B^2+1} \left(\lambda - \lambda^{-1} \right) \end{split} \end{equation} Note that for all $\lambda \in \mathbb{C}^\ast$ we have \begin{equation} \label{eq:mu-symmetry} \mu_-(\lambda^{-1}) = \mu_+(\lambda)\,. \end{equation} Now $\mu_-$ has simple roots at $\varrho,\,r$, while $\mu_+$ has simple roots at $\varrho^{-1},\,r^{-1}$ , with $\varrho,\,r,\,\varrho^{-1},\,r^{-1} $ as in \eqref{eq:roots}. While the eigenvalues of a K-matrix depend on $\lambda$ as well as both constants $A$ and $B$, the eigenvectors do not depend on $\lambda$, nor the constant $A$. \begin{proposition} The eigenvectors of $K$ are given by \begin{equation} \label{eq:eigenvectors} \mathbf{v} = \begin{pmatrix} - \sqrt{4 B^2+1}-2 B \\ 1 \end{pmatrix} \quad \mbox{ and } \quad \mathbf{v}^\perp = \begin{pmatrix} \sqrt{4 B^2+1}-2 B \\ 1 \end{pmatrix} \,. \end{equation} so for all $\lambda \in \mathbf{C}^\ast$ we have \begin{equation*} K(\lambda) \,\mathbf{v} = \mu_-(\lambda) \,\mathbf{v} \quad \mbox{ and } \quad K(\lambda) \,\mathbf{v}^\perp = \mu_+ (\lambda) \,\mathbf{v}^\perp \,. \end{equation*} \end{proposition} For the orthogonal matrix $V$ whose columns are $\mathbf{v},\mathbf{v}^\perp$, that is \begin{equation*} V = \begin{pmatrix} - \sqrt{4 B^2+1}-2 B & \sqrt{4 B^2+1}-2 B \\ 1 & 1 \end{pmatrix} \end{equation*} we have the diagonalization of $K$ by \begin{equation} \label{eq:Kdiagonal} K = V\, \mathrm{diag}[\,\mu_-,\,\mu_+\,] \,V^{-1}\,. \end{equation} The span $\langle \,\,\rangle$ of the eigenvectors $\mathbf{v},\,\mathbf{v}^\perp$ of $K$ coincide with the kernels in \eqref{eq:kernels}, and we have \begin{equation*} \begin{split} \langle \mathbf{v} \rangle &= \ker K (\varrho) = \ker K (r) \\ \langle \mathbf{v}^\perp \rangle &= \ker K (\varrho^{-1}) = \ker K (r^{-1})\,. \end{split} \end{equation*} Suppose we have a K-symmetric potential $\xi_\lambda$. Evaluating the symmetry \eqref{eq:Sklyanin-potentials} at $\lambda = \varrho$ and applying to the vector $\mathbf{v} \in \ker K(\varrho)$ gives \begin{equation*} K(\varrho)\,\xi_\varrho\,\mathbf{v} + \overline{\xi_\varrho}^t K(\varrho)\,\mathbf{v} = K(\varrho)\,\xi_\varrho\,\mathbf{v} = 0\,, \end{equation*} and hence $\xi_\varrho\,\mathbf{v} \in \ker K(\varrho) = \langle \mathbf{v} \rangle$. Similarly, $\xi_r\,\mathbf{v} \in \ker K(r) = \langle \mathbf{v} \rangle$, $\xi_{\varrho^{-1}}\,\mathbf{v}^\perp \in \ker K(\varrho^{-1}) = \langle \mathbf{v}^\perp \rangle$ and $\xi_{r^{-1}}\,\mathbf{v}^\perp \in \ker K(r^{-1}) = \langle \mathbf{v}^\perp \rangle$. We summarize this in the following \begin{proposition} At each of the four points $\lambda \in \{ \varrho,\,r,\,\varrho^{-1},\,r^{-1} \}$ where $K(\lambda)$ drops rank, the kernel of $K(\lambda)$ is an eigenspace of $\xi_\lambda$. We may assume without loss of generality that \begin{equation*} \xi_\varrho \mathbf{v} = \xi_r \mathbf{v} = -\nu\,\mathbf{v} \quad \mbox{ and } \quad \xi_{\varrho^{-1}} \mathbf{v}^\perp = \xi_{r^{-1}} \mathbf{v}^\perp = \nu\,\mathbf{v}\,. \end{equation*} \end{proposition} \subsection{The Inverse and Residues.} Now $K^{-1} = V\, \mathrm{diag}[\,\mu_-^{-1},\,\mu_+^{-1}\,] \,V^{-1}$ has simple poles at the four points $\lambda \in \{ \varrho,\,r,\,\varrho^{-1},\,r^{-1} \}$ since $\mu_\pm$ have simple roots there. Since the matrix $V$ does not depend on $\lambda$, the residues of $K^{-1}$ at the four points $\lambda \in \mathcal{N}$ as in \eqref{eq:nullset} are given by \[ \mathrm{res}[K^{-1},\lambda] = V\, \mathrm{diag}\bigl[ \,\mathrm{res}[\mu_-^{-1},\,\lambda],\,\mathrm{res}[\mu_+^{-1},\,\lambda \,\bigr] \,V^{-1}\,, \qquad \lambda \in \mathcal{N}. \] \begin{lemma} The kernels of the residues of $K^{-1}$ coincide with the kernels of $K$ as follows \begin{equation*} \begin{split} \ker \mathrm{res}[K^{-1},\varrho] &= \ker K(\varrho^{-1}) = \ker \mathrm{res}[K^{-1},r] = \ker K(r^{-1}) \,,\\ \ker \mathrm{res}[K^{-1},\varrho^{-1}] &= \ker K(\varrho) = \ker \mathrm{res}[K^{-1},r^{-1}] = \ker K(r)\,. \end{split} \end{equation*} \end{lemma} \begin{proof} To compute the residues, first multiply away the poles \begin{equation*} \begin{split} (\lambda-\varrho) \,\mu_-^{-1} &= ( (\sqrt{4 A^2+1}+2 A)\lambda^{-1} - \sqrt{4 B^2+1}-2 B)^{-1} \\ (\lambda-r) \,\mu_-^{-1} &= - ((\sqrt{4 A^2+1}-2 A)\lambda^{-1}+\sqrt{4 B^2+1} +2 B )^{-1} \\ (\lambda - \varrho^{-1})\, \mu_+^{-1} &= ((-\sqrt{4 A^2+1}+2 A)\lambda^{-1}+\sqrt{4 B^2+1} -2 B )^{-1} \\ (\lambda- r^{-1}) \,\mu_+^{-1} &= ((\sqrt{4 A^2+1}+2 A)\lambda^{-1}+\sqrt{4 B^2+1} -2 B )^{-1} \end{split} \end{equation*} then evaluate to obtain the four residues \begin{equation*} \begin{split} \mathrm{res}[\mu_-^{-1},\,\varrho] = \lim_{\lambda \to \varrho} (\lambda-\varrho)\,\mu_-^{-1} &= (\tfrac{1}{2} - \tfrac{A}{\sqrt{4A^2+1}} ) (2B - \sqrt{4 B^2+1} )\\ \mathrm{res}[\mu_-^{-1},\,r] = \lim_{\lambda \to r}(\lambda-r)\,\mu_-^{-1} &= ( \tfrac{1}{2} + \tfrac{A}{\sqrt{4 A^2+1}} ) (2B - \sqrt{4 B^2+1}) \\ \mathrm{res}[\mu_+^{-1},\,\varrho^{-1}] = \lim_{\lambda \to \varrho^{-1}}(\lambda - \varrho^{-1})\,\mu_+^{-1} &= ( \tfrac{1}{2} + \tfrac{A}{\sqrt{4 A^2+1}}) ( 2B + \sqrt{4 B^2+1}) \\ \mathrm{res}[\mu_+^{-1},\,r^{-1}] = \lim_{\lambda \to r^{-1}}(\lambda- r^{-1})\,\mu_+^{-1} &= ( \tfrac{1}{2}-\tfrac{A}{\sqrt{4 A^2+1}}) ( 2B + \sqrt{4 B^2+1}) \end{split} \end{equation*} The residues of $K^{-1}$ at the four simple poles compute to \begin{equation*} \begin{split} \mathrm{res}[K^{-1},\varrho] &= V \left( \begin{smallmatrix}\mathrm{res}[\mu_-^{-1},\varrho] & 0 \\ 0 & 0 \end{smallmatrix} \right) V^{-1} \\ &= \tfrac{\sqrt{4 A^2+1}-2A}{4 \sqrt{4 A^2+1} \sqrt{4 B^2+1}} \left( \begin{smallmatrix} -1 & \sqrt{4B^2+1}-2 B \\ \sqrt{4 B^2+1}-2 B & - (\sqrt{4 B^2+1}-2 B)^2\\ \end{smallmatrix} \right) \sim \left( \begin{smallmatrix} 1 & 2 B - \sqrt{4 B^2 + 1} \\ 0 & 0 \end{smallmatrix} \right) \\ \mathrm{res}[K^{-1},r] &= V \left( \begin{smallmatrix} \mathrm{res}[\mu_-^{-1},r] & 0 \\ 0 & 0 \end{smallmatrix} \right) V^{-1} \\ &= \tfrac{\sqrt{4 A^2+1}+2A}{4 \sqrt{4 A^2+1} \sqrt{4 B^2+1}} \left( \begin{smallmatrix} -1 & \sqrt{4B^2+1}-2 B \\ \sqrt{4 B^2+1}-2 B & - (\sqrt{4 B^2+1}-2 B)^2\\ \end{smallmatrix} \right) \sim \left( \begin{smallmatrix} 1 & 2 B - \sqrt{4 B^2 + 1} \\ 0 & 0 \end{smallmatrix} \right) \\ \mathrm{res}[K^{-1},\varrho^{-1}] &= V \left( \begin{smallmatrix} 0 & 0 \\ 0 & \mathrm{res}[\mu_+^{-1},\varrho^{-1}] \end{smallmatrix} \right) V^{-1} \\ &= \tfrac{\sqrt{4 A^2+1}+2A}{4 \sqrt{4 A^2+1} \sqrt{4 B^2+1}} \left( \begin{smallmatrix} -1 & \sqrt{4B^2+1}+2 B \\ \sqrt{4 B^2+1}+2 B & - (\sqrt{4 B^2+1}+2 B)^2\\ \end{smallmatrix} \right) \sim \left( \begin{smallmatrix} 1 & 2 B + \sqrt{4 B^2 + 1} \\ 0 & 0 \end{smallmatrix} \right) \\ \mathrm{res}[K^{-1},r^{-1}] &= V \left( \begin{smallmatrix} 0 & 0 \\ 0 & \mathrm{res}[\mu_+^{-1},r^{-1}]\end{smallmatrix} \right) V^{-1} \\ &= \tfrac{\sqrt{4 A^2+1}-2A}{4 \sqrt{4 A^2+1} \sqrt{4 B^2+1}} \left( \begin{smallmatrix} -1 & \sqrt{4B^2+1}+2 B \\ \sqrt{4 B^2+1}+2 B & - (\sqrt{4 B^2+1}+2 B)^2\\ \end{smallmatrix} \right) \sim \left( \begin{smallmatrix} 1 & 2 B + \sqrt{4 B^2 + 1} \\ 0 & 0 \end{smallmatrix} \right) \end{split} \end{equation*} Comparing row-reduced matrices in \eqref{eq:kernels} and the above row-reduced residues proves the claim. \end{proof} \section{Products of K-matrices} We next look at products and ratios of K-matrices. Suppose we have two K-matrices \begin{equation} \label{eq:K0K1} K_0 = \begin{pmatrix} 4 A_0 -4 B_0 \lambda & \lambda - \lambda^{-1} \\ \lambda - \lambda^{-1} & 4 A_0 - 4 B_0 \lambda^{-1} \end{pmatrix},\, K_1 = \begin{pmatrix} 4 A_1 -4 B_1 \lambda & \lambda - \lambda^{-1} \\ \lambda - \lambda^{-1} & 4 A_1 - 4 B_1 \lambda^{-1} \end{pmatrix}. \end{equation} Then \[ [K_0,\, K_1] = 4 \left( \lambda - \lambda^{-1} \right)^2 (B_0 - B_1) \begin{pmatrix} 0 & -1 \\ 1 & 0 \end{pmatrix} \] and thus \begin{equation} \label{eq:Commutator0} [K_0,\, K_1] = 0 \quad \mbox{for all } \lambda \in \mathbb{C}^\ast \Longleftrightarrow B_0 = B_1\,. \end{equation} \begin{proposition} \label{th:K1K0} Suppose $K_0,\,K_1$ are two K-matrices with \begin{equation} \label{eq:sign-change} A_0 B_1 = - A_1 B_0\,. \end{equation} Then there exist unique rational functions $p,\,q$, and a degree one potential $\eta_\lambda$ with \begin{equation} \label{eq:K1K0} K_1^{-1} K_0 = p\,\mathbbm{1} + q\,\eta_\lambda \end{equation} \end{proposition} \begin{proof} We compute \begin{equation*} \begin{split} \mathrm{adj} K_1 K_0 &= \begin{pmatrix} 4 A_1 -4 B_1 \lambda^{-1} & -(\lambda - \lambda^{-1}) \\ -(\lambda - \lambda^{-1}) & 4 A_1 - 4 B_1 \lambda \end{pmatrix} \begin{pmatrix} 4 A_0 -4 B_0 \lambda & \lambda - \lambda^{-1} \\ \lambda - \lambda^{-1} & 4 A_0 - 4 B_0 \lambda^{-1} \end{pmatrix} \\ = &\bigl( 16(A_0A_1+B_0B_1) - (\lambda-\lambda^{-1})^2 \bigr) \,\mathbbm{1} \,\, + \\ \lambda^{-1}&\begin{pmatrix} -16(A_0B_1 + A_1B_0 \lambda^2) & 4(A_1-A_0-(B_1-B_0)\lambda^{-1})(\lambda^2-1) \\ 4(A_1-A_0-(B_1-B_0)\lambda)(\lambda^2-1) & -16(A_1 B_0 + A_0 B_1\lambda^2) \end{pmatrix} \end{split} \end{equation*} The last matrix is trace-free if and only if \eqref{eq:sign-change} holds, and then computes to \begin{equation*} \begin{split} \begin{pmatrix} -16(A_0B_1 +A_1 B_0 \lambda^2) & 4(A_1-A_0-(B_1-B_0)\lambda^{-1})(\lambda^2-1) \\ 4(A_1-A_0-(B_1-B_0)\lambda)(\lambda^2-1) & -16(A_1B_0 + A_0 B_1\lambda^2) \end{pmatrix}& \\ = 4 (\lambda^2-1) \begin{pmatrix} 4A_0 B_1 & A_1-A_0-(B_1-B_0)\lambda^{-1} \\ A_1-A_0-(B_1-B_0)\lambda & -4A_0 B_1 \end{pmatrix}&\,. \end{split} \end{equation*} Replacing $\mathrm{adj}K_1 = \det K_1 K_1^{-1}$ and putting the above computation together we set \begin{equation} \label{eq:fgK1K0} \begin{split} p(\lambda) &=\bigl( 16(A_0A_1+B_0B_1) - (\lambda-\lambda^{-1})^2 \bigr)/ \det K_1 (\lambda)\,, \\ q(\lambda) &= - 4\mi (\lambda - \lambda^{-1})/ \det K_1 (\lambda)\,, \\ \eta_\lambda &= \mi \begin{pmatrix} 4A_0B_1 & A_1-A_0-(B_1-B_0)\lambda^{-1} \\ A_1-A_0-(B_1-B_0)\lambda & -4A_0B_1 \end{pmatrix}\,. \end{split} \end{equation} \end{proof} We next show that potentials that are K-symmetric with respect to two distinct K-matrices only give rise to Delaunay surfaces. \begin{corollary} Let $K_0,\,K_1$ be two K-matrices satisfying \eqref{eq:sign-change}, and $\xi_\lambda$ a potential that is K-symmetric with respect to both $K_0$ and $K_1$. Then $\xi_\lambda$ is a polynomial multiple of a degree 1 potential. \end{corollary} \begin{proof} Suppose we have two different K-matrices $K_0$ and $K_1$ that satisfy the condition \eqref{eq:sign-change}. Suppose we have a potential $\xi_\lambda$ that is K-symmetric with respect to both $K_0$ and $K_1$. Then \[ \overline{\xi_{\bar\lambda}}^t = -K_0\xi_\lambda K_0^{-1} = -K_1\xi_\lambda K_1^{-1} \] holds if and only if \begin{equation} [ K_1^{-1} K_0,\,\xi_\lambda ] = 0\,. \end{equation} Using \eqref{eq:K1K0} gives $[ p\,\mathbbm{1} + q\,\eta_\lambda\,,\,\xi_\lambda ] = 0$ and hence $[\eta_\lambda,\,\xi_\lambda ] =0$. Thus $\xi_\lambda = r_\lambda \eta_\lambda$ for some polynomial $r_\lambda$. \end{proof} \subsection{The case $B_0 = B_1$.} If we diagonalize as in \eqref{eq:Kdiagonal}, then $K_0 = V_0\, \mathrm{diag}[\,\mu^0_-,\,\mu^0_+\,] \,V_0^{-1}$ and $K_1^{-1} = V_1\, \mathrm{diag}[\,1/\mu^1_-,\,1/\mu^1_+\,] \,V_1^{-1}$ . If the constant $B$ in the K-matrices $K_0$ and $K_1$ coincide $B_0 = B_1$, then the matrix of eigenvectors $V=V_0 = V_1$ are equal, and \begin{equation} K_1^{-1} K_0 = V \,\mathrm{diag}[\,\mu^0_-/\mu^1_-,\,\mu^0_+/\mu^1_+\,] V^{-1} \,. \end{equation} \section{Two integrable boundary conditions} Suppose in our domain $U$ we have beside the curve $y=0$ a second curve $y=y_1 \neq 0$, and a solution $\omega$ of the sinh-Gordon equation that satisfies two boundary conditions \begin{equation} \label{eq:2-boundary1} \begin{split} &\omega_y = e^\omega A_0 + e^{-\omega} B_0 \quad \mbox{ along } y=0 \,,\\ & \omega_y = e^\omega A_1 + e^{-\omega} B_1 \quad \mbox{ along } y=y_1\,. \end{split} \end{equation} We then have two K-matrices $K_0,\,K_1$ as in \eqref{eq:K0K1} and \begin{equation}\begin{split} K_0 \mathbf{U}_\lambda &= \mathbf{U}_{\lambda^{-1}} K_0 \qquad \mbox{along} \quad y=0 \\ K_1 \mathbf{U}_\lambda &= \mathbf{U}_{\lambda^{-1}} K_1 \qquad \mbox{along} \quad y=y_1\,. \end{split} \end{equation} In the 1-boundary case the $y=0$ curve goes through the base point $z=0$, and Lemma \ref{lem:Fswitch2} assures us $K\,\mathbf{F}_\lambda = \mathbf{F}_{\lambda^{-1}}\,K$ along $y=0$. The $y=y_1$ curve does not contain the base point at which $\mathbf{F}_\lambda (0) = \mathbbm{1}$, and we only get a \emph{dressed} $K$-symmetry. \begin{lemma} \label{lem:Fswitch} $\mathrm{(1)}$ Suppose $\tfrac{\partial}{\partial x}\mathbf{F}_\lambda = \mathbf{F}_\lambda \mathbf{U}_\lambda$. Then $K_1\,\mathbf{U}_\lambda = \mathbf{U}_{\lambda^{-1}} \,K_1$ along $y=y_1$, if and only if there exists a $z$-independent matrix $C_\lambda$ such that \begin{equation} \label{eq:FK1} K_1\,\mathbf{F}_\lambda = C_\lambda \,\mathbf{F}_{\lambda^{-1}}\,K _1 \qquad \mbox{\rm{along}} \quad y=y_1\,. \end{equation} $\mathrm{(2)}$ The map $\lambda \mapsto C_\lambda$ is holomorphic on $\mathbb{C}^\ast$. Further, $\det C_\lambda = 1$ for all $\lambda \in \mathbb{C}^\ast$, and $C_\lambda \in \mathrm{SU}_2$ for all $|\lambda |=1$. \end{lemma} \begin{proof} (1) Suppose $K_1\,\mathbf{U}_\lambda = \mathbf{U}_{\lambda^{-1}} \,K_1$ along $y=y_1$. Consider $G_\lambda = K_1\,F_\lambda\,K_1^{-1}$. Then $G'_\lambda = G_\lambda \mathbf{U}_{\lambda^{-1}}$ and $F'_{\lambda^{-1}} = F_{\lambda^{-1}} \mathbf{U}_{\lambda^{-1}}$. Since $G_\lambda$ and $F_{\lambda^{-1}}$ are both fundamental solutions of the same ordinary differential equation, there exists a $z$-independent, $\lambda$-dependent matrix $C_\lambda$ such that $G_\lambda = C_\lambda F_{\lambda^{-1}}$. Thus $ K_1\,F_\lambda\,K_1^{-1} = C_\lambda F_{\lambda^{-1}}$ and the claim follows. Note that \begin{equation}\label{eq:defC} C_\lambda = K_1\,F_\lambda\,K_1^{-1} F_{\lambda^{-1}}^{-1} = K_1\,F_\lambda\,K_1^{-1} \overline{F_{\bar\lambda}}^{t} \,. \end{equation} Conversely, differentiating \eqref{eq:FK1} gives $K_1\,\mathbf{F}_\lambda \mathbf{U}_\lambda= C_\lambda \,\mathbf{F}_{\lambda^{-1}} \mathbf{U}_{\lambda^{-1}}\,K _1$ and using \eqref{eq:FK1} gives the claim. (2) By definintion $C_\lambda = G_\lambda F_{\lambda^{-1}}^{-1}$, and since $G_\lambda, \,F_\lambda$ are holomorphic on $\mathbb{C}^\ast$, the same is true for $C_\lambda$. Similarly, $C_\lambda$ inherits its other asserted properties. \end{proof} \begin{corollary} Suppose $\mathbf{F}_\lambda$ satisfies \eqref{eq:FK1} and $\zeta_\lambda = \mathbf{F}_\lambda^{-1} \xi_\lambda \mathbf{F}_\lambda$ is a polynomial Killing field. Then along $y=y_1$ we have \begin{equation} \label{eq:K1pKf} K_1\,\zeta_\lambda + \overline{\zeta_{\bar{\lambda}}}^t K_1 = \mathbf{F}_{\lambda^{-1}}^{-1} \bigl( \, C_\lambda^{-1} K_1 \,\xi_\lambda + \overline{\xi_{\bar{\lambda}}}^t C_\lambda^{-1} K_1 \,\bigr) \,\mathbf{F}_\lambda\,. \end{equation} \end{corollary} Hence $\zeta_\lambda$ is $K_1$-symmetric along $y=y_1$ if and only if its potential $\xi_\lambda$ satisfies the dressed $K_1$-symmetry \begin{equation}\label{eq:dressedK-symmetry} C_\lambda^{-1} K_1 \xi_\lambda + \overline{\xi_{\bar\lambda}}^t C_\lambda^{-1} K_1 = 0\,. \end{equation} If in addition the potential is $K_0$-symmetric, then we can replace $\overline{\xi_{\bar{\lambda}}}^t = - K_0 \xi_\lambda K_0^{-1}$ in \eqref{eq:K1pKf} and obtain along $y=y_1$ that \begin{equation} \label{eq:K1pKf2} K_1\,\zeta_\lambda + \overline{\zeta_{\bar{\lambda}}}^t K_1 = \mathbf{F}_{\lambda^{-1}}^{-1} \, K_0 \,\, \bigl[ \, K_0^{-1} C_\lambda^{-1} K_1 ,\,\xi_\lambda \,\bigr] \,\mathbf{F}_\lambda\,. \end{equation} Then $\zeta_\lambda$ is $K_1$-symmetric along $y=y_1$ if and only if \begin{equation} \label{eq:M} [M_\lambda,\,\xi_\lambda ] = 0 \quad \mbox{ for } \quad M_\lambda = K_1^{-1} C_\lambda K_0 \,. \end{equation} \begin{corollary} Suppose $\mathbf{F}_\lambda$ satisfies \eqref{eq:FK1} and $\zeta_\lambda = \mathbf{F}_\lambda^{-1} \xi_\lambda \mathbf{F}_\lambda$ is a polynomial Killing field with $K_0$-symmetric potential. Then $\zeta_\lambda$ is $K_1$-symmetric along $y=y_1$ if and only if \eqref{eq:M} holds. \end{corollary} The proof of the next result is analogous to the proof of Lemma~\ref{th:KPhi}. \begin{lemma} Let $\xi_\lambda$ satisfy the dressed $K_1$-symmetry \eqref{eq:dressedK-symmetry}. Then \begin{equation} \label{eq:dressedPhiK-symmetry} K_1\,\mathbf{\Phi}_\lambda (z) = C_\lambda \,\overline{\mathbf{\Phi}_{\bar\lambda} (\bar z)} ^{t^{-1}} C_\lambda^{-1} K_1 \qquad \mbox{for all $z \in U$}\,. \end{equation} \end{lemma} Finally, we compute the dressed $K_1$-summetry for $\mathbf{B}_\lambda$ from \eqref{eq:FK1} and \eqref{eq:dressedPhiK-symmetry}. \begin{corollary} Let $\xi_\lambda$ satisfy the dressed $K_1$-symmetry \eqref{eq:dressedK-symmetry} and the corresponding $\mathbf{F}_\lambda$ satisfy the $K_1$-symmetry \eqref{eq:FK1}along $y=y_1$. Then the dressed $K_1$-symmetry of $\mathbf{B}_\lambda$ is \begin{equation} \label{eq:dressedBK-symmetry} K_1 \mathbf{B}_\lambda = \overline{\mathbf{B}}_{\bar\lambda}^{t^{-1}} C_\lambda^{-1} K_1 \qquad \mbox{ along \quad $y=y_1$}\,. \end{equation} \end{corollary} In summary, in the 2-boundary case we have a polynomial Killing field $\zeta_\lambda = \mathbf{F}_\lambda^{-1} \xi_\lambda \mathbf{F}_\lambda$ where $\mathbf{F}_\lambda$ satisfies \eqref{eq:KF} and \eqref{eq:FK1}, and \begin{equation} \label{eq:2-boundary} \begin{split} K_0\,\zeta_\lambda + \overline{\zeta_{\bar{\lambda}}}^t K_0 &= 0 \quad \mbox{ along } y=0 \quad \Longleftrightarrow \quad K_0\,\xi_\lambda + \overline{\xi_{\bar{\lambda}}}^t K_0 = 0 \,,\\ K_1\,\zeta_\lambda + \overline{\zeta_{\bar{\lambda}}}^t K_1 &= 0 \quad \mbox{ along } y=y_1 \quad \Longleftrightarrow \quad C_\lambda^{-1} K_1 \xi_\lambda + \overline{\xi_{\bar\lambda}}^t C_\lambda^{-1} K_1 = 0\,. \end{split} \end{equation} \section{The commutator} \begin{proposition} The map $\lambda \mapsto M_\lambda$ in \eqref{eq:M} is meromorphic on $\mathbb{C}^\ast$ with exactly four simple poles at the four simple roots of $\det K_1$. Also $\det M_\lambda = \det K_0 /\det K_1$. \end{proposition} \begin{proof} The four simple poles are the four simple roots of $\det K_1$. The last assertion follows from $\det C_\lambda = 1$ for all $\lambda \in \mathbb{C}^\ast$. \end{proof} Since $[M_\lambda,\,\xi_\lambda] = 0$, there exist $\lambda$-dependent functions $f,\,g$ such that \begin{equation}\label{eq:f-g} M_\lambda = f\,\mathbbm{1} + g\, \xi_\lambda\,. \end{equation} Taking determinants gives \begin{equation*} \frac{\det K_0}{\det K_1} = f^2 + g^2 \det \xi_\lambda\,. \end{equation*} In terms of the eigenvalues, $\det K_0 = \mu^0_- \mu^0_+,\,\det K_1 = \mu^1_- \mu^1_+$, where $\mu_\pm^0,\,\mu_\pm^1$ are as in \eqref{eq:mu_pm} with $A,\,B$ replaced by $A_0,\,B_0$ respectively $A_1,\,B_1$, and writing $\nu^2 = -\det \xi_\lambda$ we obtain \begin{equation*} f^2 - g^2 \nu^2 = (f-g\,\nu)(f + g\,\nu) = \mu^0_- \mu^0_+/(\mu^1_- \mu^1_+) \end{equation*} Setting $f-g\,\nu = \mu^0_-/\mu^1_-$ and $f +g\,\nu = \mu^0_+/ \mu^1_+$ gives \begin{equation*} f = \frac{1}{2} \left( \frac{\mu^0_-}{ \mu^1_-} + \frac{\mu^0_+}{ \mu^1_+} \right) \qquad \mbox{ and } \qquad g = \frac{1}{2\,\nu} \left( \frac{\mu^0_+}{ \mu^1_+} - \frac{\mu^0_-}{ \mu^1_-} \right) \,. \end{equation*} By \eqref{eq:mu-symmetry}, and since $\nu(\lambda^{-1}) = \lambda^{1-d} \nu ( \lambda) $ we have \begin{equation*} f(\lambda^{-1}) = f(\lambda) \quad \mbox{ and } \quad g(\lambda^{-1}) = - \lambda^{d-1} g(\lambda) \,. \end{equation*} \subsection{Kernels.} We briefly verify that $f\mathbbm{1} + g\,\xi_\lambda$ has the given kernels at the four roots of $\det K_0$. Suppose the four roots of $\det K_0$ are $\{\varrho_0,\,r_0,\,\varrho_0^{-1},\,r_0^{-1} \}$ as in \eqref{eq:kernels} with $A,\,B$ replaced by $A_0,\,B_0$. Suppose further that $\mu^0_- (\varrho_0) = \mu^0_- (r_0) = 0$, and that $\langle \mathbf{v}_0 \rangle = \ker K_0(\varrho_0) = \ker K_0 (r_0)$. Then $M_{\varrho_0} \mathbf{v}_0 = M_{r_0} \mathbf{v}_0 = 0$ and as we may assume that $\xi_{\varrho_0} \mathbf{v}_0 = \xi_{r_0} \mathbf{v}_0 =-\nu\,\mathbf{v}_0$ we obtain \begin{equation*} \begin{split} \left( f(\varrho_0)\,\mathbbm{1} + g(\varrho_0) \,\xi_{\varrho_0} \right) \mathbf{v}_0 &= \frac{1}{2} \frac{\mu^0_+(\varrho_0)}{ \mu^1_+(\varrho_0)} \left( \mathbbm{1} + \frac{1}{\nu}\xi_{\varrho_0} \right) \mathbf{v}_0 = 0\,, \\ \left( f(r_0)\,\mathbbm{1} + g(r_0) \,\xi_{r_0} \right) \mathbf{v}_0 &= \frac{1}{2} \frac{\mu^0_+(r_0)}{ \mu^1_+(r_0)} \left( \mathbbm{1} + \frac{1}{\nu}\xi_{r_0} \right) \mathbf{v}_0 = 0\,. \end{split} \end{equation*} Similarly, if $\mu^0_+ (\varrho_0^{-1}) = \mu^0_+ (r_0^{-1}) = 0$, and that $\langle \mathbf{v}_0^\perp \rangle = \ker K_0(\varrho_0^{-1}) = \ker K_0 (r_0^{-1})$, then $M_{\varrho_0^{-1}} \mathbf{v}_0^\perp = M_{r_0^{-1}} \mathbf{v}_0^\perp = 0$ and as we may assume that $\xi_{\varrho_0^{-1}} \mathbf{v}_0^\perp = \xi_{r_0^{-1}} \mathbf{v}_0^\perp =\nu\,\mathbf{v}_0^\perp$, we obtain \begin{equation*} \begin{split} \left( f(\varrho_0^{-1})\,\mathbbm{1} + g(\varrho_0^{-1}) \,\xi_{\varrho_0^{-1}} \right) \mathbf{v}_0^\perp &= \frac{1}{2} \frac{\mu^0_-(\varrho_0^{-1})}{ \mu^1_-(\varrho_0^{-1})} \left( \mathbbm{1} - \frac{1}{\nu}\xi_{\varrho_0^{-1}} \right) \mathbf{v}_0^\perp = 0\,, \\ \left( f(r_0^{-1})\,\mathbbm{1} + g(r_0^{-1}) \,\xi_{r_0^{-1}} \right) \mathbf{v}_0^\perp &= \frac{1}{2} \frac{\mu^0_-(r_0^{-1})}{ \mu^1_-(r_0^{-1})} \left( \mathbbm{1} - \frac{1}{\nu}\xi_{r_0^{-1}} \right) \mathbf{v}_0^\perp = 0\,. \end{split} \end{equation*} \subsection{Spectrum.} The eigenvalues of $f\mathbbm{1} + g\,\xi_\lambda$ are $f \pm g\nu$. From above we have $f-g\,\nu = \mu^0_-/ \mu^1_-$ and $f+ g\,\nu = \mu^0_+/\mu^1_+$ so both eigenvalues of $M_\lambda$ are rational with two simple roots, and two simple poles, and principal divisors \begin{equation*} (f - g\,\nu ) = \varrho_0 + r_0 - \varrho_1 - r_1 \quad \mbox { and } \quad (f + g\,\nu) = \varrho_0^{-1} + r_0^{-1} - \varrho_1^{-1} - r_1^{-1}\,. \end{equation*} Since $[ M_\lambda,\,\xi_\lambda ] =0$, these two matrices have the same eigenvectors, and thus \begin{equation*} \xi_\lambda \mathbf{v} = \pm \nu \,\mathbf{v} \Longleftrightarrow M_\lambda \mathbf{v} = (f \pm g\,\nu)\, \mathbf{v} \,. \end{equation*} \subsection{Complementary boundary conditions} Of particular interest is the case when the two integrable boundary conditions are \emph{complimentary}, that is when the constants in the two K-matrices $K_0,\,K_1$ have opposite signs so that \begin{equation*} A_1=-A_0 \quad \mbox{ and } B_1 = -B_0. \end{equation*} Omitting subscripts, and reverting back to constants $A,\,B$ we thus consider \begin{equation*} K_0 = \begin{pmatrix} 4 A -4 B \lambda & \lambda - \lambda^{-1} \\ \lambda - \lambda^{-1} & 4 A - 4 B \lambda^{-1} \end{pmatrix} \, \mbox{ and } \, K_1 = \begin{pmatrix} -4 A +4 B \lambda & \lambda - \lambda^{-1} \\ \lambda - \lambda^{-1} & -4 A + 4 B \lambda^{-1} \end{pmatrix}\,. \end{equation*} From \eqref{eq:mu_pm} we know that $K_0$ has eigenvalues \begin{equation*} \begin{split} \mu_- ^0(\lambda) &= 4 A - 2 B \left(\lambda + \lambda^{-1} \right) - \sqrt{4 B^2+1} \left(\lambda - \lambda^{-1} \right) \\ \mu_+^0 (\lambda) &= 4 A - 2 B \left(\lambda + \lambda^{-1} \right) + \sqrt{4 B^2+1} \left(\lambda - \lambda^{-1} \right) \end{split} \end{equation*} so under $(A,\,B) \mapsto (-A,\,-B)$ we see that $K_1$ has eigenvalues \begin{equation} \label{eq:K1EV} \begin{split} \mu_- ^1 &= -\mu_+^0\\ \mu_+^1 &= -\mu_-^0 \end{split} \end{equation} Thus $\det K_0 = \det K_1$, and hence $\det M_\lambda = f^2 - g^2\nu^2 = (f-g\,\nu)(f+g\,\nu) = 1$. As in the previous section, and using \eqref{eq:K1EV} we have \begin{equation*} f-g\,\nu = \frac{\mu^0_-}{ \mu^1_-} = - \frac{\mu^0_-}{ \mu^0_+} \quad \mbox{ and } \quad f+g\,\nu = \frac{\mu^0_+}{ \mu^1_+} = - \frac{\mu^0_+}{ \mu^0_-} \end{equation*} and so indeed $(f-g\,\nu)(f+g\,\nu) = 1$. Further, solving for $f,\,g$ gives \begin{equation*} f = -\frac{1}{2} \left( \frac{\mu^0_-}{ \mu^0_+} + \frac{\mu^0_+}{ \mu^0_-} \right) \quad \mbox{ and } \quad g = \frac{1}{2\,\nu} \left( \frac{\mu^0_-}{ \mu^0_+} - \frac{\mu^0_+}{ \mu^0_-} \right) \,. \end{equation*} \subsection{The dressing matrix} From \eqref{eq:M} and \eqref{eq:f-g} we have $M_\lambda = K_1^{-1} C_\lambda K_0 = f\,\mathbbm{1} + g \,\xi_\lambda$ and using the $K_0$-symmetry of $\xi_\lambda$ proves the next \begin{proposition} The dressing matrix $C_\lambda$ is given by \begin{equation} C_\lambda = K_1 K_0^{-1} \bigl( f\,\mathbbm{1} - g\,\, \overline{\xi_{\bar\lambda}}^t \bigr) \,. \end{equation} \end{proposition} \bibliographystyle{amsplain} \bibliography{ref} \end{document}
2412.05067v1
http://arxiv.org/abs/2412.05067v1
Exotic newforms constructed from a linear combination of eta quotients
\documentclass[ prd, amssymb, preprintnumbers, secnumarabic, nofootinbib, superscriptaddress ]{revtex4-2} \usepackage{hyperref} \hypersetup{ colorlinks=true, linkcolor=blue, filecolor=magenta, urlcolor=cyan, pdftitle={Overleaf Example}, pdfpagemode=FullScreen, } \usepackage{placeins} \usepackage{amssymb} \usepackage{float} \usepackage{microtype} \usepackage{amsmath} \usepackage{amsthm} \usepackage[english]{babel} \usepackage{amsthm} \usepackage{csquotes} \theoremstyle{definition} \newtheorem{definition}{Definition}[section] \newtheorem{theorem}{Theorem}[section] \newtheorem{lemma}[theorem]{Lemma} \usepackage{geometry} \usepackage{blindtext} \parindent=0pt \usepackage{graphicx} \geometry{left = 1.5cm, right = 1.5cm, top = 3cm, bottom = 3cm} \usepackage{xcolor} \begin{document} \title{Exotic newforms constructed from a linear combination of eta quotients} \author{Anmol Kumar} \email{[email protected]} \affiliation{Indian Institute of Science, Bengaluru, India} \date{\today} \begin{abstract} \sloppy \raggedright K{\"o}hler, in [1], presented a weight 1 newform on $\Gamma_0(576)$ constructed from a linear combination of weight 1 eta quotients and asked, ``What would be a suitable $L$ and representation $\rho$ such that Deligne\text{-}Serre correspondence holds?" In this paper, we find the Galois field extension $L$ and representation $\rho$ such that the Deligne\text{-}Serre correspondence holds for this newform, and also study the splitting of primes in $L$ using the coefficients $a(p)$ of the newform. We also discuss an exotic newform on $\Gamma_0(1080)$ constructed from a linear combination of weight 1 eta quotients, find the corresponding Galois extension and representation, and study the splitting of primes in this extension. Furthermore, we find all such newforms that can be constructed from a linear combination of weight 1 eta quotients listed in [2] with $q$-expansion of the form $q+\sum_{k=2}^{\infty}a(k)q^k$.\\ \end{abstract} \maketitle \section{Introduction} \begin{definition} Let $\chi$ be a $\textit{dirichlet character modulo N}$ with $\chi(-1)=(-1)^k$. Let $$\Gamma_0(N):=\left\{\begin{pmatrix}a & b\\ c & d\end{pmatrix}\in \text{SL}_2(\mathbb{Z}):N\mid c \right\}$$ A \textit{modular form of} \textit{type} $(k,\chi)$ on $\Gamma_0(N)$ is a function $f:\mathbb{H}\rightarrow\mathbb{C}$ such that \begin{itemize} \item $f$ is holomorphic on $\mathbb{H}$ \item $f$ is holomorphic at all cusps of $\Gamma_0(N)$ \item $f\left(\frac{az+b}{cz+d}\right)=\chi(d)(cz+d)^kf(z)$ \end{itemize} The vector space formed by modular forms of type $(k,\chi)$ on $\Gamma_0(N)$ is denoted by $M_k(\Gamma_0(N),\chi)$. \end{definition} \begin{definition}[Dedekind eta function] The \textit{Dedekind eta function} is defined by $$\eta(z)=q^\frac{1}{24}\prod_{n=1}^{\infty} (1-q^n)$$ where $q=e^{2i\pi z}$, and $z$ lies in the upper half complex plane $\mathbb{H}$. \end{definition} \begin{definition} Define an \textit{eta-quotient} on level $N$ by the product $$ \prod_{0<m\mid N} \eta(mz)^{a_m}$$ where $a_m \in \mathbb{Z}$ $\forall$ $m\mid N$. \end{definition} \raggedright The function $\eta(z)$ is a modular form of weight $\frac{1}{2}$ with a multiplier system on SL$_2(\mathbb{Z})$. In general, an eta-quotient $f(z)=\prod_{0<m\mid N} \eta(mz)^{a_m}$ is a meromorphic modular form of weight $k=\frac{1}{2}\sum_{m\mid N} a_m$ with a multiplier system on the congruence subgroup $\Gamma_0(N)$ for some $N$. Yves Martin, in \cite{YM}, found all holomorphic eta-quotients $f$ of integral weight such that both $f(z)$ and its image under $\textit{Frickie involution}$ are eigenforms of all \textit{Hecke operators.} Bhattacharya, in \cite{Bha2}, gave a list of simple-holomorphic irreducible eta-quotients of weight 1 and also gave a simplified proof of Zagier's conjecture/Mersmann's theorem, which states that of any particular weight, there are only finitely many holomorphic eta quotients, none of which is an integral rescaling of another eta quotient or a product of two holomorphic eta quotients other than 1 and itself. Kohler in \cite{GK} constructed a linear combination of weight 1 eta quotients that was an exotic newform and asked the question, ``What would be a suitable $L$ and representation $\rho$ such that Deligne–Serre correspondence holds?'' In this paper, we determine $L$ and $\rho$ for this newform and study the splitting of primes in $L$ using the Fourier coefficients. We use the eta quotients of weight 1 listed in \cite{Bha1} to find a list of linear combinations of eta quotients of weight 1 that are newforms with q-expansion of the form $q+\sum_{k=2}^{\infty}a(k)q^k$, and also find out the projective image of the associated Galois representation. \\ \section{Prelimnaries} We will need the following definitions and theorems. \begin{theorem}\cite{Ono} If $f(z)=\prod_{0<m\mid N} \eta(mz)^{a_m}$ is an eta-quotient with $k=\frac{1}{2}\sum_{m\mid N} a_m \in \mathbb{Z},$ with the additional properties that $$\sum_{m\mid N}ma_m\equiv 0 \pmod {24}$$ and $$\sum_{m\mid N}\frac{N}{m}a_m\equiv 0 \pmod {24}$$, then $f(z)$ satisfies $$f\left(\frac{az+b}{cz+d}\right)=\chi(d)(cz+d)^kf(z)$$ for every $\begin{pmatrix}a & b\\ c & d\end{pmatrix}\in \Gamma_0(N).$ Here the character $\chi$ is a Dirichlet character modulo $N$ given by $$\chi(d)=\left(\frac{(-1)^ks}{d}\right)$$ where $s:=\displaystyle \prod_{d|N}d^{a_d}$. Moreover, if $f(z)$ is holomorphic at all cusps of $\Gamma_0(N)$, then $f(z)\in M_k(\Gamma_0(N),\chi)$. \end{theorem} \begin{definition} Let $\mathcal{L/K}$ be a Galois extension of number fields and denote the ring of integers of $\mathcal{K}$ and $\mathcal{L}$ by $\mathcal{O_K}$ and $\mathcal{O_L}$ respectively. Let $\mathfrak{p}$ be a prime in $\mathcal{O_K}$ and $\mathfrak{q}$ be a prime in $\mathcal{O_L}$ lying above $\mathfrak{p}$. Let $G=\operatorname{Gal}(\mathcal{L/K})$. Then, the $\textit{decomposition group}$ $D_{\mathfrak{q}}$ is defined as the stabilizer of $\mathfrak{q}$ in $G$. \end{definition} The ramification index $e_\mathfrak{q}$ and residue field degree $f_\mathfrak{q}$ of $\mathfrak{q}$ are given by $e_\mathfrak{q}=\nu_\mathfrak{q}(\mathfrak{p}\mathcal{O_L})$ and $[\mathcal{O_L}/\mathfrak{q}:\mathcal{O_K/\mathfrak{p}}]$. Recall that $e_\mathfrak{q}$ and $f_\mathfrak{q}$ are independent of the choice of $\mathfrak{q}|\mathfrak{p}$. Hence, we can write $e_\mathfrak{p}$ and $f_\mathfrak{p}$ for $e_\mathfrak{q}$ and $f_\mathfrak{q}$ respectively for the rest of the paper.\\ \begin{lemma}\cite{Mar} The decomposition groups $D_\mathfrak{q}$ are all conjugates in $G$ for $\mathfrak{q}|\mathfrak{p}$ and $e_\mathfrak{p}f_\mathfrak{p}=|D_\mathfrak{q}|$. If $\mathfrak{p}$ is unramified in $\mathcal{K},$ then $e_\mathfrak{p}=1$, hence $f_p=|D_\mathfrak{q}|.$\\ \end{lemma} Now, we want to determine the splitting behavior of an unramified prime $\mathfrak{p}$ of $\mathcal{O_K}$ in $\mathcal{O_L}$. Note that, by the $\textit{primitive element theorem,}$ there exists $\alpha$ in $\mathcal{O_L}$ such that $\mathcal{L}$ is generated over $\mathcal{K}$ by $\alpha$. Let the minimal polynomial of $\alpha$ over $\mathcal{O_K}$ be $h(x)$. We know that $\mathfrak{p}$ factorises in $\mathcal{O_L}$ as $$\mathfrak{p}\mathcal{O_L}=\prod_{i=1}^r\mathfrak{q}_i$$ where $r=|G|/f_\mathfrak{p}$ and $\mathfrak{q}_i$ are distinct prime ideals in $\mathcal{O_L}$ lying above $\mathfrak{p}$. Hence, we have $$h(x)\equiv\prod_{i=1}^rh_i(x)\text{ mod }\mathfrak{p}$$ where $h_i's$ are irreducible polynomials of degree $f_\mathfrak{p}$. Since $\mathfrak{p}$ is unramified, the map $$D_\mathfrak{q}\rightarrow \operatorname{Gal}\big(\mathcal{O_L/\mathfrak{q}}\big{/}{\mathcal{O_K/\mathfrak{p}}}\big)$$ is an isomorphism of groups. The group $\operatorname{Gal}\big(\mathcal{O_L/\mathfrak{q}}\big{/}{\mathcal{O_K/\mathfrak{p}}}\big)$ is cyclic of order $f_\mathfrak{p}$. Therefore, $D_\mathfrak{q}$ is a cyclic group of order $f_\mathfrak{p}$ and is generated by the $\textit{Frobenius automorphism}$ $\phi_\mathfrak{q}$ defined as $$\phi_\mathfrak{q}:O_L/\mathfrak{q}\rightarrow O_L/\mathfrak{q}$$ $$x\mapsto x^{\mathcal{N(\mathfrak{p})}}$$ where $\mathcal{N(\mathfrak{p})}=|\mathcal{O_K}/\mathfrak{p}|$ is the norm of $\mathfrak{p}$. We define the $\textit{Frobenius element}$ $\sigma_\mathfrak{q}$ for an unramified prime $\mathfrak{q}|\mathfrak{p}$ to be the unique element in $G$ such that for all $x$ in $\mathcal{O_L}$, $\sigma_{\mathfrak{q}}(x)=x^{\mathcal{N}(\mathfrak{p})} \pmod{\mathfrak{q}}$. Note that if $\mathfrak{q}|\mathfrak{p}$ is unramified, then for all $\mathfrak{q'}|\mathfrak{p}$, $\sigma_{\mathfrak{q}}$ and $\sigma_{\mathfrak{q'}}$ are conjugates in $G$. Therefore, for $\mathfrak{q}|\mathfrak{p}$ unramified, we define the $\textit{Frobenius Class}$ of $\mathfrak{p}$ to be the conjugacy class of $\sigma_\mathfrak{q}$ in $G$ and denote any representative of the class by $\textrm{Frob}_\mathfrak{p}$. \begin{definition} Let $\mathcal{L/}\mathbb{Q}$ be a Galois extension with Galois group $G=\operatorname{Gal}(\mathcal{L/}\mathbb{Q})$ and a two-dimensional irreducible representation $\rho$. The $\textit{Artin L-function of $\rho$}$, denoted by $L(s,\rho)$, is defined as $$L(s,\rho)=\prod_pL_p(s,\rho)$$ where the local factors are given by \begin{align*} L_p(s,\rho)= \textrm{det}(I-\rho(\textrm{Frob}_p)p^{-s})^{-1} &=((1-\lambda_{1,p}p^{-s})(1-\lambda_{2,p}p^{-s}))^{-1} \\ &=(1-\textrm{Tr}(\rho(\textrm{Frob}_p))p^{-s}+ \textrm{det}(\rho(\textrm{Frob}_p))p^{-2s})^{-1}, \end{align*} where $\lambda_{1,p}$ and $\lambda_{2,p}$ are eigenvalues of $\textrm{Frob}_p$. Note that for ramified primes $p$, $\textrm{det}(\rho(\textrm{Frob}_p))=0$. \\ \end{definition} \begin{theorem}[Chebotarev Density Theorem]\cite{Lag} Let \( C \) be a conjugacy class of the Galois group \( G \) of a Galois extension \( K/\mathbb{Q} \). Then the density of the prime ideals \(\mathfrak{p} \in \mathcal{P}\) that are unramified in \( K \) and satisfy \(\mathrm{Frob}_{\mathfrak{p}} \in C\) is given by \[ \delta\left(\{\mathfrak{p} \in \mathcal{P} \mid \mathfrak{p} \text{ is unramified and } \mathrm{Frob}_{\mathfrak{p}} \in C\}\right) = \frac{|C|}{|G|}. \] \end{theorem} \begin{theorem}[Deligne-Serre] Let $f(z)=\sum_{n=1}^{\infty}a(n)q^n$ with $a(1)=1$ be a Hecke newform of weight 1 on $\Gamma_0(N)$ with character $\chi$ modulo $N$ satisfying $\chi(-1)=-1$. Assume that $f$ is an eigenform for all $T_p$ with $p\nmid N$, with eigenvalue $a_p$. Then there exists a Galois extension $L$ of $\mathbb{Q}$ and a 2-dimensional irreducible representation $\rho:\operatorname{Gal}{(L/\mathbb{Q})}\rightarrow GL_2(\mathbb{C})$ such that $\textrm{Tr}(\rho_f(\textrm{Frob}_p))=a_p$ and $\textrm{det}(\rho_f(\textrm{Frob}_p))=\chi(p)$ for all $p\nmid N$. Equivalently, there exists a Galois extension $L$ of $\mathbb{Q}$ and a 2-dimensional irreducible representation $\rho$ of G such that $L(s,\rho)=L_f(s)=\sum_{n=1}a(n)q^n$. \end{theorem} \begin{theorem}[Weil-Langlands-Khare-Wintenberger] Given $\rho: G_\mathbb{Q}\rightarrow GL_2(\mathbb{C})$ be an irreducible, continuous, odd representation with Artin conductor $N$ and determinant $\chi$, let $L(s,\rho)=\sum_{n=1}\frac{a(n)}{n^s}$ be its Artin $L$-function. Then $f=\sum_{n=0}a(n)q^n$ is a normalized newform lying in $S_1(N,\chi)$. \end{theorem} \begin{definition}[Sturm Bound] For any space $M_k(\Gamma_0{(N)},\chi)$ of modular forms of weight $k$, level $N$, and character $\chi$, the Sturm bound is the integer $$ B(M_k(\Gamma_0(N),\chi)) := \left\lfloor \frac{km}{12}\right\rfloor,$$ where $$ m:=[SL_2(\mathbb{Z}):\Gamma_0(N)]=N\prod_{p|N}\left(1+\frac{1}{p}\right). $$ \end{definition} \begin{theorem}\cite{LMFDB} If $f=\sum_{n\ge 0}a_n q^n$ and $g=\sum_{n\ge 0}b_n q^n$ are elements of $M_k(N,\chi)$ with $a_n=b_n$ for all $n\le B(M_k(N,\chi))$ then $f=g$. \end{theorem} \begin{theorem}\cite{Lag} Let $f(z)=\sum_{n=1}^{\infty}a(n)q^n$ with $a(1)=1$ be a Hecke newform of weight 1 on $\Gamma_0(N)$ with character $\chi$ modulo $N$ satisfying $\chi(-1)=-1.$ Assume that $f$ is an eigenform for all $T_p$ with $p\nmid N$, with eigenvalue $a_p$. Let $L$ be the Galois number field such that the Deligne-Serre correspondence holds. Let $c(\rho(\textrm{Frob}_p)):=a_p^2/\chi(p)$ Then, the projective image of $G=\operatorname{Gal}(L/\mathbb{Q})$ is \begin{itemize} \item $A_4$ if the proportion of primes $p$ with $c(\rho(\textrm{Frob}_p))$ equal to 1, 2, and 3 converges to $\frac{1}{12}$, $\frac{1}{4}$, and $\frac{2}{3}$ respectively. \item $S_4$ if the proportion of primes $p$ with $c(\rho(\textrm{Frob}_p))$ equal to 1, 2, 3 and 4 converges to $\frac{1}{25}$, $\frac{3}{8}$, $\frac{1}{3}$, and $\frac{1}{4}$ respectively. \item $A_5$ if the proportion of primes $p$ with $c(\rho(\textrm{Frob}_p))$ equal to 1, 2, 3 and 5 converges to $\frac{1}{60}$, $\frac{1}{4}$, $\frac{1}{3}$, and $\frac{2}{5}$ respectively. \end{itemize} \end{theorem} \section{Constructing newforms and the corresponding Number Fields} Let $f(z)=\sum_{n=1}^\infty a(n)q^n$ with $a(1)=1$ be an eta quotient in $M_1(\Gamma_0(N),\chi)$, where $\chi(-1)=-1$. We compute the action of Hecke operator $T_{p_l}$ on $f$ for primes $p_l$ less than and coprime to $N$ and take a complex linear combination $\sum_{l}c_lT_{p_l}(f)$ of all linearly independent $T_{p_l}(f)$ which are eta quotients. The coefficients $c_l$ are chosen such that $\sum_{l}c_lT_{p_l}(f)$ is an eigenform for all $T_{p_l}$, where $p_l$ ranges through primes less than and coprime to N. This gives us a weight 1 modular form $F(z)=\sum_{n=1}^\infty b(n)q^n$ with character $\chi$ on $\Gamma_0(N)$. We calculate the approximate proportion of primes $p$ such that $b(p)=2$ by considering primes up to 10000. By $\textbf{Theorem 2.3}$, the reciprocal of this proportion gives us the order of group $G$. Next, we search for 2-dimensional complex irreducible representations of groups with this order, for which the fraction of elements with trace equal to $x$ is the same as the limiting proportion of primes $p$ with $b(p)=x$. Now, the primes $p$ such that $b(p)=2$ split completely in $L$; hence they split completely in all subfields of $L$. Using the \cite{LMFDB} website, we determine all subfields of $L$ and, hence, the composite field $L$. Using the fundamental theorem of Galois theory, we determine the subgroup structure of $G$, and hence the group $G$. We calculate the Frobenius element of prime $p$ up to the Sturm bound using \textit{sagemath}. Now, we find the Artin-$L$ function $L(s,\rho)$ for the representation $\rho$ and the $L$-function $L_F(s)=\sum_{n=1}\frac{b(n)}{n^s}$ up to the Sturm bound $B(M_k(\Gamma_0(N),\chi))$. $F$ is a newform if $L_F(s)=L(s,\rho)$, in which case the number field extension and representation such that the Deligne-Serre correspondence holds are $L$ and $\rho$, respectively. \section{Level 576} In this section, we discuss a newform on $\Gamma_0(576)$ given in \cite{GK}. Consider the following $\eta$ quotients: \begin{align*} f_{1}^{576}(z) &= \frac{\eta(4z)\eta(6z)^2\eta(12z)}{\eta(3z)\eta(24z)}, & f_{5}^{576}(z) &= \frac{\eta(2z)\eta(6z)\eta(12z)^2}{\eta(3z)\eta(24z)}, \\ f_{7}^{576}(z) &= \frac{\eta(2z)\eta(4z)^2\eta(6z)}{\eta(z)\eta(8z)}, & f_{11}^{576}(z) &= \frac{\eta(2z)^2\eta(4z)\eta(12z)}{\eta(z)\eta(8z)}, \\ f_{13}^{576}(z) &= \frac{\eta(z)\eta(8z)\eta(6z)}{\eta(2z)}, & f_{17}^{576}(z) &= \frac{\eta(z)\eta(8z)\eta(12z)}{\eta(4z)}, \\ f_{19}^{576}(z) &= \frac{\eta(4z)\eta(3z)\eta(24z)}{\eta(12z)}, & f_{23}^{576}(z) &= \frac{\eta(2z)\eta(3z)\eta(24z)}{\eta(6z)}. \end{align*} The function $$F^{576}(24z)=f_{1}^{576}(24z)+f_{5}^{576}(24z)+f_{7}^{576}(24z)+f_{11}^{576}(24z)+i\sqrt{2}f_{13}^{576}(24z)-i\sqrt{2}f_{17}^{576}(24z)-i\sqrt{2}f_{19}^{576}(24z)+i\sqrt{2}f_{23}^{576}(24z)$$ defines a modular form of weight 1. The character is given by $\chi(d)=\Big(\frac{-24}{d}\Big)$. Consider the field extensions $K_1=\mathbb{Q}(\alpha)$, $K_2=\mathbb{Q}(\beta)$ and $K_3=\mathbb{Q}(\delta)$ of $\mathbb{Q}$ given by the defining polynomials $x^2 + 6$, $x^3 + 3x - 2$, and $x^8 + 12x^6 + 42x^4 + 88x^2 - 6$ respectively. Let $L$ denote the composite field of the number fields $K_1$, $K_2$ and $K_3$. The Galois group $G=\operatorname{Gal}(L/\mathbb{Q})$ is $GL_2(\mathbb{F}_3)$. Now, consider the two-dimensional irreducible representation $\rho$ of $G$ $$\rho:G\rightarrow SL_2(\mathbb{C})$$ \FloatBarrier \[ \begin{aligned} &\rho\left(\begin{pmatrix} 1 & 0 \\ 0 & -1 \end{pmatrix}\right) = \begin{pmatrix} 1 & 0 \\ 0 & -1 \end{pmatrix}, &&\rho\left(\begin{pmatrix} 1 & 1 \\ 0 & 1 \end{pmatrix}\right) = \begin{pmatrix} \omega & 0 \\ 0 & \omega^2 \end{pmatrix}, \\ &\rho\left(\begin{pmatrix} 0 & 1 \\ -1 & 0 \end{pmatrix}\right) = \begin{pmatrix} i & 0 \\ 0 & -i \end{pmatrix}, &&\rho\left(\begin{pmatrix} -1 & 1 \\ 0 & -1 \end{pmatrix}\right) = \begin{pmatrix} -\omega & 0 \\ 0 & -\omega^2 \end{pmatrix}, \\ &\rho\left(\begin{pmatrix} 0 & 1 \\ 1 & -1 \end{pmatrix}\right) = \begin{pmatrix} e^{\frac{i\pi}{4}} & 0 \\ 0 & -e^{\frac{-i\pi}{4}} \end{pmatrix}, &&\rho\left(\begin{pmatrix} 0 & 1 \\ 1 & 1 \end{pmatrix}\right) = \begin{pmatrix} e^{\frac{-i\pi}{4}} & 0 \\ 0 & -e^{\frac{i\pi}{4}} \end{pmatrix}. \end{aligned} \] \begin{table}[h] $$ \begin{array}{c|rrrrrrrr} \rm \text{Class rep.}&\rm\big(\begin{smallmatrix} 1 & 0\\ 0 & 1 \end{smallmatrix}\big)&\rm\big(\begin{smallmatrix} -1 & 0\\ 0 & -1 \end{smallmatrix}\big)&\rm\big(\begin{smallmatrix} 1 & 0\\ 0 & -1 \end{smallmatrix}\big)&\rm\big(\begin{smallmatrix} 1 & 1\\ 0 & 1 \end{smallmatrix}\big)&\rm\big(\begin{smallmatrix} 0 & 1\\ -1 & 0 \end{smallmatrix}\big)&\rm\big(\begin{smallmatrix} -1 & 1\\ 0 & -1 \end{smallmatrix}\big)&\rm\big(\begin{smallmatrix}0 & 1\\ 1 & -1 \end{smallmatrix}\big)&\rm\big(\begin{smallmatrix}0 & 1\\ 1 & 1 \end{smallmatrix}\big)\cr \rm size&1&1&12&8&6&8&6&6\cr \hline Tr(\rho)&2&-2&0&-1&0&1&-i\sqrt{2}&i\sqrt{2}\cr \end{array} $$ \caption{Characters of the representation $\rho$} \end{table} \FloatBarrier By $\textbf{Theorem 2.4}$, we know that there exists a Hecke eigenform $g(z)=\sum_{k=0}r(k)q^k$ with character $\chi'$ such that $\textrm{Tr}(\rho(\textrm{Frob}_p))=r(p)$ and $\textrm{det}(\rho(\textrm{Frob}_p))=\chi'(p)$. For primes $p<100$, we determined $\textrm{Frob}_p$ using \textit{sagemath} and the following behaviour was observed: \begin{table}[H] \centering \begin{tabular}{||c c c c||} \hline $a_p$ & Frobenius class representative of $p$ & $f_p$ & $L_p(s,\rho)^{-1}$ \\ [0.5ex] \hline\hline 2 & $\big(\begin{smallmatrix} 1 & 0\\ 0 & 1 \end{smallmatrix}\big)$ & 1 & $1-2p^{-s}+p^{-2s}$ \\ -2 & $\big(\begin{smallmatrix} -1 & 0\\ 0 & -1 \end{smallmatrix}\big)$ & 2 & $1+2p^{-s}+p^{-2s}$ \\ 1 & $\big(\begin{smallmatrix} -1 & 1\\ 0 & -1 \end{smallmatrix}\big)$ & 6 & $1-p^{-s}+p^{-2s}$ \\ -1 & $\big(\begin{smallmatrix} 1 & 1\\ 0 & 1 \end{smallmatrix}\big)$ & 3 & $1+p^{-s}+p^{-2s}$ \\ 0 & $\big(\begin{smallmatrix} 1 & 0\\ 0 & -1 \end{smallmatrix}\big)$ & 2 & $1-p^{-2s}$ \\ 0 & $\big(\begin{smallmatrix} 0 & 1\\ -1 & 0 \end{smallmatrix}\big)$ & 4 & $1+p^{-2s}$ \\ $i\sqrt{2}$ & $\big(\begin{smallmatrix} 0 & 1\\ 1 & -1 \end{smallmatrix}\big)$ & 8 & $1-i\sqrt{2}p^{-s}-p^{-2s}$ \\ $-i\sqrt{2}$ & $\big(\begin{smallmatrix} 0 & 1\\ 1 & 1 \end{smallmatrix}\big)$ & 8 & $1+i\sqrt{2}p^{-s}-p^{-2s}$ \\[1ex] \hline \end{tabular} \caption{} \end{table} Computing the first 100 terms of $L(s,\rho),$ we observe that $a(n)=r(n) \text{ }\forall \text{ }n<100$. Since $B(M_1(\Gamma_0(576),\chi))=96,$ by $\textbf{Theorem 2.5}$ we have $F^{576}=g$. Hence, $F^{576}$ is a newform and $L$ is the unique field extension of $\mathbb{Q}$ such that the Deligne-Serre correspondence holds. Using $\textbf{Theorem 2.6}$ and analyzing the data in $\textit{Python},$ we see that $F^{576}$ is an $S_4$ form.\\ \begin{theorem} [Splitting of Primes] We present the splitting of primes in $L$ for primes $p$ not dividing the discriminant of $L$ in the following table: \FloatBarrier \begin{table}[H] \centering \begin{tabular}{||c c c c||} \hline $a_p$ & Frobenius class representative of $p$ & $f_p$ & Number of primes $p$ splits into in $L$ \\ [0.5ex] \hline\hline 2 & $\big(\begin{smallmatrix} 1 & 0\\ 0 & 1 \end{smallmatrix}\big)$ & 1 & 48 \\ -2 & $\big(\begin{smallmatrix} -1 & 0\\ 0 & -1 \end{smallmatrix}\big)$ & 2 & 24 \\ 1 & $\big(\begin{smallmatrix} -1 & 1\\ 0 & -1 \end{smallmatrix}\big)$ & 6 & 8 \\ -1 & $\big(\begin{smallmatrix} 1 & 1\\ 0 & 1 \end{smallmatrix}\big)$ & 3 & 16 \\ 0 & $\big(\begin{smallmatrix} 1 & 0\\ 0 & -1 \end{smallmatrix}\big)$ & 2 & 24 \\ 0 & $\big(\begin{smallmatrix} 0 & 1\\ -1 & 0 \end{smallmatrix}\big)$ & 4 & 12 \\ $i\sqrt{2}$ & $\big(\begin{smallmatrix} 0 & 1\\ 1 & -1 \end{smallmatrix}\big)$ & 8 & 6 \\ $-i\sqrt{2}$ & $\big(\begin{smallmatrix} 0 & 1\\ 1 & 1 \end{smallmatrix}\big)$ & 8 & 6 \\[1ex] \hline \end{tabular} \caption{} \end{table} \FloatBarrier \end{theorem} \section{Level 1152} In this section, we discuss a newform on $\Gamma_0(1152).$ $$f_{1}^{1152}(z)=\frac{\eta(144z)\eta(24z)^7\eta(16z)}{\eta(72z)^2\eta(8z)^2\eta(48z)^3}$$ $$f_{2}^{1152}(z)=\frac{T_{17}(f_{1}^{1152}(z))}{4}=\frac{\eta(72z)\eta(48z)^7\eta(8z)}{\eta(144z)^2\eta(24z)^3\eta(16z)^2}$$ The function $$ F^{1152}(z)=\frac{f_{1}^{1152}(z)+2f_{2}^{1152}(z)}{3} $$ defines a modular form of weight 1 on $\Gamma_0(1152)$. The character is given by $$ \chi(d)=\Big(\frac{-2}{d}\Big). $$ Consider the field extension \( L \) defined by the polynomial $$ p(x)=x^{8} - 4x^{7} +8x^{5} + 14x^{4} - 32x^{3} + 28x^{2} - 48x + 34. $$ The Galois group \( G=\mathrm{Gal}(L/\mathbb{Q}) \) is the Quaternion group \( Q_8 = \langle a, b \mid a^4=1, b^2=a^2, bab^{-1}=a^{-1} \rangle \). \FloatBarrier \begin{table}[H] \centering \renewcommand{\arraystretch}{1.2} $$ \begin{array}{c|ccccc} \text{Class} & 1 & a^2 & a & b & ab \\ \text{Size} & 1 & 1 & 2 & 2 & 2 \\ \hline \rho & 2 & -2 & 0 & 0 & 0 \\ \end{array} $$ \caption{Characters of a 2-dimensional irreducible representation of $Q_8$} \end{table} \FloatBarrier Using the method of analysis from \textit{section 4,} we see that $F^{1152}$ is a newform on $\Gamma_0(1152)$ and $L$ is the field extension such that the Deligne-Serre Correspondence holds. Moreover, $F^{1152}$ is a \textit{Dihedral form}. \section{Level 5760} $$ f_1^{5760}(z) = \frac{\eta(120z)^4 \eta(48z)}{\eta(240z)^2 \eta(24z)} $$ $$ f_2^{5760}(z) = T_{29}(f_1^{240}(z)) = \frac{\eta(240z)^4 \eta(24z)}{\eta(48z) \eta(120z)^2} $$ The function $$ F^{5760}(z) = f_1^{5760}(z) + 2i f_2^{5760}(z) $$ defines a weight 1 modular form on \(\Gamma_0(5760)\). The character is $$ \chi(d) = \Big(\frac{-2}{d}\Big). $$ Let \( L \) be the composite field of \( \mathbb{Q}[i] \) and the field extension of $\mathbb{Q}$ given by the defining polynomial $$ x^8 - 18x^4 + 9. $$ The Galois group \( G = \operatorname{Gal}(L/\mathbb{Q}) \) is the Pauli group on 1-qubit: $$ G = \langle a, b, c \mid a^4 = c^2 = 1, \, b^2 = a^2, \, ab = ba, \, ac = ca, \, cbc = a^2 b \rangle. $$ \FloatBarrier \begin{table}[H] \centering \renewcommand{\arraystretch}{1.3} $$ \begin{array}{c|cccccccccc} \text{Class} & 1 & a^2 & \text{$c$} & \text{$b^2$} & \text{$ab$} & a & a^3 & \text{$b$} & \text{$ac$} & \text{$abc$} \\ \text{Size} & 1 & 1 & 2 & 2 & 2 & 1 & 1 & 2 & 2 & 2 \\ \hline \rho & 2 & -2 & 0 & 0 & 0 & -2i & 2i & 0 & 0 & 0 \\ \end{array} $$ \caption{Characters of a 2-dimensional irreducible representation of \( G \)} \end{table} \FloatBarrier Using the method of analysis from \textit{section 4,} we see that $F^{5760}$ is a newform on $\Gamma_0(5760)$ and $L$ is the field extension such that the Deligne-Serre Correspondence holds. Moreover, $F^{5760}$ is a $\textit{Dihedral form.}$ \section{Level 1080} \[ f_1^{1080}(z) = \frac{\eta(36z)^2 \eta(60z) \eta(90z)}{\eta(18z) \eta(180z)}, \quad f_2^{1080}(z) = \frac{\eta(12z)^2 \eta(30z) \eta(180z)}{\eta(60z) \eta(6z)} \] \[ f_3^{1080}(z) = \frac{\eta(12z) \eta(18z) \eta(180z)^2}{\eta(36z) \eta(90z)}, \quad f_4^{1080}(z) = \frac{\eta(6z) \eta(36z) \eta(60z)^2}{\eta(12z) \eta(30z)} \] The function \[ F^{1080}(z) = f_1^{1080}(z) + i f_2^{1080}(z) + \zeta_8 f_3^{1080}(z) + i \zeta_8 f_4^{1080}(z), \] where \(\zeta_8 = \frac{\sqrt{2}i}{1-i}\), defines a weight 1 modular form on \(\Gamma_0(1080)\). The character is given by \(\chi(d) = \left(\frac{-15}{d}\right)\). Consider the field extensions $K_1$, $K_2$, $K_3$, $K_4$ and $K_5$ of $\mathbb{Q}$ given by the defining polynomials $x^2-x+1$, $x^2-x-1$, $x^3-2$, $x^4-2x^3-4x-1$ and $x^{16}-4x^{14}-2x^{13}+10x^{12}-16x^{11}-4x^{10}+52x^9-41x^8+2x^7+80x^6-50x^5+8x^4+38x^3-20x^2+6x-1$ respectively. Let $L$ be the composite field of $K_1$, $K_2$, $K_3$, $K_4$ and $K_5$. The Galois group \( G = \operatorname{Gal}(L/\mathbb{Q}) \) is a central extension by \( C_4 \) of \( S_4 \): \begin{multline*} G = \langle a, b, c, d, e \mid a^4 = d^3 = e^2 = 1, \, b^2 = c^2 = a^2, \, ab = ba, \, ac = ca, \, ad = da, \, ae = ea, \\ cbc^{-1} = a^2b, \, dbd^{-1} = a^2bc, \, ebe = bc, \, dcd^{-1} = b, \, ece = a^2c, \, ede = d^{-1} \rangle. \end{multline*} \FloatBarrier \begin{center} \begin{small} \begin{table}[!ht] $ \begin{array}{c|rrrrrrrrrrrrrrrr} \rm \text{Class rep.}& 1 & a^2& ab & e&d& a& a^3& b& ae& a^2bcd& abce& bcd^2e& abcd^2e& bce& a^3bcd& ad\cr \rm size&1&1&6&12&8&1&1&6&12&8&6&6&6&6&8&8\cr \hline \textrm{Tr}(\rho)&2&-2&0&0&-1&2i&-2i&0&0&1&-\sqrt{2}&-\sqrt{-2}&\sqrt{2}&\sqrt{-2}&-i&i\cr \end{array} $ \caption{Character table of $G$} \end{table} \end{small} \end{center} \FloatBarrier Using the method of analysis from \textit{section 4,} we see that $F^{1080}$ is a newform on $\Gamma_0(1080)$ and $L$ is the field extension such that the Deligne-Serre Correspondence holds. Moreover, $F^{1080}$ is a $S_4$ \textit{form}. \begin{theorem} We present the splitting of primes in $L$ for primes $p$ not dividing the discriminant of $L$ in the following table: \FloatBarrier \begin{table}[H] \centering \begin{tabular}{||c c c c||} \hline $a_p$ & Frobenius class representative of $p$ & $f_p$ & Number of primes $p$ splits into in $L$\\ [0.5ex] \hline\hline 2 & $1$ & 1 & $96$ \\ -2 & $a^2$ & 2 & $48$ \\ 0 & $ab$ & 2 & $48$ \\ 0 & $e$ & 2 & $48$ \\ 0 & $b$ & 3 & $32$ \\ 0 & $ae$ & 4 & $24$ \\ $2i$ & $a$ & 4 & $24$ \\ $-2i$ & $a^3$ & 4 & $24$ \\ 1 & $a^2bcd$ & 4 & $24$ \\ -1 & $d$ & 6 & $16$ \\ $i$ & $ad$ & 8 & $12$ \\ $-i$ & $a^3bcd$ & 8 & $12$ \\ $\sqrt{2}$ & $abcd^2e$ & 8 & $12$ \\ $-\sqrt{2}$ & $abce $ & 8 & $12$ \\ $i\sqrt{2}$ & $bce$ & 12 & $8$ \\ $-i\sqrt{2}$ & $bcd^2e$ & 12 & $8$ \\[1ex] \hline \end{tabular} \caption{} \end{table} \FloatBarrier \end{theorem} \section{Level 9216} $$f_1^{9216}(z)=\frac{\eta(192z)\eta(96z)^2\eta(48z)}{\eta(384z)\eta(24z)} \text{ , } f_2^{9216}(z)=\frac{T_5(f_1^{9216}(z))}{2}=\frac{\eta(48z)^3\eta(192z)^3}{\eta(24z)\eta(96z)^2\eta(384z)}$$ $$f_3^{9216}(z)=\frac{T_{13}(f_1^{9216}(z))}{2}=\frac{\eta(24z)\eta(96z)^4\eta(384z)}{\eta(192z)^2\eta(48z)^2} \text{ , } f_4^{9216}(z)=\frac{T_{17}(f_1^{9216}(z))}{4}=\eta(24z)\eta(384z)$$ The function $$F^{9216}(z)=f_1^{384}(z)+\sqrt{2}f_2^{384}(z)+\sqrt{2}f_3^{384}(z)-2f_4^{384}(z)$$ defines a modular form of weight 1 on $\Gamma_0(9216)$ and the character $\chi(d)=\big(\frac{-1}{d}$\big). Consider the field extensions $K_1$ and $K_2$ of $\mathbb{Q}$ given by the defining polynomials $x^4+1$ and $x^8+72$, respectively. Let $L$ be the composite field of $K_1$ and $K_2$. The Galois group \( G = \operatorname{Gal}(L/\mathbb{Q}) \) is \( D_{16} \): \[ G = \langle a, b \mid a^8 = b^2 = 1, \, bab = a^{-1} \rangle. \] \FloatBarrier \begin{table}[H] \centering $$ \begin{array}{c|rrrrrrr} \text{Class} & \text{1} & \text{$a^4$} & \text{$b$} &\text{$ab$} & \text{$a^2$} & \text{$a$} & \text{$a^3$} \\ \text{Size} & 1 & 1 & 4 & 4 & 2 & 2 & 2 \\ \hline \rho & 2 & -2 & 0 & 0 & 0 & -\sqrt{2} & \sqrt{2} \\ \end{array} $$ \caption{Characters of a 2-dimensional irreducible representation of \( D_{16} \)} \end{table} \FloatBarrier Using the method of analysis from \textit{section 4,} we see that $F^{9216}$ is a newform on $\Gamma_0(9216)$ and $L$ is the field extension such that the Deligne-Serre Correspondence holds. Moreover, $F^{9216}$ is a $\textit{Dihedral form.}$ \section{Level 23040} \raggedright $$f_1^{23040}(z)=\frac{\eta(120z)^2\eta(96z)^2\eta(48z)}{\eta(240z)\eta(192z)\eta(24z)} \text{ , } f_2^{23040}(z)=\frac{T_7(f_1^{23040}(z))}{2}=\frac{\eta(240z)^2\eta(192z)\eta(48z)^2}{\eta(480z)\eta(96z)\eta(24z)}$$ $$f_3^{23040}(z)=\frac{T_{31}(f_1^{23040}(z))}{2}=\frac{\eta(24z)\eta(192z)\eta(240z)^5}{\eta(48z)\eta(120z)^2\eta(480z)^2}\text{ , } f_{4}^{23040}(z)=-\frac{T_{29}(f_1^{23040}(z))}{2}=\frac{\eta(24z)\eta(96z)^2\eta(480z)^2}{\eta(48z)\eta(192z)\eta(240z)}$$ $$f_{5}^{23040}(z)=-\frac{T_{53}(f_1^{23040}(z))}{2}=\frac{\eta(192z)\eta(24z)\eta(480z)^5}{\eta(96z)\eta(240)^2\eta(960z)^2}\text{ , } f_{6}^{23040}(z)=\frac{T_{59}(f_1^{23040}(z))}{4}=\frac{\eta(480z)^2\eta(192z)\eta(48z)^3}{\eta(240z)\eta(24z)\eta(96z)^2}$$ $$f_{7}^{23040}(z)=\frac{T_{73}(f_1^{23040}(z))}{2}=\frac{\eta(24z)\eta(96z)^3\eta(240z)^2}{\eta(48z)^2\eta(192z)\eta(480z)}\text{ , } f_{8}^{23040}(z)=-\frac{T_{83}(f_1^{23040}(z))}{4}=\frac{\eta(48z)^2\eta(96z)\eta(960z)^2}{\eta(24z)\eta(192z)\eta(480z)}$$ \begin{align*} F^{23040}(z) &= \frac{ f_1^{23040}(z) + \sqrt{2} f_2^{23040}(z) + i\sqrt{2} f_3^{23040}(z) + i\sqrt{2} f_4^{23040}(z) }{1+i} \\ &\quad + \frac{ \sqrt{2} f_5^{23040}(z) + 2 f_6^{23040}(z) + i f_7^{23040}(z) + 2i f_8^{23040}(z) }{1+i} \end{align*} The function $F^{23040}$ defines a weight 1 modular form of weight 1 on $\Gamma_0(23040)$. The character is $\chi(d)=\Big(\frac{-10}{d}\Big)$. Consider the field extensions $K_1$ and $K_2$ of $\mathbf{Q}$ given by the defining polynomials $x^2+2$ and $x^{16} - 60x^{14} + 1098x^{12} - 8280x^{10} + 28710x^8 - 43200{x^6} + 19116x^{4} + 6480x^{2} + 324$ respectively. Let $L$ be the composite field of $K_1$ and $K_2$. The Galois group \( G = \mathrm{Gal}(L/\mathbb{Q}) \) is the central product of \( C_4 \) and \( D_8 \): \[ G = C_4 \circ D_8 = \langle a, b, c \mid a^4 = c^2 = 1, \, b^4 = a^2, \, ab = ba, \, ac = ca, \, cbc = a^2b^3 \rangle. \] \FloatBarrier \begin{table}[H] \centering $$ \begin{array}{c|rrrrrrrrrrrrrr} \text{Class} & \text{1} & \text{$a^2$} & \text{$c$} & \text{$bc$} & \text{$ab^2$} & \text{$a$} & \text{$a^3$} & \text{$b^2$} & \text{$ac$} & \text{$abc$} & \text{$b$} & \text{$b^3$} & \text{$ab$}& \text{$ab^3$} \\ \text{Size} & 1 & 1 & 2 & 4 & 4 & 1 & 1 & 2 & 4 & 4 & 2 & 2 & 2 & 2 \\ \hline \rho & 2 & -2 & 0 & 0 & 0 & -2i & 2i & 0 & 0 & 0 & -\sqrt{-2} & \sqrt{-2} & -\sqrt{2} & \sqrt{2} \\ \end{array} $$ \caption{Characters of a 2-dimensional irreducible representation of \( C_4 \circ D_8 \)} \end{table} \FloatBarrier Using the method of analysis from \textit{section 4,} we see that $F^{23040}$ is a newform on $\Gamma_0(23040)$ and $L$ is the field extension such that the Deligne-Serre Correspondence holds. Moreover, $F^{23040}$ is a $\textit{Dihedral form.}$ \section{Main result} The analysis done in previous sections proves the result: \begin{theorem}Within the weight 1 eta quotients listed in \cite{Bha1} but $\textbf{not}$ in \cite{GK2}, the following are all possible linear combinations that give a Hecke eigenform with $q$-expansion starting at $q$ \setlength{\tabcolsep}{12pt} \renewcommand{\arraystretch}{1.5} \begin{table}[H] \centering \begin{tabular}{|| c | c ||} \hline \textbf{Newform} & \textbf{Projective Image of Galois Representation} \\ [1ex] \hline\hline $F^{1152}$ & Dihedral \\ \hline $F^{576}$ & $S_4$ \\ \hline $F^{5760}$ & Dihedral \\ \hline $F^{1080}$ & $S_4$ \\ \hline $F^{9216}$ & Dihedral \\ \hline $F^{23040}$ & Dihedral \\ \hline \end{tabular} \caption{Projective images of Galois representations corresponding to the newforms.} \label{tab:proj_image} \end{table} \end{theorem} \section{Conclusion} We have discussed the splitting of primes in the corresponding Galois number fields using the coefficients $a(p)$ of the newforms. We also proved that the representation is of type $S_4$ for both the newforms. Bhattacharya, in \cite{Bha1}, lists simple holomorphic eta quotients on $\Gamma_0(N)$ of weight-1 up to level 768. Mersmann, in \cite{Mer}, shows that there are only finitely many simple holomorphic eta quotients of weight-1. Using the list of weight-1 simple holomorphic eta quotients, we attempted to find all exotic newforms that can be constructed from a linear combination of holomorphic eta quotients. Since there are only finitely many simple holomorphic eta quotients of weight-$1$, the number of such newforms is finite. The two newforms discussed in this paper are the only exotic newforms with the $q$-series being $q+\sum_{n=2}a(n)q^n$ that can be constructed from a linear combination of holomorphic eta quotients contained in this list. One can attempt to find a complete list of all exotic newforms of weight-1 that can be constructed from a linear combination of simple holomorphic eta quotients. The newforms discussed in the paper further raise a question: If an exotic newform is constructed from a linear combination of simple holomorphic eta quotients, is the representation $\rho$ such that the Deligne-Serre correspondence holds, always of type $S_4$? \section{Acknowledgements} I would like to express my deepest gratitude to Prof. Yingkun Li for encouraging and guiding me throughout the course of this research. The research was carried out at Technische Universität Darmstadt in Germany, and I am grateful to DAAD-WISE for the funding. Lastly, I would like to extend my heartfelt thanks to my family and friends for their constant support and motivation. \bibliographystyle{apsrev4-1} \bibliography{references} \end{document}
2412.05238v1
http://arxiv.org/abs/2412.05238v1
Some splitting and rigidity results for sub-static spaces
\documentclass[10pt,a4paper]{amsart} \usepackage{geometry} \geometry{a4paper,top=3cm,bottom=3cm,left=3cm,right=3cm} \usepackage{amsmath} \usepackage{amssymb} \usepackage{amsthm} \usepackage{xcolor} \usepackage{verbatim} \usepackage{tabularx} \usepackage{mathrsfs} \usepackage{appendix} \usepackage[pdftex]{hyperref} \theoremstyle{theorem} \newtheorem{theorem}{Theorem}[section] \newtheorem{lemma}[theorem]{Lemma} \newtheorem{proposition}[theorem]{Proposition} \newtheorem{corollary}[theorem]{Corollary} \newtheorem{theoremletter}{Theorem} \renewcommand{\thetheoremletter}{\Alph{theoremletter}} \theoremstyle{remark} \newtheorem{example}{Example}[section] \newtheorem{remark}[theorem]{Remark} \numberwithin{equation}{section} \theoremstyle{definition} \newtheorem{definition}{Definition}[section] \DeclareMathOperator{\diver}{div} \DeclareMathOperator{\Riem}{Riem} \DeclareMathOperator{\Ric}{Ric} \DeclareMathOperator{\Sect}{Sect} \DeclareMathOperator{\Hess}{Hess} \DeclareMathOperator{\trace}{tr} \DeclareMathOperator{\vol}{Vol} \DeclareMathOperator{\supp}{supp} \newcommand{\TRicc}{{}^T\!\Ric} \newcommand{\TScal}{{}^T\!S} \newcommand{\TWeyl}{{}^T\!W} \newcommand{\TV}{{}^T\!V} \newcommand{\TU}{{}^T\!U} \newcommand{\TA}{{}^T\!A} \newcommand{\TZ}{{}^T\!Z} \newcommand{\TC}{{}^T\!C} \newcommand{\TG}{{}^T\!G} \newcommand{\R}{\mathbb{R}} \newcommand{\N}{\mathbb{N}} \newcommand{\C}{\mathbb{C}} \newcommand{\MM}{\mathcal{M}} \newcommand{\LL}{\mathcal{L}} \newcommand{\II}{\mathrm{II}} \newcommand{\scal}{\mathcal{S}} \newcommand{\sphere}{\mathbb{S}} \newcommand{\lip}{\mathrm{Lip}} \newcommand{\loc}{\mathrm{loc}} \newcommand{\eps}{\varepsilon} \newcommand{\tcr}{\textcolor{red}} \newcommand{\tcb}{\textcolor{blue}} \newcommand{\tco}{\textcolor{orange}} \newcommand{\di}{\mathrm{d}} \newcommand{\Sym}{\mathrm{Sym}} \newcommand{\MS}{\mathsf{MS}} \newcommand{\disp}{\displaystyle} \newcommand{\cotg}{\mathrm{cotg}\,} \newcommand{\haus}{\mathcal{H}} \newcommand{\Mo}{\mathring{M}} \newcommand{\ch}{\mathrm{ch}} \newcommand{\sh}{\mathrm{sh}} \newcommand{\seg}{{\rm seg}} \renewcommand{\div}{\diver} \newcommand{\metric}{\langle\,,\,\rangle} \renewcommand{\phi}{\varphi} \renewcommand{\emptyset}{\varnothing} \makeatletter \newcommand*\owedge{\mathpalette\@owedge\relax} \newcommand*\@owedge[1]{ \mathbin{ \ooalign{ $#1\m@th\bigcirc$\cr \hidewidth$#1\m@th\wedge$\hidewidth\cr } } } \makeatother \title[Splitting and rigidity of sub-static spaces]{Some splitting and rigidity results for sub-static spaces} \author{Giulio Colombo} \address{Dipartimento di Matematica e Applicazioni ``R. Caccioppoli'', Universit\`a degli Studi di Napoli Fe\-de\-ri\-co II, Via Vicinale Cupa Cintia 21, I-80126 Napoli, Italy} \email{[email protected]} \author{Allan Freitas} \address{Departamento de Matem\'{a}tica, Universidade Federal da Para\'{\i}ba, 58.051-900 Jo\~{a}o Pessoa, Para\'{\i}ba, Brazil} \email{[email protected]/[email protected]} \author{Luciano Mari} \address{Dipartimento di Matematica ``F. Enriques", Universit\`a degli studi di Milano, Via Saldini 50, I-20133 Milano, Italy.} \email{[email protected]} \author{Marco Rigoli} \address{Dipartimento di Matematica ``F. Enriques", Universit\`a degli Studi di Milano, Via Saldini 50, I-20133 Milano, Italy} \email{[email protected]} \begin{document} \maketitle \noindent \textbf{MSC 2020} { Primary: 53C21, 53C24, 53C42, Secondary: 53C25, 53C43, 83C20. } \noindent \textbf{Keywords} { Sub-static $\cdot$ Rigidity $\cdot$ Stable Minimal $\cdot$ Wave maps } \begin{abstract} In this paper we study the rigidity problem for sub-static systems with possibly non-empty boundary. First, we get local and global splitting theorems by assuming the existence of suitable compact minimal hypersurfaces, complementing recent results in the literature. Next, we prove some boundary integral inequalities that extend works by Chru\'sciel and Boucher-Gibbons-Horowitz to non-vacuum spaces. Even in the vacuum static case, the inequalities improve on known ones. Lastly, we consider the system arising from static solutions to the Einstein field equations coupled with a $\sigma$-model. The Liouville theorem we obtain allows for positively curved target manifolds, generalizing a result by Reiris. \end{abstract} \section{Introduction} The purpose of this paper is to study some rigidity problems for static solutions to the Einstein field equations \begin{equation}\label{einsteq} \Ric_{\hat{g}}+\left(\Lambda-\frac{1}{2}S_{\hat{g}}\right)\hat{g}=T. \end{equation} Here, $(\hat{M}^{m+1},\hat{g})$ is a Lorentzian manifold of dimension $m+1\geq 4$ with Ricci and scalar curvature, respectively, $\Ric_{\hat{g}}$ and $S_{\hat{g}}$; $T$ is the stress-energy tensor, which accounts for the distribution of matter, energy, and momentum in the manifold, and $\Lambda$ is the cosmological constant. Looking for static solutions, that is, solutions possessing a timelike Killing field whose orthogonal distribution is integrable, leads to study the following warped product manifolds: \begin{equation}\label{static_model} \hat{M} = \mathbb{R} \times M, \qquad \hat{g} = - u^2 \, \di t \otimes \di t + g, \end{equation} where $(M^m,g)$ is a Riemannian manifold and $0 < u \in C^\infty(M)$. The lapse function $u$ determines how time stretches and compresses in different spatial locations in $M$. Physical reasons demand that $T$ satisfies the null energy condition (NEC), that is, $T$ is non-negative on null vectors. Condition (NEC) is satisfied by well-known sources including electrostatic ones, scalar fields, and certain perfect-fluid models, and its geometric consequences are highlighted, for instance, by Penrose's Singularity Theorem \cite{penrose} (see also \cite[Page 263]{hawking}). Letting $\pi : (\hat M,\hat g) \to (M,g)$ be the projection onto the second factor, for each null vector $Y$ a computation gives \begin{equation}\label{eq_Q} T(Y,Y)=\left[\Ric -\frac{\Hess u}{u}+\frac{\Delta u}{u}g\right](\pi_*Y,\pi_*Y), \end{equation} where \(\Ric \), \(\Hess \), \(\Delta \) are the Ricci curvature, the Hessian and the Laplacian on \((M^m, g)\), respectively. The (NEC) condition is therefore equivalent to \begin{equation}\label{sub_static} u \Ric - \Hess u + (\Delta u) g \doteq uQ\geq 0, \end{equation} \begin{definition} A \emph{sub-static triple} $(M^m,g,u)$ is the data of a smooth, complete Riemannian manifold $(M^m,g)$ of dimension $m \ge 3$, possibly with boundary $\partial M$, and a solution $u \in C^\infty(M)$ to \eqref{sub_static} with $u>0$ in the interior $\mathring{M}$. If $\partial M\neq\emptyset$, we assume $u=0$ on $\partial M$. \end{definition} \begin{remark} A priori, we do not assume that $Q$ can be extended continuously to $\partial M$ when this latter is non-empty. However, some result will need the condition, which is however meaningful due to \eqref{eq_Q} if we assume that $T$ is defined up to $\partial \hat M$. \end{remark} In the above setting, $\partial M$ models the event horizon of a static black-hole. If $Q \equiv 0$, system \eqref{sub_static} describes static solutions to \eqref{einsteq} with $T \equiv 0$, named vacuum static spaces. A famous conjecture by Boucher, Gibbons, and Horowitz \cite{BGH} states: \textit{(Cosmic No-Hair Conjecture) The only compact vacuum static triple \((M^m, g, u)\) with positive scalar curvature and connected boundary is given by a standard round hemisphere \(\mathbb{S}^m_{+}\), with static potential \(u\) being the height function.} In particular, the Cosmic No-Hair Conjecture suggests that, under certain conditions, the evolution of the universe leads to the dominance of a simple, highly symmetric geometry, ruling out more complicated ``hairy" solutions. Several works have established the conjecture under additional hypotheses. Reilly \cite[Theorem 4]{reilly} showed that the conjecture is true if $M$ is Einstein. Boucher, Gibbons, and Horowitz \cite{BGH} obtained a similar result, demonstrating the uniqueness of spacetime (the anti-de Sitter space) in the case of negative scalar curvature. Boucher \cite{bouch} and Friedrich \cite{fried} proved the result by assuming a Penrose compactification of spacetime, coupled with certain conditions at the conformal infinity. The same positive answer is obtained when $(M^{m},g)$ is conformally flat, as proved independently by Kobayashi \cite{koba} and Lafontaine \cite{laf}. Despite these developments, Gibbons, Hartnoll, and Pope \cite{ghp} constructed counterexamples to the Cosmic No-Hair Conjecture in dimensions $4 \leq m \leq 8$, and Costa, Di\'ogenes, Pinheiro and Ribeiro \cite{costa} constructed a simply-connected counterexample in any dimension $m \geq 4$. To our knowledge, the conjecture in its full generality is still open in dimension $m=3$. It is natural to wonder how the behavior of \( \partial M = u^{-1}(0)\) can influence the rigidity of the entire manifold. If we assume that $Q$ extends continuously to $\partial M$, equation \eqref{sub_static} implies that $\partial M$ is totally geodesic, and thus \(|\nabla u|\) is constant on each connected component of \(\partial M\). If \((M^m, g, u)\) is a compact vacuum static triple, then necessarily its scalar curvature $S$ is a positive constant which we can assume to be $m(m-1)$ by scaling. In this case, denoting by $\{\Sigma_i\}$, $1 \le i \le l$ the connected components of $\partial M$, Chru\'sciel \cite{Crus} showed that \begin{equation}\label{crus} \sum_{i=1}^l \kappa_i \int_{\Sigma_i} \left(S_{\Sigma_i} - (m-2)(m-1) \right) \geq 0, \end{equation} where $S_{\Sigma_i}$ is the scalar curvature of $\Sigma_i$ and \(\kappa_i\) is the restriction of \(|\nabla u|\) to \(\Sigma_i\), named the surface gravity of $\Sigma_i$. Furthermore, equality implies that \(M\) is a round hemisphere and, consequently, that \(l = 1\), i.e., \(\Sigma\) is connected. Chrusciel's inequality is a key step to prove an important rigidity result in dimension \(m = 3\): \begin{theorem}[Boucher, Gibbons, and Horowitz \cite{BGH}; Shen \cite{sh}]\label{bgh} Let \((M^3, g, u)\) be a compact oriented vacuum static triple with connected boundary and scalar curvature $6$. Then the area of \(\partial M\) satisfies the inequality \[ |\partial M| \leq 4\pi. \] Moreover, equality holds if and only if \((M^3, g)\) is isometric to the standard hemisphere. \end{theorem} For general sub-static triples, we prove a family of inequalities extending \eqref{crus}, depending on a parameter $b$. The inequalities are a consequence of some Shen's type identities which may have independent interest. We obtain the following extension of \eqref{crus}: \begin{theoremletter}\label{thm_BGH} Let \((M^m, g, u)\) be a compact sub-static triple such that $\partial M \neq \emptyset$. Assume that $Q$ extends continuously to $\partial M$ and that the scalar curvature $S$ of $M$ satisfies \[ S - \trace Q \qquad \text{is constant on } \, M. \] Then, for any real parameter \[ b \geq - \frac{1}{m-1}, \] the following inequality holds: \begin{equation} \label{BGHtype_intro} \sum_{i=1}^{l} \kappa_i^b \int_{\Sigma_i} \left(S_{\Sigma_i} - \frac{m-2}{m}S - \frac{2}{m} \trace Q\right) \ge 0 \end{equation} where \(\Sigma_1, \dots, \Sigma_l\) are the connected components of \(\partial M\), $S_{\Sigma_i}$ is the scalar curvature of $\Sigma_i$ and \(\kappa_i > 0\) is the surface gravity of $\Sigma_i$. Moreover, the inequality in \eqref{BGHtype_intro} is strict unless \(M\) is isometric to a round hemisphere and \(Q \equiv 0\) on \(M\). \end{theoremletter} Exploiting the dependence on $b$ in the above result and applying the Gauss-Bonnet Theorem, in dimension \(m=3\) we get the following refinement of the inequality by Boucher-Gibbons-Horowitz, where $\partial M$ is allowed to be disconnected: \begin{corollary}\label{coro_bgh} Let $(M^3,g,u)$ be a compact sub-static triple such that $\partial M \neq \emptyset$. Assume that $Q$ extends in a $C^1$-way to $\partial M$ and that the scalar curvature $S$ of $M$ satisfies \[ S - \trace Q \qquad \text{is constant on } \, M. \] Then, $S - \trace Q >0$. Moreover, if $0 < \kappa_1 < \kappa_2 < \ldots < \kappa_j$ are the distinct surface gravities, and we decompose $\partial M = \bigcup_{a=1}^j \hat{\Sigma}_a$ where $\hat \Sigma_a$, possibly disconnected, has surface gravity $\kappa_a$, then: \begin{itemize} \item[(i)] at least one of the connected components of $\hat{\Sigma}_j$ is topologically a sphere, and \begin{equation}\label{ine_enhanced_BGH} |\hat{\Sigma}_j|\leq \frac{24\pi}{S - \trace Q} \ ; \end{equation} \item[(ii)] if, for some $i \in \{2,\ldots, j\}$, it holds \begin{equation}\label{ine_enhanced_BGH_a} |\hat{\Sigma}_a|= \frac{24\pi}{S - \trace Q} \qquad \forall \, i \le a \le j, \end{equation} then at least one of the connected components of $\hat{\Sigma}_{i-1}$ is topologically a sphere, and \begin{equation}\label{ine_enhanced_BGH_am1} |\hat{\Sigma}_{i-1}|\leq \frac{24\pi}{S - \trace Q} \ ; \end{equation} \item[(iii)] equality \begin{equation}\label{ine_enhanced_BGH_tutti} |\hat{\Sigma}_a|= \frac{24\pi}{S - \trace Q} \qquad \forall \, 1 \le a \le j \end{equation} holds if and only if $M$ is isometric to a round 3-hemisphere and $Q \equiv 0$ on $M$. \end{itemize} \end{corollary} A second type of results addressed in this paper are splitting theorems. In Lorentzian manifolds, various works explored the influence of curvature on splitting phenomena (see, e.g., \cite{esch,garcia,javaloyes}). Given a sub-static triple $(M^{m},g,u)$, the analysis of the so-called optical metric $\bar{g}=u^{-2}g$ turns out to be a particularly useful tool. In this respect, Borghini and Fogagnolo \cite{fogborg} discovered a striking connection between the sub-static condition and the ${\rm CD}(0,1)$ condition: by setting $f = -(m-1) \ln u$, the manifold $(\mathring{M},\bar g)$ satisfies \[ \overline{\Ric}_f^1 \doteq \overline{\Ric} + \overline{\Hess} f + \frac{1}{m-1}\di f \otimes \di f = Q \ge 0 \] that is, by definition, $(\mathring{M},\bar g, e^{-f}\di x_g)$ is a ${\rm CD}(0,1)$ space. A tightly related link was also observed in the work by Li and Xia \cite{lixia_17}, by means of an interpolating family of connections (see also the discussion in \cite[Appendix A.3]{fogborg}). We recall that, for $N \neq m$ and a function $f \in C^\infty(M)$, the Bakry-Emery Ricci tensor $\Ric_f$ and its $N$-modified counterpart $\Ric_f^N$ are defined as \[ \Ric_f \doteq \Ric + \Hess f, \qquad \Ric_f^N \doteq \Ric + \Hess f - \frac{1}{N-m}\di f \otimes \di f. \] Recently, Wylie and Yeroshkin \cite{wylie1,wylie} developed a comparison theory for ${\rm CD}(0,1)$ spaces, and their results were adapted to sub-static manifolds to prove elegant splitting theorems in \cite[Theorems C, 3.7, 3.8 and Corollary 3.9]{fogborg} by assuming that the ends of the manifold are either {\it $u$-complete} or {\it conformally compact}. Let $E$ be an end of $M$, that is, a connected component with non-compact closure of the complementary of a compact set $K$. We set: \begin{definition}\label{def_u_complete} An end \(E \subset M\) is said to be \emph{$u$-complete} if, for any $g$-unit speed diverging curve $\gamma: [0, \infty) \longrightarrow \overline{E} \subset M$, it holds $$ \int_0^\infty u^{-1}(\gamma(t))\di t = \infty \qquad \mbox{and}\qquad \int_{0}^{\infty}u(\gamma(t)) \di t= \infty. $$ \end{definition} \noindent This is a mild request satisfied, for instance, if \[ C^{-1} r(x)^{-1} \le u(x) \le Cr(x) \qquad \text{for } \, x \in E, \ r(x) >>1, \] where $C>1$ is a constant and $r$ is the distance from a fixed compact set (see Proposition \ref{prop_suffcond}). Definition \ref{def_u_complete} looks a bit different from that in \cite{fogborg}, see however Remark \ref{rem_ucomplete}. If the compact set $K$ is not too pathological (for instance, if $K$ is a Lipschitz embedded hypersurface), the property corresponds to the completeness of $\overline{E}$ endowed with the distance induced by any of the metrics $u^{-2}g$ and $u^2 g$. The presence of area minimizing hypersurfaces often foresees splitting properties. By analyzing the flow introduced by Galloway in \cite[Lemma 3]{galloway}, Ambrozio \cite[Proposition 14]{ambrozio} (for $m=3$) and Huang, Martin, and Miao \cite{hmm} studied the local splitting behavior of vacuum static triples, see also Cruz, Lima and de Sousa \cite{tiarlos} (for $m=3$) and Coutinho and Leandro \cite{benedito} for analogous results in electrostatic systems. One contribution of the present paper is to explicitly observe a connection between area-minimizing hypersurfaces in sub-static systems and $f$-area-minimizing hypersurfaces with respect to the conformal metric $\bar{g} = u^{-2}g$. This connection and the link between sub-staticity and the ${\rm CD}(0,1)$ condition allows us to extend the results in \cite{ambrozio,benedito,tiarlos,hmm} to any sub-static space, streamlining their proofs, and to complement those in \cite{fogborg}. We obtain: \begin{theoremletter}\label{local_split} Let $(M^{m}, g, u)$ be a sub-static triple and $\Sigma \to M$ be a closed, possibly disconnected, two-sided stable minimally embedded hypersurface such that $u>0$ on $\Sigma$. \begin{itemize} \item[(A)] Assume that $\Sigma$ is locally area minimizing. Then there is $\eps>0$ and a diffeomorphism \[ \Phi : (-\eps,\eps) \times \Sigma \to M \] such that in coordinates $(s,y) \in (-\eps,\eps) \times \Sigma$ it holds \[ \Phi^* g = r^{2}(y)\di s^2 + h^{\Sigma}, \] where $h^\Sigma$ is the induced metric on $\Sigma \to (M,g)$ and $r : \Sigma \to (0,\infty)$. Moreover, there exists a function $\sigma : (-\eps,\eps) \to (0,\infty)$ such that $u(\Phi(s,y)) = r(y)\sigma(s)$. \item[(B)] If an end $E$ with respect to $\Sigma$ is $u$-complete, then: \begin{itemize} \item[(i)] the topological boundary $\partial E \subset M$ is a single connected component of $\Sigma$, and separates $E$ from $M \backslash E$; \item[(ii)] the closure $\overline{E} \subset M$ is an embedded submanifold isometric to \[ [0,\infty) \times \partial E \qquad \text{with metric} \qquad g = r^{2}(y)\di s^2 + h^{\Sigma}, \] where $h^\Sigma$ is the induced metric on $\Sigma \to (M,g)$ and $r : \Sigma \to (0,\infty)$. Moreover in coordinates $(s,y)\in [0,\infty) \times \partial E$, it holds $u(s,y) = r(y)\sigma(s)$ on $E$ for some function $\sigma : [0,\infty) \to (0,\infty)$. \end{itemize} \end{itemize} \end{theoremletter} \begin{remark}\label{rem_covering} In (A), one can require that $\varsigma : \Sigma \to M$ is merely an immersion with image $\varsigma(\Sigma)$ area minimizing in the sense of Almgren (see \cite{almgren76,taylor76}), that is, satisfying $|\varsigma(\Sigma)| \leq |\psi(\varsigma(\Sigma))|$ for any Lipschitz continuous map $\psi : M \to M$ with $\psi(\varsigma(\Sigma)) \subset \mathring{M}$. In this case, in our assumptions $\varsigma$ turns out to factorize through a Riemannian covering $\Sigma \to \hat{\Sigma}$ and an embedding $\hat\Sigma \to M$, see Theorem \ref{teo_splitting_1}. \end{remark} Some remarks are in order: \begin{itemize} \item In (A), the extra information drawn in the particular cases treated in \cite{ambrozio,benedito,tiarlos,hmm} regards the behaviour of the function $r$. In the vacuum static case, it is shown in \cite{ambrozio} and \cite[Proposition 5]{hmm} that $r(y)$ is constant, thus $g$ is locally the product metric. In the presence of an electromagnetic field and for $m=3$, this is the case if the electric field vanishes identically, see \cite[Proposition 7]{tiarlos}. \item The proof of (B) follows the standard splitting techniques by means of Busemann and distance functions, see in particular \cite{kasue}, adapted by Wylie \cite{wylie1} to ${\rm CD}(0,1)$ manifolds. However, despite \cite[Section 5]{wylie1} treats manifolds with boundary, our theorem cannot be directly drawn from \cite{wylie1}. Likewise, the results in \cite{fogborg} do not apply in the setting of (B); note, in particular, that the $u$-completeness assumption in (B) is localized to a single end $E$. This is different from \cite{fogborg}, where \emph{every} end needs to be either $u$-complete or conformally compact, and requires some additional arguments compared to \cite{wylie,fogborg}. In particular, to get our conclusion we shall ask for $\Sigma$ to be stable. We currently do not know whether this assumption is removable. \item The condition of $u$-completeness is highlighted, for instance, in a paper by Reiris \cite{reiris_compa}. There, the author conjectured that a $3$-dimensional vacuum static triple $(M^3,g,u)$ with non-empty boundary and $u$-complete ends must either be the Schwarzschild solution or a flat solution, namely, $M = [0,\infty) \times \Sigma$ with the product metric $\di s^2 + h^\Sigma$ and potential $u(x,y) = s$. Here, $h^\Sigma$ is a flat metric on $\Sigma$. This would be a far-reaching extension of the known no-hair theorems for Schwarzschid space, in which an asymptotic flatness assumption is required (for example, see \cite{agost, bunting, cerd, israel, robinson}). \item If $\partial M = \emptyset$, in \cite{case} Case proved the triviality of complete, vacuum static triples with non-negative scalar curvature (meaning, $u$ is constant and $M$ is Ricci-flat) in any dimension. The result is somehow implicit in \cite{case}, but can be seen as follows: the substitution $u = e^{-f}$ transforms \eqref{sub_static} with $Q=0$ into \[ \Ric_f^{m+1} = (\Delta_{f}f)g, \qquad \Delta_f f = \frac{S}{m-1}. \] Taking into account that $S \ge 0$ is constant on a vacuum static space, by applying \cite[Theorem 1.5]{case} and letting $a\to \infty$ one gets the constancy of $u$. \item Regarding the more general case of \emph{stationary} solutions to the Einstein vacuum equation, rigidity in dimension $m=3$ by only assuming $(M^3,g)$ complete was obtained by Anderson \cite{anderson}. A simpler proof under the further assumption that $\tilde{g}= u^2 g$ is complete was later given by Cortier and Minerbe \cite{cortier_minerbe}. The use of the metric $\tilde{g}$ is well-established in the literature. For vacuum static triples $(M^3,g,u)$, the system obtained by rewriting \eqref{sub_static} in the metric $\tilde{g}$ is called the harmonic map representation of $M$ (the reason for the name is apparent, for instance, in \cite{anderson}). In \cite{reiris_compa}, the author developed an interesting comparison theory for sub-static triples based on the use of $\tilde{g}$. It would be nice to clarify the interplay with the theory developed in \cite{fogborg}, as they may reveal further properties of sub-static spaces. \end{itemize} The last part of the paper focuses on a special sub-static triple: the one arising from the Einstein field equations on $\hat{M}$ with source a nonlinear $\sigma$-model, described by a map $\Phi: (\hat{M}^{m+1}, \hat{g}) \to (N^n, h)$ valued in a Riemannian manifold $(N,h)$ and a potential $V: (N^n, h) \to \mathbb{R}$. The associated gravitational+matter Lagrangian is \[ \mathcal{L}(\hat{g},\Phi) = \int_{\hat{M}} S_{\hat g} \di x_{\hat{g}} + \int_{\hat{M}} \left[ |\di\Phi|_{\hat{g}}^2 + (m-1)V(\Phi) \right] \di x_{\hat{g}}. \] For notational convenience, we incorporate the cosmological constant into $V$. Critical points of $\mathcal{L}$ solve \eqref{einsteq} with \[ T = \Phi^{*}h - \frac{1}{2} \left( |\di \Phi|_{\hat{g}}^2 + (m-1)V(\Phi) \right)\hat{g} \] and the equation of motion $\delta_\varphi \mathcal{L} = 0$. If $\Phi$ factorizes as $\pi \circ \varphi$ for some smooth map $\varphi : (M,g) \to (N,h)$, static solutions to $(\delta_{\hat g} \mathcal{L}, \delta_\varphi \mathcal{L}) = (0,0)$ therefore solve \[ \left\{ \begin{array}{r@{\;}c@{\;}l} u\Ric - \Hess u + (\Delta u)g & = & u\varphi^{*}h, \\[0.2cm] -\Delta u & = & V(\varphi) u, \\[0.2cm] u \tau(\varphi) + \di \varphi(\nabla u) & = & (m-1) \frac{D V(\varphi)}{2} u, \end{array} \right. \] where $D$ is the Levi-Civita connection of $h$ and $\tau(\varphi)$ is the tension field of $\varphi$ (see Section 4 and \cite{ans21}). A relevant example is given by the Klein-Gordon field, for which \[ \varphi : M \to \mathbb{C}, \qquad V(\varphi) = m^2 |\varphi|^2. \] When the target manifold of $\varphi$ is $N = \R,\mathbb{C}$ and $M$ has no boundary, Reiris \cite{reiris} obtained sharp rigidity results by only assuming the completeness of the underlying manifold $M$ and suitable conditions on $V$, general enough to encompass the Klein-Gordon and other scalar fields of interest. His approach employs techniques from comparison geometry \`a la Bakry-\'Emery. In Theorem \ref{vanishing_map}, we allow for manifolds with $\partial M \neq \emptyset$ and targets $N$ be positively curved (in a controlled way). We obtain: \begin{theoremletter}\label{vanishing_map} Let $(M^m, g, u)$ be a sub-static triple with either $\partial M = \emptyset$ or $\partial M$ compact, and let $\varphi : M \to (N,h)$ be a smooth map. Assume that \begin{equation}\label{map_source} \left\{ \begin{array}{r@{\;}c@{\;}l} u\Ric - \Hess u + (\Delta u)g & \ge & u\varphi^{*}h, \\[0.2cm] -\Delta u & = & V(\varphi) u, \\[0.2cm] u \tau(\varphi) + \di \varphi(\nabla u) & = & (m-1) \frac{D V(\varphi)}{2} u, \end{array} \right. \end{equation} and that \[ \inf_{M}V(\varphi)>-\infty, \qquad(m-1)\Hess V + 2Vh \geq -ah\qquad\mbox{on}\quad\varphi(M), \] for some constant $a\geq 0$. If the sectional curvatures of $N$ satisfy \begin{equation}\label{est_sec} \sup_{N} \mathrm{Sec}_{N}\leq\kappa < \frac{1}{m-1}, \end{equation} for some constant $\kappa$, then $$ \sup_{M}|\di\varphi|^{2}\leq \left[\frac{am}{2(1-(m-1)\kappa)}\right]^{\frac{1}{2}}. $$ In particular, if $a=0$ then $\varphi$ is constant. \end{theoremletter} For instance, by applying the result to wave maps with $V \equiv 0$, we get \begin{theorem}\label{vanishing_map_2} Let $(M^m, g, u)$ be a sub-static triple with either $\partial M = \emptyset$ or $\partial M$ compact, solving \begin{equation}\label{map_source_2} \left\{ \begin{array}{r@{\;}c@{\;}l} u\Ric - \Hess u + (\Delta u) g & \ge & u\varphi^{*}h \\[0.2cm] -\Delta u & = & 0, \\[0.2cm] u \tau(\varphi) + \di \varphi(\nabla u) & = & 0, \end{array} \right. \end{equation} for some smooth map $\varphi : (M,g) \to (N,h)$ into a Riemannian manifold $N$. If $N$ satisfies \begin{equation}\label{est_sec_2} \sup_{N} \mathrm{Sec}_{N} < \frac{1}{m-1}, \end{equation} then $\varphi$ is constant.\\ In particular, a static solution \eqref{static_model} to Einstein's equation with $M$ complete, $\partial M$ compact or empty, and source a static wave map $\varphi : M \to N$ in a manifold whose curvature satisfies \eqref{est_sec_2} is vacuum static. \end{theorem} We emphasize that Theorems \ref{vanishing_map} and \ref{vanishing_map_2} do not impose any decay hypotheses such as asymptotic flatness. The general approach follows \cite{reiris} by performing the change of variables \( f = -\ln u \) and computing the Bochner formula for the \( f \)-Laplacian of \( |\di\varphi|^{2} \). However, Reiris obtained the desired Liouville theorem via gradient estimates which do not seem easy to extend to manifolds with boundary. For this reason, we proceed differently by means of integral maximum principles at infinity in the spirit of \cite{rigoli2}, which we shall apply to the energy density $|\di \varphi|^2$. In general, however, such results do not hold when $\partial M \neq \emptyset$ without growth assumptions on $|\di \varphi|^2$ on $\partial M$, a request we would rather avoid. The insight here is that the equality $u=0$ on $\partial M$ allows, perhaps surprisingly, to cancel boundary integrals. \vspace{0.5cm} The paper is organized as follows. In Section 2, we discuss the equivalence between (NEC) and \eqref{sub_static}, and review the classical examples of sub-static triples. In Section 3, we prove Theorem \ref{local_split}. In Section 3, we establish some Shen's type identities and prove Theorem \ref{thm_BGH}. The concluding Section 4 will be devoted to the proof of Theorem \ref{vanishing_map}. \section{Preliminaries} In this work, we consider a static spacetime $(\hat M, \hat g)$ of dimension $m+1$, where \begin{equation}\label{static_space} \hat M = \R \times M \, , \qquad \hat g = - u^2 \di t \otimes \di t + g, \end{equation} for $(M^m,g)$ a Riemannian manifold and $u \in C^\infty(M)$ a positive function. In this setting, the components of the Ricci tensor are given by \begin{equation}\label{Ric_hat} \left\{\begin{array}{r@{\;}c@{\;}l} \hat R_{00} & = & \dfrac{\Delta u}{u} \\[0.3cm] \hat R_{0i} & = & 0 \\[0.2cm] \hat R_{ij} & = & R_{ij} - \dfrac{u_{ij}}{u}. \end{array} \right. \end{equation} Furthermore, the scalar curvatures satisfies \begin{equation}\label{scal_hat} S_{\hat g} = S -\frac{2}{u}\Delta u \, . \end{equation} We particularly investigate a static spacetime that satisfies the Einstein field equation, expressed as \begin{equation}\label{Einst_eq} \Ric_{\hat{g}}+\left(\Lambda-\frac{1}{2}S_{\hat{g}}\right)\hat{g}=T, \end{equation} where $T$ represents a stress-energy tensor, and $\Lambda$ denotes the cosmological constant. When expressed in coordinates, we can use \eqref{Ric_hat} and \eqref{scal_hat} to compute the components of the stress-energy tensor as follows: \begin{equation}\label{energy} \left\{\begin{array}{r@{\;}c@{\;}l} T_{00} & = & -\Lambda+\dfrac{S}{2} \\[0.4cm] T_{0i} & = & 0 \\[0.2cm] T_{ij} & = & R_{ij} - \dfrac{u_{ij}}{u}+\left(\Lambda-\dfrac{S}{2}+\dfrac{\Delta u}{u}\right)g_{ij} \end{array} \right. \end{equation} \begin{definition} A Lorentzian manifold $(\hat{M},\hat{g})$ satisfies the \textit{null energy condition} (NEC) if $T(Y,Y)\geq 0$ for all null vectors $Y$ (meaning $\hat{g}(Y,Y)=0$). \end{definition} To relate (NEC) to the sub-staticity of $(M,g,u)$, we follow the computations outlined, for instance, in \cite[Lemma 3.8]{wang}. Up to scaling, we can consider a null vector $Y = \widehat{e_0} + X$, where $\widehat{e_0} = \frac{1}{u}\frac{\partial}{\partial t}$ and $X \in \mathscr{X}(M)$ satisfies $g(X,X)=1$. From \eqref{static_space} and $\hat{g}(\widehat{e_0},\widehat{e_0})=-1$, applying the null energy condition (NEC) to $Y$ yields: \begin{eqnarray*} 0\leq T(Y,Y)&=&T_{00}+T_{ij}X^{i}X^{j}\\ &=&\left(-\Lambda+\frac{S }{2}\right)+\left[\Ric -\frac{\Hess u}{u}+\frac{\Delta u}{u}g\right](X,X)+\left(\Lambda-\frac{S }{2}\right)g(X,X)\\ &=&\left[\Ric -\frac{\Hess u}{u}+\frac{\Delta u}{u}g\right](X,X), \end{eqnarray*} which is \eqref{eq_Q}. Taking traces in \eqref{sub_static}, the scalar curvature $S$ of $M$ relates to $u$ as follows: \begin{equation}\label{sub_trace} \Delta u = \frac{u}{m-1} (\trace Q-S ), \end{equation} Sub-static triples include the following classical examples. \begin{example}[\textbf{Vacuum static system}] The simplest example arises from the Vacuum Static equation, where $T$ (and consequently $Q$) vanishes. In this case, a triple $(M^m, g, u)$ that satisfies \begin{equation}\label{eq_vacuum_static} u \Ric - \Hess u + (\Delta u) g = 0 \end{equation} is called a \textit{vacuum static triple}. As stressed in \cite{fm75}, the left hand-side of vacuum static equation also appears as the formal adjoint of the linearized scalar curvature (up to sign), and therefore ties to the scalar curvature prescription problem. For more details, see also \cite[Section 4.2]{corvino}. \end{example} \begin{example}[\textbf{Electrostatic system}] Given a function $\eta \in C^{\infty}(M)$ and a triple $(M^m, g, u)$, we consider $$ Q := \frac{1}{u^2} \left(|\nabla \eta|^2 g - \di \eta \otimes \di\eta \right). $$ Clearly, $Q \geq 0$ by the Cauchy-Schwarz inequality. This tensor gives rise to the so-called Electrovacuum static system, and $\eta$ relates to the electric potential (see, for example, \cite[Definition 1]{andrade}). For details of this construction from a specific stress-energy tensor, see \cite{tiarlos}. \end{example} \begin{example}[\textbf{Maps source}] Let $\hat{M}^{m+1}$ be spacetime and $(N^n, h)$ a Riemannian manifold. If $\Phi: (\hat{M}^{m+1}, \hat{g}) \to (N^n, h)$ and $V: (N^n, h) \to \mathbb{R}$ are smooth maps, we consider the following stress-energy tensor: $$T = \Phi^* h - \frac{1}{2} \left( |\di \Phi|_{\hat{g}}^2 + (m-1) V(\Phi) \right) \hat{g},$$ where $\Phi^* h$ is the pull-back of $h$. In this case, the matter is described by a ``wave map'' $\Phi$ and a scalar potential $V$ (for details, see Section \ref{sec_map} and also \cite{CB,reiris,ans21}). Einstein's equation is then equivalent to $$ \Ric_{\hat{g}} = \Phi^* h + V(\Phi) \hat{g}. $$ In the static case, we assume that $\Phi$ factorizes as $\Phi = \phi \circ \pi_M$, with $\pi_M: \hat{M} \to M$ the natural projection and $\varphi: M \to (N^n, h)$. Then, the system rewrites as \begin{equation*} \left\{\begin{array}{r@{\;}c@{\;}l} u\Ric - \Hess u + (\Delta u)g & = & u\varphi^* h \\[0.2cm] -\Delta u & = & V(\phi) u, \\[0.2cm] \end{array} \right. \end{equation*} thus $Q = \varphi^* h$ is a positive definite tensor, and $(M^m, g, u)$ forms a sub-static system. \end{example} \begin{example}[\textbf{Perfect fluid}] In the theory of perfect fluids for a spacetime $(\hat{M}^{m+1}, \hat{g})$ (see, for example, \cite[Chapter III, Section 8]{CB}), the stress-energy tensor is given by $$T = (\rho + P) v \otimes v + P \hat{g},$$ where $\rho$ is the energy density, $P$ is the pressure, and $v$ is a unit time-like covector field that represents the fluid's velocity. In particular, if $(M^{m+1}, \hat{g})$ is static as in \eqref{static_space}, we can write $T = \rho u^2 \di t^2 + P g$. Einstein's equation then reduces to \begin{equation*} \left\{\begin{array}{r@{\;}c@{\;}l} R_{ij} - \frac{u_{ij}}{u} + \left(\Lambda - \frac{S }{2} + \frac{\Delta u}{u}\right) g_{ij} & = & P g_{ij} \\[0.2cm] -\Lambda + \frac{S }{2} & = & \rho, \end{array} \right. \end{equation*} and hence, $$Q = (\rho + P) g.$$ Therefore, $(M, g,u)$ is a sub-static triple if $\rho + P \geq 0$. \end{example} \section{Compact minimal hypersurfaces and splitting in sub-static manifolds} In this section, we describe the connection between stable minimal hypersurfaces in sub-static systems and $f$-minimal stable hypersurfaces in manifolds with $\overline{\Ric}_f^1 \geq 0$. This approach allows us to provide simpler proofs and extend the analysis presented in \cite{galloway} and \cite{hmm}, where static systems were thoroughly studied (see also \cite[Section 4.1]{tiarlos} and \cite{benedito} for electrostatic systems). Our focus is on the rigidity problem for sub-static triples $(M^m,g,u)$ that admit a compact, minimal hypersurface contained within the interior $\mathring{M} = \{u>0\}$. \begin{remark} If the set $\partial M= u^{-1}(0)$ is supposed to be a minimal hypersurface (for instance, in the case where $\partial M$ is a horizon boundary or if $Q$ extends continuously to $\partial M$), as a consequence of the strong tangency principle for minimal hypersurface any connected minimal hypersurface either is a component of $\partial M$ or lies in $\Mo$. \end{remark} Let $\bar g = u^{-2}g$, which is often called the optical metric in the physical literature. Choose $f = - (m-1)\ln u$. Then, the formulas relating the Ricci and Hessian tensors in the metrics $g$ and $\bar g$ yield \begin{equation}\label{eq_link_Q_Ricpsi} \disp \overline{\Ric}_{f}^1 =\Ric - \frac{\Hess u}{u}+\frac{\Delta u}{u}g\doteq Q. \end{equation} For details, see \cite{fogborg}. Whence, the sub-static condition is equivalent to \[ \overline{\Ric}_f^1 \ge 0. \] Let $\Sigma \to (M,g)$ be a compact immersed hypersurface. Denote by $\di x$ and $\di \bar x$, respectively, the volume measures of $g$ and $\bar g$, and by $\di \sigma$, $\di \bar \sigma$ induced volume densities on $\Sigma$. Note that $\di \bar x = u^{-m}\di x$, $\di \bar \sigma = u^{1-m}\di \sigma$. The natural weighted densities associated to $\overline{\Ric}_f^1$ are \[ \di \bar x_f \doteq e^{-f}\di \bar x, \qquad \di \bar \sigma_f \doteq e^{-f} \di \bar \sigma \equiv \di \sigma. \] The latter identity implies that the weighted $(m-1)$-area of $\Sigma$ in $(M, \bar g)$ (which we name the $f$-area) is given by \begin{equation}\label{eq_magic} \overline{\vol}_f(\Sigma) \doteq \int_{\Sigma} \di \bar \sigma_f = \int_{\Sigma} \di \sigma = \vol(\Sigma). \end{equation} Denote by $\nu$ a local $g$-unit normal field along $\Sigma$, and $\bar \nu = u \nu$ the corresponding $\bar g$-unit normal. Stationary points of $\overline{\vol}_f$ are called $f$-minimal hypersurfaces, and are characterized by the vanishing of the $f$-mean curvature \[ \bar H_f \doteq \bar H + \di f(\bar \nu), \] where $\bar H$ is the mean curvature of $\Sigma \to (M, \bar g)$ in direction $\bar \nu$. The identity \eqref{eq_magic} establishes that \[ \begin{array}{c} \text{$\Sigma \to (M,g)$ is minimal}\\ \text{(resp. stable minimal,}\\ \text{or area minimizing)} \end{array} \qquad \Longleftrightarrow \qquad \begin{array}{c} \text{$\Sigma \to (M,\bar g,e^{-f}\di \bar x)$ is $f$-minimal}\\ \text{(resp. stable $f$-minimal,}\\ \text{or $f$-area minimizing)} \end{array} \] The second fundamental forms $A$ of $\Sigma \to (M,g)$ and $\bar{A}$ of $\Sigma \to (M,\bar{g})$ (in the directions $\nu$ and $\bar{\nu}$, respectively) satisfy the following relation: \begin{equation}\label{second_conf} \bar{A}(X,Y) = u^{-1} \left[ A(X,Y) +\di \ln u(\nu) g(X,Y) \right] \qquad \forall \, X,Y \in TM. \end{equation} Taking traces, the corresponding mean curvatures relate as follows: \[ \bar H = u \left[ H + (m-1) \di \ln u(\nu) \right]. \] Whence, the $f$-mean curvature of $\Sigma \to (M, \bar g)$ is \begin{equation}\label{eq_barHf_H} \bar H_f = u \left[ H + (m-1) \di \ln u(\nu) \right] - (m-1)u \di \ln u(\nu) = uH. \end{equation} in accordance to \eqref{eq_magic}. Assume now that $\Sigma \to (M,g)$ is a two-sided minimal immersion. The $f$-stability operator $\bar L_f$ of $\Sigma$, viewed as an $f$-minimal hypersurface in $(M, \bar g)$, is given by $$ \bar L_f \doteq \bar{\Delta}_f + \|\bar{A}\|^{2}+\overline{\Ric}_f(\bar{\nu},\bar{\nu}), $$ where $\|\cdot\|$ is the $\bar g$-norm (see for example \cite{detang}). From \eqref{second_conf} and the minimality of $\Sigma$ in $(M,g)$, we compute \begin{eqnarray*} \|\bar{A}\|^{2}&=& u^{2}[|A|^{2}+(m-1)(\di \ln u(\nu))^{2}]\\ &=&u^{2}|A|^{2}+\frac{1}{m-1}(\di f(\bar{\nu}))^{2}, \end{eqnarray*} hence, using \eqref{eq_link_Q_Ricpsi}, \begin{equation}\label{l_psi_stability} \bar L_f = \bar{\Delta}_f +u^{2}|A|^{2} + \overline{\Ric}_f^1(\bar{\nu},\bar{\nu}) = \bar{\Delta}_f +u^{2}\big[|A|^{2} + Q(\nu,\nu)\big]. \end{equation} We remember that a two-sided $f$-minimal hypersurface $\Sigma$ is said to be $f$-stable if \begin{equation}\label{stability_inequality} 0 \le -\int_{\Sigma}\psi (\bar L_f \psi) \di \bar x_f = \int_\Sigma \Big\{ \|\di \psi\|^2 - u^{2}\big[|A|^{2} + Q(\nu,\nu)\big]\psi^2\Big\} \di \bar x_f, \qquad \forall \, \psi \in C^\infty_c(\Sigma). \end{equation} This parallelism allows us to provide a short proof of the following result that was shown by Huang-Martin-Miao \cite{hmm} in the static case. \begin{proposition}\label{tot_geod} Let $\Sigma \to (M,g)$ be a closed, two-sided, stable minimal hypersurface in a sub-static manifold $(M^{m},g,u)$. Then $\Sigma$ is totally geodesic and $Q(\nu,\nu) = 0$ along $\Sigma$. \end{proposition} \begin{proof} Substituting $\psi = 1$ into the stability inequality \eqref{stability_inequality} and using \eqref{l_psi_stability}, we obtain \[ 0 \leq -\int_{\Sigma} u^{2} \big[ |A|^{2} + Q(\nu,\nu) \big] \leq 0, \] which implies $|A|^{2} + Q(\nu,\nu) \equiv 0$. \end{proof} Denote by $\varsigma : \Sigma \to (M,g)$ the isometric immersion. We next prove (A) in Theorem \ref{local_split}, a local splitting theorem first proved in \cite{hmm} in the static case. \begin{definition}\label{def_localmin} Hereafter, we say that $\varsigma$ is locally area-minimizing if there exists a neighbourhood $V \supset \varsigma(\Sigma)$ such that for every variation $\{\varsigma_t\}$ of $\varsigma$ with image in $V$ and compactly supported variation vector field, the hypersurfaces $\Sigma_t = (\Sigma, \varsigma_t^*g)$ satisfy $\vol_f(\Sigma_t) \ge \vol_f(\Sigma)$. \end{definition} In the stated assumption, $\varsigma : \Sigma \to M$ can also be viewed as an $f$-area minimizing hypersurface in $(M, \bar{g}, e^{-f} d\bar{x})$. Let $\bar{\nu}$ be a choice of $\bar g$-unit normal field along $\Sigma$, and define \begin{equation}\label{def_Phi_t} \Phi : \Sigma \times (-\eps,\eps) \to M, \qquad \Phi(x, t) = \overline{\exp}_{\varsigma(x)}\big(t \bar{\nu}(x)\big), \qquad \Sigma_t = \Phi(t,\Sigma), \end{equation} where $\eps$ is small enough so that $\Phi_t$ is an immersion for each $t$ and $\Sigma_0$ is $f$-area minimizing in $\Phi(\Sigma \times( -\eps, \eps))$. Along this flow, we use \eqref{l_psi_stability} to obtain the following at the point $(x,t)$: \[ \frac{\di}{\di t}\bar{H}_{f} = \bar{L}_f 1 = u^{2} \left( |A|^{2} + Q(\nu, \nu) \right) \geq 0. \] Therefore, since $\bar{H}_f = 0$ at $t = 0$, we have $\bar{H}_f(\cdot, t) \geq 0$ for all $t \in [0, \varepsilon)$. In particular, we have: \[ \overline{\vol}_f(\Sigma_{t}) - \overline{\vol}_f(\Sigma) = \int_0^t \left( - \int_{\Sigma_s} \bar{H}_f(\cdot, s) \, d\bar{\sigma}_f \right) ds \leq 0. \] This inequality cannot be strict for any $t \in (0, \varepsilon)$ because $\Sigma_0$ is $f$-area minimizing. Hence, $\bar{H}_f(\cdot, t) \equiv 0$ and $\overline{\vol}_f(\Sigma_t) = \overline{\vol}_f(\Sigma)$ for all $t \in [0, \varepsilon)$. The case $t \in (-\eps,0]$ is analogous. Thus, Proposition \ref{tot_geod} implies the following result in terms of the metric $g$: \begin{proposition}\label{lam_split} Consider $(M^m, g, u)$ a sub-static system, and let $\Sigma$ be a locally area-minimizing, closed, connected two-sided minimal hypersurface where $u > 0$. Then, the family $\{\Sigma_t\}$ described above is totally geodesic, the area of $\Sigma_t$ is constant, and $Q(\nu_t, \nu_t) = 0$. \end{proposition} We are ready to prove (A) in Theorem \ref{local_split} in its more general form (see Remark \ref{rem_covering}). For the definition of a minimizer in Almgren's sense, see \cite{almgren76,taylor76}. \begin{theorem}\label{teo_splitting_1} Let $(M^{m}, g, u)$ be a sub-static triple and $\varsigma : \Sigma \to M$ be a closed, connected two-sided minimally immersed hypersurface such that $u>0$ on $\Sigma$. Assume that $\Sigma$ is locally area minimizing. Then, \begin{itemize} \item[(i)] If $\Sigma$ is embedded, there exists $\eps>0$ and a diffeomorphism $\Phi : (-\eps,\eps) \times \Sigma \to M$ such that, in coordinates $(s,y) \in (-\eps,\eps) \times \Sigma$, \[ \Phi^* g = r^{2}(y)\di s^2 + h^{\Sigma}, \] where $h^\Sigma$ is the induced metric on $\Sigma \to (M,g)$ and $r : \Sigma \to (0,\infty)$. Moreover, there exists a function $\sigma : (-\eps,\eps) \to (0,\infty)$ such that $u(\Phi(s,y)) = r(y)\sigma(s)$. \item[(ii)] If $\varsigma(\Sigma)$ is a minimizer in Almgren's sense, then $\varsigma$ factorizes as $\hat\varsigma \circ \pi$ with $\pi : \Sigma \to \hat \Sigma$ a Riemannian covering and $\hat\varsigma : \hat \Sigma \to M$ an embedding. Moreover, $\hat\Sigma$ is locally area-minimizing and thus (i) holds for $\hat\Sigma$. \end{itemize} \end{theorem} \begin{proof} (i). Define $\Phi$, $\Sigma_t$, $\eps$ as in \eqref{def_Phi_t}, and choose $\eps$ so that $\Phi$ is a diffeomorphism onto a tubular neighbourhood of $\Sigma_0$. Proposition \ref{lam_split} asserts that $\Sigma_{t}$ is totally geodesic. The identity \eqref{second_conf} shows that $\Sigma_{t}$ is $\bar{g}$-totally umbilical, with second fundamental form \begin{equation}\label{eq_At} \bar{A}(X,Y) = \frac{1}{u} \di \ln u(\nu) g(X,Y) = \frac{\di u}{u}(\bar{\nu}) \bar{g}(X,Y). \end{equation} By the definition of $\Phi$, $\Sigma_{t}$ are the level sets of the $\bar{g}$-distance. Choose local coordinates $\{ y^\alpha\}$ on $\Sigma$ and write $\bar{h}^\Sigma = \bar{h}^\Sigma_{\alpha \beta} \di y^\alpha \otimes \di y^\beta$ for the induced metric on $\Sigma \hookrightarrow (M,\bar g)$. Write also \[ \Phi^*\bar g = \di t^2 + \bar{h}_{\alpha\beta} \di y^\alpha \otimes \di y^\beta. \] Along the normal flow $\Phi$, we have the known variation formula \[ \frac{\partial}{\partial t} \bar h_{\alpha\beta}(t,y) = -2\bar{A}_{\alpha\beta} = -2\frac{\partial}{\partial t}(\ln u) \bar h_{\alpha\beta}(t,y), \] and the initial condition $\bar h_{\alpha\beta}(0,y) = \bar h_{\alpha\beta}^{\Sigma}(y)$. Solving this ODE yields \[ \bar h_{\alpha\beta}(t,y) = \left(\frac{u(t,y)}{u(0,y)}\right)^{-2} \bar h_{\alpha\beta}^{\Sigma}(y), \] therefore we obtain \begin{equation}\label{eq_nice_split} \Phi^*\bar{g} = \di t^{2} + \left(\frac{u(t,y)}{u(0,y)}\right)^{-2} \bar h^{\Sigma}_{\alpha\beta}(y) \di y^\alpha \otimes \di y^\beta. \end{equation} From $\overline{\mathrm{Ric}}_f^1 \geq 0$ and applying a result of Wylie (see \cite[Proposition 2.2]{wylie1}) we have that \[ u(t,y) = r(y)\xi(t). \] (without loss of generality, $\xi(0)=1$). Therefore, coming back to the metric $g$, we have \[ \Phi^*g = r^2(y)\xi^2(t)\di t^{2} + h^{\Sigma}_{\alpha\beta}(y) \di y^\alpha \otimes \di y^\beta, \] where $h_\Sigma$ is the induced metric on $\hat\Sigma \hookrightarrow (M,g)$. Choosing \[ s(t) = \int_0^t \xi(\tau) \di \tau, \qquad \sigma(s) = \xi(t(s)), \] we conclude the desired expression for $g$.\\[0.2cm] (ii) Assume that $\varsigma$ is not an embedding, so by compactness $\varsigma$ is not injective. Pick distinct points $x_1,x_2 \in \Sigma$ with $\varsigma(x_1)=\varsigma(x_2) = p$, and let $\delta < \min\{{\rm inj}(\Sigma),{\rm inj}(p)\}$ such that $\varsigma : B_\delta(x_j) \to M$ is a diffeomorphism onto its image. Up to composing with the inverse of the exponential map of $M$ at $p$, since $\Sigma$ is totally geodesic the images $V_j = \varsigma(B_\delta(x_j))$ are hyperplanes in $B_\delta(p)$. If they are transverse, then $\varsigma(\Sigma)$ cannot be a minimizer in the sense of Almgren. In fact, the local picture around codimension $1$ singularities of such minimizers can only consist of three hypersurfaces meeting at $120$ degrees (see Theorem 8.1 and subsequent discussion in \cite{delellis}). Hence, $V_1 \equiv V_2$ and $\hat \Sigma \doteq \varsigma(\Sigma)$ can therefore be given the structure of a smooth embedded manifold, with local charts $\exp_{x_j}^{-1} \circ \varsigma^{-1} : V_j \to \R^{m-1}$, and we can write $\varsigma = \hat \varsigma \circ \pi$, where $\hat\varsigma : \hat\Sigma \to M$ is the inclusion. The induced metric via $\hat \varsigma$ makes $\pi$ a local isometry. Since $\Sigma$ is compact, Ambrose's theorem implies that $\pi$ is a Riemannian covering. To conclude that $\hat \Sigma$ is locally area minimizing, simply observe that each variation $\hat\varsigma_t$ of $\hat\Sigma$ induces a variation $\varsigma_t = \hat \varsigma_t \circ \pi$ of $\Sigma$, and that $\vol(\Sigma_t) = k \vol(\hat\Sigma_t)$, with $k$ the number of sheets of $\pi$. \end{proof} \begin{remark} If $M$ is orientable, the assumption that $\varsigma(\Sigma)$ is minimizing in the sense of Almgren can be replaced by the requirement that $\Sigma$ is a mass-minimizing current. Note that $\Sigma$, being $2$-sided, is orientable as well. Indeed, the singular set of mass minimizing currents has Hausdorff codimension $7$, cf \cite{delellis}. \end{remark} In general, the above local splitting theorem cannot be extended to a global one, as shown by the simple example of a right cylinder $[-T,T] \times \mathbb{S}^{n-1}$ with two spherical caps attached to its boundaries in such a way that the resulting manifold has non-negative Ricci curvature (a sub-static manifold with $u \equiv 1$). Global splitting theorems were recently obtained in \cite[Theorems C, 3.7, 3.8 and Corollary 3.9]{fogborg}, and (B) in Theorem \ref{local_split} provides a further result in this direction. We begin by commenting on Definition \ref{def_u_complete}. \begin{remark}\label{rem_ucomplete} Assume that $E$ is isometric to the interior of a manifold with boundary $E^*$. Since, by standard theory, the completeness of a given metric on $E^*$ is equivalent to the request that every diverging curve (i.e. eventually leaving any compact set) has infinite length, Definition \ref{def_u_complete} can be rephrased as the completeness of $(E^*, u^{-2}g)$ and of $(E^*, u^2g)$. By the Hopf-Rinow's theorem, the first condition in Definition \ref{def_u_complete} is also equivalent to say that \begin{equation}\label{ucomp_fogborg} \lim_{t\to \infty}\rho(\gamma(t))= \infty, \end{equation} where $\rho$ is the distance from a point in the metric $u^{-2}g$. In the setting of the present remark, our definition of $u$-completeness therefore agrees with the one in \cite[Definition 3.4]{fogborg}. \end{remark} The following is a sufficient condition for the $u$-completeness of $E$, which improves on \cite[Proposition 3.2]{fogborg} \begin{proposition}\label{prop_suffcond} Let $(M^m,g,u)$ be a substatic triple and $E\subset M$ is an end. If \[ C^{-1}(1 + r)^{-1} \le u \le C( 1+ r) \qquad \text{on } \, E, \] for some constant $C>0$, where $r$ is the distance to a fixed compact set of $M$, then $E$ is $u$-complete. \end{proposition} \begin{proof} Let $K$ be the compact set in the statement. Suppose that $\gamma : [0,\infty) \to E$ be a unit speed divergent curve in $E$, say starting from a point of $\partial E$. By the triangle inequality, \[ t \ge \di (\gamma(t),\gamma(0)) \ge \di(\gamma(t), \partial E) \ge r(\gamma(t)) - \di(K, \partial E). \] Therefore, since in our assumption both $u(\gamma(t))$ \and $u^{-1}(\gamma(t))$ are larger than $C^{-1}(1+r(\gamma(t)))^{-1}$, and since \[ \int_0^\infty \frac{\di t}{1+r(\gamma(t))} \ge \int_0^\infty \frac{\di t}{1+ t + \di(K, \partial E)} = \infty, \] the thesis readily follows. \end{proof} We are ready to prove (B) in Theorem \ref{local_split}. \begin{theorem}\label{teo_splitting_2} Let $(M^{m}, g, u)$ be a sub-static triple, and assume that $\Sigma$ is a (possibly disconnected) compact, embedded, two-sided stable minimal hypersurface contained in $\Mo$. If an end $E$ with respect to $\Sigma$ is $u$-complete, then: \begin{itemize} \item[(i)] the topological boundary $\partial E \subset M$ is a single connected component of $\Sigma$, and it locally separates $E$ from $M \backslash E$. \item[(ii)] the closure $\overline{E} \subset M$ is an embedded submanifold isometric to \[ [0,\infty) \times \partial E \qquad \text{with metric} \qquad g = r^{2}(y)\di s^2 + h^{\Sigma}, \] where $h^\Sigma$ is the induced metric on $\Sigma \to (M,g)$ and $r : \Sigma \to (0,\infty)$. Moreover in coordinates $(s,y)\in [0,\infty) \times \partial E$, it holds $u(s,y) = r(y)\sigma(s)$ on $E$ for some function $\sigma : [0,\infty) \to (0,\infty)$. \end{itemize} \end{theorem} \begin{proof} The proof follows the methods in \cite{kasue,wylie1,fogborg} but needs extra arguments in order to deal with the presence of a boundary. In our assumptions, setting $\bar g = u^{-2}g$ we have that $(\bar{E},\bar g)$ is a complete metric space but in principle $(\mathring{M}, \bar g)$ may not be complete. Moreover, a component of the topological boundary $\partial E \subset M$ may not separate $E$ from $M \backslash E$. For this reason, we consider the metric completion $E^*$ of $(E,\bar g)$. Since $E$ is $u$-complete, $E^*$ is a manifold with boundary and $\partial E^*$ is isometric to the union of some of the connected components of $\Sigma$, possibly repeated twice. Indeed, components of $\Sigma$ that do not locally separate $E$ from $M \backslash E$ appear twice as components of $\partial E^*$. By Proposition \ref{tot_geod}, $\Sigma$ (and thus $\partial E^*$) is totally geodesic. Hereafter, we identify $E$ with the interior of $E^*$. A $\bar g$-minimizing unit speed Lipschitz curve in $(E^*,\bar g)$ will be called a $\bar g$-segment. By the completeness of $(E^*,\bar g)$, any pair of points of $E^*$ is joined by a $\bar{g}$-segment, which by \cite{aleale} is $C^1$ and clearly a $\bar g$-geodesic whenever it lies in $E$. Note the following property: \begin{equation}\label{eq_P}\tag{P} \text{every $\bar g$-segment $\gamma$ in $E^*$ joining points of $E$ lies in $E$.} \end{equation} indeed, if by contradiction $\gamma^{-1}(\partial E^*) \neq \emptyset$ let $a = \inf \gamma^{-1}(\partial E^*) > 0$. Then, $\gamma$ restricted to $[0,a]$ is a $\bar g$-geodesic between its endpoints. As $\gamma \in C^1$, $\gamma'(a) \in T\partial E^*$. Since $\partial E^*$ is totally geodesic, $\gamma([0,a]) \subset \partial E^*$, contradiction. Let $\Gamma \subset E^*$ be either a point $p \in E$ or $\partial E^*$. We shall apply the weighted laplacian comparison theorems in \cite{wylie1,wylie} to the $\bar{g}$-distance to $\Gamma$ in the complete manifold $(E^*,\bar g)$: \[ \rho : E^* \to (0,\infty), \quad \rho(x) \doteq \di_{\bar{g}}(x,\Gamma) \] However, a preliminary analysis of the regularity of $\rho$ is needed. \\[0.2cm] \noindent \textbf{Step 1}: regularity of $\rho$.\\[0.2cm] If $\Gamma = \partial E^*$, things are well-known: $\rho$ is smooth on a relatively open subset $U \subset E^*$ containing $\partial E^*$, where the normal exponential map from $\partial E^*$ realizes a diffeomorphism; moreover, $U$ contains the interior of any segment from $\partial E^*$ to points in $E$, and $E^* \backslash U= {\rm cut}(\Gamma)$. We check an analogous property for the case $\Gamma = \{p\}$. For each vector $v \in T_{p}M$, define $\gamma_v(t) = \overline{\exp}_p(tv)$ whenever the latter is defined, and set \[ \begin{array}{l} \seg(p) = \left\{ v \in T_xM : \begin{array}{l} \gamma_v \ \text{is defined on $[0,1]$}, \ \gamma_v([0,1]) \subset \overline{E}\\ \text{and $\gamma_v$ is a $\bar g$-segment on $[0,1]$} \end{array}\right\} \\[0.3cm] \seg^0(p) = \Big\{ tv : v \in \seg(p), \ t \in [0,1)\Big\} \end{array} \] If $v \in \seg(p)$, again because $\partial E^*$ is totally geodesic we deduce $\gamma_v([0,1)) \subset E$. This fact, together with the completeness of $(E^*,\bar g)$ and property (P), implies \[ \overline{\exp}_p\big(\seg(p)\big) = E^*, \qquad \overline{\exp}_p\big(\seg^0(p)\big) \subset E. \] Crucially using property (P) again, minor adaptations of \cite[Section 5.7.3]{petersen} enable to prove the following: \begin{itemize} \item[-] $\overline{\exp}_p$ is injective on $\seg^0(p)$; \item[-] the differential $(\overline{\exp}_p)_*$ is nonsingular at each point of $\seg^0(p)$; \item[-] if $v \in \seg(p)\backslash \seg^0(p)$, then either $\overline{\exp}_p(v) \in \partial E^*$, or there exists $w \neq v$ with $\overline{\exp}_p(w) =\overline{\exp}_p(v)$, or $(\overline{\exp}_p)_*$ is singular at $v$. \item[-] $\seg^0(p)$ is open. Consequently, $\overline{\exp}_p$ is a diffeomorphism between $\seg^0(p)$ and its image $U \subset E$, the open set we search for. By construction, $\rho(x) = \|\overline{\exp}_p^{-1}(x)\|$ is smooth on $U\backslash \{p\}$. We define the cut-locus ${\rm cut}(p) = E \backslash U$. \end{itemize} \vspace{0.2cm} \noindent \textbf{Step 2}: $f$-Laplacian comparison for $\rho$.\\[0.2cm] Suppose that $x \in U$, and let $\gamma : [0,\rho(x)] \to E^*$ be a $\bar g$-segment joining $\Gamma$ to $x$. By property (P) and the above, $\gamma((0,\rho(x)]) \subset U$ thus $\rho$ is smooth therein. Writing \[ \bar H(t) \doteq \bar \Delta\rho (\gamma(t)), \] it is known that $\bar H$ is the mean curvature of the level set $\{\rho = t\}$ at the point $\gamma(t)$ in direction $-\bar \nabla \rho = -\gamma'$, and that along $\gamma$ the following Riccati equation holds: $$ \bar{H}'+\frac{\bar{H}^2}{m-1}+ \overline{\Ric} (\gamma',\gamma')= 0, $$ where $'$ means derivative with respect to the $\bar g$-arclength $t$. The sub-static condition implies $$ \bar{H}'+\frac{\bar{H}^{2}}{m-1}- \overline{\Hess} f(\gamma',\gamma')-\frac{1}{m-1}[(f \circ \gamma)']^{2} \leq 0. $$ Observing that $$ \bar{H}_f = \bar{H} -(f \circ \gamma)', \qquad \overline{\Hess} f(\gamma',\gamma') = (f \circ \gamma)'' $$ we have (leaving implicit the restriction to $\gamma$ for the ease of notation) \begin{eqnarray*} \bar{H}_f'=\bar{H}'-f''&\leq&-\frac{\bar{H}^{2}}{m-1}+f''+\frac{1}{m-1}(f')^{2}-f''\\ &=&-\frac{1}{m-1}\left[\bar{H}_f+ f'\right]^{2}+\frac{1}{m-1}(f')^{2}\\ &=&-\frac{\bar{H}_f^{2}}{m-1}-\frac{2}{m-1}f'\bar{H}_f. \end{eqnarray*} This implies that the function \[ \lambda(t)=e^{\frac{2f(\gamma(t))}{m-1}}\bar{H}_f(\gamma(t)) \] solves $$ \frac{\di \lambda}{\di t} \leq- e^{-\frac{2f}{m-1}}\frac{\lambda^{2}}{m-1}. $$ Changing variables according to \[ s(t) = \int_0^t e^{-\frac{2f(\gamma(\tau))}{m-1}}\di \tau = \int_0^t u^2(\gamma(\tau)) \di \tau \] we rewrite the above inequality as \[ \frac{\di \lambda}{\di s} \leq -\frac{\lambda^{2}(s)}{m-1}. \] One may now apply the Riccati comparison in two differente cases: \begin{itemize} \item[-] if $\Gamma = \{p\}$, then from $\bar H(\gamma(t)) \sim \frac{m-1}{t}$ as $t \to 0$ we get \[ \lambda(s) \sim \frac{m-1}{s} \qquad \text{as } \, s \to 0. \] Then, Riccati comparison gives \[ \lambda(s) \le \frac{m-1}{s}, \qquad \text{namely} \qquad \bar \Delta_f \rho (\gamma(t(s)) \le e^{\frac{-2f(\gamma(t(s)))}{m-1}} \frac{m-1}{s} \] and, because of our choice of $f$, at the point $x$ it holds \begin{equation}\label{eq_point} \bar \Delta_f \rho (x) \le u(x)^2 \frac{m-1}{s(\rho(x))} \end{equation} \item[-] If $\Gamma$ is a hypersurface, denote by $o = \gamma(0)$. From \[ \lambda(0) = e^{\frac{2f(o)}{m-1}}\bar H_f(o), \] Riccati comparison (see \cite[Theorem 3.8]{bmpr}) gives $s < s^* \doteq \frac{m-1}{-\lambda(0)}$ if $\lambda(0)<0$ and, regardless the sign of $\lambda(0)$, \[ \lambda(s) \le \frac{\lambda(0)}{1+ \frac{\lambda(0)}{m-1}s} \le \lambda(0) \qquad \text{on } \, [0,s^*). \] The inequality rewrites as \[ \bar \Delta_f \rho (x) \le e^{\frac{2[f(o)-f(x)]}{m-1}}\bar H_f(o) \] and, due to our choice of $f$ and to the minimality of $\Gamma = \Sigma_1$, \begin{equation}\label{eq_Lapla_ipers} \bar \Delta_f \rho (x) \le \left( \frac{u(x)}{u(o)}\right)^2 \bar H_f(o) = 0. \end{equation} \end{itemize} Assume now that $x$ belongs to ${\rm cut}(\Gamma)$, we shall prove that \eqref{eq_point} and \eqref{eq_Lapla_ipers} hold in the barrier sense at $x$. If $\Gamma$ is a point, by Calabi's trick, we can consider $\varepsilon>0$ and the function $$ \rho_{\varepsilon}\doteq\varepsilon+ \di_{\bar{g}}(\cdot,\gamma(\varepsilon)), $$ which satisfies $\rho_{\varepsilon}\geq\rho$ in $M$ with equality at $x$. The smoothness of $\rho_\eps$ at $x$ is proved as in the boundaryless case. If $\Gamma = \partial E^*$, we can proceed as follows: we enlarge $E^*$ to a boundaryless manifold $V$ by gluing a collar neighbourhood $T \approx (0,\delta] \times \partial E^*$ of $\partial E^*$, and smoothly extend $\bar g$, $u$ to the entire $V$ in such a way that $u>0$ on $V$ and $(V,\bar g)$ is a complete manifold. Since $\partial E^*$ separates $V$, for $\varepsilon >0$ we can consider a compact hypersurface ${\Gamma}_{\varepsilon}\subset V\backslash E$ touching $\partial E^*$ at $o$ and whose second fundamental form $A_{\varepsilon}$ in direction $-\gamma'(0)$ satisfies, at $o$, $$\bar{A}_{\varepsilon}=\bar{A}+\varepsilon\bar{g}.$$ An explicit construction can be found in \cite[Lemma 2.1]{glmm}. Define $$ \rho_{\varepsilon}= \di_{\bar{g}}(\cdot,\Gamma_{\varepsilon}). $$ The separating property of $\partial E^*$ ensures that $\rho_{\varepsilon}\geq \rho$ in $E$, with equality at $x$. Also, $\rho_{\varepsilon}$ is smooth in a neighbourhood of $x$, as shown in \cite{glmm}. The computation leading to \eqref{eq_Lapla_ipers} applied to $\rho_{\varepsilon}$, guarantees that $$ \bar \Delta_f \rho_{\varepsilon} (x) \le \left( \frac{u(x)}{u(o)}\right)^2 \left(\bar H_f(o)+ (m-1)\varepsilon\right), $$ which shows that \eqref{eq_Lapla_ipers} holds in the barrier sense on $E$. Observe that the computation does not depend on the way $\bar g,u$ are extended on $V \backslash E^*$.\\[0.2cm] \noindent \textbf{Step 3}: Busemann functions and splitting.\\[0.2cm] Take a divergent sequence $x_j \in E$, $x_j \to \infty$ and $\bar g$-segments $\gamma_{j}:[0,t_{j}]\to$ $ E^*$ from $\partial E^*$ to $x_j$. By minimality, $\gamma_{j}((0,t_{j}])\subset E$. Since $\partial E^*$ is compact the sequence subconverges to a $\bar g$-ray $\gamma : [0,\infty) \to E^*$ issuing orthogonally from $\partial E^*$, and satisfying $\gamma((0,\infty)) \subset E$. For $x \in E$, we consider the Busemann function $$ b_\gamma(x) \doteq \lim_{t\to\infty}(\di_{\bar{g}}(x,\gamma(t))-t) = \lim_{t\to\infty}(\rho_{t}(x)-t), $$ where $\rho_{t}(x)=d_{\bar{g}}(x,\gamma(t))$. Pick a $\bar g$-segment $\sigma_t : [0, \rho_{t}(x)] \to E$ from $\gamma(t)$ to $x$ (note: we use again property (P)), and set \[ s_{t}(x)=\int_{0}^{\rho_{t}(x)} u^2(\sigma_{t}(\tau)) \di \tau. \] Inequality \eqref{eq_point} gives \begin{equation}\label{lapl_est2} (\bar{\Delta}_f\rho_{t})(x)\leq u(x)^2 \frac{m-1}{s_{t}(x)}. \end{equation} Since $E$ is $u$-complete, we claim that $s_t(x) \to \infty$ as $t \to \infty$. First, we reparametrize $\tilde{\sigma}_{t}(\xi)=\sigma_{t}(\rho_{t}(x)-\xi):[0,\rho_{t}(x)]\longrightarrow E$. Since $\tilde{\sigma}_{t}(0)=x$ for each $t$, $\tilde{\sigma}_{t}$ subconverges to a $\bar{g}$-ray $\sigma:[0,\infty)\longrightarrow E^*$. Again by applying property (P), we deduce that $\sigma((0,\infty)) \subset E$. Fix $T>0$. By $u$-completeness, $\rho_t(x) \to \infty$ as $t \to \infty$, whence for $\rho_t(x) \ge T$ it holds $$ s_{t}(x)=\int_{0}^{\rho_{t}(x)} u^2(\tilde{\sigma}_{t}(\xi)) \di \xi\geq \int_{0}^{T}u^2(\tilde{\sigma}_{t}(\xi)) \di \xi, $$ and then $$ \lim_{t\to\infty}s_{t}(x)\geq \int_{0}^{T}u^2({\sigma}(\xi)) \di \xi. $$ Since $\sigma$ is a ray, hence a divergent curve, letting $T\to\infty$ and using the $u$-completeness we conclude the claim. From \eqref{lapl_est2}, we get \begin{equation}\label{eq_lapla_buse} \bar{\Delta}_f b_\gamma \le 0 \qquad \text{in the barrier sense on } \, E, \end{equation} the barriers at $x$ being $b_\gamma(x) + \di_{\bar g}(\cdot, \sigma(t)) - t$ for large $t$, see \cite[Proposition 7.3.8 and Lemma 7.3.9]{petersen}. Consider the distance $\rho$ to $\partial E^*$ and the function \[ w\doteq \rho + b_\gamma \qquad \text{on } \, E. \] We claim that $w \ge 0$, with equality on $\gamma$. First, $w(\gamma(t)) = t - t = 0$ for each $t \ge 0$. On the other hand, if $x \in E$, pick a $\bar g$-segment from $\partial E^*$ to $x$ and let $o \in \partial E^*$ be its initial point, and then a segment from $x$ to $\gamma(t)$. We compute \[ \rho(x) + \rho_{\gamma(t)}(x) = \di_{\bar g}(x,o) + \di_{\bar g}(x,\gamma(t)) \ge \di_{\bar g}(o,\gamma(t)) \ge \di_{\bar g}(\partial E^*,\gamma(t)) = t, \] whence $\rho(x) + (\rho_{\gamma(t)}(x)-t) \ge 0$. The thesis follows by letting $t \to \infty$. By using \eqref{eq_Lapla_ipers} and \eqref{eq_lapla_buse}, $\bar\Delta_f w \le 0$ holds in the barrier sense on $E$, thus from $w\geq 0$ and $w=0$ on $\gamma$ we get $w \equiv 0$ by the strong maximum principle. In particular, $$ \bar{\Delta}_{f}\rho=\bar{\Delta}_{f}b_\gamma =0, $$ and $\rho,b_\gamma$ are smooth. To see that $\bar{\nabla}\rho$ is the splitting direction, and the corresponding metric is warped as in the statement, we proceed as in \cite[Theorem C]{fogborg} (see also \cite[Lemma 3.5]{wylie1}). We consider the normal exponential map \[ \Phi :[0,\infty) \times \partial E^* \longrightarrow (E^*,\bar{g}), \qquad \Phi(t,y) = \overline{\exp}_{y}(t\bar{\nu}(y)), \] and let \[ T = \sup \left\{ t>0 \ : \ (\Phi_t)_* \ \text{ is nonsingular on $[0,t) \times \partial E$}\right\}. \] As before, we write \[ \Phi^*\bar{g} = \di t^2 + \bar{h}_{\alpha\beta} \di y^\alpha \otimes \di y^\beta \qquad \text{on } \, [0,T) \times \partial E. \] The weighted Bochner formula applied to $\bar{\nabla} \rho$ and the identity \[ 0 = \bar{\Delta}_f \rho = \bar \Delta \rho - \bar{g}(\bar{\nabla} f, \bar{\nabla} \rho) \] imply that \[ 0 = \frac{1}{2}\bar{\Delta}_{f} \|\bar{\nabla}\rho\|^{2} \geq \|\overline{\Hess}\rho \|^{2} - \frac{1}{m-1} \bar{g}(\bar{\nabla} f, \bar{\nabla} \rho)^{2} \ge 0, \] (the last inequality is Newton's one, once we recall that $\overline{\Hess} \rho(\bar \nabla \rho,\cdot) = 0$). Hence, \[ \overline{\Hess} \rho = \frac{\bar{g}(\bar{\nabla} f, \bar{\nabla} \rho)}{m-1} \bar{h} = - \bar{g}(\bar{\nabla} (\ln u), \bar{\nabla} \rho) \bar{h} \qquad \text{on } \, \bar \nabla \rho^\perp \times \bar \nabla \rho^\perp. \] The level sets of $\rho$ are therefore totally umbilic, and the identities \[ \Phi^*\bar{g} = \di t^{2} + \left(\frac{u(t,y)}{u(0,y)}\right)^{-2} \bar h^{\Sigma}, \qquad u(t,y) = r(y)\xi(t) \qquad \text{on } \, [0,T) \times \partial E. \] follow as in the proof of Theorem \ref{teo_splitting_1}. If $T < \infty$, we thus observe that $\Phi_t^* \bar g$ is non-degenerate on $\{t = T\}$, contradicting the definition of $T$. Thus, $T = \infty$ and $\Phi$ is a local diffeomorphism on $[0,\infty) \times \partial E^*$. In order to prove that $\Phi$ is bijective, the surjectivity easily follows by the completeness of $(E^*,\bar g)$. Regarding injectivity, if $\Phi(t_1, y_1) = \Phi(t_2, y_2)$, we can apply $\rho$ to both sides and conclude that $t_1 = t_2 = t$. Moreover, since $\Phi_t: \left\{0\right\} \times \partial E^* \longrightarrow \left\{t\right\} \times \partial E$ is a diffeomorphism, being the flow map of the smooth vector field $\bar \nabla \rho$, we conclude from $\Phi(t, y_1) = \Phi(t, y_2)$ that $y_1 = y_2$. \\ Having proved that $\Phi$ is an isometry, since $E^*$ is connected then $\partial E^*$ is connected, hence the topological boundary of $E$ is connected and separates $E$ from $M \backslash E$. Therefore, the closure $\bar{E}$ in $M$ is a manifold with boundary, isometric to $E^*$. This concludes the proofs of (i) and (ii). \end{proof} \section{A Boucher-Gibbons-Horowitz-type inequality for sub-static manifolds} In this section we prove Theorem \ref{thm_BGH}. Let $(M^m,g,u)$ be a sub-static triple, and assume that $Q$ extends continuously to $\partial M$. This is equivalent to require that \[ \dfrac{\Hess u}{u} \text{ extends continously up to } \partial M. \] \begin{remark} In particular, since \( \partial M = u^{-1}(0) \), the boundary \( \partial M \) is a totally geodesic hypersurface, and \( |\nabla u| \) is constant along \( \partial M \). If \( \Sigma_1, \dots, \Sigma_k \) are the connected components of \( \partial M \), the positive constants \( \kappa_i = |\nabla u|_{\Sigma_i} > 0 \), for \( i = 1, \dots, k \), are called the surface gravities. \end{remark} By taking the trace of the equation above, we obtain \[ uS = u \, \trace Q - (m-1) \Delta u. \] \begin{lemma} In the above assumptions, we have \begin{equation} \label{Qdu} 2 Q(\nabla u,\,\cdot\,) = u (\di S - 2 \div Q) \qquad \text{in } \, \mathring{M} \, . \end{equation} In particular, if $S\in C^1(M)$ and $\div Q\in C^0(M)$ then \begin{equation} \label{Qdu0} Q(\nabla u,\,\cdot\,) = 0 \qquad \text{on } \, \partial M \, . \end{equation} \end{lemma} \begin{remark} Clearly, \eqref{Qdu0} still holds if one requires only that $(\di S - 2\div Q)$ remains bounded in a neighbourhood of $\partial M$. Furthermore, \eqref{Qdu} can be equivalently expressed as \begin{equation} \label{Qdu1} (m-1) \di\frac{\Delta u}{u} = -\frac{2}{u} Q(\nabla u,\,\cdot\,) - 2\div Q + \di\trace Q \qquad \text{in } \, \mathring{M}. \end{equation} \end{remark} \begin{proof} Recalling Schur's and Ricci's identities \begin{align*} \div \Ric & = \frac{1}{2} \di S \\ \Ric(\nabla u,\,\cdot\,) & = \div\Hess u - \di \Delta u \\ & \equiv \div(\Hess u - (\Delta u)g) \end{align*} we have \begin{align*} \div(uQ) & = \div(u\Ric) - \div(\Hess u - (\Delta u)g) \\ & = u\div\Ric + \Ric(\nabla u,\,\cdot\,) - \div(\Hess u - (\Delta u)g) \\ & = u\div\Ric \\ & = \frac{u}{2} \di S \end{align*} in $\mathring{M}$. Since $\div(u Q) = u \div Q + Q(\nabla u,\,\cdot\,)$, we prove \eqref{Qdu}. \end{proof} \begin{lemma} In the above setting, we have \begin{equation} \label{Sh1} \frac{1}{2} \div\left(\frac{\nabla|\nabla u|^2}{u}\right) = \frac{|\Hess u|^2}{u} + \frac{Q(\nabla u,\nabla u)}{u} + \left\langle\nabla u,\nabla\frac{\Delta u}{u}\right\rangle \end{equation} and \begin{equation} \label{Sh2} \begin{split} \div\left(\frac{\mathring{\Hess u}(\nabla u,\,\cdot\,)^\sharp}{u}\right) & = \frac{|\mathring{\Hess u}|^2}{u} + \frac{Q(\nabla u,\nabla u)}{u} + \frac{m-1}{m} \left\langle\nabla u,\nabla\frac{\Delta u}{u}\right\rangle \end{split} \end{equation} where $\mathring{\Hess u} = \Hess u - \frac{1}{m}(\Delta u) g$ is the traceless part of $\Hess u$. \end{lemma} \begin{proof} Recalling that $\frac{1}{2} \di|\nabla u|^2 = \Hess u(\nabla u,\,\cdot\,)$, an application of Bochner's formula \[ \frac{1}{2} \Delta|\nabla u|^2 = |\Hess u|^2 + \Ric(\nabla u,\nabla u) + \langle\nabla u,\nabla\Delta u\rangle \] and of \eqref{sub_static} yields \begin{align*} \frac{1}{2} \div\left(\frac{\nabla|\nabla u|^2}{u}\right) & = \frac{1}{2} \frac{\Delta|\nabla u|^2}{u} - \frac{1}{2} \frac{\langle\nabla|\nabla u|^2,\nabla u\rangle}{u^2} \\ & = \frac{|\Hess u|^2}{u} + \frac{\Ric(\nabla u,\nabla u)}{u} + \frac{\langle\nabla u,\nabla\Delta u \rangle}{u} - \frac{\Hess u(\nabla u,\nabla u)}{u^2} \\ & = \frac{|\Hess u|^2}{u} + \frac{\langle\nabla u,\nabla\Delta u \rangle}{u} + \frac{Q(\nabla u,\nabla u)}{u} - \frac{|\nabla u|^2\Delta u}{u^2} \\ & = \frac{|\Hess u|^2}{u} + \frac{Q(\nabla u,\nabla u)}{u} + \left\langle \nabla u,\nabla\frac{\Delta u}{u} \right\rangle , \end{align*} that is, \eqref{Sh1}. Then, from the identities \begin{align*} \div \left( \frac{\Delta u}{u} \nabla u \right) & = \frac{(\Delta u)^2}{u} + \left\langle \nabla u,\nabla\frac{\Delta u}{u} \right\rangle \\ |\Hess u|^2 & = |\mathring{\Hess u}|^2 - \frac{(\Delta u)^2}{m} \\ \mathring{\Hess u}(\nabla u,\,\cdot\,)^\sharp & = \Hess u(\nabla u,\,\cdot\,)^\sharp - \frac{\Delta u}{m} \nabla u = \frac{1}{2} \nabla|\nabla u|^2 - \frac{\Delta u}{m} \nabla u \end{align*} together with \eqref{Sh1} we obtain \begin{align*} \div\left(\frac{\mathring{\Hess u}(\nabla u,\,\cdot\,)^\sharp}{u}\right) & = \frac{1}{2} \div\left(\frac{\nabla|\nabla u|^2}{u}\right) - \frac{1}{m} \div\left(\frac{\Delta u}{u} \nabla u\right) \\ & = \frac{|\Hess u|^2}{u} - \frac{1}{m} \frac{(\Delta u)^2}{u} + \frac{Q(\nabla u,\nabla u)}{u} + \left(1-\frac{1}{m}\right) \left\langle \nabla u,\nabla\frac{\Delta u}{u} \right\rangle \\ & = \frac{|\mathring{\Hess u}|^2}{u} + \frac{Q(\nabla u,\nabla u)}{u} + \frac{m-1}{m} \left\langle \nabla u,\nabla\frac{\Delta u}{u} \right\rangle \end{align*} proving \eqref{Sh2}. \end{proof} On the set $M_0 := \{x \in M : \nabla u(x)\neq 0\}$, we define \[ \nu = -\frac{\nabla u}{|\nabla u|} \, . \] For each $p\in M_0$, the intersection $\Sigma = \Sigma_{u(p)} \cap \Omega_p$ of the level set $\Sigma_{u(p)} = \{x \in M : u(x) = u(p)\}$ with a sufficiently small neighbourhood $\Omega_p\subseteq M_0$ of $p$ is an embedded smooth hypersurface of $M$ with normal vector field $\nu$. We denote by $g_\Sigma$ the metric on $\Sigma$ induced by $g$, \[ g_\Sigma = g - \nu^\flat \otimes \nu^\flat \, . \] We also denote by $H$ the unnormalized mean curvature of $\Sigma$ in the direction of $\nu$, \[ H \doteq -\div\nu \] and by $A$ the second fundamental form of $\Sigma$ in the same direction. We have \[ A(X,Y) = \frac{\Hess u(X,Y)}{|\nabla u|} \qquad \forall \, X,Y \in T_x\Sigma \, , \, x \in \Sigma \] and also \begin{equation} \label{H_Hessu} H = \trace_{g_\Sigma} A \equiv \frac{\Delta u - \Hess u(\nu,\nu)}{|\nabla u|} \, . \end{equation} We also set \[ \mathring{A} = A - \frac{H}{m-1} g_\Sigma \] for the traceless (with respect to $g_\Sigma$) part of $A$. \begin{lemma} For any $C^2$ function $u$ on a Riemannian manifold $M$, at any point where $\nabla u\neq 0$ we have, with the notation introduced above, \begin{equation} \label{Hu0} |\mathring{\Hess u}|^2 = |\nabla u|^2 |\mathring{A}|^2 + \frac{m-2}{m-1}|\nabla^\top|\nabla u||^2 + \frac{m}{m-1} |\mathring{\Hess u}(\nu,\,\cdot\,)^\sharp|^2 \end{equation} \end{lemma} \begin{proof} With respect to an orthonormal basis $\{e_i\}$ for $T_x M$ with $e_1 = \nu$, we have \begin{align*} |\mathring{\Hess u}|^2 & = |\Hess u|^2 - \frac{(\Delta u)^2}{m} = \sum_{i,j=1}^m u_{ij} u_{ij} - \frac{(\Delta u)^2}{m}, \\ |\nabla u|^2|\mathring{A}|^2 & = |\nabla u|^2|A|^2 - \frac{|\nabla u|^2 H^2}{m-1} = \sum_{i,j=2}^m u_{ij} u_{ij} - \frac{(\Delta u - u_{11})^2}{m-1} \\ & = \sum_{i,j=2}^m u_{ij} u_{ij} - \frac{(u_{11})^2}{m-1} + \frac{2 (\Delta u) u_{11}}{m-1} - \frac{(\Delta u)^2}{m-1} \, , \end{align*} hence \begin{align*} |\mathring{\Hess u}|^2 - |\nabla u|^2 |\mathring{A}|^2 & = 2 \sum_{i=2}^m u_{1i} u_{1i} + \frac{m}{m-1} (u_{11})^2 - \frac{2 (\Delta u) u_{11}}{m-1} + \frac{(\Delta u)^2}{m(m-1)} \\ & = \frac{m-2}{m-1} \sum_{i=2}^m u_{1i} u_{1i} + \frac{m}{m-1} \sum_{i=1}^m u_{1i} u_{1i} - \frac{2(\Delta u) u_{11}}{m-1} + \frac{(\Delta u)^2}{m(m-1)} \end{align*} and then the conclusion follows since \begin{align*} |\nabla^\top|\nabla u||^2 & = \sum_{i=2}^m u_{1i} u_{1i} \, , \\ |\mathring{\Hess u}(\nu,\,\cdot\,)^\sharp|^2 & = \left|\Hess u(\nu,\,\cdot\,)^\sharp - \frac{\Delta u}{m} \nu\right|^2 \\ & = |\Hess u(\nu,\,\cdot\,)^\sharp|^2 - \frac{2}{m} (\Delta u) \Hess u(\nu,\nu) + \frac{(\Delta u)^2}{m^2} \\ & = \sum_{i=1}^m u_{1i} u_{1i} - \frac{2}{m} (\Delta u) u_{11} + \frac{(\Delta u)^2}{m^2} \, . \end{align*} \end{proof} We also need the following lemma. Before stating it, we recall that under our general assumptions the tensor field \[ \frac{\mathring{\Hess u}}{u} \] extend continuously to the whole manifold $M$. With a little abuse of notation, we are going to denote such an extension by the same symbol. Define also \begin{equation}\label{def_lambda} \Lambda \doteq \frac{S-\trace Q}{m-1}. \end{equation} \begin{lemma} If $u$ solves \eqref{sub_static} with $Q\in C^1(M)$, then \begin{equation} \label{Hu0_bd} \begin{array}{lcl} \disp \frac{\mathring{\Hess u}(\nu,\nu)}{u} & = & \disp \frac{(m-1)(m-2)}{2m} \Lambda + \frac{1}{2} \trace Q - \frac{1}{2} S_{\partial M} \\[0.5cm] & = & \disp \frac{1}{m} \big( \trace Q - S\big) \qquad \text{on } \, \partial M \, . \end{array} \end{equation} \end{lemma} \begin{proof} From the definition of $Q$ we have \[ \frac{\mathring{\Hess u}}{u} = \mathring{\Ric} - \mathring{Q} \equiv \Ric - \frac{S}{m} g - Q + \frac{\trace Q}{m} g \] on $\mathring{M}$, and thus on the whole $M$ by continuity of $Q$ and of $\Ric$. Hence, \[ \frac{\mathring{\Hess u}(\nu,\nu)}{u} = \Ric(\nu,\nu) - \frac{S}{m} - Q(\nu,\nu) + \frac{\trace Q}{m} \, . \] By Gauss equations and from the fact that $\partial M$ is totally geodesic, we have \[ 2\Ric(\nu,\nu) = S - S_{\partial M} - |A_{\partial M}|^2 + H_{\partial M}^2 \equiv S - S_{\partial M} \qquad \text{on } \, \partial M \, . \] On the other hand, under assumptions $Q \in C^1(M)$ and \eqref{DuL} we have $\div Q \in C^0(M)$. Thus, by using $S \in C^1(M)$ we can apply \eqref{Qdu} to deduce $Q(\nu,\nu) = 0$ on $\partial M$. Substituting these relations into the above one we obtain \[ \frac{\mathring{\Hess u}(\nu,\nu)}{u} = \frac{m-2}{2m} S + \frac{1}{m} \trace Q - \frac{1}{2} S_{\partial M} \equiv \frac{(m-1)(m-2)}{2m} \Lambda + \frac{1}{2} \trace Q - \frac{1}{2} S_{\partial M} \, . \] \end{proof} We next assume that \[ S - \trace Q \qquad \text{is constant on } \, M, \] so that, by \eqref{sub_trace}, the function \( u \) satisfies \begin{equation} \label{DuL} \Delta u + \Lambda u = 0 \qquad \text{in } \, M, \qquad \Lambda \doteq \frac{S-\trace Q}{m-1} \ \ \ \text{ constant.} \end{equation} Remarkably, under this assumption the vector field $\mathring{\Hess u}(\nabla u,\,\cdot\,)^\sharp$ happens to be a gradient vector field, namely the gradient of the function \begin{equation} \label{Fdef} F = \frac{1}{2} |\nabla u|^2 + \frac{\Lambda}{2m} u^2 \end{equation} and, in this setting, the content of \eqref{Sh2} and \eqref{Hu0} can be restated as follows. \begin{lemma} \label{lem_dF} In the above setting, if \eqref{DuL} holds, \[ \div\left(\frac{\nabla F}{u}\right) = \frac{|\nabla u|^2}{u} |\mathring{A}|^2 + \frac{m-2}{m-1} \frac{|\nabla^\top|\nabla u||^2}{u} + \frac{Q(\nabla u,\nabla u)}{u} + \frac{m}{m-1} \frac{|\nabla F|^2}{u|\nabla u|^2} \qquad \text{on } \, M_0, \] with $F$ as in \eqref{Fdef} and $M_0 = \{\nabla u \neq 0\}$. In particular, if $Q\geq 0$, then \[ \div\left(\frac{\nabla F}{u}\right) \geq 0 \qquad \text{in } \, M, \] and the equality holds if and only if $\mathring{\Hess}u=0$. \end{lemma} We are ready to prove Theorem \ref{thm_BGH} and Corollary \ref{coro_bgh}. First, because of \eqref{DuL} we have the identity \begin{equation}\label{eq_with_lambda} S_{\partial M} - \frac{m-2}{m}S - \frac{2}{m} \trace Q = S_{\partial M}- \frac{(m-1)(m-2)}{m}\Lambda - \trace Q \end{equation} where $S_{\partial M}$ is the scalar curvature of $\partial M$. \begin{proof}[Proof of Theorem \ref{thm_BGH}] First, observe that by \eqref{eq_with_lambda} inequality \eqref{BGHtype_intro} is equivalent to \begin{equation} \label{BGHtype} \sum_i \kappa_i^b \int_{\Sigma_i} S_{\Sigma_i} \geq \frac{(m-1)(m-2)}{m} \Lambda \sum_i \kappa_i^b |\Sigma_i| + \sum_i \kappa_i^b \int_{\Sigma_i} \trace Q \, . \end{equation} The function $F$ defined in \eqref{Fdef} is smooth and positive on the whole of $M$ (note that $F=\frac{1}{2}|\nabla u|^2>0$ on $\partial M$), so the function $F^a$ is also smooth and positive for any exponent $a\in\R$. The vector field \[ X = \frac{(2F)^a\nabla F}{u} \] is smooth in the interior of $M$ and continous up to $\partial M$, so by the divergence theorem \[ \int_{\partial M} \langle X,\nu\rangle = \int_M \div X \] where, on $\partial M$, $\nu = - \nabla u/|\nabla u|$ coincides with the outward pointing unit normal. We have \[ \langle X,\nu\rangle = |\nabla u|^{2a} \frac{\mathring{\Hess u}(\nabla u,\nu)}{u} = - |\nabla u|^{2a+1} \frac{\mathring{\Hess u}(\nu,\nu)}{u} \qquad \text{on } \, \partial M, \] and then by \eqref{Hu0_bd} we get \[ \int_{\partial M} |\nabla u|^{2a+1} S_{\partial M} = \frac{(m-1)(m-2)}{m} \Lambda \int_{\partial M} |\nabla u|^{2a+1} + \int_{\partial M} |\nabla u|^{2a+1} \trace Q + 2\int_M \div X \, . \] From Lemma \ref{lem_dF} we deduce \begin{align*} 2^{-a} \div X = \div\left(\frac{F^a\nabla F}{u}\right) & = \frac{F^a}{u} \left(|\nabla u|^2|\mathring{A}|^2 + \frac{m-2}{m-1} |\nabla^\top|\nabla u||^2 + Q(\nabla u,\nabla u) \right) \\ & \phantom{=\;} + \frac{F^{a-1}}{u|\nabla u|^2} \left(\frac{m}{m-1} F + a |\nabla u|^2\right) |\nabla F|^2 \end{align*} on $M_0\cap \mathring{M}$, where $M_0 = \{\nabla u \neq 0\}$. By the definition of $F$ we have \[ \frac{m}{m-1} F + a |\nabla u|^2 = \left(\frac{m}{2(m-1)} + a\right)|\nabla u|^2 + \frac{\Lambda}{2(m-1)} u^2 > 0 \qquad \text{on } \, \mathring{M}, \] as long as $a\geq-\frac{m}{2(m-1)}$, or, equivalently, \begin{equation} \label{a_cond} 2a+1 \geq -\frac{1}{m-1} \, . \end{equation} Thus, assumption \eqref{a_cond} gives $\div X \geq 0$ on $M_0\cap \mathring{M}$, hence on the whole interior of $M$ by continuity of $\div X$ and density of $M_0$. In fact, note that by $\Delta u + \Lambda u = 0$ the set of critical points $\{|\nabla u|=0\}$ has zero measure, see \cite{uh76}. Consequently, after renaming $b = 2a+1$ we obtain \eqref{BGHtype}. If \eqref{BGHtype} is satisfied with the equality sign, then necessarily $\div X \equiv 0$ on $\mathring{M}$. This implies that $\mathring{A}$, $\nabla^\top|\nabla u|$ and $\nabla F$ vanish everywhere on $M_0\cap \mathring{M}$, so we also have $\mathring{\Hess u} \equiv 0$ on $\mathring{M}$ in view of \eqref{Hu0}. Therefore, $\Hess u \equiv -\frac{\Lambda}{m} u$ on $M$. The conclusion follows by Reilly's generalization of Obata's theorem to compact manifolds with boundary (see \cite[Lemma 3]{reilly}). \end{proof} \begin{proof}[Proof of Corollary \ref{coro_bgh}] First, integrating \eqref{DuL} on $M$ against $u$ we get \[ \Lambda \int_M u^2 = \int_M |\nabla u|^2, \] thus $\Lambda > 0$. Having split $\partial M$ into pieces $\{\hat \Sigma_a\}_{a=1}^j$ according to the value of the respective surface gravities $\{\kappa_a\}$, inequality \eqref{BGHtype_intro} rewrites as \begin{equation}\label{eq_itera} \sum_a \kappa_a^b \int_{\hat{\Sigma}_a} \left(S_{\hat{\Sigma}_a} - \frac{m-2}{m}S - \frac{2}{m} \trace Q\right) \ge 0. \end{equation} Dividing by $\kappa_j^b$ and letting $b \to \infty$, only the integral on $\hat{\Sigma}_j$ survives and \[ \int_{\hat{\Sigma}_j} \left(S_{\hat{\Sigma}_j} - \frac{m-2}{m}S - \frac{2}{m} \trace Q\right) \ge 0. \] Considering $m=3$ and using \eqref{eq_with_lambda}, the Gauss-Bonnet theorem and $Q \ge 0$ implies \[ 4\pi \chi(\hat{\Sigma}_j) \ge \frac{2}{3} \Lambda |\hat{\Sigma}_j|. \] In particular, $\chi(\hat{\Sigma}_j)>0$. Since $\chi(\hat{\Sigma}_j)$ is an even number, $\chi(\hat{\Sigma}_j) \ge 2$ and one of the components of $\hat \Sigma_j$ is a topological sphere. We therefore deduce \eqref{ine_enhanced_BGH} after replacing the value of $\Lambda$. Regarding (ii), if \eqref{ine_enhanced_BGH_a} holds for $i \le a \le j$, then in \eqref{eq_itera} only the addenda from $a=1$ to $a=i-1$ survive. Dividing \eqref{eq_itera} by $\kappa_{i-1}^b$, letting $b \to \infty$ and proceeding as above we obtain \eqref{ine_enhanced_BGH_am1}. If \eqref{ine_enhanced_BGH_tutti} is satisfied, then equality holds in \eqref{eq_itera}, thus by Theorem \ref{thm_BGH} the manifold $(M^3,g)$ is a round hemisphere and $Q \equiv 0$ on $M$. \end{proof} In the context of Theorem \ref{thm_BGH}, it is also possible to establish a constraint on the scalar curvature of the boundary, extending the analysis presented in \cite{ses}. Supposing that all the surface gravities are the same constant \( \kappa_{i} = 1 \) (in particular, this occurs when \( \partial M \) is connected), the function \[ F = \frac{1}{2}|\nabla u|^2 + \frac{\Lambda}{2m} u^2 \] is constant and equal to \( \frac{1}{2} \) on \( \partial M \). Using \eqref{Sh2}, we have \[ \diver\left(\frac{\nabla F}{u}\right) = \frac{1}{u}|\mathring{\Hess}u|^2 + \frac{1}{u} Q(\nabla u, \nabla u) \geq 0. \] By applying the maximum principle, we conclude that \( F \) attains its maximum on \( \partial M \), unless \( F \) is constant. In the case where \( F \) is constant, it follows directly that \( \mathring{\Hess} u = 0 \), implying that \( M \) is isometric to a hemisphere. Assume that \( F \) achieves its global maximum on \( \partial M \). Since \( u \) reaches its global minimum on \( \partial M \), let \( p \in \partial M \) and, by considering any point \( x \) sufficiently close to \( p \), we have \[ 0 \geq \langle \nabla F, \nabla u \rangle (x) = u |\nabla u|^2 \left( \frac{\Lambda}{m} + \frac{X(X(u))}{u} \right)(x), \] where \( X = \frac{\nabla u}{|\nabla u|} \). Therefore, \begin{equation} \label{ineq_X} \frac{X(X(u))}{u} \leq -\frac{\Lambda}{m}. \end{equation} Since \( X(X(u)) = \Hess u(X,X) \), using \eqref{sub_static}, we obtain \[ [\Ric(X,X) - \Lambda - Q(X,X)](x) \leq -\frac{\Lambda}{m}. \] Taking the limit as \( x \to p \), and applying Gauss' equation along with \eqref{Qdu0}, we get \[ S_{\partial M} \geq \Lambda \frac{(m-1)(m-2)}{m} + \trace Q. \] Note that, by the rigidity statement in Theorem \ref{thm_BGH} (see also \eqref{BGHtype}), equality holds if and only if \( (M^n, g) \) is isometric to a round hemisphere. \tcr{} \section{Einstein equations with a map and Potential Sources}\label{sec_map} In this section, we describe a rigidity result for a static Einstein system whose stress-energy tensor $T$ is given by a static wave map. In this setting, $T$ is given by the metric variation of a natural Lagrangian, as described below. For more details on the Lagrangian formulation in General Relativity, see \cite[Appendix E.1]{wald} or \cite[Section 3.3]{hawking}, for example. \subsection{Maps between manifolds} To make computations, we adopt the moving frame formalism introduced by É. Cartan (for more details of this formalism, we refer to \cite[Section 1.7]{amr}). Let \( \phi: (M^m, g) \to (N^n, h) \) be a smooth map between two Riemannian manifolds. Consider orthonormal frames $\{e_i\}$ and coframes $\{\theta^i\}$ on an open subset $U \subseteq M$, and orthonormal frames $\{E_a\}$ and coframes $\{\omega^a\}$ on an open subset $V \subseteq N$ such that $\varphi^{-1}(V)\subseteq U$. We define \begin{equation*} \varphi^*\omega^a = \varphi^a_i \theta^i, \end{equation*} so that the differential \( \di \varphi \), viewed as a $1$-form on $M$ with values in the pullback bundle $\varphi^{-1}TN$, is expressed as \begin{equation*} \di\varphi = \varphi^a_i \theta^i \otimes E_a. \end{equation*} The energy density \( e(\varphi) \) of the map $\varphi$ is defined as \begin{equation}\label{def dens en} e(\varphi) = \frac{1}{2} |\di\varphi|^2, \end{equation} where $|\di\varphi|^2 = \varphi^a_i \varphi^a_i$. The second fundamental form of the map $\varphi$ is given by $\nabla\di\varphi : TM \otimes TM \to TN$, the covariant derivative of $\di\varphi$ regarded as a section of $T^\ast M \otimes \varphi^{-1}TN$ equipped with the connection $\nabla \otimes \varphi^\ast D$ induced by the Levi-Civita connections $\nabla$ and $D$ of $M$ and $N$, respectively. Explicitly, \begin{equation*} \nabla \di\varphi = \varphi^a_{ij} \theta^j \otimes \theta^i \otimes E_a, \end{equation*} where the coefficients \( \varphi^a_{ij} \) are defined by the relation \begin{equation*} \varphi^a_{ij} \theta^j = \di\varphi^a_i - \varphi^a_k \theta^k_i + \varphi^b_i \omega^a_b, \end{equation*} and $\{\theta^i_j\}$, $\{\omega^a_b\}$ are the connection forms of $\nabla$ and $D$, respectively. The tension field \( \tau(\varphi) \) is defined by \begin{equation}\label{def tension field} \tau(\varphi) = \mathrm{tr} (\nabla \di\varphi)=\varphi_{ii}^aE_{a}. \end{equation} In this setting, $|\di \varphi|^2$ satisfies the the Bochner formula \begin{equation} \label{boch} \frac{1}{2} \Delta |\di \varphi|^2 = |\nabla \di \varphi|^2 + \varphi_i^a \varphi_{kki}^a + R_{ij} \varphi_i^a \varphi_j^a - R^N_{abcd} \varphi_i^a \varphi_j^b \varphi_i^c \varphi_j^d, \end{equation} where $R_{ij}$ and $R^N_{abcd}$ denote the local components of the Ricci tensor of $g$ and the Riemann tensor of $h$ in the given orthonormal frames, respectively. For a proof of this identity, see, for example, \cite[Proposition 1.5]{amr}. \subsection{The related Einstein's equation} Let $\Phi:(\hat{M}^{m+1},\hat{g}) \longrightarrow (N^n,h)$ and $V:(N^n,h)\longrightarrow\mathbb{R}$ be two smooth maps. Consider the matter Lagrangian defined by: \[ \mathcal{L}(\hat{g},\Phi) = \int_{\hat{M}} \left[ |\di\Phi|_{\hat{g}}^2 + (m-1)V(\Phi) \right] \di x_{\hat{g}}. \] The stress-energy tensor associated to this system is obtained by varying $\mathcal{L}$ with respect to $\hat{g}$, which gives \[ T = \Phi^{*}h - \frac{1}{2} \left( |\di \Phi|_{\hat{g}}^2 + (m-1)V(\Phi) \right)\hat{g}. \] Therefore, Einstein's equation \eqref{Einst_eq} can be written as \begin{equation}\label{eq_maps} \Ric_{\hat{g}}+\Lambda\hat{g} = \Phi^{*}h + V(\Phi)\hat{g}. \end{equation} We observe that the cosmological constant can be incorporated in the function $V$ by adding a constant in $V$. Thus, without loss of generality, we assume $\Lambda = 0$ in the subsequent analysis. The equation of motion for this system, obtained as the Euler Lagrangian equation with respect to $\Phi$ is given by \[ \hat{\tau}(\Phi) = \frac{m-1}{2} D V(\Phi), \] where $\hat{\tau}(\Phi)$ denotes the tension field. For detailed computations, see \cite[Section 5]{ans21}. For the special case where $N=\mathbb{R}$, we refer to \cite{reiris}. We now focus on the static case, where $(\hat{M}, \hat{g})$ is given by \( \hat{M} = \mathbb{R} \times M \) and \( \hat{g} = -u^2 \, \mathrm{d}t \otimes \mathrm{d}t + g \), with $(M^m, g)$ Riemannian and $0<u \in C^\infty(M)$. We assume that $\Phi$ is static as well, that is, it factorizes trough the projection \(\pi:\hat{M}\to M\) as the composition \(\Phi = \phi \circ \pi \), for some map \( \phi: M \to (N^n, h) \). In this framework, system \eqref{eq_maps} becomes \begin{equation}\label{mapfieldequation} \left\{ \begin{array}{r@{\;}c@{\;}l} \Ric - \frac{\Hess u}{u} & = & \varphi^{*}h+V(\phi)g \\[0.2cm] -\Delta u & = & V(\phi) u. \end{array} \right. \end{equation} Additionally, given that \(u \hat{\tau}(\Phi) = \di\phi(\nabla u) + u\tau(\phi)\), equation of motion equation becomes \begin{equation}\label{motion_eq_map} u\tau(\varphi)+\di\varphi(\nabla u)=(m-1)\frac{D V(\phi)}{2}u. \end{equation} By introducing the known change of variable \( u = e^{-f} \), the coupled system \eqref{mapfieldequation}-\eqref{motion_eq_map} transforms into the following coupled system: \begin{equation}\label{eq_field_f} \left\{ \begin{array}{r@{\;}c@{\;}l} \Ric_f^{m+1} & = & \varphi^{*}h + V(\varphi)g \\[0.2cm] \Delta_f f & = & V(\varphi) \\[0.2cm] \tau(\varphi) - \di\varphi(\nabla f) & = & \frac{m-1}{2} D V(\varphi) \end{array} \right. \end{equation} Note that the components of the weighted Ricci curvature \(\Ric_f^{m+1}\) are given by \[ (\Ric_f^{m+1})_{ij} = R_{ij} + f_{ij} - f_i f_j = R_{ij}^{f} - f_i f_j, \] where \( R_{ij}^{f} \) are the components of the Bakry-Émery Ricci tensor. The third equation in \eqref{eq_field_f} suggests to define a weighted operator related to the tension field: $$ \tau_{f}(\varphi)=\tau(\varphi)-\di\varphi(\nabla f). $$ In coordinates, $$(\varphi_{kk}^{a})^{f}=\varphi_{kk}^{a}-\varphi_{j}^{a}f_{j}.$$ We have the following Bochner's formula: \begin{lemma}\label{bochner} \begin{equation} \frac{1}{2}\Delta_{f}|\di\varphi|^{2}=|\nabla \di\varphi|^{2}+\langle \nabla \tau_{f}(\varphi),\di\varphi\rangle_{N}+Q(\di\varphi), \end{equation} where $$ Q(\di\varphi)=R_{ij}^{f} \varphi^a_i \varphi^a_j - R^N_{abcd} \varphi^a_i \varphi^b_j \varphi^c_i \varphi^d_j. $$ \end{lemma} \begin{proof} The identity follows by coupling the Bochner formula \eqref{boch} with the following identity \begin{eqnarray*} \varphi_{i}^{a}[(\varphi_{kk}^{a})^{f}]_{i}+R_{ij}^{f} \varphi^a_i \varphi^a_j &=&\varphi_{i}^{a}\varphi_{kki}^{a}-\varphi_{i}^{a}(\varphi_{j}^{a}f_{j})_{i}+R_{ij} \varphi^a_i \varphi^a_j+f_{ij}\varphi^a_i \varphi^a_j\\ &=&\varphi_{i}^{a}\varphi_{kki}^{a}+R_{ij} \varphi^a_i \varphi^a_j-\varphi_{i}^{a}\varphi_{ij}^{a}f_{j}\\ &=&\varphi_{i}^{a}\varphi_{kki}^{a}+R_{ij} \varphi^a_i \varphi^a_j-\frac{1}{2}f_{j}(\varphi_{i}^{a})_{j}. \end{eqnarray*} \end{proof} We now consider the more general setting of Theorem \ref{vanishing_map}, where \eqref{eq_field_f} is replaced by \eqref{map_source}. The first equation in \eqref{map_source} implies $$ {(R_{ij}^{f})^{m+1}}\varphi_{i}^{a}\varphi_{j}^{a} \ge \varphi^b_i\varphi^b_j\varphi^a_i\varphi^a_j + V(\varphi)|\di\varphi|^2, $$ while the third one in \eqref{map_source} gives $$(\varphi_{kk}^a)^{f} = \frac{m-1}{2} V^a(\varphi) \qquad \mbox{and} \qquad[(\varphi_{kk}^{a})^{f}]_{i}=\frac{m-1}{2} V^a_b(\varphi)\varphi^b_i.$$Inserting into the Bochner's formula in Lemma \ref{bochner} we get \begin{eqnarray*} \frac{1}{2}\Delta_{f}|d\varphi|^{2}& \ge &|\nabla d\varphi|^{2}+\frac{m-1}{2}V_{ab}(\varphi) \varphi^a_i \varphi^b_i +\varphi^a_i\varphi^a_j\varphi^b_i\varphi^b_j+V(\varphi)|d\varphi|^{2}\\ & & +f_{i}\varphi_{i}^{a}f_{j}\varphi_{i}^{a}- R^N_{abcd} \varphi^a_i \varphi^b_j \varphi^c_i \varphi^d_j\\ &\geq & \left[\frac{m-1}{2}\Hess V+Vh\right]_{ab}(\varphi) \varphi^a_i\varphi^b_i+\varphi^a_i\varphi^a_j\varphi^b_i\varphi^b_j-R^N_{abcd} \varphi^a_i \varphi^b_j \varphi^c_i \varphi^d_j. \end{eqnarray*} Hence, \begin{equation}\label{ineq_liouv} \frac{1}{2}\Delta_{f}|d\varphi|^{2}\geq\left[\frac{m-1}{2}\Hess V+Vh\right]_{ab}(\varphi) \varphi^a_i\varphi^b_i+Q_{0}(d\varphi), \end{equation} where we set $$ Q_{0}(d\varphi)=\varphi^a_i\varphi^a_j\varphi^b_i\varphi^b_j-R^N_{abcd} \varphi^a_i \varphi^b_j \varphi^c_i \varphi^d_j. $$ We shall examine $Q_{0}$. \begin{lemma} \label{lemma_estimate} If $$\sup_{N}\sec_{N}\leq\kappa\leq\frac{1}{m-1},$$ then $$Q_{0}(\di\varphi) \geq \frac{1-(m-1)\kappa}{m} |\di\varphi|^4.$$ \end{lemma} \begin{proof} We decompose \[ Q_0(\di\varphi) = Q_1(\di\varphi) + Q_2(\di\varphi), \] where \begin{align*} Q_1(\di\varphi) & = (m-1)\kappa \varphi^a_i \varphi^a_j \varphi^b_i \varphi^b_j - R^N_{abcd} \varphi^a_i \varphi^b_j \varphi^c_i \varphi^d_j, \\ Q_2(\di\varphi) & = (1 - (m-1)\kappa) \varphi^a_i \varphi^a_j \varphi^b_i \varphi^b_j = (1 - (m-1)\kappa) \|\varphi^\ast h\|^2. \end{align*} Since \( \sec_N \leq \kappa \), we have \( Q_1(\di\varphi) \geq 0 \) by \cite[Lemma 2.2]{cmrigoli}. On the other hand, since \( 1 - (m-1)\kappa > 0 \) by assumption, and applying the Newton inequality, we obtain \[ \|\varphi^\ast h\|^2 \geq \frac{(\mathrm{tr} \, \varphi^\ast h)^2}{m} = \frac{1}{m} |\di\varphi|^4, \] which implies \[ Q_0(\di\varphi) \geq Q_2(\di\varphi) \geq \frac{1 - (m-1)\kappa}{m} |\di\varphi|^4. \] \end{proof} From Lemma \ref{lemma_estimate}, if $\displaystyle\sup_{N}\sec_{N}\leq\kappa\leq \frac{1}{m-1}$, the map $\varphi$ satisfies the following inequality: \begin{equation}\label{eq_dphi} \frac{1}{2}\Delta_{f}|\di\varphi|^{2} \geq \left[\frac{m-1}{2}\Hess V + Vh \right]_{ab}(\varphi) \varphi^a_i \varphi^b_i + \frac{1 - (m-1)\kappa}{m}|\di\varphi|^{4}. \end{equation} In view of Keller-Osserman's theory (see \cite{bmpr} for a detailed account) a Liouville-type result for the map $\varphi$ may follow under suitable hypotheses on the potential $V$ and a control of the $f$-volume of balls. Given a complete Riemannian manifold \( M \) we fix an origin \( \mathcal{O}\subset M \) as either a single point or a relatively compact, open subset with a smooth boundary \( \partial\mathcal{O} \). We define the function \( r(x) = \operatorname{dist}(x, \mathcal{O}) \), and for \( R > 0 \), the ball \[ B_{R}(\mathcal{O}) = \left\{x \in M; r(x) \in (0, R)\right\}. \] The next result could be viewed as a version of \cite[Lemma 4.6]{rigoli2} to (weighted) manifolds with boundary. In general, \cite[Lemma 4.6]{rigoli2} is not expected to hold for manifolds with boundary. The main point here is that, in our setting, the lapse function $u$ (hence, the density $e^{-f}$ of the weighted measure) vanishes on $\partial M$, allowing us to get rid of the boundary terms. In what follows, we set \[ \di \mu_f = e^{-f}\di x, \qquad \vol_f(A) = \int_A \di \mu_f \qquad \forall A \subset M. \] \begin{proposition}\label{vol_control1} Let $(M^{m},g,u)$ be a complete manifold with $\partial M=u^{-1}(0)$ and $u>0$ in $\mathring{M}$. Let $v\in {\rm Lip}_{\rm{loc}}(M)$ and $\gamma>0$ such that $$\Omega_{\gamma}=\left\{x\in M:\:\ v(x)>\gamma\right\}\neq\emptyset.$$ For $f=-\ln{u}$, suppose that $v$ satisfies \begin{equation}\label{lap_estimate} \Delta_{f}v\geq bv^{\sigma}\qquad\mbox{on}\quad\Omega_{\gamma}, \end{equation} for positive constants $b>0$ and $\sigma>1$. If \begin{equation}\label{hip_volume} \displaystyle\liminf_{r\to\infty}\frac{\ln{\vol_f ( B_{r}(\mathcal{O}))}}{r^2}<\infty, \end{equation} then $$\sup v<\infty.$$ \end{proposition} \begin{proof} The proof closely follows that of \cite[Lemma 4.5]{rigoli2}. We decide to provide full details to underline the role of the boundary terms. The core step is the following growth inequality\\[0.2cm] \noindent\textbf{Claim 1}: There exists a constant $C>0$ such that, for every $r>0$ and $\alpha>1$, \begin{equation}\label{bound_eq} \vol_{f}(\Omega_{\gamma}\cap B_{r}(\mathcal{O})) \leq\left[\frac{1}{\gamma}\frac{C}{r^{2}}\frac{1}{b}\frac{(\alpha+\sigma-1)^{2}}{\alpha-1}\right]^{\alpha+\sigma-1}\vol_{f}(\Omega_{\gamma}\cap B_{2r}(\mathcal{O})). \end{equation} \begin{proof}[Proof of Claim 1:] Fix a constant $\zeta>1$ such that $$2+\frac{2}{\sigma-1}\left(\frac{1}{\zeta}-1\right)>0.$$ Choose a cut-off function $\psi:M\longrightarrow [0,1]$ such that \begin{enumerate} \item $\psi\equiv 1$ on $B_{r}$; \item $\psi\equiv 0$ on $M\setminus B_{2r}$; \item $|\nabla\psi|\leq\frac{C_{0}}{r}\psi^{\frac{1}{\zeta}}$, for a constant $C_{0}=C_{0}(\zeta)>0$. \end{enumerate} Fix constants $\alpha>1$, $\varepsilon>0$ and a $C^{1}$ non-decreasing function $\lambda:\mathbb{R}\longrightarrow [0,\infty)$ such that $\lambda(t)=0$ for $t\leq\gamma$, and test the equation \eqref{lap_estimate} against the function $$\psi^{2(\alpha+\sigma-1)}\lambda(v)v^{\alpha-1}\eta_{\varepsilon}(u),$$ where $$\eta_{\varepsilon}(u)=\left\{ \begin{array}{l@{\;}c@{\;}l} 0 & {\rm if} & u\leq\varepsilon \\[0.2cm] \frac{u-\varepsilon}{\varepsilon}, & {\rm if} & \varepsilon\leq u\leq 2\varepsilon \\[0.2cm] 1, & {\rm if} & u\geq 2\varepsilon. \end{array} \right.$$ Observe that \begin{eqnarray*} & &\int\psi^{2(\alpha+\sigma-1)}\lambda(v)v^{\alpha+1}\langle\nabla u,\nabla\eta_{\varepsilon}(u)\rangle \di x_{f}\\ &=&\int_{\left\{\varepsilon<u<2\varepsilon\right\}}\psi^{2(\alpha+\sigma-1)}\lambda(v)v^{\alpha+1}\frac{|\nabla u|^{2}}{\varepsilon}\di x_{f} \to 0\quad{\rm as} \quad\varepsilon\to 0 \end{eqnarray*} which accounts for the fact that the measure density $e^{-f}=u$ vanishes on $\partial M$. Therefore, by letting $\varepsilon\to 0$ and using $\lambda'\geq 0$ we obtain \begin{eqnarray*} \displaystyle\int \psi^{2(\alpha+\sigma-1)}\lambda(v)bv^{\alpha+\sigma-1}\di x_{f}&\leq& -(\alpha-1)\displaystyle\int\psi^{2(\alpha+\sigma-1)}\lambda(v)v^{\alpha-2}|\nabla v|^{2}\di x_{f}\\ & &+2(\alpha+\sigma-1)\displaystyle\int\psi^{2(\alpha+\sigma-1)-1}\lambda(v)v^{\alpha-1}|\nabla\psi||\nabla v|\di x_{f}. \end{eqnarray*} By Young's inequality, we can estimate \begin{eqnarray*} & &2(\alpha+\sigma-1)\displaystyle\int\psi^{2(\alpha+\sigma-1)-1}\lambda(v)v^{\alpha-1}|\nabla\psi||\nabla v|\di x_{f} \\ &=&\int\left[\psi^{\alpha+\sigma-1}\lambda(v)^{\frac{1}{2}}v^{\frac{\alpha-2}{2}}|\nabla u|\right]\left[2\psi^{\alpha}\lambda(v)^{\frac{1}{2}}v^{\frac{\alpha}{2}}|\nabla\psi|(\alpha+1)\right]\di x_{f}\\ &\leq&(\alpha-1)\displaystyle\int\psi^{2(2\alpha+\sigma-1)}\lambda(v)v^{\alpha-2}|\nabla v|^{2}\di x_{f}+\frac{(\alpha+\sigma-1)^{2}}{\alpha-1}\displaystyle\int\psi^{2(\alpha+\sigma-1)-2}\lambda(v)v^{\alpha}|\nabla\psi|^{2}\di x_{f}. \end{eqnarray*} Thus, \begin{eqnarray} \displaystyle\int \psi^{2(\alpha+\sigma-1)}\lambda(v)bv^{\alpha+\sigma-1}\di x_{f}&\leq&\displaystyle\int\psi^{2(\alpha+\sigma-1)}\lambda(v)v^{\alpha}\di x_{f}\nonumber\\ &&+\frac{(\alpha+\sigma-1)^2}{\alpha-1}\displaystyle\int\psi^{2(\alpha+\sigma-1)-2(1-\frac{1}{\zeta})}\lambda(v)v^{\alpha}(\psi^{-\frac{1}{\zeta}}|\nabla\psi|)^{2}\di x_{f}.\label{estimate1} \end{eqnarray} We use Hölder inequality (with coeficients $p,q$ to be chosen) in order to estimate the last integral of the (RHS) that we named $(I)$. In fact, \begin{eqnarray*} (I)&=&\displaystyle\int\left\{\left[\psi^{2(\alpha+\sigma-1)}\lambda(v)b\right]^{\frac{1}{p}}v^{\alpha}\right\}\left\{\left[\psi^{2(\alpha+\sigma-1)}\lambda(v)b\right]^{\frac{1}{q}}b^{-1}\psi^{-2(1-\frac{1}{\zeta})}\right\}\left\{\psi^{-\frac{1}{\zeta}}|\nabla\psi|\right\}^{2}\di x_{f}\\ &\leq&\frac{C_{0}^{2}}{r^{2}}\left[\displaystyle\int\psi^{2(\alpha+\sigma-1)}\lambda(v)bv^{\alpha p}\di x_{f}\right]^{\frac{1}{p}}\left[\displaystyle\int \psi^{2(\alpha+\sigma-1)+2q(\frac{1}{\zeta}-1)}\lambda(v)b^{1-q}\di x_{f}\right]^{\frac{1}{q}}. \end{eqnarray*} Choosing $$p=\frac{\alpha+\sigma-1}{\alpha}\quad \mbox{and}\quad q=\frac{\alpha+\sigma-1}{\sigma-1},$$ we have $$2(\alpha+\sigma-1)+2q\left(\frac{1}{\zeta}-1\right)=(\alpha+\sigma-1)\left[2+\frac{2}{\sigma-1}\left(\frac{1}{\zeta}-1\right)\right]>0,$$ by our choice of $\zeta$. Since $\psi\leq 1$ and $\psi\equiv 0$ off $B_{2r}$, we conclude \begin{eqnarray*} (I)&\leq&\frac{C_{0}^{2}}{r^{2}}\left[\displaystyle\int\psi^{2(\alpha+\sigma-1)}\lambda(v)bv^{\alpha+\sigma-1}\di x_{f}\right]^{\frac{\alpha}{\alpha+\sigma-1}}\left[\displaystyle\int_{B_{2r}}\lambda(v)b^{-\frac{\alpha}{\sigma-1}}\di x_{f}\right]^{\frac{\sigma-1}{\alpha+\sigma-1}}\\ &=&\frac{C_{0}^2}{r^{2}b}\left[\displaystyle\int\psi^{2(\alpha+1)}\lambda(v)bv^{\alpha+1}\di x_{f}\right]^{\frac{\alpha}{\alpha+\sigma-1}}\left[\displaystyle\int_{B_{2r}}\lambda(v)b\di x_{f}\right]^{\frac{\sigma-1}{\alpha+\sigma-1}}. \end{eqnarray*} Then, replacing in \eqref{estimate1}, we have \begin{eqnarray*} \left[\displaystyle\int\psi^{2(\alpha+\sigma-1)}\lambda(v)bv^{\alpha+\sigma-1}\di x_{f}\right]^{\frac{\sigma-1}{\alpha+\sigma-1}}\leq \frac{C_{0}^2}{r^{2}b}\frac{(\alpha+\sigma-1)^2}{\alpha-1} \left[\displaystyle\int_{B_{2r}}\lambda(v)b\di x_{f}\right]^{\frac{\sigma-1}{\alpha+\sigma-1}} \end{eqnarray*} To conclude we just estimate the (LHS) from below since $\psi\equiv 1$ on $B_{r}$ and $\lambda(v)=0$ if $v\leq \gamma$: $$\displaystyle\int\psi^{2(\alpha+\sigma-1)}\lambda(v)bv^{\alpha+\sigma-1}\di x_{f}\geq \displaystyle\int_{B_{r}\cap\Omega_{\gamma}}\psi^{2(\alpha+\sigma-1)}\lambda(v)bv^{\alpha+\sigma-1}\di x_{f}\geq \gamma^{\alpha+\sigma-1}\int_{B_{r}\cap \Omega_{\gamma}}b\lambda(v)\di x_{f}.$$ Then $$\int_{B_{r}(\mathcal{O})}\lambda(v)\di x_{f}\leq \left[\frac{1}{\gamma}\frac{C}{r^{2}}\frac{1}{b}\frac{(\alpha+\sigma-1)^{2}}{\alpha-1}\right]^{\alpha+\sigma-1}\int_{B_{2r}(\mathcal{O})}\lambda(v)\di x_{f}.$$ The thesis follows by choosing an increasing sequence of functions $\lambda_{j}$ pointwise converging to the characteristic function of $\left\{t>\gamma\right\}$. \end{proof} We are ready to prove that $$\sup v<\infty.$$ Assume by contradiction that this is not the case, so $\Omega_{\gamma}\neq\emptyset$ for each $\gamma>0$. Fix $\gamma$, define $$G(r)= \vol_{f}(\Omega_{\gamma}\cap B_{r}(\mathcal{O})),$$ and choose $R>0$ and $\alpha$ so that $$G(r)>0, \quad \alpha+\sigma-1=\frac{b\gamma r^{2}}{8C}\quad \forall r>R.$$ Then by Claim 1, $$G(r)\leq \left[\frac{1}{\gamma}\frac{C}{r^{2}}\frac{1}{b}\frac{(\alpha+\sigma-1)^{2}}{\alpha-1}\right]^{\alpha+\sigma-1}G(2r)\leq \left[\frac{4C}{b\gamma r^{2}}(\alpha+\sigma-1)\right]^{\alpha+\sigma-1}G(2r)=\left(\frac{1}{2}\right)^{\frac{b\gamma}{8 C}r^2}G(2r),$$ for $r>R$. By iterating (see \cite[Lemma 4.7]{rigoli2}), we can conclude $$ \displaystyle\liminf_{r\to\infty}\frac{1}{r^2}\ln{\vol_{f}(B_{r}(\mathcal{O}))}\geq S \gamma \ln{2}, $$ where $S$ is a positive constant independent of $\gamma$. This contradicts the assumption if $\gamma$ is large enough. \end{proof} The second proposition we need is the following weak maximum principle at infinity: \begin{proposition}\label{prop_Fsup} Let $(M^{m},g,u)$ be a complete manifold with $\partial M=u^{-1}(0)$ and $u>0$ in $\mathring{M}$. Let $v\in {\rm Lip}_{\rm{loc}}(M)$ solve \begin{equation}\label{lap_estimate1} \Delta_{f}v\geq F(v)\qquad\mbox{on}\quad\Mo \end{equation} for some $F\in C(\mathbb{R})$, and where $f=-\ln{u}$. If \begin{equation}\label{hip_volume} \displaystyle\liminf_{r\to\infty}\frac{\ln{\vol_{f}(B_{r}(\mathcal{O}))}}{r^2}<\infty, \end{equation} for some origin $\mathcal{O}\subset M$ and $$\sup_{M}v<\infty,$$ then $$F(\sup_{M} v)\leq 0$$ \end{proposition} Proposition \ref{prop_Fsup} is a version, for weighted operators and manifolds with boundary, of \cite[Theorem 4.1]{rigoli2}. The proof follows verbatim the one therein once we observe the following: \begin{itemize} \item as in Proposition \ref{vol_control1}, the introduction of the weight $e^{-f}$ in measures and operators does not make any difference, and since $e^{-f}=u=0$ on $\partial M$, boundary terms do not appear; \item the regularity $v\in C^{1}(M)$ required in the theorem can be weakened to $v\in {\rm Lip}_{\rm{loc}}(M)$ by using the weak formulation of \eqref{lap_estimate1}. \end{itemize} Putting together Propositions \ref{vol_control1} and \ref{prop_Fsup} we obtain the next result. \begin{theorem}\label{liouville_type} Let $(M^{m},g,u)$ be a complete manifold with $\partial M=u^{-1}(0)$ and $u>0$ in $\mathring{M}$. Let $v\in {\rm Lip}_{\rm{loc}}(M)$ solve \begin{equation}\label{lap_estimate} \Delta_{f}v\geq bv^{\sigma}-av\qquad\mbox{on}\quad\Mo \end{equation} for constants $b>0$, $a\geq 0$ and $\sigma>1$, where $f=-\ln{u}$. If \begin{equation}\label{hip_volume} \displaystyle\liminf_{r\to\infty}\frac{\ln{\vol_{f}(B_{r}(\mathcal{O}))}}{r^2}<\infty, \end{equation} for some origin $\mathcal{O}\subset M$, then $$\displaystyle\sup_{M} v\leq \left(\frac{a}{b}\right)^{\frac{1}{\sigma-1}}.$$ \end{theorem} \begin{proof} By contradiction assume that the thesis fails and pick $$\gamma\in \left(\left(\frac{a}{b}\right)^{\frac{1}{\sigma-1}},\sup_{M} v\right).$$ There exist a constant $C_{\gamma}>0$ such that $$\Delta_{f}v\geq C_{\gamma}v^{\sigma}\qquad\mbox{on}\quad \Omega_{\gamma} $$ By Proposition \ref{vol_control1}, $\sup v<\infty$. Using Proposition \ref{prop_Fsup} with $F(t)=bt^{\sigma}-at$, we conclude the thesis. \end{proof} \begin{remark} We stress that in the above result $\partial M$ may be non-compact, and no assumption on $v_{|_{\partial M}}$ is made besides its local Lipschitz coninuity. \end{remark} We eventually prove Theorem \ref{vanishing_map}. Since $v=|\di \varphi|^{2}$ satisfies \eqref{eq_dphi}, which in our assumptions imply $$\Delta_{f}v\geq -av+\frac{2}{m}(1-(m-1)\kappa)v^{2},$$ the result is a direct application of Theorem \ref{liouville_type} once we prove \eqref{hip_volume}. This folows by standard comparison theory. Consider a smooth relatively compact open set $\mathcal{O}$ containing $\partial M$. By \eqref{map_source} and since $V(\varphi)$ is bounded from below, there exists a constant $\bar{\kappa}>0$ such that $$\Ric_{f}^{m+1}\geq -\bar{\kappa}^{2}m g.$$ Fix a point $x\in M\setminus\mathcal{O}$, which is not in the cut locus ${\rm cut}(\mathcal{O})$, and a unit speed minimizing geodesic $\gamma:[0,r(x)]\to M$ from $\partial\mathcal{O}$ to $x$. We have the following weighted Bochner formula for the $f$-laplacian of $r$ (see \cite[Lemma 2.2]{mrs}) \[ \frac{1}{m} (\Delta_{f} r)^2 + \langle \nabla \Delta_{f} r, \nabla r \rangle + \Ric_{f}^{m+1} (\nabla r, \nabla r) \leq 0, \] which implies $$z'+\frac{z^{2}}{m}-m\bar{\kappa}^{2}\leq 0,$$ where $z(t)=\Delta_{f}r(\gamma(t))$. Furthermore $$z(0)\leq\bar{\lambda}=\max_{\partial\mathcal{O}}H_{f},$$ where $H_{f}$ is the weighted mean curvature of $\partial\mathcal{O}$ in the direction pointing towards $\mathcal{O}$. The classical Riccati comparison (see \cite[Theorem 3.8]{bmpr} for a version of this inequality using the distance from a compact set, as described above) gives $$\Delta_{f}r\leq m\frac{h'(r)}{h(r)}\qquad {\rm at }\, x,$$ where $h$ is a solution of \begin{equation} \left\{ \begin{array}{ll} h''-\bar{\kappa}^{2}h= 0 & \text{on}\quad \R^+, \\[0.2cm] h(0)=1, & h'(0)=\bar{\lambda}. \end{array} \right. \end{equation} The inequality also hold in the weak sense, see (see \cite[Lemma 2.5]{rigoli3}). By integration and since $h=\cosh (\sqrt{\bar{\kappa}t})+ \frac{\bar{\lambda}}{\bar{\kappa}}\sinh(\sqrt{\bar{\kappa}}t)$ we obtain the volume inequality (see \cite[Theorem 2.14]{rigoli3}) \[ \vol_{f}(B_{r}(\mathcal{O}) \setminus B_{r_{0}}(\mathcal{O})) \leq C \left[\int_{r_{0}}^{r} h^{m-1}(t) \, dt\right]\leq C_{1}e^{C_{2}r} \] for sonstants $C_1,C_2>0$. The desired inequality \eqref{hip_volume} follows at once. \vspace{0.3cm} \noindent \textbf{Acknowledgements.} A.F. would like to thank the hospitality of the Mathematics Department of Università degli Studi di Torino, where part of this work was carried out. A.F. was partially supported by Conselho Nacional de Desenvolvimento Científico e Tecnológico (CNPq) of the Ministry of Science, Technology and Innovation of Brazil, Grants 316080/2021-7 and 200261/2022-3. This work also was funded by Paraíba State Research Foundation - Programa Primeiros Projetos (Grant 2021/3175) and by Program Research in Pairs CIMPA-ICTP 2024. L.M. and M.R. were supported by the PRIN project no. 20225J97H5 ``Differential-geometric aspects of manifolds via Global Analysis''. \bigskip \bibliographystyle{plain} \begin{thebibliography}{99} \bibitem{agost} V. Agostiniani and L. Mazzieri, \emph{On the geometry of the level sets of bounded static potentials.} Commun. Math. Phys. {\bf 355} (2017), 261--301. \bibitem{aleale} R. Alexander and S. Alexander, \emph{Geodesics in Riemannian manifolds-with-boundary.} Indiana Univ. Math. J. \textbf{30} (1981), no. 4, 481--488. \bibitem{almgren76} F. J. Almgren, \emph{Existence and regularity almost everywhere of solutions to elliptic variational problems with constraints}, Mem. Amer. Math. Soc. \textbf{4} (1976), no. 165, viii+199. \bibitem{ambrozio} L. Ambrozio, \emph{On static three-manifolds with positive scalar curvature.} J. Differential Geom. \textbf{107} (2017), no. 1, 1--45. MR3698233 \bibitem{rigoli1} G. Albanese, M. Rigoli, \emph{A Schwarz-type lemma for noncompact manifolds with boundary and geometric applications}. Comm. Anal. Geom. {\bf 25} (4) (2017) 719--749. \bibitem{amr} L. J. Al\'ias, P. Mastrolia, M. Rigoli, \emph{Maximum principles and geometric applications.} Springer Monographs in Mathematics, Springer, Cham, 2016. MR3445380 \bibitem{anderson} M.T. Anderson, \emph{On stationary vacuum solutions to the Einstein equations.} Ann. Henri Poincar\'e 1 (2000), no.5, 977--994. \bibitem{andrade} M. Andrade, B. Leandro, R. Lousa, \emph{On the Geometry of Electrovacuum Spaces in Higher Dimensions.} Ann. Henri Poincaré {\bf 24} (2023), 3153--3184. \bibitem{ans21} A. Anselli, \emph{Bach and Einstein's equations in presence of a field.} Int. J. Geom. Methods Mod. Phys. \textbf{18} (2021), no. 5, 2150077, 68 pp. MR4254759 \bibitem{acr21} A. Anselli, G. Colombo, M. Rigoli, \emph{On the geometry of Einstein-type structures.} Nonlinear Anal. \textbf{204} (2021), 112198, 84 pp. MR4184679 \bibitem{fogborg} S. Borghini and M. Fogagnolo, \emph{Comparison geometry for substatic manifolds and a weighted Isoperimetric Inequality}. Preprint (2023). Available at arXiv:2307.14618 [math.DG] \bibitem{BGH} {W. Boucher, G. W. Gibbons and G. T. Horowitz}, \emph{Uniqueness theorem for anti-de Sitter spacetime}, {Phisical Review} D (3) \textbf{30} (1984), no. 12, 2447--2451. \bibitem{bmpr} B. Bianchini, L. Mari, P. Pucci, and M. Rigoli, \emph{Geometric analysis of quasilinear inequalities on complete manifolds - Maximum and compact support principles and detours on manifolds.} Frontiers in Mathematics. Birkhäuser/Springer, Cham, 2021, x+286 pp. \bibitem{bouch} {W. Boucher}, \emph{Cosmic no-hair theorems}. Classical general relativity (London, 1983), 43-52, Cambridge Univ. Press, Cambridge, 1984. \bibitem{brendle} S. Brendle, \emph{Constant mean curvature surfaces in warped product manifolds.} Publ. Math. Inst. Hautes Études Sci. {\bf 117} (2013), 247–269. \bibitem{bunting} G. L. Bunting, and A. K. M. Masood-Ul-Alam, \emph{Nonexistence of multiple black holes in asymptotically Euclidean static vacuum space-time.} Gen. Relativ. Gravit. {\bf 19} (1987), 147--154. \bibitem{case} J.S. Case, \emph{The nonexistence of quasi-Einstein metrics.} Pacific J. Math. 248 (2010), no.2, 277-284. \bibitem{cerd} C. Cederbaum, \emph{Uniqueness of photon spheres in static vacuum asymptotically flat space-times}, Complex Analysis \& Dynamical Systems VI, Contemp. Math. AMS {\bf 667} (2015), 86–99. \bibitem{cheeger} J. Cheeger and D. Gromoll, \emph{The splitting theorem for manifolds of nonnegative Ricci curvature}. J. Differential Geometry {\bf 6} (1971/72), 119–128. \bibitem{detang} X. Cheng, T. Mejia and D. Zhou, \emph{Stability and compactness for complete $f$-minimal surfaces}. Trans. Amer. Math. Soc. {\bf 367} (2015), 4041-4059. \bibitem{Crus} {P. T. Chru\'sciel}, \emph{Remarks on rigidity of the de Sitter metric}, {homepage.univie.ac.at/piotr.chrusciel/papers/ deSitter/deSitter2.pdf} \bibitem{CB} Y. Choquet-Bruhat, \emph{General Relativity and the Einstein Equations.} Oxford University Press (2009). \bibitem{cmrigoli} G. Colombo, M. Mariani, M. Rigoli, \emph{A sharp Eells-Sampson type theorem under positive sectional curvature upper bounds}, J. Math. Anal. Appl. {\bf 540} (1) (2024), 128584. \bibitem{cortier_minerbe} J. Cortier and V. Minerbe, \emph{On complete stationary vacuum initial data.} J. Geom. Phys. {\bf 99} (2016), 20-27. \bibitem{corvino} J. Corvino, \emph{Scalar curvature deformation and a gluing construction for the Einstein constraint equations}, Commun. Math. Phys. {\bf 214} (2000), 137–189. \bibitem{benedito} F. Coutinho, B. Leandro, \emph{Mean-stable surfaces in static Einstein–Maxwell theory.} Lett. Math. Phys. {\bf 112} (2022) no. 128. https://doi.org/10.1007/s11005-022-01623-1 \bibitem{costa} J. Costa, R. Diógenes, N. Pinheiro, and E. Ribeiro Jr. \emph{Geometry of static perfect fluid space-time}. Class. Quantum Grav. {\bf 40} (2023) 205012. \bibitem{tiarlos} T. Cruz, V. Lima, A. de Sousa, \emph{Min-max minimal surfaces, horizons and electrostatic systems.} To appear in J. Differential Geom. arXiv:1912.08600v3 [math.DG] \bibitem{delellis} C. De Lellis, \emph{The regularity theory for the area functional (in geometric measure theory).} ICM—International Congress of Mathematicians. Vol. 2. Plenary lectures, 872–913. \bibitem{esch} J.H. Eschenburg, {\emph The splitting theorem for space-times with strong energy condition.} J. Differential Geom. {\bf 27} (1988), n. 3, 477–491. \bibitem{eschenburg} J.H. Eschenburg, E. Heintze, \emph{Comparison theory for Riccati equations.} Manuscr. Math. {\bf 68} (1990), 209–214. \bibitem{fried} {H. Friedrich}, \emph{Existence and structure of past asymptotically simple solutions of Einstein's field equations with positive cosmological constant,} {J. Geom. Phys.} \textbf{3} (1986), no. 1, 101-117. \bibitem{fm75} A.E. Fischer, J.E. Marsden, \emph{Deformations of the scalar curvature.} Duke Math. J. \textbf{42} (1975), no. 3, 519--547. \bibitem{galloway} G. Galloway, \emph{On the topology of black holes.} Comm. Math. Phys. {\bf 151} (1993), no. 1, 53-66. \bibitem{gal1} G. Galloway, \emph{Some global results for asymptotically simple space-times. The conformal structure of space-time,} {Lecture Notes in Phys.} \textbf{604}(2002), 51-60. \bibitem{glmm} E. Gama, J. de Lira, L. Mari, and A. Medeiros, \emph{A Barrier principle at infinity for varifolds with bounded mean curvature.} J. Lond. Math. Soc. {\bf 2} (2022), 1–35. \bibitem{garcia} E. García-Río and D.N. Kupeli, {\emph Some splitting theorems for stably causal spacetimes}, Gen. Rel. Grav. {\bf 30} (1998), no. 1, 35-44. \bibitem{ghp} G. W. Gibbons, S. A. Hartnoll, C. N. Pope, \emph{Bohm and Einstein-Sasaki metrics, black holes, and cosmological event horizons.} Phys. Rev. D. {\bf 67} (8) (2003) 084024. \bibitem{hawking} S. Hawking and G. Ellis, \emph{The Large Scale Structure of Space-Time}, Cambridge University Press, Cambridge (1973). \bibitem{hmm} L.-H. Huang, D. Martin, P. Miao, \emph{Static potentials and area minimizing hypersurfaces.} Proc. Am. Math. Soc. {\bf 146}(6) (2018), 2647–2661. \bibitem{israel} W. Israel, \emph{Event horizons in static vacuum space-times.} Phys. Rev. {\bf 164} (1967), 1776–1779. \bibitem{javaloyes} M.A. Javaloyes and M. Sanchez, {\emph A note on the existence of standard splittings for conformally stationary spacetimes.} Class. Quantum Gravity {\bf 25} (2008) 168001. \bibitem{kasue} A. Kasue, \emph{Ricci curvature, geodesics and some geometric properties of Riemannian manifolds with boundary}. J. Math. Soc. Japan {\bf 35.1} (1983), 117-131. \bibitem{koba} O. Kobayashi, \emph{A differential equation arising from scalar curvature.} J. Math. Soc. Japan {\bf 34} (1982), 665. \bibitem{laf} J. Lafontaine, \emph{Sur la géométrie d'une généralisation de l'equation différentielle d'Obata.} J. Math. Pures Appl. {\bf 9} (1983), 63. \bibitem{lixia_17} J. Li and C. Xia, \emph{An integral formula for affine connections}. J. Geom. Anal. 27 (2017), no.3, 2539-2556. \bibitem{mrs} L. Mari, M. Rigoli, A.G. Setti, \emph{Keller-Osserman conditions for diffusion-type operators on Riemannian manifolds.} J. Funct. Anal. {\bf 258} (2010) 665–712. \bibitem{penrose} R. Penrose, \emph{Gravitational Collapse and Space-Time Singularities}. Phys. Rev. Lett. {\bf 14}, 57 (1965). \bibitem{petersen} P. Petersen, \emph{Riemannian Geometry (3rd ed.).} Graduate texts in Mathematics {\bf 171} (2016) Springer-Verlag, xiii+499. \bibitem{rigoli2} S. Pigola, M. Rigoli, and A. G. Setti, \emph{Maximum principles on Riemannian manifolds and applications}. Mem. Amer. Math. Soc. {\bf 174} (2005), no. 822, x+99. \bibitem{rigoli3} S. Pigola, M. Rigoli, and A. G. Setti, \emph{Vanishing and Finiteness Results in Geometric Analysis: a Generalization of the Bochner technique} Progress in Mathematics {\bf 266} (2008) Birkh\"auser. \bibitem{reilly} R. C. Reilly, \emph{Applications of the Hessian operator in a Riemannian manifold}. Indiana Univ. Math. J. {\bf 26} (3) (1977), 459-472. \bibitem{reiris} M. Reiris, \emph{On static solutions of the Einstein-scalar field equations}. Gen. Relativ. Gravit. 49 (2017), no. 3, 46. \bibitem{reiris_compa} M. Reiris, \emph{Static solutions from the point of view of comparison geometry.} J. Math. Phys. 53 (2012), no. 1, 012501, 31 pp. \bibitem{robinson} D. C. Robinson, \emph{A simple proof of the generalization of Israel's theorem.} Gen. Relat. Gravit. {\bf 8} (1977), 695--698. \bibitem{ses} {H. Seshadri,} \emph{On Einstein four-manifolds with $S^{1}$-actions}, { Math. Z.} \textbf{247} (2004), no. 3, 487-503. \bibitem{sh} {Y. Shen,} \emph{A note on Fischer-Marsden's conjecture,} { Proc. Am. Math. Soc.} \textbf{125} (1997), 901-905. \bibitem{taylor76} J. E. Taylor, \emph{The structure of singularities in soap-bubble-like and soap-film-like minimal surfaces}, Ann. Math. \textbf{103} (1976), 489--539. \bibitem{uh76} K. Uhlenbeck, \emph{Generic properties of eigenfunctions.} Amer. J. Math. {\bf 98} (4) (1976), 1059--1078. \bibitem{wald} R. M. Wald, \emph{General Relativity}. The University of Chicago Press, Chicago, USA, 1984. \bibitem{wang} M.-T. Wang, Y.-K. Wang and X. Zhang, \emph{Minkowski formulae and Alexandrov theorems in spacetime.} J. Differential Geom. {\bf 105} (2017), no. 2, 249–290, MR3606730, Zbl 06696281. \bibitem{wylie1} W. Wylie, \emph{A warped product version of the Cheeger–Gromoll splitting theorem.} Trans. Amer. Math. Soc. {\bf 369} (9) (2017), 6661–6681. \bibitem{wylie} W. Wylie and D. Yeroshkin, \emph{ On the geometry of Riemannian manifolds with density.} Preprint (2016). Available at arXiv:1602.08000 [math.DG] \end{thebibliography} \end{document}
2412.06829v2
http://arxiv.org/abs/2412.06829v2
Stably unactivated neurons in ReLU neural networks
\documentclass[11pt]{amsart} \usepackage[margin=1.25in]{geometry} \usepackage[colorlinks=true,citecolor=blue,linkcolor=blue]{hyperref} \usepackage{amsmath, amssymb, amsthm} \usepackage[dvipsnames]{xcolor} \usepackage{soul} \usepackage{graphicx} \usepackage{wrapfig} \usepackage{subcaption} \usepackage{soul,xcolor} \usepackage{tikz} \usepackage{todonotes} \usepackage{verbatim} \usepackage{float} \newtheorem{theorem}{Theorem}[section] \newtheorem{cor}[theorem]{Corollary} \newtheorem{prop}[theorem]{Proposition} \newtheorem{conj}[theorem]{Conjecture} \newtheorem{remark}{Remark} \newtheorem{lem}[theorem]{Lemma} \newtheorem{defn}[theorem]{Definition} \DeclareMathOperator{\Ima}{Im} \DeclareMathOperator{\code}{code} \newcommand{\R}{\mathbb{R}} \newcommand{\PP}{\overline{P}} \renewcommand{\P}{\mathbb{P}} \newcommand{\ImF}{\Ima(F_1)} \newcommand{\ImFP}{\Ima(F_1)'} \newcommand{\x}{\mathbf{x}} \newcommand{\thetaa}{\theta^\textbf{A}} \newcommand{\deltaa}{\delta^\textbf{A}} \newcommand{\Su}{\mathcal S} \newcommand{\Hp}[1]{\mathcal H_{#1}} \newcommand{\halfax}[1]{\mathtt{X}_{#1\ge0}} \newcommand{\bb}[1]{\mathbb{#1}} \newcommand{\df}{\dim_{\operatorname{fun}}} \definecolor{cornflower}{rgb}{0.5067352811380239, 0.4923960137178009, 0.7184954982852749} \newcommand{\commentNa}[1]{{\color{NavyBlue} #1}} \newcommand{\commentC}[1]{{\color{OliveGreen} #1}} \newcommand{\commentN}[1]{{\color{red!100!black} #1}} \newcommand{\commentG}[1]{{\color{blue!100!black} #1}} \newcommand{\commentE}[1]{{\color{yellow!50!red} #1}} \newcommand{\commentGQ}[1]{{\color{blue!55!red} #1}} \title{Stably Unactivated Neurons in ReLU Neural Networks} \author[N.\ Brownlowe]{Natalie Brownlowe} \author[C.R.\ Cornwell]{Christopher R.\ Cornwell} \author[E.\ Montes]{Ethan Montes} \author[G.\ Quijano]{Gabriel Quijano} \author[G.\ Stulman]{Grace Stulman} \author[N.\ Zhang]{Na Zhang} \date{\today} \begin{document} \begin{abstract} The choice of architecture of a neural network influences which functions will be realizable by that neural network and, as a result, studying the expressiveness of a chosen architecture has received much attention. In ReLU neural networks, the presence of \textit{stably unactivated} neurons can reduce the network's expressiveness. In this work, we investigate the probability of a neuron in the second hidden layer of such neural networks being stably unactivated when the weights and biases are initialized from symmetric probability distributions. For networks with input dimension $n_0$, we prove that if the first hidden layer has $n_0+1$ neurons then this probability is exactly $\frac{2^{n_0} + 1}{4^{n_0+1}}$, and if the first hidden layer has $n_1$ neurons, $n_1\le n_0$, then the probability is $\frac{1}{2^{n_1+1}}$. Finally, for the case when the first hidden layer has more neurons than $n_0+1$, a conjecture is proposed along with the rationale. Computational evidence is presented to support the conjecture. \end{abstract} \maketitle \section{Introduction and Main Results}\label{sec:Intro} A feedforward neural network, with selected parameters, has an associated \emph{network function} $F:\bb R^{n_0}\to\bb R^{n_L}$ for some $n_0, n_L \in \bb N$. This associated function takes as input a data point ${\bf x}$ with $n_0$ \emph{features} (i.e., ${\bf x}\in \bb R^{n_0}$) and, commonly, the output is used for network \emph{prediction} on how to classify ${\bf x}$ into one of $n_L$ possible classes. Throughout this paper, we exclusively consider \emph{ReLU networks} that use activation function $\sigma(x) = \max\{0, x\}$ and have \emph{fully-connected} layers (see Section \ref{sec:prelim}). We study the occurrence of a phenomenon for the network, the existence of what are called \emph{stably unactivated neurons} (see Definition \ref{defn:stably-unactivated}), which influence not only what the network function may be, but the degree to which it can be altered via ``small updates'' to the parameters. Neural networks are described, in part, by an \emph{architecture} that consists of a list of natural numbers. Feedforward neural networks with $L$ fully-connected layers and architecture $(n_0, n_1, \ldots, n_L)$ are parameterized by $\theta \in \bb R^D$, where $D = \sum_{i=1}^Ln_{i}(n_{i-1}+1)$. For each $\theta\in\bb R^D$, denote the network function for that neural network by $F^\theta$. Due to symmetries that arise from permuting neurons (coordinates) within a layer, it is well-known that the mapping $\theta\mapsto F^\theta$, from the parameter space $\bb R^D$ to the space of functions realizable by this architecture, is not one-to-one. Moreover, as discussed in \cite{BuiLampert2020} and \cite{KordingRolnick2020}, for ReLU networks the mapping $\theta\mapsto F^\theta$ is even farther from being one-to-one (provided $L\ge 2$), since it is possible to perform scaling/inverse-scaling on the weights and the bias connected to a neuron in a hidden layer without producing any change to $F^\theta$. In order to further study the relationship between the parameter space of a given network architecture and the space of realizable functions, the \emph{functional dimension} of $\theta$, denoted $\df(\theta)$, was introduced in \cite{GLMW2022}. As stated in \cite{GLMW2022}, intuitively speaking, $\df(\theta)$ is the number of degrees of freedom within the space of realizable functions that are attainable by arbitrarily small perturbations of $\theta$. Given $\theta_0\in\bb R^D$, the larger that $\df(\theta_0)$ is, the less ``redundancy'' the mapping $\theta\mapsto F^\theta$ exhibits near $\theta_0$. Additionally, when training a ReLU network with gradient descent, updates are restricted to a subspace of the tangent space of $\bb R^D$ at $\theta_0$, which at most has dimension $\df(\theta_0)$. Interestingly, it was found in \cite{GLMW2022} that, with a fixed architecture, the value of $\df(\theta)$ varies across parameter space. Hence, initialization, or a round of training, might produce a $\theta$ with relatively small functional dimension. As a consequence, the freedom to alter the network function within the space of realizable functions would then be significantly restricted, affecting the ability to train the neural network model effectively. A consequence of the fact that scaling/inverse scaling on any neuron of a hidden layer leaves the network function unchanged is that the functional dimension of a parameter in $\bb R^D$ is always strictly less than $D$ (provided that there is at least one hidden layer). Using this observation, given an architecture $(n_0, n_1,\ldots, n_L)$ and $D$, as defined above, a tight upper bound on the functional dimension of $\theta\in\bb R^D$ was found in \cite[Theorem 7.1]{GLMW2022}, \begin{equation}\label{eqn:dimfun-upperbound} \df(\theta) \le n_L + \sum_{i=1}^Ln_in_{i-1}. \end{equation} The upper bound (\ref{eqn:dimfun-upperbound}) is tight in the sense that there are architectures for which there exists a parameter $\theta$ which has functional dimension equal to the right-hand side of (\ref{eqn:dimfun-upperbound}) \cite[Theorem 8.11, for example]{GLMW2022}. However, as is well-known to experts and practitioners, ReLU networks will often contain some neurons in hidden layers that are ``dead,'' or \emph{unactivated}. If such a neuron of a network, given by parameter $\theta$, is \emph{stably unactivated} (the generic case of being unactivated; see Definition \ref{defn:stably-unactivated}) then it is impossible for $\df(\theta)$ to achieve equality in (\ref{eqn:dimfun-upperbound}) \cite[Theorem 7.3]{GLMW2022}. It is not difficult to see that if the bias of a neuron in layer $i$, $i\in\{2,\ldots, L-1\}$, and every weight ``leading to'' that neuron are all negative, then the neuron will be stably unactivated. When the weights and biases are independent and identically distributed random variables with a distribution symmetric about $0$, then the probability that this occurs is $\frac{1}{2^{1+n_{i-1}}}$. As has been observed, experimental results show that the probability of a neuron in a hidden layer being unactivated is significantly larger. In this article, as a step in the direction of understanding the functional dimension as a random variable, we study the probability that a given neuron of the network is stably unactivated. The earliest hidden layer where this probability is non-zero is the second layer of the network, at which our results are focused. As we note after our main results, generally the (unconditional) probability of a neuron in later layers being stably unactivated increases with the number of the layer. For our results, we make a mild assumption about the weights and biases that comprise the components of $\theta$. \begin{theorem} \label{thm:small-n1} Let $(n_0,n_1,\ldots,n_L)$ be an architecture of a ReLU neural network. Suppose that the weights and biases in each single layer are selected i.i.d.\ from a probability distribution on $\bb R$ that is symmetric about $0$. If $n_1 \leq n_0$, then for any one of the $n_2$ neurons in the second layer, the probability of that neuron being stably unactivated is precisely \[\frac{1}{2^{{n_1}+1}}.\] \end{theorem} \begin{theorem} Let $(n_0,n_1,\ldots,n_L)$ be an architecture of a ReLU neural network. Suppose that the weights and biases in each single layer are selected i.i.d.\ from a probability distribution on $\bb R$ that is symmetric about $0$. If $n_1 = n_0 + 1$, then for any one of the $n_2$ neurons in the second layer, the probability that the neuron is stably unactivated is precisely \[\frac{2^{n_0} + 1}{4^{{n_0}+1}}.\] \label{thm:main} \end{theorem} As mentioned above, there is a condition that is clearly sufficient for a neuron from the second layer to be stably unactivated and that occurs with probability $\frac{1}{2^{n_1+1}}$. By Theorems \ref{thm:small-n1} and \ref{thm:main}, when $n_1 \le n_0+1$ and $n_1$ is sufficiently large, this probability is either equal or approximately equal to the probability that the neuron is stably unactivated. However, this conclusion finely depends on the conditions, including the layer of the neuron in question. For example, there is a positive probability that the image of the function for the network truncated at the second layer is compact or, at least, the projection of the image to one of the coordinates is compact. This increases the probability of a stably unactivated neuron. \begin{remark}\label{rem:precomposing} For a neural network with architecture $(n_0,n_1,\ldots,n_L)$ and $\theta$ selected as in the above theorems, let $p(n_0,n_1)$ be the probability in question in those theorems. Additionally, let $m_0,m_1,\ldots,m_{k-1}$ be positive integers and consider a ReLU network with architecture $(m_0,m_1,\ldots,m_{k-1},n_0,\ldots, n_L)$, with the weights and biases in layers $k+1,\ldots,k+L$ selected identically to those in $\theta$. Choosing a neuron in hidden layer $k+2$, the probability that it is stably unactivated is strictly larger than $p(n_0,n_1)$. \end{remark} Somewhat related to Remark \ref{rem:precomposing} is the main result in \cite{Lu2020}. A careful reading of the proof of that main result indicates the following. Given any fixed $N > 0$, if we impose the constraint that $n_\ell \le N$ for all $n_\ell$ in networks with architecture $(n_0,n_1,\ldots,n_L)$, then the probability that some layer of the network consists of \emph{only} stably unactivated neurons at $\theta$ limits to 1, as $L \to \infty$. (The assumptions on $\theta$ being the same as in Theorem \ref{thm:main}.) Remark \ref{rem:precomposing} hints at the idea of understanding a neural network as a composition of networks. The possibility of stably unactivated neurons in some layer then makes it relevant to consider architectures where $n_1$ is large, relative to $n_0$. We provide some evidence and rationale that, in this case, the behavior of the probability addressed in Theorems \ref{thm:small-n1} and \ref{thm:main} is controlled by $n_0$ rather than $n_1$. \begin{conj} \label{conj:large-n1} Consider architectures $(n_0,n_1,\ldots,n_L)$ with $n_0$ fixed and suppose that the weights and biases in each single layer are selected i.i.d.\ from a probability distribution on $\bb R$ that is symmetric about $0$. Then there exists a constant $c > 0$ such that for $n_1$ sufficiently large, the probability for any one of the $n_2$ neurons in the second layer being stably unactivated is at least $\frac{c}{4^{n_0}}$. \end{conj} The remainder of the paper is organized as follows. After reviewing relevant definitions and background in Section \ref{sec:prelim}, we prove in Section \ref{sec:lemmas} some results on hyperplane arrangements, and we discuss a partition on hyperplanes in $\bb R^{n_1}$ that will be helpful in proving a key lemma, needed for the proof of Theorem \ref{thm:main}. Sections \ref{sec:proof-main} and \ref{sec:configs} contain the proofs of Theorems \ref{thm:small-n1} and \ref{thm:main}, the key Lemma \ref{Lemma: Sum of delta} being proved in Section \ref{sec:configs}. Finally, in Section \ref{sec:general architectures} we discuss Conjecture \ref{conj:large-n1}. In addition to the rationale for the conjecture, we provide some supporting computational evidence. \section{Notation and Background}\label{sec:prelim} In this section we introduce terminology and notation that will be needed throughout the paper. Hyperplane arrangements and ideas related to them will be important for the discussion. After reviewing hyperplane arrangements, we discuss ReLU neural networks and their stably unactivated neurons, which are the objects of our main interest. \subsection{Hyperplane arrangements, regions, and codewords of regions.} For an integer $n \ge 1$, a hyperplane in $\bb R^{n}$ is the solution set to an affine-linear equation, \begin{equation}\label{eqn:affine-linear} \{{\bf x}\in\R^n\ |\ {\bf w}\cdot{\bf x} + b = 0\}, \end{equation} where $b\in\R$ and ${\bf w} \in \R^n$ is a nonzero vector. \begin{defn} \label{defn:arrangement} A $\textbf{hyperplane arrangement}$ in $\bb R^n$ is a finite ordered set of hyperplanes in $\R^n$. If $\mathcal{A}$ is a hyperplane arrangement in $\bb R^n$, the connected components of \mbox{$\bb R^n\setminus\bigcup_{H\in\mathcal{A}}H$} are called the \textbf{regions} of $\mathcal{A}$. \end{defn} A vector ${\bf w}$, that helps to determine a hyperplane as above, is called a \emph{normal vector} to that hyperplane. While not unique, a choice of normal vector for a hyperplane $H$ provides a natural way to specify which component of $\bb R^n \setminus H$ is called the \emph{positive half-space} $H^+$, and which is the \emph{negative half-space} $H^-$. Specifically, given ${\bf w}\in\bb R^n$ and $b\in\bb R$ so that $H = \{{\bf x}\in\R^n\ |\ {\bf w}\cdot{\bf x} + b = 0\}$, we may set \[H^{+} = \{{\bf x}\ |\ {\bf w}\cdot{\bf x} + b >0\};\] \[H^{-} = \{{\bf x}\ |\ {\bf w}\cdot{\bf x} + b <0\}.\] \begin{defn} \label{defn:coorientation} Let $\mathcal{A} = \{H_1, H_2,\ldots, H_m\}$ be a hyperplane arrangement in $\R^n$. A \textbf{coorientation} on $\mathcal{A}$ is a specification, for each $i=1,2,\ldots,m$, of which component of $\R^n\setminus H_i$ is the positive half-space {--} the other component being the negative half-space. \end{defn} \begin{remark} In this paper, to refer to a hyperplane arrangement with a given coorientation we use bold-face notation. For example, a cooriented hyperplane arrangement may be written as $\textbf{A} = \{H_1, H_2, \ldots, H_m\}$, indicating that the half-spaces $H_1^+, H_2^+,\ldots, H_m^+$ have been specified. \end{remark} Given a choice of normal vector for each hyperplane in a hyperplane arrangement $\mathcal{A}$, our previous comments indicate a corresponding natural choice of coorientation on $\mathcal A$. In much of the paper we will assume a coorientation determined by such a choice. Additionally, in the cooriented setting, note that each region of the hyperplane arrangement is determined by knowing which positive half-space it is contained in. This information will play a useful role. Following \cite{ItskovKuninRosen20}, we make the following definition.\footnote{Our codes use vectors in $\{+,-\}^m$ rather than subsets of $\{1,2,\ldots,m\}$ (to indicate containment in a positive halfspace) as done for \emph{hyperplane codes} in \cite{ItskovKuninRosen20}. Other than notational, the only difference in approach is that we care only about points in the union of the regions of the hyperplane arrangement, i.e., in $\bb R^n\setminus\bigcup_{H\in\mathcal A}H$, whereas in \cite{ItskovKuninRosen20} each codeword may apply to part of the boundary of a region also.} \begin{defn} \label{defn:code} Let $\textbf{\emph{A}} = \{H_1,H_2,\ldots, H_m\}$ be a hyperplane arrangement in $\bb R^n$, equipped with a coorientation, and let $R$ be a region of $\textbf{\emph{A}}$. There exists a unique $(c_1,c_2,\ldots,c_m)$ in $\{+,-\}^m$ such that $R = \bigcap_{i=1}^m H_i^{c_i}$. We call $(c_1,c_2,\ldots,c_m)$ the \textbf{codeword} of $R$ and will denote it by $\operatorname{code}(R)$. The set of codewords for all regions of $\textbf{\emph{A}}$ is referred to as the \textbf{code} of $\textbf{\emph{A}}$, and is written as $\operatorname{code}(\textbf{\emph{A}})$. \end{defn} \begin{figure}[H] \centering \begin{tikzpicture}[scale=0.6] \draw[black, thick, font=\small] (-1.5,-1.5) -- (4.4,4.4)node[anchor=west]{$H_1$}; \draw[black, thick, font=\small] (-2.25,-0.6) -- (6,0.9)node[anchor=west]{$H_2$}; \draw[black, thick, font=\small] (4,-2) -- (2.6,4.4)node[anchor=east]{$H_3$}; \draw[black, thick, font=\small] (6,-2) -- (-1.75,1.75)node[anchor=east]{$H_4$}; \node[font=\tiny] at (4.2, 1.85) {\scalebox{0.8}{$++++$}}; \node[font=\tiny] at (2.3, 1.1) {\scalebox{0.8}{$++-+$}}; \node[font=\tiny] at (5, -0.2) {\scalebox{0.8}{$+-++$}}; \node[font=\tiny] at (0.9, 2.2) {\scalebox{0.8}{$-+-+$}}; \node[font=\tiny] at (1.5, -.9) {\scalebox{0.8}{$+---$}}; \node[font=\tiny] at (-1.8, -0.9) {\scalebox{0.8}{$----$}}; \node[font=\tiny] at (3.4, 4.2) {\scalebox{0.8}{$-+++$}}; \node[font=\tiny] at (2.85, -0.07) {\scalebox{0.75}{$+--+$}}; \node[font=\tiny] at (-1.2, 0.3) {\scalebox{0.8}{$-+--$}}; \node[font=\tiny] at (0.75, 0.15) {\scalebox{0.7}{$++--$}}; \node[font=\tiny] at (4.8, -1.9) {\scalebox{0.8}{$+-+-$}}; \end{tikzpicture} \caption{An example generic hyperplane arrangement $\textbf{A}$ in $\bb R^2$ equipped with a coorientation. Codewords of regions are indicated, with $\operatorname{code}(\textbf{A})$ consisting of eleven elements in $\{+,-\}^4$.} \label{fig:facet sharing} \end{figure} The code of an example hyperplane arrangement with coorientation is illustrated in Figure \ref{fig:facet sharing}. This hyperplane arrangement is \emph{generic} (Definition \ref{defn:generic}). In other work on ReLU networks, some authors have instead used \emph{ternary labelings} at a point (cf.~\cite[\S 2.4]{GLMW2022}) to encapsulate information similar to that contained in the codeword of a region, albeit in a more general setting. For any system of affine-linear equations, each equation of the form (\ref{eqn:affine-linear}) for some non-zero vector ${\bf w}$ and some $b\in\bb R$, the solution set of the system is either empty or agrees with an \emph{affine subspace} of $\bb R^n$. Hence, the intersection of some set of hyperplanes in $\bb R^n$ is either empty or an affine subspace of some dimension. We will be interested in hyperplane arrangements where the intersection of any number of the hyperplanes has a ``typical'' dimension. \begin{defn} \label{defn:generic} Let $\mathcal A =\{H_1, H_2, \ldots, H_m\}$ be a hyperplane arrangement in $\R^n$. Then $\mathcal A$ is said to be \textbf{generic} if for each of its subsets \{$H_{i_1},H_{i_2}, \dots, H_{i_q} \}$, of any size $q \in \{1, \dots, m\}$, the intersection $H_{i_1} \cap H_{i_2} \cap \dots \cap H_{i_q}$ is an affine subspace with dimension $n-q$ if $q \leq n$, and is the empty set if $q>n$. \end{defn} It is clear from definitions that any sub-arrangement $\mathcal A' \subset\mathcal A$ is generic if $\mathcal A$ is generic. \begin{remark} \label{remark:generic-almosteverywhere} If $\mathcal A$ consists of $m$ hyperplanes, suppose that $H_i\in\mathcal A$ is the solution set to an equation ${\bf w}_i\cdot{\bf x} + b_i = 0$ for $1\le i\le m$. Let $W$ be the $m\times n$ matrix with the $i^{th}$ row equal to ${\bf w}_i$ and let ${\bf b}\in\bb R^m$ have the $i^{th}$ component equal to $b_i$. Identifying such pairs $(W, {\bf b})$ with points in $\bb R^{mn+m}$, it is well-known that the hyperplane arrangement so determined is generic for a full-measure subset of $\bb R^{mn+m}$ (cf.~\cite[Lemma 2.6]{GrigsbyLindsey2022}). \end{remark} On occasion, given a generic hyperplane arrangement $\mathcal A$ and some $H\in\mathcal A$, it will be convenient to consider a certain \emph{induced arrangement} in $H$, obtained through intersections. \begin{defn} \label{defn:induced-arrangement} Let $\mathcal A = \{H_1,H_2,\ldots, H_m\}$ be a generic hyperplane arrangement in $\bb R^n$ and consider $H_i\in \mathcal A$ for some $1\le i\le m$. Define the ordered collection of affine subspaces $\mathcal A_{H_i} = \{H_i\cap H_1, \ldots, H_i\cap H_{i-1}, H_i\cap H_{i+1},\ldots, H_i\cap H_m\}$. We call $\mathcal A_{H_i}$ the \textbf{induced arrangement} (from $\mathcal A$) in $H_i$. \end{defn} By the assumption in Definition \ref{defn:induced-arrangement} that $\mathcal A$ is generic, each element of $\mathcal A_{H_i}$ is an affine subspace with dimension $n-2$. Moreover, it is possible to identify $\mathcal A_{H_i}$ with a generic hyperplane arrangement in $\bb R^{n-1}$ in a way that preserves face relations. \subsection{Polyhedral sets} Given a subset $S\subset\bb R^n$, we use the common notation $\overline{S}$ for the closure of the set $S$ in the following definition and throughout this paper. \begin{defn} \label{defn:simplex} A subset $P\subset\bb R^n$ is called a \textbf{\emph{polyhedral set}} in $\R^n$ if there is some finite collection of cooriented hyperplanes $\{H_1, H_2, \dots, H_k\}$ so that $P$ is the intersection of the closed positive half-spaces {--} i.e., $P = \overline{H}_1^+\cap\overline{H}_2^+\cap\ldots\cap\overline{H}_k^+$. When the polyhedral set is a bounded set, we call it a \textbf{\emph{convex polytope}}. For any subset $S\subset \R^n$, the \textbf{\emph{convex hull}} of $S$ is the intersection of all convex sets in $\R^n$ containing $S$. An $\textbf{n-dimensional simplex}$ is a convex polytope in $\mathbb{R}^{n} $ which agrees with the convex hull of $(n+1)$ points, with the property that no hyperplane in $\bb R^n$ contains all $(n+1)$ points. \end{defn} For any region $P$ in $\bb R^n$, we say that a hyperplane $H$ \textbf{\emph{cuts through}} $P$ if there exist $p_1, p_2\in P$ such that $p_1\in H^+$ and $p_2\in H^-$. Given a polyhedral set $P$, a hyperplane $H$ is called a \textbf{\emph{supporting hyperplane}} for $P$ if $H$ does not cut through $P$ and $H\cap P\neq \emptyset$. \begin{defn} \label{defn:face} Let $P$ be a polyhedral set. A subset $F\subset P$ is called a \textbf{\emph{face}} of $P$ if $F=\emptyset$, or $F=P$, or for some supporting hyperplane $H$ of $P$, $F=H\cap P$. \end{defn} Note that Definition \ref{defn:simplex} allows for the possibility of a polyhedral set $P \subset \bb R^n$ which is contained in an affine subspace with dimension less than $n$. In that case, any hyperplane $H$ which contains $P$ as a subset will be a supporting hyperplane. We call a face of $P$ \textbf{\emph{proper}} if it is neither empty nor equal to $P$. The \textbf{\emph{dimension}} of a polyhedral set $P$ is the dimension of its \emph{affine hull} {--} that is, the minimal dimension of an affine subspace of $\bb R^n$ containing $P$. A face $F$ of a polyhedral set $P$ is itself a polyhedral set (e.g., if $F=H\cap P$ where $H$ is a supporting hyperplane for $P$, then give $H$ a coorientation so that $P\cap H^+ = \emptyset$, and append $H$ to a set of cooriented hyperplanes that give $P$ as a polyhedral set). If $F\ne\emptyset$, we call $F$ a $k$-dimensional face of $P$ (or, simply, a \textbf{\emph{$k$-face}}) if $F$ has dimension $k$ as a polyhedral set. A \textbf{\emph{vertex}} is a $0$-face of $P$ and a \textbf{\emph{facet}} of $P$ is a face that has dimension $\dim(P)-1$. Every polyhedral set has an \emph{irredundant realization} as an intersection of closed half-spaces; i.e., for a polyhedral set $P\subset \bb R^n$, there are cooriented hyperplanes $H_1,H_2,\ldots,H_m$ in $\bb R^n$ so that $\displaystyle{P = \bigcap_{1\le j\le m}\overline{H}_j^+}$ and also, for every $i=1,\ldots,m$, we have \[P \ne \bigcap_{1\le j\le m, j\ne i}\overline{H}_j^+.\] Let $H_1,H_2,\ldots, H_m$ provide an irredundant realization of polyhedral set $P\subset\bb R^n$, and say that the dimension of $P$ is $n$. Then $F$ is a facet of $P$ if and only if $F$ is a proper face and $F = P\cap H_i$ for some $i$, $1\le i\le m$ \cite[\S 26]{Grunbaum}. Further, it can be shown that for each $(n-k)$-face $F$ of $P$, there exist $i_1,\ldots,i_k \in\{1,2,\ldots,m\}$ so that $F = P\cap H_{i_1}\cap\ldots\cap H_{i_k}$. For any hyperplane arrangement $\mathcal A$ in $\bb R^n$, the closure of any region of $\mathcal A$ is a polyhedral set of dimension $n$. As we will see below, for a generic, cooriented hyperplane arrangement $\textbf{A}$ in $\bb R^n$ with $n+1$ hyperplanes, only one region of $\textbf{A}$ has closure that is a convex polytope, and it is a simplex. \subsection{Neural networks, fully connected layers, and stably unactivated neurons.}\label{subsec:notation} This subsection introduces the notation and concepts related to neural networks that we use throughout the paper. We start by recalling the definition of the Rectified Linear Unit function, denoted by $\text{ReLU}$: \[ \text{ReLU}(x)= max\{0,x\}, \text{ for any } x\in\R. \] For any $n\in\mathbb{N}$, we denote by $\sigma:\R^n\to\R^n$ the function that applies the ReLU function to each coordinate. That is, \[ \sigma({\bf x}):=(\max\{x_1,0\},\max\{x_2,0\}, \ldots, \max\{x_n,0\} ), \] where ${\bf x}\in\R^n$. Throughout the manuscript, we focus exclusively on feedforward ReLU neural networks that have fully-connected layers. \begin{defn} \label{defn::neural net and network-function} Let $n_0, L\in\mathbb{N}$. A \textbf{feedforward ReLU neural network} $\mathcal N$ defined on $\R^{n_0}$, with $L$ \emph{layers}, is a list of $L$ positive integers $n_1,n_2,\ldots,n_L\in\bb N$, along with an ordered sequence of affine maps $A_1, A_2, \ldots, A_L$, such that \[ A_\ell: \R^{n_{\ell-1}}\to \R^{n_\ell} \] for each $\ell =1, \ldots, L$. Each feedforward ReLU neural network produces a function, which we call the associated \textbf{network function} $F\colon\mathbb{R}^{n_{0}}\to\mathbb{R}^{n_{L}}$, defined by \begin{equation}\label{eq: network function} F({\bf x}):= A_{L}\circ\sigma\circ A_{L-1}\circ\ldots\circ\sigma\circ A_{1}(\bf x), \end{equation} with $\sigma$ as above, applying ReLU coordinate-wise. We commonly abbreviate, calling $\mathcal N$ a \textbf{ReLU neural network} (or, simply, ReLU network), and $(n_0, n_1, \ldots, n_L)$ is called the \textbf{architecture} of $\mathcal N$. For each $\ell=1, 2, \ldots, L$, the $\ell$-th \textbf{layer map} $F_\ell: \R^{\ell-1}\to\R^{\ell}$ of the ReLU neural network defined above is given by \[ F_\ell:=\begin{cases} \sigma\circ A_{\ell}, &\text{ for } \ell =1, \ldots, L-1; \\ A_L, &\text{ for } \ell =L. \end{cases} \] \end{defn} Given a ReLU network with architecture $(n_0,n_1,\ldots,n_L)$, a \textbf{\emph{neuron}} of the network (or, \emph{neuron in layer $\ell$}) is a pair of indices $(\ell, j)$ with $1\le \ell\le L$ and $1\le j\le n_\ell$. Hence, the coordinate function of $F_\ell$ \emph{corresponding to a neuron}, $(\ell, j)$, refers to composition of projection to coordinate $j$ in $\bb R^{n_\ell}$ with $F_\ell$. As our neural networks have ReLU activation functions, for each $\ell=1,2,\ldots, L-1$ the image of the layer map $F_\ell$ is contained in the closed positive orthant \[\bb R^{n_\ell}_{\ge 0} = \{(x_1,\ldots,x_{n_\ell})\in\bb R^{n_\ell}\ |\ x_i\ge0,\ 1\le i\le n_\ell\}.\] Furthermore, for any $n$, $\bb R^{n}_{\ge0}$ may be viewed as a polyhedral set, determined by those hyperplanes that are defined by a coordinate being zero (the positive half-space being where that coordinate is positive). In this manuscript, our discussion will involve faces of $\bb R^n_{\ge0}$. Given $I\subset\{1,2,\ldots,n\}$, define $\mathtt{X}_{I} = \{(x_1,x_2,\ldots,x_n)\in\bb R^{n}\ |\ x_{j}=0 \text{ for each } j\not\in I\}$, and define $\halfax{I} = \mathtt{X}_{I}\cap\bb R^n_{\ge0}$. The $k$-faces of $\bb R^n_{\ge0}$ are described by those $\halfax{I}$ such that $|I| = k$. In the case that $I = \{i\}$, for some $1\le i\le n$, we call $\halfax{\{i\}}$ a (coordinate) half-axis and will write simply $\halfax{i}$. \textbf{Fully-connected layers.} The \emph{hidden layers} of a ReLU neural network refers to all layers $\ell$ with $0<\ell< L$. For a ReLU neural network with architecture $(n_0,n_1,\ldots, n_L)$, consider an $n_{\ell}\times n_{\ell - 1}$ matrix $W_{\ell}$, for each $\ell=1,2,\ldots,L$. We say the matrix $W_{\ell}$ consists of the \emph{weights} in layer $\ell$, and those in some row $j$ are the weights corresponding to neuron $(\ell,j)$. In addition, consider a vector ${\bf b_{\ell}}\in\R^{n_\ell}$ for each $\ell$, the \emph{bias vector} in layer $\ell$. We say a neural network with architecture $(n_0,n_1,\ldots, n_L)$ has $L$ \textbf{fully-connected layers} if, for each $\ell =1, 2, \ldots, L$, the affine function $A_{\ell}\colon \mathbb{R}^{n_{\ell - 1}}\to\mathbb{R}^{n_{\ell}}$ is given by \[ A_{\ell}({\bf x})=W_{\ell}\cdot{\bf x}+{\bf b_{\ell}}, \] for any ${\bf{x}}\in\mathbb{R}^{n_{\ell-1}}$. For each $\ell =1, \ldots, L$, we denote by $W_{\ell,j}$ the $j^{th}$ row of the weight matrix $W_\ell$ and by ${ b}_{\ell,j}$ the $j^{th}$ component of the bias vector ${\bf b}_{\ell}$, $j=1, \ldots, n_{\ell}$. A feedforward neural network with fully-connected layers is parameterized by a parameter space whose dimension equal to the total number of weights and biases in all layers of that neural network. \begin{defn} Define the Euclidean space $\Omega:=\R^{D}$ to be the \textbf{parameter space} for neural networks of architecture $(n_0, n_1, \ldots, n_L)$, where a parameter $\theta\in\Omega$ is given by the set of all weight matrices and bias vectors, i.e. \[ \theta:=(W_1,{\bf b}_1, W_2, {\bf b}_{2}, \ldots, W_L, {\bf b}_L) \] and \[ D:=\sum_{i=1}^{L}(n_{i-1}+1)n_i \] is the dimension of the parameter space. \end{defn} \begin{remark} For each $\theta$ in $\Omega=\R^D$, we denote by $F^\theta$ the network function associated with the ReLU neural network with $L$ fully-connected layers, determined by the weights and biases in $\theta$. Note that for each $\theta\in\Omega$, $F^\theta$ is a piecewise linear function. The superscript $\theta$ will be dropped when clear from the context. \end{remark} \begin{defn}[] \label{defn: A1(theta)}Throughout this paper, we mainly focus on the interaction between the first and the second hidden layers. For a neural network with architecture $(n_0, n_1, \ldots, n_L)$, if $\theta\in\R^D$ is understood, we denote by $\emph{\textbf{A}}_1(\theta)$ the cooriented hyperplane arrangement in $\R^{n_0}$ determined by the components of $\theta$ from the first layer map $F_1$. For example, when $\theta =(W_1,{\bf b}_1, W_2, {\bf b}_{2}, \ldots, W_L, {\bf b}_L)$ then $\emph{\textbf{A}}_1(\theta)=\{H_1, H_2, \ldots, H_{n_1}\}$, where \[ H_j=\{{\bf x}\in\R^{n_0}|W_{1,j}\cdot {\bf x}+{ b}_{1,j}=0\} \] for each $1\leq j\leq n_1$, and the coorientation on $H_j$ is the natural one determined by $W_{1,j}$ and $b_{1,j}$, as indicated above Definition \ref{defn:coorientation}. \end{defn} \begin{defn} \label{defn:stably-unactivated} Let $\theta\in\Omega=\R^D.$ We say a neuron in the $\ell$-th layer is \textbf{stably unactived} at the parameter $\theta$ if there exists an open set $O\subset\Omega$ containing $\theta$ such that for every $u\in O$, for all ${\bf x}\in\R^{n_0}$ the component of the output corresponding to that neuron in $F^u_\ell\circ \ldots\circ F^u_2\circ F^u_1({\bf x})$ is zero. \end{defn} Given a neuron in layer $\ell$, for $1<\ell<L$, let $H$ denote the hyperplane associated with that neuron when the neural network has parameter $\theta$, with the natural coorientation determined by $\theta$ as in Definition \ref{defn: A1(theta)}. If the given neuron is stably unactivated at $\theta$ then $\operatorname{Im}(F^\theta_{\ell-1}\circ\ldots\circ F^\theta_2\circ F^\theta_1) \subset H^{-}$, as being in a negative half-space is an open condition. In fact, the neuron is stably unactivated at $\theta$ if and only if $\operatorname{Im}(F^\theta_{\ell-1}\circ\ldots\circ F^\theta_2\circ F^\theta_1) \subset H^{-}$, since distance between the image and hyperplane, two closed subsets, is continuous in the parameter. \section{Supporting lemmas on hyperplanes}\label{sec:lemmas} The first few lemmas below state classical results about combinatorial questions on hyperplane arrangements and their regions, and provide some basic observations about the code of a cooriented hyperplane arrangement.\footnote{Recall that if $\textbf{A}$ is a cooriented arrangement of $m$ hyperplanes then $\operatorname{code}(\textbf{A})$ is the subset of $\{+,-\}^m$ such that $(c_1,c_2,\ldots,c_m)\in\operatorname{code}(\textbf{A})$ precisely when there exists $R\ne\emptyset$, a region of $\textbf{A}$, so that $R\subset H_1^{c_1}\cap H_2^{c_2}\cap \ldots\cap H_m^{c_m}$.} Near the end of the section we discuss a partition of certain collections of hyperplanes that aids our discussion of stably unactivated neurons. For the remainder of the paper, given a hyperplane arrangement $\mathcal A$ in $\bb R^n$ we use $r(\mathcal A)$ to denote the number of regions of $\mathcal A$, and $b(\mathcal A)$ to denote the number of bounded regions (i.e., those with compact closure). Lemmas \ref{lem:not-more-hyperplanes-than-n} and \ref{lem:more-hyperplanes-than-n}, which we write separately for emphasis, are well-known and follow from a theorem of Zaslavsky \cite{Zaslavsky} (cf.~ \cite[Section 2]{Stan07}). \begin{lem} \label{lem:not-more-hyperplanes-than-n} Let $\mathcal A = \{H_1, H_2, \ldots, H_m\}$ be a generic hyperplane arrangement in $\bb R^n$, with $m\le n$. Then $r(\mathcal A) = 2^m$ and $b(\mathcal A) = 0$. \end{lem} \begin{lem} \label{lem:more-hyperplanes-than-n} Let $\mathcal A = \{H_1, H_2, \ldots, H_m\}$ be a generic hyperplane arrangement in $\bb R^n$, with $m \ge n$. Then \[r(\mathcal A) = \sum_{k=0}^n\binom{m}{k} \quad\text{and}\quad b(\mathcal A) = \binom{m-1}{n}.\] \end{lem} As an illustration of Lemma \ref{lem:more-hyperplanes-than-n}, refer to Figure \ref{fig:facet sharing}, which shows a generic arrangement of 4 hyperplanes in $\bb R^2$. The number of regions of that arrangement is $\binom{4}{0}+\binom{4}{1}+\binom{4}{2} = 11$, and the number of bounded regions is $\binom{3}{2} = 3$. \begin{cor} \label{cor:unique-bounded-region} The number of regions of a generic hyperplane arrangement of $n+1$ hyperplanes in $\bb R^n$ is $2^{n+1}-1$. Exactly one of these regions is bounded. \end{cor} \begin{lem} \label{lem:nPlus1-hyperplanes} Suppose that $\textbf{A}$ is a generic, cooriented hyperplane arrangement of $n+1$ hyperplanes in $\bb R^n$. Then the only element from $\{+,-\}^{n+1}$ that is not contained in $\operatorname{code}(\textbf{A})$ is the negation of the codeword representing the unique bounded region. \end{lem} \begin{proof} Given $i$ with $1\le i\le n+1$, let $\textbf{A}_i$ be the cooriented subarrangement of $\textbf{A}$ obtained by excluding the $i^{th}$ hyperplane $H_i$ (and leaving the relative ordering of the other hyperplanes unchanged). The intersection of the $n$ hyperplanes in $\textbf{A}_i$ is a single point $v_i$ (one of the vertices of the bounded region of $\textbf{A}$, a simplex, guaranteed by Corollary \ref{cor:unique-bounded-region}), and we have that $v_i$ must be in $H_i^{\epsilon_i}$ for some $\epsilon_i\in\{+,-\}$. Note that $\epsilon_i$ is determined for every $1\le i\le n+1$. Moreover, $(\epsilon_1,\epsilon_2,\ldots,\epsilon_{n+1})$ must be the codeword of the simplex of $\textbf{A}$. Since $r(\textbf{A}_i) = 2^n$, we have that $\operatorname{code}(\textbf{A}_i) = \{+,-\}^n$. Furthermore, there is a neighborhood $V$ of $v_i$ that does not intersect $H_i$. Say that a region $R$ of $\textbf{A}_i$ has codeword $(c_1,c_2,\ldots,c_n)$ in $\operatorname{code}(\textbf{A}_i)$. Then, the region of $\textbf{A}$ that contains the points in $R\cap V$ will have codeword $(c_1,\ldots,c_{i-1},\epsilon_i,c_i,\ldots,c_n)$. Since every element of $\{+,-\}^n$ is in $\operatorname{code}(\textbf{A}_i)$, every element of $\{+,-\}^{n+1}$ which has $\epsilon_i$ in position $i$ is contained in $\operatorname{code}(\textbf{A})$, and this holds for all $1\le i\le n+1$. The only ${\bf c}\in\{+,-\}^{n+1}$ which does not satisfy this for some $i$ is $(-\epsilon_1,-\epsilon_2,\ldots,-\epsilon_{n+1})$, which means that we have proven that there are $2^{n+1}-1$ codewords in $\operatorname{code}(\textbf{A})$, excluding $(-\epsilon_1,-\epsilon_2,\ldots,-\epsilon_{n+1})$. Since $r(\textbf{A}) = 2^{n+1}-1$, there cannot be a region with codeword $(-\epsilon_1,-\epsilon_2,\ldots,-\epsilon_{n+1})$. \end{proof} \begin{cor} \label{cor:codeword-simplex} Let $R$ be a region of a generic, cooriented hyperplane arrangement $\textbf{A}$ consisting of $n+1$ hyperplanes in $\bb R^n$. Suppose that $\operatorname{code}(R) = (+,+,\ldots,+)$ and that the intersection of $\overline R$ with the unique simplex of $\textbf{A}$ is a $k$-dimensional face for some $0 \le k \le n$. Then the number of negative signs in the codeword of the bounded region is $n-k$; if $0< k < n-1$, then every codeword with exactly one positive sign is in the code of $\textbf{A}$ and represents an unbounded region of $\textbf{A}$. \end{cor} \begin{proof} Let $B$ be the unique bounded region of $\textbf{A}$. Each $k$-dimensional face of $B$ is contained in the intersection of $n-k$ hyperplanes in $\textbf{A}$. In order to pass through facets from $R$ to $B$, one must pass through each of these hyperplanes once. This changes the codeword in $n-k$ positions, resulting in $\operatorname{code}(B)$ having $n-k$ negative signs. Now, if $0 < k < n-1$ then $1 < n-k < n$. By Lemma \ref{lem:nPlus1-hyperplanes}, every element of $\{+,-\}^{n+1}$ is in $\operatorname{code}(\textbf{A})$ except for $-\operatorname{code}(B)$. Since $\operatorname{code}(B)$ has $n-k\ge 2$ negative signs, $-\operatorname{code}(B)$ cannot have exactly one positive sign. Finally, $B$ is the only bounded region and $\operatorname{code}(B)$ has $k+1\ge 2$ positive signs. Hence, every word with exactly one positive sign represents some unbounded region of $\textbf{A}$. \end{proof} We briefly highlight the last conclusion in Corollary \ref{cor:codeword-simplex}, as it helps in the proof of the main theorem. For any $1\le i\le n+1$, let ${\bf c}_i\in\{+,-\}^{n+1}$ be such that ${\bf c}_i$ is positive in its $i^{th}$ component and negative in all other components. Under the assumptions in Corollary \ref{cor:codeword-simplex}, if $0 < k < n-1$, then there is an unbounded region $R_i$ of $\textbf{A}$ so that $\operatorname{code}(R_i) = {\bf c}_i$. In addition, if $k=0$ or $k=n-1$, then this statement is true for all but one value of $i$ in $\{1,2,\ldots, n+1\}$. \begin{lem} \label{lem:more-hyperplanes} Suppose that $\textbf{A}$ is a generic, cooriented hyperplane arrangement of $m$ hyperplanes in $\bb R^n$ with $m > n$. Let $R$ be one of the bounded regions of $\textbf{A}$. Then $-\operatorname{code}(R)$ is not in the code of $\textbf{A}$. \end{lem} \begin{proof} Suppose that $\textbf{A} = \{H_1,H_2,\ldots, H_m\}$ and let $\operatorname{code}(R) = (c_1,c_2,\ldots,c_m)$. There are at least $n+1$ facets for the closure $\overline{R}$, and so at least $n+1$ hyperplanes of $\textbf{A}$ that contain a facet of $\overline{R}$. We claim that there is a choice of exactly $n+1$ of these hyperplanes, giving a generic subarrangement $\textbf{A}' = \{H_{i_1},\ldots,H_{i_{n+1}}\} \subset \textbf{A}$, such that $R$ is a subset of the unique bounded region of $\textbf{A}'$ guaranteed by Corollary \ref{cor:unique-bounded-region}. Call this bounded region $R'$. Using the coorientation and ordering on $\textbf{A}'$ given by that on $\textbf{A}$, we have that $\operatorname{code}(R') = (c_{i_1},\ldots,c_{i_{n+1}})$. Every region of $\textbf{A}$ is formed through a subdivision of a region of $\textbf{A}'$ with some of the hyperplanes not in $\textbf{A}'$ (potentially a \emph{trivial} subdivision if no hyperplane in $\textbf{A}\setminus\textbf{A}'$ cuts through the region). Hence, components $i_1,i_2,\ldots$, $i_{n+1}$ of the codeword for a region of $\textbf{A}$ agree with the codeword of a region of $\textbf{A}'$. By Lemma \ref{lem:nPlus1-hyperplanes}, we know that $-\operatorname{code}(R') \not\in\operatorname{code}(\textbf{A}')$ and so $-\operatorname{code}(R)$ cannot be in $\operatorname{code}(\textbf{A})$. In order to prove that the claimed subarrangement $\textbf{A}'$ exists, first note that it is immediate if $m = n+1$ (and so $\textbf{A}' = \textbf{A}$), by Lemma \ref{lem:nPlus1-hyperplanes}. Moreover, the claim holds when $n=1$, since every closed bounded region has exactly $2$ facets in that case. Now, suppose that $n\ge2$ and that $m > n+1$. Let $k$ be the number of facets of $\overline{R}$ and let $\{H_{i_1}, H_{i_2},\ldots, H_{i_k}\}\subset\textbf{A}$ denote those hyperplanes that contain these $k$ facets. We may assume that $k>n+1$, since we take $\textbf{A}'=\{H_{i_1}, H_{i_2},\ldots, H_{i_k}\}$ and $R'=R$ otherwise. We will proceed by induction on both $m$ and the dimension $n$. For each $1\le j\le k$, exactly one region $S_j$ of $\textbf{A}$ has the property that $\overline{R}\cap\overline{S}_j$ is contained in $H_{i_j}$ and is a facet of $\overline{R}$. If $S_1$ is bounded then define ${\bf A}_1 = \textbf{A} \setminus \{H_{i_1}\}$ and let $R_1$ be the interior of $\overline{R}\cup \overline{S}_1$. As ${\bf A}_1$ consists of $m-1$ hyperplanes and $R_1$ is a bounded region of $\textbf{A}_1$ containing $R$, the result follows by induction. Hence, we assume that $S_1$ is unbounded. Recalling Definition \ref{defn:induced-arrangement}, consider $\textbf{A}_{H_{i_1}}$, the induced arrangement in $H_{i_1}$ from $\textbf{A}$. Using that $\textbf{A}$ is generic, we may identify $\textbf{A}_{H_{i_1}}$ with a hyperplane arrangement in $\bb R^{n-1}$, the elements $\{H_{i_1}\cap H_\ell\ |\ \ell\ne i_1, 1\le \ell\le m\}$ being identified with hyperplanes in that dimension. Write $T_1 = \overline{R}\cap\overline{S}_1$ for the facet of $\overline{R}$ that is contained in $H_{i_1}$ and note that $T_1$ agrees with a bounded region of $\textbf{A}_{H_{i_1}}$. By induction on dimension, there is a subarrangement of $n$ hyperplanes in $\textbf{A}_{H_{i_1}}$ whose unique bounded region contains $T_1$ as a subset. That is, we have a set of indices $\ell_1, \ldots, \ell_n$, with $i_1\ne \ell_j$ for all $1\le j\le n$, so that $T_1$ is a subset of the unique bounded region of $\{H_{i_1}\cap H_{\ell_1}, \ldots, H_{i_1}\cap H_{\ell_n}\}$. Define $\textbf{A}' = \{H_{i_1}, H_{\ell_1}, \ldots, H_{\ell_n}\}$. Let $S'_1$ be the region of $\textbf{A}'$ which contains $S_1$ and has a codeword (in $\operatorname{code}(\textbf{A}')$) that agrees with components $i_1,\ell_1,\ldots,\ell_n$ of $\operatorname{code}(S_1)$. Since $S_1$ is unbounded, $S'_1$ must also be unbounded {--} we obtain it from $S_1$ by removing hyperplanes that are not in $\textbf{A}'$. Let $c_1$ be component $i_1$ of $\operatorname{code}(S_1)$, and so the region $S'_1$ is in the half-space $H_{i_1}^{c_1}$. Since $\textbf{A}'$ is a generic arrangement of $n+1$ hyperplanes and $S'_1$ is an unbounded region, on the $c_1$-side of $H_{i_1}$, that has $n+1$ facets ($n$ of them coming from those hyperplanes that intersect $H_{i_1}$ to give the $n$ facets of the bounded region we found, containing $T_1$), the region we get by crossing the facet that is in $H_{i_1}$ {--} i.e., switching only the sign of $c_1$ in the codeword {--} must be the unique bounded region of $\textbf{A}'$. By the way that we obtained the facet we crossed, a superset of $T_1$, our unique bounded region contains $R$. \end{proof} \subsection{Partitioning a collection of hyperplanes} \label{subsec:hyperplane-partition} Let $H$ be a hyperplane in $\bb R^n$, determined by a set of continuous random variables $w_1,w_2,\ldots,w_n,b$ where, using ${\bf w}$ to denote $(w_1,w_2,\ldots,w_n)$, we have $H = \{{\bf x}\in\bb R^n\ |\ \textbf{w}\cdot{\bf x} + b = 0\}$. Almost surely, $w_i\ne0$ for all $i=1,2,\ldots,n$ and $b\ne0$, in which case $H$ intersects all of the coordinate axes away from the origin. Conversely, the set of such intersection points determines $H$. If the $n$ intersection points are ${\bf q}_1, \ldots, {\bf q}_n$ then, more simply, we may refer to their one nonzero coordinate: writing $(q_1, \ldots, q_n)$ such that ${\bf q}_1 = (q_1,0,\ldots,0)$, and so on. In the setting that it is well-defined, we call $(q_1,q_2,\ldots,q_n)$ the {\bf \emph{intercept tuple}} of $H$. Given a fixed $n$-tuple, ${\bf p} = (p_1, \ldots, p_n)$ in $\bb R^n$, such that no $p_i$ is $0$, define $\Hp{\bf p}$ to be the set of hyperplanes with a well-defined intercept tuple $(q_1, q_2,\ldots,q_n)$, such that $q_i\ne 0$ for all $1\le i\le n$ and $q_i$ is positive if and only if $p_i$ is positive. We separate hyperplanes in $\Hp{\bf p}$ into subsets $\mathcal P, \Su_1, \Su_2,\ldots, \Su_n$ as follows. Let $H\in \Hp{\bf p}$ and let $(q_1,\ldots,q_n)$ be its intercept tuple. For each $j=1,\ldots,n$, there is a scalar $\lambda_j > 0$ such that $q_j = \lambda_jp_j$. Define $\mathcal P\subset\Hp{\bf p}$ by \[\mathcal P = \{H\in\Hp{\bf p}\ |\ \text{there is } j\ne j',\ 1\le j, j'\le n, \text{ such that } \lambda_j = \lambda_{j'}\}.\] Also, for each $j=1,\ldots,n$, define \[\Su_j = \{H\in\Hp{\bf p}\setminus\mathcal P\ |\ \lambda_j > \lambda_i\ \text{for all}\ i\ne j, 1\le i\le n \}.\] Given $j$ with $1\le j\le n$, note that if $H\in\Su_j$ then for each $i\ne j$ we have $|q_i|=|\lambda_ip_i| < |\lambda_jp_i|$. Thus, on the $i^{th}$ coordinate axis, $i\ne j$, $H$ intercects the axis closer to the origin than the hyperplane associated to $(\lambda_jp_1,\lambda_jp_2,\ldots,\lambda_jp_n)$. The proof of the following lemma is straightforward and we leave its verification to the reader. \begin{lem} \label{lem: partition} For any ${\bf p}\in\bb R^n$, with $p_i\ne 0$ for all $1\le i\le n$, $\Hp{\bf p}$ is partitioned by $\{\mathcal P, \Su_1, \ldots, \Su_n\}$. \end{lem} \begin{remark} For a fixed ${\bf p}$, a hyperplane $H\in\Hp{\bf p}$ is determined by $\lambda_1,\lambda_2,\ldots, \lambda_n$. \end{remark} \subsection{Network layer maps and stably unactivated neurons}\label{sec:node-layer2} In the remainder of the section, we consider a ReLU neural network with parameter $\theta$. As in Definition \ref{defn: A1(theta)}, $W_1$ (resp.\ $\textbf{b}_1$) is the weight matrix (resp.\ bias vector) in the first layer of $\mathcal N(\theta)$. As in Definition \ref{defn::neural net and network-function}, we have the layer map $F_1:\bb R^{n_0}\to\bb R^{n_1}$ given by $F_1(\textbf{x}) = \sigma(W_1\cdot{\bf x} + {\bf b}_1)$. The affine map that sends ${\bf x}\mapsto W_1\cdot{\bf x}+{\bf b}_1$ is the \emph{pre-activation map} for the first layer. Weights and biases are considered as continuous random variables. \begin{remark} \label{remark:regions-convex-image} Given a non-empty region $R$ of $\textbf{A}_1(\theta)$, say that $I\subset\{1,2,\ldots,n_1\}$ is the set of components in $\operatorname{code}(R)$ that are positive. Then it is clear that $F_1(\overline R) \subset \halfax{I}$. Since the pre-activation map for the first layer sends every point of $R$ to just one of the orthants of $\bb R^{n_1}$, the restriction of $F_1$ to $\overline R$ is an affine map. Therefore, as $\overline R$ is convex, $F_1(\overline R)$ is also convex for any region $R$ of the hyperplane arrangement $\textbf{A}_1(\theta)$. \end{remark} Let $n_1 = n_0+1$. If the hyperplane arrangement $\textbf{A}_1 = \textbf{A}_1(\theta)$ is generic, it has one bounded region (Corollary \ref{cor:unique-bounded-region}), which we call $B$. For every $i = 1,2, \ldots, n_0+1$, let ${\bf v}_i$ be the vertex of $\overline{B}$ comprising the intersection of all but the $i^{th}$ hyperplane in $\textbf{A}_1$. The image of the pre-activation map is a hyperplane in $\bb R^{n_1}$, which does intersect the $i^{th}$ coordinate axis at $W_{1,i}\cdot{\bf v}_i + b_{1,i}$ for each $1\le i\le n_0+1$. Since $\textbf{A}_1$ is generic, ${\bf v}_i$ is well-defined and not in the $i^{th}$ hyperplane. Therefore, the intercept $W_{1,i}\cdot{\bf v}_i + b_{1,i}$ is defined and non-zero; it is positive if and only if $\operatorname{code}(B)$ is positive in the $i^{th}$ component. \begin{remark} \label{remark:codeword-and-intercepts} Use $P_+=P_+(W_1,{\bf b}_1)$ for the hyperplane that is the image of the pre-activation map and write ${\bf p} = (p_0,p_1,\ldots, p_{n_0+1})$ for the intersection tuple of $P_+$. By the previous paragraph, we conclude $\operatorname{code}(B) = (\operatorname{sign}(p_1), \operatorname{sign}(p_2), \ldots, \operatorname{sign}(p_{n_0+1}))$. \end{remark} Consider a region $R$ of the arrangement $\textbf{A}_1$ whose closure intersects $\overline{B}$ in a common $k$-face, for some $0\le k\le n_0$, and such that $\operatorname{code}(R)$ has only positive components. Since $\operatorname{code}(R)$ is positive in every component, the hyperplane $P_+(W_1,{\bf b}_1)$ in Remark \ref{remark:codeword-and-intercepts} agrees with the unique hyperplane containing $F_1(R)$. In the remainder of the paper, we may refer to this hyperplane simply as $P_+$ (with an understood first layer map determined by $W_1, {\bf b}_1$). By Corollary \ref{cor:codeword-simplex} we have that $\operatorname{code}(B)$ contains exactly $n_0-k$ negative signs and so, by the above comments, we have shown the following. \begin{lem} \label{lem:intercepts-signs} Let a neural network have an architecture $(n_0,n_0+1, n_2,\ldots,n_L)$ and suppose that $\textbf{A}_1$ is generic. If $R_+$, a region of $\textbf{A}_1$, has a codeword $\operatorname{code}(R_+)=(+,+,\ldots,+)$ and shares a $k$-face with the unique bounded region of the hyperplane arrangement, then the hyperplane in $\bb R^{n_1}$ which contains $F_1(R_+)$ as a subset intersects exactly $n_0-k$ of the coordinate axes negatively. \end{lem} In Lemma \ref{lem:intercepts-signs} above, if we have that $k=n_0-1$, then by Lemma \ref{lem:nPlus1-hyperplanes} there is no region with codeword $(-,\ldots,-,+,-,\ldots,-)$, where the position of $+$ corresponds to the facet that $\overline{R}_+$ shares with $\overline{B}$. Consequently, $\ImF$ and the corresponding axis in $\bb R^{n_1}$ intersect in only the origin if $\overline{R}_+\cap\overline{B}$ is a facet of the simplex. To connect our discussion with the previous subsection, suppose that $W_1$ and ${\bf b}_1$ are the weight matrix and bias vector, respectively, of the first layer of a neural network, and that $W_1, {\bf b}_1$ are such that $\textbf{A}_1$ is a generic hyperplane arrangement. Further, let $P_+ = P_+(W_1,{\bf b}_1)$, as in Remark \ref{remark:codeword-and-intercepts}. \begin{lem} \label{lem:prob-mathcalP=0} If ${\bf p}=(p_1,\ldots,p_{n_1})$ is the intercept tuple of $P_+=P_+(W_1,{\bf b}_1)$, with assumptions as above, let $\{\mathcal P, \Su_1, \ldots, \Su_{n_1}\}$ be the partition of $\Hp{\bf p}$ discussed in Lemma \ref{lem: partition}. If $n_1 = n_0+1$, and the weights and bias corresponding to a given neuron in the second layer determine a hyperplane $H$, then $\bb P(H\in\mathcal P) = 0$. \end{lem} \begin{proof} Write the weights and bias for the given neuron in the second layer, which determine $H$, as ${\bf w}=(w_1,\ldots,w_{n_0+1})$ and $b$, respectively. If $H$ has a well-defined intercept tuple $(q_1,q_2,\ldots,q_{n_0+1})$, with only nonzero intercepts, then $q_j = -b/w_j$ for every $1\le j\le n_0+1$. As discussed above Remark \ref{remark:codeword-and-intercepts}, we have $p_j = W_{1,j}\cdot{\bf v}_j+b_{1,j}$, where ${\bf v}_j$ is the vertex obtained by intersecting all but the $j^{th}$ hyperplane in $\textbf{A}_1$. As a continuous random variable, ${\bf v}_j$ is completely determined by the weights in $W_1$ and biases in ${\bf b}_1$ (excluding those from row $j$). As a consequence, for $j\ne j'$, the variables $\lambda_j = q_j/p_j$ and $\lambda_{j'} = q_{j'}/p_{j'}$ are identically distributed random variables (provided weights within the first layer are identically distributed, as are biases in that layer). From definitions, we have $\bb P(H\in\mathcal P) = \bb P(\exists\ j\ne j'\ \text{s.t.}\ \lambda_j = \lambda_{j'})$. The equation $\lambda_j = \lambda_{j'}$ is equivalent to $p_{j'}q_j = p_jq_{j'}$, which is a rational equation, \[(W_{1,j'}\cdot{\bf v}_{j'}+b_{1,j'})bw_{j'} = (W_{1,j}\cdot{\bf v}_{j}+b_{1,j})bw_j.\] Therefore, the solution set has positive codimension in $\bb R^D$, where $D = n_0n_1+2n_1+1$ is the number of weights and biases. This remains true when we remove the finite set of affine algebraic curves to guarantee that $\textbf{A}_1$ is generic, and those given by $\{b=0\}$, $\{w_i = 0\}$, $i=1,2,\ldots,n_0+1$, to guarantee that $H$ has a well-defined intercept tuple. \end{proof} \section{Proof of the Main Results} \label{sec:proof-main} Consider a ReLU neural network $\mathcal N$ with architecture $(n_0,n_1,\ldots,n_L)$. Given a neuron in the second layer of $\mathcal N$, use $E\subset\Omega$ to denote those $\theta$ at which the neuron is stably unactivated. Let $E^+\subset E$ denote the event that at least one of the weights and bias associated with that neuron, written $({\bf w}|b)$, is positive. Let $H$ be the cooriented hyperplane associated to $({\bf w}|b)$. As above, $F_1 = F^{\theta}_1$ is the first layer map of $\mathcal N(\theta)$. As is implied by the discussion after Definition \ref{defn:stably-unactivated}, $E=\{\theta\in\Omega | \ImF\subset H^-\}$. In the sequel, we fix a generic, not cooriented, hyperplane arrangement $\mathcal A_1$ of $n_1$ hyperplanes in $\bb R^{n_0}$ and a subset $\Omega_1 \subset \Omega$ which has the property that for all $\theta\in\Omega_1$, $\textbf{A}_1(\theta)$ is the arrangement $\mathcal A_1$ equipped with a coorientation. We restrict to parameters $\theta$ for $\mathcal N$ which are in $\Omega_1$. Our strategy for proving Theorem \ref{thm:small-n1} and Theorem \ref{thm:main} is to show that $\bb P(E\ |\ \theta\in\Omega_1)$ is independent of $\mathcal A_1$ and equal to the value in the theorem statements. We may then conclude that $\bb P(E\ |\ \theta\in\Omega_1) = \bb P(E\ |\ \textbf{A}_1(\theta) \text{ is generic})$, which equals $\bb P(E)$ by Remark \ref{remark:generic-almosteverywhere}. First, we prove Theorem \ref{thm:small-n1} on the probability of $E$ in the case that $n_0\ge n_1$. The proof starts with the following lemma. \begin{lem}[]\label{Lemma:ImF1} Let $(n_0, n_1, \ldots, n_L)$ be the architecture of a neural network and assume that ${\bf A}_1$ is generic. \begin{itemize} \item[(a)] If $n_0\geq n_1$, then all coordinate half-axes $\halfax{j}$, $j=1,2,\ldots,n_1$, are subsets of $\ImF$. \item[(b)] If $n_1 = n_0+1$, denote by $B$ the bounded region in ${\bf A}_1$. If $\operatorname{code}(B)$ has exactly one $+$ or exactly one $-$ in the $i^{th}$ component, then $\halfax{j} \subset \ImF$ if $j\ne i$; otherwise, unless $\operatorname{code}(B)$ consists of only positive components, $\halfax{j} \subset \ImF$ for all $1\le j\le n_1$. \end{itemize} \end{lem} \begin{proof}[Proof of Lemma \ref{Lemma:ImF1}] Let $\textbf{A}_1=\{H_1, \ldots, H_{n_1}\}$ and denote by $R$ a region of ${\bf A}_1$ which has a codeword with exactly one positive sign, in the $j^{th}$ component for some $1\le j\le n_1$. That is, $j$ is the unique index so that $R\subset H_j^+$. By Remark \ref{remark:regions-convex-image}, $F_1(\overline{R})$ is convex and a subset of $\halfax{j}$. It follows that if $F_1(\overline{R})$ is unbounded and $R$ is adjacent to a region whose codeword has only negative components (which is sent to the origin by $F_1$), then $\halfax{j} \subset \ImF$. In the case of $n_0\geq n_1$, by Lemma \ref{lem:not-more-hyperplanes-than-n} each of the $2^{n_1}$ regions of $\textbf{A}_1$ are unbounded, so $R$ is unbounded. It is, then, straightforward to check that the coordinate function of the pre-activation map, given by ${\bf x}\mapsto W_{1, j}\cdot{\bf x} + b_{1,j}$, is unbounded along any infinite ray in $R$. Thus, $F_1(\overline{R})$ is unbounded. Since every element of $\{+,-\}^{n_1}$ is in $\operatorname{code}(\textbf{A}_1)$, there is a region with all components being negative, sent to the origin by $F_1$. As this region of $\bb R^{n_0}$ is determined by being on the negative side of all of its facets, it must be adjacent to $R$, proving that $\halfax{j}\subset\ImF$. Now let us show part (b) of Lemma \ref{Lemma:ImF1} when $n_1=n_0+1$. By Corollary \ref{cor:unique-bounded-region}, there are $2^{n_0+1}-1$ regions of ${\bf A}_1$, and exactly one of these regions is bounded, denoted by $B$. If $\operatorname{code}(B)$ has exactly one negative sign in the $i^{th}$ component then $-\operatorname{code}(B)$, which has exactly one positive sign in the $i^{th}$ component, is not in $\operatorname{code}(\textbf{A}_1)$ by Lemma \ref{lem:nPlus1-hyperplanes}. Hence, the only point in $\halfax{i}\cap\ImF$ is the origin. If $\operatorname{code}(B)$ has exactly one positive sign in the $i^{th}$ component, then $\halfax{i}\cap\ImF$ is compact since $\overline{B}$ is compact\footnote{In fact, the intersection of $\halfax{i}\cap\ImF$ is a line segment starting from the origin in this case.}. By Lemma \ref{lem:nPlus1-hyperplanes}, $-\operatorname{code}(B)$ is the only word in $\{+,-\}^{n_0+1}\setminus\operatorname{code}(\textbf{A}_1)$, and so in both cases all the other codewords with exactly one positive component represent an unbounded region of ${\bf A}_1$ which shares a facet with the region that is negative in all components. As in the proof for part (a), we find that $\halfax{j}\subset\ImF$ for all $j\ne i$. Finally, the remaining cases to consider are when $\operatorname{code}(B)$ has $k$ negative components, with $k > 1$ and $k \ne n_0$. In these cases, by Corollary \ref{cor:codeword-simplex}, each codeword with exactly one positive sign will be in $\operatorname{code}(\textbf{A}_1)$ and will represent an unbounded region of $\textbf{A}_1$. Moreover, as $k\ne0$, a non-empty region whose codeword has all negative components is sent to the origin and we have that $\halfax{j}\subset\ImF$ for all $j=1,2,\ldots, n_0+1$. \end{proof} \begin{proof}[\textbf{Proof of Theorem \ref{thm:small-n1}}] Let $H$ be the hyperplane, determined by $({\bf w}|b)$ and associated with our given node in layer 2. Then $\ImF\subset H^-$ requires that $b<0$ (since the origin is in $\ImF$). Furthermore, by part (a) of Lemma \ref{Lemma:ImF1}, we know that $\halfax{j}\subset\ImF$ for all $1\le j\le n_1$. Consequently, we must have that each intersection that $H$ has with a coordinate axis in $\bb R^{n_1}$ is negative. Thus all coordinates of ${\bf w}$ are negative, almost surely. As $\ImF\subset\bb R^{n_1}_{\ge0}$, knowing ${\bf w}<0$ and $b<0$ is also sufficient for $\ImF\subset H^{-}$. Hence, \[\bb P(E|\theta\in\Omega_1) = \bb P(\{{\bf w} < 0\} \cap \{b < 0\}) = \left(\frac{1}{2}\right)^{n_1+1}.\] \end{proof} Next, we focus on the case that $n_1=n_0+1$. We know there is a unique bounded region of $\textbf{A}_1$ according to Corollary \ref{cor:unique-bounded-region}, the closure of which is an $n_0$-dimensional simplex. As before, we use $B$ to denote the bounded region, then $\overline{B}$ is the simplex. Based on how the closed positive half-spaces of the hyperplanes in $\textbf{A}_1$ intersect $\overline{B}$ and considering that the weights and biases are independent and symmetrically distributed around the origin, there are $2^{n_1} = 2^{n_0+1}$ possible equally likely cases for the coorientation. We call each case a {\bf \textit{configuration}}, denoted by $C_i$, $i=0,\ldots, 2^{n_0+1}-1$. To establish our main result we require a key lemma, stated below, which records the sum of the conditional probabilities of $E^+$ given that $\textbf{A}_1$ is a particular configuration. Breaking down the probability $\bb P(E^+|\theta\in\Omega_1)$ according to the configuration is essential in the proof of the lemma. \begin{lem}\label{Lemma: Sum of delta} Let $\delta_i = \P( E^+ | \theta\in C_i)$, $i=0,\ldots, {2^{n_1}-1}$. If $n_1=n_0+1$, then we have \[ \sum_{i=0}^{2^{n_1}-1}\delta_i =\frac{1}{2^{n_1}}. \] \end{lem} The proof of this lemma will be postponed to Section \ref{sec:configs}. Next, we prove our main result Theorem \ref{thm:main}. \begin{proof}[\textbf{Proof of Theorem \ref{thm:main}}] Based on the definition of $C_i$, for each $i=0,\ldots, 2^{n_1}-1$, it is clear that $\{C_0,C_1, C_2, \ldots, C_{2^{n_1}-1}\}$ is a set of mutually exclusive and collectively exhaustive events. By the total probability formula, $\P(E^+|\theta\in\Omega_1)$ is given by: \[ \P(E^+|\theta\in\Omega_1)= \sum_{i=0}^{2^{n_1}-1}\P(E^+|\theta\in C_i)\P(\theta\in C_i)=\sum_{i=0}^{2^{n_1}-1}\delta_i\P(\theta\in C_i). \] Let $E^-$ denote the event that all weights and bias in $({\bf{w}}|b)$ are negative, then $E^-$ and $E^+$ are mutually exclusive and $E^{+}\cup E^{-} =E$. In addition, as the weights and bias are independent and symmetric distributed around the origin, we have $\P(\theta\in C_i)=\frac{1}{2^{n_1}}$ for any $i=0, 1, \ldots, 2^{n_1}-1$ and $P(E^-|\theta\in\Omega_1)=\frac{1}{2^{n_1+1}}$. Thus \begin{align*} \P(E|\theta\in\Omega_1)&=\P(E^-|\theta\in\Omega_1)+\P(E^+|\theta\in\Omega_1)=\frac{1}{2^{n_1+1}}+\sum_{i=0}^{2^{n_1}-1}\delta_i\P(\theta\in C_i)\\ &=\frac{1}{2^{n_0+2}}+\frac{1}{2^{n_0+1}}\frac{1}{2^{n_0+1}}=\frac{2^{n_0}+1}{4^{n_0+1}}. \qedhere \end{align*} \end{proof} \hfill \section{Proof of Lemma \ref{Lemma: Sum of delta}}\label{sec:configs} This section is devoted to proving Lemma \ref{Lemma: Sum of delta}. Throughout the section, we consider a ReLU neural network where $n_1=n_0+1$; that is, the network has an architecture of $(n_0,n_0+1,n_2,\ldots,n_L)$ with $L\ge 2$. Otherwise, assumptions are the same as in Section \ref{sec:proof-main}. To describe the \emph{configurations} $C_0,C_1,\ldots, C_{2^{n_1}-1}$ which were referenced in Lemma \ref{Lemma: Sum of delta}, we index the elements of $\{+,-\}^{n_0+1}$ as follows. Set ${\bf c}_0\in\{+,-\}^{n_0+1}$ to be the element in which every component is positive. Next, for each $i\in\{1,2,\ldots, n_0+1\}$, let ${\bf c}_i\in\{+,-\}^{n_1}$ be the element with $i^{th}$ component negative, and all other components positive. For each $i\in\{n_0+2, \ldots, 2n_0+2\}$, define ${\bf c}_i = -{\bf c}_{i-n_0-1}$. Finally, index the remaining elements of $\{+,-\}^{n_1}$ in an arbitrary order, ${\bf c}_{2n_0+3},\ldots, {\bf c}_{2^{n_1}-1}$. As in the previous sections, we use $B$ to denote the the unique bounded region of $\mathcal A_1$ and use $\overline{B}$ to denote the closure of $B$. That is, $\overline{B}$ is the simplex of $\mathcal A_1$. Now, for $i=0,1,\ldots, 2^{n_1}-1$ define $C_i = \{\theta\in\Omega_1\ |\ \operatorname{code}(B) = {\bf c}_i\}$ where $\Omega_1$ is a subset of $\Omega=\mathbb{R}^D$ with full measure. Finally, we define the conditional probabilities $\delta_i = \bb P(E^+\ |\ \theta\in C_i)$. \begin{remark} If $\textbf{A}_1$ is such that $(+,+,\ldots,+)\in\operatorname{code}(\textbf{A}_1)$, use $R_+$ to denote the region of $\textbf{A}_1$ which has this codeword. The indexing of the configurations above is such that $\theta\in\Omega_1$ is in one of $C_1,C_2,\ldots, C_{n_0+1}$ if and only if there is a such a region $R_+$ for $\textbf{A}_1(\theta)$ and $\overline{R}_+ \cap \overline{B}$ is a facet of $\overline{B}$, as in Figure \ref{fig:facet-config}. On the other hand, $\theta$ is in one of $C_{n_0+2},C_{n_0+3},\ldots, C_{2n_0+2}$ if and only if there is a region $R_+$ of $\textbf{A}_1(\theta)$ and $\overline{R}_+ \cap \overline{B}$ is a vertex of $\overline{B}$, as in Figure \ref{fig:vertex-config}. \end{remark} As discussed above, the neuron of the second layer in question is stably unactivated if and only if $\ImF \subset H^-$. In particular, it is necessary that $\ImF\cap H = \emptyset$. In addition, to ensure that $({\bf w}|b)$ are drawn from the event $E^+$, rather than $E^-$, we need (almost surely) that $H$ cuts through the positive orthant $\bb R^{n_1}_{\ge0}$, as we now explain. \begin{remark} \label{rem:H-positiveorthant} The event $E^+$ occurs when the weights and bias are not all negative. Note that, almost surely,\footnote{One of the weights, or the bias, being equal to zero happens with probability zero.} this implies that $H$ cuts through the positive orthant. To see this, if some component of ${\bf w}$, say $w_i$, and $b$ have opposite signs, then any point in $\halfax{i}$ with a sufficiently large $i^{th}$ coordinate will not be in the same half-space of $H$ as the origin, which implies that $H$ cuts through $\bb R^{n_1}_{\ge0}$. The only remaining possibility is that the weights and bias are all positive. But then the positive orthant is in $H^+$, not $H^-$, and thus $E$ cannot occur. \end{remark} By our indexing, we see that for each $i=2n_0+3, \ldots, 2^{n_1}-1$, if $\theta\in C_i$ then the codeword of the bounded region $B$ either consists of all negative components, or it has at least two negative and at least two positive components. By part (b) of Lemma \ref{Lemma:ImF1}, we can conclude that the positive half-axes $\halfax{j}$, $j=1,2,\ldots,n_0+1$, are each a subset of $\ImF$. By Remark \ref{rem:H-positiveorthant}, we obtain the following result. \begin{remark}\label{rem:zero-deltas} For $j=2n_0+3, \ldots, 2^{n_1}-1$, we have $\delta_j=0$. Thus, the proof of Lemma \ref{Lemma: Sum of delta} is reduced to showing that \[ \sum_{i=0}^{2n_0+2}\delta_i=\frac{1}{2^{n_1}} \text{ where } n_1=n_0+1. \] \end{remark} In the following subsections, we consider $\ImF$ when $\theta$ is in one of the configurations $C_0, C_1,\ldots, C_{2n_0+2}$ and we provide a useful reformulation of the condition that $H$ satisfies $\ImF\subset H^-$. For this purpose, the configurations are separated into three cases depending on the number of negative components of the codeword of the unique bounded region $B$. Moreover, in each of the configurations $C_0,C_1,\ldots,C_{2n_0+2}$, there is a positive region, denoted $R_{+}$, whose codeword is $\operatorname{code}(R_{+}) = (+,+,\ldots,+)$. First, configuration $C_0$ is considered on its own, in which case $R_+ = B$. Secondly, we consider $C_1,C_2,\ldots,C_{n_0+1}$, in which cases $\overline{B}\cap \overline{R}_{+}$ is a facet of $\overline{B}$. Finally, we consider $C_{n_0+2},\ldots, C_{2n_0+2}$, where $\overline{B}\cap \overline{R}_{+}$ is a vertex of $\overline{B}_{+}$. Our reformulation in the second case allows us to compute the sum $\sum_{i=1}^{n_0+1}\delta_i$. By the first and third cases, we are able to relate these $\delta_i$'s to the sum of the others and arrive at our conclusion. After considering these three cases, we combine all the findings at the end of Section \ref{subsec:vertex-case} to conclude the proof of Lemma \ref{Lemma: Sum of delta}. As in Remark \ref{remark:codeword-and-intercepts}, in each configuration $P_+$ denotes the hyperplane in $\bb R^{n_0+1}$ that is the image of the first layer pre-activation map. We denote the intercept tuple of $P_+$ by ${\bf p} = (p_0,p_1,\ldots,p_{n_0+1})$, which is well-defined since $\textbf{A}_1$ is generic. Then $P_+$ is the affine hull of $F_1(R_{+})$. For example, Figure \ref{fig:C0-config} depicts an example hyperplane arrangement $\textbf{A}_1(\theta)$ and image set $\ImF$ when $n_0=2$, $n_1=3$, and we have $\theta\in C_0$. The region that is in the positive half space of every hyperplane is indicated and its image in $\bb R^3$ is the shaded darkest in the figure. \subsection{ Case 1: $C_0$} \label{subsec:simplex-case} When $\theta \in C_0$, $R_{+}=B$, and $\ImF$ looks like a hyperplane, with normal vector in the positive orthant, that has been ``bent'' into the positive orthant. By Lemma \ref{lem:intercepts-signs}, each element of the intercept tuple $\bf p$ is positive. Rather than calculate $\delta_0 = \P(E^+|\ \theta\in C_0)$ directly, we indicate a collection of hyperplanes so that $\delta_0$ can be understood via the probability of $H$ being in that collection. For ${\bf p}$ as above, consider the collection of hyperplanes $\mathcal H_{\bf p}$, first discussed in Section \ref{subsec:hyperplane-partition}. Furthermore, recall the scalars $\lambda_1, \lambda_2, \ldots, \lambda_{n_0+1}$ that determine a hyperplane in $\mathcal H_{\bf p}$. We will show that $\delta_0 = \frac12\bb P(H\in\mathcal{H}^1_{\bf p})$, where $\mathcal H^1_{\bf p}$ is the subset of $\mathcal H_{\bf p}$ defined as \[\mathcal H^1_{\bf p} = \{H\in \mathcal H_{\bf p}\ |\ \lambda_i \le 1, i=1,2,\ldots,n_0+1\}.\] First, let $H$ be a hyperplane in $\R^{n_0+1}$ with an intercept tuple ${\bf q}=(q_1,\ldots,q_{n_0+1})$. We claim that in Case 1, and in the event $E^+$, we have $\ImF \subset H^-$ if and only if $H \in \mathcal H^1_{\bf p}$ and $H$ has a positive bias {--} which is equal, in probability, to the condition $0 < q_i < p_i$, for all $i$, and $H$ has a positive bias (using Lemma \ref{lem:prob-mathcalP=0}). \begin{figure}[t] \begin{tikzpicture}[>=stealth, scale=0.6] \draw[<->,cornflower!80!blue] (-1,3) --node[at start,right]{{\footnotesize $H_1$}} (-1,-3); \draw[<->,cornflower!80!blue] (-3,-2.6) --node[at end,above left]{{\footnotesize $H_2$}} (3,1); \draw[<->,cornflower!80!blue] (-3,1.128) --node[at end,below left]{{\footnotesize $H_3$}} (3, 1.128 - 2.4); \node[cornflower!80!blue] at (-0.31,-0.45) {{\tiny $+++$}}; \node[cornflower!80!blue] at (-2.0,-0.45) {{\tiny $-++$}}; \node[cornflower!80!blue] at (-2.0,1.4) {{\tiny $-+-$}}; \node[cornflower!80!blue] at (-2.0,-2.6) {{\tiny $--+$}}; \node[cornflower!80!blue] at (0.61,0.95) {{\tiny $++-$}}; \node[cornflower!80!blue] at (0.61,-1.85) {{\tiny $+-+$}}; \node[cornflower!80!blue] at (2.15,-0.45) {{\tiny $+--$}}; \node[inner sep=0pt] (surface) at (8,0) {\includegraphics[width=0.35\textwidth]{simplex_config.png}}; \end{tikzpicture} \caption{Example hyperplane arrangement $\textbf{A}_1$ (left) and corresponding image of first layer map $F_1:\bb R^2\to\bb R^3$ (right), where $\theta\in C_0$.} \label{fig:C0-config} \end{figure} To show the claim, note that an element of $\{+,-\}^{n_0+1}$ which has exactly one positive component, the $i^{th}$ component say, must be contained in $\operatorname{code}(\textbf{A}_1)$. It will be associated to some unbounded region, $R_i$. A similar argument to the one found in the proof of Lemma \ref{Lemma:ImF1}, shows that, for every $1\le i\le n_0+1$, the intersection $\ImF\cap\halfax{i}$ is unbounded. Letting ${\bf e}_i$ be the $i^{th}$ standard basis vector, the point $p_i{\bf e}_i$ is the image of the vertex $v_i$ of $\overline{B}$ that is in the boundary of $\overline{R}_i$. Since $F_1(\overline{R}_i)$ is convex (Remark \ref{remark:regions-convex-image}), it must be that for every $x_i \ge p_i$, the point $x_i{\bf e}_i$ is in $\ImF\cap\halfax{i}$. Thus, if $\ImF\cap H = \emptyset$ then $q_i < p_i$ for all $1\le i\le n_0+1$. By Remark \ref{rem:H-positiveorthant}, we must have that $H$ cuts through the positive orthant, so there is some $i$ with $q_i > 0$, almost surely. Suppose there were some $j$, $1\le j\le n_0+1$, so that $q_{j} < 0$. Then the intersection of $H$ with the plane $\mathtt{X}_{\{i,j\}}$ is a line with a positive slope. Since $P_+\cap\mathtt{X}_{\{i,j\}}$ has negative slope and $0 < q_i < p_i$, we see that $H$ and $P_+$ would necessarily intersect in $\halfax{\{i,j\}}$, giving a point in $H\cap \ImF$. Hence, if $\ImF\subset H^-$ then, almost surely, $0 < q_i < p_i$, meaning that $H\in\mathcal H^1_{\bf p}$.The bias for $H$ must be positive since $H$ separates any one of the points $p_i{\bf e}_i$ from the origin and $p_i{\bf e}_i\in H^-$. The other direction is clear, that is, if $H$ has a positive bias and $H\in\mathcal H^1_{\bf p}$ then $\ImF\subset H^-$. In summary, given that $\theta\in C_0$, the event $E^+$ has the same probability as the event that $H\in\mathcal H^1_{\bf p}$ and $H$ has a positive bias. Hence, $\delta_0 = \frac12\bb P(H\in\mathcal{H}^1_{\bf p})$. \subsection{Case 2: $C_1,\ldots,C_{n_0+1}$} \label{subsec:facet-case} When $\theta \in {C_i}$, $i=1, \ldots, n_0+1$, then $\overline{R}_{+}$ intersects $\overline{B}$ at a facet and $\operatorname{code}(B)$ has exactly $1$ negative sign in the $i^{th}$ component (see Figure \ref{fig:facet-config} for the case $n_0=2$). By Lemma \ref{Lemma:ImF1} and its proof, $\halfax{i}\cap\ImF$ consists of just the origin and $\halfax{j}\subset\ImF$, for each $1\le j\le n_0+1$ with $j\ne i$. In this case the hyperplane $P_+$ has an intercept tuple ${\bf p} = (p_0,p_1,\ldots, p_{n_0+1})$ which satisfies $p_i < 0$, and $p_j > 0$ for $j\ne i$, by Lemma \ref{lem:intercepts-signs}. As with $\delta_0$, we do not find each of $\delta_1, \delta_2, \ldots, \delta_{n_0+1}$ individually. However, we can use the symmetries of the configurations and results on hyperplanes from Section \ref{sec:lemmas} to calculate the sum $\sum_{i=1}^{n_0+1}\delta_i$. \begin{figure}[t] \begin{tikzpicture}[>=stealth, scale=0.6] \draw[<->,cornflower!80!blue] (-1,3) --node[at start,right]{{\footnotesize $H_1$}} (-1,-3); \draw[<->,cornflower!80!blue] (-3,-2.6) --node[at end,above left]{{\footnotesize $H_2$}} (3,1); \draw[<->,cornflower!80!blue] (-3,1.128) --node[at end,below left]{{\footnotesize $H_3$}} (3, 1.128 - 2.4); \node[cornflower!80!blue] at (-0.31,-0.45) {{\tiny $++-$}}; \node[cornflower!80!blue] at (-2.0,-0.45) {{\tiny $-+-$}}; \node[cornflower!80!blue] at (-2.0,1.4) {{\tiny $-++$}}; \node[cornflower!80!blue] at (-2.0,-2.6) {{\tiny $---$}}; \node[cornflower!80!blue] at (0.61,0.95) {{\tiny $+++$}}; \node[cornflower!80!blue] at (0.61,-1.85) {{\tiny $+--$}}; \node[cornflower!80!blue] at (2.15,-0.45) {{\tiny $+-+$}}; \node[inner sep=0pt] (surface) at (8,0) {\includegraphics[width=0.35\textwidth]{facet_config.png}}; \end{tikzpicture} \caption{Hyperplane arrangement $\textbf{A}_1$ (left) and image of first layer map $F_1:\bb R^2\to\bb R^3$ (right) for example $\theta\in C_3$; $n_0=2$, $n_1=3$.} \label{fig:facet-config} \end{figure} \begin{lem} \label{lem:facet-hyperplanes} Let the hyperplane $P_+$ and the $(n_0+1)$-tuple ${\bf p}$ be as in the setup above, and let $\mathcal{H}_{-{\bf p}}$ have partition $\{\mathcal P, \mathcal S_1,\ldots, \mathcal S_{n_0+1}\}$ as in Lemma \ref{lem: partition}. Then $H\in\mathcal H_{-\bf p}$ if the parameter $\theta$ is in the event $E^+$. Furthermore, $\P(E^+| \theta \in C_i) = \frac{1}{2}\P(H \in \mathcal{S}_i)$, for $i\in\{1,\ldots, n_0+1\}$. \end{lem} \begin{proof} Let $i$ be such that $\theta\in C_i$, with $1\le i\le n_0+1$. As in Lemma \ref{lem: partition}, we consider $\mathcal{H}_{\bf -p}$, the set of all hyperplanes with intercept tuples ${\bf q}=(q_1,\ldots,q_{n_0+1})$ where $q_j$ has the same sign as $-p_j$, for $j=1,2,\ldots,n_0+1$. From Lemma \ref{Lemma:ImF1}, we know that $\ImF$ contains $\halfax{j}$ for all $j \neq i$. Thus, we know that the set of hyperplanes in $E^+$ must be a subset of $\mathcal{H}_{\bf -p}$ since, almost surely, a hyperplane not in $\mathcal{H}_{\bf -p}$ either intersects $\halfax{j}$ away from the origin for some $j\ne i$ (recall that $p_j > 0$ if $j\ne i$) or, since $-p_i > 0$, it has the property that $q_j < 0$ for all $j=1,2,\ldots, n_0+1$ which, as we have remarked, cannot occur if the event $E^+$ occurs. The slope of the intersection of $P_+$ with the coordinate plane $\mathtt{X}_{\{j,i\}}$ (oriented with $j^{th}$ component first and $i^{th}$ component second) is given by $m_{ji}=-\frac{p_i}{p_j}$. Let $H' \in \mathcal{S}_k \subset \mathcal{H}_{\bf -p}$, where $k \neq i$, have intersection tuple ${\bf q'}=(q_1',\ldots,q_{n_0+1}')$. Then as in Lemma \ref{lem: partition}, $\lambda_k > \lambda_{k'}$ for all $k' \neq k$, where $\lambda_k$ is the scalar such that $q_k' = \lambda_k(-p_k)$. Then the slope of the intersection of $H'$ with the plane $\mathtt{X}_{\{k,i\}}$ is given by $m'_{ki}=-\frac{q_i'}{q_k'}=-\frac{\lambda_i(-p_i)}{\lambda_k(-p_k)}$. On the other hand, the slope of the intersection of $P_+$ in the plane $\mathtt{X}_{\{k,i\}}$ is $-\frac{p_i}{p_k}$, which is positive. Since $\lambda_k > \lambda_i$, $\frac{\lambda_i}{\lambda_k} < 1$, and so \[-\frac{\lambda_i(-p_i)}{\lambda_k(-p_k)} < -\frac{p_i}{p_k}.\] Since $p_k$ is positive, and we know $q_k'$ and $-p_k$ have the same sign, $q_k' < 0 < p_k$. Thus, $H'$ has a smaller $k^{th}$ intercept than $P_+$, and a smaller (positive) slope in the $\mathtt{X}_{\{k,i\}}$ plane, and so $H'$ must intersect $P_+$ in the positive quadrant $\halfax{\{k,i\}}$. Therefore, when $\theta \in E^+ \cap C_i$ \[\P(\ImF \subset (H')^-| H' \in \mathcal{S}_k) = 0, \quad \forall k \neq i.\] Using similar reasoning, one can show that any $H'\in\mathcal{S}_i$ does not intersect $P_+$ in $\halfax{\{j,i\}}$ for any $j\ne i$. Thus, with probability 1, $H'$ does not intersect $P_+$ in the positive orthant.\footnote{If $H'$ and $P_+$ have non-empty intersection then, generically, the intersection is an $(n_0-1)$-dimensional affine subspace in $\bb R^{n_0+1}$ and that affine subspace would intersect any plane $\mathtt{X}_{\{j,i\}}$ in a point.} From our reasoning above, except for the possibility that $H\in\mathcal P$, we have that $\ImF \subset H^-$ if and only if $H \in \mathcal{S}_i$ and $H$ has a negative bias (the origin is in $\ImF$ in this case, so it needs be in $H^-$). From Lemma \ref{lem:prob-mathcalP=0}, $\P(H \in \mathcal{P})= 0$. Therefore, \[\P(E^+ | \theta \in C_i) = \frac{1}{2}\P(H \in \mathcal{S}_i\cup\mathcal{P})= \frac{1}{2}\P(H \in \mathcal{S}_i).\] \end{proof} With Lemma \ref{lem:facet-hyperplanes} in hand, we are able to prove that \[\sum_{i=1}^{n_0+1}\delta_i=\frac{1}{2^{n_0+2}}.\] To do so, consider any intercept tuple ${\bf p}=(p_1, \ldots, p_{n_0+1})\in\R^{n_0+1}$, let $\{\mathcal{S}_1,\ldots,\mathcal{S}_{n_0+1},\mathcal{P}\}$ be the partition of $\mathcal{H}_{\bf p}$ from Lemma \ref{lem: partition}. A useful property of this partition is that the size of each of $\mathcal{P}, \mathcal{S}_1,\ldots, \mathcal{S}_{n_0+1}$ only depends on the magnitude of each component in $\bf p$, not the sign. Indeed, let $H \in \mathcal{H}_{\bf p}$ be a hyperplane with intersection tuple ${\bf q}=(q_1,\ldots,q_{n_0+1})$ and, for some $\epsilon=(\epsilon_1,\ldots,\epsilon_{n_0+1})\in\{1,-1\}^{n_0+1}$, consider $H' \in \mathcal{H}_{\epsilon\odot\bf p}$ to be the hyperplane with intersection tuple $\epsilon\odot{\bf q}$, where $\odot$ is the Hadamard product (e.g., the $j^{th}$ component of $\epsilon\odot{\bf p}$ is $\epsilon_jp_j$). For each $i$, as previously, write $\lambda_i$ for the scalar such that $q_i = \lambda_i p_i$. We have $H \in \mathcal{S}_i$ if and only if $\lambda_i > \lambda_j$, for all $j \neq i$ and the scalars $\lambda_1,\ldots,\lambda_{n_0+1}$ are pairwise distinct. Note that $\epsilon_jq_j = \lambda_j\epsilon_jp_j$ for every $j=1,2,\ldots,n_0+1$ and so $H'$ is determined in $\Hp{\epsilon\odot{\bf p}}$ by the same scalars $\lambda_1,\lambda_2,\ldots,\lambda_{n_0+1}$. Letting $\mathcal{S}'_1,\ldots,\mathcal{S}'_{n_0+1},\mathcal{P}'$ be the partition of $\mathcal{H}_{\epsilon\odot{\bf p}}$, we see that $H\in\mathcal{S}_i$ if and only if $H'\in\mathcal{S}'_i$. Recall the discussion from Section \ref{sec:lemmas} of how the intercepts of $P_+$ and the intercepts of $H$ are determined from the weights and biases. As the weights and biases are assumed to be identically and symmetrically distributed around the origin, then the components of ${\bf q}$ and those of $\varepsilon\odot{\bf q}$ have identical distributions. Hence, for the hyperplane $H \in \mathcal H_{-\bf p}$ associated to a neuron in the second layer, and for a given $\epsilon\in\{1,-1\}^{n_0+1}$, we have that $\bb P(H\in\mathcal{S}_i) = \bb P(H\in\mathcal{S}'_i)$ (using the notation of the previous paragraph). To compare different configurations on $\textbf{A}_1$ with the same underlying arrangement, changes in $(\operatorname{sign}(p_1), \operatorname{sign}(p_2), \ldots, \operatorname{sign}(p_{n_0+1}))$ produce all intercept tuples that arise from the cases $C_1,\ldots,C_{n_0+1}$ (see Remark \ref{remark:codeword-and-intercepts}).\footnote{By considering only those that make exactly one of the signs negative.} Combining this with Lemma \ref{lem:facet-hyperplanes}, we have found that we may simply consider ${\bf p}$ to be determined from some $\theta\in C_1$, say, in order to compute the sum of $\delta_1,\delta_2,\ldots,\delta_{n_0+1}$. Letting the partition of $\mathcal H_{-\bf p}$ be $\{\mathcal{S}_1,\ldots,\mathcal{S}_{n_0+1},\mathcal{P}\}$, then \[\sum_{i=1}^{n_0+1}\delta_i = \frac12 \sum_{i=1}^{n_0+1}\bb P(H\in \mathcal{S}_i) = \frac12 \bb P(H\in \mathcal H_{-\bf p}).\] From our assumption that weights and the bias for $H$ are independently and symmetrically distributed about 0, it is clear that each $q_i$ in the intercept tuple of $H$ has a $1/2$ probability of being positive. Thus, it is clear that $\P(H\in \mathcal{H}_{-\bf p})=\frac{1}{2^{n_0+1}}$. We have, therefore, found that \[\sum_{i=1}^{n_0+1}\delta_i=\frac{1}{2^{n_0+2}}.\] \subsection{Case 3: $C_{n_0+2},\ldots, C_{2n_0+2}$} \label{subsec:vertex-case} \begin{figure}[t] \begin{tikzpicture}[>=stealth, scale=0.6] \draw[<->,cornflower!80!blue] (-1,3) --node[at start,right]{{\footnotesize $H_1$}} (-1,-3); \draw[<->,cornflower!80!blue] (-3,-2.6) --node[at end,above left]{{\footnotesize $H_2$}} (3,1); \draw[<->,cornflower!80!blue] (-3,1.128) --node[at end,below left]{{\footnotesize $H_3$}} (3, 1.128 - 2.4); \node[cornflower!80!blue] at (-0.31,-0.45) {{\tiny $--+$}}; \node[cornflower!80!blue] at (-2.0,-0.45) {{\tiny $+-+$}}; \node[cornflower!80!blue] at (-2.0,1.4) {{\tiny $+--$}}; \node[cornflower!80!blue] at (-2.0,-2.6) {{\tiny $+++$}}; \node[cornflower!80!blue] at (0.61,0.95) {{\tiny $---$}}; \node[cornflower!80!blue] at (0.61,-1.85) {{\tiny $-++$}}; \node[cornflower!80!blue] at (2.15,-0.45) {{\tiny $-+-$}}; \node[inner sep=0pt] (surface) at (8,0) {\includegraphics[width=0.35\textwidth]{vertex_config.png}}; \end{tikzpicture} \caption{Hyperplane arrangement $\textbf{A}_1$ (left) and image of first layer map $F_1:\bb R^2\to\bb R^3$ (right) for example $\theta\in C_6$; $n_0=2$, $n_1=3$.} \label{fig:vertex-config} \end{figure} When $\theta \in C_i$, $i=n_0+2, \ldots, 2n_0+2$, then $\overline{R}_+$ intersects $\overline{B}$ at a vertex and $\operatorname{code}(B)$ has only one positive component in the $(i-n_0-1)^{th}$ component; so $B$ is in the positive half-space of only the $(i-n_0-1)^{th}$ hyperplane in $\textbf{A}_1$. As discussed in Lemma \ref{Lemma:ImF1} and its proof, $F_1(B)$ is contained in a bounded portion of the coordinate axis associated with that hyperplane. Furthermore, for an index $j$, with $j\ne i-(n_0+1)$, we have $\halfax{j}\subset\ImF$. By definition, $\theta \in C_i$ means that $\code(B)$ has the exact opposite signs compared to the codeword of the bounded region when a parameter is in $C_{i-n_0-1}$. Let $\bf p$ be the intersection tuple for the hyperplane $P_+$, as above, when $\theta \in C_i, i=n_0+2,\ldots,2n_0+2$. We negate every weight and bias associated to the first layer in $\theta$, getting a new parameter $\theta'$. Let $P'_+$ be the hyperplane that is likewise determined by $\operatorname{Im}(F^{\theta'}_1)$, and let ${\bf p}'$ be its corresponding intersection tuple. We have that $\theta' \in C_{i-n_0-1}$ and ${\bf p}=-{\bf p}'$. Since $\ImF$ contains every $\halfax{j}$ that is a subset of $\operatorname{Im}(F^{\theta'}_1)$ (see Lemma \ref{Lemma:ImF1}(b)), if $H \not\in \Hp{-{\bf p}'}=\Hp{\bf p}$ then $H$ must either intersect $\ImF$ or have an intercept tuple $\bf q$ such that $q_j < 0$ for all $1\le j\le n_0+1$. As this would keep it from being in $E^+$, a hyperplane $H$ in the event $E^+$ must be in $\Hp{\bf p}$, given that $\theta \in C_i$. The slope of the intersection of $P_+$ in the coordinate plane $\mathtt{X}_{\{j,i\}}$ is given by $-\frac{p_i}{p_j}$, which is the same as the slope of the intersection of $P'_+$ with $\mathtt{X}_{\{j,i\}}$. Thus, we can use the same reasoning as in Lemma \ref{lem:facet-hyperplanes} to show that $H \in \Su_{i-n_0-1} \subset \Hp{\bf p}$ is necessary in order to have $\ImF\cap H = \emptyset$. While $H\in \Su_{i-n_0-1}$ is sufficient to have $\operatorname{Im}(F^{\theta'}_1)\cap H = \emptyset$, it does not imply $\ImF\cap H = \emptyset$. Since the origin is contained in $\ImF$, a connected set, the cooriented hyperplanes that satisfy $E^+$ when $\theta \in C_i, i=n_0+2,\ldots,2n_0+2$ are precisely those which have a negative bias and also satisfy $\ImF\cap H = \emptyset$. And so we have that these hyperplanes are precisely those with correct coorientation in the set \[\Su_{i-n_0-1} \setminus \{H| \ImF\cap H\ne\emptyset\}.\] Recall the definition of $\Hp{\bf p}^{1}$, where $Q\in\Hp{\bf p}^{1}$ if and only if $Q\in\Hp{\bf p}$ and $\lambda_j \le 1$ for all $j$ (and $\lambda_1,\ldots,\lambda_{n_0+1}$ are the scalars for $Q\in\Hp{\bf p}$). Now, suppose that $H\in \Su_{i-n_0-1}$ is a hyperplane such that $\ImF\cap H\ne\emptyset$ and write $(q_1,q_2,\ldots,q_{n_0+1})$ for its intercept tuple. We have a unique intercept that is positive, namely $q_{i-n_0-1} > 0$, as $H\in\Hp{\bf p}$ and the configuration in this case requires $p_{i-n_0-1}$ to be the only positive element of ${\bf p}$. For simplicity of notation, write $i' = i - n_0 - 1$. Suppose it were the case that $q_{i'} > p_{i'}$. Then, for all $j\ne i'$, the intersection of $H$ with any coordinate plane $\mathtt{X}_{\{j,i'\}}$ {--} using orientation so the $j^{th}$ coordinate is first {--} has slope $\frac{q_{i'}}{|q_j|}$; this is larger than $\frac{p_{i'}}{|p_j|}$, which is the slope of the intersection of $P_+$ with $\mathtt{X}_{\{j,i'\}}$. With $q_{i'} > p_{i'}$, this makes it impossible to have a point in $\ImF\cap H$. Hence, the assumption that $\ImF\cap H \ne \emptyset$ tells us that $q_{i'}=\lambda_{i'}p_{i'} \le p_{i'}$. Now, $H\in\Su_{i'}$ implies that $\lambda_j \le \lambda_{i'} \le 1$ for all $j=1,2,\ldots,n_0+1$, showing that $H\in\Hp{\bf p}^{1}$. Moreover, if $H\in\Hp{\bf p}^1$ then we must have $\ImF\cap H\ne\emptyset$ (the sets intersect along $\halfax{i'}$). Setting $\Su_j^{1} = \Su_j\cap\Hp{\bf p}^{1}$ for each $j=1,2,\ldots,n_0+1$, we have that all but a probability zero subset of hyperplanes in $\Hp{\bf p}^{1}$ are in $\Su^{1}_1 \cup \Su^1_2\ldots\cup \Su^{1}_{n_1}$. We have shown that if $\theta\in C_i$, with $n_0+2\le i\le 2n_0+2$, and $\theta'\in C_{i-n_0-1}$ is the related parameter defined above, then if a hyperplane $H\in \Su_{i-n_0-1}$ is such that $\ImF\cap H\ne\emptyset$, then, almost surely, $H\in\Su^1_{i-n_0-1}\subset\mathcal{H}^1_{\bf p}$, where ${\bf p}$ is the intercept tuple for$P_+$. By the same reasoning as in subsection \ref{subsec:facet-case}, while this statement about set containment requires a different tuple ${\bf p}$ for each $i=n_0+2,\ldots,2n_0+2$, for the sake of the probability of $E^+$, any ${\bf p}$ with a single positive component suffices. Using the observation above that a negative bias is needed, we have that for any $n_0+2\le i\le 2n_0+2$, \[\bb P(E^+\ |\ \theta\in C_{i-n_0-1}) - \bb P(E^+\ |\ \theta\in C_{i}) = \frac12\bb P(H \in \Su^1_{i-n_0-1}),\] where $H$ is the hyperplane given by $({\bf w}|b)$ which comprises part of $\theta$, and the indexed set $\Su_{i-n_0-1}$ is in the partition of $\Hp{\bf p}$, as in Lemma \ref{lem: partition}. Shifting indices, \[\sum_{i=1}^{n_0+1}(\delta_i - \delta_{i+{n_0+1}}) = \sum_{i=1}^{n_0+1}\frac12\bb P(H\in\Su_i^{1}) = \frac12\bb P(H \in \Hp{\bf p}^{1}) = \delta_0.\] By Remark \ref{rem:zero-deltas} and findings in subsections \ref{subsec:simplex-case} through \ref{subsec:vertex-case}, we have \begin{align*} \sum_{i=0}^{2^{n_1}-1} \delta_i = \delta_0 + 2\sum_{i=1}^{n_0+1}\delta_i + \sum_{i=1}^{n_0+1}(\delta_{i+n_0+1} - \delta_i) = \delta_0 + 2\frac{1}{2^{n_0+2}} - \delta_0 = \frac{1}{2^{n_0+1}} \end{align*} which concludes the proof of Lemma \ref{Lemma: Sum of delta}. \section{Other architectures}\label{sec:general architectures} In this section we discuss Conjecture \ref{conj:large-n1}. Throughout the section, we consider neural networks $\mathcal N$ with architecture $(n_0,n_1,\ldots)$ such that $n_0$ is fixed and $n_1 > n_0$. Conjecture \ref{conj:large-n1} states that there is a constant $c > 0$ such that, for $n_1$ sufficiently large, the probability of a given neuron in the second layer of $\mathcal N$ being stably unactivated is at least $c\frac{1}{4^{n_0}}$. Below we give some empirical evidence of this conjecture, as well as some rationale that it holds generally. Note that this conjecture, if true, would imply that for $n_1$ sufficiently large $\bb P(E) \approx \bb P(E^+)$, since it is necessarily the case that $\bb P(E^-) = \frac{1}{2^{n_1+1}}$. \subsection{Computational evidence} In support of the Conjecture \ref{conj:large-n1}, we present empirical results on the value of $\bb P(E)$ when $2\le n_0\le 7$, for values of $n_1$ satisfying $n_0+2 \le n_1 \le n_0+16$. For each pair $(n_0, n_1)$ in that range, we sampled 100,000 parameters $\theta$ for a ReLU network with architecture $(n_0, n_1, 1)$, applying the He-uniform initialization to both weights and biases \cite{HeZhangRenSun2015}. We tested whether a neuron is stably unactivated using a Monte Carlo method, sampling the domain to test for only negative values at the given neuron. A computational challenge with this approach lies in the density of the sampling; small regions in the domain where the values could be positive might be missed by the sampling. As a result, only testing at $\theta$ would result in a significant number of false positives (returning that the neuron is stably unactivated when it is not), while false negatives never occur with that approach. Considering Definition \ref{defn:stably-unactivated}, we attempted to address this shortcoming as follows. For each parameter $\theta$, we randomly selected a set of parameters $\theta_1,\ldots,\theta_{4n_0}$ from a small neighborhood in $\Omega$ centered at $\theta$. The values of the given neuron were then tested at each of these nearby parameters within the domain sample. This creates a possibility of returning a false negative since one of the selected nearby parameters might lie outside of a neighborhood $O$ that satisfies Definition \ref{defn:stably-unactivated}. Greater accuracy of the results may then be obtained, having both false positives and false negatives, despite the computational constraints. \begin{figure}[t] \includegraphics[width=\textwidth]{exp_data2-4.png} \caption{Empirical probabilities of a neuron in the second layer being stably unactivated for architectures with $n_0 = 2, 3,$ and $4$. Along the horizontal axis we show the amount by which $n_1$ is larger than $n_0$; the dashed horizontal line is at height $1/4^{n_0+1}$.} \label{fig:data2-4} \end{figure} Our results are plotted in Figure \ref{fig:data2-4} and Figure \ref{fig:data5-7}. In these plots, we include the value of $\bb P(E)$ when $n_1=n_0+1$ that is known from Theorem \ref{thm:main}. Each of the given curves indicates that the ratio of the empirical probabilities to $1/4^{n_0}$ does not decrease as $n_0$ grows. In fact, we observe an increasing ratio, which may indicate that the bound in Conjecture \ref{conj:large-n1} could be tightened. \begin{figure}[h] \includegraphics[width=\textwidth]{exp_data5-7.png} \caption{Empirical probabilities of a neuron in the second layer being stably unactivated for architectures with $n_0 = 5, 6,$ and $7$. Along the horizontal axis we show the amount by which $n_1$ is larger than $n_0$; the dashed horizontal line is at height $1/4^{n_0+1}$.} \label{fig:data5-7} \end{figure} \subsection{Rationale for the Conjecture} As in Sections \ref{sec:proof-main} and \ref{sec:configs}, we assume the parameter $\theta$ for the network is such that $\textbf{A}_1 = \textbf{A}_1(\theta)$ is a generic hyperplane arrangement; moreover, as above we simply write $F_1$ to denote the first layer map $F^{\theta}_1:\bb R^{n_0}\to\bb R^{n_1}$, and $H$ refers to the cooriented hyperplane in $\bb R^{n_1}$ determined by the weights and bias for the given neuron in the second layer of $\mathcal N$. Moreover, as above we assume that weights and biases in each layer are continuous i.i.d.\ random variables with a distribution that is symmetric about $0$. To begin, we remark that the probability of $E$ decreases as a function of $n_1$, under typical assumptions on the underlying distribution for the weights and biases. Indeed, consider $F_1$ and $H$ as above, corresponding to a network with architecture $(n_0, m, \ldots)$. Let $F'_1$ be the first layer map of a network with the same values of $n_0$, but with $n_1 = m+1$, and let $H'$ be a random hyperplane in $\bb R^{m+1}$ determined by $m+1$ weights and a bias. Assume that there is a positive scalar so that, as random variables, the weights and bias for the hyperplane $H'$ are equivalent to that scalar times the weights and biases determining $H$.\footnote{This is a common situation. For example, if the ReLU networks are initialized with the He-uniform or He-normal distribution \cite{HeZhangRenSun2015}, then increasing $n_1$ by one will be equivalent to scaling all weights and biases in the second layer by a constant determined by $n_1$.} Since the hyperplane distribution for $H$ is invariant under overall scaling (importantly, the biases are \emph{not} initialized as zero), we have that $H'$ and $H$ have equal distributions. Write $H'_0$ for the projection of the intersection of $H'$ with the coordinate hyperplane $\mathtt{X}_{\{1,\ldots,m\}}$ to its first $m$ coordinates; $H'_0\subset \bb R^m$ inherits a coorientation from that of $H'$. Additionally, use the notation $\operatorname{Im}(F_1')_0$ for the set defined in the analagous way from $\operatorname{Im}(F_1')$. Now, clearly if $\operatorname{Im}(F'_1)\subset (H')^-$ then $\operatorname{Im}(F'_1)_0 \subset (H'_0)^-$. However, by our assumptions on the weights and biases, $\bb P\left(\operatorname{Im}(F'_1)_0 \subset (H'_0)^-\right) = \bb P(\ImF \subset H^-)$, which implies that $\bb P(\operatorname{Im}(F'_1)\subset (H')^-) \le \bb P(\ImF \subset H^-)$. As in Section \ref{sec:configs}, use $P_+$ to denote the image of the first layer pre-activation map of $\mathcal N$. With our assumption of a generic $\textbf{A}_1$, $P_+$ is an $n_0$-dimensional affine subspace of $\bb R^{n_1}$. In Section \ref{sec:configs}, since we had $n_1 = n_0+1$, this is a hyperplane and it was beneficial to describe $P_+$ in terms of its intercept tuple. Moreover, we used in this case that a hyperplane that does not intersect $P_+$ would necessarily be parallel {--} with an intercept tuple that is one multiple of that of $P_+$. Those hyperplanes which do not intersect $P_+\cap\bb R^{n_1}_{\ge0}$ were described by using a partition of $\Hp{\bf p}$ which separated hyperplanes according to intervals in which the components of their intercepts tuples lied. However, in the present context the generic assumption does not provide an intercept tuple for $P_+$. We have that $\dim P_+ = n_0$. Choosing a hyperplane $H_+$ in $\bb R^{n_1}$ which satisfies $P_+\subset H_+$, we have that $H_+$ may be chosen from an $(n_1-n_0-1)$-dimensional space of hyperplanes which contain $P_+$. Any non-zero translation in its normal direction of such an $H_+$ is a hyperplane that has empty intersection with $P_+$. Thus, for an $H$ that does not intersect $P_+$, we have a freedom of choice for $n_1 - n_0 - 1$ of the coordinate axis intercepts of $H$ with the remaining $n_0+1$ of the intercepts fixed. And so, for the hyperplane to have empty intersection with $P_+\cap\bb R_{\ge0}$, we should then expect that there is a subset of hyperplanes whose intercepts in $n_0+1$ of the coordinates are restricted to some interval, but are not restricted in the other coordinate directions. By Lemmas \ref{lem:bounded-dominates} and \ref{lem:average-number-of-facets}, which we prove below, it can be expected that most of the coordinate axes will have compact intersection with $\ImF$. In fact, we check that, on average, less than $2n_0+1$ of the $n_1$ coordinate axes intersect $\ImF$ anywhere except, possibly, the origin (see Corollary \ref{cor:average-axis-intersections}). Now, suppose we fix the coorientations of $n_0+1$ of the $n_1$ hyperplanes in $\textbf{A}_1$. We discussed in Section \ref{sec:configs} hyperplanes in $\bb R^{n_0+1}$ that will have empty intersection with the slice of $\ImF$ that is zero in all but these $n_0+1$ coordinates. In the remaining coordinates, the above discussion hints at the ability to allow (at least on average) for the intercepts of $H$, in those remaining coordinates, to be unrestrained, positive or negative {--} except, possibly, needing to avoid some interval in around $2n_0$ of them. There are $2^{n_1 - n_0 - 1}$ coorientations of $\textbf{A}_1$ which restrict to our one fixed choice of coorientation on these $n_0+1$ hyperplanes. Also, by the reasons above and relying on the work to prove Theorem \ref{thm:main}, we would guess that there is some constant $c > 0$ so that if we put together those conditional probabilities of $E^+$ (conditioned on the coorientation of $\textbf{A}_1$), where the coorientation on the unselected $n_1-n_0-1$ hyperplanes is fixed, then these probabilities sum to at least $c \frac{1}{2^{n_0+1}}$. Since each coorientation of the underlying hyperplane arrangement of $\textbf{A}_1$ has equal likelihood, this results in an estimate of the probability of $E^+$ being at least \[\frac1{2^{n_1}}2^{n_1 - n_0 - 1}\left(c \frac{1}{2^{n_0+1}}\right) = \frac{c}{4}\frac1{4^{n_0}}.\] We end the section by proving Lemmas \ref{lem:bounded-dominates} and \ref{lem:average-number-of-facets} that are referenced above, as well as Corollary \ref{cor:average-axis-intersections}. \begin{lem} \label{lem:bounded-dominates} For large $n_1$, the number of bounded regions of $\textbf{A}_1$ dominates the number of unbounded regions. \end{lem} \begin{proof} As a direct consequence of Lemma \ref{lem:more-hyperplanes-than-n}, we have that for $n_1 = m$, the number of bounded regions $b(\textbf{A}_1) = \binom{m-1}{n_0}$ is asymptotically $O(m^{n_0})$. On the other hand, asymptotically the number of unbounded regions is \begin{align*}r(\textbf{A}_1) - b(\textbf{A}_1) &= \binom{m}{n_0} - \binom{m-1}{n_0} + \binom{m}{n_0-1} + O(m^{n_0-2}) \\ &= \binom{m-1}{n_0-1}+\binom{m}{n_0-1} + O(m^{n_0-2}), \end{align*} which is $O(m^{n_0-1})$. \end{proof} Recall Lemma \ref{Lemma:ImF1}(b), which says that, in the case $n_1 = n_0+1$, there can be at most one $j$ between $1$ and $n_1$ with $\halfax{j}\not\subset\ImF$, regardless of the configuration in which $\theta$ is found. Additionally, this lemma says that $2^{n_1} - 2n_1$ of the configurations $C_i$ are such that $\halfax{j}\subset\ImF$ for all $j$, which has the effect that $\delta_i = \bb P(E^+\ |\ \theta\in C_i) = 0$ for all such configurations. In part, this result followed from the unicity of $B$, the bounded region of $\textbf{A}_1$, and that it shared a facet with $n_1$ unbounded regions. In general, for $n_1 > n_0$, the bounded regions of $\textbf{A}_1$ do not have a uniform number of facets. Moreover, how many regions (bounded or unbounded) of $\textbf{A}_1$ there are with a given number of facets is not the same for all generic hyperplane arrangements of $n_1$ hyperplanes in $\bb R^{n_0}$. However, we can understand the average number of facets among the regions of $\textbf{A}_1$. Doing so, in the interest of simpler notation, we write that $\textbf{A}_1$ has $m$ hyperplanes, instead of $n_1$. \begin{lem} \label{lem:average-number-of-facets} For a generic hyperplane arrangement $\textbf{A}_1$ of $m$ hyperplanes in $\bb R^{n_0}$, the limit of the average number of facets of a region of $\textbf{A}_1$, as $m\to\infty$, is equal to $2n_0$. \end{lem} \begin{proof} Let $s(\textbf{A}_1)$ denote the \emph{total} number of ($(n_0-1)$-dimensional) facets that appear in $\textbf{A}_1$. Since each is a facet for exactly two regions, the average number of facets of a region of $\textbf{A}_1$ is $\frac{2s(\textbf{A}_1)}{r(\textbf{A}_1)}$. Suppose that the hyperplanes in $\textbf{A}_1$ are listed as $\textbf{A}_1 = \{H_1,H_2,\ldots, H_{m}\}$. For each $i$, $1\le i\le m$, recall the induced arrangement in $H_i$, from Definition \ref{defn:induced-arrangement}, denoted $\textbf{A}_{H_i}$. As $\textbf{A}_1$ is generic, it is clear that each induced arrangement $\textbf{A}_{H_i}$ is equivalent to a generic arrangement in $\bb R^{n_0-1}$ of $m-1$ hyperplanes. Furthermore, we may consider the closure of each region of $\textbf{A}_{H_i}$ as corresponding to one of the facets, of a region of $\textbf{A}_1$, that is a subset of $H_i$. Hence, we have that $s(\textbf{A}_1) = m \sum_{k=0}^{n_0-1}\binom{m-1}{k}$, and so the average number of facets of a region of $\textbf{A}_1$ is \[\frac{2m \sum_{k=0}^{n_0-1}\binom{m-1}{k}}{\sum_{k=0}^{n_0}\binom{m}{k}}.\] Considering the average number of facets of $\textbf{A}_1$ as a function of $m$, we compute that \[\frac{2s(\textbf{A}_1)}{r(\textbf{A}_1)} = \frac{2\left(\frac{m^{n_0}}{(n_0-1)!} - \frac{1}{2(n_0-3)!}m^{n_0-1} + O(m^{n_0-2})\right)}{\frac{m^{n_0}}{n_0!} - \frac{n_0-3}{2(n_0-1)!}m^{n_0-1} + O(m^{n_0-2})}.\] As a consequence, we find that $\lim_{m\to\infty}\frac{2s(\textbf{A}_1)}{r(\textbf{A}_1)} = 2n_0$. \end{proof} In fact, for a slight improvement, we can refine the result of Lemma \ref{lem:average-number-of-facets} and check that as a function of $m$ the average number of facets is increasing, at least for sufficiently large $m$. To do this, let $\textbf{A}_1$ be as above and let $\textbf{A}'_1$ be a generic hyperplane arrangement of $m+1$ hyperplanes in $\bb R^{n_0}$. By a lengthy, but straightforward, computation in which highest and second-highest order terms vanish, one can check that \[\frac{s(\textbf{A}'_1)}{r(\textbf{A}'_1)} - \frac{s(\textbf{A}_1)}{r(\textbf{A}_1)} = \frac{m^{2n_0-2} + O(m^{2n_0-3})}{n_0!(n_0-1)!r(\textbf{A}'_1)r(\textbf{A}_1)},\] and so the difference is positive for large enough $m$. Hence, for large $m$ the average number of facets is strictly less than $2n_0$. \begin{cor} \label{cor:average-axis-intersections} For sufficiently large $n_1$, the expected number of coordinate axes whose intersection with $\ImF$ consists of more than the origin is at most $2n_0+1$. \end{cor} \begin{proof} Given a region $R$ of $\textbf{A}_1$, define $R$ to be \emph{\bf maximally negative} if there does not exist a region of $\textbf{A}_1$ whose codeword has more negative components than does $\operatorname{code}(R)$. Given that $R$ is a maximally negative region of $\textbf{A}_1$, say that $k$ is the number of facets of the closure $\overline{R}$ and that $H_1,H_2,\ldots, H_k$ are the hyperplanes in $\textbf{A}_1$ that are equal to the affine hulls of these facets. We remark that $R \subset H_1^{-}\cap H_2^{-}\cap\ldots\cap H_k^{-}$. To the contrary, if $R$ were in the positive half space of $H_i$, for some $1\le i\le k$, and $R_i$ is the region across that facet (so that $\overline{R}\cap\overline{R}_i$ is the facet in $H_i$), then $\operatorname{code}(R_i)$ and $\operatorname{code}(R)$ agree in all components but the $i^{th}$; yet, $\operatorname{code}(R)$ is positive in that component and $\operatorname{code}(R_i)$ is negative, contradicting that $R$ was maximally negative. In addition, since all facets of $R$ arise from $H_1,H_2,\ldots, H_k$, a point in $\bb R^{n_0}$ is contained in $H_1^{-}\cap H_2^{-}\cap\ldots\cap H_k^{-}$ only if it is in $R$, which means that $R$ is the \emph{only} region of $\textbf{A}_1$ whose codeword is negative in components $1,2,\ldots, k$. Any region of $\textbf{A}_1$ which is mapped by $F_1$ into some coordinate axis must have a codeword with exactly one positive component. By what we have just observed, this requires either the positive component to be one of the first $k$ components or the region must be $R$ itself. Hence, if a maximally negative region has $k$ facets then at most $k+1$ regions of $\textbf{A}_1$ can be mapped to a subset of $\halfax{j}$ for some $1\le j\le n_1$. Our assumptions make each coorientation of the arrangement underlying $\textbf{A}_1$ be equally likely, which makes the expected number of facets of a maximally negative region of $\textbf{A}_1$ equal to the average number of facets. Hence, for sufficiently large $n_1$, the expected number of $j$ such that $\halfax{j}\cap\ImF$ consists of more than the origin is at most $2n_0+1$, using Lemma \ref{lem:average-number-of-facets} and the comments immediately following its proof. \end{proof} \vspace{12pt} \subsection*{Acknowledgments} This work is the result of an REU project that was completed at Towson University in the summer of 2024. The REU was financially supported by the National Science Foundation, under grant DMS-2149865. Additionally, the authors wish to acknowledge the generous support from the TU Department of Mathematics and the Fisher College of Science and Mathematics. \bibliographystyle{amsalpha} \bibliography{refs} \end{document}
2412.05521v1
http://arxiv.org/abs/2412.05521v1
Random attractors for the stochastic Nernst-Planck-Navier-Stokes system with multiplicative white noise
\documentclass[a4paper,twoside]{article} \usepackage{geometry} \usepackage{fancyhdr} \usepackage{amsmath} \usepackage{amssymb} \usepackage{amsthm} \usepackage{enumerate} \usepackage{mathrsfs} \usepackage{xcolor} \usepackage{indentfirst} \textheight 22cm \textwidth 15cm \topmargin -12pt \evensidemargin 0,4cm \oddsidemargin 0,4cm \marginparsep 0pt \marginparwidth 60pt \headsep 8,1mm \headheight 14,5pt \footskip 12mm \topskip 0pt \mathsurround 1pt \renewcommand{\thefootnote}{\fnsymbol{footnote}} \newtheorem{thm}{Theorem}[section] \newtheorem{lem}[thm]{Lemma} \newtheorem{cor}[thm]{Corollary} \newtheorem{prop}[thm]{Proposition} \newtheorem{rem}[thm]{Remark} \newtheorem{defi}[thm]{Definition} \numberwithin{equation}{section} \begin{document} \begin{center} {\Large \bf Random attractors for the stochastic Nernst-Planck-Navier-Stokes system with multiplicative white noise}\\ \vspace{0.5cm} {Yang-yang WU$^*$ }\\ {\small Department of Mathematics,\\ Nanjing University of Aeronautics and Astronautics,\\ Nanjing, 211106, P.R. China }\\ \vspace{0.5cm} {Gao-cheng YUE}\\ {\small Department of Mathematics,\\ Nanjing University of Aeronautics and Astronautics,\\ Nanjing, 211106, P.R. China } \end{center} \footnote[0]{ \hspace*{0.6cm}$^*$ Corresponding author.\\ \hspace*{0.6cm}\textit{E-mail address}: [email protected]} \begin{abstract} In this paper, we consider the 2D periodic stochastic Nernst-Planck-Navier-Stokes equations with body forces perturbed by multiplicative white noise. We first transform the stochastic Nernst-Planck-Navier-Stokes system into the deterministic system and address the problem of global well-posedness of the solution. Then, we generate a corresponding random dynamical system and dedicate to proving the existence of a compact random attractor for such random dynamical system. Furthermore, upper semicontinuity of the random attractor is established when the noise intensity approaches to zero. \end{abstract} \indent \hspace{1.0cm}{\bf Keywords:} Electrodiffusion; Random attractor; Stochastic Nernst-Planck-Navier-Stokes equations; White noise.\\ \indent \hspace{0.4cm}{\bf 2020 Mathematics Subject Classification:} 35B41; 35Q35; 35R60; 76D03. \section{\large Introduction} In this paper, we consider the Nernst-Planck-Navier-Stokes (NPNS) system with fluid body forces. We study two ionic species with equal diffusivities $D>0$ and valences $z_1=1$ and $z_2=-1$, respectively, which is the simplest setting for the ionic concentrations. Mathematically, the evolution of ionic concentrations $c_1$ and $c_2$ obeys the Nernst-Planck equations \begin{equation}\label{y1} \partial_{t}c_1+u\cdot\nabla c_1=D\Delta c_1 +D\nabla\cdot(c_1\nabla \Phi), \end{equation} \begin{equation}\label{y2} \partial_{t}c_2+u\cdot\nabla c_2=D\Delta c_2 -D\nabla\cdot(c_2\nabla \Phi). \end{equation} The fluid velocity $u$ is described by the Navier-Stokes system and the divergence-free condition \begin{equation}\label{y3} \frac{du}{dt} + u\cdot\nabla u + \nabla p- \nu\Delta u =-\rho \nabla \Phi +f,~\nabla \cdot u=0. \end{equation} The potential $\Phi$ is given by the Poisson equation \begin{equation}\label{y4} -\varepsilon_0 \Delta\Phi=\rho=c_1-c_2. \end{equation} Above $\rho$ denotes the charge density, $\nu>0$ is the kinematic viscosity of the fluid, $\varepsilon_0>0$ is a positive constant proportional to the square of the Debye length, $p$ denotes the pressure, $\Phi$ represents the electrical potential, and $f$ is a time independent, smooth and divergence-free body force. From \eqref{y1} to \eqref{y4}, the NPNS system is obtained as follows \begin{equation}\label{a1} \left\{ \begin{aligned} \displaystyle{\frac{du}{dt} + u\cdot\nabla u + \nabla p- \nu\Delta u =-\rho \nabla \Phi +f}, \\ \nabla \cdot u = 0, \\ \rho=c_1-c_2, \\ -\varepsilon_0 \Delta\Phi=\rho, \\ \partial_{t}c_1+u\cdot\nabla c_1=D\Delta c_1 +D\nabla\cdot(c_1\nabla \Phi), \\ \partial_{t}c_2+u\cdot\nabla c_2=D\Delta c_2 -D\nabla\cdot(c_2\nabla \Phi). \\ \end{aligned} \right. \end{equation} The unknowns are the velocity $u$ and the ionic concentrations $c_1$ and $c_2$. From mathematical point of perspective, the system we are discussing encompasses classical Nernst-Planck equations and Navier-Stokes equations. There has been a tremendous literature on the well-posedness of the Nernst-Planck \cite{E.F.W2023, M.J2021, M.J2022} and Navier-Stokes systems \cite{P.C1988}. Electrodiffusion in fluids, which is described by the NPNS equations \cite{I.R1990}, has a variety of outstanding applications in neuroscience \cite{N.T1989}, semiconductors \cite{H.K1986}, water purification, ion separations \cite{A.J2016, H.J2018}, and ion selective membranes \cite{G.J1989}. Due to its importance in real world, the deterministic Nernst-Planck-Navier-Stokes equations have been studied extensively in different dimensions and situations. For example, it has been proved that the NPNS system has global and stable solutions with blocking and selective boundaries in 2D \cite{P.M2019}. In the case of mixed blocking and selective boundary conditions for the ionic concentrations and Robin boundary conditions for the electric potential, the global existence of three-dimensional strong solutions was shown in \cite{Lee.2022}. When both the ionic concentrations and the electrical potential are under the Dirichlet boundary conditions, the 3D strong solutions of the NPNS system are global if the fluid velocity is regular in \cite{P.M.F2021}. In 2D periodic domains, the authors explored long time dynamical behavior of solutions and showed the convergence of the solutions with the presence of body charges and body forces in \cite{E.M2021}. In the periodic boundary conditions, the authors considered the long-time dynamics of the NPNS system and established the existence of a global unique smooth solution, see \cite{P.M2024}. In addition, the existence of a global analytic solution for the forced NPNS system was obtained in \cite{E.M2022}. More Recently, the authors studied the global well-posedness and long-time dynamics of Nernst-Planck Darcy system and further proved the existence of a finite-dimensional global attractor in \cite{E.D2024}. However, in many practical circumstances, random effects play a key role that can not be ignored. These effects include turbulence, random fluctuations and uncertainties in fluid systems, which are essential for achieving an accurate estimate of dynamical systems in realistic world. There exist some literature about stochastic models and their properties that are related to our further study. In \cite{B.W2014}, the author studied the existence and uniqueness of random attractors for stochastic Reaction-Diffusion equations with multiplicative noise on unbounded domains and proved the upper semicontinuity of attractors. Moreover, Some results and models concerning the stochastic Navier-Stokes equations have been obtained in \cite{L.F2020, M.T2024, M.P2021, R.B2004}. As is known to us, multiplicative noise provides an appropriate description for stochastic disturbance and has thus attracted extensive attention. There exist numerous results with respect to the stochastic PDEs driven by linear multiplicative noise in \cite{Z.B.M1998,Z.B.M2016,Z.D.B.G2023,BX.W2014,D.Z2022}. In the two-dimensional case, the stochastic Navier-Stokes equation driven by linear multiplicative white noise has been studied in \cite{S.T2010}, with solutions of the 2D Navier-Stokes equation generating a locally compact cocycle. More recently, the authors established the existence of a global pullback attractor and proved the upper semicontinuity for damped Navier-Stokes equations of 2D as well as 3D in \cite{K.M2023}. To the best of our knowledge, there is still a scarcity of research about random attractors for stochastic NPNS system. Motivated by the literature above, we consider the stochastic Nernst-Planck-Navier-Stokes (SNPNS) equations perturbed by multiplicative white noise as follows: \begin{equation} \left\{ \begin{aligned}\label{a2} \displaystyle{\frac{du}{dt} + u\cdot\nabla u + \nabla p- \nu\Delta u =-\rho \nabla \Phi +f+ \varepsilon u\circ\ \frac{dW(t)}{dt} }, \\ \nabla \cdot u = 0, \\ \rho=c_1-c_2, \\ -\varepsilon_0 \Delta\Phi=\rho, \\ \partial_{t}c_1+u\cdot\nabla c_1=D\Delta c_1 +D\nabla\cdot(c_1\nabla \Phi), \\ \partial_{t}c_2+u\cdot\nabla c_2=D\Delta c_2 -D\nabla\cdot(c_2\nabla \Phi). \\ \end{aligned} \right. \end{equation} We address the SNPNS system in the two-dimensional periodic domain $\mathbb{T}^{2}$=$[0,2\pi]$$\times[0,2\pi]$ with periodic boundary conditions. Here, $\varepsilon > 0$ represents the noise intensity, $\circ$ means that the stochastic integral is understood in the sense of Stratonovich and $W(\cdot)$ is the two-sided Wiener process on the probability space $(\Omega,\mathscr{F},P)$, where $\Omega =\{\omega \in C(\mathbb{R},\mathbb{R}) :\omega(0)=0\} $, $\mathscr{F}$ is the Borel sigma-algebra induced by the compact-open topology of $\Omega$, and $P$ is a Wiener measure. In this paper, our main goal is to prove the existence of a compact random attractor and establish upper semicontinuity of the random attractor. The upper semicontinuity of random attractors was introduced in \cite{T.J.J1998}, that is, random attractors $\mathscr{A_\varepsilon}(\omega)$ converge towards a deterministic global attractor $\mathscr{A}(\omega)$ when the noise intensity $\varepsilon$ approaches to zero. The rest of this article is organized as follows. In section \ref{sec1}, we review some functional spaces and inequalities needed throughout the paper. Section \ref{sec2} contains basic concepts about random dynamical system and random attractors. Section \ref{sec3} is concerned with the existence and uniqueness of the solution, as well as the continuous dependence of the solution with respect to the initial data. We finally obtain a compact absorbing set and show the existence of a compact random attractor in $H$ in section \ref{sec4}. Section \ref{sec5} is devoted to the upper semicontinuity of such random attractor. \section{\large Preliminaries}\label{sec1} In this section, we will introduce some functional spaces, classical notations, and common inequalities that will be used in the further analysis. For more details of these topics, we refer to \cite{P.C1988}. We denote by $L^p$ the Lebesgue spaces with the norm \begin{align*} \|u\|_{L^p}=(\int _{\mathbb{T}^2}|u(x)|^p dx)^{\frac{1}{p}},~ ~\mathrm{if}~ p \in [1,\infty) \end{align*} and \begin{align*} \|u\|_{L^\infty}=\mathop {sup }\limits_{x \in \mathbb{T}^2}|u(x)|,~ ~\mathrm{if}~ p =\infty. \end{align*} We also define the following general abstract space $$\displaystyle{\tilde{V} = \left \{u \in \left (C_{0}^{\infty}({\mathbb{T}^{2}})\right )^{2}\ : \ \mathrm{div}\,u = 0\right \},}$$ where $C_{0}^{\infty}({\mathbb{T}^{2}})$ denotes the space of all infinitely differentiable functions with compact support in $\mathbb{T}^{2}$. Let $H$ denote the closure of $\tilde{V}$ in $(L^2(\mathbb{T}^{2}))^2$ with the norm $\left \vert \cdot \right \vert$, and the inner product $(\cdot , \cdot)$, where for $u,\upsilon\in(L^2(\mathbb{T}^{2}))^2$, we set $$\displaystyle{(u,\upsilon) =\sum_{i=1}^{2}\int_{\mathbb{T}^{2}}u_{i}(x)\upsilon_{i}(x)\,dx.}$$ Moreover, let $V$ denote the closure of $\tilde{V}$ in $(H_0^1(\mathbb{T}^{2}))^2$ with the norm $\|\cdot \|$, and the associated scalar product $((\cdot , \cdot))$, where for $u,\upsilon\in(H_0^1(\mathbb{T}^{2}))^2$, we have $$\displaystyle{((u,\upsilon)) =\sum_{ i,j=1}^{2}\int_{\mathbb{T}^{2}}\frac{\partial u_{j}} {\partial x_{i}} \frac{\partial \upsilon_{j}} {\partial x_{i}}\,dx.}$$ It follows that $V \subset H \equiv H^{{\prime}}\subset V ^{{\prime}}$, where the injections are continuous and dense. Let $\left \langle \cdot,\cdot \right \rangle$ denote the duality pairing between $V$ and $V ^{{\prime}}$. We recall the Stokes operator $A:V\rightarrow V^{\prime}$ by \begin{align*} \langle Au,\upsilon\rangle =\int_{\mathbb{T}^{2}}\nabla u \cdot \nabla \upsilon\,dx\quad \mathrm{for\,\,all}\quad \upsilon \in V. \end{align*} $A$ is defined as \begin{align*} Au:=-\mathcal{P}\Delta u,~u \in D(A)=V \cap H^2, \end{align*} where the operator $\mathcal{P}:L^2 \rightarrow H$ is the Leray orthogonal projection. The nonlinear operator $B: V \rightarrow V'$ is defined as \begin{align*} B(u)=B(u,u)=\mathcal{P}[(u \cdot \nabla)u], \end{align*} and the trilinear form is defined as \begin{align*} b(u,\upsilon,w)=\left((u\cdot\nabla\upsilon),w\right)\quad \mathrm{for\,\,all}\quad u,\upsilon,w\in V. \end{align*} It is clear that the trilinear form $b(\cdot,\cdot,\cdot)$ satisfies the equalities \begin{align*} b(u,\upsilon,\upsilon)=0,~~~b(u,\upsilon,w)=-b(u,w,\upsilon), \end{align*} and the following estimates. \begin{lem}[See Proposition $9.2$ in \cite{R.T1986}] For $n=2$, if $u,~\upsilon,~w \in V$, then \begin{align*} |b(u,\upsilon,w)|\leq k\|u\|_{L^2}^{\frac{1}{2}}\|\nabla u\|_{L^2}^{\frac{1}{2}}\|\nabla \upsilon\|_{L^2}\|w\|_{L^2}^{\frac{1}{2}}\|\nabla w\|_{L^2}^{\frac{1}{2}}, \end{align*} and if $u \in V$, $\upsilon \in D(A)$, and $w \in H$,then \begin{align*} |b(u,\upsilon,w)|\leq k\|u\|_{L^2}^{\frac{1}{2}}\|\nabla u\|_{L^2}^{\frac{1}{2}}\|\nabla \upsilon\|_{L^2}^{\frac{1}{2}}\|\Delta \upsilon\|_{L^2}^{\frac{1}{2}}\|w\|_{L^2}. \end{align*} \end{lem} Next, we recall Gronwall's inequality, which provides exponentially decaying bounds. (See Lemma $2.8$ in \cite{R.T1986}). \begin{lem}[\textbf{Gronwall's inequality}]\label{lem1} Let $t_0$ be a real number and $\varphi(t)\in L_{loc}^{1}(t_{0},\infty )$ be a function satisfying $\varphi'(t)\in L_{loc}^{1}(t_{0},\infty )$. Let $h(t)\in L_{loc}^{1}(t_{0},\infty)$ and $k(t)\in L_{loc}^{1}([t_{0},\infty ))$. Let $\varphi(t)$ satisfy the differential inequality $$\displaystyle{ \varphi'(t) \leq \varphi(t)k(t) + h(t)\quad \mathrm{for\ a.e.}\quad t \in (t_{0},\infty ), }$$ then $$\displaystyle{\varphi(t) \leq \varphi(t_{0})e^{\int_{t_{0}}^{t}k(\xi)\,d\xi } +\int_{t_{0}}^{t}h(\xi)e^{\int_{\xi}^{t}k(s)\,ds }\,d\xi\quad \mathrm{for\ all}\quad t \in [t_{0},\infty ). }$$ In particular, if $k(t) = C$ for all $t\geq t_0$ is a real constant, then $$\displaystyle{ \varphi(t)\leq \varphi(t_{0})e^{-C(t_{0}-t) } + \int_{t_{0}}^{t}h(\xi)e^{-C(\xi-t)}\,d\xi\quad \mathrm{for\ all}\quad t \in [t_{ 0},\infty ). }$$ Furthermore, if $h(t) = D$ for all $t\geq t_0$ is a real constant, then $$\displaystyle{ \varphi(t)\leq \varphi(t_{0})e^{-C(t_{0}-t) } + \frac{D}{C}(e^{-C(t_{0}-t) } - 1)\quad \mathrm{for\ all}\quad t \in [t_{ 0},\infty ). }$$ \end{lem} Finally, we note that in this paper $C$ denotes a positive constant that may depend on the parameters of the problem or universal constants, $C_f$ denotes a positive constant depending on the parameter $f$, $R(\omega)$ denotes a random variable, and $R_{f,\nu}(\omega)$ denotes a random variable depending on $f$ and $\nu$. As for other constants, we denote them by the same pattern. Notice in particular that these constants may vary from line to line. \section{\large The Basic Set-up}\label{sec2} In this section, we first give some basic knowledge about random dynamical system (RDS). For a more detailed introduction about RDS, we can refer to \cite{I2002, H.F1994}. Let $X$ be a Banach space with Borel sigma-algebra and $(\Omega,\mathcal{F},P)$ be a probability space. If the following holds$:\theta_t:\Omega \to \Omega,~t \in \mathbb{R}$ is measurable, $\theta_0=id$(identity on $\Omega$), and $\theta_{t+s}=\theta_t \theta_s$ for all $t,s \in \mathbb{R}$, then $(\Omega,\mathcal{F},P,{\{\theta_t\}}_{t \in \mathbb{R}})$ is called a metric dynamical system. Based on these, definitions of a continuous RDS, attraction, absorption and random attractors are given by the following four definitions, respectively. \begin{defi}[See Definition $2.1$ in \cite{H.F1994}]\label{def0} A continuous random dynamical system on X over a metric dynamical system $(\Omega,\mathcal{F},P,{\{\theta_t\}}_{t \in \mathbb{R}})$ is a measurable mapping \begin{align*} \varphi:\mathbb{R}^{+} \times \Omega \times X \rightarrow X,~~(t,\omega,x) \mapsto \varphi(t,\omega,x), \end{align*} satisfying for $P$-a.e.$~\omega \in \Omega:$\par $(i)~\varphi(0,\omega)=id~\mathrm{on}~X;$\par $(ii)~\varphi(t+s,\omega)=\varphi(t,\theta_s\omega) \varphi(s,\omega)$ for all $t,s\in \mathbb{R}^+;$\par $(iii)~\varphi(t,\omega):X \to X$ is continuous.\newline A family of maps satisfyng $(ii)$ ia called a cocycle. \end{defi} \begin{defi}[See Definition $3.3$ in \cite{H.F1994}] Given a random set $\mathscr{A}(\omega)$ and $B$ is another random set, we say $\mathscr{A}(\omega)$ attracts $B$ if $P$-a.s. \begin{align*} {\lim_{t\to + \infty}}dist (\varphi(t,\theta_{-t}\omega)B,\mathscr{A}(\omega))=0, \end{align*} where dist$(\cdot,\cdot)$ denotes the Hausdorff semi-distance. \end{defi} \begin{defi}[See Definition $3.5$ in \cite{H.F1994}] Let $\mathscr{A}(\omega)$ and $B$ be random sets, for $P$-a.e.$~\omega \in \Omega$ if there exists $t_B(\omega)$ such that for all $t \geq t_B(\omega)$, \begin{align*} \varphi(t,\theta_{-t}\omega)B \subset \mathscr{A}(\omega), \end{align*} then we say $\mathscr{A}(\omega)$ absorbs $B$. \end{defi} \begin{defi}[See Definition $3.9$ in \cite{H.F1994}] A random set $\mathscr{A}(\omega)$ is said to be a random attractor for the $\mathrm{RDS}$ $\varphi$ if \par $(i)\mathscr{A}(\omega)$ is a random compact set; \par $(ii)\mathscr{A}(\omega)$ is invariant; \par $(iii)\mathscr{A}(\omega)$ attracts all deterministic bounded sets $B \subset X$. \end{defi} The following theorem is the main result in this paper, that is, the existence theorem of a random attractor for a continuous RDS. \begin{thm}[See Theorem $3.11$ in \cite{H.F1994}]\label{thm1} If there exists a random compact set absorbing every bounded set $B \subset X$, then the $\mathrm{RDS}$ possesses a random attractor $\mathcal A(\omega)$, $$\displaystyle{\mathcal A(\omega)=\overline{\mathop {\bigcup }\limits_{B \subset X} \Lambda_B(\omega)}},$$ where $\Lambda_B(\omega)=\mathop {\bigcap }\limits_{s \geq 0} \overline{\mathop {\bigcup }\limits_{t \geq s}\varphi(t,\theta_{-t}\omega)B}$ is the omega-limit set of $B$. \end{thm} Next, applying Leray projection $\mathcal{P}$ onto the first equation in \eqref{a2} to get \begin{equation*} \frac{du}{dt} + B(u) + \nu Au =-\rho \nabla \Phi +f+ \varepsilon u\circ\ \frac{dW(t)}{dt}. \end{equation*} Finally, we generate the random dynamical system concerning SNPNS. Define $\{{\theta_t}\}_{t \in \mathbb{R}}$ by \begin{align*} \theta_t \omega(\cdot)=\omega(\cdot+t)-\omega(t),t \in \mathbb{R}, ~ \omega \in \varOmega. \end{align*} Hence, $(\Omega,\mathcal{F},P,{\{\theta_t\}}_{t \in \mathbb{R}})$ is a metric dynamical system. Moreover, there exists a $\theta_t$-invarient set $\tilde{\Omega \subseteq \Omega}$ of full $P$ measure such that for every $\omega$ $\in$ $\tilde{\Omega}$, the following holds: \begin{align}\label{l1} \frac{w(t)}{t} \to 0~~\mathrm{as}~~ t \to \pm \infty. \end{align} For convenience, we will not distinguish $\Omega$ and $\tilde{\Omega}$. Given $t \in \mathbb{R}$ and $\omega$ $\in$ $\Omega$, let the process \begin{align*} z(t,\omega)=e^{-\varepsilon\omega(t)}. \end{align*} Obviously, $z$ satisfies the equation \begin{align*} dz=-\varepsilon z \circ dW. \end{align*} Let $v^\varepsilon$ be a new variable given by \begin{align*} v^\varepsilon=zu. \end{align*} Then, we obtain the new equations without stochastic differential \begin{equation}\label{b1} \left\{ \begin{aligned} \displaystyle{\frac{dv^\varepsilon}{dt} + \nu\ Av^\varepsilon + \frac{1}{z(t,\omega)}B(v^\varepsilon)=-z(t,\omega)\rho\nabla \Phi+z(t,\omega)f}, \\ \nabla \cdot v^\varepsilon = 0, \\ \rho=c_1-c_2, \\ -\varepsilon_0 \Delta\Phi=\rho, \\ \partial_t{c_1}+z^{-1}(v^\varepsilon\cdot\nabla c_1)=D\Delta c_1 +D\nabla\cdot(c_1\nabla \Phi), \\ \partial_t{c_2}+z^{-1}(v^\varepsilon\cdot\nabla c_2)=D\Delta c_2-D\nabla\cdot(c_2\nabla \Phi). \\ \end{aligned} \right. \end{equation} Let $\sigma=c_1+c_2$, system \eqref{b1} can be rewritten in its equivalent form as follows: \begin{equation} \left\{ \begin{aligned}\label{b2} \displaystyle{\frac{dv^\varepsilon}{dt} + \nu\ Av^\varepsilon + \frac{1}{z(t,\omega)}B(v^\varepsilon) =-z(t,\omega)\rho\nabla \Phi+z(t,\omega)f}, \\ \nabla \cdot v^\varepsilon = 0, \\ \rho=c_1-c_2, \\ -\varepsilon_0 \Delta\Phi=\rho, \\ \partial_{t}\sigma+z^{-1}(v^\varepsilon\cdot\nabla \sigma)=D\Delta \sigma +D\nabla\cdot(\rho\nabla \Phi), \\ \partial_{t}\rho+z^{-1}(v^\varepsilon\cdot\nabla \rho)=D\Delta \rho +D\nabla\cdot(\sigma\nabla \Phi). \\ \end{aligned} \right. \end{equation} \section{\large Well-posedness of Solutions}\label{sec3} In this section, we will focus on the global well-posedness of solutions for system \eqref{b1}. The following calculations can be done rigorously by using a Faedo-Galerkin approximation method in \cite{R.T2012}. First of all, we give the existence and uniqueness theorem of local solutions with respect to the stochastic NPNS system. \begin{thm}[\textbf{Local Solution}]\label{thm2} Suppose $v^\varepsilon(0) \in H^1$ and $c_i(0) \in {L}^{2}$. Then, there exists $T_0$ depending on $\|v^\varepsilon(0)\|_{H^1}$, $\|c_i(0)\|_{L^2}$, and the parameters of the problem such that system \eqref{b1} has a unique solution obeying $v^\varepsilon \in L^{\infty}(0,T;H^1) \cap L^2(0,T;H^2)$ and $c_i \in L^{\infty}(0,T;L^2) \cap L^2(0,T;H^1)$ on $[0,T_0]$. \begin{proof} Taking the ${L}^{2}$ inner product of the first equation in \eqref{b1} with $-\Delta v^\varepsilon $, we obtain \begin{align}\label{b3} \frac{1}{2}&\frac{d}{dt}\|\nabla v^\varepsilon(t)\|_{L^2}^{2}\ +\nu \|\Delta v^\varepsilon\|_{L^2}^{2}\notag \\&=\frac{1}{z(t,\omega)} \int (v^\varepsilon \cdot \nabla v^\varepsilon)\cdot \Delta v^\varepsilon+z(t,\omega)\int \rho\nabla \Phi\cdot\Delta v^\varepsilon-z(t,\omega) \int f \Delta v^\varepsilon. \end{align} Using the fact that $v^\varepsilon$ is divergence free and integrating by parts, we get \begin{align*} |\frac{1}{z(t,\omega)}\int (v^\varepsilon \cdot \nabla v^\varepsilon)\cdot \Delta v^\varepsilon| \leq \frac{1}{z(t,\omega)}\|\nabla v^\varepsilon \|_{L^4}^{2}\|\nabla v^\varepsilon \|_{L^2}. \end{align*} In view of Ladyzhenskaya's and Young's inequalities, we finally estimate \begin{align}\label{j1} \frac{1}{z(t,\omega)}\|\nabla v^\varepsilon \|_{L^4}^{2}\|\nabla v^\varepsilon \|_{L^2} &\leq \frac{C}{z(t,\omega)}\|\nabla v^\varepsilon \|_{L^2}^{2}\|\Delta v^\varepsilon \|_{L^2}\notag \\ &\leq \frac{\nu}{6}\|\Delta v^\varepsilon \|_{L^2}^2+\frac{C}{z^2(t,\omega)}\|\nabla v^\varepsilon \|_{L^2}^{4}. \end{align} Thanks to elliptic regularity of the fourth equation in \eqref{b1} \begin{align*} \|\nabla \Phi\|_{L^\infty} \leq C\|\rho\|_{L^4}, \end{align*} we estimate \begin{align*} \int \rho\nabla \Phi\cdot\Delta v^\varepsilon \leq \|\nabla \Phi\|_{L^\infty}\|\rho \|_{L^2}\|\Delta v^\varepsilon \|_{L^2} \leq C \|\rho\|_{L^4}\|\rho \|_{L^2}\|\Delta v^\varepsilon \|_{L^2}. \end{align*} Applying Ladyzhenskaya's inequality, we finally have \begin{align*} \int \rho\nabla \Phi\cdot\Delta v^\varepsilon \leq C \|\nabla \rho\|_{L^2}^{\frac{1}{2}}\|\rho \|_{L^2}^{\frac{3}{2}}\|\Delta v^\varepsilon \|_{L^2}. \end{align*} Using Young's inequality again yields \begin{align}\label{j2} z(t,\omega)\int \rho\nabla \Phi\cdot\Delta v^\varepsilon \leq \frac{\nu}{6}\|\Delta v^\varepsilon \|_{L^2}^2+C{z^2 (t,\omega)}\|\nabla \rho \|_{L^2}\|\rho \|_{L^2}^3. \end{align} Similarly, we also estimate \begin{align}\label{j3} |z(t,\omega) \int f \Delta v^\varepsilon| \leq z(t,\omega)\|f\|_{L^2}\|\Delta v^\varepsilon\|_{L^2} \leq \frac{\nu}{6}\|\Delta v^\varepsilon \|_{L^2}^2+C_f+z^4(t,\omega). \end{align} Adding \eqref{j1}, \eqref{j2} and \eqref{j3}, we obtain \begin{align}\label{b4} \frac{d}{dt}&\|\nabla v^\varepsilon\|_{L^2}^{2}\ +\nu \|\Delta v^\varepsilon\|_{L^2}^{2}\notag \\ &\leq \frac{D}{2}\|\nabla \rho \|_{L^2}^2+C{z}^4(t,\omega)\|\rho \|_{L^2}^6+\frac{C}{z^2(t,\omega)}\|\nabla v^\varepsilon \|_{L^2}^{4}+C_f+Cz^4(t,\omega). \end{align} Let $\sigma=c_1+c_2$. Then, $\sigma$ and $\rho$ satisfy \begin{equation} \left\{ \begin{aligned}\label{b5} \partial_{t}\sigma+u\cdot\nabla \sigma=D\Delta \sigma +D\nabla\cdot(\rho\nabla \Phi), \\ \partial_{t}\rho+u\cdot\nabla \rho=D\Delta \rho +D\nabla\cdot(\sigma\nabla \Phi). \\ \end{aligned} \right. \end{equation} Taking the ${L}^{2}$ inner product of the first equation in \eqref{b5} with $\sigma$ and of the second equation with $\rho$, adding them, we obtain \begin{align}\label{ad1} \frac{1}{2}\frac{d}{dt}(\|\sigma\|_{L^2}^{2}\ + \|\rho\|_{L^2}^{2}\ ) =-D (\|\nabla\sigma\|_{L^2}^{2}\ + \|\nabla\rho\|_{L^2}^{2})+ \int \rho\Delta \Phi\sigma. \end{align} Using the fourth equation in \eqref{b1} \begin{align*} -\varepsilon_0 \Delta\Phi=\rho, \end{align*} we find \begin{align*} \int \rho\Delta \Phi\sigma \leq C\| \rho \|_{L^4} \|\rho\|_{L^2} \|\sigma\|_{L^4}. \end{align*} Thanks to Ladyzhenskaya's and Young's inequalities, we estimate \begin{align}\label{ad2} \int \rho\Delta \Phi\sigma \leq \frac{D}{2}(\|\nabla\rho\|_{L^2}^{2} + \|\nabla\sigma\|_{L^2}^{2})+C\|\sigma\|_{L^2}^{4}+C\|\rho\|_{L^2}^{4}. \end{align} Then, adding \eqref{ad1} and \eqref{ad2}, we obtain \begin{align}\label{b6} \frac{d}{dt}( \|\sigma\|_{L^2}^{2}\ + \|\rho\|_{L^2}^{2}\ ) +D(\|\nabla\rho\|_{L^2}^{2} + \|\nabla\sigma\|_{L^2}^{2}) \leq C\|\sigma\|_{L^2}^{4}+C\|\rho\|_{L^2}^{4}. \end{align} Let \begin{align*} M(t)=\|\nabla v^\varepsilon\|_{L^2}^{2}+\|\rho\|_{L^2}^{2}+\|\sigma\|_{L^2}^{2}. \end{align*} Combining \eqref{b4} and \eqref{b6} yields \begin{align}\label{b7} M'&(t)+\frac{D}{2}(\|\nabla\rho\|_{L^2}^{2} + \|\nabla\sigma\|_{L^2}^{2})+\nu \|\Delta v^\varepsilon\|_{L^2}^{2}\notag \\& \leq C{z}^8(t,\omega)+ \frac{C}{{z}^4(t,\omega)}+C{M(t)}^6+C_f+z^4(t,\omega). \end{align} This differential inequality guarantees the boundedness of the desired norms. Consequently, we conclude that \begin{align*} v^\varepsilon \in L^{\infty}(0,T;H^1) \cap L^2(0,T;H^2)~~\mathrm{and}~~c_i \in L^{\infty}(0,T;L^2) \cap L^2(0,T;H^1) . \end{align*} Then we prove the uniqueness. For uniqueness, suppose $(v^\varepsilon_1,c_1^1,c_2^1)$ and $(v^\varepsilon_2,c_1^2,c_2^2)$ are two solutions of system \eqref{b1}. Let $\rho_1=c_1^1-c_2^1$, $\rho_2=c_1^2-c_2^2$, $\sigma_1=c_1^1+c_2^1$, $\sigma_2=c_1^2+c_2^2$. We write $v^\varepsilon=v^\varepsilon_1-v^\varepsilon_2$, $\rho=\rho_1-\rho_2$, $\sigma=\sigma_1-\sigma_2$. Then $v^\varepsilon$, $\sigma$, $\rho$ obey the system \begin{equation} \left\{ \begin{aligned}\label{b8} \displaystyle{\partial_{t}v^\varepsilon + \nu\ Av^\varepsilon =- \frac{1}{z(t,\omega)}\{B(v^\varepsilon_1)-B(v^\varepsilon_2)\}+ ({\rho}_2 \nabla {\Phi}_2-{\rho}_1\nabla {\Phi}_1 )z(t,\omega)}, \\ \partial_{t}\sigma+\frac{1}{z(t,\omega)}(v^\varepsilon_1\cdot\nabla {\sigma}_1-v^\varepsilon_2\cdot\nabla {\sigma}_2)=D\Delta \sigma +D\nabla\cdot({\rho}_1 \nabla {\Phi}_1-{\rho}_2 \nabla {\Phi}_2), \\ \partial_{t}\rho+\frac{1}{z(t,\omega)}(v^\varepsilon_1\cdot\nabla {\rho}_1-v^\varepsilon_2\cdot\nabla {\rho}_2)=D\Delta \rho +D\nabla\cdot({\sigma}_1\nabla {\Phi}_1-{\sigma}_2\nabla {\Phi}_2). \\ \end{aligned} \right. \end{equation} We take the $L^2$ inner product of the first equation of \eqref{b8} with $v^\varepsilon$ and get \begin{align}\label{b9} \frac{1}{2}&\frac{d}{dt} \|v^\varepsilon\|_{L^2}^{2}\ + \nu \|\nabla v^\varepsilon\|_{L^2}^{2}\notag\\& =-\frac{1}{z(t,\omega)}\langle B(v^\varepsilon_1)-B(v^\varepsilon_2),v^\varepsilon \rangle +z(t,\omega)\langle {\rho}_2\nabla {\Phi}_2-{\rho}_1\nabla {\Phi}_1,v^\varepsilon \rangle. \end{align} We estimate the term \begin{align*} |\frac{1}{z(t,\omega)}\langle B(v^\varepsilon_1)-B(v^\varepsilon_2),v^\varepsilon \rangle|&=|\frac{1}{z(t,\omega)} \int (v^\varepsilon_1\cdot\nabla v^\varepsilon_1-v^\varepsilon_2\cdot\nabla v^\varepsilon_2)\cdot v^\varepsilon dx|\\ &=|\frac{1}{z(t,\omega)} \int (v^\varepsilon\cdot\nabla v^\varepsilon_1+v^\varepsilon_2\cdot\nabla v^\varepsilon)\cdot v^\varepsilon dx|. \end{align*} Applying Ladyzhenskaya's and Young's inequalities, we have \begin{align}\label{b10} |\frac{1}{z(t,\omega)} \int (v^\varepsilon\cdot\nabla v^\varepsilon_1+v^\varepsilon_2\cdot\nabla v^\varepsilon)\cdot v^\varepsilon dx| &\leq \frac{C}{z(t,\omega)} \|v^\varepsilon\|_{L^2}^{\frac{3}{2}} \|\nabla v^\varepsilon\|_{L^2}^{\frac{1}{2}}\|\nabla v^\varepsilon_1\|_{L^2}^{\frac{1}{2}}\|\Delta v^\varepsilon_1\|_{L^2}^{\frac{1}{2}}\notag\\&\leq \frac{\nu}{4} \|\nabla v^\varepsilon\|_{L^2}^{2}+{\frac{C}{z^{\frac{4}{3}}(t,\omega)}}\|v^\varepsilon\|_{L^2}^2\|\nabla v^\varepsilon_1\|_{L^2}^{\frac{2}{3}}\|\Delta v^\varepsilon_1\|_{L^2}^{\frac{2}{3}}. \end{align} We estimate the term \begin{align}\label{b11} |z(t,\omega)\langle {\rho}_2\nabla {\Phi}_2-{\rho}_1\nabla {\Phi}_1,v^\varepsilon \rangle|=|z(t,\omega)\int ({\rho}\nabla {\Phi_1}+{\rho}_2\nabla {\Phi})\cdot v^\varepsilon dx|. \end{align} In view of elliptic regularity \begin{align}\label{b12} \|\nabla \Phi\|_{L^\infty} \leq C\|\rho\|_{L^4}, \end{align} we get \begin{align}\label{b13} |z&(t,\omega)\int ({\rho}\nabla {\Phi_1}+{\rho}_2\nabla {\Phi})\cdot v^\varepsilon dx|\notag\\& \leq Cz(t,\omega)[\|\nabla \Phi_1\|_{L^\infty}\|\rho\|_{L^2}\|v^\varepsilon\|_{L^2}+\|\rho_2\|_{L^2}\|\rho\|_{L^2}^{\frac{1}{2}}\|\nabla \rho\|_{L^2}^{\frac{1}{2}}\|v^\varepsilon\|_{L^2}]. \end{align} Then, we take the $L^2$ inner product of the second equation of \eqref{b8} with $\sigma$ and obtain \begin{align}\label{b14} \frac{1}{2}&\frac{d}{dt} \|\sigma\|_{L^2}^{2}\ + D \|\nabla \sigma\|_{L^2}^{2}\notag \\& =-\frac{1}{z(t,\omega)}\int (v^\varepsilon_1\cdot\nabla \sigma_1-v_2\cdot\nabla \sigma_2) \sigma+D\int (\nabla \cdot (\rho_1\nabla \Phi_1-\rho_2\nabla \Phi_2))\sigma. \end{align} We have \begin{align}\label{b15} |\frac{1}{z(t,\omega)}\int (v^\varepsilon_1\cdot\nabla \sigma_1-v^\varepsilon_2\cdot\nabla \sigma_2) \sigma|&=\frac{1}{z(t,\omega)}|\int (v^\varepsilon\cdot\nabla \sigma_1+v^\varepsilon_2\cdot\nabla \sigma) \sigma|\notag \\&\leq \frac{C}{z(t,\omega)}\|\nabla \sigma_1\|_{L^2}\|v^\varepsilon\|_{L^2}^{\frac{1}{2}}\|\nabla v^\varepsilon\|_{L^2}^{\frac{1}{2}}\|\sigma\|_{L^2}^{\frac{1}{2}}\|\nabla \sigma\|_{L^2}^{\frac{1}{2}}, \end{align} and \begin{align}\label{b16} |\int (\nabla \cdot (\rho_1\nabla \Phi_1-\rho_2\nabla \Phi_2))\sigma|&=|\int (\nabla \cdot (\rho\nabla \Phi_1+\rho_2\nabla \Phi))\sigma|\notag\\ &\leq C[\|\nabla \Phi_1\|_{L^\infty}\|\rho \|_{L^2}\|\nabla \sigma\|_{L^2}+\|\rho_2 \|_{L^2}\|\nabla \Phi\|_{L^\infty}\|\nabla \sigma \|_{L^2}]\notag\\&\leq C[\|\nabla \Phi_1\|_{L^\infty}\|\rho \|_{L^2}\|\nabla \sigma\|_{L^2}+\|\rho_2 \|_{L^2}\|\rho \|_{L^2}^{\frac{1}{2}}\|\nabla \rho \|_{L^2}^{\frac{1}{2}}\|\nabla \sigma \|_{L^2}]. \end{align} Finally, we take the $L^2$ inner product of the third equation of \eqref{b8} with $\rho$ and obtain \begin{align}\label{b17} \frac{1}{2}&\frac{d}{dt} \|\rho\|_{L^2}^{2}\ + D \|\nabla \rho\|_{L^2}^{2}\notag\\& =-\frac{1}{z(t,\omega)}\int (v^\varepsilon_1\cdot\nabla \rho_1-v^\varepsilon_2\cdot\nabla \rho_2)\rho+D\int (\nabla \cdot (\sigma_1\nabla \Phi_1-\sigma_2\nabla \Phi_2))\rho. \end{align} Estimating the terms as above, we also have \begin{align}\label{b18} |\frac{1}{z(t,\omega)}\int (v^\varepsilon_1\cdot\nabla \rho_1-v^\varepsilon_2\cdot\nabla \rho_2)\cdot \rho|&=\frac{1}{z(t,\omega)}|\int (v^\varepsilon\cdot\nabla \rho_1+v^\varepsilon_2\cdot\nabla \rho)\cdot \rho|\notag\\ &\leq \frac{C}{z(t,\omega)}\|\nabla \rho_1\|_{L^2}\|v^\varepsilon\|_{L^2}^{\frac{1}{2}}\|\nabla v^\varepsilon\|_{L^2}^{\frac{1}{2}}\|\rho\|_{L^2}^{\frac{1}{2}}\|\nabla \rho\|_{L^2}^{\frac{1}{2}}, \end{align} and \begin{align}\label{b19} |\int (\nabla \cdot (\sigma_1\nabla \Phi_1-\sigma_2\nabla \Phi_2))\rho|&=|\int (\nabla \cdot (\sigma\nabla \Phi_1+\sigma_2\nabla \Phi))\rho| \notag\\&\leq C[\|\nabla \Phi_1\|_{L^\infty}\|\sigma \|_{L^2}\|\nabla \rho\|_{L^2}+\|\sigma_2 \|_{L^2}\|\rho \|_{L^2}^{\frac{1}{2}}\|\nabla \rho \|_{L^2}^{\frac{3}{2}}]. \end{align} Adding \eqref{b10} to \eqref{b19} together and applying Young's inequality, we deduce \begin{align}\label{b20} \frac{d}{dt}[\|v^\varepsilon\|_{L^2}^2+\|\rho\|_{L^2}^2+\|\sigma\|_{L^2}^2] \leq CC(t)[\|v^\varepsilon\|_{L^2}^2+\|\rho\|_{L^2}^2+\|\sigma\|_{L^2}^2], \end{align} where \begin{align*} C(t)=&\|\nabla v^\varepsilon_1\|_{L^2}\|\Delta v^\varepsilon_1\|_{L^2}+\|\nabla \rho_1\|_{L^2}^{4}+\|\nabla \sigma_1\|_{L^2}^{4}+\| \sigma_2\|_{L^2}^{4}+\|\rho _2\|_{L^2}^{4}\notag \\&+{z^2(t,\omega)}+\frac{1}{{z^4(t,\omega)}}+1. \end{align*} Since \begin{align*} \int_{0}^{t} C(s)ds \textless \infty, \end{align*} for any $t \in[0,T_0]$, then we can prove the uniqueness. \end{proof} \end{thm} Next, the following theorem shows that the solution is continuous corresponding to initial data. \begin{thm}\label{thm3} Suppose $v^\varepsilon(0) \in H^1$ and $c_i(0) \in {L}^{2}$. Then, the solution of \eqref{b1} is continuous in initial data. \begin{proof} Applying Lemma \ref{lem1} to \eqref{b20}, we get \begin{align}\label{b21} \|v^\varepsilon\|_{L^2}^2+\|\rho\|_{L^2}^2+\|\sigma\|_{L^2}^2 \leq e^{C\int_{0}^{T_0} C(s)ds}(\|v^\varepsilon(0)\|_{L^2}^2+\|\rho(0)\|_{L^2}^2+\|\sigma(0)\|_{L^2}^2). \end{align} Hence the proof is completed. \end{proof} \end{thm} Then, we explore the existence of global regular solutions under the following condition \eqref{b22}. \begin{thm}\label{thm4} Suppose $v^\varepsilon(0) \in H^1$ and $c_i(0) \in H^1$. Let $T > 0$. Suppose $(v^\varepsilon,c_1,c_2)$ is the solution of \eqref{b1} on the interval $[0,T]$ with \begin{align}\label{b22} \int_{0}^{T}(\|c_1(t)\|_{L^2}^2+\|c_2(t)\|_{L^2}^2)dt \textless \infty. \end{align} Then, $v^\varepsilon \in L^{\infty}(0,T;H^1) \cap L^2(0,T;H^2)$ and $c_i \in L^{\infty}(0,T;H^1) \cap L^2(0,T;H^2) $. \begin{proof} The differential inequality \eqref{b6} gives \begin{align}\label{b23} \frac{d}{dt}( \|\sigma\|_{L^2}^{2}\ + \|\rho\|_{L^2}^{2}\ ) +D(\|\nabla\rho\|_{L^2}^{2} + \|\nabla\sigma\|_{L^2}^{2}) \leq C\|\sigma\|_{L^2}^{4}+C\|\rho\|_{L^2}^{4} \leq C(\|\sigma\|_{L^2}^{2}+C\|\rho\|_{L^2}^{2})^2. \end{align} Under the assumption \eqref{b22}, we conclude that $c_i \in L^{\infty}(0,T;L^2) \cap L^2(0,T;H^1) $. \newline The differential inequality \eqref{b4} gives \begin{align}\label{b24} \frac{d}{dt}&\|\nabla v^\varepsilon\|_{L^2}^{2}\ +\nu \|\Delta v^\varepsilon\|_{L^2}^{2}\notag \\&\leq \frac{D}{2}\|\nabla \rho \|_{L^2}^2+C{z}^4(t,\omega)\|\rho \|_{L^2}^6+\frac{C}{z^2(t,\omega)}\|\nabla v^\varepsilon \|_{L^2}^{4}+C_f+Cz^4(t,\omega). \end{align} Thus, we obtain that $v^\varepsilon \in L^{\infty}(0,T;H^1) \cap L^2(0,T;H^2)$. \newline Taking the $L^2$ inner product of the equation satisfied by $\sigma$ in \eqref{b5} with -$\Delta \sigma$, we get \begin{align}\label{b25} \frac{1}{2}\frac{d}{dt} \|\nabla \sigma\|_{L^2}^{2}\ + D \|\Delta \sigma\|_{L^2}^{2} =\frac{1}{z(t,\omega)}\int (v^\varepsilon\cdot \nabla \sigma)\Delta \sigma -D\int \nabla\cdot(\rho \nabla \Phi)\Delta \sigma. \end{align} We estimate \begin{align}\label{b26} |\int \rho \Delta \Phi \Delta \sigma| \leq \frac{1}{6}\|\Delta \sigma\|_{L^2}^2+C\|\rho\|_{L^2}^4+C\|\nabla \rho\|_{L^2}^4, \end{align} \begin{align}\label{b27} |\int (\nabla \rho \cdot \nabla \Phi) \Delta \sigma| \leq \frac{1}{6}\|\Delta \sigma\|_{L^2}^2+C\|\nabla \rho\|_{L^2}^4, \end{align} and \begin{align}\label{b28} |\frac{1}{z(t,\omega)}\int (v^\varepsilon\cdot \nabla \sigma)\Delta \sigma| \leq \frac{D}{6}\|\Delta \sigma\|_{L^2}^2+\frac{C}{z^4(t,\omega)}+C\|\nabla v^\varepsilon\|_{L^2}^4\|\nabla \sigma\|_{L^2}^4, \end{align} where we used elliptic regularity, Young's inequality, Ladyzhenskaya's inequality and Poincar$\acute{\mathrm{e}}$'s inequality. Then, we take the $L^2$ inner product of the equation satisfied by $\rho$ in \eqref{b5} with -$\Delta \rho$ to obtain \begin{align}\label{b29} \frac{1}{2}\frac{d}{dt} \|\nabla \rho\|_{L^2}^{2}\ + D \|\Delta \rho\|_{L^2}^{2} =\frac{1}{z(t,\omega)}\int (v^\varepsilon\cdot \nabla \rho)\Delta \rho -D\int \nabla\cdot(\sigma \nabla \Phi)\Delta \rho. \end{align} Similarly, we estimate \begin{align}\label{b30} |\int \sigma \Delta \Phi \Delta \rho| \leq \frac{1}{6}\|\Delta \rho\|_{L^2}^2+C\|\sigma\|_{L^2}^2\|\nabla \sigma\|_{L^2}^2+C\|\nabla \rho\|_{L^2}^4, \end{align} \begin{align}\label{b31} |\int (\nabla \sigma \cdot \nabla \Phi) \Delta \rho| \leq \frac{1}{6}\|\Delta \rho\|_{L^2}^2+C\|\nabla \rho\|_{L^2}^4+C\|\nabla \sigma\|_{L^2}^4, \end{align} and \begin{align}\label{b32} |\frac{1}{z(t,\omega)}\int (v^\varepsilon\cdot \nabla \rho)\Delta \rho|&=|z^{-1}\int (\nabla v^\varepsilon \nabla \rho \nabla \rho|\notag\\& \leq \frac{D}{6}\|\Delta \rho\|_{L^2}^2+\frac{C}{z^4(t,\omega)}+C\|\nabla v^\varepsilon\|_{L^2}^4\|\nabla \rho\|_{L^2}^4. \end{align} Adding \eqref{b25} to \eqref{b32} together, we conclude that $c_i \in L^{\infty}(0,T;H^1) \cap L^2(0,T;H^2) $ with bounds depending on the initial data and $T$. \end{proof} \end{thm} The following remark is cited to provide the needed estimates. \begin{rem}[See \cite{P.M2019}]\label{Remark 1} Under the conditions of Theorem \ref{thm4}, if $c_i(0) \geq 0$, then $c_i(t) \geq 0$ for $0 \leq t \leq T.$ \end{rem} Before presenting the final result in this section, we also need to give a priori $L^2$ bounds. \begin{prop}\label{Prop5} Let $v^\varepsilon(0) \in H^1$ and $c_i(0) \in H^1$. Assume that $c_i(t) \geq 0$ holds for all $t \geq 0 $. Then, there exists an absolute constant $C > 0 $ such that \begin{align}\label{b33} &\int_{t}^{t+T}(\|\nabla \rho (s)\|_{L^2}^2+\|\nabla \sigma (s)\|_{L^2}^2+\frac{1}{\varepsilon}\|\nabla \rho (s)\|_{L^3}^3)ds\notag\\& \leq \frac{1}{2D}(2\|\sigma(t_0)\|_{L^2}^2+2\|\overline {\sigma}\|_{L^2}^2+\|\rho(t_0)\|_{L^2}^2)Te^{-2CDt}, \end{align} for all $t \geq 0$, $T>0$. \begin{proof} The proof is similar to Proposition $3$ in \cite{E.M2021}, so we omit it. \end{proof} \end{prop} Finally, with the results we have just proved, we are now ready to show that this local solution can be extended to a strong analytic solution on $[0,T]$ for any $T > 0$. \begin{thm}\label{thm5} Let $v^\varepsilon(0)$ $\in$ $H^1$ and $c_i(0) \in H^1$ be nonnegative with $c_i(0) \geq 0 $. Let $T > 0$. Then, there exists a unique solution $(v^\varepsilon,c_1,c_2)$ satisfying $v^\varepsilon \in L^{\infty}(0,T;H^1) \cap L^2(0,T;H^2)$ and $c_i \in L^{\infty}(0,T;H^1) \cap L^2(0,T;H^2) $. Moreover $c_i(t) \geq 0$ holds on $[0,T]$. \begin{proof} By the local existence theorem (Theorem \ref{thm2}), there exists $T_0 \geq 0$ depending only on the norms of initial data in $H^1$ such that the solution exists and belongs to $H^1$. By Remark \ref{Remark 1}, we find $c_i(t) \geq 0$. Moreover, the inequality \eqref{b33} holds on $[0,T_0]$. It follows from Theorem \ref{thm4} that the solution is bounded in $H^1$. Applying the local existence theorem again and starting from $T_0$, we deduce that the solution can be extended for $T_1$ $\geq T_0$. The inequality \eqref{b33} holds on $[0,T_1]$. Since the inequality holds as long as $c_i$ $\geq 0$, reasoning by contradiction we conclude that the solution can extend to the whole interval $[0,T]$. \end{proof} \end{thm} \section{\large Random Attractors}\label{sec4} In the previous section, we have proved existence, uniqueness, and continuous dependence of solutions with respect to initial data. Let us denote the solution of system \eqref{b2} by \begin{align*} (v^\varepsilon(t,\omega,t_0,v^\varepsilon_0),\sigma(t,\omega,t_0,\sigma_0),\rho(t,\omega,t_0,\rho_0)). \end{align*} By Theorem \ref{thm3}, $v^\varepsilon$ is continuous with respect to initial data and measurable in $\omega \in \Omega$. Thus it is clear that the mapping $(v^\varepsilon_0,\sigma_0,\rho_0)\mapsto(v^\varepsilon(t,\omega,t_0,v^\varepsilon_0),\sigma(t,\omega,t_0,\sigma_0),\rho(t,\omega,t_0,\rho_0))$ is continuous for all $t \geq t_0$. These guarantee that we can define a cocycle $S(t,\omega)$ by \begin{align*} S(t,\omega)x_0= (u,\sigma,\rho)=(z^{-1}(t,\omega)v^\varepsilon(t,\omega,0,u_0),\sigma(t,\omega,0,\sigma_0),\rho(t,\omega,0,\rho_0)),~t \geq 0, \end{align*} where $x_0= ( u_0,\sigma_0,\rho_0 )$. By Theorem \ref{def0}, it is easy to find that $S(t,\omega)$ is a continuous RDS on $H$ over ${\{\theta_t\}}_{t \in \mathbb{R}}$ and $(\Omega,\mathcal{F},P,{\{\theta_t\}}_{t \in \mathbb{R}})$, which implies that \begin{align*} S(t,\theta_{-t}\omega) x_0=S(0,\omega,-t,x_0). \end{align*} In this section, we proceed in several steps. First it can be shown that the random dynamical system associated with system \eqref{b2} has a random absorbing set in $H$. Next we are able to prove that there also exists a random absorbing set in $V$. Finally, the existence of a compact random attractor in $H$ is a direct consequence by making use of the Sobolev embedding theorem and Theorem \ref{thm1}. For more basic theory concerning random attractors, we can refer to \cite{Z.M.F1993, H.F1994, H.A.F1997, F.B1996,R.Z2017}. In order to prove these conclusions, we need to provide some auxiliary lemmas as follows. \begin{lem}\label{lem5} Let $(v^\varepsilon,\sigma,\rho)$ be a solution of system \eqref{b2}. Let $v^\varepsilon(t_0)$ $\in$ $H^1$ and $c_i(t_0) \in {L}^{2}$. Then, \begin{align}\label{d1} \|\sigma\|_{L^2}^{2}\ + \|\rho\|_{L^2}^{2}\ \leq (\|\sigma(t_0)\|_{L^2}^{2}\ + \|\rho(t_0)\|_{L^2}^{2})e^{2D(t_0-t)},~t \geq t_0. \end{align} \begin{proof} We recall $\sigma$ and $\rho$ obey \begin{equation} \left\{ \begin{aligned}\label{d2} \partial_{t}\sigma+z^{-1}(v^\varepsilon\cdot\nabla \sigma)=D\Delta \sigma +D\nabla\cdot(\rho\nabla \Phi) \\ \partial_{t}\rho+z^{-1}(v^\varepsilon\cdot\nabla \rho)=D\Delta \rho +D\nabla\cdot(\sigma\nabla \Phi) \\ \end{aligned} \right. \end{equation} Taking the ${L}^{2}$ inner product of the first equation in \eqref{d2} with $\sigma$ and of the second equation with $\rho$, we add them and have \begin{align*} \frac{1}{2}\frac{d}{dt}( \|\sigma\|_{L^2}^{2}\ + \|\rho\|_{L^2}^{2}\ ) =-D (\|\nabla\sigma\|_{L^2}^{2}\ + \|\nabla\rho\|_{L^2}^{2}\ )+ \int \rho\Delta \Phi\sigma. \end{align*} In view of the fourth equation in \eqref{b2}, we know that \begin{align*} \int \rho\Delta \Phi\sigma=-\frac{1}{\varepsilon_0} \int {\rho}^2\sigma, \end{align*} where $\sigma > 0$. Then, we get \begin{align}\label{d3} \frac{d}{dt}( \|\sigma\|_{L^2}^{2}\ + \|\rho\|_{L^2}^{2}\ ) +2D(\|\nabla\rho\|_{L^2}^{2} + \|\nabla\sigma\|_{L^2}^{2}) \leq 0. \end{align} Thanks to Lemma \ref{lem1}, we obtain \eqref{d1}. \end{proof} \end{lem} \begin{lem}\label{lem6} Let $v^\varepsilon(t_0)$ $\in$ $H^1$ and $c_i(t_0) \in H^1$. Then, there exists a positive constant $a$ depending on $D$, $\varepsilon_0$ and $\nu$, and a positive constant $A$ depending on the initial data, universal constants, $\varepsilon$ and $\omega$ such that \begin{align}\label{d4} \|\nabla \rho (t)\|_{L^2}^2+\|\nabla \sigma (t)\|_{L^2}^2 \leq Ae^{-a(t-t_0)}, \end{align} for all $t \geq t_0$. \begin{proof} Adding \eqref{b26} to \eqref{b32} together, the conclusion can be proved. \end{proof} \end{lem} Lemma \ref{lem5} and Lemma \ref{lem6} show that $\sigma$ and $\rho$ decay exponentially in $H$ and $V$. \begin{lem}\label{lem7} Let $(v^\varepsilon,\sigma,\rho)$ be a solution of system \eqref{b2}. Let $v^\varepsilon(t_0)$ $\in$ $H^1$ and $c_i(t_0) \in H^1$. Then, \begin{align}\label{d5} \|v^\varepsilon(t)\|_{L^2}^{2}\ \leq {e}^{-\nu t}[ \|v^\varepsilon(t_0)\|_{L^2}^{2}\ {e}^{\nu t_0} +\frac{2}{\nu}\int_{t_{0}}^{t} {z^{2}(s,w)}(\|\rho(s) \|_{L^2}^{3}\|\nabla \rho(s) \|_{L^2}+\|f\|_{L^2}^2) {e}^{\nu s}ds ], \end{align} for $t \geq t_0$. \begin{proof} Taking the ${L}^{2}$ inner product of the first equation in \eqref{b2} with $v^\varepsilon$ yields \begin{align}\label{jia1} \frac{1}{2}\|v^\varepsilon\|_{L^2}^{2}\ +\nu \|\nabla v^\varepsilon\|_{L^2}^{2}=-(\rho\nabla\Phi,v^\varepsilon)z(t,w)+(f,v^\varepsilon)z(t,w). \end{align} In view of Young's and Ladyzhenskaya's inequalities, we estimate \begin{align}\label{jia2} |(\rho\nabla\Phi,v^\varepsilon)z(t,w)| \leq \frac{\nu}{4}\|\nabla v^\varepsilon\|_{L^2}^{2}+\frac{1}{\nu}{z^{2}(t,w)}\|\rho \|_{L^2}^{3}\|\nabla \rho \|_{L^2}, \end{align} and \begin{align}\label{jia3} |(f,v)z(t,w)| \leq \frac{\nu}{4}\|\nabla v^\varepsilon\|_{L^2}^{2}+\frac{1}{\nu}{z^{2}(t,w)}\|f\|_{L^2}^2. \end{align} Putting \eqref{jia1} to \eqref{jia3} together, we get \begin{align}\label{d6} \frac{d}{dt} \|v^\varepsilon\|_{L^2}^{2}\ + \nu \|\nabla v^\varepsilon\|_{L^2}^{2} \leq\frac{2}{\nu}{z^{2}(t,w)}\|\rho \|_{L^2}^{3}\|\nabla \rho \|_{L^2}+\frac{2}{\nu}{z^{2}(t,w)}\|f\|_{L^2}^2. \end{align} By Lemma \ref{lem1}, we obtain \eqref{d5}. \end{proof} \end{lem} Lemma \ref{lem7} says $v^\varepsilon$ decays exponentially in $H$. By making full use of the lemmas above, we prove further that there exists a random absorbing set in $H$. \begin{lem}\label{lem8} There exist a positive number $R_0$ and a random variable $R_1(\omega)$ depending on the initial data, $\varepsilon_0$, $\nu$, $f$ and universal constants, such that: For every $E >0 $, there exists $t(\omega,E) \leq -1$, such that for all $x_0=(u_0,\rho_0,\sigma_0) \in {H^1}$ with $|x_0| < E$, and for any $t_0 < t(\omega,E)$, we have \begin{align}\label{d7} \|\sigma(t,\omega,t_0,\sigma_0)\|_{L^2} \leq R_0,~for~all~t \in [-1,0], \end{align} \begin{align}\label{d8} \|\rho(t,\omega,t_0,\rho_0)\|_{L^2} \leq R_0,~for~all~t \in [-1,0], \end{align} \begin{align}\label{d9} \|v^\varepsilon(t,\omega,t_0,z(t_0,\omega)u_0)\|_{L^2} \leq R_1(\omega),~for~all~t \in [-1,0]. \end{align} Therefore, $B(0,2R_0+R_1(\omega))$ is a random absorbing set in H. \begin{proof} By Lemma \ref{lem5}, for all $t \in [-1,0]$ and $t_0 \leq 1$, we get \begin{align*} \|\sigma\|_{L^2}^{2}\ + \|\rho\|_{L^2}^{2}\ \leq (\|\sigma(t_0)\|_{L^2}^{2}\ + \|\rho(t_0)\|_{L^2}^{2}\ )e^{2D(t_0+1)} \leq 2{E^2}e^{2D(t_0+1)}. \end{align*} Choose $t_1$$(E)$ $\leq -1$ such that \begin{align*} 2{E^2}e^{2D(t_1+1)} \leq {R_0}^2, \end{align*} provided $t_0 \leq t_1(E)$, which proves \eqref{d7} and \eqref{d8}.\newline We see from Lemma \ref{lem7} that \begin{align}\label{d10} \|v^\varepsilon(t)\|_{L^2}^{2}\ \leq {e}^{-\nu t}[ {z}^{2}(t_0,\omega) \|u_0\|_{L^2}^{2}\ {e}^{\nu t_0} +\frac{2}{\nu}\int_{t_{0}}^{t} {z^{2}(s,w)}(\|\rho(s) \|_{L^2}^{3}\|\nabla \rho(s) \|_{L^2}+\|f\|_{L^2}^2) {e}^{\nu s}ds], \end{align} for all $t \in [-1,0]$ and $t_0 \leq -1$. Then by \eqref{l1}, we notice that \begin{align*} {z}^{2}(t_0,\omega)e^{\nu t_0} \to 0,~ t \to -\infty, \end{align*} which implies that there exists $t_2(\omega,E) \leq -1$ such that if $t_0 \leq t_2$, \begin{align*} {z}^{2}(t_0,\omega) \|u_0\|_{L^2}^{2}\ {e}^{\nu t_0} \leq E^2{z}^{2}(t_0,\omega) {e}^{\nu t_0} \leq 1. \end{align*} Then, we estimate the integrals on the right hand of \eqref{d10}. \newline Let \begin{align*} I&=\int_{t_{0}}^{t} {z(s,w)}^{2}\|\rho(s) \|_{L^2}^{3}\|\nabla \rho(s) \|_{L^2} {e}^{\nu s}ds\\ &=(\int_{\frac{t_0}{2}}^{t} + \int_{t_0}^{\frac{t_0}{2}}) [{z^{2}(s,w)}\|\rho(s) \|_{L^2}^{3}\|\nabla \rho(s) \|_{L^2} {e}^{\nu s}]ds \\ &=I_1+I_2. \end{align*} To estimate $I_1$, if $\frac{t_0}{2} \textless s \textless t$, then \begin{align*} \|\rho(s)\|_{L^2} \leq R_0, \end{align*} provided $t_0$ $\leq$ $t_3(E)$ for some chosen $t_3(E)$ $\leq -1$. \newline We see from Lemma \ref{lem6} that \begin{align*} \|\nabla \rho (t)\|_{L^2}^2+\|\nabla \sigma (t)\|_{L^2}^2 \leq Ae^{-a(t-t_0) }. \end{align*} If $\frac{t_0}{2}< s < t$, then \begin{align*} \|\nabla \rho (s)\|_{L^2}^2 \leq A, \end{align*} $A$ is given in lemma \ref{lem6}, provided $t_0$ $\leq$ $t_3(E)$ for some chosen $t_3(E)$ $\leq -1$. \newline For $t_0 \leq t_3(E)$, $t \in [-1,0]$, \begin{align*} I_1 \leq \sqrt A{R_0}^3\int_{-\infty}^{0} {z}^{2}(s,w) {e}^{\nu s} ds = r_1(\omega), \end{align*} $r_1(\omega)$ is a random variable being independent of $E$. Since $\frac{w(t)}{t}$ $\to 0$ as $t \to$ $\pm$ $\infty$, the ergodic property implies that $s \mapsto$ ${z}^{2}(s,w) {e}^{\nu s}$ is pathwise integrable on $(-\infty,0]$.\newline To estimate $I_2$, we choose $t_4(E) \leq -1$ such that \begin{align*} 8\sqrt A E^3e^{\frac{\nu}{2}t_4} \leq 1. \end{align*} By Lemma \ref{lem5}, we deduce \begin{equation*} \|\rho(s)\|_{L^2}\ \leq 2E, \end{equation*} for all s $\in [t_0,\frac{t_0}{2}]$. If $\frac{t_0}{2} \leq t_4(E)$, then we derive \begin{align*} I_2&=\int_{t_0}^{\frac{t_0}{2}} {z}^{2}(s,w)\|\rho(s)\|_{L^2}^{3}\|\nabla \rho(s)\|_{L^2} {e}^{\nu s}ds \\ &\leq \int_{t_0}^{\frac{t_0}{2}} [8\sqrt A E^3{e}^{\frac{\nu}{2} s}]{z}^{2}(s,w) {e}^{\frac{\nu}{2} s}ds \\ &\leq \int_{t_0}^{\frac{t_0}{2}} {z}^{2}(s,w) {e}^{\frac{\nu}{2} s}ds \\ &\leq \int_{-\infty}^{0} {z}^{2}(s,w) {e}^{\frac{\nu}{2} s}ds=r_2(\omega). \end{align*} Let \begin{align*} J=\int_{t_{0}}^{t} {z(s,w)}^{2}\|f\|_{L^2}^{2}{e}^{\nu s}ds. \end{align*} Then, we estimate \begin{align*} J \leq \int_{-\infty}^{0} {z(s,w)}^{2}\|f\|_{L^2}^{2}{e}^{\nu s}ds=R_{f,\nu}(\omega), \end{align*} Let $t(\omega,E)=min\{t_1,t_2,t_3,2t_4\}$ and \begin{align*} R_1(\omega)=e^{\nu}[1+\frac{2}{\nu}(r_1(\omega)+r_2(\omega)+R_{f,\nu}(\omega))]. \end{align*} Finally, since \begin{align*} S(-t_0,\theta_{t_0} \omega)x_0=x(0,\omega,t_0,x_0)=\{v^\varepsilon(0,\omega,t_0,z(t_0,\omega)u_0),\rho(0,\omega,t_0,\rho_0),\sigma(0,\omega,t_0,\sigma_0)\}, \end{align*} we can obtain that $|S(-t_0,\theta_{t_0} \omega)x_0| \leq 2R_0+R_1(\omega)$ if $ t_0 \leq t(\omega,E)$, which means $B(0,2R_0+R_1(\omega))$ is a random absorbing set in $H$. \end{proof} \end{lem} Lemma \ref{lem8} gets a random absorbing set in $H$. In order to obtain the asymptotic compactness of the solution in $H$, we need to show that $v^\varepsilon(t)$, $\rho(t)$ and $\sigma(t)$ are bounded in $V$. The following lemma plays a crucial role in the proof of our desired conclusion. \begin{lem}\label{lem9} There exist a positive number $R_2$ depending on $D$ and universal constants, and a random variable $R_3(\omega)$ depending on the initial data, $\varepsilon_0$, $\nu$, $f$ and universal constants, such that: For every $E > 0$, there exists $t(\omega,E) \leq -1$, such that for all $t_0 \leq t(\omega,E)$ and $x_0=(u_0,\rho_0,\sigma_0)$ $\in{H}^{1}$ with $|x_0| < E$, we have \begin{align}\label{f1} \int_{-1}^{0} \|\nabla \sigma(s,\omega,t_0,\sigma_0)\|_{L^2}^{2}ds \leq R_2, \end{align} \begin{align}\label{f2} \int_{-1}^{0} \|\nabla \rho(s,\omega,t_0,\rho_0)\|_{L^2}^{2}ds \leq R_2, \end{align} \begin{align}\label{f3} \int_{-1}^{0} \|\nabla v^\varepsilon(s,\omega,t_0,z(t_0,\omega)u_0)\|_{L^2}^{2}ds \leq R_3(\omega). \end{align} \begin{proof} The differential inequality \eqref{d3} gives \begin{align}\label{d12} \frac{d}{dt}( \|\sigma\|_{L^2}^{2}\ + \|\rho\|_{L^2}^{2}\ ) +2D(\|\nabla\rho\|_{L^2}^{2} + \|\nabla\sigma\|_{L^2}^{2}) \leq 0. \end{align} Integrating \eqref{d12} between $-1$ and $0$ yields \begin{align*} &\|\sigma(0)\|_{L^2}^{2}-\|\sigma(-1)\|_{L^2}^{2}+\|\rho(0)\|_{L^2}^{2}-\|\rho(-1)\|_{L^2}^{2}+2D\int_{-1}^{0} (\|\nabla \sigma(s)\|_{L^2}^{2}+\|\nabla \rho(s)\|_{L^2}^{2})ds \leq 0. \end{align*} It follows that \begin{align*} \int_{-1}^{0} (\|\nabla \sigma(s)\|_{L^2}^{2}+\|\nabla \rho(s)\|_{L^2}^{2})ds &\leq \frac{1}{2D}(\|\sigma(-1)\|_{L^2}^{2}+\|\rho(-1)\|_{L^2}^{2}) \leq \frac{{R_0}^2}{2D}=R_2, \end{align*} which proves \eqref{f1} and \eqref{f2}. Integrating \eqref{d6} between -1 and 0, we get \begin{align*} \|&v^\varepsilon(0)\|_{L^2}^{2}-\|v^\varepsilon(-1)\|_{L^2}^{2}+\nu \int_{-1}^{0} \|\nabla v^\varepsilon(s)\|_{L^2}^{2}ds\\& \leq \frac{1}{\nu}\int_{-1}^{0} {z^2(s,\omega)}(\|\rho(s)\|_{L^2}^{3}\|\nabla \rho(s)\|_{L^2}+\|f\|_{L^2}^2)ds. \end{align*} Thus, we have \begin{align*} \int_{-1}^{0} \|\nabla v^\varepsilon(s)\|_{L^2}^{2}ds \leq \frac{1}{\nu}{R_1}^2(\omega)+\frac{{R_0}^3}{{\nu}^2}(\int_{-1}^{0} z^4(s,\omega) ds +R_2+C_f)=R_3(\omega), \end{align*} which proves \eqref{f3}. \end{proof} \end{lem} Then, we provide the $H^1$ boundedness of the solution. \begin{lem}\label{lem10} There exist two random variables $R_4(\omega)$ and $R_5(\omega)$ depending on the initial data, $\varepsilon_0$, $\nu$, $D$, $f$ and universal constants, such that for every $E > 0$, there exists $t(\omega,E) \leq -1$, such that for all $t_0 \leq t(\omega,E)$ and $x_0=(u_0,\rho_0,\sigma_0) \in {H}^{1}$ with $|x_0| < E$, we have \begin{align}\label{d13} \|\nabla v^\varepsilon(0,\omega,t_0,z(t_0,\omega)u_0)\|_{L^2} \leq R_4(\omega),~for~all~t \in [-1,0], \end{align} \begin{align}\label{d14} \|\nabla \sigma(0,\omega,t_0,\sigma_0)\|_{L^2} \leq R_5(\omega),~for~all~t \in [-1,0], \end{align} \begin{align}\label{d15} \|\nabla \rho(0,\omega,t_0,\rho_0)\|_{L^2} \leq R_5(\omega),~for~all~t \in [-1,0]. \end{align} Therefore, $B(0,R_4(\omega)+2{R_5(\omega)})$ is a random absorbing set in $V$. \begin{proof} The differential inequality \eqref{b4} gives \begin{align*} \frac{d}{dt}\|\nabla v^\varepsilon\|_{L^2}^{2}\ +\nu \|\Delta v^\varepsilon\|_{L^2}^{2} \leq& \frac{C}{{z^2(t,\omega)}}\|\nabla v^\varepsilon\|_{L^2}^{4}+\frac{D}{2}\|\nabla \rho \|_{L^2}^2 +C{z^4(t,\omega)}\|\rho \|_{L^2}^6\\&+C_f+Cz^4(t,\omega), \end{align*} which implies that \begin{align*} \frac{d}{dt}\|\nabla v^\varepsilon\|_{L^2}^{2}\ \leq& \frac{C}{{z^2(t,\omega)}}\|\nabla v^\varepsilon \|_{L^2}^{2}\|\nabla v^\varepsilon \|_{L^2}^{2}+\frac{D}{2}\|\nabla \rho \|_{L^2}^2+C{z^4(t,\omega)}\|\rho \|_{L^2}^6\\&+C_f+Cz^4(t,\omega). \end{align*} Let \begin{align*} f(t)=\frac{C}{{z^2(t,\omega)}}\|\nabla v^\varepsilon(t) \|_{L^2}^{2}. \end{align*} Applying Lemma \ref{lem1} on an arbitrary interval $[s,0] \subset [-1,0]$, we find \begin{align*} \|\nabla v^\varepsilon(0)\|_{L^2}^{2} \leq& e^{\int_{s}^{0} f(\xi)d\xi}[\|\nabla v^\varepsilon(s)\|_{L^2}^{2}+\frac{D}{2}\int_{s}^{0}\|\nabla \rho(\xi) \|_{L^2}^{2}d\xi+C\int_{s}^{0} {z^4(\xi,\omega)}\|\rho(\xi) \|_{L^2}^6 d\xi\\ &+\int_{s}^{0} (C_f+Cz^4(\xi,\omega))d\xi]. \end{align*} Integrating $s$ on the interval $[-1,0]$ yields \begin{align*} \|\nabla v^\varepsilon(0)\|_{L^2}^{2} \leq& e^{\int_{-1}^{0} f(\xi)d\xi}[\int_{-1}^{0}\|\nabla v^\varepsilon(s)\|_{L^2}^{2}ds+\frac{D}{2}\int_{-1}^{0}\|\nabla \rho(s) \|_{L^2}^2ds+C\int_{-1}^{0} {z^4(\xi,\omega)}\|\rho(\xi) \|_{L^2}^6d\xi\\&+\int_{-1}^{0} (C_f+Cz^4(\xi,\omega))d\xi]. \end{align*} We estimate \begin{align}\label{d16} \int_{-1}^{0} f(\xi)d\xi &\leq C\mathop {sup}\limits_{t \in [-1,0]} \frac{1}{z^2(t,\omega)} \int_{-1}^{0}\|\nabla v^\varepsilon(\xi) \|_{L^2}^2d\xi \notag\\&\leq CR_3(\omega)\mathop {sup}\limits_{t \in [-1,0]} \frac{1}{z^2(t,\omega)}=r_3(\omega), \end{align} and \begin{align*} \int_{-1}^{0} {z^4(\xi,\omega)}\|\rho(\xi) \|_{L^2}^6d\xi &\leq \mathop {sup}\limits_{t \in [-1,0]} z^4(t,\omega) \mathop {sup}\limits_{t \in [-1,0]}\|\rho(t) \|_{L^2}^6\\& \leq {R_0}^6\mathop {sup}\limits_{t \in [-1,0]} z^4(t,\omega)=r_4(\omega). \end{align*} We also estimate \begin{align*} \int_{-1}^{0} (C_f+Cz^4(\xi,\omega))d\xi \leq r_5(f,\omega), \end{align*} and \begin{align*} \frac{D}{2}\int_{-1}^{0} \|\nabla \rho(s)\|_{L^2}^{2}ds \leq \frac{D}{2}R_2. \end{align*} Consequently, let \begin{align*} R_4(\omega)=e^{r_3(\omega)}[{R_3}^2(\omega)+\frac{D}{2}R_2+r_4(\omega)+r_5(f,\omega)]. \end{align*} This proves \eqref{d13}. Similarly, using \eqref{b26} to \eqref{b32}, we can get the following differential inequality \begin{align*} \frac{d}{dt}(\|\nabla \rho\|_{L^2}^{2}+\|\nabla \sigma\|_{L^2}^{2}) \leq \frac{C}{{z^2(t,\omega)}}\|\nabla v^\varepsilon \|_{L^2}^{2} ( \| \nabla \rho\|_{L^2}^{2} + \|\nabla \sigma\|_{L^2}^{2})(\|\nabla \rho\|_{L^2}^{2}+\|\nabla \sigma\|_{L^2}^{2}). \end{align*} Let \begin{align*} h(t)=\frac{C}{{z^2(t,\omega)}}\|\nabla v^\varepsilon(t) \|_{L^2}^{2} ( \| \nabla \rho(t)\|_{L^2}^{2} + \|\nabla \sigma(t)\|_{L^2}^{2}). \end{align*} Applying Lemma \ref{lem1} on an arbitrary interval $[s,0] \subset [-1,0]$ again, we have \begin{align*} \|\nabla \rho(0)\|_{L^2}^{2}+\|\nabla \sigma(0)\|_{L^2}^{2} \leq e^{\int_{s}^{0} h(\xi)d\xi} (\|\nabla \rho(s)\|_{L^2}^{2}+\|\nabla \sigma(s)\|_{L^2}^{2}). \end{align*} Integrating $s$ on the interval $[-1,0]$ yields \begin{align*} \|\nabla \rho(0)\|_{L^2}^{2}+\|\nabla \sigma(0)\|_{L^2}^{2} \leq e^{\int_{-1}^{0} f(\xi)d\xi}\int_{-1}^{0}(\|\nabla \rho(s)\|_{L^2}^{2}ds+\|\nabla \sigma(s)\|_{L^2}^{2})ds. \end{align*} Lemma \ref{lem6} gives \begin{align*} \|\nabla \rho (t)\|_{L^2}^2+\|\nabla \sigma (t)\|_{L^2}^2 \leq A. \end{align*} From Lemma \ref{lem9}, we know that \begin{align*} \int_{-1}^{0}(\|\nabla v^\varepsilon(s)\|_{L^2}^{2}ds \leq R_3(\omega). \end{align*} Thus we estimate \begin{align*} \int_{-1}^{0} h(\xi)d\xi \leq CAR_3(\omega)\mathop {sup}\limits_{t \in [-1,0]} \frac{1}{z^2(t,\omega)}=r_6(\omega). \end{align*} Finally, let \begin{align*} R_5(\omega)=e^{r_6(\omega)}R_2, \end{align*} which proves \eqref{d14} and \eqref{d15}. Therefore, we can conclude that $B(0,R_4(\omega)+2{R_5(\omega)})$ is a random absorbing set in $V$. \end{proof} \end{lem} Now, applying Theorem \ref{thm1}, we are in a position to present the final conclusion in this section, that is, the existence of a compact random attractor in $H$. \begin{thm}\label{Theorem7} The random dynamical system associated with the stochastic Nernst-Planck-Navier-Stokes equations \eqref{b2} has a compact random attractor in $H$. \begin{proof} It follows from Lemma \ref{lem10} that there exists a random closed ball in $V$, which absorbs any deterministic bounded sets in $H$. Applying the compactness of embedding $V \hookrightarrow H$, it is clear to deduce that every bounded closed sets in $V$ are asymptotically compact in $H$. Hence, one can easily obtain the existence of a compact absorbing set. With the help of Theorem \ref{thm1}, the desired conclusion is proved. \end{proof} \end{thm} \section{\large Upper Semicontinuity of Random Attractors}\label{sec5} In this section, we consider the asymptotic behavior of random attractors for the system \eqref{b2} when $\varepsilon \rightarrow 0$, and establish upper semicontinuity of random attractors for the system \eqref{b2}. To begin with, we introduce the concepts about upper semicontinuity of random attractors in \cite{B.W2014}. \begin{thm}\label{Theorem10} Let X be a Banach space and $\varphi$ be a dynamical system defined on X. Given $\varepsilon>0$, we suppose that $\varphi_\varepsilon$ is a random dynamical system over a metric system $(\Omega, \mathcal{F}, P, (\theta_t)_{t \in R})$. Suppose the following three statements are satisfied:\par (1)for $P$-a.e.$~\omega \in \Omega$, $t \geq 0$, $\varepsilon_n \rightarrow 0$, $x_n$, $x \in X$, $x_n \rightarrow x$, there holds: \begin{align}\label{h1} \mathop {lim}\limits_{n \rightarrow \infty} \varphi_{\varepsilon_n}(t, \omega)x_n=\varphi(t)x. \end{align} \par (2)Let $\mathcal{D}$ be a collection of subsets of X. $\mathscr {A}_0$ is the global attractor of $\varphi$ in $X$. Assume that $\varphi_\varepsilon$ has a random attractor $\mathscr{A}_\varepsilon=\{\mathscr{A}_\varepsilon(\omega)\}_{\omega \in \Omega} \in \mathcal{D}$ and a random absorbing set $\mathcal{K}_\varepsilon=\{\mathcal{K}_\varepsilon(\omega)\}_{\omega \in \Omega} \in \mathcal{D}$ such that for some deterministic positive constant c and for $P$-a.e.$~\omega \in \Omega$, \begin{align}\label{h2} \mathop {lim}\limits_{\varepsilon \rightarrow 0} sup\|\mathcal{K}_\varepsilon(\omega)\|_X \leq c. \end{align} \par (3)there exists $\varepsilon_0 > 0$ such that for $P$-a.e.$~\omega \in \Omega$, \begin{align}\label{h3} \cup_{0<\varepsilon \leq \varepsilon_0} \mathscr{A}_\varepsilon(\omega) ~is ~precompact~ in~ X. \end{align} Then, $\mathscr{A}_\varepsilon$ is said to be upper semi-continuous, that is, for $P$-a.e.$~\omega \in \Omega$, \begin{align*} dist_X(\mathscr{A_\varepsilon}(\omega),\mathscr {A}_0) \rightarrow 0,~as~\varepsilon \rightarrow 0. \end{align*} Here, $dist_X(\cdot,\cdot)$ denotes the Hausdorff semi-distance given by \begin{align*} dist_X(A,B) =\mathop {sup}\limits_{a \in A}\mathop {inf}\limits_{b \in B}d(a,b), \end{align*} for nonempty sets $A,B \subset X$ on a metric space $(X,d)$. \end{thm} Then, the following Theorem \ref{Theorem11} will be presented to show that condition \eqref{h1} is fulfilled. \begin{thm}\label{Theorem11} For $0 < \varepsilon \leq 1$, let $u$ and $v^\varepsilon$ be the solutions of systems \eqref{a1} and \eqref{b2}, respectively. Then, for every $\omega \in \Omega$, $t > 0$, \begin{align*} \mathop {lim}\limits_{\varepsilon \rightarrow 0} \|v^\varepsilon(t,\omega,{v_0}^\varepsilon)-u(t,u_0)\|_{L^2}^2=0. \end{align*} \begin{proof} Let $y^\varepsilon=v^\varepsilon-u$. Then from \eqref{a1} and \eqref{b2}, we find \begin{align}\label{e1} \frac{dy^\varepsilon(t)}{dt}-\nu \Delta y^\varepsilon(t)=-\frac{1}{z(t,\omega)}B(v^\varepsilon(t))+B(u(t))+(1-z(t,\omega))\rho \nabla \phi+(z(t,\omega)-1)f. \end{align} Taking the $L^2$ inner product with $y^\varepsilon$ to the equation \eqref{e1}, we get \begin{align}\label{add1} \frac{1}{2}\frac{d}{dt}\|y^\varepsilon\|_{L^2}^2+\nu \|\nabla y^\varepsilon \|_{L^2}^2=&-e^{\varepsilon \omega(t)}b(v^\varepsilon,v^\varepsilon,y^\varepsilon)+b(u,u,y^\varepsilon)+(1-e^{\varepsilon \omega(t)})\langle \rho \nabla \phi,y^\varepsilon\rangle\notag \\&+(e^{\varepsilon \omega(t)}-1)\langle f,y^\varepsilon \rangle. \end{align} We can rewrite \eqref{add1} as \begin{align*} \frac{1}{2}\frac{d}{dt}\|y^\varepsilon\|_{L^2}^2+\nu \|\nabla y^\varepsilon \|_{L^2}^2=&-e^{\varepsilon \omega(t)}b(y^\varepsilon,u,y^\varepsilon)+(1-e^{\varepsilon \omega(t)})b(u,u,y^\varepsilon)+(1-e^{\varepsilon \omega(t)})\langle \rho \nabla \phi,y^\varepsilon\rangle\\&+(e^{\varepsilon \omega(t)}-1)\langle f,y^\varepsilon \rangle. \end{align*} Applying Holder's, Ladyzhenskaya’s and Young's inequalities, we estimate \begin{align*} |e^{\varepsilon \omega(t)}b(y^\varepsilon,u,y^\varepsilon)| &\leq Ce^{\varepsilon \omega(t)}\| y^\varepsilon\|_{L^2}\|\nabla y^\varepsilon\|_{L^2}\|\nabla u\|_{L^2}\\ &\leq \frac{\nu}{6}\|\nabla y^\varepsilon \|_{L^2}^2+Ce^{2\varepsilon \omega(t)}\| y^\varepsilon\|_{L^2}^2\|\nabla u\|_{L^2}^2, \end{align*} and \begin{align*} |(1-e^{\varepsilon \omega(t)})b(u,u,y^\varepsilon)| &\leq C|1-e^{\varepsilon \omega(t)}|\|u\|_{L^4} \|\nabla u\|_{L^2} \|y^\varepsilon\|_{L^4} \\&\leq C|1-e^{\varepsilon \omega(t)}| \|u\|_{L^2}^{\frac{1}{2}} \|\nabla u\|_{L^2}^{\frac{3}{2}} \|y^\varepsilon\|_{L^2}^{\frac{1}{2}} \|\nabla y^\varepsilon \|_{L^2}^{\frac{1}{2}} \\&\leq C\|\nabla v^\varepsilon\|_{L^2}^2\|y^\varepsilon\|_{L^2}^2+C\|\nabla u\|_{L^2}^2\|y^\varepsilon\|_{L^2}^2+C|1-e^{\varepsilon \omega(t)}|^{\frac{4}{3}}\| u\|_{L^2}^{\frac{2}{3}}\|\nabla u \|_{L^2}^2. \end{align*} Proceeding as above, we also estimate \begin{align*} |(1-e^{-\varepsilon \omega(t)})\langle \rho \nabla \Phi,y^\varepsilon \rangle| &\leq |1-e^{-\varepsilon \omega(t)}|\|\rho\|_{L^2}\|\nabla \Phi\|_{L^\infty}\|y^\varepsilon\|_{L^2} \\&\leq |1-e^{-\varepsilon \omega(t)}|\|\rho\|_{L^2}\|\rho \|_{L^4}\|y^\varepsilon\|_{L^2} \\&\leq \frac{3}{2\nu}|1-e^{-\varepsilon \omega(t)}|^2\|\rho\|_{L^3}^2 \| \rho\|_{L^2}^2 +\frac{\nu}{6}\|\nabla y^\varepsilon \|_{L^2}^2, \end{align*} and \begin{align*} |(e^{-\varepsilon \omega(t)}-1)\langle f,y^\varepsilon \rangle| &\leq \frac{3}{2\nu}|1-e^{-\varepsilon \omega(t)}|^2\|f\|_{L^2}^2 +\frac{\nu}{6}\|\nabla y^\varepsilon \|_{L^2}^2. \end{align*} Summing the above equalities up, we derive that \begin{align*} \frac{d}{dt} \|y^\varepsilon\|_{L^2}^2 \leq W_1(t) \|y^\varepsilon\|_{L^2}^2+W_2(t), \end{align*} where \begin{align*} W_1(t)=C[(e^{2\varepsilon \omega(t)}+1) \|\nabla u(t)\|_{L^2}^2+\|\nabla v^\varepsilon(t)\|_{L^2}^2], \end{align*} and \begin{align*} W_2(t)=C|1-e^{\varepsilon \omega(t)}|^{\frac{4}{3}}\| u\|_{L^2}^{\frac{2}{3}}\|\nabla u \|_{L^2}^2+\frac{3}{2\nu}|1-e^{-\varepsilon \omega(t)}|^2\|f\|_{L^2}^2+\frac{3}{2\nu}|1-e^{-\varepsilon \omega(t)}|^2\|\rho\|_{L^3}^2 \|\rho\|_{L^2}^2. \end{align*} Applying Lemma \ref{lem1} on the interval $[0,t]$ yields \begin{align*} \|v^\varepsilon(t,\omega,{v_0}^\varepsilon)-u(t,u_0)\|_{L^2}^2 \leq (\|e^{-\varepsilon \omega(t)}u_0-u_0\|_{L^2}^2+\int_{0}^{t} W_2(s)ds)e^{\int_{0}^{t} W_1(s)ds}. \end{align*} We deduce \begin{align*} \int_{0}^{t} W_1(s)ds \leq C\mathop {sup}\limits_{s \in [0,t]}(e^{2\varepsilon \omega(s)}+1) \int_{0}^{t} \|\nabla u(s)\|_{L^2}^2+\|\nabla v^\varepsilon(s)\|_{L^2}^2 ds < \infty. \end{align*} Similarly, we also deduce that \begin{align*} \int_{0}^{t} W_2(s)ds < \infty. \end{align*} Taking limit $\varepsilon \rightarrow 0$, then we obtain \begin{align*} \mathop {lim}\limits_{\varepsilon \rightarrow 0} \|v^\varepsilon(t,\omega,{v_0}^\varepsilon)-u(t,u_0)\|_{L^2}^2=0, \end{align*} which completes the proof. \end{proof} \end{thm} Finally, based on the above theorems, we are ready to establish upper semicontinuity of random attractors with respect to the system \eqref{b2}. \begin{thm}\label{Theorem12} For $0 < \varepsilon \leq 1$, let $\mathscr{A}(\omega)$ be the global random attractor of system \eqref{a1} and $\mathscr{A_\varepsilon}(\omega)$ be the random attractor of system \eqref{b2}. Then for every $\omega \in \Omega$, $t > 0$, \begin{align*} \mathop {lim}\limits_{\varepsilon \rightarrow 0} dist_H(\mathscr{A_\varepsilon}(\omega),\mathscr A)=0. \end{align*} \begin{proof} Define $\mathcal{K}_\varepsilon(\omega)$ as follows: \begin{align*} \mathcal{K}_\varepsilon(\omega)=\{u \in H:\|u\|_H^2 \leq \mathcal{M}_\varepsilon(\omega)\}, \end{align*} where $\mathcal{M}_\varepsilon(\omega)$ is given by \begin{align*} \mathcal{M}_\varepsilon(\omega)=z^{-2}(t,\omega)e^{\nu}[1+\frac{1}{\nu}(r_1(\omega)+r_2(\omega)+R_{f,\nu}(\omega))]. \end{align*} Then for every $0 \leq \varepsilon <1$, $\mathcal{K}_\varepsilon(\omega)$ is a closed absorbing set. For every $\omega \in \Omega$, we have \begin{align*} \mathop {lim}\limits_{\varepsilon \rightarrow 0}\|\mathcal{K}_\varepsilon(\omega)\|_H^2 \leq \mathop {lim}\limits_{\varepsilon \rightarrow 0}\mathcal{M}_\varepsilon(\omega) =\mathcal{M}_0(\omega). \end{align*} Then, condition \eqref{h2} is satisfied. By Lemma \ref{lem10}, $E_\varepsilon(\omega)$ can be defined as: \begin{align*} E_\varepsilon(\omega)=\{u \in H:\|u\|_V^2 \leq N_\varepsilon(\omega)\}, \end{align*} where $\mathcal{N}_\varepsilon(\omega)$ is denoted by \begin{align*} \mathcal{N}_\varepsilon(\omega)=R_4(\omega)+2R_5(\omega). \end{align*} Following from the invariance of the random attractor $\mathscr{A_\varepsilon}(\omega)$, we find \begin{align*} \cup_{0<\varepsilon \leq 1} \mathscr{A}_\varepsilon(\omega) \subset \cup_{0<\varepsilon \leq 1} E_\varepsilon(\omega), \end{align*} which implies that $\cup_{0<\varepsilon \leq \varepsilon_0} \mathscr{A}_\varepsilon(\omega)$ is bounded in $H^1$. Thanks to the compactness of embedding $H^1 \hookrightarrow L^2$, we conclude that $\cup_{0<\varepsilon \leq \varepsilon_0} \mathscr{A}_\varepsilon(\omega)$ is precompact in $H$. Therefore, by Theorem \ref{Theorem10} along with Theorem \ref{Theorem11}, the proof is completed. \end{proof} \end{thm} \bibliographystyle{plain} \begin{thebibliography}{9999} \bibitem{E.M2021} E. Abdo and M. Ignatova, Long time finite dimensionality in charged fluids, \emph{Nonlinearity}, {\bf 34}(2021), 6173-6209. \bibitem{E.M2022} E. Abdo and M. Ignatova, On the space analyticity of the Nernst-Planck-Navier-Stokes-system, \emph{J. Math. Fluid Mech.}, {\bf 24}(2022), 51. \bibitem{E.F.W2023} E. Abdo, F. Lee and W. Wang, On the global well-posedness and analyticity of some electrodiffusion models in ideal fluids and porous media, \emph{SIAM J. Math. Anal.}, {\bf 55}(2023), 6838-6866. \bibitem{P.M2024} E. Abdo and M. Ignatova, Long time dynamics of Nernst-Planck-Navier-Stokes systems, \emph{J. Differential Equations}, {\bf 379}(2024), 794-828. \bibitem{E.D2024} E. Abdo and D. Nikoli$\acute{\mathrm{c}}$, Long time dynamics of the three-dimensional Nernst-Planck-Darcy model, \emph{arxiv preprint arxiv:2408.07241}, 2024. \bibitem{L.F2020} L. Bianchi and F. Flandoli, Stochastic Navier-Stokes equations and related models, \emph{Milan J. Math.}, {\bf 88}(2020), 225-246. \bibitem{Z.M.F1993} Z. Brzeźniak, M. Capiński and F. Flandoli, Pathwise global attractors for stationary random dynamical systems, \emph{Prob. Th. Rel. Fields}, {\bf 95}(1993), 87-102. \bibitem{Z.B.M1998} Z. Brzeźniak, M. Capiński and F. Flandoli, A convergence result for stochastic partial differential equations, \emph{Stochastics}, {\bf 24}(1998), 423-445. \bibitem{Z.B.M2016} Z. Brzeźniak, F. Flandoli and M. Capiński, Existence and uniqueness for stochastic 2D Euler flows with bounded vorticity, \emph{Arch. Rational Mech. Anal.}, {\bf 221}(2016), 107-142. \bibitem{T.J.J1998} T. Caraballo, J. Langa and J. Robinson, Upper semicontinuity of attractors for small random perturbations of dynamical systems, \emph{Comm. Partial Differential Equations}, {\bf 23}(1998), 1557-1581. \bibitem{I2002} I. Chueshov, Monotone Random Systems Theory and Applications, \emph{Springer}, 2004. \bibitem{P.C1988} P. Constantin and C. Foiaş, Navier-stokes Equations, \emph{University of Chicago Press}, 1988. \bibitem{P.M2019} P. Constantin and M. Ignatova, On the Nernst-Planck-Navier-Stokes system, \emph{Arch. Rational Mech. Anal.}, {\bf 232}(2019), 1379-1428. \bibitem{P.M.F2021} P. Constantin, M. Ignatova and F. Lee, Nernst-Planck-Navier-Stokes systems far from equilibrium, \emph{Arch. Rational Mech. Anal}, {\bf 240}(2021), 1147-1168. \bibitem{H.F1994} H. Crauel and F. Flandoli, Attractors for random dynamical systems, \emph{Probab. Th. Re. Fields}, {\bf 100}(1994), 365-393. \bibitem{H.A.F1997} H. Crauel, A. Debussche and F. Flandoli, Random attractors, \emph{J. Dyn. Differ. Equ.}, {\bf 9}(1997), 307-341. \bibitem{Z.D.B.G2023} Z. Dong, B. Guo, J. Wu and G. Zhou, Global well-posedness and regularity of stochastic 3D Burgers equation with multiplicative noise, \emph{SIAM J. Math. Anal.}, {\bf 55}(2023), 1847-1882. \bibitem{F.B1996} F. Flandoli and B. Schmalfuss, Random attractors for the 3D stochastic Navier-Stokes equation with multiplicative white noise, \emph{Stoch. Stoch. Rep.}, {\bf 59}(1996), 21-45. \bibitem{H.K1986} H. Gajewski and K. Gröger, On the basic equations for carrier transport in semiconductors, \emph{J. Math. Anal. Appl.}, {\bf 113}(1986), 12-35. \bibitem{M.T2024} M. Hairer and T. Rosati, Global existence for perturbations of the 2D stochastic Navier–Stokes equations with space-time white noise, \emph{Ann. PDE}, {\bf 10}(2024), 3. \bibitem{M.P2021} M. Henandez and P. Nguyen, Global well‐posedness for the stochastic non-Newtonian fluid equations and convergence to the Navier‐Stokes equations, \emph{Math. Methods Appl. Sci.}, {\bf 44}(2021), 1252-1284. \bibitem{M.J2021} M. Ignatova and J. Shu, Global solutions of the Nernst-Planck-Euler equations, \emph{SIAM J. Math. Anal.}, {\bf 53}(2021), 5507-5547. \bibitem{M.J2022} M. Ignatova and J. Shu, Global smooth solutions of the Nernst-Planck-Darcy system, \emph{J. Math. Fluid Mech.}, {\bf 24}(2022), 26. \bibitem{G.J1989} G. Kasting and J. Keister, Application of electrodiffusion theory for a homogeneous membrane to iontophoretic transport through skin, \emph{J. Control. Release}, {\bf 8}(1989), 195-210. \bibitem{K.M2023} K. Kinra and M. Mohan, Existence and upper semicontinuity of random pullback attractors for 2D and 3D non-autonomous stochastic convective Brinkman-Forchheimer equations on whole space, \emph{Differ. Integr. Equ.}, {\bf 36}(2023), 367-412. \bibitem{A.J2016} A. Lee, J. Elam and S. Darling, Membrane materials for water purification: design, development, and application, \emph{Environ. Sci. Water Res. Technol.}, {\bf 2}(2016), 17-42. \bibitem{H.J2018} H. Lee, J. Kim, J. Yang, S. Seo and S. Kim, Diffusiophoretic exclusion of colloidal particles for continuous water purification, \emph{Lab Chip}, {\bf 18}(2018), 1713-1724. \bibitem{Lee.2022} F. Lee, Global regularity for Nernst-Planck-Navier-Stokes systems with mixed boundary conditions, \emph{Nonlinearity}, {\bf 36}(2022), 255-286. \bibitem{R.B2004} R. Mikulevicius and B. Rozovskii, Stochastic Navier-Stokes equations for turbulent flows, \emph{SIAM J. Math. Anal.}, {\bf 35}(2004), 1250-1310. \bibitem{S.T2010} S. Mohammed and T. Zhang, Dynamics of stochastic 2D Navier-Stokes equations, \emph{J. Funct. Anal.}, {\bf 258}(2010), 3543-3591. \bibitem{N.T1989} N. Qian and T. Sejnowsk, An electro-diffusion model for computing membrane potentials and ionic concentrations in branching dendrites, spines and axons, \emph{Biol. Cybern.}, {\bf 62}(1989), 1-15. \bibitem{I.R1990} I. Rubinstein, Electro-diffusion of ions, \emph{SIAM Studies in Applied Mathematics}, 1990. \bibitem{R.T1986} R. Temam, Infinite-dimensional dynamical systems, \emph{Nonlinear Funct. Anal. Appl.}, 1986. \bibitem{R.T2012} R. Temam, Infinite-dimensional dynamical systems in mechanics and physics, \emph{Springer}, 2012. \bibitem{BX.W2014} B. Wang, Random attractors for non-autonomous stochastic wave equations with multiplicative noise, \emph{Discrete Continuous Dyn. Syst.}, {\bf 34}(2014), 269-300. \bibitem{B.W2014} B. Wang, Existence and upper semicontinuity of attractors for stochastic equations with deterministic non-autonomous terms, \emph{Stoch. Dyn.}, {\bf 14}(2014), 1-31. \bibitem{D.Z2022} D. Zhang, Strichartz and local smoothing estimates for stochastic dispersive equations with linear multiplicative noise, \emph{SIAM J. Math. Anal.}, {\bf 54}(2022), 5981-6017. \bibitem{R.Z2017} R. Zhu and X. Zhu, Random attractor associated with the quasi-geostrophic equation, \emph{J. Dyn. Differ. Equ.}, {\bf 29}(2017), 289-322. \end{thebibliography} \end{document}
2412.05513v1
http://arxiv.org/abs/2412.05513v1
Distributional Solution and Spectral Shift Function of Heun Differential Equation
\documentclass[10pt,reqno,titlepage]{amsart} \usepackage{graphicx} \baselineskip=16pt \usepackage{footmisc} \usepackage{indentfirst,csquotes} \bibliographystyle{amsplain} \topmargin= .5cm \textheight= 20cm \textwidth= 32cc \baselineskip=16pt \evensidemargin= .9cm \oddsidemargin= .9cm \usepackage{amssymb,amsthm,amsmath,mathrsfs,scalerel,stackengine} \usepackage{xcolor, hyperref, fancyhdr, etoolbox, paralist} \usepackage{lipsum} \theoremstyle{definition} \newtheorem{teo}{Theorem} \newtheorem{defn}[teo]{Definition} \newtheorem{lem}[teo]{Lemma} \newtheorem{prop}[teo]{Proposition} \newtheorem{exa}[teo]{Example} \newtheorem{cor}[teo]{Corollary} \newtheorem{rem}[teo]{Remark} \newcounter{mybibstartvalue} \setcounter{mybibstartvalue}{9} \newcommand{\tabs}{$\quad$} \newcommand{\vd}{\mathrm{d}} \newcommand{\lap}{$\mathcal{L}$} \newcommand{\laps}{\mathcal{L}} \newcommand{\set}[1]{$\{#1\}$} \newcommand{\lrb}[1]{\left({#1}\right)} \newcommand{\lrs}[1]{\left[{#1}\right]} \newcommand{\lrbb}[1]{\left\{{#1}\right\}} \newcommand{\deri}[2]{\frac{\vd{#1}}{\vd{#2}}} \newcommand{\deris}[2]{\frac{\vd^2{#1}}{\vd{#2}^2}} \makeatletter \newcommand{\newsectionstyle}{ \renewcommand{\@secnumfont}{\bfseries} \renewcommand\section{\@startsection{section}{2} \z@{.5\linespacing\@plus.7\linespacing}{-.5em} {\normalfont\bfseries}}} \let\oldsection\section\let\old@secnumfont\@secnumfont\newcommand{\originalsectionstyle}{ \let\@secnumfont\old@secnumfont \let\section\oldsection } \makeatother \hypersetup{ colorlinks=true, linkcolor=black, filecolor=black, urlcolor=black } \def\proof{\noindent {\it Proof. $\, $}} \def\endproof{\hfill $\Box$ \vskip 5 pt } \usepackage{lipsum} \def\quotient#1#2{ \raise1ex\hbox{$#1$}\Big/\lower1ex\hbox{$#2$}} \stackMath \newcommand\reallywidecheck[1]{\savestack{\tmpbox}{\stretchto{ \scaleto{ \scalerel*[\widthof{\ensuremath{#1}}]{\kern-.6pt\bigwedge\kern-.6pt} {\rule[-\textheight/2]{1ex}{\textheight}} }{\textheight}}{0.5ex}}\stackon[1pt]{#1}{\scalebox{-1}{\tmpbox}}} \addtolength{\itemsep}{0.5\baselineskip} \renewenvironment{titlepage}{ \thispagestyle{empty}\setcounter{page}{0} \centering ll} }{ \vspace{3\baselineskip} ll} \newpage } \makeatletter \patchcmd{\@maketitle} {\if@titlepage \newpage \else} {\if@titlepage \vspace{\baselineskip} \else} {}{} \makeatother \title{Distributional Solution and Spectral Shift Function of Heun Differential Equation} \author{Ubong Sam IDIONG} \begin{document} \begin{abstract} In this work, the Heun operator is written as an element in the universal enveloping algebra of the Lie algebra $\mathscr{G}=\mathscr{L}(G)$ of the Lie group $G=SL(2,\mathbb{C})$. The Green function and the spectral shift function of the exactly solvable Heun operator from the resulting Lie algebraic equation are obtained via Fourier transform over $G$. \end{abstract} \maketitle \let\thefootnote\relax \footnote{MSC2020: Primary 34M46, Secondary 34B27, 16S30.} \footnote{Keywords: Distributional Solutions, Green function, Spectral Shift Function, Universal enveloping algebra} \date{\today} \section{Introduction} Heun differential equation (HDE) is a second-order linear ordinary differential equation in the complex domain defined by \begin{equation}\label{qhd} z(z-1)(z-a)\Psi''(z)+[\gamma(z-1)(z-a)+\delta z(z-a)+ \varepsilon z(z-1)]\Psi'(z)+(\alpha\beta z-q)\Psi(z)=0, \end{equation} such that $\alpha,\beta,\gamma,\delta,\varepsilon$ satisfy the constraint equation \begin{equation}\label{cstr} \alpha+\beta+1=\gamma+\delta+\varepsilon. \end{equation} (see ~\cite{RA1}, \S(4.2.3), p. 47). Here, $a\in\mathbb{C}\setminus \{0,1\}$ and $q\in\mathbb{C}$ is the accessory parameter, which plays the role of spectral parameter in most applications. This equation has regular singularities $0,1, a,\infty$ with corresponding exponents $\{0, 1-\gamma\},\{1,1-\delta\}, \{a, 1-\varepsilon\}$ and $\{\infty, \alpha\beta\}$ respectively. The search for solutions to HDE has attracted much research interest. The HDE, using the formula, $2^{n-1}n!,$ is known to have 192 solutions, which corresponds with the number of regular singularities $n=4$ (\cite{MRS}:~811). The concept of distributional solutions of Fuchsian equations has been studied in \cite{LJK, ERK1, ERK2}. These studies have been limited to studying ordinary differential equations with three regular singularities $0,1$ and $\infty$ using Laplace transform. This paper extends the study to operators with four regular singularities, $0,1, a,\infty$ of which the Heun equation is typical. A grasp of Fuchsian differential operators' spectral shift functions (SSFs) is necessary to comprehend their spectrum properties, particularly how they vary in response to disturbances on the Reimann surfaces. In quantum mechanics, mathematical physics, and inverse spectrum problems, it is essential to extend the SSFs which are being studied on the real line to the projective complex line. A thorough analysis of these functions improves the mathematical context and produces robust tools. It also illustrates the stability properties of solutions to differential equations under small disturbances. Research in this area may lead to novel techniques and mathematical theorems. Furthermore, the literature on applications of SSFs to Fuchsian-type equations is scarce. This is because SSFs are difficult to study on the Riemann sphere. In studying SSFs of Fuchsian equations, the computation of Green functions is inevitable. Our motivation and main objective in this paper include computing the Green kernel of the exactly solvable HDE, and by way of application, examine the compactness of its associated Green integral relation and its SSF. In its canonical form, the Heun operator given by $$H_c=\frac{\vd^2}{\vd z^2}+\left(\frac{\gamma}{z}+\frac{\delta}{z-1}+\frac{\varepsilon}{z-a}\right)\frac{\vd}{\vd z}+\frac{\alpha\beta z-q}{z(z-1)(z-a)}.$$ All other Fuchsian equations having four regular singularities in the extended complex plane $\mathbb{CP}^{1}=\mathbb{C}\cup\{\infty\}$ can be transformed into \eqref{qhd} (\cite{OFW}, \S 31.2:~711). For our interest, equation~\eqref{qhd} can be re-written in the form \begin{equation}\label{qhd1} (z^{3}-(1+a)z^{2}+az)\Psi''(z)+[(\gamma+\delta+ \varepsilon)z^{2}-((1+a)\gamma+\delta+\varepsilon)z+\gamma a]\Psi'(z) +(\alpha\beta z-q)\Psi(z)=0. \end{equation} The outline of follows: Section~\ref{prel} shall consist of mathematical preliminaries leading to Lie algebraization of the Heun differential equation and eigenvalue of an exactly solvable Heun differential equation. In Section \ref{GrHu}, we present the main results in this paper, which include: the distributional solution of the HDE is also evaluated; Green function of the equation; the SSF of Heun operator; and finally, and compactness integral relation is also determined. In the last Section, vital conclusions are drawn on the work. \section{Prelimnaries}\label{prel} In this section, a preliminary result which transforms the Heun operator as an element in the universal enveloping algebra $U(\mathscr{G})$ of the Lie algebra $\mathscr{G}=\mathscr{L}(G)$ of the Lie Group $G=SL(2,\mathbb{C})$ is given without proof. $U(\mathscr{G})$ is also a Lie algebra and can be a finite-dimensional algebra. Elements of $U(\mathscr{G})$ are called invariant differential operators acting on the Lie group $G.$ Standard reference materials on UEA include ~\cite{DJ} and (\cite{KJA}, \S4.8:85-86) among others. The Lie algebra under consideration here is the three-dimensional Lie algebra $sl(2,\mathbb{C})$ (cf: \cite{MIL}, \S 1, Eq.(1.61), p. 20) which is spanned by first-order differential operators in complex variable $z$: \begin{equation}\label{Lgen} J_{+}:=z^{2}\frac{\vd}{\vd z}-2jz,\;\;\; J_{0}:=z\frac{\vd}{\vd z}-j,\;\;\; J_{-}:=\frac{\vd}{\vd z} \end{equation} where $j=\frac{n}{2}, n\in\mathbb{Z}.$ Thus, the operators \eqref{Lgen} satisfy the commutation relations, \begin{equation}\label{qalg} [J_{+},J_{-}]=2J_{0},\;[J_{0},J_{+}]=J_{+} , \; [J_{0},J_{-}]=-J_{-}. \end{equation} which is isomorphic to the matrix generators of the Lie algebra $sl(2,\mathbb{C}).$ In what follows, we present some preliminary results on the transformations of Heun differential operator to an element of the UEA of $SL(2,\mathbb{C})$ and its gauge transformation. \begin{teo} The Heun differential expression in ~\eqref{qhd1} as an element of universal enveloping algebra of $SL(2,\mathbb{C})$ can be written in terms of the generators of the Lie algebra $sl(2,\mathbb{C})$ as \begin{eqnarray*} -H\psi &:=&\bigg\lbrace\frac{1}{2}[ J_{+}J_{0}+J_{0}J_{+}]-\frac{(1+a)}{2}[J_{+}J_{-}+J_{-}J_{+}]+\frac{a}{2}[ J_{0}J_{-}+J_{-}J_{0}]\\ &&+[\gamma+\delta+\varepsilon +\frac{3}{2}(2j-1)]J_{+}+[(2j-1-\gamma)(1+a)-\delta-\varepsilon]J_{0}\\ &&+a[\gamma-\frac{(2j-1)}{2}]J_{-} +j[(2(1-j)+\gamma)(1+a)+\delta+\varepsilon]-q\bigg\rbrace\psi . \end{eqnarray*} provided that $8j^{2}+2j(\alpha+\beta-1)+\alpha\beta=0$ and its explicit form depending on the spin number $j$ becomes $$-H_{j}=z(z-1)(z-a)D^{2}+[\rho_{j} z^{2} +\sigma_{j} z+\tau_{j}]D+\alpha_{j}\beta_{j} z, \;\; (D=\vd/\vd z)$$ with \begin{eqnarray*} \rho_{j} &=& \gamma +\delta +\varepsilon=\alpha+\beta+1;\\ \sigma_{j} &=& [2(2j-1)-\gamma] (a+1) -\delta - \varepsilon;\\ \tau_{j} &=& a(\gamma -2j+1));\\ \alpha_{j}\beta_{j} &=&-2j(2j+\alpha+\beta);\;\;\mathrm{and} \\ q_{j}&=& j[\big(2(1-j)+\gamma\big)(1+a)-\delta-\varepsilon]. \end{eqnarray*} \end{teo} The condition for the Heun operator $H$ to be exactly solvable is that the coefficient of the term $J_{+}$ in positive grading must be equal to zero. \begin{cor} Exactly solvable Heun Hamiltonian is given by \begin{eqnarray*} - H_{e}&=&\frac{1}{2}[ J_{+}J_{0}+J_{0}J_{+}]-\frac{(1+a)}{2}[J_{+}J_{-}+J_{-}J_{+}]+\frac{a}{2}[ J_{0}J_{-}+J_{-}J_{0}]\\ &&+[(2j-1-\gamma)(1+a)-\delta-\varepsilon]J_{0}+a[\gamma-\frac{(2j-1)}{2}]J_{-}\\ &&+j[(2(1-j)+\gamma)(1+a)+\delta+\varepsilon]-q. \end{eqnarray*} and setting $j=\frac{n}{2}$, in the expanded form in terms the product of generators $J_{+},J_{0},J_{-}$ in \eqref{Lgen} is given by \begin{multline*} -H_{\frac{n}{2},e}=z(z-1)(z-a)D^{2}+[\rho_{\frac{n}{2}} z^{2} +\sigma_{\frac{n}{2}} z+\tau_{\frac{n}{2}}]D+\alpha_{\frac{n}{2}}\beta_{\frac{n}{2}} z, \;\; (D=\vd/\vd z) \end{multline*} with \begin{eqnarray*} \rho_{\frac{n}{2}} &=& \frac{3(1-n)}{2};\\ \sigma_{\frac{n}{2}} &=&( n-1-\gamma)(1+a)-\delta-\varepsilon];\\ \tau_{\frac{n}{2}} &=& a(\gamma- \frac{n-1}{2});\\ \alpha_{\frac{n}{2}}\beta_{\frac{n}{2}} &=& \frac{n(n-1)}{2};\;\;\mathrm{and} \\ q_{\frac{n}{2}}&=& -\frac{n}{2}\left(2-n+\gamma\right)(1+a)-\delta-\varepsilon). \end{eqnarray*} and its associated eigenvalue $$E_{\frac{n}{2},e}=q_{\frac{n}{2}}+\Xi_{\frac{n}{2}}=n[(2-n+\gamma)(a+1)+\delta+\varepsilon]-q.$$ \end{cor} \begin{proof} By using the constraint equation~\eqref{cstr} and coefficient of $J_{+}$ \begin{eqnarray} \gamma+\delta+\varepsilon+\frac{3}{2}(2j-1) &=& \alpha+\beta+1+\frac{3}{2}(2j-1)\nonumber \\ &=& \alpha+\beta+3j-\frac{1}{2}.\label{aggx} \end{eqnarray} so that \begin{eqnarray}\label{qhd4} -H&=&\frac{1}{2}[ J_{+}J_{0}+J_{0}J_{+}]-\frac{(1+a)}{2}[J_{+}J_{-}+J_{-}J_{+}]+\frac{a}{2}[ J_{0}J_{-}+J_{-}J_{0}]\nonumber\\ &&+[\gamma+\delta+\varepsilon +\frac{3}{2}(2j-1)]J_{+}+[(2j-1-\gamma)(1+a)-\delta-\varepsilon]J_{0}\nonumber\\ &&+a[\gamma-\frac{(2j-1)}{2}]J_{-} +j[(2(1-j)+\gamma)(1+a)+\delta+\varepsilon]-q. \end{eqnarray} By setting $\alpha+\beta+3j-\frac{1}{2}=0$, the QES Heun operator in equation~\eqref{qhd4} reduces to an exactly solvable Heun Hamiltonian $H_e$ given by \begin{eqnarray}\label{He} - H_{e}&=&\frac{1}{2}[ J_{+}J_{0}+J_{0}J_{+}]-\frac{(1+a)}{2}[J_{+}J_{-}+J_{-}J_{+}]+\frac{a}{2}[ J_{0}J_{-}+J_{-}J_{0}]\nonumber\\ &&+[(2j-1-\gamma)(1+a)-\delta-\varepsilon]J_{0}+a[\gamma-\frac{(2j-1)}{2}]J_{-}\nonumber\\ &&+j[(2(1-j)+\gamma)(1+a)+\delta+\varepsilon]-q. \end{eqnarray} Now, setting $j=\frac{n}{2}$, \eqref{He} can be rewritten in the differential expression below as by expanding \eqref{He} in terms the product of generators $J_{+},J_{0},J_{-}$ in \eqref{Lgen} to get \begin{multline}\label{qham1} -H_{\frac{n}{2},e}=z(z-1)(z-a)D^{2}+[\rho_{\frac{n}{2}} z^{2} +\sigma_{\frac{n}{2}} z+\tau_{\frac{n}{2}}]D+\alpha_{\frac{n}{2}}\beta_{\frac{n}{2}} z, \;\; (D=\vd/\vd z) \end{multline} with \begin{eqnarray*} \rho_{\frac{n}{2}} &=& \frac{3(1-n)}{2};\\ \sigma_{\frac{n}{2}} &=&( n-1-\gamma)(1+a)-\delta-\varepsilon;\\ \tau_{\frac{n}{2}} &=& a(\gamma- \frac{n-1}{2});\\ \alpha_{\frac{n}{2}}\beta_{\frac{n}{2}} &=& \frac{n(n-1)}{2};\;\;\mathrm{and} \\ q_{\frac{n}{2}}&=& -\frac{n}{2}\left[(n-\gamma)(1+a)-\delta-\varepsilon\right]. \end{eqnarray*} The eigenvalue for the exactly solvable operator $-H_{\frac{n}{2},e}$ is $$E_{\frac{n}{2},e}=q_{\frac{n}{2}}+\Xi_{\frac{n}{2}}=n[(n-\gamma)(a+1)-\delta-\varepsilon]-q,\;\;n\in\mathbb{Z}.$$ \end{proof} The group $G=SL(2,\mathbb{C})$ acts on the complex projective space $\mathbb{CP}^{1}=P(\mathbb{C}^{2})$ by linear fractional (M\"{o}bius) transformation \begin{equation}\label{mob} \zeta\mapsto g\cdot \zeta = \frac{a\zeta +b}{c\zeta +d},\;\; g=\left( \begin{array}{cc} a & b \\ c & d \\ \end{array} \right),\;\; \det(g) =ad-bc=1. \end{equation} The finite-dimensional irreducible representation $\pi_{n,p}(g)$ of $SL(2,\mathbb{C})$ associated with above action in \eqref{mob} parameterized by pairs of non-negative integers $n,p$ may be described as follows. Let $$\mathcal{P}^{n,p}:=\left\{P(z_1,z_2)| P(\alpha z_1,\alpha z_2)=\alpha^{n}\bar{\alpha}^{p}P(z_1,z_2),\;\;\alpha\in\mathbb{C}^{\times}\right\}$$ be the space of homogeneous polynomials of degrees $n$ and $p$ in the variables $z_1$ and $z_2$. The left regular representation of $SL(2,\mathbb{C})$ on $\mathcal{P}^{n,p}$ is given by $$\pi_{n,p}(g)P(z_{1},z_{2})=P\left(g^{-1}\cdot \binom{z_{1}}{z_{2}}\right),\;\;\;\; g\in SL(2,\mathbb{C}).$$ (cf:~\cite{RW}, \S3-4:~54). The class of holomorphic irreducible representations of $SL(2,\mathbb{C})$ consists of representations of $\pi_{n}(g)$ ($\equiv \pi_{n,0}(g)$) which acts on the space $\mathcal{P}^{n}$ of polynomials $\displaystyle P(z_{1},z_{2})=\sum_{k=0}^{n}c_{k}z_{1}^{k}z_{2}^{k}.$ For our purpose, we consider the following realization of $\mathcal{P}^{n,p}$ with the associated function $\widetilde{\psi}$ given by $$P(z_1,z_2)=z_{1}^{n}\overline{z}_{1}^{p}\widetilde{\psi}(\frac{z_2}{z_1}).$$ Now put $z_1=1, z_2=\zeta.$ and define the function $\widetilde{\psi}$ as $\widetilde{\psi}(\zeta)=P(1,\zeta).$ Then, the left group action \begin{equation}\label{mob2} g^{-1}\cdot \zeta = g=\left(\begin{array}{cc} a & -c \\ -b& d \\ \end{array} \right)\cdot \zeta=\left(\frac{a\zeta-c}{-b\zeta+d}\right),\;\; \zeta\in\mathbb{C} \end{equation} gives the one dimensional representation of $SL(2,\mathbb{C})$ on the space of polynomials $\mathcal{P}^{n}$ as $$\pi_{n}(g)\widetilde{\psi}(\zeta)= (-b\zeta+d)^{n}\widetilde{\psi}\left(\frac{a\zeta-c}{-b\zeta+d}\right).$$ \begin{teo}[\cite{VVS}:~72] Let $\mathbf{B}$ be the subgroup of matrices of $G$ in the form $\left(\begin{array}{cc} a & b \\ 0 & a^{-1} \end{array}\right).$ $G$ acts on $\mathbb{CP}^1=\mathbb{C}\cup\{\infty\}$ by the M\"{o}bius transformation $$m(g):~\zeta\mapsto \frac{a\zeta+b}{c\zeta+d}=m(g)(\zeta),\;\;g=\left(\begin{array}{cc} a & b \\ c & d \end{array}\right).$$ The group action $\zeta\mapsto g[\zeta]$ is given by $$g[\zeta]=m((g^{-1})^{t})(\zeta)=\frac{d\zeta-c}{-b\zeta+a}.$$ For this action the stabilizer of $0$ is $\mathbf{B}$ so that $G/\mathbf{B}\cong \mathbb{CP}^{1}.$ We have a section defined on $\mathbb{C}$ given by $$s(z)=\left(\begin{array}{cc} 1 & 1 \\ -z & 1 \end{array}\right),\;\;s(z)[0]=z.$$ \end{teo} \begin{proof} Since $g\in \mathbf{B}\subset G=SL(2,\mathbb{C}), \det(g)=ad-bc=1$ and $$g^{-1}=\frac{1}{ad-bc}\left(\begin{array}{cc} d & -b \\ -c & a \end{array}\right)=\left(\begin{array}{cc} d & -b \\ -c & a \end{array}\right)$$ and $$(g^{-1})^{\top}=\left(\begin{array}{cc} d & -c \\ -b & a \end{array}\right).$$ Thus by Mobi\"{u}s transformation $$g[\zeta]=m((g^{-1})^{\top})=\frac{d\zeta-c}{-b\zeta+a}.$$ Let $g$ be a $\mathbb{C}$-segment \[s(z)=\left(\begin{array}{cc} 1 & 1 \\ -z & 1 \\ \end{array}\right),\] then \[s(z)[\zeta]=\frac{\zeta+z}{-\zeta+1}=\frac{\zeta+z}{1-\zeta}\] whence $s(z)[0]=z$ and $\vd s(z)[0]=\vd z.$ Therefore, the stabilizer of $0$ is $\mathbf{B}$ as required. \end{proof} \begin{rem} The measure $\vd x\vd y=\dfrac{i}{2}\vd z\vd \overline{z}$ is quasi-invariant under the action of $G$ since $z'=\dfrac{az+b}{cz+d}$ gives $\vd z'=(cz+d)^{-2}\vd z$ and thus $$\vd x'\vd y'=|cz+d|^{-4}\vd x\vd y=\frac{i}{2}|cz+d|^{-4}\vd z\vd \overline{z}=:\vd g[z].$$ The quasi-invariant measure $\vd g[z]$ is required when handling differential expressions involving the Laplacian $\Delta:=\dfrac{\partial^{2}}{\partial x^{2}}+\dfrac{\partial^{2}}{\partial y^{2}}= 4\dfrac{\partial}{\partial z}\dfrac{\partial}{\partial \overline{z}}$ (see~\cite{WR}, \S11.3, p.223) but the Heun operator in this case does not involve $\Delta.$ However, considering the Borel set $\left( \begin{array}{cc} a & b \\ 0 & a^{-1} \\ \end{array} \right)\in \mathbf{B}\subset SL(2,\mathbb{C}), c=0,d=a^{-1}$ so that $\vd z'=(a^{-1})^{-2}\vd z=a^{2}\vd z,a=\varsigma_{n}^{k}=\exp\left(i\dfrac{2\pi k }{n}\right)\in \mathbb{C}^{\times}, k=0,\ldots,n-1, n\in\mathbb{Z}.$ The special case, suitable for defining the measure required for defining the Fourier transform will require that $n=2$ for which $a=\pm 1$ so that $\vd z'=\vd z$. It is also known (~\cite{MCM}, p.4) that some other subsets of $SL(2,\mathbb{C})$ up to conjugacy include $SL(2,\mathbb{C})$, compact subgroup $K=SU(2)$,unipotent matrices $N=\mathbb{C}$ and diagonal matrices $A=\mathbb{C}^{\times}.$ It is also good to note here that $\mathbf{B}=AN.$ The choice of the measure on the Borel set $\mathbf{B}$ here is intuitive for the purpose of defining a suitable Fourier transform for obtaining a distributional solution of the Lie algebraic operator and its Green function. \end{rem} \section{Main Results}\label{GrHu} In what follows, we present the distributional solution associated with Lie algebraic HDO, $H_{j}.$ \begin{teo} Consider the Lie algebraic HDO, $H_{j}\in U_{sl(2)}$. The a distributional solution $\Psi(z)$ associated to $H_{j}$ is given by $$\Psi(z)=\sum_{k=0}^{\infty}c_{k}\delta^{(k)}(z).$$ provided that $c_{k}$ has either of these values $$c_{k}=A(\varepsilon_{k,j}^{(1)}+\varepsilon_{k,j}^{(2)})^k+B(\varepsilon_{k,j}^{(1)}-\varepsilon_{k,j}^{(2)})^k$$ or $$c_{k}=A[\eta_{k,j}^{(1)}+\eta_{k,j}^{(2)}]^{k}+B[\eta_{k,j}^{(1)}-\eta_{k,j}^{(2)}]^{k},$$ where, \begin{eqnarray*} \varepsilon_{k,j}^{(1)} &=& \frac{\rho_j(k+1)_{l_j+1}-\tau_j(k+1)_{l_j-1}}{2\alpha_j\beta_j(k)_{l_j}}, \\ \varepsilon_{k,j}^{(2)} &=& \frac{\sqrt{\left(\rho_j(k+1)_{l_j+1}-\tau_j(k+1)_{l_j-1}\right)^2-4\alpha_j\beta_j(k)_{l_j}\big[(k+2)_{l_j+2}-a(k+2)_{l_j}\big]}}{2\alpha_j\beta_j(k)_{l_j}},\\ \eta_{k,j}^{(1)} &=& -\frac{(k+1)_{l_j}\sigma_j}{E_j(k)_{l_j-1}}, \\ \eta_{k,j}^{(2)} &=& \frac{\sqrt{((k+1)_{l_j}\sigma_j)^2-4( E_j(k)_{l_j-1})([(1+a)(k+2)_{l_j+1}-a(k+2)_{l_j}])}}{E_j(k)_{l_j-1}}. \end{eqnarray*} Provided in each case, respectively, $k\geq l_{j}$ and $k\geq l_{j-1}.$ \end{teo} \begin{proof} Let $\Psi(z)$ be the distributional solution of the Lie-algebraic Heun-type equation \begin{equation}\label{qhf1} (H_{j}-E_{j})\Psi(z)=0. \end{equation} Let $\Gamma:=\mathbb{CP}^1\setminus\{0,1,a,\infty\}$ be an open subset of the set of complex numbers, $\mathbb{C}$. Consider the Hilbert space $\mathfrak{H}=L^2(\Gamma,\vd\mu_{\omega}(z))$ and its Fourier transformed space $\widetilde{\mathfrak{H}}=L^2(\widetilde{\Gamma},\vd\mu_{\omega} (u)), \widetilde{\Gamma}\subset\widehat{\mathbb{C}}$. Let the character of $\mathbb{C}$ be given as $$\chi_{u}(z)=e^{iz\cdot u},\;\;z,u\in\mathbb{C}.$$ The Fourier transform (isometry) $\mathscr{F}:~\mathfrak{H}\rightarrow\widetilde{\mathfrak{H}}$ is defined by $$\langle f,\chi_{u}(z)\rangle_{\omega}=\mathscr{F}[f(u)]=\frac{1}{4\pi^{2}}\int_{\Gamma}f(z)\chi_{-u}(z)\,\omega(z)\vd z,\;\;\Gamma\subset \mathbb{C}$$ (Some references use $-iz\cdot u$ instead of $iz\cdot u,\;i=\sqrt{-1},z\in\mathbb{C}, \widehat{\mathbb{C}}$ is the dual of $\mathbb{C}$). The Fourier scaling factor here is $4\pi^2$ because $\mathbb{C}\cong \mathbb{R}^{2}$. Here, $\widehat{\mathbb{C}}$ is defined as $$\widehat{\mathbb{C}}:=\{1,i,u, iu|u^2=0=i^2u^2, iu\cdot u=0=u\cdot iu\}.$$ In what follows in this section, a basic assumption that $\sigma_j,\tau_j, (a\neq 0,1)\in\mathbb{Z}$ is required. The weight function associated with $H_j$ is given by $$\omega(z)=z^{\rho_j-1}(z-1)^{\sigma_j-1}(z-a)^{\tau_j-1}=\sum_{m=0}^{\sigma_j-1}\sum_{n=0}^{\tau_j-1}h_{mn}^{j}z^{m+n+\rho_j-1}$$ where $$h_{mn}^{j}=\binom{\sigma_j-1}{m}\binom{\tau_j-1}{n}(-1)^{\sigma_j+\tau_j+m+n}a^{\tau_j-n-1}.$$ The domain of $H_j$ denoted by $Dom(H_j)=\mathscr{C}_{c}^{\infty}(\Gamma)\subset \mathfrak{H}\subset \mathscr{D}'(\Gamma)$ where $\mathscr{D}'(\Gamma)$ is the space of distributions on $\Gamma$. This implies that $H_j$ admits distributional solutions, say $\displaystyle\Psi(z)=\sum_{k=0}^{\infty}c_{k}\delta^{(k)}(z), \;\; ^{(k)}=\frac{\vd^{k}}{\vd z^{k}}$. To obtain the values of $a_k$, one sets $$\big\langle(H_j-E_j)\Psi(z),\chi_{u}(z)\big\rangle_\omega=0.$$ This yields \begin{multline}\label{dsn} \sum_{k=0}^{\infty}\sum_{m=0}^{\sigma_j-1}\sum_{n=0}^{\tau_j-1} c_k h_{mn}^{j}\bigg[\bigg\{\left(-i\frac{\vd}{\vd u}\right)^{m+n+\rho_j+2} -(1+a)\left(-i\frac{\vd}{\vd u}\right)^{m+n+\rho_j+1}\\ +a\left(-i\frac{\vd}{\vd u}\right)^{m+n+\rho_j}\bigg\}(-iu)^2 +\bigg\{\rho_j\left(-i\frac{\vd}{\vd u}\right)^{m+n+\rho_j+1}+\sigma_j\left(-i\frac{\vd}{\vd u}\right)^{m+n+\rho_j}\\ +\tau_j\bigg(-i\frac{\vd}{\vd u}\bigg)^{m+n+\rho_j-1}\bigg\}(-iu) +\alpha_j\beta_j\left(-i\frac{\vd}{\vd u}\right)^{m+n+\rho_j}-E_j\left(-i\frac{\vd}{\vd u}\right)^{m+n+\rho_j-1}\bigg](-iu)^{k}=0. \end{multline} Equation~\eqref{dsn} becomes \begin{eqnarray}\label{dsn1-1} && \sum_{k=0}^{\infty}\sum_{m=0}^{\sigma_j-1}\sum_{n=0}^{\tau_j-1} c_k h_{mn}^{j}(-i)^{m+n+\rho_j+k}\bigg[\bigg\{(-i)^{4}\left(\frac{\vd}{\vd u}\right)^{m+n+\rho_j+2}u^{k+2}\nonumber\\ && -(1+a)(-i)^3\left(\frac{\vd}{\vd u}\right)^{m+n+\rho_j+1}u^{k+2} +(-i)^{2} a\left(\frac{\vd}{\vd u}\right)^{m+n+\rho_j}u^{k+2}\bigg\}\nonumber\\ &&+\bigg\{\rho_j(-i)^{2}\left(\frac{\vd}{\vd u}\right)^{m+n+\rho_j+1}u^{k+1}+\sigma_j(-i)\left(\frac{\vd}{\vd u}\right)^{m+n+\rho_j}u^{k+1} +\tau_j\bigg(\frac{\vd}{\vd u}\bigg)^{m+n+\rho_j-1}u^{k+1}\bigg\}\nonumber\\ &&+\alpha_j\beta_j\left(\frac{\vd}{\vd u}\right)^{m+n+\rho_j}u^{k}-E_j(-i)^{-1}\left(\frac{\vd}{\vd u}\right)^{m+n+\rho_j-1}u^{k}\bigg]=0. \end{eqnarray} Equation~\eqref{dsn1-1} in terms of Pochhammer symbol is expressed as \begin{multline}\label{dsn1} \sum_{k=0}^{\infty}\sum_{m=0}^{\sigma_j-1}\sum_{n=0}^{\tau_j-1} (-i)^{m+n+\rho_j+k}h_{mn}^{j}\\ \times\bigg[\bigg\{(k+2)_{m+n+\rho_j+2} -i(1+a)(k+2)_{m+n+\rho_j+1}-a(k+2)_{m+n+\rho_j}\bigg\}c_{k-2} \\ -\bigg\{\rho_j(k+1)_{m+n+\rho_j+1}-i(k+1)_{m+n+\rho_j}\sigma_j-\tau_j(k+1)_{m+n+\rho_j-1}\bigg\}c_{k-1}\\ +[\alpha_j\beta_j(k)_{m+n+\rho_j}-i E_j(k)_{m+n+\rho_j-1}]c_k\bigg]=0. \end{multline} Here, $(k)_m=k(k-1)(k-2)\cdots(k-m+1).$ From equation~\eqref{dsn1}, one obtains 2 three-term recurrence relations by equating the real part and imaginary parts to zero respectively \begin{multline}\label{dsn2} \bigg[\big[(k+2)_{m+n+\rho_j+2}-a(k+2)_{m+n+\rho_j}\big]c_{k-2} \\ -\big[\rho_j(k+1)_{m+n+\rho_j+1}-\tau_j(k+1)_{m+n+\rho_j-1}\big]c_{k-1}\\ +[\alpha_j\beta_j(k)_{m+n+\rho_j}]c_k\bigg]=0. \end{multline} and \begin{equation}\label{dsn3} -((1+a)(k+2)_{m+n+\rho_j+1}-a(k+2)_{m+n+\rho_j})c_{k-2} +(k+1)_{m+n+\rho_j}\sigma_jc_{k-1} + E_j(k)_{m+n+\rho_j-1}c_k=0. \end{equation} For convenience we set $l_j=m+n+\rho_j$ so that equation~\eqref{dsn2} and \eqref{dsn3} respectively become \begin{multline}\label{dsn3a} \bigg[\big[(k+2)_{l_j+2}-a(k+2)_{l_j}\big]c_{k-2} -\big[\rho_j(k+1)_{l_j+1}-\tau_j(k+1)_{l_j-1}\big]c_{k-1} +[\alpha_j\beta_j(k)_{l_j}]c_k\bigg]=0 \end{multline} and \begin{equation}\label{dsn3b} -[(1+a)(k+2)_{l_j+1}-a(k+2)_{l_j}]c_{k-2} +(k+1)_{l_j}\sigma_jc_{k-1} + E_j(k)_{l_j-1}c_k=0. \end{equation} Solving the recurrence equation~\eqref{dsn3a} by setting $c_{k}=s^{k}$, we have $$\bigg[\big[(k+2)_{l_j+2}-a(k+2)_{l_j}\big]s^{k-2} -\big[\rho_j(k+1)_{l_j+1}-\tau_j(k+1)_{l_j-1}\big]s^{k-1} +[\alpha_j\beta_j(k)_{l_j}]s^k\bigg]=0.$$ Dividing through by $s^{k-2}$ gives $$\bigg[\big[(k+2)_{l_j+2}-a(k+2)_{l_j}\big] -\big[\rho_j(k+1)_{l_j+1}-\tau_j(k+1)_{l_j-1}\big]s +[\alpha_j\beta_j(k)_{l_j}]s^2\bigg]=0.$$ Solving the resulting quadratic equation yields \begin{multline*} s=\frac{\rho_j(k+1)_{l_j+1}-\tau_j(k+1)_{l_j-1}}{2\alpha_j\beta_j(k)_{l_j}}\pm \\ \frac{\sqrt{\left(\rho_j(k+1)_{l_j+1}-\tau_j(k+1)_{l_j-1}\right)^2-4\alpha_j\beta_j(k)_{l_j}\big[(k+2)_{l_j+2}-a(k+2)_{l_j}\big]}}{2\alpha_j\beta_j(k)_{l_j}} \end{multline*} Setting $s=\varepsilon_{k,j}^{(1)}\pm \varepsilon_{k,j}^{(2)}$ so that \begin{eqnarray*} \varepsilon_{k,j}^{(1)} &=& \frac{\rho_j(k+1)_{l_j+1}-\tau_j(k+1)_{l_j-1}}{2\alpha_j\beta_j(k)_{l_j}} \\ \varepsilon_{k,j}^{(2)} &=& \frac{\sqrt{\left(\rho_j(k+1)_{l_j+1}-\tau_j(k+1)_{l_j-1}\right)^2-4\alpha_j\beta_j(k)_{l_j}\big[(k+2)_{l_j+2}-a(k+2)_{l_j}\big]}}{2\alpha_j\beta_j(k)_{l_j}} \end{eqnarray*} we get the coefficient $$c_{k}=A(\varepsilon_{k,j}^{(1)}+\varepsilon_{k,j}^{(2)})^{k}+B(\varepsilon_{k,j}^{(1)}-\varepsilon_{k,j}^{(2)})^{k}$$ where $A+B=1.$ We have non-trivial solution here provided $k\geq l_{j}$. Alternatively, we consider the recurrence equation from the imaginary part given in equation~\eqref{dsn3b} and set $c_{k}=t^{k}$ so that $$-[(1+a)(k+2)_{l_j+1}-a(k+2)_{l_j}]t^{k-2} +(k+1)_{l_j}\sigma_jt^{k-1} + E_j(k)_{l_j-1}t^k=0.$$ Dividing through by $t^{k-2}$ we obtain $$-[(1+a)(k+2)_{l_j+1}-a(k+2)_{l_j}] +(k+1)_{l_j}\sigma_jt + E_j(k)_{l_j-1}t^2=0.$$ Solving the resulting quadratic equation we get $$t=-\,\frac{(k+1)_{l_j}\sigma_j}{E_j(k)_{l_j-1}}\pm\frac{\sqrt{((k+1)_{l_j}\sigma_j)^2-4( E_j(k)_{l_j-1})([(1+a)(k+2)_{l_j+1}-a(k+2)_{l_j}])}}{E_j(k)_{l_j-1}}.$$ Let $t=\eta_{k,j}^{(1)}\pm\eta_{k,j}^{(2)}$ so that \begin{eqnarray*} \eta_{k,j}^{(1)} &=& -\frac{(k+1)_{l_j}\sigma_j}{E_j(k)_{l_j-1}}, \\ \eta_{k,j}^{(2)} &=& \frac{\sqrt{((k+1)_{l_j}\sigma_j)^2-4( E_j(k)_{l_j-1})([(1+a)(k+2)_{l_j+1}-a(k+2)_{l_j}])}}{E_j(k)_{l_j-1}}. \end{eqnarray*} Thus, $$c_{k}=A[\eta_{k,j}^{(1)}+\eta_{k,j}^{(2)}]^{k}+B[\eta_{k,j}^{(1)}-\eta_{k,j}^{(2)}]^{k},$$ where $A+B=1.$ In this non-trivial solutions are obtained only when $k\geq l_{j}-1.$ The distributional solution $\Psi(z)$ is therefore be written as $$ \Psi(z)=\sum_{k=0}^{\infty}c_{k}\delta^{(k)}(z). $$ \end{proof} The next result leads to the computation of Green kernels $G\equiv G(z,w)$ and spectral shift function of the HDO $H_{\frac{n}{2},e}$. The technique adopted here is close in spirit with the one applied by~\cite{DGD}. \begin{teo} Consider the exactly solvable Heun operator $H_{\frac{n}{2},e}$. By Fourier transform approach, the Green function associated with $H_{\frac{n}{2},e}$ is given by $$ G(z,w)=\sum_{m=1}^{p} \frac{(i w)^{m-1}}{(m-1)!}\sum_{m=1}^{p}\sum_{k=0}^{\sigma_{\frac{n}{2}}-1}\sum_{l=0}^{\tau_{\frac{n}{2}}-1}h_{kl}a^{-l}(-1)^{m-1} \epsilon_{0} (m-1)!\delta(z).$$ with $\epsilon_{0}\in \mathbb{C}$ is a constant to be determined and $\displaystyle\phi(w)=\sum_{m=1}^{p}(iw)^{m-1}$. The spectral shift function in this case is $$\xi^{\pm}(\lambda,H_{\frac{n}{2},e},H_{0})=G^{\pm}(w)\mathbb{H}(\lambda),w\in\Omega^{\pm}$$ where $\Omega^{\pm}\subset\mathbb{C}_{\pm}:=\{z\in\mathbb{C}|\pm\Im z>0\}$ and $G^{\pm}(w)=G^{\pm}(w,w)$ given by $$ G^{\pm}(w)= \sum_{m=1}^{p}\sum_{k=0}^{\sigma_{\frac{n}{2}}-1}\sum_{l=0}^{\tau_{\frac{n}{2}}-1}h_{kl}a^{-l}(-1)^{m-1} \epsilon_{0} \delta(w).$$ \end{teo} \begin{proof} Consider the non-homogeneous equation satisfied by Green function $G$ \begin{equation}\label{ghde} H_{\frac{n}{2},e}G=\delta(z-w) \end{equation} then \begin{equation}\label{ghde2} \widehat{H_{\frac{n}{2},e}G}=\widehat{H_{\frac{n}{2},e}}\widehat{G}=\widehat{\delta(z-w)}. \end{equation} This yields \begin{equation}\label{ghde3} \widehat{G}=\widehat{H_{\frac{n}{2},e}}^{-1}\widehat{\delta(z-w)}. \end{equation} Thus, \begin{equation}\label{ghde4} G(z,w)=\mathscr{F}^{-1}\bigg[\widehat{H_{\frac{n}{2},e}}^{-1}\widehat{\delta(z-w)}\bigg] \end{equation} and trace value of its integral operator occurs when $z=w$ so that \begin{equation}\label{ghde4a} G(w)= G(w,w)=\mathscr{F}^{-1}\bigg[\widehat{H_{\frac{n}{2},e}}^{-1}\widehat{\delta(z-w)}\bigg]\bigg|_{z=w} \end{equation} The associated weight function for $H_{\frac{n}{2},e}$ is given by \begin{eqnarray*} \omega(z) &=& z^{\rho_{\frac{n}{2}}-1}(z-1)^{\sigma_{\frac{n}{2}}-1}(z-a)^{\tau_{\frac{n}{2}}-1} \\ &=& (-1)^{\sigma_{\frac{n}{2}}+\tau_{\frac{n}{2}}-2}a^{\tau_{\frac{n}{2}}-1}z^{\rho_{\frac{n}{2}}-1}\sum_{k=0}^{\sigma_{\frac{n}{2}}-1}\binom{\sigma_{\frac{n}{2}}-1}{k}z^k\sum_{l=0}^{\tau_{\frac{n}{2}}-1}\binom{\tau_{\frac{n}{2}}-1}{l}\left(\frac{z}{a}\right)^{l} \\ &=& (-1)^{\sigma_{\frac{n}{2}}+\tau_{\frac{n}{2}}-2}a^{\tau_{\frac{n}{2}}-1}z^{\rho_{\frac{n}{2}}-1}\sum_{k=0}^{\sigma_{\frac{n}{2}}-1}\sum_{l=0}^{\tau_{\frac{n}{2}}-1}\binom{\sigma_{\frac{n}{2}}-1}{k}\binom{\tau_{\frac{n}{2}}-1}{l}z^{k+l}a^{-l} \\ &=& (-1)^{\sigma_{\frac{n}{2}}+\tau_{\frac{n}{2}}-2}a^{\tau_{\frac{n}{2}}-1} \sum_{k=0}^{\sigma_{\frac{n}{2}}-1}\sum_{l=0}^{\tau_{\frac{n}{2}}-1}\binom{\sigma_{\frac{n}{2}}-1}{k}\binom{\tau_{\frac{n}{2}}-1}{l}z^{k+\rho_{\frac{n}{2}}+l-1}a^{-l} \\ &=& (-1)^{\sigma_{\frac{n}{2}}+\tau_{\frac{n}{2}}-2}a^{\tau_{\frac{n}{2}}-1} \sum_{k=0}^{\sigma_{\frac{n}{2}}-1}\sum_{l=0}^{\tau_{\frac{n}{2}}-1}h_{kl}z^{k+\rho_{\frac{n}{2}}+l-1}a^{-l}, \end{eqnarray*} where $\displaystyle h_{kl}=\binom{\sigma_{\frac{n}{2}}-1}{k}\binom{\tau_{\frac{n}{2}}-1}{l}.$ In what follows, let $\chi_{-s},f:L^{2}(\Omega,\vd\mu_{w}(z))\rightarrow L^{2}(\Omega,\vd\mu_{w}(z)),$ $\Omega=\mathbb{CP}^{1}\setminus\{0,a,1,\infty\}$ then the Fourier transform $$\widehat{f}:=\langle f,\chi_{s}\rangle_{\omega}=\int_{\Omega} f(z)\chi_{-s}(z)\vd\mu_{\omega}(z)$$ whence $\chi_{-s}(z)=e^{-is\cdot z}=\overline{\chi_{s}}(z)$ with $s=\sigma+i\tau, \tau=\Im s>0$ and Radon measure $\vd\mu_{\omega}(z)=\omega(z)\vd z.$ By using the inverse formula (cf:~\cite{VVS}, \S1.5, eq.(INV), p.8) $$\reallywidecheck{f\,}=\mathscr{F}^{-1}\widehat{f}(z)=\int_{\Omega}\widehat{f}(s)\chi_{-s}(z)\vd\mu_{\omega}(z).$$ Computing the Fourier transforms of operators in \eqref{ghde4}, one obtains \begin{eqnarray} \widehat{\delta(z-w)} &=& (-1)^{\sigma_{\frac{n}{2}}+\tau_{\frac{n}{2}}-2}a^{\tau_{\frac{n}{2}}-1} \sum_{k=0}^{\sigma_{\frac{n}{2}}-1}\sum_{l=0}^{\tau_{\frac{n}{2}}-1}h_{kl}a^{-l}\int_{\Omega}\delta(z-w)z^{k+\rho_{\frac{n}{2}}+l-1}\chi_{-s}(z)\vd z \nonumber \\ &=& (-1)^{\sigma_{\frac{n}{2}}+\tau_{\frac{n}{2}}-2}a^{\tau_{\frac{n}{2}}-1} \sum_{k=0}^{\sigma_{\frac{n}{2}}-1}\sum_{l=0}^{\tau_{\frac{n}{2}}-1}h_{kl}a^{-l} w^{k+\rho_{\frac{n}{2}}+l-1}\chi_{-s}(w) \label{ghde5} \end{eqnarray} Next, it is obvious that \begin{eqnarray*} \widehat{H_{\frac{n}{2},e}} &=& \mathscr{F}[H_{\frac{n}{2},e}] \\ &=& \mathscr{F}\left[(z^3-(1+a)z^2+az)D^2+(\rho_{\frac{n}{2}}z^2+\sigma_{\frac{n}{2}}z+\tau_{\frac{n}{2}})D+\frac{n(n-1)}{2}z\right] \end{eqnarray*} can be simplified term-wisely as \begin{eqnarray} \mathscr{F}[z^3D^2] &=& (-1)^{\sigma_{\frac{n}{2}}+\tau_{\frac{n}{2}}-2}a^{\tau_{\frac{n}{2}}-1}\sum_{k=0}^{\sigma_{\frac{n}{2}}-1}\sum_{l=0}^{\tau_{\frac{n}{2}}-1}h_{kl}a^{-l} \int_{\Omega} z^3D^2(z^{k+l+\rho_{\frac{n}{2}}-1}\chi_{-s}(z))\vd z\nonumber\\ &=&(-1)^{\sigma_{\frac{n}{2}}+\tau_{\frac{n}{2}}-2}a^{\tau_{\frac{n}{2}}-1}\sum_{k=0}^{\sigma_{\frac{n}{2}}-1}\sum_{l=0}^{\tau_{\frac{n}{2}}-1}h_{kl}a^{-l} \int_{\Omega}[(k+l+\rho_{\frac{n}{2}}-1)(k+l+\rho_{\frac{n}{2}}-2)z^{k+l+\rho_{\frac{n}{2}}}\nonumber\\ &&+2is(k+l+\rho_{\frac{n}{2}}-1)z^{k+l+\rho_{\frac{n}{2}}+1}-s^2z^{k+l+\rho_{\frac{n}{2}}+2}]\chi_{-s}(z)\vd z\nonumber\\ &=&(-1)^{\sigma_{\frac{n}{2}}+\tau_{\frac{n}{2}}-2}a^{\tau_{\frac{n}{2}}-1}\sum_{k=0}^{\sigma_{\frac{n}{2}}-1}\sum_{l=0}^{\tau_{\frac{n}{2}}-1}h_{kl}a^{-l} [(k+l+\rho_{\frac{n}{2}}-1)(k+l+\rho_{\frac{n}{2}}-2)\left(\frac{\vd}{\vd s}\right)^{k+l+\rho_{\frac{n}{2}}}\nonumber\\ &&+2is(k+l+\rho_{\frac{n}{2}}-1)\left(\frac{\vd}{\vd s}\right)^{k+l+\rho_{\frac{n}{2}}+1}-s^2\left(\frac{\vd}{\vd s}\right)^{k+l+\rho_{\frac{n}{2}}+2}];\label{ghde6} \end{eqnarray} \begin{eqnarray} -(1+a)\mathscr{F}[z^2D^2] &=& -(1+a)(-1)^{\sigma_{\frac{n}{2}}+\tau_{\frac{n}{2}}-2}a^{\tau_{\frac{n}{2}}-1}\sum_{k=0}^{\sigma_{\frac{n}{2}}-1}\sum_{l=0}^{\tau_{\frac{n}{2}}-1}h_{kl}a^{-l} \nonumber\\ &&\times \int_{\Omega}z^{2}[(k+l+\rho_{\frac{n}{2}}-1)(k+l+\rho_{\frac{n}{2}}-2)z^{k+l+\rho_{\frac{n}{2}}-3}\nonumber\\ &&+2is(k+l+\rho_{\frac{n}{2}}-1)z^{k+l+\rho_{\frac{n}{2}}-2}-s^2z^{k+l+\rho_{\frac{n}{2}}-1}]\chi_{-s}(z)\vd z\nonumber\\ &=& -(1+a)(-1)^{\sigma_{\frac{n}{2}}+\tau_{\frac{n}{2}}-2}a^{\tau_{\frac{n}{2}}-1}\nonumber\\ &&\times \sum_{k=0}^{\sigma_{\frac{n}{2}}-1}\sum_{l=0}^{\tau_{\frac{n}{2}}-1}h_{kl}a^{-l} [(k+l+\rho_{\frac{n}{2}}-1)(k+l+\rho_{\frac{n}{2}}-2)\left(\frac{\vd}{\vd s}\right)^{k+l}\nonumber\\ &&+\rho_{\frac{n}{2}-1}+2is(k+l+\rho_{\frac{n}{2}}-1)\left(\frac{\vd}{\vd s}\right)^{k+l+\rho_{\frac{n}{2}}}-s^2\left(\frac{\vd}{\vd s}\right)^{k+l+\rho_{\frac{n}{2}}+1}]; \label{ghde7} \end{eqnarray} \begin{eqnarray} a\mathscr{F}[zD^2] &=& a^{\tau_{\frac{n}{2}}}(-1)^{\sigma_{\frac{n}{2}}+\tau_{\frac{n}{2}}-2}\sum_{k=0}^{\sigma_{\frac{n}{2}}-1}\sum_{l=0}^{\tau_{\frac{n}{2}}-1}h_{kl}a^{-l}\int_{\Omega}z[(k+l+\rho_{\frac{n}{2}}-1)(k+l+\rho_{\frac{n}{2}}-2)z^{k+l+\rho_{\frac{n}{2}}-1}\nonumber\\ &&+2is(k+l+\rho_{\frac{n}{2}}-1)z^{k+l+\rho_{\frac{n}{2}}}-s^2z^{k+l+\rho_{\frac{n}{2}}+1}]\chi_{-s}(z)\vd z;\nonumber\\ &=& a^{\tau_{\frac{n}{2}}}(-1)^{\sigma_{\frac{n}{2}}+\tau_{\frac{n}{2}}-2}\sum_{k=0}^{\sigma_{\frac{n}{2}}-1}\sum_{l=0}^{\tau_{\frac{n}{2}}-1}h_{kl}a^{-l}[(k+l+\rho_{\frac{n}{2}}-1)(k+l+\rho_{\frac{n}{2}}-2)\left(\frac{\vd}{\vd s}\right)^{k+l+\rho_{\frac{n}{2}}}\nonumber\\ &&+2is(k+l+\rho_{\frac{n}{2}}-1)\left(\frac{\vd}{\vd s}\right)^{k+l+\rho_{\frac{n}{2}}+1}-s^2\left(\frac{\vd}{\vd s}\right)^{k+l+\rho_{\frac{n}{2}}+2}];\label{ghde8} \end{eqnarray} \begin{eqnarray} \rho_{\frac{n}{2}}\mathscr{F}[z^{2}D] &=& \rho_{\frac{n}{2}}a^{\tau_{\frac{n}{2}}-1}(-1)^{\sigma_{\frac{n}{2}}+\tau_{\frac{n}{2}}-2}\sum_{k=0}^{\sigma_{\frac{n}{2}}-1}\sum_{l=0}^{\tau_{\frac{n}{2}}-1}h_{kl}a^{-l} \int_{\Omega}z^{2}D(z^{k+l+\rho_{\frac{n}{2}}-1}\chi_{-s}(z))\vd z\nonumber\\ &=& \rho_{\frac{n}{2}}a^{\tau_{\frac{n}{2}}-1}(-1)^{\sigma_{\frac{n}{2}}+\tau_{\frac{n}{2}}-2}\nonumber\\ &&\times\sum_{k=0}^{\sigma_{\frac{n}{2}}-1}\sum_{l=0}^{\tau_{\frac{n}{2}}-1}h_{kl} a^{-l} \int_{\Omega}[(k+l+\rho_{\frac{n}{2}}-1)z^{k+l+\rho_{\frac{n}{2}}}+isz^{k+l+\rho_{\frac{n}{2}}+1}]\chi_{-s}(z)\vd z\nonumber\\ &=& \rho_{\frac{n}{2}}a^{\tau_{\frac{n}{2}}-1}(-1)^{\sigma_{\frac{n}{2}}+\tau_{\frac{n}{2}}-2}\nonumber\\ &&\times\sum_{k=0}^{\sigma_{\frac{n}{2}}-1}\sum_{l=0}^{\tau_{\frac{n}{2}}-1}h_{kl}a^{-l} [(k+l+\rho_{\frac{n}{2}}-1)\left(\frac{\vd}{\vd s}\right)^{k+l+\rho_{\frac{n}{2}}}+is\left(\frac{\vd}{\vd s}\right)^{k+l+\rho_{\frac{n}{2}}+1}];\nonumber\\ \label{ghde9} \end{eqnarray} \begin{eqnarray} \sigma_{\frac{n}{2}}\mathscr{F}[zD] &=& \sigma_{\frac{n}{2}}a^{\tau_{\frac{n}{2}}-1}(-1)^{\sigma_{\frac{n}{2}}+\tau_{\frac{n}{2}}-2}\sum_{k=0}^{\sigma_{\frac{n}{2}}-1}\sum_{l=0}^{\tau_{\frac{n}{2}}-1}h_{kl} a^{-l} \int_{\Omega}zD(z^{k+l+\rho_{\frac{n}{2}}-1}\chi_{-s}(z))\vd z\nonumber\\ &=& \sigma_{\frac{n}{2}}a^{\tau_{\frac{n}{2}}-1}(-1)^{\sigma_{\frac{n}{2}}+\tau_{\frac{n}{2}}-2}\nonumber\\ &&\times\sum_{k=0}^{\sigma_{\frac{n}{2}}-1}\sum_{l=0}^{\tau_{\frac{n}{2}}-1}h_{kl} a^{-l} \int_{\Omega}[(k+l+\rho_{\frac{n}{2}}-1)z^{k+l+\rho_{\frac{n}{2}}-1}+isz^{k+l+\rho_{\frac{n}{2}}}]\chi_{-s}(z)\vd z\nonumber\\ &=& \sigma_{\frac{n}{2}}a^{\tau_{\frac{n}{2}}-1}(-1)^{\sigma_{\frac{n}{2}}+\tau_{\frac{n}{2}}-2}\nonumber\\ &&\times\sum_{k=0}^{\sigma_{\frac{n}{2}}-1}\sum_{l=0}^{\tau_{\frac{n}{2}}-1}h_{kl} a^{-l} [(k+l+\rho_{\frac{n}{2}}-1)\left(\frac{\vd}{\vd s}\right)^{k+l+\rho_{\frac{n}{2}}-1}+is\left(\frac{\vd}{\vd s}\right)^{k+l+\rho_{\frac{n}{2}}}];\nonumber\\ \label{ghde10} \end{eqnarray} \begin{eqnarray} \tau_{\frac{n}{2}}\mathscr{F}[zD] &=& \tau_{\frac{n}{2}}a^{\tau_{\frac{n}{2}}-1}(-1)^{\sigma_{\frac{n}{2}}+\tau_{\frac{n}{2}}-2}\sum_{k=0}^{\sigma_{\frac{n}{2}}-1}\sum_{l=0}^{\tau_{\frac{n}{2}}-1}h_{kl} a^{-l} \int_{\Omega}D(z^{k+l+\rho_{\frac{n}{2}}-1}\chi_{-s}(z))\vd z\nonumber\\ &=& \tau_{\frac{n}{2}}a^{\tau_{\frac{n}{2}}-1}(-1)^{\sigma_{\frac{n}{2}}+\tau_{\frac{n}{2}}-2}\nonumber\\ &&\times\sum_{k=0}^{\sigma_{\frac{n}{2}}-1}\sum_{l=0}^{\tau_{\frac{n}{2}}-1}h_{kl}a^{-l} \int_{\Omega}[(k+l+\rho_{\frac{n}{2}}-1)z^{k+l+\rho_{\frac{n}{2}}-2}+isz^{k+l+\rho_{\frac{n}{2}}-1}]\chi_{-s}(z)\vd z\nonumber\\ &=& \tau_{\frac{n}{2}}a^{\tau_{\frac{n}{2}}-1}(-1)^{\sigma_{\frac{n}{2}}+\tau_{\frac{n}{2}}-2}\nonumber\\ &&\times\sum_{k=0}^{\sigma_{\frac{n}{2}}-1}\sum_{l=0}^{\tau_{\frac{n}{2}}-1}h_{kl} a^{-l} [(k+l+\rho_{\frac{n}{2}}-1)\left(\frac{\vd}{\vd s}\right)^{k+l+\rho_{\frac{n}{2}}-2}+is\left(\frac{\vd}{\vd s}\right)^{k+l+\rho_{\frac{n}{2}}-1}];\nonumber\\\label{ghde11} \end{eqnarray} \begin{eqnarray} \frac{n(n-1)}{2}\mathscr{F}[z] &=& \frac{n(n-1)}{2} a^{\tau_{\frac{n}{2}}-1}(-1)^{\sigma_{\frac{n}{2}}+\tau_{\frac{n}{2}}-2}\sum_{k=0}^{\sigma_{\frac{n}{2}}-1}\sum_{l=0}^{\tau_{\frac{n}{2}}-1}h_{kl}a^{-l} \int_{\Omega}z^{k+l+\rho_{\frac{n}{2}}-1}\chi_{-s}(z)\vd z\nonumber\\ &=& \frac{n(n-1)}{2}a^{\tau_{\frac{n}{2}}-1}(-1)^{\sigma_{\frac{n}{2}}+\tau_{\frac{n}{2}}-2} \sum_{k=0}^{\sigma_{\frac{n}{2}}-1}\sum_{l=0}^{\tau_{\frac{n}{2}}-1}h_{kl} a^{-l}\left(\frac{\vd}{\vd s}\right)^{k+l+\rho_{\frac{n}{2}}-1}.\label{ghde12} \end{eqnarray} Adding up equations~\eqref{ghde6}-\eqref{ghde12} yields \begin{eqnarray} \widehat{H_{\frac{n}{2},e}} &=& a^{\tau_{\frac{n}{2}}-1}(-1)^{\sigma_{\frac{n}{2}}+\tau_{\frac{n}{2}}-2} \sum_{k=0}^{\sigma_{\frac{n}{2}}-1}\sum_{l=0}^{\tau_{\frac{n}{2}}-1}h_{kl}a^{-l}\nonumber\\ &&\cdot\bigg\{[(k+l+\rho_{\frac{n}{2}}-1)[(1+a)(k+l+\rho_{\frac{n}{2}}-2-2is)+\rho_{\frac{n}{2}}]+is\sigma_{\frac{n}{2}}]\left(\frac{\vd}{\vd s}\right)^{k+l+\rho_{\frac{n}{2}}} \nonumber\\ &&+[2is(1+a)(k+l+\rho_{\frac{n}{2}}-1)+\rho_{\frac{n}{2}}is-s^2]\left(\frac{\vd}{\vd s}\right)^{k+l+\rho_{\frac{n}{2}}+1}-2s^{2}\left(\frac{\vd}{\vd s}\right)^{k+l+\rho_{\frac{n}{2}}+2}\nonumber\\ &&+[(k+l+\rho_{\frac{n}{2}}-1)(\sigma_{\frac{n}{2}}-(1+a)(k+l+\rho_{\frac{n}{2}}-2))+is\tau_{\frac{n}{2}}+\frac{n(n-1)}{2}]\left(\frac{\vd}{\vd s}\right)^{k+l+\rho_{\frac{n}{2}}-1}\nonumber\\ && +\tau_{\frac{n}{2}}(k+l+\rho_{\frac{n}{2}}-1)\left(\frac{\vd}{\vd s}\right)^{k+l+\rho_{\frac{n}{2}}-2}\bigg\}.\label{ghde13} \end{eqnarray} Let $m_{kl}=k+l+\rho_{\frac{n}{2}}$ and $c_{n}=a^{\tau_{\frac{n}{2}}-1}(-1)^{\sigma_{\frac{n}{2}}+\tau_{\frac{n}{2}}-2}$ then the expression in \eqref{ghde13} becomes \begin{eqnarray*} \widehat{H_{\frac{n}{2},e}} &=& c_{n} \sum_{k=0}^{\sigma_{\frac{n}{2}}-1}\sum_{l=0}^{\tau_{\frac{n}{2}}-1}h_{kl}a^{-l}\left\{[(m_{kl}-1)[(1+a)(m_{kl}-2-2is)+\rho_{\frac{n}{2}}]+is\sigma_{\frac{n}{2}}]\left(\frac{\vd}{\vd s}\right)^{m_{kl}} \right.\nonumber\\ &&+[2is(1+a)(m_{kl}-1)+\rho_{\frac{n}{2}}is-s^2]\left(\frac{\vd}{\vd s}\right)^{m_{kl}+1}-2s^{2}\left(\frac{\vd}{\vd s}\right)^{m_{kl}+2}\nonumber\\ &&+[(m_{kl}-1)(\sigma_{\frac{n}{2}}-(1+a)(m_{kl}-2))+is\tau_{\frac{n}{2}}+\frac{n(n-1)}{2}]\left(\frac{\vd}{\vd s}\right)^{m_{kl}-1}\nonumber\\ &&\left. +\tau_{\frac{n}{2}}(m_{kl}-1)\left(\frac{\vd}{\vd s}\right)^{m_{kl}-2}\right\} \end{eqnarray*} which further becomes \begin{eqnarray*} \widehat{H_{\frac{n}{2},e}} &=& c_{n} \sum_{m_{kl}=0}^{\sigma_{\frac{n}{2}}-\rho_{\frac{n}{2}}-l-1}\;\sum_{l=0}^{\tau_{\frac{n}{2}}-1}h_{kl}a^{-l}\left\{[(m_{kl}-1)[(1+a)(m_{kl}-2-2is)+\rho_{\frac{n}{2}}]+is\sigma_{\frac{n}{2}}]\left(\frac{\vd}{\vd s}\right)^{m_{kl}} \right.\\ &&+[2is(1+a)(m_{kl}-1)+\rho_{\frac{n}{2}}is-s^2]\left(\frac{\vd}{\vd s}\right)^{m_{kl}+1}-2s^{2}\left(\frac{\vd}{\vd s}\right)^{m_{kl}+2}\\ &&+[(m_{kl}-1)(\sigma_{\frac{n}{2}}-(1+a)(m_{kl}-2))+is\tau_{\frac{n}{2}}+\frac{n(n-1)}{2}]\left(\frac{\vd}{\vd s}\right)^{m_{kl}-1}\\ &&\left. +\tau_{\frac{n}{2}}(m_{kl}-1)\left(\frac{\vd}{\vd s}\right)^{m_{kl}-2}\right\}. \end{eqnarray*} By index shifting \begin{eqnarray}\label{ghde14} \widehat{H_{\frac{n}{2},e}}&=& c_{n} \sum_{m_{kl}=2}^{\sigma_{\frac{n}{2}}-\rho_{\frac{n}{2}}-l+1}\;\sum_{l=0}^{\tau_{\frac{n}{2}}-1}h_{kl}a^{-l}\left\{\left[(m_{kl}-1)[(1+a)(m_{kl}-2-2is)+\rho_{\frac{n}{2}}]+is\sigma_{\frac{n}{2}}\right.\right.\nonumber\\ &&+\tau_{\frac{n}{2}}(m_{kl}+1)]\left(\frac{\vd}{\vd s}\right)^{m_{kl}} +[\left(2(1+a)(m_{kl}-1)+(\rho_{\frac{n}{2}}+\tau_{\frac{n}{2}})\right)is-s^2\nonumber\\ && \left.+(m_{kl}+1)(\sigma_{\frac{n}{2}}-(1+a)m_{kl})+\frac{n(n-1)}{2}]\left(\frac{\vd}{\vd s}\right)^{m_{kl}+1}-2s^{2}\left(\frac{\vd}{\vd s}\right)^{m_{kl}+2}\right\}\nonumber\\ \end{eqnarray} and by \eqref{ghde5} \begin{equation}\label{ghde15} \widehat{\delta(z-w)}=c_{n}\sum_{m_{kl}=1}^{\sigma_{\frac{n}{2}}-\rho_{\frac{n}{2}}-l}\;\sum_{l=0}^{\tau_{\frac{n}{2}}-1}h_{kl}a^{-l}w^{m_{kl}-1}\chi_{-s}(w). \end{equation} Let \begin{eqnarray}\label{ghde16} \Delta(m_{kl},l,\frac{\vd}{\vd s})&\equiv&\sum_{m_{kl}=2}^{\sigma_{\frac{n}{2}}-\rho_{\frac{n}{2}}-l+1}\;\sum_{l=0}^{\tau_{\frac{n}{2}}-1}h_{kl}a^{-l}\bigg\{[(m_{kl}-1)[(1+a)(m_{kl}-2-2is)+\rho_{\frac{n}{2}}]+is\sigma_{\frac{n}{2}}\nonumber\\ && +\tau_{\frac{n}{2}}(m_{kl}+1)] +[\left(2(1+a)(m_{kl}-1)+(\rho_{\frac{n}{2}}+\tau_{\frac{n}{2}})\right)is-s^2\nonumber\\ &&+(m_{kl}+1)(\sigma_{\frac{n}{2}}-(1+a)m_{kl})+\frac{n(n-1)}{2}]\frac{\vd}{\vd s}-2s^{2}\left(\frac{\vd}{\vd s}\right)^{2}\bigg\}. \end{eqnarray} Then $$\widehat{H_{\frac{n}{2},e}}=c_{n}\Delta(m_{kl},l,\frac{\vd}{\vd s})\left(\frac{\vd}{\vd s}\right)^{m_{kl}}.$$ Now, the Fourier of Green function is given by \begin{eqnarray}\label{gde17} \widehat{ G}&=&\Delta(m_{kl},l,\frac{\vd}{\vd s})^{-1}\left(\frac{\vd}{\vd s}\right)^{-m_{kl}}\sum_{m_{kl}=1}^{\sigma_{\frac{n}{2}}-\rho_{\frac{n}{2}}-l}\;\sum_{l=0}^{\tau_{\frac{n}{2}}-1}h_{kl}a^{-l}w^{m_{kl}-1}\chi_{-s}(w)\nonumber\\ &=&\Delta(m_{kl},l,\frac{\vd}{\vd s})^{-1}\sum_{m_{kl}=1}^{\sigma_{\frac{n}{2}}-\rho_{\frac{n}{2}}-l}\;\sum_{l=0}^{\tau_{\frac{n}{2}}-1}h_{kl}a^{-l}w^{m_{kl}-1}\underbrace{\int\cdots\int}_{m_{kl}}\chi_{-s}(w)\underbrace{\vd s\cdots\vd s}_{m_{kl}}\nonumber\\ &=&\Delta(m_{kl},l,\frac{\vd}{\vd s})^{-1}\sum_{m_{kl}=1}^{\sigma_{\frac{n}{2}}-\rho_{\frac{n}{2}}-l}\;\sum_{l=0}^{\tau_{\frac{n}{2}}-1}h_{kl}a^{-l}\underbrace{\int\cdots\int}_{m_{kl}-1} \delta(w)\underbrace{\vd s\cdots\vd s}_{m_{kl}-1} \nonumber\\ &=&\Delta(m_{kl},l,\frac{\vd}{\vd s})^{-1}\sum_{m_{kl}=1}^{\sigma_{\frac{n}{2}}-\rho_{\frac{n}{2}}-l}\;\sum_{l=0}^{\tau_{\frac{n}{2}}-1}h_{kl}a^{-l}w^{m_{kl}-1}\frac{s^{m_{kl}-1}}{(m_{kl}-1)!}.\nonumber\\ \end{eqnarray} Let the operator $\Delta(m_{kl},l,\frac{\vd}{\vd s})$ be written in a factorable form $$\Delta(m_{kl},l,\frac{\vd}{\vd s})=\sum \epsilon_{0}+\epsilon_{1}\frac{\vd}{\vd s}+\epsilon_{2}\left(\frac{\vd}{\vd s}\right)^{2}\equiv\sum \left(\eta_{+}+\frac{\vd}{\vd s}\right)\left(\eta_{-}+\frac{\vd}{\vd s}\right)$$ where $\displaystyle\sum\equiv\sum_{m_{kl}=1}^{\sigma_{\frac{n}{2}}-\rho_{\frac{n}{2}}-l}\;\sum_{l=0}^{\tau_{\frac{n}{2}}-1}h_{kl}a^{-l}$ and \begin{eqnarray*} \epsilon_{0} &=& (m_{kl}-1)[(1+a)(m_{kl}-2-2is)+\rho_{\frac{n}{2}}]+is\sigma_{\frac{n}{2}}+\tau_{\frac{n}{2}}(m_{kl}+1)\\ \epsilon_{1} &=& \left(2(1+a)(m_{kl}-1)+(\rho_{\frac{n}{2}}+\tau_{\frac{n}{2}})\right)is-s^2+(m_{kl}+1)(\sigma_{\frac{n}{2}}-(1+a)m_{kl})+\frac{n}{2}(n-1)\\ \epsilon_{2} &=& -2s^{2} \end{eqnarray*} with \begin{equation*} \eta_{\pm}=\frac{-\epsilon_{1}\pm\sqrt{\epsilon_{1}^{2}-4\epsilon_{0}\epsilon_{2}}}{2\epsilon_{0}}. \end{equation*} Let $$\phi(s,w)=\sum_{m_{kl}=1}^{\sigma_{\frac{n}{2}}-\rho_{\frac{n}{2}}-l}\;\sum_{l=0}^{\tau_{\frac{n}{2}}-1}h_{kl}a^{-l}w^{m_{kl}-1}\frac{s^{m_{kl}-1}}{(m_{kl}-1)!}.$$ By (\cite{AF}, Chapter 16, short methods (c), p.99), given function of operators $$F(\frac{\vd}{\vd s})=\prod_{p=0}^{n}(\eta_{p}+\frac{\vd}{\vd s})\equiv\sum_{k=0}^{n}a_{k}\left(\frac{\vd}{\vd s}\right)^{k}$$ with constant coefficients $a_k$ then \begin{equation}\label{gde16b} \frac{1}{F(\frac{\vd}{\vd s})}s^{m}=\left(\sum_{k=0}^{m}a_{k}\left(\frac{\vd}{\vd s}\right)^{k}\right) s^{m}. \end{equation} Then by equations~\eqref{gde16b} and \eqref{gde17} one gets \begin{equation*} \Delta(m_{kl},l,\frac{\vd}{\vd s})^{-1}\phi(w,s)=\sum_{m_{kl}=1}^{\sigma_{\frac{n}{2}}-\rho_{\frac{n}{2}}-l}\;\sum_{l=0}^{\tau_{\frac{n}{2}}-1}h_{kl}a^{-l}\frac{w^{m_{kl}-1}}{(m_{kl}-1)!}\left(\sum \epsilon_{0}+\epsilon_{1}\left(\frac{\vd}{\vd s}\right)+\epsilon_{2}\left(\frac{\vd}{\vd s}\right)^{2}\right)s^{m_{kl}-1}. \end{equation*} Thus, $\widehat{G}$ associated with the Heun differential operator is $$\widehat{G}=\sum\frac{w^{m_{kl}-1}}{(m_{kl}-1)!}\left(\sum \epsilon_{0}s^{m_{kl}-1}+\epsilon_{1}(m_{kl}-1)s^{m_{kl}-2}+\epsilon_{2}(m_{kl}-1)(m_{kl}-2)s^{m_{kl}-3}\right).$$ $s=\sigma+i\tau$ and $-H_0=D^2$. It can be recalled that given a polynomial $P:\mathbb{C}\rightarrow\mathbb{C}, P(\frac{\vd}{\vd z})\delta(z)=\mathscr{F}^{-1}[P(-is)].$ Thus, one obtains the Green function $G(z,w)=\reallywidecheck{\widehat{G}\;}$ by \begin{multline}\label{ddff} G(z,w) =\sum i^{m_{kl}-1}\frac{w^{m_{kl}-1}}{(m_{kl}-1)!}\\ \times\left(\sum\reallywidecheck{\epsilon_{0}(-is)^{m_{kl}-1}}-i(m_{kl}-1)\epsilon_{1}\reallywidecheck{(-is)^{m_{kl}-2}}-(m_{kl}-1)(m_{kl}-2)\epsilon_{2}\reallywidecheck{(-is)^{m_{kl}-3}}\right). \end{multline} For convenience, let $m=m_{kl}, c_{m}=\frac{i^{m_{kl}}}{m_{kl}!}, d_{m}=-im_{kl}, e_{m}=-m_{kl}(m_{kl}-1)$, $\displaystyle p=\sigma_{\frac{n}{2}}-\rho_{\frac{n}{2}}-l, \sum\equiv\sum_{m=1}^{p}.$ By using the property of Dirac delta function derivatives (See~\cite{KRP}, \S 9.11 (4):~243) \begin{equation*} w^{n}\delta^{(m)}(w)=\left\{\begin{array}{cc} 0 & m<n \\ {} & {}\\ \displaystyle (-1)^{n}\frac{m!}{(m-n)!}\delta^{(m-n)}(w)& m\geq n. \end{array} \right. \end{equation*} one evaluates the inverse Fourier transforms \begin{eqnarray} \reallywidecheck{(-is)^{m-1}} &=& \int_{\Omega}(-is)^{m-1}e^{s\cdot z}\sum_{k=0}^{\sigma_{\frac{n}{2}}-1}\sum_{l=0}^{\tau_{\frac{n}{2}}-1}h_{kl}a^{-l}z^{m-1}\vd s \nonumber\\ &=& \sum_{k=0}^{\sigma_{\frac{n}{2}}-1}\sum_{l=0}^{\tau_{\frac{n}{2}}-1}h_{kl}a^{-l}z^{m-1}\int_{\Omega}(-is)^{m-1}e^{s\cdot z}\vd s \nonumber\\ &=& \sum_{k=0}^{\sigma_{\frac{n}{2}}-1}\sum_{l=0}^{\tau_{\frac{n}{2}}-1}h_{kl}a^{-l}z^{m-1}\delta^{(m-1)}(z)\nonumber\\ &=& \sum_{k=0}^{\sigma_{\frac{n}{2}}-1}\sum_{l=0}^{\tau_{\frac{n}{2}}-1}h_{kl}a^{-l}(-1)^{m-1}(m-1)!\delta(z) \label{dff9a}\\ \reallywidecheck{(-is)^{m-2}} &=& \sum_{k=0}^{\sigma_{\frac{n}{2}}-1}\sum_{l=0}^{\tau_{\frac{n}{2}}-1}h_{kl}a^{-l}z^{m-1}\delta^{(m-2)}(z)=0 \label{dff9b} \\ \reallywidecheck{(-is)^{m-3}} &=& \sum_{k=0}^{\sigma_{\frac{n}{2}}-1}\sum_{l=0}^{\tau_{\frac{n}{2}}-1}h_{kl}a^{-l}z^{m-1}\delta^{(m-3)}(z)=0.\nonumber\\ \label{dff9c} \end{eqnarray} Re-substituting equations~\eqref{dff9a}-\eqref{dff9c} into equation~\eqref{ddff}, one gets \begin{equation}\label{ddff1} G(z,w)=\sum_{m=1}^{p} \frac{(i w)^{m-1}}{(m-1)!}\sum_{m=1}^{p}\sum_{k=0}^{\sigma_{\frac{n}{2}}-1}\sum_{l=0}^{\tau_{\frac{n}{2}}-1}h_{kl}a^{-l}(-1)^{m-1} \epsilon_{0} (m-1)!\delta(z). \end{equation} In terms of the eigenvalue $E_{\frac{n}{2},e},$ the Green function $G(E_{\frac{n}{2},e},z,w)$ of $H_{\frac{n}{2},e}$ with $Dom(H_{\frac{n}{2},e})\in\mathscr{C}_{c}^{\infty}(\Omega)$ is defined by the integral $$(H_{\frac{n}{2},e}-E_{\frac{n}{2},e})^{-1}f(z)=\int_{\Omega}G(E_{\frac{n}{2},e},z,w)f(w)\vd w, \;\;\;\Im E_{\frac{n}{2},e}\neq 0.$$ In what follows, it is of particular interest, to obtain the Green function at the point $z=w\in\Omega^{\pm}$ (the upper and lower complex half-plane excluding the singular points $0,1,a,\infty$). By setting $z=w$, one obtains \begin{eqnarray*} G(E_{\frac{n}{2},e}, w)=G(E_{\frac{n}{2},e}, w,w) &=&\left\{\begin{array}{cc} G^{+}(E_{\frac{n}{2},e}, w,w)&, w\in\Omega^{+} \\ &\\ G^{-}(E_{\frac{n}{2},e}, w,w)&, w\in \Omega^{-} \end{array} \right. \end{eqnarray*} where, $G^{\pm}(E_{\frac{n}{2},e}, w,w)=G^{\pm}(E_{\frac{n}{2},e},w)$ can now be written in the form \begin{equation}\label{grsum} G^{\pm}(E_{\frac{n}{2},e},w)= \sum_{m=1}^{p} \frac{(iw)^{m-1}}{(m-1)!}\sum_{m=1}^{p}\sum_{k=0}^{\sigma_{\frac{n}{2}}-1}\sum_{l=0}^{\tau_{\frac{n}{2}}-1}h_{kl}a^{-l}\frac{(-1)^{m-1}}{E_{\frac{n}{2},e}} \epsilon_{0} (m-1)!\delta(\pm w). \end{equation} It is obvious that $\displaystyle\sum_{m=1}^{p} \frac{(iw)^{m-1}}{(m-1)!}$ is a $(p-1)$th approximation of $e^{iw}$. By setting $\displaystyle\phi(w)=\sum_{m=1}^{p}(iw)^{m-1}$ and following ~(\cite{JAB}, Definition~3, Eq.(9), p.3), then equation~\eqref{grsum} may be written as \begin{equation}\label{grsum1} G^{\pm}(E_{\frac{n}{2},e},w)= \sum_{m=1}^{p}\sum_{k=0}^{\sigma_{\frac{n}{2}}-1}\sum_{l=0}^{\tau_{\frac{n}{2}}-1}h_{kl}a^{-l}\frac{(-1)^{m-1}}{E_{\frac{n}{2},e}} \epsilon_{0} \phi(w)\delta(\pm w). \end{equation} It should be remarked here that $\delta(w)$ is an even function for $\ell\in\mathbb{Z}_{0}^{+}$, thus $$\delta^{(m)}(\pm w)=\left\{\begin{array}{cc} +\delta^{(m)}(w), & m=2\ell, \\ & \\ - \delta^{(m)}(w), & m=2\ell+1 , \end{array}\right.$$ and \begin{equation}\label{grsu} \phi(w)\delta^{(m)}(w)=(-1)^{m}\sum_{r=0}^{m}\binom{m}{r}(-1)^{r}\phi^{(m-r)}(0)\delta^{(r)}(w),\;\;\binom{m}{r}=\frac{m!}{r!(m-r)!}. \end{equation} From these data, the expression ~\eqref{grsum1} for Green function becomes \begin{eqnarray}\label{grsum2} G^{\pm}(E_{\frac{n}{2},e},w)&=& \sum_{m=1}^{p}\sum_{k=0}^{\sigma_{\frac{n}{2}}-1}\sum_{l=0}^{\tau_{\frac{n}{2}}-1}h_{kl}a^{-l}\frac{(-1)^{m-1}}{E_{\frac{n}{2},e}}\epsilon_{0} \phi(w)\delta(w)\nonumber\\ &=&\sum_{m=1}^{p}\sum_{k=0}^{\sigma_{\frac{n}{2}}-1}\sum_{l=0}^{\tau_{\frac{n}{2}}-1}h_{kl}a^{-l}\frac{(-1)^{m-1}}{E_{\frac{n}{2},e}} \epsilon_{0} \phi(0)\delta(w), \end{eqnarray} where $\phi(0)=1$ so that equation~\eqref{grsum2} becomes \begin{equation}\label{grsum4} G^{\pm}(E_{\frac{n}{2},e},w)= \sum_{m=1}^{p}\sum_{k=0}^{\sigma_{\frac{n}{2}}-1}\sum_{l=0}^{\tau_{\frac{n}{2}}-1}h_{kl}a^{-l}\frac{(-1)^{m-1}}{E_{\frac{n}{2},e}} \epsilon_{0} \delta(w). \end{equation} It should be remarked here that $$G^{\pm}(\cdot,w)\in\mathscr{C}_{c}^{\infty}(\Omega)\subset L^{2}(\Omega^{+}, \vd_{\omega}\mu(w))\oplus L^{2}(\Omega^{-}, \vd_{\omega}\mu(w)).$$ By setting the eigenvalue $E_{\frac{n}{2}}=\lambda+i\epsilon$ were $\lambda=\Re E_{\frac{n}{2}}$ and $\epsilon=\Im E_{\frac{n}{2}}$ one defines the SSF using the Privalov's representation (cf:~\cite{GES}, \S (4.8):~760) as \begin{equation}\label{ghde17} \xi^{\pm}(\lambda,H_{\frac{n}{2},e},H_{0})=\frac{1}{\pi}\lim_{\epsilon\downarrow 0}\textrm{Arg}\; G^{\pm}(\lambda+i\epsilon,w). \end{equation} Further simplification leads to \begin{eqnarray} \xi^{\pm}(\lambda,H_{\frac{n}{2},e},H_{0}) &=& \frac{1}{\pi}G^{\pm}(w,w)\lim_{\epsilon\downarrow 0}\textrm{Arg}\frac{1}{\lambda+i\epsilon} \nonumber\\ &=& \frac{1}{\pi}G^{\pm}(w,w)\lim_{\epsilon\downarrow 0}\textrm{Arg}\frac{\lambda-i\epsilon}{\lambda^{2}+\epsilon^{2}} \nonumber\\ &=& \frac{1}{\pi}G^{\pm}(w,w)\lim_{\epsilon\downarrow 0}\tan^{-1}\left(-\frac{\epsilon}{\lambda}\right),\label{ghde18} \end{eqnarray} where $\lim_{\epsilon\downarrow 0}\tan^{-1}\left(-\frac{\epsilon}{\lambda}\right)=\pi \mathbb{H}(\lambda)$ is the Heisenberg distribution (see~\cite{KRP}, \S2.7, Example 4, p.45) and $G^{\pm}(w,w)\equiv G^{\pm}(w)$ is given by $$G^{\pm}(w)= \sum_{m=1}^{p}\sum_{k=0}^{\sigma_{\frac{n}{2}}-1}\sum_{l=0}^{\tau_{\frac{n}{2}}-1}h_{kl}a^{-l}(-1)^{m-1} \epsilon_{0} \delta(w).$$ Therefore, one gives the spectral shift function in times of Heisenberg distribution as $$\xi^{\pm}(\lambda,H_{\frac{n}{2},e},H_{0})=G^{\pm}(w)\mathbb{H}(\lambda).$$ This completes the proof. \end{proof} \begin{defn} An operator $T\in B(\mathscr{H})$ is called Hilbert-Schmidt if and only if $tr (T^{\ast}T)<\infty$. The family of Hilbert-Schmidt operators is denoted by $B_{2}(\mathscr{H}).$ \end{defn} Next, it is noteworthy to consider the Hilbert-Schmidt property of $G^{\pm}(w)$ in $L^{2}(\Omega\times\Omega,\vd \mu_{\omega}(w)\otimes\vd \mu_\omega(w)).$ $G^{\pm}(w)$ is defined in $L^{2}(\Omega\times\Omega,\vd \mu_{\omega}(w)\otimes\vd \mu_\omega(w)).$ \begin{proof} Let $c_{n}=a^{\tau_{\frac{n}{2}}-1}(-1)^{\sigma_{\frac{n}{2}}+\tau_{\frac{n}{2}}-2}$ and $\mathscr{K}_{p}$ be given by \begin{equation*} \mathscr{K}_{p} = \sum_{m=1}^{p}\sum_{m=0}^{\sigma_{\frac{n}{2}}-1}\sum_{n=0}^{\tau_{\frac{n}{2}}-1}h_{kl} a^{-l}(-1)^{m-1}\epsilon_{0}. \end{equation*} Then \begin{eqnarray} \|G^{\pm}\|_{2}^{2} &=& \int_{\Omega}\int_{\Omega}G^{\pm}(\cdot,w,z)\overline{G^{\pm}(\cdot,w,z)}\vd\mu_{\omega}(w)\vd\mu_{\omega}(w)\nonumber \\ &=&\int_{\Omega}\int_{\Omega} \mathscr{K}_{p}^{2}\left(\sum_{m=0}^{\sigma_\frac{n}{2}-1}\sum_{n=0}^{\tau_\frac{n}{2}-1}c_{n}h_{mn}w^{m+n+\rho_\frac{n}{2}-1}\right)^2\delta^2(w)\vd w\vd w\nonumber\\ &=&\left(\int_{\Omega}\mathscr{K}_{p}\sum_{m=0}^{\sigma_\frac{n}{2}-1}\sum_{n=0}^{\tau_\frac{n}{2}-1}c_{n}h_{mn}w^{m+n+\rho_\frac{n}{2}-1}\delta(w)\vd w\right)^{2} .\nonumber\\ &=&\left(\int_{\Omega}\mathscr{K}_{p}\sum_{m=0}^{\sigma_\frac{n}{2}-1}\sum_{n=0}^{\tau_\frac{n}{2}-1}c_{n}h_{mn}w^{m+\beta}\delta(w)\vd w\right)^{2} .\nonumber\\ \label{Hsc} \end{eqnarray} Let $\displaystyle\omega(w)=\sum_{m=0}^{\sigma_\frac{n}{2}-1}\sum_{n=0}^{\tau_\frac{n}{2}-1}c_{n}h_{mn}w^{m+n+\rho_\frac{n}{2}-1}$ then equation~\eqref{Hsc} becomes \begin{equation*} \|G^{\pm}\|_{2}^{2}=\mathscr{K}_{p}^{2}\omega^{2}(0)<\infty. \end{equation*} Hence, the result follows. \end{proof} In what follows, using equation~\eqref{ddff1} as $p\rightarrow\infty, \mathscr{K}_{p}\rightarrow\mathscr{K}_{\infty}, \phi(w)\rightarrow \phi_{\infty}, G_{\infty}^{\pm}(\cdot,w,z)$ takes the form \begin{equation*} G_{\infty}^{\pm}(\cdot,w,z)=\mathscr{K}_{\infty}\phi_{\infty}(w)\delta(z). \end{equation*} \begin{teo}\label{GSI} Let $\mathscr{H}=L^2(\Omega,\vd \mu_{\omega})$ and $$T_{G_{\infty}}^{\pm}\Psi(z):=\int_{\Omega}G_{\infty}^{\pm}(\cdot,w,z)\Psi(w)\vd \mu_{\omega}(w)$$ then $T_{G_{\infty}}^{\pm}\in B(\mathscr{H})$ is Hilbert-Schmidt if and only if there is a function $G_{\infty}^{\pm}\in L^{2}(\Omega\times\Omega,\vd \mu_{\omega}(w)\otimes\vd \mu_\omega(w)).$ Moreover, $$\|T_{G_{\infty}}^{\pm}\|_{2}^{2}=\int_{\Omega}|G_{\infty}^{\pm}(\cdot,w,z)|^{2}\vd \mu_{\omega}(w)\vd \mu_{\omega}(w).$$ \end{teo} \begin{proof} It has been shown by Theorem~\ref{GSI} that a finite rank operator $G^{\pm}\in L^{2}(\Omega\times\Omega, \vd\mu_{\omega}\otimes\vd\mu_{\omega})$ (say of rank $p$) and $T_{G}^{\pm}$ its associated integral operator. Now, as $p\rightarrow \infty, G^{\pm}\rightarrow G_{\infty}^{\pm}\in L^{2}(\Omega\times\Omega, \vd\mu_{\omega}\otimes\vd\mu_{\omega}), T_{G}^{\pm}\rightarrow T_{G_{\infty}}^{\pm}.$ For any two functions $\Psi_{1},\Psi_{2}\in L^{2}(\Omega,\vd\mu_{\omega}), T_{G_{\infty}}^{\pm}\Psi_{1}=T_{G_{\infty}}^{\pm}\Psi_{2}\implies T_{G_{\infty}}^{\pm}(\Psi_{1}-\Psi_{2}) = 0\implies \Psi_{1}-\Psi_{2}=0$ as $T_{G_{\infty}}^{\pm}\neq 0.$ This means that $\Psi_{1}=\Psi_{2}$ and thus $T_{G_{\infty}}^{\pm}$ is well defined on $\mathscr{H}.$ However, \begin{eqnarray*} \|T_{G_{\infty}}^{\pm}\Psi(z)\|_{L^1}&=&\bigg\|\int_{\Omega}G_{\infty}^{\pm}(\cdot,w,z)\Psi(w)\vd\mu_{\omega}(w)\bigg\|_{L^1} \\ \implies \|T_{G_{\infty}}^{\pm}\Psi(z)\|_{L^1}\leqslant\|T_{G_{\infty}}^{\pm}\|_{L^2}\|\Psi(z)\|_{L^2} &\leqslant& \|G_{\infty}^{\pm}(\cdot,w,z)\|_{L^2}\|\Psi(w)\|_{L^2}\\ \implies \|T_{G_{\infty}}^{\pm}\|_{L^2}&\leqslant& \|G_{\infty}^{\pm}(\cdot,w,z)\|_{L^2} \end{eqnarray*} (by Cauchy-Schwartz inequality). From the last inequality, it is obvious that as $p\rightarrow\infty$ $$\|G^{\pm}-G_{\infty}^{\pm}\|_{L^2}\rightarrow 0\implies \|T_{G}^{\pm}- T_{G_{\infty}}^{\pm}\|_{L^2}\rightarrow 0. $$ Thus, $T_{G_{\infty}}^{\pm}$ is compact and it is however remarkable that if $\{\Psi_{\nu}(z)\overline{\Psi_{\nu'}(w)}\}_{\nu,\nu'\in\mathbb{N}}$ is orthonormal basis of $L^{2}(\Omega\times\Omega,\vd\mu_{\omega}\otimes\vd\mu_{\omega})$ then the trace operator \begin{eqnarray*} tr (T_{G_{\infty}}^{\pm\,\ast}T_{G_{\infty}}^{\pm}) &=& \langle T_{G_{\infty}}^{\pm}\Psi_{\nu},T_{G_{\infty}}^{\pm}\Psi_{\nu}\rangle, \\ &=&\langle T_{G_{\infty}}^{\pm\,\ast}T_{G_{\infty}}^{\pm}\Psi_{\nu},\Psi_{\nu}\rangle\\ &=& \| T_{G_{\infty}}^{\pm}\|_{L^2}^2\|\Psi_{\nu}\|_{L^2}^2\\ &=&\| T_{G_{\infty}}^{\pm}\|_{L^2}^2\leqslant \|G_{\infty}^{\pm}(\cdot,w,z)\|_{L^2}<\infty.\\ \end{eqnarray*} when $\| T_{G_{\infty}}^{\pm}\|_{L^2}^2= \|G_{\infty}^{\pm}(\cdot,w,z)\|_{L^2}, T_{G_{\infty}}^{\pm}\in B_{2}(\mathscr{H}).$ \end{proof} It is observed that the kernel $G(z,w)$ is a differentiable function. Hence, the following result. \begin{teo} Let $\psi\in\mathscr{C}^{\infty}(\mathbb{CP}^1)$- then the integral operator $$T(\psi(z))=\int_{\mathbb{CP}^1}G(z,w)\psi(z)\vd g[z].$$ defined on $L^{2}(\mathbb{CP}^1)=L^2(\mathbb{CP}^1,\vd g[z])$ is a Hilbert-Schmidt operator, hence is compact. \end{teo} The trace of $T$ is given by $$\mathrm{tr}\;(T)=\int_{\mathbb{CP}^1}G(z,z)\vd g[z].$$ Since $\mathbb{CP}^1$ is a compact Riemann manifold, we observe that the closed graph theorem implies that the map $$\psi\mapsto\mathrm{tr}\;(T(\psi)):\mathscr{C}^{\infty}(\mathbb{CP}^1)\longrightarrow \mathbb{C}$$ is a distribution on $\mathbb{CP}^1.$ This result shows that the Lie algebraic HDO is a compact operator defined on a compact Lie group $SL(2,\mathbb{C})$. \section{Conclusion} In this work, the quasi-exact and exact solvability of the Lie algebraic equations associated with the $sl(2)$-algebra has been studied relative to the Heun operator. The distributional solution of QES Heun equation, the Green kernel of exactly solvable Heun equation, the integral relations associated with Heun- Green kernels obtained, the compactness of the integral operator SSF and Hilbert-Schmidt condition of the integral operator associated with Heun-Green kernel has been discussed. The spectral shift functions of the exactly solvable operator has been investigated using the Fourier transform technique. \begin{thebibliography}{99} \bibitem{AF} Ayres F.~Jr. \emph{ Theory and Problems of Differential Equations. Schaum outline series}, McGraw-Hill Inc., USA (1952). \bibitem{DJ} Dixmier J. Enveloping Algebras, Graduate studies in Mathematics, American Mathematical Society, North-Holland Mathematical Library, Vol. 14 (1977). \bibitem{DGD}Duffy D.~G. \emph{ Green Functions with Applications,} Studies in Advanced Mathematics, Chapman and Hall, CRC. (2001) \bibitem{ERK1} Estrada R. and Kanwal R.~P. Applications of distributional derivatives to wave propagation \emph{J. Inst. Math. Appl.} 26(1980) 39-63. \bibitem{ERK2} Estrada R. and Kanwal R.~P. \emph{ A distributional approach to asymptotics theory and applications},Birkh\"{a}user. Boston, Basel, Berlin (2002). \bibitem{GES}Gesztesy F. Inverse Spectral Theory as Influenced by Barry Simon. In A Festschrift in Honor of Barry Simon's 60th Birthday. Proceedings of Symposia in Pure Mathematics titled:~\emph{ Spectral Theory and Mathematical Physics}, Vol.76.2. (2007) (78pp). \bibitem{JAB}Jarad~F., Adjabi~Y., Baleanu~D., and Thabet~A., On defining the distributions $\delta^{r}$ and $\delta'^{r}$ by conformable derivatives. \emph{Advances in Difference Equations} (2018) (20 pages), \url{https://doi.org/10.1186/s13662-018-1865-7}. \bibitem{KRP} Kanwal R.~P.,\emph{ Generalised Functions, Theory and Applications} 3 edition 2009, \copyright 2004 Springer Science+Business Media New York, Originally published by Birkh\"{a}user Boston in 2004. \bibitem{KJA} Kirillov Jr.~A. \emph{ An Introduction to Lie Groups and Lie Algebras}. Cambridge University Press, New York (2008). \bibitem{KYN} Kynsinski J. On Semigroups Generated by Differential Operators on Lie Groups \emph{ Journ. of Funct. Anal.} 31, 234--244 (1979). \bibitem{LJK}Littlejohn L.~L. and Kanwal R.P. 1987. Distributional Solutions of Hypergeometric Equation. \emph{Journ. of Math. Anal. and Appl.} 122, 325--345. \bibitem {MRS} Maier R.~S. 192 Solutions of The Heun Equation. \emph{ Mathematics of Computation} Volume 76, no. 258, April 2007, 811 -- 843. S 0025-5718(06)01939-9. Article electronically published on November 28, 2006. \bibitem{MCM} McMullen C., \emph{Complex Analysis on Reimann Surface}, Lecture note, Math213-Harvard University. \bibitem{MIL}Miller W.~Jr., \emph{ Lie Theory and Special Functions}. Academic Press (1968) p. 20. \bibitem{OFW} Olver F.~W.~J. (Ed.), Lozier D.W., Boisvert R.F. and Clark C.W. \emph{NIST Handbook of Mathematical Function.} Cambridge University Press. New York (2010). \bibitem{RA1} Ronveaux A. (Ed.). \emph{ Heun’s Differential Equations}. Oxford University Press, Oxford, 1995. \bibitem{WR} Rudin W. \emph{Real and Complex Analysis}, 1st Edition, McGraw-Hill Inc., London, 1970. \bibitem{RW} Ruhl W. \emph{Lorentz Group and Harmonic Analysis}. The Mathematical Physics Monograph Series. W.A. Benjamin Inc. (1970). \bibitem{VVS} Varadarajan V.~S. \emph{ Introduction to Harmonic Analysis on Semi-simple Lie groups}. Cambridge University Press. New York (2001). \end{thebibliography} \vspace{.5cm} \address{Department of Mathematics, \\ Adeyemi Federal University of Education,\\ 143 Ondo-Ore Road, 351103 Ondo State, NIGERIA} \noindent{\emph{E-mail address}:~\email{[email protected], [email protected]}} \end{document}
2412.05503v1
http://arxiv.org/abs/2412.05503v1
Critical scaling profile for trees and connected subgraphs on the complete graph
\documentclass[11pt,hidelinks]{article} \usepackage{orcidlink} \title{Critical scaling profile for trees and connected subgraphs \\ on the complete graph} \author{ Yucheng Liu\,\orcidlink{0000-0002-1917-8330}\thanks{Department of Mathematics, University of British Columbia, Vancouver, BC, Canada V6T 1Z2. Liu: \href{mailto:[email protected]}{[email protected]}. Slade: \href{mailto:[email protected]}{[email protected]}. } \and Gordon Slade\,\orcidlink{0000-0001-9389-9497}$^*$ } \date{\vspace{-5ex}} \usepackage{amsmath, amssymb, amscd, amsthm, amsfonts} \usepackage{graphicx} \usepackage{hyperref} \usepackage{booktabs} \usepackage{xcolor} \usepackage{ dsfont, bm} \usepackage[title]{appendix} \usepackage{comment} \usepackage{enumerate} \usepackage{enumitem} \usepackage{cite} \usepackage[textwidth=480pt,textheight=650pt,centering]{geometry} \theoremstyle{plain} \newtheorem{theorem}{Theorem}[section] \newtheorem{lemma}[theorem]{Lemma} \newtheorem{conjecture}[theorem]{Conjecture} \newtheorem{proposition}[theorem]{Proposition} \newtheorem{corollary}[theorem]{Corollary} \newtheorem{definition}[theorem]{Definition} \newtheorem{assumption}[theorem]{Assumption} \newtheorem{remark}[theorem]{Remark} \numberwithin{equation}{section} \newcommand{\etal}{\textit{et al}.} \newcommand{\ie}{i.e.} \newcommand{\eg}{e.g.} \renewcommand{\ae}{a.e.} \newcommand{\eps}{\varepsilon} \newcommand{\pho}{\rho} \newcommand{\N}{\mathbb{N}} \newcommand{\Z}{\mathbb{Z}} \newcommand{\Q}{\mathbb{Q}} \newcommand{\R}{\mathbb{R}} \newcommand{\C}{\mathbb{C}} \newcommand{\E}{\mathbb{E}} \newcommand{\F}{\mathbb{F}} \newcommand{\K}{\mathbb{K}} \renewcommand{\P}{\mathbb{P}} \newcommand{\T}{\mathbb{T}} \newcommand{\backmatter}{} \newcommand{\Fcal}{\mathcal{F}} \newcommand{\Lcal}{\mathcal{L}} \newcommand{\Scal}{\mathcal{S}} \newcommand{\del}{\partial} \newcommand{\grad}{\nabla} \newcommand{\inv}{^{-1}} \renewcommand{\(}{\left(} \renewcommand{\)}{\right)} \newcommand{\half}{\frac{1}{2}} \newcommand{\Del}{\Delta} \newcommand{\1}{\mathds{1}} \newcommand{\stle}{\preccurlyeq} \newcommand{\nl}{\nonumber \\} \newcommand{\loc}{_{\mathrm{loc}}} \renewcommand{\Re}{\mathrm{Re}\,} \renewcommand{\Im}{\mathrm{Im}\,} \DeclareMathOperator{\supp}{supp} \DeclareMathOperator\arctanh{arctanh} \providecommand{\abs}[1]{\lvert#1\rvert} \providecommand{\bigabs}[1]{\big\lvert#1\big\rvert} \providecommand{\Bigabs}[1]{\Big\lvert#1\Big\rvert} \providecommand{\biggabs}[1]{\bigg\lvert#1\bigg\rvert} \providecommand{\norm}[1]{\lVert#1\rVert} \providecommand{\bignorm}[1]{\big\lVert#1\big\rVert} \providecommand{\Bignorm}[1]{\Big\lVert#1\Big\rVert} \providecommand{\biggnorm}[1]{\bigg\lVert#1\bigg\rVert} \newcommand{\Ann}{B} \newcommand{\mz}{m(z)} \renewcommand{\mp}{m(p)} \newcommand{\btil}{\tilde{b}} \newcommand{\chip}{{\chi(p)}} \newcommand{\chim}{\chi \supm} \newcommand{\mo}{{\color{violet} m_0}} \newcommand{\muz}{{\mu_z}} \newcommand{\mup}{{\mu_p}} \newcommand{\supm}{^{(m)}} \newcommand{\supmz}{^{(\mz)}} \newcommand{\supmp}{^{(\mp)}} \newcommand{\supmS}{^{(m_S(z))}} \newcommand{\supM}{^{(M)}} \newcommand{\supN}{^{(N)}} \newcommand{\supNm}{^{(N,m)}} \newcommand{\supT}{^{ \mathbb{T} }} \newcommand{\supk}[1]{^{(#1)}} \newcommand{\subrz}{_{r,z}} \newcommand{\rz}{{r,z}} \newcommand{\st}{{st}} \newcommand{\supeps}{^{(\eps)}} \newcommand{\supzero}{^{(0)}} \newcommand{\supzerom}{^{(0,m)}} \newcommand{\supinfty}{^{(\infty)}} \newcommand{\supLambda}{^{(\Lambda)}} \newcommand{\supSAW}{^{\text{(SAW})}} \newcommand{\supperc}{^{\rm {perc}}} \newcommand{\supNN}{^{(\text{NN})}} \newcommand{\blue}[1]{{\color{blue}#1}} \newcommand{\red}[1]{{\color{red}#1}} \usepackage{mathrsfs} \newcommand{\Lscr}{\mathscr{L}} \newcommand{\Yscr}{\mathscr{Y}} \newcommand{\nnb}{\nonumber\\} \newcommand{\uvec}{\tilde{u}} \newcommand{\vecu}{\uvec} \newcommand{\veee}[1]{|\!|\!|#1|\!|\!|} \newcommand{\xvee}{\veee{x}} \providecommand{\nnnorm}[1]{\veee #1} \newcommand{\const}{\mathrm{const}} \providecommand{\floor}[1]{\lfloor #1 \rfloor} \providecommand{\ceil}[1]{\lceil#1\rceil} \providecommand{\integer}[1]{[#1]} \newcommand{\Dnn}{P} \newcommand{\degree}{\Omega} \newcommand{\crit}{_{p_c}} \newcommand{\Rd}{{\mathbb R^d}} \newcommand{\Zd}{\mathbb Z^d} \newcommand{\Td}{\mathbb T^d} \newcommand{\dc}{d_{\mathrm{c}}} \newcommand{\m}{ { \color{orange} m } } \newcommand{\chgs}[1]{ \red {#1} } \newcommand{\chYL}[1]{ \blue {#1} } \newcommand{\commentgs}[1] { \begin{quote} { \bf Comment from GS: #1 } \end{quote} } \newcommand{\commentYL}[1]{ \begin{quote} { \bf Comment from YL: #1 } \end{quote} } \newcommand{\commentYLblue}[1]{ \begin{quote} { \color{blue} \bf Comment from YL: #1 } \end{quote} } \begin{document} \maketitle \begin{abstract} We analyse generating functions for trees and for connected subgraphs on the complete graph, and identify a single scaling profile which applies for both generating functions in a critical window. Our motivation comes from the analysis of the finite-size scaling of lattice trees and lattice animals on a high-dimensional discrete torus, for which we conjecture that the identical profile applies in dimensions $d \ge 8$. \end{abstract} \section{Main results} \subsection{Results} The enumeration of trees and connected graphs has a long history. We are motivated by problems arising in the critical behaviour of branched polymers in equilibrium statistical mechanics, to consider certain generating functions for the number of trees and connected subgraphs in the complete graph $\K_V$ on $V$ labelled vertices. The vertices are labelled as $\mathbb V = \{ 0, \dots, V-1 \}$ and the edge set is $\mathbb E = \{ \{ x,y \}: x,y \in \mathbb V \}$. Our interest is in the asymptotic behaviour as $V \to \infty$. We define \emph{one-point functions} \begin{align} \label{eq:G0def} G_{V,0}^t(p) = \sum_{T \ni 0} \Bigl(\frac{p}{eV} \Bigr)^{\abs T} , \qquad G_{V,0}^a(p) = \sum_{A \ni 0} \Bigl(\frac{p}{eV} \Bigr)^{\abs A} , \end{align} where the first sum is over all labelled trees $T$ in $\K_V$ containing the vertex $0$, the second sum is over all labelled connected subgraphs containing $0$, and $|T|$ and $|A|$ denote the number of edges in $T$ and $A$. The division of $p$ by $eV$ is a normalisation to make $p=1$ correspond to a critical value. We also study the \emph{two-point functions} \begin{align} \label{eq:G01def} G_{V,01}^t(p) = \sum_{T \ni 0,1} \Bigl(\frac{p}{eV} \Big)^{\abs T} , \qquad G_{V,01}^a(p) = \sum_{A \ni 0,1} \Bigl(\frac{p}{eV} \Big)^{\abs A} , \end{align} where the sums now run over trees or connected subgraphs containing the distinct vertices $0,1$. To avoid repetition, when a formula applies to both trees and connected subgraphs we often omit the superscripts $t,a$. With this convention, we define the \emph{susceptibility} \begin{equation} \label{eq:chidef} \chi_V(p) = G_{V,0}(p)+(V-1)G_{V,01}(p). \end{equation} We are particularly interested in values of $p$ in a \emph{critical window} $p=1+sV^{-1/2}$ around the critical point, with $s \in \R$. We define the profile \begin{equation} \label{eq:Iprofile} I(s) = \frac{e}{\sqrt{2\pi}}\int_0^\infty e^{-\frac 12 x^2+sx}\frac{1}{\sqrt{x}} \mathrm d x \qquad (s \in \R). \end{equation} The profile can be rewritten in terms of a \emph{Fax\'en integral} \cite[p.~332]{Olve97} as $I (s) = e\pi^{-1/2}2^{-5/4} \mathrm{Fi} ( \frac{1}{2}, \frac{1}{4} ; \sqrt{2} s )$, and its asymptotic behaviour is given by \cite[Ex.~7.3, p.~84]{Olve97} to be \begin{align} I (s) \sim \begin{cases} e |2s|^{-1/2} & (s \to -\infty) \\ e s^{-1/2}e^{s^2/2} & (s \to +\infty) , \end{cases} \end{align} where $f \sim g$ means $\lim f/g = 1$. Our main result is the following theorem. \begin{theorem} \label{thm:profile} For both trees and connected subgraphs, and for all $s \in \R$, as $V \to \infty$ we have \begin{align} \label{eq:G01-profile} G_{V,01}(1 + sV^{-1/2}) & \sim V^{-3/4} I(s) , \\ \label{eq:LTK-profile} \chi_V(1 + sV^{-1/2}) &\sim V^{1/4} I(s) . \end{align} \end{theorem} The proof of Theorem~\ref{thm:profile} uses a uniform bound on the one-point function. The following theorem gives a statement that is more precise than a bound. It involves the principal branch $W_0$ of the Lambert function \cite{CGHJK96}, which solves $W_0e^{W_0} =z$ and has power series \begin{equation} \label{eq:Lambert} W_0(z) = \sum_{n=1}^\infty \frac{(-n)^{n-1}}{n!}z^n. \end{equation} The solution to $W_0e^{W_0} = -1/e$ is achieved by the particular value $W_0(-1/e)=-1$. \begin{theorem} \label{thm:G0} For both trees and connected subgraphs, for all $s \ge 0$, and for all sequences $p_V$ with $p_V \le 1+sV^{-1/2}$ and $\lim_{V\to\infty}p_V=p\in [0,1]$, \begin{align} \label{eq:G0lim} \lim_{V\to\infty} G_{V,0}(p_V) &= \sum_{n=1}^\infty \frac{ n^{n-1} }{n!} \Big( \frac{p}{e} \Big)^{n-1} = -\frac{e}{p}W_0\Big(\!\!-\frac{p}{e} \Big) . \end{align} In particular, if $p=1$ then $\lim_{V\to\infty} G_{V,0}(p_V) = e$. \end{theorem} \noindent \textbf{Notation:} We write $f \lesssim g$ if there is a $C>0$ such that $f(x) \le C g(x)$ for all $x$ of interest. \subsection{Method of proof} To prove \eqref{eq:G01-profile}, it suffices to prove \eqref{eq:LTK-profile} and \eqref{eq:G0lim}, since when $p=1+sV^{-1/2}$, by definition of $\chi_V$ we then have \begin{equation} \label{eq:Gchi-ratio} G_{V,01} = \frac{ \chi_V - G_{V,0} } {V - 1 } \sim \frac {\chi_V} V . \end{equation} \subsubsection{Trees} By Cayley's formula, the number of trees on $n$ labelled vertices is $n^{n-2}$. By decomposing the sum defining $G_{V,0}^t(p)$ according to the number $n$ of vertices in the tree, and by counting the number of ways to choose $n-1$ vertices other than $0$, we have \begin{align} \label{eq:G0} G_{V,0}^t(p) &= \sum_{n=1}^{V} \binom{V-1}{n-1} n^{n-2} \Big(\frac{p}{eV} \Big)^{n-1} . \end{align} Similarly, \begin{equation} \label{eq:G01t} G_{V,01}^t(p) = \sum_{n=2}^{V} \binom{V-2}{n-2} n^{n-2} \Big(\frac{p}{eV} \Big)^{n-1} . \end{equation} Since \begin{equation} \binom{V-1}{n-1} + (V-1) \binom{V-2}{n-2} = n\binom{V-1}{n-1}, \end{equation} it follows from \eqref{eq:chidef} that the susceptibility is given by \begin{align} \label{eq:chit} \chi_{V}^t(p) &= \sum_{n=1}^{V} \binom{V-1}{n-1} n^{n-1} \Big(\frac{p}{eV} \Big)^{n-1}. \end{align} For trees, we prove Theorems~\ref{thm:profile}--\ref{thm:G0} by directly analysing the above series for $\chi_V^t$ and $G_{V,0}^t$. The profile $I(s)$ for $\chi_V^t(1 + sV^{-1/2})$ arises from a Riemann sum limit. \subsubsection{Connected subgraphs} For connected subgraphs, we will show that the contribution to $\chi_V^a, G_{V,01}^a$ from connected subgraphs with cycles is much smaller than the contribution from trees. Let $C(n,n-1+\ell)$ denote the number of connected graphs on $n$ labelled vertices with exactly $n-1+\ell$ edges, i.e., with $\ell$ surplus edges. The surplus must be zero for $n=1,2$. For $n\ge 3$, we define the \emph{surplus generating function} \begin{equation} \label{eq:def_S} S(n,z) = \sum_{\ell= 1}^\infty C(n,n-1+\ell) z^{\ell} . \end{equation} Note that terms in the above sum are zero unless $\ell \le \binom{n} 2 - (n-1)$, and that the tree term ($\ell=0$) is absent. We extract the tree contributions and define remainder terms by \begin{align} G_{V,0}^a(p) &= G_{V,0}^t(p) + \Delta_{V,0}(p), \\ \chi_{V}^a(p) &= \chi_{V}^t(p) + \Delta_{V}(p), \end{align} with \begin{align} \label{eq:Delta0} \Delta_{V,0}(p) &= \sum_{n=3}^{V} \binom{V-1}{n-1} S\Big(n,\frac p {eV}\Big) \Big(\frac{p}{eV} \Big)^{n-1} , \\ \label{eq:chiDelta} \Delta_{V}(p) & = \sum_{n=3}^{V} \binom{V-1}{n-1} n S\Big(n,\frac p {eV}\Big) \Big(\frac{p}{eV} \Big)^{n-1}. \end{align} Given Theorems~\ref{thm:profile}--\ref{thm:G0} for trees, we prove Theorems~\ref{thm:profile}--\ref{thm:G0} for connected subgraphs by showing that, for all $s\in \R$, \begin{align} \label{eq:Delta0bd} \lim_{V\to\infty} \Delta_{V,0}(1+sV^{-1/2}) & =0, \\ \label{eq:Deltabd} \lim_{V\to\infty} V^{-1/4}\Delta_V(1 + sV^{-1/2}) &= 0. \end{align} The proof is more subtle than for trees and requires estimates on the surplus generating function. As we discuss later, a precise but cumbrous asymptotic formula for $C(n,n+k)$ is given in \cite[Corollary~1]{BCM90}. We use that formula to prove the following useful explicit bound. By convention, $k^k = 1$ when $k=0$. \begin{proposition} \label{prop:Cnk-intro} Let $n \ge 3$ and $N = \binom n 2$. For $0 \le k \le n$, we have \begin{align} \label{eq:Cnk-intro} C(n, n+k) \lesssim \binom N {n+k} \bigg( \frac 2 e \bigg)^n \bigg( \frac{ e n }{ k }\bigg)^{k/2} . \end{align} \end{proposition} Proposition~\ref{prop:Cnk-intro} is most useful when the surplus $\ell = k+1$ is small but of order $n$. This is a delicate region when controlling the surplus generating function, and the precise constant $e$ in the last factor of \eqref{eq:Cnk-intro} is important. For a larger surplus, we simply bound $C(n,n+k)$ by the total number of graphs (connected or not) on $n$ vertices with $n+k$ edges, which is $\binom{N}{n+k}$. Together, these bounds provide enough control on $S(n,p/(eV))$ to prove \eqref{eq:Delta0bd}--\eqref{eq:Deltabd}. \subsection{Motivation} Theorem~\ref{thm:profile} is motivated by a broader emerging theory of finite-size scaling in statistical mechanical models above their upper critical dimensions. The theory involves a family of profiles expressed in terms of the functions \begin{equation} I_k(s) = \int_0^\infty x^{k} e^{-\frac 14 x^4 - \frac 12 s x^2} \mathrm dx \qquad (s\in \R, \; k>-1) . \end{equation} A change of variables transforms the profile $I$ of \eqref{eq:Iprofile} into $I(s)= e 2^{1/4}\pi^{-1/2} I_0(-\sqrt{2}s)$. The general theory is described in \cite{LPS25-universal} with references to the extensive physics and mathematics literature. Given an integer $d \ge 2$, infinite-volume models can be formulated on a transitive graph $\mathbb{G}=(\Z^d,\mathbb{E})$, whose edge set $\mathbb{E}$ has a finite number of edges containing the origin and is invariant under the symmetries of $\Z^d$. Above an \emph{upper critical dimension} $d_{\rm c}$, for many models it has been proven that the critical exponents that describe the critical behaviour are the same as the corresponding exponents when the model is formulated on a regular tree or on the complete graph. The tree and complete graph settings are easy to analyse. Finite-volume models (with periodic boundary conditions) can instead be formulated on a discrete torus $\mathbb{G}_r = (\T_r^d,\mathbb{E}_r)$ of period~$r$. At and above the upper critical dimension, the torus models are known or conjectured to have critical behaviour analogous to that seen on the complete graph, with an interesting ``plateau'' phenomenon involving a universal profile which is often expressed in terms of $I_k$. The value of $k$ depends on the model. Dimensions $d<d_{\rm c}$ are conjectured to exhibit different scaling, with no plateau or profile. \smallskip\noindent {\bf Lattice trees and lattice animals:} A \emph{lattice animal} is a finite connected subgraph of $\mathbb{G}$, and a \emph{lattice tree} is an acyclic lattice animal. The critical behaviour of lattice trees and lattice animals is at least as difficult as is the case for the notoriously difficult self-avoiding walk. Despite significant interest from chemists and physicists for over half a century, due to applications to branched polymers \cite{Jans15}, the critical behaviour is understood mathematically only in dimensions $d>d_{\rm c} = 8$. For $d>8$, it has been proved using the lace expansion that for sufficiently large edge sets $\mathbb{E}$ (or for nearest-neighbour edges with $d$ sufficiently large), lattice trees and lattice animals at the critical point both have the same behaviour as a critical branching process \cite{HS90b,HS92c,DS98,CFHP23}. For $x\in \Z^d$, let $c_m(x)$ denote the number of lattice trees or lattice animals containing $0,x$ and having exactly $m$ bonds. The one-point functions, two-point functions, and susceptibilities are defined by \begin{equation} g(z) = \sum_{m=0}^\infty c_m(0)z^m, \qquad G_z(x) = \sum_{m=0}^\infty c_m(x)z^m, \qquad \chi(z) = \sum_{x\in \Z^d} G_z(x). \end{equation} The radius of convergence $z_{\rm c}$ (the \emph{critical point}) of these series is finite and positive, and is strictly smaller for animals than for trees \cite{GPSW94}. High-dimensional versions and extensions of Theorem~\ref{thm:G0} for $g(z_c)$ are proved in \cite{MS13,KS24}. The analogous quantities for trees and animals on the torus $\T_r^d$ are denoted $g_r(z)$, $G_{r,z}(x)$, $\chi_r(z)$. These are polynomials in $z$, so they define entire functions of $z$. Nevertheless, for large $r$ the infinite-volume critical point $z_c$ plays a role in the scaling. We denote the volume of the torus by $V=r^d$. Our computation of the profile $I$ for the two-point function and susceptibility in Theorem~\ref{thm:profile} supports the following conjecture from \cite{LS25a} that the profile $I_0$ (just a rescaled $I$) occurs for both lattice trees and lattice animals on the torus, above the upper critical dimension. \begin{conjecture} For lattice trees and lattice animals on $\T_r^d$ with $d>8$, there are constants $a_d<0$ and $b_d>0$ (different constants for trees and animals) such that, as $V=r^d \to \infty$, \begin{equation}\begin{aligned} \label{eq:LTLAplateau} G_{r,z_c+sV^{-1/2}}(x) -G_{z_c}(x) &\sim b_d V^{-3/4} I_0(a_d s), \\ \chi_r(z_c+sV^{-1/2}) &\sim b_d V^{1/4} I_0(a_d s). \end{aligned}\end{equation} \end{conjecture} In \eqref{eq:LTLAplateau}, the torus point $x$ is identified with its representative in $\Z^d \cap (-\frac r2,\frac r2]^d$ in the evaluation of $G_{z_c}(x)$. For $d>8$, $G_{z_c}(x)$ decays as $|x|^{-(d-2)}$ \cite{Hara08,HHS03}, and the constant term of order $V^{-3/4}= r^{-3d/4}$ dominates the Gaussian decay over most of the torus. This is the ``plateau'' phenomenon. On the complete graph, only the plateau term occurs for the two-point function at distinct points, as in \eqref{eq:G01-profile}. For $d=d_{\rm c}=8$, the conjecture is modified to include logarithmic corrections to the window scale $V^{-1/2}$, the plateau scale $V^{-3/4}$, and the susceptibility scale $V^{1/4}$, but with the identical profile $I_0$. \smallskip\noindent {\bf Self-avoiding walk:} Self-avoiding walk on the complete graph $\mathbb{K}_V$ is exactly solvable \cite{Slad20}. Let $c_{V,n}(0,1) = \prod_{j=1}^n (V-j)$ denote the number of $n$-step self-avoiding walks from $0$ to $1$ on $\mathbb{K}_V$. Let $S_{V,01}(p) = \sum_{n=1}^\infty c_{V,n}(0,1)(p/V)^n$ and let $\chi_V^{\rm SAW}(p) = 1+(V-1)S_{V,01}(p)$. It is proved in \cite{Slad20} (see also \cite[Appendix~B]{MPS23}) that, as $V \to \infty$, \begin{equation}\begin{aligned} S_{V,01}(1 + sV^{-1/2}) & \sim (2V)^{-1/2} I_1(-\sqrt{2} s) , \\ \chi_V^{\rm SAW}(1 + sV^{-1/2}) &\sim 2^{-1/2}V^{1/2} I_1(-\sqrt{2}s) . \end{aligned}\end{equation} In \cite{MPS23,PS25}, the same profile $I_1$ is conjectured to apply to the self-avoiding walk on $\T_r^d$ for $d \ge 4$, in the sense that the two-point function and susceptibility obey the analogue of \eqref{eq:LTLAplateau} with the right-hand sides replaced respectively by $b_d V^{-1/2} I_1(a_d s)$ and $ b_d V^{1/2} I_1(a_d s)$. The conjectured log corrections for $d=4$ are indicated in \cite[Section~1.6.3]{MPS23}. \smallskip\noindent {\bf Spin systems:} The plateau for spin systems in dimensions $d \ge d_{\rm c}=4$ is discussed in \cite{LPS25-universal,PS25,LPS25-Ising}, including rigorous results for a hierarchical $|\varphi|^4$ model and conjectures for spin systems on the torus. The relevant profile for $n$-component spin systems is \begin{equation} f_n(s) = \frac{\int_{\R^n} |x|^2 e^{-\frac 14 |x|^4 - \frac s2 |x|^2} dx} {n\int_{\R^n} e^{-\frac 14 |x|^4 - \frac s2 |x|^2} dx} = \frac{ I_{n+1} (s)}{n I_{n-1} (s)}. \end{equation} The profile $f_1$ has been proven to occur for the Ising model on the complete graph (Curie--Weiss model); a recent reference is \cite{BBE24}. As $n \to 0$, the profile $f_n(s)$ converges to $I_1(s)$, which is consistent with the conventional wisdom that the spin model with $n=0$ corresponds to the self-avoiding walk. \smallskip\noindent {\bf Percolation:} Percolation has been extensively studied both on infinite lattices \cite{Grim99} and on the complete graph (the Erd\H{o}s--R\'enyi random graph) \cite{JLR00}. This is a probabilistic model in which the cluster containing $0$ is a connected subgraph $A \ni 0$ with weight $p^{\abs A} (1-p)^{\abs{\del A}}$, where $\abs A$ denotes the number of edges in $A$, and $\del A$ denotes the set of edges which are not in $A$ but are incident to one or two vertices in $A$. On the complete graph, we divide $p$ by $V$ (not by $eV$ as in \eqref{eq:G01def}) to make the critical value $p=1$. Thus we define the \emph{two-point function} \begin{equation} \tau_{V,01}(p) = \P_{p/V}(0 \leftrightarrow 1) = \sum_{A \ni 0,1} \Bigl(\frac{p}{V} \Big)^{\abs A} \Bigl(1-\frac{p}{V} \Big)^{\abs {\del A}} \end{equation} and the \emph{susceptibility} (expected cluster size) $\chi_V \supperc(p) = 1 +(V-1)\tau_{V,01}(p)$. Our conjecture for an analogue of Theorem~\ref{thm:profile} for percolation on the complete graph is as follows. It involves the Brownian excursion $W^*$ of length $1$, and the moment generating function $\Psi(x) = \mathbb{E} \exp[ x\int_0^1 W^*(t)\mathrm dt] $ for the Brownian excursion area. \begin{conjecture} \label{conj:perc} For $s\in \R$, let \begin{equation} \label{eq:fperc} f_{\rm perc}(s) = \int_0^\infty x^2 \mathrm d\sigma_s, \qquad \mathrm d \sigma_s = \frac 1 { \sqrt{2\pi} } x^{-5/2} \Psi(x^{3/2}) e^{-\frac 1 6 x^3 + \frac s 2 x^2 - \frac{ s^2} 2 x} \mathrm dx . \end{equation} Then, for some $a,b>0$, as $V\to\infty$ we have \begin{equation} \label{eq:perc_conj} \begin{aligned} \tau_{V,01}(1+sV^{-1/3}) & \sim b V^{-2/3} f_{\rm perc}(as), \\ \chi_{V} \supperc(1+sV^{-1/3}) & \sim b V^{1/3} f_{\rm perc}(as). \end{aligned} \end{equation} \end{conjecture} Note the different powers of $V$ in \eqref{eq:perc_conj} compared to \eqref{eq:LTLAplateau} and \eqref{eq:G01-profile}--\eqref{eq:LTK-profile}. The powers of $V$ in \eqref{eq:perc_conj} are well-known, but to our knowledge the occurrence of the profile has not been proved. On the torus $\T_r^d$ with $d>6$, the powers $V^{-1/3}, V^{-2/3}, V^{1/3}$ are proved in \cite{HMS23}, and the role of $f_{\rm perc}$ was first conjectured in \cite[Appendix~C]{LPS25-Ising}. The origin of the conjecture is as follows. The properly rescaled cluster size (without expectation) is known to converge in distribution to a random variable described by the Brownian excursion \cite{Aldo97}, and the limiting random variable is characterised by a point process \cite{JS07}. The measure $\sigma_s$ is the intensity of the point process and is found in \cite[Theorem~4.1]{JS07}. The point process describes cluster sizes, in the sense that \begin{equation} n^{-2k/3} \sum_i \abs{ C_i }^k \Rightarrow \int_0^\infty x^k \mathrm d\sigma_s \qquad (k \ge 2) \end{equation} in distribution \cite{Aldo97}. The case $k=2$ case corresponds to $\chi_V\supperc$ and identifies $f_{\rm perc}(s)$. \section{Proof for trees} We begin with an elementary lemma. \begin{lemma} \label{lem:b} Let $\gamma \ge 0$, $\kappa > 0$, and $\lambda \in \R$. There is a constant $C_{\kappa, \lambda}>0$ such that \begin{align} \label{eq:b_bound} \sum_{n=b \sqrt V}^{V} \frac{ 1 }{n^\gamma} e^{- \kappa n^2/V} e^{\lambda n/\sqrt{V}} \le C_{\kappa,\lambda} b^{-\gamma} V^{ (1 - \gamma) /2} . \end{align} for all $V$ and for all $b$ sufficiently large (depending on $\kappa,\lambda$). \end{lemma} \begin{proof} Since $n \ge b\sqrt V$ and $\gamma \ge 0$, the left-hand side of \eqref{eq:b_bound} is bounded by \begin{align} \frac{1}{b^\gamma V^{\gamma / 2}} \sum_{n=b \sqrt V}^{\infty} e^{- \kappa n^2/V} e^{\lambda n/\sqrt{V}} . \end{align} For $b$ sufficiently large (depending on $\kappa,\lambda$), the summand above is monotone decreasing in $n$, so we can bound the sum by the integral \begin{align} \int_{b \sqrt V - 1}^\infty e^{- \kappa ( y / \sqrt V )^2} e^{\lambda( y/\sqrt{V}) } \mathrm dy \le C_{\kappa, \lambda} \sqrt V , \end{align} and the desired result follows. \end{proof} \begin{proof}[Proof of Theorem~\ref{thm:profile} for trees] We use \eqref{eq:chit} and drop the superscript $t$. Fix $s\in \R$. For $p = 1 + s V^{-1/2}$, by combining $V^{-(n-1)}$ with the binomial coefficient, we have \begin{equation} \label{eq:chit-sum} \chi_V(1+sV^{-1/2}) = \sum_{n=1}^{V} \Bigl(\prod_{j=1}^{n-1}( 1 - \frac jV )\Bigr) \frac 1 {(n-1)!} \frac{n^{n-1}}{e^{n-1}} \Big(1+\frac s {\sqrt{V}} \Big)^{n-1}. \end{equation} Let $0 < a < 1 < b < \infty$. We divide the sum over $n$ into three parts $\chi_V \supk 1, \chi_V \supk 2, \chi_V \supk 3$, which respectively sum over $n$ in the intervals $[1, a \sqrt V)$, $[ a \sqrt V, b \sqrt V]$, $(b \sqrt V , V]$. We will prove that \begin{align} \label{eq:claimLTK13} \chi_V \supk 1 \lesssim e^{a\abs s} ( 1 + a^{1/2} V^{1/4}), \qquad \chi_V \supk 3 \le C_{\abs s} b^{-1/2} V^{1/4} \end{align} for all $a>0$ and all $b$ sufficiently large, and that \begin{align} \label{eq:claimLTK2} \lim_{V\to \infty} V^{-1/4} \chi_V^{(2)} = \int_a^b f(x) \mathrm d x , \qquad f(x) = \frac{e}{\sqrt{2\pi}}e^{-x^2/2}\frac{1}{\sqrt{x}} e^{sx} \end{align} for all $a,b$. These claims imply that \begin{align} \int_a^b f(x) \mathrm d x \le \liminf_{V\to \infty} \frac{ \chi_V }{V^{1/4}} \le \limsup_{V\to \infty} \frac{ \chi_V }{V^{1/4}} \le C e^{a\abs s}a^{1/2}+ \int_a^b f(x) \mathrm d x + C_{|s|} b^{-1/2} \end{align} for all $a > 0$ and all $b$ sufficiently large. Since $\chi_V$ does not depend on $a$ or $b$, by taking the limits $a\to 0$, $b\to \infty$, we obtain $\lim_{V\to \infty} V^{-1/4} \chi_V = \int_0^\infty f$, which is the desired result \eqref{eq:LTK-profile}. It remains to prove the claims \eqref{eq:claimLTK13}--\eqref{eq:claimLTK2}. Let \begin{equation} b_{n} = \frac{n^{n-1}}{(n-1)!e^{n-1}} , \end{equation} which obeys $b_{n} \lesssim 1/\sqrt{n}$, by Stirling's formula. Using this in the sum for $\chi_V \supk 1$, and using $1 + s/\sqrt V \le e^{ \abs s / \sqrt V}$, we get \begin{equation} \label{eq:pf_tree_1} \chi_V^{(1)} \lesssim \sum_{n=1}^{a \sqrt V} 1 \frac{1}{\sqrt {n} } e^{ |s| n / \sqrt V} \le e^{a\abs s} \sum_{n=1}^{a \sqrt V} \frac{1}{\sqrt {n} } \lesssim e^{a\abs s} ( 1 + a^{1/2} V^{1/4} ) , \end{equation} as claimed. For $\chi_V \supk 3$, we also need a bound on the product over $j$. Using $\log(1+x) \le x$, we have \begin{align} \prod_{j=1}^{n-1}\big( 1 - \frac jV \big) = \exp\Bigl\{\sum_{j=1}^{n-1} \log(1-\frac j V)\Bigr\} \le \exp\Bigl\{ - \frac 1 V \sum_{j=1}^{n-1} j \Bigr\} = \exp \Bigl\{ - \frac { n (n-1)} {2V} \Bigr\} . \end{align} By Lemma~\ref{lem:b} with $\gamma = \kappa = \half$ and $\lambda=|s|$, this implies that, for all $b$ sufficiently large, \begin{equation} \label{eq:pf_tree_3} \chi_V^{(3)} \lesssim \sum_{n=b \sqrt V}^{V} e^{-n^2/2V} e^{n/2V} \frac{1}{\sqrt{ n}} e^{|s|n/\sqrt{V}} \lesssim e^{1/2} b^{-1/2} V^{1/4}. \end{equation} Finally, for $\chi_V \supk 2$ we fix $a,b$ and use the asymptotic formulas \begin{align} \Big( 1 + \frac s {\sqrt V} \Big)^{n-1} &= \exp \Big\{ (n-1) \log( 1 + \frac s {\sqrt V} ) \Big\} = e^{s n/ \sqrt V} \Big[ 1 + O\Big(\frac 1 { \sqrt V }\Big) + O\Big(\frac n V\Big) \Big] , \\ \prod_{j=1}^{n-1}\big( 1 - \frac jV \big) &= \exp\Big\{\sum_{j=1}^{n-1} \log(1-\frac j V)\Big\} = e^{-n^2/2V} \Big[ 1 +O\Big(\frac n V\Big) + O\Big(\frac {n^3} {V^2}\Big)\Big] , \end{align} which follow from Taylor expansion of the logarithm. Since $n \in [a\sqrt V, b\sqrt V]$, the above, together with the fact that $b_{n} = \frac{ e }{\sqrt{2\pi n}}[1+O(1/n)]$ by Stirling's formula, give \begin{align} \chi_V^{(2)} & = \sum_{n=a \sqrt V}^{b \sqrt V} e^{-n^2 / 2V} \frac{e}{\sqrt{2\pi n}} e^{sn/\sqrt{V}} \Big[ 1 + O\Big(\frac 1 {\sqrt V}\Big) \Big] . \end{align} The desired limit then follows from the observations that the leading term of $V^{-1/4} \chi_V \supk 2$ is a Riemann sum for the integral $\int_a^b f$ with mesh size $V^{-1/2}$. \end{proof} \begin{proof}[Proof of Theorem~\ref{thm:G0} for trees] We use \eqref{eq:G0} and again drop the superscript $t$. Fix $s\ge 0$. Let $p_V$ be a sequence with $p_V \le 1 + s V^{-1/2}$ and $p_V \to p \in [0,1]$. Similarly to \eqref{eq:chit-sum} and with an additional factor of $n$ in the denominator, \begin{equation} \label{eq:G0t-sum} G_{V,0}(p_V) = \sum_{n=1}^{V} \Big(\prod_{j=1}^{n-1}( 1 - \frac jV )\Big) \frac 1 {n!} \frac{n^{n-1}}{e^{n-1}} p_V^{n-1}. \end{equation} Let $N, b\ge 1$. We divide the sum over $n$ into three parts $G_V \supk 1, G_V \supk 2, G_V \supk 3$, which respectively sum over $n$ in the intervals $[1, N]$, $( N, b \sqrt V]$, $(b \sqrt V , V]$. For a fixed $N$, we immediately get \begin{align} \label{eq:G0N} \lim_{V\to \infty} G^{(1)}_{V}(p_V) = \sum_{n=1}^N \frac{1}{n!} \frac{n^{n-1}}{e^{n-1}}p^{n-1}, \end{align} which dominates the sum. Indeed, using monotonicity of the generating function, for $G_V \supk 2$ we can proceed as in \eqref{eq:pf_tree_1} to bound \begin{align} G_V \supk 2 (p_V) \le G_V \supk 2 (1 + s V^{-1/2}) \le e^{bs} \sum_{n=N}^{b\sqrt V} \frac 1 {n^{3/2}} \lesssim \frac{ e^{bs} } { \sqrt N} . \end{align} For $G_V \supk 3$, we can argue as in \eqref{eq:pf_tree_3} but with an additional factor $n$ in the denominator, and use Lemma~\ref{lem:b} with $\gamma = \frac 3 2$ to get $G_V \supk 3 (p_V) \lesssim b^{-3/2}V^{-1/4}$ for $b$ sufficiently large. Together, we obtain \begin{align} \sum_{n=1}^N \frac{1}{n!} \frac{n^{n-1}}{e^{n-1}}p^{n-1} \le \liminf_{V\to \infty} G_{V,0} \le \limsup_{V\to \infty} G_{V,0} \le \sum_{n=1}^N \frac{1}{n!} \frac{n^{n-1}}{e^{n-1}}p^{n-1} + \frac{ C e^{bs} } { \sqrt N} \end{align} for all $N \ge 1$ and all $b$ sufficiently large. Since $G_{V,0}$ does not depend on $N$, we can take the limit $N \to \infty$ to conclude the desired result \eqref{eq:G0lim}. \end{proof} \section{Proof for connected subgraphs} \subsection{Bound on $C(n,n+k)$} We use the asymptotic formula for $C(n,n+k)$ proved in \cite{BCM90}. We follow the notation in \cite{BCM90} and write \begin{align} x = 1 + \frac k n . \end{align} For $x > 1$, we define the function $y = y(x) \in (0,1)$ implicitly by \begin{align} \label{eq:Taylor_xy} x = \frac{1}{2y} \log \biggl( \frac { 1+ y } { 1 - y } \biggr) = \frac{1}{y}\arctanh y = \sum_{m = 0}^\infty \frac{ y^{2m} } { 2m+1 } , \end{align} and we define the functions $\varphi(x)$, $a(x)$ by \begin{gather} \label{eq:phi} e^{\varphi(x)} = \frac{ 2 e^{-x} y^{1-x} }{ \sqrt{ 1 - y^2} } , \\ a(x) = x(x+1)(1-y) + \log( 1 - x + xy) - \half \log( 1 - x + x y^2 ) . \end{gather} Both $\varphi$ and $a$ extend continuously to $x=1$ by defining $y^{1-x} = 1$ at $x=1$ and defining $a(1) = 2 + \half \log\frac 3 2$. Let $N = \binom n 2$. It is proved in \cite[Corollary~1]{BCM90} that there are constants $w_k = 1 + O(1/k)$ for which \begin{align} \label{eq:BCM} C(n,n+k) = w_k \binom N {n+k} e^{n \varphi(x)} e^{a(x)} \bigg[ 1 + O\bigg( \frac { (k+1)^{1/16} }{ n^{9/50} } \bigg) \bigg] \end{align} uniformly in $0 \le k \le N - n$. The constants $w_k$ are related to Wright's constants for the the asymptotics of $C(n,n+k)$ with $k$ fixed \cite{Wrig77}, and they are related to the Brownian excursion area \cite{Spen97}. We will simply bound $w_k$ by a constant. The next lemma gives estimates for $\varphi(x)$ and $a(x)$. \begin{lemma} \label{lem:aphi} Let $x\ge 1$. \begin{enumerate} \item [(i)] The function $a(x)$ is bounded. \item [(ii)] Let $t = \sqrt{3e}$ and $y = y(x)$. Then \begin{equation} \label{eq:phi_bdd} e^{\varphi(x) } \le \frac 2 e \exp\Bigl\{ - \frac 1 3 y^2 \log \frac y t \Bigr\} , \end{equation} and the right-hand side is monotonically increasing for $0 < y \le t/\sqrt e$. \end{enumerate} \end{lemma} By considering the limit $x\to \infty$ ($y\to 1$), we expect that the inequality \eqref{eq:phi_bdd} becomes optimal with $t = (e/2)^3 \approx 2.51$, but we do not pursue this. The weaker version with $t= \sqrt{3e}$ is sufficient for our purposes, but to show the role of $t$ we keep it in our formulas. \begin{proof} (i) The function $a(x)$ is continuous on $[1,\infty)$ by definition, and it satisfies $\abs{ a(x) } \lesssim x^2 (1-y) \sim 2x^2 e^{-2x}$ as $x\to \infty$ by \cite[Lemma~3.2]{BCM90}, so it is bounded. \smallskip \noindent (ii) By the definitions of $\varphi(x)$ and $x$, and by the Taylor series for $\log (1-y^2)$, \begin{align} \varphi(x) - \log \frac 2 e &= (1-x) ( 1 + \log y ) - \half \log ( 1 - y^2 ) \nl &= - \sum_{m=1}^\infty \frac{ y^{2m} } { 2m+1 } ( 1 + \log y ) + \sum_{m=1}^\infty \frac{ y^{2m} } { 2m } \nl &= - \frac 1 3 y^2 \log y + \frac 1 6 y^2 + \sum_{m=2}^\infty \frac{ y^{2m} }{2m+1} \Bigl(-\log y + \frac 1 {2m} \Bigr) . \end{align} We bound the series in the last line by a quadratic function, term by term. For any $m\ge 2$, by calculus, \begin{equation} \max_{0 \le y \le 1} \bigl[ y^{2m-2} (-2m \log y + 1) \bigr] = \frac{2m}{2m-2} e^{-1/m} . \end{equation} Then, with $K = \max_{m\ge 2} \{ \frac{2m}{2m-2} e^{-1/m} \} = 2 e^{-1/2}$, by \cite[0.234.8]{GR07} we have \begin{align} \sum_{m=2}^\infty \frac{ y^{2m} }{2m+1} \Bigl(-\log y + \frac 1 {2m} \Bigr) \le \sum_{m=2}^\infty \frac{ K y^2 }{(2m+1)(2m)} = (1 - \log 2 - \frac 1 6) K y^2 . \end{align} Therefore, \begin{equation} \varphi(x) - \log \frac 2 e \le - \frac 1 3 y^2 \log y + \biggl[ \frac 1 6 + (1 - \log 2 - \frac 1 6)K \biggr] y^2 . \end{equation} This implies \eqref{eq:phi_bdd} with any $t$ that obeys $ \frac 1 3 \log t \ge \frac 1 6 + (1 - \log 2 - \frac 1 6)K \approx 0.3367$. In particular, we can take any $t \ge 2.75$, including $t = \sqrt{3e} \approx 2.85$. Monotonicity of the upper bound in $0 < y \le t/\sqrt e$ is another calculus exercise. \end{proof} We now restate and prove Proposition~\ref{prop:Cnk-intro}. \begin{proposition} \label{prop:Cnk} Let $n \ge 3$, $N = \binom n 2$, and $t=\sqrt{3e}$. For $0 \le \frac k n \le \frac {t^2}{3e}$, we have \begin{align} \label{eq:Cnk} C(n, n+k) \lesssim \binom N {n+k} \bigg( \frac 2 e \bigg)^n \bigg( \frac{ t^2 n }{ 3 k }\bigg)^{k/2} . \end{align} \end{proposition} \begin{proof} We use the asymptotic formula \eqref{eq:BCM}, and use that $w_k = 1 + O(1/k)$ is bounded. The error term in \eqref{eq:BCM} is bounded by a constant since $k \le N-n \le n^2$. The factor $e^{a(x)}$ is also bounded by a constant, by Lemma~\ref{lem:aphi}(i). We therefore only need to estimate $e^{n\varphi(x)}$. Since $0 \le \frac k n \le \frac {t^2}{3e}$ and $x = 1 + \frac k n$, by \eqref{eq:Taylor_xy} we have $y(x) \le \sqrt{3(x-1)} = \sqrt{ 3k / n } \le t / \sqrt e$, so Lemma~\ref{lem:aphi}(ii) gives \begin{align} e^{\varphi(x)} \le \frac 2 e \exp\Bigl\{ - \frac 1 3 y^2 \log \frac y t \Bigr\} \le \frac 2 e \exp\Bigl\{ - \frac {x-1}2 \log \frac {3(x-1)} {t^2} \Bigr\} = \frac 2 e \bigg( \frac {t^2 n}{3k} \bigg)^{k/2n} . \end{align} The desired result then follows by inserting the above into \eqref{eq:BCM}. \end{proof} For larger $\frac kn$ we simply use the fact that $C(n,n+k)$ is less than the total number of graphs (connected or not) on $n$ vertices with $n+k$ edges, which is $\binom{N}{n+k}$. For all $n \ge 2$ and $k \ge -1$, we have \begin{equation} \label{eq:C_crude} C(n,n+k) \le \binom{N}{n+k} \le \frac{N^{n+k}}{(n+k)!}. \end{equation} \subsection{Bound on the surplus generating function} We now prove useful bounds on the surplus generating function defined in \eqref{eq:def_S}: \begin{align} S(n,z) & = \sum_{\ell= 1}^\infty C(n,n-1+\ell) z^{\ell} = \sum_{k=0}^\infty C(n,n+k) z^{k+1}. \end{align} The terms in the series are zero unless $k \le \binom{n} 2 - n$. The goal is to prove that $S(n,z)$ is small relative to the number of trees $C(n,n-1) = n^{n-2}$. We do this by decomposing the series into two parts corresponding to sparse and dense graphs. We define \begin{align} A(n,z) & = \frac 1 { n^{n-2} } \sum_{k=0}^{n} C(n,n+k) z^{k+1} , \qquad B(n,z) = \frac 1 { n^{n-2} } \sum_{k= \frac 12 n}^\infty C(n,n+k) z^{k+1} , \end{align} so that \begin{equation} \label{eq:SAB} S(n,z) \le n^{n-2} \big( A(n,z) + B(n,z) \big) . \end{equation} \begin{lemma}[Sparse connected graphs] \label{lemma:sparse} Let $n \ge 3$, $z \ge 0$, and $t=\sqrt{3e}$. \begin{enumerate} \item[(i)] If $n^{3/2} z \le b$, then $A(n,z) \le C_b n^{3/2} z$ for some $C_b > 0$. \item[(ii)] If $\eps > 0$, then \begin{equation} A(n,z) \le C_\eps \exp\Bigl\{ \big(\frac{1}{24}+\eps \big) e t^2 z^2 n^3 \Bigr\} \end{equation} for some $C_\eps > 0$. \end{enumerate} \end{lemma} \begin{proof} Since $\frac {t^2}{3e} = 1$, we can apply Proposition~\ref{prop:Cnk} to estimate $C(n,n+k)$. For the binomial coefficient in \eqref{eq:Cnk}, we use Stirling's formula, $n+k \ge n$, and $N = \binom n 2 = \half n(n-1)$ to see that \begin{align} \binom N {n+k} \le \frac{ N^{n+k} }{ (n+k)! } \lesssim \frac 1 { \sqrt{n+k} } \bigg( \frac {eN}{n+k} \bigg)^{n+k} \le \frac{1}{\sqrt{n}} \bigg( \frac {e (n-1) }{2} \bigg)^{n+k}. \end{align} Then, by extending the sum to run over all $k \ge 0$, we obtain \begin{align} \frac 1 z A(n,z) &= \frac 1 {n^{n-2}} \sum_{k=0}^{n} C(n,n+k) z^{k} \nl& \lesssim \frac{1}{n^{n-2}\sqrt n} \sum_{k=0}^{n} \bigg( \frac {e n }{2} \bigg)^{n+k} \bigg( \frac 2 e \bigg)^n \bigg( \frac{ t^2 n }{ 3 k }\bigg)^{k/2} z^k \le n^{3/2} \sum_{k=0}^{\infty} \frac 1 {k^{k/2}} \biggl( \frac{etn^{3/2}z}{2\sqrt{3}} \bigg)^k , \end{align} which converges for all $z>0$. \smallskip \noindent (i) If $n^{3/2}z \le b$ then the series on the right-hand side is bounded by a constant $C_b$, as required. \smallskip \noindent (ii) We set $x = \frac{etn^{3/2}z}{2\sqrt{3}}$ and use the asymptotic formula \cite[Lemma~4.1(i)]{JC04} \begin{equation} \label{eq:kk_asymp} \sum_{k=0}^{\infty} \frac{1}{k^{k/2}} x^k \sim (4\pi e^{-1})^{1/2} x e^{\frac{1}{2e}x^2} \qquad \text{as $x \to\infty$} \end{equation} to get a bound for large $x$. For smaller $x\ge0$, we simply bound by a constant. The desired result then follows by absorbing the prefactor of \eqref{eq:kk_asymp} and another factor of $n^{3/2}z = \const\,x$ into the exponential. This completes the proof. \end{proof} \begin{lemma}[Dense connected graphs] \label{lemma:dense} Let $n\ge 3$ and $z\le \frac{3}{en}$. Then $B(n,z)\lesssim z^2$. \end{lemma} \begin{proof} Let $\nu = \lfloor n/2 \rfloor \ge 1$. The crude bound \eqref{eq:C_crude} gives \begin{align} \label{eq:B_pf} B(n,z) \le \frac 1 {n^{n-2}} \sum_{k = \nu}^{\infty} \frac{N^{n+k}}{(n+k)!} z^{k+1} & = \frac {z^{1+\nu}} {n^{n-2}} \frac{N^{n+\nu}}{(n+\nu)!} \sum_{k = \nu}^{\infty} (Nz)^{k-\nu}\frac{(n+\nu)!}{(n+k)!} \nnb & \le \frac {z^{1+\nu}} {n^{n-2}} \frac{N^{n+\nu}}{(n+\nu)!} \sum_{m=0}^{\infty} \biggl( \frac{Nz}{n+\nu} \bigg)^{m} , \end{align} since $(n+k)! \ge (n+\nu)! (n+ \nu)^{k - \nu}$. Note that by our hypothesis \begin{align} \frac { Nz } { n+ \nu } \le \frac { \half n (n-1) z } { n + (\half n - \half)} < \frac { n (n-1)}{3n-3} z = \frac 1 3 {nz} \le \frac 1 e , \end{align} so the geometric series in \eqref{eq:B_pf} is bounded by a constant. For the prefactor in \eqref{eq:B_pf}, since $1+\nu \ge 2$ and $z\le \frac{3}{en}$ by hypothesis, we have \begin{equation} z^{1+\nu} = z^2 z^{ \nu - 1} \le \frac e 3 z^2 \Big( \frac{3}{e}\Big)^{\nu } n^{1 - \nu} . \end{equation} Also, using $N = \half n(n-1)$ and Stirling's formula, \begin{equation} \frac{ N^{n+\nu} }{(n+\nu)!} \lesssim \frac{ [\half n (n-1) ]^{n+\nu} } { \sqrt n ( \frac{n+\nu}e )^{n+\nu} } = \frac {n^{n+\nu} } {\sqrt n } \Big( \frac{e}{2}\Big)^{n+\nu} \Big( \frac{n-1}{n+\nu} \Big)^{n+\nu} . \end{equation} Since $\frac{n-1}{n+\nu} \le \frac 2 3$, together we obtain \begin{align} \frac {z^{1+\nu}} {n^{n-2}} \frac{N^{n+\nu}}{(n+\nu)!} \lesssim z^2 \Big( \frac{3}{e}\Big)^{\nu } n^{5/2} \Big( \frac{e}{3}\Big)^{n+\nu} = z^2 n^{5/2} \Big( \frac{e}{3}\Big)^{n} . \end{align} It follows that $B(n,z) \lesssim z^2 \sup_{n \ge 3} \{ n^{5/2} (e/3)^n \} , $ and the proof is complete since $e < 3$. \end{proof} \subsection{Proof for connected subgraphs} \begin{proof}[Proof of Theorem~\ref{thm:profile} for connected subgraphs] Fix $s\in \R$ and let $p = 1 + sV^{-1/2}$. We assume $V$ is large enough so that $p \le 3$. As discussed around \eqref{eq:Deltabd}, it suffices to prove \begin{align} \label{eq:animal_goal} \lim_{V\to\infty} V^{-1/4}\Delta_V(1 + sV^{-1/2}) &= 0. \end{align} By the definition of $\Delta_V$ in \eqref{eq:chiDelta} and by \eqref{eq:SAB}, \begin{align} \label{eq:diff} \Delta_{V}(p) &\le \sum_{n=3}^{V} \binom{V-1}{n-1} \Big( \frac{p}{eV} \Big)^{n-1}n^{n-1} \Big( A \big(n, \frac{p}{eV} \big) + B\big(n, \frac{p}{eV}\big) \Big) . \end{align} We write the part of the upper bound \eqref{eq:diff} that contains $A,B$ as $\Delta_V \supk A, \Delta_V \supk B$ respectively. We start with $\Delta_V \supk B$ and use Lemma~\ref{lemma:dense} to bound $B$. Let $z = p/(eV)$. Since $n \le V$ and $p \le 3$ (for large $V$), we have $nz \le p / e \le 3/e$, so Lemma~\ref{lemma:dense} applies and gives $B(n,z) \lesssim V^{-2}$. Then, by comparing to $\chi_V^t$ in \eqref{eq:chit}, we find that \begin{equation} \label{eq:DeltaB} \Delta_V \supk B(p) \lesssim V^{-2} \chi_V^t(p). \end{equation} For $\Delta_V \supk A$, we claim that if both $b \ge 1$ and $V$ are sufficiently large, then \begin{align} \label{eq:DeltaA} \Delta_V \supk A(p) \le C_{b} V^{-1/4} \chi_V^t(p) + C_s b^{-1/2} V^{1/4} . \end{align} Since we already know that $V^{-1/4} \chi_V^t$ converges, \eqref{eq:DeltaA} implies that \begin{align} 0 \le \limsup_{V\to \infty} \frac{ \Delta_V}{V^{1/4}} \le \limsup_{V\to \infty} \frac{ \Delta_V \supk A + \Delta_V \supk B }{V^{1/4}} \le C_{s} b^{-1/2} \end{align} for all $b$ sufficiently large. But $\Delta_V$ does not depend on $b$, so by taking the limit $b\to \infty$, we obtain \eqref{eq:animal_goal}, as desired. It remains to prove \eqref{eq:DeltaA}. We divide the sum defining $\Delta_V\supk A$ into two parts $\Delta_V \supk 1, \Delta_V \supk 2$, which sum over $n$ in the intervals $[3, b \sqrt V ]$, $(b \sqrt V , V]$ respectively. For $\Delta_V \supk 1$, we have $n \le bV^{1/2}$ so $n^{3/2} z = n^{3/2}p/(eV) \le c_{b} V^{-1/4}$, so we can apply Lemma~\ref{lemma:sparse}(i) to obtain $A(n,z) \le C_b' n^{3/2} z \le C_b V^{-1/4}$. With the formula for $\chi^t_V$ in \eqref{eq:chit}, this gives \begin{align} \label{eq:pf_A1} \Delta_V \supk 1 & \le C_b V^{-1/4}\sum_{n=3}^{b\sqrt V} \binom{V-1}{n-1} \Big( \frac{p}{eV} \Big)^{n-1}n^{n-1} \le C_b V^{-1/4}\chi_V^t(p). \end{align} This provides the first term on the right-hand side of \eqref{eq:DeltaA}. For $\Delta_V \supk 2$, we use $z=p/(eV)$ and Lemma~\ref{lemma:sparse}(ii) to see that \begin{equation} A(n,z) \le C_\eps \exp\Bigl\{ \big(\frac{1}{24}+\eps \big) e\inv t^2 p^2 n^{3} / V^2 \Big\}, \end{equation} Since $t = \sqrt{3e}$, we have $\frac 1 {24} e\inv t^2 = \frac1 8$. By choosing $\eps$ small, and by using $p = 1 + s V^{-1/2} \to 1$ as $V\to \infty$, for $V$ sufficiently large (depending only on $\eps,s$) we have \begin{align} A(n,z) \le C_\eps \exp \Bigl\{ \frac 1 5 \frac { n^3 }{V^2 } \Bigr\} . \end{align} For these values of $V$, we thus have \begin{align} \Delta_V \supk 2(1 + s V^{-1/2}) \lesssim \sum_{n=b\sqrt V }^{V} \binom{V-1}{n-1} \bigg( \frac{1 + s V^{-1/2}}{eV} \bigg)^{n-1} n^{n-1} \exp \Bigl\{ \frac 1 5 \frac { n^3 }{V^2 } \Bigr\} . \end{align} We now follow the argument used for $\chi_V\supk 3$ of trees in the paragraph containing \eqref{eq:pf_tree_3}. Using $\frac { n^3 }{V^2 } \le \frac { n^2 }{V }$ and Lemma~\ref{lem:b} with $\gamma = \half$ and $\kappa = \half - \frac 15 > 0$, we find that if $b$ is sufficiently large then \begin{align} \label{eq:Delta2} \Delta_V \supk 2(1 + s V^{-1/2}) \lesssim \sum_{n=b \sqrt V}^{V} e^{- (\frac 1 2 - \frac 1 5) n^2 / V} \frac{1}{\sqrt{ n}} e^{|s|n/\sqrt{V}} \le C_{\abs s} b^{-1/2} V^{1/4}. \end{align} This gives the second term on the right-hand side of \eqref{eq:DeltaA} and concludes the proof. \end{proof} \begin{proof}[Proof of Theorem~\ref{thm:G0} for connected subgraphs] As noted at \eqref{eq:Delta0bd}, it suffices to prove that $\Delta_{V,0}(1+sV^{-1/2}) \to 0$ for all $s \ge 0$. We write $p = 1 + sV^{-1/2}$ and follow the proof of Theorem~\ref{thm:profile}. Compared to $\Delta_V$ for the susceptibility, there is one less factor $n$ in $\Delta_{V,0}$, so instead of \eqref{eq:diff} we now have \begin{align} \label{eq:diffG} \Delta_{V,0}(p) &\le \sum_{n=3}^{V} \binom{V-1}{n-1} \Big( \frac{p}{eV} \Big)^{n-1}n^{n-2} \Big( A \big(n, \frac{p}{eV} \big) + B\big(n, \frac{p}{eV}\big) \Big) . \end{align} As in \eqref{eq:DeltaB}, the contribution from $B$ obeys \begin{equation} \label{eq:diffG} \Delta_{V,0}^{(B)}(p) \lesssim V^{-2}G_{V,0}^t(p) \lesssim V^{-2}, \end{equation} so it vanishes in the limit. For $\Delta_{V,0}^{(1)}(p)$, the same bound on $A$ that was used in \eqref{eq:pf_A1} now gives $\Delta_{V,0}^{(1)}(p) \le C_b V^{-1/4} G_{V,0}(p)$. For $\Delta_{V,0}^{(2)}(p)$, in \eqref{eq:Delta2} we now have an extra factor $n$ in the denominator, so Lemma~\ref{lem:b} with $\gamma = \frac 3 2$ gives \begin{align} \Delta_{V,0} \supk 2(1 + s V^{-1/2}) \lesssim \sum_{n=b \sqrt V}^{V} e^{- (\frac 1 2 - \frac 1 5) n^2 / V} \frac{1}{n^{3/2}} e^{|s|n/\sqrt{V}} \le C_{\abs s} b^{-3/2} V^{-1/4}. \end{align} Altogether, we have $\Delta_{V,0}(p) \lesssim V^{-1/4} \to 0$, and the proof is complete. \end{proof} \section*{Acknowledgements} The work of both authors was supported in part by NSERC of Canada. \begin{thebibliography}{10} \bibitem{Aldo97} D.~Aldous. \newblock Brownian excursions, critical random graphs and the multiplicative coalescent. \newblock {\em Ann. Probab.}, {\bf 25}:812--854, (1997). \bibitem{BBE24} Y.~Barhoumi-Andr\'{e}ani, M.~Butzek, and P.~Eichelsbacher. \newblock A surrogate by exchangeability approach to the {Curie}--{Weiss} model. \newblock {\em Electron. J. Probab.}, {\bf 29}:1--51, (2024). \bibitem{BCM90} E.A. Bender, E.R. Canfield, and B.D. McKay. \newblock The asymptotic number of labeled connected graphs with a given number of vertices and edges. \newblock {\em Random Struct.\ Alg.}, {\bf 1}:127--169, (1990). \bibitem{CFHP23} M.~Cabezas, A.~Fribergh, M.~Holmes, and E.~Perkins. \newblock Historical lattice trees. \newblock {\em Commun. Math. Phys.}, {\bf 401}:435--496, (2023). \bibitem{CGHJK96} R.M. Corless, G.H. Gonnet, D.E.G. Hare, D.J. Jeffrey, and D.E. Knuth. \newblock On the {Lambert} ${W}$ function. \newblock {\em Adv. Comput. Math.}, {\bf 5}:329--359, (1996). \bibitem{DS98} E.~Derbez and G.~Slade. \newblock The scaling limit of lattice trees in high dimensions. \newblock {\em Commun.\ Math.\ Phys.}, {\bf 193}:69--104, (1998). \bibitem{GPSW94} D.S. Gaunt, P.J. Peard, C.E. Soteros, and S.G. Whittington. \newblock Relationships between growth constants for animals and trees. \newblock {\em J. Phys. A: Math. Gen.}, {\bf 27}:7343--7351, (1994). \bibitem{GR07} I.S. Gradshteyn and I.M. Ryzhik. \newblock {\em Table of Integrals, Series and Products}. \newblock Academic Press, New York, 7th edition, (2007). \bibitem{Grim99} G.~Grimmett. \newblock {\em Percolation}. \newblock Springer, Berlin, 2nd edition, (1999). \bibitem{Hara08} T.~Hara. \newblock Decay of correlations in nearest-neighbor self-avoiding walk, percolation, lattice trees and animals. \newblock {\em Ann. Probab.}, {\bf 36}:530--593, (2008). \bibitem{HHS03} T.~Hara, R.~van~der Hofstad, and G.~Slade. \newblock Critical two-point functions and the lace expansion for spread-out high-dimensional percolation and related models. \newblock {\em Ann. Probab.}, {\bf 31}:349--408, (2003). \bibitem{HS90b} T.~Hara and G.~Slade. \newblock On the upper critical dimension of lattice trees and lattice animals. \newblock {\em J. Stat. Phys.}, {\bf 59}:1469--1510, (1990). \bibitem{HS92c} T.~Hara and G.~Slade. \newblock The number and size of branched polymers in high dimensions. \newblock {\em J. Stat. Phys.}, {\bf 67}:1009--1038, (1992). \bibitem{HMS23} T.~Hutchcroft, E.~Michta, and G.~Slade. \newblock High-dimensional near-critical percolation and the torus plateau. \newblock {\em Ann. Probab.}, {\bf 51}:580--625, (2023). \bibitem{Jans15} E.~J. Janse~van Rensburg. \newblock {\em The Statistical Mechanics of Interacting Walks, Polygons, Animals and Vesicles}. \newblock Oxford University Press, Oxford, 2nd edition, (2015). \bibitem{JC04} S.~Janson and P.~Chassaing. \newblock The center of mass of the {ISE} and the {Wiener} index of trees. \newblock {\em Electron. Commun. Probab.}, {\bf 9}:178--187, (2004). \bibitem{JLR00} S.~Janson, T.~{\L}uczak, and A.~Ruci\'{n}ski. \newblock {\em Random Graphs}. \newblock John Wiley and Sons, New York, (2000). \bibitem{JS07} S.~Janson and J.~Spencer. \newblock A point process describing the component sizes in the critical window of the random graph evolution. \newblock {\em Combin. Probab. Comput.}, {\bf 16}:631--658, (2007). \bibitem{KS24} N.~Kawamoto and A.~Sakai. \newblock Spread-out limit of the critical points for lattice trees and lattice animals in dimensions $d > 8$. \newblock {\em Combin. Probab. Comput.}, {\bf 33}:238--269, (2024). \bibitem{LPS25-Ising} Y.~Liu, R.~Panis, and G.~Slade. \newblock The torus plateau for the high-dimensional {Ising} model. \newblock Preprint, \url{https://arxiv.org/pdf/2405.17353}, (2024). \bibitem{LPS25-universal} Y.~Liu, J.~Park, and G.~Slade. \newblock Universal finite-size scaling in high-dimensional critical phenomena. \newblock Preprint, (2024). \bibitem{LS25a} Y.~Liu and G.~Slade. \newblock Near-critical and finite-size scaling for high-dimensional lattice trees and animals. \newblock Preprint, (2024). \bibitem{MS13} Y.~Mej\'ia~Miranda and G.~Slade. \newblock Expansion in high dimension for the growth constants of lattice trees and lattice animals. \newblock {\em Combin. Probab. Comput.}, {\bf 22}:527--565, (2013). \bibitem{MPS23} E.~Michta, J.~Park, and G.~Slade. \newblock Boundary conditions and universal finite-size scaling for the hierarchical $|\varphi|^4$ model in dimensions $4$ and higher. \newblock Preprint, \url{https://arxiv.org/pdf/2306.00896}, (2023). \bibitem{Olve97} F.W.J. Olver. \newblock {\em Asymptotics and Special Functions}. \newblock CRC Press, New York, (1997). \bibitem{PS25} J.~Park and G.~Slade. \newblock Boundary conditions and the two-point function plateau for the hierarchical $|\varphi|^4$ model in dimensions $4$ and higher. \newblock Preprint, \url{https://arxiv.org/pdf/2405.17344}, (2024). \bibitem{Slad20} G.~Slade. \newblock Self-avoiding walk on the complete graph. \newblock {\em J. Math. Soc. Japan}, {\bf 72}:1189--1200, (2020). \bibitem{Spen97} J.~Spencer. \newblock Enumerating graphs and {Brownian} motion. \newblock {\em Comm. Pure Appl. Math.}, {\bf 50}:291--294, (1997). \bibitem{Wrig77} E.M. Wright. \newblock The number of connected sparsely edged graphs. \newblock {\em J. Graph Theory}, {\bf 1}:317--330, (1977). \end{thebibliography} \end{document}
2412.05500v2
http://arxiv.org/abs/2412.05500v2
Syzygies of canonical ribbons on higher genus curves
\documentclass[11pt,twoside]{amsart} \usepackage[dvipsnames]{xcolor} \usepackage{hyperref} \usepackage{cleveref} \hypersetup{colorlinks=true, linkcolor=Blue, citecolor=Maroon} \usepackage{amsthm, amsmath, amscd, amssymb,centernot,txfonts} \usepackage{tikz-cd} \usepackage{lipsum} \usepackage{multicol} \usepackage[normalem]{ulem} \usepackage{amsfonts} \usepackage{adjustbox} \usepackage{mathrsfs} \usepackage[all]{xy} \usepackage{geometry} \usepackage{dirtytalk} \usepackage{mathtools} \usepackage{enumitem} \usepackage{comment} \usepackage{fourier} \usepackage{capt-of} \usetikzlibrary{shapes,arrows} \usepackage{todonotes} \usepackage{cancel} \setlength{\headheight}{15.2pt} \DeclarePairedDelimiter\ceil{\lceil}{\rceil} \DeclarePairedDelimiter\floor{\lfloor}{\rfloor} \DeclareMathOperator{\LCliff}{LCliff} \DeclareMathOperator{\RCliff}{RCliff} \DeclareMathOperator{\gon}{gon} \DeclareMathOperator{\Ext}{Ext} \DeclareMathOperator{\Sec}{Sec} \DeclareMathOperator{\Proj}{Proj} \DeclareMathOperator{\Sym}{Sym} \DeclareMathOperator{\Hom}{Hom} \theoremstyle{plain} \numberwithin{equation}{section} \newtheorem{theorem}{Theorem}[section] \newtheorem{proposition}[theorem]{Proposition} \newtheorem{lemma}[theorem]{Lemma} \newtheorem{corollary}[theorem]{Corollary} \newtheorem{conjecture}[theorem]{Conjecture} \newtheorem{question}[theorem]{Question} \theoremstyle{definition} \newtheorem{remark}[theorem]{Remark} \newtheorem{definition}[theorem]{Definition} \newtheorem{notation}[theorem]{Notation} \newtheorem{example}[theorem]{Example} \newtheorem{set-up}[theorem]{Set-up} \newcommand{\te}{\otimes} \newcommand{\myeq}{\overset{\mathrm{duality}}{=\joinrel=}} \newcommand*{\QEDA}{\hfill\ensuremath{\blacksquare}} \newcommand*{\QEDB}{\hfill\ensuremath{\square}} \tikzstyle{decision} = [diamond, draw, , text width=4.5em, text badly centered, node distance=3cm, inner sep=0pt] \tikzstyle{block} = [rectangle, draw, , text width=10em, text centered, rounded corners, minimum height=2em] \tikzstyle{block1} = [rectangle, draw, , text width=5em, text centered, rounded corners, minimum height=2em] \tikzstyle{line} = [draw, -latex'] \tikzstyle{cloud} = [draw, ellipse,, node distance=3cm, minimum height=2em] \definecolor{light-gray}{gray}{0.95} \begin{document} \title[Syzygies of canonical ribbons on higher genus curves]{Syzygies of canonical ribbons on higher genus curves} \author[A. Deopurkar]{Anand Deopurkar} \address{Mathematical Sciences Institute, The Australian National University} \email{[email protected]} \author[J. Mukherjee ]{Jayan Mukherjee} \address{Department of Mathematics, Oklahoma State University, Stillwater, USA} \email{[email protected]} \subjclass[2020]{14H51, 13D02} \keywords{canonical ribbons, syzygies, Green's conjecture, gonality conjecture, Koszul cohomology} \maketitle \begin{abstract} We study the syzygies of the canonical embedding of a ribbon $\widetilde{C}$ on a curve $C$ of genus $g \geq 1$. We show that the linear series Clifford index and the resolution Clifford index are equal for a general ribbon of arithmetic genus $p_a$ on a general curve of genus $g$ with $p_{a} \geq \operatorname{max}\{3g+7, 6g-4\}$. Among non-general ribbons, the case of split ribbons is particularly interesting. Equality of the two Clifford indices for a split ribbon is related to the gonality conjecture for $C$ and it implies Green's conjecture for all double covers $C'$ of $C$ with $g(C') \geq \textrm{max}\{3g+2, 6g-4\}$. We reduce it to the vanishing of certain Koszul cohomology groups of an auxiliary module of syzygies associated to $C$, which may be of independent interest. \end{abstract} \section{Introduction} Let $X \subset \mathbb{P}^N$ be a projective variety. Let $I \subset k[X_0, \dots, X_N] =: S$ be its homogeneous ideal, $R := S/I$ the homogeneous coordinate ring, and $L = \mathcal O_X(1)$. A fundamental problem is to understand the connection between geometric properties of $(X, L)$ and homological/algebraic properties of $R$ as an $S$ module. In the mid 1980's, Mark Green used the Koszul cohomology groups $K_{p,q}(X,L)$ to formulate such a connection \cite{G84,G84b}. Out of this work arose his conjecture about the Clifford index of a smooth projective curve $C$ and the shape of the minimal free resolution of the homogeneous coordinate ring of its canonical embedding \cite{G84,GL86}. This conjecture states that the Clifford index of $C$, a geometric quantity defined using linear series on $C$, is equal to its resolution Clifford index, an algebraic quantity related to the vanishing of the Koszul cohomology groups. In \cite{BE95} and \cite{EG95}, the authors laid out an approach to prove Green's conjecture for a general curve by proving it for an everywhere non-reduced curve called a ribbon. We recall the definition. \begin{definition}\label{ribbon} A ribbon $\widetilde{Y}$ on a reduced connected scheme $Y$ is a non-reduced scheme with $\widetilde{Y}_{\textrm{red}} = Y$ such that \begin{enumerate} \item the ideal sheaf $\mathcal{I}_{Y/\widetilde{Y}} \subset \mathcal O_{\widetilde Y}$ satisfies $\mathcal{I}_{Y/\widetilde{Y}}^2 = 0$ and \item viewed as an $\mathcal O_{Y}$ module, $\mathcal{I}_{Y/\widetilde{Y}}$ is a locally free sheaf $L$ of rank one (called the conormal bundle of $\widetilde Y$). \end{enumerate} \end{definition} The canonical ribbon conjecture, formulated in \cite{BE95} and \cite{EG95}, states that the analogue of Green's conjecture holds for a rational ribbon, that is, a ribbon $\widetilde C$ with $\widetilde{C}_{\textrm{red}} \cong \mathbb{P}^1$. By semicontinuity, it implies Green's conjecture for a general curve. In a series of pioneering papers, Voisin proved Green's conjecture for a general curve using different methods \cite{V02}, \cite{V05}. Using the results of Voisin and Hirschowitz--Ramanan \cite{HR98}, the first author proved the canonical ribbon conjecture for rational ribbons \cite{D18}. Eventually, Raicu--Sam \cite{RS19} and Park \cite{P22} gave independent proofs of the canonical ribbon conjecture, without using the results of Voisin and Hirschowitz--Ramanan, obtaining a proof of Green's conjecture for general curves via ribbons. In this context, we study the minimal free resolutions of canonical ribbons over higher genus curves. It has been shown in \cite{Gon06} and \cite{GGP08}, that in almost all cases such ribbons arise as flat limits of smooth curves. We see many compelling reasons to take up this study. First, the canonical ribbon conjecture for ribbons is interesting for the same reasons as it is for smooth curves. We have a geometric notion of linear series on ribbons, formulated in \cite{EG95}. By the same argument as for smooth curves, presence of linear series produces non-linear syzygies, or equivalently, non-vanishing $K_{p,2}$'s. It is natural to ask whether there are other reasons for non-linear syzygies, whose origins we do not understand. It is worthwhile to understand the answer in as many situations as we can. Ribbons on higher genus curves is one such setting. Second, by semi-continuity Green's conjecture for split ribbons on $C$ imply Green's conjecture for all double covers of $C$. Thus, understanding the minimal free resolution of the split ribbon has immediate payoff. Third, ribbons provide one of the few ways of producing higher genus curves from lower genus ones--a kind of ``induction''. A ribbon $\widetilde C$ on a smooth curve $C$ is given purely in terms of linear algebraic data: a rank 2 bundle obtained as an extension of $\Omega_{C}$ by the conormal bundle. Thus, ribbons allow us to connect questions about higher genus curves to questions about bundles on lower genus curves. The geometry of bundles on higher genus curves is much richer than that on $\mathbb{P}^1$. It is natural to ask if it leads to a richer connection. Finally, the geography of the moduli space of ribbons over higher genus curves is much more interesting. In genus $0$, the space of ribbons is naturally the ambient space of a rational normal curve. This space has many different statifications: $(1)$ the stratification by the secant varieties of the rational normal curve, $(2)$ the stratification by the blow-up index of the ribbon, $(3)$ the stratification by the splitting type of the rank $2$ bundle defining the ribbon, and $(4)$ the stratification by the shape of the minimal free resolution of the canonically embedded ribbon. A consequence of the canonical ribbon conjecture is that \emph{all} these stratifications agree. All stratifications have analogues for ribbons on higher genus curves. Understanding whether and how they differ is an intrinsically interesting question. \subsection{Results} Our first result is the canonical ribbon conjecture for a general ribbon of large genus on a general curve \begin{theorem} [\Cref{odd genus maximum blow-up index}, \Cref{generic green even}, \Cref{any genus and any blowup index} in the main text] \label{Green's conjecture intro} Let $C$ be a general curve of genus $g$. Fix a line bundle $L$ on $C$. Let $\widetilde{C}$ be a general ribbon with conormal bundle $L$. Let $p_{a}(\widetilde C)$ be the arithmetic genus, $\gon(\widetilde C)$ the gonality, $\LCliff(\widetilde C)$ the linear series Clifford index, and $\RCliff(\widetilde C)$ the resolution Clifford index of $\widetilde C$. If $p_{a}(\widetilde C) \geq \operatorname{max}\{3g+7, 6g-4\}$, then we have \[ \LCliff(\widetilde C) = \gon(\widetilde C)-2 = \RCliff(\widetilde C) = \left\lfloor \frac{p_a(\widetilde C) - 1}{2} \right \rfloor. \] \end{theorem} To prove \Cref{Green's conjecture intro}, we study two stratifications on the space $\mathbb{P}(H^0(2K_C-L)^*)$ of ribbons on $C$ with conormal bundle $L$. The first is the stratification by the blow-up index of the ribbon, and the second is the stratification by gonality. These two stratifications coincide with the secant stratification in the case when $C \cong \mathbb{P}^1$. In higher genus, the blow-up index stratification coincides with the secant stratification, but the gonality stratification is different and more interesting. Let $\textrm{Sec}_k(C)$ denote the $k$th secant variety of $C$ in the embedding $C \hookrightarrow \mathbb{P}H^0(2K_C-L)^*$. \begin{theorem}[\Cref{gonality stratification} and \Cref{inclusion of gonality stratification in secant variety}] \label{gonality stratification introduction} Let $C$ be a smooth curve of genus $g$ and gonality $m$. Then the subvariety $W_d$ of $\mathbb{P}(H^0(2K_C-L)^*)$ parameterizing ribbons $\widetilde{C}$ with conormal bundle $L$ containing a $g^1_d$ is the subvariety of $\Sec_{d+2g-2}(C)$ given by \begin{align*} W_d = \bigcup_{e \leq d/2} \left\{\text{\parbox{.7\linewidth}{Secant planes spanned by divisors of length $d+2g-2$ that contain a ramification divisor of a degree $e$ map $C \to \mathbb{P}^1$}}\right\}. \end{align*} In particular, we have \[\textrm{Sec}_{d-2m}(C) \subset W_d \subset \textrm{Sec}_{d+2g-2}(C).\] \end{theorem} We turn next to the split ribbon. This is a particularly interesting case---all smooth double covers isotrivially degenerate to a split ribbon, and hence, a proof of equality of the linear series and resolution Clifford index for a split ribbon establishes Green's conjecture for a smooth double cover. We first show that the linear series Clifford index of a split ribbon is $2m-2$, where $m$ is the gonality of the underlying curve (\Cref{lower bounds to linear series gonality and Clifford index}). The rest of the section is devoted to proving that the following statements are equivalent to the fact that the resolution Clifford index of a split ribbon is $2m-2$. We prove the following. \begin{theorem}\label{green for split intro} (see \Cref{green for split}, \Cref{green for split implies green for double cover}) Let $\widetilde{C}$ be a split ribbon on a curve $C$ of genus $g$ and gonality $m$ with conormal bundle $L$. \begin{enumerate} \item The linear series Clifford index of $\widetilde C$ is $\LCliff(\widetilde C) = 2m-2$. \item If $p_a(\widetilde{C}) \geq \textrm{max}\{2g+2m-1, 6g-4\}$, then the following are equivalent \smallskip \begin{enumerate} \item The resolution Clifford index $\RCliff(\widetilde C) = 2m-2$. \smallskip \item For all $i,j$ with $i,j \geq 0$ and $i + j = 2m-3$, the map \[\Phi_{i,j}: \bigwedge^{i+1}H^0(K_C) \otimes K_{j,1}(C, K_C-L) \xrightarrow{} \bigwedge^{i}H^0(K_C) \otimes K_{j,1}(C, K_C, K_C-L)\] is surjective. \smallskip \item Set $M^j = \bigoplus_q K_{j,1}(C, qK_C, K_C-L)$. For all $i,j$ with $i,j \geq 0$ and $i + j = 2m-3$, we have \[K_{i,1}(M^{j}, H^0(K_C)) = 0.\] \smallskip \end{enumerate} \smallskip \item If any of the equivalent conditions of part $(2)$ holds, then any smooth curve $C'$ which is a double cover of $C$ branched along $-2L$ satisfies Green's conjecture. \end{enumerate} \end{theorem} We believe that it is interesting to study whether the equivalent statements are true for any smooth curve $C$ (see \Cref{split ribbon conjecture}). We show that the question has a positive answer when the curve is either elliptic or hyperelliptic. We end with \Cref{does not satisfy Green}, where we give an example of a split ribbon of low arithmetic genus, i.e, arithmetic genus $9$ over a general curve of genus $3$, which does \textit{not} satisfy Green's conjecture. \subsection{Organization and conventions} In \Cref{sec:can}, we recall basic results about the canonical embedding of a ribbon. In \Cref{sec:cliff}, we recall and establish the basic properties of the Clifford index for ribbons. In \Cref{sec:blowup} and \Cref{sec:gonality}, we study the blow-up index and gonality stratification of the space of ribbons. In \Cref{sec:green_general}, we use these stratifications to prove Green's conjecture for a general ribbon. In \Cref{sec:split}, we treat the case of a split ribbon. All schemes are of finite type over an algebraically closed field of characteristic zero. For us, the projectivisation of $V$ is $\mathbb{P} V = \Proj \Sym^{*} (V^{*})$, which is the space of one-dimensional subspaces of $V$. Unless stated otherwise, a \emph{curve} is a projective connected scheme of pure dimension 1. \subsection*{Acknowledgements} The second author thanks Purnaprajna Bangere and Debaditya Raychaudhury for motivating discussions on syzygies of $K3$ carpets. The second author also thanks the Australian National University for its hospitality. \section{Canonical embedding of ribbons on curves of higher genus}\label{sec:can} In this section, we study the canonical map of a ribbon on a higher genus curve. Let $C$ be a smooth curve, $L$ a line bundle on $C$, and $\widetilde{C}$ a ribbon on $C$ with conormal bundle $L$. We begin by analyzing sections of positive line bundles on $\widetilde C$. \begin{proposition}\label{very ampleness on ribbons} In the setup above, let $\widetilde M$ be a line bundle on $\widetilde C$ and set $M := \widetilde{M}|_C$. \begin{enumerate} \item Assume that $M$ is very ample, $M \otimes L$ is base point free and $H^0(\widetilde{M}) \to H^0(M)$ surjects. Then $\widetilde{M}$ is very ample. \item Assume that $M$ is projectively normal and for all $k \geq 1$, the maps \[H^0(k\widetilde{M}) \to H^0(kM) \text{ and } H^0(M \otimes L) \otimes H^0(kM) \to H^0((k+1)M \otimes L)\] are surjective. Then the multiplication map $H^0(k\widetilde{M}) \otimes H^0(\widetilde{M}) \to H^0((k+1)\widetilde{M}) $ is surjective for all $k \geq 1$. \end{enumerate} \end{proposition} \begin{proof} Assuming (1), we have an exact sequence, \begin{equation*} 0 \to H^0(M \otimes L) \to H^0(\widetilde{M}) \to H^0(M) \to 0 \end{equation*} Let $\widetilde{\zeta}$ be a length two subscheme of $\widetilde{C}$. We must show that the restriction $H^0(\widetilde M) \to H^0(\widetilde M|_{\widetilde \zeta})$ surjects. Let $\zeta$ be the intersection of $\widetilde \zeta$ with $C$, so that $\mathcal O_\zeta = \mathcal O_{\widetilde \zeta} |_C$. Then $\zeta$ is a subscheme of $C$ of length one or two. If it is of length two, then $\widetilde \zeta$ is a subscheme of $C$. Since $M$ is very ample, and $H^0(\widetilde{M}) \to H^0(M)$ surjects, we conclude that $H^{0}(\widetilde M) \to H^{0}(\widetilde M|_{\widetilde \zeta})$ surjects. we get a section of $\widetilde{M}$ separating $\mathscr{O}_{\widetilde{\zeta}}$. On the other hand, if $\mathscr{O}_{\zeta}$ is of length one, then we have the exact sequence \begin{equation*} 0 \to \mathscr{O}_{\zeta} \to \mathscr{O}_{\widetilde{\zeta}} \to \mathscr{O}_{\zeta} \to 0 \end{equation*} Tensoring by $\widetilde{M}$ and taking global sections gives \begin{equation*} 0 \to H^0(M \otimes L \otimes \mathscr{O}_{\zeta}) \to H^0(\widetilde{M} \otimes \mathscr{O}_{\widetilde{\zeta}}) \to H^0(M \otimes \mathscr{O}_{\zeta}) \to 0, \end{equation*} and a commutative diagram of exact rows as follows \[ \begin{tikzcd} 0 \arrow[r] & H^0(M \otimes L ) \arrow[r] \arrow[d] & H^0(\widetilde{M}) \arrow[r] \arrow[d] & H^0(M) \arrow[r] \arrow[d] & 0 \\ 0 \arrow[r] & H^0(M \otimes L \otimes \mathscr{O}_{\zeta}) \arrow[r] & H^0(\widetilde{M} \otimes \mathscr{O}_{\widetilde{\zeta}}) \arrow[r] & H^0(M \otimes \mathscr{O}_{\zeta}) \arrow[r] & 0. \end{tikzcd} \] Since both $M$ and $M \otimes L$ are base point free, the flanking vertical maps surject and hence the middle vertical map surjects. Assuming (2), we need to show that $H^0(k\widetilde{M}) \otimes H^0(\widetilde{M}) \to H^0((k+1)\widetilde{M})$ surjects. We have the following commutative diagram \[ \begin{tikzcd} 0 \arrow[r] & H^0(M \otimes L ) \otimes H^0(k\widetilde{M}) \arrow[r] \arrow[d] & H^0(\widetilde{M}) \otimes H^0(k\widetilde{M}) \arrow[r] \arrow[d] & H^0(M) \otimes H^0(k\widetilde{M}) \arrow[r] \arrow[d] & 0 \\ 0 \arrow[r] & H^0((k+1)M \otimes L)) \arrow[r] & H^0((k+1)\widetilde{M}) \arrow[r] & H^0((k+1)M) \arrow[r] & 0 \end{tikzcd} \] The conditions in (2) imply that the two flanking vertical maps surject, and hence the middle vertical map also surjects. \end{proof} We now apply the previous analysis to the canonical bundle. \begin{proposition}\label{canonical morphism of ribbons} Let $\widetilde{C}$ be a ribbon on a smooth irreducible curve $C$ of genus $g \geq 1$ with conormal bundle $L$. \begin{enumerate} \item If $p_a \geq 2g+2$, then $ K_{\widetilde{C}} $ is very ample. \item If $p_a \geq 2g+2$ and either $g = 1$ or $h^0(K_C+L) \leq g-2$, then the map $$H^0(kK_{\widetilde{C}}) \otimes H^0(K_{\widetilde{C}}) \to H^0((k+1)K_{\widetilde{C}})$$ is surjective for all $k \geq 1$. In particular if $g \geq 2$, the above surjectivity holds if $p_a \geq 4g-2$. \end{enumerate} \end{proposition} \begin{proof} The canonical bundle $K_{\widetilde{C}}$ of a ribbon sits in an exact sequence \begin{equation*} 0 \to K_C \to K_{\widetilde{C}} \to K_{\widetilde{C}}|_C \to 0, \end{equation*} and we have $K_{\widetilde{C}}|_C = K_C \otimes L^{-1}$. For $k \geq 2$, the space $H^1(k(K_C \otimes L^{-1}) \otimes L)$ vanishes while for $k = 1$, the map $H^1(K_C) \to H^1(K_{\widetilde{C}})$ is injective. Hence $H^0(kK_{\widetilde{C}}) \to H^0(k(K_C \otimes L^{-1}))$ is surjective for all $k \geq 1$. Observe that $K_{C}$ is base point free and $K_C \otimes L^{-1}$ is very ample if $-\deg L \geq 3$. So (1) follows from \Cref{very ampleness on ribbons}. Further, under the conditions of (2) the map \[H^0(K_C) \otimes H^0(k(K_C\otimes L^{-1})) \to H^0((k+1)(K_C \otimes L^{-1}) \otimes L)\] is surjective by \cite[Theorem $4.e.1$]{G84} Hence part (2) follows from the part (2) of \Cref{very ampleness on ribbons}. \end{proof} \section{Linear Series Clifford index and resolution Clifford index of a ribbon}\label{sec:cliff} Let $C$ be a smooth curve of genus $g$ and let $\widetilde C$ be a ribbon on $C$ of arithmetic genus $p_a$ and conormal bundle $L$. The exact sequence \[ 0 \to L \to \mathcal{O}_{\widetilde C} \to O_C \to 0\] implies \[ \chi\left(\mathcal{O}_{\widetilde{C}}\right) = \chi(\mathcal{O}_C) + \chi(L),\] and hence \[ p_a = (2g-1) -\deg L.\] We recall some defintions as introduced in \cite[Section~1]{EG95}. A \emph{generalized line bundle} on $\widetilde{C}$ is a torsion free coherent sheaf which is generically free of rank $1$. We define the \emph{degree} of a generalized line bundle $\widetilde{M}$ by \[\deg(\widetilde{M}) = \chi(\widetilde{M})-\chi(\mathcal{O}_{\widetilde{C}}).\] The restriction of $\widetilde M$ to $C$ may have torsion; let $\tau \subset \widetilde M|_{C}$ be the torsion subsheaf. We set ${M} = (\widetilde{M}|_C/\tau)$. Sections of $\widetilde M$ are of two kinds, those that yield a non-zero section of $M$ and those that do not. The sections of the first kind define an injective map $\mathcal O_{\widetilde C} \to \widetilde M$. The scheme-theoretic vanishing locus of such a section is a Cartier divisor on $\widetilde C$. We call such sections \emph{Cartier} sections. The sections of the second kind do not give an injective map $\mathcal O_{\widetilde C} \to \widetilde M$. The entire reduced curve $C$ is contained in their scheme-theoretic zero locus. Let \(\widetilde M\) be a generalised line bundle on \(\widetilde C\). By \cite[Theorem~1.1]{EG95}, there exists a unique divisor \(\beta \subset C\) and a line bundle \(\widetilde M'\) on the blow up \(\widetilde C'\) of \(\widetilde C\) along \(\beta\) such that \(\widetilde M\) is the push-forward of \(\widetilde M'\). Then \(M = \widetilde {M'}|_{C}\). A \emph{generalized linear series} of rank $r$ and degree $d$, or simply a $g^r_d$, on $\widetilde{C}$ is pair $\Phi = (V, \widetilde{M})$ where $\widetilde{M}$ is a generalized line bundle and $V \subset H^0(\widetilde{M})$ is of dimension $r+1$, such that the restriction map $V \to H^0(M)$ is injective. The \emph{Clifford index} of $\Phi$ is $d-2r$. The \emph{linear series Clifford index} of $\widetilde{C}$, denoted by $\LCliff(\widetilde C)$, is the minimum of the Clifford indices of all generalized linear series $g^r_d$ such that $r \geq 1$ and $d \leq p_a-1$. The \emph{gonality} of $\widetilde{C}$, denoted by $\gon(\widetilde{C})$, is the smallest $d$ such that there exists a $g^1_d$ on $\widetilde{C}$. For a line bundle $\widetilde{H}$ on $\widetilde{C}$, we let $K_{p,q}(\widetilde{C}, \widetilde{H})$ be the Koszul cohomology group as defined in \cite{G84}. For $\widetilde H = K_{\widetilde C}$, we know that $K_{p,q}(\widetilde C, K_{\widetilde C})$ is possibly non-zero only for $0 \leq p \leq p_a-2$ and $0 \leq q 3$. Within this range, the group vanishes for $(p > 0, q = 0)$ and $(p < p_a-2, q = 3)$. So the most interesting cases are $q = 1$ and $q = 2$. Owing to the duality, \[ K_{p,q}(\widetilde C, K_{\widetilde C}) = K_{p_a-p-2,3-q}(\widetilde C, K_{\widetilde C})^{\vee},\] understanding $K_{p,1}$'s is equivalent to understanding $K_{p,2}$'s. We also know that if $K_{p,2}(\widetilde C, K_{\widetilde C}) = 0$ then for all $i \geq p$, we have $K_{i,2}(\widetilde C, K_{\widetilde C}) = 0$. So it is important to understand the smallest $p$ such that $K_{p,2}(\widetilde C, K_{\widetilde C}) \neq 0$. This $p$ is called the \emph{resolution Clifford index} of $\widetilde C$. We denote it by $\RCliff(\widetilde C)$. By duality, it is the smallest $p$ such that $K_{p_a-2-p,1}(\widetilde{C}, K_{\widetilde{C}}) \neq 0$. \subsection{Semicontinuity of gonality and Clifford index} The resolution Clifford index is lower semi-continuous by the semi-continuity of cohomology. We now establish the lower semicontinuity of gonality and linear series Clifford index. Fix a smooth curve $C$ of genus $g$ and a ribbon $\widetilde C$ on $C$ of arithmetic genus $p_a$. Let $(T,0)$ be a smooth pointed curve and $\mathcal C \to T$ a flat proper morphism of relative dimension 1. Suppose for all $t \in T$ with $t \neq 0$, the fiber $\mathcal C_t$ is a smooth curve of genus $p_a$ and the fiber $\mathcal C_0$ is $\widetilde C$. The following proposition constructs a limiting generalized $g^r_d$. \begin{proposition}\label{limit grd} In the setup above, assume that for all $t \neq 0$, the curve $\mathcal C_t$ has a $g^{r}_{d}$. Assume that $p_a > d + 2g - 1$ and $r \geq 1$. Then $\widetilde C$ has a generalised $g^r_{d'}$ with $d' \leq d$. \end{proposition} \begin{proof} We follow the proof of \cite[Theorem~2.1]{EG95}, replacing \(\omega_{\mathcal C/T}\) by a more suitable line bundle. Let \(\eta\) be the generic point of \(T\). After a finite base change, we may assume that we have a line bundle \(\mathcal G_{\eta}\) on \(\mathcal C_{\eta}\) and an $(r+1)$-dimensional subspace \(V_{\eta} \subset H^0(\mathcal C_{\eta})\). Choose a line bundle \(\mathcal E\) on \(\mathcal C\) of relative degree \(2g-4\), for example, by starting with a line bundle of degree \(2g-4\) on \(\widetilde C\) and deforming it. Then $\mathcal E|_{C}$ is a line bundle of degree $g-2$. Assume that it is a general line bundle of this degree. Set \(\mathcal F = \omega_{\mathcal C/T} \otimes \mathcal E^{-1}\). \begin{align*} \deg\left(\mathcal F_{\eta} \otimes \mathcal G_{\eta}^{-1}\right) &= (2p_a-2)-(2g-4)-d \\ &\geq p_a. \end{align*} The last inequality follows from our assumption $p_a > d + (2g-1)$. In particular, \(\mathcal F_{\eta} \otimes \mathcal G_{\eta}^{-1}\) is effective. One of its section gives an inclusion \(\mathcal G_{\eta} \subset \mathcal F_{\eta}\). Via this inclusion, we may think of \(V_{\eta}\) as a subspace of \(H^0(\mathcal F_{\eta})\). By the theorem on cohomology and base change \cite[Section~5]{mum}, there exists a map \(K^0 \to K^1\) of locally free \(\mathcal O_T\) modules of finite rank such that for every \(S \to T\), we have a canonical isomorphism \[ \pi_{*} (\mathcal C_T \times S, \mathcal F \times S) = \ker(K^0_S \to K^1_{S}).\] We have the subspace \(V_{\eta} \subset K^0_{\eta}\) of dimension $r+1$. It extends to a locally free $\mathcal O_{T}$ module of the same rank \(V \subset K^{0}\) such that the map \(V|_0 \to K^0|_0\) remains injective. Since \[V_{\eta} \subset \ker(K^0_{\eta} \to K^1_{\eta}) = H^0(\mathcal F_{\eta}),\] we have \[V \subset \ker (K^0 \to K^1) = \pi_{*}(\mathcal F)\] and \[V|_0 \subset \ker(K^0|_0 \to K^1|_0) = H^0(\mathcal F|_0)\] by continuity. We have the exact sequence \[ 0 \to K_C \to K_{\widetilde C} \to K_{\widetilde C}|_C \to 0.\] Tensoring by \(\mathcal E^{-1}\) yields the exact sequence \begin{equation} 0 \to K_C \otimes \mathcal E^{-1} \to \mathcal F|_{0} \to \mathcal F|_{C} \to 0. \end{equation} We have \[\deg (K_{C} \otimes \mathcal E^{-1})= (2g-2)-(g-2) = g.\] Since $\mathcal E|_{C}$ is general, so is $K_C \otimes \mathcal E^{-1}$, and hence $h^0(K_C \otimes \mathcal E^{-1}|_C) = 1$. By the long exact sequence, we see that \begin{equation}\label{eqn:negative kernel} \dim \ker \left( H^0(\mathcal F|_0) \to H^0(\mathcal F|_C) \right) = 1. \end{equation} Let \(\mathcal G\) be the subsheaf of \(\mathcal F\) generated by \(V\) and let \(G_0 \subset \mathcal F|_0\) be the image of \(\mathcal G\). Equivalently, \(G_0\) is the subsheaf of \(\mathcal F|_0\) generated by \(V_0\). Then $G_0$ is torsion free. Since $\dim V_0 = (r+1)$ is greater than the dimension $1$ of the kernel of $H^0(\mathcal F|_0) \to H^0(\mathcal F|_C)$, the restriction map $V_0 \to H^0(\mathcal F|_C)$ is non-zero. Then it follows that the map $G_0 \subset \mathcal F|_0$ is an isomorphism at the generic point. Therefore, $G_0$ is a generalized line bundle. Since we have a surjection $\mathcal G|_0 \to G_{0}$, and $\deg \mathcal G|_0 = d$, we conclude that $\deg G_0 \leq d$. Set \(G = G_0|_C / \textrm{torsion}\). We know that the map $V_0 \to H^0(G)$ is non-zero, so $\deg G \geq 0$. It remains to check that the map $V_0 \to H^0(G)$ is injective. Let $\beta \subset C$ be the divisor such that $G_0$ is the push-forward of a line bundle from the blow-up of $\widetilde C$ along $\beta$. Let $L$ be the conormal bundle of $\widetilde C$. Then we have the exact sequence \[ 0 \to G \otimes L(\beta) \to G_{0} \to G \to 0;\] see \cite[\S~1]{EG95}. Let $d' = \deg G_0$. Then \[\deg G + \deg \beta \leq 2\deg G + \deg \beta = d',\] so \[ \deg (G \otimes L(\beta)) \leq d' + \deg L = d' +(2g-1) - p_a \leq d + (2g-1) - p_a < 0,\] where the last inequality follows from $p_a > d + (2g-1)$. As a result, $H^0(G \otimes L(\beta)) = 0$, and hence the map $H^0(G_0) \to H^0(G)$ is injective. Therefore, the composite $V_0 \subset H^0(G_0) \to H^0(G)$ is injective. \end{proof} \begin{corollary}\label{semicontinuity of gonality} In the setup of \Cref{limit grd}, suppose the generic fibers $\mathcal C_t$ have gonality $d$. If $p_a > d+2g-1$, then $\mathcal C$ has gonality at most $d$. In particular, if $p_a > 4g+1$, then $\mathcal C$ has gonality at most $d$. \end{corollary} \begin{proof} The first statement follows directly from Proposition\ref{limit grd}. For the second statement, observe that $d \leq (p_a+3)/2$. So $p_a > 4g+1$ implies $p_a > d+2g-1$. \end{proof} In \Cref{semicontinuity of gonality}, the condition \(p_a > d+2g-1\) is indeed necessary. See \Cref{ex:ellipticg13} for the failure of the existence of a limiting \(g^1_3\) without this condition. \begin{corollary}\label{semicontinuity of Clifford index and Clifford dimension} In the setup of \Cref{limit grd}, suppose the generic fibers $\mathcal C_t$ have Clifford index $c$ and Clifford dimension $r$. If $p_a > c+2r + (2g-1)$, then $\LCliff(\widetilde C) \leq c$. In particular, if $p_a > 4g-3 + 4r$, then $\LCliff(\widetilde C) \leq c$. \end{corollary} \begin{proof} The first statement follows directly from Proposition\ref{limit grd}. For the second statement, observe that $c \leq (p_a-1)/2$. So $p_a > 4g-3 + 4r$ implies $p_a > c+2r+(2g-1)$. \end{proof} Using the results of \cite{ELMS89}, we eliminate the dependence of $p_a$ on the Clifford dimension $r$ in Corollary~\ref{semicontinuity of Clifford index and Clifford dimension}. \begin{corollary}\label{semicontinuity of Clifford index} In the setup of \Cref{limit grd}, assume that the fibers $\mathcal C_t$ have Clifford index $c$ and Clifford dimension $r$. Then $\LCliff(\widetilde C) \leq c$ holds under any of the following hypotheses: \begin{enumerate} \item $r = 1$ and $p_a > 4g+1$, \item $p_a$ is odd and $p_a > 8g+1$, \item $p_a$ is even, \((p_{a}, c, d) \neq (4r-2, 2r-3,4r-3)\), and $p_a > 8g+1$. \end{enumerate} \end{corollary} \begin{proof} Under the first hypothesis, the conclusion is Corollary\ref{semicontinuity of gonality}. Under the second or third hypothesis, \cite[Corollary~3.5]{ELMS89} says that $p_a\geq 8r-7$. Then $p_a > 8g+1$ implies $p_a > 4g-3 + 4r$. Therefore, the conclusion follows from Corollary~\ref{semicontinuity of Clifford index and Clifford dimension}. \end{proof} \subsection{Green-Lazarsfeld non-vanishing theorem for ribbons} In this section, we relate the linear series Clifford index and the resolution Clifford index of a ribbon. We show that, for $p_a$ large compared to $g$, we have the inequality $\RCliff \leq \LCliff$. \begin{theorem}\label{GL vanishing 1} Let $\widetilde{C}$ be a ribbon of arithmetic genus $p_a$ on smooth curve $C$ of genus $g$ with $h^0(\mathcal{O}_{\widetilde C}) = 1$. Let $\widetilde{M}_1$ and $\widetilde{M}_2$ be line bundles on $\widetilde C$ and set \[\widetilde{M} = \widetilde{M}_1 \otimes \widetilde{M}_2.\] For $i = 1,2$, let $H^0(\widetilde{M}_i)$ be of dimension $r_i+1$, with $r_i \geq 1$. Assume that \begin{enumerate} \item $\widetilde{M}_1$ is base point free, and \item the zero locus of a general element of $H^0(\widetilde{M}_2)$ is zero dimensional. \end{enumerate} Then \begin{equation*} K_{r_1+r_2-1,1}(\widetilde{C}, \widetilde{M}) \neq 0 \end{equation*} \end{theorem} \begin{proof} Let $s_1 \in H^0(\widetilde M_2)$ be a section whose scheme-theoretic zero locus $D_1$ is zero dimensional. Let $s_2 \in H^0(\widetilde M_1)$ be a section whose scheme-theoretic zero locus $D_2$ is disjoint from $D_1$. Note that both $D_1$ and $D_2$ are Cartier diviors on $\widetilde C$. Then, up to scaling, there is a unique section $s_0$ of $H^0(\widetilde{M})$ that vanishes on both $D_1$ and $D_2$; it is the section whose scheme-theoretic zero locus is $D_1+D_2$. The rest of the proof follows verbatim from \cite[Appendix]{G84}. We sketch it for the convenience of the reader. Recall that a Cartier section of a line bundle on $\widetilde C$ is a section that defines an injection from $\mathcal{O}_{\mathcal{C}}$. The scheme theoretic zero locus of a Cartier section is a Cartier divisor. Since $s \in H^0(\widetilde{M})$ is Cartier, a general element of $H^0(\widetilde M)$ is Cartier. Consider $H^0(\widetilde{M}(-D_1))$ (resp. $H^0(\widetilde{M}(-D_2))$) seen as subspaces of $H^0(\widetilde{M})$. These two subspaces intersect along the one-dimensional subspace spanned by $s$. These subspaces contain Cartier sections $s_2$ and $s_1$, respectively, so their general section is Cartier. Choose bases of these subspaces consisting of Cartier sections as follows. Let \[s_0, s_1, ..., s_{r_2}\] be a basis of $H^0(\widetilde{M}(-D_2))$, and \[s_0, s_{r-r_1+1},...,s_r\] a basis of $H^0(\widetilde{M}(-D_1))$. Extend to a basis of $H^0(\widetilde M)$ by adding Cartier sections \[s_{r_2+1},...,s_{r-r_1}.\] Let $\{e_0, \dots, e_{r}\}$ be the dual basis of $H^0(\widetilde{M})^*$. Let $\iota = \sum_{i=1}^{r-r_1} e_i \otimes s_i$ and $s = \sum_{i=0}^{r} e_i \otimes s_i $. Let $$ \alpha = \iota \wedge e_{r_2+1} \wedge...\wedge e_{r-r_1} $$ Then $\alpha \in \bigwedge^{r-r_1-r_2+1} H^0(\widetilde{M})^* \otimes H^0(\widetilde{M}(-D_2))$. Consider $s \wedge \alpha$. We have that \begin{align*} s \wedge \alpha &\in \bigwedge^{r-r_1-r_2+2} H^0(\widetilde{M})^* \otimes H^0(\widetilde{M}(-D_2)\otimes\widetilde{M}(-D_1)) \\ & = \bigwedge^{r-r_1-r_2+2} H^0(\widetilde{M})^* \otimes H^0(\widetilde{M}) \\ &= \bigwedge^{r_1+r_2-1} H^0(\widetilde{M}) \otimes H^0(\widetilde{M}). \end{align*} Since $s \wedge s \wedge \alpha = 0$, we see that $s$ defines a Koszul cocycle, that is, an element of $K_{r_1+r_2-1,1}(\widetilde{C}, \widetilde{M})$. The fact that this element is non-zero follows exactly as in \cite[Appendix]{G84}. \end{proof} We now examine when line bundles residual to the canonical carry Cartier sections. \begin{lemma}\label{Cartier divisor in residual} Let $\widetilde{C}$ be a ribbon of arithmetic genus $p_a$ on a smooth curve $C$ of genus $g$. Let $\widetilde{M}_1$ be a line bundle on $\widetilde{C}$ with $h^0(\widetilde{M}_1) \geq r+1$ and $r \geq 1$. Set $d = \deg \widetilde M_1$ and $c = d-2r$. Let $\widetilde M_2 = K_{\widetilde C} \otimes \widetilde M_1^{-1}$. Assume that $d \leq p_a-1$ and $H^{0}(\widetilde M_1)$ contains a Cartier section. If $p_a > 3g-2+c$ then $h^0(\widetilde M_2) \geq r+1$ and $H^0(\widetilde{M}_2)$ contains a Cartier section. \end{lemma} \begin{proof} By Riemann--Roch, we have \[ h^{0}(\widetilde M_2) = p_a-d - 1 + h^0(\widetilde M_1).\] Since $d \leq p_a-1$ and $h^0(\widetilde M_1)\geq r+1$, we have $h^0(\widetilde M_2) \geq r+1$. Set $\widetilde{M}_2|_C = M_2$. Let the conormal bundle of $\widetilde{C}$ be $L$. We have the exact sequence \[0 \to M_2 \otimes L \to \widetilde{M}_2 \to M_2 \to 0,\] and hence \begin{equation*} \deg(M_2 \otimes L) = \displaystyle\frac{1}{2}\deg(\widetilde{M}_2)+\deg(L) = \displaystyle\frac{1}{2}(2p_a-2-d)-(p_a-2g+1) = 2g-2-\displaystyle\frac{d}{2}. \end{equation*} If $d > 4g-4$, then every section of $H^0(\widetilde M_2)$ is Cartier. Assume that $d \leq 4g-4$. Then \[r = \displaystyle\frac{1}{2}(d-c) \leq 2g-2.\] Multiplication by a Cartier section of $\widetilde M_1$ gives an injection \[\widetilde M_2 \hookrightarrow K_{\widetilde C}.\] Recall the short exact sequence \[0 \to H^0(K_C) \to H^0(K_{\widetilde{C}}) \to H^0(K_C \otimes L^{-1}).\] The inclusion $\widetilde M_2 \hookrightarrow K_{\widetilde C}$ induces an injection \[H^0(M_2 \otimes L) \to H^0(K_C).\] In particular, $h^0(M_2 \otimes L) \leq g$. On the other hand, we know that \[h^0(\widetilde M_2) = p_a - d -1+ h^0(\widetilde M_1) \geq p_a - d + r.\] From $p_a > 3g-2 + c$ and $d \leq 4g-4$, it follows that $p_a-d+r > g$ and hence $h^0(\widetilde M_2) > h^0(M_2 \otimes L)$. So there exists a Cartier section of $\widetilde M_2$. \end{proof} We combine \Cref{GL vanishing 1} and \Cref{Cartier divisor in residual} to obtain the following non-vanishing result. \begin{theorem}\label{lin and res} Let $\widetilde{C}$ be a ribbon of arithmetic genus $p_a$ on a smooth curve $C$ of genus $g$ and linear series Clifford index $\LCliff(\widetilde C)$. If $p_a > 3g-2+\LCliff(\widetilde C)$, then $\RCliff(\widetilde C) \leq \LCliff(\widetilde C)$, that is, \[ K_{p_a-2-\LCliff(\widetilde C),1}(\widetilde{C}, K_{\widetilde{C}}) \neq 0.\] In particular, the non-vanishing holds if $p_a > 6g-5$. \end{theorem} \begin{proof} Let $L$ be the conormal bundle of $\widetilde{C}$. Since $p_a > 2g-1$, we have $\deg L = -(p_a-2g+1) < 0$, so $h^0(\mathcal{O}_{\widetilde{C}}) = 1$. Let $g^r_d$ be a generalized linear series on $\widetilde{C}$ with $d \leq p_a-1$ and $r \geq 1$ and $d-2r = c$. Recall that this is a pair $(V, \widetilde M_1)$ where $\widetilde{M}_1$ is a generalized line bundle of degree $d$ and $V \subset H^0(\widetilde{M}_1)$ is an $(r+1)$ dimensional vector subspace of global sections which injects into $H^0(M_1)$. We may assume that $\widetilde{M}_1$ is globally generated by $V$. Otherwise, we simply replace $\widetilde{M}_1$ by the subsheaf generated by $V$ (see \cite[Lemma~1.3]{EG95}). By \cite[Theorem~1.1]{EG95}, there exists a divisor $\beta \subset C$ of degree $b$, such that after blowing up $\widetilde{C}$ along $\beta$ we have a line bundle $\widetilde{M}_1'$ of degree $d' = d-b$ whose pushforward is $\widetilde{M}_1$. The blown up ribbon $\widetilde{C}'$ has conormal bundle $L(\beta)$ and arithmetic genus $p_a' = p_a-b$. Since $H^0(\widetilde{M}_1') = H^0(\widetilde{M}_1)$, we may treat $V$ as a subspace of $H^0(\widetilde{M}'_1)$. Since $d \leq p_a-1$, we also have $d' \leq p_a'-1$. Since $H^0(\widetilde{M}_1')$ contains $V$ of dimension $r+1$, we have $h^0(\widetilde M_1') \geq r+1$. Since $V \subset H^0(\widetilde M_1)$ generates $\widetilde M_1$, it follows that $V \subset H^0(\widetilde M_1')$ generates $\widetilde M_1'$. Set $c' = d'-2r = c-b$. Since $p_a > 3g-2+c$, we have $p_a' > 3g-2+c'$. Set $\widetilde M_2' = K_{\widetilde C'} \otimes (\widetilde{M}_1')^{-1}$. By \Cref{Cartier divisor in residual}, $H^0(\widetilde M_2')$ contains a Cartier section. Write $h^0(\widetilde{M}'_1) = r+1+a$ for some $a \geq 0$. By Riemann-Roch, we have $h^0(\widetilde{M}_2') = p_a'-d'+r+a$. By \Cref{GL vanishing 1}, we get \begin{equation}\label{van 1} K_{p_a'-d'+2r-2+2a, 1}(\widetilde{C}', K_{\widetilde{C}'}) \neq 0. \end{equation} Since $a \geq 0$, we have \[p'_a - d'+2r+2a \geq p'_a-d'+2r-2 = p'_a-c'-2 = p_a-c-2.\] So from \eqref{van 1}, we conclude \[K_{p_a-c-2, 1}(\widetilde{C}', K_{\widetilde{C}'}) \neq 0.\] Then by \cite[Lemma~1]{D18}, we deduce \[K_{p_a-c-2, 1}(\widetilde{C}, K_{\widetilde{C}}) \neq 0.\] The proof is now complete. \end{proof} \section{Blow-up index stratification of the space of ribbons}\label{sec:blowup} Recall that a \emph{split} ribbon $\widetilde C$ on $C$ is one that admits a retraction map $\widetilde C \to C$. Every ribbon admits a blow-up that is a split ribbon. The \emph{blow up index} of a ribbon is the minimum number of simple blow-ups necessary to make the ribbon split. In this section, we study the set of ribbons of a given blow-up index, its relationship with a secant variety, and with the linear series Clifford index. \subsection{Blow-up index of a ribbon as a pushout and relations to secant variety} Fix a smooth curve $C$. A ribbon $\widetilde{C}$ on $C$ with conormal bundle $L$ gives an extension \[ 0 \to L \to \Omega_{\widetilde{C}}|_C \to K_C \to 0.\] Conversely, given an extension \[ 0 \to L \to E \to K_C \to 0,\] there is a unique ribbon $\widetilde C$ on $C$ with an isomorphism $E \cong \Omega_{\widetilde C}|_{C}$ compatible with the extension. Therefore, the space of ribbons on $C$ with conormal bundle $L$ is identified with $\mathbb{P} \Ext^1(\Omega_C, L)$ (see \cite[Theorem~1.2]{BE95}). Fix a ribbon $\widetilde C$. Given a divisor $\beta \subset C$, we have the map $L \to L(\beta)$. Construct the push-out diagram \[ \begin{tikzcd} 0 \arrow[r] & L \arrow[r] \arrow[d] & \Omega_{\widetilde{C}}|_C \arrow[r] \arrow[d] & K_C \arrow[r] \arrow[equal,d] & 0 \\ 0 \arrow[r] & L(\beta) \arrow[r] & E \arrow[r] & K_C \arrow[r] & 0. \end{tikzcd} \] The extension in the second row corresponds to a ribbon $\widetilde{C}'$ with conormal bundle $L(\beta)$. By \cite[Theorem~1.9]{BE95}, the ribbon $\widetilde{C}'$ is precisely the blow-up of $\widetilde{C}$ along the Weil divisor $\beta$ of $\widetilde{C}$. Let $e$ be the class of the extension in the first row and $e'$ the extension in the second row. Let \begin{equation}\label{pushout} \mathbb{P}\Ext^1(K_{C}, L) \to \mathbb{P} \Ext^1(K_C, L(\beta)) \end{equation} be the map induced by $L \to L(\beta)$. Then $e'$ is the image of $e$. If $\beta$ is of sufficiently large degree, then the group on the right vanishes, and hence $e'$ vanishes. Then $\widetilde C'$ is the split ribbon. \begin{definition}\label{blow-up index} Let $\widetilde{C}$ be a ribbon on a smooth projective curve $C$. The \emph{blow-up index} $b(\widetilde{C})$ of $\widetilde C$ is the smallest $b$ such that there exists a divisor $\beta$ on $C$ of degree $b$ such that the blow-up of $\widetilde C$ along $\beta$ is split. \end{definition} Let $e \in \mathbb P \Ext^1(K_C, L)$ be the extension class corresponding to $\widetilde C$. Then $b(\widetilde C)$ is the smallest such that there exists an effective divisor $\beta \subset C$ such that the image of $e$ in $\mathbb P \Ext^1(K_C, L(\beta))$ vanishes. Recall that $C$ is a smooth curve. By Serre duality, we have the identification \[ \Ext^1(\Omega_C, L) = H^1(L\otimes K_C^{-1}) = H^0(K_C^{2} \otimes L^{-1})^{*}.\] \begin{proposition}\label{blow-up index and secant variety} Let $L$ be a line bundle of negative degree on $C$. Let $i \colon C \hookrightarrow \mathbb{P}H^0(K^{2}_C\otimes L^{-1})^*$ be the embedding given by the complete linear series of the very ample line bundle $K_C^2 \otimes L^{-1}$. Then the ribbons $\widetilde{C}$ on $C$ with conormal bundle $L$ and $b(\widetilde{C}) \leq k$ correspond to the points of the $k$-secant variety of the embedding $i$. \end{proposition} \begin{proof} Let $\widetilde{C}$ be a ribbon on $C$ with conormal bundle $L$ and extension class \[e \in H^0(K_C^{2}\otimes L^{-1})^{*} = H^{1}(L \otimes K_C^{-1}).\] Let $\beta \subset C$ be an effective divisor. Consider the map \[ H^1(L \otimes K_C^{-1}) \xrightarrow{f} H^1(L(\beta) \otimes K^{-1}_C). \] The map above is Serre dual to the map \[ H^0(K^{2}_C\otimes L^{-1})^* \to H^0(K^{2}_C \otimes L^{-1}(-\beta))^*, \] which in turn is dual to the map \[ H^0(K_C^2 \otimes L^{-1}(-\beta)) \xrightarrow{g} H^{0}(K_C^2\otimes L^{-1}) \] induced by the inclusion $\mathcal O(-\beta) \to \mathcal O$. Observe that $e$ lies in the kernel of $f$ if and only if the composite \[ H^0(K_C^2 \otimes L^{-1}(-\beta)) \xrightarrow{g} H^{0}(K_C^2\otimes L^{-1}) \xrightarrow{e} \mathbb k\] vanishes. But the points $\lambda \in \mathbb P H^0(K^2 \otimes L^{-1})$ such that the composite $\lambda \circ g$ vanishes are precisely the points that lie on the span of $\beta$ in the embedding $i$. It follows that the ribbons $\widetilde C$ of blow-up index at most $k$ correspond the points lying on the span of an effective divisor of degree at most $k$, which is the $k$-secant variety of $C$. \end{proof} \begin{corollary}\label{Using secant varieties to compute blow-up index} Let \(C\) be a smooth curve of genus \(g\). Fix \(p_a > 2g-1\) and let \(L\) be a line bundle on \(C\) of degree \(-p_a+2g-1\). Let $\widetilde{C}$ be a ribbon of arithmetic genus with conormal bundle \(L\). Then \begin{enumerate} \item $0 \leq b(\widetilde{C}) \leq \lceil (p_a+g-2)/2 \rceil$ \item If $\widetilde{C} \in \mathbb P \Ext^1(K_C, L)$ is general, then $b(\widetilde{C}) = \lceil (p_a+g-2)/2 \rceil$ \end{enumerate} \end{corollary} \begin{proof} The $k-$th secant variety of a curve $C \hookrightarrow \mathbb{P}^N$ is of expected dimension $\min(2k-1, N)$ (see \cite{Lan84}). So the result follows from \Cref{blow-up index and secant variety}. \end{proof} \subsection{Blow-up index as pullback and relations to stability of $\Omega_{\widetilde{C}|_C}$} There is another interpretation of the blow-up index, which is useful for measuring gonality. As usual, let $\widetilde{C}$ be a ribbon on $C$ with conormal bundle $L$ defined by an extension $$ 0 \to L \to \Omega_{\widetilde{C}}|_C \to K_C \to 0.$$ An effective divisor \(\beta \subset C\) gives a map of line bundle \(K_C(-\beta) \to K_{C}\). This map yields the pull-back diagram \begin{equation}\label{pullback} \begin{tikzcd} 0 \arrow[r] & L \arrow[r] \arrow[d] & E \arrow[r] \arrow[d] & K_C(-\beta) \arrow[r] \arrow[d] & 0 \\ 0 \arrow[r] & L \arrow[r] & \Omega_{\widetilde{C}}|_C \arrow[r] & K_C \arrow[r] & 0. \end{tikzcd} \end{equation} Denoting by \(e \in \Ext^1(K_C,L)\) the class of the extension in the bottom row, the class of the extension in the top row is simply the image of \(e\) under the map \begin{equation}\label{eq:pullback} \Ext^1(K_C, L) \to \Ext^1(K_C(-\beta), L). \end{equation} The observation leads to the following. \begin{proposition}\label{equivalent definitions of blow-up index} Let \(C\) be a smooth curve and \(L\) a line bundle on \(C\). Let \(\widetilde C\) be a ribbon on \(C\) of arithmetic genus with conormal bundle \(L\). Let \(e \in \Ext^1(K_C,L)\) be the extension class of \[ 0 \to L \to \Omega_{\widetilde C}|_C \to K_C \to 0.\] The blow-up index of \(\widetilde C\) is any of the following equal quantities: \begin{enumerate} \item the minimum \(b\) such that there exists an effective divisor \(\beta\) on \(C\) of degree \(b\) such that the image of \(e\) under the push-out by \(L \to L(\beta)\) vanishes, \item the minimum \(b\) such that there exists an effective divisor \(\beta\) on \(C\) of degree \(b\) such that the image of \(e\) under the pull-back by \(K_C(-\beta) \to K_{C}\) vanishes. \end{enumerate} If $p_a > g+1$, then it is also equal to \(2g-2-k\) where \(k\) is the maximum such that \(\Omega_{\widetilde C}|_C\) has a sub line-bundle of degree \(k\). \end{proposition} \begin{proof} The quantity (1) is the definition of the blow-up index (\Cref{blow-up index}). The equality of (1) and (2) follows because the push-out \(L \to L(\beta)\) and pull-back \(K_C(-\beta) \to K_C\) induce the same map \[ \Ext^{1}(K_C, L) \to \Ext^1(K_C, L(\beta)) = \Ext^1(K_C(-\beta), L).\] We now prove the last statement. Note that the extension obtained by the pull-back along \(K_C(-\beta) \to K_C\) splits if and only if there is a map \(K_C(-\beta) \to \Omega_{\widetilde C}|_{C}\) such that the following diagram commutes \[ \begin{tikzcd} K_C(-\beta) \arrow[d] \arrow[dr] & \\ \Omega_{\widetilde{C}}|_C \arrow[r] & K_C \end{tikzcd} \] In this case, \(K_C(-\beta)\) is a sub-bundle of \(\Omega_{\widetilde C}|_C\) of degree \(2g-2-b\). So \((3) \leq (2)\). For the reverse inequality, let \(M \subset \Omega_{\widetilde C}|_{C}\) be a sub line-bundle of largest degree \(k\). By \Cref{Using secant varieties to compute blow-up index} we know that the blow-up index of \(\widetilde C\) is at most \(\lceil 1/2(p_a+g-2)\rceil\). So \(\Omega_{\widetilde C}|_C\) has a line sub-bundle of degree at least \(2g-2-\lceil 1/2(p_a+g-2)\rceil\). The condition \(p_a > g+1\) ensures that \[ 2g-2-\lceil 1/2(p_a+g-2)\rceil > \deg L = -p_a+2g-1.\] Therefore \(\deg M = k> \deg L\), and so \(M\) does not admit a non-zero map to \(L\). It follows that the composite \(M \to \Omega_{\widetilde C}|_C \to K_C\) is non-zero. So \(M\) has the form \(K_C(-\beta)\) for some effective divisor \(\beta\) of degree \(2g-2-k\). We conclude that \((2) \leq (3)\). \end{proof} As a corollary, we relate the blow-up index with the stability of the rank 2 bundle \(\Omega_{\widetilde C}|_{C}\). \begin{corollary} Let $\widetilde{C}$ be a ribbon of arithmetic genus $p_a > g+1$ on a smooth curve of genus $g$. Then $\Omega_{\widetilde{C}}|_C$ is stable (resp. semistable) if and only if $b(\widetilde{C}) > 1/2(p_a+3)-2$ (resp. $b(\widetilde{C}) \geq 1/2(p_a+3)-2$). \end{corollary} \begin{proof} Note that \[\deg(\Omega_{\widetilde{C}}|_C) = -\deg(L)+2g-2 = -(p_a-2g+1)+2g-2 = -(p_a+3)+4g.\] So the slope of $\Omega_{\widetilde{C}}|_C$ is $-1/2(p_a+3)+2g$. The statement now follows from \Cref{equivalent definitions of blow-up index}. \end{proof} \begin{remark} For ribbons of blow-up index between $-1/2(p_a+3)-2$ and the maximum, which is $\lceil 1/2(p_a+g-2)\rceil$, the bundles $\Omega_{\widetilde C}|_{C}$ are semi-stable. But they are still distinguished by the maximum degree of sub line-bundles. \end{remark} \subsection{Blow-up index, gonality, and linear series Clifford index} We end this section with bounds on the gonality of a ribbon in terms of the blow-up index. Recall that the gonality of $\widetilde C$ is the minimum $d$ such that there exists a generalized $g^1_d$ on $\widetilde C$. The following gives an upper bound on gonality. \begin{proposition}\label{blow up index and linear series Cliff of a ribbon} Let $C$ be a smooth curve of genus $g$ and gonality $m$. Let $\widetilde{C}$ be a ribbon $C$ of arithmetic genus $p_a$, blow-up index $b$, and gonality $d$. Assume there exist a smooth divisor in $|-2L|$ and that $p_a > 2g-1+2m$, then \begin{equation*} d \leq \min(b+2m, \lfloor 1/2(p_a+3) \rfloor). \end{equation*} \end{proposition} \begin{proof} Let $\widetilde{C}'$ be the split ribbon obtained by blowing up $\widetilde{C}$ along a divisor of degree $b$. Then we have a degree 2 map $\widetilde{C'} \to C$. The pull-back of a $g^{1}_m$ on $C$ yields a $g^1_{2m}$ on $\widetilde C'$. Its push-forward to $\widetilde C$ gives a $g^1_{b+2m}$ on $\widetilde C$. So $d \leq b+2m$. Finally, in the prescribed range, all ribbons are smoothable \cite{Gon06}. A smooth curve of genus $p_a$ has gonality at most $\lfloor 1/2(p_a+3)\rfloor$. So the bound $d \leq \lfloor 1/2(p_a+3)\rfloor$ follows from semi-continuity (\Cref{semicontinuity of gonality}). \end{proof} The following gives a lower bound on gonality. \begin{proposition}\label{lower bound to gonality} Let $C$ be a smooth curve of genus $g$ and gonality $m$. Let $\widetilde{C}$ be a ribbon $C$ of arithmetic genus $p_a$, blow-up index $b$, and gonality $d$. Then \[d \geq b-(2g-2).\] \end{proposition} \begin{proof} Let $\widetilde{M}$ denote the generalized line bundle associated to the $g^1_d$ on $\widetilde{C}$. We observe that $\widetilde{M}$ must be generated by the 2 sections of the $g^1_d$. If not, the subsheaf generated by these two sections gives a generalised $g^1_{d'}$ with $d' < d$, contradicting the minimality of $d$. Let us first treat the case that $\widetilde{M}$ is a line bundle. Since $\widetilde{M}$ is a line bundle, $d$ is even. Then the $g^1_d$ on $\widetilde C$ induces a finite map $\widetilde C \to \mathbb P^1$ of degree $d$, leading to the diagram \[ \begin{tikzcd} C \arrow[d, hook] \arrow[dr]{f} & \\ \widetilde{C} \arrow[r, ] & \mathbb{P}^1. \end{tikzcd} \] The diagram above induces the diagram \[ \begin{tikzcd} f^*(K_{\mathbb{P}^1}) \arrow[d, hook]\arrow[dr, hook] & \\ \Omega_{\widetilde{C}}|_C \arrow[r] & K_C. \end{tikzcd} \] Note that $f \colon C \to \mathbb P^1$ is finite of degree $d/2$. We have the identification $f^{*}(K_{\mathbb P^1}) = K_C(-\beta)$, where $\beta$ is the ramification divisor of $f$, which has degree $2g-2+d$. Owing to this diagram, the pull-back of the extension \[ 0 \to L \to \Omega_{\widetilde C}|_{C} \to K_C \to 0\] to $K_C(-\beta)$ is split. Therefore, by \Cref{equivalent definitions of blow-up index}, we get $b \leq 2g-2+d$. Let us treat the general case, where $\widetilde M$ is only a generalized line bundle. Then there exists an effective divisor $\alpha$ of degree $a$ such that $\widetilde M$ is the push-forward of a line bundle $\widetilde M'$ on the blow-up $\widetilde C'$ of $\widetilde C$ at $a$. Then we have a $g^1_{d-a}$ on $\widetilde C'$ with the line bundle $\widetilde M'$. Plainly, the blow-up index $b'$ of $\widetilde C'$ is bounded below by $b-a$. So we get \[ d-a \geq b' - (2g-2) \geq b-a - (2g-2),\] and hence $d \geq b-(2g-2)$, as required. \end{proof} \section{Gonality stratification of the space of ribbons}\label{sec:gonality} In this section we describe the stratification of the space of ribbons by gonality. Fix a smooth curve $C$ of genus $g$ and a line bundle $L$ on $C$ of negative degree. Then the space of ribbons on $C$ with conormal bundle $L$ is naturally the projective space $\mathbb P H^0(K_C^{2} \otimes L^{-1})$. Let \[W_d \subset \mathbb P H^0(K_C^{2} \otimes L^{-1})\] be the subset of ribbons that carry a generalized $g^1_d$. Let \[\Sec_k(C) \subset \mathbb P H^0(K_C^{2} \otimes L^{-1})\] be the $k$-secant variety of $C \subset \mathbb P H^0(K_C^{2} \otimes L^{-1})$. \begin{theorem}\label{gonality stratification} With the notation above, $W_d \subset \Sec_{d+2g-2}(C)$ is the union of linear subspaces spanned by divisors of degree $d+2g-2$ that contain the ramification divisor of a map $C \to \mathbb P^1$ of degree at most $d/2$. \end{theorem} \begin{proof} Let $\widetilde C \in W_{d}$. Then $\widetilde{C}$ carries a $g^1_d$, say $\widetilde M$. We show that $\widetilde C$ lies on a linear subspace spanned by a divisor of degree $d+2g-2$ that contains a ramification divisor of a map $C \to \mathbb P^1$ of degree at most $d/2$. It suffices to treat the case when the two sections of the $g^1_d$ genarate $\widetilde M$. (If not, we simply replace $\widetilde M$ by the subsheaf they generate). The generalised line bundle $\widetilde M$ is the push-forward of a line bundle $\widetilde M'$ on a blow-up $\widetilde C'$ of $\widetilde C$ in a divisor $\beta \subset C$ of degree $b$. Note that the degree of $\widetilde M'$ is $d-b$. The two sections of $\widetilde M'$ generate $\widetilde M'$ and hence yield a map $\widetilde C' \to \mathbb P^1$ of degree $d-b$. Its restriction $f \colon C \to \mathbb P^{1}$ is a map of degree $e = (d-b)/2$. Let $R$ be the ramification divisor of $f$. By \cite[Theorem~1.6]{BE95}, we have a map $K_C(-R) \to \Omega_{\widetilde C'}|_{C}$ that makes the following diagram commute \begin{equation}\label{eqn:Rsplit} \begin{tikzcd} & & & f^*(K_{\mathbb{P}^1}) = K_C(-R) \arrow[d]\arrow[dl] & \\ 0 \arrow[r] & L(\beta) \arrow[r] & \Omega_{\widetilde{C}'}|_C \arrow[r] & K_C \arrow[r] & 0. \end{tikzcd} \end{equation} By \Cref{equivalent definitions of blow-up index}, this means that $\widetilde C'$ splits after blowing it up in $R$. Then $\widetilde C$ splits after blowing it up in $\beta + R$. Note that $\beta + R$ has degree $d+2g-2$ and contains the ramification divisor $R$. Conversely, let $R \subset C$ be the ramification divisor of a map $f \colon C \to \mathbb P^{1}$ of degree $e \leq d/2$, and let $\widetilde C$ lie in the linear span of a divisor of degree $d+2g-2$ of the form $\beta + R$, where $\beta$ is effective. We produce a generalized $g^1_d$ on $\widetilde C$. Since $\widetilde C$ lies in the span of $\beta + R$, the blow-up of $\widetilde C$ in $\beta + R$ is split (\Cref{blow-up index and secant variety}). Let $\widetilde C'$ be the blow-up of $\widetilde C$ in $\beta$, so that the blow-up of $\widetilde C'$ in $R$ is split. Then we have a map $K_C(-R) \to \Omega_{\widetilde C'}|_{C}$ making the diagram \eqref{eqn:Rsplit} commute. Applying \cite[Theorem $1.6$]{BE95} again, we conclude that the map $f \colon C \to \mathbb P^1$ extends to a map $\widetilde C' \to \mathbb P^{1}$. Then the pull-back of $\mathcal O(1)$ is a $g^1_{2e}$ on $\widetilde C'$, whose push-forward to $\widetilde C$ is a $g^1_d$ on $\widetilde C$. \end{proof} \begin{corollary}\label{inclusion of gonality stratification in secant variety} We have the inclusions \[ \Sec_{d-2m}(C) \subset W_d \subset \Sec_{d+2g-2}(C).\] \end{corollary} \begin{proof} The second inclusion is a part of \Cref{gonality stratification}. For the first, take $x \in \Sec_{d-2m}(C)$. Then $x$ is in the span of a divisor $\alpha$ of degree $d-2m$. Since $m$ is the gonality of $C$, we have a map $C \to \mathbb P^1$ of degree $m$. Its ramification divisor $R$ has degree $2g-2+2m$. Then $x$ is also in the span of $\alpha + R$. \end{proof} \Cref{gonality stratification} allows us to count the dimension of each gonality stratum. To do so, let us further stratify $W_d$. For a non-negative integer $e \leq d/2$, set $b = d-2e$. Let $W_{e,b} \subset W_d$ be the union of linear subspaces spanned by divisors of degree $d+2g-2$ that contain the ramification divisor of a map $C \to \mathbb P^1$ of degree $e$. Then, by definition, we have \[W_d = \bigcup_{e = 0}^{d/2} W_{e,b}.\] From the proof of \Cref{gonality stratification}, we see that the points of $W_{e,b}$ correspond to ribbons $\widetilde C$ that carry a generalised $g^1_d$ that is the push-forward of a line bundle from a blow-up of $\widetilde C$ in a divisor of degree $b$. Let $m$ be the gonality of $C$ and let $W^1_e(C)$ be the moduli space of base-point free linear series of rank $1$ and degree $e$ on $C$. \begin{theorem}\label{dimension of W_d} Retain the setup above and assume that $L$ is a line bundle of degree $\leq -2m-5$. Set $p_a = 2g-1- \deg L$. Then \[\dim(\overline W_{e,b}) \leq \dim W^1_e(C)+ 2g+2e+2b-3.\] In addition, if $C$ is a general curve of genus $g$, the following hold. \begin{enumerate} \item If $2e < g+2$, then $\overline W_{e,b}$ is empty, and otherwise, we have \[\dim(\overline W_{e,b}) \leq g+2d-5.\] \item The gonality of a general ribbon on \(C\) with conormal bundle $L$ is the maximum, namely $\lfloor 1/2(p_a+3)\rfloor$. \end{enumerate} \end{theorem} \begin{proof} Consider a point \((\phi, \beta) \in W^1_e(C) \times \Sym^b(C)\). Let \(R_{\phi}\) be the ramification divisor of \(\phi\). Consider the span of \(R + \beta\) in \(\mathbf{P} H^0(K_C ^{2} \otimes L^{-1})\). For \((\phi, \beta)\) in a non-empty Zariski open subset \(U \subset W^1_e(C) \times \Sym^b(C)\), this span has a constant dimension \(n\). Since \(\deg R = 2g+2e-2\) and \(\deg \beta = b\), we have \(n \leq 2g+2e+b-3\). Consider the incidence variety \[ I \subset U \times \mathbb P H^0(K_C^{2} \otimes L^{-1})\] consisting of $(\phi, \beta, x)$ such that $x$ lies on the span of $R_{\phi} + \beta$. Then $I$ is a closed subvariety and its projection to $\mathbf{P} H^0(K_C^2 \otimes L^{-1})$ contains a Zariski open subset of $W_{e,b}$. The fibers of $I \to U$ are projective spaces of dimension $n$. Therefore, we get \begin{align*} \dim \overline W_{e,b} &\leq \dim I = \dim U + n = \dim W^1_e(C) + b + n\\ &\leq \dim W^1_e(C) + b + (2g+2e+b-3), \end{align*} as required. If $C$ is general, then $\dim W^1_e(C) = 2e-g-2$ (and $W^1_e(C)$ is empty if $2e < g+2$). Since $d = 2e + b$, we get \[ \dim \overline W_{e,b} \leq g+2d-5.\] The dimension of the ambient $\dim \mathbf{P} H^0(K_c^2 \otimes L^{-1})$ is $g+p_a-3$. Therefore, if \[g+2d-5 < g+p_a-3,\] then $\overline W_{e,b} \subset \mathbf{P} H^0(K_c^2 \otimes L^{-1})$ is a proper subset. That is, a generic point of the ambient space is contained in $W_{d}$ only when $d \geq \lfloor (p_a+3)/2 \rfloor$. In other words, a generic ribbon has gonality at least $\lfloor (p_a+3)/2 \rfloor$. To see that a generic ribbon has gonality at most $\lfloor (p_a+3)/2 \rfloor$, observe that such a ribbon is a limit of smooth curves and smooth curves have gonality at most $\lfloor (p_a+3)/2 \rfloor$. Then apply the semi-continuity of gonality (\Cref{semicontinuity of gonality}). \end{proof} We end the section with some examples of the gonality loci of ribbons. In all the examples, \(C\) will be a smooth curve of genus \(g\) and \(L\) a line bundle of negative degree on \(C\). We set \[p_a = -\deg L + 2g - 1.\] We consider \(C\) embedded in \(\mathbf{P}H^0(K_C^{\otimes 2} \otimes L^{-1})^{*}\) by the complete linear series. Recall that the points of \(\mathbf{P}H^0(K_C^{\otimes 2} \otimes L^{-1})^{*}\) correspond to ribbons on \(C\) of arithmetic genus \(p_a\) and with conormal bundle \(L\). \begin{example}[Ribbons on hyperelliptic curves] \label{ex:hyperelliptic} Let \(C\) be a hyperelliptic curve of genus \(g\). We describe \(W_{e,b}\) explicitly for \(e = 2\). Assume for simplicity that \(p_a \geq 2g+5\). Let \(R\) be the ramification divisor of the degree 2 map \(C \to \mathbf{P}^1\). Fix \(e = 2\) and \(d \geq 4\) and \(b = d-2e = d-4\). By \Cref{gonality stratification}, \(W_{2,d-4} \subset \mathbf{P}H^0(K_C^{\otimes 2} \otimes L^{-1})^{*}\) is the union of the span of \(R + \beta\) as \(\beta\) varies in \(\operatorname{Sym}^b(C)\). Let \(\langle R \rangle \subset \mathbf{P}H^0(K_C^{\otimes 2} \otimes L^{-1})^{*}\) be the span of \(R\). Then \(\langle R \rangle\) is a projective space of dimension \(2g+1\). We have the linear projection \[ \mathbf{P}H^0(K_C^{\otimes 2} \otimes L^{-1})^{*} \dashrightarrow \mathbf{P}H^0(K_C^{\otimes 2} \otimes L^{-1}(-R))^{*}\] with center \(\langle R \rangle\). The projection of \(C\) yields an embedding \(C \subset \mathbf{P}H^0(K_C^{\otimes 2} \otimes L^{-1}(-R))^{*}\) by the complete linear series. Then \(\overline W_{2,d-4} \subset \mathbf{P}H^0(K_C^{\otimes 2} \otimes L^{-1})^{*}\) is the cone over the \(b\)-secant variety of \(C\) in \(\mathbf{P}H^0(K_C^{\otimes 2} \otimes L^{-1}(-R))^{*}\). Note that the ambient space \(\mathbf{P}H^0(K_C^{\otimes 2} \otimes L^{-1}(-R))^{*}\) has dimension \(p_a-g-5\). Hence the \(b\)-secant variety has dimension \( \min(2b-1, p_a-g-5)\). Therefore, we get \[ \dim \overline W_{2,d-4} = \min(2d+2g-7, p_a+g-3).\] Note that \(2g+2g-7\) is the bound given by \Cref{dimension of W_d} and \(p_a+g-3\) is simply the dimension of the ambient projective space \(\mathbf{P}(K_C^{\otimes 2} \otimes L^{-1})^{*}\). Observe that if \(d \geq (p_a-g+4)/2\), then \(\overline W_{2,d-4}\) is the ambient projective space. In particular, the gonality of a generic ribbon over a hyperelliptic curve is less than the maximum \(\lfloor(p_a+3)/2\rfloor\). \end{example} \begin{example}[Non-existence of a limiting \(g^1_3\)] \label{ex:ellipticg13} Let \(C\) be an elliptic curve and take \(p_a = 4\). Then \(C\) is a limit of smooth curves of genus 4, which have gonality 3. Nevertheless, from \Cref{gonality stratification} it follows that \(W_3\) is empty. So \(C\) does not admit a generalised \(g^1_3\). This does not contradict the semi-continuity of gonality (\Cref{semicontinuity of gonality}), since the condition \(p_a > d + 2g-1\) is not satisfied. \end{example} \begin{example}[An elliptic normal curve in \(\mathbf{P}^5\)] Let \(C\) be a smooth curve of genus \(1\) and take \(p_a = 7\). Then the conormal bundle \(L\) has degree \(-6\). The complete linear series gives an embedding \[C \subset \mathbf{P}(K_C^{\otimes 2} \otimes L^{-1})^{*} = \mathbf{P}^5.\] \Cref{gonality stratification} implies that \(W_{d}\) is empty for \(d = 1,2,3\). We describe \(W_4\). We have \(W_4 = W_{2,0}\) and it is the union of the spans of the ramification divisors of degree \(2\) maps \(C \to \mathbf{P}^1\). Fix one degree \(2\) map \(f \colon C \to \mathbf{P}^1\); all others are translates of \(f\). Let \(R = p_1 + p_2 + p_3 + p_4\) be the ramification divisor of \(f\) and \(P \subset \mathbf{P}^5\) the \(\mathbf{P}^3\) spanned by \(R\). Then \(W_4\) is the union of the translates of \(P\) by the points of the Jacobian of \(C\). This is a hypersurface of degree 6. \end{example} \section{ Green's conjecture and resolution Clifford index for a general ribbon on a general curve}\label{sec:green_general} In this section, we relate the linear series Clifford index to the resolution Clifford index. Specifically, we prove Green's conjecture for a general ribbon using Green's conjecture for a general smooth curve. The proof follows the ideas in \cite{D18}. We begin with two results about embedded and abstract deformations of ribbons. \begin{proposition}\label{unobstructedness of embedded ribbons} Let $\widetilde{C}$ be a ribbon of arithmetic genus $p_a$ on a smooth curve $C$ of genus $g$. Suppose $p_a \geq 4g-2$. Let $\widetilde C \subset \mathbf{P}^N$ be the embedding by a complete linear series of a very ample line bundle of degree at least $2p_a$. Then the deformations of $\widetilde C \subset \mathbf{P}^N$ are unobstructed. That is, the Hilbert scheme of $\mathbf{P}^N$ is smooth at the point represented by $[\widetilde C]$. \end{proposition} \begin{proof} Let $L$ be the conormal bundle of the ribbon $\widetilde{C}$. We have the sequence \begin{equation}\label{eqn:c1} 0 \to T_C \to T_{\mathbf{P}^N} \otimes \mathcal{O}_C \to N_{C/\mathbf{P}^N} \to 0 \end{equation} and the Euler exact sequence \begin{equation}\label{eqn:c2} 0 \to \mathcal{O}_C \to \mathcal{O}_C(1)^{\oplus N} \to T_{\mathbf{P}^N} \otimes \mathcal{O}_C \to 0. \end{equation} Note that \(\deg \mathcal{O}_C(1) \geq p_a\) and \(\deg \mathcal{O}_C(1) \otimes L \geq 2g-1\). So both \(\mathcal{O}_C(1)\) and \(\mathcal{O}_C(1) \otimes L\) have vanishing \(H^1\). From the long exact sequence in cohomology applied to \eqref{eqn:c2} and its twist by \(L\), we get \begin{equation}\label{eqn:ct} H^{1}(T_{\mathbf{P}^N} \otimes \mathcal{O}_C) = 0 \text{ and } H^{1}(T_{\mathbf{P}^N} \otimes L) = 0. \end{equation} As a result, from the long exact sequence in cohomology applied to \eqref{eqn:c1} and its twist by \(L\), we get \begin{equation}\label{eqn:c} H^1(N_{C/\mathbf{P}^N}) = 0 \text{ and } H^1(N_{C/\mathbf{P}^N} \otimes L) = 0. \end{equation} We now turn to the normal bundle of \(\widetilde C\). Since $\widetilde{C}$ is a local complete intersection, its normal bundle is locally free of rank \(N-1\). Restricting it to \(C\) yields the exact sequence \begin{equation}\label{1} 0 \to N_{\widetilde{C}/\mathbb{P}^N} \otimes L \to N_{\widetilde{C}/\mathbb{P}^N} \to N_{\widetilde{C}/\mathbb{P}^N} \otimes \mathcal{O}_C \to 0. \end{equation} We have the exact sequence \[ 0 \to I_C^2/ (I_C \cdot I_{\widetilde C}) \to I_{\widetilde C} / (I_C \cdot I_{\widetilde C}) \to I_{\widetilde C}/I_C^2 \to 0,\] whose terms are locally free \(\mathcal{O}_C\)-modules of ranks \(1\), \(N-1\), and \(N-2\), respectively. Specifically, the kernel is the line bundle \(L^2\) and the middle term is the conormal bundle of \(\widetilde C\) restricted to \(C\). Applying \(\Hom(-,\mathcal{O}_C)\) yields the sequence \begin{equation}\label{2} 0 \to \Hom(I_{\widetilde{C}}/I_{C}^2, \mathcal{O}_C) \to N_{\widetilde{C}/\mathbb{P}^N} \otimes \mathcal{O}_C \to L^{-2} \to 0 \end{equation} Finally, we have the sequence \[ 0 \to I_{\widetilde C}/I_C^{2} \to I_C/I_C^2 \to I_C/I_{\widetilde C} \to 0,\] whose terms are locally free \(\mathcal{O}_C\)-modules of ranks \(N-2\), \(N-1\), and \(1\), respectively. Specifically, the cokernel is the line bundle \(L\) and the middle term is the conormal bundle of \(C\). Applying \(\Hom(-,\mathcal{O}_C)\) yields the sequence \begin{equation}\label{3} 0 \to L^{-1} \to N_{C/\mathbb{P}^N} \to \Hom(I_{\widetilde{C}}/I_{C}^2, \mathcal{O}_C) \to 0. \end{equation} Using the vanishings \eqref{eqn:c}, the long exact sequence in cohomology applied to \eqref{3} and its twist by \(L\) yields \begin{equation}\label{eqn:int} H^1(\Hom(I_{\widetilde C}/I_C^2, \mathcal{O}_C)) = 0 \text{ and } H^1(\Hom(I_{\widetilde C}/I_C^2, \mathcal{O}_C) \otimes L) = 0. \end{equation} Note that \(\deg L^{-2} \geq \deg L^{-1} = p_a-2g+1 \geq 2g-1\). So \(H^1(L^{-2}) = H^1(L^{-1}) = 0\). Combined with the vanishing \eqref{eqn:int}, the long exact sequence in cohomology applied to \eqref{2} and its twist by \(L\) yields \begin{equation}\label{eqn:cc} H^1(N_{\widetilde C/\mathbf{P}^N} \otimes \mathcal{O}_C) = 0 \text{ and } H^1(N_{\widetilde C/\mathbf{P}^N} \otimes \mathcal{O}_C \otimes L) = 0. \end{equation} Finally, the long exact sequence in cohomology applied to \eqref{1} yields \[ H^1(N_{\widetilde C/\mathbf{P}^N}) = 0.\] So the embedded deformations of \(\widetilde C \subset \mathbf{P}^N\) are unobstructed. \end{proof} We now turn to the abstract deformations of \(\widetilde C\). \begin{proposition}\label{unobstructedness of ribbons} Let $\widetilde{C}$ be a ribbon of arithmetic genus $p_a$ on a smooth curve $C$ of genus $g$. Suppose $p_a \geq 4g-2$. Then the deformations of $\widetilde{C}$ are unobstructed. That is, a versal deformation space of $\widetilde{C}$ is smooth. \end{proposition} \begin{proof} Choose an embedding $\widetilde C \subset \mathbf{P}^{N}$ by the complete linear series of a very ample line bundle of degree at least $2p_a$. We relate the abstract versus embedded deformations of $\widetilde C$. Consider the conormal sequence \[0 \to \mathcal{I}/\mathcal{I}^2 \to \Omega_{\mathbb{P}^N}|_{\widetilde{C}} \to \Omega_{\widetilde{C}} \to 0.\] Applying $\Hom(-, \mathcal{O}_{\widetilde{C}})$ yields \begin{equation}\label{eqn:longext} \Hom(\mathcal{I}/\mathcal{I}^2, \mathcal{O}_{\widetilde{C}}) \to \Ext^1(\Omega_{\widetilde{C}}, \mathcal{O}_{\widetilde{C}}) \to \Ext^1(\Omega_{\mathbb{P}^N}|_{\widetilde{C}}, \mathcal{O}_{\widetilde{C}}) \to \Ext^1(\mathcal{I}/\mathcal{I}^2, \mathcal{O}_{\widetilde{C}}) \to \Ext^2(\Omega_{\widetilde{C}}, \mathcal{O}_{\widetilde{C}}). \end{equation} Consider $\Ext^1(\Omega_{\mathbb{P}^N}|_{\widetilde{C}},\mathcal{O}_{\widetilde{C}}) = H^1(T_{\mathbb{P}^N}|_{\widetilde{C}}).$ We have the exact sequence \[ 0 \to T_{\mathbf{P}^N}|_C \otimes L \to T_{\mathbf{P}^N}|_{\widetilde C} \to T_{\mathbf{P}^N|_C} \to 0.\] By \eqref{eqn:ct}, the \(H^1\) of the first and the third term vanishes. So \(H^1\) of the middle term also vanishes. Now, \eqref{eqn:longext} says that \[ H^0(N_{\widetilde C/\mathbf{P}^N}) \to \Ext^1(\Omega_{\widetilde C}, \mathcal{O}_{\widetilde C})\] is surjective, and \[ H^1(N_{\widetilde C/\mathbf{P}^N}) \to \Ext^2(\Omega_{\widetilde C}, \mathcal{O}_{\widetilde C})\] is injective. That is, the forgetful map from embedded to abstract deformations is surjective on tangent spaces and injective on obstruction spaces. As a result, the forgetful map is smooth. We know that the embedded deformation space is smooth (\Cref{unobstructedness of embedded ribbons}). We conclude that the abstract deformation space is also smooth. \end{proof} We examine the non-nodal locus in a versal deformation. \begin{proposition}\label{worse than nodal} Let $\widetilde{C}$ be a ribbon of arithmetic genus $p_a$ on a smooth curve of genus $g$ with $p_a \geq 3g-1$. Let \((U,0)\) be a versal deformation space of \(\widetilde C\) with a versal family of deformations $\mathcal{C} \to U$. Let $N \subset U$ be the closed subset containing $u \in U$ such that $\mathcal{C}_u$ has worse than nodal singularities. Then \(N \subset U\) has codimension at least $2$ at \(0\). \end{proposition} \begin{proof} We follow the proof of \cite[Proposition~3]{D18}. Since every ribbon $\widetilde{C}$ can be isotrivially deformed to a split ribbon, it is enough to prove the theorem for the split ribbon. So assume that $\widetilde{C}$ is the split ribbon. Let the conormal bundle be $L$ and let \(V = H^0(-2L)\). Every \(v \in V\) gives a double cover of \(C\) whose branch divisor is the zero locus of \(v\) and whose structure sheaf is \(\operatorname{O}_C \oplus L\). Consider the family of curves \(\mathcal C_V \to V\), whose fiber over \(v \in V\) is the double of \(C\) defined by \(v\). Note that the fiber over \(0\) is the split ribbon \(\mathcal C\). By versality, possibly after passing to an \'etale neighborhood of \(0\), we have a map \((V,0) \to (U,0)\) and an isomorphism of \(\mathcal C_V \to V\) with the pull-back of the versal family. Let \(N_V \subset V\) be the pre-image of \(N \subset U\). Then \(N_V \subset V\) is the locus of \(v \in V\) that define a worse than nodal \(\mathcal C_v\). But the double cover \(\mathcal C_v\) is has worse than nodal singularities if and only if \(v\) has a zero of multiplicity at least 3. Since \(\deg (-L) = p_a-2g+1 \geq g\), we have \(\deg(-2L) \geq 2g\). Then it follows that \(N_V \subset V\) has codimension 2 at \(0\). Therefore, \(N_U \subset U\) has codimension at least 2 at \(0\). \end{proof} As in \cite{D18}, let $\mathcal{U}$ denote the open sub-stack of the stack of all projective curves whose points corresond to curves $X$ satisfying the following conditions: \begin{enumerate} \item $X$ is Gorenstein of arithmetic genus $p_a$ and $h^0(\mathcal{O}_{X}) = 1$. \item $K_{X}$ embeds $X$ as a arithmetically Gorenstein subscheme of $\mathbb{P}^{p_a-1}$. \item A versal deformation space of $X$ is smooth. \end{enumerate} Let $M_{p_a}$ be the stack of smooth projective curves of genus $p_a$, and let $M_{p_a}^{nh} \subset M_{p_a}$ be the stack of non-hyperelliptic curves. Let $C$ be a smooth curve of genus $g$ and let $\widetilde C$ be a ribbon on $C$ of arithmetic genus $p_a$ with $p_a \geq 4g-2$. Then $X = \widetilde C$ satisfies the three conditions above: the first is clear; the second is \Cref{canonical morphism of ribbons}; and the third is \Cref{unobstructedness of ribbons}. Therefore, $\widetilde{C}$ defines a point $[\widetilde{C}]$ of $\mathcal{U}$. Let $p_a$ be odd, say $p_{a} = 2k+1$. As shown in \cite[Proposition~4]{D18}, we have a divisor $D \subset \mathcal U$ whose points represent curves $X$ such that $K_{k-1,1}(X, K_X) \neq 0$. We also have a divisor in $M_{p_a}^{nh}$ whose points represent smooth curves of gonality $k+1$. Let $D_{k+1} \subset \mathcal U$ be its closure. \begin{proposition}\label{equality of the divisors} Let $\widetilde{C}$ be a ribbon of arithmetic genus $p_a = 2k+1$ on a smooth projective curve $C$ of genus $g$ with $p_a \geq 4g-2$. Then $D_{k+1} = D$ on an open subset of $\mathcal{U}$ containing $[\widetilde{C}]$. \end{proposition} \begin{proof} The proof of \cite[Proposition~5]{D18} applies verbatim, thanks to \Cref{worse than nodal}. \end{proof} \begin{proposition}[Green's conjecture for generic ribbons of odd genus] \label{odd genus maximum blow-up index} Let $\widetilde{C}$ be a ribbon of arithmetic genus $p_a = 2k+1$ on a smooth projective curve $C$ of genus $g$ with $p_a \geq \operatorname{max}\{4g+2, 6g-4\}$. Suppose $\widetilde{C}$ has gonality $k+2$. Then its resolution Clifford index is $k$. That is, we have \[ \RCliff(\widetilde C) = \LCliff(\widetilde C) = k.\] In particular, the statement applies to a general ribbon of odd arithmetic genus $p_a \geq \operatorname{max}\{3g+7, 6g-4\}$ on a general curve of genus $g$. \end{proposition} \begin{proof} By the semi-continuity of gonality (\Cref{semicontinuity of gonality}), we see that $[\widetilde{C}] \notin D_{k+1}$. Therefore, by \Cref{equality of the divisors}, $[\widetilde{C}] \notin D$, so $K_{k-1,1}(\widetilde{C}, K_{\widetilde{C}}) = 0$. That is, $\RCliff(\widetilde C) \geq k$. But the lower bound on $p_a$ further ensures that $\widetilde{C}$ is smoothable (see \cite{Gon06} or \cite{GGP08}). By semi-continuity, $\RCliff(\widetilde C)$ is at most $\RCliff$ of a general curve of genus $2k+1$; that is, $\RCliff(\widetilde C) \leq k$. So $\RCliff(\widetilde C) = k$. Since $\widetilde C$ is $(k+2)$-gonal, we have $\LCliff(\widetilde C) \leq k$. But we know that $\LCliff(\widetilde C) \geq \RCliff(\widetilde C)$ (\Cref{lin and res}). So $\LCliff(\widetilde C) = k$. The last assertion follows from \Cref{dimension of W_d}, $(2)$. \end{proof} The case of general ribbons of even arithmetic genus follows using blow-ups. \begin{proposition}[Green's conjecture for generic ribbons of even genus] \label{generic green even} Let $C$ be a general smooth curve of genus $g$. Fix an even non-negative integer $p_a = 2k \geq \operatorname{max}\{3g+7, 6g-4\}$ and a line bundle $L$ of degree $-p_a+2g-1$ on $C$. Then a general ribbon $\widetilde C$ on $C$ with conormal bundle $L$ satisfies \[ \RCliff(\widetilde C) = \LCliff(\widetilde C) = k-1.\] \end{proposition} \begin{proof} We have $\RCliff(\widetilde C) \leq \LCliff(\widetilde C)$ by \Cref{lin and res}, and $\LCliff(\widetilde C) \leq k-1$ by \Cref{semicontinuity of gonality} (or \Cref{blow up index and linear series Cliff of a ribbon}) since $\widetilde{C}$ is smoothable under the condition on $p_a$. It remains to prove that $k-1 \leq \RCliff(\widetilde C)$, that is, \[ K_{k,1}(\widetilde C, K_{\widetilde C}) = 0.\] By semi-continuity, it suffices to exhibit one $\widetilde C$ such that $K_{k,1}(\widetilde C, K_{\widetilde C}) = 0$. Choose a point $x \in C$ and let $\widetilde C'$ be a generic ribbon on $C$ with conormal bundle $L(-x)$. Let $\widetilde C$ be the blow-up of $\widetilde C'$ at $x$. Then $\widetilde C$ has conormal bundle $L$. Applying \Cref{odd genus maximum blow-up index} to $\widetilde C'$, we get \[ K_{k,1}(\widetilde C', K_{\widetilde C'}) = 0.\] By \cite[Lemma~1]{D18}, we deduce \[ K_{k,1}(\widetilde C, K_{\widetilde C}) = 0.\] The proof is now complete. \end{proof} The same idea allows us to obtain a lower bound on the resolution Clifford index of general ribbons of every blow-up index. But we have to relinquish control over the conormal bundle. \begin{proposition}\label{any genus and any blowup index} Let \(C\) be a general smooth curve of genus \(g\). Fix a non-negative integer $\operatorname{max}\{3g+7, 6g-4\}$ . Let \(\widetilde C\) be a general ribbon on \(C\) of arithmetic genus $p_a$ and blow-up index \(b\). Set $m = \lfloor (g+3)/2\rfloor$. Except in the case where $p_a$ is even and $g$ is odd and $b = \lceil (p_a+g-2)/2\rceil$ is the maximum possible, we have \[b-\lfloor g/2 \rfloor \leq \RCliff(\widetilde C) \leq \LCliff(\widetilde C) \leq \min(b+2m-2, \lfloor 1/2(p_a-1) \rfloor).\] \end{proposition} Note that the excluded case is handled by \Cref{generic green even}. \begin{proof} Again, the second and the third inequalities follow from \Cref{lin and res} and \Cref{blow up index and linear series Cliff of a ribbon}. So we are left to prove the first inequality. By semi-continuity, it suffices to exhibit one ribbon $\widetilde C$ of arithmetic genus $p_a$ and blow-up index $b$ such that $b-\lfloor g/2 \rfloor \leq \RCliff(\widetilde C)$. The proof depends on the parity of $p_a$ and $g$. We fully explain the case when $p_a$ is even and $g$ is odd, and indicate the changes necessary in the other cases. Take \[ k = (p_a+g-2) - 2b.\] (If $g$ is even, add 1 to the right hand side.) Observe that $k \geq 0$ (in the excluded case, we have $k = -1$). Set $p_a' = p_a +k$, and observe that it is odd. Let $\widetilde C'$ be a general ribbon on $C$ of arithmetic genus $p_a'$. By \Cref{Using secant varieties to compute blow-up index}, the blow-up index of $\widetilde C'$ is \[ b' = (p'_a+g-2)/2 = b + k.\] Consider the sequence of $b'$ blow-ups that takes $\widetilde C'$ to the split ribbon. Let $\widetilde C$ be the ribbon obtained after $k$ blow-ups in this sequence. Then $\widetilde C$ has arithmetic genus $p_a$ and blow-up index $b$. By \Cref{odd genus maximum blow-up index} applied to $\widetilde C'$, we have \[ \RCliff(\widetilde C') = (p_a'-1)/2.\] That is, \[ K_{(p_a'-1)/2}(\widetilde C', K_{\widetilde C'}) = 0.\] By \cite[Lemma~1]{D18}, we conclude \[ K_{(p_a'-1)/2}(\widetilde C, K_{\widetilde C}) = 0,\] and therefore \begin{align*} \RCliff(\widetilde C) &\geq (p_a-2) - (p_a'-1)/2 + 1\\ &= (p_a-k-1)/2 \\ &= b-\lfloor g/2 \rfloor. \end{align*} The proof is now complete. \end{proof} \section{Gonality and Clifford index of a split ribbon}\label{sec:split} In this section we concentrate on the linear series Clifford index, gonality, and the resolution Clifford index of the split ribbon. \subsection{Linear series Clifford index of a split ribbon} We first find the linear series Clifford index. \begin{proposition}\label{lower bounds to linear series gonality and Clifford index} Let $\widetilde{C}$ be a ribbon of arithmetic genus $p_a$ on a smooth curve $C$ of genus $g$ and gonality $m$. \begin{enumerate} \item If $\widetilde{C}$ has a $g^1_d$ then $d \geq 2m$. \item Suppose the Clifford index of \(C\) is \(m-2\). Then, if $\widetilde{C}$ has a $g^r_d$ with \(d \leq p_a-1\), then $d-2r \geq 2m-2$ \end{enumerate} \end{proposition} \begin{proof} We begin with some set-up for both statements. Let \(L\) be the conormal bundle of \(\widetilde C\). Let \((V, \widetilde M)\) be a \(g^r_d\) on \(\widetilde C\). Then there exists a divisor \(\beta \subset C\) such that \(\widetilde M\) is the push-forward of a line bundle on the blow-up of \(\widetilde C\) at \(\beta\). Let \(b\) be the degree of \(\beta\). Let \(\widetilde C'\) be the blow-up of \(\widetilde C\) at \(\beta\) and \(\widetilde M'\) such a line bundle on \(\widetilde C'\) whose push-forward is \(\widetilde M\). Note that \(\widetilde C'\) has arithmetic genus \(p_a-b\) and conormal bundle \(L(\beta)\). Set \(M = \widetilde M|_C / \text{torsion}\). On \(\widetilde C'\), we have the exact sequence \[ 0 \to M \otimes L (\beta) \to \widetilde M' \to M \to 0.\] Set \(e = \deg M\), so that \(\deg \widetilde M' = 2e = d -b \). Having set-up the basic objects, we turn to the proof of \((1)\). Assume that \(r = 1\). Since \((V, \widetilde M)\) is a generalised \(g^1_d\), the restriction map \(V \to H^0(M)\) is injective. Therefore, \(h^0(M) \geq 2\). Since \(C\) has gonality \(m\), we conclude that \(e = \deg M \geq m\). As a result, \(d = 2e + b \geq 2m\). We now assume $r = 2$ and turn to the proof of \((2)\). We make two cases: \begin{enumerate} \item Suppose \(e \geq 2g-2\). Then \(h^1(M) \leq 1\) and so \(h^0(M) \leq 2-g+e\). As a result, \(r \leq 1-g+e\), and \[ d-2r = 2e+b-2r \geq 2g-2+b \geq 2m-2.\] \item Suppose \(e < 2g-2\). Then \(M\) contributes to the linear series Clifford index of \(C\). That is, we have \(e - 2r \geq \LCliff(C)\). As a result, using the fact that \(\LCliff(C) \geq m-3\), (see \cite{MG91}) we get \[ d-2r = 2e + b - 2r \geq 2\LCliff(C)+2r+b \geq 2m-2.\] \end{enumerate} \end{proof} \begin{corollary}\label{linear series Cliff of a split ribbon} Let \(C\) be a smooth curve of gonality \(m\). Let \(\widetilde C\) be a split ribbon of arithmetic genus \(p_a\). \begin{enumerate} \item The gonality of $\widetilde{C}$ is $2m$. \item The linear series Clifford index of $\widetilde{C}$ is \(2m-2\). \end{enumerate} \end{corollary} \begin{proof} The pull-back of a \(g^1_m\) on \(C\) yields a \(g^1_{2m}\) on \(\widetilde C\). Therefore, the gonality of \(\widetilde C\) is at most \(2m\) and the linear series Clifford index is at most \(2m-2\). The lower bounds follow from \Cref{lower bounds to linear series gonality and Clifford index}. \end{proof} As a result of \Cref{linear series Cliff of a split ribbon}, we see that Green's conjecture for the split ribbon implies Green's conjecture for any smooth double cover of \(C\). \begin{corollary}\label{green for split implies green for double cover} Let \(D\) be a smooth curve of genus \(p_a\) and \(f \colon D \to C\) a finite map of degree 2. Let \(L\) be the line bundle on \(C\) such that \(f_{*} \mathcal{O}_D \cong \mathcal{O}_C \oplus L\). Let \(\widetilde C\) be the split ribbon on \(C\) with conormal bundle \(L\). If \(\RCliff(\widetilde C) = \LCliff(\widetilde C)\), then \(\RCliff(D) = \LCliff(D)\). \end{corollary} \begin{proof} Let \(m\) be the gonality of \(C\). By \Cref{linear series Cliff of a split ribbon}, we have \[ \RCliff(\widetilde C) = 2m-2 = \gon(\widetilde C) - 2.\] Note that \(f \colon D \to C\) is specified by a global section of \(L^{-2}\). Scaling this section by \(t \in \mathbf{A}^1\) gives a family of double covers of \(C\) whose fiber at \(t = 0\) is \(\widetilde C\) and whose all other fibers are isomorphic to \(D\). By the semi-continuity of Koszul cohomology, we have, \[ \RCliff(\widetilde C) \leq \RCliff(D).\] Note that the pull-back of a \(g^1_{m}\) on \(C\) gives a \(g^{1}_{2m}\) on \(D\), so \[ \gon(D) \leq 2m,\] and hence \[ \LCliff(D) \leq 2m-2.\] The inequalities \[ 2m-2 = \RCliff(\widetilde C) \leq \RCliff(D) \leq \LCliff(D) \leq 2m-2\] yield the conclusion. \end{proof} \subsection{Resolution Clifford index of a split ribbon} We now take up the resolution Clifford index of the split ribbon. Unsurprisingly, we can describe the Koszul complex of the split ribbon entirely in terms of the underlying curve and the conormal bundle. We begin by defining a graded module that governs the Koszul complex of the split ribbon. Let \(C\) be a smooth curve of genus \(g\) and fix a line bundle \(L\) on \(C\). Fix a non-negative integer \(p\). Let \(S = \Sym(H^0(C, K_C))\) with the usual grading where the elements of \(H^0(C, K_C)\) have degree \(1\). The graded \(S\)-module \(M^p_L\), or simply \(M^p\) if \(L\) is clear from the context, is defined by \[ M^p = \bigoplus_{q \geq 0} K_{p,1}\left(C, K_C^{q}, K_C \otimes L^{-1}\right).\] The \(S\)-module structure is induced by the multiplication maps \[ K_{p,1}(C, K_C^{q}, K_C \otimes L^{-1}) \otimes H^0(C, K_C) \to K_{p,1}(C, K_C^{q+1}, K_C \otimes L^{-1}).\] Consider the Koszul complex of the graded \(S\)-module \(M^p\), namely \begin{equation}\label{eqn:mpkoszul} \cdots \to \bigwedge^{i+1} H^0(K_C) \otimes M^p_{q-1} \to \bigwedge^{i} H^0(K_C) \otimes M^p_q \to \bigwedge^{i-1}H^0(K_C) \otimes M^p_{q+1} \to \cdots. \end{equation} Let \(\Phi_{i,p,q}\) be the first map shown above. Writing out the descriptions of the graded pieces of \(M^p\), we have \begin{equation}\label{Phi} \Phi_{i,p,q} \colon \bigwedge^{i+1}H^0(K_C) \otimes K_{p,1}(C, K_C^{q-1}, K_C\otimes L^{-1}) \to \bigwedge^{i}H^0(K_C) \otimes K_{p,1}(C, K^q_C, K_C \otimes L^{-1}). \end{equation} Since \(M^p_{q}\)'s are themselves Koszul cohomology groups, the complex \eqref{eqn:mpkoszul} arises as a row in a double complex after taking vertical cohomology. We now write this double complex. For brevity, we write \((A)\) for \(H^0(C,A)\) and \(V^{(i)}\) for \(\bigwedge^i V\). The double complex is \begin{equation}\label{eqn:mpdouble} \begin{tikzcd} & \ar[d] & \ar[d] \\ \ar[r]& (K_C)^{(i+1)} \otimes (K_C \otimes L^{-1})^{(p+1)} \otimes (K_C^{q-1}) \ar[d] \ar[r] & (K_C)^{(i)} \otimes (K_C \otimes L^{-1})^{(p+1)} \otimes (K_C^{q}) \ar[d]\ar[r]& {}\\ \ar[r]& (K_C)^{(i+1)} \otimes (K_C \otimes L^{-1})^{(p)} \otimes (K_C^{q}\otimes L^{-1}) \ar[d] \ar[r,"d_{i,p,q}"] & (K_C)^{(i)} \otimes (K_C \otimes L^{-1})^{(p)} \otimes (K_C^{q+1}\otimes L^{-1}) \ar[d]\ar[r]&{}\\ \ar[r]& (K_C)^{(i+1)} \otimes (K_C \otimes L^{-1})^{(p-1)} \otimes (K_C^{q+1}\otimes L^{-2})\ar[r]\ar[d] & (K_C)^{(i)} \otimes (K_C \otimes L^{-1})^{(p-1)} \otimes (K_C^{q+2}\otimes L^{-2})\ar[r]\ar[d]&{}\\ {} & {} & {} \end{tikzcd}. \end{equation} The map \(\Phi_{i,p,q}\) in \eqref{Phi} is the map induced on vertical cohomology by the map \(d_{i,p,q}\). \begin{lemma}\label{surjectivity as koszul cohomology} In the setup above, assume that \(h^1(-L) = 0\) and \(p \leq 2g-4\). Then \(\Phi_{i,p,1}\) is surjective if and only if \(K_{i,1}(M^p) = 0\). \end{lemma} \begin{proof} Note that $K_{i,1}(M^p)$ is the middle cohomology of the complex \[(K_C)^{(i+1)} \otimes K_{p,1}(C, K_C\otimes L^{-1}) \xrightarrow{\Phi_{i,p,1}} (K_C)^{(i)} \otimes K_{p,1}(C, K_C, K_C\otimes L^{-1}) \to (K_C)^{(i-1)} \otimes K_{p,1}(C, K^2_C, K_C\otimes L^{-1}).\] It suffices to prove that the last term vanishes. Let \(r + 1 = h^0(K_C \otimes L^{-1})\). By duality \cite[2.6.c]{G84}, we have \[ K_{p,1}(C, K^2_C, K_C\otimes L^{-1}) = K_{r-1-p,1}(C, K_C^{-1}, K_C \otimes L^{-1})^{*}.\] Shifting by 1 gives \[ K_{r-1-p,1}(C, K_C^{-1}, K_C \otimes L^{-1}) = K_{r-1-p,0}(C, L^{-1}, K_C \otimes L^{-1}).\] Since \(h^1(-L) = 0\), we get \[ h^0(-L) = h^0(K_{C}-L) - (2g-2).\] Since \(p \leq 2g-4\), the right hand side is bounded above by \(r-1-p\). By \cite[Theorem~3.a.1]{G84}, we conclude that \(K_{r-1-p,0}(C, L^{-1}, K_C \otimes L^{-1}) = 0\). \end{proof} The following theorem relates the syzygies of the canonically embedded split ribbon with the Koszul cohomology of the module \(M^p\). \begin{theorem}\label{green for split} Let \(C\) be a smooth curve of genus \(g\) and gonality \(m\). Let \[p_a \geq \max(2g+2m-1, 6g-4)\] and let \(L\) be a line bundle on \(C\) of degree \(-p_a+2g-1\). Let \(\widetilde C\) be the split ribbon with conormal bundle \(L\). The following are equivalent. \begin{enumerate} \item \(\widetilde C\) satisfies Green's conjecture, that is, \[ \RCliff(\widetilde C) = \LCliff(\widetilde C).\] \item for all non-negative integers \(i,j\) such that \(i+j = 2m-3\), the map \[ \Phi_{i,j,1}: \bigwedge^{i+1}H^0(K_C) \otimes K_{j,1}(C, K_C-L) \xrightarrow{} \bigwedge^{i}H^0(K_C) \otimes K_{j,1}(C, K_C, K_C-L)\] is surjective. \item for all non-negative integers \(i,j\) such that \(i+j = 2m-3\), we have $K_{i,1}(M^{j}) = 0$. \end{enumerate} \end{theorem} \begin{proof} Let \(\phi \colon \widetilde C \to C\) be the projection. For any line bundle \(A\) on \(C\), we have a canonically split exact sequence of \(\mathcal{O}_{\widetilde C}\)-modules \[ 0 \to A \otimes L \to \phi^{*} A \to A \to 0.\] The splitting \(A \to \phi^{*}A\) is given by the pull-back map. In particular, for the \(q\)-th tensor power of \(K_{\widetilde C} = \phi^{*}(K_C \otimes L^{-1})\), we have the splitting \[ K_{\widetilde C}^q = (K_C^q \otimes L^{-q}) \oplus (K_{C}^q \otimes L^{-q+1}).\] As a result, we have \begin{equation}\label{eqn:canring} H^0(\widetilde C, K_{\widetilde C}^q) = H^0(C,K_C^q \otimes L^{-q}) \oplus H^0(K_{C}^q \otimes L^{-q+1}). \end{equation} Let \(\widetilde S\) and \(S\) be the graded rings \begin{align*} \widetilde S &= \bigoplus_{q \geq 0} H^0(\widetilde C, K_{\widetilde C}^q), \text{ and }\\ S &= \bigoplus_{q \geq 0} H^0(C, K_{C}^q \otimes L^{-q}), \end{align*} and let \(J\) be the graded \(S\)-module \[ J = \bigoplus_{q \geq 0} H^0(C, K_C^q \otimes L^{-q+1}). \] Let \(\epsilon\) be a formal variable with \(\epsilon^2 = 0\). Taking the direct sum over all \(q\) in \eqref{eqn:canring} gives an isomorphism of graded rings \[ \widetilde S = S \oplus \epsilon J.\] The Koszul complex that computes \(K_{p,q}(\widetilde C, K_{\widetilde C})\) is \[\mathcal{K} = \cdots \to \bigwedge^{p+1} \widetilde S_1 \otimes \widetilde S_{q-1} \to \bigwedge^{p} \widetilde S_1 \otimes \widetilde S_{q} \to \bigwedge^{p-1} \widetilde S_1 \otimes \widetilde S_{q+1} \to \cdots \] Here, we take the three terms above to be in homological degres \(q-1, q, q+1\), respectively. The compelx \(\mathcal{K}\) has a subcomplex \[ \mathcal{S} = \cdots \to \bigwedge^{p+1} \widetilde S_1 \otimes \epsilon J_{q-1} \to \bigwedge^{p} \widetilde S_1 \otimes \epsilon J_{q} \to \bigwedge^{p-1} \widetilde S_1 \otimes \epsilon J_{q+1} \to \cdots,\] and the quotient complex is \[ \mathcal{Q} = \cdots \to \bigwedge^{p+1} \widetilde S_1 \otimes S_{q-1} \to \bigwedge^{p} \widetilde S_1 \otimes S_{q} \to \bigwedge^{p-1} \widetilde S_1 \otimes S_{q+1} \to \cdots.\] Observe that \[ \bigwedge^p \widetilde S_1 = \bigoplus_i \bigwedge^{i} S_1 \otimes \epsilon \bigwedge^{p-i} J_1.\] The action of \(\epsilon \bigwedge^{p-i} J_1\) on \(\epsilon J_q\) and on \(S_q\) is zero. As a result, we see that \begin{align*} H^q(\mathcal{S}) &= \bigoplus_i \bigwedge^{p-i} J_1 \otimes K_{i,q}(C, L, K_C \otimes L^{-1}), \text{ and }\\ H^q(\mathcal{Q}) &= \bigoplus_i \bigwedge^{p-i} J_1 \otimes K_{i,q}(C, K_C \otimes L^{-1}). \end{align*} The short exact sequence of complexes \[ 0 \to \mathcal{S} \to \mathcal{K} \to \mathcal{Q} \to 0\] induces a long exact sequence in cohomology \begin{equation}\label{eq:les} \cdots \to H^{q-1}(\mathcal{Q}) \to H^q(\mathcal{S}) \to H^q(\mathcal{K}) \to \cdots. \end{equation} Consider the connecting map \[ \bigoplus_i \bigwedge^{p+1-i} J_1 \otimes K_{i,q-1}(C, K_C \otimes L^{-1}) \to \bigoplus_i \bigwedge^{p-i} J_1 \otimes K_{i,q}(C, L, K_C \otimes L^{-1}).\] It is easy to check that this map is diagonal, that is, the direct sum of maps \begin{equation}\label{eqn:conn} \bigwedge^{p+1-i} J_1 \otimes K_{i,q-1}(C, K_C \otimes L^{-1}) \to \bigwedge^{p-i} J_1 \otimes K_{i,q}(C, L, K_C \otimes L^{-1}). \end{equation} Note that \(J_{1} = H^0(C, K_C)\) and \[ K_{i,q}(C, L, K_C \otimes L^{-1}) = K_{i,q-1}(C, K_C, K_C \otimes L^{-1}),\] so the map \eqref{eqn:conn} becomes \begin{equation}\label{eq:conmap} \bigwedge^{p+1-i} H^0(K_C) \otimes K_{i,q-1}(C, K_C \otimes L^{-1}) \to \bigwedge^{p-i} H^0(K_C) \otimes K_{i,q-1}(C, K_C, K_C \otimes L^{-1}). \end{equation} For \(q = 2\), this is precisely the map \(\Phi_{p-i, i, 1}\) from \eqref{Phi}. We are now ready to prove the equivalence of (1) and (2). By \Cref{linear series Cliff of a split ribbon}, we know that \(\LCliff(\widetilde C) = 2m-2\). By \Cref{lin and res}, we know that \(\RCliff(\widetilde C) \leq \LCliff(\widetilde C)\). So, (1) is equivalent to the vanishing \[ K_{2m-3, 2}(\widetilde C, K_{\widetilde C}) = 0.\] Taking \(p = 2m-3\) and \(q = 2\), we have \[ K_{2m-3, 2}(\widetilde C, K_{\widetilde C}) = H^q(\mathcal{K}).\] Observe that we have \[ H^2(\mathcal{Q}) = \bigoplus_{i=0}^{2m-3} \bigwedge^{2m-3-i}H^0(K_C) \otimes K_{i,2}(C,K_C\otimes L^{-1}).\] By our hypothesis, we have \[ \deg(L^{-1}) = p_a - 2g+1 \geq 2m,\] and therefore \(\deg(K_C \otimes L^{-1}) \geq 2g+2m-2.\) By \cite[Theorem~4.a.1]{G84}, for \(0 \leq i \leq 2m-3\), we have \[K_{i,2}(C,K_C\otimes L^{-1}) = 0. \] So \(H^2(\mathcal{Q}) = 0\). By the long exact sequence \eqref{eq:les}, the vanishing of \(H^2(\mathcal{K})\) is equivalent to the surjectivity of \[ H^1(\mathcal{Q}) \to H^2(\mathcal{S}).\] By \eqref{eq:conmap}, this surjectivity is equivalent to (2). The equivalence of (2) and (3) is \Cref{surjectivity as koszul cohomology}. \end{proof} In light of \Cref{green for split}, it is natural to ask the following. \begin{question}\label{split ribbon conjecture} Are the equivalent statements in \Cref{green for split} are true ? \end{question} We observe that they are true for hyperelliptic curves. \begin{proposition}\label{split over hyperelliptic} The conditions in \Cref{green for split} hold for a hyperelliptic curve $C$. \end{proposition} \begin{proof} We begin with a more general statement. Let \(C\) be a smooth curve of genus \(g\) and let \(m\) be its gonality. We show that the map \[\bigwedge^{i+1} H^0(K_C) \otimes K_{j,1}(C, K_C\otimes L^{-1}) \to \bigwedge^i H^0(K_C) \otimes K_{j,1}(C, K_C, K_C\otimes L^{-1})\] is surjective for \(i = 0\) and \(j = 2m-3\). To see this, we use the results and notation of \cite{But94}. For line bundles \(A\) and \(B\), the kernel of the map \[\bigwedge^{j}H^0(A) \otimes H^0(B) \to \bigwedge^{j-1} H^0(A) \otimes H^0(A \otimes B)\] is \(H^0(\bigwedge^j M_A \otimes A \otimes B)\) (see, for example, \cite[Lemma $1.4$]{EL93}). So it is enough to show that for $j = 2m-3$, the map \[H^0(K_C) \otimes H^0\left(\bigwedge^j M_{(K_C \otimes L^{-1})} \otimes K_C \otimes L^{-1}\right) \to H^0\left(\bigwedge^j M_{(K_C \otimes L^{-1})} \otimes K^2_C \otimes L^{-1}\right)\] is surjective. Since $K_C\otimes L^{-1}$ is semistable of degree (= slope) at least \(2g\), the bundle \(M_{K_C \otimes L^{-1}}\) is semistable (\cite[Theorem $1.2$]{But94}). The surjection follows from \cite[Proposition $2.2$]{But94}. \end{proof} We end with an example of a split ribbon that does \emph{not} satisfy Green's conjecture. \begin{example}\label{does not satisfy Green} Let \(C\) be a general curve of genus 3, and take \(L = K_C^{-1}\). Note that \(C\) has gonality \(3\). Let \(\widetilde C\) be the split ribbon with conormal bundle \(L\). Then \(p_a(\widetilde C) = 9\). By \Cref{linear series Cliff of a split ribbon}, we have \(\LCliff(\widetilde C) = 4\). A \texttt{Macaulay2} computation---done over the base field \(\mathbf{Z}/101 \mathbf{Z}\) with a randomly chosen plane quartic \(C\)---shows that the minimal free resolution of the canonical embedding of \(\widetilde C\) has the betti table \[ \begin{matrix} & 0 & 1 & 2 & 3 & 4 & 5 & 6 & 7\\ \text{total:} & 1 & 21 & 84 & 154 & 154 & 84 & 21 & 1\\ 0: & 1 & . & . & . & . & . & . & .\\ 1: & . & 21 & 64 & 90 & 64 & 20 & . & .\\ 2: & . & . & 20 & 64 & 90 & 64 & 21 & .\\ 3: & . & . & . & . & . & . & . & 1 \end{matrix},\] which means \(\RCliff(\widetilde C) = 2\). \end{example} \color{black} \bibliographystyle{plain} \begin{thebibliography}{100} \bibitem[BE95]{BE95} Bayer, Dave; Eisenbud, David \emph{Ribbons and their canonical embeddings} Transactions of the American Mathematical Society Volume 347, Number 3, March 1995 \bibitem[But94]{But94} Butler, David C. \emph{Normal generation of vector bundles over a curve}. J. Differential Geom. 39 (1994), no. 1, 1--34. \bibitem[CS19]{CS19} Coelho, Juliana; Sercio, Frederico \emph{On the gonality of stable curves.} Math. Nachr.292(2019), no.11, 2338–2351. \bibitem[D18]{D18} Deopurkar, Anand. \emph{The canonical syzygy conjecture for ribbons.} Math. Z. 288 (2018), no. 3-4, 1157--1164 \bibitem[Dr\'e15]{Dr\'e15} Drézet, Jean-Marc \emph{Reducible deformations and smoothing of primitive multiple curves.} Manuscripta Math.148(2015), no.3-4, 447–469. \bibitem[Dr\'e21]{Dr\'e21} Drézet, Jean-Marc \emph{Primitive multiple schemes.} Eur. J. Math.7(2021), no.3, 985–1045. \bibitem[EG95]{EG95} Eisenbud, David; Green, Mark \emph{Clifford Indices of Ribbons} Transactions of the American Mathematical Society Vol. 347, No. 3 (Mar., 1995), pp. 757-765 (9 pages) \bibitem[ELMS89]{ELMS89} Eisenbud, David; Lange, Herbert; Martens, Gerriet; Schreyer, Frank-Olaf \emph{The Clifford dimension of a projective curve.} Compositio Math.72(1989), no.2, 173–204. \bibitem[EL93]{EL93} Ein, Lawrence; Lazarsfeld, Robert. \emph{Syzygies and Koszul cohomology of smooth projective varieties of arbitrary dimension}. Invent. Math. 111 (1993), no. 1, 51--67. \bibitem[EL12]{EL12} Ein, Lawrence; Lazarsfeld, Robert \emph{Asymptotic syzygies of algebraic varieties.} Invent. Math.190(2012), no.3, 603–646. \bibitem[EL15]{EL15} Ein, Lawrence; Lazarsfeld, Robert \emph{The gonality conjecture on syzygies of algebraic curves of large degree.} Publ. Math. Inst. Hautes Études Sci.122(2015), 301–313. \bibitem[F]{F} Fischer, Tom. \emph{The higher secant varieties of an elliptic normal curve} \bibitem[F93]{F93} Fong, Lung-Ying. \emph{Rational ribbons and deformation of hyperelliptic curves.} J. Algebraic Geom. 2 (1993), no. 2, 295--307. \bibitem[G84]{G84} Green, Mark L. \emph{Koszul cohomology and the geometry of projective varieties.} J. Differential Geom. 19 (1984), no. 1, 125--171 \bibitem[G84b]{G84b} Green, M.L \emph{Koszul cohomology and the geometry of projective varieties II} J. Differential Geom. 20 (1984), no. 1, 279–289. \bibitem[GGP08]{GGP08} Gallego, Francisco Javier; González, Miguel; Purnaprajna, Bangere P. \emph{Deformation of finite morphisms and smoothing of ropes.} Compos. Math.144(2008), no.3, 673–688. \bibitem[GL86]{GL86} Green, Mark; Lazarsfeld, Robert. \emph{On the projective normality of complete linear series on an algebraic curve}. Invent. Math. 83 (1986), no. 1, 73–90. \bibitem[GL87]{GL87} Green, Mark; Lazarsfeld, Robert. \emph{Special divisors on curves on a K3 surface.} Inventiones Mathematicae, 89(2):357–370, 1987 \bibitem[Gon06]{Gon06} Gonz\'alez, Miguel. \emph{Smoothing of ribbons over curves} J. reine angew. Math. 591 (2006), 201—235 \bibitem[Gre84]{Gre84} Green, Mark L. \emph{Koszul cohomology and the geometry of projective varieties}. J. Differential Geom. 19 (1984), no. 1, 125--171. \bibitem[HR98]{HR98} Hirschowitz, A.; Ramanan, S. \emph{New evidence for Green's conjecture on syzygies of canonical curves.} Ann. Sci. École Norm. Sup. (4)31(1998), no.2, 145–152. \bibitem[Lan84]{Lan84} Lange, Herbert \emph{Higher secant varieties of curves and the theorem of Nagata on ruled surfaces.} Manuscripta Math.47(1984), no.1-3, 263–269. \bibitem[MG91]{MG91} Coppens, Marc; Martens, Gerriet \emph{Secant spaces and Clifford's theorem.} Compositio Math.78(1991), no.2, 193–212. \bibitem[Mum70]{mum} Mumford, David. \emph{Varieties defined by quadratic equations}. 1970 Questions on Algebraic Varieties (C.I.M.E., III Ciclo, Varenna, 1969) pp. 29--100 Edizioni Cremonese, Rome. \bibitem[PR88]{PR88} Paranjape, Kapil; Ramanan, S. \emph{On the canonical ring of a curve.} Algebraic geometry and commutative algebra, Vol. II, 503–516. Kinokuniya Company Ltd., Tokyo, 1988 \bibitem[P22]{P22} Park, Jinhyung \emph{Syzygies of tangent developable surfaces and K3 carpets via secant varieties} https://arxiv.org/abs/2212.07584 \bibitem[RS19]{RS19} Raicu, Claudiu; Sam, Steven V. Bi-graded Koszul modules, K3 carpets, and Green's conjecture.Compos. Math.158(2022), no.1, 33–56. \bibitem[vBH04]{vBH04} H.-Chr. Graf v. Bothmer; Hulek, Klaus \emph{Geometric syzygies of elliptic normal curves and their secant varieties} Manuscripta Math. 113 (2004), no. 1, 35–68. \bibitem[V02]{V02} Voisin, C. \emph{Green’s generic syzygy conjecture for curves of even genus lying on a K3 surface.} J. Eur. Math. Soc., 4, 363–404 (2002). \bibitem[V05]{V05} Voisin, C. \emph{Green’s canonical syzygy conjecture for generic curves of odd genus.} Compositio Math., 141 (5), 1163–1190 (2005). \end{thebibliography} \end{document}
2412.05494v1
http://arxiv.org/abs/2412.05494v1
On a bipartite graph defined on groups
\documentclass[12pt]{amsart} \usepackage{tikz} \usepackage{array} \usepackage{caption} \usetikzlibrary{automata} \usetikzlibrary{positioning} \usepackage{tikz-network} \newtheorem{defn}{Definition}[section] \newtheorem{eg}{Example}[section] \newtheorem{conj}{Conjecture}[section] \newtheorem{obj}{Observation}[section] \newtheorem{remark}{Remark}[section] \newtheorem{theorem}{Theorem}[section] \newtheorem{prop}[theorem]{Proposition} \newtheorem{lemma}[theorem]{Lemma} \newtheorem{cor}[theorem]{Corollary} \newtheorem{question}[theorem]{Question} \renewcommand{\theenumi}{\alph{enumi}} \renewcommand{\labelenumi}{\rm (\theenumi)} \DeclareMathOperator{\sol}{sol} \DeclareMathOperator{\nil}{nil} \DeclareMathOperator{\cyc}{cyc} \DeclareMathOperator{\girth}{girth} \DeclareMathOperator{\diam}{diam} \DeclareMathOperator{\ABC}{ABC} \DeclareMathOperator{\GA}{GA} \DeclareMathOperator{\SCI}{SCI} \DeclareMathOperator{\Nbd}{Nbd} \DeclareMathOperator{\gen}{gen} \DeclareMathOperator{\ir}{ir} \renewcommand{\labelenumi}{\rm(\alph{enumi})} \allowdisplaybreaks \setlength{\topmargin}{-0.4in} \setlength{\topskip}{0.2in} \setlength{\textheight}{9in} \setlength{\textwidth}{6.25in} \setlength{\oddsidemargin}{0.1in} \setlength{\evensidemargin}{0.1in} \begin{document} \title[On a bipartite graph defined on groups]{On a bipartite graph defined on groups} \author[S. Das, A. Erfanian and R. K. Nath]{Shrabani Das, Ahmad Erfanian and Rajat Kanti Nath*} \address{S. Das, Department of Mathematical Sciences, Tezpur University, Napaam-784028, Sonitpur, Assam, India.} \email{[email protected]} \address{A. Erfanian, Department of Pure Mathematics, Ferdowsi University of Mashhad, P.O. Box 1159-91775, Mashhad, Iran.} \email{[email protected]} \address{R. K. Nath, Department of Mathematical Sciences, Tezpur University, Napaam-784028, Sonitpur, Assam, India.} \email{ [email protected]} \thanks{*Corresponding author} \begin{abstract} Let $G$ be a group and $L(G)$ be the set of all subgroups of $G$. We introduce a bipartite graph $\mathcal{B}(G)$ on $G$ whose vertex set is the union of two sets $G \times G$ and $L(G)$, and two vertices $(a, b) \in G \times G$ and $H \in L(G)$ are adjacent if $H$ is generated by $a$ and $b$. We establish connections between $\mathcal{B}(G)$ and the generating graph of $G$. We also discuss about various graph parameters such as independence number, domination number, girth, diameter, matching number, clique number, irredundance number, domatic number and minimum size of a vertex cover of $\mathcal{B}(G)$. We obtain relations between $\mathcal{B}(G)$ and certain probabilities associated to finite groups. We also obtain expressions for various topological indices of $\mathcal{B}(G)$. Finally, we realize the structures of $\mathcal{B}(G)$ for the dihedral groups of order $2p$ and $2p^2$ and dicyclic groups of order $4p$ and $4p^2$ (where $p$ is any prime) including certain other small order groups. \end{abstract} \thanks{ } \subjclass[2020]{20D60, 05C25} \keywords{Graphs on groups; Bipartite graph; Dihedral group; Dicyclic group.} \maketitle \section{Introduction} Finite groups are being characterized through various graphs defined on it for a long time now. There are quite a few graphs whose vertex set contains elements from a group $G$ and edges defined by properties of $G$. Some popular graphs defined on groups include the commuting graph (pioneered by Brauer and Fowler in \cite{brauer1955groups}), the non-commuting graph (pioneered by Erd{\"o}s and Neumann \cite{neumann1976problem}), the generating graph (pioneered by Liebeck and Shalev \cite{LS96}), the power graph (pioneered by Kelarev and Quinn \cite{KQ2000}), cyclic/non-cyclic graph (pioneered by Abdollahi and Hassanabadi \cite{AH2007}), nilpotent/non-nilpotent graph (pioneered by Abdollahi and Zarrin \cite{az2010}), solvable/non-solvable graph (pioneered by Hai-Reuven \cite{Hai-Reuven-2013}), and this list has been growing. A survey on the above mentioned graphs defined on groups can be found in \cite{cameron2021graphs}. Let $G$ be a group and $\mathcal{X}$ be a group property, viz. cyclic, abelian, nilpotent, solvable etc. A graph defined on $G$ is called a $\mathcal{X}$ graph of $G$ if the vertex set is $G$ and two distinct vertices $x$ and $y$ are adjacent if $\langle x, y\rangle$ is a $\mathcal{X}$-group. Thus, commuting graph of $G$ is nothing but `abelian graph' of $G$. Recently, the explicit formulas for the number of spanning trees of commuting graphs associated with some specific groups are obtained in \cite{CMMS-2022}. The complement of $\mathcal{X}$ graph is called non-$\mathcal{X}$ graph of $G$. Note that the set $\mathcal{X}(G) := \{x \in G : \langle x, y \rangle \text{ is a $\mathcal{X}$-group for all } y \in G\}$ is the set of all dominant vertices of $\mathcal{X}$ graph of $G$. We have $\mathcal{X}(G) =$ Cyc$(G)$ (the cyclicizer of $G$), $Z(G)$ (the center of $G$), Nil$(G)$ (the hypercenter of $G$) and Sol$(G)$ (the solvable radical of $G$) when $\mathcal{X}$ represents ``cyclic", ``abelian", ``nilpotent" and ``solvable" respectively. While studying the connectedness, genus and many other graph parameters of $\mathcal{X}$ graph and non-$\mathcal{X}$ graph often $\mathcal{X}(G)$ is removed from the vertex set. Recently, two counter examples (see \cite{Das-NN-2024, SN-2024}) to a conjecture of Gutman (see \cite[Conjecture 3.1]{Gutman-2011} and \cite{Gutman-78}) regarding the existence of hyperenergetic graphs are provided through non-commuting graphs of finite groups. The power graph of $G$ has vertex set $G$ and two vertices $x$ and $y$ are adjacent if one of $x$ and $y$ is the power of the other. Recent works on power graph can be found in \cite{KSCC-2021, MPS23, PPS21}. The generating graph of a finite group $G$, denoted by $\Gamma_{\gen}(G)$, is a simple undirected graph with vertex set $V(\Gamma_{\gen}(G))= G$ and two distinct vertices $x$ and $y$ are adjacent if $\langle x, y \rangle = G$. There are other classes of graphs defined on groups whose vertices are the orders of the elements (see \cite{MN-2024}) or the orders of the conjugacy classes (see \cite{Lewis-2008}). A survey on graphs whose vertex set consists of conjugacy classes of a group can be found in \cite{CJSN-2024}. Another class of graphs have been defined on groups by considering the vertex set as the set of subgroups of the group. For instance, intersection graph (introduced by Cs$\acute{\rm a}$k$\acute{\rm a}$ny and Poll$\acute{\rm a}$k \cite{CP69}), inclusion graph (introduced by Devi and Rajkumar \cite{DR16}) and permutability graph (introduced by Rajkumar and Devi \cite{RD14}) of $G$ are such graphs. In these graphs, if $H$ and $K$ are two vertices (subgroups of $G$) then $H$ and $K$ are adjacent if (i) $H \cap K \neq \{1\}$ (in case of intersection graph) (ii) $H \subset K$ or $K \subset H$ (in case of inclusion graph) (iii) $HK = KH$ (in case of permutability graph). Various results on these graphs can be found in \cite{ahm2015,SK-2017,OWW20,RD16,RDG16}. Note that none of the above mentioned graphs are bipartite in nature, while the use of bipartite graphs in solving real-world problems has been known. Bipartite graphs are extensively used in modern coding theory while receiving code words from a channel. They can be used in medical field in the detection of lung cancer, throat cancer etc. In economics, one can see how markets work when buyers and sellers do business. Bipartite graphs are also used for optimizing transportation systems, route planning, and logistics management. Reader may have a look to \cite{Arun-Koma-15} and the references therein for these applications of bipartite graphs. In this paper, we introduce a bipartite graph $\mathcal{B}(G)$ on a group $G$ whose vertex set $V(\mathcal{B}(G))$ is the union of two sets $G \times G$ and $L(G) := \{H : H \text{ is a subgroup of } G\}$, and two vertices $(a, b) \in G \times G$ and $H \in L(G)$ are adjacent if $H = \langle a, b \rangle$, the subgroup generated by $a$ and $b$. We write $V(\mathcal{B}(G)) = G \times G \sqcup L(G)$, where $\times$ denotes the Cartesian product of sets and $\sqcup$ denotes the union of disjoint sets. We shall also use $\sqcup$ to denote the union of disjoint graphs. The neighborhood of any vertex $x$ of $\mathcal{B}(G)$ is denoted by $\Nbd_{\mathcal{B}(G)}(x):= \{y \in V(\mathcal{B}(G)) : y \text{ is adjacent to } x\}$. Further, if $S$ is a subset of $V(\mathcal{B}(G))$ then we write $\mathcal{B}(G)[S]$ to denote the induced subgraph of $\mathcal{B}(G)$ induced by $S$. For any group $G$, it is obvious that every element of $G \times G$ is adjacent to some element of $L(G)$. Also, if $G$ is a $2$-generated group then every element of $L(G)$ is adjacent to some elements of $G \times G$. We also have the following observations. \begin{obj}\label{vrtex_deg_of_X_in_B(G)} Let $G$ be any group. Then $\mathcal{B}(G)$ has the following properties. \begin{enumerate} \item For all $(a, b) \in G \times G$, the degree of $(a, b)$ in $\mathcal{B}(G)$ is one. That is, $\deg_{\mathcal{B}(G)}((a, b))$ $ = 1$. \item $\mathcal{B}(G)$ has no cycle and it is a forest having $|L(G)|$ components. In particular, $\mathcal{B}(G)$ is a union of \, $|L(G)|$ \, star graphs. If $G$ is a cyclic group of prime order then $\mathcal{B}(G) = K_2 \sqcup K_{1, |G|^2-1}$, where $K_n$ and $K_{1, n}$ denote complete graphs on $n$ vertices and star graphs on $n+1$ vertices respectively. Let $p$ be any prime and $G = \langle a \rangle$. If $|G|=2p$ then $V(\mathcal{B}(G)) = G \times G \sqcup \{\{1\}, \langle a^p \rangle, \langle a^2 \rangle, \langle a \rangle\}$. Since \quad $|\langle a^p \rangle| = 2$ and $|\langle a^2 \rangle| =p$ we have $\mathcal{B}(G)[\{\langle a^p \rangle\}$ $ \sqcup \Nbd_{\mathcal{B}(G)}(\langle a^p \rangle)] = K_{1, 3}$ and $\mathcal{B}(G)[\{\langle a^2 \rangle\} \sqcup \Nbd_{\mathcal{B}(G)}(\langle a^2 \rangle)] = K_{1, p^2 - 1}$. Also, $\mathcal{B}(G)[\{\langle a \rangle\} \sqcup \Nbd_{\mathcal{B}(G)}(\langle a \rangle)] = K_{1, 3p^2 - 3}$ noting that \quad $|\Nbd_{\mathcal{B}(G)}(\langle a \rangle)| = 4p^2 - $ $(1 + 3 + p^2 - 1) = 3p^2 - 3$. Thus, $\mathcal{B}(G) = K_2 \sqcup K_{1, 3}\sqcup K_{1, p^2 - 1} \sqcup K_{1, 3p^2 - 3}$. If $|G|=p^2$ then $V(\mathcal{B}(G)) = G \times G \sqcup \{\{1\}, \langle a^p \rangle, \langle a \rangle\}$. Since $|\langle a^p \rangle| =p$ we have $\mathcal{B}(G)[\{\langle a^p \rangle\} \sqcup \Nbd_{\mathcal{B}(G)}(\langle a^p \rangle)] = K_{1, p^2 - 1}$. Also, $\mathcal{B}(G)[\{\langle a \rangle\} \sqcup \Nbd_{\mathcal{B}(G)}(\langle a \rangle)] = K_{1, p^4 - p^2}$ noting that $|\Nbd_{\mathcal{B}(G)}(\langle a \rangle)| = p^4 - (1 + p^2 - 1)$. Thus, $\mathcal{B}(G) = K_2 $ $\sqcup K_{1, p^2 - 1} \sqcup K_{1, p^4 - p^2}$. If $|G|= 2p^2$ (for odd prime $p$) then $V(\mathcal{B}(G)) = G \times G$ $ \sqcup \{\{1\}, \langle a^{p^2} \rangle, \langle a^{2p} \rangle, \langle a^p \rangle$, $\langle a^2 \rangle, \langle a \rangle\}$. Since $|\langle a^{p^2} \rangle| = 2$, $|\langle a^{2p} \rangle| = p$, $|\langle a^p \rangle|= 2p$ and $|\langle a^2 \rangle| =p^2$ we have $\mathcal{B}(G)[\{\langle a^{p^2} \rangle\} \sqcup \Nbd_{\mathcal{B}(G)}(\langle a^{p^2} \rangle)] = K_{1, 3}$, $\mathcal{B}(G)[\{\langle a^{2p} \rangle\} \sqcup \Nbd_{\mathcal{B}(G)}(\langle a^{2p} \rangle)] = K_{1, p^2-1}$, $\mathcal{B}(G)[\{\langle a^p \rangle\} \sqcup \Nbd_{\mathcal{B}(G)}(\langle a^p \rangle)] = K_{1, 3p^2 - 3}$, $\mathcal{B}(G)[\{\langle a^2 \rangle\} \sqcup \Nbd_{\mathcal{B}(G)}(\langle a^2 \rangle)] = K_{1, p^4 - p^2}$. Also, $\mathcal{B}(G)[\{\langle a \rangle\} \sqcup \Nbd_{\mathcal{B}(G)}(\langle a \rangle)] = K_{1, 3p^4 - 3p^2}$ noting that $|\Nbd_{\mathcal{B}(G)}(\langle a \rangle)| = 4p^4 - (1 + 3 + p^2 - 1 + 3p^2 - 3 + p^4 - p^2)$ $ = 3p^4 - 3p^2$. Thus, $\mathcal{B}(G) = K_2 \sqcup K_{1, 3} \sqcup K_{1, p^2 - 1} \sqcup K_{1, 3p^2 - 3} \sqcup K_{1, p^4 - p^2} \sqcup K_{1, 3p^4 - 3p^2}$. If $G$ is a non-cyclic group of order $p^2$ then $G$ has one subgroup of order one, $p + 1$ subgroups of order $p$ and one subgroup of order $p^2$. Let $I = \{1\}$, $H_1, H_2, \dots, H_{p+1}$ and $K = G$ be the subgroups of $G$, where $H_i \cong \mathbb{Z}_p$ for $1 \leq i \leq p+1$. Then $\mathcal{B}(G)[\{I\}\sqcup \Nbd_{\mathcal{B}(G)}(I)] = K_2$, \quad $\mathcal{B}(G)[\{H_i\}\sqcup \Nbd_{\mathcal{B}(G)}(H_i)] = \mathcal{B}(\mathbb{Z}_p)[\{\mathbb{Z}_p\}\sqcup \Nbd_{\mathcal{B}(\mathbb{Z}_p)}(\mathbb{Z}_p)] = K_{1, p^2 - 1}$ for $1 \leq i \leq p+1$. Further, $\mathcal{B}(G)[\{G\}$ $ \sqcup \Nbd_{\mathcal{B}(G)}(G)]$ $ = K_{1, p(p-1)(p^2 - 1)}$ noting that $|\Nbd_{\mathcal{B}(G)}(G)| = p^4 - (p+1)(p^2 -1) - 1 = p(p-1)(p^2 - 1)$. Thus, $\mathcal{B}(G) = K_2 \sqcup (p+1)K_{1, p^2 - 1} \sqcup K_{1, p(p-1)(p^2 - 1)}$, where $mK_{1, n}$ denotes the disjoint union of $m$ copies of the star $K_{1, n}$. \item $\mathcal{B}(G)$ is connected if and only if $G = \{1\}$. In this case, $\mathcal{B}(G)= K_2$. \end{enumerate} \end{obj} In Section 2, we obtain some properties of $\mathcal{B}(G)$. In particular, we establish connections between $\mathcal{B}(G)$ and $\Gamma_{\gen}(G)$. We also discuss about various graph parameters such as independence number, domination number, girth, diameter, matching number, clique number, irredundance number, domatic number and minimum size of a vertex cover of $\mathcal{B}(G)$. One big motivation in defining the graph $\mathcal{B}(G)$ is to obtain various probabilities associated to finite groups through this graph. In Section 3, we obtain relations between $\mathcal{B}(G)$ and certain probabilities associated to finite groups. Using those relations, we calculate the exact probabilities for some well-known small order finite groups. We shall also obtain expressions for various topological indices such as first and second Zagreb indices, Randic Connectivity index, Atom-Bond Connectivity index, Geometric-Arithmetic index, Harmonic index and Sum-Connectivity index of $\mathcal{B}(G)$. In Section 4, we first realize the structures of $\mathcal{B}(G)$ when $G = S_3, D_8, Q_8, D_{10}, D_{12}, A_4$ and $S_4$. After that we realize the structures of $\mathcal{B}(G)$ when $G = D_{2p}$ and $D_{2p^2}$ the dihedral groups of order $2p$ and $2p^2$ for any prime $p$, where $D_{2n}$ is the dihedral group presented by $\langle a, b: a^n=b^2=1, bab=a^{-1} \rangle$. We conclude the paper realizing the structures of $\mathcal{B}(G)$ when $G = Q_{4p}$ and $Q_{4p^2}$ the dicyclic groups of order $4p$ and $4p^2$ for any prime $p$, where $Q_{4n}$ is the dicyclic group presented by $\langle a, b : a^{2n} = 1, b^2 = a^n, bab^{-1} = a^{-1} \rangle$. \section{Some properties of \, $\mathcal{B}(G)$} We begin with the following properties of $\mathcal{B}(G)$. \begin{theorem} If $G$ is a non-trivial finite group, then $\deg_{\mathcal{B}(G)}(x) \leq |G|^2-1$ for all $x \in V(\mathcal{B}(G))$. Further $\deg_{\mathcal{B}(G)}(G) = |G|^2-1$ if and only if $G$ is a cyclic group of prime order. \end{theorem} \begin{proof} We have $V(\mathcal{B}(G))=G \times G \, \sqcup \, L(G)$ and $\deg_{\mathcal{B}(G)}(a, b) =1$ for all $(a, b) \in G \times G$. Also, $\{ 1 \} \in L(G)$ and $\{1\}$ is adjacent to $(1, 1)$ only. Therefore, for all $x \in L(G)\setminus \{1\}$, we have $\deg_{\mathcal{B}(G)}(x) \leq |G|^2-1$. If $G$ is a cyclic group of prime order, then all the non-identity elements of $G$ are its generators. Also, $L(G)=\{\{1\}, G\}$. As such, $\deg_{\mathcal{B}(G)}(\{1\})=1$ since $\{1\}$ is adjacent to $(1, 1)$ only and $\deg_{\mathcal{B}(G)}(G)=|G|^2-1$. Conversely, suppose that $\deg_{\mathcal{B}(G)}(G)=|G|^2-1$. Then for every element $(1, 1) \ne (a, b) \in G \times G$ we have $\langle a, b\rangle = G$. In particular, $\langle a\rangle = G$ for all $1\ne a \in G$. This shows that $G$ is cyclic group of prime order. \end{proof} In the following theorem we obtain degree of any vertex $H \in L(G)$ in the graph $\mathcal{B}(G)$ using the size of the generating graph $\Gamma_{\gen}(H)$. \begin{theorem}\label{relatn B(G) and generating graph} Let $G$ be a finite group and $H \in L(G)$. Then \[ \deg_{\mathcal{B}(G)}(H)=\begin{cases} 1, & \text{ if } H=\{1\} \\ 2|e(\Gamma_{\gen}(H))|+\phi(|H|), & \text{ if } H \text{ is cyclic } \\ 2|e(\Gamma_{\gen}(H))|, & \text{ otherwise. } \end{cases} \] Here, $\Gamma_{\gen}(H)$ is the generating graph of $H$ and $\phi(|H|)$ is the number of generators of $\mathbb{Z}_{|H|}$. \end{theorem} \begin{proof} Clearly, $(1,1)$ is the only vertex adjacent to $\{1\}$ in $\mathcal{B}(G)$ and so $\deg_{\mathcal{B}(G)}(H)=1$ if $H=\{1\}$. If $H \ne \{1\}$ is a cyclic group then $\phi(|H|)$ gives the number of generators of $H$. We have \begin{align*} \deg_{\mathcal{B}(G)}(H)&=\left|\{(a,b) \in G \times G: \langle a,b \rangle =H\}\right| \\ &=\phi(|H|)+\left|\{(a,b) \in G \times G: \langle a,b \rangle =H, a \neq b\}\right|. \end{align*} Now, for $a \neq b$, if $\langle a,b \rangle=\langle b,a \rangle=H$ then $(a,b)$ and $(b,a)$ are adjacent to $H$ in $\mathcal{B}(G)$ and $a$ is adjacent to $b$ in $\Gamma_{\gen}(H)$. It follows that, the pairs $(a,b), (b,a), a \neq b$ that generates $H$, contribute one edge in $\Gamma_{\gen}(H)$ and two edges in $\mathcal{B}(G)$. Therefore, $|e(\Gamma_{\gen}(H))|=\frac{1}{2}\left|\{(a,b) \in G \times G: \langle a,b \rangle =H,\right. $ $\left. a \neq b\}\right|$. Thus, $\deg_{\mathcal{B}(G)}(H)=2|e(\Gamma_{\gen}(H))|+\phi(|H|)$. If $H$ is non-cyclic then \quad $\deg_{\mathcal{B}(G)}(H)=\left|\{(a,b) \in G \times G: \langle a,b \rangle =H, a \neq b\}\right|$, since $\{(a, a) \in G \times G: \langle a, a \rangle =H\}$ is an empty set. Therefore, by similar arguments as above, it follows that $\deg_{\mathcal{B}(G)}(H)=2|e(\Gamma_{\gen}(H))|$. \end{proof} The following theorem is useful in obtaining independence and domination number of $\mathcal{B}(G)$. \begin{theorem}\label{size of A bigger than that of B} For any $2$-generated finite group $G$, if $A=G \times G$ and $B=L(G)$, then $|A| \geq |B|$ with equality when $G$ is a group of order $1$. \end{theorem} \begin{proof} Define a map $f:G \times G \rightarrow L(G)$ by $f((a, b))= \langle a, b \rangle$ for all $a, b \in G$. We have $f((1, 1))=\{1\}$ and $f((a, 1))=f((1, a))=f((a, a))=\langle a \rangle$ for all $a \in G$. So $f$ is a many-one function. Also, if $G$ is a 2-generated group, then any $H \in L(G)$ is adjacent to some elements of $G \times G$. As such, $f$ is an onto function. Therefore, $|G \times G| > |L(G)|$ when $|G| > 1$. For $|G|=1$, $G \times G$ and $L(G)$ have same cardinality equal to one. \end{proof} Let $\Gamma$ be any graph. An independent vertex set of $\Gamma$ is a subset of the vertex set of $\Gamma$ such that no two vertices in the subset represent an edge of $\Gamma$. The cardinality of the largest independent vertex set of $\Gamma$ is the independence number of $\Gamma$. A subset $S$ of $V(\Gamma)$ is said to be a dominating set of vertices in $\Gamma$ if $\left(\cup_{s \in S}\Nbd(s)\right) \cup S = V(\Gamma)$. A dominating set of smallest size is called a minimum dominating set and its cardinality is called the domination number of $\Gamma$. \begin{theorem}\label{independence-domination no. of B(G)} If $A=G \times G$ and $B=L(G)$, then independence and domination number of $\mathcal{B}(G)$ are the sizes of $A$ and $B$ respectively where $G$ is any $2$-generated finite group. \end{theorem} \begin{proof} By Theorem \ref{size of A bigger than that of B} we have $|A| \geq |B|$. Since $\mathcal{B}(G)$ is a bipartite graph, by definition, the independence number of $\mathcal{B}(G)$ is $|A|$. Also, every element of $A$ is adjacent to some elements of $B$ and if $G$ is a $2$-generated finite group, then any element of $B$ is adjacent to some elements of $A$. Therefore, by definition, the domination number of $\mathcal{B}(G)$ is $|B|$. \end{proof} \begin{remark}\label{first remark for r-generated} Let $G$ be an $r$-generated finite group where $r\geq3$. \begin{enumerate} \item It can be easily seen that a vertex $H \in B=L(G)$ is isolated in $\mathcal{B}(G)$ if $H$ is generated by 3 or more elements. For example, if $G= \mathbb{Z}_{4} \times \mathbb{Z}_{4} \times \mathbb{Z}_{4}$ then $\langle \Bar{2} \rangle \times \langle \Bar{2} \rangle \times \langle \Bar{2} \rangle, \langle \Bar{2} \rangle \times\langle \Bar{2} \rangle \times \langle \mathbb{Z}_{4} \rangle, \langle \Bar{2} \rangle \times \langle \mathbb{Z}_{4} \rangle \times \langle \Bar{2} \rangle, \langle \mathbb{Z}_{4} \rangle \times \langle \Bar{2} \rangle \times \langle \Bar{2} \rangle, \langle \mathbb{Z}_{4} \rangle \times \langle \mathbb{Z}_{4} \rangle \times \langle \Bar{2} \rangle, \langle \mathbb{Z}_{4} \rangle \times \langle \Bar{2} \rangle \times \langle \mathbb{Z}_{4} \rangle, \langle \Bar{2} \rangle \times \langle \mathbb{Z}_{4} \rangle \times \langle \mathbb{Z}_{4} \rangle, \mathbb{Z}_{4} \times \mathbb{Z}_{4} \times \mathbb{Z}_{4}$ etc. are some isolated vertices in $\mathcal{B}(G)$. We also have that $|A| =|G \times G| =4096 \geq 129=|L(G)|=|B|$. Thus the conclusion of Theorem \ref{size of A bigger than that of B} is true for $G=\mathbb{Z}_{4} \times \mathbb{Z}_{4} \times \mathbb{Z}_{4}$. In general, the conclusion of Theorem \ref{size of A bigger than that of B} may also be true for any finite $r$-generated group where $r\geq 3$. However, the proof we have given will not work in this case as there are isolated vertices. \item Let $L_2(G)=\{H \in L(G): H \text{ is generated by 1 or 2 elements} \}$. Then $|A|\geq |L_2(G)|$ and $A \sqcup (L(G) \setminus L_2(G))$ is the largest independent set of $\mathcal{B}(G)$. Hence, independence number of $\mathcal{B}(G)$ is $|A|+|L(G)|-|L_2(G)|$. Further, if $|A|\geq |B|$ then domination number of $\mathcal{B}(G)$ is $|B|$. \end{enumerate} \end{remark} Let $\Gamma$ be any graph. The girth of $\Gamma$, denoted by $\girth(\Gamma)$, is the size of the smallest cycle in it. The diameter of $\Gamma$, denoted by $\diam(\Gamma)$, is defined as the maximum distance of any two vertices of it. A matching in $\Gamma$ is a subset of the edge set of $\Gamma$ such that no two edges in the subset share common vertices. A maximum matching is a matching that contains the largest possible number of edges. The number of edges in a maximum matching of $\Gamma$ is called the matching number, denoted by $\nu(\Gamma)$, of $\Gamma$. A clique of $\Gamma$ is defined as a subset of $V(\Gamma)$ such that every two distinct vertices of the subset are adjacent. A maximum clique is a clique such that there is no clique with more vertices. The number of vertices in a maximum clique of $\Gamma$ is called the clique number, denoted by $\omega(\Gamma)$, of $\Gamma$. The bondage number of $\Gamma$, denoted by $b(\Gamma)$, is the cardinality of the smallest set $E$ of edges such that the domination number of $\Gamma$ after removing the edges in $E$ is strictly greater than that of original $\Gamma$. A subset $S$ of $V(\Gamma)$ is said to be an irredundant set of $\Gamma$ if $\left(\cup_{s \in S \setminus \{v\}}\Nbd(s)\right) \cup \left(S \setminus \{v\}\right) \neq \left(\cup_{s \in S}\Nbd(s)\right) \cup S$, for every vertex $v \in S$. A maximal irredundant set of $\Gamma$ is an irredundant set that cannot be expanded to another irredundant set by addition of any vertex of $\Gamma$. The irredundance number, denoted by $\ir(\Gamma)$, is the minimum size of a maximal irredundant set of $\Gamma$. A domatic partition of $\Gamma$ is a partition of $V(\Gamma)$ into disjoint sets $V_1, V_2, \ldots, V_k$ such that each $V_i$ is a dominating set of $\Gamma$. The maximum size of a domatic partition is called domatic number of $\Gamma$, denoted by $d(\Gamma)$. A vertex cover of $\Gamma$ is a set of vertices of $\Gamma$ that includes at least one endpoint of every edge of $\Gamma$. We write $\beta(\Gamma)$ to denote the minimum size of a vertex cover of $\Gamma$. Note that $\alpha(\Gamma) + \beta(\Gamma) = |V(\Gamma)|$ (see \cite[Corollary 7.1]{BM1977}). We obtain all the above mentioned graph parameters for $\mathcal{B}(G)$ in the following result. \begin{theorem} For any group $G$, the graph $\mathcal{B}(G)$ has the following properties: \begin{enumerate} \item $\girth(\mathcal{B}(G))= 0$ and $\diam(\mathcal{B}(G)) =1$ or $\infty$. \item $\nu(\mathcal{B}(G))=|L_2(G)|$. \item $\omega(\mathcal{B}(G)) = 2$. \end{enumerate} \end{theorem} \begin{proof} \begin{enumerate} \item By Observation \ref{vrtex_deg_of_X_in_B(G)}(b), it follows that $\mathcal{B}(G)$ has no cycle. Therefore, $\girth(\mathcal{B}(G))= 0$. The second part follows from Observation \ref{vrtex_deg_of_X_in_B(G)}(c). \item Note that every edge of $\mathcal{B}(G)$ is incident to some $H \in L_2(G)$. Consider a subset $E$ of $e(\mathcal{B}(G))$, the edge set of $\mathcal{B}(G)$, such that there exists only one edge in $E$ with $H$ as its endpoint for each $H \in L_2(G)$. Clearly, $E$ is a matching in $\mathcal{B}(G)$ and $|E|=|L_2(G)|$. Now, if we include any more edge to $E$, there will be two edges in $E$ having a common endpoint $H$ for some $H \in L_2(G)$. Therefore, $E$ is a maximum matching and $\nu(\mathcal{B}(G))=|E|$. Hence, the result follows. \item Note that $(1, 1)$ and $\{1\}$ are always adjacent in $\mathcal{B}(G)$. Therefore, $\mathcal{B}(G)$ has a clique of size two. Thus, $\omega(\mathcal{B}(G)) \geq 2$. Suppose that $\omega(\mathcal{B}(G)) \geq 3$. Then $\mathcal{B}(G)$ must have a cycle of length greater than or equal to three, which is a contradiction. Therefore, $\omega(\mathcal{B}(G))=2$. \end{enumerate} \end{proof} \begin{theorem} For a $2$-generated group $G$, we have $b(\mathcal{B}(G)) =1$, $\ir(\mathcal{B}(G))=\beta(\mathcal{B}(G))=|L(G)|$ and $d(\mathcal{B}(G))=2$. \end{theorem} \begin{proof} By Theorem \ref{independence-domination no. of B(G)}, we have domination number of $\mathcal{B}(G)=|B|=|L(G)|$. Also, $\deg_{\mathcal{B}(G)}((a,b))=1$ for any $(a,b)\in G \times G$. If we remove any edge from $\mathcal{B}(G)$, $L(G)$ will not be a dominating set and any other dominating set will have size at least one more than $|L(G)|$. This increases the domination number of the new graph by at least one. Therefore, $b(\mathcal{B}(G))=1$. By definition, we have $G \times G$ and $L(G)$ both are maximal irredundant sets of $\mathcal{B}(G)$. From Theorem \ref{size of A bigger than that of B}, we know $|G|^2 \geq |L(G)|$. Therefore, $\ir(\mathcal{B}(G))=|L(G)|$. We have $\alpha(\mathcal{B}(G))+\beta(\mathcal{B}(G))=|V(\mathcal{B}(G))|$, where $\alpha(\mathcal{B}(G))$ is the independence number of $\mathcal{B}(G)$. From Theorem \ref{independence-domination no. of B(G)}, we have $\alpha(\mathcal{B}(G))=|G|^2$. Therefore $\beta(\mathcal{B}(G))=|G|^2+|L(G)|-|G|^2=|L(G)|$. We have $V(\mathcal{B}(G))$ is the disjoint union of $G \times G$ and $L(G)$. Also, both $G \times G$ and $L(G)$ are dominating sets of $\mathcal{B}(G)$. As such, $d(\mathcal{B}(G))\geq 2$. It was shown in \cite{CH-1977} that $d(\Gamma) \leq \delta(\Gamma) + 1$ for any graph $\Gamma$, where $\delta(\Gamma)$ is the minimum degree of $\Gamma$. In our case, $\delta(\mathcal{B}(G)) = 1$ and so $d(\mathcal{B}(G)) \leq 1+1=2$. Hence, $d(\mathcal{B}(G))=2$. \end{proof} \begin{remark} Let $G$ be an $r$-generated group where $r \geq 3$. Then \begin{enumerate} \item The domination number of $\mathcal{B}(G)$ is \, $\min\left\{|G|^2+|L(G)|-|L_2(G)|, |L(G)|\right\}$. Removing any edge from $\mathcal{B}(G)$ will increase the domination number strictly by one. Therefore, $b(\mathcal{B}(G))=1$. \item Both $G \times G \sqcup (L(G) \setminus L_2(G))$ and $L(G)$ are maximal irredundant sets. If $|G|^2 \geq |L(G)|$ then $\ir(\mathcal{B}(G))=|L(G)|$. In general, $$\ir(\mathcal{B}(G))=\min\left\{|G|^2+|L(G)|-|L_2(G)|, |L(G)|\right\}.$$ \item From Remark \ref{first remark for r-generated}(b), we have independence number of $\mathcal{B}(G)$ is $|G|^2+|L(G)|-|L_2(G)|$. As such, $\beta(\mathcal{B}(G))=|G|^2+|L(G)|-(|G|^2+|L(G)|-|L_2(G)|)=|L_2(G)|$. \item Domatic partition of $V(\mathcal{B}(G))$ does not exist since $L(G)$ is the only dominating set in $\mathcal{B}(G)$. \end{enumerate} \end{remark} \section{Relations between \, $\mathcal{B}(G)$ \, and probabilities associated to finite groups} In this section, we obtain relations between $\mathcal{B}(G)$ and certain probabilities associated to finite groups. Let $G$ be a finite group and $H$ be any given subgroup of $G$. The probability that a randomly chosen pair of elements of $G$ generate $H$ is called the probability generating a given subgroup. We write $\Pr_H(G)$ to denote this probability. Therefore, \begin{equation}\label{SGP} {\Pr}_H(G)= \frac{|\{(a, b) \in G \times G : \langle a, b \rangle = H\}|}{|G|^2}. \end{equation} \begin{obj} \begin{enumerate} \item $\Pr_H(G)= 1$ if and only if $H=G=\{1\}$. \item $\Pr_H(G)= 1-\frac{1}{|G|^2}$ if and only if $H=G$ is a group of prime order. \item $\Pr_H(G)= \frac{1}{|G|^2}$ if and only if $H=\{1\}$. \end{enumerate} \end{obj} \noindent Note that $\Pr_G(G) := \varphi_2(G)$ is the probability that a randomly chosen pair of elements of $G$ generate $G$. Dixon \cite{Di69} obtained a lower bound for $\Pr_{A_5}(A_5)$ for the first time. Results on $\Pr_G(G)$ for symmetric group and finite simple groups can be found in \cite{Ba89, LS95, LS96}. The study of the generalized version of $\varphi_2(G)$, viz. \[ \varphi_n(G) = \frac{|\{(x_1, \dots, x_n) \in G \times \cdots \times G : \langle x_1, \dots, x_n\rangle = G\}|}{|G|^n} \] goes back to Hall \cite{Hall36}. Results on $\varphi_n(G)$ can be found in \cite{Pak99}. The probability that a randomly chosen pair of elements of $G$ commute is called the commuting probability of $G$. It is also known as commutativity degree of $G$. We write $\Pr(G)$ to denote this probability. Therefore, \begin{align*} \Pr(G) &= \frac{|\{(a, b) \in G \times G : ab = ba \}|}{|G|^2} \\ &= \frac{|\{(a, b) \in G \times G : \langle a, b \rangle \text{ is abelian} \}|}{|G|^2}. \end{align*} The origin of $\Pr(G)$ lies in a paper of Erd$\ddot{\rm{o}}$s and Tur$\acute{\rm a}$n \cite{ET69}. Results on $\Pr(G)$ can be found in the survey \cite{DNP-13}. A relation between the number of edges in commuting/non-commuting graph and $\Pr(G)$ of $G$ was observed in \cite{AKM06, TE-13}. Notions similar to $\Pr(G)$, viz. cyclicity degree (denoted by $\Pr_{\cyc}(G)$ and introduced in \cite{PSSW93}), nilpotency degree (denoted by $\Pr_{\nil}(G)$ and introduced in \cite{DGMW92}) and solvability degree (denoted by ${\Pr}_{\sol}(G)$ and introduced in \cite{FGSV2000}) are defined as follows: \[ {\Pr}_{\cyc}(G)= \frac{|\{(a, b) \in G \times G : \langle a, b \rangle \text{ is cyclic} \}|}{|G|^2}, \] \[ {\Pr}_{\nil}(G)= \frac{|\{(a, b) \in G \times G : \langle a, b \rangle \text{ is nilpotent} \}|}{|G|^2} \] and \[ {\Pr}_{\sol}(G)= \frac{|\{(a, b) \in G \times G : \langle a, b \rangle \text{ is solvable} \}|}{|G|^2}. \] Relation between the number of edges in solvable/non-solvable graph and ${\Pr}_{\sol}(G)$ of $G$ was observed in \cite{BNN2020}. Relations similar to \cite[Lemma 3.27]{AKM06} and \cite[Theorem 4.5]{BNN2020} can also be determined for cyclic graph and nilpotent graph. In this section, we obtain certain relations among $\mathcal{B}(G)$, $\Pr(G)$, $\Pr_{\cyc}(G)$, $\Pr_{\nil}(G)$ and $\Pr_{\sol}(G)$. The following lemma is useful in this regard. \begin{lemma}\label{deg(H in L(G))} Let $G$ be a finite group and $H$ be a subgroup of $G$. Then \[ {\Pr}_H(G) = \frac{\deg_{\mathcal{B}(G)}(H)}{|G|^2}. \] \end{lemma} \begin{proof} We have $\deg_{\mathcal{B}(G)}(H)=|\Nbd_{\mathcal{B}(G)}(H)|$, where \begin{align*} \Nbd_{\mathcal{B}(G)}(H) & =\{(a, b) \in G \times G: (a, b) \text{ is adjacent to H}\}\\ & =\{(a, b) \in G \times G: \langle a, b\rangle = H\}. \end{align*} Hence, the result follows from \eqref{SGP}. \end{proof} \begin{theorem} Let $G$ be a finite group and $e(\mathcal{B}(G))$ be the set of edges of the graph $\mathcal{B}(G)$. Then \[ \sum_{(a, b)\in G\times G} \deg_{\mathcal{B}(G)}((a, b))= \sum_{H \in L(G)} \deg_{\mathcal{B}(G)}(H)= |G|^2=|e(\mathcal{B}(G))| \] \end{theorem} \begin{proof} For any bipartite graph $\mathcal{G}$ with partitions $A$ and $B$ of $V(\mathcal{G})$, we have \begin{equation}\label{deg_sum=num_of_edges} \sum_{x\in A} \deg_{\mathcal{G}}(x)= \sum_{y \in B} \deg_{\mathcal{G}}(y)=|e(\mathcal{G})|. \end{equation} Therefore, for the graph $\mathcal{B}(G)$ we have \[ \sum_{(a, b)\in G\times G} \deg_{\mathcal{B}(G)}((a, b))= \sum_{H \in L(G)} \deg_{\mathcal{B}(G)}(H)= |e(\mathcal{B}(G))|. \] Since $\sum_{H \in L(G)}{\Pr}_H(G) = 1$, by Lemma \ref{deg(H in L(G))}, we have \[ \sum_{H \in L(G)} \deg_{\mathcal{B}(G)}(H)= \sum_{H \in L(G)}|G|^2 {\Pr}_H(G) = |G|^2. \] Hence the result follows. \end{proof} \begin{theorem}\label{relation between B(G) and varphi_2(G)} Let $G$ be a finite group and $H \in L(G)$. Then $ \varphi_2(H) = \dfrac{\deg_{\mathcal{B}(G)}(H)}{|H|^2}. $ \end{theorem} \begin{proof} For $a \neq b$, the pairs $(a,b)$ and $(b,a)$ that generate $H$, contribute one edge in $\Gamma_{\gen}(H)$ and two edges in $\mathcal{B}(G)$. It follows that \begin{align*} 2|e(\Gamma_{\gen}(H))| &=\left|\{(a,b) \in H \times H: \langle a,b \rangle =H, a \neq b\}\right| \\ &=\left|\{(a,b) \in H \times H: \langle a,b \rangle =H\}\right| - \left|\{(a,a) \in H \times H: \langle a \rangle=H\}\right| \\ &= |H|^2 \varphi_2(H)-\phi(|H|), \end{align*} noting that $\varphi_2(H)=\frac{|\{(a,b) \in H \times H: \langle a, b \rangle=H\}|}{|H|^2}$ and $\phi(|H|) = 0$ or the number of generators of $\mathbb{Z}_{|H|}$ according as $H$ is non-cyclic or cyclic. Thus, \[ |H|^2 \varphi_2(H) = 2|e(\Gamma_{\gen}(H))| + \phi(|H|). \] Hence, the result follows from Theorem \ref{relatn B(G) and generating graph}. \end{proof} \begin{theorem} Let $G$ be a finite group and $L_A(G)=\{H \in L(G): H \text{ is abelian}\}$. If $S=(G \times G) \sqcup L_{A}(G)$, then \[ \Pr(G) = \frac{\sum_{H \in L_A(G)} \deg_{\mathcal{B}(G)[S]}(H)}{|G|^2}=\frac{|e(\mathcal{B}(G)[S])|}{|G|^2}. \] \end{theorem} \begin{proof} Since $L_A(G)$ is the set of all abelian subgroups of $G$, from the definitions of $\Pr_H(G)$ and $\Pr(G)$, we have \[ \Pr(G)=\sum_{H \in L_A(G)}{\Pr}_H(G). \] Also, $\deg_{\mathcal{B}(G)[S]}(H) = \deg_{\mathcal{B}(G)}(H)$ for all $H \in L_A(G)$. Therefore, using Lemma \ref{deg(H in L(G))}, we get \begin{align*} \sum_{H \in L_A(G)} \deg_{\mathcal{B}(G)[S]}(H)&=\sum_{H \in L_A(G)}|G|^2 {\Pr}_H(G) \\ &=|G|^2\sum_{H \in L_A(G)}{\Pr}_H(G) \\ &=|G|^2 \Pr(G). \end{align*} By \eqref{deg_sum=num_of_edges} we have \[ \sum_{H \in L_A(G)} \deg_{\mathcal{B}(G)[S]}(H)=|e(\mathcal{B}(G)[S])|. \] Hence, the result follows. \end{proof} \begin{theorem} Let $G$ be a finite group and $L_C(G)=\{H \in L(G): H \text{ is cyclic}\}$. If $S=(G \times G) \sqcup L_{C}(G)$, then \[ {\Pr}_{\cyc}(G)= \frac{\sum_{H \in L_C(G)} \deg_{\mathcal{B}(G)[S]}(H)}{|G|^2}= \frac{|e(\mathcal{B}(G)[S])|}{|G|^2}. \] \end{theorem} \begin{proof} Since $L_C(G)$ is the set of all cyclic subgroups of $G$, from the definitions of $\Pr_H(G)$ and $\Pr_{\cyc}(G)$, we have \[ {\Pr}_{\cyc}(G)=\sum_{H \in L_C(G)}{\Pr}_H(G). \] Also, $\deg_{\mathcal{B}(G)[S]}(H) = \deg_{\mathcal{B}(G)}(H)$ for all $H \in L_C(G)$. Therefore, using Lemma \ref{deg(H in L(G))}, we get \begin{align*} \sum_{H \in L_C(G)} \deg_{\mathcal{B}(G)[S]}(H)&=\sum_{H \in L_C(G)}|G|^2 {\Pr}_H(G) \\ &=|G|^2\sum_{H \in L_C(G)}{\Pr}_H(G) \\ &=|G|^2 {\Pr}_{\cyc}(G). \end{align*} By \eqref{deg_sum=num_of_edges} we have \[ \sum_{H \in L_C(G)} \deg_{\mathcal{B}(G)[S]}(H)=|e(\mathcal{B}(G)[S])|. \] Hence, the result follows. \end{proof} \begin{theorem} Let $G$ be a finite group and $L_N(G)=\{H \in L(G): H \text{ is nilpotent}\}$. If $S=(G \times G) \sqcup L_{N}(G)$, then \[ {\Pr}_{\nil}(G)= \frac{\sum_{H \in L_N(G)} \deg_{\mathcal{B}(G)[S]}(H)}{|G|^2}= \frac{|e(\mathcal{B}(G)[S])|}{|G|^2}. \] \end{theorem} \begin{proof} Since $L_N(G)$ is the set of all nilpotent subgroups of $G$, from the definitions of $\Pr_H(G)$ and $\Pr_{\nil}(G)$, we have \[ {\Pr}_{\nil}(G)=\sum_{H \in L_N(G)}{\Pr}_H(G). \] Also, $\deg_{\mathcal{B}(G)[S]}(H) = \deg_{\mathcal{B}(G)}(H)$ for all $H \in L_N(G)$. Therefore, using Lemma \ref{deg(H in L(G))}, we get \begin{align*} \sum_{H \in L_N(G)} \deg_{\mathcal{B}(G)[S]}(H)&=\sum_{H \in L_N(G)}|G|^2 {\Pr}_H(G) \\ &=|G|^2\sum_{H \in L_N(G)}{\Pr}_H(G) \\ &=|G|^2 {\Pr}_{\nil}(G). \end{align*} By \eqref{deg_sum=num_of_edges} we have \[ \sum_{H \in L_N(G)} \deg_{\mathcal{B}(G)[S]}(H)=|e(\mathcal{B}(G)[S])|. \] Hence, the result follows. \end{proof} \begin{theorem} Let $G$ be a finite group and $L_S(G)=\{H \in L(G): H \text{ is solvable}\}$. If $S=(G \times G) \sqcup L_{S}(G)$, then \[ {\Pr}_{\sol}(G)=\frac{\sum_{H \in L_S(G)} \deg_{\mathcal{B}(G)[S]}(H)}{|G|^2} = \frac{|e(\mathcal{B}(G)[S])|}{|G|^2}. \] \end{theorem} \begin{proof} Since $L_S(G)$ is the set of all solvable subgroups of $G$, from the definitions of $\Pr_H(G)$ and $\Pr_{\sol}(G)$, we have \[ {\Pr}_{\sol}(G)=\sum_{H \in L_S(G)}{\Pr}_H(G). \] Also, $\deg_{\mathcal{B}(G)[S]}(H) = \deg_{\mathcal{B}(G)}(H)$ for all $H \in L_S(G)$. Therefore, using Lemma \ref{deg(H in L(G))}, we get \begin{align*} \sum_{H \in L_S(G)} \deg_{\mathcal{B}(G)[S]}(H)&=\sum_{H \in L_S(G)}|G|^2 {\Pr}_H(G) \\ &=|G|^2\sum_{H \in L_S(G)}{\Pr}_H(G) \\ &=|G|^2 {\Pr}_{\sol}(G). \end{align*} By \eqref{deg_sum=num_of_edges} we have $ \sum_{H \in L_S(G)} \deg_{\mathcal{B}(G)[S]}(H)=|e(\mathcal{B}(G)[S])|. $ Hence, the result follows. \end{proof} Let $\mathcal{G}$ be the set of all graphs. A topological index is a function $T : \mathcal{G} \to \mathbb{R}$ such that $T(\Gamma_1) = T(\Gamma_2)$ whenever the graphs $\Gamma_1$ and $\Gamma_2$ are isomorphic. Some of the well-known degree-based topological indices are Zagreb indices, Randic Connectivity index, Atom-Bond Connectivity index, Geometric-Arithmetic index, Harmonic index, Sum-Connectivity index etc. A survey on degree-based topological indices can be found in \cite{MNJ-FA-2020}. Let $\Gamma \in \mathcal{G}$. The first and second Zagreb indices of $\Gamma$, denoted by $M_{1}(\Gamma)$ and $M_{2}(\Gamma)$ respectively, are defined as \[ M_{1}(\Gamma) = \sum\limits_{v \in V(\Gamma)} \deg(v)^{2} \text{ and } M_{2}(\Gamma) = \sum\limits_{uv \in e(\Gamma)} \deg(u)\deg(v). \] The Randic Connectivity index of $\Gamma$, denoted by $R(\Gamma)$, is defined as \[ R(\Gamma)=\sum_{uv \in e(\Gamma)}\left(\deg(u)\deg(v)\right)^{\frac{-1}{2}}. \] The Atom-Bond Connectivity index of $\Gamma$, denoted by $\ABC(\Gamma)$, is defined as \[ \ABC(\Gamma)=\sum_{uv\in e(\Gamma)}\left(\frac{\deg(u)+\deg(v)-2}{\deg(u)\deg(v)}\right)^{\frac{1}{2}}. \] The Geometric-Arithmetic index of $\Gamma$, denoted by $\GA(\Gamma)$, is defined as \[ \GA(\Gamma)=\sum_{uv\in e(\Gamma)} \frac{\sqrt{\deg(u)\deg(v)}}{\frac{1}{2}(\deg(u)+\deg(v))}. \] The Harmonic index of $\Gamma$, denoted by $H(\Gamma)$, is defined as \[ H(\Gamma)=\sum_{uv\in e(\Gamma)}\frac{2}{\deg(u)+\deg(v)}. \] The Sum-Connectivity index of $\Gamma$, denoted by $\SCI(\Gamma)$, is defined as \[ \SCI(\Gamma)=\sum_{uv \in e(\Gamma)} \left(\deg(u)+\deg(v)\right)^{\frac{-1}{2}}. \] In the following theorem we obtain the above mentioned topological indices of $\mathcal{B}(G)$ in terms of $\varphi_2(G)$ using Theorem \ref{relation between B(G) and varphi_2(G)}. \begin{theorem} For any finite group $G$ we have the following: \begin{enumerate} \item $M_1(\mathcal{B}(G))=|G|^2+ \sum\limits_{H \in L(G)}|H|^4\left(\varphi_2(H)\right)^2$ and \qquad\qquad\qquad\qquad\qquad\qquad\qquad\qquad\,$M_2(\mathcal{B}(G))=\sum\limits_{H \in L(G)}|H|^4 \left(\varphi_2(H)\right)^2$. \item $R(\mathcal{B}(G))=\sum\limits_{H \in L(G)}|H|\left(\varphi_2(H)\right)^{\frac{1}{2}}$. \item $\ABC(\mathcal{B}(G))=\sum\limits_{H \in L(G)}|H|\left(\left(|H|\varphi_2(H)\right)^2-\varphi_2(H)\right)^{\frac{1}{2}}$. \item $\GA(\mathcal{B}(G))=\sum\limits_{H \in L(G)} \frac{2|H|^3\left( \varphi_2(H)\right)^{\frac{3}{2}}}{(1+|H|^2 \varphi_2(H))}$. \item $H(\mathcal{B}(G)=\sum\limits_{H \in L(G)}\frac{2|H|^2 \varphi_2(H)}{1+|H|^2 \varphi_2(H)}$. \item $\SCI(\mathcal{B}(G))=\sum\limits_{H \in L(G)} |H|^2 \varphi_2(H)\left(1+|H|^2 \varphi_2(H)\right)^{\frac{-1}{2}}$. \end{enumerate} \end{theorem} \begin{proof} For $(a, b) \in G \times G$ and $H \in L(G)$, by Observation \ref{vrtex_deg_of_X_in_B(G)}(a) and Theorem \ref{relation between B(G) and varphi_2(G)}, we have $\deg_{\mathcal{B}(G)}((a, b))=1$ and $\deg_{\mathcal{B}(G)}(H)=|H|^2 \varphi_2(H)$. \begin{enumerate} \item We have \begin{align*} M_{1}(\mathcal{B}(G)) &= \sum_{v \in V(\mathcal{B}(G))} \deg(v)^{2} \\ &=\sum_{(a, b) \in G \times G}\left(\deg_{\mathcal{B}(G)}((a, b))\right)^2+\sum_{H \in L(G)}\left(\deg_{\mathcal{B}(G)}(H)\right)^2\\ &= \sum_{(a, b) \in G \times G} 1 + \sum_{H \in L(G)}\left(|H|^2\varphi_2(H)\right)^2 =|G|^2+ \sum_{H \in L(G)}|H|^4\left(\varphi_2(H)\right)^2. \end{align*} Also, \begin{align*} M_2&(\mathcal{B}(G))= \sum_{uv \in e(\mathcal{B}(G))} \deg(u)\deg(v)\\ &\quad=\sum_{(a, b)H \in e(\mathcal{B}(G))}\deg_{\mathcal{B}(G)}((a, b))\deg_{\mathcal{B}(G)}(H) = \sum_{(a, b)H \in e(\mathcal{B}(G))} \deg_{\mathcal{B}(G)}(H). \end{align*} In the above sum, $\deg_{\mathcal{B}(G)}(H)$ appears $\deg_{\mathcal{B}(G)}(H)$ many times for each $H \in L(G)$. Therefore, \begin{align*} M_2(\mathcal{B}(G))&= \sum_{H \in L(G)} \left(\deg_{\mathcal{B}(G)}(H)\right)^2 \\ &= \sum_{H \in L(G)}\left(|H|^2 \varphi_2(H)\right)^2 = \sum_{H \in L(G)}|H|^4 \left(\varphi_2(H)\right)^2. \end{align*} \item We have \begin{align*} R(\mathcal{B}(G))&=\sum_{uv \in e(\mathcal{B}(G))}\left(\deg(u)\deg(v)\right)^{\frac{-1}{2}} \\ &= \sum_{(a, b)H \in e(\mathcal{B}(G))}\left(\deg_{\mathcal{B}(G)}((a, b))\deg_{\mathcal{B}(G)}(H)\right)^{\frac{-1}{2}} \\ &= \sum_{(a, b)H \in e(\mathcal{B}(G))}\left(\deg_{\mathcal{B}(G)}(H)\right)^{\frac{-1}{2}}. \end{align*} In the above sum, \quad $\left(\deg_{\mathcal{B}(G)}(H)\right)^{\frac{-1}{2}}$ appears $\deg_{\mathcal{B}(G)}(H)$ many times for each $H \in L(G)$. Therefore, \begin{align*} R&(\mathcal{B}(G))= \sum_{H \in L(G)}\left(\deg_{\mathcal{B}(G)}(H)\right)^{\frac{-1}{2}} \deg_{\mathcal{B}(G)}(H) \\ &= \sum_{H \in L(G)}\left(\deg_{\mathcal{B}(G)}(H)\right)^{\frac{1}{2}} = \sum_{H \in L(G)}\left(|H|^2 \varphi_2(H)\right)^{\frac{1}{2}}=\sum_{H \in L(G)}|H|\left(\varphi_2(H)\right)^{\frac{1}{2}}. \end{align*} \item We have \begin{align*} \ABC(\mathcal{B}(G))&=\sum_{(a, b)H \in e(\mathcal{B}(G))}\left(\frac{\deg_{\mathcal{B}(G)}((a, b))+\deg_{\mathcal{B}(G)}(H)-2}{\deg_{\mathcal{B}(G)}((a, b))\deg_{\mathcal{B}(G)}(H)}\right)^{\frac{1}{2}} \\ &=\sum_{(a, b)H \in e(\mathcal{B}(G))}\left(\frac{1+\deg_{\mathcal{B}(G)}(H)-2}{\deg_{\mathcal{B}(G)}(H)}\right)^{\frac{1}{2}} \\ &=\sum_{(a, b)H \in e(\mathcal{B}(G))}\left(\frac{\deg_{\mathcal{B}(G)}(H)-1}{\deg_{\mathcal{B}(G)}(H)}\right)^{\frac{1}{2}}. \end{align*} In the above sum, \quad $\left(\frac{\deg_{\mathcal{B}(G)}(H)-1}{\deg_{\mathcal{B}(G)}(H)}\right)^{\frac{1}{2}}$ appears $\deg_{\mathcal{B}(G)}(H)$ many times for each $H \in L(G)$. Therefore, \begin{align*} \ABC(\mathcal{B}(G)) &=\sum_{H \in L(G)}\left(\frac{\deg_{\mathcal{B}(G)}(H)-1}{\deg_{\mathcal{B}(G)}(H)}\right)^{\frac{1}{2}} \deg_{\mathcal{B}(G)}(H) \\ &= \sum_{H \in L(G)}\left(\left(\deg_{\mathcal{B}(G)}(H)\right)^2-\deg_{\mathcal{B}(G)}(H)\right)^{\frac{1}{2}} \\ &=\sum_{H \in L(G)}\left(\left(|H|^2 \varphi_2(H)\right)^2-|H|^2 \varphi_2(H)\right)^{\frac{1}{2}}. \end{align*} Hence, the result follows. \item We have \begin{align*} \GA(\mathcal{B}(G))&=\sum_{(a, b)H \in e(\mathcal{B}(G))} \frac{\sqrt{\deg_{\mathcal{B}(G)}((a, b))\deg_{\mathcal{B}(G)}(H)}}{\frac{1}{2}(\deg_{\mathcal{B}(G)}((a, b))+\deg_{\mathcal{B}(G)}(H))} \\ &=\sum_{(a, b)H \in e(\mathcal{B}(G))} \frac{2\sqrt{\deg_{\mathcal{B}(G)}(H)}}{(1+\deg_{\mathcal{B}(G)}(H))} \end{align*} In the above sum, \quad $\frac{2\sqrt{\deg_{\mathcal{B}(G)}(H)}}{(1+\deg_{\mathcal{B}(G)}(H))}$ \quad appears $\deg_{\mathcal{B}(G)}(H)$ many times for each $H \in L(G)$. Therefore, \begin{align*} \ABC(\mathcal{B}(G)) &=\sum_{H \in L(G)} \frac{2\sqrt{\deg_{\mathcal{B}(G)}(H)}}{(1+\deg_{\mathcal{B}(G)}(H))}\deg_{\mathcal{B}(G)}(H)\\ &= \sum_{H \in L(G)} \frac{2\left(\deg_{\mathcal{B}(G)}(H)\right)^{\frac{3}{2}}}{(1+\deg_{\mathcal{B}(G)}(H))} = \sum_{H \in L(G)} \frac{2\left(|H|^2 \varphi_2(H)\right)^{\frac{3}{2}}}{(1+|H|^2 \varphi_2(H))}. \end{align*} Hence, the result follows. \item We have \begin{align*} H(\mathcal{B}(G))&=\sum_{(a, b)H \in e(\mathcal{B}(G))}\frac{2}{\deg_{\mathcal{B}(G)}((a, b))+\deg_{\mathcal{B}(G)}(H)} \\ &= \sum_{(a, b)H \in e(\mathcal{B}(G))}\frac{2}{1+\deg_{\mathcal{B}(G)}(H)} \end{align*} In the above sum, $\frac{2}{1+\deg_{\mathcal{B}(G)}(H)}$ appears $\deg_{\mathcal{B}(G)}(H)$ many times for each $H \in L(G)$. Therefore, \begin{align*} H(\mathcal{B}(G)) &=\sum_{H \in L(G)}\frac{2}{1+\deg_{\mathcal{B}(G)}(H)}\deg_{\mathcal{B}(G)}(H) =\sum_{H \in L(G)}\frac{2|H|^2 \varphi_2(H)}{1+|H|^2 \varphi_2(H)}. \end{align*} \item We have \begin{align*} \SCI(\mathcal{B}(G))&=\sum_{(a, b)H \in e(\mathcal{B}(G))} \left(\deg_{\mathcal{B}(G)}((a, b))+\deg_{\mathcal{B}(G)}(H)\right)^{\frac{-1}{2}} \\ &= \sum_{(a, b)H \in e(\mathcal{B}(G))} \left(1+\deg_{\mathcal{B}(G)}(H)\right)^{\frac{-1}{2}}. \end{align*} In the above sum, $\left(1+\deg_{\mathcal{B}(G)}(H)\right)^{\frac{-1}{2}}$ appears $\deg_{\mathcal{B}(G)}(H)$ many times for each $H \in L(G)$. Therefore, \begin{align*} \SCI(\mathcal{B}(G))&=\sum_{H \in L(G)} \left(1+\deg_{\mathcal{B}(G)}(H)\right)^{\frac{-1}{2}}\deg_{\mathcal{B}(G)}(H) \\ &=\sum_{H \in L(G)} |H|^2 \varphi_2(H)\left(1+|H|^2 \varphi_2(H)\right)^{\frac{-1}{2}}. \end{align*} \end{enumerate} \vspace{-.8cm} \end{proof} We conclude this section with the following table describing $|L_C(G)|$, $|L_A(G)|$, $|L_N(G)|$, $|L_S(G)|$, $|e(\mathcal{B}(G))|$ and various probabilities defined on finite groups for certain small order groups. \begin{table}[h] \begin{center} {{ \begin{tabular}{|c|c|c|c|c|c|c|c|} \hline $G$& $S_3$ & $D_8$ & $Q_8$ & $D_{10}$ & $D_{12}$ & $A_4$ & $S_4$ \\ \hline $|G|$ & 6 & 8 & 8 & 10 & 12 & 12 & 24 \\ \hline $|L_C(G)|$ & 5 & 7 & 5 & 7 & 10 & 8 & 17 \\ \hline $|L_A(G)|$ & 5 & 9 & 5 & 7 & 13 & 9 & 21 \\ \hline $|L_N(G)|$ & 5 & 10 & 6 & 7 & 13 & 9 & 24 \\ \hline $|L_S(G)|$ & 6 & 10 & 6 & 8 & 16 & 10 & 30 \\ \hline $|e(\mathcal{B}(G))|$ & 36 & 64 & 64 & 100 & 144 & 144 & 576 \\ \hline $\Pr_{\cyc}(G)$ & $\frac{1}{2}$ & $\frac{7}{16}$ & $\frac{5}{8}$ & $\frac{2}{5}$ &$ \frac{3}{8}$ & $\frac{7}{24}$ & $\frac{1}{6}$ \\ \hline $\Pr(G)$ & $\frac{1}{2}$ & $\frac{5}{8}$ & $\frac{5}{8}$ & $\frac{2}{5}$ & $\frac{1}{2}$ & $\frac{1}{3}$ & $\frac{5}{24}$ \\ \hline $\Pr_{\nil}(G)$ & $\frac{1}{2}$ & 1 & 1 & $\frac{2}{5}$ & $\frac{1}{2}$ & $\frac{1}{3}$ & $\frac{1}{3}$ \\ \hline $\Pr_{\sol}(G)$ & 1 & 1 & 1 & 1 & 1 & 1 & 1 \\ \hline $\varphi_2(G)$ &$\frac{1}{2}$ & $\frac{3}{8}$ & $\frac{3}{8}$ & $\frac{3}{5}$ & $\frac{3}{8}$ & $\frac{2}{3}$ & $\frac{3}{8}$ \\ \hline \end{tabular} }} \caption{Various probabilities of small order groups}\label{Table 1} \end{center} \end{table} \newpage \section{Realization of $\mathcal{B}(G)$} Graph realization is one of the major aspects in studying graphs defined on algebraic systems. In Table \ref{Table 1}, while computing $|e(\mathcal{B}(G))|$ for various groups we realized the structures of $\mathcal{B}(G)$ for $G = S_3, D_8, Q_8, D_{10}, D_{12}, A_4$ and $S_4$. For instance, $V(\mathcal{B}(S_3)) = S_3 \times S_3 \sqcup \{H_0, H_1, \dots, H_4, S_3\}$ where $H_0=\{(1)\}$, $H_1=\{(1), (12)\}$, $H_2=\{(1), (13)\}$, $H_3=\{(1), (23)\}$ and $H_4=\{(1), (123), (132)\}$. We have $\Nbd_{\mathcal{B}(S_3)}(H_0)=\{((1),(1))\}$, $\Nbd_{\mathcal{B}(S_3)}(H_i)=H_i \times H_i \setminus \{((1),(1))\}$ for $1 \leq i \leq 4$ and $\Nbd_{\mathcal{B}(S_3)}(S_3)=S_3 \times S_3 \setminus \left(\sqcup_{i=0}^{4} \Nbd_{\mathcal{B}(S_3)}(H_i)\right)$. Since the vertices from $S_3 \times S_3$ have degree one, we have the following structure of $\mathcal{B}(S_3)$. \begin{center} \begin{tikzpicture} \tikzstyle{vertex}=[circle,minimum size=0.1pt,fill=black!30,inner sep=1.5pt] \node[vertex](A) at (-9.7,0){}; \node[vertex](B) at (-9.7,-1){$H_0$}; \node[vertex](C) at (-7.9,-0.8){$H_1$}; \node[vertex](D) at (-7.9,0){}; \node[vertex](E) at (-8.5,-1.6){}; \node[vertex](F) at (-7.3,-1.6){}; \node[vertex](C1) at (-6.1,-0.8){$H_2$}; \node[vertex](D1) at (-6.1,0){}; \node[vertex](E1) at (-6.7,-1.6){}; \node[vertex](F1) at (-5.5,-1.6){}; \node[vertex](C2) at (-4.3,-0.8){$H_3$}; \node[vertex](D2) at (-4.3,0){}; \node[vertex](E2) at (-4.9,-1.6){}; \node[vertex](F2) at (-3.7,-1.6){}; \node[vertex](G) at (-2,-0.8){$H_4$}; \node[vertex](H) at (-2,0){}; \node[vertex](I) at (-2,-1.6){}; \node[vertex](J) at (-2.8,-0.8){}; \node[vertex](K) at (-1.2,-0.8){}; \node[vertex](L) at (-2.7,-0.2){}; \node[vertex](M) at (-1.3,-0.2){}; \node[vertex](N) at (-2.7,-1.4){}; \node[vertex](O) at (-1.3,-1.4){}; \node[vertex](P) at (0.8,-0.8){$S_3$}; \node[vertex](Q) at (0.8,0){}; \node[vertex](R) at (0.3,0){}; \node[vertex](S) at (-0.2,0){}; \node[vertex](T) at (1.3,0){}; \node[vertex](U) at (1.8,0){}; \node[vertex](V) at (2.1,-0.3){}; \node[vertex](W) at (2.1,-0.6){}; \node[vertex](X) at (2.1,-0.9){}; \node[vertex](Y) at (2.1,-1.2){}; \node[vertex](Z) at (1.8,-1.6){}; \node[vertex](a) at (1.3,-1.6){}; \node[vertex](b) at (0.8,-1.6){}; \node[vertex](c) at (0.3,-1.6){}; \node[vertex](d) at (-0.2,-1.6){}; \node[vertex](e) at (-0.5,-0.3){}; \node[vertex](f) at (-0.5,-0.6){}; \node[vertex](g) at (-0.5,-0.9){}; \node[vertex](h) at (-0.5,-1.2){}; \path (A) edge (B) (C) edge (D) (C) edge (E) (C) edge (F) (C1) edge (D1) (C1) edge (E1) (C1) edge (F1) (C2) edge (D2) (C2) edge (E2) (C2) edge (F2) (G) edge (H) (G) edge (K) (G) edge (J) (G) edge (I) (G) edge (L) (G) edge (M) (G) edge (N) (G) edge (O) (P) edge (Q) (P) edge (R) (P) edge (S) (P) edge (T) (P) edge (U) (P) edge (V) (P) edge (W) (P) edge (X) (P) edge (Y) (P) edge (Z) (P) edge (a) (P) edge (b) (P) edge (c) (P) edge (d) (P) edge (e) (P) edge (f) (P) edge (g) (P) edge (h); \end{tikzpicture} \captionof{figure}{Graph structure of $\mathcal{B}(S_3)$} \label{fig:fig1} \end{center} From Figure \ref{fig:fig1}, it is clear that $\mathcal{B}(S_3)=K_2 \sqcup 3K_{1, 3} \sqcup K_{1, 8} \sqcup K_{1, 18}$. We know that $D_6 \cong S_3$. Therefore, $\mathcal{B}(D_6) \cong \mathcal{B}(S_3) =K_2 \sqcup 3K_{1, 3} \sqcup K_{1, 8} \sqcup K_{1, 18}$. For the group $Q_8$, we have $V(\mathcal{B}(Q_8)) =Q_8\times Q_8 \sqcup \{H_0, H_1, \dots, H_4, Q_8\}$ where $H_0=\{1\}$, $H_1=\{1, a^2\}$, $H_2=\{1, a, a^2, a^3\}$, $H_3=\{1, a^2, b, a^2b\}$ and $H_4=\{1, a^2, ab, a^3b\}$. We have $\Nbd_{\mathcal{B}(Q_8)}(H_0)=\{(1,1)\}$, $\Nbd_{\mathcal{B}(Q_8)}(H_1)=H_1 \times H_1 \setminus \{(1,1)\}$, $\Nbd_{\mathcal{B}(Q_8)}(H_i)=H_i \times H_i \setminus \{\sqcup_{j=0}^{1}\Nbd_{\mathcal{B}(Q_8)}(H_j)\}$ for $2 \leq i \leq 4$ and $\Nbd_{\mathcal{B}(Q_8)}(Q_8)=Q_8 \times Q_8 \setminus \left(\sqcup_{i=0}^{4} \Nbd_{\mathcal{B}(Q_8)}(H_i)\right)$. Since the vertices from $Q_8 \times Q_8$ have degree one, we have the following structure of $\mathcal{B}(Q_8)$. \begin{center} \begin{tikzpicture} \tikzstyle{vertex}=[circle,minimum size=0.1pt,fill=black!30,inner sep=1.5pt] \node[vertex](A) at (-8.6,0){}; \node[vertex](B) at (-8.6,-1){$H_0$}; \node[vertex](C) at (-6,-0.8){$H_1$}; \node[vertex](D) at (-6,0){}; \node[vertex](E) at (-6.8,-1.6){}; \node[vertex](F) at (-5.2,-1.6){}; \node[vertex](C2) at (-3,-0.8){$H_2$}; \node[vertex](D2) at (-3,0){}; \node[vertex](E2) at (-3,-1.6){}; \node[vertex](F2) at (-3.9,-0.8){}; \node[vertex](G2) at (-2.1,-0.8){}; \node[vertex](H2) at (-3.5,-0.1){}; \node[vertex](I2) at (-3.8,-0.4){}; \node[vertex](J2) at (-2.5,-0.1){}; \node[vertex](K2) at (-2.2,-0.4){}; \node[vertex](L2) at (-3.8,-1.2){}; \node[vertex](M2) at (-3.5,-1.5){}; \node[vertex](N2) at (-2.2,-1.2){}; \node[vertex](O2) at (-2.5,-1.5){}; \node[vertex](G) at (0.5,-0.8){$H_3$}; \node[vertex](H) at (0.5,0){}; \node[vertex](I) at (0.5,-1.6){}; \node[vertex](J) at (-0.4,-0.8){}; \node[vertex](K) at (1.4,-0.8){}; \node[vertex](L) at (0,-0.1){}; \node[vertex](L1) at (-0.3,-0.4){}; \node[vertex](M) at (1,-0.1){}; \node[vertex](M1) at (1.3,-0.4){}; \node[vertex](N) at (1.3,-1.2){}; \node[vertex](N1) at (0,-1.5){}; \node[vertex](O) at (-0.3,-1.2){}; \node[vertex](O1) at (1,-1.5){}; \node[vertex](C3) at (-6,-3.2){$H_4$}; \node[vertex](D3) at (-6,-2.2){}; \node[vertex](E3) at (-6,-4.2){}; \node[vertex](F3) at (-7.1,-3.2){}; \node[vertex](G3) at (-4.9,-3.2){}; \node[vertex](H3) at (-6.6,-2.3){}; \node[vertex](I3) at (-7,-2.7){}; \node[vertex](J3) at (-5.4,-2.3){}; \node[vertex](K3) at (-5,-2.7){}; \node[vertex](L3) at (-7,-3.7){}; \node[vertex](M3) at (-6.6,-4.1){}; \node[vertex](N3) at (-5,-3.7){}; \node[vertex](O3) at (-5.4,-4.1){}; \node[vertex](P) at (-2,-3.2){$Q_8$}; \node[vertex](Q) at (-2,-2.2){}; \node[vertex](R) at (-2.5,-4.2){}; \node[vertex](S) at (-3,-2.2){}; \node[vertex](m) at (-3.35,-2.35){}; \node[vertex](n) at (-0.65,-2.35){}; \node[vertex](T) at (-1.5,-2.2){}; \node[vertex](U) at (-1,-2.2){}; \node[vertex](V) at (-0.5,-2.6){}; \node[vertex](W) at (-0.5,-2.9){}; \node[vertex](X) at (-0.5,-3.2){}; \node[vertex](Y) at (-0.5,-3.5){}; \node[vertex](k) at (-0.5,-3.8){}; \node[vertex](l) at (-0.6,-4.1){}; \node[vertex](Z) at (-1,-4.2){}; \node[vertex](a) at (-1.5,-4.2){}; \node[vertex](b) at (-2,-4.2){}; \node[vertex](c) at (-2.5,-2.2){}; \node[vertex](d) at (-3,-4.2){}; \node[vertex](e) at (-3.5,-2.6){}; \node[vertex](f) at (-3.5,-2.9){}; \node[vertex](g) at (-3.5,-3.2){}; \node[vertex](h) at (-3.5,-3.5){}; \node[vertex](i) at (-3.5,-3.8){}; \node[vertex](j) at (-3.35,-4.1){}; \path (A) edge (B) (C) edge (D) (C) edge (E) (C) edge (F) (C2) edge (D2) (C2) edge (E2) (C2) edge (F2) (C2) edge (G2) (C2) edge (H2) (C2) edge (I2) (C2) edge (J2) (C2) edge (K2) (C2) edge (L2) (C2) edge (M2) (C2) edge (N2) (C2) edge (O2) (G) edge (H) (G) edge (K) (G) edge (J) (G) edge (I) (G) edge (L) (G) edge (M) (G) edge (N) (G) edge (O) (G) edge (L1) (G) edge (M1) (G) edge (N1) (G) edge (O1) (C3) edge (D3) (C3) edge (E3) (C3) edge (F3) (C3) edge (G3) (C3) edge (H3) (C3) edge (I3) (C3) edge (J3) (C3) edge (K3) (C3) edge (L3) (C3) edge (M3) (C3) edge (N3) (C3) edge (O3) (P) edge (Q) (P) edge (R) (P) edge (S) (P) edge (T) (P) edge (U) (P) edge (V) (P) edge (W) (P) edge (X) (P) edge (Y) (P) edge (Z) (P) edge (a) (P) edge (b) (P) edge (c) (P) edge (d) (P) edge (e) (P) edge (f) (P) edge (g) (P) edge (h) (P) edge (i) (P) edge (j) (P) edge (k) (P) edge (l) (P) edge (m) (P) edge (n); \end{tikzpicture} \captionof{figure}{Graph structure of $\mathcal{B}(Q_8)$} \label{fig:fig2} \end{center} From Figure \ref{fig:fig2}, it is clear that $\mathcal{B}(Q_8) = K_2 \sqcup K_{1, 3} \sqcup 3K_{1, 12} \sqcup K_{1, 24}$. For the group $D_8$, we have $V(\mathcal{B}(D_8))=D_8 \times D_8 \sqcup \{H_0, H_1, \dots, H_8, D_8\}$ where $H_0=\{1\}$, $H_1=\{1, a^2\}$, $H_2=\{1, b\}$, $H_3=\{1, ab\}$, $H_4=\{1, a^2b\}$, $H_5=\{1, a^3b\}$, $H_6=\{1, a^2, b, a^2b\}$, $H_7=\{1, a^2, ab, a^3b\}$ and $H_8=\{1, a, a^2, a^3\}$. We have $\Nbd_{\mathcal{B}(D_8)}(H_0)=\{(1,1)\}$, $\Nbd_{\mathcal{B}(D_8)}(H_i)=H_i \times H_i \setminus \{(1,1)\}$ for $1 \leq i \leq 5$, $\Nbd_{\mathcal{B}(D_8)}(H_6)= H_6 \times H_6 \setminus \left(\sqcup_{j=0}^{2}\Nbd_{\mathcal{B}(D_8)}(H_j) \sqcup \Nbd_{\mathcal{B}(D_8)}(H_4)\right)$, $\Nbd_{\mathcal{B}(D_8)}(H_7)= H_7 \times H_7 \setminus \left(\sqcup_{j=0}^{1}\Nbd_{\mathcal{B}(D_8)}(H_j)\right.$ $\left. \sqcup \Nbd_{\mathcal{B}(D_8)}(H_3) \sqcup \Nbd_{\mathcal{B}(D_8)}(H_5)\right)$, $\Nbd_{\mathcal{B}(D_8)}(H_8)=H_8 \times H_8 \setminus \left(\sqcup_{j=0}^{1}\Nbd_{\mathcal{B}(D_8)}(H_j)\right)$ and $\Nbd_{\mathcal{B}(D_8)}(D_8)=D_8 \times D_8$ $ \setminus \left(\sqcup_{j=0}^{8}\Nbd_{\mathcal{B}(D_8)}(H_j)\right)$. Since the vertices from $D_8 \times D_8$ have degree one, we have the following structure of $\mathcal{B}(D_8)$. \begin{center} \begin{tikzpicture} \tikzstyle{vertex}=[circle,minimum size=0.1pt,fill=black!30,inner sep=1.5pt] \node[vertex](A) at (-10.7,0){}; \node[vertex](B) at (-10.7,-1){$H_0$}; \node[vertex](C) at (-8.7,-0.8){$H_1$}; \node[vertex](D) at (-8.7,0){}; \node[vertex](E) at (-9.3,-1.6){}; \node[vertex](F) at (-8.1,-1.6){}; \node[vertex](C4) at (-6.5,-0.8){$H_2$}; \node[vertex](D4) at (-6.5,0){}; \node[vertex](E4) at (-7.1,-1.6){}; \node[vertex](F4) at (-5.9,-1.6){}; \node[vertex](C5) at (-4.2,-0.8){$H_3$}; \node[vertex](D5) at (-4.2,0){}; \node[vertex](E5) at (-4.8,-1.6){}; \node[vertex](F5) at (-3.6,-1.6){}; \node[vertex](C2) at (-2,-0.8){$H_4$}; \node[vertex](D2) at (-2,0){}; \node[vertex](E2) at (-2.6,-1.6){}; \node[vertex](F2) at (-1.4,-1.6){}; \node[vertex](G) at (0.2,-0.8){$H_5$}; \node[vertex](H) at (0.2,0){}; \node[vertex](I) at (-0.4,-1.6){}; \node[vertex](J) at (0.8,-1.6){}; \node[vertex](a1) at (-9.8,-3.2){$H_6$}; \node[vertex](b1) at (-9.8,-2.2){}; \node[vertex](c1) at (-9.8,-4.2){}; \node[vertex](d1) at (-10.9,-2.7){}; \node[vertex](e1) at (-10.9,-3.7){}; \node[vertex](f1) at (-8.7,-2.7){}; \node[vertex](g1) at (-8.7,-3.7){}; \node[vertex](a2) at (-6.8,-3.2){$H_7$}; \node[vertex](b2) at (-6.8,-2.2){}; \node[vertex](c2) at (-6.8,-4.2){}; \node[vertex](d2) at (-7.9,-2.7){}; \node[vertex](e2) at (-7.9,-3.7){}; \node[vertex](f2) at (-5.7,-2.7){}; \node[vertex](g2) at (-5.7,-3.7){}; \node[vertex](C3) at (-3.8,-3.2){$H_8$}; \node[vertex](D3) at (-3.8,-2.2){}; \node[vertex](E3) at (-3.8,-4.2){}; \node[vertex](F3) at (-4.9,-3.2){}; \node[vertex](G3) at (-2.7,-3.2){}; \node[vertex](H3) at (-4.4,-2.3){}; \node[vertex](I3) at (-4.8,-2.7){}; \node[vertex](J3) at (-3.2,-2.3){}; \node[vertex](K3) at (-2.8,-2.7){}; \node[vertex](L3) at (-4.8,-3.7){}; \node[vertex](M3) at (-4.4,-4.1){}; \node[vertex](N3) at (-2.8,-3.7){}; \node[vertex](O3) at (-3.2,-4.1){}; \node[vertex](P) at (-0.6,-3.2){$D_8$}; \node[vertex](Q) at (-0.6,-2.2){}; \node[vertex](R) at (-1.1,-4.2){}; \node[vertex](S) at (-1.6,-2.2){}; \node[vertex](m) at (-1.95,-2.35){}; \node[vertex](n) at (0.75,-2.35){}; \node[vertex](T) at (-0.1,-2.2){}; \node[vertex](U) at (0.4,-2.2){}; \node[vertex](V) at (0.9,-2.6){}; \node[vertex](W) at (0.9,-2.9){}; \node[vertex](X) at (0.9,-3.2){}; \node[vertex](Y) at (0.9,-3.5){}; \node[vertex](k) at (0.9,-3.8){}; \node[vertex](l) at (0.9,-4.1){}; \node[vertex](Z) at (0.4,-4.2){}; \node[vertex](a) at (-0.1,-4.2){}; \node[vertex](b) at (-0.6,-4.2){}; \node[vertex](c) at (-1.1,-2.2){}; \node[vertex](d) at (-1.6,-4.2){}; \node[vertex](e) at (-2.1,-2.6){}; \node[vertex](f) at (-2.1,-2.9){}; \node[vertex](g) at (-2.1,-3.2){}; \node[vertex](h) at (-2.1,-3.5){}; \node[vertex](i) at (-2.1,-3.8){}; \node[vertex](j) at (-1.95,-4.1){}; \path (A) edge (B) (C) edge (D) (C) edge (E) (C) edge (F) (C2) edge (D2) (C2) edge (E2) (C2) edge (F2) (C4) edge (D4) (C4) edge (E4) (C4) edge (F4) (C5) edge (D5) (C5) edge (E5) (C5) edge (F5) (G) edge (H) (G) edge (I) (G) edge (J) (a1) edge (b1) (a1) edge (c1) (a1) edge (d1) (a1) edge (e1) (a1) edge (f1) (a1) edge (g1) (a2) edge (b2) (a2) edge (c2) (a2) edge (d2) (a2) edge (e2) (a2) edge (f2) (a2) edge (g2) (C3) edge (D3) (C3) edge (E3) (C3) edge (F3) (C3) edge (G3) (C3) edge (H3) (C3) edge (I3) (C3) edge (J3) (C3) edge (K3) (C3) edge (L3) (C3) edge (M3) (C3) edge (N3) (C3) edge (O3) (P) edge (Q) (P) edge (R) (P) edge (S) (P) edge (T) (P) edge (U) (P) edge (V) (P) edge (W) (P) edge (X) (P) edge (Y) (P) edge (Z) (P) edge (a) (P) edge (b) (P) edge (c) (P) edge (d) (P) edge (e) (P) edge (f) (P) edge (g) (P) edge (h) (P) edge (i) (P) edge (j) (P) edge (k) (P) edge (l) (P) edge (m) (P) edge (n); \end{tikzpicture} \captionof{figure}{Graph structure of $\mathcal{B}(D_8)$} \label{fig:fig3} \end{center} From Figure \ref{fig:fig3}, it is clear that $\mathcal{B}(D_8) = K_2 \sqcup 5K_{1, 3} \sqcup 2K_{1, 6} \sqcup K_{1, 12} \sqcup K_{1, 24}$. Thus, it follows that $\mathcal{B}(Q_8)$ and $\mathcal{B}(D_8)$ are not isomorphic. It is worth mentioning that the commuting and non-commuting graphs of the groups $Q_8$ and $D_8$ are isomorphic. \begin{center} \begin{tikzpicture} \Vertex[x=0, y=-3, size=0.05, color=black, label=$a$, position=left]{A} \Vertex[x=1, y=-3, size=0.05, color=black, label=$a^3$, position=right]{B} \Vertex[x=0, y=-2, size=0.05, color=black, label=$a^2$, position=left]{C} \Vertex[x=0, y=-1, size=0.05, color=black, label=$ab$, position=above]{D} \Vertex[x=-1, y=-1, size=0.05, color=black, label=$a^3b$, position=above]{E} \Vertex[x=1, y=-2, size=0.05, color=black, label=1, position=right]{F} \Vertex[x=1, y=-1, size=0.05, color=black, label=$a^2b$, position=above]{G} \Vertex[x=2, y=-1, size=0.05, color=black, label=$b$, position=above]{H} \path (A) edge (B) (A) edge (C) (A) edge (F) (C) edge (B) (F) edge (B) (C) edge (F) (C) edge (E) (C) edge (D) (C) edge (G) (C) edge (H) (F) edge (E) (F) edge (D) (F) edge (G) (F) edge (H) (C) edge (E) (C) edge (D) (C) edge (G) (D) edge (E) (C) edge (E) (C) edge (D) (C) edge (G) (G) edge (H); \end{tikzpicture} \captionof{figure}{Commuting graphs of $D_8$ and $Q_8$ } \label{fig:figA} \end{center} For the group $D_{10}$, we have $V(\mathcal{B}(D_{10}))= D_{10} \times D_{10} \sqcup \{H_0, H_1, \ldots, H_6, D_{10}\}$ where $H_0=\{1\}$, $H_1=\langle b \rangle$, $H_2=\langle ab \rangle$, $H_3=\langle a^2b \rangle$, $H_4=\langle a^3b \rangle$, $H_5=\langle a^4b \rangle$ and $H_6=\langle a \rangle$. We have $\Nbd_{\mathcal{B}(D_{10})}(H_0)=\{(1,1)\}$, $\Nbd_{\mathcal{B}(D_{10})}(H_i)=H_i \times H_i \setminus \{(1,1)\}$ for $1 \leq i \leq 6$ and $\Nbd_{\mathcal{B}(D_{10})}(D_{10})=D_{10} \times D_{10} \setminus \left(\sqcup_{i=0}^{6}\Nbd_{\mathcal{B}(D_{10})}(H_i)\right)$. Since the vertices from $D_{10} \times D_{10}$ have degree one, it follows that $\mathcal{B}(D_{10}) = K_2 \sqcup 5K_{1, 3} \sqcup K_{1, 24} \sqcup K_{1, 60}$. For the group $D_{12}$, we have $V(\mathcal{B}(D_{12}))= D_{12} \times D_{12} \sqcup \{H_0, H_1, \ldots, H_{14}, D_{12}\}$ where $H_0=\{1\}$, $H_1=\langle b \rangle$, $H_2=\langle ab \rangle$, $H_3=\langle a^2b \rangle$, $H_4=\langle a^3b \rangle$, $H_5=\langle a^4b \rangle$, $H_6=\langle a^5b \rangle$, $H_7=\langle a^3 \rangle$, $H_8= \langle a^2 \rangle$, $H_9=\langle a \rangle$, $H_{10}=\{1, a^3, b, a^3b\}$, $H_{11}=\{1, a^3, ab, a^4b\}$, $H_{12}=\{1, a^3, a^2b, a^5b\}$, $H_{13}=\{1, a^2, a^4, b, a^2b, a^4b\}$ and $H_{14}=\{1, a^2, a^4, ab, a^3b, a^5b\}$. We have $\Nbd_{\mathcal{B}(D_{12})}(H_0)=\{(1,1)\}$, $\Nbd_{\mathcal{B}(D_{12})}(H_i)=H_i \times H_i \setminus \{(1,1)\}$ for $1 \leq i \leq 8$, $\Nbd_{\mathcal{B}(D_{12})}(H_9)=H_9 \times H_9 \setminus (\sqcup_{j=7}^{8}\Nbd_{\mathcal{B}(D_{12})}(H_j)$ $ \sqcup \{(1,1)\})$ and $\Nbd_{\mathcal{B}(D_{12})}(H_{10})=H_{10} \times H_{10} \setminus (\Nbd_{\mathcal{B}(D_{12})}(H_1) \sqcup \Nbd_{\mathcal{B}(D_{12})}(H_4) \sqcup \Nbd_{\mathcal{B}(D_{12})}(H_7) \sqcup \{(1,1)\})$. Now, since $H_{10} \cong H_{11} \cong H_{12}$ and $\mathcal{B}(D_{12})[\{H_{10}\} \sqcup \Nbd_{\mathcal{B}(D_{12})}(H_{10})] \cong K_{1, 6}$ so $\mathcal{B}(D_{12})[\{H_{i}\} \sqcup \Nbd_{\mathcal{B}(D_{12})}(H_i)] \cong K_{1, 6}$ for $i=11$ and $12$. Also, since $H_{13} \cong H_{14} \cong S_3$ so $\mathcal{B}(D_{12})[\{H_i\} \sqcup \Nbd_{\mathcal{B}(D_{12})}(H_i)] \cong K_{1, 18}$ for $i=13$ and $14$. Now, $\Nbd_{\mathcal{B}(D_{12})}(D_{12})=D_{12} \times D_{12} \setminus \left(\sqcup_{i=0}^{14}\Nbd_{\mathcal{B}(D_{12})}(H_i)\right)$. Since the vertices from $D_{12} \times D_{12}$ have degree one, it follows that $\mathcal{B}(D_{12}) = K_2 \sqcup 7K_{1, 3} \sqcup K_{1, 8} \sqcup K_{1, 24} \sqcup 3K_{1, 6} \sqcup 2K_{1, 18} \sqcup K_{1, 54}$. For the group $A_4$, we have $V(\mathcal{B}(A_4))=A_4 \times A_4 \sqcup \{H_0, H_1, \dots, H_8, A_4\}$ where $H_0=\{(1)\}$, $H_1=\{(1), (12)(34)\}$, $H_2=\{(1), (13)(24)\}$, $H_3=\{(1), (14)(23)\}$, $H_4=\{(1), (123)$ $, (132)\}$, $H_5=\{(1), (134), (143)\}$, $H_6=\{(1), (234), (243)\}$, $H_7=\{(1), (124), (142)\}$ and $H_8=\{(1), (12)(34), (13)(24), (14)(23)\}$. We have $\Nbd_{\mathcal{B}(A_4)}(H_0)=\{((1), (1))\}$, $\Nbd_{\mathcal{B}(A_4)}(H_i)=H_i \times H_i \setminus \{((1), (1))\}$ for $1 \leq i \leq 7$, $\Nbd_{\mathcal{B}(A_4)}(H_8)=H_8 \times H_8 \setminus \left(\sqcup_{j=0}^{3}\Nbd_{\mathcal{B}(A_4)}(H_j)\right)$ and $\Nbd_{\mathcal{B}(A_4)}(A_4)=A_4 \times A_4$ $ \setminus \left(\sqcup_{j=0}^{8}\Nbd_{\mathcal{B}(A_4)}(H_j)\right)$. Since the vertices from $A_4 \times A_4$ have degree one, it follows that $\mathcal{B}(A_4) = K_2 \sqcup 3K_{1, 3} \sqcup 4K_{1, 8} \sqcup K_{1, 6} \sqcup K_{1, 96}$. For the group $S_4$, we have $V(\mathcal{B}(S_4))= S_4 \times S_4 \sqcup \{H_0, H_1, \ldots, H_{28}, S_4\}$, where \begin{center} $H_0=\{(1)\}$, \end{center} \begin{center} $\begin{array}{llll} H_1 =\langle (12) \rangle, &\quad H_2=\langle (13) \rangle, & \quad H_3=\langle (14) \rangle, & \quad H_4=\langle (23) \rangle, \\ H_5=\langle (24) \rangle, & \quad H_6 =\langle (34) \rangle, &\quad H_7=\langle (12)(34) \rangle, &\quad H_8=\langle (13)(24) \rangle,\\ H_9=\langle (14)(23) \rangle, &\quad H_{10}=\langle (123) \rangle, &\quad H_{11}=\langle (134) \rangle, &\quad H_{12}=\langle (124) \rangle, \\ H_{13}=\langle (234) \rangle, &\quad H_{14}=\langle (1234) \rangle, &\quad H_{15}=\langle (1324) \rangle, &\quad H_{16}=\langle (1243) \rangle, \end{array}$ \end{center} \begin{center} $\begin{array}{ll} H_{17}=\langle (12), (34) \rangle, & \quad H_{18}=\langle (13), (24) \rangle,\\ H_{19}=\langle (14), (23) \rangle, & \quad H_{20}=\langle (12)(34), (13)(24) \rangle, \end{array}$ \end{center} \noindent $H_{21}=\{(1), (12), (13), (23), (123), (132)\}$, $H_{22}=\{(1), (13), (14), (34), (134), (143)\}$, \noindent $H_{23}=\{(1), (12), (14), (24), (124), (142)\}$, $H_{24}=\{(1), (23), (24), (34), (234), (242)\}$, \noindent $H_{25}=\{(1), (12), (34), (12)(34), (13)(24), (14)(23), (1423), (1324)\}$, \noindent $H_{26}=\{(1), (13), (24), (12)(34), (13)(24), (14)(23), (1234), (1432)\}$, \noindent $H_{27}=\{(1), (14), (23), (12)(34), (13)(24), (14)(23), (1342), (1243)\}$ and \noindent $H_{28}=$ $\{(1), (12)(34), (13)(24), (14)(23)$, \qquad\qquad\qquad\qquad\qquad\qquad$(123), (132), (134), (143), (234), (243), (124), (142)\}$. \noindent Note that the vertices from $S_4 \times S_4$ have degree one in $\mathcal{B}(S_4)$. We have $\Nbd_{\mathcal{B}(S_4)}(H_0)=\{((1), (1))\}$ and so $\mathcal{B}(S_4)[\{H_0\} \sqcup \Nbd_{\mathcal{B}(S_4)}(H_0)] = K_2$. For $1 \leq i \leq 9$, we have $H_i \cong \mathbb{Z}_2$ and so $\mathcal{B}(S_4)[\{H_i\} \sqcup \Nbd_{\mathcal{B}(S_4)}(H_i)] = \mathcal{B}(\mathbb{Z}_2)[\{\mathbb{Z}_2\} \sqcup \Nbd_{\mathcal{B}(\mathbb{Z}_2)}(\mathbb{Z}_2)] = K_{1, 3}$. For $10 \leq i \leq 13$, we have $H_i \cong \mathbb{Z}_3$ and so $\mathcal{B}(S_4)[\{H_i\} \sqcup \Nbd_{\mathcal{B}(S_4)}(H_i)] = \mathcal{B}(\mathbb{Z}_3)[\{\mathbb{Z}_3\} \sqcup \Nbd_{\mathcal{B}(\mathbb{Z}_3)}(\mathbb{Z}_3)] = K_{1, 8}$. For $14 \leq i \leq 16$, we have $H_i \cong \mathbb{Z}_4$ and so $\mathcal{B}(S_4)[\{H_i\} \sqcup \Nbd_{\mathcal{B}(S_4)}(H_i)]= \mathcal{B}(\mathbb{Z}_4)[\{\mathbb{Z}_4\} \sqcup \Nbd_{\mathcal{B}(\mathbb{Z}_4)}(\mathbb{Z}_4)] = K_{1, 12}$. For $17 \leq i \leq 20$, we have $H_i \cong \mathbb{Z}_2 \times \mathbb{Z}_2$ and so $\mathcal{B}(S_4)[\{H_i\} \sqcup \Nbd_{\mathcal{B}(S_4)}(H_i)]= \mathcal{B}(\mathbb{Z}_2 \times \mathbb{Z}_2)[\{\mathbb{Z}_2 \times \mathbb{Z}_2\} \sqcup \Nbd_{\mathcal{B}(\mathbb{Z}_2 \times \mathbb{Z}_2)}(\mathbb{Z}_2 \times \mathbb{Z}_2)] = K_{1, 6}$. For $21 \leq i \leq 24$, we have $H_i \cong S_3$ and so $\mathcal{B}(S_4)[\{H_i\} \sqcup \Nbd_{\mathcal{B}(S_4)}(H_i)]= \mathcal{B}(S_3)[\{S_3\} \sqcup \Nbd_{\mathcal{B}(S_3)}(S_3)] = K_{1, 18}$. For $25 \leq i \leq 27$, we have $H_i \cong D_8$ and so $\mathcal{B}(S_4)[\{H_i\} \sqcup \Nbd_{\mathcal{B}(S_4)}(H_i)]= \mathcal{B}(D_8)[\{D_8\} \sqcup \Nbd_{\mathcal{B}(D_8)}(D_8)] = K_{1, 24}$. We have $H_{28} \cong A_4$ and so $\mathcal{B}(S_4)[\{H_{28}\} \sqcup \Nbd_{\mathcal{B}(S_4)}(H_{28})]= \mathcal{B}(A_4)[\{A_4\} \sqcup \Nbd_{\mathcal{B}(A_4)}(A_4)]= K_{1, 96}$. Lastly, for the subgroup $S_4$ we have $\Nbd_{\mathcal{B}(S_4)}(S_4) = S_4 \times S_4 \setminus (\sqcup_{i=0}^{28}\Nbd_{\mathcal{B}(S_4)}(H_i))$ and $\mathcal{B}(S_4)[\{S_4\} \sqcup \Nbd_{\mathcal{B}(S_4)}(S_4)] = K_{1, 216}$ noting that $|\Nbd_{\mathcal{B}(S_4)}(S_4)| = 576 - 360 = 216$. Hence, \begin{center} $\mathcal{B}(S_4) = K_2 \sqcup 9K_{1, 3} \sqcup 4K_{1, 8} \sqcup 3K_{1, 12} \sqcup 4K_{1, 6} \sqcup 4K_{1, 18} \sqcup 3K_{1, 24} \sqcup K_{1, 96} \sqcup K_{1, 216}$. \end{center} \subsection{Realization of $\mathcal{B}(D_{2p})$ and $\mathcal{B}(D_{2p^2})$} In this section, we realize the graph structures of $\mathcal{B}(G)$ for the dihedral groups $D_{2p}$ and $D_{2p^2}$ where $p$ is a prime number. Let us begin with the group $D_{2p}$. \begin{theorem}\label{structure_of_D_2p} Let $D_{2p}=\langle a, b: a^p=b^2=1, bab=a^{-1} \rangle$ be the dihedral group of order $2p$, where $p$ is a prime. Then $ \mathcal{B}(D_{2p})=K_2 \sqcup pK_{1, 3} \sqcup K_{1, p^2-1} \sqcup K_{1, 3p(p-1)}. $ \end{theorem} \begin{proof} \textbf{Case 1.} $p=2$. We have $D_4= \langle a, b: a^2=b^2=1, bab=a^{-1} \rangle = \{1, a, b, ab \}$. The subgroups of $D_4$ are $H_1=\{1\}, H_2=\{1, a\}, H_3=\{1, b\}, H_4=\{1, ab\}$ and $H_5=D_4$. Clearly, $(1, 1)$ is the only vertex adjacent to $H_1$ and so $\deg_{\mathcal{B}(D_4)}(H_1)=1$. Therefore, the subgraph induced by $\Nbd_{\mathcal{B}(D_4)}(H_1) \sqcup \{H_1\}$ in $\mathcal{B}(D_4)$ is $K_{1, 1}=K_2$. The vertices adjacent to $H_2$ are $(1, a), (a, 1)$ and $(a, a)$. Therefore, $\deg_{\mathcal{B}(D_4)}(H_2)=3$ and so the subgraph induced by $\Nbd_{\mathcal{B}(D_4)}(H_2) \sqcup \{H_2\}$ in $\mathcal{B}(D_4)$ is $K_{1, 3}$. Similarly, $H_{3}$ is adjacent to $(1, b), (b, 1), (b, b)$ and $H_4$ is adjacent to $(1, ab), (ab, 1), (ab, ab)$. So, $\deg_{\mathcal{B}(D_4)}(H_3)=\deg_{\mathcal{B}(D_4)}(H_4)=3$ and \[ \mathcal{B}(D_4)[\Nbd_{\mathcal{B}(D_4)}(H_3) \sqcup \{H_3\}] = K_{1, 3} = \mathcal{B}(D_4)[\Nbd_{\mathcal{B}(D_4)}(H_4) \sqcup \{H_4\}]. \] Lastly, $H_5$ is adjacent to \quad $(a, b), (b, a), (a, ab), (ab, a), (b, ab)$ and $(ab , b)$. Therefore, $\deg_{\mathcal{B}(D_4)}(H_5)$ $=6$ and so the subgraph induced by $\Nbd_{\mathcal{B}(D_4)}(H_5) \sqcup \{H_5\}$ in $\mathcal{B}(D_4)$ is $K_{1, 6}$. Thus, \begin{align*} \mathcal{B}(D_4) &= \underset{H \in L(D_4)}{\sqcup}\mathcal{B}(D_4)[\Nbd_{\mathcal{B}(D_4)}(H) \sqcup \{H\}]\\ &= K_2 \sqcup 3K_{1, 3} \sqcup K_{1, 6} = K_2 \sqcup 2K_{1, 3} \sqcup K_{1, 2^2-1} \sqcup K_{1, 3\cdot2(2-1)}. \end{align*} \textbf{Case 2.} $p$ is an odd prime. The subgroups of $D_{2p}=\langle a, b: a^p=b^2=1, bab=a^{-1} \rangle=\{1, a, a^2, a^3, \ldots, a^{p-1}, b, ab,$ $a^2b, \ldots, a^{p-1}b\}$ are $H_0=\{1\}, H_1=\{1, b\}, H_2=\{1, ab\}, H_3=\{1, a^2b\}, H_4=\{1, a^3b\},$ $\ldots, H_p=\{1, a^{p-1}b\}, H_{p+1}=\{1, a, a^2, \ldots, a^{p-1}\}$ and $H_{p+2}=D_{2p}$. Clearly, $(1, 1)$ is the only vertex adjacent to $H_0$ and so $\deg_{\mathcal{B}(D_{2p})}(H_0)=1$. Therefore, the subgraph induced by $\Nbd_{\mathcal{B}(D_{2p})}(H_0) \sqcup \{H_0\}$ in $\mathcal{B}(D_{2p})$ is $K_2$. The vertices adjacent to $H_1$ are $(1, b), (b, 1), (b, b)$. Therefore, $\deg_{\mathcal{B}(D_{2p})}(H_1)=3$ and so the subgraph induced by $\Nbd_{\mathcal{B}(D_{2p})}(H_1) \sqcup \{H_1\}$ in $\mathcal{B}(D_{2p})$ is $K_{1, 3}$. Similarly, for each $i = 2, 3, \dots, p$ the subgraph induced by $\Nbd_{\mathcal{B}(D_{2p})}(H_i) \sqcup \{H_i\}$ in $\mathcal{B}(D_{2p})$ is $K_{1, 3}$. The vertices adjacent to $H_{p+1}$ are $(1, a), (1, a^2), \ldots, (1, a^{p-1}), (a, 1),$ $ (a^2, 1), \ldots,$ $(a^{p-1}, 1)$ and $(a^i, a^j)$ where $1 \leq i, j \leq p-1$. Therefore, $\deg_{\mathcal{B}(D_{2p})}(H_{p+1})=(p-1)+(p-1)+(p-1)^2=(p-1)(p+1)=p^2-1$ and so the subgraph induced by $\Nbd_{\mathcal{B}(D_{2p})}(H_{p+1}) \sqcup \{H_{p+1}\}$ in $\mathcal{B}(D_{2p})$ is $K_{1, p^2-1}$. Lastly, the vertices adjacent to $H_{p+2}$ are $(a^i, a^jb)$, where $1 \leq i \leq p-1$ and $0 \leq j \leq p-1$; $(a^jb, a^i)$, where $1 \leq i \leq p-1$ and $0 \leq j \leq p-1$ and $(a^ib, a^jb)$, where $0 \leq i\ne j \leq p-1$. Therefore, $\deg_{\mathcal{B}(D_{2p})}(H_{p+2})=p(p-1)+p(p-1)+p^2-p=3p(p-1)$ and the subgraph induced by $\Nbd_{\mathcal{B}(D_{2p})}(H_{p+2}) \sqcup \{H_{p+2}\}$ in $\mathcal{B}(D_{2p})$ is $K_{1, 3p(p-1)}$. Thus, \begin{align*} \mathcal{B}(D_{2p}) &= \underset{H \in L(D_{2p})}{\sqcup}\mathcal{B}(D_{2p})[\Nbd_{\mathcal{B}(D_{2p})}(H) \sqcup \{H\}]\\ &= K_2 \sqcup pK_{1, 3} \sqcup K_{1, p^2-1} \sqcup K_{1, 3p(p-1)}. \end{align*} This completes the proof. \end{proof} \begin{theorem}\label{structure_of_D_2p2} Let $D_{2p^2}=\langle a, b: a^{p^2}=b^2=1, bab=a^{-1} \rangle$ be the dihedral group of order $2p^2$, where $p$ is a prime. Then \[ \mathcal{B}(D_{2p^2})=K_2 \sqcup p^2K_{1, 3} \sqcup K_{1, p^2-1} \sqcup K_{1, p^4-p^2} \sqcup pK_{1, 3p(p-1)} \sqcup K_{1, 3p^2(p^2-p)}. \] \end{theorem} \begin{proof} If $p = 2$ then we have already obtained that $\mathcal{B}(D_8)= K_2 \sqcup 5K_{1, 3} \sqcup 2K_{1, 6} \sqcup K_{1, 12} \sqcup K_{1, 24}$. Therefore, we consider the case when $p$ is an odd prime. The subgroups of $D_{2p^2}= \langle a, b: a^{p^2}=b^2=1, bab=a^{-1} \rangle = \{1, a, a^2, \ldots a^{p^2-1}, b, ab, a^2b,$ $\ldots, a^{p^2-1}b\}$ are \begin{center} $I=\{1\}$; \quad $H_i=\langle a^ib \rangle=\{1, a^ib\}$ for $0 \leq i \leq p^2-1$; $K=\langle a^p \rangle=\{1, a^p, a^{2p}, \ldots, a^{(p-1)p}\}$; \quad $T=\langle a \rangle=\{1, a, a^2, \ldots, a^{p^2-1}\}$; \noindent $M_r=\langle a^p, a^rb \rangle=\{1, a^p, a^{2p}, \ldots, a^{(p-1)p}, a^{p+r}b, a^{2p+r}b, a^{3p+r}b, \ldots, a^{(p-1)p+r}b, a^rb\}$ for $0 \leq r \leq p-1$; and $G=D_{2p^2}$. \end{center} Thus, $L(D_{2p^2})=\{I, H_0, H_1, \ldots, H_{p^2-1}, K, T, M_0, M_1, M_2, \ldots, M_{p-1}, G\}$. Clearly, $(1, 1)$ is the only vertex adjacent to $I$ and so $\deg_{\mathcal{B}(D_{2p^2})}(I)=1$. Therefore, $\mathcal{B}(D_{2p^2})[\Nbd_{\mathcal{B}(D_{2p^2})}(I)$ $\sqcup \{I\}] = K_2$. We have $\Nbd_{\mathcal{B}(D_{2p^2})}(H_i) = \{(1, a^ib), (a^ib, 1), (a^ib, a^ib)\}$ and so $\deg_{\mathcal{B}(D_{2p^2})}(H_i)$ $=3$ for $0 \leq i \leq p^2-1$. Therefore, $\mathcal{B}(D_{2p^2})[\Nbd_{\mathcal{B}(D_{2p^2})}(H_i) \sqcup \{H_i\}] = K_{1, 3}$ for $0 \leq i \leq p^2-1$. We have $\Nbd_{\mathcal{B}(D_{2p^2})}(K) =\{(a^{ip}, a^{jp}): 0 \leq i, j \leq p-1\} \setminus \{(1, 1)\}$ and so $\deg_{\mathcal{B}(D_{2p^2})}(K)=p^2-1$. Therefore, $\mathcal{B}(D_{2p^2})[\Nbd_{\mathcal{B}(D_{2p^2})}(K)\sqcup \{K\}] = K_{1, p^2-1}$. For the subgroup $T$, we have $\Nbd_{\mathcal{B}(D_{2p^2})}(T) = \{(a^r, a^s): \langle a^r, a^s \rangle = T\}$. We know that $\langle a^r, a^s \rangle = \langle a^{\gcd(r,s)} \rangle$ and $\langle a^l \rangle =T$ if and only if $\gcd(l, p^2)=1$. Therefore, $|\Nbd_{\mathcal{B}(D_{2p^2})}(T)| = p^4-p^2 = \deg_{\mathcal{B}(D_{2p^2})}(T)$ and so $\mathcal{B}(D_{2p^2})[\Nbd_{\mathcal{B}(D_{2p^2})}(T)\sqcup \{T\}] = K_{1, p^4-p^2}$. Finally, for the subgroups $M_r$, we have $\Nbd_{\mathcal{B}(D_{2p^2})}(M_r) = \{(a^{ip}b, a^{jp+r}), (a^{jp+r}, a^{ip}b), (a^{ip+r}b, a^{jp+r}) : 0 \leq i \ne j \leq p-1\}$ for $0 \leq r \leq p-1$. Therefore, $|\Nbd_{\mathcal{B}(D_{2p^2})}(M_r)| = 2p(p-1)+p^2-p=3p(p-1) = \deg_{\mathcal{B}(D_{2p^2})}(M_r)$ and so $\mathcal{B}(D_{2p^2})[\Nbd_{\mathcal{B}(D_{2p^2})}(M_r)\sqcup \{M_r\}] = K_{1, 3p(p-1)}$ for $0 \leq r \leq p-1$. By Lemma \ref{deg_sum=num_of_edges}, we get $\deg_{\mathcal{B}(D_{2p^2})}(D_{2p^2}) = 4p^4-(1+3p^2+p^2-1+p^4-p^2+3p^3-3p^2)=3p^2(p^2-p)$. Therefore, $\mathcal{B}(D_{2p^2})[\Nbd_{\mathcal{B}(D_{2p^2})}(G) \sqcup \{G\}] = K_{1, 3p^2(p^2-p)}$. Hence, \begin{align*} \mathcal{B}(D_{2p^2}) &= \underset{H \in L(D_{2p^2})}{\sqcup}\mathcal{B}(D_{2p^2})[\Nbd_{\mathcal{B}(D_{2p^2})}(H) \sqcup \{H\}]\\ &=K_2 \sqcup p^2K_{1, 3} \sqcup K_{1, p^2-1} \sqcup K_{1, p^4-p^2} \sqcup pK_{1, 3p(p-1)} \sqcup K_{1, 3p^2(p^2-p)}. \end{align*} This completes the proof. \end{proof} \subsection{Realization of $\mathcal{B}(Q_{4p})$ and $\mathcal{B}(Q_{4p^2})$} In this section, we realize the graph structures of $\mathcal{B}(G)$ for the dicyclic groups $Q_{4p}$ and $Q_{4p^2}$, where $p$ is a prime number. We begin with the group $Q_{4p}$. \begin{theorem} Let $Q_{4p} = \langle a, b : a^{2p} = 1, b^2 = a^p, bab^{-1} = a^{-1} \rangle$ be the dicyclic group of order $4p$, where $p$ is a prime. Then \[ \mathcal{B}(Q_{4p})=\begin{cases} K_2 \sqcup K_{1, 3} \sqcup 3K_{1, 12} \sqcup K_{1, 24}, & \text{ when } p=2 \\ K_2 \sqcup K_{1, 3} \sqcup pK_{1, 12} \sqcup K_{1, p^2-1} \sqcup K_{1, 3p^2-3} \sqcup K_{1, 12p^2-12p}, & \text{ when } p \geq 3. \end{cases} \] \end{theorem} \begin{proof} If $p=2$ then we have already obtained that $\mathcal{B}(Q_8)=K_2 \sqcup K_{1, 3} \sqcup 3K_{1, 12} \sqcup K_{1, 24}$. Therefore, we consider the case when $p$ is an odd prime. The subgroups of $Q_{4p}= \langle a, b : a^{2p} = 1, b^2 = a^p, bab^{-1} = a^{-1} \rangle$ are $I=\{1\}$, $K=\{1, a^p\}$, $T=\langle a^2 \rangle =\{a^2, a^4, \ldots, a^{2p}=1\}$, $S=\langle a \rangle =\{a, a^2, \ldots, a^{2p}=1\}$, $H_i=\langle a^ib \rangle=\{1, a^ib, b^2, a^{p+i}b\}$ for $1 \leq i \leq p$; and $G=Q_{4p}$. Thus, $L(Q_{4p})=\{I, K, T, S, H_1, H_2, \ldots, H_p,$ $ G\}$. Clearly, $(1, 1)$ is the only vertex adjacent to $I$. Therefore, $\mathcal{B}(Q_{4p})[\{I\} \sqcup \Nbd_{\mathcal{B}(Q_{4p})}(I)]$ $=K_2$. Since $K, T$ and $S$ are cyclic subgroups of order two, $p$ and $2p$ respectively, by Observation \ref{vrtex_deg_of_X_in_B(G)}(b), we have $\mathcal{B}(Q_{4p})[\{K\} \sqcup \Nbd_{\mathcal{B}(Q_{4p})}(K)]=K_{1, 3}$, $\mathcal{B}(Q_{4p})[\{T\} \sqcup \Nbd_{\mathcal{B}(Q_{4p})}(T)]=K_{1, p^2-1}$ and $\mathcal{B}(Q_{4p})[\{S\} \sqcup \Nbd_{\mathcal{B}(Q_{4p})}(S)]=K_{1, 3p^2-3}$. Also, since $H_i$'s are cyclic subgroups of order four, by Observation \ref{vrtex_deg_of_X_in_B(G)}(b), we have $\mathcal{B}(Q_{4p})[\{H_i\} \sqcup \Nbd_{\mathcal{B}(Q_{4p})}(H_i)]=K_{1, 12}$ for $1 \leq i \leq p$. By Lemma \ref{deg_sum=num_of_edges}, we get $\deg_{\mathcal{B}(Q_{4p})}(Q_{4p})=|\Nbd_{\mathcal{B}(Q_{4p})}(Q_{4p})|=16p^2-(1+3+p^2-1+3p^2-3+12p)=12p^2-12p$. Therefore, $\mathcal{B}(Q_{4p})[\{G\} \sqcup \Nbd_{\mathcal{B}(Q_{4p})}(G)]=K_{1, 12p^2-12p}$. Hence, \begin{align*} \mathcal{B}(Q_{4p})&=\underset{H \in L(Q_{4p})}{\sqcup} \mathcal{B}(Q_{4p})[\{H\} \sqcup \Nbd_{\mathcal{B}(Q_{4p})}(H)] \\ &=K_2 \sqcup K_{1, 3} \sqcup pK_{1, 12} \sqcup K_{1, p^2-1} \sqcup K_{1, 3p^2-3} \sqcup K_{1, 12p^2-12p}. \end{align*} This completes the proof. \end{proof} \begin{theorem} Let $Q_{4p^2} = \langle a, b : a^{2p^2} = 1, b^2 = a^{p^2}, bab^{-1} = a^{-1} \rangle$ be the dicyclic group of order $4p^2$, where $p$ is a prime. Then \[ \mathcal{B}(Q_{4p^2})=\begin{cases} K_2 \sqcup K_{1, 3} \sqcup 5K_{1, 12} \sqcup 2K_{1, 24} \sqcup K_{1, 48} \sqcup K_{1, 96}, & \text{ when } p=2 \\ K_2 \sqcup K_{1, 3} \sqcup p^2K_{1, 12} \sqcup K_{1, p^2-1} \sqcup K_{1, 3p^2-3} \sqcup K_{1, 3p^4-3p^2} \\ \qquad \qquad \sqcup (p-1)K_{1, 12p^2-12p} \sqcup K_{1, 13p^4-12p^3+11p^2-12p}, & \text{ when } p \geq 3. \end{cases} \] \end{theorem} \begin{proof} \textbf{Case 1.} $p=2$. We have $Q_{16}=\{1, a, a^2, \ldots, a^7, b, ab, a^2b, \ldots, a^7b\}$. The subgroups of $Q_{16}$ are \begin{center} $I=\{1\}$; \quad $J=\langle a^4 \rangle =\{1, a^4\}$; \quad $K=\langle a^2 \rangle=\{1, a^2, a^4, a^6\}$; $L=\langle a \rangle=\{1, a, a^2, \ldots, a^7\}$; \quad $H_i=\langle a^ib \rangle =\{1, a^ib, a^4, a^{p^2+i}b\}$ for $1 \leq i \leq 4$; $M_x=\langle a^2, x: (a^2)^2=x^2, (a^2)^4=1, xa^2x^{-1}=(a^2)^{-1} \rangle$ for $x=b$ and $ab$; and $G=Q_{16}$. \end{center} Thus, $L(Q_{16})=\{I, J, K, L, H_1, \ldots, H_4, M_b, M_{ab}, G\}$. Clearly, $(1, 1)$ is the only vertex adjacent to $I$. Therefore, $\mathcal{B}(Q_{16})[\{I\} \sqcup \Nbd_{\mathcal{B}(Q_{16})}(I)]=K_2$. Since $J \cong \mathbb{Z}_2$, by Observation \ref{vrtex_deg_of_X_in_B(G)}(b), $\mathcal{B}(Q_{16})[\{J\} \sqcup \Nbd_{\mathcal{B}(Q_{16})}(J)]=K_{1, 3}$. Also, for $1 \leq i \leq 4$, $K \cong H_i \cong \mathbb{Z}_4$ and so $\mathcal{B}(Q_{16})[\{K\} \sqcup \Nbd_{\mathcal{B}(Q_{16})}(K)]=\mathcal{B}(Q_{16})[\{H_i\} \sqcup \Nbd_{\mathcal{B}(Q_{16})}(H_i)]=K_{1, 12}$. Now, $\Nbd_{\mathcal{B}(Q_{16})}(L)=L \times L \setminus (\Nbd_{\mathcal{B}(Q_{16})}(I)\sqcup \Nbd_{\mathcal{B}(Q_{16})}(J)\sqcup \Nbd_{\mathcal{B}(Q_{16})}(K))$. That is, $|\Nbd_{\mathcal{B}(Q_{16})}(L)|=64-16=48$ and so $\mathcal{B}(Q_{16})[\{L\} \sqcup \Nbd_{\mathcal{B}(Q_{16})}(L)]=K_{1, 48}$. Also, $M_b \cong M_{ab} \cong Q_8$ and so $\mathcal{B}(Q_{16})[\{M_b\} \sqcup \{M_{ab}\} \sqcup \Nbd_{\mathcal{B}(Q_{16})}(M_a) \sqcup \Nbd_{\mathcal{B}(Q_{16})}(M_{ab})]=2K_{1, 24}$. By Lemma \ref{deg_sum=num_of_edges}, we get $\deg_{\mathcal{B}(Q_{16})}(Q_{16})=|\Nbd_{\mathcal{B}(Q_{16})}(Q_{16})|=256-(1+3+60+48+48)=96$. Therefore, $\mathcal{B}(Q_{16})[\{G\} \sqcup \Nbd_{\mathcal{B}(Q_{16})}(G)]=K_{1, 96}$. Hence, \begin{align*} \mathcal{B}(Q_{16})&=\underset{H \in L(Q_{16})}{\sqcup} \mathcal{B}(Q_{16})[\{H\} \sqcup \Nbd_{\mathcal{B}(Q_{16})}(H)] \\ &=K_2 \sqcup K_{1, 3} \sqcup 5K_{1, 12} \sqcup 2K_{1, 24} \sqcup K_{1, 48} \sqcup K_{1, 96}. \end{align*} \textbf{Case 2.} $p$ is an odd prime. The subgroups of $Q_{4p^2}=\{1, a, a^2, \ldots, a^{2p^2-1}, b, ab, a^2b, \ldots, a^{2p^2-1}b\}$ are \begin{center} $I=\{1\}$; \quad $J=\{1, a^{p^2}\}$; \quad $K=\langle a^{2p} \rangle=\{a^{2p}, a^{4p}, \ldots, (a^{2p})^p=1\}$; $L=\langle a^p \rangle=\{a^p, a^{2p}, \ldots, (a^p)^{2p}=1\}$; \quad $T=\langle a \rangle=\{a, a^2, \ldots, a^{2p^2}=1\}$; $H_i=\langle a^ib \rangle=\{a^ib, a^{p^2}, a^{p^2+i}b, 1\}$ for $1 \leq i \leq p^2$; $M_{x_i}=\langle (a^p)^i, x: ((a^p)^i)^p=x^2, ((a^p)^i)^{2p}=1, x(a^p)^ix^{-1}=((a^p)^i)^{-1}\rangle$, where $x=b$ and $ab$, $i < p$ and $\gcd(i, 2p)=1$; and $G=Q_{4p^2}$. \end{center} Thus, \quad $L(Q_{4p^2}) \quad =\{I, J, K, L, T, H_1, H_2, \ldots, H_{p^2}, M_{b_1}, M_{b_3} \ldots, M_{b_{p-2}}, M_{ab_1}, M_{ab_3}, \ldots,$ \noindent $ M_{ab_{p-2}}, G\}$. Clearly, $(1, 1)$ is the only vertex adjacent to $I$. Therefore, $\mathcal{B}(Q_{4p^2})[\{I\} \sqcup \Nbd_{\mathcal{B}(Q_{4p^2})}(I)]=K_2$. Since $J \cong \mathbb{Z}_2$, $K \cong \mathbb{Z}_p$, $L \cong \mathbb{Z}_{2p}$, $T \cong \mathbb{Z}_{2p^2}$ and $H_i \cong \mathbb{Z}_4$ for $1 \leq i \leq p^2$, by Observation \ref{vrtex_deg_of_X_in_B(G)}(b), we have \begin{center} $\mathcal{B}(Q_{4p^2})[\{J\} \sqcup \Nbd_{\mathcal{B}(Q_{4p^2})}(J)]=K_{1, 3}$; \quad $\mathcal{B}(Q_{4p^2})[\{K\} \sqcup \Nbd_{\mathcal{B}(Q_{4p^2})}(K)]=K_{1, p^2-1}$; \quad $\mathcal{B}(Q_{4p^2})[\{L\} \sqcup \Nbd_{\mathcal{B}(Q_{4p^2})}(L)]=K_{1, 3p^2-3}$; \quad $\mathcal{B}(Q_{4p^2})[\{T\} \sqcup \Nbd_{\mathcal{B}(Q_{4p^2})}(T)]=K_{1, 3p^4-3p^2}$; \quad and $\mathcal{B}(Q_{4p^2})[\{H_i\} \sqcup \Nbd_{\mathcal{B}(Q_{4p^2})}(H_i)]=K_{1, 12}$ for $1 \leq i \leq p^2$. \end{center} Also, for odd $i$, $1 \leq i \leq p-2$, we have $M_{b_i} \cong M_{ab_i} \cong Q_{4p}$. As such, $\mathcal{B}(Q_{4p^2})[\{M_{b_i}\} \sqcup \Nbd_{\mathcal{B}(Q_{4p^2})}(M_{b_i})]=K_{1, 12p^2-12p}=\mathcal{B}(Q_{4p^2})[\{M_{ab_i}\} \sqcup \Nbd_{\mathcal{B}(Q_{4p^2})}(M_{ab_i})]$ for odd $i$, $1 \leq i \leq p-2$. By Lemma \ref{deg_sum=num_of_edges}, we get $\deg_{\mathcal{B}(Q_{4p^2})}(Q_{4p^2})=|\Nbd_{\mathcal{B}(Q_{4p^2})}(Q_{4p^2})|=16p^4-(1+3+p^2-1+3p^2-3+3p^4-3p^2+12p^2+12p^3-12p^2-12p^2+12p)=13p^4-12p^3+11p^2-12p$. Therefore, $\mathcal{B}(Q_{4p^2})[\{G\} \sqcup \Nbd_{\mathcal{B}(Q_{4p^2})}(G)]=K_{1, 13p^4-12p^3+11p^2-12p}$. Hence, \begin{align*} \mathcal{B}(Q_{4p^2})&=\underset{H \in L(Q_{4p^2})}{\sqcup} \mathcal{B}(Q_{4p^2})[\{H\} \sqcup \Nbd_{\mathcal{B}(Q_{4p^2})}(H)] \\ &=K_2 \sqcup K_{1, 3} \sqcup p^2K_{1, 12} \sqcup K_{1, p^2-1} \sqcup K_{1, 3p^2-3} \sqcup K_{1, 3p^4-3p^2} \\ & \qquad \qquad \qquad \sqcup (p-1)K_{1, 12p^2-12p} \sqcup K_{1, 13p^4-12p^3+11p^2-12p}. \end{align*} This completes the proof. \end{proof} It may be interesting to realize the structures of $\mathcal{B}(G)$ for the dihedral group and dicyclic group (in general), quasi-dihedral group $QD_{2^{n}} = \langle a, b : a^{2^n} = b^2 = 1, bab^{-1} = a^{-1} \rangle$ ($n\geq 3$) etc. We leave this as a problem for further research. In a separate paper we shall compute various spectra, energies and certain topological indices of $\mathcal{B}(G)$ for the groups considered in this paper. \section*{Acknowledgements} The authors would like to thank the referee for his/her valuable comments. The first author is thankful to Council of Scientific and Industrial Research for the fellowship (File No. 09/0796(16521)/2023-EMR-I). \begin{thebibliography}{00} \bibitem{AKM06} A. Abdollahi, S. Akbari and H. R. Maimani, Non-commuting graph of a group, {\it J. Algebra} \textbf{298}(2006) 468--492. \bibitem{AH2007} A. Abdollahi and A. M. Hassanabadi, Noncyclic graph of a group, {\it Comm. Algebra} \textbf{35}(2007), 2057--2081. \bibitem{az2010} A. Abdollahi and M. Zarrin, Non-nilpotent graph of a group, {\it Comm. Algebra} \textbf{38}(2010) 4390--4403. \bibitem{ahm2015} S. Akbari, F. Heydari and M. Maghasedi, The intersection graph of a group, \textit{J. Algebra Appl.} \textbf{14}(2015) Article No. 1550065. \bibitem{Arun-Koma-15} B. R. Arunkumar and R. Komala, Applications of bipartite graph in diverse fields including cloud computing, {\it Int. J. Mod. Eng. Res.} \textbf{5}(2015) 1--7. \bibitem{Ba89} L. Babai, The probability of generating the symmetric group, {\it J. Comb. Theory, Series A} \textbf{52}(1989) 148--153. \bibitem{BNN2020} P. Bhowal, D. Nongsiang and R. K. Nath, Solvable graphs of finite groups, {\it Hacettepe J. Math. Stat.} \textbf{49}(2020) 1955--1964. \bibitem{BM1977} J. A. Bondy and U. S. R. Murty, { \it Graph Theory with Applications} (Elsevier, North-Holland, New York, 1977). \bibitem{brauer1955groups} R. Brauer and K. A. Fowler, On groups of even order, { \it Ann. Math.} \textbf{62}(1995) 565--583. \bibitem{cameron2021graphs} P. J. Cameron, Graphs defined on groups, {\it Int. J. Group Theory} \textbf{11}(2022) 53--107. \bibitem{CJSN-2024} P. J. Cameron, F. E. Jannat, R. K. Nath and R. Sharafdini, A survey on conjugacy class graphs of groups, \textit{Expo. Math.} \textbf{42}(2024) Article No. 125585. \bibitem{CMMS-2022} X. Y. Chen, A. Mahmoudifar, A. R. Moghaddamfar and F. Salehzadeh. The complexity of specific commuting graphs, {\it J. Algebra Appl.} \textbf{21}(2022) Article No. 2250119. \bibitem{CH-1977} E. J. Cockayne and S. T. Hedetniemi, Towards a theory of domination in graphs, {\it Networks} \textbf{7}(1977) 247--261. \bibitem{CP69} B. Cs$\acute{\rm a}$k$\acute{\rm a}$ny and G. Poll$\acute{\rm a}$k, The graph of subgroups of a finite group, {\it Czech. Math. J.} \textbf{19}(1969) 241--247. \bibitem{DNP-13} A. K. Das, R. K. Nath and M. R. Pournaki, A survey on the estimation of commutativity in finite groups, {\it Southeast Asian Bull. Math.} \textbf{37}(2013) 161--180. \bibitem{Das-NN-2024} S. Das, D. Nongsiang and R. K. Nath, Characterization of commuting graphs of finite groups having small genus, {\it Algebra Discrete Math.} \textbf{37}(2024), 191--214. \bibitem{DR16} P. Devi and R. Rajkumar, Inclusion graph of subgroups of a group, Online, https://arxiv.org/abs/1604.08259v1. \bibitem{Di69} J. D. Dixon, The probability of generating the symmetric group, {\it Math. Zeitschrift} \textbf{110}(1969) 199--205. \bibitem{DGMW92} H. Dubose-Schmidt, M. D. Galloy and D. L. Wilson, Counting nilpotent pairs in finite groups: some conjectures, Mathematical Sciences Technical Reports (MSTR), 132, 1992. https://scholar.rose-hulman.edu/math\_mstr/132 \bibitem{ET69} P. Erd$\ddot{\rm{o}}$s and P. Tur$\acute{\rm a}$n, On some problems of a statistical group-theory. IV, {\it Acta Math. Hungarica} \textbf{19}(1968) 413--435. \bibitem{FGSV2000} J. E. Fulman, M. D. Galloy, G. J. Sherman and J. M. Vanderkam, Counting nilpotent pairs in finite groups, {\it Ars Combinatoria} \textbf{54}(2000) 161--178. \bibitem{Gutman-78} I. Gutman, The energy of a graph, {\em Ber. Math-Statist. Sekt. Forschungsz. Graz} \textbf{103} (1978) 1--22. \bibitem{Gutman-2011} I. Gutman, Hyperenergetic and Hypoenergetic Graphs, \textit{Zbornik Radova} \textbf{22}(2011) 113--135. \bibitem{Hai-Reuven-2013} D. Hai-Reuven, Non-solvable graph of a finite group and solvabilizers, https://doi.org/10.48550/arXiv.1307.2924. \bibitem{Hall36} P. Hall, The Eulerian functions of a group, {\it Q. J. Math.} \textbf{7}(1936) 134--151. \bibitem{MNJ-FA-2020} M. N. Jauhari and F. Ali, Survey on topological indices and graphs associated with a commutative ring. In {\it Journal of Physics: Conference Series}, vol 1562, 012008(9 pages), 2020. \bibitem{KQ2000} A. V. Kelarev and S. J. Quinn, A combinatorial property and power graphs of groups, {\it Contrib. General Algebra} \textbf{12}(2000) 3--6. \bibitem{KSCC-2021} A. Kumar, L. Selvaganesh, P. J. Cameron and T. T. Chelvam, Recent developments on the power graph of finite groups - a survey, AKCE Int. J. Graphs Comb. \textbf{18}(2021) 65--94. \bibitem{Lewis-2008} M. L. Lewis, An overview of graphs associated with character degrees and conjugacy class sizes in finite groups, \textit{Rocky Mountain J. Math.} \textbf{38}(2008) 175--211. \bibitem{LS95} M. W. Liebeck and A. Shalev, The probability of generating a finite simple group, {\it Geom. Dedicata} \textbf{56}(1995) 103--113. \bibitem{LS96} M. W. Liebeck and A. Shalev, Simple Groups, Probabilistic Methods, and a Conjecture of Kantor and Lubotzky, {\it J. Algebra} \textbf{184}(1996) 31--57. \bibitem{MN-2024} S. Madhumitha and S. Naduvath, Graphs on groups in terms of the order of elements: A review, {\it Discrete Math., Algorithms Appl.} \textbf{16}(2024) Article No. 2330003. \bibitem{MPS23} S. Mukherjee, K. L. Patra, B. K. Sahoo, On the minimum cut-sets of the power graph of a finite cyclic group (to appear), {\it J. Algebra Appl.} https://doi.org/10.1142/S0219498824501767. \bibitem{neumann1976problem} B. H. Neumann, A problem of Paul Erd{\"o}s on groups, { \it J. Aust. Math. Soc.} \textbf{21}(1976) 467--472. \bibitem{OWW20} S. Ou, D. Wong, Z. Wang, Diameters and automorphism groups of inclusion graphs over nilpotent groups, {\it J. Algebra Appl.} \textbf{19}(2020) Article No. 2050097. \bibitem{Pak99} I. Pak, On probability of generating a finite group, preprint available at https://www.math.ucla.edu/~pak/papers/sim.pdf \bibitem{PPS21} R. P. Panda, K. L. Patra and B. K. Sahoo, On the minimum degree of the power graph of a finite cyclic group, {\it J. Algebra Appl.} \textbf{20}(2021) Article No. 2150044. \bibitem{PSSW93} D. M. Patrick, G. J. Sherman, C. A. Sugar and E. K. Wepsic, What's the probability of generating a cyclic group?, {\it Irish Math. Soc. Bull.} \textbf{31}(1993) 22--27. \bibitem{RD14} R. Rajkumar and P. Devi, Planarity of permutability graphs of subgroups of groups, {\it J. Algebra Appl.} \textbf{13}(2014) Article No. 1350112. \bibitem{RD16} R. Rajkumar and P. Devi, Permutability graphs of subgroups of some finite non-abelian groups, {\it Discrete Math., Algorithms Appl.} \textbf{8}(2016) Article No. 1650047. \bibitem{RDG16} R. Rajkumar, P. Devi and A. Gagarin, Classification of finite groups with toroidal or projective-planar permutability graphs, \textit{Comm. Algebra} \textbf{44}(2016) 3705--3726. \bibitem{SK-2017} H. Shahsavari and B. Khosravi, On the intersection graph of a finite group, \textit{Czech. Math. J.} \textbf{67}(2017) 1145--1153. \bibitem{SN-2024} M. Sharma and R. K. Nath, Signless Laplacian energies of non-commuting graphs of finite groups and related results, {\it Discrete Math., Algorithms Appl.} https://doi.org/10.1142/S1793830924500605. \bibitem{TE-13} B. Tolue and A. Erfanian, Relative non-commuting graph of a finite group, {\it J. Algebra Appl.} \textbf{12}(2013) Article No. 1250157. \end{thebibliography} \end{document}
2412.05612v1
http://arxiv.org/abs/2412.05612v1
The buckling and clamped plate problems on differential forms
\documentclass[11pt,a4paper,twoside]{article} \usepackage{enumerate} \usepackage{amsmath} \usepackage{fix-cm} \usepackage{pb-diagram} \usepackage[english]{babel} \usepackage{amssymb,latexsym} \usepackage{hyperref} \usepackage{color} \usepackage{authblk} \textheight 25.cm \textwidth 16.5cm \voffset= -3.cm \hoffset=-2.2cm \DeclareMathOperator{\dive}{div} \DeclareMathOperator{\ric}{Ric} \DeclareMathOperator{\dist}{dist} \DeclareMathOperator{\rol}{roll} \def \lra{\longrightarrow} \def \bui#1#2{\mathrel{\mathop{\kern 0pt#1}\limits^{#2}}} \def \buil#1#2{\mathrel{\mathop{\kern 0pt#1}\limits_{#2}}} \newcommand{\mnote}[1]{\marginpar{\tiny\em #1}} \newcommand{\paa}{{\partial}} \newcommand{\HH}{{\mathbb H}} \newcommand{\CC}{{\mathbb C}} \newcommand{\DD}{{\mathbb D}} \let\<\langle \let\>\rangle \newcommand{\R}{{\mathbb R}} \newcommand{\N}{{\mathbb N}} \renewcommand{\Re}{\mathrm{Re}} \def\SU{{\rm{SU}}} \def \U{{\rm{U}}} \newcommand{\lquot}[2]{\raisebox{-0.5ex}{$#2$}\backslash\!\raisebox{0.5ex}{$#1$}} \newtheorem{example}{Examples}[section] \newtheorem{thm}{Theorem}[section] \newtheorem{lemma}[thm]{Lemma} \newtheorem{prop}[thm]{Proposition} \newtheorem{TTT}[thm]{ } \newtheorem{cor}[thm]{Corollary} \newtheorem{remark}[thm]{Remark} \newtheorem{remarks}[thm]{Remarks} \newtheorem{definition}[thm]{Definition} \newtheorem{notation}[thm]{Notation} \newtheorem{exabout:ample}[thm]{Example} \newtheorem{conjecture}[thm]{Conjecture} \newtheorem{assumption}[thm]{Assumption} \newcommand{\Remark}[1]{\begin{remark}{\rm #1}\end{remark}} \newcommand{\Remarks}[1]{\begin{remarks}{\rm #1}\end{remarks}} \newcommand{\Definition}[1]{\begin{definition}{\rm #1}\end{definition}} \newcommand{\Notation}[1]{\begin{notation}{\rm #1}\end{notation}} \newcommand{\Example}[1]{\begin{example}{\rm #1}\end{example}} \frenchspacing \parindent0cm \parskip=.5\baselineskip \sloppy \title{The buckling and clamped plate problems on differential forms} \author[1]{Fida El Chami\thanks{\texttt{[email protected]}}} \author[2]{Nicolas Ginoux\thanks{\texttt{[email protected]}}} \author[1,2]{Georges Habib\thanks{\texttt{[email protected]}}} \author[1]{Ola Makhoul\thanks{\texttt{[email protected]}}} \author[3]{Simon Raulot\thanks{\texttt{[email protected]}}} \affil[1]{\footnotesize Lebanese University, Faculty of Sciences II, Department of Mathematics, P.O. Box 90656 Fanar-Matn, Lebanon} \affil[2]{\footnotesize Universit\'e de Lorraine, CNRS, IECL, F-57000 Metz} \affil[3]{\footnotesize Univ Rouen Normandie, CNRS, Normandie Univ, LMRS UMR 6085, F-76000 Rouen, France} \begin{document} \maketitle \noindent\begin{center}\begin{tabular}{p{115mm}} \begin{small}{\bf Abstract.} We extend the buckling and clamped-plate problems to the context of differential forms on compact Riemannian manifolds with smooth boundary. We characterize their smallest eigenvalues and prove that, in the case of Euclidean domains, their spectra on forms coincide with the spectra of the corresponding problems. We obtain various estimates involving the first eigenvalues of the mentioned problems and the ones of the Hodge Laplacian with respect to Dirichlet and absolute boundary conditions on forms. These estimates generalize previous ones in the case of functions. \end{small}\\ \end{tabular}\end{center} \section{Introduction} Let $(M,g)$ be an $n$-dimensional compact Riemannian manifold with smooth boundary $\partial M$ and let $\nu$ be the inward unit vector field normal to $\partial M$. For a smooth function $f$ on $M$, we consider the following two problems \begin{equation}\label{clamped_plate_functions} \left\{ \begin{array}{lll} \Delta^2 f & = \Gamma f & \textrm{ on } M\\ f & = 0 & \textrm{ on } \partial M\\ \frac{\partial f}{\partial\nu} & = 0 & \textrm{ on } \partial M \end{array} \right. \end{equation} and \begin{equation}\label{buckling_functions} \left\{ \begin{array}{lll} \Delta^2 f &=\Lambda \Delta f &\textrm{ on } M\\ f & = 0 & \textrm{ on } \partial M\\ \frac{\partial f}{\partial\nu} & = 0 & \textrm{ on } \partial M \end{array} \right. \end{equation} called the clamped plate and the buckling problem respectively. Note that $\Delta f=- \hbox{tr}(\nabla^2f)$ is the Laplace operator of $f$ and $\Delta^2 $ its square, which is sometimes called the bi-Laplace operator. It is well known that these two problems have discrete spectra consisting of eigenvalues of finite multiplicities $$ 0< \Gamma_1\le \Gamma_2 \le \ldots \le \Gamma_k \le \ldots \longrightarrow \infty$$ and $$ 0< \Lambda_1 \le \Lambda_2 \le \ldots \le \Lambda_k \le \ldots \longrightarrow \infty,$$ where each eigenvalue is repeated according to its multiplicity. Physically, problem $(\ref{clamped_plate_functions})$ describes the vibrations of a clamped plate, whereas problem $(\ref{buckling_functions})$ describes the critical buckling load of a clamped plate subjected to a uniform compressive force around its boundary. \\ These two problems were studied by numerous authors. In 1955, Payne \cite{Payne1955} proved that, if $M$ is a planar domain, then \begin{equation*} \Lambda_1 \ge \lambda_2, \end{equation*} where $\lambda_2$ is the second eigenvalue of the Dirichlet problem on $M$ (see also \cite{Friedlander2004} for a corrected proof). In 1996, Ashbaugh and Laugesen showed, in their work in \cite{AL}, that whenever $M$ is a bounded and connected open subset of the Euclidean space $\R^n$, \begin{equation}\label{AL} \Lambda_1^2 \ge \Gamma_1 \ge \Lambda_1 \lambda_1 > \lambda_1^2\,,\end{equation} where $\lambda_1$ is the first eigenvalue of the Dirichlet problem on $M$. In \cite{ChenChengWangXia2012}, Chen, Cheng, Wang and Xia proved that, if the Ricci curvature is bounded below by $n-1$, then \begin{equation}\label{CCWX} \Gamma_1 > n\lambda_1\,\,\,\,\,\hbox{and}\,\,\,\,\, \Lambda_1>n.\end{equation} Ilias and Shouman gave in \cite{IliasShouman2019} an estimate relating the first eigenvalue $\Lambda_1$ of the buckling problem to $\mu_1$, the first nonzero eigenvalue of the Neumann problem: \begin{equation} \label{IS} \mu_1 < \Lambda_1. \end{equation} In this paper, we first generalize problems (\ref{clamped_plate_functions}) and (\ref{buckling_functions}) to the context of differential forms on the manifold $M$. We prove that each problem has a discrete spectrum consisting of a nondecreasing sequence of real eigenvalues of finite multiplicities and the corresponding eigenforms form a Hilbert basis of $L^2$-integrable $p$-forms on $M$. We also characterize the first eigenvalue of each problem (see Theorems \ref{BucKPb} and \ref{CP_Pb}).\\ In Section \ref{s:ebs}, we prove that if $(M,g)$ is a compact domain of the Euclidean space $\R^n$, the spectra without multiplicities of both problems on $p$-forms, for $p=1,\ldots,n$, and on functions on $M$ coincide (see Proposition \ref{BCP_Euclidean}). This allows, for example, to determine the first eigenvalues of both of the problems on $p$-forms for the Euclidean ball of arbitrary radius. In the same section, namely in Theorem \ref{BuckCP}, we establish a relationship between the first eigenvalues of the buckling and clamped-plate problems on an arbitrary compact Riemannian manifold $(M,g)$ with smooth boundary. In the same context, we consider the following problem, called the Dirichlet problem on forms, \begin{equation}\label{Dirichlet-forms} \left\{ \begin{array}{lll} \Delta \omega &=\lambda \omega &\textrm{ in } M\\ \omega & = 0 & \textrm{ on } \partial M\\ \end{array} \right. \end{equation} and give estimates relating the first eigenvalues of the buckling, clamped-plate and Dirichlet problems. The estimates that we obtain generalize inequalities (\ref{AL}) to differential forms on $M$. We prove as well that there exists a connection with the first eigenvalue of the absolute boundary value problem on forms \begin{equation}\label{absolute-forms} \left\{ \begin{array}{lll} \Delta \omega &=\mu \omega &\textrm{ on } M\\ \nu\lrcorner \omega & = 0 & \textrm{ on } \partial M\\ \nu \lrcorner d\omega & = 0 & \textrm{ on } \partial M \end{array} \right. \end{equation} given in Theorem \ref{BuckAbs}. This connection allows to extend (\ref{IS}) to differential forms.\\ Further in the same section, we show that, under the condition that the Weitzenb\"ock curvature operator is bounded below by a positive constant $\gamma$, the first eigenvalue $\lambda_{1,p}$ of problem (\ref{Dirichlet-forms}) is also bounded below by a quantity depending on $\gamma$, see Theorem \ref{GM_Dirichlet}. This gives new estimates of the first eigenvalues of the buckling and clamped-plate problems under the same conditions, see Corollary \ref{BCDL}.\\ Finally, we end Section \ref{s:ebs} by considering domains $M$ of the unit sphere $\mathbb{S}^n$ and derive inequalities relating the first eigenvalues of the buckling and clamped plate problems on forms of different degrees (see Theorem \ref{t:domainSn}).\\ {\bf Acknowledgments:} This collaboration is supported by the International Emerging Action ``HOPF'' (Higher Order boundary value Problems for differential Forms) of the French CNRS, which the authors would like to thank. We also acknowledge the support of Universit\'e de Rouen Normandie, Universit\'e de Lorraine, and Universit\'e Libanaise. \section{The buckling and clamped plate problems} In order to make this work self-contained, we collect here some classical and useful formulae in the study of $p$-forms on manifolds with boundary. In the following, $(M^n,g)$ denotes an $n$-dimensional compact Riemannian manifold with smooth boundary and $\nu$ the inner unit normal to $\partial M$. Then we recall from \cite[Lemma 18]{RaulotSavo2011} that for any $p$-form $\omega$ on $M$, we have \begin{eqnarray}\label{eq:iotastarnablanuomega} \iota^*(\nabla_\nu\omega)=\nu\lrcorner\,d\omega+d(\nu\lrcorner\,\omega)+S^{[p]}(\iota^*\omega) \end{eqnarray} and \begin{eqnarray}\label{eq:nuintnablanuomega} \nu\lrcorner\,\nabla_\nu\omega = \delta^{\partial M}(\iota^*\omega)-\iota^*(\delta\omega)-S^{[p-1]}(\nu\lrcorner\,\omega)+(n-1)H\nu\lrcorner\,\omega. \end{eqnarray} Here $\delta^{\partial M}$ denotes the codifferential on $\partial M$, $S^{[p]}$ is the natural extension as an endomorphism of $\Omega^{p}(\partial M)$ of the shape operator $S:=-\nabla\nu$ of the embedding $\iota$ of $\partial M$ in $M$ and $H=\frac{1}{n-1}{\rm tr\,}(S)$ is its mean curvature, see e.g. \cite[p. 624]{RaulotSavo2011}. We are especially interested in $p$-forms $\omega\in\Omega^p(M)$ which satisfy the boundary condition $\omega_{|_{\partial M}}=0$. Then, using (\ref{eq:iotastarnablanuomega}) and (\ref{eq:nuintnablanuomega}), it is not difficult to compute that, along $\partial M$, we have $\iota^*\nabla_\nu\omega=\nu\lrcorner\,d\omega$ and $\nu\lrcorner\nabla_\nu\omega=-\iota^*\delta\omega$. So it is straightforward to see that \begin{equation}\label{Zcharacterization} \left\{\begin{array}{lll} \omega_{|\partial M} & = & 0\\ \nabla_\nu\omega_{|\partial M} & = & 0 \end{array}\right. \quad\Longleftrightarrow\quad \left\{\begin{array}{lll} \iota^*\omega & = & 0\\ \nu\lrcorner\omega & = & 0\\ \iota^*\delta\omega & = & 0\\ \nu\lrcorner\,d\omega & = & 0. \end{array}\right. \end{equation} In the following, we will denote by $Z$ the vector space of smooth $p$-forms on $M$ which satisfy these boundary conditions that is \begin{equation}\label{DefZ} Z:=\left\{\omega\in\Omega^p(M)\,|\,\omega_{|\partial M} = 0\text{ and }\nabla_\nu\omega_{|\partial M} = 0\right\}. \end{equation} In this work, we will also often use integration by parts formulae which are cumbersome to write down in the general frame work of $p$-forms on manifolds with boundary. However, as we restrict our attention to elements in $Z$, it may be very useful to observe that in this context they become very simple. In fact, for any $\omega,\omega'\in\Omega^p(M)$, it holds that: \begin{eqnarray} \label{eq:laplacianG1}\int_M\langle\Delta\omega,\omega'\rangle d\mu & = & \int_M\Big(\langle d\omega,d\omega'\rangle +\langle\delta\omega,\delta\omega'\rangle\Big) d\mu+\int_{\partial M}\Big(\langle \nu\lrcorner d\omega,\iota^*\omega'\rangle-\langle\iota^*\delta\omega,\nu\lrcorner\,\omega'\rangle\Big) d\sigma \\ \label{eq:laplacianG}&\hspace{-3.5cm}=&\hspace{-2.2cm} \int_M\langle\omega,\Delta\omega'\rangle d\mu + \int_{\partial M}\Big(\langle\nu\lrcorner d\omega,\iota^*\omega'\rangle-\langle\iota^*\omega,\nu\lrcorner d\omega'\rangle+\langle\nu\lrcorner\omega, \iota^*\delta\omega'\rangle-\langle\iota^*\delta\omega, \nu\lrcorner\omega'\rangle\Big)d\sigma \end{eqnarray} and so we immediately deduce from (\ref{Zcharacterization}) that if $\omega,\omega'\in Z$ then \begin{eqnarray}\label{eq:laplacian} \int_M\langle\Delta\omega,\omega'\rangle d\mu=\int_M\Big(\langle d\omega,d\omega'\rangle+\langle\delta\omega,\delta\omega'\rangle\Big)d\mu=\int_M\langle\omega,\Delta\omega'\rangle d\mu. \end{eqnarray} Here $d\mu$ (resp. $d\sigma$) denotes the Riemmanian measure density of $(M^n,g)$ (resp. $\partial M$ endowed with the induced metric). Now by replacing $\omega$ by $\Delta \omega$ in (\ref{eq:laplacianG}), we obtain \begin{eqnarray*} \int_M\langle\Delta^2\omega,\omega'\rangle d\mu & = & \int_M\langle\Delta \omega,\Delta\omega'\rangle d\mu+\int_{\partial M}\Big(\langle\nu\lrcorner d\Delta\omega,\iota^*\omega'\rangle-\langle\iota^*\Delta\omega,\nu\lrcorner d\omega'\rangle\Big)d\sigma\\ & & +\int_{\partial M}\Big(\langle\nu\lrcorner\Delta\omega,\iota^*\delta\omega'\rangle-\langle\iota^*\delta\Delta\omega,\nu\lrcorner\omega'\rangle\Big)d\sigma \end{eqnarray*} and so if $\omega,\omega'\in Z$, we get \begin{eqnarray}\label{eq:partialintDeltacons} \int_M\langle\Delta^2\omega,\omega'\rangle d\mu=\int_M\langle\Delta \omega,\Delta\omega'\rangle d\mu. \end{eqnarray} In the following, we will also denote by $(\cdot,\cdot)_{L^2(M)}$ the $L^2$-scalar product on $\Omega^p(M)$ and $\|\cdot\|_{L^2(M)}$ its associated norm. We finally notice that the boundary conditions studied here turn out to be elliptic in the sense of Lopatinski\u{\i}-Shapiro (see \cite[Def. 1.6.1]{Sc}). This was proved in a more general setting by the first four authors (see \cite[Lemma 6.1]{EGHM}) and we restate this result in our context. \begin{lemma}\label{elliptic_problem} Let $(M^n,g)$ be a compact Riemannian manifold with smooth boundary $\partial M$ and let $\nu$ be the inward unit normal vector field to the boundary. The following boundary value problem $$ \left\{ \begin{array}{lll} \Delta^2\omega&=f&\textrm{ on }M\\ \omega & = \omega_1 & \textrm{ on }\partial M\\ \iota^*\delta\omega & =\omega_2 &\textrm{ on }\partial M\\ \nu\lrcorner d\omega & =\omega_3 & \textrm{ on }\partial M \end{array} \right. $$ for given $f\in\Omega^p(M)$, $\omega_1\in\Omega^p(M)_{|_{\partial M}}$, $\omega_2\in\Omega^{p-1}(\partial M)$, $\omega_3\in\Omega^p(\partial M)$, is elliptic in the sense of Lopatinski\u{\i}-Shapiro. \end{lemma} \subsection{The buckling problem} The buckling eigenvalue problem on differential forms is \begin{equation}\label{buckling_forms} \left\{ \begin{array}{lll} \Delta^2 \omega&=\Lambda \Delta \omega &\textrm{ on } M\\ \omega &=0&\textrm{ on }\partial M\\ \nabla_\nu\omega_{|\partial M}&=0&\textrm{ on }\partial M \end{array}\right. \end{equation} for some real constant $\Lambda$. Let us begin with the following existence result. \begin{thm}\label{BucKPb} There exists a Hilbert basis of the space of $L^2$-integrable $p$-forms on $(M^n,g)$ consisting of eigenforms solutions of the problem (\ref{buckling_forms}) associated to an unbounded and positive sequence of eigenvalues $(\Lambda_{i,p})_{i\geq 1}$. Moreover, each eigenspace has a finite multiplicity and the corresponding eigenforms are smooth. Finally, the first eigenvalue $\Lambda_{1,p}$ is characterized by \begin{equation}\label{eq:charb} \Lambda_{1,p}=\inf\left\{\frac{\|\Delta\omega\|_{L^2(M)}^2}{ \| d\omega\|_{L^2(M)}^2+\|\delta \omega \|_{L^2(M)}^2},\, \omega\in\Omega^p(M)\setminus\{0\},\,\omega_{|_{\partial M}}=0\textrm{ and }\nabla_\nu\omega_{|\partial M}=0\right\}. \end{equation} Equality holds if and only if $\omega$ is an eigenform associated to the first eigenvalue. \end{thm} {\it Proof.} The proof is classical and so we only recall the main steps. For all $p$-forms $\omega,\omega'$, the two bilinear forms \[ \left(\omega,\omega'\right)_V:=\int_M\langle\Delta\omega,\Delta\omega'\rangle d\mu\;\textrm{ and }\;\left(\omega,\omega'\right)_W:=\int_{M}\Big( \langle d\omega,d\omega'\rangle+\langle \delta\omega,\delta\omega'\rangle\Big) d\mu \] define scalar products on $Z$ whose associated norms will be denoted by $\|\cdot\|_V$ and $\|\cdot\|_W$. We will also denote by $V$ and $W$ the completions of $Z$ with respect to these norms. Then one can easily show that there exists a positive constant $C$ such that $\|\cdot\|_W\leq C\|\cdot\|_V$ on $Z$ so that there is a natural bounded linear operator $\mathcal{I}\colon V\rightarrow W$ extending the identity map on $Z$. Since by \cite{Anne89} -- see \cite{ChadrakarGittinsHabibPeyerimhoff2025} for a corrected proof -- any closed and co-closed $p$-form vanishing along $\partial M$ must vanish identically on $M$, the operator $\mathcal{I}$ is actually injective.\\ Now let $\mathcal{K}\colon V\to V$ be the linear operator defined by \[ \left(\mathcal{K}\omega,\omega'\right)_V=\left(\mathcal{I}\omega, \mathcal{I}\omega'\right)_W \] for all $(\omega,\omega')\in V^2$. By definition, the operator $\mathcal{K}$ is self-adjoint and positive-definite. On the other hand, since from standard elliptic estimates both norms $\|\cdot\|_V$ and $\|\cdot\|_{H^2(M)}$ are equivalent on $Z$, the Rellich theorem ensures that $\mathcal{I}$ is compact and so is $\mathcal{K}$. The spectral theorem for positive compact self-adjoint operators applies and yields the existence of a countable Hilbert orthonormal basis $(\omega_i)_{i\geq 1}$ of $V$ associated to a monotonously nonincreasing positive real sequence of eigenvalues of finite multiplicities $(\alpha_{i,p})_{i\geq1}$ going to $0$ such that $\mathcal{K}\omega_i=\alpha_{i,p}\omega_i$ for all $i\geq1$. Now fixing $i\geq 1$ and using the definition of $\mathcal{K}$ as well as the integration by part formula (\ref{eq:laplacian}), it can be computed that, for every $\omega\in Z$, \begin{eqnarray*}\label{eq:DeltaomegaDeltaomegaibis} \alpha_{i,p}\left(\Delta\omega_i,\Delta\omega\right)_{L^2(M)} =\left(\mathcal{K}\omega_i,\omega\right)_V =\left(\omega_i,\omega\right)_W=\left(\Delta\omega_i,\omega\right)_{L^2(M)}. \end{eqnarray*} At the same time, we also have by \eqref{eq:partialintDeltacons} that \begin{eqnarray*}\label{eq:DeltaomegaDeltaomegai} \left(\Delta\omega_i,\Delta\omega\right)_{L^2(M)}= \left(\Delta^2\omega_i,\omega\right)_{L^2(M)} \end{eqnarray*} for every $\omega\in Z$ and so the previous equality now reads as \[ \left(\Delta^2\omega_i-\Lambda_{i,p}\Delta \omega_i, \omega\right)_{L^2(M)}=0 \] where we let $\Lambda_{i,p}=\frac{1}{\alpha_{i,p}}$. It follows that $\omega_i$ is a weak solution of the eigenvalue problem (\ref{buckling_forms}) which, by ellipticity (see in Lemma \ref{elliptic_problem}), is in fact smooth. Thus the form $\omega_i$ becomes a smooth eigenform to problem \eqref{buckling_forms} associated with the eigenvalue $\Lambda_{i,p}=\frac{1}{\alpha_{i,p}}$ which is of finite multiplicity, since $\alpha_{i,p}$ is. Conversely, observe that if there exists a nontrivial solution $\omega$ to \eqref{buckling_forms} for a certain $\Lambda\in\R$, then by \eqref{eq:partialintDeltacons}, we have \[ \left(\omega,\omega'\right)_V=\Lambda\left(\omega,\omega'\right)_W \] for every $\omega'\in Z$. Note that $\Lambda>0$, since otherwise $\Delta^2\omega=0$ which from \eqref{eq:partialintDeltacons} implies that $\Delta\omega=0$ and then $\omega=0$ by \cite{Anne89} since $\omega_{|\partial M}=0$. By definition of $\mathcal{K}$, we have $\left(\omega,\omega'\right)_V=\Lambda\left(\mathcal{K}\omega,\omega'\right)_V$ for all $\omega'\in Z$ and hence in $V$, therefore $\mathcal{K}\omega=\frac{1}{\Lambda}\omega$. This shows that $\omega$ is an eigenform of $\mathcal{K}$ associated to the eigenvalue $\alpha=\frac{1}{\Lambda}$. Finally, given any eigenform $\omega$ associated to a positive eigenvalue $\Lambda$ of \eqref{buckling_forms}, we have by formula \eqref{eq:partialintDeltacons} that \[ \Lambda\int_M\langle\Delta\omega,\omega\rangle d\mu=\int_M |\Delta \omega|^2 d\mu. \] Applying \eqref{eq:laplacian} to the left-hand side of this equality ensures that \[ \Lambda_{1,p}\leq \frac{\|\Delta\omega\|_{L^2(M)}^2}{ \|d\omega\|_{L^2(M)}^2+\|\delta\omega\|_{L^2(M)}^2} \] for every such eigenform, with equality for $\omega$ associated to $\Lambda_{1,p}$. Finally, if $\omega\in V$, one may write its decomposition in the Hilbert basis $(\omega_i)_{i\geq 1}$ so that \begin{eqnarray*} \|d\omega\|_{L^2(M)}^2+\|\delta\omega\|_{L^2(M)}^2 =\left(K\omega,\omega\right)_V=\sum_{i\geq 1}\frac{1}{\Lambda_{i,p}}\left|\left(\omega,\omega_i\right)_V\right|^2 \leq \frac{1}{\Lambda_{1,p}}\sum_{i\geq 1}\left|\left(\omega,\omega_i\right)_V\right|^2=\frac{1}{\Lambda_{ 1 , p}}\|\Delta\omega\|_{ L^2(M)}^2. \end{eqnarray*} This prove the characterization \eqref{eq:charb} since $Z$ is dense in $V$. \hfill$\square$ \begin{remark}\label{SymmetryBuck} The Hodge $\star$ operator is an isometry commuting with the Laplacian and preserving the boundary conditions in the buckling problem so that $\Lambda_{i,p}=\Lambda_{i,n-p}$ for any $i\geq 1$ and $1\leq p\leq n$. \end{remark} \subsection{The clamped plate problem} The clamped plate eigenvalue problem on differential forms is \begin{equation}\label{clamped_plate_forms} \left\{ \begin{array}{lll}\Delta^2 \omega&=\Gamma \omega &\textrm{ on } M\\ \omega &=0&\textrm{ on }\partial M\\ \nabla_\nu\omega_{|\partial M}&=0&\textrm{ on }\partial M \end{array} \right. \end{equation} for some real constant $\Gamma$. As previously, we immediately get the following existence result. \begin{thm}\label{CP_Pb} There exists a Hilbert basis of the space of $L^2$-integrable $p$-forms on $(M^n,g)$ consisting of eigenforms solutions of the problem (\ref{clamped_plate_forms}) associated to an unbounded and positive sequence of eigenvalues $(\Gamma_{i,p})_{i\geq 1}$. Moreover, each eigenspace has a finite multiplicity and the corresponding eigenforms are smooth. Finally, the first eigenvalue $\Gamma_{1,p}$ is characterized by \begin{equation}\label{eq:charcp} \Gamma_{1,p}=\inf\left\{\frac{\|\Delta\omega\|_{L^2(M)}^2}{ \| \omega\|_{L^2(M)}^2},\, \omega\in\Omega^p(M)\setminus\{0\},\,\omega_{|_{\partial M}}=0\textrm{ and }\nabla_\nu\omega_{|\partial M}=0\right\}. \end{equation} Equality holds if and only if $\omega$ is an eigenform associated to the first eigenvalue. \end{thm} {\it Proof.} It is enough to take $\left(\,,\,\right)_W$ to be the $L^2(M)$-scalar product on $\Omega^p(M)$ in the proof of Theorem \ref{BucKPb} and then the proof goes the same. Note that if $\omega\in\Omega^p(M)$ is an eigenform associated to $\Gamma_{1,p}$, it follows from (\ref{eq:partialintDeltacons}) that $\Gamma_{1,p}\geq 0$. Moreover if $\Gamma_{1,p}=0$ then any associated eigenform $\omega$ has to be harmonic with $\omega_{|\partial M}=0$ and so $\omega=0$ by \cite{Anne89}. In particular, $\Gamma_{1,p}>0$. \hfill$\square$ \begin{remark} As for the buckling problem, the Hodge $\star$ operator preserves the clamped plate problem so that $\Gamma_{i,p}=\Gamma_{i,n-p}$ for any $i\geq 1$ and $1\leq p\leq n$. \end{remark} \section{Eigenvalues of the buckling and clamped plate operators}\label{s:ebs} \subsection{Eigenvalues for Euclidean domains} In this section, we completely describe the spectrum of the buckling and clamped plate problems for compact domains in the Euclidean space. More precisely, we prove the following characterization. \begin{prop}\label{BCP_Euclidean} Let $(M^n,g)$ be a compact domain in the Euclidean space $\mathbb{R}^n$. Then the spectrum of the buckling problem on $p$-forms on $(M^n,g)$ coincide with the spectrum of the buckling problem on functions that is $\Lambda_{i,p}=\Lambda_{i,0}$ for all $i \geq 1$ and $p\in\{1,\ldots,n\}$. The same holds for the clamped plate problem that is $\Gamma_{i,p}=\Gamma_{i,0}$ for all $i \geq 1$ and $p\in\{1,\ldots,n\}$. \end{prop} {\it Proof.} First recall that on $\mathbb{R}^n$, there exists for each $p\in\{1,\ldots,n\}$ a maximal number of parallel $p$-forms. Fix $p\in\{1,\ldots,n\}$ and denote by $\omega_0$ a nontrivial parallel $p$-form on $M$. Then note that for any smooth function $f$ on $M$ with $f_{|\partial M}=0$ and $\frac{\partial f}{\partial \nu}_{|\partial M}=0$, the $p$-form $\omega_f:=f \omega_0$ satisfies \begin{eqnarray}\label{fomega0BC} \omega_{f|\partial M}=0\quad\text{and}\quad \nabla_\nu\omega_{f|\partial M}=0. \end{eqnarray} On the other hand, since $\omega_0$ is parallel, we have $d\omega_f=df\wedge\omega_0$ and $\delta\omega_f=-df\lrcorner\omega_0$ and therefore $\Delta\omega_f=(\Delta f)\omega_0$. Applying twice this formula leads to \begin{eqnarray}\label{fomega0BL} \Delta^2\omega_f=\Delta\left((\Delta f)\omega_0\right)=\left(\Delta^2 f\right)\omega_0. \end{eqnarray} Now if we take $f_1$ (resp. $f_2$) to be an eigenfunction for the buckling (resp. clamped plate) problem (\ref{buckling_functions}) (resp. (\ref{clamped_plate_functions})) associated with the eigenvalue $\Lambda$ (resp. $\Gamma$), we conclude combining (\ref{fomega0BC}) and (\ref{fomega0BL}) that $\omega_{f_1}$ (resp. $\omega_{f_2}$) is a $p$-eigenform for (\ref{buckling_forms}) (resp. (\ref{clamped_plate_forms})) associated with the eigenvalue $\Lambda$ (resp. $\Gamma$). Conversely, first note that if $f\in C^\infty(M)$ is a smooth nontrivial function such that $f=\<\omega,\omega_0\>$ where $\omega$ is a smooth $p$-form and $\omega_0$ is a smooth parallel $p$-form we have \begin{eqnarray}\label{DeltaFunctionEuc} \Delta f=\<\nabla^\ast\nabla\omega,\omega_0\>=\<\Delta\omega,\omega_0\> \end{eqnarray} where the last equality follows from the Bochner formula (see (\ref{eq:weitzenboeckpforms}) below) and the fact that $M$ is Euclidean. Note also that if $\omega$ satisfies the boundary condition (\ref{Zcharacterization}) then $f$ and $\partial f/\partial\nu$ vanish on $\partial M$. Now if $\omega_1$ and $\omega_2$ denote respectively $p$-eigenforms to the problems (\ref{buckling_forms}) and (\ref{clamped_plate_forms}) associated to the eigenvalues $\Lambda$ and $\Gamma$, then there exist two parallel $p$-forms $\omega^i_0$ on $\mathbb{R}^n$ such that $f_i:=\<\omega_i,\omega^i_0\>$ are smooth nontrivial functions for $i=1,2$. Therefore we easily deduce from (\ref{DeltaFunctionEuc}) that these functions are smooth eigenfunctions of (\ref{buckling_functions}) and (\ref{clamped_plate_functions}) respectively associated to $\Lambda$ and $\Gamma$. \hfill$\square$ \begin{remarks} \noindent\begin{enumerate} \item Proposition \ref{BCP_Euclidean} gives immediately the value of the first eigenvalues of the buckling and clamped plate problems on $p$-forms for the Euclidean ball of radius $R=1/H_0$. More precisely, it follows from \cite[Sec. 1]{AL} that \begin{eqnarray*} \Lambda_{1,p}=j_{\frac{n}{2},1}^2 H_0^2\quad\text{and}\quad\Gamma_{1,p}=k_{\frac{n}{2}-1,1}^4 H_0^4, \end{eqnarray*} where $j_{\frac{n}{2},1}$ is the first positive zero of the Bessel function $J_{\frac{n}{2}}$ and $k_{\frac{n}{2}-1,1}$ is the first positive zero of $J_{\frac{n}{2}-1}I_{\frac{n}{2}}+J_{\frac{n}{2}} I_{\frac{n}{2}-1}$, with $I_\ell$ being the corresponding modified Bessel function of the first kind. \item If $(M^n,g)$ is a compact Riemannian manifold carrying a nontrivial parallel $p$-forms for a certain $p\in\{1,\ldots,n\}$, we can mimic the first part of the proof of Proposition \ref{BCP_Euclidean} to ensure that if ${\rm Spec}_{\Lambda,p}(M)$ and ${\rm Spec}_{\Gamma,p}(M)$ denote respectively the buckling and clamped plate spectra on $p$-forms for $p\in\{0,\ldots,n\}$ then we have \begin{eqnarray*} {\rm Spec}_{\Lambda,0}(M)\subset {\rm Spec}_{\Lambda,p}(M) \quad\text{and}\quad {\rm Spec}_{\Gamma,0}(M)\subset {\rm Spec}_{\Gamma,p}(M) \end{eqnarray*} \end{enumerate} \end{remarks} \subsection{General estimates} In this section, we prove general estimates between the first eigenvalues of the buckling and the clamped pate problems and other classical problems. These estimates hold on $(M^n,g)$ without any extra assumption, except being a compact Riemannian manifold with smooth boundary. The first result of this part establishes a direct link between the buckling and the clamped plate problems. \begin{thm}\label{BuckCP} On an $n$-dimensional compact Riemannian manifold $(M^n,g)$ with smooth boundary, we have $\Gamma_{1,p}<\Lambda_{1,p}^2$ for all $p\in\{0,\ldots,n\}$. \end{thm} Before giving the proof of this result, we first note that if $\omega$ is a $p$-form satisfying $\omega_{|_{\partial M}}=\nabla_\nu \omega_{|_{\partial M}}=0$ then it follows from the identity $\Delta=d\delta+\delta d$ and from an integration by parts that \begin{eqnarray*}\label{normDeltaomega} \|\Delta \omega \|^2_{L^2(M)} = \|\delta d\omega \|^2_{L^2( M)}+\|d \delta\omega \|^2_{L^2(M)} \end{eqnarray*} and from (\ref{eq:laplacian}) that \begin{equation}\label{Deltaomega_scal_omega} (\Delta \omega,\omega)_{L^2( M)} = \|d\omega \|^2_{L^2( M)}+\|\delta\omega \|^2_{L^2(M)}. \end{equation} {\it Proof.} Let $\omega$ be a smooth nonzero $p$-form on $M$ with $\omega_{|_{\partial M}}=0$ and $\nabla_\nu\omega_{|\partial M}=0$. By \eqref{Deltaomega_scal_omega} and the Cauchy-Schwarz inequality, we get \[ \|d\omega \|^2_{L^2( M)}+\|\delta\omega \|^2_{L^2( M)}\leq\| \Delta \omega\|_{L^2( M)} \| \omega\|_{L^2( M)} \] and this implies that \[ \frac{\| \Delta \omega\|_{L^2( M)}}{\|\omega\|_{L^2(M)}}\leq\frac{\| \Delta \omega\|_{L^2( M)}^2}{\|d\omega \|^2_{L^2( M)}+\|\delta\omega \|^2_{L^2( M)}}. \] Taking $\omega$ to be an eigenform of the buckling problem associated to $\Lambda_{1,p}$ gives the desired inequality. Moreover, if equality holds, then for any eigenform $\omega$ of the buckling problem associated to $\Lambda_{1,p}$, the $p$-form $\Delta\omega$ is pointwise proportional to $\omega$, therefore $\Delta\omega=\Lambda_{1,p}\omega$ on $M$ and thus $\omega$ is an eigenform of the Dirichlet problem associated to the eigenvalue $\Lambda_{1,p}$. But because $\omega$ satisfies $\nabla_\nu\omega_{|\partial M}=0$ along $\partial M$, the unique continuation theorem for elliptic second-order linear operators (see e.g. \cite[Theorem 1.4]{Salolectnotes2014} and \cite[chap. VIII]{HoermanderlinearI}) implies that $\omega=0$ on $M$. This shows that the inequality must actually be strict. \hfill$\square$ Now we consider $\lambda_{1,p}$ the smallest eigenvalue of the Hodge Laplacian of $M$ with Dirichlet boundary condition. It is well-known that there exists a Hilbert basis of the space of $L^2$-integrable $p$-forms on $(M^n,g)$ consisting of smooth eigenforms solutions of the problem \begin{equation}\label{DirichletBC} \left\{ \begin{array}{lll} \Delta \omega &=\lambda \omega &\textrm{ on } M\\ \omega &=0&\textrm{ on }\partial M \end{array} \right. \end{equation} associated to an unbounded and positive sequence of eigenvalues $(\lambda_{i,p})_{i\geq 1}$. Moreover the first eigenvalue is characterized by \[ \lambda_{1,p}=\inf\left\{\frac{\|d\omega\|_{L^2(M)}^2+\|\delta\omega\|_{L^2(M)}^2}{\|\omega\|_{L^2(M)}^2}\,,\;\omega\in\Omega^p(M)\setminus\{0\},\;\omega_{|_{\partial M}}=0\right\} \] and has to be positive because of \cite{Anne89}. \begin{remarks}\label{RemarksDHL} \noindent\begin{enumerate} \item The Hodge $\star$ operator preserves the Dirichlet problem so that $\lambda_{i,p}=\lambda_{i,n-p}$ for any $i\geq 1$ and $1\leq p\leq n$. \item If $(M^n,g)$ is a compact domain in the Euclidean space, one can show, reasoning as in the proof of Proposition \ref{BCP_Euclidean}, that the Dirichlet eigenvalues on $p$-forms do not depend on $p$ and correspond to the Laplacian Dirichlet eigenvalues on functions. \end{enumerate} \end{remarks} We are now in position to give explicit estimates between the first eigenvalues of the three previous problems, restricting our attention to $1\leq p\leq[\frac{n}{2}]$ by Hodge symmetry of these spectra. Namely we get: \begin{thm}\label{BCP_Dirichlet} On an $n$-dimensional compact Riemannian manifold $(M^n,g)$ with smooth boundary, the following inequalities hold: \begin{enumerate} \item $\Lambda_{1,p} \lambda_{1,p}< \Gamma_{1,p}$ for $0\leq p\leq n$; \item $\inf(\lambda_{1,p+1}, \lambda_{1,p-1}) \le \Lambda_{1,p}$ for $1\leq p\leq n$; \item $\lambda_{1,1}\leq \Lambda_{1,0}$. \end{enumerate} \end{thm} \begin{remarks} \noindent\begin{enumerate} \item The inequalities of Theorems \ref{BuckCP} and \ref{BCP_Dirichlet} give the analogue of some well-known results in the case of functions (see \cite{AL} for example). \item In general, it is not clear whether equality can occur in the second and third inequalities of Theorem \ref{BCP_Dirichlet}. We will see that this cannot occur for Euclidean domains. \end{enumerate} \end{remarks} {\it Proof.} To prove the first inequality, we consider $\omega$ an eigenform of the clamped plate problem \eqref{clamped_plate_forms} associated with $\Gamma_{1,p}$. Then the form $\omega$ can be considered as a test-form for the first eigenvalue of the buckling problem as well as for the first eigenvalue of the Dirichlet problem on differential forms. Therefore we first get \[ \Lambda_{1,p}\Big(\|d\omega\|^2_{L^2(M)} +\|\delta \omega\|^2_{L^2(M)}\Big) \le \|\Delta \omega\|^2_{L^2(M)}. \] which can be rewritten as \[ \Lambda_{1,p} \left(\dfrac{\|d\omega\|^2_{L^2(M)} +\|\delta \omega\|^2_{L^2(M)}}{\| \omega\|^2_{L^2(M)}}\right)\le \dfrac{\|\Delta \omega\|^2_{L^2(M)}}{\| \omega\|^2_{L^2(M)}}, \] from which the estimate follows. If the inequality is an equality, then any eigenform $\omega$ of the clamped plate problem must be an eigenform of the Dirichlet problem for the Hodge Laplacian on $M$. As above, the unique continuation theorem implies that $\omega=0$. Therefore the inequality is strict. For the second estimate, we consider $\omega$ an eigenform of the buckling eigenvalue problem (\ref{buckling_forms}) associated with the first eigenvalue $\Lambda_{1,p}$ for $1\leq p\leq [\frac{n}{2}]$. We first notice that the two differential forms $\omega_1=d\omega\in\Omega^{p+1}(M)$ and $\omega_2=\delta\omega\in\Omega^{p-1}(M)$ cannot be both trivial otherwise $\omega$ would be a harmonic $p$-form which vanishes on the boundary which is impossible by \cite{Anne89}. Moreover, it follows from (\ref{Zcharacterization}) that \begin{eqnarray*} \iota^*\omega_1=d\iota^*\omega=0,\quad \nu\lrcorner\omega_1=\nu\lrcorner d\omega=0 \end{eqnarray*} and \begin{eqnarray*} \iota^*\omega_2=\iota^*(\delta\omega)=0,\quad\nu\lrcorner\omega_2=-\delta^{\partial M}(\nu\lrcorner\omega)=0 \end{eqnarray*} which imply that both $\omega_1$ and $\omega_2$ vanish along the boundary. So one can respectively take these forms as test-forms in (\ref{DirichletBC}) leading to \[ \lambda_{1,p+1}\| d \omega\|^2_{L^2(M)} \le \|\delta d \omega\|^2_{L^2(M)} \quad \mbox{ and } \quad \lambda_{1,p-1}\| \delta \omega\|^2_{L^2(M)} \le \|d\delta \omega\|^2_{L^2(M)}. \] By adding these two inequalities, we get \[ \inf(\lambda_{1,p+1}, \lambda_{1,p-1}) \left(\| d \omega\|^2_{L^2(M)} + \| \delta \omega\|^2_{L^2(M)} \right) \le \|\delta d \omega\|^2_{L^2(M)}+\|d\delta \omega\|^2_{L^2(M)} \] and the variational characterization \eqref{eq:charb} allows to conclude. For the last one, take a smooth eigenfunction $f$ for the buckling problem (\ref{buckling_functions}) on functions associated to $\Lambda_{1,0}$ and let $\omega=df\in\Omega^1(M)$. Then as above we observe that $\omega$ vanishes on $\partial M$ so that it can be used as a test-form in the variational characterization of $\lambda_{1,1}$ leading to the inequality \begin{eqnarray*} \lambda_{1,1}\leq \frac{\int_M\big(\Delta f)^2d\mu}{\int_M |df|^2d\mu}=\Lambda_{1,0}. \end{eqnarray*} This proves the third inequality in the broad sense. \hfill$\square$ A direct consequence of Theorem \ref{BuckCP} and the first inequality in Theorem \ref{BCP_Dirichlet} is the following estimate. \begin{cor}\label{DBCP} Let $(M^n,g)$ be a compact Riemannian manifold with boundary then \begin{eqnarray}\label{DirichletBounds} \lambda_{1,p}<\sqrt{\Gamma_{1,p}}<\Lambda_{1,p} \end{eqnarray} for all $0\leq p\leq n$. \end{cor} Since for domains in Euclidean space, the eigenvalues $\lambda_{1,p}$ and $\Lambda_{1,p}$ do not depend on $p$, it follows from the previous corollary with $p=0$ that equality cannot occur in the second and third inequalities of Theorem \ref{BCP_Dirichlet}. \begin{remark} In \cite{elchamiginouxhabib19}, the Robin problem is defined as $$ \left\{ \begin{array}{lll} \Delta \omega &=\lambda \omega &\textrm{ on } M\\ \nu\lrcorner\omega &=0&\textrm{ on }\partial M\\ \nu\lrcorner d\omega & = \tau \iota^*\omega & \textrm{ on }\partial M \end{array} \right. $$ and it is proven that the first eigenvalue satisfies $\lambda_{1,p}(\tau)\leq \lambda_{1,p}$ for any parameter $\tau>0$. Hence, it follows from \eqref{DirichletBounds} that $\lambda_{1,p}(\tau)<\sqrt{\Gamma_{1,p}}<\Lambda_{1,p}$. \end{remark} \begin{remark} It is in fact not difficult to show that $\lambda_{j,p}\leq\Lambda_{j,p}$ for all $j\geq 1$ and $p=0,\ldots,n$, where the inequality is strict for $j=1$ by Corollary \ref{DBCP}. Indeed, it can be checked that the eigenvalue problem (\ref{DirichletBC}) is equivalent to the problem $$ \left\{ \begin{array}{lll} \Delta^2 \omega &=\lambda \Delta\omega &\textrm{ on } M\\ \omega &=0&\textrm{ on }\partial M\\ \Delta\omega & = 0 & \textrm{ on }\partial M \end{array} \right. $$ in such a way that the Dirichlet eigenvalues are determined by applying the min-max formula to the functional \begin{eqnarray*} \frac{\int_M|\Delta \omega|^2d\mu}{\int_M|d\omega|^2+|\delta\omega|^2d\mu} \end{eqnarray*} on subspaces of $p$-forms which are in $(H^2\cap H_0^1)(M)$. Namely, any critical point $\omega\in(H^2\cap H_0^1)(M)$ of that functional associated with a critical value $\ell$ must satisfy \[ \int_M\langle\Delta\omega,\Delta\omega'\rangle d\mu=\ell\cdot\int_M\left(\langle d\omega,d\omega'\rangle+\langle \delta\omega,\delta\omega'\rangle\right)d\mu\] for all $\omega'\in(H^2\cap H_0^1)(M)$, that is, \[\int_M\langle\Delta^2\omega,\omega'\rangle d\mu+\int_{\partial M}\left(\langle\iota^*\Delta\omega,\nu\lrcorner d\omega'\rangle-\langle\nu\lrcorner\Delta\omega,\iota^*\delta\omega'\rangle\right)d\mu=\ell\cdot\left(\int_M\Delta\omega,\omega'\rangle\right),\] which reduces to \[\int_M\langle\Delta^2\omega-\ell\Delta\omega,\omega'\rangle d\mu=\int_{\partial M}\left(\langle\nu\lrcorner\Delta\omega,\iota^*\delta\omega'\rangle-\langle\iota^*\Delta\omega,\nu\lrcorner d\omega'\rangle\right)d\mu.\] Here we have used that $\omega'$ vanishes along $\partial M$. It can be deduced in the usual way that $\Delta^2\omega-\ell\Delta\omega=0$ on $M$ as well as $\nu\lrcorner\Delta\omega=0$ and $\iota^*\Delta\omega=0$ along $\partial M$, using the fact that, for $\omega'\in H_0^1(M)$, we have $\iota^*\delta\omega'=-\nu\lrcorner\nabla_\nu\omega'$ and $\nu\lrcorner d\omega'=\iota^*\nabla_\nu\omega'$, and both the tangential and normal components of $\nabla_\nu\omega'$ can be prescribed independently. Thus $\Delta\omega=0$ along $\partial M$, which yields the above equivalent problem.\\ Comparing now the above characterization to that of $\Lambda_{j,p}$, where the same quotient is considered on the smaller space $H^2_0(M)$, allows to conclude. Mind that it cannot be deduced for $j\geq2$ whether the equality can be attained or not. \end{remark} Now for $p\in\{0,\ldots,n\}$, we consider the Hodge Laplacian with respect to the absolute boundary condition which satisfies the eigenvalue problem \begin{equation}\label{eq:Neumannpblonpforms} \left\{ \begin{array}{lll} \Delta \omega &=\mu \omega &\textrm{ on } M\\ \nu\lrcorner\omega &=0&\textrm{ on }\partial M\\ \nu\lrcorner\,d\omega & = 0&\textrm{ on }\partial M. \end{array} \right. \end{equation} We will denote by $\mu_{1,p}$ the first {\it positive} eigenvalue which is given by \begin{equation}\label{VC_Absolute} \mu_{1,p}=\inf_{\omega\in\Omega^p(M)\setminus\{0\}}\left\{\frac{\|d\omega\|_{L^2(M)}^2+\|\delta\omega\|_{L^2(M)}^2}{\|\omega\|_{L^2(M)}^2}\,,\;\nu\lrcorner\omega=0\textrm{ and }\omega\in H_A^p(M)^\perp\right\}. \end{equation} The eigenspace associated with the zero eigenvalue, if non empty, corresponds to the absolute de Rham cohomology group in degree $p$ defined by $$ \displaystyle{H_A^p(M)=\left\{\omega\in \Omega^p(M)\,|\, d\omega=\delta\omega=0\,\, \text{on}\,\, M\text{ and } \nu\lrcorner\omega=0\right\}}. $$ With these notations, it is clear that $\mu_{1,0}$ corresponds to the first nonzero eigenvalue of the Laplace operator under the Neumann boundary condition. Note also that the dual eigenvalue problem is the relative one whose zero eigenvalue reflects the relative de Rham cohomology group $$ \displaystyle{H_R^p(M)=\left\{\omega\in \Omega^p(M)\,|\, d\omega=\delta\omega=0\,\, \text{on}\,\, M\text{ and } \iota^*\omega=0\right\}} $$ and is related to the absolute boundary condition by the Hodge star operator which induces an isomorphism $H_A^p(M)\simeq H_R^{n-p}(M)$ for all $p=0,\ldots,n$. The Hodge star operator maps (\ref{eq:Neumannpblonpforms}) onto the boundary problem $$ \left\{ \begin{array}{lll} \Delta \omega &=\kappa \omega &\textrm{ on } M\\ \iota^*\omega &=0&\textrm{ on }\partial M\\ \iota^*(\delta\omega) & = 0&\textrm{ on }\partial M \end{array} \right. $$ where $\kappa$ is the corresponding eigenvalue. As usual, up to the possible eigenvalue $0$, the (monotonously nondecreasing) sequence of real eigenvalues is denoted by $(\kappa_{i,p})_{i\geq1}$, where $\kappa_{1,p}$ is the smallest positive one. It is a straightforward consequence of that correspondence via the Hodge star operator that, for all $i\geq1$ and $p\in\{0,\ldots,n\}$, the identity $\mu_{i,p}=\kappa_{i,n-p}$ holds. Moreover, $\mu_{1,n}=\kappa_{1,0}>0$ holds because of $H^0_R(M)=\{0\}$. By definition, the identity $\kappa_{1,0}=\lambda_{1,0}$ also holds. \begin{remark}\label{RemarkPolya} If $H_A^p(M)=\{0\}$, an eigenform for the Dirichlet problem (\ref{DirichletBC}) can be taken as a test-form in the variational characterization of $\mu_{1,p}$ leading to the estimate $\mu_{1,p}\leq\lambda_{1,p}$ for all $p=1,\ldots,n$. Note that this inequality does not hold in general for $p=0$. However, it is a well-known result of P\'olya \cite{Polya1952} (see also \cite{Friedlander91}) that $\mu_{1,0}<\lambda_{1,0}$ for Euclidean domains. On the other hand, for $p=0,\ldots,[\frac{n}{2}]$, Guerini and Savo proved in \cite[Theorem 2.6, b)]{GueriniSavo2004} that $\mu_{1,p}\leq\kappa_{1,p}$ for a convex domain $M$ in the Euclidean space. Even more, if $M$ is strictly convex, this inequality is strict for $0\leq p<[\frac{n}{2}]$ (see \cite[Theorem 2.6, c)]{GueriniSavo2004}). Therefore, for a convex Euclidean domain with $H^{p}_R(M)=\{0\}$ for some $p=0,\ldots,[\frac{n}{2}]$, we have \begin{eqnarray*} \mu_{1,p}\leq \kappa_{1,p}=\mu_{1,n-p}\leq\lambda_{1,n-p}=\lambda_{1,p} \end{eqnarray*} This is the case for example for strictly convex domains in $\mathbb{R}^n$ and in addition the first inequality is strict for $0\leq p<[\frac{n}{2}]$ by the above discussion. \end{remark} It turns out that the first nonzero eigenvalues of the Hodge Laplacian under the absolute boundary condition can be related with the first eigenvalue of the buckling problem, as is stated below. \begin{thm}\label{BuckAbs} On an $n$-dimensional compact Riemannian manifold $(M^n,g)$ with smooth boundary, we have \begin{eqnarray}\label{BuckAbs1} \max\left(\mu_{1,p},\mu_{1,n-p}\right)\leq\Lambda_{1,p} \end{eqnarray} for all $p=0,\ldots,n$. Moreover, equality occurs if and only if there exist $k\in\{p,n-p\}$, $\omega\in\Omega^k(M)$ and $\omega_0\in H^k_A(M)\setminus\{0\}$ satisfying the overdetermined boundary value problem \begin{equation}\label{ODP} \left\{ \begin{array}{lll} \Delta \omega &=\mu_{1,k} \omega &\textrm{ on } M\\ \omega & = \omega_0 & \textrm{ on }\partial M\\ \iota^*(\delta\omega) & =0,\,\nu\lrcorner d\omega =0 & \textrm{ on }\partial M. \end{array} \right. \end{equation} \end{thm} {\it Proof.} Let $\omega_1\in\Omega^p(M)$ be an eigenform of the buckling problem associated to the eigenvalue $\Lambda_{1,p}$. We denote by $\omega_0$ the $L^2(M)$-orthogonal projection of $\omega_1$ onto $H_A^p(M)$ and we set $\omega:=\omega_1-\omega_0\in \Omega^p(M)$. Then $d\omega=d\omega_1$ and $\delta\omega=\delta\omega_1$ on $M$, and in particular $\Delta\omega=\Delta\omega_1$, as well as $\nu\lrcorner\,\omega=0$ and $\nu\lrcorner\,d\omega=0$ along $\partial M$. For this form $\omega$, using the first equality in (\ref{eq:laplacian}) and the Cauchy-Schwarz inequality leads to \begin{eqnarray*} \left(\|d\omega\|_{L^2(M)}^2+\|\delta\omega\|_{L^2(M)}^2\right)^2 = \left(\Delta\omega,\omega\right)_{L^2(M)}^2 \leq \|\Delta\omega\|_{L^2(M)}^2 \|\omega\|_{L^2(M)}^2, \end{eqnarray*} so that \[ \frac{\|d\omega\|_{L^2(M)}^2+\|\delta\omega\|_{L^2(M)}^2}{\|\omega\|_{L^2(M)}^2}\leq\frac{\|\Delta\omega\|_{L^2(M)}^2}{\|d\omega\|_{L^2(M)}^2+\|\delta\omega\|_{L^2(M)}^2}. \] Note that $\|d\omega\|_{L^2(M)}^2+\|\delta\omega\|_{L^2(M)}^2=\|d\omega_1\|_{L^2(M)}^2+\|\delta\omega_1\|_{L^2(M)}^2>0$. Moreover since $\Delta\omega=\Delta\omega_1$ and $\omega_1$ is an eigenform associated with $\Lambda_{1,p}$, we deduce that \[ \frac{\|d\omega\|_{L^2(M)}^2+\|\delta\omega\|_{L^2(M)}^2}{\|\omega\|_{L^2(M)}^2}\leq\frac{\|\Delta\omega_1\|_{L^2(M)}^2}{\|d\omega_1\|_{L^2(M)}^2+\|\delta\omega_1\|_{L^2(M)}^2}=\Lambda_{1,p}. \] But $\nu\lrcorner\omega=0$ and $\left(\omega,\omega'\right)_{L^2(M)}=0$ for all $\omega'\in H_A^p(M)$ so that the $p$-form $\omega$ can be taken as a test-form in (\ref{VC_Absolute}) and therefore $\displaystyle\mu_{1,p}\leq \Lambda_{1,p}$. The same reasoning ensures that $\mu_{1,n-p}\leq\Lambda_{1,n-p}$ and the result follows since $\Lambda_{1,n-p}=\Lambda_{1,p}$ as noticed in Remark \ref{SymmetryBuck}. Now if equality occurs one can assume without loss of generality that $k=p$ (recall that $k\in\{p,n-p\}$ by assumption). Then the $p$-form $\omega$ is an eigenform with respect to the absolute boundary condition for the eigenvalue $\mu_{1,p}$, so that it satisfies the problem (\ref{VC_Absolute}). On the other hand, it is clear that since $\omega=\omega_1-\omega_0$ with $\omega_1$ an eigenform for the buckling problem and $\omega_0\in H^p_A(M)$ we have $\iota^*\omega=-\iota^*\omega_0$ and $\iota^*(\delta\omega)=0$ which gives the first part of the equality case. Conversely, assume that there exists $\omega\in\Omega^p(M)$ satisfying (\ref{ODP}) with $\omega_0\in H^p_A(M)\setminus\{0\}$. Now if we let $\omega_1:=\omega-\omega_0$, it is not difficult to check using (\ref{Zcharacterization}) that \begin{eqnarray*} \omega_{1|\partial M}=0\quad\text{and}\quad\nabla_\nu\omega_{1|\partial M}=0 \end{eqnarray*} so that $\omega_1$ can be taken as a test-form in the variational characterization of $\Lambda_{1,p}$. On the other hand, since $\omega_0$ is an harmonic field we have $d\omega_1=d\omega$ and $\delta\omega_1=\delta\omega$ and then \begin{eqnarray*} \Lambda_{1,p}\leq\frac{\|\Delta\omega_1\|^2_{L^2(M)}}{\|d\omega_1\|^2_{L^2(M)}+\|\delta\omega_1\|^2_{L^2(M)}}=\frac{\|\Delta\omega\|^2_{L^2(M)}}{\|d\omega\|^2_{L^2(M)}+\|\delta\omega\|^2_{L^2(M)}}=\mu_{1,p}. \end{eqnarray*} This implies that equality occurs in our eigenvalue estimate. The same holds if we replace $p$ by $n-p$. \hfill$\square$ Note that problem (\ref{ODP}) would not be elliptic without the boundary condition $\nu\lrcorner\,d\omega=0$. \begin{remark} In other words, Theorem \ref{BuckAbs} states that \begin{eqnarray}\label{BuckAbs2} \max\left(\mu_{1,p},\kappa_{1,p}\right)\leq\Lambda_{1,p} \end{eqnarray} for all $p=0,\ldots,n$. Moreover, equality occurs if and only if there exist either $\omega\in\Omega^p(M)$ and $\omega_0\in H^p_A(M)\setminus\{0\}$ satisfying (\ref{ODP}), or $\widetilde{\omega}\in\Omega^p(M)$ and $\widetilde{\omega}_0\in H^p_R(M)\setminus\{0\}$ such that \begin{equation}\label{ODP2} \left\{ \begin{array}{lll} \Delta \widetilde{\omega} &=\kappa_{1,p} \widetilde{\omega} &\textrm{ on } M\\ \widetilde{\omega} & = \widetilde{\omega}_0 & \textrm{ on }\partial M\\ \iota^*(\delta\widetilde{\omega}) & =0,\,\nu\lrcorner d\widetilde{\omega} =0 & \textrm{ on }\partial M. \end{array} \right. \end{equation} \end{remark} As a direct consequence of Theorem \ref{BuckAbs}, we get: \begin{cor} Let $(M^n,g)$ be an $n$-dimensional compact Riemannian manifold with boundary, then the following hold: \begin{enumerate} \item if $H^p_A(M)=\{0\}$ then $\mu_{1,p}<\Lambda_{1,p}$ for $p=0,\ldots,n-1$; \item if $H^p_A(M)=\{0\}$ and $H^p_R(M)=\{0\}$ then Inequality (\ref{BuckAbs1}) is strict for $p=0,\ldots,[\frac{n}{2}]$. \end{enumerate} \end{cor} Note that, when $H^p_A(M)=\{0\}$, the inequality $\mu_{1,p}<\Lambda_{1,p}$ is a straightforward consequence of Corollary \ref{DBCP} because of $\max(\mu_{1,p},\mu_{1,n-p})\leq \lambda_{1,p}$ by \cite[Prop. 5.4]{elchamiginouxhabib19}. \begin{remarks} \noindent\begin{enumerate} \item Since $H^n_A(M)=\{0\}$ it follows from Theorem \ref{BuckAbs} that $\mu_{1,n}<\Lambda_{1,n}$ which is just another formulation of the inequality $\lambda_{1,0}<\Lambda_{1,0}$. \item Let $\omega_i\in\Omega^p(M)$, $i=1,2$, be two $p$-forms on $M$ such that $\omega_{1|\partial M}=\omega_{2|\partial M}$. Then it follows directly from (\ref{eq:iotastarnablanuomega}) and (\ref{eq:nuintnablanuomega}) that: \begin{eqnarray*} \Big(\iota^*(\delta\omega_1)=\iota^*(\delta\omega_2)\quad\text{and}\quad\nu\lrcorner d\omega_1=\nu\lrcorner d\omega_2\Big)\quad\Longleftrightarrow\quad\nabla_\nu\omega_{1|\partial M}=\nabla_\nu\omega_{2|\partial M}. \end{eqnarray*} Using this characterization, it is not difficult to show that $\omega\in\Omega^p(M)$ and $\omega_0\in H^p_A(M)$ satisfy (\ref{ODP}) if and only if $$ \left\{ \begin{array}{lll} \Delta \omega &=\mu_{1,p} \omega &\textrm{ on } M\\ \omega & = \omega_0 & \textrm{ on }\partial M\\ \nabla_\nu\omega_{|\partial M} & = \nabla_\nu\omega_{0|\partial M} &\textrm{ on }\partial M. \end{array} \right. $$ Obviously, the same holds for the eigenvalue boundary problem (\ref{ODP2}) with $\widetilde{\omega}\in\Omega^p(M)$ and $\widetilde{\omega}_0\in H^p_R(M)$. \end{enumerate} \end{remarks} From Corollary \ref{DBCP}, the estimate (\ref{BuckAbs2}) for $p=0$ reads \begin{eqnarray*} \mu_{1,0}\leq\Lambda_{1,0}\quad\text{and}\quad\lambda_{1,0}<\Lambda_{1,0}. \end{eqnarray*} The first inequality was proved in \cite{IliasShouman2019} and states more precisely that $\mu_{1,0}<\Lambda_{1,0}$ on any compact Riemannian manifold with boundary. However the arguments given in the proof of \cite[Lemma 3.1]{IliasShouman2019} which establishes that the inequality is strict are not clear. Indeed if $M$ is assumed to be connected, we have $H^0_A(M)\simeq \mathbb{R}$ and so, as indicated in Theorem \ref{BuckAbs}, the equality $\mu_{1,0}=\Lambda_{1,0}$ ensures the existence of a smooth function $f$ on $M$ satisfying the overdetermined problem $$ \left\{ \begin{array}{lll} \Delta f &=\mu_{1,0} f &\textrm{ on } M\\ f & = \frac{1}{{\rm Vol}(M,g)}\int_M fd\mu & \textrm{ on }\partial M\\ \frac{\partial f}{\partial \nu} & = 0 &\textrm{ on }\partial M \end{array} \right. $$ which is a particular case of the so-called Schiffer conjecture (see for example \cite{ProvenzanoSavo2023} and the references therein). However the fact that the inequality is strict can be proved at least for Euclidean domains. \begin{cor} Let $(M^n,g)$ be a compact domain in the Euclidean space. Then $\mu_{1,0}<\Lambda_{1,0}$. \end{cor} {\it Proof.} As recall in Remark \ref{RemarkPolya}, we have $\mu_{1,0}<\lambda_{1,0}$ for any domains in the Euclidean space and so the result follows from Corollary \ref{DBCP}. \hfill$\square$ On a convex Euclidean domain with $H^{p}_R(M)=\{0\}$, since the spectrum of the Dirichlet and the buckling problems do not depend on $p$, we deduce from Remark \ref{RemarkPolya} that \begin{eqnarray*} \mu_{1,p}\leq\lambda_{1,p}=\lambda_{1,0}<\lambda_{2,0}\leq\Lambda_{1,0}=\Lambda_{1,p} \end{eqnarray*} for $p=0,\ldots,[\frac{n}{2}]$. The last inequality is due to Payne \cite{Payne1955} (with a gap in the proof filled by Friedlander \cite{Friedlander2004}). \subsection{Manifolds with Weitzenb\"ock curvature operator bounded from below} In this section, we give a lower bound for the first eigenvalue $\lambda_{1,p}$ of the Dirichlet Hodge Laplacian under the condition that the Weitzenb\"ock curvature operator of the Riemannian manifold $(M^n,g)$ is bounded from below by a positive constant $\gamma>0$. Combined with some results of the former part, we obtain estimates for the first eigenvalues of the buckling and clamped plate problems in this context. These last results generalize previous results by Chen, Cheng, Wang and Xia \cite{ChenChengWangXia2012}. For this, we first recall that the Weitzenb\"ock formula for $p$-forms states that \begin{equation}\label{eq:weitzenboeckpforms} \Delta\omega=\nabla^*\nabla\omega+W^{[p]}\omega \end{equation} for any $\omega\in\Omega^p(M)$ where $\nabla$ (resp. $\nabla^*$) is the Levi-Civita connection (resp. its $L^2$-adjoint) on forms and $W^{[p]}$ is the curvature term. This last term is usually called the Weitzenb\"ock curvature operator and it defines a self-adjoint endomorphism acting on $p$-forms. In the following we will say that $W^{[p]}$ is bounded from below by $\gamma p(n-p)\in\mathbb{R}$ for a fixed $p=1,\ldots,n$ if it satisfies \begin{equation}\label{BCOperator} \<W^{[p]}\omega,\omega\>\geq \gamma p(n-p)|\omega|^2 \end{equation} for all $\omega\in\Omega^p(M)$. For $p=1$, this is equivalent to the fact that the Ricci curvature satisfies $\mathrm{Ric}\geq (n-1)\gamma g$. From \cite{GM}, this condition is satisfied for all $p$ if the curvature operator is bounded from below by $\gamma\in\mathbb{R}$. Now integrating (\ref{eq:weitzenboeckpforms}) on $M$ leads to the so-called {\it Reilly formula} on $p$-forms \cite[Theorem 3]{RaulotSavo2011} which writes as \begin{eqnarray}\label{eq:Reillyformula} \int_M\left(|d\omega|^2+|\delta\omega|^2\right)d\mu=\int_M\left(|\nabla\omega|^2+\langle W^{[p]}\omega,\omega\rangle\right)d\mu+\int_{\partial M}\mathcal{B}(\omega,\omega)d\sigma \end{eqnarray} for all $\omega\in\Omega^p(M)$ and where \begin{eqnarray*} \mathcal{B}(\omega,\omega)=2\langle\nu\lrcorner\omega,\delta^{\partial M}(\iota^*\omega)\rangle+\langle S^{[p]}(\iota^*\omega),\iota^*\omega\rangle+(n-1)H|\nu\lrcorner\omega|^2-\langle S^{[p-1]}(\nu\lrcorner\omega),\nu\lrcorner\omega\rangle. \end{eqnarray*} As a direct consequence of this formula, we get the following estimate on $\lambda_{1,p}$. \begin{thm}\label{GM_Dirichlet} Let $(M^n,g)$ be a compact Riemannian manifold with boundary whose Weitzenb\"ock curvature operator $W^{[p]}$ is bounded from below by a positive constant $\gamma p(n-p)$ for some $1\leq p\leq [\frac{n}{2}]$. Then we have $\lambda_{1,p}>\gamma p(n-p+1)$. \end{thm} {\it Proof.} We first observe that if $\omega$ is a $p$-form on $M$ satisfying $\omega_{|_{\partial M}}=0$, the boundary term in the Reilly formula (\ref{eq:Reillyformula}) vanishes and then, from (\ref{BCOperator}), we get \begin{eqnarray*} \int_M\left(|d\omega|^2+|\delta\omega|^2\right)d\mu\geq\int_M\left(|\nabla\omega|^2+\gamma p(n-p)|\omega|^2\right)d\mu. \end{eqnarray*} On the other hand, with the help of the pointwise inequality \begin{eqnarray*} |\nabla\alpha|^2\geq\frac{1}{p+1}|d\alpha|^2+\frac{1}{n-p+1}|\delta\alpha|^2 \end{eqnarray*} which is true for any $p$-form $\alpha$ (see \cite[Lemme 6.8]{GM}), we obtain \begin{eqnarray*} \|d \omega \|^2_{L^2(M)}+\|\delta \omega \|^2_{L^2(M)}\ge \dfrac{1}{n-p+1}\big(\|d \omega \|^2_{L^2(M)}+\|\delta \omega \|^2_{L^2(M)}\big)+\gamma p(n-p)\|\omega \|^2_{L^2(M)} \end{eqnarray*} since $1\leq p\leq [\frac{n}{2}]$. Note that equality occurs if $\omega$ is a conformal Killing form (see Lemma \ref{CKFB} below for a precise definition and \cite{Semmelmann02} for more general properties) and if $n=2p$ or if $1\leq p<[\frac{n}{2}]$ and $d\omega=0$. We finally get \begin{eqnarray}\label{LaplaceBounds} \|d\omega\|^2_{L^2(M)}+\|\delta\omega \|^2_{L^2(M)} \geq \gamma p(n-p+1) \|\omega\|^2_{L^2(M)} \end{eqnarray} which, by taking $\omega$ an eigenform associated to $\lambda_{1,p}$, leads to the desired estimate in a broad sense. Assume now that equality holds so that $\omega$ is a conformal Killing $p$-form which vanishes on the boundary. Then Lemma \ref{CKFB} ensures that $\nabla_\nu\omega_{|\partial M}=0$ and this is impossible since $\omega$ is also a $p$-eigenform for the Dirichlet Hodge Laplacian. \hfill$\square$ In the previous proof, we considered conformal Killing $p$-forms $\omega$, that is $p$-forms satisfying the following equation \begin{equation}\label{CKForms} \nabla_X\omega=\frac{1}{p+1}X\lrcorner d\omega-\frac{1}{n-p+1}X^\flat\wedge\delta\omega \end{equation} for all $X\in\Gamma(TM)$ and used the following result that we prove now. \begin{lemma}\label{CKFB} On a (not necessarily compact) Riemannian manifold $(M^n,g)$ with boundary $\partial M$, a conformal Killing $p$-form with $1\leq p\leq n-1$ which vanishes on $\partial M$ satisfies $\nabla_\nu\omega_{|\partial M}=0$. \end{lemma} {\it Proof.} It is a direct consequence of (\ref{CKForms}) that \begin{eqnarray*} \iota^*(\nabla_\nu\omega)=\frac{1}{p+1}\nu\lrcorner d\omega\quad\text{and}\quad\nu\lrcorner\nabla_\nu\omega=-\frac{1}{n-p+1}\iota^*(\delta\omega). \end{eqnarray*} On the other hand, since $\omega\in\Omega^p(M)$ vanishes on $\partial M$ it is straightforward from (\ref{eq:iotastarnablanuomega}) and (\ref{eq:nuintnablanuomega}) to compute that \begin{eqnarray*} \iota^*(\nabla_\nu\omega)=\nu\lrcorner d\omega\quad\text{and}\quad\nu\lrcorner\nabla_\nu\omega=-\iota^*(\delta\omega). \end{eqnarray*} Putting these relations together imply that \begin{eqnarray*} \nu\lrcorner d\omega=0 \quad\text{and}\quad\frac{n-p}{n-p+1}\iota^*(\delta\omega)=0 \end{eqnarray*} which, with the help of (\ref{Zcharacterization}), allows to conclude. \hfill$\square$ Combining Theorem \ref{GM_Dirichlet} with (\ref{DirichletBounds}) leads to the following estimates for the buckling and clamped plate first eigenvalues. \begin{cor}\label{BCDL} Let $(M^n,g)$ be a compact Riemannian manifold with boundary whose Weitzenb\"ock curvature operator $W^{[p]}$ is bounded from below by $\gamma p(n-p)>0$ for some $1\leq p\leq [\frac{n}{2}]$. Then we have \begin{eqnarray*} \Lambda_{1,p}>\gamma p(n-p+1)\quad\text{and}\quad\Gamma_{1,p}>\gamma^2 p^2(n-p+1)^2. \end{eqnarray*} \end{cor} \begin{remarks}\label{r:knownlowerboundsbucklingclampedplate} \noindent\begin{enumerate} \item From these estimates, known lower bounds for the first eigenvalues of the buckling and clamped plate problems on functions \cite{ChenChengWangXia2012} can be deduced. Indeed, for $p=1$, the fact that $W^{[1]}$ is bounded from below by $n-1$ is equivalent to the fact that the Ricci tensor of $(M^n,g)$ satisfies $\mathrm{Ric}\geq (n-1)g$. Then the first estimate in Corollary \ref{BCDL} reads $\Lambda_{1,1}>n$ under this curvature assumption. \item Independently, the third inequality of Theorem \ref{BCP_Dirichlet}, which reads $\Lambda_{1,0}\geq \lambda_{1,1}$, combined with the inequality $\lambda_{1,1}>n$ from Theorem \ref{GM_Dirichlet}, yields $\Lambda_{1,0}>n$. This is exactly \cite[Theorem 1.6]{ChenChengWangXia2012}. Now putting together this estimate with the first inequality in Theorem \ref{BCP_Dirichlet} gives $\Gamma_{1,0}>n\lambda_{1,0}$, which is precisely \cite[Theorem 1.5]{ChenChengWangXia2012}. \end{enumerate} \end{remarks} \subsection{Domains in the unit sphere} In this section, we derive an inequality which relates the first eigenvalues of the clamped plated problem for different degrees for domains of the $n$-dimensional unit sphere $\mathbb{S}^n$. This result is a consequence of a more general estimate for submanifolds isometrically immersed in the Euclidean space $\mathbb{R}^{n+m}$ which can be obtained using \cite[Lemma 3.8]{EGHM}. However, it is in general difficult to control all the terms which appear in these estimates so that we shall restrict ourselves to the case when $M$ is a domain of $\mathbb{S}^n$. We also obtain a similar result for the first buckling eigenvalue. More precisely, we obtain: \begin{thm}\label{t:domainSn} Let $(M^n,g)$ be a compact domain in the unit sphere $\mathbb{S}^n$. \begin{enumerate} \item For $1\leq p\leq [\frac{n}{2}]$, we have \begin{equation}\label{eq:upperboundGammainSn} p \Gamma_{1,p-1}+ (n-p) \Gamma_{1,p+1}< C_{n,p} \Gamma_{1,p}, \end{equation} and \begin{equation}\label{eq:upperboundLambdainSn} \min( p\Lambda_{1,p-1}, (n-p)\Lambda_{1,p+1})< \dfrac{C_{n,p}}{2}\cdot\Lambda_{1,p}, \end{equation} where $C_{n,p}=n+\dfrac{4+2(n-2p)^2}{p(n-p+1)}+\dfrac{n(n-2p)^2}{ p^2(n-p+1)^2}$. \item For $n=2p$, we have \begin{equation}\label{eq:upperboundLambdainSnforn=2p} \Lambda_{1,\frac{n}{2}-1} < \left(1+\frac{16}{n^2(n+2)}\right)\cdot \Lambda_{1,\frac{n}{2}}. \end{equation} \end{enumerate} \end{thm} {\it Proof.} Let $\omega$ be a smooth $p$-form on $\Omega$ which vanishes on $\partial M$ and whose normal covariant derivative is also zero on the boundary. For any $1\leq i\leq n+1$, we consider the $(p-1)$-form $\partial_{x_i}^T\lrcorner\,\omega$ on $M$ where $\partial_{x_i}^T$ denotes the tangential part in $TM$ of the unit parallel vector field $\partial_{x_i}$ of $\mathbb{R}^{n+1}$. It is not difficult to check that \begin{eqnarray*} \big(\partial_{x_i}^T\lrcorner\,\omega\big)_{|\partial M}=0\quad\text{and}\quad\nabla_{\nu}\big(\partial_{x_i}^T\lrcorner\omega\big)_{|\partial M}=0 \end{eqnarray*} so that it can be used as a test-form in the variational characterizations of $\Gamma_{1,p-1}$ and $\Lambda_{1,p-1}$. \begin{enumerate} \item From (\ref{eq:charcp}), we get \[ \Gamma_{1,p-1}\int_M|\partial_{x_i}^T\lrcorner\,\omega|^2 d\mu\leq\int_M|\Delta(\partial_{x_i}^T\lrcorner\,\omega)|^2d\mu. \] Summing that inequality over $i$ and using the pointwise identity $\displaystyle{\sum_{i=1}^{n+1}|\partial_{x_i}^T\lrcorner\,\omega|^2=p|\omega|^2}$ (see e.g. \cite[Eq. (22)]{EGHM}), we obtain \[ p\Gamma_{1,p-1}\int_M|\omega|^2 d\mu\leq\int_M\sum_{i=1}^{n+1}|\Delta(\partial_{x_i}^T\lrcorner\,\omega)|^2d\mu. \] Now the right-hand side of that inequality was computed for every $p$-form $\omega$ in the right-hand side of \cite[Eq. (31) and (33)]{EGHM} and we obtain \begin{eqnarray}\label{eq:inequalitysphere} p\Gamma_{1,p-1} \|\omega\|^2_{L^2(M)} \leq 4\|\delta\omega\|^2_{L^2(M)}+p\|\Delta\omega+(2p-n)\omega\|^2_{L^2(M)}. \end{eqnarray} Since Inequality \eqref{eq:inequalitysphere} is true for any $p$-eigenform $\omega$, we can apply it to the $(n-p)$-eigenform $\star\omega$ to get $$ (n-p) \Gamma_{1,n-p-1} \|\omega\|^2_{L^2(M)}\leq 4\|d\omega\|^2_{L^2(M)}+(n-p)\|\Delta\omega+(n-2p)\omega\|^2_{L^2(M)}. $$ Summing both inequalities and using the fact that $\Gamma_{1,n-p-1}=\Gamma_{1,p+1}$ yields \begin{eqnarray*}\label{eq:inesphere} \left(p \Gamma_{1,p-1}+ (n-p) \Gamma_{1,p+1}\right)\|\omega\|^2_{L^2(M)} & \leq & n\|\Delta\omega\|^2_{L^2(M)}+n(2p-n)^2\|\omega\|^2_{L^2(M)}\nonumber\\ & & +(4+2(n-2p)^2) (\Delta\omega,\omega)_{L^2(M)}\nonumber\\ & \leq & n\|\Delta\omega\|^2_{L^2(M)}+n(2p-n)^2\|\omega\|^2_{L^2(M)}\nonumber\\ & & +(4+2(n-2p)^2) \|\Delta\omega\|_{L^2(M)}\|\omega\|_{L^2(M)}\nonumber \end{eqnarray*} where we used the Cauchy-Schwarz inequality in the last inequality. On the other hand, since $M$ is in the unit sphere, the estimate (\ref{LaplaceBounds}) holds with $\gamma=1$, and therefore we get \begin{eqnarray*} \left(p \Gamma_{1,p-1}+ (n-p) \Gamma_{1,p+1}\right)\|\omega\|^2_{L^2(M)}\leq C_{n,p}\|\Delta\omega\|^2_{L^2(M)} \end{eqnarray*} which allows to deduce \eqref{eq:upperboundGammainSn} in the broad sense by taking $\omega$ to be an eigenform associated to $\Gamma_{1,p}$. If \eqref{eq:upperboundGammainSn} were an equality, then for any clamped-plate-eigenform $\omega$ associated to $\Gamma_{1,p}$, the $p$-form $\Delta\omega$ would be pointwise proportional to $\omega$ because of the equality in the Cauchy-Schwarz inequality. But, because of $\Delta^2\omega=\Gamma_{1,p}\omega$ on $M$, we would have $\Delta\omega=\sqrt{\Gamma_{1,p}}\omega$ on $M$, therefore $\omega$ would be a Dirichlet eigenform associated to the eigenvalue $\sqrt{\Gamma_{1,p}}$. Again, because $\omega$ satisfies $\nabla_\nu\omega_{|\partial M}=0$ along $\partial M$, the unique continuation property for elliptic second-order linear operators would imply that $\omega=0$ on $M$. This would lead to a contradiction and shows that \eqref{eq:upperboundGammainSn} must be strict. \\ \\ Now, we prove \eqref{eq:upperboundLambdainSn}. Using the variational characterization (\ref{eq:charb}) of $\Lambda_{1,p-1}$ for $p=1,\ldots,n$ gives \[ \Lambda_{1,p-1}\int_{M}\Big(|d(\partial_{x_i}^T\lrcorner\omega)|^2+|\delta(\partial_{x_i}^T\lrcorner\omega)|^2\Big)d\mu\leq \int_M|\Delta(\partial_{x_i}^T\lrcorner\omega)|^2d\mu \] and summing as above on $i$ from $1$ to $n+1$ leads to \[ \Lambda_{1,p-1}\sum_{i=1}^{n+1}\int_{M}\Big(|d(\partial_{x_i}^T\lrcorner\omega)|^2+|\delta(\partial_{x_i}^T\lrcorner\omega)|^2\Big)d\mu\leq 4\|\delta\omega\|^2_{L^2(M)}+p\|\Delta\omega+(2p-n)\omega\|^2_{L^2(M)}. \] By the Cartan identity and \cite[id. (4.3)]{GueriniSavo2004} (see also \cite[id. (20)]{EGHM}), we have, for every $1\leq i\leq n+1$, \begin{eqnarray*} d(\partial_{x_i}^T\lrcorner\omega)&=&\mathcal{L}_{\partial_{x_i}^T}\omega-\partial_{x_i}^T\lrcorner\,d\omega\\ &=&\nabla_{\partial_{x_i}^T}\omega+\mathrm{I\!I}_{\partial^\perp_{x_i}}^{[p]}\omega-\partial_{x_i}^T\lrcorner\,d\omega, \end{eqnarray*} where $\mathcal{L}_X\omega$ is the Lie derivative of $\omega$ in the $X$-direction, $\mathrm{I\!I}_{\partial^\perp_{x_i}}^{[p]}$ is the natural extension onto $\Lambda^p T^*M$ of the pointwise endomorphism field $\mathrm{I\!I}_{\partial^\perp_{x_i}}$ of $TM$ defined for all $X,Y\in TM$ by $\langle \mathrm{I\!I}_{\partial^\perp_{x_i}}(X),Y\rangle=\langle\mathrm{I\!I}(X,Y),\partial^\perp_{x_i}\rangle$ and $\mathrm{I\!I}$ is the second fundamental form of $M$ in $\R^{n+1}$. Here $\partial^\perp_{x_i}$ denotes the normal component of $\partial_{x_i}$ that is, $\partial^\perp_{x_i}=\partial_{x_i}-\partial^T_{x_i}$. Note that, because $\mathrm{I\!I}_x=-g_x\otimes x$ for every $x\in\mathbb{S}^n$ and hence $x\in M$, we have $\mathrm{I\!I}_{\partial^\perp_{x_i}}=-x_i\cdot\mathrm{Id}$ at $x=(x_1,\ldots,x_{n+1})$, so that $\mathrm{I\!I}_{\partial^\perp_{x_i}}^{[p]}=-px_i\cdot\mathrm{Id}$ at $x$. It can be deduced that \begin{eqnarray*} |d(\partial_{x_i}^T\lrcorner\omega)|^2&=&\left|\nabla_{\partial_{x_i}^T}\omega+\mathrm{I\!I}_{\partial^\perp_{x_i}}^{[p]}\omega-\partial_{x_i}^T\lrcorner\,d\omega\right|^2\\ &=&\left|\nabla_{\partial_{x_i}^T}\omega\right|^2+\left|\mathrm{I\!I}_{\partial^\perp_{x_i}}^{[p]}\omega\right|^2+\left|\partial_{x_i}^T\lrcorner\,d\omega\right|^2\\ &&+2\langle\nabla_{\partial_{x_i}^T}\omega,\mathrm{I\!I}_{\partial^\perp_{x_i}}^{[p]}\omega\rangle-2\langle \nabla_{\partial_{x_i}^T}\omega,\partial_{x_i}^T\lrcorner\,d\omega\rangle-2\langle\mathrm{I\!I}_{\partial^\perp_{x_i}}^{[p]}\omega,\partial_{x_i}^T\lrcorner\,d\omega\rangle. \end{eqnarray*} Choosing any orthonormal basis $(e_j)_{1\leq j\leq n}$ of $T_x M$ for some $x\in M$, we have \begin{eqnarray*} \sum_{i=1}^n\left|\nabla_{\partial_{x_i}^T}\omega\right|^2&=&\sum_{i=1}^{n+1}\sum_{j,k=1}^n\langle\partial^T_{ x_i},e_j\rangle\langle\partial^T_{ x_i},e_k\rangle\langle\nabla_{e_j}\omega,\nabla_{e_k}\omega\rangle\\ &=&\sum_{i=1}^{n+1}\sum_{j,k=1}^n\langle\partial_{ x_i},e_j\rangle\langle\partial_{ x_i},e_k\rangle\langle\nabla_{e_j}\omega,\nabla_{e_k}\omega\rangle\\ &=&\sum_{j,k=1}^n\left(\sum_{i=1}^{n+1}\langle\partial_{ x_i},e_j\rangle\langle\partial_{ x_i},e_k\rangle\right)\cdot\langle\nabla_{e_j}\omega,\nabla_{e_k}\omega\rangle\\ &=&\sum_{j,k=1}^n\langle e_j,e_k\rangle\langle\nabla_{e_j}\omega,\nabla_{e_k}\omega\rangle\\ &=&\sum_{j=1}^n\left|\nabla_{e_j}\omega\right|^2\\ &=&\left|\nabla\omega\right|^2. \end{eqnarray*} As a second step, at every $x\in M$, because of $|x|=1$, \[\sum_{i=1}^{n+1}\left|\mathrm{I\!I}_{\partial^\perp_{x_i}}^{[p]}\omega\right|^2=p^2\sum_{i=1}^{n+1}x_i^2|\omega|^2=p^2|x|^2|\omega|^2=p^2|\omega|^2.\] By \cite[eq. (22)]{EGHM}, \[\sum_{i=1}^{n+1}\left|\partial_{x_i}^T\lrcorner\,d\omega\right|^2=(p+1)|\omega|^2.\] Moreover, \begin{eqnarray*} \sum_{i=1}^{n+1}2\langle\nabla_{\partial_{x_i}^T}\omega,\mathrm{I\!I}_{\partial^\perp_{x_i}}^{[p]}\omega\rangle&=&p\sum_{i=1}^{n+1}2x_i\langle\nabla_{\partial_{x_i}^T}\omega,\omega\rangle\\ &=&p\sum_{i=1}^{n+1}x_i\partial_{x_i}^T(|\omega|^2)\\ &=&px^T(|\omega|^2)\\ &=&0 \end{eqnarray*} because of $x^T=0$ for every $x\in\mathbb{S}^n$. For the same reason, \begin{eqnarray*} \sum_{i=1}^{n+1}\langle\mathrm{I\!I}_{\partial^\perp_{x_i}}^{[p]}\omega\ ,\partial_{x_i}^T\lrcorner\,d\omega\rangle&=&p\sum_{i=1}^{n+1}x_i\langle\omega,\partial_{x_i}^T\lrcorner\,d\omega\rangle\\ &=&p\langle\omega,x^T\lrcorner\,d\omega\rangle\\ &=&0. \end{eqnarray*} Applying the same computational method as above, we also have, in any pointwise orthonormal basis $(e_j)_{1\leq j\leq n}$ of $TM$, \begin{eqnarray*} \sum_{i=1}^{n+1}\langle \nabla_{\partial_{x_i}^T}\omega,\partial_{x_i}^T\lrcorner\,d\omega\rangle&=&\sum_{j=1}^n\langle \nabla_{e_j}\omega,e_j\lrcorner\,d\omega\rangle\\ &=&\sum_{j=1}^n\langle e_j^\flat\wedge\nabla_{e_j}\omega,d\omega\rangle\\ &=&|d\omega|^2. \end{eqnarray*} On the whole, we obtain \begin{eqnarray*} \sum_{i=1}^{n+1}|d(\partial_{x_i}^T\lrcorner\,\omega)|^2&=&|\nabla\omega|^2+p^2|\omega|^2+(p+1)|d\omega|^2-2|d\omega|^2\\ &=&|\nabla\omega|^2+p^2|\omega|^2+(p-1)|d\omega|^2. \end{eqnarray*} Therefore \[ \sum_{i=1}^{n+1}\|d(\partial_{x_i}^T\lrcorner\,\omega)\|_{L^2(M)}^2= \|\nabla\omega\|_{L^2(M)}^2+p^2\|\omega\|_{L^2(M)}^2+(p-1)\|d\omega|_{L^2(M)}^2. \] Using the Weitzenb\"ock formula (\ref{eq:weitzenboeckpforms}) with $W^{[p]}=p(n-p)\cdot\mathrm{Id}$ and that $\omega_{|_{\partial M}}=\nabla_\nu\omega_{|_{\partial M}}=0$ leads to \begin{eqnarray*} \sum_{i=1}^{n+1}\left\|d(\partial_{x_i}^T\lrcorner\,\omega)\right\|_{L^2(M)}^2&=&\left(\nabla^*\nabla\omega,\omega\right)_{L^2(M)}+p^2\|\omega\|_{L^2(M)}^2+(p-1)\|d\omega|_{L^2(M)}^2\\ &=&\left(\Delta\omega,\omega\right)_{L^2(M)}-\left(W^{[p]}\omega,\omega\right)_{L^2(M)}+p^2\|\omega\|_{L^2(M)}^2+(p-1)\|d\omega|_{L^2(M)}^2\\ &=&\left(\Delta\omega,\omega\right)_{L^2(M)}+(p^2-p(n-p))\|\omega\|_{L^2(M)}^2+(p-1)\|d\omega|_{L^2(M)}^2\\ &=&p(2p-n)\|\omega\|_{L^2(M)}^2+(p-1)\|d\omega\|_{L^2(M)}^2+(\Delta\omega,\omega)_{L^2(M)}. \end{eqnarray*} On the other hand, for every $i\in\{1,\ldots,n+1\}$, we can compute, using a local orthonormal basis $(e_j)_{1\leq j\leq n}$ of $TM$ as well as $\nabla_X\partial_{x_i}^T=-x_i X$ for every $X\in T_xM$, \begin{eqnarray*} \delta(\partial_{x_i}^T\lrcorner\,\omega)&=&-\sum_{j=1}^ne_j\lrcorner\,\nabla_{e_j}(\partial_{x_i}^T\lrcorner\,\omega)\\ &=&-\sum_{j=1}^ne_j\lrcorner\left((\nabla_{e_j}\partial_{x_i}^T)\lrcorner\,\omega+\partial_{x_i}^T\lrcorner\,\nabla_{e_j}\omega\right)\\ &=&-\sum_{j,k=1}^n\langle\nabla_{e_j}\partial_{x_i}^T,e_k\rangle e_j\lrcorner\,e_k\lrcorner\,\omega-\partial_{x_i}^T\lrcorner\,\delta\omega\\ &=&\underbrace{\sum_{j,k=1}^nx_i\langle e_j,e_k\rangle e_j\lrcorner\,e_k\lrcorner\,\omega}_{0}-\partial_{x_i}^T\lrcorner\,\delta\omega\\ &=&-\partial_{x_i}^T\lrcorner\,\delta\omega, \end{eqnarray*} where we used the skew-symmetry of $\langle e_j,e_k\rangle e_j\lrcorner\,e_k\lrcorner\,\omega$ in $(j,k)$. As a consequence, again by \cite[eq. (22)]{EGHM}, \begin{eqnarray*} \sum_{i=1}^{n+1}|\delta(\partial_{x_i}^T\lrcorner\omega)|^2&=&\sum_{i=1}^{n+1}|\partial_{x_i}^T\lrcorner\,\delta\omega|^2\\ &=&(p-1)|\delta\omega|^2, \end{eqnarray*} from which $\displaystyle{\sum_{i=1}^{n+1}\|\delta(\partial_{x_i}^T\lrcorner\omega)\|_{L^2(M)}^2=(p-1)\|\delta\omega\|_{L^2(M)}^2}$ follows. Finally, we deduce that \begin{eqnarray*} &&\Lambda_{1,p-1}\left(p(2p-n)\|\omega\|_{L^2(M)}^2+(p-1)(\|d\omega\|_{L^2(M)}^2+\|\delta\omega\|_{L^2(M)}^2)+(\Delta\omega,\omega)_{L^2(M)}\right)\\ &&\leq4\|\delta\omega\|^2_{L^2(M)}+p\|\Delta\omega+(2p-n)\omega\|^2_{L^2(M)}, \end{eqnarray*} that is, using $\|d\omega\|_{L^2(M)}^2+\|\delta\omega\|_{L^2(M)}^2=\left(\Delta\omega,\omega\right)_{L^2(M)}$, \begin{eqnarray} \label{buckling_shpere_pminus1} \nonumber p\Lambda_{1,p-1}\left((\Delta\omega,\omega)_{L^2(M)}+(2p-n)||\omega||^2_{L^2(M)}\right)\leq 4||\delta\omega||^2_{L^2(M)}+p||\Delta\omega+(2p-n)\omega||^2_{L^2(M)}. \end{eqnarray} Notice that the l.h.s. of that last identity must be positive since it is actually $\displaystyle{\sum_{i=1}^{n+1}\|d(\partial_{x_i}^T\lrcorner\omega)\|_{L^2(M)}^2+\|\delta(\partial_{x_i}^T\lrcorner\omega)\|_{L^2(M)}^2}$. Replacing $\omega$ by $\star\omega$ and using the Hodge symmetry of the buckling eigenvalues ensure that \begin{eqnarray*} (n-p)\Lambda_{1,p+1}\left((\Delta\omega,\omega)_{L^2(M)}+(n-2p)\|\omega\|^2_{L^2(M)}\right)\\ &&\hspace{-2cm}\leq 4\|d\omega\|^2_{L^2(M)}+(n-p)\|\Delta\omega+(n-2p)\omega\|^2_{L^2(M)}. \end{eqnarray*} Adding those two inequalities, we obtain \begin{eqnarray} \label{buckling_sphere} \nonumber2\min(p\Lambda_{1,p-1}, (n-p)\Lambda_{1,p+1})\cdot (\Delta\omega,\omega)_{L^2(M)} & \le & 4(\|d\omega\|^2_{L^2(M)}+\|\delta\omega\|^2_{L^2(M)})\\ \nonumber&&+p||\Delta\omega+(2p-n)\omega||^2_{L^2(M)}\\ \nonumber&&+(n-p)\|\Delta\omega+(n-2p)\omega\|^2_{L^2(M)} \\ \nonumber&=&4\left(\Delta\omega,\omega\right)_{L^2(M)}+n\|\Delta\omega\|_{L^2(M)}^2\\ \nonumber&&+2\left(p(2p-n)+(n-p)(n-2p)\right)\left(\Delta\omega,\omega\right)_{L^2(M)}\\ \nonumber&&+\left(p(2p-n)^2+(n-p)(n-2p)^2\right)\|\omega\|_{L^2(M)}^2\\ &=&n\|\Delta\omega\|^2_{L^2(M)}+n(n-2p)^2\|\omega\|^2_{L^2(M)}\nonumber\\ &&+(4+2(n-2p)^2)(\Delta\omega,\omega)_{L^2(M)}. \end{eqnarray} But \eqref{LaplaceBounds} with $\gamma=1$ gives $$\|\omega\|_{L^2(M)} \le \dfrac{1}{p(n-p+1)} \|\Delta \omega\|_{L^2(M)}.$$ Substituting that estimate in \eqref{buckling_sphere} yields the following inequality: $$2 \min(p\Lambda_{1,p-1}, (n-p)\Lambda_{1,p+1})\cdot\left(\|d\omega\|_{L^2(M)}^2+\|\delta\omega\|_{L^2(M)}^2\right) \le C_{n,p} \|\Delta \omega\|^2_{L^2(M)},$$ where $C_{n,p}$ is the constant defined in the first statement of Theorem \ref{t:domainSn}. We deduce inequality (\ref{eq:upperboundLambdainSn}) by taking $\omega$ as an eigenform of the buckling eigenvalue problem associated to $\Lambda_{1,p}$. Similarly to the first case, the equality cannot occur in (\ref{eq:upperboundLambdainSn}). \item If $n=2p$, then (\ref{eq:upperboundLambdainSn}) becomes, because of $\Lambda_{1,p-1}=\Lambda_{1,n-p+1}=\Lambda_{1,p+1}$, \[\Lambda_{1,\frac{n}{2}-1}<\frac{C_{n,\frac{n}{2}}}{n}\cdot\Lambda_{1,\frac{n}{2}},\] which is (\ref{eq:upperboundLambdainSnforn=2p}). \end{enumerate} \hfill$\square$ Notice that, by the assumption $p\leq n-p$, Inequality (\ref{eq:upperboundLambdainSn}) implies that \[\min(\Lambda_{1,p-1},\Lambda_{1,p+1})< \dfrac{C_{n,p}}{2p}\cdot\Lambda_{1,p}.\] \begin{thebibliography}{99} \bibitem{Anne89} C.~Ann\'e, \emph{Principe de Dirichlet pour les formes diff\'erentielles}, Bull. Soc. Math. France {\bf 117} (1989), no. 4, 445--450. \bibitem{AL} M.S.~Ashbaugh and R.S.~Laugesen, {\it Fundamental tones and buckling loads of clamped plates}, Ann. Sc. Norm. Super. Pisa (4) \textbf{23} (1996), 383--402. \bibitem{ChadrakarGittinsHabibPeyerimhoff2025} T.~Chakradhar, K.~Gittins, G.~Habib and N.~Peyerimhoff, \emph{Magnetic Steklov operator on differential forms}, in progress. \bibitem{ChenChengWangXia2012} D.~Chen, Q.-M.~Cheng, Q.~Wang and C.~Xia, {\it On eigenvalues of a system of elliptic equations and of the biharmonic operator}, J. Math. Anal. Appl. \textbf{387} (2012), no. 2, 1146--1159. \bibitem{elchamiginouxhabib19} F.~El Chami, N.~Ginoux and G.~Habib, \emph{New eigenvalue estimates involving Bessel functions}, Publ. Mat. \textbf{65} (2021), 681--726. \bibitem{EGHM} F.~El Chami, N.~Ginoux, G.~Habib and O.~Makhoul, {\it Biharmonic Steklov operator on differential forms}, to appear in Ann. Math. Blaise Pascal. \bibitem{Friedlander91} L.~Friedlander, \emph{Some inequalities between Dirichlet and Neumann eigenvalues}, Arch. Ration. Mech. Anal. \textbf{116}, No. 2, 153--160 (1991). \bibitem{Friedlander2004} L.~Friedlander, \emph{Remarks on the membrane and buckling eigenvalues for planar domains}, Mosc. Math. J \textbf{4} (2004), no. 2, 369--375. \bibitem{GM} S.~Gallot and D.~Meyer, \emph{Op\'erateur de courbure et laplacien des formes diff\'erentielles d'une vari\'et\'e riemannienne}, J. Math. Pures Appl. \textbf{54} (1975), 259--284. \bibitem{GueriniSavo2004} P.~Guerini and A.~Savo, {\it Eigenvalue and gap estimates for the Laplacian acting on $p$-forms}, Trans. Amer. Math. Soc. {\bf 356} (2004), no. 1, 319--344. \bibitem{HoermanderlinearI} L.~H\"ormander, \emph{The analysis of linear partial differential operators. I. Distribution theory and Fourier analysis}, Springer, 2003. \bibitem{IliasShouman2019} S.~Ilias and A.~ Shouman, \emph{Inequalities between Dirichlet, Neumann and buckling eigenvalues on Riemannian manifolds}, Calc. Var. Partial Differential Equations \textbf{59} (2020), no. 4, Paper No. 127, 15 pp. \bibitem{Payne1955} L.E.~Payne, \emph{Inequalities for eigenvalues of membranes and plates}, J. Rat. Mech. Anal. \textbf{4} (1955), 517--529. \bibitem{Polya1952} G.~P\'olya, \emph{Remarks on the foregoing paper}, J. Math. Phys. \textbf{31} (1952), 55--57 . \bibitem{ProvenzanoSavo2023} L.~Provenzano and A.~Savo, \emph{Isoparametric foliations and the Pompeiu property}, Mathematics in Engineering \textbf{2} (2023), no. 5, 1--27. \bibitem{RaulotSavo2011} S.~Raulot and A.~Savo, \emph{A Reilly formula and eigenvalue estimates for differential forms}, J. Geom. Anal. \textbf{21} (2011), no. 3, 620--640. \bibitem{Salolectnotes2014} M.~Salo (ed.), \emph{Unique continuation for elliptic equations}, lecture notes, 2014. \bibitem{Sc} G.~Schwarz, \emph{Hodge decomposition - A method for solving boundary value problems}, Lecture notes in Mathematics, Springer, 1995. \bibitem{Semmelmann02} U.~Semmelmann, \emph{Conformal Killing forms on Riemannian manifolds}, Math. Z. \textbf{245} (2003), no. 3, 503--527. \end{thebibliography} \end{document}
2412.05588v2
http://arxiv.org/abs/2412.05588v2
Prime rings having nontrivial centralizers of (skew) traces of Lie ideals
\documentclass[12pt]{article} \usepackage[british]{babel} \usepackage{amsmath} \usepackage{amsthm} \usepackage{amsfonts} \usepackage{amssymb} \usepackage{fancyhdr} \usepackage[all]{xy} \usepackage{graphicx} \usepackage[left=3cm, right=3cm]{geometry} \geometry{} \usepackage{enumerate} \usepackage{cite} \usepackage{hyperref} \theoremstyle{plain} \newtheorem{thm}{Theorem}[section] \newtheorem{cor}[thm]{Corollary} \newtheorem{lem}[thm]{Lemma} \newtheorem{pro}[thm]{Proposition} \newtheorem{question}[thm]{Question} \newtheorem{problem}[thm]{Problem} \theoremstyle{definition} \newtheorem{examp}[thm]{Example} \newtheorem{defi}{Definition} \theoremstyle{remark} \newtheorem{remark}[thm]{Remark} \def\alp{\alpha} \def\Lam{\Lambda} \def\Del{\Delta} \def\del{\delta} \def\lam{\lambda} \def\M{\text{\rm M}} \def\gam{\gamma} \def\GF{\text{\rm GF}} \def\E{{\operatorname E}} \def\hgb{{\hat g}_b} \def\hhb{{\hat h}_b} \def\Rdel{R^{(\del)}} \def\Qdel{Q^{(\del)}} \def\CalL{{\Cal L}} \def\ba{{\bold a}} \def\bb{{\bold b}} \def\bc{{\bold c}} \def\bh{{\bold h}} \def\bl{{\bold l}} \def\bx{{\bold x}} \def\byy{{\bold y}} \def\bff{{\bold f}} \def\bS{{\bold S}} \def\bTheta{{\bold\Theta}} \def\bPsi{{\bold\Psi}} \def\bPhi{{\bold\Phi}} \def\Cotimes{{\underset C\to\otimes}} \def\deg{\text{\rm deg}} \def\del{{\delta}} \def\dim{\text{\rm dim}} \def\ov{\overline} \def\soc{\text{soc}} \def\Ref{\text{\rm Ref}\,} \newcommand{\overbar}[1]{\mkern 1.5mu\overline{\mkern-1.5mu#1\mkern-1.5mu}\mkern 1.5mu} \author{Tsiu-Kwen Lee$^\flat$ and Jheng-Huei Lin$^\natural$} \title{Prime rings having nontrivial centralizers of (skew) traces of Lie ideals} \date{} \begin{document} \maketitle \centerline {Department of Mathematics, National Taiwan University${^\flat,}$${^\natural}$} \centerline {Taipei, Taiwan} \centerline {[email protected]$^\flat$; [email protected]${^\natural}$} \begin{abstract}\vskip6pt \noindent Let $R$ be a prime ring with center $Z(R)$ and with involution $*$. Given an additive subgroup $A$ of $R$, let $ T(A):=\{x+x^*\mid x\in A\} $ and $ K_0(A):=\{x-x^*\mid x\in A\}. $ Let $L$ be a non-abelian Lie ideal of $R$. It is proved that if $d$ is a nonzero derivation of $R$ satisfying $d(T(L))=0$ (resp. $d(K_0(L))=0$), then $T(R)^2\subseteq Z(R)$ (resp. $K_0(R)^2\subseteq Z(R)$). These results are applied to the study of $d(T(M))=0$ and $d(K_0(M))=0$ for noncentral $*$-subrings $M$ of a division ring $R$ such that $M$ is invariant under all inner automorphisms of $R$, and for noncentral additive subgroups $M$ of a prime ring $R$ containing a nontrivial idempotent such that $M$ is invariant under all special inner automorphisms of $R$. The obtained theorems also generalize some recent results on simple artinian rings with involution due to M. Chacron. \end{abstract} { \hfill\break \noindent 2020 {\it Mathematics Subject Classification.}\ 16N60, 16W10, 16K40. \vskip6pt \noindent {\it Key words and phrases:}\ Prime ring, division ring, involution, derivation, transpose, symplectic, (skew) trace, Lie ideal, centralizer.\ \vskip6pt \section{Introduction} In several branches of mathematics, science and technology, it is always interesting to study how local behavior of a given structure affects the entire one. On the topic of rings with involution, a lot of researchers have investigated the connection between properties on the whole ring and behavior of a specific subset relating to the involution. For instance, the study on subsets of the set comprising all symmetric (or skew) elements is very popular. On such type of subsets, Amitsur et al. considered polynomial identities (see \cite{amitsur1968, baxter1968,herstein1967,martindale1969,montgomery1971a}), Osborn discussed invertibility (see \cite{osborn1967}), and Chacron, Herstein and Montgomery investigated some algebraic conditions such as Jacobson condition of periodicity (see \cite{chacron1975,herstein1974,herstein1971,montgomery1971b,montgomery1973}). Inspired by Amitsur \cite{amitsur1968}, similar studies also appear in the theory of group algebras. See \cite{balogh2012,balogh2008,catino2014,lee2010,lee2015,shalev1992}. On the other hand, in the theory of functional identities on prime rings, there is a very important theorem relating to this topic, given by Beidar and Martindale, saying that the standard type of functional identities with involution can be completely solved by the standard solution if the maximal degree of all symmetric and skew elements is large enough (see \cite[Theorem 3.1]{beidar1998}). During the first quarter of the century, there are still several researchers studying different topics on subsets of the set comprising all symmetric (or skew) elements. See, for example, \cite{bien2019,ferreira2015,goodaire2013,lin2010,mosic2009,siciliano2011,thu2022}. In 2022, Bien, Hai and Hue investigated how the commutativity of trace or norm elements of units affects the whole ring and the involution (see \cite{bien2022}). In the recent papers \cite{chacron2022, chacron2023, chacron2024}, Chacron continued the work and discussed the effects of properties, such as commutativity, of trace or skew trace elements of some specific substructures. Throughout this paper, let $R$ be an associative ring with center $Z(R)$, not necessarily with unity. For $a, b\in R$, let $[a, b]:=ab-ba$, the additive commutator of $a$ and $b$. For additive subgroups $A, B$ of $R$, we let $AB$ (resp. $[A, B]$) stand for the additive subgroup of $R$ generated by all elements $ab$ (resp. $[a, b]$) for $a\in A$ and $b\in B$. An additive subgroup $L$ is said to be a {\it Lie ideal} of $R$ if $[L, R]\subseteq L$. A Lie ideal is called {\it abelian} if $[L, L]=0$. Otherwise, it is called {\it non-abelian}. Finally, let $A$ be a subset of $R$, we denote by $\overline A$ the subring of $R$ generated by $A$. Let $R$ be equipped with an involution $*$, that is, $*$ is an anti-automorphism of $R$ with period $\leq 2$. An element $x\in R$ is called {\it symmetric} (resp. {\it skew}) if $x^*=x$ (resp. $x^*=-x$). The involutions on a primitive ring, not a division ring, with nonzero socle are completely determined (see \cite[Theorem 1]{montgomery1976} and \cite[Theorem 1.2.2]{herstein1976}). Depending on \cite[Theorem 1.2.2]{herstein1976}, we have the following.\vskip6pt \begin{defi}\label{def1} Let $R$ be a primitive ring, not a division ring, with nonzero socle. An involution $*$ on $R$ is said to be of the {\it transpose} type if $R$ has a symmetric minimal idempotent, that is, there exists a minimal idempotent $e\in R$ such that $e^*=e$. Otherwise, the involution $*$ is said to be of the {\it symplectic} type. \end{defi} The following is well-known (see \cite[Theorem 1]{montgomery1976} and \cite[Corollary, p.19]{herstein1976}). \begin{thm}\label{thm5} Let $R:=\text{\rm M}_n(D)$ be equipped with an involution $*$, where $n>1$ and $D$ is a division ring. Case 1:\ The involution $*$ is of the symplectic type. Then $n=2m$ and $D$ is a field. Let $(A_{ij})\in R=\text{\rm M}_m(\text{\rm M}_2(D))$, where $A_{ij}\in \text{\rm M}_2(D)$, $1\leq i, j\leq m$. We have $(A_{ij})^*=(B_{ij})$, where $B_{ij}=A_{ji}^\sigma$ for $1\leq i, j\leq m$. Here $$\left[ \begin{array}{cc} \alpha & \beta \\ \gamma & \delta\end{array}\right]^\sigma=\left[ \begin{array}{cc} \delta &- \beta \\ -\gamma & \alpha\end{array}\right]\in \text{\rm M}_2(D). $$ Case 2:\ The involution $*$ is of the transpose type. Then there exist an involution $-$ on $D$ and nonzero $\pi_i\in D$, $1\leq i\leq n$, such that $\bar{\pi_i}=\pi_i$ for all $i$, and $$ (a_{ij})^*=\text{\rm Diag}(\pi_1^{-1},\ldots,\pi_n^{-1})(\bar{a_{ij}})^t\text{\rm Diag}(\pi_1,\ldots,\pi_n)\in R. $$ \end{thm} Let $A$ be an additive subgroup of $R$, and let $S(A)$ (resp. $K(A)$, $T(A)$, $K_0(A)$) be the set of all {\it symmetric} (resp. {\it skew, trace, skew trace}) elements of $A$. Precisely, we have $$ S(A):=\{x\in A\mid x=x^*\},\ \ K(A):=\{x\in A\mid x^*=-x\}, $$ $$ T(A):=\{x+x^*\mid x\in A\}\ \text{\rm and}\ K_0(A):=\{x-x^*\mid x\in A\}. $$ A ring $R$ is called {\it prime} if, given $a, b\in R$, $aRb=0$ implies that either $a=0$ or $b=0$. Let $R$ be a prime ring, and let $Q_s(R)$ be the Martindale symmetric ring of quotients of $R$. It is known that $Q_s(R)$ is also a prime ring with center, denoted by $C$, which is called the {\it extended centroid} of $R$. Note that $C$ is always a field. It is also known that if $R$ is a prime PI-ring then $Q_s(R)=RC$ and $IC=RC$ for any nonzero ideal $I$ of $R$. See \cite{beidar1996} for details. Let $R$ be a prime ring with involution $*$. It is known that the involution $*$ can be uniquely extended to an involution, denoted by the same $*$, on $Q_s(R)$, and $C^*=C$. We say that $*$ is of the {\it first kind} if $*$ is the identity map on $C$. Otherwise, we say that $*$ is of the {\it second kind}. \vskip6pt \begin{defi}\label{def2} A prime ring $R$ is called {\it exceptional} if both $\text{\rm char}\,R= 2$ and $\dim_CRC=4$. \end{defi} An additive map $d\colon R\to R$ is called a {\it derivation} if $d(xy)=d(x)y+xd(y)$ for all $x, y\in R$. The derivation $d$ of the ring $R$ is called {\it inner} if there exists $a\in R$ such that $d(x)=[a, x]$ for all $x\in R$. It is known that if $R$ is a prime ring, every derivation $d$ of $R$ can be uniquely extended to a derivation, denoted by the same $d$, of $Q_s(R)$. The derivation $d$ of $R$ is called {\it X-inner} if the extension of $d$ to $Q_s(R)$ is inner, that is, there exists $b\in Q_s(R)$ such that $d(x)=[b, x]$ for all $x\in Q_s(R)$. Otherwise, $d$ is called {\it X-outer}. \vskip6pt \begin{defi}\label{def3} Let $R$ be a prime ring with involution $*$ such that $RC$ is a primitive ring, not a division ring, with nonzero socle. We say that $*$ is of the symplectic (resp. transpose) type on $R$ if $*$ is of the symplectic (resp. transpose) type on $RC$. \end{defi} Given a unital ring $R$, we let $R^\times$ be the set of all units in the ring $R$. By a {\it $*$-subset} we mean a subset $M$ of $R$ such that $M^*=M$. In recent papers \cite{chacron2022, chacron2023, chacron2024}, Chacron studied $*$-submonoids $M$ (i.e., multiplicatively closed $*$-subsets with $1$) in a simple artinian ring $R$ such that $M$ is invariant under all inner automorphisms of $R$ (i.e., $uMu^{-1}\subseteq M$ for all $u\in R^\times$). We mention the main theorems proved in \cite{chacron2023, chacron2024}. \begin{thm} (\cite[Theorem 6]{chacron2023})\label{thm33} Let $R$ be a simple artinian ring with involution $*$, $Z(R)\ne \text{\rm GF}(2)$, and let $M$ be a noncentral $*$-submonoid of $R$, which is invariant under all inner automorphisms of $R$. If $[T(M), T(M)]=0$, then $[T(R), T(R)]=0$, $\dim_{Z(R)}R=4$, and $*$ is of the first kind. \end{thm} We remark that, in the above theorem, the ring $R$ and the involution $*$ can be described in more details according to either $R\cong \text{\rm M}_2(Z(R))$ or $R$ a $4$-dimensional central division algebra. The following theorem complements the above theorem under the current $2$-torsion free assumption. \begin{thm} (\cite[Theorem 6]{chacron2024})\label{thm34} Let $R$ be a $2$-torsion free simple artinian ring with involution $*$, and let $M$ be a noncentral $*$-submonoid of $R$, which is invariant under all inner automorphisms of $R$. If $[K_0(M), K_0(M)]=0$, then $[K(R), K(R)]=0$, $\dim_{Z(R)}R=4$, and $*$ is of the orthogonal (i.e., transpose) type. \end{thm} \begin{remark}\label{remark1} {\rm (i)\ We remark that Corollaries \ref{cor10} and \ref{cor11} generalize both of the above two theorems in the division case and in the non division case, respectively. (ii)\ In Theorems \ref{thm33} and \ref{thm34}, let $M^+$ denote the additive subgroup of $R$ generated by $M$. Then $M^+$ is a noncentral $*$-subring of $R$, which is invariant under all inner automorphisms of $R$. Moreover, $[T(M), T(M)]=0$ if and only if $[T(M^+), T(M^+)]=0$, and $[K_0(M), K_0(M)]=0$ if and only if $[K_0(M^+), K_0(M^+)]=0$. Hence it suffices to assume from the start that $M$ is a noncentral $*$-subring of $R$, which is invariant under all inner automorphisms of $R$.} \end{remark} We first give some observations for the case that $T(R)\subseteq Z(R)$ (resp. $K_0(R)\subseteq Z(R)$). In a unital ring $R$ with involution $*$, Lim proved that $R$ is a central $*$-trace ring (i.e., $x+x^*\in Z(R)$ for all $x\in R$) if and only if $R$ is a central $*$-norm ring (i.e., $xx^*\in Z(R)$ for all $x\in R$) (see \cite[Theorem 3]{lim1979}). Also, a $*$-commuting ring $R$ (i.e., $[x, x^*]=0$ for all $x\in R$) with $2R=0$ must be a central $*$-trace ring (see \cite[Theorem 2]{lim1979}). Lim also gave an example to show that in general a $*$-commuting ring is not necessary to be a central $*$-trace ring (see \cite[Example 1, p.127]{lim1979}). We remark that the example is an algebra over a field $F$ of characteristic not $2$, which is not a semiprime ring. Recall that a ring $R$ is called {\it semiprime} if, for $a\in R$, $aRa=0$ implies $a=0$. This is equivalent to saying that $R$ has no nonzero nilpotent ideals. Given an involution $*$ on a unital semiprime ring $R$, Chacron proved that the following are equivalent: (1) $R$ is a $*$-commuting ring; (2) $R$ is a central $*$-trace ring; (3) $R$ is a central $*$-norm ring (see \cite[Theorem 2.3]{chacron2016}). In fact, commuting anti-automorphisms on semiprime rings, not necessarily with unity, are completely characterized (see \cite[Theorem 1.2]{lee2017}, and also \cite[Theorem 1.1]{lee2018}). \begin{pro}\label{pro2} Let $R$ be a semiprime ring with involution $*$. Then $K_0(R)\subseteq Z(R)$ if and only if $T(R)\subseteq Z(R)$ and $2R\subseteq Z(R)$. \end{pro} \begin{proof} Assume that $K_0(R)\subseteq Z(R)$. Let $x\in R$. Then $x-x^*\in Z(R)$ and so $[x, x^*]=0$. Therefore $R$ is a $*$-commuting ring. In view of \cite[Theorem 1.1]{lee2018} (or \cite[Theorem 2.3]{chacron2016} for semiprime rings with unity), $R$ is a central $*$-trace ring, i.e., $T(R) \subseteq Z(R)$. Given $x\in R$, we have $ 2x=(x-x^*)+(x+x^*)\in Z(R) $ and hence $2R\subseteq Z(R)$. The converse is obvious. \end{proof} The following is an immediate consequence of Proposition \ref{pro2}. \begin{cor}\label{cor8} Let $R$ be a noncommutative prime ring with involution $*$. Then $K_0(R)\subseteq Z(R)$ if and only if $T(R)\subseteq Z(R)$ and $\text{\rm char}\,R=2$. \end{cor} Our first aim is to characterize $[T(R), T(R)]=0$ (resp. $[K_0(R), K_0(R)]=0$) for a given prime ring $R$ with involution $*$. The following two characterizations are Theorems \ref{thm2} and \ref{thm4}.\vskip6pt \noindent{\bf Theorem A.} {\it Let $R$ be a prime ring with involution $*$. If $T(R)\nsubseteq Z(R)$, then the following are equivalent: (i)\ $[T(R), T(R)]=0$; (ii)\ $d(T(R))=0$ for some nonzero derivation $d$ of $R$; (iii)\ $[b, T(R)]=0$ for some $b\in RC\setminus C$; (iv)\ $R$ is exceptional and $*$ is of the first kind; (v)\ $T(R)^2\subseteq Z(R)$.} \vskip6pt \noindent{\bf Theorem B.} {\it Let $R$ be a prime ring with involution $*$. If $K_0(R)\nsubseteq Z(R)$, then the following are equivalent: (i)\ $[K_0(R), K_0(R)]=0$; (ii)\ $d(K_0(R))=0$ for some nonzero derivation $d$ of $R$; (iii)\ $[b, K_0(R)]=0$ for some $b\in RC\setminus C$; (iv)\ $K_0(R)^2\subseteq Z(R)$; (v)\ $\dim_CRC=4$, $*$ is of the transpose type, and $*$ is of the first kind.} \vskip6pt In the statements of Theorems A and B, we need the notion of the transpose or symplectic type on division algebras. It will be given in Definition \ref{def4} of the next section. For division rings, we will prove the following main theorem (i.e., Theorem \ref{thm14}). \vskip6pt \noindent{\bf Theorem C.} {\it Let $R$ be a division ring with involution $*$ and let $M$ be a noncentral $*$-subring of $R$ such that $M$ is invariant under all inner automorphisms of $R$. If $d$ is a nonzero derivation of $R$ such that $ d(T(M))=0 $ (resp.\ $d(K_0(M))=0$), then $T(R)^2\subseteq Z(R)$ (resp. $K_0(R)^2\subseteq Z(R)$).}\vskip6pt Finally, we extend Theorems A and B to the case of Lie ideals (i.e., Theorems \ref{thm20} and \ref{thm32}).\vskip6pt \noindent{\bf Theorem D.}\ {\it Let $R$ be a prime ring with involution $*$, $L$ a non-abelian Lie ideal of $R$, and $d$ a nonzero derivation of $R$. If $d(T(L))=0$, then $T(R)^2\subseteq Z(R)$.}\vskip6pt \noindent{\bf Theorem E.}\ {\it Let $R$ be a prime ring with involution $*$, $L$ a non-abelian Lie ideal of $R$, and $d$ a nonzero derivation of $R$. If $d(K_0(L))=0$, then $K_0(R)^2\subseteq Z(R)$.}\vskip6pt By a {\it special inner automorphism} $\phi$ of a ring $R$, not necessarily with unity, we mean that there exists a square zero element $t\in R$ such that $ \phi(x)=(1+t)x(1+t)^{-1} $ for all $x\in R$. Chuang proved that, in a prime ring $R$ containing a nontrivial idempotent, every noncentral additive subgroup, which is invariant under all special inner automorphisms, contains a Lie ideal of the form $[I, R]$ with $I$ a nonzero ideal of $R$ except when $R$ is exceptional (see \cite[Theorem 1]{chuang1987}). Note that $[I, R]$ is a non-abelian Lie ideal of $R$ (see Lemma \ref{lem5} (iii)). Applying it, the following is an immediate consequence of Theorems D and E (cf. Theroem \ref{thm19}).\vskip6pt \begin{cor}\label{cor7} Let $R$ be a prime ring with involution $*$, containing a nontrivial idempotent. Suppose that $A$ is a noncentral additive subgroup of $R$, which is invariant under all special inner automorphisms of $R$. If $d$ is a nonzero derivation of $R$ such that $d(T(A))=0$ (resp. $d(K_0(A))=0$), then $T(R)^2\subseteq Z(R)$ (resp. $K_0(R)^2\subseteq Z(R)$) except when $R$ is exceptional. \end{cor} Whenever it is more convenient, we will use the widely accepted shorthand \textquotedblleft iff\textquotedblright\ for \textquotedblleft if and only if\textquotedblright\ in the text. \section{Preliminaries} We refer the following lemma to \cite[Lemmas 1 and 5]{lee1995}. \begin{lem}\label{lem1} Let $R$ be a noncommutative prime ring with involution $*$. (i)\ If $d$ is a nonzero derivation of $R$ such that either $d(T(R))\subseteq Z(R)$ or $d(K_0(R))\subseteq Z(R)$, then $\dim_CRC=4$. (ii)\ If either $T(R)^2\subseteq Z(R)$ or $K_0(R)^2\subseteq Z(R)$, then $\dim_CRC=4$. \end{lem} \begin{lem} (\cite[Lemma 5]{bergen1981})\label{lem2} Let $R$ be a noncommutative prime ring, $\text{\rm char}\,R\ne 2$. If $d$ is a nonzero derivation of $R$, and $U$ a Lie ideal of $R$ such that $d(U) = 0$, then $U\subseteq Z(R)$. \end{lem} We need a special case of Theorem \ref{thm5} in the proofs below. \begin{lem}\label{lem7} Let $R=\text{\rm M}_2(C)$, where $C$ is a field, with involution $*$. \vskip4pt (i) If the involution $*$ is of the symplectic type, then $\left[ \begin{array}{cc} \alpha & \beta \\ \gamma & \delta\end{array}\right]^*=\left[ \begin{array}{cc} \delta & -\beta \\ -\gamma & \alpha\end{array}\right]\in R.$ In this case, we have $T(R)=C$, $K_0(R)=\{\left[ \begin{array}{cc} \alpha & 2\beta \\ 2\gamma & -\alpha\end{array}\right]\mid \alpha, \beta, \gamma\in C\}$, and $K_0(R)^2\subseteq C$ iff $\text{\rm char}\,R=2$. In particular, if $\text{\rm char}\,R\ne 2$, then $K_0(R)=[R, R]$ and $K_0(R)^2=R$.\vskip4pt (ii)\ If the involution $*$ is of the transpose type, then there exist an involution $-$ on $C$ and $\pi_1, \pi_2\in C\setminus \{0\}$, $\bar{\pi_i}=\pi_i$, $i=1, 2$, such that $ \left[ \begin{array}{cc} \alpha & \beta \\ \gamma & \delta\end{array}\right]^*=\left[ \begin{array}{cc} \bar\alpha & \pi\bar\gamma \\ \pi^{-1}\bar\beta & \bar\delta\end{array}\right], $ where $\pi:=\pi_1^{-1}\pi_2$. In this case, we have $$ T(R)=\{\left[ \begin{array}{cc} \alpha +\bar\alpha & \gamma \\ \pi^{-1}\bar\gamma& \delta+\bar\delta\end{array}\right]\mid \alpha, \gamma, \delta\in C\}\ \text{\rm and}\ \ K_0(R)=\{\left[ \begin{array}{cc} \alpha -\bar\alpha & \gamma \\ -\pi^{-1}\bar\gamma& \delta-\bar\delta\end{array}\right]\mid \alpha, \gamma, \delta\in C\}. $$ (iii)\ $T(R)\subseteq C$ iff $*$ is of the symplectic type. In this case, $T(R)=C$. \end{lem} \begin{lem}\label{lem3} Let $R=\text{\rm M}_2(C)$, where $C$ is a field, with involution $*$, which is of the transpose type. If $*$ is of the first kind, then the following hold (with notation in Lemma \ref{lem7}): (i)\ $ T(R)=\{\left[ \begin{array}{cc} 2\alpha & \gamma \\ \pi^{-1}\gamma& 2\delta\end{array}\right]\mid \alpha, \gamma, \delta\in C\}$ and $K_0(R)=\{\left[ \begin{array}{cc} 0 & \gamma \\ -\pi^{-1}\gamma & 0\end{array}\right]\mid \gamma\in C\}; $ (ii)\ $T(R)^2\subseteq C$ iff $\text{\rm char}\,R=2$. In this case, $T(R)^2=C$; (iii)\ $K_0(R)^2=C$; (iv)\ $1\in T(R)$ iff $\text{\rm char}\,R\ne 2$. \end{lem} \begin{proof} (i) follows directly from Lemma \ref{lem7} (ii). For (ii), if $\text{\rm char}\,R=2$, then $T(R)=\{\left[ \begin{array}{cc} 0 & \gamma \\ \pi^{-1}\gamma& 0\end{array}\right]\mid \gamma\in C\}$ and so $T(R)^2=C$. Conversely, assume that $T(R)^2\subseteq C$. Then $$ \left[ \begin{array}{cc} 2 &1\\ \pi^{-1}& 2\end{array}\right]\left[ \begin{array}{cc} 0 & 1 \\ \pi^{-1}& 0\end{array}\right]=\left[ \begin{array}{cc} \pi^{-1}&2\\ 2\pi^{-1}& \pi^{-1}\end{array}\right]\in C, $$ implying $2=0$, that is, $\text{\rm char}\,R=2$. Finally, (iii) and (iv) hold trivially. \end{proof} Let $R$ be a $4$-dimensional central division algebra with involution $*$, which is of the first kind, and let $F$ be a maximal subfield of $R$. Then, clearly, we have the canonical involution, denoted by the same $*$, on $R\otimes_{Z(R)}F$ as follows: $ \big(\sum_ix_i\otimes \beta_i\big)^*=\sum_ix_i^*\otimes \beta_i, $ where $x_i\in R$ and $\beta_i\in F$ for all $i$. \begin{lem}\label{lem10} Let $R$ be a $4$-dimensional central division algebra with involution $*$, which is of the first kind, and let $F$ be a maximal subfield of $R$. If the canonical involution $*$ on $R\otimes_{Z(R)}F$ is of the symplectic type, then so is the canonical involution $*$ on $R\otimes_{Z(R)}L$ for any maximal subfield $L$ of $R$. \end{lem} \begin{proof} Note that $R\otimes_{Z(R)}F\cong \text{\rm M}_2(F)$, and that $T(R\otimes_{Z(R)}F)=T(R)\otimes_{Z(R)}F$. Therefore, we have $T(R\otimes_{Z(R)}F)\subseteq F$ iff $T(R)\subseteq Z(R)$. By Lemma \ref{lem7}, the canonical involution of $R\otimes_{Z(R)}F$ is of the symplectic type iff $T(R)\subseteq Z(R)$. Since the condition that $T(R)\subseteq Z(R)$ is independent of the choice of any maximal subfield of $R$, this completes the proof. \end{proof} By Lemma \ref{lem10} and its proof, we are ready to give the following. \vskip6pt \begin{defi}\label{def4} Let $R$ be a $4$-dimensional central division algebra with involution $*$, which is of the first kind. The involution $*$ on $R$ is said to be of the symplectic (resp. transpose) type if $T(R)\subseteq Z(R)$ (resp. $T(R)\nsubseteq Z(R)$). \end{defi} In Definition \ref{def4}, if $T(R)\subseteq Z(R)$, then the involution $*$ is of the first kind. This fact holds for any noncommutative prime ring $R$ with involution $*$. Indeed, suppose that $\beta\ne \beta^*$ for some $\beta\in C$. There exists a nonzero ideal $I$ of $R$ such that $\beta I\subseteq R$. For $x, y\in I$, we have $ \big[\beta x+\beta^*x^*, y\big]=0 $ and $ \big[\beta^*x+\beta^*x^*, y\big]=0, $ implying that $ (\beta^*-\beta)[x, y]=0. $ Thus $[x, y]=0$ for all $x, y\in I$, implying that $R$ is commutative, a contradiction. This proves that $*$ is of the first kind. We can also define the symplectic or transpose type of the first kind involution $*$ on a finite-dimensional central division algebra but Definition \ref{def4} is sufficient for our purpose.\vskip6pt \begin{lem}\label{lem6} Let $R$ be a $4$-dimensional central division algebra with involution $*$, which is of the first kind. \noindent Case 1:\ $*$ is of the transpose type. Then the following hold: (i)\ $T(R)^2\subseteq Z(R)$ iff $\text{\rm char}\,R=2$; (ii)\ $K_0(R)^2=Z(R)$. \noindent Case 2:\ $*$ is of the symplectic type. Then $T(R)=Z(R)$, and $K_0(R)^2\subseteq Z(R)$ iff $\text{\rm char}\,R=2$. \end{lem} \begin{proof} Let $\widetilde R:=R\otimes_{Z(R)}L$, where $L$ is a maximal subfield of $R$, $\widetilde T:=T(\widetilde R)$, and ${\widetilde K}_0:=K_0(\widetilde R)$. Then $*$ is of the first kind on $\widetilde R$. Clearly, $\widetilde T=T(R)\otimes L$ and $\widetilde {K}_0=K_0(R)\otimes L$. Moreover, we have $ {\widetilde T}^2=T(R)^2\otimes L $ and $ {\widetilde K_0}^2={K_0}(R)^2\otimes L. $ Case 1:\ $*$ is of the transpose type. Since $*$ is of the transpose type on $\widetilde R$, it follows from Lemma \ref{lem3} (ii) that $ {\widetilde T}^2\subseteq L\ \text{\rm iff}\ \ \text{\rm char}\,R=2. $ Moreover, by Lemma \ref{lem3} (iii) we have ${\widetilde K_0}^2=L$. That is, $T(R)^2\subseteq Z(R)$ iff $\text{\rm char}\,R=2$. Thus (i) is proved. We also have $K_0(R)^2=Z(R)$, proving (ii). Case 2:\ $*$ is of the symplectic type. It follows directly from Lemma \ref{lem7} (i). \end{proof} Let $R$ be a prime ring, and let $a, b\in Q_s(R)$. If $aRb=0$, then either $a=0$ or $b=0$. Indeed, there exists a nonzero ideal $I$ of $R$ such that $aI, Ib\subseteq R$. Then $aIRIb=0$. The primeness of $R$ implies that either $aI=0$ or $Ib=0$. That is, either $a=0$ or $b=0$. \begin{lem}\label{lem4} Let $R$ be a noncommutative prime ring with involution $*$, and let $I$ be a nonzero ideal of $R$. If $aT(I)=0$ (resp. $aK_0(I)=0$) where $a\in Q_s(R)$, then $a=0$. \end{lem} \begin{proof} Assume that $aT(I)=0$, where $a\in Q_s(R)$. Given $x\in R$ and $y\in I$, we have $$ axy=-a(xy)^*=-ay^*x^*=ayx^*=-axy^*. $$ That is, $aRT(I)=0$ and so either $a=0$ or $T(I)=0$. Assume that $T(I)=0$. Then, for $x, y\in I$, we have $xy=-y^*x^*=-yx$ and so $xy+yx=0$. It is easy to prove that $R$ is commutative, a contradiction. Hence $a=0$ follows. The case $aK_0(I)=0$ has a similar argument by a slight modification. \end{proof} \begin{lem}\label{lem8} Let $R$ be a ring with involution $*$. Then $T(R)^2$ and $K_0(R)^2$ are Lie ideals of $R$. \end{lem} \begin{proof} We only prove that $T(R)^2$ is a Lie ideal of $R$. The skew case has almost the same argument. Let $t_1, t_2\in T(R)$ and $x\in R$. Then $$ [t_1t_2, x]=t_1(t_2x+x^*t_2)-(t_1x^*+xt_1)t_2\in T^2. $$ This proves that $T(R)^2$ is a Lie ideal of $R$. \end{proof} \begin{lem}\label{lem5} Let $R$ be a ring. (i)\ If $A$ is an additive subgroup of $R$, then $[A, R]=[\overline A, R]$. (ii)\ If $L$ is a Lie ideal of $R$, then $R[L, L]R\subseteq L+L^2$ and $\big[R[L, L]R, R\big]\subseteq L$ (see \cite[Lemma 2.1]{lee2022}). (iii)\ If $R$ is a prime ring and if $\big[a, [R, R]\big]=0$ where $a\in Q_s(R)$, then $a\in C$ (see \cite[Lemma 1.5]{herstein1969}). (iv)\ Let $R$ be a prime ring, and $U$ and $V$ two Lie ideals of $R$. If $[U, V]\subseteq Z(R)$, then either $U\subseteq Z(R)$ or $V\subseteq Z(R)$ except when $R$ is exceptional (\cite[Lemma 7]{lanski1972}). \end{lem} We only give the short proof of (i). Let $a_1,\ldots,a_n\in A$ and $x\in R$. By induction on $n$, we get \begin{equation*} [a_1a_2\cdots a_n, x]=[a_2\cdots a_n, xa_1]+ [a_1, a_2\cdots a_nx]\in [A, R]. \end{equation*} \vskip6pt \noindent {\bf The Skolem-Noether Theorem.}\ {\it Let $R$ be a finite-dimensional central simple algebra. Then every derivation $d$ of $R$ satisfying $d(Z(R))=0$ is inner.}\vskip6pt \noindent {\bf The Brauer-Cartan-Hua Theorem.}\ {\it Let $R$ be a division ring, and let $A$ be a subdivision ring of $R$. If $A$ is invariant under all inner automorphisms, then either $A\subseteq Z(R)$ or $A=R$.}\vskip6pt \section{Theorem A} In this section, except Lemma \ref{lem9}, we always assume that {\it $R$ is a prime ring with involution $*$}. The aim of this section is to characterize $[T(R), T(R)]=0$. \begin{lem}\label{lem26} Let $d$ be a nonzero derivation of $R$. If $d(\beta)=0$ for all $\beta=\beta ^*\in C$, then $d(C)=0$. \end{lem} \begin{proof} Let $\mu\in C$. Then $d(\mu+\mu^*)=0$ and $d(\mu\mu^*)=0$, implying that $(\mu-\mu^*)d(\mu)=0$. Thus either $\mu=\mu^*$ or $d(\mu)=0$. In either case, we get $d(\mu)=0$. That is, $d(C)=0$. \end{proof} \begin{lem}\label{lem12} Let $R$ be noncommutative, and let $d$ be a nonzero derivation of $R$. Suppose that either $d(T(R))=0$ or $d(K_0(R))=0$. Then $*$ is of the first kind. \end{lem} \begin{proof} Assume that $d(T(R))=0$. Let $\beta=\beta^*\in C$. There exists a nonzero $*$-ideal $I$ of $R$ such that $\beta I\subseteq R$. Let $x\in I$. Then $d(x+x^*)=0$ and $$ 0=d( \beta x+(\beta x)^*)=d(\beta (x+x^*))=d(\beta)(x+x^*). $$ By Lemma \ref{lem4}, we get $d(\beta)=0$. Hence $d(\beta)=0$ for all $\beta=\beta^*\in C$. In view of Lemma \ref{lem26}, we get $d(C)=0$. Let $\beta\in C$. Then $\beta I\subseteq R$ for some nonzero $*$-ideal $I$ of $R$. Let $x\in I$. Then $d(x+x^*)=0$ and $$ 0=d( \beta x+(\beta x)^*)= \beta d(x)+\beta^*d(x^*)=(\beta-\beta^*)d(x). $$ So $(\beta-\beta^*)d(I)=0$. Hence $(\beta-\beta^*)d(IR)=0$ and so $(\beta-\beta^*)Id(R)=0$. Since $d\ne 0$, we get $\beta=\beta^*$. This proves that $*$ is of the first kind. The case $d(K_0(R))=0$ has almost the same argument by a slight modification. \end{proof} The following is Theorem A mentioned in the introduction. \begin{thm}\label{thm2} If $T(R)\nsubseteq Z(R)$, then the following are equivalent: (i)\ $[T(R), T(R)]=0$; (ii)\ $d(T(R))=0$ for some nonzero derivation $d$ of $R$; (iii)\ $[b, T(R)]=0$ for some $b\in RC\setminus C$; (iv)\ $R$ is exceptional and $*$ is of the first kind; (v)\ $T(R)^2\subseteq Z(R)$. \end{thm} \begin{proof} In view of Lemma \ref{lem1}, if one of (i)--(v) is satisfied, then $\dim_CRC=4$. Also, by Lemma \ref{lem12}, any one of (i)--(iv) implies that $*$ is of the first kind. Let $F:=C$ if $RC\cong \text{\rm M}_2(C)$, and let $F$ be a maximal subfield of $RC$ if $RC$ is a division algebra. Let $\widetilde R:=RC\otimes_CF\cong\text{\rm M}_2(F)$. Since $T(R)\nsubseteq Z(R)$, in either case, any one of (i)--(iv) implies that the canonical involution $* $ on $\widetilde R$ is of the transpose type (see Lemma \ref{lem7}). That is, the involution $*$ on $RC$ is of the transpose type. (i) $\Rightarrow$ (ii):\ Since $T(R)\nsubseteq Z(R)$, we have $[b, T(R)]=0$ for some $b\in T(R)\setminus Z(R)$. Let $d$ be the inner derivation of $R$ induced by the element $b$. Then $d$ is nonzero and $d(T(R))=0$, as desired. (ii) $\Rightarrow$ (iii):\ Recall that $\dim_CRC=4$. Let $\beta=\beta^*\in C$. There exists a nonzero $*$-ideal $I$ of $R$ such that $\beta I\subseteq R$. Then, for $x\in I$, we have $$ 0=d((\beta x)+(\beta x)^*)=d(\beta(x+x^*))=d(\beta)(x+x^*). $$ That is, $d(\beta)(x+x^*)=0$ for all $x\in I$. It follows from Lemma \ref{lem4} that $d(\beta)=0$. Therefore, $d(\beta)=0$ for all $\beta=\beta^*\in C$. In view of Lemma \ref{lem26}, we get $d(C)=0$. By the Skolem-Noether theorem, the extension of $d$ to $RC$ is inner. That is, there exists $b\in RC\setminus C$ such that $d(x)=[b, x]$ for all $x\in R$. By (ii), it follows that $[b, T(R)]=0$. (iii) $\Rightarrow$ (iv):\ In view of Lemma \ref{lem8}, $T(R)^2$ is a Lie ideal of $R$. Since $[b, T(R)]=0$, we have $[b, T(R)^2]=0$. If $\text{\rm char}\,R\ne 2$, by Lemma \ref{lem2} we see that $T(R)^2\subseteq Z(R)$. Hence, by Lemmas \ref{lem3} and \ref{lem6}, we get $\text{\rm char}\,R=2$, a contradiction. Therefore, $\text{\rm char}\,R=2$ and so $R$ is an exceptional prime ring. (iv) $\Rightarrow$ (v):\ In view of Lemmas \ref{lem3} and \ref{lem6}, it follows that $T(R)^2\subseteq Z(R)$. (v) $\Rightarrow$ (i):\ Let $t_1, t_2\in T(R)\setminus \{0\}$. Since $T(R)^2\subseteq Z(R)$, we have $ 0=[t_1t_2, t_2]=[t_1, t_2]t_2, $ and so $[t_1, t_2]t_2T(R)=0$. Note that $0\ne t_2T(R)\subseteq Z(R)$ (see Lemma \ref{lem4}). Thus $[t_1, t_2]=0$. Therefore, $[T(R), T(R)]=0$. \end{proof} \begin{cor}\label{cor5} Let $d$ be a nonzero derivation of $R$. If $d(T(R))=0$, then $T(R)^2\subseteq Z(R)$. \end{cor} \begin{proof} If $T(R)\subseteq Z(R)$, it is clear that $T(R)^2\subseteq Z(R)$. Suppose that $T(R)\nsubseteq Z(R)$. Since $d(T(R))=0$, it follows from Theorem \ref{thm2} that $T(R)^2\subseteq Z(R)$, as desired. \end{proof} Given a subset $A$ of a ring $R$, let $\mathfrak{C}_R(A)$ denote the {\it centralizer} of $A$ in $R$, that is, $$ \mathfrak{C}_R(A)=\{x\in R\mid xa=ax\ \forall a\in A\}. $$ A prime ring $R$ is called {\it centrally closed} if $R=RC$. The following corollary itself is interesting. \begin{cor}\label{cor1} If $R$ is centrally closed, then $\mathfrak{C}_R(T(R))$ is equal to $R$, $Z(R)$, or $T(R)+Z(R)$. \end{cor} \begin{proof} Clearly, if $T(R)\subseteq Z(R)$, then $\mathfrak{C}_R(T(R))=R$. Assume that $T(R)\nsubseteq Z(R)$. If $\mathfrak{C}_R(T(R))\subseteq Z(R)$, then $\mathfrak{C}_R(T(R))=Z(R)$. Hence we assume that $\mathfrak{C}_R(T(R))\nsubseteq Z(R)$. That is, there exists $b\in R\setminus Z(R)$ such that $[b, T(R)]=0$. In view Theorem \ref{thm2} (iv), $R$ is exceptional and $*$ is of the first kind. In this case, $Z(R)=C$. Also, one of the following holds: Case 1:\ $R\cong \text{\rm M}_2(C)$. Since $T(R)\nsubseteq Z(R)$, $*$ is of the transpose type on $R$. As given in Lemma \ref{lem3}, $$ T(R)=\{\left[ \begin{array}{cc} 0 & \gamma \\ \pi^{-1}\gamma& 0\end{array}\right]\mid \gamma\in C\}. $$ Let $x\in R$. Then $x\in \mathfrak{C}_R(T(R))$ iff $\big[x, \left[ \begin{array}{cc} 0 & 1\\ \pi^{-1}&0\end{array}\right]\big]=0$. A direct computation shows that $x\in T(R)+C$. That is, $ \mathfrak{C}_R(T(R))=T(R)+Z(R)$. Case 2:\ $R$ is a $4$-dimensional central division algebra, and $\text{\rm char}\,R=2$. Since $T(R)\nsubseteq Z(R)$, $*$ is of the transpose type on $R$. Let $F$ be a maximal subfield of $R$, and let $\widetilde R:=R\otimes_{Z(R)}F\cong \text{\rm M}_2(F)$. As given in Case 1, we have $$ \mathfrak{C}_{\widetilde R}(T({\widetilde R}))=T({\widetilde R})+Z({\widetilde R})=T(R)\otimes F+Z(R)\otimes F. $$ On the other hand, $ \mathfrak{C}_{\widetilde R}(T({\widetilde R}))= \mathfrak{C}_{\widetilde R}(T(R)\otimes F)=\mathfrak{C}_R(T(R))\otimes F. $ Hence we get $\mathfrak{C}_R(T(R))=T(R)+Z(R)$. \end{proof} \begin{lem}\label{lem9} Let $R$ be a prime ring with a Lie ideal $L$. Then $\mathfrak{C}_R(L)=R$, $\mathfrak{C}_R(L)=Z(R)$, or both $R$ is exceptional and $LC=[a, RC]=Ca+C$ for any $a\in L\setminus Z(R)$. \end{lem} \begin{proof} Let $x\in \mathfrak{C}_R(L)$ and $r\in R$. Then $[x, L]=0$, $[r, L]\subseteq L$ and so $$ \big[[x, r], L\big]\subseteq \big[[x, L], r\big]+\big[x, [r, L]\big]\subseteq \big[x, L\big]=0. $$ That is, $[\mathfrak{C}_R(L), R]\subseteq \mathfrak{C}_R(L)$, proving that $\mathfrak{C}_R(L)$ is a Lie ideal of $R$. Clearly, $[\mathfrak{C}_R(L), L]=0$. In view of Lemma \ref{lem5} (iv) and \cite[Lemma 6.1]{lee2025}, $\mathfrak{C}_R(L)\subseteq Z(R)$, $L\subseteq Z(R)$, or both $R$ is exceptional and $LC=[a, RC]=Ca+C$ for any $a\in L\setminus Z(R)$. The first two cases imply that $\mathfrak{C}_R(L)=Z(R)$ and $\mathfrak{C}_R(L)=R$, respectively. The proof is now complete. \end{proof} \begin{cor}\label{cor3} If $R$ is centrally closed, then $\mathfrak{C}_R(T(R)^2)$ is equal to either $R$ or $Z(R)$. \end{cor} \begin{proof} In view of Lemma \ref{lem8}, $T(R)^2$ is a Lie ideal of $R$. By Lemma \ref{lem9}, we have $\mathfrak{C}_R(T(R)^2)=R$, $\mathfrak{C}_R(T(R)^2) =Z(R)$, or both $R$ is exceptional and $T(R)^2C=[a, RC]=Ca+C$ for any $a\in T(R)^2\setminus C$. Assume the third case. Then $\dim_CT(R)^2C=2$ and $T(R)\nsubseteq Z(R)$. In view Theorem \ref{thm2}, $*$ is of the second kind. There exists $\beta\in C$ such that $\beta\ne \beta^*$. Let $I$ be a nonzero $*$-ideal of $R$ such that $\beta I\subseteq R$. Then, for $x\in I$, $$ (\beta^*-\beta)T(R)x=T(R)\big(\beta^*(x+x^*)-(\beta x+(\beta x)^*)\big)\subseteq T(R)^2C. $$ Applying the same argument, we get $(\beta^*-\beta)^2yx\subseteq T(R)^2C$ for all $x, y\in I$. Hence $I^2C \subseteq T(R)^2C$. Note that $I^2C=R$. Therefore $T(R)^2C=R$, a contradiction. This completes the proof. \end{proof} \section{Theorem B} In this section we always assume that {\it $R$ is a prime ring with involution $*$}. Relative to Theorem \ref{thm2}, we have another corresponding skew trace result, i.e., Theorem B mentioned in the introduction. \begin{thm}\label{thm4} If $K_0(R)\nsubseteq Z(R)$, then the following are equivalent: (i)\ $[K_0(R), K_0(R)]=0$; (ii)\ $d(K_0(R))=0$ for some nonzero derivation $d$ of $R$; (iii)\ $[b, K_0(R)]=0$ for some $b\in RC\setminus C$; (iv)\ $K_0(R)^2\subseteq Z(R)$; (v)\ $\dim_CRC=4$, $*$ is of the transpose type, and $*$ is of the first kind. \end{thm} \begin{proof} We first consider the case that $\text{\rm char}\,R=2$. In this case, we have $T(R)=K_0(R)$. It follows from Theorem \ref{thm2} that (i)--(iv) are equivalent. Moreover, by Lemma \ref{lem12}, $*$ is of the first kind. Since $T(R)=K_0(R)\nsubseteq Z(R)$, it follows that $*$ is of the transpose type. So (i) implies (v). By Theorem \ref{thm2}, (v) also implies (i) and hence we are done in this case. Hence we always assume that $\text{\rm char}\,R\ne 2$. In view of Lemma \ref{lem1}, if one of (i)--(iv) is satisfied, then $\dim_CRC=4$. (i) $\Rightarrow$ (ii):\ Since $K_0(R)\nsubseteq Z(R)$, choose $b\in K_0(R)\setminus Z(R)$. Let $d(x)=[b, x]$ for $x\in R$. Then $d$ is a nonzero derivation of $R$ and $d(K_0)=0$. (ii) $\Rightarrow$ (iii):\ We have a similar argument as given in the proof of ``(ii) $\Rightarrow$ (iii)'' of Theorem \ref{thm2}. (iii) $\Rightarrow$ (iv):\ Note that $K_0(R)^2$ is a Lie ideal of $R$ (see Lemma \ref{lem8}). Since $[b, K_0(R)]=0$, we get $[b, K_0(R)^2]=0$. In view of Lemma \ref{lem2}, it follows that $K_0(R)^2\subseteq Z(R)$, as desired. (iv) $\Rightarrow$ (i):\ Let $k_1, k_2\in K_0(R)\setminus \{0\}$. Then $[k_1k_2, k_2]=0$ and so $[k_1, k_2]k_2K_0(R)=0$. In view of Lemma \ref{lem4}, $0\ne k_2K_0(R)\subseteq Z(R)$. We conclude that $[k_1, k_2]=0$, as desired. Up to now, we have proved that (i)--(iv) are equivalent. In this case, it follows from Lemma \ref{lem12} that $*$ is of the first kind. (iv) $\Rightarrow$ (v):\ Suppose on the contrary that $*$ is of the symplectic type on $RC$. Hence, by Lemma \ref{lem7} (i) and Case 2 of Lemma \ref{lem6}, $K_0(R)^2\subseteq Z(R)$ iff $\text{\rm char}\,R=2$, a contradiction. Thus $*$ is of the transpose type. (v) $\Rightarrow$ (iv):\ By Lemma \ref{lem3} and Case 1(ii) of Lemma \ref{lem6}, we get $K_0(R)^2\subseteq C$. \end{proof} \begin{cor}\label{cor6} Let $d$ be a nonzero derivation of $R$. If $d(K_0(R))=0$, then $K_0(R)^2\subseteq Z(R)$. \end{cor} \begin{cor}\label{cor2} If $R$ is centrally closed, then $\mathfrak{C}_R(K_0(R))$ is equal to $R$, $Z(R)$, or $K_0(R)+Z(R)$. \end{cor} \begin{proof} Clearly, if $K_0(R)\subseteq Z(R)$, then $\mathfrak{C}_R(K_0(R))=R$. Assume that $K_0(R)\nsubseteq Z(R)$. If $\mathfrak{C}_R(K_0(R))\subseteq Z(R)$, then $\mathfrak{C}_R(K_0(R))=Z(R)$. Hence we assume that $\mathfrak{C}_R(K_0(R))\nsubseteq Z(R)$. That is, there exists $b\in R\setminus Z(R)$ such that $[b, K_0(R)]=0$. In view Theorem \ref{thm4} (v), $\dim_CR=4$, $*$ is of the transpose type on $R$, and $*$ is of the first kind. In this case, $Z(R)=C$. Also, one of the following holds: Case 1:\ $R\cong \text{\rm M}_2(C)$. As given in Lemma \ref{lem3}, $$ K_0(R)=\{\left[ \begin{array}{cc} 0 & \gamma \\ -\pi^{-1}\gamma & 0\end{array}\right]\mid \gamma\in C\}. $$ Let $x\in R$. Then $x\in \mathfrak{C}_R(K_0(R))$ iff $\big[x, \left[ \begin{array}{cc} 0 & 1\\ -\pi^{-1}&0\end{array}\right]\big]=0$. A direct computation shows that $x\in K_0(R)+C$. That is, $ \mathfrak{C}_R(K_0(R))=K_0(R)+Z(R)$. Case 2:\ $R$ is a $4$-dimensional central division algebra. Note that $*$ is of the transpose type on $R$. Let $F$ be a maximal subfield of $R$, and let $\widetilde R:=R\otimes_{Z(R)}F\cong \text{\rm M}_2(F)$. As given in Case 1, we have $$ \mathfrak{C}_{\widetilde R}(K_0({\widetilde R}))=K_0({\widetilde R})+Z({\widetilde R})=K_0(R)\otimes F+Z(R)\otimes F. $$ On the other hand, $ \mathfrak{C}_{\widetilde R}(K_0({\widetilde R}))= \mathfrak{C}_{\widetilde R}(K_0(R)\otimes F)=\mathfrak{C}_R(K_0(R))\otimes F. $ Hence we get $\mathfrak{C}_R(K_0(R))=K_0(R)+Z(R)$. \end{proof} \begin{cor}\label{cor4} If $R$ is centrally closed, then $\mathfrak{C}_R(K_0(R)^2)$ is equal to either $R$ or $Z(R)$. \end{cor} \begin{proof} In view of Lemma \ref{lem8}, $K_0(R)^2$ is a Lie ideal of $R$. By Lemma \ref{lem9}, we have $\mathfrak{C}_R(K_0(R)^2)=R$, $\mathfrak{C}_R(K_0(R)^2) =Z(R)$, or both $R$ is exceptional and $K_0(R)^2C=[a, RC]=Ca+C$ for any $a\in K_0(R)^2\setminus C$. Assume the third case, implying that $\dim_CK_0(R)^2C=2$ and $K_0(R)\nsubseteq Z(R)$. Suppose on the contrary that $*$ is of the second kind. There exists $\beta\in C$ such that $\beta\ne \beta^*$. Let $I$ be a nonzero $*$-ideal of $R$ such that $\beta I\subseteq R$. Then, for $x\in I$, $$ (\beta^*-\beta)K_0(R)x=K_0(R)\big(\beta^*(x-x^*)-(\beta x-(\beta x)^*)\big)\subseteq K_0(R)^2C. $$ Applying the same argument, we get $(\beta^*-\beta)^2yx\subseteq K_0(R)^2C$ for all $x, y\in I$. Hence $I^2C \subseteq K_0(R)^2C$. Note that $I^2C=R$. Therefore $K_0(R)^2C=R$, a contradiction. This proves that $*$ is of the first kind. If $*$ is of the transpose type on $R$, it follows from Theorem \ref{thm4} that $K_0(R)^2\subseteq C$, a contradiction. Thus $*$ is of the symplectic type. By Lemmas \ref{lem7} and \ref{lem6}, it follows that $\text{\rm char}\,R\ne 2$, a contradiction. This completes the proof. \end{proof} \section{Theorem C} Let $R$ be a prime ring with involution $*$. We say that $R$ satisfies a $*$-generalized polynomial $f$ if $f(X_1,\ldots,X_n, Y_1,\ldots,Y_n)$ is a generalized polynomial in noncommutative variables $X_i$ and $Y_i$, $1\leq i\leq n$, with coefficients in $Q_s(R)$ and $ f(x_1,\ldots,x_n, x_1^*,\ldots,x_n^*)=0 $ for all $x_i\in R$ (see \cite{chuang1989}). We also say that $R$ satisfies the $*$-generalized polynomial $f(X_1,\ldots,X_n, X_1^*,\ldots,X_n^*)$. If $f$ has coefficients in $C$, we say that $R$ satisfies the $*$-polynomial $f(X_1,\ldots,X_n, X_1^*,\ldots,X_n^*)$. \begin{pro} ( \cite[Proposition 4]{chuang1989}) \label{pro1} Suppose that $R$ is a prime ring with involution $*$, and that $f$ is a $*$-generalized polynomial. If $f$ vanishes on a nonzero ideal of $R$, then $f$ vanishes on $Q_s(R)$. \end{pro} To keep the statements of our theorems below neat, throughout this section, we always make the following assumption:\vskip4pt {\it Let $R$ be a division ring with involution $*$ and let $M$ be a noncentral $*$-subring of $R$ such that $M$ is invariant under all inner automorphisms of $R$.}\vskip4pt We begin with the following. \begin{thm}\label{thm9} If $M$ is a PI-ring, then the following hold: (i)\ $R=MZ(R)$, where $Z(R)$ is the quotient field of $Z(M)$; (ii)\ If $R$ is equipped with involution $*$, then $R$ and $M$ satisfy the same $*$-generalized polynomials with coefficients in $R$. \end{thm} \begin{proof} (i)\ We claim that $[M, M]\ne 0$. Otherwise, $[M, M]=0$. Let $m\in M$ and $u\in R\setminus \{0, -1\}$. Then there exist $m_1, m_2\in M$ such that $$ um=m_1u\ \ \text{\rm and}\ \ (1+u)m=m_2(1+u). $$ Then $m=(m_2-m_1)u+m_2$ and so $(m_2-m_1)[m, u]=0$. Therefore, either $m_1=m_2$ or $[m, u]=0$. The former case implies that $m=m_2$ and so $[m, u]=0$. This implies that $m\in Z(R)$. Thus $M\subseteq Z(R)$, a contradiction. By \cite[Theorem 2 and Corollary 1]{rowen1973}, $Z(M)\ne 0$, $MC_M$ is a finite-dimensional central simple $C_M$-algebra and $Z(MC_M)=C_M$, where $C_M$ is the quotient field of $Z(M)$. In this case, $MC_M$ is a subdivision ring of $R$. Moreover, $uC_Mu^{-1}\subseteq C_M$ for all $0\ne u\in R$, implying that $MC_M$ is invariant under all inner automorphisms of $R$. Since $[M, M]\ne 0$, $MC_M$ is a noncentral subdivision ring of $R$, it follows from the Brauer-Cartan-Hua theorem that $R=MC_M$. Clearly, $C_M=Z(R)$ and so $Z(R)$ is the quotient field of $Z(M)$. (ii)\ It follows directly from (i) and Proposition \ref{pro1}. \end{proof} \begin{thm}\label{thm10} If $ [T(M), T(M)]=0\ \ (resp.\ [K_0(M), K_0(M)]=0), $ then $T(R)^2\subseteq Z(R)$ (resp. $K_0(R)^2\subseteq Z(R)$). \end{thm} \begin{proof} Assume that \begin{eqnarray} [x+x^*, y+y^*]=0, \label{eq:3} \end{eqnarray} for all $x, y\in M$. Since the prime ring $M$ satisfies the $*$-polynomial $[X+X^*, Y+Y^*]$, in view of \cite[Theorem 1]{amitsur1969}, $M$ is a PI-ring. It follows from Theorem \ref{thm9} (ii) that Eq.\eqref{eq:3} holds for all $x\in R$. That is, $[T(R), T(R)]=0$. In view of Theorem \ref{thm2}, either $T(R)\subseteq Z(R)$ or $T(R)^2\subseteq Z(R)$. In either case, $T(R)^2\subseteq Z(R)$. A similar argument can be applied to proving that $[K_0(R), K_0(R)]=0$ implies $K_0(R)^2\subseteq Z(R)$ by Theorem \ref{thm4}. Hence the proof is complete. \end{proof} The following generalizes the division case of Theorems \ref{thm33} and Theorems \ref{thm34}. \begin{cor}\label{cor10} (i)\ If $ [T(M), T(M)]=0, $ then $\dim_{Z(R)}R=4$ and $*$ is of the first kind. (ii)\ If $ [K_0(M), K_0(M)]=0 $ and $\text{\rm char}\,R\ne 2$, then $\dim_{Z(R)}R=4$, $*$ is of the transpose type, and $*$ is of the first kind. \end{cor} \begin{proof} (i)\ In view of Theorem \ref{thm10}, we have $T(R)^2\subseteq Z(R)$ and hence $\dim_{Z(R)}R=4$ (see Lemma \ref{lem1} (ii)). If $T(R)\subseteq Z(R)$, then $*$ is of the first kind (see the argument below Definition \ref{def4}). Suppose next that $T(R)\nsubseteq Z(R)$. It follows from Theorem \ref{thm2} (iv) that $*$ is of the first kind. (ii)\ In view of Theorem \ref{thm10}, we have $K_0(R)^2\subseteq Z(R)$ and hence $\dim_{Z(R)}R=4$ (see Lemma \ref{lem1} (ii)). If $K_0(R)\subseteq Z(R)$, then, by Corollary \ref{cor8}, $T(R)\subseteq Z(R)$ and $\text{\rm char}\,R=2$, a contradiction. Thus $K_0(R)\nsubseteq Z(R)$. It follows from Theorem \ref{thm4} (v) that $*$ is of the transpose type and $*$ is of the first kind. \end{proof} The following is the inner case of Theorem C. \begin{thm}\label{thm13} If $ \mathfrak{C}_R(T(M))\nsubseteq Z(R) $ (resp.\ $\mathfrak{C}_R(K_0(M))\nsubseteq Z(R))$, then $T(R)^2\subseteq Z(R)$ (resp. $K_0(R)^2\subseteq Z(R)$). \end{thm} \begin{proof} We only prove the trace case. The skew trace case is similar by a slight modification. Since $\mathfrak{C}_R(T(M))\nsubseteq Z(R)$, there exists $b\in R\setminus Z(R)$ such that $[b, x+x^*]=0$ for all $x\in M$. Then $[b, T(M)^2]=0$. Note that $T(M)^2$ is a Lie ideal of $M$ (see Lemma \ref{lem8}). We claim that $[T(M)^2, T(M)^2]=0$. Suppose not. By Lemma \ref{lem5} (ii), we have $$ I:=M[T(M)^2, T(M)^2]M\subseteq T(M)^2+T(M)^4 $$ is a nonzero ideal of $M$. Then $[b, I]=0$. Since $[b, IM]=0$, we get $I[b, M]=0$ and so $[b, M]=0$. Note that $\mathfrak{C}_R(M)$ is a subdivision ring of $R$ and $u\mathfrak{C}_R(M)u^{-1}\subseteq \mathfrak{C}_R(M)$ for all $u\in R^\times$. By the Brauer-Cartan-Hua theorem, either $\mathfrak{C}_R(M)\subseteq Z(R)$ or $\mathfrak{C}_R(M)=R$. The latter case implies that $M$ is central, a contradiction. Thus $\mathfrak{C}_R(M)\subseteq Z(R)$ and so $b\in Z(R)$, a contradiction. This proves our claim. Therefore, $M$ satisfies a nontrivial $*$-polynomial and so $M$ is a PI-ring (see \cite[Theorem 1]{amitsur1969}). In view of Theorem \ref{thm9} (ii), we get $[b, T(R)]=0$ since $[b, x+x^*]=0$ for all $x\in M$. We are now done by Theorem \ref{thm2}. \end{proof} The following is Theorem C mentioned in the introduction. \begin{thm}\label{thm14} If $d$ is a nonzero derivation of $R$ such that $ d(T(M))=0 $ (resp.\ $d(K_0(M))=0$), then $T(R)^2\subseteq Z(R)$ (resp. $K_0(R)^2\subseteq Z(R)$). \end{thm} \begin{proof} Assume that $d(T(M))=0$. Then $d(T(M)^4)=0$. Since $T(M)^2$ is a Lie ideal of the prime ring $M$ (see Lemma \ref{lem8}), it follows from \cite[Theorem 1.6]{lee2025} that either $T(M)^4$ contains a nonzero ideal, say $I$, of $M$ or $[T(M)^2, T(M)^2]=0$. Case 1:\ $T(M)^4$ contains a nonzero ideal $I$ of $M$. Then $d(I)=0$ and so $d(IM)=0$. This implies that $Id(M)=0$ and so $d(M)=0$. Let $m\in M$ and $u\in R^\times$. Then $umu^{-1}\in M$ and so $d(umu^{-1})=0$. Therefore, $d(u)mu^{-1}+umd(u^{-1})=0$. Note that $d(u^{-1})=-u^{-1}d(u)u^{-1}$. We see that $[u^{-1}d(u), m]=0$. That is, $[u^{-1}d(u), M]=0$. In particular, $[u^{-1}d(u), T(M)]=0$. If $u^{-1}d(u)\notin Z(R)$ for some $u\in R^\times$, it follows from Theorem \ref{thm13} that $T(R)^2\subseteq Z(R)$. Suppose next that $u^{-1}d(u)\in Z(R)$ for all $u\in R^\times$. Since $R$ is a division ring, we get $[x, d(x)]=0$ for all $x\in R$. By Posner's theorem (see \cite[Theorem 2]{posner1957}), either $d=0$ or $R$ is commutative, a contradiction. Case 2:\ $[T(M)^2, T(M)^2]=0$. That is, $M$ satisfies the $*$-polynomial $$ f:=\big[(X_1+X_1^*)(X_2+X_2^*), (Y_1+Y_1^*)(Y_2+Y_2^*)\big]. $$ In view of \cite[Theorem 1]{amitsur1969}, $M$ is a PI-ring. In view of Theorem \ref{thm9} (ii), $R$ also satisfies the $*$-polynomial $f$. Hence $R$ is a division PI-ring, implying $\dim_{Z(R)}R<\infty$. Let $\beta\in Z(M)$ and $x\in M$. Then $d(x+x^*)=0$, $d(\beta+\beta^*)=0$, and so \begin{eqnarray} 0 &=&d(\beta x+(\beta x)^*)\nonumber \nonumber \\ &=&d(\beta)x+\beta d(x)+\beta^*d(x^*)+d(\beta^*)x \nonumber \\ &=&(\beta-\beta^*)d(x).\nonumber \end{eqnarray} Thus, either $\beta=\beta^*$ or $d(x)=0$. If $d(M)=0$, then, by applying the same argument in Case 1, we are done in this case. Hence we may assume that $\beta=\beta^*$ for all $\beta\in Z(M)$. Let $\beta\in Z(M)$ and $x\in M$. Then $d(\beta x+(\beta x)^*)=0$ and so $d(\beta(x+x^*))=0$, implying $d(\beta)(x+x^*)=0$. That is, $d(\beta)T(M)=0$. Note that $T(M)\ne 0$ since $M$ is noncentral. So $d(\beta)=0$. Thus $d=0$ on $Z(M)$. This implies that $d=0$ on $Z(R)$ as $Z(R)$ is the quotient field of $Z(M)$. By the Skolem-Noether theorem, $d$ is inner on $R$. By Theorem \ref{thm13}, $T(R)^2\subseteq Z(R)$. This completes the trace case. The skew trace case has almost the same argument by a slight modification. \end{proof} \section{Lie ideals: Theorem D} We begin with the case of ideals. \begin{thm}\label{thm17} Let $R$ be a noncommutative prime ring with involution $*$, $d$ a nonzero derivation of $R$, and $I$ a nonzero ideal of $R$. If $d(T(I))=0$ (resp. $d(K_0(I))=0$), then $T(R)^2\subseteq Z(R)$ (resp. $K_0(R)^2\subseteq Z(R)$). \end{thm} \begin{proof} Replacing $I$ by $I\cap I^*$, we may assume that $I$ is a nonzero $*$-ideal of $R$. Case 1: Assume that $d(T(I))=0$. Let $x\in I$ and $w\in T(I)$. Then $$ xw\in I, w=w^*,d(x^*)=-d(x), d(w)=0\ \text{\rm and}\ d(xw+(xw)^*)=0. $$ We have \begin{eqnarray*} 0 &=&d(xw+(xw)^*)\nonumber\\ &=&d(x)w+xd(w)+w^*d(x^*)+d(w^*)x^*\nonumber\\ &=&d(x)w-wd(x).\nonumber \end{eqnarray*} That is, $[d(I), y+y^*]=0$ for all $y\in I$. Applying Proposition \ref{pro1}, we get $[d(I), y+y^*]=0$ for all $y\in R$. In view of Theorem \ref{thm2}, either $d(I)\subseteq Z(R)$ or $T(R)^2\subseteq Z(R)$. If $d(I)\subseteq Z(R)$, then, by \cite[Theorem 2]{lee1986}, either $d=0$ or $R$ is commutative, a contradiction. Thus $T(R)^2\subseteq Z(R)$. Case 2: Assume that $d(K_0(I))=0$. By Case 1, we may assume that $\text{\rm char}\,R\ne 2$. Let $x\in I$ and $w\in K_0(I)$. Then $$ xw\in I, w^*=-w, d(x^*)=d(x), d(w)=0\ \text{\rm and}\ d(xw-(xw)^*)=0. $$ We have \begin{eqnarray*} 0 &=&d(xw-(xw)^*)\nonumber\\ &=&d(x)w+xd(w)-w^*d(x^*)-d(w^*)x^*\nonumber\\ &=&d(x)w+wd(x).\nonumber \end{eqnarray*} That is, $[d(I), w_1w_2]=0$ for all $w_1, w_2\in K_0(I)$. It follows from \cite[Theorem 4]{bergen1981} that $w_1w_2\in Z(R)$ for all $w_1, w_2\in K_0(I)$. That is, $K_0(I)^2\subseteq Z(R)$. Applying Proposition \ref{pro1}, we get $K_0(R)^2\subseteq Z(R)$, as desired. \end{proof} Herstein proved that, in a prime ring $R$ containing a nontrivial idempotent, every noncentral subring invariant under all special inner automorphisms of $R$ contains a nonzero ideal (see \cite[Theorem, p.26]{herstein1983}). Hence the following is an immediate consequence of Theorem \ref{thm17} (cf. Corollary \ref{cor7}). \begin{thm} \label{thm19} Let $R$ be a prime ring with involution $*$, possessing a nontrivial idempotent. Suppose that $M$ is a noncentral subring of $R$, which is invariant under all special inner automorphisms of $R$. If $d$ is a nonzero derivation of $R$ such that $d(T(M))=0$ (resp. $d(K_0(M))=0$), then $T(R)^2\subseteq Z(R)$ (resp. $K_0(R)^2\subseteq Z(R)$) except when $R\cong \text{\rm M}_2(\text{\rm GF}(2))$. \end{thm} The following generalizes the non division case of Theorems \ref{thm33} and Theorems \ref{thm34}. \begin{cor}\label{cor11} Let $R$ be a prime ring with involution $*$, possessing a nontrivial idempotent, and $R\ncong \text{\rm M}_2(\text{\rm GF}(2))$. Suppose that $M$ is a noncentral subring of $R$, which is invariant under all special inner automorphisms of $R$. (i)\ If $ [T(M), T(M)]=0, $ then $\dim_CRC=4$ and $*$ is of the first kind. (ii)\ If $ [K_0(M), K_0(M)]=0 $ and $\text{\rm char}\,R\ne 2$, then $\dim_CRC=4$, $*$ is of the transpose type, and $*$ is of the first kind. \end{cor} \begin{proof} In view of \cite[Theorem, p.26]{herstein1983}, $M$ contains a nonzero ideal $I$ of $R$. Replacing $I$ by $I\cap I^*$, we may assume that $I$ is a nonzero $*$-ideal of $R$. (i)\ Since $ [T(M), T(M)]=0, $ it follows that $[T(I), T(I)]=0$. By Proposition \ref{pro1}, we get $[T(R), T(R)]=0$. This implies that $\dim_CRC=4$ (see Lemma \ref{lem1} (ii) and Theorem \ref{thm2} (iv)). If $T(R)\nsubseteq Z(R)$, then it follows from Theorem \ref{thm2} (iv) that $*$ is of the first kind. Suppose that $T(R)\subseteq Z(R)$. By Lemma \ref{lem7}, $*$ is of the symplectic type and hence is of the first kind. (ii)\ Since $ [K_0(M), K_0(M)]=0, $ it follows that $[K_0(I), K_0(I)]=0$. By Proposition \ref{pro1}, we get $[K_0(R), K_0(R)]=0$. If $K_0(R)\subseteq Z(R)$, then, by Corollary \ref{cor8}, $T(R)\subseteq Z(R)$ and $\text{\rm char}\,R=2$, a contradiction. Thus $K_0(R)\nsubseteq Z(R)$. It follows from Theorem \ref{thm4} (v) that $\dim_CRC=4$, $*$ is of the transpose type and $*$ is of the first kind. \end{proof} We now turn to the case of Lie ideals. The following will play a key role in the proofs below. \begin{thm} (\cite[Theorem 5.2 Case 1]{lee2025})\label{thm23} Let $R$ be a prime ring, $L$ a non-abelian Lie ideal of $R$, and $a\in R\setminus Z(R)$. Then $L+aL$ contains a nonzero ideal of $R$. \end{thm} \begin{lem} \label{lem24} Let $R$ be a prime ring with involution $*$, $\text{\rm char}\,R=2$, and $b\in R\setminus Z(R)$. Suppose that $ \big[b, [x, y]+[x, y]^*\big]=0 $ for all $x, y\in R$. Then $T(R)^2\subseteq Z(R)$. \end{lem} \begin{proof} By hypothesis, we have \begin{eqnarray} \big[b, [x, y]+[x, y]^*\big]=0 \label{eq:5} \end{eqnarray} for all $x, y\in R$. Suppose first that $b^*=b$. Replacing $x$ by $b$ in Eq.\eqref{eq:5}, we get $ \big[b, [b, y]+[b, y^*]\big]=0 $ and hence $[b^2, y+ y^*]=0$ for all $y\in R$. That is, $[b^2, T(R)]=0$. In view of Theorem \ref{thm2}, either $T(R)^2\subseteq Z(R)$ or $b^2\in Z(R)$. Assume that $b^2\in Z(R)$. Thus $[b, z]b=b[b, z]$ for all $z\in R$. Note that $\big[[R, R], [R, R]\big]\ne 0$. In view of Theorem \ref{thm23}, $I\subseteq [R, R]+b[R, R]$ for some nonzero ideal $I$ of $R$. Let $x\in I$. Write $$ x=\sum_i[a_i, b_i]+\sum_jb[c_j, d_j] $$ for some finitely many $a_i, b_i, c_j, d_j\in R$. Then \begin{eqnarray*} [b, x+x^*]&=&\Big[b, \sum_i([a_i, b_i]+[a_i, b_i]^*)\Big]+\Big[b, \sum_jb[c_j, d_j]+\sum_j[c_j, d_j]^*b\Big]\\ &=&b\Big[b, \sum_j[c_j, d_j]\Big]+\Big[b, \sum_j[c_j, d_j]^*\Big]b\\ &=&b\Big[b, \sum_j[c_j, d_j]\Big]+b\Big[b, \sum_j[c_j, d_j]^*\Big]\\ &=&b\sum_j\Big[b,[c_j, d_j]+[c_j, d_j]^*\Big]\\ &=&0. \end{eqnarray*} That is, $[b, T(I)]=0$ and so, by Theorem \ref{thm17}, we get $T(R)^2\subseteq Z(R)$ in the case of $b^*=b$. We consider the general case. By Eq.\eqref{eq:5}, $\big[b+b^*, [x, y]+[x, y]^*\big]=0$ for all $x, y\in R$. By the case above, either $T(R)^2\subseteq Z(R)$ or $b+b^*\in Z(R)$. Assume that $b^*=b+\beta$. Then $b^*=b+\beta$ for some $\beta\in Z(R)$. It suffices to prove that $\beta=0$. Suppose on the contrary that $\beta\ne 0$. Replacing $x$ by $b$ in Eq.\eqref{eq:5}, we get $\big[b, [b, y]+[b^*, y^*]\big]=0$ and hence $\big[b, [b, y]+[b+\beta, y^*]\big]=0$, implying $[b^2, y+ y^*]=0$ for all $y\in R$. That is, $[b^2, T(R)]=0$. In view of Theorem \ref{thm2}, we have $b^2\in Z(R)$. Replacing $x$ by $bx$ in Eq.\eqref{eq:5}, we get \begin{eqnarray} \big[b, [bx, y]+[bx, y]^*\big]=0. \label{eq:6} \end{eqnarray} Expanding Eq.\eqref{eq:6}, we have \begin{eqnarray*} 0&=&\big[b, [b, y]x+b[x, y]+[x^*(b+\beta), y^*]\big]\\ &=&[b, y][b, x]+b\big[b, [x, y]\big]+\big[b, x^*[b, y^*]\big]+\big[b, [x^*, y^*]\big](b+\beta)\\ &=&[b, y][b, x]+b\big[b, [x, y]\big]+[b, x^*][b, y^*]+(b+\beta)\big[b, [x^*, y^*]\big]\\ &=&[b, y][b, x]+[b, x^*][b, y^*]+\beta\big[b, [x^*, y^*]\big]+b\Big(\big[b, [x, y]+[x^*, y^*]\big]\Big)\\ &=&[b, y][b, x]+[b, x^*][b, y^*]+\beta\big[b, [x^*, y^*]\big].\\ \end{eqnarray*} That is, \begin{eqnarray} [b, y][b, x]+[b, x^*][b, y^*]+\beta\big[b, [x^*, y^*]\big]=0 \label{eq:7} \end{eqnarray} for all $x, y\in R$. Replacing $y$ by $by$ in Eq.\eqref{eq:7}, we get \begin{eqnarray} [b, by][b, x]+[b, x^*][b, y^*(b+\beta)]+\beta\big[b, [x^*, y^*(b+\beta)]\big]=0. \label{eq:8} \end{eqnarray} Note that $[b, x^*][b, y^*(b+\beta)]=(b+\beta)[b, x^*][b, y^*]$ and $$ \big[b, [x^*, y^*(b+\beta)]\big]=(b+\beta)\big[b, [x^*, y^*]\big]+[b, y^*][x^*, b]. $$ Thus it follows from Eq.\eqref{eq:8} that \begin{eqnarray*} b[b, y][b, x]+(b+\beta)[b, x^*][b, y^*]+\beta(b+\beta)\big[b, [x^*, y^*]\big]+\beta[b, y^*][x^*, b]=0 \end{eqnarray*} and so \begin{eqnarray*} &&b\Big([b, y][b, x]+[b, x^*][b, y^*] +\beta\big[b, [x^*, y^*]\big]\Big)\\ &&\ \ \ \ \ \ \ \ \ \ \ \ \ \ +\beta\Big([b, x^*][b, y^*]+[b, y^*][x^*, b]\Big)+\beta^2\big[b, [x^*, y^*]\big]=0. \end{eqnarray*} By Eq.\eqref{eq:7}, we get $[b, x^*][b, y^*]+[b, y^*][x^*, b]+\beta\big[b, [x^*, y^*]\big]=0$ and hence \begin{eqnarray} [b, x][b, y]+[b, y][b, x]+\beta\big[b, [x, y]\big]=0 \label{eq:9} \end{eqnarray} for all $x, y\in R$. Replacing $x$ by $bx$ in Eq.\eqref{eq:9}, we get $$ b\Big([b, x][b, y]+[b, y][b, x]+\beta\big[b, [x, y]\big]\Big)+\beta [b, y][b,x]=0 $$ and so $[b, y][b,x]=0$ for all $x, y\in R$. The primeness of $R$ implies that $b\in Z(R)$, a contradiction. \end{proof} \begin{lem}\label{lem14} Let $R$ be a noncommutative prime ring with involution $*$, $\text{\rm char}\,R=2$, and $d$ a nonzero derivation of $R$. If $d(T(R))\subseteq Z(R)$, then $*$ is of the first kind. \end{lem} \begin{proof} Suppose that $*$ is of the second kind. Let $\beta\in C$ be such that $\beta\ne \beta^*$. Choose a nonzero $*$-ideal $I$ of $R$ such that $\beta^2I\subseteq R$. Then $d(\beta^2)=0$ and $d((\beta^*)^2)=0$. Let $x\in I$. We compute \begin{eqnarray*} &&d(\beta^2x+(\beta^2 x)^*)\\ &=& d(\beta^2 (x+x^*)+((\beta^2)^*+\beta^2)x^*)\\ &=& \beta^2 d(x+x^*)+d(\beta^2)(x+x^*)+d((\beta^*)^2+\beta^2)x^*+((\beta^*)^2+\beta^2)d(x^*)\\ &=& \beta^2 d(x+x^*)+((\beta^*)^2+\beta^2)d(x^*)\in Z(R) \end{eqnarray*} and so $((\beta^*)^2+\beta^2)d(x^*)\in Z(R)$. Since $(\beta^*)^2+\beta^2\ne 0$, we get $d(x^*)\in Z(R)$ for all $x\in I$. As $I=I^*$, we have $d(I)\subseteq Z(R)$, implying that $R$ is commutative (see \cite[Theorem 2]{lee1986}), a contradiction. This completes the proof. \end{proof} \begin{lem}\label{lem25} Let $R$ be a ring with involution $*$, and let $L$ be a Lie ideal of $R$. Then $[K_0(L), K_0(R)] \subseteq K_0(L)$. \end{lem} \begin{proof} Let $u \in L$ and $r \in R$. Then $$ [u-u^{\ast}, r-r^{\ast}] = ([u,r]-[u,r]^{\ast})-([u,r^{\ast}]-[u,r^{\ast}]^{\ast}) \in K_0(L). $$ Thus $[K_0(L), K_0(R)] \subseteq K_0(L)$, as desired. \end{proof} The following is Theorem D. \begin{thm}\label{thm20} Let $R$ be a prime ring with involution $*$, $L$ a non-abelian Lie ideal of $R$, and $d$ a nonzero derivation of $R$. If $d(T(L))=0$, then $T(R)^2\subseteq Z(R)$. \end{thm} \begin{proof} Case 1:\ $\text{\rm char}\,R\ne 2$. Let $u\in L$ and $t\in T(L)$. Then $[u, t]\in L$, $[u, t]^*=[t, u^*]$ and $d(t)=0$. Thus \begin{eqnarray*} 0&=&d([u, t]+[u, t]^*)\\ &=&d([u, t]+[t, u^*])\\ &=&[d(u), t]+[t, d(u^*)]\\ &=&[d(u), t]-[t, d(u)]\\ &=&2[d(u), t]. \end{eqnarray*} That is, $ \big[T(L), d(L)\big]=0. $ In view of \cite[Theorem 2]{lee1983}, $T(L)\subseteq Z(R)$. Let $u\in L$ and $r\in R$. Then $u+u^*\in Z(R)$ and $$ [u, r]+[u, r]^*=[u, r]+[r^*, u^*]=[u, r]-[r^*, u]=[u, r+r^*]\in Z(R), $$ implying $[L, T(R)]=0$. Since $L$ is noncentral, it follows from Corollary \ref{cor5} that $T(R)^2\subseteq Z(R)$. Case 2:\ $\text{\rm char}\,R=2$. Let $J:=R[L, L]R$ and $I:=J\cap J^*$. Then $I$ is a nonzero $*$-ideal of $R$ such that $[I, R]\subseteq L$ (see Lemma \ref{lem5} (ii)). Since $R$ is noncommutative, $[I, R]$ is a non-abelian Lie ideal of $R$. By the fact that $d(T(L))=0$, we get $d(T([I, R]))=0$. Suppose first that $d$ is X-inner. There exists $b\in Q_s(R)$ such that $d(x)=[b, x]$ for all $x\in R$. Hence \begin{eqnarray} \big[b, [x, r]+[x, r]^*\big]=0 \label{eq:11} \end{eqnarray} for all $x\in I$ and $r\in R$. In view of Proposition \ref{pro1}, Eq.\eqref{eq:11} holds for all $x, r\in Q_s(R)$. By Lemma \ref{lem24}, $T(Q_s(R))^2\subseteq C$ and so $T(R)^2\subseteq Z(R)$, as desired. We next consider the general case. In view of Lemma \ref{lem25}, $[T(L),T(R)] \subseteq T(L)$, which implies $[T(L), d(T(R))]=0$. Applying the X-inner case above, either $T(R)^2\subseteq Z(R)$ or $d(T(R))\subseteq Z(R)$. For the latter case, assume that $d(T(R))\subseteq Z(R)$. It follows from Lemma \ref{lem1} that $\dim_CRC=4$ as $R$ is noncommutative. In view of Lemma \ref{lem14}, $*$ is of the first kind. Up to now, we have proved that $R$ is exceptional and $*$ is of the first kind. It follows from Theorem \ref{thm2} that $T(R)^2\subseteq Z(R)$. \end{proof} \section{Lie ideals: Theorem E} Let $R$ be a prime ring with involution $*$, and $d$ a derivation of $R$. We let $d^*$ be the derivation of $R$ defined by $ d^*(x)=d(x^*)^* $ for $x\in R$. We say that $d$ is a {\it $*$-derivation} if $d=d^*$ and is a {\it skew $*$-derivation} if $d=-d^*$. Note that if $b\in S(R)$ then the inner derivation defined by $b$ is a skew $*$-derivation. Similarly, if $b\in K(R)$, then the inner derivation defined by $b$ is a $*$-derivation. \begin{lem} \label{lem28} Let $R$ be a prime ring with involution $*$, $\text{\rm char}\,R\ne 2$, and $b\in R\setminus Z(R)$. Suppose that $ \big[b, [x, y]-[x, y]^*\big]=0 $ for all $x, y\in R$. Then $K_0(R)^2\subseteq Z(R)$. \end{lem} \begin{proof} By hypothesis, \begin{eqnarray} \big[b, [x, y]-[x, y]^*\big]=0 \label{eq:12} \end{eqnarray} for all $x, y\in R$. We claim that $*$ is of the first kind. Otherwise, $\beta\ne \beta^*$ for some $\beta\in C$. Choose a nonzero $*$-ideal $I$ of $R$ such that $\beta \subseteq R$. Let $x, y\in I$. Then $\beta^*\big[b, [x, y]-[x, y]^*\big]=0$ and $$ \big[b, [\beta x, y]-[\beta x, y]^*\big]=0, $$ implying that $(\beta^*-\beta)[b, [x, y]]=0$. That is, $\big[b, [I, I]\big]=0$ and so $b\in Z(R)$, a contradiction. Suppose first that $b^*=-b$. Then the inner derivation of $R$ induced by $b$ is a $*$-derivation. Replacing $x$ by $b$ in Eq.\eqref{eq:12}, $$ 0=\big[b, [b, y]-[b, y]^*\big]=\big[b, [b, y]-[y^*, b^*]\big]=\big[b, [b, y-y^*]\big] $$ for all $y\in R$. In view of \cite[Theorem 2.6]{lin1986}, it follows that $\dim_CRC=4$. Suppose that $*$ is of the symplectic type. Then $T(R)\subseteq Z(R)$. Let $x, y\in R$. Then $[x, y]+[x, y]^*\in Z(R)$. Together with Eq.\eqref{eq:12}, we get $ \big[b, 2[x, y]\big]=0. $ Since $\text{\rm char}\,R\ne 2$, $[b, [R, R]]=0$ and so $b\in Z(R)$, a contradiction. Thus $*$ is of the transpose type. In view of Theorem \ref{thm4}, $K_0(R)^2\subseteq Z(R)$, as desired. For the general case, it follows from Eq.\eqref{eq:12} that $$ \big[b^*-b, [x, y]-[x, y]^*\big]=0 $$ for all $x, y\in R$. Suppose on the contrary that $K_0(R)^2\nsubseteq Z(R)$. By the skew case proved above, $b^*-b\in Z(R)$. Since $*$ is of the first kind, we have $b=b^*$. Replacing $x$ by $b$ in Eq.\eqref{eq:12}, we get $$ 0=\big[b, [b, y]-[b, y]^*\big]=\big[b, [b, y+y^*]\big] $$ for all $y\in R$. In particular, $\big[b, [b, [x, y]+[x, y]^*]\big]=0$ for all $x, y\in R$. Together with Eq.\eqref{eq:12}, we get $[b, [b, [x, y]]=0$ for all $x, y\in R$. In view of \cite[Theorem 1]{bergen1981}, it follows that $b\in Z(R)$, a contradiction. Hence $K_0(R)^2\subseteq Z(R)$. \end{proof} Finally, we prove the following theorem, i.e., Theorem E.\vskip6pt \begin{thm}\label{thm32} Let $R$ be a prime ring with involution $*$, $L$ a non-abelian Lie ideal of $R$, and $d$ a nonzero derivation of $R$. If $d(K_0(L))=0$, then $K_0(R)^2\subseteq Z(R)$. \end{thm} \begin{proof} Case 1:\ $\text{\rm char}\,R=2$. Then $K_0(L)=T(L)$. It follows from Theorem \ref{thm20} that $K_0(R)^2=T(R)^2\subseteq Z(R)$, as desired. Case 2:\ $\text{\rm char}\,R\ne 2$. In view of Lemma \ref{lem25}, $[K_0(L), K_0(R)] \subseteq K_0(L)$. Since $d(K_0(L))=0$, we get $[K_0(L), d(K_0(R))]=0$. As $L$ is a non-abelian Lie ideal of $R$, there exists a nonzero $*$-ideal $I$ of $R$ such that $[I, R]\subseteq L$ (see Lemma \ref{lem5} (ii)). Let $z\in d(K_0(R))$. Then \begin{eqnarray} \big[z, [x, y]-[x, y]^*\big]=0 \label{eq:13} \end{eqnarray} for all $x\in I$ and $y\in R$. In view of Proposition \ref{pro1}, Eq.\eqref{eq:13} holds for all $x, y\in R$. By Lemma \ref{lem28}, either $K_0(R)^2\subseteq Z(R)$ or $z\in Z(R)$ for all $z\in d(K_0(R))$. Assume the latter case. That is, $d(K_0(R))\subseteq Z(R)$. In view of Lemma \ref{lem1} (i), it follows that $\dim_CRC=4$. Step 1:\ $d(C)=0$. Let $\beta=\beta^*\in C$. Choose a nonzero $*$-ideal $J$ of $R$ such that $\beta J\subseteq I$ and $J\subseteq I$. Let $x\in I$ and $r\in J$. Then $d([x, r]-[x, r]^*)=0$ and $$ 0=d([x, \beta r]-[x, \beta r]^*)=d(\beta([x, r]-[x, r]^*))=d(\beta)([x, r]-[x, r]^*). $$ Suppose on the contrary that $d(\beta)\ne 0$. Then $[x, r]=[x, r]^*$ for all $x\in I$ and $r\in J$. Since $xr\in J$, we get $$ [x, xr]=[x, xr]^*=(x[x, r])^*=[x, r]^*x^*=[x, r]x^*. $$ Hence $[x, [x, J]]=0$ for all $x=x^*\in I$. In view of \cite[Theorem 1]{bergen1981}, $x\in Z(R)$ for all $x=x^*\in I$. It is easy to prove that $R$ is commutative, a contradiction. Up to now, we have proved that if $\beta=\beta^*\in C$, then $d(\beta)=0$. In view of Lemma \ref{lem26}, we get $d(C)=0$. It follows from the Skolem-Noether theorem that $d$ is X-inner, that is, there exists $b\in Q_s(R)$ such that $d(x)=[b, x]$ for all $x\in R$. Step 2:\ The involution $*$ is of the first kind. Let $\beta\in C$. The aim is to prove that $\beta^*=\beta$. Choose a nonzero ideal $J$ of $R$ such that $\beta J\subseteq R$. Then, for $x\in I$ and $r\in J$, we have $ \big[b, [x, r]-[x, r]^*\big]=0 $ and $$ \big[b, [x, \beta r]-[x, \beta r]^*\big]=\beta^*\big[b, [x, r]-[x, r]^*\big]-(\beta^*-\beta)\big[b, [x, r]\big]=0. $$ Therefore $(\beta^*-\beta)\big[b, [x, r]\big]=0$. That is, $(\beta^*-\beta)\big[b, [I, J]\big]=0$. By Lemma \ref{lem2} and $b\notin Z(R)$, it follows that $\beta^*=\beta$, as desired. Up to now, we have proved that $\dim_CRC=4$ and $*$ is of the first kind. Suppose that $*$ is of the symplectic type, that is, $T(R)\subseteq Z(R)$. Let $x\in I$ and $r\in R$. Then $\big[b, [x, r]-[x, r]^*\big]=0$ and $[x, r]+[x, r]^*\in Z(R)$, implying that $\big[b, 2[x, r]\big]=0$. That is, $[b, [I, R]]=0$ and so $b\in Z(R)$, a contradiction. This proves that $*$ is of the transpose type. It follows from Theorem \ref{thm4} that $K_0(R)^2\subseteq Z(R)$, as desired. \end{proof} \begin{thebibliography}{99} \bibitem{amitsur1968} S. A. Amitsur, {\it Rings with involution}, Israel J. Math. {\bf 6} (1968), 99--106. \bibitem{amitsur1969} S. A. Amitsur, {\it Identities in rings with involutions}, Israel J. Math. {\bf 7}(1) (1969), 63--68. \bibitem{balogh2012} Z. Balogh, {\it Lie derived length and involutions in group algebras}, J. Pure Appl. Algebra {\bf 216} (2012), 1282--1287. \bibitem{balogh2008} Z. Balogh and T. Juh\'{a}sz, {\it Derived lengths of symmetric and skew symmetric elements in group algebras}, JP J. Algebra Number T. {\bf 12}(2) (2008), 191--203. \bibitem{baxter1968} W. E. Baxter and W. S. Martindale III, {\it Rings with involution and polynomial identities}, Canad. J. Math. {\bf 20} (1968), 465--473. \bibitem{beidar1998} K. I. Beidar and W. S. Martindale III, {\it On functional identities in prime rings with involution}, J. Algebra {\bf 203} (1998), 491--532. \bibitem{beidar1996} K. I. Beidar, W. S. Martindale III and A. V. Mikhalev, ``Rings with Generalized Identities", Monographs and Textbooks in Pure and Applied Mathematics, {\bf 196}. Marcel Dekker, Inc., New York, 1996. \bibitem{bergen1981} J. Bergen, I. N. Herstein and J. W. Kerr, {\it Lie ideals and derivations of prime rings}, J. Algebra {\bf 71}(1) (1981), 259--267. \bibitem{bien2019} M. H. Bien, {\it A note on local commutators in division rings with involution}, Bull. Korean Math. Soc. {\bf 56}(3) (2019), 659--666. \bibitem{bien2022} M. H. Bien, B. X. Hai and D. T. Hue, {\it On the unit groups of rings with involution}, Acta Math. Hungar. {\bf 166}(2) (2022), 432--452. \bibitem{catino2014} F. Catino, G. T. Lee and E. Spinelli, {\it Group algebras whose symmetric elements are Lie metabelian}, Forum Math. {\bf 26}(5) (2014), 1459--1471. \bibitem{chacron2016} M. Chacron, {\it Commuting involution}, Comm. Algebra {\bf 44}(9) (2016), 3951--3965. \bibitem{chacron2022} M. Chacron, {\it Certain invariant multiplicative subset of a simple Artinian ring with involution}, JP J. Algebra Number T. {\bf 53}(2) (2022), 151--163. \bibitem{chacron2023} M. Chacron, {\it Certain invariant multiplicative subset of a simple Artinian ring with involution. II}, Acta Math. Hungar. {\bf 170}(2) (2023), 437--454. \bibitem{chacron2024} M. Chacron, {\it On invariant submonoids}, Comm. Algebra, published online in 2024, https://doi.org/10.1080/00927872.2024.2400198. \bibitem{chacron1975} M. Chacron and I. N. Herstein, {\it Powers of skew and symmetric elements in division rings}, Houston J. Math. {\bf 1}(1) (1975), 15--27. \bibitem{chuang1987} C.-L. Chuang, {\it On invariant additive subgroups}, Israel J. Math. {\bf 57}(1) (1987), 116--128. \bibitem{chuang1989} C.-L. Chuang, {\it $*$-differential identities of prime rings with involution}, Trans. Amer. Math. Soc. {\bf 316}(1) (1989), 251--279. \bibitem{ferreira2015} V. O. Ferreira and J. Z. Gon\c{c}alves, {\it Free symmetric and unitary pairs in division rings infinite-dimensional over their centers}, Israel J. Math. {\bf 210} (2015), 297--321. \bibitem{goodaire2013} E. G. Goodaire and C. P. Milies, {\it Involutions and anticommutativity in group rings}, Canad. Math. Bull. {\bf 56}(2) (2013), 344--353. \bibitem{herstein1967} I. N. Herstein, {\it Special simple rings with involution}, J. Algebra {\bf 6}(3) (1967), 369--375. \bibitem{herstein1969} I. N. Herstein, ``Topics in ring theory'', The University of Chicago Press, Chicago, Ill.-London {\bf 1969} xi+132 pp. \bibitem{herstein1971} I. N. Herstein and S. Montgomery, {\it A note on division rings with involutions}, Michigan Math. J. {\bf 18} (1971), 75--79. \bibitem{herstein1974} I. N. Herstein, {\it Rings with periodic symmetric or skew elements}, J. Algebra {\bf 30} (1974), 144--154. \bibitem{herstein1976} I. N. Herstein, ``Rings with involution", Chicago Lectures in Mathematics. University of Chicago Press, Chicago, Ill.-London, {\bf 1976}. x+247 pp. \bibitem{herstein1983} I. N. Herstein, {\it A theorem on invariant subrings}, J. Algebra {\bf 83}(1) (1983), 26--32. \bibitem{lanski1972} C. Lanski and S. Montgomery, {\it Lie structure of prime rings of characteristic $2$}, Pacific J. Math. {\bf 42}(1) (1972), 117--136. \bibitem{lee1983} P.-H. Lee and T.-K. Lee, {\it Lie ideals of prime rings with derivations}, Bull. Inst. Math. Acad. Sinica {\bf 11}(1) (1983), 75--80. \bibitem{lee1986} P.-H. Lee and T.-K. Lee, {\it Note on nilpotent derivations}, Proc. Amer. Math. Soc. {\bf 98}(1) (1986), 31--32. \bibitem{lee1995} T.-K. Lee, {\it On nilpotent derivations of semiprime rings with involution}, Chinese J. Math. {\bf 23} (1995) 155--166. \bibitem{lee2017} T.-K. Lee, {\it Anti-automorphisms satisfying an Engel condition}, Comm. Algebra {\bf 45}(9) (2017), 4030--4036. \bibitem{lee2018} T.-K. Lee, {\it Commuting anti-homomorphisms}, Comm. Algebra {\bf 46}(3) (2018), 1060--1065. \bibitem{lee2022} T.-K. Lee, {\it On higher commutators of rings}, J. Algebra Appl. {\bf 21}(6) (2022), Paper No. 2250118, 6 pp. \bibitem{lee2025} T.-K. Lee and J.-H. Lin, {\it Commutators and products of Lie ideals of prime rings}, arXiv:2410.00444v2 [math.RA], https://doi.org/10.48550/arXiv.2410.00444. \bibitem{lee2010} G. T. Lee, ``Group identities on units and symmetric units of group rings", Algebra and Applications, {\bf 12}. Springer–Verlag London, Ltd., London, 2010. xii+194 pp. \bibitem{lee2015} G. T. Lee, S. K. Sehgal and E. Spinelli, {\it Lie identities on skew elements in group algebras}, Contemp. Math. {\bf 652} (2015), 103--121. \bibitem{lim1979} T. P. Lim, {\it Some classes of rings with involution satisfying the standard polynomial of degree $4$}, Pacific J. Math. {\bf 85}(1) (1979), 125--130. \bibitem{lin1986} J.-S. Lin, {\it On derivations of prime rings with involution}, Chinese J. Math. {\bf 14}(1) (1986), 37--51. \bibitem{lin2010} J.-S. Lin and C.-K. Liu, {\it Strong commutativity preserving maps in prime rings with involution}, Linear Algebra Appl. {\bf 432}(1) (2010), 14--23. \bibitem{martindale1969} W. S. Martindale III, {\it Rings with involution and polynomial identities}, J. Algebra {\bf 11}(2) (1969), 186--194. \bibitem{montgomery1971a} S. Montgomery, {\it Polynomial identity algebras with involution}, Proc. Amer. Math. Soc. {\bf 27}(1) (1971), 53--56. \bibitem{montgomery1971b} S. Montgomery, {\it A generalization of a theorem of Jacobson}, Proc. Amer. Math. Soc. {\bf 28}(2) (1971), 366--370. \bibitem{montgomery1973} S. Montgomery, {\it A generalization of a theorem of Jacobson II}, Pacific J. Math. {\bf 44}(1) (1973), 233--240. \bibitem{montgomery1976} S. Montgomery, {\it A structure theorem and a positive-definiteness condition in rings with involution}, J. Algebra {\bf 43}(1) (1976), 181--192. \bibitem{mosic2009} D. Mosi\'{c} and D. S. Djordjevi\'{c}, {\it Moore–Penrose-invertible normal and Hermitian elements in rings}, Linear Algebra Appl. {\bf 431}(5--7) (2009), 732--745. \bibitem{osborn1967} J. M. Osborn, {\it Jordan algebras of capacity two}, Proc. Natl. Acad. Sci. U.S.A. {\bf 57}(3) (1967), 582--588. \bibitem{posner1957} E. C. Posner, {\it Derivations in prime rings}, Proc. Amer. Math. Soc. {\bf 8}(6) (1957), 1093--1100. \bibitem{rowen1973} L. Rowen, {\it Some results on the center of a ring with polynomial identity}, Bull. Amer. Math. Soc. {\bf 79}(1) (1973), 219--223. \bibitem{shalev1992} A. Shalev, {\it The derived length of Lie soluble group rings I}, J. Pure Appl. Algebra {\bf 78}(3) (1992), 291--300. \bibitem{siciliano2011} S. Siciliano, {\it On the Lie algebra of skew-symmetric elements of an enveloping algebra}, J. Pure Appl. Algebra {\bf 215}(1) (2011), 72--76. \bibitem{thu2022} V. H. M. Thu, {\it A note on symmetric elements of division rings with involution}, Acta Math. Vietnam. {\bf 47} (2022), 495--502. \end{thebibliography} \end{document}
2412.05682v1
http://arxiv.org/abs/2412.05682v1
On the Rank of a bicomplex matrix
\documentclass[16pt,oneside,reqno]{amsart} \usepackage{amsfonts,amssymb,amscd,amsmath,enumerate,verbatim,calc} \usepackage{color,soul} \usepackage{mathtools} \usepackage{graphicx} \everymath{\displaystyle} \usepackage{xcolor} \usepackage{fancyhdr} \usepackage{tikz} \usetikzlibrary{shapes.geometric, arrows} \tikzstyle{arrow} = [thick,->,>=stealth] \tikzstyle{process} = [rectangle, minimum width=4cm, minimum height=2cm, text centered, text width=2.5cm, draw=green, fill=green!10] \textwidth=15cm \textheight=20.5cm \topmargin=0.5cm \oddsidemargin=0.5cm \evensidemargin=0.5cm \pagestyle{plain} \newcommand{\re}{\text{Re}} \newcommand{\Hom}{\text{Hom}} \newcommand{\ran}{\text{Im }} \newcommand{\rank}{\text{rank }} \newcommand{\nul}{\text{nullity}} \newcommand{\Log}{\text{Log}} \newcommand{\Arg}{\text{Arg}} \def\res{\mathop{\text{Res}}\limits} \newcommand{\TFAE}{The following conditions are equivalent:} \newcommand{\IFF}{\text{if and only if}} \newcommand{\wrt}{with respect to} \newcommand{\N}{\mathbb{N} } \newcommand{\Q}{\mathbb{Q} } \newcommand{\Z}{\mathbb{Z} } \newcommand{\R}{\mathbb{R} } \newcommand{\C}{\mathbb{C} } \newcommand{\n}{\mathfrak{n} } \newcommand{\m}{\mathfrak{m} } \newcommand{\q}{\mathfrak{q} } \newcommand{\p}{\mathfrak{p} } \theoremstyle{plain} \newtheorem{thm}{Theorem} \newtheorem{First}{Theorem A} \newtheorem{Second}{Theorem B} \newtheorem{Third}{Theorem C} \newtheorem{theorem}{Theorem}[section] \newtheorem{corollary}[theorem]{Corollary} \newtheorem{lemma}[theorem]{Lemma} \newtheorem{proposition}[theorem]{Proposition} \newtheorem{question}[theorem]{Question} \newtheorem{conjecture}[theorem]{Conjecture} \theoremstyle{definition} \newtheorem{definition}[theorem]{Definition} \newtheorem{s}[theorem]{} \newtheorem{remarks}[theorem]{Remarks} \newtheorem{remark}[theorem]{Remark} \newtheorem{example}[theorem]{Example} \newtheorem{examples}[theorem]{Examples} \newtheorem{construction}[theorem]{Construction} \begin{document} \title{On the Rank of a bicomplex matrix} \author{amita} \address{Department of Mathematics, IGNTU, Amarkantak, Madhya Pradesh 484887, India} \email{[email protected]} \author{Mamta Amol Wagh} \address{Department of Mathematics, University of Delhi, New Delhi 110078, India} \email{[email protected]} \author{Suman Kumar} \address{Department of Mathematics, IGNTU, Amarkantak, Madhya Pradesh 484887, India} \email{[email protected]} \author{Akhil Prakash} \address{Department of Mathematics, Aligarh Muslim University, Aligarh, Uttar Pradesh 202002, India} \email{[email protected]} \keywords{Bicomplex Numbers, Bicomplex matrix, Vector Space, Rank.} \subjclass[IMS]{Primary 15A04, 15A30; Secondary 30G35} \begin{abstract} The paper explores the concept of the rank of a bicomplex matrix, delving into four distinct types of ranks and investigating conditions under which these ranks are equivalent. It also defines and analyzes the concept of idempotent row space and idempotent column space of a bicomplex matrix. Some examples and counter examples have been presented to substantiate the study. \end{abstract} \maketitle \section{Introduction and preliminaries} In recent years, the theory of bicomplex numbers has become a thriving area of mathematical research, undergoing significant advancements and branching into new directions. In the historical development of bicomplex numbers, Segre is credited with its initial introduction \cite{segre1892}. Although not exhaustive, resources such as Price \cite{price2018introduction} offer a comprehensive foundation for multicomplex number theory. Notably, recent research efforts, (see \cite{alpay2014basics, futagawa1928, futagawa1932, {gervais2011finite}, {luna2015bicomplex}, riley1953, rochon2004, srivastava2008}) have further propelled advancements in this field. In this section, we provide a summary of several properties associated with bicomplex numbers. \noindent {\bf Bicomplex numbers:} A bicomplex number can be represented as ${u}_{1} + {i}_{1} {u}_{2} + {i}_{2} {u}_{3} + {i}_{1} {i}_{2} {u}_{4}$, where ${u_k}$ are the real numbers for $k=1$ to $4$ and $i_1$, $i_2$ are unit vectors satisfying the properties $i_1 i_2=i_2 i_1, i_1^2=i_2^2=-1$. For the sake of convenience, we employed certain symbols $\C_0, \C_1$, and $\C_2$, for the set of real numbers, the set of complex numbers, and the set of bicomplex numbers respectively. The set of bicomplex numbers can be represented in three different ways \begin{eqnarray*} \mathbb{C}_{2} &=& \{{u}_{1} + {i}_{1} {u}_{2} + {i}_{2} {u}_{3} + {i}_{1} {i}_{2} {u}_{4}\;:\; {u}_{1} , {u}_{2} , {u}_{3} , {u}_{4} \in\mathbb{C}_{0}\}, \\ \mathbb{C}_{2} &=& \{{z}_{1} + {i}_{2} {z}_{2}\;:\; {z}_{1}, {z}_{2} \in \mathbb{C}_{1}\},\\ \mathbb{C}_{2} &=& \{1_\xi e_1 + 2_\xi e_2\;:\; {1_\xi=({z}_{1}-i_1 {z}_{2}), 2_\xi=({z}_{1}+i_1 {z}_{2}) \in \mathbb{C}_{1}}\}. \end{eqnarray*} The introduction of two numbers, $e_1 = \frac{(1 + i_1 i_2)}{2}$ and $e_2 = \frac{(1 - i_1 i_2)}{2}$, significantly simplifies the system of bicomplex numbers $\C_2$. These numbers, $e_1$ and $e_2$ serve as zero divisors in $\C_2$. Furthermore, the numbers $e_1$ and $e_2$ form a basis for $\C_2$, which is called the idempotent basis. Thus, every bicomplex number $\xi$ can be written in a unique way as: $\xi = 1_\xi e_1 + 2_\xi e_2$. This representation is called the idempotent representation of a bicomplex number $\xi$. The idempotent representation gives algebraic structure to the bicomplex space, for instance the product of two bicomplex numbers can be seen component-wise in the above idempotent representation, i.e. if $\xi=1_\xi e_1 + 2_\xi e_2$ and $\eta=1_\eta e_1 + 2_\eta e_2$ in $\C_2$, then \begin{eqnarray*} \xi \cdot \eta = (1_\xi \cdot 1_\eta) e_1 + (2_\xi \cdot 2_\eta) e_2. \end{eqnarray*} The set of singular elements in $\C_2$ is denoted by $O_2$ and defined as the collection of all non-invertible elements in $\C_2$. \begin{definition} \noindent({\bf Principal ideals} \cite{price2018introduction}) The principal ideals in $\C_2$ determined by $e_1$ and $e_2$, denoted by $I_1$ and $I_2$, respectively are defined as: \begin{eqnarray*} I_1=:\{\xi e_1:\xi \in \C_2\}=\{1_\xi e_1:1_\xi \in \C_1\},\\ I_2=:\{\xi e_2:\xi \in \C_2\}=\{2_\xi e_2:2_\xi \in \C_1\}. \end{eqnarray*} \end{definition} \begin{theorem}\label{th1} \cite{price2018introduction} A bicomplex number $\xi=(Z_1+i_2Z_2)$ is singular if and only if $\xi \in I_1 \cup I_2$. \end{theorem} \begin{theorem}\label{th2} \cite{price2018introduction} A bicomplex number $\xi=(Z_1+i_2Z_2)$ is non-singular if and only if $\xi \notin I_1 \cup I_2$. \end{theorem} \begin{definition}\label{th5} (\noindent{\bf Cartesian product} \cite{anjali2023matrix}) The $n$-times Cartesian product of $\C_2$ is denoted by $\C_2^n$ and defined as \begin{eqnarray*} \mathbb C_2^n=\{(\xi_1,\xi_2,\ldots,\xi_n):\xi_i \in \C_2;i=1,2,\ldots,n\}. \end{eqnarray*} Moreover, analogous to the idempotent representation of an element in $\C_2$, every element of $\C_2^n$ can be expressed uniquely as \begin{eqnarray*} (\xi_1,\xi_2,\ldots,\xi_n)= (1_{\xi_1},1_{\xi_2},\ldots,1_{\xi_n})e_1 + (2_{\xi_1},2_{\xi_2},\ldots,2_{\xi_n}) e_2, \end{eqnarray*} where $(1_{\xi_1},1_{\xi_2},\ldots,1_{\xi_n}),(2_{\xi_1},2_{\xi_2},\ldots,2_{\xi_n})$ are $n$-tuples of complex numbers. \end{definition} \begin{remark}\label{th49}\cite{anjali2023matrix} If $(\xi_1,\xi_2,\ldots,\xi_n), (\eta_1,\eta_2,\ldots,\eta_n)\in \C_2^n$ and $\kappa \in \C_2$, then \begin{itemize} \item $(\xi_1,\xi_2,\ldots,\xi_n)\cdot (\eta_1,\eta_2,\ldots,\eta_n)=(\xi_1\eta_1,\xi_2\eta_2,\ldots,\xi_n\eta_n).$ \item $(\xi_1,\xi_2,\ldots,\xi_n)=(\eta_1,\eta_2,\ldots,\eta_n) \iff 1\xi_i=1\eta_i \ and \ 2\xi_i=2\eta_i\ \forall \ i=1,2,\ldots n$. \item $\kappa(\xi_1,\xi_2,\ldots,\xi_n) = (\xi_1,\xi_2,\ldots,\xi_n) \kappa =(\kappa\xi_1 ,\kappa \xi_2,\ldots,\kappa \xi_n)$. \end{itemize} Consequently, the product \begin{eqnarray*} (\xi_1,\xi_2,\ldots,\xi_n)e_i=(\xi_1e_i,\xi_2e_i,\ldots,\xi_n e_i)=(i_{\xi_1}e_i,i_{\xi_2}e_i,\ldots,i_{\xi_n} e_i) ; i=1,2. \end{eqnarray*} This conclusion arises from the fact that $\C_2^n$ is not only a $\C_2$ module but also exhibits the structure of a $\C_1$-algebra. \end{remark} \begin{definition}\label{th6} \noindent{\bf (Bicomplex matrix)} The set $\C_2^{n\times m}$ of bicomplex matrices of order ${n\times m}$ is defined as \begin{eqnarray*} \C_2^{n\times m}=:\{[\xi_{ij}]_{n\times m}:\xi_{ij}\in \C_2;1\leq i \leq n, 1\leq j \leq m \}. \end{eqnarray*} The set $\C_2^{n\times n}$ with respect to ordinary multiplication, addition and scalar multiplication forms an algebra over $\C_1$. Similar to the concept of idempotent representation for elements in $\C_2$, every bicomplex matrix can be uniquely expressed as \[ [\xi_{ij}]_{n\times m}=[1_{\xi_{ij}}]_{n\times m}e_1 + [2_{\xi_{ij}}]_{n\times m}e_2; 1\leq i \leq n, 1\leq j \leq m, \] where $[1_{\xi_{ij}}]_{n\times m}$ and $[2_{\xi_{ij}}]_{n\times m}$ are complex matrices of order $n\times m$, i.e. if $A=[\xi_{ij}]_{n\times m}\in \C_2^{n\times m}$ then $A=1_Ae_1+2_Ae_2$, where $1_A=[1_{\xi_{ij}}]_{n\times m}\in \C_1^{n\times m}$, $2_A=[2_{\xi_{ij}}]_{n\times m}\in \C_1^{n\times m}$ and $\C_1^{n\times m}$ denotes the set of all complex matrices of order ${n\times m}$. \end{definition} \begin{definition} \noindent{\bf (Bicomplex singular and non-singular matrix)} If $A\in C_2^{n\times n}$, then $A$ is said to be non-singular if determinant of $A$ is non-singular element, i.e. $\det(A)\notin O_2$ and if determinant of $A$ is singular element, i.e. $\det(A)\in O_2$, then it is called singular. \end{definition} \begin{theorem}\label{th7}\cite{price2018introduction} If $A$ is a bicomplex matrix of order $n\times n$, then $\det(A) = \det(1_A) e_1 + \det(2_A) e_2 $. \end{theorem} \begin{corollary}\label{th9}\cite{price2018introduction} Let $A$ be a bicomplex matrix of order $n\times n$, then $A$ is non-singular if and only if $\det(1_A) \neq0$ and $\det(2_A) \neq0$. \end{corollary} \begin{corollary}\label{th10}\cite{price2018introduction} Let $A$ be a bicomplex matrix of order $n\times n$, then $A$ is singular if and only if either $\det(1_A)=0$ or $\det(2_A)=0$. \end{corollary} \section{rank of Bicomplex matrix} \label{th11} This section deals with the rank, row rank, and column rank of a bicomplex matrix. Here we explore some results on these type of ranks. We define the idempotent row space and idempotent column space of a bicomplex matrix, also investigate some results. \begin{definition} Let $V$ be a subset of $\C_1^{n}$, then we define the set $Ve_i$ as follows \begin{eqnarray*} Ve_i=\{(x_1,x_2,\ldots,x_n )e_i;(x_1,x_2,\ldots,x_n) \in V \}, \mbox{for} \ i \in \{1,2\}. \end{eqnarray*} Furthermore, the set $Ve_i$ is a subset of $\C_2^n$. \end{definition} \begin{definition}({\bf $e_1$-matrix})\label{th12} If $A=[\xi_{ij}]_{n\times m}\in \C_2^{n\times m}$ and $\xi_{ij}={1_\xi}_{ij}e_1$ for all $i$ and $j$, then $A$ is called $e_1$-matrix. \end{definition} \begin{definition}({\bf $e_2$-matrix})\label{th13} If $A=[\xi_{ij}]_{n\times m}\in \C_2^{n\times m}$ and $\xi_{ij}={2_\xi}_{ij}e_2$ for all $i$ and $j$, then $A$ is called $e_2$-matrix. \end{definition} \begin{definition}({\bf $e_1e_2$-matrix})\label{th14} If $A=[\xi_{ij}]_{n\times m}\in \C_2^{n\times m}$ and $\xi_{ij}\in I_1\cup I_2$ for all $i$ and $j$, then $A$ is called $e_1e_2$- matrix. \end{definition} \begin{remark} Every $e_1$-matrix is an $e_1e_2$-matrix, also every $e_2$-matrix is an $e_1e_2$-matrix, but converse need not be true. For example \[ A =\begin{bmatrix} e_1 & e_2\\ e_2 & e_1 \end{bmatrix} \] is $e_1e_2$-matrix, but neither $A$ is $e_1$-matrix nor $A$ is $e_2$-matrix. \end{remark} \begin{theorem}\label{th15} If $B$ is a sub-matrix of a bicomplex matrix $A$, then $1_B$ and $2_B$ are sub-matrices of $1_A$ and $2_A$ respectively. \end{theorem} \begin{proof} Let $B$ be a sub-matrix of a bicomplex matrix $A$. Therefore, $B$ is obtained from $A$ by omitting some rows and columns of $A$. Thus, we have sub-matrices $1_B$ and $2_B$, which are obtained from matrix $B$ by omitting some rows and some columns of $1_A$ and $2_A$ respectively. Hence, $1_B$ and $2_B$ are sub-matrices of $1_A$ and $2_A$ respectively. \end{proof} \begin{remark} The converse of Theorem \ref{th15} is not true in general. Consider a bicomplex matrix \begin{align*} A = \begin{bmatrix} 1 & e_1 & 0 \\ e_1 & e_2 & e_1\\ 0 & 0 & 1 \end{bmatrix} =\begin{bmatrix} 1 & 1 & 0 \\ 1 & 0 & 1 \\ 0 & 0 & 1 \end{bmatrix} e_1 + \begin{bmatrix} 1 & 0 & 0 \\ 0 & 1 & 0 \\ 0 & 0 & 1 \end{bmatrix} e_2. \end{align*} We see that \[\begin{bmatrix} 1 & 1\\ 0 & 0 \end{bmatrix} \quad and \quad \begin{bmatrix} 1 & 0\\ 0 & 1 \end{bmatrix} \] are sub-matrices of $1_A$ and $2_A$, respectively. But \begin{align*} B =\begin{bmatrix} 1 & 1\\ 0 & 0 \end{bmatrix} e_1 + \begin{bmatrix} 1 & 0\\ 0 & 1 \end{bmatrix} e_2 =\begin{bmatrix} 1 & e_1\\ 0 & e_2 \end{bmatrix} \end{align*} is not a sub-matrix of $A$. \end{remark} \begin{definition}({\bf Rank of a bicomplex matrix})\label{th17} A number $r$ is said to be the rank of a matrix $A\in \C_2^{n\times m}$, if it possesses the following properties: \begin{itemize} \item There exists a non-singular sub-matrix $B_{r}$ of $A$ and a series of non-singular sub-matrices of the matrix $B_{r}$ such that \begin{equation}\label{eq1} B_1 \preccurlyeq B_2\preccurlyeq \ldots \preccurlyeq B_{r-1} \preccurlyeq B_{r}, \end{equation} where $B_i \preccurlyeq B_{i+1}$ represents that $B_i$ is a sub-matrix of $B_{i+1}$, and $i$ represents the order of the matrix $B_i$. \item If $B'$ is any sub-matrix of $A$ with order $t; t>r$, then no series of the type (\ref{eq1}) exists for the matrix $B'$. \end{itemize} \end{definition} The rank of a matrix A is denoted by $\rho(A)$. Furthermore, If no such $r$ exists, then we define $\rho(A)=0$. \begin{remark} The definition \ref{th17} yields that \begin{enumerate} \item If $r$ is the rank of a bicomplex matrix $A$, then there exists a non-singular matrix $B$ of order $r$ such that $B\preccurlyeq A$ and $\rho(B)=r$, and any matrix $B'$ of order $t; t> r$ such that $B'\preccurlyeq A$, then $\rho(B') \leq r$. \item If $A\in \C_2^{n\times m}$, then $\rho(A) \le$ min$(m,n)$. \item If $A\in \C_2^{n\times n}$ and $\rho(A) = n$, then $\det(A) \notin O_2$. But converse need not be true. For example, if $A =\begin{bmatrix} e_1 & e_2\\ e_2 & e_1 \end{bmatrix}$, then $\rho(A) = 0$, but $\det(A) \notin O_2$. \item If $A\in \C_2^{n\times m}$ and $\rho(A) = 0$, then $A$ need not be null matrix. For example, the rank of $e_1e_2$-matrix is zero, but it is not a null matrix. \end{enumerate} \end{remark} \begin{theorem} Let $A$ be a bicomplex matrix of order $n\times m$. Then, $\rho(A)=0$ if and only if $A$ is $e_1e_2$- matrix. \begin{proof} Let $A=[\xi_{ij}]_{n\times m} \in \C_2^{n \times m}$, and $\rho(A)=0$. Let us assume that there exists an entry $\xi_{ps}$ of $A$ such that $\xi_{ps} \notin O_2$ for some $p \in \{1,2,...,n\}$ and $s \in \{1,2,...,m\}$. Therefore the matrix $B=[\xi_{ps}]_{1\times 1}$ is a non-singular sub-matrix of $A$ and has a series of the type (\ref{eq1}). There are two cases here:\\ \textbf{Case (i):} If no series of the type (\ref{eq1}) exists for all sub-matrix $B'$ of $A$ with order $t; t>1$, then $\rho(A)=1$.\\ \textbf{Case (ii):} If there is a non-singular sub-matrix $B'$ of $A$ with order $t; t>1$ such that a series of the type (\ref{eq1}) exists for the matrix $B'$, then $\rho(A) \geq t$.\\ Clearly, in each case $\rho(A) \geq 1$, which contradicts as $\rho(A)=0$. So, our assumption, $\xi_{ps} \notin O_2$ for some $p \in \{1,2,...,n\}$ and $s \in \{1,2,...,m\}$, is not true. Thus, $\xi_{ps} \in O_2$ for all $p \in \{1,2,...,n\}$ and $s \in \{1,2,...,m\}$. Hence $A$ is $e_1e_2$- matrix.\\ \noindent{\bf Conversely} Suppose $A=[\xi_{ij}]_{n\times m}$ is an $e_1e_2$- matrix. Let us assume that $\rho(A)=r,r \geq 1$. Then there exists a non-singular sub-matrix $B_{r}$ of $A$ with order $r; r \geq 1$ such that a series $B_1 \preccurlyeq B_2\preccurlyeq \ldots \preccurlyeq B_{r-1} \preccurlyeq B_{r},$ of the type (\ref{eq1}) exists for the matrix $B_{r}$. Clearly, $B_1$ is a non-singular sub-matrix of $A$ with order 1. This implies that there exists an entry $\xi_{ps}$ of $A$ such that $\xi_{ps} \notin O_2$ for some $p \in \{1,2,...,n\}$ and $s \in \{1,2,...,m\}$, which contradicts as $A$ is $e_1e_2$- matrix. Therefore our assumption $\rho(A)=r,r \geq 1$ is not true. Hence $\rho(A)=0$. \end{proof} \end{theorem} \begin{definition}({\bf Row rank of a matrix})\label{th19} If $A=[\xi_{ij}]_{n\times m} \in \C_2^{n \times m}$, then the row space of $A$ is defined as the subspace of vector-space $\C_2^m(\C_1)$ spanned by the rows of $A$. The row rank of $A$ is denoted by "$\rho_r(A)$" and defined as the dimension of row space of $A$. \end{definition} \begin{definition}({\bf Column rank of a matrix})\label{th20} If $A=[\xi_{ij}]_{n\times m} \in \C_2^{n \times m}$, then the column space of $A$ is defined as the subspace of vector-space $\C_2^n(\C_1)$ spanned by the columns of $A$. The column rank of $A$ is denoted by "$\rho_c(A)$" and defined as the dimension of column space of $A$. \end{definition} \begin{theorem} \label{th18} Let $A$ be a bicomplex matrix of order $n\times m$. Then, \begin{center} $\rho(A)\le min \ (\rho(1_A),\rho(2_A))$. \end{center} \end{theorem} \begin{proof} Let $A$ be a bicomplex matrix of order $n\times m$ and $\rho(1_A)=r_1$, $\rho(2_A)=r_2$. Let us consider $\rho(A)=l$ and $l>min(r_1,r_2)$. Then, there exists a sub-matrix $B$ of $A$ of order $l\times l$ such that $\rho(B)=l$. This implies that $\det(B)\notin O_2$. \begin{eqnarray*} &\Rightarrow& \det(1_B)\neq 0 \ \mbox{and} \ \det(2_B)\neq 0\ \{\mbox{by Corollary\ref{th9}}\}\\ &\Rightarrow& \rho(1_B) = l \ \mbox{and} \ \rho(2_B) = l\\ &\Rightarrow& \rho(1_B) > r_1 \ \mbox{or} \ \rho(2_B) > r_2 \quad \{\mbox{as} \ l>min(r_1,r_2)\}\\ &\Rightarrow& \rho(1_B) > \rho(1_A) \ \mbox{or} \ \rho(2_B) > \rho(2_A), \end{eqnarray*} which is a contradiction, because $(1_B)$ and $(2_B)$ are the sub-matrices of $(1_A)$ and $(2_A)$ respectively. Therefore, our assumption $\rho(A)=l$, and $l>min(r_1,r_2)$ is not true. Thus, $\rho(A) \leq min(r_1,r_2)$. Hence, $\rho(A) \leq min(\rho(1_A),\rho(2_A))$. \end{proof} The following remark is immediate consequence of Theorem \ref{th18}. \begin{remark} If $A$ is a bicomplex matrix of order $n\times m$, then \begin{enumerate} \item $\rho(A)\leq \rho(1_A)$ and $\rho(A)\leq \rho(2_A)$. \item $\rho(A)\leq$ $\rho_r(1_A)$ and $\rho(A)\leq$ $\rho_r(2_A)$. \item $\rho(A)\leq$ $\rho_c(1_A)$ and $\rho(A)\leq$ $\rho_c(2_A)$. \end{enumerate} \end{remark} \begin{theorem}\label{th21} A matrix $A\in \C_2^{n\times n}$ is non-singular if and only if $\rho(1_A) = n$ and $\rho(2_A) = n$. \end{theorem} \begin{proof} Let $A\in \C_2^{n\times n}$ be a non-singular matrix. Then, by using Corollary \ref{th9} we have \begin{eqnarray*} \det(1_A) \ne 0 \ \mbox{and} \ \det(2_A)\neq 0 \Rightarrow \rho(1_A) = n \ \mbox{and} \ \rho(2_A) = n. \end{eqnarray*} \noindent{\bf Conversely} Let $\rho(1_A)=n$ and $\rho(2_A)=n$, where $1_A,\ 2_A\in \C_1^{n\times n}(\C_1)$. Then, \begin{eqnarray*} \det(1_A)\neq0 \ \mbox{and} \ \det(2_A)\neq0. \end{eqnarray*} Using Corollary \ref{th9}, we get $A$ is non singular matrix. The proof of theorem is completed. \end{proof} The following corollary is immediate consequence of Theorem \ref{th21}. \begin{corollary}\label{th22} If $A\in \C_2^{n\times m}$, and $\rho(A)=n($respectively $m)$, then $\rho(1_A)=n($respectively $m)$ and $\rho(2_A) =n($respectively $m)$. \end{corollary} \begin{remark} The converse of the Corollary \ref{th22} is not true. For instance, if \begin{align*} A =\begin{bmatrix} 1 & 0 & 0\\ 0 & 1 & 6 \end{bmatrix} e_1 + \begin{bmatrix} 0 & 1& 0\\ 1 & 0 & 6 \end{bmatrix} e_2 =\begin{bmatrix} e_1 & e_2 & 0\\ e_2 & e_1 & 6 \end{bmatrix}, \end{align*} then $\rho(1_A)=\rho(2_A)=2$, but $\rho(A)=1$. \end{remark} \begin{remark} If $A\in \C_2^{n\times m}$, then in general $\rho_r(A) \neq$ $\rho_c(A)$. \end{remark} \begin{example}\label{th23} Consider \[ A=\begin{bmatrix} 2 & e_1 & 1\\ 0 & 0 & e_1 \end{bmatrix}. \] It is evident that columns of A and rows of A are linearly independent with complex scalars. Thus, $\rho_r(A) \neq$ $\rho_c(A)$. \end{example} \begin{remark} Example \ref{th23} shows that a bicomplex matrix $A\in \C_2^{n\times m}$ does not exhibit analogous notion as observed in a complex matrix, i.e. all columns of a bicomplex matrix $A$ are linearly independent or all rows of a bicomplex matrix $A$ are linearly independent does not imply $\rho(A)=m$ or $n$. \end{remark} \begin{remark} If $A\in\C_2^{n\times n}$, and $\det(A)\in O_2$, then rows of A and columns of A may be linearly independent. \end{remark} \begin{example}\label{th24} Consider \[ A=\begin{bmatrix} 1 & 2\\ e_1 & 0 \end{bmatrix}. \] The rows and columns of matrix A are linearly independent, i.e. $\rho_r(A) =\rho_c(A)=2$, but $\det(A)\in O_2$. \end{example} \begin{remark} If $A\in \C_2^{n\times m}$, then in general $\rho_r(A) \neq \rho_c(A) \neq \rho(A)$. \end{remark} \begin{example}\label{th25} Consider \[ A=\begin{bmatrix} 1 & 0 & 1\\ e_2 & e_1 & 0\\ 0 & e_2 & e_1\\ 0 & 0 & e_1 \end{bmatrix}. \] Then, we have $\rho_r(A)=4$, $\rho_c(A)=3$ and $\rho(A)=1$, i.e. $\rho_r(A)\neq$ $\rho_c(A)\neq \rho(A)$. \end{example} \begin{definition}\label{th26} ({\bf Idempotent row rank of a matrix}) If $A = [\xi_{ij}]_{n\times m}\in \C_2^{n\times m}$, then we define idempotent row space of $A$ as \begin{eqnarray*} \mbox{Idempotent row space of}\ A &=& (\mbox{row space of}\ 1_A) e_1 + (\mbox{row space of}\ 2_A) e_2\\ &=& \left\{ \left(\sum_{k=1}^{n}\alpha_k({1_\xi}_{k1},{1_\xi}_{k2},\ldots,{1_\xi}_{km})\right)e_1 \right. \\ &&\left. +\left(\sum_{k=1}^{n}\beta_k({2_\xi}_{k1},{2_\xi}_{k2},\ldots,{2_\xi}_{km})\right)e_2;\ \alpha_k,\beta_k\in \C_1, 1\le k\le n \right\}. \end{eqnarray*} The Idempotent row rank of $A$ is denoted by "$\rho_{ir}(A)$" and defined as the dimension of idempotent row space of $A$. \end{definition} \begin{definition}({\bf Idempotent column rank of a matrix})\label{th27} If $A = [\xi_{ij}]_{n\times m}\in \C_2^{n\times m}$, then we define idempotent column space of $A$ as \begin{eqnarray*} \mbox{Idempotent column space of}\ A&=&(\mbox{column space of}\ 1_A) e_1 + (\mbox{column space of}\ 2_A) e_2\\ &=& \left\{ \left(\sum_{k=1}^{m}\alpha_k({1_\xi}_{1k},{1_\xi}_{2k},\ldots{1_\xi}_{nk})\right)e_1 \right. \\ &&\left. +\left(\sum_{k=1}^{m}\beta_k({2_\xi}_{1k},{2_\xi}_{2k},\ldots,{2_\xi}_{nk})\right)e_2;\ \alpha_k,\beta_k\in \C_1, 1\le k\le m \right\}. \end{eqnarray*} The Idempotent column rank of $A$ is denoted by "$\rho_{ic}(A)$" and defined as the dimension of idempotent column space of $A$. \end{definition} \begin{theorem}\label{th28} If $A=[\xi_{ij}]_{n\times m}\in \C_2^{n\times m}$, then row space of \ $A \subseteq$ Idempotent row space of \ $A$. \end{theorem} \begin{proof} Let $A=[\xi_{ij}]_{n\times m}\in \C_2^{n\times m}$ and $X \in$ row space of $A$. Then, $X = \sum_{k=1}^{n}\alpha_{k}(\xi_{k1},\xi_{k2},\ldots,\xi_{km})$, where $\alpha_k \in \C_1$. Now, by using Definition \ref{th5} and Remark \ref{th49}, we have \begin{eqnarray*} X &=& \sum_{k=1}^{n}\alpha_{k}\{({1_\xi}_{k1},{1_\xi}_{k2},\ldots,{1_\xi}_{km}) e_1 + ({2_\xi}_{k1},{2_\xi}_{k2},\ldots,{2_\xi}_{km}) e_2\}\\ &=& \sum_{k=1}^{n}\alpha_{k}\{({1_\xi}_{k1}e_1,{1_\xi}_{k2}e_1,\ldots,{1_\xi}_{km}e_1) +({2_\xi}_{k1}e_2,{2_\xi}_{k2}e_2,\ldots,{2_\xi}_{km}e_2)\}\\ &=& \sum_{k=1}^{n} \alpha_{k}({1_\xi}_{k1} e_1,{1_\xi}_{k2} e_1,\ldots,{1_\xi}_{km} e_1) + \sum_{k=1}^{n}\alpha_{k}({2_\xi}_{k1} e_2,{2_\xi}_{k2} e_2,\ldots,{2_\xi}_{km} e_2)\\ &=& \sum_{k=1}^{n}(\alpha_{k} ({1_\xi}_{k1} e_1),\alpha_{k}({1_\xi}_{k2} e_1),\ldots,\alpha_{k}({1_\xi}_{km} e_1)) \\ &&\hspace{3.5 cm}+ \sum_{k=1}^{n}(\alpha_{k}({2_\xi}_{k1} e_2),\alpha_{k}({2_\xi}_{k2} e_2),\ldots,\alpha_{k}({2_\xi}_{km} e_2))\\ &=& \sum_{k=1}^{n}((\alpha_{k} {1_\xi}_{k1}) e_1,(\alpha_{k}{1_\xi}_{k2}) e_1,\ldots,(\alpha_{k}{1_\xi}_{km} )e_1)\\ &&\hspace{3.5 cm}+\sum_{k=1}^{n}((\alpha_{k}{2_\xi}_{k1}) e_2,(\alpha_{k}{2_\xi}_{k2}) e_2,\ldots,(\alpha_{k}{2_\xi}_{km}) e_2)\\ &=& \sum_{k=1}^{n}(\alpha_{k} {1_\xi}_{k1} ,\alpha_{k}{1_\xi}_{k2},\ldots,\alpha_{k}{1_\xi}_{km} )e_1 + \sum_{k=1}^{n}(\alpha_{k}{2_\xi}_{k1},\alpha_{k}{2_\xi}_{k2},\ldots,\alpha_{k}{2_\xi}_{km})e_2\\ &=& \sum_{k=1}^{n}\{\alpha_{k} ( {1_\xi}_{k1} ,{1_\xi}_{k2},\ldots,{1_\xi}_{km} )\}e_1+ \sum_{k=1}^{n}\{\alpha_{k}({2_\xi}_{k1},{2_\xi}_{k2},\ldots,{2_\xi}_{km})\}e_2 \\ &=& \left \{\sum_{k=1}^{n}\alpha_{k} ( {1_\xi}_{k1} ,{1_\xi}_{k2},\ldots,{1_\xi}_{km} )\right\}e_1+\left \{\sum_{k=1}^{n}\alpha_{k}({2_\xi}_{k1},{2_\xi}_{k2},\ldots,{2_\xi}_{km})\right \}e_2. \end{eqnarray*} Therefore, $X \in$ (row space of $1_A ) e_1 +$ (row space of $2_A ) e_2$. Hence, row space of $A \subseteq$ idempotent row space of $A$. \end{proof} \noindent{Dually, we may prove the following theorem.} \begin{theorem} \label{th29} If $A=[\xi_{ij}]_{n\times m}\in \C_2^{n\times m}$, then column space of $A \subseteq$ idempotent column space of $A$. \end{theorem} \begin{theorem} \label{th30} Let $A=[\xi_{ij}] \in \C_2^{n\times m}$ and either rows of $1_A$ or rows of $2_A$ are linearly independent. Then, $\rho_r(A)$ = n, i.e. rows of A are linearly independent. \end{theorem} \begin{proof} Let $A=[\xi_{ij}]_{n\times m}\in \C_2^{n\times m}$ and rows of $1_A$ be linearly independent.\\ Suppose $\sum_{k=1}^{n}\alpha_k (\xi_{k1},\xi_{k2},\dots,\xi_{km})=0$, where $\alpha_k \in \C_1$. Then, by Definition \ref{th5} and Remark \ref{th49}, it follows that \begin{eqnarray*} && \sum_{k=1}^{n} \alpha_k \{({1_\xi}_{k1},{1_\xi}_{k2},\dots,{1_\xi}_{km}) e_1 + ({2_\xi}_{k1},{2_\xi}_{k2},\dots,{2_\xi}_{km}) e_2 \} = (0,0,\dots,0)\\ &\Rightarrow& \sum_{k=1}^{n} \alpha_k \{({1_\xi}_{k1}e_1, {1_\xi}_{k2}e_1, \dots,{1_\xi}_{km}e_1) + ({2_\xi}_{k1}e_2, {2_\xi}_{k2}e_2, \dots,{2_\xi}_{km}e_2)\}= (0,0,\dots,0) \\ &\Rightarrow&\sum_{k=1}^{n} \alpha_k({1_\xi}_{k1}e_1, {1_\xi}_{k2}e_1, \dots,{1_\xi}_{km}e_1) + \sum_{k=1}^{n} \alpha_k ({2_\xi}_{k1}e_2, {2_\xi}_{k2}e_2, \dots,{2_\xi}_{km}e_2)= (0,0,\dots,0)\\ &\Rightarrow&\sum_{k=1}^{n} (\alpha_k({1_\xi}_{k1}e_1), \alpha_k({1_\xi}_{k2}e_1),\dots, \alpha_k({1_\xi}_{km}e_1)) \\ &&\hspace{3.0 cm} +\sum_{k=1}^{n} (\alpha_k ({2_\xi}_{k1}e_2), \alpha_k({2_\xi}_{k2}e_2),\dots, \alpha_k({2_\xi}_{km}e_2)) = (0,0,\dots,0)\\ &\Rightarrow&\sum_{k=1}^{n} ((\alpha_k{1_\xi}_{k1})e_1, (\alpha_k{1_\xi}_{k2})e_1,\dots, (\alpha_k{1_\xi}_{km})e_1) \\ &&\hspace{3.0 cm}+\sum_{k=1}^{n} ((\alpha_k {2_\xi}_{k1})e_2, (\alpha_k{2_\xi}_{k2})e_2,\dots, (\alpha_k{2_\xi}_{km})e_2)= (0,0,\dots,0)\\ &\Rightarrow&\sum_{k=1}^{n} (\alpha_k{1_\xi}_{k1}, \alpha_k{1_\xi}_{k2},\dots, \alpha_k{1_\xi}_{km})e_1\\ &&\hspace{3.0 cm}+\sum_{k=1}^{n} (\alpha_k {2_\xi}_{k1}, \alpha_k{2_\xi}_{k2},\dots, \alpha_k{2_\xi}_{km})e_2 = (0,0,\dots,0)\\ &\Rightarrow& \left\{\sum_{k=1}^{n} (\alpha_k{1_\xi}_{k1}, \alpha_k{1_\xi}_{k2},\dots, \alpha_k{1_\xi}_{km})\right\}e_1\\ &&\hspace{3.0 cm}+\left\{\sum_{k=1}^{n} (\alpha_k {2_\xi}_{k1}, \alpha_k{2_\xi}_{k2},\dots, \alpha_k{2_\xi}_{km})\right\}e_2 = (0,0,\dots,0). \end{eqnarray*} Implies that, \[ \sum_{k=1}^{n} \alpha_k ({1_\xi}_{k1},{1_\xi}_{k2},\dots,{1_\xi}_{km}) = (0,0,\dots,0)\quad and \quad \sum_{k=1}^{n} \alpha_k ({2_\xi}_{k1},{2_\xi}_{k2},\dots,{2_\xi}_{km}) =(0,0,\dots,0). \] Since rows of $1_A$ are linearly independent, therefore $\alpha_{k} =0\ \forall\ k; 1\le k\le n$. So, it is clear that rows of $A$ are linearly independent. Similarly, if rows of $2_A$ are linearly independent. It gives rows of $A$ are linearly independent. Hence $\rho_r(A) = n$. \end{proof} \begin{remark} Some remarks can be easily made here. \begin{enumerate} \item The converse of Theorem \ref{th30} is not true. For example, consider \[ A=\begin{bmatrix} 0 & 0 & e_1\\ 0 & 0 & e_2 \end{bmatrix}. \] Then, we have $\rho_r(A)$ = 2. But, rows of $1_A$ are linearly dependent and $2_A$ are linearly dependent. \item If $A\in \C_2^{n\times m}$ and $\rho(A) = n$, then Theorem \ref{th30} and Corollary \ref{th22} implies that $\rho_r(A)$ = $\rho(A)$ = $\rho_r(1_A)$ = $\rho_r(2_A) = n$. \end{enumerate} \end{remark} The following Corollary \ref{th31} is immediate consequence of Theorems \ref{th21} and \ref{th30}. \begin{corollary}\label{th31} Let $A=[\xi_{ij}]_{n\times n} \in \C_2^{n\times n}$ and $\det(A)\notin O_2$, then $\rho_r(1_A)$ = $\rho_r(2_A)$ = $\rho_r(A)=n$. \end{corollary} Dually we may prove the following result. \begin{theorem}\label{th32} If $A\in \C_2^{n\times m}$ and either columns of $1_A$ or columns of $2_A$ are linearly independent then $\rho_c(A)$ = m. \end{theorem} \begin{remark} The following remarks can be made here. \begin{enumerate} \item The converse of the Theorem \ref{th32} is not true. For example, consider \[ A=\begin{bmatrix} e_1 & 0 \\ 0 & e_2\\ \end{bmatrix}. \] It is trivial to find that $\rho_c(A)$ = 2, but columns of $1_A$ are linearly dependent and columns of $2_A$ are linearly dependent. \item If $A\in \C_2^{n\times m}$ and $\rho(A)=m$, then Theorem \ref{th32} and Corollary \ref{th22} imply that $\rho_c(A)$ = $\rho(A)$ = $\rho_c(1_A)$ = $\rho_c(2_A) = m$. \end{enumerate} \end{remark} The following corollary is immediate consequence of Theorems \ref{th21} and \ref{th32}. \begin{corollary}\label{th33} If $A=[\xi_{ij}]_{n\times n} \in \C_2^{n\times n}$ and $\det(A) \notin O_2$, then $\rho_c(1_A)$ = $\rho_c(2_A)$ = $\rho_c(A) = n$. \end{corollary} The following Corollaries \ref{th34} and \ref{th35} are immediate consequences of Theorems \ref{th30} and \ref{th32}. \begin{corollary}\label{th34} If $A=[\xi_{ij}]_{n \times n} \in \C_2^{n\times n}$ and either rows of $1_A$ (respectively $2_A$) or columns of $1_A$ (respectively $2_A$) are linearly independent, then $\rho_r(A)$ = $\rho_c(A) = n$. \end{corollary} \begin{corollary}\label{th35} If $A\in \C_2^{n\times n}$ and $det(A)\notin O_2$, then $\rho_r(A)$ = $\rho_c(A) = n$. \end{corollary} \begin{proposition}\label{th36} If $A=[\xi_{ij}]_{n\times n} \in \C_2^{n\times n}$ and $\rho(A) = n$, then $\rho(A)$ = $\rho_c(A)$ = $\rho_r(A)$. \end{proposition} \begin{proof} Let $A=[\xi_{ij}]_{n \times n} \in \C_2^{n\times n}$ and $\rho(A) = n$, then $\det(A)\notin O_2$. Thus by using corollary \ref{th35}, $\rho(A)$ = $\rho_r(A)$ = $\rho_c(A)$. \end{proof} \begin{theorem}\label{th37} Let $A=[\xi_{ij}]_{n\times n} \in \C_2^{n\times n}$ be a non-singular matrix such that $\xi_{ij}\notin O_2$ for all $i,j$ and $n\le 3$. Then, $\rho(A) = n$. \end{theorem} \begin{proof} Let $A=[\xi_{ij}]_{n\times n} \in \C_2^{n\times n}$ be non-singular matrix such that $\xi_{ij}\notin O_2$ for all $i,j$.\\ \textbf{Case (i):} If $n=1$, then clearly $\rho(A) = 1$.\\ \textbf{Case (ii):} If $n=2$, then $A =\begin{bmatrix} \xi_{11} & \xi_{12}\\ \xi_{21} & \xi_{22}\\ \end{bmatrix}$.\\ Consider \begin{align*} A_1= \begin{bmatrix} \xi_{11} \end{bmatrix}\quad \mbox{and} \quad A_2=\begin{bmatrix} \xi_{11} & \xi_{12} \\ \xi_{21} & \xi_{22} \end{bmatrix}. \end{align*} Since $\xi_{ij}\notin O_2$ for all $i,j$ and $\det(A) \notin O_2$, therefore we have a series of non-singular sub-matrices of $A$ such that $A_1 \preccurlyeq A_2=A$. Thus $\rho(A)$ will be $2$.\\ \textbf{Case (iii):} $n=3$.\\ Let, $B=1_B e_1 +2_B e_2$ be any sub-matrix of $A$ of order $2\times 3$. Then \begin{align*} 1_B=\begin{bmatrix} C_1 & C_2 & C_3 \end{bmatrix} \quad \mbox{and} \quad 2_B=\begin{bmatrix} C_1' & C_2' & C_3' \end{bmatrix}, \end{align*} where $C_i,C_i';i\in\{1,2,3\}$ are columns of $1_B$ and $2_B$ respectively. Since $\det(A) \notin O_2$ and $\xi_{ij}\notin O_2$ for all $i,j$, therefore $\rho_c(1_B) = \rho_c(2_B) = 2$ and $C_i\neq 0,C_i'\neq 0;\forall \ i\in\{1,2,3\}$.\\ Since $\rho_c(1_B) = 2$ , this implies that there exist $i, j\in\{1,2,3\};i<j$ such that $C_i$ and $C_j$ are linearly independent.\vspace{0.05cm} Let us suppose \textquotedblleft $C_i \ \mbox{and} \ C_k$\textquotedblright \ and \textquotedblleft$C_j\ \mbox{and} \ C_k$\textquotedblright \ be linearly dependent columns, where $k\in\{1,2,3\};i\neq k$ and $j\neq k$. Then there exist $\alpha\neq0$ and $\beta\neq0 \in C_1$ such that $C_i=\alpha C_k$ and $C_j=\beta C_k$. It follows that $C_i$ and $C_j$ are linearly dependent, which is a contradiction because $C_i$ and $C_j$ are linearly independent. Therefore, our assumption \textquotedblleft$C_i \ \mbox{and} \ C_k$\textquotedblright \ and \textquotedblleft$C_j\ \mbox{and} \ C_k$\textquotedblright are linearly dependent is not true, i.e. either \textquotedblleft$C_i \ \mbox{and} \ C_k$\textquotedblright are linearly independent or \textquotedblleft$C_j\ \mbox{and} \ C_k$\textquotedblright are linearly independent, which evidently implies that the set of vectors $\{C_i,C_k\}$ and $\{C_i,C_j\}$ are linearly independent or the set of vectors $\{C_i,C_j\}$ and $\{C_j,C_k\}$ are linearly independent for $i,j \ \mbox{and} \ k \in \{1,2,3\}$; $i\neq j, j\neq k$ and $k\neq i$. In same manner, for the matrix $2_B$ we can find that either the set of vectors $\{C_l',C_m'\}$ and $\{C_l',C_n'\}$ or the set of vectors $\{C_l',C_m'\}$ and $\{C_m',C_n'\}$ are linearly independent. Now, we list the possible pair of non-singular sub-matrices of $1_B$ and $2_B$ of order $2\times2$: \begin{center} \begin{tabular}{||c | c||} \hline \textbf{Possible pair of non singular} & \textbf{Possible pair of non singular} \\ \textbf{sub-matrices of $1_B$ of order 2} & \textbf{sub-matrices of $2_B$ of order 2} \\ [0.5ex] \hline\hline $[C_1,C_2]$ and $[C_1,C_3]$ & $[C_1',C_2']$ and $[C_1',C_3']$ \\ \hline $[C_1,C_2]$ and $[C_2,C_3]$ & $[C_1',C_2']$ and $[C_2',C_3']$ \\ \hline $[C_1,C_3]$ and $[C_2,C_3]$ & $[C_1',C_3']$ and $[C_2',C_3']$ \\ \hline \end{tabular} \end{center} It is completely clear from the above that there are nine cases formed from the above possibilities to form non singular sub-matrix of $B$ of order $2$. In each case, there exist $\alpha,\beta$ such that $[C_\alpha, C_\beta] \ and \ [C'_\alpha, C'_\beta] $ are non singular sub-matrices of order $2$ of $1_B$ and $2_B$ respectively for some $\alpha,\beta \in \{1,2,3\}$, where $\alpha<\beta$. Now, we construct a bicomplex matrix \[ M=[C_\alpha \ C_\beta]e_1+[C'_\alpha \ C'_\beta]e_2. \] It is evident that $M$ is a non-singular sub-matrix of order $2$ of matrix $B$. This implies that $M$ is the sub-matrix of order $2$ of matrix $A$ such that $\det(M)\notin O_2$. Then, there exist $\xi_{pq}\notin O_2$ such that $[\xi_{pq}]_{1\times 1}$ is a sub-matrix of $M$ of order $1$, where $p,q\in \{1,2,3\}$. Let $A_1=[\xi_{pq}]_{1 \times 1}$, $A_2=M$ and $A_3=A$. Since $\xi_{pq}\notin O_2$, $\det(M)\notin O_2$ and $\det(A)\notin O_2$, therefore we have a series of non- singular sub-matrices of A such that \[ A_1\preccurlyeq A_2 \preccurlyeq A_3=A .\] Hence $\rho(A) = 3$.\\ The proof of theorem is completed. \end{proof} \begin{theorem}\label{th38} Let $V$ be a subspace of $\C_1^n(\C_1)$. Then, $V e_1$ is a subspace of $\C_2^n(\C_1)$. \end{theorem} \begin{proof} Let $V$ be a subspace of $\C_1^n(\C_1)$. Then, $Ve_1$ is a subset of $\C_2^n$.\\ Let us consider $X,Y\in Ve_1$. Then, there exists $(x_1,x_2,\ldots,x_n),(y_1,y_2,\ldots,y_n)\in V$ such that $X=(x_1,x_2,\ldots,x_n)e_1$ and $Y=(y_1,y_2,\ldots,y_n)e_1$. Now, by using Remark \ref{th49}, we have \begin{eqnarray*} X+Y &=& (x_1,x_2,\ldots,x_n)e_1+(y_1,y_2,\ldots,y_n)e_1\\ &=&(x_1 e_1,x_2 e_1,\ldots,x_n e_1)+(y_1 e_1,y_2 e_1,\ldots,y_n e_1)\\ &=&(x_1 e_1+y_1 e_1,x_1 e_1+y_1 e_1,\ldots,x_n e_1+y_n e_1)\\ &=&((x_1+y_1)e_1,(x_2+y_2)e_1,\ldots,(x_n+y_n)e_1)\\ &=&(x_1+y_1, x_2+y_2, \ldots,x_n+y_n)e_1\in V e_1 .\end{eqnarray*} and \begin{eqnarray*} \alpha X &=& \alpha\{(x_1,x_2,\ldots,x_n)e_1\}\\ &=&\alpha(x_1 e_1,x_2 e_1,\ldots,x_n e_1)\\ &=&(\alpha(x_1 e_1),\alpha(x_2 e_1),\ldots,\alpha(x_n e_1))\\ &=&((\alpha x_1) e_1,(\alpha x_2) e_1,\ldots,(\alpha x_n) e_1)\\ &=&(\alpha x_1, \alpha x_2, \ldots, \alpha x_n) e_1 \in V e_1 .\end{eqnarray*} Since $V e_1$ is a subset of $\C_2^n$. Thus, $V e_1$ forms a subspace of $\C_2^n(\C_1)$. \end{proof} Dually we may prove the following result. \begin{theorem}\label{th39} Let $V$ be a subspace of $\C_1^n(\C_1)$. Then, $V e_2$ is subspace of $C_2^n(\C_1)$. \end{theorem} Combining Theorems \ref{th38} and \ref{th39} we get following result. \begin{proposition}\label{th40} If $1_V$ and $2_V$ are two sub-spaces of $\C_1^n(\C_1)$, then $1_V e_1 + 2_V e_2$ is a subspace of $\C_2^n(\C_1)$. \end{proposition} \begin{remark} It follows from Proposition \ref{th40}, if $A\in \C_2^{n\times m}$, then idempotent row space of $A$ and idempotent column space of $A$ are sub-spaces of $\C_2^m$ and $\C_2^n$, respectively. \end{remark} The following corollary is immediate consequence of Remark 2.42 and Theorem \ref{th29}. \begin{corollary}\label{th41} If $A\in \C_2^{n\times m}$, then column space of $A$ is a subspace of idempotent column space of $A$. \end{corollary} The following corollary is immediate consequence of Remark 2.42 and Theorem \ref{th28}. \begin{corollary}\label{th42} If $A\in \C_2^{n\times m}$, then row space of $A$ is a subspace of idempotent row space of $A$. \end{corollary} \begin{proposition}\label{th43} If $1_V$ and $2_V$ be two sub-spaces of $\C_1^n$. Then $1_Ve_1\cap 2_V e_2=\{0\}$. \end{proposition} \begin{proof} Let $1_V$ and $2_V$ be two sub-spaces of $\C_1^n$ and $(\xi_{1},\xi_{2},\cdots, \xi_{n}) \in 1_Ve_1 \cap 2_V e_2$. Then, \begin{center} $(\xi_{1},\xi_{2},\cdots, \xi_{n}) \in 1_V e_1$ and $(\xi_{1},\xi_{2},\cdots, \xi_{n}) \in 2_V e_2$. \end{center} This implies that there exist \ $(z_{1},z_{2},\cdots, z_{n})\in 1_V$ and $( w_{1},w_{2},\cdots, w_{n})\in 2_V$ such that \[ (\xi_{1},\xi_{2},\cdots, \xi_{n}) =(z_{1},z_{2},\cdots, z_{n})e_1 \mbox{and} (\xi_{1},\xi_{2},\cdots, \xi_{n}) =(w_{1},w_{2},\cdots, w_{n})e_2. \] Then, by using Remark \ref{th49}, we have $(\xi_{1},\xi_{2},\cdots, \xi_{n}) =(z_{1}e_1,z_{2}e_2,\cdots, z_{n}e_1) \ and \ (\xi_{1},\xi_{2},\cdots, \xi_{n}) =(w_{1}e_2,w_{2}e_2,\cdots, w_{n}e_2)$ \begin{eqnarray*} &\Rightarrow&\xi_{i} = z_{i} e_1\ \ and\ \ \xi_{i} = w_{i}e_2 \ \forall \ i; \ 1\le i \le n\\ &\Rightarrow& \ z_{i} e_1=w_{i} e_2 \ \forall \ i ;\ 1\le i \le n\\ &\Rightarrow& z_{i}=0 \quad and \quad w_{i}=0\ \forall \ i;\ 1\le i \le n\\ &\Rightarrow& (\xi_{1},\xi_{2},\cdots, \xi_{n})=(0,0,\cdots, 0) .\end{eqnarray*} Therefore, \begin{align}\label{1} 1_V e_1 \cap 2_V e_2 \subseteq \{0\}. \end{align} Since $(0,0,\cdots, 0)\in 1_V e_1$ and $(0,0,\cdots, 0) \in 2_V e_2$. Clearly $(0,0,\cdots, 0)\in 1_V e_2 \cap 2_V e_2$. So, we have \begin{align}\label{2} \{0\} \subseteq 1_V e_2 \cap 2_V e_2 .\end{align} Using \eqref{1} and \eqref{2}, we get \[ 1_V e_2 \cap 2_V e_2 = \{0\} .\] The proof of proposition is completed. \end{proof} The following corollary is immediate consequence of Proposition \ref{th40} and Proposition \ref{th43}. \begin{corollary}\label{th44} If $1_V$ and $2_V$ are two sub-spaces of $\C_1^n(\C_1)$. Then, $1_V e_1 + 2_V e_2 = 1_V e_1 \oplus 2_V e_2$. \end{corollary} \begin{theorem}\label{th45} Let $1_V$ be a subspace of $\C_1^n(\C_1)$ and $\dim(1_V)=k$. Then, $\dim(1_V e_1)=k$. \end{theorem} \begin{proof} Let $1_V$ be a subspace of $\C_1^n(\C_1)$ and $\dim(1_V)=k$. So we take \begin{center} $S=\{(\beta_{11},\beta_{12},\ldots,\beta_{1n}), (\beta_{21},\beta_{22},\ldots,\beta_{2n}), \ldots, (\beta_{k1},\beta_{k2},\ldots,\beta_{kn})\}$ \end{center} as a basis for $1_V$. Now, We claim that the set \begin{center} $S'=\{(\beta_{11},\beta_{12},\ldots,\beta_{1n})e_1, (\beta_{21},\beta_{22},\ldots,\beta_{2n})e_1, \ldots, (\beta_{k1},\beta_{k2},\ldots,\beta_{kn})e_1\}$ \end{center} forms a basis for $1_Ve_1$. Suppose $\sum_{i=1}^{k} \alpha_i \{(\beta_{i1},\beta_{i2},\ldots,\beta_{in})e_1\} = 0, \alpha_i\in \C_1$. Using Remark \ref{th49}, we have \begin{eqnarray*} &&\sum_{i=1}^{k} \alpha_i (\beta_{i1}e_1,\beta_{i2}e_1,\ldots,\beta_{in}e_1) = 0\\ &\Rightarrow& \ \sum_{i=1}^{k} (\alpha_i (\beta_{i1}e_1),\alpha_i(\beta_{i2}e_1),\ldots,\alpha_i(\beta_{in}e_1)) = 0\\ &\Rightarrow& \sum_{i=1}^{k} ((\alpha_i \beta_{i1})e_1,(\alpha_i \beta_{i2})e_1,\ldots,(\alpha_i\beta_{in})e_1) =0\\ &\Rightarrow& \sum_{i=1}^{k} (\alpha_i \beta_{i1},\alpha_i \beta_{i2},\ldots,\alpha_i\beta_{in})e_1 = 0\\ &\Rightarrow& \left\{\sum_{i=1}^{k} (\alpha_i \beta_{i1},\alpha_i \beta_{i2},\ldots,\alpha_i\beta_{in})\right\}e_1 = 0 \\ &\Rightarrow& \sum_{i=1}^{k} (\alpha_i \beta_{i1},\alpha_i \beta_{i2},\ldots,\alpha_i\beta_{in}) = 0\\ &\Rightarrow& \sum_{i=1}^{k} \alpha_i( \beta_{i1}, \beta_{i2},\ldots,\beta_{in}) = 0 .\end{eqnarray*} Since $S$ is a basis of $1_V$, implies that $\alpha_i=0 \ \forall i$. Therefore $S'$ is a linearly independent set.\\ Let $X\in 1_Ve_1$. Then, there exists $1_X\in 1_V$ such that $X=1_Xe_1$. Since $1_X\in 1_V$, therefore \begin{eqnarray*} 1_X=\sum_{i=1}^{k} \alpha_i (\beta_{i1},\beta_{i2},\ldots,\beta_{in}) &\Rightarrow& X= \left\{\sum_{i=1}^{k} \alpha_i (\beta_{i1},\beta_{i2},\ldots,\beta_{in})\right\}e_1 .\end{eqnarray*} Using Remark \ref{th49}, it follows that \begin{eqnarray*} X&=&\left\{\sum_{i=1}^{k} (\alpha_i\beta_{i1},\alpha_i\beta_{i2},\ldots,\alpha_i \beta_{in})\right\}e_1\\ \ &=&\sum_{i=1}^{k} (\alpha_i\beta_{i1},\alpha_i\beta_{i2},\ldots,\alpha_i \beta_{in})e_1\\ &=&\sum_{i=1}^{k} ((\alpha_i\beta_{i1})e_1,(\alpha_i\beta_{i2})e_1,\ldots,(\alpha_i \beta_{in})e_1)\\ &=&\sum_{i=1}^{k} (\alpha_i(\beta_{i1}e_1),\alpha_i(\beta_{i2}e_1),\ldots,\alpha_i (\beta_{in}e_1))\\ &=&\sum_{i=1}^{k} \alpha_i(\beta_{i1}e_1,\beta_{i2}e_1,\ldots,\beta_{in}e_1)\\ &=&\sum_{i=1}^{k} \alpha_i\{(\beta_{i1},\beta_{i2},\ldots,\beta_{in})e_1\} .\end{eqnarray*} This implies that \begin{eqnarray*} &&X\in \langle S' \rangle\\ &\Rightarrow& 1_V e_1 \subseteq \langle S' \rangle \\ &\Rightarrow& 1_V e_1 = \langle S' \rangle .\end{eqnarray*} Therefore, $S'$ is a basis for $1_V e_1$. This implies that $\dim(1_V e_1)=k$.\\ The proof of theorem is completed. \end{proof} Dually we may prove the following result. \begin{theorem}\label{th46} Let $2_V$ be a subspace of $\C_1^n(\C_1)$ and $\dim(2_V)=k$. Then $\dim(2_V e_2)=k$. \end{theorem} The next result follows from the Corollary 2.46, Theorem 2.47 and Theorem 2.48. \begin{theorem}\label{th47} Let $1_V$ and $2_V$ be two sub-spaces of $\C_1^n(\C_1 )$, $\dim(1_V)=k_1$ and $\dim(2_V)=k_2$. Then $\dim(1_V e_1 + 2_V e_2) = k_1 + k_2$. \end{theorem} The next result follows from the Theorem \ref{th47}. \begin{proposition}\label{th48} If $A=1_A e_1 + 2_A e_2 \in \C_2^{n\times m}$, then \\ $\dim($idempotent column space of $A)$= $\dim($idempotent row space of $A)$, i.e. $\rho_{ir}(A)$ = $\rho_{ic}(A)$ .\end{proposition} In the view of the results 2.36 of section 2. The following problem still remain open for the research.\\ {\bf Problem 1.} If $A\in \C_2^{n\times n}$ is a non-singular matrix such that $\xi_{ij}\notin O_2$ for all $i, j$, then for $n\le 3$ we have shown that $\rho(A) = n$. Is this true for all n? \bibliographystyle{amsplain} \bibliography{references} \end{document}
2412.05698v1
http://arxiv.org/abs/2412.05698v1
Controlled rough SDEs, pathwise stochastic control and dynamic programming principles
\documentclass[11pt]{amsart} \usepackage[left=3.4cm,right=3.4cm,top=2.2cm,bottom=2.2cm]{geometry} \usepackage{amsmath,amssymb,mathrsfs,stmaryrd} \usepackage{amsthm} \usepackage{mathtools,bm} \usepackage[LGR,T1]{fontenc} \usepackage[utf8]{inputenc} \usepackage{lmodern} \usepackage[english]{babel} \usepackage{cancel} \usepackage{dsfont} \usepackage[colorlinks,linkcolor=NavyBlue,citecolor=Gray,urlcolor=Periwinkle] {hyperref} \usepackage[nameinlink,capitalize]{cleveref} \usepackage[dvipsnames]{xcolor} \usepackage[shortlabels]{enumitem} \usepackage[babel=true,kerning=true]{microtype} \usepackage[abbrev,msc-links,alphabetic]{amsrefs} \usepackage{lscape} \usepackage{afterpage} \usepackage[textsize=tiny]{todonotes} \newcommand{\khoa}[2][] {\todo[color=SeaGreen,caption={}, #1]{#2}} \newcommand{\peter}[2][] {\todo[color=yellow,caption={}, #1]{#2}} \newcommand{\huilin}[2][] {\todo[color=green,caption={}, #1]{#2}} \newcommand{\mnNorm}[3]{ \| {#1} | \cff_{#2} \|_{#3}} \theoremstyle{plain} \newcommand{\vep}{\varepsilon} \newcommand{\ome}{\omega} \newcommand{\CB}{\mathcal{B}} \newcommand{\CK}{\mathcal{K}} \newcommand{\Cb}{\ensuremath{\mathcal C_b}} \newcommand{\nbh}{\text{neighbourhood}} \newcommand{\be}{\begin{equation}} \newcommand{\ee}{\end{equation}} \newcommand{\bea}{\begin{eqnarray}} \newcommand{\eea}{\end{eqnarray}} \newcommand{\beaa}{\begin{eqnarray*}} \newcommand{\eeaa}{\end{eqnarray*}} \newcommand{\lan}{ \langle } \newcommand{\ran}{ \rangle} \newcommand{\gam}{\gamma} \newcommand{\lam}{ \lambda} \newcommand{\ka}{\kappa} \newcommand{\eps}{\varepsilon} \newcommand{\MH}{\mathcal{H}} \newcommand{\MF}{\cff} \newcommand{\MT}{\mathcal{T}} \newcommand{\MD}{\mathcal{D}} \newcommand{\MC}{\mathscr{C}} \newcommand{\ML}{\mathrm{Lin}} \newcommand{\MY}{\mathcal{Y}} \newcommand{\MZ}{\mathcal{Z}} \newcommand{\MP}{\mathcal{P}} \newcommand{\mC}{\mathbb{C}_{T,d}} \newcommand{\MK}{\mathcal{K}} \newcommand{\MA}{\mathcal{A}} \newcommand{\MB}{\mathcal{B}} \newcommand{\MG}{\mathcal{G}} \newcommand{\MS}{\mathcal{S}} \newcommand{\MO}{\ooo} \newcommand{\MU}{\mathcal{U}} \newcommand{\hD}{\hat{\mathbb{D}}_{T,d}} \newcommand{\hC}{\hat{\mathbb{C}}_{T,d}} \newcommand{\BM}{\mathbb{M}} \newcommand{\BS}{\mathbb{S}} \newcommand{\Bx}{\mathbf{x}} \newcommand{\BW}{\mathbf{W}} \newcommand{\BD}{\mathbf{D}} \newcommand{\cK}{\mathcal{K}} \newcommand{\hY}{\hat{Y}} \newcommand{\hZ}{\hat{Z}} \newcommand{\hf}{\hat{f}} \newcommand{\hPhi}{\hat{\Phi}} \newcommand{\hc}{\hat{c}} \newcommand{\hu}{\hat{u}} \newcommand{\heta}{\hat{\eta}} \newcommand{\td}{\tilde{d}} \newcommand{\tR}{\tilde{R}} \newcommand{\MV}{\mathcal{V}} \newcommand{\tBY}{\tilde{\mathbf{Y}}} \newcommand{\tBZ}{\tilde{\mathbf{Z}}} \newcommand{\tx}{\tilde{x}} \newcommand{\tS}{\tilde{S}} \newcommand{\tv}{\tilde{v}} \newcommand{\bv}{\bar{v}} \newcommand{\tB}{\tilde{B}} \newcommand{\teta}{\tilde{\eta}} \newcommand{\txi}{\tilde{\xi}} \newcommand{\tX}{\tilde{X}} \newcommand{\tY}{\tilde{Y}} \newcommand{\tU}{\tilde{U}} \newcommand{\tMY}{\tilde{\MY}} \newcommand{\ty}{\tilde{y}} \newcommand{\tz}{\tilde{z}} \newcommand{\tZ}{\tilde{Z}} \newcommand{\tBX}{\tilde{\mathbf{X}}} \newcommand{\te}{\tilde{\vep}} \newcommand{\tb}{\tilde{b}} \newcommand{\tu}{\tilde{u}} \newcommand{\tA}{\tilde{A}} \newcommand{\tome}{\tilde{\omega}} \newcommand{\tMF}{\tilde{\MF}} \newcommand{\tc}{\tilde{c}} \newcommand{\tP}{\tilde{P}} \newcommand{\tpsi}{\tilde{\psi}} \newcommand{\tC}{\tilde{C}} \newcommand{\tPsi}{\tilde{\Psi}} \newcommand{\tI}{\tilde{I}} \newcommand{\tJ}{\tilde{J}} \newcommand{\tf}{\tilde{f}} \newcommand{\tg}{\tilde{g}} \newcommand{\tMA}{\tilde{\MA}} \newcommand{\tMV}{\tilde{\MV}} \newcommand{\tsigma}{\tilde{\sigma}} \newcommand{\tMM}{\widetilde{\mathcal{M}}} \newcommand{\tOmega}{\tilde{\Omega}} \newcommand{\bE}{\bar{E}} \newcommand{\bY}{\bar{Y}} \newcommand{\bB}{\bar{B}} \newcommand{\bareta}{\bar{\eta}} \newcommand{\bxi}{\bar{\xi}} \newcommand{\bZ}{\bar{Z}} \newcommand{\bpsi}{\bar{\psi}} \newcommand{\bF}{\bar{F}} \newcommand{\bg}{\bar{g}} \newcommand{\bMG}{\bar{\MG}} \newcommand{\bMF}{\bar{\MF}} \newcommand{\bbP}{\bar{\bP}} \newcommand{\bMV}{\bar{\MV}} \newcommand{\bBX}{\bar{\mathbf{X}}} \newcommand{\bt}{\bar{t}} \newcommand{\by}{\bar{y}} \newcommand{\bX}{\bar{X}} \newcommand{\gamp}{\gam'} \newcommand{\mup}{\mu'} \newcommand{\etap}{\eta'} \newcommand{\E}{\mathbb{E}} \newcommand{\tE}{\tilde{\mathbb{E}}} \newcommand{\R}{\mathbb{R}} \newcommand{\N}{\mathbb{N}} \newcommand{\Oc}{ \Omega^c} \newcommand{\Od}{ \Omega^D} \newcommand{\X}{\mathbb{X}} \newcommand{\T}{\mathbb{T}} \newcommand{\bS}{\mathbb{S}} \newcommand{\bH}{\mathbb{H}} \newcommand{\BX}{\mathbf{X}} \newcommand{\By}{\mathbf{y}} \newcommand{\bP}{\mathbb{P}} \newcommand{\tK}{\tilde{\mathcal{K}}} \newcommand{\FC}{\mathscr{C}} \newcommand{\FF}{\mathscr{F}} \newcommand{\B}{\mathbb{B}} \newcommand{\BB}{\mathbf{B}} \newcommand{\op}{\mathcal{P}} \newcommand{\FD}{\mathscr{D}} \newcommand{\oq}{\mathcal{Q}} \newcommand{\oor}{\mathcal {R}} \newcommand{\BI}{\mathbf{1}} \newcommand{\BZ}{\mathbf{Z}} \newcommand{\I}{\mathcal{I}} \newcommand{\gl}{{\gamma_t, \eta_t}} \newcommand{\BF}{\mathbb{F}} \newcommand{\Z}{\mathbb{Z}} \newcommand{\bx}{\mathbf{x}} \newcommand{\bz}{\mathbf{z}} \newcommand{\s}{\mathbb{S}} \newcommand{\A}{\mathbb{A}} \newcommand{\LL}{\mathbb{L}} \newcommand{\tiop}{\tilde{\mathcal{P}}} \newcommand{\BBI}{\mathbf{I}} \newcommand{\BY}{\mathbf{Y}} \newcommand{\FKF}{\mathfrak{K}_F} \newcommand{\Bf}{\mathbf{f}} \newcommand{\yop}{y^\op} \newcommand{\lf}{\lfloor} \newcommand{\rf}{\rfloor} \newcommand{\strela}{\rightharpoonup} \newtheorem{thm}{Theorem}[section] \newtheorem{theorem}[thm]{Theorem} \newtheorem{assumption}[thm]{Assumption} \newtheorem{lem}[thm]{Lemma} \newtheorem{coro}[thm]{Corollary} \newtheorem{corollary}[thm]{Corollary} \newtheorem{rem}[thm]{Remark} \newtheorem{remark}[thm]{Remark} \newtheorem{prop}[thm]{Proposition} \newtheorem{proposition}[thm]{Proposition} \newtheorem{exam}[thm]{Example} \newtheorem{defi}[thm]{Definition} \newtheorem{definition}[thm]{Definition} \newcommand{\cdummy}{\cdot} \newcommand{\tmem}[1]{{\em #1\/}} \newcommand{\tmtextbf}[1]{\text{{\bfseries{#1}}}} \crefname{assumption}{Assumption}{Assumptions} \numberwithin{equation}{section} \newcommand{\assign}{:=} \newcommand{\backassign}{=:} \newcommand{\nospace}{} \newcommand{\tmmathbf}[1]{\ensuremath{\boldsymbol{#1}}} \newcommand{\tmop}[1]{\ensuremath{\operatorname{#1}}} \newcommand{\tmstrong}[1]{\textbf{#1}} \newcommand{\tmverbatim}[1]{{\ttfamily{#1}}} \newcommand{\wei}[1]{\langle#1\rangle} \newcommand{\bk}[1]{\llbracket#1\rrbracket} \newcommand{\tand}{\quad\textrm{and} \quad} \newcommand{\tif}{\quad\textrm{if} \quad} \newcommand{\geo}{{\mathrm{geo}}} \newcommand{\ctrl}{\eta} \newcommand{\tctrl}{\teta} \newcommand{\bctrl}{\bar{\eta}} \newcommand{\law}{\mathrm{Law}} ll}$\Box$\medskip} \colorlet{darkred}{red!90!black} \newcommand{\comment}[2][darkred]{\marginpar{\begin{flushleft}\scriptsize{\textcolor{#1}{#2}}\end{flushleft}}} \long\def\ant#1{\textcolor{blue}{A: #1}} \long\def\rmk#1{\textcolor{red}{#1}} \def\KL#1{\comment[teal]{KL: #1}} \long\def\col#1{\textcolor{blue}{#1}} \newcommand{\caa}{\mathcal A} \newcommand{\cbb}{\mathcal B} \def\cdd{{\mathcal D}} \newcommand{\aaa}{\mathfrak A} \newcommand{\cff}{{\mathfrak F}} \newcommand{\ccc}{\mathfrak C} \newcommand{\uuu}{\mathfrak U} \newcommand{\bbb}{\mathfrak B} \newcommand{\ooo}{\mathfrak O} \newcommand{\mmm}{\mathfrak M} \newcommand{\cll}{\mathcal L} \def\cgg{{\mathcal G}} \def\cmm{{\mathcal M}} \def\cjj{{\mathcal J}} \def\coo{{\mathcal O}} \newcommand{\cpp}{{\mathcal P}} \newcommand{\M}{{\mathcal M}} \def\czz{{\mathcal Z}} \def\bww{{\bm X}} \newcommand{\sym}{{\mathrm{Sym} }} \newcommand{\tr}{{\mathrm{Tr} }} \def\lt{\left} \def\rt{\right} \newcommand{\We}{\ensuremath{\bm{W}}} \newcommand{\Y}{\ensuremath{\bm{Y}}} \newcommand{\XX}{\ensuremath{\mathbb{X}}} \newcommand{\Rd}{{\R^d}} \renewcommand{\P}{\ensuremath{{\mathbb P}}} \newcommand{\D}{\ensuremath{{\mathbf D}}} \newcommand{\DD}{\ensuremath{{\mathscr D}}} \newcommand{\DL}[3]{\ensuremath{\mathbf{D}_X^{#1}L_{#2,#3}}} \newcommand{\DLL}[2]{\ensuremath{\mathbf{D}_X^{#1}L_{#2}}} \newcommand{\bDL}[3]{\ensuremath{\mathbf{\bar D}_X^{#1}L_{#2,#3}}} \newcommand{\DDL}[3]{\ensuremath{\mathscr{D}_X^{#1}L_{#2,#3}}} \newcommand{\DDLL}[2]{\ensuremath{\mathscr{D}_X^{#1}L_{#2}}} \newcommand{\DLtilde}[3]{\ensuremath{\mathbf{D}_{\widetilde X}^{#1}L_{#2,#3}}} \newcommand{\Vone}{{\bar{V}}} \newcommand{\wass}{{\mathcal W}} \newcommand{\cont}{{\mathscr W}} \newcommand{\abx}{{\mathcal X}} \newcommand{\var}{\ensuremath{ \textrm{-}{\mathrm{var}}}} \newcommand{\lip}{\ensuremath{{\mathrm{Lip}}}} \newcommand{\dd}{\ensuremath{{{\partial }}}} \DeclareMathOperator*{\esssup}{ess\,sup} \newcommand{\bxx}{\mathbb{X}} \newcommand\bmA{\bm{A}} \newcommand{\bmxx}{\X} \newcommand{\C}{{\mathcal C}} \newcommand{\CC}{{\mathscr{C}}} \newcommand{\1}{{\mathds 1}} \newcommand{\db}{{\bar d}} \newcommand{\EE}{{{\mathbb E}_{\bigcdot}}} \newcommand{\W}{\mathbb{W}} \usepackage{graphicx} \makeatletter \newcommand*\bigcdot{{\mathpalette\bigcdot@{.5}}} \newcommand*\bigcdot@[2]{\mathbin{\vcenter{\hbox{\scalebox{#2}{$\m@th#1\bullet$}}}}} \makeatother \newcommand{\loc}{\ensuremath{{\mathrm{loc}}}} \newcommand{\lin}{\ensuremath{{\mathrm{lin}}}} \newcommand{\ot}{\leftarrow} \DeclareFontEncoding{FMS}{}{} \DeclareFontSubstitution{FMS}{futm}{m}{n} \DeclareFontEncoding{FMX}{}{} \DeclareFontSubstitution{FMX}{futm}{m}{n} \DeclareSymbolFont{fouriersymbols}{FMS}{futm}{m}{n} \DeclareSymbolFont{fourierlargesymbols}{FMX}{futm}{m}{n} \DeclareMathDelimiter{\vvert}{\mathord}{fouriersymbols}{152}{fourierlargesymbols}{147} \DeclarePairedDelimiter{\nn}{\vvert}{\vvert} \DeclarePairedDelimiter{\fl}{\lfloor}{\rfloor} \DeclarePairedDelimiter{\cl}{\lceil}{\rceil} \begin{document} \title[\tiny{Controlled RSDEs, pathwise stochastic control, dynamic programming principles}]{Controlled rough SDEs, pathwise stochastic control and dynamic programming principles} \author{Peter K.~Friz} \address{TU Berlin and WIAS Berlin} \email{[email protected]} \author{Khoa L\^e} \address{University of Leeds} \email{[email protected]} \author{Huilin Zhang} \address{Shandong U. and Humboldt U.} \email{[email protected]} \subjclass[2020]{Primary 60L20, 60H10} \keywords{Rough SDEs, pathwise stochastic control.} \begin{abstract} We study stochastic optimal control of rough stochastic differential equations (RSDEs). This is in the spirit of the pathwise control problem (Lions--Souganidis 1998, Buckdahn--Ma 2007; also Davis--Burstein 1992), with renewed interest and recent works drawing motivation from filtering, SPDEs, and reinforcement learning. Results include regularity of {\em rough} value functions, validity of a {\em rough} dynamic programming principles and new {\em rough} stability results for HJB equations, removing excessive regularity demands previously imposed by flow transformation methods. Measurable selection is used to to relate RSDEs to ``doubly stochastic'' SDEs under conditioning. In contrast to previous works, Brownian statistics for the to-be-conditioned-on noise are not required, aligned with the ``pathwise'' intuition that these should not matter upon conditioning. Depending on the chosen class of admissible controls, the involved processes may also be anticipating. The resulting stochastic value functions coincide in great generality for different classes of controls. RSDE theory offers a powerful and unified perspective on this problem class. \end{abstract} \maketitle \tableofcontents \section{Introduction} \label{sec.introduction} Consider a {partially} controlled multidimensional diffusion $Y = Y^{\theta}$ with It\^o dynamics \be dY_t = b (t,Y_t ; \theta_t) dt + \sigma (t,Y_t ; \theta_t) d B_t + f (t,Y_t) d W_t . \label{equ:11} \ee Here $B$ and (for the moment) $W$ are independent Brownian motions, $\theta$ is taken in some class of ``admissible'' controls $\Theta$. {\em Pathwise stochastic control} is concerned with minimizing some expected cost, conditionally on $W$. Specifically, \be V^{\Theta} (s, y, \omega) := \mathrm{essinf}_{\theta \in \Theta} \mathbb{E}^{s, y} \left( g (Y _T) + \int_s^T \ell (t, Y_t, \theta_r)dr \Big| \mathfrak{F}_T^W \right) \label{equ:12} \ee defines a random field, which should relate to non-linear stochastic partial differential equations (SPDEs) of Hamilton--Jacobi--Bellmann (HJB) type, of the form \be \label{equ:sHJB} - d_t v = H (y, t, D v, D^2 v) dt + (f (t,y) \cdot Dv) \circ d W_t, \qquad v(T,\cdot) \equiv g. \ee Classical It\^o theory is not well equipped to treat this SPDE. Taking a {\em pathwise} view, with $W(\omega)$ replaced by a generic continuous path $X$, it was proposed in \cite{LS98} to understand such value functions as ``stochastic viscosity solution'' of \eqref{equ:sHJB}: their analysis is based on a deterministic path $X$, the problem only becomes ``stochastic'' in the very end upon {\em randomization}, \[ X \rightsquigarrow W(\omega) ; \] with no Brownian assumptions necessary, it suffices that $W$ has continuous realizations. The same logic applies to multidimensional $W$ and general vector fields $f$, upon taking a {\em rough-pathwise} perspective \cite{CFO11}. (The notion of pathwise stochastic control actually goes back at least to Davis and Burstein \cite{DB92}, cf. \cite{DFG17, AC20} for a discussion.) A direct stochastic approach to \eqref{equ:sHJB} and the associated control problem was pursued in \cite{BM07}, assuming Brownian statistics for $(B,W)$, essentially relying on stochastic flow transformations such as to ``transform away'' the $dW$-term in \eqref{equ:11} and \eqref{equ:sHJB}, respectively. Importantly, \cite{BM07} offers profound results to the control problem itself. A key subtly lies in the ambiguity what one means by ``admissible''. While classical stochastic control theory suggests to consider controls adapted to the information generated by $(B,W)$ (for which the authors show that one may not even have a “minimizing'' sequence), the particular form of the conditioning in \eqref{equ:12} suggests to work with the augmented filtration that contains all the information of $W$ until terminal time $T$. Such controls however, if employed in \eqref{equ:11} would lead to considerations of anticipating stochastic differential equations, avoided in \cite{BM07} through stochastic flow transformations. Surprisingly perhaps, the resulting (random) value functions coincide and stochastic dynamic programming principles are seen to hold, though formulations of these results are somewhat indirect due to the said transformation approach. If one aims to connect \eqref{equ:12} with classical stochastic control theory, one can add constraints to enforce suitable adaptness of the controls, as was carried out (in case $\sigma \equiv 0$) in \cite{DFG17}, building on ideas of \cite{DB92, rogers2007pathwise}, see also \cite{AC20, BCO23, CHT24} for recent contributions to pathwise stochastic control in the context of filtering, SPDEs, and reinforcement learning, respectively. Classically, the restriction to adapted controls is tied to the interpretation of the time variable $t$ in \eqref{equ:11} and \eqref{equ:12} as {\em physical time} and the idea that one does not know the future. Numerous works in machine learning have led to other interpretations: for instance, a controlled evolution on $[0,T]$ has seen fruitful interpretations as continuous limit of a deep learning networks in the infinite layer limit, in which case the ``future'' of the noise (e.g. induced by initialization of deep neural network weights) is fully available. (It is not the purpose of this work to offer an interpretation of \eqref{equ:12} in this direction, but see \cite{bayer2023stability, gassiat2024gradient} for works in this spirit.) The purpose of this work is to revisit \eqref{equ:11} and \eqref{equ:12} through the lenses of {\em rough stochastic differential equations} \cite{FHL21}. In essence, this allows to replace $W$ in \eqref{equ:11},\eqref{equ:12} by a (purely deterministic) rough path $\BX$, while maintaining Brownian statistics for $B$, in the form of a $(\mathfrak{F}_t)$-Brownian motion on some probability space $(\Omega, (\mathfrak{F}_t)_{t\ge0}, \mathfrak{F} , \mathbb{P})$. We consider the {\em rough} SDE \footnote{ When it is clear from the context that $Y'=f(Y)$, as is usually the case for R(S)DEs, we write $(f, f') (t, Y_t) : = (f (t, Y_t), ((D_y f) f) (t, Y_t) + f' (t, Y_t))$ and sometimes abuse notation in writing $f(t,Y_t)$ rather than $(f,0)(t,Y_t)$ (note that $f' \equiv 0$ requires sufficiently regular time-dependence of $f$); cf. Section \ref{sec:RDE}.} \be dY^\BX_t = b (t,Y^\BX_t ; \eta_t) dt + \sigma (t,Y^\BX_t ; \eta_t) d B_t + (f,f') (t,Y^\BX_t) d \BX_t , \label{equ:11r} \ee and then, with $\eta \in \mathcal{A}$, essentially consisting of adapted controls, \be \mathcal{V} (s, y ; \mathbf{X} ) := \inf_{\eta \in \mathcal{A} } \mathbb{E}^{s, y} \left( g (Y^{\BX}_T) + \int_s^T \ell (t, Y^{\BX}_t, \eta_r)dr \right) , \label{equ:12r} \ee which we call {\em rough} value function. Our approach offers a variety of advantages compared to earlier works. \begin{itemize} \item No more ambiguity on the notion of admissible control: with $\BX$ being deterministic, there is only one filtration $(\mathfrak{F}_t)$. \item With \eqref{equ:11r}, direct (rough)path-wise meaning is given to \eqref{equ:11}, i.e. without imposing (ultimately irrelevant) Brownian statistics for $W$. \item Dynamic programming principle (DPP) for \eqref{equ:12r} is formulated with ``$\inf$'' rather than ``$\mathrm{essinf}$'', the existence of minimizing sequences is trivially guaranteed. \item Regularity of the value function comes directly from \eqref{equ:12r}, bypassing any need to construct careful modifications of \eqref{equ:12}. \item Reduction of excessive regularity demands \cite{CFO11}, from $f \in \C_b^4$ to $\C_b^{2+}$, in rough path stability of HJB equations. \end{itemize} All this is achieved in \cref{DPP_for_RSDEs} and \cref{sec:HJB}. Let us note here that $(f,f')$ is a deterministic controlled (in sense of Gubinelli) vector field which allows for ``rough'' time dependence in $f$. At first reading the reader may consider $f' \equiv 0$ which accommodates the case of autonomous $f$, and also Young-complementary $t$-regularity, neither of which holds if $f$ were to depend on $\mathrm{Law}(Y_t)$: the generality of our setup is precisely rooted in preparing the grounds for mean-field controlled rough stochastic differential equations, which (as also suggested in \cite{carmona2014master}) ultimately may offer new tools to the analysis of mean field games conditionally on common noise. \medskip \noindent {\em Randomization: } To some extent, this paper could have ended here, at least if one is prepared to regard a rough path as perfect model for a noise that one wants to treat as ``frozen'' analytic object. That said, the remainder of this work is devoted to the randomization of the afore-mentioned results, $ \mathbf{X} \rightsquigarrow \mathbf{W} (\omega).$ This is natural, on the one hand, since many examples of rough paths actually do come from stochastic processes, enhanced with iterated integrals or L\'evy's area, e.g. \cite{FV10}. On the other other hand, this allows to reconnect with previous works, notably \cite{BM07}, leading to a number of improvements, including \begin{itemize} \item flexible meaning to \eqref{equ:11}, without Brownian assumptions on $W$ (it is immediate to treat e.g. fractional Brownian motion in the regime $H>1/3$), \item reduction of excessive regularity demands \cite{BM07}, again from $f \in \C_b^4$ to $\C_b^{2+}$, \item modulus of continuity for the random field \eqref{equ:12r}, left as an open problem in \cite{BM07}, \item unified treatment of \eqref{equ:11} for different classes of admissible controls (not depending on $W$, adapted, $W$-anticipating), \item general understanding why all lead to the same stochastic value function, validity of stochastic DPP . \end{itemize} We start with the {\em 1st randomization} of the value function, \[ \bar{\mathcal{V}} (s, y ; \mathbf{} \omega) \assign \mathcal{V} (s, y ; \mathbf{} \mathbf{W} (\omega)) =\mathcal{V} (s, y ; \mathbf{} \mathbf{X}) |_{\mathbf{X} = \mathbf{W} (\omega)}. \] The {\em 2nd randomization} concerns the RSDE itself. To this end we move to a product space that supports a Brownian motion $B=B(\omega')$ and a random rough path $\mathbf{W} (\omega'') \perp B (\omega')$, writing $\omega = (\omega', \omega'')$. Consider now a rough SDE $Y = Y^{\eta, \mathbf{X}} (\omega')$ on $\Omega'$ such that \[ d Y_t = b (t, Y_t; \eta_t (\omega' , \BX)) d t + \sigma (t, Y_t ; \eta_t (\omega' , \BX)) d B_t (\omega') + f (t,Y_t) d \BX_t \] and write $\bar{\eta}_t (\omega) = \eta_t (\omega' , \mathbf{W} (\omega'') \mathbf{})$ and then $\bar{Y}^{\bar{\eta}} (\omega) = Y^{\bar{\eta}, \mathbf{W} (\omega'')} (\omega')$. Note $\bar{\eta}_t \in \cff'_t \vee \cff''_T$. \ Thanks to RSDE theory \cite{FHL21}, reviewed in \cref{sec:RSDE}, and a number of new measurable selection results (interesting in their own right, \cref{sec:measurable_prelim}) this defines measurable process, with \[ \bar{Y}^{\bar{\eta}}_t (\omega', \omega'') \in \cff'_t \vee \cff''_T . \] (Cf. \cref{sec:notation} for notation.) Consider controls $\eta = \eta_t (\ome' , \mathbf{X})$ in \[ \mathcal{A}^1 : = \left\{ \eta \text{ is } \mathfrak{O}' \otimes \{ \emptyset, \mathscr{C}_T \} \text{-measurable} \right\} \subset \mathcal{A}^2 : = \left\{ \eta \text{ is } \mathfrak{O}' \otimes \mathfrak{C}_T \text{-measurable} \right\} ; \] and their randomizations $\bar{\eta} = \bar{\eta}_t (\omega) \in \bar{\mathcal{A}}^i$ if $\bar{\eta}_t (\omega) = \eta_t (\omega' , \mathbf{W} (\omega''))$ for $\eta \in \mathcal{A}^i$, $i = 1, 2.$ (In other words, \(\bar{\mathcal{A}}^i\) are exactly the controls that arise from ``$\BX = \BW (\omega'')$''-randomization of controls in $\mathcal{A}^i$. This defines different classes of admissible controls, that range from no dependence to anticipating full dependence on $\BW$.) If we now define \[ \mathcal{V}^i (s, y ; \mathbf{} \omega) \assign {\tmop{essinf}_{\eta \in \mathcal{A}^i}} \mathbb{E}^{s, y} (g (\bar{Y}^{\bar{\eta}}_T) {| \cff_T^{\mathbf{W}} }) = {\tmop{essinf}_{\bar{\eta} \in \bar{\mathcal{A}}^i}} \mathbb{E}^{s, y} (g (\bar{Y}^{\bar{\eta}}_T) {| \cff_T^{\mathbf{W}} }), \] for $i = 1, 2,$ our main result in \cref{sec:pathwise-control}, valid in the full generality of (level $2$, H\"older) random rough paths, can be summarized by saying that the randomized rough value function $ \bar{\mathcal{V}}$ is a continuous modification of both $\mathcal{V}^1$ and $\mathcal{V}^2$ with a.s. regularity inherited from the regularity of the rough value function $\mathcal{V}$ (\cref{thm:RoughValueReg}). In particular, for every $(s,y) \in [0,T] \times \R^{d_Y}$, with probability one, $$ \bar{\mathcal{V}} (s, y ; \omega) = \mathcal{V}^1 (s, y ; \omega) =\mathcal{V}^2 (s, y ; \omega).$$ We furthermore see that a stochastic DPP holds for the random fields $\mathcal{V}^1$ and $\mathcal{V}^2$, essentially as consequence of the deterministic dynamic programming principle for the rough value function $\mathcal{V}$. Our final \cref{sec:BMcase} is devoted to specialize to the case when $B,W$ has jointly Brownian dynamics. More specifically, assume that $\BW$ is the (It\^o) Brownian rough path, and $\bar{\eta} = \bar{\eta}_t (\omega) \in \bar{\mathcal{A}}^1$ then $\bar{Y}^{\bar{\eta}} (\omega)$ solves the It\^o SDE \[ d Y = b (t,Y_t; \eta_t (\omega')) d t + \sigma (t,Y_t; \eta_t (\ome')) d B (\omega') + f (t,Y_t) d W (\omega'') \] and this remains true for $\mathbf{X}$-causal controls ($\mathfrak{C}_T$-optional, with $\eta_t (\ome' , \mathbf{X}) = \eta_t (\ome' , \mathbf{X}_{. \wedge t})$) in which case $\bar{\eta}$ is \ \{$\cff'_t \vee \cff''_t$\}-optional, situation(s) considered in [BM07]. For $\bar{\eta} = \bar{\eta}_t (\omega) \in \bar{\mathcal{A}}^2$, $\bar{\eta} $ is not $\cff'_t \vee \cff''_t$ adapted, so that $\bar{Y}^{\bar{\eta}}$ can be seen as solution to some anticipating SDE: this is provided by our randomized RSDE approach, with no reliance on any results from anticipating stochastic calculus, or stochastic flow. Randomized RSDEs thus also provide a unification, that deals simultaneously with {partially anticipating coefficients} and {non-Brownian W-noise} (independent of Brownian $B$-noise). \medskip \noindent {\bf Acknowledgement}: PKF and HZ acknowledge support from DFG CRC/TRR 388 ``Rough Analysis, Stochastic Dynamics and Related Fields'', Projects A07, B04 and B05. Part of this work was carried out during a visit of the first author to Shandong University. KL acknowledges supports from EPSRC [grant number EP/Y016955/1] and from the Humboldt fellowship while at TU Berlin where this project was commenced. HZ is partially supported by NSF of China and Shandong (Grant Numbers 12031009, ZR2023MA026), Young Research Project of Tai-Shan (No.tsqn202306054). \section{Setups and notations} \label{sec:notation} A filtered probability space is denoted by $(\Omega, \cff, (\cff_t)_{t \geq 0}, \P)$ and said to satisfy the {\em usual conditions} if $\cff_t = \bigcap_{s > t} \cff_s \ \text{for all } t \geq 0$ and each \(\cff_t\) contains all \( \mathbb{P} \)-null sets of \( \cff \). Given generic measure spaces $(M_i,\mmm_i), i =1,2$ we say $f:M_1 \to M_2$ is $\mmm_1 / \mmm_2$-measurable if $f^{-1} (\mmm_2) \subset \mmm_1$. If $U$ is a topological space, we equip if with its Borel sets, $\uuu = \bbb (U)$, unless otherwise stated; if furthermore $U$ is Polish, we call $(U,\uuu) = (U,\bbb (U))$ a {\em Polish measure space}. We also write $\bbb_T := \bbb ([0,T])$ and $\bbb^d := \bbb (\R^d)$. Throughout the paper, \(T>0\) is a fixed time horizon and \(\Delta_T\) denotes the simplex \(\{(s,t)\in[0,T]^2:s\le t\}\). For a path \((Z_t)_{t\in[0,T]}\) and a doubly parameter path \((A_{s,t})_{(s,t)\in \Delta_T}\), we denote \(\delta Z_{s,t}\) for the increment \(Z_t-Z_s\) for each \(s,t\) and for each \(\beta>0\) we set \begin{align*} |A|_\beta=\sup_{(s,t)\in \Delta_T}\frac{|A_{s,t}|}{(t-s)^\beta}. \end{align*} Write $\mathscr{C}^\alpha_T = \mathscr{C}^\alpha_T (\R^d)$ for the space of $\alpha$-H\"older rough paths on $[0,T]$ over $\R^d$, and $\mathscr{C}^{0,\alpha}_T$ for the (Polish) subspace of (obtained as the closure of smooth rough paths). When $\alpha$ is fixed, we also write $\mathscr{C}_T := \mathscr{C}^{0,\alpha}_T$ and $\ccc_T$ for its Borel sets. The $\alpha$-H\"older rough path metric is given by \begin{equation} \label{def.rho_metric} \rho_{\alpha}(\BX,\bar\BX)=|\delta X- \delta\bar X|_\alpha+|\XX-\bar\XX|_{2\alpha}, \end{equation} the homogenous rough path norm by $ \nn{\BX}_\alpha= |\delta X|_\alpha \vee \sqrt{|\XX|_{2 \alpha}}$. We will be interested in (adapted) stochastic processes that depend additionally on $\mathbf{X} \in \mathscr{C}_T$ as parameter; we then call a process $g$ {\em causal in} $\BX$ (short: $\BX$-causal) if $g(t,\omega; \BX) = g(t,\omega;\BX_{\cdot \wedge t})$. For some normed vector spaces \(K,\bar{K}\), we denote by \(\C_b(K,\bar K)\) the space continuous bounded functions from \(K\) to \(\bar K\) equipped with the norm \begin{align*} f\mapsto |f|_\infty=\sup_{y\in K}|f(y)|. \end{align*} For each real number \(\gamma>0\), we write $\C_b^\gamma (K;\bar K)$ for the classical Lipschitz space of functions from \(K\) to \(\bar K\) with regularity $\gamma$. More precisely, for $\gamma=N+\beta$ where $N$ is a non-negative integer and $0<\beta\le 1$, $\C^\gamma_b(K;\bar K)$ consists of bounded functions $f\colon K\to \bar K$ such that $f$ has Fr\'echet derivatives up to order $N$, $D^jf$, $j=1,\ldots,N$ are bounded functions and $D^Nf$ is globally H\"older continuous with exponent $\beta$. For each $f$ in $\C_b^\gamma$, we denote \[ [f]_\gamma=\sum\nolimits_{k=1}^N|D^kf|_\infty +\sup_{x,y\in K}\frac{|f(x)-f(y)|}{|x-y|^\beta} \tand |f|_\gamma=|f|_\infty+[f]_\gamma. \] \section{Snapshots of RDEs and RSDE} \label{sec:RSDE} We briefly review the main results for rough (stochastic) differential equations that are closely relevant to our study. Our goal is to present simpler statements with more straightforward conditions while still achieving sufficiently general conclusions. \subsection{Rough paths and its integration} \label{sec:RDE} The purely deterministic theory of rough paths \cite{MR1654527} gives well-posedness to rough differential equations (RDEs) of the form \begin{equation}\label{eq:RDE} dY_t= b_t (Y_t)dt + f_t(Y_t)d\BX_t ,\quad t\in[0,T]. \end{equation} Here $\BX = (X,\XX) \in \mathscr{C}^\alpha$ is a $\alpha$-H\"older rough path, $\alpha \in (1/3,1/2]$. The solution $Y=Y^\BX$ is an example of a {\em controlled rough path} \cite{MR2091358}, or simply {\em controlled} (w.r.t. $X$), in the sense that it looks like $X$ on small scales: $Y_t \approx Y_s + Y_s' (X_t - X_s)$, with $Y'_s := f_s (Y_s)$. Crucially, the definition of $\int f (Y) d \BX$, and then integral meaning to \eqref{eq:RDE}, requires $f(Y)$ itself to be controlled. Many works on this subject, including \cite{FH20}, consider autonomous situation where $f_t(\cdot) \equiv f(\cdot)$, but there is no difficulty in assuming $f_t (\cdot) \approx f_s(\cdot) + f'_s (\cdot) (X_t -X_s)$ to accommodate (controlled) rough time dependence, in which case we write $$ dY_t= b_t (Y_t)dt + (f_t,f_t')(Y_t)d\BX_t . $$ With $Y'_s = f_s (Y_s)$ and $Y''_s = ((Df_s) f_s +f'_s) (Y_s)$, one expects a Davie-type expansion $$ Y_t = Y_s + Y_s' (X_t - X_s) + Y''_s \XX_{s,t} + o(t-s). $$ In quantified form, cf. \cite{MR2387018,FH20}, this characterizes RDE solutions. \begin{rem} \label{rem:nodimRDE} We tacitly assume that $X$ and $Y$ take values in $\mathbb{R}^{d_X}$ and $\mathbb{R}^{d_Y}$, respectively, and may write $\BX \in \mathscr{C}^\alpha([0,T];\mathbb{R}^{d_X})$ to be fully explicit. Viewing $b(t, . ): \mathbb{R}^{d_Y} \to \mathbb{R}^{d_Y}$ as (time-dependent) vector field, we also (ab)use this terminology for $f(t,. ): \mathbb{R}^{d_Y} \to \mathrm{Lin}(\mathbb{R}^{d_X}, \mathbb{R}^{d_Y})$. We typically drop this explicit information about the dimensions (as it only enters trivially in the argument, in dealing with summations implicit in expressions like $Y_s' (X_t - X_s)$ or $Y''_s \XX_{s,t}$). \end{rem} Important examples of rough paths come from the typical realization of a multidimensional Brownian motion enhanced with iterated (It\^o) integrals, \[ \BX = (X, \X) = \left(B(\omega), (\int \delta B \otimes d B) (\omega) \right)=: \mathbf{B}^{\text{It\^o}} (\omega). \] As is well-known, e.g. \cite[Ch.9]{FH20}, under natural conditions, $\bar{Y} (\omega) := Y^\BX|_{\BX = \mathbf{B}^{\text{It\^o}} (\omega)}$ yields a (beneficial) version of the It\^o solution to $dY_t = b_t (Y_t) dt + \sigma_t (Y_t) dB_t$, the Stratonovich case is similar. \subsection{Rough stochastic differential equations (RSDEs)} Let $(\Omega,\cff,(\cff_t)_{t \in [0,T]},\mathbb{P})$ be a complete filtered probability space that supports a $\mathbb{R}^{d_B}$-dimensional Brownian motion $B$. It was an open problem until \cite{FHL21} to provide a unified approach to SDEs and RDEs, such as to give intrinsic meaning and well-posedness to {\em rough stochastic differential equations} (RSDEs), aiming for an adapted solution process $Y=Y^{\BX} (\omega)$ to \begin{equation}\label{eq:dRDE} d Y_t = b_t (Y_t)dt + \sigma_t ( Y_t) dB_t + (f_t,f_t')(Y_t) d \BX_t . \end{equation} The coefficients \(b,\sigma,f,f'\) are allowed to be random but progressively measurable. As before, $\BX \in \mathscr{C}^\alpha$ with $\alpha \in (1/3,1/2]$ and again (cf. \cref{rem:nodimRDE}) we need not be explicit about the dimensions. The afore-mentioned paper provides both a Davie expansion, which quantifies $$ \delta Y_{s, u} \approx \int_s^u b_t (Y_t) dt + \int_s^u \sigma_t (Y_t) dB_t + f_s (Y_s) \delta X_{s, u} + ((D f) f + f') (Y_s) \mathbb{X}_{s, u}, $$ and a rough stochastic integration theory which gives intrinsic meaning and estimates for rough stochastic integrals, $$ \int_s^u (Z, Z') d \BX \approx Z_s \delta X_{s, u} + Z'_s\mathbb{X}_{s, u} $$ where $(Z,Z')$ is a so-called stochastic controlled rough path (see \cref{def:stochasticcontrolledroughpaths} below). It is given by $Z=f(Y),Z' = ((D f) f + f')(Y)$ in the above example and provides integral meaning to \eqref{eq:dRDE}. It is an important feature of that only $f$ needs to be Taylor-expanded, whereas $b,\sigma$ can have progressive $(t,\omega)$ dependence and only need to have Lipschitz spatial regularity, as in classical It\^o theory, to have well-posedness. \begin{rem} The progressive generality (in $b,\sigma)$ is crucial if one considers stochastic control problems. A commonly used trick is to regard \eqref{eq:dRDE} as RDE driven by a random rough path,``$(B,\BX)$'' $\in \mathscr{C}([0,T];\mathbb{R}^{d_B + d_X})$ obtained by a joint lift of $\mathbb{R}^{d_B}$-dimensional Brownian motion $B$ and $\BX \in \mathscr{C}([0,T];\mathbb{R}^{d_X})$, essentially by supplying the missing integrals $\int \delta B d B, \int \delta X dB$ by It\^o integration, $\int \delta B dX$ via integration by parts. However, this method fails to treat such general setup: a progressive path-dependent coefficient field $\sigma (t,\omega)$, comes with no H\"older or $p$-variation regularity in $t$ whatsoever, leave alone controlled $t$-dependence, which prohibits the use of this joint lifting method. \end{rem} We now review some key material of \cite{FHL21}, to the extent needed later on. Let \(p \in [2,\infty)\) and \(q \in [p,\infty]\), and lets \(\mathcal{G}\subseteq\cff\) be a sub-\(\sigma\)-field. Given a random variable \(\xi\), we define—if it exists—its conditional \(L^p\)-norm with respect to \(\mathcal{G}\) as the (unique) \(\mathcal{G}\)-measurable \(\mathbb{R}\)-valued random variable given by \[ \|\xi|\mathcal{G}\|_{p} := \mathbb{E}\left( |\xi|^p|\mathcal{G}\right)^{\frac{1}{p}}. \] In particular, the mixed \(L_{p,q}\)-norm \(\|\|\xi|\mathcal{G}\|_p\|_q\) denotes the \(L^q(\Omega)\)-norm of \(\|\xi|\mathcal{G}\|_p\). Let \(\kappa,\kappa' \in [0,1]\) and let \(X :[0,T]\to \R^{d_X} \) be an \(\alpha\)-H\"older continuous path. \begin{defi} \label{def:stochasticcontrolledroughpaths} {\cite[Definition 3.1]{FHL21}} A pair \((Z,Z')\) is called \textit{stochastic controlled rough path} of $(p,q)$-integrability and $(\kappa,\kappa')$-H\"older regularity if \begin{enumerate} \renewcommand{\labelenumi}{\alph{enumi})} \item \(Z=(Z_t)_{t \in [0,T]}\) is progressively measurable such that \[ \|\delta Z\|_{\kappa;p,q} := \sup_{0 \leq s < t \leq T} \frac{\|\|\delta Z_{s,t}|\cff_s\|_p\|_q}{|t-s|^\kappa} < +\infty; \] \item \(Z'=(Z'_t)_{t \in [0,T]}\) is progressively measurable such that \[ \sup_{t \in [0,T]} \|Z'_t\|_q < +\infty \quad \text{and} \quad \|\delta Z'\|_{\kappa';p,q} := \sup_{0 \leq s < t \leq T} \frac{\|\|\delta Z'_{s,t}|\cff_s\|_p\|_q}{|t-s|^{\kappa'}} < +\infty; \] \item writing \(R^Z_{s,t} = \delta Z_{s,t} - Z'_s \delta X_{s,t}\) for \((s,t)\in\Delta_T\), we have \[ \|\mathbb{E}_\cdot R^Z\|_{\kappa+\kappa';q} := \sup_{0 \leq s < t \leq T} \frac{\|\mathbb{E}_s(R^Z_{s,t})\|_q}{|t-s|^{\kappa+\kappa'}} < + \infty. \] \end{enumerate} We write \((Z,Z') \in \mathbf{D}_X^{\kappa,\kappa'} L_{p,q}\); and also \(\mathbf{D}_X^{2\kappa} L_{p,q}\), \(\mathbf{D}_X^{\kappa,\kappa'} L_p\) if \(\kappa = \kappa'\) and \(p=q\), respectively. \end{defi} A seminorm is then defined by \[ \|(Z,Z')\|_{\mathbf{D}_X^{\kappa,\kappa'}L_{p,q}} := \|\delta Z\|_{\kappa;p,q} + \|\delta Z'\|_{\kappa';p,q} + \|\mathbb{E}_\cdot R^Z\|_{\kappa+\kappa';q}. \] Given additionally \((\bar{Z},\bar{Z}') \in \mathbf{D}_{\bar{X}}^{\kappa,\kappa'} L_{p,q}\) for some \(\bar{X} \in C^\alpha([0,T];\mathbb{R}^{d_X})\), we define \begin{align*} \|Z,Z';\bar{Z},\bar{Z}'\|_{X,\bar{X};\kappa,\kappa';p} &:= \|\delta (Z-\bar{Z})\|_{\kappa;p} +\| Z'-\bar{Z}'\|_{\kappa';p} + \|\mathbb{E}_\cdot R^Z - \mathbb{E}_\cdot \bar{R}^{\bar{Z}}\|_{\kappa,\kappa';p}, \end{align*} where \(\bar{R}^{\bar{Z}}_{s,t} = \delta \bar{Z}_{s,t} - \bar{Z}'_s \delta \bar{X}_{s,t}\). If \(\bar{X} = X\), we often write \(\|\ \cdot \ ; \ \cdot \ \|_{X;\kappa,\kappa';p}\) instead of \(\|\ \cdot \ ; \ \cdot \|_{X,X;\kappa,\kappa';p}\). When \(X\) has a rough path lift \(\BX \in \CC^\alpha\), the integration of \((Z,Z')\) against \(\BX\) is a rough stochastic integral defined rigorously in the following result, which is an excerpt from {\cite[Theorem 3.5]{FHL21}}. \begin{proposition}\label{prop.rsint} Suppose that \((Z,Z')\) belongs to \(\mathbf{D}_X^{\kappa,\kappa'} L_{p,q}\) with \(\alpha+\kappa>1/2\), \(\alpha+\min(\alpha,\kappa)+\kappa'>1\). Then the Riemann sums \begin{align*} \sum_{[u,v]\in\cpp, u\le t}Z_u X_{u,v\wedge t}+Z'_u\XX_{u,v\wedge t} \end{align*} converges uniformly in time \(t\in[0,T]\) in \(L_p\), as \(|\cpp|\) goes to 0, to a continuous adapted process, denoted by \(\int_0^\cdot (Z,Z')d\BX\). \end{proposition} We now define integrable solutions to \eqref{eq:dRDE}. \begin{defi} \label{def:solutionRSDEs_compendium}{\cite[Definition 4.2]{FHL21}} An $L_{p,q}$-integrable solution of \eqref{eq:dRDE} on $[0,T]$ is a continuous adapted process $Y$ such that the following conditions are satisfied: \begin{enumerate}\renewcommand{\labelenumi}{\alph{enumi})} \item $\int_0^T |b_r(Y_r)|dr$ and $\int_0^T |(\sigma \sigma^\top)_r(Y_r)|dr$ are finite $\mathbb{P}$-a.s.; \item $(f(Y),D_y f(Y)(f(Y))+f'(Y))$ belongs to $\mathbf{D}_X^{\bar\alpha,\bar\alpha'}L_{p,q}$, for some $\bar\alpha,\bar\alpha'\in(0,1]$ such that $\alpha+\min\{\alpha,\bar\alpha\}>\frac12$ and $\alpha+\min\{\alpha,\bar\alpha\}+\bar\alpha'>1$; \item $Y$ satisfies the following stochastic Davie-type expansion for every $(s,t)\in \Delta_T$: \begin{equation*} \begin{aligned} \delta Y_{s,t} &= \int_s^t b_r(Y_r)dr + \int_s^t \sigma_r(Y_r)dB_r \\ & \quad + f_s(Y_s) \delta X_{s,t} + (D_y f_s(Y_s)(f_s(Y_s))+f'_s(Y_s))\mathbb{X}_{s,t} + Y^\natural_{s,t}, \end{aligned} \end{equation*} where \begin{equation*} \|\|Y^\natural_{s,t}|\cff_s\|_p\|_q=o(|t-s|^\frac{1}{2}) \qquad \text{and} \qquad \|\mathbb{E}_s(Y^\natural_{s,t})\|_q=o(|t-s|). \end{equation*} \end{enumerate} When the starting position $Y_0=\xi$ is specified, we say that $Y$ is a solution starting from $\xi$. Furthermore, by \cite[Proposition 4.3]{FHL21}, c) can be replaced by \begin{itemize} \item[c')] $\mathbb{P}$-almost surely and for any $t \in [0,T]$, \begin{equation*} Y_t = \xi + \int_0^t b_r(Y_r) dr + \int_0^t \sigma_r(Y_r) dB_r + \int_0^t (f_r(Y_r),D_y f_r(Y_r)(f_r(Y_r)) + f'_r(Y_r)) d\mathbf{X}_r. \end{equation*} \end{itemize} \end{defi} To state the well-posedness result for \eqref{eq:dRDE}, we recall some terminologies related to the regularity of vector fields. \begin{definition} \label{def:stochasticboundedandLipschitzvectorfield} {\cite[Definition 4.1]{FHL21}} Let $W,\bar W$ be some finite dimensional Euclidean spaces and fix a Borel set \( S\subset W \). Let \( (t,\omega)\mapsto g_t(\omega,\cdot) \) be a progressively measurable stochastic process from \( \Omega\times [0,T] \to \Cb(S;\bar W)\). We say that: \begin{enumerate}[label=(\alph*)] \item $g$ is \textit{random bounded continuous} if is uniformly bounded, namely, there exists a deterministic constant $\|g\|_\infty$ such that \[ \sup_{t\in[0,T]}\esssup_{\omega\in \Omega}\sup_{x\in S}|g_t(\omega,x)|\le\|g\|_\infty. \] \item $g$ is \textit{random bounded Lipschitz} if it is random bounded continuous, progressively measurable from \( \Omega\times[0,T]\to \Cb^1 (S;\bar W) \) and uniformly bounded in the sense that \[ \sup_{t\in[0,T]}\esssup_{\omega\in \Omega}\sup_{x,\bar x\in S}\frac{|g_t(\omega,x)-g_t(\omega,\bar x)|}{|x-\bar x|}\le \|g\|_\lip \] for some constant \( \|g\|_{\lip} \). \end{enumerate} \end{definition} \begin{definition}[Stochastic controlled vector fields, {\cite[Definition 3.8]{FHL21}}]\label{def.scvec} Let $\beta, \beta' \in (0,1]$ and $\gamma >1$ be some fixed parameters. We call $(f,f')$ {\em stochastic controlled vector field on \( \R^d \)} and write \( (f,f')\in \D^{\beta,\beta'}_XL_{p,q}\C^\gamma_{b}\) if the following conditions are satisfied. \begin{enumerate}[label=(\alph*)] \item\label{scvec.f} The pair \[ (f,f')\colon\Omega \times [0,T] \to \C^\gamma_{b}(\R^{d}; \R^m) \times \C^{\gamma-1}_{b} (\R^d ; \mathrm{Lin}(\R^{d_X},\R^m)) \] is progressively measurable in the strong sense and uniformly $n$-integrable in the sense that \begin{align}\label{def.normff} \|(f,f')\|_{\gamma;q}:=\sup_{s\in [0,T]}\||f_s|_{\gamma}\|_q + \sup_{s\in [0,T]}\||f'_s|_{\gamma-1}\|_q \end{align} is finite. \item\label{scvec.brakets} Letting \begin{align}\label{def.bk} \bk{Z}_{\zeta;p,q}:=\sup_{(s,t)\in \Delta_T:s\neq t}\frac{\Big\|\big\|\sup_{y\in \Rd}|Z_{s,t}(y)|\,\big|\,\cff_s \big\|_p\Big\|_q}{(t-s)^\zeta}\,, \end{align} the quantities $\bk{\delta f}_{\beta;p,q}$, $\bk{\delta f'}_{\beta';p,q}$, \( \bk{\delta Df}_{\beta';p,q} \) are finite. \item\label{scvec.remainder} The map $(s,t)\mapsto \E_s R^f_{s,t}=\E_sf_t-f_s-f'_s \delta X_{s,t}$ satisfies \[ \bk{\E_\bigcdot R^f}_{\beta+\beta';q} =\sup_{(s,t)\in \Delta_T:s\neq t}\frac{\|\sup_{y\in \Rd}|\E_sR^{f}_{s,t}(y)|\|_q}{(t-s)^{\beta+\beta'}} <\infty\,. \] \end{enumerate} \end{definition} For stochastic controlled vector fields as above, we introduce the quantity \begin{equation} \label{def.norms_scvf} \bk{(f,f')}_{X;\beta,\beta';p,q} :=\bk{\delta f}_{\beta;p,q}+\bk{\delta Df}_{\beta';p,q}+\bk{\delta f'}_{\beta';p,q}+\bk{\E_\bigcdot R^f}_{\beta+\beta';p,q}, \end{equation} which is abbreviated as $\bk{(f,f')}_{X;\beta,\beta';p}$ if $p=q$, as $\bk{(f,f')}_{X;2\beta;p,q}$ if \( \beta=\beta' \) and as $\bk{(f,f')}_{X;2\beta;p}$ when both conditions are met. Furthermore, if $(\bar f,\bar f')\in \D^{\beta,\beta'}_{\bar X}L_{p,\infty}\C^\gamma_b$ for another such \(\bar X\in C^\alpha(V) \), we define \begin{equation} \label{def.bk_metric} \begin{aligned} \bk{f,f';\bar f,\bar f'}_{X,\bar X;\beta, \beta';p}&=\bk{\delta f-\delta \bar f}_{\beta;p}+\bk{\delta f'-\delta \bar f'}_{\beta';p}+\bk{\delta Df-\delta D\bar f}_{\beta';p} \\&\quad+\bk{\E_\bigcdot R^f-\E_\bigcdot \bar R^{\bar f}}_{\beta+\beta';p}. \end{aligned} \end{equation} \begin{definition}[Controlled vector fields] In the case of deterministic vector fields, stochastic controlled vector fields reduce to controlled vector fields and we write \(\D^{\beta,\beta'}_X\C^\gamma_{b}\) instead of \(\D^{\beta,\beta'}_XL_{p,q}\C^\gamma_{b}\). In such cases, we will also drop the indices \(p,q\) in the notations in \eqref{def.normff}-\eqref{def.bk_metric}. \end{definition} The following results on well-posedness of \eqref{eq:dRDE} are consequences of more general ones from \cite{FHL21}. \begin{thm}[see {\cite[Theorem 4.6 and Proposition 4.5]{FHL21}}] \label{thm:wellposed} Let $\BX \in \mathscr{C}^\alpha, \alpha \in (1/3,1/2]$ and $p \in [2,\infty)$. Assume\footnote{For simplicity, take \(\beta=\alpha\) at first reading.} that $\beta \in [0,\alpha]$ and $\gamma \in \left(\frac{1}{\alpha},3 \right]$ such that $\alpha + (\gamma-1)\beta > 1 $. Let $b$ and $\sigma$ be random bounded Lipschitz vector fields. Let $(f, f')$ be a stochastic controlled vector field in \(\mathbf{D}_X^{2\beta} L_{p,\infty} \C_b^\gamma\) such that \((D_yf,D_yf')\) belongs to \(\mathbf{D}_X^{2\beta} L_{p,\infty} \C_b^{\gamma-1}\). Then, for any $\xi \in L^p(\Omega,\cff_0;\mathbb{R}^d)$, there exists a unique \(L_{p,\infty}\)- integrable solution $Y$ to \eqref{eq:dRDE}. Moreover, \begin{align}\label{est.apri.m} \|(Y,Y')\|_{\mathbf{D}_X^{\alpha,\beta}L_{p,\infty}} \lesssim_{T,p,\alpha,\beta,\gamma} \left(1+\|b\|_\infty + \|\sigma\|_\infty + M \nn{\BX}_\alpha \right)^{\frac{1}{\beta}}, \end{align} where \(M>0\) is any constant such that \(\| (f, f') \|_{\gamma-1;\infty} + \llbracket(f, f') \rrbracket_{X;2\beta; p,\infty} \leq M\). \end{thm} \begin{thm}[see {\cite[Theorem 4.9]{FHL21}}] \label{thm:stabilityforRSDEs_compendium} In the setting of \cref{thm:wellposed}, let \(\bar{\mathbf{X}} = (\bar{X}, \bar{\mathbb{X}}) \in \mathscr{C}^\alpha([0,T]; \mathbb{R}^{d_X})\), let \(\bar{b}\) and \(\bar{\sigma}\) be two random bounded continuous vector fields and let \((\bar{f}, \bar{f'}) \in \mathbf{D}_{\bar{X}}^{2\beta} L_{p,\infty} \C_b^\gamma\). Denote by \(\bar{Y} = (\bar{Y}_t)_{t \in [0,T]}\) the solution of the rough SDE starting from \(\bar{\xi} \in L^p(\Omega,\cff_0;\mathbb{R}^d)\) and driven by \(\bar{\mathbf{X}}\), with coefficients \(\bar{b}, \bar{\sigma}\) and \((\bar{f}, \bar{f'})\). Then \[ \begin{aligned} \|\sup_{t\in[0,T]}| Y_{t}- \bar Y_{t}|\|_p&+ \|Y, f(Y); \bar{Y}, \bar{f}(\bar{Y})\|_{X, \bar{X}; \alpha, \beta; p} \\ &\lesssim_{M, \alpha, \beta, p, T} \|\xi - \bar{\xi}\|_p + \rho_\alpha(\mathbf{X}, \bar{\mathbf{X}}) \\&\quad+ \sup_{t \in [0,T]} \| |b_t(\cdot) - \bar{b}_t(\cdot)|_\infty \|_p +\sup_{t \in [0,T]} \| |\sigma_t(\cdot) - \bar{\sigma}_t(\cdot)|_\infty \|_p \\&\quad + \|(f - \bar{f}, f' - \bar{f'}) \|_{\gamma-1; p} + \llbracket f, f'; \bar{f}, \bar{f}' \rrbracket_{X, \bar{X}; 2\beta; p}, \end{aligned} \] where \(M > 0\) is any constant such that \[ \nn{\BX}_\alpha + \nn{\bar \BX}_\alpha + \|b\|_{\lip} + \|\sigma\|_{\lip} + \| (f, f') \|_{\gamma; \infty} + \llbracket (f, f') \rrbracket_{X; 2\beta; p; \infty}+\bk{(Df,Df')}_{X;2\beta;p,\infty} \leq M. \] \end{thm} \section{Measurable selection for RSDEs} \label{sec:measurable_prelim} When solving RSDEs, the rough path is fixed and plays the role of a deterministic parameter. In the randomization step, one needs to substitute the rough path by a random process which requires certain measurability properties of the solutions to RSDEs with respect to the rough path input. We provide answers to these measurability issues in this section, building upon the classical results from Stricker--Yor \cite{SY78}, which we revise for the non-french reader's convenience in the appendix. Let \((U,\uuu)\) be a measurable space and \((M,\mmm)\) be a Polish measurable space. In later sections, the abstract measurable parameter space $(U, \uuu)$ is taken as the rough path space $\MC^{\alpha}$, equipped with its Borel sets, $(M,\mmm)$ is often just $(\R^d , \bbb^d)$. \begin{defi} \label{def:optional} Let $(\Omega, \cff, (\cff_t)_{t \geq 0}, \P)$ be a filtered probability space satisfying the usual conditions. A measurable map $$ Y:(([0,T] \times \Omega) \times U , (\bbb_T \otimes \cff) \otimes \uuu ) \to (M,\mmm) , \quad (t,\omega,u) \mapsto Y^u(t,\omega) $$ is called {\em $\uuu$-optional} if it is $\ooo \otimes \uuu / \mmm$-measurable where $\ooo$ denotes the optional $\sigma$-field on $[0,T] \times \Omega$. We say that $Y$ has a {\em $\uuu$-optional version} if there exists $ ^{\circ}Y$ which is $\uuu$-optional and {\em indistinguishable} from $Y$, in the sense that, for all $u \in U$, the processes $ ^{\circ} Y^u$ and $Y^u$ are indistinguishable. \end{defi} Thanks to Theorem \ref{thm:wellposed}, under the conditions stated there, there exists a unique \(L_{p,\infty}\)-integrable solution on $[0,T]$ to \footnote{As earlier, $(f_r, f_r') (Y_r ; \BX) = (f_r (Y_r ; \BX), ((D_y f_r) f_r) (Y_r ; \BX) + f_r' (Y_r ; \BX ) )$.} \be\label{rand-rsde} Y_t = \xi + \int_0^t b_r(Y_r; \BX) \, dr + \int_0^t \sigma_r(Y_r; \BX)\, dB_r + \int_0^t \Big(f_r, f'_r \Big)(Y_r; \BX) d\mathbf{X}_r, \ee for any fixed rough path $\BX \in \mathscr{C}_T = \mathscr{C}^{0,\alpha}([0,T])$. To show that the solution is measurable with respect to \(\BX\), we allow $\xi$ and $g \in \{ b, \sigma, f,f' \}$ to depend on $\BX$ in a measurable way. Specifically, we assume that $g=g(t,\omega,y,\BX)$ to be $\bbb^{d_Y} \otimes \ccc_T$-optional in the above sense with $U = \R^{d_Y} \times \mathscr{C}_T$, equipped with the obvious product $\sigma$-field. (All assumptions on $g=g(\cdot,\cdot,\cdot,\BX)$ for fixed $\BX$ imposed by Theorem \ref{thm:wellposed} remain.) These conditions are stated precisely in \cref{assum-jt-measu} below. Note that, at this stage, we do not assume that $g$ is causal in $\BX$. \begin{exam} Consider a control $\eta = \eta (t,\omega,\BX)$ which is $\ccc_T$-optional, with values in some Polish measurable space $(A,\aaa)$, and measurable fields $g \in \{ b, \sigma, f, f' \}$, with $ g(t,y,a)$ defined on $[0,T] \times \R^{d_Y} \times A$. Then $g(t,\ome,y;\BX):= g(t,y,\eta(t,\ome,\BX))$ is $\bbb^{d_Y} \otimes \ccc_T$-optional. \end{exam} \begin{assumption}\label{assum-jt-measu} Assume that $\xi:\Omega \times \MC_T \rightarrow \R^{d_Y}$ is $\MF_0 \otimes \ccc_T/\bbb^{d_Y}$-measurable, and for any $\BX \in \MC_T,$ $\xi(\cdot,\BX) \in L_p$. Assume that $g\in \{b,\sigma,f,f'\}$ defined on $[0,T] \times \Omega \times \R^{d_Y} \times \MC_T$ is $\bbb^{d_Y} \otimes \ccc_T$-optional. Moreover, we assume the following. \begin{itemize} \item[$(1)$.] $(b,\sigma)$ is bounded and uniformly Lipschitz in $y$ in the sense that for any $y_1, y_2 \in \R^{d_Y},$ $g\in \{b,\sigma\},$ $$ \sup_{(t,\BX)\in [0,T]\times \CC_T}\esssup_{\omega\in \Omega} |g(t,\omega,y_1;\BX )- g(t,\omega,y_2; \BX )| \le \|g\|_\lip |y_1-y_2| $$ for some positive constant \(\|g\|_\lip\). \item[$(2)$.] For any $\BX \in \MC_T$, $(f,f') (\cdot \ ;\BX) $ belongs to $ \BD_X^{2\beta }L_{p,\infty} \C^{\gamma}_b$ for some $p \ge 2,$ $\beta \in (0,\alpha]$, $\gamma > \frac{1}{\alpha}$ such that $\alpha+\beta>\frac12$ and $\alpha + (\gamma-1)\beta>1.$ \end{itemize} \end{assumption} \begin{exam} We give examples of $(f,f')$ which satisfies \cref{assum-jt-measu}, (2). \begin{enumerate}[(i)] \item If there is a finite constant \( C\) so that additionally \(f\) satisfies \begin{align*} \esssup_{\omega}\sup _{y}|f(t,\omega,y)-f(s,\omega,y)|\le C(t-s)^{2 \beta} \end{align*} for every \((s,t)\in \Delta_T\), then \((f,f'):=(f,0)\) belongs to $ \BD_X^{2\beta }L_{p,\infty} \C^{\gamma}_b$. \item Let $h: \R^{d_Y} \times \R^{d_X} \rightarrow \R^d$ be in $\C^3_b$. Then $(t,\ome,y,\BX) \mapsto (f,f')(t,\ome,y;\BX) := ( h(y ;X_t), D_x h(y; X_t) ) $ belongs to $ \BD^{2\alpha}_X \C^3_b$. \item Let $\ell : \R^{d_B} \rightarrow \R$ be in $\C^{3}_b.$ Then $(t,\ome,y,\BX) \mapsto (f,f')(t,\ome,y;\BX):= (\ell(B_t(\ome)), 0) $ belongs to $ \BD_X^{1/2,1/2} L_{2,\infty} \C_b^{\gamma}$ (equivalently in our notation, \(\BD_X^{1} L_{2,\infty} \C_b^{\gamma}\)) for any \(\gamma>0\). \end{enumerate} \end{exam} The following result is the main result of the current section, which shows that the solution to \eqref{rand-rsde} has a \(\ccc_T\)-optional version. \begin{thm}[Measurable selection] \label{thm-rsde-optional} Suppose that \cref{assum-jt-measu} holds. For each \(\BX\in \CC_T\), let $Y=Y (t,\omega; \BX)$ be the solution to \eqref{rand-rsde} on $[0,T]$ which is supplied by \cref{thm:wellposed}. Then $Y$ has a $\ccc_T$-optional version (denoted by $\tY$) which is $\P$-a.s. continuous for any fixed $\BX \in \MC_T$. In particular, $\tY_T$ is $ \cff_T \otimes \ccc_T / \bbb^{d_Y}$-measurable. \end{thm} We prepare the proof with the following measurable selection result, closely related to \cite{SY78} which we include for the reader's convenience as \cref{SY_Prop1} in the appendix (see also \cite[Theorem62]{MR2273672}). Recall the measurable parameter space $(U, \uuu)$. \begin{lem}\label{optional-limit} Suppose that $Z^n:[0,T] \times \Omega \times U \rightarrow \R^d$ has a $ \uuu$-optional version in the sense of Definition \ref{def:optional}, for each $n \in \N$. Assume for each $u \in U, $ $\{Z^n(\cdot, u)\}_n$ are c\`adl\`ag, and converge to $Z(\cdot,u)$ uniformly in time in $\P$-probability, where $Z:[0,T] \times \Omega \times U \rightarrow \R^d$. Then $Z$ has a $ \uuu$-optional version, which is $\P$-a.s. c\`adl\`ag for each $u \in U.$ \end{lem} \begin{proof} Without loss of generality, we assume $Z^n$ is $ \uuu$-optional. For each \(i,j\), define the process $$ \Delta_t^{i,j}(\ome, u):=|Z^i-Z^j|(t,\ome,u) $$ which is $\uuu$-optional. Put $n^u_0:=1, $ and for any $k \ge 1,$ define $$ n_k^u:= \inf \Big\{ n> (k \vee n_{k-1}^u): \sup_{i,j \ge n} \P(\sup_{t\in[0,T]}\Delta^{i,j}_t(u) > 2^{-k} ) \le 2^{-k} \Big\}. $$ Since for any $u$, $\Delta^{i,j}(u)$ is c\`adl\`ag and $\Delta^{i,j}$ is $\uuu$-optional, the mapping $$(u,n) \mapsto \sup_{i,j \ge n} \P(\sup_{t\in[0,T]}\Delta^{i,j}_t(u) > 2^{-k} )$$ is $\uuu \otimes \bbb(\N)/ \bbb^{1}$-measurable. It follows that the map \(u\mapsto n^u_k\) from \( U\) to \(\N\) is measurable, and thus for each \(k\), the process \(\tZ^k\), defined by $\tZ^k_t(\ome,u):= Z^{n^u_k}_t(\ome,u)$ for any $(t,\ome,u)$, is $\uuu$-optional. Now let \be \tZ_t(\ome,u):=\left\{ \begin{array}{ll} \lim_k \tZ^k_t(\ome,u) , \ \ & \text{if the limit exists, for fixed $(t,\ome,u)$}, \\[2mm] 0, \ \ &\text{otherwise.} \end{array}\right. \ee Then $\tZ$ is $\uuu$-optional. For each $u \in U,$ since $\tZ^k(\cdot,u)$ converges uniformly in time to $Z(\cdot,u)$ in probability, we have $\P(\text{for any $t \in [0,T],$ } \tZ_t(\cdot,u)=Z_t(\cdot,u))=1$, and thus $\tZ$ is an $\uuu$-optional version of $Z.$ Moreover, since for each fixed $u,$ $Z^n(\cdot, u)$ is c\`adl\`ag and converges to $Z$ uniformly in time, we see that $\tZ(\cdot, u)$ is $\P$-a.s. c\`adl\`ag. \end{proof} \begin{coro}\label{optional-r.i.} Suppose that for any $\BX=(X,\X) \in \MC_T,$ $(Z,Z'){(\cdot\ ; \BX)} \in \BD^{\gamma,\gamma'}_X L_{p,q}$ with \(p\in[2,\infty)\), \(q\in[p,\infty]\), \(\gamma,\gamma'\in[0,1]\) such that \(\alpha+\gamma>1/2\) and \(\alpha+\min(\alpha,\gamma)+\gamma'>1\). Furthermore, assume that $(Z,Z')$ as a mapping on $[0,T] \times \Omega \times \MC_T$ has a $\ccc_T$-optional version. Then the rough integral $\int_0^.(Z,Z')d\BX$ has a $\ccc_T$-optional version, which is $\P$-a.s. continuous for any fixed $\BX \in \MC_T$. \end{coro} \begin{proof} Without loss of generality, we assume $(Z,Z')$ is $\ccc_T$-optional. According to \cref{prop.rsint}, for any $\BX \in \MC_T$, the process $\int_0^.(Z,Z')d\BX$ is the uniform in time limit in probability of $$ I^{\op,(\BX)}_t:= \sum_{[u,v] \in \op, u \le t} Z_u(\cdot\ ; \BX) X_{u,v \wedge t} + Z'_u(\cdot\ ; \BX) \X_{u,v \wedge t}, $$ as $|\op|$ goes to zero. Then the result follows by \cref{optional-limit}. \end{proof} \begin{lem}\label{meas-coef} Suppose that $g:[0,T] \times \Omega \times \R^d \times \MC_T \rightarrow \R^m$ is $\bbb^{d} \otimes \ccc_T$-optional. Suppose $Y:[0,T] \times \Omega \times \MC_T \rightarrow \R^d$ has a $\ccc_T$-optional version. Then $Z_t(\ome,\BX):= g(t,\ome,Y_t(\ome,\BX), \BX)$ has a $\ccc_T$-optional version. \end{lem} \begin{proof} Without loss of generality, assume $Y$ is $\ccc_T$-optional. Then let $$(S_T,\MS):=(([0,T]\times \Omega) \times \MC_T, \MO \otimes \ccc_T ),$$ and thus $g:S_T \times \R^d \rightarrow \R^m$ is $\MS \otimes \bbb^d/\bbb^m$-measurable. Note that $Y$ is $\MS/\bbb^d$-measurable, and thus $Z$ is $\MS/\bbb^m$-measurable, which means $\ccc_T$-optional. \end{proof} \begin{proof}[Proof of \cref{thm-rsde-optional}] Since global-on-$[0,T]$ solution are constructed by concatenation of local solutions, it suffices to check the stated measurability for local solutions (with random initial data). Following Theorem 4.6 in \cite{FHL21}, these are constructed by Picard iteration in a space of $X$-stochastic controlled rough paths, started from the process $t\mapsto (\xi +f_0(\xi)\delta X_{0,t},f_0(\xi)) =: (Y^{\BX;(0)}, Y^{\prime,\BX;(0)})=: \mathrm{Y}^{\BX;(0)}$. Then we define inductively $\mathrm{Y}^{\BX;(n+1)} \equiv (Y^{\BX;(n+1)}, Y'^{,\BX;(n+1)}) := (Y^{\BX;(n+1)}, f(Y^{\BX;(n)}) ), $ where \be\label{meas-picard} \begin{split} Y^{\BX;(n+1)}:=\ & \xi + \int_0^t b_r(Y^{\BX;(n)}_r ; \BX)dr + \int_0^t \sigma_r (Y^{\BX;(n)}_r ; \BX)dB_r\\ & +\int_0^t \Big(f_r, (D_y f_r) Y'^{,\BX;(n)}+ f'_r \Big) (Y^{\BX;(n)}_r; \BX )d\BX_r. \end{split} \ee We can see inductively that $(t,\omega, \BX) \mapsto \mathrm{Y}^{\BX;(n)} (t, \omega)= \mathrm{Y}^{(n)} (t, \omega; \BX)$ is $\ccc_T$-optional. Indeed, the case $n=0$ being obvious in view of $\cff_0$-measurability of $\xi$ and $f_0.$ To see that $\ccc_T$-optional measurability of $\mathrm{Y}^{(n)}$ implies the same for $\mathrm{Y}^{(n+1)}=({Y}^{(n+1)}, f(Y^{(n)}) )$, we have to deal with Lebesgue integration $\int b (...) dt$, It\^o integration $\int \sigma (...) dB$ and last not least stochastic rough integration $\int (... ) d \BX$, where in the these three cases the integrand $ (...)$ is itself an $\ccc_T$-optional in view of Lemma \ref{meas-coef}. The desired $\ccc_T$-optional measurability of the first two cases then follows from results on measurable selection for stochastic integration with parameters, more precisely Lemma \ref{SY78-Lem2} and Proposition \ref{SY78-Prop5} in the appendix. It remains to understand that the rough stochastic integral as map $$ (t, \omega, \BX) \mapsto \int_0^t \left(f_r, (D_y f_r) Y'^{,\BX;(n)} + f'_r \right)(Y^{\BX; (n)}_r ;\BX) d\mathbf{X}_r $$ is $\ccc_T$-optional. By \cref{assum-jt-measu} $(2)$ and Lemma \ref{meas-coef} again, we see that the integrand, abbreviated to $(Z,Z')$ is $\ccc_T$-optional. The $\ccc_T$-optional measurability of the rough integral then follows from Corollary \ref{optional-r.i.} above. This shows that all ${Y}^{(n)}$, $n=1,2,...$ are $\ccc_T$-optional. By Lemma \ref{optional-limit} we can then find $Y=Y(t,\omega;\BX)$ which is $\ccc_T$-optional and the limit in $n$, uniformly on $[0,T]$ and in probability, of ${Y}^{\BX;(n)}$ for all $\BX$, which completes the proof. \end{proof} \begin{rem} If we want to work with c\`adl\`ag rough paths, we should work with ``$\ccc_T$-predictable'' rather than $\ccc_T$-optional integrands, already needed when including BV integrators, as is the case in the semimartingale setting of Theorem 1 in {\cite{SY78}}. \end{rem} \section{Rough stochastic control} \label{DPP_for_RSDEs} \subsection{Setup and assumptions} \label{sub.setupdpp} Let $\Omega = \C([0,T];\R^{d_B})$, and $B$ the canonical process on the Wiener space $(\Omega,\MF, ( \MF_t )_{t \ge 0}, \P)$ with $\{\MF_t\}_{t\ge 0}$ as the augmented filtration generated by $B$. Let $A$ be a Polish space (a complete separable metric space) equipped with its Borel sigma algebra $\aaa$, and $\MA$ be the space of {\em admissible} controls, by which we mean {\em optional}\footnote{The use of optional controls is related to Lemma \ref{lem:op} below.} processes $\eta: [0,T] \times \Omega \to A$, in the sense of measurability with respect to the optional $\sigma$-algebra $\ooo$. Some authors, motivated e.g by linear-quadratic control problems, impose integrability conditions of the form $\E[\int_0^T|\ctrl|_A^2 dt]<\infty$, which is not necessary for us, since the control will only be used in uniformly bounded coefficient fields. Suppose that $(b,\sigma, \ell): [0,T] \times \R^{d_Y} \times A \rightarrow \R^{d_Y} \times \ML(\R^{d_B}, \R^{d_Y}) \times \R,$ $g:\R^{d_Y} \rightarrow \R,$ and $f:[0,T] \times \R^{d_Y} \rightarrow \ML(\R^{d_X}, \R^{d_Y})$. For any $\BX \in \MC_T,$ we consider the following controlled RSDEs: for any $0<s\le t \le T$, $\ctrl \in \MA$, and $y \in \R^{d_Y},$ \be\label{c-rsde}\left\{ \begin{split} &dY_t(\omega)=b(t,Y_t(\omega),\ctrl_t (\omega))dt+\sigma(t,Y_t(\omega),\ctrl_t(\omega))dB_t(\omega)+(f,f')(t,Y_t(\omega)) d\BX_t,\\ & Y_s = y, \end{split} \right. \ee and the cost function \be\label{cost-rp} J (s,y, \ctrl;\BX):=\E\left[ g(Y_T^{(s,y,\ctrl, \BX)})+ \int_s^T \ell (r, Y_r^{(s,y,\ctrl, \BX)}, \ctrl_r)dr \right], \ee with $Y^{(s,y,\ctrl,\BX)}$, also written as $Y^{s,y} (\ctrl,\BX)$, is the solution of \eqref{c-rsde}. The rough stochastic conntrol problem is concern about the minimisaion of the cost functional, directly related to the rough value function \be\label{vf-rsde11} \mathcal V (s,y; \BX ) := \mathrm{inf}_{\ctrl \in {\mathcal{A}}} J (s,y, \ctrl;\BX). \ee The coefficients $(b,\sigma,f,g,\ell)$, which are implicit in \eqref{vf-rsde11}, are the data for the above control problem. For the well-posedness of controlled RSDE \eqref{c-rsde} and the cost function, we suppose the following assumption. \begin{assumption}\label{assum-dpp} \phantom{new line} \item[$(1)$] Suppose that $ (b,\sigma, \ell) :[0,T] \times \R^{d_Y} \times A \rightarrow \R^{d_Y} \times \mathrm{Lin}(\R^{d_B}, \R^{d_Y}) \times \R $ is bounded and measurable. Moreover, each $\psi\in \{b,\sigma, \ell\}$ is uniformly Lipschitz in the sense that for any $y,\bar{y} \in \R^{d_Y}$, \be \sup_{u \in A}\sup_{t\in [0,T]} |\psi(t,y,u)-\psi(t, \bar{y}, u)| \le \|\psi\|_{\lip} |y-\bar{y}|, \label{LipAss} \ee for some positive constant $\|\psi\|_{\lip}.$ \item[$(2)$] Suppose $(f,f') (\cdot \ ;\BX) $ belongs to $ \BD_X^{2\beta } \C^{\gamma}_b$ for $\beta \in (0,\alpha]$, $\gamma > \frac{1}{\alpha}$ such that $\alpha+\beta>\frac12$ and $\alpha + (\gamma-1)\beta>1.$ \item[$(3)$] Suppose $g \in \mathrm{BUC}(\R^{d_Y})$, i.e. bounded uniformly continuous, with concave modulus of continuity $\lambda_g (.)$.\end{assumption} \subsection{Regularity of rough value function} Under the above assumption, we show the continuity of the rough value function \eqref{vf-rsde11}, which plays a key role in the proof of dynamical programming principle. \begin{thm} \label{thm:RoughValueReg} Suppose $\BX,\bBX \in \MC_T$, and $( b, \sigma, f , f', g, \ell)$, $(\bar b, \bar \sigma, \bar f, {\bar f}', \bar g, \bar \ell)$ satisfy Assumption \ref{assum-dpp}. For any $\ctrl \in \MA,$ let $\MV(t,y;\BX)$ (resp. $J(t,y,\ctrl;\BX)$) and $ \bar{\mathcal{V}} (\bar t,\bar y;{\bar \BX})$ (resp. $\bar{J}(\bt,\by,\ctrl;\bBX)$) be the corresponding value functions (resp. cost function) given by \eqref{vf-rsde11} (resp. \eqref{cost-rp}) with data $(b,\sigma, g, f, f' , \ell, \BX)$ and $(\bar b, \bar \sigma, \bar g, \bar f ,{\bar f}', \bar \ell , \bar \BX)$ respectively. When $\ell \equiv 0$, \begin{multline} \label{localest-costfct} \sup_{\ctrl \in \MA } |\bar {J}(\bar t,\bar y, \ctrl;\bar \BX)- {J}(t,y, \ctrl;\BX)| \lesssim | (\bar{g} - g)_+ |_\infty \\ +\lambda_{g} \left(|T-t\vee\bar t|^\alpha( |(b,\sigma)-(\bar b, \bar \sigma) |_\infty + F + \rho_\alpha(\bar\BX,\BX) )+|t-\bar t|^{\alpha}+|y-\bar y| \right) \end{multline} with $F = \|(f-\bar f, f'-\bar f')\|_{\gamma-1}+ \llbracket f, f'; \bar{f}, \bar{f}' \rrbracket_{X, \bar{X}; 2 \beta}$. Moreover, \begin{multline} \label{equ:cRSDEestimate} |\bar {\mathcal{V}}(\bar t,\bar y;\bar \BX)- {\mathcal{V}}(t,y;\BX)| \lesssim | (\bar{g} - g)_+ |_\infty \\ +\lambda_{g} \left(|T-t\vee\bar t|^\alpha( |(b,\sigma)-(\bar b, \bar \sigma) |_\infty+ F + \rho_\alpha(\bar\BX,\BX) )+|t-\bar t|^{\alpha}+|y-\bar y| \right), \end{multline} otherwise (with non-trivial \(\ell\)) \begin{multline} \label{equ:cRSDEestimate} |\bar {\mathcal{V}}(\bar t,\bar y;\bar \BX)- {\mathcal{V}}(t,y;\BX)| \lesssim | (\bar{g} - g)_+ |_\infty \\ +\tilde\lambda_{g} \left(|T-t\vee\bar t|^\alpha( |(b,\sigma,\ell)-(\bar b, \bar \sigma, \bar \ell)|_\infty+ F + \rho_\alpha(\bar\BX,\BX) )+|t-\bar t|^{\alpha}+|y-\bar y| \right), \end{multline} where \(\tilde{\lambda}_g(\cdot)=\lambda_g(\cdot)+|\cdot |\) and the implicit constant depends on the parameter $M$ from \cref{thm:stabilityforRSDEs_compendium}, in particular on $\nn{\bBX}_\alpha+\nn{\BX}_\alpha$. \end{thm} \begin{remark} \label{rem:cont_val}At first reading, take $f = \bar{f} \in \C^\gamma_b, f' = \bar{f}' \equiv 0$, so that $F = 0$. In this case, \cref{thm:RoughValueReg} immediately implies that the rough value function depends continuously on the rough path. \end{remark} \begin{proof} We may assume $\ell = 0$ for otherwise it suffices to consider the enhanced process $(Y,Z)$ with $Y$ as in \eqref{c-rsde} and additional dynamics $$ dZ_t (\omega) = \ell (t, Y_t(\omega),\ctrl_t(\omega)) dt; $$ in terms of $(Y,Z)$, with drift $(b,\ell)$, the value function \eqref{cost-rp} can then be written as $$ \tilde{\mathcal{V}} (s,(y,0); \BX) = \inf_{\ctrl } \E^{s, (y,0)} [\tilde g (Y_T,Z_T)] $$ with $\tilde g \in \mathrm{BUC}$ such that $\tilde g (Y_T,Z_T) = g (Y_T) + Z_T$, possible since $g \in \mathrm{BUC}$ and $\ell$, hence $Z$, bounded. Note that \(\tilde g\) has concave modulus of continuity \(\tilde{\lambda}_g\). We revert to our original notation, and assume $\ell = 0$. Applying triangle inequality, we have, from the very definition of the value function, \begin{align*} |\bar {\mathcal{V}}(t,y;\bar{\BX})- {\mathcal{V}}(t,y;\BX)|\le |\bar g-g|_\infty+ \sup_{\ctrl } |\E g(Y^{\bar t,\bar y}_T(\ctrl,\bar\BX))-\E g( Y^{ t, y}_T(\ctrl,\BX))|, \end{align*} where we write $Y^{(t,y,\ctrl,\BX )}_s$ as $Y^{ t, y}_s(\ctrl,\BX).$ Hence, it suffices to consider the case $\bar g=g$. By regularity of $g$ and Jensen inequality, \begin{align*} | \E [g( Y^{\bar t,\bar y}_T(\ctrl,\bar{\BX}))]-\E [g( Y^{t,y}_T(\ctrl,\BX))]| &\le \E \lambda_g(|Y^{\bar t,\bar y}_T(\ctrl,\bar{\BX})-Y^{t,y}_T(\ctrl,\BX)|) \\&\le \lambda_g(\E| Y^{\bar t,\bar y}_T(\ctrl,\bar{\BX})-Y^{t,y}_T(\ctrl,\BX)|). \end{align*} We assume that $t\le\bar t$. By uniqueness of \eqref{c-rsde}, we can write $Y^{t,y}_T(\ctrl,\BX)=Y^{\bar t,Y^{t,y}_{\bar t}(\ctrl,\BX)}_T(\ctrl,\BX)$ and apply \cref{thm:stabilityforRSDEs_compendium} to get that \begin{align*} \E| Y^{\bar t,\bar y}_T(\ctrl,\bar{\BX})-Y^{t,y}_T(\ctrl,\BX)| &=\E| Y^{\bar t,\bar y}_T(\ctrl,\bar\BX)-Y^{\bar t,Y^{t,y}_{\bar t}(\ctrl,\BX)}_T(\ctrl,\BX)| \\&\le\|Y^{t,y}_{\bar t}(\ctrl,\BX) -\bar y\|_2+\|\delta Y^{\bar t,\bar y}_{\bar t,T}(\ctrl,\bar \BX)-\delta Y^{\bar t,Y^{t,y}_{\bar t}(\ctrl,\BX)}_{\bar t,T}(\ctrl, \BX)\|_2 \\&\lesssim\|Y^{t,y}_{\bar t}(\ctrl,\BX)-\bar y\|_2+|T-\bar t|^\alpha\rho_\alpha(\bar\BX,\BX). \end{align*} Using triangle inequality, the identity $Y^{t,y}_t(\ctrl,\BX)=y$ and estimate \eqref{est.apri.m} (noting that $p\ge2$ can be chosen arbitrarily), we have \[ \|Y^{t,y}_{\bar t}(\ctrl,\BX)-\bar y\|_2 \le \|Y^{t,y}_{\bar t}(\ctrl,\BX)-Y^{t,y}_t(\ctrl,\BX) \|_2+|\bar y-y| \lesssim|\bar t-t|^\alpha+|\bar y-y|. \] The implicit constants in the previous estimates are uniform in $t,y,\bar t,\bar y$ and crucially also in $\ctrl$, thanks to \cref{assum-dpp}. Putting these estimates altogether, in conjunction with elementary estimates of the form $| \inf A - \inf B | \le \sup | A - B |$, we obtain the announced inequality. \end{proof} \subsection{Rough dynamical programming principle} To show the Dynamical Programming Principle (DPP) in the canonical space, we exploit the causal structure of optional processes. \begin{lem} \label{lem:op} Let $(\Omega, \cff, (\cff_t)_t, \P)$ be the augmented canonical space as described in \cref{sub.setupdpp}. Suppose that $\ctrl:[0,T] \times \Omega \rightarrow A$ is measurable. Then $\ctrl$ is optional w.r.t. $(\cff_t)$ if and only if for a.e. $\ome,$ $\ctrl_t(\ome)= \ctrl_t(\ome_{t \wedge .})$ for all \(t\). \end{lem} \begin{proof} Indeed, the proof follows by a sequence of facts. Let $(\MF^B_t)$ is the raw filtration generated by the canonical process $B.$ Then according to \cite[Theorem IV.78 and Remark IV.74]{DM1978}, we see that $(\MF_t)$-predictable processes are indistinguishable from $(\MF^B_t)$-predictable ones. On the other hand, since $(\MF_t)_t$ is augmented from $(\MF^B_t)_t$, by \cite[p.30 Proposition and p.31 Example]{chung13}, which claims every $(\MF_t)_t$-stopping time\footnote{Note that in that book, stopping times are called optional times.} is a $(\MF_t)_t$-predictable stopping time (so the optional sigma algebra agrees with the predictable sigma algebra), we see that any optional process $\ctrl(\cdot)$ is indistinguishable from a $(\MF^B_t)$-predictable process. Finally, according to \cite[p.147 Theorem 97]{DM1978}, which claims $\eta$ is $(\MF^B_t)$-predictable if and only if $\ctrl_t(\ome)= \ctrl_t(\ome_{t \wedge .})$, we see our claim follows. \end{proof} The following lemma, adapted from \cite[Section 3.2]{NT13} and \cite[Proposition 4]{CTT16}, provides convenient presentation of the value function, which is used to prove the ``harder'' part of the DPP. \begin{lem}\label{equiv-vf} Suppose that Assumption \ref{assum-dpp} holds. Let $t \in [0,T)$, \(y\in \R^{d_Y}\) and \(\BX\in \MC_T\). Put \be \MA^t:=\{\ctrl \in \MA \ \Big| \ \ctrl \text{ independent of $\MF_t$ under }\P \}, \ee \be\label{vf2} \tilde \MV (t,y; \BX ) := \mathrm{inf}_{\ctrl \in {\mathcal{A}^t} } J (t,y, \ctrl; \BX ), \ee where $J$ is given by \eqref{cost-rp}. Then we have \be \MV(t,y;\BX)=\tilde \MV(t,y;\BX). \ee \end{lem} We leave the proof of the above lemma later and present the DPP for rough stochastic control problem. \begin{thm}\label{thm-RBP} (Rough DPP) Let $\BX \in \MC^\alpha_T$ and let Assumption \ref{assum-dpp} be in force. Then, for $0 \le s \le s+h \le T$, \be\label{eq:rdpp} \mathcal V (s,y; \BX) = \inf_{\ctrl \in \mathcal{A}} \E \left( \mathcal V(s+h,Y^{(s,y,\ctrl, \BX)}_{s+h}; \BX) + \int_s^{s+h} \ell (t, Y_t^{(s,y,\ctrl, \BX)}, \eta_t) d t \right). \ee \end{thm} \begin{proof} Denote by $\tMV(s,y;\BX)$ the right hand side of \eqref{eq:rdpp}. We first offer a simpler proof under the stronger assumption that $\BX$ is a geometric rough path, hence the limit of smooth path in $\rho_\alpha$, and for autonomous $f = f(y)$. \smallskip (1st variant) We use continuity of $\mathcal{V}= \mathcal V (s,y; \BX)$ in the rough path argument $\BX$, as follows from \cref{thm:RoughValueReg} (cf. Remark \ref{rem:cont_val}). In particular, this makes the right-hand side in the statement, denoted by $\tilde{\mathcal{V}} (s,y; \BX)$ for the rest of this proof, well-defined. As before, we may assume $\ell = 0$ without loss of generality. The first one assumes validity of the (classical) DPP in the case when $\BX$ is replaced by smooth $X^\varepsilon$, followed by taking $\varepsilon \to 0$. By definition, a geometric rough path $\BX$ is the limits of smooth paths. That is, $\rho_\alpha (\BX,\BX^\varepsilon) \to 0$ as $\varepsilon \to 0$, where $\BX^\varepsilon$ is the canonical rough path lift of smooth $X^{\varepsilon}$. Set $\mathcal{V}^\varepsilon (s,y) := \mathcal V (s,y; \BX^\varepsilon)$ and similarly for $\tilde{\mathcal{V}}^\varepsilon$. For fixed $\varepsilon > 0$ we are in a classical setting (e.g. \cite{krylov2008controlled, SA21}) so that $\mathcal{V}^\varepsilon = \tilde{\mathcal{V}}^\varepsilon$, any $\epsilon > 0$. By \cref{thm:RoughValueReg}, we know $\mathcal{V}^\varepsilon \to \mathcal{V}(\cdot, \cdot; \BX)$ so it suffices to see the corresponding statement for $$ \tilde{\mathcal{V}}^\varepsilon (s,y) = \inf_{\ctrl } \E \left( \mathcal V(s+h,Y^{s, y}(\ctrl, \BX_{s+h}^\varepsilon); \BX^\varepsilon) \right) $$ But this is straight-forward, using the elementary $| \inf A - \inf B | \le \sup | A - B |$, continuous dependence of $Y(\ctrl, \BX)$, fixed $\ctrl$, in $\BX$, joint continuity of $V(s,y;\BX)$ in $(y,\BX)$, boundedness of $V$, and bounded convergence. This justifies passage to the limit, and the rough DPP is established.\\ (2nd variant) We now give the full proof, which applies to each fixed $\BX$, and thus we omit \(\BX\) in the notation, writing $Y^{s,y, \ctrl} = Y^{(s,y,\ctrl,\BX)}$ accordingly. This argument does not require continuity of the value function in $\BX$, hence also applicable to suitable $X$-controlled vector fields $(f,f')$. We do need however continuity of the value function in the time-space argument $(s,y)$, as follows immediately from our estimate \eqref{equ:cRSDEestimate} applied with $t, \bar{t}, y, \bar{y}$, and identical data ($b, \sigma, \ell, g, \BX, f,f'$) otherwise. Here $\BX$ is fixed, and henceforth omitted from the notation for the rest of this proof. ``$\ge$'': By the definition of $\MV$ and Lemma \ref{lem:op}, for any $\vep>0,$ there exists $\ctrl^{\vep}_t(\ome)= \ctrl^{\vep}(t,B(\ome)_{t\wedge .}) \in \MA$ such that $$ \MV(s,y) > J(s,y,\ctrl^\vep)-\vep. $$ On the other hand, note that for any $\eta_t(\ome)=\eta(t, B_{t\wedge .}(\ome)) \in \MA,$ we have $ \P$-a.s. \be\label{eq:r-markov} \E \left[ g(Y_T^{s+h, Y_{s+h}^{s,y,\eta}, \eta }) + \int_{s+h}^T \ell(r,Y^{s,y,\eta}_r, \eta_r) dr \Big| \MF_{s+h} \right](\ome) =J(s+h,Y^{s,y,\eta}_{s+h}(\ome),\eta^\ome), \ee where for almost surely $\ome \in \Omega,$ $\eta^{\ome}_t(\ome'):= \eta(t, B^{s+h, \ome}_{t \wedge .}(\ome')),\ \text{for any } (t,\ome') \in [0,T] \times \Omega,$ with $$ B^{s+h, \ome}_{r}(\ome')=\left\{ \begin{array}{ll} B_{r}(\ome), & r\le s+h,\\ B_{r}(\ome')-B_{s+h}(\ome')+ B_{s+h}(\ome), & r >s+h. \end{array} \right. $$ According to Lemma \ref{lem:op}, we see that $\eta^{\ome} \in \MA.$ Then by taking $\ctrl=\ctrl^{\vep}$ in \eqref{eq:r-markov} and the tower property of conditional expectation, \begin{equation} \begin{split} J(s,y,\ctrl^\vep) \ge \E \left[ \MV(s+h,Y_T^{s,y,\ctrl^\vep}) + \int_s^{s+h} \ell(r, Y^{s,y,\ctrl^\vep}, \ctrl_r) dr \right], \end{split} \end{equation} which implies $\MV(s,y)> \tMV(s,y)-\vep.$ ``$\le$'': For any $\vep >0,$ in view of the uniform continuity of $\MV$ in the space variable, i.e. \eqref{equ:cRSDEestimate}, there exists $\delta>0$ and a sequence of balls $\{B_i \}_{i\ge 1}$ covering $\R^{d_Y}$, such that for any $y,y'\in B_i$ and $t > 0,$ \begin{equation}\label{close-vf} |\MV(t,y)-\MV(t,y')|<\vep. \end{equation} Moreover, in view of the continuity of $Y^{s,y,\ctrl}$ in $y\in \R^{d_Y}$ uniformly on $\eta \in \MA,$ i.e. Theorem \ref{thm:stabilityforRSDEs_compendium}, we have \be \left\| \sup_{t\in[0,T]} |Y_t^{t,y,\ctrl} - Y_t^{t,y',\ctrl}| \right\|_2 \lesssim |y-y'|, \ee where the implicit constant depends only on ${\nn{\BX}}_{\alpha}$ and $(b,\sigma,f,f').$ Thus by Lipschitzness of $(g,\ell),$ we can choose $\{B_i \}_{i\ge 1},$ such that for any $y,y'\in B_i$, \begin{equation}\label{close-cost} \E\left[|g(Y_T^{t,y,\ctrl})- g(Y_T^{t,y',\ctrl})|+ \int_t^T |\ell(r,Y_r^{t,y,\ctrl}, \ctrl)-\ell(r,Y_r^{t,y',\ctrl}, \ctrl)|dr \right] <\vep. \end{equation} Then it is standard to obtain a Borel partition of $\R^{d_Y}$, denoted by $ \{\Gamma_i\}_{i\ge 1}$ with some $y^i \in \Gamma_i$ and $\Gamma_i \cap \Gamma_j= \emptyset $ for any $i \neq j$, such that \eqref{close-vf} and \eqref{close-cost} hold for any $y \in \Gamma_i$ and $y'=y^i.$ For any $y^i$, according to Lemma \ref{equiv-vf}, there exists $\eta^{\vep,i} \in \MA^{s+h},$ such that \begin{equation}\label{rep-vf} \MV(s+h,y^i) \ge J(s+h,y^i, \ctrl^{\vep,i})- \vep. \end{equation} Now for any $\eta \in \MA,$ let \begin{equation} {\bar{\ctrl}}^\vep_t(\ome):=\left\{ \begin{array}{ll} \ctrl^{\vep,i}_t(\ome), & t > s+h, \ Y^{s,y,\ctrl}_{s+h}(\ome) \in \Gamma_i,\\ \ctrl_t(\ome), & t\in [s,s+h]. \end{array} \right. \end{equation} According to \eqref{equiv-vf} and \eqref{rep-vf}, we have \begin{equation}\label{appro-vf} \begin{split} \MV(s+h, Y^{s,y,\ctrl}_{s+h}) & \ge \sum_{i} \MV(s+h,y^i) 1_{\{Y^{s,y,\ctrl }_{s+h} \in \Gamma_i \}}- \vep\\ & \ge \sum_{i} J(s+h,y^i, \ctrl^{\vep,i}) 1_{\{Y^{s,y,\ctrl }_{s+h} \in \Gamma_i \}}- 2\vep. \end{split} \end{equation} On the other hand, since $\ctrl^{\vep,i} \in \MA^{s+h}$ is independent of $\MF_{s+h}$ which implies $Y^{s+h,y^i,\ctrl^{\vep,i}}$ as well is, we have \begin{equation}\label{appro-cost} \begin{split} & \E \left[\sum_{i} J(s+h,y^i, \ctrl^{\vep,i}) 1_{\{Y^{s,y,\ctrl }_{s+h} \in \Gamma_i \}} \right] \\ = & \ \E\left[ \sum_i g(Y_{T}^{s+h,y^i,\ctrl^{\vep,i} }) 1_{\{Y^{s,y,\ctrl }_{T} \in \Gamma_i \}} + \sum_i \int_{s+h}^T \ell(r,Y_r^{s+h,y^i, \ctrl^{\vep,i}}, \ctrl^{\vep,i} )1_{\{Y^{s,y,\ctrl }_{s+h} \in \Gamma_i \}} dr \right]\\ \ge & \ \E \left[ g(Y_T^{s+h,Y^{s,y,\ctrl}, \bctrl^\vep}) + \int_{s+h}^T \ell(r,Y_r^{s+h,Y^{s,y,\ctrl}, \bctrl^\vep}, \bctrl^{\vep}_r ) dr \right] - \vep\\ = & \ \E \left[ g(Y_T^{s,y,\bctrl^\vep}) + \int_{s+h}^T \ell(r,Y_r^{s,y,\bctrl^\vep}, \bctrl_r^{\vep}) dr \right]-\vep, \end{split} \end{equation} where we apply \eqref{close-cost} in the last inequality. In view of \eqref{appro-vf} and \eqref{appro-cost}, we have \begin{equation} \begin{split} & \E \left[ \MV(s+h,Y^{s,y,\ctrl}_{s+h}) + \int_{s }^{s+h} \ell(r,Y^{s,y,\ctrl}_r, \ctrl_r) dr \right]\\ & \ge \ \E \left[ g(Y_T^{s,y,\bctrl^\vep}) + \int_{s }^{T} \ell(r,Y_r^{s ,y, \bctrl^\vep}, \bctrl^{\vep}_r ) dr \right] - 3 \vep \\ & \ge \MV(s,y)-3 \vep, \end{split} \end{equation} which implies $\tMV(s,y) \ge \MV(s,y)-3\vep$ and so our claim follows. \end{proof} \begin{proof}[Proof of Lemma \ref{equiv-vf}] These ideas being standard, we only provide a sketch. (Also see \cite[Proposition 4]{CTT16} for another proof, noting that the authors deal with predictable controls, which is equivalent to our setting, as pointed out in the proof of Lemma \ref{lem:op}.) Fix $(s,y) \in [0,T] \times \R^{d_Y}$ and omit \(\BX\) in notations. We only need to show $\MV(s,y) \ge \tMV(s,y) .$ Indeed, according to Lemma \ref{lem:op}, for any $\ctrl \in \MA$ is of the form $\ctrl= \ctrl_t( \ome_{t \wedge .} )$ and hence for any $t\ge s,$ we can write $ \ctrl_t(\ome)= \Phi(t, \ome_{s \wedge . } , (\ome_{r} - \ome_s )_{s\le r \le t} ), $ for a suitable $\Phi$. By independence of Brownian increments, for any fixed $\ome_{s\wedge .} \in \Omega |_{[0,s]},$ $\ctrl^{s,\ome}_t:=\Phi(t, \ome_{s \wedge . }, \cdot-\ome_s): \Omega_{[s,t]} \rightarrow U$ is independent of $\MF_s$ and thus belongs to $\MA^s.$ Then by the Fubini theorem, we have \begin{equation} \begin{split} J(s,y,\ctrl) & = \int \E \Big[ g(Y_T^{s,y,\ctrl^{s,\ome}}) + \int_s^T \ell(r,Y_r^{s,y,\ctrl^{s,\ome}}, \ctrl^{s,\ome}_r ) \Big] d \P(\ome_{s\wedge .})\\ & \ge \int \tMV(s,y) d \P(\ome_{s\wedge .})= \tMV(s,y), \end{split} \end{equation} which implies the claim. \end{proof} We remark that our proof for the DPP, in particular the first variant, extends in a straight-forward way to the case of two-player, zero-sum stochastic differential games \cite{FS89}. \subsection{Explicit example} The following example is inspired by \cite{BM07} where the authors consider a linear, scalar situation ($d_X = d_Y =1$), which allows for pathwise solutions. We here consider general dimensions with linear $f: \R^{d_Y} \rightarrow \mathrm{Lin}(\R^{d_X}, \R^{d_Y})$, zero It\^o vector fields $b,\sigma$, so that \eqref{rand-rsde} is a genuine controlled rough differential equations, of the form $$ \begin{array}{ll} & dY^{\eta}_t = \eta_t dt + f (Y^{\eta}_t) d\BX_t, \quad t \in [s, T],\\ & Y_s^{\eta} = y. \end{array} $$ Call $P^{\BX}_{t \leftarrow s}: \R^{d_Y} \to \R^{d_Y}$ the (linear) solution flow to the RDE solution flow of this equation in uncontrolled case ($\eta \equiv 0$). Then controlled process then can be given in mild formulation, $$ Y_T^{(s, y, \eta , \BX)} = P^{\BX}_{T \leftarrow s} \hspace{0.17em} y + \int_s^T P^{\BX}_{T \leftarrow r} \eta_r dr, \qquad Y_T^{(0, 0 , \eta , \BX)} = \int_0^T P^{\BX}_{T \leftarrow r} \eta_r dr. $$ Assume now that $| \eta | \leqslant 1$, fix $v \in \mathbb{R}^{d_Y}$ and consider distance-to-$1$ cost $| \Pi_T^{\eta} - 1|$, where \[ {\Pi_T^{\eta}} : = \langle v, Y_T^{ (0, 0 , \eta , \BX) } \rangle = \int_0^T \langle \Theta_r, \eta_r \rangle dr, \qquad \Theta_r : = (P^{\BX}_{T \leftarrow r})^{\star} v .\] Clearly ${\Pi_T^{\eta}} \leqslant M_T^{\BX} \assign \int_0^T | \Theta_r | d r$ with equality when $\eta_r = \hat{\Theta}_r \assign \Theta_r / | \Theta_r |$. If $M_T^{\BX} > 1$ we overshoot the target $1$ and the optimal control (zero cost) is plainly given by $\eta^{\star}_r = \hat{\Theta}_r / M_T^{\BX}$, thanks to linearity of ${\Pi_T^{\eta}} $ in $\eta$. If $M_T^{\BX} \leqslant 1$ the best we can is full speed ahead, i.e. $\eta^{\star}_r = \hat{\Theta}_r$ which results in a cost $(1 - M_T^{\BX})$. Summarizing, we have \[ \mathcal{V} (0, 0, \BX) \assign \inf_{\eta \in \mathcal{A}} | \langle v, Y_T^{(0, 0 , \eta , \BX)} \rangle - 1| = [1 - M_T^{\BX}]^+ \] with optimal control \[ \gamma_t^{\star} = \left\{ \begin{array}{ll} \hat{\Theta}_r / M_T^{\BX} & \text{on } \{ M_T^{\BX} > 1 \},\\ \hat{\Theta}_r & \text{on } \{ M_T^{\BX} \leq 1 \} . \end{array} \right. \] Remark that $\mathscr{C}_T \ni \BX \mapsto M^{\mathbf{X}}$ is $\ccc_T$-measurable, which (trivially) implies that the process $ \gamma^{\star}$ is $\ccc_T$-optional, but certainly not causal in $\BX$. \subsection{Joint measurability of $\eps$-minimizers} To conclude the current section, we provide a result on the $\vep$-approximation of the optimal control to the rough stochastic control problem, which turns out to be quite helpful for pathwise control problem considered in \cref{sec:pathwise-control}. As before, we assume without loss of generality zero running cost $\ell$. \begin{lem} \label{lem1} Suppose Assumption \ref{assum-dpp} holds. Recall that $\mathcal{A}$ is the class of optional (in sense of $\ooo / \aaa$-measurable) controls and that \[ \mathcal V (s,y; \BX ) = \mathrm{inf}_{\ctrl \in {\mathcal{A}}} \E\left[ g(Y_T^{(s,y,\ctrl, \BX)}) \right] \] where $Y^{(s,y,\ctrl,\BX)}$ is the solution to \eqref{c-rsde}. Then for every $\varepsilon > 0$, there exists a $(\ooo \otimes \ccc_T)/\aaa$-measurable map $\hat{\eta} = \{ \eta^{\varepsilon}_t (\ome, \BX) : 0 \leqslant t \leqslant T \}$ such that \[ \mathcal V (s,y; \BX )) \leqslant \mathbb{E} [g (Y^{(s,y,\hat \ctrl,\BX)}_T)] \leqslant \mathcal V (s,y; \BX ) + \varepsilon. \] As a consequence, $\mathcal{V}$ is also the value function when $\mathcal{A}$ is replaced by the class of $\ccc_T$-optional (i.e. $(\ooo \otimes \ccc_T) / \aaa$-measurable) controls. \end{lem} \begin{rem}\label{g-dep-X} With $g$ bounded, continuous, so is the value function (w.l.o.g. with $\ell \equiv 0$) from the definition of $\mathcal{V}$. If the $\mathcal{V}(s,y;\BX)$ is furthermore continuous in $\BX$, the above lemma applies directly with $g (Y^{(s,y,\hat \ctrl,\BX)}_T)$ replaced by $\mathcal V(s+h,Y^{(s,y,\ctrl, \BX)}_{s+h}; \BX)$, term seen on the right-hand side of \eqref{eq:rdpp}. \end{rem} \begin{proof} (As heuristic motivation, assume $\mathscr{C}_T$ is replaced by $\{ \mathbf{x}_1, \ldots ., \mathbf{x}_n \}$, with power $\sigma$-field ${\mathfrak{P}= 2^{\{ \mathbf{x}_1, \ldots ., \mathbf{x}_n \}}} .$ Let $\eta^{\varepsilon}_t (\ome, \mathbf{x }_i)$ be an $\ooo $-measurable $\varepsilon$-optimal control, for fixed $\mathbf{x }_i$. Then \[ \eta^{\varepsilon}_t (\ome, \BX) \assign \eta^{\varepsilon}_t (\ome, \mathbf{x }_i) \quad \text{when } \mathbf{X} = \mathbf{x}_i, 1 \leqslant i \leqslant n, \] which is clearly $(\ooo \otimes \mathfrak{P})$-measurable.) In the general case, employ continuiuty (at $\mathbf{X}$): for every $\varepsilon, \mathbf{X}$ have $\delta(\varepsilon, \mathbf{X}) >0$ such that for any $\bx\in \MC_T,$ \[ d (\mathbf{x}, \mathbf{X }) < \delta (\varepsilon, \mathbf{X})\Rightarrow \sup_{\eta \in \mathcal{A}} \left(\mathbb{E} [g (Y^{\eta, \mathbf{X}}_T)] -\mathbb{E} [g (Y^{\eta, \mathbf{x}}_T)]\right) < \varepsilon . \] Consider the open cover of $\mathscr{C}_T$ given by $\{ B (\mathbf{X}, \delta (\mathbf{X}, \varepsilon)) : \mathbf{X} \in \mathscr{C}_T \}$. \ By the Lindeloef property of Polish spaces, one may switch to a countable subcover, for some $\{\mathbf{x}_i\}_i \subset \mathscr{C}_T,$ \[ B_i : = \{ B (\mathbf{x}_i, \delta (\varepsilon, \mathbf{x}_i)) : i \in \mathbb{N} \} . \quad \] We can turn this into a (Borel) measurable partition $\{P_i\}_i$ of $\mathscr{C}_T$ by considering $P_1 = B_1, P_2 = B_2 \backslash B_1, \ldots .$, so that every $\mathbf{X} \in P_{\mathfrak{i} (\mathbf{X})}$ for some unique $\mathfrak{i} (\mathbf{X}) \in \mathbb{N}$. We can now define \[ \eta^{\varepsilon}_t (\ome, \mathbf{X}) \assign \eta^{\varepsilon}_t (\ome, \mathbf{x }_{\mathfrak{i} (\mathbf{X})}), \quad \tmop{when }\ \mathbf{X} \in P_{\mathfrak{i} (\mathbf{X})}, \] which is clearly $(\ooo \otimes \mathfrak{B} (\mathscr{C}_T))$-measurable. \end{proof} \section{Rough stability of Hamilton--Jacobi--Bellmann, revisited} \label{sec:HJB} In the setting and notation of the previous section, cf. \eqref{c-rsde}, \eqref{vf-rsde11}, we studied the cost functional \be\label{vf-rsde1} v (s,y; \BX ) = \mathrm{inf}_{\ctrl \in {\mathcal{A}}} \E\left[ g(Y_T^{(s,y,\ctrl, \BX)})+ \int_s^T \ell (r, Y_r^{(s,y,\ctrl, \BX)}, \ctrl_r)dr \right] \ee with \be\label{c-rsde1}\left\{ \begin{split} &dY_t(\omega)=b(t,Y_t(\omega),\ctrl_t (\omega))dt+\sigma(t,Y_t(\omega),\ctrl_t(\omega))dB_t(\omega)+f(t,Y_t(\omega)) d\BX_t,\\ & Y_s = y. \end{split} \right. \ee If the (geometric) rough path $\BX$ is replaced by a continuously differentiable path $X$, so that $d X = \dot{X} d t$, it is classical that the value function is the unique (in viscosity sense; see e.g. \cite{fleming2006controlled}) solution to the HJB terminal value problem \[ - \partial_t v = H (y, t, D v, D^2 v) + (f (t,y) \cdot Dv) \dot{X}, \qquad v(T,\cdot) \equiv g, \] with \[ H (y, t, D v, D^2 v) = \inf_{\ctrl \in A} \left( \tfrac{1}{2} \mathrm{Tr} \left( (\sigma \sigma^T) \left(t, y, \ctrl \right) D^2 v \right) + b \left( t, y, \ctrl \right) \cdot Dv \right). \] In the general case, one expects a ``rough'' HJB equation of the form,$$ - d_t v = H (y, t, D v, D^2 v) dt + (f (t,y) \cdot Dv) d \BX. $$ The pragmatic interpretation of such an equation, point of view also taken in \cite{Friz2016}, is to view $v$ as limit of $v^\eps$, whenever $X^\eps \to \BX$ in rough path sense, provided $v$ only depends on $\BX$ and not on the approximating sequence. Such a construction was carried out in \cite{CFO11}, in case of autonomous coefficients fields, relying fundamentally on representing $v$ via a ``rough flow'' transformation, induced by the auxiliary RDE $- d \Psi= f (\Psi) d \BX$; the (formally) transformed equation can also serve as a definition, as pointed out in cf. \cite[Ch.13]{FH20} and references therein.\footnote{Most authors using flow transformation techniques consider autonomous coefficients, i.e. without $t$-dependence, but the extension is not difficult, provided one works with the correct backward flows.} Equivalently, one formulates the transformation via the transport equation associated to the RDE, see \cite[Ch.12]{FH20} and \cite{CHT24}. See also \cite{BCO23} for a discussion how this relates to the original ``local'' transform of test. For the reader's convenience we restate the relevant result in our situation. \begin{thm}\label{thm51} Assume $b=b(y,\eta), \sigma=\sigma(y,\eta)$ are bounded Lipschitz in $y$, uniformly in $\eta$. Assume further $f \in \C_b^{\gamma+2}$, with $\gamma > 1/\alpha$. Then solution map $(\BX, g) \mapsto v$ is continuous, seen as map from $\MC^\alpha {\times} \mathrm{BUC}(\R^{d_Y}) \mapsto \C_b([0,T]\times \R^{d_Y})$, provided $f \in \C_b^{\gamma + 2}$, with $\gamma > 1/\alpha$. \end{thm} \begin{proof} This is \cite[Theorem 1]{CFO11}, noting that $f \in \C_b^{\gamma+2}$ implies the required $\Phi^{(3)}$-invariant comparison put forward in \cite{CFO11}. \end{proof} We can now improve on this result in several ways. \begin{thm} \label{thm52} Assume $b=b(t,y,\eta), \sigma=\sigma(t,y,\eta)$ are bounded Lipschitz in $y$, uniformly in the $t, \eta$. Assume further $f \in \C_b^{\gamma}$, with $\gamma > 1/\alpha$. Then the solution map $(\BX, g) \mapsto v$ is continuous, seen as map from $\MC^\alpha {\times} \mathrm{BUC}(\R^{d_Y}) \mapsto \mathrm{BUC}([0,T]\times \R^{d_Y})$ and this continuity statement is fully quantifiable via estimate \eqref{equ:cRSDEestimate}. In particular, the map $(\BX, g) \mapsto v$ is Lipschitz continuous when the rough path norm of $\BX$ and $\C_b^{1}$-norm of $g$ remain bounded. \end{thm} \begin{proof} Direct consequence of \cref{thm:RoughValueReg}. \end{proof} In particular, in the case of interest when $\alpha = 1/2 - \eps$ (which covers the case when $\BX$ is a typical realization of Brownian motion enhanced to a rough path in the Stratonovich sense) the excessive regularity requirement $f \in \C_b^{4+\eps}$ of Theorem \ref{thm51} is replaced by $f \in \C_b^{2+\eps}$ in Theorem \ref{thm52}, which is the natural condition (with $\eps = 0+$) one expects from Stratonovich SDE and transport SPDE theory. \begin{exam} We revisit \cite[Ex 3.17]{AC20}. The interest in doing so comes from the $(...)dB$ term which fits directly in our setup, while outside the scope of that work.\footnote{From \cite{AC20}: ``we expect all of the analysis [with $(...)dB$] to follow with appropriate technical adjustments.''} \begin{align*} d \left(\begin{array}{c} Y_t^{s, y, z, \eta, \BX }\\ Z_t^{s, z, \eta, \BX } \end{array}\right) &= \left(\begin{array}{c} 0\\ \eta_t \end{array}\right) d t + \left(\begin{array}{c} Z_t^{s, z, \eta, \BX}\\ 0 \end{array}\right) \sigma d B + \left(\begin{array}{c} Z_t^{s, z, \eta, \BX}\\ 0 \end{array}\right) d \BX, \\ \left(\begin{array}{c} Y_s^{s, y, z, \eta, \BX }\\ Z_s^{s, z, \eta, \BX } \end{array}\right) &= \left(\begin{array}{c} y\\ z \end{array}\right) . \end{align*} where $\BX$ is a (scalar) rough path that we assume geometric. It is implicit in this rough SDE that $Z ^{s, z, \eta, \BX}$ has zero stochastic Gubinelli derviative, the stochastic rough integral $\int Z d \BX$ then makes sense without the compensating terms (of the form $\sum Z'_u \mathbb{X}_{u, v}$) i.e. converges directly as Riemann-Stieltjes integral (so that this particular example can also be understood pathwise, via integration by parts, exploiting finite variation of $Z ^{s, z, \eta, \BX}$. With transaction cost $\varepsilon \eta^2$ for any $\vep>0$, the agent's expected terminal wealth is then given by the value function \[ v (s, y, z) = \sup_{\eta} \mathbb{E} \left[ Y_T^{s, y, z, \eta; \BX} - \int_s^T \varepsilon \eta_t^2 \hspace{0.17em} dt \right] . \] An easy localization argument shows that this example fits in the setting of Theorem \ref{thm52}. Replacing $\BX$ by a smooth path $X $, $v$ satisfies \[ - d_t v = \frac{1}{2} z^2 \sigma^2 \frac{\partial^2 v}{\partial y^2} \hspace{0.17em} dt + \frac{1}{4 \epsilon} \left( \frac{\partial v}{\partial z} \right)^2 \hspace{0.17em} dt + z \frac{\partial v}{\partial x} \hspace{0.17em} d X, \qquad v (T, y, z) = y, \] the explicit solution of which is $$ v (s, y, z) = y + (X_T - X_s) z + \frac{1}{4 \varepsilon} \int_s^T (X_T - X_s)^2 d s;$$ the expression is clearly robust in $X$ and so illustrates the abstract stability result of Theorem \ref{thm52}, also verifiable of course from the stochastic representation of $v$. \end{exam} \section{Pathwise stochastic control with general rough path noise}\label{sec:pathwise-control} \subsection{Pathwise stochastic control via randomised RSDEs}\label{sec-randomised-rsde} Consider \begin{equation} \label{sec51productbasis} ( {\Omega},\MG,(\MF_t)_t,\P) = (\Omega',\mathcal G',(\cff'_t)_t,\P') \otimes (\Omega'',\mathcal G'',(\cff''_t)_t, \P''), \end{equation} where $(\Omega',\mathcal G',(\MF'_t)_t,\P')$ is the canonical space with $B$ the $d_B$-dimensional Brownian motion, $\MF'$ the augmented filtration of $\MF^B$, and $\P'$ the Wiener measure. Write $\omega = (\omega',\omega'')$. Consider also the rough path valued random variable\footnote{The actual filtration on $\Omega''$ and the ambient $\sigma$-field $\mathcal G''$ will play little role in this section. Of course, one can also work with the filtration generated by $\BW$ and also set $\mathcal G'' = \MF_T''$, ifso desired.} $$\BW: (\Omega'',\MF_T'' ,\P'') \rightarrow (\MC_T ,\ccc_T )$$ and controls $\eta = \eta_t (\ome' , \mathbf{X})$, for $\mathbf{X} \in \MC_T$, taken in \[ \mathcal{A}^1 : = \left\{ \eta \text{ is } \mathfrak{O}' \otimes \{ \emptyset, \mathscr{C}_T \}/ \aaa \text{-measurable} \right\} \subset \mathcal{A}^2 : = \left\{ \eta \text{ is } \mathfrak{O}' \otimes \mathfrak{C}_T / \aaa \text{-measurable} \right\} . \] Also write $\bar{\eta} = \bar{\eta}_t (\omega) \in \bar{\mathcal{A}}^i$ if $\bar{\eta}_t (\omega) = \eta_t (\ome' , \mathbf{W} (\omega'') )$ for $\eta \in \mathcal{A}^i$, $i = 1, 2.$ Note $\bar{\eta}_t \in \cff'_t \vee \cff''_T$. To simplify the discussion, for the remainder of this section we assume running cost $\ell=0$ and $f=f(t,y)$ is sufficiently regular in $t$ so that $f'=0$). More precisely, we make the following \begin{assumption} \label{assum-path-stoch} \item[$(1)$] Assumption \ref{assum-dpp}, (1). \item[$(2)$] $f$ is defined on $[0,T] \times \R^{d_Y}$ such that \(f(t,.)\in \C^\gamma_b\) for each \(t\) and \footnote{With regard to Assumption 5.1. (2), this implies that $(f, 0) \in \mathbf{D}^{2\beta}_X \mathcal{C}_b^{\gamma}$ for any $\alpha$-H\"older path $X$.} \[ \sup_{(s,t)\in \Delta_T}\frac{| f (t,.) - f (s,.) |_\infty}{ | t - s |^{2 \beta}}<\infty \] for $\beta \in (0,\alpha]$, $\gamma > \frac{1}{\alpha}$ such that $\alpha+\beta>\frac12$ and $\alpha + (\gamma-1)\beta>1.$ \item[$(3)$] Assumption \ref{assum-dpp}, (3). \end{assumption} Theorem \ref{thm:wellposed} shows that for $\eta \in \MA^2$, we have a unique solution $Y^{\ctrl, \BX}(\ome')$ to the RSDE \be\label{r-dsde'}\left\{ \begin{split} & dY^{\ctrl,\BX}_t(\omega')=b(t,Y^{ \ctrl,\BX}_t ,\ctrl_t(\ome',\BX) )dt+\sigma(t,Y^{ \ctrl,\BX}_t ,\ctrl_t(\ome',\BX) )dB_t(\omega')+f(t,Y_t^{\ctrl,\BX}) d\BX_t, \\ & Y_s = y, \end{split}\right. \ee which under the given assumptions we can and will take a $\ccc_T$-optional version, cf. Theorem \ref{thm-rsde-optional} (measurable selection). We then have that \be\label{randomized-rsde} \bar{Y}^{\bar{\eta}} (\omega) := Y^{\bar{\eta}, \BW (\omega'')} (\omega'):= Y^{\eta, \mathbf{X}}|_{\BX=\BW(\ome'')} \ee defines a measurable process (see Section \ref{sec:BMcase} for a discussion when it coincides with solution to a classical SDE) and more precisely \[ \bar{Y}^{\bar{\eta}}_t (\omega', \omega'') \in \cff'_t \vee \cff''_T . \] Define \[ \mathcal{V}^i (s, y ; \omega) \assign {\tmop{essinf}_{\eta \in \mathcal{A}^i}} \mathbb{E}^{s, y} (g (\bar{Y}^{\bar{\eta}}_T) {| \cff_T^{\mathbf{W}} }) = {\tmop{essinf}_{\bar{\eta} \in \bar{\mathcal{A}}^i}} \mathbb{E}^{s, y} (g (\bar{Y}^{\bar{\eta}}_T) | \cff_T^{\mathbf{W}} ), \] for $i = 1, 2$, where $\cff_T^{\mathbf{W}} $ is the $\sigma$-algebra generated by $\BW,$ and we recall \begin{defi}\label{def-essinf} Let $\chi$ be a nonempty set of random variables on a probability space $(\Omega,\MF, P)$. The essential infimum of $\chi$, denoted by $P\text{-}\mathrm{essinf} \chi$, is a random variable $\xi,$ satisfying: $(i)$ for any $X \in \chi,$ $\xi \le X$ $P$-a.s.; $(ii)$ for any random variable $Y$ with $Y \le X$ \(P\)-a.s. for all $X \in \chi,$ it is necessary that $Y\le \xi$ $P$-a.s.. \end{defi} On the other hand, to apply our dynamical programming for RSDEs to the random case, let \be\label{mv'} \bar{\MV}(s,y; \ome):= \MV(s,y;\BW(\ome)) \equiv \mathcal{V} (s, y ; \mathbf{X}) |_{\mathbf{X} = \mathbf{W} (\omega)}:= \mathrm{inf}_{\ctrl \in \MA} \E' [g(Y^{\ctrl,\BX})]\Big|_{\BX=\BW(\ome)}, \ee where \be\label{rough-ad-contr} \MA:=\{\ctrl: ([0,T]\times \Omega', \MO') \rightarrow (A,\aaa) \text{ is measurable} \}. \ee To build the relation between the different random value functions we need the following proposition. \begin{prop}\label{equ-condi-ep} Suppose that $(b,\sigma,f)$ satisfies \cref{assum-path-stoch} $(1), (2)$. For any $\ctrl \in \MA^2,$ let $Y^{\ctrl,\BX}$ be the solution to \eqref{r-dsde'} with any $\BX \in \MC_T$, and $\bar{Y}^{\bar{\eta}}$ be the randomized solution given by \eqref{randomized-rsde}. Let $g$ be bounded and measurable. Then we have \be \label{equiv-condi} \mathbb{E} [g (\bar{Y}^{\bar{\eta}}_T) | {\text{$\cff$}}_T^{\mathbf{W}} ] = \mathbb{E} [g (Y^{\eta, \mathbf{X}}_T)] \Big|_{\mathbf{X} = \mathbf{W}}. \ee \end{prop} \begin{proof} According to Theorem \ref{thm-rsde-optional}, we see that $\bar{Y}^{\bar{\eta}}_T$ is measurable on $(\Omega' \times \Omega'', \cff'_T \otimes \cff''_T),$ and thus the left hand side of \eqref{equiv-condi} is well-defined. For any bounded measurable function $\phi$ on $(\MC_T, \ccc_T ),$ let $\xi:=\phi(\BW).$ Since $Y^{\eta, \mathbf{X}}_T$ is jointly measurable on $\Omega' \times \MC_T,$ by the Fubini's lemma and identity \eqref{randomized-rsde}, we have for any $\ome'' \in \Omega'',$ $$ \mathbb{E}' [g (Y^{\eta, \mathbf{X}}_T)] |_{\mathbf{X} = \mathbf{W} (\omega'')}= \mathbb{E}' [g (Y^{\eta, \mathbf{X}}_T) |_{\mathbf{X} = \mathbf{W} (\omega'')} ]= \mathbb{E}' [g ( \bar{Y}^{\bar{\eta}}_T ) ]. $$ It follows that $$ \E \left[ \xi \mathbb{E}' [g (Y^{\eta, \mathbf{X}}_T)] |_{\mathbf{X} = \mathbf{W} (\omega'')} \right]= \E[\xi g(\bar{Y}^{\bar \eta }_T)], $$ which completes the proof. \end{proof} \begin{rem} \label{cost-g-X} Similar to Remark \ref{g-dep-X}, if $\mathcal{V}:[0,T] \times \R^{d_Y} \times \MC_T \rightarrow \R$ bounded, jointly measurable, the above proposition applies directly with $g (Y^{ {\eta}, \mathbf{X}}_T)$ replaced by $\mathcal{V}(r,Y^{(s,y,\ctrl, \BX)}_{r}; \BX)$ for any $r\in[s,T]$, a term seen on the right-hand side of \eqref{DPP-path-stoch}. \end{rem} Then we have the following equivalence between $\bMV$ and $\MV^i$, $i=1,2.$ \begin{theorem}\label{rep-value-fct} Suppose that \cref{assum-path-stoch} holds. Then $ \bar{\mathcal{V}}$ given by \eqref{mv'} is a continuous modification of both $\mathcal{V}^1$ and $\mathcal{V}^2$ with a.s. regularity inherited by the regularity of $\mathcal{V}$ provided by \cref{thm:RoughValueReg}. In particular, for every $(s,y) \in [0,T] \times \R^{d_Y}$, with probability one, $$ \bar{\mathcal{V}} (s, y ; \omega) = \mathcal{V}^1 (s, y ; \omega) =\mathcal{V}^2 (s, y ; \omega).$$ \end{theorem} \begin{remark} In \cite[Thm 5.5]{BM07} the authors consider a stochastic value function, that can be seen to coincide with ours in case of Brownian (rough path) randomization, and show the existence of a continuous version. Our improvement here is two-fold. (1) At no point have we assumed Brownian (rough path) statistics of $\mathbf{W}$, as one might expect since the entire analysis is conditional on the information provided by $\mathbf{W}$. (2) \cref{thm:RoughValueReg} provides a modulus of regularity for $\mathcal{V}$ which is inherited by $\bar{\mathcal{V}}$ and thus yields regularity results for the stochastic value function, which is new even in the case previously considered by \cite{BM07}. \end{remark} \begin{proof}[Proof of \cref{rep-value-fct}] Note that $\MA^1 \subseteq \MA^2 ,$ and thus $\MV^1 \ge \MV^2$ a.s.. Hence we only need to show that $\bMV \ge \MV^1$ and $\MV^2 \ge \bMV$ a.s.. We omit \((s,y)\) in the notations in the remaining of the proof below. (i) ``$\MV^2 \ge \bMV$'': Let $0 \leqslant$ $\phi \in \C_b$($\mathscr{C }_T$). For any $u = u_t (\omega' , \mathbf{X}) \in \mathcal{A}^2$, note that for $\mathbf{X}$ fixed, $(t, \omega') \mapsto u_t (\omega' , \mathbf{X})$ is in $\mathcal{A}$. We have \[ \begin{split} \mathbb{E} [\phi (\mathbf{W} (\ome'')) \MV ( \mathbf{W} (\omega''))] = & \ \mathbb{E} [\phi (\mathbf{W} (\ome'')) (\inf_{\eta \in \mathcal{A}} \mathbb{E}' [g (Y^{\eta, \mathbf{X}}_T)]) |_{\mathbf{X} = \mathbf{W} (\omega'')}]\\ \le & \ \mathbb{E} [\phi (\mathbf{W} (\ome'')) (\mathbb{E}' [g (Y^{u , \mathbf{X}}_T)]) |_{\mathbf{X} = \mathbf{W} (\omega'')}]. \end{split} \] In view of Proposition \ref{equ-condi-ep}, we have \[ \begin{split} \mathbb{E} [\phi (\mathbf{W} (\ome'')) (\mathbb{E}' [g (Y^{u, \mathbf{X}}_T)]) |_{\mathbf{X} = \mathbf{W} (\omega'')}] & = \mathbb{E} [\phi (\mathbf{W} (\ome'')) \mathbb{E} [g (\bY^{ \bar{u} }_T) \mid \cff^{\mathbf{W}}_T] ],\\ \end{split} \] which implies that \[ \bMV= \mathcal{V} ( \mathbf{W} (\omega'')) \leqslant \tmop{essinf}_{ {u} \in \mathcal{A}^2 } \mathbb{E} [g (\bY^{ \bar u }_T) \mid \cff^{\mathbf{W}}_T] (\omega'')=\MV^2. \] (ii) ``$\bMV \ge \MV^1$'': For every $\varepsilon >0, \mathbf{X} \in \MC_T,$ by \cref{thm:RoughValueReg}, there exists $\delta=\delta(\vep,\BX) >0$ such that for any $\bx\in \MC_T,$ \[ d (\mathbf{x}, \mathbf{X }) < \delta \Rightarrow \sup_{\eta \in \mathcal{A} } \left(\mathbb{E}' [g (Y^{\eta, \mathbf{X}}_T)] -\mathbb{E}' [g (Y^{\eta, \mathbf{x}}_T)]\right) < \varepsilon, \] which implies \be\label{vep-appr} | \MV(\BX)-\MV(\Bx) | < \vep, \ \ \ |J(\ctrl;\BX)- J(\ctrl;\Bx)|< \vep, \ee where we recall $J(\ctrl;\BX):= \mathbb{E}' [g (Y^{\eta, \mathbf{X}}_T)].$ Then by the same argument in the proof of Lemma \ref{lem1}, there exists $\{\mathbf{x}_i\}_i \subset \mathscr{C}_T,$ and \[ B_i : = \{ B (\mathbf{x}_i, \delta (\varepsilon, \mathbf{x}_i)) : i \in \mathbb{N} \}, \] such that $\{B_i\}_i$ is a countable open cover of $\MC_T$. Let $\{P_i\}_i$ be a (Borel) measurable partition of $\mathscr{C}_T$ by $P_1 := B_1, P_2 := B_2 \backslash B_1, \ldots ,$ with some $ \By_i \in P_i$ for any $i$. For any $i,$ let $\ctrl^{i} \in \MA$ be $\vep$-optimal for $\MV(\By_i ),$ i.e. $$ \MV(\By_i ) \le J(\ctrl^{i};\By_i ) \le \MV(\By_i ) + \vep. $$ By the definition of $P_i$, \eqref{vep-appr} and the above inequality, we have \begin{align} \label{est-3} \MV(\BX)|_{\BX=\BW(\ome'') } & = \sum_{i\in \N} \MV(\BW (\ome'') ) 1_{P_i}(\BW(\ome'')) \\ \nonumber & \ge \sum_{i\in \N } (\MV(\By_i ) - \vep ) 1_{P_i}(\BW (\ome'')) \\ \nonumber & \ge \sum_{i\in \N } (J(\ctrl^i; \By_{i}) - 2\vep ) 1_{P_i}(\BW (\ome'')). \end{align} In view of the second inequality in \eqref{vep-appr}, for any $i,$ \be\label{est-4} (J(\ctrl^i; \By_i ) - 2\vep ) 1_{P_i}(\BW (\ome'')) \ge (J(\eta^i; \BW (\ome'') )-3 \vep ) 1_{P_i}(\BW (\ome'')). \ee Note that $u^{i}(\ome',\BX) := \ctrl^{i}(\ome') \in \MA^1 \subseteq \MA^2 ,$ which by \cref{equ-condi-ep}, implies that \be \label{rela-5} J(\eta^{i}; \BW (\ome'')) = \E'[g(Y_T^{\ctrl^{i},\BX })]|_{\BX=\BW (\ome'')}= \E[ g(\bY^{\bar{u}^i }_T ) | \MF^{\BW}_T ](\ome''). \ee Take \eqref{est-4}, \eqref{rela-5} to \eqref{est-3}, and we have \begin{align*} \MV(\BX)|_{\BX=\BW(\ome'') } & \ge \sum_{i\in \N } \E[ g( \bY^{\bar{u}^i }_T ) | \MF^{\BW}_T ](\ome'') 1_{P_i}(\BW(\ome'')) - 3\vep \\ & \ge \sum_{i\in \N } \MV^1(\ome'') 1_{P_i}(\BW (\ome'')) - 3\vep \\ & = \MV^1(\ome'') -3 \vep. \end{align*} Thus upon $\varepsilon \downarrow 0$, we see $\mathcal{V} ( \mathbf{W} (\omega'')) \ge \MV^1 (\omega'')$ a.s.. \end{proof} \subsection{Stochastic dynamical programming principle} In view of \cref{thm:RoughValueReg} and \eqref{mv'}, $\bMV:[0,T]\times \R^{d_Y} \times \Omega \rightarrow \R$ is $\bbb_T \otimes \bbb^{d_Y} \otimes \{\emptyset, \Omega' \} \otimes \cff''_T $-measurable. Thus $\ome \mapsto \bMV(t,\bY^{\bctrl}_t(\ome); \ome)$ is $\cff'_t \otimes \cff''_T$-measurable for any fixed $t\in[s,T]$ and \(\eta\in\caa^2\) (recall we took $\ell=0$ w.l.o.g.). By our dynamical programming principle for rough SDEs (\cref{thm-RBP}), we have the following version for the pathwise stochastic control problem. \begin{theorem}[DPP for the pathwise stochastic control] Suppose that \cref{assum-path-stoch} holds. For any $y \in \R^{d_Y}$, $s\in[0,T],$ and $\ctrl \in \MA^2,$ let $\bY^{s,y,\bctrl}$ and $ \bar{\mathcal{V}}(s,y;\ome)$ be given by \eqref{randomized-rsde} and \eqref{mv'} respectively. Then we have for any $h\in[0,T-s]$ and $i=1,2$ \be\label{DPP-path-stoch} \bMV(s,y;\ome)= {\tmop{essinf}_{\eta \in \mathcal{A}^i}}\ \E[ \bMV(s+h, \bY^{s,y,\bctrl}_{s+h}; \ome) \mid \cff^{\BW}_T], \ee where the left-hand side is continuous on $[0,T]\times \R^{d_Y}$ a.s. as in Theorem \ref{rep-value-fct}. \end{theorem} \begin{proof} Let $\bMV^i(s,h,y,\ome),$ $i=1,2$ be the right-hand side of \eqref{DPP-path-stoch}. Similar to the proof of Theorem \ref{rep-value-fct}, since $\MA^1 \subseteq \MA^2, $ we easily have $ \bMV^1 \ge \bMV^2, $ a.s. Then we only need to show that $\bMV^2 \ge \bMV$ and $\bMV \ge \bMV^1$ a.s.. For ``$\bMV^2 \ge \bMV$'' part, take any $u \in \MA^2.$ By \cref{cost-g-X}, we have \[ \E[ \bMV(s+h, \bY^{s,y,\bar{u}}_{s+h}; \ome) \mid {\text{$\cff$}}^{\BW}_T]= \E'[\MV(s+h, Y^{(s,y,u,\BX) } ; \BX)] \Big|_{\BX=\BW(\ome'')}. \] Recall $\MA$ is defined by \eqref{rough-ad-contr}, and thus $u(\cdot,\BX) \in \MA$ for any fixed $\BX$. By Theorem \ref{thm-RBP}, it follows that \be \begin{split} \E[ \bMV(s+h, \bY^{s,y,\bar{u}}_{s+h}; \ome) \mid \cff^{\BW}_T] \ge & \left(\inf_{\ctrl \in \MA} \E'[\MV(s+h,Y^{(s,y,\ctrl,\BX)}_{s+h};\BX)] \right) \Big|_{\BX=\BW(\ome'')} \\ =& \ \MV(s,y;\BX)\Big|_{\BX=\BW(\ome'')}= \bMV(s,y;\ome). \end{split} \ee For ``$\bMV \ge \bMV^1$'' part, again by Theorem \ref{thm-RBP}, we have $$ \bMV= \left(\inf_{\ctrl \in \MA} \E'[\MV(s+h,Y^{(s,y,\ctrl,\BX)}_{s+h};\BX)] \right) \Big|_{\BX=\BW(\ome'')}. $$ We claim that for any $\vep >0,$ \[ \left(\inf_{\ctrl \in \MA} \E'[\MV(s+h,Y^{(s,y,\ctrl,\BX)}_{s+h};\BX)] \right) \Big|_{\BX=\BW(\ome'')} \ge \bMV^1(\ome) - 3 \vep, \] which implies our result. Indeed, since $\MV(s+h,\cdot; \cdot)$ is continuous on $\R^{d_Y} \times \MC_T$ in view of \cref{thm:RoughValueReg}, we can repeat the second part of the proof of Theorem \ref{rep-value-fct} with $g(Y^{\ctrl,\BX}_T)$ replaced by $\MV(s+h,Y^{s,y,\ctrl,\BX}_{s+h};\BX),$ and thus the claim follows. \end{proof} \section{Pathwise stochastic control with Brownian rough path noise } \label{sec:BMcase} Now in particular, we take $(\Omega'',\mathcal G'',(\cff''_t)_t,\P'')$ as any filtered probability space with a $d_W$-dimensional Brownian motion $W.$ Again consider \begin{equation} \label{sec51productbasis} ( {\Omega},\MG,(\MF_t)_t,\P) = (\Omega',\mathcal G',(\cff'_t)_t,\P') \otimes (\Omega'',\mathcal G'',(\cff''_t)_t,\P''), \end{equation} where $(\Omega',\mathcal G',(\cff'_t)_t,\P')$ is defined as in Section \ref{sec-randomised-rsde}. This section is devoted to case when $\BX$ happens to be some realization of some Brownian rough path $\BW = \BW (\omega'')$, over some a Brownian motion $W=W(\omega'')$, independent of $B = B(\omega')$, in which case one may expect connections to ``doubly stochastic'' It\^o stochastic differential equations, provided controls are causal in $\BX$. We review the standard construction of the It\^o and Stratonovich Brownian rough path, following Chapter 3 in \cite{FH20}. For each $(s,t)\in \Delta_T$, define $\mathbb{W}^{\text{It\^o}}_{s,t}=\int_s^t \delta W_{s,r}dW_r$ as an It\^o integral and $\mathbb{W}_{s,t}^{\mathrm{Strato}}:=\int_s^t \delta W_{s,r}\circ dW_r=\mathbb{W}^{\text{It\^o}}_{s,t}+\frac12 (t-s)I$ as a Stratonovich integral. It is well-understood that $\BW^{\text{It\^o}}(\omega'')=(W(\omega''),\mathbb{W}^{\text{It\^o}}(\omega''))$ and $\BW^{\mathrm{Strato}}(\omega'')=(W(\omega''),\mathbb{W}^{\mathrm{Strato}}(\omega''))$ are elements of $\cap_{\alpha\in(1/3,1/2)} \CC^{0,\alpha}$ for all $\omega''\in N_1^c$, where $N_1$ is a $\P''$-null set of $\Omega''$. Moreover, for any $t\in[0,T]$, the {\it lifting mapping } \be\label{ito-lift} \begin{array}{llll} \BW:=\BW^{\text{It\^o}}: & [0,T]\times \Omega'' & \rightarrow & (\MC_T, \ccc_T) \\ & (t,\ome'') &\mapsto & \BW^{\text{It\^o}}(\ome'')_{.\wedge t} \end{array} \ee is progressively measurable w.r.t. $(\MF''_t )_t$. Moreover, we have the following result on the progressive measurability for the composition of $\BW^{\text{It\^o}}$ with $\ccc_T$-optional process introduced in Definition \ref{def:optional}. \begin{prop}\label{prop:rou-inte-agr-ito} Suppose that $(Z,Z')$ defined on $[0,T] \times \Omega' \times \MC_T$ is $ \ccc_T$-optional, and satisfies $(1)$ $(Z_0, Z'_0)(\ \cdot \ ; \BW_0) \in L_q(\MF_0;\P)$; $(2)$ for any $\BX=(X,\X) \in \MC_T,$ $(Z,Z')^{\BX}(\cdot):= (Z,Z')(\ \cdot \ ; \BX) \in \BD^{\gamma,\gamma'}_X L_{p,q}(\Omega')$, where $q \ge p \ge 2,$ \(\gamma,\gamma'\in[0,1]\) such that \(\alpha+\gamma>1/2\) and \(\alpha+\min(\alpha,\gamma)+\gamma'>1\); $(3)$ $(Z, Z')$ is $\BX$-{\it causal}.\ Then for any $t\in[0,T],$ \[ \int_0^t Z^{\mathbf{W} (\ome'')}_s (\omega') d W_s (\omega'')= \Big( \int_0^t (Z, Z')^{\mathbf{X}} (\omega') d \mathbf{X} \Big) \Big|_{\mathbf{X} = \mathbf{W} (\omega'')}, \ \ \P\text{-a.s.} \] where the left-hand side is the It\^o integral. \end{prop} \begin{proof} By the $\mathbf{X}$-causality of $(Z, Z')^{\mathbf{X}}$, we have that $(Z,Z')^{\BW(\cdot)}(\cdot)$ is $(\MF_s)$-adapted, and thus the It\^o integral is well-defined. Let \(J=(J^\BX_t)\) be the a \(\ccc_T\)-optional version of the rough stochastic integral $ \int_0^t (Z, Z')^{\mathbf{X}} (\omega') d \mathbf{X} $ whose existence is guaranteed by \cref{optional-r.i.}. From \cref{prop.rsint}, we see that for any fixed \(t\) and \(\BX\), \(J^\BX_t\) is the limit in \(\P'\)-probability of compensated Riemann sums approximation, denoted by $\{I_n^{\mathbf{X}} (\omega') : n \geqslant 1 \}$. By Corollary \ref{coro-ito-rough}, it also holds that \[ \lim_{n \to \infty} I_n^{\mathbf{W} (\ome'')} (\omega') = \Big( \int_0^t (Z, Z')^{\mathbf{X} } (\omega') d \mathbf{X} \Big) (\omega', \mathbf{X}) \Big|_{ \mathbf{X} = \mathbf{W} (\omega'')} \quad \text{in $(\mathbb{P}' \otimes \mathbb{P}'')$-probability. } \] By assumption, $W = W (\omega'')$ is a $\left( {\cff''_s} \right)$-Brownian motion on $({\Omega''}, ( {\cff}''_{s})_s, \ensuremath{\mathbb{P}}^{''})$, thus also a $(\cff_s)$-Brownian motion on the product space $(\Omega, (\cff_s)_s, \mathbb{P}) = (\Omega', ( \cff'_s)_s, \mathbb{P}') \otimes (\Omega'', ( \cff''_s)_s, \mathbb{P}'')$. By an argument similiar to [FH20], we see that the limits in $(\mathbb{P}' \otimes \mathbb{P}'')$-probability, \[ \lim_{n \to \infty} I_n^{\mathbf{W} (\ome'') } (\omega') \] and \[ \lim_{n \to \infty} \sum_{[s, r] \in \pi_n} Z_s^{\mathbf{W} (\ome'') } (\omega') (W_r - W_s) (\omega'') \] exist and coincide. Noting that the latter is the It\^o integral, we obtain the claimed identity. \end{proof} To study the equivalence between the randomised rough system \eqref{randomized-rsde} and the stochastic system with two Brownian motions, we need to consider the following $\mathbf{X}$-causal controls, in which case there is no problem in the definition of the later system. $$ \MA^c:=\{ \ctrl= \ctrl_t(\ome', {\text{$\BX$}}_{.\wedge t} ) \text{ is measurable w.r.t. }\MO'\otimes \ccc_T \}. $$ \begin{prop} Suppose that $(b,\sigma,f)$ satisfies Assumption \ref{assum-path-stoch}, and $\BW=\BW^{\text{It\^o}}$ is given by \eqref{ito-lift}. For any $\ctrl \in \MA^c$, let $\bY^{\bar \eta }$ be defined by \eqref{randomized-rsde}, and $Y$ be the solution to the SDE \be\label{BB-rde} \left\{ \begin{split} & dY_t =b(t,Y_t ,\bctrl_t )dt+\sigma(t,Y_t ,\bctrl_t )dB_t(\omega')+f(t,Y_t ) dW_t(\ome''), \\ & Y_s = y. \end{split}\right. \ee with $\bctrl_t(\ome',\ome''):= \ctrl_t(\ome',\BW(\ome''))$. Then for any $\ctrl \in \MA^c$ and $t\in[0,T], $ we have $\bY^{\bar \eta }_t= Y_t$, $\P$-a.s.. \end{prop} \begin{proof} We recall that \(Y^{\eta,\BX}\) is the \(\ccc_T\)-optional solution to \eqref{r-dsde'}. Moreover, because \(\eta\in\caa^c\), it is evident that $\P'$-a.s. $Y^{\ctrl, \BX}_t = Y^{\ctrl, \BX_{\cdot \wedge t}}_t$ for all $t \in [0,T].$ By \cref{def:solutionRSDEs_compendium}, we see that \((Z,Z'):=(f(Y^{\eta,\BX} ), D_yf(Y^{\eta,\BX} )f(Y^{\eta,\BX} ))\) belongs to \(\textbf{D}^{\bar \alpha,\bar \alpha'} _XL_2\) for some suitable parameters \(\bar \alpha,\bar{\alpha}' \) stated there. Then according to Proposition \ref{prop:rou-inte-agr-ito}, \be\label{eq-rs1} \int_s^t f_r( Y^{\ctrl, \BX}_r ) d\BX_r \Big|_{\BX= \BW(\ome'')}= \int_s^t f_r (\bY^{\bar \eta}_r) dW(\ome''). \ee On the other hand, note that for any $\BX \in \MC_T,$ the following convergence holds in $\P'$-probability, \[ \begin{split} \int_s^t b_r(Y^{\ctrl, \BX }_r , \ctrl_r(\ome', \BX) )dr = \lim_{|\pi_t|\rightarrow 0} \sum_{[u,v] \in \pi_t } b_u(Y^{\ctrl, \BX }_u, \ctrl_u(\ome', \BX ) )(v-u), \end{split} \] where $\pi_t$ is any partition of $[s,t].$ By Corollary \ref{coro-ito-rough}, we have the following convergence in $\P$-probability \be\label{eq-rs2} \begin{split} \int_s^t b_r(Y^{\ctrl, \BX }_r , \ctrl_r(\ome', \BX) )dr \Big|_{\BX=\BW(\ome'')}& = \lim_{|\pi_t|\rightarrow 0} \sum_{[u,v] \in \pi_t } b_u(Y^{\ctrl, \BX }_u, \ctrl_u(\ome', \BX ) ) \Big|_{\BX=\BW(\ome'')} (v-u), \\ & = \lim_{|\pi_t|\rightarrow 0} \sum_{[u,v] \in \pi_t } b_u( \bY^{\bctrl }_u, \bctrl_u ) (v-u)\\ & = \int_s^t b_r( \bY^{\bctrl }_r, \bctrl_r ) dr. \end{split} \ee Similarly, we have \be\label{eq-rs3} \int_s^t \sigma_r(Y^{\ctrl, \BX }_r , \ctrl_r(\ome', \BX) )dB_r \Big|_{\BX=\BW(\ome'')} = \int_s^t \sigma_r( \bY^{\bctrl }_r, \bctrl_r ) dB_r. \ee In view of \eqref{eq-rs1}, \eqref{eq-rs2} and \eqref{eq-rs3}, we obtain \be \bY^{\bctrl}_t = y + \int_s^t b_r( \bY^{\bctrl }_r, \bctrl_r ) dr + \int_s^t \sigma_r( \bY^{\bctrl }_r, \bctrl_r ) dB_r + \int_s^t f_r (\bY^{\bar \eta}_r) dW(\ome''), \ee which implies that $\bY^{\bctrl}$ is a solution to \eqref{BB-rde}, and our result follows by the uniqueness of solutions to \eqref{BB-rde}. \end{proof} \begin{rem} One may consider \be\label{mvob} \MV^c ( \ome''):= \mathrm{essinf}_{\ctrl \in {\MA^c} } \E[g({\bY}^{\bar \ctrl }_T) \mid {\MF}^{\BW}_T ]. \ee Note that $\MA^1 \subseteq \MA^c \subseteq \MA^2 $, it follows by Theorem \ref{rep-value-fct} that $\MV^c= \MV^1= \MV^2= \bMV $. Indeed, $\MA^c$ is the admissible considered in \cite{BM07}. \end{rem} \section*{Appendix: Some measurable selection results} We collect some facts on measurable selection, mostly from Stricker--Yor {\cite{SY78}}. \medskip \noindent Let $(\Omega, \cff, (\cff_t)_{t \geq 0}, P)$ is a filtered probability space satisfying the usual conditions. We agree that $\cff_{0_-} =\cff_0$, and that $\cff=\cff_{\infty} = \bigvee_t \cff_t$. We denote by $\ooo$ (resp. $\mathcal{P}$) the optional (resp. predictable) sigma-algebra on $\mathbb{R}_+ \times \Omega$ associated with this family. Assume a given measurable space $(U, \uuu)$, and we consider functions $X : (u, t, \omega) \mapsto X^u_t (\omega)$ with real values on $U \times \mathbb{R}_+ \times \Omega$. We will say that $X$ is $\uuu$-{\em{measurable}}\footnote{Stricker--Yor call such $X$ simply measurable, we prefer to be more explicit.} without further clarification to indicate that $X$ is measurable with respect to $\uuu \otimes \mathcal{B} (\mathbb{R}_+) \otimes \cff$. We always interpret $X$ as a family, indexed by $u$, of stochastic processes $X^u = (X^u_t)_{t \geq 0}$, and we will consider the phrase ``the process $X^u$ depends measurably on $u$'' as equivalent to ``$X$ is $\uuu$-measurable''. Let $X$ and $\bar{X}$ be two functions as described above; we will sometimes say that $X$ and $\bar{X}$ are {\tmem{indistinguishable}} if, for all $u \in U$, the processes $X^u$ and $\bar{X}^u$ are indistinguishable. \begin{lem}[\cite{SY78}, Lemme 1] Let $X$ be a function on $U \times \mathbb{R}_+ \times \Omega$ such that: \begin{itemize} \item[a)] For all $u \in U$, the process $X^u$ is indistinguishable from a c{\`a}dl{\`a}g process. \item[b)] For all $t \in \mathbb{R}_+$, there exists a function $H_t$ on $U \times \Omega$, measurable with respect to $\uuu \otimes \cff$, such that: $\forall u \in U, X^u_t (\cdummy) = H_t (u, \cdot)$ almost surely $P$-a.s. \end{itemize} Then there exists a $\uuu$-measurable function $Y$ on $U \times \mathbb{R}_+ \times \Omega$, indistinguishable from $X$, such that for all $u$, the trajectories of $Y^u$ are all c{\`a}dl{\`a}g. \end{lem} Suppose that $H_t$ is $\cff_t$-measurable for all $t$. The family $(\cff_t)$ satisfies the usual conditions, the $P$-negligible set $C_u$ belongs to $\cff_0$, and the process $(Y^u_t)$ is adapted to $(\cff_t)$ for all $t$ and is c{\`a}dl{\`a}g, hence optional, for all $u \in U$. However, as pointed out in \cite{SY78} (after Lemme 1), the map $(u, (t, \omega)) \mapsto Y^u_t (\omega)$ is not $\uuu \otimes \ooo$-measurable a priori (see however Proposition \ref{L3SY78} and the remark which follows). \begin{lem}\label{SY78-Lem2} Let $H : (u, t, \omega) \mapsto H^u_t (\omega)$ be a $\uuu \otimes \ooo$-measurable function such that, for all $u$ and all finite $t$, we have \[ \int_0^t |H^u_s (\omega) |ds < \infty \quad P \text{-a.s.} \] Then there exists $Z$ measurable w.r.t. $\uuu \otimes \ooo$ such that, for all $u$, the processes $(Z^u_t)$ and $\left( \int_0^t H^u_s d s \right)$ are indistinguishable. \end{lem} \begin{remark} This lemma is a variation of Lemme 2 in {\cite{SY78}}, that deals with the integration against general finite variation process. \end{remark} \begin{proof} For any fixed $t,$ by the jointly measurability of $H$ on $(U \times [0,t] \times \Omega , \uuu \otimes \bbb_t \otimes \MF_t) $, we see that $ \int_0^t H^u_s d s $ is $\uuu \otimes \MF_t$-measurable. Note that $\int_0^t H^u_s d s$ is continuous in $t$, and our conclusion follows. \end{proof} \begin{proposition}[{\cite{SY78}}, Proposition 1 and Corollary 1] \label{SY_Prop1} (i) Let $(X_n)$ be a sequence of $\uuu \otimes \cff$-measurable functions on $U \times \Omega$. Suppose that for all $u \in U$, the sequence $X_n (u, \cdot)$ converges in probability on $\Omega$. Then there exists a $\uuu \otimes \cff$-measurable function $X$ such that for all $u \in U$, $X (u, \cdot) = \lim_{n \to \infty} X_n (u, \cdot)$ in probability. (ii) Suppose that $X_n (u, \cdot)$ converges in $L^p$ for all $u \in U$. Then there exists a $\uuu$-measurable function $X$ on $U \times \Omega$ such that, for all $u$, $X_n (u, \cdot)$ converges in $L^p$ to $X (u, \cdot)$. \end{proposition} \begin{corollary}\label{coro-ito-rough} Let $Q$ be a probability measure on the parameter space $(U, \uuu)$. For $X$ and $X_n$ as in Proposition \ref{SY_Prop1}, part (i). and (ii), respectively. Then (i) \ $X = \lim_{n \to \infty} X_n$ in $(Q \otimes P)$-probability. (ii) $X = \lim_{n \to \infty} X_n$ in $L^p (Q \otimes P)$, provided $\{ \mathbb{E}^P (| X_n (u, \cdot) |^p) : n \geqslant 1 \}$ is uniformly $Q$-integrable. \end{corollary} \begin{proof} (i) We know from Proposition \ref{SY_Prop1}, part (i), that all $u \in U$ \[ \phi_n (u) : =\mathbb{E}^P (| X (u, \cdot) - X_n (u, \cdot) | \wedge 1) \rightarrow 0. \] By ($\uuu \otimes \cff$) -measurability of $X$, Fubini and bounded convergence \[ \mathbb{E}^{(Q \otimes P)} (| X - X_n | \wedge 1) =\mathbb{E}^Q \phi_n \rightarrow 0 \] which shows that $X_n$ converges to $X$ in $(Q \otimes P)$-probability. \ (ii) We know $\phi^p_n (u) : =\mathbb{E}^P (| X (u, \cdot) - X_n (u, \cdot) |^p) \rightarrow 0$. Assuming $\{ \mathbb{E}^P (| X_n (u, \cdot) |^p) : n \geqslant 1 \}$ to be uniformly $Q$-integrable, implies that the $\{ \phi^p_n : n \geqslant 1 \}$ are UI w.r.t $Q$ and so \[ \mathbb{E}^{(Q \otimes P)} (| X - X_n |^p) =\mathbb{E}^Q \phi_n \rightarrow 0. \] \end{proof} For a measurable process $H : \mathbb{R}_+ \times \Omega \to \mathbb{R}$ (which we assume, for simplicity, to be bounded or positive), and let $^{\circ} H$ and $^p H$ be its optional and predictable projections. Recall that $^{\circ} H$, for example, is closely related to certain conditional expectation operators: \[ \text{-- If } T \text{ is an } (\cff_t) \text{-stopping time, }^{\circ} H_T =\mathbb{E} [H_T |\cff_T],\ \ P\text{-a.s. on } \{T < \infty\} . \] \begin{proposition}[{\cite{SY78}}, Lemme 3] \label{L3SY78}Let $H : U \times \mathbb{R}_+ \times \Omega \to \mathbb{R}$ be a measurable function, bounded or positive. There exist two functions $K$ and $L : U \times (\mathbb{R}_+ \times \Omega) \to \mathbb{R}$, measurable respectively with respect to $\uuu \otimes \ooo$ and $\uuu \otimes \mathcal{P}$, such that for all $u \in U$, $K^u$ (resp. $L^u$) is a version of $^{\circ} H^u$ (resp. $^p H^u$). We will write $K = {^{\circ} H}$, $L = {^p H}$ from now. \end{proposition} \begin{rem} \label{rem_optional}Suppose that a measurable function $H$ on $U \times \mathbb{R}_+ \times \Omega$ is such that, for all $u$, the process $(H^u_t)$ is optional. Then there exists a function $K$ indistinguishable from $H$ such that $(u, (t, \omega)) \mapsto K (u, t, \omega)$ is $\uuu \otimes \ooo$-measurable. Indeed, one immediately reduces this by means of a bijection from $\mathbb{R}$ onto $] 0, 1 [$ in the case where $H$ is bounded, and it is then sufficient to take $K = {^{\circ} H}$. \end{rem} \begin{prop}[{\cite{SY78}}, Proposition 5]\label{SY78-Prop5} Let $M$ be a local martingale, and $J : U \times \mathbb{R}_+ \times \Omega \to \mathbb{R}$ a $\uuu \otimes \ooo$-measurable function such that \[ \forall u \in U, \quad \mathbb{E} \left[ \left( \int_0^{\infty} (J^u_s)^2 d [M, M]_s \right)^{\frac12} \right] < \infty . \] Then there exists a $\uuu \otimes \ooo$-measurable function $Y : U \times \mathbb{R}_+ \times \Omega \to \mathbb{R}$ such that, for all $u \in U$, the process $(Y_t^u)$ is indistinguishable from the optional stochastic integral $\left( \int_0^t J^u_s dM_s \right)_{t \geq 0}$. \end{prop} (If $M$ is only a semimartingale, similar results hold but $J$ needs to be $\uuu$-predictable ($\uuu \otimes \mathcal{P}$-measurable), cf. Theorem 1 in {\cite{SY78}}.) \bibliography{processes_martingale} \end{document}
2412.06848v1
http://arxiv.org/abs/2412.06848v1
Application of Random Matrix Theory in High-Dimensional Statistics
\documentclass[12pt]{article} \usepackage{latexsym,amsmath} \usepackage[dvips]{graphicx} \usepackage{lscape} \usepackage{color} \usepackage[pagewise]{lineno} \usepackage{graphicx} \usepackage {epstopdf} \usepackage {subfigure} \usepackage{hyperref} \usepackage{booktabs} \usepackage{amsmath} \usepackage{multicol} \usepackage{multirow} \usepackage{xcolor} \usepackage{float} \usepackage{amsthm} \usepackage{amsfonts} \usepackage[margin=1in]{geometry} \usepackage[toc]{appendix} \usepackage{stackengine} \usepackage{bbold} \RequirePackage[OT1]{fontenc} \usepackage[authoryear]{natbib} \bibliographystyle{apalike} \usepackage{adjustbox} \usepackage{amsmath,amssymb,amsthm,bm,latexsym} \usepackage{graphicx,psfrag,epsf} \usepackage[ruled,vlined]{algorithm2e} \usepackage{epigraph,xcolor} \usepackage{enumerate} \usepackage{url} \usepackage{booktabs} \usepackage{multirow} \usepackage{hyperref} \usepackage{pifont} \hypersetup{colorlinks=true,citecolor=blue, pdfpagemode=FullScreen,linktocpage=true} \usepackage{cleveref} \usepackage{amsfonts} \usepackage{amsmath} \usepackage{orcidlink} \usepackage{amsthm} \usepackage[T1]{fontenc} \usepackage{multirow} \usepackage{array} \usepackage{booktabs} \newtheorem{lemma}{Lemma} \newtheorem{corollary}{Corollary} \newtheorem{remark}{Remark} \newtheorem{theorem}{Theorem} \newtheorem{proposition}{Proposition} \setlength{\topmargin}{-1in} \setlength{\oddsidemargin}{0in} \setlength{\evensidemargin}{0in} \setlength{\textwidth}{6.5in} \setlength{\textheight}{8.5in} \setlength{\parindent}{.25in} \setlength{\parskip}{1ex} \def\doublespace{\baselineskip=24pt} \def\singlespace{\baselineskip=12pt} \renewcommand{\qed}{\hfill\hbox{\vrule height1.5ex width.5em}} \newcommand{\pf}{\noindent{\bf Proof.} } \newcommand{\XX}{{\cal X}} \def\a{\alpha} \def\b{\beta} \def\d{\delta} \def\e{\epsilon} \def\g{\gamma} \def\I{\vartheta} \def\l{\lambda} \def\m{\mathrm} \def\p{\partial} \def\r{\rho} \def\s{\sigma} \def\t{\theta} \def\x{\times} \def\D{\Delta} \def\Sg{\Sigma} \def\vt{\vartheta} \def\ol{\overline} \def\ul{\underline} \newcommand{\uu} {\mbox{\boldmath$u$}} \newcommand{\ua} {\mbox{\boldmath$a$}} \newcommand{\ug} {\mbox{\boldmath$g$}} \newcommand{\up} {\mbox{\boldmath$p$}} \newcommand{\uB} {\mbox{\boldmath$B$}} \newcommand{\uV} {\mbox{\boldmath$V$}} \newcommand{\uv} {\mbox{\boldmath$v$}} \newcommand{\uW} {\mbox{\boldmath$W$}} \newcommand{\uw} {\mbox{\boldmath$w$}} \newcommand{\uX} {\mbox{\boldmath$X$}} \newcommand{\ux} {\mbox{\boldmath$x$}} \newcommand{\uY} {\mbox{\boldmath$Y$}} \newcommand{\uy} {\mbox{\boldmath$y$}} \newcommand{\uZ} {\mbox{\boldmath$Z$}} \newcommand{\uz} {\mbox{\boldmath$z$}} \def \bfmOmega {{\mbox{\boldmath$\Omega$}}} \def \z {{\mbox{\boldmath$z$}}}\def \y {{\mbox{\boldmath$y$}}} \def \bmu {{\mbox{\boldmath$\mu$}}} \newcommand{\ualpha} {\mbox{\boldmath$\alpha$}} \newcommand{\ubeta} {\mbox{\boldmath$\beta$}} \newcommand{\ugamma} {\mbox{\boldmath$\gamma$}} \newcommand{\udelta} {\mbox{\boldmath$\delta$}} \newcommand{\uepsilon} {\mbox{\boldmath$\epsilon$}} \newcommand{\uvarepsilon} {\mbox{\boldmath$\varepsilon$}} \newcommand{\uzeta} {\mbox{\boldmath$\zeta$}} \newcommand{\ueta} {\mbox{\boldmath$\eta$}} \newcommand{\utheta} {\mbox{\boldmath$\theta$}} \newcommand{\uvartheta} {\mbox{\boldmath$\vartheta$}} \newcommand{\uiota} {\mbox{\boldmath$\uiota$}} \newcommand{\ukappa} {\mbox{\boldmath$\kappa$}} \newcommand{\ulambda} {\mbox{\boldmath$\lambda$}} \newcommand{\umu} {\mbox{\boldmath$\mu$}} \newcommand{\unu} {\mbox{\boldmath$\nu$}} \newcommand{\uxi} {\mbox{\boldmath$\xi$}} \newcommand{\uGamma} {\mbox{\boldmath$\Gamma$}} \newcommand{\uDelta} {\mbox{\boldmath$\Delta$}} \newcommand{\uTheta} {\mbox{\boldmath$\Theta$}} \newcommand{\uLambda} {\mbox{\boldmath$\Lambda$}} \newcommand{\uXi} {\mbox{\boldmath$\Xi$}} \newcommand{\uPi} {\mbox{\boldmath$\Pi$}} \newcommand{\uSigma} {\mbox{\boldmath$\Sigma$}} \def\beq{\begin{equation}} \def\eeq{\end{equation}} \def\beqa{\begin{eqnarray}} \def\eeqa{\end{eqnarray}} \def\beqan{\begin{eqnarray*}} \def\eeqan{\end{eqnarray*}} \def\nn{\nonumber} \def\bc{\begin{center}} \def\ec{\end{center}} \def\btable{\begin{table}[htbp]} \def\etable{\end{table}} \def\bfig{\begin{figure}[htbp]} \def\efig{\end{figure}} \def\pbox{\parbox[t]{5truein}} \def\bi{\begin{itemize}} \def\ei{\end{itemize}} \topmargin=-23pt \def\E{\mathbb{E}} \def\I{\mathbf{I}} \def\P{\mathbb{P}} \def\X{\bm{X}} \def\L{\bm{L}} \def\Y{\bm{Y}} \def\Z{\mathbb{Z}} \def\F{\mathcal{F}} \def\M{\mathcal{M}} \def\S{\mathbb{S}} \def\var{{\textrm{var}}\,} \def\Cov{{\textrm{Cov}}\,} \def\ave{{\textrm{ave}}\,} \def\N{\mathbb{N}} \def\Z{\mathbb{Z}} \def\Q{\mathbb{Q}} \def\R{\mathbb{R}} \def\C{\mathbb{C}} \newcommand{\convD}{\xrightarrow[]{\mathcal{L}}} \newcommand{\convP}{\xrightarrow[]{\mathcal{P}}} \newcommand{\convAS}{\xrightarrow[]{a.s.}} \newcommand{\sgn}{\mathrm{sgn}} \newcommand{\cor}{\mathrm{Corr}} \newcommand{\cov}{\mathrm{Cov}} \newcommand{\logit}{\mathrm{logit}} \newcommand{\ber}{\mathrm{Bernoulli}} \newcommand{\ind}{\mathbb{I}} \newcommand{\G}{\mathcal{G}} \renewcommand{\P}{\mathbb{P}} \renewcommand{\L}{\mathcal{L}} \renewcommand{\vec}[1]{\mathbf{#1}} \newcommand{\eqA}{\overset{A}{\sim}} \newcommand{\RNum}[1]{\uppercase\expandafter{\romannumeral #1\relax}} \DeclareMathOperator*{\argmin}{arg\,min} \DeclareMathOperator*{\argmax}{arg\,max} \newcommand{\abs}[1]{\left\lvert #1 \right\rvert} \newcommand{\norm}[1]{\left\| #1 \right\|} \newcommand{\floor}[1]{\left\lfloor #1 \right\rfloor} \numberwithin{equation}{section} \newtheoremstyle{general} {3mm} {3mm} {\it} {} {\bfseries} {.} {.5em} {} \theoremstyle{general} \newtheorem{definition}{Definition} \newcommand{\sd}[1]{{\color{red}(SD: #1)}} \begin{document} \doublespace \noindent{\huge\bf Application of Random Matrix Theory in High-Dimensional Statistics }\\ \ \noindent Swapnaneel Bhattacharyya,$^{1,2}$, Srijan Chattopadhyay,$^{1,3}$, Sevantee Basu,$^{1,4}$ \\ \ \noindent{$^1$ Indian Statistical Institute, 203 B.T. Road, Kolkata-700108, India}\\ \ \noindent{$^2$ \href{mailto:[email protected]}{[email protected]}\\ \ \noindent{$^3$ \href{mailto:[email protected]}{[email protected]}}\\ \ \noindent{$^4$ \href{mailto:[email protected]}{[email protected]}} \noindent{\bf Abstract}: This review article provides an overview of random matrix theory (RMT) with a focus on its growing impact on the formulation and inference of statistical models and methodologies. Emphasizing applications within high-dimensional statistics, we explore key theoretical results from RMT and their role in addressing challenges associated with high-dimensional data. The discussion highlights how advances in RMT have significantly influenced the development of statistical methods, particularly in areas such as covariance matrix inference, principal component analysis (PCA), signal processing, and changepoint detection, demonstrating the close interplay between theory and practice in modern high-dimensional statistical inference. \\ \noindent{\bf Key words}: Random Matrix Theory(RMT), Empirical Spectral Distribution (ESD), Limiting Spectral Distribution (LSD), Principal Component Analysis (PCA), Canonical Correlation Analysis (CCA), Changepoint Detection (CPD) \newpage \tableofcontents \newpage \section{Introduction} \label{sec:intro} In recent years, the field of statistics has seen a significant shift driven by the rapid generation of large, complex datasets across diverse disciplines such as genomics, atmospheric science, communications, biomedical imaging, and economics. These datasets, often high-dimensional due to their representation in standard coordinate systems, pose challenges that extend beyond the scope of classical multivariate statistical methods. This evolving landscape has necessitated the integration of advanced mathematical frameworks, including convex analysis, Differential geometry, topology and combinatorics, into statistical methodologies. Among these, random matrix theory has emerged as a powerful tool for addressing key theoretical and practical problems in the analysis of high-dimensional data. In this review article, we focus on several application areas of random matrix theory (RMT) in high-dimensional statistics. These include problems in dimension reduction, hypothesis testing for high-dimensional data, regression analysis, and covariance estimation. We also briefly describe the important role played by RMT in enabling certain theoretical analyses in wireless communications and changepoint detection. The challenges posed by high-dimensional data have sparked renewed interest in several classical phenomena within random matrix theory (RMT). Among these, the concept of universality holds particular significance, offering insights into the applicability of statistical techniques beyond the traditional framework based on the multivariate Gaussian distribution. This article focuses on aspects of RMT that are most relevant to statistical questions in this context. In particular, attention is directed toward the behavior of the bulk spectrum, represented by the empirical spectral distribution, and the edge of the spectrum, characterized by the extreme eigenvalues of random matrices. Given the central role of the sample covariance matrix in multivariate analysis, a significant portion of this work is devoted to examining its spectral properties and their implications for statistical applications. A detailed discussion of these topics is present in \citet{bai2010spectral}, \citet{couillet2022random}. Also \citet{johnstone2006high}, \citet{paul2014random} discuss many RMT-based approaches to modern high-dimensional problems. Though RMT has a wide variation of applications beyond statistics, e.g. wireless communications, finance, and econometrics, in this article our key focus is to discuss the RMT-based approaches to standard high-dimensional problems and their novelties in comparison to traditional methods. The article is organized as follows. In \Cref{sec:RMT}, we discuss the key theoretical results from random matrix theory which provides a framework for the statistical methods. We focus mainly on the asymptotic theory of the spectrum of the two kinds of random matrices - covariance matrix and the ratio of covariance matrices, for their widespread applications in statistics. For each of the two kinds of matrices, we discuss the properties of their bulk spectrum and behavior at the edge of the spectrum, when the matrices are of large dimensions. In the next \Cref{sec:application}, we discuss statistical applications of RMT. We focus on four problems: inference on covariance matrices, application in PCA, application in statistical signal detection removing noise, and changepoint detection. \Cref{th:th1} being our contribution, we present the proof of that theorem in the appendix section. We also present a few theoretical applications demonstrating the novelty of this result in \Cref{sebsec: infcovmat}. The rest of the theorems have appeared in the cited papers and we refer to those cited papers for their proof. \subsection{Notations and Abbreviations} In this paper, $\convP$ means convergence in probability, $\convD$ means convergence in distribution. For a random variable $X$, $F_X(\cdot)$ denotes its CDF. $\mathbf{1}(\cdot)$ denotes the indicator function. RMT means Random Matrix Theory, ESD stands for Empirical SPectral Distribution, LSD indicates Limiting Spectral Distribution. \textit{as} $\mu$ means almost surely wrt the measure $\mu$. \section{Background and Motivation} \label{sec:motivation} Random matrices play a fundamental role in statistical analysis, particularly in the study of multivariate data. Classical multivariate analysis, as detailed in influential works such as \citet{mardia2024multivariate}, and \citet{muirhead2009aspects}, frequently addresses key problems through the analysis of random matrices. These problems are typically formulated in terms of the eigen-decomposition of Hermitian or symmetric matrices and can be broadly classified into two categories. The first category involves the eigen-analysis of a single Hermitian matrix, often referred to as the single Wishart problem, encompassing methods such as principal component analysis (PCA), factor analysis, and tests for population covariance matrices in one-sample problems. The second category includes generalized eigenvalue problems involving two independent Hermitian matrices of the same dimension, commonly known as the double Wishart problem. This includes applications like multivariate analysis of variance (MANOVA), canonical correlation analysis (CCA), tests for equality of covariance matrices, and hypothesis testing in multivariate linear regression. Beyond these, random matrices also play a natural role in defining and characterizing estimators in multivariate linear regression, classification (involving sample covariance matrices), and clustering (using pairwise distance or similarity matrices). The analysis of eigenvalues and eigenvectors of random symmetric or Hermitian matrices has a long history in statistics, dating back to \citet{pearson1901liii} pioneering work on dimensionality reduction through PCA. This article provides a concise overview of these classical problems to set the stage for the broader discussion of random matrix theory in statistical applications. Principal component analysis (PCA) is very useful tool for data reduction and model building. The formulation of PCA in classical multivariate analysis at the population level is as follows. Suppose that we measure \( p \) variables (assume real-valued, for simplicity), expressed as a random vector \(\mathbf{X} = \left(X^{(1)}, \ldots, X^{(p)}\right)^{T}\). Suppose also that the random vector \(\mathbf{X}\) has finite variance \(\mathbf{\Sigma} = \mathbb{E}\big[(\mathbf{X} - \mathbb{E}[\mathbf{X}])(\mathbf{X} - \mathbb{E}[\mathbf{X}])^{T}\big]\). The primary goal of PCA is to obtain a lower-dimensional representation of the data in the form of linear transformations of the original variable, subject to the condition that the residual variance is as small as possible. This can be achieved by considering a sequence of linear transformations given by \(\mathbf{v}_k^T \mathbf{X}, k = 1, 2, \ldots, p\), satisfying the requirement that \(\mathrm{Var}(\mathbf{v}_k^T \mathbf{X})\) is maximized subject to the conditions that \(\mathbf{v}_k\) are unit norm vectors in \(\mathbb{R}^p\) (or \(\mathbb{C}^p\) if the data is complex-valued), and \(\mathbf{v}_k\) is orthonormal to \(\{\mathbf{v}_j : j = 1, \ldots, k-1\}\), i.e., \(\mathbf{v}_k^T \mathbf{v}_j = 0\) for \(j = 1, \ldots, k-1\). This optimization problem can be solved in terms of the spectral decomposition of the nonnegative definite Hermitian matrix \(\mathbf{\Sigma}\): \begin{equation} \mathbf{\Sigma} \mathbf{v}_k = \ell_k \mathbf{v}_k, \quad k = 1, \ldots, p, \end{equation} where \(\mathbf{v}_k\) are the orthonormal vectors. Here, \(\ell_k\) (always real-valued) is an eigenvalue associated with \(\mathbf{v}_k\). Note that in this formulation the eigenvalues \(\ell_k\) are ordered, i.e., \(\ell_1 \geq \cdots \geq \ell_p \geq 0\). If \(\ell_k\) is of multiplicity one, then \(\mathbf{v}_k\) is unique up to a sign change. In practice, \(\mathbf{\Sigma}\) is unknown and we typically observe a sample \(\mathbf{X}_1, \ldots, \mathbf{X}_n\) for the variable \(\mathbf{X}\). In that case, the empirical version of PCA replaces \(\mathbf{\Sigma}\) by the sample covariance matrix, \(\mathbf{S}_n = (n-1)^{-1} \sum_{i=1}^n (\mathbf{X}_i - \overline{\mathbf{X}})(\mathbf{X}_i - \overline{\mathbf{X}})^T\), and performs the spectral decomposition for \(\mathbf{S}_n\). The corresponding eigenvectors \(\widehat{\mathbf{v}}_k\) are often referred to as the sample principal components. The corresponding ordered eigenvalues \(\widehat{\ell}_k\) are typically used to detect the dimension of the reduction subspace. One of the commonly used techniques is to plot the eigenvalues against their indices (so-called "scree plot") and then look for an "elbow" in the plot. There are formal tests based on likelihood ratios (see, \citet{mardia2024multivariate}, \citet{muirhead2009aspects}) that assume that, after a certain index, the eigenvalues are all equal and that the observations are Gaussian. Notice that the name "single Wishart" arises from the fact that if \(\mathbf{X}_1, \ldots, \mathbf{X}_n\) are i.i.d. \(N_p(\mathbf{0}, \mathbf{\Sigma})\), then \((n-1)\mathbf{S}_n\) has $W_p(\mathbf{\Sigma},n-1)$ distribution. Under Gaussianity, one of the commonly used tests for sphericity, i.e., the hypothesis \(H_0 : \mathbf{\Sigma} = \mathbf{I}_p\), is Roy's largest root test (\citet{roy1953heuristic}), which rejects \(H_0\) if \(\widehat{\ell}_1\), the largest eigenvalue of \(\mathbf{S}\), exceeds a threshold determined by the level of significance. If \(H_0 : \mathbf{\Sigma} = \sigma^2 \mathbf{I}_p\) for some unknown \(\sigma^2\), the corresponding generalized likelihood ratio test, under an alternative that assumes \(\mathbf{\Sigma}\) to be a rank-one perturbation of \(\sigma^2 \mathbf{I}_p\), rejects for large values of \(\widehat{\ell}_1 / \big(\sum_{j=2}^p \widehat{\ell}_j\big)\) (\citet{johnson1972analysis}; \citet{nadler2008finite}). A \textit{factor analysis problem} can be seen as a generalization of PCA in that it assumes a certain signal-plus-noise decomposition of the observation vector \(\mathbf{X}\): \begin{equation} \mathbf{X} - \boldsymbol{\mu} = \mathbf{L} \mathbf{f} + \boldsymbol{\varepsilon}, \end{equation} where \(\mathbf{f}\) is an \(m \times 1\) dimensional random vector, \(\mathbf{L}\) is a \(p \times m\) dimensional nonrandom matrix, \(\mathbf{f}\) and \(\boldsymbol{\varepsilon}\) are uncorrelated, and \(\boldsymbol{\varepsilon}\) has mean \(0\) and variance \(\mathbf{\Psi}\), a \(p \times p\) diagonal matrix. For identifiability, it is typically assumed that \(\mathbb{E}[\mathbf{f}] = 0\) and \(\mathbb{E}[\mathbf{f} \mathbf{f}^T] = \mathbf{I}_m\). Under this setting, the covariance matrix of \(\mathbf{X}\) is of the form \(\mathbf{\Sigma} = \mathbf{L} \mathbf{L}^T + \mathbf{\Psi}\). Thus, if \(\mathbf{\Psi}\) is a multiple of the identity, the problem of estimating \(\mathbf{L}\) from data can be formulated in terms of a PCA of the sample covariance matrix. One important distinction between PCA and factor analysis is that, in the latter case, the practitioner implicitly assumes a causal model for the data. In general, factor analysis problems are often solved through a maximum likelihood approach (see \citet{tipping1999probabilistic}). A more enhanced version of the factor analysis model, the so-called dynamic factor model, is used extensively in econometrics, where the factors \(\mathbf{f}\) are taken to be time-dependent (\citet{forni2000tthe}). A detailed discussion of various versions of the double Wishart eigenproblem, including a summary of the associated distribution theory when the observations are Gaussian, can be found in \citet{johnstone2009sparse}. We first consider the canonical correlation analysis (CCA) problem within this framework. Again, first, we deal with the formulation at the population level. Suppose that real-valued random vectors $\mathbf{X}$ and $\mathbf{Y}$ are jointly observed, where $\mathbf{X}$ is of dimension $p$ and $\mathbf{Y}$ is of dimension $q$. Then a generalization of the notion of correlation between $\mathbf{X}$ and $\mathbf{Y}$ is expressed in terms of the sequence of canonical correlation coefficients defined as \begin{equation} \label{eq: rho} \rho_k = \max_{(\mathbf{u}, \mathbf{v}) \in S_k} |\mathrm{Cor}(\mathbf{u}^\top \mathbf{X}, \mathbf{v}^\top \mathbf{Y})|, \quad k = 1, 2, \dots, \min\{p, q\}, \end{equation} where \[ S_k := \{ (\mathbf{u}, \mathbf{v}) \in \mathbb{R}^{p+q} : \mathbf{u}^\top \mathbf{\Sigma}_{\mathbf{XX}} \mathbf{u} = \mathbf{v}^\top \mathbf{\Sigma}_{\mathbf{YY}} \mathbf{v} = 1; \, \mathbf{u}^\top \mathbf{\Sigma}_{\mathbf{XX}} \mathbf{u}_j = \mathbf{v}^\top \mathbf{\Sigma}_{\mathbf{YY}} \mathbf{v}_j = 0, \, j = 1, \dots, k-1 \}, \] with $\mathbf{\Sigma}_{\mathbf{XX}} = \mathrm{Var}(\mathbf{X})$, $\mathbf{\Sigma}_{\mathbf{YY}} = \mathrm{Var}(\mathbf{Y})$, and $(\mathbf{u}_k, \mathbf{v}_k)$ denoting the pair of vectors for which the maximum in (\ref{eq: rho}) is attained. If $\mathbf{\Sigma}_{\mathbf{XY}} = \mathrm{Cov}(\mathbf{X}, \mathbf{Y})$, then the optimization problem (\ref{eq: rho}) can be formulated as the following generalized eigenvalue problem: the successive canonical correlations $\rho_1 \geq \cdots \geq \rho_{\min\{p,q\}} \geq 0$ satisfy the generalized eigen-equations \begin{equation} \label{eq: char poly} \det (\mathbf{\Sigma}_{\mathbf{XY}} \mathbf{\Sigma}_{\mathbf{YY}}^{-1} \mathbf{\Sigma}_{\mathbf{YX}} - \rho^2 \mathbf{\Sigma}_{\mathbf{XX}}) = 0. \end{equation} When we have $n$ samples $\{(\mathbf{X}_i, \mathbf{Y}_i) : i = 1, \dots, n\}$ we can replace $\mathbf{\Sigma}_{\mathbf{XX}}, \mathbf{\Sigma}_{\mathbf{XY}}$ and $\mathbf{\Sigma}_{\mathbf{YY}}$ by their sample counterparts and, assuming $n > \max\{p, q\}$, the corresponding sample canonical correlations $r_1 \geq \cdots \geq r_{\min\{p,q\}} \geq 0$ satisfy the sample version of \Cref{eq: char poly}. It is shown in \citet{mardia2024multivariate} that in the latter case, we can reformulate the corresponding generalized eigenanalysis problem as solving \begin{equation} \det (\mathbf{U} - r^2 (\mathbf{U} + \mathbf{V})) = 0, \end{equation} where $\mathbf{U}$ and $\mathbf{V}$ are independent Wishart matrices if $(\mathbf{X}_i, \mathbf{Y}_i)$ are i.i.d. Gaussian and $\mathbf{\Sigma}_{\mathbf{XY}} = \mathbf{0}$, i.e., $\mathbf{X}$ and $\mathbf{Y}$ are independently distributed. Next, we consider the multivariate Linear Regression model, \begin{equation} \label{eq: High dim Lin Reg} \mathbf{Y} = \mathbf{XB} + \mathbf{E} \end{equation} where $\mathbf{Y} = [Y_1: \cdots: Y_m] \in \R^{n \times m}$ is the response matrix consisting $n$ observations for each of the $m$ response variable, $\mathbf{X} \in \R^{n \times p}$ is the design matrix where $p$ is the number of covariates. $\mathbf{E}$ denotes the error matrix. For inference purposes, it is further assumed that $\mathbf{E} \sim NDM(0,\mathbf{\Sigma})$ i.e. rows of $\mathbf{E}$ are iid from $N(0,\mathbf{\Sigma})$. Then, as described in \citet{mardia2024multivariate}, the union-intersection test for the linear hypothesis of the form $H_0 : \mathbf{C}\mathbf{B}\mathbf{D} = \mathbf{0}$ where $\mathbf{C}$ and $\mathbf{D}$ are specified conformable matrices, can be expressed in terms of the largest eigenvalue of $\mathbf{U} (\mathbf{U} + \mathbf{V})^{-1}$ where $\mathbf{U}$ and $\mathbf{V}$ are appropriately specified independent Wishart matrices (under Gaussianity of the entries of $\mathbf{E}$). The two-sample test for equality of variances assumes that we have i.i.d. samples from two normal populations $N_p(\boldsymbol{\mu}_1, \mathbf{\Sigma}_1)$ and $N_p(\boldsymbol{\mu}_2, \mathbf{\Sigma}_2)$ of sizes $n_1$ and $n_2$, say. Then several tests for the hypothesis $H_0 : \mathbf{\Sigma}_1 = \mathbf{\Sigma}_2$ can be formulated in terms of functionals of the eigenvalues of $\mathbf{U} (\mathbf{U} + \mathbf{V})^{-1}$ where $\mathbf{U} = (n_1 - 1)\mathbf{S}_1$ and $\mathbf{V} = (n_2 - 1)\mathbf{S}_2$ are the sample covariances for the two samples, which would follow independent Wishart distributions in $p$ dimensions with d.f. $n_1 - 1$ and $n_2 - 1$ and dispersion matrix $\mathbf{\Sigma}_1 = \mathbf{\Sigma}_2$ under $H_0$. Now it is to be noted that the traditional methods to deal with the above problems assume the dimension of the data to be fixed and relatively small compared to the number of data points. But in the modern era, most of the high-dimensional data arising in fields such as genomics, economics, atmospheric science, chemometrics, and astronomy, to name a few, are of enormously large dimensions which makes it very challenging to apply the traditional methods directly to those datasets. And so, to accommodate the analysis of such datasets, it is imperative to either modify or reformulate some of the statistical techniques. This is where RMT has been playing a significant role, especially over the last decade. In the next sections, we develop these modern RMT-based methods with a key focus on their applications in statistics. \section{High Dimensional Random Matrices} \label{sec:RMT} In Random Matrix Theory, two particular kinds of random matrices draw special attention for their remarkable application in statistics - Covariance Matrices and F-type Matrices. In this section, we discuss the theoretical properties of these two kinds of random matrices, which play a key role in most modern developments in high-dimensional statistics. Most of these results focus on the spectrum's behavior when the matrix has a large dimension. Hence to address those properties, we first give a basic introduction to these matrix models and describe a couple of key questions associated with it. In classical random matrix theory, in the context of covariance matrices, one of the most fundamental and crucially studied matrices is the Wishart Matrix. The Wishart matrix is defined as specifying two sequences of integers $n$, the sample size, and $p = p(n)$, the data dimension. Most of the results additionally assume that the sequences are related so that, as $n \to \infty$, $p = p(n) \to \infty$ satisfying, \begin{equation*} \underset{n \to \infty}{\text{lim}} \frac{p}{n} = \gamma \in (0,\infty) \end{equation*} So if, $X_1,\cdots,X_n \overset{iid}{\sim} N_p(\mathbf{0},\mathbf{\Sigma})$, and $\mathbf{X} = [X_1:\cdots:X_n]$, then the distribution of $\mathbf{X}\mathbf{X^T}$ is called Wishart distribution with parameter $\mathbf{\Sigma}$, degree of freedom $n$ and dimension $p$ and abbreviated as $W_p(\mathbf{\Sigma},m)$. A density of the distribution is given by \[ f_W(\mathbf{X}) = \frac{|\mathbf{X}|^{(n-p-1)/2} e^{-\frac{1}{2} \text{tr}(\Sigma^{-1} \mathbf{X})}}{2^{np/2} |\Sigma|^{n/2} \Gamma_p\left(\frac{n}{2}\right)} \] where $\Gamma_p(\cdot)$ is the multivariate gamma function. The distribution was first studied by \citet{wishart1928generalised} and continues to be a fundamental focus in multivariate statistics thereafter. The central reason for Wishart matrices being so frequent and useful in statistics is their association with sample covariance matrices - the sample covariance matrix of a normal random sample follows a Wishart distribution i.e. If the data $X_1,\cdots,X_n \overset{iid}{\sim} N_p(\mathbf{0},\mathbf{\Sigma})$, and $\mathbf{S} = \frac{1}{n}\sum_{i=1}^n (X_i - \overline{X})(X_i - \overline{X})^T$ is the sample covariance matrix, then $n\mathbf{S} \sim W_p(\mathbf{\Sigma},n-1)$. Further detailed properties of the distribution can be found in \citet{muirhead2009aspects}, \citet{mardia2024multivariate}. From the definition, it can be seen that the Wishart distribution is the analog of the chi-square distribution of the univariate case. Similarly, the univariate $F-$distribution has also an extension for matrices, which is called Matrix $F-$distribution. If $\mathbf{A} \sim W_p(I_p,\nu)$ and $\mathbf{B}\sim W_p(I_p,\delta)$ and $\mathbf{A,B}$ are independent then the distribution of $\mathbf{B}^{-1/2}\mathbf{A}\mathbf{B}^{-1/2}$ is called a matrix $\mathbf{F}(I_p,\nu,\delta)$ distribution. The distribution was originally derived by \citet{olkin1964multivariate}. \citet{perlman1977note} discusses several interesting properties of this distribution. Usually if $\mathbf{A,B}$ are independent random matrices such that $\mathbf{A} \sim W_p(\mathbf{\Sigma},m)$ and $\mathbf{B} \sim W_p(\mathbf{\Sigma},n)$ and $\Sigma$ is positive definite and $m \geqslant p$, then $\mathbf{A}^{-1}\mathbf{B}$ is called an $F-$type matrix in the literature. For their widespread applications such as in Linear Discriminant Analysis, Canonical Correlation Analysis, etc, $F-$type matrices are also extensively studied. However, the properties of these matrices, which played a crucial role in statistical inference and related fields over decades, have been studied beyond the parametric framework, under minimalistic assumptions mostly for the high-dimensional setup. We discuss the properties of those matrices in terms of their spectrum in both parametric and nonparametric frameworks. \subsection{Spectral Properties of large Sample Covariance Matrices} \label{subsec:cov matrices} The sample covariance matrix is one of the most important random matrices in multivariate statistical inference. It is fundamental in hypothesis testing, principal component analysis, factor analysis, and discrimination analysis. Many test statistics are defined by their eigenvalues. However, for large matrices, it is more convenient to study the asymptotic behavior of their spectrum as they exhibit nice properties. \subsubsection{Properties of the whole Spectrum} \label{subsubsec: spectrum} Suppose $\mathbf{X}$ is a $n \times n$ random matrix having eignevalues $\lambda_1,\cdots,\lambda_n \in \mathbb{C}$. Then the empirical distribution of the eigenvalues of $\mathbf{X}$ is called the empirical spectral distribution(ESD) of $\mathbf{X}$. If the matrix $\mathbf{X}$ is real, symmetric then all of its eigenvalues are real and hence the empirical spectral distribution is given by $\hat{F}(x) = \frac{1}{n} \sum_{i=1}^n \mathbf{1}(\lambda_i \leqslant x)$ which is the case for Wishart matrices. In random matrix theory, the ESD is crucial to study as most of the properties of a matrix can be reformulated in terms of its eigenvalues and hence is of key interest. In different domains like machine learning, signal processing, and wireless communications, several functions of the eigenvalues give important objects to study (eg. \citet{tulino2004random}, \citet{couillet2022random} etc). In the parametric setup, under the normality assumption of the data, when we have the exact distribution of the sample covariance matrix, one of the most fundamental questions one can ask is how to characterize the joint distribution of the spectrum. If $\mathbf{X} \sim W_p(\mathbf{\Sigma},n)$, then the rank of $\mathbf{X}$ is min$\{p,n\}$ almost surely. Now for $n \leqslant p-1$, the rank of $\mathbf{X} \leqslant p - 1$ and hence the eigenvalues do not have a joint pdf. However, for $n > p-1$, the joint probability density function (pdf) for the eigenvalues of $\mathbf{X}$ exists and can be found in \citet{muirhead2009aspects} and a detailed study on their joint distribution can be found in \citet{james2014concise}. Furthermore, for $n > p-1$, the following central limit theorem holds for log-transformed eigenvalues of $\mathbf{X}$. \begin{theorem} \label{th:th1} Let $\mathbf{X}_n \sim W_p(\mathbf{\Sigma},n)$ where $n \geqslant p$ and $\mathbf{\Sigma}$ is positive definite. Let $\lambda_1^{(n)},\cdots,\lambda_p^{(n)}$ be the eigenvalues of $\mathbf{X}_n$ and $\lambda_1,\cdots,\lambda_p$ be the eigenvalues of $\mathbf{\Sigma}$. Then, \begin{equation*} \underset{x \in \mathbb{R}}{\text{sup}} \hspace{0.1cm} \left| P \left( \sqrt{\frac{n}{2p}} \left( \sum_{i=1}^p \text{log}\left( \frac{\lambda_i^{(n)}}{\lambda_i}\right) - \sum_{i=1}^p\text{log} \hspace{0.1cm} (n-p+i) \right) \leqslant x \right) - \Phi(x) \right| = O\left(\frac{p}{\sqrt{n}} \right) \end{equation*} where $\Phi(x)$ denotes the Standard Normal CDF. \end{theorem} The proof of \Cref{th:th1} can be found in the appendix. Hence, for $\frac{p}{\sqrt{n}} \to 0$ as $n \to \infty$, we have \begin{equation*} \sqrt{\frac{n}{2p}} \left( \sum_{i=1}^p \text{log}\left( \frac{\lambda_i^{(n)}}{\lambda_i}\right) - \sum_{i=1}^p\text{log} \hspace{0.1cm} (n-p+i) \right) \convD N(0,1) \end{equation*} It is to be noted that the above central limit theorem is very useful for one-sample and two-sample testing for covariance matrices, to approximate the power function of such tests, and also for inference on covariance matrices in high dimensional linear models. A detailed discussion of these applications can be found in \Cref{sebsec: infcovmat}. The theorem also provides a rate of convergence of the eigenvalues of the sample covariance matrix to the population covariance matrix. Often for approximation purposes, functions of the spectrum of the population covariance matrices are estimated using that of the corresponding sample covariance matrix. In such cases, the theorem provides an upper bound of the error. Hence, it is useful for sample size determination if there is a predetermined allowable upper bound on the error. In this context, a natural follow-up question is whether the weak limit of the ESD for Wishart matrices exists. The celebrated \textit{Marcenko-Pastur Law} answers this question in the context of sample covariance matrices. With an assumption of the finiteness of the fourth moment of the entries of the data matrix, \citet{marchenko1967distribution} showed that depending on the value of $\gamma = \underset{n \to \infty}{\text{lim}} \frac{p}{n}$, the weak limit of the ESD of sample covariance matrices exist. \begin{theorem}[Marcenko-Pastur Law] \label{th: th2} Suppose that \( \mathbf{X} \) is a \( p \times n \) matrix with i.i.d. real- or complex-valued entries with mean 0 and variance 1. Suppose $\underset{n \to \infty}{\text{lim}} \frac{p}{n} = \gamma \in (0,\infty) $ . Then, as \( n \to \infty \), the empirical spectral distribution (ESD) of \( \mathbf{S} = \frac{1}{n}\mathbf{XX}^T \) converges almost surely in distribution to a nonrandom distribution, known as the Marcenko–Pastur law and denoted by \( F_\gamma \). If \( \gamma \in (0, 1] \), then \( F_\gamma \) has the p.d.f.: \begin{equation} f_\gamma(x) = \frac{\sqrt{(b_+(\gamma) - x)(x - b_-(\gamma))}}{2\pi \gamma x}, \quad b_-(\gamma) \leq x \leq b_+(\gamma), \end{equation} where \[ b_\pm(\gamma) = \left(1 \pm \sqrt{\gamma}\right)^2. \] For \( x \) outside this interval, \( f_\gamma(x) = 0 \). If \( \gamma \in (1, \infty) \), then \( F_\gamma \) is a mixture of a point mass at 0 and the p.d.f. \( f_{1/\gamma}(x) \), with weights \( 1 - 1/\gamma \) and \( 1/\gamma \), respectively. \end{theorem} \begin{figure} \centering \includegraphics[width=0.5\linewidth]{marcenko_pastur_density.png} \caption{Marcenko–Pastur density functions for $\gamma = 0.1,0.25,0.5,1$} \label{fig:marcenko pastur law} \end{figure} It is to be noted that the above result is distribution-free, in the sense the limiting distribution only depends on the limiting ratio of data dimension and sample size ($\gamma$) and is free of the data distribution. As $\gamma$ increases from $0$ to $1$, the spread of the eigenvalues also increases. However, a necessary condition for the weak limit of the ESD to exist is to $\gamma > 0$. For $\gamma = 0$, as illustrated in \Cref{fig:marcenko pastur law}, the maximum and minimum eigenvalues converge to $1$ and hence Marcenko-Pastur law does not hold for this case. However, with an assumption of the finiteness of the fourth moment of the entries of $\mathbf{X}$, applying a suitable centering and scaling to the matrix $\mathbf{S}$, \citet{bai1988convergence} derived the weak limit of the ESD of the transformed matrix when $\frac{p}{n} \to 0$. \begin{theorem}[\citet{bai1988convergence}] \label{th: th3} Suppose that \( \mathbf{X} \) is a \( p \times n \) matrix with i.i.d. real-valued entries with mean 0 and variance 1 with finite fourth moment. Suppose $\underset{n \to \infty}{\text{lim}} \frac{p}{n} = 0 $ . Then, as \( p \to \infty \), the empirical spectral distribution (ESD) of \( \mathbf{S}_p = \frac{1}{2\sqrt{np}} \left(\mathbf{XX}^T - n I_p \right) \) converges almost surely in distribution to a nonrandom distribution, known as semi-circular distribution having the pdf \begin{equation} f(x) = \frac{1}{2\pi} \sqrt{4 - x^2}, \quad -2 \leq x \leq 2 \end{equation} \end{theorem} Like the Marcenko-Pastur law, the above theorem is also distribution-free and provides the rate at which the eigenvalues of $\mathbf{S} = \frac{1}{n}\mathbf{XX}^T$ goes to 1 when $\frac{p}{n} \to 0$. However, one potential disadvantage of the above two theorems is the requirement of the i.i.d data. In both of the theorem, in the data matrix $\mathbf{X} = [X_1,\cdots,X_n]$, where each of the columns ($X_i$) represents a data point of dimension $p$, though each of the columns can be assumed to be independent, in practice it is very unlikely that the entries of a single data point in $\mathbb{R}^p$ will be mutually independent as well. Henceforth substantial progress has been made to generalize these results by relaxing the conditions of independence within the columns. If $Y_1,\cdots,Y_n$ are i.i.d. $p-$dimensional data points with covariance matrix $\mathbf{\Sigma}$, then $\mathbf{Y} = [Y_1,\cdots,Y_n] = \mathbf{\Sigma}^{\frac{1}{2}} \mathbf{X}$, where $\mathbf{X}$ satisfies the conditions of \Cref{th: th2} and \ref{th: th3}. In this context, \citet{silverstein1995strong} develops the Marcenko-Pastur law for the ESD of $\frac{1}{n} \mathbf{\Sigma}^{\frac{1}{2}}\mathbf{X}\mathbf{X}^T\mathbf{\Sigma}^{\frac{1}{2}}$ when $\frac{p}{n} \to \gamma \in (0,\infty)$ under the same conditions as of \Cref{th: th2}. For $\frac{p}{n} \to 0$, under the conditions of \Cref{th: th3}, \citet{bao2012strong} showed the ESD of $\sqrt{\frac{n}{p}} \left( \frac{1}{n} \mathbf{\Sigma}^{\frac{1}{2}}\mathbf{X}\mathbf{X}^T\mathbf{\Sigma}^{\frac{1}{2}} - \mathbf{\Sigma} \right) = \sqrt{\frac{n}{p}} \mathbf{\Sigma}^{\frac{1}{2}} \left( \frac{1}{n} \mathbf{X}\mathbf{X}^T - \mathbf{I} \right) \mathbf{\Sigma}^{\frac{1}{2}}$ converges almost surely in distribution to a nonrandom distribution. Further research has been done to develop similar results under different forms of dependence. For instance, \citet{yin1987limit} derived similar results when $X_1,\cdots,X_n$ are i.i.d from a spherically symmetric distribution. \citet{hui2010limiting}, \citet{wei2016limiting} considered the case when the data points come from a $m-$dependent process. \citet{hofmann2008wigner} and \citet{friesen2013gaussian} assumed that the entries of the data matrix $\mathbf{X} = [X_1,\cdots,X_n]$ can be partitioned into independent subsets while allowing the entries from the same subset to be dependent. \citet{gotze2006limit} replaced the independent assumption by a technical martingale-type condition. \citet{yao2012note} develops a version of the Marcenko-Pastur law when $X_1,\cdots,X_n$ are independent and entries of each $X_i$ comes from a linear time series process. \subsubsection{Properties of extreme eigenvalues} \label{subsubsec: extreme eval} In the previous section, we have a detailed description of the limit of the ESD of random matrices. However, in many situations, it is important to know whether the sample eigenvalues of $\mathbf{S}$ (as in \Cref{th: th2}) lie inside the support of $F_\gamma$ as well. For instance, in signal processing, pattern recognition, edge detection, and many other areas, the support of the LSD of the population covariance matrices consists of several disjoint pieces. So it is essential to know whether or not the LSD of the sample covariance matrices is also separated into the same number of disjoint pieces, and under what conditions this is true. Also, many statistics can be written as a function of the integrals of the ESD of the random matrix. For example, the determinant of the sample covariance matrix is very useful in wireless communication and signal processing (\citet{paul2014random}) which can be written as \begin{equation} \label{eq:det} \det(\mathbf{A}) = \prod_{j=1}^{n} \lambda_j = \exp \left( n \int_0^{\infty} \log x \, F^{\mathbf{A}}(dx) \right) \end{equation} So under the knowledge of the asymptotic distribution of the ESD, usually the Helly-Bray theorem (\citet{billingsley2013convergence}) is used to obtain an approximation of the statistic. But often such functions are not bounded (e.g. the function in \ref{eq:det} is $log x$ which is unbounded. As a result, the LSD and Helly-Bray theorem cannot be used to approximate the statistics. This limitation reduces the usefulness of the LSD. However, in many cases, the supports of the LSDs are compact intervals. Still, this alone does not guarantee that the Helly-Bray theorem can be applied unless one also proves in addition that the extreme eigenvalues of the random matrix stay within certain bounded intervals. These examples demonstrate that knowledge about the weak limit of the ESD is not sufficient. Furthermore, extreme eigenvalues of random matrices themselves occur naturally in many problems such as principal component analysis. Henceforth studies regarding the asymptotic properties of the extreme eigenvalues of random matrices are extremely important. Under the assumption of the finiteness of the fourth moment of the i.i.d. entries, \citet{yin1984limit} proved that the maximum eigenvalue of $\mathbf{S}$ (as in \Cref{th: th2}) converges almost surely. \begin{theorem}[\citet{yin1984limit}] \label{th:th 4} Suppose that \( \mathbf{X} \) is a \( p \times n \) matrix with i.i.d. real-valued entries with mean 0 and variance $\sigma^2$ and finite fourth moment. Suppose $\underset{n \to \infty}{\text{lim}} \frac{p}{n} = \gamma \in (0,\infty) $ . Suppose $\lambda_{\text{max}}(n)$ is the maximum eigenvalue of the $p \times p$ random matrix \( \mathbf{S} = \frac{1}{n}\mathbf{XX}^T \). Then \begin{equation} \underset{n\to \infty}{\text{lim}} \lambda_{\text{max}}(n) = (1 + \sqrt{\gamma})^2 \sigma^2 \hspace{0.3cm} \text{a.s.} \end{equation} \end{theorem} A similar result was developed in \citet{bai2008limit} for the smallest eigenvalue of $\mathbf{S}$ as well under the same set of assumptions as of \Cref{th:th 4} as well when $p < n$. \begin{theorem}[\citet{bai2008limit}] \label{th:th 4} Suppose that \( \mathbf{X} \) is a \( p \times n \) matrix with i.i.d. real-valued entries with mean 0 and variance $\sigma^2$ and finite fourth moment. Suppose $\underset{n \to \infty}{\text{lim}} \frac{p}{n} = \gamma \in (0,1) $ . Suppose $\lambda_{\text{min}}(n)$ is the smallest eigenvalue of the $p \times p$ random matrix \( \mathbf{S} = \frac{1}{n}\mathbf{XX}^T \). Then \begin{equation} \underset{n\to \infty}{\text{lim}} \lambda_{\text{min}}(n) = (1 - \sqrt{\gamma})^2 \sigma^2 \hspace{0.3cm} \text{a.s.} \end{equation} \end{theorem} These results give an accurate idea of the asymptotic range of the eigenvalues of sample covariance matrices under very mild assumptions. However many classical tests in multivariate analysis consist of the largest eigenvalues of sample covariance matrices (eg. Roy's largest root test) which makes the asymptotic distributions of the maximum eigenvalue of special interest. In the celebrated paper \citet{johnstone2001distribution}, the limiting distribution of the largest eigenvalue of the sample covariance matrix was derived when the entries of the data matrix are i.i.d from the standard normal distribution. Suppose that \( \mathbf{X} = ((X_{ij})) \) is an \( p \times n \) matrix with entries are i.i.d. from standard normal distribution, \[ X_{ij} \sim N(0, 1). \] Let $l_1$ be the largest sample eigenvalue of the Wishart matrix \( \mathbf{X}\mathbf{X}^T \). Define the centering and scaling constants as follows: \begin{equation} \label{eq:mu} \mu_{n,p} = \left( \sqrt{n-1} + \sqrt{p} \right)^2 \end{equation} \begin{equation} \label{eq:sigma} \sigma_{n,p} = \left( \sqrt{n-1} + \sqrt{p} \right) \left( \frac{1}{\sqrt{n-1}} + \frac{1}{\sqrt{p}} \right)^{1/3} \end{equation} The Tracy-Widom law of order 1 has the distribution function defined by: \begin{equation} \label{eq:TW cdf} F_1(s) = \exp \left( -\frac{1}{2} \int_s^\infty \left[ q(x) + (x - s) q^2(x) \right] dx \right) , \hspace{0.2cm} s \in \mathbb{R} \end{equation} where \( q(x) \) solves the nonlinear Painlevé II differential equation: \begin{equation} q''(x) = x q(x) + 2 q^3(x) \end{equation} with the asymptotic condition: \begin{equation} q(x) \sim \text{Ai}(x) \quad \text{as} \quad x \to +\infty, \end{equation} where \( \text{Ai}(x) \) denotes the Airy function. This distribution was found by \citet{tracy1996orthogonal} as the limiting law of the largest eigenvalue of an \( n \times n \) Gaussian symmetric matrix. In terms of these distributions, the asymptotic distribution of $l_1$ can be stated as follows, \begin{theorem}[\citet{johnstone2001distribution}] \label{th: tracy widom} Suppose that \( \mathbf{X} = ((X_{ij})) \) is an \( p \times n \) matrix whose entries are i.i.d. from standard normal distribution i.e. $X_{ij} \overset{i.i.d.}{\sim} N(0, 1)$. If $\frac{p}{n} \to \gamma \in (0,\infty)$, and $l_1$ denotes the highest eigenvalue of $\mathbf{X}\mathbf{X}^T$ then, \begin{equation} \frac{l_1 - \mu_{n,p}}{\sigma_{n,p}} \convD W_1 \sim F_1 \end{equation} where $\mu_{n,p},\sigma_{n,p}$ are as in \ref{eq:mu} and \ref{eq:sigma} respectively and $W_1$ is a random variable following Tracy-Widom distribution defined in \ref{eq:TW cdf}. \end{theorem} In \citet{karoui2003largest}, the same result was extended for the cases $\gamma=0,\infty$ as well. \begin{figure} \centering \includegraphics[width=0.6\linewidth]{image.png} \caption{Density function for the Tracy-Widom Distribution} \label{fig:enter-label} \end{figure} These results are very useful in single Wishart (e.g. principal component analysis (PCA), factor analysis, and tests for population covariance matrices in one-sample problems) and double Wishart problems (e.g. multivariate analysis of variance (MANOVA), canonical correlation analysis (CCA), tests for equality of covariance matrices and tests for linear hypotheses in multivariate linear regression problems). Many asymptotic generalizations of classical tests (e.g. Roy's largest root test) have been obtained using the above results which have a wide range of applications in signal processing, wireless communication (\citet{paul2014random}), and machine learning (\citet{couillet2022random}). \Cref{sec:application} has a detailed discussion on these applications. To check the practical applicability of \Cref{th: tracy widom}, for the purpose of approximation, a simulation study was done (\citet{johnstone2001distribution}). First, for square cases $n = p = 5, 10$ and 100, using $R = 10,000$ replications, results are shown in \Cref{table: table tracy widom}. Even for $5 \times 5$ and $10 \times 10$, the approximation seems to be quite good in the right-hand tail at conventional significance levels of 10\%, 5\%, and 1\%. At $100 \times 100$, the approximation seems reasonable throughout the range. The same general picture holds for $n/p$ in the ratio 4:1. Even for $5 \times 20$ matrices, the approximation is reasonable, if not excellent, at the conventional upper significance levels. A further summary message from these computations is that in the null Wishart case, about 80\% of the distribution lies below $\mu_{np}$ and 95\% below $\mu_{np}+ \sigma_{np}$. \citet{ma2012accuracy} has a detailed discussion on the accuracy of the Tracy-Widom laws. \begin{table}[ht] \centering \caption{(\citet{johnstone2001distribution}) Simulations for finite \( n \times p \) versus Tracy–Widom Limit. The first column shows the probabilities of the \( F_1 \) limit distribution corresponding to fractions in the second column. The next three columns show estimated cumulative probabilities for \( l_1 \), centered and scaled as in \Cref{eq:mu} and \ref{eq:sigma}, in \( R = 10,000 \) repeated draws from \( W_p(n, I) \) with \( n = p = 5, 10, 100 \). The following three cases have \( n:p \) in the ratio 4:1. The final column gives approximate standard errors based on binomial sampling. The bold font highlights some conventional significance levels. The Tracy–Widom distribution \( F_1 \) was evaluated on a grid of 121 points \( -6(0.1)6 \) using the Mathematica package \texttt{p2Num} written by Craig Tracy. The remaining computations were done in MATLAB, with percentiles obtained by inverse interpolation and using \texttt{randn()} for normal variates and \texttt{norm()} to evaluate the largest singular values.} \label{table: table tracy widom} \hspace{0.2cm} \begin{tabular}{cccccccccc} \toprule Percentile & TW & 5 $\times$ 5 & 10 $\times$ 10 & 100 $\times$ 100 & 5 $\times$ 20 & 10 $\times$ 40 & 100 $\times$ 400 & 2 $\times$ SE \\ \midrule -3.90 & 0.01 & 0.000 & 0.001 & 0.007 & 0.002 & 0.003 & 0.010 & (0.002) \\ -3.18 & 0.05 & 0.003 & 0.015 & 0.042 & 0.029 & 0.039 & 0.049 & (0.004) \\ -2.78 & 0.10 & 0.019 & 0.049 & 0.089 & 0.075 & 0.089 & 0.102 & (0.006) \\ -1.91 & 0.30 & 0.211 & 0.251 & 0.299 & 0.304 & 0.307 & 0.303 & (0.009) \\ -1.27 & 0.50 & 0.458 & 0.480 & 0.500 & 0.539 & 0.524 & 0.508 & (0.010) \\ -0.59 & 0.70 & 0.697 & 0.707 & 0.703 & 0.739 & 0.733 & 0.714 & (0.009) \\ 0.45 & 0.90 & 0.901 & 0.907 & 0.903 & 0.919 & 0.918 & 0.908 & (0.006) \\ 0.98 & 0.95 & 0.948 & 0.954 & 0.950 & 0.960 & 0.961 & 0.957 & (0.004) \\ 2.02 & 0.99 & 0.988 & 0.991 & 0.991 & 0.992 & 0.993 & 0.992 & (0.002) \\ \bottomrule \end{tabular} \end{table} \subsection{Asymptotic Properties of $F-$type matrices} \label{subsec: Fmatrices} In this section, we discuss the asymptotic properties of a multivariate $F-$ matrix. Multivariate F-distribution plays a crucial role in several areas of multivariate data analysis, especially when the relationships between multiple variables are tested simultaneously. It has primary application in two-sample tests on covariance matrices, MANOVA (multivariate analysis of variance), multivariate linear regression, and in Canonical Correlation Analysis. Pioneering work by \citet{wachter1980limiting} examined the limiting distribution of the solutions to the equation: \[ \det\left( \mathbf{X}_{1,n_1} \mathbf{X}_{1,n_1}^T - \lambda \mathbf{X}_{2,n_2} \mathbf{X}_{2,n_2}^T \right) = 0, \] where \( \mathbf{X}_{j,n_j} \) is a \( p \times n_j \) matrix with i.i.d. entries from \( N(0,1) \), and \( \mathbf{X}_{1,n_1} \) is independent of \( \mathbf{X}_{2,n_2} \). When \( \mathbf{X}_{2,n_2} \mathbf{X}_{2,n_2}^T \) is of full rank, the solutions to this equation are \( \frac{n_2}{n_1} \) times the eigenvalues of the multivariate F-matrix: \[ \left( \frac{1}{n_1} \mathbf{X}_{1,n_1} \mathbf{X}_{1,n_1}^T \right) \left( \frac{1}{n_2} \mathbf{X}_{2,n_2} \mathbf{X}_{2,n_2}^T \right)^{-1}. \] \citet{yin1983limit} proved the existence of the limiting spectral distribution (LSD) of the matrix sequence \( \{ \mathbf{S}_n \mathbf{T}_n \} \), where \( \mathbf{S}_n \) is a standard Wishart matrix of dimension \( p \) with \( n \) degrees of freedom, and \( \frac{p}{n} \to \gamma \in (0, \infty) \), \( \mathbf{T}_n \) is a positive definite matrix with \( \beta_k(\mathbf{T}_n) \to \mathbf{H}_k \), and the sequence \( \mathbf{H}_k \) satisfies the Carleman condition. In \citet{yin1986limiting}, this result was extended to the case where the sample covariance matrix is formed from i.i.d. real random variables with mean zero and variance one. Building on the work of \citet{yin1983limit}, later \citet{yin1983limiting} demonstrated the existence of the LSD of the multivariate F-matrix. The explicit form of the LSD for multivariate F-matrices was derived by \citet{bai1988limiting} and \citet{silverstein1995strong} and is given by the following theorem. \begin{theorem}[\citet{bai1988limiting}] \label{th:th7} Let \( \mathbf{F} = \mathbf{S}_{n_1} \mathbf{S}_{n_2}^{-1} \), where \( \mathbf{S}_{n_i} \) (for \( i = 1,2 \)) is a sample covariance matrix with dimension \( p \) and sample size \( n_i \), and the underlying distribution has mean 0 and variance 1. If \( \mathbf{S}_{n_1} \) and \( \mathbf{S}_{n_2} \) are independent, \( \frac{p}{n_1} \to \gamma \in (0,\infty) \), and \( \frac{p}{n_2} \to \gamma' \in (0,1) \), then the limiting spectral distribution (LSD) \( F_{\gamma,\gamma'} \) of \( \mathbf{F} \) exists and has a density function given by: \[ f_{\gamma,\gamma'}(x) = \begin{cases} \frac{(1 - \gamma')\sqrt{(b - x)(x - a)}}{2 \pi x (\gamma + x\gamma')}, & \text{if } a < x < b, \\ 0, & \text{otherwise}, \end{cases} \] where \[ a = \left( \frac{1 - \sqrt{\gamma + \gamma' - \gamma\gamma'}}{1 - \gamma'} \right)^2, b = \left( \frac{1 + \sqrt{\gamma + \gamma' - \gamma\gamma'}}{1 - \gamma'} \right)^2. \] Further, if \( \gamma > 1 \), then \( F_{\gamma,\gamma'} \) has a point mass \( 1 - \frac{1}{\gamma} \) at the origin. \end{theorem} Besides the entire ESD, the extreme eigenvalues of multivariate $F-$matrices are also immensely important in many high-dimensional problems such as testing sphericality in covariance matrices, testing equality of multiple covariance matrices, correlated noise detection, etc (\citet{han2016tracy}). The following result from the phenomenal work of \citet{han2016tracy} obtains the limiting distribution of the generalized $F-$type matrices under mild assumptions. Before starting the actual theorem, we first state a condition the data matrix needs to satisfy. \begin{definition} \label{def:def 1} A real random matrix $\mathbf{Z}$ is said to satisfy \textbf{Condition 1}, if it consists of entries \( \{ Z_{ij} \} \) where \( \{ Z_{ij} \} \) are independent random variables with \( \mathbb{E}[Z_{ij}] = 0 \) and \( \mathbb{E}[|Z_{ij}|^2] = 1 \) and for all \( k \in \mathbb{N} \), there exists a constant \( C_k \) such that \( \mathbb{E}[|Z_{ij}|^k] \leq C_k \). \end{definition} In conjunction with the above definition, the following theorem presents the desired limiting distribution. \begin{theorem}[\citet{han2016tracy}] \label{th: tw for Fmat} Also assume that the real random matrices \( \mathbf{X} = (X_{ij})_{p \times n} \) and \( \mathbf{Y} = (Y_{ij})_{p \times m} \) are independent and satisfies \textbf{Condition 1}. Set \( m = m(p) \) and \( n = n(p) \). Suppose that \[ \lim_{p \to \infty} \frac{p}{m} = d_1 > 0, \quad \lim_{p \to \infty} \frac{p}{n} = d_2 > 0, \quad 0 < \lim_{p \to \infty} \frac{p}{m+n} < 1. \] satisfies $0 < d_1 < 1$ and, $0 < d_2 < \infty$. Let, \[ \breve{m} = \max\{m, p\}, \quad \breve{n} = \min\{n, m+n-p\}, \quad \breve{p} = \min\{m, p\}. \] Moreover, let \[ \sin^2\left(\frac{\gamma}{2}\right) = \frac{\min\{\breve{p}, \breve{n}\} - \frac{1}{2}}{\breve{m} + \breve{n} - 1}, \quad \sin^2\left(\frac{\psi}{2}\right) = \frac{\max\{\breve{p}, \breve{n}\} - \frac{1}{2}}{\breve{m} + \breve{n} - 1}, \] \begin{equation} \mu_{J,p} = \tan^2 \left(\frac{\gamma + \psi}{2}\right), \end{equation} \begin{equation} \sigma^3_{J,p} = \frac{16 \mu^3_{J,p}}{ (\breve{m} + \breve{n} - 1)^2} \cdot \frac{1}{\sin(\gamma)\sin(\psi)\sin^2(\gamma + \psi)}. \end{equation} Set \[ \mathbf{B}_p = \frac{\mathbf{X}\mathbf{X}^T }{\breve{n}} \quad \text{and} \quad \mathbf{A}_p = \frac{\mathbf{Y}\mathbf{Y}^T }{\breve{m}} . \] Denote the largest root of \[ \det(\lambda \mathbf{A}_p - \mathbf{B}_p) = 0 \] by $\lambda_1$. Then \[ \lim_{p \to \infty} P\left( \frac{\frac{\breve{n}}{\breve{m}} \lambda_1 - \mu_{J,p}}{\sigma_{J,p}} \leqslant s \right) = F_1(s) \] where $F_1(s)$ is the cumulative distribution function of the Tracy-Widom distribution defined in \Cref{eq:TW cdf}. \end{theorem} The above theorem has a couple of interesting remarks. For instance, this immediately implies the distribution of the largest root of $\det(\lambda(\mathbf{B}_p + \mathbf{A}_p) - \mathbf{B}_p) = 0$. In fact, the largest root of $\det(\lambda(\mathbf{B}_p + \mathbf{A}_p) - \mathbf{B}_p) = 0$ is $\frac{\lambda_1}{1 + \lambda_1}$ if $\lambda_1$ is the largest root of the F matrices $\mathbf{B}_p \mathbf{A}_p^{-1}$ in \Cref{th: tw for Fmat} when $0 < d_1 < 1$. When $d_1 > 1$, the largest root of $\det(\lambda(\mathbf{B}_p + \mathbf{A}_p) - \mathbf{B}_p) = 0$ is one with multiplicity $(p - m)$. In that case, instead one considers the $(p - m + 1)$th largest root of $\det(\lambda(\mathbf{B}_p + \mathbf{A}_p) - \mathbf{B}_p) = 0$. It turns out that the $(p - m + 1)$th largest root of $\det(\lambda(\mathbf{B}_p + \mathbf{A}_p) - \mathbf{B}_p) = 0$ is $\frac{\lambda_1}{1 + \lambda_1}$ if $\lambda_1$ is the largest root of $\det(\lambda \mathbf{A}_p - \mathbf{B}_p) = 0$. The exact order of the centering and scaling parameters $\mu_{J,p}$ and $\sigma_{J,p}$ can also be obtained along the lines of this result in terms of $\breve{m},\breve{n}$ and $p$. \Cref{sec:application} has a detailed discussion on the applications of these results on high-dimensional inference. \section{Applications in Statistics} \label{sec:application} In this section, we discuss various applications of random matrix theory in statistics and related fields. So far there are a lot of ground-breaking applications of RMT that helped to develop robust, efficient high dimensional data handling methods, enriched complex machine learning algorithms, optimized signal processing techniques and motivated a lot of crucial discoveries in genomics, finance, climate science, and social network analysis. Henceforth, in this section, the key focus is on these applications to real-world problems in conjunction with the theoretical discussion above. \subsection{Inference on Covariance Matrices} \label{sebsec: infcovmat} One of the primary applications of the theory of large random matrices in high-dimensional statistics is inference on covariance matrices. Since the results provide asymptotic properties of the spectra of large random matrices, using those results one can check whether one estimate is consistent under certain conditions, and also for hypothesis testing, one can approximate the power functions if the test statistic is a function of the spectrum. In this regard, One of the earliest uses of the distribution of the largest eigenvalue of the sample covariance matrix is in testing the hypothesis \( H_0 : \Sigma = \mathbf{I}_p \) when i.i.d. samples are drawn from a \( N(\mu, \Sigma) \) distribution. The Tracy--Widom law for the largest sample eigenvalue under the null Wishart case, i.e., when the population covariance matrix \( \Sigma = \mathbf{I}_p \), allows a precise determination of the cut-off value for this test, which, with a careful calibration of the centering and normalizing sequences, is very accurate even for relatively small \( p \) and \( n \) (\citet{johnstone2001distribution},\citet{johnstone2009sparse}). The behavior of the power of the test requires formulating suitable alternative models. For instance, for data matrix $\mathbf{X} = [X_1,\cdots,X_p] \in \R^{p \times n}$ with iid columns $x_i$, consider the testing problem, \begin{equation} \begin{split} H_0&: \mathbf{X} = \sigma \mathbf{Z} \\ H_1&: \mathbf{X} = a \mathbf{s}^T + \sigma \mathbf{Z} \end{split} \end{equation} where $\mathbf{Z} = [\mathbf{z}_1, \ldots, \mathbf{z}_n] \in \mathbb{R}^{p \times n}$ with $\mathbf{z}_i \sim N(\mathbf{0}, \mathbf{I}_p)$, $\mathbf{a} \in \mathbb{R}^p$ deterministic with unit norm $\|\mathbf{a}\| = 1$, $\mathbf{s} = [s_1, \ldots, s_n]^\top \in \mathbb{R}^n$ with $s_i$ i.i.d. random scalars, and $\sigma > 0$. We also denote $c = p / n$ (and demand as usual that $0 < \lim \inf c \leq \lim \sup c < \infty$). This model describes the observation of either pure Gaussian noise data $\sigma \mathbf{z}_i$ with zero mean and covariance $\sigma^2 \mathbf{I}_p$, or of deterministic information $\mathbf{a}$ possibly modulated by a scalar (random) signal $s_i$ (which could simply be $\pm 1$) added to the noise. If the parameters $\mathbf{a}$, $\sigma$ as well as the statistics of $s_i$ are known, a mere Neyman-Pearson test allows one to discriminate between $H_0$ and $H_1$ with optimal detection probability, for all finite $n, p$; precisely, one will decide on the genuine hypothesis according to the ratio of posterior probabilities \begin{equation} \label{eq: LRT} \frac{\mathbb{P}(\mathbf{X} \mid H_1)}{\mathbb{P}(\mathbf{X} \mid H_0)} \underset{H_0}{\overset{H_1}{\gtrless}} \alpha \end{equation} for some $\alpha > 0$ controlling the desired Type I and Type II error rates (that is, the probability of false positives and of false negatives). However, in practice, unless the existence of a set of previous pure-noise acquisitions is assumed, it is quite unlikely that $\sigma$ be assumed known or consistently estimated. Similarly, if the ultimate objective (post-decision) is to estimate the data structure $\mathbf{a}$ under $H_1$, $\mathbf{a}$ is naturally assumed partially or completely unknown (it may be known to belong to a subset of $\mathbb{R}^p$ in which case more elaborate procedures than proposed here can be carried on). In the most generic scenario where $\mathbf{a}$ is fully unknown, assuming additionally the data of zero mean, we may thus impose without generality the restriction that Under this (very restricted) prior knowledge, instead of the maximum likelihood test in (\ref{eq: LRT}), one may resort to a \textit{generalized likelihood ratio test (GLRT)} defined as \[ \frac{\sup_{\sigma, \mathbf{a}} \mathbb{P}(\mathbf{X} \mid \sigma, \mathbf{a}, \mathcal{H}_1)}{\sup_{\sigma, \mathbf{a}} \mathbb{P}(\mathbf{X} \mid \sigma, \mathbf{a}, \mathcal{H}_0)} \underset{\mathcal{H}_0}{\overset{\mathcal{H}_1}{\gtrless}} \alpha. \] Under both Gaussian noise and signal $s_i$ assumption, the GLRT has an explicit expression that appears to be a monotonously increasing function of $\|\mathbf{X}\mathbf{X}^\top\| / \mathrm{tr}(\mathbf{X}\mathbf{X}^\top)$. That is, the test is equivalent to \[ T_p \equiv \frac{\|\frac{1}{n}\mathbf{X}\mathbf{X}^\top\|}{\frac{1}{p} \mathrm{tr} \left( \frac{1}{n}\mathbf{X}\mathbf{X}^\top \right)} \underset{\mathcal{H}_0}{\overset{\mathcal{H}_1}{\gtrless}} f(\alpha), \] (\citet{wax1985detection} and \citet{anderson1963asymptotic} has a detailed discussion on this idea) for some known monotonously increasing function $f$. Here we introduced the normalizations $1/p$ and $1/n$ so that both the numerator and denominator are of order $O(1)$ as $n, p \to \infty$. Since the ratio $T_p$ has limit $(1+\sqrt{c})^2$ under the $H_0$ asymptotics, $f(\alpha)$ must be of the form $f(\alpha) = (1+\sqrt{c})^2 + g(\alpha)$ for some $g(\alpha) > 0$. Also, as we know that $\frac{1}{p} \mathrm{tr} \left( \frac{1}{n} \mathbf{X}\mathbf{X}^\top \right)$ fluctuates at the speed $O(n^{-1})$, while $\|\frac{1}{n}\mathbf{X}\mathbf{X}^\top\|$ fluctuates at the slower speed $O(n^{-2/3})$ (as per \Cref{th: tracy widom}), the global fluctuation is dominated by the numerator at a rate of order $O(n^{-2/3})$, i.e., we have under $H_0$, \[ T_p \stackrel{H_0}{=} (1 + \sqrt{c})^2 + O(n^{-2/3}). \] Since the denominator essentially converges (at an \( O(n^{-1}) \) rate) while the numerator still fluctuates (at an \( O(n^{-2/3}) \) rate), despite the dependence between both, only the fluctuations of the numerator \( \frac{1}{n} \mathbf{X}\mathbf{X}^\top \) influence the behavior of the ratio \( T_p \), and thus \[ T_p \overset{H_0}{\sim} (1 + \sqrt{c})^2 + (1 + \sqrt{c}) \frac{4}{3} c^{-\frac{1}{6}} n^{-\frac{2}{3}} \text{TW} + o(n^{-2/3}), \] where TW denotes the Tracy-Widom Distribution. As a consequence, in order to set a maximum false alarm rate (or false positive, or Type I error) of \( r > 0 \) in the limit of large \( n, p \), one must choose a threshold \( f(\alpha) \) for \( T_p \) such that \[ \mathbb{P}(T_p \geq f(\alpha)) = r, \] that is, such that \[ \mu_{\text{TW}}([A_p, +\infty)) = r, \quad A_p = (f(\alpha) - (1 + \sqrt{c})^2)(1 + \sqrt{c})^{-\frac{4}{3}} c^{\frac{1}{6}} n^{\frac{2}{3}} \tag{3.2} \] with \( \mu_{\text{TW}} \), the Tracy-Widom measure. For testing problems on covariance matrices, with a both-sided alternative, based on a normal random sample, instead of Tracy-Widom law, one can also use \Cref{th:th1}. For $X_1,\cdots,X_n \overset{iid}{\sim} N_p(\mu,\mathbf{\Sigma})$ where $\mu,\Sigma$ are unknown and $p,n$ are both large and, $p \sim n^{\frac{1}{2} - \epsilon}$, $\epsilon > 0$. Consider the testing problem with a two-sided alternative, \begin{equation} \label{eq: two sided test} \begin{split} H_0: \mathbf{\Sigma} = \mathbf{\Sigma}_0 \\ H_1: \mathbf{\Sigma} \neq \mathbf{\Sigma}_0 \end{split} \end{equation} From \Cref{th:th1}, it turns out \begin{equation} \phi(\mathbf{X}) = \mathbf{1}\left( \sqrt{\frac{n-1}{2p}} \left| \sum_{i=1}^p \log\left( \frac{\hat{\lambda}_i}{\lambda_i} \right) - \sum_{i=1}^p \log(n-p+i) \right| > z_{\alpha/2} \right) \end{equation} is an asymptotically size $\alpha$ test, where $\hat{\lambda}_1,\cdots,\hat{\lambda}_p$ are the eigenvalues of the sample covariance matrix $S = \frac{1}{n-1} \sum_{i=1}^n (X_i - \overline{X}_n)(X_i - \overline{X}_n)^T$, $\lambda_1,\cdots,\lambda_p$ are the eigenvalues of $\mathbf{\Sigma}_0$, $\alpha \in (0,1)$ and $z_{\alpha}$ is the $(1-\alpha)$th quantile of N(0,1). The same idea can be generalized for testing problems of the covariance matrices in high-dimensional Linear Regression when the number of response variables and number of data points are both large. Consider the multivariate Linear Regression model, \begin{equation} \label{eq: High dim Lin Reg} \mathbf{Y} = \mathbf{XB} + \mathbf{E} \end{equation} where $\mathbf{Y} = [Y_1: \cdots: Y_m] \in \R^{n \times m}$ is the response matrix consisting $n$ observations for each of the $m$ response variable, $\mathbf{X} \in \R^{n \times p}$ is the design matrix where $p$ is the number of covariates. $\mathbf{E}$ denotes the error matrix. For inference purposes, it is further assumed that $\mathbf{E} \sim NDM(0,\mathbf{\Sigma})$ i.e. rows of $\mathbf{E}$ are iid from $N(0,\mathbf{\Sigma})$. Consider the testing problem with a two-sided alternative, as in (\ref{eq: two sided test}) i.e. \begin{equation*} \begin{split} \begin{split} H_0: \mathbf{\Sigma} = \mathbf{\Sigma}_0 \\ H_1: \mathbf{\Sigma} \neq \mathbf{\Sigma}_0 \end{split} \end{split} \end{equation*} Under $H_0$, the sum of squares of error (SSE) defined as $\mathbf{Y}^T(\mathbf{I - P_X})\mathbf{Y}$, where $\mathbf{P_X}$ is the orthogonal projection matrix of $\mathcal{C}(\mathbf{X})$ follows $W_m(\mathbf{\Sigma}_0,n-r)$ with $r$ to be the rank of $\mathbf{X}$. Therefore an asymptotically level $\alpha$ test to test (\ref{eq: two sided test}) is given by \begin{equation} \phi_{\mathcal{R}} := \mathbf{1}\left( \sqrt{\frac{n-r}{2m}} \left| \sum_{i=1}^m \log\left( \frac{\hat{\lambda}_i}{\lambda_i} \right) - \sum_{i=1}^m \log(n-r-k+i) \right| > z_{\alpha/2} \right) \end{equation} where $\hat{\lambda}_1,\cdots,\hat{\lambda}_p$ are the eigenvalues of SSE $= \mathbf{Y}^T(\mathbf{I - P_X})\mathbf{Y}$, $\lambda_1,\cdots,\lambda_p$ are the eigenvalues of $\mathbf{\Sigma}_0$, $\alpha \in (0,1)$ and $z_{\alpha}$ is the $(1-\alpha)$th quantile of N(0,1). The same idea can be generalized for the two sample tests of equality for covariance matrices as well. Suppose we have data points $X_1,\cdots,X_m \overset{iid}{\sim} N_p(\mu_1,\mathbf{\Sigma}_1)$ and $Y_1,\cdots,Y_n \overset{iid}{\sim} N_p(\mu_2,\mathbf{\Sigma}_2)$ where $\mu_1,\mu_2,\mathbf{\Sigma}_1,\mathbf{\Sigma}_2$ are unknown. $n \equiv n(m)$ satisfies $\lim_{m \to \infty} \frac{n}{m} = c \in (0,\infty)$. Consider the testing problem, \begin{equation} \begin{split} H_0: \mathbf{\Sigma_1 = \Sigma_2} \\ H_1: \mathbf{\Sigma_1 \neq \Sigma_2} \end{split} \end{equation} Then by \Cref{th:th1} it turns out that \begin{equation} \begin{split} \phi(\mathbf{X,Y}) := \mathbf{1}\left( \sqrt{\frac{m}{2p\left( 1 + \frac{1}{c} \right)}} \left| \sum_{i=1}^p \log\left( \frac{\hat{\lambda}_i}{\hat{\lambda}_i^*} \right) - \sum_{i=1}^p \log \left( \frac{n-p+i}{m-p+i} \right) \right| > z_{\alpha/2} \right) \end{split} \end{equation} is an asymptotically level $\alpha$ test where $\hat{\lambda}_1,\cdots,\hat{\lambda}_p$ are the eigenvalues of the sample covariance matrix $\mathbf{S_x} = \frac{1}{m-1} \sum_{i=1}^m (X_i - \overline{X}_m)(X_i - \overline{X}_m)^T$, $\hat{\lambda}_1^*,\cdots,\hat{\lambda}_p^*$ are the eigenvalues of $\mathbf{S_Y} = \frac{1}{n-1} \sum_{i=1}^n (Y_i - \overline{Y}_n)(Y_i - \overline{Y}_n)^T$, $\alpha \in (0,1)$ and $z_{\alpha}$ is the $(1-\alpha)$th quantile of N(0,1). The power function for this test is given by, $\beta(\mathbf{\Sigma_1,\Sigma_2}) := 1 - \Phi\left( z_{\alpha/2} - \sqrt{\frac{m}{2p\left( 1 + \frac{1}{c} \right)}}\sum_{i=1}^p \log\left(\frac{\lambda_i}{\lambda_i^*} \right) \right) + \Phi\left( - z_{\alpha/2} - \sqrt{\frac{m}{2p\left( 1 + \frac{1}{c} \right)}}\sum_{i=1}^p \log\left(\frac{\lambda_i}{\lambda_i^*} \right) \right) + f(n,m)$ where $f(n,m) = O\left( p\left( \frac{1}{\sqrt{m}} + \frac{1}{\sqrt{n}} \right) \right)$. The same test can be done using the asymptotic theory of the largest root of $F-$type matrices as well. Since $\mathbf{S_Y}$ is almost surely invertible, we can define the test, \begin{equation} \phi_{\mathcal{F}} := \mathbf{1}\left( \frac{\frac{\breve{n}}{\breve{m}} \lambda_1 - \mu_{J,p}}{\sigma_{J,p}} > F_1^{-1}(1-\alpha) \right) \end{equation} where $\breve{n},\breve{m},\mu_{J,p},\sigma_{J,p},F_1(\cdot)$ are as in \Cref{th: tw for Fmat} and $\lambda_1$ is the largest root of \[\big((n-1)\mathbf{S_Y}\big)^{-1}\big((m-1)\mathbf{S_X} \big)\] By \Cref{th: tw for Fmat}, $\phi_{\mathcal{F}}$ turns out to be an asymptotic size $\alpha$ test. For the high-dimensional linear regression model defined in (\ref{eq: High dim Lin Reg}), one can also obtain a high-dimensional generalization of Wald's test using the asymptotic theories of the $F-$type matrices. Consider the wald's testing problem \begin{equation} \begin{split} H_0: \mathbf{L^TB = B_0}\\ H_1: \mathbf{L^TB \neq B_0} \end{split} \end{equation} where $\mathbf{L} \in \R^{p \times k}$ and rank of $\mathbf{L}$ is $k$. If the design matrix $\mathbf{X}$ has full column rank, i.e. $\rho(\mathbf{X}) = p$, the Ordinary Least Squares (OLS) estimate for $\mathbf{B}$ is given by $\mathbf{\hat{B}} = \mathbf{(X^TX)^{-1}X^TY}$. Then under $H_0$, \begin{equation} \mathbf{A_p} := \mathbf{(L^T\hat{B} - B_0)^T (L^T(X^TX)^{-1}L)^{-1}(L^T\hat{B} - B_0)} \overset{H_o}{\sim} W_m(\mathbf{\Sigma},k) \end{equation} and \begin{equation} \mathbf{B_p = Y^T(I-P_X)Y} \sim W_m(\mathbf{\Sigma},n-p) \end{equation} and $\mathbf{A_p,B_p}$ are independent. Let $\lambda_1$ be the largest root of $det(\lambda A_p - B_p)$. Then from \Cref{th: tw for Fmat}, it turns out that a normalized version of $\lambda_1$ asymptotically follows Tracy-Widom Distribution under $H_0$. Therefore, in view of \Cref{th: tw for Fmat}, one can construct an asymptotic size $\alpha$ test using $\lambda_1$. \subsection{Application in PCA} \label{subsec: app in PCA} In \Cref{subsec:cov matrices} we discussed the LSD and asymptotic properties of the sample covariance matrix under the Gaussianity assumption when the eigenvalues of the population covariance matrix are either identical or are evenly spread out so that none of them “sticks out” from the bulk. \citet{soshnikov2002note} proved the distributional limits under weaker assumptions, in addition to deriving distributional limits of the $k-$th largest eigenvalue, for fixed but arbitrary $k$. Under the Gaussianity assumption of the data, the asymptotic distribution of the eigenvalues of the sample covariance matrix also turns out to be Gaussian if the eigenvalues of the population covariance matrix are distinct. \begin{theorem}[\citet{mardia2024multivariate}] Let $X_1,\cdots,X_n \overset{iid}{\sim} N_p(0,\mathbf{\Sigma})$ where $\mathbf{\Sigma}$ is positive definite with all distinct eigenvalues $\lambda_1 > \cdots \lambda_p > 0$. Let $l_{n,1} \geqslant \cdots \geqslant l_{n,p}$ be the eigenvalues of $\frac{1}{n} \sum_{i=1}^nX_iX_i^T$ and $\mathbf{\Lambda} = \text{diag}(\lambda_1,\cdots,\lambda_p)$. Then \begin{equation} \sqrt{n}(l_n - \mathbf{\lambda}) \convD N_p(0,2\mathbf{\Lambda}^2) \end{equation} as $n \to \infty$ where $l_n = \begin{bmatrix} l_{n,1}\\ \vdots \\ l_{n,p} \end{bmatrix}$ and $\lambda = \begin{bmatrix} \lambda_1\\ \vdots \\ \lambda_p \end{bmatrix}$ \end{theorem} So the consistency of the sample eigenvalues of the sample covariance matrix holds when the population covariance matrix has either all eigenvalues identical or all distinct from each other. However, in recent years, researchers in various fields have been using different versions of covariance matrices of growing dimensions with special patterns. For instance, in speech recognition (\citet{hastie1995penalized}), wireless communication (\citet{telatar1999capacity}), and statistical learning (\citet{hoyle2003limiting}) a few of the sample eigenvalues have limiting behavior that is different from the behavior when the covariance is the identity. While high-dimensional data often exhibits complex patterns, it's frequently characterized by a simple underlying structure. This structure can be modeled as a low-dimensional "signal" obscured by high-dimensional "noise." Assuming an additive relationship between these components, we can represent the data using a factor model. Factor models are particularly useful for detecting and estimating low-dimensional signals within isotropic or nearly isotropic noise. Key statistical questions, such as those related to dimension reduction, can be effectively addressed by analyzing the eigenvalues and eigenvectors of the sample covariance matrix. A particularly useful idealized model of this kind, named the spiked covariance model by \citet{johnstone2001distribution} has been in use for quite some time in statistics. Under this model, the population covariance matrix $\mathbf{\Sigma}$ is expressed as \begin{equation} \label{eq: spike} \mathbf{\Sigma} = \sum_{j=1}^M \lambda_j \theta_j \theta_j^* + \sigma^2 I_p, \end{equation} where $\theta_1, ..., \theta_M$ are orthonormal; $\lambda_1 \geq ... \geq \lambda_M > 0$ and $\sigma^2 > 0$. This model implies that, except for $M$ leading eigenvalues $l_j = \lambda_j + \sigma^2$ for $j = 1, ..., M$, the rest of the eigenvalues are all equal. This model has been studied extensively in the context of high-dimensional PCA since it brings out several key issues associated with dimension reduction in the high-dimensional context. \citet{johnstone2009sparse} first demonstrated that if $\frac{p}{n} \to \gamma \in (0,\infty)$ the sample principal components are inconsistent estimates of the population principal components under (\ref{eq: spike}). This phase transition phenomenon is described in its simplest form in the following theorem, where, for convenience, we assume in (\ref{eq: spike}) $\sigma^2 = 1$. \begin{theorem}[\citet{baik2006eigenvalues}] \label{th:th 10} Suppose that $\Sigma$ is a $p \times p$ positive definite matrix with eigenvalues $\ell_1 \geq \cdots \geq \ell_M > 1 = \cdots = 1$, and let $\hat{\ell}_1 \geq \cdots \geq \hat{\ell}_p$ be the eigenvalues of the sample covariance matrix $S = n^{-1} \Sigma^{1/2} Z Z^* \Sigma^{1/2}$ where the $p \times n$ data matrix $Z$ has i.i.d. real or complex entries with zero mean, unit variance and finite fourth moment. Suppose that $p, n \to \infty$ such that $p/n \to \gamma \in (0, \infty)$. Then, for each fixed $j=1,2,\cdots,M$ \begin{equation} \hat{\ell}_j \xrightarrow{\text{a.s.}} \begin{cases} (1 + \sqrt{\gamma})^2 & \text{if } \ell_j \leq 1 + \sqrt{\gamma}, \\ \ell_j \left(1 + \frac{\gamma}{\ell_j - 1}\right) & \text{if } \ell_j > 1 + \sqrt{\gamma}. \end{cases} \end{equation} \end{theorem} Therefore, when the population covariance matrix is of the spike form, it might not be such a good idea to use Principal Component Analysis (PCA) for dimension reduction in a high-dimensional setting, at least not in its standard form. In this regard, one natural question is how one can test if the population covariance matrix $\mathbf{\Sigma}$ is in the form of (\ref{eq: spike}) and how bad the inconsistency of the sample principal components if $\mathbf{\Sigma}$ is in spike form. The following theorem provides an answer to these questions. \begin{theorem}[\citet{paul2007asymptotics}] \label{th: th 11} Suppose that $X_1,\cdots,X_n \overset{iid}{\sim} N_p(0,\Sigma)$ where $\Sigma$ is a $p \times p$ positive definite matrix with eigenvalues $\ell_1 \geq \cdots \geq \ell_M > 1 = \cdots = 1$, and let $\hat{\ell}_1 \geq \cdots \geq \hat{\ell}_p$ be the eigenvalues of the sample covariance matrix $S = \frac{1}{n}\sum_{i=1}^nX_iX_i^T$. Suppose that $p, n \to \infty$ such that $\frac{p}{n} - \gamma = o(n^{-1/2})$ for a $\gamma \in (0,\infty)$. For a fixed $j \in \{1,2,\cdots,M\}$ if $\ell_j > 1 + \sqrt{\gamma}$, then \begin{equation} \sqrt{n}\left(\hat{\ell}_j - \ell_j \left(1 + \frac{\gamma}{\ell_j - 1}\right) \right) \convD N(0,\sigma^2(\ell_j)) \end{equation} as $n \to \infty$ where $\sigma^2(\ell) := 2\ell^2\left(1 - \frac{\gamma}{(\ell-1)^2} \right)$ \end{theorem} Suppose that we test the hypothesis $H_0: \Sigma = I$ versus the alternative that $H_1: \Sigma =$ diag$(\ell_1, \ldots, \ell_M, 1, \ldots, 1)$ with $\ell_1 \geq \cdots \geq \ell_M > 1$, based on i.i.d. observations from $N(0, \Sigma)$. If $\ell_1 > 1 + \sqrt{\gamma}$, it follows from \Cref{th:th 10} that the largest root test is asymptotically consistent. For the special case when $\ell_1$ is of multiplicity one, \Cref{th: th 11} gives an expression for the asymptotic power function, assuming that $p / n$ converges to $\gamma$ fast enough, as $n \to \infty$. One has to view this in context since the result is derived under the assumption that $\ell_1, \ldots, \ell_M$ are all fixed, and we do not have a rate of convergence for the distribution of $\hat{\ell}_1$ toward normality. However, \Cref{th: th 11} can be used to find confidence intervals for the larger eigenvalues under the non-null model. Under the same set of assumptions as of \Cref{th: th 11}, \citet{paul2007asymptotics} proved further that, if $\ell_j \leqslant 1 + \sqrt{\gamma}$ and $\ell_j$ is of arithmetic multiplicity one, then the angle between the $j-$th sample and population eigenvectors converges to $\frac{\pi}{2}$ almost surely which essentially shows in which extent the sample principal components can be inconsistent and provides a generalization of \citet{johnstone2009sparse}. Later on \citet{bai2008large} extends the results of \citet{paul2007asymptotics} in the context of spiked covariance matrix by dropping the Gaussianity assumption. \begin{figure}[h] \centering \includegraphics[width=0.7\linewidth]{spike_cov.png} \caption{(\citet{paul2007asymptotics}) An illustration of the phase transition of eigenvalues in a spiked covariance model: here, $p=50$, $n=200$ and eigenvalues of the covariance matrix are $\ell_1=2.5$, $\ell_2=1.5$, $\ell_j=1$ for $j=3,\ldots,p$. So, $\ell_1 > 1+\sqrt{p/n}$ and $\ell_2 = 1+\sqrt{p/n}$. Blue dots correspond to the population eigenvalues. Black circles correspond to the sample eigenvalues (based on i.i.d. Gaussian samples) for 50 replicates. Solid red circles indicate the theoretical limits of the first two eigenvalues for $\gamma=p/n=0.25$.} \label{fig:enter-label} \end{figure} \subsection{On signal processing and wireless communications} \label{subsec: signal pro} Large random matrices come up often in signal processing, especially in wireless communication. \citet{bai2010spectral}, \citet{couillet2011random}, and \citet{tulino2004random} highlight several such cases, including: (i) finding the channel capacity of MIMO (multiple-input-multiple-output) systems, which involves calculating the logarithm of the determinant of the matrix \( \mathbf{I + S} \), where \( \mathbf{S} \) is a Wishart matrix that reflects the signal-to-noise ratio in transmission; (ii) finding the limiting SINR (signal-to-interference-noise ratio) in random channels and in linearly precoded systems, like CDMA (code-division-multiple-access) systems (\citet{bai2007signal}); (iii) analyzing the performance of receivers as the system size grows; and (iv) estimating energy from multiple sources (\citet{couillet2011random}). Besides, random matrices are useful in a variety of signal-processing problems, such as detecting input signals (\citet{nadakuditi2010fundamental}; \citet{silverstein1992signal}) and estimating subspaces in sensor networks (\citet{hachem2013subspace}). Furthermore, the asymptotic distribution of the spectra of large random matrices and the idea behind Roy's largest root test (\citet{roy1953heuristic}, \citet{johnstone2017roy}) can be used to construct nonparametric tests to detect the number of signals embedded in noise (\citet{kritchman2009non}). The standard setup for signals impinging on an array with sensors consists of $n$ i.i.d $p-$dimensional observations $\{\mathbf{x}_i\}_{i=1}^n$ from the model, \begin{equation} \mathbf{x}(t) = \mathbf{A} \mathbf{s}(t) + \sigma \mathbf{n}(t) \end{equation} sampled at $n$ distinct times $t_i$, where $\mathbf{A} = [\mathbf{a}_1, \dots, \mathbf{a}_K]$ is the $p \times K$ steering matrix of $K$ linearly independent $p$-dimensional vectors. The $K \times 1$ vector $\mathbf{s}(t) = [s_1(t), \dots, s_K(t)]^T$ represents the random signals, assumed zero mean and stationary with full rank covariance matrix. $\sigma$ is the unknown noise level, and $\mathbf{n}(t)$ is a $p \times 1$ additive Gaussian noise vector, distributed $\mathcal{N}(0, \mathbf{I}_p)$ and independent of $\mathbf{s}(t)$. Under these assumptions, the population covariance matrix $\Sigma$ of $\mathbf{x}(t)$ has a diagonal form, \begin{equation} \mathbf{W}^H \Sigma \mathbf{W} = \sigma^2 \mathbf{I}_p + \text{diag}(\lambda_1, \dots, \lambda_K, 0, \dots, 0) \end{equation} where columns of $\mathbf{W}$ forms a basis of $\mathbb{C}^p$ (or of $\mathbb{R}^p$ if the signals are real valued). Let $\mathbf{S}_n$ be the sample covariance matrix of $\{\mathbf{x}_i\}_{i=1}^n$, defined as \begin{equation*} \mathbf{S}_n = \frac{1}{n} \sum_{i=1}^n \mathbf{x}_i\mathbf{x}_i^H \end{equation*} having the eigenvalues $l_1 \geqslant l_2 \geqslant \cdots \geqslant l_p$. The number of signals $K$, can then be estimated with the number of eigenvalues of the sample covariance matrix $\mathbf{S}_n$ which are \textit{significanly larger} i.e. bigger than a certain threshold, where the individual thresholds for the eigenvalues can be determined using the Tracy Widom laws (\Cref{th: tracy widom}). The following algorithm, which is deeply motivated by Roy's largest root test (\citet{roy1953heuristic}, \citet{johnstone2017roy}), takes the eigenvalues $l_1,\cdots,l_p$ of the sample covariance matrix $\mathbf{S}_n$ as input and gives the estimated number of signals $\hat{K}_{\text{RMT}}$ as output. The algorithm works as follows: For $k = 1, \dots, \min(p, n) - 1$, we test \[ H_0 : \text{at most } k - 1 \text{ signals} \quad \text{vs.} \quad H_1 : \text{at least } k \text{ signals}. \] Under the null hypothesis, $\ell_k$ arises from noise. Thus, we reject $H_0$ if $\ell_k$ is too large, i.e. \[ \ell_k > \hat{\sigma}^2(k) C_{n,p,k}(\alpha) \] where $\hat{\sigma}^2(k)$ is an estimate for the unknown noise level $\sigma^2$ taken to be, \begin{equation} \label{eq: noise est} \hat{\sigma}^2(k) = \frac{1}{p-k} \sum_{j = k+1}^p l_j \end{equation} and \begin{equation} \label{eq: cut off} C_{n,p,k}(\alpha) = \mu_{n,p - k} + s(\alpha) \xi_{n,p - k} \end{equation} where $\mu_{n,p}$ and $\xi_{n,p}$ are the centering and scaling parameters defined as \begin{equation} \begin{split} \mu_{n,p} &= \frac{1}{n} (\sqrt{n-1/2} + \sqrt{p-1/2})^2 \\ \xi_{n,p} &= \sqrt{\frac{\mu_{n,p}}{n}} \left(\frac{1}{\sqrt{n-1/2}} + \frac{1}{\sqrt{p-1/2}}\right)^{1/3} \end{split} \end{equation} and $s(\alpha)$ is the $1-\alpha$ quantile of the Tracy Widom distribution. \citet{kritchman2009non} showed, \[ \Pr\{\text{reject } H_0|H_0\} = \Pr\{\ell_k > \sigma^2 C_{n,p,k}(\alpha)|H_0\} \approx \alpha. \] Hence, $\alpha$ controls the probability of model overestimation. We stop at the smallest index $k$ where the above condition fails, i.e., the first time we accept $H_0$. Our estimate of the number of signals is then $\hat{K}_{\text{RMT}} = k - 1$. Hence, the estimator of the number of signals is, \[ \hat{K}_{\text{RMT}} = \arg \min_k \left\{ \ell_k < \hat{\sigma}^2(k)(\mu_{n,p - k} + s(\alpha) \xi_{n,p - k}) \right\} - 1. \] \begin{algorithm}[H] \caption{Algorithm for detecting number of signals (\citet{kritchman2009non})} \KwIn{Confidence level $\alpha$, observations $\ell_k$ for $k = 1, \dots, \min(p, n) - 1$} \KwOut{Estimated number of signals $\hat{K}_{\text{RMT}}$} \For{$k = 1$ \textbf{to} $\min(p, n) - 1$}{ Compute the threshold $\hat{\sigma}^2(k) C_{n,p,k}(\alpha)$ using \ref{eq: noise est}, \ref{eq: cut off}\; \eIf{$\ell_k > \hat{\sigma}^2(k) C_{n,p,k}(\alpha)$}{ conclude that there are at least $k$ signals and set $k = k+1$ \; }{ conclude that there are at most $k-1$ signals\; Set $\hat{K}_{\text{RMT}} = k - 1$\; \textbf{break}\; } } \Return $\hat{K}_{\text{RMT}} = \arg \min_k \left\{ \ell_k < \hat{\sigma}^2(k)(\mu_{n,p - k} + s(\alpha) \xi_{n,p - k}) \right\} - 1$\; \end{algorithm} For a suitably chosen sequence of $\{\alpha\}_n$, $\hat{K}_{RMT,n}$ can be shown to be consistent i.e. $\underset{n \to \infty}{\text{lim}} \mathbb{P}(\hat{K}_{RMT,n} = K) = 1$ (\citet{kritchman2009non}) where $K$ is the original number of signals. To demonstrate the performance of the above algorithm, We plot the number of estimated signals when the actual number of signals is in a range of $2$ to $5$ and the errors are from Standard Normal, $t-$distribution with 5 df, Cauchy and Laplace distribution. Also, we vary the sample size $n$ in a range up to 5000. From \Cref{fig:est sig} it can be seen that, except when the noise has standard Cauchy distribution, if the sample size exceeds 1000, the estimated number of signals is the same as the original number of signals. The algorithm overestimates the number of signals for noise arriving from the Cauchy distribution. \begin{figure}[H] \centering \includegraphics[width=0.4\linewidth]{signal_g.jpeg} \includegraphics[width=0.4\linewidth]{signal_t5.jpeg} \includegraphics[width=0.4\linewidth]{signal_de.jpeg} \includegraphics[width=0.4\linewidth]{signal_c.jpeg} \caption{Number of Estimated Signals for different noise distributions: (in clockwise order) Standard Normal, t distribution with 5 df, Standard Cauchy, Double Exponential. The x-axis represents the sample size. } \label{fig:est sig} \end{figure} \subsection{On Changepoint Detection} \label{subsec: change point, cov} Change point detection (CPD) is a statistical method used to identify points in a dataset where the distribution of the data changes significantly. Studies of change-point detection problems date back to 1950. Since then, this topic has been of interest to statisticians and researchers in many other fields such as engineering, economics, climatology, biosciences, genomics, and linguistics due to its diverse applications. Different methods in parametric and nonparametric setups have been discovered (\citet{niu2016multiple}, \citet{aminikhanghahi2017survey}) for univariate and multivariate time series data. However, when the data is ultrahigh dimensional, most of these traditional methods struggle due to computational complexity or the failure to meet the underlying distributional assumptions. For example, to determine the change of covariance in high dimensional time series data, the sample covariance matrices are used which are extremely large dimensional. In this section, we discuss some methods of detecting change in covariance pattern of high dimensional time series which are motivated by the results of random matrix theory. Change point detection algorithms are traditionally classified as “online” or “offline.” We focus on the Offline setting, which considers the entire data set at once and looks back in time to recognize where the change occurred. The literature on detecting changes in covariance for high dimensional time series has grown substantially in the last few years. Based on the theory of large-scale random matrices in \citet{hero2012hub}, \citet{banerjee2015non} has developed a method for covariance CPD when the data points are independently drawn from an unknown elliptically contoured distribution. \citet{avanesov2018change}, \citet{wang2017optimal} obtain method based on the distance between sample covariance matrices, using the operator norm and $l_\infty$ norm of matrices, respectively. In particular, many authors consider changes in the moderate dimensional setting, that is, where the number of the parameters of the model is of the order of the number of data points. \citet{ryan2023detecting} proposes a novel method for detecting changes in the covariance structure of moderate dimensional time series. Let \( X_1, \dots, X_n \in \mathbb{R}^p \) be independent \( p \)-dimensional vectors with \begin{equation*} \operatorname{cov}(X_i) = \Sigma_{i,p}, \quad \text{for } 1 \leq i \leq n, \end{equation*} where each \( \Sigma_{i,p} \in \mathbb{R}^{p \times p} \) is of full rank. Furthermore, let \( \mathbf{X}_{n,p} \) denote an \( n \times p \) matrix defined by \( \mathbf{X}_{n,p} := (X_1^T, \dots, X_n^T)^T \). The method primarily aims to develop a testing procedure that can identify a change in the covariance structure of the data over time. For now, let us consider the case of a single changepoint. We compare a null hypothesis of the data sharing the same covariance versus an alternative setting that allows a single change at time $\tau$. Formally we have \begin{equation} \begin{split} \label{eq:hypo} H_0 &: \Sigma_{1,p} = \dots = \Sigma_{n,p} \\ H_1 &: \Sigma_{1,p} = \dots = \Sigma_{\tau,p} \neq \Sigma_{\tau+1,p} = \dots \Sigma_{n,p} \end{split} \end{equation} where $\tau$ is unknown. We are interested in distinguishing between the null and alternative hypothesis, and under the alternative locating the changepoint $\tau$, when the dimension of the data $p$, is of comparable to the sample size, $n$. In particular, we require that for all pairs $n, p$, the set \[T_{n,p}(\ell):=\{t\in \mathbb{Z}^+ \text{ such that } \ell < t < n-\ell\} \quad (2.4)\] is nonempty, where $\ell > p$ is a problem dependent positive constant. Note $T_{n,p}(\ell)$ defines the set of possible candidate changepoints, while $\ell$ is the minimum distance between changepoints or minimum segment length. Then for each candidate changepoint $t\in T_{n,p}(\ell)$, a two-sample test statistic $T(t)$ can be used to determine if the data to the left and right of the changepoint have different distributions. If the two sample test statistic for a candidate exceeds some threshold, then we say a change has occurred and an estimator for $\tau$ is given by the value $t\in T_{n,p}(\ell)$ that maximizes $T(t)$. In their method \citet{ryan2023detecting}, constructs the two sample test statistics using the eigenvalues of the sample covariance matrices of the two samples as follows. For two sample covariance matrices $\mathbf{A,B}$, in this context, we need to test whether $\mathbf{A}$ and $\mathbf{B}$ are equal or not. So in case they are identical, all of the eigenvalues of $R(\mathbf{A,B}) := \mathbf{B}^{-1}\mathbf{A}$ is 1. Therefore, the following function of the ratio matrix (or $F-$type matrix) $R(\mathbf{A,B})$, gives a suitable measure of deviance from the equality of the two matrices, \begin{equation} \label{eqn:Tstat} T(\mathbf{A}, \mathbf{B}) = \sum_{j=1}^{p} \left( 1 - \lambda_j(R(\mathbf{A}, \mathbf{B})) \right)^2 + \left( 1 - \lambda_j^{-1}(R(\mathbf{A}, \mathbf{B})) \right)^2 \end{equation} where $\lambda_j(R(\mathbf{A}, \mathbf{B}))$ is the $j$th largest eigenvalue of the matrix $R(\mathbf{A}, \mathbf{B})$. The function $T$ has valuable properties that may not be immediately obvious. \begin{proposition}[\citet{ryan2023detecting}] \label{prop:Tstat prop} Let $\mathbf{\Sigma}_1, \mathbf{\Sigma}_2 \in \mathbb{R}^{p \times p}$ be the covariance matrices of data $\mathbf{Z}_1 \in \mathbb{R}^{n_1 \times p}$ and $\mathbf{Z}_2 \in \mathbb{R}^{n_2 \times p}$, respectively, and define $T$ as in (2.5). Then we have that, for any covariance matrix $\mathbf{\Sigma}_0$: \begin{enumerate} \item $T$ is symmetric, that is, $T(\mathbf{\Sigma}_1, \mathbf{\Sigma}_2) = T(\mathbf{\Sigma}_2, \mathbf{\Sigma}_1)$; \item $T$ is symmetric with respect to the inversion of matrices, that is, \[ T(\mathbf{\Sigma}_1, \mathbf{\Sigma}_2) = T(\mathbf{\Sigma}_1^{-1}, \mathbf{\Sigma}_2^{-1}); \] \item If $\mathbf{\Sigma}_1 = \mathbf{\Sigma}_0 \mathbf{Z}_1^T \mathbf{Z}_1 \mathbf{\Sigma}_0$ and $\mathbf{\Sigma}_2 = \mathbf{\Sigma}_0 \mathbf{Z}_2^T \mathbf{Z}_2 \mathbf{\Sigma}_0$, then \[ T(\mathbf{\Sigma}_1, \mathbf{\Sigma}_2) = T(\mathbf{Z}_1^T \mathbf{Z}_1, \mathbf{Z}_2^T \mathbf{Z}_2). \] \end{enumerate} \end{proposition} The symmetry property is important for a changepoint analysis as the segmentation should be the same regardless of whether the data is read forward or backward. The second property states that $T$ is the same whether we examine the covariance matrix or the precision matrix. This ensures that differences between both small and large eigenvalues can be detected. The third property is particularly important as we can translate \Cref{prop:Tstat prop} from two separate datasets $\mathbf{Z}_1, \mathbf{Z}_2$ to two subsets of a single dataset $\mathbf{X}_{n,p}$. This implies that $T$ provides a test statistic that is independent of the underlying covariance of the data. it is to be noted that the function involves ratio matrices which are widely used in multivariate analysis to compare covariance matrices (\citet{finn1974general}). In particular functions of the eigenvalues of the ratio matrices are standard in literature (\citet{wilks1932certain}, \citet{lawley1938generalization}, \citet{potthoff1964generalized}) for inference and methodologies involving covariance matrices. \Cref{th:th7} also discusses the the LSD of the ratio matrics $R(\mathbf{A},\mathbf{B})$ under suitable conditions. Using the LSD of the ratio matrices, \citet{ryan2023detecting} finds the asymptotic distribution of $T(\mathbf{A},\mathbf{B})$ for two sample covariance matrices as presented in the following theorem, which gives the framework of the changepoint detection method. \begin{theorem}[\citet{ryan2023detecting}] \label{th:th9} Let \( X_{n_1,p} \in \mathbb{R}^{n_1 \times p} \) and \( X_{n_2,p} \in \mathbb{R}^{n_2 \times p} \) be random matrices with independent not necessarily identically distributed entries \( \{ X_{n_1,i,j}, 1 \leq i \leq n_1, 1 \leq j \leq p \} \) and \( \{ X_{n_2,k,j}, 1 \leq k \leq n_2, 1 \leq j \leq p \} \) with mean 0, variance 1 and fourth moment \( 1 + \kappa \). Furthermore, for any fixed \( \eta > 0 \), \begin{align} \frac{1}{n_1 p} \sum_{j=1}^p \sum_{i=1}^{n_1} \mathbb{E} |X_{n_1,i,j}|^4 \mathbf{1}(|X_{n_1,j,k}| \geq \eta \sqrt{n_1}) &\rightarrow 0 \tag{3.7} \\ \frac{1}{n_2 p} \sum_{j=1}^p \sum_{i=1}^{n_2} \mathbb{E} |X_{n_2,i,j}|^4 \mathbf{1}(|X_{n_2,j,k}| \geq \eta \sqrt{n_2}) &\rightarrow 0 \tag{3.8} \end{align} as \( n_1, n_2, p \) tend to infinity such that $ \frac{p}{n_1} \to \gamma_1 \in (0,1), \frac{p}{n_2} \to \gamma_2 \in (0,1), \gamma = (\gamma_1,\gamma_2)$ and $\mathbf{1}(\cdot)$ denotes the indicator function. Then as \( n \to \infty \), \begin{equation*} T \left( \frac{1}{n_1} X_{n_1,p}^T X_{n_1,p}, \frac{1}{n_2} X_{n_2,p}^T X_{n_2,p} \right) - p \int f^*(x) dF_{\gamma}(x) \rightarrow N(\mu(\gamma), \sigma^2(\gamma)) \end{equation*} where \begin{equation} T(A,B) = \sum_{j=1}^p \big[(1-\lambda_j (B^{-1}A))^2 + (1-\lambda_j^{-1} (B^{-1}A))^2\big] \hspace{0.1cm} (\lambda_j \hspace{0.2cm} \text{is $j$th maximum eigenvalue}), \end{equation} \begin{equation} f^*(x) = (1 - x)^2 + (1 - 1/x)^2, \end{equation} \begin{equation} \mu(\gamma) = 2K_{3,1} \left(1 - \gamma_2 / h^2\right) + 2K_{2,1} \gamma_2 / h + 2K_{3,2} \left(1 - \gamma_1^2 / h^2\right) + 2K_{2,2} \gamma_1 / h, \end{equation} \begin{align} \sigma^2(\gamma) &= \frac{2(K_{2,1}^2 + K_{3,1}^2 + 2K_{3,2}^2)}{h(h^2 - 1)} + \frac{(J_1 K_{2,1} / h - J_1 K_{3,1} (h^2 + 1))}{h^2 + (h^2 - 1)} \\ &+ \frac{(J_2 K_{2,1}2h) / (h^2 - 1)^3 + J_2 K_{3,1}(1 - 3h^2))}{h(h^2 - 1)^3)} \end{align} \begin{equation} K_{2,1} = \frac{2h(1 + h^2)}{(1 - \gamma_2)^4 - 2h / (1 - \gamma_2)^2}, \quad K_{2,2} = \frac{2h(1 + h^2)^2}{(1 - \gamma_1)^4} - 2h / (1 - \gamma_1)^2, \end{equation} \begin{equation} K_{3,1} = \frac{h^2}{(1 - \gamma_1)^4}, \quad K_{3,2} = \frac{-2(1 - \gamma_2)^2}{(1 - \gamma_2)^4}, \quad J_2 = (1 - \gamma_2)^4, \quad J_1 = -2 (1-\gamma_2)^2, \end{equation} \begin{equation} h = \sqrt{\gamma_1 + \gamma_2 - \gamma_1 \gamma_2}, \quad \gamma_1 = p / n_1, \quad \gamma_2 = p / n_2, \end{equation} \begin{equation} F_\gamma(dx) = \frac{1 - \gamma_2}{2\pi x (\gamma_1 + \gamma_2 x)} \sqrt{(b - x)(x - a)} \mathbf{I}_{[a,b]}(x) dx, \end{equation} \begin{equation} a = \frac{(1 - h)^2}{(1 - \gamma_2)^2}, \quad b = \frac{(1 + h)^2}{(1 - \gamma_2)^2}. \end{equation} \end{theorem} So, using \Cref{th:th9} we can immediately have a normalized version of $T$, i.e. \begin{equation} \label{eq:normalised Tstat} \Tilde{T} = \sigma^{-1}(\gamma) \left(T \left( \frac{1}{n_1} X_{n_1,p}^T X_{n_1,p}, \frac{1}{n_2} X_{n_2,p}^T X_{n_2,p} \right) - p \int f^*(x) dF_{\gamma}(x) - \mu(\gamma) \right) \end{equation} which will be asymptotically standard normal, and hence we can use the quantile of standard normal with multiple testing corrections (\citet{haynes2013bonferroni}) to test hypothesis \ref{eq:hypo}. So using \Cref{th:th9}, given one dataset we can test whether the data has one changepoint or not. For the case of Multiple changepoints, the method is generalized using the classic binary segmentation procedure (\citet{scott1974cluster}). The binary segmentation method extends a single changepoint test as follows. First, the test is run on the whole data. While running on a particular interval of time $(s,e)$, for each timepoint $\tau$ in that range (except leaving $l$ many timepoints from both sides of the interval, for efficiency purposes as the testing procedure is asymptotic) the algorithm finds the normalized test statistic $\Tilde{T}(\tau)$ (as in \Cref{eq:normalised Tstat}) by breaking the datapoints into two parts pivoting $\tau$ and then finds the maximum value of the test statistic $\Tilde{T}(\tau)$ over $\tau$ in that interval $(s+l,e-l)$ and check if that exceeds a cutoff $\nu$ to guarantee the existence of a changepoint in the interval $(s,e)$. If no change is found then the algorithm terminates. If a changepoint is found, it is added to the list of estimated changepoints, and the binary segmentation procedure is then run on the data to the left and right of the candidate change. This process continues until no more changes are found. Note the threshold, $\nu$, and the minimum segment length, $\ell$, remain the same. Note that several extensions of the traditional binary segmentation procedure have been proposed in recent years (\citet{olshen2004circular}; \citet{fryzlewicz2014wild}) which may be used to generalize the algorithm of \citet{ryan2023detecting}. The full proposed procedure is described in algorithm \ref{algo:RatioBinseg}. \begin{algorithm}[H] \label{algo:RatioBinseg} \caption{Ratio Binary Segmentation (RatioBinSeg) (\citet{ryan2023detecting})} \KwIn{Data matrix $X$, interval $(s, e)$, set of changepoints $C$, minimum segment length $\ell$, significance level $\alpha$} \KwOut{Set of changepoints $C$} Set $\nu = \Phi^{-1}(1 - \frac{\alpha}{n^2}$), where $\Phi(\cdot)$ N(0,1) CDF\; \For{$\tau = s + \ell$ \textbf{to} $e - \ell$}{ Compute $\gamma := \left( \frac{p}{\tau}, \frac{p}{n - \tau} \right)$\; Compute $\widetilde{T}(\tau) := \sigma^{-1/2}(\gamma) \left( T\left( \overline{\Sigma}(s, \tau), \overline{\Sigma}(\tau, e) \right) - p \int f^*(x) dF_y - \mu(\gamma) \right)$\; } \textbf{end}\\ Set $\hat{\tau} := \arg \max_{\tau} \widetilde{T}(\tau)$ for $s + \ell < \tau < e - \ell$\; \If{$\widetilde{T}(\hat{\tau}) > \nu$}{ Set $C_l := \text{RatioBinSeg}(X, (s, \hat{\tau}), C, \ell, \alpha)$\; Set $C_r := \text{RatioBinSeg}(X, (\hat{\tau}, e), C, \ell, \alpha)$\; Update $C = C \cup \{\hat{\tau}\} \cup C_l \cup C_r$\; } \textbf{end}\\ \Return{Set of changepoints $C$}\; \end{algorithm} In the algorithm, $\Bar{\Sigma}$ is the natural estimate of $\Sigma$ based on the data in the corresponding time interval. For the multiple changepoint setting, let $\tau:= \{\tau_1, . . . , \tau_m\}$ and $\hat{\tau}:= \{\hat{\tau}_1, . . . , \hat{\tau}_m \}$ to denote the set of true changepoints and the set of estimated changepoints, respectively. The changepoint $\tau_i$ is said to be detected correctly if $|\hat{\tau}_j - \tau_i| \le h$ for some $1 \le j \le \hat{m}$ and denote the set of correctly estimated changes by $\hat{\tau}_c$. h = 20 is chosen for simulation, although it should be noted that in reality, the desired accuracy would be application-specific and dependent on the minimum segment length $l$. Then the False Positive Rate (FPR) is defined as the number of wrongly detected changepoints out of the detected ones, i.e. \[FPR = \frac{|\hat{\tau}| - |\hat{\tau}_c|}{|\hat{\tau}|}\] Table for FPR for this method and \citet{wang2017optimal} for various $n$ and $p$ are in \Cref{tab:FPR comp} \begin{table}[h!] \centering \caption{Comparison of FPR for various $n,p$} \label{tab:FPR comp} \begin{tabular}{cccccc} \hline \multicolumn{2}{c}{} & \multicolumn{2}{c}{Assumptions of \citet{ryan2023detecting}} & \multicolumn{2}{c}{Assumptions of \citet{wang2017optimal}} \\ \cline{3-6} \textbf{p} & \textbf{n} & \textbf{Ratio} & \textbf{Wang} & \textbf{Ratio} & \textbf{Wang} \\ \hline 3 & 500 & 0.24 & 0.63 & \textbf{0.10} & 0.64 \\ 3 & 1000 & 0.28 & 0.77 & \textbf{0.14} & 0.80 \\ 3 & 2000 & 0.31 & 0.85 & \textbf{0.16} & 0.88 \\ 3 & 5000 & 0.31 & 0.90 & \textbf{0.17} & 0.92 \\ 10 & 500 & 0.16 & 0.27 & 0.23 & 0.39 \\ 10 & 1000 & 0.13 & 0.43 & 0.24 & 0.62 \\ 10 & 2000 & 0.10 & 0.53 & 0.24 & 0.75 \\ 10 & 5000 & \textbf{0.09} & 0.62 & 0.19 & 0.80 \\ 30 & 2000 & \textbf{0.02} & 0.32 & \textbf{0.03} & 0.58 \\ 30 & 5000 & \textbf{0.02} & 0.31 & \textbf{0.03} & 0.78 \\ 100 & 5000 & \textbf{0.00} & 0.45 & \textbf{0.00} & 0.18 \\ \hline \end{tabular} \end{table} \section{Conclusion and Future directions} \label{sec: conclusion} This article highlights the profound role of random matrix theory (RMT) in addressing challenges arising in high-dimensional statistics. By leveraging the asymptotic spectral properties of large random matrices, particularly covariance matrices, and ratios of covariance matrices, RMT provides a novel theoretical foundation for statistical methods. The exploration of both the bulk spectrum and the extreme eigenvalues underscores the versatility of these tools in understanding high-dimensional data structures. The applications discussed in this article demonstrate the practical relevance of RMT. From inference on covariance matrices to dimensionality reduction through PCA, noise reduction in signal processing, and changepoint detection, RMT proves to be an indispensable framework for tackling modern statistical problems. The unifying principles of RMT not only enhance the theoretical understanding of high-dimensional phenomena but also drive the development of innovative methodologies in diverse fields. This work provides an inspection of the bridge between the mathematical elegance of random matrix theory and its impactful applications in statistics, emphasizing the potential for further exploration and development in this vibrant intersection of disciplines. We discuss some of the future directions of application of RMT, which has a great potential: \begin{itemize} \item Most of the results in RMT are based on iid observations. Though work has been done for certain covariance patterns as well, however, there is a great potential for extending the current theory on the eigenvalues of Wishart-type matrices, when the columns of the data matrix can be viewed as a realization of a high-dimensional multivariate time series, and that can have a significant impact on econometrics and finance. \item The RMT-based methods can be generalized when there are missing values in the dataset. For example, in high-dimensional spatiotemporal statistics, for each timepoint, the spatial data is in the form of a matrix. In most cases, for each time point, there are a couple of missing values in the matrix consisting of the spatial data at that time point. In literature, in case it is assumed that the spatial data arises from a random field. \citet{deb2017asymptotic} has a detailed discussion about the spectral analysis of such datasets coming from a random field. However, the asymptotic theory provided there assumes the data dimension to be fixed. So generalization of these results using the asymptotic theories of large random matrices is an open problem yet to be solved. \item A potentially useful avenue for the application of RMT is in numerical optimization algorithms that use gradient-based methods for large dimensional data. While there has been explosive growth in mathematical descriptions in the RMT literature, computational tools have not kept pace with the theoretical developments. Integration of computational tools with tools for the analysis of large dimensional data using RMT principles has the potential to create a new paradigm for statistical practices. \end{itemize} \section*{} \input{custom.bbl} \newpage \section{Appendix} \textbf{Proof of Theorem 1:} Given $\mathbf{X}_n \sim W_p(\mathbf{\Sigma},n)$ where $n \geqslant p$. Let $\lambda_1^{(n)},\cdots,\lambda_p^{(n)}$ be the eigenvalues of $\mathbf{X}_n$ and $\lambda_1,\cdots,\lambda_p$ be the eigenvalues of $\mathbf{\Sigma}$. For a random variable $X$, let $F_X(\cdot)$ be its CDF and for two CDFs F and G, define \begin{equation} \Delta(F,G) := \sup_{x \in \mathbb{R}}|F(x) - G(x)| \end{equation} We first prove a couple of lemmas needed for the proof. \textbf{Lemma 1:} Let $X_n,Y_n,X,Y$ be real-valued random variables having a joint distribution such that $(X_n,X)$ is independent of $(Y_n,Y)$. Then \begin{equation} \Delta(F_{X_n + Y_n}, F_{X+Y}) \leqslant \Delta(F_{X_n},F_X) + \Delta(F_{Y_n},F_Y) \end{equation} \textbf{Proof of Lemma 1:} Fix $x \in \mathbb{R}$. Then observe that, \begin{flalign*} &\qquad|\P(X_n + Y_n \leqslant x) - \P(X + Y \leqslant x)| &&\\ &\quad= |\E \left(\P(X_n \leqslant x - Y_n) - \P(X \leqslant x - Y) | Y_n, Y \right)| &&\\ &\quad= |\E (\P(X_n \leqslant x - Y_n) - \P(X_n \leqslant x - Y) + \P(X_n \leqslant x - Y) - \P(X \leqslant x - Y) \big| Y_n, Y \big)| &&\\ &\quad= |\E (\P(X_n \leqslant x - Y_n) - \P(X_n \leqslant x - Y)\big| Y_n, Y \big)| + \E\big|\P(X_n \leqslant x - Y) - \P(X \leqslant x - Y) \big| Y_n, Y \big)&&\\ &\quad= I + II \hspace{0.2cm}(say)&& \end{flalign*} Observe that $II \leqslant \Delta(F_{X_n},F_X)$ and \begin{flalign*} I &= \big|\P(X_n+Y_n \leqslant x) - \P(X_n+Y \leqslant x)\big| \\ &= \big| \E \big( \P(Y_n \leqslant x - X_n) - \P(Y \leqslant x - X_n) \big| X_n \big) \big| \\ &\leqslant \E \big| \P(Y_n \leqslant x - X_n) - \P(Y \leqslant x - X_n) \big| X_n \big)\\ &\leqslant \Delta(F_{Y_n},F_Y) \end{flalign*} Thus for all $x \in \R$, we have $\big|\P(X_n + Y_n \leqslant x) - \P(X + Y \leqslant x)\big| \leqslant \Delta(F_{X_n},F_X) + \Delta(F_{Y_n},F_Y)$. Hence \textbf{Lemma 1} follows. $\blacksquare$ \textbf{Lemma 2:} Let $\Phi(\cdot)$ be the N(0,1) CDF and $m$ be a natural number. Then there exists $c >0$ such that \begin{equation} \Delta\left(\Phi \left(\sqrt{\frac{m}{2}} \left( e^{\sqrt{\frac{2}{m}}x} - 1 \right)\right), \Phi(x)\right) \leqslant \frac{c}{\sqrt{m}} \end{equation} \textbf{Proof of Lemma 2:} Let $U_1,\cdots,U_m \overset{iid}{\sim} N(0,1)$. Observe that, $Y_m := \sqrt{\frac{m}{2}} \log \left( 1 + \sqrt{2} \cdot \overline{U}_m \right)$ have cdf $F_{Y_m}(x) := \Phi \left(\sqrt{\frac{m}{2}} \left( e^{\sqrt{\frac{2}{m}}x} - 1 \right)\right)$ where $\overline{U}_m := \frac{1}{m} \sum_{i=1}^m U_i$. Consider, $f(x) := \log (1 + \sqrt{2}\cdot x)$. Since $f(0) = 0$ and $f'(0) = \sqrt{2}$, by theorem 2.10 of \citet{pinelis2016optimal}, there exists $c > 0$ such that \begin{equation} \sup_{x \in \R} \left| \P\left( \frac{\sqrt{m}f(\overline{U}_m)}{|f'(0)|} \leqslant x \right) - \Phi(x) \right| \leqslant \frac{c}{\sqrt{m}} \end{equation} Since $\frac{\sqrt{m}f(\overline{U}_m)}{|f'(0)|} = Y_m$, we have \begin{equation*} \Delta(F_{Y_m},\Phi) \leqslant \frac{c}{\sqrt{m}} \end{equation*} Hence \textbf{Lemma 2} follows. $\blacksquare$ \textbf{Lemma 3:} \hspace{0.2cm}[Berry Esseen type Bounds for log of $\chi^2$ random variables] Let $Z_m \sim \chi^2_m$. Then, there exists $c > 0$ such that, \begin{equation} \sup_{x \in \R}\left| \P\left( \sqrt{\frac{m}{2}} \log\left(\frac{Z_m}{m}\right) \leqslant x \right) - \Phi(x) \right| \leqslant \frac{c}{\sqrt{m}} \end{equation} \textbf{Proof of Lemma 3:} Observe that by triangle inequality, we have \begin{equation} \label{eq: tineq} \Delta(G,\Phi) \leqslant \Delta(G, F_{Y_m}) + \Delta(F_{Y_m},\Phi) \end{equation} where $G(x) := \P\left( \sqrt{\frac{m}{2}} \log\left(\frac{Z_m}{m}\right) \leqslant x \right)$ and $F_{Y_m}$ is as in \textbf{Lemma 2}. By \textbf{Lemma 2}, there exists $c_1 > 0$, such that $\Delta(F_{Y_m},\Phi) \leqslant \frac{c_1}{\sqrt{m}}$. By the Berry–Esseen theorem, there exists $c_2 > 0$, such that for all $x \in \R$, \begin{equation} \label{eq: BE phi} \Phi(x) - \frac{c_2}{\sqrt{m}} \leqslant \P\left( \sqrt{\frac{m}{2}} \left(\frac{Z_m}{m} - 1\right) \leqslant x \right) \leqslant \Phi(x) + \frac{c_2}{\sqrt{m}} \end{equation} Now $G(x) = \P\left( \sqrt{\frac{m}{2}} \log\left(\frac{Z_m}{m}\right) \leqslant x \right) = \P\left(\sqrt{\frac{m}{2}} \left(\frac{Z_m}{m} - 1\right) \leqslant \sqrt{\frac{m}{2}} \left( e^{\sqrt{\frac{2}{m}}x} - 1 \right) \right)$. So by (\ref{eq: BE phi}), $\left| G(x) - F_{Y_m}(x) \right| \leqslant \frac{c_2}{\sqrt{m}}$ i.e. $\Delta(G,F_{Y_m}) \leqslant \frac{c_2}{\sqrt{m}}$. Thus by (\ref{eq: BE phi}), $\Delta(G,\Phi) \leqslant \frac{c}{\sqrt{m}}$ where $c:= c_1+c_2$ completing the proof of \textbf{Lemma 3}. $\blacksquare$ Now we complete the proof of the theorem. Observe that $|\mathbf{X}_n| = |\mathbf{\Sigma}|U_1\cdots U_p$ where $U_j \sim \chi^2_{m-p+j}, j=1,\cdots,p$; $U_1,\cdots,U_p$ mutually independent and for a matrix $\mathbf{M}$, $|\mathbf{M}|$ denotes the determinant of $\mathbf{M}$. Thus, \begin{flalign*} &\qquad \sup_{x\in\R}\left| \P \left( \sqrt{\frac{n}{2p}} \left( \sum_{i=1}^p \text{log}\left( \frac{\lambda_i^{(n)}}{\lambda_i}\right) - \sum_{i=1}^p\text{log} \hspace{0.1cm} (n-p+i) \right) \leqslant x \right) - \Phi(x) \right| &&\\ &\quad= \sup_{x \in \R} \left| \P \left( \sqrt{\frac{n}{2}}\left( \sum_{i=1}^p \log \left( \frac{U_i}{n-p+i} \right) \right) \leqslant \sqrt{p}x \right) - \Phi(x) \right| &&\\ &\quad= \sup_{x \in \R} \left| \P \left( \sqrt{\frac{n}{2}}\left( \sum_{i=1}^p \log \left( \frac{U_i}{n-p+i} \right) \right) \leqslant x \right) - \Phi\left(\frac{x}{\sqrt{p}}\right) \right| &&\\ &\quad= \sup_{x \in \R} \left| \P \left( \sqrt{\frac{n}{2}}\left( \sum_{i=1}^p \log \left( \frac{U_i}{n-p+i} \right) \right) \leqslant x \right) - \P(Y_1+\cdots+Y_p \leqslant x) \right| \hspace{0.2cm} ,Y_1,\cdots,Y_p \overset{iid}{\sim} N(0,1) &&\\ &\quad \leqslant \sum_{i=1}^p \sup_{x\in \R}\left| \P \left( \sqrt{\frac{n}{2}}\left( \sum_{i=1}^p \log \left( \frac{U_i}{n-p+i} \right) \right) \leqslant x \right) - \Phi(x) \right| \hspace{0.2cm} (\text{by \textbf{Lemma 1}}) &&\\ &\quad \leqslant C \sum_{i=1}^p \frac{1}{\sqrt{n-p+i}} \hspace{0.2cm} (\text{by \textbf{Lemma 3}}) &&\\ &\quad = O\left(\frac{p}{\sqrt{n}}\right) \end{flalign*} Hence \textbf{Theorem 1} follows. $\blacksquare$ \end{document} @article{wishart1928generalised, title={The generalised product moment distribution in samples from a normal multivariate population}, author={Wishart, John}, journal={Biometrika}, pages={32--52}, year={1928}, publisher={JSTOR} } @book{mardia2024multivariate, title={Multivariate analysis}, author={Mardia, Kanti V and Kent, John T and Taylor, Charles C}, volume={88}, year={2024}, publisher={John Wiley \& Sons} } @book{muirhead2009aspects, title={Aspects of multivariate statistical theory}, author={Muirhead, Robb J}, year={2009}, publisher={John Wiley \& Sons} } @article{tulino2004random, title={Random matrix theory and wireless communications}, author={Tulino, Antonia M and Verd{\'u}, Sergio and others}, journal={Foundations and Trends{\textregistered} in Communications and Information Theory}, volume={1}, number={1}, pages={1--182}, year={2004}, publisher={Now Publishers, Inc.} } @book{couillet2022random, title={Random matrix methods for machine learning}, author={Couillet, Romain and Liao, Zhenyu}, year={2022}, publisher={Cambridge University Press} } @article{james2014concise, title={Concise probability distributions of eigenvalues of real-valued Wishart matrices}, author={James, Oliver and Lee, Heung-No}, journal={arXiv preprint arXiv:1402.6757}, year={2014} } @article{marchenko1967distribution, title={Distribution of eigenvalues for some sets of random matrices}, author={Marchenko, Vladimir Alexandrovich and Pastur, Leonid Andreevich}, journal={Matematicheskii Sbornik}, volume={114}, number={4}, pages={507--536}, year={1967}, publisher={Russian Academy of Sciences, Steklov Mathematical Institute of Russian~…} } @article{bai1988convergence, title={Convergence to the semicircle law}, author={Bai, Zhidong D and Yin, Yong Q}, journal={The Annals of Probability}, volume={16}, number={2}, pages={863--875}, year={1988}, publisher={Institute of Mathematical Statistics} } @article{yin1987limit, title={Limit Theorems for the Eigenvalues of a Product of Large Dimensional Random Matrices When the Underlying Distribution is Isotropic}, author={Yin, Yong-Quan and Krishnaiah, Paruchuri Rama}, journal={Theory of Probability \& Its Applications}, volume={31}, number={2}, pages={342--346}, year={1987}, publisher={SIAM} } @article{hui2010limiting, title={Limiting spectral distribution for large sample covariance matrices with m-dependent elements}, author={Hui, Jun and Pan, Guangming}, journal={Communications in Statistics—Theory and Methods}, volume={39}, number={6}, pages={935--941}, year={2010}, publisher={Taylor \& Francis} } @article{wei2016limiting, title={The limiting spectral distribution for large sample covariance matrices with unbounded m-dependent entries}, author={Wei, Meng and Yang, Guangyu and Yang, Lingling}, journal={Communications in Statistics-Theory and Methods}, volume={45}, number={22}, pages={6651--6662}, year={2016}, publisher={Taylor \& Francis} } @article{hofmann2008wigner, title={Wigner theorems for random matrices with dependent entries: ensembles associated to symmetric spaces and sample covariance matrices}, author={Hofmann-Credner, Katrin and Stolz, Michael}, year={2008} } @article{friesen2013gaussian, title={Gaussian fluctuations for sample covariance matrices with dependent data}, author={Friesen, Olga and L{\"o}we, Matthias and Stolz, Michael}, journal={Journal of Multivariate Analysis}, volume={114}, pages={270--287}, year={2013}, publisher={Elsevier} } @article{gotze2006limit, title={Limit theorems for spectra of positive random matrices under dependence}, author={Gotze, F and Tikhomirov, A}, journal={Journal of Mathematical Sciences}, volume={133}, pages={1257--1276}, year={2006}, publisher={Springer} } @article{yao2012note, title={A note on a Mar{\v{c}}enko--Pastur type theorem for time series}, author={Yao, Jianfeng}, journal={Statistics \& probability letters}, volume={82}, number={1}, pages={22--28}, year={2012}, publisher={Elsevier} } @article{silverstein1995strong, title={Strong convergence of the empirical distribution of eigenvalues of large dimensional random matrices}, author={Silverstein, Jack W}, journal={Journal of Multivariate Analysis}, volume={55}, number={2}, pages={331--339}, year={1995}, publisher={Elsevier} } @article{bao2012strong, title={Strong convergence of ESD for the generalized sample covariance matrices when p/n→ 0}, author={Bao, Zhigang}, journal={Statistics \& Probability Letters}, volume={82}, number={5}, pages={894--901}, year={2012}, publisher={Elsevier} } @book{billingsley2013convergence, title={Convergence of probability measures}, author={Billingsley, Patrick}, year={2013}, publisher={John Wiley \& Sons} } @article{paul2014random, title={Random matrix theory in statistics: A review}, author={Paul, Debashis and Aue, Alexander}, journal={Journal of Statistical Planning and Inference}, volume={150}, pages={1--29}, year={2014}, publisher={Elsevier} } @article{yin1984limit, title={On limit of the largest eigenvalue of the large dimensional sample covariance matrix}, author={Yin, YQ and Bai, ZD and Krishnaiah, PR}, year={1984} } @incollection{bai2008limit, title={Limit of the smallest eigenvalue of a large dimensional sample covariance matrix}, author={Bai, Zhi-Dong and Yin, Yong-Qua}, booktitle={Advances In Statistics}, pages={108--127}, year={2008}, publisher={World Scientific} } @article{johnstone2001distribution, title={On the distribution of the largest eigenvalue in principal components analysis}, author={Johnstone, Iain M}, journal={The Annals of statistics}, volume={29}, number={2}, pages={295--327}, year={2001}, publisher={Institute of Mathematical Statistics} } @article{tracy1996orthogonal, title={On orthogonal and symplectic matrix ensembles}, author={Tracy, Craig A and Widom, Harold}, journal={Communications in Mathematical Physics}, volume={177}, pages={727--754}, year={1996}, publisher={Springer} } @article{karoui2003largest, title={On the largest eigenvalue of Wishart matrices with identity covariance when n, p and p/n tend to infinity}, author={Karoui, Noureddine El}, journal={arXiv preprint math/0309355}, year={2003} } @article{wachter1980limiting, title={The limiting empirical measure of multiple discriminant ratios}, author={Wachter, Kenneth W}, journal={The Annals of Statistics}, pages={937--957}, year={1980}, publisher={JSTOR} } @article{yin1983limit, title={A limit theorem for the eigenvalues of product of two random matrices}, author={Yin, Yong Q and Krishnaiah, Paruchuri R}, journal={Journal of Multivariate Analysis}, volume={13}, number={4}, pages={489--507}, year={1983}, publisher={Elsevier} } @article{yin1986limiting, title={Limiting spectral distribution for a class of random matrices}, author={Yin, Yong Q}, journal={Journal of multivariate analysis}, volume={20}, number={1}, pages={50--68}, year={1986}, publisher={Elsevier} } @article{yin1983limiting, title={Limiting behavior of the eigenvalues of a multivariate F matrix}, author={Yin, YQ and Bai, ZD and Krishnaiah, PR}, journal={Journal of multivariate analysis}, volume={13}, number={4}, pages={508--516}, year={1983}, publisher={Elsevier} } @article{bai1988limiting, title={On the limiting empirical distribution function of the eigenvalues of a multivariate F matrix}, author={Bai, Zhidong D and Yin, Yong Q and Krishnaiah, Paruchuri R}, journal={Theory of Probability \& Its Applications}, volume={32}, number={3}, pages={490--500}, year={1988}, publisher={SIAM} } @article{han2016tracy, title={The Tracy--Widom law for the largest eigenvalue of F type matrices}, author={Han, Xiao and Pan, Guangming and Zhang, Bo}, year={2016} } @article{ryan2023detecting, title={Detecting changes in covariance via random matrix theory}, author={Ryan, Sean and Killick, Rebecca}, journal={Technometrics}, volume={65}, number={4}, pages={480--491}, year={2023}, publisher={Taylor \& Francis} } @article{bhattacharjee2016large, title={Large sample behaviour of high dimensional autocovariance matrices}, author={Bhattacharjee, Monika and Bose, Arup}, year={2016} } @book{bai2010spectral, title={Spectral analysis of large dimensional random matrices}, author={Bai, Zhidong and Silverstein, Jack W}, volume={20}, year={2010}, publisher={Springer} } @article{johnstone2006high, title={High dimensional statistical inference and random matrices}, author={Johnstone, Iain M}, journal={arXiv preprint math/0611589}, year={2006} } @book{couillet2011random, title={Random matrix methods for wireless communications}, author={Couillet, Romain and Debbah, Merouane}, year={2011}, publisher={Cambridge University Press} } @article{bai2007signal, title={On the signal-to-interference ratio of CDMA systems in wireless communications}, author={Bai, ZD and Silverstein, Jack W}, year={2007} } @article{nadakuditi2010fundamental, title={Fundamental limit of sample generalized eigenvalue based detection of signals in noise using relatively few signal-bearing and noise-only samples}, author={Nadakuditi, Raj Rao and Silverstein, Jack W}, journal={IEEE Journal of selected topics in Signal Processing}, volume={4}, number={3}, pages={468--480}, year={2010}, publisher={IEEE} } @article{silverstein1992signal, title={Signal detection via spectral theory of large dimensional random matrices}, author={Silverstein, Jack W and Combettes, Patrick L}, journal={IEEE Transactions on Signal Processing}, volume={40}, number={8}, pages={2100--2105}, year={1992}, publisher={IEEE} } @article{hachem2013subspace, title={A subspace estimator for fixed rank perturbations of large random matrices}, author={Hachem, Walid and Loubaton, Philippe and Mestre, Xavier and Najim, Jamal and Vallet, Pascal}, journal={Journal of Multivariate Analysis}, volume={114}, pages={427--447}, year={2013}, publisher={Elsevier} } @article{roy1953heuristic, title={On a heuristic method of test construction and its use in multivariate analysis}, author={Roy, Samarendra Nath}, journal={The Annals of Mathematical Statistics}, volume={24}, number={2}, pages={220--238}, year={1953}, publisher={Institute of Mathematical Statistics} } @article{johnstone2017roy, title={Roy’s largest root test under rank-one alternatives}, author={Johnstone, Iain M and Nadler, Boaz}, journal={Biometrika}, volume={104}, number={1}, pages={181--193}, year={2017}, publisher={Oxford University Press} } @article{kritchman2009non, title={Non-parametric detection of the number of signals: Hypothesis testing and random matrix theory}, author={Kritchman, Shira and Nadler, Boaz}, journal={IEEE Transactions on Signal Processing}, volume={57}, number={10}, pages={3930--3941}, year={2009}, publisher={IEEE} } @article{niu2016multiple, title={Multiple change-point detection: a selective overview}, author={Niu, Yue S and Hao, Ning and Zhang, Heping}, journal={Statistical Science}, pages={611--623}, year={2016}, publisher={JSTOR} } @article{aminikhanghahi2017survey, title={A survey of methods for time series change point detection}, author={Aminikhanghahi, Samaneh and Cook, Diane J}, journal={Knowledge and information systems}, volume={51}, number={2}, pages={339--367}, year={2017}, publisher={Springer} } @inproceedings{banerjee2015non, title={Non-parametric quickest change detection for large scale random matrices}, author={Banerjee, Taposh and Firouzi, Hamed and Hero, Alfred O}, booktitle={2015 IEEE International Symposium on Information Theory (ISIT)}, pages={146--150}, year={2015}, organization={IEEE} } @article{hero2012hub, title={Hub discovery in partial correlation graphs}, author={Hero, Alfred and Rajaratnam, Bala}, journal={IEEE Transactions on Information Theory}, volume={58}, number={9}, pages={6064--6078}, year={2012}, publisher={IEEE} } @article{avanesov2018change, title={Change-point detection in high-dimensional covariance structure}, author={Avanesov, Valeriy and Buzun, Nazar}, year={2018} } @article{wang2017optimal, title={Optimal covariance change point localization in high dimension}, author={Wang, Daren and Yu, Yi and Rinaldo, Alessandro}, journal={arXiv preprint arXiv:1712.09912}, year={2017} } @article{bhattacharjee2016large, title={Large sample behaviour of high dimensional autocovariance matrices}, author={Bhattacharjee, Monika and Bose, Arup}, year={2016} } @book{finn1974general, title={A general model for multivariate analysis.}, author={Finn, Jeremy D}, year={1974}, publisher={Holt, Rinehart \& Winston} } @article{wilks1932certain, title={Certain generalizations in the analysis of variance}, author={Wilks, Samuel S}, journal={Biometrika}, pages={471--494}, year={1932}, publisher={JSTOR} } @article{lawley1938generalization, title={A generalization of Fisher's z test}, author={Lawley, Derrick N}, journal={Biometrika}, volume={30}, number={1/2}, pages={180--187}, year={1938}, publisher={JSTOR} } @article{potthoff1964generalized, title={A generalized multivariate analysis of variance model useful especially for growth curve problems}, author={Potthoff, Richard F and Roy, Samarendra N}, journal={Biometrika}, volume={51}, number={3-4}, pages={313--326}, year={1964}, publisher={Oxford University Press} } @article{haynes2013bonferroni, title={Bonferroni correction}, author={Haynes, Winston}, journal={Encyclopedia of systems biology}, pages={154--154}, year={2013}, publisher={Springer New York} } @article{scott1974cluster, title={A cluster analysis method for grouping means in the analysis of variance}, author={Scott, Andrew Jhon and Knott, Martin}, journal={Biometrics}, pages={507--512}, year={1974}, publisher={JSTOR} } @article{olshen2004circular, title={Circular binary segmentation for the analysis of array-based DNA copy number data}, author={Olshen, Adam B and Venkatraman, E Seshan and Lucito, Robert and Wigler, Michael}, journal={Biostatistics}, volume={5}, number={4}, pages={557--572}, year={2004}, publisher={Oxford University Press} } @article{fryzlewicz2014wild, title={Wild binary segmentation for multiple change-point detection}, author={Fryzlewicz, Piotr}, year={2014} } @article{ma2012accuracy, title={Accuracy of the Tracy--Widom limits for the extreme eigenvalues in white Wishart matrices}, author={Ma, Zongming}, year={2012} } @article{soshnikov2002note, title={A note on universality of the distribution of the largest eigenvalues in certain sample covariance matrices}, author={Soshnikov, Alexander}, journal={Journal of Statistical Physics}, volume={108}, pages={1033--1056}, year={2002}, publisher={Springer} } @article{hastie1995penalized, title={Penalized discriminant analysis}, author={Hastie, Trevor and Buja, Andreas and Tibshirani, Robert}, journal={The Annals of Statistics}, volume={23}, number={1}, pages={73--102}, year={1995}, publisher={Institute of Mathematical Statistics} } @article{telatar1999capacity, title={Capacity of multi-antenna Gaussian channels}, author={Telatar, Emre}, journal={European transactions on telecommunications}, volume={10}, number={6}, pages={585--595}, year={1999}, publisher={Wiley Online Library} } @article{hoyle2003limiting, title={Limiting form of the sample covariance eigenspectrum in PCA and kernel PCA}, author={Hoyle, David and Rattray, Magnus}, journal={Advances in Neural Information Processing Systems}, volume={16}, year={2003} } @article{johnstone2009sparse, title={Sparse principal components analysis}, author={Johnstone, Iain M and Lu, Arthur Yu}, journal={arXiv preprint arXiv:0901.4392}, year={2009} } @article{baik2006eigenvalues, title={Eigenvalues of large sample covariance matrices of spiked population models}, author={Baik, Jinho and Silverstein, Jack W}, journal={Journal of multivariate analysis}, volume={97}, number={6}, pages={1382--1408}, year={2006}, publisher={Elsevier} } @article{paul2007asymptotics, title={Asymptotics of sample eigenstructure for a large dimensional spiked covariance model}, author={Paul, Debashis}, journal={Statistica Sinica}, pages={1617--1642}, year={2007}, publisher={JSTOR} } @article{bai2008large, title={Large sample covariance matrices without independence structures in columns}, author={Bai, Zhidong and Zhou, Wang}, journal={Statistica Sinica}, pages={425--442}, year={2008}, publisher={JSTOR} } @article{pinelis2016optimal, title={Optimal-order bounds on the rate of convergence to normality in the multivariate delta method}, author={Pinelis, Iosif and Molzon, Raymond}, year={2016} } @article{olkin1964multivariate, title={Multivariate beta distributions and independence properties of the Wishart distribution}, author={Olkin, Ingram and Rubin, Herman}, journal={The Annals of Mathematical Statistics}, pages={261--269}, year={1964}, publisher={JSTOR} } @article{perlman1977note, title={A note on the matrix-variate F distribution}, author={Perlman, Michael D}, journal={Sankhy{\=a}: The Indian Journal of Statistics, Series A}, pages={290--298}, year={1977}, publisher={JSTOR} } @article{wax1985detection, title={Detection of signals by information theoretic criteria}, author={Wax, Mati and Kailath, Thomas}, journal={IEEE Transactions on acoustics, speech, and signal processing}, volume={33}, number={2}, pages={387--392}, year={1985}, publisher={IEEE} } @article{anderson1963asymptotic, title={Asymptotic theory for principal component analysis}, author={Anderson, Theodore Wilbur}, journal={The Annals of Mathematical Statistics}, volume={34}, number={1}, pages={122--148}, year={1963}, publisher={Institute of Mathematical Statistics} } @article{pearson1901liii, title={LIII. On lines and planes of closest fit to systems of points in space}, author={Pearson, Karl}, journal={The London, Edinburgh, and Dublin philosophical magazine and journal of science}, volume={2}, number={11}, pages={559--572}, year={1901}, publisher={Taylor \& Francis} } @article{johnson1972analysis, title={An analysis of a two-way model with interaction and no replication}, author={Johnson, Dallas E and Graybill, Franklin A}, journal={Journal of the American Statistical Association}, volume={67}, number={340}, pages={862--868}, year={1972}, publisher={Taylor \& Francis} } @article{nadler2008finite, title={Finite sample approximation results for principal component analysis: A matrix perturbation approach}, author={Nadler, Boaz}, year={2008} } @article{tipping1999probabilistic, title={Probabilistic principal component analysis}, author={Tipping, Michael E and Bishop, Christopher M}, journal={Journal of the Royal Statistical Society Series B: Statistical Methodology}, volume={61}, number={3}, pages={611--622}, year={1999}, publisher={Oxford University Press} } @article{forni2000tthe, title={TThe generalized factor model: identification and estimation}, author={Forni, M and Hallin, M and Lippi, M and Reichlin, L}, journal={U\%"" 3\&}, volume={4}, year={2000} } @article{deb2017asymptotic, title={An asymptotic theory for spectral analysis of random fields}, author={Deb, Soudeep and Pourahmadi, Mohsen and Wu, Wei Biao}, year={2017} }
2412.05805v1
http://arxiv.org/abs/2412.05805v1
Exact Hausdorff dimension of some sofic self-affine fractals
\documentclass[reqno, a4paper,12pt]{amsart} \usepackage[capposition=bottom]{floatrow} \usepackage{euscript,eufrak,verbatim} \usepackage{graphicx} \usepackage[usenames]{color} \usepackage[colorlinks,linkcolor=red,anchorcolor=blue,citecolor=blue]{hyperref} \usepackage{amsmath, mathtools} \usepackage{mathtools} \usepackage{amsthm} \usepackage[all]{xy} \usepackage{amssymb} \usepackage{bm} \usepackage[all]{xy} \usepackage{mathrsfs} \usepackage{amscd} \usepackage{enumitem} \usepackage[numbers]{natbib} \usepackage{mathptmx} \usepackage[T1]{fontenc} \usepackage{lipsum} \def\acts{\curvearrowright} \makeatletter \numberwithin{equation}{section} \setlength{\textheight}{23cm} \setlength{\textwidth}{16cm} \setlength{\oddsidemargin}{0cm} \setlength{\evensidemargin}{0cm} \setlength{\topmargin}{0cm} \renewcommand{\baselinestretch}{1.2} \theoremstyle{cuplain} \newtheorem{main theorem}{Main Theorem} \newtheorem{theorem}{Theorem}[section] \newtheorem{lemma}[theorem]{Lemma} \newtheorem{conjecture}[theorem]{Conjecture} \newtheorem{corollary}[theorem]{Corollary} \newtheorem{proposition}[theorem]{Proposition} \newtheorem{claim}[theorem]{Claim} \newtheorem{sublemma}[theorem]{Sublemma} \newtheorem{property}[theorem]{Property} \newtheorem{subclaim}[theorem]{Subclaim} \newtheorem*{theorem*}{Theorem} \theoremstyle{definition} \newtheorem{definition}[theorem]{Definition} \newtheorem{remark}[theorem]{Remark} \newtheorem{example}[theorem]{Example} \newtheorem{notation}[theorem]{Notation} \newtheorem{assertion}[theorem]{Assertion} \newtheorem{fact}[theorem]{Fact} \newtheorem{problem}[theorem]{Problem} \newtheorem{condition}[theorem]{Condition} \newtheorem{data}[theorem]{Data} \newtheorem{question}[theorem]{Question} \newtheorem*{example*}{Example} \newtheorem*{remark*}{Remark} \newtheoremstyle{break} {\topsep}{\topsep} {\itshape}{} {\bfseries}{} {\newline}{}\theoremstyle{break} \newtheorem{breaktheorem}{Theorem}[section] \newtheorem{breaklemma}[theorem]{Lemma} \newtheorem{breakconjecture}[theorem]{Conjecture} \newtheorem{breakcorollary}[theorem]{Corollary} \newtheorem{breakproposition}[theorem]{Proposition} \newtheorem{breakclaim}[theorem]{Claim} \newtheorem{breakproperty}[theorem]{Property} \newtheoremstyle{break} {\topsep}{\topsep} {\normalshape}{} {\bfseries}{} {\newline}{}\newtheorem{breakdefinition}[theorem]{Definition} \newtheorem{breakremark}[theorem]{Remark} \newtheorem{breakexample}[theorem]{Example} \newtheorem{breaknotation}[theorem]{Notation} \newtheorem{breakassertion}[theorem]{Assertion} \newtheorem{breakfact}[theorem]{Fact} \newtheorem{breakproblem}[theorem]{Problem} \newtheorem{breakcondition}[theorem]{Condition} \newtheorem{breakquestion}[theorem]{Question} \numberwithin{equation}{section} \renewcommand{\proofname}{Proof.} \newcommand{\spa}{\hspace{1pt}} \newcommand{\vep}{\varepsilon} \newcommand{\flo}{\mathscr} \newcommand{\diam}{\mathrm{diam}} \DeclarePairedDelimiter{\abs}{\lvert}{\rvert} \DeclarePairedDelimiter{\ceil}{\lceil}{\rceil} \newcommand{\norm}[1]{\left\lVert#1\right\rVert} \newcommand{\setcond}{\hspace{2pt} \middle| \hspace{2pt}} \DeclareFontFamily{U}{stix2bb}{\skewchar\font127 } \DeclareFontShape{U}{stix2bb}{m}{n} {<-> stix2-mathbb}{} \DeclareMathAlphabet{\mathbb}{U}{stix2bb}{m}{n} \begin{document} \newcommand\titlelowercase[1]{\texorpdfstring{\lowercase{#1}}{#1}} \font\mathptmx=cmr12 at 12pt \title[\fontsize{13}{12}\mathptmx {\it{E\titlelowercase{xact} H\titlelowercase{ausdorff dimension of some sofic self-affine fractals}}}]{\Huge E\titlelowercase{xact} H\titlelowercase{ausdorff dimension} \protect{\\[9pt]} \titlelowercase{of some sofic self-affine fractals}} \author[\fontsize{13}{12}\mathptmx {\it{N\titlelowercase{ima} A\titlelowercase{libabaei}}}]{\fontsize{13}{12}\mathptmx Nima Alibabaei} \subjclass{28A80, 28D20, 37B40} \keywords{Self-affine fractals, Sofic systems, Sofic affine-invariant sets, Dynamical systems, Hausdorff dimension} \maketitle \begin{abstract} Previous work has shown that the Hausdorff dimension of sofic affine-invariant sets is expressed as a limit involving intricate matrix products. This limit has typically been regarded as incalculable. However, in several highly non-trivial cases, we demonstrate that the dimension can in fact be calculated explicitly. Specifically, the dimension is expressed as the solution to an infinite-degree equation with explicit coefficients, which also corresponds to the spectral radius of a certain linear operator. Our result provides the first non-trivial calculation of the exact Hausdorff dimension of sofic sets in $\mathbb{R}^3$. This is achieved by developing a new technique inspired by the work of Kenyon and Peres (1998). \end{abstract} \section{Introduction and results} \label{section: introduction} \subsection{Overview} \hfill\\ \quad Self-affine fractals, such as Bedford-McMullen carpets, have been extensively studied. Figure \ref{figure: self-affine fractals} illustrates the construction of these sets. Consider integers $1 < m_1 \leq m_2$, and partition the unit square into $m_1 m_2$ numbers of congruent rectangles (with $m_1 = 2$ and $m_2 = 3$ in Figure \ref{figure: self-affine fractals}). We then select some of these rectangles and repeat the process indefinitely, choosing which rectangles to retain at each step based on the initial selection. This process generalizes to higher Euclidean dimensions, and the resulting fractals are known as {\it{\textbf{self-affine sponges}}}. The Hausdorff dimension and the Minkowski (box) dimension of these sets are well understood \cite{Kenyon--Peres}. The underlying structure of these sets involves the symbolic dynamical systems known as full shifts, an infinite Cartesian product of finite symbols: $\{1, 2, \ldots, n\}^{\mathbb{N}}$. The shift-invariant nature of these systems ensures the affine-invariant (or "fractal") property of the self-affine sponges. \begin{figure}[h!] \includegraphics[width=15.8cm]{BM_carpet_and_sofic_set.eps} \caption{The first few generations of self-affine fractals} \label{figure: self-affine fractals} \end{figure} Next, we consider a subshift, a subset of a full shift that is shift-invariant. Using it, we can define a fractal in Euclidean space in a manner analogous to the construction of self-affine sponges. A particularly interesting class of subshifts is {\it{\textbf{sofic systems}}}, which arise from finite directed graphs. The fractals associated with sofic systems are referred to as {\it{\textbf{sofic sets}}} (see Figure \ref{figure: self-affine fractals}). Sofic sets in $\mathbb{R}^2$ were studied in \cite{Kenyon--Peres: sofic}, where the Hausdorff dimension is expressed as a limit involving complicated matrix products. They introduced clever methods to calculate the Hausdorff dimension from this expression for various examples. Despite these insightful results, no further investigations have been made into the estimate of exact dimension of sofic sets. This paper aims to derive an exact expression for the Hausdorff dimension of certain sofic sets in $\mathbb{R}^2$ and $\mathbb{R}^3$. We address the complexities that arise when the matrices lack the simplifying structures, such as commutativity or shared eigenvectors. By analyzing an example presented in \cite{Kenyon--Peres: sofic}, we introduce a technique we call {\it{tower-decomposition}} to break down the matrix product. For sofic sets in $\mathbb{R}^3$, this description requires operators defined on a space indexed by a tree, phenomena not encountered in planar cases. The Hausdorff dimension will be expressed as a solution to an equation of infinite degree, which coincides with the spectral radius of a certain linear operator. Our result provides the first exact calculation of the Hausdorff dimension for non-trivial sofic sets in $\mathbb{R}^3$, an achievement that has generally been considered highly challenging. \subsection{Results in $\mathbb{R}^2$} \label{subsection: results in 2-dim} \hfill\\ \quad In \S \ref{subsection: results in 2-dim} and \S \ref{subsection: results in 3-dim}, we briefly outline our main results. The precise formulations of the main theorems require some preparation, so here we present only the ``rough versions'' of the statements along with illustrating examples. The rigorous statements and proofs of the theorems will be provided in \S \ref{section: theorems and proofs}, and the detailed calculations for the examples are given in \S \ref{section: examples}. We now turn to the construction of sofic sets. Let $I$ be a set of labels. For instance, $I = \{0, 1\} \times \{0, 1, 2\}$ in Figure \ref{figure: self-affine fractals}. We begin by dividing the square into congruent rectangular pieces, each labeled with an element from $I$. These rectangles are then recursively subdivided into smaller rectangles, with the same labeling scheme applied at each stage. This process continues inductively, so that each element of $I^{\mathbb{N}}$ corresponds to a point within the square. It is important to note that this correspondence is not injective. Next, we consider a directed graph $G$ where each edge is labeled with an element from $I$. An infinite path in $G$ corresponds to an element in $I^{\mathbb{N}}$, and we define $S \subset I^{\mathbb{N}}$ as the set of all such paths. The set $S$ is referred to as a sofic system. (We note that an additional condition on $G$ is required, which we omit here.) Via the aforementioned correspondence, $S$ defines a self-affine fractal in Euclidean space, commonly known as a sofic set. An example of such a set, with $I = \{0, 1\} \times \{0, 1, 2, 3, 4\}$, is illustrated in Figure \ref{figure: a sofic set}. (For a precise definition, see \S \ref{section: background}.) \begin{figure}[h!] \includegraphics[width=15cm]{Coincidence_of_dimensions_example.eps} \caption{A sofic set in $\mathbb{R}^2$ with $I = \{0,1\} \times \{0,1,2,3,4\}$} \label{figure: a sofic set} \end{figure} Let $\mathbb{N}_0 = \{0, 1, 2, \ldots\}$ and $\mathrm{M}_n(\mathbb{N}_0)$ be the set of $n \times n$ matrices with components in $\mathbb{N}_0$. Consider integers $1 < m_1 \leq m_2$. Let $I = \{0, 1, \ldots, m_1-1\} \times \{0, 1, \ldots, m_2-1\}$ be the set of indices, and let $X \subset \mathbb{R}^2$ be the resulting sofic set. Let $I_1 = \{0, 1, \ldots, m_1-1\}$. By \cite[Theorem 3.2]{Kenyon--Peres: sofic}, there is an ``adjacency matrix'' $A_i \in \mathrm{M}_n(\mathbb{N}_0)$ for each $i \in I_1$ which yields the following formula of the Hausdorff dimension $\mathrm{dim_H}(X)$. \begin{equation*} \mathrm{dim_H}(X) = \lim_{N \to \infty} \spa \frac{1}{N} \log_{m_1^{}}{ \sum_{ (u_1^{}, \ldots, u_N^{}) \in I_1^{N}} {\norm{A_{u_1^{}} \cdots A_{u_N^{}}} }^{\alpha}}. \end{equation*} Here, $\alpha = \log_{m_2}{m_1}$ and the norm of the matrices is arbitrary. A matrix $A$ is said to be \textbf{primitive} if there is an integer $d$ such that every entry in $A^d$ is positive. Denote by $x^{\top}$ the transpose of a column vector $x \in \mathbb{R}^n$. We say that $A$ has a $1$-dimensional image when its image $\left\{ x^{\top} \spa A \setcond x \in \mathbb{R}^n \right\}$ is spanned by a single non-zero vector. (We consider matrices to be acting on $\mathbb{R}^n$ from right.) The following is the main result for sofic sets in $\mathbb{R}^2$. A rigorous statement of this result is given in Theorem \ref{theorem: tower decomposition for 2d}. \begin{theorem*} Suppose $\sum_{i \in I_1} A_i$ is primitive. Also assume that we have a natural number $L$ and a string $s = (s_1^{}, \ldots, s_L^{}) \in I_1^L$ such that the corresponding matrix $A_{s_1^{}} \cdots A_{s_L^{}}$ has a $1$-dimensional image. Then, there is an ``explicitly calculable'' constant $C_k \geq 0$ for each non-negative integer $k$ such that \[\mathrm{dim}_{\mathrm{H}}(X) = \log_{m_1^{}}{r}, \] where $r$ is the unique positive solution to the equation: \[ r^L = C_0 + \frac{C_1}{r^1} + \frac{C_2}{r^2} + \cdots. \] \end{theorem*} This theorem provides an exact value for the Hausdorff dimension, as illustrated by the following examples. \begin{example*} Let $I = \{0, 1\} \times \{0, 1, 2\}$. Consider the directed graph in Figure \ref{figure: digraph for planar 01} labeled with $I$. \begin{figure}[h!] \includegraphics[width=\textwidth-5.5cm]{sofic_system_01_has_1dim.eps} \caption{A digraph labeled with $I = \{0, 1\} \times \{0, 1, 2\}$} \label{figure: digraph for planar 01} \end{figure} \noindent Then, the adjacency matrices are \begin{equation*} A_0 = \begin{pmatrix} 2 & 0 & 0\\ 0 & 1 & 0\\ 0 & 1 & 0 \end{pmatrix}, \hspace{4pt} A_1 = \begin{pmatrix} 1 & 1 & 1 \\ 0 & 0 & 0 \\ 2 & 1 & 1 \end{pmatrix}. \end{equation*} Let \begin{align*} C_{N, k} = 2^{N-k+1} \Big( ( 2 + 2\sqrt{2} )(1+ \sqrt{2})^k - ( 2 - 2\sqrt{2} ) (1- \sqrt{2})^k \Big). \end{align*} We have $\mathrm{dim}_{\mathrm{H}}(X) = \log_2{r} = 1.6416\cdots$, where $r$ satisfies \begin{equation*} r = \sum_{N =1}^\infty \left( \sum_{k = 0}^N {C_{N, k}}^{\log_3{2}} \right) r^{-N-1} = 3.1201\cdots. \end{equation*} If we denote by $C_N$ the coefficient of $r^{-N}$, then $r$ is also the spectral radius of the operator \begin{equation*} \begin{pmatrix} C_0 & C_1 & C_2 & \cdots \\ 1 & 0 & 0 & \cdots \\ 0 & 1 & 0 & \\ 0 & 0 & 1 & \\ & \vdots & & \ddots \\ \end{pmatrix}. \end{equation*} \end{example*} \spa \begin{example*} Let $G$ be a directed graph with $2$ vertices, and $I = \{0, 1, 2\} \times \{0, 1, 2, 3\}$. Consider a sofic system with the following adjacency matrices. (The digraph $G$ in this example has $19$ edges, so we omit it.) \begin{equation*} A_0 = \begin{pmatrix} 1 & 0 \\ 2 & 0 \\ \end{pmatrix}, \hspace{4pt} A_1 = \begin{pmatrix} 2 & 1 \\ 1 & 2 \\ \end{pmatrix}, \hspace{4pt} A_2 = \begin{pmatrix} 3 & 2 \\ 2 & 3 \\ \end{pmatrix}. \end{equation*} Then, $\mathrm{dim}_{\mathrm{H}}(X) = \log_3{r} = 1.6994\cdots$, where $r$ satisfies \begin{align*} r = \sum_{N=0}^{\infty} \left(\sum_{k = 0}^N \begin{pmatrix} N \\ k \end{pmatrix} \left( \frac{3^k \spa 5^{N-k} -1}{2} \right)^{\log_4{3}} \right) r^{-N} = 6.4693\cdots. \end{align*} \end{example*} \subsection{Results in $\mathbb{R}^3$} \label{subsection: results in 3-dim} \hfill\\ \quad Next, we state our results on sofic sets in $\mathbb{R}^3$. Let $1 < m_1 \leq m_2 \leq m_3$ be natural numbers, and let $I = \{0, 1, \ldots, m_1-1\} \times \{0, 1, \ldots, m_2-1\} \times \{0, 1, \ldots, m_3-1\}$ be the set of indices used to label edges in a directed graph $G$. In a manner analogous to the planar case, we define a sofic set $X \subset \mathbb{R}^3$. Let $I_1 = \{0, 1, \ldots, m_1-1\}$ and $I_2 = \{0, 1, \ldots, m_2-1\}$. It has been proved that there is a matrix $A_{(s,t)} \in M_n(\mathbb{Z}_{\geq 0})$ for each $(s,t) \in I_1 \times I_2$ such that the Hausdorff dimension of $X$ is given by the following expression. (\cite[Theorems 1.1 and 4.1]{Barral--Feng}, \cite[Lemma 2.5]{Z.Feng}) \begin{align} \label{equation: dimension formula for 3d sofic} \mathrm{dim_H}(X) = \lim_{N \to \infty} \spa \frac{1}{N} \log_{m_1}{\hspace{-3pt}\sum_{(s_1^{}, \ldots, s_N^{}) \in I_1^N} \left( \spa \sum_{ (t_1^{}, \ldots, t_N^{}) \in I_{2}^N} \hspace{-2pt} {\norm{A_{(s_1^{},t_1^{})} \cdots A_{(s_N^{},t_N^{})}} }^{a_{2}^{}} \right)^{\hspace{-2pt} a_1^{}}}. \end{align} Here, $a_1 = \log_{m_2}{m_1}$ and $a_2 = \log_{m_3}{m_2}$. We say that a sofic set $X \subset \mathbb{R}^3$ has a {\textbf{recursive structure}} with $\boldsymbol{v} \in \mathbb{R}^n$, if for any $s \in I_1$, there is $t \in I_2$ such that the image of $A_{(s, t)}$ is spanned by $\boldsymbol{v}^{\top}$. (Here, the matrices act on $\mathbb{R}^n$ from right.) Suppose $X$ satisfies this. For each $s \in I_1$, we will introduce (in \S \ref{subsection: tower decomposition for sofic sets in 3d}) a linear operator $M_s$ on a space indexed by a certain tree. The following theorem shows that we can reduce the double summations in equation (\ref{equation: dimension formula for 3d sofic}) to a single one (see Theorem \ref{theorem: tower decomposition for 3d} for the details). \begin{theorem*} Suppose that $X$ has a recursive structure, and that $\sum_{w \in I_1 \times I_2} A_w$ is primitive. Then, \begin{equation*} \mathrm{dim}_{\mathrm{H}}(X) = \lim_{N \to \infty} \spa \frac{1}{N} \log_{m_1^{}}{\hspace{-3pt}\sum_{(s_1^{}, \ldots, s_N^{}) \in I_1^N} {\norm{ M_{s_N^{}} \spa M_{s_{N-1}^{}} \cdots \spa M_{s_1^{}} \spa \Phi_0 }_1 }^{a_1^{}}}, \end{equation*} where $\Phi_0$ is a certain vector in the space indexed by a tree and $\norm{\cdot}_1$ is the $l^1$-norm. \end{theorem*} This expression can sometimes be further simplified (Proposition \ref{proposition: reapplication}), leading to explicit calculations of the dimension. The following examples illustrate such phenomena. \begin{example*} Let $I = \{0, 1\} \times \{0, 1, 2\} \times \{0, 1, 2, 3\}$. Consider the directed graph in Figure \ref{figure: digraph for 3d example} labeled with $I$. \begin{figure}[h!] \includegraphics[width=\textwidth-4cm]{digraph_for_sofic_set_with_calculation_using_infinite_matrix.eps} \caption{A digraph labeled with $I = \{0, 1\} \times \{0, 1, 2\} \times \{0, 1, 2, 3\}$} \label{figure: digraph for 3d example} \end{figure} Then, the adjacency matrices are \begin{equation*} A_{(0, 0)} = \begin{pmatrix} 0 & 0 \\ 0 & 1 \\ \end{pmatrix}, \hspace{4pt} A_{(1, 0)} = \begin{pmatrix} 0 & 1 \\ 0 & 0 \\ \end{pmatrix}, \hspace{4pt} A_{(1, 2)} = \begin{pmatrix} 1 & 0 \\ 1 & 1 \\ \end{pmatrix}. \end{equation*} For a natural number $k$, let \begin{equation*} b_k = \norm{ {\begin{pmatrix} 0 & 1 & 2^{\log_4{3}} & 3^{\log_4{3}} & 4^{\log_4{3}} & \cdots \\ 1 & 0 & 0 & \cdots \\ 0 & 1 & 0 & 0 & \cdots \\ 0 & 0 & 1 & 0 & 0 & \cdots \\ & \vdots & & & \ddots \\ \end{pmatrix}}^k \begin{pmatrix} 1 \\ 0 \\ 0 \\ 0 \\ \vdots \\ \end{pmatrix} }_{1}^{\log_3{2}}. \end{equation*} Then, $\mathrm{dim}_{\mathrm{H}}(X) = \log_2{r} = 1.1950\cdots$, where $r$ satisfies \begin{align*} r = \sum_{k = 0}^{\infty} \frac{b_k}{r^k} &= 1 + \frac{1}{r} + \frac{\sqrt{2}}{r^2} + \frac{ (2 + 2^{\log_4{3}})^{\log_3{2}} }{r^3} + \frac{ (3 + 2^{\log_4{3}} + 3^{\log_4{3}})^{\log_3{2}} }{r^4} + \cdots & \\ & = 2.2894\cdots. & \end{align*} \end{example*} \begin{example*} Let $I = \{0, 1, 2\} \times \{0, 1, 2, 3\} \times \{0, 1, 2, 3, 4\}$. Consider the directed graph in Figure \ref{figure: digraph for 3d example 2} labeled with $I$. \begin{figure}[h!] \includegraphics[width=\textwidth-4cm]{2nd_digraph_for_sofic_set_with_calculation_using_infinite_matrix.eps} \vspace{3pt}\caption{A digraph labeled with $I = \{0, 1, 2\} \times \{0, 1, 2, 3\} \times \{0, 1, 2, 3, 4\}$} \label{figure: digraph for 3d example 2} \end{figure} \\ Then, the adjacency matrices are \begin{equation*} A_{(0, 0)} = \begin{pmatrix} 0 & 1 \\ 0 & 0 \\ \end{pmatrix}, \hspace{4pt} A_{(1, 0)} = \begin{pmatrix} 0 & 0 \\ 0 & 1 \\ \end{pmatrix}, \hspace{4pt} A_{(1, 1)} = \begin{pmatrix} 1 & 0 \\ 1 & 1 \\ \end{pmatrix}, \hspace{4pt} A_{(2, 0)} = \begin{pmatrix} 0 & 0 \\ 0 & 1 \\ \end{pmatrix}, \hspace{4pt} A_{(2, 3)} = \begin{pmatrix} 1 & 0 \\ 2 & 1 \\ \end{pmatrix}. \end{equation*} Let $a_1^{} = \log_4{3}$ and $a_2^{} = \log_5{4}$. Define \begin{align*} \hspace{10pt} b_N &= \sum_{(s_1, \ldots, s_N) \in \{1, 2\}^N} \Bigg( \big( N + \#\left\{ j \setcond 1 \leq j \leq N, s_j = 2\right\} \big)^{a_2^{}} \hspace{20pt} &\\ &\hspace{150pt} + \sum_{k = 1}^{N-1} 2^{k-1} \big( N + \#\left\{j \setcond k \leq j \leq N \text{\hspace{1pt} and \hspace{1pt}} s_j = 2\right\} \big)^{a_2^{}} \Bigg)^{a_1^{}}. & \end{align*} Then, $\mathrm{dim}_{\mathrm{H}}(X) = \log_2{r} = 2.224\cdots$, where $r$ satisfies \begin{align*} r = \sum_{k = 0}^{\infty} \frac{b_k}{r^k} = 4.673\cdots. \end{align*} \end{example*} \begin{remark*} Regarding the dimension theory of sofic sets, \cite{Olivier} discussed the uniqueness of the measure with full dimension in the planar case. Additionally, \cite{Z.Feng} studied the conditions under which general self-affine fractals, including sofic sets, have the same Hausdorff dimension and box dimension. \end{remark*} \section{Background} \label{section: background} This section introduces the definition of sofic systems and sofic sets, and then proceeds to their dimension formula in terms of matrix products. Let $\mathbb{T} = \mathbb{R}/\mathbb{Z}$. We begin by reviewing the definition of sofic systems. Weiss \cite{Weiss} defined {\it{sofic systems}} as subshifts which are factors of shifts of finite type. Using the results by \cite{Boyle--Kitchens--Marcus}, sofic systems can also be characterized in the following way. \begin{definition}[{{\cite[Proposition 3.6]{Kenyon--Peres: sofic}}}] \label{definition: sofic systems} Consider a finite directed graph $G = \langle V, E \rangle$ with possible loops and multiple edges. Let $I$ be a set of labels. Suppose that edges of $G$ are labeled using elements of $I$ in a ``right-resolving'' fashion: every two edges emanating from the same vertex have different labels. Then, the set $S \subset I^{\mathbb{N}}$ of sequences of labels that arise from infinite paths in $G$ is called the \textbf{sofic system}. \end{definition} Fix a positive integer $d$, the Euclidean dimension of the space in which we will be working. Consider natural numbers $1 < m_1 \leq m_2 \leq \cdots \leq m_d$, and define $I_i = \{0, 1, \ldots, m_i-1\}$ for $1 \leq i \leq d$. Let $I = I_1 \times I_2 \times \cdots \times I_d$, and define $R: I^{\mathbb{N}} \rightarrow {\mathbb{T}}^d$ by \begin{equation*} R \left( \left( e^{(n)}_1, e^{(n)}_2, \ldots, e^{(n)}_d \right)_{n=1}^{\infty} \right) = \left( \sum_{n=1}^{\infty} \frac{e^{(n)}_1}{{m_1}^n}, \cdots, \sum_{n=1}^{\infty} \frac{e^{(n)}_d}{{m_d}^n} \right). \end{equation*} Let $S$ be a sofic system with the set of labels $I$. Then, $R(S)$ is a compact set invariant under the map $T_A: {\mathbb{T}}^d \rightarrow {\mathbb{T}}^d$, which is induced from the multiplication of the diagonal matrix $A = \mathrm{diag}(m_1, m_2, \ldots, m_d)$. This set $R(S)$ is referred to as the \textbf{sofic set}. When $d=2$, Kenyon and Peres \cite{Kenyon--Peres: sofic} proved that the Hausdorff dimension of sofic sets can be expressed as the limit of a certain combinatorial sum involving matrix products. This result extends to general $d$, as we shall explain. Suppose $S$ is a sofic system defined from a directed graph $G = \langle V, E \rangle$ with $n$ number of vertices; $V = \{v_1, v_2, \ldots, v_n\}$. Let $W_i = I_1 \times \cdots \times I_i$ for $1 \leq i \leq d-1$, and for each $s \in W_{d-1}$, define the restricted adjacency matrix $A_{s} = \left( a_{ij}(s) \right)_{i,j} \in \mathrm{M}_n(\mathbb{Z}_{\geq 0})$ by \[ a_{ij}(s) = \# \left\{ e \in E \setcond \text{$e$ is from vertex $v_i$ to $v_j$ and its label is $(s, k) \in I$ for some $k \in I_d$} \right\}. \] For a natural number $N$ and $s = (s_1^{}, \ldots, s_N^{}) \in W_i^N$, define \[ W_{i+1}^N(s) = \left\{ s' \in W_{i+1}^N \setcond \text{ There is $(t_1^{}, \ldots, t_N^{}) \in I_{i+1}^N$ with $s' = \big( (s_1^{}, t_1^{}), \ldots, (s_N^{}, t_N^{}) \big)$ } \right\}. \] The following theorem is a special case of \cite[Theorems 1.1 and 4.1]{Barral--Feng}, as explained in \cite[Lemma 2.5]{Z.Feng}. \begin{theorem}[{{\cite[Lemma 2.5]{Z.Feng}}}] \label{theorem: combinatorial formula for the Hausdorff dimension of sofic sets} Let $a_i^{} = \log_{m_{i+1}^{}}{m_i^{}} \hspace{2pt}$ for $ \spa 1 \leq i \leq d-1$. For any sofic system $S \subset I^{\mathbb{N}}$, we have \begin{equation*} \mathrm{dim}_{\mathrm{H}}(R(S)) = \lim_{N \to \infty} \spa \frac{1}{N} \log_{m_1^{}}{\hspace{-5pt}\sum_{s_1^{} \in W_1^N} \hspace{-3pt} \left( \hspace{-1pt} \sum_{s_2^{} \in W_2^N(s_1^{})} \hspace{-3pt} \left( \hspace{-2pt} \cdots \hspace{-2pt} \left( \sum_{\substack{ s_{d-1}^{} \in W_{d-1}^N(s_{d-2}^{}) \\ s_{d-1}^{} = (s^{(1)}, \ldots, s^{(N)})}} \hspace{-12pt} {\norm{ A_{s^{(1)}} \cdots A_{s^{(N)}} } }^{a_{d-1}^{}} \hspace{-2pt} \right)^{\hspace{-2pt} a_{d-2}^{}} \hspace{-14pt} \cdots \right)^{\hspace{-2pt} a_2^{}} \hspace{1pt} \right)^{\hspace{-2pt} a_1^{}}}. \end{equation*} Here, the norm for matrices can be arbitrary. \end{theorem} This theorem can also be proved using a concept called {\it{weighted topological entropy}} as discussed in \cite[Claim 1.6]{Alibabaei}. In \cite{Alibabaei}, the Hausdorff dimension of a sofic set in $\mathbb{T}^3$ is computed, although the example considered there was trivial in the following sense. Suppose that either all the matrices commute and $\sum_{u \in W_{r-1}} A_u$ is primitive, or the matrices share a common eigenvector. In either case, it can be shown that \begin{equation*} \mathrm{dim}_{\mathrm{H}}(R(S)) = \log_{m_1^{}}{\sum_{s_1^{} \in W_1} \left( \sum_{s_2^{} \in W_2(s_1^{})} \left( \cdots \left( \sum_{s_{d-1}^{} \in W_{d-1}(s_{d-2}^{})} {\lambda_{\spa s_{d-1}^{}}}^{a_{d-1}^{}} \right)^{a_{d-2}^{}} \cdots \right)^{ a_2^{}} \right)^{a_1^{}}}, \end{equation*} where $\lambda_{\spa u}$ is the spectral radius of $A_u$. The aim of this paper is to provide sophisticated calculations of the dimension for much more non-trivial cases. For planar cases, the following structure behind the dimension formula is explained after Proposition 3.4 in \cite{Kenyon--Peres: sofic}. Let $\flo{L}$ be an operator acting on the continuous functions on $S^{\spa \# V -1}$ by \[ (\flo{L}f)(x) = \sum_{s = 0}^{m_1^{} -1} \norm{A_s x}^{a_1^{}} f \left( \frac{A_s \spa x}{ \norm{A_s \spa x} } \right). \] Then, we have $\mathrm{dim_H} (X) = \log_{m_1^{}} {\rho(\flo{L})}$, where $\rho(\flo{L})$ is the spectral radius of $\flo{L}$. When one of the matrices has a $1$-dimensional image, the sphere $S^{\spa \#V -1}$ can be replaced with a countable set of directions, resulting in an expression for $\flo{L}$ as an infinite matrix. This structure provides valuable intuition for the arguments in \S \ref{subsection: tower decomposition for planar sofic sets}. In contrast, we were not able to uncover this type of ``bigger picture'' in $3$-dimensional cases due to the nesting of the summations. At present, the technique presented in \S \ref{subsection: tower decomposition for sofic sets in 3d} appears to be a ``fortunate discovery''. Hopefully, a deeper underlying structure will be uncovered in the future. \section{Theorems and proofs} \label{section: theorems and proofs} \subsection{Tower decomposition for planar sofic sets} \label{subsection: tower decomposition for planar sofic sets} \hfill\\ \quad In this section, we generalize the technique introduced by Kenyon and Peres \cite[Example 4.2]{Kenyon--Peres: sofic} to compute the Hausdorff dimension of sofic sets in $\mathbb{T}^2$. Recall that $m_1 \leq m_2$ are natural numbers and $I_1 = \{0, 1, \ldots, m_1-1\}$. Let $X \subset \mathbb{T}^2$ be a sofic set introduced in \S \ref{section: background}. By Theorem \ref{theorem: combinatorial formula for the Hausdorff dimension of sofic sets}, the Hausdorff dimension of $X$ is given by the following expression, where $\alpha = \log_{m_2}{m_1}$. \begin{equation*} \mathrm{dim}_{\mathrm{H}}(X) = \lim_{N \to \infty} \spa \frac{1}{N} \log_{m_1^{}}{ \sum_{ (u_1^{}, \ldots, u_N^{}) \in I_1^{N}} {\norm{A_{u_1^{}} \cdots A_{u_N^{}}} }^{\alpha}}. \end{equation*} We define the norm as the sum of the absolute values of all entries of the matrix. \begin{breakdefinition} (1) A matrix $A$ is said to be \textbf{primitive} if there is an integer $d$ such that every entry in $A^d$ is positive. \noindent(2) For $A \in M_n(\mathbb{R})$, define $\mathrm{Im_R^{}}(A) = \left\{ x^{\top} A \setcond x \in \mathbb{R}^n \right\}$. \end{breakdefinition} For $s \in I_1^L$ with a natural number $L$, define the \textbf{exclusion set} for each natural number $k$ by \[ I_1^k \backslash \{s\} = \left\{ u \in I_1^k \setcond \text{The string $s$ does not appear in $u$} \right\}. \] Finally, let $A = \sum_{i \in I_1} A_i$. We are now ready to state and prove the following main theorem for planar sofic sets. \begin{theorem} \label{theorem: tower decomposition for 2d} Suppose $A$ is primitive. Additionally, assume we have an integer $L > 0$ and a string $s = (s_1^{}, \ldots, s_L^{}) \in I_1^L$ such that there is $\boldsymbol{v} \in \mathbb{R}^n \backslash \{0\}$ with \[ \mathrm{Im_R^{}}( A_{s_1^{}} \cdots A_{s_L^{}} ) = \mathrm{Span}\{ \boldsymbol{v}^\top \}. \] For $u = (u_1^{}, \ldots, u_k^{}) \in I_1^k$, define $J_u \geq 0$ by \[ \boldsymbol{v}^{\top} \spa A_{u_1^{}} \cdots A_{u_{k}^{}} \hspace{2pt} A_{s_1^{}} \cdots A_{s_L^{}} = J_u \spa {\boldsymbol{v}}^{\top}. \] Set $C_k = \sum_{u \in I_1^k \backslash \{s\}} J_u^{\alpha}$ for each non-negative integer $k$. Then, we have \[\mathrm{dim}_{\mathrm{H}}(X) = \log_{m_1^{}}{r}, \] where $r$ is the unique positive solution to the equation \[ r^L = C_0 + \frac{C_1}{r} + \frac{C_2}{r^2} + \cdots. \] \end{theorem} \begin{proof} We break down the summand by constructing what can be described as a tower-decomposition. Specifically, we define a vector $\Phi_N = \left( \Phi_{N, k} \right)_{k = 0}^{\infty}$ for each natural number $N \geq L$, whose norm closely approximates that of the original matrix product. Let $\mathbb{e}$ be the column vector with $1$ in every entry. Let \[ \Phi_{N, N} = \left[ \boldsymbol{v}^{\top} \mathbb{e} \right]^{\alpha}, \] \[ \Phi_{N, N-L} = \left[ \boldsymbol{v}^{\top} \spa A_{s_1^{}} \cdots A_{s_{L}^{}} \spa \mathbb{e} \right]^{\alpha}, \] and when $0 \leq k < N-L$, let \[ \Phi_{N, k} = \sum_{(u_i)_i \in I_1^{N-L-k}} \left[ \boldsymbol{v}^{\top} A_{u_{1}^{}} \ldots A_{u_{N-L-k}^{}} \spa A_{s_1^{}} \cdots A_{s_{L}^{}} \spa \mathbb{e} \right]^{\alpha}. \] Set $\Phi_{N, k} = 0$ for all other $k$. \begin{claim} \label{claim: tower decomposition} We have the following relation: \begin{equation*} \mathrm{dim}_{\mathrm{H}}(X) = \lim_{N \to \infty} \frac{1}{N} \log_{m_1^{}} \norm{ \Phi_N }_1, \end{equation*} where $\norm{\cdot}_1$ is the $l^1$-norm. \end{claim} \begin{proof}[Proof of Claim \ref{claim: tower decomposition}] Since $A = \sum_{i \in I_1} A_i \in \mathrm{M}_n(\mathbb{Z}_{\geq0})$ is primitive, there is an integer $d$ such that every entry of $A^d$ is at least 1. From this, we can conclude that \[ \norm{ \boldsymbol{x}^{\top} }_1 \leq \norm{ \boldsymbol{x}^{\top} \spa A^{d} \spa A_{s_1^{}} \cdots A_{s_L^{}} }_1 \] for every $\boldsymbol{x} \in (\mathbb{R}_{\geq 0})^n$. Also, for each natural number $k$, we can take a string $t^{(k)} = (t_1^{(k)}, \ldots, t_k^{(k)}) \in I_1^k$ such that $A_{t_1^{(k)}} \cdots A_{t_k^{(k)}} \ne O$, again because $A$ is primitive. Then, we have \[ \norm{ \boldsymbol{x}^{\top} }_1 \leq \norm{ \boldsymbol{x}^{\top} \spa A^{d} \spa A_{t_1^{(k)}} \cdots A_{t_k^{(k)}} }_1. \] Using the fact that $(x + y)^{\alpha} \leq x^{\alpha} + y^{\alpha}$ for $x, y \geq 0$ when $0 \leq \alpha \leq 1$, we can make the following evaluation: \begin{align*} \norm{ \Phi_N }_1 & \leq \sum_{k = 0}^{N-L} \hspace{4pt} \sum_{(u_i)_i \in I_1^{N-L-k}} \left[ \boldsymbol{v}^{\top} A_{u_1^{}} \cdots A_{u_{N-L-k}^{}} \spa A_{s_1^{}} \cdots A_{s_L^{}} \spa A^d \spa A_{t_1^{(k)}} \cdots A_{t_k^{(k)}} \spa \mathbb{e} \right]^{\alpha} + \Phi_{N, N} \\ & = \sum_{k = 0}^{N-L} \hspace{4pt} \sum_{(u_i)_i \in I_1^{N-L-k}} \left[ \boldsymbol{v}^{\top} A_{u_1^{}} \cdots A_{u_{N-L-k}^{}} \spa A_{s_1^{}} \cdots A_{s_L^{}} \spa \left( \sum_{q = 0}^{m-1} A_q \right)^d \spa A_{t_1^{(k)}} \cdots A_{t_k^{(k)}} \spa \mathbb{e} \right]^{\alpha} + \left[ \boldsymbol{v}^{\top} \mathbb{e} \right]^{\alpha} \\ & \leq m_1^d \spa N \spa \sum_{(u_i^{})_i \in I_1^{N + d}} {\left[ \boldsymbol{v}^{\top} A_{u_{1}^{}} \cdots A_{u_{N+d}^{}} \mathbb{e} \right] }^{\alpha} + \left[ \boldsymbol{v}^{\top} \mathbb{e} \right]^{\alpha} \\ & \leq m_1^d \spa N \spa \norm{\boldsymbol{v}}_{\infty} \sum_{(u_i^{})_i \in I_1^{N + d}} {\left[ \mathbb{e}^{\top} A_{u_{1}^{}} \cdots A_{u_{N+d}^{}} \mathbb{e} \right] }^{\alpha} + \left[ \boldsymbol{v}^{\top} \mathbb{e} \right]^{\alpha} \\ & \leq m_1^d \spa N \spa \norm{\boldsymbol{v}}_{\infty} \sum_{(u_i^{})_i \in I_1^{N + d}} {\left[ \boldsymbol{v}^{\top} \spa A^{d} \spa A_{u_{1}^{}} \cdots A_{u_{N+d}^{}} \spa A^d \spa A_{s_1^{}} \cdots A_{s_L^{}} \spa \mathbb{e} \right] }^{\alpha} + \left[ \boldsymbol{v}^{\top} \mathbb{e} \right]^{\alpha} \\ & \leq m_1^{3d} \spa N \spa \norm{\boldsymbol{v}}_{\infty} \spa \norm{\Phi_{N+3d+L}}_1. \end{align*} Taking the logarithm and letting $N \rightarrow \infty$, we have \begin{equation*} \lim_{N \to \infty} \frac{1}{N} \log_{m_1^{}} \norm{ \Phi_N }_1 = \lim_{N \to \infty} \spa \frac{1}{N} \log_{m_1^{}}{ \sum_{ (u_1^{}, \ldots, u_N^{}) \in I_1^{N}} {\norm{A_{u_1^{}} \cdots A_{u_N^{}}} }^{\alpha}}. \end{equation*} \end{proof} Now, the vector $\boldsymbol{v}^{\top} A_{u_{1}^{}} \ldots A_{u_{N-L-k}^{}} \spa A_{s_1^{}} \cdots A_{s_{L}^{}}$ in the definition of $\Phi_{N,k}$ is already a constant multiple of $\boldsymbol{v}^{\top}$. Therefore, for any $L-1 \leq k \leq N-L$ we have \begin{flalign*} &\; & \sum_{(w_1^{}, \ldots, w_{k-L+1}^{}) \in I_1^{k-L+1} \backslash \{s\}} \hspace{2pt} \sum_{(u_i)_i \in I_1^{N-L-k}} \left[ \boldsymbol{v}^{\top} \spa A_{u_{1}^{}} \ldots A_{u_{N-L-k}^{}} \spa A_{s_1^{}} \cdots A_{s_{L}^{}} \spa A_{w_1^{}} \cdots A_{w_{k-L+1}^{}} \spa A_{s_1^{}} \cdots A_{s_{L}^{}} \spa \mathbb{e} \right]^{\alpha} & \end{flalign*} \begin{flalign*} & \hspace{15pt} = \sum_{w \in I_1^{k-L+1} \backslash \{s\}} J_w^{\alpha} \Phi_{N, k} & \\ & \hspace{15pt} = C_{k-L+1} \Phi_{N, k}.& \end{flalign*} Also, \begin{flalign*} &\sum_{(w_1^{}, \ldots, w_{N-L+1}^{}) \in I_1^{N-L+1} \backslash \{s\}} \hspace{2pt} \left[ \boldsymbol{v}^{\top} \spa A_{w_1^{}} \cdots A_{w_{N-L+1}^{}} \spa A_{s_1^{}} \cdots A_{s_{L}^{}} \spa \mathbb{e} \right]^{\alpha} = \sum_{w \in I_1^{N-L+1} \backslash \{s\}} J_w^{\alpha} \Phi_{N, N} & \end{flalign*} \begin{flalign*} & \hspace{273.5pt} = C_{N-L+1} \Phi_{N, N}. & \end{flalign*} We thus conclude that \[ \Phi_{N+1, 0} = \sum_{k = 0}^{\infty} \spa C_{k} \spa \Phi_{N, k+L-1}. \] Also, we have $\Phi_{N+1, k+1} = \Phi_{N, k}$. Thus, we can express the sum in question using a linear operator as follows. Let $\mathbb{R}^{\bigoplus \mathbb{N}_0}$ denote the direct sum of $\mathbb{R}$ indexed by $\mathbb{N}_0$; an element in this direct sum has only finitely many non-zero entries. Define $M: \mathbb{R}^{\bigoplus \mathbb{N}_0} \rightarrow \mathbb{R}^{\bigoplus \mathbb{N}_0}$ by \\[-6pt] \begin{equation*} \begin{matrix} \phantom{ \begin{matrix} \overbrace{ \hphantom{\begin{matrix} M \end{matrix}} }^{\text{$L$}} \end{matrix} \phantom{ \begin{matrix} M \end{matrix}}}\\ M = \end{matrix} \begin{matrix} \begin{matrix} \overbrace{ \hphantom{\begin{matrix} 0 & 0 & 0 & \cdots & 0 \end{matrix}} }^{\text{$(L-1)$ columns}} \end{matrix} \phantom{ \begin{matrix} 0 & C_0 & C_1 & C_2 & \cdots \end{matrix}}\\ \begin{pmatrix} 0 & 0 & 0 & \cdots & 0 & C_0 & C_1 & C_2 & \cdots \\ 1 & 0 & 0 & \cdots & & 0 & 0 & \cdots \\ 0 & 1 & 0 & 0 & \cdots \\ 0 & 0 & 1 & 0 & \cdots \\ & \vdots & & \ddots \\ \end{pmatrix}. \end{matrix} \end{equation*} \\[4pt] Then, $\Phi_{N+1} = M \Phi_N$. This yields for $N > L$, \[ \norm{\Phi_N}_1 = \norm{M^{N-L} \spa \Phi_L }_1. \] Thus, \[ \mathrm{dim}_{\mathrm{H}}(X) = \lim_{N \to \infty} \log_{m_1^{}}{ \norm{M^{N-L} \spa \Phi_L}_1^{\frac{1}{N}} }. \] Let $r$ be the unique solution to the following equation (which exists since $C_k$ grows at most exponentially). \begin{equation*} r = \sum_{k = 0}^{\infty} C_k r^{-k-L+1}. \end{equation*} Define $\boldsymbol{b} \in \mathbb{R}^{\mathbb{N}}$ by $\boldsymbol{b} = (1, r^{-1}, r^{-2}, \spa \ldots)^{\top}$. We see that $M \boldsymbol{b}$ is well-defined and $M \boldsymbol{b} = r\boldsymbol{b}$, which implies \[ \norm{ M^{N-L} \spa \Phi_L }_1 \leq r^L \spa \norm{\Phi_L}_1 \spa \norm{ M^{N-L} \spa \boldsymbol{b} }_1 = \norm{\Phi_L}_1 \spa \norm{ \boldsymbol{b} }_1 \spa r^N. \] The other direction is done via an application of Perron-Frobenius theorem. Let $M_k$ be the upper $k \times k$ submatrix of $M$. By the Perron-Frobenius theorem, \[ \liminf_{N \to \infty} \norm{ M^{N-L} \spa \Phi_L }_1^{\frac{1}{N}} \geq \liminf_{N \to \infty} \norm{ M_k^{N-L} \spa \Phi_L^{(k)} }_1^{\frac{1}{N}} = r_k. \] Here, $r_k$ is the spectral radius of $M_k$ which satisfies \begin{equation*} r_k = \sum_{j = 0}^{k} C_j {r_k}^{-j-L+1}, \end{equation*} Since $r_k \to r$ as $k \to \infty$, we obtain \[ \mathrm{dim}_{\mathrm{H}}(X) = \log_{m_1^{}}{r}. \qedhere \] \end{proof} \begin{remark} For comparison with the operator appearing in the next section \S\ref{subsection: tower decomposition for sofic sets in 3d} (Figure \ref{figure: operator in 3d}), we look at the operator $M$ as the sum of a shift operator and a ``return'' map. See Figure \ref{figure: operator in 2d}. \begin{figure}[h!] \includegraphics[width=\textwidth-2.5cm]{operator_explanation_2d.eps} \vspace{3pt}\caption{A description of the operator $M$ when $L = 1$} \label{figure: operator in 2d} \end{figure} \end{remark} \subsection{Tower decomposition for sofic sets in $\mathbb{R}^3$} \label{subsection: tower decomposition for sofic sets in 3d} \hfill \\ \quad Let $m_1 \leq m_2 \leq m_3$ be natural numbers, and consider a sofic set $X \subset \mathbb{T}^3$ introduced in \S \ref{section: background}. Let $I_1 = \{0, 1, \ldots, m_1-1\}$ and $I_2 = \{0, 1, \ldots, m_2-1\}$. By Theorem \ref{theorem: combinatorial formula for the Hausdorff dimension of sofic sets}, for each pair $(s,t) \in I_1 \times I_2$, there is a matrix $A_{(s,t)} \in M_n(\mathbb{Z}_{\geq 0})$ such that the Hausdorff dimension of $X$ can be expressed as \begin{align*} \mathrm{dim}_{\mathrm{H}}(X) = \lim_{N \to \infty} \spa \frac{1}{N} \log_{m_1^{}}{\hspace{-3pt}\sum_{(s_1^{}, \ldots, s_N^{}) \in I_1^N} \left( \spa \sum_{ (t_1^{}, \ldots, t_N^{}) \in I_{2}^N} \hspace{-2pt} {\norm{A_{(s_1^{},t_1^{})} \cdots A_{(s_N^{},t_N^{})}} }^{a_{2}^{}} \right)^{\hspace{-2pt} a_1^{}}}. \end{align*} Here, $a_1^{} = \log_{m_2^{}}{m_1^{}}$ and $a_2^{} = \log_{m_3^{}}{m_2^{}}$. \begin{definition} We say that a sofic set $X \subset \mathbb{T}^3$ has a {\textbf{recursive structure}} with $\boldsymbol{v} \in \mathbb{R}^n$ if for any $s \in I_1$ there is $t \in I_2$ with $\mathrm{Im_R^{}}( A_{(s,t)} ) = \mathrm{Span} \{ \boldsymbol{v}^{\top} \}$. When this condition is satisfied, we define for each $u \in I_1$ \[ P(u) = \left\{ t \in I_2 \setcond \text{ Either $A_{(u, t)} = O$ or $\mathrm{Im_R^{}}( A_{(u, t)} ) = \mathrm{Span} \{ \boldsymbol{v}^{\top} \}$} \right\}. \] We say that $u \in I_1$ is \textbf{removable} if $P(u) = I_2$. \end{definition} In the rest of this subsection, we assume that a sofic set $X$ has a recursive structure with $\boldsymbol{v}$. Define $J$ to be the collection of $u \in I_1$ that are not removable. As before, for $s \in I_1^N$ let $W_2^N(s) = \left\{ w \in (I_1 \times I_2)^N \setcond \text{The projection of $w$ onto $I_1^N$ is $s$.} \right\}$. Also, let $Q(u) = \{ u \} \times P(u)$ for $u \in I_1$. Take $u \in I_1$. Define the constant $D_w(p) \geq 0$ for $w = (w_1^{}, \ldots, w_N^{}) \in (I_1 \times I_2)^N$ and $p \in Q(u)$ by \[ \boldsymbol{v}^{\top} \spa A_{w_1^{}} \cdots A_{w_N^{}} \spa A_p = D_w(p) \boldsymbol{v}^{\top}, \] and $D_{\varnothing}(p)$ by $\boldsymbol{v}^{\top} \spa A_p = D_{\varnothing}(p) \boldsymbol{v}^{\top}.$ Define $C_{\varnothing}(u) \geq 0$ by \[ C_{\varnothing}(u) = \sum_{p \in Q(u)} {D_{\varnothing}(p)}^{a_2}, \] and $C_s(u) \geq 0$ for $s \in J^N$ by \[ C_s(u) = \sum_{p \in Q(u)} \spa \spa \sum_{w \in R^N(s)} {D_w(p)}^{a_2}, \] where $R^N(s) = \left\{ (s_i^{}, t_i^{})_i \in (I_1 \times I_2)^N \setcond \text{$t_i^{} \notin P(s_i^{})$ for each $1 \leq i \leq N$} \right\}$. Let $\Gamma$ be the set of all finite words formed from the letters in $J$, including the null word, which we interpret as a rooted tree: \[ \Gamma = \{\varnothing\} \cup \bigcup_{k = 1}^\infty J^k. \] Let $\mathbb{R}^{\bigoplus \Gamma}$ denote the direct sum of $\mathbb{R}$ indexed by $\Gamma$, where an element has only finitely many non-zero entries. For $u \in J$, define a linear operator $M_u: \mathbb{R}^{\bigoplus \Gamma} \rightarrow \mathbb{R}^{\bigoplus \Gamma}$ as the sum of a directed shift and a return map (see Figure \ref{figure: operator in 3d}): \begin{equation*} \left( M_u \left( (x_{\mu})_{\mu \in \Gamma} \right) \right)_\lambda = \begin{dcases} \sum_{s \in \Gamma} C_s(u) x_s \text{\quad ( if $\lambda = \varnothing$. )} \\ x_{\lambda'}^{} \text{\quad ( if $\lambda = \lambda' \spa u$, the concatenation of $\lambda'$ and $u$. )} \\ 0 \text{\quad ( otherwise. )} \end{dcases} \end{equation*} \begin{figure}[h!] \includegraphics[width=\textwidth-1.5cm]{operator_in_3d_shift_and_return_M0.eps} \vspace{3pt}\caption{A description of the operator $M_u$ when $I_1 =\{0, 1\}$ and $u = 0 \in J$} \label{figure: operator in 3d} \end{figure} When $u$ is removable ($u \notin J$), $M_u$ is defined similarly, although without the shift operator as \begin{equation*} \left( M_u \left( (x_{\mu})_{\mu \in \Gamma} \right) \right)_\lambda = \begin{dcases} \sum_{s \in \Gamma} C_s(u) x_s \text{\quad ( if $\lambda = \varnothing$. )} \\ 0 \text{\quad ( otherwise. )} \end{dcases} \end{equation*} Define $\Phi_0 \in \mathbb{R}^{\Gamma}$ by $(\Phi_0)_{\varnothing} = 1$ and $0$ elsewhere. Let $A = \sum_{w \in I_1 \times I_2} A_w$. \begin{theorem} \label{theorem: tower decomposition for 3d} Suppose that $X$ has a recursive structure with $\boldsymbol{v} \in \mathbb{R}^n$, and that $A$ is primitive. Then, \begin{equation*} \mathrm{dim}_{\mathrm{H}}(X) = \lim_{N \to \infty} \spa \frac{1}{N} \log_{m_1^{}}{\hspace{-3pt}\sum_{(u_1^{}, \ldots, u_N^{}) \in I_1^N} {\norm{ M_{u_N^{}} \spa M_{u_{N-1}^{}} \cdots \spa M_{u_1^{}} \spa \Phi_0 }_1 }^{a_1^{}}}. \end{equation*} \end{theorem} \begin{proof} The proof proceeds by constructing a tower decomposition of the summand over the tree $\Gamma$ and expressing it as a composition of linear operators $M_u$. Let $N$ be a natural number and $s = (s_1^{}, \ldots, s_N^{}) \in I_1^N$. We begin by defining $\Phi_N(s) \in \mathbb{R}^{\Gamma}$. For a natural number $k$, define $s[k]$ to be the last $k$ components of $s$ from the end, that is, $s[k] = (s_{N-k+1}^{}, \ldots, s_{N}^{})$. Also, $s|_k$ is the first $k$ components; $s|_k = (s_1^{}, \ldots, s_k^{})$. Set $s[0] = \varnothing$. Let \[ \left( \Phi_N(s) \right)_{s} = \left[ \boldsymbol{v}^{\top} \spa \mathbb{e} \right]^{a_2^{}}, \] \[ \left( \Phi_N(s) \right)_{s[N-1]} = \sum_{p \in Q(s_{1}^{})} \hspace{2.5pt} \left[ \boldsymbol{v}^{\top} \spa A_p \spa \mathbb{e} \right]^{a_2^{}}, \] and for $0 \leq k \leq N-2$, \[ \left( \Phi_N(s) \right)_{s[k]} = \sum_{p \in Q(s_{N-k}^{})} \hspace{2.5pt} \sum_{(w_i^{})_i \in W^{N-k-1}(s|_{N-k-1})} \left[ \boldsymbol{v}^{\top} \spa A_{w_1^{}} \cdots A_{w_{N-k-1}^{}} \spa A_{p} \spa \mathbb{e} \right]^{a_2^{}}. \] Here, $\mathbb{e}$ denotes the column vector with $1$ in every entry. We define all other entries of $\Phi_N(s)$ to be $0$. $\big($ Although the above definition depends only on $k$ and not on $s[k]$, we will need this distinction for later discussion.$\big)$ \begin{claim} \label{claim: tower decomposition for 3d} We have the following relation: \begin{equation*} \mathrm{dim}_{\mathrm{H}}(X) = \lim_{N \to \infty} \frac{1}{N} \log_{m_1^{}} \sum_{s \in I_1^N} \norm{ \Phi_N(s) }_1^{a_1}. \end{equation*} \end{claim} \begin{proof}[Proof of Claim \ref{claim: tower decomposition for 3d}] Since $A = \sum_{w \in I_1 \times I_2} A_w \in \mathrm{M}_n(\mathbb{Z}_{\geq0})$ is primitive, there is an integer $d$ such that every entry of $A^d$ is at least 1. For a natural number $k$, take a string $w^{(k)} = (w_1^{(k)}, \ldots, w_k^{(k)}) \in I_2^k$ such that $A_{w_1^{(k)}} \cdots A_{w_k^{(k)}} \ne O$. Then, \[ \norm{\boldsymbol{x}^{\top} }_1 \leq \norm{ \boldsymbol{x}^{\top} \spa A^{d} \spa A_{w_1^{(k)}} \cdots A_{w_k^{(k)}} }_1 \] for every $\boldsymbol{x} \in (\mathbb{R}_{\geq 0})^n$. Using the fact that $(x + y)^{\alpha} \leq x^{\alpha} + y^{\alpha}$ for $x, y \geq 0$ when $0 \leq \alpha \leq 1$, we can evaluate as follows. \begin{flalign*} & \sum_{s \in I_1^N} \norm{ \Phi_N(s) }_1^{a_1}& \end{flalign*} \begin{flalign*} &\leq \sum_{s \in I_1^N} \hspace{-2pt} \left( \hspace{-2pt} (\Phi_N(s))_s + \sum_{k = 0}^{N-1} \hspace{0.5pt} \sum_{p \in Q(s_{N-k}^{})} \hspace{1pt} \sum_{\substack{ (w_i)_i \spa \in \\ W_2^{N-k-1}(s|_{N-k-1}) }} \hspace{-1pt} \left[ \boldsymbol{v}^{\top} \spa A_{w_1^{}} \cdots A_{w_{N-k-1}^{}} \spa A_{p} \spa A^d \spa A_{w_1^{(k)}} \cdots A_{w_k^{(k)}} \spa \mathbb{e} \right]^{a_2} \right)^{\hspace{-2pt} a_1^{}} & \end{flalign*} \begin{flalign*} &= \sum_{s \in I_1^N} \left( \left[ \boldsymbol{v}^{\top} \spa \mathbb{e} \right]^{a_2^{}} + \sum_{k} \hspace{2pt} \sum_{p} \hspace{2.5pt} \sum_{(w_i)_i} \left[ \boldsymbol{v}^{\top} \spa A_{w_1^{}} \cdots A_{w_{N-k-1}^{}} \spa A_{p} \spa \left( \sum_{w \in I_1 \times I_2} A_w \right)^d \spa A_{w_1^{(k)}} \cdots A_{w_k^{(k)}} \spa \mathbb{e} \right]^{a_2} \hspace{-1pt} \right)^{a_1^{}} & \end{flalign*} \begin{flalign*} & \leq {(m_1m_2)}^{d+1} \spa N \spa \sum_{s \in I_1^{N+d}} \left( \sum_{(w_i)_i \in W_2^{N+d}(s)} {\left[ \boldsymbol{v}^{\top} \spa A_{w_1^{}} \cdots A_{w_{N+d}^{}} \spa \mathbb{e} \right] }^{a_2^{}} \right)^{a_1^{}} & \end{flalign*} \begin{flalign*} & \leq {(m_1m_2)}^{d+1} \spa N \spa \norm{\boldsymbol{v}}_{\infty} \sum_{s \in I_1^{N+d}} \left( \sum_{(w_i)_i \in W_2^{N+d}(s)} { \left[ \mathbb{e}^{\top} \spa A_{w_1^{}} \cdots A_{w_{N+d}^{}} \spa \mathbb{e} \right] }^{a_2^{}} \right)^{a_1^{}} & \end{flalign*} \begin{flalign*} & \leq {(m_1m_2)}^{d+1} \spa N \spa \norm{\boldsymbol{v}}_{\infty} \sum_{s \in I_1^{N+d}} \left( \sum_{p \in Q(0)} \sum_{(w_i)_i \in W_2^{N+d}(s)} { \left[ \boldsymbol{v}^{\top} \spa A^d \spa A_{w_1^{}} \cdots A_{w_{N+d}^{}} \spa A^d \spa A_p \spa \mathbb{e} \right] }^{a_2^{}} \right)^{a_1^{}} & \end{flalign*} \begin{flalign*} & \leq {(m_1m_2)}^{3d+1} \spa N \spa \norm{\boldsymbol{v}}_{\infty} \sum_{s \in I_1^{N+3d+1}} \norm{ \Phi_N(s) }_1^{a_1}. & \end{flalign*} Taking the logarithm and letting $N \rightarrow \infty$ finishes the proof. \end{proof} Now, take $s \in I_1^N$ and $u \in I_1$, and consider the concatenation $su \in I_1^{N+1}$. We see from the definition that for a non-negative integer $k$, \[ \left( \Phi_{N+1}(su) \right)_{s[k] \spa u} = \left( \Phi_{N+1}(su) \right)_{(su)[k+1]} = \left( \Phi_{N}(s) \right)_{s[k]}. \] Also, the vector $\boldsymbol{v}^{\top} \spa A_{w_1^{}} \cdots A_{w_{N-k-1}^{}} \spa A_p$ in the definition of $\Phi_N(s)$ is already a constant multiple of $\boldsymbol{v}^{\top}$. Therefore, we have for any $0 \leq k \leq N$ and $u \in I_1$, \begin{flalign*} &\; & \sum_{q \in Q(u)} \hspace{3.5pt} \sum_{(z_i^{})_i \in R^k(s[k])} \hspace{3.5pt} \sum_{p \in Q(s_{N-k}^{})} \hspace{3.5pt} \sum_{(w_i^{})_i \in W_2^{N-k-1}(s|_{N-k-1})} \left[ \boldsymbol{v}^{\top} A_{w_1^{}} \cdots A_{w_{N-k-1}^{}} \spa A_{p} \spa A_{z_1^{}} \cdots A_{z_k^{}} \spa A_{q} \spa \mathbb{e} \right]^{a_2^{}} & \end{flalign*} \begin{flalign*} & \phantom{\hspace{25pt}} = \sum_{q \in Q(u)} \hspace{3.5pt} \sum_{w \in R^k(s[k])} {D_w(q)}^{a_2} \spa \left( \Phi_N(s) \right)_{s[k]} & \\ & \phantom{\hspace{25pt}} = C_{s[k]}(u) \left( \Phi_N(s) \right)_{s[k]}. & \end{flalign*} Since $W_2^N(s) = \bigcup_{k = 0}^N W_2^{N-k-1}(s|_{N-k-1}) \times Q(s_{N-k}^{}) \times R^k (s[k]) \footnote{Here, the sets $W_2^{-1}$, $Q(s_0^{})$, and $R^0$ are regarded as empty.}$ is a disjoint partition (by considering the last index, say $N-k$, that multiplies $A_p$ with some $p \in Q(s_{N-k}^{})$,) we conclude \[ \left( \Phi_{N+1}(su) \right)_{\varnothing} = \sum_{\lambda \in \Gamma} C_{\lambda}(u) \spa \left( \Phi_N(s) \right)_\lambda. \] Here, we used the fact that $\left( \Phi_N(s) \right)_\lambda = 0$ for all $\lambda$ except for $\lambda = s[k]$ with some $k$. By the definition of $M_u$, we have \[ \Phi_{N+1}(su) = M_u \spa \Phi_N(s). \] We conclude that for any string $s = (s_1^{}, \ldots, s_N^{}) \in I_1^N$, \[ \Phi_N(s) = M_{s_N^{}} \cdots M_{s_1^{}} \spa \Phi_0. \] We combine this with Claim \ref{claim: tower decomposition for 3d} and complete the proof. \end{proof} When there is a removable index $u \in I_1$, the operator $M_u$ has a $1$-dimensional image (since it does not include the ``shift'' operator), leading to an intriguing result similar to the one in Theorem \ref{theorem: tower decomposition for 2d}. \begin{proposition} \label{proposition: reapplication} Suppose that a sofic set $X \subset \mathbb{T}^3$ satisfies the following conditions. \\ (1) $X$ has a recursive structure with $\boldsymbol{v} \in \mathbb{R}^n$, and $A$ is primitive. \\ (2) There is at least one non-removable index in $I_1$\footnote{Otherwise, the dimension is easily determined.}. \\ (3) There is a removable index $u \in I_1$, and a string $(t_1^{}, \ldots, t_L^{}) \in I_1^L$ such that $M_u \spa M_{t_L^{}} \cdots M_{t_1^{}}$ is increasing with respect to the $l^1$-norm\footnote{That is, $\norm{M_u \spa M_{t_L^{}} \cdots M_{t_1^{}} \spa \boldsymbol{x}}_1 \geq \norm{\boldsymbol{x}}_1$ for any $\boldsymbol{x} \in \mathbb{R}^{\bigoplus \Gamma}$. This is easily satisfied in most cases.}. \\ Let $s = (s_1^{}, \ldots, s_k^{}) \in I_1^k$ be a string, and define $c(s) \geq 0$ by \[ M_u \spa M_{s_{N}^{}} \cdots M_{s_1^{}} \spa \Phi_0 = c(s) \spa \Phi_0. \] Next, define $b_k$ by \[ b_k = \sum_{s \in (I_1 \backslash \{u\})^k} c(s)^{a_1}. \] We then conclude that \[ \mathrm{dim}_{\mathrm{H}}(X) = \log_{m_1^{}} r, \] where $r$ is the unique positive solution to the equation \[ r = b_0 + \frac{b_1}{r} + \frac{b_2}{r^2} + \cdots. \] \end{proposition} \begin{proof} Without loss of generality, assume that $u = 0$. We begin by constructing the tower decomposition $\Psi_N = \left( \Psi_N(k) \right)_{k = 0}^\infty \in \mathbb{R}^{\mathbb{N}_0}$ for a natural number $N$. First, set $\Psi_0 = (1, 0, 0, \ldots)^{\top}$, and let for $0 \leq k \leq N-1$ \[ \Psi_N(k) = \sum_{(s_1^{}, \ldots, s_{N-k-1}^{}) \in I_1^{N-k-1}} \norm{ M_0 \spa M_{s_{N-k-1}^{}} \cdots M_{s_1^{}} \spa \Phi_0 }_1^{a_1^{}}. \] The following evaluation follows from assumption (2) and (3). \begin{align*} \norm{ \Psi_N }_1 \leq & \sum_{k = 0}^{N-1} \hspace{2pt} \sum_{(w_1^{}, \ldots, w_k^{}) \in I_1^{k}} \hspace{2pt} \sum_{(s_1^{}, \ldots, s_{N-k-1}^{}) \in I_1^{N-k-1}} \norm{ M_{w_k^{}} \cdots M_{w_1^{}} \spa M_0 \spa M_{t_L^{}} \cdots M_{t_1^{}} \spa M_0 \spa M_{s_{N-k-1}^{}} \cdots M_{s_1^{}} \spa \Phi_0 }_1^{a_1^{}}. & \\ \leq & \spa N \sum_{(s_1^{}, \ldots, s_{N+L+2}^{}) \in I_1^{N+L+2}} \norm{ M_{s_{N+L+2}^{}} \cdots M_{s_1^{}} \spa \Phi_0 }_1^{a_1^{}} \\ \leq & \spa N \sum_{(s_1^{}, \ldots, s_{N+L+2}^{}) \in I_1^{N+L+2}} \norm{ M_0 \spa M_{t_L^{}} \cdots M_{t_1^{}} \spa M_{s_{N+L+2}^{}} \cdots M_{s_1^{}} \spa \Phi_0 }_1^{a_1^{}} \\ \leq & \spa N \spa \norm{\Psi_{N+2L+2}}_1. \end{align*} The estimation above implies \[ \lim_{N \to \infty} \spa \frac{1}{N} \log_{m_1^{}}{\hspace{-3pt}\sum_{(s_1^{}, \ldots, s_N^{}) \in I_1^N} {\norm{ M_{s_N^{}} \spa M_{s_{N-1}^{}} \cdots \spa M_{s_1^{}} \spa \Phi_0 }_1 }^{a_1^{}}} = \lim_{N \to \infty} \spa \frac{1}{N} \log_{m_1^{}} \norm{\Psi_N}_1. \] Next, we define a linear operator $L: \mathbb{R}^{\bigoplus \mathbb{N}_0} \rightarrow \mathbb{R}^{\bigoplus \mathbb{N}_0}$ by \begin{equation*} L = \begin{pmatrix} b_0 & b_1 & b_2 & \cdots \\ 1 & 0 & 0 & \\ 0 & 1 & 0 & \cdots \\ 0 & 0 & 1 & \\ & \vdots & & \ddots \\ \end{pmatrix}. \end{equation*} Then, $\Psi_{N+1} = L \spa \Psi_N$, and $\Psi_N = L^N \Psi_0$. The remainder of the proof follows in the same way as in Theorem \ref{theorem: tower decomposition for 2d}. \end{proof} \section{Examples} \label{section: examples} In this section, we provide a detailed explanation to the examples mentioned in the introduction, starting with the planar cases. \begin{example} Let $I = \{0, 1\} \times \{0, 1, 2\}$. Consider the directed graph in Figure \ref{figure: digraph for planar 01 redrawn} labeled with $I$. \begin{figure}[h!] \includegraphics[width=\textwidth-5.5cm]{sofic_system_01_has_1dim.eps} \caption{A digraph labeled with $I = \{0, 1\} \times \{0, 1, 2\}$} \label{figure: digraph for planar 01 redrawn} \end{figure} Then, the adjacency matrices are \begin{equation*} A_0 = \begin{pmatrix} 2 & 0 & 0\\ 0 & 1 & 0\\ 0 & 1 & 0 \end{pmatrix}, \hspace{4pt} A_1 = \begin{pmatrix} 1 & 1 & 1 \\ 0 & 0 & 0 \\ 2 & 1 & 1 \end{pmatrix}. \end{equation*} The summed matrix $A = A_0 + A_1$ is primitive. Furthermore, \begin{equation*} A_0 \spa A_1 = \begin{pmatrix} 2 & 2 & 2\\ 0 & 0 & 0\\ 0 & 0 & 0 \end{pmatrix} \end{equation*} has a $1$-dimensional image, $\mathrm{Span}\{(1, 1, 1)\}$, satisfying the condition in Theorem \ref{theorem: tower decomposition for 2d}. Excluding the string ``$01$'', the only possible strings are of the form $11\cdots100\cdots0$. For such a string $(u_1^{}, \ldots, u_N^{})$, we have for some $k$ \begin{flalign*} (1, 1, 1) \spa A_{u_1^{}} \cdots A_{u_N^{}} \spa A_0 \spa A_1 & = (1, 1, 1) \spa A_1^{k} \spa A_0^{N-k} \spa A_0 \spa A_1 & \\ & = 2^{N-k+1} \Big( ( 2 + 2\sqrt{2} )(1+ \sqrt{2})^k - ( 2 - 2\sqrt{2} ) (1- \sqrt{2})^k \Big) \spa (1, 1, 1). & \end{flalign*} Let \begin{align*} C_{N, k} = 2^{N-k+1} \Big( ( 2 + 2\sqrt{2} )(1+ \sqrt{2})^k - ( 2 - 2\sqrt{2} ) (1- \sqrt{2})^k \Big). \end{align*} Then, by Theorem \ref{theorem: tower decomposition for 2d}, $\mathrm{dim}_{\mathrm{H}}(X) = \log_2{r} = 1.6416\cdots$, where $r = 3.1201\cdots$ satisfies \begin{equation*} r^2 = \sum_{N =1}^\infty \left( \sum_{k = 0}^N {C_{N, k}}^{\log_3{2}} \right) r^{-N}. \end{equation*} \end{example} \spa \begin{example} Let $G$ be a directed graph with $2$ vertices, and $I = \{0, 1, 2\} \times \{0, 1, 2, 3\}$. Consider a sofic system with the following adjacency matrices. \begin{equation*} A_0 = \begin{pmatrix} 1 & 0 \\ 2 & 0 \\ \end{pmatrix}, \hspace{4pt} A_1 = \begin{pmatrix} 2 & 1 \\ 1 & 2 \\ \end{pmatrix}, \hspace{4pt} A_2 = \begin{pmatrix} 3 & 2 \\ 2 & 3 \\ \end{pmatrix}. \end{equation*} In this case, $A_0$ has a $1$-dimensional image, $\mathrm{Span}\{ (1, 0) \}$. Furthermore, $A_1$ and $A_2$ commute, which implies that for every string $(u_1, \ldots, u_N) \in \{1, 2\}^N$, \begin{align*} A_{u_1^{}} \cdots A_{u_N^{}} = A_1^k \spa A_2^{N-k} \end{align*} with some $k$. Then, \vspace{-8pt} \begin{align*} (1, 0) \spa A_{u_1^{}} \cdots A_{u_N^{}} \spa A_0 &= \frac{3^k \spa 5^{N-k} -1}{2} (1, 0). \end{align*} Counting the number of strings with such a $k$, we have \begin{equation*} C_k = \sum_{k = 0}^N \begin{pmatrix} N \\ k \end{pmatrix} \left( \frac{3^k \spa 5^{N-k} -1}{2} \right)^{\log_4{3}}. \end{equation*} Then, $\mathrm{dim}_{\mathrm{H}}(X) = \log_3{r} = 1.6994\cdots$, where $r$ satisfies \begin{align*} r = \sum_{N=0}^{\infty} \left(\sum_{k = 0}^N \begin{pmatrix} N \\ k \end{pmatrix} \left( \frac{3^k \spa 5^{N-k} -1}{2} \right)^{\log_4{3}} \right) r^{-N} = 6.4693\cdots. \end{align*} \end{example} Next, we provide a detailed explanation of the examples of sofic sets in $\mathbb{T}^3$ introduced earlier. \begin{example} Let $I = \{0, 1\} \times \{0, 1, 2\} \times \{0, 1, 2, 3\}$. Consider the directed graph in Figure \ref{figure: digraph for 3d example reappear} labeled with $I$. \begin{figure}[h!] \includegraphics[width=\textwidth-4cm]{digraph_for_sofic_set_with_calculation_using_infinite_matrix.eps} \caption{A digraph labeled with $I = \{0, 1\} \times \{0, 1, 2\} \times \{0, 1, 2, 3\}$} \label{figure: digraph for 3d example reappear} \end{figure} \\ Then, the adjacency matrices are \begin{equation*} \hspace{4pt} A_{(0, 0)} = \begin{pmatrix} 0 & 0 \\ 0 & 1 \\ \end{pmatrix}, \hspace{4pt} A_{(1, 0)} = \begin{pmatrix} 0 & 1 \\ 0 & 0 \\ \end{pmatrix}, \hspace{4pt} A_{(1, 2)} = \begin{pmatrix} 1 & 0 \\ 1 & 1 \\ \end{pmatrix}. \end{equation*} This sofic set has a recursive structure with $(0, 1)$. Moreover, $0 \in I_1$ is removable, so we have $J = \{1\}$. The coefficients are; $C_\varnothing (0) = 1$, $C_s(0) = 1$ for $s \in \{ 1 \}^N$, $C_\varnothing (1) = 0$, and $C_s(1) = N^{\log_4{3}}$ for $s \in \{ 1 \}^N$. The operator $M_1$ acts on $\mathbb{R}^{\bigoplus \mathbb{N}_0}$, and it is given by \begin{equation*} M_1 = \begin{pmatrix} 0 & 1 & 2^{\log_4{3}} & 3^{\log_4{3}} & 4^{\log_4{3}} & \cdots \\ 1 & 0 & 0 & \cdots \\ 0 & 1 & 0 & 0 & \cdots \\ 0 & 0 & 1 & 0 & 0 & \cdots \\ & \vdots & & & \ddots \\ \end{pmatrix}. \end{equation*} Let $\Psi_0 = (1, 0, 0, \ldots)^{\top}$. We have for any natural number $k$ \begin{equation*} b_k = \norm{ M_1^k \Phi_0 }_{1}^{\log_3{2}}. \end{equation*} Then, $\mathrm{dim}_{\mathrm{H}}(X) = \log_2{r} = 1.1950\cdots$, where $r$ satisfies \begin{align*} r = \sum_{k = 0}^{\infty} \frac{b_k}{r^k} &= 1 + \frac{1}{r} + \frac{\sqrt{2}}{r^2} + \frac{ (2 + 2^{\log_4{3}})^{\log_3{2}} }{r^3} + \frac{ (3 + 2^{\log_4{3}} + 3^{\log_4{3}})^{\log_3{2}} }{r^4} + \cdots & \\ & = 2.2894\cdots. & \end{align*} \end{example} \begin{example} Let $I = \{0, 1, 2\} \times \{0, 1, 2, 3\} \times \{0, 1, 2, 3, 4\}$. Consider the directed graph in Figure \ref{figure: digraph for 3d example 2 redrawn} labeled with $I$. \begin{figure}[h!] \includegraphics[width=\textwidth-4cm]{2nd_digraph_for_sofic_set_with_calculation_using_infinite_matrix.eps} \vspace{3pt}\caption{A digraph labeled with $I = \{0, 1, 2\} \times \{0, 1, 2, 3\} \times \{0, 1, 2, 3, 4\}$} \label{figure: digraph for 3d example 2 redrawn} \end{figure} \\ Then, the adjacency matrices are \begin{equation*} A_{(0, 0)} = \begin{pmatrix} 0 & 1 \\ 0 & 0 \\ \end{pmatrix}, \hspace{4pt} A_{(1, 0)} = \begin{pmatrix} 0 & 0 \\ 0 & 1 \\ \end{pmatrix}, \hspace{4pt} A_{(1, 1)} = \begin{pmatrix} 1 & 0 \\ 1 & 1 \\ \end{pmatrix}, \hspace{4pt} A_{(2, 0)} = \begin{pmatrix} 0 & 0 \\ 0 & 1 \\ \end{pmatrix}, \hspace{4pt} A_{(2, 3)} = \begin{pmatrix} 1 & 0 \\ 2 & 1 \\ \end{pmatrix}. \end{equation*} Let $a_1^{} = \log_4{3}$ and $a_2^{} = \log_5{4}$. This sofic set has a recursive structure with $(0, 1)$, and $J = \{1, 2\}$ since $0 \in I_1$ is removable. The coefficients for $u = 0$ are; $C_\varnothing (0) = 0$, $C_s(0) = (N + k)^{a_2^{}}$ for $s \in J^N$ that has $k$ number of $2$s. Also, $C_s(1) = C_s(2) = 1$ for every $s \in \Gamma$. Then, by considering \[ M_u \spa M_{s_{N}^{}} \cdots M_{s_1^{}} \spa \Phi_0, \] we see that \begin{align*} \hspace{10pt} b_N &= \sum_{(s_1, \ldots, s_N) \in \{1, 2\}^N} \Bigg( \big( N + \#\left\{ j \setcond 1 \leq j \leq N, s_j^{} =2\right\} \big)^{a_2^{}} \hspace{20pt} &\\ &\hspace{150pt} + \sum_{k = 1}^{N-1} 2^{k-1} \big( N + \#\left\{j \setcond k \leq j \leq N \text{\hspace{1pt} and \hspace{1pt}} s_j^{} = 2\right\} \big)^{a_2^{}} \Bigg)^{a_1^{}}. & \end{align*} We conclude that $\mathrm{dim}_{\mathrm{H}}(X) = \log_2{r} = 2.224\cdots$, where $r$ satisfies \begin{align*} r = \sum_{k = 0}^{\infty} \frac{b_k}{r^k} = 4.673\cdots. \end{align*} \end{example} \section*{Acknowledgement} I am deeply grateful to my mentor, Masaki Tsukamoto, whose expertise and insightful feedback have been instrumental in shaping both this paper and my understanding of ergodic theory. I also want to acknowledge my family and friends for their unwavering support, which has been a foundation throughout my life. This paper owes much to the collective support and intellectual environment of the academic community I have been fortunate to be a part of. \begin{thebibliography}{99} \bibitem[Ali24]{Alibabaei} N.~Alibabaei, Weighted topological pressure revisited, {\it{Ergod. Th. \& Dynam. Sys.}}, \textbf{45} (2025), 34-70. \bibitem[BF12]{Barral--Feng} J. Barral and D. J. Feng, Weighted thermodynamic formalism on subshifts and applications, {\it{Asian J. Math.}} \textbf{16} (2012), 319–352. \bibitem[BKM85]{Boyle--Kitchens--Marcus} M. Boyle, B. Kitchens and B. Marcus, A note on minimal covers for sofic systems, {\it{Proc. Amer. Math. Soc.}} \textbf{95} (1985), 403-411. \bibitem[Fe24]{Z.Feng} Z. Feng, On the coincidence of the Hausdorff and box dimensions for some affine-invariant sets, arXiv:2405.03213 \bibitem[FH16]{Feng--Huang} D.-J. Feng, W. Huang, Variational principle for weighted topological pressure, {\it{J. Math. Pures Appl.}} \textbf{106} (2016), 411-452. \bibitem[KP96]{Kenyon--Peres} R.~Kenyon, Y.~Peres, Measures of full dimension on affine-invariant sets, {\it{Ergod. Th. \& Dynam. Sys.}} \textbf{16} (1996), 307-323. \bibitem[KP96-2]{Kenyon--Peres: sofic} R.~Kenyon, Y.~Peres, Hausdorff dimensions of sofic affine-invariant sets, {\it{Israel J. Math.}} \textbf{94} (1996) 157-178. \bibitem[Ol10]{Olivier} E. Olivier, Uniqueness of the measure with full dimension on sofic affine-invariant subsets of the 2-torus. {\it{Ergod. Th. \& Dynam. Sys.}} \textbf{30} (2010), 1503-1528 \bibitem[We82]{Weiss} B Weiss, Subshifts of finite type and sofic systems, {\it{Monatsh. Math.}} \textbf{77} (1973), 462-474 \end{thebibliography} \vspace{0.5cm} \address{ Department of Mathematics, Kyoto University, Kyoto 606-8501, Japan} \textit{E-mail}: \texttt{[email protected]} \end{document}
2412.05902v1
http://arxiv.org/abs/2412.05902v1
Long-Time behavior of the tangential surface Navier-Stokes equation
\documentclass[a4paper]{amsart} \setlength{\textheight}{24.2cm} \setlength{\textwidth}{16cm} \setlength{\oddsidemargin}{0.1cm} \setlength{\evensidemargin}{0.1cm} \setlength{\topmargin}{0cm} \setlength{\parindent}{0.5cm} \def\rhead{Allen-Cahn equation} \parskip1mm \usepackage{amsthm,amsmath,amssymb} \usepackage{bm} \usepackage[square,numbers,sectionbib]{natbib} \bibliographystyle{abbrvnat} \usepackage{color} \usepackage[dvipsnames]{xcolor} \usepackage{times} \usepackage[shortlabels]{enumitem} \usepackage{enumitem} \newcommand{\subscript}[2]{$#1 _ #2$} \newcommand{\UUU}{\color{blue}} \newcommand{\TTT}{\color{teal}} \newcommand{\EEE}{\color{black}} \newtheorem{teor}{Theorem}[section] \newtheorem{defin}[teor]{Definition} \newtheorem{lemm}[teor]{Lemma} \newtheorem{osse}[teor]{Remark} \newtheorem{prop}[teor]{Proposition} \newtheorem{defi}[teor]{Definition} \newtheorem{coro}[teor]{Corollary} \newtheorem{prob}[teor]{Problem} \newcommand{\xe}{\bm{\xi}} \newcommand{\ve}{\boldsymbol{v}} \newcommand{\He}{\boldsymbol{H}} \newcommand{\qe}{\boldsymbol{q}} \newcommand{\no}{\boldsymbol{n}_{\partial \Omega}} \newcommand{\opdiv}{\operatorname{div}} \newcommand{\dx}{\mathrm{d}x} \newcommand{\dt}{\mathrm{d}t} \def\an#1{{\color{blue}#1}} \def\d{{\rm d}} \def \l {\langle} \def \r {\rangle} \def\V{{\boldsymbol{V}}} \def\H{\boldsymbol{H}} \def\W{\boldsymbol{W}} \def\A{\boldsymbol{A}} \def\L{\boldsymbol{L}} \def\ff{\textbf{\textit{f}}} \def\uu{\textbf{\textit{u}}} \def\vv{\textbf{\textit{v}}} \def\ww{\textbf{\textit{w}}} \def\g{\textbf{\textit{g}}} \def\ddt{\frac{\d}{\d t}} \def\C{\mathcal{C}} \def\n{\boldsymbol{n}} \def\E{\boldsymbol \varepsilon} \def\e{\rm e} \def \HH{\mathcal{H}} \def\VV{\mathcal{V}} \def\la{\lambda} \def\pphi{\boldsymbol{\varphi}} \def\mmu{\boldsymbol{\mu}} \newcommand{\Fsec}{F^{\prime\prime}} \newcommand{\numberset}{\mathbb} \newcommand{\N}{\numberset{N}} \newcommand{\R}{\numberset{R}} \def\nablag{\nabla_\Gamma} \def\divg{\mathrm{div}_{\Gamma}} \def\J{\boldsymbol{J}} \def\cd{\cdot} \def\Jr{\J_\rho} \def\vn{v_\n} \def\intg{\int_{\Gamma(t)}} \def\ints#1{\int_{#1(t)}} \def\Vn{v_\n} \def\VVn{\V_\n} \def\dtb{\partial^\bullet_t} \def\dt{\partial^\circ_t} \def\P{\boldsymbol{P}} \def\v{\boldsymbol{v}} \def\ig{\intg} \def\u{\boldsymbol{u}} \def\vphi{\varphi} \def\tf{\widetilde{\vphi}} \def\D{\boldsymbol{D}} \def\D{\boldsymbol{D}^-} \def\tphi{\widetilde{\phi}} \def\eps{\epsilon} \def\tmu{\widetilde{\mu}} \def\A{\boldsymbol{A}} \def\phmt{\phi_{-t}} \def\phit_{\phi_t} \def\dtast{\partial^\ast_t} \def\tphimt{\tphi_{-t}} \def\nablagz{\nabla_{\Gamma_0}} \def\D{\boldsymbol{D}} \def\Dm{\D^-} \def\tphit{\tphi_t} \def\dr{(\rho_1-\rho_2)} \def\ro{\rho} \def\tphimt{\tphi_{-t}} \def\tvphi{\widetilde{\vphi}} \def\a{\boldsymbol{a}} \def\b{\boldsymbol{b}} \def\c{\boldsymbol{c}} \def\di{\boldsymbol{d}} \def\nablagphi{\nabla_{\Gamma(t)}^\phi} \def\tT{{T}} \def\H{\boldsymbol{H}} \def\Zut{Z^1_{\tT}} \def\Zdt{Z^2_{\tT}} \def\Ztt{Z^3_{\tT}} \def\Ng{\mathcal{N}_\Gamma} \def\w{\boldsymbol{w}} \def\XT{X_{{T}}} \def\YT{Y_{{T}}} \def\norm#1{\left\Vert#1\right\Vert } \def\norma#1{\left\vert#1\right\vert } \def\trho{\rho} \def\f{\boldsymbol f} \def\dtn{\partial_t} \def\ddt{\frac{d}{dt}} \def \Pt{P(t)} \def\LQs{L^2(0,\widetilde{T};\L^2_\sigma(\Gamma_0))} \def\LQ{L^2(0,\widetilde{T};\L^2(\Gamma_0))} \def\CT{C(T)} \def\thetaq{\theta(1-\frac1q)} \def\gz{\Gamma_0} \def\LLQ(#1,#2){L^{#1}(0,\widetilde{T};\L^{#2}(\Gamma_0))} \def\EE{\mathcal{E}^{\gz}} \def\div#1{\text{div}_{#1}} \def\dts{\partial_t_*} \def\Lqp{L^q(0,\tT;L^p(\gz))} \def\Lqpb{L^q(0,\tT;\L^p(\gz))} \def\gam{\Gamma} \def\gt{(\gam)} \def\NN{\mathcal{N}} \def\Linfg{\L^\infty\gt} \def\wu{\widehat{\u}} \def\na{\nablag} \def\nus{\frac{\nu_*}{16}\norm{\E_\Gamma(\V)}^2} \def\non{\nonumber} \def\Phitn{\Phi_t^n} \def\z{\boldsymbol z} \def\X{\boldsymbol X} \def \ds{d \sigma} \def\A{\boldsymbol{A}} \def\B{\boldsymbol{B}} \def\Ya{L^p(0,T;\L^p_\sigma(\Omega))} \def\Yan{L^p(0,T;\L^p(\Omega))} \def\Yb{L^q(0,T;L^q(\Omega))} \def\Yc{L^2(0,T;L^2(\Omega))} \def\th{\theta} \def\Pk{\P_{\mathcal{K}}} \def\Lts{{\L^2_\sigma(\Gamma)}} \def\I{\boldsymbol I} \def\KK{\mathcal K} \def\BB{\mathcal B} \def\AA{\mathcal A} \def\BBB{\mathbb B} \def\tA{\widetilde{\AA}} \def\JJ{\mathcal J} \def\AP#1{{\color{red}#1}} \def\Af#1{{\color{violet}#1}} \newcommand{\T}{{\rm T}} \newcommand{\Rz}{{\mathbb R}} \newcommand{\zero}{\boldsymbol 0} \numberwithin{equation}{section} \begin{document} \title[Long-Time behavior of the tangential surface Navier--Stokes equation]{Long-Time behavior of the \\ tangential surface Navier--Stokes equation} \author[A.~Poiatti]{Andrea Poiatti} \address[A.~Poiatti]{Faculty of Mathematics, University of Vienna, Oskar-Morgenstern-Platz 1, A-1090 Vienna, Austria} \email[A.~Poiatti]{[email protected]} \author[U.~Stefanelli]{Ulisse Stefanelli} \address[U.~Stefanelli]{ Faculty of Mathematics, University of Vienna, Oskar-Morgenstern-Platz 1, A-1090 Vienna, Austria, Vienna Research Platform on Accelerating Photoreaction Discovery, University of Vienna, W\"ahringerstrasse 17, A-1090 Wien, Austria, \& Istituto di Matematica Applicata e Tecnologie Informatiche {\it E. Magenes}, via Ferrata 1, I-27100 Pavia, Italy.} \email[U.~Stefanelli]{[email protected]} \begin{abstract} We investigate the initial-value problem for the incompressible tangential Navier--Stokes equation {with variable viscosity} on a given {two-dimensional} surface without boundary. Existence of global weak and strong solutions under inhomogeneous forcing is proved by a fixed-point and continuation argument. Continuous dependence on data, backward uniqueness, and instantaneous regularization are also discussed. Depending on the effect of the inhomogeneous forcing on the dissipative and the nondissipative components of the system, we investigate the long-time behavior of solutions. We prove the existence and properties of the $\sigma$-{global} attractor, in the case of bounded trajectories, and of the so-called unbounded attractor, for unbounded trajectories. \end{abstract} \subjclass{35Q30, 37L30,} \keywords{Tangential surface Navier--Stokes equation, well-posedness, long-time behavior, attractor} \maketitle \section{Introduction} \label{sec:intro} This paper is concerned with the initial-value problem for the {\it incompressible tangential surface Navier--Stokes equation} \cite{V1} {with variable viscosity}, namely, \begin{align} &\label{t1}\partial_t\u+(\u\cdot \nabla_\Gamma)\u-2\P_\Gamma\divg(\nu(\cdot)\E_\Gamma(\u))+\nabla_\Gamma p=\f(\cdot,\u),\\& \divg\u=0,\label{t12}\\& \u(0)=\u_0. \label{tangential} \end{align} This equation describes the motion of a viscous incompressible fluid on a given, sufficiently smooth, and connected two-dimensional surface $\Gamma\subset \Rz^3$ without boundary. The state of the fluid is determined by its {\it velocity} $\u : \Gamma \times \Rz_+ \to \T \Gamma $ and its {\it pressure} $p:\Gamma\times \Rz_+ \to \Rz$. Here, $\T \Gamma$ denotes the tangent bundle of $\Gamma$. Fluid flows on closed surfaces arise in connection with models at different scales, from biological membranes, to the dynamics and coating of microfluidic droplets, to soap bubbles. Atmospheric flows around planets, as well as oceanic flows on completely fluid-covered bodies, are also examples of surface flows. At an even larger scale, magnetohydrodynamics is the basis of different astrophysical models for surface flows of charged fluids, like plasma, to model star dynamics. Surface flows have in fact already attracted mathematical attention and the reader is referred to \cite{Arnaudon,Chan,V1} and \cite{AGP2024,AGP2024b,Koba,Koba2} for some discussion of the case of stationary and evolving surfaces, respectively. See also \cite{Brandner} for an overview and a comparison on the different derivations of the surface Navier--Stokes equation. In equation \eqref{t1}, the symbol $\partial_t$ denotes the partial derivative with respect to time and $\P_\Gamma:=\I - \n \otimes \n$ is the projection on $\T\Gamma$, where $\I$ is the identity and $\n$ is the outward pointing normal to $\Gamma$. For given differentiable fields $\u$ and $p$ defined on a neighborhood of $\Gamma$, we indicate by $\nabla_\Gamma \u = {\P_\Gamma}\nabla \u \P_\Gamma= \nabla \u- (\n\otimes \n)\nabla \u - \nabla \u (\n\otimes \n) + \n\otimes \n$ and $\nabla_\Gamma p = \nabla p \P_\Gamma = \nabla p - (\nabla p \cdot \n)\n$ the {covariant derivative and the scalar surface gradient, respectively}, by ${\rm div}_\Gamma \u = \P_\Gamma {:} \nabla \u - {\rm div} \u - \n \cdot (\nabla \u) \n$ the surface divergence, and by $\E_\Gamma(\u):= (\nabla_\Gamma\u+\nabla_\Gamma^T\u)/2 $ the surface rate-of-strain tensor (the symbols $\cdot$, $:$, and $\otimes$ denote the scalar, the contraction, and the tensor product, respectively). We refer to Chapter 2 in \cite{V1}, \cite{greenbook}, or the Appendix of \cite{Simonett} for additional information and details on these objects. The function $\nu$ in \eqref{t1} is the strictly positive {\it interface shear viscosity} which we assume to be space dependent to allow for the modeling of binary or multi-phase fluids (see, e.g., \cite{AGG,AGP}), {or even of nonisothermal fluids (e.g., \cite{Bernardi,GPPV})}. Eventually, $\f:\Gamma\times \T\Gamma\to \T\Gamma$ is a suitable divergence-free tangential forcing term, which we also allow to depend on $\u$. The aim of this paper is to investigate the well-posedness and the long-time behavior of solutions to the initial-value problem \eqref{t1}--\eqref{tangential}, especially in the nonhomogeneous case $\f \not =\zero$. Note that in most settings, nontrivial forcings $\f$ naturally occur. Referring to the above list of examples, we remark that microdroplets are subject to friction and surface tension, biological membranes are driven by cell electrochemistry, and gravity influences soap bubbles. At another scale, atmospheric and oceanica flows are driven by thermal, chemical, geophysical, and tidal processes, while astrophysical flows are subject to electromagnetic, thermal, and gravitational effects. Existence results for the Navier--Stokes equation in three dimensions are a mainstay of nonlinear PDE theory. The reader is referred to the monographs \cite{Constantin88,Galdi94,Girault86,Temam84} for a collection of material. Existence for the surface Navier--Stokes case has also been considered. The surface {stationary} Stokes equation has been considered in \cite{V1} while the surface {evolutionary} Navier--Stokes equation, {with $\ff=\mathbf 0$ and constant viscosity $\nu$}, has been studied in \cite{Simonett} and checked to be locally well-posed in time. {Additionally, solutions to the same equation on two dimensional surfaces, departing from a sufficiently regular divergence-free initial datum, has been shown to exist globally in \cite{Simonett2}.} Our first result concerns the global well-posedness of problem \eqref{t1}--\eqref{tangential} in the general inhomogeneous case $\f \not = \zero$ {and variable viscosity}. We prove that weak solutions to problem \eqref{t1}--\eqref{tangential} globally exist, regardless of the size of the initial datum. In case the initial datum is in $\H^1$ weak solutions are actually strong. Moreover, we show continuous dependence on data, entailing uniqueness, as well as instantaneous regularization, see Theorems \ref{thm1} and \ref{insta}. In the setting of the tangential equation, this extends the results in \cite{Simonett,Simonett2} to the general, global-in-time, inhomogeneous case. In addition, we show the so-called {\it backward uniqueness} property, namely, that two trajectories coinciding at some point in time necessarily coincide for all times, see Theorem \ref{backuni}. This property, which is well known in the case of the Navier--Stokes problem in a three-dimensional domain \cite{Escauriaza,Iskauriaza,Seregin}, was apparently not yet investigated for surface flows. The main focus of this paper is however on the long-time behavior of solutions. The asymptotic behavior of a solution to the Navier--Stokes equation for large times in three-dimensional domains is classical, see the monographs \cite{WeakGlobalNSAttractor,Foias2,Temam} and the references therein. The long-time behavior for the surface Navier--Stokes equation was initiated in \cite{Simonett,Simonett2}, {concerning the equilibria of a single trajectory}. In this setting a key role is played by the so-called {\it Killing fields} on $\Gamma$, namely, fields $\u_K \in C^\infty(\Gamma;\T\Gamma)$ with the property that $\E_\Gamma(\u_K)=\zero$. Note that the occurrence of Killing fields is completely determined by the geometry of the given $\Gamma$. In particular, Killing fields correspond to rotational symmetries of $\Gamma$. Hence, the space of Killing field is $n$-dimensional, with $n=0,1,2$, or $3$. In case $\f=\zero$, Killing fields are stationary solutions to \eqref{t1}--\eqref{t12}. In particular, the tangential surface Navier--Stokes equation is nondissipative on Killing fields, which in fact correspond to equilibria. In \cite{Simonett}, killing fields are identified as equilibria and are proved to be stable. Eventually, trajectories starting close enough to equilibria are shown to exist globally and to converge to equilibria exponentially fast in time. {Also, in \cite{Simonett2} it is shown that any weak solution, departing from a sufficiently smooth divergence-free initial datum, converges exponentially fast to an equilibrium, that is, to a Killing field.} {Inspired by these results,} we turn our attention on sets of trajectories instead {of single ones}. Moving from our global well-posedness results, we discuss the long-time behavior in terms of the dynamical system generated by equation \eqref{t1}--\eqref{t12}. One decomposes $\u = \u_K + \u_{NK}$ in its Killing and non-Killing components by projecting on the finite-dimensional space of Killing fields. As { only }the component $\u_{NK}$ dissipates {thanks to the tensor $\boldsymbol\varepsilon_\Gamma$}, one is asked to investigate the properties of $\f$ with respect to the dynamics of the Killing component $\u_{K}$. In Theorems \ref{thm:323} and \ref{thm:325}, we consider the case of bounded trajectories. Different assumptions entailing such boundedness are presented, including, for instance, $$\int_\Gamma\f_{NK}(\cdot,\u)\cdot \u \leq \delta \|\u_{NK}\|^2_{\L^2} + C \|\u_K\|^2_{\L^2}, \quad \int_\Gamma\f_{K}(\cdot,\u)\cdot \u \leq 0 \quad \forall \u \in \L^2_\sigma(\Gamma)$$ for some $C>0$ and $\delta >0 $ small, where $\f = \f_K + \f_{NK}$ is the decomposition of $\f$ in its Killing and non-Killing components and we have used the space $\L^2_\sigma(\Gamma) :=\{\v \in \L^2(\Gamma)\::\: {\rm div}\, \v=0 \ \text{a.e. in} \ \Gamma\}$. In the case of bounded trajectories, we prove that the generated semigroup restricted to $\{\u\in \L^2_\sigma(\Gamma)\::\: \|\u_{K}\|_{\L^2}\leq r\}$ for $r>0$ admits a global attractor $\mathcal A_r$ (nonempty, compact, connected, invariant, and attracting) of finite fractal dimension and that $\cup_{r>0}\mathcal A_r{=\cup_{m\in \mathbb N}\mathcal A_m}$ is minimal among closed sets attracting all bounded sets {of the phase space $\L^2_\sigma(\Gamma)$}. {Since this is a countable union, such} {set is called {\it $\sigma$-global attractor.}} Moreover, we prove that for all $r>0$ there exists an exponential attractor $\mathcal M_r$ (nonempty, compact, positively invariant, and exponentially attracting) of finite fractal dimension, so that {$\cup_{m\in \mathbb N}\mathcal M _m$, which we} term {\it $\sigma$-exponential attractor}, exponentially attracts all bounded sets {of $\L^2_\sigma(\Gamma)$}. The case of possibly unbounded trajectories is discussed in Theorems \ref{thm:332} and \ref{t1b}, instead. If $$\exists R>0: \quad \int_\Gamma\f_K(\cdot,\u)\cdot \u \geq 0 \quad \forall \u \in \L^2_\sigma(\Gamma) \ \ \text{such that} \ \ \| \u_K\|_{\L^2} \geq R, $$ one can consider the notion of {\it unbounded attractor}, \cite{Chepyzov}, which consists of the values of all bounded-in-the-past complete trajectories of the dynamical system. This {set} has many of the properties of a global attractor. In particular, it is nonempty, closed, and invariant. It is moreover bounded compact: its intersection with any closed bounded set is compact. On the other hand, its attractivity can proved just for bounded trajectories departing from any given bounded set. The paper is organized as follows. In Section \ref{sec:strong}, we introduce notation and recall some basic facts on Killing fields. All statements are collected in Section \ref{main}. In particular, well-posedness (Theorem \ref{thm1}) and backward uniqueness (Theorem \ref{backuni}) are in Section \ref{sec:well}. Some preliminaries on long-time behavior, including the decomposition $\u = \u_K + \u _{NK}$, the bounds on these components, and instantaneous regularization (Theorem \ref{insta}), are collected in Section \ref{longtt}. The notion of attractor for the underlying dynamical system is discussed in Section \ref{notion}. Eventually, the statements on the existence of attractors for the case of bounded (Theorems \ref{thm:323} and \ref{thm:325}) and unbounded (Theorems \ref{thm:332} and \ref{t1b}) trajectories are given in Sections \ref{sec:attr} and \ref{unbd}, respectively. The proof of the statements from Sections \ref{sec:well}, \ref{longtt}, \ref{sec:attr}, and \ref{unbd} is then given in Sections \ref{sec:pr1}, \ref{sec:prel}, \ref{bdda}, and \ref{unbdd}, respectively. Eventually, Appendix \ref{sec:appendix} contains a lemma on the existence of the unbounded attractor. \section{Notation and functional setting} \label{sec:strong} Let us consider a smooth, compact, connected, embedded hypersurface $\Gamma\subset \R^3$ without boundary. In the following, the classical Sobolev spaces are denoted as usual by $W^{k,p}(\Gamma)$ , where $k\in \mathbb{N}$ and $1\leq p\leq \infty $, with norm $\Vert \cdot \Vert _{W^{k,p}(\Gamma)}$. The Hilbert space $W^{k,2}\gt$ is denoted by $H^{k}\gt$ with norm $\Vert \cdot \Vert _{H^{k}\gt}$. We then set $H^{-1}\gt=(H^1\gt)'$. Given a vector space $X$ of functions defined on $\Gamma$, we denote by $\boldsymbol{X}$ the generic space of tangential vectors or matrices, with each component in $X$. We recall that a tangential vector $\v$ is such that $\v\cdot\n=0$ on $\gam$, i.e., $\P_\Gamma\v=\v$. In a similar way, a tangential matrix $\boldsymbol M$ is a matrix such that $\P_\gam\boldsymbol M\P_\Gamma=\boldsymbol M$. An example of a tangential matrix is the covariant derivative of a vector field $\v$, denoted by $\nabla_\Gamma\v$. The symbol $\vert \boldsymbol{v} \vert$ stands for the Euclidean norm of $\boldsymbol{v}\in \boldsymbol{X}$, i.e., $\vert\boldsymbol{v}\vert^2=\sum_{j}\|v_j\|_X^2$. We then denote by $(\cdot,\cdot )$ the inner product in $\boldsymbol{L}^{2}\gt$ and by $\Vert \cdot \Vert $ the induced norm. We also denote by $(\cdot ,\cdot )_{H}$ and $\Vert \cdot \Vert _{H}$ the natural inner product and its corresponding induced norm in the Hilbert space $H$. Moreover, given a Banach space $X$, $T>0$, and $q\in[2,\infty]$, we denote by $L^q (0,T; X )$ the Bochner space of $X$-valued $q$-integrable (or essentially bounded) functions. We then denote by $BC([0,T];X)$ the Banach space of bounded continuous functions on $[0,T]$, equipped with the supremum norm. The space $BUC([0,T];X)$ is then its subspace of bounded and uniformly continuous functions. Finally, $W^{1,p} (0, T ; X)$, $1 \leq p < \infty$, is the space of functions $f$ such that $\partial_t f\in L^p(0,T;X)$ and $f\in L^p(0,T;X)$, where $\partial_t$ is the vector-valued distributional derivative of $f$. We set $H^1 (0, T ; X) =W^{1,2}(0,T;X)$. In order to handle divergence-free vector fields, we introduce the spaces \begin{align*} &\L^2_\sigma\gt:=\{\v\in \L^2\gt: \divg \v=0\text{ a.e. on } \Gamma \}, \\ &\H^1_\sigma\gt:=\{\v\in \H^1\gt: \divg \v=0\text{ a.e. on } \Gamma \}, \end{align*} and observe that $\H^1_\sigma\gt,\L^2_\sigma\gt ,\H^1_\sigma\gt')$ forms a Hilbert triplet. We also introduce the surface Helmholtz orthogonal projector on $\Gamma$, denoted by \begin{align} \P_0:\L^2(\Gamma)\to \L^2_\sigma(\gam), \label{Leray} \end{align} see, e.g., \cite{Simonett2} for details. We now introduce the notion of Killing fields as \begin{align*} \mathcal K:=\{\u\in \L^2_\sigma(\Gamma)\cap \boldsymbol C^\infty( \Gamma;\T\Gamma ):\ \E_\Gamma(\u)\equiv \boldsymbol 0\}, \end{align*} where we recall that the divergence free condition is already entailed by the condition $\E_\Gamma(\u)\equiv \boldsymbol 0$, by $\divg \u=\text{tr}(\E_\Gamma(\u))=\boldsymbol 0$. Moreover, as noticed in \cite{Simonett,Simonett2}, one can show that any vector field $\u\in \W^{1,q}(\Gamma)$ such that $\E_\Gamma(\u)\equiv \boldsymbol 0$ is smooth, see for instance \cite[Lemma 3]{Priebe}. In particular, the Killing fields on a Riemannian manifold form a Lie sub-algebra of the Lie algebra of all tangential fields. We also recall that the Killing fields of a Riemannian manifold $(M, g)$ are the infinitesimal generators of the isometries $I(M, g)$ on $(M, g)$, i.e., the generators of flows that are isometries on $(M, g)$. Furthermore, if $(M, g)$ is complete, which is our case, the Lie algebra of Killings fields is isometric to the Lie algebra of $I(M, g)$, (see for example \cite[Corollary III.6.3]{Sakai}). From this observation we can deduce that $\KK$ is a finite dimensional vector space, which is thus closed in $\L^2_\sigma(\Gamma)$. Indeed, in the case of a surface embedded in $\R^{d+1}$, from \cite[Proposition III.6.5]{Sakai} one deduces that $\text{dim}\ \KK \leq d(d+1)/2$, with the equal sign characterizing the case where $\Gamma$ is isometric to an Euclidean sphere. In our case $d=2$, and thus $\text{dim}\ \KK \leq 3$. Additionally, if $(M, g)$ is compact and the Ricci tensor is negative-definite everywhere, then any Killing field on $M$ is equal to zero and $I (M, g)$ is a finite group, see, for example \cite[Proposition III.6.6]{Sakai}. Namely, if $(M, g)$ is a two-dimensional Riemannian manifold with negative Gaussian curvature then any Killing field is $\boldsymbol 0$. Note that, among two-dimensional compact closed surfaces only those of genus 0 and 1 may have nonzero Killing fields \cite[Thm.~6]{Myers}. Some possible examples for which the Killing fields are nontrivial are then the following (see, e.g., \cite{Simonett, Olshsup}): \begin{itemize} \item $ \Gamma= \mathbb S^2$. Then, $\text{dim } \KK = 3$ and any $\u\in \KK$ is a rotation about an axis spanned by a vector $\omega = (\omega_1,\omega_2,\omega_3) \in \R^3$, so that $\u \in \KK$ is given by $$\u(x) = \omega \times x,\quad x \in \mathbb S^2,$$ for some $\omega \in\R^3$. \item $\Gamma=\mathbb T^2$ with parametrization \begin{align*} & x_1 = (R + r \cos \phi) \cos \theta,\\& x_2 = (R + r \cos \phi) \sin \theta,\\& x_3 = r \sin \phi, \end{align*} where $\phi, \theta \in [0, 2\pi)$ and $ 0 < r < R$. Then, the velocity field $\u = \omega\boldsymbol e_3 \times x$, with $ \omega \in \R$, is an element of $\KK$, i.e., the fluid on the torus rotates about the $x_3$-axis with angular velocity $\omega\boldsymbol e_3$, and these rotations can be proven to be the only isometries on $\mathbb T^2$. This means that $\text{dim}\ \KK=1$. \item As observed in \cite{Olshsup}, any surface $\Gamma$ of revolution supports a nonzero Killing field. Moreover, it looks plausible (although apparently still not discussed in the literature) that among closed compact smooth surfaces isometrically embedded in $\R^3$ only surfaces of revolution support nontrivial Killing fields. In the case of a surface of revolution, one has KIlling fields $\u\in \KK$ is of the form $$ \u=\omega\boldsymbol e_z \times x, $$ where $\omega\in \R$ and $\boldsymbol e_z$ is the unit vector in the direction of the axis of rotation. This corresponds to a rotation about the $z$-axis with angular velocity $\omega\boldsymbol e_z$. \end{itemize} The importance of the Killing fields $\KK$ lies in the lack of dissipativity properties of the tangential Navier--Stokes system on this vector space. In order to take this issue into account, solutions need to be decomposed in a Killing component in $\KK$ and a non-Killing one. More precisely, we define the orthogonal $\L^2$ projector on the (closed) Killing vector space $\mathcal K$: $$ \Pk: \Lts\to \mathcal{K}. $$ Since the space $\mathcal K$ is finite dimensional and at most of dimension $n=3$, we can always find an orthonormal $\L^2$-basis $\{\v_n\}$, where $n$ can be $0$, $1$, $2$, or $3$. Therefore, any element $\v\in \L^2_\sigma(\Gamma)$ can be uniquely decomposed into $$ \v=\Pk \v+(\boldsymbol I-\Pk)\v:=\v_{K}+\v_{NK}, $$ where $\boldsymbol I$ is the identity operator on $\L^2_\sigma(\Gamma)$. Note that, as shown in \cite[Remark 4.10]{Simonett2}, it also holds $$ \H^1_\sigma(\Gamma)=\KK\oplus {(}(\I-\Pk)\L^2_\sigma(\Gamma)\cap \H^1(\Gamma){)}. $$ In conclusion, we recall the following Korn's inequality, which is fundamental to deal with the lack of dissipativity properties of the system (see, e.g. \cite[(4.7)]{V1}): \begin{align} \norm{\v}_{\H^1(\Gamma)}\leq C(\norm{\v}+\norm{\E_\Gamma(\v)}),\quad \forall \v\in \H^1(\Gamma), \label{Korn1} \end{align} which entails that \begin{align} \norm{\v}_{\H^1(\Gamma)}\leq C\norm{\v},\quad \forall \v\in \KK. \label{Korn2} \end{align} Furthermore, we also have (see, for instance, \cite[Theorem A.3]{Simonett2}) \begin{align} \norm{\v}_{\H^1(\Gamma)}\leq C_P\norm{\E_\Gamma(\v)},\quad \forall \v\in (\I-\Pk)\L^2_\sigma(\Gamma)\cap \H^1(\Gamma), \label{Korn3} \end{align} for some $C_P>0$. \section{Main results}\label{main} \subsection{Well-posedness and backward uniqueness} \label{sec:well} We state here a well-posedness result concerning the tangential surface Navier--Stokes equation. This is an extension of the former \cite{Simonett2}, where $\f=\boldsymbol 0$ and a constant viscosity $\nu$ are assumed. %, we refer to The proof of this result is postponed to Section \ref{sec:pr1}. We have: \begin{teor}[Well-posedness] \label{thm1} Let $\nu\in W^{1,\infty}(\Gamma)$ be such that $\nu\geq \nu_*>0$, and $\f:\Gamma\times \T\Gamma\to \T\Gamma$ be a Carath\'eodory function such that there exist $C_1>0$ so that \begin{align} \norm{\f(x,\boldsymbol 0)}\leq C_1, \quad \text{for a.e.} \ x \in \Gamma, \label{C1} \end{align} and there exists $C_2>0$ so that \begin{align} \norm{ \f(x, \v_1(x))-\f(x,\v_2(x))}\leq C_2\norm{ \v_1-\v_2},\quad \forall \v_1,\v_2\in \L^2(\Gamma), \quad \text{for a.e.} \ x \in \Gamma . \label{C2} \end{align} Let $\u_0\in \L^2_\sigma(\Gamma)$. Then there exists a unique global weak solution $\u:\Gamma\times[0,\infty)\to \T\Gamma$ to \eqref{t1}--\eqref{tangential} such that, for any $T>0$, \begin{align} &\u\in C([0,T]; \L^2_\sigma(\Gamma))\cap L^2(0,T;\H^1_\sigma(\Gamma))\cap H^1(0,T;\H_\sigma^1(\Gamma)'),\label{regga} \end{align} and, for almost every $t>0$, \begin{align*} \langle \partial_t\u, \boldsymbol\eta\rangle_{\H^1_\sigma(\gam)',\H^1_\sigma(\Gamma)}+\int_\Gamma (\u\cdot\nabla_\Gamma)\u\cdot \boldsymbol \eta+\int_\Gamma 2\nu(x)\E_\Gamma(\u):\E_\Gamma(\boldsymbol{\eta})=\int_\Gamma \f(x,\u)\cdot \boldsymbol{\eta},\quad \forall \boldsymbol{\eta}\in \H^1_\sigma(\gam). \end{align*} Moreover, given two weak solutions $\u_1,\u_2$ as above, departing from initial data $\u_{0,1},\u_{0,2}\in \L^2_\sigma(\Gamma)$, for all $T>0 $ the following continuous-dependence estimate holds \begin{align} \sup_{t\in[0,T]}\norm{\u_1(t)-\u_2(t)}^2+2\nu_*\int_0^T\norm{\mathcal E_S(\u_1(t)-\u_2(t))}^2d t\leq C \norm{\u_{0,1}-\u_{0,2}}^2, \label{contdep2} \end{align} where the constant $C>0$ depends on $T$ and $\norm{\u_{0,i}}$, $i=1,2$. Additionally, if $\u_0\in \H^1_\sigma(\Gamma)$ the unique weak solution is also a global strong solution. Namely, for all $T>0$ we have \begin{align} &\u\in C([0,T]; \H^1_\sigma(\Gamma))\cap L^2(0,T;\H^2(\Gamma))\cap H^1(0,T;\L^2_\sigma(\Gamma))\label{strong} \end{align} and \eqref{t1}--\eqref{tangential} are satisfied almost everywhere on $\Gamma\times(0,\infty)$. Eventually, the unique global weak solution departing from a given $\u_0\in \L^2_\sigma(\Gamma)$ instantaneously regularizes. In particular, it is such that, for any $T>0$ and any {$\tau\in(0,T)$}, there exists $ C>0$ depending on $T$, $\norm{\u_0}$, $\f$, $\Gamma$, and on the parameters of the problem, but not on $\tau$, so that \begin{align} \norm{\u}_{C([\tau,T];\H^1_\sigma (\Gamma))}+\norm{\u}_{L^2(\tau,T;\H^2(\Gamma))}+\norm{\partial_t\u}_{L^2(\tau,T;\L^2_\sigma(\Gamma))}\leq \frac{ C }{\sqrt \tau}. \label{regulari} \end{align} \end{teor} \begin{osse}\rm (Divergence-free condition) Without loss of generality one can directly assume that $\f(\cdot,\v)$ is divergence free for any suitably regular divergence free vector field $\v$. Indeed, only the divergence-free part of the function $\f$ has an active role in the system. \end{osse} \begin{osse}\rm (Pressure) By the classical De Rahm's theorem, in the case of strong solutions we can also deduce the existence of a unique pressure $p$ such that $\int_\Gamma p\equiv 0$ and $p\in L^2(0,T;H^1(\Gamma))$ for any $T>0$. \end{osse} We additionally point out that the system \eqref{t1}--\eqref{tangential} has the property of backward uniqueness for strong solutions, i.e., if two strong solutions coincide at some instant of time, then they coincide for all previous times. This property has been shown for the 2D Navier--Stokes equations on bounded domains (see, e.g., \cite{Tartar}), but, as far as we know, it has not been shown in the case of closed surfaces. Indeed, in the latter case one cannot exploit the full dissipativity nature of the equations as in the bounded domains case. This is to some extent similar to what has been adressed in \cite{Foias} in the case of semi-dissipative Boussinesq equations, where a part of the system is not dissipative. In our problem, the main ingredients will be a careful use of Korn's inequality to deal with the lack of dissipativity, together with the decomposition of the solution in its Killing and non-Killing components. An additional difficulty comes from the presence of a variable viscosity $\nu$. We thus have the following, which is also proved in Section \ref{sec:pr1}. \begin{teor}[Backward uniqueness]\label{backuni} Let assumptions \eqref{C1}--\eqref{C2} hold, together with $\nu\in W^{1,\infty}(\Gamma)$ and $\nu\geq \nu_*>0$. For $i=1, 2$, let $\u_i$ be two {global} strong solutions to \eqref{t1}--\eqref{tangential} according to Theorem \emph{\ref{thm1}}. If there exists a time $T^*>0$ such that $\u_1(T^*) =\u_2(T^*)$ almost everywhere on $\Gamma$, then for any $t\in[0,T^*]$, we have $\u_1(t) =\u_2(t)$ almost everywhere on $\Gamma$. \end{teor} \begin{osse}\rm (Solutions do not cross)\label{coincidenza} Notice that, as a consequence of Theorem \emph{\ref{backuni}} and the uniqueness result of Theorem \emph{\ref{thm1}}, if two (global) strong solutions coincide at some instant of time $T^*\geq0$, then they must coincide for any $t\geq0$. \end{osse} \subsection{Long-Time behavior: Preliminaries} \label{longtt} Observe that the main peculiarity of the surface tangential Navier--Stokes equation is related to the fact that the system is not dissipative in general. Indeed, in case the Killing field space $\mathcal{K}\not=\{\boldsymbol 0\}$, the dissipation due to the stress tensor $\E_\Gamma$ does not act on the component of the velocity in the direction of a Killing field (since $\E_\Gamma(\v)=\boldsymbol 0$ for any $\v\in \mathcal K$). In order to study the long-time behavior of the solutions to \eqref{t1}--\eqref{tangential}, we consider the dynamical system in the space $\Lts$. For the sake of readability, all proofs of the statements of this section are postponed to Section \ref{sec:prel}. Under the same assumptions of Theorem \ref{thm1}, we can now define a dynamical system $(\Lts,S(t))$ where \begin{equation*} S(t):\Lts\rightarrow \Lts, \quad S(t)\boldsymbol{u}_{0}=\boldsymbol{u}(t),\quad \forall \, t\geq 0. \end{equation*}Observe that $S(t)$ is a continuous semigroup, since it satisfies the following properties: \begin{itemize} \item $S(0)=\boldsymbol I$; \item $S(t+\tau)=S(t)S(\tau)$, for every $t,\, \tau \geq 0$; %$\boldsymbol{u}_0\in \Lts$; \item $t\mapsto S(t)\boldsymbol{u}_0\in C([0,\infty);\Lts)$, for every $\boldsymbol{u}_0\in\Lts$; \item $\boldsymbol{u}_0\mapsto S(t)\boldsymbol{u}_0\in C(\Lts;% \Lts)$, for any $t\in[0,\infty)$. \end{itemize} In particular, the last property is a direct consequence of the continuous dependence estimate \eqref{contdep2}. Observe also that the backward uniqueness property, together with the well-posedness result (see Theorems \ref{thm1} and \ref{backuni}) implies that if any two (sufficiently smooth) trajectories intersect, then they must be identical. As already anticipated, due to the lack of dissipation of the entire velocity, it is useful to decompose the semigroup into the sum of two other operators. First, we introduce $\f_K:\Gamma\times \L^2_\sigma(\Gamma)\to \L^2_\sigma(\Gamma)$ via \begin{align*} \f_K(\cdot,\u):=\P_{\mathcal{K}}\f(\cdot,\u)=\sum_{j=1}^n\left(\int_\Gamma \f(x,\u(x))\cdot \v_j(x)\right)\v_j, \end{align*} where $\{\v_j\}$ is a given orthonormal basis of $\mathcal K$. We also define $\f_{NK}:=\f-\f_K$. \noindent Now, given $\u_0\in \Lts$, setting $\overline{\u}:=S(t)\u_0$, we define $\u_{NK}(t)$ as the solution to \begin{align} &\label{t11}\partial_t\u_{NK}+\overline{\u}\cdot \nabla_\Gamma\overline{\u}-2\P\divg(\nu\boldsymbol\varepsilon_\Gamma(\u_{NK}))+\nabla_\Gamma p=\f_{NK}(\cdot,\overline{\u}),\\& \divg\u_{NK}=0,\\& \u_{NK}(0)=(\boldsymbol I-\P_{\mathcal{K}})\u_0, \label{tangential1} \end{align} and $\u_K(t)$ to be the solution to \begin{align} &\label{K1}\partial_t\u_K=\f_K(\cdot,\overline{\u}),\\& \label{K2}\u_K(0)=\P_{\mathcal{K}}\u_0. \end{align} We then have the following. \begin{prop}[Decomposition of $\u$] Let $\u_0\in \Lts$ and set $\overline{\u}=S(t)\u_0$. Under assumptions \eqref{C1}--\eqref{C2} it holds that \begin{align*} \u_{NK}=(\boldsymbol I-\P_{\mathcal{K}})\overline{\u},\quad \u_{K}=\P_{\mathcal{K}}\overline{\u}, \end{align*} where $\u_{NK}$ and $\u_{K}$ are defined in \eqref{t11}--\eqref{tangential1} and \eqref{K1}--\eqref{K2} respectively. \label{dec} \end{prop} In order to analyze the asymptotic behavior of the Killing projection $\u_K$, we need to further distinguish some cases, depending on the dissipative nature of the forcing term $\f_K$. We collect some observations in the following proposition, which is proved in Section \ref{sec:prel}. \begin{prop}[Bounds on the Killing component $\u_K$] \label{lemmadec} Under assumptions \eqref{C1}--\eqref{C2}, let us assume, additionally, that there exists $ C_3 >0$ such that \begin{align} \int_\Gamma \f_K(x,\u)\cdot \u\leq C_3 \norm{\P_{\mathcal{K}}\u}^2+ C_3 ,\quad \forall \u\in \Lts. \label{uk1} \end{align} Then there exists $ C_4 >0$ such that \begin{align} \norm{\u_K(t)}^2\leq e^{2 C_4 t}(\norm{\u_K(0)}^2+ C_4 ),\quad \forall t\geq 0, \label{exp} \end{align} for any $\u_0\in \Lts$. Moreover, we distinguish three more specific cases: \begin{itemize} \item[(i)] If $\f_K$ is independent of $\u$, i.e.,$\f_K:\Gamma\to \T\Gamma$, then \begin{align} \int_{\Gamma}\f_K(x)\cdot \u_{K}(x,t)= \int_{\Gamma}\f_K(x)\cdot \u_{K}(x,0)+t\norm{\f_K}^2,\quad \forall t\geq 0, \label{fd} \end{align} whereas \begin{align} \norm{\u_K(t)}^2- \left(\int_{\Gamma}\f_K(x)\cdot \u_{K}(x,t)\right)^2= \norm{\u_K(0)}^2- \left(\int_{\Gamma}\f_K(x)\cdot \u_{K}(x,0)\right)^2,\quad \forall t\geq 0. \label{uk} \end{align} As a consequence, \begin{align} \norm{\u_K(t)}^2=\norm{\u_K(0)}^2+t^2\norm{\f_K}^4+2t\norm{\f_K}^2\int_{\Gamma}\f_K(x)\cdot \u_{K}(x,0),\quad \forall t\geq 0,\label{uk2} \end{align} entailing $\norm{\u_K(t)}\to \infty$ as $t\to \infty$ in case $\f_K \not = 0$. \item[(ii)] If \begin{align} \int_\Gamma \f_K(x, \u)\cdot \u\leq 0, \label{nega} \end{align} for any $\u\in \L^2_\sigma(\Gamma)$, then $$ \norm{\u_K(t)}\leq \norm{\u_K(0)},\quad \forall t\geq 0. $$ \item[(iii)] If \begin{align} \int_\Gamma \f_K(x, \u)\cdot \u\geq 0, \label{pos}\end{align} for any $\u\in \L^2_\sigma(\Gamma)$, then $$ \norm{\u_K(t)}\geq \norm{\u_K(0)},\quad \forall t\geq 0. $$ \end{itemize} \end{prop} \begin{osse}\rm ($\f_{K}$ antisymmetric) Notice that a necessary condition for \eqref{nega} or \eqref{pos} to hold (and $\f_K\not = 0$) is that $\f_{K}$ is antisymmetric with respect to $\u$, i.e., $\f_K(x,\u)=-\f_K(x,-\u)$. \end{osse} \begin{osse}\rm (Weaker assumptions) We observe that, in case (iii) of Proposition \emph{\ref{lemmadec}} one can also assume the weaker assumption $\int_\Gamma \f_K(x, \u)\cdot \u\geq 0$, for any $\u\in \L^2_\sigma(\Gamma)$ such that $\norm{\Pk\u}\geq R_0$, for some $R_0\geq0$. In this case, it is immediate to deduce by a similar argument that, if there exists $t_0\geq0$ such that $\norm{\u_K(t_0)}\geq R_0$, then $\norm{\u_K(t)}\geq \norm{\u_K(t_0)}$ for any $t\geq t_0$. In particular, if $\norm{\u_K(0)}\geq R_0$, then $\norm{\u_K(t)}\geq \norm{\u_K(0)}\geq R_0$ for any $t\geq 0$. %{\color{red}[Forse qui andrebbe messa una dimostrazione, \label{relax} \end{osse} Our second general result is showing that, possibly depending on the behavior of $\u_{K}$, the function $\u_{NK}$ has an exponential decay {(up to some additive uniform constant)} as time goes to infinity, as long as the constant {$C_1,C_2$} appearing in the Theorem \ref{thm1} are sufficiently small compared to the viscosity lower bound $\nu_*$. This reflects the fact that the interaction between $\u_K$ and $\u_{NK}$ is mostly due to the transport term $\overline{\u}\cdot \nabla_\Gamma\overline{\u}$, which does not appear in the energy estimate. The proof is presented in Section \ref{sec:prel}. \begin{prop}[Bounds on the non-Killing component $\u_{NK}$] \label{pp} Under assumptions \eqref{C1}--\eqref{C2} and \eqref{uk1}, if additionally there exist $C_5,\, C_6>0$ such that \begin{align} \int_\Gamma{\f_{NK}(x,\u)\cdot \u}\leq C_5 \norm{(\boldsymbol I-\P_{\mathcal{K}})\u}^2+ C_6 \norm{(\boldsymbol I-\P_{\mathcal{K}})\u},\quad \forall \u\in \Lts, \label{extra} \end{align} with $\zeta:=\frac{2\nu_*}{C_P^2}-2 C_5 >0$ ($C_P$ is defined in \eqref{Korn3}), then there exists $\omega>0$ such that \begin{align}\label{gr} \norm{\u_{NK}(t)}^2\leq e^{-\zeta t}\norm{\u_{NK}(0)}^2+\omega,\quad \forall t\geq0, \end{align} for any $\u_0\in \Lts$. \noindent On the other hand, under the additional assumption \eqref{nega}, assuming \begin{align} \int_\Gamma{\f_{NK}(x,\u)\cdot \u}\leq C_5 \norm{(\boldsymbol I-\P_{\mathcal{K}})\u}^2+ C_6 \norm{(\boldsymbol I-\P_{\mathcal{K}})\u}+ C_6 \norm{\Pk\u}^2,\quad \forall \u\in \Lts, \label{extra2} \end{align} and again $\zeta:=\frac{2\nu_*}{C_P^2}-2 C_5 >0$, there exists $\omega>0$ such that \begin{align}\label{gr2} \norm{\u_{NK}(t)}^2\leq e^{-\zeta t}\norm{\u_{NK}(0)}^2+\omega(1+\norm{\u_K(0)}^2),\quad \forall t\geq0, \end{align} for any $\u_0\in \Lts$. \end{prop} \begin{osse}\rm (More general $\f_{NK}$) Thanks to the results of Proposition \ref{lemmadec} case (ii), i.e., under the additional assumption \eqref{nega}, we can assume more general hypotheses of the behavior of $\f_{NK}$ {(see \eqref{extra2})}, which is now allowed to depend also on $\u_K$ in the estimate from above. This means that we can assume some influence of the Killing component on the non-Killing component of the trajectory. \end{osse} \begin{osse}\rm (Case $\f_{NK}= 0$) Note that, in the case $\f_{NK}= 0$, we deduce $$ \norm{\u_{NK}(t)}^2\leq e^{-\frac{4\nu_*}{C_P^2}t}\norm{\u_{NK}(0)}^2, $$ so that $\u_{NK}\to 0$ exponentially as $t\to \infty$, see \eqref{energy} in the proof in Section \ref{sec:prel}, having a right-hand side $0$ in this case. {This is in agreement with the results in \cite{Simonett2}.} \end{osse} \begin{osse}\rm (Examples) Some possible examples of forcing terms satisfying the assumptions of Proposition \ref{lemmadec} and \eqref{pp} are, for instance, \begin{align*} &\f_1=\f_1(x)\in \L^2_\sigma(\Gamma),\\& \f_2^{\pm}=\f_2^{\pm}(x,\u)=\v\pm \P_\mathcal K\u,\quad \text{for some fixed }\v\in (\boldsymbol I-\P_\mathcal K)\L^2_\sigma(\Gamma),\\& \f_3^{\pm}=\f_3^{\pm}(\u)=\pm\u,\\& \f^{\pm}_4=\f_4^{\pm}(x,\u)=(\boldsymbol I-\P_\mathcal K)\u\pm\P_\mathcal{K}(\norma{x-p}\P_\mathcal K\u), \quad \text{for some fixed }p\in \Gamma,\\& \f_5=\f_5(x,\u)=(\I-\Pk)(\vert x\vert \u)-\u. \end{align*} Note that in the case of $\f_4$ it holds \begin{align*} \f_{4,K}^\pm(x,\u):=\pm\vert x-p\vert \Pk\u,\quad \f_{4,NK}^\pm(x,\u):=(\I-\Pk)\u, \end{align*} whereas, in the case $\f_5$, \begin{align*} \f_{5,K}(x,\u):= -\Pk\u,\quad \f_{5,NK}(x,\u):=(\I-\Pk)(\vert x\vert \u-\u), \end{align*} and it clearly satisfies both case (ii) of Proposition \ref{lemmadec} as well as \eqref{extra2}, since $$ \int_\Gamma \f_{5,NK}(x,\u)\cdot \u\leq C\norm{(\I-\Pk)(\vert x\vert \u)}\norm{(\I-\Pk)\u}+C\norm{(\I-\Pk)\u}^2\leq C\norm{(\I-\Pk)\u}^2+C\norm{\Pk\u}^2, $$ recalling $\norm{\u}^2=\norm{\Pk\u}^2+\norm{(\I-\Pk)\u}^2.$ \label{remf} \end{osse} \begin{osse}\rm (Discussion of the examples) Concerning the forcing terms proposed in Remark \ref{remf}, we see that case (i) of Proposition \emph{\ref{lemmadec}} corresponds to $\f_1$. Then $\f_2^-,\f_3^-,\f_4^-,\f_5$ are related to case (ii), whereas $\f_2^+,\f_3^+,\f_4^+$ fall into case (iii). Concerning $\f_2^\pm,\f_3^\pm$, we observe that they both correspond to $\f_{K}=\pm\u_K$, so that, more precisely we deduce $$ \frac12\frac{d}{dt}\norm{\u_K}^2=\norm{\u_K}^2, $$ i.e., $$ \norm{\u_K(t)}^2=e^{\pm 2t}\norm{\u_K(0)}^2, $$ so that $\u_K(t)\to 0$ as $t\to \infty$ in the cases $\f_2^-,\f_3^-$, whereas $\norm{\u_K(t)}\to +\infty$ as $t\to\infty$ for $\f_2^+,\f_3^+$. \noindent In conclusion, in the case $\f_K= \boldsymbol 0$, by uniqueness (see Proposition \ref{dec}) we simply have $$\u_K(t)\equiv \u_K(0),\quad \forall t\geq0.$$ \end{osse} Under assumptions \eqref{uk1}, \eqref{nega}, and \eqref{extra2}, we can show that any weak solution instantaneously regularizes and, in particular, all its norms are uniformly bounded by the initial energy. Namely, we have the following (see Section \ref{sec:prel} for the proof) \begin{teor}[Instantaneous regularization]\label{insta} Let assumptions \eqref{C1}--\eqref{C2} hold, together with \eqref{uk1}, \eqref{nega}, and \eqref{extra2}. Then the unique weak solution $\u$ instantaneously regularizes, i.e., $\u$ is such that, for any $\tau>0$, \begin{align} & \boldsymbol{u}\in BUC([\tau,\infty );\boldsymbol{H}^{1}(\Gamma )),\label{regg1}\\& \u \in H^1(t,t+1;\L^2_\sigma(\Gamma))\cap L^2(t,t+1,\H^2_\sigma(\Gamma)),\quad \forall t\geq \tau.\label{regg2} \end{align} The solution $\u$ is uniformly bounded in the above spaces just in terms of $\tau$, $\f$, $\Gamma$, $\norm{\u_0}$ and of the the parameters of the problem. Moreover, if $\u_0\in \H^1_\sigma(\Gamma)$, one can choose $\tau=0$ in \eqref{regg1}--\eqref{regg2}. In this case, the above-mentioned bounds also depend on $\norm{\u_0}_{\H^1(\Gamma)}$. \end{teor} \begin{osse}\rm (Initial datum) We point out that the instantaneous regularization in \eqref{regg1}--\eqref{regg2} strongly depends on the initial energy of the complete datum $\u_0$, meaning that also the effect of the $\L^2$-norm of the $\L^2$-projection of $\u_0$ on the Killing field space $\mathcal K$, i.e., $\norm{\Pk\u_0}$, has an influence on the $L^\infty_t\H_\sigma^1$-norm of the complete solution $\u$. {This means that, even in the case $\ff=\zero$, the Killing and non-Killing components of $\u$ cannot be decoupled when obtaining higher-order \textit{a priori }bounds, in contrast to what happens at the level of the basic energy estimate. This is of course due to the convective term $(\u\cdot\nabla_\Gamma)\u$.} \label{effect} \end{osse} \subsection{The notion of attractor for the dynamical system $(S(t),\Lts)$}\label{notion} The natural object to look for in the case of a dissipative dynamical system is the global attractor, which is usually defined as either the maximal compact invariant set, the minimal closed set which uniformly attracts all bounded sets, or the set of points on complete bounded trajectories. For many dissipative systems, and in particular for the Navier--Stokes equations in 2D bounded domains (see, e.g., \cite{Foias2}), these definitions are equivalent. For these standard dissipative systems (i.e., with bounded absorbing sets), the global attractor $\mathcal{A}$ is compact, invariant, and attracting, entailing in particular that the following properties hold \begin{enumerate} \item If $\mathcal{F}$ is a bounded invariant set, then $\mathcal{F}\subset \mathcal A$, i.e., $\mathcal A$ is the maximal bounded invariant set. \item If $\mathcal G$ is a closed attracting set, then $\mathcal A\subset \mathcal G$, i.e., $\mathcal A$ is the minimal closed attracting set. \end{enumerate} It is easy to see that these two properties entail uniqueness of the global attractor $\mathcal A$, since it is itself closed and bounded. For the dynamical system $(S(t),\Lts)$ in study, the definition asks for a modification. In particular, the compactness requirement (and, more in general, the boundedness) must be dropped, in consideration of the possible choices of the forcing term $\f$. In the following, we introduce suitable notions of attractors, which to some extent recover the properties (1) and (2) above. Note on the other hand that uniqueness may be lost, since the attractor might be unbounded. The basic features of the global attractor which we might aim at preserving in our new notion are the invariance property and the fact that it is the minimal closed set which attracts all bounded sets (i.e., property (2) above). This means that a good candidate is any set $\mathcal{C}\subset \Lts$ such that \begin{align*} &S(t)\mathcal C=\mathcal C\quad\forall t\geq 0, \end{align*} and, if there exists a closed set $\mathcal{D}$ such that $$ \lim_{t\to +\infty}\text{\rm dist}(S(t)B,\mathcal D)=0,\quad \forall B\subset \Lts, \quad B\text{ bounded set}, $$ then $\mathcal{C}\subset \mathcal{D}$. As a preliminary result, we can immediately show that, if we simply choose $\f_K$ to be independent of $\u$, i.e., $\f_K:\Gamma \to \T\Gamma$, as in the standard case of 2D Navier--Stokes equations on bounded domains (\cite{Foias2}), such a set $\mathcal C$ is necessarily empty. To this aim, we first state the following general result. \begin{lemm}[Emptyness of invariant sets] Consider the dynamical system $(S(t),\Lts)$. If there exists a closed set $\mathcal{D}\subset \Lts$ such that \begin{align} \lim_{t\to +\infty}\text{\rm dist}(S(t)B,\mathcal D)=0,\quad \forall B \ \text{bounded in} \ \Lts,\label{bdd} \end{align} and such that \begin{align} \lim_{t\to\infty}\inf_{\u\in \mathcal{D}}\norm{S(t)\u}=+\infty, \label{inf} \end{align} then any invariant set $\mathcal C\subset \Lts$ with $\mathcal C\subset \mathcal D$ is empty.\label{empty} \end{lemm} \begin{proof} Assume by contradiction that $\mathcal C\not=\emptyset$. Since by the invariance property $S(t)\mathcal{C}=\mathcal{C}$, for any $t\geq0$ and $\u\in \mathcal{C}$ there exists a sequence $\{\u_n\}_n\subset \mathcal{C}$ such that $$ \u=S(n)\u_n,\quad \forall n\in\N. $$ Hence, using the fact that $\mathcal C\subset \mathcal D$, \begin{align*} \infty>\norm{\u}=\norm{S(n)\u_n}\geq \inf_{\v\in \mathcal C}\norm{S(n)\v}\geq \inf_{\v\in \mathcal D}\norm{S(n)\v}\to +\infty, \end{align*} a contradiction. \end{proof} Exploiting this lemma and Proposition \ref{dec}, we immediately obtain the following proposition, which applies to the case of $\f_K$ being constant in time. \begin{prop}\label{const} Under the assumptions of Propositions \emph{\ref{dec}--\ref{pp}} (in particular, by assuming \eqref{extra}), if $\f_K\not= \boldsymbol 0$ satisfies case (i) of Proposition \emph{\ref{lemmadec}}, i.e., it is independent of $\u\in \Lts$, then any invariant set $\mathcal C\subset \Lts$ which is also the minimal closed set uniformly attracting all bounded sets, is empty. \end{prop} \begin{proof} We first introduce the set $\mathcal B\subset \Lts$ as \begin{align} \mathcal B:=\left\{\v\in \Lts:\ \norm{(\boldsymbol I-\P_\mathcal K)\v}\leq \sqrt{\frac12+\omega},\quad \int_\Gamma \P_{\mathcal{K}}\v\cdot \f_K\geq 0 \right\},\label{BB} \end{align} where $\omega>0$ is given in \eqref{gr}. Thanks to \eqref{fd} and \eqref{gr} we can prove that for any bounded set $B\subset \Lts$ there exists a finite $t_B>0$ such that \begin{align} S(t)B\subset \mathcal B,\quad \forall t\geq t_B,\label{absorbing} \end{align} i.e., $\mathcal B$ is an absorbing set. To see this, we set $\u_{NK}(t):=(\boldsymbol I-\P_K) S(t)\u_0$, and $\u_K(t):=\P_{\mathcal{K}}S(t)\u_0$, for $\u_0\in B$. By \eqref{gr}, there exists $t_{B,1}>0$, depending only on $\norm{(\boldsymbol I-\P_{\mathcal{K}} )B}:=\sup_{\v\in (\boldsymbol I-\P_{\mathcal{K}} ) B}\norm{\v}\leq \sup_{b\in B}\|b\|=: \norm{B}\leq C_B$, such that $$ \norm{\u_{NK}(t)}\leq \frac12+\omega,\quad \forall t\geq t_{B,1}. $$ Moreover, since $B$ is a bounded set, also $\norm{\Pk B}\leq C_B$, so that $$ \norma{ \int_\Gamma \f_K\cdot \v} \leq \norm{\f_K}\norm{\v}\leq C_B\norm{\f_K}, $$ and thus, from \eqref{fd}, $$ \int_{\Gamma}\f_K\cdot \u_K\geq -C_B\norm{\f_K}+t\norm{\f_K}^2\to +\infty \quad\text{as }t\to \infty, $$ entailing that there exists $t_{B,2}>0$, depending only on $\norm{B}$, such that $$ \int_\Gamma \f_K\cdot \u_K\geq 0,\quad \forall t\geq t_{B,2}. $$ Choosing $t_B:=\max\{t_{B,1},t_{B,2}\}$, we have shown \eqref{absorbing}, since this time does not depend on the specific $\u_0\in B$, but only on $\norm{B}$. Now, we show that the absorbing (closed) set $\mathcal D:=\mathcal B$ satisfies the assumptions of Lemma \ref{empty}. The fact that $\mathcal{B}$ satisfies \eqref{bdd} is straightforward from its absorbing property. Concerning property \eqref{inf}, we have, for any $\u_0\in \mathcal{B}$, \begin{align*} \norm{\u_K(t)}^2=\norm{\u_K(0)}^2+t^2\norm{\f_K}^4+2t\norm{\f_K}^2\int_{\Gamma}\f_K(x)\cdot \u_{K}(x,0)\geq t^2\norm{\f_K}^4, \end{align*} since on $\mathcal B$ the last summand is always nonnegative. Therefore, this implies \begin{align*} \norm{\P_{\mathcal{K}}S(t)\u_0}\geq t \|\f_K\|^2, \forall \u_0\in \mathcal{B}. \end{align*} Since $\norm{S(t)\u_0}\geq \norm{\P_{\mathcal{K}}S(t)\u_0}$, this entails \begin{align*} \inf_{\u_0\in \mathcal{B}}\norm{S(t)\u_0}\geq t\norm{\f_K}^2\to +\infty \quad\text{as } t\to \infty, \end{align*} i.e., property \eqref{inf} holds. Moreover, since $\mathcal C$ is an invariant set which is the minimal closed set attracting all bounded sets, it holds that $\mathcal C\subset \mathcal D=\mathcal B$. Therefore, Lemma \ref{empty} shows that $\mathcal C=\emptyset$, i.e., any invariant set $\mathcal C\subset \Lts$ which is also the minimal closed set uniformly attracting all bounded sets is empty. This concludes the proof. \end{proof} In constrast with the classical results for the Navier--Stokes equations in 2D bounded domains, here the case $\f_K$ independent of the solution $\u$ does not allow to give a satisfying notion of attractor. Therefore we will assume from now on that $\f_K$, if not exactly equal to zero, also depends on $\u$. In general, the task to find an attractor with good properties is not easy, and strongly depends on the characteristics of the forcing term $\f_K$. Following \cite{Chepyzov}, we are interested in the set of \textit{bounded-in-the-past} complete trajectories \begin{align} \mathcal{J}:=\{\xi(0)\in \Lts:\ \xi \text{ is a bounded-in-the-past complete trajectory for $S(t)$}\},\label{J} \end{align} where $\xi : \R \to \Lts$ is a bounded-in-the-past complete trajectory of $S$ if $S(t)\xi(s) = \xi(t +s)$ for all $t\geq 0$ and $s\in\R$, and $\xi((-\infty, 0])$ is a bounded subset of $\Lts$. In the standard case of dissipative systems, where all trajectories are bounded, this set coincides with the set \begin{align} \mathcal{I}:=\{\xi(0)\in \Lts:\ \xi \text{ is a bounded complete trajectory for $S(t)$}\},\label{I} \end{align} where $\xi((-\infty, +\infty))$ is a bounded subset of $\Lts$, and the global attractor exactly coincides with this set. In our case of the system $(S(t),\Lts)$, it is not \textit{a priori} ensured that all trajectories are bounded in the future, and thus in general we can only expect $\mathcal I \subset \mathcal J$. In general, the interest in $\mathcal J$ as a descriptor of the long-time behavior of the system is given by the following trivial lemma (see for instance \cite[Proposition 3]{Bortolan}), which we prove here for completeness. \begin{lemm}[Properties of $\mathcal J$]\label{obv} The set $\mathcal J$ satisfies the following properties: \begin{enumerate} \item[A.] $\mathcal J$ is invariant, i.e., $S(t)\mathcal J=\mathcal J$ for any $t\geq 0$. \item[B.] If a closed set $\mathcal D\subset \Lts$ attracts all bounded sets, then $\mathcal J\subset \mathcal D$. \item[C.] If $A\subset \Lts$ is a nonempty bounded invariant set, then $A\subset \mathcal I\subset \mathcal J$ and thus $\mathcal J$ is nonempty. \end{enumerate} \end{lemm} \begin{proof} The invariance property of $\mathcal J$ is easily checked. Let $\v\in \mathcal J$, so that there exists a complete trajectory $\xi$ bounded-in-the-past with $\xi(0)=\v$. Note that $\xi_r:=\xi(\cdot+r)$ for any $r\in \R$ is still a bounded-in-the-past complete trajectory, so that, for any $t\geq0$, $S(t)\v=\xi_t(0)$ and thus by definition $S(t)\mathcal J\subset \mathcal J$. Analogously, since $\xi_{-t}(0)\in \mathcal J$ for any $t\geq 0$, it holds that $\v=S(t)\xi_{-t}(0)\in S(t)\mathcal J$, and thus $\mathcal J\subset S(t)\mathcal J$, i.e., $\mathcal J$ is invariant. To prove assertion B., let us fix $\v\in \mathcal J$ and let $\xi$ be a complete trajectory which is bounded-in-the-past with $\xi(0)=\v$. Consider $B:=\xi((-\infty,0])$, which is a bounded set containing $\v$ {and such that $B\subset S(t)B$ for all $t\geq0$}, and observe that $$ \text{\rm dist}(B,\mathcal D){\leq}\text{\rm dist}(S(t)B,\mathcal D)\to 0,\quad \text{as} \ t\to\infty. $$ Since $\mathcal D$ is closed, this means $\v\in B\subset \mathcal D$, entailing $\mathcal J\subset \mathcal D$. To prove property C., we recall that, by \cite[Lemma 1.4]{Robinsonbook}, a set $A\subset \Lts$ is invariant if and only if for any $\v\in A$ there exists a complete trajectory $\xi$ such that $\xi(\R)\subset A$. Therefore if we assume $A\subset \Lts$ to be bounded and invariant, then clearly $A\subset \mathcal I$. \end{proof} \begin{osse}\rm ($\mathcal I$ is invariant) It is immediate to verify that also $\mathcal I$ is an invariant set, i.e., $S(t)\mathcal I=\mathcal I$, for any $t\geq 0$. Note also that, given any complete bounded or bounded-in-the-past trajectory $\xi$, it clearly holds $\xi(\R)\subset \mathcal I$ or $\xi(\R)\subset \mathcal J$, respectively. \end{osse} \begin{osse}\rm (Case $\mathcal J\not =\emptyset$) We point out that if we prove that $\mathcal J$ is nonempty, closed, and attracts all bounded sets of $\Lts$, then by property B. of Lemma \emph{\ref{obv}} we deduce that it is also the minimal closed attracting set. Then $\mathcal J$ satisfies properties (1) and (2) described above for the classical global attractor for dissipative dynamical systems: the only property which is missing is the compactness (in particular, the boundedness). This property will be retrieved in a weaker sense: we will prove that $\mathcal J$ is in many cases \emph{bounded compact}, i.e., the intersection of $\mathcal J$ with any closed and bounded set is compact. \end{osse} A first nontrivial task is to prove that $\mathcal J$ is nonempty. Note that this is generally false. Indeed, for instance in the case of $\f_K$ independent of the velocity it is immediate to see that $\mathcal J=\emptyset$: \begin{lemm}[Case $\mathcal J =\emptyset$] Under the assumptions of Propositions \emph{\ref{dec}--\ref{pp}}, if $\f_K\not = \boldsymbol 0$ satisfies case (i) of Proposition \emph{\ref{lemmadec}}, i.e., it is independent of $\u\in \Lts$, then $\mathcal J=\emptyset$. \end{lemm} \begin{proof} Since the set $\mathcal{B}$ introduced in \eqref{BB} is absorbing, then, by property B. of Lemma \ref{obv}, it holds $\JJ\subset \mathcal{B}$. Since $\JJ$ is invariant and $\JJ\subset \mathcal B$, we can apply Lemma \ref{empty}, with $\mathcal{D}=\mathcal{B}$ and $\mathcal C=\JJ$ and conclude that $\JJ=\emptyset$. \end{proof} Our objective then becomes to first show that $\mathcal J$ is nonempty, and then further ascertain the properties of the set $\mathcal J$ (or $\mathcal I$, in some cases), such as being the minimal closed set attracting all bounded sets. We distinguish two cases, corresponding to cases (ii) and (iii) of Proposition \ref{lemmadec}. They will be treated with a completely different approach. \subsection{Attractors for bounded trajectories} \label{sec:attr} We first focus on case (ii) of Proposition \ref{lemmadec}. Namely, we assume \eqref{nega}, that is $$\int_\Gamma \f_K(x, \u)\cdot \u\leq 0,\quad\text{for any } \u\in \Lts.$$ In this situation, it is clear from Propositions \ref{dec}--\ref{pp} that any trajectory is bounded, i.e., given any $\u_0\in \Lts$, $S([0,+\infty))\u_0$ is bounded in $\Lts$, entailing that $$ \mathcal J=\mathcal I. $$ In this case, we can fully characterize the set $\mathcal J$ (which is proven to be nonempty), and we call this set \textit{$\sigma$-attractor}, in analogy to what has been first introduced in \cite{Foias} to deal with systems with similar dissipativity properties. This name comes from the fact $\mathcal J$ can be obtained as the countable union of nonempty, compact, finite-dimensional (with respect to the fractal dimension) sets. We recall that the fractal dimension of a compact set $A\subset \Lts$ is defined as $$ \text{dim}_{\Lts}(A)=\limsup_{\epsilon\to0}\frac{\log N(\epsilon)}{-\log \epsilon}, $$ and $N(\epsilon)$ is the minimum number of $\epsilon$-balls of $ \Lts$ necessary to cover $A$. We now introduce the following notation: for any $r\geq0$, we define \begin{align} \mathbb B_r:=\{\v\in \Lts:\ \norm{\P_{\mathcal{K}}\v}\leq r\}, \label{Ar} \end{align} which is a complete metric space if endowed with the distance induced by the norm of $\Lts$, i.e., the $\L^2(\Gamma)$-norm. We will show that, for any $r\geq0$, the dynamical system $(S(t),\mathbb B_r)$ admits a finite-dimensional global attractor $\mathcal A_r$, namely, fulfilling the following properties \begin{itemize} \item $\mathcal A_r$ is nonempty, compact, connected, and of finite fractal dimension, \item $\mathcal A_r$ is invariant, i.e., $S(t)\mathcal A_r=\mathcal A_r$, for any $t\geq0$, \item $\mathcal A_r$ is attracting, i.e., for any $B\subset \mathbb B_r$ bounded it holds ${\rm dist}(S(t)B,\mathcal A_r)\to 0$ as $t\to \infty$. \end{itemize} Clearly, from these properties we can also deduce that $\mathcal A_r$ is the maximal bounded invariant set, as well as the minimal closed attracting set. Moreover, it also holds \begin{align} \mathcal A_r=\{\xi(0)\in \mathbb B_r:\ \xi \text{ is a bounded complete trajectory for $S(t)$ in }\mathbb B_r \}, \label{bdd_traj} \end{align} which clearly entails $\mathcal A_r\subset \mathcal A_s$ if $0\leq r\leq s$. Indeed, if $\xi$ is a complete bounded trajectory in $\mathcal A_r$, one has that $\xi(\R)\subset \mathcal A_r\subset \mathbb B_r$, but, since $\mathbb B_r\subset \mathbb B_s$, then also $\xi$ is a complete bounded trajectory in $\mathbb B_s$, and thus $\xi(\R)\subset \mathcal A_s$. We can now state our main result of this section (which is proven in Section \ref{bdda}). \begin{teor}[$\sigma$-attractor]\label{thm:323} Under the assumptions \eqref{C1}--\eqref{C2}, \eqref{uk1}, and \eqref{extra2}, if $\f_K$ is such that \begin{align} \int_\Gamma \f_K(x, \u)\cdot \u\leq 0,\quad\text{for any } \u\in \Lts, \label{neg} \end{align} then the nonempty set $\mathcal J$ defined in \eqref{J}, coinciding with $\mathcal I$ defined in \eqref{I}, is the $\sigma$-attractor and enjoys the following properties: \begin{enumerate} \item $\mathcal J=\bigcup_{r\geq 0}\mathcal A_r=\bigcup_{n\in \N}\mathcal A_n$, where $\mathcal A_r\subset\H^1_\sigma(\Gamma)$ are the global attractors to the dynamical system $(S(t), \mathbb B_r)$ for any $r\geq 0$. Namely, $\mathcal A_r$ are nonempty, compact, of finite fractal dimension, invariant, and connected. \item $\mathcal J\subset \H^1_\sigma(\Gamma)$ has empty interior in $\Lts$. \item $\mathcal J$ attracts all bounded sets. More precisely, if $B\subset \Lts$ is a bounded set, then there exists $r>0$ such that $${\rm dist}(S(t)B,\JJ)\leq {\rm dist}(S(t)B, \mathcal A_r)\to 0$$ as $t\to \infty$. \item $\mathcal J$ is invariant, i.e., $S(t)\mathcal J =\mathcal J$ for all $t\geq0$. \item $\mathcal J$ is minimal among closed sets attracting all bounded sets, which means that if there exists a closed set $\mathcal{D}$ such that $$ \lim_{t\to +\infty}\text{\rm dist}(S(t)B,\mathcal D)=0,\quad \forall B\subset \Lts, \quad B\text{ bounded set}, $$ then $\mathcal{J}\subset \mathcal{D}$. \end{enumerate} \label{th1} \end{teor} \begin{osse}\rm (Complete trajectories do not cross) We point out that, thanks to the backward-uniqueness result of Theorem \ref{backuni}, together with the well-posedness given in Theorem \ref{thm1}, since any complete trajectory $\xi$ belongs to $\JJ\subset \H^1_\sigma(\Gamma)$, and thus it is a strong solution to \eqref{t1}--\eqref{tangential}, if any two complete trajectories intersect, then they must be identical. In particular, two distinct complete trajectories $\xi$ cannot intersect. This means than one could require the semigroup $S(t)$ on $\JJ$ to be defined for negative times, as well. \end{osse} As a by-product of Theorem \ref{thm:323}, we also have the existence of a set $\mathcal M$ which can be defined as a $\sigma$-exponential attractor, in the sense that it is the {countable} union of the exponential attractors $\mathcal M_m$ for each dynamical system $(S(t),\BBB_m)$, $m\in \mathbb N$. We recall that an exponential attractor $\mathcal M_r$ (which is possibly not unique) for the dynamical system $(S(t),\BBB_r)$, {$r\geq0$}, enjoys the properties: \begin{itemize} \item $\mathcal M_r$ is compact and of finite fractal dimension $N_r$, possibly increasing with $r\geq0$, \item $\mathcal M_r$ is positively invariant, i.e., $S(t)\mathcal M_r\subset \mathcal M_r$, for any $t\geq0$, \item $\mathcal M_r$ is exponentially attracting, i.e., for any $B\subset \mathbb B_r$ bounded it holds $${\rm dist}(S(t)B,\mathcal M_r)\leq Q_r(\norm{B}_\Lts)e^{- \gamma_r t},$$ where $Q_r>0$ is an increasing function of $\norm{B}_ \Lts$, only depending on $r$, and $\gamma_r>0$ is a universal constant depending only on $r$. Here, we have used the notation $\norm{A}_{\Lts}=\sup_{\u\in A}\norm{\u}$, for any bounded set $A$. \end{itemize} As $\mathcal A_r\subset \mathcal M_r$, if $ \mathcal M_r$ has finite fractal dimension, so does $\mathcal A_r$. All the following results are proven in Section \ref{bdda}. We then have the following \begin{teor}[$\sigma$-exponential attractor]\label{thm:325} Under the assumptions \eqref{C1}--\eqref{C2}, \eqref{uk1}, and \eqref{extra2}, if $\f_K$ is such that $$\int_\Gamma \f_K(x, \u)\cdot \u\leq 0,\quad\text{for any } \u\in \Lts,$$ then there exists a set $\mathcal M$ enjoying the following properties: \begin{enumerate} \item ${\mathcal M=\bigcup_{m\in \mathbb N} \mathcal M_m}$, {where $\mathcal M_m$ are exponential attractors for the dynamical system $(S(t),\BBB_m)$, for any $m\in \mathbb N$}, \item $\mathcal M$ is positively invariant, i.e., $S(t)\mathcal M\subset \mathcal M$, for any $t\geq0$, \item {$\mathcal M$ is exponentially attracting, i.e., for any bounded set $B\subset \mathbb B_m$, for some $m\in \mathbb N$, it holds $${\rm dist}(S(t)B,\mathcal M)\leq {\rm dist}(S(t)B,\mathcal M_m)\leq Q_m(\norm{B}_\Lts)e^{-\gamma_m t},\quad \forall t\geq0,$$ where $Q_m>0$ is an increasing function of $\norm{B}_ \Lts$, only depending on $m$, and $\gamma_m>0$ is a universal constant depending only on $m$.} \end{enumerate} \end{teor} {\begin{osse}(Nonuniqueness of the $\sigma$-exponential attractor) We point out here that, since for any $m\in \mathbb N$ an exponential attractor might be in general not unique, also the $\sigma$-exponential attractor we construct in the theorem above is not unique. Additionally, also the uncountable union $\bigcup_{r\geq 0}\mathcal M_r$, where $\mathcal M_r$ is an exponential attractor for the dynamical system $(S(t),\BBB_r)$, {$r\geq0$}, has the same properties (1)-(3) given in Thorem \ref{thm:325}. Neverheless, since in general {it is not ensured that $\mathcal M_r\subset \mathcal M_s$ for any $0\leq r\leq s$, the $\sigma$-exponential attractor $\mathcal M$ given by Theorem \ref{thm:325} is possibly smaller (and thus more desirable), since it clearly holds $\mathcal M\subset \bigcup_{r\geq 0}\mathcal M_r$. } \end{osse}} In conclusion, we can further refine the results of Theorem \ref{th1} if we give some specific structure to the forcing term $\f$. In particular, we have the following \begin{teor}[Case $\f_K=0$]\label{spe} Under the assumptions \eqref{C1}--\eqref{C2} and \eqref{extra2}, if $\f_K= \boldsymbol 0$ or $\f_K(\cdot,\u)=\Pk((\v\cdot \nabla_\Gamma) \u_K)$, for $\v\in \L^2_\sigma(\Gamma)\cap \L^\infty(\Gamma)$, then the $\sigma$-attractor $\JJ$, defined in Theorem \emph{\ref{th1}}, possesses a so-called \emph{pancake-like} structure, { \cite{Foias}}, i.e., $$ \JJ=\bigcup_{r\geq 0}\widetilde{\AA}_r, $$ where, having defined the complete metric space (endowed with $\Lts$ topology) \begin{align} \label{tB} \widetilde{\BBB}_r:=\{\u\in \Lts:\ \norm{\Pk\u}=r\}, \end{align} the compact, invariant, attracting, finite-fractal dimensional set $\widetilde{\AA}_r$ is the global attractor of the dynamical system $(S(t),\widetilde{\BBB}_r)$, for any $r\geq 0$. Furthermore, $\Pk \JJ=\KK$ and $\JJ$ is closed, bounded compact, and of finite fractal dimension, i.e., the intersection of $\mathcal J$ with any closed and bounded set $B\subset \Lts$ is compact and of finite fractal dimension. In particular, there exists $r\geq 0$ such that $\mathcal A\cap B\subset \mathcal A_r$, where $\AA_{r}$ is the corresponding global attractor for the dynamical system $(S(t),\BBB_{r})$. In conclusion, if additionally $\f_{NK}=\boldsymbol 0$, then $ \JJ=\KK$. \end{teor} \begin{osse}\rm (Advective field) Notice that the theorem above also holds in case of the presence of an {external }advective field $\v\in \L^2_\sigma(\Gamma)\cap \L^\infty(\Gamma)$, in which the $\L^2$-norm of $\u_K$ does not change, but $\u_K$ is not trivially constant in time. The $\u_K$ component on the attractor $\JJ$ is then a rearranged version of the initial component $\Pk\u_0$. An example of this kind is when $\f=(\v\cdot\nabla_\Gamma)\u$, $\v\in \L^2_\sigma(\gam)\cap \L^\infty(\gam)$, which also satisfies \eqref{C1}--\eqref{C2} and \eqref{extra2}, thanks to Korn's inequality \eqref{Korn2}. \end{osse} In the case $\f_K$ is such that there exists a bounded absorbing set, we can say much more on the attractor. Notice that in this case we do not strictly need assumption \eqref{neg} on $\f_K$, since we can directly operate on the dynamical system $(S(t),\Lts)$, without any restriction. In this case, the set $\JJ$ is exactly the global attractor in the standard definition. Indeed, we have the following. \begin{teor}[Properties of $\mathcal J$, II] \label{interesting} Let the assumptions \eqref{C1}--\eqref{C2}, \eqref{uk1}, and \eqref{extra} hold. If $\f_K$ is such that there exists $C\subset \Lts$ closed and bounded, such that, for any bounded set $B\subset \Lts$, there exists $t_B\geq0$ so that $S(t)B\subset C$, for any $t\geq t_B$ (i.e., $C$ is a bounded absorbing set for the dynamical system $(S(t),\Lts)$), then the unique global attractor $\JJ$ defined in \eqref{J}, coinciding with $\mathcal I$ from \eqref{I}, is such that \begin{enumerate} \item $\mathcal J\subset C$ is compact, connected, and of finite fractal dimension, \item $\mathcal J$ is invariant, i.e., $S(t)\mathcal J=\mathcal J$, for any $t\geq0$, \item $\mathcal J$ is attracting, i.e., for any $B\subset \Lts$ bounded it holds $${\rm dist}(S(t)B,\mathcal J)\to 0,$$ as $t\to \infty$. \item $$\JJ \subset \left\{\u\in\Lts:\ \norm{(\I-\Pk)\u}\leq \sqrt{\frac 12 +\omega}\right\}.$$ \end{enumerate} Moreover, there exists an exponential attractor $\mathcal M$, which is a compact, finite-dimensional, positively invariant set such that, for any $B\subset \Lts$ bounded it holds $${\rm dist}(S(t)B,\mathcal M)\leq C(\norm{B}_\Lts)e^{-\gamma t},$$ where $C>0$ depends on $\norm{B}_{\Lts}$, and $\gamma>0$ is a universal constant. \noindent If also $\Pk S(t)B\to \boldsymbol 0$ as $t\to \infty$ for any set $B\subset \Lts$ such that $\Pk B$ is bounded, then \begin{align}\label{best} \Pk\JJ=\{\boldsymbol 0\}. \end{align} \end{teor} \begin{osse}\rm (Affine $\f$) The case $\f(x,\u):=-\u+c_0\v$, with $\v\in \KK$ and $c_0\geq0$, corresponding to $\f_K(x,\u)=-\Pk\u+c_0\v$ is clearly one example for which Theorem \emph{\ref{interesting}} applies. Notice that in this case assumption \eqref{neg} is \textit{not} satisfied. To see that the theorem holds, fixing $\u(t)=S(t)\u_0$, for some $\u_0\in \Lts$, and multiplying the equation for $\Pk\u(t)$ by $\Pk\u(t)$ we have $$ \frac12\frac{d}{dt}\norm{\Pk\u}^2= -\norm{\Pk\u}^2+\int_\Gamma c_0\v\cdot \Pk\u\leq -\norm{\Pk\u}^2+\frac12\norm{\Pk\u}^2+\frac {c_0^2}{2}\norm{\v}^2, $$ entailing \begin{align} \frac12\frac{d}{dt}\norm{\Pk\u}^2+\frac12\norm{\Pk\u}^2\leq \frac12c_0^2\norm{\v}^2 \label{p} \end{align} and thus by Gronwall's Lemma $$\norm{\Pk\u}\leq e^{-\zeta_0 t}\norm{\Pk\u_0}+\omega_0,$$ for some $\zeta_0,\omega_0>0$. Then the assumption of Theorem \ref{interesting} is satisfied for any set $B\subset \Lts$ such that $\Pk B$ is bounded. Indeed, recalling that also \eqref{gr} holds, it is easy to see that the set $$ C:=\left\{\u\in\Lts:\ \norm{\Pk\u}\leq \sqrt{\frac12+\omega_0},\quad \norm{(\I-\Pk)\u}\leq \sqrt{\frac12+\omega}\right\} $$ is a bounded absorbing set for the dynamical system. \noindent When $c_0=0$, the global attractor $\JJ$ can be further characterized as in \eqref{best}, since $\omega_0$ from \eqref{p} is $0$ in this case, and thus the $\Lts$-norm of $\Pk\u$ is subject to an exponential decay to zero. \end{osse} \begin{osse}\rm (Interactions of Killing and non-Killing components) We point out that the assumptions of Theorem \ref{interesting} are satisfied also in some cases when we assume some interaction of the non-Killing component $\f_{NK}$ on the Killing component of the trajectory. For instance, let us assume $\f$ satisfying \eqref{C1}--\eqref{C2}, and $\f_{NK}$ satisfying \eqref{extra}. If we choose, for instance, $$\f(x,\u):=-\u+\vert (\I-\Pk)\u\vert \v,$$ for some $\v\in \L^\infty(\Gamma)$, then $\f_{NK}(x,\u)=-(\I-\Pk)\u+(\I-\Pk)(\vert (\I-\Pk)\u\vert \v)$, whereas $\f_K(x,\u)=-\Pk\u+\Pk(\vert (\I-\Pk)\u\vert \v)$. Property \eqref{gr} immediately follows, whereas for the Killing component of the trajectory, we have by standard estimates, together with \eqref{gr}, \begin{align*} \frac12\frac{d}{dt}\norm{\Pk\u}^2&=-\norm{\Pk\u}^2+\int_\Gamma \vert (\I-\Pk)\u\vert \v\cdot \Pk\u \\& \leq -\norm{\Pk\u}^2+\norm{(\I-\Pk)\u}\norm{\Pk\u}\norm{\v}_{\L^\infty(\Gamma)}\\& \leq -\frac12\norm{\Pk\u}^2+\frac12\norm{(\I-\Pk)\u}^2\norm{\v}_{\L^\infty(\Gamma)}^2\\& \leq -\frac12\norm{\Pk\u}^2+\frac12(e^{-\zeta t}\norm{(\I-\Pk)\u_0}^2+\omega)\norm{\v}_{\L^\infty(\Gamma)}^2, \end{align*} where we recall $\v\in\L^\infty(\Gamma)$. By Gronwall's Lemma, this entails that \begin{align*} \norm{\Pk\u(t)}^2\leq e^{-\frac12 t}\norm{\Pk\u_0}^2+\omega_1(1+q(t)\norm{(\I-\Pk)\u_0}^2),\quad \forall t\geq 0, \end{align*} where $q(t)\to 0$ as $t\to \infty$ and $\omega_1>0$. This means that the set $$ C:=\left\{\u\in\Lts:\ \norm{\Pk\u}\leq \sqrt{\frac12+\frac32\omega_1},\quad \norm{(\I-\Pk)\u}\leq \sqrt{\frac12+\omega}\right\} $$ is an absorbing set for the whole dynamical system $(S(t),\Lts)$, and thus the assumptions of Theorem \ref{interesting} hold. \end{osse} \begin{osse}\rm (Case $\KK=\{\boldsymbol 0\}$) If $\KK=\{\boldsymbol 0\}$ one obviously has that $\Pk S(t)\u_0=\boldsymbol 0$ for any $\u_0\in\Lts$, so that also in this case the global attractor $\JJ$ is characterized by Theorem \ref{interesting}, and satisfies \eqref{best}. \end{osse} \subsection{Attractors in case of possibly unbounded trajectories} \label{unbd}In this section, we discuss the case when trajectories can be unbounded, and thus in general we do not have the coincidence between $\mathcal J$ and $\mathcal I$. In particular, we assume that $$\int_\Gamma \f_K(x, \u)\cdot \u\geq 0,\quad\text{for any } \u\in \Lts\text{ such that }\norm{\Pk\u}\geq R_0,\quad\text{for some }R_0>0,$$ corresponding to case (iii) of Proposition \ref{lemmadec} (see in particular Remark \ref{relax}). In this case we can characterize the set $\mathcal J$, which we already know that is nonempty and which we call {\it unbounded attractor} following \cite{Chepyzov}, see also the recent works \cite{Carvalho,Bortolan}. Still, we will see that many properties we expect from a global attractor are here preserved. The main drawback, which is due the fact that there might be unbounded trajectories, is the attraction property, which holds only on the bounded (as $t\to \infty$) trajectories departing from a given bounded set (see property (4) in Theorem \ref{th2} below). The other properties, especially concerning the invariance and the property of being the minimal invariant attracting closed set (in a slightly weaker version) are here present as for Theorem \ref{th1}. Also in this case, $\mathcal J$ is bounded compact. Still, we cannot recover the precise characterization of $\mathcal J$ as the countable union of compact or finite-fractal-dimensional sets. To be precise, we have the following main result, which is proven in Section \ref{unbdd}. \begin{teor}[Properties of $\mathcal J$, III]\label{thm:332} Under the assumptions \eqref{C1}--\eqref{C2}, together with assumptions \eqref{uk1} and \eqref{extra}, if $\f_K$ is such that $$\int_\Gamma \f_K(x, \u)\cdot \u\geq 0,\quad\text{for any } \u\in \Lts \text{ such that }\norm{\Pk\u}\geq R_0,\quad \text{ for some }R_0>0,$$ then the nonempty set $\mathcal J$ defined in \eqref{J} is the unbounded attractor, satisfying: \begin{enumerate} \item $\mathcal J=\overline{\bigcap_{t\geq0}S(t)Q}$, where $Q$ is given by $$ Q:=\overline{\bigcup_{t\geq 0}S(t)\left\{\v\in \Lts: \norm{(\boldsymbol I-\P_{\mathcal{K}})\v}\leq \frac12+\omega\right\}}, $$ where $\omega>0$ is given in Proposition \emph{\ref{pp}}. \item $\mathcal J$ is closed and invariant, i.e., $S(t)\mathcal J =\mathcal J$ for all $t\geq0$. \item $\JJ\subset \H^1_\sigma(\Gamma)$ and it has empty interior in the topology of $\Lts$. \item $\mathcal J$ is bounded compact, i.e., the intersection of $\mathcal J$ with any closed and bounded set $B\subset \Lts$ is compact. \item If for some $R\geq 0$, $B \in\Lts$ bounded, and $t_1 > 0$ the sets $S(t)B \cap \{\v\in \Lts:\ \norm{\P_\mathcal K \v}\leq R\}$ are nonempty for every $t \geq t_1$, then $$ \lim_{t\to\infty} {\rm dist}(S(t)B \cap \{\v\in \Lts:\ \norm{\P_\mathcal K \v}\leq R\}, \mathcal J ) = 0,$$ and $\mathcal J$ is the minimal closed set with the above property. \item $\P_\mathcal K \mathcal J = \mathcal K$. \end{enumerate} \label{th2} \end{teor} Since in general in this case one has that $\mathcal I\subset \JJ$ but the two may not coincide, we specify here the properties of $\mathcal I$, as well. \begin{coro}[Properties of $\mathcal I$] Under the same assumptions of Theorem \ref{th2} the set $\mathcal I$ defined in \eqref{I} is such that it attracts all bounded sets with bounded trajectories, i.e., if a bounded set $B\in \Lts$ is such that there exist $t_1 > 0$ and $R \geq R_0$ with $S(t)B \subset H_R$ for $t \geq t_1$, where $H_R:=\{\u\in Q:\ \norm{\Pk\u}\leq R\}$ and $Q$ is defined in Theorem \ref{th1}, then $$ \text{\rm dist} (S(t)B, \mathcal I)\to 0\quad\text{as }t\to \infty. $$ Moreover, if $\u\in \JJ\setminus \mathcal I$, then $$ \lim_{t\to \infty}\norm{S(t)\u}=+\infty. $$ \end{coro} \begin{proof} The proof of this corollary is simply an application of \cite[Theorem 7, Lemma 6]{Carvalho}, which holds as a consequence of the validity of Lemma \ref{A1}. \end{proof} In conclusion, if we make some further assumptions on the behavior of the forcing term $\f_K$, we can also obtain that the set $\mathcal I$ is nonempty, which is in general not trivial to prove. In particular, we have (see Section \ref{unbdd} for the proof) \begin{teor}[{Case $\f_K(\cdot,\mathbf 0)\equiv\mathbf 0 $}]\label{t1b} Under the same assumptions of Theorem \emph{\ref{th2}}, assume additionally that $\f_K(x, \boldsymbol 0)= \boldsymbol 0$ for any $x\in \Gamma$. Then, by defining as $\mathcal A_0$ the global attractor to the system $(S(t),(\I-\Pk)\Lts)$, we have $$ \mathcal A_0\subset \mathcal I, $$ i.e., the set of complete bounded trajectories $\mathcal I$ defined in \eqref{I} is nonempty. \label{tt} \end{teor} \begin{osse}\rm The theorem above can be easily generalized by assuming that there exists $\u_A\in \Lts$ such that $\f_K(x,\u_A)= \boldsymbol 0$ for any $x\in \Gamma$. \end{osse} \begin{osse}\rm Recalling Remark \emph{\ref{remf}}, examples of forcing terms satisfying the assumptions of Theorem \ref{t1b} are $\f_2^+,\ \f_3^+,\ \f_4^+$, and $ \f_5$. \end{osse} \section{ Well-posedness and backward uniqueness: Proofs of Section \ref{main}} \label{sec:pr1} \subsection{Proof of Theorem \ref{thm1}} The proof can be carried out in many ways. Since we need to consider both weak and strong solutions, we first prove the short-time existence of a unique strong solution exploiting the $L^p-L^q$ maximal regularity properties of the surface Stokes operator with variable viscosity (see \cite[Lemma 7.4]{AGP2024b}). Then, by means of energy estimates, we show the global existence of both weak and strong solutions to the problem. Henceforth, we use the symbol $C$ to indicate a positive constant, possibly depending on data. The value of $C$ may change from line to line. \subsubsection{Local well-posedness of strong solutions} Let us first assume that $\u_0\in \H^1_\sigma(\Gamma)$. Given $T\in(0,T_0)$, {where $T_0>0$ is chosen arbitrarily large}, we introduce the space $$ Z_T:=L^2(0,T;\H^2(\Gamma))\cap H^1(0,T;\L^2_\sigma(\Gamma)), $$ equipped with the norm \begin{align} & \Vert \f \Vert_{Z_T}:=\Vert \f \Vert_{L^2(0,T; \H^2(\Gamma))}+\Vert \f\Vert_{ H^{1}(0,T;\L^2_\sigma(\Gamma))}+\Vert \f(0) \Vert_{\H^1_\sigma(\Gamma)}, \end{align} and we recall that, as in \cite[Lemma 2]{Saal1}, by standard embeddings, there exists $C(T_0)>0$ such that \begin{align}\label{BUC} \norm{\u}_{BUC([0,T];\L^2_\sigma(\Gamma))}\leq C(T_0)\norm{\u}_{X_T}, \end{align} for any $T\in(0,T_0)$. We also denote $\YT:=L^2(0,T;\L^2_\sigma(\Gamma))$ and we introduce the space $$ \XT:=\{\u\in Z_T:\ \u(0)=\u_0\}, $$ and we rewrite system \eqref{t1}--\eqref{tangential} as follows \begin{align*} \mathcal{L}(\u)=\mathcal{F}(\u). \end{align*} Here, the linear operator $\mathcal{L}:\XT\to\YT$ is defined as \begin{align} \label{L} \mathcal{L}(\v):=\partial_t\v-2\P_0\P_\Gamma\mathrm{div}_{\Gamma}(\nu(x)\E_\Gamma(\v))+\omega \v \end{align} for some arbitrary $\omega>0$, where $\P_0$ is the Leray--Helmholtz projector defined in \eqref{Leray} and the possibly nonlinear operator $\mathcal{F}:\XT\to\YT$ given by \begin{align} \label{F} \mathcal{F}(\v):=-\P_0(\v\cdot \nabla_\Gamma)\v+\P_0\f(\cdot,\u)+\omega\v. \end{align} First, we notice that the operator $\mathcal F$ is well defined. Indeed, by standard embeddings and the continuity of the operator $\P_0$, $$ \norm{\P_0(\v\cdot \nabla_\Gamma)\v}_{\YT}\leq C\norm{\v}_{L^\infty(0,T;\L^4(\Gamma))}\norm{\v}_{L^2(0,T;\H^2(\Gamma))}\leq C\norm{\v}^2_{\XT}<+\infty, $$ as well as, from assumptions \eqref{C1}--\eqref{C2}, \begin{align*} \norm{\P_0\f(\cdot,\v)}_{\YT}&\leq C\norm{\f(\cdot,\v)-\f(\cdot,\boldsymbol 0)}_{L^2(0,T;\L^2(\Gamma))}+C\norm{\f(\cdot,\boldsymbol 0)}_{L^2(0,T;\L^2(\Gamma))}\\&\leq C\norm{\v}_{\XT}+CT<+\infty, \end{align*} so that $\mathcal F(\u)$ is well defined for any $\u\in\XT$. We aim now at proving that, under the assumptions of Theorem \ref{thm1}, there is a constant $C(T, R)>0$ such that \begin{align} \Vert\mathcal{F}(\v_1) -\mathcal{F}(\v_2)\Vert_{\YT} \leq C(T, R)\Vert(\v_1 - \v_2)\Vert_{\XT}, \label{CR}\end{align} for all $\v_i\in \XT$ with $\Vert\v_i\Vert_{\XT} \leq R$, $R>0$, and $i = 1, 2$. Furthermore it holds $C(T, R) \to0$ as ${T} \to0$. To see this, let us fix $\v_i\in \XT$ with $\Vert\v_i\Vert_{\XT} \leq R$, $R>0$, split $\mathcal F$ in its summands, and use Gagliardo--Nirenberg's inequality, the embedding $\H^1(\Gamma)\hookrightarrow \L^4(\Gamma)$, and \eqref{BUC} to write \begin{align*} &\norm{\P_0((\v_1\cdot \nabla_\Gamma)\v_1-(\v_2\cdot \nabla_\Gamma)\v_2)}_{\YT}\\&\leq \norm{\P_0((\v_1\cdot \nabla_\Gamma)\v_1-(\v_2\cdot \nabla_\Gamma)\v_2)}_{L^2(0,T;\L^2(\Gamma))}\\& \leq \norm{\v_1}_{L^\infty(0,T;\L^4(\Gamma))}\norm{\v_1-\v_2}_{L^2(0,T;\W^{1,4}(\Gamma))}+\norm{\v_1-\v_2}_{L^\infty(0,T;\L^4(\Gamma))}\norm{\v_2}_{L^2(0,T;\W^{1,4}(\Gamma))}\\& \leq C\norm{\v_1}_{\XT}\norm{\v_1-\v_2}_{L^\infty(0,T;\H^1(\Gamma))}\norm{\v_1-\v_2}_{L^1(0,T;\H^2(\Gamma))}\\&\quad +\norm{\v_1-\v_2}_{ \XT}\norm{\v_2}_{L^\infty(0,T;\H^1(\Gamma))}\norm{\v_2}_{L^1(0,T;\H^2(\Gamma))}\\& \leq C(R)T^\frac12\norm{\v_1-\v_2}_{\XT} \end{align*} Then, considering the forcing term $\f$, it holds, recalling assumption \eqref{C2}, \begin{align*} \norm{\P_0(\f(\cdot,\v_1)-\f(\cdot,\v_2))}_{\YT}&\leq C\norm{\f(\cdot,\v_1)-\f(\cdot,\v_2)}_{L^2(0,T;\L^2(\Gamma))}\\&\leq C(R)T\norm{\v_1-\v_2}_{\XT}, \end{align*} which, together with the previous estimate, leads to \eqref{CR}. We now focus on the operator $\mathcal L$. The fact that, for any ${T}>0$, this operator is invertible from $\YT$ to $\XT$ can be deduced from \cite[Lemma 7.8]{AGP2024b}, by setting, in the notations of that lemma, $\nu\equiv 2$, $\widetilde{\vphi}_0=\nu(\cdot)\in W^{1,\infty}(\Gamma)$, $\rho\equiv 1$ (so that the assumption $\vert\widetilde{\vphi}_0\vert\leq 1$ is not needed here, since $\rho_0\equiv1$). Therefore, by the Bounded-Inverse Theorem, for any $T>0$ there exists $C(T)>0$, possibly depending on $T$, such that $$ \norm{\mathcal{L}^{-1}}_{\mathcal{L}(\YT,\XT)}\leq C(T),\quad \forall T>0. $$ It is then enough to show that the constant above does not actually change with $T$. This can be obtained by a simple extension argument (see for instance the proof of \cite[Lemma 7]{AWe}), leading to the fact that there exits $C(T_0)$ such that \begin{align} \norm{\mathcal{L}^{-1}}_{\mathcal{L}(\YT,\XT)} \leq C(T_0),\quad \forall 0<T< T_0. \label{map}\end{align} We can now complete the existence proof. We aim at solving via a fixed-point argument the equation $$ \u=\mathcal{L}^{-1}\mathcal{F}\u \quad\text{in }\XT, $$ {for some } $T\in(0,T_0)$. First, we consider a generic $\overline{\v}\in \XT$. Then we fix $R>0$ such that it holds $\overline{\v}\in\overline{B}_{ R}^{X_{T_0}}(0)$, where $\overline{B}_R^{\XT}(0)$ is the closed ball of $\XT$ of radius $R$ centered at $0$. Clearly $\overline{B}_R^{X_{T_0}}(0)\subset \overline{B}_R^{\XT}(0)$, since $T\leq T_0$ and the norm of $\XT$ is nondecreasing. We also set $R$ such that $$\frac R2>\norm{\mathcal{L}^{-1}\mathcal{F}\overline{\v}}_{X_{T_0}}.$$ Note that in this way $R$ does not depend on $T$, since $T\leq T_0$, and it is also finite, since we have shown that $\mathcal F$ is well defined from $X_{T_0}$ to $Y_{T_0}$. Then one fixes $0<T< T_0$ (possibly depending also on $R$) such that the operator $\mathcal{L}^{-1}\mathcal{F}$ is a $(1/4)$-contraction mapping from $\XT$ to $\XT$. This is possible thanks to \eqref{CR} and \eqref{map}, since it holds \begin{align*} \Vert\mathcal{L}^{-1}\mathcal{F}(\v_1) -\mathcal{L}^{-1}\mathcal{F}(\v_2)\Vert_{\XT} \leq \norm{\mathcal{L}^{-1}}_{\mathcal{L}(\YT,\XT)}C(T, R)\Vert(\v_1 - \v_2)\Vert_{\XT}\leq C(T_0) C(T, R)\Vert(\v_1 - \v_2)\Vert_{\XT}, \end{align*} and thus we choose $T$ sufficiently small so that $C(T_0) C(T, R)\leq \frac14$. Then, thanks to the estimates, one shows that $\mathcal{L}^{-1}\mathcal{F}$ is well defined from $\overline{B}_R^{\XT}(0)$ to itself. Indeed, for any $\v\in \overline{B}_R^{\XT}(0)$ we have \begin{align*} \norm{\mathcal{L}^{-1}\mathcal{F}\v}_{\XT}&\leq \norm{\mathcal{L}^{-1}\mathcal{F}\v-\mathcal{L}^{-1}\mathcal{F}\overline{\v}}_{\XT}+ \norm{\mathcal{L}^{-1}\mathcal{F}\overline{\v}}_{\XT}\\& \leq \frac14\norm{\v-\overline{\v}}_{\XT}+\norm{\mathcal{L}^{-1}\mathcal{F}\overline{\v}}_{X_{T_0}} < R, \end{align*} since $\v,\overline{\v}\in \overline{B}_R^{\XT}(0)$. Thus, by Banach fixed point theorem applied on $\mathcal {L}^{-1}\mathcal F: \overline{B}_R^{\XT}(0)\to \overline{B}_R^{\XT}(0)$, there exists a unique solution $\u\in \overline{B}_R^{\XT}(0)\subset \XT$ to the problem under study. By a standard argument it is also easy to show that the solution $\u\in\overline{B}_R^{\XT}(0)\subset\XT$ we just found is unique in $\XT$. Indeed, let us assume that there exists another solution $\v\in \XT$. Then consider $\tilde{R}>0$ larger than $R$ used in the previous argument, so that $\v\in\overline{B}_{\tilde{R}}^{\XT}(0)$. Then, by repeating the same argument we deduce that there exists $T_1(\tilde{R})\in(0,T_0)$ such that the solution in $\overline{B}_{\tilde{R}}^{X_{T_1} }(0)$ is unique on $[0,T_1(\tilde{R})]$ and coincides with $\v_{\vert [0,T_1]}$. Thus, since also $\u_{\vert [0,\tT_1]}\in \overline{B}_{\tilde{R}}^{X_{T_1} }(0)$, it is immediate to infer $\v_{\vert [0,T_1]}=\u_{\vert [0,T_1]}$. If $T_1>T$ we are done, otherwise we can restart the same argument on the interval $[T_1,T_2]$, for some $T_2\in(T_1,T_0)$, and repeat the iterative continuation argument until we see that the identity holds on the entire interval $[0, T]$. The procedure terminates in a finite number of steps, since the time step in the contraction argument only depends on the radius of the ball containing $\v$, i.e., $\tilde{R}$. In conclusion, the pressure can be retrieved by standard arguments exploiting De Rahm's Theorem. \subsubsection{Existence of a global strong solution} In order to prove the existence of a global strong solution, we can use a standard continuation argument. Given $\u_0\in \H^1_\sigma(\Gamma)$, we know from the previous section that there exists a local strong solution $\u$. Let us assume that the maximal time $T_m$ of existence is finite. Then it holds \begin{align*} \u\in H^1_{loc}(0,T_m;\L^2_\sigma(\Gamma))\cap L^2_{loc}(0,T_m;\H^2(\Gamma)), \end{align*} entailing $\u\in BUC([0,T_m);\H^1(\Gamma))$. \noindent By means of suitable energy estimates we now show that $\u\in C([0,T_m];\H^1(\Gamma))$, which is a contradiction, since then we can define $\u(T_m)$ and extend the maximal existence interval of the solution outside $[0,T_m]$. First, we multiply \eqref{t1} by $\u$ and integrate over $\Gamma$. After some integration by parts we obtain the energy inequality \begin{align*} \frac12\frac{d}{dt}\norm{\u}^2+2\nu_*\int_\Gamma \norma{\E_\Gamma(\u)}^2\leq \frac12\frac{d}{dt}\norm{\u}^2+2\int_\Gamma \nu\norma{\E_\Gamma(\u)}^2=\int_\Gamma \f(\cdot,\u)\cdot \u. \end{align*} Recalling assumptions \eqref{C1}--\eqref{C2}, we obtain, by Young's inequality, \begin{align*} \int_\Gamma \f(\cdot,\u)\cdot \u\leq \norm{\f(\cdot,\u)-\f(\cdot,\boldsymbol 0)}\norm{\u}+\norm{\f(\cdot,\boldsymbol 0)}\norm{\u}\leq C_1\norm{\u}^2+C_2\norm{\u}\leq C(\norm{\u}^2+1), \end{align*} so that we infer \begin{align} \frac12\frac{d}{dt}\norm{\u}^2+2\nu_*\int_\Gamma \norma{\E_\Gamma(\u)}^2\leq C(\norm{\u}^2+1),\quad \forall t<T_m. \label{energia} \end{align} From Korn's inequality \eqref{Korn1} we infer by Gronwall's Lemma that \begin{align} \norm{\u}_{L^\infty(0,T_m;\L^2_\sigma(\Gamma))}+\norm{\u}_{L^2(0,T_m;\H^1(\Gamma))}\leq C(T_m). \label{reg1} \end{align} We can now pass to show higher-order estimates, namely, we multiply \eqref{t1} by $\partial_t\u$ and obtain, after integration by parts, \begin{align*} \frac{d}{dt}\int_\Gamma \nu \norma{\E_\Gamma(\u)}^2+\norm{\partial_t\u}^2+\int_\Gamma (\u \cdot\nabla_\Gamma)\u\cdot \partial_t\u=\int_\Gamma \f(\cdot,\u)\cdot \partial_t\u. \end{align*} Then, we have, by Gagliardo--Nirenberg's and Korn's inequalities, recalling \eqref{reg1} and $\nu\geq \nu_*$, \begin{align*} & \norma{\int_\Gamma (\u \cdot\nabla_\Gamma)\u\cdot \partial_t\u}\\&\leq \norm{\u}_{\L^4(\Gamma)}\norm{\nabla_\Gamma\u}_{\L^4(\Gamma)}\norm{\partial_t\u}\\&\leq C\norm{\u}^\frac12\norm{\u}_{\H^1(\Gamma)}\norm{\u}_{\H^2(\Gamma)}^\frac12\norm{\partial_t\u} \\&\leq C\norm{\u}^\frac12(\norm{\u}+\norm{\E_\Gamma(\u)})\norm{\u}_{\H^2(\Gamma)}^\frac12\norm{\partial_t\u}\\& \leq C(T_m) (1+\norm{\E_\Gamma(\u)})\norm{\u}_{\H^2(\Gamma)}^\frac12\norm{\partial_t\u}\\& \leq C(T_m)\norm{\E_\Gamma(\u)}^2\left(\int_\Gamma \nu\norma{\E_\Gamma(\u)}^2\right)+\frac12\norm{\u}_{\H^2(\Gamma)}^2+\frac14\norm{\partial_t\u}^2+C(T_m). \end{align*} Moreover, recalling \eqref{C1}--\eqref{C2} and \eqref{reg1} it holds \begin{align*} &\norma{\int_\Gamma \f(\cdot,\u)\cdot \partial_t\u}\leq \norm{\f(\cdot,\u)}\norm{\partial_t\u}\\&\leq \norm{\f(\cdot,\u)-\f(\cdot,\boldsymbol 0)}\norm{\partial_t\u}+\norm{\f(\cdot,\boldsymbol 0)}\norm{\partial_t\u}\\& \leq C\norm{\u}\norm{\partial_t\u}+\norm{\f(\cdot,\boldsymbol 0)}\norm{\partial_t\u}\\& \leq C(T_m)+\frac14\norm{\partial_t\u}^2. \end{align*} Therefore, we can write \begin{align} \frac{d}{dt}\int_\Gamma \nu \norma{\E_\Gamma(\u)}^2+\frac12\norm{\partial_t\u}^2\leq C(T_m)\norm{\E_\Gamma(\u)}^2\left(\int_\Gamma \nu\norma{\E_\Gamma(\u)}^2\right)+C(T_m)+\frac12\norm{\u}_{\H^2(\Gamma)}^2.\label{time} \end{align} In order to close the estimate, we use \cite[Lemma 7.4]{AGP2024b}. Indeed, we can rewrite equation \eqref{t1} in weak formulation as \begin{align*} 2\int_\Gamma \nu \E_\Gamma(\u):\E_\Gamma(\boldsymbol \eta)+\omega\int_\Gamma \u \cdot \boldsymbol \eta=\int_\Gamma \widetilde{\f}\cdot \boldsymbol \eta,\quad \forall \boldsymbol \eta \in \L^2_\sigma(\Gamma)\cap \H^1(\Gamma), \end{align*} for some $\omega>0$ and for $$ \widetilde{\f}:=\f(\cdot,\u)-(\u\cdot \nabla_\Gamma)\u-\partial_t\u. $$ Thus, we can apply \cite[Lemma 7.4]{AGP2024b} with, in the notations of the lemma, $t=0$, $\Gamma(0)=\Gamma$, $\nu\equiv 2$, $\varphi_0=\nu\in W^{1,\infty}(\Gamma)$, $\f=\widetilde{\f}$, and obtain, similarly as above, recalling \eqref{C1}--\eqref{C2}, and \eqref{reg1}, \begin{align*} &\norm{\u}_{\H^2(\Gamma)}\leq C(\omega)\norm{\widetilde{\f}}\\&\leq C(\norm{\f(\cdot,\u)}+\norm{(\u\cdot\nabla_\Gamma)\u}+\norm{\partial_t\u})\\& \leq C(\norm{\f(\cdot,\u)-\f(\cdot,\boldsymbol 0)}+\norm{\f(\cdot,\boldsymbol 0)}+\norm{\u}_{\L^4(\Gamma)}\norm{\nabla_\Gamma\u}_{\L^4(\Gamma)}+\norm{\partial_t\u})\\& \leq C\norm{\u}+C\norm{\f(\cdot,\boldsymbol 0)}+C\norm{\u}_{\L^4(\Gamma)}\norm{\u}_{\H^1(\Gamma)}^\frac12\norm{\u}_{\H^2(\Gamma)}^\frac12 +C_0\norm{\partial_t\u} \\& \leq C+C_0\norm{\partial_t\u}+\frac12 \norm{\u}_{\H^2(\Gamma)}, \end{align*} for some $C,C_0>0$. This entails, taking the squares, $$ \norm{\u}_{\H^2(\Gamma)}^2\leq C+4C_0^2\norm{\partial_t\u}^2, $$ so that, summing this inequality multiplied by $\gamma:=\frac1{16C_0^2}$ to inequality \eqref{time}, we get \begin{align} \label{final} \frac{d}{dt}\int_\Gamma \nu \norma{\E_\Gamma(\u)}^2+\gamma \norm{\u}_{\H^2(\Gamma)}^2+\frac14\norm{\partial_t\u}^2\leq C(T_m)\norm{\E_\Gamma(\u)}^2\left(\int_\Gamma \nu\norma{\E_\Gamma(\u)}^2\right)+C(T_m),\quad \forall t<T_m. \end{align} Recalling that, due to \eqref{reg1}, $\u \in L^2(0,T_m;\H^1_\sigma(\Gamma))$, we can apply Gronwall's Lemma and infer \begin{align} \label{more_reg} \norm{\u}_{L^2(0,T_m;\H^2(\Gamma))}+\norm{\u}_{L^\infty(0,T_m;\H^1_\sigma(\Gamma))}+\norm{\u}_{H^1(0,T_m;\L^2_\sigma(\Gamma))}\leq C(T_m), \end{align} which clearly entails also \begin{align*} \norm{\u}_{C([0,T_m];\H^1(\Gamma)\cap \L^2_\sigma(\Gamma))}\leq C(T_m), \end{align*} allowing to conclude the continuation argument, as outlined at the beginning of the proof. This concludes the argument to show that there exists a global in time strong solution. \subsubsection{Existence of a global weak solution} The proof of the existence of a global weak solution is now straightforward. Let us fix $\u_0\in \L^2_\sigma(\Gamma)$. Due to the density of $\H^1_\sigma(\Gamma)$ in $\L^2_\sigma(\Gamma)$, as $\varepsilon\to0$, we can approximate the initial datum by a sequence $\{\u_0^\varepsilon\}\subset \H^1_\sigma(\Gamma)$ such that $\u_0^\varepsilon\to \u_0$ in $\L^2_\sigma(\Gamma)$. Then, for any $\varepsilon>0$ we have just shown that there exists a global strong solution $\u^\varepsilon$, which also satisfies the following energy estimate (see \eqref{energia}) \begin{align} \frac12\frac{d}{dt}\norm{\u^\varepsilon}^2+2\nu_*\int_\Gamma \norma{\E_\Gamma(\u^\varepsilon)}^2\leq C(\norm{\u^\varepsilon}^2+1),\quad \forall t\geq 0. \label{energia2} \end{align} Applying Gronwall's Lemma and recalling that $\norm{\u_0^\varepsilon}\leq C$ uniformly in $\varepsilon$, we infer the following uniform estimates for any $T>0$ \begin{align} \norm{\u^\varepsilon}_{L^\infty(0,T;\L^2_\sigma(\Gamma))}+\norm{\u^\varepsilon}_{L^2(0,T;\H^1_\sigma(\Gamma))}\leq C(T),\quad \forall \varepsilon>0. \label{unif1} \end{align} We now obtain an estimate for $\partial_t\u^\varepsilon$, namely we consider $\v\in \H^1_\sigma(\Gamma)$ and observe that \begin{align*} \int_\Gamma \partial_t\u^\varepsilon\cdot \v=-\int_\Gamma (\u^\varepsilon\cdot \nabla_\Gamma)\u^\varepsilon\cdot \v+\int_\Gamma \f(\cdot,\u^\varepsilon)\cdot\v. \end{align*} By Gagliardo--Nirenberg's inequalities and \eqref{unif1} we get \begin{align*} &\norma{\int_\Gamma (\u^\varepsilon\cdot \nabla_\Gamma)\u^\varepsilon\cdot \v}=\norma{\int_\Gamma (\u^\varepsilon\cdot \nabla_\Gamma)\v\cdot \u^\varepsilon}\\&\leq \norm{\u^\varepsilon}_{\L^4(\Gamma)}^2\norm{\v}_{\H^1(\Gamma)}\leq C\norm{\u^\varepsilon}\norm{\u^\varepsilon}_{\H^1(\Gamma)}\norm{\v}_{\H^1(\Gamma)}\leq C\norm{\u^\varepsilon}_{\H^1(\Gamma)}\norm{\v}_{\H^1(\Gamma)}. \end{align*} Moreover, recalling assumptions \eqref{C1}--\eqref{C2} we can handle the $\f$-term as follows \begin{align*} &\norma{\int_\Gamma \f(\cdot,\u^\varepsilon)\cdot\v}\leq \norm{\f(\cdot,\u^\varepsilon)}\norm{\v}\\& \leq \norm{\f(\cdot,\u^\varepsilon)-\f(\cdot,\boldsymbol 0)}\norm{\v}+\norm{\f(\cdot,\boldsymbol 0)}\norm{\v}\\& \leq C\norm{\u^\varepsilon}\norm{\v}+\norm{\f(\cdot,\boldsymbol 0)}\norm{\v} \leq C(1+\norm{\u_\varepsilon})\norm{\v}. \end{align*} Therefore, since by \eqref{unif1} we know that $\norm{\u_\varepsilon}_{L^2(0,T;\H^1_\sigma(\Gamma))}\leq C(T)$ uniformly in $\varepsilon$, we infer that, for any $T>0$, \begin{align*} \norm{\partial_t\u}_{L^2(0,T; \H^1_\sigma(\Gamma)')}\leq C(T),\quad \forall \varepsilon>0. \end{align*} This result, together with \eqref{unif1} allows to deduce by standard compactness arguments that there exists $\u:\Gamma\times[0,\infty)\to \T\Gamma$ such that, up to not relabeled subsequences, for any $T>0$, \begin{align*} &\u^\varepsilon\overset{*}{\rightharpoonup} \u,\quad \text{in }L^\infty(0,T;\L^2_\sigma(\Gamma)),\\& \u^\varepsilon \rightharpoonup \u,\quad\text{ in }L^2(0,T;\H^1_\sigma(\Gamma)),\\& \partial_t\u^\varepsilon\rightharpoonup \partial_t\u,\quad \text{ in }L^2(0,T;{\H^1_\sigma}(\Gamma)'), \end{align*} as $\varepsilon\to 0$. This also entails, by Aubin-Lions Lemma, $$ \u^\varepsilon\to \u,\quad \text{ in }L^2(0,T;\H^s(\Gamma)),\quad \forall s\in[0,1),\quad\text{and almost everywhere in }\Gamma\times[0,T]. $$ These convergences are enough to pass to the limit in the equations satisfied by $\u^\varepsilon$ and conclude that $\u$ is a global weak solution to \eqref{t1}--\eqref{tangential}. \subsubsection{Weak uniqueness and continuous-dependence estimate} Thanks to the first part of Theorem \ref{thm1}, given $\u_{0,1},\u_{0,2}\in \L^2_\sigma(\Gamma)$ there exist two weak solutions $\u_1,\u_2$ on $[0,\infty)$ to \eqref{t1}--\eqref{tangential} departing from those initial data. Their regularity is enough to perform rigorously the next computations, leading to \eqref{contdep2}. In particular, let us notice that $\u:=\u_1-\u_2$ satisfies \begin{align} &\label{t2}\int_\Gamma\langle\partial_t\u,\v\rangle_{\H^1_\sigma(\Gamma)',\H^1_\sigma(\Gamma)}+\int_\Gamma(\u_1\cdot \nabla_\Gamma)\u\cdot \v+\int_\Gamma(\u\cdot \nabla_\Gamma)\u_2\cdot \v+2\int_\Gamma \nu\boldsymbol\varepsilon_\Gamma(\u):\E_\Gamma(\v)\\&=\int_\Gamma(\f(\cdot,\u_1)-\f(\cdot,\u_2))\cdot \v,\quad \forall \v\in \H^1_\sigma(\Gamma)\\nonumber\\& \u(0)=\u_{0,1}-\u_{0,2}, \label{tangential2222} \end{align} Let us now set $\v=\u$ in \eqref{t2}. Recalling that $\nu\geq \nu_*>0$, we obtain \begin{align*} \frac12\frac{d}{dt}\norm{\u}^2+2\nu_*\norm{{\boldsymbol \varepsilon}_\Gamma(\u)}^2\leq-\int_\Gamma (\u\cdot\nabla_\Gamma) \u_2\cdot \u +\int_\Gamma (\f(\cdot,\u_1)-\f(\cdot,\u_2))\cdot \u. \end{align*} Note that, by H\"{o}lder's, Gagliardo--Nirenberg's inequalities, and Korn's inequalities we have that \begin{align*} \norma{\int_\Gamma (\u\cdot\nabla_\Gamma) \u_2\cdot \u}&\leq \norm{\u}_{\L^4(\Gamma)}\norm{\nabla_\Gamma \u_2}\norm{\u}_{\L^4(\Gamma)}\\&\leq C\norm{\u}\norm{\u}_{\H^1(\Gamma)}\norm{\u_2}_{\H^1(\Gamma)}\\&\leq C\norm{\u}(\norm{\u}+\norm{{\boldsymbol \varepsilon}_\Gamma(\u)})\norm{\u_2}_{\H^1(\Gamma)}\\& \leq C(1+\norm{\u_2}_{\H^1(\Gamma)}^2)\norm{\u}^2+\nu_*\norm{{\boldsymbol \varepsilon}_\Gamma(\u)}^2. \end{align*} Moreover, exploiting assumption \eqref{C2} on $\f$, we deduce, again by H\"{o}lder's, Gagliardo--Nirenberg's, and Korn's inequalities \begin{align*} \norma{\int_\Gamma (\f(\cdot,\u_1)-\f(\cdot,\u_2))\cdot \u}&\leq \norm{\f(\cdot,\u_1)-\f(\cdot,\u_2)}\norm{\u} \\& \leq C\norm{\u}^2. \end{align*} Putting all these estimates together we end up with \begin{align*} \frac12\frac d{dt}\norm{\u}^2+\nu_*\norm{{\boldsymbol \varepsilon}_\Gamma(\u)}^2\leq C(1+\norm{\u_2}_{\H^1(\Gamma)}^2)\norm{\u}^2, \end{align*} recalling that the regularity of weak solutions implies that $\u_i\in L^2(0,T;\H^1_\sigma(\Gamma))$, $i=1,2$. Using the embedding $\H^1(\Gamma)\hookrightarrow \L^4(\Gamma)$, we can apply Gronwall's lemma and deduce \eqref{contdep2}. \subsubsection{Instantaneous regularization} An immediate consequence of the above results is the instantaneous regularization of a weak solution. Let us assume $\u_0\in \L^2_\sigma(\Gamma)$. Then there exists a unique global weak solution $\u$ departing from that datum. Let us fix an arbitrary $\tau>0$. Since $\u\in L^2(0,T;\H^1_\sigma(\Gamma))$ for any $T>0$, there exists $\tau_1\in(0,\tau]$ such that $\u(\tau_1)\in \H^1_\sigma(\Gamma)$. Therefore, by the first part of Theorem \ref{thm1}, there exists a unique global strong solution $\u_1$ departing from $\u(\tau_1)$. Since the solutions are unique, it holds $\u\equiv \u_1$ on $[\tau_1,+\infty)$, and thus the solution $\u$ becomes instantaneously strong. Let us fix $\tau>0$. In order to prove \eqref{regulari} we can repeat, for any $t\geq \tau$, the estimates leading to \eqref{final}, obtaining, for any $T>0$, \begin{align} \label{finalb} \frac{d}{dt}\int_\Gamma \nu \norma{\E_\Gamma(\u)}^2+\gamma \norm{\u}_{\H^2(\Gamma)}^2+\frac14\norm{\partial_t\u}^2\leq C(T)\norm{\E_\Gamma(\u)}^2\left(\int_\Gamma \nu\norma{\E_\Gamma(\u)}^2\right)+C(T),\quad \forall t\geq \tau, \end{align} where the constants $C$ depend on $T$ due to the energy estimates leading to \eqref{regga}. Multiplying by $t$ this inequality we are led to \begin{align*} &\frac{d}{dt}t\int_\Gamma \nu \norma{\E_\Gamma(\u)}^2+\gamma t\norm{\u}_{\H^2(\Gamma)}^2+\frac t4\norm{\partial_t\u}^2\\&\leq C(T)t\norm{\E_\Gamma(\u)}^2\left(\int_\Gamma \nu\norma{\E_\Gamma(\u)}^2\right)+C(T)t\frac12+\int_\Gamma \nu \norma{\E_\Gamma(\u)}^2\\& \leq C(T)T\norm{\E_\Gamma(\u)}^2\left(\int_\Gamma \nu\norma{\E_\Gamma(\u)}^2\right)+C(T)T+\frac12\int_\Gamma \nu \norma{\E_\Gamma(\u)}^2,\quad \forall t\geq \tau. \end{align*} Since then $\norm{\u}_{L^2(0,T;\H^1_\sigma(\Gamma))}\leq C(T)$ by \eqref{regga}, we can apply Gronwall's Lemma and deduce \eqref{regulari}, recalling also Korn's inequality. The proof is concluded. \subsection{Proof of Theorem \ref{backuni}} In order to prove the theorem, we follow a somehow similar argument as in \cite{Foias}. There, one needs to show that the Dirichlet quotient $\lambda := \frac{\norm{\u}_{\H^1(\Gamma)}^2}{\norm{\u}^2}$ grows at most exponentially in time, exploiting the dissipativity properties of the system. In this case, due to the lack of dissipativity, a suitable quantity to consider is \begin{align} \label{qty} \Lambda:=\frac{\norm{\sqrt{2\nu}\E_\Gamma(\u)}^2}{\norm{\u}^2}=\frac{\norm{\sqrt{2\nu}\E_\Gamma(\u)}^2}{\norm{\u_K}^2+\norm{\u_{NK}}^2},\quad \forall \u\in \H^1_\sigma(\Gamma),\quad \u\not\equiv0, \end{align} where we set $\u_K:=\Pk\u$ and $\u_{NK}:=(\I-\Pk)\u$. Note that this is not a Dirichlet quotient, since $\Lambda=0$ for any $\u\in \KK$, and in general $\KK$ is nontrivial. Let us consider the two solutions $\u_i$, $i=1,2$ given in the statement, and introduce $\u=\u_1-\u_2$, and $p=p_1-p_2$, so that they solve \begin{align} \partial_t\u +(\u\cdot\nabla_\Gamma)\u_1+(\u_2\cdot\nabla_\Gamma)\u-2\P_\Gamma\divg(\nu\E_\Gamma(\u))+\nabla_\Gamma p=\f(\cdot,\u_1)-\f(\cdot,\u_2),\label{a0} \end{align} where we recall that we assumed $\nu\in W^{1,\infty}(\Gamma)$ such that $\nu\geq \nu_*>0$. Applying the Helmholtz projector $\P_0$, we can also write \begin{align} \partial_t\u +\P_0(\u\cdot\nabla_\Gamma)\u_1+\P_0(\u_2\cdot\nabla_\Gamma)\u+\A_S\u=\P_0\f(\cdot,\u_1)-\P_0\f(\cdot,\u_2),\label{a1} \end{align} where we set $\A_S\u:=-2\P_0\P_\Gamma\divg(\nu\E_\Gamma(\u))$ as the (modified) Stokes operator. We observe that, after an integration by parts, it holds \begin{align} \int_\Gamma{\A_S(\v)}\cdot \v=\norm{\sqrt{2\nu}\E_\Gamma(\v)}^2,\quad \forall \v\in \H^1_\sigma(\Gamma). \label{intbyparts} \end{align} Now, assume by contradiction that there exists $t_*\in [0,T_*)$ such that $\u_1(t_*)\not=\u_2(t_*)$, i.e., there exists a set of positive Lebesgue measure on which the two solutions are different. Then $\norm{\u(t_*)}>0$, and, recalling that, as $\u_1,\u_2\in C([0,\infty);\H^1_\sigma(\Gamma))$, this entails that there exists an interval $I_\delta:=[t_*,t_*+\delta)$, $\delta>0$, such that $\norm{\u(t)}>0$ for any $t\in I_\delta$. Assuming $I_\delta$ to be the largest of such intervals with this property, it must be $\norm{\u(t_*+\delta)}=0$. We will now consider $t\in I_\delta$ and show that we obtain a contradiction. Let us also introduce the quantity \begin{align*} L(t):=-\frac12 \ln(\norm{\u(t)}^2),\quad t\in I_\delta, \end{align*} which is clearly well defined on $I_\delta$, and observe that we have the following identities, which are obtained by multiplying (and integrating) \eqref{a0} by $\u$ and \eqref{a1} by $\A_S\u$, respectively: \begin{align} \label{e1} &\frac12\frac{d}{dt}\norm{\u}^2=-\int_\Gamma (\u\cdot\nabla_\Gamma )\u_1\cdot \u -\norm{\sqrt{2\nu}\E_\Gamma(\u)}^2+\int_\Gamma (\f(\cdot,\u_1)-\f(\cdot,\u_2))\cdot \u,\\&\nonumber \frac12\frac d{dt}\norm{\sqrt{2\nu}\E_\Gamma(\u)}^2\\& \label{e2}=-\int_\Gamma (\u\cdot\nabla_\Gamma )\u_1\cdot \A_S\u-\int_\Gamma (\u_2\cdot\nabla_\Gamma )\u\cdot \A_S\u-\norm{\A_S\u}^2+\int_\Gamma (\f(\cdot,\u_1)-\f(\cdot,\u_2))\cdot \A_S\u. \end{align} Now, let us consider the time derivative of $L(t)$: we have \begin{align*} \frac d{dt}L(t)=-\frac{\frac12\frac d{dt}\norm{\u}^2}{\norm{\u}^2}=\frac{\int_\Gamma (\u\cdot\nabla_\Gamma )\u_1\cdot \u}{\norm{\u}^2}+\Lambda(t)-\frac{\int_\Gamma (\f(\cdot,\u_1)-\f(\cdot,\u_2))\cdot \u}{\norm{\u}^2}, \end{align*} and are left with estimating the first and the last terms in the right-hand side in terms of $\Lambda$. First observe that, by Korn's inequality \eqref{Korn1}, we have \begin{align} \norm{\u}_{\H^1(\Gamma)}\leq C(\norm{\u}+\norm{\E_\Gamma(\u)})=C(\norm{\u_K}+\norm{\u_{NK}}+\norm{\E_\Gamma(\u)})\leq C(\norm{\u_K}+\norm{\E_\Gamma(\u)}). \label{essential} \end{align} Then, by H\"{o}lder's and Gagliardo--Nirenberg's inequalities, we have, recalling \eqref{essential} and $\nu\geq \nu_*>0$, \begin{align*} &\frac{\int_\Gamma (\u\cdot\nabla_\Gamma )\u_1\cdot \u}{\norm{\u}^2}\leq \frac{1}{\norm{\u}^2}\norm{\u}_{\L^4(\Gamma)}^2\norm{\u_1}_{\H^1(\Gamma)}\\&\leq C\frac{\norm{\u}_{\H^1(\Gamma)}\sqrt{\norm{\u_K}^2+\norm{\u_{NK}}^2}}{\norm{\u_K}^2+\norm{\u_{NK}}^2}\norm{\u_1}_{\H^1(\Gamma)}\leq C\frac{\norm{\u_K}+\norm{\E_\Gamma(\u)}}{\sqrt{\norm{\u_K}^2+\norm{\u_{NK}}^2}}\norm{\u_1}_{\H^1(\Gamma)}\\& \leq C\norm{\u_1}_{\H^1(\Gamma)}^2+\frac{\norm{\u_K}^2}{{\norm{\u_K}^2+\norm{\u_{NK}}^2}}+\frac{1}{2\nu_*}\frac{\norm{\sqrt{2\nu}\E_\Gamma(\u)}^2}{{\norm{\u_K}^2+\norm{\u_{NK}}^2}}\\& \leq C\norm{\u_1}_{\H^1(\Gamma)}^2+1+\frac1{2\nu_*}\Lambda(t)\leq C(T)+\frac1{2\nu_*}\Lambda(t), \end{align*} where $C(T)$ depends on some time horizon $T>T^*$, since $\u_1\in L^\infty(0,T;\H^1_\sigma(\Gamma))$ for any $T>0$. Concerning the terms related to the forcing $\f$, recalling assumption \eqref{C2} and by Gagliardo--Nirenberg's and Young's inequalities, we have \begin{align*} &\norma{\frac{\int_\Gamma (\f(\cdot,\u_1)-\f(\cdot,\u_2))\cdot \u}{\norm{\u}^2}}\leq \frac{C}{\norm{\u}^2}\norm{\u}^2=C. \end{align*} To sum up, we have obtained that \begin{align} \frac{d}{dt}L(t)\leq C(T)+C\Lambda(t),\quad \forall t\in I_\delta.\label{L1} \end{align} Let us now establish some control on $\Lambda(t)$. To this aim, similar to \cite{Foias}, recalling \eqref{intbyparts}, we observe that \begin{align*} &\norm{\A_S\u-\Lambda(t)\u}^2=\norm{\A_S\u}^2+\frac{\norm{\sqrt{\nu}\E_\Gamma(\u)}^4}{\norm{\u}^2}-2\frac{\norm{\sqrt{2\nu}\E_\Gamma(\u)}^2}{\norm{\u}^2}\int_\Gamma \A_S\u\cdot\u\\& =\norm{\A_S\u}^2-\frac{\norm{\sqrt{2\nu}\E_\Gamma(\u)}^4}{\norm{\u}^2}=\norm{\A_S\u}^2-\Lambda(t)\norm{\sqrt{2\nu}\E_\Gamma(\u)}^2. \end{align*} Then, we can compute the time derivative of $\Lambda$. By H\"{o}lder's inequality and exploiting \eqref{e1}--\eqref{e2} we get \begin{align*} & \frac d{dt}\Lambda(t)=\frac{\frac{d}{dt}\norm{\sqrt{2\nu}\E_\Gamma(\u)}^2}{\norm{\u}^2}-\Lambda(t)\frac{\frac d {dt}\norm{\u}^2}{\norm{\u}^2}\\& =\frac{2}{\norm{\u}^2}\left(-\int_\Gamma (\u\cdot\nabla_\Gamma )\u_1\cdot \A_S\u-\int_\Gamma (\u_2\cdot\nabla_\Gamma )\u\cdot \A_S\u-\norm{\A_S\u}^2+\int_\Gamma (\f(\cdot,\u_1)-\f(\cdot,\u_2))\cdot \A_S\u\right)\\& -\frac{2\Lambda(t)}{\norm{\u}^2}\left(-\int_\Gamma (\u\cdot\nabla_\Gamma )\u_1\cdot \u-\int_\Gamma (\u_2\cdot\nabla_\Gamma )\u\cdot \u -\norm{\sqrt{2\nu}\E_\Gamma(\u)}^2+\int_\Gamma (\f(\cdot,\u_1)-\f(\cdot,\u_2))\cdot \u\right)\\& =\frac{2}{\norm{\u}^2}\left((\Lambda(t)\norm{\sqrt{2\nu}\E_\Gamma(\u)}^2-\norm{\A_S\u}^2)+\int_\Gamma (\u\cdot\nabla_\Gamma )\u_1\cdot (\Lambda(t)\u-\A_S\u)\right.\\& \left.+\int_\Gamma (\u_2\cdot\nabla_\Gamma )\u\cdot (\Lambda(t)\u-\A_S\u) +\int_\Gamma (\f(\cdot,\u_1)-\f(\cdot,\u_2))\cdot (\A_S\u-\Lambda(t)\u) \right)\\& \leq \frac{2}{\norm{\u}^2}\left(-\norm{\Lambda(t)\u-\A_S\u}^2+\norm{\u}_{\L^4(\Gamma)}\norm{\u_1}_{\W^{1,4}(\Gamma)}\norm{\Lambda(t)\u-\A_S\u}\right.\\&\left.+\norm{\u_2}_{\L^\infty(\Gamma)}\norm{\u}_{\H^1(\Gamma)}\norm{\Lambda(t)\u-\A_S\u}+\norm{\f(\cdot,\u_1)-\f(\cdot,\u_2)}\norm{\Lambda(t)\u-\A_S\u} \right)\\& =\frac{2}{\norm{\u}^2}\left(-\norm{\Lambda(t)\u-\A_S\u}^2+\mathcal I_0+\mathcal I_1+\mathcal I_2\right). \end{align*} To estimate $\mathcal I_0$, by Young's and Gagliardo--Nirenberg's inequalities, together with \eqref{essential}, we have \begin{align*} \mathcal I_0&\leq C\norm{\u}\norm{\u}_{\H^1(\Gamma)}\norm{\u_1}_{\H^2(\Gamma)}\norm{\u_1}_{\H^1(\Gamma)}+\frac14\norm{\Lambda(t)\u-\A_S\u}^2\\& \leq C(T)\norm{\u}^2\norm{\u_1}_{\H^2(\Gamma)}^2+C(\norm{\u_K}^2+\norm{\E_\Gamma(\u)}^2)+\frac14\norm{\Lambda(t)\u-\A_S\u}^2\\&\leq C(T)\norm{\u}^2(1+\norm{\u_1}_{\H^2(\Gamma)}^2)+C\norm{\sqrt{2\nu}\E_\Gamma(\u)}^2)+\frac14\norm{\Lambda(t)\u-\A_S\u}^2, \end{align*} where we used $\norm{\u_K}\leq \norm{\u}$ as well as $\norm{\u_1}_{L^\infty(0,T;\H^1_\sigma(\Gamma))}\leq C(T)$, choosing $T>T^*$. We estimate $\mathcal I_1$ analogously, by using Agmon's, Young's, Korn's inequalities, and \eqref{essential}, as \begin{align*} \mathcal I_1&\leq C\norm{\u_2}_{\H^1(\Gamma)}^\frac12\norm{\u_1}_{\H^2(\Gamma)}^\frac12\sqrt{\norm{\u_K}^2+\norm{\E_\Gamma(\u)}^2}\norm{\Lambda(t)\u-\A_S\u}\\& \leq C(T)\norm{\u_2}_{\H^2(\Gamma)}(\norm{\sqrt{2\nu}\E_\Gamma(\u)}^2+\norm{\u}^2)+\frac14\norm{\Lambda(t)\u-\A_S\u}^2, \end{align*} where again $\norm{\u_2}_{L^\infty(0,T;\H^1_\sigma(\Gamma))}\leq C(T)$, choosing $T>T^*$. In conclusion, recalling \eqref{C2}, we have, by Gagliardo--Nirenberg's and Young's inequalities, together with \eqref{essential}, \begin{align*} \mathcal I_2&\leq C\norm{\u}\norm{\Lambda(t)\u-\A_S\u}\\& \leq C\norm{\u}^2+\frac14\norm{\Lambda(t)\u-\A_S\u}^2. \end{align*} To sum up, we have obtained \begin{align*} \frac d{dt}\Lambda(t)+ \frac12 \frac{\norm{\Lambda(t)\u-\A_S\u}^2}{\| u \|^2}\leq C(T)(1+\norm{\u_2}_{\H^2(\Gamma)})\Lambda(t)+C(T)(1+\norm{\u_1}_{\H^2(\Gamma)}^2),\quad \forall t\in I_\delta. \end{align*} Recalling that, for $T>T^*$, we have $\u_i\in L^2(0,T;\H^2(\Gamma))$, $i=1,2$, we can apply Gronwall's Lemma on the interval $I_\delta$ and obtain that \begin{align*} \sup_{t\in[t_*,t_*+\delta)}\Lambda(t)\leq \Lambda(t_0)e^{C(T)}+C(T), \end{align*} entailing that $\Lambda$ is bounded on $I_\delta$. From this, recalling \eqref{L1}, we deduce by integrating over $I_\delta$ that also $L(t)$ is bounded on $[t_*,t_*+\delta)$. In particular, we can deduce that also $L(t_*+\delta)$ is bounded, contradicting the fact that $\norm{\u(t_*+\delta)}=0$. This concludes the contradiction argument and $t_*\in[0,T^*)$ with the property that $ \norm{\u(t_*)}>0$ does not exist. %cannot exist. \section{ Preliminary results on long-time behavior: Proofs of Section \ref{longtt}}\label{sec:prel} \subsection{Proof of Proposition \ref{dec}} In the case $\mathcal{K}=\{\boldsymbol 0\}$ the proof is trivial. We thus consider the case of nontrivial Killing field space $\mathcal{K}$ of dimension $n\geq1$. We see by summing up \eqref{t11} and \eqref{K1} that it holds, by uniqueness, $\overline{\u}=\u_{K}+\u_{NK}$ (recall that $\E_\Gamma(\u_K)=\boldsymbol 0$ by definition of Killing vector field), and thus $\u_{NK}$ {satisfying \eqref{t11}} can be obtained as the difference between $\overline{\u}$ and $\u_K$, as soon as we prove that $\u_K$ exists. In order to prove the proposition we then need first to show that $\u_K$ exists and is uniquely defined. To see this, notice that, from its definition, $\u_K\in \mathcal K$ for all $t\geq0$. Thus, considering the basis $\{\v_i\}$ of $\mathcal{K}$, it can be written as $\u_K(t)=\sum_{i=1}^n\alpha_i(t)\v_i$, where $\alpha_i$ can be assumed to be in $C^1([0,\infty))$. Let us multiply \eqref{K1} by $\v_i$ and integrate over $\Gamma$. Since the basis $\{\v_j\}$ is orthonormal, this gives a system of ODEs in $\alpha_i$ of the form \begin{align} \partial_t\alpha_i=\int_\Gamma\f_K\left(x,\overline{\u}\right)\cdot \v_i, \quad i=1,\ldots,n. \label{ode} \end{align} The right-hand side of \eqref{ode} is continuous in time. Indeed, thanks to the regularity of $\overline{\u}$, $\overline{\u}(s)\to\overline{\u}(t)$ as $s\to t$, $t\geq0$, in $\Lts$, and thus, by the continuity of $\Pk$, \begin{align*} &\norma{\int_\Gamma(\f_K(\cdot,\overline{\u}(t))-\f_K(\cdot,\overline{\u}(s)))\cdot \v_i}\\&\leq \norm{\f(\cdot,\overline{\u}(t))-\f(\cdot,\overline{\u}(s))}\norm{\v_i}\\& \leq C\norm{\overline{\u}(t)-\overline{\u}(s)}\to 0\quad\text{as }s\to t. \end{align*} Therefore, by standard theory there exists $t_*>0$ and a unique vector $(\alpha_i)_i\in C^1([0,t_*];\mathbb R^n)$ such that \eqref{ode} is satisfied. This corresponds to a unique solution $\u_K$ to \eqref{K1}--\eqref{K2}. Moreover, by multiplying \eqref{K1} by $\u_K$, applying the Cauchy-Schwartz inequality, and recalling \eqref{C1}--\eqref{C2}, we infer that \begin{align*} \frac 12\partial_t\sum_{i=1}^n\norma{\alpha_i}^2&\leq \norm{\f(\cdot, \overline{\u})-\f(\cdot,\boldsymbol 0)}\sqrt{\sum_{i=1}^n\norma{\alpha_i}^2}+\norm{\f(\cdot,\boldsymbol 0)}\sqrt{\sum_{i=1}^n\norma{\alpha_i}^2}\\&\leq C\sum_{i=1}^n\norma{\alpha_i}^2+C(1+\norm{\overline\u}^2), \end{align*} entailing that $t_*=+\infty$, i.e., the solution $\u_K\in \mathcal K$ is indeed global. It is now trivial to deduce that $\u_{NK}=\overline{\u}-\u_{K}$ {satisfies \eqref{t11}}, is uniquely determined, and belongs to $(\I-\Pk)\Lts$, by simply multiplying \eqref{t11} by $\z\in \mathcal K$, integrating over $\Gamma$, and recalling (see again \cite{Simonett}) that $\int_\Gamma (\overline{\u}\cdot\nabla_\Gamma)\overline{\u}\cdot \z=0$. Since also $\divg\z=0$, we deduce that one has $$ \partial_t\int_{\Gamma}\u_{NK}\cdot \z=0,\quad \forall \z\in \mathcal K. $$ On the other hand, we also have that $\int_\Gamma \u_{NK}(0)\cdot \z=\int_\Gamma (\boldsymbol I-\P_{\mathcal{K}})\u_0\cdot \z=0$ for any $\z\in \mathcal K$. This entails that $\u_{NK}(t)\in (\I-\Pk)\Lts$ for any $t\geq0$. Since $\L^2_\sigma(\Gamma)=\mathcal K\oplus (\I-\Pk)\Lts$, we have just proved the assertion of the proposition, by the uniqueness of this decomposition, i.e., $\u_K=\Pk\overline{\u}$ and $\u_{NK}=(\I-\Pk)\overline{\u}$. \subsection{Proof of Proposition \ref{lemmadec}} In order to prove \eqref{exp}, it is enough to multiply equation \eqref{K1} by $\u_K$ to obtain, by the assumptions on $\f_K$ and Proposition \ref{dec}, $$ \frac12\frac{d}{dt}\norm{\u_K}^2=\int_\Gamma \f_K(x,\overline{\u})\cdot \u_K=\int_\Gamma \f_K(x,\overline{\u})\cdot \overline{\u}\leq C_3 \norm{\u_K}^2+ C_4 , $$ where we recall that $\overline{\u}=S(t)\u_0$ is the entire unique global solution corresponding to $\u_0$. This entails the result by Gronwall's Lemma. For case (i), we multiply \eqref{K1} by $\f_K$ and integrate over $\Gamma$ getting \begin{align*} \frac{d}{dt}\int_\Gamma \u_K\cdot \f_K=\norm{\f_K}^2, \end{align*} so that $$ \int_\Gamma \u_K(t)\cdot \f_K=\int_\Gamma \u_K(0)\cdot \f_K+t\norm{\f_K}^2 $$ which is \eqref{fd}. Moreover, if we consider the subspace $\mathcal{K}\cap \f_K^{\perp_{\L^2}}$, it holds \begin{align*} \frac{d}{dt}\int_\Gamma \u_K\cdot \v=0,\quad \forall \v\in \mathcal{K}\cap \f_K^{\perp_{\L^2}}, \end{align*} entailing \eqref{uk1}. Summing up \eqref{fd} squared with \eqref{uk1} one obtains \eqref{uk2}. In order to check case (ii), one multiplies \eqref{K1} with $\u_K$ and integrates on $\Gamma$ getting $$ \frac12 \frac{d}{dt}\norm{\u_K}^2=\int_\Gamma \f_K(x,\overline{\u})\cdot \u_K=\int_\Gamma \f_K(x,\overline{\u})\cdot \overline{\u}\leq 0. $$ Analogously, for the case (iii) we have $$ \frac12 \frac{d}{dt}\norm{\u_K}^2=\int_\Gamma \f_K(x,\overline{\u})\cdot \u_K=\int_\Gamma \f_K(x,\overline{\u})\cdot \overline{\u}\geq 0. $$ The assertion follows by integrating in time. \subsection{Proof of Proposition \ref{pp}} Multiply \eqref{t11} by $\u_{NK}$ and integrate over $\Gamma$. Observe that (see \cite{Simonett}) $$ \int_\Gamma (\overline{\u}\cdot\nabla_\Gamma)\overline{\u}\cdot \u_{NK}=\int_\Gamma (\overline{\u}\cdot\nabla_\Gamma)\overline{\u}\cdot \overline{\u}=0, $$ since $\int_\Gamma (\overline{\u}\cdot\nabla_\Gamma)\overline{\u}\cdot {\u}_K=0$ from the divergence-free property of $\overline{\u}$. Therefore, from \eqref{extra} and Proposition \ref{dec}, recalling that $\nu\geq \nu_*>0$ we get that \begin{align} &\nonumber \frac12\ddt\norm{\u_{NK}}^2+2\nu_*\norm{\E_\Gamma(\u_{NK})}^2\\& \leq { C_5 }\norm{\u_{NK}}^2+ C_6 \norm{\u_{NK}}\nonumber\\& \leq 2 C_5 \norm{\u_{NK}}^2+C.\label{energy} \end{align} From Korn's inequality \eqref{Korn3} we deduce from \eqref{energy} that \begin{align} & \frac12\ddt\norm{\u_{NK}}^2+\left(\frac{2\nu_*}{C_P^2}-2 C_5 \right)\norm{\u_{NK}}^2\leq C,\label{energy2} \end{align} which gives the desired result \eqref{gr} by Gronwall's Lemma, since by assumption $\zeta:=\frac{2\nu_*}{C_P^2}-2 C_5>0$. Under the additional assumption \eqref{nega} on $\f_K$, we have $$ \norm{\u_K (t) }\leq \norm{\u_K(0)},\quad \forall t\geq0, $$ so that, working now with \eqref{extra2} and performing similar energy estimates as above we obtain \begin{align} &\nonumber \frac12\ddt\norm{\u_{NK}}^2+2\nu_*\norm{\E_\Gamma(\u_{NK})}^2\\& \leq { C_5}\norm{\u_{NK}}^2+ C_6\norm{\u_{NK}}+ C_6\norm{\u_K}^2\nonumber\\&\nonumber \leq C_5\norm{\u_{NK}}^2+ C_6\norm{\u_{NK}}+ C_6\norm{\u_K(0)}^2 \\& \leq 2 C_5\norm{\u_{NK}}^2+C(1+\norm{\u_K(0)}^2).\label{energy3} \end{align} Again by Korn's inequality we have \begin{align} & \frac12\ddt\norm{\u_{NK}}^2+\left(\frac{2\nu_*}{C_P^2}-2 C_5\right)\norm{\u_{NK}}^2\leq C(1+\norm{\u_K(0)}^2),\label{energy2b} \end{align} allowing to deduce \eqref{gr2} by Gronwall's Lemma. \subsection{Proof of Theorem \ref{insta}} The instantaneous regularization of (global) weak solutions is already pointed out in Theorem \ref{thm1}. Let us consider a weak solution $\u$ departing from an initial datum $\u_0\in \L^2_\sigma(\Gamma)$. From any positive time onward, this solution is also strong. We now aim at finding uniform estimates for all positive times. First, from Proposition \ref{lemmadec} and \eqref{gr2} we have \begin{align*} \norm{\u (t) }^2&=\norm{\u_{NK} (t) }^2+\norm{\u_{K} (t) }^2\\&\leq e^{-\zeta t}\norm{\u_{NK}(0)}^2+\omega(1+\norm{\u_K(0)}^2)+\norm{\u_K(0)}^2 \\& \leq C\norm{\u_0}^2,\quad \forall t\geq 0, \end{align*} entailing that \begin{align} \norm{\u}_{L^\infty(0,\infty;\L^2_\sigma(\Gamma))}\leq C,\label{unif} \end{align} where $C$ depends on the $\L^2$-norm of $\u_0$. Then, repeating the same estimates leading to \eqref{energy}, we get \begin{align} 2\nu_*\norm{\E_\Gamma(\u_{NK})}^2 \leq 2 C_5 \norm{\u_{NK}}^2+C,\label{energybb} \end{align} so that, integrating over $[t,t+1]$, $t\geq0$, using Korn's inequality \eqref{Korn1} we obtain $$ \norm{\u_{NK}}_{L^2(t,t+1;\H^1_\sigma(\Gamma))}\leq C,\quad \forall t\geq 0. $$ Now recall that, again by Korn's inequality \eqref{Korn2}, it holds \begin{align*} \norm{\u_K}_{\H^1(\Gamma)}\leq C\norm{\u_K}, \end{align*} so that, together with \eqref{unif}, we immediately end up with \begin{align} \norm{\u}_{L^2(t,t+1;\H^1_\sigma(\Gamma))}\leq C,\quad \forall t\geq 0,\label{H1} \end{align} where $C$ only depends on $\norm{\u_0}$, $\f$, $\Gamma$, and the parameters of the problem. We can now deal with higher-order estimates. Let us fix an arbitrary $\tau>0$. The solution $\u$ is strong on $[\tau,\infty)$, so that, by repeating the same computations leading to \eqref{final} and exploiting \eqref{unif} we get \begin{align} \label{final2} \frac{d}{dt}\int_\Gamma \nu \norma{\E_\Gamma(\u)}^2+\gamma \norm{\u}_{\H^2(\Gamma)}^2+\frac14\norm{\partial_t\u}^2\leq C\norm{\E_\Gamma(\u)}^2\left(\int_\Gamma \nu\norma{\E_\Gamma(\u)}^2\right)+C,\quad \forall t\geq \tau. \end{align} Indeed, the constant in estimate \eqref{final} depends on the final time. Nonetheless, given the information that we now have, the argument towards \eqref{final} could be refined in order to prove that no dependence on $T$ is actually needed in that constant. Recalling \eqref{H1} and $\nu\geq \nu_*>0$, we can apply the uniform Gronwall's Lemma (\cite{Temam}) to infer \begin{align*} \norm{\E_\Gamma(\u)}_{L^\infty(\tau,\infty;\L^2(\Gamma))}+\norm{\u}_{L^2(t,t+1;\H^2(\Gamma))}+\norm{\partial_t\u}_{L^2(t,t+1,\L_\sigma^2(\Gamma))}\leq C(\tau),\quad \forall t\geq \tau, \end{align*} where $C$ depends on $\tau$, $\norm{\u_0}$, $\f$, $\Gamma$, and the parameters of the problem. Again by Korn's inequality, this also implies, thanks to \eqref{unif}, \begin{align*} \norm{\u}_{L^\infty(\tau,\infty;\H^1_\sigma(\Gamma))}\leq C, \end{align*} entailing also \eqref{regg1} by standard embeddings. In conclusion, if we assume $\u_0\in \H^1_\sigma(\Gamma)$, by Theorem \ref{thm1} there exists a unique strong solution $\u$ departing from this initial datum. Then, having fixed $\tau=1$, we know that estimates \eqref{regg1}--\eqref{regg2} hold on $[1,\infty)$. Moreover, from Theorem \ref{thm1} we infer that \begin{align*} \norm{\u}_{L^\infty((0,1);\H^1_\sigma(\Gamma))}+\norm{\u}_{L^2((0,1);\H^2(\Gamma))}+\norm{\partial_t\u}_{L^2((0,1),\L_\sigma^2(\Gamma))}\leq C(\norm{\u_0}_{\H^1_\sigma(\Gamma)}), \end{align*} and thus, putting together the estimates, we deduce that \eqref{regg1}--\eqref{regg2} also hold on $[0,\infty)$, as long as we recall that the constants appearing also depend on the $\H^1$-norm of $\u_0$. This concludes the proof. \section{ Attractors for bounded trajectories: Proofs of Section \ref{sec:attr}}\label{bdda} \subsection{Proof of Theorem \ref{th1}} We begin the proof by fixing $r\geq0$ and considering the complete metric space $\mathbb B_r$ defined in \eqref{Ar}. Recall that $$ \norm{A}_\Lts:=\sup_{\u\in A}\norm{\u},\quad \forall A\subset \Lts. $$ Thanks to Proposition \ref{pp}, in the notation of the (restricted) dynamical system $(S(t),\BBB_r)$, we deduce that \begin{align*} \norm{(\I-\P_\mathcal K)S(t)B}_{\Lts}\leq \sqrt{e^{-\zeta t}\norm{(\I-\P_\mathcal K)B}_{\Lts}^2+\omega(1+r^2)},\quad \forall t\geq 0, \end{align*} for any bounded $B\subset \BBB_r$, where the constants $\zeta,\omega$ do not depend on the size of $B$, i.e., on $\norm{B}_{\Lts}$. Moreover, due to the assumptions on $ \f_K$, exploiting Proposition \ref{lemmadec} point (ii) we also have \begin{align*} \norm{\Pk S(t)B}_\Lts\leq \norm{\Pk B}_\Lts\leq r,\quad \forall t\geq 0, \end{align*} for \textit{any} set $B\subset \BBB_r$. This shows in particular that the map $S(t)$ satisfies \begin{align*} S(t):\ \BBB_r\to \BBB_r, \quad \forall t\geq 0. \end{align*} Therefore, we can consider the dynamical system $(S(t),\BBB_r)$. Next, we check that \begin{align} \mathcal B_0^r:= \left\{\u\in \BBB_r:\ \norm{(\I-\Pk)\u}\leq \sqrt{\frac12+\omega(1+r^2)}\right\}\subset \BBB_r, \label{B0r} \end{align} which is an absorbing set for $(S(t),\BBB_r)$. Indeed, from the estimates above we have that, given $B\subset \BBB_r$ bounded set, there exists $t_B:=\ln(2\norm{(I-\Pk)B}_\Lts)/\zeta$, depending only on the size of $B$ (i.e., $\norm{B}_{\Lts}$), so that $$ S(t)B\subset \BB_0^r,\quad \forall t\geq t_B. $$ Since also $\BB_0^r$ is bounded, there exists $t_{\BB_0^r}\geq0$, depending only on $\zeta,\omega,r$ through $\norm{(\I-\Pk)\BB_0^r}_\Lts$, such that \begin{align} S(t)\BB_0^r\subset \BB_0^r,\quad \forall t\geq t_{\BB_0^r}. \label{absorbing1} \end{align} We can now observe that, thanks to the instantaneous-regularization property \eqref{regg1} of any solution $\u$, for the fixed $\tau=t_{\BB_0^r}$ it holds (see Theorem \ref{insta}) $$ \sup_{t\geq t_{\BB_0^r}}\norm{S(t)\BB_0^r}_{\H^1(\Gamma)}\leq C(t_{\BB_0^r})\norm{\BB_0^r}_{\Lts}\leq C(t_{\BB_0^r},r,\omega,\zeta), $$ where the dependencies of $C$ come from the definition of $\BB_0$. Therefore, we can introduce the set \begin{align} \BB_1^r:=\{\u\in \BB_0^r\cap \H^1(\Gamma):\ \norm{\u}_{\H^1(\Gamma)}\leq C(t_{\BB_0^r}, r,\omega,\zeta)\}, \label{B1r} \end{align} so that $$S(t)\BB_0^r\subset \BB_1^r,\quad \forall t\geq t_{\BB_0^r},$$ entailing that $\BB_1^r\subset \BB_r$ is also a bounded absorbing set. Since this set is also compact, by a standard application of the theory for dissipative dynamical systems on the complete metric space $\BBB_r$ (see, e.g., \cite{Temam}) we infer that there exists the (unique) global attractor $\AA_r\subset \mathcal B_1^r\subset \H^1_\sigma(\Gamma)$ for the dynamical system $(S(t),\BBB_r)$, such that \begin{itemize} \item $\mathcal A_r$ is nonempty, compact and connected, \item $\mathcal A_r$ is invariant, i.e., $S(t)\mathcal A_r=\mathcal A_r$, for any $t\geq0$, \item $\mathcal A_r$ is attracting, i.e., for any $B\subset \mathbb B_r$ bounded it holds ${\rm dist}(S(t)B,\mathcal A_r)\to 0$ as $t\to \infty$. \end{itemize} Moreover, it also holds \begin{align} \mathcal A_r=\{\xi(0)\in \mathbb B_r:\ \xi \text{ is a bounded complete trajectory for $S(t)$ in }\mathbb B_r \}. \label{bdd_traj2} \end{align} This result can be obtained for any $r\geq 0$, so that we have constructed a family $\{\AA_r\}_{r\geq 0}\subset \Lts$ of global attractors for the dynamical systems $(S(t),\BBB_r)$. As already noticed, the characterization property \eqref{bdd_traj2} allows to deduce, since $\BBB_{r_1}\subset \BBB_{r_2}$ for any $r_1\leq r_2$, that $\mathcal A_{r_1}\subset \mathcal{A}_{r_2}$ for any $r_1\leq r_2$, i.e., the family $\{\AA_r\}_{r\geq 0}\subset \Lts$ is increasing. Our aim is now to link this family to the attractor of the \textit{full} dynamical system $(S(t),\Lts)$, which is expected to be the set $\mathcal J$ (coinciding in this case with $\mathcal I$, as already observed). We thus define the following $$ \widetilde{\AA}:={\bigcup_{r\geq 0}\AA_r}={\bigcup_{ r \in \N}\AA_r}, $$ where the last identity is due to the fact that the family of global attractors is increasing with $r\geq 0$. First, we show that $\widetilde{\AA}$ is attracting for $(S(t),\Lts)$. Indeed, let us consider a bounded set $B\subset \Lts$. There exists $r>0$ sufficiently large such that $B\subset \BBB_r$. From the attracting property of the corresponding set $\AA_r\subset \widetilde{\AA}$, it then holds \begin{align*} \text{\rm dist}_\Lts(S(t)B, \widetilde{\AA})\leq \text{\rm dist}_\Lts(S(t)B, \AA_r)\to 0,\quad \text{as }t\to \infty. \end{align*} As a consequence, $\tA$ is attracting. As $\overline{\tA}^{\Lts}$ is obviously closed, we immediately deduce from Property B. of Lemma \ref{obv} that $\mathcal J\subset \overline{\tA}^{\Lts}$. Now, by construction any bounded set $\AA_r$ is fully invariant, in the sense that $S(t)\AA_r=\AA_r$ for any $t\geq 0$. Then, by Lemma \ref{obv} property C., $\AA_r\subset \mathcal J$ for any $r\geq 0$, and thus $$ \bigcup_{r\geq 0}\AA_r\subset \mathcal J. $$ Clearly, this immediately entails that $\JJ$ is nonempty. In order to conclude the identification and show that $\JJ=\tA$, we thus need to further show that $\JJ\subset \tA$. Notice that we cannot in general show that $\JJ$ is closed, which would also entail, from what observed above, that $\overline{\tA}^{\Lts}=\tA$. We will see some specific cases in Theorem \ref{spe} in which this result is true. To show $\JJ\subset \tA$, let us fix $\u\in \JJ$. By construction there exists a complete bounded trajectory $\xi$ such that $\xi(0)=\u$. Since the trajectory is bounded, it clearly holds $\xi(\R)\subset \BBB_r$ for some $r\geq0$. From property \eqref{bdd_traj2} of the corresponding set $\AA_r$, this also means that $\xi(0)=\u\in \AA_r$, entailing that $\JJ\subset \tA$. We then conclude that $\JJ=\tA$. Having identified the two sets, we can proceed with the proof. First, since $\AA_r\subset \H^1_\sigma(\Gamma)$ for any $r\geq0$, then also $\JJ\subset \H^1_\sigma(\Gamma)$ and thus, since $\H^1_\sigma(\Gamma)\hookrightarrow \Lts$ compactly, $\JJ$ must have empty interior in $\Lts$, which is property 2 of Theorem \ref{th1}. Moreover, as $\tA$ attracts and $\tA=\JJ$ we also have Property 3 of Theorem \ref{th1}. Property 4 is then a consequence of Proposition \ref{obv}, whereas Property 5 is again a consequence of Proposition \ref{obv} point B. In order to conclude the proof of Theorem \ref{th1}, we need to show that each set $\AA_r$ is of finite fractal dimension. We prove that for any $r\geq0$ there exists an exponential attractor $\mathcal M_r$ for the dynamical system $(S(t),\BBB_r)$, whose properties are the following \begin{itemize} \item $\mathcal M_r$ is compact and of finite fractal dimension $N_r$, possibly increasing with $r\geq0$. \item $\mathcal M_r$ is positively invariant, i.e., $S(t)\mathcal M_r\subset \mathcal M_r$, for any $t\geq0$, \item $\mathcal M_r$ is exponentially attracting, i.e., for any $B\subset \mathbb B_r$ bounded it holds $${\rm dist}(S(t)B,\mathcal M_r)\leq C(\norm{B}_\Lts)e^{-\gamma_r t},$$ where $C>0$ depends on the size of $B$ (i.e., $\norm{B}_{\Lts}$), and $\gamma_r>0$ is a universal constant depending only on $r$. \end{itemize} Since $\mathcal M_r$ is closed and attracting, it holds $\AA_r\subset \mathcal M_r$ and thus $\AA_r$ is of finite fractal dimension as well. In order to prove the existence of exponential attractors, we need some preliminary lemmas. First, for any $r\geq0$, recalling the definition of $\BB_1^r$ in \eqref{B1r}, we know that there exists $t_1^r=t_1^r(r,\zeta,\omega)$ such that $$ S(t)\BB_1^r\subset \BB_1^r,\quad \forall t\geq t_1^r. $$ Therefore, we can introduce the set \begin{align*} \mathbb{C}_r:=\overline {\bigcup_{t\geq t_1^r}S(t)\mathcal{B}_1^r}^{\Lts}\subset \BB_1^r, \end{align*} which is compact, positively invariant, and absorbing. Let us then prove the following. \begin{lemm} Under assumptions \eqref{C1}--\eqref{C2}, given $\u_{0,1},\u_{0,2}\in \Lts$, it holds \begin{align} \norm{\Pk \left(S(t) \u_{0,1} -S(t) \u_{0,2} \right)}_{\H^1(\Gamma)} \leq C\norm{\Pk \left(S(t) \u_{0,1} -S(t) \u_{0,2} \right)},\quad \forall t\geq 0, \label{regKill} \end{align} where $C>0$ only depends on $\Gamma$. \noindent Moreover, given $\u_{0,1},\u_{0,2}\in \mathbb C_r$, for any $r\geq0$ and $T> 0$ there exists $C=C(T,r,\zeta,\omega)>0$ such that \begin{align} \Vert {\boldsymbol \varepsilon}_\Gamma\left(S(t)\uu_{0,1}-S(t)\uu_{0,2} \right)\Vert^2\leq \frac{C}{t}\Vert \uu_{0,1}-\uu_{0,2} \Vert^2,\quad \forall t\in(0,T], \label{expo2} \end{align} \end{lemm} \begin{proof} Thanks to Theorem \ref{thm1}, since $\mathbb C_r\subset \H^1_\sigma(\Gamma)$, given $\u_{0,1},\u_{0,2}\in \Lts$ there exist (unique) strong solutions $\u_1,\u_2$ on $[0,\infty)$ to \eqref{t1}--\eqref{tangential} corresponding to these initial data. Their regularity is enough to perform rigorously the next computations, leading to \eqref{expo2}. In particular, let us notice that $\u:=\u_1-\u_2$ satisfies \begin{align} &\label{t22}\partial_t\u+(\u_1\cdot \nabla_\Gamma)\u+(\u\cdot \nabla_\Gamma)\u_2-2\P_\Gamma\divg(\nu\boldsymbol\varepsilon_\Gamma(\u))+\nabla_\Gamma p=\f(\cdot,\u_1)-\f(\cdot,\u_2),\\& \divg\u=0,\\& \u(0)=\u_{0,1}-\u_{0,2}, \label{tangential2} \end{align} where $p$ as a suitable zero-integral-mean pressure, corresponding to $p_1-p_2$. First, to show \eqref{regKill}, it is enough to observe that, by Korn's inequality \eqref{Korn2}, \begin{align} \norm{\u}_{\H^1(\Gamma)}\leq C\norm{\u},\quad \forall \u\in \mathcal K, \end{align} thanks to the fact that ${\boldsymbol \varepsilon}_\Gamma(\u)=\boldsymbol 0$ for any $\u\in \mathcal K$. Let us now multiply \eqref{t22} by $\partial_t\u$ and integrate over $\Gamma$. After an integration by parts, this gives \begin{align} &\label{**}\frac d{dt}\int_\Gamma\nu(x)\norma{\E_\Gamma(\u)}^2+\norm{\partial_t\u}^2\\ &\quad =\int_\Gamma (\u_1\cdot \nabla_\Gamma)\u\cdot \partial_t\u+\int_\Gamma (\u\cdot \nabla_\Gamma)\u_2\cdot \partial_t\u+\int_\Gamma(\f(\cdot,\u_1)-\f(\cdot,\u_2))\cdot \partial_t\u.\nonumber \end{align} Now, by H\"{o}lder's and Korn's inequalities and by recalling the embedding $\H^2(\Gamma)\hookrightarrow \L^\infty(\gam)$, we have \begin{align*} &\norma{\int_\Gamma (\u_1\cdot \nabla_\Gamma)\u\cdot \partial_t\u}\\&\leq \norm{\u_1}_{\L^\infty(\gam)}\norm{\nabla_\Gamma \u}\norm{\partial_t\u}\\&\leq C\norm{\u_1}_{\H^2(\Gamma)}(\norm{\u}+\norm{{\boldsymbol \varepsilon}_\Gamma(\u)})\norm{\partial_t\u} \\& \leq C\norm{\u_1}_{\H^2(\Gamma)}^2\norm{\E_\Gamma(\u)}^2+C\norm{\u_1}_{\H^2(\Gamma)}^2\norm{\u}^2+\frac14\norm{\partial_t\u}^2. \end{align*} In a similar way, by Gagliardo--Nirenberg's and Korn's inequalities we have \begin{align*} \norma{\int_\Gamma (\u\cdot \nabla_\Gamma)\u_2\cdot \partial_t\u}&\leq \norm{\u}_{\L^4(\Gamma)}\norm{\nabla_\Gamma\u_2}_{\L^4(\Gamma)}\norm{\partial_t\u} \\& \leq \norm{\u}^\frac12\norm{\u}_{\H^1(\Gamma)}^\frac12\norm{\u_2}_{\W^{1,4}(\Gamma)}\norm{\partial_t\u}\\& \leq C\norm{\u}^\frac12(\norm{\E_\Gamma(\u)}^\frac12+\norm{\u}^\frac12)\norm{\u_2}_{\W^{1,4}(\Gamma)}\norm{\partial_t\u}\\& \leq C(1+\norm{\u_2}_{\W^{1,4}(\Gamma)}^4)\norm{\u}^2+C\norm{\E_\Gamma(\u)}^2+\frac14\norm{\partial_t\u}^2. \end{align*} Eventually, by assumption \eqref{C2} on $\f$ we have \begin{align*} \norma{\int_\Gamma(\f(\cdot,\u_1)-\f(\cdot,\u_2))\cdot \partial_t\u}&\leq \norm{\f(\cdot,\u_1)-\f(\cdot,\u_2)}\norm{\partial_t\u}\\&\leq C\norm{\u}\norm{\partial_t\u}\\&\leq C \norm{\u}^2+ \frac14 \norm{\partial_t\u}^2. \end{align*} As a consequence, we can sum up all the estimates and, recalling that $\nu\geq \nu_*>0$ and multiplying equation \eqref{**} by $t$ we get \begin{align} \nonumber&\frac d{dt}t\int_\Gamma\nu(x)\norma{{\boldsymbol \varepsilon}_\Gamma(\u)}^2+\frac18t\norm{\partial_t\u}^2\\&\leq\nonumber Ct(1+\norm{\u_1}_{\H^2(\Gamma)}^2)\int_\Gamma\nu(x)\norma{{\boldsymbol \varepsilon}_\Gamma(\u)}^2\\&+ Ct(1+\norm{\u_1}_{\H^2(\Gamma)}^2+\norm{\u_2}_{\W^{1,4}(\Gamma)}^4)\norm{\u}^2+C\int_\Gamma\nu(x)\norma{{\boldsymbol \varepsilon}_\Gamma(\u)}^2.\label{ree2} \end{align} Now, recalling \eqref{contdep2} we deduce \begin{align} \sup_{t\in[0,T]}\norm{\u(t)}^2+2\nu_*\int_0^T \norm{\E_\Gamma(\u(t))}^2dt\leq C(T,r,\zeta,\omega)\norm{\u_{0,1}-\u_{0,2}}^2, \label{ff} \end{align} so that \begin{align} \nonumber&\frac d{dt}t\int_\Gamma\nu(x)\norma{\E_\Gamma(\u)}^2+ \frac14 t\norm{\partial_t\u}^2\\&\leq\nonumber Ct(1+\norm{\u_1}_{\H^2(\Gamma)}^2)\int_\Gamma\nu(x)\norma{{\boldsymbol \varepsilon}_\Gamma(\u)}^2\\&+ Ct(1+\norm{\u_1}_{\H^2(\Gamma)}^2+\norm{\u_2}_{\W^{1,4}(\Gamma)}^4)\norm{\u_{0,1}-\u_{0,2}}^2+\int_\Gamma\nu(x)\norma{{\boldsymbol \varepsilon}_\Gamma(\u)}^2,\label{ree} \end{align} and, applying Gronwall's Lemma, since, by Theorem \ref{thm1}, $\u_i$, $i=1,2$, are strong solutions, and thus, by interpolation, $\u_i\in L^2(0,T;\H^2(\Gamma))\cap \L^\infty(0,T;\H^1(\Gamma))\cap L^4(0,T;\W^{1,4}(\Gamma))$, we obtain \begin{align*} t\norm{\E_\Gamma(\u)(t)}^2&\leq C(T,r,\zeta,\omega)\norm{\u_{0,1}-\u_{0,2}}^2+C\int_0^T\norm{\E_\Gamma(\u(t))}^2dt\\&\leq C(T,r,\zeta,\omega)\norm{\u_{0,1}-\u_{0,2}}^2,\quad \forall t\in(0,T], \end{align*} where we also used \eqref{ff}. This concludes the proof of the lemma. \end{proof} Moving from the continuous-dependence estimate \eqref{contdep2} and \eqref{regKill}--\eqref{expo2}, we can infer the following smoothing estimate \begin{align} \norm{S(t)\u_{0,1}-S(t)\u_{0,2}}_{\H^1(\Gamma)}\leq \frac{C(T,r,\zeta,\omega)}{\sqrt t}\norm{\u_{0,1}-\u_{0,2}},\quad \forall t\in(0,T].\label{smoothing} \end{align} We can now continue the proof of Theorem \ref{thm:323}, following, for instance, \cite{Zelik}. Observe that, thanks to the uniform regularity of strong solutions given in Theorem \ref{insta}, it holds \begin{align*} \norm{\partial_t S(t)\u_0}_{L^2(t,t+1;\Lts)}\leq C(r,\omega,\zeta,\tau),\quad \forall \u_0\in \mathbb C_r,\quad \forall t\geq \tau . \end{align*} Setting $ \u (t)=S(t) \u_0 $, with $ \u_0 \in\mathbb{C}_r$, we have, for any given $T>0$, \begin{align} \label{lipt} \Vert \u(t)-\u(s)\Vert\leq \int_s^t\Vert \partial_t\u(\tau)\Vert d\tau\leq \vert t-s\vert^{1/2}\left(\int_s^t\Vert \partial_t\u(\tau)\Vert^2 d\tau\right)^{1/2}\leq C(T,r,\omega,\zeta)\vert t-s\vert^{1/2} \end{align} for any $s,t\in[0,T]$, i.e., $t \mapsto S(t) u_0 $ is $(1/2)$-H\"{o}lder continuous in $[0,T]$, with $C$ depending only on $T,r,\omega,\zeta$. This, together with the continuous-dependence estimate \eqref{contdep2} allows to deduce that $S$ is $(1/2)$-H\"{o}lder continuous in $[0,T]\times \mathbb C_r$, for any $T>0$. Let us now fix $t_*>0$. Thanks to the smoothing property \eqref{smoothing} valid at $t=t_*>0$, the discrete dynamical system generated by the iterations of $(S(t_*),\mathbb C_r)$ possesses an exponential attractor $\mathcal{M}^*_r\subset \mathbb C_r$ (see, e.g., \cite[Thm.(2)7]{Zelik}). Moreover \eqref{contdep2} and \eqref{lipt} entail that $$ S:[0,t_*]\times \mathbb{C}_r\to \mathbb{C}_r, \quad S(t, \u_0 ):=S(t) \u_0 , $$ is H\"{o}lder continuous, when $\mathbb{C}_r$ is endowed with the $\Lts$ topology. Therefore, we can define $$ \mathcal{M}_r:=\bigcup_{t\in[0,t_*]}S(t)\mathcal{M}_r^*\subset \mathbb{C}_r, $$ and, following again \cite{Zelik}, show that $\mathcal{M}_r$ is an exponential attractor for $S(t)$ on $\mathbb{C}_r$. Since $\mathbb{C}_r$ is also a compact absorbing set, the basin of exponential attraction of $\mathcal{M}_r$ is the whole phase space $\BBB_r$. This means that $\mathcal{M}_r$ is an exponential attractor on $\BBB_r$. Note that the finite fractal dimension $N_r$ of $\mathcal M_r$ only depends on $r$ and that there exists an increasing function $Q_r$ and $\gamma_r$, only depending on $r$ (and also $\zeta,\omega$), such that, for any bounded set $B\subset \BBB_r$, \begin{align*} \text{\rm dist}(S(t)B,\mathcal M_r)\leq Q_r(\norm{B}_\Lts)e^{-\gamma_r t},\quad \forall t\geq 0. \end{align*} This comes from the properties \eqref{smoothing} and \eqref{lipt}, since the constants involved only depend on $T,r,\omega$, and $\zeta$. Since then $\mathcal A_r\subset \mathcal M_r$ for any $r\geq0$, we deduce that $\AA_r$ is also of finite fractal dimension. This concludes the proof of Theorem \ref{th1}. \subsection{Proof of Theorem \ref{thm:325}} In the previous section we have shown the existence of an exponential attractor $\mathcal M_r$ for the system $(S(t),\BBB_r)$, for any $r\geq0$. Let us define {$\mathcal M:=\bigcup_{m\in \mathbb N}\mathcal M_m$}. Since $\mathcal M_r$ is positively invariant for any $r\geq0$, we have $$ S(t)\mathcal M\subset \bigcup_{m\in \mathbb N} S(t)\mathcal M_m\subset \mathcal M, \quad \forall t\geq0, $$ i.e., also $\mathcal M\subset \Lts$ is positively invariant. In conclusion, the exponential attraction stated in property (3) directly comes from the exponential attraction of each {$\mathcal M_m$ in $\BBB_m$, for any $m\in \mathbb N$}. \subsection{Proof of Theorem \ref{spe}} To prove this statement, we need first to show that $\f_K=\Pk((\v\cdot\nabla_\Gamma) \u_K)$ satisfies assumptions \eqref{C1}--\eqref{C2} and \eqref{uk1} ( as $\f$ is assumed to satisfy these assumptions, also $\f_{NK}$ will satisfy them). Assumption \eqref{C1} is trivially checked, since \begin{align} \int_\Gamma \Pk((\v\cdot\nabla_\Gamma) \u_K)\cdot \u=\int_\Gamma (\v\cdot\nabla_\Gamma) \u_K\cdot \u_K=0, \label{ll} \end{align} for any $\u\in \L^2(\Gamma)$, since $\v\in \L_\sigma^2(\Gamma)$. Note that $\u_K=\Pk\u$ belongs also to $\H^1_\sigma(\Gamma)$, since, by Korn's inequality \eqref{Korn2}, $\norm{\u_K}_{\H^1_\sigma(\Gamma)}\leq C\norm{\u_K}\leq C\norm{\u}$. Again by Korn's inequality, also assumption \eqref{C2} is verified, since $\f_K$ is linear in $\u$ and $\v\in \L^\infty(\Gamma)$. Eventually, assumption \eqref{uk1} is easily verified, again thanks to \eqref{ll}. Therefore, Theorem \ref{th1} concerning the $\sigma$-attractor $\JJ$ also applies in this case. Clearly, the same holds if $\f_K= \boldsymbol 0$. We now aim at refining the results of the theorem. In the case $\f_K= \boldsymbol 0$, it is immediate to deduce from \eqref{K1}--\eqref{K2} that $$ \partial_t \u_K(t)=\boldsymbol 0, $$ so that $\Pk S(t)\u_0=\u_K(t)=\u_K(0)=\Pk\u_0$, for any $\u_0\in \Lts$ and any $t\geq0$, i.e., the Killing component of any trajectory is always constant. In particular, it holds \begin{align} \norm{\Pk S(t)\u_0}=\norm{\Pk\u_0},\quad \forall t\geq0.\label{conserved} \end{align} On the other hand, when $\f_K=\Pk((\v\cdot\nabla_\Gamma) \u_K)$, the $\L^2$-norm of $\u_K$ does not change in time, since we have \begin{align*} \frac12\frac d {dt} \norm{\u_K}^2=\int_\Gamma (\v\cdot\nabla_\Gamma) \u_K\cdot \u_K=0, \end{align*} since $\v\in \L^2_\sigma(\Gamma)$. Thus, also in this case \eqref{conserved} holds. We can thus consider the two cases with the same approach. Let us now introduce, for any fixed $r\geq0$, the (closed) set \begin{align} \label{tB1} \widetilde{\BBB}_r:=\{\u\in \Lts:\ \norm{\Pk\u}=r\}, \end{align} as in \eqref{tB1}, which is a complete metric space if endowed with the $\Lts$ topology. Note that in this case the family $\{\widetilde{\BBB}_r\}_{r\geq0}$ is not monotone. Then, by \eqref{conserved}, the map $S(t)$ is such that $$ S(t):\quad \widetilde{\BBB}_r\to \widetilde{\BBB}_r, $$ and thus we can define the dynamical system $(S(t),\widetilde{\BBB}_r)$. Repeating the same arguments as in the proof of Theorem \ref{thm1}, by simply substituting $\BBB_r$ with $\widetilde{\BBB}_r$, we can show that, for any $r\geq0$, there exists a compact, invariant, attracting set $\widetilde{\AA}_r$ of finite fractal dimension, which is the global attractor for the dynamical system $(S(t),\widetilde{\BBB}_r)$. Therefore, it immediately follows from the properties of $\JJ$ that $\bigcup_{r\geq 0}\widetilde{\AA}_r\subset \JJ$. For the reverse inclusion, let us consider $\u\in \JJ$. Then, there exists a complete bounded trajectory $\xi$ such that $\xi(0)=\u$. Since the trajectory is bounded and by \eqref{conserved} it holds $\norm{\Pk\xi(t)}=\norm{\Pk\u}$ for any $t\geq0$, we have that $\xi(\R)\subset \widetilde{\BBB}_r$ for $r=\norm{\Pk\u}$. From property \eqref{bdd_traj2} of the corresponding set $\widetilde{\AA}_r$, which is the global attractor of system $(S(t),\widetilde{\BBB}_r)$, this also means that $\xi(0)=\u\in \widetilde{\AA}_r$, entailing that $\JJ\subset \bigcup_{r\geq 0}\widetilde{\AA}_r$. We thus conclude that $\JJ=\bigcup_{r\geq 0}\widetilde{\AA}_r$, i.e., $\JJ$ has the desired pancake-like structure. In order now to prove that $\JJ$ is bounded compact and bounded finite dimensional, we need to consider a closed and bounded set $B\subset \Lts$ and prove that $\JJ\cap B \subset \AA_r$, for some $r\geq0$, where $\AA_r$ is the global attractor for the system $(S(t),\BBB_r)$ introduced in Theorem \ref{th1}. Indeed, this will entail that also $\JJ\cap B$ is compact and of finite fractal dimension. To prove $\JJ\cap B \subset \AA_r$ for some $r\geq0$, let us observe that, since $B$ is bounded, there exists $r\geq 0$ such that $\JJ\cap B\subset \BBB_r$. We then consider $\u\in \JJ\cap \BBB_r$. By definition of $\JJ$ there exists a complete bounded trajectory $\xi$ such that $\xi(0)=\u$. By \eqref{conserved} it holds $\norm{\Pk\xi(t)}=\norm{\Pk\u}\leq r$, for any $t\geq0$, so that clearly $\xi(\R) \subset \BBB_r$. Then, by property \eqref{bdd_traj2} of the corresponding global attractor ${\AA}_r$ of system $(S(t),\BBB_r)$, we deduce $\u\in \AA_r$, entailing $\JJ\cap B\subset \AA_r$. As we have proved that $\JJ$ is bounded closed, it is immediate to prove that $\JJ$ is closed. Indeed, let us consider a sequence $\{\u_n\}_{n\in\N}\subset \JJ$ such that $\u_n\to \u^*$ in $\Lts$ as $n\to \infty$. Then the set $B=\overline{\{\u_n\}_{n\in\N}}^\Lts$ is closed and bounded, so that $\JJ\cap B$ is also closed. Since $\{\u_n\}_{n\in\N}\subset \JJ\cap B$, this means that $\u^*\in \JJ\cap B\subset\JJ$, entailing that $\JJ$ is closed in $\Lts$. To prove that $\Pk\JJ=\KK$, we refer to the proof of Theorem \ref{th2}. Indeed, both $\f_K=\boldsymbol 0$ and $\f_K=\Pk((\v\cdot\nabla_\Gamma) \u_K)$ satisfy the assumptions there, and thus property 5 in the statement of Theorem \ref{th2} gives the identification. To conclude the proof we only need to consider the case when, additionally, $\f_{NK}=\boldsymbol 0$. In this case, by repeating the same proof leading to Proposition \ref{pp}, we immediately see that, for any $\u_0\in \Lts$ it holds (see also \cite{Simonett2}) \begin{align*} \norm{(\I-\Pk)S(t)\u_0}^2\leq e^{-\zeta t}\norm{(\I-\Pk)\u_0}^2\to 0\quad\text{as }t\to \infty, \end{align*} so that the set $$\BB_s:=\{\u\in \Lts:\ \norm{(\I-\Pk)\u}\leq s\},\quad s> 0,$$ is an absorbing set, for any $s>0$. Therefore, since $\JJ$ is the minimal closed set attracting bounded sets, it holds that $$ \JJ\subset \bigcap_{s>0}\BB_s, $$ entailing that $\norm{(\I-\Pk)\u}\leq s$ for any $s>0$ and any $\u\in \JJ$. Therefore, it must be $\norm{(\I-\Pk)\u}=0$ for any $\u\in \JJ$, and thus $\JJ=\KK$, concluding the proof. \subsection{Proof of Theorem \ref{interesting}} The proof of this theorem is analogous to the one of Theorem \ref{th1}. In particular, in this case we can directly consider the map $$ S(t):\quad \Lts\to \Lts,\quad \forall t\geq 0, $$ since by assumption there exists $C\subset \Lts$ which is a bounded absorbing set, so that the system is dissipative. Then, we can define an absorbing $\Lts$-ball $\BB_r^\Lts$ of radius $r\geq0$ sufficiently large such that $C\subset\BB_r^\Lts$, and repeat the very same proof of Theorem \ref{th1}, substituting the absorbing ball $\BB_r^0$ with $\BB_r^\Lts$ and the phase space $\BBB_r$ with the whole $\Lts$. Note that in this case we are assuming the additional hypothesis \eqref{extra} on $\f_{NK}$, while omitting \eqref{extra2}, since $\f_K$ does not satisfy \eqref{nega}, but \eqref{pos}. In this way we can retrieve the existence of a global attractor $\AA\subset \Lts$ to the system, which is nonempty, compact, invariant, attracting, and of finite fractal dimension. Since $C$ is an absorbing set, the global attractor is also contained in $C$. Clearly it also holds that $\AA$ coincides with $\JJ$ defined in \eqref{J}, by the natural property \eqref{bdd_traj} of any global attractor (in this case the phase space is directly the whole $\Lts$), i.e., $\AA$ only contains the complete and bounded trajectories in $\Lts$. Note also that the existence of an exponential attractor $\mathcal M$ is a by-product of the aforementioned proof, from which we deduce the finite dimensionality of $\JJ$. Since any trajectory is bounded by assumption, it also holds $\JJ=\mathcal I$, where $\mathcal I$ is defined in \eqref{I}. Note also that, since we are assuming \eqref{extra}, estimate \eqref{gr} holds and thus the set $$ \BB:=\left\{\u\in\Lts:\ \norm{(\I-\Pk)\u}\leq \sqrt{\frac 12 +\omega}\right\} $$ is an absorbing set for the system, entailing $\JJ\subset \mathcal B$. To conclude, if we assume that $\Pk S(t)B\to 0$ as $t\to \infty$ for any set $B\subset \Lts$ such that $\Pk B$ is bounded, then we can define the absorbing sets \begin{align} C_s:=\left\{\u\in\Lts:\ \norm{\Pk\u}\leq s\right\},\quad \forall s>0. \end{align} Indeed, by assumption, for any $s>0$ and any bounded set $B\subset \Lts$ there exists $t_{s,B}>0$ such that $S(t)B\subset C_s$ for any $t\geq t_{s,B}$. Therefore, by the minimal attracting property of $\JJ$, $\JJ\subset \bigcap_{s>0}C_s$. This entails that, for any $\u\in\Lts$, $\norm{\Pk\u}=0$, proving \eqref{best}. \section{ Attractors for unbounded trajectories: Proofs of Section \ref{unbd}}\label{unbdd} \subsection{Proof of Theorem \ref{th2}} To prove the statement, we apply Lemma \ref{A1}, so that we only need to check its assumptions. Let us define the set $$ Q_0:=\left\{\v\in \Lts: \norm{(\boldsymbol I-\P_{\mathcal{K}})\v}\leq \frac12+\omega\right\}, $$ where $\omega$ is given in \eqref{gr}. Moroever, let $$ Q:=\overline{\bigcup_{t\geq 0}S(t)Q_0}. $$ Recall that in this case we are assuming \eqref{extra} for $\f_{NK}$. Thanks to estimate \eqref{gr}, it is then immediate to deduce that $Q$ is an absorbing set, i.e., for any bounded set $B\subset \Lts$ there exists $t_B>0$ such that $S(t)B\subset Q$, for any $t\geq t_B$. We now need to show that $Q$ is positively invariant. Let us fix $t\geq0$ and consider $\u\in S(t)Q$. This means that there exists $\u_0\in Q$ such that $S(t)\u_0=\u$. Therefore, from the definition of $Q$ there exists a sequence $\{t_n\}_n$, $t_n\geq0$ and a sequence $\{\u_n\}_n\subset Q_0$ such that $S(t_n)\u_n \to \u_0$ as $n\to \infty$. Thus, by the continuity properties of the semigroup $S(t)$, we have $$ S(t)S(t_n)\u_n=S(t+t_n)\u_n \to S(t)\u_0=\u,\quad\text{as }n\to \infty. $$ Since $\{S(t+t_n)\u_n\}_{n}\subset \bigcup_{t\geq 0}S(t)Q_0$, this entails that $\u\in Q$, and thus $S(t)Q\subset Q$ for any $t\geq0$, i.e., $Q$ is positively invariant. This means that assumption (H1) of Lemma \ref{A1} is satisfied by this choice of $Q$, and by setting $D_1=D_2=\frac12+\omega$. Concerning assumption (H2), we recall that, by assumption, $\f$ is chosen so that we are in case (iii) of Proposition \ref{lemmadec} (see also Remark \ref{relax}). Then, for any $\u_0\in\Lts$ such that $\norm{\Pk\u_0}\geq R_0$, we have \begin{align}\label{exploding} \norm{\Pk S(t)\u_0}\geq \norm{\Pk\u_0},\quad \forall t\geq 0. \end{align} We can thus define the set \begin{align*} H_R:=\{\u\in Q:\ \norm{\Pk\u_0}\leq R\},\quad \forall R\geq R_0. \end{align*} By setting $S(R)=R$ and $R_1=R_0$, we see that $\{\u\in\Lts:\ \norm{\Pk\u}\leq S(R)\}\cap Q\subset H_R$, and, obviously, $S(R)=R\to \infty$ as $R\to \infty$. Moreover, for every $R\geq R_1=R_0$ it holds $H_R \subset \{\u\in \Lts:\ \norm{\Pk\u}\leq R\}$. In conclusion, thanks to \eqref{exploding} (notice that this condition justifies the assumptions on $\f_K$), we have $$ S(t)(Q\setminus H_R)\subset Q\setminus H_R,\quad \forall t\geq 0,\quad \forall R\geq R_1=R_0. $$ Indeed, $Q\setminus H_R=\{\u\in Q:\ \norm{\Pk\u}>R\}$ and, by \eqref{exploding}, $\norm{\Pk S(t)\u}\geq \norm{\Pk \u}>R$, for any $\u\in Q\setminus H_R$. Moreover, since $Q$ is positively invariant, $S(t)(Q\setminus H_R)\subset Q$ for any $t\geq 0$. This means that $S(t)(Q\setminus H_R)\subset Q\setminus H_R$, as desired. Hence, assumption (H2) of Lemma \ref{A1} is satisfied. Eventually assumption (H3) of Lemma \ref{A1} is easily verified. Indeed, by Theorem \ref{thm1} (see, in particular, \eqref{regulari}) we know that, for any $\u_0\in \Lts$ $$ \norm{S(\cdot)\u_0}_{L^\infty(\frac t2,2t;\H^1_\sigma(\Gamma))}\leq C(\norm{\u_0},t,T),\quad \forall t>0. $$ Let us fix $t>0$. Then, for any $B\subset \Lts$ bounded, it holds $$ \sup_{\u_0\in B}\norm{S(t)\u_0}_{\H^1_\sigma(\Gamma)}\leq C(\norm{B}_{\Lts},t), $$ where the constant $C>0$ only depends on $t$ and the $\L^2$-diameter of $B$. The ball $K(t,B):=\{\u\in \H^1_\sigma(\Gamma):\ \norm{\u}_{\H^1_\sigma(\gam)}\leq C(\norm{B}_{\Lts},t) \}$ is compact in $\Lts$ and % also it is such that \begin{align*} {S(t)B}\subset K(t,B). \end{align*} Since both $t>0$ and the bounded set $B$ are arbitrary, this means that the semigroup $S(t)$ is generalized asymptotically compact, verifying assumption (H3) of Lemma \ref{A1}. Therefore, having verified all the assumptions, Lemma \ref{A1} can be applied, which proves assertions 1--2 and 4--6 in the statement of Theorem \ref{th2}. To prove property 3 let us consider $\u_0\in \JJ$. Since $S(t)\JJ=\JJ$ for any $t\geq0$ there exists $\u_1\subset \JJ$ such that $\u_0=S(1)\u_1$. From the regularization property \eqref{regulari}, it holds that $S(1)\u_1\in \H^1_\sigma(\Gamma)$, entailing that $\u_0\in \H^1_\sigma(\Gamma)$. This means $\JJ\subset \H^1_\sigma(\Gamma)$ and thus $\JJ$ must have empty interior in $\Lts$. \subsection{Proof of Theorem \ref{t1b}} To check the statement we observe that, by restricting the semigroup to $(S(t), (\I-\Pk)\Lts)$, since $\f_K(x,\boldsymbol 0)= \boldsymbol 0$ for any $x\in \Gamma$, we obtain by uniqueness that $\Pk S(t)\u_0\equiv \boldsymbol 0$ for any $\u_0\in (\I-\Pk)\Lts$. This implies that $S(t)=(\I-\Pk)S(t)$ on $(\I-\Pk)\Lts$, hence $$ S(t): \ (\I-\Pk)\Lts\to (\I-\Pk)\Lts, $$ for any $t\geq 0$. Therefore, by exploiting the regularization properties of the system as well as the dissipative estimate \eqref{gr}, we can argue as in the proof of Theorem \ref{th1} and deduce that there exists a compact absorbing set of the same form as \eqref{B1r}, but with $r=0$. By the standard theory of dynamical systems, this entails that there exists the unique global attractor $\mathcal A_0$ for the dynamical system $(S(t), (\I-\Pk)\Lts)$. This attractor is nonempty, compact, invariant, and attracting. Moreover, $\AA_0$ can be shown (see property \eqref{bdd_traj}) to be composed of complete and bounded trajectories. Therefore we immediately infer $\mathcal A_0\subset \mathcal I$, where $\mathcal I$ is defined in \eqref{I}, concluding the proof. pendix \section{A Lemma on the existence of the unbounded attractor}\label{sec:appendix} We conclude with an Appendix presenting a technical lemma, following some ideas in \cite{Carvalho}. The proof is essentially mutated from \cite{Carvalho}, up to minor modifications. \begin{lemm}\label{A1} Under the assumptions \eqref{C1}--\eqref{C2}, let us consider the dynamical system $(S(t),\Lts)$ defined in Section \emph{ \ref{longtt}}. If \begin{enumerate}[label=(\subscript{H}{{\arabic*}})] \item There exist $D_1, D_2 > 0$ and a closed set $Q\subset \Lts$ with $$\{\u\in \Lts:\ \norm{(\I - \Pk)\u} \leq D_1\} \subset Q \subset \{ \u\in \Lts:\ \norm{(\I - \Pk)\u} \leq D_2\}$$ such that $Q$ is an absorbing set, i.e., for any bounded set $B\subset \Lts$ there exists $t_B>0$ such that $$ S(t)B\subset Q,\quad \forall t\geq t_B, $$ and positively invariant, i.e., for every $B\subset \Lts$ there exists $t_B > 0$ such that $S(t)Q \subset Q$ for every $t \geq t_B$; \item There exist two constants $R_0$ and $R_1$ with $0 < R_0 \leq R_1$ and an increasing family of closed and bounded sets $\{H_R\}_{R\geq R_0}$ with $H_R \subset Q$, such that \begin{itemize} \item for every $R \geq R_1$ we can find $S(R) \geq R_0$ such that $\{\u\in\Lts:\ \norm{\Pk \u} \leq S(R)\} \cap Q \subset H_R$ and moreover $\lim_{R\to\infty} S(R)=+\infty$, \item for every $R \geq R_1$ we have $H_R \subset \{\u\in \Lts:\ \norm{\Pk\u}\leq R\}$, \item for every $R \geq R_1$, $S(t)(Q \setminus H_R) \subset Q \setminus H_R $ for every $t\geq0$; \end{itemize} \item The semigroup $\{S(t)\}_{t\geq0}$ is generalized asymptotically compact, i.e., if for every bounded set $B\subset \Lts$ and every $t > 0$ there exists a compact set $K(t, B) \subset \Lts$ and $\varepsilon(t, B) \to 0$ as $t\to\infty$ such that \begin{align} S(t)B\subset O_\varepsilon(K(t,B)), \label{ascmp} \end{align} i.e., for any $\u\in S(t)B$ it holds $\text{\rm dist}_\Lts(\u,K(t,B))\leq \varepsilon$, \end{enumerate} then the nonempty set $\mathcal J$ defined in \eqref{J} is the unbounded attractor, satisfying: \begin{enumerate} \item $\mathcal J=\overline{\bigcap_{t\geq0}S(t)Q}$, \item $\mathcal J$ is closed and invariant, i.e., $S(t)\mathcal J =\mathcal J$ for all $t\geq0$, \item $\mathcal J$ is bounded compact, i.e., the intersection of $\mathcal J$ with any closed and bounded set $B\subset \Lts$ is compact, \item If for some $R\geq 0$, $B \in\Lts$ bounded, and $t_1 > 0$ the sets $S(t)B \cap \{\v\in \Lts:\ \norm{\P_\mathcal K \v}\leq R\}$ are nonempty for every $t \geq t_1$, then $$ \lim_{t\to\infty} {\rm dist}(S(t)B \cap \{\v\in \Lts:\ \norm{\P_\mathcal K \v}\leq R\}, \mathcal J ) = 0,$$ and $\mathcal J$ is the minimal closed set with the above property, \item $\P_\mathcal K \mathcal J = \mathcal K$. \end{enumerate} \end{lemm} \begin{proof} Concerning the proof, we refer to \cite[Theorem 3]{Carvalho}. In the notations of such theorem, we have $X=\Lts$, $E^+=\KK$, {which is a finite dimensional subspace of $X$}, and $E^-=(\I-\Pk)\Lts$, {which is clearly closed}. To be precise, in the aforementioned proof there is a stronger assumption on the semigroup $S(t)$, namely $$ S\in C([0,+\infty)\times \Lts;\Lts), $$ i.e., it is jointly time-space continuous. Actually, this {additional }assumption can be relaxed to \begin{align} S\in C([0,+\infty)\times \KK;\Lts), \label{KK}\end{align} i.e., that $S$ is jointly continuous if we restrict the phase space to $\KK$. Indeed, the only part in the proof of \cite[Theorem 3]{Carvalho} which exploits this regularity is to show that $\JJ$ is nonempty and $\Pk\JJ=\KK$, i.e., \cite[Lemma 3]{Carvalho}. In this case, all the proof is based on the fact that, having defined $$ B_R:=\{\u\in \KK:\ \norm{\u}<R+1\}, $$ for any $R\geq R_0$, we need $$ S\in C([0,T]\times \overline{B}_R;\Lts),\quad \forall T>0,\quad \forall R\geq R_0, $$ and this is ensured by the weaker \eqref{KK}, as well. In our case, the dynamical system $(S(t),\Lts)$ under consideration satisfies this assumption and thus \cite[Theorem 3]{Carvalho} can be applied. Indeed, note that, by Korn's inequality \eqref{Korn2}, $$ \norm{\u}_{\H^1(\Gamma)}\leq C\norm{\u},\quad \forall \u\in\KK, $$ and thus, by Theorem \ref{thm1} we have that the semigroup $S(t)$, when restricted to $\KK$, is such that $\partial_t S(t)\u_0\in L^2(0,T;\Lts)$ for any $T\geq0$ and any $\u_0\in \KK$ (see \eqref{strong}), where the bounding constants only depend on $\norm{\u_0}_{\H^1(\Gamma)}$, the parameters of the problem, $\f$, and $\Gamma$. This shows, as in \eqref{lipt}, that the map $S(\cdot)\u_0: [0,T]\to \Lts$ is $(1/2)$-H\"{o}lder continuous for any $T>0$ and any fixed $\u_0\in \KK$. Together with the continuous dependence estimate \eqref{contdep2} (in which the constants appearing depend on $T>0$, $\norm{\u_{0,i}}$, $i=1,2$, the parameters of the problem, $\f$, and $\Gamma$), this allows to show that $$ S\in C^{\frac12}([0,T]\times D;\Lts), $$ for any $T>0$ and any $D\subset \KK$ closed bounded set, i.e., $S$ is locally $(1/2)$-H\"{o}lder continuous in $[0,+\infty)\times \KK$, which entails \eqref{KK} and concludes the proof. \end{proof} \section*{Acknowledgments} This research was funded in whole or in part by the Austrian Science Fund (FWF) projects 10.55776/ESP552, 10.55776/F65, 10.55776/I5149, 10.55776/P32788, as well as by the OeAD-WTZ project CZ 09/2023. AP is a member of Gruppo Nazionale per l’Analisi Matematica, la Probabilità e le loro Applicazioni (GNAMPA) of Istituto Nazionale per l’Alta Matematica (INdAM), and gratefully acknowledges support from the Alexander von Humboldt Foundation. For open-access purposes, the authors have applied a CC BY public copyright license to any author-accepted manuscript version arising from this submission. Part of this research was conducted during a visit to the Mathematical Institute at Tohoku University, whose warm hospitality is gratefully acknowledged. \bibliography{Bibliography} \bibliographystyle{abbrv} \end{document}
2412.05945v3
http://arxiv.org/abs/2412.05945v3
The Multiplicity of Powers of a Class of Non-Square-free Monomial Ideals
\documentclass[12pt]{amsart} \usepackage{amsmath} \usepackage{amsthm} \usepackage{amssymb} \usepackage{graphics} \usepackage{pgf} \usepackage{tikz} \usepackage{pifont} \usetikzlibrary{arrows} \usepackage[all]{xy} \usepackage{lineno} \newcommand{\abs}[1]{\lvert#1\rvert} \newcommand{\blankbox}[2]{ \parbox{\columnwidth}{\centering \setlength{\fboxsep}{0pt} \fbox{\raisebox{0pt}[#2]{\hspace{#1}}} } } \unitlength=0.7cm \def\Box#1#2#3{\multiput(#1,#2)(1,0){2}{\line(0,1)1} \multiput(#1,#2)(0,1){2}{\line(1,0)1} \put(#1,#2){\makebox(1,1){$#3$}}} \def\LBox#1#2#3#4{\multiput(#1,#2)(#4,0){2}{\line(0,1)1} \multiput(#1,#2)(0,1){2}{\line(1,0){#4}} \put(#1,#2){\makebox(#4,1){$#3$}}} \def\Ci#1#2{\put(#1.5,#2.5){\circle{0.7}}} \def\NZQ{\Bbb} \def\NN{{\NZQ N}} \def\QQ{{\NZQ Q}} \def\ZZ{{\NZQ Z}} \def\RR{{\NZQ R}} \def\CC{{\NZQ C}} \def\AA{{\NZQ A}} \def\PP{{\NZQ P}} \def\FF{{\NZQ F}} \def\GG{{\NZQ G}} \def\HH{{\NZQ H}} \def\frk{\frak} \def\aa{{\frk a}} \def\pp{{\frk p}} \def\Pp{{\frk P}} \def\qq{{\frk q}} \def\Qq{{\frk Q}} \def\mm{{\frk m}} \def\Mm{{\frk M}} \def\Phi{{\frk n}} \def\Phi{{\frk N}} \def\MA{{\mathcal A}} \def\MB{{\mathcal B}} \def\MI{{\mathcal I}} \def\MJ{{\mathcal J}} \def\MP{{\mathcal P}} \def\MQ{{\mathcal Q}} \def\MR{{\mathcal R}} \def\MV{{\mathcal V}} \def\ME{{\mathcal E}} \def\MK{{\mathcal K}} \def\MF{{\mathcal F}} \def\MT{{\mathcal T}} \def\MH{{\mathcal H}} \def\MS{{\mathcal S}} \def\MM{{\mathcal M}} \def\MY{{\mathcal Y}} \def\ab{{\bold a}} \def\bb{{\bold b}} \def\xb{{\bold x}} \def\yb{{\bold y}} \def\zb{{\bold z}} \def\gb{{\bold g}} \def\cb{{\bold c}} \def\db{{\bold d}} \def\fb{{\bold f}} \def\eb{{\bold e}} \def\opn#1#2{\def#1{\operatorname{#2}}} \opn\chara{char} \opn\length{\ell} \opn\pd{pd} \opn\rk{rk} \opn\projdim{proj\,dim} \opn\injdim{inj\,dim} \opn\rank{rank} \opn\depth{depth} \opn\grade{grade} \opn\height{height} \opn\embdim{emb\,dim} \opn\codim{codim} \def\OO{{\mathcal O}} \opn\Tr{Tr} \opn\bigrank{big\,rank} \opn\superheight{superheight}\opn\lcm{lcm} \opn\trdeg{tr\,deg}\opn\reg{reg} \opn\lreg{lreg} \opn\ini{in} \opn\lpd{lpd} \opn\size{size}\opn\bigsize{bigsize} \opn\cosize{cosize}\opn\bigcosize{bigcosize} \opn\sdepth{sdepth}\opn\sreg{sreg} \opn\link{link}\opn\fdepth{fdepth} \opn\index{index} \opn\index{index} \opn\indeg{indeg} \opn\N{N} \opn\mult{mult} \opn\SSC{SSC} \opn\SC{SC} \opn\lk{lk} \opn\HS{HS} \opn\div{div} \opn\Div{Div} \opn\cl{cl} \opn\Cl{Cl} \opn\Spec{Spec} \opn\Supp{Supp} \opn\supp{supp} \opn\Sing{Sing} \opn\Ass{Ass} \opn\Min{Min}\opn\Mon{Mon} \opn\dstab{dstab} \opn\astab{astab} \opn\Syz{Syz} \opn\reg{reg} \opn\Ann{Ann} \opn\Rad{Rad} \opn\Soc{Soc} \opn\Im{Im} \opn\Ker{Ker} \opn\Coker{Coker} \opn\Am{Am} \opn\Hom{Hom} \opn\Tor{Tor} \opn\Ext{Ext} \opn\End{End}\opn\Der{Der} \opn\Aut{Aut} \opn\id{id} \def\Frob{{\mathcal F}} \opn\nat{nat} \opn\pff{pf}\opn\Pf{Pf} \opn\GL{GL} \opn\SL{SL} \opn\mod{mod} \opn\ord{ord} \opn\Gin{Gin} \opn\Hilb{Hilb}\opn\sort{sort} \opn\initial{init} \opn\ende{end} \opn\height{height} \opn\type{type} \opn\aff{aff} \opn\con{conv} \opn\relint{relint} \opn\st{st} \opn\lk{lk} \opn\cn{cn} \opn\core{core} \opn\vol{vol} \opn\link{link} \opn\Link{Link}\opn\lex{lex} \opn\gr{gr} \def\Rees{{\mathcal R}} \def\poly#1#2#3{#1[#2_1,\dots,#2_{#3}]} \def\pot#1#2{#1[\kern-0.28ex[#2]\kern-0.28ex]} \def\Pot#1#2#3{\pot{#1}{#2_1,\dots,#2_{#3}}} \def\konv#1#2{#1\langle #2\rangle} \def\Konv#1#2#3{\konv{#1}{#2_1,\dots,#2_{#3}}} \opn\dirlim{\underrightarrow{\lim}} \opn\inivlim{\underleftarrow{\lim}} \let\union=\cup \let\sect=\cap \let\dirsum=\oplus \let\tensor=\otimes \let\iso=\cong \let\Union=\bigcup \let\Sect=\bigcap \let\Dirsum=\bigoplus \let\Tensor=\bigotimes \let\to=\rightarrow \let\To=\longrightarrow \def\Implies{\ifmmode\Longrightarrow \else } \def\implies{\ifmmode\Rightarrow \else } \def\iff{\ifmmode\Longleftrightarrow \else } \let\gets=\leftarrow \let\Gets=\longleftarrow \let\followsfrom=\Leftarrow \let\Followsfrom=\Longleftarrow \let\:=\colon \newtheorem{Theorem}{Theorem}[section] \newtheorem{Lemma}[Theorem]{Lemma} \newtheorem{Corollary}[Theorem]{Corollary} \newtheorem{Proposition}[Theorem]{Proposition} \newtheorem{Remark}[Theorem]{Remark} \newtheorem{Remarks}[Theorem]{Remarks} \newtheorem{Example}[Theorem]{Example} \newtheorem{Examples}[Theorem]{Examples} \newtheorem{Definition}[Theorem]{Definition} \newtheorem{Question}[Theorem]{Question} \newtheorem{Questions}[Theorem]{Questions} \newtheorem{Problem}[Theorem]{} \newtheorem{Conjecture}[Theorem]{Conjecture} \newtheorem{Observation}[Theorem]{Observation} \newtheorem{Construction}[Theorem]{Construction} \newtheorem{Setup}[Theorem]{Setup} \newtheorem{Notation}[Theorem]{Notation} \let\epsilon\varepsilon \let\kappa=\varkappa \textwidth=15cm \textheight=22cm \topmargin=0.5cm \oddsidemargin=0.5cm \evensidemargin=0.5cm \pagestyle{plain} } \def\textqed{\unskip\nobreak\penalty50 \hskip2em\hbox{}\nobreak\hfil\qedsymbol nalhyphendemerits=0} \def\dispqed{\rlap{\qquad\qedsymbol}} \def\noqed{\def\qed{\relax}} \opn\dis{dis} \def\pnt{{\raise0.5mm\hbox{\large\bf.}}} \def\lpnt{{\hbox{\large\bf.}}} \opn\Lex{Lex} \def\F{{\mathcal F}} \opn\Spec{Spec} \opn\Supp{Supp} \opn\supp{supp} \opn\Ass{Ass} \opn\p{Ass} \opn\min{min} \opn\max{max} \opn\MIN{Min} \opn\p{\mathfrak{p}} \opn\Deg{Deg} \def\Coh#1#2{H_{\mm}^{#1}(#2)} \def\hchst#1{for all $u\in G(I)$ there exists $i\notin {#1}$ such that $\nu_i(u) > a_i \geq 0$} \newcommand{\Shan}[1]{{\color{blue} \sf $\heartsuit$ [#1]}} \begin{document} \title{The Multiplicity of Powers of a Class of Non-Square-free Monomial Ideals} \author {Liuqing Yang, Zexin Wang*} \footnote{* Corresponding author} \address{School of Mathematical Science, Soochow University, 215006 Suzhou, P.R.China} \email{[email protected]} \address{School of Mathematical Science, Soochow University, 215006 Suzhou, P.R.China} \email{[email protected]} \begin{abstract} Let $R = \mathbb{K}[x_1, \ldots, x_n]$ be a polynomial ring over a field $\mathbb{K}$, and let $I \subseteq R$ be a monomial ideal of height $h$. We provide a formula for the multiplicity of the powers of $I$ when all the primary ideals of height $h$ in the irredundant reduced primary decomposition of $I$ are irreducible. This is a generalization of \cite[Theorem 1.1]{TV}. Furthermore, we present a formula for the multiplicity of powers of special powers of monomial ideals that satisfy the aforementioned conditions. Here, for an integer $m>0$, the $m$-th special power of a monomial ideal refers to the ideal generated by the $m$-th powers of all its minimal generators. Finally, we explicitly provide a formula for the multiplicity of powers of special powers of edge ideals of weighted oriented graphs. \end{abstract} \subjclass[2010]{13A02, 13H15, 05E40.} \keywords{multiplicity, powers of an ideal, special powers of an ideal, weighted oriented graph, edge ideal } \maketitle \section{Introduction} Let $R=\mathbb{K}[x_1,\ldots,x_n]$ be a standardly graded polynomial ring over a field $\mathbb{K}$, and let $M$ be a finitely generated graded $R$-module. We denote by $M_k$ the degree $k$ component of $M$. The {\it Hilbert function} $H_M(k)$ of $M$ is a function from $\mathbb{Z}$ to $\mathbb{N}$ given by $H_M(k):=\dim_{\mathbb{K}}M_k$ for all $k\in \mathbb{ Z}$. The {\it Hilbert series} $\HS(M,t)$ of $M$ is defined to be the formal power series: $$\HS(M,t):=\sum_{k\in \mathbb{Z}}H_M(k)t^k.$$ Assuming $\dim M=d+1$, David Hilbert showed that that $\HS(M,t)$ is a rational function of the following form $$\HS(M,t)=\frac{Q_M(t)}{(1-t)^{d+1}}.$$ Here, $Q_M(t)\in \mathbb{Q}[t,t^{-1}]$ is a Laurent polynomial such that $Q_M(1)\neq 0$. As a consequence, there exists a polynomial $p_M(t)\in \mathbb{Q}[t]$ of degree $d$ such that $H_M(k)=p_M(k)$ for all $k\gg 0$. The polynomial $p_M(t)$ is referred to as the {\it Hilbert polynomial} of $M$. \begin{Definition} \em Let $M$ be a finitely generated graded $R$-module of dimension $d+1$. The Hilbert polynomial $p_M(t)$ of $M$ can be written as $$p_M(t)=\sum_{i=0}^d(-1)^ie_i(M)\binom{t+d-i}{d-i}.$$ The integer coefficients $e_i(M)$ for $i=0,\ldots,d$ are called the {\it Hilbert coefficients} of $M$. \end{Definition} According to \cite[Proposition 4.1.9]{BH}, we have $e_i(M)=\frac{Q_M^{(i)}(1)}{i!}$ for $i=0,\ldots,d$. The first Hilbert coefficients $e_0(M)$ is also called the {\it multiplicity} of $M$ and denoted by $\mult(M)$. The multiplicity of a graded ideal is a significant invariant in algebraic geometry and commutative algebra. For the study of the multiplicity of graded ideals, we refer to \cite{A1,A2}, \cite{HS1,HS2}, \cite{TY1,TY2,TY3} and the references therein. Let $I$ be a graded ideal with $\dim R/I=d$. Herzog-Puthenpurakal-Verma showed in \cite[Theorem 1.1]{HPV08} that {\it $e_i(R/I^s)$ is of polynomial type in $s$ of degree $\leq n-d+i$ for $i=0,1,\ldots,d$}. Recall that a function $f:\mathbb{N}\rightarrow \mathbb{Q}$ is of {\it polynomial type} of degree $d$ if there exists a polynomial $p(t)\in \mathbb{Q}[t]$ of degree $d$ such that $f(k)=p(k)$ for all $k\gg 0$. In particular, $\mult(R/I^s)$ is of polynomial type in $s$. Naturally, the question arises: Is it possible to explicitly compute the multiplicity of powers of graded ideals in certain instances? To the best of our knowledge, the first instance of a non-trivial graded ideal, for which the multiplicity of its powers is explicitly provided, is the path ideal of a line graph, as computed previously in \cite{SWL}, where it was proved that if $I$ is the path ideal of a line graph, then the formula $\mult(R/I^s) = \mult(R/I) \binom{\height(I)+s-1}{s-1}$ holds. Recently, Thuy and Vu extended this formula to encompass arbitrary square-free monomial ideals in their work \cite{TV}. Building upon their findings, we further extend \cite[Theorem 1.1 and Lemma 2.1]{TV} to compute the multiplicity of powers for a specific class of non-square-free monomial ideals, notably including the edge ideals of weighted oriented graphs and edge-weighted graphs. \begin{Definition}\em Let $I$ be a graded ideal of $R$ with a primary decomposition given by \[ I = Q_1 \cap Q_2 \cdots \cap Q_t. \] We refer to this decomposition as \emph{reduced} if the radicals $\sqrt{Q_i}$ are pairwise distinct for all $i = 1, \ldots, t$. Furthermore, we refer to it as \emph{irredundant} if for any $1 \leq i \leq t$, the ideal $Q_i$ is not a superset of the intersection of the other primary ideals, i.e., $Q_i \nsupseteq \bigcap_{j \neq i} Q_j$. \end{Definition} \begin{Theorem}{\em (Theorem~\ref{Main result})} Let $I$ be a monomial ideal of $R$ with height $h$. Suppose $I$ admits an irredundant reduced primary decomposition \[ I = Q_1 \cap \cdots \cap Q_r \cap Q_{r+1} \cap \cdots \cap Q_t, \] where $\height(Q_i) = h$ for $i = 1,\ldots,r$ and $\height(Q_i) > h$ for $i = r+1,\ldots,t$. Then following statements hold. \textup{(1)} For every integer $s \geq 1$, \[ \mult(R/I^s) = \sum_{i=1}^r \mult(R/Q_i^s). \] \textup{(2)} If each $Q_i$ ($1 \leq i \leq r$) is an irreducible monomial ideal generated by pure powers of variables with exponents $a_{i_1},\ldots,a_{i_h}$, then for any $s \geq 1$, \[ \mult(R/I^s) = \mult(R/I) \binom{h+s-1}{s-1} = \sum_{i=1}^r \left( \prod_{j=1}^h a_{i_j} \right) \binom{h+s-1}{s-1}. \] \end{Theorem} We remark that \cite[Lemma 2.1]{TV} is a special case of formula (\dag) when $s=1$, and \cite[Theorem 1.1]{TV} is a special case of formula (\ddag) when $I$ is a square-free monomial ideal. Let $I$ be a monomial ideal. The ideal generated by the $m$-th powers of all its minimal generators is called the $m$-th \emph{special power} of $I$, denoted as $I^{\{m\}}$. We have proved the following theorem: \begin{Theorem} {\em (Theorem~\ref{main2})} If $I$ satisfies the hypotheses of Theorem~\ref{Main result}(2), then $I^{\{m\}}$ also satisfies them for all integers $m \geq 1$. Furthermore, let $\height(I)=h$, then for all $m,s \geq 1$, \[ \mult(R/(I^{\{m\}})^s) = m^h \begin{pmatrix}h+s-1\\ s-1\end{pmatrix} \mult(R/I). \] \end{Theorem} We provide some notations and definitions that will be used throughout this paper. \begin{Notation}\label{graph} \em Let $G = (V(G), E(G))$ be a simple graph (without loops or multiple edges) with vertices $V(G) = \{x_1, \ldots, x_n\}$ and edge set $E(G)$. By identifying the variables of the polynomial ring $R = \mathbb{K}[x_1, \ldots, x_n]$ with the vertices of $V(G)$, we can associate to $G$ a square-free monomial ideal $I(G) = (\{x_ix_j \mid \{x_i, x_j\} \in E(G)\})$, called the edge ideal of $G$. \end{Notation} \begin{Definition}\label{cover} \em For a vertex $x_i\in V(G)$, the {\it neighbor set} of $x_i$ is defined to be the set $N_G(x_i) = \{x_j |\ \{x_i,x_j\}\in E(G)\} $. A {\it vertex cover} of $G$ is a subset $C \subseteq V(G)$ such that for each edge $\{x_i, x_j\}$ in $G$, either $x_i \in C$ or $x_j \in C$. A vertex cover is {\it minimal} if it does not properly contain another vertex cover of $G$. The minimum number of vertices in a minimal vertex cover of $G$ is called the {\it vertex covering number} of $G$, denoted as $\alpha(G)$. Let $r(G)$ denote the number of minimal vertex covers of $G$ that contain exactly $\alpha(G)$ vertices. \end{Definition} \begin{Definition}\label{oriented graph} \em A {\it weighted oriented graph} $D$, whose underlying graph is $G$, is a triplet $(V(D), E(D), w)$ where $V(D) = V(G)$, $E(D) \subseteq V(D)\times V(D)$ such that $\{\{ x_i, x_j \} | (x_i, x_j) \in E(D)\} = E(G)$, and $w$ is a function $w: V(D) \rightarrow \mathbb{N}$. The vertex set of $D$ and the edge set of $D$ are $V(D)$ and $E(D)$, respectively. Sometimes, for brevity, we denote these sets by $V$ and $E$ respectively. The {\it weight} of $x_i \in V$ is $w(x_i)$. \end{Definition} \begin{Definition}\label{L} \em The edge ideal of a weighted oriented graph $D$ is a monomial ideal given by \[ I(D) = (x_i x_j^{w(x_j)} \mid (x_i, x_j) \in E(D)) \subseteq R. \] \end{Definition} Edge ideals of weighted oriented graph arose in the theory of Reed-Muller codes as initial ideals of vanishing ideals of projective spaces over finite fields (see \cite{MPV}, \cite{PS}). In recent years, its algebraic properties have been studied by many researchers. Relevant research can be referred to in \cite{CK}, \cite{HLMRV}, \cite{MP}, \cite{WZXZ} and \cite{ZXWT}, etc. We provide a formula for the multiplicity of powers of special powers of the edge ideal of any weighted oriented graph using combinatorial properties. \begin{Definition} \em Let $D = (V, E, w)$ be a vertex-weighted oriented graph and $G$ its underlying graph. For a vertex cover $C$ of $G$, define \begin{align*} &L_1(C) = \{x_i \in C\ | \ \exists \ x_j \text{ such that } (x_i,x_j) \in E \text{ and } x_j \notin C\}, \\ &L_3(C) = \{x_i \in C\ | \ N_G(x_i) \subseteq C\}, \\ &L_2(C) = C \setminus (L_1(C) \cup L_3(C)). \end{align*} \end{Definition} \begin{Theorem}{\em (Theorem~\ref{main3})} \em Let $D = (V, E, w)$ be a vertex-weighted oriented graph and $G$ its underlying graph. Let $C_1, \ldots, C_{r(G)}$ be all minimal vertex covers of graph $G$ that contain exactly $\alpha(G)$ vertices. Then, for all $m, s \geq 1$, $$\mult(R/(I(D)^{\{m\}})^s) = m^{\alpha(G)} \sum_{i=1}^{r(G)} \left( \prod_{x_j\in L_2(C_i)} w(x_j)\right) \binom{\alpha(G)+s-1}{s-1} .$$ \end{Theorem} When $m=1$, this formula reduces to the multiplicity formula for powers of the edge ideal of a weighted oriented graph. The paper is structured as follows: Section 2 provides an explicit formula for the multiplicity of the powers of $I$ when all the primary ideals of height $h$ in the irredundant reduced primary decomposition of $I$ are irreducible. Then, we introduce the concept of special powers, and further derive a formula for the multiplicity of the powers of these special powers for such ideals. Section 3 provides a formula for the multiplicity of powers of special powers of the edge ideal of any weighted oriented graph using combinatorial properties. \section{Multiplicity} In this section, we will present a formula for computing the multiplicity of the powers of a class of non-square-free monomial ideals. Then, we introduce the concept of special powers, and further derive a formula for the multiplicity of the powers of these special powers for such ideals. Throughout this paper, the polynomial ring $\mathbb{K}[x_1, \ldots, x_n]$ will be uniformly denoted by $R$. \begin{Definition}\em Let $I$ be a graded ideal of $R$ with a primary decomposition given by \[ I = Q_1 \cap Q_2 \cdots \cap Q_t. \] We refer to this decomposition as \emph{reduced} if the radicals $\sqrt{Q_i}$ are pairwise distinct for all $i = 1, \ldots, t$. Furthermore, we refer to it as \emph{irredundant} if for any $1 \leq i \leq t$, the ideal $Q_i$ is not a superset of the intersection of the other primary ideals, i.e., $Q_i \nsupseteq \bigcap_{j \neq i} Q_j$. \end{Definition} \begin{Theorem}\label{Main result} Let $I$ be a monomial ideal of $R$ with height $h$. Suppose $I$ admits an irredundant reduced primary decomposition \[ I = Q_1 \cap \cdots \cap Q_r \cap Q_{r+1} \cap \cdots \cap Q_t, \] where $\height(Q_i) = h$ for $i = 1,\ldots,r$ and $\height(Q_i) > h$ for $i = r+1,\ldots,t$. Then following statements hold. \textup{(1)} For every integer $s \geq 1$, we have \[ \mult(R/I^s) = \sum_{i=1}^r \mult(R/Q_i^s). \] \textup{(2)} If each $Q_i$ ($1 \leq i \leq r$) is an irreducible monomial ideal generated by pure powers of variables with exponents $a_{i_1},\ldots,a_{i_h}$, then for any $s \geq 1$, we have \[\tag{\dag} \mult(R/I^s) = \mult(R/I) \binom{h+s-1}{s-1} = \sum_{i=1}^r \left( \prod_{j=1}^h a_{i_j} \right) \binom{h+s-1}{s-1}. \] \end{Theorem} We first prove that formula $(\dag)$ holds for irreducible monomial ideals. According to \cite[Corollary 1.3.2]{HH}, a monomial ideal is irreducible if and only if it is generated by pure powers of variables. As usual, we use $G(I)$ to denote the minimal generating set for a monomial ideal $I$. \begin{Notation}\label{Notation} Without loss of generality, let $I_m = (x_1^{a_1}, x_2^{a_2}, \ldots, x_m^{a_m})$ be an irreducible monomial ideal of $R$, where $1 \leq m \leq n$ and $a_i$ are positive integers for all $i$. When $m > 1$, denote $I_{m-1} = (x_1^{a_1}, x_2^{a_2}, \ldots, x_{m-1}^{a_{m-1}})$. \end{Notation} \begin{Lemma} \label{colon} Following Notation \ref{Notation}, for every integer $s\geq 2$, $$I_m^s:x_m^{a_m}=I_m^{s-1}.$$ \end{Lemma} \begin{proof} Because $x_m^{a_m} \in G(I_m)$, the relation ``$\supseteq$'' is obvious. We only need to prove the reverse inclusion ``$\subseteq$'' holds. Let $u \in I_m^s:x_m^{a_m}$ be a monomial, then $ux_m^{a_m} \in I_m^s$. This means there exists $v \in G(I_m^s)$ such that $v | ux_m^{a_m}$. Note that we may write $v$ as $x_1^{k_1a_1}x_2^{k_2a_2}\cdots x_m^{k_ma_m}$, where $k_1+\cdots+k_m=s$ with $k_i\geq 0$ for $i=1,\ldots,m$. If $k_m=0$, then $v | u$, and thus $u \in I_m^s \subseteq I_m^{s-1}$. If $k_m\geq 1$, then, since $\frac{v}{x_m^{a_m}}=x_1^{k_1a_1}x_2^{k_2a_2}\cdots x_m^{(k_m-1)a_m}$, we conclude that $\frac{v}{x_m^{a_m}}$ belongs to $I_m^{s-1}$. It follows that $u$ belongs to $I_m^{s-1}$ since $\frac{v}{x_m^{a_m}} \mid u$, as required. \end{proof} \begin{Lemma} \label{irreducible} Following Notation \ref{Notation}, for every integer $s\geq 1$, $$\mult (R/I_m^s)=\mult (R/I_m)\begin{pmatrix}m+s-1\\s-1\end{pmatrix}=a_1\ldots a_m\begin{pmatrix}m+s-1\\s-1\end{pmatrix}.$$ \end{Lemma} \begin{proof} Since $I_m$ is a monomial ideal generated by a regular sequence of monomials, according to \cite[Exercise~16.9]{P}, we have $\mult(R/I_m) = a_1\ldots a_m$. Therefore, it suffices to prove that the first term is equal to the third term in the equality. We proceed by induction on both $s$ and $m$. The case when $s=1$ or $m=1$ follows from \cite[Exercise~16.9]{P}. Suppose now that $s>1$ and $m>1$. Consider the following two short exact sequences of graded $R$-modules: \begin{equation} 0 \To\frac{R}{I_m^s:x_m^{a_m}}[-a_m] \To\frac{R}{I_m^s} \To\frac{R}{(I_m^s,x_m^{a_m})} \To 0, \end{equation} \begin{equation} 0 \To\frac{R}{I_{m-1}^{s}:x_m^{a_m}}[-a_m] \To\frac{R}{I_{m-1}^{s}} \To\frac{R}{(I_{m-1}^{s},x_m^{a_m})} \To 0. \end{equation} Firstly, by Lemma~\ref{colon}, we have $\sqrt{I_m^s:x_m^{a_m}} = \sqrt{I_m^{s-1}} = (x_1, \ldots, x_m)$. Therefore, \[ \dim \frac{R}{I_m^s:x_m^{a_m}} = \dim \frac{R}{I_m^{s-1}} = n - m. \] Note that $(I_m^s,x_m^{a_m})=(I_{m-1}^s,x_m^{a_m})$ and $\sqrt{(I_{m-1}^s,x_m^{a_m})}=(x_1,\ldots,x_m)$, we obtain $$\dim \frac{R}{(I_m^s,x_m^{a_m})}=\dim \frac{R}{(I_{m-1}^s,x_m^{a_m})}=n-m.$$ Applying \cite[Lemma 3.9]{SWL} to the exact sequence (1), we obtain \[ \mult(R/I_m^s) = \mult(R/I_m^{s-1}) + \mult(R/(I_{m-1}^s, x_m^{a_m})). \] Next, consider the second short exact sequence, where $I_{m-1}^{s}:x_m^{a_m}=I_{m-1}^{s}$ holds trivially, leading to the following equality: \[ \HS(R/(I_{m-1}^{s},x_m^{a_m}),t) =(1-t^{a_m})\HS(R/I_{m-1}^s,t)=\frac{(1+t+\cdots+t^{a_m-1})Q(t)}{(1-t)^{d-1}}. \] Here, we assume that $ \dim \frac{R}{I_{m-1}^s}=d$ and $\HS(R/I_{m-1}^s,t)=\frac{Q(t)}{(1-t)^d}$. Since $\sqrt{I_{m-1}^s} = (x_1, \ldots, x_{m-1})$ and $\sqrt{(I_{m-1}^{s}, x_m^{a_m})} = (x_1, \ldots, x_m)$, we obtain \[ \dim \frac{R}{(I_{m-1}^s, x_m^{a_m})} = \dim \frac{R}{I_{m-1}^s} - 1=d-1. \] It follows \cite[Proposition 4.1.9]{BH} that \[ \mult(R/(I_{m-1}^s, x_m^{a_m})) = a_m \mult(R/I_{m-1}^s). \] Combining the above equality with the induction hypothesis, we can deduce that: \begin{align*} & \mult (R/I_m^s)= \mult (R/I_m^{s-1})+a_m\mult (R/I_{m-1}^s) \\ =&a_1\ldots a_m\begin{pmatrix}m+s-2\\ s-2\end{pmatrix} +a_m a_1\ldots a_{m-1}\begin{pmatrix}m+s-2\\s-1\end{pmatrix}\\ =&a_1\ldots a_m\begin{pmatrix}m+s-1\\s-1\end{pmatrix}. \end{align*} \end{proof} \begin{proof}[Proof of Theorem \ref{Main result}] Suppose that $I^s$ admits an irredundant reduced primary decomposition \[ I^s = Q_1' \cap \cdots \cap Q_r' \cap Q_{r+1}' \cap \cdots \cap Q_k', \] such that $\sqrt{Q_i'}$ are pairwise distinct for $i=1,\ldots,k$. Assume further that $\sqrt{Q_i'} = \sqrt{Q_i}$ for $i = 1, \ldots, r$. Since $\sqrt{Q_1'}, \ldots, \sqrt{Q_r'}$ are all minimal prime ideals of $I$, it is easy to see that \[ Q_i' = Q_i^s, \text{ for any } 1 \leq i \leq r. \] One may also look at the proof of \cite[Lemma 2]{GMSVV} for this observation. So, applying \cite[Lemma 2.1]{TV} directly yields the first assertion. The second assertion follows from Theorem \ref{Main result} (1) and Lemma~\ref{irreducible}. \end{proof} Using the concept of special powers of monomial ideals as defined below, we can construct numerous monomial ideals satisfying the hypotheses of Theorem~\ref{Main result}(2). \begin{Definition} \em Let $I$ be a monomial ideal with $G(I)=\{u_1,\ldots,u_t\}$. For any $m \geq 1$, define the $m$-th {\it special power} of $I$ as \[ I^{\{m\}} = (u_1^m, \ldots, u_t^m). \] \end{Definition} We collect some easy facts regarding the special power. \begin{Lemma}\label{special power basic properties} Let $I_1, I_2, \ldots, I_t$ be monomial ideals, then for $m \geq 1$, we have: \begin{enumerate} \item $(I_1\cap I_2\cap \cdots \cap I_t)^{\{m\}}=I_1^{\{m\}}\cap I_2^{\{m\}}\cap \cdots \cap I_t^{\{m\}}$; \item $(I_1I_2\cdots I_t)^{\{m\}}=I_1^{\{m\}}I_2^{\{m\}}\cdots I_t^{\{m\}}$; \item $(I_1+I_2+\cdots +I_t)^{\{m\}}=I_1^{\{m\}}+I_2^{\{m\}}+\cdots +I_t^{\{m\}}$; \item $I$ is an irreducible monomial ideal if and only if so is $I^{\{m\}}$; \item $I$ is a primary monomial ideal if and only if so is $I^{\{m\}}$. \end{enumerate} \end{Lemma} \begin{proof} (1) By induction, we only consider the case $t=2$. For any $u \in G(I_1 \cap I_2)$, it holds that $u \in I_1,I_2$, and thus $u^m \in I_1^{\{m\}}, I_2^{\{m\}}$ for all $m$. Since all $u^m$ generate $(I_1 \cap I_2)^{\{m\}}$, we have $(I_1 \cap I_2)^{\{m\}} \subseteq I_j^{\{m\}}$ for $j=1,2$, which further implies $(I_1 \cap I_2)^{\{m\}} \subseteq I_1^{\{m\}}\cap I_2^{\{m\}}$. Next, we prove the reverse inclusion. For any $u \in G(I_1)$ and any $v \in G(I_2)$, we have $\text{lcm}(u^m, v^m) = (\text{lcm}(u, v))^m \in (I_1 \cap I_2)^{\{m\}}$ for all $m$. And since all $\text{lcm}(u^m, v^m)$ generate $I_1^{\{m\}} \cap I_2^{\{m\}}$, the reverse inclusion also holds. The proofs of (2) and (3) is similar as (1) and we omit it. (4) The conclusion follows immediately from the generating structure of irreducible monomial ideals by pure powers of variables. (5) A monomial is a primary ideal if and only if it is the intersection of irreducible monomial ideals with the same support. In view of this fact, the assertion follows from (1) together with (4). \end{proof} \begin{Lemma}\label{special power property} If a monomial ideal $I$ admits an irredundant reduced primary decomposition $I = \bigcap_{i=1}^t Q_i$, then \[ I^{\{m\}} = \bigcap_{i=1}^t Q_i^{\{m\}} \] is an irredundant reduced primary decomposition of $I^{\{m\}}$. \end{Lemma} \begin{proof} From Lemma~\ref{special power basic properties}(1) and (5), we can deduce that \[\tag{\S} I^{\{m\}} = \bigcap_{i=1}^t Q_i^{\{m\}} \] is a primary decomposition of $I^{\{m\}}$. Furthermore, since $\sqrt{Q_i} = \sqrt{Q_i^{\{m\}}}$, and given that $I = \bigcap_{i=1}^t Q_i$ is a reduced primary decomposition, it can be concluded that $(\S)$ is also a reduced primary decomposition. We only need to prove that this decomposition is irredundant. If there exists an $i$ such that $Q_i^{\{m\}} \supseteq \bigcap_{j \neq i} Q_j^{\{m\}}$, then by Lemma~\ref{special power basic properties}(1), we have $Q_i^{\{m\}} \supseteq \left( \bigcap_{j \neq i} Q_j \right)^{\{m\}}$. For any $u \in G\left( \bigcap_{j \neq i} Q_j \right)$, there exists a $v \in G(Q_i)$ such that $v^m | u^m$, thereby $v | u$. Therefore, $Q_i \supseteq \bigcap_{j \neq i} Q_j$, which contradicts the fact that $I = \bigcap_{i=1}^t Q_i$ is irredundant. This completes the proof. \end{proof} From Theorem \ref{Main result}, we can derive the following results. \begin{Theorem}\label{main2} If $I$ satisfies the hypotheses of Theorem~\ref{Main result}(2), then $I^{\{m\}}$ also satisfies them for all integers $m \geq 1$. Furthermore, let $\height(I)=h$, then for all $m,s \geq 1$, \[ \mult(R/(I^{\{m\}})^s) = m^h \begin{pmatrix}h+s-1\\ s-1\end{pmatrix} \mult(R/I). \] \end{Theorem} \begin{proof} The first assertion directly follows from Lemmas \ref{special power property} and \ref{special power basic properties}(4). For the second assertion, by Theorem~\ref{Main result}(2), it suffices to prove that \[ \mult(R/I^{\{m\}}) = m^h \mult(R/I). \] If the irreducible monomial ideal $Q$ is generated by pure powers of variables with degrees $a_1, \ldots, a_h$, then $Q^{\{m\}}$ is generated by pure powers of variables with degrees $ma_1, \ldots, ma_h$. Additionally, according to \cite[Exercise~16.9]{P}, we have $$\operatorname{mult}(R/Q^{\{m\}}) = m^h a_1 \ldots a_h=m^h \cdot \operatorname{mult}(R/Q).$$ Thus, combining Theorem \ref{Main result}(2) and Lemma \ref{special power property}, the conclusion is obvious. \end{proof} Let $I$ be a square-free monomial ideal in $R$. According to \cite[Corollary~6.2.3]{HH}, the multiplicity $\text{mult}(R/I)$ is the count of associated prime ideals of $I$ having the minimal height. Furthermore, since a square-free monomial ideal can be decomposed as the intersection of monomial prime ideals, by applying Theorem \ref{main2}, we can derive the following corollary. \begin{Corollary} \label{special power square-free} \em Let $I$ be a square-free monomial ideal of height $h$ in $R$, and let $r$ be the number of height-$h$ associated primes of $R/I$. Then, for all $m, s \geq 1$, $$\mult(R/(I^{\{m\}})^s) = r m^h\begin{pmatrix}h+s-1\\ s-1\end{pmatrix}.$$ \end{Corollary} \begin{Remark}\em When $m=1$, the above corollary reduces to \cite[Theorem 1.1]{TV}. \end{Remark} In the conclusions below, we use the notation from Notation \ref{graph} and Definition \ref{cover}. \begin{Corollary}\label{special power graph} \em Let $I(G)$ be the edge ideal of the graph $G$. Then, for all $m, s \geq 1$, $$\mult(R/(I(G)^{\{m\}})^s) = r(G) m^{\alpha(G)}\begin{pmatrix}\alpha(G)+s-1\\ s-1\end{pmatrix}.$$ \end{Corollary} \begin{proof} We know that $\height(I(G))=\alpha(G)$ and $\mult(R/I(G))= r(G)$, so the conclusion is a direct corollary of Corollary \ref{special power square-free}. \end{proof} \begin{Remark}\em The $m$-th special power $I(G)^{\{m\}}$ of $I(G)$ is precisely the edge ideal of trivially edge-weighted graphs $G_w$ where each edge has a weight of $m$, as defined in \cite{PS}. \end{Remark} \section{weighted oriented graph} To compute multiplicities of powers for special powers in edge ideals of weighted oriented graphs, we employ Theorem~\ref{Main result} in this section. In this section, we continue to use Notation \ref{graph}, Definition \ref{cover} and Definition \ref{oriented graph}. Recall a weighted oriented graph $D$, whose underlying graph is $G$, is a triplet $(V(D), E(D), w)$ where $V(D) = V(G)$, $E(D) \subseteq V(D)\times V(D)$ such that $\{\{ x_i, x_j \} | (x_i, x_j) \in E(D)\} = E(G)$, and $w$ is a function $w: V(D) \rightarrow \mathbb{N}$. The vertex set of $D$ and the edge set of $D$ are $V(D)$ and $E(D)$, respectively. The edge ideal of a weighted oriented graph $D$ is a monomial ideal given by $I(D) = (x_i x_j^{w(x_j)} \mid (x_i, x_j) \in E(D)) \subseteq R$. For reference, we repeat Definition \ref{L} as follows: \begin{Definition}\em Let $D = (V, E, w)$ be a weighted oriented graph and $G$ its underlying graph. For a vertex cover $C$ of $G$, define \begin{align*} &L_1(C) = \{x_i \in C\ | \ \exists \ x_j \text{ such that } \{x_i,x_j\} \in E \text{ and } x_j \notin C\}, \\ &L_3(C) = \{x_i \in C\ | \ N_G(x_i) \subseteq C\}, \\ &L_2(C) = C \setminus (L_1(C) \cup L_3(C)). \end{align*} \end{Definition} A vertex cover $C$ of $G$ is called a {\it strong vertex cover} of $D$ if $C$ is a minimal vertex cover of $G$ or for each $x_i \in L_3(C)$ there is an edge $(x_j,x_i) \in E$ such that $x_j \in L_2(C) \cup L_3(C)$ with $w(x_j) \geq 2$. \begin{Theorem}\label{main3} \em Let $D = (V, E, w)$ be a weighted oriented graph and $G$ its underlying graph. Let $C_1, \ldots, C_{r(G)}$ be all minimal vertex covers of graph $G$ that contain exactly $\alpha(G)$ vertices. Then, for all $m, s \geq 1$, $$\mult(R/(I(D)^{\{m\}})^s) = m^{\alpha(G)} \sum_{i=1}^{r(G)} \left( \prod_{x_j\in L_2(C_i)} w(x_j)\right) \binom{\alpha(G)+s-1}{s-1} .$$ \end{Theorem} \begin{proof} According to \cite[Remark 26]{PRT}, if $\mathcal{C}_{s}$ denotes the set of strong vertex covers of $D$, then the irredundant reduced primary decomposition of the ideal $I(D)$ is expressed as \[ I(D) = \bigcap_{C \in \mathcal{C}_{s}} I_{C}, \] where \[ I_{C} = \left( L_1(C) \cup \{x_j^{w(x_j)} \mid x_j \in L_2(C) \cup L_3(C)\} \right). \] Therefore, $I(D)$ satisfies the hypotheses of Theorem~\ref{Main result}(2). Using Theorem \ref{main2}, we only need to prove that \[ \mult(R/I(D)) = \sum_{i=1}^{r(G)} \left( \prod_{x_j \in L_2(C_i)} w(x_j) \right). \] According to \cite[Proposition 6]{PRT}, if $C$ is a minimal vertex cover of $D$, then $L_{3}(C) = \emptyset$. Therefore, for any $1 \leq i \leq r(G)$, we have $\mult(R/I_{C_i}) = \prod_{x_j \in L_2(C_i)} w(x_j)$. By Theorem~\ref{Main result}(1), the above equality follows as required. \end{proof} \begin{Remark} \em From this paper, we know that $\mult(R/I^s) = \mult(R/I) \binom{\height(I)+s-1}{s-1}$ holds for many monomial ideals $I$. Naturally, one would ask whether this rule holds for all monomial ideals. The answer is no, as we will see in the following example. \end{Remark} \begin{Example} \em Let $I = (x_1^2, x_2^2, x_3^4) \cap (x_1^3, x_2^3, x_3^2) = (x_2^3, x_1^3, x_2^2x_3^2, x_1^2x_3^2, x_3^4)$ be a non-irreducible primary monomial ideal. We have $\height(I) = 3$. By utilizing CoCoA \cite{CoCoA}, we obtain $\mult(R/I) = 26$. Furthermore, \[ \mult(R/I^2) = 112 \neq 26\binom{3+2-1}{2-1} = 104; \] \[ \mult(R/I^3) = 294 \neq 26\binom{3+3-1}{3-1} = 260; \] \[ \mult(R/I^4) = 608 \neq 26\binom{3+4-1}{4-1} = 520. \] \end{Example} \vspace{2mm} {\bf \noindent Acknowledgment:} The first author acknowledges the assistance of Jiawei Bao in computations. Many of the computations related to this project were done using CoCoA \cite{CoCoA}. \vspace{2mm} {\bf\noindent Statement:} On behalf of all authors, the corresponding author states that there is no conflict of interest. \begin{thebibliography}{9999} \bibitem{A1} G. Alesandroni, {\it Monomial multiplicities in explicit form}, Journal of Algebra and Its Applications, Vol. 19, No. 09 (2020) 2050181. \bibitem{A2} G. Alesandroni, {\it Monomial invariants applied to graph coloring}, Journal of Algebraic Combinatorics (2023) 58:95-112. \bibitem{BH} W. Bruns, J. Herzog: \textit{Cohen-Macaulay rings}, Revised Edition. Cambridge (1996) \bibitem{CK} B. Casiday, S. Kara, {\it Betti numbers of weighted oriented graphs}, Electron. J. Combin., 28(2) (2021), P2.33. \bibitem{CoCoA} CoCoATeam, {\it CoCoA: a system for doing Computations in Commutative Algebra.} Available at {\tt http://cocoa.dima.unige.it} \bibitem{GMSVV} P. Gimenez, J. M. Bernal, A. Simis, R. H. Villarreal, C. E. Vivares, {\it Symbolic powers of monomial ideals and Cohen-Macaulay vertex-weighted digraphs}, Singularities, Algebraic Geometry, Commutative Algebra, and Related Topics, pp. 491-510. Springer, Cham (2018) \bibitem{HH} J. Herzog, T. Hibi, {\it Monomial Ideals}, Graduate Texts in Mathematics, vol. 260. Springer, 2011.\par \bibitem{HPV08} J. Herzog, T. J. Puthenpurakal and J. K. Verma, {\it Hilbert polynomials and powers of ideals.} Math. Proc. Cambridge Philos. Soc. \textbf{145}(2008), 623--642.\par \bibitem{HLMRV} H. T. H\`{a}, Kn. Lin, S. Morey, E. Reyes, R. H. Villarreal, {\it Edge ideals of oriented graphs}, International Journal of Algebra and Computation, 29(3)(2019),535-559. \bibitem{HS1} J. Herzog, H. Srinivasan, {\it Bounds for multiplicities}, Trans. Amer. Math. Soc. 350 (1998) 2879-2902. \bibitem{HS2} J. Herzog, H. Srinivasan, {\it Multiplicities of monomial ideals}, Journal of Algebra 274 (2004) 230-244. \bibitem{MP} M. Mandal, D.K. Pradhan, {\it Regularity in weighted oriented graphs}, Indian J. Pure App Math 52, 1055-1071 (2021). \bibitem{MPV} J. Mart\'{i}nez-Bernal, Y. Pitones, R. H. Villarreal, {\it Minimum distance functions of graded ideals and Reed-Muller-type codes}, J. Pure Appl. Algebra, 221 (2017), 251-275. \bibitem{P} I. Peeva, {\it Graded syzygies}, Algebra and Applications, vol. 14, Springer-Verlag London, Ltd., London~(2011). \bibitem{PRT} Y. Pitones, E. Reyes, J. Toledo, {\it Monomial ideals of weighted oriented graphs}, Electron. J. Combin., 26(3) (2019), 1-18. \bibitem{PS} C. Paulsen, S. Sather-Wagstaff, {\it Edge ideals of weighted graphs}, J. Algebra Appl., 12 (5) (2013), 1250223-1-24. \bibitem{SWL} J. Shan, Z. Wang, D. Lu, {\it Multiplicity of powers of path ideals of a line graph}, Bull. Malays. Math. Sci. Soc. 48, 19 (2025). \bibitem{TV} P. T. Thuy, T. Vu, {\it Multiplicity of powers of squarefree monomial ideals}, arXiv:2411.01287. \bibitem{TY1} N. Terai, Ki. Yoshida, {\it Stanley-Reisner rings with large multiplicities are Cohen-Macaulay}, Journal of Algebra 301 (2006) 493-508. \bibitem{TY2} N. Terai, Ki. Yoshida, {\it Buchsbaum Stanley-Reisner rings with minimal multiplicity}, Proceedings of the American Mathematical Society 134, no. 1 (2006): 55-65. \bibitem{TY3} N. Terai, Ki. Yoshida, {\it Multiplicity and Castelnuovo-Mumford regularity of Stanley-Reisner rings}, Acta Math Vietnam 40, 61-69 (2015). \bibitem{WZXZ} H. Wang, G. Zhu, L. Xu, J. Zhang, {\it Algebraic properties of edge ideals of some vertex-weighted oriented cyclic graphs}, Algebra Colloquium 28:2 (2021) 253-268. \bibitem{V} R. H. Villarreal, {\it Monomial algebras}, 2nd ed., Monographs and Research Notes in Mathematics, CRC Press, Boca Raton, FL~(2015). \bibitem{ZXWT} G. Zhu, L. Xu, H. Wang, Z. Tang, {\it Projective dimension and regularity of edge ideal of some weighted oriented graphs}, Rocky Mountain J. Math. 49 (4) (2019) 1391-1406. \end{thebibliography} \end{document}
2412.06123v1
http://arxiv.org/abs/2412.06123v1
An Upper Bound on the Length of an Algebra and Its Application to the Group Algebra of the Dihedral Group
\documentclass{baustms} \citesort \theoremstyle{cupthm} \newtheorem{Theorem}{Theorem}[section] \newtheorem{Proposition}[Theorem]{Proposition} \newtheorem{Corollary}[Theorem]{Corollary} \newtheorem{Lemma}[Theorem]{Lemma} \theoremstyle{cupdefn} \newtheorem{Definition}[Theorem]{Definition} \theoremstyle{cuprem} \newtheorem{Remark}[Theorem]{Remark} \numberwithin{equation}{section} \newtheorem{Conjecture}[Theorem]{Conjecture} \newtheorem{Example}[Theorem]{Example} \begin{document} \def\F{{\mathbb F}} \def\A{{\cal A}} \def\L{{\cal L}} \def\SS{{\cal S}} \def\B{{\cal B}} \def\K{{\mathbb K}} \def\C{{\cal C}} \def\D{{\cal D}} \def\R{{\cal R}} \def\P{{\cal P}} \def\Z{{\mathbb Z}} \def\T{{\cal T}} \def\X{{\cal X}} \def\N{{\cal N}} \def\FF{{\cal F}} \def\DD{{\mathbb D}} \def\RR{{\mathbb R}} \def\NN{{\mathbb N}} \def\CC{{\mathbb C}} \def\ZZ{{\mathbb Z}} \def\chr{{\rm char}\,} \def\Re{{\rm Re}\,} \def\Im{{\rm Im}\,} \newcommand{\diag}{{\text {diag}}} \runningtitle{An upper bound on the length of an algebra} \title{An Upper Bound on the Length of an Algebra and Its Application to the Group Algebra of the Dihedral Group} \author[1]{M. A. Khrystik} \address[1]{HSE University, Faculty of Computer Science, Moscow, 101000, Russia.} \address[2]{Moscow Center of Fundamental and Applied Mathematics, Moscow, 119991, Russia.\email{good\[email protected]}} \authorheadline{M. A. Khrystik} \support{This research was supported by Russian Science Foundation, grant 20-11-20203, https://rscf.ru/en/project/20-11-20203/} \begin{abstract} Let $\A$ be an $\F$-algebra and let $\SS$ be its generating set. The length of $\SS$ is the smallest number $k$ such that $\A$ equals the $\F$-linear span of all products of length at most $k$ of elements from $\SS$. The length of $\A$, denoted by $l(\A)$, is defined to be the maximal length of its generating set. In this paper, it is shown that the $l(\A)$ does not exceed the maximum of $\dim \A / 2$ and $m(\A)-1$, where $m(\A)$ is the largest degree of the minimal polynomial among all elements of the algebra $\A$. For arbitrary odd $n$, it is proven that the length of the group algebra of the dihedral group of order $2n$ equals $n$. \end{abstract} \classification{primary 16S34; secondary 20C05, 20C30} \keywords{Finite-dimensional algebras, length of an algebra, group algebras, dihedral group, representations of dihedral groups.} \maketitle \section{Introduction} All algebras considered in this paper are {\bf associative finite-dimensional algebras with an identity over a field}. First, we recall the notion of the {\em length} of the algebra $\A$. Let $\A$ be an algebra. Any product of a finite number of elements from a finite subset $\SS \subset \A$ is called a word over the alphabet $\SS$. The length of a word equals the number of letters in this product that are different from $1_{\A}$. We consider $1_{\A}$ to be an empty word of length 0. If $\SS$ is a generating system (or a generating set) of the algebra $\A$, i.e., $\A$ is the minimal subalgebra of $\A$ containing $\SS$, then any element of the algebra $\A$ can be expressed as a linear combination of words over $\SS$. The minimal $k$ such that all elements of $\A$ can be expressed using words of length no more than $k$ is called the length of the generating system $\SS$. The length of the algebra $\A$ is defined as the maximum length among its generating systems and will be denoted by $l(\A)$ (see definition \ref{alg_len}). In defining the length of algebra $ \A $, we consider the set of {\bf all} generating systems for $ \A $. This explains the difficulty of calculating the length even for classical algebras. The general problem of calculating the length was first formulated by A.~Paz in 1984 for the full matrix algebra $M_n(\F)$ over a field in \cite{Paz} and still remains open. \begin{Conjecture}[\cite{Paz}] Let $\F$ be an arbitrary field. Then $l(M_n(\F))=2n-2.$ \end{Conjecture} A nontrivial upper bound on $l(\A)$ in terms of $\dim \A$ and $m(\A)$ (the largest degree of the minimal polynomial among all elements of the algebra $\A$) was obtained in \cite{Pap} by C.~Pappacena. The study of upper bounds on length in these terms will be continued in this paper. Calculating the length in general is a rather difficult task. The main algebraic properties of the length function were studied by O.V.~Markova in the work \cite{OVM}. The question of calculating the lengths of group algebras is of particular interest. Due to their matrix representations, solving this question is closely linked to solving Paz's problem. For group algebras of small-order groups it is possible to calculate the length precisely over arbitrary fields. For the permutation group $S_3$, Klein four-group $K_4$, and quaternion group $Q_8$, the lengths were found by A.E. Guterman and O.V. Markova in \cite{GutM18,GutM19}. Systematic study of the general problem of finding the lengths of group algebras of finite abelian groups was dedicated to the joint works of the author with A.E. Guterman and O.V. Markova \cite{GMK1,GutKhM20p2}. The works of O.V.~Markova \cite{Mar20} and the author \cite{Kh23} continued the study of the lengths of group algebras of finite abelian groups in the modular case. Studying all non-abelian groups appears to be too difficult due to the diversity of their structure. Therefore, it is proposed to study the length function separately for families of classic non-abelian groups. Thus, in the joint work of the author with O.V. Markova \cite{KhMar20}, the study of the lengths of group algebras of dihedral groups began, and the length was calculated in the semisimple case. This series of groups in the semisimple case is a natural next step after the abelian case. Indeed, for group algebras of abelian groups in the decomposition into a direct sum of matrix algebras all terms are one-dimensional, whereas the sizes of the matrix algebras in the decomposition into a direct sum of group algebras of dihedral groups do not exceed two. The work \cite{KhMar20POMI} continued the study of the lengths of group algebras of dihedral groups of order $2^k$ and calculated their length in the modular case. This paper will consider the length of the group algebra of the dihedral group over an arbitrary field. In Section \ref{main_def}, the main definitions and notations of the considered theory are introduced. In Section \ref{genbound}, the upper bound on the length is proven. In Section \ref{lendih}, the concept of bicirculant algebra is introduced and studied, in particular, its length is calculated. A bicirculant representation of the group algebra of the dihedral group is constructed and its properties are studied. Using the bicirculant representation, $l(\F \mathcal D_n)$ and $m(\F \mathcal D_n)$ are estimated. \section{Main Definitions and Notations}\label{main_def} Denote by $\langle S \rangle$ the linear span (the set of all finite linear combinations with coefficients from $\F$) of a subset $S$ of some vector space over $\F$. Let $B=\{b_1,\ldots,b_m\}$ be a non-empty finite set (alphabet). Finite sequences of letters from $B$ are called words. Let $B^*$ denote the set of all words in the alphabet $B$, $F_B$ be the free semigroup over the alphabet $B$, i.e. $B^*$ with the operation of concatenation. \begin{Definition}\label{word_len} {\em The length\/} of the word $b_{i_1}\ldots b_{i_t}$, where $b_{i_j}\in B$, is equal to $t$. We will consider $1$ (the empty word) a word from the elements $B$ {\em of length $0$\/}. \end{Definition} Let $B^i$ denote the set of all words in the alphabet $B$ of length no greater than $i$, $i\geq 0$. Then by $B^{=i}$ denote the set of all words in the alphabet $B$ of length equal to $i$, $i\geq 1$. \begin{Remark} Products of elements from the generating set $\SS$ can be considered as images of elements of the free semigroup $F_{\SS}$ under the natural homomorphism, and they can also be called words from the generators and use the natural notations $\SS^i$ and $\SS^{=i}$. \end{Remark} Denote by $\L_i(\SS)$ the linear span of words from $\SS^i$. Note that $\L_0(\SS)=\langle 1_{\A}\rangle=\F$. Let also $\L(\SS)=\bigcup\limits_{i=0}^\infty \L_i(\SS)$ denotes the linear span of all words in the alphabet $\SS=\{a_1,\ldots, a_k\}$. \begin{Definition}\label{sys_len} {\em The length of a generating system $\SS$\/} of algebra $\A$ is $l(\SS)=\min\{k\in \ZZ_+: \L_k(\SS)=\A\}$. \end{Definition} \begin{Definition}\label{alg_len} {\em The length of an algebra $\A$} is $l(\A)=\max \{l(\SS): \L(\SS)=\A\}$. \end{Definition} Let $\A$ be an algebra, $\tau \in \A$. Denote the minimal polynomial of $\tau$ by $\mu_{\tau}(x)$. Then $m(\tau)=\deg \mu_{\tau}(x)$, $m(\A)=\max_{\tau \in \A} m(\tau)$. Denote by $\F G$ or $\F[G]$ the group algebra of the group $G$ over the field $\F$, $E_{i,j}$ for the matrix unit, $\mathcal D_n$ for the dihedral group of order $2n$, $S_n$ for the symmetric group. \begin{Definition}\label{equiv} We say that two words $u$ and $v$ of length $i$ from the generators are {\em equivalent}, if $u-\alpha v\in \L_{i-1}(\SS)$ for some nonzero $\alpha \in \F$. We will use the notation $u\sim v$ in this case. \end{Definition} \begin{Definition} We say that a word $u$ of length $i$ from the generators {\em reducible} if $u\in \L_{i-1}(\SS)$. Otherwise, we will call the word {\em irreducible}. \end{Definition} \section{General Bound on Length}\label{genbound} \subsection{Equivalence of Words}\ Before proceeding to prove the main statement of the section let us note some properties of the introduced concept of word equivalence as it is significantly used in the proof of this statement. \begin{Lemma}\label{eqrel} Equivalence of words is an equivalence relation on the set of words. \end{Lemma} \begin{proof} {\em Reflexivity.} $u-\alpha u \in \L_{i-1}(\SS)$ with $\alpha=1.$ {\em Symmetry.} Let $u-\alpha v \in \L_{i-1}(\SS)$. Then, by multiplying the element $u-\alpha v$ by $-\alpha^{-1}$, we get $v-\alpha^{-1} u \in \L_{i-1}(\SS).$ {\em Transitivity.} Let $u-\alpha_1 v \in \L_{i-1}(\SS)$, $v-\alpha_2 w \in \L_{i-1}(\SS)$. Then, by adding the second element multiplied by $\alpha_1$ to the first one, we obtain $u-\alpha_1 \alpha_2 w \in \L_{i-1}(\SS).$ \end{proof} \begin{Lemma}\label{eqred} Let $u \sim v$. Then $u$ is reducible if and only if $v$ is reducible. \end{Lemma} \begin{proof} Straightforward. \end{proof} \begin{Lemma}\label{eqsub} Let the word $u$ be irreducible. Then any subword of $u$ is irreducible. \end{Lemma} \begin{proof} Straightforward. \end{proof} \begin{Lemma}\label{eqrep} Let the word $w$ of length $i$ contain a subword $u$ of length $j$, $u \sim v$. Then $w \sim w'$, where $w'$ is a word obtained from $w$ by replacing the subword $u$ with the subword $v$. \end{Lemma} \begin{proof} By condition, $u-\alpha v \in \L_{j-1}(\SS)$, $w=w_1uw_2$, for some words $w_1$, $w_2$. Then, by multiplying the expression $u-\alpha v$ on the left by $w_1$ and on the right by $w_2$, we get $w-\alpha w' \in \L_{i-1}(\SS).$ \end{proof} \subsection{Estimating $l(\A)$ Using $\dim \A$ and $m(\A)$}\ \begin{Theorem}\label{ldm} Let $\A$ be an associative finite-dimensional algebra with an identity. Then $$l(\A)\leq max\{m(\A)-1,\frac{\dim\A}{2}\}.$$ \end{Theorem} \begin{proof} Let $l(\A)\geq m(\A)$ (otherwise the statement is proven). Let $\SS$ be a generating set of length $l(\A)$ of the algebra $\A$ (in the case of other generating sets the length of the algebra will be no greater). Consider an irreducible word $a_1a_2\cdots a_{l(\A)}$ of length $l(\A)$ in the alphabet $\SS$ (such exists by definition of the length of the algebra). We will prove that $\forall k\in [1,l(\A)-1]$ it holds that $\dim\L_k(\SS)-\dim \L_{k-1}(\SS)\geq 2.$ We will reason by contradiction. Suppose $\exists k\in [1,l(\A)-1]$ such that $\dim\L_k(\SS)-\dim \L_{k-1}(\SS)=1$ (this difference cannot be zero by definition of the length of the algebra). We will break the reasoning into steps and lead it to a contradiction. {\em First step.} The word $a_1a_2\cdots a_{l(\A)}$ is irreducible. Therefore, its subword $a_1a_2\cdots a_k$ is irreducible by Lemma \ref{eqsub}. By assumption $a_2a_3\cdots a_{k+1} \sim a_1a_2\cdots a_k$ (here we use the fact that $k$ is no greater than $l(\A)-1$). Indeed, if this were not the case, we would get $\dim\L_k(\SS)-\dim \L_{k-1}(\SS)\geq 2$, since the dimension would increase by at least 2 due to these two words. Thus, $a_1a_2\cdots a_{l(\A)} \sim a_2 a_3\cdots a_k a_{k+1} a_{k+1} a_{k+2} \cdots a_{l(\A)}$ by Lemma \ref{eqrep}. Therefore, the word $ a_2 a_3\cdots a_k a_{k+1} a_{k+1} a_{k+2} \cdots a_{l(\A)}$ is irreducible. {\em Second step.} Now consider the irreducible word $ a_2 a_3\cdots a_k a_{k+1} a_{k+1} a_{k+2} \cdots a_{l(\A)}$ of length $l(\A)$ obtained in the previous step. By reasoning similarly (considering subwords of length $k$ starting from the first and second letters), we will get rid of the letter $a_2$ similarly to how we got rid of the letter $a_1$ in the first step. We obtain that the word $ a_3 a_4\cdots a_k a_{k+1} a_{k+1} a_{k+1} a_{k+2} \cdots a_{l(\A)}$ is irreducible. After conducting $k$ steps of this reasoning, we obtain that the word $a_{k+1}\cdots a_{k+1} a_{k+2} \cdots a_{l(\A)}$ of length $l(\A)$ is irreducible. Now we can proceed to the last step and obtain a contradiction. {\em $(k+1)$-st step.} The word $a_{k+1}^{k+1} a_{k+2} \cdots a_{l(\A)}$ is irreducible. Therefore, its subword $a_{k+1}^{k}$ is irreducible. By assumption, all words of length $k$ are expressed through the word $a_{k+1}^{k}$ and words of shorter length. Thus, $a_1a_2\cdots a_{l(\A)} \sim a_{k+1}^{l(\A)}$. Therefore, the word $a_{k+1}^{l(\A)}$ is irreducible and $l(\A)< m(\A)$. Contradiction. We return to the proof of the main statement. Represent the dimension of the algebra in the following form $\dim \A=\dim\L_{l(\A)}(\SS)=(\dim\L_{l(\A)}(\SS)-\dim\L_{l(\A)-1}(\SS))+(\dim\L_{l(\A)-1}(\SS)-\dim\L_{l(\A)-2}(\SS))+\cdots+(\dim\L_1(\SS)-\dim\L_0(\SS))+\dim\L_0(\SS)$. The first term of this sum is not less than 1, the last one equals 1, and all the others are not less than 2. Thus, $\dim \A \geq 1+2(l(\A)-1)+1$. Therefore, $l(\A) \leq \frac{\dim\A}{2}$. Thus, $l(\A)\leq max\{m(\A)-1,\frac{\dim\A}{2}\}.$ \end{proof} \subsection{Comparison with Other Estimates}\ In conclusion of this section we will compare the obtained bound with other similar bounds. Let us compare the obtained bound with the following bound presented in the joint work of the author with O.V. Markova. \begin{Lemma}[{\cite[Lemma 2.10]{KhMar20POMI}}]\label{d<m+4} Let $\mathcal A$ be an $\F$-algebra, $\dim\mathcal A\leq m(\mathcal A)+4$, $m({\mathcal A}) \geq 3$. Then $l(\mathcal A) \leq m(\mathcal A)$. \end{Lemma} Since $m(\A)-1$ is unequivocally less than $m(\A)$, we see that the new estimate will be worse than the estimate from Lemma \ref{d<m+4} only if $\dfrac{\dim\A}{2} \geq m(\A)+1$ (that is, if $\dim\A \geq 2m(\A)+2$). Also, by the condition of Lemma \ref{d<m+4} it must be fulfilled that $\dim\mathcal A\leq m(\mathcal A)+4$. From the last two inequalities, it follows that $m(\A) \leq 2$. But in the condition of Lemma \ref{d<m+4} it is also required that $m({\mathcal A}) \geq 3$. Therefore, the new bound is better in any case. Next we will compare with the following Pappacena's estimate. \begin{Theorem}[{\cite[Theorem 3.1]{Pap}}]\label{Pap} Let $\A$ be any algebra. Then $ l(\A)< f(\dim \A,m(\A))$, where $$f(d,m)=m\sqrt{\frac{2d}{m-1}+\frac{1}{4}}+\frac{m}{2}-2.$$ \end{Theorem} Since $\dim\A \geq m(\A)-1$, we have $m\sqrt{\dfrac{2d}{m-1}+\dfrac{1}{4}}+\dfrac{m}{2}-2 \geq m\sqrt{\dfrac{9}{4}}+\dfrac{m}{2}-2 = 2m-2.$ Since $m(\A)-1$ is less than $2m(\A)-2$, we see that the new estimate will be worse than Pappacena's estimate only if $\dfrac{\dim\A}{2} > 2m(\A)-2$ (that is, if $\dim\A > 4(m(\A)-1)$). That is, the new bound can be worse than Pappacena's bound only if the dimension of the algebra is 4 times greater than the expression $m(\A)-1$. In particular, the new estimate is unequivocally better when considering group algebras of dihedral groups, which will be discussed in the next section. However, Theorem \ref{ldm} may give a more accurate estimate than Theorem \ref{Pap} even if $\dim\A \leq 4(m(\A)-1)$. Let us show that by the following example. \begin{Example} Let $\A = M_3(\mathbb F)$. Then $\dim \A = 9$, $m(\A)=3$. Theorem \ref{Pap} gives an estimate $l(\A) \leq 8$. Theorem \ref{ldm} gives an estimate $l(\A) \leq 4$, which corresponds to the value $l(M_3(\mathbb F))$ in Paz's conjecture. \end{Example} \section{Calculating $l(\F\D_n$)}\label{lendih} \subsection{Bicirculant Algebra}\ Let us consider two matrices. The circulant $A_n=E_{n,1}+E_{1,2}+\cdots+E_{n-1,n}$ and the anti-circulant $B_n=E_{1,n}+\cdots +E_{n,1}$. $$ A_n= \begin{pmatrix} 0 & 1 & 0 &\ldots & 0\\ 0 & 0 & 1 &\ldots & 0\\ 0 & 0 & 0 &\ldots & 0\\ \vdots& \vdots & \vdots &\ddots & \vdots\\ 0 & 0 & 0 &\ldots & 1\\ 1 & 0 & 0 &\ldots & 0 \end{pmatrix} ,\quad B_n= \begin{pmatrix} 0 & 0 &\ldots & 0 & 1\\ 0 & 0 &\ldots & 1 & 0\\ \vdots& \vdots & \ddots &\vdots & \vdots\\ 0 & 0 &\ldots & 0 & 0\\ 0 & 1 &\ldots & 0 & 0\\ 1 & 0 &\ldots & 0 & 0 \end{pmatrix}. $$ Let us define the algebra generated by these two matrices. \begin{Definition} {\em The algebra of bicirculants of order n} over the field $\F$ is $\C_n(\F)=\L(\{A_n,B_n\})$. \end{Definition} Let us study the structure of this algebra. \begin{Lemma}\label{bcrel} $A_n^n=E$, $B_n^2=E$, $A_nB_n=B_nA_n^{n-1}$. \end{Lemma} \begin{proof} The equalities are checked directly by multiplying matrices. \end{proof} \begin{Lemma}\label{bcdim} $\dim \C_n(\F)=\begin{cases} 2n-2,\ \mbox{for even}\; n;\\ 2n-1, \ \mbox{for odd}\; n. \end{cases}$ \end{Lemma} \begin{proof} Due to Lemma \ref{bcrel} we may consider that $\C_n(\F)=\C_n'(\F)+\C_n''(\F)$, where $\C_n'(\F)=\langle E,A_n,A_n^2,\dots,A_n^{n-1}\rangle$, $\C_n''(\F)=\langle B_n,B_nA_n,B_nA_n^2,\dots,B_nA_n^{n-1}\rangle$. Note that $\C_n'(\F)$ is nothing else but the space of circulants, and $\C_n''(\F)$ is the space of anti-circulants, each of which has a dimension of $n$. The basis of the intersection of the spaces $\C_n'(\F)$ and $\C_n''(\F)$ in the odd case is the matrix in which each element equals 1, and in the even case, the basis will be the following two matrices $$ \begin{pmatrix} 1 & 0 & 1 &\ldots & 0\\ 0 & 1 & 0 &\ldots & 1\\ 1 & 0 & 1 &\ldots & 0\\ \vdots& \vdots & \vdots &\ddots & \vdots\\ 1 & 0 & 1 &\ldots & 0\\ 0 & 1 & 0 &\ldots & 1 \end{pmatrix} \ \mbox{and } \begin{pmatrix} 0 & 1 & 0 &\ldots & 1\\ 1 & 0 & 1 &\ldots & 0\\ 0 & 1 & 0 &\ldots & 1\\ \vdots& \vdots & \vdots &\ddots & \vdots\\ 0 & 1 & 0 &\ldots & 1\\ 1 & 0 & 1 &\ldots & 0 \end{pmatrix}. $$ Thus, the statement of the lemma follows from the formula for the dimension of the sum of subspaces. \end{proof} \begin{Theorem}\label{bclen} $l(\C_n(\F))=n-1.$ \end{Theorem} \begin{proof} Let us first prove the lower bound $l(\C_n(\F))\geq n-1.$ Consider a generating set $\SS=\{u,v\}$, where $u=B_n, v=A_nB_n$. This is indeed a generating set, as $\C_n(\F)=\L(\{A_n,B_n\})=\L(\{vu,u\})\subseteq \L(\{u,v\})=\L(\{B_n,A_nB_n\})\subseteq \L(\{A_n,B_n\})=\C_n(\F)$. At the same time, $u^2=v^2=E$, meaning that there are no more than two irreducible words of each length (of the form $uvuv\dots$ and $vuvu\dots$). Thus, $\dim\L_{n-2}(\SS)=(\dim\L_{n-2}(\SS)-\dim\L_{n-3}(\SS))+(\dim\L_{n-3}(\SS)-\dim\L_{n-4}(\SS))+\cdots+(\dim\L_1(\SS)-\dim\L_0(\SS))+\dim\L_0(\SS)\leq 2(n-2)+1<\dim\C_n(\F)$, from which it follows that the length of the algebra is at least $n-1$. The upper bound $l(\C_n(\F))\leq n-1$ follows from Theorem \ref{ldm}. Indeed, by the Cayley-Hamilton theorem, $m(\C_n(\F))\leq n$. By Lemma \ref{bcdim}, $\dim \C_n(\F)\leq 2n-1$. Applying Theorem \ref{ldm}, we obtain the inequality $l(\C_n(\F)) \leq max\{n-1,\frac{2n-1}{2}\}$. This completes the proof. \end{proof} \subsection{Bicirculant Representation of $\F\D_n$}\label{bcsect}\ Let us number the vertices of a regular $n$-gon. Let $d\in \D_n$ map the vertex $i$ to the vertex $\sigma(i)$ $\forall i$, where $\sigma\in S_n$. Then we can consider a group homomorphism, defining its values on elements of $\D_n$ by the rule $f(d)=\sigma$, and then extend it to an algebra homomorphism $f:\F\D_n\rightarrow \F S_n$ by linearity. Let us now consider a group homomorphism $g: S_n \rightarrow M_n(\{0,1\})$, which maps a permutation from $S_n$ into the corresponding permutation matrix. We extend it to an algebra homomorphism $g: \F S_n \rightarrow M_n(\F)$ by linearity. Note that the composition $g\circ f$ defines a linear representation of the algebra $\F\D_n$. This representation is called the {\em bicirculant representation} in this paper. Let us study some properties of this composition. \begin{Lemma}\label{imgf} $\Im g\circ f = \C_n(\F)$. \end{Lemma} \begin{proof} Let $a$ be the rotation by an angle $\frac{2\pi}{n}$, $b$ be the symmetry about the axis passing through the vertex $\left[\dfrac{n}{2}\right]+1$. Then $\F \D_n=\langle e, a, a^2, \dots, a^{n-1}, b, ba, \dots , ba^{n-1} \rangle$. It is easy to notice that $g\circ f(a)=A_n$, $g\circ f(b)=B_n$. Since $g\circ f$ is a homomorphism, $g\circ f(b^ia^j)=B_n^iA_n^j$, from which the statement of the lemma follows. \end{proof} \begin{Lemma}\label{kergf} $\ker g\circ f = \langle e+a+\cdots+a^{n-1}-b-ba-\cdots-ba^{n-1} \rangle$, for odd n. $\ker g\circ f = \langle e+a^2+\cdots+a^{n-2}-b-ba^2-\cdots-ba^{n-2}, a+a^3+\cdots+a^{n-1}-ba-ba^3-\cdots-ba^{n-1}\rangle$, for even n. \end{Lemma} \begin{proof} The dimension of the kernel is established using Lemmas \ref{bcdim} and \ref{imgf}. The fact that the specified elements lie in the kernel and are linearly independent (in the case of even $n$) is checked directly. \end{proof} \subsection{Length of $\F\D_n$}\ First, let us present known results about the length of $\F\D_n$. \begin{Lemma}[{\cite[Lemma 2.1]{KhMar20}}]\label{Died} Let ${\cal D}_{n}$ be the dihedral group of order $2n$, $n\geq3$, $\F$ be an arbitrary field. Then $l(\F{\cal D}_{n})\geq n$. \end{Lemma} \begin{Theorem}[{\cite[Theorem 1.15]{KhMar20}}]\label{Died_n} Let $\F$ be a field such that $\chr \F$ does not divide $2n$. Then $l(\F {\cal D}_n)=n$, for $n\geq 3$. \end{Theorem} \begin{Theorem}[{\cite[Theorem 4.10]{KhMar20POMI}}] Let $\chr \F = 2$, $k\geq 2$. Then $l(\F\D_{2^k})=2^k$.\label{lfd2k} \end{Theorem} In this paper, we will try to generalize the last two theorems, namely, to eliminate the condition on the field. Hereinafter in the work, it is assumed that $n\geq 3$. Let us prove the main result of the section. In the proof of the following lemma the author uses the idea of proving Lemma 3.11 from \cite{GutM18}. \begin{Lemma}\label{sur} Let there exist a surjective homomorphism of algebras $\varphi:\cal A \rightarrow \cal B$. Then $$l(\mathcal A) \leq l (\mathcal B)+\dim \mathcal A -\dim \cal B.$$ \end{Lemma} \begin{proof} Consider an arbitrary generating set $\SS=\{a_1,\dots,a_k\}$ of the algebra $\A$. Since the homomorphism $\varphi$ is surjective, we see that the set $\SS_{\mathcal{B}}=\{c_1=\varphi(a_1),\dots,c_k=\varphi(a_k)\}$ is a generating set of the algebra $\mathcal{B}$. Therefore, $\dim\L_{l(\mathcal{B})}(\SS_{\B})=\dim \B$. On the other hand, $\L_{l(\B)}(\SS_{\B})=\L_{l(\B)}(\varphi(\SS))=\varphi(\L_{l(\B)}(\SS))$. Therefore, $\dim\L_{l(\B)}(\SS)\geq \dim \varphi (\L_{l(\B)}(\SS))=\dim \L_{l(\B)}(\SS_{\B})=\dim \B$. Since the dimensions $\L_{i}(\SS)$ must increase with $i$ until stabilization, we have $\dim\L_{l(\B)+\dim \A - \dim \B}(\SS)\geq \dim \B + (\dim \A - \dim \B) = \dim \A$. At the same time, the minimal $i$ such that $\dim \L_{i}(\SS) = \dim \A$, by definition, is $l(\SS)$. Due to the arbitrariness of $\SS$, we obtain $l(\mathcal A) \leq l (\mathcal B)+\dim \mathcal A -\dim \cal B$. \end{proof} \begin{Theorem}\label{lendn} Let ${\cal D}_{n}$ be the dihedral group of order $2n$, $n\geq3$, $\F$ be an arbitrary field. Then\\ $l(\F{\cal D}_{n})=\begin{cases} n,\ \mbox{for odd}\; n;\\ n \ \mbox{or}\; n+1, \ \mbox{for even}\; n. \end{cases}$ \end{Theorem} \begin{proof} The lower bound is given by Lemma \ref{Died}. Let us prove the upper bound. From Theorem \ref{bclen} it follows that $l(\C_n(\F))=n-1$. From Lemma \ref{bcdim} it follows that $\dim \C_n(\F)=2n-1$ for odd $n$, $\dim \C_n(\F)=2n-2$ for even $n$. Consider the homomorphism of algebras $g \circ f : \F \mathcal{D}_n \rightarrow \C_n(\F)$, described in Section \ref{bcsect}. Since by Lemma \ref{imgf} the homomorphism $g \circ f$ is surjective, we can apply Lemma \ref{sur} and get the upper bound $l(\F \mathcal{D}_n) \leq l (\C_n(\F))+\dim \F \mathcal{D}_n -\dim \C_n(\F)$. Then application of Theorem \ref{bclen}, Lemma \ref{bcdim} and the fact that $\dim \F \mathcal{D}_n = 2n$ completes the proof. \end{proof} \begin{Remark} Despite the fact that among the possible values of $l(\F{\cal D}_{n})$ there is $n+1$, no real examples of algebras with this length have been found (and are not expected given Theorem \ref{Died_n}). The developed technique allows finding the exact value only for odd $n$, however, the obtained result is a noticeable advancement in the study of the lengths of group algebras of dihedral groups, demonstrating the usefulness of the bound proven in Theorem \ref{ldm} and the bicirculant representation. \end{Remark} \subsection{Bound for $m(\F{\cal D}_{n})$}\ Using the bicirculant representation, we get an estimate of $m(\F{\cal D}_{n})$. \begin{Theorem}\label{mdn} Let ${\cal D}_{n}$ be the dihedral group of order $2n$, $n\geq3$, $\F$ be an arbitrary field. Then\\ $m(\F{\cal D}_{n})\leq \begin{cases} n+1,\ \mbox{for odd}\; n;\\ n+2, \ \mbox{for even}\; n. \end{cases}$ \end{Theorem} \begin{proof} Let $\tau \in \F{\cal D}_{n}$, $g \circ f : \F \mathcal{D}_n \rightarrow \C_n(\F)$ be the homomorphism of algebras described in Section \ref{bcsect}, $a$ be the rotation by an angle $\frac{2\pi}{n}$, $b$ be the symmetry. Let $g \circ f(\tau) = T \in M_n(\F)$. Then by the Cayley-Hamilton theorem $m(T)=\deg \mu_T(x) \leq n$. Since $g \circ f(\mu_{T}(\tau))=\mu_T(T)=0$, we get $\mu_{T}(\tau) \in \ker g \circ f$. Next, consider two cases separately. First case. Let $n$ be odd. Then from Lemma \ref{kergf} it follows that $\ker g \circ f$ is one-dimensional. On the other hand, the kernel of a homomorphism of algebras is an ideal, which means $\mu_{T}(\tau)$ and $\mu_{T}(\tau)\tau$ are linearly dependent. Thus, $m(\F{\cal D}_{n})\leq n+1$. Second case. Let $n$ be even. Then from Lemma \ref{kergf} it follows that $\ker g \circ f$ is two-dimensional. On the other hand, the kernel of a homomorphism of algebras is an ideal, which means $\mu_{T}(\tau)$, $\mu_{T}(\tau)\tau$, and $\mu_{T}(\tau)\tau^2$ are linearly dependent. Thus, $m(\F{\cal D}_{n})\leq n+2$. \end{proof} \begin{Remark} The main conjecture regarding the lengths of group algebras in the case of dihedral groups is that $l(\F{\cal D}_{n})=n$ for all $n\geq 3$ over an arbitrary field. Due to Theorem \ref{ldm}, to prove the conjecture, it is sufficient to obtain an estimate $m(\F{\cal D}_{n})\leq n+1$. However, using an estimate from Theorem \ref{mdn}, we get the same result as presented in Theorem \ref{lendn}. Nevertheless, estimating $m(\F{\cal D}_{n})$ allows us to demonstrate another application of the Theorem \ref{ldm} and the bicirculant representation, and the study of numerical characteristics of algebras is of interest in itself. \end{Remark} \begin{thebibliography}{99} \bibitem{GutM18} A. E.~Guterman, O. V.~Markova, {\em The length of group algebras of small-order groups}. --- Zap. Nauchn. Sem. POMI, {\bf 472} (2018), 76--87; English transl. in J. Math. Sci. {\bf 240}:6 (2019), 754--761. \bibitem{Mar20} O. V.~Markova, {\em An example of length computation for a group algebra of a noncyclic abelian group in the modular case}. --- Fundam. Prikl. Mat., {\bf 23}:2 (2020), 217–229; English transl. in J. Math. Sci. {\bf 262}:5 (2022) 740--748. \bibitem{Kh23} M.A. Khrystik, {\em Length of the group algebra of the direct product of a cyclic group and a cyclic p-group in the modular case}. --- Zap. Nauchn. Semin. POMI, {\bf 524}(2023), 166--176. \bibitem{OVM} O. V.~Markova, {\em The length function and matrix algebras}. --- Fundam. Prikl. Mat., {\bf 17}:6 (2012), 65--173; English transl. in J. Math. Sci. {\bf 193}:5 (2013) 687--768. \bibitem{KhMar20POMI} M. A. Khrystik, O. V. Markova {\em The length of the group algebra of the dihedral group of order $2^k$}. --- Zap. Nauchn. Sem. POMI, {\bf 496}(2020), 169–181; English transl. in J. Math. Sci. (N. Y.), {\bf 255}:3 (2021), 324--331. \bibitem{GutKhM20p2} A.~E.~Guterman, M.~A.~Khrystik, O.~V.~Markova, {\em On the lengths of group algebras of finite abelian groups in the modular case}. --- J. Algebra its Appl., {\bf 21}:6 (2022), 2250117–2250130. \bibitem{GutM19} A.~E.~Guterman, O.~V.~Markova, {\em The length of the group algebra of the group ${\mathbf Q_8}$}. --- New Trends in Algebra and Combinatorics. Proceedings of the 3rd International Congress in Algebra and Combinatorics (Ed. by K.P. Shum, E. Zelmanov, P. Kolesnikov, A. Wong), World Sci., Singapore, (2019), 106--134. \bibitem{GMK1} A.~E.~Guterman, O.~V.~Markova, M.~A.~Khrystik, {\em On the lengths of group algebras of finite abelian groups in the semi-simple case}. --- J. Algebra its Appl., {\bf 21}:7 (2022), 2250140--2250153. \bibitem{KhMar20} M.~A.~Khrystik, O.~V.~Markova, {\em On the length of the group algebra of the dihedral group in the semi-simple case}. --- Commun Algebra, {\bf 50}:5 (2022), 2223–2232. \bibitem{Pap} C.~J.~Pappacena, {\em An upper bound for the length of a finite-dimensional algebra}. --- J. Algebra, {\bf 197} (1997), 535--545. \bibitem{Paz} A.~Paz, {\em An application of the Cayley--Hamilton theorem to matrix polynomials in several variables}. --- Linear Multilinear Algebra, {\bf 15} (1984), 161--170. \end{thebibliography} \end{document} To typeset a list of items number (i), (ii), ... use \begin{enumerate}[(i)] \item first item \item second item \end{enumerate} To typeset a list of items number H1, H2, ... use \begin{enumerate}[H1] \item first item \item second item \end{enumerate} \begin{figure} \centering \caption{Caption text}\label{fig1:} \end{figure} \begin{table} \caption{Caption text}\label{tab:1} \centering \begin{tabular}{cccc} \toprule \multicolumn{2}{c}{text} & \multicolumn{2}{c}{text}\\ \cmidrule(r){1-2}\cmidrule(l){3-4} \multicolumn{1}{c}{One} & Two & \multicolumn{1}{c}{Three} & Four\\ \midrule 1 & 2 & 3 & 4 \\ 1 & 2 & 3 & 4 \\ \bottomrule \end{tabular} \end{table}
2412.06109v1
http://arxiv.org/abs/2412.06109v1
Permutation clones that preserve relations
\documentclass[runningheads]{llncs} \usepackage[T1]{fontenc} \usepackage[normalem]{ulem} \usepackage{graphicx} \usepackage{tikz} \usepackage{amsmath} \usepackage{amsfonts} \usepackage{amssymb}\usepackage{makecell} \usepackage[]{mdframed} \newcommand{\framednote}[1]{\par\noindent\fbox{ \parbox{\dimexpr\linewidth-2\fboxsep-2\fboxrule}{#1}}} \begin{document} \title{Permutation clones that preserve relations} \newcommand{\N}{\mathbb N} \newcommand{\Z}{\mathbb Z} \newcommand{\Q}{\mathbb Q} \newcommand{\B}{\mathbb B} \newcommand{\A}{\mathbb A} \newcommand{\C}{\mathbb C} \newcommand{\OO}{{\cal O}} \newcommand{\RR}{{\cal R}} \newcommand{\Pow}{{\cal P}} \newcommand{\Fix}{\operatorname{Fix}} \newcommand{\Pol}{\operatorname{Pol}} \newcommand{\PPol}{\operatorname{PPol}} \newcommand{\Inv}{\operatorname{Inv}} \newcommand{\Cons}{\operatorname{Cons}} \newcommand{\Aff}{\operatorname{Aff}} \newcommand{\Deg}{\operatorname{Deg}} \newcommand{\Par}{\operatorname{Par}} \newcommand{\Aut}{\operatorname{Aut}} \newcommand{\Inn}{\operatorname{Inn}} \newcommand{\Out}{\operatorname{Out}} \newcommand{\bigboxplus}{\scalebox{1.6}{$\boxplus$}} \author{Tim Boykett\inst{1}\orcidID{0000-0002-3003-0927} } \authorrunning{T. Boykett} \institute{Institute for Algebra, Johannes-Kepler University, Linz, Austria \\ Time's Up Research, Linz, Austria \email{[email protected]}\\ \url{http://timesup.org} \\ University for Applied Arts, Vienna} \maketitle \begin{abstract} Permutation clones generalise permutation groups and clone theory. We investigate permutation clones defined by relations, or equivalently, the automorphism groups of powers of relations. We find many structural results on the lattice of all relationally defined permutation clones on a finite set. We find all relationally defined permutation clones on two element set. We show that all maximal borrow closed permutation clones are either relationally defined or cancellatively defined. Permutation clones generalise clones to permutations of $A^n$. Emil Je\v{r}\'{a}bek found the dual structure to be weight mappings $A^k\rightarrow M$ to a commutative monoid, generalising relations. We investigate the case when the dual object is precisely a relation, equivalently, that $M={\mathbb B}$, calling these relationally defined permutation clones. We determine the number of relationally defined permutation clones on two elements (13). We note that many infinite classes of clones collapse when looked at as permutation clones. \keywords{reversible gates \and permutation clones \and weight mappings \and borrow closure} \end{abstract} \section{Introduction} Given a finite set of logic signals, the reversible gates for these signals are the basis upon which a reversible computer can be designed and built. The number of logic signals, whether binary logic, ternary logic or some higher arity logic, will affect which gates are possible. In this paper we deepen our understanding of these gates and the ways that they can be combined to engineer reversible computer systems. In particular we will borrow strongly from clone theory, itself developed as an abstract theory of circuits. For a given finite set of logic signals $A$, a $k$-ary gate is a bijection from $A^k$ to itself, that is, a permutation of $A^k$. Thus we investigate collections of permutations of $A^k$ for all $k$, closed under natural operations of parallel and serial composition. Taking the lead from \cite{jerabek18} we call closed systems of bijections \emph{permutation clones}. We consider ancilla and borrow closure, where an extra input and output is allowed; an \emph{ancilla} logic state is provided and returned in a particular state, whereas a \emph{borrowed} logic state is provided and returned in an arbitrary state. In previous papers, Aaronson, Grier and Schaeffer \cite{aaronsonetal15} have determined all ancilla closed gates on a set of order 2, and the author, together with Kari and Salo, has investigated generating sets and other topics \cite{b19,bks17}. More related work is outlined in the next section. In this paper, we examine permutation clones that are naturally related to clones. We consider permutation clones that are defined by their components maps as clones. Clones are defined by the relations that they respect using a Galois connection. Through these results, we gain some insight into the general structure of reversible gate systems. We start Section 2 by introducing the background properties of permutation clones and the duality results from Je\v{r}\'{a}bek. We then investigate the properties of relationally defined permutation clones and investigate the various maximal classes. We conclude with some open questions and ideas for further work. \section{Background} We start by introducing some clone theory and permutation group theory. In particular we are interested in co-clones or relational clones as well as automorphism groups of relations. We will then bring these together as permutation clones, and introduce weight preservation as the appropriate dual structure. Let $A$ be a finite set. Define ${\cal O}_A^n$ to be the set of all $n$-ary mappings $f:A^n\rightarrow A$. Let $\OO(A) = \{f:A^n\rightarrow A \mid n \in \N\}= \cup_n{\cal O}_A^n$ be the collections of multivariate mappings of $A$ to itself. Such mappings can be transformed by \begin{enumerate} \item permuting variables: if $f:A^n\rightarrow A$ and $\sigma\in S_n$ is a permutation, define $g:A^n\rightarrow A$ by $g(x_1,\dots,x_n)= f(x_{\sigma^{-1}(1)},\dots, x_{\sigma^{-1}(n)})$. \item identifying variables: if $f:A^n\rightarrow A$ and $n > 1$ , define $g:A^{n-1}\rightarrow A$ by $g(x_1,\dots,x_{n-1})= f(x_1,\dots, x_{n-1},x_{n-1})$. \item composition of functions: if $f:A^n\rightarrow A$ and $g:A^{m}\rightarrow A$, define $h:A^{m+n-1}\rightarrow A$ by $h(x_1,\dots,x_{m+n-1})= f(g(x_1,\dots, x_{n}),x_{n+1},\dots,x_{n+m-1})$. \item inserting dummy variables: if $f:A^n\rightarrow A$ , define $g:A^{n+1}\rightarrow A$ by $g(x_1,\dots,x_{n+1})= f(x_1,\dots, x_{n})$. \end{enumerate} The projection mapping $\pi_i^n:A^n\rightarrow A$ is defined by $\pi_i^n(x_1,\dots,x_n)=x_i$. A collection of mappings $C \subseteq \OO(A) $ is called a \emph{clone} if it contains all projections and is closed under the transformation operations above. A set that is closed under the operations but does not include the projections is called an \emph{iterative algebra}. Let $R\subseteq A^k$ be a $k$-ary relation on $A$. Let $f:A^n\rightarrow A$ be a $n$-ary map on $A$. We can extend $f$ to act on $A^k$ componentwise, so $f:(A^k)^n \rightarrow A^k$ in a natural fashion. If $f$ maps $R$ to itself, then we say that $f$ \emph{respects} the relation $R$, written $f \triangleright R$. We call this a \emph{polymorphism} of $R$. This gives rise to a Galois connection between maps and relations on $A$, a duality. \begin{definition}[\protect{\cite[Section 2.4]{lau_2006}}] A \emph{relational clone} is a collection of relations closed under the following operations: \begin{enumerate} \item permutation of entries \item projection; ignoring one entry \item cartesian products \item intersection \item includes the equality relation $\{(a,a,b) \mid a,b \in A\}$ \end{enumerate} \end{definition} \begin{lemma} Let $C$ be a clone. Then there exists a relational clone $\RR$ such that for all $f \in \OO(A)$, $f \in C$ iff $f \triangleright R$ for all $R \in \RR$. \end{lemma} The collection of maps that respect a relation $R$ is written $\Pol(R)$, the \emph{polymorphisms} of $R$. All clones are of the form $\bigcap_i \Pol(R_i)$ for some (possibly infinite) set of relations $R_i$. We call relations the \emph{dual structure} of clones. Clone theory has been well developed for several decades, see e.g.\ \cite{kerkhoffetal,lau_2006} for an overview. One of the classical results in clone theory is the determination of the maximal clones. \begin{theorem}[Rosenberg] Let $A$ be a finite set. Then the maximal subclones of ${\mathcal O}(A)$ are one of the following. \begin{enumerate} \item monotone mappings, that is respecting a bounded partial order on $A$ \item respecting a graph of prime length loops \item respecting a nontrivial equivalence relation \item affine mappings for a prime $p$: that is, respecting the relation $\{(a,b,c,d) \mid a+b = c+d\}$ where $(A,+)$ is an elementary abelian group \item respecting a central relation \item respecting a h-generated relation \end{enumerate} \end{theorem} Let $A$ be a finite set. We will assume a basic understanding of permutation group theory, but clarify some terminology here, for details see e.g.\ \cite{mortimerdixon}. $Sym(A)=S_A$ is the set of permutations or bijections of $A$. If $A=\{1,\dots,n\}$ we will often write $S_n$. We write permutations in cycle notation and act from the right, so we write the action of a permutation $g\in G \leq Sym(A)$ on an element $a\in A$ as $a^g$. Given a set $A$ and a $K$-ary relation $R \subseteq A^k$, the collection $\Aut(R) \leq S_A$ of automorphisms of that relation $R$ is a group. For example the relation defining the edges of a square on $A=\{1,2,3,4\}$ with \begin{equation} R=\{ (1,2),(2,3),(3,4),(4,1),(2,1),(3,2), (4,3),(1,4)\} \subseteq A^2 \end{equation} has $\Aut(R)$ the dihedral group with 8 elements. We can speak of $(A,R)$ as a \emph{relational structure} in the same way that an operation $f:A^n\rightarrow A$ gives $(A,f)$ an algebraic structure. Let $G$ be a group of permutations of a set $A$. Let $n \in \N$. The \emph{wreath product} $G wr S_n$ is a group of permutations acting on $A^n$. The elements of $G wr S_n$ are $\{(g_1,\dots,g_n,\alpha) \mid g_i \in G,\,\alpha \in S_n\}$ with action defined by: for $(a_1,\dots,a_n)\in A^n$, \begin{eqnarray} (a_1,\dots,a_n)^{(g_1,\dots,g_n,\alpha)} = (a_{\alpha^{-1}(1)}^{g_1},\dots,a_{\alpha^{-1}(n)}^{g_n}) \end{eqnarray} Let $B_n(A) = Sym(A^n)$ and $B(A) = \bigcup_{n\in \N} B_n(A)$. We call $B_n(A)$ the set of \emph{$n$-ary reversible gates} on $A$, $B(A)$ the set of \emph{reversible gates}. For $\alpha \in S_n$, let $\pi_\alpha \in B_n(A)$ be defined by $\pi_\alpha(x_1,\dots,x_n) = (x_{\alpha^{-1}(1)},\dots,x_{\alpha^{-1}(n)})$. We call this a \emph{wire permutation}. Let $\Pi=\{\pi_\alpha \mid \alpha \in S_n,\,n\in\N\}$. In the case that $\alpha$ is the identity, we write $i_n=\pi_\alpha$, the $n$-ary identity. Let $f\in B_n(A)$, $g\in B_m(A)$. Note that we can write $f$ as $(f_1,\dots,f_n)$ with each $f_i:A^n\rightarrow A$, calling $f_i$ the \emph{components} of $f$. Define the \emph{parallel composition} as $f\oplus g \in B_{n+m}(A)$ with \begin{align*} (f\oplus g)(x_1,\dots,x_{n+m}) &= (f_1(x_1,\dots,x_n),\dots,f_n(x_1,\dots,x_n),\\ &\hspace{10mm}g_1(x_{n+1},\dots,x_{n+m}),\dots \dots,g_m(x_{n+1},\dots,x_{n+m})). \end{align*} For $f,g\in B_n(A)$ we can compose $f\bullet g$ in $Sym(A^n)$. If $f,g$ have distinct arities we ``pad'' them with identity. Let $f\in B_n(A)$ and $g\in B_m(A)$, $n< m$, and define \begin{eqnarray*} f\bullet g = (f \oplus i_{m-n}) \bullet g \end{eqnarray*} Let $f\in B_n(A)$ and $g\in B_m(A)$, $n> m$, and define \begin{eqnarray*} f\bullet g =f \bullet (g \oplus i_{n-m}) \end{eqnarray*} We can thus serially compose all elements of $B(A)$. We call a subset $C \subseteq B(A)$ that includes $\Pi$ and is closed under $\oplus$ and $\bullet$ a \emph{permutation clone} \cite{jerabek18}. These have also been investigated with ideas from category theory \cite{lafont93} and as \emph{memoryless computation} \cite{mgpc14,mgsr15}. If we do not insist upon the inclusion of $\Pi$, then we have \emph{reversible iterative algebras} \cite{bks17}. If $C,D$ are permutation clones, with $C\subseteq D$, then we call $C$ a \emph{sub permutation clone} of $D$. For a set $F \subseteq B(A)$ we write $\langle F \rangle$ as the smallest permutation clone that includes $F$, the permutation clone \emph{generated} by $F$. Let $C$ be a permutation clone. We write $C^{[n]} = C \cap B_n(A)$ for the elements of $C$ of arity $n$. We will occasionally write $(a_1,\dots,a_n)\in A^n$ as $a_1a_2\dots a_n$ for brevity. We observe that $C$ is a $\N$-indexed collection of permutation groups, $C^{[i]} \leq Sym(A^i)$, where the index corresponds to the gate arity. A permutation clone $C$ can be seen as an inverse monoid $(C, \bullet, i_1)$, with $\{i_n\mid n \in \N\}$ the set of idempotents. In any permutation clone $C$, the unary part $C^{[1]}$ is found as a wreath product in all other parts, $C^{[1]} wr S_n \leq C^{[n]}$ because the wire permutations give us the right hand factor while $f_1\oplus\dots\oplus f_n$ for $f_i \in C^{[1]}$ gives us the left hand factor. \begin{example} Let $q$ be a prime power, $GF(q)$ the field of order $q$, $AGL_n(q)$ the collection of affine invertible maps of $GF(q)^n$ to itself. We note that for all $m \in \N$, $AGL_n(q^m) \leq AGL_{nm}(q)$ \cite[p. 56]{mortimerdixon}. For a prime $p$, let \begin{eqnarray*}\operatorname{Aff}(p^m) = \bigcup_{n\in \N} AGL_{nm}(p) \end{eqnarray*} be the permutation clone of affine maps over $A=GF(p)^m$. \end{example} \begin{example} Let $A$ be a set. Let $\beta_1,\dots,\beta_n \in S_A$, so $(\beta_1,\dots,\beta_n) =\beta_1\oplus\dots\oplus\beta_n\in B_n(A)$. Let \begin{eqnarray*} \Deg(A)=\{\pi_\alpha \bullet (\beta_1,\dots,\beta_n) \mid n\in \N,\, \alpha\in S_n,\, \beta_i\in S_A\} = \bigcup_n S_A wr S_n \end{eqnarray*} be the collection of \emph{essentially unary} or \emph{degenerate} maps. \end{example} \begin{example} Let $A$ be a set, let $o\in A$ be some arbitrary element. Let \begin{eqnarray*} P_o(A)=\{f\in B(A) \mid f(o,\dots,o)=(o,\dots,o)\} \end{eqnarray*} be the collection of \emph{$o$-preserving} maps. \end{example} We say that a permutation clone $C \leq B(A)$ is \emph{borrow closed} if for all $f\in B(A)$, $f\oplus i_1 \in C$ implies that $f\in C$. We say that a permutation clone $C \leq B(A)$ is \emph{ancilla closed} if for all $f\in B_n(A)$, $g \in C^{[n+1]}$ with some $a\in A$ such that for all $x_1,\dots,x_n \in A$, for all $i\in\{1,\dots,n\}$, $f_i(x_1,\dots,x_n) = g_i(x_1,\dots,x_n,a)$ and $g_{n+1}(x_1,\dots,x_n,a)=a$ implies that $f\in C$. If a permutation clone is ancilla closed then it is borrow closed. For any prime power $q$, $\operatorname{Aff}(q)$ is borrow and ancilla closed. For any set $A$, $\Deg(A)$ is borrow and ancilla closed. For all $o\in A$, $P_o(A)$ is borrow but not ancilla closed. \begin{lemma} \label{lemma_borrowclosed} A permutation clone $C$ is borrow closed iff if $f,f\oplus g \in C$ then $g\in C$. \end{lemma} \begin{proof} Let $g$ have arity $n$, $f$ have arity $m$. Suppose $C$ is borrow closed. Since $f\in C$ we know that $f^{-1} \in C$, so $f^{-1}\oplus i_n \in C$. Then $(f^{-1}\oplus i_n)\bullet(f \oplus g) = i_m\oplus g \in C$. By borrow closure, then $g\in C$ and we are done. For the other direction we always have that $f=i_1\in C$ and this is the definition of borrow closed. \qed \end{proof} Let $F\subseteq \OO(A)$ be a set of maps on $A$. Define $PC(F) = \{f \in B(A) \mid \forall i \in 1,\dots,ar(f), \, f_i \in F\}$ as the set of permutations with component functions in $F$. We can ask ourselves, when is $PC(F)$ a permutation clone? From \cite{jerabek18} we know that permutation clones are defined as dual to collections of weight maps, similar to the duality by relations for clones. \begin{definition} Let $k\in \N$, let $(M,\cdot)$ be a commutative monoid. A \emph{weight map} is some map $w:A^k\rightarrow M$. Let $f \in B_n(A)$. Then $f$ \emph{respects} $w$, $f \triangleright w$, if for every $a \in A^{k\times n}$, \begin{eqnarray} \prod_{i=1}^{n} w(a_{1i},\dots,a_{ki}) = \prod_{i=1}^{n} w(f(a_{11},\dots,a_{1n})_{i},\dots,f(a_{k1},\dots,a_{kn})_{i}). \label{eqn_weight} \end{eqnarray} Then $\Pol(w) = \{f \in B(A) \mid f \triangleright w\}$ is then the set of \emph{polymorphisms} of the weight $w$. \end{definition} Such arrays will be written in square brackets, so $a$ above would be \begin{equation} a=\left[ \begin{array}{ccccc} a_{11} & \dots & a_{1n} \\ \vdots & & \vdots \\ a_{k1} & \dots & a_{kn} \end{array} \right] \end{equation} We will often use shorthand, for instance if $a,b\in A^n$, then $\left[ \begin{array}{c} a \\ b \end{array} \right] \in A^{2 \times n}$. For simplicity we will write $w(a)$ for $\prod_{i=1}^{n} w(a_{1i},\dots,a_{ki})$ and \begin{eqnarray*} f(a)= \left[ \begin{array}{c} f(a_{11},\dots,a_{1n}) \\ \vdots \\ f(a_{k1},\dots,a_{kn}) \end{array} \right] \end{eqnarray*} for the array obtained by applying $f$ to each row of $a$. Then the equation (\ref{eqn_weight}) above becomes $w(a) = w(f(a))$. \begin{definition} Let $c \in A^m$ and $f\in B_n(A)$. The \emph{controlled permutation} $CP(c,f)\in B_{m+n}(A)$ applies $f$ to the last $n$ entries of its argument if the first $m$ entries match $c$. \end{definition} On the set $\{0,1\}$ the controlled permutation $CP(1,(01\;10))$ is a ternary map that swaps the second and third arguments if the first argument is a 1. It is called the \emph{Fredkin gate} \cite{fredkintoffoli82} and has an important role in reversible computation. \begin{example} Let $A=\{0,1\}$, $w:{0,1} \rightarrow (\N_0,+)$, $w(0)=0$, $w(1)=1$. Then $w$ counts the number of $1$s in a tuple, so $f \triangleright w$ iff $f$ conserves the number of $1$s, i.e. the weight of the tuple. \end{example} Such gates are called \emph{conservative}, the Fredkin gate is an example. Let $(\B,\wedge)$ be the two element monoid on $\{0,1\}$ with logical and operation $\wedge$. \begin{example} Let $(A,\leq)$ be a partially ordered set, $w:A^2 \rightarrow (\B,\wedge)$, $w_\leq(a,b)=1$ if $a\leq b$ and $w_\leq(a,b)=0$ otherwise. Then for $f \in B_n(A)$, $f \triangleright w_\leq$ iff $a\leq b \Rightarrow f(a)\leq f(b)$ iff $f$ is a monotone map, i.e.\ an endomorphism and thus automorphism of $(A^n,\leq)$. \end{example} \begin{example} Let $(A,+)$ be an abelian group, $w:A^3 \rightarrow (\B,\wedge)$, $w(a,b,c)=1$ if $a+b=c$ and $w(a,b,c)=0$ otherwise. Then for $f \in B_n(A)$, $f \triangleright w$ iff $f$ is a linear map, i.e.\ an automorphism of $(A,+)$. \end{example} \begin{example} Let $(A,+)$ be an abelian group, $w:A^4 \rightarrow (\B,\wedge)$, $w(a,b,c,d)=1$ if $a+b=c+d$ and $w(a,b,c,d)=0$ otherwise. Then for $f \in B_n(A)$, $f \triangleright w$ iff $f$ is an affine permutation, i.e.\ the sum of an automorphism of $A$ and a constant map. \end{example} In particular, if $A=\Z_p$ for a prime $p$, then $\Pol(w) = \Aff(A)$. \begin{definition} \label{exampleRelationWeight} Let $R \subset A^k$ be a relation. Let $w_R:A^k\rightarrow (\B,\wedge)$ with $w_R(a)$ true iff $a\in R$. Then $f\triangleright w_R$ iff every component $f_i$ respects the relation $R$, i.e.\ $f_i\triangleright R$ in the clone sense. \end{definition} We call such polymorphisms \emph{relationally defined}. This type of weight will be the main concern of this paper. Then $PC(\Pol(R \mid R \in \RR))$ $= \Pol(w_R\mid R \in \RR)$. We note that in order to check whether $f\in \Pol(w_R)$ we need only check for matrices with columns in $R$, that is, for matrices $M$ such that $w_R(M)=1$ and $w_R(f(M))=1$. By reversibility, the other matrices all map to $0$. We note that we can look at the individual arities of a relationally defined permutation clone as the automorphism group of a relational structure. \begin{lemma} \label{lemma_AutR} Let $\A=(A,R)$, $R \subseteq A^k$ be a relational structure. Let $\A^n=(A^n,R^n)$ where $R^n= \{ (a_1,\dots,a_k) \mid a_i\in A^n,\, (a_{i1},\dots,a_{ik})\in R\; \forall i\in \{1,\dots,n\}\}$. Then $\Pol(w_R)^{[n]} = Aut(\A^n)$. \end{lemma} \begin{proof} \begin{eqnarray*} f \in \Pol(w_R)^{[n]} & \Leftrightarrow & f\in B_n(A),\, \forall a\in A^{k\times n},\, \\ && \hspace{10mm}(a_{1i},\dots,a_{k,i})\in R\, \forall i, (f(a)_{1i},\dots,f(a)_{k,i})\in R\, \forall i \\ &\Leftrightarrow & \forall a \in \A^n,\, f(a)\in \A^n \\ &\Leftrightarrow & f \in \Aut(\A^n) \end{eqnarray*} \qed \end{proof} This also reminds us that $f\triangleright w_R$ iff every array with columns in $R$ is mapped by $f$ to an array with columns in $R$, and every array with at least one column not in $R$ is mapped to an array with at least one column not in $R$. An algebraic structure $(S,+,*,0,1)$ is called a \emph{(commutative) semiring} if $(S,+,0)$ is a commutative monoid, $(S,*,1)$ is a commutative monoid and the two distribution laws hold, i.e.\ for all $a,b,c \in S$ \begin{eqnarray} a*(b+c) &= a*b + a* c \\ (a+b)*c &= a*c+b*c \end{eqnarray} Examples include commutative rings, fields, bounded lattices and $(S,+,+,0,0)$ when $(S,+,0)$ is a join semilattice with $0$. Je\v{r}\'{a}bek defines a closure process for sets of weights \cite[p. 11]{jerabek18}. \begin{definition} \label{defn_coclone} A set $D$ of weight maps is \emph{closed} if: \begin{enumerate} \item If $w:A^k\rightarrow M$ is in $D$, and $\rho:\{1,\dots,k\}\rightarrow \{1,\dots,l\}$, then the weight $w\circ \rho:A^l\rightarrow M$ is in $D$, where $w\circ \rho: (x_1,\dots,x_l) \mapsto w(x_{\rho(1)},\dots,x_{\rho(k)})$. \item If $w:A^k\rightarrow M$ is in $D$, and $\phi:M\rightarrow N$ is a commutative monoid homomorphism, then $\phi\circ w_k:A^k\rightarrow N$ is in $D$. \item If $w:A^k\rightarrow M$ is in $D$, and $N\leq M$ is a submonoid with $w(A^k) \subseteq N$, then $w:A^k\rightarrow N$ is in $D$. \item If $w_i:A^k\rightarrow M_i$ is in $D$ for all $i\in I$, then $w: A^k \rightarrow \prod_{i \in I} M_i$ is in $D$. \item The weight $c_1:A \rightarrow (\N_0,+)$ with $c_1(a)=1$ for all $a\in A$ is in $D$. \item The weight $\delta:A^2 \rightarrow (\B,\wedge)$ with $\delta(a,b)$ true iff $a=b$ is in $D$. \item If $w:A^k\rightarrow (M,\cdot)$ is in $D$, $k\geq 2$, and $(M,\boxplus,\cdot)$ is a semiring, then $w^\boxplus$ is in $D$, where $w^\boxplus:A^{k-1}\rightarrow M$, \begin{align*} w^\boxplus(a_1,\dots,a_{k-1}) = \underset{a\in A}\bigboxplus w(a_1,\dots,a_{k-1},a) \end{align*} \end{enumerate} \end{definition} Je\v{r}\'{a}bek calls \cite[Defn 5.17]{jerabek18} a set of weight maps that are closed in this way a \emph{permutation co-clone}. The important result is then the following, telling us that every permutation clone is defined by a collection of weights. \begin{theorem}[\protect{\cite[Thm 5.18]{jerabek18}}] \label{theoremJerabek} Let $C$ be a permutation clone. Then there exists a permutation co-clone $D$ such that for all $f \in B(A)$, $f \in C$ iff $f \triangleright w$ for all $w \in D$. \end{theorem} That is, a permutation clone is defined precisely by the permutation co-clone that it respects. We write $\Inv(C)$, the \emph {invariants} of $C$, for the collection of weight maps that are respected by a set of bijections $C \subseteq B(A)$ and $\Pol(D) \subseteq B(A)$, the \emph{polymorphisms} that respect $D$, for the set of bijections that respect a collection of weight maps $D$. $D$ is closed iff $\Inv(\Pol(D))=D$ and $C$ is a permutation clone iff $\Pol(\Inv(C))=C$. If $w$ is a weight mapping, we write $\Pol(w)$ to mean $\Pol(\langle w \rangle)$ where $\langle w \rangle$ is the permutation co-clone generated by $w$. $\Pol$ and $\Inv$ By duality, a maximal permutation clone is defined by a minimal co-clone, which is generated by one weight. In order to reduce confusion we will write $\Pol(R)$ to mean the clone of mappings that respect the relation $R$, $\Pol(w_R)$ to mean the permutation clone of bijections that respect the relation $R$ encoded as a weight function $w_R$, and $\PPol(R)=\Pol(w_R)$ (for permutation polymorphism) as shorthand, especially when talking about multiple relations. Thus $\PPol(R,S,T) = \Pol(w_R) \cap \Pol(w_S) \cap \Pol(w_T)$. It is still unknown what precise form of weight is necessary in order to define ancilla and borrow closed permutation clones. Some sufficient conditions (for example, cancellative monoids) are known (see below), but they are not necessary \cite[Section 5.2]{jerabek18}. \begin{theorem} Let $w:A^k\rightarrow (M,\cdot)$ be a weight map, let $a\in A^k$ be such that $w(a)$ is cancellative in $M$. Then $Pol(w)$ is borrow closed. \end{theorem} \begin{proof} Suppose $f\oplus i_1 \in Pol(w)$, $f\in B_n(A)$. Let $b \in A^{k\times n}$. Let $d\in A^{k\times (n+1)}$ be the matrix consisting of $b$ with the $n+1$-th column being all $a$, that is, $d=b \oplus a$. Then because $f\oplus i_1 \in Pol(w)$, $w(d) = w((f\oplus i_1)(d))$, thus $w(b) \cdot w(a) = w((f\oplus i_1)(d))=w(f(b)\oplus a)= w(f(b))\cdot w(a)$. Now because $w(a)$ is cancellative, this implies that $w(b)=w(f(b))$ so $f \triangleright w$ and thus $f\in Pol(w).$ Thus $Pol(w)$ is borrow closed and we are done. \qed \end{proof} There is a similar result \cite{jerabek18} for ancilla closure. \begin{theorem} \label{thm_ancillaclosed} Let $w:A^k\rightarrow (M,+)$ be a weight map, such that $w(a,\dots,a)$ is cancellative in $M$ for all $a\in A$. Then $Pol(w)$ is ancilla closed. \end{theorem} These conditions are sufficient but not neccesary. The following example shows this for borrow closure. \begin{example} Let $A=\{0,1\}$, let $w:A\rightarrow \B^2$, $w(0)=(1,0)$ and $w(1)=(0,1)$. Neither of these two images is cancellative, but $\Pol(w)$ is borrow closed. This can be seen because the weight $v:A^2\rightarrow \B^2$ defined by \begin{equation*} \begin{array}{|c|c|} \hline 00 & 10 \\ 01 & 11 \\ 10 & 00 \\ 11 & 01 \\ \hline \end{array} \end{equation*} is in the permutation co-clone defined by $w$, as is the weight $\bar v:A^2\rightarrow \B$ with $\bar v (a)= 1 $ iff $a=01$. It can also be seen that $w$ is in the permutation co-clone defined by $v$ and $\bar v$. Both $v$ and $\bar v$ have cancellative images, showing that $\Pol(v)$ is borrow closed. This is the set of permutations that fix the zero vector and the vector of all 1, i.e.\ $f\in \Pol(w)$ iff $f(0\dots 0) = 0\dots 0$ and $f(1\dots 1) = 1\dots 1$. \end{example} \section{Some basic properties} In this section we will look at some properties of the collection of relationally defined permutation clones. Firstly we will show that all permutation co-clones contain a relational part that is closed as a co-clone, thus permutation co-clone closure is stronger than relational co-clone closure. Then we will examine relational clone closure and show that maximal borrow closed permutation clones are either relationally defined or have cancellative weights. \begin{lemma} \label{lemma_relCloneClosure} Let $\RR$ be a collection of relations on $A$. Let $Wt(\RR) = \{w_R \mid R \in \RR\}$ be the collection of weights obtained from $\RR$. Let $D$ be a permutation coclone. Let $Rl(D) = \{R \mid w_R \in D\}$ be the collection of relations which have characteristic functions in $D$. Then $\langle \RR \rangle_{rc} \subset Rl(\langle Wt(\RR)\rangle_{pcc}$ where $rc$ means relation clone (coclone) closure and $pcc$ means permutation coclone closure. Moreover $Rl(\langle Wt(\RR)\rangle_{pcc})$ is a coclone. \end{lemma} \begin{proof} We show that each of the coclone closure operations can be obtained from the permutation coclone closure operations. The permutation of entries follows from the first closure operation. Projection can be obtained by using the semigroup closure with the semigroup $(\B,\vee,\wedge)$. Let $R\subseteq A^k,\,S\subset A^l$ be two relations. Then $w_R$ and $w_S$ are two characteristic functions for these relations. Let $p_1:\{1,\dots,k\} \rightarrow \{1,\dots,k+l\}$ be defined by $p_1:i\mapsto i$ and $p_2:\{1,\dots,l\} \rightarrow \{1,\dots,k+l\}$ be defined by $p_2:i\mapsto k+i$. Then $w_R^{p_1}:A^{k+l} \rightarrow \B$ and $w_S^{p_2}:A^{k+l} \rightarrow \B$ are in the permutation coclone. We then create the product $(w_R^{p_1},w_S^{p_2}):A^{k+l} \rightarrow \B\times \B$ that is in the permutation coclone, then apply the homomorphism $\wedge: \B\times \B\rightarrow \B$ to obtain a characteristic function of the cartesian product of $R$ and $S$. Let $R,S \subseteq A^k$. Let $w_R$ and $w_S$ be the characteristic functions for these relations. Then $(w_R,w_S): A^k \rightarrow \B \times \B$ is in the permutation coclone, we apply the homomorphism $\wedge: \B\times \B\rightarrow \B$ to obtain a characteristic function of the intersection $R \cap S$. The equality relation characteristic function $\delta$ is in the permutation coclone. Let $p:\{1,2\}\rightarrow \{1,2,3\}$ be the identity injection, so $\delta \circ p$ is the characteristic function of the equality relation is in a co-clone. \qed \end{proof} Thus the lattice of relationally defined permutation clones will be a homomorphic image of the lattice of clones. Relationally defined permutation clones have a stronger closure property. \begin{lemma} Every relationally defined permutation clone is borrow closed. \end{lemma} \begin{proof} Let $w_R$ be the weight function from a relation $R$. If $R$ is non trivial, then $w_R(a)=1$ for at least one $a$. $1$ is cancellative as it is an identity. Thus $Pol(w_R)$ is borrow closed. \qed \end{proof} Relationally defined permutation clones have a stronger factoring property than borrow closure. \begin{lemma} Let $C$ be a relationally defined permutation clone. If $f \oplus g \in C$, then $f,g\in C$. \end{lemma} \begin{proof} Let $C =\PPol(\rho)$ be our relationally defined permutation clone. Let $f$ have arity $n$ and $g$ have arity $m$. Let $a,b$ be two appropriately sized arrays of elements from $A$ such that every column is in the relation defining $C$. Apply $f \oplus g$ to $a\oplus b$ and the columns of $(f\oplus g)(a\oplus b)$ are all in the relation defining $C$. Thus since $a$ had columns in $\rho$ and the first $n$ columns of $(f\oplus g)(a\oplus b)$ are all in $\rho$ then $f(a)$ has columns in $\rho$ so $f\triangleright \rho$ and $f \in C$. Similarly $g \in C$. \qed \end{proof} We can determine the relations that give us the full permutation clone. \begin{lemma} \label{lemma_relationFull} Let $A$ be a finite set. Let $\emptyset \neq R\subseteq A^k$ be a relation with $\PPol(R)=B(A)$. Then there exists an equivalence relation $E$ such that \[R = \{a\in A^k \mid a_i=a_j,\,\forall i,j: (i,j)\in E\}.\] \end{lemma} \begin{proof} Let $k=1$, so $R\subseteq A$. Let $f$ be the cycle that rotates all the elements of $A$. Then $f\triangleright w_R$ means that $f(R)=R$ so $R=A$. We proceed by induction. Assume the result is true for $k-1$. Suppose there exist $a_1,\dots,a_{k-1}\in R$ such that for all $i$, $(a_i)_i \neq (a_i)_{i+1}$. Then the $(k-1) \times k$ array $\left[ a_1\dots a_{k-1}\right]$ has $k$ distinct rows, so it can be mapped to some $(k-1) \times k$ array $b$ such that the first column $b_1$ of $b$ is arbitrary. Thus $b_1\in R$ so $R=A^k$ and we are done. If such a set of $a_i$ do not exist, assume without loss of generality that for all $a\in R$, $a_{k-1}=a_k$. Let $w_R$ be the characteristic weight of $R$, let $(\B,\vee,\wedge)$ be a semiring and note that $w^\vee_R(a_1,\dots,a_{k-1})=1$ iff $w_R(a_1,\dots,a_{k-1},a_{k-1})=1$. From $w^\vee_R$ we can construct the weight $v:A^k\rightarrow (\b,\wedge)^2$ such that $v( a_1,\dots,a_{k})=(w^\vee_R(a_1,\dots,a_{k-1}, \delta(a_{k-1},a_{k})$. Applying the AND homomorphism from $\B^2$ to $\B$ to $v$, we find that $\wedge \circ v( a_1,\dots,a_{k})=w_R( a_1,\dots,a_{k})$, so $w^\vee_R$ and $w_R$ define the same permutation clone. Then define $R_1\subset A^{k-1}$ such that $w_{R_1}=w^\vee_R$,. Apply the induction hypothesis to find an equivalence relation $E_1$ on $\{1,\dots,k-1\}$. Extend $E_1$ by $(k-1,k)$ to obtain an equivalence relation $E$ and we have proven our result. \qed \end{proof} A \emph{subelementary} monoid $S=C\cup N$ is a disjoint union of a cancellative monoid $C$ and a nil semigroup $N$. Note that $N$ is an ideal of $S$. The two element boolean algebra $(\B,\wedge)$ is the simplest subelementary monoid; a union of the one element cancellative monoid $\{1\}$ and the one element nil semigroup $\{0\}$. For any subelementary monoid $S$, the map $h:S\rightarrow \B$ with $h(c)=1$ for all $c \in C$ and $h(n)=0$ for all $n \in N$ is a homomorphism. Thus $\B$ is the only simple subelementary monoid. It is known (e.g.\ \cite{grillet2001}) that a finitely generated commutative monoid is a subdirect product of a cancellative and a direct product of subelementary monoids, i.e. $C \times \prod_i S_i$. We obtain the following result immediately. \begin{lemma} A meet irreducible permutation clone is defined by a cancellative or subelementary monoid weight. \end{lemma} For instance the conservative permutation clones are of this form, as are the orthogonal permutation clones over the field $(\Z_p,+,\cdot)$ by the weight $w_O:\Z_p^2\rightarrow (\Z_p,+)$ with $w_O(x,y)=x\cdot y$. The following shows that for borrow closure (and thus for ancilla closure) we can annihilate the nil part of a subelementary monoid and still define the same permutation clone. \begin{lemma} \label{lemma_subelementaryNil} Let $w:A^k\rightarrow C \cup N$ be a subelementary weight. Let $\Pol(w)$ be borrow closed. Let $\phi: C \cup N \rightarrow C \cup \{0\}$ be the natural homomorphism annihilating the nil part. Then $\Pol(w)=\Pol(\phi \circ w)$. \end{lemma} \begin{proof} Suppose this is not true, so there is some $f\in B_m(A)$ with $f \triangleright \phi \circ w$ and $f \not\triangleright w$. Let $a\in A^{k \times m}$ be the contradicting array, so $\phi \circ w(f(a))=\phi \circ w(a)$ but $w(f(a))\neq w(a)$. Then $w(f(a)),\, w(a)\in N$. Let $b\in A^k$ such that $w(b)\in N$, let $h\in \N$ be the nil degree of $N$, so $\forall n \in N$, $n^h=0$. Let $\bar a \in A^{k \times (m+h)}$ be the array $a$ with $h$ copies of $b$ adjoined to the right. Then $w(\bar a) = w(a)\cdot w(b)^h=0$ and $w((f\oplus i_h)(\bar a)) = w(f(a) \cdot w(b)^h=0$ so $(f\oplus i_h)\triangleright w$, so $(f\oplus i_h) \in \Pol(w)$. But $\Pol(w)$ is borrow closed, so this implies that $f\in \Pol(w)$ and we are done. \end{proof} The following shows that maximal borrow closed permutation clones are defined either by cancellative weights or relational weights. \begin{lemma} \label{lemma_maximalSubEl} Let $w:A^k\rightarrow (C \cup N,\cdot)$ be a nontrivially subelementary weight, so $w(A^k)\cap N \neq \emptyset$ and $w(A^k)\cap C \neq \emptyset$. Let $\Pol(w)$ be borrow closed and maximal. Let $\phi: C \cup N \rightarrow \{0,1\}$ be the natural homomorphism annihilating the nil part and mapping the cancellative part to $1$. Then $\Pol(w)=\Pol(\phi \circ w)$. \end{lemma} \begin{proof} We note that by Lemma \ref{lemma_subelementaryNil} we can take $N=\{0\}$ to be trivial. Let $1\in C$ be the identity. We proceed by induction. Let $k=1$. Let $R\subseteq A$ such that $w_R=\phi \circ w$. By the nontriviality of the subelementary weight, $R$ is nontrivial, and $\PPol(R)$ is a maximal borrow closed permutation clone, so $\Pol(w)=\Pol(\phi \circ w)$. Suppose this is not true for some $k>1$, but true for $k-1$. Then $\Pol(w)\neq\Pol(\phi \circ w)$ so $\Pol(\phi \circ w)=B(A)$ by maximality. From Lemma \ref{lemma_relationFull} above we know that the relation $R$ such that $w_R=\phi \circ w$ is defined by an equivalence relation $E$. If $E$ is the equality relation, then $R=A^k$ and $w(A^k)\subseteq C$, a contradiction. Without loss of generality, let $(k-1,k) \in E$. Thus $w(a_1,\dots,a_k)\in C$ implies that $a_{k-1}=a_k$. We define a binary operation $+$ on $C\cup N$ such that $(C\cup N,+,\cdot)$ is a semiring. For all $c\in C$, $c+0=0+c=c$, for $c,d\in C$ let $c+d=0$. Then because $w^+$ has only one nonzero summand, $w^+(a_1,\dots,a_{k-1})=w(a_1,\dots,a_{k-1},a_{k-1})$. Now let $w^+ \times \delta: A^k \rightarrow (C \cup N) \times \{0,1\}$ be defined by \begin{eqnarray*} w^+ \times \delta (a_1,\dots,a_k) = (w^+(a_1,\dots,a_{k-1}), \delta(a_{k-1},a_k)). \end{eqnarray*} Then $\psi:(C \cup N) \times \{0,1\} \rightarrow (C \cup N) $ defined by $\psi(c,1)=c$ and $\psi(c,0)=0$ is a monoid homomorphism. Then $\psi \circ (w^+ \times \delta)=w$ so $w$ and $w^+$ define the same permutation clone. By the induction hypothesis, $\Pol(w^+)=\Pol(\phi \circ w^+)$. We now define the relation $R^+\subseteq A^{k-1}$ such that $w_{R^+}= \phi \circ w^+$. Then \begin{eqnarray*} R= \{(a_1,\dots,a_k) \mid (a_1,\dots,a_{k-1}) \in R^+ \wedge a_{k-1}=a_k\}, \end{eqnarray*} so $R$ is in the relational clone defined by $R^+$. Similarly \begin{eqnarray*} R^+= \{(a_1,\dots,a_{k-1}) \mid (a_1,\dots,a_{k-1},a_{k-1}) \in R\} \end{eqnarray*} so $R$ and $R^+$ define the same relational clone. Thus $\Pol(w)=\Pol(w^+)=\Pol(\phi \circ w^+)=\Pol(\phi \circ w)$ and we are done. \qed \end{proof} Thus maximal borrow closed permutation clones are defined by a cancellative weight or a relational weight. By Lemma \ref{lemma_relCloneClosure} we know that all maximal relationally defined permutation clones must be defined by relations corresponding to maximal clones, so by Rosenberg's result we know them all. We will investigate these in the next section. The following example shows that Lemma \ref{lemma_maximalSubEl} does not apply in general. \begin{example} \label{example_three} Let $M=\N_0\cup \{\infty\}$ be the subelementary monoid natural numbers with absorbing element $\infty$ adjoined. Let $A=\{0,1,2\}$ and $w:A\rightarrow M$ have $w(0)=0$, $w(1)=1$ and $w(2)=\infty$. Then $\Pol(w)$ will act as $\Cons(\{0,1\})$ on the set $\{0,1\}$. Let $\phi: \N_0\cup \{\infty\} \rightarrow \{0,1\}$ be the natural homomorphism mapping the cancellative part $\N_0$ to $1$ and $\infty$ to $0$. Then $\Pol(\phi \circ w)$ is $\PPol(\{0,1\})$, that is, the collection of permutations that respect the subset $\{0,1\}$. We find that $\Pol(w) \subsetneq \Pol(\phi \circ w) \subsetneq B(A)$. \end{example} \section{Learning from clones} As clone theory is well developed, there is a wealth of results that can be utilised for investigating the relationally defined permutation clones. We will see some of this here but there is much more. As a guiding principle we will use Rosenberg's classification of maximal clones, as all relational clones lie below one or more maximal clones, and thus all relationally defined permutation clones lie below one or more of the permutation clines defined by the Rosenberg relations. First we will look at some tools, then we will look at the Rosenberg relations in more detail. The property of being defined by a relation is not always clear. There are permutation clones that can be defined by relations as well as by cancellative weights, such as the degenerate or essentially unary maps. \begin{definition} Let $m,n\in \N$. The Hamming graph $H(m,n)$ has node set $\{1,\dots,m\}^n$ with two nodes adjacent if they differ at exactly one position. Equivalently it is the direct product of $n$ copies of the complete graph $K_m$. \end{definition} \begin{theorem}[\cite{mifzal_zaiee_2021,praegerSchneider}] \label{theoremHamming} The automorphism group $\Aut(H(n,m))$ of the Hamming graph is $S_m wr S_n$. \end{theorem} \begin{lemma} Let $R=\{(a,b,c)\mid a=b \bigvee b=c\} \subseteq A^3$. Let $w:A^2\rightarrow (\N_0,+)$ be defined by $w(a,a)=1$ and $w(a,b)=0$ for $a \neq b$. Then $\Deg(A)=\Pol(w_R)=\Pol(w)$. \end{lemma} \begin{proof} ($\Deg(A)\subseteq \Pol(w_R)$): Let $f\in \Deg(A)$, $f = \pi_\alpha \bullet (g_1,\dots,g_n)$ for some $g_i\in S_A$. Let $w_R(a)=1$ so each column of $a$ is in $R$. If $a_{1i}=a_{2i}$ then $f(a)_{1i^\alpha}=f(a)_{2i^\alpha}$ and similarly if $a_{2i}=a_{3i}$ then $f(a)_{2i^\alpha}=f(a)_{2i^\alpha}$ so every column of $f(a)$ is in $R$ and thus $w_R(f(a))=1$. ($\Pol(w_R) \subseteq \Deg(A) )$: Suppose $f\not\in Deg(A)^{[n]}$. Without loss of generality, let $f_1$ depend upon inputs 1 and 2, so there exists some $a_1,\dots, a_n,b_1,b_2\in A$ such that \begin{eqnarray} f_1(a_1,\dots,a_n) &\neq& f_1(b_1,a_2,\dots,a_n) \\ f_1(a_1,\dots,a_n) &\neq& f_1(a_1,b_2,a_3,\dots,a_n) \end{eqnarray} Then the matrix \[ a=\left[ \begin{array}{ccccc} b_1 & a_2 & a_3 &\dots & a_n \\ a_1 & a_2 & a_3 &\dots & a_n \\ a_1 & b_2 & a_3 & \dots & a_n \end{array} \right] \] has every column in $R$, but the first column of $f(a)$ is not in $R$, so $f \not\in \Pol(w_R)$. ($\Pol(w) = \Deg(A) )$): Fix $n\in \N$. Define a graph on $A^n$ with edges $(a,b)$ iff the matrix with rows $a$ and $b$ has weight $n-1$. This occurs iff $a$ and $b$ differ in exactly one position. Thus this is the $n$-Hamming graph on the alphabet $A$. Any $f\in \Pol(w)$ must map matrices of weight $n-1$ to each other, so it preserves the edges of this graph. From Theorem\ref{theoremHamming} we know that the automorphism group of the Hamming graph is the wreath product $S_A wr S_n$ which is precisely $Deg(A)$ so we are done. \qed \end{proof} Thus while $Deg(A)$ can be defined by a cancellative weight, it can also be defined by a relational weight. We cannot yet define the necessary conditions on a relation $R$ such that $\Pol(w_R)=\Pi$, but the understanding is that imbalances in the relation completion possibilities will force this, such as that used in the previous lemma. The first consideration is that all component maps $f_i$ for some $f\in C^{[n]}$ for some permutation clone are balanced \cite{toffoli80}. A map $g:A^n\rightarrow A$ is \emph{balanced} if the preimage $\vert g^{-1}(a)\vert = \vert A\vert^{n-1}$ for all $a\in A$. It is easy to see that the component maps of wire permutations are balanced. Let $C$ be a clone, let $\beta(C)$ be the collection of bijections that can be created from the maps in $C$. Then $\beta$ induces an equivalence relation of clones. Two clones will be $\beta$-equivalent if they contain the same balanced maps. We will see below that some balanced maps do not occur as a component map of a bijection. There are some manipulations that determine permutation clones that include the permutation clone of interest, including the homomorphism closure and semiring closure from Definition \ref {defn_coclone} above. The following is clear. \begin{lemma} Let $\rho_1,\rho_2 \subseteq A^k$ be two relations. Then $\PPol(\rho_1,\rho_2) \subseteq \PPol(\rho_1 \cap \rho_2)$. \end{lemma} Let $R \subseteq A^k$ be a relation. For any $i\in\{1,\dots,k\}$ let $c_{R,i}:A^{k-1}\rightarrow (\N,\cdot)$ be defined by \begin{equation} c_{R,i}(a_1,\dots,a_{k-1}) = \vert \{x \mid (a_1,\dots,a_{i-1},x,a_i,\dots,a_{k-1})\in R\} \vert \end{equation} \begin{lemma} \label{lemma_countingCons} For any relation $R \subseteq A^k$, $k\geq 2$, for any $i\in\{1,\dots,k\}$, $Pol(w_R) \subseteq Pol(c_{R,i})$. \end{lemma} \begin{proof} We show that for all $f\in B(A)$, $f\triangleright w_R \Rightarrow f \triangleright c_{R,i}$. Let $f\in B_n(A)$, $R \subseteq A^k$, $k\geq 2$. Suppose $f\triangleright w_R$. Without loss of generality, we will consider $c_{R,k}$. Let $a\in A^{(k-1)\times n}$. For any $b\in A^n$ such that $\left[ \begin{array}{c} a \\ b \end{array} \right] \in R^n$, $\left[ \begin{array}{c} f(a) \\ f(b) \end{array} \right] \in R^n$. Thus $\vert \{ b \mid \left[ \begin{array}{c} a \\ b \end{array} \right] \in R^n\}\vert = \vert \{ b \mid \left[ \begin{array}{c} f(a) \\ b \end{array} \right] \in R^n\}\vert$. But this is simple rewriting that $c_{R,k}(a)=c_{R,k}(f(a))$ which means that $f\triangleright c_{R,k}$, which is what we wanted. \qed \end{proof} There is an alternative proof using the closure results from Theorem \ref{theoremJerabek}. \begin{proof} Note that $\{0,1\} \leq (\N,\cdot,1)$ as a monoid. Treat $w_R:A^k\rightarrow (\N_0,\cdot,1)$ as the weight map. Then one semiring addition on $\N_0$ is the normal addition and \begin{equation*} c_{R,k}(x_1,\dots,x_{k-1})= w_R^+= \sum_{a\in A} w(x_1,\dots,x_{k-1},a) \end{equation*} Thus $c_{R,k} $ is in the permutation coclone defined by $w_R$ so $\Pol(w_R) \leq \Pol(c_{R,k})$. This holds for all indices using the wire permutations, so we are done. \qed \end{proof} Note that $c$ maps to the natural numbers under multiplication. This has a few implications. If $c_{R,k}(a)=0$ for some $a$, this absorbing zero gives an example of a properly subelementary weight. If $c_{R,k}(a)\neq0$ for all $a$, then we obtain something more like the conservative permutation clones defined above, but (for $k\geq 3$) on tuples rather than individual elements of $A$. We will have a number of conservative weights based upon the prime factors of $c_{R,k}(a)$, i.e.\ the prime numbers less that $\vert A \vert$. \begin{example} Let $A=\{0,1,2\}$. Let $\rho=\{01,10,11\} \subset A^2$. Then $c_{\rho,1}=c_{\rho,2}$ with $c_{\rho,1}(0)=1$, $c_{\rho,1}(2)=2$ and $c_{\rho,1}(3)=0$. By the mapping $\infty\mapsto 0$, $n\mapsto 2^n$ we see that this is the same weight that we saw in Example \ref{example_three}. \end{example} An opposite construction, from conservative weights to relational weights, can also be undertaken. \begin{lemma} \label{lemma_maxCons} Let $w:A^k\rightarrow \N_0$ be a weight. Let $m\in \N$ be maximal in $w(A^k)$. Let $\rho=\{a\in A^k\mid w(a)=m\}$. Then $\Pol(w) \subseteq \PPol(\rho)$. \end{lemma} \begin{proof} Suppose $f\in \Pol(w)^{[n]}$. Let $a \in A^{k \times n}$ with all columns taken from $\rho$, i.e.\ $a \in \rho^n$. Then $w(a) = nm$ and since $f\triangleright w$, $w(f(a))=nm$. Thus each column of $f(a)$ has weight $m$ so each column is in $\rho$, so $f\in \PPol(\rho)$. \qed \end{proof} It is clear that the same result holds for the relation defined by the tuples that map to the minimal value in $\N_0$. There is an alternate proof that uses the coclone closure operations. \begin{proof} Let $\psi:\N_0 \rightarrow \N_0$ be the homomorphism $\psi(x)=mx$. Then the weight $w \times \psi\circ c_1:A^k \rightarrow \N_0 \times \N_0$ is in the permutation coclone. Let $\phi: \N_0 \times \N_0 \rightarrow \Q\cup \{\infty\}$ be the homomorphism $\phi(x,y)=x/y$. Then the weight $\phi\circ (w \times \psi\circ c_1)$ is in the permutation coclone. Let $M\subseteq \Q\cup \{\infty\}$ be the image of $\phi\circ (w \times \psi\circ c_1)$. It will consist of $1$ and rational numbers between $0$ and $1$. Let $\zeta: M \rightarrow \B$ be defined by $\zeta(1)=1$ and $\zeta(x) =0$ otherwise. Then $\zeta$ is a homomorphism and $\zeta \circ \phi\circ (w \times \psi\circ c_1)$ is the characteristic function of the relation $\rho$. \end{proof} This proof makes clear that this result applies for any value $m$ that is not the product of other values in $w(A^k)$, i.e.\ for $a_i \in w(A^k)$, $\prod_{i=1}^n a_i = m^n$ iff $a_i=m$ for all $i$. \subsection {Partial Orders} A partial order $\A=(A,\leq)$ is \emph{exponentially decomposable} if $\A = \C^n$ for some partial order $\C=(C,\leq)$. \begin{lemma}[ \cite{Jonsson_1982}] \label{lemma_partial_order} Let $(A,\leq)$ be an exponentially indecomposable partial order. Then $\PPol(\leq)^{[n]}= \Aut ( A^n,\leq) = S_n wr Aut(A)$. \end{lemma} \begin{proof} The first equality follows from Lemma \ref{lemma_AutR}, the second from Jonsson. \qed \end{proof} \begin{example} \label{ex_totalorder} Let $(A,\leq)$ be a total order. Then $\Aut(A)$ is trivial, so $\Aut ( A^n,\leq)$ is the symmetric group on $n$ elements, $S_n$ acting on coordinates, which gives us $\Pi_n$, so $\PPol(\leq)=\Pi$ the trivial permutation clone. \end{example} This is in stark contrast to the situation for clones, where $\Pol(\leq)$ for a bounded partial order is a maximal clone \cite{Rosenberg1970,pinskerthesis2002}. \subsection{Unary Relations} The simplest relations are unary relations $\rho \subseteq A$. \begin{lemma}[\cite{boykett2021}] Let $\rho \subseteq A$. Then $\PPol(\rho)$ is a maximal borrow closed permutation clone. \end{lemma} The following result is clear. \begin{lemma} Let $\rho_1,\rho_2 \subseteq A$. Then $\PPol(\rho_1,\rho_2) \leq \PPol(\rho_1 \cap\rho_2) $. \end{lemma} Thus the collection of sub permutation clones defined by unary relations is isomorphic to the collection of subsets of $\Pow(A)$ that are closed under intersection. The smallest such permutation clone is $\PPol(\Pow(A))$. Each unary relation defined permutation clone is defined by a collection of subsets of $A$ closed under intersection. These are called Moore Collections and can be counted using sequence A102894 of the Online Encyclopedia of Integer Sequences. Chapter 16 of \cite{lau_2006} investigates the case of $\Pol(\{a\} \mid a \in Q)$ for $Q\subseteq A$. In particular we know that the maximal subclones of these clones are intersections with the known maximal clones. It is clear that $\Cons(A) \leq \PPol(\Pow(A))$. \begin{lemma} Let $A$ be of prime order $p \neq 2$. Then $\PPol(\Pow(A)) \cap \Aff(A) = \Pi$ \end{lemma} \begin{proof} Let $f \in \PPol({\Pow}(A)) \cap \Aff(A)^{[n]} $. Then $f(0)=0$ so $f$ is linear. For all $i$, $f(e_i^{0,1}) \in \{0,1\}^n$ so the matrix of $f$ is a 0-1 matrix. $f(11\dots1)=11\dots 1$ so each row of the matrix contains $1 \mod p$ 1s. Let $x_i=1$ for all $i$ except $i=j,k$, where $x_j=x_k=0$. Then $f(x) \in \{0,1\}^n$. If the matrix of $f$ had a 1 in column $j$ and column $k$ for the same row $l$, then $f(x)_l=-1$, but $-1 \not\in \{0,1\}$ so this is a contradiction. Thus the matrix has only one 1 in each row and is a permutation matrix, so $f$ is a wire permutation, and $\PPol(\Pow(A)) \cap \Aff(A) = \Pi$ \qed \end{proof} \subsection{Equivalence Relations} In this section we discuss the case when a permutation clone preserves an equivalence relation. An equivalence relation is a binary relation that is reflexive, transitive and symmetric, that is, for all $x,y,z\in A$, $x \rho x$, $x \rho y \wedge y \rho z \Rightarrow x \rho z$ and $x \rho y \Leftrightarrow y \rho x$. An equivalence relation is equivalent to a partition of $A$, where each part of the partition is an equivalence class. The following result shows a class of maximal borrow and ancilla closed permutation clones. \begin{theorem} Let $\rho$ be an equivalence relation with all equivalence classes of the same size. Then $\PPol(\rho)$ is a maximal borrow and ancilla closed permutation clone. \end{theorem} \begin{proof} For any $n\in \N$, $\rho^n$ is an equivalence relation with all equivalence classes the same size. Thus $\PPol(\rho)^{[n]} = \Aut(\rho^n)$. By \cite{LPS87} we know that $\Aut(\rho^n)$ is maximal in $S_{A^n}=B_n(A)$. Let $f\not \in \PPol(\rho)$ of arity $n$. Then $\langle \PPol(\rho),f\rangle^{[n]}= B_n(A)$. By borrow (or ancilla) closure, this means that for all $m\leq n$, $\langle \PPol(\rho),f\rangle^{[m]}= B_m(A)$. Then $\langle \PPol(\rho),f\rangle^{[1]}= B_1(A)$. Thus there is some unary map $g$ that breaks the equivalence relation $\rho$. Then for any $n$, $g\oplus i_n$ is an $(n+1)$-ary map that is not in $\PPol(\rho)^{[n+1]}$, so $\langle \PPol(\rho),g\rangle^{[n+1]}= B_{n+1}(A)$. Thus $\langle \PPol(\rho),g\rangle= B(A)$, so $ \PPol(\rho)$ is maximal. \qed \end{proof} \begin{lemma} Let $\rho$ be an equivalence relation on $A$. Let $f\triangleright \rho$. Then $f$ is well-defined on $A/\rho$. \end{lemma} Let $\epsilon:B(A) \rightarrow B(A / \rho)$ be the natural homomorphism. Then $\ker \epsilon$ is the set of bijections that are mapped to the identity. This is not a permutation clone, as it does not include the wire permutations $\Pi$, but it is a reversible iterative algebra. \begin{lemma} Let $\rho$ be an equivalence relation on $A$. Let $C \leq \PPol(\rho)$ be a sub-permutation clone. Then $\ker \epsilon ^{[n]}$ is a normal subgroup of $C^{[n]}$ and $C^{[n]}$ is a group extension of $\epsilon C$ by $\ker \epsilon ^{[n]}$. \end{lemma} \begin{example} Let $A=\{0,1,2,3\}$ and $\rho=01\mid 23$. Then $\ker \epsilon^{[n]} = B_n(\{0,1\})\bullet B_n(\{2,3\})$ and $\epsilon (\PPol(\rho))\cong B(\{0,1\})$ with the isomorphism $[0]\mapsto 0$ and $[2]\mapsto 1$. We see that $CP(\{0,1\},(01)(23)) \in \ker \epsilon$ because the permutation is within the equivalence classes. Let $f=CP(\{0,1\},(02)(13))$. Then $f \triangleright \rho$, $\epsilon(f)=CP(0,(01\,10))$ is the Fredkin gate. \end{example} This is a very clean example, as all the equivalence classes are the same size, so $\PPol(\rho)$ can permute them. \begin{example} Let $A=\{0,1,2,3\}$ and $\rho=0\mid 123$. Then $\ker \epsilon^{[n]} = B_n(\{0\})\bullet B_n(\{1,2,3\})$ and $\epsilon (\PPol(\rho))\leq Cons_{[0]} \leq B(\{[0],[1]\})$ because $\PPol(\rho)$ cannot map between the equivalence classes because of their different sizes (Lemma \ref{lemma_countingCons}). \end{example} Thus the lattice of (relationally defined) permutation clones below the permutation clone defined by an equivalence relation can be determined using inverse monoid extensions, see e.g.\ \cite{CARVALHO_GRAY_RUSKUC_2011} for some details. \subsection{Affine Maps} The following result has been found various times, see for instance \cite{Demetrovics_Bagyinszki_1982} for the inclusion diagram for $\vert A \vert=3$, Proposition 2.9 in \cite{szendrei1986clones} or \cite{salomaa_1964}. \begin{lemma} Let $\vert A \vert$ be a prime. Then $\Aff(A)$ has finitely many relationally defined sub-permutation clones. \end{lemma} Note that this does not show that there are only finitely many sub-permutation clones of the affine permutation clone. There are sub-permutation clones of the affine maps that are not defined by relations. Let $A=\Z_p$ be of prime order, let the weight $w:A^3 \rightarrow A$ be defined by $(a,b,c) \mapsto (a-c)\cdot (b-c)\in \Z_p$. Then $\Pol(w)\cap \Aff(A)$ will be the collection of affine maps for which the linear part of the map is orthogonal. This cannot be a relationally defined permutation clone. \subsection{Central Relations} Let $\rho \subseteq A^h$ be a relation. The \emph{center} of $\rho$ is $c(\rho)= \{a \in A \mid \forall a_2,\dots,a_h \in A,\, (a,a_2,\dots,a_h)\in \rho\}$. A relation $\rho \subseteq A^h$ is \emph{central} iff it is totally reflexive, totally symmetric and has a non void center that is a proper subset of $A$. \subsection{Degenerate or Essentially Unary Maps} We saw above that the essentially unary maps have the interesting property that they can be defined as a relational as well as by a cancellative weight. The maps are known by two names. In the field of reversible computation, they are known as degenerate maps \cite{aaronsonetal15}, probably because they do not involve any interaction between their inputs. In the clone theory field, they are known as essentially unary maps, because the output is only dependent upon a single input, thus they are generalise being unary. In clone theory, there is some use of the transformation monoid that is at the core of a clone of essentially unary maps. In our case, that will be a collections of permutations of our set $A$, a permutation group. \begin{theorem} \label{theorem_neq_degenerate} Let $A$ be a finite set with three or more elements. Let $\eta = \{(a,b) \mid a \neq b\} \subset A^2$ be the binary non-equality relation. Then $\PPol(\eta)=\Deg(A)$. \end{theorem} \begin{proof} Let $f\in \Deg(A)^{]n]}$. Let $ \left[ \begin{array}{c} x \\ y \end{array} \right] \in \eta^n$. Then for all $i$, $f(x)_i$ depends only upon one input, wlog, $x_j$. Then $f(x)_i = f(y)_i$ iff $x_j=y_j$, but $x_j\neq y_j$ so $f(x)_i \eta f(y)_i$. Thus $ \left[ \begin{array}{c} f(x) \\ f(y) \end{array} \right] \in \eta^n$, so $f \triangleright \eta$. Thus $\Deg(A) \leq \PPol(\eta)$. Let $A=\{0,\dots,m-1\}$. Choose some $n$. Let $\Gamma$ be the graph $(A^n,\eta)$, let $G=\Aut(\Gamma)$. $G$ is transitive on $A^n$ so $G_{e_0}$ is index $\vert A \vert^n$ in $G$. $G_{e_0}$ is transitive on $(A \setminus \{0\})^n$ so $G_{e_0,e_1}$ has index $(A \setminus \{0\})^n=(\vert A\vert-1)^n$ in $G_{e_0}$. Thus $G_{e_0,e_1,\dots,e_{m-1}}$ has index $\vert A \vert^n (\vert A\vert-1)^n \dots 2.1 = (\vert A\vert !)^n$ in $G$. Let $H=G_{e_0,e_1,\dots,e_{m-1}}$. $H$ fixes every subset of $A$. Then for $x\in \{0,a\}^n$, $a\not\in\{0,1\}$, $(e_i^{0,1},x)\in \Gamma$ or equivalently, $ \left[ \begin{array}{c} e_i^{0,1} \\ x \end{array} \right] \in \eta^n$ iff $x=e_i^{a,0}$ or $x=e_a$. Then if $f(e_i^{0,1})$ has more than one $1$, there will be more than two neighbours of $f(e_i^{0,1})$ in $\{0,a\}^n$, which would mean that $f$ is not an automorphism of $\Gamma$. Thus $f(e_i^{0,1})=e_j{0,1}$ for some $j$. Thus $H$ fixes the set $\{e_i^{0,1}\mid i=1,\dots,n\}$ as a set, and is transitive on them. Let $K\leq H$ fix these pointwise, so $K$ is of index $n!$ in $H$. Then by the adjacency argument above, for all $a\not\in\{0,1\}$, $e_i^{a,0}$ is fixed by all $f\in K$ and similarly for all $a\neq b$, $e_i^{a,b}$ is fixed by $K$. Lt $f\in K$. Let $x\in \{0,1\}^n$, $a\not\in\{0,1\}$. Suppose $x_i=1$. Then $(x,e_i^{a,0})\in \Gamma$ so $(f(x),e_i^{a,0})\in \Gamma$ thus $f(x)_i=1$. Similarly for $x_i=0$ so we see that $K$ fixes $\{0,1\}^n$ pointwise. Similarly $K$ fixes all $\{a,b\}^n$ pointwise. Let $x\in A^n$. Assume $x_i= a\in A$. Let $c\in A$, $c\neq a$. Then for all $j$, let $\bar x_j=c$ if $x_j=a$, $\bar x_j=a$ otherwise. Then $\bar x$ is fixed by $K$. Then $(x,\bar x) \in \Gamma$ so $(f(x),f(\bar x)=(f(x),\bar x) \in \Gamma$, so $f(x)_i \neq c$. Since $c$ was arbitrary and not equal to $a$, we see that $f(x)_i=a$. Thus $f$ fixes all of $A^n$ so $K$ is trivial. Thus the size of $G$ is $(G:H)(H:K) = ((\vert A\vert !)^n)(n!)$ which is equal to the size of $\Deg(A)^{[n]}$ so by inclusion, the two are the same. Thus $\PPol(\eta)=\Deg(A)$ and we are done. \qed \end{proof} When $A$ has two elements, the $\eta$ relation corresponds to the self-duality relation and so $\PPol(\eta)$ is not degenerate. The following notes are taken from \cite[Theorem 2.2]{szendrei2012} Let $S$ be the clone of mappings that are either essentially unary or nonsurjective, called S{\l}upecki's clone. Then for $2\leq i \leq \vert A\vert$, there exist a series of subsets, $S_i \subseteq \OO(A)$, that include all maps with range of order $i$ or less, together with the essentially unary maps. $S_{\vert A\vert} = \OO(A)$ and $S_{\vert A\vert-1} = S$. Then there exists a uniquely defined $S_1$ and we define $S_0$ to be the transformation monoid on $A$, so that $S_0\subsetneq S_1\subsetneq \dots \subsetneq S_{\vert A\vert-1} \subsetneq S_{\vert A\vert} =\OO(A)$ is an unrefinable strict chain, meaning that $S_i\leq S_{i-1}$ is a maximal subclone. \begin{definition} Let $A$ be a finite set. Define $\beta = \{(a_1,a_2,a_3,a_4) \in A^4\mid a_1=a_i \wedge a_j=a_k, \, \{1,a_i,a_j,a_k\}=\{1,2,3,4\}\}$. For $2\leq m \leq \vert A\vert$, define $\iota_m=\{(a_1,\dots,a_m) \mid \exists i\neq j:\:a_i=a_j \}$. That is, all $m$-tuples that contain at least one repetition. \end{definition} \begin{theorem}[\protect{\cite[Theorem 2.2]{szendrei2012}}] \label{thm_szendrei_slupecki} $S_1=\Pol(\beta)$ and for all $m\leq \vert A\vert$; $S_{r-1}=\Pol(\iota_r)$. \end{theorem} When we look at these relations for defining permutation clones, we note that the various $S_i$ contain unbalanced maps and essentially unary maps. So the collection of permutation clones defined by these relations collapses. \begin{definition} Let $R \subseteq A^k$ be a relation. We say that $R$ is \emph{totally symmetric} if for all permutations $\alpha \in S_k$, $(a_1,\dots,a_k) \in R$ iff $(a_{\alpha(1)},\dots,a_{\alpha(k)}) \in R$. \end{definition} \begin{theorem} \label{thm_include_iota} Let $3 \leq m \leq \vert A\vert$. Let $\rho\subseteq A^m$ be a totally symmetric relation that includes $\iota_m$. Then $\PPol(\rho)\leq\PPol(\iota_m)=Deg(A)$. \end{theorem} \begin{proof} First we show that $\PPol(\iota_m)=Deg(A)$ using Theorem \ref{thm_szendrei_slupecki} above. $S_{m-1}=\Pol(\iota_m)$ consists of essentially unary maps and those with image of size less than or equal to $m-1$. Maps with image of size less than $\vert A\vert$ cannot be balanced, cannot appear as components of bijections. Thus all components maps of $\PPol(\iota_m)$ are essentially unary so all elements of $\PPol(\iota_m)$ are degenerate. Applying Lemma \ref{lemma_countingCons} to $\iota_m$ we obtain \begin{eqnarray} c_{\iota_m,m}(a_1,\dots,a_{m-1})&=& \left\{ \begin{array}{l} (m-1) \mbox{ if } a_1,\dots,a_{m-1} \mbox{ are all distinct}\\ \vert A \vert \mbox{ otherwise } \end{array} \right. \end{eqnarray} All tuples $(a_1,\dots,a_{m}) \in \rho \setminus \iota_m$ contain no repetitions, they are sets of $m$ distinct elements of $A$. Thus \begin{eqnarray} c_{\rho,m}(a_1,\dots,a_{m-1})&=& \left\{ \begin{array}{l} m \mbox{ or more, if } a_1,\dots,a_{m-1} \mbox{ are all distinct } \\ \hspace{15mm}\wedge(a_1,\dots,a_{m-1},a)\in \rho \setminus \iota_m \exists a\in A\\ (m-1) \mbox{ if } a_1,\dots,a_{m-1} \mbox{ are all distinct}\\ \vert A \vert \mbox{ otherwise } \end{array} \right. \end{eqnarray} Applying Lemma \ref{lemma_maxCons} to this weight, using the fact that $\vert A\vert \geq m > m-1$, we let $\gamma = \{(a_1,\dots,a_{m-1})\mid c_{\rho,m})(a_1,\dots,a_{m-1})=m-1\}$. Then $\PPol(\rho) \leq \PPol(\gamma)$ Thus $\gamma$ consists of tuples that contain no repeats. Let $\theta\subset A^2$ consisting of the first two entries of $\gamma$, so $\PPol(\gamma) \leq \PPol(theta)$. But $\theta=\eta$, so by Theorem \ref{theorem_neq_degenerate} we know that $\PPol(\rho) \leq \PPol(\eta)=\Deg(A)$. \qed \end{proof} If $\rho$ properly contains $\iota_m$ then $\rho \setminus \iota_m$ will define a set of $m$-sets of $A$. Then $\PPol(\rho) = \langle \Gamma \rangle$ where $\Gamma \leq S_A$ is the permutation group on $A$ that fixes this set of $m$-sets. This can also be seen as the automorphism of the $m$-uniform hypergraph with hyperedges given by the $m$-sets. In any case, there are only a finite number of sub-permutation clones of $\PPol(\rho)$. \subsection{$h$-regular // $h$-universal // $h$-generated } The $h$ generated relations are defined by a complex reduction. However the definition implies that these relations are totally reflexive and totally symmetry, thus they include $\iota_m$ and by Theorem \ref{thm_include_iota} above, the permutation clone generated by a $h$-regular relation is essemtially unary. \subsection{Self dual} Self dual mappings on a set $A$ of order $n$ respect a permutation of $A$ of prime order $p$ with no fixed points. Thus $p$ divides $n$. If $A$ is of prime order, then these is one cycle and every map that respects that cycle is balanced. Thus every map in the clone could be a component map of a bijection. We have no further results in this case. \section{ $\Pol(w_R)$ on a two element set} The collection of clones on a two element set has been completely described by Post. Thus we can determine all relationally defined permutation clones on a two element set by inspecting each of the clones in the Post lattice. \begin{table}[htp] \caption{The $\beta$ classes of clones on the two element set, with their defining relations.} \label{table_binary} \begin{center} \begin{tabular}{|c|c|c|} \hline Name & Description & Defining Relation \\ \hline $\top$ & All maps & \{0,1\}\\ \hline $D$ & Self dual & \{01,10\}\\ \hline $DP$ & \makecell{Self dual, 0-preserving \\and 1-preserving} & \{0110,1010\}\\ \hline $A$ & Affine & \{0000,0011,0101,0110,1001,1010,1100,1111\}\\ \hline $AP_0$ & \makecell{Affine and 0-preserving,\\ i.e. linear} & \{000,011,101,110\}\\ \hline $AD$ & Affine Self dual & $\{0000,0011,0101,0110,1001,1010,1100,1111\} \times \{01,10\}$\\ \hline $AP_1$ & Affine and 1-preserving & \{00001,00111,01011,01101,10011,10101,11001,11111\}\\ \hline $AP$ & Linear and 1-preserving & \{0001,0111,1011,1101\}\\ \hline $P_0$ & 0-preserving & \{0\}\\ \hline $P_1$ & 1-preserving & \{1\}\\ \hline $P$ & 0- and 1-preserving & \{01\}\\ \hline $U$ & \makecell{Essentially Unary \\ (Degenerate)} & \{000,001,011,100,110,111\}\\ \hline $\Pi$ & Projections & something trivial\\ \hline \end{tabular} \end{center} \label{default} \end{table} \begin{theorem} \label{thm_binary} There are exactly 13 relationally defined permutation clones on a set of order 2. \end{theorem} \begin{proof} We start by showing that the clones defined by $T_0^2,\,M, T_1^2$ all lie in the same $\beta$-class as the projections. Let us consider $M$ first. A bounded partial order on two elements is a total order. From Lemma \ref{lemma_partial_order} we know that $\PPol(\leq)=\Pi$. $T_0^2=\{01,10,00\}$. Let $f \in \Pol(T_0^2)^{[n]}$, suppose $f$ is balanced. For all $x\in A^n$, $\left[\begin{array}{c} x\\ \bar x \end{array}\right] \in (T_0^2)^n$, so $\left[\begin{array}{c} f(x)\\ f(\bar x) \end{array}\right] \in T_0^2$ so one of them is zero. In order for $f$ to be balanced, one of them must be 0 and the other 1, so $f(\bar x) = \bar f(x)$, so $f\in D$. However $ \Pol(T_0^2)\cap D \subseteq M$ and the only balanced maps in $M$ are projections. Similarly $\Pol(T_1^2)$ contains no nontrivial balanced maps, so each of these clones lie in the same $\beta$ class as the projections. Now we show that $(U,UD) \in \beta$. The only balanced essentially unary maps on $A$ are the identity and NOT. Which are self-dual, so $\beta(U)=\beta(UD)$. We now look at the remaining 13 classes and show that each inclusion is proper, by demonstrating a bijection that is in one but not the other. We will do this from the top down in the inclusion lattices in Figure \ref{figure_lattice_binary}. We represent our permutations as two columns with the domain on the left and the image of each element in the domain on the right, except when we can reprsent them by affine maps. \begin{itemize} \item $B(A)$: The map \begin{equation*} \begin{array}{|c|c|} \hline 000 & 001 \\ 001 & 010 \\ 010 & 000 \\ 011 & 100 \\ 100 & 011 \\ 101 & 101 \\ 110 & 111\\ 111 & 110\\ \hline \end{array} \end{equation*} is neither self dual, affine, 0-fixing nor 1-fixing. \item $D$: The map \begin{equation*} \begin{array}{|c|c|} \hline 000 & 111 \\ 001 & 001 \\ 010 & 010 \\ 011 & 011 \\ 100 & 100 \\ 101 & 101 \\ 110 & 110\\ 111 & 000\\ \hline \end{array} \end{equation*} is self dual, but not affine, 0-fixing nor 1-fixing. \item $DP$: The map \begin{equation*} \begin{array}{|c|c|} \hline 000 & 000 \\ 001 & 001 \\ 010 & 101 \\ 011 & 011 \\ 100 & 100 \\ 101 & 010 \\ 110 & 110\\ 111 & 111\\ \hline \end{array} \end{equation*} is self dual, 0-fixing and 1-fixing, but not affine. \item $A=\Aff(2)$: The map $x \mapsto \left[ \begin{array}{ccc} 1 & 1 & 0 \\ 0 & 1 &0 \\ 0& 0 & 1 \end{array} \right] x + \left[ \begin{array}{c} 1 \\ 1 \\ 1 \end{array} \right] $ is affine, but not linear, 1-fixing, nor self dual. \item $AP_0$: Affine and fixing zero means linear. The map $x \mapsto \left[ \begin{array}{ccc} 1 & 1 & 0 \\ 0 & 1 &0 \\ 0& 0 & 1 \end{array} \right] x $ is linear, but not 1-fixing, nor self dual. \item $AP_1$: Affine and fixing zero means linear. The map $x \mapsto \left[ \begin{array}{ccc} 1 & 1 & 0 \\ 0 & 1 &0 \\ 0& 0 & 1 \end{array} \right] x + \left[ \begin{array}{c} 1 \\ 0 \\ 0 \end{array} \right]$ is 1-fixing, but not linear, so not in $AP$. \item $AD$: The map $x \mapsto \left[ \begin{array}{ccc} 1 & 1 & 1 \\ 0 & 1 &0 \\ 0& 0 & 1 \end{array} \right] x + \left[ \begin{array}{c} 1 \\ 0 \\ 0 \end{array} \right]$ is self dual, but not linear, so not in $AP$. \item $AD$: The map $x \mapsto \left[ \begin{array}{ccc} 1 & 1 & 1 \\ 0 & 1 &0 \\ 0& 0 & 1 \end{array} \right] x $ is self dual, linear, and 1-fixing, but not in $\Pi$. \item $U=Deg(A)$: The map $NOT$ is unary, affine and non trivial, so not in $\Pi$. \item $P_0$: The map \begin{equation*} \begin{array}{|c|c|} \hline 000 & 000 \\ 001 & 001 \\ 010 & 010 \\ 011 & 111 \\ 100 & 100 \\ 101 & 101 \\ 110 & 110\\ 111 & 011\\ \hline \end{array} \end{equation*} is 0-fixing , but not affine or 1-fixing. \item $P_1$: The map \begin{equation*} \begin{array}{|c|c|} \hline 000 & 011 \\ 001 & 001 \\ 010 & 010 \\ 011 & 111 \\ 100 & 100 \\ 101 & 101 \\ 110 & 110\\ 111 & 111\\ \hline \end{array} \end{equation*} is 1-fixing , but not affine or 0-fixing. \item $P$: The map \begin{equation*} \begin{array}{|c|c|} \hline 000 & 000 \\ 001 & 010 \\ 010 & 011 \\ 011 & 001 \\ 100 & 100 \\ 101 & 101 \\ 110 & 110\\ 111 & 111\\ \hline \end{array} \end{equation*} is 0-fixing and1-fixing , but not affine or self dual. \end{itemize} \qed \end{proof} Note that there are non-projection balanced monotone maps, e.g. 000,100,010,001 mapto 0, others map to 1. But this map cannot be extended to a permutation on $A^3$. \begin{figure}[htbp] \begin{center} \begin{tikzpicture} \node (top) at (2,9) {$\top$}; \node (A) at (2,8) {$A$}; \node (AP1) at (3,7){$AP_1$}; \node [below of=A] (AP0){$AP_0$}; \node (AD) at (1,7){$AD$}; \node (AP) at (2,4){$AP$}; \node (P0) at (3,8) {$P_0$}; \node (P1) at (4,8) {$P_1$}; \node (P) at (4,7) {$P$}; \node (D) at (0,8) {$D$}; \node (DP) at (1,5) {$DP$}; \node (UD) at (0,5) {$U$}; \node (bot) at (1,3) {$\Pi$}; \draw [] (top) -- (A); \draw [] (top) -- (D); \draw [] (top) -- (P0); \draw [] (top) -- (P1); \draw [] (P1) -- (P); \draw [] (P0) -- (P); \draw [] (P) -- (DP); \draw [] (P1) -- (AP1); \draw [] (P0) -- (AP0); \draw [] (DP) -- (AP); \draw [] (A) -- (AP0); \draw [] (A) -- (AP1); \draw [] (A) -- (AD); \draw [] (AD) -- (AP); \draw [] (AP0) -- (AP); \draw [] (AP1) -- (AP); \draw [] (AP) -- (bot); \draw [] (AD) -- (UD); \draw [] (UD) -- (bot); \draw [] (D) -- (DP); \draw [] (D) -- (AD); \end{tikzpicture} \caption{The inclusion diagram of the clones on two elements that give distinct permutation clones. This is a sublattice of the Post lattice.} \label{figure_lattice_binary} \end{center} \end{figure} From Table \ref{table_binary} we see that only two nontrivial permutation clones on two elements are ancilla closed by Theorem \ref{thm_ancillaclosed}, the affine maps and the degenerate maps. And in fact these are the only relationally defined ancilla closed permutation clones on the binary alphabet according to \cite{aaronsonetal15}. \section{Conclusions and further work} The collection of relationally defined permutation clones is restricted enough to be comprehensible, but still contains many interesting permutation clones. These permutation clones are those that are defined by a property of their components and nothing else other than reversibility. We have found the cluster definition for balanced maps. We find that for a binary alphabet, there are only finitely many relationally defined permutation clones. We cannot expect the general case to be so clean. We observed that many relations $R$ give rise to trivial permutation clones. Determining what relational properties imply triviality would be of value. Can we recognise a relation that gives $\Pol(w_R)=\Pi$? \subsubsection{Acknowledgements} Supported by the Austrian Science Fund (FWF): P33878 and PEEK Project AR561. \bibliographystyle{splncs04} \bibliography{RelPermClones} \end{document}
2412.06246v1
http://arxiv.org/abs/2412.06246v1
The smallest singular value for rectangular random matrices with Lévy entries
\documentclass[11pt,reqno]{amsart} \usepackage[utf8]{inputenc} \usepackage[margin=1.25in]{geometry} \parindent=.25in \usepackage{hyperref} \usepackage{appendix} \usepackage{amsfonts} \usepackage{amsthm} \usepackage{amssymb} \usepackage{stmaryrd} \usepackage{amsmath} \usepackage{amsthm} \usepackage[dvipsnames]{xcolor} \usepackage{mathrsfs} \usepackage{lipsum} \theoremstyle{plain} \newtheorem{theorem}{Theorem}[section] \newtheorem{corollary}[theorem]{Corollary} \newtheorem{lemma}[theorem]{Lemma} \newtheorem{Proposition}[theorem]{Proposition} \newtheorem{Assumption}[theorem]{Assumption} \newtheorem{Notation}[theorem]{Notation} \newtheorem{Example}[theorem]{Example} \newtheorem{Definition}[theorem]{Definition} \newtheorem{Sampling}[theorem]{Sampling} \theoremstyle{remark} \newtheorem{example}[theorem]{Example} \newtheorem{Claim}[theorem]{Claim} \newtheorem{remark}[theorem]{Remark} \usepackage{biblatex} \addbibresource{sample.bib} \numberwithin{equation}{section} \title[Smallest singular value Lêvy entries]{The smallest singular value for rectangular random matrices with Lévy entries} \author{Yi HAN} \address{Department of Mathematics, Massachusetts Institute of Technology, Cambridge, MA } \email{[email protected]} \begin{document} \begin{abstract} Let $X=(x_{ij})\in\mathbb{R}^{N\times n}$ be a rectangular random matrix with i.i.d. entries (we assume $N/n\to\mathbf{a}>1$), and denote by $\sigma_{min}(X)$ its smallest singular value. When entries have mean zero and unit second moment, the celebrated work of Bai-Yin and Tikhomirov show that $n^{-\frac{1}{2}}\sigma_{min}(X)$ converges almost surely to $\sqrt{\mathbf{a}}-1.$ However, little is known when the second moment is infinite. In this work we consider symmetric entry distributions satisfying $\mathbb{P}(|x_{ij}|>t)\sim t^{-\alpha}$ for some $\alpha\in(0,2)$, and prove that $\sigma_{min}(X)$ can be determined up to a log factor with high probability: for any $D>0$, with probability at least $1-n^{-D}$ we have $$C_1n^{\frac{1}{\alpha}}(\log n)^\frac{5(\alpha-2)}{2\alpha}\leq \sigma_{min}(X)\leq C_2n^{\frac{1}{\alpha}}(\log n)^\frac{\alpha-2}{2\alpha}$$ for some constants $C_1,C_2>0$. This appears to be the first determination of $\sigma_{min}(X)$ in the $\alpha$-stable case with a correct leading order of $n$, as previous ant-concentration arguments only yield lower bound $n^\frac{1}{2}$. The same lower bound holds for $\sigma_{min}(X+B)$ for any fixed rectangular matrix $B$ with no assumption on its operator norm. The case of diverging aspect ratio is also computed. Geometrically, the lower bound shows that the random polytope $X^*(B_1^N)$ generated by heavy-tail distributions will with very high probability contain Euclidean balls $B_2^n$ of a much larger radius compared to its Gaussian counterpart. \end{abstract} \maketitle \section{Introduction} This paper is concerned with the smallest singular value of a rectangular random matrix with i.i.d. entries. A classical theorem of Bai and Yin \cite{MR1235416} states that \begin{theorem}\label{theorem1.111}[\cite{MR1235416}] Let $X=(x_{ij})\in\mathbb{R}^{N\times n}$ with independent, identically distributed (i.i.d.) entries $x_{ij}$ satisfying $\mathbb{E}[x_{ij}]=0$, $\mathbb{E}[|x_{ij}|^2]=1$, $\mathbb{E}[|x_{ij}|^4]<\infty$. Assume that the aspect ratio satisfies $N/n\to \mathbf{a}$ as $n\to\infty$ for some $\mathbf{a}>1$. Then almost surely $$n^{-1/2} \sigma_{min}(X)\to \sqrt{\mathbf{a}}-1,\quad\text{ as } n\to\infty, $$ where $\sigma_{min}(X)$ denotes the smallest singular value of $X$. \end{theorem} Under the assumptions of Theorem \ref{theorem1.111}, it is also proven that $n^{-\frac{1}{2}}\sigma_{max}(X)$ converges almost surely to $\sqrt{\mathbf{a}}+1$, where $\sigma_{max}(X)$ is the largest singular value of $X$. The empirical measure of singular values of $X$ converge to the Marchenko-Pastur distribution \cite{marchenko1967distribution} and when $x_{ij}$ are sub-Gaussian, Tracy-Widom fluctuations of $\sigma_{min}(X)$ and $\sigma_{max}(X)$ are proven in \cite{soshnikov2002note}, \cite{feldheim2010universality}. When $x_{ij}$ has less than four moments, the distribution of $\sigma_{max}(X)$ is asymptotically Poisson \cite{auffinger2009poisson}. Recently, \cite{han2024deformed} determined the law of $\sigma_{max}(X)$ when $\mathbb{P}(|x_{ij}|\geq t)\sim t^{-4}$. It remained an open problem for some time whether the finite fourth moment condition is necessary for a.s. convergence of $n^{-1/2}\sigma_{min}(X)$ to $\sqrt{\mathbf{a}}-1$, until Tikhomirov finally removed the fourth moment condition in \cite{tikhomirov2015limit}. \begin{theorem}[\cite{tikhomirov2015limit}] The claim of Theorem \ref{theorem1.111} is true without assuming $\mathbb{E}[|x_{ij}|^4]<\infty.$ \end{theorem} Recently, \cite{bao2024phase} investigated the finer asymptotics of $\sigma_{min}(X)$ when $\mathbb{P}(|x_{ij}|\geq t)\sim t^{-\alpha}$, $\alpha\in(2,4)$ and obtained three different scaling limits in different ranges of $\alpha$. Having a good understanding of $\sigma_{min}(X)$ when $x_{ij}$ has a finite second moment, a natural question is to study $\sigma_{min}(X)$ when $x_{ij}$ has heavier tail, say they have a slow varying tail with $\mathbb{P}(|x_{ij}|\geq t)\sim t^{-\alpha}$ for some $\alpha\in(0,2)$. In this direction only partial progress has been achieved so far. A lower bound for $\sigma_{min}(X)$ has been derived in \cite{tikhomirov2016smallest} without assuming any moment condition on $x_{ij}$, and is purely based on anti-concentration properties: \begin{theorem}\label{theorem1.2345}[\cite{tikhomirov2016smallest}](Informal statement) For any $\delta>1$ and $N\geq \delta n$, consider a rectangular random matrix $X=(X_{ij})\in\mathbb{R}^{N\times n}$ with i.i.d. entries that are uniformly anti-concentrated (measured in terms of Lévy concentration function \eqref{levyconcentrationf}). Then we can find $u,v>0$ such that, for any non-random $B\in\mathbb{R}^{N\times n}$, we have $$ \mathbb{P}(\sigma_{min}(X+B)\leq u\sqrt{n})\leq \exp(-vn). $$ \end{theorem} Applying Theorem \ref{theorem1.2345} to a rectangular matrix $X$ with $\alpha$-stable distribution, we deduce that with very high probability $\sigma_{min}(X)\geq C\sqrt{n}$ for some $C>0$. However, one may expect that that this bound significantly underestimates the magnitude of $\sigma_{min}(X)$ when $\alpha\in(0,2)$, as we shall see $\sigma_{min}(X)$ should be of order $n^{\frac{1}{\alpha}}$ modulo some log factor. A closely related setting is square Lévy matrices, where the correct asymptotics of $\sigma_{min}(X)$ is recently derived in Louvaris \cite{louvaris2022universality}. Specifically, \begin{theorem}\label{theorem1.4gagag}[\cite{louvaris2022universality}] For $\alpha\in(0,2)$ Let $\sigma$ be defined by equation (1.2) in \cite{louvaris2022universality}. let $Z$ be a random variable following a $(0,\sigma)$ $\alpha$-stable law in the sense that $$ \mathbb{E}[e^{itZ}]=\exp(-\sigma^\alpha|t|^\alpha)\text{ for all } t\in\mathbb{R}. $$ Let $D_n(\alpha)=(d_{ij})_{1\leq i,j\leq n}$ be a squared random matrix with i.i.d. entries, each entry has the law $n^{-\frac{1}{\alpha}}Z$. Then there is a countable set $\mathcal{A}\subset(0,2)$ such that for all $\alpha\in(0,2)\setminus\mathcal{A}$, we have $$\mathbb{P}(n\xi \sigma_{min}(D_n(\alpha))\leq r)=1-\exp(-\frac{r^2}{2}-r)+\mathcal{O}_r(n^{-c}) $$ for some $c>0$, where $\xi$ is a constant depending only on $\alpha$ defined in \cite{louvaris2022universality}, eq. (1.4). \end{theorem} Theorem \ref{theorem1.4gagag} generalizes the celebrated universality result of Tao and Vu \cite{tao2010random} for least singular value of i.i.d. (light-tailed) matrices to i.i.d. Lévy matrices. Now we turn back to a rectangular matrix with i.i.d. stable distribution. In this regime the spectral measure was characterized in \cite{belinschi2009spectral}, which is a heavy-tail analogue of the Marchenko-Pastur distribution and, in contrast to the Marchenko-Pastur law, has a nonzero mass in any neighborhood of zero. Not much is known since then. A recent investigation \cite{bao2024phase2} identified a change from localized to delocalized eigenvector statistics as we switch from $\alpha\in(0,2)$ to $\alpha>2$, but asymptotics of $\sigma_{min}(X)$ remain elusive. Meanwhile, the proof methods in these existing works (in particular on squared Lévy matrices \cite{MR4310816},\cite{MR4260468},\cite{louvaris2022universality}) do not easily generalize to this heavy-tail rectangular setting. \subsection{Two-side least singular value estimates} The first contribution of this paper is a lower bound for $\sigma_{min}(X)$ that is asymptotically sharp up to a log factor. \begin{theorem} \label{theorem1.56} Fix $\alpha\in(0,2),$ $\delta>1$, and let $N=\lceil \delta n\rceil$. Let $\xi$ be a random variable with a symmetric distribution $\xi \overset{\operatorname{law}}{=}-\xi$ satisfying that, for some $C_u>C_\ell>0$ we have for any $t>0$, \begin{equation}\label{stablelaw} C_\ell t^{-\alpha}\leq \mathbb{P}(|\xi|\geq t)\leq C_ut^{-\alpha}. \end{equation} Let $X=(x_{ij})_{1\leq i\leq N,1\leq j\leq n}$ be a rectangular random matrix with independent identically distributed (i.i.d.) entries having the distribution $\xi$. Then for any $D>0$ we can find $C_1,C_2>0$ and some $N_0\in\mathbb{N}_+$ such that, whenever $N\geq N_0$, \begin{equation}\label{twosideestimate1} \mathbb{P}\left(C_1n^\frac{1}{\alpha}(\log n)^\frac{5(\alpha-2)}{2\alpha}\leq \sigma_{min}(X)\leq C_2n^\frac{1}{\alpha}(\log n)^\frac{\alpha-2}{2\alpha}\right)\leq 3n^{-D}, \end{equation} where the constants $C_1,C_2,N_0$ depend only on $C_\ell$,$C_u$,$\alpha$,$\delta$,$D$. \end{theorem} The proof of Theorem \ref{stablelaw} and other main results in this work are based on a surprising interplay between two distinctive powerful techniques: a recent matrix universality principle by Brailovskaya and Van Handel \cite{brailovskaya2024universality}, and a careful adaptation of an earlier argument of Tikhomirov \cite{tikhomirov2016smallest}. Such a successful combined usage seems completely new: we need to consider both anti-concentration phenomena and the increased variance due to heavy-tail, and negligence on either side would not lead to our almost optimal estimate in \eqref{twosideestimate1}. Recall that a random variable $Z$ is said to be $(0,\sigma)$ $\alpha$-stable for some $\alpha\in(0,2)$ and $\sigma>0$, if $$ \mathbb{E}[e^{itZ}]=\exp(-\sigma^\alpha|t|^\alpha)\text{ for all }t\in\mathbb{R}. $$ By standard properties of stable distribution, the estimate \eqref{stablelaw} is satisfied by all $\alpha$-stable laws for all $\alpha\in(0,2)$. The proof of Theorem \ref{theorem1.56} (and all other forthcoming theorems) can be adapted to any symmetric random variable $\xi$ having a slow varying tail in the domain of attraction of $\alpha$-stable law, i.e. for some slow varying function $L(t)$: $$\mathbb{P}(|\xi|\geq t)=L(t)t^{-\alpha}.$$ All the technical arguments remain unchanged, but the precise asymptotic of $\sigma_{min}(X)$ will depend on $L(t)$ up to a sub-polynomial factor in $n$. Theorem \ref{theorem1.56} shows that typically the smallest singular value $\sigma_{min}(X)$ is concentrated in a very small window. We believe that the lower bound in \eqref{twosideestimate1} can be improved to $n^\frac{1}{\alpha}(\log n)^\frac{\alpha-2}{2\alpha}$, i.e. the upper bound should give the optimal magnitude. Currently the discrepancy of the upper and lower bound by a log factor is due to the technique we use, see Remark \ref{majorremarks}, (4) for more details. Removing the log factor discrepancy may require considerable effort and innovative ideas, which we leave for future work. Despite this mismatch of log factors, Theorem \ref{theorem1.56} appears to be the first time that we can capture the effect of heavy tails on $\sigma_{min}(X)$ that is not existing for Gaussian distribution. Another major advantage is that the proof is robust to perturbations, even without any control on operator norm of perturbation, see Theorem \ref{lowergeneral}. A further open problem is to derive the exact distribution of $\sigma_{min}(X)$, but we feel this problem is far beyond the scope of existing techniques. While the estimate \eqref{twosideestimate1} holds with polynomially good probability $1-O(n^{-D})$ for any $D>0$, it does not hold with exponentially good probability $1-e^{-\Omega(n)}$. This can be seen from the simple fact that for any $c\in(0,1)$, with probability $\Omega(e^{-n^c})$, we can find a column of $X$ whose every entry is conditioned to have absolute value less than $n^{\frac{1}{\alpha}-\frac{c}{\alpha}}$. Then, applying a computation as in the proof of Theorem \ref{theorem2.22} shows the Euclidean norm of this column is with high probability much smaller in magnitude than the lower bound in \eqref{twosideestimate1}, so $\sigma_{min}(X)$ is smaller than this quantity. For dense random matrices with uniformly anti-concentrated entry distribution, we often have least singular value bounds with exponentially good probability, see \cite{litvak2005smallest}, \cite{rudelson2009smallest}, \cite{livshyts2021smallest}. For spare random matrices, small singular value estimates usually hold with probability $1-e^{\Omega(np)}$ where $p$ is the sparsity, see \cite{basak2017invertibility}, \cite{gotze2023largest}, \cite{dumitriu2024extreme}. This is no surprise given that heavy-tail random matrix ensembles share some common features with sparse matrix ensembles. Finally, we note that the factor $5(\alpha-2)$ in the left hand side of \eqref{twosideestimate1} can be easily replaced by $(4+\epsilon)(\alpha-2)$ for any $\epsilon>0$, but further improvement is not clear. \subsection{Diverging aspect ratio and general perturbations} The lower bound in Theorem \ref{theorem1.56} is valid in a much wider generality, as it continues to hold when the random variables are no longer i.i.d. and the matrix is perturbed by some other deterministic matrix $B$. It is also important to consider unbounded aspect ratio $N/n$, which we discuss here. In the next theorem, we (1)consider possibly unbounded aspect ratio $N/n$; (2)remove the i.i.d. assumption and (3)consider a general perturbation $B$. \begin{theorem}\label{lowergeneral} Fix $\alpha\in(0,2),$ $\delta>1$, and let $N,n$ be integers such that $N\geq \delta n$. Let $(x_{ij})_{1\leq i\leq N,1\leq j\leq n}$ be i.i.d. random variables with a symmetric distribution and satisfy \eqref{stablelaw}. Fix $0<A_1<A_2$ and let $(a_{ij})_{1\leq i\leq N,1\leq j\leq n}$ be fixed real numbers such that $$A_1\leq a_{ij}\leq A_2$$ for each $i=1,\cdots,N$ and $j=1,\cdots,n$. Let $A=(a_{ij}x_{ij})$ be an $N\times n$ random matrix and let $B$ be any deterministic $N\times n$ matrix. We do not make any assumption on $B$. Then for any $D>0$ we can find $C_3>0$ and $N_0\in\mathbb{N}_+$ such that, whenever $N\geq N_0$ \begin{equation}\label{onesideestimate} \mathbb{P}(\sigma_{min}(A+B)\leq C_3N^\frac{1}{\alpha}(\log N)^\frac{5(\alpha-2)}{2\alpha})\leq 2N^{-D}, \end{equation} where $C_3$ and $N_0$ depend only on $D,\delta,\alpha,C_\ell,C_u,A_1,A_2$, and notably do not depend on $B$. \end{theorem} In Theorem \ref{theorem1.56} we assumed a bounded aspect ratio $N/n$ and do not aim for a sharp estimate on the leading constant, so we can freely interchange $n$ by $N$, or vice versa, in estimate \eqref{twosideestimate1}. But in Theorem \ref{lowergeneral} the aspect ratio is unbounded, so we need to be careful that it is the factor $N$, rather than $n$, appearing in \eqref{onesideestimate}. An upper bound for $\sigma_{min}(X)$ can also be proven for diverging $N/n$. \begin{Proposition}\label{propositionunbounded} (Upper bound, diverging $N/n$) Fix $\alpha\in(0,2)$, $\delta>1$ and let integers $N,n$ satisfy $N\geq \delta n$. Consider an $N\times n$ random matrix $X=(x_{ij})$ with i.i.d. symmetric entries satisfying \ref{stablelaw}. Then for any $D>0$ and $\mathbf{b}\in(0,1)$ we can find $C_4>0$ such that $$ \mathbb{P}(\sigma_{min}(X)\geq C_4N^\frac{1}{\alpha}(\log n)^\frac{\alpha-2}{2\alpha})\leq N^{-D}+\exp(-n^\mathbf{b}), $$ where $C_4$ depends only on $D,\delta,\alpha,C_\ell,C_u,\mathbf{b}$. \end{Proposition} We stress that the upper bound in Proposition \ref{propositionunbounded} involves a factor $(\log n)$ but the lower bound in Theorem \ref{lowergeneral} involves the much larger factor $(\log N)$. They cannot be interchanged when $N/n$ is very large. The reason is, for the lower bound we need a form of global universality principle, and $N$ is the typical size of the rectangular matrix $X$. For the upper bound we only need to find some columns of $X$ with certain good properties, and there are only $n$ columns, so we only need factor $\log n$. From this reasoning, we believe that in the general case of unbounded $\log N/\log n$, the idea of this work will never lead to lower and upper bounds of $\sigma_{min}(X)$ having the same order of magnitude. \subsection{A geometric application: Euclidean balls inside random polytopes} An elegant way to visualize the main result of this paper is to investigate polytopes spanned by randomly generated vectors with heavy-tail entries. Fix $\delta>1$ and $N\geq \delta n$. We independently generate $N$ Gaussian vectors in $\mathbb{R}^n$, denoted $v_1,\cdots,v_N$, where each $v_i$ has i.i.d. coordinates of standard real-Gaussian distribution. A folklore example in Banach space theory (see for instance \cite{litvak2002randomized}, \cite{mankiewicz2000compact}, \cite{wojtaszczyk2010stability}) states that, $\text{Conv}(v_1,\cdots,v_N)$, the convex hull of $v_1,\cdots, v_N$ in $\mathbb{R}^n$, will contain a Euclidean ball $\epsilon B_2^n$ with probability 1-$e^{-\Omega(n)}$, where $\epsilon=\epsilon(N,n)>0$ is some fixed function of $N$ and $n$. Let $\Gamma$ be an $N\times n$ rectangular random matrix with i.i.d. standard Gaussian coordinates, then the conclusion in the last paragraph can be reformulated as requiring \begin{equation}\label{l1quotient}\Gamma^*(B_1^N)\supset \epsilon B_2^n,\end{equation} where $\Gamma^*$ is the transpose matrix of $\Gamma$, and for any $p\in[1,\infty)$ $B_p^n$ denotes the unit $\ell_p^n$ ball in $\mathbb{R}^n$. This property \eqref{l1quotient} is called $\ell^1$ quotient property by some authors \cite{wojtaszczyk2010stability}, \cite{guedon2020random}, and has important applications in analyzing compressive sensing algorithms \cite{donoho2006compressed}, \cite{candes2006stable}, \cite{candes2006compressive}. The concentration of $\sigma_{min}(X)$ can be used to derive the $\ell_1$ quotient property and many other more elaborate results on the geometry of random polytopes, as proposed in the work \cite{litvak2005smallest}. Moreover, \cite{litvak2005smallest} shows that these properties of random polytopes are universal and do not depend on the specific entry distribution, so it covers Gaussian, Bernoulli, and all normalized sub-Gaussian random variables. Specifically, \cite{litvak2005smallest}, Theorem 4.2 further shows that with probability exponentially close to 1, we have $\Gamma^*(B_1^N)\supset\frac{1}{8}(B_\infty^n\cap \sqrt{C_1\ln(N/n)}B_2^n)$ for $2^n\geq N\geq C_2n$, and for two given constants $C_1,C_2>0$. This was followed by a series of refinements including \cite{guedon2022geometry} \cite{hayakawa2023estimating}. We note that \cite{guedon2020random} extended these geometric characterizations in \cite{litvak2005smallest} to all random variables with unit variance and uniform anti-concentration, without assuming further moments are finite. In \cite{guedon2022geometry} the authors further considered the polytope generated by $\alpha$-stable distributions, $\alpha\in(1,2)$. Theorem \ref{lowergeneral} immediately implies an improved estimate on convex hull geometry. \begin{corollary}\label{corollary} Fix $\delta>1$, $\alpha\in(0,2)$ and let integers $N,n$ satisffy $N\geq \delta n$. Let $X=(x_{ij})$ be an $N\times n$ rectangular matrix with i.i.d. entries satisfying \eqref{stablelaw}. Then for any $D>0$ we can find $N_0\in\mathbb{N}$ and $C_4>0$ such that whenever $N\geq \max(N_0,\delta N)$, with probability at least $1-N^{-D}$ we have $$ X^*(B_1^N)\supset C_4 N^{\frac{1}{\alpha}-\frac{1}{2}}(\log N)^\frac{5(\alpha-2)}{2\alpha}B_2^n. $$ where $C_4,N_0$ depend only on $D,\alpha,\delta,C_\ell,C_u$. \end{corollary} The remarkable feature of Corollary \ref{corollary} is that it already yields significant improvement over the Gaussian case for any bounded aspect ratio $\delta<N/n<\infty.$ In comparison, for Gaussian random polytopes (and all light-tailed distributions), we only have $X^*(B_1^N)\supset\epsilon B_2^n$ for some finite $\epsilon$ when aspect ratio is bounded. While it is intuitively clear that random polytopes spanned by heavy-tailed entries will have a much larger volume than its Gaussian counterpart, proving that with high probability it contains a very large Euclidean ball is unexplored in previous literature, and may be inaccessible by other methods based on anti-concentration arguments. Corollary \ref{corollary} can be compared to the recent Theorem 1.8 of \cite{guedon2022geometry}. This cited work works with $\alpha$-stable distribution for $\alpha\in(1,2)$ (while we work with $\alpha\in(0,2)$), and considers the inclusion of general $\ell_q^n$ balls in $X^*(B_1^N)$ whereas we consider the $\ell_2^n$ ball. The estimate in \cite{guedon2022geometry} appears to be better when $N/n$ is very large. On the other hand, if $N/n$ is bounded, then \cite{guedon2022geometry}, Theorem 1.8 yields no improvement to the Gaussian case whereas Corollary \ref{corollary} well captures this heavy-tail inflation effect in the interior of $X^*(B_1^N)$. We expect this work to be a starting point for finer investigations on the geometry of random polytopes generated by $\alpha$-stable distributions, $\alpha\in(0,2)$. The proof of Corollary \ref{corollary} is elementary and outlined as follows. \begin{proof}[\proofname\ of Corollary \ref{corollary}] This follows from the fact that, if for any $y\in \mathbb{R}^n$ we have $t|y|\leq \|Xy\|_\infty,$ then $tB_2^n\in X^*(B_1^N).$ Thus if $\|Xy\|\geq t\sqrt{N}\|y\|$ for any $y\in\mathbb{R}^n$ then $tB_2^n\subset X^*(B_1^N).$ Now we make a direct use of the conclusion of Theorem \ref{lowergeneral}. The claimed fact is an elementary corollary of the hyperplane separation theorem. Indeed, consider any $\tilde{y}\notin X^*(B_1^N)$ with $\|\tilde{y}\|\leq t$. This set $X^*(B_1^N)$ is convex in $\mathbb{R}^n$, so the separation theorem provides some $z\in\mathbb{R}^n,\|z\|=1$, satisfying $$ t>\langle z,\tilde{y}\rangle\geq\sup\{|\langle z,y\rangle|:y\in X^*(B_1^N)\}, $$ so that $\sup_{j=1,\cdots,N}|\langle z,X_{ j\cdot}\rangle|<t$ and $\|Xz\|_\infty<t$. We use $X_{j\cdot}$ to denote the $j$-th row of $X$. \end{proof} \section{Proof outline of main results} To study the heavy-tail random matrix $X$, an (by now standard, see \cite{aggarwal2019bulk}, \cite{MR4310816},\cite{louvaris2022universality},\cite{bao2024phase},\cite{han2024deformed}) approach is to resample the entries of $X$. Specifically, we first assign a label taking value in $\{0,1\}$ to each entry, where a label 1 is assigned to site $(i,j)$ meaning that $x_{ij}$ takes a small value, and value 0 assigned to $(i,j)$ meaning that $x_{ij}$ takes s large value. Then we sample the small and large entry parts of $X$ separately via conditional laws of $x_{ij}$ conditioned to be small or large. Conditioned on the label, the entries of $X$ remain independent. \subsection{Resampling for bounded aspect ratio} We outline this resampling procedure for $X$. In this section we assume the aspect ratio is bounded: for some fixed $1<\delta<T<\infty$, $$ \delta n\leq N\leq Tn. $$ The diverging aspect ratio case is dealt in Section \ref{divergingaspect}. \begin{Definition} The sampling procedure (for bounded aspect ratio) has the following steps: \end{Definition} \begin{enumerate} \item Fix $\tilde{\epsilon}_n\in(0,\frac{1}{\alpha})$ whose value may be $n$-independent and will be determined later. Different values of $\tilde{\epsilon}_n$ will be used in each specific application. \item Consider $\Psi=(\psi_{ij})$ an $N\times n$ random matrix with independent Bernoulli entries distributed as \begin{equation}\label{generationoflabels} \psi_{ij}=\begin{cases}1,\quad\text{with probability } \mathbb{P}(|x_{ij}|\leq n^{\frac{1}{\alpha}-\tilde{\epsilon}_n}),\\0,\quad \text{with probability } \mathbb{P}(|x_{ij}|\geq n^{\frac{1}{\alpha}-\tilde{\epsilon}_n}). \end{cases} \end{equation} \item Define two classes of random variables $y_{ij}$ and $z_{ij}$, that are conditional versions of $x_{ij}$ such that $|x_{ij}|\leq n^{\frac{1}{\alpha}-\tilde{\epsilon}_n}$ and $|x_{ij}|\geq n^{\frac{1}{\alpha}-\tilde{\epsilon}_n}$. More precisely, we define $y_{ij}$ and $z_{ij}$ to satisfy, for any interval $\tilde{I}\subset\mathbb{R}$, \begin{equation} \mathbb{P}(y_{ij}\in\tilde{I})=\frac{\mathbb{P}\left(x_{ij}\in\tilde{I}\cap[-n^{\frac{1}{\alpha}-\tilde{\epsilon}_n},n^{\frac{1}{\alpha}-\tilde{\epsilon}_n}]\right)}{\mathbb{P}(x_{ij}\in[-n^{\frac{1}{\alpha}-\tilde{\epsilon}_n},n^{\frac{1}{\alpha}-\tilde{\epsilon}_n}])}, \end{equation} \begin{equation}\label{lawofz} \mathbb{P}(z_{ij}\in\tilde{I})=\frac{\mathbb{P}(x_{ij}\in\tilde{I}\cap((-\infty,-n^{\frac{1}{\alpha}-\tilde{\epsilon}_n}]\cup[n^{\frac{1}{\alpha}-\tilde{\epsilon}_n},\infty)))}{\mathbb{P}(x_{ij}\in(-\infty,-n^{\frac{1}{\alpha}-\tilde{\epsilon}_n}]\cup[n^{\frac{1}{\alpha}-\tilde{\epsilon}_n},\infty))}. \end{equation} \item It is not hard to check that the matrix $X$ has the same distribution as \begin{equation}\label{resamplingdecision}X\overset{\operatorname{law}}{=} {\Psi}\circ Y+(\mathbf{1}-{\Psi})\circ Z, \end{equation} where $Y=(y_{ij})_{1\leq i\leq N,1\leq j\leq n}$ and $z=(z_{ij})_{1\leq i\leq N,1\leq j\leq n}$, and entries of $Y$ and $Z$ are independent. The symbol $\circ$ denotes the entry-wise (Hadamard) product of two $N\times n$ matrices: $(A\circ B)_{ij}=a_{ij}b_{ij}$, and $\mathbf{1}$ denotes the all-ones $N\times n$ matrix. \end{enumerate} \subsection{The upper bound} The upper bound claimed in Theorem \ref{theorem1.56} is proven in \cite{bao2024phase2}, Theorem 3.10. For sake of completeness, we outline the complete proof from \cite{bao2024phase2}. \begin{theorem}\label{theorem2.22}[\cite{bao2024phase2}, Theorem 3.10] Fix $\alpha\in(0,2)$, $\delta>1$ and $N=\lceil \delta n\rceil$. For any $D>0$ we can find a constant $C_\eqref{theorem2.22}>0$ and $N_\eqref{theorem2.22}\in\mathbb{N}$ depending only on $\alpha,\delta,C_\ell,C_u,D$ such that $$ \mathbb{P}(\sigma_{min}(X)\geq C_\eqref{theorem2.22}n^{\frac{1}{\alpha}}(\log n)^{\frac{\alpha-2}{2\alpha}})\leq n^{-D},\quad N\geq N_\eqref{theorem2.22}. $$ \end{theorem} \begin{proof} This proof is taken from \cite{bao2024phase2}. For each $j=1,\cdots,n$ denote by $\mathbf{ \psi}_j=(\psi_{1j}\cdots\psi_{Nj})^T$ the $j$-th column of the label $\Psi$, and $\mathbf{1}_n$ the all-ones vector. Then $$ \exp(-\delta C_u n^{\alpha\tilde{\epsilon}_n})\leq \mathbb{P}(\mathbf{\psi}_j=\mathbf{1}_n)\leq \exp(-C_\ell n^{\alpha\tilde{\epsilon}_n}),\quad j\in[n] $$ and that $$ \mathbb{P}(\Psi\text{ has no all-one columns})\leq\exp(-n\exp(-\delta\cdot C_un^{\alpha\tilde{\epsilon }_n})). $$ Next we let $Y^{\mathbf{m}}$ denote the minor of $Y$ after removing all columns $j$ such that $\psi_j\neq \mathbf{1}_n$. We obviously have $$ \sigma_{min}({\Psi}Y+(1-{\Psi})Z)=\inf_{v\in\mathbb{S}^{n-1}}\|({\Psi}Y+(1-{\Psi})Z)v\|\leq \|Y^{\mathbf{m}}\|. $$ Since $x_{ij}$ has a symmetric distribution, $\mathbb{E}[y_{ij}]=0$ and $\mathbb{E}[y_{ij}^2]\leq C_u n^{-1}n^{{\frac{2}{\alpha}-(2-\alpha})\tilde{\epsilon}_n}$. We have by Rosenthal’s inequality that for any $q>0,$ (where $(Y^{\mathbf{m}})_{i\cdot}$ denotes the $i$-th row of $Y^{\mathbf{m}}$), $$ \mathbb{E}[\|(Y^{\mathbf{m}})_{i\cdot}\|_2^q]\leq C^q (n^{\frac{1}{\alpha}-\tilde{\epsilon}_n})^q(n^{\alpha\tilde{\epsilon}_n})^\frac{q}{2}+q^\frac{q}{4}(n^{\alpha\tilde{\epsilon}_n})^{\frac{q}{4}}, $$ where $C>0$ depends only on $C_u$, $\alpha$ and $\delta$. A similar computation applies to the columns of $Y^{\mathbf{m}}$. Now apply Lemma \ref{kemma2.2spectralradius} with the choice $q=2\log n$, we have $$ \mathbb{E}\|Y^{\mathbf{m}}\|_2^q\leq 2C^q(n^{\frac{1}{\alpha}-\tilde{\epsilon}_n})^qn(n^{\alpha\tilde{\epsilon}_n})^\frac{q}{2}. $$ Then applying Markov's inequality, for any $D>0$, we can find $C_\eqref{theorem2.22}>0$ such that $$ \mathbb{P}\{\|Y^{\mathbf{m}}\|\geq C_\eqref{theorem2.22}n^{\frac{1}{\alpha}-(1-\frac{\alpha}{2})\tilde{\epsilon}_n}\}\leq\frac{1}{2} n^{-D}+\exp(-n\exp(-\delta\cdot C_u n^{\alpha\tilde{\epsilon}_n})) $$ where $C_\eqref{theorem2.22}>0$ depends on $D,C_u,C_\ell$ and $\delta$. Now we take some $\mathbf{b}\in(0,1)$ and define $\tilde{\epsilon}_n$ via \begin{equation} \label{choiceofepsilonn}n^{\alpha\tilde{\epsilon}_n}=\frac{\mathbf{b}\log n}{\delta\cdot C_u}. \end{equation} Finally we take $N_\eqref{theorem2.22}$ to be such that, for any $N\geq N_\eqref{theorem2.22}$, we have $\exp(-n^{1-\mathbf{b}})\leq \frac{1}{2}n^{-D}.$ \end{proof} In the proof we have used the following Lemma from \cite{seginer2000expected}: \begin{lemma}\label{kemma2.2spectralradius}[\cite{seginer2000expected}, Corollary 2.2] Consider $Y=(y_{ij})$ some $m_1\times m_2$ random matrix with i.i.d. zero mean entries. If we use $Y_{i\cdot}$ and $Y_{\cdot j}$ to denote the $i$-th row or $j$-th column of $X$, then we can find $C>0$ so that for any $q\leq 2\log\max(m_1,m_2)$, we have $$ \mathbb{E}\|Y\|^q\leq C^q\left(\mathbb{E}\max_{1\leq i\leq n_1}\|Y_{i\cdot}\|_2^q+\mathbb{E}\max_{1\leq j\leq m_2}\|Y_{\cdot j}\|_2^q \right). $$ \end{lemma} \subsection{The lower bound} In the proof of lower bound, which is the major contribution of this paper, we take the same decomposition $X={\Psi}\circ Y+(\mathbf{1}-{\Psi})\circ Z$ but take the truncation level $\tilde{\epsilon}_n$ slightly differently. We first compute the variance of $y_{ij}$: for any $\tilde{\epsilon}_n=o(1),$ we must have $\mathbb{P}(|y_{ij}|\leq n^{\frac{1}{\alpha}-\tilde{\epsilon}_n})=1-o(1)$, so that $$\begin{aligned} \mathbb{E}[|y_{ij}|^2]&\in(1+o(1))[C_\ell, C_u]\cdot \int_0^{n^{\frac{1}{\alpha}-\tilde{\epsilon}_n}}2x^{1-\alpha}dx\\& \in \frac{2+o(1)}{2-\alpha}[C_\ell,C_u]\cdot n^{\frac{2-\alpha}{\alpha}-(2-\alpha)\tilde{\epsilon}_n} .\end{aligned} $$Then we take the renormalization and deduce $$ \mathbb{E}[|n^{-\frac{1}{\alpha}+(1-\frac{\alpha}{2})\tilde{\epsilon}_n}y_{ij}|^2]=C_n\cdot n^{-1} $$ for some constant $C_n\in \frac{2+o(1)}{2-\alpha}[C_\ell,C_u]$. We summarize the computations so far as follows \begin{lemma}\label{lemma2.34} Let $\xi_n$ be the distribution of $n^{-\frac{1}{\alpha}+(1-\frac{\alpha}{2})\tilde{\epsilon}_n}y_{ij}$, then $$\mathbb{E}[\xi_n]=0,\quad \mathbb{E}[|\xi_n|^2]=C_n\cdot n^{-1} ,\quad |\xi_n|\leq n^{-\frac{\alpha}{2}\epsilon_n} \text{ a.s. }.$$ \end{lemma} Instead of choosing $\tilde{\epsilon}_n$ as in \eqref{choiceofepsilonn}, we will choose $\tilde{\epsilon}_n$in the proof of lower bound as \begin{equation}\label{choicelowerbound} n^{\alpha\tilde{\epsilon}_n}=(\log n)^5. \end{equation} This change is made for a purely technical purpose, so that we can use Theorem \ref{derivation2.414}. See Remark \ref{majorremarks}, (4) for more discussions on the optimality of $\tilde{\epsilon}_n$. We let $\mathcal{F}_\Psi$ be a sigma subalgebra of the probability space $(\Omega,\mathcal{F},\mathbb{P})$ spanned by all the random variables $\psi_{ij}$ and the events $\{\mathbf{\psi}=W\}$ for any $W\in\{0,1\}^{N\times n}$. Then conditioning on the filtration $\mathcal{F}_\Psi$, the random variables $y_{ij}$ and $z_{ij}$ are mutually independent. Now we outline a heuristic argument to prove Theorem \ref{theorem1.56}. For any fixed choice of label ${\Psi}$, we first sample the entries $Z=(z_{ij})$, and we show that no matter what value $z_{ij}$ takes, we can always use the randomness of $y_{ij}$ to derive high probability lower bounds for $\sigma_{min}(X)$ that are independent of $Z$. Via the normalization in Lemma \ref{lemma2.34}, we see that $n^{-\frac{1}{\alpha}+(1-\frac{\alpha}{2})\tilde{\epsilon}_n}X$ has most entries distributed as $\xi_n$ which has zero mean, variance $n^{-1}$ and are almost surely bounded. Since $X$ is rectangular, we can heuristically deduce that $\sigma_{min}(n^{-\frac{1}{\alpha}+(1-\frac{\alpha}{2})\tilde{\epsilon}_n}X)\geq \sqrt{\mathbf{a}}-1$ by the classical Bai-Yin Theorem (Theorem \ref{theorem1.111}). This heuristic argument has several flaws. First, higher moments of the random variable $\xi_n$ are not well-controlled, and $\xi_n$ are not uniformly anti-concentrated in $n$. Indeed, it is clear that for some fixed $M>0,C>0$ we have $\mathbb{P}(|y_{ij}|\leq M)\geq C,$ so that we have $\mathbb{P}(|\xi_n|\leq Mn^{-\frac{1}{\alpha}+(1-\frac{\alpha}{2})\tilde{\epsilon}_n})\geq C$, so that anti-concentration arguments (as in the proof of Theorem \ref{theorem1.2345}) cannot be used directly. Second, by our sampling procedure there are still $\gg n$ entries $(i,j)$ of $X$ where $\psi_{ij}=0$ and thus the randomness at site $(i,j)$ cannot be used directly. With high possibility each row has such entries, so we have to deal with $X$ with some randomness removed. Third, we have very weak control on the operator norm of the conditionally non-random part $\|(\mathbf{1}-\Psi)\circ Z\|$ containing the large entries, and we need a method to lower bound the smallest singular value for a rectangular matrix with normalized entries perturbed by a large deterministic matrix. This is very similar to the setting of \cite{tikhomirov2016smallest}. \subsection{A two-moment replacement principle for minimal singular value} We now address the first caveat raised in the last section. We show that we can replace $n^{\frac{1}{2}}\xi_n$ by some random variable which is uniformly anti-concentrated with all moments finite while not changing the value of $\sigma_{min}(X)$ too much, conditioned on the value of ${\Psi}$ and $Z$. Actually, we can replace $n^{\frac{1}{2}}\xi_n$ by a standard real Gaussian variable. This is the main result of \cite{brailovskaya2024universality}. The next theorem is essentially a corollary of \cite{brailovskaya2024universality}, Theorem 2.6 and 3.16. \begin{theorem}\label{derivation2.414} Let $N\geq n$. Let $T$ be a $N\times n$ matrix with decomposition $T=T_0+T_1$, both of which are $N\times n$ matrices, where $T_0$ is deterministic, $T_1$ has independent, mean zero coordinates $(t_{ij})$ such that $\sup_{i,j}\mathbb{E}[|t_{ij}|^2]\leq \frac{M^2}{n}$ for some fixed $M>1$ and let $q>0$ be such that $$\sup_{i,j}|t_{ij}|\leq q,\quad a.s.$$ Let $G=G_0+G_1$ be another $N\times n$ matrix where $G_0=T_0$, $G_1=(g_{ij})$ has independent mean zero Gaussian entries and share the same variance profile as $T_1$ i.e. $\mathbb{E}[g_{ij}^2]=\mathbb{E}[t_{ij}^2]$ for each $i,j$. On a common probability space supporting an independent copy of $G$ and $T$, we have: for any $t>0$, $$ \mathbb{P}(|\sigma_{min}(T)-\sigma_{min}(G)|\geq Cn^{-\frac{1}{2}}t^{\frac{1}{2}}+Cq^{\frac{1}{3}}t^{\frac{2}{3}}(\frac{N}{n})^\frac{1}{3}+Cqt)\leq 8N e^{-t}, $$ where $C>0$ is a universal constant that only depends on $M>0$. \end{theorem} When the aspect ratio $N/n$ is bounded, for the estimate to be meaningful we need $t\gg\log n$ and, as one can easily check that $\sigma_{min}(G)=O(1),$ we need $q\ll (\log n)^{-2}$, which is the reason we set \eqref{choicelowerbound}. The proof of Theorem \ref{derivation2.414} is deferred to Section \ref{section3.5}. When the aspect ratio $N/n$ is diverging, we will set the magnitude of $q$ differently, see Section \ref{divergingaspect}. \begin{remark}\label{majorremarks} We give several important remarks before applying this theorem. \begin{enumerate} \item Theorem \ref{derivation2.414} does not require that entries of $T_1$, or any of its normalized version, are uniformly anti-concentrated. \item All parameters in Theorem \ref{derivation2.414} depend only on the random part $T_1$ and do not depend on the deterministic part $T_0$. This is very important for our application. \item We can also compare $T$ to its free probability version $T_{\text{free}}$ and compute $\sigma_{min}(T_{\text{free}})$ using Lehner's formula \cite{lehner1999computing}. This innovative approach is proposed and developed carefully in \cite{bandeira2023matrix}. However, we have almost no control over the very large deterministic part $T_0$, and some randomness is removed from the random part $T_1$. This means that the computation of $\sigma_{min}(T_{\text{free}})$ will be very difficult. Instead, we take the other approach by lower bounding $\sigma_{min}(G)$, which draws us on the techniques in the subfield of quantitative invertibility of random matrices (see review \cite{MR4680362}) and in particular in a very similar situation as \cite{tikhomirov2016smallest}. \item The choice of $\tilde{\epsilon}_n$ in \eqref{choicelowerbound} is made only for Theorem \ref{derivation2.414} to produce meaningful quantitative estimates. This choice of $\tilde{\epsilon}_n$ is the source that produces a mismatch in the lower and upper bounds of Theorem \ref{theorem1.56} by a factor of $(\log n)^\frac{4(\alpha-2)}{2\alpha}$, and we believe that the choice of $\tilde{\epsilon}_n$ such that $n^{\alpha\tilde{\epsilon}_n}=C(\log n)^{-\frac{1}{2}}$, as in \eqref{choiceofepsilonn}, should be the optimal one under which a similar comparison theorem as Theorem \ref{derivation2.414} can be proven. Currently, there exists another method to bound the least singular value of a sparse rectangular matrix, which is \cite{dumitriu2024extreme}. The work \cite{dumitriu2024extreme} remarkably achieves the optimal $(\log n)^{-\frac{1}{2}}$ sparsity scale for a Bai-Yin type result to hold, but the proof requires the entries to have zero mean. In our setting, considering a large chunk of fixed non-zero entries is fundamentally important, and it is unclear how to generalize the proof of \cite{dumitriu2024extreme} to a matrix with arbitrary mean, so we do not take this approach. \end{enumerate} \end{remark} \subsection{When anti-concentration takes in} Having Theorem \ref{derivation2.414} in hand, the next step is to lower bound $\sigma_{min}(G)$ for a rectangular random matrix $G$ with Gaussian entries. As some randomness of $G$ is removed, the most direct proof does not work. Also, it appears that assuming a Gaussian entry does not simplify much the proof, so we make a more general statement for independent sub-Gaussian entries with uniform anti-concentration. Recall that a mean zero, variance one random variable $\xi$ is said to be $K$-sub-Gaussian for some $K>0$ if $\mathbb{E}[\exp(|\xi|^2/K^2)]\leq 2.$ \begin{Definition}\label{line276definition} For a given label $\Psi=(\psi_{ij})\in\{0,1\}^{N\times n}$, we define a random matrix $G$ with label $\Psi$ via $$G=G_0+G_1,$$ where $G_0$ is an arbitrary deterministic $N\times n$ matrix with no constraint imposed. We define $$G_1=(\psi_{ij}g_{ij})_{ij}$$ where $g_{ij}$ are i.i.d. mean zero, variance one, $K$- sub-Gaussian random variables satisfying an anti-concentration estimate: for some given $\alpha_\eqref{line276definition},\beta_\eqref{line276definition}>0,$ $$ \sup_{\lambda\in\mathbb{R}}\mathbb{P}\{|g_{ij}-\lambda|\leq\alpha_\eqref{line276definition} \}\leq 1-\beta_\eqref{line276definition}. $$ The matrix $G$ depends on its deterministic part $G_0$ and the label ${\Psi}$, but we shall use the notation $G$ to simplify the notations and keep the dependence on ${\Psi}$ implicit. \end{Definition} A lower bound on $\sigma_{min}(G)$ depends on the labels ${\Psi}$, as, for example, if there are too many $\psi_{ij}=0$ so that $G$ has a zero column, then $\sigma_{min}(G)=0$. We extract a high probability event and show that most labels $\Psi$ lead to a good lower bound on $\sigma_{min}(G).$ For a label $\Psi\in \{0,1\}^{N\times n}$ we denote by $P_{\Psi}$ the probability distribution of $G$ conditioning on its label being $\Psi$. More precisely, for any event $\mathcal{A}$ in the $\sigma$-algebra $\mathcal{F}$ of the probability space $(\Omega,\mathcal{F},\mathbb{P})$, we define \begin{equation}\label{definition1w} \mathbb{P}_{\Psi}(G\in\mathcal{A}):=\frac{\mathbb{P}(G\in \mathcal{A}, G\text{ has label }\Psi)}{\mathbb{P}(G\text{ has label }\Psi)}. \end{equation} This definition makes sense because, as $\Psi$ has distribution \eqref{generationoflabels}, $\mathbb{P}(G\text{ has label }\Psi)>0$ for any $\Psi\in\{0,1\}^{N\times n}.$ More generally, for a $\mathcal{F}_\Psi$-measurable event $\Omega_\Psi$ (i.e. an event depending only on the choice of label $\Psi$), we define analogously, for any $\mathcal{A}\in\mathcal{F}$, \begin{equation}\label{definition2label} \mathbb{P}_{\Omega_\Psi}(G\in A):=\frac{\mathbb{P}(G\in \mathcal{A}, \Psi\in\Omega_\Psi)}{\mathbb{P}(\Psi\in\Omega_\Psi)}. \end{equation} In the next theorem, the distribution of random label $\Psi$ does not need to follow \eqref{generationoflabels}. \begin{theorem}\label{universalbacks} Fix any $\delta>1$, consider $N\geq\delta n$ and let $G$ be as in Definition \eqref{line276definition}. Then there exists a subset of labels $\mathcal{D}\subset\{0,1\}^{N\times n}$ which satisfies, for a randomly generated label $\Psi$ such that each $\psi_{ij}$ is i.i.d. taking value in $\{0,1\}$ with $\mathbb{P}(\psi_{ij}=1)\geq \frac{3\delta+1}{4\delta}$, then there exists a positive integer $N_\eqref{universalbacks}\in\mathbb{N}$ such that for any $N\geq N_\eqref{universalbacks}$, \begin{enumerate} \item For this random choice of label $\Psi$, $$\mathbb{P}(\Psi\in\mathcal{D})\geq 1-\exp(-w_\eqref{universalbacks}N). $$ \item For any fixed label $\Psi\in \mathcal{D}$ and any deterministic $N\times n$ matrix $G_0$, we have $$ \mathbb{P}_{\Psi}(\sigma_{min}(G)\leq h_\eqref{universalbacks}\sqrt{N})\leq \exp(-w_\eqref{universalbacks}N), $$\end{enumerate} where the constants $h_\eqref{universalbacks}>0,w_\eqref{universalbacks}>0,N_\eqref{universalbacks}\in\mathbb{N}$ depend only on $\delta$, $\alpha_\eqref{line276definition}$ and $\beta_\eqref{line276definition}$ and they are independent of the choice of the deterministic part $G_0$ and the label $\Psi\in\mathcal{D}$. \end{theorem} The proof of Theorem \ref{universalbacks} draws much ideas from \cite{tikhomirov2016smallest}, yet we cannot use its main result directly. The difference is we need to fix a single label $\Psi$ a-priori and show $\sigma_{min}(G)$ is lower bounded uniformly for each $\Psi\in\mathcal{D}$, whereas \cite{tikhomirov2016smallest} does not involve this quasi-random selection procedure in its statement of main result. Nonetheless, the validity of our quasi-random selection is implicit in the proof in \cite{tikhomirov2016smallest} so that we only need to wrap up its proof in a different way. The construction of the subset $\mathcal{D}$ is inexplicit and depends on the choice of several families of $\epsilon$-nets. We will defer the proof of Theorem \ref{universalbacks} to Section \ref{section3.4theends}, and use it to now prove the main result of this paper. \subsection{Proof of main results: bounded aspect ratio} Now we are ready to prove the main theorems of this paper assuming Theorem \ref{derivation2.414} and Theorem \ref{universalbacks}. \begin{proof}[\proofname\ of Theorem \ref{theorem1.56}] The upper bound is already proven in Theorem \ref{theorem2.22}. For the lower bound, we take $\tilde{\epsilon}_n$ satisfying \eqref{choicelowerbound}. Then we can rewrite the resampling decomposition \eqref{resamplingdecision} as $$n^{-\frac{1}{\alpha}+(1-\frac{\alpha}{2})\tilde{\epsilon}_n}X\overset{\text{law}}{\equiv} \Psi\circ n^{-\frac{1}{\alpha}+(1-\frac{\alpha}{2})\tilde{\epsilon}_n}Y+(\mathbf{1}-\Psi)\circ n^{-\frac{1}{\alpha}+(1-\frac{\alpha}{2})\tilde{\epsilon}_n}Z.$$ By our choice of $\tilde{\epsilon}_n$, Lemma \ref{lemma2.34} implies that, if we set \begin{equation}\label{symbolT1} T_1:= n^{-\frac{1}{\alpha}+(1-\frac{\alpha}{2})\tilde{\epsilon}_n}\Psi\circ Y,\quad T_0:= n^{-\frac{1}{\alpha}+(1-\frac{\alpha}{2})\tilde{\epsilon}_n}(\mathbf{1}-\Psi)\circ Z ,\end{equation} then $T=T_0+T_1$ satisfies the assumption of Theorem \ref{derivation2.414} with $q=(\log n)^{-2.5}$ and for some $M>0$ depending only on $\alpha,C_\ell,C_u$. Then Theorem \ref{derivation2.414} implies that, for any $D>0$ we can find a constant $C_D>0$ such that, uniformly over the choice of label $\Psi\in\{0,1\}^{N\times n}$, \begin{equation} \mathbb{P}_{\Psi}(|\sigma_{min}(T)-\sigma_{min}(G)|\geq C_D(\log n)^{-1/6})\leq N^{-D},\end{equation} where $G$ is the Gaussian model associated to $T$, as defined in Theorem \ref{derivation2.414}. Then we apply Theorem \ref{universalbacks} to lower bound $\sigma_{min}(G)$. We may assume $T_1$ has normalized entry variance $\frac{1}{n}$: this can be achieved by multiplying a constant that depends only on $C_\ell,C_u,\alpha$, There exists $N_0\geq N_{\eqref{universalbacks}}$ such that for all $N\geq N_0$, $\mathbb{P}(\xi_{ij}=1)\geq\frac{3\delta+1}{4}$. Then by Theorem \ref{universalbacks} we find a set of labels $\mathcal{D}$ with $\mathbb{P}(\Psi\in\mathcal{D})\geq 1-\exp(-w_\eqref{universalbacks}N)$ and for each $\Psi\in\mathcal{D}$, Theorem \ref{universalbacks} implies $$ \mathbb{P}_{\Psi}(\sigma_{min}(G)\leq h)\geq 1-\exp(-w_\eqref{universalbacks}N), $$ where $h>0$ depends only on $\delta,\alpha,C_\ell,C_u$. We have dropped the $\sqrt{N}$ factor here as in this proof, the variance of each entry of $G$ is $\frac{1}{n}$ whereas in Theorem \ref{universalbacks} the variance of each entry of $G$ is 1, and also note that $N/n\to\delta<\infty$. We may enlarge $N_0$ so that $C_D(\log n)^{-\frac{1}{6}}\leq \frac{h}{2}$ and $\exp(-w_{\ref{universalbacks}}N)\leq N^{-D}$ for all $N\geq N_0$. Then for all $N\geq N_0$ and all $W\in\mathcal{D}$, $$ \mathbb{P}_W(\sigma_{min}(T)\leq \frac{h}{2})\leq N^{-D}+\exp(-w_{\ref{universalbacks}}N)\leq 2N^{-D}. $$ Now we can conclude the proof. For all $N\geq N_0$, $$\begin{aligned} \mathbb{P}(\sigma_{min}(X)&\leq \frac{h}{2}n^{\frac{1}{\alpha}}(\log n)^\frac{5(\alpha-2)}{2\alpha})=\mathbb{P}(\sigma_{min}(T)\leq \frac{h}{2})\\&\leq\mathbb{P}(\Psi\not\in D)+\sum_{W\in\mathcal{D}}\mathbb{P}(\Psi=W)\mathbb{P}_W(\sigma_{min}(T)\leq \frac{h}{2})\\&\leq \exp(-w_{\ref{universalbacks}}N)+2N^{-D}\leq 3N^{-D}. \end{aligned}$$ This completes the proof. \end{proof} \subsection{Diverging aspect ratio}\label{divergingaspect} Now we drop the assumption that $N/n$ is bounded. We sample $\Psi$ and $Y,Z$ slightly differently by using threshold depending on $N$ instead of $n$. More precisely, we now define \begin{equation}\label{generationoflabelsunbound} \psi_{ij}=\begin{cases}1,\quad\text{with probability } \mathbb{P}(|x_{ij}|\leq N^{\frac{1}{\alpha}-\tilde{\epsilon}_N}),\\0,\quad \text{with probability } \mathbb{P}(|x_{ij}|\geq N^{\frac{1}{\alpha}-\tilde{\epsilon}_N}), \end{cases} \end{equation} and for each interval $\tilde{I}\subset\mathbb{R}$, we define \begin{equation}\label{resampling1} \mathbb{P}(y_{ij}\in\tilde{I})=\frac{\mathbb{P}\left(x_{ij}\in\tilde{I}\cap[-N^{\frac{1}{\alpha}-\tilde{\epsilon}_N},N^{\frac{1}{\alpha}-\tilde{\epsilon}_N}]\right)}{\mathbb{P}(x_{ij}\in[-N^{\frac{1}{\alpha}-\tilde{\epsilon}_N},N^{\frac{1}{\alpha}-\tilde{\epsilon}_N}])},\end{equation} where $\tilde{\epsilon}_N$ is a cutoff value to be determined later, which is either a function of $N$, or a function of both $N$ and $n$. We define the law of $z_{ij}$ similarly, by swapping all factors of $n$ by $N$ in the definition \eqref{lawofz}. The proof of Theorem \ref{lowergeneral} is very similar to that of Theorem \ref{theorem1.56}. \begin{proof}[\proofname\ of Theorem \ref{lowergeneral}] We only sketch the places where we need to do differently compared to the proof of Theorem \ref{theorem1.56}. We again take the truncation \eqref{generationoflabelsunbound} and resampling \ref{resampling1} procedure to define the label $\Psi$, with the procedure applied to $x_{ij}$. To account for the coefficients, we replace $y_{ij}$ by $y_{ij}a_{ij}$ and $z_{ij}$ by $z_{ij}a_{ij}$ in a similar decomposition as in \eqref{resamplingdecision}. That is, we now have the equivalence in law $A\overset{\text{law}}{\equiv} {\Psi}\circ Y+(\mathbf{1}-{\Psi})\circ Z$, where $Y=(a_{ij}y_{ij})$ and $Z=(a_{ij}z_{ij})$. Then we define $T_0,T_1$ via $$ T_1:= N^{-\frac{1}{\alpha}+(1-\frac{\alpha}{2})\tilde{\epsilon}_N}\Psi\circ Y,\quad T_0:= N^{-\frac{1}{\alpha}+(1-\frac{\alpha}{2})\tilde{\epsilon}_N}((\mathbf{1}-\Psi)\circ Z+B) $$ so that $$ A+B=N^{\frac{1}{\alpha}+(\frac{\alpha}{2}-1)\tilde{\epsilon}_n}T=N^{\frac{1}{\alpha}+(\frac{\alpha}{2}-1)\tilde{\epsilon}_n}(T_0+T_1) $$ where we absorb the deterministic part $B$ into $T_0$ and replace $n$ by $N$ in both $T_0,T_1$. We take $\tilde{\epsilon}_N$ such that $$N^{\alpha\tilde{\epsilon}_N}=(\log N)^5.$$ As can be checked similarly to Lemma \ref{lemma2.34}, each entry of $T_1$ has mean zero, variance bounded by $CN^{-1}$ for some $C>0$ depending on $A_2,C_\ell,C_u$, and each entry is almost surely bounded by $(\log N)^{-2.5}$. Then we apply Theorem \ref{derivation2.414} to $\sqrt{\frac{N}{n}}T=\sqrt{\frac{N}{n}}(T_0+T_1)$, where we multiply a $\sqrt{\frac{N}{n}}$ factor because Theorem \ref{derivation2.414} took a normalization of entry variance to be $n^{-1}$ . Again let $\sqrt{\frac{N}{n}}G=\sqrt{\frac{N}{n}}(G_0+G_1)$, where $G$ is the Gaussian model of $T$ defined in Theorem \ref{derivation2.414}. In applying Theorem \ref{derivation2.414} we will take $q=\sqrt{\frac{N}{n}}(\log N)^{-2.5}$. Then for any $D>0$ we can find a constant $C_D>0$ such that, uniformly over the choice of label $\Psi\in\{0,1\}^{N\times n}$, \begin{equation} \label{495proofofupperbound} \mathbb{P}_{\Psi}\left(\left|\sigma_{min}(\sqrt{\frac{N}{n}}T)-\sigma_{min}(\sqrt{\frac{N}{n}}G)\right|\geq C_D\sqrt{\frac{N}{n}}(\log N)^{-1/6}\right)\leq N^{-D}.\end{equation} The next step is to apply Theorem \ref{universalbacks} to prove that we can find a set of labels $\mathcal{D}$ with $\mathbb{P}(\Psi\in\mathcal{D})\geq 1-\exp(-w_\eqref{lemma3.10}D)$, and that for any $W\in \mathcal{D}$ we have $$ \mathbb{P}_W(\sigma_{min}(\sqrt{N}G)\leq \frac{h}{2}\sqrt{N})\leq 2N^{-D} $$ for some $h=h(C_\ell,C_u,A_1,A_2,\delta,\alpha)>0$ (The variance of each entry of $\sqrt{N}G$ is normalized to be $\sim 1$). Combined with \eqref{495proofofupperbound}, this completes the proof of Theorem \ref{lowergeneral} following exactly the same lines as in the proof of Theorem \ref{theorem1.56}. There is an issue when applying Theorem \ref{universalbacks} to $G$: the entries of $G_1$ are not identically distributed. However, by assumption on $a_{ij}$ the variance of each entry of $\sqrt{N}G_1$ is uniformly bounded from below. As the Gaussian distribution is infinitely divisible, we can decompose $G_1$ as $G_1=G_2+G_3$ with $G_2$, $G_3$ independent and entries of $G_2$ are i.i.d. Then we can apply Theorem \ref{universalbacks} to $(G_0+G_3)+G_2$ and finish the proof. \end{proof} Finally we prove the upper bound $\sigma_{min}(X)$ in Proposition \ref{propositionunbounded} when $N/n$ is unbounded. \begin{proof}[\proofname\ of Proposition \ref{propositionunbounded}] This will be a careful adaptation of the proof of Theorem \ref{theorem2.22}. Recall $\psi_j$ denotes the $j$-th column of $\Psi$, then for each $j$ $$ \mathbb{P}(\psi_j=\mathbf{1}_N)\geq \exp(-C_uN^{\alpha\tilde{\epsilon}_N}), $$ and since different $\psi_j$ are independent, $$ \mathbb{P}(\psi\text{ has no all-ones column})\leq \exp(-n\exp(-C_u N^{\alpha\tilde{\epsilon}_N})). $$ Denoting by $Y^\mathbf{m}$ the minor of $Y$ after removing all columns $j$ that are not all-ones column, i.e., those $j$ such that $\psi_j\neq \mathbf{1}_N$. Then we have $\sigma_{min}(X)\leq\|Y^\mathbf{m}\|$. Denote by $(Y^\mathbf{m})_{i\cdot}$, resp. $(Y^\mathbf{m})_{\cdot j}$ the $i$-th row, resp. the $j$-th column of $Y^\mathbf{m}$. Then by Rosenthal inequality for any $q>0$, $$\mathbb{E}[\|(Y^{\mathbf{m}})_{i\cdot}\|_2^q]\leq C^q (N^{\frac{1}{\alpha}-\tilde{\epsilon}_N})^q(N^{\alpha\tilde{\epsilon}_N})^\frac{q}{2}+q^\frac{q}{4}(N^{\alpha\tilde{\epsilon}_N})^{\frac{q}{4}}, $$ where $C>0$ depends only on $C_u$, $\alpha$ and $\delta$. We take $q=2\log N$ and apply Lemma \ref{kemma2.2spectralradius} with this choice of $q$ to deduce $$ \mathbb{E}\|Y^{\mathbf{m}}\|_2^q\leq 2C^q(N^{\frac{1}{\alpha}-\tilde{\epsilon}_N})^qN(N^{\alpha\tilde{\epsilon}_N})^\frac{q}{2}. $$ Then applying Markov's inequality, for any $D>0$, we can find $C_D>0$ such that $$ \mathbb{P}\{\|Y^{\mathbf{m}}\|\geq C_DN^{\frac{1}{\alpha}-(1-\frac{\alpha}{2})\tilde{\epsilon}_N}\}\leq N^{-D}+\exp(-n\exp(-C_u N^{\alpha\tilde{\epsilon}_N})) $$ where $C_D>0$ depending on $D,C_u,C_\ell$ and $\delta$. Now we take some $\mathbf{b}\in(0,1)$ and define $\tilde{\epsilon}_N$ via \begin{equation} N^{\alpha\tilde{\epsilon}_N}=\frac{(1-\mathbf{b})\log n}{ C_u}. \end{equation} This choice of $\tilde{\epsilon}_N$ completes the proof. \end{proof} \section{Proof of technical results} This section collects the proof of Theorem \ref{derivation2.414} and \ref{universalbacks}. Section \ref{subsection3.1} towards Section \ref{section3.4theends} contain the proof of Theorem \ref{universalbacks} and Section \ref{section3.5} contains the proof of Theorem \ref{derivation2.414}. \subsection{Anti-concentration: preliminary results}\label{subsection3.1} As a preparation for the proof of Theorem \ref{universalbacks}, we recall several facts from \cite{tikhomirov2016smallest}. The first is about bounding the least singular value without an operator norm control. For a subspace $E\subset\mathbb{R}^n$ we denote by $\operatorname{proj}_E$ the orthogonal projection of $\mathbb{R}^n$ onto $E$, and denote by $E^\perp$ the orthogonal complement of $E$ in $\mathbb{R}^n$. \begin{Proposition}\label{proposition3.13.13.1}[\cite{tikhomirov2016smallest},Proposition 3] Let $D_1,D_2$ be $N\times n$ (deterministic) matrices and $D=D_1+D_2$. Fix a set of vectors $S\subset\mathbb{S}^{n-1}$. Assume we can find $h,\epsilon>0$, some subset $\mathcal{N}\subset\mathbb{R}^n$, and linear subspaces $\{E_{y'}\subset\mathbb{R}^n:y'\in\mathcal{N}\}$ such that $y'\in E_{y'}$ for all $y'\in\mathcal{N}$, and that (1) For all $y'\in\mathcal{N}$, $$ \operatorname{dist}(D_1y',D(E_{y'}^\perp)+D_2(E_{y'}))\geq h; $$ (2) For each $y\in S$ we can find $y'\in\mathcal{N}$ with $$ \|\operatorname{Proj}_{E_{y'}}(y)-y'\|\leq\epsilon. $$ Then we have $$ \inf_{y\in S}\|Dy\|\geq h-\epsilon\|D_1\|. $$\end{Proposition} We also need a few facts about anti-concentration and subspace projection. For a random variable $\xi$ we define its Lévy concentration function via \begin{equation} \label{levyconcentrationf}\mathcal{Q}(\xi,\alpha)=\sup_{\lambda\in\mathbb{R}}\mathbb{P}(|\xi-\lambda|\leq\alpha). \end{equation} We will also need a conditional version of Lévy concentration function. For any label $\Psi\in\{0,1\}^{N\times n}$ or any event $\Omega_\Psi\in\mathcal{F}_\Psi$ we define analogously \begin{equation}\label{631analogously} \mathcal{Q}_\Psi(\xi,\alpha)=\sup_{\lambda\in\mathbb{R}}\mathbb{P}_\Psi(|\xi-\lambda|\leq\alpha),\quad \mathcal{Q}_{\Omega_\Psi}(\xi,\alpha)=\sup_{\lambda\in\mathbb{R}}\mathbb{P}_{\Omega_\Psi}(|\xi-\lambda|\leq\alpha), \end{equation}where we recall \eqref{definition1w}, \eqref{definition2label} for the notation of $\mathbb{P}_\Psi$ and $\mathbb{P}_{\Omega_\Psi}.$ \begin{theorem}\label{theorem3.2}[\cite{MR131894}] Fix $k\in\mathbb{N}$ and consider $\xi_1,\cdots,\xi_k$ independent random variables. Fix real numbers $h_1,\cdots,h_k>0$. Then given any $h\geq \max_{j=1,\cdots,k}h_j$, $$ \mathcal{Q}(\sum_{j=1}^k\xi_j,h)\leq C_{\eqref{theorem3.2}}h\left(\sum_{j=1}^k(1-\mathcal{Q}(\xi_j,h_j))h_j^2\right)^{-1/2} $$ for a universal constant $C_{\eqref{theorem3.2}}>0$. \end{theorem} Then we have a convenient bound on anti-concentration of subspace projection \begin{corollary}\label{corollary3.3}[\cite{tikhomirov2016smallest}, Corollary 6] Consider $X=(X_1,\cdots,X_m)$ a vector with independent coordinates with $\mathcal{Q}(X_i,h)\leq 1-\tau$ for some $h>0,\tau\in(0,1)$ and for each $i=1,\cdots,m$. For any $\ell\in\mathbb{N}$, and any $d$-dimensional fixed subspace ($d\leq m$) $E\subset\mathbb{R}^m$, we have $$ \mathcal{Q}(\operatorname{Proj}_EX,h\sqrt{d}/\ell)\leq (C_{\eqref{corollary3.3}}/\sqrt{\ell\tau})^{d/\ell}, $$ where $C_{\eqref{corollary3.3}}$ is some universal constant. \end{corollary} We recall a well-known operator norm bound for inhomogeneous random matrices with sub-Gaussian entries. This can be found in various places such as \cite{rudelson2010non} or \cite{benhamou2018operator}. Whereas these proofs are stated for matrix with i.i.d. entries, they are based on the method of moments and adding a label $\psi_{ij}\in\{0,1\}$ only decreases the moments, so the result still applies. \begin{lemma}\label{lemma3.444} Fix a label $\Psi=(\psi_{ij})\in\{0,1\}^{N\times n}$, and let $W=(\psi_{ij}w_{ij})$ be an $N\times n$ random matrix ($N\geq n$) where $w_{ij}$ are i.i.d. random variables with mean zero, variance one and are $K$-sub-Gaussian. Then we can find $C_{\eqref{lemma3.444}}>0$ depending only on $K$ (and independent of $\Psi$) such that $$ \mathbb{P}\{\|W\|\geq C_{\eqref{lemma3.444}}\sqrt{N}\}\leq\exp(-N). $$ \end{lemma} We finally quote a purely technical result from \cite{tikhomirov2016smallest} which gives improved cardinality counting of the net. This lemma is possibly trivial when $\xi$ is a standard Gaussian, but let us state it that way. \begin{lemma}\label{lemma3.56}(\cite{tikhomirov2015limit}, Lemma 14) Consider $\xi$ a random variable satisfying, for $z\in\mathbb{R},\gamma>0,N\in\mathbb{N}$: $$ \min(\mathbb{P}(z-\sqrt{N}\geq\xi\leq z-1),\mathbb{P}(z+1\leq\xi\leq z+\sqrt{N}))\geq\gamma. $$ Then we can find some integer $\ell\in[0,\lfloor\log_2\sqrt{N}\rfloor]$, some $\lambda\in\mathbb{R}$, some Borel subsets $H_1,H_2\subset[-2^{\ell+2},2^{\ell+2}]$ satisfying $$\operatorname{dist}(H_1,H_2)\geq 2^\ell, \quad \min(\mathbb{P}\{\xi-\lambda\in H_1\},\mathbb{P}\{\xi-\lambda\in H_2\})\geq c_{\eqref{lemma3.56}}\gamma 2^{-\ell/8}$$ where $c_{\eqref{lemma3.56}}>0$ is a universal constant, and such that for $H=H_1\cup H_2$ we have $$\mathbb{E}[(\xi-\lambda)1_{\xi-\lambda\in H}]=0.$$ \end{lemma} \subsection{Sphere decomposition and cardinality of nets} We now outline a decomposition of the sphere $\mathbb{S}^{n-1}$ into three subsets, following \cite{tikhomirov2016smallest}. \begin{Definition} For fixed $\theta>0,m>0$ we define subsets $\mathbb{S}_p^{n-1}(\theta)$ and $\mathbb{S}_a^{n-1}(m)$ of $\mathbb{S}^{n-1}$ via \begin{enumerate} \item The set $\mathbb{S}_p^{n-1}(\theta)$ of $\theta$-peaky vectors consist of unit vectors in $\mathbb{R}^n$ that have $\ell_\infty^n$ norm at least $\theta$. \item A vector $y\in \mathbb{S}^{n-1}$ is called $m$-sparse if $|\operatorname{supp}y|\leq m$. We then call $y\in\mathbb{S}^{n-1}$ almost $m$-sparse if we can find $J\subset\{1,\cdots,n\}$ with cardinality bounded by $m$, and $\|y\chi_J\|\geq\frac{1}{2}$. We use the notation $\mathbb{S}_a^{n-1}(m)$ to denote the set of almost $m$-sparse vectors of $\mathbb{S}^{n-1}$. \end{enumerate}\end{Definition} The cardinality of nets for $\mathbb{S}_a^{n-1}$ can be bounded by the following lemma. \begin{lemma}\label{lemma3.7}[\cite{tikhomirov2015limit}, Lemma 12] We can find a universal constant $C_{\eqref{lemma3.7}}>0$ so the following holds. Given $n,m\in\mathbb{N},n\geq m$, $\epsilon\in(0,1]$, a subset $S\subset\mathbb{S}^{n-1}$, and let $T\subset\mathbb{B}_2^n$ (the unit ball in $\ell_2^n$) be a subset of $m$-sparse vectors that satisfy \begin{equation}\label{line444} \text{for each }y\in S\text{ we can find }x=x(y)\in T\text{ such that }y\chi_{\text{supp}x}=x. \end{equation} Then we can find a set $\mathcal{N}\subset T$ with cardinality bounded by $(\frac{C_{\eqref{lemma3.7}}n}{\epsilon m})^m$ so that given any $y\in S$ we can find $y'=y'(y)\in\mathcal{N}$ satisfying $\|y'-\chi_{\operatorname{supp}y'}y\|\leq\epsilon.$ \end{lemma} Finally we consider the rest of the sphere $\mathbb{S}^{n-1}\setminus (\mathbb{S}_p^{n-1}(\theta)\cup\mathbb{S}_a^{n-1}(m))$, and find a net. \begin{lemma}\label{lemma3.82}(\cite{tikhomirov2015limit}, Lemma 16) Fix some $N\geq n\geq m\geq 1$. For any $y\in\mathbb{S}^{n-1}\setminus \mathbb{S}_a^{n-1}(\sqrt{N})$ we can find s subset $J=J(y)\subset\{1,2,\cdots,n\}$ with $|J|\leq m$, $\|y\chi_J\|\geq\frac{1}{2}\sqrt{\frac{m}{n}}$, and that $\|y\chi_J\|_\infty\leq\frac{1}{\lfloor N^{1/4}\rfloor}.$ \end{lemma} \subsection{Quasi-random selection of labels} We first prove that quasi-random selection of labels holds for a single vector. \begin{lemma}\label{lemma3.9} Fix some $d,r>0$. Let $(g_{i1},g_{i2},\cdots g_{in})$ be a random vector of i.i.d. coordinates. Consider $H\subset\mathbb{R}$ a Borel subset with $H=H_1\cup H_2$ for two Borel sets $H_1,H_2$, where $\operatorname{dist}(H_1,H_2)\geq d$, and $\min(\mathbb{P}(g_{ij}\in H_1),\mathbb{P}(g_{ij}\in H_2))\geq r$. Given any $t>0$ we define $$ h_{\eqref{lemma3.9}}=\frac{1-\delta^{-1/4}}{C_{\eqref{theorem3.2}}}\sqrt{ \frac{r}{16} }td, $$ and let $y\in\mathbb{R}^n$ be a vector which satisfy $\|y\|_2\geq t$, $\|y\|_\infty\leq \frac{2h_{\eqref{lemma3.9}}}{d}$. Let $(g_{i1}',g_{i2}',\cdots,g_{in}')$ be an i.i.d. copy of $(g_{i1},\cdots,g_{in})$. Then \begin{equation}\label{andletsumover} \mathbb{P}\left\{\left|\sum_{j=1}^n \psi_{ij}(g_{ij}-g_{ij}')y_j\right|\leq h_{\eqref{lemma3.9}}\right\} \leq 1-\delta^{-1/4}.\end{equation} \end{lemma} We note that the randomness is taken jointly over $g_{ij},g_{ij}'$ and $\psi_{ij}$. \begin{proof} From joint independence and the definition of $\psi_{ij}$, we have $$ \mathbb{P}\{\psi_{ij}|g_{ij}-g_{ij}'|\geq d\}\geq \mathbb{P}\{g_{ij}\in H_1,g_{ij}'\in H_2,\psi_{ij}=1\}+\mathbb{P}\{g_{ij}\in H_2,g_{ij}'\in H_1,\psi_{ij}=1\} \geq \frac{r}{2}, $$ where we use $\mathbb{P}(\psi_{ij}=1)>\frac{1}{2}$. Since $g_{ij}-g_{ij}'$ has symmetric distribution, we deduce that $\mathcal{Q}(g_{ij}-g_{ij}',\frac{d}{2})\leq 1-\frac{r}{4}.$ By our assumption, $h_{\eqref{lemma3.9}}\geq \frac{d|y_j|}{2}$ for each $j$. Thus, applying Theorem \ref{theorem3.2}, we, have that $$\begin{aligned} \text{LHS of } \eqref{andletsumover}&\leq C_{\eqref{theorem3.2}}h_{\eqref{lemma3.9}}(\frac{1}{4}\sum_{j=1}^n(1-\mathcal{Q}((g_{ij}-g_{ij}')y_j,\frac{|y_j|d}{2})(y_jd)^2)^{-1/2}\\&\leq C_{\eqref{theorem3.2}} h_{\eqref{lemma3.9}}(\frac{r}{16}\sum_{j=1}^d(y_jd)^2)^{-1/2}=1-\delta^{-1/4}. \end{aligned}$$ This concludes the proof. \end{proof} Now we can conclude the quasi-random label selection for a fixed vector $y$ with sufficient mass and which is not peaky. \begin{lemma}\label{lemma3.10} Fix $\delta>1$ and $N\geq\delta n$. Fix $t>0,d>0.$ For any $y\in\mathbb{R}^n$ with $\|y\|\geq t,$ $\|y\|_\infty\leq \frac{2h_{\eqref{lemma3.9}}}{d}$ we can find a subset of labels $\Omega_y\in\{0,1\}^{N\times n}$ such that for a randomly chosen label $\Psi,$ $$ \mathbb{P}(\Psi\in\Omega_y)\geq 1-\exp(-w_{\eqref{lemma3.10}}N) $$ and that for any $W_y\in\Omega_y$, $$ \mathbb{P}_{W_y}\{\operatorname{dist}(G_1y,V_{G_0,G_1}(E))\leq h_{\eqref{lemma3.10}}h_{\eqref{lemma3.9}}\sqrt{N})\}\leq 2\exp(-w_{\eqref{lemma3.10}}N) $$ for constants $h_{\eqref{lemma3.10}},w_{\eqref{lemma3.10}}$ depending only on $\delta$. Here $E:=\operatorname{span}\{e_j\}_{j\in\operatorname{supp}y},$ and for any subspace $E\subset\mathbb{R}^n$, we denote by $V_{G_0,G_1}(E)$ the following subspace \begin{equation}\label{vgog1e}V_{G_0,G_1}(E):=(G_0+G_1)(E^\perp)+G_0(E).\end{equation} \end{lemma} \begin{proof} For $i\in\{1,2,\cdots,N\}$ and $J\subset\{1,2,\cdots,n\}$ denote by $$ \Omega_J^i=\{\omega\in\Omega: \psi_{ij}(\omega)=1,j\in J;\quad \psi_{ij}(\omega)=0,j\notin J\}. $$ For each $i,j$ let $g_{ij}'$ be an i.i.d. copy of $g_{ij}$ with $g_{ij}'$ mutually independent. In this proof we take $$\tau=\frac{1}{2}(\delta^{-1/4}-\delta^{-1/3}).$$ Suppose that a subset $J\subset\{1,2,\cdots,n\}$ satisfies, for the fixed $y\in\mathbb{R}^n$, \begin{equation} \label{assumptiononthej}\mathbb{P}_{\Omega_J^i}\left\{|\sum_{j=1}^n \psi_{ij}(g_{ij}-g_{ij}')y_j|>h_{\eqref{lemma3.9}}\right\}\geq 2\tau, \end{equation} then for any $\lambda\in\mathbb{R}$ we have $$\begin{aligned} &\mathbb{P}_{\Omega^i_J}\{\lambda-\frac{h_{\eqref{lemma3.9}}}{2}\leq\sum_{j=1}^n\psi_{ij}g_{ij}y_j\leq\lambda+\frac{h_{\eqref{lemma3.9}}}{2}\}^2\\&\leq\mathbb{P}_{\Omega^i_J}\{|\sum_{j=1}^n\psi_{ij}(g_{ij}-g_{ij}')y_j|\leq h_{\eqref{lemma3.9}}\}\leq 1-2\tau, \end{aligned}$$ whence we have, recalling the definition in \eqref{631analogously}, $$ \mathcal{Q}_{\Omega^i_J}(\sum_{j=1}^n\psi_{ij}g_{ij}y_j,\frac{h_{\eqref{lemma3.9}}}{2})\leq\sqrt{1-2\tau}\leq 1-\tau. $$ For each $i$ let $L_i$ be a subset of $\{0,1\}^{\{1,2,\cdots,n\}}$ defined as $$ L_i:=\left\{J\subset\{1,2,\cdots,n\}:\quad \mathcal{Q}_{\Omega^i_J}(\sum_{j=1}^n \psi_{ij}g_{ij}y_j,\frac{h_{\eqref{lemma3.9}}}{2})\leq 1-\tau\right\};\quad \mathcal{E}_i=\cup_{j\in L_i}\Omega_J^i, $$ then the previous argument implies that any $J$ satisfying \eqref{assumptiononthej} must satisfy $J\in L_i.$ From the law of total probability, we have $$ \mathbb{P}\{|\sum_{j=1}^n\psi_{ij}(g_{ij}-g_{ij}')y_j|>h_{\eqref{lemma3.9}}\}=\sum_J \mathbb{P}_{\Omega^i_J}\{|\sum_{j=1}^n\psi_{ij}(g_{ij}-g_{ij}')y_j|>h_{\eqref{lemma3.9}}\}\mathbb{P}(\Omega^i_J). $$ Then combining Lemma \ref{lemma3.9} and the previous deductions we get that $$ \delta^{-1/4}\leq \sum_{J\in L_i}\mathbb{P}(\Omega^i_J)+2\tau\sum_{J\notin L_i}\mathbb{P}(\Omega^i_J)\leq 2\tau+\mathbb{P}(\mathcal{E}_i). $$ We conclude that $\mathbb{P}(\mathcal{E}_i)\geq\delta^{-1/3}.$ Now each $\mathcal{E}_i$ are independent and $\mathbb{P}(\mathcal{E}_i)\geq \delta^{-1/3}$. Then by Bernstein's inequality, if we set $$ \Omega_y=\{\Psi\in\{0,1\}^{N\times n}:|\{i\in\{1,2,\cdots,N\}: \{j\in\{1,\cdots,n\}:\psi_{i,j}=1\}\in L_i \}|\geq N\delta^{-1/2}\}, $$ then for a randomly selected label $\Psi$ we have $\mathbb{P}(\Psi\in\Omega_y)\geq 1-\exp(-w_{\eqref{lemma3.10}}N)$ for some $w_{\eqref{lemma3.10}}>0$ only relying on $\delta$. We have taken the notation $\Omega_y$ as this event implicitly depends on the vector $y\in\mathbb{R}^n$ chosen, and we have made a slight abuse of notation by identifying an event on the labels $\Psi$ with a collection of labels $\Psi\in\{0,1\}^{N\times n}$, so that $\Omega_y$ means the collection of labels $\Psi=(\psi_{ij})$ on $\{0,1\}^{N\times n}$ such that for at least $N\delta^{-1/2}$ many $i\in [n]$ we have $\mathcal{Q}(\sum_{j=1}^n\psi_{ij}g_{ij}y_j,\frac{h_{\eqref{lemma3.9}}}{2})\leq 1-\tau.$ Now for this $y\in\mathbb{R}^n$ let $E_y=\operatorname{span}\{e_j\}_{j\in \operatorname{supp}y}$. For each $\Psi\in\Omega_y$, denote by $$ m:=|\{i\in \{1,2,\cdots,N\}:\mathcal{Q}_{\Psi}(\sum_{j=1}^n \psi_{ij}y_j,\frac{h_{\eqref{lemma3.9}}}{2})\leq 1-\tau\}|, $$ then $m\geq N\delta^{-1/2}\geq \delta^\frac{1}{2}n.$ Thus, by Corollary \ref{corollary3.3}, we choose $\kappa=\delta^{-1/2}-\delta^{-1}$ and consider any $n$-dimensional fixed subspace $F\subset\mathbb{R}^N$, take $\ell=4(C_{\eqref{theorem3.2}}C_{\eqref{corollary3.3}})^2/\tau$, we get that for any label $W_y\in \mathcal{E}_y$, $$ \mathbb{P}_{\Omega_{W_y}}\{\text{dist}(G_1y,F)\leq \frac{h_{\eqref{lemma3.9}}}{2\ell}\sqrt{\kappa N}\}\leq 2^{-\kappa N/\ell}. $$ Since the $n$-dimensional subspace $V_{G_0,G_1}(E)$ is independent of $G_1y,$ we may take $F=V_{G_0,G_1}(E)$ and conclude the proof. \end{proof} Now we can take the union bound over almost sparse vectors. \begin{Proposition}\label{proposition3.11}(Almost sparse vectors from $\mathbb{S}_a^{n-1}(\sqrt{N})$). Fix $\gamma>0$ and $\delta>1$ we can find $N_\eqref{proposition3.11}$ and $h_\eqref{proposition3.11}>0$ depending only on $\gamma,\delta$. We define $\theta_\eqref{proposition3.11}=\frac{1-\delta^{-1/4}}{C_\eqref{corollary3.3}}\sqrt{\frac{\gamma}{16}}$ and let $N\geq\max(N_\eqref{proposition3.11},\delta n)$. Suppose that $g_{ij}$ satisfies, for some $z\in\mathbb{R}$, that $$ \min(\mathbb{P}\{z-\sqrt{n}\leq g_{ij}\leq z-1\},\mathbb{P}\{z+1\leq g_{ij}\leq z+\sqrt{n}\}\geq \gamma. $$ Then denote by $S:=S_a^{n-1}(\sqrt{N})\setminus \mathbb{S}_p^{n-1}(\theta)$, we can find a collection of labels $\Omega_a\subset\{0,1\}^{N\times n}$ with $\mathbb{P}(\Psi\in\Omega_a)\geq 1-\exp(-w_\eqref{lemma3.10}N/2)$, such that for any $W_a\in \Omega_a$, $$ \mathbb{P}_{W_a}\{\inf_{y\in S}\|G_0y+G_1y\|\leq h_\eqref{proposition3.11}\sqrt{N} \}\leq\exp(-w_\eqref{lemma3.10}N/2)-\exp(-N). $$ \end{Proposition} \begin{proof} For any fixed $\gamma>0$, $\delta>1$ we take $d=2$, $r=\gamma$ and $t=\frac{1}{2}$. Let $N_\eqref{proposition3.11}$ be the smallest integer larger than $\frac{2\max(1,C_\eqref{lemma3.444})}{h_\eqref{lemma3.9}h_\eqref{lemma3.10}}$ such that $$ 2(C_\eqref{lemma3.7}N)^{3\sqrt{N}}\leq\exp(w_\eqref{lemma3.10}N/2). $$ Assume without loss of generality $z=0$ (otherwise, we replace $G_0,G_1$ by $G_0-z\mathbf{1},G_1+z\mathbf{1}$). Take $H_1=[-\sqrt{N},1],H_2=[1,\sqrt{N}]$ and $H=H_1\cup H_2$. Now we denote by $T\subset B_2^n$ the $\sqrt{N}$-sparse vectors with $\ell^2$ norm no less than $\frac{1}{2}$ and $\ell^\infty$ norm bounded by $\theta$. Thanks to Lemma \eqref{lemma3.7} we find a finite net $\mathcal{N}\subset T,$ $|\mathcal{N}|\leq ( C_\eqref{lemma3.7}N)^{3\sqrt{N}}$, and that for any $y\in S$ we have $y'=y'(y)\in \mathcal{N}$ having $|y\chi_{\operatorname{supp}y'}-y'\|\leq N^{-2}.$ Let $$ \Omega_a:=\cap_{y\in\mathcal{N}}\Omega_y, $$where $\Omega_y$ was defined in Lemma \ref{lemma3.10}. Then combining Lemma \ref{lemma3.10} with a simple union bound, we have $\mathbb{P}(\Psi\in\Omega_a)\geq 1-\exp(-w_{\eqref{lemma3.10}}N/2)$. For any $W_a\in\Omega_a$, consider the event $$\mathcal{E}_{W_a}=\{w\in\Omega:\Psi=W_a, \operatorname{dist}(G_1y,V_{G_0,G_1}(E_{y'})\geq h_\eqref{lemma3.9}h_\eqref{lemma3.10}\sqrt{N}\text{ for any }y'\in\mathcal{N}\}. $$ where we recall that $E_{y'}=\operatorname{span}\{e_j\}_{j\in\operatorname{supp}y'}$. Then $$\mathbb{P}_{W_a}(\mathcal{E}_{w_a})\geq 1-2|\mathcal{N}|\exp(-w_\eqref{lemma3.10}N)\geq 1-\exp(-w_\eqref{lemma3.10}N/2). $$ Now we take any $w\in\mathcal{E}_{W_a}$, on this event we have $$ \operatorname{dist}(G_1(\omega)y',G_1(\omega)(E_{y'}^\perp)+G_0(\mathbb{R}^n))\geq h_\eqref{lemma3.9}h_\eqref{lemma3.10}\sqrt{N}. $$ Now we apply Proposition \ref{proposition3.13.13.1} and Lemma \eqref{lemma3.444} which shows $\|G_1\|\leq C_\eqref{lemma3.444}\sqrt{N}$ with probability at least $1-\exp(-N)$, to deduce that for this $w\in\mathcal{E}_{W_a}$, $$ \inf_{y\in S}\|(G_0+G_1)(\omega)y\|\geq h_\eqref{lemma3.9}h_\eqref{lemma3.10}\sqrt{N}-C_\eqref{lemma3.444}N^{-1/2}\geq \frac{1}{2}h_\eqref{lemma3.9}h_\eqref{lemma3.10}\sqrt{N}. $$ \end{proof} Next we take the bound over peaky vectors, i.e. vectors that have a large $\ell^\infty$ norm. \begin{lemma}\label{lemma3.12} Fix $\delta>1$, $\gamma>0$ and let $N,n$ be that $N\geq \delta n$. Assume that $\mathcal{Q}(g_{ij},1)\leq 1-\gamma$. Then for any fixed $\theta>0$ there exists a collection of labels $\Omega_p\subset\{0,1\}^{N\times n},$ with $\mathbb{P}(\Psi\in\Omega_p)\geq 1-\exp(-w_\eqref{lemma3.12}n)$ such that for any $W_p\in\Omega_p,$ $$ \mathbb{P}_{W_p}\{\inf_{y\in \mathbb{S}_p^{n-1}(\theta)}\|Gy\|\leq h_\eqref{lemma3.12}\theta\sqrt{N}\}\leq n\exp(-w_\eqref{lemma3.12}N), $$ where $h_\eqref{lemma3.12}$ and $w_\eqref{lemma3.12}$ depend only on $\delta$ and $\gamma$. \end{lemma} \begin{proof} In the proof we fix $$d=N-n+1,\quad d'=\lceil \frac{N+n}{2}\rceil+1.$$ Let $\Omega_p$ be a subset of labels that satisfy $$\Omega_p:=\{\Psi\in\{0,1\}^{N\times n}: \text{ for each } j\in [n],\quad |i\in\{1,\cdots,N\}:\psi_{ij}=1|\geq N-\frac{1}{2}d \}. $$ Then since $\mathbb{P}(\psi_{ij}=1)\geq \frac{1+\frac{3}{4}(\delta-1)}{\delta}$, an application of Chernoff's inequality implies $$ \mathbb{P}(\Omega_p)\geq 1-n\exp(-w_\eqref{lemma3.12}N). $$ Now we take any $W_p\in\Omega_p$, then for any subspace $F\subset\mathbb{R}^N$ of dimension $n-1$, we have for each $j$, and for each $\ell>0$, \begin{equation}\label{whatline6323} \mathbb{P}_{W_p}\{\operatorname{dist}(\text{col}_j(G),F)\leq\sqrt{d'}/\ell\} \leq \mathcal{Q}(\operatorname{Proj}_{F_0^{\perp}}(\mathbf{v}),\sqrt{d'}/\ell)\leq (C_\eqref{corollary3.3}/\sqrt{\ell\gamma})^{d'/\ell}, \end{equation} where $F_0$ is a linear subspace of dimension at most $d'$ and $\mathbf{v}$ is a $N$-dimensional random vector $\mathbf{v}=(b_{1j},b_{2j},\cdots b_{Nj})^t$ where each $b_{ij}$ is independent and $\mathcal{Q}(b_{ij},1)\leq 1-\gamma$. More precisely, for this fixed $j$ we can form $F_0$ as the linear span of $F$ and all the $i$-th rows of $G$ such that $\psi_{ij}=0$, that is, we take the further projection over an additional $\frac{d}{2}$ rows of $G$. Then $F_0$ has dimension at most $\frac{N+n}{2}+1.$ The vector $\mathbf{v}$ is nothing but the $j$-th column of $G$ where we set all labels $\psi_{ij}=1,i=1,\cdots,N$. The second inequality in \eqref{whatline6323} uses Corollary \ref{corollary3.3}. Here we use the obvious fact that $\mathcal{Q}(t+g_{ij},1)\leq1-\gamma$ for any $t\in\mathbb{R}$. Then if we take $\ell=\lfloor 4 C_\eqref{corollary3.3}^2/\gamma\rceil$, by independence of $\operatorname{col}_j(G)$, we conclude that $$ \mathbb{P}_{W_p}(\operatorname{dist}(\operatorname{Col}_j(G),\text{span}\{\text{col}_k(G)_{k\neq j}\})\leq h\sqrt{d'})\leq \exp(-wd'), $$ where $h,w>0$ are two constants only relying on $\gamma$ and $\delta$. For each $W_p\in\Omega_p$ denote by $$ \mathcal{E}_{W_p}:=\{\Psi=W_p,\text{dist}(\text{col}_j(G(\omega)),\text{span}(\text{col}_k(G(\omega))_{k\neq j})\geq h\sqrt{d'}\text{ for any }j=1,\cdots,n\}. $$Then for any $\omega\in \mathcal{E}_{W_p},$ take any $y=(y_1,\cdots,y_n)\in \mathbb{S}_p^{n-1}(\theta),$ we can find $j=j(y)$ with $|y_j|\geq\theta$ and that $$ \|G(\omega)y\|\geq\theta \text{dist}(\text{col}_j(G(\omega)),\text{span}\{\text{col}_k(G(\omega))\}_{k\neq j})\geq h\theta\sqrt{d'}. $$ Then we have, for any $W_p\in \Omega_p$, $$ \mathbb{P}_{W_p}\{\inf_{y\in\mathbb{S}_p^{n-1}(\theta)}\|Gy\|\leq h\theta\sqrt{d'}\}\leq n\exp(-wd') $$ which completes the proof. \end{proof} Finally we consider the complement $\mathbb{S}^{n-1}\setminus(\mathbb{S}_a^{n-1}(\sqrt{N})\cup \mathbb{S}_p(\theta))$. The next Proposition and its proof is a reformulation of \cite{tikhomirov2016smallest}, Proposition 17. \begin{Proposition} \label{propos13.13} Fix $\delta>1,\gamma>0$, and let $z\in\mathbb{R}$ be such that $$\min(\mathbb{P}\{z-\sqrt{N}\leq g_{ij}\leq z-1\},\mathbb{P}\{z+1\leq g_{ij}\leq z+\sqrt{N}\})\geq\gamma. $$ Denote by $S:=\mathbb{S}^{n-1}\setminus \mathbb{S}_a^{n-1}(\sqrt{N})$. Then we can find $N_\eqref{propos13.13}\in\mathbb{N},$ $h_\eqref{propos13.13}>0$ depending only on $\gamma,\delta$ such that, for $N\geq\max(N_\eqref{propos13.13},\delta n)$, we can find a subset of labels $\Omega_g\subset\{0,1\}^{N\times n}$ with $\mathbb{P}(\Psi\in\Omega_g)\geq 1-\exp(-w_\eqref{lemma3.10}N/2)$, such that for any $W_g\in \Omega_g,$ we have $$ \mathbb{P}_{W_g}(\inf_{y\in S}\|Gy\|\leq h_\eqref{propos13.13}\sqrt{N})\leq\exp(-w_\eqref{lemma3.10}N/2). $$\end{Proposition} \begin{proof} We denote by $f_0:=\frac{(1-\delta^{-1/4})\sqrt{c_\eqref{lemma3.56}\gamma}}{C_\eqref{theorem3.2}}$, and denote by $\tau_0=\tau_0(\gamma,\delta)$ the largest real number in $(0,1]$ satisfying that for all $s\geq 0,$ $$ (\frac{64C_{\eqref{lemma3.444}}C_{\eqref{lemma3.7}}2^{s/2}}{h_\eqref{lemma3.10}f_0\tau_0^{3/2}})^{2^{-s/4}\tau_0}\leq\exp(w_\eqref{lemma3.10}/4). $$ Next we take $N_\eqref{propos13.13}=N_\eqref{propos13.13}(\tau,\delta)$ the minimal integer so that for $N\geq N_\eqref{propos13.13}$, \begin{equation}\label{firstbound123} \frac{1}{\lfloor N^{1/4}\rfloor}\leq \frac{f_0\sqrt{\tau_0}}{16}N^{-3/16} ,\quad \frac{192\sqrt{N}C_{\eqref{lemma3.444}}C_{\eqref{lemma3.7}}}{h_{\eqref{lemma3.10}}f_0\tau_0^{3/2}}\leq \exp(w_\eqref{lemma3.10}N/4). \end{equation} From Lemma \ref{lemma3.56} we can find some $\ell\in[0,\lfloor\log_2\sqrt{N}\rfloor]$, $\lambda\in\mathbb{R}$, two subsets $H_1,H_2\subset[-2^{\ell+2},2^{\ell+2}]$ with $\operatorname{dist}(H_1,H_2)\geq 2^\ell$, $\mathbb{E}[(g_{ij}-\lambda)1_{g_{ij}\in H_1\cup H_2}]=0$, and that $\min(\mathbb{P}(g_{ij}-\lambda\in H_1),\mathbb{P}(g_{ij}-\lambda\in H_2))\geq c_\eqref{lemma3.56}\gamma 2^{-\ell/8}.$ Now we take the parameters $$ R:=2^{\ell+2},d:=2^{\ell}, r:=c_\eqref{lemma3.56}\gamma 2^{-\ell/8},m:=\lceil \frac{\tau_0n}{2^{\ell/4}}\rceil,t:=\frac{1}{2}\sqrt{\frac{m}{n}},\epsilon:=\frac{h_\eqref{lemma3.10}h_{\eqref{lemma3.9}}}{2C_{\eqref{lemma3.444}}R}. $$ When $S$ is nonempty, we denote by $T\subset B_2^n$ the set of all $m$-sparse vectors satisfying $\|y\|\geq t,\|y\|_\infty\leq \frac{2h_\eqref{lemma3.9}}{d}$. The first equation in \eqref{firstbound123} implies $\frac{1}{\lfloor N^{1/4\rfloor}}\leq\frac{2h_\eqref{lemma3.9}}{d}$. Thus, thanks to Lemma \ref{lemma3.82}, the net $T$ is nonempty and has the property \eqref{line444}. By Lemma \ref{lemma3.7} we find a finite net $\mathcal{N}\subset T$, $|\mathcal{N}|\leq (\frac{c_\eqref{lemma3.7}n}{m\epsilon})^m$, satisfying that for each $y\in S$ we can find $y'=y'(y)\in\mathcal{N}$ such that $\|y\chi_{\operatorname{supp}y'}-y''\|\leq\epsilon$. Given each $y'\in\mathcal{N}$, denote by $E_{y'}=\operatorname{span}\{e_j\}_{j\in\operatorname{supp}y'}$. Then applying Lemma \ref{lemma3.10}, we can find a subset of labels $\Omega_{y'}$ such that $\mathbb{P}(\Psi\in\Omega_{y'})\geq 1-\exp(-w_\eqref{lemma3.10}N)$ and for each $W_0\in \Omega_{y'}$, we have, recalling definition \eqref{vgog1e} of $V_{G_0,G_1}(E)$, $$ \mathbb{P}_{W_0}\{\operatorname{dist}(G_1y',V_{G_0,G_1}(E_{y'}))\leq h_\eqref{lemma3.9}h_{\eqref{lemma3.10}}\sqrt{N}\}\leq 2\exp(-w_\eqref{lemma3.10}N). $$ Now denote by $\Omega_g=\cap_{y'\in \mathcal{N}}\Omega_{y'},$ we have by our assumption on $N$ that $$\begin{aligned}\mathbb{P}( \Psi\in\Omega_g)&\geq 1-|\mathcal{N}|\exp(-w_\eqref{lemma3.10}N)\\&\geq 1-(\frac{8C_\eqref{lemma3.444}\eqref{lemma3.7}}{\tau_0h_\eqref{lemma3.9}h_\eqref{lemma3.10}})^{2^{-\ell/4}\tau_0n+1}\exp(-w\eqref{lemma3.10}N)\\&\geq 1-(\frac{64C_\eqref{lemma3.444}\eqref{lemma3.7}2^{\ell/2}}{h_\eqref{lemma3.10}f_0\tau_0^{3/2}})^{2^{-\ell/4}\tau_0n+1}\exp(-w_\eqref{lemma3.10} N)\\& \geq 1-\exp(-w_\eqref{lemma3.10}N/2).\end{aligned}$$ For any $W_g\in\Omega_g$ consider $$\begin{aligned} \mathcal{E}_{W_g}:=\{\omega\in\Omega:\Psi=W_g,\operatorname{dist}(G_1(\omega)y';V_{G_0,G_1}(E_{y'})(\omega)\geq h_\eqref{lemma3.9}h_\eqref{lemma3.10}\sqrt{N}\\\text{for any } y'\in\mathcal{N}, \quad \|A\|\leq C_\eqref{lemma3.444}\sqrt{N} \},\end{aligned} $$ then similarly to the computation of $\mathbb{P}(\Omega_y)$, we have by the second equation of \eqref{firstbound123} $$\begin{aligned}\mathbb{P}_{W_g}(\mathcal{E}_{W_g})&\geq 1-\exp(-N)-2|\mathcal{N}|\exp(-w_\eqref{lemma3.10}N)\\&\geq 1-3(\frac{64C_\eqref{lemma3.444}C_\eqref{lemma3.7}2^{\ell/2}}{h_\eqref{lemma3.10}f_0\tau_0^{3/2}})^{2^{-\ell/4}\tau_0n+1}\exp(-w_\eqref{lemma3.10}N)\geq 1-\exp(-w_\eqref{lemma3.10}N/2).\end{aligned}$$ Now we take any $\omega\in \mathcal{E}_{W_g}$, then using Proposition \ref{proposition3.13.13.1} we have $$ \inf_{y\in S}\|G(\omega)y\|\geq h_\eqref{lemma3.9}h_\eqref{lemma3.10}\sqrt{N}-\epsilon C_\eqref{lemma3.444}\sqrt{N}\geq \frac{h_{\eqref{lemma3.10}}f_0\sqrt{\tau_0}}{16}\sqrt{N}, $$ which completes the proof of the theorem. \end{proof} \subsection{Lower bounding singular value: conclusion} \label{section3.4theends} We have collected all the necessary materials for the proof of Theorem \ref{universalbacks}. \begin{proof}[\proofname\ of Theorem \ref{universalbacks}] After rescaling we shall prove the theorem for $\alpha_\eqref{line276definition}=1$. Fix $\delta>1$ and any $\beta\in(0,1)$, set $\gamma=\beta/4$, and let $N_0=N_0(\beta,\delta)$ be the minimal integer satisfying $N_0\geq \max(N_{\eqref{proposition3.11}},N_{\eqref{propos13.13}},16/(1-\gamma))$ and for any $N\geq N_0$ we have $$ N\leq\exp(w_\eqref{lemma3.12}N/2),\quad 3\leq \exp(\min(w_\eqref{lemma3.12},w_\eqref{lemma3.10})N/4). $$ The entries $g_{ij}$ satisfy $\mathcal{Q}(g_{ij},1)\leq1-\beta$, then we choose $z\in\mathbb{R}$ such that $$ \mathbb{P}(g_{ij}\leq z-1)\geq\frac{\beta}{2},\quad \mathbb{P}(g_{ij}<z-1)\leq\frac{\beta}{2}. $$ We define the collection of labels $\mathcal{D}$ via $$ \mathcal{D}=\Omega_p\cap\Omega_g\cap\Omega_a, $$ where the three sets on the right hand side are defined in Proposition \eqref{proposition3.11}, Lemma\eqref{lemma3.12} and Proposition \eqref{propos13.13} respectively. Then via a direct union bound, for a randomly chosen label $\Psi$ as in Theorem \ref{universalbacks}, we have $\mathbb{P}(\Psi\in\mathcal{D})\geq 1-2\exp(-w_\eqref{lemma3.10}n)-\exp(w_\eqref{lemma3.12}n)$. As $g_{ij}$ has unit second moment, for $N\geq 16/(1-\gamma)$ it is impossible to have $\mathbb{P}(z+1\leq g_{ij}\leq z+\sqrt{N})\leq \gamma$ or $\mathbb{P}\{z-\sqrt{N}\leq g_{ij}\leq z-1\}\leq\gamma$.Thus we assume $\min(\mathbb{P}\{z-\sqrt{N}\leq g_{ij}\leq z-1\},\mathbb{P}\{z+1\leq g_{ij}\leq z+\sqrt{N}\})\geq\gamma.$ Then we define $\theta_\eqref{proposition3.11}$ as in Proposition \ref{proposition3.11} and combine the conclusion of Lemma \ref{lemma3.12} for peaky vectors, Proposition \ref{proposition3.11} for almost sparse vectors and Proposition \ref{propos13.13} for generic vectors to deduce that for any $W\in \mathcal{D}$, $$ \mathbb{P}_W\{\sigma_{min}(G)\leq h\sqrt{N}\}\leq \exp(-w_\eqref{lemma3.12}N/2)+2\exp(-w_{\eqref{lemma3.10}}N/2), $$ where $h=\min(h_\eqref{proposition3.11},h_\eqref{propos13.13},h_\eqref{lemma3.12}\theta_{\eqref{proposition3.11}})$. This justifies the claim of Theorem \ref{universalbacks}. \end{proof} \subsection{Proof of universality results}\label{section3.5} This section contains the proof of Theorem \ref{derivation2.414}. We begin with some notations. The Hausdorff distance of two subsets $A,B\subset\mathbb{R}$ is defined via $$ d_H(A,B):=\inf\{\epsilon>0:A\subset B+[-\epsilon,\epsilon]\text{ and } B\subset A+[-\epsilon,\epsilon]\}. $$ For any bounded operator $X$ on a Hilbert space we use the notation $\operatorname{sp}(X)$ to denote the spectrum of $X$. We will use the following spectrum universality result from \cite{brailovskaya2024universality}, Theorem 2.6. Let $Z_0$ be an $d\times d$ self-adjoint deterministic matrix, and let $Z_1,\cdots,Z_n$ be independent $d\times d$ self-adjoint random matrices with $\mathbb{E}[Z_i]=0$. Consider \begin{equation} X:=Z_0+\sum_{i=1}^n Z_i, \end{equation} and let $\operatorname{Cov}(X)$ be $d^2\times d^2$ covariance matrix such that \begin{equation} \operatorname{Cov}(X)_{ij,kl}:=\mathbb{E}[(X-\mathbb{E}X)_{ij}\overline{(X-\mathbb{E}X)}_{kl}]. \end{equation} Let $G$ be a $d\times d$ self-adjoint random matrix where $\{Re G_{ij},\Im G_{ij}:i,j\in[d]\}$ are jointly Gaussian, satisfying that $\mathbb{E}[G]=\mathbb{E}[X]$ and $\operatorname{Cov}(G)=\operatorname{Cov}(X)$. Then $G$ can be expressed as $$ G=Z_0+\sum_{i=1}^N A_ig_i $$ for deterministic matrices $A_1,\cdots,A_N\in M_d(\mathbb{C})_{sa}$ and for i.i.d. real Gaussians $g_1,\cdots,g_N$. Following \cite{brailovskaya2024universality} we introduce matrix parameters $$ \sigma(X):=\|\mathbb{E}[(X-\mathbb{E}X)^2]\|^\frac{1}{2},\quad \sigma_*(X):=\sup_{\|v\|=\|w\|=1}\mathbb{E}[|\langle v,(X-\mathbb{E}X)w\rangle|^2]^\frac{1}{2}, $$ $$ R(X):=\left\|\max_{1\leq i\leq n}\|Z_i\|\right\|_\infty. $$ \begin{theorem}\label{theorem3.14goods}[\cite{brailovskaya2024universality}] There exists a universal constant $C>0$ such that for any $t>0$, \begin{equation} \mathbb{P}[d_H(\operatorname{sp}(X),\operatorname{sp}(G))\geq C\epsilon(t)]\leq de^{-t}, \end{equation} where $$\epsilon(t)=\sigma_*(X)t^\frac{1}{2}+R(X)^\frac{1}{3}\sigma(X)^\frac{2}{3}t^\frac{2}{3}+R(X)t. $$ \end{theorem} The following lemma (for deterministic matrices) is similar to \cite{bandeira2023matrix}, Lemma 3.13 and has a similar proof. \begin{lemma}\label{740new} For any $\epsilon>0$ let $A_\epsilon=4\epsilon^2\mathbf{1}$. For the random matrices $H,G$ we define $$ \widetilde{G}_\epsilon=\begin{pmatrix}0&0&G&A_\epsilon^\frac{1}{2},\\0&0&0&0\\G^*&0&0&0\\A_\epsilon^\frac{1}{2}&0&0&0,\quad \end{pmatrix},\quad \widetilde{H}_\epsilon=\begin{pmatrix}0&0&H&A_\epsilon^\frac{1}{2},\\0&0&0&0\\H^*&0&0&0\\A_\epsilon^\frac{1}{2}&0&0&0,\end{pmatrix}. $$ Then $\operatorname{sp}(\widetilde{H}_\epsilon)\subseteq \operatorname{sp}(\widetilde{G}_\epsilon)+[-\epsilon,\epsilon]$ implies that $$\begin{cases} \lambda_+(HH^*+A_\epsilon)^\frac{1}{2}\leq \lambda_+(GG^*+A_\epsilon)^\frac{1}{2}+\epsilon,\\ \lambda_{-}(HH^*+A_\epsilon)^\frac{1}{2}\geq \lambda_{-}(GG^*+A_\epsilon)^\frac{1}{2}-\epsilon,\\ \end{cases}$$ which holds for any $\epsilon>0$, and the same holds if we replace the order of $G$ and $H$. We denote by $\lambda_+(Z):=\sup\operatorname{sp}(Z)$ and $\lambda_{-}(Z):=\inf\operatorname{sp}(Z)$ for each self-adjoint operator $Z$. Furthermore, $\operatorname{sp}(\widetilde{ H}_\epsilon)\subseteq \operatorname{sp}(\widetilde{G}_\epsilon)+[-\epsilon,\epsilon]$ implies that $$ \sigma_{min}(H)\geq\sigma_{min}(G)-3\epsilon. $$ \end{lemma} \begin{proof} A standard linear algebra fact implies that $$ \operatorname{sp}(\widetilde{G}_\epsilon)\cup\{0\}=\operatorname{sp}((GG^*+A_\epsilon)^\frac{1}{2})\cup -\operatorname{sp}((GG^*+A_\epsilon)^\frac{1}{2})\cup\{0\}, $$ and the same holds for $\widetilde{H}_\epsilon$. Then $\operatorname{sp}(\widetilde{H}_\epsilon)\subset\operatorname{sp}(\widetilde{G}_\epsilon)+[-\epsilon,\epsilon]$ implies that $$ \lambda_+(HH^*+A_\epsilon)^\frac{1}{2}\leq\lambda_+(GG^*+A_\epsilon)^\frac{1}{2}+\epsilon. $$ Meanwhile, noting that we must have $\lambda_{-}(GG^*+A_\epsilon)^\frac{1}{2}\geq 2\epsilon$ and $\lambda_{-}(HH^*+A_\epsilon)^\frac{1}{2}\geq 2\epsilon$, so that even though the spectrum of $\widetilde{H}_\epsilon$ has a zero, we still conclude that $$ \lambda_{-}(HH^*+A_\epsilon)^\frac{1}{2}\geq \lambda_{-}(GG^*+A_\epsilon)^\frac{1}{2}-\epsilon. $$ Finally, using $$ \sigma_{min}(H)\leq \lambda_{-}(HH^*+A_\epsilon)^\frac{1}{2}\leq \sigma_{min}(H)+2\epsilon $$ and a similar expression for $\sigma_{min}(G),$ we conclude that $$ \sigma_{min}(H)\geq \sigma_{min}(G)-3\epsilon. $$ \end{proof} Let $H$ and $G$ be as defined in Theorem \ref{derivation2.414}. We note the following computation of matrix parameters: for any $\epsilon>0$ we have $$ \sigma(\widetilde{T}_\epsilon)\leq M\sqrt{\frac{N}{n}},\quad \sigma_*(\widetilde{T}_\epsilon)\leq\frac{M}{\sqrt{n}}, \quad R(\widetilde{T}_\epsilon)\leq q. $$ We have collected necessary ingredients for proving Theorem \ref{derivation2.414}. \begin{proof}[\proofname\ of Theorem \ref{derivation2.414}] We define random matrices $\widetilde{H}_{\epsilon(t)},$ $\widetilde{G}_{\epsilon(t)}$ as in the statement of Lemma \eqref{740new}, where we replace $\epsilon$ by $\epsilon(t)$ defined by $$ \epsilon(t)=CMn^{-\frac{1}{2}}t^\frac{1}{2}+CM^\frac{2}{3}q^{\frac{1}{3}}t^{\frac{2}{3}}(\frac{N}{n})^\frac{1}{3}+Cqt, $$ where the constant $C>0$ is defined in Theorem \ref{theorem3.14goods} and $M>1$ is the upper bound of entry variance in the statement of Theorem \ref{derivation2.414}. Applying Theorem \ref{theorem3.14goods}, we get that for any $t>0$, $$ \mathbb{P}\left(d_H\left(\operatorname{sp}(\widetilde{T}_{\epsilon(t)}),\operatorname{sp}(\widetilde{G}_{\epsilon(t)})\right)\geq \epsilon(t)\right)\leq 4N e^{-t}. $$ Then Lemma \ref{740new} implies that for any $t\geq 0,$ $$ \mathbb{P}(\sigma_{min}(T)\geq \sigma_{min}(G)-3\epsilon(t))\geq 1-4N e^{-t} $$ and $$ \mathbb{P}(\sigma_{min}(G)\geq \sigma_{min}(T)-3\epsilon(t))\geq 1-4N e^{-t}. $$ It suffices to replace the universal constant $C$ by $3CM$, which is a universal constant depending only on $M>0,$ (and replace $M^\frac{2}{3}$ by $M$ since $M\geq 1$) and we conclude the proof. \end{proof} \section*{Funding} The author is supported by a Simons Foundation Grant (601948, DJ) \printbibliography \end{document}
2412.06331v1
http://arxiv.org/abs/2412.06331v1
The maximum forcing numbers of quadriculated tori
\documentclass[12pt, a4paper]{article} \usepackage{amsmath} \usepackage{comment} \usepackage{amsfonts} \usepackage{amssymb} \usepackage{epsfig} \usepackage{graphicx} \usepackage{color} \usepackage{amsthm} \usepackage{enumerate} \usepackage [latin1]{inputenc} \usepackage[numbers, sort&compress]{natbib} \usepackage{url} \setcounter{MaxMatrixCols}{10} \textheight 25.5cm \textwidth 17 cm \topmargin -2.0 cm \oddsidemargin -0.5 cm \newtheorem{thm}{Theorem}[section] \newtheorem{lem}[thm]{Lemma} \newtheorem{cor}[thm]{Corollary} \newtheorem{pro}[thm]{Proposition} \newtheorem{exa}[thm]{Example} \newtheorem{con}[thm]{Conjecture} \newtheorem{prob}[thm]{Problem} \newtheorem{ex}[thm]{Example} \theoremstyle{definition} \newtheorem{den}[thm]{Definition} gurename}{Fig.} \newtheorem{remark}[thm]{Remark} \graphicspath{{figures/}} \newcommand{\meng}[1]{\textcolor{blue}{Xiaomeng: #1}} \usepackage{url} \usepackage{authblk} \long\def\delete#1{} \usepackage{xcolor} \usepackage[normalem]{ulem} \begin{document} \openup 0.5\jot \title{The maximum forcing numbers of quadriculated tori} \author[1]{Qianqian Liu\thanks{ E-mail: \texttt{[email protected].}}} \author[2]{Yaxian Zhang\thanks{E-mail: \texttt{[email protected].}}} \author[2]{Heping Zhang\footnote{The corresponding author. E-mail: \texttt{[email protected].}}} \affil[1]{\small College of Science, Inner Mongolia University of Technology, Hohhot, Inner Mongolia 010010, China} \affil[2]{\small School of Mathematics and Statistics, Lanzhou University, Lanzhou, Gansu 730000, China} \date{} \maketitle \setlength{\baselineskip}{20pt} \noindent {\bf Abstract}: Klein and Randi\'{c} (1985) proposed the concept of forcing number, which has an application in chemical resonance theory. Let $G$ be a graph with a perfect matching $M$. The forcing number of $M$ is the smallest cardinality of a subset of $M$ that is contained only in one perfect matching $M$. The maximum forcing number of $G$ is the maximum value of forcing numbers over all perfect matchings of $G$. Kleinerman (2006) obtained that the maximum forcing number of $2n\times 2m$ quadriculated torus is $nm$. By improving Kleinerman's approach, we obtain the maximum forcing numbers of all 4-regular quadriculated graphs on torus except one class. \vspace{2mm} \noindent{\textbf{Keywords}} Perfect matching, maximum forcing number, quadriculated torus \vspace{2mm} \noindent{\textbf{MSC2020}} 05C70, 05C92 \section{\normalsize Introduction} Let $G$ be a graph with a perfect matching $M$. A subset $S\subseteq M$ is called a \emph{forcing set} of $M$ if it is contained in no other perfect matchings of $G$. The smallest cardinality of a forcing set of $M$ is called the \emph{forcing number} of $M$, denoted by $f(G,M)$. The \emph{minimum} and \emph{maximum forcing number} of $G$, denoted by $f(G)$ and $F(G)$, are respectively defined as the minimum and maximum values of $f(G,M)$ over all perfect matchings $M$ of $G$. The concept of the forcing number of a perfect matching was first introduced by Klein and Randi\'{c} \cite{3,klein85} in 1985 when they studied the molecular resonance structures, which was called ``innate degree of freedom'' in chemical literatures. It was turned out that the perfect matchings with the maximum forcing number contribute more to the stability of molecule\cite{32}. Afshani, Hatami and Mahmoodian \cite{5} pointed out that the computational complexity of the maximum forcing number of a graph is still an open problem. Xu, Bian and Zhang \cite{27} obtained that maximum forcing numbers of hexagonal systems are equal to the resonant numbers. The same result also holds for polyominoes \cite{zhou2016,lin2017} and BN-fullerene graphs \cite{40}. Abeledo and Atkinson \cite{13} had already obtained that resonant numbers of 2-connected plane bipartite graphs can be computed in polynomial time. Thus, the maximum forcing numbers of such three classes of graphs can be solved in polynomial time. The cartesian product of graphs $G$ and $H$ is denoted by $G\square H$. The maximum forcing numbers of the cartesian product of some special graphs, such as paths and cycles, have been obtained. Let $P_n$ and $C_n$ denote a path and a cycle with $n$ vertices, respectively. Pachter and Kim \cite{6}, Lam and Pachter \cite{9} obtained that $F(P_{2n}\square P_{2n})=n^2$ using different methods. In general, Afshani et al. \cite{5} proved that $F(P_m\square P_n)=\lfloor\frac{m}{2}\rfloor\cdot\lfloor\frac{n}{2}\rfloor$ for even $mn$. Besides, they \cite{5} obtained that $F(P_{2m}\square C_{2n})=mn$ and $F(P_{2m+1}\square C_{2n})=mn+1$, and asked such a question: what is the maximum forcing number of a non-bipartite cylinder $P_{2m}\square C_{2n+1}$? Jiang and Zhang \cite{29} solved this problem and obtained that $F(P_{2m}\square C_{2n+1})=m(n+1)$. By a method of marking independent sets, Kleinerman \cite{16} obtained that $F(C_{2m}\square C_{2n})=mn$. Obviously, $C_{2m}\square C_{2n}$ is a special type of 4-regular quadriculated graphs on torus. As early as 1991, Thomassen \cite{Tho} classified all 4-regular quadriculated graphs on torus (abbreviated to ``\emph{quadriculated tori}'') into two classes, which were reduced into one class by Li \cite{classfy}. For $n\geq1$ and $m\geq 2$, a \emph{quadriculated torus} $T(n,m,r)$ is obtained from an $n\times m$ chessboard ($n$ rows, each consists of $m$ squares) by sticking the left and right sides together and then identifying the top and bottom sides with a torsion of $r$ squares where $1\leq r\leq m$ (see Fig. \ref{torsion}). Obviously, $T(n,m,m)$ is isomorphic to $C_n\square C_m$. Based on the parity of three parameters, quadriculated tori with perfect matchings can be divided into six classes $T(2n,2m,2r)$, $T(2n,2m,2r-1)$, $T(2n+1,2m,2r)$, $T(2n+1,2m,2r-1)$, $T(2n,2m+1,2r)$ and $T(2n,2m+1,2r-1)$. \begin{figure}[h] \centering \includegraphics[height=3cm,width=6cm]{torsion-eps-converted-to.pdf} \caption{\label{torsion}Quadriculated torus $T(3,8,4)$.} \end{figure} In this paper, we obtain a simple expression for the maximum forcing numbers of all quadriculated tori except for $T(2n+1,2m,2r-1)$. In Section 2, we give some notations and terminologies, and prove some crucial lemmas. In Section 3, we prove that $F(T(2n,2m+1,t))=n(m+1)$ for $1\leq t\leq 2m+1$ by choosing a fixed independent set. In Section 4, we obtain that $F(T(2n,2m,r))=mn+1$ if $(r,2m)=2$, and $F(T(2n,2m,r))=mn$ otherwise, where $(r,2m)$ represents the greatest common factor of $r$ and $2m$, and $1\leq r\leq 2m$. In Section 5, by another representation of the quadriculated torus, we obtain the maximum forcing number of $T(2n+1,2m,2r)$ for $1\leq r\leq m$. \section{\normalsize Preliminaries}In this section, we give some notations and terminologies, and prove some important lemmas. Let $T(n,m,r)$ be a quadriculated tori. According to positions of vertices in the chessboard, we label the vertices of $T(n,m,r)$ as $\{v_{i,j}| i\in Z_n, j \in Z_m\}$ (see Fig. \ref{nota}), where $Z_m:=\{0,1,\dots,m-1\}$. Hence $v_{i,0}$ is adjacent to $v_{i,m-1}$ for $i\in Z_{n}$, and $v_{0,j}$ is adjacent to $v_{n-1,m-r+j}$ for $j\in Z_{m}$. \begin{figure}[h] \centering \includegraphics[height=3.3cm,width=7cm]{newnotation-eps-converted-to.pdf} \caption{\label{nota}Labels of the vertices in $T(4,8,2)$.} \end{figure} For $j\in Z_m$, let $v_{0,j}v_{1,j}\cdots v_{n-1,j}$ be a path called \emph{$j$-column}, and $v_{0,j}$ and $v_{n-1,j}$ are \emph{initial} and \emph{terminal} of $j$-column. For convenience, we call $j$-column a \emph{column} for $j\in Z_{m}$. If initial $v_{0,j_2}$ of $j_2$-column is adjacent to terminal $v_{n-1,j_1}$ of $j_1$-column, that is, $j_2\equiv j_1+r$ (mod $m$), then $j_2$-column is the \emph{successor} of $j_1$-column. Let $j_0$-, $j_1$-, \dots, $j_{g-1}$-columns be pairwise different such that $j_{k+1}$-column is the successor of $j_k$-column for each $k\in Z_g$. Then these $g$ columns form a cycle, called an \emph{$\mathrm{I}$-cycle}. In \cite{LYZ}, we had proved the following lemma. \begin{lem}\rm{\cite{LYZ}}\label{lem1} $T(n,m,r)$ has $(r,m)$ $\mathrm{I}$-cycles and each $\mathrm{I}$-cycle contains $\frac{m}{(r,m)}$ columns. Moreover, any consecutive $(r,m)$ columns lie on different $\mathrm{I}$-cycles. \end{lem} Intuitively, we call $v_{i,j}v_{i,j+1}$ a \emph{horizontal edge} and $v_{i,j}v_{i+1,j}$ a \emph{vertical edge} for $i\in Z_n$ and $j\in Z_{m}$. Obviously, all vertical edges form $(r,m)$ $\mathrm{I}$-cycles, and all horizontal edges form $n$ $\mathrm{II}$-cycles (consisting of all vertices and edges on a row). Preserving the horizontal and vertical edges, we can obtain another representation of this quadriculated tori, denoted by $T^*(n,m,r)$, in which all vertices of a $\mathrm{I}$-cycle of $T(n,m,r)$ lie on a column and all vertices of a $\mathrm{II}$-cycle of $T(n,m,r)$ are divided into different rows (see Fig. \ref{obsev}). Therefore, $\mathrm{I}$-cycles (resp. $\mathrm{II}$-cycles) in $T(n,m,r)$ corresponds to $\mathrm{II}$-cycles (resp. $\mathrm{I}$-cycles) in $T^*(n,m,r)$. For $i\in Z_{n}$, the subgraph of $T(n,m,r)$ induced by all vertices of any consecutive two rows $$\{v_{i,0},v_{i,1},\dots, v_{i,m-1}\}\cup \{v_{i+1,0},v_{i+1,1},\dots, v_{i+1,m-1}\}$$ is denoted by $R_{i,i+1}$. Then $R_{i,i+1}$ contains a subgraph isomorphic to $C_{m}\square P_2$. Particularly, $R_{i,i+1}$ is isomorphic to $C_{m}\square P_2$ for $n\geq 2$ where $i\in Z_n$. Relabeling the vertices of $T(n,m,r)$ according to $\mathrm{I}$-cycle, we can obtain the following lemma. For details, see Section 2 of ref. \cite{LYZ}. \begin{figure}[h] \centering \includegraphics[height=5.7cm,width=13cm]{obsev-eps-converted-to.pdf} \caption{\label{obsev} Quadriculated tori $T(3,12,8)$ and $T(4,9,3)=T^*(3,12,8)$.} \end{figure} \begin{lem}\rm{\cite{LYZ}}\label{drawing} For $n\geq1$, $m\geq 2$ and $1\leq r\leq m$, $T^*(n,m,r)=T((r,m), \frac{mn}{(r,m)},(\frac{m}{(r,m)}-k)n)$, where $0\leq k\leq \frac{m}{(r,m)}-1$ is an integer satisfying the equation $(r,m)\equiv rk\ (\text{mod\ } m).$ Furthermore, $T^{**}(n,m,r)=T(n,m,r)$. \end{lem} For a non-empty subset $S\subseteq V(G)$, the \emph{subgraph induced by $S$}, denoted by $G[S]$, is a graph whose vertex set is $S$ and edge set consists of those edges of $G$ that have both end vertices in $S$. The induced subgraph $G[V(G)\setminus S]$ is denoted by $G-S$. For an edge subset $F\subseteq E(G)$, we use $V(F)$ to denote the set of all end vertices of edges in $F$. Let $G$ be a graph with a perfect matching $M$. We give an independent set $T$ of $G$ called \emph{marked vertices} of $G$. Define $M_T=\{e\in M\ |\ e \text{\ has an end vertex in }T\}.$ Then $M_T\subseteq M$ and $|M_T|=|T|$. A cycle of $G$ is \emph{$M$-alternating} if its edges appear alternately in $M$ and off $M$. \begin{lem}\label{forcingset} Let $G$ be a graph with a perfect matching $M$. If the union of all paths of length 2 whose initial and terminal lie in $T$ contains no $M$-alternating cycles, then $f(G,M)\leq |M|-|T|$. \end{lem} \begin{proof}We prove that $G[V(M_T)]$ contains no $M$-alternating cycles. Suppose to the contrary that $G[V(M_T)]$ contains an $M$-alternating cycle $C$. Then $C$ is also an $M_T$-alternating cycle. Since $T$ is an independent set, half vertices of $C$ are marked, and marked and unmarked vertices appear alternately. Thus, $C$ can be viewed as the union of paths of length two whose initial and terminal lie in $T$, which is a contradiction. Since $G[V(M_T)]$ contains no $M$-alternating cycles, $G[V(M_T)]$ has a unique perfect matching. Thus, $M\setminus M_T$ is a forcing set of $M$, and $f(G,M)\leq |M\setminus M_T|=|M|-|T|$. \end{proof} For convenience, ``the union of all paths of length 2 whose initial and terminal are marked vertices'' is defined as ``\emph{marked subgraph}''. Next we give the concept of $2\times 2$-polyomino, which is a kind of general ``marked subgraph''. A \emph{polyomino} is a finite connected subgraph in the infinite plane square grid in which every interior face is surrounded by a square and every edge belongs to at least one square. A \emph{$2\times 2$-polyomino} is also a polyomino which is obtained by replacing each square in a polyomino by a $2\times 2$ chessboard (see Fig. \ref{polyominog}). \begin{figure}[h] \centering \includegraphics[height=3.2cm,width=7cm]{polyomino-eps-converted-to.pdf} \caption{\label{polyominog} A polyomino and its corresponding $2\times 2$-polyomino.} \end{figure} An \emph{interior vertex} of a plane graph is a vertex which is not on the boundary of the unbounded face. For a polyomino, an interior vertex means a vertex of degree 4. By the proof of Theorem 3.2 in \cite{29}, Jiang and Zhang obtained the following result. \begin{lem}\label{polyomino}\rm{\cite{29}} A $2\times 2$-polyomino has an odd number of interior vertices. \end{lem} \section{\normalsize The maximum forcing number of $T(2n,2m+1,r)$ for $1\leq r\leq 2m+1$} In this section, we will obtain the maximum forcing number of $T(2n,2m+1,r)$ by the method of marking independent sets for $1\leq r\leq 2m+1$. For $T(2n,m,r)$, we define some subsets of vertices and edges. For $i\in Z_{n}$, let $$X_{i}=\{v_{i,2k}|k\in Z_{\lfloor\frac{m}{2}\rfloor}\} \text{ and } Y_{i}=\{v_{i,2k+1}|k\in Z_{\lfloor\frac{m}{2}\rfloor}\}.$$ For $j\in Z_{m}$, let $W_{j}=\{v_{2k,j}v_{2k+1,j}|k\in Z_{n}\}$, $$W^{1}_{j}=\{v_{4k+2,j}v_{4k+3,j}|k\in Z_{\lfloor\frac{n}{2}\rfloor}\} \text{ and } W^{2}_{j}=\{v_{4k,j}v_{4k+1,j}|k\in Z_{\lfloor\frac{n+1}{2}\rfloor}\}$$ be two subsets of $W_j$. \begin{thm}\label{odd} For $n, m\geq 1$ and $1\leq r\leq 2m+1$, $F(T(2n,2m+1,r))=(m+1)n$. \end{thm} \begin{proof} Let $M_1=W_0\cup W_1\cup \cdots \cup W_{2m}$ be a perfect matching of $T(2n,2m+1,r)$ (see Fig. \ref{fig111}). We will prove that $f(T(2n,2m+1,r),M_1)=(m+1)n$. \begin{figure}[h] \centering \includegraphics[height=3.6cm,width=11.8cm]{fig111-eps-converted-to.pdf} \caption{\label{fig111}The perfect matching $M_1$ of $T(4,7,5)$, and a forcing set of $M_1$ shown in red lines.} \end{figure} For $i\in Z_n$, since $R_{2i,2i+1}$ contains a subgraph isomorphic to $C_{2m+1}\square P_2$, any forcing set of $M_1\cap E(R_{2i,2i+1})$ has size at least $m+1$. Thus, $M_1$ has the forcing number at least $n(m+1)$. Let $S=W_0\cup W^1_1\cup W^2_2\cup W^1_3\cup W^2_4\cup \cdots \cup W^1_{2m-1}\cup W^2_{2m}$ be a subset of $M_1$ shown as red lines in Fig. \ref{fig111}(b), so that exactly $m+1$ edges of $R_{2i,2i+1}$ are chosen to belong to $S$. Obviously, $S$ is a forcing set of $M_1$ with size $n(m+1)$. Hence, we obtain that $f(T(2n,2m+1,r), M_1)=n(m+1)$. For any perfect matching $M$ of $T(2n,2m+1,r)$, we will choose an independent set $T$ of size $mn$ such that ``marked subgraph'' contains no $M$-alternating cycles. By Lemma \ref{forcingset}, we have $$f(T(2n,2m+1,r),M)\leq |M|-|T|=(2m+1)n-mn=(m+1)n.$$ By the arbitrariness of $M$, we have $F(T(2n,2m+1,r))\leq(m+1)n$. To achieve this goal, we will take $m$ appropriate vertices on 1, 3, $\dots$, $2n-1$ rows. Let $X'_{i}=(X_i-\{v_{i,0}\})\cup \{v_{i,2m}\}$ for $i\in Z_{2n-1}$ and $$X^*=\{v_{2n-1,2m+1-r}\}\cup\{v_{2n-1,2m+1-r+j}|j=3,5,\dots,2m-1\}.$$ Take marked vertices $T=X'_1\cup X'_3\cup \cdots \cup X'_{2n-3}\cup X^*$ shown as Fig. \ref{fig112}. \begin{figure}[h] \centering \includegraphics[height=4.8cm,width=16cm]{fig114-eps-converted-to.pdf} \caption{\label{fig112}Marked vertices of $T(6,11,5)$ and $T(6,11,6)$.} \end{figure} From left to right, we choose 1'st, 4'th, 6'th, $\dots$, $(2m)$'th vertices in the first row and 3'th, 5'th, $\dots$, $(2m+1)$'th vertices in the third row as marked vertices. Hence, all edges incident with $v_{0,j}$ are not contained in ``marked subgraph'' for $0\leq j\leq 2m$. Thus such $2m+1$ vertices are not contained in ``marked subgraph'', and ``marked subgraph'' is a plane graph. The ``marked subgraph'' formed by all paths of length two whose initial and terminal are in $X'_{1}\cup X'_{3}\cup \cdots \cup X'_{2n-3}$ is a $2\times 2$-polyomino corresponding to a $(n-2)\times (m-1)$ chessboard, and the ``marked subgraph'' formed by all paths of length two whose initial and terminal are in $X'_{2n-3}\cup X^*$ is a $2\times 2$-polyomino corresponding to some $1\times t$ $(0\leq t\leq m-1)$ chessboard attaching a path. Thus, ``marked subgraph'' is a $2\times 2$-polyomino attaching a path. Suppose to the contrary that $C$ is an $M$-alternating cycle contained in ``marked subgraph''. Then $\text{Int}[C]$ (the subgraph of $T(2n,2m+1,r)$ induced by the vertices of $C$ and its interior) is a $2\times 2$-polyomino. By Lemma \ref{polyomino}, $\text{Int}[C]$ has an odd number of interior vertices, which contradicts that $C$ is $M$-alternating. Thus, ``marked subgraph'' contains no $M$-alternating cycles. \end{proof} \section{\normalsize The maximum forcing number of $T(2n,2m,r)$ for $1\leq r\leq 2m$}In this section, we are to obtain the maximum forcing number of $T(2n,2m,r)$ for $1\leq r\leq 2m$. In the proof of Theorem \ref{odd}, we fix $mn$ marked vertices to prove that ``marked subgraph'' contains no $M$-alternating cycles for any perfect matching $M$ of $T(2n,2m+1,r)$, where $1\leq r\leq 2m+1$. But for a perfect matching $M$ of $T(2n,2m,r)$, ``marked subgraph'' contains an $M$-alternating cycle no matter which sets with size $mn$ we mark. For the case that each $\mathrm{II}$-cycle is not $M$-alternating, we can prove the following result. \begin{lem}\label{modifiedcycle}For $n,m\geq 2$ and $1\leq r\leq 2m$, assume that $M$ is a perfect matching of $T(2n,2m,r)$ and each $\mathrm{II}$-cycle is not $M$-alternating. Then we can mark $mn$ vertices so that ``marked subgraph'' contains no $M$-alternating cycles. \end{lem} \begin{proof} First we choose an independent set $T$ of $T(2n,2m,r)$ with size $mn$ as marked vertices. If $n$ is odd, then take $$T=\{Y_{4k+1}|k=0,1,2, \dots, \frac{n-1}{2}\} \bigcup \{X_{4k+3}|k=0,1,2, \dots, \frac{n-3}{2}\}.$$ Otherwise, take $$T=\{Y_{4k+1}|k=0,1,2, \dots, \frac{n-2}{2}\} \bigcup \{X_{4k+3}|k=0,1,2, \dots, \frac{n-2}{2}\}.$$ See two examples in Fig. \ref{em81}. \begin{figure}[h] \centering \includegraphics[height=6cm,width=13cm]{em81-eps-converted-to.pdf} \caption{\label{em81}Marked vertices and ``marked subgraph'' of $T(6,8,3)$ and $T(8,8,3)$.} \end{figure} If $r$ is odd (resp. even), then marked vertices on the first and last rows are located at different (resp. same) columns. For the case that $r$ and $n$ have the same parity, ``marked subgraph'' consists of $n$ $\mathrm{II}$-cycles. By the assumption, each $\mathrm{II}$-cycle is not $M$-alternating. Thus, ``marked subgraph'' contains no $M$-alternating cycles, and $T$ is the marked vertices we require. It suffices to consider the case that $r$ and $n$ have different parity. In the sequel, we only prove the lemma for the case that $r$ is odd and $n$ is even, and the proof is similar for the other case. Now marked vertices on the first and third rows are located at the same columns. Thus ``marked subgraph'' consists of $m$ paths of length two $\{v_{2n-1,2m-r+j}v_{0,j}v_{1,j}|j=1,3,\dots,2m-1\}$ and $n$ $\mathrm{II}$-cycles shown as red lines in Fig. \ref{em81}(b). By the assumption, each $\mathrm{II}$-cycle is not $M$-alternating. Hence, each $M$-alternating cycle (if exists) of ``marked subgraph'' is contained in the subgraph induced by all vertices of the first three rows, and contains at least two vertices on the second row. By Lemma \ref{polyomino}, an $M$-alternating cycle cannot form the boundary of a $2\times 2$-polyomino which corresponds to a $1\times l$ chessboard for $1\leq l\leq m-1$. Therefore, any $M$-alternating cycle of ``marked subgraph'' has the following form: it starts with a $\mathrm{II}$-cycle in the first row and moves to the third row and backs at specified intervals shown as green lines in Fig. \ref{emmm}(a). Notice that each such cycle contains exactly $2m$ horizontal edges, divided in some way between the two rows. \begin{figure}[h] \centering \includegraphics[height=2.6cm,width=17cm]{emmm-eps-converted-to.pdf} \caption{\label{emmm}$M$-alternating cycle of ``marked subgraph''.} \end{figure} Translating the marked vertices down by one row shown as Fig. \ref{emmm}(b), we also have an $M$-alternating cycle lying on the subgraph induced by the vertices of the second, third and fourth rows (otherwise, new marked vertices we obtained is what we want). We will demonstrate that the new $M$-alternating cycle has more horizontal edges in the bottom (i.e., the fourth) row than the first one does. Consider the set of horizontal edges in the bottom row of the first $M$-alternating cycle, which is partitioned into subsets naturally by proximity: there is a set of horizontal edges, then a cross-over, then perhaps a cross-back, then another set of horizontal edges, and so forth. Consider one of these sets, say $\{v_{1,1}v_{1,2},v_{1,2}v_{1,3},\cdots, v_{1,2t}v_{1,2t+1}\}$ shown as green lines on the third row of Fig. \ref{emm8}(a), where $t\geq 1$. By the form of $M$-alternating cycles, edges of $\{v_{1,1}v_{0,1},v_{0,1}v_{2n-1,2m-r+1}\}$ and $\{v_{1,2t+1}v_{0,2t+1},v_{0,2t+1}v_{2n-1,2m-r+2t+1}\}$ are contained in the first $M$-alternating cycle. It suffices to prove that the set of edges $$\{v_{2,0}v_{2,1},v_{2,1}v_{2,2},v_{2,2}v_{2,3},\cdots, v_{2,2t}v_{2,2t+1}\} \text{ or } \{v_{2,1}v_{2,2},v_{2,2}v_{2,3},\cdots, v_{2,2t}v_{2,2t+1},v_{2,2t+1}v_{2,2t+2}\}$$ is contained in the bottom row of the new $M$-alternating cycle. \begin{figure}[h] \centering \includegraphics[height=2.6cm,width=17cm]{emm82-eps-converted-to.pdf} \caption{\label{emm8}Part of the two $M$-alternating cycles lying in corresponding ``marked subgraphs''.} \end{figure} Since all horizontal edges of the first $M$-alternating cycle lie on the first and third rows, and these of the new $M$-alternating cycle lie on the second and fourth rows, only vertical edges in $\{v_{0,2k+1}v_{1,2k+1}|k=0,1,\dots, m-1\}$ may be intersected. If $v_{0,1}v_{1,1}$ belongs to the new $M$-alternating cycle, then $v_{0,1}v_{1,1}\in M$, and $v_{1,1}v_{2,1}$ is contained in the new $M$-alternating cycle. We claim that $v_{0,0}v_{0,1}$ is contained in the new $M$-alternating cycle. Otherwise, $v_{0,1}v_{0,2}$ and $v_{0,2}v_{0,3}\in M$ are contained in the new $M$-alternating cycle. Since $v_{1,2}v_{1,3}\in M$, $v_{0,3}v_{1,3}$ does not lie on the new $M$-alternating cycle. Hence the path $v_{0,1}v_{0,2}v_{0,3}\cdots v_{0,2t}v_{0,2t+1}$ lies on the new $M$-alternating cycle (see Fig. \ref{emm8}(a)). Note that $v_{0,2t}v_{0,2t+1}\in M$, which contradicts that $v_{2n-1,2m-r+2t+1}v_{0,2t+1}$ and $v_{0,2t+1}v_{1,2t+1}$ belong to the first $M$-alternating cycle. Now we prove the claim. Thus, $v_{0,0}v_{0,1}$ and $v_{1,1}v_{2,1}$ lie on the new $M$-alternating cycle (see Fig. \ref{emm8}(b)). Since $v_{1,1}v_{1,2}v_{1,3}\cdots v_{1,2t}v_{1,2t+1}$ is on the first $M$-alternating cycle, we can obtain that the path $v_{2,1}v_{2,2}v_{2,3}\cdots v_{2,2t}v_{2,2t+1}v_{2,2t+2}$ lies on the second $M$-alternating cycle by a simple argument. If $v_{0,2t+1}v_{1,2t+1}$ belongs to the new $M$-alternating cycle, then, by a similar argument, we can obtain that $$v_{0,2t+2}v_{0,2t+1}v_{1,2t+1}v_{2,2t+1}v_{2,2t}\cdots v_{2,2}v_{2,1}v_{2,0}$$ lies on the second $M$-alternating cycle. If neither $v_{0,1}v_{1,1}$ nor $v_{0,2t+1}v_{1,2t+1}$ belongs to the new $M$-alternating cycle (see Fig. \ref{emm82222}), then, by the form of $M$-alternating cycles, such two $M$-alternating cycles have no common edges in this area, and the result holds naturally. This means that all horizontal edges in the bottom row of the first $M$-alternating cycle give rise to abutting horizontal edges in the bottom row of the second one. Because the intersected vertical edges cannot overlap, there is at least one more horizontal edge in the bottom row of the second $M$-alternating cycle. \begin{figure}[h] \centering \includegraphics[height=2cm,width=8cm]{emm82222-eps-converted-to.pdf} \caption{\label{emm82222}Part of the two $M$-alternating cycles lying in corresponding ``marked subgraphs''.} \end{figure} Each time we translate the marked vertices down by one row, we obtain an abutting $M$-alternating cycle which contains more horizontal edges in the bottom row than the first one does. Since any $M$-alternating cycle contains no more than $2m$ horizontal edges on its bottom row, there is a placement of marked vertices such that ``marked subgraph'' contains no $M$-alternating cycles. \end{proof} \subsection{\small The maximum forcing number of $T(2n,2m,2r)$ for $1\leq r\leq m$} By Lemma \ref{lem1}, $T(n,m,r)$ contains $(r,m)$ $\mathrm{I}$-cycles, and each $\mathrm{I}$-cycle contains $\frac{mn}{(r,m)}$ vertices. For $(r,m)\geq 2$ and $j\in Z_{(r,m)}$, the subgraph induced by all vertices of the two $\mathrm{I}$-cycles containing $j$-column and $(j+1)$-column contains a subgraph isomorphic to $C_{\frac{mn}{(r,m)}}\square P_2$, denoted by $C_{j,j+1}$. Particularly, $C_{j,j+1}$ is isomorphic to $C_{\frac{mn}{(r,m)}}\square P_2$ for $(r,m)\geq 3$ where $j\in Z_{(r,m)}$. \begin{thm}\label{mqps1}For $n,m\geq 2$ and $1\leq r\leq m$, we have \begin{equation*} F(T(2n,2m,2r))= \begin{cases} mn+1, & \quad {if\ (r,m)=1};\\ mn,&\quad {otherwise}. \end{cases} \end{equation*} \end{thm} \begin{proof}First we prove the case that $(r,m)\neq 1$. Let $M_1=E_0\cup E_2\cup \dots \cup E_{2m-2}$ be a perfect matching of $T(2n,2m,2r)$ shown as Fig. \ref{em1}(a), where $E_j=\{v_{i,j}v_{i,j+1}|i\in Z_{2n}\}$. Then $C_{2j,2j+1}$ contains a subgraph isomorphic to $C_{\frac{2mn}{(r,m)}}\square P_2$ for $j\in Z_{(r,m)}$ and contains $\frac{mn}{(r,m)}$ disjoint $M_1$-alternating cycles. Hence, $T(2n,2m,2r)$ contains $mn$ disjoint $M_1$-alternating cycles and $f(T(2n,2m,2r),M_1)\geq mn$. Form a forcing set of size $mn$ so that half horizontal edges of $C_{2j,2j+1}$ are chosen for $j\in Z_{(r,m)}$. Precisely, from top to bottom we choose 1'th, 3'th, $\dots$, $(\frac{2mn}{(r,m)}-1)'$th horizontal edges of $C_{4j,4j+1}$ for $j\in \lceil\frac{(r,m)}{2}\rceil$ and 2'th, 4'th, $\dots$, $\frac{2mn}{(r,m)}$'th horizontal edges of $C_{4j+2,4j+3}$ for $j\in \lfloor\frac{(r,m)}{2}\rfloor$ (red lines of $T^*(2n,2m,2r)$ in Fig. \ref{em1}(b) and that of $T(2n,2m,2r)$ in Fig. \ref{em1}(c) form a forcing set). Hence, $f(T(2n,2m,2r),M_1)= mn$. \begin{figure}[h] \centering \includegraphics[height=5.5cm,width=14cm]{em11-eps-converted-to.pdf} \caption{\label{em1}The perfect matching $M_1$ of $T(4,8,4)$, where red lines form a forcing set of $M_1$.} \end{figure} Let $M$ be any perfect matching of $T(2n,2m,2r)$. It suffices to prove that $$f(T(2n,2m,2r),M)\leq mn.$$ If none of $\mathrm{II}$-cycles is $M$-alternating, then we can mark $mn$ vertices so that ``marked subgraph'' contains no $M$-alternating cycles by Lemma \ref{modifiedcycle}. Otherwise, there is an $M$-alternating $\mathrm{II}$-cycle. Then each $\mathrm{I}$-cycle is not $M$-alternating. By Lemma \ref{drawing}, $T(2n,2m,2r)$ has another representation $$T^*(2n,2m,2r)=T(2(r,m), \frac{2nm}{(r,m)},2n(\frac{m}{(r,m)}-k)),$$ in which each $\mathrm{II}$-cycle is not $M$-alternating. By Lemma \ref{modifiedcycle}, we can mark $mn$ vertices so that ``marked subgraph'' contains no $M$-alternating cycles. By Lemma \ref{forcingset}, $$f(T(2n,2m,2r),M)=f(T^*(2n,2m,2r),M)\leq |M|-|T|=mn.$$ By the arbitrariness of $M$, we have $F(T(2n,2m,2r))\leq mn$. Next we prove the case that $(r,m)= 1$. By Lemma \ref{lem1}, $T(2n,2m,2r)$ has exactly two $\mathrm{I}$-cycles. Let $M_1=E_0\cup E_2\cup \dots \cup E_{2m-2}$ be a perfect matching of $T(2n,2m,2r)$ shown as bold lines in Fig. \ref{em12}(a). \begin{figure}[h] \centering \includegraphics[height=3.5cm,width=14cm]{em12222-eps-converted-to.pdf} \caption{\label{em12}The perfect matching $M_1$ of $T(4,10,4)$, and red lines cannot form a forcing set of $M_1$.} \end{figure} Since $C_{0,1}$ contains a subgraph isomorphic to $C_{2nm}\square P_2$, $T(2n,2m,2r)$ contains $mn$ disjoint $M_1$-alternating cycles. Since a forcing set of $M_1$ contains at least one edge from each $M_1$-alternating cycle, any forcing set of $M_1$ has size at least $mn$. To find a forcing set of size $mn$, we need to choose one of the horizontal edges in any two consecutive ones of $C_{0,1}$. In $C_{0,1}$, starting with the two consecutive edges $v_{0,0}v_{0,1}$ and $v_{1,0}v_{1,1}$, in which the latter are chosen, we choose a set of horizontal edges with size $mn$ shown as red lines in Fig. \ref{em12}(b), where each $E_{2j}$ for $j\in Z_{m}$ has $n$ edges $\{v_{2i+1,2j}v_{2i+1,2j+1}|i\in Z_n\}$ being chosen. But the chosen $mn$ edges cannot form a forcing set of $M_1$ for there are still $n$ $\mathrm{II}$-cycles being not intersected with such $mn$ edges (see red lines in Fig. \ref{em12}(a)). Hence, $f(T(2n,2m,2r),M_1)\geq mn+1$. It's easy to find a forcing set of size $mn+1$. Thus $f(T(2n,2m,2r),M_1)=mn+1$. For any perfect matching $M$ of $T(2n,2m,2r)$, we are to prove that $$f(T(2n,2m,2r),M)\leq mn+1.$$ By Lemma \ref{forcingset}, it suffices to prove that we can mark at least $mn-1$ vertices in $T(2n,2m,2r)$ such that ``marked subgraph'' contains no $M$-alternating cycles. If each $\mathrm{II}$-cycle is not $M$-alternating, then we can mark $mn$ vertices so that ``marked subgraph'' contains no $M$-alternating cycles by Lemma \ref{modifiedcycle}. Otherwise, assume that $v_{2n-1,0}v_{2n-1,1}\cdots v_{2n-1,2m-1}v_{2n-1,0}$ is an $M$-alternating cycle, and $\{v_{2n-1,2j}v_{2n-1,2j+1}|j\in Z_{m}\}\subseteq M$. Let $$X_*=\{v_{0,1},v_{0,3},\dots,v_{0,2r-1},v_{0,2r+3},v_{0,2r+5},\dots,v_{0,2m-1}\} \text{ and } Y_*=\{v_{3,0},v_{5,0},\dots,v_{2n-1,0}\}.$$ Take $T=Y_*\cup X_*\cup X'_2\cup X'_4\cup \dots \cup X'_{2n-2}$ as marked vertices shown as Fig. \ref{em122}, where $X'_{i}=X_{i}-\{v_{i,0}\}$ for $i\in Z_{2n}$. Then all vertices on the third row don't lie on the ``marked subgraph'', and ``marked subgraph'' is a plane graph shown as red lines in Fig. \ref{em122}. \begin{figure}[h] \centering \includegraphics[height=5.5cm,width=12.5cm]{emm5-eps-converted-to.pdf} \caption{\label{em122}Marked vertices and ``marked subgraph'' of $T(8,6,2)$ and $T(8,10,4)$.} \end{figure} The ``marked subgraph'' formed by all paths of length two whose initial and terminal are in $X'_2\cup X'_4 \cup \cdots \cup X'_{2n-2}$ is a $2\times 2$-polyomino corresponding to a $(n-2)\times (m-2)$ chessboard. Noting that both $v_{2n-1,0}$ and $v_{0,2r-1}$ are marked vertices, $v_{2n-1,0}v_{2n-1,2m-1}v_{0,2r-1}v_{0,2r}v_{2n-1,0}$ is contained in ``marked subgraph'', and the ``marked subgraph'' formed by all paths of length two whose initial and terminal are in $X_*\cup Y_*$ is a cycle of length 4 attaching a path on $2m-2$ vertices and a path on $2n-3$ vertices. Furthermore, ``marked subgraph'' consists of a $2\times 2$-polyomino corresponding to a $(n-2)\times (m-2)$ chessboard and a 4-cycle attaching a path on $2m-2$ vertices and a path on $2n-3$ vertices. Since $v_{2n-1,0}v_{2n-1,1}\in M$, such 4-cycle $v_{2n-1,0}v_{2n-1,2m-1}v_{0,2r-1}v_{0,2r}v_{2n-1,0}$ is not $M$-alternating. By Lemma \ref{polyomino}, a $2\times 2$-polyomino contains no $M$-alternating cycles. Thus, ``marked subgraph'' contains no $M$-alternating cycles. By Lemma \ref{forcingset}, $M\setminus E_{T}$ is a forcing set of $M$ and $$f(T(2n,2m,2r),M)\leq |M|-|T|\leq 2mn-(mn-1)=mn+1.$$ By the arbitrariness of $M$, we have $F(T(2n,2m,2r))\leq nm+1$. \end{proof} \subsection{\small The maximum forcing number of $T(2n,2m,2r-1)$ for $1\leq r\leq m$} Next we will obtain the maximum forcing number of $T(2n,2m,2r-1)$ for $1\leq r\leq m$. \begin{thm}\label{even}For $n\geq1$, $m\geq 2$ and $1\leq r\leq m$, $F(T(2n,2m,2r-1))=mn$. \end{thm} \begin{proof} Let $M_1=W_0\cup W_1\cup \cdots \cup W_{2m-1}$ be a perfect matching of $T(2n,2m,2r-1)$. Since $R_{2i,2i+1}$ contains a subgraph isomorphic to $C_{2m}\square P_2$, it contains $m$ disjoint $M_1$-alternating cycles for $i\in Z_n$. Thus, any forcing set of $M_1$ has size at least $mn$. Clearly, $W^2_0\cup W^1_1\cup W^2_2\cup \cdots \cup W^2_{2m-2}\cup W^1_{2m-1}$ shown as red lines in Fig. \ref{fig11} is a forcing set of $M_1$ with size $mn$. Hence, we obtain that $f(T(2n,2m,2r-1), M_1)=mn$. \begin{figure}[h] \centering \includegraphics[height=4.2cm,width=15cm]{fig11.png} \caption{\label{fig11}Perfect matchings $M_1$ of $T(4,10,5)$ and $T(6,10,5)$, where red lines form a forcing set.} \end{figure} Let $M$ be any perfect matching of $T(2n,2m,2r-1)$, we are to prove that $$f(T(2n,2m,2r-1),M)\leq mn.$$ It suffices to mark $mn$ vertices of $T(2n,2m,2r-1)$ such that ``marked subgraph'' contains no $M$-alternating cycles. If we have done, then by Lemma \ref{forcingset}, we have $$f(T(2n,2m,2r-1),M)\leq |M|-mn=mn.$$ By the arbitrariness of $M$, we have $F(T(2n,2m,2r-1))\leq mn$. For $n\geq 2$, we only suffice to prove the case that there is a $\mathrm{II}$-cycle is $M$-alternating by Lemma \ref{modifiedcycle}. For $n=1$, $n$ and $2r-1$ are of the same parity, by the proof of Lemma \ref{modifiedcycle}, we also need to prove the same case as $n\geq 2$. Without loss of generality, we suppose that $v_{2n-1,0}v_{2n-1,1}\cdots v_{2n-1,2m-1}v_{2n-1,0}$ is an $M$-alternating $\mathrm{II}$-cycle, and $\{v_{2n-1,2j}v_{2n-1,2j+1}|j\in Z_m\}\subseteq M.$ Let $T=Y_*\cup X'_0 \cup X'_2\cup \cdots \cup X'_{2n-2}$ (see Fig. \ref{mmark2}) as marked vertices, where $$Y_*=\{v_{2n-1,2m-2r+1},v_{1,0}, v_{3,0},\dots, v_{2n-3,0}\} \text{ and } X'_{i}=X_{i}-\{v_{i,0}\} \text{ for } i\in Z_{2n}.$$ Then $T$ is of size $mn$. Since any vertices of $Y_*$ and that of $X'_{2i}$ belong to no same rows for $i\in Z_{n}$, any vertices of $\{v_{i,1}, v_{i,2m-1}|i\in Z_{2n}\}$ are not contained in ``marked subgraph''. Furthermore, any vertices of $\{v_{2n-1,2m-2r+1+j}|j=2,3,\dots,2m-2\}$ are not contained in ``marked subgraph''. Thus, ``marked subgraph'' is a plane graph shown as red lines in Fig. \ref{mmark2}. The ``marked subgraph'' formed by all paths of length two whose initial and terminal are in $X'_0\cup X'_2\cup X'_4 \cup \cdots \cup X'_{2n-2}$ is a $2\times 2$-polyomino corresponding to a $(n-1)\times (m-2)$ chessboard, which contains no $M$-alternating cycles by Lemma \ref{polyomino}. \begin{figure}[h] \centering \includegraphics[height=4.6cm,width=13.5cm]{mmark2-eps-converted-to.pdf} \caption{\label{mmark2}Marked vertices and ``marked subgraph'' of $T(6,10,5)$ and $T(6,6,3)$.} \end{figure} Since $v_{2n-1,2m-2r+1}$, $v_{2n-2,2m-2r}$ and $v_{2n-2,2m-2r+2}$ are marked vertices, four paths of length two $v_{2n-2,2m-2r}v_{2n-1,2m-2r}v_{2n-1,2m-2r+1}$, $v_{2n-2,2m-2r}v_{2n-2,2m-2r+1}v_{2n-1,2m-2r+1}$, $v_{2n-2,2m-2r+1}\\v_{2n-2,2m-2r+2}v_{2n-1,2m-2r+2}$ and $v_{2n-2,2m-2r+1}v_{2n-1,2m-2r+1}v_{2n-1,2m-2r+2}$ are contained in ``marked subgraph''. Let $C$ be an $M$-alternating cycle of ``marked subgraph''. Then $C$ contains the vertex $v_{2n-1,2m-2r+1}$. Since $C$ is $M$-alternating, it also contains three edges $v_{2n-1,2m-2r}v_{2n-2,2m-2r}$, $v_{2n-1,2m-2r}v_{2n-1,2m-2r+1}$ and $v_{2n-1,2m-2r+1}v_{2n-2,2m-2r+1}$, and such four vertices $v_{2n-1,2m-2r}$,\\ $v_{2n-1,2m-2r+1}$, $v_{2n-2,2m-2r}$ and $v_{2n-2,2m-2r+1}$ are on the boundary of $\text{Int}[C]$. Next, we prove that $C$ contains exactly such four vertices. If $C$ contains at least six vertices, then $\text{Int}[C]$ and $\text{Int}[C]-\{v_{2n-1,2m-2r}, v_{2n-1,2m-2r+1}\}$ have the same number of interior vertices. Since $\text{Int}[C]-\{v_{2n-1,2m-2r}, v_{2n-1,2m-2r+1}\}$ is a $2\times 2$-polyomino, it has an odd number of interior vertices by Lemma \ref{polyomino}. Thus, $\text{Int}[C]$ has an odd number of interior vertices, which contradicts that $C$ is $M$-alternating. Thus $$C=v_{2n-1,2m-2r}v_{2n-1,2m-2r+1}v_{2n-2,2m-2r+1} v_{2n-2,2m-2r}v_{2n-1,2m-2r}.$$ If $v_{2n-2,2m-2r}v_{2n-2,2m-2r+1}\notin M$, then $C$ is not $M$-alternating. Hence none of cycles in ``marked subgraph'' is $M$-alternating. So we assume that $v_{2n-2,2m-2r}v_{2n-2,2m-2r+1}\in M$. Translating marked vertices right by two columns, by a similar argument, we suffice to consider the case that $v_{2n-2,2m-2r+2}v_{2n-2,2m-2r+3}\in M$. Proceeding like this, it suffices to consider the case that $M$ has the same matching form on the last $2n$ rows, i.e., $\{v_{i,2j}v_{i,2j+1}|j\in Z_m\}\subseteq M$ for $0\leq i\leq 2n-1$. Since the torsion is $2r-1$, $M$ has different matching form on the first two rows. By the previous argument, we have done. \end{proof} \section{\normalsize Discussion of the maximum forcing number of $T(2n+1,2m,r)$ for $1\leq r\leq 2m$} By Theorems \ref{odd} and \ref{even}, we obtain the maximum forcing number of $T(2n+1,2m,2r)$ for $1\leq r\leq m$. \begin{thm}\label{mqps0} For $n\geq 1$, $m\geq 2$ and $1\leq r\leq m$, we have \begin{equation*} F(T(2n+1,2m,2r))= \begin{cases} \frac{m(2n+1)+(r,m)}{2}, & \quad {if\ \frac{m}{(r,m)}\ is\ odd};\\ \frac{m(2n+1)}{2},&\quad {otherwise}. \end{cases} \end{equation*} \end{thm} \begin{proof}By Lemma \ref{drawing}, $T(2n+1,2m,2r)$ has another representation $$T^*(2n+1,2m,2r)=T(2(r,m),\frac{m(2n+1)}{(r,m)},(2n+1)(\frac{m}{(r,m)}-k))$$ where $0\leq k\leq \frac{m}{(r,m)}-1$ satisfies the equation $(2r,2m)\equiv 2rk$ (mod $2m$). If $\frac{m}{(r,m)}$ is even, then $2rk-(2r,2m)= 2mp$ for some non-negative integer $p$. That is, $rk-(r,m)= mp$. Thus $\frac{r}{(r,m)}k= \frac{m}{(r,m)}p+1$. Since $\frac{m}{(r,m)}$ is even and $\frac{m}{(r,m)}p+1$ is odd, we obtain that $k$ is an odd number. Hence $\frac{m}{(r,m)}-k$ and $(2n+1)(\frac{m}{(r,m)}-k)$ are also odd numbers. Let $n'=(r,m)$, $m'=\frac{m(2n+1)}{2(r,m)}$ and $2r'-1=(2n+1)(\frac{m}{(r,m)}-k)$. Then $T^*(2n+1,2m,2r)=T(2n',2m',2r'-1).$ Since $0\leq k\leq \frac{m}{(r,m)}-1$, we have $2n+1\leq 2r'-1 \leq (2n+1)\frac{m}{(r,m)}=2m'$. Thus $n+1\leq r'<m'$. By Theorem \ref{even}, we have $$F(T(2n+1,2m,2r))=F(T(2n',2m',2r'-1))=m'n'=\frac{m(2n+1)}{2}.$$ If $\frac{m}{(r,m)}$ is odd, then $2(r,m)$ is even, $\frac{m(2n+1)}{(r,m)}$ is odd. Let $n'=(r,m)$, $2m'+1=\frac{m(2n+1)}{(r,m)}$ and $r'=(2n+1)(\frac{m}{(r,m)}-k)$. Since $0\leq k\leq \frac{m}{(r,m)}-1$, we have $2n+1\leq r'\leq (2n+1)\frac{m}{(r,m)}=2m'+1$. By Theorem \ref{odd}, we have $$F(T(2n+1,2m,2r))=F(T(2n',2m'+1,r'))=(m'+1)n'=\frac{m(2n+1)+(r,m)}{2}.$$ Now we finish the proof. \end{proof} For $T(2n+1,2m,2r-1)$, we have not been able to obtain a general expression for the maximum forcing number for $1\leq r\leq m$. Therefore, computing the maximum forcing number of $T(2n+1, 2m, 2r-1)$ is an open problem. \vspace{2mm} \noindent{\normalsize \textbf{Acknowledgments }} This work is supported by NSFC\,(Grant No. 12271229), start-up funds of Inner Mongolia Autonomous Region (Grant No. DC2400002165) and Inner Mongolia University of Technology (Grant No. BS2024038). \vspace{3mm} \noindent{\normalsize \textbf{Conflict of interest }} The authors declare no conflict of interest. \begin{thebibliography}{99} \setlength{\itemsep}{-.2mm} \bibitem{13}Abeledo H., Atkinson G. W., Unimodularity of the Clar number problem. Linear Algebra Appl., 2007, 420: 441-448 \bibitem{5}Afshani P., Hatami H., Mahmoodian E. S., On the spectrum of the forcing matching number of graphs. Australas. J. Combin., 2004, 30: 147-160 \bibitem{29}Jiang X., Zhang H., The maximum forcing number of cylindrical grid, toroidal 4-8 lattice and Klein bottle 4-8 lattice. J. Math. Chem., 2016, 54: 18-32 \bibitem{3}Klein K. J., Randi\'{c} M., Innate degree of freedom of a graph. J. Comput. Chem., 1987, 8: 516-521 \bibitem{16}Kleinerman S., Bounds on the forcing numbers of bipartite graphs. Discrete Math., 2006, 306: 66-73 \bibitem{9}Lam F., Pachter L., Forcing numbers of stop signs. Theoret. Comput. Sci., 2003, 303: 409-416 \bibitem{classfy}Li Q., Study on the Matching Extension of Graphs in Surfaces. Ph.D. Thesis, Lanzhou University, 2012 \bibitem{lin2017}Lin Y., Wang M., Xu L., Zhang F., The maximum forcing number of a polyomino. Australas. J. Combin., 2017, 69: 306-314 \bibitem{LYZ}Liu Q., Zhang Y., Zhang H., Components of domino tilings under flips in quadriculated tori. preprint, 2023, \url{https://arxiv.org/pdf/2307.08332v2.pdf} \bibitem{6}Pachter L., Kim P., Forcing matchings on square girds. Discrete Math., 1998, 190: 287-294 \bibitem{klein85}Randi\'{c} M., Klein D. J., Kekul\'{e} valence structures revisited. Innate degrees of freedom of $\pi$-electron couplings, in: N. Trinajsti'c (Eds). Mathematical and Computational Concepts in Chemistry, Wiley, New York, 1985: 274-282 \bibitem{32}Randi\'{c} M., Vuki\u{c}evi\'{c} D., Kekul\'{e} structures of fullerene C$_{70}$. Croat. Chem. Acta, 2006, 79(3): 471-481 \bibitem{40}Shi L., Wang H., Zhang H., On the maximum forcing and anti-forcing numbers of (4,6)-fullerenes. Discrete Appl. Math., 2017, 233: 187-194 \bibitem{27}Xu L., Bian H., Zhang F., Maximum forcing number of hexagonal systems. MATCH Commun. Math. Comput. Chem., 2013, 70: 493-500 \bibitem{Tho}Thomassen C., Tilings of the torus and the Klein bottle and vertex-transitive graphs on a fixed surface. Trans. Amer. Math. Soc., 1991, 323(2): 605-635 \bibitem{zhou2016}Zhang H., Zhou X., A maximum resonant set of polyomino graphs. Discuss. Math. Graph Theory, 2016, 36: 323-337 \end{thebibliography} \end{document}
2412.06423v1
http://arxiv.org/abs/2412.06423v1
Equilibrium States for Piecewise Weakly Convex Interval Maps
\documentclass[letterpaper,10pt,reqno]{amsart} \usepackage{graphicx} \usepackage{caption} \usepackage{subcaption} \usepackage{indentfirst} \usepackage{mathtools} \usepackage{mathabx} \usepackage{amsthm} \usepackage{thmtools} \usepackage{enumitem} \usepackage{fancyhdr} \usepackage{lipsum} \pagestyle{fancy} \headheight=14pt \fancyhf{} \chead{Equilibrium states for geometric potentials} \thispagestyle{fancy} \rhead{\thepage} \usepackage[colorlinks=true]{hyperref} \usepackage[usenames,dvipsnames]{xcolor} \usepackage{ifthen} \usepackage{tikz} \usetikzlibrary{decorations.pathreplacing} \usepackage{tikz-cd} \usepackage{dsfont} } \makeatletter \def\MRbibitem{\@ifnextchar[\my@lbibitem\my@bibitem} \def\mybiblabel#1#2{\@biblabel{{\hyperref{http://www.ams.org/mathscinet-getitem?mr=#1}{}{}{#2}}}} \def\myhyperanchor#1{\Hy@raisedlink{\hyper@anchorstart{cite.#1}\hyper@anchorend}} \def\my@lbibitem[#1]#2#3#4\par{ \item[\mybiblabel{#2}{#1}\myhyperanchor{#3}\hfill]#4 \@ifundefined{ifbackrefparscan}{}{\BR@backref{#3}} \if@filesw{\let\protect\noexpand\immediate\ignorespaces } \def\my@bibitem#1#2#3\par{ \refstepcounter\@listctr \item[\mybiblabel{#1}{\the\value\@listctr}\myhyperanchor{#2}\hfill]#3 \@ifundefined{ifbackrefparscan}{}{\BR@backref{#2}} \if@filesw\immediate\write\@auxout\ignorespaces } \makeatother \DeclareFontFamily{U} {MnSymbolA}{} \DeclareFontShape{U}{MnSymbolA}{m}{n}{ <-6> MnSymbolA5 <6-7> MnSymbolA6 <7-8> MnSymbolA7 <8-9> MnSymbolA8 <9-10> MnSymbolA9 <10-12> MnSymbolA10 <12-> MnSymbolA12}{} \DeclareFontShape{U}{MnSymbolA}{b}{n}{ <-6> MnSymbolA-Bold5 <6-7> MnSymbolA-Bold6 <7-8> MnSymbolA-Bold7 <8-9> MnSymbolA-Bold8 <9-10> MnSymbolA-Bold9 <10-12> MnSymbolA-Bold10 <12-> MnSymbolA-Bold12}{} \DeclareSymbolFont{MnSyA} {U} {MnSymbolA}{m}{n} \DeclareFontFamily{U} {MnSymbolC}{} \DeclareFontShape{U}{MnSymbolC}{m}{n}{ <-6> MnSymbolC5 <6-7> MnSymbolC6 <7-8> MnSymbolC7 <8-9> MnSymbolC8 <9-10> MnSymbolC9 <10-12> MnSymbolC10 <12-> MnSymbolC12}{} \DeclareFontShape{U}{MnSymbolC}{b}{n}{ <-6> MnSymbolC-Bold5 <6-7> MnSymbolC-Bold6 <7-8> MnSymbolC-Bold7 <8-9> MnSymbolC-Bold8 <9-10> MnSymbolC-Bold9 <10-12> MnSymbolC-Bold10 <12-> MnSymbolC-Bold12}{} \DeclareSymbolFont{MnSyC} {U} {MnSymbolC}{m}{n} \DeclareMathSymbol{\top}{\mathord}{MnSyA}{219} \DeclareMathSymbol{\plus}{\mathord}{MnSyC}{20} \declaretheorem[numberwithin=section]{theorem} \declaretheorem[sibling=theorem]{lemma} \declaretheorem[sibling=theorem]{corollary} \declaretheorem[sibling=theorem]{proposition} \declaretheorem[sibling=theorem,style=definition]{definition} \declaretheorem[sibling=theorem]{remark} \declaretheorem[sibling=theorem]{example} \declaretheorem[sibling=theorem,style=remark]{conjecture} \declaretheorem[name=Acknowledgements, style=remark, numbered=no]{ack} \newtheorem{claim}{Claim} \setcounter{tocdepth}{1} \setcounter{secnumdepth}{3} \hypersetup{bookmarksdepth = 3} \numberwithin{equation}{section} \setlist[enumerate,1]{label={\upshape(\alph*)},ref=\alph*} \setlist[enumerate,2]{label={\upshape(\arabic*)},ref=\arabic*} \newcommand{\M}{\mathcal{M}} \newcommand{\R}{\mathbb{R}} \newcommand{\Z}{\mathbb{Z}} \newcommand{\N}{\mathbb{N}} \newcommand{\E}{\mathbb{E}} \renewcommand{\P}{\mathbb{P}} \newcommand{\cA}{\mathcal{A}}\newcommand{\cB}{\mathcal{B}}\newcommand{\cC}{\mathcal{C}} \newcommand{\cD}{\mathcal{D}}\newcommand{\cE}{\mathcal{E}}\newcommand{\cF}{\mathcal{F}} \newcommand{\cG}{\mathcal{G}}\newcommand{\cH}{\mathcal{H}}\newcommand{\cI}{\mathcal{I}} \newcommand{\cJ}{\mathcal{J}}\newcommand{\cK}{\mathcal{K}}\newcommand{\cL}{\mathcal{L}} \newcommand{\cM}{\mathcal{M}}\newcommand{\cN}{\mathcal{N}}\newcommand{\cO}{\mathcal{O}} \newcommand{\cP}{\mathcal{P}}\newcommand{\cQ}{\mathcal{Q}}\newcommand{\cR}{\mathcal{R}} \newcommand{\cS}{\mathcal{S}}\newcommand{\cT}{\mathcal{T}}\newcommand{\cU}{\mathcal{U}} \newcommand{\cV}{\mathcal{V}}\newcommand{\cW}{\mathcal{W}}\newcommand{\cX}{\mathcal{X}} \newcommand{\cY}{\mathcal{Y}}\newcommand{\cZ}{\mathcal{Z}} \def\c{{\bf C}} \def\eps{\varepsilon} \def\phi{\varphi} \def\R{{\mathbb R}} \def\C{{\mathbb C}} \def\N{{\mathbb N}} \def\Z{{\mathbb Z}} \def\disk{{\mathbb D}} \def\H{{\mathbb H}} \def\E{{\mathcal E}} \def\O{{\mathcal O}} \def\J{{\mathcal J}} \def\L{{\mathcal L}} \def\B{{\mathcal B}} \def\I{{\mathcal I}} \def\P{{\mathcal P}} \def\Q{{\mathcal Q}} \def\F{{\mathcal F}} \def\D{{\mathcal D}} \def\M{{\mathcal M}} \def\A{{\mathcal A}} \def\S{{\mathcal S}} \def\T{{\mathcal T}} \def\CA{{\mathcal CA}} \def\cont{{\mathcal C}} \def\es{{\emptyset}} \def\sm{\setminus} \def\dist{\mbox{dist}} \def\Dist{\mbox{Dist}} \def\supp{\mbox{\rm supp}} \def\diam{\mbox{\rm diam} } \def\orb{\mbox{\rm orb}} \def\Crit{\mbox{\rm Crit}} \def\crit{{\mathcal Cr}} \def\cut{{\mathcal Cut}} \def\esc{\mbox{E}} \def\conv{\mbox{conv\ }} \def\bd{\partial } \def\le{\leqslant} \def\ge{\geqslant} \def\st{such that } \def\e{\epsilon} \def\F{\mathcal{F}} \def\M{\mathcal{M}} \def\dimspec{\mathfrak{D}} \def\htop{h_{top}} \def\trans{\mathcal{T}} \def\G{\mathcal{G}} \newcommand{\margin}[1]{\marginpar{{\scriptsize \sf \ensuremath{\triangleright}{#1}}}} \newcommand{\vertiii}[1]{{\left\vert\kern-0.25ex\left\vert\kern-0.25ex\left\vert #1 \right\vert\kern-0.25ex\right\vert\kern-0.25ex\right\vert}} \newcommand{\invertiii}[1]{{\vert\kern-0.25ex\vert\kern-0.25ex\vert #1 \vert\kern-0.25ex\vert\kern-0.25ex\vert}} \newcommand{\parti}[1]{{\left\{ #1 \right\}}} \begin{document} \title{Equilibrium states for piecewise weakly convex interval maps} \date{\today} \thanks{Supported by ANID Doctorado Nacional 21210037.} \subjclass[2020]{Primary: 37D35; Secondary:37D25, 37E05} \keywords{Equilibrium states, piecewise monotone transformations, geometric potential.} \author{ Nicol\'as Ar\'evalo H.} \address{Facultad de Matem\'aticas, Pontificia Universidad Cat\'olica de Chile (UC), Avenida Vicu\~na Mackenna 4860, Santiago, Chile} \email{\href{[email protected]}{[email protected]}} \urladdr{\url{https://sites.google.com/view/nicolasarevalomath/}} \begin{abstract} We establish the existence of equilibrium states for geometric potentials in a family of piecewise weakly convex interval maps. This family includes systems with indifferent fixed points and non-Markov partitions. Under specific conditions, we also prove the uniqueness of equilibrium states. \end{abstract} \maketitle \section{Introduction} Consider $T:[0,1]\rightarrow [0,1]$ to be a non-singular \textit{piecewise monotone and, orientation-preserving transformation}, i.e., there are numbers $0=a_{0}<\cdots < a_{N}=1$ such that for each $1\leq k \leq N$ the restriction $T_{k}:=T|_{[a_{k-1},a_{k})}$ is strictly increasing and continuous, and $m\circ T^{-1}_{k}$ is absolutely continuous with respect to $m$, where $m$ is the Lebesgue measure. Since each branch $T_{k}$ is injective, an extended inverse function can be defined as follows: for each $x\in [0,1]$ \begin{align*} \psi_{k}(x):=\begin{cases}a_{k-1} & \text{if $x\leq \inf_{y\in [a_{k-1},a_{k})} T_{k}(y)$},\\ T^{-1}_{k}(x) & \text{if $x\in T_{k}(a_{k-1},a_{k})$},\\ a_{k} & \text{if $x\geq \sup_{y\in [a_{k-1},a_{k})} T_{k}(y)$}. \end{cases} \end{align*} Since $\psi_{k}$ is differentiable for Lebesgue almost every point, we may assume that for each $1\leq k \leq N$, the function $\sum^{k}_{i=1}\psi_{i}'$ is upper-semicontinuous. In \cite{bmvs}, Bose, Maume-Deschamps, Schmitt, and Shin considered non-singular piecewise monotone and orientation preserving maps such that \begin{enumerate} \item \label{Cond1} $T'(0)>1$ and $T_{k}'(a_{k-1})>0$ for each $1\leq k \leq N$. \item \label{Cond2} the function $\sum^{k}_{i=1}\psi_{i}'$ is non-increasing for each $1\leq k\leq N$. \end{enumerate} For each map $T$ satisfying these two conditions, the existence of a $T$-invariant measure, absolutely continuous (ACIM) with respect to the Lebesgue measure, was established (see \cite[Theorem 1.1]{bmvs}). This generalizes Lasota and Yorke's results on a class $\mathcal{C}$ of piecewise monotone, orientation-preserving transformations in which each $T_{k}$ is convex, $T_{k}(a_{k-1})=0$ and $T'(0)>1$ (see Example \ref{ConvexTransformationsLasotaYorke} and \cite{ly}). Our objective is to extend the study of the ergodic properties of a subfamily satisfying conditions (\ref{Cond1}) and (\ref{Cond2}) but still containing $\mathcal{C}$. Since we may work with different ergodic measures, we need more than $\psi_{i}'$ to exist in a set of full Lebesgue measure. We say that $T$ is an \textit{$a$-convex} transformation if it satisfies condition (\ref{Cond1}) and that for each $1\leq k\leq N$, the function $\sum^{k}_{i=1}\psi_{i}$ is differentiable except, for at most on countable many points, and $\sum^{k}_{i=1}\psi_{i}'$ is not increasing. The weak convexity of these maps is due to the fact that if $T_{i}$ is convex, then $\psi_{i}$ is concave, so $\psi_{i}'$ exists except, at most, on a countable set and is a non-increasing function. This family, referred to as $a$-convex transformations due to their average convexity, may include concave branches (so some $\psi_{i}'$ may be increasing functions), indifferent fixed points, and non-Markov partitions. These characteristics limit our ability to apply conjugations with topological Markov shifts or to assume hyperbolicity, where thermodynamic formalism is more fully developed. Moreover, $a$-convex transformations share ordering properties with the family of attractive $g$-functions defined over the full shift with finite symbols (see \cite{hu} and Remark \ref{PHulseRemark}). Let $s\in \R$, and suppose $\mu_{s}$ is an equilibrium state for the geometric potential $-s\log|T'|$. It is well known that, since $\mu_{s}$ is ergodic, the Lyapunov exponent of $\mu_{s}$-almost every point is constant. Consequently, equilibrium states of geometric potentials are related to the possible values that the Lyapunov exponent can attain (see \cite{io,an,we}). In the hyperbolic context, the ACIM with respect to the Lebesgue measure is an equilibrium state for the potential $-\log|T'|$ (see \cite[Theorem 9.7.1]{BG}). We aim to develop Thermodynamic Formalism for geometric potentials of $a$-convex transformations—an area not addressed in \cite{bmvs}. Let $C([0,1])$ be the space of real-valued continuous functions on $[0,1]$. For each $s\in \R$ and for each $f\in C([0,1])$, we define the operator \begin{align*} F_{s}(f)=\sum^{N}_{i=1}(\psi_{i}')^{s}f\circ \psi_{i}. \end{align*} The expression of the Frobenius-Perron operator (when $s=1$) is well known in the literature (see \cite[Chapter 4]{ly}). Following \cite{hk1}, in section 3, we will define an extension $\overline{[0,1]}$ of $[0,1]$ with the order topology such that the corresponding extension of $F_{s}$ is the transfer operator for $-s\log|T'|$, where $T'$ is continuously extended. We will show the existence of a \textit{conformal measure} $m_{s}$ related to $F_{s}$ on $\overline{[0,1]}$, i.e., for each Borel set $A$ on $\overline{[0,1]}$ and for any $f\in L^{1}_{m_{s}}(\overline{[0,1]})$ \begin{align*} \int_{T^{-1}A}fdm_{s}=\int_{A}F_{s}fdm_{s}, \end{align*}up to a constant multiplication. Keller and Hofbauer (see \cite[Lemma 2,Theorem 6]{hk1}) showed that the measure $m_{s}$ assigns zero measure to the set of extended points for piecewise monotone transformations $T$ with an expansive iterate, i.e. $|(T^{n})'|>1$ for some $n\in \N$. Hence $m_{s}$ is, indeed, a probability measure over $[0,1]$. They also established the existence of a unique equilibrium state $\mu_{s}$ for $-s\log|T'|$, which is absolutely continuous with respect to $m_{s}$. In particular, their theorem applies to each map in $\mathcal{C}$ (see Example \ref{ConvexTransformationsLasotaYorke}). Here, our goal is to generalize this result for $a$-convex transformations. It is important to note that Keller and Hofbauer's arguments cannot be applied directly due to the presence of indifferent fixed points in some $a$-convex transformations. Moreover, even under the existence of indifferent fixed points, we do not employ inducing to study the thermodynamic formalism of $a$-convex transformations. A different approach has to be implemented to consider non-Markov partitions. Now we define sufficient conditions to prove our results. Given $T$ an $a$-convex transformation, there exists a point $\beta\in (0,1]$ such that $\bigcup^{\infty}_{i=0}T^{i}[0,a_{1}]=[0,\beta]$ (see \cite[Lemma 2]{bmvs}). Without loss of generality, we may assume that $\beta=a_{N^{*}}$ for some $1\leq N^{*}\leq N$ (see Remark \ref{SomeIterationExpansive}). This forward invariant interval is, in principle, the one that carries all interesting ergodic properties. For every $1\leq k\leq N$, let $I _{k}=[a_{k-1},a_{k})$, and for every array of integers $(d_{i})^{r}_{i=1}\in \{1,...,N\}^{r}$, define the cylinder \begin{align*} I_{(d_{i})^{r}_{i=1}}=\bigcap^{r-1}_{i=0}T^{-i}I_{d_{i+1}}. \end{align*} Note that $I_{(d_{i})^{r}_{i=1}}$ may be empty. Let $\mathds{1}$ be the constant function equal to 1. We say that $T$ satisfies \textit{condition $(B)$} for $s>0$ if there exists a sequence of positive numbers $\{M_{r}\}_{r\in \N}$ that converges to zero such that for every cylinder $I_{(d_{i})^{r}_{i=1}}$ that does not contain $\beta$, we have \begin{align*} \inf_{n\in \N} \dfrac{\|F^{n}_{s}\chi_{I_{(d_{i})^{r}_{i=1}}}\|_{\infty}}{(\int F_{s}\mathds{1}dm_{s})^{n}}\leq M_{r}, \end{align*} where $\|\cdot\|_{\infty}$ denotes the supremum norm. Let $\mathcal{J}$ be the cone of non-negative non-increasing functions over $[0,1]$. We say $T$ satisfies \textit{condition $(C)$} for $s>0$ if $\sum^{k}_{i=1}(\psi_{i}')^{s}\in \mathcal{J}$ for every $1\leq k \leq N$. \begin{theorem}\label{PrincipalThmChap52} Let $T$ be an $a$-convex transformation satisfying conditions $(B)$ and $(C)$ for a given $s>0$. If either \begin{align*} \lim_{x\rightarrow \beta^{-}}\psi_{N^{*}}'(x)<1\text{, or $\lim_{x\rightarrow \beta^{+}}\psi_{N^{*}+1}(x)=1$}, \end{align*} then there exists an equilibrium state $\mu_{s}$ for the potential $-s\log|T'|$ which is absolutely continuous with respect to $m_{s}$, and such that $\frac{d\mu_{s}}{dm_{s}}\in \mathcal{J}$. Furthermore, if $\beta=1$ and $m_{s}\neq \delta_{\beta}$, then $\mu_{s}$ is the unique equilibrium state for $-s\log|T'|$, which is absolutely continuous with respect to $m_{s}$. \end{theorem} This article is organized as follows: Section 2 presents examples of $a$-convex transformations not covered in \cite{bmvs}, illustrating different dynamics to which Theorem \ref{PrincipalThmChap52} applies. Section 3 addresses the existence of the conformal measure $m_{s}$ for the operator $F_{s}$. In Section 4, we study the space of the $F_{s}$-invariant functions over the space of bounded variation functions. Finally, Section 5 contains the proof of Theorem \ref{PrincipalThmChap52}. \section{On $a$-convex transformations} Given an $a$-convex transformation $T$, we say that a fixed point $\beta\in (0,1]$ is \textit{indifferent} if $|T'(\beta)|=1$. We choose the same letter $\beta$ as the point mentioned in Theorem \ref{PrincipalThmChap52} because, indeed, will be the only posible indifferent fixed point in $[0,\beta]$ (see Remark \ref{SomeIterationExp2}). The partition defined by $\{a_{i}\}_{i=0}^{N}$ is called a \textit{Markov partition} if for any $1\leq i,j \leq n$ such that $(a_{i-1},a_{i})\cap T((a_{j-1},a_{j}))$ is non-empty, then $(a_{i-1},a_{i})\subset T((a_{j-1},a_{j})).$ \begin{remark}\label{SomeIterationExpansive} Here we summarize the necessary properties of the point $\beta$ mentioned in the introduction to prove our results. Let $T$ be an $a$-convex transformation. Then, there exists a point $\beta \in (0,1]$ and a positive integer $r$ such that (see \cite[Lemma 4.1 and Lemma 4.2]{bmvs}) \begin{align}\label{ZeroBetaForwardInvariant} \bigcup^{r}_{k=0}T^{k}[0,a_{1}]=\bigcup^{\infty}_{k=0}T^{k}[0,a_{1}]=[0,\beta]. \end{align} Moreover, let $N^{*} \leq N$ be such that $\beta \in [a_{N^{*}-1}, a_{N^{*}}]$. We can split the $N^{*}$-th branch of $T$ so that $T|_{[0,\beta]}$ is again an $a$-convex transformation over the interval $[0, \beta]$. So, for now on let $\beta=a_{N^{*}}$. At first appearance, the subsystem $(T, [0, \beta])$ carries all the relevant dynamical properties, as it inherits the hyperbolicity of $T$ on $[0,a_{1}]$: \begin{itemize} \item If $\lim_{x\rightarrow \beta^{-}}\psi_{N^{*}}'(x)<1$, then $(T,[0,\beta])$ is exact, has exponential decay of correlations, and some iterate of $T$ is expansive on $[0,\beta]$ (see \cite[Theorem 4.5, Remark 4.2]{bmvs}). \item If for each $d\in (\beta,1]$, we have $T[\beta,d]\nsubseteq [\beta,d]$, then there exists an unique ACIM for the Lebesgue measure (see \cite[Corollary 6.3]{bmvs}). \end{itemize} \end{remark} Recall that, for each measurable set $A$ and for $f\in L^{1}_{m}([0,1])$, the Frobenius-Perron operator $F_{1}$ with respect to the Lebesgue measure $m$ satisfies \begin{align}\label{Perron-Frobenius} \int_{T^{-1}A}fdm=\int_{A}F_{1}fdm, \end{align}where $L^{1}_{m}([0,1])$ denotes the space of Lebesgue integrable functions on $[0,1]$ . \begin{example}\label{ExampleParabolicAconvex} For every $x\in [0,1]$, consider the map defined by (see Figure \ref{fig:$a$-convexParabolic}) \begin{align*} T(x)=\begin{cases} \frac{1}{3}(2+3x-2\sqrt{1-3x}) & \text{if $x\in [0,1/3)$,}\\ x-\frac{3}{4}(1-x)^{2} & \text{if $x\in [1/3,1]$}. \end{cases} \end{align*} This example is constructed to satisfy \begin{align}\label{ExamploParaAconvex} \psi_{1}'=1-\psi_{2}'. \end{align} Which by the Inverse Function Theorem implies \begin{align*} -\dfrac{T_{1}''\circ T_{1}^{-1}}{(T_{1}'\circ T_{1}^{-1})^{3}}=\dfrac{T_{2}''\circ T_{2}^{-1}}{(T_{2}'\circ T_{2}^{-1})^{3}}. \end{align*} Thus, $T_{1}$ is convex, $T_{2}$ is concave, and equation \eqref{ExamploParaAconvex} implies that $\psi_{1}'+\psi_{2}'=1$. Note also that $T_{1}'(0)>1$. Consequently, $T$ is an $a$-convex transformation, and the Lebesgue measure is $T$-invariant because $F_{1}\mathds{1}=\mathds{1}$ (see equation \ref{Perron-Frobenius}). Moreover there is an indifferent fixed point at $x=1$, which implies $\lim_{x\rightarrow 1/3^{-}}T_{1}'(x)=\infty$ by equation \eqref{ExamploParaAconvex}. \begin{figure}\label{ExampleFigure 1} \centering \begin{subfigure}[b]{0.45\textwidth} \centering \includegraphics[width=\textwidth]{Example_A-convex_non-uniformly_hyperbolic.png} \caption{Parabolic fixed point at $x=1$.} \end{subfigure} \hfill \begin{subfigure}[b]{0.45\textwidth} \centering \includegraphics[width=\textwidth]{Example_A-convex_non-uniformly_hyperbolic_2.png} \caption{Parabolic fixed point and non-markov partition.} \end{subfigure} \caption{The graph of $a$-convex transformations from Example \ref{ExampleParabolicAconvex} (A) and Example \ref{ExampleParabolicNonMarkovianAconvex} (B).} \label{fig:$a$-convexParabolic} \end{figure} Since $\psi_{1}'$ is non-increasing, it follows that $(\psi_{1}')^{s}$ is also non-increasing for every $s>0$. Additionally, for $s>0$ with $s\neq 1$, we have \begin{align*} \dfrac{d}{dx}((\psi_{1}')^{s}+(\psi_{2}')^{s})=s\psi_{2}''(-(1-\psi_{2}')^{s-1}+(\psi_{2}')^{s-1}). \end{align*} Because $\psi_{2}''>0$, the function $(\psi_{1}')^{s}+(\psi_{2}')^{s}$ is non-increasing if and only if \begin{align}\label{Ecu537} (\psi_{2}')^{s-1}-(1-\psi_{2}')^{s-1}\leq 0. \end{align} Given that $\psi_{2}'$ is increasing and $\psi_{2}'(0)=1/2$, we have $\psi_{2}'\geq \frac{1}{2}$. Therefore, equation \eqref{Ecu537} is only satisfied when $0<s<1$. Consequently, $T$ is an $a$-convex transformation that satisfies condition $(C)$ for any $0<s\leq 1$. In example \ref{ConditionBExamplesInfFixPt} we show this map satisfies condition $(B)$ for $0<s<1$.\end{example} \begin{example}\label{ExampleParabolicNonMarkovianAconvex} We can also construct an $a$-convex transformation with an indifferent fixed point at $x=1$, and without a Markov partition (see Figure \ref{fig:$a$-convexParabolic}). For each $x\in [0,1]$ consider \begin{align*} T(x)=\begin{cases} \frac{1}{4}\left(3-3\sqrt{1-4x}+4x\right) & \text{if $x\in [0,\frac{1}{4})$},\\ 15x-\frac{15}{4} & \text{if $x\in [\frac{1}{4},\frac{17}{60})$},\\ \frac{15}{2}x-\frac{17}{8} & \text{if $x\in [\frac{17}{60},\frac{2}{5})$},\\ 15x-\frac{11}{2} & \text{if $x\in [\frac{2}{5},\frac{32}{75})$}. \end{cases} \end{align*} The construction of $T_{2},T_{3}$, and $T_{4}$ is done so that the $\sum^{k}_{i}\psi_{i}'< 1$ and $\sum^{k}_{i}\psi_{i}'\in \mathcal{J}$ for each $1\leq k \leq 4$. Note also that $T'(0)>1$. Finally, $T_{5}$ is implicitly defined on $[\frac{32}{75},1]$ by the equation $\sum^{5}_{i=1}\psi_{i}'=1$. Thus, $T$ is an $a$-convex transformation, and the Lebesgue measure is $T$-invariant. Since $T[\frac{17}{60},\frac{2}{5}]=[0,\frac{7}{8}]$ is not a union of intervals in the partition $\mathcal{P}=\{[a_{j-1},a_{j})\}_{j=1}^{5}$, $\mathcal{P}$ is a non-Markov partition. Furthermore, $x=1$ is an indifferent fixed point because $\sum^{5}_{j=1}\psi'_{j}(1)=0+\psi_{5}'(1)=1$. \end{example} \section{Conformal measure for $-s\log|T'|$} Let $T$ be an $a$-convex transformation satisfying condition $(C)$, $s > 0$, and $h \in C([0,1])$. The \textit{transfer operator} of $T$ with respect to $h$ is defined on $C([0,1])$ by the map \begin{align*} f(x)\mapsto \sum_{y\in T^{-1}x}\exp(h(y))f(y). \end{align*} Notice that $\psi_{i}'(x)= \exp(-\log|T'T_{i}^{-1}x|)\chi_{\overline{T(a_{i-1},a_{i})}}$ for each $1\leq i \leq N$, where $\chi_{\overline{T(a_{i-1},a_{i})}}$ denotes the characteristic function of the closure of $T(a_{i-1},a_{i})$. In this section we construct an extension $\overline{[0,1]}$ of $[0,1]$ as a topological space in which $T'$ and the function $\sum_{i=1}^{k} (\psi_{i}')^{s}$ is continuous for every $1 \leq k \leq N$. Thus, the extended operator $F_{s}$ acts as the transfer operator for the potential $h=-s\log|T'|$ on $\overline{[0,1]}$. We follow the construction presented in \cite{hk1} to achieve this. Define \begin{align*} V_{k}:=\left\{x\in[0,1]:\lim_{y\rightarrow x^{+}} \sum^{k}_{i=1}\psi_{i}'(y)\neq \lim_{y\rightarrow x^{-}}\sum^{k}_{i=1}\psi_{i}'(y)\right\}. \end{align*} Let $V=\left(\bigcup^{N}_{k=1}V_{k}\right)\cup \{0,1\}$. Then, all possible orbits with discontinuities of $\sum^{k}_{i=1}(\psi_{i}')^{s}$ and $T$ generated by the dynamics are contained in the set \begin{align*} W=\bigcup^{\infty}_{i=0}T^{-i}\left(\bigcup^{\infty}_{j=0}T^{j}V\right)\setminus \{0,1\}. \end{align*} Since $V$ is countable by the differentiability condition of $a$-convex transformations, it follows that $W$ is a countable $T$-invariant set. For each $x\in W$, we replace $x$ in $[0,1]$ with $x^{+}$ and $x^{-}$ to define \begin{align*} \overline{[0,1]}=W^{c}\cup \bigcup_{x\in W}\{x^{+}\}\cup \bigcup_{x\in W}\{x^{-}\}. \end{align*} The order relation on $[0,1]$ is extended to $\overline{[0,1]}$ as follows: $ y<x^{-}<x^{+}<z$ whenever $y<x<z$ in $[0,1]$. The space $\overline{[0,1]}$ endowed with the order topology is a compact topological space (\cite[Theorem 12, Chapter X, section 7]{bg}). Since $[0,1]\setminus W$ is dense in $\overline{[0,1]}$, for every $x\in W$ we can define $\sum^{k}_{i=1}(\psi_{i}')(x^{+})$, $\sum^{k}_{i=1}(\psi_{i}')(x^{-})$, $T(x^{+})$, and $T(x^{-})$ continuously on $\overline{[0,1]}$. Indeed, for any $x,y\in W$, we have \begin{align*} [y^{+},x^{-}]=\{z\in \overline{[0,1]}:y^{+}\leq z\leq x^{-}\}=\{z\in \overline{[0,1]}:y^{-}<z<x^{+}\}=(y^{-},x^{+}), \end{align*} which implies that the extensions of $\sum_{i=1}^{k}(\psi_{i}')^{s}$ and $T$ are continuous with respect to the order topology. Thus, the extended $F_{s}$ acts as an operator from $C(\overline{[0,1]})$ to $C(\overline{[0,1]})$. Since $F_{s}$ is linear, for any $f, g \in C(\overline{[0,1]})$, we have \begin{align*} \|F_{s}f-F_{s}g\|_{\infty}\leq K\|f-g\|_{\infty}, \end{align*} where $K = \left\|\sum_{i=1}^{N}(\psi_{i}')^{s}\right\|_{\infty}$ is finite by condition $(C)$. This implies that $F_{s}$ is continuous on $(C(\overline{[0,1]}),\|\cdot\|_{\infty})$. Therefore, the dual operator $F_{s}^{*}$ is continuous on $C^{*}(\overline{[0,1]})$, the dual space of $C(\overline{[0,1]})$. By the Riesz Representation Theorem, $C^{*}(\overline{[0,1]})$ is isomorphic to the Banach space of regular finite measures on $\overline{[0,1]}$ with the weak* topology. Since the space $\mathcal{M}_{1}(\overline{[0,1]})$ of positive Borel probability measures on $\overline{[0,1]}$ is convex and compact in $C^{*}(\overline{[0,1]})$, we apply the Schauder-Tychonoff Theorem to the continuous map \begin{align*} \mu \mapsto \dfrac{F_{s}^{*}\mu}{F_{s}^{*}\mu(\mathds{1})},\quad \mu\in \mathcal{M}_{1}(\overline{[0,1]}). \end{align*} Thus, we obtain a fixed point $m_{s}$. In other words, for every $f \in C(\overline{[0,1]})$, \begin{align}\label{Msmeasure} m_{s}(F_{s}f)=\gamma m_{s}(f), \end{align} where $\gamma = m_{s}(F_{s}\mathds{1})$. Hence, $m_{s}$ is a fixed point of $\gamma^{-1}F^{*}_{s}$. Note that $F_{s}\mathds{1}(1)\leq \gamma \leq F_{s}\mathds{1}(0)$. Let us denote $\overline{F}_{s}=\gamma^{-1}F_{s}$ for simplicity. To deduce properties of $m_{s}$ on $(T, [0,1])$, we require $m_{s}$ to assign zero measure on the set of duplicate points $\bigcup_{x\in W}\{x^{+},x^{-}\}$ (see, for example, Lemma \ref{Lema2HK1}). Define $\mathcal{P} = \{[a_{k-1}^{+}, a_{k}^{-}]\}_{k=1}^{N}$. For each $n \in \mathbb{N}$, the partition $\mathcal{P}^{n} = \bigvee_{i=0}^{n-1} T^{-i} \mathcal{P}$ is the finite partition of $\overline{[0,1]}$ into intervals on which $T^{n}$ is monotone and continuous, where $\bigvee$ denotes the join operator for partitions. \begin{lemma}\cite[Lemma 2]{hk1}\label{Lema2HK1} Suppose there exists $n \in \mathbb{N}$ such that $\|(T^{n})'\|_{\infty} \geq d$ for some $d > 1$. Then for all $A \in \mathcal{P}^{nk}$, we have $m_{s}(\overline{A}) \leq d^{k}$. \end{lemma} Consequently, we deduce that $m_{s}(\{x\}) = 0$ for all $x \in \overline{[0,1]}$ whenever $T$ has an expansive iteration. Since $W$ is countable, $m_{s}$ assigns zero measure to duplicate points and so $m_{s} \in \mathcal{M}_{1}([0,1])$. Bose et al. provide sufficient conditions for $T$ to have some expansive iteration (see \cite[Theorem 4.10]{bmvs} and Remark \ref{SomeIterationExpansive}). To address cases where $T$ includes indifferent fixed points, condition $(B)$ will be sufficient to imply $m_{s}\in \mathcal{M}_{1}([0,1])$ (see Remark \ref{SomeIterationExp2}). \begin{proposition}\cite[Lemma 5, Lemma 6]{hk2}\label{PmarkovOp} Suppose that $m_{s}(\{x\}) = 0$ for every $x \in \overline{[0,1]}$. Then, $\overline{F_{s}}$ is a Markov operator on $L^{1}_{m_{s}}([0,1])$, i.e., for all $f \in L^{1}_{m_{s}}([0,1])$ \begin{enumerate} \item $m_{s}(\overline{F_{s}}f)=m_{s}(f)$. \item $\|\overline{F_{s}}f\|_{L^{1}_{m_{s}}}\leq \|f\|_{L^{1}_{m_{s}}}$. \end{enumerate} \end{proposition} \begin{example}\label{ConvexTransformationsLasotaYorke} Consider the family $\mathcal{C}$ of piecewise monotone convex transformations studied in \cite{ly}. Let $T \in \mathcal{C}$. Since each $T_k$ is convex, it is differentiable except at most in countably many points. Therefore, for each $1 \leq i \leq N$, we have $\psi_i' \in \mathcal{J}$ up to a countable set. This implies that $\sum_{i=1}^k (\psi_i')^s \in \mathcal{J}$ for every $1 \leq k \leq N$ and any $s>0$, which shows that $T$ is an $a$-convex transformation satisfying condition $(C)$ for all $s > 0$. We now show that $T^n$ is expansive for some $n \in \mathbb{N}$. Notice that either $T(\beta) = \beta$ or $\psi_{N^{*}}'(\beta^{-}) = 0$. The second case satisfies our requirement. Suppose $T(\beta) = \beta$. By convexity, we have \begin{align*} 1< \dfrac{\beta}{\beta-a_{N^{*}-1}}\leq T'(\beta^{-})=(\psi_{N^{*}}'(\beta^{-}))^{-1}, \end{align*}since $T_{N^{*}}(a_{N^{*}-1})=0$. Hence, $\psi_{N^{*}}(\beta^{-})<1$. By Remark \ref{SomeIterationExpansive}, there exists $n_0 \in \mathbb{N}$ and $\alpha>1$ such that $(T^{n_0})' > \alpha $. Therefore, $T$ satisfies condition $(B)$ by Lemma \ref{Lema2HK1} and equation \eqref{Msmeasure} applied to $\chi_{I_{(d_{i})_{i=1}^{r}}}$ the characteristic function of any cylinder $I_{(d_{i})_{i=1}^{r}}$. \end{example} \begin{remark}\label{SomeIterationExp2} Now, we discuss why $\psi_{N^{*}}'(\beta) \geq 1$ implies that the point $\beta^{-}$ is indifferent. First, since $\sum^{N^{*}}_{i}\psi_{i}'\in \mathcal{J}$ it implies that $\sum_{i=1}^{N^{*}} \psi_{i}'(x) \geq 1$ for any $x \in [0,\beta]$. Integrating both sides with respect to Lebesgue measure, we obtain for any $x \in [0,\beta]$: \begin{align}\label{QQ} \sum^{N^{*}}_{i=1}(\psi_{i}(x)-a_{i-1})\geq x. \end{align} Hence, we have $\psi_{N^{*}}(\beta) \geq \beta$ when evaluated at $x = \beta$. Since $[0, \beta]$ is forward invariant by equation \eqref{ZeroBetaForwardInvariant} we must have equality on inequality \eqref{QQ}. So $\lim_{x\rightarrow \beta^{-}}T_{N^{*}}(x)=\beta$, and $\sum_{i=1}^{N^{*}} \psi_{i}'(\beta) = 1$. Be warned that we can have that $T$ is not continuous at $\beta$, so $T(\beta^{-})$ may differ from $T(\beta^{+})$. Therefore $\psi_{N^{*}}'(\beta^{-}) = 1$, meaning that $\beta^{-}$ is an indifferent fixed point. Furthermore, by equation \eqref{Perron-Frobenius} $F_{1} \chi_{[0,\beta]} = \chi_{[0,\beta]}$, so $\beta^{-1} \chi_{[0,\beta]}$ is the density of an ACIM for the Lebesgue measure. This argument also implies that $\psi_{N^{*}}'(x) <1$ for all $x \in [a_{N^{*}-1}, \beta)$. Indeed, if there exists an $a_{N^{*}-1} \leq x < \beta$ such that $\psi_{N^{*}}(x) = 1$, then we must have $\psi_{i}'(x) = 0$ for every $i < N^{*}$. In this case, $T_{i}[a_{i-1},a_{i}] \subset [0,x)$, which contradicts the forward invariance of $[0,\beta]$. \end{remark} Recall that $\mathcal{P}$ is said to be a generating partition for $m_{s}$ if $\bigvee_{i=0}^{\infty} T^{-i} \mathcal{P}$ is the partition into points of $\overline{[0,1]}$ up to zero $m_{s}$-measure. Typically, the argument that $\mathcal{P}$ is a generating partition is established by the existence of an expansive iteration $T^{k}$ (Lemma \ref{Lema2HK1}), a property that we do not necessarily have when $\beta^{-}$ is indifferent. \begin{proposition}\label{IKPartition} Suppose $T$ satisfies condition $(B)$ and $(C)$ for some $s > 0$. Then, either $m_{s}\in\{\delta_{\beta^{-}},\delta_{\beta^{+}}\}$, or $\mathcal{P}$ is a generating partition for $m_{s}$. \end{proposition} \begin{proof} Let $r \in \mathbb{N}$ and $(d_{i})_{i=1}^{r} \in \{1, \dots, N\}^{r}$. Equation \eqref{Msmeasure} implies that for every $n \in \mathbb{N}$, \begin{align}\label{Ecu538} m_{s}(I_{(d_{i})_{i=1}^{r}})& =m_{s}(\overline{F_{s}}^{n}(\chi_{I_{(d_{i})_{i=1}^{r}}})). \end{align}Since $T$ satisfies condition $(B)$, for any $I_{(d_{i})_{i=1}^{r}}$ not containing $\beta$, we have \begin{align*} m_{s}(I_{(d_{i})_{i=1}^{r}})\leq M_{r}. \end{align*}Notice that it is enough to consider the case where $\lim_{x \to \beta^{-}} \psi_{N^{*}}(x) = 1$. Since an expansive iteration implies that we can choose $\{M_{r}\}_{r \in \mathbb{N}}$ to decrease exponentially. Hence, we assume that $\beta^{-}$ is an indifferent fixed point. Define $w_{0} = 0$; for every $n \in \mathbb{N}$, define recursively $w_{n+1}$ by the equation $w_{n} = T_{N^{*}}(w_{n+1})$. Since $T' > 1$ over $[a_{N^{*}-1}, \beta)$, the sequence $\{w_{n}\}_{n \in \mathbb{N}}$ is strictly increasing and converges to $\beta$. Let $x \in \overline{[0,1]} \setminus \{\beta^{+}, \beta^{-}\}$. Then, either there exists $n \in \mathbb{N}$ such that $x \leq w_{n}$, or $x > \beta$. Denote by $I_{(d_{i})_{i=1}^{r}}(x)$ to the cylinder of length $r$ containing $x$. If $r \geq n$, then by equation \eqref{Ecu538}, \begin{align*} m_{s}(I_{(d_{i})_{i=1}^{r}}(x)) \leq M_{r}. \end{align*} The result follows whenever $m_{s}\notin \{ \delta_{\beta^{+}},\delta_{\beta^{-}}\}$. \end{proof} Thus, unless $m_{s}\in \{\delta_{\beta^{+}},\delta_{\beta^{-}}\}$, condition $(B)$ implies that $m_{s}(\{x\}) = 0$ for all $x \in \overline{[0,1]}$. We can go further and say that, in case $m_{s}=\delta_{\beta^{+}}$, we have $\psi_{N^{*}+1}'(\beta^{+})=1$ and $\psi_{N^{*}}'(\beta^{-})=1$. This implies that $T'$ is continuous at $\beta$, so $\beta$ is an indifferent fixed point by Remark \ref{SomeIterationExpansive}. Since the set $W$ is countable, we conclude that $m_{s} \in \mathcal{M}_{1}([0,1])$ whenever $\psi_{N^{*}}(\beta^{-})<1$ or $\psi_{N^{*}+1}'(\beta^{+})=1$, giving some light on the assumptions of theorem \ref{PrincipalThmChap52}. \begin{example}\label{ConditionBExamplesInfFixPt} Consider $T$, the $a$-convex transformation from Example \ref{ExampleParabolicAconvex}. Let $w_1 = 1/3$ and $w_0 = 0$. For each $r \in \mathbb{N}$, define $w_{r+1}$ recursively by the equation $w_r = T_2(w_{r+1})$. Since $T_2$ is increasing, for any $x \in [w_r, w_{r+1})$, we have that $T^i x \in [w_{r-i}, w_{r+1-i})$ for all $1 \leq i \leq r$. Thus, the sequence $\{w_r\}_{r \in \mathbb{N}}$ is monotone increasing converging to 1. Moreover, there exist constants $C_3, C_4 > 0$ such that for every $x \in [w_r, w_{r+1})$ (see \cite[Proposicion 5.1.1]{mf}), we have \begin{align}\label{InequalityExampleParabolic} \dfrac{r^{2}}{C_{4}}\leq (T^{r})'(x) \leq \dfrac{r^{2}}{C_{3}}. \end{align} Recall that $T_1'$ is strictly increasing, and $T_2'(1/3^+) = 1/2 = T_1'(0)$. By inequality \eqref{InequalityExampleParabolic}, for any $x \leq w_{r+1}$, we obtain \begin{align*} \dfrac{r^{2}}{C_{4}} \leq (T^{r})'(w_{r+1}^{+}) \leq (T^{r})'(x). \end{align*} Let $r \in \mathbb{N}$ and $I_{(a_i)_{i=1}^r}$ be any cylinder that does not contain 1, i.e., $I_{(a_i)_{i=1}^r} \neq [w_r^+, 1]$. By equation \eqref{Msmeasure}, we have \begin{align*} m_{s}(I_{(a_{i})^{r}_{i=1}})=\int \overline{F}^{r}_{s}\chi_{I_{(a_{i})^{r}_{i=1}})}dm_{s}\leq \sup \||(T^{r})'|^{s}\gamma^{r}\|^{-1}_{\infty}\leq \dfrac{C_{4}}{r^{2s}\gamma{r}}. \end{align*} Recall that, condition $(C)$ implies $1=\sum^{N}_{i=1}(\psi_{i}'(1))^{s}\leq \gamma$. Defining $M_r = \frac{C_4}{r^2}$, we see that $T$ satisfies conditions $(B)$ and $(C)$ for $0 < s \leq 1$ whenever $m_s \neq \delta_1$. It is noteworthy that analogous results can be shown for Example \ref{ExampleParabolicNonMarkovianAconvex}. \end{example} \section{Spectral Decomposition of the operator $\overline{F}_{s}$.} In this section, we study the space of $\overline{F_s}$-invariant functions. For the remainder of the document, we assume that $T$ satisfies hypothesis of Theorem \ref{PrincipalThmChap52}. \begin{definition}\label{BoundedVDefinition} We say that a function $f:[a,b]\rightarrow \R$ has \textit{bounded variation} on $[a,b]$ if there exists $M>0$ such that for any finite set $\{x_0, x_1, \dots, x_n\} \subset [a,b]$, the following inequality holds \begin{align*} \sum^{n}_{k=0}|f(x_{k})-f(x_{k+1})|\leq M. \end{align*} The \textit{variation} of $f$ on $[a,b]$ is defined by \begin{align*} V_{[a,b]}f=\sup\left\{\sum^{n-1}_{k=0}|f(x_{k})-f(x_{k+1})|: \text{$\{x_{0},x_{1},...,x_{n}\}\subset [a,b]$}\right\}. \end{align*} \end{definition} \begin{proposition}\cite[Theorem 2.3.1.]{BG}\label{boundedIneq} If $f$ has bounded variation on $[a,b]$, then for all $x\in [a,b]$ \begin{align*} |f(x)|\leq |f(a)|+V_{[a,b]}f. \end{align*} Furthermore, for any probability measure $\mu$ and for all $x \in [0,1]$, we have \begin{align*} |f(x)|\leq V_{[0,1]}f+\|f\|_{L^{1}_{\mu}}. \end{align*} \end{proposition} Therefore, the set of bounded variation functions, $BV[0,1]$, is a subspace of $L^1_\mu([0,1])$ for any probability measure $\mu$. This motivates us to look for a fixed point of $\overline{F_{s}}$ in the subspace $BV[0,1]$ (see also \cite{ly,BG}). Recall that a real-valued function $g$ is said to be a density for $m_s$ if it integrates to 1. To prove the existence of an ACIM for $m_s$, it is sufficient to take a suitable bounded variation function $f$ such that the iterations $\{\overline{F}_s^n f\}_{n \in \mathbb{N}}$ are contained in $BV[0,1]$, and the sequence $\{V_{[0,1]} \overline{F}_s^n f\}_{n \in \mathbb{N}}$ has an upper bound. Then, by applying Helly's First Theorem (see \cite[Theorem 2.3.9]{BG}), we show that $\{\overline{F}_s^n f\}_{n \in \mathbb{N}}$ is precompact in $L^1_{m_s}([0,1])$. The limit point $g$ of any convergent subsequence will be a fixed point of $\overline{F}_s$. Therefore, equation \eqref{Msmeasure} implies the existence of a $T$-invariant measure, absolutely continuous for $m_s$. \begin{proposition}\cite[Lemma 2.4]{bmvs}\label{VarInequality} For every $f \in \mathcal{J}$, we have that $\overline{F}_s f \in \mathcal{J}$. Furthermore, for each $\alpha$ satisfying $\frac{1}{T'(0)} < \alpha < 1$, there exists a positive constant $b(\alpha)$ such that for all $f \in \mathcal{J}$, \begin{align}\label{LasotaYorkeInequality} V_{[0,1]}\overline{F}_{s}f \leq \alpha V_{[0,1]}f+b \|f\|_{L^{1}_{m_{s}}}. \end{align} \end{proposition} Inequality \eqref{LasotaYorkeInequality} is commonly referred to as a \textit{Lasota-Yorke type inequality}. \begin{remark}\label{PHulseRemark} We now explain the similarities between $a$-convex transformations and attractive $g$-functions considered in \cite{hu}. Let $(\Sigma,\sigma)=(\{1,...,N\}^{\N},\sigma)$ denote the fullshift on $N$ symbols. We write $x=(x_{i})_{i\geq 0}$ for elements in $\Sigma$. For each $1\leq j \leq N$, denote $[j]$ the set of points $x\in \Sigma$ such that $x_{0}=j$, and, for any $x\in \Sigma$ denote $\sigma^{-1}(x)\cap [j]$ by $jx$. We define a partial order on $\Sigma$ given by $x\leq y$ whenever $x_{i}\leq y_{i}$ for all non-negative integers $i$. A function $g:\Sigma \rightarrow \R$ is said to be \textit{attractive} if there exists $\delta>0$ such that for all $x\in \Sigma$, we have $g(x)\geq \delta$, $\sum_{i=k}^{N}g(ix)=1$, and, for each $1\leq k \leq N$, the function $\sum^{N}_{i=k}g(ix)$ is non-decreasing. We obtain an analog condition to $\sum^{k}_{i=1}\psi_{i}'\in \mathcal{J}$ since \begin{align*} \sum^{k}_{i=1}g(ix)=1-\sum^{N}_{i=k+1}g(ix), \end{align*} is non-increasing, where $g(ix)$ plays the role of $\psi_{i}'$. Hulse gave a condiition in \cite[Theorem 2.2]{hu}, which guarantees the uniqueness of $g$-measures (equilibrium states) for attractive $g$-functions. This condition in the context of $a$-convex transformations would be the following: for each $1 \leq k \leq N$, we have \begin{align}\label{HulseCondition} \lim_{n\rightarrow \infty}\overline{F}^{n}_{s}\chi_{[0,a_{k}]}(0)-\overline{F}_{s}^{n}\chi_{[0,a_{i}]}(1)=0. \end{align} Equation \eqref{HulseCondition} is stronger than Condition $(B)$, since $\overline{F}_s^n \chi_{[0, a_i]} \in \mathcal{J}$, but harder to verify. In contrast, condition $(B)$ focuses on how $|(T^n)'|^{-1}$ decreases to zero near indifferent fixed points. Furthermore, attractive $g$-functions require the constant function $\mathds{1}$ to be the eigenfunction of the transfer operator. \end{remark} As a consequence of Proposition \ref{VarInequality} and the fact that $m_{s}$ is conformal, we obtain Theorem \ref{ACIM} following arguments in \cite[Theorem 1.1]{bmvs}. \begin{theorem}\label{ACIM} There exists a $T$-invariant measure $\mu_{s}$, which is absolutely continuous with respect to $m_{s}$. Furthermore, the density $g = \frac{d\mu_{s}}{dm_{s}}$ may be chosen from $\mathcal{J}$. \end{theorem} \begin{lemma}\cite[Lemma 4.4]{bmvs}\label{Lemma4.4} Let $g$ be the $F_{s}$-invariant function given by Theorem \ref{ACIM}, and $A=\int^{\beta}_{0}gdm_{s}$. Define \begin{align*} g_{\beta}(x)=\begin{cases} g(x)/A & \text{if $0\leq x\leq \beta,$}\\ 0 & \text{if $\beta<x\leq 1$.} \end{cases} \end{align*} Then $g_{\beta}$ is $\overline{F}_{s}$-invariant. \end{lemma} For what is left of this section, we will study the non-empty set of $T$-invariant and absolutely continuous measures with respect to $m_{s}$. \begin{lemma}\cite[Lemma 3]{hk1}\label{SupportMS} Suppose $m_{s}\neq \delta_{\beta}$. Let \begin{align*} Y=\bigcap\{F:\text{$F\subseteq [0,1]$ closed with $m_{s}(F)=1$}\} \end{align*} be the support of $m_{s}$. Then $T$ can be modified at a finite number of points to satisfy $T(Y) \subseteq Y$ and $T^{-1}\{x\} \subseteq Y$ for all $x \in Y$ except for finitely many points. Furthermore, $Y$ is a compact ordered set, and every non-trivial interval $J \subseteq Y$ has a non-zero $m_{s}$-measure. \end{lemma} Hence, up to a change on a finite set (zero measure by Proposition \ref{IKPartition}), $Y$ is a $T$-invariant set. We then focus on $(Y, T)$. Similarly to Definition \ref{BoundedVDefinition}, a complex-valued function $f : Y \to \mathbb{C}$ is said to have \textit{bounded variation} if \begin{align*} V_{Y}f=\sup\left\{\sum^{k-1}_{i=1}|f(x_{i})-f(x_{i-1})|: \{x_{1},\cdots ,x_{k}\}\subset Y\right\}<\infty, \end{align*} where we abuse notation and use $|\cdot|$ to denote the norm on the complex numbers. Similar to the preceding section, we obtain $m_{s}$ as a complex measure satisfying equation \eqref{Msmeasure} where $F_{s}$ is an operator from the set of complex-valued continuous functions to itself. Denote by $L^{1}_{m_{s}}(Y, \mathbb{C})$ the set of complex-valued integrable functions over $Y$ with respect to $m_{s}$. We define the \textit{variation} of the equivalence class of $f \in L^{1}_{m_{s}}(Y, \mathbb{C})$ by \begin{align*} v(f)=\inf\{V_{Y}\widehat{f}: \text{$\widehat{f}$ is a version of $f$ in $L^{1}_{m_{s}}(Y,\mathbb{C})$}\}. \end{align*} We denote by $BV_{m_{s}}$ the set of functions with bounded variation in $L^{1}_{m_{s}}(Y, \mathbb{C})$. It is known that $BV_{m_{s}}$ is a linear subspace of $L^{1}_{m_{s}}(Y, \mathbb{C})$, but it is not closed with respect to the norm $\|\cdot\|_{L^{1}_{m_{s}}(Y, \mathbb{C})}$. The following defines a norm for $BV_{m_{s}}$ (see \cite[Remark 1]{hk1}): \begin{align}\label{BV-Norm} \|f\|_{v}=\|f\|_{L^{1}_{m_{s}}(Y,\mathbb{C})}+v(f), && \text{for each $f\in BV_{m_{s}}$.} \end{align} \begin{lemma}\label{ConditionalExp} Let $f \in L^{1}_{m_{s}}(Y, \mathbb{C})$ be such that $v(f) \leq c$ for some $c > 0$. Let $\{M_{r}\}_{r \in \mathbb{N}}$ be the sequence of positive numbers given by condition $(B)$. Then for every $r \in \mathbb{N}$, \begin{align*} \int|f-E_{m_{s}}(f|\mathcal{U}_{r})|dm_{s}\leq c\left(M_{r}+m_{s}(I_{(N^{*})_{i=1}^{r}})\right), \end{align*} where $E_{m_{s}}(f|\mathcal{U}_{r})$ is the conditional expectation of $f$ with respect to the $\sigma$-algebra $\mathcal{U}_{r}$ generated by $\mathcal{P}^{r}$. \end{lemma} \begin{proof} By Lemma \ref{SupportMS}, every non-trivial interval in $Y$ has non-zero measure. Thus \begin{align*} \int |f-E_{m_{s}}(f|\mathcal{U}_{r})|dm_{s} & = \int \left|f-\sum_{I_{(d_{i})_{i=1}^{r}}}\dfrac{ \chi_{I_{(d_{i})_{i=1}^{r}}}}{m_{s}(I_{(d_{i})_{i=1}^{r}})}\int_{I_{(d_{i})_{i=1}^{r}}}fdm_{s}\right|dm_{s}\\ & \leq \sum_{I_{(d_{i})_{i=1}^{r}}}\int_{I_{(d_{i})_{i=1}^{r}}}\left|f-\dfrac{1}{m_{s}(I_{(d_{i})_{i=1}^{r}})}\int_{I_{(d_{i})_{i=1}^{r}}}fdm_{s}\right|dm_{s}, \end{align*} Since $\frac{1}{m_{s}(I_{(d_{i})_{i=1}^{r}})} \int_{I_{(d_{i})_{i=1}^{r}}} f \, dm_{s}$ is the expected value of $f$ in the cylinder $I_{(d_{i})_{i=1}^{r}}$, \begin{align*} \int \left|f-\dfrac{1}{m_{s}(I_{(d_{i})_{i=1}^{r}})}\int_{I_{(d_{i})_{i=1}^{r}}}fdm_{s}\right|dm_{s}& \leq m_{s}(I_{(d_{i})_{i=1}^{r}})V_{I_{(d_{i})_{i=1}^{r}}}(f). \end{align*} By condition $(B)$, we obtain \begin{align*} \int |f-E_{m_{s}}(f|\mathcal{U}_{r})|dm_{s} &\leq M_{r} \sum_{I_{(d_{i})_{i=1}^{k}}\neq I_{(N^{*})_{i=1}^{r}}}V_{I_{(d_{i})_{i=1}^{r}}}(f)+m_{s}(I_{(N^{*})_{i=1}^{r}})V_{I_{(N^{*})_{i=1}^{r}}}\\ &\leq M_{r}V_{[0,1]}(f) +m_{s}(I_{(N^{*})_{i=1}^{r}})V_{I_{(N^{*})_{i=1}^{r}}}. \end{align*} The result follows by taking a suitable version of $f$ and using that $v(f) \leq c$. \end{proof} Lemma \ref{SupportMS}, equation \eqref{BV-Norm}, and Lemma \ref{ConditionalExp} imply the following Proposition, using similar arguments as in \cite[Lemma 5]{hk1}. \begin{proposition}\label{PropoBVBanachSpace} If $m_{s}\neq \delta_{\beta}$, then \begin{enumerate} \item The set $E_{c}=\{f\in L^{1}_{m_{s}}(Y,\mathbb{C}):\|f\|_{v}\leq c\}$ is compact in $L^{1}_{m_{s}}(Y,\mathbb{C})$ for every $c>0$. \item $(BV_{m_{s}},\|\cdot\|_{v})$ is a Banach space. \item $BV_{m_{s}}$ is dense in $L^{1}_{m_{s}}(Y,\mathbb{C})$. \end{enumerate} \end{proposition} Denote $L^{1}_{m_{s}}(Y)$ as the set of real-valued integrable functions over $Y$. Let $E $ be the set of all $\overline{F}_{s}$-invariant non-negative functions in $L^{1}_{m_{s}}(Y)$ which is non-empty by Theorem \ref{ACIM}. Let $E_{1} = E \cap \left\{ f : \|f\|_{L^{1}_{m_{s}}} = 1 \right\}$ be the set of $\overline{F}_{s}$-invariant densities of $m_{s}$. From this point, it is standard to verify that the operator $\overline{F}_{s}:BV_{m_{s}} \rightarrow BV_{m_{s}}$ satisfies all the hypotheses of the Ionescu-Tulcea and Marinescu Theorem \cite{tm} (see \cite[Section 7.1]{BG}) for the Banach spaces $BV_{m_{s}}$ and $L^{1}_{m_{s}}(Y, \mathbb{C})$. Thus, $E \subset BV_{m_{s}}$ is a convex set and is finite-dimensional. Furthermore, $E_{1}$ has a finite number of extreme points with disjoint supports spanning $E$ (see \cite[Proposition 7.2.3]{bg}). We obtain Theorem \ref{SpectralDescomp} following \cite[Chapter 7]{bg}. \begin{theorem}\label{SpectralDescomp} Let $T$ be an $a$-convex transformation satisfying conditions $(B)$ and $(C)$ for a given $s > 0$. Suppose either $\psi_{N^{*}}(\beta^{-})<1$ or $\psi_{N^{*}+1}'(\beta^{+})=1$. Then $T$ has a finite number of $T$-invariant ergodic measures $\{\mu_{1},...,\mu_{n}\}$ absolutely continuous with respect to $m_{s}$. Furthermore, each $\frac{d\mu_{i}}{dm_{s}}\in E_{1}$, and $\{\frac{d\mu_{1}}{dm_{s}},\cdots, \frac{d\mu_{n}}{dm_{s}}\}$ span the space of $\overline{F}_{s}$-invariant densities with bounded variation. \end{theorem} Indeed, it is possible to have more than one ACIM for $m_{s}$ (see \cite[Example 6.1]{bmvs}). \section{Equilibrium states} This section aims to prove that the $T$-invariant measure $\mu_{s}$ from Theorem \ref{ACIM} is an equilibrium state with respect to the potential $-s \log |T'|$ on $Y$, i.e., \begin{align}\label{PressureVariationalPrinciple} h(\mu_{s})-\int s\log |T'|d\mu_{s}=\sup\left\{h(\nu)-\int s\log |T'|d\nu:\text{$\nu$ is $T$-invariant on $Y$}\right\}, \end{align} where $Y\subseteq[0,1]$ is the support of the conformal measure $m_{s}$ from section 4. The value on the right-hand side of equation \eqref{PressureVariationalPrinciple} is also known as the topological pressure of $T$ for the potential $-s \log |T'|$ over $Y$, and we denote it by $P(s)$. \begin{proposition}\cite[Theorem 8.1.2]{bg}\label{Thm8.1.2} Suppose $m_{s} \neq \delta_{\beta}$. If $f$ is an $\overline{F}_{s}$-invariant density of bounded variation in $L^{1}_{m_{s}}(Y)$, then the support of $f$ is open for $m_{s}$-almost every point. \end{proposition} Let $g$ be the $\overline{F}_{s}$-invariant density from Theorem \ref{ACIM}. Recall that $g_{\beta} = (g / A) \chi_{[0, \beta]}$ is $\overline{F}_{s}$-invariant, where $A = \int_0^{\beta} g \, dm_{s}$ by Lemma \ref{Lemma4.4}. \begin{proposition}\label{PositiveDensity} Suppose $m_{s} \neq \delta_{\beta}$ and $\psi_{j}'(\beta)>0$ for some $1\leq j < N$. Then $g_{\beta}$ is the only $\overline{F}_{s}$-invariant density whose support is $[0, \beta]$ and $g_{\beta}|_{[0,\beta]} \geq c$ for some $c > 0$. \end{proposition} \begin{proof} Recall that $g_{\beta}\in BV_{m_{s}}$ since $g_{\beta} \in \mathcal{J}$. By Theorem \ref{SpectralDescomp}, we have, for some constants $\alpha_{1},...,\alpha_{n}$, that \begin{align*} g_{\beta}=\sum^{n}_{i=1}\alpha_{i}\dfrac{d\mu_{i}}{dm_{s}}. \end{align*} The support of each $\frac{d\mu_{i}}{dm_{s}}$ is an open set for $m_{s}$-almost every point by Proposition \ref{Thm8.1.2}. There exists $x > 0$ and $1 \leq q \leq k$ such that $(0, x)$ supports $\frac{d\mu_{q}}{dm_{s}}$. Then expansiveness on $(0, a_{1})$ implies that $(0, \beta) \subseteq T^{r}(0, x)$ for some $r \in \mathbb{N}$ (see Remark \ref{SomeIterationExpansive}). Therefore, $(0, \beta)$ must be in the support $\frac{d\mu_{q}}{dm_{s}}$. Since the $\overline{F}_{s}$-invariant density functions from Theorem \ref{SpectralDescomp} have disjoint supports (as the respective measures are ergodic), then $g_{\beta} = \frac{d\mu_{q}}{dm_{s}}$, and $(T, g_{\beta} m_{s})$ is ergodic. Recall $g_{\beta} $ attains its minimum at $g_{\beta}(\beta)$. On the other hand, by $\overline{F}_{s}$-invariance, we have \begin{align*} g_{\beta}(\beta)=\dfrac{1}{A\gamma}\sum^{N}_{i=1}(g\circ \psi_{i}(\beta))(\psi_{i}'(\beta))^{s}. \end{align*} Now suppose that $\lim_{x \to \beta^{-}} g_{\beta}(x) = 0$. Then, \begin{align*} \lim_{x\rightarrow \beta^{-}}g_{\beta}(x)\geq \lim_{x\rightarrow \beta^{-}}\dfrac{(g\circ \psi_{j}(x))(\psi_{j}'(x))^{s}}{A\gamma}=0. \end{align*} Thus, by our hypothesis $g_{\beta}(a_{j}) = 0$ since $\lim_{x \to \beta^{-}} g \circ \psi_{j}(x) = g(a_{j})$. This implies that the support of $g_{\beta}$ is in $[0, a_{j}]$. Let $\mu_{s}=g_{\beta}m_{s}$. Therefore, $\mu_{s}((a_{j}, \beta)) = 0$. On the other hand, for any $x \in (0, a_{1}]$, there exists $N \in \mathbb{N}$ and $1\leq i \leq j$ such that for each $k\geq N$ we have $T_{i}^{-1}(a_{j}) \leq T^{k}(x)$ by Remark \ref{SomeIterationExpansive}. Thus, $0 = \mu_{s}(T^{-k+1} [a_{j}, \beta)) \geq \mu_{s}((x, \beta))$ by $T$-invariance. Since $x$ is arbitrary, we obtain $\mu_{s}((0, \beta)) = 0$. Therefore, $g_{\beta} m_{s} = \delta_{0}$, where $\delta_{0}$ is the Dirac delta measure at $x = 0$. This leads to a contradiction, since $m_{s}$ assigns zero measure on singleton sets by Proposition \ref{IKPartition}. \end{proof} By proposition \ref{PositiveDensity}, the function $\overline{g}=|T'|^{-s}\dfrac{g_{\beta}}{ g_{\beta}\circ T}$ is well defined whenever $\psi_{j}'(\beta)>0$ for some $1\leq j < N$. Also, define $G_{s}:C(Y)\rightarrow C(Y)$ by \begin{align}\label{Ecu5413} G_{s}(f)=\dfrac{1}{\gamma}\sum^{N}_{i=1}(\overline{g}f)\circ \psi_{i}=\dfrac{1}{g_{\beta}\gamma}\sum^{N}_{i=1}(\psi_{i}')^{s}(g_{\beta}f\circ \psi_{i}), \end{align} where $\gamma= m_{s}(F_{s}\mathds{1})$ (see equation \eqref{Msmeasure}). \begin{lemma}\cite[Lemma 15]{hk1}\label{Lemma 15} Suppose $\psi_{j}'(\beta)>0$ for some $1\leq j < N$. If $\mu$ is a $T$-invariant probability measure on $Y$, then \begin{align*} \int \log(\overline{g})d\mu-\log(\gamma) =-s\int \log|T'|d\mu. \end{align*} \end{lemma} Hence, for any $T$-invariant measure $\mu$ on $Y$, we have \begin{align}\label{ASTERISCO} h_{\mu}+\int \log \overline{g}d\mu =h_{\mu}-s\int \log|T'|d\mu-\log(\gamma). \end{align}Thus, $T$-invariant measures that attain the supremum in equation \eqref{ASTERISCO} are equilibrium states for both $\overline{g}$ and $-s \log |T'|$. \begin{theorem}\cite[Theorem 2.1]{wp}\label{Ledrapierthm} Suppose $\psi_{j}'(\beta)>0$ for some $1\leq j < N$. Let $\mu$ be a Borel probability measure on $Y$. The following statements are equivalent: \begin{enumerate} \item The dual operator $G^{*}_{s}:C^{*}(Y)\rightarrow C^{*}(Y)$ has $\mu$ as a fixed point. \item $\mu$ is a $T$-invariant measure on $Y$ and is an equilibrium state for $\log(\overline{g})$. \end{enumerate} \end{theorem} \textit{Proof of Theorem \ref{PrincipalThmChap52}} First, assume $\psi_{i}'(\beta)=0$ for all $1\leq i < N$. This implies that some iteration is expansive, so by \cite[Theorem 6]{hk1} $\mu_{s}$ is an equilibrium state for $-s\log|T'|$. Suppose now $\psi_{j}'(\beta)>0$ for some $1\leq j < N$. Recall that by Proposition \ref{IKPartition} $\psi_{N^{*}}(\beta^{-})<1$ or $\psi_{N^{*}+1}'(\beta^{+})=1$ implies $m_{s}\in \mathcal{M}_{1}([0,1])$. Since $\mu_{s} = g_{\beta} m_{s}$ and $\overline{F}^{*}_{s} m_{s} = m_{s}$, we obtain for every $f \in C(Y)$ \begin{align*} \mu_{s}(G_{s}f)=\int \frac{1}{\gamma}\sum^{N}_{i=1}(\psi_{i}')^{s}g_{\beta}f\circ \psi_{i}dm_{s}=\int \overline{F}_{s}(g_{\beta}f)dm_{s}=\int fg_{\beta}dm_{s}=\mu_{s}(f). \end{align*}Therefore, $\mu_{s}$ is an equilibrium state of $-s \log |T'|$, according to Theorem \ref{Ledrapierthm}. By Proposition \ref{PositiveDensity}, the uniqueness follows when $\beta = 1$ and $m_{s}\neq \delta_{\beta}$. \qedsymbol \begin{example} Recall that we showed that transformations in Examples \ref{ExampleParabolicAconvex} and \ref{ExampleParabolicNonMarkovianAconvex} satisfy condition $(B)$ for each $0 < s < 1$ such that $m_{s} \neq \delta_{1}$, and condition $(C)$ for all $0 < s \leq 1$ (Example \ref{ConditionBExamplesInfFixPt}). We now show that the function $s \mapsto P(s)$ is non-increasing in $(0, \infty)$, constant and equal to zero for $s \geq 1$, and differentiable except for at most countably many points. Using equation \eqref{PressureVariationalPrinciple}, we deduce that $s \mapsto P(s)$ is non-increasing. Moreover, since $|T' x| = 1$ if and only if $x = 1$, we have for every $\varepsilon > 0$ that $P(s + \varepsilon) = P(s)$ if and only if for each $t\in (s,s+\varepsilon)$ an equilibrium state of $-t \log |T'|$ is $\delta_{1}$. However, for $s = 1$, there is an equilibrium state different from $\delta_{1}$, which is the ACIM for the Lebesgue measure ( \cite[Theorem 1.1]{bmvs}). The proof of Theorem \ref{Ledrapierthm} given in \cite[Theorem 2.1]{wp} shows that an equilibrium state $\mu$ of $\log(\overline{g})$ satisfies\begin{align*} h_{\mu} + \int \log(\overline{g}) \, d\mu = 0. \end{align*} Since $\gamma= m_{1}(F_{1} \mathds{1}) = 1$, we conclude that $P(1) = 0$ by Lemma \ref{Lemma 15}. Finally, the pressure function is differentiable except for, at most, countably many points since it is a convex function (\cite[Theorem 3.6.2]{pu}). We can also deduce the existence of equilibrium states in Example \ref{ExampleParabolicAconvex} by inducing and obtaining the exact description of the pressure function. However, we will not develop this point here (see \cite{sa, mf}). Nevertheless, our method allows us to study the thermodynamic formalism of $a$-convex transformations with non-Markov partitions, such as in Example \ref{ExampleParabolicNonMarkovianAconvex}, which, to our knowledge, cannot be achieved by inducing. \end{example} \section*{Acknowledments}I want to thank my Ph.D. advisor, Godofredo Iommi, for suggesting this problem, his patience, and numerous helpful remarks. This research was supported by ANID Doctorado Nacional 21210037. \input{Bibliography} \end{document} \begin{thebibliography}{CHMW} \bibitem[An]{an} Arevalo, N. \emph{The Lyapunov spectrum as the Newton-Raphson method for countable Markov interval maps.} Journal of Mathematical Analysis and Applications. 534 (2024), no. 2, 128091. \bibitem[BG]{BG} Boyarsky, A. and Gòra, P. \emph{Laws of chaos: invariant measures and dynamical systems in one dimension}. Springer Science \& Business Media (2012). \bibitem[Bg]{bg} Birkhoff, G. \emph{Lattice theory.}American Mathematical Soc. 25 (1940). \bibitem[BMVS]{bmvs} Bose, C.; Maume-Deschamps, V.; Schmitt, B. and Shin, S. \emph{Invariant measures for piecewise convex transformations of an interval.} 152 (2002), no.3, 263--297. \bibitem[DS]{ds} Dunford, N. and Schwartz, J. \emph{Linear operators, part 1: general theory.} John Wiley \& Sons. 10 (1988). \bibitem[HK1]{hk1} Hofbauer, F. and Keller, G. \emph{Ergodic properties of invariant measures for piecewise monotonic transformations.} The Theory of Chaotic Attractors. 1982. 120--141. \bibitem[HK2]{hk2} Hofbauer, F. and Keller, G. \emph{Equilibrium states for piecewise monotonic transformations.} Ergodic Theory and Dynamical Systems. 2 (1982), no.1, 23--43. \bibitem[Hu]{hu} Hulse, P. \emph{Uniqueness and ergodic properties of attractive g-measures.} 11 (1991), no.1, 65--77. \bibitem[Io1]{io} Iommi, G. \emph{Multifractal analysis of the Lyapunov exponent for the backward continued fraction map.} Ergodic Theory and Dynamical Systems. 30 (2010). no. 1, 211--232. \bibitem[LY]{ly} Lasota, A and Yorke, J. \emph{Exact dynamical systems and the Frobenius-Perron operator.} Transactions of the American Mathematical Society. 273 (1982), no. 1, 375--384. \bibitem[Le]{le} Ledrappier, F. \emph{Principe variationnel et systemes dynamiques symboliques.} Zeitschrift f{\"u}r Wahrscheinlichkeitstheorie und Verwandte Gebiete. 30 (1974), 185--202. \bibitem[Mf]{mf} Monardes, F. \emph{Formalismo Termodinámico para Dinámicas con Puntos Fijos Indiferentes.} 2023. Master’s thesis, Pontifical Catholic University of Chile. \bibitem[PU]{pu} Przytycki, F. and Urba{\'n}ski, M. \emph{Conformal fractals: ergodic theory methods.} Cambridge University Press. 371 (2010). \bibitem[PM]{pm} Pomeau, Y. and Manneville, P. \emph{Intermittent transition to turbulence in dissipative dynamical systems}. Communications in Mathematical Physics. 74 (1980), no. 2, 189-197. \bibitem[Sa]{sa} Sarig, O. \emph{Phase transitions for countable Markov shifts.} Communications in Mathematical Physics. 217 (2001), 555--577. \bibitem[Sh]{sh}Schaefer, H. \emph{Locally convex topological vector spaces.} Springer. 1971. \bibitem[TM]{tm} Tulcea, CT I. and Marinescu, G. \emph{Th{\'e}orie ergodique pour des classes d'op{\'e}rations non compl{\`e}tement continues.} Annals of Mathematics. 1950, 140--147. \bibitem[Wp]{wp} Walters, P. \emph{Ruelle’s operator theorem and $g$-measures.} Transactions of the American Mathematical Society. 214 (1975), 375--387. \bibitem[We]{we} Weiss, H. \emph{The Lyapunov spectrum for conformal expanding maps and axiom-A surface diffeomorphisms.} Journal of statistical physics. 95 (1999), no. 3, 615--632. \end{thebibliography}
2412.06406v1
http://arxiv.org/abs/2412.06406v1
Invariant Probability Measures under $p$-adic Transformations
\documentclass{amsart} \usepackage[nobreak]{cite} \usepackage[utf8]{inputenc} \usepackage[T1]{fontenc} \usepackage{hyperref,amssymb,mdwlist,microtype,mathtools,enumitem,color,soul} \usepackage[capitalize,nameinlink]{cleveref} \allowdisplaybreaks \newtheorem{theorem}{Theorem}[section] \newtheorem{corollary}[theorem]{Corollary} \newtheorem{lemma}[theorem]{Lemma} \newtheorem{prop}[theorem]{Proposition} \theoremstyle{definition} \newtheorem{definitionn}[theorem]{Definition} \newtheorem{problem}[theorem]{Problem} \theoremstyle{remark} \newtheorem{remark}[theorem]{Remark} \newtheorem{example}[theorem]{Example} \crefname{equation}{}{} \Crefname{enumi}{}{} \DeclareMathOperator{\id}{id} \DeclareMathOperator{\card}{card} \DeclareMathOperator{\B}{B} \DeclareMathOperator{\T}{T} \newcommand{\BB}{\mathbb{B}} \newcommand{\Borel}{\mathcal{B}} \newcommand{\FF}{\mathcal{F}} \newcommand{\FP}{\mathcal{P}} \newcommand{\FI}{\mathcal{I}} \newcommand{\FN}{\mathcal{N}} \newcommand{\FPS}{\mathcal{P}_{S}} \newcommand{\FPSp}{\mathcal{P}_{S_p}} \newcommand{\FPSdwa}{\mathcal{P}_{S_2}} \newcommand{\FM}{\mathcal{M}} \newcommand{\FMS}{\mathcal{M}_{S}} \newcommand{\FMSp}{\mathcal{M}_{S_p}} \newcommand{\abs}[1]{\left\lvert#1\right\rvert} \newcommand{\norm}[1]{\left\lVert#1\right\rVert} \newcommand{\clopen}[2]{\char"5B #1,#2\char"29}\newcommand{\opencl}[2]{\char"28 #1,#2\char"5D} \renewcommand{\theenumi}{\roman{enumi}} \renewcommand{\labelenumi}{{\rm (\theenumi)}} \renewcommand{\subjclassname}{Mathematics Subject Classification (2020)} eld}[1]{\mathbb{#1}} \makeatletter \def\cref@thmoptarg[#1]#2#3#4{ \normalfont \trivlist \let\thmheadnl\relax \let\thm@swap\@gobble \thm@notefont{\fontseries\mddefault\upshape} \thm@headpunct{.} \thm@headsep 5\p@ plus\p@ minus\p@\relax \thm@space@setup #2 \@topsep \thm@preskip \@topsepadd \thm@postskip \def\@tempa{#3}\ifx\@empty\@tempa \def\@tempa{\@oparg{\@begintheorem{#4}{}}[]} \else \refstepcounter[#1]{#3} \@namedef{cref@#3@alias}{#1} \def\@tempa{\@oparg{\@begintheorem{#4}{\csname the#3\endcsname}}[]} \@tempa}\makeatother \crefname{theorem}{Theorem}{Theorems} \crefname{lemma}{Lemma}{Lemmas} \crefname{proposition}{Proposition}{Propositions} \crefname{prop}{Proposition}{Propositions} \crefname{corollary}{Corollary}{Corollaries} \crefname{definition}{Definition}{Definitions} \begin{document} \title{Invariant probability measures under $P$-adic transformations} \author[J.~Morawiec]{Janusz Morawiec} \address{Institute of Mathematics, University of Silesia, Bankowa 14, PL-40-007 Katowice, Poland} \email{[email protected]} \author[T.~Zürcher]{Thomas Zürcher} \address{Institute of Mathematics, University of Silesia, Bankowa 14, PL-40-007 Katowice, Poland} \email{[email protected]} \subjclass{Primary 37E05, 39B12; Secondary 26A30, 28D05} \keywords{invariant measures, iterative functional equations, singular functions, jump functions, beta-transformations} \begin{abstract} It is well-known that the Lebesgue measure is the unique absolutely continuous invariant probability measure under the $p$-adic transformation. The purpose of this paper is to characterize the family of all invariant probability measures under the $p$-adic transformation and to provide some description of them. In particular, we describe the subfamily of all atomic invariant measures under the $p$-adic transformation as well as the subfamily of all continuous and singular invariant probability measures under the $p$-adic transformation. Iterative functional equations play the base role in our considerations. \end{abstract} \maketitle \renewcommand{\theequation}{1.\arabic{equation}}\setcounter{equation}{0} \section{Introduction}\label{S1} The dyadic transformation is an example of the simplest deterministic dynamical system that has so-called chaotic dynamics. It is a special case of the so called $\beta$-transformation, which has been studied by many authors during the last 70 years. The $\beta$-transformation was introduced in~\cite{Renyi1957}, where it was proved that there exists exactly one probability measure that is absolutely continuous (with respect to the Lebesgue measure) and invariant under the $\beta$-transformation. An explicit formula for this invariant probability measure was obtained in~\cite{Gelfond1959} and, independently, in~\cite{Parry1960}, where it was also proved that the $\beta$-transformation is weakly mixing, and hence ergodic. Exactness of the $\beta$-transformation was demonstrated in~\cite{Rohlin1961}. Additional information on the $\beta$-transformation with extensive literature and broadly discussed topics connected with it can be found in~\cite{Vapstas2024}. Results about the existence of the unique absolutely continuous invariant probability measure under the $\beta$-transformation, on its explicit formula and ergodicity behaviour have been motivation for many authors to extend them for more general transformations but still connected with the original $\beta$-transformations (see e.g.\ \cite{TanakaIto1982, DajaniKraaikamp2003, Gora2007, DajaniVries2007, DajaniKalle2010, Kempton2014, KalleMaggioni2022, Suzuki2024}). All the results on the existence of absolutely continuous invariant probability measures are based on a method introduced in~\cite{LasotaYork1973} for deterministic dynamical systems. These methods were later effectively extended to random dynamical systems (see e.g.\ \cite{Buzzi2000, GoraBoyarski2003, BahsounGora2005, Inoue2012}). In many situations, there are also continuous and singular invariant measures or atomic invariant probability measures among the absolutely continuous invariant probability measures under a given transformation. Moreover, the family of all invariant probability measures under a given transformation can be quite large, however among them, there exists only one invariant probability measure that is absolutely continuous. An example of such a situation is the $p$-adic transformation, which we will consider in this paper. The problem with determining invariant probability measures under a given transformation that are not absolutely continuous is that there is no method for this purpose. To the best of our knowledge, one can get some information on singular invariant probability measures considering the iterated function system associated with the given transformation (if it is even possible) and to show that its attractor is of Lebesgue measure zero. However, by this way we may obtain only very specific singular invariant probability measures. The purpose of this paper is to describe the family of all invariant probability measures under the $p$-adic transformation. As we know that the family has exactly one absolutely continuous measure (which is exactly the Lebesgue measure) it suffices to determine its subfamily of all atomic measures and its subfamily of all continuous and singular measures. Let us note that large families of continuous and singular invariant probability measures under the dyadic transformation were determined in~\cite{MorawiecZurcher2018, MorawiecZurcher2021, MorawiecZurcher2024}. Our main tool in this paper is the functional equation associated with the probability distribution functions of invariant measures that is slightly different from that which arises considering density functions of invariant measures (see e.g.~\cite{Suzuki2019}). The main part of this paper is~\cref{S6}, in which we describe the structure of atomic invariant probability measures under the $p$-adic transformation that are supported on minimal finite orbits. As noted in the introduction of~\cite{DownarowiczHuczek2022} such a description can help in progresses of solving the Furstenberg's $\times 2,\times 3$ conjecture, posed in~\cite{Furstenberg1967}, that is one of the major unsolved problems in the field of ergodic theory in dynamical systems. \renewcommand{\theequation}{2.\arabic{equation}}\setcounter{equation}{0} \section{Preliminaries}\label{S2} Denote by $\Borel$ the $\sigma$-algebra of all Borel subsets of $\clopen{0}{1}$ and by $\FF$ the family of all $\Borel$-measurable maps $\Phi\colon\clopen{0}{1}\to\clopen{0}{1}$. Any measure defined on $\Borel$ is called a \emph{Borel measure} and any $\Phi\in\FF$ is called a \emph{Borel transformation}. A Borel probability measure $\mu$ is said to be \emph{invariant under $\Phi\in\FF$} if \begin{equation*} \mu(\Phi^{-1}(B))=\mu(B)\quad\text{for every }B\in\Borel. \end{equation*} Throughout this paper, we fix an integer number $p\geq 2$ and consider the \emph{$p$-adic transformation} $S_p\in\FF$ defined by \begin{equation*} S_p(x)=px\,(\hspace*{-2ex}\mod 1). \end{equation*} The aim of this paper is to characterize the set of all invariant measures under $S_p$ and provide some description of them. However, we will formulate some results for more general transformations than $S_p$. For this purpose, we fix numbers $a_1,\ldots,a_{p+1}\in[0,1]$ such that $0=a_1<a_2<\cdots<a_{p+1}=1$. Next, for every $k\in\{1,\ldots,p\}$ we fix an increasing bijection $f_k\colon[a_k,a_{k+1}]\to[0,1]$ and consider $S\in\FF$ defined by \begin{equation*} S(x)=f_k(x)\quad\text{ for }x\in\clopen{a_k}{a_{k+1}}\text{ with }k\in\{1,\ldots,p\}. \end{equation*} The definition of $S$ does not require defining the functions $f_1,\ldots,f_p$ on the right endpoints of their domains, however it will be important in~\cref{S3} to simplify writing. Note that in the case where $S$ is the $p$\nobreakdash-adic transformation~$S_p$, we have $a_{k+1}-a_k=\frac{1}{p}$ and $f_k(x)=px-k+1$ for every $k\in\{1,\ldots,p\}$. Denote by $\FM$ the family of all Borel probability measures and put \begin{equation*} \FMS=\big\{\mu\in\FM\,\big|\mu(S^{-1}(B))=\mu(B)\text{ for every }B\in\Borel\big\}. \end{equation*} We begin with two simple observations on $\FMS$. \begin{remark}\label{rem:Continuity} Every $\mu\in\FMS$ vanishes on each point of the set $\{a_2,\ldots,a_{p}\}$. In particular, if $\mu\in\FMSp$, then $\mu(\{\frac{k}{p}\})=0$ for every $k\in\{1,\ldots,p-1\}$. \end{remark} Denote by $\delta_x$ the Dirac measure concentrated at the point $x\in\mathbb R$. \begin{remark}\label{rem:FixedPoints} Assume that $x\in[0,1]$. Then $\delta_x\in\FMS$ if and only if $S(x)=x$. In particular, $\delta_x\in\FMSp$ if and only if $x\in\{\frac{k}{p-1}\,|\,k\in\{0,\ldots,p-2\}\}$. \end{remark} \renewcommand{\theequation}{3.\arabic{equation}}\setcounter{equation}{0} \section{Invariant measures vs.\ functional equations}\label{S3} Put \begin{align*} \FI&=\left\{\phi\colon[0,1]\to[0,1]\,\big|\, \phi \text{ is non-decreasing, }\phi(0)=0,\text{ and } \phi(1)=1\right\},\\ \FP&=\left\{\phi\in\FI\,|\,\phi \text{ is left-continuous}\right\}. \end{align*} Recall (cf.\ e.g.~\cite[Theorem~12.4]{Billingsley1995} or~\cite[9.1.1]{Dudley2002} and note that left- and right-continuous functions correspond to each other) that $\FP$ and $\FM$ are with one-to-one correspondence by the formula \begin{equation}\label{mu-phi} \phi(x)=\mu(\clopen{0}{x})\quad\text{ for every }x\in[0,1]. \end{equation} \begin{prop}\label{prop:FunctionalEquation} Assume that $\mu\in\FM$ and let $\phi\in\FP$ correspond to $\mu$ by~\cref{mu-phi}. Then $\mu\in\FMS$ if and only if \begin{equation}\label{FE} \phi(x)=\sum_{k=1}^{p}\left[\phi(f_k^{-1}(x))-\phi(f_k^{-1}(0))\right]\quad\text{for every }x\in[0,1]. \end{equation} \end{prop} \begin{proof} ($\Rightarrow$) Fix $\mu\in\FMS$ and $x\in[0,1]$. Applying~\cref{mu-phi}, we obtain \begin{align*} \phi(x)&=\mu(\clopen{0}{x})=\mu\left(S^{-1}(\clopen{0}{x})\right) =\sum_{k=1}^{p}\mu\left(\left[a_k,f_k^{-1}(x)\right)\right)\\ &=\sum_{k=1}^{p}\left[\mu\left(\left[0,f_k^{-1}(x)\right)\right) -\mu\left([0,a_k)\right)\right] =\sum_{k=1}^{p}\left[\phi\left(f_k^{-1}(x)\right)-\phi(a_k)\right]\\ &=\sum_{k=1}^{p}\left[\phi(f_k^{-1}(x))-\phi(f_k^{-1}(0))\right]. \end{align*} ($\Leftarrow$) Fix $\phi\in\mathcal P$ satisfying \cref{FE}. Applying \cref{mu-phi}, for every $y\in[0,1)$, we get \begin{equation*} \mu(S^{-1}([0,y)))=\mu([0,y)). \end{equation*} Hence for all $x,y\in(0,1)$ with $x<y$ we have \begin{align*} \mu(S^{-1}(\clopen{x}{y}))&=\mu(S^{-1}([0,y)))-\mu(S^{-1}([0,x)))=\mu([0,y))-\mu([0,x))\\ &=\mu(\clopen{x}{y}). \end{align*} Now, it suffices apply \cite[Theorem~3.3]{Billingsley1995}. \end{proof} In the case of $p$-adic transformation \cref{FE} takes the form \begin{equation}\label{FESp} \phi(x)=\sum_{k=0}^{p-1}\left[\phi\left(\frac{x+k}{p}\right) -\phi\left(\frac{k}{p}\right)\right]\quad\text{for every }x\in[0,1]. \end{equation} Put \begin{equation*} \FPS=\left\{\phi\in\FP\,\big|\,\phi\text{ satisfies }\cref{FE}\right\}. \end{equation*} By \cref{prop:FunctionalEquation} each $\phi\in\FPS$ determines exactly one $\mu\in\FMS$ and each $\mu\in\FMS$ is determined by some $\phi\in\FPS$. Therefore, we see that for describing $\FMS$, it suffices to describe $\FPS$. Clearly, any $\phi\in\FPS$ can be decomposed in a canonical way into the absolutely continuous part, the continuous and singular part, and the jump part, i.e.\ the function defined by $\phi_j(x)=\sum_{y\leq x}[\lim_{z\to y+}\phi(z)-\phi(y)]$ for every $x\in[0,1]$ (see~\cite[page~58]{LebesgueNew}, cf.~\cite[page~139]{AmbrosioFuscoPallara2000}). Note that by a jump function we understand any $\phi\in\FPS$ for which the measure $\mu\in\FMS$ that corresponds to $\phi$ by~\cref{mu-phi} is atomic (or discrete), i.e.\ there are a countable set $\{x_j\,|\,j\in J\}\subset[0,1)$ and a sequence $(\alpha_j)_{j\in J}$ with $\sum_{j\in J}\alpha_j=1$ such that $\mu=\sum_{j\in J}\alpha_j\delta_{x_j}$. We say that the transformation $\Phi\in\FF$ is \emph{nonsingular} if $\Phi^{-1}(B)$ is of measure zero whenever $B\in\Borel$ is of measure zero (cf.~\cite[Definition~3.2.2]{LasotaMackey1994}). Note that each $p$\nobreakdash-adic transformation is nonsingular. \begin{prop}\label{prop:ASJ} Assume that $S$~and~$S^{-1}$ are nonsingular. If $\phi\in\FPS$, then its absolutely continuous, continuous and singular, and jump parts satisfy \cref{FE}. \end{prop} \begin{proof} Fix $\phi\in\FPS$. By the canonical decomposition (see \cite[Chapter 16, Section E]{Jones1993}, cf.~\cite[Theorem 7.1.45]{KannanKrugerKing1996}) there exist exactly one (non-decreasing) absolutely continuous function $\phi_a$, exactly one (non-decreasing) continuous and singular function $\phi_s$, and exactly one (non-decreasing) jump function $\phi_j$ such that $\phi_a(0)=\phi_s(0)=0$ and \begin{align*} \phi_a(x)+\phi_s(x)+\phi_j(x)&=\phi(x)=\sum_{k=1}^{p}\left[\phi(f_k^{-1}(x))-\phi(f_k^{-1}(0))\right]\\ &=\sum_{k=1}^{p}\phi_a(f_k^{-1}(x))+\sum_{k=1}^{p}\phi_s(f_k^{-1}(x))\\ &\quad+\sum_{k=1}^{p}\phi_j(f_k^{-1}(x))-\sum_{k=1}^{p}\phi(f_k^{-1}(0)) \end{align*} for every $x\in[0,1]$. To see that $\sum_{k=1}^{p}\phi_a\circ f_k^{-1}$ is absolutely continuous, one can apply the Banach-Zarecki theorem (see \cite[Theorem~7.14]{BrucknerBrucknerThomson} or \cite[Theorem 7.1.38]{KannanKrugerKing1996}). It is easy to check that the function $\sum_{k=1}^{p}\phi_s\circ f_k^{-1}$ is continuously singular and $\sum_{k=1}^{p}\phi_j\circ f_k^{-1}$ is a jump function. Then from the uniqueness in the canonical decomposition there exists a constant $c\in\mathbb R$ such that for every $x\in[0,1]$ we have \begin{equation*} \phi_a(x)=\sum_{k=1}^{p}\phi_a(f_k^{-1}(x))-\sum_{k=1}^{p}\phi(f_k^{-1}(0))+c \end{equation*} and since $0=\phi_a(0)=\sum_{k=1}^{p}\phi_a(f_k^{-1}(0))-\sum_{k=1}^{p}\phi(f_k^{-1}(0))+c$ we see that $\phi_a$ satisfies \cref{FE} and \begin{equation*} \phi_s(x)+\phi_j(x)=\sum_{k=1}^{p}\phi_s(f_k^{-1}(x)) +\sum_{k=1}^{p}\phi_j(f_k^{-1}(x))-c. \end{equation*} Again by the canonical decomposition there exists a constant $d\in\mathbb R$ such that for every $x\in[0,1]$ we have \begin{equation*} \phi_s(x)=\sum_{k=1}^{p}\phi_s(f_k^{-1}(x))-c+d \end{equation*} and since $0=\phi_s(0)=\sum_{k=1}^{p}\phi_s(f_k^{-1}(0))-c+d$ we see that $\phi_s$ satisfies \cref{FE} and \begin{equation*} \phi_j(x)=\sum_{k=1}^{p}\phi_j(f_k^{-1}(x))-d. \end{equation*} Finally, since $0=\phi(0)=\phi_a(0)+\phi_s(0)+\phi_j(0)=\phi_j(0)$ we conclude that $0=\phi_j(0)=\sum_{k=1}^{p}\phi_j(f_k^{-1}(0))-d$, and hence $\phi_j$ satisfies \cref{FE}. \end{proof} From \Cref{prop:ASJ} we see that for describing the family $\FPS$ it suffices to describe each of its three subfamilies: \begin{align*} \FPS^a&=\{\phi\in\FPS\,|\,\phi\text{ is an absolutely continuous function}\},\\ \FPS^s&=\{\phi\in\FPS\,|\,\phi\text{ is a continuous and singular function}\},\\ \FPS^j&=\{\phi\in\FPS\,|\,\phi\text{ is a jump function}\}.\\ \end{align*} \renewcommand{\theequation}{4.\arabic{equation}}\setcounter{equation}{0} \section{The family \texorpdfstring{$\mathcal P_S$}{Pₛ}}\label{S4} Although we know that it is sufficient to describe the families $\FPS^a$, $\FPS^s$, and $\FPS^j$, we will start with a general description of the family $\FPS$. In this section, the main tool are Banach limits. Recall that a Banach limit $B\colon l^\infty\to\mathbb R$ is a linear and continuous extension of the functional $\lim\colon c \to\mathbb R$, where $l^\infty$ is the space of all bounded sequences of real numbers, and $c$ is its subspace consisting of all convergent sequences. In other words, a Banach limit is a linear, positive, shift invariant and normalized functional defined on~$l^\infty$. The reader interested in Banach limits can consult, e.g., \cite{AlekhnoSemenovSukochevUsachev2018, SemenovSukochevUsachev2019, SemenovSukochevUsachev2020, Sofi2021, DasNanda2022, SemenovSukochevUsachev2023} and the references therein. From now on, we fix a Banach limit $\B$. Define $\T_S\colon \FI\to\FI$ putting \begin{equation*} \T_S\phi(x)=\sum_{k=1}^{p}\left[\phi(f_k^{-1}(x))-\phi(f_k^{-1}(0))\right] \quad\text{for every }x\in[0,1]. \end{equation*} Obviously, \begin{equation}\label{TSpsi} \T_S\phi=\phi\quad\text{for every }\phi\in\FPS. \end{equation} With any $\phi\in\FI$ we associate the function $\B_S^\phi\colon[0,1]\to\FI$ defined by \begin{equation*} \B_S^\phi(x)=\B((\T_S^m\phi(x))_{m\in\mathbb N}). \end{equation*} The function $\B_S^\phi$ may not be left-continuous. Moreover, it can happen that $\lim_{x\to 1-}\B_S^\phi(x)\in[0,1)$. Therefore, we define the function $\BB_S^\phi\colon[0,1]\to\FP$ putting \begin{equation*} \BB_S^\phi(x)=\begin{cases*} \frac{1}{\alpha}\lim_{z\to x+}\B_S^\phi(z),& if $x\in[0,1)$,\\ 1,&if $x=1$, \end{cases*} \end{equation*} where \begin{equation*} \alpha=\begin{cases*} \lim_{x\to 1-}\B_S^\phi(x),& if $\lim_{x\to 1-}\B_S^\phi(x)\in(0,1]$,\\ 1,&if $\lim_{x\to 1-}\B_S^\phi(x)=0$. \end{cases*} \end{equation*} Denote by $\chi_A$ the characteristic function of the set $A\subset\mathbb R$. \begin{remark}\label{rem:BB}\ \begin{itemize} \item[\rm (i)] If $\lim_{x\to 1-}\B_S^\phi(x)=0$, then $\BB_S^\phi=\chi_{\{1\}}$. \item[\rm (ii)] If $\lim_{x\to 1-}\B_S^\phi(x)\in(0,1]$, then $\BB_S^\phi$ is continuous at $1$. \end{itemize} \end{remark} \begin{theorem}\label{thm:BB} We have $\{\BB_S^\phi\,|\,\phi\in\mathcal I\}=\mathcal P_S\cup\{\chi_{\{1\}}\}$. \end{theorem} \begin{proof} ($\supset$) Since $\chi_{\{1\}}\in\mathcal I$ and $\BB_S^{\chi_{\{1\}}}=\chi_{\{1\}}$, we have $\chi_{\{1\}}\in\{\BB_S^\phi\,|\,\phi\in\mathcal I\}$. Fix now $\phi\in\mathcal P_S$ and $x\in[0,1]$. Repeatedly applying~\cref{TSpsi}, we get \begin{equation*} \B_S^\phi(x)=\B((T_S^m\phi(x))_{m\in\mathbb N})=\B((\phi(x))_{m\in\mathbb N})=\phi(x). \end{equation*} Hence $\BB_S^\phi=\phi$, which yields $\mathcal P_S\subset\{\BB^\phi_S\,|\,\phi\in\mathcal I\}$. ($\subset$) Fix $\phi\in\mathcal I$ and $x\in[0,1]$. Then \begin{align*} \B_S^\phi(x)&=\B\left(\left(\T_S^m\phi(x)\right)_{m\in\mathbb N}\right) =\B\left(\left(\T_S^{m+1}\phi(x)\right)_{m\in\mathbb N}\right)\\ &=\B\left(\left(\sum_{k=1}^{p}\left[\T_S^m\phi(f_k^{-1}(x)) -\T_S^m\phi(f_k^{-1}(0))\right]\right)_{m\in\mathbb N}\right)\\ &=\sum_{k=1}^{p}\left[\B\left(\left(\T_S^m\phi(f_k^{-1}(x))\right)_{m\in\mathbb N}\right) -\B\left(\left(\T_S^m\phi(f_k^{-1}(0))\right)_{m\in\mathbb N}\right)\right]\\ &=\sum_{k=1}^{p}\left[\B_S^\phi(f_k^{-1}(x))-\B_S^\phi(f_k^{-1}(0))\right]. \end{align*} In consequence, \begin{equation*} \BB_S^\phi=\sum_{k=1}^{p}\left[\BB_S^\phi\circ f_k^{-1}-\BB_S^\phi(f_k^{-1}(0))\right], \end{equation*} which jointly with \Cref{rem:BB} yields $\{\BB^\phi_S\,|\,\phi\in\FI\}\subset\FPS\cup\{\chi_{\{1\}}\}$. \end{proof} Before showing how \Cref{thm:BB} works in the $p$-adic case, let us note that \begin{equation*} \T_{S_p}\phi(x)=\sum_{k=0}^{p-1}\left[\phi\left(\frac{x+k}{p}\right) -\phi\left(\frac{k}{p}\right)\right]\quad\text{for every }x\in[0,1] \end{equation*} and, by induction, \begin{equation}\label{Tpm} \T_{S_p}^m\phi(x)=\sum_{k=0}^{p^m-1}\left[\phi\left(\frac{x+k}{p^m}\right) -\phi\left(\frac{k}{p^m}\right)\right]\quad\text{for all }m\in\mathbb N,\,x\in[0,1]. \end{equation} \begin{example}\label{ex:x^2} Fix $\phi\in\mathcal I$ of the form $\phi(x)=x^2$. By \cref{thm:BB} we have $\BB_{S_p}^\phi\in\mathcal P_{S_p}\cup\{\chi_{\{1\}}\}$. Fix $x\in[0,1]$. Then using \cref{Tpm} we obtain \begin{align*} \B_{S_p}^\phi(x)&=\B((T_{S_p}^m\phi(x))_{m\in\mathbb N}) =\B\left(\left(\sum_{k=0}^{p^m-1}\left[\left(\frac{x+k}{p^m}\right)^2 -\left(\frac{k}{p^m}\right)^2\right]\right)_{m\in\mathbb N}\right)\\ &=\B\left(\left(x+\frac{x^2-x}{p^m}\right)_{m\in\mathbb N}\right) =\lim_{m\to\infty}\left(x+\frac{x^2-x}{p^m}\right)=x. \end{align*} Therefore, $\BB_{S_p}^\phi=\id_{[0,1]}$ and the invariant measure that corresponds to $\id_{[0,1]}$ by~\cref{mu-phi} is the one-dimensional Lebesgue measure on $[0,1]$. \end{example} \renewcommand{\theequation}{5.\arabic{equation}}\setcounter{equation}{0} \section{The family \texorpdfstring{$\mathcal P_{S_p}^a$}{Pₛᵃ}}\label{S5} It is well-known (see \cite{Renyi1957, Rohlin1961}) that \begin{equation}\label{PSpa} \mathcal P_{S_p}^a=\{\id_{[0,1]}\}, \end{equation} however it can be deduced from the following more general result about the transformation~$S$ in~\cite{LasotaMackey1994}, which can be stated as follows. \begin{theorem}[{see \cite[Theorem 6.2.1]{LasotaMackey1994}; cf.~\cite[Theorem 1]{LasotaYork1973}}]\label{thm:621} Assume that $f_k\in C^2([a_k,a_{k+1}))$ for every $k\in\{0,\ldots,p-1\}$ and there exist $\alpha\in(1,\infty)$ and $\beta\in\mathbb R$ such that $\alpha\leq f_k'(x)$ and $-f_k''(x)\leq\beta [f_k'(x)]^2$ for all $k\in\{0,\ldots,p-1\}$ and $x\in[a_k,a_{k+1})$, then $\mathcal P_S$ consists of exactly one function. \end{theorem} Since $\id_{[0,1]}$ is the unique absolutely continuous function belonging to the family $\mathcal P_{S_p}$, in view of \Cref{thm:BB}, it would be interesting (see \Cref{ex:x^2}) to find the biggest (in the sense of inclusion) subfamily of the family $\mathcal I$ such that for any function $\phi$ from this subfamily we have $\BB_{S_p}^\phi=\id_{[0,1]}$. We do not know what this subfamily is, but we have the following result. \begin{theorem}\label{thm:PSpa=id} If $\phi\in\mathcal I$ is absolutely continuous, then $\lim_{m\to\infty}\T_{S_p}^m\phi(x)=x$ for every $x\in[0,1]$. In particular, $\BB_{S_p}^\phi=\B_{S_p}^\phi=\id_{[0,1]}$. \end{theorem} \begin{proof} Fix an absolutely continuous function $\phi\in\FI$ and let $\Phi\colon\mathbb R\to [0,1]$ be the $1$-periodic extension of $\phi|_{[0,1)}$. According to \cite[Theorem 7.4.4]{Lojasiewicz1988} for every $m\in\mathbb N$ the function \begin{equation*} R_m(\phi')=\frac{1}{p^m}\sum_{k=0}^{p^m-1}\Phi'\left(\cdot+\frac{k}{p^m}\right) \end{equation*} belongs to $L^1([0,1])$, and moreover, \begin{equation}\label{RiemannSum} \lim_{m\to\infty}\left\|R_m(\phi')-\int_{0}^{1}\phi'(s)ds\right\|_1=0; \end{equation} see e.g.~\cite[Section 2]{RuchWeber2006}. Fix $x\in[0,1]$ and $m\in\mathbb N$. Applying again~\cite[Theorem 7.4.4]{Lojasiewicz1988}, \cref{Tpm}, and the periodicity of $\Phi$, we obtain \begin{align*} |\T_{S_p}^m\phi(x)-x|&=\left|\int_{0}^x \left[(\T_{S_p}^m\phi)'(t)-1\right]dt\right| \leq\int_{0}^{x}|(\T_{S_p}^m\phi)'(t)-1|dt\\ &=\int_{0}^{x}\left|\frac{1}{p^m}\sum_{k=0}^{p^m-1}\phi'\left(\frac{t+k}{p^m}\right) -\int_{0}^{1}\phi'(s)ds\right|dt\\ &\leq\int_{0}^{1}\left|\frac{1}{p^m}\sum_{k=0}^{p^m-1}\Phi'\left(\frac{t+k}{p^m}\right) -\int_{0}^{1}\phi'(s)ds\right|dt\\ &=\sum_{i=0}^{p^m-1}\frac{1}{p^m}\int_{0}^{1}\left|\frac{1}{p^m} \sum_{k=0}^{p^m-1}\Phi'\left(\frac{t+i+k}{p^m}\right)-\int_{0}^{1}\phi'(s)ds\right|dt\\ &=\sum_{i=0}^{p^m-1}\int_{\frac{i}{p^m}}^{\frac{i+1}{p^m}}\left|\frac{1}{p^m} \sum_{k=0}^{p^m-1}\Phi'\left(z+\frac{k}{p^m}\right)-\int_{0}^{1}\phi'(s)ds\right|dz\\ &=\int_{0}^{1}\left|\frac{1}{p^m}\sum_{k=0}^{p^m-1}\phi'\left(z+\frac{k}{p^m}\right) -\int_{0}^{1}\Phi'(s)ds\right|dz\\ &=\left\|R_m(\phi')-\int_{0}^{1}\phi'(s)ds\right\|_1. \end{align*} This jointly with \cref{RiemannSum} completes the proof. \end{proof} \renewcommand{\theequation}{6.\arabic{equation}}\setcounter{equation}{0} \section{The family \texorpdfstring{$\mathcal{P}_{S_p}^j$}{Pₛʲ}}\label{S6} For $\phi\in\FI$ we set $\phi(x+)=\lim_{y\to x+}\phi(y)$. The next remark can be easily verified. \begin{remark}\label{rem:lim+} Assume that $\phi\in\FPS$ and $\phi(0+)=0$. Then the function $\psi\colon[0,1]\to[0,1]$ defined by $\psi(x)=\phi(x+)-\phi(x)$ satisfies~\cref{FESp} and is such that $\psi(x)=0$ if and only if $\phi$ is continuous at $x$. \end{remark} Note that if $\phi\in \mathcal{P}_{S_p}$ fails to be continuous at a point~$x$, then there are further points~$\frac{x+k}{p}$, where it fails to be continuous as well, and the jumps have to add up. But now, all of these points generate themselves further points of discontinuity. However, this process cannot continue forever as the sum of all possible jumps is bounded from above by~$1$. Therefore, points have to start to coincide, forcing them to be of a certain form. This is the motivation behind the definition of the following set: \begin{equation*} D:=\bigcup_{m\in\mathbb N}\left\{\frac{i}{p^m-1}\,\big|\,i\in\{0,\ldots,p^m-2\}\right\}. \end{equation*} \begin{prop}\label{prop:ContinuouityPoints} Every $\phi\in\FPSp$ is continuous at each point outside the set $D$. \end{prop} \begin{proof} Fix $\phi\in\FPSp$ and $z_0\in [0,1]\setminus D$. If $\phi(0+)=1$, then $\phi$ is constant on $(0,1]$, and hence continuous at $z_0$. Thus we assume that $\phi(0+)\in[0,1)$. We also assume that $\phi(0+)=0$, otherwise we replace $\phi$ with $\Phi\colon[0,1]\to[0,1]$ given by \begin{equation*} \Phi(x)=\begin{cases*} 0,&if $x=0$,\\ \frac{\phi(x)-\phi(0+)}{1-\phi(0+)},&if $x\in(0,1]$. \end{cases*} \end{equation*} Here, we gloss over a small technical issue, namely that constant functions differing from~$0$ are not contained in~$\FPSp$. However, $\phi\in\FPSp$ if and only if for all $x\in [0,1]$ \begin{equation*} \varphi(x)=\sum_{k=0}^{p-1}\varphi\left(\frac{x+k}{p}\right)-\sum_{k=1}^{p-1}\varphi\left(\frac{k}{p}\right). \end{equation*} and thus, we might have to work with this equality instead of~\cref{FESp}. Put \begin{equation*} \alpha=\phi(z_0+)-\phi(z_0) \end{equation*} and note that $\alpha\geq 0$ as $\phi$ is non-decreasing. For every $m\in\mathbb N$ we set \begin{equation*} C_m=\left\{\frac{z_0+i}{p^{m-1}}\,\big|\,i\in\{0,\ldots,p^{m-1}-1\}\right\}. \end{equation*} We want to show that \begin{equation}\label{Cm} \sum_{x\in C_m}[\phi(x+)-\phi(x)]=\alpha \end{equation} for every $m\in\mathbb N$. For $m=1$, this is clear by the definition of the number $\alpha$. Fix $m\in\mathbb N$ and assume that \cref{Cm} holds. By \cref{rem:lim+}, we have \begin{equation*} \alpha=\sum_{x\in C_m}\sum_{k=0}^{p-1}\left[\phi\left(\frac{x+k}{p} +\right)-\phi\left(\frac{x+k}{p}\right)\right] =\sum_{x\in C_{m+1}}[\phi(x+)-\phi(x)]. \end{equation*} It is easy to check that $C_n\cap C_m=\emptyset$ for all $n,m\in\mathbb N$ with $n\neq m$. This jointly with \cref{Cm} and the fact that $\phi$ is non-decreasing gives \begin{equation*} \alpha M=\sum_{m=1}^M\sum_{x\in C_m}[\phi(x+)-\phi(x)]\leq \sum_{x\in[0,1]}[\phi(x+)-\phi(x)]\leq 1 \end{equation*} for every $M\in\mathbb N$. Thus $\alpha=0$, and by \Cref{rem:lim+} we see that $\phi$ is continuous at $z_0$. \end{proof} In principle, if $f$ is not continuous at a point $x$, then there is at least one~$k$ such that $f$ also fails to be continuous at the point~$\frac{x+k}{p}$. It turns out that there is exactly one such~$k$. \begin{lemma}\label{lem:D} Let $x\in [0,1]$. Then $\card\left(D\cap\{\frac{x+k}{p}\,|\,k\in\{0,\ldots,p-1\}\right)\leq 1$ and equality holds if and only if $x\in D$. \end{lemma} \begin{proof} Fix $x\in[0,1]$ and assume by contradiction that there exist $k,l\in\{0,\ldots,p-1\}$ such that $\frac{x+k}{p},\frac{x+l}{p}\in D$, i.e.\ there are $m_1,m_2\in\mathbb N$, $i_1\in\{0,\ldots,p^{m_1}-2\}$, and $i_2\in\{0,\ldots,p^{m_2}-2\}$ such that $\frac{x+k}{p}=\frac{i_1}{p^{m_1}-1}$ and $\frac{x+l}{p}=\frac{i_2}{p^{m_2}-1}$. Then \begin{equation*} \frac{k-l}{p}=\frac{i_1(p^{m_2}-1)-i_2(p^{m_1}-1)}{(p^{m_1}-1)(p^{m_2}-1)}. \end{equation*} Since $\gcd(p,p^{m_1}-1)=\gcd(p,p^{m_2}-1)=1$, we conclude that $k=l$. To prove the second part of the lemma assume first that $x\in D$. Then there exist $m\in\mathbb N$, $k\in\{0,\ldots,p-1\}$, and $l\in\{0,\ldots,p^{m-1}-1\}$ such that $x=\frac{k+lp}{p^m-1}$; note that $1\not\in D$ yields $k\neq p-1$ or $l\neq p^{m-1}-1$. Then \begin{equation*} \frac{x+k}{p}=\frac{k+lp +k(p^m-1)}{p(p^m-1)}=\frac{l+kp^{m-1}}{p^m-1}\in D. \end{equation*} Conversely, if there exists $k\in\{0,\ldots,p-1\}$ such that $\frac{x+k}{p}\in D$, then an easy calculation shows that $x\in D$. \end{proof} The next observation is an obvious consequence of the definition of $D$, the fact that $1\notin D$, and \cref{lem:D}, and thus we omit the proof. \begin{lemma}\label{lem:Dml} If $x_0\in D$, then there exist $m\in\mathbb N$ and $(k_0,\ldots,k_{m-1})\in\{0,\ldots,p-1\}^m$ such that $k_0+k_1p+\cdots+k_{m-1}p^{m-1}\neq p^m-1$ and \begin{equation}\label{formulaDml} \frac{x_0+k_0+k_1p+\cdots+k_{m-1}p^{m-1}}{p^{m}}=x_0; \end{equation} moreover, the formulas \begin{equation}\label{cycle} x_{(n+1)(\hspace{-1.7ex}\mod m)}=\frac{x_n+k_n}{p}\quad\text{for every }n\in\{0,\ldots,m-1\} \end{equation} define the cycle $(x_0,\ldots,x_{m-1})$ with \begin{equation*}\label{formulaxn} x_n=\frac{\sum_{j=0}^{m-1}k_jp^{j+m-n}(\hspace{-2ex}\mod p^{m}-1)}{p^m-1}\in D\quad\text{for every } n\in\{1,\ldots,m-1\}. \end{equation*} \end{lemma} eld{N}$. Let $m\in\mathbb N$ be the smallest number such that $x_m=x_0$. Then the set $\{x_0,\ldots,x_{m-1}\}$ has $m$ distinct points and if $\phi\in\FPSp$ is discontinuous at a point of the set $\{x_0,\ldots,x_{m-1}\}$, then it is discontinuous at every point of that set, by the periodicity of $(x_n)_{n\in\mathbb N_0}$. This allows us to decompose the set $D$ into disjoint subsets $D^m_l$, where each of these sets is minimal (in the sense of inclusion) with the following two properties: \begin{enumerate} \item[\rm (i)] $\card(D^m_l)=m$ (the number $m\in\mathbb N$ will be called the \emph{level} of $D^m_l$, whereas the index $l\in\mathbb N_0$ is used to number the sets of level $m$); \item[\rm (ii)] If $\phi\in\FPSp$ is discontinuous at a point of $D^m_l$, then it is discontinuous at every point of $D^m_l$. \end{enumerate} In fact, each of the sets from the decomposition of $D$ is of the form described above. More precisely, fix a sequence $(k_0,\ldots,k_{m-1})\in\{0,\ldots,p-1\}^{m}$ such that $l=k_0+k_1p+\cdots+k_{m-1}p^{m-1}\neq p^m-1$ and choose $x_0\in D$ satisfying \cref{formulaDml}. Then the formula \cref{cycle} gives raise to the definition of the cycle $(x_0,\ldots,x_{m-1})$ with \begin{equation*} x_n=\frac{lp^{m-n}(\hspace{-2ex}\mod p^{m}-1)}{p^m-1} \quad\text{for every }n\in\{0,\ldots,m-1\}, \end{equation*} and $\{x_0,\ldots,x_{m-1}\}\subset D$. If $x_n\neq x_0$ for every $n\in\{0,\ldots,m-1\}$, then the cycle $(x_0,\ldots,x_{m-1})$ leads to the set $D^m_l=\{x_0,\ldots,x_{m-1}\}$ and since $(x_0,\ldots,x_{m-1})$ is a cycle, we have \begin{equation*} D^m_l=D^m_{lp^n(\hspace{-2ex}\mod p^{m}-1)}\quad\text{for every }n\in\{1,\ldots,m-1\}. \end{equation*} Assume now that $j=\min\{n\in\{0,\ldots,m-1\}\,|\,x_n=x_0\}<m$. Then $j=\card(\{x_0,\ldots,x_{m-1}\})$ and $l=lp^{j}(\hspace{-1ex}\mod p^{m}-1)$, which means that $k_{j+n}=k_n$ for every $n\in\{0,\ldots,m-1-j\}$. Since $(x_0,\ldots,x_{m-1})$ is a cycle, we see that $m=jq$ with some $q\in\mathbb N$. Hence \begin{align*} l&=k_0+\cdots+k_jp^{j-1}+k_0p^j+\cdots+k_jp^{2j-1}+\cdots+k_0p^{j(q-1)}+\cdots+k_jp^{jq-1}\\ &=(k_0+\cdots+k_jp^{j-1})(1+p^j+\cdots+p^{(q-1)j}), \end{align*} and putting $l_0=k_0+\cdots+k_jp^{j-1}$ we obtain \begin{align*} \frac{l}{p^m-1}&=\frac{l_0(1+p^j+\cdots+p^{(q-1)j})}{p^{jq}-1}= \frac{l_0(1+p^j+\cdots+p^{(q-1)j})}{(p^j-1)(1+p^j+\cdots+p^{(q-1)j})}\\ &=\frac{l_0}{p^j-1}. \end{align*} This leads to the set $D^j_{l_0}=\{x_0,\ldots,x_{j-1}\}$, and as previously, we have \begin{equation*} D^j_{l_0}=D^j_{l_0p^n(\hspace{-2ex}\mod p^{j})}\quad\text{for every }n\in\{1,\ldots,j-1\}. \end{equation*} Summarizing, we see that for each $m\in\mathbb N$ the sequence $(k_0,\ldots,k_{m-1})\in\{0,\ldots,p-1\}^{m}$ with $l=k_0+k_1p+\cdots+k_{m-1}p^{m-1}\neq p^m-1$ leads either to the set $D^m_l$ or to the set $D^j_{l_0}$ with $l_0=k_0+k_1p+\cdots+k_{j-1}p^{j-1}$ and $j$ being a divisor of $m$. In particular, we have: \begin{itemize} \item[(1)] $p-1$ sets of level $1$ of the form \begin{equation*} D^1_l=\left\{\frac{l}{p-1}\right\}, \end{equation*} where $l\in\mathbb L_1=\{0,\ldots,p-2\}$ (cf.~\cref{rem:FixedPoints}); \item[(2)] $\frac{p(p-1)}{2}$ sets of level $2$ of the form \begin{equation*} D^2_{k_0+k_1p}=D^2_{k_1+k_0p}=\left\{\frac{k_0+k_1p}{p^2-1},\frac{k_1+k_0p}{p^2-1}\right\}, \end{equation*} where $k_0,k_1\in\{0,\ldots,p-1\}$ and $k_0\neq k_1$; \item[(3)] $\frac{p(p^2-1)}{3}$ sets of level $3$ of the form \begin{align*} D^3_{k_0+k_1p+k_2p^2}&=D^3_{k_2+k_0p+k_1p^2}=D^3_{k_1+k_2p+k_0p^2}\\ &=\left\{\frac{k_0+k_1p+k_2p^2}{p^3-1}, \frac{k_1+k_2p+k_0p^2}{p^3-1},\frac{k_2+k_0p+k_1p^2}{p^3-1}\right\}, \end{align*} where $k_0,k_1,k_2\in\{0,\ldots,p-1\}$ and $k_0\neq k_1$ or $k_0\neq k_2$ or $k_1\neq k_2$; \item[($m$)] $\card \mathbb L_m$ sets of level $m\geq 2$ of the form \begin{align*} D^m_l&=D^m_{lp(\hspace{-2ex}\mod p^{m}-1)}=\cdots=D^m_{lp^{m-1}(\hspace{-2ex}\mod p^{m}-1)}\\ &=\left\{\frac{l}{p^m-1},\frac{lp\,(\hspace{-2ex}\mod p^{m}-1)}{p^m-1},\ldots,\frac{lp^{m-1}(\hspace{-2ex}\mod p^{m}-1)}{p^m-1}\right\}, \end{align*} where \begin{equation*} l\in\mathbb L_m=\left\{\sum_{i=0}^{m-1}k_ip^{i}\,\Big|\,(k_0,\ldots,k_{m-1})\in\{0,\ldots,p-1\}^m\text{ is acyclic}\right\}. \end{equation*} \end{itemize} Now, with each of the set $D^m_l$ we want to associate a jump function belonging to $\FPSp$ that is discontinuous exactly at the points of the set $D^m_l$. The first lemma concerns sets of level~$1$, and its proof is a simple verification. \begin{lemma}\label{lem:jumpFunction1} For every $l\in\{0,\ldots,p-2\}$ the function $\phi^1_l\colon[0,1]\to[0,1]$ given by \begin{equation*} \phi^1_l(x)=\chi_{(\frac{l}{p-1},1]}(x) \end{equation*} belongs to $\FPSp^j$ and is discontinuous only at the point of the set $D^1_l$. \end{lemma} The next remark is an immediate consequence of the description of sets $D^m_l$; cf.~\cref{lem:D}. \begin{remark}\label{rem:yi} Let $x\in D^m_l$. Then: \begin{enumerate}[label=(\roman*)] \item\label{xy} there exist unique $y\in D^m_l$ and $k\in\{0,\ldots,p-1\}$ such that $x=\frac{y+k}{p}$; \item\label{zx} there exist unique $z\in D^m_l$ and $q\in\{0,\ldots,p-1\}$ such that $z=\frac{x+q}{p}$. \end{enumerate} \end{remark} \begin{remark}\label{rem:unique} Let $x\in(0,1)$ and $y\in D^m_l$. Then there exists at most one $k\in\{0,\ldots,p-1\}$ such that $\frac{k}{p}\leq y<\frac{x+k}{p}$. \end{remark} \begin{proof} Assume by contradiction that $\frac{k}{p}\leq y<\frac{x+k}{p}$ and $\frac{q}{p}\leq y<\frac{x+q}{p}$ with $0\leq k<q\leq p-1$, then $\frac{q}{p}\leq y<\frac{x+k}{p}<\frac{1+k}{p}\leq\frac{q}{p}$, a contradiction. \end{proof} Before formulating the second lemma let us recall that $1\notin D$ and since $D^1_0=\{0\}$, we have $0\notin D^m_l$ for any $l\in\mathbb L_m$, whenever $m\geq 2$. \begin{lemma}\label{lem:jumpFunction2} Assume that $D^m_l=\{y_0,\ldots,y_{m-1}\}$, where $m\geq 2$, $l=\sum_{i=0}^{m-1}k_ip^{i}$ with $(k_0,\ldots,k_{m-1})\in\{0,\ldots,p-1\}^m$, and $0<y_0<\cdots<y_{m-1}<1$. Then the function $\phi^m_l\colon[0,1]\to[0,1]$ given by \begin{equation*} \phi^m_l(x)=\frac{1}{m}\sum_{i=0}^{m-1}\chi_{(y_i,1]}(x) \end{equation*} belongs to $\FPSp^j$ and is discontinuous exactly at points of the set $D^m_l$. \end{lemma} \begin{proof} It is clear from the definition of $\phi^m_l$ that it is non-decreasing, left-continuous, and discontinuous only at points of the set $D^m_l$. Moreover, $\phi^m_l(0)=0$ and $\phi^m_l(1)=1$. So, to complete the proof it suffices to show that $\phi^m_l$ satisfies \cref{FESp}, which means that \begin{equation}\label{FEchi} \phi^m_l(x)=\frac{1}{m}\sum_{i=0}^{m-1}\sum_{k=0}^{p-1} \left[\chi_{(y_i,1]}\left(\frac{x+k}{p}\right)-\chi_{(y_i,1]}\left(\frac{k}{p}\right)\right] \end{equation} for every $x\in[0,1]$. It is easy to check that \Cref{FESp} holds for $x\in\{0,1\}$. Fix $x\in(0,1)$. We will consider three cases: \begin{itemize} \item[(a)] $x\in(0,y_0]$, \item[(b)] $x\in(y_{j-1},y_j]$ for some $j\in\{1,\ldots,m-1\}$, \item[(c)] $x\in(y_{m-1},1)$. \end{itemize} Case (a). In this case we have $\phi^m_l(x)=0$. For proving that \cref{FEchi} holds, it suffices to show that for no $i\in\{0,\ldots,m-1\}$ and $k\in \{0,\ldots,p-1\}$ we have \begin{equation}\label{yi} \frac{k}{p}\leq y_i<\frac{x+k}{p}; \end{equation} indeed, then for all $i\in\{0,\ldots,m-1\}$ and $k\in \{0,\ldots,p-1\}$ we have \begin{equation*} \chi_{(y_i,1]}\left(\frac{x+k}{p}\right)=\chi_{(y_i,1]}\left(\frac{k}{p}\right). \end{equation*} Let us assume by contradiction that there are $i\in\{0,\ldots,m-1\}$ and $k\in\{0,\ldots,p-1\}$ satisfying \cref{yi}. By \Cref{rem:yi}~\cref{xy} there exists $y_j\in D^m_l$ and $k_0\in \{0,\ldots,p-1\}$ such that $y_i=\frac{y_j+k}{p}$. Actually, $k$~and~$k_0$ are the same: \begin{equation*} \frac{k}{p}\leq \frac{y_j+k_0}{p} <\frac{x+k}{p}, \end{equation*} yielding \begin{equation*} -1<y_j-x <k-k_0\leq y_j<1, \end{equation*} and thus $k=k_0$. Then \begin{equation*} \frac{k}{p}\leq\frac{y_j+k}{p}<\frac{x+k}{p}\leq\frac{y_0+k}{p}, \end{equation*} and hence $y_j\in(0,y_0)$, a contradiction to the fact that $y_0$ is minimal. Case (b). In this case we have $\phi^m_l(x)=\frac{j}{m}$. To prove that \cref{FEchi} holds, it is enough to show that there are exactly $j$ distinct points $y_{i_1},\ldots,y_{i_j}\in D^m_l$ such that for each $i\in\{i_1,\ldots,i_j\}$ there exists only one $k\in\{0,\ldots,p-1\}$ satisfying $\cref{yi}$; indeed, then \begin{equation*} \sum_{k=0}^{p-1}\left[\chi_{(y_i,1]}\left(\frac{x+k}{p}\right) -\chi_{(y_i,1]}\left(\frac{k}{p}\right)\right]= \begin{cases*} 1,&if $i\in\{i_1,\ldots,i_j\}$,\\ 0,& if $i\notin\{i_1,\ldots,i_j\}.$ \end{cases*} \end{equation*} Assume first that $y_i$ satisfies \cref{yi} with some $k\in\{0,\ldots,p-1\}$. Note that $k$ is unique by \Cref{rem:unique}. From \Cref{rem:yi}~\cref{xy} there exists $y_q\in D^m_l$ and $k_0\in \{1,\ldots,p-1\}$ such that $y_i=\frac{y_q+k_0}{p}$. As \begin{equation*} \frac{k_0}{p}\leq \frac{y_q+k_0}{p} =y_i <\frac{k_0+1}{p}, \end{equation*} the uniqueness just mentioned now guarantees that $k=k_0$. Then, by~\cref{yi}, we have \begin{equation*} \frac{k}{p}\leq\frac{y_q+k}{p}<\frac{x+k}{p}\leq\frac{y_j+k}{p}, \end{equation*} and hence $y_q\in \{y_0,\ldots,y_{j-1}\}$. This shows that there exist at most $j$ distinct points in $D^m_l$ with the required property. Fix now $y_q\in \{y_0,\ldots,y_{j-1}\}$. By \Cref{rem:yi}~\cref{zx} there are $y_i\in D^m_l$ and $k\in \{0,\ldots,p-1\}$ such that $y_i=\frac{y_q+k}{p}$, and hence \begin{equation*} \frac{k}{p}<y_i=\frac{y_q+k}{p}\leq\frac{y_{j-1}+k}{p}<\frac{x+k}{p}, \end{equation*} which means that \cref{yi} holds. Therefore, we showed that each of the elements in $\{y_0,\ldots,y_{s-1}\}$ leads to some $y_i\in D^m_l$ and $k\in\{0,\ldots,p-1\}$ satisfying~\cref{yi}. Actually more is true: the $y_i$ are distinct, and in consequence, the required property holds. Case (c). In this case $\phi_l^m(x)=1$. Let $y_i\in\{y_0,\ldots,y_{m-1}\}$. From \Cref{rem:yi}~\cref{xy} there are unique $y_q\in D^m_l$ and $k\in \{0,\ldots,p-1\}$ such that $y_i=\frac{y_q+k}{p}$. Then \begin{equation*} \frac{k}{p}<y_i=\frac{y_q+k}{p}\leq\frac{y_{m-1}+k}{p}<\frac{x+k}{p} \end{equation*} and, by \Cref{rem:unique}, we see that for each $i\in \{0,\ldots,m-1\}$ there is exactly one $k\in\{0,\ldots,p-1\}$ satisfying \cref{yi}. Therefore, for every $i\in \{0,\ldots,m-1\}$ we have \begin{equation*} \sum_{k=0}^{p-1}\left[\chi_{(y_i,1]}\left(\frac{x+k}{p}\right) -\chi_{(y_i,1]}\left(\frac{k}{p}\right)\right]=1, \end{equation*} which implies that \cref{FEchi} holds. \end{proof} We conclude our investigations with the following result. \begin{theorem}\label{thm:jamp} Let $\phi\in\FPSp$. Then $\phi\in\FPSp^j$ if and only if there exists a set $\{\alpha^m_l\,|\,m\in\mathbb N, l\in \mathbb L_m\}$ of non-negative real numbers with $\sum_{m\in\mathbb N}\sum_{l\in\mathbb L_m}\alpha^m_l=1$ such that \begin{equation}\label{jumpSolution} \phi=\sum_{m\in\mathbb N}\sum_{l\in\mathbb L_m}\alpha^m_l\phi^m_l. \end{equation} \end{theorem} \begin{proof} Making use of~\cref{lem:jumpFunction1,lem:jumpFunction2} it is easy to check that the function $\phi$ given by~\cref{jumpSolution} belongs to $\FPSp^j$. Let now $\phi\in\FPSp^j$. Choose $x_0\in D$ with $s_1:=\phi(x_0+)-\phi(x_0)\in(0,1)$. Then there are $m_1\in\mathbb N$ and $l_1\in\mathbb L_{m_1}$ such that $x_0\in D^{m_1}_{l_1}$. By \Cref{lem:D} for every $x\in D^{m_1}_{l_1}$ we have $\phi(x+)-\phi(x)=s_1$. Moreover, $m_1s_1\in \opencl{0}{1}$. Therefore, the function $\phi-m_1s_1\phi^{m_1}_{l_1}$ is non-decreasing, continuous at every point of the set $D^{m_1}_{l_1}$, and $\phi(1)-m_1s_1\phi^{m_1}_{l_1}(1)=1-m_1s_1$. If $m_1s_1=1$, then $\phi=m_1s_1\phi^{m_1}_{l_1}$ and we are done. If $m_1s_1\in(0,1)$, then $\frac{1}{1-m_1s_1}(\phi-m_1s_1\phi^{m_1}_{l_1})\in\FPSp^j$ and repeating the same arguments as before we can choose $m_2\in\mathbb N$, $l_2\in\mathbb L_{m_2}$, and $s_2\in(0,1)$ such that the function $\frac{1}{1-m_1s_1}(\phi-m_1s_1\phi^{m_1}_{l_1})-m_2s_2\phi^{m_2}_{l_2}$ is non-decreasing, continuous at every point of the set $D^{m_1}_{l_1}\cup D^{m_2}_{l_2}$, and $\frac{1}{1-m_1s_1}(\phi(1)-m_1s_1\phi^{m_1}_{l_1}(1))-m_2s_2\phi^{m_2}_{l_2}(1)=1-m_2s_2$. Again, if $m_2s_2=1$, then $\phi=m_1s_1\phi^{m_1}_{l_1}+m_2s_2\phi^{m_2}_{l_2}$ and we are done. Otherwise we continue the reasoning. Since the set of all discontinuity points of $\phi$ is countable (see \Cref{prop:ContinuouityPoints}), the proof can be completed by induction. \end{proof} From \cref{thm:jamp} we see that there are solutions of equation \cref{FESp} that are discontinuous exactly at the points of the set $D$. \renewcommand{\theequation}{7\arabic{equation}}\setcounter{equation}{0} \section{The family \texorpdfstring{$\mathcal P_{S_p}^s$}{Pₛˢ}}\label{S7} One can ask if it would be possible to formulate any counterpart of \Cref{thm:PSpa=id} for the family $\FPSp^s$. Unfortunately, we do not know if, for a continuous and singular $\phi\in\FI$, the sequence $(\T^m_{S_p}\phi)_{m\in\mathbb N}$ converges pointwise, and if so, to which function. We do not even know if, for a continuous (and singular) $\phi\in\FI$, the function $\B_{S_p}^\phi$ is continuous (and singular). So, we cannot determine members of the family $\FPSp^s$ by any formula similar to that in~\cref{thm:PSpa=id}. Fix $\phi\in\FI$. By \cref{thm:BB} we have $\BB_{S_p}^{\phi}\in\FPSp$ or $\BB_{S_p}^{\phi}=\chi_{\{1\}}$. If $\BB_{S_p}^{\phi}\neq\chi_{\{1\}}$, then applying \cref{prop:ASJ} together with \cref{PSpa} and \cref{thm:jamp} we conclude that there are constants $\alpha\in[0,1]$ and $\alpha^m_l\in[0,1]$, for all $m\in\mathbb N$ and $l\in \mathbb L_m$, such that $\alpha+\sum_{m\in\mathbb N}\sum_{l\in\mathbb L_m}\alpha^m_l\in[0,1]$ and \begin{equation*} \BB_{S_p}^{\phi}-\alpha\id_{[0,1]}-\sum_{m\in\mathbb N}\sum_{l\in\mathbb L_m}\alpha^m_l\phi^m_l\in\FPSp^s. \end{equation*} All the constants can be easily calculated. As $\BB_{S_p}^{\phi}$ is differentiable almost everywhere, we have (almost everywhere) \begin{equation*} \alpha=(\BB_{S_p}^{\phi})', \end{equation*} whereas for all $m\in\mathbb N$ and $l\in \mathbb L_m$ we have \begin{equation*} \alpha^m_l=m\left[\BB_{S_p}^{\phi}\left(\frac{l}{p^m-1}+\right)-\BB_{S_p}^{\phi}\left(\frac{l}{p^m-1}\right)\right]. \end{equation*} In consequence, for any $\phi\in\FI$ such that $\BB_{S_p}^{\phi}\neq\chi_{\{1\}}$ the formula \begin{equation*} \BB_{S_p}^{\phi}-(\BB_{S_p}^{\phi})'\id_{[0,1]}-\sum_{m\in\mathbb N}m\sum_{l\in\mathbb L_m}\left[\BB_{S_p}^{\phi}\left(\frac{l}{p^m-1}+\right)-\BB_{S_p}^{\phi}\left(\frac{l}{p^m-1}\right)\right]\phi^m_l \end{equation*} defines a function belonging to $\FPSp^s$. Note that $\phi\in\FPSp^s$ gives $\BB_{S_p}^{\phi}=\phi$, so the above formula holds as $(\BB_{S_p}^{\phi})'=0$ and $\BB_{S_p}^{\phi}(\frac{l}{p^m-1}+)=\BB_{S_p}^{\phi}(\frac{l}{p^m-1})$ for all $m\in\mathbb N$ and $l\in \mathbb L_m$. Let us finish this short section with some information on previous results concerning the family $\FPSdwa^s$. Namely, in~\cite{MorawiecZurcher2018} (see also \cite[Section 5C]{Kairies1997}), it was observed that $\FPSdwa^s$ contains a large family of strictly increasing and H\"{o}lder continuous functions that are convex combinations of the singular de~Rham functions from \cite{Rham1956} (studied earlier in \cite{Cesaro}, \cite{Faber}, and \cite{Salem}). Next in~\cite{MorawiecZurcher2021} more new families of strictly increasing functions belonging to $\FPSdwa^s$ were found. Among them a family of functions that are not H\"{o}lder continuous. Recently, in~\cite{MorawiecZurcher2024}, it was observed that $\FPSdwa^s$ also contains a quite large family of Cantor-type functions. All the results from the mentioned papers concerns the family $\FPSdwa^s$, however some of them can be generalized to the family $\FPSp^s$. \section*{Acknowledgement} The research was supported by the University of Silesia Mathematics Department (Iterative Functional Equations and Real Analysis program). \bibliographystyle{plain} \bibliography{bibliography} \end{document}
2412.06402v2
http://arxiv.org/abs/2412.06402v2
VC-dimensions for set familes between partially ordered set and totally ordered set
\documentclass[a4paper,11pt]{article} \usepackage[left = 2cm, right = 2cm, top = 2cm, bottom = 2cm]{geometry} \usepackage{amsmath,amsthm,amssymb, amsfonts, mathrsfs} \usepackage{mathtools} \usepackage[shortlabels]{enumitem} \usepackage[hidelinks]{hyperref} \usepackage[nameinlink,capitalise,noabbrev]{cleveref} \usepackage{graphicx, float, tikz, subcaption} \usepackage[linesnumbered,boxed,ruled,vlined]{algorithm2e} \usepackage{multirow} \theoremstyle{definition} \newtheorem{definition}{Definition}[section] \newtheorem{example}[definition]{Example} \newtheorem{construction}[definition]{Construction} \newtheorem{remark}[definition]{Remark} \newtheorem{problem}[definition]{Problem} \theoremstyle{plain} \newtheorem{conjecture}[definition]{Conjecture} \newtheorem{theorem}[definition]{Theorem} \newtheorem{lemma}[definition]{Lemma} \newtheorem{proposition}[definition]{Proposition} \newtheorem{claim}[definition]{Claim} \newtheorem{question}[definition]{Question} \newtheorem{corollary}[definition]{Corollary} \def \tri {\triangle} \def \ex {\mathrm{ex}} \def \sm {\setminus} \def \cl {\colon} \def \ce {\coloneqq} \def \A {\mathbb{A}} \def \E {\mathbb{E}} \def \F {\mathbb{F}} \def \N {\mathbb{N}} \def \P {\mathbb{P}} \def \Z {\mathbb{Z}} \renewcommand{\le}{\leqslant} \renewcommand{\ge}{\geqslant} \renewcommand{\leq}{\leqslant} \renewcommand{\geq}{\geqslant} \makeatletter \def \eps {\varepsilon} \def \es {\varnothing} \renewcommand \b[2] {\binom{#1}{#2}} \newcommand*{\rom}[1]{\expandafter\@slowromancap\romannumeral #1@} \newcommand{\rI}{\rom{1}} \newcommand{\rII}{\rom{2}} \newcommand{\rIII}{\rom{3}} \def \a {\mathbf{a}} \def \mA {\mathcal{A}} \def \mB {\mathcal{B}} \def \c {\mathbf{c}} \def \mC {\mathcal{C}} \def \mE{\mathcal{E}} \def \mF{\mathcal{F}} \def \mG {\mathcal{G}} \def \mH{\mathcal{H}} \def \mI {\mathcal{I}} \def \mK {\mathcal{K}} \def \mP{\mathcal{P}} \def \mS {\mathcal{S}} \def \mT {\mathcal{T}} \newcommand{\floor}[1]{\left \lfloor #1 \right \rfloor} \newcommand{\ceil}[1]{\left \lceil #1 \right \rceil} \DeclareMathOperator{\poly}{poly} \DeclareMathOperator{\VC}{VC} \renewcommand {\vec}[1]{\overrightarrow{#1}} \title{VC-dimensions between partially ordered sets and totally ordered sets} \author{ Boyan Duan\thanks{School of Computer Science, ETH Z\"urich, Z\"urich 8092, Switzerland. \texttt{[email protected]}. } \and Minghui Ouyang\thanks{School of Mathematical Sciences, Peking University, Beijing 100871, China. \texttt{[email protected]}. } \and Zheng Wang\thanks{School of Mathematical Sciences, Peking University, Beijing 100871, China. \texttt{[email protected]}. } } \date{\vspace{-5ex}} \begin{document} \maketitle \begin{abstract} We say that two partial orders on $[n]$ are compatible if there exists a partial order that is finer than both of them. Under this compatibility relation, the set of all partial orders $\mathcal{F}$ and the set of all total orders $\mathcal{G}$ on $[n]$ naturally define set families on each other, where each order is identified with the set of orders that are compatible with it. In this note, we determine the VC-dimension of $\mathcal{F}$ on $\mathcal{G}$ by showing that $\operatorname{VC}_{\mathcal{G}}(\mathcal{F}) = \lfloor\frac{n^2}{4}\rfloor$ for $n \ge 4$. We also prove $2(n-3) \le \operatorname{VC}_{\mathcal{F}}(\mathcal{G}) \le n \log_2 n$ for $n \ge 1$. \medskip \noindent \textit{Keywords:} Compatible posets, VC-dimension. \end{abstract} \section{Introduction} For a family $\mF$ of subsets of a set $X$, a subset $S \subseteq X$ is said to be \emph{shattered} by $\mF$ if, for every $A\subseteq S$ there exists $B \in \mF$ such that $B \cap S = A$. The \emph{VC-dimension} of $\mF$ is the largest cardinality of a subset of $X$ that is shattered by $\mF$. We denote the VC-dimension of $\mF$ on $X$ by $\VC_{X}(\mF)$. Since its introduction by Vapnik and Chervonenkis~\cite{VC71} in the context of learning theory, VC-dimension has played a central role in various areas of mathematics and computer science. If $\mF$ shatters $S$, each subset of $S$ corresponds to a distinct element in $\mF$. Hence the VC-dimension of a set family $\mF$ is at most $\log_2 |\mF|$ on any set. Fix a ground set $[n]$, we consider the set of partial orders and total orders on $[n]$ under the \emph{compatibility} relation defined below. \begin{definition} \label{def:compatibility} Given an integer $n$, let $\mF$ (resp. $\mG$) denote the set of all partial (resp. total) orders on $[n]$. Clearly, $\mG \subseteq \mF$. We say that two partial orders $<_1$ and $<_2$ on $[n]$ are \emph{compatible} if there exists a partial order that is finer than both $<_1$ and $<_2$. Equivalently, the directed graph $<_1 \cup <_2$ is acyclic. In the special case where $<_1$ is a total order, the compatibility relation is equivalent to requiring that $<_1$ is a linear extension of $<_2$. \end{definition} Under this compatibility relation, $\mF$ and $\mG$ naturally define set families on each other. \begin{definition} \label{def:set_families} For each element $A \in \mF$ (resp. $A \in \mG$), we define the \emph{associated} subset of $\mG$ (resp. $\mF$) as the set of elements $B \in \mG$ (resp. $B \in \mF$) that are compatible with $A$. We define $\VC_{\mG}(\mF)$ as the VC-dimension of $\mF$ when $\mF$ is considered as the associated set family grounding on $\mG$. We define $\VC_{\mF}(\mF), \VC_{\mG}(\mF)$ and $\VC_{\mG}(\mG)$ in the same manner. \end{definition} Kleitman and Rothschild~\cite{KR70} showed that the size of $\mF$ is approximately $2^{(1+o(1)) \frac{n^2}{4}}$. Hence, we have $\VC_{\mG}(\mF) \le \VC_{\mF}(\mF) \le \log_2 2^{(1+o(1)) \frac{n^2}{4}} = (1+o(1))\frac{n^2}{4}$. We show that actually we can drop the $(1+o(1))$ term here. \begin{theorem} \label{thm:vc_dim_partial_to_total} For $n \ge 1$, we have \[ \VC_{\mG}(\mF) = \begin{cases} 3,& n = 3 \\ \floor{\frac{n^2}{4}} ,& n \neq 3. \end{cases} \] \end{theorem} We believe $\VC_{\mF}(\mF)$ also equals to $\floor{\frac{n^2}{4}}$. However, our proof does not directly extend to this case. For the VC-dimension of $\mG$, it is easy to see that $\VC_{\mG}(\mG) = 1$ when considering $\mG$ as a set family on itself, since the only compatible total order with a given total order $<$ is $<$ itself. Thus, the only meaningful question is to determine the value of $\VC_{\mF}(\mG)$. We establish the following lower bound by explicitly constructing a shattered set. However, we believe the correct asymptotic value for $\VC_{\mF}(\mG)$ is $\Theta(n \log n)$. \begin{theorem} \label{thm:vc_dim_total_to_partial} For $n \ge 1$, we have \[ 2(n-3) \le \VC_{\mF}(\mG) \le n \log_2 n. \] \end{theorem} \section{Proofs} \begin{proof}[\underline{Proof of \Cref{thm:vc_dim_partial_to_total}}] For $n = 1,2$, it is easy to see that $\VC_\mG(\mF) = n-1 = \lfloor \frac{n^2}{4} \rfloor$. For $n = 3$, $S = \{123, 231, 312\}$ is a shattered set of size $3$ (notice $\varnothing$ is a poset which is compatible with both of them), here the string ``$abc$'' means the total order $a < b < c$ on $\{1,2,3\}$. And it is easy to check there is no such set of size $4$. Now assume $n \ge 4$. We represent partial orders as directed graphs, where an edge $a \to b$ corresponds to the relation $a < b$ in the partial order. Suppose $S \subseteq \mG$ is a shattered set, we aim to show that $|S| \le \floor{\frac{n^2}{4}}$. Since $n \le \floor{\frac{n^2}{4}}$ for every $n \ge 4$, we may assume $|S| \ge n+1$ for contradiction; otherwise, the bound is already established. By the definition of shattering, for each $A \in S$, there exists a partial order $G_A$ on $[n]$ which is incompatible with $A$ but compatible with all elements in $S \setminus \{ A \}$. Since $G_A$ is incompatible with $A$ and $A$ is a total order, there is an edge $e_A \in G_A$ that contradicts $A$. Define $G \ce \{e_A \cl A \in S\}$ as the directed graph formed by the set of edges $e_A$. We now establish the following two claims: \textbf{Claim 1: $G$ is acyclic.} Suppose, for contradiction, that $G$ contains a directed cycle $C = e_{A_1} e_{A_2} \cdots e_{A_k}$. Assume without loss of generality that $C$ is a simple cycle, so $k \le n$. Since $|S| \ge n+1$, there exists some $B \in S \setminus \{A_1, \cdots, A_k\}$. By the definition of $G_{A_i}$, each edge $e_{A_i}$ is compatible with $B$. Since $B$ is a total order, which decides the order of each pair of elements, this implies that $e_{A_i} \in B$ for all $i$. Hence, the cycle $C$ is completely contained in $B$, which contradicts the assumption that $B$ is an order. \textbf{Claim 2: $G$ does not contain a directed path of length $\ge 2$ between any pair $(x,y)$ if $\vec{xy} \in G$.} Assume for contradiction that there exists a directed path $P = e_{A_1} e_{A_2} \cdots e_{A_k}$ from $x$ to $y$ with $k \ge 2$, and that $\vec{xy} \in G$ via some $e_B = \vec{xy}$. By the same argument as before, each $e_{A_i}$ must belong to $B$, and since $B$ is an order, we conclude $\vec{xy} \in B$. However, this contradicts the fact that $\vec{xy} = e_B \in G_B$, since $e_B$ was chosen to contradict $B$. From the above to claims, $G$ is a triangle-free graph, since any ordering of a triangle would contradict with one of the assertions above. Hence, $|S| = |E(G)| \le \floor{\frac{n^2}{4}}$, as desired. Next, we show that there is a shattered set $S$ of size $\floor{\frac{n^2}{4}}$ on $\mG$. Consider the pairs $(i,j)$ where $1 \le i \le \floor{\frac{n}{2}}$ and $\floor{\frac{n}{2}}+1 \le j \le n$. Define the directed edge $e_{i,j} = i \to j$ and its reversse $\overline{e_{i,j}} = i \gets j$. For each pair $(i,j)$, let $A_{i,j}$ be a topological ordering of the acyclic graph $\{\overline{e_{i,j}}\} \cup \{e_{i',j'} \cl (i',j') \neq (i,j)\}$. We claim $S \ce \{A_{i,j} \cl 1 \le i \le \floor{\frac{n}{2}}, \floor{\frac{n}{2}}+1 \le j \le n\}$ is a shattered set. For any subset $\mA \subseteq S$, define a partial order $G = \{ e_{i,j} \cl A_{i,j} \in \mA\}$. By construction, $G$ is compatible with elements of $S \setminus \mA$ but incompatible with elements in $\mA$, proving that $S$ is a shattered set. \end{proof} \begin{proof}[\underline{Proof of \Cref{thm:vc_dim_total_to_partial}}] Since $\mG$ consists of the set of all total orders on $[n]$, we have $|\mG| = n!$. Therefore, $\VC_{\mF}(\mG) \le \log_2 |\mG| \le n \log_2 n$. For the lower bound of $\VC_{\mF}(\mG)$, we present two different constructions of shattered sets. These constructions yield the bounds $\VC_{\mF}(\mG) \ge 3\left( \floor{\frac{n}{2}} - 1 \right)$ and $\VC_{\mF}(\mG) \ge 2(n-3)$ respectively. Since the mechanics are different, we think it is worth to present both of them here. Each of the constructions is a family of directed graphs, denoted $\{G_i\}$ and $\{H_i\}$, where the transitive closures of the elements of them, considered as a partially ordered set, form a shattered set. For any subset $S \subseteq \{G_i\}$ or $S \subseteq \{H_i\}$, there is a way to select an edge from each subgraph in $S$ and delete the rest of the edges, such that if we flip the direction of those edges and leave the subgraphs not belonging to $S$ unchanged, the resulting graph would always have no direct cycle. We call this \textbf{property ($\boldsymbol{\ast}$)}. This is simply a restatement of the definition of shattered sets, as any topological ordering of this acyclic graph would be compatible with the subgraphs that are precisely not in $S$. \begin{figure}[ht!] \centering \begin{subfigure}{.5\textwidth} \centering \includegraphics[width=.9\linewidth]{P1} \caption{The graph $G$} \end{subfigure} \begin{subfigure}{.5\textwidth} \centering \includegraphics[width=.8\linewidth]{P2} \caption{The graph $H$} \end{subfigure} \end{figure} Let $k \ce \floor{\frac{n}{2}}$. We take \[ G_1 \ce \left\{ \vec{u_1u_2}, \vec{u_2u_{k+2}} \right\},\ G_2 \ce \left\{ \vec{u_2u_3} \right\}, \cdots,\ G_k \ce \left\{ \vec{u_ku_{k+1}} \right\},\ G_{k+1} \ce \left\{ \vec{u_1u_{k+1}}, \vec{u_{k+1}u_{k+2}} \right\}, \] \[ G_{k+2} \ce \left\{ \vec{u_1v_3}, \vec{v_3u_{k+2}} \right\}, \cdots,\ G_{2k-1} \ce \left\{ \vec{u_1v_k}, \vec{v_ku_{k+2}} \right\}, \] \[ G_{2k} \ce \left\{ \vec{u_3v_3} \right\}, \cdots,\ G_{3k-3} \ce \left\{ \vec{u_kv_k} \right\}, \] and \[ H_1 \ce \left\{ \vec{w_1w_4}, \vec{w_4w_2} \right\},\ H_2 \ce \left\{ \vec{w_1w_5}, \vec{w_5w_2} \right\}, \cdots,\ H_{n-3} \ce \left\{ \vec{w_1w_n}, \vec{w_nw_2} \right\}, \] \[ H_{n-2} \ce \left\{ \vec{w_3w_4} \right\}, \cdots,\ H_{2n-6} \ce \left\{ \vec{w_3w_n} \right\}. \] We now verify \textbf{($\boldsymbol{\ast}$)} for the families $\{G_i\}$ and $\{H_i\}$ respectively. It is easy to see that both $G \ce \bigcup_{i = 1}^{3k-3} G_i$ and $H \ce \bigcup_{i = 1}^{2n-6} H_i$ have $n$ vertices (we add an isolated vertex $n$ to $G$ when $n$ is odd), are acyclic, and are unions of $3\left( \floor{\frac{n}{2}} - 1 \right)$ and $2(n-3)$ subgraphs $\{G_i\}$ and $\{H_i\}$ respectively. \textbf{$\boldsymbol{\{G_i\}}$ satisfies ($\boldsymbol{\ast}$):} Suppose $S$ is an arbitrary subset of $\{G_i\}$. The graphs $G_2, \cdots, G_k$ and $G_{2k}, \cdots, G_{3k-3}$ each contain only a single edge, so the direction of edges in these graphs after the flip is determined. If at least one of $G_2, \cdots, G_k$ belongs to $S$, we flip $u_2u_{k+2}$ and delete $u_1u_2$ whenever $G_1 \in S$, and flip $u_1u_{k+1}$ and delete $u_{k+1}u_{k+2}$ whenever $G_{k+1} \in S$. Otherwise, we flip $u_1u_2$ and delete $u_2u_{k+2}$ whenever $G_1 \in S$, and flip $u_{k+1}u_{k+2}$ and delete $u_1u_{k+1}$ whenever $G_{k+1} \in S$. Since $u_1u_2 \cdots u_{k+1}u_1$ and $u_2u_3 \cdots u_{k+2}u_2$ are the only two potential cycles in $\{u_1,\cdots, u_{k+2}\}$ regardless of the orientation, these are the only ways for the vertices $\{u_1, \cdots, u_{k+2}\}$ to form a cycle internally. Under the above strategy, it is easy to see that neither of these cycles can exist in the resulting graph. Therefore, we may consider $\{ u_1, \cdots, u_{k+2} \}$ as a single unified vertex to form a new graph. Without directed cycle in this graph implies that the original graph has no directed cycle either. We flip the subgraphs in $S \cap \{G_{k+2}, \cdots, G_{3k-3}\}$ in the same manner as the following contruction $\{H_i\}$. \textbf{$\boldsymbol{\{H_i\}}$ satisfies ($\boldsymbol{\ast}$):} Suppose $S$ is an arbitrary subset of $\{H_i\}$. Let $S_1 \ce S \cap \{H_i \cl H_{i+(n-3)} \in S\}$, $S_2 \ce S \cap \{H_i \cl H_{i+(n-3)} \notin S\}$ and $S_3 \ce S \cap \{H_{n-2}, \cdots, H_{2(n-3)}\}$. Then $S = S_1 \sqcup S_2 \sqcup S_3$ is a decomposition of $S$. We flip subgraphs in $S$ according to the following strategy: For each $H_i \in S_1$, we flip $w_1w_{i+3}$ and delete $w_{i+3}w_2$; for $H_i \in S_2$, we flip $w_{i+3}w_2$ and delete $w_1w_{i+3}$; for $H_i \in S_3$, we flip the only edge $w_3w_{i-(n-6)}$. It is easy to see that there is no directed path from $w_2$ to $w_1$ after the flip. Therefore, any directed cycle in the resulting graph cannot pass through $w_1$ and $w_2$ simultaneously. Hence, if a cycle exists, it must pass through both $w_1$ and $w_3$ or both $w_2$ and $w_3$. However, under the above flipping strategy, there is no directed path from $w_3$ to $w_1$ or from $w_2$ to $w_3$. Hence, there is no directed cycle after the flip. \end{proof} \bibliographystyle{abbrv} \bibliography{reference} \end{document}
2412.06437v2
http://arxiv.org/abs/2412.06437v2
Minimization of the first eigenvalue for the Lamé system
\documentclass[11pt]{article} \usepackage{amsbsy} \usepackage{amssymb,amsthm, amsmath, latexsym} \usepackage{mathrsfs} \usepackage{physics} nt') \usepackage{braket} \usepackage{color} \usepackage[colorlinks=true,citecolor={Plum},linkcolor={red}]{hyperref} \usepackage{pdfsync} \usepackage{footmisc} \usepackage{dsfont} \usepackage{graphicx,epsfig,subfigure,psfrag} \hoffset=-2cm \voffset=-1cm \setlength{\textwidth}{16cm} \setlength{\textheight}{22cm} \setlength{\footskip}{1.5cm} \pagestyle{plain} \setcounter{page}{1} \usepackage{color} \usepackage{enumerate} \usepackage[usenames,dvipsnames]{pstricks} \newtheorem{theorem}{Theorem}[section] \newtheorem{proposition}{Proposition}[section] \newtheorem{fact}{Fact}[section] \newtheorem{lemma}{Lemma}[section] \newtheorem{corollary}{Corollary}[section] \newtheorem{definition}{Definition}[section] \newtheorem{remark}{Remark}[section] \newcommand{\tex}{\textstyle} \numberwithin{equation}{section} \numberwithin{theorem}{section} \numberwithin{proposition}{section} \numberwithin{lemma}{section} \numberwithin{corollary}{section} \numberwithin{definition}{section} \numberwithin{remark}{section} \newcommand{\dm}{\,\mathrm{d}} \newcommand{\LL}{\mathcal{L}} \newcommand{\HH}{\mathcal{H}} \newcommand{\R}{\mathbb{R}} \newcommand{\N}{\mathbb{N}} \newcommand{\C}{{\mathcal C}} \newcommand{\Ccap}{{\operatorname{Cap}}} \newcommand{\diam}{{\rm diam}} \newcommand{\D}{{\mathcal D}} \newcommand{\ol}{\overline} \newcommand{\Ms}{{\mathbb M}^{2{\times}2}_{\rm sym}} \newcommand{\com}{\color{black}} \newcommand{\verde}{\textcolor{black!55!green}} \date{\today} \title{Minimization of the first eigenvalue for the Lam\'e system} \author{Antoine Henrot\footnote{Universit\'e de Lorraine, CNRS, Institut Elie Cartan de Lorraine, BP 70239 54506 Vand\oe uvre-l\`es-Nancy Cedex, France ({\tt [email protected]}).} \and Antoine Lemenant\footnote{Universit\'e de Lorraine, CNRS, Institut Elie Cartan de Lorraine, BP 70239 54506 Vand\oe uvre-l\`es-Nancy Cedex, France ({\tt [email protected]}).}~\footnote{Institut Universitaire de France (IUF)} \and Yannick Privat\footnote{Universit\'e de Lorraine, CNRS, Institut Elie Cartan de Lorraine, Inria, BP 70239 54506 Vandœuvre-l\`es-Nancy Cedex, France. ({\tt [email protected]}).}~\footnotemark[3] } \date{\today} \begin{document} \maketitle \begin{abstract} In this article, we address the problem of determining a domain in $\R^N$ that minimizes the first eigenvalue of the Lamé system under a volume constraint. We begin by establishing the existence of such an optimal domain within the class of quasi-open sets, showing that in the physically relevant dimensions $N = 2$ and $3$, the optimal domain is indeed an open set. Additionally, we derive both first and second-order optimality conditions. Leveraging these conditions, we demonstrate that in two dimensions, the disk cannot be the optimal shape when the Poisson ratio is below a specific threshold, whereas above this value, it serves as a local minimizer. We also extend our analysis to show that the disk is nonoptimal for Poisson ratios $\nu$ satisfying $\nu \leq 0.4$. \end{abstract} \tableofcontents \section{Introduction} The Faber–Krahn inequality is one of the most fundamental results in spectral geometry. It states that, among all sets of a given volume (in any dimension), the ball uniquely minimizes the first eigenvalue of the Dirichlet–Laplacian, see \cite{Faber1923}, \cite{Krahn}. Similar results hold for other boundary conditions. For instance: \begin{itemize} \item the ball maximizes the first non-trivial eigenvalue of the Neumann–Laplacian (Szeg\"o-Weinberger see \cite{Szego}, \cite{Weinberger}), \item the ball minimizes the first eigenvalue of the Robin-Laplacian (Bossel-Daners (when the boundary parameter is positive) see \cite{Bossel}, \cite{Daners}), \item the ball maximizes the first (non-trivial) eigenvalue of the Steklov-Laplacian (Brock see\cite{Brock}). \end{itemize} In all of these problems, a volume constraint is imposed. For further discussion on eigenvalue optimization problems, see \cite{henrot06}, \cite{henrot18}. The question of minimizing or maximizing the first eigenvalue in the case of systems is far less understood. Notably, unlike the scalar cases discussed earlier, the ball does not necessarily serve as the extremal domain. Recent studies have explored this question for the Stokes operator. It is shown that in three dimensions, the ball does not minimize the first eigenvalue among sets of a given volume. In two dimensions, however, the disk is found to be a local minimizer and is conjectured to be the global minimizer. A numerical investigation of this problem is presented in \cite{li-zhu23}. Another operator that has recently attracted attention in a series of studies is the curl operator, see \cite{Cantarella_2000_Bis,Cantarella_2000},\cite{Gerner23}, \cite{enciso-peralta}, \cite{enciso2022optimal}. These studies also examine various properties of potential optimal domains, revealing that the ball does not minimize the first eigenvalue. In \cite{enciso-peralta}, the authors demonstrate, under certain regularity conditions, that an optimal domain cannot possess axial symmetry. Conversely, it is conjectured in \cite{Cantarella_2000_Bis} that the optimal domain is a {\it spheromak}, namely a torus in $\mathbb{R}^3$ with its central hole minimized to form an almost spherical shape, often likened to a cored apple. In the recent paper \cite{KLZ24}, the authors investigate the Maxwell operator (or vectorial Laplacian) with the boundary condition $ u \times \nu = 0$. They demonstrate that, in three dimensions, the ball is neither a minimizer nor a maximizer for the first eigenvalue under volume or perimeter constraints. Specifically, the authors show that the infimum of the first eigenvalue is zero, while the supremum is $ +\infty $ under both constraints. In this article, we focus on the Lamé system with Dirichlet boundary conditions, which is fundamental to the theory of linear elasticity. Let $\Omega$ be a bounded open set in $\R^N$ and $H^1_0(\Omega)^N$ denotes the space of vectors $u=(u_1, \ldots, u_N)$ where each $u_i$ belongs to the Sobolev space $H^1_0(\Omega)$. The first eigenvalue of $\Omega$ for the Lam\'e system is defined by \begin{equation}\label{defLambda} \Lambda(\Omega):=\min_{u\in H^1_0(\Omega)^N \setminus \{0\}} \frac{\mu\int_{\Omega}|\nabla u|^2 \;dx + (\lambda + \mu) \int_{\Omega} ({\rm div}(u))^2\;dx}{\int_{\Omega} |u|^2 \;dx}, \end{equation} where $\lambda , \mu$ are the Lam\'e coefficients that satisfy $\mu>0,\lambda+\mu >0$. In the previous expression, $|\nabla u|^2$ denotes $|\nabla u_1|^2+\ldots |\nabla u_N|^2$ and $|u|^2$ denotes $u_1^2+\ldots u_N^2$. The associated PDE solved by the minimizer $u$ is \begin{equation}\label{pdeLame0} \left\lbrace \begin{array}{cc} -\mu \Delta u -(\lambda+\mu) \nabla ({\rm div}(u)) = \Lambda u & \mbox{ in } \Omega,\\ u=0 & \mbox{ on } \partial\Omega .\\ \end{array} \right. \end{equation} We will explain below that a natural motivation for introducing the first eigenvalue $\Lambda$ arises from the famous Korn inequality. In that context, let us mention the paper \cite{Lewicka-Muller} where optimal constants for the Korn inequality are also computed but under tangential boundary conditions. It is worth noting that it will be convenient to introduce the Poisson coefficient $\nu$, as it is related to the Lam\'e parameters through the following formulae: \begin{equation}\label{lamepoi} \lambda=\frac{E\nu}{(1+\nu)(1-2\nu)},\qquad \mu=\frac{E}{2(1+\nu)} \end{equation} where $E$ is the Young modulus and $\nu \in (-1,0.5)$ (for many materials $\nu \in [0.2,0.4]$). Indeed, dividing the eigenvalue $\Lambda$ by $\mu$ lead us to introduce the ratio $$\frac{\lambda+\mu}{\mu}= \frac{1}{1-2\nu}$$ and therefore, we can see that the minimization of $\Lambda$ will primarily depend on the Poisson coefficient $\nu$. In some papers, like \cite{kawohl-sweers}, the authors look at an eigenvalue defined as \begin{equation}\label{defLambda2a} \Lambda(\Omega,a):=\min_{u\in H^1_0(\Omega)^N \setminus \{0\}} \frac{\int_{\Omega}|\nabla u|^2 \;dx + a \int_{\Omega} ({\rm div}(u))^2\;dx}{\int_{\Omega} |u|^2 \;dx}, \end{equation} where $a$ stands for $1/(1-2\nu)$. To make the underlying physics more apparent, we will explicitly retain the Lam\'e parameters and the Poisson coefficient in our paper. Thus, this paper is dedicated to the study of the following shape optimization problem: \begin{equation}\label{shop1} \boxed{ \inf\{ \Lambda(\Omega), \Omega\subset \R^N \mbox{ bounded }, |\Omega|=V_0\}} \end{equation} or equivalently (since $\Lambda(t\Omega)=\Lambda(\Omega)/t^2$) to the unconstrained optimization problem \begin{equation}\label{shop2} \inf\{|\Omega|^{2/N} \Lambda(\Omega), \Omega\subset \R^N \}. \end{equation} Here $|\Omega|$ denotes the Lebesgue measure of the open set (or quasi-open set) $\Omega$. The precise definition of quasi-open set will be given at the beginning of Section \ref{secexistence}. In Section \ref{secexistence}, we first establish an existence result for quasi-open sets in any dimension. To achieve this, we employ the standard strategy based on a concentration-compactness argument, that we had to adapt in a vectorial context. Subsequently, we demonstrate some mild regularity, specifically that the optimal domain is an open set in the physical dimensions 2 and 3. This is accomplished by proving the equivalence of the minimization problem with a penalized problem and introducing the concept of Lamé quasi-minimizers. We then show that these quasi-minimizers are globally H\"older continuous. Therefore our first important result is \begin{theorem}\label{theoexis} There exists a quasi-open set $\Omega^*$ solution of \eqref{shop1} or \eqref{shop2}. Moreover in dimension $N=2$ and $N=3$ this set is open and any eigenfunction associated with the eigenvalue $\Lambda(\Omega^*)$ belongs to $\mathscr{C}^{0,\alpha}(\R^N)$ for all $\alpha<1$ if $N=2$, and for all $\alpha<\frac{1}{2}$ if $N=3$. \end{theorem} In Section \ref{secoptim}, we derive first and second order optimality conditions by calculating the first and second shape derivatives of the eigenvalue. These computations prove to be particularly useful in the subsequent Section \ref{secdisk}, where we examine the potential optimality of the disk in two dimensions. In this context, we are able to prove: \begin{theorem} If the Poisson coefficient $\nu$ is less than $0.4$, the disk {\bf is not} the minimizer of $\Lambda$ (among sets of given area). \end{theorem} The proof involves several steps. First, we explicitly compute the first eigenvalue of the disk, which is a non-trivial task, and show that if $\nu \leq 0.349\ldots$, the first eigenvalue is double. This finding allows us, through a straightforward variational argument, to conclude that the disk cannot be optimal in this case. It is worth noting that the value $0.349\ldots$ is explicitly related to the first zero of the Bessel function $J_1$ and its derivative. Next, in Section \ref{secrhombi}, we identify explicit rhombi that yield a better first eigenvalue than the disk for the range $0.349\ldots \leq \nu \leq 0.3878\ldots$. We further extend our analysis by considering suitable rectangles in Section~\ref{secrectangle}. For these rectangles, we cannot compute the first eigenvalue explicitly, but we can provide precise estimates through a clever choice of test functions. This approach allows us to rule out the disk as a possible minimizer for $\nu \leq 0.4$. For values of $\nu$ between $0.4$ and $0.5$, we cannot reach a conclusion using analytical arguments. However, in Section \ref{seconddisk}, we demonstrate that once the first eigenvalue is simple (i.e., for $\nu > 0.349\ldots$), the disk serves as a local minimizer for our problem. This is established by showing that the second shape derivative is non-negative, and we can also estimate the quadratic form using the $H^1$-norm of the perturbation. Finally, we present heuristic arguments suggesting that there exists a threshold $\nu^*$ such that the disk could be a minimizer when $\nu^* \leq \nu < 0.5$. This conclusion is based on the property that the Lamé eigenvalue $\Gamma$-converges to the Stokes eigenvalue as $\nu \to 1/2$, in conjunction with the previously established local minimality of the disk. \section{Motivation and elementary comparisons} \subsection{Reminders on the Korn inequalities} Let $\Omega$ denote any bounded open set in $\R^N$. For $u:\Omega \subset \R^N \to \R^N$ we denote by $e(u)$ the symmetric gradient defined by $$e(u):=\frac{\nabla u+\nabla u^T}{2}.$$ Let us recall two standard Korn inequalities. \begin{theorem}\label{theokorn} For all $u \in H^1_0(\Omega)^N$, one has \begin{equation} \tag{\text{Korn}} \|\nabla u\|_{L^2(\Omega))}\leq 2 \| e(u)\|_{L^2(\Omega)}, \label{korn1} \end{equation} and \begin{equation}\tag{\text{Poincar\'e-Korn}} \|u\|_{L^2(\Omega))}\leq C(\Omega) \| e(u)\|_{L^2(\Omega)}. \label{korn2} \end{equation} Moreover we can take $C(\Omega)= 2/\lambda_1^D(\Omega)$, where $\lambda_1^D(\Omega)$ is the usual scalar first eigenvalue for the Dirichlet Laplacian. \end{theorem} \begin{proof} An elementary ``integration by parts'' reveals that the following identity is always true for any $u \in C^\infty_c(\Omega, \R^N)$, $$\int_{\Omega}|e(u)|^2 dx=\frac{1}{2}\left(\int_{\Omega} |\nabla u|^2 + ({\rm div}(u))^2\right).$$ Inequality \eqref{korn1} follows immediately. Notice that here the constant does not depend on $\Omega$. Now for a proof of \eqref{korn2}, we apply \eqref{korn1} and the following Poincaré inequality, for a scalar function $v \in H^1_0(\Omega))$: $$\lambda_1(\Omega)\int_{\Omega} v^2 \; dx \leq \int_{\Omega} |\nabla v|^2 \; dx.$$ We deduce that, for $u=(u_1,u_2,\ldots u_N)$: \begin{eqnarray} \frac{\int_{\Omega} |e(u)|^2 \;dx }{\int_{\Omega} u^2 \;dx}&=&\frac{\frac{1}{2}\left(\int_{\Omega} |\nabla u|^2 + ({\rm div}(u))^2\right)}{\int_{\Omega}\sum_{i=1}^N (u_i)^2 } \geq \frac{\frac{1}{2}\left(\int_{\Omega} \sum_{i=1}^N |\nabla u_i|^2 \right)}{\int_{\Omega} \sum_{i=1}^N (u_i)^2} \notag \\ &\geq & \min_{i=1,2,\ldots N} \left( \frac{\frac{1}{2}\int_{\Omega} |\nabla u_i|^2}{\int_{\Omega}(u_i)^2} \right) \geq \frac{1}{2}\lambda_1(\Omega). \notag \end{eqnarray} \end{proof} Therefore, looking at the best constant in the \eqref{korn2} leads us to compute the eigenvalue $\Lambda$ defined in \eqref{defLambda} for the particular choice $\mu=1/2$ and $\lambda=0$. \subsection{Link with other eigenvalues} \subsubsection{Link with the eigenvalues of the Stokes operator}\label{remark1} For $\Omega$, a bounded open set, let us introduce the so-called {\it Dirichlet Stokes first eigenvalue} $\lambda_1^{\rm Stokes}(\Omega)$ by $$ \lambda_1^{\rm Stokes}(\Omega):= \inf_{\substack{u \in (H^1_0(\Omega))^N\setminus \{0\} \\ {\rm div}(u)=0\text{ in }\Omega}} \frac{\int_{\Omega} |\nabla u|^2 } {\int_{\Omega} |u|^2 }. $$ Then for all $\Omega$ it holds \begin{equation}\label{comp:StKorn} \mu \lambda_1^{\rm Stokes}(\Omega) \geq \Lambda(\Omega). \end{equation} Indeed this inequality comes from the fact that the space on which we minimize is a subspace of $H^1_0(\Omega)^N$ on which the energy coincides with our Rayleigh quotient. In some sense, the divergence term may appear as a penalization term, in particular when the Poisson coefficient goes to $1/2$ (or equivalently when the Lam\'e coefficients are such that $(\lambda + \mu)/\mu \to +\infty$). We will make this more precise in Section \ref{secconclusion} by proving the strong convergence of the Lam\'e operator to the Stokes operator when $\nu \to 1/2$. For that purpose, we will use the tool of $\Gamma$-convergence. \subsubsection{Comparison with Dirichlet Eigenvalues} In this section we retrieve some results that already appeared, for example in \cite{kawohl-sweers}. We recall that $\lambda_1^D(\Omega)$ denotes the first eigenvalue of the Dirichlet-Laplacian. \begin{proposition} \label{boundlambda1} For any bounded domain $\Omega$ it holds \begin{eqnarray} \mu \lambda_1^D(\Omega) <\Lambda(\Omega) \leq \frac{(\lambda + (N+1)\mu)}{N} \, \lambda_1^D(\Omega).\label{ineqlambda1} \end{eqnarray} Moreover, $$\inf_{\Omega} \frac{\Lambda(\Omega)}{\lambda_1^D(\Omega)}=\mu ,$$ and is achieved by a sequence of thin cuboids shrinking to a line. \end{proposition} \begin{remark} From the left inequality in \eqref{ineqlambda1} and the famous Faber-Krahn inequality, we observe that $$|\Omega|^{2/N} \Lambda(\Omega) \geq \mu \, |\Omega|^{2/N} \lambda_1^D(\Omega) \geq \mu j_{N/2-1,1}^2$$ where $j_{N/2-1,1}$ is the first zero of the Bessel function $J_{N/2-1}$. Thus we see that the infimum in \eqref{shop2} is strictly positive. \end{remark} \begin{proof} The inequality $$\mu \lambda_1^D(\Omega) \leq \Lambda(\Omega)$$ follows the chain of inequalities that appear in the proof of Theorem \ref{theokorn} (multiplied by $\mu$ instead of $1/2$). Now we demonstrate that the inequality must be strict. Indeed, assuming that $\mu \lambda_1^D(\Omega) = \Lambda(\Omega)$ and applying the previously established chain of inequalities, we observe that each $u_i$ must be a Dirichlet eigenfunction associated with $\lambda_1^D(\Omega)$. But since we also have ${\rm div}(u)=0$ we will get a contradiction according to Lemma \ref{div0} below, and this achieves the proof of the strict inequality. We now prove the upper bound. For that purpose we consider $u_1$ being the (normalized) Dirichlet eigenfunction associated to $\lambda_1(\Omega)$ and we consider the vector test functions $(0,0,\ldots,u_1,0,\ldots)$ composed of null functions except $u_1$ in $i$-th position. Then $$\Lambda\leq \frac{\mu \int_{\Omega}|\nabla u_1|^2\;dx + (\lambda +\mu)\int_{\Omega} (\partial_i u_1)^2 \;dx}{\int_{\Omega} u_1^2 \;dx}.$$ Summing these $N$ inequalities yields $$N \Lambda \leq N\mu \int_{\Omega}|\nabla u_1|^2\;dx +(\lambda +\mu) \int_{\Omega}|\nabla u_1|^2\;dx =(\lambda + (N+1)\mu)\lambda_1^{D},$$ which finishes the proof of \eqref{ineqlambda1}. Let us now prove the last assertion. For that purpose, we consider the cuboid $\Omega_L=(0,L)\times \prod_{i=2}^N (0,1)$ and take a first Dirichlet eigenfunction of $\Omega_L$ namely $$u_1(X)=\sin\left(\pi \frac{x_1}{L}\right) \prod_{i=2}^N \sin(\pi x_i).$$ We will use the fact that $$\lambda_1^D(\Omega)=\pi^2 (N-1+\frac{1}{L^2}).$$ Now, we plug in the Rayleigh quotient defining $\Lambda(\Omega_L)$ the vector $u=(u_1,0,\ldots,0)$. Since $$\int_{\Omega_L} |u|^2 \;dx=\frac{L}{2^N}$$ $$\int_{\Omega_L}|\nabla u|^2 \; dx= \int_{\Omega_L}|\nabla u_1|^2 \; dx=\frac{L \pi^2}{2^N}(N-1+\frac{1}{L^2})= \frac{L}{2^N} \lambda_1^D(\Omega_L$$ $$\int_{\Omega_L}({\rm div}(u))^2 \; dx =\frac{ \pi^2}{L^2}\frac{L}{2^N}$$ we deduce that \begin{eqnarray} \Lambda (\Omega_L)\leq \frac{\mu \frac{L}{2^N} \lambda_1^D(\Omega_L) + (\lambda+\mu) \frac{L}{2^N} \frac{\pi^2}{L^2}}{\frac{L}{2^N}} \notag \end{eqnarray} or $$\Lambda(\Omega_L) \leq \mu \lambda_1^D(\Omega_L) + (\lambda+\mu) \frac{L}{2^N} \frac{\pi^2}{L^2}$$ and finally letting $L\to +\infty$ we conclude that $$\inf_{\Omega} \frac{\Lambda(\Omega)}{\lambda_1^D(\Omega)}=\mu,$$ as claimed in the proposition. \end{proof} \begin{lemma}\label{div0} Let $u=(u_1,u_2,\ldots, u_N)$ be a $N$-uple of functions in $H^1_0(\Omega)$ such that ${\rm div}(u)=0$ and for all $i$, $u_i=\alpha_i u_1$ for some $\alpha_i \in \R$. Then $u_1$ and then all the $u_i$ are identically zero.\\ In particular, if $\Omega$ is connected and all the $u_i$ are eigenfunctions associated to the first eigenvalue $\lambda_1^D(\Omega)$, it is not possible that ${\rm div}(u)=0$. \end{lemma} \begin{proof} From the assumptions ${\rm div}(u)=0$ and $u_i=\alpha_i u_1$ we deduce that $u_1$ satisfies $$\frac{\partial u_1}{\partial x_1} + \sum_{i=2}^N \alpha_i \frac{\partial u_1}{\partial x_i} =0$$ which means that $u_1$ must be constant on all affine lines directed by $(1,\alpha_2,\ldots,\alpha_N)$. Since all those lines touches the boundary of $\Omega$, from the Dirichlet condition on $u_1$ we deduce that $u_1$ must be identically $0$.\\ The last assertion comes from the fact that the first Dirichlet eigenvalue (for the Laplacian) of a connected domain is simple. \end{proof} \section{Existence and regularity }\label{secexistence} \subsection{Existence of an optimal quasi-open set} In this section, we will fix the values of the Lamé coefficients, specifically choosing $\mu = 1/2$ and $\lambda = 0$, which corresponds to the Korn inequality. This choice does not affect the proof of existence (since the general case would simply involve multiplying the terms by positive constants), but it simplifies the proof and enhances its clarity. We establish the existence of an optimal shape within the class of quasi-open sets, employing a standard concentration-compactness strategy first introduced by Lions \cite{lions}. This approach has been utilized to solve shape optimization problems for the Laplace operator, initially by Dorin Bucur (see \cite{dorin1,dorin2,dorin3,depveli}), and subsequently by several other authors. Recently, this strategy has also been applied to the Stokes operator \cite{henrot-mazari-privat}. We denote by $\operatorname{Cap}(A)$ the $H^1$-capacity of $A$ (for instance the Bessel capacity). A set $A\subset \R^N$ is said to be quasi-open if, for every $\varepsilon>0$ there exists an open set $\Omega_\varepsilon$ such that $A\subset \Omega_{\varepsilon}$ and $\operatorname{Cap}(\Omega_{\varepsilon}\setminus A)\leq \varepsilon$. We first introduce the class $$\mathcal{O}:=\{ \Omega \subset \R^N \text{ quasi-open such that } 0<|\Omega|< +\infty \}.$$ The space $H^1_0(\Omega)$ is defined as functions $u \in H^1(\R^N)$ such that $u=0$ quasi-everywhere on $\Omega^c$. Notice that a domain $\Omega \in \mathcal{O}$ is not necessarily bounded. However, the space $H^1_0(\Omega)$ is known to be a closed subspace of $H^1(\R^N)$ which is compactly embedded into $L^2(\R^N)$, when $|\Omega|<\infty$ (because by definition of being quasi-open there exists an open set $E$ with $|E|<+\infty$ such that $\Omega \subset E$ thus $H^1_0(\Omega)\subset H^1_0(E)$ and the standard compact embedding of $H^1_0(E)$ into $L^2$ applies). Notice also that thanks to Proposition \ref{remarkK} below, the space of all $u \in L^2(\Omega)^N$ such that $e(u)\in L^2(\Omega)$ and $u=0$ $\operatorname{Cap}_{1,2}$-q.e. in $\Omega^c$ coincides with the space $H^1_0(\Omega)^N$. Then we can relax the definition of $\Lambda(\Omega)$ for $\Omega \in \mathcal{O}$ by considering $$\Lambda(\Omega):= \min_{u \in H^1_0(\Omega)^N} \frac{\int_{\R^N} |e(u)|^2 \; dx}{\int_{\R^N}|u|^2 \;dx}.$$ Notice here that $\Omega$ is merely quasi-open and not necessarily open, but the definition coincides with the standard one when $\Omega$ is open. Also, it is easy to check that the minimum in the definition of $\Lambda$ is achieved by an $H^1_0(\Omega)$ function, thanks to the compact embedding of $H^1_0(\Omega)$ into $L^2(\R^N)$ and the semicontinuity behavior of the convex functional $u\mapsto \int_{\R^N} |e(u)|^2 \; dx$ for the weak topology of $H^1$. In the sequel we will need the famous Korn inequality but now in the whole $\R^N$, in particular valid for $u\in H^1_0(\Omega)^N$ with $\Omega \in \mathcal{O}$ quasi-open, as stated in the following proposition. \begin{proposition}[Korn inequality in $\R^N$] \label{remarkK}If $u \in L^2(\R^N)^N$ is such that $e(u)\in L^2(\R^N)$, then $u\in H^1(\R^N)^N$ and \begin{eqnarray} 2\int_{\R^N } |e(u)|^2 \;dx = \int_{\R^N } |\nabla u|^2 +({\rm div} (u))^2 \; dx. \label{kornkornkorn} \end{eqnarray} \end{proposition} \begin{proof} Let $R>0$ be given and let $\varphi_{ R} \in [0,1]$ be a cut-off function such that $\varphi_{\lambda, R}=1$ on $B(0,R)$, $\varphi_{\lambda,R}=0$ on $B(0,2 R)^c$, and $$|\nabla \varphi_{\lambda,R}|\leq C\frac{1}{R}.$$ Then the function $u_R:=\varphi_{R} u$ clearly belongs to $H^1_0(B(0,2 R))^N$ and Korn's inequality \eqref{kornkornkorn} holds true for the function $\varphi_{R} u$. For simplicity we will by now denote by $\varphi$ the function $\varphi_{R}$. Notice that $$ e(\varphi u)_{ij}=\frac{u^i\partial_j\varphi +u^j\partial_i\varphi }{2}+\varphi e(u)_{i,j}, $$ so pointwisely in $\mathbb{R}^N$ it holds the following estimate $$|e(u \varphi )| \leq |u| |\nabla \varphi | + |\varphi | |e(u)|\leq \frac{C}{ R} |u| + |e(u)|,$$ $$|{\rm div}(u \varphi)|\leq \frac{C}{ R} |u| + |{\rm div}( u)|,$$ $$|D (u\varphi)|\leq \frac{C}{R} |u| + |D(u)|.$$ Recall also that $u\varphi = u$ in $B(0,R)$. Now applying \eqref{kornkornkorn} in $B(0, R)$ to the function $\varphi u$ we obtain \begin{eqnarray} 2\int_{\R^N\cap B(0,R)} |e(u)|^2 \;dx = \int_{\R^N \cap B(0,R)} |\nabla u|^2 +({\rm div} (u))^2 \; dx +E(R), \label{aap} \end{eqnarray} with $$|E(R)| \leq \frac{C}{R^2} \int_{\Omega \setminus B(0,2 R)} |u|^2 \;dx.$$ We now let $R\to +\infty$ which yields, $$|E(R)| \underset{R\to +\infty}{\longrightarrow} 0.$$ Thus passing \eqref{aap} to the limit, and using that $e(u) \in L^2(\R^N)$ we can use Fatou lemma to get first $\nabla u \in L^2(\R^N)$, after which the monotone convergence theorem allows to conclude \begin{eqnarray} 2\int_{\R^N } |e(u)|^2 \;dx = \int_{\R^N } |\nabla u|^2 +({\rm div} (u))^2 \; dx. \notag \end{eqnarray} This proves that $u \in H^1(\R^N)^N$ and finishes the proof. \end{proof} The purpose of this section is to prove the following result. \begin{theorem}{\label{existence} } For all $V>0$ there exists a solution for the problem $$\min_{\Omega \in \mathcal{O} \; \text{ such that }\; |\Omega| \leq V} \Lambda(\Omega).$$ \end{theorem} \begin{proof} The proof follows the same approach as in \cite{henrot-mazari-privat} reasoning on the scalar function $|u|$ and using the concentration-compactness strategy of Lions \cite{lions}. More precisely, we let $\Omega_k$ be a minimizing sequence with $|\Omega_k|\leq V$ and we consider $w_k:=|u_k|$ where $u_k$ is a chosen normalized eigenvalue for $\Lambda(\Omega_k)$. In other words, $\|w_k\|_{L^2(\R^N)}=1$ and by Proposition~\ref{remarkK}, \begin{eqnarray} \int_{\R^N} |\nabla (w_k)|^2 \;dx \leq \int_{\R^N} |\nabla u_k|^2 \; dx \leq 2\int_{\R^N} |e(u_k)|^2 \;dx = 2\Lambda(\Omega_k)\leq C_0, \label{boundD} \end{eqnarray} so that $w_k$ is uniformly bounded in $H^1(\R^N)$. Let $Q_n:\R^+\to \R^+$ be the sequence of concentration functions\footnote{According to Lions \cite{lions} this notion was first introduced by L\'evy.} defined by $$Q_n(R):= \sup_{y\in \R^N}\int_{B(y,R)}|u_k|^2 \;dx.$$ Then $Q_n$ is a sequence of nondecreasing functions on $\R^n$ which are uniformly bounded by $1$. By Dini's theorem, up to extract a subsequence (not relabelled), $(Q_n)_n$ admits a pointwize limit function which is nondecreasing, bounded by 1, and that we denote $Q:\R^+ \to \R^+$. Then we let $$\alpha:=\lim_{R\to +\infty} Q(R) \in [0,1].$$ The value of $\alpha$ is usually referred to the ``maximal concentration''. Depending on the value of $\alpha$, we know that one of the following occurs by the concentration-compactness principle of Lions \cite[Lemma I.1]{lions}. \begin{itemize} \item {\bf If $\alpha=1$: Compactness:} There exists a sequence $(y_k)_{k \in \mathbb{N}}$ such that $|w_k|^2(\cdot-y_k)$ is tight: $$\forall \varepsilon >0, \exists R< +\infty , \forall k \quad \int_{y_k+B_R} w_k^2 \;dx\geq 1- \varepsilon.$$ \item {\bf If $\alpha \in (0,1)$: Dichotomy:} There exist $(y_k)_{k \in \mathbb{N}}$ and two sequences of positive radii $(R_k)_{k}$, $(R_k')_k$ satisfying $$R'_k - R_k \to +\infty \text{ and } R_k,R_k'\to +\infty,$$ and such that \begin{eqnarray} \int_{B(y_k,R_k)} w_k^2 \to \alpha, \text{ } \int_{B(y_k,R_k')^c} w_k^2 \to 1-\alpha. \label{information1} \end{eqnarray} \item {\bf If $\alpha=0$: Vanishing.} For every $R>0$, $$\lim_{k\to +\infty} \sup_{y \in \R^N} \int_{B(y,R)} w_k^2 =0.$$ \end{itemize} As usual, our aim is to prove that only the compactness case can occur, by ruling out the two other cases. Let us first prove that the compactness situation implies the desired existence. \medskip \noindent{\bf Step 1.} \emph{Compactness implies existence.} We consider the sequence of translated functions $u_k(\cdot -y_k)$ that we still denote by $u_k$. We know by assumption that this sequence is uniformly bounded in $H^1(\R^N)$ thus admits a weakly converging subsequence. Since $H^1(\R^N)$ is compactly embedded in $L^2_{loc}(\R^N)$, using a diagonal argument we can extract a subsequence (not relabelled) and a function $u \in L^2_{loc}(\R^N)$ such that $u_k\to u$ strongly in $L^2_{loc}$ and weakly in $H^1(\R^N)$. Now we use that $(w_k)$ is in the situation of compactness, and in particular for every $\varepsilon >0$ there exists $R>0$ such that $$\int_{B_R} |u_k|^2 \;dx\geq 1 - \varepsilon.$$ Passing to the limit and using the convergence of $u_k$ in $L^2(B_R)$ we deduce that $\int_{B_R} |u|^2\; dx \geq \; 1-\varepsilon$, which means in particular that $$\int_{\R^N} |u|^2\; dx \geq \; 1-\varepsilon,$$ and since $\varepsilon$ is arbitrary, we finally get $\int_{\R^N} |u|^2\; dx \geq \; 1$. But of course the reverse is also true so in conclusion $\|u\|_{L^2(\R^N)}=1$. But we already knew that $u_k$ was converging weakly in $L^2(\R^N)$ to $u$. We just have proved that the sequence of norms are also converging so finally $u_k$ converges strongly in $L^2(\R^N)$ to $u$. Passing to the limit in the Rayleigh quotient, strongly in $L^2$ for $u_k$ and weakly in $L^2$ for $e(u_k)$ we deduce that \begin{eqnarray} \inf_{\Omega \in \mathcal{O} \; \text{ such that }\; |\Omega| \leq V} \Lambda(\Omega)= \frac{\int_{\R^N} |e(u)|^2 \;dx}{\int_{\R^N} |u|^2 \;dx}. \label{infimum} \end{eqnarray} Let us denote $\Omega=\{|u|>0\}$, which is a quasi-open set, and from the equality in \eqref{infimum} we know that $u$ must be an eigenfunction associated to $\Lambda(\Omega)$. Furthermore, since $\int_{\R^N} |u|^2 \;dx=1$ we know that $|\Omega|>0$. We can also assume that $|u_k|$ converges a.e. in $\R^N$ to $|u|$. This implies, for a.e. $x\in \R^N$, $$\mathds{1}_{\{|u|>0\}}(x) \leq \liminf_{k}\mathds{1}_{\{|u_k|>0\}}(x),$$ and since $|\{|u_k|>0\}|\leq V$, we deduce by Fatou Lemma that $|\Omega|\leq V$ and finally $\Omega$ is a solution. \medskip \noindent{\bf Step 2.} \emph{Vanishing does not occur.} This case is easy to exclude by standard arguments. Indeed, Lemma 3.3 in \cite{zbMATH01482128} says that up to a subsequence, $w_k(\cdot +y_k)$ does not weakly converge in $H^1(\mathbb{R}^N)$, which is a contradiction with the uniform bound in \eqref{boundD} together with the fact that $\|w_k\|_2=1$, implying that $w_k$ is uniformly bounded in $H^1(\mathbb{R}^N)$ thus admits a weakly converging subsequence. \medskip \noindent{\bf Step 3.} \emph{Dichotomy cannot occur.} Assume that $(w_k)_{k\in \mathbb{N}}$ is in the dichotomy situation. Then the idea is to split the minimizing sequence in two disjoint pieces. For that purpose we define $\eta_k:=(R_k'-R_k)/4$ and then we construct two cut-off functions: the first one $\varphi_{k,1}$ supported in $B(y_k,R_k+2\eta_k)$ is such that $\varphi_{k,1}=1$ in $B(y_k,R_k)$, and the second one $\varphi_{k,2}$ equal to 1 in $B(y_k,R_k'-\eta_n)^c$ and $0$ on $B(y_k,R'_k-2\eta_k)$ satisfying $$|\nabla \varphi_{k,1}|+|\nabla \varphi_{k,2}|\leq 1/\eta_k \to 0.$$ Next, we define $$v_{k,1}=\varphi_{k,1} u_k \quad \text{ and } \quad v_{k,2}=\varphi_{k,2} u_k.$$ We want to prove that the sum $v_{k,1}+v_{k,2}$ has almost the same $L^2$ norm as the original function $u_k$ because $w_k$ is in a dichotomy situation. Let us define the annulus $A_k:=B(y_k,R'_k)\setminus B(y_k,R_k)$. Because of \eqref{information1} and the fact that $\|w_k\|_2=1$ for all $k$, we directly get $$\int_{A_k} |w_k|^2 \;dx \to 0,$$ and since $|u_k|=|w_k|$ we also have for $i=1,2$, $$\int_{A_k} |u_k \varphi_{k,i}|^2 \;dx \leq \int_{A_k} |u_k |^2 \;dx= \int_{A_k} |w_k |^2 \;dx \to 0.$$ We deduce that \begin{eqnarray} \int_{\mathbb{R}^N}|v_{k,1}|^2 \;dx =\int_{B(y_k,R_k)}|w_{k}|^2 \;dx + \int_{A_k} |u_k \varphi_{k,1}|^2 \to \alpha_1 \label{inin1}\\ \int_{\mathbb{R}^N}|v_{k,2}|^2 \;dx =\int_{B(y_k,R_k')}|w_{k}|^2 \;dx + \int_{A_k} |u_k \varphi_{k,2}|^2 \to 1-\alpha_1.\label{inin2} \end{eqnarray} Then we want to estimate the difference of the symmetrized gradients. For that purpose we compute $e(\varphi u)$ as follows. From the identity $$\partial_j (\varphi u^i)= u^i\partial_j\varphi +\varphi \partial_j u^i,$$ we get \begin{eqnarray} e(\varphi u)_{ij}&=& \frac{ u^i\partial_j\varphi +\varphi \partial_j u^i}{2}+\frac{ u^j\partial_i\varphi +\varphi \partial_i u^j}{2} \notag \\ &=& \frac{u^i\partial_j\varphi +u^j\partial_i\varphi }{2}+\varphi e(u)_{i,j}. \end{eqnarray} Therefore, pointwisely in $\mathbb{R}^N$ it holds the following estimate $$|e(u_k \varphi_{k,1})| \leq |u| |\nabla \varphi_{k,1}| + |\varphi_{k,1}||e(u_k)|\leq \frac{1}{\eta_k} |u| + |e(u_{k})|$$ and the same holds true for $v_{k,2}$, $$|e(u_k \varphi_{k,2})| \leq \frac{1}{\eta_k} |u| + |e(u_{k})|.$$ Taking the square we get, for $i=1,2$, \begin{eqnarray} |e(v_{k,i})|^2 \leq \frac{1}{\eta_k^2} |u_k|^2 + 2\frac{1}{\eta_k}|u_k||e(u_k)|+ |e(u_{k})|^2. \label{corona} \end{eqnarray} Now remember that $v_{k,1}$ and $v_{k,2}$ have disjoint support, and that their sum coincide with $u_k$ outside $A_k$, in which we can use \eqref{corona} to estimate \begin{eqnarray} \int_{\mathbb{R}^N} |e(u_k)|^2 \; dx - \int_{\mathbb{R}^N} |e(v_{k,1})|^2 \; dx &-& \int_{\mathbb{R}^N} |e(v_{k,2})|^2 \;dx \notag \\ &\geq& -2\int_{A_k} \frac{1}{\eta_k^2} |u_k|^2 + 2\frac{1}{\eta_k}|u_k||e(u_k)| \;dx. \end{eqnarray} Since $\frac{1}{\eta_k}\to 0$ and both $u_k$ and $e(u_k)$ are uniformly bounded in $L^2$, we deduce that the term on the right-hand side converges to zero thus \begin{eqnarray} \liminf_{k \to +\infty}\int_{\mathbb{R}^N} |e(u_k)|^2 \; dx - \int_{\mathbb{R}^N} |e(v_{k,1})|^2 \; dx- \int_{\mathbb{R}^N} |e(v_{k,2})|^2 \;dx\geq 0. \label{liminf} \end{eqnarray} This allows to compare the Rayleigh quotient of $u_k$ with the one of $v_{k,1}+v_{k,2}$. More precisely, using \eqref{liminf}, the standard inequality on real nonnegative numbers $a$, $b$ and positive numbers $c$, $d$, $$\frac{a+b}{c+d}\geq \min\left\{\frac{a}{c}, \frac{b}{d}\right\},$$ and also \eqref{inin1} and \eqref{inin2}, we obtain \begin{eqnarray} \lambda^*:=\inf_{|\Omega|\leq V}\Lambda(\Omega)&=&\lim_{k\to +\infty}\int_{\R^N} |e(u_k)|^2 \;dx \notag \\ &\geq& \liminf \int_{\mathbb{R}^N} |e(v_{k,1})|^2 \;dx+ \int_{\mathbb{R}^N} |e(v_{k,2})|^2 \;dx \notag \\ &=&\liminf \frac{\int_{\mathbb{R}^N} |e(v_{k,1})|^2+ \int_{\mathbb{R}^N} |e(v_{k,2})|^2}{\int_{\mathbb{R}^N} |v_{k,1}|^2 \; dx+ \int_{\mathbb{R}^N} |v_{k,2}|^2 \; dx} \label{return} \\ &\geq & \min \left\{ \liminf \frac{\int_{\mathbb{R}^N} |e(v_{k,1})|^2}{\int_{\mathbb{R}^N} |v_{k,1}|^2 \; dx} \;, \;\liminf \frac{ \int_{\mathbb{R}^N} |e(v_{k,2})|^2}{ \int_{\mathbb{R}^N} |v_{k,2}|^2 \; dx} \right\}. \label{yannickQ} \end{eqnarray} Notice that applying the concentration principle on the sequence $v_k^1$, we obtain that $v_k^1$ is in the compactness situation, with concentration value $\alpha$. In particular, arguing as in the compactness case, we can assume that $v_k^1$ converges strongly in $L^2$ (and weakly in $H^1$) to a function $v\in H^1(\mathbb{R}^N)$. Then, if the minimum above is achieved for $v_{k,1}$, we deduce that the quasi-open set $\Omega^*= \{|v| > 0\}$ is an optimal domain, and the proof is concluded from the compactness situation. So we have to consider that it is not the case. But then it means that $v$, being the $L^2$ limit of $v_k^1$, satisfies $$\frac{\int_{\R^N} |\nabla v|^2 \;dx}{\int_{\R^N} |v|^2 \;dx} > \lambda^*,$$ or differently, $$ \int_{\R^N} |\nabla v|^2 \;dx> \alpha \lambda^*.$$ We also know that $$\liminf \frac{ \int_{\mathbb{R}^N} |e(v_{k,2})|^2}{ \int_{\mathbb{R}^N} |v_{k,2}|^2 \; dx}=\lambda^*,$$ because by assumption the minimum in \eqref{yannickQ} is achieved with the sequence $v_{k,2}$, and since $\lim_{k\to +\infty} \int_{\mathbb{R}^N} |v_{k,2}|^2 \; dx=1-\alpha$ we deduce that $$\liminf_{k\to +\infty} \int_{\mathbb{R}^N} |e(v_{k,2})|^2 = (1-\alpha)\lambda^*.$$ Now returning back to \eqref{return}, we have obtained $$\lambda^*\geq \liminf_{k\to +\infty} \frac{\int_{\mathbb{R}^N} |e(v_{k,1})|^2+ \int_{\mathbb{R}^N} |e(v_{k,2})|^2}{\int_{\mathbb{R}^N} |v_{k,1}|^2 \; dx+ \int_{\mathbb{R}^N} |v_{k,2}|^2 \; dx} =\frac{ \int_{\R^N} |\nabla v|^2 \;dx + \liminf_{k\to +\infty} \int_{\mathbb{R}^N} |e(v_{k,2})|^2}{\alpha + (1-\alpha)}>\lambda^*,$$ a contradiction. This achieves the proof of the Theorem. \end{proof} \subsection{Regularity} The purpose of this section is to prove that any quasi-open solution of our shape optimisation problem, is actually an open set. We will achieve this conclusion only for $N=2$ or $N=3$. The reason is that we need an apriori $L^p$ bound on an eigenfunction which, up to our knowledge, is not know in any dimension (see also Remark \ref{bound}) below. Here is a general regularity result valid in any dimension. \begin{theorem} \label{open} Let $\Omega^* \subset \R^N$ be a quasi-open solution to the problem $$\min_{\Omega \in \mathcal{O} \; \text{ such that }\; |\Omega| \leq V} \Lambda(\Omega).$$ Assume moreover that $u \in L^p(\R^N)$ with $p >N$. Then $u\in \mathscr{C}^{0,\alpha}(\R^N)$, for all $\alpha<1-\frac{N}{p}$. As a consequence, $\Omega^*$ is an open set. \end{theorem} In particular in dimension $N=2$ and $N=3$ we obtain the following. \begin{corollary} Assume that the dimension $N=2$ or $N=3$. Then for all $V>0$ there exists an open solution $\Omega^*$ for the problem $$\min \left\{ \Lambda(\Omega) \; , \; \Omega\subset \R^N \text{ open set such that }\; |\Omega| \leq V \right\}.$$ Moreover, any eigenfunction associated with $\Lambda(\Omega^*)$ belongs to $\mathscr{C}^{0,\alpha}(\R^N)$ for all $\alpha<1$ if $N=2$ and for all $\alpha<\frac{1}{2}$ if $N=3$. \end{corollary} \begin{proof} Let $u$ be an eigenfunction associated with $\Lambda(\Omega^*)$. Since $u\in H^1(\R^N)$, by the Sobolev embedding we know that $u \in L^{p}(\R^N)$ with $p$ arbitrary large for $N=2$ or $p=2^*=\frac{2N}{N-2}$ for $N>2$. Then by Proposition \ref{quasiminimal} below we know that $u$ is a Lam\'e quasi-minimizer with exponent $\gamma=N-\frac{2N}{p}$. To conclude that $u\in \mathscr{C}^{0,\alpha}$ we would need that $p>N$. This is true for $N=2$ or $N=3$. For $N=2$ we deduce from Proposition \ref{aga} that $u \in \mathscr{C}^{0,\alpha}$ for all $\alpha<1$. If $N=3$ we deduce, from Proposition \ref{aga}, that $u \in \mathscr{C}^{0,\alpha}$ for all $\alpha <\frac{1}{2}$. \end{proof} \begin{remark} \label{bound} Let us stress that the conclusion of Theorem \ref{open} does not imply that $u\in L^\infty(\R^N)$. In other words by $u\in \mathscr{C}^{0,\alpha}(\R^N)$ we mean, that for a representative of $u$ it holds $|u(x)-u(y)|\leq C|x-y|^\alpha$ which is enough to prove that $u$ is continuous. Since $\Omega^*$ may not be a bounded set, we do not conclude that $u$ is bounded. Let us mention that in the scalar case it is well known that any eigenfunction associated to the first eigenvalue of the Dirichlet Laplacian belongs to $L^\infty(\R^N)$ together with the following nice bound, for which one usually refers to \cite[Example 2.1.8]{zbMATH00194234}: $$\|u\|_{L^\infty}\leq e^{\frac{1}{8\pi}}\lambda_1^D(\Omega)^{\frac{N}{4}} \|u\|_2.$$ It would be very interesting to know whether a similar bound is true for the Lam\'e eigenfunctions. This would directly imply the existence of an open solution in any dimension. \end{remark} The strategy of proof for Theorem \ref{open} is inspired by the seminal paper of Brian\c{c}on, Hayouni and Pierre \cite{briancon}, also declined later in different directions, see for instance \cite{lamboleybri,mazzoleni}. The general approach involves showing that a solution to the original problem is also a solution to a penalized version of the problem. We then exploit the regularity theory for free-boundary-type problems to conclude that the eigenfunction is globally Hölder continuous. In our case, however, this strategy requires a non-trivial adaptation due to the presence of the symmetrized gradient. For instance in \cite{briancon}, the first step is to use a truncated test function and the co-area formula, which are not available for the symmetric gradient, thus in our context we have to argue differently from \cite{briancon}. In particular we are not able to arrive up to Lipschitz regularity but merely continuous regularity, which is enough to conclude that the optimal set is open.\subsubsection{Equivalence with a penalized problem} \begin{proposition} \label{penalization}Let $V>0$ be given and $u$ be a solution for the problem \begin{eqnarray} \lambda_V:=\min \left\{ \int_{\R^N}|e(u)|^2 dx \quad {\text s.t. } \quad u \in H^1(\R^N), \; \int_{\R^N} |u|^2=1, \;\text{ and } \; |\{|u|>0\}|\leq V \right\}. \label{problemu} \end{eqnarray} Then for all $\lambda>\frac{\lambda_V}{V}$, and all $v\in H^1(\R^N)$ we have \begin{eqnarray} \int_{\R^N}|e(u)|^2 \; dx \leq \int_{\R^N} |e(v)|^2 dx +\lambda_V\left(1-\int_{\R^N} |v|^2\right)^+ + \lambda \Big( |\{|v|>0\}|-V\Big)^+. \label{problemlam} \end{eqnarray} \end{proposition} \begin{proof}For $v\in H^1(\R^N)$ and $\lambda>0$ we introduce $$F_\lambda(v):=\int_{\R^N} |e(v)|^2 dx +\lambda_V\left(1-\int_{\R^N} |v|^2\right)^+ + \lambda\Big( |\{|v|>0\}|-V\Big)^+.$$ We first notice that, arguing as in Theorem \ref{existence}, $F_\lambda$ admits a minimizer $u_\lambda \in H^1(\R^N)$. Our next goal is to prove that for $\lambda$ large enough, then $|\{|u_\lambda|>0\}|\leq V$. Assume for a contradiction that $|\{|u_\lambda|>0\}|> V$. In the sequel we will write $\Omega:=\{|u_\lambda|>0\}$. Then we compare $u_\lambda$ with the competitor $v:= u_\lambda(t x)$ with the choice $$t:=\left(\frac{|\Omega|}{V}\right)^{\frac{1}{N}}\geq 1.$$ Since $u_\lambda \in H^1_0(\Omega)$, it follows that $v\in H^1_0(\frac{1}{t}\Omega)$. Moreover, $$|\{|v|>0\}|=|\frac{1}{t}\Omega|=\frac{1}{t^N}|\Omega|=V.$$ Next, we use that $u_\lambda$ is a minimizer thus $$F_{\lambda}(u_\lambda) \leq F_\lambda(v),$$ which yields in particular, \begin{eqnarray} \int_{\R^N}|e(u_\lambda)|^2 \;dx + \lambda\big( |\{|u_\lambda|>0\}|-V\big)^+\leq F_\lambda(u_\lambda)\leq \int_{\Omega} |e(v)|^2 \; dx +\lambda_V\left(1-\int_{\R^N} |v|^2\right)^+ \label{test1} \end{eqnarray} because $$\big( |\{|v|>0\}|-V\big)^+=0.$$ Now notice that $$\int_{\Omega} |e(v)|^2 \; dx =t^{2-N} \int_{\R^N} |e(u_\lambda)|^2 \;dx= \left(\frac{|\{|u_\lambda|>0\}|}{V}\right)^{\frac{2-N}{N}} \int_{\R^N} |e(u_\lambda)|^2 \;dx \leq \int_{\R^N} |e(u_\lambda)|^2 \;dx,$$ and $$\int_{\R^N} |v|^2=t^{-N} \int_{\R^N} |u_\lambda|^2=\frac{V}{|\{|u_\lambda|>0\}|}.$$ Therefore, we deduce from \eqref{test1} that \begin{eqnarray} \lambda \Big( |\{|u_\lambda|>0\}|-V\Big)^+ &\leq& \lambda_V\left(1-\frac{V}{|\{|u_\lambda|>0\}|}\right)^+. \label{test2} \end{eqnarray} But then we obtain a bound $\lambda\leq M_0$, where $$M_0=\max_{s \geq V}\left(\lambda_V \frac{1}{s-V}\left(1-\frac{V}{s}\right)\right) =\max_{s \geq V} \frac{\lambda_V}{s}= \frac{\lambda_V}{V}.$$ We arrive to the conclusion that for $\lambda >\frac{\lambda_V}{V}$, then we necessarily have $$|\{|u_\lambda|>0\}|\leq V.$$ Now let us fix $\lambda >\frac{\lambda_V}{V}$, and finish the proof of the Proposition. We denote by $u$ the minimizer for the problem \eqref{problemu}, and we pick any $v\in H^1(\R^N)$. From the inequality $$F_\lambda(u_\lambda)\leq F_\lambda(u),$$ and the fact that $|\{|u>0\}|\leq V$ and $\int_{\R^N}|u|^2=1$, we deduce that $$F_\lambda(u_\lambda)\leq \int_{\R^N} |e(u)|^2 dx.$$ On the other hand by the definition of $\lambda_V$, we know that $$\int_{\R^N}|e(u_\lambda)|^2 \;dx -\lambda_V \int_{\R^N} |u_\lambda|^2 \;dx \geq 0,$$ so that $$F_\lambda(u)=\int_{\R^N}|e(u)|^2 \;dx =\lambda_V \leq \int_{\R^N}|e(u_\lambda)|^2 \;dx + \lambda_V\left(1- \int_{\R^N} |u_\lambda|^2 \;dx\right) \leq F_\lambda(u_\lambda),$$ where for the last inequality we have used that $|\{|u_\lambda|>0\}|\leq V$. All together we have proved that $$F_\lambda(u)=F_\lambda(u_\lambda),$$ and therefore $u$ is also a minimizer of $F_\lambda$. But then \eqref{problemlam} holds true for every $v\in H^1(\R^N)$ because $F_\lambda(u)=\int_{\R^N} |e(u)|^2\;dx$ and $F_\lambda(u)\leq F_\lambda(v)$. This achieves the proof of the proposition. \end{proof} \subsubsection{Lam\'e quasi-minimizers} In order to investigate the regularity properties of an optimal domain we introduce the following definition. \begin{definition}[Quasi-minimizer] \label{defQuasi}We say that $u \in H^1(\R^N)^N$ is a quasi-minimizer for the Lam\'e energy if and only if $u$ satisfies the following minimality property: there exists $C>0$ and $\gamma>0$ such that for all ball $B_r\subset \R^N$ of radius $r \in (0,1)$ and for all $v\in H^1(\R^N)^N$ such that $u=v$ on $\R^N\setminus B_r$ we have \begin{eqnarray} \int_{B_r}|\nabla u|^2 +({\rm div}(u))^2 \; dx \leq \int_{B_r} |\nabla v|^2 +({\rm div}(v))^2dx + Cr^\gamma. \label{problemlam2} \end{eqnarray} \end{definition} The definition is motivated by the following observation. \begin{proposition}\label{quasiminimal}Let $u$ be a solution for the problem \begin{eqnarray} \min \left\{ \int_{\R^N}|e(u)|^2 dx \quad {\text s.t. } \quad u \in H^1(\R^N)^N, \; \int_{\R^N} |u|^2=1, \;\text{ and } \; |\{|u|>0\}|\leq V \right\}. \label{problemu2} \end{eqnarray} Assume moreover that $u\in L^p(\R^N)^N$ with $p\geq 2$. Then $u$ is a quasi-minimizer for the Lam\'e energy in the sense of Definition~\ref{defQuasi} with $\gamma=N-\frac{2N}{p}$ . \end{proposition} \begin{proof}By Proposition \ref{penalization} we already know that $u$ satisfies the following minimality property: for all $v\in H^1(\R^N)^N$ we have \begin{eqnarray} \int_{\R^N}|e(u)|^2 \; dx \leq \int_{\R^N} |e(v)|^2 dx +\lambda_V\left(1-\int_{\R^N} v^2\right)^+ + \lambda \Big( |\{|v|>0\}|-V\Big)^+,\label{problemlam2ter} \end{eqnarray} or equivalently, using \eqref{kornkornkorn}, \begin{eqnarray} \frac{1}{2}\int_{\R^N}|\nabla u|^2 + ({\rm div}(u))^2 \; dx \notag & \leq& \frac{1}{2}\int_{\R^N} |\nabla v|^2 + ({\rm div}(v))^2\, dx \notag \\ &&+\lambda_V\left(1-\int_{\R^N} v^2\right)^+ + \lambda \Big( |\{|v|>0\}|-V\Big)^+.\label{problemlam2bis} \end{eqnarray} Then let $v$ be equal to $u$ outside $B_r$, so that the volume $|\{|v|>0\}|$ has at most increased by $\omega_Nr^N$, and since $\int_{\R^N} |u|^2 \;dx=1$, we can compute $$1-\int_{\R^N} |v|^2 =1- \int_{\R^N}|u|^2 + \int_{\R^N} |u|^2 - \int_{\R^N} |v|^2 =\int_{B_r} |u|^2 - \int_{B_r} |v|^2. $$ In other words from the minimality of $u$ we obtain \begin{eqnarray} \int_{B_r}|\nabla u|^2 + ({\rm div}(u))^2 \; dx \leq \int_{B_r} |\nabla v|^2 + ({\rm div}(v))^2dx +C\left( \int_{B_r} |u|^2 - \int_{B_r} |v|^2 \right)^+ + Cr^{N}.\notag \end{eqnarray} If moreover $u \in L^p(\R^N)^N$ with $p\geq 2$, then denoting by $q$ the exponent satisfying $2q=p$ we can estimate $$\left( \int_{B_r} |u|^2 - \int_{B_r} |v|^2 \right)^+\leq \int_{B_r} |u|^2 \leq |B_r|^{\frac{1}{q'}}\left(\int_{B_r} |u|^{2q}\right)^{\frac{1}{q}} =C r^{\frac{N}{q'}}\|u\|_p^{\frac{p}{q}}=Cr^{\gamma}$$ with $\gamma=\frac{N}{q'}=N-\frac{2N}{p}$. Since $\gamma <N$ and $r\leq 1$, it follows that $Cr^N\leq Cr^{\gamma}$ and finally $u$ is a quasi-minimizer for the Lam\'e energy in the sense of Definition~\ref{defQuasi} with $\gamma=N-\frac{2N}{p}$. \end{proof} Now Theorem \ref{open} follows from gathering together Proposition \ref{quasiminimal} with the following one after noticing that the condition $\gamma> N-2$ with $\gamma=N-\frac{2N}{p}$ is equivalent to $p>N$. \begin{proposition} \label{aga} Let $u \in H^1(\R^N)^N$ be a quasi-minimizer for the Lam\'e energy with exponent $\gamma \in (N-2,N]$. Then $u\in C^{0,\alpha}(\R^N)^N$, for all $\alpha<\frac{\gamma-(N-2)}{2}$. \end{proposition} \begin{proof} The proof is inspired by standard arguments in free boundary theory, such as for instance \cite[Theorem 2.1.]{david_toro}. The novelty here is that we have to take care of the Lam\'e energy instead of the standard Dirichlet energy. However, since the Lam\'e system is elliptic in the sense of systems, (which means that it satisfies the strong Legendre-Hadamard ellipticity condition), we can conclude by use of similar techniques from the regularity theory for elliptic equations. In this proof we will keep denoting by $C>0$ a universal constant that could change from line to line. Let $B_r\subset \R^N$ be a given ball of radius $r\in (0,1)$ and let $v$ be the solution for the problem $$ \min_{v\in u+H_0^1(B_r)}\Big \{ \int_{B_r} |\nabla (v)|^2 +({\rm div}(v))^2 \;dx \Big\}. $$ In other words $v$ is the replacement of $u$ in $B_r$, by a function satisfying $v=u$ on $\partial B_r$ and solution for the homogeneous Lam\'e system \begin{eqnarray} -\Delta v - \nabla ({\rm div} v)=0 \text{ in } B_r. \label{lameB} \end{eqnarray} Since the Lam\'e system satisfies the strong Legendre-Hadamard ellipticity condition, then $v$ enjoys some nice decaying properties. Indeed by applying standard regularity theory for elliptic systems (see for instance \cite[Theorem 4.11]{giaquinta}) we know that \begin{eqnarray} \sup_{B_{r/2}}|\nabla v|^2 \leq C \frac{1}{|B_r|}\int_{B_r}|\nabla v|^2 \;dx . \label{estimate} \end{eqnarray} Let now $Q_s(v)$ be the quadratic form defined by $$Q_s(v):= \int_{B_s} |\nabla v|^2 +({\rm div}(v))^2 \;dx.$$ Then for $s\leq r/2$, using \eqref{estimate} we get \begin{eqnarray} Q_s(v)= \int_{B_s} |\nabla v|^2 +({\rm div}(v))^2 \;dx &\leq &(1+N^2)\sup_{B_{r/2}}| \nabla v|^2 |B_s| \notag \\ & \leq & C \left(\frac{s}{r}\right)^{N}\int_{B_r}|\nabla v|^2 \;dx, \notag \\ &\leq & C \left(\frac{s}{r}\right)^{N} Q_r(v), \label{decay} \end{eqnarray} where $C$ depends only on dimension $N$. Moreover, the weak formulation of \eqref{lameB} says that for all $\varphi \in H^1_0(B_r)^N$, $$ \int_{B_r}\nabla v : \nabla \varphi+ ({\rm div} v)({\rm div \varphi})\;dx=0.$$ In other words, if $A_r(u,v)$ denotes the bilinear form associated with $Q_r$ and defined by $$A_r(u,v):= \int_{B_r} \nabla u:\nabla v +{\rm div}(u){\rm div}(v) \;dx,$$ we have $A_r(v,\varphi)=0$ for all $\varphi \in H^1_0(B_r)^N$. This applies in particular to $\varphi=u-v\in H^1_0(B_r)^N$ and we deduce from Pythagoras equality that $Q_r(u-v)+Q_r(v)=Q_r(u)$ or differently, \begin{eqnarray} Q_r(u-v)=Q_r(u)-Q_r(v). \label{pythagore} \end{eqnarray} We will use this property later. Notice also that $Q_s$ is a nonnegative quadratic form for any $s>0$ and using that $Q_s(b-a)\geq 0$ we obtain, for arbitrary $a,b$, $$2|A_s(a,b)|\leq Q_s(a)+Q_s(b),$$ so that, for all $s<r/2$, $$Q_{s}(u)=Q_{s}(u-v+v)\leq 2Q_{s}(u-v)+2Q_{s}(v).$$ Then using \eqref{decay} we arrive to \begin{eqnarray} Q_{s}(u) &\leq& 2Q_{s}(u-v)+2Q_{s}(v)\leq C \left(\frac{s}{r}\right)^{N} Q_r(v) +2 Q_r(u-v) \notag\\ &\leq & C \left(\frac{s}{r}\right)^{N} Q_r(u) +2 Q_r(u-v), \notag \end{eqnarray} where for the last line we have used that $v$ is a minimizer of $Q_r$ and $u$ is a competitor. Now we recall that $Q_r(u-v)=Q_r(u)-Q_r(v)$ (by \eqref{pythagore}) and we use that $u$ is a quasi-minimizer, so that $$Q_r(u-v)=Q_r(u)-Q_r(v)\leq Cr^\gamma.$$ All in all, we have proved that for all $s\leq r/2$ we have \begin{eqnarray} Q_{s}(u) \leq C \left(\frac{s}{r}\right)^{N} Q_r(u)+Cr^\gamma. \label{decCay} \end{eqnarray} Of course we can assume $C\geq 2$. The decaying in \eqref{decCay} looks promising but we would prefer $s^\gamma$ on the last term instead of $r^\gamma$. We can obtain this up to decrease a bit the power $\gamma$ and use a technical dyadic argument. This is standard (see for e.g. \cite[Lemma 5.6.]{filippo}) but let us write the full details for the reader's convenience. Indeed, to lighten the notation we denote by $f(s)$ the non-increasing function $f:s\mapsto Q_s(u)$. Let $a\in (0,1/2)$ be chosen later and let $r_k:=a^kr_0$. Let us prove by induction that for all $k \in \N$ it holds \begin{eqnarray} f(a^k r_0)\leq C^k a^{Nk}f(r_0)+ Ca^{(k-1)\gamma }r_0^\gamma \frac{C^k-1}{C-1}. \label{somme} \end{eqnarray} For $k=0$ the inequality is obvious. Now let us assume that it holds true for some $k$. Then from the decaying property \eqref{decCay} we infer that (using in particular that $\gamma\leq N$ in \eqref{etaa}), \begin{eqnarray} f(a^{k+1} r_0)&\leq& Ca^N f(a^k r_0) + C (a^k r_0)^\gamma \notag \\ &\leq & Ca^N \Big( C^k a^{Nk}f(r_0)+ Ca^{(k-1)\gamma}r_0^\gamma \frac{C^k-1}{C-1} \Big)+ C (a^k r_0)^\gamma \notag \\ &\leq & C^{k+1} a^{N(k+1)} f(r_0) + Ca^{\gamma k}r_0^\gamma \frac{C^{k+1}-C}{C-1} + C (a^k r_0)^\gamma \label{etaa} \\ &= & C^{k+1} a^{N(k+1)} f(r_0) + Ca^{\gamma k}r_0^\gamma \left(\frac{C^{k+1}-1}{C-1}\right),\notag \end{eqnarray} which proves \eqref{somme}. To simplify a bit we can write it differently, taking into account that $C\geq 2$, \begin{eqnarray} f(r_k)\leq C^k \left(\frac{r_k}{r_0}\right)^{N}f(r_0)+ C^{k+1}a^{-1}r_k^\gamma. \label{decayingbis} \end{eqnarray} The nice thing with \eqref{decayingbis} is that we have now $r_k^\gamma$ on the last term (compare with \eqref{decCay}), but the price to pay is the $C^k$ in factor. We will beat this factor by choosing well the constant $a$, and decreasing a bit the powers $N$ and $\gamma$. Indeed, let $\alpha \in (0,1)$ be given and let us fix $$a:=\frac{1}{C^{\frac{1}{\alpha}}},$$ so that $$r_k^\alpha C^k = a^{\alpha k}r_0^\alpha C^k=r_0^\alpha.$$ Then from \eqref{decayingbis} we deduce that \begin{eqnarray} f(r_k)&\leq &C^k \left(\frac{r_k}{r_0}\right)^{N-\alpha}\left(\frac{r_k}{r_0}\right)^{\alpha} f(r_0)+ C^{k+1}a^{-1}r_k^{\gamma-\alpha} r_k^{\alpha} \notag \\ &\leq & \left(\frac{r_k}{r_0}\right)^{N-\alpha} f(r_0) + C' r_k^{\gamma-\alpha}, \end{eqnarray} where $$C'=Ca^{-1}r_0^\alpha.$$ Now let $s\in (0,1/2)$ be given. There exists $k$ such that $r_{k+1} \leq s \leq r_k$. In particular, $r_k\leq \frac{1}{a}s$. Moreover, $s\mapsto f(s)$ is non decreasing so \begin{eqnarray} f(s)\leq f(r_k)&\leq& \left(\frac{r_k}{r_0}\right)^{N-\alpha} f(r_0) + C' r_k^{\gamma-\alpha} \notag \\ &\leq& a^{-(N-\alpha)} \left(\frac{s}{r_0}\right)^{N-\alpha} f(r_0) + a^{-(\gamma-\alpha)}C' s^{\gamma-\alpha}. \notag \end{eqnarray} In conclusion we have proved that there exists a constant $C>0$ (depending on $N$, $\alpha$, $\gamma$, $r_0$) such that for all $s\leq 1/2$ we have $$f(s)\leq C s^{N-\alpha} f(r_0)+ Cs^{\gamma-\alpha}\leq C s^{\gamma-\alpha} f(r_0)+ Cs^{\gamma-\alpha},$$ where for the last inequality we have used $N\geq \gamma$. Returning back to $u$, and estimating $f(r_0)$ by $\int_{\R^N}|e(u)|^2 \;dx$, we conclude, using also the Poincar\'e inequality, that for all $r\leq 1/2$, $$\int_{B_r}|u-m_u|^2 \; dx\leq Cr^2\int_{B_r} |\nabla u|^2 \;dx \leq C r^{2+\gamma -\alpha}=C r^{N+(2+\gamma-N -\alpha)},$$ where $m_u$ denotes the (vectorial) average of $u$. Remember that here $\alpha$ is arbitrary close to $0$. Then by standard results about Campanato spaces (see for e.g. \cite[Theorem 5.4]{filippo}), provided that $$ 2+\gamma-N -\alpha>0,$$ then $u \in \mathscr{C}^{0,\beta}(\R^N)$ with $\beta= \frac{2+\gamma-N -\alpha}{2}$. Since $\alpha$ is arbitrary, this means that $u$ belongs to $\mathscr{C}^{0,\beta}(\R^N)$ for all $\beta<\frac{\gamma-(N-2)}{2}$, as soon as $\gamma>N-2$, and the Proposition follows. \end{proof} \section{Characterization of minimizers}\label{secoptim} \subsection{Optimality conditions: first and second orders} We give first general formulae for the first and second order shape derivative of the Lam\'e eigenvalue. We will then apply it to get optimality conditions (assuming that the minimizer is smooth enough to justify our computations). As kindly mentioned by D. Buoso, these computations (and the criticality of the ball) already appeared in the review paper \cite{Buoso-Lamberti15}. Moreover, they use weaker regularity assumptions on the domain $\Omega$ in this paper. For sake of completeness, we give the main results here that will be useful in Section \ref{secdisk}. Let $\Omega$ be a bounded domain with $\mathscr C^3$ boundary. This regularity assumption allows us to ensure that all quantities we will handle belong to $L^2(\partial\Omega)$. Let ${V}\in W^{4,\infty}(\R^N,\R^N)$ and introduce, for any $t\in (-1,1)$ small enough, \[ \Omega_{t{V}}:=(\mathrm{Id}+t{V})\Omega. \] Recall that, for any $t$ small enough, $(\mathrm{Id}+t\Phi)$ is a smooth diffeomorphism. \paragraph{First order optimality conditions.} For a given shape functional $F$ and a given shape $\Omega$, we say that $F$ is differentiable at $\Omega$ if, for any ${V}\in W^{4,\infty}(\R^N,\R^N)$ compactly supported, the limit \[ \langle dF(\Omega),{V}\rangle:=\lim_{t\to 0}\frac{F(\Omega_{t{V}})-F(\Omega)}t\] exists and if it is a linear form in ${V}$. In this case, this limit is called the first-order shape derivative of $F$ at $\Omega$ in the direction ${V}$. We consider the case of the general eigenvalue $$ \Lambda(\Omega)=\inf_{u\in (H^1_0(\Omega))^N}\frac{\mu \int_\Omega |\nabla u|^2\, dx+(\lambda+\mu)\int_\Omega (\operatorname{div}(u))^2\, dx}{\int_\Omega |u|^2\, dx}. $$ Let us assume moreover that $\Omega$ is such that $\Lambda(\Omega)$ is simple. The associated PDE solved by the minimizer $u$ is \begin{equation}\label{pdeLame1} \left\lbrace \begin{array}{cc} -\mu \Delta u -(\lambda+\mu) \nabla ({\rm div}(u)) = \Lambda u & \mbox{ in } \Omega\\ u=0 & \mbox{ on } \partial\Omega .\\ \end{array} \right. \end{equation} It is standard that $W^{4,\infty}(\R^N,\R^N)\ni {V}\mapsto {u}_{(\operatorname{Id}+{V})}\in [H^1(\Omega)]^N$ is differentiable, see for example \cite{Buoso-Lamberti15}, \cite{zbMATH06838450}. Its Eulerian derivative $\dot{{u}}_{{V}}$, solves \begin{equation} \label{Pb:derLame} \begin{cases} -\mu \Delta \dot{{u}}_{{V}}-(\lambda+\mu)\nabla \operatorname{div} \dot{{u}}_{{V}} =\dot \Lambda {u}_{{V}}+\Lambda (\Omega)\dot{{u}}_{{V}} & \textrm{in }\Omega \\ \dot{{u}}_{{V}}=-\nabla {{u}_\Omega} n ({V}\cdot n) & \textrm{on }\partial \Omega\,, \\ \int_{\Omega} {u}_\Omega\cdot \dot{{u}}_{{V}}=0. \end{cases} \end{equation} Let us multiply the main equation by ${u}_\Omega$ and then integrate by parts. We get $$ \mu \int_\Omega \nabla u_\Omega :\nabla \dot{u}_V+(\lambda+\mu)\int_\Omega \operatorname{div}\dot{u}_V \operatorname{div}{u}_\Omega=\Lambda(\Omega)\int_\Omega u_\Omega\cdot \dot{u}_V+\dot{\Lambda}\int_\Omega |{u}_\Omega|^2. $$ Similarly, let us multiply the main equation \eqref{pdeLame1} solved by $u_\Omega$ by $\dot{u}_V$ and then integrate by parts. Using the boundary conditions, we get \begin{eqnarray*} \mu \int_\Omega \nabla u_\Omega :\nabla \dot{u}_V+(\lambda+\mu)\int_\Omega \operatorname{div}\dot{u}_V \operatorname{div}{u}_V&=& \Lambda(\Omega)\int_\Omega u_\Omega\cdot \dot{u}_V-\mu \int_{\partial \Omega}|(\nabla u_\Omega )n|^2(V\cdot n)\\ && -(\lambda+\mu)\int_\Omega (\operatorname{div}u_\Omega)^2 V\cdot n, \end{eqnarray*} by using that $\nabla u_\Omega n \cdot n = \operatorname{div}u_\Omega$ on $\partial \Omega$. Finally, combining the two identities above yields $$ \dot{\Lambda}=-\mu \int_{\partial \Omega}|(\nabla u_\Omega) n|^2(V\cdot n) -(\lambda+\mu)\int_\Omega (\operatorname{div}u_\Omega)^2 V\cdot n. $$ We have then obtained the following result. \begin{proposition}\label{Pr:DifferentiabilityFormulaeBis} Let $\Omega$ denote a $\mathscr C^3$ domain such that $\Lambda(\Omega)$ is simple. Let ${u}_\Omega$ be its associated (normalized) first eigenfunction. For any ${V}\in W^{4,\infty}(\R^N,\R^N)$, the mapping $W^{4,\infty}(\R^N,\R^N)\ni {V}\mapsto \Lambda(\Omega_{{V}})$ is differentiable. Denoting by $\dot \Lambda$ its differential, the first order derivative of $\Lambda$ is \begin{equation}\label{Eq:D1Lambda} \dot{\Lambda}=\langle d\Lambda(\Omega),{V}\rangle=-\mu \int_{\partial \Omega}|(\nabla u_\Omega) n|^2(V\cdot n) -(\lambda+\mu)\int_{\partial \Omega} (\operatorname{div}u_\Omega)^2 V\cdot n. \end{equation} \end{proposition} \begin{remark}[Shape gradient] Observe that $\nabla u_\Omega \nu\cdot n = \operatorname{div}u_\Omega$ on $\partial \Omega$ and denote by $[\nabla u_{i,\Omega}]_\tau:= \nabla u_{i,\Omega}-\frac{\partial u_{i,\Omega}}{\partial nu} n$ the tangential part of the gradient $\nabla u_{i,\Omega}$. According to the result above, the shape gradient $\nabla \Lambda(\Omega)$ reads \begin{equation}\label{eq1106} \nabla \Lambda(\Omega)=-\mu |(\nabla u_\Omega) n|^2 -(\lambda+\mu) (\operatorname{div}u_\Omega)^2 \end{equation} and can be decomposed as: $$ \nabla \Lambda(\Omega)=-\mu \sum_i |[\nabla u_{i,\Omega}]_\tau |^2 -(\lambda+2\mu) (\operatorname{div}u_\Omega)^2 . $$ \end{remark} \begin{corollary} Let $\Omega^*$ by a solution with $\mathscr{C}^3$ boundary of the extremal eigenvalue problem $$ \min_{|\Omega|=V_0}\Lambda(\Omega) $$ such that $\Lambda(\Omega^*)$ is simple. Then, denoting by $u_{\Omega^*}$ any associated eigenfunction, $$\mu |(\nabla u_\Omega^*) n|^2 -(\lambda+\mu) (\operatorname{div}u_\Omega^*)^2 $$ is constant on $\partial\Omega^*$. \end{corollary} \begin{proof} This is a consequence of Proposition~\ref{Pr:DifferentiabilityFormulaeBis}. Indeed, since we work with a volume constraint, there exists a Lagrange multiplier such that the shape gradient of the eigenvalue is proportional to the derivative of the volume, namely $\int_{\partial\Omega^*} V\cdot n$ whence the result. As a particular case, if we take $\mu>0$ and $\lambda=0$ we obtain that $|e(u_{\Omega^*}|$ in that case is constant on the boundary. \end{proof} \paragraph{Second order optimality conditions.} According for instance to \cite{zbMATH03656460}, for any $\mathscr C^3$ domain $\Omega$ such that $\Lambda(\Omega)$ is simple, the mapping $\Omega\mapsto \Lambda(\Omega)$ is twice shape differentiable at $\Omega$ in the following sense: for any compactly supported vector field $V\in W^{4,\infty}(\R^N,\R^N)$, the map $t\mapsto \Lambda\left((\mathrm{Id}+t\Phi)\Omega\right)$ is twice differentiable at $t=0$. We will use the notations \[ \langle d\Lambda(\Omega),V\rangle :=f'(0),\qquad \langle d^2\Lambda(\Omega)V,V\rangle :=f''(0). \] Similarly, the mapping $\Omega\mapsto u_\Omega$ is twice shape-differentiable at $\Omega$, where $u_\Omega$ is the first normalized eigenfunction of System \eqref{pdeLame1} on $\Omega$, in the sense that the mapping $g:t\mapsto u_{(\mathrm{Id}+tV)\Omega}$ is twice differentiable at $t=0$. We let $\dot{u}_V$ be its first order derivative at $t=0$ (often called Eulerian derivative in the standard shape optimization literature). \begin{proposition}\label{Pr:DifferentiabilityFormulae} For any $\mathscr C^3$ domain $\Omega$ such that $\Lambda(\Omega)$ is simple, let $u_\Omega$ be its associated first eigenfunction. For any $V\in W^{4,\infty}(\R^N,\R^N)$ compactly supported, the shape derivative $\dot{u}_V$ solves the PDE \begin{equation} \label{Pb:der} \begin{cases} -\mu \Delta \dot{u}_V-(\lambda+\mu)\nabla \operatorname{div}\dot{u}_V=\Lambda(\Omega)\dot{u}_V+ \langle d\Lambda(\Omega),V\rangle {u_\Omega} & \textrm{in }\Omega\\ \dot{u}_V=-\nabla {u_\Omega} n (V\cdot n) & \textrm{on }\partial \Omega\,, \\ \int_{\Omega} u_\Omega\cdot \dot{u}_V=0. \end{cases} \end{equation} If, in addition, the vector field $V$ is normal to $\partial \Omega$, meaning that $V=(V\cdot n) n$ on $\partial\Omega$, the second-order shape derivative of $\Lambda$ at $\Omega$ is given by \begin{eqnarray}\label{Eq:D2Lambdabis} \langle d^2\Lambda(\Omega)V,V\rangle &=& -\mu\int_{\partial \Omega}\left(H \left| \frac{\partial u_\Omega}{\partial n}\right|^2+2\frac{\partial^2 u_\Omega }{\partial n^2}\cdot \frac{\partial u_\Omega }{\partial n}\right)(V\cdot n)^2 \nonumber \\ && -(\lambda+\mu) \int_{\partial \Omega}\left(H (\operatorname{div} u_\Omega)^2+\frac{\partial (\operatorname{div} u_\Omega)^2}{\partial n}\right)(V\cdot n)^2\nonumber \\ && -2\Lambda \int_\Omega |\dot{u}_V|^2+2\mu \int_\Omega |\nabla \dot{u}_V|^2+2(\lambda+\mu)\int_\Omega (\operatorname{div}\dot{u}_V)^2 \end{eqnarray} where $H$ is the mean curvature of $\partial \Omega$. Furthermore, when $N=2$, one has $$ \frac{\partial (\operatorname{div} u_\Omega)^2}{\partial n}=-\frac{2\mu}{\lambda+\mu}\operatorname{div}u_\Omega \left(H\frac{\partial u_\Omega}{\partial n}\cdot n+\frac{\partial^2u_\Omega}{\partial n^2}\cdot n \right)\qquad \text{on }\partial\Omega . $$ \end{proposition} \begin{proof} Let us denote by $u_\Omega=(u_1,\dots, u_N)^\top$ the solution of \eqref{pdeLame1}. General formulae for the shape differentiation of Dirichlet boundary value problem yield that $\dot{u}_V$ solves \eqref{Pb:der}, we refer to \cite[Chapter 5]{zbMATH06838450} for the detailed computations. Let us apply the Hadamard formula for integrals on variable boundaries \cite[Proposition 5.4.18]{zbMATH06838450} to \eqref{Eq:D1Lambda}. This yields \begin{eqnarray*} \langle d^2\Lambda(\Omega)V,V\rangle &=&-2\mu \int_{\partial \Omega} (\nabla \dot{u}_V)n\cdot (\nabla u_\Omega)n (V\cdot n) -\mu\int_{\partial \Omega}\left(H | (\nabla u_\Omega)n|^2+\frac{\partial |(\nabla u_\Omega)n|^2}{\partial n}\right)(V\cdot n)^2 \\&& -2(\lambda+\mu)\int_{\Omega}\operatorname{div} u_\Omega \operatorname{div} \dot{u}_V (V\cdot n)\\ && -(\lambda+\mu) \int_{\partial \Omega}\left(H (\operatorname{div} u_\Omega)^2+\frac{\partial (\operatorname{div} u_\Omega)^2}{\partial n}\right)(V\cdot n)^2+R, \end{eqnarray*} where $H$ denotes the mean curvature of $\partial\Omega$ and \begin{eqnarray*} R&=&-2\mu\int_{\partial \Omega}\langle (\partial_n u_\Omega)n',(\partial_nu_\Omega)n\rangle ( V\cdot n)-\mu\int_{\partial \Omega} \Vert (\partial_n u_\Omega )n\Vert^2( V\cdot n)'\\ &&-(\lambda+\mu)\int_{\partial\Omega}(\operatorname{div}u_\Omega)^2(V\cdot n)' \end{eqnarray*} where $n'$ is the Eulerian derivative of $n$. The expression of \( d^2 \Lambda(\Omega) \) is independent of the specific extension chosen for \( n \), which allows us to consider a regular extension of \( n \), that is unitary in a neighborhood of \( \partial\Omega \), without loss of generality. As a consequence, $n'=-\nabla_\Gamma(V\cdot n)$, the notation $\nabla_\Gamma$ standing for the tangential gradient. First, note that \( ( V\cdot n)' = 0 \) since we are dealing with vector fields $V$ that are normal to \( \partial\Omega \). Furthermore, because \( u_\Omega = 0 \) on \( \partial\Omega \) and \( n' \) is orthogonal to \( n \), it follows that \( \nabla u_i \cdot n' = 0 \) for \( i \in \{1, 2, \ldots N\} \), which implies \( (\nabla u_\Omega)n' = 0 \). From this, we deduce that \( R = 0 \). Let us multiply the main equation of \eqref{Eq:D2Lambdabis} by $\dot{u}_V$ and then integrate by parts. We obtain \begin{eqnarray*} \lefteqn{\mu \int_\Omega |\nabla \dot{u}_V|^2+\mu \int_{\partial\Omega}(\nabla \dot{u}_V)n \cdot (\nabla u_\Omega)n (V\cdot n)+(\lambda+\mu)\int_\Omega (\operatorname{div}\dot{u}_V)^2}\\ &&+(\lambda+\mu)\int_{\partial\Omega}\operatorname{div}\dot{u}_V (\nabla u_\Omega) n\cdot n (V\cdot n)=\dot{\Lambda}\int_{\Omega}\dot{u}_V\cdot u_\Omega+\Lambda \int_\Omega |\dot{u}_V|^2. \end{eqnarray*} Using that $\int_{\Omega}\dot{u}_V\cdot u_\Omega=0$ and that $$ (\nabla u_\Omega) n\cdot n=\sum_{i,j}\frac{\partial u_i}{\partial x_j}n_in_j=\sum_{i,j}\frac{\partial u_i}{\partial n}n_in_j^2=\sum_{i}\frac{\partial u_i}{\partial n}n_i=\sum_{i}\frac{\partial u_i}{\partial x_i}=\operatorname{div}u_\Omega $$ on $\partial\Omega$, the equality above simplifies into \begin{eqnarray*} \lefteqn{\mu \int_{\partial\Omega}(\nabla \dot{u}_V)n \cdot (\nabla u_\Omega)n (V\cdot n)+(\lambda+\mu)\int_{\partial\Omega}\operatorname{div}\dot{u}_V \operatorname{div}u_\Omega (V\cdot n)} \\ &&=\Lambda \int_\Omega |\dot{u}_V|^2-\mu \int_\Omega |\nabla \dot{u}_V|^2-(\lambda+\mu)\int_\Omega (\operatorname{div}\dot{u}_V)^2. \end{eqnarray*} As a result, the second order derivative of $\Lambda$ rewrites \begin{eqnarray*} \langle d^2\Lambda(\Omega)V,V\rangle &=& -\mu\int_{\partial \Omega}\left(H | (\nabla u_\Omega)n|^2+\frac{\partial |(\nabla u_\Omega)n|^2}{\partial n}\right)(V\cdot n)^2 \\&& -2\Lambda \int_\Omega |\dot{u}_V|^2+2\mu \int_\Omega |\nabla \dot{u}_V|^2+2(\lambda+\mu)\int_\Omega (\operatorname{div}\dot{u}_V)^2\\ && -(\lambda+\mu) \int_{\partial \Omega}\left(H (\operatorname{div} u_\Omega)^2+\frac{\partial (\operatorname{div} u_\Omega)^2}{\partial n}\right)(V\cdot n)^2. \end{eqnarray*} Let us simplify the term $$ A:=\frac{\partial |(\nabla u_\Omega)n|^2}{\partial n} . $$ By expanding $|(\nabla u_\Omega)n|^2$, we get $$ |(\nabla u_\Omega)n|^2=\sum_i \left(\frac{\partial u_i}{\partial n}\right)^2. $$ Now since $u_i=0$ on $\partial \Omega$, one has $$ \frac{\partial}{\partial n}\left(\frac{\partial u_i}{\partial n}\right)^2=2 \left(\frac{\partial u_i}{\partial n}\right) \left(\frac{\partial^2 u_i}{\partial n^2}\right) $$ and therefore $A=2\frac{\partial u_\Omega}{\partial n}\cdot \frac{\partial^2 u_\Omega}{\partial n^2}$. Now we look at the term $$B:=\frac{\partial (\operatorname{div} u_\Omega)^2}{\partial n}.$$ To simplify this term, we will use the main equation in \eqref{pdeLame1}. We have $$ \frac{\partial \operatorname{div}u_\Omega}{\partial n}= \sum_i \frac{\partial \operatorname{div}u_\Omega}{\partial x_i} \,n_i. $$ According to \eqref{pdeLame1} and the decomposition of the Laplacian $$ \Delta u_i=\Delta_\tau u_i+H\frac{\partial u_i}{\partial n}+\frac{\partial^2 u_i}{\partial n^2}\qquad \text{on }\partial \Omega, $$ we get $$ (\lambda+\mu)\frac{\partial \operatorname{div}u_\Omega}{\partial n}=-\mu \left(H\frac{\partial u_\Omega}{\partial n}\cdot n+\frac{\partial^2u_\Omega}{\partial n^2}\cdot n\right)\qquad \text{on }\partial\Omega , $$ and thus $$ \frac{\partial (\operatorname{div} u_\Omega)^2}{\partial n}=-\frac{2\mu}{\lambda+\mu}\operatorname{div}u_\Omega \left(H\frac{\partial u_\Omega}{\partial n}\cdot n+\frac{\partial^2u_\Omega}{\partial n^2}\cdot n\right)\qquad \text{on }\partial\Omega , $$ whence the last claim of the theorem. \end{proof} \subsection{Multiplicity of minimal eigenvalues} \begin{lemma} Assume that $N=2$ and let $\Omega^*$ be a minimizing domain with $\mathscr{C}^3$ boundary for the problem $$ \min_{|\Omega|=V_0}\Lambda(\Omega). $$ Then, $\Lambda(\Omega^*)$ is at most of multiplicity 2. \end{lemma} \begin{proof} In what follows, let us denote by $[y(x)]_\tau$ the tangential part of a vector field $y\in L^2(\partial\Omega,\R^2)$ at $x\in \partial\Omega$, in other words $$ [y(x)]_\tau=y(x)-(y(x)\cdot n(x))n(x). $$ Let us assume that $\Lambda(\Omega)$ has multiplicity $m\geq 3$. According to classical results for the derivative of multiple eigenvalues (see e.g. \cite{henrot06}, \cite{Buoso-Lamberti15}, \cite{CDM21}), the first order optimality conditions read: let $V$ denote a smooth vector field, then the directional derivative of $|\Omega|\Lambda(\Omega)$ exists and it is the smallest eigenvalue of the $m\times m$ matrix $\mathcal{M}$ whose entries are $$V_0\mathcal{M}-\Lambda \int_{\partial\Omega} (V\cdot n)I_2, \quad \text{where }\mathcal{M}_{i,j}=-\int_{\partial\Omega} \left\lbrack \mu [\nabla u^i:\nabla u^j] + (\lambda+\mu) \operatorname{div} u^i \operatorname{div} u^j\right\rbrack V\cdot n,$$ where $(u^1,\dots,u^m)$ is an orthonormal basis of associated eigenfunctions. By minimality, this directional derivative has to be nonnegative. Since we can take both $ V$ and $-V$, this shows that $ \mathcal{M}=\frac{\Lambda}{V_0} \int_{\partial\Omega} (V\cdot n) I_2$. In particular, $$ \mu [\nabla u^i:\nabla u^j] + (\lambda+\mu) \operatorname{div} u^i \operatorname{div} u^j=0\quad \text{on $\partial\Omega$, for }i\neq j, $$ which rewrites $$ \mu \frac{\partial u^i}{\partial n}\cdot \frac{\partial u^j}{\partial n} + (\lambda+\mu) \operatorname{div} u^i \operatorname{div} u^j=0\quad \text{on $\partial \Omega$ for }i\neq j, $$ by using the Dirichlet boundary condition. Observe moreover that $\partial u^i/\partial n \cdot n=\sum_k \partial u^i_k/\partial n n_k=\operatorname{div}u^i$ and therefore, the condition above rewrites $$ \mu \left[\frac{\partial u^i}{\partial n}\right]_\tau\cdot \left[\frac{\partial u^j}{\partial n}\right]_\tau + (\lambda+2\mu) \operatorname{div} u^i \operatorname{div} u^j=0\quad \text{on $\partial\Omega$ for }i\neq j, $$ or equivalently $$ \left(\sqrt{\mu} \left[\frac{\partial u^i}{\partial n}\right]_\tau+\sqrt{\lambda+2\mu} \operatorname{div}(u^i)n)\right)\cdot\left(\sqrt{\mu} \left[\frac{\partial u^j}{\partial n}\right]_\tau+\sqrt{\lambda+2\mu} \operatorname{div}(u^j)n)\right)=0\quad \text{on $\partial\Omega$ for }i\neq j. $$ We have obtained a family of (at least) three orthogonal vectors in $\R^2$, and thus a contradiction. \end{proof} \section{The case of the disk}\label{secdisk} Our first aim is to compute the first eigenvalue of the unit disk in $\R^2$. We recall that the Lam\'e coefficients $\lambda,\mu$ are such that $\mu>0, \lambda+\mu >0$. The first eigenvalue is then defined by \begin{equation}\label{LambdaLame} \Lambda:= \min_{u=(u_1,u_2) \in H^1_0(\Omega)^2} \frac{ \mu\left(\int_{\Omega}|\nabla u_1|^2 \; dx+\int_{\Omega}|\nabla u_2|^2 \; dx\right) +(\lambda+\mu) \int_{\Omega} ({\rm div}(u))^2 \;dx}{\int_{\Omega}(u_1)^2 \; dx+\int_{\Omega}( u_2)^2 \; dx} \end{equation} and the PDE solved by the minimizer $u=(u_1,u_2)$ is \begin{equation}\label{pdeLame} \left\lbrace \begin{array}{cc} -\mu \Delta u -(\lambda+\mu) \nabla ({\rm div}(u)) = \Lambda u & \mbox{ in } \Omega\\ u=0 & \mbox{ on } \partial\Omega .\\ \end{array} \right. \end{equation} \subsection{Eigenvalues and eigenvectors of the unit disk}\label{sec;eig_disk} We follow the strategy proposed in Capoferri et al, \cite{CFLV23}. We will need some Helmholtz decomposition of the vector $u$. Let us state a more general Lemma that will be also useful for the derivative later. \begin{lemma}\label{lem:helm} Let $v=(v_1,v_2)$ be a smooth function satisfying the equation \begin{equation}\label{eqsatv} -\mu \Delta v -(\lambda + \mu) \nabla({\rm div}(v))=\Lambda v +f \end{equation} in a smooth domain $\Omega$ with a given function $f$. There exist two function $\psi_1$ and $\psi_2$ in $C^\infty(\Omega)$ such that \begin{equation}\label{helmoltz} v+\frac{1}{\Lambda}\,f=\nabla \psi_1+\operatorname{curl}\psi_2\quad \text{in }\Omega. \end{equation} Furthermore $\psi_1$ and $\psi_2$ respectively satisfy the PDE $$ -(\lambda+2\mu)\Delta \psi_1=\Lambda \psi_1\quad \text{in }\Omega, $$ and $$ -\mu\Delta \psi_2=\Lambda \psi_2-\frac{\mu}{\Lambda}\operatorname{curl}f \quad \text{in }\Omega . $$ \end{lemma} \begin{proof} This proof has been suggested by M. Levitin. Let us first set $$ \psi_1=-\frac{\lambda+2\mu}{\Lambda}\operatorname{div}\ v \quad\text{and}\quad \psi_2=-\frac{\mu}{\Lambda}\operatorname{curl}v=-\frac{\mu}{\Lambda}(\partial_x v_2-\partial_y v_1). $$ According to \eqref{eqsatv}, one has \begin{eqnarray*} v+\frac{f}{\Lambda} &=& \frac{1}{\Lambda}\left(-\mu \Delta v-(\lambda+\mu)\nabla \operatorname{div}(v)\right)\\ &=& \frac{1}{\Lambda}\left(-\mu \left(\Delta v-\nabla \operatorname{div}(v)\right)-(\lambda+2\mu)\nabla \operatorname{div}(v)\right). \end{eqnarray*} Note that $$ \Delta v-\nabla \operatorname{div}(v)=\begin{pmatrix} \partial_{yy}\ v_1 - \partial_{xy} v_2\\ \partial_{xx} v_2 - \partial_{xy} v_1 \end{pmatrix}=\begin{pmatrix} -\partial_{y}\\ \partial_{x} \end{pmatrix}(\partial_x v_2-\partial_y v_1). $$ Combining the identities above, we thus infer that $v$ satisfies \eqref{helmoltz}. Now, observe that we can write the equation \eqref{eqsatv} as $$\mu \operatorname{curl}\operatorname{curl}(v)-(\lambda+2\mu) \operatorname{grad}\operatorname{div}(v)= \lambda v +f$$ Now, passing to the divergence in this equation and using $\operatorname{div}\operatorname{curl}=0$ and $\operatorname{div}\operatorname{grad}\operatorname{div}=\Delta \operatorname{div}$ yields $$ -(\lambda+2\mu)\Delta \psi_1=\Lambda \psi_1\quad \text{in }\Omega . $$ In the same way, taking the curl in this equation and using $\operatorname{curl}\operatorname{grad}=0$ and $\operatorname{curl}\operatorname{curl}= - \Delta $ yields $$ -\mu\Delta \psi_2=\Lambda \psi_2-\frac{\mu}{\Lambda}\operatorname{curl} f , \quad \text{in }\Omega. $$ The conclusion follows. \end{proof} Now, to compute the eigenvalues and eigenvectors of the unit disk, we use the decomposition provided by Lemma \ref{lem:helm} (with $v=u$ and $f=0$), \begin{equation}\label{helm0} u=\nabla \psi_1 + \operatorname{curl}\psi_2=\left(\begin{array}{c} \frac{\partial \psi_1}{\partial x} \,+ \frac{\partial \psi_2}{\partial y} \\ \frac{\partial \psi_1}{\partial y} \, - \frac{\partial \psi_2}{\partial x} \end{array}\right) \end{equation} and we use that the scalar potentials $\psi_i,i=1,2$ satisfy an Helmholtz equation \begin{equation}\label{helm1} -\Delta \psi_i=\omega_i^2 \psi_i \quad \mbox{in } \Omega \end{equation} where $$\omega_1^2=\frac{\Lambda}{\lambda+2\mu},\quad \omega_2^2=\frac{\Lambda}{\mu}\,.$$ We introduce $$\omega=\sqrt{\Lambda},\ a_1=\frac{1}{\sqrt{\lambda+2\mu}},\ a_2=\frac{1}{\sqrt{\mu}}$$ therefore, $\omega_1 =a_1\omega, \omega_2=a_2\omega$. In polar coordinates, the general solution of \eqref{helm1} is given, for $i=1,2$ by \begin{equation}\label{exp1} \psi_i(r,\theta)=a_{i,0} J_0(\omega_i r) +\sum_{k=1}^\infty J_k(\omega_i r) [a_{i,k} \cos k\theta + b_{i,k} \sin k\theta]. \end{equation} It remains to express the Dirichlet boundary conditions $u_1=u_2=0$ for $r=1$. Using the expression of the derivatives in polar coordinates, this leads to the system $$\left\lbrace \begin{array}{c} \cos\theta [\frac{\partial \psi_1}{\partial r} + \frac{\partial \psi_2}{\partial \theta}] - \sin\theta [\frac{\partial \psi_1}{\partial \theta} - \frac{\partial \psi_2}{\partial r}]=0\\ \sin\theta [\frac{\partial \psi_1}{\partial r} + \frac{\partial \psi_2}{\partial \theta}] + \cos\theta [\frac{\partial \psi_1}{\partial \theta} - \frac{\partial \psi_2}{\partial r}]=0 \end{array} \right.$$ for which we infer \begin{equation}\label{exp2} \frac{\partial \psi_1}{\partial r} + \frac{\partial \psi_2}{\partial \theta}=0 ,\qquad \frac{\partial \psi_1}{\partial \theta} - \frac{\partial \psi_2}{\partial r}=0 \end{equation} these equalities being true for $r=1$. Using the expression of $\psi_1,\psi_2$ given in \eqref{exp1}, we get by identification for the constant term (and using the fact that $J_0'=-J_1$): $$a_{1,0}J_1(a_1\omega)=0,\quad a_{2,0} J_1(a_2\omega)=0.$$ This provides the sequence of eigenvalues $(\lambda+2\mu) j_{1,k}^2$ and $\mu j_{1,k}^2$ where $j_{1,k}$ is the sequence of zeros of the Bessel function $J_1$. Among all these values, the smallest one is $\mu j_{1,1}^2$ since $\lambda+2\mu > \mu$ by assumption. Therefore, \begin{equation}\label{candidat1} \mbox{a candidate to be the first eigenvalue $\Lambda$ is } \mu j_{1,1}^2. \end{equation} Now we look at the coefficients in $\cos k\theta$ and $\sin k\theta$. coming from \eqref{exp2}. We obtain the two systems \begin{equation}\label{sys1} \left\lbrace \begin{array}{l} \omega_1 J'_k(\omega_1) a_{k,1} + k J_k(\omega_2) b_{k,2} =0 \\ k J_k(\omega_1) a_{k,1} + \omega_2 J'_k(\omega_2) b_{k,2} + =0 \end{array} \right. \end{equation} and \begin{equation}\label{sys2} \left\lbrace \begin{array}{l} \omega_1 J'_k(\omega_1) b_{k,1} - k J_k(\omega_2) a_{k,2} =0 \\ k J_k(\omega_1) b_{k,1} - \omega_2 J'_k(\omega_2) a_{k,2} + =0 . \end{array} \right. \end{equation} The determinant of these two systems is the same and it must vanish if we look for a non-trivial solution. This leads to the following transcendental equations that determines the other eigenvalues \begin{equation}\label{trans1} a_1a_2\omega^2 J'_k(a_1\omega)J'_k(a_2\omega)-k^2 J_k(a_1\omega)J_k(a_2\omega) =0. \end{equation} Using the classical relations for the derivative of Bessel functions, we can rewrite \eqref{trans1} \begin{equation}\label{trans2} \frac{k}{a_1\omega} J_k(a_1\omega)J_{k-1}(a_2\omega) + \frac{k}{a_2\omega} J_{k-1}(a_1\omega)J_{k}(a_2\omega) - J_{k-1}(a_1\omega)J_{k-1}(a_2\omega)=0 \end{equation} or \begin{equation}\label{trans3} \frac{k}{a_1\omega} J_k(a_1\omega)J_{k+1}(a_2\omega) + \frac{k}{a_2\omega} J_{k+1}(a_1\omega)J_{k}(a_2\omega) - J_{k+1}(a_1\omega)J_{k+1}(a_2\omega)=0 . \end{equation} Now to determine the first eigenvalue of the elasticity operator, we need to know whether the smallest solution of the previous transcendental equations can be smaller than the value $\mu j_{1,1}^2$ already obtained. In that case, the first eigenvalue would be double, systems \eqref{sys1} and \eqref{sys2} providing two independent solutions associated to the same eigenvalue. Let us state the following characterization where we see that the first eigenvalue actually depends on the Poisson coefficient $\nu$: \begin{theorem}\label{theopoisson} Let $\nu^*$ be the number \begin{equation}\label{nustar} \nu^*:=\frac{j_{1,1}^2-2{j'_{1,1}}^2}{2j_{1,1}^2-2{j'_{1,1}}^2}\simeq 0.349895 \end{equation} where $j_{1,1}$ is the first zero of the Bessel function $J_1$ and $j'_{1,1}$ is the first zero of its derivative $J'_1$. Assume that the Poisson coefficient $\nu$ satisfies \begin{equation}\label{poisson1} \nu\leq \nu^*, \end{equation} then the first eigenvalue is given as a solution of the transcendental equation \eqref{trans2} for some $k$ and then it is at least double. Assume that the Poisson coefficient $\nu$ satisfies \begin{equation}\label{poisson2} \nu\geq \nu^* \end{equation} then the first eigenvalue is $\Lambda=\mu j_{1,1}^2$. Moreover, it is a simple eigenvalue as soon as $\nu > \nu^*$. \end{theorem} \begin{remark} Note that when $\nu=\nu^*$ the first eigenvalue is (at least) triple and equal to $\mu j_{1,1}^2$. \end{remark} \begin{proof} Let us introduce the function $F_k$ defined by $$F_k(\omega)= \frac{k}{a_1\omega} J_k(a_1\omega)J_{k-1}(a_2\omega) + \frac{k}{a_2\omega} J_{k-1}(a_1\omega)J_{k}(a_2\omega) - J_{k-1}(a_1\omega)J_{k-1}(a_2\omega).$$ When $\omega$ is small, using the Taylor expansion of the Bessel function near 0, we obtain $$F_k(\omega)=\frac{a_1^{k-1}a_2^{k-1}(a_1^2+a_2^2)}{[(k-1)!]^2 2^{2k+1}k(k+1)}\,\omega^2 + o(\omega^2)$$ that shows in particular that $F_k(\omega)>0$ for $\omega>0$ small. Now let us look at $F_1$ and evaluate it at $\omega^*=\sqrt{\mu j_{1,1}^2}=j_{1,1}/a_2$. Since $J_1(a_2\omega^*)=0$ we get $$F_1(\omega^*)=\frac{J_0(a_2\omega^*)}{a_1\omega^*}\left(J_1(a_1\omega^*)-a_1\omega^* J_0(a_1\omega^*)\right).$$ If we can prove that $F_1(\omega^*) \leq 0$, then $F_1$ changes its sign between 0 and $\omega^*$ that implies the fact that the first eigenvalue is a zero of the transcendental equation. Now $J_1(x)-xJ_0(x)=-xJ'_1(x)$ and this is negative between 0 and $j'_{1,1}$ and positive between $j'_{1,1}$ and $j'_{1,2}$. On the other hand, the term $J_0(a_2\omega^*)=J_0(j_{1,1} )<0$ therefore, we want to find the case where $x=a_1\omega^*$ belongs to the interval $[j'_{1,1},j'_{1,2}]$. Now $$a_1\omega^*=\sqrt{\frac{\mu}{\lambda+2\mu}} j_{1,1}\in [j'_{1,1},j'_{1,2}] \Leftrightarrow \frac{{j'_{1,1}}^2}{j_{1,1}^2}\leq \frac{1-2\nu}{2-2\nu} \leq \frac{{j'_{1,2}}^2}{j_{1,1}^2}$$ where we use the expression of $\mu/(\lambda+2\mu)$ in term of $\nu$. Solving the previous inequality in $\nu$ provides the desired result from the left inequality. The right inequality is automatically satisfied since $-1\leq \nu <0.5$. Now, it remains to prove that, when $\nu \geq \nu^*$ the first eigenvalue is $\mu j_{1,1}^2$ (and is simple when $\nu > \nu^*$). Let us introduce $\psi_k(x):=x J_k'(x)/J_k(x)$. It is known that the function $\psi_k$ is decreasing on all interval in $\R_+$ where it is defined, and in particular on $[0, j_{k,1})$. We refer to \cite{spigler} or \cite{landau99} for that assertion. Moreover $\psi_k(0)=k$. This implies that $\psi_k(x) < k$ for $x\in (0,j_{k,1})$ and for any $k$. Now, let us assume that $\omega$ is such that $\omega < \sqrt{\mu j_{1,1}^2}$. Since $\lambda+2\mu > \mu$, we have $a_1<a_2$ and therefore $$a_1 \omega < a_2 \omega < a_2 \sqrt{\mu j_{1,1}^2} = j_{1,1} \leq j_{k,1} \quad \mbox{for all } k\geq 1 .$$ Therefore $J_k(a_1 \omega) >0$ and $J_k(a_2\omega) >0$ for all $\omega < \sqrt{\mu j_{1,1}^2}$. Now, let us rewrite the transcendental equation \eqref{trans1} as (we can divide by $J_k(a_1 \omega) J_k(a_2 \omega) $ that is positive) \begin{equation}\label{eqpsik} \psi_k(a_1 \omega) \psi_k(a_2 \omega) - k^2 =0 . \end{equation} Now, the properties we recalled on $\psi_k$ and the fact that $a_i \omega < j_{k,1}$ show that the first member of \eqref{eqpsik} is strictly negative when $\omega < \sqrt{\mu j_{1,1}^2}$. This proves the thesis. \end{proof} \subsection{Optimality of the disk: first order arguments} We wonder whether a Faber-Krahn type inequality holds for the elasticity operator. We will see that it depends actually of the Poisson coefficient. Roughly speaking, when the first eigenvalue $\Lambda$ is double, we can prove that the disk is not a minimizer, while when $\Lambda$ is simple, we can prove that the disk is at least a local minimizer. Let us start by the first possibility: \begin{theorem}\label{theonotdisk} Assume that the Poisson coefficient $\nu$ satisfies \eqref{poisson1} with a strict inequality. Then the disk does not minimize $\Lambda$ among open sets of given volume. \end{theorem} \begin{proof} We will use a first order optimality argument for which we need the expression of the eigenvectors. As we have seen in Theorem \ref{theopoisson}, when $\nu$ satisfies \eqref{poisson1}, the eigenvalue is (at least) double and the two eigenvectors can be obtained through the systems \eqref{sys1} and \eqref{sys2} with $\omega$ defined as the smallest solution of all the equations \eqref{trans1} (or \eqref{trans2}, \eqref{trans3}). The value of the integer $k$ will not be really important here. Let us choose for example $$a_{1,k}=k J_k(\omega_2), \qquad b_{2,k}=-\omega_1 J'_k(\omega_1)$$ that satisfy system \eqref{sys1}. (we recall that $\omega_1=a_1\omega$ and $\omega_2=a_2\omega$). Then $$\psi_1(r,\theta)=a_{1,k} J_k(\omega_1 r)\cos k\theta,\qquad \psi_2(r,\theta)=b_{2,k} J_k(\omega_2 r)\sin k\theta.$$ Using \eqref{helm0}, we obtain $u=(u_1,u_2)$ with \begin{eqnarray*} u_1=a_{1,k}\left(\omega_1 \cos\theta \cos k\theta J'_k(\omega_1 r) + \frac{k\sin\theta}{r} \sin k\theta J_k(\omega_1 r)\right) +\\ b_{2,k}\left(\omega_2 \sin\theta \sin k\theta J'_k(\omega_2 r) + \frac{k\cos\theta}{r} \cos k\theta J_k(\omega_2 r)\right) \end{eqnarray*} \begin{eqnarray*} u_2=a_{1,k}\left(\omega_1 \sin\theta \cos k\theta J'_k(\omega_1 r) - \frac{k\cos\theta}{r} \sin k\theta J_k(\omega_1 r)\right) +\\ b_{2,k}\left(-\omega_2 \cos\theta \sin k\theta J'_k(\omega_2 r) + \frac{k\sin\theta}{r} \cos k\theta J_k(\omega_2 r)\right) . \end{eqnarray*} In principle, we must multiply the previous expressions by a normalization factor in order to satisfy $\int_\Omega u_1^2+u_2^2 =1$, but it turns out that this factor has no importance in the computation we present now. The shape derivative of a multiple eigenvalue is now a classical topic: in the case of the elasticity operator, we refer for example to the recent paper \cite{CDM21}. To sum up, let us assume that the eigenvalue has multiplicity $m$ and denote by $u^1,u^2, \ldots u^m$ a set of orthonormal eigenvectors. Then if we perturb the boundary of $\Omega$ by a vector field $V$, the first eigenvalue has a semi-derivative (or directional derivative) that is given as the smallest eigenvalue of the $m\times m$ matrix $\mathcal{M}$ whose entries are $$\mathcal{M}_{i,j}=-\int_{\partial\Omega} \left\lbrack \mu [\nabla u^i:\nabla u^j] + (\lambda+\mu) \operatorname{div} u^i \operatorname{div} u^j\right\rbrack V\cdot n$$ (where $n$ is here the exterior normal vector). So our thesis will be proved if we can prove that this matrix has a negative eigenvalue for a vector field preserving the area, i.e. a vector field $V$ such that $\int_{\partial\Omega} V\cdot n =0$. For that purpose, it is sufficient to look at the first term $\mathcal{M}_{1,1}$ and prove that it can be chosen negative (that will imply that the symmetric matrix $\mathcal{M}$ is not positive and therefore has a negative eigenvalue). This term being given by $$\mathcal{M}_{1,1}=-\int_{\partial\mathbb{D}} \left\lbrack \mu (|\nabla u_1|^2+|\nabla u_2|^2) + (\lambda+\mu) (\operatorname{div} u)^2\right\rbrack V\cdot n$$ we have to compute on the unit circle $|\nabla u_1|^2, |\nabla u_2|^2$ and $(\operatorname{div} u)^2$. From the Helmholtz decomposition \eqref{helm0}, it comes $$\operatorname{div} u=\Delta \psi_1=-\omega_1^2 \psi_1=-a_{1,k} \omega_1^2J_k(\omega_1 r) \cos k\theta$$ so, on the unit circle \begin{equation}\label{div1} (\operatorname{div} u)^2=a_{1,k}^2 \omega_1^4J_k(\omega_1 )^2 \cos^2 k\theta. \end{equation} Now, $u_1$ and $u_2$ being constant on the unit circle, we have $|\nabla u_i|^2=\left(\frac{\partial u_i}{\partial r}\right)^2$ with $r=1$. Using the formula of $u_1,u_2$, we can write \begin{equation}\label{gradu1} \frac{\partial u_1}{\partial r}=A_1 \cos\theta \cos k\theta + B_1 \sin\theta \sin k\theta \end{equation} with \begin{eqnarray*} A_1=a_{1,k} \omega_1^2 {J_k}''(\omega_1) -k b_{2,k} J_k(\omega_2)+b_{2,k} k \omega_2 J'_k(\omega_2) \\ B_1=-k a_{1,k} {J_k}(\omega_1) +a_{1,k} k \omega_1 J'_k(\omega_1) +b_{2,k} \omega_2^2 {J_k}''(\omega_2). \end{eqnarray*} Using the Bessel differential equation to replace ${J_k}''(\omega_i), i=1,2$ by a combination of $J'_k(\omega_i)$ and $J_k(\omega_i)$, together with the choice we have done for $a_{1,k}$ and $b_{2,k}$ and the transcendental equation \eqref{trans1}, we can simplify the previous expressions as \begin{equation}\label{gradu12} A_1=-k \omega_1^2 J_k(\omega_1) J_k(\omega_2), \quad \; B_1=\omega_1\omega_2^2 J'_k(\omega_1) J_k(\omega_2). \end{equation} In the same way, we obtain \begin{equation}\label{gradu2} \frac{\partial u_2}{\partial r}=A_2 \sin\theta \cos k\theta + B_2 \cos\theta \sin k\theta \end{equation} with \begin{equation}\label{gradu123} A_2=-k \omega_1^2 J_k(\omega_1) J_k(\omega_2)=A_1, \quad \; B_2=-\omega_1\omega_2^2 J'_k(\omega_1) J_k(\omega_2)=-B_1. \end{equation} Therefore \begin{eqnarray*} |\nabla u_1|^2+|\nabla u_2|^2=A_1^2\cos^2 k\theta +B_1^2 \sin^2 k\theta = \\ \omega_1^2 J_k^2(\omega_2) \left(\omega_1^2 k^2 J_k^2(\omega_1) \cos^2 k\theta +\omega_2^4 {J'_k}^2(\omega_1)\sin^2 k\theta\right). \end{eqnarray*} With $(\operatorname{div} u)^2$ given by \eqref{div1} we finally get \begin{equation}\label{m11} \mathcal{M}_{1,1}=\omega_1^2 J_k^2(\omega_2) \int_0^{2\pi} \left((\lambda+2\mu)k^2\omega_1^2J_k^2(\omega_1)\cos^2 k\theta +\mu \omega_2^4 {J'_k}^2(\omega_1)\sin^2 k\theta\right) V\cdot n . \end{equation} As explained before, in order to conclude the proof, it suffices to find a deformation field $V$ such that $\int_0^{2\pi} V\cdot n =0$ and $ \mathcal{M}_{1,1} <0$. Let us choose $V$ such that $V(1,\theta)=\alpha \cos (2k\theta)$. Plugging this value in $ \mathcal{M}_{1,1}$ yields \begin{equation}\label{m112} \mathcal{M}_{1,1}=\omega_1^2 J_k^2(\omega_2) \frac{\pi \alpha}{2} \,\left((\lambda+2\mu)k^2\omega_1^2J_k^2(\omega_1) -\mu \omega_2^4 {J'_k}^2(\omega_1)\right). \end{equation} The quantity $ \mathcal{M}_{1,1}$ being linear in $\alpha$, in order to conclude we just need to prove that the right-hand side of \eqref{m112} cannot be zero. According to Theorem \ref{theopoisson} we know that the eigenvalue satisfies $\Lambda < \mu j_{1,1}^2$, therefore $\omega_2=\sqrt{\frac{\Lambda}{\mu}}<j_{1,1}$ and then $J_k(\omega_2)>0$ (for $k\geq 1$, the first zero of $J_k$ is always greater or equal to $j_{1,1}$). It remains to consider the quantity $(\lambda+2\mu)k^2\omega_1^2J_k^2(\omega_1) -\mu \omega_2^4 {J'_k}^2(\omega_1)$. Using the expression of $\omega_1,\omega_2$ and up to the factor $\omega^2$, it is equal to $$Q=k^2 J_k^2(a_1\omega)-\frac{\omega^2}{\mu} {J'_k}^2(a_1\omega).$$ Since $a_1<a_2$, we know that $J_k(a_1 \omega)>0$. Therefore, $Q=0$ means \begin{equation}\label{derj1} kJ_k(a_1\omega)-a_2\omega J'_k(a_1\omega) = 0 \quad \mbox{or} \quad kJ_k(a_1\omega)+a_2\omega J'_k(a_1\omega) = 0 . \end{equation} Let us analyze the first case. From the transcendental equation, we see that $$kJ_k(a_1\omega)=a_2\omega J'_k(a_1\omega) \Rightarrow kJ_k(a_2\omega)=a_1\omega J'_k(a_2\omega) .$$ Rewriting this in term of the function $\psi_k$ already introduced, this means $$\psi_k(a_2 \omega) = \frac{k a_2}{a_1}$$ but since $ka_2/a_1 > k$ and $\psi_k(x) \leq k$ in this range we see that it is impossible. Now in the other case, in the same way thanks to the transcendental equation, we get \begin{equation}\label{psiw} \psi_k(a_2 \omega) =- \frac{k a_2}{a_1}. \end{equation} When $k\geq 3$ this is impossible since then $J_k(a_2\omega)$ and $J'_k(a_2\omega)$ are both positive (we recall that we are in the case where $\omega \leq \sqrt{\mu}j_{1,1} \Rightarrow a_2 \omega \leq j_{1,1}$). It remains the case $k=2$. In that case, due to the fact that $\psi_2$ is decreasing, we infer $\psi_2(a_2 \omega)\geq \psi_2(j_{1,1})$. But since $j_{1,1} J'_2(j_{1,1}) = -2 J_2(j_{1,1} )$ this would imply with \eqref{psiw} $$-\frac{2 a_2}{a_1} \geq -2 \Rightarrow a_2 \leq a_1$$ a contradiction since we know that $a_2> a_1$. This finishes the proof of non optimality of the disk in that case. \end{proof} \subsection{Optimality of the disk: second order arguments}\label{seconddisk} Let us assume that $\Omega$ is the unit disk $\Omega=\mathbb{B}_2$ and assume that $\Lambda$ is simple. We know, according to Theorem \ref{theopoisson} that it is the case as soon as $\nu > \nu^*$ and moreover $\Lambda(\Omega)=\mu j_{1,1}^2$. We also know, from the proof of Theorem \ref{theopoisson} that the associated eigenspace is spanned by the normalized vector $U=[u_1,u_2]^\top$, reading in polar coordinates $(r,\theta)$ as $$ u_1=-\alpha \sin\theta J_1(j_{1,1}r)\quad \text{and}\quad u_2=\alpha \cos\theta J_1(j_{1,1}r) $$ where $\alpha=\frac{1}{\sqrt \pi |J_0(j_{1,1})|}$. Our aim is to prove that in that case, the first order shape derivative of the functional $\mathcal{F}(\Omega):=|\Omega| \Lambda(\Omega)$ is zero (for any vector field $V$) while the second order shape derivative is a positive quadratic form. A consequence of the general formulae for the second shape derivative given in Proposition~\ref{Pr:DifferentiabilityFormulae} is: \begin{proposition}\label{cor:LambSecBall} Assume that $\Omega=\mathbb{B}_2$ is the unit disk in $\R^2$. Assume that the Poisson coefficient $\nu$ satisfies $ \nu> \nu^*$. Then, the second order derivative of $\Lambda$ at $\Omega=\mathbb{B}_2$ reads \begin{eqnarray}\label{Eq:D2Lambda} \langle d^2\Lambda(\Omega)V,V\rangle &=& -\mu\int_{\partial \Omega}\frac{\partial^2 u_\Omega }{\partial n^2}\cdot \frac{\partial u_\Omega }{\partial n}(V\cdot n)^2 \nonumber \\ && -2\Lambda \int_\Omega |\dot{u}_V|^2+2\mu \int_\Omega |\nabla \dot{u}_V|^2+2(\lambda+\mu)\int_\Omega (\operatorname{div}\dot{u}_V)^2. \end{eqnarray} \end{proposition} \begin{proof} This follows by observing that, in such a case, \begin{itemize} \item one has $\operatorname{div}u_\Omega=0$ in $\Omega$ ; \item furthermore, using the standard decomposition of the Laplacian on $\partial\Omega$ yields $$ 0=-\Lambda(\Omega)u_\Omega-(\lambda+\mu)\nabla \operatorname{div}u_\Omega=\mu \Delta u_\Omega=\frac{\partial^2 u_\Omega }{\partial n^2}+H \frac{\partial u_\Omega}{\partial n}+\Delta_{\partial\Omega} u_\Omega, $$ where $\Delta_{\partial\Omega}$ stands for the Laplace-Beltrami tangential operator on $\partial\Omega$. It follows that $\Delta_{\partial\Omega} u_\Omega=0$ on $\partial\Omega$, and thus, $$ H \left| \frac{\partial u_\Omega}{\partial n}\right|^2+2\frac{\partial^2 u_\Omega }{\partial n^2}\cdot \frac{\partial u_\Omega }{\partial n}=\frac{\partial^2 u_\Omega }{\partial n^2}\cdot \frac{\partial u_\Omega }{\partial n}\quad \text{on }\partial\Omega . $$ \end{itemize} \end{proof} Our first task is to compute explicitly the second derivative of $\Lambda$. According to Proposition~\ref{cor:LambSecBall}, one has $$ \langle d^2\Lambda(\Omega)V,V\rangle = -\mu A_1-2\Lambda \int_\Omega |\dot{u}_V|^2+ A_2+2(\lambda+\mu)A_3, $$ where \begin{eqnarray*} A_1 &=& \int_{\partial \Omega}\frac{\partial^2 u_\Omega }{\partial n^2}\cdot \frac{\partial u_\Omega }{\partial n}(V\cdot n)^2\\ A_2 &=& 2\mu\int_\Omega |\nabla \dot{u}_V|^2\\ A_3 &=& \int_\Omega (\operatorname{div}\dot{u}_V)^2. \end{eqnarray*} Let us compute each term separately. To this aim, it is convenient to denote by $\varphi$ the function $V\cdot n$, defined on the boundary of $\mathbb{B}_2$, expanding in the $\theta$-coordinate as \begin{equation}\label{expand:varphiFourier} \varphi(\theta)=\sum_{k=0}^{+\infty} \alpha_k\cos (k\theta)+\beta_k\sin (k\theta). \end{equation} \paragraph{A simplified expression of $ \langle d^2\Lambda(\Omega)V,V\rangle $.} According to the Green formula and Proposition~\ref{Pb:der}, one has \begin{eqnarray*} A_2&=&2\mu \int_{\partial\Omega}\dot{u}_V\cdot \frac{\partial \dot{u}_V}{\partial n}-2\mu\int_{\Omega}\dot{u}_V\cdot \Delta \dot{u}_V\\ &=& 2\mu\int_{\partial\Omega}\dot{u}_V\cdot \frac{\partial \dot{u}_V}{\partial n}+2\Lambda(\Omega)\int_\Omega |\dot{u}_V|^2+2(\lambda+\mu)\int_\Omega \dot{u}_V\cdot \nabla \operatorname{div}\dot{u}_V . \end{eqnarray*} Let us now use the equation satisfied by $\dot{u}_V$. We obtain \begin{eqnarray*} A_2 &=& 2\mu\int_{\partial\Omega}\dot{u}_V\cdot \frac{\partial \dot{u}_V}{\partial n}+\Lambda(\Omega)\int_\Omega |\dot{u}_V|^2-2(\lambda+\mu)\int_\Omega ( \operatorname{div}\dot{u}_V)^2+2(\lambda+\mu)\int_{\partial \Omega}\operatorname{div}\dot{u}_V(\dot{u}_V\cdot n) . \end{eqnarray*} Note that \begin{eqnarray*} \dot{u}_V\cdot n&=&-\varphi \nabla u_\Omega n \cdot n = -\varphi \sum_{i,j}\frac{\partial u_{\Omega,i}}{\partial x_j}n_jn_i= -\varphi \sum_i \frac{\partial u_{\Omega,i}}{\partial n}n_i=-\varphi \operatorname{div}u_\Omega=0\quad \text{on }\partial\Omega. \end{eqnarray*} We get \begin{eqnarray}\label{train0954} A_2 &=& 2\mu\int_{\partial\Omega}\dot{u}_V\cdot \frac{\partial \dot{u}_V}{\partial n}+2\Lambda(\Omega)\int_\Omega |\dot{u}_V|^2-2(\lambda+\mu)\int_\Omega ( \operatorname{div}\dot{u}_V)^2 . \end{eqnarray} As a consequence, $$ \langle d^2\Lambda(\Omega)V,V\rangle = -\mu A_1+2\mu\int_{\partial\Omega}\dot{u}_V\cdot \frac{\partial \dot{u}_V}{\partial n}. $$ Let us now expand this expression into a sum of squares. \paragraph{Computation of $A_1$.} One has $$ \frac{\partial^2 u_\Omega }{\partial n^2}\cdot \frac{\partial u_\Omega }{\partial n}=\alpha^2j_{1,1}^3J_1'(j_{1,1})J_1''(j_{1,1})=-\alpha^2j_{1,1}^2J_0(j_{1,1})^2, $$ by noting that $J_1'(j_{1,1})=J_0(j_{1,1})$ and $j_{1,1}^2J_1''(j_{1,1})=-j_{1,1}J_1'(j_{1,1})$. It follows that $$ A_1=\mu\alpha^2 j_{1,1}^2J_0(j_{1,1})^2\int_{\partial\Omega}(V\cdot n)^2 = \Lambda\left(2\alpha_0^2+\sum_{k=1}^{+\infty}(\alpha_k^2+\beta_k^2)\right) . $$ \paragraph{Computation of $\boldsymbol{I:=\int_{\partial\Omega}\dot{u}_V\cdot \frac{\partial \dot{u}_V}{\partial n}}$.} Recall that $\dot{u}_V$ satisfies \begin{equation}\label{dotuBoundary} \dot{u}_V=\alpha j_{1,1}J_0(j_{1,1})\varphi(\theta)[\sin\theta,-\cos \theta]^\top\quad \text{on }\partial\Omega. \end{equation} To compute $\dot{u}_V$ inside the domain $\Omega$, we will use Lemma ~\ref{lem:helm} with $v=\dot{u}_V$ and $f=d\Lambda(\Omega,V) u_\Omega$. According to formulae \eqref{Pr:DifferentiabilityFormulaeBis} and since $\mathrm{div} u_\Omega=0$, we finally obtain for the unit disk \begin{equation}\label{firstderdisk} d\Lambda(\Omega,V)=-2\mu j_{1,1}^2 \alpha_0= -2\Lambda(\Omega) \alpha_0. \end{equation} Since the first derivative of the area is $dA(\Omega,V)=\int_{\partial \Omega} V\cdot n=2\pi \alpha_0$, we recover the fact that the first derivative of the functional $\mathcal{F}$ is zero at the disk (in other terms, the disk is a critical point). We now use the decomposition of Lemma~\ref{lem:helm} on the unit circle taking profit that $u_\Omega$ vanishes on the boundary. Therefore, by writing $\psi_1$ and $\psi_2$ in the polar coordinates $(r,\theta)$, one has on the boundary $$ \left\{\begin{array}{l} \dot{u}_{V,1}=\partial_x\psi_1-\partial_y\psi_2=\cos\theta \left(\partial_r\psi_1-\frac{1}{r}\partial_\theta\psi_2\right)-\sin\theta \left(\frac{1}{r}\partial_\theta\psi_1+\partial_r\psi_2\right)\\ \dot{u}_{V,2}=\partial_y\psi_1+\partial_x\psi_2=\sin\theta \left(\partial_r\psi_1-\frac{1}{r}\partial_\theta\psi_2\right)+\cos\theta \left(\frac{1}{r}\partial_\theta\psi_1+\partial_r\psi_2\right) . \end{array} \right. $$ By using \eqref{dotuBoundary}, we infer that \begin{equation}\label{eq0953} \partial_r\psi_1-\partial_\theta\psi_2=0\quad \text{and}\quad \partial_\theta\psi_1+\partial_r\psi_2=-\frac{j_{1,1}}{\sqrt \pi}\varphi(\theta). \end{equation} For the sake of notational clarity, let us introduce $$ \omega=\sqrt{\frac{\mu}{\lambda+2\mu}}. $$ According to Lemma~\ref{lem:helm}, the functions $\psi_1$ and $\psi_2$ solve the PDEs $$ -\Delta \psi_1=\omega^2 j_{1,1}^2\psi_1\quad \text{and}\quad -\Delta \psi_2=j_{1,1}^2\psi_2\quad \text{in }\Omega. $$ We infer that $\psi_1$ and $\psi_2$ expand as \begin{eqnarray*} \psi_1 &=& a_{1,0}J_0(\omega j_{1,1}r)+\sum_{k=1}^{+\infty}\left(a_{1,k}\cos (k\theta)+b_{1,k}\sin(k\theta) \right)J_k(\omega j_{1,1}r)\\ \psi_2 &=& a_{2,0}J_0( j_{1,1}r)+\sum_{k=1}^{+\infty}\left(a_{2,k}\cos (k\theta)+b_{2,k}\sin(k\theta) \right)J_k( j_{1,1}r) . \end{eqnarray*} Plugging these expressions into \eqref{eq0953} allows us to compute the Fourier coefficients characterizing $\psi_1$ and $\psi_2$: $$ \left\{\begin{array}{l} a_{1,0}=0, \quad b_{1,0}\text{ is arbitrary}\\ a_{1,k}j_{1,1}\omega J_k'(j_{1,1}\omega)-jJ_1(j_{1,1})b_{2,k}=0, \qquad k\geq 1\\ -ka_{1,k}J_k(j_{1,1}\omega)+j_{1,1}J_k'(j_{1,1})b_{2,k}=-\frac{j_{1,1}}{\sqrt \pi}\beta_k\\ kb_{1,k}J_j(j_{1,1}\omega)+j_{1,1}J_k'(j_{1,1})a_{2,k}=-\frac{j_{1,1}}{\sqrt \pi}\alpha_k\\ b_{1,k}j_{1,1}\omega J_k'(j_{1,1}\omega)+kJ_k(j_{1,1})a_{2,k}=0 \end{array}\right. $$ which, after easy computations, reduces into \begin{eqnarray*} a_{1,k} & =& -\frac{kj_{1,1}J_k(j_{1,1})}{\sqrt \pi (j_{1,1}^2\omega J_{k}'(\omega j_{1,1})J_k'(j_{1,1})-k^2J_k(j_{1,1})J_k(\omega j_{1,1}))}\beta_k\\ a_{2,k} & =& \frac{kj_{1,1}J_k'(j_{1,1})}{\sqrt \pi (j_{1,1}^2\omega J_{k}'(\omega j_{1,1})J_k'(j_{1,1})-k^2J_k(j_{1,1})J_k(\omega j_{1,1}))}\alpha_k\\ b_{1,k} & =& -\frac{\omega j_{1,1}^2J_k(j_{1,1}\omega)}{\sqrt \pi (j_{1,1}^2\omega J_{k}'(\omega j_{1,1})J_k'(j_{1,1})-k^2J_k(j_{1,1})J_k(\omega j_{1,1}))}\alpha_k\\ b_{2,k} & =& -\frac{\omega j_{1,1}^2J_k'(j_{1,1}\omega)}{\sqrt \pi (j_{1,1}^2\omega J_{k}'(\omega j_{1,1})J_k'(j_{1,1})-k^2J_k(j_{1,1})J_k(\omega j_{1,1}))}\beta_k .\\ \end{eqnarray*} We know the explicit expression of $\psi_1$ and $\psi_2$. We are now in position to compute $I$. One has \begin{eqnarray*} I &=& \int_0^{2\pi}\left(\dot{u}_{V,1}\frac{\partial \dot{u}_{V,1}}{\partial r}+\dot{u}_{V,2}\frac{\partial \dot{u}_{V,2}}{\partial r}\right)\, d\theta \\ &=& \frac{j_{1,1}}{\sqrt \pi}\int_0^{2\pi}\left(\frac{\partial\psi_1}{\partial\theta}-\left(\frac{\partial^2\psi_1}{\partial r\partial\theta}+\frac{\partial^2\psi_2}{\partial r^2}\right)\right)\varphi(\theta)\, d\theta +(2\alpha_0)^2 \int_0^{2\pi} u_\Omega . \frac{\partial u_\Omega}{\partial r}\,d\theta. \end{eqnarray*} Note that \begin{eqnarray*} \frac{\partial^2\psi_1}{\partial r\partial\theta}&=& \omega j_{1,1}\sum_{k=1}^{+\infty}kJ_k'(\omega j_{1,1})\left(b_{1,k}\cos (k\theta)-a_{1,k}\sin (k\theta)\right)\\ \frac{\partial^2\psi_2}{\partial r^2}&=& j_{1,1}^2\sum_{k=1}^{+\infty} J_k''(j_{1,1})\left(a_{2,k}\cos (k\theta)+b_{2,k}\sin (k\theta)\right)+ j_{1,1}^2a_{2,0}J_1''( j_{1,1}) \end{eqnarray*} on $\partial\Omega$. Now, using $$ j_{1,1}^2J_k''( j_{1,1})=- j_{1,1}J_k'( j_{1,1})+(k^2- j_{1,1}^2)J_k( j_{1,1}), $$ it follows from easy, but lengthly computations that \begin{eqnarray*} -\frac{j_{1,1}}{\sqrt \pi}\int_0^{2\pi}\left(\frac{\partial^2\psi_1}{\partial r\partial\theta}+\frac{\partial^2\psi_2}{\partial r^2}\right)\varphi(\theta)\, d\theta &=& -\omega j_{1,1}^3\sum_{k=1}^{+\infty}\frac{k^2J_k(j_{1,1})J_k'(\omega j_{1,1})(\alpha_k^2+\beta_k^2)}{j_{1,1}^2\omega J_{k}'(\omega j_{1,1})J_k'(j_{1,1})-k^2J_k(j_{1,1})J_k(\omega j_{1,1})}\\ && +\omega j_{1,1}^3\sum_{k=1}^{+\infty}\frac{(-j_{1,1}J_k'(j_{1,1})+(k^2-j_{1,1}^2)J_k(j_{1,1}))J_k'(\omega j_{1,1})(\alpha_k^2+\beta_k^2)}{j_{1,1}^2\omega J_{k}'(\omega j_{1,1})J_k'(j_{1,1})-k^2J_k(j_{1,1})J_k(\omega j_{1,1})}\\ &=& -\omega j_{1,1}^4\sum_{k=1}^{+\infty}\frac{(J_k'(j_{1,1})+j_{1,1}J_k(j_{1,1}))J_k'(\omega j_{1,1})(\alpha_k^2+\beta_k^2)}{j_{1,1}^2\omega J_{k}'(\omega j_{1,1})J_k'(j_{1,1})-k^2J_k(j_{1,1})J_k(\omega j_{1,1})} . \end{eqnarray*} Similarly, since $$ \frac{\partial\psi_1}{\partial \theta}=\sum_{k=1}^{+\infty}kJ_k(\omega j_{1,1})\left(b_{1,k}\cos (k\theta)-a_{1,k}\sin (k\theta)\right)\qquad \text{on }\partial \Omega, $$ it follows that \begin{eqnarray*} \frac{j_{1,1}}{\sqrt \pi}\int_0^{2\pi} \frac{\partial\psi_1}{\partial\theta}\, d\theta &=& - j_{1,1}^2\sum_{k=1}^{+\infty}\frac{k^2J_k(\omega j_{1,1})J_k(j_{1,1})(\alpha_k^2+\beta_k^2)}{j_{1,1}^2\omega J_{k}'(\omega j_{1,1})J_k'(j_{1,1})-k^2J_k(j_{1,1})J_k(\omega j_{1,1})} . \end{eqnarray*} \paragraph{Conclusion.} Finally, we obtain the following expression of $ \langle d^2\Lambda(\Omega)V,V\rangle$ by combining all the results above: $$ \langle d^2\Lambda(\Omega)V,V\rangle=\mu j_{1,1}^2\left(6\alpha_0^2+\sum_{k=1}^{+\infty}c_k(\alpha_k^2+\beta_k^2)\right), $$ where $$ c_k=\frac{k^2J_k(\omega j_{1,1})J_k(j_{1,1})-\omega j_{1,1}^2J_k'(j_{1,1})J_k'(\omega j_{1,1})-2k \omega j_{1,1}^2 J_k(j_{1,1})J_k'(\omega j_{1,1})}{j_{1,1}^2\omega J_{k}'(\omega j_{1,1})J_k'(j_{1,1})-k^2J_k(j_{1,1})J_k(\omega j_{1,1})}. $$ These computations allow us to state: \begin{theorem} Let $\mathcal{F}$ the shape functional defined by $\mathcal{F}(\Omega)=|\Omega| \Lambda(\Omega)$ and $\Omega$ be the unit disk. Then $d\mathcal{F}(\Omega,V)=0$ and \begin{equation}\label{der2F} \langle d^2 \mathcal{F}(\Omega),V,V\rangle \geq A_0 \|\hat{V}\|_{H^1(\partial\Omega)}^2, \end{equation} where $\hat{V}$ denotes the projection of $V\cdot n$ on the orthogonal space to $span\{1,\cos\theta,\sin\theta\}$. Therefore the unit disk is a local minimum in a weak sense. \end{theorem} \begin{proof} The fact that the first derivative of $\mathcal{F}$ vanishes at the disk has already been proved. Let us compute the second shape derivative. Denoting by $A$ the area, we have $$d^2\mathcal{F}=\Lambda d^2A +2 d\Lambda dA +A d^2\Lambda.$$ Using $dA=\int_{\partial\Omega} \varphi$, $d^2A=\int_{\partial\Omega} H \varphi^2$ where the mean curvature $H$ equals $1$ and $d\Lambda=-2\Lambda \int_{\partial\Omega} \varphi$, we finally get with the above expression of $d^2\Lambda$ the following expansion for the second derivative $$ \langle d^2\mathcal{F}(\Omega)V,V\rangle=\pi \Lambda \sum_{k=1}^{+\infty}C_k(\alpha_k^2+\beta_k^2) $$ where $\alpha_k$ and $\beta_k$ are the coefficients in the expansion \eqref{expand:varphiFourier} of $\varphi$ and $$ C_k=2j_{1,1}^2 \omega \frac{kJ'_k(\omega j_{1,1})J_k(j_{1,1})}{k^2J_k(\omega j_{1,1})J_k(j_{1,1})-j_{1,1}^2\omega J'_k(\omega j_{1,1})J'_k(j_{1,1})} $$ with $\omega=\sqrt{\mu/(\lambda+2\mu)}<1$. We remark that no terms come from $k=0$ and $k=1$ ($C_1=0$). This is due to the invariance of the functional $\mathcal{F}$ under dilation and rotation. We claim that each $C_k$ is positive for $k\geq 2$. Indeed, we have already seen in the proof of Theorem \ref{theopoisson} that the denominator of $C_k$ is positive. The first term in the numerator is also positive since, for $k\geq 2$, $J_k(j_{1,1}) >0$. For the second term we need to be more precise. Since we are in the case where the first eigenvalue of the disk is $\mu j_{1,1}^2$ we know that $\nu >\nu^*$. Now, $$\omega=\sqrt{\frac{\mu}{\lambda+2\mu}}=\sqrt{\frac{1-2\nu}{2-2\nu}} < \sqrt{\frac{1-2\nu^*}{2-2\nu^*}}= \frac{j'_{1,1}}{j_{1,1}} <\frac{1}{2}.$$ Therefore $\omega j_{1,1} < j_{1,1}/2 < j'_{k,1}$ for any $k\geq 2$ and $J'_k(\omega j_{1,1})>0$. To conclude the proof, we look at the asymptotic behaviour of $C_k$ for $k$ large. When $x$ is fixed, we have for $k\,$ large $$J_k(x)\sim \frac{x^k}{2^k k!} -\frac{x^{k+2}}{2^{k+2} (k+1)!} \quad \mbox{ and } \quad J'_k(x)\sim \frac{x^{k-1}}{2^k (k-1)!}-\frac{(k+2)x^{k+1}}{2^{k+2} (k+1)!} .$$ Then the numerator $N_k$ of $C_k$ satisfies $$N_k\sim \frac{ j_{1,1}^{2k+1}\omega^k}{2^{2k-1}[(k-1)!]^2}$$ while the denominator $D_k$ of $C_k$ satisfies $$D_k\sim \frac{ j_{1,1}^{2k+2}\omega^k}{2^{2k+2}(k-1)!(k+1)!}\left(2(1+\omega^2)-\frac{\omega^2j_{1,1}^2}{k}\right)\quad \text{as }k\to +\infty.$$ Finally $$C_k\sim \frac{4k(k+1)}{j_{1,1}\left(2(1+\omega^2)-\frac{\omega^2j_{1,1}^2}{k}\right)} \geq k(k+1)\quad \text{as }k\to +\infty.$$ The conclusion follows since the $H^1$ norm of the projection of $\varphi$ on the orthogonal space to $\operatorname{span}\{1,\cos\theta,\sin\theta\}$ is $$\|\varphi\|_{H^1}^2=\sum_{k+2}^{+\infty} (k^2+1) (\alpha_k^2+\beta_k^2).$$ \end{proof} \section{Some particular domains} The aim of this section is to find (simple) domains which may have a lower first eigenvalue than the disk, at least when $\nu\geq \nu^*$. For that purpose, we will give first explicit examples for which we can give the exact value of $\Lambda$. Let us mention that these examples are very similar to the ones found by Kawohl-Sweers in \cite{kawohl-sweers}. Then we will consider the case of rectangles. In that case, we are not able to give the exact value of $\Lambda$ but we can estimate it from above with a good precision. \subsection{Rhombi}\label{secrhombi} In this section, we discuss the following question: does there exist some domain in the plane for which the eigenvector is given by twice the same function, i.e. $U(x,y)=(u(x,y),u(x,y))$. As we will see, this is possible and we can even, in that case, find an explicit eigenvector and an explicit eigenvalue. More precisely we will find some parallelograms, actually rhombi, (depending on the Lam\'e coefficients $\lambda,\mu$) fulfilling this condition and the associated eigenvalue will be quite simple and only depend on the area of the parallelogram. We work by analysis and synthesis. \paragraph{Analysis.} Let us assume that the domain $\Omega \subset \mathbb{R}^2$ has the property that its eigenvector is given by $U=(u(x,y),u(x,y))$. Thus $\mathrm{div}(U)=\frac{\partial u}{\partial x} + \frac{\partial u}{\partial y}$. We replace in the eigenvector equation \eqref{pdeLame1} and we make the difference of the two equations to obtain \begin{equation} \frac{\partial }{\partial x} \mathrm{div} U - \frac{\partial }{\partial y} \mathrm{div} U =0 . \end{equation} Therefore (locally, but then globally by analyticity), we have \begin{equation}\label{transp} \mathrm{div} U = \frac{\partial u}{\partial x} + \frac{\partial u}{\partial y} = f(x+y) \end{equation} for some analytic function $f$. Solving this transport equation \eqref{transp} provides the existence of two analytic functions $\varphi$ and $\psi$ such that finally \begin{equation}\label{defu} u(x,y)= \varphi(x-y) + \psi(x+y). \end{equation} Now we come back to the system \eqref{pdeLame1}: we have $\Delta u=2 (\varphi^{\prime\prime}(x-y) +\psi^{\prime\prime}(x+y) )$ and $ \mathrm{div} U = 2 \psi^\prime (x+y)$. Therefore, using the change of variable $v=x-y, w=x+y$ we see that $\varphi$ and $\psi$ must satisfy $$-2\mu (\varphi^{\prime\prime}(v) +\psi^{\prime\prime}(w) ) - 2(\lambda+\mu) \psi^{\prime\prime}(w) = \Lambda (\varphi(v) + \psi(w)).$$ In this equation, we can separate variables to get the existence of some constant $C$ such that $$-2(\lambda+2\mu) \psi^{\prime\prime}(w) - \Lambda \psi(w) = C = 2\mu (\varphi^{\prime\prime}(v) + \Lambda \varphi(v)).$$ Solving this equation separately in $\psi$ and $\varphi$ yields \begin{equation}\label{eqpsi} \psi(w)=A_1\cos \omega_1 w + B_1\sin \omega_1 w -\frac{C}{\Lambda} \quad\mbox{ with } \omega_1^2=\frac{\Lambda}{2\lambda+4\mu} \end{equation} and \begin{equation}\label{eqphi} \varphi(v)=-A_2\cos \omega_2 v - B_2\sin \omega_2 v +\frac{C}{\Lambda} \quad\mbox{ with } \omega_2^2=\frac{\Lambda}{2\mu} . \end{equation} Adding \eqref{eqphi} and \eqref{eqpsi}, we get by \eqref{defu} $$ u(x,y)=u(v,w)= A_1\cos \omega_1 w + B_1\sin \omega_1 w -A_2\cos \omega_2 v - B_2\sin \omega_2 v $$ that can also be rewritten as \begin{equation}\label{formulau} u(v,w)=C_1 \sin(\omega_1 w-\theta_1) - C_2 \sin(\omega_2 v-\theta_2). \end{equation} With this expression of $u$ we have completely taken into account the eigen-equation. It just remain to express the Dirichlet boundary condition. In other words, domains $\Omega$ that will satisfy the property (that the eigenvector is of the kind $(u,u)$) are those domains on which a function $u(v,w)$ given by \eqref{formulau} vanishes on the boundary of $\Omega$. \paragraph{Synthesis.} We will prove below that necessarily $C_1=C_2$ in the expression \eqref{formulau}. So let us assume that $C_1=C_2$ and let us investigate the set of points where $u$ vanishes. In that case we have to solve $\sin(\omega_1 w-\theta_1) = \sin(\omega_2 v-\theta_2)$, therefore, coming back to the variables $x,y$: $$u=0 \Leftrightarrow \left\lbrace \begin{array}{l} \omega_1(x+y)-\omega_2(x-y)=\theta_1-\theta_2 + 2k\pi, \;k\in \mathbb{Z} \\ \omega_1(x+y)+\omega_2(x-y)=\theta_1+\theta_2 + (2k'+1) \pi, \;k'\in \mathbb{Z} \\ \end{array}\right.$$ or, it can also be written using the definition of $\omega_1,\omega_2$ and introducing the real numbers $a_1=\theta_1-\theta_2$ and $a_2=\theta_1+\theta_2$ \begin{equation}\label{charp} \left\lbrace \begin{array}{l} \left(\frac{1}{\sqrt{\lambda+2\mu}}-\frac{1}{\sqrt{\mu}}\right) x+ \left(\frac{1}{\sqrt{\lambda+2\mu}}+\frac{1}{\sqrt{\mu}}\right) y = \sqrt{\frac{2}{\Lambda}}(a_1+ 2k\pi), \;k\in \mathbb{Z} \\ \left(\frac{1}{\sqrt{\lambda+2\mu}}+\frac{1}{\sqrt{\mu}}\right) x+ \left(\frac{1}{\sqrt{\lambda+2\mu}}-\frac{1}{\sqrt{\mu}}\right) y = \sqrt{\frac{2}{\Lambda}}(a_2+ + (2k'+1) \pi), \;k'\in \mathbb{Z} .\\ \end{array}\right. \end{equation} This corresponds to equations of line segments with two specific normal vectors. Therefore, the domain $\Omega$ should be a parallelogram delimited by such parallel line segments. But we have to make more precise what line segments. To simplify the notations, let us introduce $$\alpha=\frac{1}{\sqrt{\lambda+2\mu}}-\frac{1}{\sqrt{\mu}} , \quad \beta=\frac{1}{\sqrt{\lambda+2\mu}}+\frac{1}{\sqrt{\mu}}$$ and the normal vectors $$\mathbf{e_1}=\left(\begin{array}{c} \alpha \\ \beta \end{array}\right)\qquad \mathbf{e_2}=\left(\begin{array}{c} \beta \\ \alpha \end{array}\right).$$ Let us assume that the parallelogram is defined by the four equations $$\left\lbrace \begin{array}{l} \mathbf{e_1}\cdot X=\xi_1 \\ \mathbf{e_1}\cdot X=\hat{\xi}_1 \end{array} \right. \qquad \left\lbrace \begin{array}{l} \mathbf{e_2}\cdot X=\xi_2 \\ \mathbf{e_2}\cdot X=\hat{\xi}_2 . \end{array} \right. $$ According to \eqref{charp}, we must have $$\xi_1= \sqrt{\frac{2}{\Lambda}}(a_1+ 2k\pi) \quad \hat{\xi}_1= \sqrt{\frac{2}{\Lambda}}(a_1+ 2\hat{k}\pi)$$ therefore $\hat{\xi}_1-\xi_1= \sqrt{\frac{2}{\Lambda}} 2m\pi$ for some integer $m$ that cannot be zero. Let us take the smallest possible value $m=1$ (or $m=-1$). This shows that \begin{equation} \hat{\xi}_1-\xi_1= \sqrt{\frac{2}{\Lambda}} 2\pi . \end{equation} Exactly in the same way, we get \begin{equation}\label{hatxi} \hat{\xi}_2-\xi_2= \sqrt{\frac{2}{\Lambda}} 2\pi . \end{equation} In particular we see that the parallelogram must satisfy $\hat{\xi}_1-\xi_1=\hat{\xi}_2-\xi_2$ and therefore, it is a rhombus. We are going to give a simple relation between the area of the parallelogram and the eigenvalue $\Lambda$. Assume that the parallelogram has vertices $A,B,C,D$ with $B=A+\rho_1 \mathbf{e_1}^\perp$ and $D=A+\rho_2 \mathbf{e_2}^\perp$ where $\mathbf{e_1}^\perp$ and $\mathbf{e_2}^\perp$ are the vectors respectively orthogonal to $\mathbf{e_1}$ and $\mathbf{e_2}$ with the same norm. The line $(AB)$ corresponds to $\xi_1$ and the line $(AD)$ to $\hat{\xi}_2$ in the previous notations. Then the length of the basis $AB$ is $AB=\rho_1 \|\mathbf{e_1}^\perp\|$. On the other hand, the height $h$ of the parallelogram is given by the distance between $B$ and its orthogonal projection $B_1$ on the line $(CD)$. In other words the height is given by $$h=\frac{1}{\|\mathbf{e_1}\|} BB_1\cdot\mathbf{e}_1.$$ Now $BB_1\cdot \mathbf{e}_1=AB_1\cdot \mathbf{e}_1=\hat{\xi}_1 - \xi_1$ by definition of the two lines. Finally the area of the parallelogram $\Omega$, that is $AB\times h$ is given by $$|\Omega|=\rho_1 \|\mathbf{e_1^\perp}\| \frac{1}{\|\mathbf{e_1}\|}( \hat{\xi}_1 - \xi_1 )= \rho_1 \sqrt{\frac{2}{\Lambda}} 2\pi.$$ It remains to express $\rho_1$ taken into account the relation \eqref{hatxi}. Let $B_2$ be the orthogonal projection of $B$ on the line $(AD)$. By definition we have $BB_2\cdot \mathbf{e_2}=-\hat{\xi}_2+\xi_2$. Now $$\rho_1 \mathbf{e_1^\perp}\cdot \mathbf{e_2}=AB\cdot \mathbf{e_2}=B_2B\cdot \mathbf{e_2}=\hat{\xi}_2-\xi_2 .$$ Thus $$\rho_1=\frac{(\hat{\xi}_2 - \xi_2)}{\mathbf{e_1}^\perp \cdot\mathbf{e_2}}= \sqrt{\frac{2}{\Lambda}} 2\pi \frac{\sqrt{\mu(\lambda+2\mu)}}{4}.$$ Therefore we have proved that the area of the parallelogram is given by \begin{equation}\label{areaparall} |\Omega|=\frac{2\pi^2 \sqrt{\mu(\lambda+2\mu)}}{\Lambda}. \end{equation} Let us rephrase this formula in stating the following theorem: \begin{theorem} Let $\Omega$ be a parallelogram defined by the four lines $$\left\lbrace \begin{array}{l} \mathbf{e_1}\cdot X=\xi_1 \\ \mathbf{e_1}\cdot X=\hat{\xi}_1 \end{array} \right. \qquad \left\lbrace \begin{array}{l} \mathbf{e_2}\cdot X=\xi_2 \\ \mathbf{e_2}\cdot X=\hat{\xi}_2 \end{array} \right. $$ where $$ \mathbf{e_1}=\begin{pmatrix} \alpha \\ \beta \end{pmatrix}\qquad \mathbf{e_2}=\begin{pmatrix} \beta \\ \alpha \end{pmatrix} $$ and $$\alpha=\frac{1}{\sqrt{\lambda+2\mu}}-\frac{1}{\sqrt{\mu}} \quad \beta=\frac{1}{\sqrt{\lambda+2\mu}}+\frac{1}{\sqrt{\mu}}.$$ Assume that $\hat{\xi}_1-\xi_1=\hat{\xi}_2-\xi_2$. Then an eigenvalue of the parallelogram is given by \begin{equation}\label{eigparall} \Lambda=\frac{2\pi^2 \sqrt{\mu(\lambda+2\mu)}}{|\Omega|} \end{equation} with an eigenvector $U$ of the form $U=(u,u)$ where \begin{equation}\label{formulau2} u(x,y)=\sin(\omega_1 (x+y)-\theta_1) - \sin(\omega_2 (x-y)-\theta_2) \end{equation} with $$\omega_1^2=\frac{\Lambda}{2\lambda+4\mu} \quad \omega_2^2=\frac{\Lambda}{2\mu}.$$ \end{theorem} \begin{remark} In the above synthesis, we have studied the case $C_1=C_2$. We claim that in the case $C_1\not= C_2$ there are no (bounded) domain $\Omega$ in the plane such that $$u(v,w)=C_1 \sin(\omega_1 w-\theta_1) - C_2 \sin(\omega_2 v-\theta_2)=0\quad \mbox{on the boundary }\partial\Omega.$$ Indeed if we would have two level lines of the function $u(v,w)$ crossing at some point $A$, necessarily the gradient of $u$ must vanish at $A$. That would provide the three relations $$\left\lbrace \begin{array}{l} C_1 \sin(\omega_1 w-\theta_1) - C_2 \sin(\omega_2 v-\theta_2) = 0\\ C_1 \cos(\omega_1 w-\theta_1) = 0\\ C_2 \cos(\omega_2 v-\theta_2) = 0\\ \end{array}\right.$$ that are clearly incompatible since we can assume $C_1\not= 0$ and $C_2\not= 0$ for a bounded domain. \end{remark} \begin{remark} If we look for domains for which the eigenvector is $U=(u(x,y), Au(x,y))$ for some real number $A$, following the same approach we get other parallelograms but their eigenvalue is still given by the formula \eqref{eigparall}. \end{remark} \medskip Let us come back to the possible minimality of the disk. We have seen in Theorem \ref{theonotdisk} that the disk cannot be a minimizer if the Poisson coefficient is less than $\nu^*\simeq 0.349...$ but we were not able to conclude for larger values of the Poisson coefficient (between $\nu^*$ and $0.5$) since we know that in this case the first eigenvalue of the disk is simple and the disk is a local minimizer (at least in a weak sense). Now our previous analysis allows us to increase the interval of values of the Poisson coefficient for which the disk is not optimal: \begin{corollary} Assume that the Poisson coefficient $\nu$ satisfies $$\nu< \frac{j_{1,1}^4-8\pi^2}{2(j_{1,1}^4-4\pi^2)} \simeq 0.3879$$ then the disk is not a minimizer of $\Lambda$ (among sets of given volume). \end{corollary} \begin{proof} According to Theorem \ref{theonotdisk}, it suffices to compare our previous parallelogram of area $\pi$ with the first eigenvalue of the disk that is $\mu j_{1,1}^2$. Thus we get the thesis as soon as $2\pi \sqrt{\mu(\lambda+2\mu)} < \mu j_{1,1}^2$. This is equivalent to $\frac{\lambda}{\mu} +2 < \frac{j_{1,1}^4}{4\pi^2}$. Now using the relation between the Lam\'e coefficents and the Poisson coefficient, we know that $\lambda/\mu = 2\nu /(1-2\nu)$. Therefore $$\frac{\lambda}{\mu} +2 < \frac{j_{1,1}^4}{4\pi^2} \Leftrightarrow 8\pi^2(1-\nu) < j_{1,1}^4 (1-2\nu) \Leftrightarrow \nu< \frac{j_{1,1}^4-8\pi^2}{2(j_{1,1}^4-4\pi^2)}.$$ \end{proof} \subsection{Rectangles}\label{secrectangle} Now in the range $\frac{3}{8} \leq \nu \leq \frac{2}{5}$, that corresponds to $a=1/(1-2\nu)$ in the range $[4,5]$, we are going to consider convenient rectangles. Note that $3/8 <0.38$, therefore we will be able to cover the whole range $\nu \in (-1,0.4]$ and prove the disk is not optimal in this range with these different arguments. We consider a rectangle $\Omega_L=(0,L)\times (0,\ell)$ of area $\pi$. It will be useful to write the length and the width of the rectangle as $$L=\sqrt{\frac{\pi}{t}}\quad\mbox{ and }\quad \ell=\sqrt{t\pi},\qquad t\in (0,1].$$ Let us denote by $\varphi_1$ the first (normalized) eigenfunction for the Dirichlet-Laplacian of $\Omega_L$, defined by $$\varphi_1(x,y)=\frac{2}{\sqrt{\pi}}\,\sin\left(\frac{\pi x}{L}\right) \sin\left(\frac{\pi y}{\ell}\right),$$ and another eigenfunction $$\varphi_2(x,y)=\frac{2}{\sqrt{\pi}}\,\sin\left(\frac{2\pi x}{L}\right) \sin\left(\frac{2\pi y}{\ell}\right).$$ This other eigenfunction could be the fourth one (for a rectangle not too far from the square), but can also have a larger index. We will explain below why we do this particular choice. Now the idea is to plug in the variational formulation defining $\Lambda(\Omega_L)$ a family of vectors, for $X=(\alpha_1,\alpha_2,\beta_1,\beta_2)$: $$U_X=\left(\begin{array}{c} u_1\\ u_2 \end{array}\right)=\left(\begin{array}{c} \alpha_1 \varphi_1 + \alpha_2 \varphi_2\\ \beta_1 \varphi_1 + \beta_2 \varphi_2\\ \end{array}\right).$$ Since the eigenfunctions of the Laplace operator define an orthonormal basis, we have $$\int_{\Omega_L} |\nabla u_1|^2 +|\nabla u_2|^2 =\left(\frac{\pi^2}{L^2}+\frac{\pi^2}{\ell^2}\right)\left(\alpha_1^2+4\alpha_2^2+\beta_1^2+4\beta_2^2 \right)$$ and $$\int_{\Omega_L} u_1^2 + u_2^2 =\alpha_1^2+\alpha_2^2+\beta_1^2+\beta_2^2.$$ It remains to compute $\int_{\Omega_L} (\mathrm{div}(U_X))^2$. We obtain \begin{equation} \int_{\Omega_L} (\mathrm{div}(U_X))^2=\frac{\pi^2}{L^2}\left(\alpha_1^2+4\alpha_2^2\right) + \frac{\pi^2}{\ell^2}\left(\beta_1^2+4\beta_2^2\right) - \frac{128}{9\pi}\left(\alpha_1 \beta_2 + \alpha_2 \beta_1\right). \end{equation} Using $a=1/(1-2\nu)$, this implies using this admissible test function, that \begin{equation}\label{ratioQ} \frac{\Lambda(\Omega_L)}{\mu} \leq \frac{Q(X)}{\alpha_1^2+\alpha_2^2+\beta_1^2+\beta_2^2} \end{equation} where $Q$ is the quadratic form defined by \begin{eqnarray*} Q(X)=\alpha_1^2\left((1+a)\frac{\pi^2}{L^2} + \frac{\pi^2}{\ell^2}\right) + \alpha_2^2\left(4(1+a)\frac{\pi^2}{L^2} + \frac{4\pi^2}{\ell^2}\right) + \beta_1^2\left(\frac{\pi^2}{L^2} + (1+a)\frac{\pi^2}{\ell^2}\right) + \\ \beta_2^2\left(\frac{4\pi^2}{L^2} + 4(1+a)\frac{\pi^2}{\ell^2}\right) - \frac{128 a}{9\pi}\left(\alpha_1 \beta_2 + \alpha_2 \beta_1\right). \end{eqnarray*} Now we have to choose $X=(\alpha_1,\alpha_2,\beta_1,\beta_2)$ that give the lowest possible value for the ratio in \eqref{ratioQ}. This lowest value exactly corresponds to the smallest eigenvalue of the $4\times 4$ matrix of the quadratic form $Q$. This matrix $\mathcal{M}$ has the simple structure $$\mathcal{M}=\left(\begin{array}{cccc} a_1 & 0 & 0 & b\\ 0 & a_2 & b & 0\\ 0 & b & a_3 & 0\\ b & 0 & 0 & a_4\\ \end{array}\right) .$$ Its characteristic polynomial factorizes as $P(x)=[(a_2-x)(a_3-x)-b^2][(a_1-x)(a_4-x)-b^2]$ with $b=-64a/9\pi$ and $$\begin{array}{l} a_1=\pi(1+a)t+\frac{\pi}{t }\\ a_2=4\pi(1+a)t+\frac{4\pi}{t}\\ a_3=\pi t+\frac{(1+a)\pi}{t }\\ a_4=4\pi t+\frac{4(1+a)\pi}{t } .\\ \end{array}$$ We observe that $a_2a_3=a_1a_4$ and $a_1+a_4 \geq a_2+a_3$ because $t\leq 1$. Therefore the trinome $(a_1-x)(a_4-x)-b^2$ is always less than $(a_2-x)(a_3-x)-b^2$ and the smallest root of $P(x)$ is the smallest root of $q_1(a,t,x):=(a_1-x)(a_4-x)-b^2$. More precisely, the question is to know whether the smallest root of $q_1$ is smaller than $j_{1,1}^2$ because our aim is to compare the rectangle $\Omega_L$ with the unit disk. Since $q_1(a,t,0)=a_1a_4-b^2$, we see that $$q_1(a,t,0)\geq q_1(a,\frac{1}{2},0) = 4\pi^2\left(a^2+4a+4-\frac{1024 a^2}{81\pi^4}\right) >0.$$ Therefore, we get the thesis as soon as we can find some $t^*\in (0,1]$ such that $q_1(a,t^*,j_{1,1}^2)<0$ for all $a\in [4,5]$. It turns out that the particular choice $t^*=2/5=0.4$ achieves this aim. This is an elementary analysis to prove that the polynomial expression $$ q_1(a,\frac{2}{5},j_{1,1}^2)=j_{1,1}^4 - \pi j_{1,1}^2 \left(\frac{29}{2} + \frac{52 a}{5}\right) + 4\pi^2 \left(a^2+\frac{169}{36}\,(a+1)\right) - \frac{4096 a^2}{81\pi^2 } $$ remains negative for all $a\in [4,5]$. Thus we have proved \begin{theorem} Let $\Omega_L$ be the rectangle of length $L=\sqrt{5\pi/2}$ and width $\ell=\sqrt{2\pi/5}$. Then its first eigenvalue satisfies $$\Lambda(\Omega_L) < \mu j_{1,1}^2$$ for all values of the Poisson coefficient $\nu \in [\frac{3}{8},\frac{2}{5}]$. Therefore the disk is not a minimizer in this range of values of $\nu$. \end{theorem} \begin{remark} Let us explain why we chose the association of $\varphi_1$ and $\varphi_2$ as test functions. The aim is to get a cross product coming from the divergence term strong enough to make the first eigenvalue of the matrix $\mathcal{M}$ as small as possible. It turns out that a choice of the two first eigenfunctions of the rectangle would not realize this and a simple analysis convince us that our choice was the better. \end{remark} \subsection{The case of ellipses} In the case where the domain $\Omega$ is an ellipse, we do not have an explicit expression for the eigenvalues (nor a good upper estimate) and eigenfunctions, as we did in Sections~\ref{secrhombi} and \ref{secrectangle}. Therefore, we have aimed to extend the previous analysis using numerical simulations, in which we computed an estimate of the eigenvalue $\Lambda(\Omega_a)$, where $\Omega_a$ denotes the ellipse with semi-axes $a$ and $1/a$. For \( a = 1 \), this corresponds to the eigenvalue of the disk, equal to \( \mu j_{1,1}^2 \) as long as \( \nu \geq 0.35 \), as stated in Theorem~\ref{theopoisson}. Numerical observations summarized on Figure~\ref{fig:caseEllipses}, performed with the software Matlab, suggest that the disk is not optimal while \( \nu \leq \bar{\nu} \), and it appears to be optimal among ellipses with area $\pi$ when \( \nu > \bar{\nu} \) where $\bar{\nu}$ is a numerical value very close to $0.41$. \begin{figure}[htbp] \centering \subfigure[Case $\nu=0.39$]{\includegraphics[width=0.32\textwidth]{fig/eig_Lame_ellipsis_nu=3.900000e-01.png}} \subfigure[Case $\nu=0.40$]{\includegraphics[width=0.32\textwidth]{fig/eig_Lame_ellipsis_nu=4.000000e-01.png}} \subfigure[Case $\nu=0.405$]{\includegraphics[width=0.32\textwidth]{fig/eig_Lame_ellipsis_nu=4.050000e-01.png}}\\ \subfigure[Case $\nu=0.41$]{\includegraphics[width=0.32\textwidth]{fig/eig_Lame_ellipsis_nu=4.100000e-01.png}} \subfigure[Case $\nu=0.42$]{\includegraphics[width=0.32\textwidth]{fig/eig_Lame_ellipsis_nu=4.200000e-01.png}} \subfigure[Case $\nu=0.45$]{\includegraphics[width=0.32\textwidth]{fig/eig_Lame_ellipsis_nu=4.500000e-01.png}} \caption{Graph of $\Lambda(\Omega_a)$ with respect to $a$. The dotted line corresponds to the first eigenvalue of the disk.} \label{fig:caseEllipses} \end{figure} \section{Conclusion}\label{secconclusion} \subsection{A conjecture} In our numerical simulations, we are not able to give a lower first eigenvalue than the disk when $\nu \geq 0.41$. For example, the best rectangles are better than the disk only when $\nu$ is less than a value not far from $0.41$. It is the same for the best ellipse. This leads us to think that it may exist a threshold value $\hat{\nu}$ such that the disk becomes the solution of our minimization problem when $\nu \geq \hat{\nu}$. An heuristic argument that supports this conjecture is the following: \begin{enumerate} \item First we prove in Section~\ref{secgamma} below that the eigenvalues of the Lam\'e system converge to the eigenvalues of the Stokes system when $\nu \to 1/2$. \item If we assume that the disk minimizes the first Stokes eigenvalue in the plane (this is another conjecture as explained in \cite{henrot-mazari-privat}), \item if we could then use the local minimality of the disk for our problem in a strong sense (for example for the Hausdorff convergence), \end{enumerate} we would get the result. Indeed, by the $\Gamma$-convergence result stated below, we see that the minimizer for the Lam\'e system must converge to the minimizer for the Stokes system. Therefore, for $\nu$ close enough to $1/2$, the minimizer should enter in the neighborhood of the disk where the disk is the solution. This could provide the expected result. \subsection{$\Gamma$-convergence as $\nu \to 1/2$}\label{secgamma} As explained just above, it is interesting to prove that when $\nu\to 1/2$, the eigenvalues of the Lam\'e system converge to those of the Stokes system. For that purpose we will renormalize the eigenvalue $\Lambda(\Omega)$ and work with the parameter $a:=\frac{\lambda+\mu}{\mu}=\frac{1}{1-2\nu}$ that we consider satisfying $a\to+\infty$. In other words the right quantity to study becomes $$\Lambda^a(\Omega):=\frac{1}{\mu}\Lambda(\Omega)=\min_{u\in H^1_0(\Omega) \setminus \{0\}} \frac{\int_{\Omega}|\nabla u|^2 \;dx + a \int_{\Omega} ({\rm div}(u))^2\;dx}{\int_{\Omega} |u|^2 \;dx}.$$ In this section we would like to investigate the limiting behavior as $a \to +\infty$ (or equivalently $\nu\to 1/2$) of $\Lambda^a(\Omega)$. In particular we will show that for $\Omega$ fixed, $\Lambda^{a}(\Omega)$ converges to the Stokes eigenvalue, and moreover under some standard geometrical restrictions on the admissible sets $\Omega$, the shape functional $\Omega \mapsto \Lambda^a(\Omega)$ $\Gamma$-converges to $\Omega \mapsto \lambda_1^{\rm Stokes}(\Omega)$. Here, $\lambda_1^{\rm Stokes}(\Omega)$ is the Stokes eigenvalue already defined in Section \ref{remark1} as $$\lambda_1^{\rm Stokes}(\Omega):=\min_{u\in H^1_0(\Omega) \setminus \{0\} \text{ s.t. } {\rm div}(u)=0} \frac{\int_{\Omega}|\nabla u|^2 \;dx }{\int_{\Omega} |u|^2 \;dx}. $$ We first establish the convergence of $\Lambda^a(\Omega)$ for $\Omega$, fixed. For this purpose we define the following two quadratic forms on $H^1_0(\Omega)$ : $$ Q_a(u):= \int_{\Omega}|\nabla u|^2 \;dx + a\int_{\Omega} ({\rm div}(u))^2\;dx. $$ $$ Q_{\infty}(u):= \left\{ \begin{array}{cc} \int_{\Omega}|\nabla u|^2 \;dx & \text{ if } {\rm div}(u)=0\\ +\infty & \text{ otherwise}. \end{array}\right. $$ \begin{proposition}\label{gamma1} Let $\Omega$ be a bounded open set. Then $Q_a$ $\Gamma$-converges to $Q_\infty$ for the $L^2$ topology. when $a\to +\infty$ As a consequence, the associated Lam\'e operator converges in the strong resolvent sense to the Stokes operator and in particular $$\lim_{a \to \infty} \Lambda^a (\Omega)= \lambda_1^{\rm Stokes}(\Omega).$$ \end{proposition} \begin{proof} The proof is somehow standard. Let us write the details. \\ \emph{Step 1. $\Gamma$-limsup}. Let $u \in H^1_0(\Omega)$ be such that $Q_\infty(u)<+\infty$ (otherwise there is nothing to prove). Then we take as a recovery sequence the constant sequence $u_\lambda=u$ and we use that ${\rm div}(u)=0$, together with Korn inequality to deduce that $$Q_a(u)=Q_{\infty}(u)$$ and a fortiori, $$\limsup_{a \to +\infty} Q_a(u)=Q_{\infty}(u),$$ which directly proves the limsup inequality.\\ \emph{Step 2. $\Gamma$-liminf}. Assume that $u_a \to u$ in $L^2(\Omega)$. We can assume that $$\sup_{a} Q_a(u_a) \leq C,$$ otherwise there is nothing to prove. But this means thanks to Korn inequality, that $u_a$ is uniformly bounded in $H^1(\Omega)$ thus converges weakly in $H^1$ and strongly in $L^2$, up to a subsequence, to some function $u \in H^1_0(\Omega)$ and $$\int_{\Omega} |\nabla u|^2 \leq \liminf_{a} \int_{\Omega} | \nabla u_a|^2 \;dx.$$ Passing to the liminf in the inequality $$ \int_{\Omega} ({\rm div}(u))^2\;dx\leq \frac{C}{a},$$ we deduce that ${\rm div}(u )= 0$ thus $$Q_{\infty}(u)= \int_{\Omega} |\nabla u|^2 dx $$ which finishes the liminf inequality, and the proof of $\Gamma$-convergence. Then the end of the statement of the Proposition follows from the standard theory of $\Gamma$-convergence that asserts that $\Gamma$-convergence of quadratic forms implies the convergence in the strong resolvent sense of the associated operators (see \cite[Chapter 12]{dalmasoGamma}). A review of these properties can also be found in \cite[Section 1.1]{PabloAntoine}. In particular, the convergence of the eigenvalues follows from the fact that the associated operators have compact resolvent. More precisely, we first notice that the quadratic forms $Q_a$ and $Q_\infty$ are equi-coercive thanks to Poincar\'e-Korn inequality. They are also semi-continuous with respect to the $L^2$ topology. The associated operators are thus self-adjoint, invertible and thanks to the compact embedding of $H^1_0(\Omega)$ into $L^2(\Omega)$, their inverse are compact operators. We then apply Proposition 7 in \cite{PabloAntoine} with $X=H^1_0(\Omega)$ and $H=L^2(\Omega)$ which establishes the convergence of the spectrum for the inverse operators, from the $\Gamma$-convergence of $Q_a$ to $Q_\infty$. The spectrum of the operators itself then follows immediately. \end{proof} Now we consider the $\Gamma$-convergence of $\Lambda^a(\Omega)$ but with respect to the variable $\Omega$. For simplicity we will work in the restricted class of domains $\Omega$ that are uniformly Lipschitz, more precisely that satisfies a uniform $\varepsilon$-cone property (see \cite[Definition 2.4.1]{zbMATH06838450}). We will endow this class with the complementary Hausdorff distance (see \cite[Definition 2.2.8]{zbMATH06838450}) and we already know that the Dirichlet problem is stable along any such converging sequence in this class (\cite[Theorem 3.2.13]{zbMATH06838450}) which will help a lot in the following proposition. \begin{proposition} Let $D\subset \R^N$, $\varepsilon_0>0$ and $V>0$, fixed, and let $\mathcal{A}_0$ be the class of domains $\Omega \subset D$ that satisfy the $\varepsilon_0$-cone property together with the further constraint $|\Omega|=V$. Then as $a\to +\infty$, the family of functional $\Lambda^a :\mathcal{A} \to \R$, $\Gamma$-converges to $\lambda_1^{\rm Stokes}$ with respect to the complementary Hausdorff distance. \end{proposition} \begin{proof} \emph{Step 1. $\Gamma$-limsup}. For $\Omega \in \mathcal{A}$ being given we take the constant sequence $\Omega_a=\Omega$ as a recovery sequence. Thanks to Proposition \ref{gamma1} we know that $$\lim_{a\to +\infty} \Lambda^a (\Omega)= \lambda_1^{\rm Stokes}(\Omega),$$ which proves the $\Gamma$-limsup property.\\ \emph{Step 2. $\Gamma$-liminf}. Let $\Omega_a$ converging to $\Omega$ for the complementary Hausdorff distance. Since $\mathcal{A}$ is closed for the complementary Hausdorff convergence (see \cite[Theorem 2.4.10]{zbMATH06838450}), it follows that $\Omega \in \mathcal{A}$, and it is easily seen that $|\Omega|=V$. Let $u_a$ be a sequence of normalized eigenfunctions, associated to $\Lambda^a(\Omega_a)$. In particular, $u_a \in H^1_0(\Omega_a)$ and $$\Lambda^a(\Omega_a)= \int_{\Omega_\lambda}|\nabla u_a|^2 \;dx + a \int_{\Omega_a} ({\rm div}(u_a))^2\;dx.$$ We may assume without loss of generality that $(\Lambda^a(\Omega_a))_{a}$ is a bounded sequence. Therefore, the sequence $u_a$ is uniformly bounded in $H^1_0(D)$ and converges up to a subsequence (not relabelled) to a function $u \in H^1_0(D)$, weakly in $H^1(D)$ and strongly in $L^2(D)$. In particular $\|u\|_{L^2(D)}=1$. Moreover by the Mosco convergence of $\Omega_a$ (see \cite{zbMATH06838450}) we know that $u \in H^1_0(\Omega)$. Finally, the bound on $\Lambda^a(\Omega_a)$ tells us that $$\int_{\Omega_a}({\rm div}(u_a))^2 \;dx \leq \frac{C}{a}\to_{a\to +\infty} 0,$$ and we deduce from the weak convergence of $u_a$ to $u$ in $H^1(D)$ that ${\rm div}(u)=0$ thus $u$ is an admissible competitor for the Rayleigh quotient that defines $\lambda_1^{\rm Stokes}(\Omega)$. Therefore, the following sequence of inequalities holds: \begin{eqnarray} \lambda_1^{\rm Stokes}(\Omega)&\leq & \int_{\Omega }|\nabla u|^2 \;dx \notag \\ &\leq & \liminf \int_{D}|\nabla u_a|^2 \;dx \notag \\ &\leq & \liminf \int_{D}|\nabla u_a|^2 \;dx + a \int_{D} ({\rm div}(u_a))^2\;dx \notag \\ &= & \liminf \Lambda^a(\Omega_a), \notag \end{eqnarray} which finishes the proof of the liminf inequality, and so follows the proof of $\Gamma$-convergence. \end{proof} \section*{Acknowledgments} This work is partially supported by the ANR project STOIQUES financed by the French Agence Nationale de la Recherche (ANR). We would like to warmly thank Michael Levitin for valuable discussions and also David Krejcirik and Davide Buoso for interesting feedback on a preliminary version of the paper. \bibliographystyle{abbrv} \bibliography{biblio_korn} \end{document}
2412.06527v1
http://arxiv.org/abs/2412.06527v1
Higher genus Gromov-Witten theory of one-parameter Calabi-Yau threefolds II: Feynman rule and anomaly equations
\documentclass[10pt]{amsart} \pdfoutput=1 \usepackage{amsmath} \usepackage{amssymb} \usepackage{amsthm} \usepackage{bm} \usepackage{accents} \usepackage{mathtools} \usepackage{tikz} \usetikzlibrary{calc} \usetikzlibrary{decorations.pathmorphing,shapes} \usetikzlibrary{automata,positioning,quotes} \usepackage{tikz-cd} \usepackage{forest} \usepackage{braket} \usepackage{listings} \usepackage{mdframed} \usepackage{verbatim} \usepackage{physics2} \usephysicsmodule{ab,ab.legacy,diagmat,xmat} \usepackage{derivative} \usepackage{fixdif} \usepackage{stmaryrd} \usepackage[mathscr]{eucal} \usepackage{stackengine} \usepackage{microtype} \usepackage{algorithmicx} \usepackage{algpseudocode} \usepackage{algorithm} \usepackage[english]{babel} \usepackage[backend=biber,style=alphabetic,maxalphanames=4,maxnames=5,hyperref]{biblatex} \usepackage[bookmarks, colorlinks, breaklinks]{hyperref} \hypersetup{linkcolor=blue,citecolor=magenta,filecolor=black,urlcolor=blue} \usepackage{cleveref} \usepackage{xurl} \usepackage{graphicx} \graphicspath{{./}} \crefname{equation}{}{} \usepackage{float} \usepackage{booktabs} \usepackage[shortlabels]{enumitem} \setitemize{noitemsep} \usepackage{csquotes} \newlist{mydescription}{description}{1} \setlist[mydescription]{style=nextline, font=\bfseries, labelindent=1cm, leftmargin =2cm, rightmargin=1cm, topsep =1ex } \usepackage{lipsum} \DeclarePairedDelimiter{\gen}{\langle}{\rangle} \DeclarePairedDelimiter{\floor}{\lfloor}{\rfloor} \DeclarePairedDelimiter{\ceil}{\lceil}{\rceil} \newtheorem{thm}{Theorem}[section] \newtheorem{cor}[thm]{Corollary} \newtheorem{prop}[thm]{Proposition} \newtheorem{lem}[thm]{Lemma} \newtheorem{conj}[thm]{Conjecture} \newtheorem{quest}[thm]{Question} \newtheorem{claim}[thm]{Claim} \newtheorem{fthm}[thm]{``Theorem''} \theoremstyle{definition} \newtheorem{defn}[thm]{Definition} \newtheorem{defns}[thm]{Definitions} \newtheorem{con}[thm]{Construction} \newtheorem{exm}[thm]{Example} \newtheorem{exms}[thm]{Examples} \newtheorem{notn}[thm]{Notation} \newtheorem{notns}[thm]{Notations} \newtheorem{conv}[thm]{Convention} \newtheorem{addm}[thm]{Addendum} \newtheorem{exer}[thm]{Exercise} \theoremstyle{remark} \newtheorem{rmk}[thm]{Remark} \newtheorem{rmks}[thm]{Remarks} \newtheorem{warn}[thm]{Warning} \newtheorem{sch}[thm]{Scholium} \theoremstyle{plain} \newtheorem*{thm*}{Theorem} \newtheorem*{prop*}{Proposition} \newtheorem*{lem*}{Lemma} \newtheorem*{cor*}{Corollary} \newtheorem*{conj*}{Conjecture} \theoremstyle{definition} \newtheorem*{defn*}{Definition} \newtheorem*{exer*}{Exercise} \newtheorem*{defns*}{Definitions} \newtheorem*{con*}{Construction} \newtheorem*{exm*}{Example} \newtheorem*{exms*}{Examples} \newtheorem*{notn*}{Notation} \newtheorem*{notns*}{Notations} \newtheorem*{addm*}{Addendum} \theoremstyle{remark} \newtheorem*{rmk*}{Remark} \newcommand{\Ima}{\mathrm{Im}} \newcommand{\A}{\mathbb{A}} \newcommand{\G}{\mathbb{G}} \newcommand{\N}{\mathbb{N}} \newcommand{\R}{\mathbb{R}} \newcommand{\C}{\mathbb{C}} \newcommand{\Z}{\mathbb{Z}} \newcommand{\Q}{\mathbb{Q}} \newcommand{\F}{\mathbb{F}} \newcommand{\W}{\mathcal{W}} \newcommand{\bd}{\mathbf{d}} \newcommand{\ba}{\mathbf{a}} \newcommand{\bb}{\mathbf{b}} \newcommand{\bA}{\mathbf{A}} \newcommand{\bB}{\mathbf{B}} \newcommand{\bx}{\mathbf{x}} \newcommand{\bt}{\mathbf{t}} \newcommand{\bp}{\mathbf{p}} \newcommand{\bO}{\mathbf{0}} \renewcommand{\k}{\Bbbk} \renewcommand{\L}{\mathbb{L}} \renewcommand{\P}{\mathbb{P}} \newcommand{\E}{\mathbb{E}} \newcommand{\M}{\mathcal{M}} \newcommand{\Mbar}{\overline{\mathcal{M}}} \newcommand{\g}{\mathfrak{g}} \newcommand{\h}{\mathfrak{h}} \newcommand{\n}{\mathfrak{n}} \newcommand{\fr}{\mathfrak{r}} \renewcommand{\b}{\mathfrak{b}} \newcommand{\ep}{\varepsilon} \newcommand*{\dt}[1]{ \accentset{\mbox{\Huge\bfseries .}}{#1}} \newcommand{\mc}[1]{\mathcal{#1}} \newcommand{\msc}[1]{\mathscr{#1}} \newcommand{\T}{\mathbf{T}} \newcommand{\mf}[1]{\mathfrak{#1}} \newcommand{\mbf}[1]{\mathbf{#1}} \newcommand{\mr}[1]{\mathrm{#1}} \newcommand{\on}[1]{\operatorname{#1}} \newcommand{\ms}[1]{\mathsf{#1}} \newcommand{\mt}[1]{\mathtt{#1}} \newcommand{\ol}[1]{\overline{#1}} \newcommand{\ul}[1]{\underline{#1}} \newcommand{\wt}[1]{\widetilde{#1}} \newcommand{\wh}[1]{\widehat{#1}} \renewcommand{\div}{\operatorname{div}} \newcommand{\1}{\mathbf{1}} \newcommand{\2}{\mathbf{2}} \newcommand{\3}{\mathbf{3}} \newcommand{\I}{\mathrm{I}} \newcommand{\II}{\mr{I}\hspace{-1.3pt}\mr{I}} \newcommand{\III}{\mr{I}\hspace{-1.3pt}\mr{I}\hspace{-1.3pt}\mr{I}} \renewcommand{\v}{\mbf{v}} \newcommand{\w}{\mbf{w}} \newcommand{\bmu}{\bm{\mu}} \newcommand{\pre}{\mr{pre}} \newcommand{\vir}{\mr{vir}} \newcommand{\fl}{\mr{fl}} \newcommand{\pt}{\mr{pt}} \newcommand{\loc}{\mr{loc}} \newcommand{\sloop}{\ms{loop}} \newcommand{\ssmall}{\ms{small}} \newcommand{\extra}{\ms{extra}} \newcommand{\tw}{\mr{tw}} \renewcommand{\top}{\mr{top}} \newcommand{\ps}[1]{\llbracket #1 \rrbracket} \DeclareMathOperator{\Der}{Der} \DeclareMathOperator{\Tor}{Tor} \DeclareMathOperator{\Hom}{Hom} \DeclareMathOperator{\End}{End} \DeclareMathOperator{\Ext}{Ext} \DeclareMathOperator{\ad}{ad} \DeclareMathOperator{\Aut}{Aut} \DeclareMathOperator{\Rad}{Rad} \DeclareMathOperator{\Pic}{Pic} \DeclareMathOperator{\NS}{NS} \DeclareMathOperator{\supp}{supp} \DeclareMathOperator{\Supp}{Supp} \DeclareMathOperator{\depth}{depth} \DeclareMathOperator{\sgn}{sgn} \DeclareMathOperator{\spec}{Spec} \DeclareMathOperator{\Spec}{Spec} \DeclareMathOperator{\proj}{Proj} \DeclareMathOperator{\Proj}{Proj} \DeclareMathOperator{\ord}{ord} \DeclareMathOperator{\Div}{Div} \DeclareMathOperator{\Bl}{Bl} \DeclareMathOperator{\coker}{coker} \DeclareMathOperator{\ev}{ev} \DeclareMathOperator{\PF}{\msc{PF}} \DeclareMathOperator{\st}{st} \DeclareMathOperator{\Res}{Res} \DeclareMathOperator{\cl}{cl} \DeclareMathOperator{\Cont}{Cont} \numberwithin{equation}{section} \setcounter{MaxMatrixCols}{12} \addbibresource{./msp.bib} \title[Feynman rule and anomaly equations for weighted $\P^4$]{Higher genus Gromov-Witten theory of one-parameter Calabi-Yau threefolds II: Feynman rule and anomaly equations} \author{Patrick Lei} \date{\today} \begin{document} \begin{abstract} We prove the Feynman rule conjectured by Bershadsky-Cecotti-Ooguri-Vafa~\cite{bcov} and the anomaly equations conjectured by Yamaguchi-Yau~\cite{yy04} for the Gromov-Witten theory of the Calabi-Yau threefolds $Z_6 \subset \P(1,1,1,1,2)$, $Z_8 \subset \P(1,1,1,1,4)$, and $Z_{10} \subset \P(1,1,1,2,5)$. These determine the generating series $F_g$ of genus $g$ Gromov-Witten invariants recursively from the lower-genus $F_{h<g}$ up to $3g-3$ unknown parameters. \end{abstract} \maketitle \tableofcontents \section{Introduction}\label{sec:Introduction} \subsection{Historical overview}\label{sub:Historical overview} Despite its origin in theoretical physics as a duality between A-model and B-model topological string theories, mirror symmetry has sparked significant interest in mathematics, starting with the landmark paper~\cite{cdgp}, which gave predictions for the number of genus zero curves of any degree on the quintic threefold. Even though their predictions differ from the actual numbers of curves, they sparked a significant change in the field of enumerative geometry -- namely, to consider deformation-invariant virtual counts of curves. The first mathematical theory constructed to satisfy this property is Gromov-Witten theory (a formalization of A-model invariants), which was developed in symplectic topology by various authors~\cite{rtqcoh,ltsymplectic,siebertsymplecticgw,arnoldgw,ruanvirtualneighborhoods} and in algebraic geometry by Behrend-Fantechi and Li-Tian~\cite{intrinsicnc,gwalggeo,ltgwfoundation}. For physical reasons, a central problem in Gromov-Witten theory is to compute the Gromov-Witten invariants of compact Calabi-Yau threefolds. For simplicity, we will restrict to the case when $h^2 = 1$ and in particlar to those which arise as complete intersections in weighted projective spaces, of which there are $13$ examples (see~\cite[\S 4]{hkq} for a complete list). We will list (very incompletely) some historical developments in mathematics and physics: \begin{itemize} \item A theorem determining the genus zero invariants generalizing the predictions of~\cite{cdgp} was proved by Givental and Lian-Liu-Yau~\cite{eqgwinv,lly} for complete intersections in projective space and by Coates-Corti-Lee-Tseng and Wang~\cite{orbqcohwproj,mirrornonconvex} for complete intersections in weighted projective spaces. \item Bershadsky-Cecotti-Ooguri-Vafa~\cite{bcov} studied the B-model Kodaira-Spencer gravity and predicted that the generating series $F_g$ of genus $g$ Gromov-Witten invariants can be computed recursively using $F_{h<g}$ by a \textit{Feynman rule} up to a finite ambiguity. Mathematically, this corresponds to a sum over stable graphs, which index combinatorial strata of the moduli space $\ol{\msc{M}}_{g,n}$ of stable curves. \item Yamaguchi-Yau~\cite{yy04} studied the B-model theory further and predicted that a normalized generating series $P_g$ of genus $g$ Gromov-Witten invariants is a polynomial in five explicit generators for $Z_5 \subset \P^4$, $Z_6 \subset \P(1^4,2)$, $Z_8 \subset \P(1^4,4)$, and $Z_{10} \subset \P(1^3,2,5)$. They also predicted that these polynomials satisfy differential equations in the generators, which we will refer to as \textit{Anomaly Equations}.\footnote{These are typically referred to as Holomorphic Anomaly Equations in the mathematics literature, which is technically inaccurate. In physics, the B-model partition function $F_g^{\bB}(t, \bar{t})$ has an antiholomorphic part, which is governed by the physical Holomorphic Anomaly Equations.} These predictions were extended to the other examples by Huang-Klemm-Quackenbush~\cite{hkq}. \item In mathematics, exact formulae for the genus $1$ invariants were proved for complete intersections in projective spaces by Zinger and Popa~\cite{reducedgenus1,popagenus1ci} and for hypersurfaces in weighted projective spaces by the author~\cite{polynomiality}. \item An exact formula for the genus $2$ invariants of the quintic was proved by Guo-Janda-Ruan~\cite{genus2logglsm} pending the proof of foundational results in logarithmic geometry. This was followed by a proof~\cite{bcovlogglsm} of the Anomaly Equations and finite generation conjecture, as well as some other results, for the quintic threefold. \item The Feynman rule, Anomaly Equations, and finite generation conjecture were proved independently by Chang-Guo-Li~\cite{nmsp2,nmsp3} using the theory of Mixed-Spin-P (MSP) fields developed in~\cite{mspfermat,msp2,msp3,nmsp}.\footnote{A parameter $N$, which is a positive integer, was introduced in~\cite{nmsp} and the resulting theory was originally referred to as $N$-Mixed-Spin-P (NMSP) fields. In this paper, we will simply refer to the theory with arbitrary $N$ as MSP fields.} Using a slight generalization of their approach, the author proved the finite generation conjecture~\cite{polynomiality} for hypersurfaces in weighted projective space. \end{itemize} \subsection{Setup and past work}\label{sub:Setup and past work} Let $\ba = (1,1,1,1,2)$, $(1,1,1,1,4)$, or $(1,1,1,2,5)$, $k \coloneqq \sum_{i=1}^5 a_i$, and set $p_k \coloneqq \frac{k}{a_1\cdots a_5}$. The $I$-function of $Z = Z_k \subset \P(\ba)$ is given by \begin{align*} I(q,z) &\coloneqq z \sum_{d \geq 0} q^d \frac{\prod_{m=1}^{kd} (kH+mz)}{\prod_{i=1}^5 \prod_{m=1}^{a_i d} (a_i H + mz)} \\ &\eqqcolon I_0(q) z + I_1(q) H + I_2(q) \frac{H^2}{z} + I_3(q) \frac{H^3}{z^2}, \end{align*} where $H = c_1(\msc{O}_{\P(\ba)}(1))$. If we set $r \coloneqq \frac{k^k}{a_1^{a_1} \cdots a_5^{a_5}}$, then $I(q,z)$ has radius of convergence $\frac{1}{r}$. \begin{rmk} In~\cite{eqgwinv} and other work, the $I$-function differs from ours by a prefactor of $q^{\frac{H}{z}}$. In particular, applying $zq\odv{}{q}$ to the usual conventions corresponds to applying $H+zq\odv{}{z}$ to our $I$-function. Our choice is natural from the perspective of quasimap theory. There, our $I$-function is obtained by localization on the stacky loop space~\cite[Proposition 4.9]{orbqmap} and appears in the wall-crossing formula of~\cite{qmapwc}. \end{rmk} Define $D \coloneqq q \odv{}{q}$ and define \[ I_{11} \coloneqq 1 + D\ab(\frac{I_1(q)}{I_0(q)}). \] Yamaguchi-Yau~\cite{yy04} defined infinitely many generators \begin{align*} A_m \coloneqq \frac{D^m I_{11}}{I_{11}}, \qquad B_m \coloneqq \frac{D^m I_{0}}{I_{0}}, \qquad Y \coloneqq \frac{1}{1-rq}, \qquad \text{and} \qquad X \coloneqq 1-Y. \end{align*} For simplicity, denote $A \coloneqq A_1$ and $B \coloneqq B_1$. \begin{rmk} Our generators differ from the generators in~\cite{yy04,hkq} slightly. The variables differ by $\psi^{\ms{HKQ}} = (rq)^{-1}$, and the generators in~\cite{hkq} are defined by \[ A_m^{\ms{HKQ}} \coloneqq \frac{\ab(\psi^{\ms{HKQ}} \odv{}{\psi^{\ms{HKQ}}})^m (q I_{11})}{qI_{11}} \qquad \text{and} \qquad B_m^{\ms{HKQ}} \coloneqq \frac{\ab(\psi^{\ms{HKQ}} \odv{}{\psi^{\ms{HKQ}}})^m I_0}{I_0}. \] In particular, we have the relations \begin{align*} A_m = (-1)^m \ab(\sum_{j=0}^m \binom{m}{j} A_j^{\ms{HKQ}}) \qquad \text{and} \qquad B_m = (-1)^m B_m^{\ms{HKQ}}. \end{align*} \end{rmk} \begin{lem}[\cite{yy04}] The ring $\msc{R} \coloneqq \Q[A,B,B_2,B_3,Y]$ contains all $A_m$ and $B_m$ for $m \geq 0$. In particular, we have the relations \begin{align*} A_2 &= 2B^2 - 2AB - 4 B_2 - X (A + 2B + r_0); \\ B_4 &= -X (2B_3 + (1+r_0)B_2 + r_0 B + r_1), \end{align*} where $r_0$ and $r_1$ are given in~\Cref{tab:r0andr1}. \begin{table}[htpb] \centering \caption{Values of $r_0$, $r_1$, $a_{0,k}$, and $a_{1,k}$ for different $k$} \label{tab:r0andr1} \begin{tabular}{ccccc} \toprule $k$ & $r_0$ & $r_1$ & $a_{0,k}$ & $a_{1,k}$ \\ \midrule $6$ & $\frac{13}{36}$ & $\frac{5}{162}$ & $\frac{1}{2}$ & $-\frac{7}{4}$ \\ \\[-0.8em] $8$ & $\frac{11}{32}$ & $\frac{105}{4096}$ & $\frac{1}{3}$ & $-\frac{11}{6}$ \\ \\[-0.8em] $10$ & $\frac{3}{10}$ & $\frac{189}{10000}$ & $\frac{1}{6}$ & $-\frac{17}{12}$ \\ \bottomrule \end{tabular} \end{table} \end{lem} We now define the generating function \[ F_g(Q) \coloneqq \delta_{g,0} a_{0,k} (\log Q)^3 + \delta_{g,1} a_{1,k} \log Q + \sum_d N_{g,d} Q^d, \] where the values of $a_{0,k} = \frac{1}{6} \int_Z H^3$ and $a_{1,k} = -\frac{1}{24} \int_Z c_2(Z) \cup H$ are given in~\Cref{tab:r0andr1}. This is not quite so well-behaved, so we will normalize it by defining \[ P_{g,m} \coloneqq \frac{(p_k Y)^{g-1} I_{11}^n}{I_0^{2g-2}} \ab(Q \odv{}{Q})^m F_g(Q) \Bigg|_{Q = qe^{\frac{I_1}{I_0}}} \] for any $(g,m)$ satisfying $2g-2+m > 0$. These satisfy the recursive relation \[ P_{g,m+1} = (D+(g-1)(2B+X)-mA)P_{g,m} \] and therefore can be computed from $P_{0,3} = 1$, $P_{1,1}$, and $P_{g \geq 2}$. The main result of our previous work~\cite{polynomiality} is the following: \begin{thm} For any $(g,m)$ satisfying $2g-2+m > 0$, $P_{g,m} \in \msc{R}$. \end{thm} We also proved the following exact formula for the genus $1$ invariants of $Z$: \begin{thm} We have \[ P_{1,1} = -\frac{1}{2}A + \ab(\frac{\chi(Z)}{24} - 2) B - \frac{1}{12}X + a_{1,k}. \] For clarity, note that $\chi(Z_6) = -204$, $\chi(Z_8) = -296$, and $\chi(Z_{10}) = -288$. \end{thm} \subsection{Feynman rule}\label{sub:Feynman rule} \begin{defn}\label{defn:propogators} Define the physicists' \textit{propogators} by the formulae \begin{align*} E_{\psi} &\coloneqq B_1, \\ E_{\varphi\varphi} &\coloneqq A_1 + 2 B_1, \\ E_{\varphi\psi} &\coloneqq -B_2, \\ E_{\psi\psi} &\coloneqq -B_3 + (B_1 - X)B_2 - r_0 B_1 X. \end{align*} For a choice of ``gauge'' $\G \coloneqq (c_{11}, c_{12}, c_2, c_3)$, where $c_{11},c_{12} \in \Q[X]_{1}$, $c_2 \in \Q[X]_{2}$, and $c_3 \in \Q[X]_{3}$, define \begin{align*} E_{\psi}^{\G} &\coloneqq E_{\psi} + c_{11}, \\ E_{\varphi\varphi}^{\G} &\coloneqq E_{\varphi\varphi} + c_{12}, \\ E_{\varphi\psi}^{\G} &\coloneqq E_{\varphi\psi} - c_{12}B_1 + c_2, \\ E_{\psi\psi}^{\G} &\coloneqq E_{\psi\psi} + c_{12} B_1^2 - 2 c_2 B_1 + c_3. \end{align*} \end{defn} Define $I_{22} \coloneqq 1 + D\ab(\frac{D\ab(\frac{I_2}{I_0}) + \frac{I_0}{I_0}}{I_{11}})$ and $I_{33} \coloneqq I_{11}$. Let $\varphi_i = I_0 \cdots I_{ii} H^i$ for $i = 0,1,2,3$ and let $\psi$ denote the ancestor class on $\ol{\msc{M}}_{g,n}$. \begin{defn} Define the B-model Gromov-Witten correlators $P_{g,m,n}$ by \[ P_{g,m,n} \coloneqq \begin{cases} (2g+m+n-3)_n P_{g,m} & 2g-2+m > 0 \\ (n-1)! \ab(\frac{\chi(Z)}{24}-1) & (g,m) = (1,0). \end{cases} \] Here, $(2g+m+n-3)_n$ is the falling Pochhammer symbol. \end{defn} Note that these agree with the GW invariants \[ \frac{(p_k Y)^{m-1}}{I_0^{2g-2+m+n}} \ab<\varphi_1^{\otimes m}, (\varphi_0\psi)^{\otimes n}>_{g,m+n}^Z \] whenever $(g,m) \neq (1,0)$, in which case the GW invariants is $(n-1)! \frac{\chi(Z)}{24}$. Later, we will discover the meaning of this term. \begin{defn}\label{defn:bmodelfeynman} Let $G_{g,\ell}$ be the set of all stable graphs of genus $g$ and $\ell$ legs and define \[ f_{g,m,n}^{\bB,\G} \coloneqq \sum_{\Gamma \in G_{g,m+n}} \frac{\on{Cont}_{\Gamma}^{\bB,\G}}{\ab|\Aut \Gamma|}, \] where the contribution of a stable graph is defined by the following construction: \begin{itemize} \item At each leg, we place $\varphi_1 - E_{\psi}^{\G}\varphi_0 \psi$ or $\varphi_0 \psi$; \item At each edge, we place the bivector \[ V_{\bB,\G} \coloneqq E_{\varphi\varphi}^{\G} \varphi_1 \otimes\varphi_1 + E_{\varphi\psi}^{\G} (\varphi_1\otimes \varphi_0\psi + \varphi_0\psi\otimes \varphi_1) + E_{\psi\psi}^{\G} \varphi_0\psi\otimes\varphi_0\psi; \] \item At each vertex, we place the linear map \[ \varphi_1^{\otimes m} \otimes (\varphi_0 \psi)^{\otimes n} \mapsto P_{g,m,n}. \] \end{itemize} \end{defn} The main result of this paper is the following polynomiality result for $f_{g,m,n}^{\bB}$. \begin{thm}[\Cref{cor:bmodelfeynman}]\label{thm:bmodelintro} For any $g,m,n$ and any choice of gauge $\G$, we have \[ f^{\bB,\G}_{g,m,n} \in \Q[X]_{3g-3+m}. \] \end{thm} If we specialize to $m=n=0$, then $G_{g,0}$ contains a leading graph $\Gamma_0$ with exactly one vertex (of genus $g$) and no edges. This graph contributes $P_g$, while other graphs contribute linear combinations of products of $P_{h<g,m,n}$ and the propogators. Therefore, if we know all $P_{h<g,m,n}$, the formula \[ P_g = f_g^{\bB} - \sum_{\Gamma \neq \Gamma_0} \frac{1}{\ab|\Aut \Gamma|} \on{Cont}^{\bB}_{\Gamma} \] implies that to compute $P_g$, it suffices to compute $f_g^{\bB} \in \Q[X]_{3g-3}$. Because the degree $0$ invariant \[ N_{g,0} = \frac{(-1)^g \chi(Z) \cdot \ab|B_{2g}| \cdot \ab|B_{2g-2}|}{4g \cdot (2g-2) \cdot (2g-2)!} \] was already computed by Faber-Pandharipande~\cite{gwhodge}, this allows us to fix the constant term of $f_g^{\bB}$ as $p_k^{g-1} N_{g,0}$. We will prove this result by reconstructing it from the A-model. We begin by constructing an A-model $R$-matrix, which will act on the Gromov-Witten potential of $Z$. It is engineered such that its edge contributions match the B-model edge contributions as much as possible. \begin{defn} In the basis $\varphi_0, \ldots, \varphi_3$ of $\msc{H}_Z$, define the $A$-model $R$-matrix by \[ R^{\bA,\G}(z)^{-1} \coloneqq I - \begin{pmatrix} 0 & z E_{\psi}^{\G} & z^2 E_{\varphi\psi}^{\G} & z^3 E_{1\psi^2}^{\G} \\ & 0 & z E_{\varphi\varphi}^{\G} & z^2 E_{1\varphi\psi}^{\G} \\ & & 0 & z E_{\psi}^{\G} \\ & & & 0 \end{pmatrix}, \] where we define $E_{1\varphi\psi}^{\G} \coloneqq -E_{\psi}^{\G} E_{\varphi\varphi}^{\G} - E_{\varphi\psi}^{\G}$ and $E_{1\psi^2}^{\G} \coloneqq -E_{\psi}^{\G} E_{\varphi\psi}^{\G} - E_{\psi\psi}^{\G}$. \end{defn} We will now define an A-model Feynman rule. \begin{defn} For $\ba \in \{0,1,2,3\}^n$ and $\bb \in \Z_{\geq 0}^n$, define \[ f_{g,m,n}^{\bA,\G} \coloneqq \sum_{\Gamma \in G_{g,m+n}} \frac{\on{Cont}_{\Gamma}^{\bA,\G}}{\ab|\Aut \Gamma|}, \] where the contribution of a stable graph is defined by the following construction: \begin{itemize} \item At each leg, we place $R^{\bA,\G}(z)^{-1} \varphi_a \psi^b$; \item At each edge, we place the bivector \begin{align*} V_{\bA,\G} \coloneqq{}& \sum_{i=0}^3 \frac{\varphi_i \otimes \varphi_{3-i} - R^{\bA}(\psi)^{-1} \varphi_i \otimes R^{\bA}(\psi')^{-1} \varphi_{3-i}}{\psi+\psi'} \\ ={}& E_{\varphi\varphi} \varphi_1 \otimes \varphi_1 + E_{\varphi\psi} (\varphi_1 \otimes \varphi_0 \psi' + \varphi_0 \psi \otimes \varphi_1) + E_{\psi\psi} (\varphi_0\psi \otimes \varphi_0\psi') \\ &+ E_{\psi} (\varphi_0 \otimes \varphi_2 + \varphi_2 \otimes \varphi_0) + E_{1\varphi\psi} (\varphi_0 \otimes \varphi_1 \psi' + \varphi_1 \psi \otimes \varphi_0) \\ &+ E_{1\psi^2} (\varphi_0 \otimes \varphi_0 (\psi')^2 + \varphi_0 \psi^2 \otimes \varphi_0). \end{align*} \item At each vertex, we place the linear map \[ \frac{(p_k Y)^{g-1}}{I_0^{2g-2+n}}\ab<->_{g,n}^Z. \] \end{itemize} \end{defn} Using the theory of MSP fields, we will prove a polynomiality result for $f_{g,\ba,\bb}^{\bA, \G}$. Along the way, we discover that the MSP $R$-matrix defined in~\cite{polynomiality} (see~\Cref{eqn:rmatrix}) factors as the product of a matrix $R^X$ which satisfies an $X$-polynomiality property and $R^{\bA, \G}$ and obtain a similar degree bound for the level $0$ part of the full MSP theory. \begin{thm}[\Cref{cor:amodelfeynman}]\label{thm:amodelintro} For any choice of gauge $\G$ and any $g,n$ such that $2g-2+n > 0$, we have \[ f_{g,\ba,\bb}^{\bA, \G} \in \Q[X]_{3g-3+n-\sum b_i}. \] \end{thm} In particular, when we set $\ba = (1^m,0^n)$ and $\bb = (0^m,1^n)$, we define \[ f_{g,m,n}^{\bA, G} \coloneqq f_{g,(1^m,0^n),(0^m,1^n)}^{\bA,\G} \in \Q[X]_{3g-3+m}. \] Note that the A-model Feynman rule has three extra edge contributions, which contain four extra variables compared to the B-model Feynman rule. Remarkably, the contributions of the correction term in $g=1$, the extra A-model edge contributions, and the four extra A-model variables cancel out to yield the following result. \begin{thm}[\Cref{thm:allgenusmirror}]\label{thm:aequalsbintro} The A-model and B-model graph sums satisfy the relation \[ f_{g,m,n}^{\bA, \G} = f_{g,m,n}^{\bB, \G} + \delta_{g,1} \delta_{m,0} (n-1)!. \] \end{thm} \subsection{Anomaly equations}\label{sub:Anomaly equations} In~\Cref{sec:Holomorphic anomaly equations}, we prove the following anomaly equations: \begin{thm}[\Cref{thm:hae}]\label{thm:haeintro} The $P_{g,m}$ satisfy the differential equations \begin{align*} - \partial_{A} P_g = \frac{1}{2} \ab(P_{g-1,2} + \sum_{g_1+g_2 = g} P_{g_1, 1} P_{g_2, 2}), \\ \ab(-2 \partial_{A} + \partial_{B} + (A+2B) \partial_{B_2} - \ab((B-X)(A+2B)-B_2-r_0 X)\partial_{B_3})P_g = 0. \end{align*} \end{thm} The first equation is proved directly by differentiating the B-model Feynman rule, while the second is equivalent to a mysterious reduction of generators (\Cref{thm:reduction}) found by Yamaguchi-Yau~\cite{yy04}. In particular, they consider particular $v_1, v_2, v_3 \in \msc{R}$ and conjecture that for all $g \geq 2$, $P_g \in \Q[v_1, v_2, v_3, X]$. \subsection{Outline}\label{sub:Outline} The paper is organized as follows: \begin{itemize} \item In \S 2, we review the construction of the MSP $[0,1]$ theory, construct the MSP $[0]$ and $[1]$ theories, and prove polynomiality of the MSP $[1]$ theory. \item In \S 3, we prove the polynomiality of the MSP $[0]$ theory and the A-model graph sum using a bootstrapping argument when $\G = (0,0,0,0)$. The A-model Feynman rule for a general gauge (\Cref{thm:amodelintro}) follows as a corollary. \item In \S 4, we rewrite both the A-model Feynman rule and B-model Feynman rule using the formalism of geometric quantization of symplectic linear transformations. We then study the effect of the extra contributions in the A-model and prove~\Cref{thm:aequalsbintro} by direct computation. The B-model Feynman rule (\Cref{thm:bmodelintro}) follows as a corollary. \item In \S 5, we prove~\Cref{thm:haeintro}. \end{itemize} \subsection{Conventions}\label{sub:Conventions} We will use the following conventions in this paper: \begin{itemize} \item We will ignore the odd cohomology of our target $Z$. All operators in this paper will preserve the $\Z/2$ grading on cohomology and are the identity on odd classes. \item The theory of MSP fields depends on a positive integer $N$. We will assume that $N$ is an odd prime. Whenever we fix $g,n$, we will always assume that $N \gg 3g-3+n$. \item We will consider $\T = (\C^{\times})^N$-equivariant invariants. Our convention is that after equivariant integration, we will specialize our equivariant parameters by $t_{\alpha} = -\zeta_N^{\alpha}t$. At various points in the paper, we will specialize $t^N = -1$. \item Whenever we compute using equivariant integration, we will make the substitution $q' = \frac{-q}{t^N}$. Note the specialization $t^N = -1$ makes $q' = q$. \end{itemize} \subsection*{Acknowledgements}\label{sub:Acknowledgements} The author is grateful to Chiu-Chu Melissa Liu for all of her helpful advice and for proposing this project. The author would like to thank Konstantin Aleshkin and Shuai Guo for helpful discussions, and Dimitri Zvonkine for his lectures about CohFTs and $R$-matrix actions at the Simons Center for Geometry and Physics in August 2023. The author would also like to thank Shuai Guo for his hospitality during the author's visit to Peking University in July 2024, when part of this work was completed. Finally, the author would like to thank Felix Janda and Yongbin Ruan for expressing interest in the results of this paper and its prequel~\cite{polynomiality}. \section{MSP $[0]$ and $[1]$ theories}\label{sec:MSP 0 and 1 theories} In this section, we will define the MSP $[0]$ theory and $[1]$ theory and prove a polynomiality property for the $[1]$ theory. We will refer the reader to~\cite{relationsvia3spin} and~\cite[\S 2, Appendix C]{nmsp3} for a discussion of CohFTs and $R$-matrix actions, including in the cases when $R_0 \neq I$ and when source and target of the $R$-matrix are different vector spaces or have different pairings. \subsection{The MSP $[0,1]$ theory}\label{sub:The MSP theory} Let $N$ be a positive integer. First, the stack $\msc{W}_{g,n,(d,0)}$ was constructed in~\cite{foundations} and MSP invariants were constructed in~\cite{polynomiality} for the state space \[ \msc{H} \coloneqq H^*(Z) \oplus \bigoplus_{\alpha=1}^N H^*(\pt_{\alpha}) \eqqcolon \msc{H}_Z \oplus \bigoplus_{\alpha}\msc{H}_{\alpha} \eqqcolon \msc{H}_Z \oplus \msc{H}_1. \] The pairing is given by \begin{align*} (x,y) &\coloneqq \int_{Z} \frac{xy}{-t^N} + \sum_{\alpha} \frac{-p_k}{N t_{\alpha}^3 t^N} xy \bigg\vert_{\pt_{\alpha}}\\ &\eqqcolon (x|_{Z},y|_{Z})^{Z, \tw} + \sum_{\alpha} (x|_{\pt_{\alpha}}, y|_{\pt_{\alpha}})^{\pt_{\alpha}, \tw}. \end{align*} We will now give several bases which we will consider in the rest of the paper. \begin{enumerate} \item We consider $Z \sqcup \bigsqcup_{\alpha=1}^N \pt_{\alpha} = (x_1^{k/a_1} + \cdots + x_5^{k/a_5} = 0)^{\T} \subset \P(\ba, 1^N)$ and let $p = c_1(\mc{O}_{\P(\ba, 1^N)}(1))$. Then define $\phi_j \coloneqq p^j$ for $j = 1, \ldots N+3$; \item Note that there is the natural basis $\{1,H,H^2, H^3\}$ of $\msc{H}_Z$ and $\{\1_{\alpha}\}_{\alpha=1}^N$ of $\msc{H}_1 \coloneqq \bigoplus_{\alpha} \msc{H}_{\alpha}$; \item We may normalize the previous basis\footnote{This is related to the transformation from flat coordinates to canonical coordinates, see~\cite[\S 5]{polynomiality}} and consider $\varphi_b = I_0 I_{11} \cdots I_{bb} H^b$, where $I_{22} = 1 + D\ab(\frac{D \ab(\frac{I_2}{I_0})+\frac{I_1}{I_0}}{I_{11}})$ and $I_{33} = I_{11}$. We will also consider $\bar{\1}_{\alpha} = L^{-\frac{N+3}{2}}\1_{\alpha}$. \end{enumerate} Before we continue, we will define several CohFTs related to $\mc{C}$ using the MSP virtual localization formula~\cite[\S 6]{foundations}. First, for any smooth projective variety $Z$, the Gromov-Witten CohFT associated to $Z$ is given by \[ \Omega_{g,n}^Z(\tau_1, \ldots, \tau_n) \coloneqq \sum_{d \in H_2(Z,\Z)} q^d \st^Z_* \ab(\prod_{i=1}^n \ev_i^*(\tau_i) \cap [\ol{\msc{M}}_{g,n}(Z,d)]^{\vir}), \] where $\st^Z \colon \ol{\msc{M}}_{g,n}(Z,d) \to \ol{\msc{M}}_{g,n}$ is the stabilization morphism and $\tau_i \in H^*(Z)$. In the MSP virtual localization formula, the contribution of a vertex at level $0$ is given by \[ (-t^N)^{-(d+1-g)} [\ol{\msc{M}}_{g,n}(Z,d)]^{\vir} \eqqcolon [\ol{\msc{M}}_{g,n}(Z,d)]^{\tw}. \] Replacing $[\ol{\msc{M}}_{g,n}(Z,d)]^{\vir}$ by $[\ol{\msc{M}}_{g,n}(Z,d)]^{\tw}$ in the formula for $\Omega^Z$, we obtain the CohFT $\Omega^{Z,\tw}$. We will need to consider a shift of the Gromov-Witten CohFT of $Z$ by the mirror map $\tau_Z(q) \coloneqq \frac{I_1(q)}{I_0(q)}H$. This is given by the formula \begin{align*} \Omega^{Z,\tau_Z(q)}_{g,n}(-) \coloneqq{}& \sum_{d,m} \frac{q^d}{m!} \Omega^Z_{g,n+m}(-,\tau_Z(q)^m) \\ ={}& \sum_{d} Q(q)^d \Omega^Z_{g,n+m}(-), \end{align*} where $Q(q) \coloneqq qe^{\frac{I_1(q)}{I_0(q)}}$ is the mirror map. For each of the isolated points $\pt_{\alpha}$, we may consider the vertex contribution \begin{align*} &[\ol{\msc{M}}_{g,n}]^{\alpha, \tw} \\ \coloneqq{} &(-1)^{1-g} \frac{p_k t_{\alpha} \cdot \prod_{i=1}^5 e_{\T}(\E_{g,n}^{\vee}\cdot (-a_i t_{\alpha})) \cdot \prod_{\beta \neq \alpha} e_{\T}(\E_{g,n}^{\vee} \cdot (t_{\beta} - t_{\alpha}))}{(-t_{\alpha})^5 \cdot e_{\T}(\E_{g,n} \cdot kt_{\alpha}) \cdot \prod_{\beta \neq \alpha} (t_{\beta} - t_{\alpha})} \cap [\ol{\msc{M}}_{g,n}]. \end{align*} These classes define a CohFT $\Omega^{\pt_{\alpha},\tw}$, and restricting to the topological part \[ [\ol{\msc{M}}_{g,n}]^{\top} = \ab(\frac{1}{p_k} N(-t_{\alpha})^{N+3})^{g-1} [\ol{\msc{M}}_{g,n}] \] gives the topological part $\omega^{\pt_{\alpha},\tw}$ of the CohFT. In~\cite[\S 2.1]{polynomiality}, we defined MSP invariants $\ab<->^M_{g,n}$ using virtual localization, whose explicit formula was proved in~\cite[\S 5]{foundations}. Recall that for any cohomological field theory, Givental's theory~\cite{symplfrob} considers the fundamental solution of the quantum differential equation (or Dubrovin connection), which is given by \[ S^M_{\tau}(z)x = x + \sum_{a, n} \frac{1}{n!} e^a \ab<\frac{x}{z-\psi}, e_a, \tau^n>_{0,n+2}^M. \] Here, $\{e_a\}$ is a basis of $\msc{H}$ and $\{e^a\}$ is its dual basis. We will now restrict to the case $\tau = 0$ and abbreviate the fundamental solution to $S^M(z)$. We also considered the corresponding fundamental solutions $S^Z \coloneqq S^Z_{\tau_Z(q)}$ where $\tau_Z = \frac{I_1(q)}{I_0(q)}H$ and $S^{\pt_{\alpha}} \coloneqq S_{\tau_{\alpha}}^{\pt_{\alpha}}$, where $\tau_{\alpha} = -t_{\alpha} \int_0^{q} (L(x)-1) \frac{\d{x}}{x}$, where $L = (1+rx)^{\frac{1}{N}}$. Finally, we defined the MSP $R$-matrix by the formula \begin{equation}\label{eqn:rmatrix} S^M(z) \begin{pmatrix} \on{diag}\ab\{ \Delta^{\pt_{\alpha}}(z)\}_{\alpha=1}^N & \\ & 1 \end{pmatrix} = R(z) \begin{pmatrix} \on{diag}\ab\{ S^{\pt_{\alpha}}(z)\}_{\alpha=1}^N & \\ & S^{Z}(z) \end{pmatrix}\biggr|_{q \mapsto q'}, \end{equation} where $\Delta^{\pt_{\alpha}}(z)$ is the Quantum Riemann-Roch~\cite{qrr} operator given by the formula \begin{align*} \Delta^{\pt_{\alpha}}(z) \coloneqq \exp &\Biggl[\sum_{m \geq 0} \frac{B_{2m}}{2m(2m-1)} \Biggl(\sum_{i=1}^5 \frac{1}{(-a_i t_{\alpha})^{2m-1}} \\ &+ \frac{1}{(kt_{\alpha})^{2m-1}} + \sum_{\beta \neq \alpha} \frac{1}{(t_{\beta} - t_{\alpha})^{2m-1}}\Biggr) z^{2m-1} \Biggr]. \end{align*} Here, the $B_{2k}$ are the Bernoulli numbers. In~\cite[\S 5]{polynomiality}, we explained how to use the explicit formula for the quantum differential equation given in~\cite[Lemma 2.18]{polynomiality} to compute the entries of $R(z)^*$ when the input basis is $\{1, p, \ldots, p^{N+3}\}$ and the output basis is $\{\varphi_0, \ldots, \varphi_3\} \cup \{\bar{\1}_{\alpha}\}_{\alpha=1}^N$. In particular, the entries are elements of $\msc{R}$ up to some normalization. \begin{defn}[{\cite[Theorem 3.6]{polynomiality}}] Define the local theory by the formula \[ \Omega^{\loc} \coloneqq \Omega^{Z,\tw} \oplus \bigoplus_{\alpha=1}^N \omega^{\pt_{\alpha},\tw} \] and the MSP $[0,1]$ theory by the formula \[ \Omega^{[0,1]} \coloneqq R.\Omega^{\loc}. \] \end{defn} \begin{rmk} Note that the $[0,1]$ theory was originally defined using virtual localization. The fixed loci of $\mc{W}_{g,n,(d,0)}$ are described using localization graphs $\Theta$ whose vertices can be partitioned as $V = V_0 \sqcup V_1 \sqcup V_{\infty}$. We then only consider those $\Theta$ for which $V_{\infty} = \emptyset$ when defining the $[0,1]$ theory. In~\cite[\S 3]{polynomiality}, we proved that it is equivalent to the definition we give here. In addition, when $N$ is very large relative to $g,n$, we obtain a simpler formula for the tail contributions at level $0$. \end{rmk} \begin{rmk} In contrast to the usual setting (see~\cite{relationsvia3spin} for example), our $R(z) = R_0 + R_1 z + \cdots$ does not satisfy $R_0 = \mr{Id}$. However, we can relate this more general case of $R$-matrix actions to the usual setting via the dilaton flow, for example see~\cite[Appendix C]{nmsp3}. \end{rmk} \subsection{The MSP $[0]$ and $[1]$ theories}\label{sub:The MSP 0 and 1 theories} We will now define restricted versions of the MSP $[0,1]$ CohFTs, which we will call the $[0]$ and $[1]$ theories. First, recall that~\cite[\S 2]{nmsp3} gives a definition of the $R$-matrix action on CohFTs when the source and target of $R$ are not the same vector space. In particular, we only need that $R(-z)^* R(z) = \mr{Id}$, which in particular implies that $R_0$ is injective. \begin{defn} Define the restricted $R$-matrices $R^{[0]}(z)$ and $R^{[1]}(z)$ by the formulae \begin{align*} R^{[0]}(z) &= R(z)|_{\msc{H}_Z}, \\ R^{[1]}(z) &= R(z)|_{\msc{H}_1}. \end{align*} \end{defn} Because the MSP $R$-matrix $R(z)$ satisfies $R(-z)^* R(z) = \mr{Id}$, $R^{[0]}(z)$ and $R^{[1]}(z)$ satisfy the identities \begin{align*} R^{[0]}(-z)^* R^{[0]}(z) &= \mr{Id}_{\msc{H}_Z}, \\ R^{[1]}(-z)^* R^{[1]}(z) &= \mr{Id}_{\msc{H}_1}, \\ R^{[0]}(-z)^* R^{[1]}(z) &= R^{[1]}(-z)^* R^{[0]}(z) = 0 . \end{align*} \begin{defn} Define the MSP $[0]$ theory by the formula \[ \Omega^{[0]} \coloneqq R^{[0]}. \Omega^{Z,\tw} \] and the MSP $[1]$ theory by the formula \[ \Omega^{[1]} \coloneqq R^{[1]}. \bigoplus_{\alpha=1}^N \omega^{\pt_{\alpha},\tw}. \] \end{defn} Our immediate goal is now to prove a polynomiality result for the MSP $[0]$ theory. We will do this by studying the MSP $[0,1]$ theory and the $[1]$ theory, which is similar to the argument used to prove the polynomiality of the $[0,1]$ theory~\cite[Theorem 4.1]{polynomiality}. We will first describe a bipartite graph decomposition of the $[0,1]$ theory, then prove the polynomiality of the $[1]$ theory. After some work, we will apply the polynomiality of the $[1]$ theory and of the $[0,1]$ theory to deduce the polynomiality of the $[0]$ theory. \begin{defn} Define $\msc{G}^{[0,1]}_{g,n}$ to be the set of stable bipartite graphs of total genus $g$ and $n$ legs. These are stable graphs with a partition $V = V_0 \sqcup V_1$ making the graph bipartite. \end{defn} \begin{thm}\label{thm:01bipartite} There is a decomposition of the MSP $[0,1]$ theory in terms of stable bipartite graphs as \begin{align*} \Omega^{[0,1]}_{g,n}(\tau_1, \ldots, \tau_n) = \sum_{\Lambda \in \msc{G}_{g,n}^{[0,1]}} & \bigotimes_{v \in V_0} \Omega^{[0]}_{g_v, n_v} \otimes \bigotimes_{v \in V_1} \Omega^{[1]}_{g_v, n_v} \\ &\ab(\bigotimes_{\substack{v \in V_0 \\ \ell \in L_v}} \tau_{\ell} \otimes \bigotimes_{\substack{v \in V_0 \\ \ell \in L_v}} \tau_{\ell} \otimes \bigotimes_{e \in E} V^{01}(\psi,\psi')), \end{align*} where we define \[V^{01}(z,w) \coloneqq \sum_{\alpha=1}^N \frac{R^{[1]}(z) - R^{[1]}(-w)}{z+w} \1_{\alpha} \otimes R^{[1]}(w) \1^{\alpha}. \] \end{thm} \begin{proof} Recall that the edge contribution in the definition of the MSP $[0,1]$ theory is given by \begin{align*} \on{Cont}_{E_{01}} &= \sum_{i=1}^{N+3} \frac{\phi_i \otimes \phi^i - R(z)^{-1} \phi_i \otimes R(w)^{-1}\phi^i}{z+w} \Bigg\vert_{\msc{H}_Z \otimes \msc{H}_1} \\ &= - \sum_{\alpha=1}^N \frac{R(-z)^* R(-w) \1_{\alpha} \otimes \1^{\alpha}}{z+w} \Bigg\vert_{\msc{H}_Z \otimes \msc{H}_1} \\ &= - \sum_{\alpha=1}^N \frac{R^{[0]}(-z)^* R^{[1]}(-w) \1_{\alpha} \otimes \1^{\alpha}}{z+w} \\ &= \sum_{\alpha=1}^N \frac{R^{[0]}(-z)^* (R^{[1]}(z) - R^{[1]}(-w))\1_{\alpha \otimes \1^{\alpha}}}{z+w} \\ &= (R^{[0]}(-z)^* \otimes R^{[1]}(-w)^* V^{01}(z,w)). \end{align*} The result then follows from the definition of the $R$-matrix action as a sum over stable graphs. \end{proof} \subsection{Polynomiality of the $[1]$ theory}\label{sub:Polynomiality of the 1 theory} Recall that we defined the edge contributions to the $[0,1]$ theory by \begin{align*} V(z,w) &= \sum_{j=0}^{N+3} \frac{\phi_j \otimes \phi^j - R(z)^{-1}\phi_j \otimes R(w)^{-1}\phi^j}{z+w} \\ &\eqqcolon \sum_{m,n} V_{mn} z^m w^n. \end{align*} \begin{lem}\label{lem:rlevel1} Let $m,n \geq 0$, $a = 0,\ldots, N+3$, and $\alpha,\beta \in \{1,\ldots,N\}$. In~\cite[\S 5.3]{polynomiality}, we defined \[ (R_m)_a^{\alpha} \coloneqq L_{\alpha}^{-a-m} (R_m \bar{\1}^{\alpha},\phi_a). \] Then \begin{enumerate} \item $(R_m)_a^{\alpha}$ is independent of $\alpha$ and $(R_m)_a^{\alpha} \in \Q[X]_{m+\floor{\frac{a}{N}}}$; \item The $V$-bivector has the form \[ V_{mn} |_{\msc{H}_1 \otimes \msc{H}_1} = L^{-3} t^N \sum_{\alpha,\beta} \sum_s L_{\alpha}^{s-m} L_{\beta}^{2-s-n} (V_{mn})^{\alpha\beta;s} \1_{\alpha} \otimes \1_{\beta}, \] where $(V_{mn})^{\alpha\beta;s} \in \Q[X]_{m+n+1}$ is independent of $\alpha,\beta$. \end{enumerate} \end{lem} \begin{proof} Note that (1) is~\cite[Lemma 5.9]{polynomiality}. The proof of (2) is the same as~\cite[Lemma C.1]{nmsp2}. \end{proof} \begin{defn} When $\star$ is either $[0]$, $[1]$, or $[0,1]$, define \[ f^{\star}_{g,(\ba,\bb)} \coloneqq \int_{\ol{\msc{M}}_{g,n}} \prod_{i=1}^n \psi_i^{b_i} \Omega_{g,n}^{\star}(\phi_{a_1}, \ldots, \phi_{a_n}) \] and the $[1]$ theory with special insertions \begin{align*} f^{[1]}_{g,(\ba,\bb)(\ba',\bb')} \coloneqq L^{\sum_{i=1}^m a_i'}& \int_{\ol{\msc{M}}_{g,n+m}} \prod_{i=1}^n \psi_i^{b_i} \prod_{j=1}^m \psi_{n+j}^{b_j'} \\ & \Omega_{g,n+m}^{[1]}(\phi_{a_1}, \ldots, \phi_{a_n}, R(\psi_{n+1}) \bar{\phi}_{a_1'},\ldots,R(\psi_{n+m}) \bar{\phi}_{a_m'}) \end{align*} for $\ba \in \{0,\ldots,N+3\}^n$, $\ba' \in \{1,\ldots,N\}^m$, $\bb \in \Z_{\geq 0}^n$, and $\bb' \in \Z_{\geq 0}^m$. Here, we define $\bar{\phi}_a \coloneqq L^{-\frac{N+3}{2}} L^a p^a|_{\msc{H}_1} = \sum_{\alpha=1}^N L_{\alpha}^a \bar{\1}_{\alpha}$. \end{defn} \begin{defn} For a tuple $\ba \in \Z_{\geq 0}^n$, define $\ab|\ba| \coloneqq \sum_{i=1}^n a_i$ and $\floor*{\frac{\ba}{N}} \coloneqq \sum_{i=1}^n \floor*{\frac{a_i}{N}}$. \end{defn} \begin{lem}\label{lem:polynomiality1theory} Let $N \gg 3g-3+n+m$. Define \[ c \coloneqq \frac{\ab|\ba| + \ab|\ba'| + \ab|\bb| + \ab|\bb'| - n-m}{N}. \] If $c \in \Z$, then \[ \ab( \frac{Y}{t^N} )^{g-1+r} f_{g,(\ba,\bb),(\ba',\bb')}^{[1]} \] is a polynomial in $X$ of degree at most $3g-3+n+m-\ab|\bb|-\ab|\bb'| + \floor*{\frac{\ba}{N}}$. Otherwise, $f_{g,(\ba,\bb),(\ba',\bb')}^{[1]} = 0$. \end{lem} \begin{proof} Recall that $f^{[1]}_{g,(\ba,\bb),(\ba',\bb')}$ is defined as a sum of stable graph contributions, where the contribution from any stable graph is computed as follows: \begin{enumerate} \item At every leg with insertion $\phi_a \psi^b$, we place \[ R^{[1]}(-\psi)^* \psi_a \psi^b = \sum_{\alpha,m} L_{\alpha}^{a-m}(-1)^m \psi^{m+b} \bar{\1}_{\alpha} = L^{-\frac{N+3}{2}}\sum_{\alpha,m} L_{\alpha}^{a-m}(-1)^m \psi^{m+b} \1_{\alpha}; \] \item At every leg with special insertion $R(\psi')\bar{\phi}_{a'}\psi^{b'}$, we place \[ R^{[1]}(-\psi)^* R(\psi) \bar{\phi}_{a'} \psi^{b'} = \sum_{\alpha} L_{\alpha}^{a'} \bar{\1}_{\alpha} \psi^{b'} = L^{-\frac{N+3}{2}} \sum_{\alpha} L_{\alpha}^{a'} \1_{\alpha}\psi^{b'}; \] \item At every edge, we place the bivector\footnote{Note that we reindex the sum from~\Cref{lem:rlevel1}.} \[ V(z,w)|_{\msc{H}_1 \otimes \msc{H}_1} = L^{-3} t^N \sum_{\alpha,\beta} \sum_{c,d,s} L_{\alpha}^{1+s-c} L_{\beta}^{1-s-d} (V_{cd})^{\alpha\beta;s+1} \1_{\alpha} \otimes \1_{\beta}; \] \item At every vertex of genus $g_v$ with $n_v$ legs, we place the map \[ \sum_{\alpha} L^{\frac{N+3}{2}(2g_v - 2 + n_v)} \sum_s \frac{1}{s!} \st^s_* \omega_{g_v,n_v+s}^{\pt_{\alpha},\tw} (-,T_{\alpha}(\psi)^s), \] where \begin{align*} T_{\alpha}(z) &= z(\1- L^{\frac{N+3}{2}} R(z)^{-1} \1) |_{\pt_{\alpha}} \\ &= \sum_{m=1}^{\infty} L_{\alpha}^{-m} (R_m)_0^{\alpha} (-z)^{m+1} \1_{\alpha} \end{align*} was defined in~\cite[Theorem 3.6]{polynomiality} and $\st^s \colon \ol{\msc{M}}_{g,n+s} \to \ol{\msc{M}}_{g,n}$ is the morphism forgetting the last $s$ marked points. Recall from~\cite[Lemma 2.4]{polynomiality} that \[ \omega_{g,n}^{\pt_{\alpha},\tw} = \ab(\frac{1}{p_k}N(-t_{\alpha})^{N+3})^{g-1} \Omega_{g,n}^{\pt}. \] \end{enumerate} We will now count the degrees of the contributions at each vertex labeled by $\pt_{\alpha}$. Let $L_v$ be the set of ordinary (first $n$) legs attached to $v$ and $L_v'$ be the set of special legs attached to $v$. The factor involving $L_{\alpha}$, $L$, and $Y$ is \begin{align*} L_{\alpha}^{(N+3)(g_v-1)} \prod_{\ell \in L_v} L_{\alpha}^{a_{\ell}-c_{\ell}} \prod_{\ell' \in L_v'} L_{\alpha}^{a'_{\ell'}} \prod_{\substack{f =(e,v)\\e \in E_v}} t^{\frac{N}{2}} L^{\frac{N}{2}} L_{\alpha}^{1+s_f-c_f} \prod_{\ell'' \in \mr{tails}} L_{\alpha}^{-c_{\ell''}} \end{align*} Using the fact that \begin{align*} \sum_{\ell' \in \mr{tails}} c_{\ell''} + \sum_{\substack{f = (e,v) \\ e \in E_v}} c_f + \sum_{\ell \in L_v} c_{\ell} + b_{\ell} + \sum_{\ell' \in L_v'} b'_{\ell'} &= 3g_v - 3 + n_v \\ &= 3g_v -3 + \ab|L_v| + \ab|L_v'| + \ab|E_v|, \end{align*} the contribution becomes \begin{align*} & (tL)^{N\ab(g-1 + \frac{\ab|E_v|}{2})} L_{\alpha}^{3g-3+n_v + \sum_{\ell} (a_{\ell}-1) + \sum_{\ell'} (a'_{\ell'}-1) + \sum_f s_f - \sum_{\ell} c_{\ell} -\sum_f c_f - \sum_{\ell''} c_{\ell''}} \\ ={}& (tL)^{N\ab(g-1+\frac{\ab|E_v|}{2})} L_{\alpha}^{\sum_{\ell} (a_{\ell}+b_{\ell}-1) + \sum_{\ell'} (a'_{\ell'} + b'_{\ell'}-1) + \sum_f s_f}. \end{align*} The remaining tail, edge, and leg contributions contribute a degree in $X$ of at most \begin{align*} & \sum_{\ell \in L_v} \ab(c_{\ell} + \floor*{\frac{a_{\ell}}{N}}) + \sum_{\substack{f = (e,v) \\ e \in E_v}} \ab(c_f + \frac{1}{2}) + \sum_{\ell'' \in \mr{tails}} c_{\ell''} \\ ={}& 3g_v - 3 + n_v + \frac{\ab|E_v|}{2} + \sum_{\ell \in L_v} \floor*{\frac{a_{\ell}}{N}} - \sum_{\ell \in L_v} b_{\ell} - \sum_{\ell' \in L_v'} b'_{\ell'} \end{align*} Note that the end result must be independent of the hours $\alpha$, so we can sum over all $\alpha$ and obtain a multiplicative factor \[ L_{\alpha}^{\frac{1}{N}\ab(\sum_{\ell \in L_v}(a_{\ell}+b_{\ell}-1) + \sum_{\ell' \in L_v'} (a'_{\ell'} + b'_{\ell'}-1) + \sum_{e \in E_v} s_{(e,v)})}. \] Denote the exponent by $c_v$. Clearly if this is not an integer, the contribution vanishes after summing over all $\alpha$. By our choice of indexing of the edge contributions, we see that $s_{(e,v_1)} + s_{(e,v_2)} = 0$ whenever $e = (v_1, v_2)$. In particular, taking the product over all vertices gives an exponent of \[ \sum_v c_v = \frac{1}{N} (\ab|\ba| + \ab|\ba'| + \ab|\bb| + \ab|\bb'| - n-m) = c. \] If this is not an integer, the total contribution vanishes. Multiplying the prefactors over all vertices, we obtain a total prefactor \[ L_{\alpha}^{N \sum_v r_v} (tL)^{\sum_v N \ab(g_v-1+\frac{\ab|E_v|}{2})} = (tL)^{Nc + N(g-1)}. \] Using the fact that $Y = L^{-N}$, the prefactor becomes \[ \ab(\frac{Y}{t^N})^{-(g-1+c)}. \] Multiplying the remaining contributions from the tails, edges, and legs, the total degree in $X$ is at most \begin{align*} & \sum_v 3g_v - 3 + n_v + \frac{\ab|E_v|}{2} + \sum_{\ell \in L_v} \floor*{\frac{a_{\ell}}{N}} - \sum_{\ell \in L_v} b_{\ell} - \sum_{\ell' \in L_v'} b'_{\ell'} \\ ={}& 3g-3+n+m + \floor*{\frac{\ba}{N}} - \ab|\bb| - \ab|\bb'|. \end{align*} The desired result follows after applying the normalization factor $\ab(\frac{Y}{t^N})^{g-1+c}$. \end{proof} \subsection{Vanishing of the $[0]$ theory}\label{sub:Vanishing of the 0 theory} We will now study the MSP $[0]$ theory. Unfortunately, we need to factorize the MSP $[0]$ theory to prove the desired polynomiality result, but we will prove a vanishing result similar to the first part of~\Cref{lem:polynomiality1theory}. \begin{defn} Define the \textit{mod-$N$ degrees} by $\deg \varphi_j = \deg \phi_j = j \mod{N}$ and $\deg \psi = 1$. \end{defn} \begin{lem}\label{lem:RpreservesmodNdegree} $R^{[0]}(z)$ preserves the mod-$N$ degree. If we define $\bar{j} = j\mod{N}$, then \[ R^{[0]}(z)^* \phi_j = c'_{j,k} q^{\floor*{\frac{j}{N}}} \varphi_{\bar{j}} + O(z^{\bar{j}-3}) \] for $j = 0,\ldots,N+3$, where we define\footnote{These are in fact the $q$ coefficients of the quantities $I_0$, $I_0 I_{11}$, $I_0 I_{11} I_{22}$, and $I_0^2 I_{11} I_{22}$.} \begin{align*} c'_{j,6} &= (1,\ldots,1,-360,-3132,-8532,-11304) \\ c'_{j,8} &= (1,\ldots,1,-1680,-17488,-48048,-63856) \\ c'_{j,10} &= (1,\ldots,1,-15120,-194640,-605360,-784880). \end{align*} \end{lem} \begin{proof} Recall that \[ zD R^{[0]}(z)^* = R^{[0]}(z)^* A^M - A^Z R^{[0]}(z)^*, \] where \[ A^Z = \begin{pmatrix} 0 \\ I_{11} & 0 \\ & I_{22} & 0 \\ & & I_{11} & 0 \end{pmatrix} \] and $A^M$ is given by the formulae \[ A^M_{j+1,j} = 1, \qquad A^M_{j+N-1,j} = c_{j,k}q - \delta_{i,4}t^N \] and all other entries being zero, where \begin{align*} c'_{j,6} &= (360,2772,5400,2772,360) \\ c'_{j,8} &= (1680,15808,30560,15808,1680) \\ c'_{j,10} &= (15120,179520,410720,179520,15120). \end{align*} The expression for $R^{[0]}\phi_j$ follows directly. The fact that $R^{[0]}$ preserves the mod-$N$ degree follows from the fact that \[ R^{[0]}(z)^{-1}x = S^Z(q',z) (S^M(z)^{-1}x)|_Z \] for all $x \in \msc{H}_Z$ and the fact that both $S^Z$ and $S^M$ preserve the mod-$N$ degrees. Here, it is helpful to recall that \begin{align*} S^{Z}(z)^* =&{}\ I + \frac{1}{z} \begin{pmatrix} 0 \\ J_1' & 0 \\ & J_2' & 0 \\ & & J_1' & 0 \end{pmatrix} \\ &+ \frac{1}{z^2} \begin{pmatrix} 0 \\ & 0 \\ J_2 & & 0 \\ & \frac{J_2'}{J_1'}J_1 - J_2 & & 0 \end{pmatrix} + \frac{1}{z^3} \begin{pmatrix} 0 \\ & 0 \\ & & 0 \\ J_3 & & & 0 \end{pmatrix}, \end{align*} where we define $J_b = \frac{I_b}{I_0}$, $J_1' = I_{11}$, and $J_2' = J_1 + D J_2$. \end{proof} \begin{lem} Define $c \coloneqq \frac{1}{N}(\ab|\ba| + \ab|\bb| - n)$. If $c \notin \Z$, then $f_{g,(\ba,\bb)}^{[0]} = 0$. \end{lem} \begin{proof} Recall that $\Omega^{[0]} = R^{[0]}.\Omega^{Z,\tw}$ is defined by a graph sum formula, where the vertex, edge and leg contributions are given by the following: \begin{itemize} \item At each leg with insertion $\phi_a \psi^b$, we place $R^{[0]}(-\psi)^* \phi_a \psi^b$; \item At each edge, we place the bivector \[ \sum_{j=0}^{N+3} \frac{\phi_j \otimes \phi^j - R^{[0]}(-z)^* \phi_j \otimes R^{[0]}(-w)^* \phi^j}{z+w}; \] \item At each vertex, we place the linear map $I_0(q')^{-(2g-2+n)} \Omega_{g,n}^{Z,\tw,\tau_Z(q')}(-)$. \end{itemize} Because $\phi_j \otimes \phi^j$ has mod-$N$ degree $3$ and $R^{[0]}$ preserves the mod-$N$ degree, we see that the edge contributions have mod-$N$ degree $2$. Then note that for $\bullet$ being either ``$Z$'' or ``$Z,\tw$'' the integral \[ \int_{\ol{\msc{M}}_{g,m}} \Omega_{g,m}^{\bullet}\ab(\bigotimes_{j=1}^m \varphi_{a_j} \psi^{b_j}) \] vanishes unless $\sum_{j=1}^m (a_j+b_j) = m$. Summing over all vertices and edges in an arbitrary stable graph, we see that the contribution can only be nonzero if \[ \sum_{i=1}^n (\bar{a}_i + \bar{b}_i) = n, \] which is equivalent to $c \in \Z$. \end{proof} We may now assume that $c \coloneqq \frac{1}{N}(\ab|\ba| + \ab|\bb| - n)$ is an integer. \begin{lem}\label{lem:vanishing0nospecial} Define \[ \bar{c} \coloneqq \frac{\ab|\bar{\ba}| + \ab|\bb| - n}{N}. \] If $N \gg 3g-3+3n$, we always have $\bar{c} \geq 0$. In addition, if $\bar{c} \neq 0$, then $f_{g,(\ba,\bb)}^{[0]} = 0$. \end{lem} \begin{proof} Using the assumption that $N > 3g-3+3n$ and the fact that $\ol{\msc{M}}_{g,n}$ is a DM stack (hence $3g-3+n \geq 0$), we obtain $N > 2n$. Because all $a_i,b_i \geq 0$ and $\bar{c}$ is an integer, we must have \[ \bar{c} \geq \ceil*{\frac{-n}{N}} = 0. \] The vanishing result is another degree-counting argument. Assume that $\bar{c} > 0$. Recall $f_{g,(\ba,\bb)}^{[0]}$ is a sum of stable graph contributions, where we place \[ R^{[0]}(-\psi)^* \phi_{a_i} \psi^{b_i} \] at the $i$-th leg. In particular, the total degree of the ancestors is at least \begin{align*} \ab|\bar{\ba}| - 3n + \ab|\bb| &= N\bar{c} - (\ab|\bb| - n) - 3n + b \\ &\geq N - 2n. \end{align*} However, the contribution vanishes if \[ \ab|\bar{\ba}_v| - 3(n_v - \ab|E_v|) + \ab|\bb_v| > 3g_v - 3 + n_v \] and therefore if \begin{align*} \ab|\bar{\ba}| - 3n + \ab|\bb| &> \sum_v (3g_v - 3 + n_v) \\ &= 3g - 3 + n - \ab|E|. \end{align*} However, by assumption, we see that \begin{align*} \ab|\bar{\ba}| - 3n + \ab|\bb| &\geq N - 2n \\ &> 3g-3+n \\ &\geq 3g-3+n-\ab|E|, \end{align*} so all graph contributions must vanish. \end{proof} \begin{cor}\label{cor:criterionforc} If $f_{g,(\ba,\bb)}^{[0]}$ is nonzero, then $c = \floor*{\frac{\ba}{N}}$ and \[ g-1+c \leq 3g-3+c+n-\ab|\bb|. \] \end{cor} \begin{defn}\label{defn:0potspecial} Define the MSP $[0]$ potential with special insertions by \[ f_{g,(\ba,\bb),(\ba',\bb')}^{[0]} \coloneqq \int_{\ol{\msc{M}}_{g,n}} \Omega_{g,n+m}^{[0]}\ab(\bigotimes_{i=1}^n \phi_{a_i} \psi^{b_i}, \bigotimes_{j=1}^m E_{a'_j, b'_j}(\psi_{n+j})), \] where we define \[ E_{a',b'}(\psi) = L^{-a'} \cdot [w^{b'}] \frac{( R(\psi) - R(-w) )\bar{\phi}^{a'}}{\psi+w}. \] Here, $[w^{b'}]$ means that we take the coefficient of $w^{b'}$ in the expression. \end{defn} By construction, we have \[ V^{01}(z,w) = \sum_{a=1}^N E_{ab}(z) w^b \otimes L^a R(w) \bar{\phi}_a. \] \begin{lem} If $a-m \not\equiv b \pmod{N}$, then $(\phi_a, R_m \bar{\phi}^b) = 0$. In the case when $a-m \equiv b \pmod{N}$, then \[ L^{-a+m}(\phi_a, R_m \bar{\phi}^b) \in \Q(t^N)[X]_{m+\floor*{\frac{a}{N}}}. \] \end{lem} \begin{proof} Note that \begin{align*} (\phi_a, R_m \bar{\phi}^b) &= \sum_{\alpha=1}^N L_{\alpha}^{-b} (\phi_a, R_m \bar{\1}^{\alpha}) \\ &= \sum_{\alpha=1}^N L_{\alpha}^{-b} L^{a-m} (R_m)_a^{\alpha}. \end{align*} The result follows by using~\Cref{lem:rlevel1} and the the fact that the total power of the roots of unity is $a-m-b$. \end{proof} \begin{lem} The $[0]$ potential with special insertions $f_{g,(\ba,\bb),(\ba',\bb')}^{[0]}$ vanishes unless $c \coloneqq \frac{\ab|\ba| + \ab|\bb| + \ab|\ba'| + \ab|\bb'| -n+m}{N} \in \Z$. \end{lem} \begin{proof} Write \[ R_m \bar{\phi}^{b} = \sum_{s=1}^{N+3} (R_m \bar{\phi}^b, \phi_s) \phi^s. \] By the previous lemma, the only nonzero terms are the ones with $s \equiv m+b (\pmod N)$, so the mod-$N$ degree of $R_m\bar{\phi}^b$ is $3-(m+b)$. This implies that the mod-$N$ degree of $E_{ab}$ is $2-(a+b)$. Computing the total ancestor degree at all vertices as in the proof of~\Cref{lem:vanishing0nospecial}, we obtain the desired result. \end{proof} \section{The $A$-model Feynman rule}\label{sec:The A model Feynman rule} In order to extract information from the $[0]$ theory, we will extract a part of it which satisfies an $X$-polynomiality property. \subsection{Factorization of the $[0]$ theory}\label{sub:Factorization of the 0 theory} \begin{defn} Define the CohFT\footnote{Note that the unit of this CohFT is $\varphi_0$ and not $1$.} $\Omega^{\bA, \G}$ by the formula \[ \Omega^{\bA, \G} = R^{\bA,\G}.\Omega^{Z,\tw}. \] Also, define the generating function \[ f_{g,(\ba,\bb)}^{\bA,\G} \coloneqq \ab( \frac{-p_kY}{t^N} )^{g-1} \int_{\ol{\msc{M}}_{g,n}} \Omega_{g,n}^{\bA,\G}(\varphi_{a_1} \psi_1^{b_1}, \ldots, \varphi_{a_n} \psi_n^{b_n}). \] \end{defn} \begin{rmk} Because the CohFT $\Omega^Z$ and $\Omega^{\bA}$ have different units, the graph sum formula for the $R^{\bA}$ action will have $T = z(1-I_0)$, which by the dilaton equation will imply that we place the linear map \[ \frac{1}{I_0^{2g-2+n}} \Omega^Z_{g,n} \] at each vertex. \end{rmk} \begin{rmk} Because the basis $\varphi_0, \ldots, \varphi_3$ is not flat, we will need to consider the transformation \[ \Psi \coloneqq \pdiagmat[empty={}]{I_0,I_0 I_{11},I_0 I_{11} I_{22},I_0I_{11^2}I_{22}} \] from this basis to the flat basis $1, H, H^2, H^3$. In particular, if we want to compute derivatives of $R^{\bA,\G}$, then we will need the input basis to be the flat basis, and in this basis $R^{\bA, \G}(z)^{-1}$ takes the form \[ \Psi \ab(I - \begin{pmatrix} 0 & z E_{\psi}^{\G} & z^2 E_{\varphi\psi}^{\G} & z^3 E_{1\psi^2}^{\G} \\ & 0 & z E_{\varphi\varphi}^{\G} & z^2 E_{1\varphi\psi}^{\G} \\ & & 0 & z E_{\psi}^{\G} \\ & & & 0 \end{pmatrix}). \] \end{rmk} \begin{defn} Define the matrix \[ R^{X}(z) \coloneqq R^{\bA}(z)^{-1} \cdot R^{[0]}(z). \] We will consider the input basis of $R^X$ to be $\varphi_0, \ldots, \varphi_3$ and the output basis to be $\phi_0, \ldots, \phi_{N+3}$. \end{defn} \begin{lem}\label{lem:matrixelementsRX} The matrix elements $(R^X_m)_j^a \coloneqq (\phi_j, R_m^X \varphi^a)$ of $R^{X}$ satisfy the following whenever $m < N-3$: \begin{enumerate} \item If $j \not\equiv m+a \pmod{N}$, then $(R^X_m)_j^a = 0$; \item If $j < N$, then $(R^X_m)_j^a \in X \Q[X]_{m-1}$; \item If $j \geq N$, then $(R^X_m)_j^a \in q \Q[X]_m$; \item We have $R^X(-z)^* C^X(z) = \mr{Id}_{\msc{H}_Z}$ for some $C^X(z)$ of the form $\begin{psmallmatrix} C(z) \\ 0 \end{psmallmatrix}$, where $C(z)$ has the form \[ C(z) = \begin{pmatrix} 1 & z \cdot C_{1} & z^2 \cdot C_{2} & z^3 \cdot C_{3}\\ 0 & 1 & z \cdot C_{4} & z^2 \cdot C_{5} \\ 0 & 0 & 1 & z \cdot C_6 \\ 0 & 0 & 0 & 1 \end{pmatrix} \] for some $C_1, C_2, C_4, C_6 \in \Q[X]_1$ and $C_3, C_5 \in \Q[X]_2$. \end{enumerate} \end{lem} \begin{proof} Using the equation \begin{equation}\label{eqn:mspqde} zD R^{[0]}(z)^* = R^{[0]}(z)^* A^M - A^Z R^{[0]}(z)^* . \end{equation} Using the definition $R^{[0]}(z)^* = R^{\bA}(z)^* R^X(z)^*$, we obtain \[ zD(R^{\bA}(z)^*R^X(z)^*) = R^{\bA}(z)^* R^X(z)^* A^M - A^Z R^{\bA}(z)^* R^X(z)^*, \] and multiplying by $R^{\bA}(-z)$, we obtain \begin{equation}\label{eqn:Xqde} R^X(z)^* A^M = zD(R^X(z)^*) + R^{\bA}(-z) [(zD+A^Z) R^{\bA}(z)^*] R^X(z)^*. \end{equation} Direct computation (here, note that the basis $\varphi_0, \ldots, \varphi_3$ is not flat, so we need the version of $R^{\bA}$ with the $\Psi$) yields \[ R^{\bA}(-z) [(zD+A^Z)R^{\bA}(z)^*] = \begin{pmatrix} 0 & 0 & 0 & -r_1 X z^4 \\ 1 & 0 & -r_0 X z^2 & 0 \\ 0 & 1 & -Xz & 0 \\ 0 & 0 & 1 & -Xz \end{pmatrix}. \] Therefore, we can compute $R^X(z)^* \phi_i$ from $R^X(z)^* \phi_0$. Because $R^{[0]}(z)^* \phi_0 = \varphi_0 + O(z^{N-3})$ and $R^{\bA}(z)^* \varphi_0 = \varphi_0$, the first entry of $R^X$ is $1 + O(z^{N-3})$. We then note that the matrices $A^M$ and $R^{\bA}(-z)[(zD+A^Z)R^{\bA}(z)^*]$ both increase the mod-$N$ degree by $1$. Therefore, we see that $R^{X}$ preserves the mod-$N$ degree, so the vanishing property holds. The degree estimates follow from the explicit formulae for $A^M$ and $R^{\bA}(-z) [(zD+A^Z)R^{\bA}(z)^*]$. The final statement is obtained by direct computation. \end{proof} \begin{cor}\label{cor:edgeX} The edge contribution \[ V_X \coloneqq \frac{\sum_{i=0}^3 \varphi_i \otimes \varphi^i - \sum_{j=0}^{N+3} R^X(-z)^* \phi_j \otimes R^X(-w)^* \phi^j}{z+w} \] satisfies the degree bound \[ \frac{Y}{t^N} [z^{m_1} w^{m_2}] V_X \in \msc{H}_Z^{\otimes 2}[X]_{m_1+m_2+1}. \] \end{cor} \begin{proof} This follows from the lemma and the fact that $\varphi_j = \frac{t^N}{p_k Y} \varphi_{3-j}$. \end{proof} \subsection{Polynomiality of the $[0]$ theory and the $\bA$ theory}\label{sub:Polynomiality of the 0 theory} We now prove the polynomiality of the $[0]$-theory. First, we will prove some lemmas about $E_{a',b'}(z)$, which was introduced in~\Cref{defn:0potspecial}. \begin{lem}\label{lem:vanishingcoeffedge} Recall the definition of $E_{a',b'}(z)$ from~\Cref{defn:0potspecial}. Then \[ (\phi^a, [z^b] E_{a',b'}(z)) = 0 \] unless $a+a'+b+b' \equiv 2 \pmod{N}$. \end{lem} \begin{proof} By definition, we have \begin{align*} (\phi_a, [z^b] E_{a',b'}(z)) &= (-1)^{b'}(\phi_a, R_{b+b'+1} \bar{\phi}^{a'}) \\ &= (-1)^{b'} \sum_{\alpha} L_{\alpha}^{-a'} (\phi_a, R_{b+b'+1} \bar{\1}^{\alpha}), \end{align*} which vanishes unless $a-a'-(b+b'+1) \equiv 0 \pmod{N}$. The result follows from the fact that $\phi^a$ has mod-$N$ degree $3-a$. \end{proof} \begin{lem}\label{lem:degreecoeffedge} Whenever $N \gg b''$, then \[ \ab(\frac{Y}{t^N})^{-c_E} \cdot [z^{b''}] (\varphi^{a''}, R^X(-z)^* E_{a',b'}(z)) \in \Q[X]_{b'+b''+1}, \] where we define $c_E \coloneqq \frac{a'+b'+a''+b''-N-2}{N}$. If $c_E \notin \Z$, then the above quantity vanishes. \end{lem} \begin{proof} The vanishing is a corollary of~\Cref{lem:matrixelementsRX} and~\Cref{lem:vanishingcoeffedge}. The degree estimate comes from the following considerations. Whenever $a = 4, \ldots, N-1$ and $m' < N-3$, then $(\varphi^{a''}, [z^{m'}] R^X(-z)^* \phi_a)$ is nonzero only if $a = a''+m'$. If this is satisfied, then it has degree $m'$ in $X$, and therefore \[ \ab(\frac{Y}{t^N})^{-c_E} \cdot [z^{b''}] (\varphi^{a''}, R^X(-z)^* \phi_a)(\phi^a, E_{a',b'}(z)) \in \Q[X]_{b'+b''+1}. \] Here, we use the fact that \[ [z^b](\phi^a, E_{a',b'}(z)) = \frac{N}{p_k} \ab(\frac{Y}{t^N})^{c_E} (-1)^{b'} (R_{b+b'+1})_{N+3-a}^{\alpha} \in \ab(\frac{Y}{t^N})^{c_E} \Q[X]_{b+b'+1} \] by~\Cref{lem:rlevel1} because $\floor*{\frac{a}{N}} = 0$. In the case where $a = 0,\ldots,3$, then for any $m' < N-3$, the vanishing still holds, so we use $\phi^a = \frac{1}{5}(\phi_{N+3-a} - t^N \phi_{3-a})$ to compute. The $\phi_{N+3-a}$ term contributes an element of $\Q[X]_{b'+b''+2}$, whereas the $t^N \phi_{3-a}$ term contributes \[ [z^b](t^N \phi_{3-a}, E_{a',b'}(z)) = N (-1)^{b'} \ab(\frac{Y}{t^N})^{c_E} (R_{b+b'+1})_{3-a}^{\alpha} Y. \] Therefore, we have \[ \ab(\frac{Y}{t^N})^{-c_E} \cdot [z^{b''}] (\varphi^{a''}, R^X(-z)^* \phi_a)(\phi^a, E_{a',b'}(z)) \in \Q[X]_{b'+b''+2}. \] In the case when $a \geq N$, the nonvanishing condition becomes $a-N = a''+m'$. Therefore,~\Cref{lem:matrixelementsRX} implies that \[ \ab(\frac{Y}{t^N})^{-c_E} \cdot [z^{b''}] (\varphi^{a''}, R^X(-z)^* \phi_a)(\phi^a, E_{a',b'}(z)) \in q\Q[X]_{b'+b''+1}. \] If we sum the contributions from the above procedure, the degree count is too high by $1$. The $X^{b'+b''+2}$-coefficient is given by (up to a constant) \begin{align*} [X^{b'+b''+2}] \sum_{\substack{i+j=b'+b''+1 \\ j \leq b'' \\ a \leq 3}} (R^X_j)_i^a (R_i)_{N+3-a} - Y (R_j^X)_i^a(R_i)_{3-a} + \frac{Y}{t^N} (R_j^X)_{i}^{N+a} (R_i)_{3-a}. \end{align*} Note that the MSP quantum connection~\Cref{eqn:mspqde} (in particular the value of $A^M$) implies that $[X^{m+1}] (R_m)_j = \frac{c'_{j,k}}{r} \cdot [X^m] (R_m)_{j-N}$ and the equation~\Cref{eqn:Xqde} for $R^X$ implies that $[X^m] (q^{-1} (R^X_m)_j^a) = c'_{j,k} \cdot [X^m] (R_m)_{j-N}^a$, where $c'_{j,k}$ was defined in~\Cref{lem:RpreservesmodNdegree}. This implies that \begin{align*} & [X^{i+j+1}] ( (R_j^X)_i^a (R_i)_{N+3-a} - Y(R_j^X)_i^a (R_i)_{3-a} + \frac{Y}{t^N} (R_j^X)_i^{N+a}(R_i)_{3-a}) \\ ={}& \ab(1+\frac{c'_{N+a}}{r} + \frac{c'_{N+3-a}}{r}) \\ ={}& 0 \end{align*} for all $i+j = b'+b''+1$ and $a = 0,1,2,3$, where we have used the fact that $c'_{N+a} + c'_{N+3-a} = -r$, which is implied by the fact that $I_0^2 I_{11}^2 I_{22} = Y$. \end{proof} We will now put a partial ordering on the set of pairs $(g,n)$ such that $(h,m) \prec (g,n)$ if $(h,m) < (g,n)$ in the lexicographic order and $3h+m \leq 3g+n$. We will use induction on $(h,m)$ under this ordering and a bootstrapping argument to prove both the polynomiality of the $[0]$ theory and of the $\bA$ theory. We introduce the following statements: \begin{enumerate} \item Denote by $\mf{P}_{g,n}$ the statement ``for all $\ba \in \{0,1,2,3\}^n$ and $\bb \in \Z_{\geq 0}^n$, we have \[ \ab(\frac{Y}{t^N})^{g-1} f_{g,(\ba,\bb)}^{[0]} \in \Q[X]_{3g-3+n-\ab|\bb|}.\text{''} \] \item Denote by $\mf{Q}_{g,s}$ the statement ``for all $m+n \leq s$, $\ba \in \{0,\ldots,N+3\}^m$, $(\bb,\bb') \in \Z_{\geq 0}^{m+n}$, and $\ba' \in \{1,\ldots,N\}^n$, \[\ab(\frac{Y}{t^N})^{g-1} f_{g,(\ba,\bb),(\ba,\bb')}^{[0]} \in \Q[X]_{3g-3+m+2n+\floor*{\frac{\ba}{N}}-\ab|\bb|+\ab|\bb'|}.\text{''} \] \end{enumerate} \begin{lem}\label{lem:Apolyassuming0poly} Suppose that $\mf{P}_{h,m}$ holds for all $(h,m) \prec (g,n)$. Then for all $(h,m) \prec (g,n)$, we have \[ f_{h,(\ba,\bb)}^{\bA} \in \Q[X]_{3h-3+m-\ab|\bb|} \] for all $\ba \in \{0,1,2,3\}^m$. \end{lem} \begin{proof} First, note that $R^{\A}$ preserves degrees, so we must have $\sum_i (a_i + b_i) = n$. Now define \[ \tilde{f}_{h,(\ba,\bb)}^{[0]} \coloneqq \int_{\ol{\msc{M}}_{h,m}} (R^X.\Omega^{\bA})_{h,m}(C^X(\psi_1)\varphi_{a_1}\psi^{b_1}, \ldots, C^X(\psi_m)\varphi_{a_m} \psi^{b_m}). \] By the assumption $\mf{P}_{h,m}$ and the fact that $C^X$ has nonzero entries only in the top four rows, preserves the mod-$N$ degree, and the fact that $(\phi^j, C_{\ell}^X \varphi_a) \in \Q[X]_{\ell}$, we see that \[ \ab(\frac{Y}{t^N})^{h-1}\tilde{f}_{h,(\ba,\bb)}^{[0]} \in \Q[X]_{3h-3+m-\ab|\bb|}. \] In the graph sum formula for the action of $R^X$ and $\Omega^{\bA}$, we see there is a graph with a single genus $h$ vertex with $m$ insertions (corresponding to the largest stratum of $\ol{\msc{M}}_{h,m}$). The contribution of this graph is $f_{h,(\ba,\bb)}^{\bA}$. The contribution of any other graph will have the form \[ \bigotimes_{v} f^{\bA}_{g_v, n_v} \ab(\bigotimes_{i=1}^m \varphi_{a_i} \psi^{b_i} \otimes \bigotimes_{e} V_X(e)), \] where $V_X(e)$ was defined in~\Cref{cor:edgeX}. We will now induct on $(h,m)$. In the base case $(h,m) = (0,3)$, the leading graph is the only graph, so the result follows directly (note here the different normalization conventions for $f^{\bA}$ and $f^{[0]}$). Now we assume the result for all $(h',m') \prec (h,m)$. Using the graph sum, we now count the degrees of all of the contributions. \begin{itemize} \item The total exponent of $\frac{Y}{t^N}$ is $\sum_{v} (g_v-1) + \ab|E| = h-1$ using~\Cref{cor:edgeX} and distributing the factors of $Y$ as in the proof of~\cite[Theorem 6.1]{polynomiality}; \item The total degree in $X$ is at most \begin{align*} &\sum_v \ab(3g_v-3+n_v-\sum_{e\in E_v} b_{(e,v)} - \sum_{i \in L_v} b_i) + \sum_e (b_{ (e,v_1) }+b_{(e,v_2)}+1) \\ ={}& 3 \ab(\sum_v g_v - \ab|V| + 3\ab|E|) + m - \sum_i b_i \\ ={}& 3h-3+m - \ab|\bb|. \qedhere \end{align*} \end{itemize} \end{proof} \begin{lem}\label{lem:pimpliesq} If $\mf{P}_{h,m}$ holds for all $(h,m) \prec (g,n)$, then $\mf{Q}_{h,m}$ also holds for all $(h,s) \prec (g,n)$. \end{lem} \begin{proof} Recall that $\Omega^{[0]} = R^X.\Omega^{\bA}$. This implies that $f^{[0]}_{h,(\ba,\bb),(\ba',\bb')}$ can be written as a graph sum, where the contribution of a stable graph $\Gamma$ is given by the following: \begin{itemize} \item For each ordinary leg $\ell$, we insert $R^X(-\psi)^* \phi_a \psi^b$. By~\Cref{lem:matrixelementsRX}, this in fact becomes \[ (\varphi^{\ol{a} - m_{\ell}}, (-1)^{m_{\ell}}\psi^{b+m_{\ell}}(R^X_{m_{\ell}})^* \phi_a) \in \psi^{b+m_{\ell}} \ab(\frac{Y}{t^N})^{-\floor*{\frac{a}{N}}} \Q[X]_{m_{\ell}+\floor*{\frac{a}{N}}}. \] for a unique $m_{\ell}$ (here, note that the ancestor degree must be at most $3g_v-3+n_v < N$). \item For each special leg $\ell'$, we insert $R^X(-\psi)^* E_{a',b'}(\psi)$. Using~\Cref{lem:vanishingcoeffedge} and~\Cref{lem:degreecoeffedge}, this becomes \[ \psi^{b''} [z^{b''}] (\varphi^{a''}, R^X(-z)^* E_{a',b'}(z)) \in \psi^{b''} \ab(\frac{Y}{t^N})^{c_{E_{\ell'}}} \Q[X]_{b''+b'+1} \] for a unique $a'', b''$. \item At every edge, we insert the bivector $V_X$. \end{itemize} We now consider the total degree of the contributions from a graph $\Gamma$. \begin{itemize} \item The total exponent of $\frac{Y}{t^N}$ is given by \[ \sum_{\ell} -\floor*{\frac{a_{\ell}}{N}} + \sum_{\ell'} c_{E_{\ell'}} + \sum_v (1-g_v) +\sum_e (-1). \] Using the fact that \[ \sum_{\ell} (\bar{a}_{\ell} + b_{\ell}) + \sum_{\ell'} (a_{\ell'}'' + b_{\ell'}'') = m+n \] by~\Cref{cor:edgeX}, we obtain \begin{align*} \sum_{\ell'} c_{E_{\ell'}} + c + n - \floor*{\frac{\ba}{N}} ={}& \sum_{\ell'} \frac{a'_{\ell'} + b'_{\ell'} + a''_{\ell'} + b''_{\ell'}-N-2}{N} \\ &+ \frac{\ab|\bar{\ba}| + \ab|\bb| - \ab|\ba'| - \ab|\bb'| -m+n}{N} + n \\ ={}& \frac{\sum_{\ell'}(a''_{\ell'}+b''_{\ell'})+\ab|\bar{\ba}| + \bar|\bb| -nN - 2n -m+n }{N} \\ ={}& 0, \end{align*} which implies that the total exponent is $-(c+n+h-1)$. \item The total degree in $X$ is at most \begin{align*} &\sum_v \bigg(3g_v-3+n_v - \sum_{\ell}(b_{\ell}+m_{\ell})-\sum_{\ell'} b''_{\ell'} - \sum_e m_{(e,v)} \\ &+ \sum_{\ell} \ab(m_{\ell}+\floor*{\frac{a_{\ell}}{N}}) + \sum_{\ell'} (b''_{\ell'}+b'_{\ell'}+1) \bigg) + \sum_e (m_{(e,v_1)}+ m_{(e,v_2)}+1) \\ ={}& \ab(\sum_v 3g_v-3+n_v) - \ab|\bb| + \ab|\bb'| + \ab|E| + n + \floor*{\frac{\ba}{N}} \\ ={}& 3h-3+m+2n+\floor*{\frac{\ba}{N}} - \ab|\bb| + \ab|\bb'|. \qedhere \end{align*} \end{itemize} \end{proof} \begin{thm} For all $\ba, \bb \in \{0,\ldots,N+3\}^n$, we have \[ \ab(\frac{Y}{t^N})^{g-1+c} f_{g,(\ba,\bb)}^{[0]} \in \Q[X]_{3g-3+n+\floor*{\frac{\ba}{N}} - \ab|\bb|}. \] \end{thm} \begin{proof} We will induct on $(g,n)$ under the ordering $\prec$. The base case is $(g,n) = (0,3)$. In this case, there is only one graph with a single vertex. Because $\dim \ol{\msc{M}}_{g,n} = 0$, no ancestor insertions are allowed, and so we calculate \begin{align*} \ab(\frac{Y}{t^N})^{0-1+c} f_{0,(\ba,\mbf{0})}^{[0]} &= \ab(\frac{Y}{t^N})^c I_0^2 I_{11}^2 I_{22} \frac{t^N}{Y} \cdot \mr{const} \cdot q^c \\ &= \mr{const} \cdot X^c. \end{align*} Here, we use the fact that $c = \floor*{\frac{\ba}{N}}$ and the computation of genus-zero three-point functions in~\cite[\S 2.4]{polynomiality}. We now assume the desired polynomiality result for $(h,m) \prec (g,n)$. This implies $\mf{P}_{h,m}$ and thus $\mf{Q}_{h,m}$ for all $(h,m) \prec (g,n)$ by~\Cref{lem:pimpliesq}. We may also assume that $c = \floor*{\frac{\ba}{N}}$ by~\Cref{cor:criterionforc}. We will now consider the $[0,1]$ theory. By~\cite[Theorem 4.1]{polynomiality}, $f_{g,(\ba,\bb)}^{[0,1]}$ is a polynomial in $q$ of degree at most $g-1+\frac{3g-3 + \ab|\ba|}{N}$. By~\Cref{lem:vanishing0nospecial}, this becomes \begin{align*} g-1+\frac{3g-3+\ab|\ba|+n-\ab|\bar{\ba}|-\ab|\bb|}{N} &= g-1+\frac{3g-3+\ab|\ba|+n-\ab|\ba|-\ab|\bb|+N\floor*{\frac{\ba}{N}}}{N} \\ &= g-1+c + \frac{3g-3+n-\ab|\bb|}{N}. \end{align*} Because $0\leq \ab|\bb| \leq 3g-3+n$ and $N \gg 3g-3+n$, we see that the degree is in fact at most $g-1+c$. Multiplying by $\ab(\frac{Y}{t^N})^{g-1+c}$, we see that \[ \ab(\frac{Y}{t^N})^{g-1+c} f_{g,(\ba,\bb)}^{[0,1]} \in \Q[X]_{g-1+c}. \] By~\Cref{cor:criterionforc}, this satisfies the desired degree bound. We will now apply the bipartite graph decomposition from~\Cref{thm:01bipartite}. There is a leading bipartite graph with only a single level $0$ vertex. We need to prove the degree estimate for the non-leading graphs. Applying $\mf{Q}_{h,m}$ at level $0$ and~\Cref{lem:polynomiality1theory} at level $1$, we now count the total degree contribution of a bipartite graph $\Lambda$. \begin{itemize} \item The total exponent of $\frac{Y}{t^N}$ is \begin{align*} & g-1+c - \sum_{v \in V_0} (g_v-1+c_v+\ab|L'_v|) - \sum_{v \in V_1} (g_v-1+c_v) \\ ={}& g-1 + \frac{\ab|\ba|+\ab|\bb|-n}{N}\\ &- \sum_{v \in V_0} \ab(g_v-1+\frac{\ab|\ba_v| + \ab|\bb|_v -\ab|\ba'_v| - \ab|\bb'_v| - \ab|L_v| + \ab|E_v|}{N} + \ab|E_v|) \\ &- \sum_{v \in V_1} \ab(g_v-1+\frac{\ab|\ba_v| + \ab|\bb|_v -\ab|\ba'_v| - \ab|\bb'_v| - \ab|L_v| - \ab|E_v|}{N}) \\ ={}& g-1 + \ab|E| - \sum_{v \in V} (g_v-1) \\ ={}& 0. \end{align*} \item The total degree in $X$ is \begin{align*} \sum_{v \in V_0} \ab(3g_v-3+n_v + \ab|E_v|+\floor*{\frac{\ba_v}{N}} - \ab|\bb_v| + \ab|\bb'_v|) \\ + \sum_{v \in V_1} \ab(3g_v - 3 + n_v + \floor*{\frac{\ba_v}{N}} - \ab|\bb_v| - \ab|\bb'_v|) \end{align*} Because a stable graph describes a codimension $\ab|E|$ stratum in $\ol{\msc{M}}_{g,n}$, this is at most $3g-3+n+\floor*{\frac{\ba}{N}} - \ab|\bb|$. \qedhere \end{itemize} \end{proof} Applying~\Cref{lem:Apolyassuming0poly}, we obtain the following. \begin{cor}\label{cor:amodelfeynman} For all $(g,n)$ such that $2g-2+n > 0$, all $\ba \in \{0,1,2,3\}^n$, and all $\bb \in \Z_{\geq 0}^n$, we have \[ f_{g,(\ba,\bb)}^{\bA} \in \Q[X]_{3g-3+n-\ab|\bb|}. \] \end{cor} \subsection{Choice of gauge}\label{sub:Choice of gauge} Note that \[ R^{\bA,\G}(z)^{-1} = R^{\bA}(z)^{-1} \G(z)^{-1}, \] where we compute \[ \G(z)^{-1} = I - \begin{pmatrix} 0 & zc_{11} & z^2c_2 & -z^3 (c_{11} c_2 + c_3) \\ & 0 & z c_{12} & -z^2 (c_{11} c_{12} + c_2) \\ & & 0 & c_{11} \\ & & & 0 \end{pmatrix}. \] Note that this is a symplectic matrix, so we have an equality \[ \Omega^{\bA, \G} = \G.\Omega^{\bA} \] of CohFTs. \begin{thm} For all $(g,n)$ such that $2g-2+n > 0$, all $\ba \in \{0,1,2,3\}^n$, and all $\bb \in \Z_{\geq 0}^n$, we have \[ f_{g,(\ba,\bb)}^{\bA,\G} \in \Q[X]_{3g-3+n-\ab|\bb|}. \] \end{thm} \begin{proof} We will write the stable graph sum formula for $\Omega^{\bA, \G}$ as the $\G$-action on $\Omega^{\bA}$. The contribution of a stable graph $\Gamma$ is given by the following assignments: \begin{itemize} \item At each leg, we place $\G(-z)^* \varphi_a \psi^b = \sum_m \G_m^* (-\psi)^m \varphi_a \psi^b$; \item At each edge, we place \begin{align*} V^{\G} &\coloneqq \sum_{i=1}^3 \frac{\varphi_i \otimes \varphi^i - \G(-\psi)^* \varphi_i \otimes \G(-\psi')^* \varphi^i}{\psi+\psi'} \\ &= Y^{-1} \sum_{a,b} V_{ab}^{\G} \psi^a (\psi')^b. \end{align*} \end{itemize} Here, note that $\G_m^*$ has degree $m$ in $X$ and $V_{ab}$ has degree $a+b+a$ in $X$ by the assumption on the gauge in~\Cref{defn:propogators}. We now compute the total degree of the contribution. The total exponent of $\frac{Y}{t^N}$ in the contribution of $\Gamma$ to $\Omega_{g,n}^{\bA, \G}$ is \[ -\ab|E| - \sum_{v \in V} (g_v-1) = -(g-1). \] On the other hand, the total degree in $X$ is at most \begin{align*} &\sum_v \ab(3g_v-3+n_v - \sum_{\ell \in L_v} (m_{\ell} + b_{\ell}) - \sum_{e \in E_v} m_{(e,v)})\\ &+ \sum_e (m_{(e,v_1)} + m_{(e,v_2)}+1) + \sum_{\ell} m_{\ell} \\ ={}& 3g-3+n - \ab|\bb|. \qedhere \end{align*} \end{proof} \section{The B-model Feynman rule}\label{sec:The B-model Feynman rule} In this section, we will define the B-model Feynman rule and prove that it equals the A-model Feynman rule. From now on, we will make the specialization $t^N = -1$. This makes $q' = q$ and makes \[ f_{g,(\ba,\bb)}^{\bA,\G} = (p_kY)^{g-1} \int_{\ol{\msc{M}}_{g,n}} \Omega_{g,n}^{\bA,\G}(\varphi_{a_1} \psi_1^{b_1}, \ldots, \varphi_{a_n} \psi_n^{b_n}). \] \begin{conv} In this section, we will omit all superscripts of $\G$, so $E_{**}$ stands for $E_{**}^{\G}$, $R^{\bA}$ stands for $R^{\bA, \G}$, and so on. \end{conv} \subsection{B-model geometric quantization}\label{sub:B model geometric quantization} We will first express the physics Feynman rule using geometric quantization. Note that the Givental formalism is a type of geometric quantization, so we will be able to compare the A-model and B-model quantizations. Consider the vector space \[\msc{H}_S \coloneqq T^*(zH^0(Z) \oplus H^2(Z)) = \on{span}\{ -\varphi_2 z^{-1}, \varphi_3 z^{-2},\varphi_1, \varphi_0 z \} \] with the symplectic form \[ \frac{1}{p_k Y} \Res_{z=0} (f(-z),g(z)) = \begin{pmatrix} 0 & I \\ -I & 0 \end{pmatrix} . \] Then define \begin{align*} R^{\bB} &\coloneqq R^{\bA}|_{\msc{H}_S} \\ &= \begin{pmatrix} 1 & -E_{\psi} \\ 0 & 1 \\ -E_{\varphi\varphi} & -E_{\varphi\psi} & 1 \\ E_{1\varphi\psi} & E_{1\psi^2} & E_{\psi} & 1 \end{pmatrix} \\ &\eqqcolon \begin{pmatrix} A & B \\ C & D \end{pmatrix}. \end{align*} Because $\msc{H}_S$ is a symplectic vector space, we will write elements as \[ (\mbf{p}, \mbf{x}) \coloneqq - p_x \varphi_2 z^{-1} +p_y \varphi_3 z^{-2} + x \varphi_1 + y \varphi_0 z. \] \begin{defn} Following~\cite{geomquantgwtheory}, we define the geometric quantization $\wh{R}^{\bB}$ by the Gaussian integral \[ \wh{R}^{\bB}F(\hbar, \mbf{x}) \coloneqq \log \int_{\R^4} e^{\frac{1}{\hbar} ( \mbf{Q}(\bx', \bp') - \bx' \cdot \bp' ) + F(\hbar, \bx')} \d{\bx'} \d{\bp'}. \] Here, $\mbf{Q}(\bx', \bp')$ is defined by the formula \begin{align*} \mbf{Q}(\bx', \bp') &\coloneqq (D^{-1}\bx') \cdot \bp' - \frac{1}{2} (D^{-1}C \bp') \cdot \bp' \\ &= (\bp')^T \begin{pmatrix} 1 & \\ -E_{\psi} & 1 \end{pmatrix} \bx' + \frac{1}{2} (\bp')^T \begin{pmatrix} E_{\varphi\varphi} & E_{\varphi\psi} \\ E_{\varphi\psi} & E_{\psi\psi} \end{pmatrix}\bp' \end{align*} and $(\bp', \bx')$ are coordinates on $\R^4 = \R^2 \times \R^2$. \end{defn} Following~\cite[\S 3.4]{geomquantgwtheory}, a standard argument involving the Fourier transform gives us the operator form \[ \wh{R}^{\bB} F(\hbar, \bx) = \log \ab(e^{-\frac{\hbar}{2} \begin{psmallmatrix} \partial_x & \partial_y \end{psmallmatrix} DC^T \begin{psmallmatrix} \partial_x \\ \partial_y \end{psmallmatrix} } e^{F(\hbar, D^{-1}\bx)}). \] Now define $\tilde{E}_{\varphi\varphi}$, $\tilde{E}_{\varphi\psi}$, and $\tilde{E}_{\psi\psi}$ by \[ -DC^T \coloneqq \begin{pmatrix} \tilde{E}_{\varphi\varphi} & \tilde{E}_{\varphi\psi} \\ \tilde{E}_{\varphi\psi} & \tilde{E}_{\psi\psi} \end{pmatrix}. \] Then if we define \[ V^{\bB}(\partial_{\bx}, \partial_{\bx}) \coloneqq \frac{1}{2} \tilde{E}_{\varphi\varphi} \pdv[order=2]{}{x} + \tilde{E}_{\varphi\psi} \pdv{}{x,y} + \frac{1}{2} \tilde{E}_{\psi\psi} \pdv[order=2]{}{y}, \] the operator form of the quantization action becomes \begin{equation}\label{eqn:operatorbmodel} \wh{R}^{\bB} F(\hbar, \bx) = \log \ab(e^{\hbar V^{\bB}(\partial_{\bx}, \partial_{\bx})} e^{F(\hbar, D^{-1}\bx)}). \end{equation} \begin{defn} Define the (normalized) Gromov-Witten correlator of $Z$ by the formula \[ P_{g,m,n} \coloneqq \frac{(p_k Y)^{g-1}}{I_0^{2g-2+m+n}} \ab< \varphi_1^{\otimes m}, (\varphi_0 \psi)^{\otimes n}>_{g,m+n}^Z \] when $(g,m) \neq (1,0)$ and define \[ P_{1,0,n} \coloneqq (n-1)! \ab(\frac{\chi(Z)}{24} - 1). \] \end{defn} \begin{defn} Define the master B-model Gromov-Witten potential function by \[ P^{\bB}(\hbar, x, y) \coloneqq \sum_{g,m,n} \hbar^{g-1} \frac{x^m y^n}{m!n!} P_{g,m,n}. \] Then, define the master B-model potential function by \[ f^{\bB}(\hbar, x, y) \coloneqq \wh{R}^{\bB} P^{\bB}(\hbar, x, y) \eqqcolon \sum_{g,m,n} \hbar^{g-1} f_{g,m,n}^{\bB}. \] \end{defn} By~\cite[Theorem 10]{geomquantgwtheory}, we can also compute $f^{\bB}$ by the construction in~\Cref{defn:bmodelfeynman}. \subsection{Factorization of the quantization action}\label{sub:Factorization of the quantization action} We will factor the quantization action~\Cref{eqn:operatorbmodel} into the change of variables and the application of differential operators. Observe that $D^{-1}\bx = (x,y-E_{\psi} x)$. Then the transformation \[ F(\hbar, x,y) \mapsto F(\hbar, x, y-E_{\psi} x) \] is given by quantizing the matrix \[ \msc{E}^{\bB} \coloneqq \begin{pmatrix} 1 & -E_{\psi} \\ & 1 \\ & & 1 \\ & & E_{\psi} & 1 \end{pmatrix}. \] Then we compute \begin{align*} \tilde{P}^{\bB} &\coloneqq \wh{\msc{E}}^{\bB} P^{\bB}(\hbar, x, y) \\ &= P^{\bB}(\hbar, x, y-E_{\psi}x) \\ &= \sum_{g,m,n} \hbar^{g-1} \frac{x^m y^n}{m!n!} \tilde{P}_{g,m,n}, \end{align*} where \[ \tilde{P}_{g,m,n} \coloneqq \frac{(p_k Y)^{g-1}}{I_0^{2g-2+m+n}}\ab<(\varphi_1 - E_{\psi} \varphi_0\psi)^{\otimes m}, (\varphi_0 \psi)^{\otimes n}>_{g,m+n}^{Z}. \] We then compute \[ \tilde{R}^{\bB} \coloneqq R^{\bB} (\msc{E}^{\bB})^{-1} = \begin{pmatrix} 1 & 0 \\ 0 & 1 \\ -\tilde{E}_{\varphi\varphi} & -\tilde{E}_{\varphi\psi} & 1 \\ -\tilde{E}_{\varphi\psi} & -\tilde{E}_{\psi\psi} & 0 & 1 \end{pmatrix}, \] so we see that $f^{\bB}(\hbar, x, y) = \wh{\tilde{R}^{\bB}} \tilde{P}^{\bB}(\hbar, x, y)$. \subsection{Modification of the A-model quantization}\label{sub:Modification of the A-model quantization} Note that in the graph sum formula for $\Omega^{\bA}$, the edge contribution is given by \begin{align*} V_{\bA} ={}& V_{\bB} + E_{\psi} (\varphi_0 \otimes \varphi_2 + \varphi_2 \otimes \varphi_0) + E_{1\varphi\psi} (\varphi_0 \otimes \varphi_1 \psi' + \varphi_1 \psi \otimes \varphi_0) \\ &+ E_{1\psi^2} (\varphi_0 \otimes \varphi_0 (\psi')^2 + \varphi_0 \psi^2 \otimes \varphi_0). \end{align*} In order to prove that the A-model and B-model Feynman rules are equivalent, we need to analyze the contributions of the three extra terms. We will begin with the terms $E_{\psi} (\varphi_0 \otimes \varphi_2 + \varphi_2 \otimes \varphi_0)$ and study a parallel construction to the modified B-model quantization. The parallel construction in the A-model is to consider the matrix\footnote{This should also implicitly have a $\Psi^{-1}$ on the right, but we will omit it.} \[ \msc{E}^{\bA} \coloneqq I + z \begin{pmatrix} 0 & E_{\psi} \\ & 0 \\ & & 0 & E_{\psi} \\ & & & 0 \end{pmatrix} \] and the factorization \[ \tilde{R}^{\bA}(z) \coloneqq R^{\bA}(z) \msc{E}^{\bA}(z)^{-1}. \] Recall that matrices are written in the basis $\varphi_0, \varphi_1, \varphi_2, \varphi_3$. \begin{defn} Define the CohFT \[ \tilde{\Omega}^Z \coloneqq \msc{E}^{\bA} \Omega^{Z} \] and the potential \[ \tilde{P}^{\bA}(\hbar, \bt) \coloneqq \sum_{g,n} \frac{\hbar^{g-1}}{n!} (p_k Y)^{g-1} \int_{\ol{\msc{M}}_{g,n}} \tilde{\Omega}^Z_{g,n}(\bt^{\otimes n}) \] for the coordinate \[ \bt = x \varphi_1 + y \varphi_0 \psi + a \varphi_1 \psi + b \varphi_0 \psi^2 + c \varphi_0. \] Specializing to the case $\bt = (\bx, 0)$, we define \[ \tilde{P}^{\bA}(\hbar, x, y) \coloneqq \tilde{P}^{\bA}(\hbar, x\varphi_0 + y \varphi_0 \psi). \] \end{defn} \begin{lem} We have the identity \[ \tilde{P}^{\bA}(\hbar, x, y) = \tilde{P}^{\bB}(\hbar, x, y) - \log(1-y). \] \end{lem} \begin{proof} By~\cite[Theorem 5]{invtauteqns}, the $R$-matrix action preserves all tautological equations on the moduli spaces of curves, in particular the string and dilaton equations, so we can apply the dilaton equation to $\tilde{\Omega}^Z$. We also note that changing $\tilde{P}^{\bB}(\hbar, x, y)$ to $\tilde{P}^{\bB}(\hbar, x, y)$ is the same as replacing $P_{1,0,n}$ by the actual Gromov-Witten invariant \[ \frac{1}{I_0^n} \ab<(\varphi_0\psi)^{\otimes n}>_{1,n}^Z = (n-1)! \frac{\chi(Z)}{24}, \] so applying the dilaton equation to both sides (where $\tilde{P}$ denotes either $\tilde{P}^{\bA}$ or $\tilde{P}^{\bB}$) yields \begin{align*} \pdv{}{y} \tilde{P}(\hbar, x, y) &= \pdv{}{y}\ab( \sum_{(g,m,n) \neq (1,0,1)} \hbar^{g-1} \frac{x^m y^n}{m!n!} \tilde{P}_{g,m,n} + \frac{\chi(Z)}{24} y ) \\ &= \sum_{g,m,n} \hbar^{g-1} \frac{x^m y^{n-1}}{m!(n-1)!} \tilde{P}_{g,m,n} + \frac{\chi(Z)}{24} \\ &= \sum_{g,m,n} \hbar^{g-1} \frac{x^m y^{n-1}}{m!(n-1)!} (2g-2+m+n-1) \tilde{P}_{g,m,n-1} + \frac{\chi(Z)}{24} \\ &= \ab(2\hbar \pdv{}{\hbar} + x \pdv{}{x} + y \pdv{}{y}) \tilde{P} + \frac{\chi(Z)}{24}. \end{align*} Therefore, we only need to prove that $\tilde{P}^{\bA}(\hbar, x, 0) = \tilde{P}^{\bB}(\hbar, x, 0)$. We will now consider the graph sum formula for the definition of $\tilde{\Omega}^Z$. The edge contributions are given by $E_{\psi}(\varphi_0 \otimes \varphi_2 + \varphi_2 \otimes \varphi_0)$. All insertions at legs are $\msc{E}^{\bA}(-\psi)^* \varphi_1 = \varphi_1 - E_{\psi} \varphi_0 \psi$, and each vertex contributes a Gromov-Witten correlator of $Z$. Applying the string equation, dilaton equation, divisor equation, and virtual dimension constraints, if the stable graph $\Gamma$ has at least one edge and one vertex of $g > 0$, then its contribution vanishes. By a similar argument, any vertex with more than two edges has vanishing contribution. Therefore, we have a decomposition \[ \tilde{P}^{\bA}(\hbar, x, 0) = P^{\bA}(\hbar, x, -E_{\psi}x) + P_1^{\sloop}(x), \] where the first term comes from the leading graphs with a single genus $g$ vertex and the second comes from loops of genus zero vertices, which contribute to the genus $1$ potential. Every vertex must have at least one $\varphi_1$ insertion by dimension reasons, and the string and dilaton equations imply that there is exactly one $\varphi_1$ insertion. Therefore, we use the dilaton equation to remove the $-E_{\psi}\varphi_0 \psi$ insertions and compute \begin{align*} P_1^{\sloop}(x) &= \sum_{\substack{\Gamma \text{ loop} \\ \ab|V| = m \\ \ab|L| = m+n}} \frac{x^{m+n}}{m!n!}\frac{\mr{Cont}_{\Gamma}}{\ab|\Aut \Gamma|} \\ &= \sum_{m > 0} \frac{(m-1)!}{m!} (E_{\psi}x)^m \prod_{i=1}^m \sum_{n_i \geq 0} (-E_{\psi}x)^{n_i} \\ &= \sum_{m > 0} \frac{1}{m} \ab(\frac{E_{\psi}x}{1+E_{\psi}x})^m \\ &= -\log \ab(1- \frac{E_{\psi}x}{1+E_{\psi}x}) \\ &= \log (1+E_{\psi}x). \end{align*} Here, we have used the following combinatorial facts: \begin{itemize} \item There are $(m-1)!$ ways to arrange $m$ vertices in a loop; \item For any partition $n = n_1 + \cdots + n_m$ of the $\varphi_0 \psi$ insertions, the number of possible assignments is $\frac{n!}{n_1! \cdots n_m!}$; \item Applying the dilaton equation to any vertex with $n_i$ insertions of $\varphi_0 \psi$ and one insertion of $\varphi_1$ produces a factor of $n!$. \end{itemize} We conclude that \begin{align*} \tilde{P}^{\bA}(\hbar, x, 0) &= P^{\bA}(\hbar, x, -E_{\psi}x) + \log(1+E_{\psi}x) \\ &= P^{\bB}(\hbar, x, -E_{\psi}x) \\ &= \tilde{P}^{\bB}(\hbar, x, 0). \qedhere \end{align*} \end{proof} \subsection{Equality of A-model and B-model potentials}\label{sub:Equality of A-model and B-model potentials} A direct computation yields \[ \tilde{R}^{\bA}(z)^{-1} = I - \begin{pmatrix} 0 & 0 & z^2 \tilde{E}_{\varphi\psi} & z^3 \tilde{E}_{\psi\psi} \\ & 0 & z \tilde{E}_{\varphi\varphi} & z^2 \tilde{E}_{\varphi\psi} \\ & & 0 & 0 \\ & & & 0 \end{pmatrix}. \] By~\cite[Proposition 7.3]{virasorofanotoric}, we see that \[ e^{f^{\bA}(\hbar, x, y)} = e^{\hbar V^{\bA}(\partial_{\bt}, \partial_{\bt})} e^{\tilde{P}^{\bA}(\hbar, x, y)}, \] where \begin{align*} V^{\bA}(\partial_{\bt}, \partial_{\bt}) &\coloneqq V^{\bB}(\partial_{\bt}, \partial_{\bt}) - \tilde{E}_{\varphi\psi} \pdv{}{a,c} - \tilde{E}_{\psi\psi} \pdv{}{b,c} \\ &\eqqcolon V^{\bB}(\partial_{\bt}, \partial_{\bt}) + V^{\extra}(\partial_{\bt}, \partial_{\bt}). \end{align*} We will first prove some technical lemmas about the geometric quantization formalism. This is \begin{lem} We have the equality \[ e^{\tilde{P}^{\bA}(\hbar, x, y, a, b, c)} = e^{\frac{c}{1-y}\ab(a \pdv{}{x} + b \pdv{}{y})} e^{\tilde{P}^{\bA}(\hbar, x, y)}. \] \end{lem} \begin{proof} The operator form of the string equation\footnote{If we write $\varphi(z) = \sum_{i=0}^3 t_i^j \varphi_i z^j$, the operator form of the string equation (see~\cite{symplfrob}) is \[ \pdv{}{t_0^0} \msc{D} = \frac{1}{2} (\bt^0, \bt^0) + \sum_{j=0}^{\infty} \sum_{i=0}^3 t_i^{j+1} \pdv{}{t_i^j} \msc{D}. \] Because there are no $\varphi_2$ or $\varphi_3$ insertions, the unstable term disappears. We then set $t_1^0 = x$, $t_1^1 = a$, $t_0^0 = c$, $t_0^1 = y$, and $t_0^2 = b$. Here, $\msc{D} = e^{\tilde{P}^{\bA}}$.} is \[ \pdv{}{c} e^{\tilde{P}^{\bA}(\hbar, x, y, a, b, c)} = \ab(a \pdv{}{x} + b \pdv{}{y} + y\pdv{}{v})e^{\tilde{P}^{\bA}(\hbar, x, y, a, b, c)}. \] By virtual dimension reasons, we obtain the initial condition \[ \tilde{P}^{\bA}(\hbar, x, y, a, b, 0) = \tilde{P}^{\bA}(\hbar, x, y). \] The desired result follows from the computation \begin{align*} (1-y)\pdv{}{c}&\ab( e^{\frac{c}{1-y}\ab(a \pdv{}{x} + b \pdv{}{y})} e^{\tilde{P}^{\bA}(\hbar, x, y)} ) \\ &= (1-y)\pdv{}{c} \sum_{n=0}^{\infty} \frac{1}{n!} \ab(\frac{c}{1-y})^n \ab(a \pdv{}{x} + b \pdv{}{y})^n e^{\tilde{P}(\hbar, x, y)} \\ &= \sum_{n=0}^{\infty} \frac{1}{(n-1)!} \ab(\frac{c}{1-y})^{n-1} \ab(a \pdv{}{x} + b \pdv{}{y})^n e^{\tilde{P}^{\bA}(\hbar, x, y)} \\ &= \ab(a \pdv{}{x} + b\pdv{}{y}) \ab(e^{\frac{c}{1-y}\ab(a \pdv{}{x} + b \pdv{}{y})} e^{\tilde{P}^{\bA}(\hbar, x, y)}). \qedhere \end{align*} \end{proof} \begin{thm}\label{thm:allgenusmirror} We have the equality \[ f^{\bA}(\hbar, x, y) = f^{\bB}(\hbar, x, y) - \log(1-y). \] \end{thm} \begin{proof} Our goal is to compute the function \begin{align*} e^{f^{\bA}(\hbar, x, y)} ={}& e^{\hbar(V^{\bB}(\partial_{\bt}, \partial_{\bt}) + V^{\extra}(\partial_{\bt}, \partial_{\bt}))} e^{\tilde{P}^{\bA}(\hbar, x, y, a, b, c)}\Big|_{a,b,c=0} \\ ={}& e^{\hbar(V^{\bB}(\partial_{\bt}, \partial_{\bt})+V^{\extra}(\partial_{\bt}, \partial_{\bt}))} e^{\frac{c}{1-y}\ab(a\pdv{}{x}+b\pdv{}{y})}e^{\tilde{P}^{\bA}(\hbar, x, y)} \Big|_{a,b,c=0} \\ ={}& e^{\hbar(V^{\bB}(\partial_{\bt}, \partial_{\bt})+V^{\extra}(\partial_{\bt}, \partial_{\bt}))} e^{\frac{c}{1-y}\ab(a\pdv{}{x}+b\pdv{}{y})}\frac{e^{\tilde{P}^{\bB}(\hbar, x, y)}}{1-y} \Bigg|_{a,b,c=0}. \end{align*} We will first consider the contribution of $\frac{c}{1-y} \ab(a\pdv{}{x}+b\pdv{}{y})$ and $V^{\extra}(\partial_{\bt}, \partial_{\bt})$. For any function $\msc{D}(x,y)$, we compute \begin{align*} & e^{\hbar V^{\extra}(\partial_{\bt}, \partial_{\bt})} e^{\frac{c}{1-y}\ab(a\pdv{}{x}+b\pdv{}{y})} \msc{D}(x,y) \\ ={}& \sum_{m,n} \frac{(-\hbar)^m}{m!n!} \ab(\tilde{E}_{\varphi\psi}\pdv{}{a,c} + \tilde{E}_{\psi\psi}\pdv{}{b,c})^m \ab(\frac{c}{1-y} \ab(a\pdv{}{x}+b\pdv{}{y}))^n \msc{D}(x,y) \Big|_{a,b,c=0} \\ ={}& \sum_{n} \frac{(-\hbar)^n}{(n!)^2} \ab(\tilde{E}_{\varphi\psi}\pdv{}{a,c} + \tilde{E}_{\psi\psi}\pdv{}{b,c})^n \ab(\frac{c}{1-y} \ab(a\pdv{}{x}+b\pdv{}{y}))^n \msc{D}(x,y) \Big|_{a,b,c=0} \\ ={}& \sum_n \frac{(-\hbar)^n}{n!}\ab(\tilde{E}_{\varphi\psi}\pdv{}{a} + \tilde{E}_{\psi\psi}\pdv{}{b})^n \ab(\frac{1}{1-y} \ab(a\pdv{}{x}+b\pdv{}{y}))^n \msc{D}(x,y) \Big|_{a,b,c=0} \\ ={}& \sum_n \ab(\frac{(-\hbar)^n}{1-y} \ab(\tilde{E}_{\varphi\psi}\pdv{}{x} + \tilde{E}_{\psi\psi} \pdv{}{y}))^n \msc{D}(x,y). \end{align*} Now set $E^{\extra}(\partial_{\bt}) \coloneqq \frac{-\hbar}{1-y}\ab(\tilde{E}_{\varphi\psi}\pdv{}{x} + \tilde{E}_{\psi\psi}\pdv{}{y})$. To deal with the contribution of $V^{\bB}(\partial_{\bt}, \partial_{\bt})$, we compute \begin{align*} e^{-\hbar V^{\bB}(\partial_{\bt}, \partial_{\bt})}&(1-y)e^{\hbar V^{\bB}(\partial_{\bt}, \partial_{\bt})} \\ &= \ab(\sum_{m\geq 0} \frac{(-\hbar V^{\bB}(\partial_{\bt}, \partial_{\bt}))^m}{m!})(1-y) \ab(\sum_{n\geq 0} \frac{(-\hbar V^{\bB}(\partial_{\bt}, \partial_{\bt}))^n}{n!}) \\ &= \sum_{n \geq 0} \sum_{\ell+m = n} \frac{(-\hbar V^{\bB}(\partial_{\bt}, \partial_{\bt}))^{\ell}}{\ell!}(1-y) \frac{(\hbar V^{\bB}(\partial_{\bt}, \partial_{\bt}))^m}{m!} \\ &= \sum_{n \geq 0}\frac{1}{n!} \ab(\ab[-,V^{\bB}(\partial_{\bt}, \partial_{\bt})])^n (1-y) \\ &= (1-y) + \hbar\ab(\tilde{E}_{\varphi\psi} \pdv{}{x} + \tilde{E}_{\psi\psi} \pdv{}{y}). \end{align*} Dividing by $1-y$, we see that \[ (1-y)^{-1} e^{-\hbar V^{\bB}(\partial_{\bt}, \partial_{\bt})}(1-y)e^{\hbar V^{\bB}(\partial_{\bt}, \partial_{\bt})} = 1-E^{\extra}(\partial_{\bt}), \] or in other words that \[ e^{-\hbar V^{\bB}(\partial_{\bt}, \partial_{\bt})}(1-y)^{-1}e^{\hbar V^{\bB}(\partial_{\bt}, \partial_{\bt})} (1-y) = \sum_{n \geq 0} E^{\extra}(\partial_{\bt})^n.\] Putting all of this together, we see that \begin{align*} e^{f^{\bA}(\hbar, x, y)} ={}& e^{\hbar(V^{\bB}(\partial_{\bt}, \partial_{\bt})+V^{\extra}(\partial_{\bt}, \partial_{\bt}))} e^{\frac{c}{1-y}\ab(a\pdv{}{x}+b\pdv{}{y})}\frac{e^{\tilde{P}^{\bB}(\hbar, x, y)}}{1-y} \Bigg|_{a,b,c=0} \\ ={}& e^{\hbar V^{\bB}(\partial_{\bt}, \partial_{\bt})} \sum_{n \geq 0} E^{\extra}(\partial_{\bt})^n (1-y)^{-1} e^{\tilde{P}^{\bB}(\hbar, x, y)} \\ ={}& (1-y)^{-1} e^{\hbar V^{\bB}(\partial_{\bt}, \partial_{\bt})} e^{\tilde{P}^{\bB}(\hbar, x, y)} \\ ={}& \frac{e^{f^{\bB}(\hbar, x, y)}}{1-y}. \end{align*} The desired result follows by taking logarithms. \end{proof} \begin{cor}[B-model Feynman rule]\label{cor:bmodelfeynman} For any $g,m,n$, we have \[ f^{\bB}_{g,m,n} \in \Q[X]_{3g-3+m}. \] \end{cor} \begin{proof} By~\Cref{thm:allgenusmirror}, we see that $f^{\bA}_{g,m,n} = f^{\bB}_{g,m,n} + \delta_{g,1}\delta_{m,0} (n-1)!$. The result then follows from~\Cref{cor:amodelfeynman} by choosing $\ba = (1^m 0^n)$ and $\bb = (0^n 1^m)$. \end{proof} \section{Anomaly equations}\label{sec:Holomorphic anomaly equations} Our goal is to prove the following theorem. \begin{thm}\label{thm:hae} The $P_{g,m}$ satisfy the differential equations \begin{align} - \partial_{A} P_g = \frac{1}{2} \ab(P_{g-1,2} + \sum_{g_1+g_2 = g} P_{g_1, 1} P_{g_2, 2}), \label{eqn:hae1} \\ \ab(-2 \partial_{A} + \partial_{B} + (A+2B) \partial_{B_2} - \ab((B-X)(A+2B)-B_2-r_0 X)\partial_{B_3})P_g = 0\label{eqn:hae2}. \end{align} \end{thm} In order to prove this, we introduce a smaller ring of modified generators which contains the $P_g$. \begin{defn} Define the modified generators \begin{align*} \msc{E}_1 \coloneqq \tilde{E}_{\varphi\varphi}, \qquad \msc{E}_2 \coloneqq \tilde{E}_{\varphi\psi}, \qquad \msc{E}_3 \coloneqq \tilde{E}_{\psi\psi} \end{align*} and then set \[ \tilde{\msc{R}} \coloneqq \Q[\msc{E}_1, \msc{E}_2, \msc{E}_3, X] \subset \msc{R}. \] \end{defn} \begin{rmk} The ring $\tilde{\msc{R}}$ is in fact invariant under the choice of gauge. Direct computation yields \begin{align*} \msc{E}_1^{\G} &= \msc{E}_1^{\bO} + c_{12}, \\ \msc{E}_2^{\G} &= \msc{E}_2^{\bO} + c_{11} \msc{E}_1^{\bO} + c_{11}c_{12} + c_2, \\ \msc{E}_3^{\G} &= \msc{E}_3^{\bO} + 2c_{11} \msc{E}_2^{\bO} + c_{11}^2 \msc{E}_1^{\bO} + c_{11}^2 c_{12} + 2 c_{11}c_2 + c_3, \end{align*} so we will write the generators with no superscript and $\G = \bO$. Here, recall that $c_{11}, c_{12}, c_2, c_3 \in \Q[X]$. \end{rmk} \begin{rmk} Our generators are related to the generators $v_1$, $v_2$, and $v_3$ introduced in~\cite{yy04} by the formulae \[ v_1 = -\msc{E}_1, \qquad v_2 = -\msc{E}_2, \qquad \text{and} \qquad v_3 = \msc{E}_3 - \msc{E}_2 X. \] \end{rmk} \begin{lem} The ring $\tilde{\msc{R}}$ is closed under the derivative $D$. \end{lem} \begin{proof} A direct computation yields \begin{align*} D \msc{E}_1 &= -X(\msc{E}_1 + r_0) - \msc{E}_1^2 + 2 \msc{E}_2, \\ D \msc{E}_2 &= -X \msc{E}_2 - \msc{E}_1 \msc{E}_2 + \msc{E}_3, \\ D \msc{E}_3 &= r_1 X - X \msc{E}_3 - \msc{E}_2^2. \end{align*} \end{proof} \begin{thm}[Reduction of generators]\label{thm:reduction} Let $g > 1$. Then $P_g \in \tilde{\msc{R}}$. \end{thm} \begin{proof} First, note that by definition, we have $P_g = \tilde{P}_g$. We will now prove that all $\tilde{P}_{g,m} \in \tilde{\msc{R}}$ by induction on the lexicographic order in $(g,m)$. Recall that $f_{g,m}^{\bB}$ can be computed from $\tilde{P}^{\bB}_{h \leq g,m,n}$ by the geometric quantization of $\msc{R}^{\bB}$. The contribution of each stable graph to this quantization is given by the following construction: \begin{itemize} \item At each leg, we place $\varphi_1$ or $\varphi_0 \psi$; \item At every edge, we place the bivector \begin{align*} &\tilde{E}_{\varphi\varphi} (\varphi_1 \otimes \varphi_1) + \tilde{E}_{\varphi\psi}(\varphi_1 \otimes \varphi_0 \psi + \varphi_0 \psi \otimes \varphi_1) + \tilde{E}_{\psi\psi}(\varphi_0\psi \otimes \varphi_0\psi) \\ =&\msc{E}_1 (\varphi_1 \otimes \varphi_1) + \msc{E}_2(\varphi_1 \otimes \varphi_0 \psi + \varphi_0 \psi \otimes \varphi_1) + \msc{E}_3(\varphi_0\psi \otimes \varphi_0\psi); \end{align*} \item At every vertex, we place the linear map $\varphi_1^{\otimes m} \otimes (\varphi_0\psi)^{\otimes n} \mapsto \tilde{P}_{g,m,n}$. \end{itemize} The base cases are $\tilde{P}_{1,0,1} = \frac{\chi(Z)}{24}-1$ and $\tilde{P}_{0,3} = 1$. The dilaton equation implies that if $\tilde{P}_{g,m} \in \tilde{\msc{R}}$, then $\tilde{P}_{g,m,n} \in \tilde{\msc{R}}$ for all $n$. Now we assume that $\tilde{P}_{h,\ell,n} \in \tilde{\msc{R}}$ for all $(h,\ell) < (g,m)$. Then we know $f_{g,m}^{\bB} \in \Q[X]$ by~\Cref{cor:bmodelfeynman}. Computing it by the stable graph sum, we see \[ f_{g,m}^{\bB} = \tilde{P}_{g,m} + \sum_{\Gamma \text{ non-leading}} \frac{1}{\ab|\Aut \Gamma|} \on{Cont}_{\Gamma}. \] By the inductive hypothesis and the formula for the edge contributions, $\on{Cont}_{\Gamma} \in \tilde{\msc{R}}$ for any non-leading $\Gamma$. The desired result follows immediately. \end{proof} \begin{proof}[Proof of~\Cref{thm:hae}] The second equation~\Cref{eqn:hae2} is equivalent to~\Cref{thm:reduction} by the results of~\cite{yy04}, so we only need to prove~\Cref{eqn:hae1}. We proceed by differentiating the quantization action. By definition, we have \[ e^{P^{\bB}(\hbar, x, y-E_{\psi}x)} = e^{-\hbar V^{\bB}(\partial_{\bt}, \partial_{\bt})} e^{f^{\bB}(\hbar, x, y)}. \] Applying $\partial$ being either $\partial_A$, $\partial_B$, $\partial_{B_2}$, or $\partial_{B_3}$, we see that \begin{align*} e^{\tilde{P}^{\bB}(\hbar, x, y)} &= -\hbar \partial V^{\bB}(\partial_{\bt}, \partial_{\bt}) e^{V^{\bB}(\partial_{\bt}, \partial_{\bt})} e^{f^{\bB}(\hbar, x, y)} \\ &= -\hbar \partial V^{\bB}(\partial_{\bt}, \partial_{\bt}) e^{\tilde{P}^{\bB}(\hbar, x, y)}. \end{align*} Making the change of variables $\bt' = (x',y') \coloneqq (x, y-E_{\psi}x)$, we then see that \[ V^{\bB}(\partial_{\bt}, \partial_{\bt}) = \frac{1}{2} E_{\varphi\varphi} \pdv[order=2]{}{x'} + E_{\varphi\psi} \pdv{}{x',y'} + E_{\psi\psi} \pdv[order=2]{}{y'} \eqqcolon V^{\bB,\ms{small}}(\partial_{\bt'}, \partial_{\bt'}). \] From now on, we will replace $x'$ by $x$ and $y'$ by $y$ for simplicity. We now see that \[ e^{P^{\bB}(\hbar, x, y)} = \partial V^{\bB, \ms{small}}(\partial_{\bt}, \partial_{\bt}) e^{P^{\bB}(\hbar, x, y)}. \] First, $\partial_A V^{\bB, \ms{small}}(\partial_{\bt}, \partial_{\bt}) = \frac{1}{2} \pdv[order=2]{}{x}$, so we obtain \[ \partial_A P^{\bB}(\hbar, x, y) = -\frac{\hbar}{2} \pdv[order=2]{}{x} P^{\bB}(\hbar, x, y) - \frac{\hbar}{2} \ab(\pdv{}{x} P^{\bB}(\hbar, x, y))^2. \] Setting $x=y=0$, we see that \[ \sum_{g \geq 2} \hbar^{g-1} \partial_A P_g = -\frac{1}{2} \ab(\sum_{g \geq 2} \hbar^{g-1} P_{g-1,2} + \ab(\sum_{g_1,g_2>0} \hbar^{g_1+g_2-1} P_{g_1} P_{g_2})). \] Taking the coefficient of $\hbar^{g-1}$ on both sides, we obtain~\Cref{eqn:hae1}. \end{proof} \printbibliography \end{document}
2412.06572v2
http://arxiv.org/abs/2412.06572v2
Quaternionic spinors and horospheres in 4-dimensional hyperbolic geometry
\documentclass{amsart} \usepackage{amsfonts} \usepackage{amsmath} \usepackage{amssymb} \usepackage{amsthm} \usepackage{multicol} \usepackage[margin=3cm]{geometry} \usepackage{mathabx} \usepackage{arydshln} \usepackage{makecell} \usepackage[pagebackref, pdftex]{hyperref} \usepackage{cite} \renewcommand{\backreftwosep}{\backrefsep} \renewcommand{\backreflastsep}{\backrefsep} \renewcommand*{\backref}[1]{} \renewcommand*{\backrefalt}[4]{ \ifcase #1 [No citations.] \or [#2] \else [#2] } \usepackage{graphicx} \usepackage{import} \usepackage{tikz} \usetikzlibrary{calc, arrows, decorations.markings, decorations.pathmorphing, positioning, decorations.pathreplacing} \setcounter{tocdepth}{1} \AtBeginDocument{ \def\MR#1{} } \newcommand{\To}{\longrightarrow} \newcommand{\0}{{\bf 0}} \newcommand{\1}{{\bf 1}} \newcommand{\A}{\mathcal{A}} \newcommand{\B}{\mathcal{B}} \newcommand{\C}{\mathbb{C}} \newcommand{\Cat}{\mathcal{C}} \newcommand{\CP}{\mathbb{CP}} \newcommand{\D}{\mathcal{D}} \newcommand{\Disc}{\mathbb{D}} \newcommand{\e}{\mathbf{e}} \newcommand{\E}{\mathbb{E}} \newcommand{\F}{\mathcal{F}} \newcommand{\FF}{\mathbb{F}} \newcommand{\G}{\mathcal{G}} \newcommand{\h}{\mathfrak{h}} \newcommand{\HH}{\mathbb{H}} \newcommand{\hyp}{\mathcal{H}} \newcommand{\I}{\mathcal{I}} \newcommand{\II}{\mathbb{I}} \newcommand{\J}{\mathcal J} \newcommand{\K}{\mathcal{K}} \renewcommand{\L}{\mathbb{L}} \newcommand{\Lag}{\mathcal L} \newcommand{\M}{\mathcal{M}} \newcommand{\Mbar}{\overline{\mathcal{M}}} \newcommand{\MF}{\mathcal{MF}} \newcommand{\N}{\mathbb{N}} \renewcommand{\P}{\mathcal{P}} \newcommand{\pH}{\mathbf{\$ H}} \newcommand{\Q}{\mathbb{Q}} \newcommand{\QQ}{\mathcal{Q}} \newcommand{\R}{\mathbb{R}} \newcommand{\Ring}{\mathcal{R}} \newcommand{\RP}{\mathbb{RP}} \newcommand{\s}{\mathfrak{s}} \renewcommand{\S}{\mathcal{S}} \newcommand{\T}{\mathbb{T}} \newcommand{\TT}{\mathcal{T}} \newcommand{\U}{\mathbb{U}} \newcommand{\UU}{\mathcal{U}} \newcommand{\V}{\mathcal{V}} \newcommand{\VI}{\mathcal{VI}} \newcommand{\x}{{\bf x}} \newcommand{\X}{\mathcal{X}} \newcommand{\Y}{\mathcal{Y}} \newcommand{\Z}{\mathbb{Z}} \newcommand{\ZZ}{\mathcal{Z}} \DeclareMathOperator{\ad}{ad} \DeclareMathOperator{\Aut}{Aut} \DeclareMathOperator{\Byp}{Byp} \DeclareMathOperator{\Cl}{Cl} \DeclareMathOperator{\Conv}{Conv} \DeclareMathOperator{\Down}{Down} \DeclareMathOperator{\End}{End} \DeclareMathOperator{\ev}{ev} \DeclareMathOperator{\For}{For} \DeclareMathOperator{\Fr}{Fr} \DeclareMathOperator{\gr}{gr} \DeclareMathOperator{\Gr}{Gr} \DeclareMathOperator{\Hom}{Hom} \DeclareMathOperator{\Hor}{Hor} \DeclareMathOperator{\Id}{Id} \let\Im\relax \DeclareMathOperator{\Im}{Im} \let\Re\relax \DeclareMathOperator{\Re}{Re} \DeclareMathOperator{\Int}{Int} \DeclareMathOperator{\inv}{inv} \DeclareMathOperator{\Inv}{Inv} \DeclareMathOperator{\Isom}{Isom} \DeclareMathOperator{\Mat}{Mat} \DeclareMathOperator{\Mor}{Mor} \DeclareMathOperator{\Ob}{Ob} \DeclareMathOperator{\Pin}{Pin} \DeclareMathOperator{\pdet}{pdet} \DeclareMathOperator{\Quad}{Quad} \DeclareMathOperator{\qdet}{qdet} \DeclareMathOperator{\Rep}{Rep} \DeclareMathOperator*{\Res}{Res} \DeclareMathOperator{\Sgn}{Sgn} \DeclareMathOperator{\Spin}{Spin} \DeclareMathOperator{\Sut}{Sut} \DeclareMathOperator{\Sym}{Sym} \DeclareMathOperator{\Top}{Top} \DeclareMathOperator{\Tr}{Tr} \DeclareMathOperator{\Trace}{Trace} \DeclareMathOperator{\Up}{Up} \numberwithin{equation}{subsection} \newtheorem{theorem}{Theorem} \newtheorem{conj}[equation]{Conjecture} \newtheorem{corollary}[equation]{Corollary} \newtheorem{cor}[equation]{Corollary} \newtheorem{lemma}[equation]{Lemma} \newtheorem{lem}[equation]{Lemma} \newtheorem{conjecture}[equation]{Conjecture} \newtheorem{prob}[equation]{Problem} \newtheorem{proposition}[equation]{Proposition} \newtheorem{prop}[equation]{Proposition} \newtheorem{qn}[equation]{Question} \newtheorem{axiom}[equation]{Axiom} \newtheorem{claim}[equation]{Claim} \theoremstyle{definition} \newtheorem{defn}[equation]{Definition} \newtheorem{example}[equation]{Example} \newcommand{\refsec}[1]{Section~\ref{Sec:#1}} \newcommand{\refdef}[1]{Definition~\ref{Def:#1}} \newcommand{\reffig}[1]{Figure~\ref{Fig:#1}} \newcommand{\reftable}[1]{Table~\ref{Table:#1}} \newcommand{\refeqn}[1]{\eqref{Eqn:#1}} \newcommand{\reflem}[1]{Lemma~\ref{Lem:#1}} \newcommand{\refprop}[1]{Proposition~\ref{Prop:#1}} \newcommand{\refthm}[1]{Theorem~\ref{Thm:#1}} \newcommand{\refcor}[1]{Corollary~\ref{Cor:#1}} \newcommand{\refeg}[1]{Example~\ref{Eg:#1}} \renewcommand{\theenumi}{(\roman{enumi})} \renewcommand{\labelenumi}{\theenumi} \begin{document} \title{Quaternionic spinors and horospheres in 4-dimensional hyperbolic geometry} \author{Daniel V. Mathews} \address{School of Mathematics, Monash University, 9 Rainforest Walk, Clayton VIC 3800, Australia; School of Physical and Mathematical Sciences, Nanyang Technological University, 21 Nanyang Link, Singapore 637371} \email{[email protected]} \author{Varsha} \address{Department of Mathematics, University College London, Gower Street, London WC1E 6BT} \email{[email protected]} \begin{abstract} We give explicit bijective correspondences between three families of objects: certain pairs of quaternions, which we regard as spinors; certain flags in (1+4)-dimensional Minkowski space; and horospheres in 4-dimensional hyperbolic space decorated with certain pairs of spinorial directions. These correspondences generalise previous work of the first author, Penrose--Rindler, and Penner in lower dimensions, and use the description of 4-dimensional hyperbolic isometries via Clifford matrices studied by Ahlfors and others. We show that lambda lengths generalise to 4 dimensions, where they take quaternionic values, and are given by a certain bilinear form on quaternionic spinors. They satisfy a non-commutative Ptolemy equation, arising from quasi-Pl\"{u}cker relations in the Gel'fand--Retakh theory of noncommutative determinants. We also study various structures of geometric and topological interest that arise in the process. \end{abstract} \maketitle \tableofcontents \section{Introduction} \label{Sec:introduction} \subsection{Overview and main results} In previous work \cite{Mathews_Spinors_horospheres}, the first author demonstrated a bijective correspondence between complex \emph{spinors} or \emph{spin vectors}, and \emph{spin-decorated horospheres} in 3-dimensional hyperbolic space. This extended the work of Penrose--Rindler \cite{Penrose_Rindler84}, which associated to such spin vectors certain \emph{flags} in Minkowski space $\R^{1,3}$. This previous work also gave several further connections between spinors and 3-dimensional hyperbolic geometry, and has been used to study knot complements \cite{HIMS_lambda_figure8} and generalise Descartes' classic circle theorem \cite{Mathews_Zymaris}. In this paper, we generalise these results from the complex numbers $\C$ to the \emph{quaternions} $\HH$, from 3- to \emph{4-dimensional} hyperbolic space $\hyp^4$, and from $(1+3)$- to \emph{$(1+4)$-dimensional} Minkowski space $\R^{1,4}$. It is well known that the progression from 2 to 3 to 4 dimensions in hyperbolic geometry is closely related to the progression from $\R$ to $\C$ to $\HH$. In particular, the orientation preserving isometry groups of hyperbolic space of dimensions 2, 3, and 4 are respectively isomorphic to certain groups of M\"{o}bius transformations over $\R$, $\C$ and $\HH$ respectively, although the quaternionic case is more subtle than the others. The results of this paper show that this progression also extends to spinors, flags, and horospheres, with corresponding subtleties. The first main result of this paper is a generalisation of the spinor-horosphere correspondence of \cite{Mathews_Spinors_horospheres}, also incorporating the corresponding notions of flags, which we call \emph{multiflags}. \begin{theorem}[Spinor--multiflag--horosphere correspondence] \label{Thm:main_thm_1} There is an explicit, smooth, bijective, $SL_2 \$$-equivariant correspondence between the following: \begin{enumerate} \item quaternionic spinors; \item spin multiflags; \item spin-decorated horospheres in $\hyp^4$. \end{enumerate} \end{theorem} Roughly, quaternionic spinors are pairs of quaternions $(\xi, \eta)$ satisfying certain conditions; multiflags consist of a pair of orthogonal 2-dimensional flags on a lightlike flagpole in $\R^{1,4}$; decorations on horospheres consist of orthogonal pairs of parallel direction fields; and $SL_2 \$ $ is a group of quaternionic matrices describing isometries of $\hyp^4$ analogously to $SL_2 \C$ for $\hyp^3$. We properly explain these and all notions involved as we proceed. In \cite{Mathews_Spinors_horospheres} the first author showed that the notion of (real) \emph{lambda length} introduced by Penner in the 2-dimensional context \cite{Penner87} as a distance between horospheres, extends to a \emph{complex} lambda length between spin-decorated horospheres in $\hyp^3$, and moreover is given by a natural antisymmetric bilinear form on spinors, taking a determinant. The second main result of this paper extends these results to quaternions and $\hyp^4$. We show there is a well-defined notion of \emph{quaternionic} lambda length between spin-decorated horospheres in $\hyp^4$, which uses the isomorphism of unit quaternions with $\Spin(3)$. This lambda length agrees a with a bilinear form on spinors, denoted $\{ \cdot, \cdot \}$, given by a certain \emph{pseudo-determinant} considered by Ahlfors in a series of papers \cite{Ahlfors_Clifford85, Ahlfors_Mobius85, Ahlfors_Mobius_86, Ahlfors_fixedpoints_85, Ahlfors_84}, which we denote $\pdet$, as follows. \begin{theorem}[Lambda lengths are pseudo-determinants] \label{Thm:main_thm_2} Let $\kappa_1 = (\xi_1, \eta_1)$ and $\kappa_2 = (\xi_2, \eta_2)$ be two quaternionic spinors, corresponding to spin-decorated horospheres $(\h_1, W_1)$ and $(\h_2, W_2)$ in $\hyp^4$. Then the lambda length $\lambda_{12}$ from $\h_1$ to $\h_2$ is given by \begin{equation} \label{Eqn:lambda_pdet} \lambda_{12} = \pdet \begin{pmatrix} \xi_1 & \xi_2 \\ \eta_1 & \eta_2 \end{pmatrix} = \{ \kappa_1, \kappa_2 \} = \xi_1^* \eta_2 - \eta_1^* \xi_2 \end{equation} \end{theorem} Here $*$ denotes Clifford algebra reversion conjugation, which applies to quaternions as $(a+bi+cj+dk)^* = a+bi+cj-dk$. The lambda length is no longer antisymmetric but instead satisfies $\lambda_{12} = -\lambda_{21}^*$. In \cite{Mathews_Spinors_horospheres}, the first author showed that given four complex spinors $\kappa_n$ corresponding to four spin-decorated horospheres $\h_n$, the six lambda lengths between them satisfy a \emph{Ptolemy equation} \[ \lambda_{01} \lambda_{23} + \lambda_{03} \lambda_{12} = \lambda_{02} \lambda_{13}, \] where $\lambda_{mn}$ is the lambda length from $\h_m$ to $\h_n$. The third main result of this paper generalises this result to the non-commutative context of quaternions, and 4 dimensions. \begin{theorem}[Non-commutative Ptolemy equation] \label{Thm:main_thm_3} Given four spin-decorated horospheres $(\h_n, W_n)$ in $\hyp^4$, $n=0,1,2,3$, let $\lambda_{mn}$ denote the lambda length from $(\h_m, W_m)$ to $(\h_n, W_n)$. Then \begin{equation} \label{Eqn:noncomm_Plucker} \lambda_{02}^{-1} \lambda_{01} \lambda_{31}^{-1} \lambda_{32} + \lambda_{02}^{-1} \lambda_{03} \lambda_{13}^{-1} \lambda_{12} = 1. \end{equation} \end{theorem} Similar non-commutative Ptolemy equations were discussed by Berenstein and Retakh \cite{Berenstein_Retakh_18, Retakh_OW_report_13} in the context of non-commutative surfaces and cluster algebras. In the rest of this introduction we summarise the results of this paper, and give further details, background, and context. \subsection{Quaternionic spinors and paravectors} \label{Sec:intro_spinors_paravectors} The spinors in \refthm{main_thm_1} are a quaternionic generalisation of the \emph{spin vectors} of Penrose and Rindler in \cite{Penrose_Rindler84}, which consisted of pairs of complex numbers. Here however, defining quaternionic spinors naively as pairs of quaternions cannot provide a bijection with spin-decorated horospheres: as we see below in \refsec{horospheres_decorations}, the space of spin-decorated horospheres is $7$-real-dimensional, so any bijection with quaternionic spinors must cut down the space of spinors from $\HH^2$ to a real-codimension-1 subset. The definition of quaternionic spinors can be motivated as follows. In the complex case, a spinor $(\xi, \eta) \in \C^2$ yields a point on the future light cone in $\R^{1,3}$ via the $2 \times 2$ matrix \begin{equation} \label{Eqn:spinor_matrix} \begin{pmatrix} \xi \\ \eta \end{pmatrix} \begin{pmatrix} \overline{\xi} & \overline{\eta} \end{pmatrix} = \begin{pmatrix} |\xi|^2 & \xi \overline{\eta} \\ \eta \overline{\xi} & |\eta|^2 \end{pmatrix} \end{equation} The diagonal elements of this matrix are non-negative, and the Pauli matrices can be used to identify a point on the future light cone in $(1+3)$-dimensional Minkowski space, by equating the above matrix with \begin{equation} \label{Eqn:R13_matrix} \frac{1}{2} \begin{pmatrix} T+Z & X+iY \\ X-iY & T-Z \end{pmatrix}, \quad \text{corresponding to} \quad (T,X,Y,Z) \in \R^{1,3}. \end{equation} If we now consider $\xi, \eta$ above as quaternions, then equation \refeqn{spinor_matrix} still makes sense, using the standard quaternion conjugation. The diagonal elements of \refeqn{spinor_matrix} are again non-negative reals, but the off-diagonal entries in general can be arbitrary quaternions. We can identify points in $(1+4)$-dimensional Minkowski space with matrices by a generalisation of \refeqn{R13_matrix}, namely \begin{equation} \label{Eqn:R14_matrix} \frac{1}{2} \begin{pmatrix} T+Z & W+iX+jY \\ W-iX-jY & T-Z \end{pmatrix} \quad \text{corresponding to} \quad (T,W,X,Y,Z) \in \R^{1,4}. \end{equation} Requiring quaternionic spinors $(\xi, \eta)$ to yield points in $\R^{1,4}$ then motivates the following definitions. \begin{defn} \ \label{Def:quaternionic_spinor} \begin{enumerate} \item The real-linear subspace of $\HH$ spanned by $1$, $i$ and $j$ is denoted $\$\R^3$. Its elements are called \emph{paravectors}. \item A \emph{quaternionic spinor}, or just \emph{spinor}, is a pair of quaternions $\kappa = (\xi, \eta)$, not both zero, such that $\xi \overline{\eta} \in \$\R^3$. The set of quaternionic spinors is denoted $S\HH$. \end{enumerate} \end{defn} Thus \[ S\HH = \left\{ \left( \xi, \eta \right) \in \HH^2 \; \mid \; (\xi, \eta) \neq (0,0), \quad \xi \overline{\eta} \in \$\R^3 \right\}. \] The notation $\$\R^3$ and terminology follows Lounesto \cite[ch. 19]{Lounesto_Clifford_book_01} and comes from the context of Clifford algebras. Given a real vector space $V$ with a nondegenerate quadratic form $Q$, the Clifford algebra $\Cl(V,Q)$ contains $V$ as its subspace of degree-1 elements or \emph{vectors}, and $\R$ as its subspace of degree-0 elements of \emph{scalars}. \emph{Paravectors} are sums of elements of degree 0 or 1, so the space of paravectors is $\R \oplus V$. Paravectors arise naturally in the study of M\"{o}bius transformations and Clifford algebras, e.g. \cite{Ahlfors_84, Ahlfors_Clifford85, Ahlfors_Mobius85, Ahlfors_fixedpoints_85, Ahlfors_Mobius_86, Cao_Waterman_98, Lounesto_Latvamaa_80, Maass_49, Vahlen_1902, Waterman_93, Kellerhals01, Gongopadhyay_12, Ahlfors_Lounesto_89}. Often they are known as \emph{vectors}; we discuss terminology in \refsec{vectors_paravectors}. When $V = \R^2$ and $Q$ is negative definite, we have $\Cl(V,Q) \cong \HH$, with the vectors being $\R i + \R j$, the scalars being $\R$, and the paravectors as in \refdef{quaternionic_spinor}. Paravectors are then precisely those quaternions $x$ such that $x^* = x$. The defining condition $\xi \overline{\eta} \in \$\R^3$ of quaternionic spinors cuts out a 7-dimensional subset of $\HH^2$, as required for a bijection with spin-decorated horospheres. Indeed, $S\HH \cong S^3 \times S^3 \times \R$ (\reflem{topology_of_SH}). The condition $\xi \overline{\eta} \in \$\R^3$ is known (e.g. \cite{Ahlfors_Clifford85, Ahlfors_Mobius85, Ahlfors_Mobius_86, Ahlfors_fixedpoints_85, Ahlfors_84}) to have multiple equivalent reformulations: it is equivalent to $\xi^* \eta \in \$\R^3$ and, when $\eta \neq 0$, is equivalent to $\xi \eta^{-1} \in \$\R^3$. The matrices of \refeqn{R14_matrix} are precisely those $2 \times 2$ matrices $S$ with paravector entries and which satisfy $S = \bar{S}^T$, i.e. \emph{paravector Hermitian} matrices. Just as Hermitian matrices with complex entries correspond to points of $\R^{1,3}$, paravector Hermitian matrices with quaternion entries correspond to points of $\R^{1,4}$. For a paravector Hermitian matrix $S$ as in \refeqn{R14_matrix} corresponding to a point $p = (T,W,X,Y,Z) \in \R^{1,4}$, as in the complex case we have \[ 4 \det S = \langle p, p \rangle, \quad \Tr S = T. \] Here $\langle \cdot, \cdot \rangle$ denotes the Minkowski inner product $dT^2 - dW^2 - dX^2 - dY^2 - dZ^2$, and $\det$ is the usual $2 \times 2$ determinant (which agrees with $\pdet$ for paravector Hermitian matrices). With \refdef{quaternionic_spinor} in hand, we can associate to a quaternionic spinor $\kappa = (\xi, \eta) \in S\HH$ a point $(T,W,X,Y,Z) \in \R^{1,4}$ by equating the matrices \refeqn{spinor_matrix} and \refeqn{R14_matrix}. Since a matrix \refeqn{spinor_matrix} arising from a spinor has determinant $0$ and positive trace, all points in $\R^{1,4}$ arising from spinors $\kappa \in S\HH$ in fact lie on the future light cone. \subsection{Multiflags} In \cite{Penrose_Rindler84}, Penrose and Rindler associated to a complex spinor $(\xi, \eta)$ certain \emph{flags}. Such a flag involves a point $p$ on the light cone and a 2-plane tangent to the light cone, containing the line $p \R$. A family of complex spinors of the form $(\xi, \eta) e^{i\theta}$ all yield the same point $p$, but multiplication by $e^{i\theta}$ rotates the 2-plane by $2\theta$ about $p \R$. As shown in \cite{Mathews_Spinors_horospheres}, the flag can be given by the derivative of the map which sends $(\xi, \eta) \mapsto (T,X,Y,Z)$ in the direction of a related spinor $(\overline{\eta}, -\overline{\xi})i$. In the quaternionic case, again we obtain a point $p$ on the future light cone, but \emph{two} such flags naturally arise, which we call the \emph{$i$-flag} and \emph{$j$-flag}. As the names suggest, these flags arise from the quaternions $i$ and $j$, and indeed they are given by the derivative of the map $(\xi, \eta) \mapsto (T,W,X,Y,Z)$ in certain directions $(\eta', -\xi')i$ and $(\eta', -\xi')j$. (Here $'$ denotes another conjugation on $\HH$, $x' = \bar{x}^*$, which arises naturally from Clifford algebras. A discussion of the conjugations on Clifford algebras is given below in \refsec{Clifford_general}, and for $\HH$ specifically in \refsec{quaternion_involution}.) See \reffig{1}. This yields a \emph{multiflag}, formally defined in \refdef{multiflag}. When $(\xi, \eta)$ are both complex, the $i$-flag agrees with the flag of \cite{Penrose_Rindler84} and \cite{Mathews_Spinors_horospheres}. Multiplying a spinor $(\xi, \eta)$ on the right by a unit quaternion $x$ yields another spinor. Generalising the complex case, the family of quaternionic spinors $(\xi, \eta)x$ all yield the same point $p$, and the multiflags rotate about $p \R$. This rotation is essentially given by the standard identification of unit quaternions with $\Spin(3)$, the double cover of the 3-dimensional rotation group. \begin{figure}[h] \begin{center} \begin{tikzpicture}[scale=0.8] \draw[black] (-4,4)--(0,0)--(4,4); \draw[blue, dashed, thick] plot[variable=\t,samples=1000,domain=-75.5:75.5] ({tan(\t)},{sec(\t)}); \draw[black] (0,4) ellipse (4cm and 0.4cm); \draw[blue, dotted, thick] (-0.2,3.7) .. controls (-1,0.25) .. (1.8,4.27); \draw[blue] (0,4) ellipse (3.85cm and 0.3cm); \draw[red, thick] (0,0)--(2,3); \node[black] at (-3.5,3){$L^+$}; ll[white](2.5,2.5)--(3.2,2.5)--(3.2,3.3)--(2.5,3.3)--cycle; \node[black] at (2.7,3){$p$}; \draw[red, thick] (2,3)--(2.2,2.3)--(1.33,2)--(2,3); \draw[green!50!black, thick] (-0.1,0)--(1.9,3); \draw[green!50!black, thick] (1.9,3)--(1.2,3.2)--(1.23,2)--(1.9,3); ll[black] (0,0) circle (0.1cm); ll[black] (2,3) circle (0.1cm); \node[blue] at (-0.75,2.5){$\h$}; \node[blue] at (-2.25,3){$\hyp^4$}; \draw[red, ->, thick] (0.1,2.7)--(0.6,2.7); \draw[green!50!black, ->, thick] (0.1,2.7)--(-0.2,3.15); \draw[red, ->, thick] (-0.2,2)--(0.1,2); \draw[green!50!black, ->, thick] (-0.2,2)--(-0.4,2.3); \end{tikzpicture} \caption{Multiflag and decorated horosphere corresponding to a quaternionic spinor.} \label{Fig:1} \end{center} \end{figure} The map sending $\kappa = (\xi, \eta)$ to $p$, which we denote $\phi_1$ as in \cite{Mathews_Spinors_horospheres}, is closely related to the quaternionic Hopf fibration. Scaling $\kappa$ by a real factor also scales $p$ by a real factor and preserves the $i$- and $j$-flags. Scaling $\kappa$ to so that it lies on $S^7$, i.e. so that $|\xi|^2 + |\eta|^2 = 1$, this map is a restriction of the Hopf fibration. These spinors $S\HH \cap S^7$ are precisely the preimage of the equatorial $S^3 \subset S^4$ given by $\$\R^3 \cup \{\infty\} \subset \HH \cup \{\infty\}$ under the Hopf fibration. We discuss this in \refsec{spinors_to_light_cone}. In defining multiflags, we find that the tangent space to $S\HH$ at a point $\kappa$ has an interesting structure: it is naturally parallelisable and decomposes into the direct sum of three orthogonal real subspaces (\reflem{TSH}). First, there is the 1-dimensional subspace in the radial direction. Second, there is the 3-dimensional subspace given by the fibres of the Hopf fibration: these are the directions in which $p$ remains constant but multiflags rotate. Third, there is a 3-dimensional subspace $(\eta', -\xi')\$\R^3$, a copy of the paravectors in the tangent space, which maps isomorphically and conformally onto the tangent space of the celestial sphere under the derivative of $\phi_1$. We prove this in \refprop{Derivs_props} and \refprop{paravectors_conformal}. These latter two subspaces span a 6-dimensional subspace of the tangent space to the $S^7$ centred at the origin through $\kappa$. The mapping $(\xi, \eta) \mapsto (\eta', -\xi')$ is somewhat analogous the almost complex structure of a K\"{a}hler structure. Multiflags in the above sense are equivalent to a choice of lightlike subspace $\ell = p \R$, and an orientation-preserving conformal linear identification $\psi$ of the Euclidean spacelike 3-dimensional space $\ell^\perp / \ell$ with $\$\R^3$. The $i$- and $j$-flags, which lie in $\ell^\perp$, project to oriented lines in $\ell^\perp/\ell$ which are identified with the directions of $i$ and $j$. The line $\ell$ can be regarded as an ideal point of $\hyp^4$, and the conformal isomorphism $\psi \colon \$\R^3 \To \ell^\perp/\ell$ as analogous to a decoration on a horosphere, so we call such pairs $(\ell, \psi)$ \emph{decorated ideal points}. We discuss decorated ideal points in \refsec{decorated_ideal_points}. The space of multiflags $\MF$ is naturally diffeomorphic to $S^3 \times SO(3) \times \R$, with the $SO(3)$ factor corresponding to rotations of flags. Its double (universal) cover $\widetilde{\MF} \cong S^3 \times S^3 \times \R$ then consists of \emph{spin multiflags}, which must rotate through $4\pi$ to return to the same flag. The proof of \refthm{main_thm_1} constructs an explicit diffeomorphism $S\HH \cong \widetilde{\MF}$. \subsection{Horospheres and decorations} \label{Sec:horospheres_decorations} As in \cite{Mathews_Spinors_horospheres, Penner87}, from a point $p$ on the future light cone in Minkowski space, one can associate a horosphere $\h$, by intersecting the hyperboloid model of hyperbolic space with the affine hyperplane given by the equation \[ \langle p, x \rangle = 1. \] See \reffig{1}. This association between the future light cone and horospheres works in any dimension. Hence, just as real and complex spinors yield horospheres in $\hyp^2$ or $\hyp^3$, quaternionic spinors yield horospheres in $\hyp^4$. In the complex case, the flag associated to a spinor $\kappa$ describes a direction field on the corresponding horosphere $\h$ in $\hyp^3$, which is in fact parallel. This is possible as $\h$ is isometric to a Euclidean plane. In \cite{Mathews_Spinors_horospheres} we defined a \emph{decoration} on a horosphere in $\hyp^3$ to be such a parallel direction field. Multiplying $\kappa$ by $re^{i\theta}$ translates $\h$ by $2 \log r$ towards its centre and rotates its decoration by $\theta$. In the quaternionic case, each of the two flags in a multiflag describes a direction field on the corresponding horosphere $\h$ in $\hyp^4$, which we call the \emph{$i$-direction field} and \emph{$j$-direction field}. Again these direction fields are parallel, which is possible since $\h$ is isometric to Euclidean 3-space; they are also perpendicular (just like $i$ and $j$ are in $\HH$). We show this in \refprop{line_fields_parallel}. We can thus define a decoration as follows. \begin{defn} \label{Def:decorated_horosphere} A \emph{decoration} on a horosphere $\h$ in $\hyp^4$ is a pair of oriented parallel tangent direction fields $L_i$ and $L_j$ on $\h$, such that $L_i$ and $L_j$ are everywhere perpendicular. \end{defn} Thus, from a quaternionic spinor $\kappa \in S\HH$, we obtain a decoration on a horosphere $\h$. Being parallel, once $L_i$ and $L_j$ are perpendicular at one point of $\h$, they are perpendicular at all points of $\h$. Multiplying $\kappa$ by $r > 0$ again translates $\h$ by $2 \log r$ towards its centre. Multiplying $\kappa$ by a unit quaternion rotates the direction fields of the decoration using the isomorphism of unit quaternions with $\Spin(3)$. In $\hyp^3$, a decoration on a horosphere $\h$, together with a choice of normal to the horosphere, determines a field of parallel oriented orthonormal frames along $\h$. A \emph{spin decoration} is roughly a lift of such a frame field to the spin double cover of the frame bundle. This spin construction entails that a rotation of a frame about an angle of $2\pi$ does not result in the same frame, but a rotation of a frame by an angle of $4\pi$ does. Thus, whereas angles are usually measured modulo $2\pi$, after lifting to the spin double cover, angles are measured modulo $4\pi$. In $\hyp^4$ a similar construction applies. Since a decoration on a horosphere $\h$ consists of two perpendicular direction fields, having a decoration, together with a choice of normal, again yields a parallel frame field along $\h$. A choice of decoration is equivalent to a choice of oriented orthonormal frame. The space of horospheres in $\hyp^4$ is diffeomorphic to $S^3 \times \R$, with a choice in $S^3 \cong \partial \hyp^4$ for the centre of the horosphere, and then a choice in $\R$ for its size. The space of decorated horospheres is diffeomorphic to $S^3 \times SO(3) \times \R$. A \emph{spin decoration} in $\hyp^4$, in a similar way to $\hyp^3$, is essentially a lift of a frame field to the spin double cover of the frame bundle. We give the formal definition is \refdef{associated_spin_decorations}. Again, this results in a situation where rotation by $4\pi$ is required to return a frame to itself, so that angles are measured modulo $4\pi$. The space of spin-decorated horospheres is diffeomorphic to $S^3 \times S^3 \times \R$, as is $S\HH$. The proof of \refthm{main_thm_1} gives an explicit diffeomorphism between these spaces. \subsection{Spin isometries of hyperbolic 4-space and M\"{o}bius transformations} \refthm{main_thm_1} includes a statement about equivariance with respect to a group $SL_2\$$. This is a naturally arising group of matrices in the study of M\"{o}bius transformations and 4-dimensional hyperbolic isometries, appearing for example in \cite{Ahlfors_Clifford85, Ahlfors_Mobius85, Ahlfors_Mobius_86, Gongopadhyay_12, Ahlfors_fixedpoints_85, Ahlfors_84, Cao_Waterman_98, Waterman_93, Kellerhals01}. However it is more subtle than $SL_2$ over a commutative ring. We use the following definition. \begin{defn} \label{Def:SL2H} The group $SL_2 \$ $ consists of \emph{Clifford matrices}, which are matrices \[ \begin{pmatrix} a & b \\ c & d \end{pmatrix}, \quad a,b,c,d \in \HH, \] such that the following hold: \begin{gather} \label{Eqn:Vahlen_conditions_1} ab^*, cd^*, c^* a, d^* b, ba^*, dc^*, a^* c, b^* d \in \$\R^3 \\ \label{Eqn:Vahlen_conditions_2} ad^* - bc^* = da^* - cb^* = d^* a - b^* c = a^* d - c^* b = 1 \end{gather} \end{defn} Such matrices go by several other names in the literature; we discuss our notation and terminology in \refsec{Mobius_hyperbolic}. The conditions of \refeqn{Vahlen_conditions_1}--\refeqn{Vahlen_conditions_2} appear onerous but in fact are greatly redundant (and in fact such matrices satisfy many more similar conditions); we have simply included them for convenience and symmetry. For instance, $SL_2\$$ can be defined simply as the set of $2 \times 2$ quaternionic matrices with pseudo-determinant $1$ whose columns are spinors. In \refsec{Clifford_conditions} we discuss various equivalent formulations. Clifford matrices have numerous interesting properties. We discuss some of them in \refsec{Clifford_properties}. In particular, the action of $SL_2\$$ on $\HH^2$ by standard matrix-vector multiplication preserves the 7-real-dimensional subspace $S\HH$. The action on $S\HH$ preserves two of the three summands of its tangent bundle discussed above, and its behaviour on the third summand is rather subtle but has interesting conformal properties, which are related to the equivariance in \refthm{main_thm_1}. We discuss this behaviour in \refsec{SL2_on_spinors} and \refsec{action_SL2_tangent_spinors}. In the complex case, the group $SL_2 \C$ has quotient $PSL_2 \C$ by $\{\pm I\}$, which is is isomorphic to the group of M\"{o}bius transformations of $\C \cup \{\infty\}$: \begin{equation} \label{Eqn:complex_Mobius} \pm \begin{pmatrix} a & b \\ c & d \end{pmatrix} \in PSL_2 \C \quad \text{corresponds to} \quad z \mapsto \frac{az+b}{cz+d}. \end{equation} Via these M\"{o}bius transformations, $PSL_2 \C$ acts on $\C \cup \{\infty\}$, which can be regarded as the boundary at infinity of the upper half space model of $\hyp^3$. Each M\"{o}bius transformation then extends to an orientation-preserving isometry of $\hyp^3$ and in fact there is an isomorphism $PSL_2 \C \cong \Isom^+ \hyp^3$ with the orientation-preserving isometry group of $\hyp^3$. A similar story is known to arise with $SL_2\$$ and 4-dimensional hyperbolic geometry, and has been studied by Ahlfors, Cao, Gongopadhyay, Kellerhals, Waterman and others; further details and references are given in \refsec{Mobius_hyperbolic} and \refsec{paravector_Mobius}. We denote the quotient of $SL_2\$$ by the normal subgroup $\{ \pm I \}$ as $PSL_2\$$. Elements of $PSL_2\$$ can be regarded as M\"{o}bius transformations, although non-commutativity means that the fractional expression in \refeqn{complex_Mobius} no longer makes sense. Instead, we have \begin{equation} \label{Eqn:Mobius_from_SL2H} \pm \begin{pmatrix} a & b \\ c & d \end{pmatrix} \in PSL_2\$ \quad \text{corresponding to} \quad v \mapsto (av+b)(cv+d)^{-1}. \end{equation} When $v$ is a paravector (i.e. $v$ lies in the 3-real-dimensional subspace $\$\R^3 \subset \HH$), $(av+b)(cv+d)^{-1}$ also lies in $\$\R^3$ (or is $\infty$). Thus, $PSL_2\$$ acts on $\$\R^3 \cup \{\infty\}$ by M\"{o}bius transformations with quaternionic coefficients. We can regard $\$\R^3 \cup \{\infty\}$ as the boundary at infinity of the upper half space model of $\hyp^4$. Each M\"{o}bius transformation of $\$\R^3 \cup \{\infty\}$ extends to an orientation-preserving isometry of $\hyp^4$ and there is an isomorphism $PSL_2\$ \cong \Isom^+ \hyp^4$. The group $\Isom^+ \hyp^4$ is diffeomorphic to $\R^4 \times SO(4)$, and its spin double cover, or \emph{spin isometry} group $\Isom^S \hyp^4$, is diffeomorphic to $\R^4 \times \Spin(4)$. There is an isomorphism $SL_2\$ \cong \Isom^S \hyp^4$. In \cite{Mathews_Spinors_horospheres}, we showed that the association of spin-decorated horospheres to complex spinors was $SL_2\C$-equivariant, where $SL_2 \C$ acts on complex spinors by matrix-vector multiplication, and on spin-decorated horospheres by spin isometries of $\hyp^3$. In a similar way here $SL_2\$$ acts on quaternionic spinors by matrix-vector multiplication, and on spin multiflags and spin-decorated horospheres by spin isometries of $\hyp^4$. \refthm{main_thm_1} asserts that these actions are equivariant. \subsection{Explicit description in the upper half space model} When the upper half space model of $\hyp^3$ is used, the spinor--horosphere correspondence has a particularly simple expression. In the 3-dimensional case, with the sphere at infinity $\partial \hyp^3 \cong S^2$ regarded as $\C \cup \{\infty\}$, horospheres appear either as Euclidean spheres tangent to $\C$, or horizontal planes (tangent to $\partial \hyp^3$ at $\infty$). In the former case, the highest point (``north pole") of a horosphere in the model has tangent plane parallel to $\C$, and a decoration can be described by a nonzero complex number, corresponding to the direction at this highest ``north pole" point. In the latter case, any point on the horosphere has tangent plane parallel to $\C$, and again the decoration can be described by a nonzero complex number. It is shown in \cite{Mathews_Spinors_horospheres} that the spinor $(\xi, \eta)$ corresponds to a horosphere centred at $\xi/\eta$. If $\eta \neq 0$ then $\xi/\eta \in \C$, and the horosphere has Euclidean diameter $|\eta|^{-2}$, with decoration described at the north pole by $i \eta^{-2}$. If $\eta = 0$, then $\xi/\eta = \infty$, and the horosphere lies at Euclidean height $|\xi|^2$, with decoration given by $i \xi^2$. These descriptions generalise to the case of quaternions and the upper half space model $\U$ of $\hyp^4$, with the paravectors $\$\R^3$ taking the place of $\C$. Regarding $\partial \hyp^4$ as $\$\R^3 \cup \{\infty\}$, horospheres appear either as Euclidean 3-spheres tangent to $\$\R^3$, or horizontal planes (tangent to $\partial \hyp^4$ at $\infty$). In the former case, at the highest ``north pole" point of a horosphere, the tangent plane is parallel to $\$\R^3$, and in the latter case, any point on the horosphere has tangent plane parallel to $\$\R^3$. Thus parallel direction fields can be described by nonzero elements of $\$\R^3$. \begin{theorem} \label{Thm:main_thm_4} Under the correspondence of \refthm{main_thm_1}, a spinor $(\xi, \eta)$ corresponds to a horosphere $\h$ centred at $\xi \eta^{-1}$ in $\U$. \begin{enumerate} \item If $\eta \neq 0$ then $\h$ appears in $\U$ as a Euclidean 3-sphere with Euclidean diameter $|\eta|^{-2}$, its $i$-direction field is north-pole described by $\eta^{-1*} i \eta^{-1}$, and its $j$-direction field is north-pole described by $\eta^{-1*} j \eta^{-1}$. \item If $\eta = 0$ then $\h$ appears in $\U$ as a Euclidean 3-plane at Euclidean height $|\xi|^2$, with $i$-direction field given by $\xi i \xi^*$ and its $j$-direction field given by $\xi j \xi^*$. \end{enumerate} \end{theorem} This description is illustrated schematically in \reffig{2}. For complex numbers, the $*$-conjugation is trivial, so when $(\xi, \eta)$ is a complex spinor, the $i$-line field points in the direction of the decorations of \cite{Mathews_Spinors_horospheres}; and since any complex number $z$ satisfies $zj = j\overline{z}$, the $j$-direction field simply points in the $j$ direction. We may also observe that, just as for complex numbers, any nonzero quaternion $\eta$ satisfies $\eta^{-1} = \overline{\eta}/|\eta|^2$, so \[ \xi \eta^{-1} = \frac{\xi \overline{\eta}}{|\eta|^2}. \] The condition $\xi \overline{\eta} \in \$\R^3$ in the definition of a quaternionic spinor thus ensures that $\xi \eta^{-1} \in \$\R^3 \cup \{\infty\}$, and hence that the description of the horosphere in \refthm{main_thm_4} makes sense. \begin{figure}[h] \begin{center} \begin{tikzpicture}[scale=1.2] \draw[black] (-2,-0.5)--(2,-0.5)--(3,0.5)--(-1,0.5)--(-2,-0.5); ll[white] (-0.1,0.5) circle (0.5cm); \shade[ball color = red!40, opacity = 0.1] (-0.1,0.5) circle (0.5cm); \draw[blue] (-0.1,0.5) circle (0.5cm); ll[black] (-0.1,0) circle (0.07cm); \draw[red, ->] (-0.1,1)--(-0.3,1.2); \draw[green!50!black, ->] (-0.1,1)--(0.1,1.2); \node[red] at (-1.1,1.2) {$\eta^{-1*} i \eta^{-1}$}; \node[green!50!black] at (1,1.2) {$\eta^{-1*} j \eta^{-1}$}; \node[black] at (-0.1,-0.3) {$\xi \eta^{-1}$}; \draw[<->] (0.8,0)--(0.8,1); ll[white] (0.6,0.3)--(1.4,0.3)--(1.4,0.7)--(0.6,0.7)--cycle; \node[black] at (1,0.5) {$|\eta|^{-2}$}; \draw[blue] (-2,1.5)--(2,1.5)--(3,2.5)--(-1,2.5)--(-2,1.5); \begin{scope}[xshift=0.5cm] \draw[red,->] (-1.1,1.7)--(-1.4,2); \draw[red,->] (-0.4,1.7)--(-1,2.4); \draw[red,->] (0.2,1.7)--(-0.4,2.4); \draw[red,->] (0.8,1.7)--(0.2,2.4); \draw[red,->] (1.2,2)--(0.8,2.4); \draw[green!50!black,->] (-1.1,1.7)--(-0.8,2); \draw[green!50!black,->] (-0.4,1.7)--(0.2,2.4); \draw[green!50!black,->] (0.2,1.7)--(0.8,2.4); \draw[green!50!black,->] (0.8,1.7)--(1.4,2.4); \node[red] at (-1.3,2.3) {$\xi i \xi^*$}; \node[green!50!black] at (1.7,2.3) {$\xi j \xi^*$}; \end{scope} \draw[<->] (2.2,0)--(2.2,2); ll[white] (1.8,0.7)--(2.6,0.7)--(2.6,1.3)--(1.8,1.3)--cycle; \node[black] at (2.2,1) {$|\xi|^2$}; \node[black] at (2,-0.2) {$\$\R^3$}; \end{tikzpicture} \caption{Decorated horospheres as they appear in the upper half space model, with $i$-decoration fields shown in red and $j$-decoration fields in green. This picture is adapted from figure 2 of \cite{Mathews_Spinors_horospheres}.} \label{Fig:2} \end{center} \end{figure} Observe that all the directions given in \refthm{main_thm_4} are of the form $qvq^*$. This operation is well known to be an important one. Indeed, let us define the map $\sigma$ by \begin{equation} \label{Eqn:rho} \sigma(q)(v) = qvq^*, \end{equation} where $q,v \in \HH$. A similar operation has been studied in, e.g., \cite{Ahlfors_Mobius85, Ahlfors_Clifford85, Ahlfors_Lounesto_89, Ahlfors_Mobius_86, Ahlfors_fixedpoints_85}. Any $\sigma(q)$ preserves $\$\R^3$, and when $|q|=1$ is a Euclidean rotation. In fact over the $S^3$ of unit quaternions, $\sigma$ provides the standard description of quaternions as rotations, $S^3 \cong \Spin(3)$. In particular, all the claimed direction fields at north poles or on horizontal horospheres in \refthm{main_thm_4} lie in $\$\R^3$, so the statement makes sense. In general, a nonzero quaternion $q$ can be written in polar form as $q = r e^{u\theta}$, where $r \geq 0$, $\theta \in \R$ is an angle, and $u$ is an imaginary unit quaternion. Then $\sigma(q)$, applied to $\$\R^3$, is a dilation by $r^2$, and rotation by $2 \theta$ about an axis determined by $u$, namely $-uk \in \$\R^3$. See \refprop{rho_rotation_dilation} for details. Extended to $\hyp^4$, $\sigma(q)$ is an isometry that translates by $2 \log r$ and rotates by $2\theta$. \subsection{Quaternionic lambda lengths in hyperbolic 4-space} \label{Sec:intro_lambda} We give a rough idea of quaternionic lambda lengths in 4 dimensions; full details are presented in Sections \ref{Sec:orientations_frames_spin}--\ref{Sec:quaternionic_lambda}. We first describe the 3-dimensional case. Any two horospheres $\h_1, \h_2$ in $\hyp^3$ with spin decorations $W_1, W_2$, have a complex translation distance $d = \rho + i \theta$ from $(\h_1, W_1)$ to $(\h_2, W_2)$ as follows. Let $\rho$ be the oriented distance from $\h_1$ to $\h_2$ along their common perpendicular geodesic $\gamma$. Translating a spin frame of $W_1$ along $\gamma$, by distance $\rho$, takes a frame from a point of $\h_1$ to a point of $\h_2$. Then, rotation by some angle $\theta$ aligns the frame of $W_1$ with that of $W_2$. If $W_1, W_2$ are just decorations (rather than spin-decorations), then $\theta$ is well defined modulo $2\pi$; if $W_1, W_2$ are spin decorations, then $\theta$ is well defined modulo $4\pi$. The lambda length $\lambda_{12}$ from $\h_1$ to $\h_2$ is defined to be \[ \lambda_{12} = e^{d/2} = \exp \left( \frac{\rho + i \theta}{2} \right). \] When dealing with non-spin decorations, $\theta/2$ is only well defined modulo $\pi$, so $\lambda$ is ambiguous up to sign. With spin decorations however $\lambda$ is well defined. \begin{figure}[h] \def\svgwidth{0.38\columnwidth} \begin{center} \input{complex_lambda_lengths.pdf_tex} \caption{Complex distance between horospheres, copied from \cite{Mathews_Spinors_horospheres}.} \label{Fig:3} \end{center} \end{figure} To define the sense of the rotation, one must proceed a little more carefully. A spin decoration is defined to include spin frames of \emph{either} orientation on a horosphere, \emph{inwards} and \emph{outwards}. In order for orientations to match, we translate the \emph{inwards} spin frame of $W_1$ along $\gamma$, and then rotate it to align with the \emph{outwards} spin frame of $W_2$. To describe rotations of frames on horospheres in $\hyp^4$, we use the operation $\sigma$, and identify the tangent spaces of horospheres with $\$\R^3$. Different identifications lead to different numbers in $\$\R^3$ describing the axis of the rotation. However, the $i$- and $j$-direction fields of a decoration on a horosphere $\h$ provide a canonical way to identify the directions of $i$ and $j$ in $T\h$. The remaining vector in an orthonormal frame, of desired orientation, can be identified with $1$. With the three basis vectors of an orthonormal frame thus identified with $i,j$ and $1$, the decoration provides a \emph{paravector identification}, i.e. an identification of each tangent space of $\h$ with $\$\R^3$. A normal direction can then be given by $k$. We describe these matters in detail in \refsec{orientations_frames_spin}. Given two spin-decorated horospheres $(\h_1, W_1)$ and $(\h_2, W_2)$, we then have notions of quaternionic translation distance and lambda length as follows. As in the 3-dimensional case, we have a signed distance $\rho$ from $\h_1$ to $\h_2$ along the oriented geodesic $\gamma$. Using the paravector identification coming from a spin-decoration (as it turns out, it does not matter which decoration we use, $W_1$ or $W_2$; see \reflem{rotation_coordinates_invariant}), translating the inwards spin frame of $W_1$ by $\rho$ along $\gamma$ and then rotating by angle $\theta$ (well defined modulo $4\pi$) about an axis of rotation given by a unit vector $v \in \$\R^3$ (using the paravector identification) aligns it with the outward spin frame of $W_2$. The quaternionic translation distance from $(\h_1, W_1)$ to $(\h_2, W_2)$ is then $d = \rho + \theta vk $; note that as $v \in \$\R^3$, $vk$ is a unit imaginary quaternion. The lambda length $\lambda_{12}$ is then given by \begin{equation} \label{Eqn:lambda_length} \lambda_{12} = e^{d/2} = \exp \left( \frac{\rho + \theta vk}{2} \right). \end{equation} Full details are given in \refsec{spin_decorations_multiflags} and \refsec{quaternionic_lambda}. When the spinors are complex, the corresponding decorated horospheres have $j$-direction fields given by the $j$-direction in the upper half space model, so the rotation of frames from $W_1$ to $W_2$ lie entirely in the real-2-dimensional tangent plane to a horosphere identified with $\C$. Thus the axis of rotation is in the $\pm j$ direction, so we have $v = \pm j$ and $vk = \pm i$. Then $e^{\theta vk/2} = e^{\pm \theta i}$ and the lambda length reduces to the definition of \cite{Mathews_Spinors_horospheres}. Although it can be deduced directly from \refthm{main_thm_2} that $\lambda_{12} = - \lambda_{21}^*$, this can also be proved directly from the geometric definition of lambda length; we prove this in \refprop{lambda_length_antisymmetric}. \subsection{Quasideterminants, Pl\"{u}cker and Ptolemy equations} The theory of determinants of matrices over noncommutative rings like $\HH$ is quite different from the commutative case. The pseudo-determinant of \refeqn{lambda_pdet} is just one among several notions of determinant. A general theory of \emph{quasideterminants} over noncommutative rings was developed by Gel'fand and Retakh in several papers \cite{Gelfand_Retakh_91, Gelfand_Retakh_92, Gelfand_Retakh_97}; see also \cite{Retakh_Wilson_lectures, GGRW_05}. We apply this theory to matrices of quaternions. For general accounts of determinants of quaternionic matrices, though not including the Gel'fand--Retakh theory, see \cite{Aslaksen_96, Zhang_97}. In \cite{Mathews_Spinors_horospheres}, with complex lambda lengths given by $2 \times 2$ complex determinants, the Pl\"{u}cker relation between determinants of $2 \times 2$ submatrices of a $2 \times 4$ matrix yields a Ptolemy equation on lambda lengths. In other words, the Ptolemy equation is a Pl\"{u}cker equation. In the noncommutative case, Gel'fand and Retakh show that there are analogues of Pl\"{u}cker relations. In particular, they define \emph{left quasi-Pl\"{u}cker coordinates} $p_{lm}^I (A)$ of a matrix $A$ over a noncommutative ring, and prove they satisfy certain relations. We show in \reflem{quasi-Plucker_lambda} that when $A$ is a quaternionic matrix with spinor columns $\kappa_l$ we obtain \begin{equation} \label{Eqn:quasi-Plucker_lambda} p_{lm}^n (A) = \lambda_{nl}^{-1} \lambda_{nm}. \end{equation} After proving \refthm{main_thm_2} and \refeqn{quasi-Plucker_lambda}, the Gel'fand--Retakh Pl\"{u}cker relation yields \refthm{main_thm_3} immediately. We show this in \refsec{Plucker}. Gel'fand--Retakh also prove a certain \emph{skew-symmetry} property of quasi-Pl\"{u}cker coordinates, namely certain triples of them multiply to $-1$. This yields the following result on lambda lengths, proved in \refsec{Plucker}. \begin{prop} \label{Prop:triangle_holonomy} Given any 3 spin-decorated horospheres $(\h_n, W_n)$ in $\hyp^4$, $n=1,2,3$, let $\lambda_{mn}$ denote the lambda length from $(\h_m, W_m)$ to $(\h_n, W_n)$. Then \[ \lambda_{12} \lambda_{32}^{-1} \lambda_{31} \in \$\R^3 \cup \{\infty\}. \] \end{prop} Berenstein--Retakh in \cite{Berenstein_Retakh_18, Retakh_OW_report_13} discussed related quantities as \emph{noncommutative angles}, satisfying \emph{triangle relations} which are similar to the above. A possible interpretation of \refthm{main_thm_3} is that it provides information about ``lambda length holonomy" around paths between horospheres. We can regard $\lambda_{mn}$ as describing the holonomy involved in translating and rotating a spin frame along the oriented geodesic $\h_m \rightarrow \h_n$, and $\lambda_{mn}^{-1}$ referring to the holonomy backwards from $\h_n$ to $\h_m$ along the oriented geodesic $\h_m \to \h_n$. Because of the alternating inwards and outwards frames, the holonomy moves alternately forwards and backwards along edges, in a manner reminiscent of the oscillating curves of the first author and Purcell \cite{Mathews_Purcell_Ptolemy_hyperbolic}. After rewriting \refeqn{noncomm_Plucker} as \[ \lambda_{02} = \lambda_{01} \lambda_{31}^{-1} \lambda_{32} + \lambda_{03} \lambda_{13}^{-1} \lambda_{12}, \] \refthm{main_thm_3} can be interpreted as describing the holonomy from $\h_0 \to \h_2$ in terms of the holonomy detouring to $\h_1$ and $\h_3$ along the way; and \refprop{triangle_holonomy} describes the holonomy around a triangle. \[ \begin{tikzpicture}[scale=0.9] ll[black] (0,0) circle (0.1cm); \node[black] at (-0.5,0) {$\h_0$}; ll[black] (1,1) circle (0cm); ll[black] (1,-1) circle (0cm); ll[black] (2,0) circle (0.1cm); \node[black] at (2.6,0) {$\h_2$}; \draw[black, -latex] (0,0) -- (1,0); \draw[black] (1,0)--(2,0); \node[black] at (1,0.5) {$\lambda_{02}$}; \draw[ultra thick, green!50!black, -latex, rounded corners] (0.2,-0.2) -- (1.8,-0.2); \node[black] at (3,0) {$=$}; \end{tikzpicture} \begin{tikzpicture}[scale=0.9] ll[black] (0,0) circle (0.1cm); \node[black] at (-0.5,0) {$\h_0$}; ll[black] (1,1) circle (0.1cm); \node[black] at (1.5,1) {$\h_3$}; ll[black] (1,-1) circle (0.1cm); \node[black] at (0.5,-1) {$\h_1$}; ll[black] (2,0) circle (0.1cm); \node[black] at (2.6,0) {$\h_2$}; \draw[black, -latex] (0,0) -- (0.5,0.5); \draw[black] (0,0)--(1,1); \node[black] at (0,0.7) {$\lambda_{03}$}; \draw[black, -latex] (1,-1)--(1,0); \draw[black] (1,0)--(1,1); \node[black] at (1.4,0.2) {$\lambda_{13}$}; \draw[black, -latex] (1,-1) -- (1.5,-0.5); \draw[black] (1.5,-0.5)--(2,0); \node[black] at (1.8,-0.7) {$\lambda_{12}$}; \draw[ultra thick, green!50!black, -latex, rounded corners] (0.3,0.1) -- (0.9,0.7) -- (0.9,-0.7) -- (1.1,-0.7) -- (1.7,-0.1); \node[black] at (3,0) {$+$}; \end{tikzpicture} \begin{tikzpicture}[scale=0.9] ll[black] (0,0) circle (0.1cm); \node[black] at (-0.5,0) {$\h_0$}; ll[black] (1,1) circle (0.1cm); \node[black] at (0.5,1) {$\h_3$}; ll[black] (1,-1) circle (0.1cm); \node[black] at (1.5,-1) {$\h_1$}; ll[black] (2,0) circle (0.1cm); \node[black] at (2.6,0) {$\h_2$}; \draw[black, -latex] (1,1) -- (1.5,0.5); \draw[black] (1.5,0.5)--(2,0); \node[black] at (1.8,0.7) {$\lambda_{32}$}; \draw[black, -latex] (1,1)--(1,0); \draw[black] (1,0)--(1,-1); \node[black] at (1.4,-0.2) {$\lambda_{31}$}; \draw[black, -latex] (0,0) -- (0.5,-0.5); \draw[black] (0.5,-0.5)--(1,-1); \node[black] at (0,-0.7) {$\lambda_{01}$}; \draw[ultra thick, green!50!black, -latex, rounded corners] (0.3,-0.1) -- (0.9,-0.7) -- (0.9,0.7) -- (1.1,0.7) -- (1.7,0.1); \end{tikzpicture}, \quad \begin{tikzpicture}[scale=0.9] \node[black] at (-2,0) {and}; ll[black] (0,0) circle (0.1cm); \node[black] at (-0.5,0) {$\h_1$}; ll[black] (1,1) circle (0.1cm); \node[black] at (1.5,1) {$\h_2$}; ll[black] (1,-1) circle (0.1cm); \node[black] at (1.5,-1) {$\h_3$}; \draw[black, -latex] (0,0) -- (0.5,0.5); \draw[black] (0,0)--(1,1); \node[black] at (0,0.7) {$\lambda_{12}$}; \draw[black, -latex] (1,-1)--(1,0); \draw[black] (1,0)--(1,1); \node[black] at (1.6,0) {$\lambda_{32}$}; \draw[black, -latex] (1,-1) -- (0.5,-0.5); \draw[black] (0.5,-0.5)--(0,0); \node[black] at (0,-0.7) {$\lambda_{31}$}; \draw[ultra thick, green!50!black, -latex, rounded corners] (0.3,0.1) -- (0.9,0.7) -- (0.9,-0.7) -- (0.3,-0.1); \node[black] at (2.5 ,0) {$\in \$\R^3$}; \end{tikzpicture} . \] \subsection{Structure and approach of this paper} This paper, roughly speaking, uses three quite distinct bodies of previous work: work of Penner, Penrose, Rindler, and the first author on complex spinors, low-dimensional hyperbolic geometry, and lambda lengths; work of Ahlfors, Cao, Kellerhals, Lounesto, Maass, and Vahlen on M\"{o}bius transformations, Clifford matrices, and 4-dimensional hyperbolic isometries; and work of Berenstein, Gel'fand and Retakh on noncommutative determinants. We do not imagine that readers familiar with all these topics are particularly common. Therefore, we have tried to make this paper as self-contained as possible. We give background and references on all these subjects as we proceed. We also need to develop some necessary background for our main theorems, particularly regarding quaternion paravectors, Clifford matrices and 4-dimensional hyperbolic isometries. For instance, we need an explicit description of paravector rotations on quaternions for our notion of quaternionic distance between horospheres; and we need some facts about parabolic translation matrices not to our knowledge in the literature. Therefore, we proceed by establishing the background for our theorems step by step. The basic structure of the proof of the main \refthm{main_thm_1} is the same as for the corresponding theorem in \cite{Mathews_Spinors_horospheres}: we construct the maps of the following commutative diagram. \begin{equation} \label{Eqn:main_thm_diagram} \begin{array}{ccccc} & & \widetilde{MF} & \stackrel{\widetilde{\Phi_2}}{\To} & \Hor^S \\ & \stackrel{\widetilde{\Phi_1}}{\nearrow} & \downarrow && \downarrow \\ S\HH & \stackrel{\Phi_1}{\To} & \MF \cong \S^{+D} & \stackrel{\Phi_2}{\To} & \Hor^D \\ & \stackrel{\phi_1}{\searrow} & \downarrow & & \downarrow \\ & & L^+ & \stackrel{\phi_2}{\To} & \Hor \end{array} \end{equation} Here $S\HH$ is the space of spinors; $\widetilde{\MF} \To \MF$ is the double cover from spin multiflags to multiflags; $\Hor^S \To \Hor^D$ is the double cover from spin-decorated horospheres to decorated horospheres; $\Hor$ is the space of horospheres; $L^+$ is the future light cone; and $\S^{+D}$ is the space of decorated ideal points. The remaining downward maps are forgetful, and the maps $\widetilde{\Phi_1}, \widetilde{\Phi_2}$ are the correspondences of \refthm{main_thm_1}. In \refsec{Clifford_geometry}, we consider Clifford algebras in general, for motivation and context, before specialising to the quaternions. We begin with background on Clifford algebras in general (\refsec{Clifford_general}), including M\"{o}bius transformations, Lipschitz groups, and hyperbolic isometries in general dimension (\refsec{Mobius_hyperbolic}). In order to use quaternions for 4-dimensional hyperbolic geometry, we use the approach to M\"{o}bius transformations from paravectors, which we introduce in \refsec{vectors_paravectors}, and discuss how M\"{o}bius transformations, Lipschitz groups, and hyperbolic isometries arise in this approach (\refsec{paravector_Mobius}). We briefly consider complex numbers as a simple special case (\refsec{complex}), then proceed to consider quaternions. We discuss their basic properties (\refsec{quaternion_involution}), their paravectors and Lipschitz group (\refsec{quaternion_para}). The standard relationship of quaternions to 3-dimensional geometry appears a little differently when we take the paravector approach, which we recount, including dot and cross product (\refsec{dot_cross_paravector}), rotations (\refsec{para_rotation}), and the operation $\sigma$ of \refeqn{rho} (\refsec{actions_on_paravectors}). We suspect none of this was unknown to Ahlfors or Lounesto, though we could not find some of the explicit descriptions in the literature. In \refsec{spinors} we turn to pairs of quaternions, and spinors. Much of this is, in some sense, new, but many of the technical properties of the spinors involved were already known from the study of Clifford matrices. We first introduce a standard inner product on $\HH^2$ (\refsec{inner_product_norm_H2}), then discuss various equivalent ways of defining spinors (\refsec{spinor_conditions}). We introduce the bracket $\{ \cdot, \cdot \}$ and some of its properties in \refsec{bracket}, which was previously studied to some extent as the quasideterminant. We introduce certain ``complementary" spinors, which arise naturally in the tangent space to spinors, in \refsec{complementary}; these are in a certain sense analogous to a K\"{a}hler structure. We then consider the space of spinors globally (\refsec{space_of_spinors}), including its tangent space and its subspace of paravectors (\refsec{paravectors_in_TSH}). We then turn to linear maps on spinors, which are given by the Clifford matrices, a well-studied topic. We discuss various known equivalent formulations of Clifford matrices in \refsec{Clifford_conditions}, and some of their properties in \refsec{Clifford_properties}. We develop required notions of parabolic Clifford matrices in \refsec{parabolic_clifford}. We then consider the action of Clifford matrices on spinors in \refsec{SL2_on_spinors}, and its derivative in \refsec{action_SL2_tangent_spinors}. We can then turn in \refsec{spinor_to_flag} to the construction of the map $\Phi_1$ of the first correspondence of \refthm{main_thm_1}, from spinors to multiflags. We introduce paravector Hermitian matrices and their relationship to Minkowski space in \refsec{Hermitian_Minkowski}, and discuss orientations on the various spaces involved in \refsec{orientations}. We construct the map $\phi_1$ from spinors to the light cone in \refsec{spinors_to_light_cone}, and then consider its derivative in \refsec{deriv_phi1}, as required to obtain flags. We consider the conformality of this derivative in \refsec{conformal_paravector}, and define flags (\refsec{flags}) and multiflags, so that we can construct $\Phi_1$ (\refsec{multiflags}). We use conformality to define and discuss decorated ideal points (\refsec{decorated_ideal_points}), and finally discuss the equivariance of the map in \refsec{SL2_on_paravectors_etc}. In \refsec{Minkowski_horospheres} we turn to hyperbolic geometry, constructing the map $\Phi_2$ of the second correspondence of \refthm{main_thm_1}, from multiflags to decorated horospheres. We establish background on 4-dimensional hyperbolic geometry in \refsec{hyp_geom_general} and then study horospheres in \refsec{horospheres_geometry}, including their isometries and relationship to parabolic matrices, and the map $\phi_2$ from the light cone to horospheres. Then in \refsec{multiflags_to_horospheres} we can define decorated horospheres, and the map $\Phi_2$. In \refsec{H4_models} we then proceed from the hyperboloid to the upper half space model, establishing the explicit description of \refthm{main_thm_4}. In \refsec{spin_decorations} we introduce spin geometry, constructing the lifts $\widetilde{\Phi_1}$ and $\widetilde{\Phi_2}$ and proving the correspondences of \refthm{main_thm_1}. In \refsec{orientations_frames_spin} we discuss the frame fields, orientations and paravector identifications associated to a decorated horosphere. In \refsec{spin_decorations_multiflags} we can define spin decorations on horospheres, the maps $\widetilde{\Phi_1}$ and $\widetilde{\Phi_2}$, and prove \refthm{main_thm_1}. In \refsec{quaternionic_lambda} we introduce quaternionic distances and define lambda lengths. In \refsec{antisym} we prove the antisymmetry result $\lambda_{12} = -\lambda_{21}^*$ geometrically. In \refsec{lambda_lengths} we show that the pseudo-determinant gives lambda length, proving \refthm{main_thm_2}. Finally, in \refsec{Ptolemy} we prove the Ptolemy equation of \refthm{main_thm_3}. In \refsec{quasidet_Plucker} we discuss Gel'fand--Retakh quasideterminants and quasi-Pl\"{u}cker coordinates, and in \refsec{Plucker} use their Pl\"{u}cker relation to prove \refthm{main_thm_3}. \subsection{Remarks on notation and terminology} In this paper, several standard conventions for notation clash, and we mitigate these issues as best we can. We discuss these issues as they arise, but the following comments apply throughout. The letter H is standard to describe many of the objects discussed. We use $\h$ for horospheres, $\HH$ for Hamilton's quaternions, $\mathbf{H}$ for Hermitian matrices, and $\hyp$ for hyperbolic space. We use $\hyp^4$ for 4-dimensional hyperbolic space in general and, when our considerations are model-dependent, for the hyperboloid model. We denote the conformal disc model by $\Disc$ and the upper half space model by $\U$. The letters $i,j,k$ denote quaternions, but are also used commonly for indices. We thus use $l,m,n, \ldots$ for indices instead. \subsection{Acknowledgments} The first author is supported by Australian Research Council grant DP210103136. \section{Geometry of Clifford algebras, quaternions, and paravectors} \label{Sec:Clifford_geometry} In this section we introduce the geometric aspects of quaternions as we need them. Much of this is well known and we simply recall results; however formulations involving paravectors may be less well known. \subsection{Clifford algebras in general} \label{Sec:Clifford_general} Following the usual definition as in e.g. Lounesto \cite[ch. 14]{Lounesto_Clifford_book_01}, given a real vector space $V$ endowed with a nondegenerate quadratic form $Q$, we define the \emph{Clifford algebra} $\Cl(V,Q)$ to be the associative algebra generated by distinct subspaces of scalars $\R$ and vectors $V$ such that for all $v \in V$ we have $v^2 = Q(v)$. For present purposes, we always have $V = \R^n$. When $Q$ has signature $p,q$ with $p+q=n$, we write $\R^{p,q}$ instead of $(V,Q)$, and may take a basis $e_1, \ldots, e_n$ such that $e_m^2 = 1$ for $1 \leq m \leq p$, $e_m^2 = -1$ for $p+1 \leq m \leq n$, and $e_l e_m = - e_m e_l$ for $1 \leq l, m \leq n$. When $q=0$ we simply write $\R^n$ rather than $\R^{n,0}$. As a real vector space, $\Cl(\R^{p,q})$ has dimension $2^n = 2^{p+q}$. A basis is given by products of distinct $e_i$ (including the empty product), i.e. $e_{m_1} \ldots e_{m_l}$ where $q \leq m_1 < \cdots < m_l \leq n$. Such a product $e_{m_1} \cdots e_{m_l}$ has \emph{degree} $l$, and the linear combinations of such products are the elements of $\Cl(\R^{p,q})$ of \emph{degree} $l$. The real vector subspace of $\Cl(\R^{p,q})$ of elements of degree $l$ has dimension $\binom{n}{l}$. It follows from $v^2 = Q(v)$ that for any $v,w \in \R^{p,q}$ we have $vw+wv = 2 \langle v,w \rangle$, where $\langle v,w \rangle = \frac{1}{2} \left( Q(v+w) - Q(v) - Q(w) \right)$ is the symmetric nondegenerate bilinear form induced by the quadratic form $Q$. We then have $\langle v,v \rangle = Q(v)$. A Clifford algebra $\Cl(\R^{p,q})$ naturally has three conjugation involutions. Several conventions exist for the notation but we follow numerous authors in the following \cite{Ahlfors_Mobius85, Ahlfors_Clifford85, Ahlfors_Lounesto_89, Ahlfors_Mobius_86, Ahlfors_84, Ahlfors_fixedpoints_85, Cao_Waterman_98, Gongopadhyay_12, Kellerhals01, Wada_90, Waterman_93}. The homomorphism $q \mapsto q'$ is induced by $v \mapsto -v$ for $v \in V$; the anti-homomorphism $q\mapsto q^*$ is induced by reversing the words $e_{m_1} \cdots e_{m_l}$ forming a basis of $\Cl(\R^{p,q})$ as a vector space; and the anti-homomorphism $q \mapsto \bar{q}$ is given by the composition of the previous two conjugations, $\bar{q} = q'^*$. An invertible vector $v \in \R^{p,q}$ acts on $\Cl(\R^{p,q})$ by conjugation, $x \mapsto vxv^{-1}$. If $x$ is a vector, i.e. $x \in \R^{p,q}$, then $vxv^{-1} \in \R^{p,q}$ also, and in fact $-vxv^{-1}$ is the reflection of $x$ in the plane orthogonal to $v$ in $\R^{p,q}$ with respect to the inner product $\langle \cdot, \cdot \rangle$. Thus, the action of $v$ is by negative reflections. The actions of $v$ and any nonzero scalar multiple of $v$ on $V$ are identical, so to obtain all reflections it suffices to consider $v$ such that $Q(v) = \pm 1$, i.e. $v^2 = \pm 1$. Following Lounesto \cite{Lounesto_Clifford_book_01}, we define the \emph{Lipschitz group} $\Gamma_{p,q}$ of $\Cl(\R^{p,q})$ to be the multiplicative subgroup of $\Cl(\R^{p,q})$ generated by invertible vectors. In the literature this group is often called the \emph{Clifford group} \cite{Ahlfors_Clifford85, Ahlfors_Lounesto_89, Ahlfors_Mobius85, Ahlfors_Mobius_86, Ahlfors_84, Ahlfors_fixedpoints_85, Kellerhals01, Cao_Waterman_98}. According to Ahlfors and Lounesto \cite{Ahlfors_Lounesto_89}, this terminology goes back to Chevalley \cite{Chevalley_54}, but the notion goes back to Lipschitz \cite{Lipschitz_1886}, and we prefer Lounesto's terminology, to avoid potential confusion with Clifford algebras and the group of Clifford matrices. Again when $q=0$ we simply write $\Gamma_n$ rather than $\Gamma_{n,0}$. Although an element of $\Gamma_{p,q}$ may not have a well-defined integer degree, it is is either a linear combination of even-degree elements of odd-degree elements and thus has a well defined parity. Each element of $\Gamma_{p,q}$ acts on $\R^{p,q}$ by conjugation, where it acts as a composition of negative reflections. The subgroup of $\Gamma_{p,q}$ of even-degree elements is denoted $\Gamma^+_{p,q}$ and acts on $\R^{p,q}$ by rotations. The subgroup of $\Gamma_{p,q}$ generated by invertible vectors $v$ such that $v^2 = \pm 1$ forms the group $\Pin(p,q)$, and the subgroup of $\Gamma_{p,q}$ generated by invertible vectors $v$ such that $v^2 = 1$ forms the group $\Pin^+ (p,q)$. The subgroup of $\Pin(p,q)$ (resp. $\Pin_+ (p,q)$) of even degree elements (i.e. $\Pin(p,q) \cap \Gamma^+_{p,q}$, resp. $\Pin^+ (p,q) \cap \Gamma^+_{p,q}$) forms the group $\Spin(p,q)$ (resp. $\Spin^+ (p,q)$). The groups $\Pin(p,q)$, $\Spin(p,q)$ and $\Spin^+ (p,q)$ are the spin double covers of $O(p,q)$, $SO(p,q)$ and $SO^+ (p,q)$ respectively. See e.g. \cite[ch. 17.2]{Lounesto_Clifford_book_01} for further details. \subsection{M\"{o}bius transformations and hyperbolic isometries} \label{Sec:Mobius_hyperbolic} M\"{o}bius transformations can be obtained from Clifford algebras in two distinct ways; see e.g. \cite{Ahlfors_Lounesto_89, Ahlfors_84} or \cite[ch. 19]{Lounesto_Clifford_book_01}. We first recount the more straightforward method, before discussing the second method in \refsec{paravector_Mobius}, which we rely on in the sequel. We follow the general approach of Ahlfors in several papers \cite{Ahlfors_Clifford85, Ahlfors_Mobius85, Ahlfors_Mobius_86, Ahlfors_84, Ahlfors_fixedpoints_85}, which in turn goes back to work of Maass from 1949 \cite{Maass_49}, Fueter from 1926\cite{Fueter_1926}, and Vahlen from 1902 \cite{Vahlen_1902}. As discussed by Lounesto in \cite[sec. 19.2]{Lounesto_Clifford_book_01}, we can regard the upper half space model $\U^n$ of $\hyp^n$ as the set of $x_1 e_1 + \cdots + x_{n} e_{n}$ in $\R^n$, where $e_1, \ldots, e_n$ are standard basis vectors, all $x_1, \ldots, x_n \in \R$ and $x_n > 0$. Forming the Clifford algebra $\Cl(\R^{n})$ of $\R^{n}$ with positive definite quadratic form, and Lipschitz group $\Gamma_n$, we may consider certain matrices \[ \begin{pmatrix} a & b \\ c & d \end{pmatrix} \quad \text{such that} \quad a,b,c,d \in \Gamma_n \cup \{0\}, \quad a b^*, b^* d, d c^*, c^* a \in \R^n, \quad ad^* - bc^* = \pm 1. \] These matrices form a group; they are designed to act on $\Cl(\R^n) \cup \{\infty\}$ by M\"{o}bius transformations, preserving the subspace $\R^n \cup \{\infty\}$, by \[ \begin{pmatrix} a & b \\ c & d \end{pmatrix} \cdot x = (ax+b)(cx+d)^{-1}. \] In fact, this group is a 4-fold cover of the group of M\"{o}bius transformations of $\R^n \cup \{\infty\}$. Following Ahlfors and others, we call these matrices \emph{Clifford matrices} \cite{Ahlfors_Clifford85, Ahlfors_Mobius85, Ahlfors_Mobius_86, Ahlfors_fixedpoints_85, Kellerhals01, Waterman_93, Cao_Waterman_98, Wada_90} and, following \cite{Ahlfors_Clifford85, Ahlfors_Mobius85, Ahlfors_Mobius_86}, we denote the group by $SL_2 \Gamma_n$. These matrices can be traced back to the work of Maass \cite{Maass_49} and Vahlen \cite{Vahlen_1902}, and are also variously known as $SL(2,C_n)$ \cite{Kellerhals01, Waterman_93, Cao_Waterman_98}, \emph{Vahlen matrices} \cite{Lounesto_Clifford_book_01}, $SL(2,\HH)$ \cite{Gongopadhyay_12}, and $\mathcal{C}^n$ \cite{Wada_90}. Ahlfors in \cite[sec. 3.3]{Ahlfors_Mobius_86} suggests the notation $SL_2 (\Gamma_n \cup \{0\})$ would be better but is needlessly cumbersome. By the same argument down a dimension, $SL_2 \Gamma_{n-1}$ acts on $\Cl(\R^{n-1})$ preserving $\R^{n-1} \cup \{\infty\}$. Regarding $\R^{n-1} \subset \R^{n}$ as the subspace spanned by $e_1, \ldots, e_{n-1}$, and $\Gamma_{n-1} \subset \Gamma_{n}$ then $SL_2 \Gamma_{n-1}$ acts on $\Cl(\R^{n})$ preserving $\R^{n-1} \cup \{\infty\}$ and $\R^n \cup \{\infty\}$. Hence, being orientation preserving, the action of $SL_2 \Gamma_{n-1}$ on $\R^n \cup \{\infty\}$ preserves the upper half space $\hyp^n$ consisting of $x_1 e_1 + \cdots + x_{n-1} e_{n-1} + x_{n} e_{n}$ where $x_1, \ldots, x_{n} \in \R$ and $x_{n} > 0$. In this way, $SL_2 \Gamma_{n-1}$ yields a 4-fold cover of the group of orientation-preserving M\"{o}bius transformations of $\R^{n-1} \cup \{\infty\}$, which is also the group $\Isom^+ \hyp^n$ of orientation-preserving isometries of $\hyp^n$. In particular, $\hyp^4$ has orientation-preserving isometry group 4-fold covered by $SL_2 \Gamma_3$, where $\Gamma_3$ is the Lipschitz group of $\Cl(\R^3)$. \subsection{Vectors and paravectors in Clifford algebras} \label{Sec:vectors_paravectors} Following Lounesto \cite[sec. 19.3]{Lounesto_Clifford_book_01}, we define a \emph{paravector} in a Clifford algebra $\Cl(V,Q)$ to be a real linear combination of a scalar and a vector, i.e. an element of $\R \oplus V \subset \Cl(V,Q)$. In the literature these elements are often called ``vectors", with the elements of $V$ called ``pure vectors" (e.g. \cite{Cao_Waterman_98, Ahlfors_Clifford85, Ahlfors_Mobius85, Ahlfors_Mobius_86, Ahlfors_84, Ahlfors_fixedpoints_85, Kellerhals01}), but we prefer to use the word ``vector" in its natural meaning, as the elements of the vector space $V$. Following Lounesto's general approach, we notate objects using paravectors with a $ \$ $ sign. In $\Cl(\R^{p,q})$ then a paravector is an element of the form $x + y$ where $x \in \R$ and $y \in \R^{p,q}$. We temporarily denote by $\$V$ the vector subspace of paravectors in $\Cl(\R^{p,q})$, so $\$ V = \R \oplus \R^{p,q}$. The approach of the present paper, and indeed much of the work on Clifford algebras and M\"{o}bius transformations, relies on the fact that, roughly speaking, \emph{paravectors in $\R^{p,q}$ behave like vectors in $\R^{q+1,p}$}. We now briefly explain why, and refer to Lounesto \cite[ch. 19]{Lounesto_Clifford_book_01} for further details. Given a paravector $v \in \$ V$, we can write $v = a+b+c$ where $a \in \R$ is a scalar, $b \in \R^{p,0}$ is a positive definite vector, and $c \in \R^{0,q}$ is a negative definite vector, so that $a^2, b^2 \geq 0$, $c^2 \leq 0$, and $bc+cb = 0$. Then defining $\overline{v} = a-(b+c)$, we have \[ v \overline{v} = (a+b+c)(a-b-c) = a^2 - b^2 - c^2 \in \R. \] Since $a^2 \geq 0$, $-b^2 \leq 0$, and $-c^2 \geq 0$, the $a,b,c$ components of $v$ have respectively become positive, negative and positive definite. Thus $\$ V$ naturally has a quadratic form $\$ Q(v) \colon \$ V \To \R$, given by $\$ Q(v) = v \overline{v}$, of signature $(q+1,p)$, and we have an isomorphism of vector spaces with quadratic forms $\$ V \cong \R^{q+1,p}$. We regard this $\$ V$ as the ``paravector version of $\R^{q+1,p}$, and thus write $\$ V$ as $\$ \R^{q+1,p}$. When $p=0$ we simply write $\$\R^{q+1}$ rather than $\$\R^{q+1,0}$. This means that the paravector version $\$\R^{p,q}$ of $\R^{p,q}$ is the set of paravectors in $\Cl(\R^{q,p-1})$. A paravector $v$ is invertible if and only if $\$Q (v) = v \overline{v} \neq 0$, in which case $v^{-1} = \overline{v} / (v \overline{v})$. The quadratic form $\$ Q$ on $\$\R^{q+1,p}$ induces a symmetric nondegenerate bilinear form, also of signature $(q+1,p)$, given by \[ \langle \cdot, \cdot \rangle_{\$} \colon \$ \R^{q+1,p} \times \$ \R^{q+1,p} \To \R, \quad \langle v, w \rangle_\$ = \frac{\$ Q(v+w) - \$ Q (v) - \$ Q (w)}{2} = \frac{v \overline{w} + w \overline{v}}{2}. \] \subsection{M\"{o}bius transformations from paravectors} \label{Sec:paravector_Mobius} The approach to M\"{o}bius transformations based on paravectors essentially replaces the vector space $\R^{p,q}$ with the paravector space $\$\R^{p,q}$. This is the approach used, for instance, by Ahlfors \cite{Ahlfors_Clifford85, Ahlfors_Mobius85, Ahlfors_Mobius_86, Ahlfors_84, Ahlfors_fixedpoints_85}, Cao--Waterman \cite{Cao_Waterman_98}, Gongopadhyay \cite{Gongopadhyay_12}, Kellerhals \cite{Kellerhals01} and Waterman \cite{Waterman_93}. We refer to those papers, or Lounesto's book \cite[ch. 19]{Lounesto_Clifford_book_01} for further details. Just as vectors in $\R^{p,q}$ have a geometric action on $\Cl(\R^{p,q})$ preserving $\R^{p,q}$, paravectors $v \in \$ \R^{q+1,p} \subset \Cl(\R^{p,q})$ also have nice geometric actions on $\Cl(\R^{p,q})$, which preserve $\$ \R^{q+1,p}$. One such action is $x \mapsto vx\overline{v}^{\, -1} = vxv/(v\overline{v})$, which preserves $\$\R^{q+1,p}$. If $v \in \R$ then this action is trivial, and otherwise one can show that this action is a rotation in the 2-plane spanned by $1$ and $v$ \cite{Ahlfors_Mobius85}; we show this explicitly for the quaternions below. Another such action is $x \mapsto vxv$, which is the same action, multiplied by $v\overline{v} = \$ Q(v)$. Thus it is another action of $\$\R^{q+1,p}$ on $\Cl(\R^{p,q})$ preserving $\$\R^{q+1,p}$, on which it acts as a rotation in the 2-plane spanned by $1$ and $v$, composed with a dilation of $\$\R^{q+1,p}$ by $\$Q(v)$. We can then define the \emph{paravector Lipschitz group} $\$\Gamma_{q+1,p}$ to be the multiplicative group generated by invertible paravectors in $\$\R^{q+1,p} \subset \Cl(\R^{p,q})$. The notation is also due to Lounesto \cite[sec. 19.3]{Lounesto_Clifford_book_01} (though he does not give the group a name); he attributes the idea to Porteous' 1969 work \cite{Porteous_69_81}. Again this group also often goes by the name of \emph{Clifford group}, and the same reasoning applies for our terminology. The conjugation $v \mapsto \overline{v}$ on paravectors $\$\R^{q+1,p}$ agrees with the conjugation $x \mapsto \overline{x}$ from $\Cl(\R^{p,q})$, so we obtain an anti-homomorphic involution $v \mapsto \overline{v}$ on $\$\Gamma_{q+1,p}$ such that $0 \neq v \overline{v} \in \R$ and $v^{-1} = \overline{v}/(v\overline{v})$ for all $v \in \$\Gamma_{q+1,p}$. The action of $\$\R^{q+1,p}$ on $\$\R^{q+1,p}$ by $x \mapsto vx\overline{v}^{-1}$ extends to an action of $\$\Gamma_{q+1,p}$ on $\$\R^{q+1,p}$ by orientation-preserving isometries: for $v \in \$\Gamma_{q+1,p}$, given as $v = v_1 \cdots v_m$ where each $v_k \in \$\R^{q+1,p}$ is invertible, and $x \in \$\R^{q+1,p}$ we have \begin{align*} x &\mapsto v_1 \cdots v_m \; x \; \overline{v_m}^{\,-1} \, \cdots \, \overline{v_1}^{\, -1} = v x (v')^{-1} = \frac{ vxv^* }{|v|^2} \end{align*} since $v^* = v_m \cdots v_1$ so that $\overline{v^*} = \overline{(v_m \cdots v_1)} = \overline{v_1} \; \overline{v_2} \; \cdots \; \overline{v_m}$ and $(\overline{v^*})^{-1} = (v')^{-1} = \overline{v_m}^{\,-1}\cdots\overline{v_1}^{\,-1}$. Similarly, the action $x \mapsto vxv$ of $\$\R^{q+1,p}$ on $\$\R^{q+1,p}$ extends to an action of $v \in \$\Gamma_{q+1,p}$ on $x \in \$\R^{q+1,p}$ given by $x \mapsto vxv^*$, which is an orientation-preserving isometry of $\$\R^{q+1,p}$ composed with a dilation by $v\overline{v}$. When $p=0$, i.e. $q=n$, and we begin from a negative definite quadratic form and the Clifford algebra $\Cl(\R^{0,n})$, then the paravectors have signature $(n+1,0)$ and so are written as $\$\R^{n+1}$. The paravector Lipschitz group of $\Cl(\R^{0,n})$ is $\$\Gamma_{n+1,0}$, which we simply write as $\$\Gamma_{n+1}$. Thus $\$\Gamma_n$ is the paravector Lipschitz group of $\Cl(\R^{0,n-1})$. In fact $\$\Gamma_{n} \cong \Gamma_{n}^+$ \cite[sec. 19.3]{Lounesto_Clifford_book_01}. There is a corresponding set of Clifford matrices, given (e.g. \cite{Lounesto_Clifford_book_01, Vahlen_1902}) by \begin{equation} \label{Eqn:Vahlen_general} \begin{pmatrix} a & b \\ c & d \end{pmatrix}, \quad a,b,c,d \in \$\Gamma_n \cup \{0\}, \quad \overline{a} \, b, \; b \, \overline{d}, \; \overline{d} \, c, \; c \, \overline{a} \in \$\R^n, \quad a d^* - b c^* = 1. \end{equation} These matrices form a group, which we denote $SL_2 \$\Gamma_n$. Again numerous other terminologies exist; our choice of terminology and notation is as in \refsec{Mobius_hyperbolic}, with the addition of a $\$$ for paravectors. Numerous other equivalent definitions of the group are possible and exist in the literature; we discuss these in \refsec{Clifford_conditions}. But we observe that the defining condition of spinors $\xi \overline{\eta} \in \$\R^3$ arises naturally. The Clifford matrices $SL_2 \$\Gamma_n$ act on $\Cl(\R^{0,n-1}) \cup \{\infty\}$ preserving the paravectors $\$\R^n \cup \{\infty\}$, by \[ \begin{pmatrix} a & b \\ c & d \end{pmatrix} \cdot v = (av+b)(cv+d)^{-1} \quad \text{for } v \in \$\R^n \cup \{\infty\}. \] The group $SL_2 \$ \Gamma_n$ forms a 2-fold cover of the group of M\"{o}bius transformations of $\$\R^n \cup \{\infty\}$. Similarly, $SL_2 \$ \Gamma_{n-1}$ acts on $\$\R^{n-1} \cup \{\infty\}$. Via the natural inclusions $\Cl(\R^{0,n-2}) \subset \Cl(\R^{0,n-1})$ and $\$\R^{n-1} \subset \$\R^n$ and $\$\Gamma_{n-1} \subset \$\Gamma_n$ then $SL_2\$\Gamma_{n-1}$ acts on $\Cl(\R^{0,n-1})$ preserving $\$\R^{n-1} \cup \{\infty\}$ and $\$\R^n \cup \{\infty\}$. Being orientation-preserving, the action of $SL_2 \$\Gamma_{n-1}$ preserves the upper half space $\hyp^n$, where it acts via orientation-preserving isometries, yielding a 2-fold cover of the orientation-preserving isometry group of $\hyp^{n}$ in the upper half space model. In other words, $SL_2\$\Gamma_{n-1} \cong \Isom^S \hyp^n$ and $PSL_2\$\Gamma_{n-1} \cong \Isom^+ \hyp^n$ \subsection{Complex numbers} \label{Sec:complex} As a preliminary example, consider $(p,q) = (0,1)$. We obtain $\Cl(\R^{0,1}) \cong \C$, with $e_1 = i$. Under this isomorphism, the real numbers in $\C$ are scalars in $\Cl(\R^{0,1})$, and pure imaginary numbers are vectors. The conjugation $x \mapsto x^*$ is trivial, and the other conjugations are the standard complex number conjugation, $\overline{x} = x'$. The paravectors naturally have signature $(2,0)$ and form $\$\R^2$, corresponding to the entire space $\C$. The Lipschitz group $\Gamma_{0,1}$ is generated by nonzero imaginary complex numbers. The odd elements of $\Gamma_{0,1}$ are nonzero imaginary elements, and the even elements, i.e. the elements of $\Gamma_{0,1}^+$, are nonzero real numbers. The action of every element of $\Gamma_{0,1}$ on $\R^{0,1}$ is by negation (reflection of $\R^{0,1}$), as the algebra is commutative. The set of invertible paravectors is $\C \setminus \{0\}$, so the paravector Lipschitz group of $\Cl(\R^{0,1})$, which is $\$\Gamma_{2}$, is isomorphic to $\C^\times$. The action of $v = re^{\theta i} \in \$\Gamma_2$ (where $r>0$ and $\theta \in \R$ as in the usual polar form of a nonzero complex number) on the space of paravectors $\C$ given by $x \mapsto vx(v')^{-1}$ sends $x$ to $x e^{2 \theta i}$. Thus $v$ acts by rotation by $2\theta$. Similarly the action $x \mapsto vxv^*$ acts as multiplication by $v^2 = r^2 e^{2\theta i}$, i.e. rotation by $2\theta$ and dilation by $r^2$ from the origin. The group $SL_2 \$\Gamma_2$ is then just the usual $SL_2 \C$, which acts on $\$\R^2 \cup \{\infty\} = \C \cup \{\infty\}$ by M\"{o}bius transformations in the usual way, forming a 2-fold cover of the group of M\"{o}bius transformations $PSL_2 \C \cong \Isom^+ \hyp^3$. \subsection{Quaternions and their basic properties} \label{Sec:quaternion_involution} \label{Sec:quaternions_inner_product} For the rest of this paper, we specialise to the case $(p,q) = (0,2)$, and $\Cl(\R^{0,2}) \cong \HH$. We begin by recalling some basic facts about quaternions. We obtain , with $e_1 = i$, $e_2 = j$, and $e_1 e_2 = k$, and for the remainder of this paper we identify these two spaces. Under this isomorphism, the real numbers in $\HH$ are scalars, the real linear combinations of $i$ and $j$ are vectors, the paravectors are as in \refdef{quaternionic_spinor}, and real multiples of $k$ are bivectors. For the remainder of this subsection we write a general quaternion $q \in \HH$ as $q = a+bi+cj+dk$ where $a,b,c,d \in \R$. The three Clifford algebra conjugations apply as follows. \begin{defn} \label{Def:involutions} For a quaternion $q = a+bi+cj+dk$, \begin{enumerate} \item $q \mapsto q'$ is the homomorphic involution given by $q' = a-bi-cj+dk$ \item $q \mapsto \overline{q}$ is the anti-homomorphic involution given by $\overline{q} = a-bi-cj-dk$ \item $q \mapsto q^*$ is the anti-homomorphic involution given by $q^* = a+bi+cj-dk$. \end{enumerate} \end{defn} The following fact is easily verified. \begin{lem} \label{Lem:conjugation_combination} For any $q \in \HH$, \[ q + \overline{q} - q^* - q' = 0. \] \qed \end{lem} A quaternion $q$ has \emph{norm} $|q| \geq 0$ given by $|q|^2 = q \overline{q} = a^2 + b^2 + c^2 + d^2$, which extends to all of $\HH$ the quadratic form $\$ Q$ which exists on paravectors in general. The related bilinear form $\langle \cdot, \cdot \rangle_\$ $, which we henceforth simply denote $\langle \cdot, \cdot \rangle$, also extends to a map $\HH \times \HH \to \R$, given by $\langle v, w \rangle = \frac{1}{2}(v \overline{w} + w \overline{v}) = \Re(v \overline{w}) = \Re (w \overline{v})$. If $q \neq 0$ then $q^{-1} = \overline{q}/|q|^2$. A \emph{unit quaternion} is a $q$ such that $|q| = 1$. An \emph{imaginary} quaternion is a real linear combination of $i,j$ and $k$. We denote the imaginary quaternions by $\II$, so as real vector spaces $\HH = \R \oplus \II$ and $\II = \R i \oplus \R j \oplus \R k$. By the \emph{real part} and \emph{imaginary part} of $q$ we mean $a$ and $bi+cj+dk$ respectively. For any non-real quaternion $u$, the real span of $1$ and $u$ is algebraically isomorphic to $\C$. Indeed, if $u$ is unit imaginary then $u^2 = -u\overline{u} = -|u|^2 = -1$ so this isomorphism can be chosen to take $u$ to $i$. We then have $e^{u \theta} = \cos \theta + u \sin \theta$ for real $\theta$. A quaternion $q$ can be written in \emph{polar} form as $q = r e^{\theta u}$ where $r, \theta \in \R$, $r = |q| \geq 0$, and $u$ is unit imaginary. Indeed, $re^{u \theta} = r\cos \theta + r u \sin \theta$ so $u$ is a real multiple of $\Im q$. In polar form the conjugations are given by $(re^{u \theta})' = re^{u' \theta}$, $\overline{re^{u \theta}} = r e^{\overline{u}\, \theta} = r e^{-u\theta}$, and $(re^{u \theta})^* = re^{u^* \theta}$. \subsection{Quaternion paravectors and Lipschitz group} \label{Sec:quaternion_para} We collect some straightforward facts about quaternion paravectors. The paravectors in $\HH$ have signature $(q+1,p) = (3,0)$ so are written as $\$\R^3$ and agree with \refdef{quaternionic_spinor}(i). The following facts are used without further comment throughout this paper. \begin{lem} Let $q \in \HH$. \begin{enumerate} \item $q \in \$\R^3$ iff $q = q^*$. \item $q \in \$\R^3$ iff $qk \in \II$, and $q \in \II$ iff $qk \in \$\R^3$. \item $qq^* \in \$\R^3$. \end{enumerate} \end{lem} \begin{proof} Straightforward checks. \end{proof} The converse of (iii) is true: any paravector can be expressed as $qq^*$. In fact, the following stronger statement is true. \begin{lem} \label{Lem:paravector_square_root} For any paravector $v$, there exists a paravector $w$ such that $v = ww^* = w^2$. \end{lem} \begin{proof} Let $v = x+yu$ where $x,y \in \R$, $u \in \II \cap \$\R^3$ and $|u| = 1$. Let $a + b i \in \C$ be a complex square root of $x+yi$, where $a,b \in \R$. Then $w = a+b u$ has the desired properties. \end{proof} Since all nonzero elements of $\HH$ are invertible, the invertible paravectors are $\$\R^3 \setminus \{0\}$. The paravector Lipschitz group $\$\Gamma_3$ is the multiplicative subgroup of $\HH$ generated by $\$\R^3 \setminus \{0\}$. \begin{lem} $\$\Gamma_{3} = \HH^\times$. \end{lem} Here and throughout, $\HH^\times$ denotes the multiplicative subgroup of $\HH$, i.e. $\HH \setminus \{0\}$. \begin{proof} By multiplying paravectors by $i,j,ij = k \in \$\Gamma_3$ we find that $\$\Gamma_3$ contains all nonzero $a+bi+cj+dk$ such that $a,b,c,d \in \R$ and at least one of $a,b,c,d$ is zero. Now any $a+bi+cj+dk$ with $a,b,c,d$ all nonzero can be expressed as $(1 + \frac{ab-cd}{a^2+d^2} i + \frac{ac+bd}{a^2+d^2} j)(a+dk)$. \end{proof} \subsection{Dot and cross products with paravectors} \label{Sec:dot_cross_paravector} It is common to identify $\R^3$ with the imaginary quaternions, $(x,y,z) \leftrightarrow xi+yj+zk$, and scalars with the real quaternions, under which we have the standard equations for the standard dot and cross product of $v,w \in \R^3$: \[ vw = v \cdot w + v \times w, \quad v \cdot w = \frac{vw + wv}{2} = \Re (vw) = \Re (wv), \quad v \times w = \frac{vw - wv}{2} = \Im(vw) = \Im (-wv). \] The observation $\Re(vw) = \Re(vw)$ is true for any $v,w \in \HH$, and implies $vw - wv$ is imaginary, so we note the following fact which will be useful in the sequel: \begin{equation} \label{Eqn:commutator_identity} ab - ba + \overline{(ab - ba)} = ab - ba + \overline{b} \, \overline{a} - \overline{a} \overline{b} = 0, \quad \text{for any } a,b \in \HH. \end{equation} Although identifying $(x,y,z)$ with $xi+yj+zk$ is common, we will identify $\R^3$ with the paravectors in a standard way: via \begin{equation} \label{Eqn:R3_R3} \R^3 \cong \$\R^3, \quad (x,y,z) = x+yi+zj. \end{equation} The space $\$\R^3$ then inherits an orientation from the standard orientation on $\R^3$, with $(1,i,j)$ forming an oriented basis. We obtain the following equations for dot and cross products, for $v,w \in \$\R^3$: \begin{gather} \label{Eqn:quaternion_geometry_1} v \overline{w} = v \cdot w - (v \times w) k, \quad v \cdot w = \frac{v \overline{w} + w \overline{v}}{2} = \Re ( v \overline{w} ) = \Re (w \overline{v} ), \\ \label{Eqn:quaternion_geometry_2} v \times w= \frac{(v \overline{w} - w \overline{v}) k}{2}, \quad (v \times w) k = \Im( - v \overline{w} ) = \Im ( w \overline{v} ). \end{gather} In particular, the dot product on $\R^3 \cong \$\R^3$ is the restriction of the inner product $\langle \cdot, \cdot \rangle $ to $\$\R^3$, and the Euclidean norm on $\R^3 \cong \$\R^3$ is the restriction of the norm on $\HH$. \subsection{Rotations with paravectors} \label{Sec:para_rotation} The rotations produced by paravectors, discussed in \refsec{paravector_Mobius} above, are standard; for example Ahlfors in \cite{Ahlfors_Mobius85} describes them explicitly. However we need the explicit formulation for quaternions of \reflem{paravector_rotation} and \refprop{rho_rotation_dilation}, and we give a self-contained proof. \begin{lem} \label{Lem:quaternion_geometry_facts} Let $u \in \II$, so that $-uk \in \$\R^3$. Let $x \in \$\R^3$. \begin{enumerate} \item \label{Lem:paravector_commutation} $uk = -ku^*$. \item $xk = k\overline{x}$. \item $(-uk) \cdot x = 0$ if and only if $ux=xu^*$. In this case, $(-uk) \times x = ux = xu^*$. \end{enumerate} \end{lem} \begin{proof} The first two statements are straightforward checks. For (iii), using \refeqn{quaternion_geometry_1}, (i), (ii), and $\overline{u^*} = -u^*$ (since $u^*$ is imaginary) we have \[ 2 (-uk) \cdot x = -uk \overline{x} + x \overline{(-uk)} = -uxk + x \overline{ku^*} = \left( -ux + x u^* \right) k. \] We can then calculate the cross product using \refeqn{quaternion_geometry_1}, (i), (ii), and $\overline{u^*} = -u^*$ as \[ 2 (-uk) \times x = \left( (-uk) \overline{x} - x \overline{(-uk)} \right) k = \left( -uxk - x \overline{(ku^*)} \right) k = \left( -ux - x u^* \right) k^2 = ux+xu^* \] and the statement follows from noting $ux=xu^*$. \end{proof} \begin{lem} \label{Lem:paravector_rotation} Let $u \in \II$, so that $-uk \in \$\R^3$, and $x \in \$\R^3$. Suppose $(-uk) \cdot x = 0$ and further $|u| = 1$. Then rotation of $\$\R^3$ by angle $\theta$ about $-uk$ sends $x$ to \[ e^{u \theta} x = x e^{u^* \theta} = e^{u \theta/2} x e^{u^* \theta/2}. \] \end{lem} \begin{proof} Using the cross product and \reflem{quaternion_geometry_facts}(iii), the stated rotation sends $x \in \$\R^3$ to \[ x \cos \theta + (-uk) \times x \sin \theta = x \cos \theta + ux \sin \theta = \left( \cos \theta + u \sin \theta \right) x = e^{u \theta} x, \] the last equality holding since $u$ is imaginary and $|u| = 1$. Since $ux=xu^*$, the above expression equals \[ x \cos \theta + x u^* \sin \theta = x \left( \cos \theta + u^* \sin \theta \right) = x e^{u^* \theta}, \] since $u*$ is also unit imaginary. Since $e^{u\theta} x = x e^{u^* \theta}$ for all real $\theta$, we also have \[ e^{u \theta} x = e^{u \theta/2} e^{u \theta/2} x = e^{u \theta/2} x e^{u^* \theta/2}. \] \end{proof} \subsection{Actions of quaternions and paravectors} \label{Sec:actions_on_paravectors} The action of $v \in \$\Gamma_3$ on paravectors $\$\R^3$ by $x \mapsto v x (v')^{-1}$, often denoted $\rho$ in the literature \cite{Ahlfors_Lounesto_89, Ahlfors_Mobius85, Ahlfors_fixedpoints_85, Waterman_93}, yields rotations. We use $\rho$ for other purposes (e.g. \refsec{intro_lambda}), and are interested in the slightly different action $x \mapsto vxv^*$, so we notate it by $\sigma$; in fact it extends to an action of $\HH$ on $\HH$ as follows. \begin{defn} \label{Def:rho} We define an action of $\HH$ into linear endomorphisms on $\HH$ as follows. \[ \sigma \colon \HH \To \End(\HH), \quad \sigma(v) (x) = vxv^*. \] \end{defn} Throughout this paper we will be considering rotations in 2-planes $\Pi$ in various ambient dimensions. We can describe the sense of such rotations by giving two vectors $v,w$ spanning the 2-plane $\Pi$ of the rotation, and saying that we rotate from $v$ \emph{towards} $w$ by some angle. This means that, for small angles, $v$ rotates into the half-2-plane $\Pi \setminus \R v$ containing $w$. Clearly, when $v \in \R$, $\sigma(v)$ is multiplication by $v^2$, so it suffices to consider non-real $\sigma(v)$. \begin{prop} \label{Prop:rho_rotation_dilation} \label{Prop:rho_rotation} Let $v \in \HH \setminus \R$ be given in polar form as $v = r e^{\theta u}$, so $\sigma(v)$ is a linear endomorphism of $\HH$. Then $\sigma(v)$ respects the splitting $\HH = \$\R^3 \oplus \R k$, and acts on each summand as follows. \begin{enumerate} \item The restriction of $\sigma(v)$ to $\$\R^3$, identified with $\R^3$ by \refeqn{R3_R3}, is a rotation of angle $2\theta$ about the axis $-uk$, composed with multiplication by $|v|^2$. \item If $v \in \$\R^3$ then the rotation described above is also a rotation of angle $2\theta$ in the 2-plane $\Pi$ spanned by $1$ and $v$, from $1$ towards $v$. \item The restriction of $\sigma(v)$ to $\R k$ is multiplication by $|v|^2$. \end{enumerate} \end{prop} Since the action $x \mapsto vx(v')^{-1}$ is obtained from $\sigma$ by post-multiplication by $|v|^{-2}$, under this action the splitting $\$\R^3 \oplus \R k$ is again preserved, $v$ acts to preserve $\R k$ pointwise, and $v$ acts on $\$\R^3$ by the rotation described above. In particular, the action is by orientation-preserving isometries. \begin{proof} Since $\sigma(v)$ is linear, it suffices to prove (i) for $x \in \$\R^3$ such that $x = -uk$, and for $x \in \$\R^3$ which are perpendicular to $-uk$. If $x = -uk$ then we have $uk = -ku^*$ (\reflem{quaternion_geometry_facts}(i)), so \begin{align*} vxv^* &= \left( r e^{u \theta} \right) (-uk) \left( r e^{u^* \theta} \right) = - r^2 \left( \cos \theta + u \sin \theta \right) u k \left( \cos \theta + u^* \sin \theta \right) \\ &= - r^2 u \left( \cos \theta + u \sin \theta \right) \left( \cos \theta - u \sin \theta \right) k = - r^2 uk = r^2 x = |v|^2 x. \end{align*} If $x \in \$\R^3$ is perpendicular to $-uk$ then $vxv^* = r^2 e^{u \theta} x e^{u^* \theta}$ by \reflem{quaternion_geometry_facts}(iv) is $|v|^2$ times the rotation of $x$ by $2\theta$ about $-uk$. If $v \in \$\R^3$, then $v = re^{\theta u}$ where $u$ is a unit imaginary paravector, and we may take $0 < \theta < \pi$, so that $\sin \theta > 0$ and $\Im(v)$ is a positive multiple of $u$. Then a rotation in $\Pi$ from $1$ towards $v$ is a rotation about the axis $1 \times v$, which by \refeqn{quaternion_geometry_2} is $\frac{1}{2}(\overline{v}-v)k = - \Im(v)k$, which is a positive multiple of $-uk$. To see (iii), note that if $v \in \$\R^3$ then we have $v^* = v$ and by \reflem{quaternion_geometry_facts}(i) $vk=k\overline{v}$ so $\sigma(v)(k) = vkv = k\overline{v} v = k |v|^2$. Successively multiplying by other paravectors we obtain $\sigma(v)(k) = k |v|^2$ for all $v \in \$\Gamma_3 = \HH \setminus \{0\}$. \end{proof} The unit quaternions form a subspace of $\HH^3$ diffeomorphic to $S^3$, which is a multiplicative subgroup; we denote this group simply by $S^3$. Restricting $\sigma$ to the action of $S^3$ on $\$\R^3$, the action is by rotations, and we obtain a homomorphism $S^3 \To SO(3)$. It is clear from the description above that this homomorphism is surjective. Two elements $e^{u_1 \theta_1}, e^{u_2 \theta_2}$ describe the same rotation iff $- u_1 k = - u_2 k$ and $2\theta_1 \equiv 2\theta_2$ mod $2\pi$, or $-u_1 k = u_2 k$ and $2\theta_1 = - 2\theta_2$ mod $2\pi$. This occurs iff $e^{u_1 \theta_1} = \pm e^{u_2 \theta_2}$. We obtain the standard 2--1 covering map $S^3 \To SO(3)$ and we regard $S^3$ as $\Spin(3)$. \section{Geometry of spinors} \label{Sec:spinors} \subsection{Inner product and norm on $\HH^2$} \label{Sec:inner_product_norm_H2} We now consider pairs of quaternions $\kappa = (\xi, \eta)$, i.e. elements of $\HH^2$. We begin by introducing a standard real-valued inner product and norm. If $\kappa_m = (\xi_m, \eta_m)$, $m=1,2$, then we define \[ \langle \cdot, \cdot \rangle \colon \HH^2 \times \HH^2 \To \R, \quad \langle \kappa_1, \kappa_2 \rangle = \frac{\xi_1 \overline{\xi_2} + \xi_2 \overline{\xi_1} + \eta_1 \overline{\eta_2} + \eta_2 \overline{\eta_1}}{2} = \Re \left( \xi_1 \overline{\xi_2} + \eta_1 \overline{\eta_2} \right), \] which is real-bilinear, nondegenerate, symmetric, and positive definite, with the following properties. \begin{lem} Let $\kappa, \kappa_1, \kappa_2 \in \HH^2$ and $x,y \in \HH$. Then we have the following. \label{Lem:quaternion_inner_product_mult} \label{Lem:imaginary_multiplication_orthogonal} \begin{enumerate} \item $\langle \kappa_1 x, \kappa_2 x \rangle = \langle x \kappa_1, x \kappa_2 \rangle = |x|^2 \langle \kappa_1, \kappa_2 \rangle$. \item $\langle \kappa x, \kappa y \rangle = |\kappa|^2 \langle x,y \rangle$. \item If $x \in \HH$ is imaginary, then $\langle \kappa, \kappa x \rangle = \langle \kappa x, \kappa \rangle = 0$. \end{enumerate} \end{lem} Note in (ii) that the first inner product is for $\HH^2$, the second for $\HH$, as in \refsec{quaternions_inner_product}. \begin{proof} For (i), we observe $\langle x \kappa_1, x \kappa_2 \rangle = x \langle \kappa_1, \kappa_2 \rangle \overline{x} = |x|^2 \langle \kappa_1, \kappa_2 \rangle$. A direct calculation also shows $\langle \kappa_1 x, \kappa_2 x \rangle = |x|^2 \langle \kappa_1, \kappa_2 \rangle$. For (ii), let $\kappa = (\xi, \eta)$. Noting that $\langle x, y \rangle = \frac{1}{2}( x \overline{y} + y \overline{x})$ is real, we obtain \begin{align*} \langle \kappa x, \kappa y \rangle &= \frac{\xi x \, \overline{y} \, \overline{\xi} + \xi y \, \overline{x} \, \overline{\xi} + \eta x \, \overline{y} \, \overline{\eta} + \eta y \, \overline{x} \, \overline{\eta}}{2} = \frac{ \xi \left( x \overline{y} + y \overline{x} \right) \overline{\xi} + \eta \left( x \overline{y} + y \overline{x} \right) \overline{\eta}}{2} \\ &= \langle x, y \rangle \left( \xi \overline{\xi} + \eta \overline{\eta} \right) = \langle x, y \rangle \; |\kappa|^2. \end{align*} The final statement then follows from (ii) upon noting that $\langle 1, x \rangle = x + \overline{x} = 0$ when $x$ is imaginary. \end{proof} The associated norm is given, for $\kappa = (\xi, \eta) \in \HH^2$, by \begin{equation} \label{Eqn:H2_norm} | \cdot | \colon \HH^2 \To \R_{\geq 0}, \quad |\kappa|^2 = \langle \kappa, \kappa \rangle = |\xi|^2 + |\eta|^2. \end{equation} We will sometimes identify $\HH^2$ with $\R^8$ in a standard way, \begin{equation} \label{Eqn:H2_R8} \HH^2 \cong \R^8, \quad (\xi,\eta) = (x_0 + x_1 i + x_2 j + x_3 k, y_0 + y_1 i + y_2 j + y_3 k) \leftrightarrow (x_0, x_1, x_2, x_3, y_0, y_1, y_2, y_3). \end{equation} Then $\langle \cdot, \cdot \rangle$ is the standard dot product, and the norm is the standard Euclidean norm. In particular, the set of unit $\kappa \in \HH^2$, i.e. $\kappa = (\xi, \eta)$ with $|\kappa|^2 = |\xi|^2 + |\eta|^2 = 1$, is the standard $S^7 \subset \R^8$. \subsection{Conditions for spinors} \label{Sec:spinor_conditions} We have defined the space $S\HH$ of quaternionic spinors in \refdef{quaternionic_spinor}. The following lemma allows equivalent characterisations, and we use it repeatedly throughout; it includes lemma 1.4 of \cite{Ahlfors_Mobius85}. \begin{lem} \label{Lem:spinor_condition} Let $\xi = x_0 + x_1 i + x_2 j + x_3 k$, $\eta = y_0 + y_1 i + y_2 j + y_3 k$ where all $x_m, y_m \in \R$. The following are equivalent. \begin{enumerate} \item $\xi \overline{\eta} \in \$\R^3$ \item $\xi^* \eta \in \$\R^3$ \item $x_0 y_3 + x_1 y_2 - x_2 y_1 - x_3 y_0 = 0$. \end{enumerate} \end{lem} \begin{proof} The rotation $\sigma(\eta^*/|\eta|)$ takes $\xi \overline{\eta}$ to $\eta^* \xi \overline{\eta} \eta / |\eta|^2 = \xi^* \eta$. Since $\sigma$ preserves $\$\R^3$, each of $\xi \overline{\eta}, \xi^* \eta$ lies in $\$\R^3$ simultaneously with the other. Alternately, and for the equivalence of (iii), we compute that the coefficient of $k$ in both $-\xi \overline{\eta}$ and $\xi^* \eta$ is $x_0 y_3 + x_1 y_2 - x_2 y_1 - x_3 y_0$. \end{proof} Since $\$\R^3$ is mapped to itself bijectively under the operations $v \mapsto \overline{v}$ and $ \mapsto v^*$, as well as their composition $v \mapsto v'$, the first two conditions above are also equivalent to any one of \begin{equation} \label{Eqn:spinor_conditions} \overline{(\xi \overline{\eta})} = \eta \overline{\xi}, \quad \left( \xi \overline{\eta} \right)^* = \eta' \xi^*, \quad \left( \xi \overline{\eta} \right)' = \xi' \eta^*, \quad \overline{\xi^* \eta} = \overline{\eta} \xi', \quad \left( \xi^* \eta \right)^* = \eta^* \xi, \quad \left( \xi^* \eta \right)' = \overline{\xi} \eta' \end{equation} lying in $\$\R^3$. Since $\eta^{-1} = \overline{\eta}/|\eta|^2$ is a real multiple of $\overline{\eta}$, when $\eta \neq 0$, we can replace $\overline{\eta}$ with $\eta^{-1}$ in any of the above. So $(\xi, \eta) \in S\HH$ if and only if any one of the above conditions (hence all of them) are satisfied. Also, spinors are preserved under right multiplication and swapping coordinates. \begin{lem} \label{Lem:spinor_right_multiplication} Let $\kappa \in S\HH$. Then we have the following. \begin{enumerate} \item If $x \in \HH^\times$ then $\kappa x \in S\HH$. \item $(\eta, \xi) \in S\HH$. \end{enumerate} \end{lem} \begin{proof} We have $\kappa \neq 0$ and $\xi \overline{\eta}, \xi^* \eta \in \$\R^3$. Then $(\xi x) \overline{(\eta x)} = \xi x \, \overline{x} \overline{\eta} = \xi \overline{\eta} |x|^2 \in \$\R^3$, and $\kappa x $ is nonzero since $x \neq 0$, so $\kappa x \in S\HH$. Similarly, $(\eta, \xi)$ is nonzero, and $\eta^* \xi = (\xi^* \eta)^* \in \$\R^3$, so $(\eta, \xi) \in S\HH$. \end{proof} \subsection{Bracket on $\HH^2$} \label{Sec:bracket} We now define another bilinear form on $\HH^2$. Unlike $\langle \cdot, \cdot \rangle$, this bilinear form $\{ \cdot, \cdot \}$ is quaternion-valued, and is not symmetric. We call it the \emph{bracket}, as seen in \refeqn{lambda_pdet}: for $\kappa_1, \kappa_2 \in \HH^2$, $\kappa_m = (\xi_m, \eta_m)$, denoting by $(\kappa_1, \kappa_2)$ the $2 \times 2$ matrix with $\kappa_1, \kappa_2$ as columns, we have \[ \{ \cdot, \cdot \} \colon \HH^2 \times \HH^2 \To \HH, \quad \{\kappa_1, \kappa_2\} = \pdet(\kappa_1, \kappa_2) = \xi_1^* \eta_2 - \eta_1^* \xi_2. \] This bilinear form is real-bilinear, nondegenerate, and satisfies the antisymmetry condition \[ \{\kappa_2, \kappa_1\} = - \{\kappa_1, \kappa_2\}^*. \] We now demonstrate several properties of the bracket. \begin{lem} \label{Lem:bracket_properties} Let $\kappa \in \HH^2$. \begin{enumerate} \item $\{\kappa, \kappa \} \in \R k$. \item $\{ \kappa, \kappa \} = 0$ if and only if $\kappa \in S\HH \cup \{0\}$. \end{enumerate} \end{lem} \begin{proof} Letting $\kappa = (\xi, \eta)$ we have $\{\kappa, \kappa\} = \xi^* \eta - \eta^* \xi$, which is sent to its negative under the $*$-involution, hence lies in $\R k$. It is zero precisely when $\xi^* \eta \in \$ \R^3$, i.e. $\kappa \in S\HH$ or $\kappa = 0$. \end{proof} The bracket also has the multiplicativity property \begin{equation} \label{Eqn:bracket_multiplication} \{\kappa_1 x_1, \kappa_2 x_2 \} = x_1^* \; \{ \kappa_1, \kappa_2 \} \; x_2 \quad \text{for any } \kappa_1, \kappa_2 \in \HH^2 \text{ and } x_1, x_2 \in \HH. \end{equation} \begin{lem} \label{Lem:nondegeneracy_of_spinor_form} Let $\kappa = (\xi, \eta) \in S\HH$ and $\nu = (\alpha, \beta) \in \HH^2$. Then $\{\kappa, \nu \} = 0$ if and only if $\nu = \kappa x$ for some $x \in \HH$ (and hence $\nu = 0$ or, by \reflem{spinor_right_multiplication}, $\nu \in S\HH$). \end{lem} \begin{proof} If $\nu = \kappa x$ then either $x = 0$ so that $\nu = 0$ and $\{\kappa, \nu\} = 0$ trivially, or $x \neq 0$ so $\nu \in S\HH$ by \reflem{spinor_right_multiplication}. Then $\{\kappa, \nu \} = \{\kappa, \kappa x \} = \{ \kappa, \kappa \} x = 0$, using \refeqn{bracket_multiplication} and antisymmetry of the bracket. For the converse, suppose \[ \{\kappa, \nu \} = \xi^* \beta - \eta^* \alpha = 0. \] If $\nu = 0$ then we may take $x=0$, so we may assume $\nu \neq 0$. If $\beta = 0$ then we have $\eta^* \alpha = 0$, but $\alpha \neq 0$ (since $\nu \neq 0$), so $\eta = 0$; then as $\kappa = (\xi, 0) \in S\HH$ we have $\xi \neq 0$; we then have $\nu = (\alpha, 0) = (\xi, 0) x = \kappa x$ where $x = \xi^{-1} \alpha$. Thus we may assume $\beta \neq 0$. Similarly, if $\eta = 0$ then we have $\xi^* \beta = 0$, but $\beta \neq 0$ (from above) and $\xi \neq 0$ (as $\kappa \in S\HH$), a contradiction. Thus we may assume $\eta \neq 0$. The equation $\xi^* \beta = \eta^* \alpha$ is then equivalent to $(\xi \eta^{-1})^* = \alpha \beta^{-1}$. As $\kappa \in S\HH$ then $\xi \eta^{-1} \in \$\R^3$, hence is invariant under $*$, so also $\alpha \beta^{-1} \in \$\R^3$ and we then have $\xi \eta^{-1} = \alpha \beta^{-1}$. Letting $x = \xi^{-1} \alpha$ we then have $\alpha = \xi x$ and $\beta = \eta \xi^{-1} \alpha = \eta x$ so $\nu = (\alpha, \beta) = (\xi, \eta) x = \kappa x$. \end{proof} \subsection{Complementary elements} \label{Sec:complementary} Given $\kappa = (\xi, \eta) \in \HH^2$, it will be useful to introduce the following ``complementary" element of $\HH^2$. It is perhaps analogous to an almost complex structure in a K\"{a}hler structure, ``$\check{\kappa}$ is like $J\kappa$". Indeed the map $J$ sending $\kappa \mapsto \check{\kappa}$ satisfies $J^2 = -1$. Similarly, ``$\langle \cdot, \cdot \rangle$ is a metric" and ``$\{ \cdot, \cdot \}$ is like a symplectic form". See \refsec{paravectors_in_TSH} for further discussion and \refsec{deriv_phi1} for observations along similar lines. \begin{defn} For $\kappa = (\xi, \eta) \in \HH^2$, let $\check{\kappa} = (\eta', -\xi')$. \end{defn} Clearly $|\kappa| = |\check{\kappa}|$. \begin{lem} \label{Lem:complementary_spinor_facts} Let $\kappa = (\xi, \eta) \in \HH^2$. \begin{enumerate} \item If $\kappa \in S\HH$ then $\check{\kappa} \in S\HH$. \item $\langle \kappa, \check{\kappa} \rangle = 0$. In fact, for any $x,y \in \HH$ such that $x \overline{y} \in \$\R^3$, we have $\langle \kappa x, \check{\kappa} y \rangle = 0$. \item If $\kappa \in S\HH$ then $\langle \kappa x, \check{\kappa} y \rangle = 0$ for any $x,y \in \HH$. \item $\{ \kappa, \check{\kappa} \} = - |\kappa|^2$. More generally, for any $x,y \in \HH$ we have $\{ \kappa x, \check{\kappa} y \} = - x^* y |\kappa|^2$. \end{enumerate} \end{lem} \begin{proof} For (i), if $(\xi, \eta) \in S\HH$ then $\xi \overline{\eta} \in \$\R^3$. Then $(\eta')\overline{(-\xi')} = - \eta' \xi^* = (\xi \overline{\eta})^* \in \$\R^3$, so $\check{\kappa} \in S\HH$. For (ii), noting that $x \overline{y} \in \$\R^3$ implies $(x \overline{y})^* = x\overline{y}$, i.e. $y' x^* = x \overline{y}$, and hence $\overline{x\overline{y}} = \overline{y' x^*}$, so $y \overline{x} = \overline{(y' x^*)} = x' y^*$, we compute \begin{align*} \langle \kappa x, \check{\kappa} y \rangle &= \langle (\xi, \eta)x , (\eta', -\xi') y \rangle = \frac{ \xi x \, \overline{y} \, \eta^* + \eta' y \, \overline{x} \, \overline{\xi} - \eta x \, \overline{y} \xi^* - \xi' y \, \overline{x} \, \overline{\eta} }{2} \\ &= \frac{ \xi x \, \overline{y} \, \eta^* + \eta' y \, \overline{x} \, \overline{\xi} - \eta y' x^* \xi^* - \xi' x' y^* \, \overline{\eta}}{2} \end{align*} which is zero, since equal to $(a + \overline{a} - a^* + a')/2$ where $a = \xi x \, \overline{y} \, \eta^*$, by \reflem{conjugation_combination}. If $\kappa \in S\HH$ then we have $\xi \eta^{-1} = v \in \$\R^3$, so $\xi = v \eta$ where $v = v^*$ and $\overline{v} = v'$, and following the above calculation we obtain \begin{align*} \langle \kappa x, \check{\kappa} y \rangle &= \frac{v \eta x \, \overline{y} \, \eta^* + \eta' y \, \overline{x} \, \overline{\eta} \, \overline{v} - \eta x \, \overline{y} \, \eta^* v^* - v' \eta' y \, \overline{x} \, \overline{\eta}}{2} \\ &= \frac{v \eta x \, \overline{y} \, \eta^* + \eta' y \, \overline{x} \, \overline{\eta} \, \overline{v} - \eta x \, \overline{y} \, \eta^* v - \overline{v} \, \eta' y \, \overline{x} \, \overline{\eta}}{2} \\ &= \frac{ab + \overline{b} \, \overline{a} - b a - \overline{a} \, \overline{b}}{2} \quad \text{where $a = v$ and $b = \eta x \, \overline{y} \eta^*$}, \end{align*} which is zero by \refeqn{commutator_identity}, proving (iii). Finally, for $\kappa \in \HH^2$ we compute \begin{align*} \{ \kappa, \check{\kappa} \} = \pdet \begin{pmatrix} \xi & \eta' \\ \eta & -\xi' \end{pmatrix} = - \xi^* \xi' - \eta^* \eta' = -|\xi|^2 - |\eta|^2 = - |\kappa|^2, \end{align*} from which statement (iv) follows by \refeqn{bracket_multiplication}. \end{proof} We will need the following technical fact about $\kappa$ and $\check{\kappa}$ in the sequel. \begin{lem} \label{Lem:factorisation_fact} Let $\kappa = (\xi, \eta) \in S\HH$, considered as a $2 \times 1$ column vector. Then there exists a $1 \times 2$ row vector $\tau = (\alpha, \beta) \in \HH^2$ such that precisely one of $\tau \kappa, \tau \check{\kappa}$ is zero. \end{lem} \begin{proof} Suppose to the contrary that $\{ (\alpha, \beta) \in \HH^2 \mid \alpha \xi + \beta \eta = 0 \} = \{ (\alpha, \beta) \in \HH^2 \mid \alpha \eta' - \beta \xi' = 0 \}$. If $\xi = 0$ or $\eta = 0$ (we cannot have $\xi = \eta = 0$ since $\kappa \in S\HH$) then the two sets are clearly distinct, so we may assume $\xi, \eta$ are both nonzero. In this case, any $(\alpha, \beta) \neq (0,0)$ in either set has $\alpha, \beta$ both nonzero. Other than $(0,0)$, the first set contains all $(\alpha, \beta)$ such that $\beta^{-1} \alpha = -\eta \xi^{-1}$, and the second set contains all $(\alpha, \beta)$ such that $\beta^{-1} \alpha = \xi' \eta'^{-1}$. So $- \eta \xi^{-1} = \xi' \eta'^{-1}$. Now let $v = \xi \eta^{-1}$. Then $v \neq 0$, and as $\kappa \in S\HH$ we have $v \in \$\R^3$. Then $\eta \xi^{-1} = v^{-1}$ and $\xi' \eta'^{-1} = v'$, so the equation $-\eta \xi^{-1} = \xi' \eta'^{-1}$ implies $-v^{-1}= v'$. Comparing norms yields $|v| = 1$, so $v^{-1} = \overline{v}$. Thus we obtain $-\overline{v} = v'$, and hence $v = -v^*$. But as $v \in \$\R^3$ then $v = v^*$, so $v=0$, a contradiction. \end{proof} \subsection{The space of spinors} \label{Sec:space_of_spinors} We now consider the space $S\HH$. Firstly we consider its topology. Note that as a smooth real manifold, $\HH^2 \cong \R^8$ and $S\HH$ is a subset of the non-compact manifold $\HH^2 \setminus \{0\} \cong S^7 \times \R$. \begin{lem} \label{Lem:topology_of_SH} $S\HH$ is diffeomorphic to $S^3 \times S^3 \times \R$. In particular, $S\HH$ is orientable. \end{lem} \begin{proof} The equation in \reflem{spinor_condition}(iii) can be written as $(x_0 + y_3)^2 - (x_0 - y_3)^2 + (x_1 + y_2)^2 - (x_1 - y_2)^2 - (x_2 + y_1)^2 + (x_2 - y_1)^2 - (x_3 + y_0)^2 + (x_3 - y_0)^2 = 0$, so changing variables to the expressions in brackets, the equation is the zero set of a standard quadratic Morse function $\R^8 \To \R$ of index 4. Hence its zero set (minus the origin), which is $S\HH$, is diffeomorphic to $S^3 \times S^3 \times \R$. \end{proof} Thus, $S\HH$ is a 7-(real-)dimensional subset of the 8-dimensional space $\HH^2 \setminus \{0\} \cong S^7 \times \R$, cut out by the real quadratic equation of \reflem{spinor_condition}(iii). A spinor $\kappa = (\xi, \eta) \in S\HH$ by definition has $\xi \eta^{-1} \in \$\R^3 \cup \{\infty\}$. Thus there is a map $S\HH \To \$\R^3 \cup \{\infty\}$ given by $(\xi, \eta) \mapsto \xi \eta^{-1}$. We now describe the fibres of this map; they are diffeomorphic to $S^3 \times \R \cong \HH^\times$. \begin{lem} \label{Lem:division_fibre} Let $z \in \$\R^3 \cup \{\infty\}$ and let $S\HH_z = S\HH \cap \{ (\xi, \eta) \mid \xi \eta^{-1} = z\}$. Then $S\HH_z$ is diffeomorphic to $S^3 \times \R$, and if $(\xi_0, \eta_0) \in S\HH_z$ then $S\HH_z = (\xi_0, \eta_0)\HH^\times$. \end{lem} \begin{proof} Take $z \in \$\R^3 \cup \{\infty\}$. Note $S\HH_z$ is nonempty; for instance it contains $(z,1)$ when $z \in \$\R^3$, and $(1,0)$ when $z = \infty$. Let $\kappa_0 = (\xi_0, \eta_0)$ be an arbitrary element of $S\HH_z$. We will show $S\HH_z = \kappa_0 \HH^\times$. For any $x \in \HH^\times$ we have $(\xi_0 x)(\eta_0 x)^{-1} = \xi_0 x x^{-1} \eta_0^{-1} = \xi_0 \eta_0^{-1} = z$. Thus $\kappa_0 \HH^\times \subseteq S\HH_z$. Conversely, suppose $(\xi, \eta) \in S\HH_z$. Then $\xi \eta^{-1} = \xi_0 \eta_0^{-1} = z$. If $z = \infty$ then we have $\eta = \eta_0 = 0$ and $\xi, \xi_0 \neq 0$, so letting $x = \xi_0^{-1} \xi \in \HH^\times$ we have $(\xi, \eta) = (\xi_0 x, 0) = (\xi_0, \eta_0)x \in \kappa_0 \HH^\times$. If $z \in \$\R^3$ then $\eta, \eta_0$ are nonzero; letting $x = \eta_0^{-1} \eta \in \HH^\times$, from $\xi \eta^{-1} = \xi_0 \eta_0^{-1} = z$ we have $\xi_0^{-1} \xi = \eta_0^{-1} \eta = x$, so $(\xi, \eta) = (\xi_0, \eta_0)x \in \kappa_0 \HH^\times$. Thus $S\HH_z \subseteq \kappa_0 \HH^\times$ and hence $S\HH_z = \kappa_0 \HH^\times$. \end{proof} \begin{lem} \label{Lem:TSH} Let $\kappa = (\xi, \eta) \in S\HH$. Then the tangent space to $S\HH$ is given by \[ T_\kappa S\HH = \left\{ (\alpha, \beta) \in \HH^2 \mid \alpha \overline{\eta} + \xi \overline{\beta} \in \$\R^3 \right\} \] and decomposes into orthogonal real subspaces as \[ T_\kappa S\HH = \kappa \, \HH \; \oplus \; \check{\kappa} \, \$ \R^3 = \kappa \R \oplus \kappa \II \oplus \check{\kappa} \$\R^3 \] \end{lem} \begin{proof} The first statement is obtained by differentiating the defining equation $(\xi + t \alpha) \overline{(\eta + t \beta)} \in \$\R^3$ with respect to $t \in \R$, as $\$\R^3$ is a real vector subspace. Taking $(\alpha, \beta) = (\xi, \eta) x$ with $x \in \HH$, we have \[ \alpha \, \overline{\eta} + \xi \, \overline{\beta} = \xi x \, \overline{\eta} + \xi \, \overline{x} \, \overline{\eta} = \xi (x + \overline{x}) \, \overline{\eta} = 2 \Re(x) \xi \overline{\eta} \in \$\R^3 \] Thus $\kappa \HH \subset T_\kappa S\HH$. Taking $(\alpha, \beta) = \check{\kappa} v = (\eta', -\xi')v$ where $v \in \$\R^3$, \[ \alpha \overline{\eta} + \xi \overline{\beta} = \eta' v \overline{\eta} - \xi \overline{v} \xi^* = \sigma(\eta')(v) - \sigma(\xi)(\overline{v}) \in \$\R^3, \] since $\sigma$ preserves $\$\R^3$. Thus $\kappa \HH$ and $\check{\kappa} \$\R^3$ are subspaces of $T_\kappa S\HH$, of real dimension 4 and 3 respectively. By \reflem{complementary_spinor_facts}(iii) they are orthogonal. Thus we have the first orthogonal direct sum. Since $\HH = \R \oplus \II$ we have the second direct sum decomposition, and by \reflem{imaginary_multiplication_orthogonal} it is also orthogonal. \end{proof} We note that it is also possible to show that $T_\kappa S\HH$ contains $(-\xi \overline{x}, \eta x)$ for all $x \in \HH$, $(v \eta, \overline{v} \xi)$ for all $v \in \$\R^3$, and $(k \xi, k \eta)$. The summand $\check{\kappa} \$\R^3$ of $T_\kappa S\HH$ provides us with a copy of the paravectors $\$\R^3$ at each point of $S\HH$, and it is naturally oriented via the orientation on $\$\R^3$ discussed in \refsec{dot_cross_paravector}. Thus $(\check{\kappa}, \check{\kappa}i,\check{\kappa}j)$ forms an oriented basis of $\check{\kappa}\$\R^3$. The other summand $\kappa \HH$ is the tangent space to the fibre $\kappa \HH^\times$ of \reflem{division_fibre}. It follows from this lemma, combined with \reflem{complementary_spinor_facts}(iii) and \reflem{imaginary_multiplication_orthogonal} that $T_\kappa S\HH$ has an orthogonal basis given by \[ \kappa, \; \kappa i, \; \kappa j, \; \kappa k, \; \check{\kappa}, \; \check{\kappa} i, \; \check{\kappa} j \] and an orthonormal basis is given by dividing each of these 7 vectors by $|\kappa| = |\check{\kappa}|$. Indeed, in this way we obtain 7 orthonormal sections hence a trivialisation of the tangent bundle of $S\HH$. If we add $\check{\kappa} k$ to these vectors, we obtain an orthogonal basis of $\HH^2$ (this follows from the same lemmas), which can be made orthonormal in the same way, so $\check{\kappa} k$ is normal to $T_\kappa S\HH$, and we have an orthogonal decomposition $\HH^2 = \kappa \, \HH \oplus \check{\kappa} \HH$. Indeed, given an element of $\HH^2$, we can explicitly express it as a quaternion combination of $\kappa$ and $\check{\kappa}$ as follows. \begin{lem} \label{Lem:explicit_decomposition_in_TSH} Let $\kappa = (\xi, \eta) \in S\HH$ and $\nu = (\alpha, \beta) \in \HH^2$. Then there exist unique $x,y \in \HH$ such that \[ \nu = \kappa x + \check{\kappa} y, \quad \text{namely} \quad x = \frac{ \overline{\xi} \, \alpha + \overline{\eta} \, \beta}{|\kappa|^2}, \quad y = \frac{ \eta^* \alpha - \xi^* \beta }{|\kappa|^2}. \] \end{lem} \begin{proof} The orthogonality argument above shows that there exist unique $x$ and $y$; to see that they in fact are as given, we can check explicitly that $\alpha = \xi x + \eta' y$ and $\beta = \eta x - \xi' y$. For the first equation we have \[ \xi x + \eta' y = \frac{ \xi \left( \overline{\xi} \, \alpha + \overline{\eta} \, \beta \right) + \eta' \left( \eta^* \alpha - \xi^* \beta \right)}{|\kappa|^2} = \frac{|\xi|^2 \alpha + \xi \overline{\eta} \beta + |\eta|^2 \alpha - \eta' \xi^* \beta}{|\kappa|^2} \] As $\kappa \in S\HH$ we have $\xi \overline{\eta} \in \$\R^3$, hence $\xi \overline{\eta} = \left( \xi \overline{\eta} \right)^* = \eta' \xi^*$. So the above expression simplifies to $\alpha$ as desired. A similar calculation establishes the second equality. \end{proof} \subsection{Paravectors in the tangent space of spinors} \label{Sec:paravectors_in_TSH} The summand $\check{\kappa} \$\R^3$ of $T_\kappa S\HH$ in \reflem{TSH} provides us with a copy of the paravectors $\$\R^3$ naturally at hand in the tangent space to $S\HH$ at any point. We therefore define the following family of sections of $TS\HH$, parametrised by paravectors. \begin{defn} \label{Def:Z} For any $v \in \$\R^3$, the section $s_v \colon S\HH \To TS\HH$ of $TS\HH$ is \[ s_v (\kappa) = \check{\kappa} v. \] More generally, the family of such sections forms a map \[ s \colon \$\R^3 \times S\HH \To TS\HH, \quad s(v, \kappa) = s_v (\kappa) = \check{\kappa} v. \] \end{defn} In particular, we have sections $s_1, s_i, s_j$ of $TS\HH$ are given by \[ s_1 (\kappa) = \check{\kappa}, \quad s_i (\kappa) = \check{\kappa} i, \quad s_j (\kappa) = \check{\kappa} j. \] These sections also behave nicely under the bracket: \reflem{complementary_spinor_facts}(iv) tells us that \begin{equation} \label{Eqn:inner_product_with_sv} \left\{ \kappa, s_v \kappa \right\} = \left\{ \kappa, \check{\kappa} v \right\} = - v \left( |\xi|^2 + |\eta|^2 \right) = - v |\kappa|^2. \end{equation} Moreover, $s_v$ and $s$ are real-linear in $v$: for $v,w \in \$\R^3$ and $x \in \R$ we have \[ s_{v+w} = s_v + s_w \quad \text{and} \quad s_{xv} = x s_v, \quad \text{i.e.} \quad s(v+w, \cdot) = s(v, \cdot) + s(w, \cdot) \quad \text{and} \quad s(xv, \cdot) = x s(v, \cdot). \] We can write \begin{equation} \label{Eqn:J_eqn} \check{\kappa} = J \kappa' \quad \text{where} \quad J = \begin{pmatrix} 0 & 1 \\ -1 & 0 \end{pmatrix}, \end{equation} so that $s_v (\kappa) = J \kappa' v$. For complex spinors, $\kappa' = \overline{\kappa}$ and $s_i$ reduces to the map $Z$ of \cite{Mathews_Spinors_horospheres}. (The $J$ there is $Ji$ here.) The copy of the paravectors inside the tangent space $T_\kappa S\HH$ at $\kappa$ is given by the map \begin{equation} \label{Eqn:conformal_paravectors_in_TSH} s( \cdot, \kappa) \colon \$\R^3 \To \check{\kappa} \$\R^3 \subset T_\kappa S\HH, \end{equation} which is an $\R$-linear isomorphism. The paravectors $\$\R^3$, identified with $\R^3$ as in \refeqn{R3_R3}, have the standard dot product and Euclidean norm, which agrees with the standard inner product and norm on $\HH$, as discussed in \refsec{dot_cross_paravector}. As $\check{\kappa} \$\R^3$ is a real-linear subspace of $\HH^2$, it has an inner product and norm as in \refsec{inner_product_norm_H2}. In fact, the map $s(\cdot, \kappa)$ of \refeqn{conformal_paravectors_in_TSH} is conformal, and we can compute the scaling factor explicitly. By scaling factor, we mean the following. \begin{defn} \label{Def:scaling_factor} Let $V,W$ be real vector spaces equipped with nondegenerate bilinear symmetric real-valued forms $\langle \cdot, \cdot \rangle_V, \langle \cdot, \cdot \rangle_W$. Let $f \colon V \to W$ be a conformal linear map. The real constant $K$ such that for all $v_1, v_2 \in V$, \[ \langle f(v_1), f(v_2) \rangle_W = K \langle v_1, v_2 \rangle_V, \] is the \emph{scaling factor} of $f$. \end{defn} Note this definition allows for bilinear forms of arbitrary signature; $f$ above scales lengths in the usual sense by $\sqrt{|K|}$. The bilinear forms above induce real-valued norms $Q_V, Q_W$ defined by $Q_V (v) = \langle v,v \rangle$ and $Q_W (w) = \langle w,w \rangle$, which may take negative values. By the polarisation identity, $f$ has scaling factor $K$ iff for all $v \in V$, $Q_W (f(v)) = K Q_V (v)$. \begin{lem} \label{Lem:paravector_first_conformal} For any $\kappa \in \HH$, the map $s(\cdot, \kappa)$ of \refeqn{conformal_paravectors_in_TSH} is conformal with scaling factor $|\kappa|^2$. \end{lem} Since $s(\cdot, \kappa)$ sends $v \in \$\R^3$ to $s(v, \kappa) = s_v (\kappa) = \check{\kappa} v$, this means that for all $v, w \in \$\R^3$, \[ \langle \check{\kappa} v, \check{\kappa} w \rangle = |\kappa|^2 \; v \cdot w. \] \begin{proof} From \reflem{quaternion_inner_product_mult}(ii) we have $\langle \check{\kappa} v, \check{\kappa} w \rangle = \left| \check{\kappa} \right|^2 \; \langle v, w \rangle$. The result then follows from $|\check{\kappa}| = |\kappa|$ and $\langle v, w \rangle = v \cdot w$. \end{proof} \subsection{Conditions for Clifford matrices} \label{Sec:Clifford_conditions} We defined a group of Clifford matrices $SL_2\$$ in \refdef{SL2H}, with numerous conditions. This is a shortened notation for the group $SL_2\$\Gamma_3$, one of the family $SL_2\$\Gamma_n$ in \refeqn{Vahlen_general}. We now discuss these groups in detail, following arguments of \cite[sec. 2]{Ahlfors_Mobius85} . Note that any matrix satisfying \refdef{SL2H} has spinor columns: for the first column, $a^* c \in \$\R^3$, and moreover $(a,c) \neq (0,0)$, since $a=c=0$ would imply $a^* d - c^* b = 0$; a similar argument applies to the second column. Therefore, consider quaternionic matrices $A$ satisfying the following minimal requirements: $A$ has spinor columns, and pseudo-determinant $1$, i.e. \begin{equation} \label{Eqn:minimal_matrix} A = \begin{pmatrix} a & b \\ c & d \end{pmatrix}, \quad \text{such that} \quad (a,c), (b,d) \in S\HH \quad \text{and} \quad \pdet A = a^* d - c^* b = 1. \end{equation} We will show that such a matrix in fact satisfies the maximal requirements of \refdef{SL2H}. \begin{lem} \label{Lem:Clifford_matrix_1} For $A$ as in \refeqn{minimal_matrix}, all of $a b^*, cd^*, c^* a, d^* b, ba^*, dc^*, a^* c, b^* d $ lie in $\$ \R^3$. \end{lem} In other words, matrices satisfying \refeqn{minimal_matrix} satisfy \refeqn{Vahlen_conditions_1}. This is equivalent to saying that $(a^*,b^*)$ and $(c^*,d^*)$ lie in $S\HH$. \begin{proof} As the columns of $A$ lie in $S\HH$, we have $a^* c, b^* d \in \$\R^3$ and, by \reflem{spinor_condition}, $a \overline{c}, b \overline{d} \in \$\R^3$ also. We then have $(b \overline{d})^* = d' b^* \in \$\R^3$. Since $\pdet A = 1$ we have $(a^* d - c^* b)' = 1$, i.e. $\overline{a} d' - \overline{c} b' = 1$. Thus \[ a b^* = a (\overline{a} d' - \overline{c} b' ) b^* = |a|^2 d' b^* - |b|^2 a \overline{c} \in \$\R^3. \] Similarly we have $\overline{(a \overline{c})} = c \overline{a}$ and $(b \overline{d})' = b' d^*$ in $\$\R^3$, so \[ cd^* = c (\overline{a} d' - \overline{c} b') d^* = |d|^2 c \overline{a} - |c|^2 b' d^* \in \$\R^3. \] Now as $a^* c$, $b^* d$, $ab^*$ and $cd^*$ are all in $\$\R^3$, so too are $c^* a = (a^* c)^*$, $d^* b = (b^* d)^*$, $ba^* = (ab^*)^*$, and $dc^* = (cd^*)^*$. \end{proof} \begin{lem} \label{Lem:Clifford_matrix_2} For $A$ as in \refeqn{minimal_matrix}, then $ad^* - bc^* = da^* - cb^* = d^* a - b^* c = 1$. \end{lem} In other words, matrices satisfying \refeqn{minimal_matrix} satisfy \refeqn{Vahlen_conditions_2}. \begin{proof} We first show $ad^* - bc^* = 1$. As $(b,d) \in S\HH$ we have $b \overline{d} \in \$\R^3$, so $b \overline{d} = (b \overline{d})^* = d' b^*$. And by \reflem{Clifford_matrix_1} we have $cd^* \in \$\R^3$ so $cd^* = (cd^*)^* = dc^*$. Thus \begin{equation} \label{Eqn:Ahlfors_adapted_1} |d|^2 ( a^* d - c^* b) = d (a^* d - c^* b) \overline{d} = |d|^2 d a^* - c d^* d' b^* = |d|^2 (da^* - c b^*). \end{equation} Similarly, using $cd^* = (cd^*)^* = dc^*$ and $a \overline{c} = (a \overline{c})^* = c' a^* \in \$\R^3$, we obtain \begin{equation} \label{Eqn:Ahlfors_adapted_2} |c|^2 ( a^* d - c^* b ) = c' ( a^* d - c^* b ) c^* = a \overline{c} c d^* - |c|^2 b c^* = |c|^2 ( ad^* - bc^* ). \end{equation} Now, since $a^* d - c^* b = 1$, $c$ and $d$ cannot both be zero. If $d \neq 0$ then \refeqn{Ahlfors_adapted_1} shows $da^* - cb^* = 1$, and taking $*$, we obtain $ad^* - bc^* = 1$; if $c \neq 0$ then \refeqn{Ahlfors_adapted_2} shows $ad^* - bc^* = 1$. Taking $*$ of $a^* d - c^* b = ad^* - bc^* = 1$ then yields $d^* a - b^* c = da^* - cb^* = 1$. \end{proof} We also show that all prospective definitions agree. \begin{lem} Let $A$ be a quaternionic $2 \times 2$ matrix. The following are equivalent. \begin{enumerate} \item $A$ satisfies \refdef{SL2H}. \item $A$ satisfies \refeqn{Vahlen_general} with $n=3$. \item $A$ satisfies \refeqn{minimal_matrix}. \end{enumerate} \end{lem} \begin{proof} Clearly (i) implies (iii), and \reflem{Clifford_matrix_1} and \reflem{Clifford_matrix_2} show (iii) implies (i). If $A$ satisfies \refeqn{Vahlen_general} with $n=3$ then the conditions $\overline{a} b, b \overline{d}, \overline{d} c, c \overline{a} \in \$\R^3$ imply that the columns $(a,c), (b,d) \in \HH$. We also have $a^* b' = (\overline{a} b)' \in \$\R^3$, so $(a^*, b^*) \in S\HH$. Similarly $c^* d' = (\overline{d} c)^* \in \$\R^3$, so $(c^*, d^*) \in S\HH$. Thus all the conditions of \refeqn{Vahlen_conditions_1} are satisfied. From \refeqn{Vahlen_general} we also have $ad^* - bc^* = 1$ and, taking $*$, $da^* - cb^* = 1$. Again we cannot have both $c$ and $d$ zero, so applying \refeqn{Ahlfors_adapted_1} if $d \neq 0$, and \refeqn{Ahlfors_adapted_2} if $c \neq 0$ as above, we conclude $a^* d - c^* b = 1$ and, taking $*$, $d^* a - b^* c = 1$. Thus all the conditions of \refeqn{Vahlen_conditions_2}, hence also \refdef{SL2H}, are satisfied. Conversely, if $A$ satisfies \refdef{SL2H} then the columns are spinors, as are $(a^*, b^*), (c^*, d^*)$, and $ad^* - bc^* = 1$, so \refeqn{Vahlen_general} is satisfied. \end{proof} Thus $SL_2\$ = SL_2\$\Gamma_3$. Preferring the less cumbersome notation, we henceforth simply write $SL_2\$$. \subsection{Properties of Clifford matrices} \label{Sec:Clifford_properties} Applying the general theory discussed in \refsec{paravector_Mobius} to the case $(p,q) = (0,2)$, and the paravectors $\$\R^3$ in $\Cl(\R^{0,2}) \cong \HH$, we have that $SL_2\$$ is a group, and that elements of $SL_2\$$ yield well defined M\"{o}bius transformations of $\$\R^3 \cup \{\infty\}$, given for $v \in \$\R^3$ by \begin{equation} \label{Eqn:Mobius_from_Clifford_for_quaternions} \begin{pmatrix} a & b \\ c & d \end{pmatrix} \quad \rightsquigarrow \quad v \mapsto (av+b)(cv+d)^{-1}. \end{equation} The only matrices in $SL_2\$$ yielding the identity M\"{o}bius transformation are $\pm 1$; we write $PSL_2\$ = SL_2\$/\{\pm 1\}$. Then $SL_2\$$ is the double cover of the group of M\"{o}bius transformations of $\$\R^3 \cup \{\infty\}$, which is also $\Isom^+ \hyp^4$. We have $SL_2\$ \cong \Isom^S \hyp^4$ and $PSL_2\$ \cong \Isom^+ \hyp^4$. As $\Isom^+ \hyp^4$ is diffeomorphic to $\R^4 \times SO(4)$, and $SL_2\$$, as its spin/universal double cover, we have diffeomorphisms $SL_2\$\Gamma \cong \R^4 \times \Spin(4) \cong \R^4 \times S^3 \times S^3$ and $PSL_2\Gamma \cong \R^4 \times SO(4)$. We now collect some elementary facts about Clifford matrices. First, we have some immediate observations, which also appear in the literature, e.g. \cite{Ahlfors_Mobius85, Ahlfors_Clifford85, Ahlfors_Mobius_86, Ahlfors_84, Ahlfors_fixedpoints_85, Kellerhals01, Waterman_93}. \begin{lem} \ \label{Lem:elementary_vahlen_properties} \begin{enumerate} \item $SL_2 \C \subset SL_2\$$. \item The diagonal matrices in $SL_2\$$ are precisely those of the form \[ \begin{pmatrix} a & 0 \\ 0 & a^{-1*} \end{pmatrix}, \quad a \in \HH^\times, \] which correspond to the M\"{o}bius transformations $v\mapsto ava^* = \sigma(a)(v)$ over $a \in \HH^\times$. \item The inverse of a Clifford matrix is given by \[ \begin{pmatrix} a & b \\ c & d \end{pmatrix}^{-1} = \begin{pmatrix} d^* & -b^* \\ -c^* & a^* \end{pmatrix}. \] \end{enumerate} \qed \end{lem} Although the pseudo-determinant is not in general multiplicative, it is preserved by $SL_2\$$. \begin{lem} \label{Lem:pdet_preserved} Let $M$ be a $2 \times 2$ quaternion matrix, and \[ A = \begin{pmatrix} a & b \\ c & d \end{pmatrix} \in SL_2\$. \quad \text{Then } \pdet(AM) = \pdet (M). \] \end{lem} \begin{proof} Writing $M$ with columns $(\xi_1, \eta_1), (\xi_2, \eta_2) \in \HH^2$, we have \[ AM = \begin{pmatrix} a & b \\ c & d \end{pmatrix} \begin{pmatrix} \xi_1 & \xi_2 \\ \eta_1 & \eta_2 \end{pmatrix} = \begin{pmatrix} a \xi_1 + b \eta_1 & a \xi_2 + b \eta_2 \\ c \xi_1 + d \eta_1 & c \xi_2 + d \eta_2 \end{pmatrix} \] which has pseudo-determinant \begin{align*} \left( a \xi_1 + b \eta_1 \right)^* \left( c \xi_2 + d \eta_2 \right) &- \left( c \xi_1 + d \eta_1 \right)^* \left( a \xi_2 + b \eta_2 \right) = \left( \xi_1^* a^* + \eta_1^* b^* \right) \left( c \xi_2 + d \eta_2 \right) - \left( \xi_1^* c^* + \eta_1^* d^* \right) \left( a \xi_2 + b \eta_2 \right) \\ &= \xi_1^* \left( a^* c - c^* a \right) \xi_2 + \xi_1^* \left( a^* d - c^* b \right) \eta_2 + \eta_1^* \left( b^* c - d^* a \right) \xi_2 + \eta_1^* \left( b^* c - d^* b \right) \eta_2. \end{align*} Now as $A \in SL_2\$$ we have $a^* c, b^* d \in \$\R^3$ so $a^* c = c^* a$ and $b^* d = d^* b$; we also have $a^* d - c^* b = d^* a - b^* c = 1$. Hence the pseudo-determinant is $\xi_1^* \eta_2 - \eta_1^* \xi_2 = \pdet M$. \end{proof} Any spinor $\kappa$ can be a column of a Clifford matrix; the complementary spinor $\check{\kappa}$ helps us find one. \begin{lem} \label{Lem:Vahlen_with_arbitrary_column} For any $\kappa \in S\HH$, $(\kappa, -\frac{\check{\kappa}}{|\kappa|^2}) \in SL_2\$$. \end{lem} Here $(\kappa_1, \kappa_2)$ denotes the matrix with columns $\kappa_1, \kappa_2$, as in \refsec{bracket}. \begin{proof} It is sufficient to show the columns are spinors and $\pdet = 1$. These follow from \reflem{complementary_spinor_facts}(i) and (iv). \end{proof} \subsection{Parabolic translation matrices} \label{Sec:parabolic_clifford} We will need to consider those Clifford matrices which, when regarded as M\"{o}bius transformations and hyperbolic isometries in the upper half space model, translate along horospheres. M\"{o}bius transformations of various types, including in 4 and higher dimensions and involving quaternions, have been studied in numerous papers of Ahlfors, Cao, Foreman, Gongopadhyay, Kellerhals, Parker, Short, Waterman, and others, e.g. \cite{Ahlfors_Clifford85, Gongopadhyay_12, Gongopadhyay_Kulkarni_09, Ahlfors_fixedpoints_85, Kellerhals01, Cao_07, CPW_04, Cao_Waterman_98, Foreman_04, Parker_Short_09, Waterman_93}. When $n=2$ or $3$, the nontrivial isometries in $\Isom^+ \hyp^n$ translating along horospheres are precisely those with a single fixed point at infinity (and no fixed points in $\hyp^n$), also known as \emph{parabolic} isometries. However in $\hyp^4$, there are isometries which have a single fixed point at infinity, and no fixed points in $\hyp^4$, but which do not translate along horospheres. For example, the isometry of $\hyp^4$ given in the upper half space model by the M\"{o}bius transformation $z \mapsto e^{i \theta} z e^{i \theta} +j$ ($0< \theta < \pi/2$ say) has the unique fixed point $\infty$, and preserves all the horospheres about $\infty$, on which it acts as screw motions. It is standard in the literature that a M\"{o}bius transformation is \emph{parabolic} if it has a single fixed point in $\overline{\hyp^n} = \hyp^n \cup \partial \hyp^n$, which is necessarily in $\partial \hyp^n$. Following the terminology of \cite{Gongopadhyay_12, Kellerhals01, Waterman_93}, we consider a smaller set of \emph{translations}, which we define below via several equivalent characterisations. Waterman \cite{Waterman_93} calls them \emph{strictly parabolic} and Cao--Parker--Wang \cite{CPW_04} call them \emph{simple parabolics}. Gongopadhyay \cite{Gongopadhyay_12} defines a notion of \emph{$k$-rotary parabolic} isometries, as those with $k$ rotation angles (see also \cite{Gongopadhyay_Kulkarni_09}); the $0$-rotary parabolics are translations. \begin{lem} \label{Lem:parabolic_conditions} For a matrix $A \in SL_2\$$, the following are equivalent. \begin{enumerate} \item $A \neq 1$ and $(A-1)^2 = 0$. \item $A$ is conjugate to $P_0 = \begin{pmatrix} 1 & 1 \\ 0 & 1 \end{pmatrix}$. \item $A = \begin{pmatrix} 1-ac^* & aa^* \\ -cc^* & 1+ca^* \end{pmatrix}$ for some $(a,c) \in S\HH$. \end{enumerate} \end{lem} \begin{proof} We show equivalence of (ii) and (iii) by a calculation. Considering a matrix \[ B = \begin{pmatrix} a & b \\ c & d \end{pmatrix} \in SL_2\$, \] we compute \begin{align} \label{Eqn:parabolic_calculation} B \begin{pmatrix} 1 & 1 \\ 0 & 1 \end{pmatrix} B^{-1} &= \begin{pmatrix} ad^* - bc^* - ac^* & -ab^* + ba^* + aa^* \\ cd^* - dc^* - cc^* & -cb^* + da^* + ca^* \end{pmatrix} = \begin{pmatrix} 1 - ac^* & aa^* \\ -cc^* & 1 + ca^* \end{pmatrix} \end{align} since $ad^* - bc^* = da^* - cb^* = 1$, and $ab^*, cd^* \in \$\R^3$ so $ab^* =ba^*$ and $cd^* = dc^*$. This shows (ii) implies (iii). If $A$ satisfies (iii), then by \reflem{Vahlen_with_arbitrary_column} there exists $B \in SL_2\$$ with $(a,c)$ as its first column, and then \refeqn{parabolic_calculation} shows $A$ satisfies (ii). Now a matrix $A \in SL_2\$$ satisfies (i) iff any conjugate of $A$ satisfies (i). Clearly the matrix $P_0$ satisfies (i), so (ii) implies (i). It is now sufficient to show (i) implies (ii). So, suppose $A = \begin{pmatrix} \alpha & \beta \\ \gamma & \delta \end{pmatrix}$ satisfies (i). We claim that $A$ is conjugate to an upper or lower triangular matrix with both diagonal entries $1$. If $\gamma = 0$, then $A \in SL_2\$$ tells us $\delta = \alpha^{*-1}$. The top left entry of $(A-1)^2 = 0$ is $(\alpha - 1)^2 = 0$, so $\alpha = \delta = 1$, satisfying the claim. If $\gamma \neq 0$, then following \cite[sec. 7]{Ahlfors_Clifford85} or \cite[sec. 3]{Ahlfors_fixedpoints_85}, $A$ has a conjugate in the normal form \[ N = \begin{pmatrix} s \gamma & s \gamma s - \gamma^{*-1} \\ \gamma & \gamma s \end{pmatrix} \quad \text{where } s \in \$\R^3. \] Then we can calculate that the lower left entry of \begin{align*} 0 = (N-1)^2 &= \begin{pmatrix} s \gamma - 1 & s \gamma s - \gamma^{*-1} \\ \gamma & \gamma s - 1 \end{pmatrix}^2 \end{align*} is $\gamma(s \gamma - 1) + (\gamma s - 1) \gamma = 2(\gamma s \gamma - \gamma)$, so $\gamma s \gamma = \gamma$, hence as $\gamma \neq 0$ we have $s = \gamma^{-1}$. Then $\gamma$ is also a paravector. So $N$ has both diagonal entries $1$ and upper right entry $0$. This proves the claim. Thus $A$ is conjugate to a matrix $N$ of the form \[ N = \begin{pmatrix} 1 & \beta \\ 0 & 1 \end{pmatrix} \quad \text{or} \quad \begin{pmatrix} 1 & 0 \\ \gamma & 1 \end{pmatrix}. \] Now by \reflem{paravector_square_root}, there is a quaternion (in fact, a paravector) $v$ such that $vv^* = \beta$ or $-\gamma$ respectively, so $N$ takes the form (iii) with $(a,c) = (v,0)$ or $(0,v)$, which is a spinor. By equivalence of (ii) and (iii), $N$ is conjugate to $P_0$. Hence $A$ is conjugate to $P_0$, satisfying (ii). \end{proof} \begin{defn} \label{Def:parabolic} We call a matrix $A \in SL_2\$$ that satisfies any, hence all, of the conditions in \reflem{parabolic_conditions} a \emph{parabolic translation}. The set of parabolic translation matrices is denoted $\P$. \end{defn} Gongopadhyay and Waterman \cite{Gongopadhyay_12, Waterman_93} use the conjugacy condition (ii) of \reflem{parabolic_conditions} for their definitions. Polynomials satisfied by M\"{o}bius transformations, as in (i), were studied in \cite{Gongopadhyay_12, Foreman_04}. Note it follows from the end of the proof of \reflem{parabolic_conditions} that any matrix of the form \begin{equation} \label{Eqn:upper_triangular_unipotent_parabolic} \begin{pmatrix} 1 & v \\ 0 & 1 \end{pmatrix} \quad \text{or} \quad \begin{pmatrix} 1 & 0 \\ v & 1 \end{pmatrix} \quad \text{with $v \in \$\R^3$ is parabolic.} \end{equation} The description given in \reflem{parabolic_conditions} demonstrates several properties of parabolic translation matrices, which we state here. Unlike $SL_2\R$ and $SL_2\C$, they do not necessarily have trace $2$, but have several similar-looking properties. See the work of Cao and Waterman \cite{Cao_Waterman_98, Waterman_93} for further results. In particular, Waterman \cite{Waterman_93} shows that the real part of $a+d^*$ is conjugation invariant, and gives a geometric interpretation. \begin{lem} \label{Lem:parabolic_facts} Let $A = \begin{pmatrix} a & b \\ c & d \end{pmatrix} \in SL_2\$$ be a parabolic translation. \begin{enumerate} \item $a + d^* = 2$. \item The off-diagonal entries $b,c$ are paravectors. \item $ab+bd = 2b$ and $dc+ca = 2c$. \item For any $B \in SL_2\$$, $ABA^{-1}$ is also a parabolic translation. \end{enumerate} \end{lem} \begin{proof} From the form in \reflem{parabolic_conditions}(iii), we immediately deduce (i) and (ii). The off-diagonal entries of the equation $(A-1)^2 =0$ of \reflem{parabolic_conditions}(i) are precisely the equations of (iii). By \reflem{parabolic_conditions}(ii), $\P$ is the conjugacy class of $P_0$, hence invariant under conjugation, giving (iv). \end{proof} \subsection{Action of $SL_2$ on spinors} \label{Sec:SL2_on_spinors} We now consider $SL_2\$$ actions on the spaces $S\HH \subset \HH^2$, which is simply by matrix-vector multiplication. We denote all actions of $SL_2\$$ by a dot: for $A \in SL_2\$$ and $\kappa \in \HH^2$, $A.\kappa = A\kappa$. This action preserves the bracket on $\HH^2$: given $\kappa_1, \kappa_2 \in \HH^2$, denoting by $(\kappa_1, \kappa_2)$ the $2 \times 2$ matrix with $\kappa_1, \kappa_2$ as columns as in \refsec{bracket}, by \reflem{pdet_preserved} we have \begin{equation} \label{Eqn:action_preserves_bracket} \{ A.\kappa_1, A.\kappa_2 \} = \pdet(A\kappa_1, A \kappa_2) = \pdet(A(\kappa_1, \kappa_2)) = \pdet(\kappa_1,\kappa_2) = \{ \kappa_1, \kappa_2\}. \end{equation} Restricting to $S\HH \subset \HH^2$, the following lemma ensures that there is a well-defined action on $S\HH$. Since spinors are those $(\xi, \eta)$ whose quotient $\xi \eta^{-1}$ is a paravector, this is essentially the same result as that M\"{o}bius transformations from $SL_2\$$ preserve $\$\R^3 \cup \{\infty\}$, proved by Ahlfors in \cite{Ahlfors_Mobius85}, going back to Maass \cite{Maass_49} and Vahlen \cite{Vahlen_1902}. \begin{lem} \label{Lem:action_preserves_spinors} If $\kappa \in S\HH$ and $A \in SL_2\$$ then $A \kappa \in S\HH$. Moreover, $SL_2\$$ acts transitively on $S\HH$. \end{lem} \begin{proof} If $\kappa = (\xi, \eta) \in S\HH$ then $\xi^* \eta \in \$\R^3$. Letting $A$ have entries $a,b,c,d$ as in \refeqn{minimal_matrix}, we have $A.\kappa = (a \xi + b \eta, c \xi + d \eta)$. Then \begin{align*} (a \xi + b \eta)^* (c \xi + d \eta) &= (\xi^* a^* + \eta^* b^*) (c \xi + d \eta) \\ &= \xi^* a^* c \xi + \xi^* a^* d \eta + \eta^* b^* c \xi + \eta^* b^* d \eta \\ &= \sigma(\xi^*) (a^* c ) + \xi^* (c^* b + 1) \eta + \eta^* b^* c \xi + \sigma(\eta^*)(b^* d) \\ &= \sigma(\xi^*)(a^* c) + \xi^* c^* b \eta + [\xi^* c^* b \eta]^* + \xi^* \eta + \sigma(\eta^*)(b^* d) \end{align*} In line 3 we used $a^*d - c^* b = 1$. Now $a^* c, b^* d \in \$\R^3$, so $\sigma(\xi^*)(a^* c), \sigma(\eta^*)(b^* d) \in \$\R^3$. The sum of a term and its $*$-conjugate lies in $\$\R^3$, so the above expression lies in $\$\R^3$, hence $A.\kappa \in S\HH$. By \reflem{Vahlen_with_arbitrary_column} above, for any two spinors $\kappa_1, \kappa_2$, there are matrices $A_1, A_2 \in SL_2\$$ whose first columns are $\kappa_1$ and $\kappa_2$ respectively. Letting $\kappa_0 = (1,0)$, we have $A_1.\kappa_0 = \kappa_1$ and $A_2.\kappa_0 = \kappa_2$. Thus $A_2 A_1^{-1}$ sends $\kappa_1$ to $\kappa_2$. \end{proof} Parabolic translation matrices can be characterised by their action on spinors, as follows. See \cite{Zhang_97} for a review of eigenvalues of quaternionic matrices. \begin{lem} \label{Lem:parabolic_on_spinors} Let $A \in SL_2\$$. \begin{enumerate} \item If $A \in \P$ and $A \kappa = \kappa x$ for some $\kappa \in S\HH$ and $x \in \HH$ then $x = 1$. \item $A \in \P$ iff $A$ is not the identity and there exists some $\kappa \in S\HH$ such that $A \kappa = \kappa$. \end{enumerate} \end{lem} Thus, the only right eigenvalue of a parabolic translation matrix is $1$; and parabolic matrices are precisely the non-identity matrices in $SL_2\$$ with $1$ as a right eigenvalue. \begin{proof} First we consider $P_0$. Note that $P_0 \kappa_0 = \kappa_0$, where $\kappa_0 = (1,0)$. Moreover, if $P_0 \kappa = \kappa x$ for some $\kappa = (\xi, \eta) \in S\HH$ and $x \in \HH$, then $\kappa = \kappa_0 y$ for some $y \in \HH^\times$, and $x=1$. To see this, $P_0 \kappa = \kappa x $ gives $(\xi + \eta, \eta) = (\xi x, \eta x)$. If $\eta \neq 0$ then we obtain $x=1$ so $\xi + \eta = \xi$, hence $\eta = 0$, a contradiction. Thus $\eta = 0$, so $\xi \neq 0$, and $\kappa_0 = (\xi, 0) = \kappa_0 \xi$ where $\xi \in \HH^\times$. Now a general $A \in \P$ is given as $A=B P_0 B^{-1}$ where $B \in SL_2\$$. Letting $\kappa_1 = B^{-1} \kappa$, we have $P_0 \kappa_1 = \kappa_1 x$. So the above gives $x = 1$ (and in fact $\kappa_1 = \kappa_0 y$ for some $y \in \HH^\times$), proving (i). Moreover, for $A \in \P$ written as $A = B P_0 B^{-1}$, letting $\kappa = B \kappa_0$, from $P_0 \kappa_0 = \kappa_0$ we have $A \kappa = \kappa$, giving the forwards direction of (ii). For the converse, suppose $A \in SL_2\$$ is not the identity and $A \kappa = \kappa$. First suppose $\kappa = \kappa_0$. Letting \[ A = \begin{pmatrix} a & b \\ c & d \end{pmatrix}, \] $A \kappa_0 = \kappa_0$ implies that $a = 1$ and $c=0$. Then $d^* a - b^* c = 0$ implies $d=1$, and $(b,d) = (b,1) \in S\HH$ implies $b \in \$\R^3$. So \[ A = \begin{pmatrix} 1 & b \\ 0 & 1 \end{pmatrix} \quad \text{where} \quad b \in \$\R^3, \] which lies in $\P$ by \refeqn{upper_triangular_unipotent_parabolic}. Now suppose $A \kappa = \kappa$ for a general $\kappa \in S\HH$ and $A \neq 1$. Since $SL_2\$$ acts transitively on $S\HH$ (\reflem{action_preserves_spinors}), there exists $B \in SL_2\$$ be such that $B \kappa = \kappa_0$. Then $BAB^{-1} \kappa_0 = \kappa_0$, and moreover $BAB^{-1} \neq 1$. So by the preceding paragraph $BAB^{-1} \in \P$. As $\P$ is closed under conjugation (\reflem{parabolic_facts}(iv)) then $A \in \P$. \end{proof} \subsection{Action of $SL_2$ on the tangent space to spinors} \label{Sec:action_SL2_tangent_spinors} The action of $A \in SL_2\$$ on $\S\HH$ is linear, so we obtain a map on tangent spaces $T_\kappa S\HH \To A_{A \kappa} S\HH$, which is just a restriction of the linear isomorphism $\HH^2 \To \HH^2$ given by $\kappa \mapsto A \kappa$. Moreover, this map respects right multiplication: for any $x \in \HH$ we have $A.(\kappa x) = A \kappa x$. Using \reflem{TSH}, we have orthogonal decompositions $T_\kappa S\HH = \kappa \R \oplus \kappa \II \oplus \check{\kappa} \$\R^3$ and $T_{A \kappa} S\HH = A \kappa \R \oplus A \kappa \II \oplus \widecheck{(A\kappa)}\$\R^3$. The action of $A$ preserves the first two summands, sending $\kappa \R \To A \kappa \R$ and $\kappa \II \To A \kappa \II$. However, its action on the third summand $\check{\kappa} \$\R^3$, the copy of the paravectors given by the section $s$ of \refdef{Z}, is more subtle. Nonetheless, it follows from the preservation of the first two summands that for all $\kappa \in S\HH$, the action of $A$ yields an $\R$-linear map of real-3-dimensional vector spaces \begin{equation} \label{Eqn:A_on_quotient_spinors} \frac{T_\kappa S\HH}{\kappa \HH} \To \frac{T_{A\kappa} S\HH}{A \kappa \HH}, \end{equation} which by \reflem{TSH} are isomorphic as vector spaces to $\check{\kappa} \$\R^3$ and $\widecheck{(A \kappa)} \$\R^3$ respectively (induced by inclusion or projection). It will be useful later in \refsec{SL2_on_paravectors_etc} to understand this map. These quotients form a vector bundle over $S\HH$, and as each quotient has an ordered basis and orientation given by the equivalence classes of $\check{\kappa}1, \check{\kappa}i, \check{\kappa}j$, we obtain a trivialisation of this bundle. Under their natural isomorphism, the orientations on $T_\kappa S\HH / \kappa \HH$ and $\check{\kappa}\$\R^3$ (\refsec{space_of_spinors}) agree. The points in $T_\kappa S\HH / \kappa \HH$ are affine 4-planes in $T_\kappa S\HH$, of the form $\widecheck{\kappa} v + \kappa \HH = s_v (\kappa) + \kappa \HH$, over all $v \in \$\R^3$. These affine 4-planes have a simple interpretation in terms of the bracket $\{ \cdot, \cdot \}$. \begin{lem} Let $\kappa \in S\HH$ and $\nu \in \HH^2$. Then $\nu \in s_v (\kappa) + \kappa \HH$ iff $\{ \kappa, \nu \} = - v |\kappa|^2$. \end{lem} \begin{proof} By \refeqn{inner_product_with_sv}, $\{ \kappa, s_v (\kappa) \} = - v |\kappa|^2$, and by \reflem{nondegeneracy_of_spinor_form}, $\{ \kappa, \kappa x \} = 0$. Thus by bilinearity of the bracket, $\nu \in s_v (\kappa) + \kappa \HH$ implies $\{\kappa, \nu \} = - v |\kappa|^2$. Conversely, if $\{ \kappa, \nu \} = - v |\kappa|^2$ then $\{\kappa, \nu - s_v (\kappa) \} = 0$ so by \reflem{nondegeneracy_of_spinor_form} $\nu - s_v(\kappa) \in \kappa \HH$. \end{proof} We can then describe the action of $A \in SL_2\$$ on these affine 4-planes, or equivalently, on quotients $T_\kappa S\HH / \kappa \HH$. \begin{lem} \label{Lem:A_on_quotient_spinors} Let $\kappa_0 \in S\HH$, $A \in SL_2\$$ and $\kappa_1 = A \kappa_0$. For each $v \in \$\R^3$, the action of $A$ restricts to a map \[ s_v (\kappa_0) |\kappa_0|^{-2} + \kappa_0 \HH \To s_v (\kappa_1) |\kappa_1|^{-2} + \kappa_1 \HH \] which are affine 4-planes in $T_{\kappa_0} S\HH$ and $T_{\kappa_1} S\HH$ respectively. \end{lem} \begin{proof} By the previous lemma, the first affine 4-plane is precisely the set of $\nu \in T_{\kappa_0} S\HH$ such that $\{\kappa_0, \nu\} = - v$, and the second 4-plane is precisely the set of $\nu \in T_{\kappa_1} S\HH$ such that $\{\kappa_1, \nu \} = -v$. By \refeqn{action_preserves_bracket}, for any $\nu \in T_{\kappa_0} S\HH$, we have $\{\kappa, \nu\} = \{ A \kappa, A \nu \}$. So if $\{ \kappa_0, \nu \} = -v$ then $\{ \kappa_1, A \nu\} = -v$. \end{proof} We note that this statement can also be proved explicitly by calculating the component of $A \kappa \in T_{\kappa_1} S\HH = \kappa_1 \HH \oplus \check{\kappa}_1 \$\R^3$ in the the second summand using \reflem{explicit_decomposition_in_TSH}. It will be useful to introduce an inner product on the quotients $T_\kappa S\HH / \kappa \HH$. As we have the orthogonal decomposition and isomorphism \[ T_\kappa S\HH \cong \kappa \HH \oplus \check{\kappa}\$\R^3, \quad \frac{T_\kappa S\HH}{\kappa \HH} \cong \check{\kappa}\$\R^3, \] we could define a norm on the quotient by minimising over equivalence classes in $T_\kappa S\HH$, and then define an inner product by the polarisation identity; or we could define an inner product on the quotient via its isomorphism with the subspace $\check{\kappa}\$\R^3$. As the decomposition is orthogonal and all spaces are positive definite, these approaches yield the same result, agreeing with the following definition. \begin{defn} \label{Def:inner_product_on_spinor_quotient} Let $\kappa \in S\HH$. We define a positive definite inner product \[ \left( \cdot, \cdot \right) \colon \frac{T_\kappa S\HH}{\kappa \HH} \times \frac{T_\kappa S\HH}{\kappa \HH} \To \R \] as follows. Each element of $T_\kappa S\HH / \kappa \HH$ is of the form $\check{\kappa} v + \kappa \HH$ for some unique $v \in \$\R^3$, and contains a unique representative $\check{\kappa} v \in \check{\kappa}\$\R^3 \subset T_\kappa S\HH$. Then for $v,w \in \$\R^3$ we define \[ \left( \check{\kappa} v + \kappa \HH, \; \check{\kappa} w + \kappa \HH \right) = \langle \check{\kappa} v, \check{\kappa} w \rangle, \] where $\langle \cdot, \cdot \rangle$ is the inner product in $\HH^2$. \end{defn} By \reflem{paravector_first_conformal} we have \[ \left( \check{\kappa} v + \kappa \HH, \; \check{\kappa} w + \kappa \HH \right) = |\kappa|^2 \; v \cdot w. \] With respect to this inner product, the map \refeqn{A_on_quotient_spinors} is conformal, and we now calculate its scaling factor (\refdef{scaling_factor}). \begin{lem} \label{Lem:action_on_spinor_quotients_conformal} Let $\kappa_0 \in S\HH$, $A \in SL_2\$$ and $\kappa_1 = A \kappa_0$. The map \[ \frac{T_{\kappa_0} S\HH}{\kappa_0 \HH} \To \frac{T_{\kappa_1} S\HH}{\kappa_1 \HH}. \] induced by the action of $A$ is an orientation-preserving conformal $\R$-linear isomorphism with scale factor $|\kappa_0|^4/|\kappa_1|^4$. \end{lem} In other words, for $v,w \in \$\R^3$, \[ \left( A \check{\kappa}_0 v + \kappa_1 \HH, A \check{\kappa}_0 w + \kappa_1 \HH \right) = \frac{|\kappa_0|^4}{|\kappa_1|^4} \; \left(\check{\kappa}_0 v + \kappa_0 \HH, \check{\kappa}_0 w + \kappa_0 \HH \right) = \frac{|\kappa_0|^4}{|\kappa_1|^4} \; \left\langle \check{\kappa}_0 v, \check{\kappa}_0 w \right\rangle. \] \begin{proof} By \reflem{A_on_quotient_spinors}, the action of $A$ sends the element represented by $\check{\kappa}_0 v = s_v (\kappa_0)$ to the element represented by $\check{\kappa}_1 v |\kappa_0|^2 |\kappa_1|^{-2} = s_v (\kappa_1) |\kappa_0|^2 |\kappa_1|^{-2} $. So for $v,w \in \$\R^3$ we have \begin{align*} \left( A \check{\kappa}_0 v + \kappa_1 \HH, A \check{\kappa}_0 w + \kappa_1 \HH \right) &= \left\langle \check{\kappa}_1 |\kappa_0|^2 |\kappa_1|^{-2} v, \check{\kappa}_1 |\kappa_0|^2 |\kappa_1|^{-2} w \right\rangle = \frac{|\kappa_0|^4}{|\kappa_1|^{4}} \left\langle \check{\kappa}_1 v, \check{\kappa}_1 w \right\rangle \end{align*} Thus we have the required conformal map. Each quotient $T_\kappa S\HH / \kappa \HH$ has an oriented basis $(s_1 (\kappa), s_i (\kappa), s_j (\kappa))$, and $A$ sends such a basis to another, up to the positive scaling factor. So $A$ is orientation-preserving. \end{proof} \section{From quaternionic spinors to flags} \label{Sec:spinor_to_flag} \subsection{Paravector Hermitian matrices and Minkowski space} \label{Sec:Hermitian_Minkowski} We now define a map from the set $S\HH$ spinors to certain matrices, analogous to $2 \times 2$ Hermitian matrices, as mentioned earlier and generalising \cite{Mathews_Spinors_horospheres}. To begin, we define this set of matrices. \begin{defn} Let $A$ be a $2 \times 2$ quaternionic matrix. \begin{enumerate} \item $A$ is \emph{Hermitian} if $A = \overline{A}^T$. \item $A$ is a \emph{paravector matrix} if its entries are paravectors. \end{enumerate} The set of Hermitian paravector matrices is denoted $\pH$. \end{defn} (A conjugate transpose is often denoted by a $*$, but that notation already being used, we write $\overline{A}^T$.) A matrix in $\pH$ is a real linear combination of the ``quaternionic Pauli matrices" \[ \begin{pmatrix} 1 & 0 \\ 0 & 1 \end{pmatrix}, \quad \begin{pmatrix} 0 & 1 \\ 1 & 0 \end{pmatrix}, \quad \begin{pmatrix} 0 & i \\ -i & 0 \end{pmatrix}, \quad \begin{pmatrix} 0 & j \\ -j & 0 \end{pmatrix}, \quad \begin{pmatrix} 1 & 0 \\ 0 & -1 \end{pmatrix}. \] We can identify paravector Hermitian matrices with points in $(1+4)$-dimensional Minkowski space $\R^{1,4}$. Indeed, both $\pH$ and $\R^{1,4}$ are isomorphic 5-dimensional real vector spaces. We take $\R^{1,4}$ to have coordinates $(T,W,X,Y,Z)$, metric $dT^2 - dW^2 - dX^2 - dY^2 - dZ^2$, denoted $\langle \cdot, \cdot \rangle$, and norm $|p|^2 = \langle p, p \rangle$. (So $|p|^2$ is positive, zero, or negative respectively as $p$ is timelike, lightlike, or spacelike; we leave $|p|$ undefined.) Two vectors $v,w$ in $\R^{1,4}$ are \emph{orthogonal} if $\langle v,w \rangle = 0$. For $v,w$ both timelike or both spacelike, the \emph{angle} $\theta$ between them is formed in the usual way, by $\cos \theta = \langle v, w \rangle / \sqrt{|v|^2 |w|^2}$. We use a similar identification as in \cite{Mathews_Spinors_horospheres}, and as discussed in the introduction (\refsec{intro_spinors_paravectors}). \[ \pH \cong \R^{1,4}, \quad (T,W,X,Y,Z) \leftrightarrow \frac{1}{2} \begin{pmatrix} T+Z & W+Xi+Yj \\ W-Xi-Yj & T-Z \end{pmatrix}. \] For a matrix in $\pH$, as its entries are all paravectors, the determinant $\pdet$ of \refeqn{lambda_pdet} agrees with the usual determinant, since all entries are in $\$\R^3$, hence invariant under $*$. We may also take the trace as usual. For a point $p = (T,W,X,Y,Z) \in \R^{1,4}$ corresponding to $S \in \pH$, we have \begin{equation} \label{Eqn:matrix_R14_correspondences} \Tr S = T, \quad 4 \det S = \langle p, p \rangle = |p|^2. \end{equation} The light cone in $\R^{1,4}$ consists of $p$ with $\langle p,p \rangle = 0$, corresponding to $S$ with $\det S = 0$, and the the future light cone $L^+$ consists of all $x$ additionally satisfying $T>0$, corresponding to $\Tr S > 0$. The tangent space to $L^+$ at $p$ is defined by the equation $\langle x, p \rangle = 0$, so we have $T_p L^+ = p^\perp$. We define the \emph{celestial sphere} $\S^+$ as the projectivisation of the light cone, i.e. the space of lightlike 1-dimensional subspaces $\ell \subset \R^{1,4}$. Intersection of $L^+$ with a 4-plane $T=T_0$ also yields a 3-sphere, $\S^+_{T_0}$, which is naturally diffeomorphic to $\S^+$ via projectivisation, so we call $\S^+_{T_0}$ the \emph{celestial sphere at constant $T$}. All $\S^+_T$ and $\S^+$ are diffeomorphic to $S^3$. If $p \in L^+$ has $T$-coordinate $T_0$ and $\ell = p \R$ then projection yields an isomorphism $T_\ell \S^+ \cong T_p \S^+_{T_0}$. At each point $p \in L^+$, the line $\ell = p \R$ is tangent to $L^+$. The family of celestial spheres at constant $T$, and such lines, provides a product structure $L^+ \cong \S^3 \times \R$. The sphere $\S_T^+$ and line $p \R$ through each point are orthogonal, so we have an orthogonal decomposition of the tangent space \begin{equation} \label{Eqn:TL_decomposition} T_p L^+ = p^\perp = \ell^\perp = p \R \oplus T_p \S_T^+ = \ell \oplus T_p \S_T^+ \end{equation} and hence an isomorphism \begin{equation} \label{Eqn:celestial_sphere_light_cone_quotient} T_p \S^+_T \cong \frac{T_p L^+}{p \R} = \frac{p^\perp}{p \R} = \frac{\ell^\perp}{\ell} \end{equation} sending each vector to its equivalence class. This isomorphism can in fact be regarded as an isometry, as follows. Let $p = (T,W,X,Y,Z) \in L^+$, let $v,w \in T_p L^+ = p^\perp$ and let $\overline{v}, \overline{w}$ be their images in $T_p L^+ / p \R$. Any other representatives $\tilde{v}, \tilde{w} \in T_p L^+$ of $\overline{v}, \overline{w}$ satisfy $\tilde{v} = v+ap$, $\tilde{w} = w+bp$ where $a,b \in \R$, so as $p,v,w \in p^\perp$ we have $\langle \tilde{v}, \tilde{w} \rangle = \langle v+ap, w+bp \rangle = \langle v,w \rangle$. Hence there is a well-defined inner product on each $T_p L^+ / p \R = \ell^\perp / \ell$, induced by the Minkowski inner product. Sending each vector in $T_p L^+$ to its representative in $T_p L^+ / p \R$ yields the same result for the inner product, and hence the isomorphism \refeqn{celestial_sphere_light_cone_quotient} is an isometry. Note that when two points $p_1, p_2 \in L^+$ are real multiples of each other, we have $p_1 \R = p_2 \R = \ell$, where $\ell$ is a lightlike line along $L^+$. Then we have $p_1^\perp = p_2^\perp = \ell^\perp$, so $T_{p_1} L^+ = T_{p_2} L^+ = \ell^\perp$. Similarly, $p_1^\perp / p_1 \R = p_2^\perp / p_2 \R = \ell^\perp / \ell$. Since each celestial sphere $\S^+_T$ is spacelike, the inner product on each $\ell^\perp/\ell$ is negative definite. The lines $\ell$ are the points of $\S^+$, we also have $T_\ell \S^+ \cong \ell^\perp / \ell$ also negative definite. \subsection{Orientations in Minkowski space} \label{Sec:orientations} We will need to consider orientations on various spaces: Minkowski space $\R^{1,4}$, the future light cone $L^+$, each celestial sphere $\S^+_T$ at constant $T$, and quotient spaces from the light cone $T_p L^+ / p \R = \ell^\perp / \ell$. We use the following conventions: \begin{enumerate} \item \emph{Orientations of codimension-1 subspaces.} Given a codimension-1 subspace $W$ of an oriented vector space $V$, a transverse vector $v$ to $W$ induces an orientation on $W$: an ordered basis $B$ for $W$ is positively oriented iff appending $v$ to $B$ yields a positively oriented basis for $V$. The opposite convention is probably more standard (prepending $v$ to an oriented basis for $W$ yields an oriented basis for $V$), but the usefulness of this convention can be seen from the fact that $(1,i,j,k)$ is a standard oriented basis of $\HH$ over $\R$, and $(1,i,j)$ is a standard oriented basis for $\$\R^3$, identified with $\R^3$ via \refeqn{R3_R3}. \item \emph{Orientations of codimension-1 quotient spaces.} Given a 1-dimensional subspace $W = \R w$ of an oriented vector space $V$, spanned by $w$, we obtain a natural orientation on the quotient $V/W$: an ordered basis $B$ for $V/W$ is positively oriented iff representatives of $B$ (arbitrarily chosen, this does not affect the orientation), followed by $w$, yield an oriented basis for $V$. Again, the opposite convention is probably more standard, but by this convention the quotient $\HH / k\R \cong \$\R^3$, obtains its standard orientation from that of $\HH$. \end{enumerate} In Minkowski space $\R^{1,4}$ with points denoted $p = (T,W,X,Y,Z)$ we have unit (i.e. norm $\pm 1$) vector fields $\partial_T, \partial_W, \partial_X, \partial_Y, \partial_Z$ in coordinate directions. We endow $\R^{1,4}$ with the standard orientation, so that this ordered basis is positively oriented. We define two radial vector fields: \[ \partial_r = T \partial_T + W \partial_W + X \partial_X + Y \partial_Y + Z \partial_Z, \quad\text{and} \quad \partial_- = W \partial_W + X \partial_X + Y \partial_Y + Z \partial_Z. \] Thus $\partial_r$ is nonzero at all $p \neq 0$, indeed is the position vector of $p$, and $\partial_-$ is nonzero on the complement of the $T$-axis, and is spacelike, pointing radially outward along 4-planes of constant $T$. Next, we define an orientation on each spacelike 4-plane $\Pi_{T_0}$ given by $T=T_0$. The vector field $\partial_T$ is normal to $\Pi_{T_0}$ and we endow it with the induced orientation, which is the same as the standard $\R^4$ with coordinates $(W,X,Y,Z)$. We then define \emph{two} orientations, \emph{outward} and \emph{inward}, on $\S^+_{T_0}$, which is a codimension-1 submanifold of $\Pi_{T_0}$, bounding a 4-ball in $\Pi_{T_0}$, with outward normal vector field $\partial_-$. Thus we define the \emph{outward} orientation to be the one induced by $\partial_-$, and the \emph{inward} orientation induced by $-\partial_-$. Any basis of a tangent space to $\S^+$ is accordingly either oriented \emph{inward} or \emph{outward}. Explicitly, at $p \in \S^+_{T_0}$, an ordered basis $(B_1, B_2, B_3)$ of $T_p \S^+_{T_0}$ has outward orientation iff $(B_1, B_2, B_3, \partial_-)$ is a positively oriented basis of $\Pi_{T_0}$, iff $(B_1, B_2, B_3, \partial_-, \partial_T)$ is a positively oriented basis of $\R^{1,4}$. Using the isomorphism \refeqn{celestial_sphere_light_cone_quotient} between $T_p \S^+_{T_0}$ and $T_p L^+ / p \R$ for $p \in L^+$, we also obtain inward and outward orientations on each $T_p L^+ / p \R = p^\perp / p \R = \ell^\perp/\ell$ where $\ell = p \R$. Alternatively, $L^+$ also obtains an orientation as a codimension-1 submanifold of $\R^{1,4}$ with transverse vector field $\partial_-$, pointing out of the solid cone formed by future-pointed timelike vectors. At any point $p \in L^+$, the 4-plane $T_p L^+ = p^\perp$ thus obtains an orientation. (Any element of $SO(1,4)^+$ sends $\partial_-$ to a vector also pointing out of the solid cone, so this orientation is Lorentz invariant.) The quotient $T_p L^+ / p \R = p^\perp / p \R$ then obtains an orientation as a quotient. With this orientation, an ordered basis $(\underline{B}_1, \underline{B}_2, \underline{B}_3)$ of $p^\perp / p \R$ is positively oriented iff $(B_1, B_2, B_3, \partial_r)$ forms a positively oriented basis of $p^\perp = T_p L^+$ (where each $B_m \in T_p L^+$ is an arbitrary representative of $\underline{B}_m$), iff $(B_1, B_2, B_3, \partial_r, \partial_-)$ forms a positively oriented basis of $\R^{1,4}$. This corresponds to the inward orientation defined above, because $(\partial_-, \partial_T) \mapsto (\partial_r, \partial_-)$ is orientation reversing. \begin{example} \label{Eg:orientation_at_p0} At $p_0 = (1,0,0,0,1) \in \S^+_1$, we have $\partial_- = \partial_Z$. The ordered basis $(\partial_W, \partial_X, \partial_Y)$ of $T_{p_0} \S^+_1$ is outward oriented, since $(\partial_W, \partial_X, \partial_Y, \partial_Z, \partial_T )$ has positive orientation in $\R^{1,4}$. \end{example} \subsection{From spinors to the light cone} \label{Sec:spinors_to_light_cone} \begin{defn} The map $\phi_1 \colon S\HH \To \pH \cong \R^{1,4}$ is defined by $\phi_1 (\kappa) = \kappa \overline{\kappa}^T$. \end{defn} Thus \begin{equation} \label{Eqn:matrix_for_phi1} \phi_1 (\kappa) = \begin{pmatrix} \xi \\ \eta \end{pmatrix} \begin{pmatrix} \overline{\xi} & \overline{\eta} \end{pmatrix} = \begin{pmatrix} |\xi|^2 & \xi \overline{\eta} \\ \overline{\xi} \eta & |\eta|^2 \end{pmatrix}, \end{equation} just as in the complex case. By definition $\phi_1 (\kappa) \in \pH$; as the trace is positive and determinant zero, so $\phi_1(\kappa) \in L^+$. Letting $p = \phi_1 (\kappa) = (T,W,X,Y,Z)$, comparing the above with \refeqn{R14_matrix}, we have \begin{equation} \label{Eqn:phi1_in_coords} T = |\xi|^2 + |\eta|^2 = |\kappa|^2, \quad W+iX+jY = 2 \xi \overline{\eta}, \quad Z = |\xi|^2 - |\eta|^2. \end{equation} In fact the image of $\phi_1$ is precisely $L^+$. To see this, note that an arbitrary element of $\pH$ with positive trace and zero determinant can be written as \[ \begin{pmatrix} a & re^{u \theta} \\ r e^{-u \theta} & b \end{pmatrix} \] where the off-diagonal elements are written in polar form. Here $a,b,r,\theta \in \R$, $a,b \geq 0$ with at least one of $a,b$ positive, $r \geq 0$, $u \in \$\R^3 \cap \II$ with $|u| = 1$, (if $r=0$ then $u,\theta$ are arbitrary), and $ab = r^2$. Such a matrix can be obtained as the image under $\phi_1$ of, say, $(\xi, \eta) = (\sqrt{a} e^{u \theta}, \sqrt{b})$, where $\sqrt{\cdot}$ denotes positive square root of a positive real. Note that as $re^{u\theta} \in \$\R^3$ we have $\sqrt{a} e^{u \theta} \in \$\R^3$ also, so this $(\xi, \eta)$ satisfies $\xi \overline{\eta} = \sqrt{ab} e^{u \theta} = r e^{u \theta}$, hence lies in $S\HH$. Thus $\phi_1$ maps a 7-dimensional domain (topologically $S^3 \times S^3 \times \R$) to a 4-dimensional domain (topologically $S^3 \times \R$). We next describe the fibres of this map. \begin{lem} \label{Lem:phi1_fibres} Let $\kappa_0 = (\xi_0, \eta_0), \kappa_1 = (\xi_1, \eta_1) \in S\HH$. Then $\phi_1 (\kappa_0) = \phi_1 (\kappa_1)$ iff $\kappa_0 = \kappa_1 \alpha$ for some unit $\alpha \in \HH$. \end{lem} \begin{proof} By definition of $\phi_1$ we have $\phi_1 (\kappa_0 ) = \phi_1 ( \kappa_0 \alpha )$ for any unit $\alpha \in \HH$. Now suppose $\phi_1 (\kappa_0) = \phi_1 (\kappa_1)$. Comparing diagonal entries yields $|\xi_0| = |\xi_1|$ and $|\eta_0| = |\eta_1|$. Both cannot be zero; suppose without loss of generality $|\xi_0| \neq 0$. Then $\xi_0 = \xi_1 \alpha$ for some unit $\alpha \in \HH$. Comparing off-diagonal entries then $\xi_0 \overline{\eta_0} = \xi_1 \alpha \, \overline{\eta_0} = \xi_1 \, \overline{\eta_1}$, so $\alpha \overline{\eta_0} = \overline{\eta_1}$ and hence $\eta_0 \overline{a} = \eta_1$. Since $|\alpha| = 1$, $\alpha^{-1} = \overline{\alpha}$ so $\eta_0 = \eta_1 \alpha$ and $\kappa_0 = \kappa_1 \alpha$. \end{proof} Thus the fibres of $\phi_1$ are of the form $\kappa S^3$ over $\kappa \in S\HH$, where we regard $S^3$ as the unit quaternions. In the complex case, the corresponding map $\phi_1$ is the cone on the Hopf fibration, and the restriction of $\phi_1$ to unit spinors $S^3 \subset \C^2$ is the Hopf fibration $S^3 \To S^2$. In the present case the quaternionic Hopf fibration arises. The quaternionic Hopf fibration is the map $S^7 \To S^4$ with $S^3$ fibres, which sends unit elements $(\xi, \eta) \in \HH^2$ with $|\xi|^2 + |\eta|^2 = 1$ to $\xi \eta^{-1} \in \HH \cup \{\infty\} \cong \R^4 \cup \{\infty\} \cong S^4$. Two points $(\xi_0, \eta_0), (\xi_1, \eta_1) \in S^7$ lie in the same fibre iff $(\xi_0, \eta_0) = (\xi_1, \eta_1)\alpha$ for some unit quaternion $\alpha$, just as for $\phi_1$. The map $\phi_1$ and the Hopf fibration essentially agree on their common domain $S\HH \cap S^7$ of unit spinors. (Here a unit spinor $\kappa$ is one satisfying $|\kappa| = 1$ as in \refeqn{H2_norm}, which lies in $S^7$ via the identification \refeqn{H2_R8}.) Indeed, $S\HH \cap S^7$ consists precisely of those $(\xi, \eta) \in S^7$ which the Hopf fibration sends to $\$\R^3 \cup \{\infty\} \subset \HH \cup \{\infty\}$, which is an equatorial $S^3 \subset S^4$. For such spinors, $\phi_1$ sends them to have $T$ coordinate $1$, so the image lies on the celestial sphere at $T=1$, i.e. $\S^+_1 \cong S^3$. Moreover $W+Xi+Yj = 2\xi \overline{\eta}$, and $Z$ is given by $Z = |\xi|^2 - |\eta|^2$. Composing with stereographic projection, which sends $S^3 \To \R^3 \cup \{\infty\}$, which we regard as a map $\S^+_1 \To \$\R^3 \cup \{\infty\}$ and from the boundary of the hyperboloid model to the upper half space model of $\hyp^4$, $\phi_1 (\kappa)$ ends up at precisely at $(W+Xi+Yj)/(T-Z) = \xi \overline{\eta} / |\eta|^2 = \xi \eta^{-1}$. (See \refeqn{boundary_hyperboloid_to_upper} in \refsec{H4_models} below, where we consider stereographic project explicitly.) Thus, $\phi_1$ composed with stereographic projection precisely equals the Hopf map on unit spinors, $S\HH \cap S^7 \To S^3$. Topologically this is a map $S^3 \times S^3 \To S^3$, which is the restriction of the quaternionic Hopf fibration $S^7 \To S^4$ to an equatorial $S^3$ subset of the base $S^4$. It follows from the above that this restriction of the Hopf fibration is a trivial $S^3$ bundle over $S^3$. \subsection{Tangent space of spinors and derivative of $\phi_1$} \label{Sec:deriv_phi1} Following the 3D case, we extend $\phi_1$ to a map to flags by including tangent data. Given a tangent vector $\nu \in T_\kappa S\HH$, we write $D_\kappa \phi_1(\nu)$ for the derivative of $\phi_1$ at $\kappa$ in the direction $\nu$. We then have, for real $t$, \[ \phi_1 \left( \kappa + t \nu \right) = \left( \kappa + t \nu \right) \left( \overline{\kappa} + t \overline{\nu} \right)^T = \kappa \overline{\kappa}^T + \left( \kappa \overline{\nu}^T + \nu \overline{\kappa}^T \right) t + \nu \overline{\nu}^T t^2 \] so the derivative is given by \begin{equation} \label{Eqn:derivative_phi1} D_\kappa \phi_1 (\nu) = \left. \frac{d}{dt} \phi_1 \left( \kappa + t \nu \right) \right|_{t=0} = \kappa \overline{\nu}^T + \nu \overline{\kappa}^T. \end{equation} We now use the structure of the tangent bundle to $S\HH$ from \reflem{TSH}, and $L^+$ from \refeqn{TL_decomposition}, so that if $\phi_1 (\kappa) = p = (T,W,X,Y,Z)$ then we have \[ D_\kappa \phi_1 \colon T_\kappa S\HH \To T_p L^+, \quad \text{i.e.} \quad \kappa \HH \oplus \check{\kappa} \$\R^3 \To p \R \oplus T_p \S^+_T. \] As we now see, $D_\kappa \phi_1$ behaves nicely on these summands. \begin{prop} \label{Prop:Derivs_props} Let $\kappa = (\xi, \eta) \in S\HH$ and $\phi_1 (\kappa) = p = (T,W,X,Y,Z)$ as above. \begin{enumerate} \item $D_\kappa \phi_1$ maps $\kappa \R$ isomorphically to $p \R$. Precisely, $D_\kappa \phi_1 (\kappa) = 2 p$. \item $D_\kappa \phi_1$ maps $\check{\kappa} \$\R^3$ isomorphically onto $T_{p} \S^+_T$. \item $\ker D_\kappa \phi_1 = \kappa \II$. \end{enumerate} \end{prop} Thus, $D_\kappa \phi_1$ is surjective onto $T_p L^+$, mapping $\kappa \R$ onto $p \R$ and $\check{\kappa} \$\R^3$ onto $T_p \S^+_T$. Thus the image of $\kappa$ forms a basis for $p \R$, and the images of $\check{\kappa}, \check{\kappa}i, \check{\kappa} j$ form a basis for $T_p \S^+_T$. A basis for the kernel is given by $\kappa i, \kappa j, \kappa k$. The kernel $\kappa \II$ is also the tangent space to the fibres of $\phi_1$, as described in \reflem{phi1_fibres}. \begin{proof} Taking $\nu = \kappa$ we have \[ D_\kappa \phi_1 (\nu) = \kappa \overline{\nu}^T + \nu \overline{\kappa}^T = 2 \kappa \overline{\kappa}^T = 2 \phi_1 (\kappa) \] which is nonzero and proportional to $p = \phi_1 (\kappa)$, so spans $p \R$, proving (i). Then taking $\nu = \check{\kappa} \, v = (\eta', -\xi') v$ for $v \in \$\R^3$ we have \[ D_\kappa \phi_1 (\nu) = \kappa \overline{\nu}^T + \nu \overline{\kappa}^T = \begin{pmatrix} \xi \\ \eta \end{pmatrix} \overline{v} \begin{pmatrix} \eta^* & -\xi^* \end{pmatrix} + \begin{pmatrix} \eta' \\ - \xi' \end{pmatrix} v \begin{pmatrix} \overline{\xi} & \overline{\eta} \end{pmatrix} \] The trace of the result, i.e. its $T$-coordinate, is \[ \xi \, \overline{v} \, \eta^* - \eta \, \overline{v} \, \xi^* + \eta' v \, \overline{\xi} - \xi' v \, \overline{\eta} = a - a^* + \overline{a} - a' \quad \text{where } a = \xi \, \overline{v} \, \eta^* \] which is zero by \reflem{conjugation_combination}. Hence $D_\kappa \phi_1 (\nu) \in T_p S_T^+$. We claim that when $v \neq 0$, $D_\kappa \phi_1 (\nu) \neq 0$. To see this, suppose to the contrary that there exists some $0 \neq v \in \$\R^3$ such that $D_\kappa \phi_1 ( \check{\kappa} v ) = 0$. Then we have $\kappa \overline{\nu}^T = - \nu \overline{\kappa}^T = -\check{\kappa} (v \overline{\kappa}^T)$, an equality of quaternionic matrices factorised as nonzero $2 \times 1$ and $1 \times 2$ vectors. Now by \reflem{factorisation_fact}, there exists a $1 \times 2$ row vector $\tau = (\alpha, \beta) \in \HH^2$ such that precisely one of $\tau \kappa$ and $\tau \check{\kappa}$ is zero, so precisely one of $\tau \kappa \overline{\nu}^T$ and $-\tau \check{\kappa}(v \overline{\kappa}^T)$ is zero. But these two matrices are equal, and we have a contradiction. Thus, when $v \neq 0$, $D_\kappa \phi_1 (\check{\kappa} v)$ is a nonzero vector in $T_p \S^+_T$. Since $\check{\kappa} \$\R^3$ and $T_p \S^+_T$ are both 3-dimensional, $D_\kappa \phi_1$ must map them to each other isomorphically. This proves (ii). From (i) and (ii), $D_\kappa \phi_1$ is surjective from $T_\kappa S\HH$ onto $T_p L^+$. As these spaces have real dimension $7$ and $4$ respectively, we have $\dim \ker D_\kappa \phi_1 = 3$. Now we calculate directly that $D_\kappa \phi_1 (\nu) = 0$ when $\nu = (\xi, \eta) u = \kappa u$ with $u$ pure imaginary: \[ D_\kappa \phi_1 (\nu) = \kappa \overline{\nu}^T + \nu \overline{\kappa}^T = \kappa \overline{u} \overline{\kappa}^T + \kappa u \overline{\kappa}^T = \kappa (\overline{u} + u) \overline{\kappa}^T. \] Since $u$ is imaginary, $\overline{u} = -u$, we obtain $D_\kappa \phi_1 (\nu) = 0$. Hence $\kappa \II \subseteq \ker D_\kappa \phi_1$, and as the kernel has dimension 3, this inclusion is in fact equality, as desired. \end{proof} To form multiflags, we will take derivatives of $\phi_1$ in certain directions of $\check{\kappa} \$\R^3$, which map to directions along $\S^+_T$. Recall we defined the section $s_v$ of $TS\HH$ for each $v \in \$\R^3$ in \refdef{Z}, which allows us to access these tangent vectors. We will see next that $\check{\kappa} \$\R^3 \subset T_\kappa S\HH$ maps conformally onto $T_p \S^+_T$. We then have the derivative in the direction $s_v$ as \begin{align*} \label{Eqn:DkappaZkappa} D_\kappa \phi_1 ( s_v \kappa ) = \kappa \, \overline{(s_v \kappa)}^T + (s_v \kappa) \, \overline{\kappa}^T = \kappa \overline{(J \kappa')v}^T + (J \kappa') v \overline{\kappa}^T = -\kappa \overline{v} \kappa^{*T} J + J \kappa' v \overline{\kappa}^T, \end{align*} using $J$ as in \refeqn{J_eqn}. When the spinors are complex and $v = i$, this reduces to the expression $\kappa \kappa^T (Ji) + (Ji) \overline{\kappa} \overline{\kappa}^T$ from (2.4) of \cite{Mathews_Spinors_horospheres}. \begin{example} \label{Eg:Dphi1_at_10} At $\kappa_0 = (1,0)$ we have $ \phi_1 (\kappa_0) = p_0 = (1,0,0,0,1) \in \S^+_1. $ The relevant tangent spaces are $T_{\kappa_0} S\HH = \kappa_0 \HH \oplus \check{\kappa}_0 \$\R^3$, where $\check{\kappa}_0 = (0,-1)$, and $T_{p_0} L^+ = p_0^\perp$, which is spanned by $p_0$ and the basis $\partial_W, \partial_X, \partial_Y$ of $T_{p_0} \S^+_1$. Then $D_{\kappa_0} \phi_1 (\kappa_0) = 2p_0$, $D_{\kappa_0} (\kappa_0 \II) = 0$, $D_{\kappa_0} (s_1 (\kappa_0)) = 2 \partial_W$, $D_{\kappa_0} (s_i (\kappa_0)) = 2 \partial_X $, $D_{\kappa_0} (s_j (\kappa_0)) = 2 \partial_Y$. So the oriented basis $s_1 (\kappa), s_i (\kappa), s_j (\kappa)$ of $\$\R^3$ (\refsec{space_of_spinors}) maps under $D_{\kappa_0} \phi_1$ to the basis $2 \partial_W, 2 \partial_X, 2 \partial_Y$ of $T_{p_0} \S^+_1$. As in \refeg{orientation_at_p0}, this is an outward oriented basis. \end{example} \subsection{Conformality on paravectors} \label{Sec:conformal_paravector} We saw in \refprop{Derivs_props} that at $\kappa \in S\HH$, mapping to $p \in L^+$ under $\phi_1$ the derivative $D_\kappa \phi_1$ restricts to an isomorphism $\widehat{\kappa} \$\R^3 \To T_p \S^+_T$. And as we saw in \refsec{paravectors_in_TSH}, there is a natural identification of the paravectors with $\check{\kappa} \$\R^3$, given by the map $s(v \cdot) \colon \$\R^3 \cong \check{\kappa} \$\R^3$ of \refeqn{conformal_paravectors_in_TSH}. Thus we have a composition of linear isomorphisms \begin{equation} \label{Eqn:paravectors_to_celestial_sphere} \$\R^3 \stackrel{s(\cdot, \kappa)}{\To} \check{\kappa} \$\R^3 \stackrel{D_\kappa \phi_1}{\To} T_p \S^+_T \end{equation} We saw in \reflem{paravector_first_conformal} that the first and second spaces have positive definite inner products, and the first map is conformal. The third space $T_p \S^+_T$ is a spacelike linear subspace of $\R^{1,4}$, so has a (negative definite) inner product and norm, as in \refsec{Hermitian_Minkowski}. We now show this composition is conformal. From \reflem{paravector_first_conformal}, the first map has scaling factor $|\kappa|^2$; we show the second has scaling factor $-4|\kappa|^2$. \begin{prop} \label{Prop:paravectors_conformal} For any $v,w \in \$\R^3$ and $\kappa \in S\HH$, \begin{equation} \label{Eqn:both_scaling_factors} \langle D_\kappa \phi_1 ( \check{\kappa} v ), D_\kappa \phi_1 ( \check{\kappa} w ) \rangle = -4 |\kappa|^2 \; \langle \check{\kappa} v, \check{\kappa} w \rangle = -4 |\kappa|^4 \; v \cdot w. \end{equation} \end{prop} The proof requires the following calculation. \begin{lem} \label{Lem:derivative_det_miracle} For $v \in \$\R^3$ and $\kappa \in S\HH$, \[ \det \left( D_\kappa \phi_1 \left( \check{\kappa} v \right) \right) = - |v|^2 |\kappa|^4. \] \end{lem} Here the derivative is considered as a matrix in $\pH$. \begin{proof} The derivative is given by \begin{align*} D_\kappa \phi_1 \left( \check{\kappa} v \right) &= \kappa \, \overline{\left( \check{\kappa} v \right)}^T + \check{\kappa} v \, \overline{\kappa}^T = \begin{pmatrix} \xi \\ \eta \end{pmatrix} \overline{v} \begin{pmatrix} \eta^* & - \xi^* \end{pmatrix} + \begin{pmatrix} \eta' \\ - \xi' \end{pmatrix} v \begin{pmatrix} \overline{\xi} & \overline{\eta} \end{pmatrix} \\ &= \begin{pmatrix} \xi \overline{v} \eta^* + \eta' v \overline{\xi} & - \xi \overline{v} \xi^* + \eta' v \overline{\eta} \\ \eta \overline{v} \eta^* -\xi' v \overline{\xi} & - \eta \overline{v} \xi^* - \xi' v \overline{\eta}. \end{pmatrix} \end{align*} As this matrix is paravector Hermitian, $\det$ or $\pdet$ yield the same result, and we calculate \begin{align*} \pdet \left( D_\kappa \phi_1 \left( \check{\kappa} v \right) \right) &= - \left( \eta \overline{v} \xi^* + \xi' v \overline{\eta} \right) \left( \eta \overline{v} \xi^* + \xi' v \overline{\eta} \right) - \left( \eta \overline{v} \eta^* - \xi' v \overline{\xi} \right) \left( - \xi \overline{v} \xi^* + \eta' v \overline{\eta} \right) \\ &= - \left( \eta \overline{v} \xi^* \right)^2 - 2 |\xi|^2 |\eta|^2 |v|^2 - \left( \xi' v \overline{\eta} \right)^2 + \left( \eta \overline{v} \eta^* \right) \left( \xi \overline{v} \xi^* \right) - |\xi|^4 |v|^2 - |\eta|^4 |v|^2 + \left( \xi' v \overline{\xi} \right) \left( \eta' v \overline{\eta} \right) \\ &= \eta \overline{v} \left( \eta^* \xi - \xi^* \eta \right) \overline{v} \xi^* - \left( |\xi|^2 + |\eta|^2 \right)^2 |v|^2 + \xi' v \left( \overline{\xi} \eta' - \overline{\eta} \xi' \right) v \overline{\eta} \\ &= - |\kappa|^4 |v|^2 \end{align*} In the first line we used $v^* = v$. In the final line we used $(\xi, \eta) \in S\HH$, so that $\xi^* \eta, \; \overline{\eta} \xi' \in \$\R^3$ (\reflem{spinor_condition}, \refeqn{spinor_conditions}). As elements of $\$\R^3$ are invariant under $*$, we have $\xi^* \eta = \eta^* \xi$ and $\overline{\eta} \xi' = \overline{\xi} \eta'$. \end{proof} \begin{proof}[Proof of \refprop{paravectors_conformal}] After \reflem{paravector_first_conformal} it is sufficient to prove that, for $v,w \in \$\R^3$ and $\kappa \in S\HH$, \begin{equation} \label{Eqn:paravectors_conformal_2} \langle D_\kappa \phi_1 ( \check{\kappa} v ), D_\kappa \phi_1 ( \check{\kappa} w ) \rangle = -4 |\kappa|^4 \; v \cdot w. \end{equation} To see this, we first recall \refeqn{matrix_R14_correspondences} that if a matrix $S \in \pH$ corresponds to $p \in \R^{1,4}$ then $4 \det S = |p|^2$. Thus from \reflem{derivative_det_miracle}, we have \begin{equation} \label{Eqn:miracle_in_norms} \left| D_\kappa \phi_1 \left( s_v (\kappa) \right) \right|^2 = - 4 |v|^2 |\kappa|^4. \end{equation} Now we apply the polarisation identity on both sides. In $\$\R^3$ we obtain $4 v \cdot w = |v+w|^2 - |v-w|^2$. In $\R^{1,4}$, using real-linearity of the derivative we have \[ 4 \left\langle D_\kappa \phi_1 \left( \check{\kappa} v \right), \; D_\kappa \phi_1 \left( \check{\kappa} w \right) \right\rangle = \left| D_\kappa \phi_1 \left( \check{\kappa} (v+w) \right) \right|^2 - \left| D_\kappa \phi_1 \left( \check{\kappa} (v-w) \right) \right|^2. \] Applying these polarisation identities to \refeqn{miracle_in_norms} then gives \refeqn{paravectors_conformal_2} as desired. \end{proof} We can also consider the effect of $D \phi_1$ quotients $T_\kappa S\HH / \kappa \HH \cong \check{\kappa}\$\R^3$, studied in \refsec{SL2_on_spinors}. This quotient has the inner product defined in \refdef{inner_product_on_spinor_quotient}. Now we consider the derivative $D_\kappa \phi_1$ at $\kappa \in S\HH$. as applied to the quotient $T_\kappa S\HH / \kappa \HH$. Letting $\phi_1 (\kappa) = p$ lie on $\S^+_T \subset L^+$, and $\ell = p \R$, then the map $D_\kappa \phi_1 \colon T_\kappa S\HH \To T_p L^+$ sends $\kappa \R$ to $\ell$, and $\kappa \II$ is the kernel, so $D_\kappa \phi_1$ yields a well defined map of quotients \begin{equation} \label{Eqn:derivative_on_quotients} D_\kappa \phi_1 \colon \frac{T_\kappa S\HH}{\kappa \HH} \To \frac{T_p L^+}{p \R} = \frac{\ell^\perp}{\ell}, \quad \text{isomorphic to} \quad \check{\kappa}\$\R^3 \To T_p \S^+_T \end{equation} The quotient $\ell^\perp/\ell$ is isometric to $T_p \S^+$ and has a well defined negative definite inner product as discussed in \refsec{Hermitian_Minkowski}, and orientations as discussed in \refsec{orientations}. On $T_\kappa S\HH / \kappa \HH \cong \check{\kappa}\$\R^3$ we have the positive definite inner product of \refdef{inner_product_on_spinor_quotient}, and orientation formed by $s_1(\kappa),s_i(\kappa),s_j(\kappa)$ as discussed in \refsec{SL2_on_spinors}. We then have the following. \begin{lem} \label{Lem:spinor_quotient_conformal} The map \refeqn{derivative_on_quotients} is conformal with scaling factor $-4 |\kappa|^2$, and is orientation-preserving with respect to the outward orientation on $\S^+$. \end{lem} \begin{proof} As discussed in \refsec{SL2_on_spinors}, every element of $T_\kappa S\HH / \kappa \HH$ has a unique representative of the form $s_\kappa (v)$, for some $v \in \$\R^3$, and for $v,w \in \$\R^3$ we have $(s_\kappa (v) + \kappa \HH, s_\kappa (w) + \kappa \HH) = \langle s_\kappa(v), s_\kappa (w)\rangle = |\kappa|^2 \, v \cdot w$, where $\langle \cdot, \cdot \rangle$ is the inner product on $\HH^2$ and $\cdot$ is the dot product on paravectors. The conformality statement is now just recalling that the second map $D_\kappa \phi_1$ of \refeqn{paravectors_to_celestial_sphere} has scaling factor $-4|\kappa|^2$. Explicitly, by \refprop{paravectors_conformal} we have \[ \langle D_\kappa \phi_1 (s_\kappa (v)), D_\kappa \phi_1 (s_\kappa (w)) \rangle = -4 |\kappa|^2 \langle s_v (\kappa), s_w (\kappa) \rangle = -4 |\kappa|^2 (s_\kappa (v) + \kappa \HH, s_\kappa (w) + \kappa \HH). \] To see that the map is orientation-preserving, note that at each $\kappa \in S\HH$, the map \refeqn{derivative_on_quotients} sends a standard oriented basis represented by $s_1(\kappa), s_i(\kappa), s_j(\kappa)$ to a basis of $T_{\phi(\kappa)} \S^+_T$. Since $S\HH$ is connected, $\phi_1$ is continuous, and a continuously varying set of oriented bases always has the same orientation, it suffices to verify the statement at a single $\kappa$. From \refeg{Dphi1_at_10}, at $\kappa_0 = (1,0)$ the standard basis is sent to an outward oriented basis of the celestial sphere. \end{proof} \subsection{Flags} \label{Sec:flags} We can now define flags. As in \cite{Mathews_Spinors_horospheres}, all flags are oriented of signature $(1,2)$, i.e. of the form $\{0\} = V_0 = V_1 \subset V_2$, where $\dim V_1 = 1$, $\dim V_2 = 2$, and $V_1/V_0 = V_1$ and $V_2/V_1$ are endowed with orientations. All vector spaces and dimensions are over $\R$. The following definition is identical to \cite{Mathews_Spinors_horospheres}. \begin{defn} A \emph{pointed oriented flag}, or just \emph{flag}, consists of a point $p \in L^+$ and an oriented flag $\{0\} \subset V_1 \subset V_2$ in $\pH \cong \R^{1,4}$ of signature $(1,2)$, such that \begin{enumerate} \item $V_1 = p \R$, the \emph{flagpole}, is future-oriented, and \item $V_2$ is a tangent 2-plane to $L^+$ at $p$, i.e. $V_2 \subset T_p L^+$. \end{enumerate} We call $p$ the \emph{basepoint} and say the flag is \emph{based} at $p$. The set of flags is denoted $\F$, and the set of flags based at $p$ is denoted $\F_p$. \end{defn} A flag can be recovered from the data of $p$ and the relatively oriented $V_2$, thus we can denote a flag by a pair $(p, V_2)$. For $p \in L^+$ and $v \in T_p L^+$, we denote by $[[p,v]]$ the flag based at $p$ with $V_2$ spanned by $p$ and $v$ and $V_2 / p \R$ oriented by (the equivalence class of) $v$. As in \cite{Mathews_Spinors_horospheres}, $V_2$ contains no timelike vectors, and $p \R$ generates the unique 1-dimensional lightlike subspace of $V_2$. Since $T_p L^+ = p^\perp$, we have $p \R \subset V_2 \subset p^\perp$. Two flags described as $[[p,v]]$, $[[p',v']]$ are equal if and only if $p=p'$ and there exist real $a,b,c$ such that $ap+bv+cv' = 0$, where $b,c$ (necessarily nonzero) have opposite sign. The set $\F_p$ of flags based at $p$ naturally corresponds to the set of oriented lines in $p^\perp / p \R$. Indeed, from a flag $(p,V)$, the quotient $V / p \R$ is a line in $p^\perp / p \R$, which obtains an orientation from the flag orientation. Conversely, an oriented line $\ell + p \R \in p^\perp / p \R$ lifts to a 2-plane $V = \ell + p \R \subset p^\perp$ such that $V / p \R$ has an orientation, so $(p, V)$ is a flag. The isometry $T_p \S^+_T \cong p^\perp / p \R$ of \refeqn{celestial_sphere_light_cone_quotient}, for any $T>0$, thus provides a bijective correspondence between $\F_p$ and oriented lines tangent to $\S^+_T$ at $p$, or equivalently, to unit tangent vectors in $T_p \S^+_T$. We have a conformal structure on $\F_p$ using the following fact, which will also be useful in the sequel. \begin{lem} \label{Lem:projection_along_light} Let $p \in L^+$ and $\Pi$ be a vector subspace of $\R^{1,4}$ such that $p \R \subseteq \Pi \subseteq p^\perp$. Let $\pi \colon \Pi \To \frac{\Pi}{p\R}$ be the projection. Then $\pi$ is a well-defined map of inner product spaces, and we have the following: \begin{enumerate} \item If $v_1, v_2, w_1, w_2 \in \Pi$ satisfy $\pi(v_1) = \pi(v_2)$ and $\pi(w_1) = \pi(w_2)$ then $\langle v_1, w_1 \rangle = \langle v_2, w_2 \rangle$. \item For any $v,w \in \Pi$, $\langle v,w \rangle = \langle \pi(v), \pi(w) \rangle$. \end{enumerate} \end{lem} In particular, if $\pi(v_1) = \pi(v_2)$ then $|v_1|^2 = |v_2|^2$, and $|v|^2 = |\pi(v)|^2$. The argument is essentially the same as the one in \refsec{Hermitian_Minkowski} showing \refeqn{celestial_sphere_light_cone_quotient} is an isometry. \begin{proof} We have $v_1 = v_2 + xp$ and $w_1 = w_2 + yp$ for some $x,y \in \R$, so $\langle v_1, w_1 \rangle = \langle v_2 + xp, w_2 + yp \rangle = \langle v_2, w_2 \rangle$, using $\Pi \subseteq p^\perp$. Thus $\langle \pi(v), \pi(w) \rangle$ is well defined and equal to $\langle v_1, w_1 \rangle = \langle v_2, w_2 \rangle$. \end{proof} \begin{lem} \label{Lem:flag_angle_well_defined} Let $[[p,v]],[[p,w]] \in \F_p$. Then the angle between $v$ and $w$ is independent of the choice of $v,w$ in the descriptions of these flags. \end{lem} Note the angle is well defined since the vectors $v, w$ must be spacelike. \begin{proof} Let $v_1, v_2$ be two choices of $v$, and $w_1, w_2$ two choices of $w$. Then $v_1 = ap + b v_2$, where $a,b \in \R$ and $b>0$; similarly $w_1 = cp + dw_2$, where $c,d \in \R$ and $d>0$. As all flags lie in $p^\perp$, by the previous lemma $|v_1|^2 = b^2 |v_2|^2$, $|w_1|^2 = d^2 |w_2|^2$, and $\langle v_1, w_1 \rangle = bd \langle v_2, w_2 \rangle$, the cosines of the angles are equal: \[ \frac{ \langle v_1, w_1 \rangle }{ \sqrt{|v_1|^2 |w_1|^2} } = \frac{ bd \langle v_2, w_2 \rangle }{ \sqrt{ b^2 d^2 |v_2|^2 |w_2|^2 }} = \frac{ \langle v_2, w_2 \rangle}{ \sqrt{|v_2|^2 |w_2|^2} }. \] \end{proof} \begin{defn} \label{Def:flag_angle} The \emph{angle} between two flags $[[p,v]],[[p,w]]$ is the angle between $v$ and $w$. \end{defn} Topologically, each $\F_p \cong UT_p \S^+_T \cong S^2$, and $\F$ is a bundle over $L^+ \cong S^3 \times \R$ with $S^2$ fibres $\F_p$. Then $\F \cong UTS^3 \times \R$, where $UTS^3$ is the unit tangent bundle of $S^3$. We regard $p \in L^+ \cong S^3 \times \R$, and then the 2-plane provides an oriented line in $T_p S^3$, corresponding to a unit vector in $UT_p S^3$. Equivalently, $\F \cong TS^3 \setminus S^3$, the complement of the zero section in $TS^3$. Since $S^3$ (like all 3-manifolds) is parallelisable we have $TS^3 \cong S^3 \times \R^3$, so $\F \cong S^3 \times S^2 \times \R$. The following lemma generalises lemma 2.8 of \cite{Mathews_Spinors_horospheres}, relating flags to the bracket, and showing when derivatives in different directions yield the same flag. \begin{lem} \label{Lem:when_flags_equal} For $\kappa \in S\HH$, $\nu \in T_\kappa S\HH$, and $v \in \$\R^3$, the following are equivalent: \begin{enumerate} \item $\{ \kappa, \nu \}$ is a negative real multiple of $v$; \item $\nu = \kappa x + b s_v (\kappa)$ where $x \in \HH$ and $b$ is real positive; \item $[[\phi_1 (\kappa), D_\kappa \phi_1 (\nu) ]] = [[\phi_1 (\kappa), D_\kappa \phi_1 \left( s_v (\kappa) \right) ]]$. \end{enumerate} \end{lem} \begin{proof} From \refeqn{inner_product_with_sv} above we have $\{\kappa, s_v (\kappa) \}$ is a negative real multiple of $v$, and from \reflem{nondegeneracy_of_spinor_form} $\{\kappa, \kappa x \} = 0$. Thus (ii) implies (i). For the converse, from \reflem{nondegeneracy_of_spinor_form} $\{\kappa, \alpha \} = \{ \kappa, \beta \}$ implies $\alpha - \beta = \kappa x$ for some $x \in \HH$. To see that (ii) implies (iii), let $x = x_0 + x_1$ where $x_0 \in \R$ and $x_1$ is imaginary. Then by \refprop{Derivs_props}, $D_\kappa \phi_1 (\kappa x) = D_\kappa \phi_1 ( \kappa x_0) = 2 x_0 \phi_1 (\kappa)$, a real multiple of $\phi_1 (\kappa)$. So $D_\kappa \phi_1 (\nu)$ is equal to a real multiple of $\phi_1 (\kappa)$, plus $D_\kappa \phi_1 (b s_v (\kappa)) = b D_\kappa \phi_1 (s_v(\kappa))$, where $b$ is positive. Thus the half-plane spanned by $\phi_1 (\kappa)$ and positive multiples of $D_\kappa \phi_1 (\nu)$ is equal to the half-plane spanned by $\phi_1 (\kappa)$ and positive multiples of $D_\kappa \phi_1 (s_v (\kappa))$. Hence $[[\phi_1 (\kappa), D_\kappa \phi_1 (\nu)]] = [[\phi_1 (\kappa), D_\kappa \phi_1 (s_v (\kappa))]]$. For the converse, if the two flags are equal, then $D_\kappa \phi_1 (\nu ) = a \phi_1 (\kappa) + b D_\kappa \phi_1 (s_v \kappa)$, where $a$ is real and $b$ positive. Since $D_\kappa \phi_1 (\kappa) = 2 \phi_1 (\kappa)$ we have $\nu - \kappa \frac{a}{2} - b s_v (\kappa) \in \ker D_\kappa \phi_1$, so by \refprop{Derivs_props} is equal to $\kappa c$ for some $c \in \II$. Thus $\nu = \kappa ( \frac{a}{2} + c ) + b s_v (\kappa)$, so (ii) holds. \end{proof} \subsection{Multiflags} \label{Sec:multiflags} We can now make use of the notion of angle between flags from \refdef{flag_angle} to define a multiflag. \begin{defn} \label{Def:multiflag} A \emph{multiflag} is a triple $(p, V^i, V^j)$ where $p \in L^+$, and $(p, V^i), (p,V^j)$ are orthogonal flags. The flags $(p, V^i)$ and $(p, V^j)$ are called the \emph{$i$-flag} and \emph{$j$-flag} respectively. We say the multiflag is \emph{based} at $p$. The set of multiflags is denoted $\MF$, and the set of multiflags based at $p$ is denoted $\MF_p$. \end{defn} If the $i$- and $j$-flags in a multiflag are described as $[[p, v^i]]$ and $[[p,v^j]]$ respectively, we denote the multiflag by $[[p,v^i,v^j]]$. By \reflem{flag_angle_well_defined}, the orthogonality of the flags means $\langle v^i, v^j \rangle = 0$. In a multiflag, since the $i$-flag and $j$-flag have the same basepoint $p$, they have the same 1-plane $p \R$, future-oriented. If two multiflags $[[p,v^i,v^j]]$, $[[p',w^i,w^j]]$ are equal then $p=p'$ and we have equalities $[[p,v^i]] = [[p,w^i]]$ and $[[p,v^j]]=[[p,w^j]]$, so $a^i p + b^i v^i + c^i w^i = 0$ and $a^j p + b^j v^j + c^j w^j = 0$, with each triple $a^\bullet,b^\bullet,c^\bullet$ real and $b^\bullet,c^\bullet$ of opposite sign. Topologically, $\MF_p$ is diffeomorphic to $SO(3)$. As we have seen, flags based at $p$ correspond to unit vectors in $T_p \S^+_T$. The $i$-flag and $j$-flag of a multiflag correspond to two orthonormal vectors in $\S^+_T \cong S^3$. This pair of vectors extends to a unique (right-handed orthonormal) frame in $T_p \S^+_T$. Thus $\MF_p$ is diffeomorphic to the space of frames at $p$ in $\S^+_T$, which is diffeomorphic to $SO(3)$ in the standard way. That is, $\MF_p \cong \Fr_p \S^+_T$, where $\Fr \S^+_T$ is the bundle of (oriented orthonormal) frames on $\S^+_T$. Then $\MF$ is a bundle over $L^+ \cong S^3 \times \R$ with fibres $\MF_p \cong \Fr_p \S^+_T \cong SO(3)$. Indeed, $\MF \cong \Fr S^3 \times \R$. A point of $S^3$ describes a ray in $L^+$, a frame there describes the 2-planes of the flags from its first two vectors, and the $\R$ factor fixes the basepoint $p$. By parallelisability $\Fr S^3 \cong S^3 \times SO(3)$, so $\MF \cong S^3 \times SO(3) \times \R$. The following map to flags generalises the construction of flags in \cite{Mathews_Spinors_horospheres} and by Penrose--Rindler \cite{Penrose_Rindler84}. \begin{defn} The map $\phi_1 \colon S\HH \To \MF$ is defined as \[ \Phi_1 (\kappa) = [[ \phi_1 (\kappa), D_\kappa \phi_1 (s_i \kappa), D_\kappa \phi_1 (s_j \kappa) ]]. \] \end{defn} Thus $\phi_1$ provides the flagpole, and its derivatives in the directions given by the sections $s_i, s_j$ yield the $i$- and $j$-flags. It follows from \refprop{Derivs_props} and \refdef{Z} that the derivatives of $\phi_1$ in the $s_i (\kappa)$ and $s_j (\kappa)$ directions are nonzero, and from \refprop{paravectors_conformal} that these derivatives are orthogonal. Indeed, the flag directions are given by derivatives of $\phi_1$ in the directions corresponding to paravectors $i,j$ under the section $s$ of \refdef{Z} and the conformal maps \refeqn{paravectors_to_celestial_sphere} discussed in \refsec{conformal_paravector}. So $\Phi_1$ is well defined. \begin{example} \label{Eg:Phi1_of_k0} Let us calculate $\Phi_1 ( \kappa_0 )$ where $\kappa_0 = (1,0)$. From \refeg{Dphi1_at_10} we have $\phi_1 (\kappa_0) = (1,0,0,0,1) = p_0$, and $D_\kappa \phi_1 (s_i \kappa_0) = 2 \partial_X$, $D_{\kappa_0} \phi_1 (s_j \kappa_0) = 2 \partial_Y$. So we have $\Phi_1 (1,0) = [[p_0, \partial_X, \partial_Y]]$. \end{example} \subsection{Decorated ideal points} \label{Sec:decorated_ideal_points} In this section, we discuss how a multiflag based at $p \in L^+$ is equivalent to a conformal identification of $\$\R^3$ with the quotient $T_p L^+ / p \R$, as follows. These are analogous to the decorations we later give on horospheres, for ideal points of $\hyp^4$, and hence we call them ``ideal decorations". Recall from \refsec{Hermitian_Minkowski} that the celestial sphere $\S^+$ is the projectivised light cones, so its points can be regarded as lightlike 1-dimensional subspaces $\ell \subset \R^{1,4}$; such an $\ell$ also corresponds to a point on the ideal boundary of $\hyp^4$ in the hyperboloid model. \begin{defn} \label{Def:decorated_ideal_point} Let $\ell \in \S^+$. A \emph{decoration} on $\ell$ is a conformal orientation-preserving $\R$-linear isomorphism \[ \psi \colon \$\R^3 \To \frac{\ell^\perp}{\ell}, \] with respect to the outward orientation on $\ell^\perp/\ell$. A \emph{decorated ideal point} is such a pair $(\ell, \psi)$. The set of all decorated ideal points is denoted $\S^{+D}$, and the set of all decorations on $\ell$ is denoted $\S^{+D}_\ell$. \end{defn} Here $\$\R^3$ are the paravectors, identified with $\R^3$ as in \refeqn{R3_R3}, endowed with its standard orientation and dot product, agreeing with the inner product on $\HH$ (\refsec{dot_cross_paravector}). The space $\ell^\perp/\ell$ has the negative definite inner product induced from $\R^{1,4}$, of which it is sub-quotient, as discussed in \refsec{Hermitian_Minkowski}. It has an outwards and inwards orientation as discussed in \refsec{orientations}; we choose the outward orientation because of \refeg{orientation_at_p0} and \reflem{spinor_quotient_conformal}, but the choice is irrelevant to the equivalence with multiflags. The key fact is that once $\psi(i), \psi(j)$ are given (necessarily orthogonal and equal in norm), $\psi$ is determined. As the inner products on $\$\R^3$ and $\ell^\perp/\ell$ are respectively positive and negative definite, the scaling factor of $\Psi$ (\refdef{scaling_factor}) must be negative. The set of conformal linear orientation-preserving isometries between 3-dimensional definite vector spaces is diffeomorphic to $SO(3) \times \R$, with the $SO(3)$ acting transitively on frames, so each $\S^{+D}_\ell \cong SO(3) \times \R$. In fact, as $\ell^\perp/\ell \cong T_\ell \S^+$ as in \refsec{Hermitian_Minkowski}, we have $\S^{+D}_\ell \cong \Fr_\ell \S^+ \times \R$, where $\Fr \S^{+}$ is the frame bundle of $\S^+$. Then $\S^{+D}$ is a bundle over $\S^+$ with fibres $\S^{+D}_\ell \cong \Fr_\ell \S^+ \times \R$, so $\S^{+D} \cong \Fr \S^+ \times \R$. \begin{prop} \label{Prop:multiflags_ideal_decorations} There is a smooth bijective correspondence $\Psi \colon \MF \To \S^{+D}$ given as follows. \begin{enumerate} \item Given $[[p, v^i, v^j]] \in \MF$, let $p$ have $T$-coordinate $T_0$, and let $a^i, a^j > 0$ be such that $a^i v^i$ and $a^j, v^j$ have Minkowski norm $-4T_0^2$. Then \[ [[p, v^i, v^j]] \mapsto (p \R, \psi) \] where $\psi$ is uniquely defined by $\psi(i) = a^i v^i + p \R$ and $\psi(j) = a^j v^j + p \R$. \item Given $(\ell, \psi) \in \S^{+D}$, let $K<0$ be the scale factor of $\psi$. Then \[ (\ell, \psi) \mapsto [[ p, \widetilde{\psi(i)}, \widetilde{\psi(j)} ]], \] where $p$ is the point on $\ell$ whose $T$-coordinate $T_0$ satisfies $K = -4T_0^2$, and $\widetilde{\psi(i)}, \widetilde{\psi(j)} \in \ell^\perp$ are arbitrary lifts of $\psi(i), \psi(j) \in \ell^\perp/\ell$ to $\ell^\perp = T_p L^+$. \end{enumerate} \end{prop} \begin{proof} Consider the multiflag $[[p, v^i, v^j]]$. Let $\ell = p \R$. Flags at $p$ correspond to oriented lines in $\ell^\perp / \ell$, and the equivalence classes of $v^i$ and $v^j$ in $\ell^\perp/\ell$ orient the lines corresponding to the $i$-flag and $j$-flag. Note $v^i, v^j$ are both spacelike, as are their equivalence classes in $\ell^\perp/\ell$. As $T_0 > 0$ and $v^i, v^j$ are spacelike, there are unique $a^i, a^j>0$ such that $a^i v^i, a^j v^j$ have Minkowski norm $-4T_0^2$. Hence $a^i v^i + \ell, a^j v^j + \ell \in \ell^\perp/\ell$ also have norm $-4T_0^2$. As $\ell^\perp / \ell$ is oriented, 3-dimensional and negative definite, there exists a unique $v^1 \in \ell^\perp\ell$, also of Minkowski norm $-4T_0^2$, such that $(a^i v^i + \ell, a^j v^j + \ell, v^1)$ from an oriented basis. There is a unique orientation-preserving conformal $\psi$ such that $\psi(i) = a^i v^i + \ell$ and $\psi(j) = a^j v^j + \ell$, and by choice of $v^1$, this $\psi$ satisfies $\psi(1) = v^1$. Since $\psi$ sends $1,i,j$ of unit norm to elements of norm $-4T_0^2$, $\psi$ has scale factor $K = -4T_0^2$. From $(\ell, \psi)$ we can set $p$ as the point on $\ell$ with $T$-coordinate $T_0>0$ such that the scale factor $K$ of $\psi$ is $K = -4T_0^2$. Then $\psi(i)$ spans a ray in $\ell^\perp/\ell = T_p L^+ / p \R$ giving the $i$-flag, and $\psi(j)$ spans the $j$-flag. Taking arbitrary lifts $\widetilde{\psi(i)}, \widetilde{\psi(j)} \in \ell^\perp$ then the $i$-flag is $[[p, \widetilde{\psi(i)}]]$ and the $j$-flag is $[[p, \widetilde{\psi(j)}]]$. As $\psi$ is conformal and $i,j \in \$\R^3$ are orthogonal, $\psi(i),\psi(j)$ are orthogonal, so the $i$-flag and $j$-flag are orthogonal, and we have a multiflag as claimed. To see these correspondences are mutually inverse, suppose $[[p,v^i,v^j]]$ maps to $(\ell, \psi)$, so $\ell = p \R$, $\psi$ has scaling factor $K = -4T_0^2$, and $\psi(i) = a^i v^i + p \R$ and $\psi(j) = a^j v^j + p \R$ as above. Let $(\ell, \psi)$ map to $[[p', \widetilde{\psi(i)}, \widetilde{\psi(j)}]]$. Since $p'$ lies on $p \R$ and has $T$-coordinate $T'$ such that $K = - 4T_0^2 = -4T'^2$ and $T,T'>0$, we have $p = p'$. Since $\psi(i) = a^i v^i + p \R$ and $a^i > 0$, any lift $\widetilde{\psi(i)} \in \ell^\perp$ yields the same flag as $v^i$; similarly $\widetilde{\psi(j)}$ yields the same flag as $v^j$. Thus $[[p,v^i,v^j]] = [[p',\widetilde{\psi(i)},\widetilde{\psi(j)}]]$. Similarly, suppose $(\ell, \psi)$ maps to $[[p,v^i,v^j]]$, where $\psi$ has scaling factor $K$, the $T$-coordinate $T_0$ of $p$ satisfies $K = -4T_0^2$ and $v^i,v^j \in \ell^\perp$ generate the same flags as $\psi(i),\psi(j) \in \ell^\perp/\ell$. Let $[[p,v^i,v^j]]$ map to $(\ell', \psi')$. Then $\ell' = p \R$ so $\ell' = \ell$. And $\psi'$ has scaling factor $-4T_0^2$, so sends $i,j$ to elements of $\ell^\perp/\ell$ of norm $-4T_0^2$, generating the flags $[[p,v^i]]$ and $[[p,v^j]]$, just like $\psi$. Thus $\psi = \psi'$ and so $(\ell, \psi) = (\ell', \psi')$. So the correspondences are mutually inverse, and they clearly depend smoothly on the input data. \end{proof} We have previously seen a decorated ideal point: the map $D_\kappa \phi_1 \circ s(\kappa \cdot)$ of \refeqn{paravectors_to_celestial_sphere}, which maps $\$\R^3 \To \widecheck{\kappa} \$\R^3$, as arises naturally in the tangent bundle to spinors, then $\widecheck{\kappa} \$\R^3 \To T_p \S^+_T$ via $D_\kappa \phi_1$. The latter space is isometric to $\ell^\perp/\ell$ by \refeqn{celestial_sphere_light_cone_quotient}. By \refprop{multiflags_ideal_decorations}, there is a corresponding flag: we show it is precisely $\Phi_1 (\kappa)$. The seemingly arbitrary $-4T_0^2$ in \refprop{multiflags_ideal_decorations} was chosen in order to produce this equality. \begin{lem} For any $\kappa \in S\HH$, let $p = \phi_1 (\kappa)$, $\ell = p \R$ and \[ \psi \colon \$ \R^3 \To T_p \S^+_T \cong \frac{\ell^\perp}{\ell}, \quad \psi(v) = D_\kappa \phi_1 \left( s_v (\kappa) \right). \] Then $(\ell, \psi)$ is the decorated ideal point corresponding to $\Phi_1 (\kappa) \in \MF$ under $\Psi$. \end{lem} \begin{proof} Let $p$ have $T$-coordinate $T_0$. By \refeqn{phi1_in_coords} we have $T_0 = |\xi|^2 + |\eta|^2 = |\kappa|^2$. By \refprop{paravectors_conformal}, since $s_v (\kappa) = \check{\kappa} v$, $\psi$ has scaling factor $K = -4|\kappa|^4 = -4T_0^2$. Since $|i|^2 = |j|^2 = 1$, we have $|\psi(i)|^2 = |\psi(j)|^2 = -4 T_0^2$; in other words, $|D_\kappa \phi_1 (s_i \kappa)|^2 = |D_\kappa \phi_1 (s_j \kappa)|^2 = - 4T_0^2$. Moreover, by \refprop{Derivs_props}, $D_\kappa \phi_1 (s_i \kappa)$ and $D_\kappa \phi_1 (s_i \kappa)$ both lie in $T_p \S^+_{T_0} \subset \ell^\perp$. Now $\Phi_1 (\kappa) = [[p, D_\kappa \phi_1 (s_i \kappa), D_\kappa \phi_1 (s_j \kappa)]]$. Under $\Psi$, this multiflag corresponds to the line $p \R = \ell$, together with the conformal map $\$\R^3 \To \ell^\perp/\ell$, with scaling factor $K = -4T_0^2$ which sends $i$ to a generator of the $i$-flag in $\ell^\perp/\ell$ and $j$ to a generator of the $j$-flag in $\ell^\perp/\ell$. As $D_\kappa \phi_1 (s_i \kappa), D_\kappa \phi_1 (s_j \kappa) \in \ell^\perp$ have precisely the correct norm and generate these flags, they must correspond precisely to $\Psi(i), \Psi(j) \in \ell^\perp/\ell$ under the isometry $T_p \S^+_{T_0} \cong \ell^\perp/\ell$. Thus the conformal map is precisely $\psi$ as claimed. \end{proof} \begin{example} \label{Eg:decorated_ideal_point_of_k0} In \refeg{Phi1_of_k0}, for $\kappa_0 = (1,0)$, we calculated $\Phi_1 (\kappa_0) = [[p_0, \partial_X, \partial_Y]]$ where $p_0 = (1,0,0,0,1) \in \S^+_1$, and in \refeg{Dphi1_at_10} we calculated that $D_{\kappa_0} \phi_1$ sends $s_i (\kappa_0) \mapsto 2 \partial_X$ and $s_j (\kappa_0) \mapsto 2 \partial_Y$. Let the corresponding ideal point of $\hyp^4$ be $\ell_0 = p_0 \R$, so $\ell_0^\perp$ has basis $\partial_W, \partial_X, \partial_Y, p_0$, and as in \refeg{orientation_at_p0}, the quotient $\ell_0^\perp/\ell \cong T_{p_0} \S^+_1$ has outward oriented basis represented by $\partial_W, \partial_X, \partial_Y$. The corresponding decorated ideal point is $(\ell_0, \psi_0)$ where $\psi_0 \colon \$\R^3 \To \ell/\ell^\perp$ sends $i \mapsto 2\partial_X$, $j \mapsto 2\partial_Y$, and $1 \mapsto 2\partial_W$. \end{example} \subsection{$SL_2$ action and equivariance} \label{Sec:SL2_on_paravectors_etc} We now consider the action of $SL_2\$$ on $\pH \cong \R^{1,4}$ and $\MF \cong \S^{+D}$, and the equivariance of $\phi_1$ and $\Phi_1$. We studied the action on $S\HH$ in \refsec{SL2_on_spinors}, including on tangent vectors in $T_\kappa S\HH = \kappa \R \oplus \kappa \II \oplus \check{\kappa} \$\R^3$. We showed that the action preserves the summands $\kappa \R$ and $\kappa \II$, on which it acts straightforwardly, and explicitly described its action on the quotients $T_\kappa S\HH / \kappa \HH \cong \check{\kappa}\$\R^3$. We now also know the effect of $D \phi_1$ on these summands, as it sends a spinor $\kappa \in S\HH$ to a point $p \in \S^+_T \subset L^+$. From \refprop{Derivs_props}, $D_\kappa \phi_1$ sends the $\kappa \R$ direction on $S\HH$ maps to the $p \R$ direction at $p \in L^+$, $\kappa \II$ is the kernel, and $\check{\kappa}\$\R^3$ is sent to $T_p \S^+_T$. Moreover, by \refprop{paravectors_conformal} and \reflem{spinor_quotient_conformal}, the map from $\check{\kappa}\$\R^3$ or equivalently $T_\kappa S\HH / \kappa \HH$ to $T_p \S^+_T$ is orientation-preserving and conformal. The flags of $\Phi_1$ are spanned by $p \R$ and the directions in which $D \phi_1$ sends $s_i (\kappa), s_j (\kappa) \in \check{\kappa}\$\R^3$. The group $SL_2\$$ acts on $\pH \cong \R^{1,4}$: for a point in $\R^{1,4}$ corresponding to a Hermitian matrix $S \in \pH$, $A \in SL_2\$$ acts by $A.S = AS \overline{A}^T$. This is clearly a group action on $2 \times 2$ Hermitian matrices; we show it preserves paravector Hermitian matrices in the following lemma. \begin{lem} If $S \in \pH$ and $A \in SL_2\$$ then $A.S = AS \overline{A}^T \in \pH$. Further, $\phi_1$ is equivalent with respect to this action. \end{lem} \begin{proof} For $\kappa \in S\HH$ we have $A \kappa \in S\HH$ (\reflem{action_preserves_spinors}) \[ \phi_1 (A \kappa) = (A \kappa) \overline{(A \kappa)}^T = A \kappa \overline{\kappa}^T \overline{A}^T = A \phi_1 (\kappa) \overline{A}^T = A.\phi_1 (\kappa). \] We claim that the action of $A$ on $\pH \cong \R^{1,4}$ preserves $L^+$: any $p \in L^+$ can be written as $p = \phi_1 (\kappa)$ for some $\kappa \in S\HH$, and then the above calculation shows $A.p = \phi_1 (A \kappa) \in L^+$. Since vectors in $L^+$ span $\R^{1,4}$ and the action of $A$ on $\pH \cong \R^{1,4}$ is linear, the action of $A$ must preserve $\R^{1,4}$. Thus for any $S \in \pH$ we have $A.S \in \pH$ also. \end{proof} Since the action of $A$ preserves $L^+$, it lies in $O(1,4)^+$. (In fact, it lies in $SO(1,4)^+$, as is well known; this also follows from arguments below.) We also note that $SL_2\$$ acts transitively on $L^+$, since $\phi_1$ has image $L^+$ and the action of $SL_2\$$ on $S\HH$ is transitive (\reflem{action_preserves_spinors}). The action of $SL_2\$$ on $L^+$, via its derivative, extends to an action of each $A \in SL_2\$$ on each tangent space to $L^+$. As $A$ acts linearly, we also denote the derivative by $A$. Precisely, if $p \in L^+$ then $A$ sends $p \mapsto A.p \in L^+$ and sends $T_p L^+ \To T_{A.p} L^+$. Since the action is by linear maps in $O(1,4)^+$, this is an isometry of tangent spaces. The equivariance of $\phi_1$ yields the following equivariance property on its derivative \[ A. D_\kappa \phi_1 (\nu) = D_{A \kappa} \phi_1 (A \nu), \] for $\nu \in T_\kappa S\HH$. Indeed, $A.D_\kappa \phi_1 (\nu) = A.(\kappa \overline{\nu}^T + \nu \overline{\kappa}^T) = (A \kappa) \overline{(A \nu)}^T + (A \nu) \overline{(A \kappa)}^T = D_{A \kappa} \phi_1 (A\nu)$. \begin{lem} The action of $A \in SL_2\$$ on $L^+$ and $TL^+$ above yields an action of $SL_2\$$ on the space $\MF$ of multiflags, given by \[ [[p, v^i, v^j]] \mapsto [[A.p, A.v^i, A.v^j]] \] \end{lem} \begin{proof} First, we show that $[[A.p, A.v^i, A.v^j]]$ is in fact a multiflag. Since $p \in L^+$ we have $A.p \in L^+$, and as $v^i, v^j \in T_p L^+$ we have $A.v^i, A.v^j \in T_{A.p} L^+$. Now $\langle v^i, v^j \rangle = 0$, and as the action of $A$ lies in $O(1,4)^+$ we have $\langle A.v^i, A.v^j \rangle = 0$. So indeed $[[A.p, A.v^i, A.v^j]] \in \MF$. To show that the map is well defined, suppose $[[p, v^i, v^j]] = [[p, w^i, w^j]]$. Then $a^i p + b^i v^i + c^i w^i = 0$ and $a^j p + b^j v^j + c^j w^j = 0$, with each triple $a,b,c$ and $b,c$ of opposite sign. Applying $A$ to these equations we have $a^i A.p + b^i A.v^i + c^i A.w^i = 0$ and $a^j A.p + b^j A.v^j + c^j A.w^j = 0$. Thus $[[A.p, A.v^i, A.v^j]] = [[A.p, A.w^i, A.w^j]]$. \end{proof} \begin{lem} The map $\Phi_1 \colon S\HH \To \MF$ is $SL_2\$$-equivariant. \end{lem} \begin{proof} We have $\Phi_1 (\kappa) = [[ \phi_1 (\kappa), D_\kappa \phi_1 (s_i \kappa), D_\kappa \phi_1 (s_j \kappa) ]]$ so we must show the following multiflags coincide: \begin{align*} \Phi_1 (A \kappa) &= [[ \phi_1 (A \kappa), D_{A\kappa} \phi_1 (s_i (A \kappa)), D_{A \kappa} \phi_1 (s_j (A \kappa)) ]], \\ A.\Phi_1 (\kappa) &= [[ A.\phi_1 (\kappa), A.D_\kappa \phi_1 (s_i \kappa), A.D_\kappa \phi_1 (s_j \kappa) ]] = [[ \phi_1 (A \kappa), D_{A \kappa} \phi_1 ( A s_i (\kappa)), D_{A \kappa} \phi_1 (A s_j (\kappa)) ]], \end{align*} where we used equivariance of $\phi_1$ and its derivative. Now by \refeqn{action_preserves_bracket}, we have $\{ A \kappa, A s_i (\kappa) \} = \{ \kappa, s_i (\kappa) \}$ and $\{ A \kappa, A s_j (\kappa) \} = \{ \kappa, s_j (\kappa) \}$, and by \refeqn{inner_product_with_sv}, these are negative multiples of $i$ and $j$ respectively. By \reflem{when_flags_equal} we then have equalities of flags $[[\phi_1 (A \kappa), D_{A \kappa} \phi_1 (A s_i (\kappa))]] = [[\phi_1 (A \kappa), D_{A \kappa} (s_i (A \kappa))]]$ and $[[\phi_1 (A \kappa), D_{A \kappa} \phi_1 (A s_j (\kappa))]] = [[\phi_1 (A \kappa), D_{A \kappa} (s_j (\kappa))]]$. Thus the two multiflags are equal. \end{proof} Being linear and preserving $L^+$, the action of $A$ preserves the rays along $L^+$. So in mapping $p \mapsto Ap$, the tangent direction $p \R \subset T_p L^+$ is mapped to the tangent direction $A p \R \subset T_{Ap} L^+$. The action of $A$ thus yields an action on the quotient spaces, \[ \frac{T_p L^+}{p\R} \To \frac{T_{A.p} L^+}{A.p\R}. \] These are isometric to tangent spaces $T_p \S^+_T$, as in \refeqn{celestial_sphere_light_cone_quotient}, with inwards and outwards orientations as in \refsec{orientations}, and by \reflem{spinor_quotient_conformal}, $D_\kappa \phi_1$ maps the quotients $T_\kappa S\HH / \kappa \HH$ onto them in conformal and orientation-preserving fashion. For $\kappa_0 \in S\HH$ and $A \in SL_2\$$, let $\kappa_1 = A \kappa_0$ and $p_0 = \phi_1 (\kappa_0)$, $p_1 = \phi_1 (\kappa_1)$. By equivariance of $\phi_1$ we have $A.p_0 = p_1$. Then we have the following diagram of maps. \[ \begin{array}{ccc} \frac{T_{\kappa_0} S\HH}{\kappa_0 \HH} & \stackrel{D_{\kappa_0} \phi_1}{\To} & \frac{T_{p_0} L^+}{p_0 \R} \\ A \downarrow && \downarrow A \\ \frac{T_{\kappa_1} S\HH}{\kappa_1 \HH} & \stackrel{D_{\kappa_1} \phi_1}{\To} & \frac{T_{p_1} L^+}{p_1 \R} \end{array} \] \begin{lem} With respect to the outward orientation on each $T_p L^+ / p \R$, all maps in the above diagram are orientation-preserving $\R$-linear conformal isomorphisms. Moreover, the diagram is ``conformally commutative'': for $\nu \in T_\kappa S\HH / \kappa \HH$ we have \[ D_{\kappa_1} \phi_1 (A \nu) = \frac{|\kappa_0|^2}{|\kappa_1|^2} A.(D_{\kappa_0} \phi_1 (\nu)). \] \end{lem} \begin{proof} By \reflem{spinor_quotient_conformal}, the top and bottom horizontal maps are conformal and orientation-preserving, with scaling factors $-4|\kappa_0|^2$ and $-4|\kappa_1|^2$ respectively. By \reflem{action_on_spinor_quotients_conformal}, the left vertical map is conformal and orientation-preserving, with scaling factor $|\kappa_0|^4 |\kappa_1|^{-4}$. As $A$ acts by linear maps in $O(1,4)^+$, the right vertical map is an isometry. The top left space has an orthogonal oriented basis represented by $(s_1(\kappa_0), s_i(\kappa_0), s_j (\kappa_0))$, which maps under $D_{\kappa_0} \phi_1$ to an orthogonal oriented basis of $T_{p_0} L^+$; the images of $s_i (\kappa_0)$ and $s_j (\kappa_0)$, together with $p_0 \R $, span the $i$-flag and $j$-flag of $\Phi_1 (\kappa_0)$ respectively. Taking a quotient by $p_0 \R$ sends the $i$-flag and $j$-flag of $\Phi_1 (\kappa_0)$ to oriented lines (the ``$i$-line'' and ``$j$-line'') directed by the equivalence class of $D_{\kappa_0} \phi_1 (s_i (\kappa_0))$ and $D_{\kappa_0} \phi_1 (s_j (\kappa_0))$. Under $A$, these map to the equivalence classes of $D_{A \kappa_0} \phi_1 (A s_i (\kappa_0)) = D_{\kappa_1} \phi_1 (A s_i (\kappa_0))$ and $D_{A \kappa_0} \phi_1 (A s_j (\kappa_0)) = D_{\kappa_1} \phi_1 (A s_j (\kappa_0))$ in $T_{p_1} L^+/p_1 \R$. On the other hand, the orthogonal basis of the top left space maps under $A$ to an oriented orthogonal oriented basis of $T_{\kappa_1} S\HH/\kappa_1 \HH$, which by \reflem{A_on_quotient_spinors} is represented by a real multiple of $(s_1 (A \kappa_0), s_i (A \kappa_0), s_j (A \kappa_0)) = (s_1 (\kappa_1), s_i (\kappa_1), s_j (\kappa_1))$. Under $D_{\kappa_1} \phi_1$, these map to an oriented equal-length oriented basis of $T_{p_1} L^+/p_1 \R$, such that the $i$-flag and $j$-flag of $\Phi_1 (A \kappa_0) = \Phi_1 (\kappa_1)$ project in this quotient to $i$- and $j$-lines directed by $D_{A \kappa_0} \phi_1 ( s_i (A \kappa_0)) = D_{\kappa_1} \phi_1 (s_i (\kappa_1))$ and $D_{A \kappa_0} \phi_1 (s_j (A \kappa_0)) = D_{\kappa_1} \phi_1 (s_j (\kappa_1))$. By equivariance of $\Phi_1$, these $i$- and $j$-lines in $T_{p_1} L^+ / p_1 \R$ yield the same flags, so they must be positive multiples of each other. A similar argument with the derivatives in the $s_1(\kappa_0)$ direction shows that, either way around the diagram, the standard basis of the top left space represented by $(s_1 \kappa_0, s_i \kappa_0, s_j \kappa_0)$ is sent to a positive multiple of the standard basis of the bottom right space represented by $(D_{\kappa_1} \phi_1 (s_1 \kappa_1), D_{\kappa_1} \phi_1 (s_i \kappa_1), D_{\kappa_1} \phi_1 (s_j \kappa_1))$. Since the scaling factors are all known, the overall factor is $|\kappa_0|^2/|\kappa_1|^2$ as claimed. Moreover, as the maps commute up to a positive factor, and as 3 of the 4 maps are orientation-preserving, the remaining map must be orientation preserving also. (This shows directly that $A$ acts by orientation-preserving linear maps, i.e. maps in $SO(1,4)^+$.) \end{proof} There is also a natural action of $SL_2\$$ on decorated ideal points, as follows. Let $(\ell, \psi) \in \S^{+D}$. Since $\ell$ is a line in $L^+$, $A$ sends $\ell$ to another line $A.\ell$ in $L^+$, and as $A$ acts on $\R^{1,4}$ by linear maps in $SO(1,4)^+$, it also sends the 4-plane $\ell^\perp$ to another 4-plane $A.\ell^\perp$. (Note $(A.\ell)^\perp = A.(\ell^\perp)$.) Thus $A$ yields a well-defined map $\ell^\perp / \ell \To A. \ell^\perp / A.\ell = A.(\ell^\perp/\ell)$. Since $A$ acts by an element of $SO(1,4)^+$, this action of $A$ is orientation-preserving. Composing with $\psi \colon \$\R^3 \To \ell^\perp / \ell$ yields a map $A. \psi \colon \$\R^3 \To A.(\ell^\perp / \ell)$. Since $\psi$ is orientation-preserving, $\R$-linear and conformal, so is $A.\psi$. We can thus define $A.(\ell, \psi) = (A.\ell, A.\psi)$. \begin{lem} The correspondence $\Psi \colon \MF \To \S^{+D}$ of \refprop{multiflags_ideal_decorations} is $SL_2\$$-equivariant. \end{lem} \begin{proof} Both actions are essentially induced by the action of $SL_2\$$ on $\R^{1,4}$. Let $[[p, v^i, v^j]]$ be a multiflag corresponding to a decorated ideal point $(\ell, \psi)$. So $\ell = p \R$ and $\psi \colon \$\R^3 \To \ell^\perp/\ell$ sends $i,j$ to the equivalence classes of $v^i,v^j$ respectively. Now consider $A.[[p,v^i,v^j]] = [[A.p,A.v^i,A.v^j]]$ and $A.(\ell, \psi) = (A.\ell, A.\Psi)$. Then we have $A.\ell = A.p \R$, and $A.\Psi(i), A.\Psi(j)$ are the equivalence classes of $A.v^i, A.v^j$ respectively. So the multiflag and decorated ideal point again correspond, and we have equivariance. \end{proof} \begin{example} \label{Eg:equivariance_of_flags_at_10} We consider $\Phi_1 (\kappa)$ for $\kappa = (x,0) = \kappa_0 x$, where $\kappa_0 = (1,0)$ and $|x| = 1$, i.e. $x \in S^3$. By \reflem{phi1_fibres} and \refeg{Dphi1_at_10} these are precisely the $\kappa$ such that $\phi_1(\kappa) = \phi_1 (\kappa_0) = p_0 = (1,0,0,0,1) \in \S^+_1$. We have \begin{equation} \label{Eqn:S3_matrices} \kappa = A \kappa_0 \quad \text{where} \quad A = \begin{pmatrix} x & 0 \\ 0 & x' \end{pmatrix} \in SL_2\$, \end{equation} since $\pdet A = x^* x' = 1$. At $p_0$, the tangent space to $\S^+_1$ is the $WXY$ 3-plane. From \refeg{Phi1_of_k0} and equivariance we have $\Phi_1 (\kappa) = A.\Phi_1 (\kappa_0) = [[p_0, A.\partial_X, A.\partial_Y]]$. We compute \[ A.\partial_X = A \partial_X \overline{A}^T = \begin{pmatrix} x & 0 \\ 0 & x' \end{pmatrix} \begin{pmatrix} 0 & i \\ -i & 0 \end{pmatrix} \begin{pmatrix} \overline{x} & 0 \\ 0 & x^* \end{pmatrix} \\ = \begin{pmatrix} 0 & xix^* \\ -x'i\overline{x} & 0 \end{pmatrix} \\ = \begin{pmatrix} 0 & \sigma(x)(i) \\ \overline{\sigma(x)(i)} & 0 \end{pmatrix}, \] and $A.\partial_Y$ is the same matrix with $i$ replaced with $j$. Thus $A.\partial_X, A.\partial_Y$, which together with $p_0$ span the $i$- and $j$-flags of $\Phi_1 (\kappa)$, are obtained from $\partial_X, \partial_Y$ by the rotation $\sigma(x)$ in the $WXY$ 3-plane. As $x$ varies over $S^3$, the multiflag $\Phi_1(\kappa)$ rotates via the representation $\sigma$ of \refdef{rho}, which provides a homomorphism and double cover $S^3 \To SO(3)$ (\refsec{actions_on_paravectors}), and hence $\Phi_1 (\kappa)$ rotates through all multiflags based at $p_0$. Moreover, $\Phi(\kappa) = \Phi(\kappa_0)$ precisely when $x = \pm 1$, i.e. $\kappa = \pm \kappa_0$. If we restrict to complex $x = e^{i \theta}$, then $\sigma(x)$ is a rotation of $2\theta$ about $-ik = j$, so the $j$-flag remains constant at $[[p_0, \partial_Y]]$, while the $i$-flag rotates in the $WX$-plane through $2\theta$. So on the copy of $\R^{1,3}$ given by $Y = 0$, multiflags reduce to the flags of \cite{Mathews_Spinors_horospheres} and \cite{Penrose_Rindler84}. \end{example} As $SL_2\$$ acts transitively on $L^+$, and by the above example acts transitively on the space $\MF_{p_0}$ of multiflags at $p_0$, then $SL_2\$$ acts transitively on $\MF$, with the stabiliser of each $p \in L^+$ being an $S^3 \cong \Spin(3)$ subgroup conjugate to the group of matrices in \refeqn{S3_matrices} above. It follows that $\Phi_1$ is surjective onto $\MF$, and $\Phi(\kappa_1) = \Phi(\kappa_2)$ iff $\kappa_1 = \pm \kappa_2$. Topologically, $\Phi_1 \colon S\HH \To \MF$ is a map $S^3 \times S^3 \times \R \To S^3 \times SO(3) \times \R$ (\reflem{topology_of_SH}, \refsec{multiflags}), which is a double cover. \section{Horospheres and decorations} \label{Sec:Minkowski_horospheres} We now turn to hyperbolic geometry, and define spaces of horospheres $\Hor$ and decorated horospheres $\Hor^D$, and maps $\phi_2, \Phi_2$ of the commutative diagram \refeqn{main_thm_diagram}. \subsection{Hyperbolic geometry in general} \label{Sec:hyp_geom_general} The hyperboloid model $\hyp^4$ of hyperbolic 4-space is \[ \hyp^4 = \left\{ x = (T,W,X,Y,Z) \in \R^{1,4} \; \mid \; \langle x,x \rangle = 1, \; T>0 \right\}. \] Thus $\hyp^4$ is a spacelike codimension-1 submanifold of $\R^{1,4}$, with tangent spaces $T_x \hyp^4 = x^\perp$. The boundary at infinity $\partial \hyp^4$ given by the celestial sphere $\S^+ \cong S^3$, whose points are the lines $\ell$ in $L^+$. The isometry group $\Isom^+ \hyp^4$ is isomorphic to $SO(1,4)^+$ which, acting linearly on $\R^{1,4}$ in the standard way, preserving $L^+$ and $\hyp^4$. This group acts on the celestial sphere $\S^+$ by conformal orientation-preserving maps, which are the orientation-preserving M\"{o}bius transformations of $\S^+ \cong S^3$. We saw in \refsec{SL2_on_paravectors_etc} that $SL_2\$$ acts on $\R^{1,4}$ by linear maps in $SO(1,4)^+$. One can check that the resulting map $SL_2\$ \To SO(1,4)^+$ has kernel $\{\pm 1\}$. Indeed, as discussed in \refsec{Clifford_properties}, this is a spin double cover, and we have isomorphisms $PSL_2\$ \cong SO(1,4)^+ \cong \Isom^+ \hyp^4$. So $SL_2\$$ is also the spin double cover of the isometry group $\Isom^+ \hyp^4$, and we may regard its elements as \emph{spin isometries} of $\hyp^4$; we denote this group $\Isom^S \hyp^4$. \subsection{Horospheres and their geometry} \label{Sec:horospheres_geometry} As in \cite{Penner87} for $\R^{1,2}$ and \cite{Mathews_Spinors_horospheres} for $\R^{1,3}$, horospheres $\h$ in $\hyp^4 \subset \R^{1,4}$ are given by intersections with affine hyperplanes $\Pi$ of the form $\{ x \mid \langle x, p \rangle = 1 \}$ where $p \in L^+$. Such planes have (Minkowski) normal $p$ lightlike and hence we call them lightlike. As $\h = \hyp^4 \cap \Pi$, the tangent space to $\h$ at a point $q$ is $q^\perp \cap p^\perp$. The line $\ell = p \R$ is a point of $\S^+$ and we say $\ell$ is the \emph{centre} of $\h$. The following definition follows \cite{Mathews_Spinors_horospheres}. \begin{defn} The set of horospheres in $\hyp^4$ is denoted $\Hor$. The map $\phi_2 \colon L^+ \To \Hor$ sends $p \in L^+$ to the horosphere with equation $\langle x, p \rangle = 1$. The map $\phi_2^\partial \colon L^+ \To \partial \hyp^4$ sends $p$ to the point at infinity of $\phi_2 (p)$. \end{defn} As in other dimensions, $\phi_2$ is a diffeomorphism, with both $L^+$ and $\partial \hyp^4$ diffeomorphic to $S^3 \times \R$. The map $\phi_2^\partial$ can be regarded as projectivisation $L^+ \To S^3$. As $SL_2\$$ acts on $\hyp^4$ via hyperbolic isometries, it also acts on $\partial \hyp^4$ and $\Hor$. As in other dimensions, a horosphere $\phi_2 (p)$ with equation $\langle x, p \rangle = 1$ is sent to a horosphere with equation $\langle A^{-1}.x, p \rangle = 1$, which is equivalent to $\langle x, A.p \rangle = 1$, so $A.\phi_2 (p) = \phi_2 (A.p)$ and $\phi_2$ is $SL_2\$$-equivariant. Indeed, the fact that $\phi_2$ is an $SL_2\$$-equivariant diffeomorphism implies that $A \in SL_2\$$ fixes a point $p \in L^+$ iff it fixes the corresponding horosphere $\h = \phi_2 (p)$. As is well known, a horosphere is isometric to a Euclidean space. We can see this directly as follows. \begin{lem} \label{Lem:projection_for_horosphere} Let $p \in L^+$ and $\h = \phi_2 (p)$. Then the projection $\pi \colon \R^{1,4} \To \frac{\R^{1,4}}{p \R}$ restricts to an isometry $\h \To \frac{\Pi}{p \R}$, which is isometric to $\frac{T_p L^+}{p \R}$, and $\E^3$ with negative definite inner product. \end{lem} Note here that $\Pi$ is an affine 4-plane, so $\Pi/p \R$ is not a quotient of vector spaces, but an affine 3-plane in $\R^{1,4}/p \R$, parallel to $p^\perp / p \R$, whose points are $x + p \R$ over $x \in \Pi$. Here $\E^3$ denotes $\R^3$ with the Euclidean metric. \begin{proof} Each ray parallel to $p \R$ in $\Pi $ intersects $\hyp^4$ at a single point of $\h$, so the restriction of $\pi$ to $\h$ yields a diffeomorphism onto $\Pi/p \R$. At each $q \in \h$, we have $T_q \h = q^\perp \cap p^\perp$. As $\Pi$ is parallel to $p^\perp$, each tangent space of $\Pi$ is $p^\perp$, and hence each tangent space of $\Pi / p \R$ is $p^\perp / p \R$. Then the derivative of $\pi|_\h \colon \h \To \Pi/p \R$ at each point $q \in \h$ is a linear isomorphism $q^\perp \cap p^\perp \To p^\perp / p \R$, which is a restriction of the projection $p^\perp \To p^\perp/p \R$. By \reflem{projection_along_light}, this map preserves inner products, hence is an isometry. As $\Pi$ is parallel to $p^\perp$, we have an isometry $\Pi/p \R \cong p^\perp/p \R = T_p L^+ / p \R$. This spacelike 3-dimensional vector space is isometric to $\E^3$ with negative definite inner product. \end{proof} \begin{example} \label{Eg:phi_2_p0} We compute $\phi_2 (p_0)$ where $p_0 = (1,0,0,0,1)$ as in \refeg{Phi1_of_k0}. This horosphere $\h_0$ is the intersection of $\hyp^4$ with the plane $\Pi_0$ defined by $\langle x, p_0 \rangle = 1$, i.e. $T-Z = 1$. It contains the point $q_0 = (1,0,0,0,0)$, at which the tangent space $q_0^\perp \cap p_0^\perp$ has basis $\partial_W, \partial_X, \partial_Y$, which also represents a basis of $p_0^\perp/p_0 \R$. \end{example} Now we consider hyperbolic isometries which preserves a horosphere $\h$; or equivalently, preserve a point $p \in L^+$ such that $\phi_2 (p) = \h$. Precisely, let \[ \B_\h = \{ A \in SL_2\$ \mid A.\h = \h\}, \quad \P_\h = \left\{ A \in \P \cup \{1\} \mid A.\h = \h, \right\}. \] Recall $\P$ is the set of parabolic translation matrices of \refdef{parabolic}, and $\P \cup \{1\}$ adjoins the identity matrix. Let $\underline{\B_\h}$, $\underline{\P_\h}$ be their images in $PSL_2\$$. As stabilisers of a group action, $\B_\h$ and $\underline{\B_\h}$ are subgroups of $SL_2\$$ and $PSL_2 \$ $respectively. If the action of $A \in SL_2\$$ takes $\h$ to another horosphere $\h'$, then we have $\B_{h'} = A \B_\h A^{-1}$ and, as $\P$ is invariant under conjugation (\reflem{parabolic_facts}(iv)), $\P_{\h'} = A \P_\h A^{-1}$. As $SL_2\$$ acts transitively on $L^+$ (\refsec{SL2_on_paravectors_etc}), then $SL_2\$$ acts transitively on $\Hor$. So for any two horospheres $\h,\h'$, the the groups $\B_\h, \B_{\h'}$ are isomorphic, indeed conjugate subgroups, as are $\P_\h, \P_{\h'}$. We can also describe $\B_\h$ and $\P_\h$ in terms of spinors. \begin{lem} \label{Lem:parabolic_preserve_spinor} Suppose $\kappa \in S\HH$, $p = \phi_1 (\kappa) \in L^+$ and $\h = \phi_2 (p)$. Then $A \in \P_\h$ iff $A.\kappa = \kappa$, and $A \in \B_\h$ iff $A.\kappa = \kappa x$ for some $x \in \HH$ such that $|x| = 1$. \end{lem} In other words, $\B_\h$ consists of those matrices in $SL_2\$$ which have $\kappa$ as an eigenvector with right eigenvalue a unit quaternion, and $\P_\h$ the subset with eigenvalue $1$. In particular, as the stabiliser of $\kappa$, this implies that $\P_\h$, and hence also $\overline{\P_\h}$, is a group. \begin{proof} We have $A \in \B_\h$ iff $A.p = p$, which by equivariance of $\phi_1$ is equivalent to $\phi_1 (A. \kappa) = \phi_1 (\kappa)$. By \reflem{phi1_fibres}, this is equivalent to $A.\kappa = \kappa x$ where $|x| = 1$. If $A \in \P_\h$ then we again have $A.\kappa = \kappa x$, but then by \reflem{parabolic_on_spinors}(i) we have $x=1$. Conversely, if $A.\kappa = \kappa$ then by equivariance of $\phi_1$ we have $A.p = p$, so $A.\h = \h$. Moreover, by \reflem{parabolic_on_spinors}(ii), $A = 1$ or $A \in \P$. As $A \in \P \cup \{1\}$ and $A.\h = \h$ then $A \in \P_\h$. \end{proof} \begin{lem} \label{Lem:action_of_parabolics} For any horosphere $\h$, $\B_\h \cong \Isom^S \E^3 \cong \R^3 \rtimes \Spin(3)$, and $\underline{\B_\h} \cong \Isom^+ \E^3 \cong \R^3 \rtimes SO(3)$. Moreover, $\P_\h$ and $\underline{\P_\h}$ are their respective $\R^3$ translation subgroups. \end{lem} Here in standard fashion we regard $\Isom^+ \E^3 = \R^3 \rtimes SO(3)$, so that $\Isom^S \E^3 = \R^3 \rtimes \Spin(3)$. \begin{proof} Since all $\B_\h \cong \B_{\h'}$, $\underline{\B_\h} \cong \underline{\B_{\h'}}$, $\P_\h \cong \P_{\h'}$, $\underline{\P_\h} \cong \underline{\P_{\h'}}$ are isomorphic via conjugation by an isometry from $\h$ to $\h'$, it suffices to consider the single horosphere $\h_0 = \phi_2 (p_0) = \phi_2 \circ \phi_1 (\kappa_0)$ of \refeg{Dphi1_at_10} and \refeg{phi_2_p0}. We now consider the spinors $\kappa$ such that $\phi_1 (\kappa) = p_0$. From \reflem{phi1_fibres} and \refeg{Dphi1_at_10}, $\phi_1 (\kappa) = p$ iff $\kappa = \kappa_0 x$ where $|x| = 1$. By equivariance of $\phi_1$ and $\phi_2$, $\B_\h$ is the set of $A \in SL_2\$$ that preserve the set of spinors $\{(x,0) \mid |x| = 1 \}$. Thus \[ \B_{\h_0} = \left\{ \begin{pmatrix} a & b \\ 0 & d \end{pmatrix} \in SL_2\$ \; \mid \; |a| = 1 \right\}. \] The requirement $A \in SL_2\$$ implies that $\pdet A = 1$ and that $ab^*, b^* d \in \$\R^3$. From $\pdet A = 1$ we have $a^* d = 1$, so $|d| = 1$ and $d = a^{-1*} = a'$. So $A \in \B_{\h_0}$ yields the M\"{o}bius transformation \[ v \mapsto (av+b)d^{-1} = ava^* + ba^* = \sigma(a)(v) + ba^*, \quad v \in \$\R^3. \] Since $|a| = 1$ and $ba^* \in \$\R^3$, these are precisely the orientation-preserving Euclidean isometries of $\$\R^3$, with $\sigma(a)$ acting as a rotation via $\sigma \colon S^3 \To SO(3)$. Precisely, identifying $S^3 \subset \HH^2$ with $\Spin(3)$ and $\$\R^3$ with $\R^3$, we have \[ \B_{h_0} \stackrel{\cong}{\To} \Isom^S \E^3 = \R^3 \rtimes \Spin(3), \quad \begin{pmatrix} a & b \\ 0 & a' \end{pmatrix} \mapsto \left( ba^*, a \right), \] which double covers an isomorphism $\underline{\B_{h_0}} \To \Isom^+ \E^3 = \R^3 \rtimes SO(3)$ sending the image of the matrix above to $(ba^*, \sigma(a))$. Considering \reflem{parabolic_conditions}, the subgroup $\P_{\h_0}$ of $\B_{\h_0}$ thus consists of those matrices of the form of \reflem{parabolic_conditions}(iii) with $c=0$, together with the identity matrix, i.e. \begin{equation} \label{Eqn:Ph0} \P_{\h_0} = \left\{ \begin{pmatrix} 1 & aa^* \\ 0 & 1 \end{pmatrix} \; \mid \; a \in \HH \right\} = \left\{ \begin{pmatrix} 1 & v \\ 0 & 1 \end{pmatrix} \; \mid \; v \in \$\R^3 \right\}, \end{equation} the latter equality by \reflem{paravector_square_root}. Thus $\P_{\h_0} \cong \$\R^3 \cong \R^3$, precisely as the translation subgroup of $\Isom^S \R^3$. Similarly $\underline{\P_{\h_0}}$ is the $\R^3$ translation subgroup of $\underline{\B_{\h_0}}$. \end{proof} \begin{lem} \label{Lem:parabolic_identity_on_quotient} Let $\h = \phi_2 (p)$. If $A \in \P_\h$ then $A$ acts as the identity on $p^\perp / p \R$. \end{lem} To see that this makes sense, note that $A \in \P_\h$ preserves $p$, hence $p\R$, and also preserves $p^\perp$, hence has a well-defined action on $p^\perp/p\R$. \begin{proof} We first argue that, by transitivity, it suffices to prove the statement for one horosphere. Given two horospheres $\h, \h'$, there is a $B \in SL_2\$$ such that $B.\h = \h'$, and $\P_{\h'} = B \P_\h B^{-1}$. Letting $\h = \phi_2 (p)$ and $\h' = \phi_2 (q)$ then $B.p = q$. So $B$ yields isomorphisms $p \R \To q \R$ and $p^\perp \To q^\perp$, hence an isomorphism $(p^\perp/p\R) \To q^\perp/q\R$. If $A \in \P_\h$ acts as the identity on $p^\perp / p\R$, then $BAB^{-1}$ acts as the identity on $q^\perp / q \R$, and conversely. Thus we consider the horosphere $\h_0 = \phi_2 (p_0)$ of \refeg{phi_2_p0}, and the group $\P_{\h_0}$ of \refeqn{Ph0}. The 4-plane $p_0^\perp$ has basis $p, \partial_W, \partial_X, \partial_Y$, so $p^\perp / p \R$ has basis represented by $\partial_W, \partial_X, \partial_Y$. To find the action of $A \in \P_{\h_0}$ on $p_0^\perp / p_0 \R $ it thus suffices to find the action on points of the form $(0,W,X,Y,0)$. Letting $S \in \pH$ correspond to $(0,W,X,Y,0)$ and $w = W+Xi+Yj \in \$\R^3$ we have \begin{align*} AS\overline{A}^T = \begin{pmatrix} 1 & v \\ 0 & 1 \end{pmatrix} \begin{pmatrix} 0 & w/2 \\ \overline{w}/2 & 0 \end{pmatrix} \begin{pmatrix} 1 & 0 \\ \overline{v} & 1 \end{pmatrix} &= \frac{1}{2} \begin{pmatrix} (v\overline{w} + w\overline{v}) & w \\ \overline{w} & 0 \end{pmatrix} = \begin{pmatrix} v \cdot w & w/2 \\ \overline{w}/2 & 0 \end{pmatrix} \end{align*} thus $(0,W,X,Y,0) \mapsto (0,W,X,Y,0) + (v\cdot w) p_0$. So the action of $A$ on $p_0^\perp / p_0 \R $ is the identity. \end{proof} \begin{lem} \label{Lem:parabolic_translation_on_quotient} If $A \in \P_\h$ and $\Pi$ is the 4-plane given by $\langle x, p \rangle = 1$, then $A$ acts on $\Pi/p \R$ as a Euclidean translation. Moreover, $\P_\h$ is isomorphic to the $\R^3$ group of translations of $\Pi/p \R$. \end{lem} \begin{proof} By a similar transitivity argument as in \reflem{parabolic_identity_on_quotient}, it suffices to prove the statement for $\h_0$ of \refeg{phi_2_p0}; let $p_0$, $\Pi_0$, $q_0$ be as there. The plane $\Pi_0$ is a translate of $p_0^\perp$, namely $\Pi_0 = p_0^\perp + q_0$. Taking a general element $A \in \P_{\h_0}$ as in \refeqn{Ph0}, and writing $v \in \$\R^3$ as $v = W+Xi+Yj$ with $W,X,Y \in \R$, we compute \[ A.q_0 = \begin{pmatrix} 1 & v \\ 0 & 1 \end{pmatrix} \begin{pmatrix} 1/2 & 0 \\ 0 & 1/2 \end{pmatrix} \begin{pmatrix} 1 & 0 \\ \overline{v} & 1 \end{pmatrix} = \frac{1}{2} \begin{pmatrix} 1 + |v|^2 & v \\ \overline{v} & 1 \end{pmatrix}. \] Thus $q_0 \mapsto (1 + \frac{1}{2}|v|^2, W,X,Y, \frac{1}{2} |v|^2) = q_0 + \frac{1}{2} |v|^2 p_0 + (0,W,X,Y,0)$, which projects to $q_0 + (0,W,X,Y,0)$ in $\Pi_0 / p_0 \R $. Since $\Pi_0 = q_0 + p_0^\perp$ we have $\frac{\Pi}{p_0 \R} = (q_0 + p_0 \R) + \frac{p_0^\perp}{p_0 \R}$. A general point in $\Pi_0 /p_0 \R $ is of the form $q_0 + r + p_0 \R$, where $r \in p_0^\perp$. By \reflem{parabolic_identity_on_quotient}, $A$ acts as the identity on $p_0^\perp/p \R$, so $A.(q_0 + r + \R p_0) = q_0 + (0,W,X,Y,0) + r + p_0 \R$. In other words, $A$ acts by translation by $(0,W,X,Y,0)$ on $\Pi_0/p_0 \R$. Thus as $v$ varies over all $W+Xi+Yj \in \$\R^3$, we obtain an isomorphism between $\P_{\h_0}$ and the $\R^3$ group of translations of $\Pi_0/p_0 \R$. \end{proof} \begin{prop} \label{Prop:parabolics_are_translations} If $A \in \P_\h$ then $A$ acts on $\h$ by a Euclidean translation. Moreover, $\P_\h$ is isomorphic to the $\R^3$ group of translations of $\h$. \end{prop} \begin{proof} We have $\h \subset \Pi$, and by \reflem{projection_for_horosphere}, the projection $\h \To \Pi/p \R$ is an isometry, where $\h = \phi_2 (p)$. This projection is along the lines of $p \R$. Any $A \in \P_\h$ fixes $p$, so acts on $p \R$ as the identity. Thus the actions of $A$ on $\h$ and $\Pi/p \R$ are equivariant with respect to this isometry. By \reflem{parabolic_translation_on_quotient}, $\P_\h$ acts on $\Pi/p \R$ by translations, and is isomorphic to the $\R^3$ translation group of $\Pi/p \R$; so under the isometry $\Pi / p \R \To \h$, $\P_\h$ acts on $\h$ by translations, and is isomorphic to the $\R^3$ group of parabolic translations of $\h$. \end{proof} \subsection{From multiflags to horospheres and line fields} \label{Sec:multiflags_to_horospheres} Suppose we now have a multiflag $(p, V^i, V^j) = [[p,v^i,v^j]]$, so the $i$-flag $[[p, v^i]]$ has 2-plane $V^i = p \R + \R v^i$ an the $j$-flag has 2-plane $V^j = p \R + \R v^j$. The horosphere $\phi_2 (p)$, being the intersection of $\hyp^4$ with the 4-plane $\langle x, p \rangle = 1$, has tangent space at a point $q \in \phi_2 (p)$ given by $T_q \phi_2 (p) = q^\perp \cap p^\perp$. Thus the $i$-flag and $j$-flag 2-planes intersect the tangent the tangent space to the horosphere in \[ T_q \phi_2 (p) \cap V^i = V^i \cap q^\perp \cap p^\perp = V^i \cap q^\perp, \quad T_q \phi_2 (p) \cap V^j = V^j \cap q^\perp \cap p^\perp = V^j \cap q^\perp, \] since $V^i, V^j$ are subspaces of $p^\perp$. The intersection of the 2-plane $V^i$ and the 4-plane $q^\perp$ is not 2-dimensional, since $V^i$ contains the lightlike $p$, while $q^\perp$ is spacelike; nor can it be 0-dimensional, as both lie in $\R^{1,4}$; hence the intersection is $1$-dimensional and spacelike. Moreover, since the intersection does not lie in the lightlike direction $p \R$, then the orientation on $V^i/p \R$ provided by the $i$-flag determines an orientation on the intersection. Thus the $i$-flag of the multiflag $(p, V^i, V^j)$ naturally provides a 1-dimensional, spacelike, oriented tangent line at each point of the horosphere $\phi_2 (p)$, i.e. and oriented line field, which we call the \emph{$i$-line field}. A similar argument applies to the $j$-flag, which provides a \emph{$j$-line field} $T \phi_2 (p) \cap V^j$ on the horosphere. In a multiflag, the $i$-flag and $j$-flag are orthogonal, as in \refdef{flag_angle}. At any $q \in \phi_2 (p)$, the $i$-line and $j$-line lie in $V^i, V^j$ respectively, and are both transverse to the lightlike $p \R$, so by \reflem{flag_angle_well_defined}, they are orthogonal. Hence a multiflag naturally produces a horosphere decorated with two orthogonal line fields, and we make the following definitions. \begin{defn} \ \begin{enumerate} \item $\Hor^L$ is the set of triples $(\h, L^i, L^j)$ where $\h \in \Hor$, and $L^i, L^j$ are oriented tangent line fields on $\h$ which are everywhere orthogonal. \item The map $\Phi_2 \colon \MF \To \Hor^L$ sends a multiflag $(p, V^i, V^j)$ to the horosphere $\phi_2 (p)$, with the $i$-line field $T \phi_2 (p) \cap V^i$ and $j$-line field $T \phi_2 (p) \cap V^j$. \end{enumerate} \end{defn} The group $SL_2\$$ acts on $\Hor^L$ via $SO(1,4)^+$: such a map acts as an orientation-preserving isometry of $\hyp^4$, sending horospheres to horospheres, with its derivative sending line fields on horospheres to line fields on horospheres. The actions of $SL_2\$$ on both $\MF$ and $\Hor^L$ are both via the linear maps of $A \in SO(1,4)^+$ on $\R^{1,4}$. We have seen $\phi_2$ is equivariant, sending horospheres $\h \to \h'$ correspondingly to points $p \to p'$ on the light cone; moreover the relatively oriented 2-planes $V^i, V^j$, whose intersections with $\h$ determine its oriented line fields, are sent to relatively oriented 2-planes whose intersections with $\h'$ determine its oriented line fields. So $\Phi_2$ is $SL_2\$$-equivariant. As in the 3-dimensional case, we now show that the line fields obtained from multiflags are parallel. As in \cite{Mathews_Spinors_horospheres}, an oriented line field on a horosphere $\h$ is \emph{parallel} if it is invariant under Euclidean translations, i.e. parabolic translations fixing $\h$. Thus, as discussed in the introduction in \refdef{decorated_horosphere}, we define a \emph{decorated horosphere} to be an $(\h, L^i, L^j) \in \Hor^L$ where $L^i, L^j$ are both parallel. $L^i$ and $L^j$ are called the \emph{$i$-decoration} and \emph{$j$-decoration} respectively. The set of decorated horospheres is denoted $\Hor^D$. In other words, a decoration on a horosphere $\h$ consists of a pair of orthogonal parallel oriented line fields. As horospheres have Euclidean geometry, to describe a decoration on a horosphere, it suffices to give two orthogonal directions at one point. Given an orientation on $\h$, the two orthogonal directions uniquely define an oriented orthonormal basis, so a decoration is equivalent to a triple of oriented orthogonal parallel line fields. \begin{prop} \label{Prop:line_fields_parallel} The image of $\Phi_2$ lies on $\Hor^D$. \end{prop} \begin{proof} Let $(p, V^i, V^j) \in \MF$. As $\Phi_1$ is surjective onto multiflags, we have $(p, V^i, V^j) = \Phi_1 (\kappa)$ for some spinor $\kappa$. Let $\Phi_2 (p, V^i, V^j) = (\h, L^i, L^j) \in \Hor^L$. So $\h = \phi_2 (p)$, and we must show that the oriented line fields $L^i = T\h \cap V^i$ and $L^j = T\h \cap V^j$ on $\h$ are parallel. By \reflem{projection_for_horosphere}, we have an isometry $\h \cong \Pi / p \R \cong p^\perp / p \R$, where $\Pi$ is the affine 3-plane given by $\langle x, p \rangle = 1$. induced by projection. The 2-plane $V^i$ satisfies $p \R \subset V^i \subset T_p L^+ = p^\perp$. At a point $q \in \h$, we have the 3-plane $T_q \h = q^\perp \cap p^\perp \subset p^\perp$, and the 2-plane $V^i \subset p^\perp$, which intersect in the line $L^i$. Under projection along $p \R$, the 3-plane $T_q \h$ maps to the Euclidean 3-plane $p^\perp / p \R$, and the 2-plane $V^i$ maps to the line $V^i / p \R$. Their intersection $L^i$ is transverse to $p \R$, so maps to a line in $p^\perp / p \R$, and as $L^i \subset V^i$, this projection must be the same as $V^i / p \R$. Thus, under the isometry $T_q \h \To p^\perp / p \R$ induced by projection along the lines parallel to $p$, $L^i$ maps to $V^i / p \R$, and similarly $L^j$ maps to $V^j / p \R$. In particular, in $p^\perp / p \R$ these two line fields appear as constant line fields (there is no dependence on $q$). Now by \refprop{parabolics_are_translations}, the group $\P_\h$ acts on $\h$ as the $\R^3$ group of Euclidean translations of $\h$. Moreover, by \reflem{parabolic_translation_on_quotient}, $\P_\h$ acts on $\Pi/p \R$ as the $\R^3$ group of Euclidean translations on $\Pi / p \R$. These two actions are equivariant with respect to the projection isometry $\h \To \Pi/p \R$. Since $\P_\h$ acts by translations in $\Pi / p \R$, in which the line fields $V^i / p \R$ and $V^j / p \R$ are constant, $\P_\h$ preserves these line fields. These line fields are the images of $L^i, L^j$ under the isometry $\h \To \Pi / p \R$, so $\P_\h$ preserves $L^i$ and $L^j$. As $\Phi_1,\Phi_2$ are $SL_2\$$-equivariant, and as each $A \in \P$ fixes $\kappa$, each $A \in \P$ also fixes the multiflag $(\h, L^i, L^j)$. As $\P_\h$ acts transitively on $\h \cong p^\perp / p \R$, then $L^i$ and $L^j$ are parallel. \end{proof} Let us also calculate what $\Phi_2$ does, when considered as a map $\S^{+D} \to \Hor^D$. Recall a multiflag $(p, V^i, V^j) \in \MF$ corresponds to a decorated ideal point $(\ell, \psi) \in \S^{+D}$, where $\ell = p \R$ and $\psi \colon \$\R^3 \To \ell^\perp / \ell$ is conformal and orientation-preserving. Writing $(p, V^i, V^j)$ as $[[p, v^i, v^j]]$ where $p$ has $T$-coordinate $T_0$ and $v^i, v^j$ have Minkowski norm $-4T_0^2$, $\psi$ is given by $\psi(i) = v^i + p \R$ and $\psi(j) = v^j + p \R$. On the corresponding horosphere $\h = \phi_2 (p)$, we have the isometry $\h \To \ell^\perp/\ell$ given by projection. Now $\Phi_2$ sends this multiflag or decorated ideal point to $(\h, L^i, L^j) \in \Hor^D$, where $\h = \phi_2 (p)$, and the orthogonal line fields $L^i, L^j$ are defined by intersection with $V^i$ and $V^j$. As we have seen, under the isometry $\h \To \ell^\perp/\ell$, these line fields are directed by $v^i$ and $v^j$. So the line fields $L^i$ and $L^j$ are directed by $\psi(i)$ and $\psi(j)$ respectively. The horosphere $\h$ can be obtained by letting the scale factor of $\psi$ be $-4T_0^2$, then taking $p$ to be the point on $\ell$ with $T$-coordinate $T_0$; we then have $\h = \phi_2 (p)$. \begin{lem} \label{Lem:Phi_2_diffeo} $\Phi_2$ is a diffeomorphism $\MF \To \Hor^D$. \end{lem} \begin{proof} We have seen $\Phi_2$ has image in $\Hor^D$. As $\phi_2$ is a diffeomorphism, every horosphere $\h$ arises in the image of $\Phi_2$. For a given basepoint $p$ and corresponding horosphere $\h$, the multiflags $(p, V^i, V^j)$ based at $p$ correspond bijectively with decorations on $\h$, so $\Phi_2$ is a bijection; indeed both are diffeomorphic to $SO(3)$. Both $\MF$ and $\Hor^D$ are naturally diffeomorphic to $S^3 \times \R \times SO(3)$, and $\Phi_2$ provides a diffeomorphism between them. \end{proof} \subsection{From hyperboloid to disc and upper half space models} \label{Sec:H4_models} We now proceed from the hyperbolic model to the upper half space model $\U$ of $\hyp^4$, denoted as \begin{equation} \label{Eqn:U} \U = \left\{ (w,x,y,z) \in \R^4 \; \mid \; z>0 \right\} \quad \text{with metric} \quad ds^2 = \frac{dw^2 + dx^2+dy^2+dz^2}{z^2}. \end{equation} The plane $z=0$ is identified with $\$\R^3$ via $(w,x,y) = w+xi+yj$, and $\partial \U$ with $\$\R^3 \cup \infty$. In $\U$, horospheres centred at $\infty$ appear as horizontal 3-planes; we call the $z$-coordinate of this plane the \emph{height} of the horosphere. Horospheres centred at other points appear as 3-spheres tangent to $\$\R^3$; we call the maximum of $z$ on the 3-sphere the \emph{Euclidean diameter} of the horosphere, achieved at its \emph{north pole}. A decoration on a horosphere $\h$ can be described by numbers in $\$\R^3$, using the following identification. \begin{defn} \label{Def:U-identification} Let $\h$ be a horosphere in $\U$. Let $p$ be a point of $\h$ at which $T_p \h$ is the $wxy$-plane. Then the \emph{paravector $\U$-identification} is the $\R$-linear isomorphism $\$\R^3 \To T_p \h$ which sends $1,i,j$ to the vectors $\partial_w, \partial_x, \partial_y \in T_p \h$ respectively. \end{defn} The point $p$ in the above definition is the north pole of $\h$, if the centre of $\h$ is not $\infty$; if $\h$ is centred at $\infty$ then $p$ can be any point on $\h$. If $\h$ has centre $\infty$, it appears as a plane parallel to $\$\R^3$, and we can describe a parallel oriented line field by a nonzero element of $\$\R^3$, well defined up to positive multiples, using the paravector $\U$-identification. A decoration $(L^i, L^j)$ is then specified by two nonzero orthogonal elements of $\$\R^3$, again each well defined up to positive multiples. If $\h$ is centred elsewhere, its tangent plane at its north pole is parallel to $\$\R^3$, and so we can \emph{north pole specify} an oriented line field by a nonzero element of $\$\R^3$ up to positive multiples, again using the paravector $\U$-identification; and we can specify a decoration by two nonzero orthogonal elements of $\$\R^3$, each up to positive multiples. We also have the disc model $\Disc$, given by the standard unit ball in $\R^4$, with boundary $\partial \Disc = S^3$. Just as in 3 dimensions, there are standard isometries $\hyp^4 \To \Disc \To \U$, where $\Disc$ is the conformal disc model. The first of these is given by \[ \hyp^4 \To \Disc, \quad (T,W,X,Y,Z) \mapsto \frac{1}{1+T}(W,X,Y,Z). \] On the boundaries, these are given by \begin{equation} \label{Eqn:hyp_models_translations} \partial\hyp^4 \To \partial\Disc, \quad (T,W,X,Y,Z) \mapsto \frac{1}{T} \left( W,X,Y,Z \right) \quad \text{and} \quad \partial \Disc \To \partial \U, \quad (w,x,y,z) \mapsto \frac{w+xi+yj}{1-z} \end{equation} where points of $\partial \hyp^4$ are represented by points in $L^+$. The composition $\partial\hyp^4 \To \partial\U$ sends \begin{align} \label{Eqn:boundary_hyperboloid_to_upper} (T,W,X,Y,Z) \mapsto \frac{W + Xi + Yj}{T-Z}. \end{align} The group $SL_2\$$ acts via isometries on each model, equivariantly with respect to these maps. A spinor $\kappa = (\xi, \eta) \in S\HH$ maps under $\phi_1$ to $p = (T,W,X,Y,Z) \in L^+$ given by \refeqn{phi1_in_coords}. In particular, $W+Xi+Yj = 2 \xi \overline{\eta}$ and $T-Z = 2 |\eta|^2$. Under $\phi_2$, $p$ maps to a horosphere centred at $p \in \partial \hyp^4$, which in $\U = \$\R^3 \cup \{\infty\}$ is given by \[ \frac{W+Xi+Yj}{T-Z} = \frac{2 \xi \overline{\eta}}{2 |\eta|^2} = \xi \eta^{-1} \] A matrix $A \in SL_2\$$ with entries $a,b,c,d$ as in \refdef{SL2H} sends $(\xi, \eta) \mapsto (a \xi + b\eta, c \xi + d \eta)$, so by equivariance, the action of $A$ on $\partial \U$ sends \[ \xi \eta^{-1} \mapsto (a\xi + b \eta)(c\xi + d\eta)^{-1} = \left( a \xi \eta^{-1} + b \right) \left( c \xi \eta^{-1} + d \right)^{-1}, \] which is the standard action by M\"{o}bius transformations, as in \refeqn{Mobius_from_SL2H} or \refeqn{Mobius_from_Clifford_for_quaternions}. We can now prove \refthm{main_thm_4}, giving the explicit description of $\Phi_2 \circ \Phi_1 (\xi, \eta)$. \begin{proof}[Proof of \refthm{main_thm_4}] We have already seen that $\h$ has centre $\xi \eta^{-1}$. We prove the above using transitivity from $\kappa_0 = (1,0)$. Let $\kappa_0, p_0, \h_0, q_0$ be as in \refeg{Phi1_of_k0} and \refeg{phi_2_p0}. So, let $\kappa = \kappa_0 = (1,0)$. From \refeg{Phi1_of_k0} we have $\Phi_1 (\kappa_0) = [[p_0, \partial_X, \partial_Y]]$. Under $\Phi_2$, by \refeg{phi_2_p0}, this maps to $\h_0$, which is centred at $p_0$ and passes through $q_0$. At $q_0$ we have $T_{q_0} \hyp^4$ is the $WXYZ$ 4-plane, and $T_{q_0} \h_0$ is the $WXY$ 3-plane. The $i$-decoration on $\h_0$ at $q_0$ is thus $V^i \cap T_{q_0} \h_0$, where $V^i$ is spanned by $\partial_X$ and $p_0$. Thus the $i$-decoration at $q_0$ is $\partial_X$, and similarly the $j$-decoration at $q_0$ is $\partial_Y$. In $\Disc$, this corresponds to a decorated horosphere centred at $(0,0,0,1)$, passing through $(0,0,0,0)$, and at that point having $i$-decoration $(0,1,0,0)$ and $j$-decoration $(0,0,1,0)$. In $\U$, this corresponds to a horosphere centred at $\infty$, passing through $(0,0,0,1)$, at that point having $i$-decoration $(0,1,0,0) \sim i \in \$\R^3$ and $j$-decoration $(0,0,1,0) \sim j \in \$\R^3$. Thus $\Phi_2 \circ \Phi_1 (\kappa_0)$ is centred at $\infty$, has height $1$, and $i$- and $j$-decorations specified by $i = \sigma(1)(i)$ and $j = \sigma(1)(j)$ respectively. Thus the proposition holds for $\kappa = (1,0)$. We now consider the following matrices and actions on spinors \[ \begin{pmatrix} 0 \\ 1 \end{pmatrix} = \begin{pmatrix} 0 & -1 \\ 1 & 0 \end{pmatrix} \begin{pmatrix} 1 \\ 0 \end{pmatrix}, \quad \begin{pmatrix} \xi \\ 0 \end{pmatrix} = \begin{pmatrix} \xi & 0 \\ 0 & \xi^{-1*} \end{pmatrix} \begin{pmatrix} 1 \\ 0 \end{pmatrix}, \quad \begin{pmatrix} \xi \\ \eta \end{pmatrix} = \begin{pmatrix} \eta^{-1*} & \xi \\ 0 & \eta \end{pmatrix} \begin{pmatrix} 0 \\ 1 \end{pmatrix}. \] So $\Phi_2 \circ \Phi_1 (0,1)$ is obtained from that of $\kappa_0$ by applying $v \mapsto -v^{-1}$, which in $\U$ is a half turn in the 2-plane bounded by $\R \cup \{\infty\}$; the resulting horosphere has Euclidean diameter $1$, $i$-decoration $i = \sigma(1)(i)$, and $j$-direction $j = \sigma(1)(j)$. Similarly, $\Phi_2 \circ \Phi_1 (\xi, 0)$ is obtained from $\Phi_2 \circ \Phi_1 (1,0)$ by applying $v \mapsto \xi v (\xi^{-1*})^{-1} = \xi v \xi^* = \sigma(\xi)(v)$, which is a rotation about $0$ given by $\sigma(\xi/|\xi|)$, followed by a dilation of $|\xi|^2$, hence has centre $\infty$, height $|\xi|^2$, $i$-direction $\sigma(\xi)(i)$, and $j$-direction $\sigma(\xi)(j)$. Similarly, when $\eta \neq 0$, $\Phi_2 \circ \Phi_1 (\xi, \eta)$ is obtained from $\Phi_2 \circ \Phi_1 (0, 1)$ by applying $v \mapsto (\eta^{-1*} v + \xi)\eta^{-1} = \sigma(\eta^{-1*})(v) + \xi \eta^{-1}$, hence has centre $\xi \eta^{-1}$, Euclidean diameter $|\eta|^{-2}$, $i$-direction $\sigma(\eta^{-1*})(i)$, and $j$-direction $\sigma(\eta^{-1*})(j)$. \end{proof} From this description, it is straightforward to see that when we multiply a $\kappa \in S\HH$ by a real $r>0$, the corresponding decorated horosphere is translated by hyperbolic distance $2 \log r$ towards its centre, and the decoration specification is unchanged in the upper half space model. Similarly, when we multiply $\kappa$ on the right by a unit in $\HH$, the horosphere is unchanged, but the decorations rotate. \section{Spin decorations and lambda lengths} \label{Sec:spin_decorations} We now define spin decorations on horospheres in $\hyp^4$, and define quaternionic lambda lengths, generalising \cite{Mathews_Spinors_horospheres}. \subsection{Orientations, frames, paravector identifications} \label{Sec:orientations_frames_spin} We consider $\hyp^4$ to be oriented in a standard fashion, with vectors in the $w,x,y,z$ directions in $\U$ of \refeqn{U} forming an oriented basis. This agrees with the orientation induced on the hyperboloid model from $\R^{1,4}$ by the transverse vector field $\partial_T$ discussed in \refsec{orientations}. A horosphere $\h \subset \hyp^4$ has two normal directions: \emph{outward} of $\h$ towards its centre, and \emph{inward} into $\hyp^4$. Denote the outward and inward unit normal vector fields on $\h$ by $N^{out}, N^{in}$. We consider orthonormal frames in $\hyp^4$ with this orientation, which we simply refer to as \emph{frames}. The set of frames $\Fr$ forms a principal $SO(4)$-bundle $\Fr \To \hyp^4$, and its spin double (universal) cover is a principal $\Spin(4)$ bundle $\Fr^S \To \hyp^4$. Points of $\Fr$ are frames, and we call points of $\Fr^S$ \emph{spin frames}. The orientation-preserving isometry group $\Isom^+ \hyp^4 \cong PSL_2\$$ acts simply transitively on $\Fr$, and indeed via a choice of base frame we may identify $\Fr \cong PSL_2\$$. Similarly, the spin isometry group $\Isom^S \hyp^4 \cong SL_2\$$ acts simply transitively on $\Fr^S$ and via a choice of base spin frame we may identify $\Fr^S \cong SL_2\$$. Two matrices $\pm A \in SL_2\$$ represent the same element of $PSL_2\$$, hence correspond to the two spin frames lifting a common frame, related by $2\pi$ rotation. We can regard elements of $SL_2\$$ as spin isometries of $\hyp^4$, or equivalently, since they form the universal cover of $\Isom^+ \hyp^4$, as homotopy classes of paths of isometries starting at the identity. A decoration $(L^i, L^j)$ on a horosphere $\h$ can be normalised to a pair or orthogonal parallel unit tangent vector fields $(v^i, v^j)$ on $\h$, and we can then construct frame fields along $\h$. \begin{defn} \label{Def:inward_outward_frame_field} Let $v = (v^i, v^j)$ be a decoration on $\h$. \begin{enumerate} \item The \emph{inward frame field} of $v$ is the unique frame field $F^{in}$ on $\h$ of the form $(v^{1,in}, v^i, v^j, N^{in})$ for some $v^{1,in}$. \item The \emph{outward frame field} of $v$ is the unique frame field $F^{out}$ on $\h$ of the form $(v^{1,out}, v^i, v^j, N^{out})$ for some $v^{1,out}$. \end{enumerate} \end{defn} Note that the first vector $v^{1,in}, v^{1,out}$ in each frame is uniquely determined by the requirement that frames are oriented. Both $v^{1,in}, v^{1,out}$ are tangent to $\h$ and orthogonal to $v^i$ and $v^j$. Indeed $v^{1,in} = - v^{1,out}$. Both $F^{in}$ and $F^{out}$ are sections of $\Fr \To \hyp^4$ over $\h$, differing by a half-turn in the 2-plane orthogonal to $v^i,v^j$. The ordering of the elements in each frame is based on the idea that the frame provides ``$1,i,j,k$ directions" and thus provides an isomorphism of the tangent space to $\hyp^4$ with the quaternions, with the tangent space to $\h$ identified with the paravectors. The first three elements of each frame $(v^{1,in/out}, v^i, v^j, v^{1,out})$ forms a basis for $T \h$ at each point of $\h$. The notation $v^i, v^j, v^1$ is intended to suggest the real basis $i,j,1$ of paravectors $\$\R^3$. Indeed, it gives us a way to identify the tangent space of a decorated horosphere with $\$\R^3$, as follows. \begin{defn} \label{Def:paravector_identification} Let $p$ be a point on a horosphere $\h$ with a decoration $(v^i, v^j)$. \begin{enumerate} \item The \emph{inward paravector identification} is the $\R$-linear isomorphism $\$\R^3 \To T_p \h$ which sends $1,i,j$ to the vectors $v^{1,in},v^i,v^j$ of $F^{in}(p)$ respectively. \item The \emph{outward paravector identification} is the $\R$-linear isomorphism $\$\R^3 \To T_p \h$ which sends $1,i,j$ to the vectors $v^{1,out},v^i,v^j$ of $F^{out}(p)$ respectively. \end{enumerate} \end{defn} Thus, each vector in $T_p \h$ can be described as a paravector, in two different ways. The two frames $F^{in}, F^{out}$ have the same $i$- and $j$-directions, but reversed direction of $1$. So the two paravectors are related by the following lemma. \begin{lem} \label{Lem:inward_outward_identification} Let $p, \h, (v^i, v^j)$ be as above, and let $w \in T_p \h$. Let $w_{in}, w_{out} \in \$\R^3$ be the paravectors identified with $w$ using the inward and outward paravector identifications respectively. Then $w_{in} = - \overline{w_{out}}$. \end{lem} Note the operation $w \mapsto -\overline{w}$ takes a general element $a+bi+cj$ of $\$\R^3$ to $-a+bi+cj$. \begin{proof} The inward paravector identification sends $(1,i,j) \mapsto (v^{1,in},v^i,v^j)$, and the outward sends $(1,i,j) \mapsto (v^{1,out}, v^i,v^j)$. Since $v^{1,in} = -v^{1,out}$, we observe that $w_{in}, w_{out}$ have the same imaginary part, but opposite real part, i.e. $w_{in} = - \overline{w_{out}}$. \end{proof} \subsection{Spin decorations and multiflags} \label{Sec:spin_decorations_multiflags} We now have several ways to describe a decoration on a horosphere: oriented line fields $(L^i, L^j)$ as in the original \refdef{decorated_horosphere}; unit vector fields $(v^i, v^j)$ as in \refsec{orientations_frames_spin}; and we also observe that a decoration is specified by its inwards or outward frame fields $F^{in}, F^{out}$. Henceforth, we will even-handedly denote a decoration by this pair of frame fields $F = (F^{in}, F^{out})$. Following \cite{Mathews_Spinors_horospheres} we make the following definitions. \begin{defn} An \emph{outward (resp. inward) spin decoration} on $\h$ is a continuous lift of an outward (resp. inward) frame field from $\Fr$ to $\Fr^S$. \end{defn} Spin decorations come in pairs $W = (W^{in}, W^{out})$, each associated to the other. To describe rotations in a 2-plane we use the convention discussed in \refsec{actions_on_paravectors}, rotating one vector towards another. \begin{defn} \ \label{Def:associated_spin_decorations} \begin{enumerate} \item Let $W^{out}$ be an outward spin decoration on $\h$. The \emph{associated inward spin decoration} is the spin decoration obtained by rotating $W^{out}$ by $\pi$ in the 2-plane orthogonal to $v^i,v^j$, from $N^{out}$ towards $v^{1,out}$. \item Let $W^{in}$ be an inward spin decoration on $\h$. The \emph{associated outward spin decoration} is the spin decoration obtained by rotating $W^{in}$ by $\pi$ in the 2-plane orthogonal to $v^i,v^j$, from $v^{1,in}$ towards $N^{in}$. \item A \emph{spin decoration} on $\h$ is a pair $W = (W^{in}, W^{out})$ of associated inward and outward spin decoration. The set of spin-decorated horospheres is denoted $\Hor^S$. \end{enumerate} \end{defn} Note the convention here is essentially opposite to that of \cite{Mathews_Spinors_horospheres}, as we order the vectors in frames differently here. Taking a basepoint in $\Fr$ to be the outward frame of $\Phi_1 \circ \Phi_1 (1,0)$ at $(0,0,0,1) \in \U$, and a lift in $\Fr^S$, the correspondence $\Fr \cong PSL_2\$$ identifies the parabolic translation subgroup $\underline{\P_{\h_0}}$ of \refsec{horospheres_geometry} and \refeqn{Ph0} with the frames of the outward frame field of $\Phi_2 \circ \Phi_1 (1,0)$, and $\P_{\h_0}$ with the spin frames of an outward spin decoration lifting $\Phi_2 \circ \Phi_1 (1,0)$. We thus have identifications \[ \frac{PSL_2\$}{\underline{\P_{\h_0}}} \cong \Hor^D, \quad \frac{SL_2\$}{\P_{\h_0}} \cong \Hor^S. \] Since $\Hor^D \cong S^3 \times SO(3) \times \R$, we have $\pi_1 (\Hor^D) \cong \Z/2$, and a nontrivial loop is given by rotating a decoration through $2\pi$ in a fixed horosphere. In the double cover, a rotation through $4\pi$ is required to form a loop. Indeed, $\Hor^S$ is the double, universal cover of $\Hor^D$. We now lift the maps $\Phi_1, \Phi_2$ to maps $\widetilde{\Phi_1}, \widetilde{\Phi_s}$ to obtain a diagram \[ \begin{array}{ccccc} S\HH & \stackrel{\widetilde{\Phi_1}}{\To} & \MF^S & \stackrel{\widetilde{\Phi_2}}{\To} & \Hor^S \\ & \stackrel{\Phi_1}{\searrow} & \downarrow & & \downarrow \\ & & \MF & \stackrel{\Phi_2}{\To} & \Hor^D \end{array} \] as follows. We have diffeomorphisms $S\HH \cong S^3 \times S^3 \times \R$ and $\MF \cong \Hor^D \cong S^3 \times SO(3) \times \R$. Moreover $\Phi_1$ is a double universal cover and $\Phi_2$ a diffeomorphism. We have $\pi_1(\MF) \cong \Z/2$, with a nontrivial loop given by fixing a flagpole and rotating the two flags of a multiflag simultaneously about a common orthogonal direction through $2\pi$. We denote the double (universal) cover $\MF^S$, and its elements \emph{spin multiflags}. Choosing basepoints for the lifts arbitrarily, we obtain lifts $\widetilde{\Phi_1}, \widetilde{\Phi_2}$ making the above diagram commute, and remaining $SL_2\$$-equivariant. \begin{proof}[Proof of \refthm{main_thm_1}] We have explicitly defined $\widetilde{\Phi_1}, \widetilde{\Phi_2}$ and shown they are $SL_2\$$-equivariant diffeomorphisms of $S^3 \times S^3 \times \R$. They provide the desired bijections. \end{proof} \subsection{Quaternionic distances and lambda lengths} \label{Sec:quaternionic_lambda} Before defining lambda lengths, we define a notion of quaternionic distance between spin frames, generalising \cite{Mathews_Spinors_horospheres}. Let $p$ be a point on an oriented geodesic $\gamma$ in $\hyp^4$. A frame $F = (f_1, f_2, f_3, f_4)$ at $p$ is \emph{adapted} to $\gamma$ if $f_4$ is positively tangent to $\gamma$. A spin frame $\widetilde{F}$ at $p$ is adapted to $\gamma$ if it lifts a frame adapted to $\gamma$. Now consider two points $p_1, p_2$ on $\gamma$, with frames $F^n = (f_1^n, f_2^n, f_3^n, f_4^n)$ adapted to $\gamma$ at each $p_n$. Parallel translation along $\gamma$ from $p_1$ to $p_2$ takes $F^1$ to a frame ${F'}^1 = ({f'}_1^1, {f'}_2^1, {f'}_3^1, {f'}_4^1)$ at $p_2$ adapted to $\gamma$ and agreeing with $F^2$ in its final vector, ${f'}_4^1 = f_4^2$. This translation has signed distance $\rho$. A rotation $R$ in the 3-plane $\Pi \subset T_{p_2} \hyp^4$ orthogonal to $\gamma$ (or ${f'}_4^1 = f_4^2$) at $p_2$ then moves $F'^1$ to $F^2$. The 3-plane $\Pi$ has an orientation induced by the normal $\gamma$ and the ambient orientation on $\hyp^4$ of \refsec{orientations_frames_spin}, using the conventions of \refsec{orientations}. This rotation $R$ is around some axis in $\Pi$, by some angle $\theta$, measured in the usual right-handed way, using the orientation on $\Pi$. Both $({f'}_1^1, {f'}_2^1, {f'}_3^1)$ or $(f_1^2, f_2^2, f_3^2)$ are oriented bases of $\Pi$. We can use such either basis to identify $\Pi$ with $\$\R^3$, identifying the three basis elements with $(i,j,1)$ respectively. In this way, the four elements of the original frame are treated like the ``$1$, $i$, $j$, normal=$k$" elements of a frame field, as in \refdef{inward_outward_frame_field}, and the identifications of $\Pi$ with $\$\R^3$ agree with the paravector identifications of \refdef{paravector_identification}. Under such an identification, $R$ is given by a rotation of $\theta \in \R / 2 \pi Z$ about an oriented axis in the direction of a unit vector $v \in \$\R^3$. Although we have two possible choices for this identification $\Pi \cong \$\R^3$, in fact we obtain the same description of $R$. In other words, the vector $v$ has the same expression as a paravector, whether we use the basis from ${F'}^1$ or $F^2$ for the paravector identification. This follows from the following elementary linear algebra fact. \begin{lem} \label{Lem:rotation_coordinates_invariant} Suppose $B_1, B_2$ are two oriented bases of an oriented real inner product space $\V$, and $R$ is the linear map taking $B_1$ to $B_2$. Let $R_i$ be the matrix of $R$ with respect to the basis $B_i$. Then $R_1 = R_2$. Moreover, if $\V$ is 3-dimensional and $R$ is a rotation with axis $\ell$, then any point $p \in \ell$ has the same coordinates with respect to $B_1$ and $B_2$. \end{lem} \begin{proof} Let $C$ be the change of basis matrix from $B_1$ to $B_2$, so the $i$th column of $C$ expresses the $i$th vector of $B_2$ in terms of the basis $B_1$. Then for any endomorphism $L$ of $\V$, the matrices $L_1, L_2$ of $L$ with respect to $B_1, B_2$ are related by $L_2 = C^{-1} L_1 C$. For the endomorphism $R$, we have $R_1 = C$. Thus $R_2 = R_1^{-1} R_1 R_1 = R_1$. Now if $R$ is a rotation with axis $\ell$ and $p \in \ell$, let $p_i$ be the vector of coordinates of $p$ with respect to $B_i$. Then we have $p_1 = C p_2$. But as $p$ lies on the axis of $R$ we have $R p = p$, so $R_1 p_1 = p_1$ and $R_2 p_2 = p_2$. Thus $p_1 = C p_2 = R_1 p_2 = R_2 p_2 = p_2$. \end{proof} Thus the unit $v \in \$\R^3$ describing the oriented axis of $R$ is the same for the two choices of identification $\Pi \cong \$\R^3$. (Note that $v,\theta$ and $-v,-\theta$ describe the same rotation.) The above equally applies to spin frames, the only difference then is $\theta \in \R / 4 \pi \Z$. In the above we considered translating from $p_1$ to $p_2$, then rotating via $R$ in $\Pi \subset T_{p_2} \hyp^4$. But instead we could have rotated at $p_1$, then translated to $p_2$. We show that even in this case we obtain the same $v \in \$\R^3$ and $\theta$. Precisely, let ${F'}^2 = ({f'}_1^2, {f'}_2^2, {f'}_3^2, {f'}_4^2)$ be the frame at $p_1$ obtained by parallel translation of $F^2$ from $p_2$ to $p_1$ along $\gamma$. Then ${F'}^2$ agrees with $F^1$ in its final vector, ${f'}_4^2 = f_4^1$. A rotation $R_1$ in the 3-plane $\Pi_1 \subset T_{p_1} \hyp^4$ orthogonal to $\gamma$ (or $f_4^1 = {f'}_4^2$) at $p_1$ takes $F^1$ to ${F'}^2$, and then translation $T_\rho$ along $\gamma$ by signed distance $\rho$ from $p_1$ to $p_2$ takes ${F'}^2$ to $F^2$. The isometries $T_\rho, R_1, R$ satisfy $T_\rho \circ R_1 = R \circ T_\rho$, so $R_1, R$ are conjugate, hence have the same angle $\theta$. Moreover, $T_\rho$ takes the oriented axis of $R_1$ to the oriented axis of $R$. Let $v_1$ be the unit vector directing the axis of $R_1$. Using $F^1$ or ${F'}^2$, we have an identification $\Pi_1 \cong \$\R^3$ and regard $v_1 \in \$\R^3$. By the above lemma, whichever identification we choose, we obtain the same $v_1 \in \$\R^3$. As $T_\rho$ sends $F^1$ to ${F'}^1$, and ${F'}^2$ to $F^2$, the expressions of $v_1$ and $v$ in $\$\R^3$ are the same in these bases, and so $v_1 = v \in \$\R^3$. \begin{defn} Let $F^1, F^2$ be frames, or spin frames, adapted to a common oriented geodesic. The \emph{quaternionic distance} from $F^1$ to $F^2$ is $\rho + \theta v k$. \end{defn} Note that as $v \in \$\R^3$ we have $vk \in \II$. Also, $v,k$ and $-v,-k$ yield the same result for the complex distance. However, the quaternionic distance between frames is only well defined modulo $2\pi vk$; between spin frames, it is well defined modulo $4\pi vk$. Since $e^{\pi v k} = -1$ and $e^{2 \pi vk} = 1$, we can obtain well-defined quaternions by taking exponentials, and this is how we define lambda lengths, as in \refeqn{lambda_length}. Now consider horospheres $\h_1, \h_2$ with centres $z_1, z_2 \in \$\R^3 \cup \{\infty\}$. Let $\gamma_{12}$ the oriented geodesic from $z_1$ to $z_2$, and $p_i = \gamma_{12} \cap \h_i$. Decorations $F_i$ on $\h_i$ yield frames $F^{in}_i (p_i), F^{out}_i (p_i)$, and spin decorations $W_i$ yield associated spin frames $W^{in}_i (p_i), W^{out}_i (p_i)$. The frames $F^{in}_1 (p_1), F^{out}_2 (p_2)$ and spin frames $W^{in}_1 (p_1), W^{out}_2 (p_2)$ are adapted to $\gamma_{12}$. \begin{defn} Let $\h_1, \h_2$ be horospheres. \begin{enumerate} \item If $F_i$ is a decoration on $\h_i$, then the \emph{lambda length} from $(\h_1, F_1)$ to $(\h_2, F_2)$ is \[ \lambda_{12} = \exp \left( \frac{d}{2} \right) \] where $d = \rho + \theta v k$ is the quaternionic distance from $F^{in}_1 (p_1)$ to $F^{out}_2 (p_2)$. \item If $W_i$ is a spin decoration on $\h_i$, then the \emph{lambda length} from $(\h_1, W_1)$ to $(\h_2, W_2)$ is \[ \lambda_{12} = \exp \left( \frac{d}{2} \right) \] where $d = \rho + \theta v k$ is the quaternionic distance from $W^{in}_1 (p_1)$ to $W^{out}_2 (p_2)$. \end{enumerate} When $\h_1, \h_2$ have common centre, we set $\lambda_{12} = 0$. \end{defn} With decorations, $d$ is well defined modulo $2\pi vk$, so $\lambda_{12}$ is only well defined up to sign. With spin decorations, $d$ is well defined modulo $4 \pi vk$, so $\lambda_{12}$ is a well defined quaternion, and lambda lengths provide a function $\Hor^S \times \Hor^S \To \HH$, which as in the 3-dimensional case is continuous. By definition, quaternionic distance between spin frames, and lambda length between spin-decorated horospheres, are invariant under the action of spin isometries. Lambda lengths can be understood to arise naturally: for the representation $\sigma$ of \refdef{rho}, $\sigma(\lambda)$ essentially achieves a lambda length of $\lambda$, as follows. Consider the spin isometry $\phi$ of $\U$ which relates frames related by quaternionic distance $d = \rho + \theta v k$, along the geodesic $\gamma$ from $0$ to $\infty$. The 3-planes orthogonal to $\gamma$ are parallel to the $\$\R^3$ at infinity in $\U$ and we identify them with $\$\R^3$ accordingly. Then $\phi$ rotates the $\$\R^3$ at infinity by angle $\theta$ along an oriented axis directed by the unit paravector $v$, then translates by $\rho$ along the geodesic from $0$ to $\infty$. The isometry $\phi$ is given by the M\"{o}bius transformation of $\partial \U = \$\R^3 \cup \{\infty\}$ which fixes $\infty$, hence is given by a linear map of $\$\R^3$. This linear map is the composition of dilation by $e^\rho$, and rotation by $\theta$ about $v$. By \refprop{rho_rotation_dilation}, this linear map is $\sigma(\lambda)$ where $\lambda = e^{\rho/2} e^{\theta vk} = \exp(d/2)$. In other words, the M\"{o}bius transformation which achieves lambda length $\lambda$ is given by $\sigma(\lambda)$. \subsection{Antisymmetry of lambda lengths} \label{Sec:antisym} In this section we prove the following proposition, generalising the 3-dimensional case where $\lambda$ is antisymmetric, $\lambda_{12} = - \lambda_{21}$. Let $\h_\bullet, z_\bullet, \gamma_{\bullet \bullet}, p_\bullet$ be as above in \refsec{quaternionic_lambda}. \begin{prop} \label{Prop:lambda_length_antisymmetric} Let $(\h_1, W_1), (\h_2, W_2)$ be spin-decorated horospheres, and let $\lambda_{ij}$ be the lambda length from $(\h_i, W_i)$ to $(\h_j, W_j)$. Then $\lambda_{12} = - \lambda_{21}^*$. \end{prop} \begin{proof} If $h_1, \h_2$ have common centre then the result is clear, so assume not. Let $d_{ij}$ be the quaternionic distance from $W_i^{in}$ to $W_j^{out}$ from $p_i$ to $p_j$ along $\gamma_{ij}$. Let $Y_1^{out}$ be the spin frame at $p_1$ obtained by a rotation of $2\pi$ from $W_1^{out}$, so $Y_1^{out}, W_1^{out}$ are the two spin lifts of $F_1^{out}$. As in the 3-dimensional case, the spin isometry $\phi$ which takes $W_1^{in} (p_1)$ to $W_2^{out} (p_2)$ also takes $Y_1^{out} (p_1)$ to $W_2^{in} (p_2)$. This spin isometry is the composition of a translation by signed distance $\rho$ along $\gamma_{12}$, followed by a rotation $R$ by angle $\theta \in \R / 4 \pi \Z$ about some unit vector $v \in T_{p_2} \h_2$. Using inward and outward paravector identifications from the decoration of $W_2$ (i.e. the frames $W_2^{in}(p_2)$ and $W_2^{out}(p_2)$), $v$ is identified with unit paravectors $v_{in}, v_{out} \in \$\R^3$. By \reflem{inward_outward_identification}, $v_{in} = - \overline{v_{out}}$. Since the spin isometry is from $W_1^{in}(p_1)$ to $W_2^{out}(p_2)$ and the rotation $R$ is at $p_2$, we use the outward paravector identification of $v$ in the quaternionic distance. Thus the quaternionic distance $d_{12}$ from $W_1^{in}(p_1)$ to $W_2^{out}(p_2)$ is given by $\rho + \theta v_{out} k$. The inverse $\phi^{-1}$ of this spin isometry takes $W_2^{in} (p_2)$ to $Y_1^{out} (p_1)$, and is the composition of the rotation $R^{-1}$ about $v$ in $T_{p_2} \h_2$, followed by a translation by signed distance $\rho$ on the oppositely oriented geodesic $\gamma_{21}$. As the basis $W_2^{in}(p_2)$ has opposite orientation to the basis $W_2^{out}(p_2)$, the rotation $R^{-1}$ has the same angle $\theta \in 4 \pi \Z$ about $v$ with respect to $W_2^{in}(p_2)$ as $R$ does about $v$ with respect to $W_2^{out}(p_2)$. Adding a $2\pi$ to the rotation gives the spin isometry from $W_2^{in} (p_2)$ to $W_1^{out} (p_1)$. As the rotation is performed at $p_2$, we use the inward paravector identification of $v$ in the quaternionic distance. Thus $d_{21} = \rho + (\theta + 2\pi) v_{in} k$. Note that here we have considered the rotation before the translation, but by the discussion of \refsec{quaternionic_lambda} this still yields the correct quaternionic distance. Hence the quaternionic distances $d_{12}, d_{21}$ have the same real part $\rho$, and their imaginary parts are $\theta v_{out} k$ and $(\theta + 2\pi) v_{in} k = -(\theta + 2\pi) \overline{v_{out}} k$ respectively. Thus \[ \lambda_{12} = \exp \left( \frac{\rho + \theta v_{out} k}{2} \right), \quad \lambda_{21} = \exp \left( \frac{\rho - (\theta + 2\pi) \overline{v_{out}} k}{2} \right) = -\exp \left( \frac{\rho -\theta \overline{v_{out}}k}{2} \right) \] Now we note that for $v \in \$\R^3$ we have $-\overline{v} k = (vk)^*$. For $(vk)^* = k^* v^* = -kv$ and from \reflem{quaternion_geometry_facts} we have $-kv = -\overline{v}k$. Thus \begin{align*} \lambda_{21} = -\exp \left( \frac{\rho + \theta (v_{out} k)^*}{2} \right) = - \exp \left( \left( \frac{\rho + \theta v_{out} k}{2} \right)^* \right) = - \left( \exp \left( \frac{\rho + \theta v_{out} k}{2} \right) \right)^* = - \lambda_{12}^*. \end{align*} \end{proof} \subsection{Pseudo-determinant and lambda length} \label{Sec:lambda_lengths} Let $\kappa_1, \kappa_2$ be spinors, $\kappa_1 = (\xi_1, \eta_1)$, $\kappa_2 = (\xi_2, \eta_2)$. Let $(\h_1, W_1), (\h_2, W_2)$ be the corresponding spin-decorated horospheres, i.e. $\Phi_2 \circ \Phi_1 (\kappa_m) = (\h_m, W_m)$ for $m=1,2$. Let $\lambda_{12}$ be the lambda length from $(\h_1, W_1)$ to $(\h_2, W_2)$. We now prove \refthm{main_thm_2}, that $\lambda_{12} = \{\kappa_1, \kappa_2\}$. \begin{proof}[Proof of \refthm{main_thm_2}] First, note $\{\kappa_1, \kappa_2 \} = 0$ precisely when $\kappa_1 = \kappa_2 x$ for some $x \in \HH$ (\reflem{nondegeneracy_of_spinor_form}), which occurs precisely when the corresponding horospheres $\h_1, \h_2$ have the same centre, which occurs precisely when $\lambda_{12} =0$. So we may assume this is not the case. Second, following \cite{Mathews_Spinors_horospheres} we prove the result when $\kappa_1 = (1,0)$ and $\kappa_2 = (0,1)$, showing $\lambda_{12} = 1$. Then $\h_1$ is centred at $\infty$ with height $1$, $\h_2$ is centred at $0$ with Euclidean diameter $1$, so $\h_1, \h_2$ are tangent at $p = (0,0,0,1) \in \U$, where $W_1, W_2$ both have $i$-direction specified by $i$, and $j$-direction specified by $j$. Hence $W_1^{in}, W_2^{out}$ have coincident frames. Recall that elements of $SL_2\$ = \Isom^s \hyp^4$ can be regarded as homotopy classes of paths in $PSL_2\$ \cong \Isom^+ \hyp^4$ starting at the identity. Consider \[ A = \begin{pmatrix} 0 & -1 \\ 1 & 0 \end{pmatrix} \in SL_2\$ \quad \text{represented by} \quad M_t = \pm \begin{pmatrix} \cos t & - \sin t \\ \sin t & \cos t \end{pmatrix} \in PSL_2\$, \quad t \in [0, \pi/2]. \] Note all $M_t \in PSL_2\R \subset PSL_2\$$. We observe $A.\kappa_1 = \kappa_2$, so by equivariance of $\Phi_1$ and $\Phi_2$ we have $A.(\h_1, W_1) = (\h_2, W_2)$. Now $M_t$ is the isometry of $\hyp^4$ which rotates the $wz$-plane by $2t$ about $p$, from the $z$-direction $\partial_z$ towards the $w$-direction $\partial_w$, fixing the 2-plane in the $xy$-directions at $p$. The frame $W_1^{in}$ at $p$ projects to $(v_1^{1,in}, v_1^i, v_1^j, N_1^{in}) = (-\partial_w, \partial_x, \partial_y, -\partial_z)$, which rotates via $M_t$ from $N_1^{in}$ towards $v_1^{1,in}$ through $\pi$ to arrive at $W_2^{in}$. This spin frame $W_2^{in}$ projects to $(v_2^{1,in}, v_2^i, v_2^j, N_2^{in}) = (\partial_w, \partial_x, \partial_y, \partial_z)$. Applying \refdef{associated_spin_decorations}, the associated outward spin frame $W_2^{out}$ is then obtained by rotating $W_2^{in}$ by $\pi$ from $v_2^{1,in}$ towards $N_2^{in}$ . These two rotations are inverses, and so $W_1^{in} = W_2^{out}$, the quaternionic distance from $W_1^{in}$ to $W_2^{out}$ is $0$, and $\lambda_{12} = 1$. Third, we prove the result when $\kappa_1 = (1,0)$ and $\kappa_2 = (0,D)$ where $D \in \HH^\times$, showing that $\lambda_{12} = D$. The oriented common perpendicular $\gamma_{12}$ runs from $\infty$ to $0$, intersecting $\h_1$ at $p_1 = (0,0,0,1)$ and $\h_2$ at $p_2 = (0,0,0,|D|^{-2})$. The translation distance along $\gamma_{12}$ from $p_1$ to $p_2$ is $\rho = 2 \log |D|$. Both $T_{p_1} \h_1$ and $T_{p_2} \h_2$ are the $wxy$ 3-plane, so we can identify them with $\$\R^3$ using the $\U$-identification of \refdef{U-identification}, $(1,i,j) \mapsto (\partial_w, \partial_x, \partial_y)$). The inward paravector identification of $W_1^{in}$ at $p_1$ is $(1,i,j) \mapsto (-\partial_w, \partial_x, \partial_y)$, as calculated in the previous case. Thus a tangent vector given by $v \in \$\R^3$ in the $\U$-identification is given by $-\overline{v}$ in the inward paravector identification of $W_1^{in}$. We now consider the rotation of frames from $W_1^{in}$ to $W_2^{out}$. Let $D= |D| e^{u \theta}$ where $u$ is unit imaginary and $\theta \in [0, \pi]$. Consider \[ A = \begin{pmatrix} D^{-1*} & 0 \\ 0 & D \end{pmatrix} \in SL_2\$, \quad \text{represented by} \quad M_t = \pm \begin{pmatrix} |D|^{-t} e^{-tu^* \theta} & 0 \\ 0 & |D|^t e^{tu\theta} \end{pmatrix} \] over $t \in [0,1]$. All $M_t$ are of the form \reflem{elementary_vahlen_properties}(ii) up to sign, hence lie in $PSL_2\$$. Now $A.(0,1) = (0,D)$, so by equivariance, $A$ sends $(\h_2, W_2)$ from the previous case to $(\h_2, W_2)$ here, and is realised by the family of isometries given by $M_t$. Each $M_t$ yields the M\"{o}bius transformation \begin{align*} z \mapsto \left( |D|^{-t} e^{-tu^* \theta} \right) z \left( |D|^{t} e^{tu} \right)^{-1} = \sigma \left( |D|^{-t} e^{-tu^* \theta} \right) (z) \end{align*} which is the isometry of $\U$ rotating $\$\R^3$ by $\sigma(e^{-tu^* \theta})$ and translating by $2t \log |D|$ along $\gamma_{12}$. By \refprop{rho_rotation}, this rotation is of angle $2t\theta$ about $-(-u^*) k = u^* k \in \$\R^3$. But this is identifying $\$\R^3$ with $T_p \h_1$ using the $\U$-identification; the inward paravector identification has the opposite orientation, with rotation angle $-2t\theta$ and axis $-(\overline{u^* k}) = k u'$. By \reflem{quaternion_geometry_facts} we have $ku' = -(u')^* k = - \overline{u} \; k = uk$, the final equality since $u \in \II$. Calculating the quaternionic distance $d_{12}$ from $W_1^{in}$ to $W_2^{out}$ via rotation at $p_1$ then translation, we obtain $d_{12} = \rho + (-2\theta)(uk)k = 2 \log |D| + 2 \theta u$. Hence $\lambda_{12} = \exp(|D| + \theta u) = |D|e^{\theta u} = D$. Finally, for general $\kappa_1, \kappa_2 \in S\HH$ there exists $A \in SL_2\$$ such that $A.\kappa_1 = (1,0)$ and $A.\kappa_2 = (0,D)$, where $0 \neq D = \{\kappa_1, \kappa_2\} = \xi_1^* \eta_2 - \eta_1^* \xi_2$. To see this, consider the matrix $B$ with columns $\kappa_1$ and $\kappa_2 D^{-1}$. As the columns of $B$ are spinors (using \reflem{spinor_right_multiplication}) and \begin{align*} \pdet B &= \pdet \begin{pmatrix} \xi_1 & \xi_2 D^{-1} \\ \eta_1 & \eta_2 D^{-1} \end{pmatrix} = \xi_1^* \eta_2 D^{-1} - \eta_1^* \xi_2 D^{-1} = DD^{-1} = 1, \end{align*} $B \in SL_2\$$. This $B$ satisfies $B.(1,0) = \kappa_1$ and $B.(0,D) = \kappa_2$. Thus $A = B^{-1}$ sends $\kappa_1 \mapsto (1,0)$ and $\kappa_2 \mapsto (0,D)$ as required. This $A$ acts on $\hyp^4$ as a spin isometry, so the lambda length from the spin-decorated horosphere of $\kappa_1$ to that of $\kappa_2$, is equal to the lambda length from the spin-decorated horosphere of $(1,0)$ to $(0,D)$, which from the previous case is $D$ as desired. \end{proof} \section{Ptolemy equation} \label{Sec:Ptolemy} We now apply Gel'fand--Retakh's theory of noncommutative determinants to prove \refthm{main_thm_3}. \subsection{Quasideterminants and quasi-Pl\"{u}cker coordinates} \label{Sec:quasidet_Plucker} In \cite[sec. II]{Gelfand_Retakh_97}, Gel'fand and Retakh consider various generalised notions of determinants Pl\"{u}cker coordinates for matrices over noncommutative rings. We consider only those notions necessary to define their \emph{left quasi-Pl\"{u}cker coordinates} in the case of $2 \times 4$ quaternionic matrices. For a square matrix $A$ of arbitrary size $N \times N$ over an arbitrary ring with unit, Gel'fand--Retakh \cite[Defn. 1.1.4, p. 520]{Gelfand_Retakh_97} define a family of \emph{quasideterminants} determinants $|A|_{p,q}$, indexed by integers $p,q$ where $1 \leq p,q \leq N$ . In the case of a $2 \times 2$ matrix, they are given by \cite[Example 1, p. 520]{Gelfand_Retakh_97} \[ \begin{array}{ll} |A|_{11} = a_{11} - a_{12} a_{22}^{-1} a_{21} & |A|_{12} = a_{12} - a_{11} a_{21}^{-1} a_{22} \\ |A|_{21} = a_{21} - a_{22} a_{12}^{-1} a_{11} & |A|_{22} = a_{22} - a_{21} a_{11}^{-1} a_{12} \end{array} \] For a general $M \times N$ matrix $A$ with $M < N$ over an arbitrary skew field, they define a family of \emph{left quasi-Pl\"{u}cker coordinates} $p_{lm}^I (A)$, indexed by integers $l,m$ such that $1 \leq l,m \leq N$, and sets $I = \{ l_1, \ldots, l_{M-1} \}$ of $M-1$ distinct integers satisfying $1 \leq l_1, \ldots, l_{M-1} \leq N$, none of which is equal to $l$ \cite[sec. II.1.1, p. 526]{Gelfand_Retakh_97}. In the case of $2 \times 4$ matrices over $\HH$, we thus have left quasi-Pl\"{u}cker coordinates $p_{lm}^{I}$ where $1 \leq l,m \leq 4$ and $I = \{l_1\}$ where $1 \leq l_1 \leq 4$ and $l_1 \neq l$. We write $n$ for $l_1$ and $p_{lm}^n (A)$ rather than $p_{ij}^{I} (A)$. Writing $a_{i,j}$ for the $(i,j)$ entry of $A$, in this case $p_{lm}^n (A)$ is defined as \begin{equation} \label{Eqn:left_quasi-plucker_2x4} p_{lm}^n (A) = \begin{vmatrix} a_{1,l} & a_{1,n} \\ a_{2,l} & a_{2,n} \end{vmatrix}^{-1}_{s,l} \begin{vmatrix} a_{1,m} & a_{1,n} \\ a_{2,m} & a_{2,n} \end{vmatrix}_{s,m}, \end{equation} where $s = 1$ or $2$; it turns out the result is the same for either choice of $s$. Let us calculate these quantities for the matrices we shall need. \begin{lem} \label{Lem:quasideterminant_for_spinors} Let $A$ be a quaternionic matrix whose columns lie in $S\HH$, \[ A = \begin{pmatrix} a & b \\ c & d \end{pmatrix}. \] Then \[ \begin{pmatrix} |A|_{11} & |A|_{12} \\ |A|_{21} & |A|_{22} \end{pmatrix} = \begin{pmatrix} d^{-1*} & -c^{-1*} \\ -b^{-1*} & a^{-1*} \end{pmatrix} \begin{pmatrix} (\pdet A)^* & 0 \\ 0 & (\pdet A) \end{pmatrix}. \] \end{lem} \begin{proof} As the columns of $A$ lie $S\HH$, we have $d^* b, c^* a \in \$\R^3$, hence $d^* b = b^* d$ and $c^* a = a^* c$. Then we compute \[ d^* |A|_{11} = d^* \left( a - bd^{-1} c \right) = d^* a - b^* d d^{-1} c = (\pdet A)^*, \] so $|A|_{11} = d^{*-1} \det A$. Similarly we compute \begin{align*} -c^* |A|_{12} &= -c^* \left( b - ac^{-1} d \right) = -c^* b + a^* d = \pdet A \\ -b^* |A|_{21} &= -b^* \left( c - db^{-1} a \right) = -b^* c + d^* b b^{-1} a = (\pdet A)^* \\ a^* |A|_{22} &= a^* \left( d - ca^{-1} b \right) = a^* d - c^* a a^{-1} b = \pdet A \end{align*} giving the desired result. \end{proof} As in \refsec{bracket}, for spinors $\kappa_1, \ldots, \kappa_n$, we denote by $(\kappa_1, \ldots, \kappa_n)$ the $2 \times n$ matrix whose $m$th column is $\kappa_m$. Let $\kappa_m = (\xi_m, \eta_m)$. Recalling that $\pdet (\kappa_1, \kappa_2) = \{ \kappa_1, \kappa_2 \}$ by definition, and letting $A = (\kappa_1, \kappa_2)$, then \reflem{quasideterminant_for_spinors} can be rewritten as \begin{equation} \label{Eqn:quasidets_for_spinors} \begin{array}{ll} |A|_{11} = \eta_2^{-1*} \{ \kappa_1, \kappa_2 \}^* & |A|_{12} = -\eta_1^{-1*} \{ \kappa_1, \kappa_2 \} \\ |A|_{21} = -\xi_2^{-1*} \{ \kappa_1, \kappa_2 \}^* & |A|_{22} = \xi_1^{-1*} \{ \kappa_1, \kappa_2 \}. \end{array} \end{equation} Moreover, \refeqn{left_quasi-plucker_2x4} can be rewritten as \begin{equation} \label{Eqn:Plucker_in_spinors} p_{lm}^n (\kappa_1, \kappa_2, \kappa_3, \kappa_4) = \left| (\kappa_l, \kappa_n) \right|_{sl}^{-1} \; \left| (\kappa_m, \kappa_n) \right|_{sm} \end{equation} \begin{lem} \label{Lem:quasi-Plucker_lambda} Let $A = (\kappa_1, \kappa_2, \kappa_3, \kappa_4)$ where each $\kappa_m \in S\HH$. Then for $1 \leq i,j,k \leq 4$ and $k \neq i$ \[ p_{lm}^n (A) = \left( \pdet (\kappa_n, \kappa_l) \right)^{-1} \, \pdet (\kappa_n, \kappa_m) = \left\{ \kappa_n, \kappa_l \right\}^{-1} \, \left\{ \kappa_n, \kappa_m \right\}. \] \end{lem} \begin{proof} Taking $s=1$ in \refeqn{Plucker_in_spinors} and applying \refeqn{quasidets_for_spinors}, we obtain \[ p_{lm}^{n} (A) = \begin{vmatrix} \xi_l & \xi_{n} \\ \eta_l & \eta_{n} \end{vmatrix}_{1l}^{-1} \begin{vmatrix} \xi_m & \xi_{n} \\ \eta_m & \eta_{n} \end{vmatrix}_{1m} = \left( \eta_{n}^{-1*} \{ \kappa_l, \kappa_{n} \}^* \right)^{-1} \eta_{n}^{-1*} \{\kappa_m, \kappa_{n} \}^* = \{\kappa_l, \kappa_{n}\}^{*-1} \{ \kappa_m, \kappa_{n} \}^*. \] Now the result follows from the antisymmetry property $\{ \kappa_1, \kappa_2 \} = - \{ \kappa_2, \kappa_1\}^*$. \end{proof} So we obtain \refeqn{quasi-Plucker_lambda}, i.e. $p_{lm}^n = \lambda_{n,l}^{-1} \lambda_{n,m}$, where $\lambda_{m,n}$ is the lambda length from the spin-decorated horosphere of $\kappa_m$ to that of $\kappa_n$. \subsection{Pl\"{u}cker relations} \label{Sec:Plucker} We can now prove \refprop{triangle_holonomy}, that $\lambda_{12} \lambda_{32}^{-1} \lambda_{31} \in \$\R^3 \cup \{\infty\}$, or equivalently, since $\$\R^3$ is closed under taking inverses, $\lambda_{31}^{-1} \lambda_{32} \lambda_{12}^{-1} \in \$\R^3 \cup \{\infty\}$. \begin{proof}[Proof of \refprop{triangle_holonomy}] Gelfand--Retakh \cite[Prop. 2.1.4 and Example 1, p. 527]{Gelfand_Retakh_97} show that for $2 \times n$ matrices, for any $l,m,n$, \[ p_{lm}^{n} p_{mn}^{l} p_{nl}^{m} = -1. \] In terms of lambda lengths we then have \[ \lambda_{nl}^{-1} \lambda_{nm} \lambda_{lm}^{-1} \lambda_{ln} \lambda_{mn}^{-1} \lambda_{ml} = -1 \] which is equivalent to \[ \lambda_{nl}^{-1} \lambda_{nm} \lambda_{lm}^{-1} = -\lambda_{ml}^{-1} \lambda_{mn} \lambda_{ln}^{-1}. \] Since $\lambda_{mn} = - \lambda_{nm}^*$, the right hand side is $\lambda_{lm}^{-1*} \lambda_{nm}^* \lambda_{nl}^{-1*} = \left( \lambda_{nl}^{-1} \lambda_{nm} \lambda_{lm}^{-1} \right)^*$. Thus we obtain \[ \lambda_{nl}^{-1} \lambda_{nm} \lambda_{lm}^{-1} = \left( \lambda_{nl}^{-1} \lambda_{nm} \lambda_{lm}^{-1} \right)^* \] and, being fixed under $*$, taking $(l,m,n) = (1,2,3)$, we conclude $\lambda_{31}^{-1} \lambda_{32} \lambda_{12}^{-1} \in \$\R^3$, unless it is $\infty$. \end{proof} Finally, we prove \refthm{main_thm_3}, the non-commutative Ptolemy equation \[ \lambda_{02}^{-1} \lambda_{01} \lambda_{31}^{-1} \lambda_{32} + \lambda_{02}^{-1} \lambda_{03} \lambda_{13}^{-1} \lambda_{12} = 1. \] \begin{proof}[Proof of \refthm{main_thm_3}] Gelfand--Retakh \cite[Prop. 2.1.4 and Example 2, p. 527]{Gelfand_Retakh_97} show that for $2 \times 4$ matrices, for any $a,b,l,m$, \[ p_{ab}^{l} p_{ba}^{m} + p_{am}^{l} p_{ma}^{b} = 1. \] Thus we obtain \[ \lambda_{la}^{-1} \lambda_{lb} \lambda_{mb}^{-1} \lambda_{ma} + \lambda_{la}^{-1} \lambda_{lm} \lambda_{bm}^{-1} \lambda_{ba} = 1. \] Taking $(a,b,l,m) = (2,1,0,3)$ then gives the desired result. \end{proof} \small \bibliography{spinref} \bibliographystyle{amsplain} \end{document}
2412.06635v1
http://arxiv.org/abs/2412.06635v1
Motivic cohomology of mixed characteristic schemes
\documentclass[10pt,twoside]{article} \usepackage[utf8]{inputenc} \title{Motivic cohomology of mixed characteristic schemes} \author{Tess Bouis} \date{} \setcounter{tocdepth}{2} \usepackage[left=3cm, right=3cm, top=1in, bottom=1in, headheight=2cm]{geometry} \usepackage{fancyhdr} \pagestyle{fancy} \usepackage{stmaryrd} \usepackage{mathdots} \usepackage{amsmath, amscd, amsfonts, amssymb, amsthm,todonotes,eurosym,enumitem,relsize} \usepackage[utf8]{inputenc} \usepackage[T1]{fontenc} \usepackage{mathrsfs} \usepackage[colorlinks]{hyperref} \usepackage[figure,table]{hypcap} \hypersetup{ bookmarksnumbered, pdfstartview={FitH}, citecolor={black}, linkcolor={black}, urlcolor={black}, pdfpagemode={UseOutlines} } \usepackage{xcolor} \usepackage{comment} \usepackage[object=pgfhan]{pgfornament} \usepackage{graphicx} \usepackage{array} \usepackage{csquotes} \usepackage{extarrows} \usepackage{needspace} \input{xy} \xyoption{all} \usepackage{tikz-cd} \usepackage{textcomp} \usepackage{sectsty} \sectionfont{\sc\centering} \subsectionfont{\bf\large} \subsubsectionfont{\bf\normalsize} \usepackage{fancyhdr} \renewcommand{\headrulewidth}{0pt} \rhead{} \lhead{} \chead{} \cfoot{} \newcommand{\BMS}{{}} \renewcommand{\labelenumi}{(\arabic{enumi})} \newcommand{\ns}{\vspace{3pt}} \newlength{\outermargin} \setlength{\outermargin}{2.5cm} \newlength{\mar} \setlength{\mar}{1cm} \newlength{\len} \newlength{\temp}\setlength{\temp}{\paperwidth} \addtolength{\len}{\paperwidth}\addtolength{\len}{-\outermargin}\addtolength{\len}{-\outermargin}\addtolength{\len}{-\mar}\addtolength{\len}{-\mar} \newcommand{\boxone}{4cm} \newcommand{\boxtwo}{13cm} \newcommand{\boxthree}{15cm} \newtheorem{theorem}{Theorem}[section] \newtheorem{lemma}[theorem]{Lemma} \newtheorem{corollary}[theorem]{Corollary} \newtheorem{proposition}[theorem]{Proposition} \newtheorem{theoremintro}{Theorem} \renewcommand{\thetheoremintro}{\Alph{theoremintro}} \newtheorem{corollaryintro}[theoremintro]{Corollary} \DeclareMathOperator{\bblacksquare}{\scalebox{0.6}{$\blacksquare$}} \theoremstyle{definition} \newtheorem{remark}[theorem]{Remark} \newtheorem{remarks}[theorem]{Remarks} \newtheorem{example}[theorem]{Example} \newtheorem{examples}[theorem]{Examples} \newtheorem{construction}[theorem]{Construction} \newtheorem{definition}[theorem]{Definition} \newtheorem{notation}[theorem]{Notation} \newtheorem{question}[theorem]{Question} \newtheorem{conjecture}[theorem]{Conjecture} \usepackage[bbgreekl]{mathbbol} \DeclareSymbolFontAlphabet{\mathbb}{AMSb} \DeclareSymbolFontAlphabet{\mathbbl}{bbold} \newcommand{\Prism}{{\mathlarger{\mathbbl{\Delta}}}} \DeclareMathOperator{\Z}{\mathbb{Z}} \DeclareMathOperator{\Q}{\mathbb{Q}} \DeclareMathOperator{\R}{\mathbb{R}} \DeclareMathOperator{\N}{\mathbb{N}} \DeclareMathOperator{\F}{\mathbb{F}} \DeclareMathOperator{\Fp}{\mathbb{F}_p} \DeclareMathOperator{\C}{\mathbb{C}} \DeclareMathOperator{\E}{\mathbb{E}} \renewcommand{\S}{\mathbb{S}} \renewcommand{\epsilon}{\varepsilon} \let\l\relax \newcommand{\l}{\ell} \DeclareFontFamily{U}{MnSymbolC}{} \DeclareFontShape{U}{MnSymbolC}{m}{n}{ <-5.5> MnSymbolC5 <5.5-6.5> MnSymbolC6 <6.5-7.5> MnSymbolC7 <7.5-8.5> MnSymbolC8 <8.5-9.5> MnSymbolC9 <9.5-11.5> MnSymbolC10 <11.5-> MnSymbolCb12 }{} \DeclareRobustCommand{\sqcdot}{ \mathbin{\text{\usefont{U}{MnSymbolC}{m}{n}\symbol{"69}}} } \begin{document} \pagestyle{fancy} \fancyhead[EC]{TESS BOUIS} \fancyhead[OC]{MOTIVIC COHOMOLOGY OF MIXED CHARACTERISTIC SCHEMES} \fancyfoot[C]{\thepage} \maketitle \begin{abstract} We introduce a theory of motivic cohomology for quasi-compact quasi-separated schemes, which generalises the construction of Elmanto--Morrow in the case of schemes over a field. Our construction is non-$\mathbb{A}^1$-invariant in general, but it uses the classical $\mathbb{A}^1$-invariant motivic cohomology of smooth $\Z$-schemes as an input. The main new input of our construction is a global filtration on topological cyclic homology, whose graded pieces provide an integral refinement of derived de Rham cohomology and Bhatt--Morrow--Scholze’s syntomic cohomology. Our theory satisfies various expected properties of motivic cohomology, including a relation to non-connective algebraic $K$-theory via an Atiyah--Hirzebruch spectral sequence, the projective bundle formula, and pro cdh descent. \end{abstract} \tableofcontents \section{Introduction} \vspace{-\parindent} \hspace{\parindent} Motivic cohomology is an analogue in algebraic geometry of singular cohomology. It was first envisioned to exist for schemes $X$ of finite type over $\Z$ by Beilinson and Lichtenbaum \cite{lichtenbaum_values_1973,lichtenbaum_values_1984,beilinson_notes_1986,beilinson_height_1987,beilinson_notes_1987}, as a way to better understand the special values of their $L$-functions. Motivic cohomology, in the form of complexes of abelian groups $\Z(i)^{\text{mot}}(X)$ indexed by integers $i \geq 0$, should be an integral interpolation between étale cohomology, and Adams eigenspaces on rationalised algebraic $K$-theory. That is, there should be a natural filtration $\text{Fil}^\star_{\text{mot}} \text{K}(X)$ on the non-connective algebraic $K$-theory $\text{K}(X)$, which splits rationally, and whose shifted graded pieces $$\Z(i)^{\text{mot}}(X) \simeq \text{gr}^i_{\text{mot}} \text{K}(X)[-2i]$$ are given mod $p$, when $p$ is invertible in $X$, and in degrees less than or equal to $i$, by the étale cohomology $R\Gamma_{\text{ét}}(X,\mu_p^{\otimes i})$: $$\tau^{\leq i} \F_p(i)^{\text{mot}}(X) \simeq \tau^{\leq i} R\Gamma_{\text{ét}}(X,\mu_p^{\otimes i}).$$ Such a theory was first developed in the smooth case at the initiative of Bloch and Voevodsky \cite{bloch_algebraic_1986,voevodsky_cycles_2000}, using algebraic cycles and $\mathbb{A}^1$-homotopy theory. In this generality, the use of $\mathbb{A}^1$\nobreakdash-invariant techniques is permitted by Quillen's fundamental theorem of algebraic $K$-theory \cite{quillen_higher_1973}, stating that algebraic $K$-theory is $\mathbb{A}^1$-invariant on regular schemes. On more general schemes, algebraic $K$-theory fails to be $\mathbb{A}^1$-invariant, so motivic cohomology itself needs to be non-$\mathbb{A}^1$-invariant in general. The first non-$\mathbb{A}^1$-invariant theory of motivic cohomology was recently introduced by Elmanto and Morrow \cite{elmanto_motivic_2023}, using recent advances in algebraic $K$-theory and topological cyclic homology. Their theory is developed in the generality of quasi-compact quasi-separated (qcqs) schemes over an arbitrary field, and recovers on smooth varieties the classical $\mathbb{A}^1$-invariant theory. In this article, we extend the work of Elmanto--Morrow to mixed characteristic, thus producing a theory of motivic cohomology in the originally expected generality of Beilinson and Lichtenbaum. Our theory relies on recent progress in integral $p$-adic Hodge theory \cite{bhatt_topological_2019,bhatt_prisms_2022,bhatt_absolute_2022}, and offers in return a complete description of mod $p$ motivic cohomology, even when $p$ is not invertible in the qcqs scheme~$X$. \needspace{5\baselineskip} \subsection{A non-\texorpdfstring{$\mathbb{A}^1$}{TEXT}-invariant theory of motivic cohomology} \vspace{-\parindent} \hspace{\parindent} The starting point of our construction is the following result, due to Kerz--Strunk--Tamme \cite{kerz_algebraic_2018} (who prove that homotopy $K$-theory is the cdh sheafification of algebraic $K$-theory) and Land--Tamme \cite{land_k-theory_2019} (who prove that the fibre $\text{K}^{\text{inf}}$ of the cyclotomic trace map satisfies cdh descent). \begin{theorem}[\cite{kerz_algebraic_2018,land_k-theory_2019}]\label{theoremKST+LT} Let $X$ be a qcqs scheme. Then the natural commutative diagram $$\begin{tikzcd} \emph{K}(X) \arrow{r} \arrow{d} & \emph{TC}(X) \ar[d] \\ \emph{KH}(X) \arrow[r] & \big(L_{\emph{cdh}} \emph{TC}\big)(X) \end{tikzcd}$$ is a cartesian square of spectra, where $\emph{KH}(X)$ is the homotopy $K$-theory of $X$, $\emph{TC}(X)$ is the topological cyclic homology of $X$, $L_{\emph{cdh}}$ is the cdh sheafification functor, the top horizontal map is the cyclotomic trace map, and the bottom horizontal map is the cdh sheafified cyclotomic trace map. \end{theorem} Theorem~\ref{theoremKST+LT} states that algebraic $K$-theory of schemes can be reconstructed purely in terms of homotopy $K$-theory ({\it i.e.}, information coming from $\mathbb{A}^1$-homotopy theory) and topological cyclic homology ({\it i.e.}, information coming from trace methods). The cdh topology is a Grothendieck topology introduced by Voevodsky \cite{suslin_bloch-kato_2000,voevodsky_unstable_2010}, as a way to apply topos theoretic techniques to the study of resolution of singularities. In particular, assuming resolution of singularities, any qcqs scheme would be locally regular in the cdh topology. While homotopy $K$-theory and topological cyclic homology were originally introduced as tools to approximate the existing algebraic $K$-theory, we construct the motivic cohomology of schemes using refinements of homotopy $K$-theory and topological cyclic homology. More precisely, our motivic filtration on algebraic $K$-theory is defined as a pullback of appropriate filtrations on homotopy $K$-theory, topological cyclic homology, and cdh sheafified topological cyclic homology. On homotopy $K$-theory, we use the recent work of Bachmann--Elmanto--Morrow \cite{bachmann_A^1-invariant_2024}, who construct a functorial multiplicative $\N$-indexed filtration $\text{Fil}^\star_{\text{cdh}} \text{KH}(X)$ on the homotopy $K$-theory of qcqs schemes $X$. The shifted graded pieces of this filtration, that we will denote by $\Z(i)^{\text{cdh}}(X)$, provide a good theory of {\it cdh-local motivic cohomology} for qcqs schemes. Their construction, which we review in Section~\ref{subsectioncdhlocalmotivicfiltration}, relies on the classical $\mathbb{A}^1$-invariant motivic cohomology of smooth $\Z$-schemes, and extends most of its properties to general qcqs schemes. Our first construction is that of a functorial multiplicative $\Z$-indexed filtration $\text{Fil}^\star_{\text{mot}} \text{TC}(X)$ for qcqs schemes $X$. This filtration recovers the HKR filtration on $\text{HC}^{-}(X/\Q)$ in characteristic zero \cite{antieau_periodic_2019,moulinos_universal_2022,raksit_hochschild_2020}, and the motivic filtration on $\text{TC}(X;\Z_p)$ after $p$-completion \cite{bhatt_topological_2019,morin_topological_2021,bhatt_absolute_2022,hahn_motivic_2022}. We describe the shifted graded pieces $$\Z(i)^{\text{TC}}(X) \simeq \text{gr}^i_{\text{mot}} \text{TC}(X)[-2i]$$ of this filtration in the following definition. \needspace{5\baselineskip} \begin{definition}[See Definition~\ref{definitionmotivicfiltrationonTC}]\label{definitionintroZ(i)^TC} For every qcqs scheme $X$ and every integer $i \in \Z$, the complex $\Z(i)^{\text{TC}}(X) \in \mathcal{D}(\Z)$ is defined by a natural cartesian square $$\begin{tikzcd} \Z(i)^{\text{TC}}(X) \ar[r] \ar[d] & R\Gamma_{\text{Zar}}\big(X,\widehat{\mathbb{L}\Omega}^{\geq i}_{-/\Z}\big) \ar[d] \\ \prod_{p \in \mathbb{P}} \Z_p(i)^{\text{BMS}}(X) \ar[r] & \prod_{p \in \mathbb{P}} R\Gamma_{\text{Zar}}\big(X,(\widehat{\mathbb{L}\Omega}^{\geq i}_{-/\Z})^\wedge_p\big), \end{tikzcd}$$ where $\Z_p(i)^{\text{BMS}}(X)$ is the syntomic cohomology of the $p$-adic formal scheme associated to $X$. \end{definition} It is straightforward from this definition that the presheaf $\Z(i)^{\text{TC}}$ is naturally identified with Bhatt--Morrow--Scholze's syntomic complex $\Z_p(i)^{\text{BMS}}$ in characteristic $p$, and with the Hodge-completed derived de Rham complex $R\Gamma_{\text{Zar}}(-,\widehat{\mathbb{L}\Omega}^{\geq i}_{-/\Q})$ in characteristic zero. Following \cite{elmanto_motivic_2023}, the motivic complex $\Z(i)^{\text{mot}}$ is defined in characteristic $p$ and zero respectively by cartesian squares $$\begin{tikzcd} \Z(i)^{\text{mot}}(X) \ar[r] \ar[d] & \Z_p(i)^{\text{BMS}}(X) \ar[d] & \Z(i)^{\text{mot}}(X) \ar[r] \ar[d] & R\Gamma_{\text{Zar}}\big(X,\widehat{\mathbb{L}\Omega}^{\geq i}_{-/\Q}\big) \ar[d] \\ \Z(i)^{\text{cdh}}(X) \ar[r] & \big(L_{\text{cdh}} \Z_p(i)^{\text{BMS}}\big)(X) & \Z(i)^{\text{cdh}}(X) \ar[r] & R\Gamma_{\text{cdh}}\big(X,\widehat{\mathbb{L}\Omega}^{\geq i}_{-/\Q}\big). \end{tikzcd}$$ The following definition is then a natural mixed characteristic generalisation of Elmanto--Morrow's definition over a field. \begin{definition}[Motivic cohomology; see Section~\ref{subsectiondefinitionmotiviccohomology}]\label{definitionintromotiviccohomology} For every qcqs scheme $X$ and every integer $i \in \Z$, the {\it weight-$i$ motivic complex} $$\Z(i)^{\text{mot}}(X) \in \mathcal{D}(\Z)$$ of $X$ is defined by a natural cartesian square $$\begin{tikzcd} \Z(i)^{\text{mot}}(X) \ar[r] \ar[d] & \Z(i)^{\text{TC}}(X) \ar[d] \\ \Z(i)^{\text{cdh}}(X) \ar[r] & \big(L_{\text{cdh}} \Z(i)^{\text{TC}}\big)(X), \end{tikzcd}$$ where the bottom horizontal map is induced by a filtered refinement of the cdh sheafified cyclotomic trace map. \end{definition} However, proving the expected relation between the motivic complexes $\Z(i)^{\text{mot}}$ and algebraic $K$\nobreakdash-theory (Theorem~\ref{theoremintroAHSSandAdams} below) requires more efforts in mixed characteristic than over a field. The main foundational obstacle is to prove that the presheaves $\Z(i)^{\text{mot}}$ vanish in weights~$i<0$, as we explain now. Note first that, by construction, the presheaves $\Z(i)^{\text{cdh}}$ do vanish on all qcqs schemes in weights $i<0$ (Section~\ref{subsectioncdhlocalmotivicfiltration}). In characteristic $p$, the presheaves $\Z_p(i)^{\text{BMS}}$ vanish in weights $i<0$, thus so do the presheaves $\Z(i)^{\text{mot}}$, and the zeroth step of the associated motivic filtration $\text{Fil}^\star_{\text{mot}} \text{K}$ recovers algebraic $K$-theory. Ignoring for a moment the completeness issues for this motivic filtration, this means that the presheaves $\Z(i)^{\text{mot}}$ provide a natural cohomological refinement of algebraic $K$-theory on arbitrary characteristic~$p$ schemes. In characteristic zero, the presheaves $R\Gamma_{\text{Zar}}(-,\widehat{\mathbb{L}\Omega}^{\geq i}_{-/\Q})$ do not vanish in weights $i<0$. Instead, they are equal to the presheaf $R\Gamma_{\text{Zar}}(-,\widehat{\mathbb{L}\Omega}_{-/\Q})$, which happens to be a cdh sheaf on qcqs $\Q$-schemes, by results of Corti\~{n}as--Haesemeyer--Schlichting--Weibel \cite{cortinas_cyclic_2008}, Antieau \cite{antieau_periodic_2019}, and Elmanto--Morrow \cite{elmanto_motivic_2023}. That is, the right vertical map of the previous diagram is an equivalence in weights $i<0$, so the presheaves $\Z(i)^{\text{mot}}$ vanish in weights $i<0$, and the zeroth step of the associated motivic filtration $\text{Fil}^\star_{\text{mot}} \text{K}$ recovers algebraic $K$-theory. This means that the presheaves $\Z(i)^{\text{mot}}$ provide a natural cohomological refinement of algebraic $K$-theory on arbitrary characteristic zero schemes. In mixed characteristic, we prove similarly that in weights $i<0$, the presheaves $\Z(i)^{\text{TC}}$ are cdh sheaves on qcqs schemes, {\it i.e.}, that the presheaves $\Z(i)^{\text{mot}}$ vanish. The result modulo a prime number $p$ is a consequence, as in characteristic $p$, of the fact that the presheaves $\Z_p(i)^{\text{BMS}}$ vanish in weights~$i<0$. The difficulty is to then prove that the presheaves $\Q(i)^{\text{TC}}$ are cdh sheaves in weight $i<0$. The main cdh descent result used in characteristic zero does not hold in mixed characteristic. That is, the presheaf $R\Gamma_{\text{Zar}}(-,\widehat{\mathbb{L}\Omega}_{-/\Z})$ (or its rationalisation) is not a cdh sheaf on qcqs schemes. We avoid this difficulty by proving a rigid-analytic analogue of this cdh descent result. To formulate this result, denote by $$R\Gamma_{\text{Zar}}\big(X,\underline{\widehat{\mathbb{L}\Omega}}^{\geq i}_{-_{\Q_p}/\Q_p}\big)$$ the {\it rigid-analytic derived de Rham cohomology} of a qcqs $\Z_{(p)}$-scheme $X$, which we define as the pushout of the diagram $$R\Gamma_{\text{Zar}}\big(X,(\widehat{\mathbb{L}\Omega}^{\geq i}_{-/\Z})^\wedge_p\big) \longleftarrow R\Gamma_{\text{Zar}}\big(X,\widehat{\mathbb{L}\Omega}^{\geq i}_{-/\Z}) \longrightarrow R\Gamma_{\text{Zar}}(X,\widehat{\mathbb{L}\Omega}^{\geq i}_{-_{\Q}/\Q})$$ in the derived category $\mathcal{D}(\Z)$. Here, we restrict our attention to qcqs $\Z_{(p)}$-schemes for simplicity, and refer to Section~\ref{sectionratonialstructure} for the relevant statements over $\Z$. As a consequence of Definition~\ref{definitionintroZ(i)^TC}, there is a natural cartesian square $$\begin{tikzcd} \Q(i)^{\text{TC}}(X) \ar[r] \ar[d] & R\Gamma_{\text{Zar}}(X,\widehat{\mathbb{L}\Omega}^{\geq i}_{-_{\Q}/\Q}) \ar[d] \\ \Q_p(i)^{\text{BMS}}(X) \ar[r] & R\Gamma_{\text{Zar}}(X,\underline{\widehat{\mathbb{L}\Omega}}^{\geq i}_{-_{\Q_p}/\Q_p}) \end{tikzcd}$$ in the derived category $\mathcal{D}(\Q)$. In weights $i<0$, the presheaves $\Q_p(i)^{\text{BMS}}$ vanish. As already mentioned, the presheaf $R\Gamma_{\text{Zar}}(-,\widehat{\mathbb{L}\Omega}_{-_{\Q}/\Q})$ is moreover a cdh sheaf on qcqs schemes. So the fact that the presheaves $\Q(i)^{\text{TC}}$ are cdh sheaves in weights $i<0$ reduces to the following result, which can be seen as a rigid-analytic analogue of the latter cdh descent over $\Q$. \begin{theoremintro}[Cdh descent for rigid-analytic derived de Rham cohomology; see Corollary~\ref{corollaryHPsolidallprimespisacdhsheafongradedpieces}]\label{theoremintrocdhdescentrigidanalytic} For every prime number $p$, the presheaf $R\Gamma_{\emph{Zar}}\big(-,\underline{\widehat{\mathbb{L}\Omega}}_{-_{\Q_p}/\Q_p}\big)$ satisfies cdh descent on qcqs $\Z_{(p)}$\nobreakdash-schemes. \end{theoremintro} The modern proof of the analogous result over $\Q$ relies on the theory of truncating invariants of Land--Tamme \cite{land_k-theory_2019} and on a theorem of Goodwillie \cite{goodwillie_cyclic_1985}, who prove respectively that every truncating invariant is a cdh sheaf on qcqs schemes and that periodic cyclic homology over $\Q$ is a truncating invariant. By definition, a truncating invariant is a localizing invariant $E$ such that for every connective $\mathbb{E}_1$-ring $R$, the natural map $E(R) \rightarrow E(\pi_0(R))$ is an equivalence. To prove Theorem~\ref{theoremintrocdhdescentrigidanalytic}, we then use the condensed mathematics of Clausen--Scholze \cite{clausen_condensed_2019}, and prove that a suitable rigid-analytic variant of periodic cyclic homology is a truncating invariant. In particular, the proof of Theorem~\ref{theoremintrocdhdescentrigidanalytic} relies on a result on associative rings (actually, on general solid connective $\mathbb{E}_1$-rings). As a consequence of Theorem~\ref{theoremintrocdhdescentrigidanalytic}, we obtain the following cohomological description of rational motivic cohomology. \begin{theoremintro}[$p$-adic and rational motivic cohomology; see Corollaries~\ref{corollarymainpadicstructureongradeds}\label{theoremintromaincartesiansquares} and~\ref{corollarycartesiansquarerational}] Let $X$ be a qcqs scheme, and $p$ be a prime number. Then for any integers $i \in \Z$ and $k \geq 1$, the natural commutative diagrams $$\begin{tikzcd} \Z/p^k(i)^{\emph{mot}}(X) \ar[r] \ar[d] & \Z/p^k(i)^{\emph{BMS}}(X) \ar[d] & \Q(i)^{\emph{mot}}(X) \ar[r] \ar[d] & R\Gamma_{\emph{Zar}}\big(X,\widehat{\mathbb{L}\Omega}^{\geq i}_{-_{\Q}/\Q}\big) \ar[d] \\ \Z/p^k(i)^{\emph{cdh}}(X) \ar[r] & \big(L_{\emph{cdh}} \Z/p^k(i)^{\emph{BMS}}\big)(X) & \Q(i)^{\emph{cdh}}(X) \ar[r] & R\Gamma_{\emph{cdh}}\big(X,\widehat{\mathbb{L}\Omega}^{\geq i}_{-_{\Q}/\Q}\big) \end{tikzcd}$$ are cartesian squares in the derived category $\mathcal{D}(\Z)$. \end{theoremintro} Together, these two cartesian squares recover the cartesian squares of Elmanto--Morrow that define the motivic complexes $\Z(i)^{\text{mot}}$ over a field, and are thus natural mixed characteristic analogues of these. The $p$-adic part of Theorem~\ref{theoremintromaincartesiansquares} is a formal consequence of Definitions~\ref{definitionintroZ(i)^TC} and~\ref{definitionintromotiviccohomology}. The rational part of Theorem~\ref{theoremintromaincartesiansquares} implies that the difference between $\Q(i)^{\text{mot}}(X)$ and $\Q(i)^{\text{cdh}}(X)$ depends only on the rationalisation $X_{\Q}$ of the scheme $X$. If $X$ is regular, this difference should vanish, and this is then more interesting in the presence of singularities. More precisely, Theorem~\ref{theoremintromaincartesiansquares} can be used to capture interesting information about the singularities of an arbitrary commutative ring $R$: cdh sheaves are typically insensitive to singularities, so the singular information in the motivic complex $\Z(i)^{\text{mot}}(R)$ is controlled by the complexes $\Z/p^k(i)^{\text{BMS}}(R)$ and $\mathbb{L}\Omega^{<i}_{(R\otimes_{\Z} \Q)/\Q}$, which are accessible to computation. Theorem~\ref{theoremintromaincartesiansquares} also implies that the presheaves $\Q(i)^{\text{mot}}$ vanish in weights~$i<0$, which was the essential missing part to establish the following fundamental properties of motivic cohomology. \begin{theoremintro}[Relation to algebraic $K$-theory]\label{theoremintroAHSSandAdams} There exists a finitary Nisnevich sheaf of filtered spectra $$\emph{Fil}^\star_{\emph{mot}} \emph{K}(-) : \emph{Sch}^{\emph{qcqs,op}} \longrightarrow \emph{FilSp}$$ with the following properties: \begin{enumerate} \item \emph{(Atiyah--Hirzebruch spectral sequence; see Section~\ref{subsectionAHSS})} For every qcqs scheme $X$, the filtration $\emph{Fil}^\star_{\emph{mot}} \emph{K}(X)$ is a multiplicative $\N$-indexed filtration on the non-connective algebraic $K$-theory $\emph{K}(X)$, whose graded pieces are naturally given by $$\emph{gr}^i_{\emph{mot}} \emph{K}(X) \simeq \Z(i)^{\emph{mot}}(X)[2i], \quad i \geq 0.$$ In particular, writing $\emph{H}^j_{\emph{mot}}(X,\Z(i)) := \emph{H}^j(\Z(i)^{\emph{mot}}(X))$ for the corresponding \emph{motivic cohomology groups}, there exists an Atiyah--Hirzebruch spectral sequence $$E_2^{i,j} = \emph{H}^{i-j}_{\emph{mot}}(X,\Z(-j)) \Longrightarrow \emph{K}_{-i-j}(X).$$ If $X$ has finite valuative dimension,\footnote{The valuative dimension of a commutative ring, defined in terms of the ranks of certain valuation rings, was introduced by Jaffard in \cite[Chapter IV]{jaffard_theorie_1960}, and generalised to schemes in \cite[Section~$2.3$]{elmanto_cdh_2021}. The valuative dimension of a scheme is always at least equal to its Krull dimension, and both notions agree on noetherian schemes. For our purposes, the valuative dimension of a qcqs scheme $X$ will be used as an upper bound on the cohomological dimension of the cdh topos of $X$ (\cite[Theorem~$2.4.15$]{elmanto_cdh_2021}).} then the filtration $\emph{Fil}^\star_{\emph{mot}} \emph{K}(X)$ is complete, and the Atiyah--Hirzebruch spectral sequence is convergent. \item \emph{(Adams decomposition; see Corollary~\ref{corollaryKtheorysplitsrationally})} For every qcqs scheme $X$, the Atiyah--Hirzebruch spectral sequence degenerates rationally and, for every integer $n \in \Z$, there is a natural isomorphism of abelian groups $$\emph{K}_n(X) \otimes_{\Z} \Q \cong \bigoplus_{i \geq 0} \big(\emph{H}^{2i-n}_{\emph{mot}}(X,\Z(i)) \otimes_{\Z} \Q\big)$$ induced by the Adams operations on rationalised algebraic $K$-theory. \end{enumerate} \end{theoremintro} One of the main historical motivations for developing motivic cohomology was to apply cohomological techniques to the study of algebraic $K$-theory \cite{beilinson_notes_1987}. The following theorem summarizes our results on the relations between motivic cohomology and previously studied cohomological invariants. When $X$ is smooth over $\Z$, we denote by $$\Z(i)^{\text{cla}}(X) := z^i(X,\bullet)[-2i]$$ the {\it weight-$i$ classical motivic complex}, where $z^i(X,\bullet)$ is Bloch's cycle complex (and $\bullet$ is the cohomological index). \needspace{5\baselineskip} \begin{theoremintro}\label{theoremintrocohomology} Let $X$ be a qcqs scheme, and $i \geq 0$ be an integer. \begin{enumerate} \item \emph{(Weight zero; see Example~\ref{exampleweightzeromotiviccohomology})} There is a natural equivalence $$\Z(0)^{\emph{mot}}(X) \xlongrightarrow{\sim} R\Gamma_{\emph{cdh}}(X,\Z)$$ in the derived category $\mathcal{D}(\Z)$. \item \emph{(Weight one; see Example~\ref{exampleweightonemotiviccohomology})} There is a natural map $$R\Gamma_{\emph{Nis}}(X,\mathbb{G}_m)[-1] \longrightarrow \Z(1)^{\emph{mot}}(X)$$ in the derived category $\mathcal{D}(\Z)$ which is an isomorphism in degrees less than or equal to three. \item \emph{(\'Etale cohomology; see Corollary~\ref{corollaryladicmotivcohomologylowdegreesisétalecohomology})} For every prime number $\l$ which is invertible in $X$ and every integer $k \geq 1$, there is a natural map $$\Z/\l^k(i)^{\emph{mot}}(X) \longrightarrow R\Gamma_{\emph{ét}}(X,\mu_{\l^k}^{\otimes i})$$ in the derived category $\mathcal{D}(\Z/\l^k)$ which is an isomorphism in degrees less than or equal to $i$. \item \emph{(Syntomic cohomology; see Corollary~\ref{corollarypadiccomparisoninsmalldegreessyntomiccoho})} For every prime number $p$ and every integer $k \geq 1$, there is a natural map $$\Z/p^k(i)^{\emph{mot}}(X) \longrightarrow \Z/p^k(i)^{\emph{syn}}(X)$$ in the derived category $\mathcal{D}(\Z/p^k)$ which is an isomorphism in degrees less than or equal to $i$, where $\Z/p^k(i)^{\emph{syn}}(X)$ denotes the weight-$i$ syntomic cohomology of $X$ in the sense of \cite{bhatt_absolute_2022}. \item \emph{(Milnor $K$-theory; see Theorem~\ref{theoremcomparisontoMilnorKtheory})} If $X=\emph{Spec}(A)$ is the spectrum of a henselian local ring~$A$, then for every integer $n \geq 1$, there is a natural isomorphism $$\widehat{\emph{K}}{}^{\emph{M}}_i(A)/n \xlongrightarrow{\cong} \emph{H}^i_{\emph{mot}}(A,\Z(i))/n$$ of abelian groups, where $\widehat{\emph{K}}{}^{\emph{M}}_i(A)$ denotes the $i^{\emph{th}}$ improved Milnor $K$-group of $A$ in the sense of \cite{kerz_milnor_2010}. \item \emph{(Classical motivic cohomology; see Section~\ref{sectionclassical})} If $X$ is smooth over $\Z$, then there is a natural map $$\Z(i)^{\emph{cla}}(X) \longrightarrow \Z(i)^{\emph{mot}}(X)$$ in the derived category $\mathcal{D}(\Z)$ which is an isomorphism in degrees less than or equal to $i+1$ in general, and an isomorphism in all degrees if $X$ has dimension less than or equal to one over $\Z$. \item \emph{(Lisse motivic cohomology; see Corollary~\ref{corollarylissemotivicmaincomparisontheorem})} If $X=\emph{Spec}(A)$ is the spectrum of a local ring $A$, then for every integer $i \geq 0$, there is a natural equivalence $$\Z(i)^{\emph{lisse}}(A) \xlongrightarrow{\sim} \tau^{\leq i} \Z(i)^{\emph{mot}}(A)$$ in the derived category $\mathcal{D}(\Z)$, where $\Z(i)^{\emph{lisse}}$ denotes the weight-$i$ lisse motivic cohomology of~$A$, defined as the left Kan extension from smooth $\Z$-algebras of the classical motivic complex $\Z(i)^{\emph{cla}}$. In particular, the functor $\tau^{\leq i} \Z(i)^{\emph{mot}}$ is left Kan extended on local rings from local essentially smooth $\Z$-algebras. \item \emph{($\mathbb{A}^1$-invariant motivic cohomology; see Theorem~\ref{theoremA1localmotiviccohomologymain})} There is a natural equivalence $$\big(L_{\mathbb{A}^1} \Z(i)^{\emph{mot}}\big)(X) \xlongrightarrow{\sim} \Z(i)^{\mathbb{A}^1}(X)$$ in the derived category $\mathcal{D}(\Z)$, where $\Z(i)^{\mathbb{A}^1}(X)$ denotes the cohomology represented by the slices of the $K$-theory motivic spectrum $\emph{KGL}_X \in \emph{SH}(X)$. \end{enumerate} \end{theoremintro} The following result is a consequence of Theorem~\ref{theoremintrocohomology}\,$(8)$ and the fact that the $\mathbb{A}^1$-invariant motivic complexes $\Z(i)^{\mathbb{A}^1}$ recover the classical motivic complexes $\Z(i)^{\text{cla}}$ on smooth $\Z$-schemes. In particular, although we expect our motivic complexes $\Z(i)^{\text{mot}}$ to actually coincide with the classical motivic complexes $\Z(i)^{\text{cla}}$ on smooth $\Z$-schemes, this means that the former at least recover the latter after enforcing $\mathbb{A}^1$-invariance. \begin{corollaryintro}[See Corollary~\ref{corollary26A1localmotiviccohomologyisclassicalmotiviccohomology}] Let $X$ be a smooth scheme over $\Z$. Then for every integer $i \geq 0$, there is a natural equivalence $$\Z(i)^{\emph{cla}}(X) \simeq \big(L_{\mathbb{A}^1} \Z(i)^{\emph{mot}}\big)(X)$$ in the derived category $\mathcal{D}(\Z)$. \end{corollaryintro} Assuming the existence of a well-behaved derived category of motives $\text{DM}$, the motivic cohomology groups of a scheme $X$ should be given by $$\text{H}^j_{\text{mot}}(X,\Z(i)) \cong \text{Hom}_{\text{DM}}(\text{M}(X),\Z(i)[j]),$$ where $\text{M}(X) \in \text{DM}$ is the motive associated to $X$, and $\Z(i) \in \text{DM}$ are the Tate motives, fitting for every integer $r \geq 0$ in a natural decomposition in $\text{DM}$: $$\text{M}(\mathbb{P}^r_{\Z}) \cong \bigoplus_{j=0}^r \Z(j)[2j].$$ In the $\mathbb{A}^1$-invariant framework, Voevodsky constructed such a derived category of motives, in which the classical motivic complexes $\Z(i)^{\text{cla}}$ can be interpreted in terms of these Tate motives. Without assuming $\mathbb{A}^1$-invariance, Annala--Iwasa \cite{annala_motivic_2023} and Annala--Hoyois--Iwasa \cite{annala_algebraic_2023,annala_atiyah_2024} recently introduced a more general derived category of motives, where the decomposition of the motive $\text{M}(\mathbb{P}^r_{\Z})$, {\it i.e.}, the {\it projective bundle formula}, is isolated as the key defining property. The following result states that the motivic complexes $\Z(i)^{\text{mot}}$ fit within this theory of non-$\mathbb{A}^1$-invariant motives. \begin{theoremintro}[Projective bundle formula; see Theorem~\ref{theoremprojectivebundleformula}]\label{theoremintroprojectivebundleformula} Let $X$ be a qcqs scheme, $i \geq 0$ be an integer, and $\mathcal{E}$ be a vector bundle of rank $r$ on $X$. Then for every integer $i \geq 0$, the powers of the motivic first Chern class $c_1^{\emph{mot}}(\mathcal{O}(1)) \in \emph{H}^2_{\emph{mot}}(\mathbb{P}_X(\mathcal{E}),\Z(1))$ induce a natural equivalence $$\bigoplus_{i=0}^{r-1} \Z(i-j)^{\emph{mot}}(X)[-2j] \xlongrightarrow{\sim} \Z(i)^{\emph{mot}}(\mathbb{P}_X(\mathcal{E}))$$ in the derived category $\mathcal{D}(\Z)$. \end{theoremintro} Theorem~\ref{theoremintroprojectivebundleformula} is proved by Elmanto--Morrow in the equicharacteristic case \cite{elmanto_motivic_2023}, where the proof relies on the projective bundle formula for the complexes $\Z(i)^{\text{cdh}}$ \cite{bachmann_A^1-invariant_2024}. In mixed characteristic, however, the cdh-local motivic complexes $\Z(i)^{\text{cdh}}$ are known to satisfy the projective bundle formula only conditionally on a certain property of valuation rings, called $F$-smoothness \cite{bhatt_syntomic_2023,bachmann_A^1-invariant_2024}. This condition can be proved in mixed characteristic for valuation rings over a perfectoid base: this is the main result of \cite{bouis_cartier_2023}. The case of general valuation rings remaining open, our proof of Theorem~\ref{theoremintroprojectivebundleformula} is different from that of Elmanto--Morrow, and uses in particular our description of motivic cohomology with finite coefficients in terms of syntomic cohomology (Theorem~\ref{theorempadicmotiviccohomologyintermsofsyntomicohomology}). \subsection{Motivic cohomology of regular schemes} \vspace{-\parindent} \hspace{\parindent} Recall that on smooth $\F_p$\nobreakdash-schemes, the Beilinson--Lichtenbaum conjecture, proved by Suslin and Voevodsky as a consequence of the Bloch--Kato conjecture \cite{suslin_bloch-kato_2000}, computes the $\l$\nobreakdash-adic part of motivic cohomology in terms of the cohomology of the étale sheaf $\mu_{\l^k}$ of $\l^k$\nobreakdash-roots of unity. To describe the $p$\nobreakdash-adic part of motivic cohomology, one needs to replace $\mu_{\l^k}^{\otimes i}$ (which is zero on smooth varieties when $\l=p$ and~$i>0$) by the logarithmic de Rham--Witt sheaves $W_k\Omega^i_{-,\text{log}}$ \cite{geisser_k-theory_2000}. The corresponding description of $p$\nobreakdash-adic algebraic $K$\nobreakdash-theory, in terms of the logarithmic de Rham--Witt sheaves, is generalised in \cite{kelly_k-theory_2021} to all Cartier smooth $\F_p$\nobreakdash-algebras, and in particular to all characteristic $p$ valuation rings. On smooth schemes over a mixed characteristic Dedekind domain, the $p$-adic part of classical motivic cohomology is similarly described in low degrees by the étale cohomology of the generic fibre \cite{geisser_motivic_2004}. This result is a consequence of the Gersten conjecture proved by Geisser \cite{geisser_motivic_2004}, and is unknown for general regular schemes. Combined with Theorem~\ref{theoremintrocohomology}\,$(6)$, the following result extends this description of classical motivic cohomology to the regular case. More precisely, the notion of $F$-smoothness was introduced by Bhatt--Mathew \cite{bhatt_syntomic_2023} as a non-noetherian generalisation of regular schemes, and our result naturally applies to general $p$-torsionfree $F$-smooth schemes. \begin{theoremintro}[Beilinson--Lichtenbaum conjecture for $F$-smooth schemes; see Corollary~\ref{corollaryFsmoothnessBeilinsonLichtenbaumcomparison}]\label{theoremintroBeilinsonLichtenbaumconjectureforFsmoothschemes} Let $X$ be a $p$-torsionfree $F$-smooth scheme ({\it e.g.}, a regular scheme flat over $\Z$). Then for any integers $i \geq 0$ and $k \geq 1$, the Beilinson--Lichtenbaum comparison map $$\Z/p^k(i)^{\emph{mot}}(X) \longrightarrow R\Gamma_{\emph{ét}}(X[\tfrac{1}{p}],\mu_{p^k}^{\otimes i})$$ is an isomorphism in degrees less than or equal to $i-1$, and is injective in degree $i$. \end{theoremintro} Valuation ring extensions of a perfectoid valuation ring being $F$-smooth by \cite{bouis_cartier_2023}, we then obtain the following complete description of the $p$-adic motivic cohomology of valuation rings over a perfectoid base. \begin{corollaryintro}[Motivic cohomology of valuation rings; see Theorem~\ref{theoremvaluationringsmotiviccohomologyfinitecoefficients}] Let $V_0$ be a $p$\nobreakdash-torsionfree valuation ring whose $p$\nobreakdash-completion is a perfectoid ring, and $V$ be a henselian valuation ring extension of $V_0$. Then for any integers $i \geq 0$ and $k \geq 1$, the motivic complex $\Z/p^k(i)^{\emph{mot}}(V)$ is in degrees at most $i$, and the Beilinson--Lichtenbaum comparison map $$\Z/p^k(i)^{\emph{mot}}(V) \longrightarrow R\Gamma_{\emph{ét}}(\emph{Spec}(V[\tfrac{1}{p}]),\mu_{p^k}^{\otimes i})$$ is an isomorphism in degrees less than or equal to $i-1$. On $\emph{H}^i$, this map is injective, with image generated by symbols, via the symbol map $$(V^\times)^{\otimes i} \rightarrow \emph{H}^i_{\emph{ét}}(\emph{Spec}(V[\tfrac{1}{p}]),\mu_{p^k}^{\otimes i}).$$ \end{corollaryintro} The proof of Theorem~\ref{theoremintroBeilinsonLichtenbaumconjectureforFsmoothschemes} relies on a syntomic-étale comparison theorem of Bhatt--Mathew \cite{bhatt_syntomic_2023} (see also \cite[Theorem~B]{bouis_cartier_2023} for a proof over perfectoid bases using relative prismatic cohomology). \subsection{Motivic cohomology of singular schemes} \vspace{-\parindent} \hspace{\parindent} For the rest of this introduction, we focus on the properties of motivic cohomology that are specific to singular schemes. One of the most interesting, yet mysterious features of the algebraic $K$-theory of singular schemes is the presence of nonzero negative $K$-groups. Most of the current understanding of negative $K$-groups relies on results on the behaviour of algebraic $K$-theory with respect to blowups \cite{cortinas_cyclic_2008,kerz_algebraic_2018}. It was proved in particular by Thomason \cite{thomason_K-groupes_1993} that algebraic $K$-theory sends the blowup square associated to a regular closed immersion to a long exact sequence of $K$-groups. The following result is a cohomological refinement of Thomason's result. \begin{theoremintro}[Regular blowup formula; see Theorem~\ref{theoremregularblowupformula}] For every regular closed immersion $Y \rightarrow X$ of qcqs schemes ({\it i.e.}, the closed subscheme $Y$ is Zariski-locally on $X$ defined by a regular sequence) and every integer $i \geq 0$, the commutative diagram $$\begin{tikzcd} \Z(i)^{\emph{mot}}(X) \ar[r] \ar[d] & \Z(i)^{\emph{mot}}(Y) \ar[d] \\ \Z(i)^{\emph{mot}}(\emph{Bl}_Y(X)) \ar[r] & \Z(i)^{\emph{mot}}(\emph{Bl}_Y(X) \times_X Y) \end{tikzcd}$$ is a cartesian square in the derived category $\mathcal{D}(\Z)$. \end{theoremintro} However, algebraic $K$-theory fails to associate long exact sequences to general blowups. Motivated by Grothendieck's theorem on formal functions for quasi-coherent cohomology \cite[Theorem~$4.1.5$]{grothendieck_elements_1961}, many people hoped for a formal analogue of these long exact sequences in algebraic $K$-theory, that would hold for general blowups. This was finally proved by Kerz--Strunk--Tamme \cite{kerz_algebraic_2018}, in the form of a pro cdh excision property for the algebraic $K$-theory of arbitrary noetherian schemes. Note that the pro cdh topology was recently introduced by Kelly--Saito \cite{kelly_procdh_2024}, as a way to encode this pro cdh excision property in a descent property for this Grothendieck topology. The following result relies in particular on Theorem~\ref{theoremintromaincartesiansquares} and on Grothendieck's theorem on formal functions, thus shedding some light on the analogy between quasi-coherent and $K$-theoretic techniques. \begin{theoremintro}[Pro cdh descent; see Theorem~\ref{theoremprocdhdescentformotiviccohomology}]\label{theoremintroprocdhdescent} For every integer $i \geq 0$, the presheaf $\Z(i)^{\emph{mot}}$ satisfies pro cdh descent on noetherian schemes. That is, for every abstract blowup square $$\begin{tikzcd} Y' \ar[r] \ar[d] & X' \ar[d] \\ Y \ar[r] & X \end{tikzcd}$$ of noetherian schemes, the associated commutative diagram $$\begin{tikzcd} \Z(i)^{\emph{mot}}(X) \ar[r] \ar[d] & \Z(i)^{\emph{mot}}(X') \ar[d] \\ \{\Z(i)^{\emph{mot}}(rY)\}_r \ar[r] & \{\Z(i)^{\emph{mot}}(rY')\}_r \end{tikzcd}$$ is a weakly cartesian square of pro objects in the derived category $\mathcal{D}(\Z)$. \end{theoremintro} Kelly--Saito moreover proved that non-connective algebraic $K$-theory not only satisfies pro cdh descent, but is the pro cdh sheafification of connective algebraic $K$-theory \cite{kelly_procdh_2024}. Combined with the observation of Bhatt--Lurie that connective algebraic $K$-theory is left Kan extended on commutative rings from smooth $\Z$-algebras \cite{elmanto_modules_2020}, this motivated the definition of the {\it pro cdh motivic complexes} $\Z(i)^{\text{procdh}}$, as the pro cdh sheafification of the left Kan extension of the classical motivic complexes~$\Z(i)^{\text{cla}}$. The following result relies on the comparison to lisse motivic cohomology (Theorem~\ref{theoremintrocohomology}\,$(7)$) and pro cdh descent (Theorem~\ref{theoremintroprocdhdescent}). \begin{corollaryintro}[Comparison to pro cdh motivic cohomology; see Theorem~\ref{theoremcomparisonprocdhmotivic}] Let $X$ be a noetherian scheme. Then for every integer $i \geq 0$, there is a natural equivalence $$\Z(i)^{\emph{procdh}}(X) \xlongrightarrow{\sim} \Z(i)^{\emph{mot}}(X)$$ in the derived category $\mathcal{D}(\Z)$. \end{corollaryintro} Note that the pro cdh motivic complexes $\Z(i)^{\text{procdh}}$ are not finitary, so they cannot coincide with the motivic complexes $\Z(i)^{\text{mot}}$ on general qcqs schemes. An important conjecture of Weibel \cite{weibel_K-theory_1980} states that for every noetherian scheme $X$ of dimension at most $d$, the negative $K$-groups $\text{K}_{-n}(X)$ vanish for integers $n > d$. This conjecture was settled by Kerz--Strunk--Tamme \cite{kerz_algebraic_2018}, as a consequence of pro cdh descent for algebraic $K$-theory. The proof of the following result uses the techniques of Kerz--Strunk--Tamme \cite{kerz_algebraic_2018} as reformulated by Elmanto--Morrow \cite{elmanto_motivic_2023}, who proved the same result over a field. In particular, Theorem~\ref{theoremintroweibelvanishing} relies on pro cdh descent for motivic cohomology (Theorem~\ref{theoremintroprocdhdescent}). \begin{theoremintro}[Motivic Weibel vanishing; see Theorem~\ref{theoremmotivicWeibelvanishing}]\label{theoremintroweibelvanishing} Let $X$ be a noetherian scheme of finite dimension $d$, and $i \geq 0$ be an integer. Then for every integer $j > i+d$, the motivic cohomology group $\emph{H}^j_{\emph{mot}}(X,\Z(i))$ is zero. \end{theoremintro} Via the Atiyah--Hirzebruch spectral sequence of Theorem~\ref{theoremintroAHSSandAdams}, Theorem~\ref{theoremintroweibelvanishing} is a motivic refinement of Weibel's vanishing conjecture in $K$-theory. \subsection{Notation} \smallskip {\bf Algebraic $K$-theory.} By default, algebraic $K$-theory means non-connective algebraic $K$-theory, as introduced by Thomason--Trobaugh \cite{thomason_higher_1990}. By a theorem of Blumberg--Gepner--Tabuada \cite{blumberg_universal_2013}, non\nobreakdash-connective algebraic $K$-theory is the universal localizing invariant. \bigskip \noindent {\bf $\mathbb{A}^1$-invariance.} A presheaf $F(-)$ on schemes is called {\it $\mathbb{A}^1$-invariant} if for every scheme $X$ and every integer~\hbox{$m \geq 0$}, the natural map $F(X) \rightarrow F(\mathbb{A}^m_X)$ is an equivalence. Given a presheaf $F(-)$ on schemes, the {\it $\mathbb{A}^1$-localisation} $L_{\mathbb{A}^1} F(-)$ of $F(-)$ is the initial $\mathbb{A}^1$-invariant presheaf with a map from $F(-)$. The $\mathbb{A}^1$-localisation functor $L_{\mathbb{A}^1}$ commutes with colimits. \bigskip \noindent {\bf Base change.} Given a commutative ring $R$, an $R$-algebra $S$, and a scheme $X$ over $\text{Spec}(R)$, denote by~$X_{S}$ the base change $X \times_{\text{Spec}(R)} \text{Spec}(S)$ of $X$ from $R$ to $S$. If $X$ is a derived scheme, this base change is implicitly the derived base change from $R$ to $S$. We sometimes use the derived base even on classical schemes, and say explicitly when we do so. \bigskip \noindent {\bf Bounded torsion.} An abelian group $A$ is said to have {\it bounded torsion} if there exists an integer $N \geq 1$ such that the multiplication by $N$ of every element of $A$ is zero. Given a commutative ring $R$ and an element $d$ of $R$, an $R$-module $M$ is said to have {\it bounded $d$-power torsion} if there exists an integer $n\geq 1$ such that $M[d^m]=M[d^n]$ for all $m \geq n$; this assumption guarantees that the derived $d$\nobreakdash-completion of $M$ is in degree zero, given by the classical $d$\nobreakdash-completion of $M$. \bigskip \noindent {\bf Cdh topology.} The cdh topology is a Grothendieck topology introduced by Voevodsky \cite{suslin_bloch-kato_2000,voevodsky_unstable_2010}; see \cite{elmanto_cdh_2021} for the definition and properties of the cdh topology in the generality of qcqs schemes. It is a completely decomposed version of the topology generated by Deligne's hypercoverings. The cdh sheafification functor $L_{\text{cdh}}$ preserves multiplicative structures. \bigskip \noindent {\bf Coefficients.} Given a functor $F(-)$ (resp. $\text{Fil}^\star F(-)$) taking values in spectra (resp. filtered spectra) and a prime number $p$, we denote the rationalisation of $F(-)$ by $F(-;\Q)$, its reduction modulo~$p$ by $F(-;\F_p)$, its derived reduction modulo powers of $p$ by $F(-;\Z/p^k)$, its $p$-completion by $F(-;\Z_p)$, and the rationalisation of its $p$-completion by $F(-;\Q_p)$. We adopt a similar notation for a functor $\text{Fil}^\star F(-)$ taking values in filtered spectra, {\it e.g.}, we denote its rationalisation by $\text{Fil}^{\star} F(-;\Q)$. Similarly, if $\Z(i)^F(-)$ is a functor taking values in the derived category $\mathcal{D}(\Z)$, we denote the rationalisation of $\Z(i)^F(-)$ by $\Q(i)^F(-)$, its derived reduction modulo $p$ by $\F_p(i)^F(-)$, etc. Following the same notation, we also write $$\prod_{p \in \mathbb{P}}{}^{'} F(-;\Q_p) := \Big(\prod_{p \in \mathbb{P}} F(-;\Z_p)\Big)_{\Q} \quad \Big(\text{resp.} \prod_{p \in \mathbb{P}}{}^{'} \text{Fil}^\star F(-;\Q_p) := \Big(\prod_{p \in \mathbb{P}} \text{Fil}^\star F(-;\Z_p)\Big)_{\Q}\Big).$$ \bigskip \noindent {\bf Derived categories and spectra.} Denote by $\text{Sp}$ the category of spectra. Given a commutative ring $R$, denote by $\mathcal{D}(R)$ the derived category of $R$-modules; it is implicitly the derived $\infty$-category of $R$-modules, and is in particular naturally identified with the category of $R$-linear spectra. Our convention for degrees is by default cohomological. In this context, the notions of fibre and cofibre sequences agree, and the fibre and cofibre of a given map satisfy the relation $\text{fib} \simeq \text{cofib}[-1]$. Given an element $d$ of $R$, also denote by $(-)^\wedge_d$ the $d$-adic completion functor in the derived category~$\mathcal{D}(R)$. \bigskip \noindent {\bf Extension by zero $j_!$.} Given a prime number $p$ (which is typically clear from context) and a scheme $X$, denote by $j : X[\tfrac{1}{p}] \rightarrow X$ the open immersion of the $p$-adic generic fibre of $X$, and by $j_! : (X[\tfrac{1}{p}])_{\text{ét}} \rightarrow X_{\text{ét}}$ the associated extension by zero functor. \bigskip \noindent {\bf Filtrations.} By default, a filtration with values in a category $\mathcal{C}$ is a $\Z$-indexed decreasing filtered object in the category $\mathcal{C}$, {\it i.e.}, a functor from the category $(\Z,\geq)^\text{op}$ to the category $\mathcal{C}$. A filtration is called {\it $\N$-indexed} if it is constant in non-positive degrees. Given a filtered object $\text{Fil}^\star\, C$ and for each integer $n \in \Z$, let $\text{gr}^n\, C \in \mathcal{D}(R)$ denote the cofibre of the transition map $\text{Fil}^{n+1}\,C \rightarrow \text{Fil}^n\,C$. A filtered object $\text{Fil}^\star\, C$ is said to be {\it complete} if the limit $\text{lim}_n\, \text{Fil}^n\, C$ vanishes. For instance, The Hodge filtration on the de Rham complex is given for each $n \in \Z$ by $\text{Fil}_\text{Hod}^n\, \Omega_{-/R} := \Omega_{-/R}^{\geq n}$; the Hodge filtration $\mathbb{L}\Omega^{\geq \star}_{-/R}$ on the derived de Rham complex $\mathbb{L}\Omega_{-/R}$ is defined as the left Kan extension of this filtration. It is $\N$-indexed, but not always complete. Its completion, the Hodge-completed derived de Rham complex, is denoted by $\widehat{\mathbb{L}\Omega}^{\geq \star}_{-/R}$. Given a commutative ring $R$, denote by $\mathcal{DF}(R) := \text{Fun}((\Z,\geq)^\text{op},\mathcal{D}(R))$ the filtered derived category of $R$-modules. Also denote by $\text{FilSp}$ the category of filtered spectra, and by $\text{biFilSp}$ the category of bifiltered spectra ({\it i.e.}, the category of filtered objects in the category of filtered spectra). \bigskip \noindent {\bf Henselian rings.} Given a commutative ring $R$ and an ideal $I$ of $R$, the pair $(R,I)$ is called {\it henselian} if it satisfies Hensel's lemma. A local ring $R$ is called {\it henselian} if the pair $(R,\mathfrak{m})$ is henselian, where $\mathfrak{m}$ is the maximal ideal of $R$. Henselian local rings are the local rings for the Nisnevich topology. A commutative ring $R$ is called {\it $d$-henselian}, for $d$ an element of $R$, if the pair $(R,(d))$ is henselian. \bigskip \noindent {\bf Ind-smooth schemes.} Given a scheme $S$, a scheme $X$ is called ind-smooth (resp. ind-regular, ind-étale) over $S$ if it is a cofiltered limit of smooth (resp. regular, étale) $S$-schemes. \bigskip \noindent {\bf Left Kan extensions.} Given a commutative ring $R$, an $\infty$-category $\mathcal{D}$ which admits sifted colimits ({\it e.g.}, $\mathcal{D}(R)$ or $\mathcal{DF}(R)$), and a functor $$F : \text{Sm}_R := \{\text{smooth }R\text{-algebras}\} \longrightarrow \mathcal{D},$$ define $$\begin{array}{ll} \mathbb{L}F : &R\text{-Alg} \longrightarrow \mathcal{D} \\ &S \longmapsto \underset{P\rightarrow S}{\text{colim}} \text{ } F(P),\end{array}$$ where the colimit is taken over all free $R$-algebras $P$ mapping to $S$. The functor $\mathbb{L}F$ is called the left Kan extension from polynomial $R$-algebras of $F$. For instance, the cotangent complex $\mathbb{L}_{-/R} := \mathbb{L}\Omega^1_{-/R}$ is the left Kan extension from polynomial $R$-algebras of the module of Kähler differentials $\Omega^1_{-/R}$, and the derived de Rham complex $\mathbb{L}\Omega_{-/R}$ is the left Kan extension from polynomial $R$-algebras of the de Rham complex $\Omega_{-/R}$. We also consider more general left Kan extensions ({\it e.g.}, from smooth $R$-algebras), which are defined similarly --see \cite[Section~$2.3$ and Remark~$3.4$]{elmanto_motivic_2023} for a quick review of this formalism. The left Kan extension from a category~$\mathcal{C}$ to a category $\mathcal{C'}$, when this makes sense, is denoted by $L_{\mathcal{C}'/\mathcal{C}}$. \bigskip \noindent {\bf Quasisyntomic rings.} A morphism $R \rightarrow S$ of commutative rings is called {\it $p$-discrete}, for $p$ a prime number, if the derived tensor product $S \otimes_{R}^{\mathbb{L}} R/p \in \mathcal{D}(R/p)$ is concentrated in degree zero, where it is given by $S/p$. It is called {\it $p$-flat} if it is $p$-discrete and if its reduction $R/p \rightarrow S/p$ modulo $p$ is flat. It is called {\it $p$-quasisyntomic} if it is $p$-flat and if the cotangent complex $\mathbb{L}_{(S/p)/(R/p)} \in \mathcal{D}(S/p)$ has Tor-amplitude in $[-1;0]$. A commutative ring $R$ is called {\it $p$-quasisyntomic} if it has bounded $p$-power torsion and if the complex $\mathbb{L}_{R/\Z} \otimes_{R}^{\mathbb{L}} R/p \in \mathcal{D}(R/p)$ has Tor-amplitude in $[-1;0]$. Beware that $p$-quasisyntomic $\Z$-algebras are $p$-quasisyntomic rings, but the converse is not true: for instance, $\F_p$ is a $p$\nobreakdash-quasisyntomic ring, but the morphism $\Z \rightarrow \F_p$ is not $p$-discrete. We refer to \cite{bhatt_topological_2019} for the definition of the associated $p$-quasisyntomic topology on $p$-quasisyntomic rings. \bigskip \noindent {\bf Rigid functor.} A functor $F(-)$ on commutative rings is called rigid if for every henselian pair $(R,I)$, the natural map $F(R) \rightarrow F(R/I)$ is an equivalence. \bigskip \noindent {\bf Rings and schemes.} Quasi-compact quasi-separated (derived) schemes are called qcqs (derived) schemes. These include all affine (derived) schemes, {\it i.e.}, (animated) commutative rings. Denote by $\text{Sch}^{\text{qcqs}}$ the category of qcqs schemes, $\text{dSch}^{\text{qcqs}}$ the category of qcqs derived schemes, $\text{Rings}$ the category of commutative rings, $\text{AniRings}$ the category of animated commutative rings. Schemes, resp. commutative rings, are sometimes called {\it classical} to emphasize that we are not working in the generality of derived schemes, resp. animated commutative rings. Given a commutative base ring $R$, also denote by $\text{Sm}_R$ the category of smooth schemes over $\text{Spec}(R)$, $\text{Sch}^{\text{fp}}$ the category of finitely presented schemes over $\text{Spec}(R)$, $\text{Poly}_R$ the category of polynomial $R$-algebras, and $\mathbb{E}_1\text{-Rings}_{R}$ the category of associative $R$-linear ring spectra. \bigskip \noindent {\bf Sheafification.} We use several Grothendieck topologies, including the Zariski, Nisnevich, étale, and cdh topologies. Denote by $L_{\text{Zar}}$, $L_{\text{Nis}}$, $L_{\text{ét}}$, and $L_{\text{cdh}}$ the sheafification functors for these topologies. \subsection{Acknowledgements} \vspace{-\parindent} \hspace{\parindent} I am very grateful to Matthew Morrow for sharing many insights on motivic cohomology, and for careful readings of this manuscript. I would also like to thank Jacob Lurie and Georg Tamme for many helpful comments and corrections on a preliminary version of this paper, and Ben Antieau, Denis\nobreakdash-Charles Cisinski, Dustin Clausen, Frédéric Déglise, Elden Elmanto, Quentin Gazda, Marc \hbox{Hoyois}, Ryomei Iwasa, Shane Kelly, Niklas Kipp, Arnab Kundu, Shuji Saito, and Georg Tamme for helpful discussions. This project has received funding from the European Research Council (ERC) under the European Union’s Horizon 2020 research and innovation programme (grant agreement No. 101001474). \newpage \section{The motivic filtration on topological cyclic homology}\label{sectionmotivicfiltrationonTC} \vspace{-\parindent} \hspace{\parindent} In this section, we introduce a motivic filtration on the topological cyclic homology of qcqs derived schemes (Definition~\ref{definitionmotivicfiltrationonTC}), whose shifted graded pieces $\Z(i)^{\text{TC}}$ will serve as a building block for the definition of the motivic complexes $\Z(i)^{\text{mot}}$ (Remark~\ref{remarkmaincartesiansquareformotiviccohomology}). We first explain how to express topological cyclic homology in terms of its profinite completion and of negative cyclic homology. Following \cite{nikolaus_topological_2018}, and given a qcqs derived scheme~$X$ and a prime number~$p$, the $p$-completed topological cyclic homology $\text{TC}(X;\Z_p)$ of $X$ is constructed from its $p$-completed topological negative cyclic homology $\text{TC}^-(X;\Z_p)$ and its $p$-completed topological periodic cyclic homology $\text{TP}(X;\Z_p)$ (see Section~\ref{subsectionBMSfiltrations}). Following \cite[Lemma~$6.4.3.2$]{dundas_local_2013} and \cite[Section~II.$4$]{nikolaus_topological_2018}, the topological cyclic homology $\text{TC}(X)$ of $X$ is then defined by a natural cartesian square of spectra $$\begin{tikzcd} \text{TC}(X) \ar[r] \ar[d] & \text{TC}^-(X) \ar[d] \\ \prod_{p \in \mathbb{P}} \text{TC}(X;\Z_p) \ar[r] & \prod_{p \in \mathbb{P}} \text{TC}^-(X;\Z_p). \end{tikzcd}$$ The comparison map $\text{THH}(X) \rightarrow \text{HH}(X)$, induced by extension of scalars along the map of $\mathbb{E}_{\infty}$-rings $\text{THH}(\Z) \rightarrow \Z$, is $\text{S}^1$-equivariant, and for every integer $n \in \Z$, the kernel and cokernel of the induced map on homotopy groups $\text{THH}_n(X) \rightarrow \text{HH}_n(X)$ are killed by an integer depending only on $n$. In particular, the natural commutative diagram $$\begin{tikzcd} \text{THH}(X) \ar[r] \ar[d] & \text{HH}(X) \ar[d] \\ \prod_{p \in \mathbb{P}} \text{THH}(X;\Z_p) \ar[r] & \prod_{p \in \mathbb{P}} \text{HH}(X;\Z_p), \end{tikzcd}$$ is a cartesian square of spectra, which in turn defines a natural cartesian square of spectra $$\begin{tikzcd} \text{TC}^-(X) \ar[r] \ar[d] & \text{HC}^-(X) \ar[d] \\ \prod_{p \in \mathbb{P}} \text{TC}^-(X;\Z_p) \ar[r] & \prod_{p \in \mathbb{P}} \text{HC}^-(X;\Z_p) \end{tikzcd}$$ by taking homotopy fixed points $(-)^{h\text{S}^1}$. Composing this cartesian square with the cartesian square defining topological cyclic homology then induces a natural cartesian square $$\begin{tikzcd} \text{TC}(X) \ar[r] \ar[d] & \text{HC}^-(X) \ar[d] \\ \prod_{p \in \mathbb{P}} \text{TC}(X;\Z_p) \ar[r] & \prod_{p \in \mathbb{P}} \text{HC}^-(X;\Z_p). \end{tikzcd}$$ We will use this cartesian square to define the motivic filtration on $\text{TC}(X)$ (Definition~\ref{definitionmotivicfiltrationonTC}), by glueing existing filtrations on the three other terms; namely, the HKR and BMS filtrations. \subsection{The HKR filtrations}\label{subsectionHKRfiltrations} \vspace{-\parindent} \hspace{\parindent} In this subsection, we review the HKR filtrations on Hochschild homology and its variants, as defined, in the generality of qcqs derived schemes, by \cite{antieau_periodic_2019} and \cite[Section~$6.3$]{bhatt_absolute_2022}. Only the HKR filtration on negative cyclic homology $\text{HC}^-(-)$ (Definition~\ref{definitionHKRfiltrationonHC-}) will be used to define the motivic filtration on topological cyclic homology $\text{TC}(-)$ (Definition~\ref{definitionmotivicfiltrationonTC}). We will use the other HKR filtrations of this section in Section~\ref{sectionratonialstructure}. The following result is \cite[Example~$6.1.3$ and Remarks~$6.1.4$ and $6.1.5$]{bhatt_absolute_2022}. \begin{proposition}[Tate filtration]\label{propositionTatefiltration} Let $X$ be a spectrum equipped with an $\emph{S}^1$-action. Then the Tate construction $X^{t\emph{S}^1} \in \emph{Sp}$ is naturally equipped with a $\Z$-indexed filtration $$\emph{Fil}^\star_{\emph{T}} X^{t\emph{S}^1} \in \emph{FilSp}.$$ This filtration is called the \emph{Tate filtration} on $X^{t\emph{S}^1}$, and satisfies the following properties: \begin{enumerate} \item The filtration $\emph{Fil}^\star_{\emph{T}} X^{t\emph{S}^1} \in \emph{FilSp}$ is complete. \item $\emph{Fil}^0_{\emph{T}} X^{t\emph{S}^1}$ is the homotopy fixed point spectrum $X^{h\emph{S}^1}$, which is thus also equipped with an $\N$\nobreakdash-indexed complete filtration $\emph{Fil}^\star_{\emph{T}} X^{h\emph{S}^1}$, which we call the {\it Tate filtration} on $X^{h\emph{S}^1}$. \item For every integer $n \in \Z$, the graded piece $\emph{gr}^n_{\emph{T}} X^{t\emph{S}^1}$ is naturally identified with the spectrum~$X[-2n]$. \end{enumerate} \end{proposition} Following \cite[Section~$5$]{bhatt_topological_2019}, a filtered spectrum $\text{Fil}^\star X$ is called {\it connective for the Beilinson $t$\nobreakdash-structure} if for every integer $i \in \Z$, the graded piece $\text{gr}^i X \in \text{Sp}$ is in cohomological degrees at most~$i$. For every integer $i \in \Z$, also denote by $\tau_{\geq i}^{\text{B}}$ the truncation functor for the Beilinson $t$-structure on filtered spectra. \begin{definition}[Décalage filtration]\label{definitiondécalagefiltration} Let $\text{Fil}^\star X \in \text{FilSp}$ be a filtered spectrum. The {\it décalage filtration} on $\text{Fil}^\star X$ is the bifiltered spectrum $$\text{Fil}^\star_{\text{B}} \text{Fil}^\star X \in \text{biFilSp}$$ where, for every integer $i \in \Z$, $\text{Fil}^i_{\text{B}} \text{Fil}^\star X$ is the $i$-connective cover of $\text{Fil}^\star X \in \text{FilSp}$ with respect to the Beilinson $t$-structure on the category of filtered spectra: $$\text{Fil}^i_{\text{B}} \text{Fil}^\star X := \tau^{\text{B}}_{\geq i} \text{Fil}^\star X.$$ \end{definition} \begin{construction}[HKR filtration on HP]\label{constructionHKRfiltrationonHP} For every integer $i \in \Z$, let $$\text{Fil}^i_{\text{HKR}} \text{Fil}^\star_{\text{T}}\text{HP}(-) := L_{\text{Zar}} L_{\text{dSch}^{\text{qcqs,op}}/\text{Poly}_{\Z}^{\text{op}}} \text{Fil}^i_{\text{B}} \text{Fil}^\star_{\text{T}} \text{HP}(-),$$ where $\text{Fil}^\star_{\text{T}} \text{HP}(-)$ is the Tate filtration on periodic cyclic homology of qcqs derived schemes, $\text{Fil}^\star_{\text{B}}$ is the décalage filtration of Definition~\ref{definitiondécalagefiltration}, and the left Kan extension $L_{\text{dSch}^{\text{qcqs,op}}/\text{Poly}_{\Z}^{\text{op}}}$ is taken in the category of filtration-complete filtered spectra. The {\it HKR filtration on periodic cyclic homology} of qcqs derived schemes is the functor $$\text{Fil}^\star_{\text{HKR}} \text{HP}(-) : \text{dSch}^{\text{qcqs,op}} \longrightarrow \text{FilSp}$$ defined as the underlying filtered object of the bifiltered functor $\text{Fil}^\star_{\text{HKR}} \text{Fil}^\star_{\text{T}} \text{HP}(-)$: $$\text{Fil}^\star_{\text{HKR}} \text{HP}(-) := {\lim\limits_{\text{ }\longrightarrow n}} \text{Fil}^\star_{\text{HKR}} \text{Fil}^n_{\text{T}} \text{HP}(-).$$ \end{construction} The following definition is the one which will appear explicitly in the definition of the motivic filtration on topological cyclic homology (Definition~\ref{definitionmotivicfiltrationonTC}). \begin{definition}[HKR filtration on $\text{HC}^-$]\label{definitionHKRfiltrationonHC-} The {\it HKR filtration on negative cyclic homology} of qcqs derived schemes is the functor $$\text{Fil}^\star_{\text{HKR}} \text{HC}^-(-) : \text{dSch}^{\text{qcqs,op}} \longrightarrow \text{FilSp}$$ defined as $$\text{Fil}^\star_{\text{HKR}} \text{HC}^-(-) := \text{Fil}^\star_{\text{HKR}} \text{Fil}^0_{\text{T}} \text{HP}(-).$$ \end{definition} The following definition is motivated by Proposition~\ref{propositionTatefiltration}$\,(3)$. \begin{definition}[HKR filtration on HH]\label{definitionHKRfiltrationonHH} The {\it HKR filtration on Hochschild homology} of qcqs derived schemes is the functor $$\text{Fil}^\star_{\text{HKR}} \text{HH}(-) : \text{dSch}^{\text{qcqs,op}} \longrightarrow \text{FilSp}$$ defined as $$\text{Fil}^\star_{\text{HKR}} \text{HH}(-) := \text{Fil}^\star_{\text{HKR}} \text{gr}^0_{\text{T}} \text{HP}(-).$$ \end{definition} Cyclic homology $\text{HC}(-)$ is defined as the homotopy orbits $\text{HH}(-)_{h\text{S}^1}$ of the $\text{S}^1$-action on Hochschild homology $\text{HH}(-)$, and is related to negative cyclic homology $\text{HC}^-(-)$ and periodic cyclic homology $\text{HP}(-)$ by a natural fibre sequence $$\text{HC}^-(-) \longrightarrow \text{HP}(-) \longrightarrow \text{HC}(-)[2].$$ \begin{definition}[HKR filtration on HC]\label{definitionHKRfiltrationonHC} The {\it HKR filtration on cyclic homology} of qcqs derived schemes is the functor $$\text{Fil}^\star_{\text{HKR}} \text{HC}(-) : \text{dSch}^{\text{qcqs,op}} \longrightarrow \text{FilSp}$$ defined, for every integer $i \in \Z$, by $$\text{Fil}^i_{\text{HKR}} \text{HC}(-) := \text{cofib}\Big(\text{Fil}^{i+1}_{\text{HKR}} \text{HC}^-(-) \longrightarrow \text{Fil}^{i+1}_{\text{HKR}} \text{HP}(-)\Big)[-2],$$ where the map on the right hand side is induced by Construction~\ref{constructionHKRfiltrationonHP} and Definition~\ref{definitionHKRfiltrationonHC-}. \end{definition} \begin{remark}[Graded pieces of the HKR filtrations]\label{remarkgradedpiecesoftheHKRfiltrations} Let $X$ be a qcqs derived scheme. The main result of \cite{antieau_periodic_2019} describes the graded pieces of the HKR filtrations on $\text{HC}^-(X)$, $\text{HP}(X)$, and $\text{HC}(X)$ in terms of the Hodge-completed derived de Rham cohomology of $X$. In particular, Definition~\ref{definitionHKRfiltrationonHC} provides a filtered refinement of the fibre sequence $$\text{HC}^-(-) \longrightarrow \text{HP}(-) \longrightarrow \text{HC}(-)[2],$$ which induces on graded pieces, for every integer~$i \in \Z$, a natural fibre sequence $$R\Gamma_{\text{Zar}}\big(X,\widehat{\mathbb{L}\Omega}^{\geq i}_{-/\Z}\big)[2i] \longrightarrow R\Gamma_{\text{Zar}}\big(X,\widehat{\mathbb{L}\Omega}_{-/\Z}\big)[2i] \longrightarrow R\Gamma_{\text{Zar}}\big(X,\mathbb{L}\Omega^{<i}_{-/\Z}\big)[2i]$$ in the derived category $\mathcal{D}(\Z)$. \end{remark} \begin{proposition}[\cite{antieau_periodic_2019,bhatt_absolute_2022}]\label{propositionHKRfiltrationonHCrationalisfinitary} For every integer $i \in \Z$, the functor $\emph{Fil}^i_{\emph{HKR}} \emph{HC}(-)$, from animated commutative rings to spectra, is left Kan extended from polynomial $\Z$-algebras, commutes with filtered colimits, and its values are in cohomological degrees at most $-i$. \end{proposition} \begin{proof} On animated commutative rings, the Tate filtrations $$\text{Fil}^{i+1}_{\text{HKR}} \text{Fil}^\star_{\text{T}} \text{HC}^-(-) \quad \text{and} \quad \text{Fil}^{i+1}_{\text{HKR}} \text{Fil}^\star_{\text{T}} \text{HP}(-)$$ are by definition left Kan extended, as complete filtered objects, from polynomial $\Z$-algebras, thus so is the Tate filtration $\text{Fil}^i_{\text{HKR}} \text{Fil}^\star_{\text{T}} \text{HC}(-)$. The Tate filtration $\text{Fil}^i_{\text{HKR}} \text{Fil}^\star_{\text{T}} \text{HC}(-)$ is also finite by construction, so the functor $\text{Fil}^i_{\text{HKR}} \text{HC}(-)$ is left Kan extended on animated commutative rings from polynomial $\Z$-algebras. In particular, the functor $$\text{Fil}^i_{\text{HKR}} \text{HC}(-)$$ commutes with filtered colimits of animated commutative rings. For every integer $j \in \Z$, the $j^{\text{th}}$ graded piece of the filtration $\text{Fil}^\star_{\text{HKR}} \text{HC}(-)$ is naturally identified with the functor $R\Gamma_{\text{Zar}}\big(-,\mathbb{L}\Omega^{\leq j}_{-/\Z}\big)[2j]$ (\cite{antieau_periodic_2019}), whose values are in degrees at most $-j$ on animated commutative rings. The filtration $\text{Fil}^\star_{\text{HKR}} \text{HC}(-)$ is moreover complete on animated commutative rings (\cite[Remark~$6.3.5$]{bhatt_absolute_2022}), hence the desired connectivity result. \end{proof} \begin{lemma}[Completeness of the HKR filtrations, after \cite{bhatt_absolute_2022}]\label{lemmaHKRfiltrationonHC-isalwayscomplete} Let $X$ be a qcqs derived scheme. Then the HKR filtrations $$\emph{Fil}^\star_{\emph{HKR}} \emph{HP}(X), \text{ }\emph{Fil}^\star_{\emph{HKR}} \emph{HC}^-(X), \text{ } \emph{Fil}^\star_{\emph{HKR}} \emph{HH}(X), \text{ and } \emph{Fil}^\star_{\emph{HKR}} \emph{HC}(X)$$ are complete. \end{lemma} \begin{proof} The result for $\text{HC}^-$ and $\text{HH}$ is a direct consequence of \cite[Remark~$6.3.5$]{bhatt_absolute_2022}. The result for $\text{HC}$ is a consequence of the connectivity result of Proposition~\ref{propositionHKRfiltrationonHCrationalisfinitary}. By Definition~\ref{definitionHKRfiltrationonHC}, the result for $\text{HP}$ is then a consequence of the result for $\text{HC}^-$ and $\text{HC}$. \end{proof} \begin{remark}[Variant over $\Q$]\label{remarkHHandvariantsrelativetoQ} Let $X$ be a qcqs derived scheme. By the base change property for Hochschild homology, the natural map $$\text{HH}(X) \otimes_{\Z} \Q \longrightarrow \text{HH}(X_{\Q}/\Q)$$ is an equivalence in the derived category $\mathcal{D}(\Q)$, where $\text{HH}(-/\Q)$ is Hochschild homology relative to~$\Q$. Applying the functors $(-)^{h\text{S}^1}$, $(-)^{t\text{S}^1}$, and $(-)_{h\text{S}^1}$ to this Hochschild homology relative to $\Q$ induces relative variants $\text{HC}^-(-_{\Q}/\Q)$ of negative cyclic homology, $\text{HP}(-_{\Q}/\Q)$ of periodic cyclic homology, and $\text{HC}(-_{\Q}/\Q)$ of cylic homology. One can then define similar HKR filtrations $$\text{Fil}^\star_{\text{HKR}} \text{HH}(X_{\Q}/\Q), \text{ } \text{Fil}^\star_{\text{HKR}} \text{HC}^-(X_{\Q}/\Q), \text{ } \text{Fil}^\star_{\text{HKR}} \text{HP}(-_{\Q}/\Q), \text{ and } \text{Fil}^\star_{\text{HKR}} \text{HC}(-_{\Q}/\Q),$$ on these functors, whose graded pieces are versions of derived de Rham cohomology relative to $\Q$. \end{remark} To introduce and study the motivic filtration on topological cyclic homology (Definition~\ref{definitionmotivicfiltrationonTC}), we will need some $p$-complete variants of the previous HKR filtrations. \begin{definition}[HKR filtration on $\text{HC}^-(-;\Z_p)$]\label{definitionHKRfiltrationHC-pcompleted} Let $p$ be a prime number. The {\it HKR filtration on $p$-completed negative cyclic homology} of qcqs derived schemes is the functor $$\text{Fil}^\star_{\text{HKR}} \text{HC}^-(-;\Z_p) : \text{dSch}^{\text{qcqs,op}} \longrightarrow \text{FilSp}$$ defined as $$\text{Fil}^\star_{\text{HKR}} \text{HC}^-(-;\Z_p) := \big( \text{Fil}^\star_{\text{HKR}} \text{HC}^-(-) \big)^\wedge_p.$$ \end{definition} \begin{remark} The HKR filtrations on $\text{HP}(-;\Z_p)$, $\text{HC}(-;\Z_p)$, and $\text{HH}(-;\Z_p)$ of qcqs derived schemes are defined as in Definition~\ref{definitionHKRfiltrationHC-pcompleted}, where $\text{HC}^-(-;\Z_p)$ is replaced by $\text{HP}(-)$, $\text{HC}(-)$, or $\text{HH}(-)$. In particular, for every qcqs derived scheme $X$, Definition~\ref{definitionHKRfiltrationonHC} induces a fibre sequence of filtered spectra $$\text{Fil}^\star_{\text{HKR}} \text{HC}^-(X;\Z_p) \longrightarrow \text{Fil}^\star_{\text{HKR}} \text{HP}(X;\Z_p) \longrightarrow \text{Fil}^{\star-1}_{\text{HKR}} \text{HC}(X;\Z_p)[2].$$ \end{remark} \begin{lemma}\label{lemmaHKRfiltrationproductpcompletionsiscomplete} Let $X$ be a qcqs derived scheme. Then the filtrations $$\prod_{p \in \mathbb{P}} \emph{Fil}^\star_{\emph{HKR}} \emph{HP}(X;\Z_p), \quad \prod_{p \in \mathbb{P}} \emph{Fil}^\star_{\emph{HKR}} \emph{HC}^-(X;\Z_p), \quad \prod_{p \in \mathbb{P}} \emph{Fil}^\star_{\emph{HKR}} \emph{HH}(X;\Z_p),$$ $$\emph{ and } \prod_{p \in \mathbb{P}} \emph{Fil}^\star_{\emph{HKR}} \emph{HC}(X;\Z_p)$$ are complete. \end{lemma} \begin{proof} The collection of complete filtered spectra is closed under limits in the category of filtered spectra, so this is a consequence of Lemma~\ref{lemmaHKRfiltrationonHC-isalwayscomplete}. \end{proof} \begin{remark}[Exhaustivity of the HKR filtrations]\label{remarkexhaustivityoftheHKRfiltrations} The HKR filtrations $\text{Fil}^\star_{\text{HKR}} \text{HC}^-$ and $\text{Fil}^\star_{\text{HKR}} \text{HP}$ are not exhaustive on general qcqs derived schemes (\cite[Remark~$6.3.6$]{bhatt_absolute_2022}). For the purpose of the motivic filtration on algebraic $K$-theory (Definition~\ref{definitionmotivicfiltrationonKtheoryofschemes}), we will however only need the fact that the HKR filtration $\text{Fil}^\star_{\text{HKR}} \text{HC}$ is always $\N$-indexed, and in particular exhaustive. \end{remark} \subsection{The BMS filtrations}\label{subsectionBMSfiltrations} \vspace{-\parindent} \hspace{\parindent} Let $p$ be a prime number. In this subsection, we review the BMS filtrations on $p$-completed topological Hochschild homology $\text{THH}(-;\Z_p)$ and its variants, as defined in \cite{bhatt_topological_2019} for $p$\nobreakdash-complete $p$-quasisyntomic rings, and generalised in \cite{antieau_beilinson_2020} to $p$-complete rings and in \cite[Section~$6.2$]{bhatt_absolute_2022} to animated commutative rings. Only the BMS filtration on $p$-completed topological cyclic homology $\text{TC}(-;\Z_p)$ (Definition~\ref{definitionBMSfiltrationonTCpcompletedofqcqsderivedschemes}) will appear in the definition of the motivic filtration on topological cyclic homology $\text{TC}(-)$ (Definition~\ref{definitionmotivicfiltrationonTC}). The other BMS filtrations are necessary to construct the BMS filtration on $p$-completed topological cyclic homology $\text{TC}(-;\Z_p)$. \begin{construction}[BMS filtration on $\text{Fil}^\star_{\text{T}} \text{TP}(-;\Z_p)$]\label{constructionBMSfiltrationonTPpcompleted} Topological Hochschild homology $\text{THH}(-)$ of qcqs derived schemes admits a natural $\text{S}^1$-action, inducing a natural Tate filtration $\text{Fil}^\star_{\text{T}} \text{TP}(-)$ on topological periodic cyclic homology $\text{TP}(-) := \text{THH}(-)^{t\text{S}^1}$ (Proposition~\ref{propositionTatefiltration}). The Tate filtration $$\text{Fil}^\star_{\text{T}} \text{TP}(-;\Z_p) : \text{dSch}^{\text{qcqs,op}} \longrightarrow \text{FilSp}$$ is then defined as the $p$-completion of the Tate filtration $\text{Fil}^\star_{\text{T}} \text{TP}(-)$. For every quasiregular semiperfectoid ring $R$ and every integer $i \in \Z$, define the filtered spectrum $$\text{Fil}^i_{\text{BMS}} \text{Fil}^\star_{\text{T}} \text{TP}(R;\Z_p) := \tau_{\geq 2i} \text{Fil}^\star_{\text{T}} \text{TP}(R;\Z_p).$$ The filtered object $$\text{Fil}^i_{\text{BMS}} \text{Fil}^\star_{\text{T}} \text{TP}(-;\Z_p) : \text{dSch}^{\text{qcqs,op}} \longrightarrow \text{FilSp}$$ is then first defined on $p$-quasisyntomic rings as the unique such functor satisfying $p$-complete faithfully flat descent (the existence and unicity of such a functor is \cite[Proposition~$4.31$]{bhatt_topological_2019}). In general, polynomial $\Z$-algebras are $p$-quasisyntomic rings, and this filtered object is defined as the Zariski sheafification of its left Kan extension from polynomial $\Z$-algebras $$\text{Fil}^i_{\text{BMS}} \text{Fil}^\star_{\text{T}} \text{TP}(-;\Z_p) := L_{\text{Zar}} L_{\text{AniRings}/\text{Poly}_{\Z}} \text{Fil}^i_{\text{BMS}} \text{Fil}^\star_{\text{T}} \text{TP}(-;\Z_p),$$ where the left Kan extension is taken in the category of $p$-complete filtration-complete spectra. By \cite[Theorem~$6.2.4$]{bhatt_absolute_2022}, the resulting functor is still given by the double-speed Postnikov filtration on quasiregular semiperfectoid rings and, as a functor from animated commutative rings to $p$-complete filtration-complete spectra, commutes with sifted colimits and satisfies $p$-complete faithfully flat descent. \end{construction} \begin{remark}\label{remarkderivedpcompletionwithBMS} The BMS filtrations were first defined in \cite{bhatt_topological_2019} in the generality of $p$\nobreakdash-com\-plete $p$\nobreakdash-quasisyntomic rings. On general animated commutative rings $R$, the BMS filtrations, by construction, depend only on the $p$-completion of $R$ --and in particular vanish on animated commutative $\Z[\tfrac{1}{p}]$-algebras. Here the $p$-completion is the derived $p$-completion, even on classical commutative rings. On commutative rings with bounded $p$-power torsion ({\it e.g.}, on $p$-quasisyntomic rings), the derived and classical $p$-completions naturally coincide, and there is no conflict between the two definitions. \end{remark} \begin{definition}[BMS filtration on $\text{TP}(-;\Z_p)$]\label{definitionBMSfiltrationonTPpcompletedofqcqsderivedschemes} The {\it BMS filtration on $p$-completed topological periodic cyclic homology} of qcqs derived schemes is the functor $$\text{Fil}^\star_{\text{BMS}} \text{TP}(-;\Z_p) : \text{dSch}^{\text{qcqs,op}} \longrightarrow \text{FilSp}$$ defined as the underlying filtered object of the bifiltered functor $\text{Fil}^\star_{\text{BMS}} \text{Fil}^\star_{\text{T}} \text{TP}(-;\Z_p)$ of Construction~\ref{constructionBMSfiltrationonTPpcompleted}: $$\text{Fil}^\star_{\text{BMS}} \text{TP}(-;\Z_p) := {\lim\limits_{\text{ }\longrightarrow n}} \text{Fil}^\star_{\text{BMS}} \text{Fil}^n_{\text{T}} \text{TP}(-;\Z_p).$$ \end{definition} Topological negative cyclic homology is to topological periodic cyclic homology what negative cyclic homology is to periodic cyclic homology. Given Definition~\ref{definitionBMSfiltrationonTPpcompletedofqcqsderivedschemes}, the following definition then mimics Definition~\ref{definitionHKRfiltrationonHC-}. \begin{definition}[BMS filtration on $\text{TC}^-(-;\Z_p)$]\label{definitionBMSfiltrationonTC-pcompleted} The {\it BMS filtration on $p$-completed topological negative cyclic homology} of qcqs derived schemes is the functor $$\text{Fil}^\star_{\text{BMS}} \text{TC}^-(-;\Z_p) : \text{dSch}^{\text{qcqs,op}} \longrightarrow \text{FilSp}$$ defined as $$\text{Fil}^\star_{\text{BMS}} \text{TC}^-(-;\Z_p) := \text{Fil}^\star_{\text{BMS}} \text{Fil}^0_{\text{T}} \text{TP}(-;\Z_p).$$ \end{definition} Similarly, topological Hochschild homology is to topological periodic and topological negative cyclic homologies what Hochschild homology is to periodic and negative cyclic homologies, and the following definition mimics Definition~\ref{definitionHKRfiltrationonHH}. \begin{definition}[BMS filtration on $\text{THH}(-;\Z_p)$]\label{definitionBMSfiltrationonTHHpcompleted} The {\it BMS filtration on $p$-completed topological Hochschild homology} of qcqs derived schemes is the functor $$\text{Fil}^\star_{\text{BMS}} \text{THH}(-;\Z_p) : \text{dSch}^{\text{qcqs,op}} \longrightarrow \text{FilSp}$$ defined as $$\text{Fil}^\star_{\text{BMS}} \text{THH}(-;\Z_p) := \text{Fil}^\star_{\text{BMS}} \text{gr}^0_{\text{T}} \text{TP}(-;\Z_p).$$ \end{definition} Topological cyclic homology is however not to topological periodic cyclic homology what cyclic homology is to periodic cyclic homology. Following \cite{nikolaus_topological_2018}, it is rather defined, after $p$-completion, by a fibre sequence $$\text{TC}(-;\Z_p) \longrightarrow \text{TC}^-(-;\Z_p) \xlongrightarrow{\phi_p - \text{can}} \text{TP}(-;\Z_p).$$ Unwinding the previous definitions, the map $\phi_p - \text{can} : \text{TC}^-(-;\Z_p) \rightarrow \text{TP}(-;\Z_p)$ admits a unique refinement as a filtered map $$\phi_p - \text{can} : \text{Fil}^\star_{\text{BMS}} \text{TC}^-(-;\Z_p) \longrightarrow \text{Fil}^\star_{\text{BMS}} \text{TP}(-;\Z_p).$$ \begin{definition}[BMS filtration on $\text{TC}(-;\Z_p)$]\label{definitionBMSfiltrationonTCpcompletedofqcqsderivedschemes} The {\it BMS filtration on $p$-completed topological cyclic homology} of qcqs derived schemes is the functor $$\text{Fil}^\star_{\text{BMS}} \text{TC}(-;\Z_p) : \text{dSch}^{\text{qcqs,op}} \longrightarrow \text{FilSp}$$ defined as $$\text{Fil}^\star_{\text{BMS}} \text{TC}(-;\Z_p) := \text{fib}\Big(\phi_p - \text{can} : \text{Fil}^\star_{\text{BMS}} \text{TC}^-(-;\Z_p) \longrightarrow \text{Fil}^\star_{\text{BMS}} \text{TP}(-;\Z_p)\Big).$$ \end{definition} The BMS filtration on $p$-completed topological cyclic homology is always complete, as a consequence of a connectivity result of \cite{antieau_beilinson_2020}. We will need the following slightly more precise result when studying the completeness of the motivic filtration on algebraic $K$-theory. \begin{lemma}\label{lemmaBMSfiltrationproductallprimesiscomplete} Let $X$ be a qcqs derived scheme. Then the filtrations $\prod_{p \in \mathbb{P}} \emph{Fil}^\star_{\emph{BMS}} \emph{TC}(X;\Z_p)$ and $\big(\prod_{p \in \mathbb{P}} \emph{Fil}^\star_{\emph{BMS}} \emph{TC}(X;\Z_p)\big)_{\Q}$ are complete. More precisely, for every integer $i \in \Z$, the values of the presheaves $\prod_{p \in \mathbb{P}} \emph{Fil}^i_{\emph{BMS}} \emph{TC}(-;\Z_p)$ and $\big(\prod_{p \in \mathbb{P}} \emph{Fil}^i_{\emph{BMS}} \emph{TC}(-;\Z_p)\big)_{\Q}$ are in cohomological degrees at most $-i+1$ on affine derived schemes. \end{lemma} \begin{proof} The presheaves $$\prod_{p \in \mathbb{P}} \text{Fil}^\star_{\text{BMS}} \text{TC}(-;\Z_p) \quad \text{and} \quad \Big(\prod_{p \in \mathbb{P}} \text{Fil}^\star_{\text{BMS}} \text{TC}(-;\Z_p)\Big)_{\Q}$$ are Zariski sheaves by construction, so it suffices to prove the result for affine derived schemes $X$. Let $R$ be an animated commutative ring, and $i \in \Z$ be an integer. The spectrum $\text{Fil}^i_{\text{BMS}} \text{TC}(R;\Z_p)$ is in cohomological degrees at most $-i+1$ for every prime number~$p$ (\cite[Theorem~$5.1$]{antieau_beilinson_2020}). Taking the product over all primes $p$ and rationalisation, this implies that the spectra $\prod_{p \in \mathbb{P}} \text{Fil}^i_{\text{BMS}} \text{TC}(R;\Z_p)$ and $\Big(\prod_{p \in \mathbb{P}} \text{Fil}^i_{\text{BMS}} \text{TC}(R;\Z_p)\Big)_{\Q}$ are also in cohomological degrees at most $-i+1$, which implies that the associated filtrations are complete. \end{proof} \begin{remark}[Exhaustivity of the BMS filtrations]\label{remarkexhaustivityofBMSfiltrations} The BMS filtrations $\text{Fil}^\star_{\text{BMS}} \text{TP}(-;\Z_p)$ and $\text{Fil}^{\star}_{\text{BMS}} \text{TC}^-(-;\Z_p)$ are not exhaustive on general qcqs derived schemes (\cite[Warning~$6.2.7$]{bhatt_absolute_2022}). For the purpose of the motivic filtration on algebraic $K$-theory (Definition~\ref{definitionmotivicfiltrationonKtheoryofschemes}), we will however only need the fact that the BMS filtration $\text{Fil}^{\star}_{\text{BMS}} \text{TC}(-;\Z_p)$ is always $\N$\nobreakdash-indexed (\cite[proof of Proposition~$7.16$]{bhatt_topological_2019}), and in particular exhaustive. \end{remark} We refer to \cite{bhatt_topological_2019,bhatt_prisms_2022,bhatt_absolute_2022} for the relation between prismatic cohomology and the graded pieces of the BMS filtrations on $\text{TP}(-;\Z_p)$, $\text{TC}^-(-;\Z_p)$, and $\text{THH}(-;\Z_p)$. We only define here the shifted graded pieces of the BMS filtration on $\text{TC}(-;\Z_p)$, which are a version of syntomic cohomology (see Remark~\ref{remarksyntomiccohomologyBMSandBL}), and which will serve as a building block for the $p$-adic motivic complexes (Corollary~\ref{corollarymainpadicstructureongradeds}). \begin{definition}[BMS syntomic cohomology]\label{definitionsyntomiccohomologyintermsofTC} For every integer $i \in \Z$, the {\it syntomic complex} $$\Z_p(i)^{\text{BMS}}(-) : \text{dSch}^{\text{qcqs,op}} \longrightarrow \mathcal{D}(\Z)$$ is the functor defined as the shifted graded piece of the BMS filtration on $\text{TC}(-;\Z_p)$: $$\Z_p(i)^{\text{BMS}}(-) := \text{gr}^i_{\text{BMS}} \text{TC}(-;\Z_p)[-2i].$$ \end{definition} \begin{remark}\label{remarksyntomiccohomologyBMSandBL} Syntomic cohomology $\Z_p(i)^{\text{syn}}(X)$ of qcqs derived (formal) schemes $X$ is defined in \cite[Section~$8.4$]{bhatt_absolute_2022} (see also Notation~\ref{notationsyntomiccohomology}), in terms of the syntomic complexes of Definition~\ref{definitionsyntomiccohomologyintermsofTC} and of étale cohomology. From this perspective, the syntomic complexes $\Z_p(i)^{\text{BMS}}(X)$ of Definition~\ref{definitionsyntomiccohomologyintermsofTC} correspond to the syntomic cohomology $\Z_p(i)^{\text{syn}}(\mathfrak{X})$ of the derived $p$-adic formal scheme $\mathfrak{X}$ associated to $X$. \end{remark} \begin{theorem}\label{theoremBMSfiltrationonTCpcompletedsatisfiesquasisyntomicdescentandisLKEfrompolynomialalgebras} \begin{enumerate} \item (\cite{bhatt_topological_2019,bhatt_absolute_2022}) The functor $\emph{Fil}^\star_{\emph{BMS}} \emph{TC}(-;\Z_p)$, viewed as a functor from $p$\nobreakdash-quasisyntomic rings to $p$-complete filtered spectra, satisfies descent for the $p$\nobreakdash-quasisyntomic topology. \item (\cite{antieau_beilinson_2020,bhatt_absolute_2022}) The functor $\emph{Fil}^\star_{\emph{BMS}} \emph{TC}(-;\Z_p)$, viewed as a functor from animated commutative rings to $p$-complete filtered spectra, is left Kan extended from polynomial $\Z$-algebras. \end{enumerate} \end{theorem} \begin{proof} $(1)$ The filtration $\text{Fil}^\star_{\text{BMS}} \text{TC}(-;\Z_p)$ is complete on $p$-quasisyntomic rings (Lemma~\ref{lemmaBMSfiltrationproductallprimesiscomplete}), so it suffices to prove the result on graded pieces. The result on graded pieces is a special case of \cite[Proposition~$7.4.7$]{bhatt_absolute_2022}. $(2)$ By \cite[Theorem~$5.1\,(2)$]{antieau_beilinson_2020}, the functor $\text{Fil}^\star_{\text{BMS}} \text{TC}(-;\Z_p)$, viewed as a functor from $p$\nobreakdash-complete animated commutative rings to $p$-complete filtered spectra, is left Kan extended from $p$-complete polynomial $\Z$-algebras.\footnote{More precisely, it is proved to be left Kan extended from $p$\nobreakdash-complete polynomial $\Z$-algebras to $p$\nobreakdash-complete $p$\nobreakdash-quasisyntomic rings. By definition, $p$\nobreakdash-quasisyntomic rings have bounded $p$-power torsion. Hence, their derived and classical $p$-completions are naturally identified, and the left Kan extension to $p$-complete animated commutative rings agrees with the left Kan extension to $p$-complete classical rings on $p$-complete $p$-quasisyntomic rings.} Let $R$ be an animated commutative ring, and $R^\wedge_p$ be its (derived) $p$-completion. The natural map $$\text{Fil}^\star_{\text{BMS}} \text{TC}(R;\Z_p) \longrightarrow \text{Fil}^\star_{\text{BMS}} \text{TC}(R^\wedge_p;\Z_p)$$ is an equivalence of filtered spectra. Indeed, the filtrations $$\text{Fil}^\star_{\text{BMS}} \text{TC}(R;\Z_p) \quad \text{and} \quad \text{Fil}^\star_{\text{BMS}} \text{TC}(R^\wedge_p;\Z_p)$$ are $\N$-indexed and complete (Lemma~\ref{lemmaBMSfiltrationproductallprimesiscomplete} and Remark~\ref{remarkexhaustivityofBMSfiltrations}), so it suffices to prove the result on graded pieces, where this is a direct consequence of \cite[Corollary~$7.4.11$]{bhatt_absolute_2022}. This implies the desired left Kan extension property. \end{proof} \begin{corollary}\label{corollaryBMSsyntomiccohomologyhasquasisyntomicdescentandLKEfrompolynomialZalgebras} Let $i \in \Z$ be an integer. \begin{enumerate} \item The functor $\Z_p(i)^{\emph{BMS}}(-)$, viewed as a functor from $p$-quasisyntomic rings to $p$-complete objects in the derived category $\mathcal{D}(\Z)$, satisfies descent for the $p$\nobreakdash-quasisyntomic topology. \item The functor $\Z_p(i)^{\emph{BMS}}(-)$, viewed as a functor from animated commutative rings to $p$\nobreakdash-complete objects in the derived category $\mathcal{D}(\Z)$, is left Kan extended from polynomial $\Z$-algebras. \end{enumerate} \end{corollary} \begin{proof} $(1)$ was already part of the proof of Theorem~\ref{theoremBMSfiltrationonTCpcompletedsatisfiesquasisyntomicdescentandisLKEfrompolynomialalgebras}\,$(1)$. $(2)$ is a direct consequence of Theorem~\ref{theoremBMSfiltrationonTCpcompletedsatisfiesquasisyntomicdescentandisLKEfrompolynomialalgebras}\,$(2)$. \end{proof} \begin{theorem}[\cite{antieau_beilinson_2020}]\label{theoremAMMNrigidity} Let $(A,I)$ be a henselian pair of commutative rings. Then for any integers $i \geq 0$ and $k \geq 1$, the fibre of the natural map $$\Z/p^k(i)^{\emph{BMS}}(A) \longrightarrow \Z/p^k(i)^{\emph{BMS}}(A/I)$$ in the derived category $\mathcal{D}(\Z/p^k)$ is in degrees at most $i$. \end{theorem} \begin{proof} By \cite[Theorem~$5.2$]{antieau_beilinson_2020}, for every henselian pair $(A,I)$ such that the commutative rings $A$ and $A/I$ are (classically) $p$-complete, the fibre of the natural map $$\Z/p^k(i)^{\text{BMS}}(A) \longrightarrow \Z/p^k(i)^{\text{BMS}}(A/I)$$ is in degrees at most $i$. The proof of \cite[Theorem~$5.2$]{antieau_beilinson_2020} proves more generally that for $(A,I)$ a general henselian pair of commutative rings, the fibre of the natural map $$\Z/p^k(i)^{\text{BMS}}(A^\wedge_p) \longrightarrow \Z/p^k(i)^{\text{BMS}}((A/I)^\wedge_p)$$ where $(-)^\wedge_p$ is the derived $p$\nobreakdash-completion, is in degrees at most $i$. By \cite[Corollary~$7.4.11$]{bhatt_absolute_2022} (see also the proof of Theorem~\ref{theoremBMSfiltrationonTCpcompletedsatisfiesquasisyntomicdescentandisLKEfrompolynomialalgebras}\,$(2)$), the natural map $\Z/p^k(i)^{\text{BMS}}(-) \rightarrow \Z/p^k(i)^{\text{BMS}}((-)^\wedge_p)$ is an equivalence on animated commutative rings, hence for every henselian pair $(A,I)$ of commutative rings, the fibre of the natural map $$\Z/p^k(i)^{\text{BMS}}(A) \longrightarrow \Z/p^k(i)^{\text{BMS}}(A/I)$$ is in degrees at most $i$. \end{proof} \subsection{The motivic filtration on TC} \vspace{-\parindent} \hspace{\parindent} In this subsection, we introduce the motivic filtration on topological cyclic homology $\text{TC}(-)$ of general qcqs derived schemes (Definition~\ref{definitionmotivicfiltrationonTC}). The following proposition is \cite[Proposition~$6.4.1$]{bhatt_absolute_2022}. \begin{proposition}[\cite{bhatt_absolute_2022}]\label{propositionfilteredmapTC-pcompletedtoHC-pcompleted} Let $p$ be a prime number. The map $$\emph{Fil}^\star_{\emph{T}} \emph{TP}(-;\Z_p) \longrightarrow \emph{Fil}^\star_{\emph{T}} \emph{HP}(-;\Z_p),$$ viewed as a map of filtered spectra-valued presheaves on the category of qcqs derived schemes, admits a unique, multiplicative extension to a map of bifiltered presheaves of spectra $$\emph{Fil}^\star_{\emph{BMS}} \emph{Fil}^\star_{\emph{T}} \emph{TP}(-;\Z_p) \longrightarrow \emph{Fil}^\star_{\emph{HKR}} \emph{Fil}^\star_{\emph{T}} \emph{HP}(-;\Z_p).$$ \end{proposition} \begin{construction}[BMS-HKR comparison map]\label{constructionfilteredmapTCpcompletedtoHC-pcompleted} Let $p$ be a prime number. The {\it BMS-HKR comparison map} is the map $$\text{Fil}^\star_{\text{BMS}} \text{TC}(-;\Z_p) \longrightarrow \text{Fil}^\star_{\text{HKR}} \text{HC}^{-}(-;\Z_p)$$ of functors from (the opposite category of) qcqs derived schemes to the category of filtered spectra defined as the composite $$\text{Fil}^\star_{\text{BMS}} \text{TC}(-;\Z_p) \longrightarrow \text{Fil}^\star_{\text{BMS}} \text{TC}^{-}(-;\Z_p) \longrightarrow \text{Fil}^\star_{\text{HKR}} \text{HC}^{-}(-;\Z_p)$$ of the maps given by Definition~\ref{definitionBMSfiltrationonTCpcompletedofqcqsderivedschemes}, and Proposition~\ref{propositionfilteredmapTC-pcompletedtoHC-pcompleted} after restricting to the zeroth step of the Tate filtration. \end{construction} \begin{definition}[Motivic filtration on TC]\label{definitionmotivicfiltrationonTC} The {\it motivic filtration on topological cyclic homology} of qcqs derived schemes $$\text{Fil}^\star_{\text{mot}} \text{TC}(-) : \text{dSch}^{\text{qcqs},\text{op}} \longrightarrow \text{FilSp}$$ is the functor defined by the cartesian square $$\begin{tikzcd} \text{Fil}^\star_{\text{mot}} \text{TC}(-) \ar[r] \ar[d] & \text{Fil}^\star_{\text{HKR}} \text{HC}^{-}(-) \ar[d] \\ \prod_{p \in \mathbb{P}} \text{Fil}^\star_{\text{BMS}} \text{TC}(-;\Z_p) \ar[r] & \prod_{p \in \mathbb{P}} \text{Fil}^\star_{\text{HKR}} \text{HC}^{-}(-;\Z_p), \end{tikzcd}$$ where the bottom horizontal map is the map of Construction~\ref{constructionfilteredmapTCpcompletedtoHC-pcompleted}, and the right vertical map is profinite completion. For every integer $i \in \Z$, also define the functor $$\Z(i)^{\text{TC}}(-) : \text{dSch}^{\text{qcqs},\text{op}} \longrightarrow \mathcal{D}(\Z)\footnote{Every value of the functor $\Z(i)^{\text{TC}}(-)$ has a natural module structure over the $\mathbb{E}_{\infty}$-ring $\Z(0)^{\text{TC}}(\Z)$, which, by unwinding the definition, is naturally identified with the $\mathbb{E}_{\infty}$-ring $H\Z$. This implies that the spectra-valued functor $\Z(i)^{\text{TC}}(-)$ takes values in $H\Z$-linear spectra, {\it i.e.}, in the derived category $\mathcal{D}(\Z)$.}$$ as the shifted graded piece of this motivic filtration: $$\Z(i)^{\text{TC}}(-) := \text{gr}^i_{\text{mot}} \text{TC}(-)[-2i].$$ \end{definition} \begin{remark}[Comparison to \cite{elmanto_motivic_2023}]\label{remarkcomparisontoEMfiltrationonTCoverafield} For every qcqs derived scheme $X$ over $\Q$, the filtered spectrum $\text{Fil}^\star_{\text{HKR}} \text{HC}^-(X)$ is $\Q$-linear by construction, so its profinite completion vanishes. The filtration $\prod_{p \in \mathbb{P}} \text{Fil}^\star_{\text{BMS}} \text{TC}(X;\Z_p)$ also vanishes (Remark~\ref{remarkderivedpcompletionwithBMS}), and the natural map $$\text{Fil}^\star_{\text{mot}} \text{TC}(X) \longrightarrow \text{Fil}^\star_{\text{HKR}} \text{HC}^-(X/\Q)$$ is then an equivalence of filtered spectra. Similarly, for every prime number $p$ and every qcqs derived scheme $X$ over $\F_p$, the filtered spectrum $\text{Fil}^\star_{\text{HKR}} \text{HC}^-(X)$ is $\Z$-linear and $p$-complete, so it is naturally identified with its profinite completion. Again using Remark~\ref{remarkderivedpcompletionwithBMS}, the natural map $$\text{Fil}^\star_{\text{mot}} \text{TC}(X) \longrightarrow \text{Fil}^\star_{\text{BMS}} \text{TC}(X;\Z_p)$$ is then an equivalence of filtered spectra. \end{remark} \begin{remark}[Comparison to \cite{bhatt_absolute_2022}] In \cite[Section~$6.4$]{bhatt_absolute_2022}, Bhatt--Lurie define filtered spectra $\text{Fil}^\star_{\text{mot}} \text{TP}(X)$ and $\text{Fil}^\star_{\text{mot}} \text{TC}^-(X)$ for qcqs derived schemes $X$, with shifted graded pieces called the global prismatic complexes $\widehat{\Prism}^{\text{gl}}_X\{i\}$ and $\mathcal{N}^{\geq i} \widehat{\Prism}^{\text{gl}}_X\{i\}$ respectively. These filtrations can be used to obtain an alternative definition of the $\text{Fil}^\star_{\text{mot}} \text{TC}(X)$ of Definition~\ref{definitionmotivicfiltrationonTC}. More precisely, for every prime number $p$ the $p$-completion of Bhatt--Lurie's filtration $\text{Fil}^\star_{\text{mot}} \text{TP}(X)$ is the filtration $\text{Fil}^\star_{\text{BMS}} \text{TP}(X;\Z_p)$ of Definition~\ref{definitionBMSfiltrationonTPpcompletedofqcqsderivedschemes}, and there is a natural fibre sequence $$\text{Fil}^\star_{\text{mot}} \text{TC}(X) \longrightarrow \text{Fil}^\star_{\text{mot}} \text{TC}^-(X) \longrightarrow \prod_{p \in \mathbb{P}} \text{Fil}^\star_{\text{BMS}} \text{TP}(X;\Z_p)$$ of filtered spectra. In particular, for every integer $i \in \Z$, this induces a natural fibre sequence $$\Z(i)^{\text{TC}}(X) \longrightarrow \mathcal{N}^{\geq i} \widehat{\Prism}^{\text{gl}}_X\{i\} \longrightarrow \prod_{p \in \mathbb{P}} \widehat{\Prism}_{X,p}\{i\}$$ in the derived category $\mathcal{D}(\Z)$, where $\widehat{\Prism}_{X,p}$ denotes the $p$-adic absolute prismatic cohomology of $X$. \end{remark} \begin{proposition}\label{propositionpcompletionofmotiviconTCisBMS} Let $X$ be a qcqs derived scheme, and $p$ be a prime number. Then the natural map $$\emph{Fil}^\star_{\emph{mot}} \emph{TC}(X;\Z_p) \longrightarrow \emph{Fil}^\star_{\emph{BMS}} \emph{TC}(X;\Z_p)$$ is an equivalence of filtered spectra. \end{proposition} \begin{proof} By definition, the natural map $$\text{Fil}^\star_{\text{HKR}} \text{HC}^-(X) \longrightarrow \prod_{\l \in \mathbb{P}} \text{Fil}^\star_{\text{HKR}}\text{HC}^-(X;\Z_{\l})$$ in Definition~\ref{definitionmotivicfiltrationonTC} is profinite completion, so its fibre becomes zero after $p$-completion. \end{proof} \begin{corollary} Let $X$ be a qcqs derived scheme, and $p$ be a prime number. Then the natural map $$\Z_p(i)^{\emph{TC}}(X) \longrightarrow \Z_p(i)^{\emph{BMS}}(X)$$ is an equivalence in the derived category $\mathcal{D}(\Z_p)$. \end{corollary} \begin{proof} This is a direct consequence of Proposition~\ref{propositionpcompletionofmotiviconTCisBMS}. \end{proof} \begin{proposition}\label{propositionfilteredTCandrationalfilteredTCarecomplete} Let $X$ be a qcqs derived scheme. Then the filtrations $$\emph{Fil}^\star_{\emph{mot}} \emph{TC}(X) \text{ and } \emph{Fil}^\star_{\emph{mot}} \emph{TC}(X;\Q)$$ are complete. \end{proposition} \begin{proof} The filtrations $\text{Fil}^\star_{\text{HKR}} \text{HC}^-(X)$, $\prod_{p \in \mathbb{P}} \text{Fil}^\star_{\text{HKR}} \text{HC}^-(X;\Z_p)$, and $\prod_{p \in \mathbb{P}} \text{Fil}^\star_{\text{BMS}} \text{TC}(X;\Z_p)$ are complete by Lemmas~\ref{lemmaHKRfiltrationonHC-isalwayscomplete}, \ref{lemmaHKRfiltrationproductpcompletionsiscomplete}, and \ref{lemmaBMSfiltrationproductallprimesiscomplete} respectively. By Definition~\ref{definitionmotivicfiltrationonTC}, the filtration $\text{Fil}^\star_{\text{mot}}\text{TC}(X)$ is then complete, as a pullback of three complete filtrations. To prove that the filtration $\text{Fil}^\star_{\text{mot}}\text{TC}(X;\Q)$ is complete, consider the cartesian square of filtered spectra $$\begin{tikzcd} \text{Fil}^\star_{\text{mot}} \text{TC}(X;\Q) \ar[r] \ar[d] & \text{Fil}^\star_{\text{HKR}} \text{HC}^-(X;\Q) \ar[d] \\ \Big(\prod_{p \in \mathbb{P}} \text{Fil}^\star_{\text{BMS}} \text{TC}(X;\Z_p)\Big)_{\Q} \ar[r] & \Big(\prod_{p \in \mathbb{P}} \text{Fil}^\star_{\text{HKR}} \text{HC}^-(X;\Z_p)\Big)_{\Q} \end{tikzcd}$$ induced by taking the rationalisation of Definition~\ref{definitionmotivicfiltrationonTC}. The filtration $$\big(\prod_{p \in \mathbb{P}} \text{Fil}^\star_{\text{BMS}} \text{TC}(X;\Z_p)\big)_{\Q}$$ is complete by Lemma~\ref{lemmaBMSfiltrationproductallprimesiscomplete}. The fibre of the natural map $$\text{Fil}^\star_{\text{HKR}} \text{HC}^-(X) \longrightarrow \prod_{p \in \mathbb{P}} \text{Fil}^\star_{\text{HKR}} \text{HC}^-(X;\Z_p)$$ is complete as an object of the filtered derived category $\mathcal{DF}(\Z)$ (Lemmas~\ref{lemmaHKRfiltrationonHC-isalwayscomplete} and \ref{lemmaHKRfiltrationHC-solidproductallprimesiscomplete}), and is zero modulo $p$ for every prime number $p$ by construction. In particular, it is naturally identified with the fibre of the natural map $$\text{Fil}^\star_{\text{HKR}} \text{HC}^-(X;\Q) \longrightarrow \Big(\prod_{p \in \mathbb{P}} \text{Fil}^\star_{\text{HKR}} \text{HC}^-(X;\Z_p)\Big)_{\Q},$$ which is thus complete as an object of the filtered derived category $\mathcal{DF}(\Q)$. This implies, by the previous cartesian square, that the filtration $\text{Fil}^\star_{\text{mot}} \text{TC}(X;\Q)$ is also complete. \end{proof} \begin{remark}[Exhaustivity of the motivic filtration on TC]\label{remarkexhaustivitymotivicfiltrationonTC} The motivic filtration $\text{Fil}^\star_{\text{mot}} \text{TC}$ is not exhaustive on general qcqs derived scheme. Although this will not be necessary to prove that the motivic filtration on algebraic $K$-theory is exhaustive (Proposition~\ref{propositionmotivicfiltrationisexhaustive}), one can prove, using \cite[Lemma~$4.10$]{antieau_periodic_2019} and its proof, that if $X$ is a quasi-lci $\Z$-scheme,\footnote{By this, we mean that Zariski-locally on the qcqs scheme $X$, the cotangent complex $\mathbb{L}_{-/\Z}$ has Tor-amplitude in~$[-1;0]$.} then the motivic filtration $\text{Fil}^\star_{\text{mot}} \text{TC}(X)$ is exhaustive. \end{remark} \begin{proposition}\label{propositionétaledescentfiltrationonTC} For every integer $i \in \Z$, the presheaf $$\emph{Fil}^i_{\emph{mot}} \emph{TC}(-) : \emph{dSch}^{\emph{qcqs,op}} \longrightarrow \emph{Sp}$$ is an étale sheaf. \end{proposition} \begin{proof} By Definition~\ref{definitionmotivicfiltrationonTC}, it suffices to prove that the presheaves $$\prod_{p \in \mathbb{P}} \text{Fil}^i_{\text{BMS}} \text{TC}(-;\Z_p), \text{ } \text{Fil}^i_{\text{HKR}} \text{HC}^-(-), \text{ and } \prod_{p \in \mathbb{P}} \text{Fil}^i_{\text{HKR}} \text{HC}^-(-;\Z_p)$$ are étale sheaves. A product of sheaves is a sheaf, and these BMS and HKR filtrations are complete (Lemmas~\ref{lemmaBMSfiltrationproductallprimesiscomplete}, \ref{lemmaHKRfiltrationonHC-isalwayscomplete}, and~\ref{lemmaHKRfiltrationproductpcompletionsiscomplete}). It then suffices to prove that for every prime number $p$, the presheaves $$\Z_p(i)^{\text{BMS}}(-), \text{ } R\Gamma_{\text{Zar}}\big(-,\widehat{\mathbb{L}\Omega}^{\geq i}_{-/\Z}\big), \text{ and } R\Gamma_{\text{Zar}}\big(-,(\widehat{\mathbb{L}\Omega}^{\geq i}_{-/\Z})^\wedge_p\big)$$ are étale sheaves. The statement for $\Z_p(i)^{\text{BMS}}(-)$ is a consequence of $p$-fpqc descent (\cite[Proposition~$7.4.7$]{bhatt_absolute_2022}). The statement for the other two presheaves reduces to the fpqc descent for the powers of the cotangent complex (\cite[Theorem~$3.1$]{bhatt_topological_2019}). \end{proof} \begin{corollary}\label{corollaryétaledescentforZ(i)TC} For every integer $i \in \Z$, the presheaf $$\Z(i)^{\emph{TC}}(-) : \emph{dSch}^{\emph{qcqs,op}} \longrightarrow \mathcal{D}(\Z)$$ is an étale sheaf. \end{corollary} \begin{proof} This is a direct consequence of Proposition~\ref{propositionétaledescentfiltrationonTC}. \end{proof} \subsection{The motivic filtration on \texorpdfstring{$L_{\text{cdh}} \text{TC}$}{TEXT}} \vspace{-\parindent} \hspace{\parindent} In this subsection, we introduce the motivic filtration on the cdh sheafification of topological cyclic homology of general qcqs schemes (Definition~\ref{definitionmotivicfiltrationonLcdhTC}). \begin{definition}[Motivic filtration on $L_{\text{cdh}} \text{TC}$]\label{definitionmotivicfiltrationonLcdhTC} The {\it motivic filtration on cdh sheafified topological cyclic homology} of qcqs schemes $$\text{Fil}^\star_{\text{mot}} L_{\text{cdh}} \text{TC}(-) : \text{Sch}^{\text{qcqs},\text{op}} \longrightarrow \text{FilSp}$$ is the functor defined as the cdh sheafification of the motivic filtration on topological cyclic homology (Definition~\ref{definitionmotivicfiltrationonTC}) $$\text{Fil}^\star_{\text{mot}} L_{\text{cdh}} \text{TC}(-) := \big(L_{\text{cdh}} \text{Fil}^\star_{\text{mot}} \text{TC}\big)(-).$$ \end{definition} \begin{remark}[Graded pieces of $\text{Fil}^\star_{\text{mot}} L_{\text{cdh}} \text{TC}$] Let $X$ be a qcqs scheme. For every integer $i \in \Z$, the canonical map $$\big(L_{\text{cdh}} \Z(i)^{\text{TC}}\big)(X) \longrightarrow \text{gr}^i_{\text{mot}} L_{\text{cdh}} \text{TC}(R)[-2i]$$ is an equivalence in the derived category $\mathcal{D}(\Z)$. We will usually refer to these shifted graded pieces by the complexes $\big(L_{\text{cdh}} \Z(i)^{\text{TC}}\big)(X)$. \end{remark} \begin{remark}[Completeness of $\text{Fil}^\star_{\text{mot}} L_{\text{cdh}} \text{TC}$] It is not clear {\it a priori} that the filtered spectrum $\text{Fil}^\star_{\text{mot}} L_{\text{cdh}} \text{TC}(X)$ is complete, even for qcqs schemes of finite valuative dimension. Modulo any prime number $p$, this is a consequence of the connectivity bound \cite[Theorem~$5.1\,(1)$]{antieau_beilinson_2020} and \cite[Theorem~$2.4.15$]{elmanto_cdh_2021}. The integral statement will be a consequence of certain cdh descent results in Section~\ref{sectionratonialstructure}. \end{remark} \begin{remark}[Exhaustivity of $\text{Fil}^\star_{\text{mot}} L_{\text{cdh}} \text{TC}$] The filtration $\text{Fil}^\star_{\text{mot}} L_{\text{cdh}} \text{TC}$ is not exhaustive on general qcqs derived schemes. We will prove however, in Section~\ref{sectionratonialstructure}, that the fibre of the natural map $\text{Fil}^\star_{\text{mot}} \text{TC} \rightarrow \text{Fil}^\star_{\text{mot}} L_{\text{cdh}} \text{TC}$ is $\N$-indexed, and in particular exhaustive. \end{remark} \newpage \section{Definition of motivic cohomology} \vspace{-\parindent} \hspace{\parindent} In this section, we introduce motivic cohomology of general quasi-compact quasi-separated derived schemes (Definition~\ref{definitionmotiviccohomologyofderivedschemes}) and establish some of the fundamental properties of the associated motivic filtration. \subsection{Classical motivic cohomology} \vspace{-\parindent} \hspace{\parindent} In this subsection, we review the classical definition of motivic cohomology of smooth schemes in mixed characteristic. Following \cite{bloch_algebraic_1986,levine_techniques_2001,geisser_motivic_2004}, the motivic cohomology of smooth $\Z$-schemes $X$ is classically defined in terms of Bloch's cycle complexes $z^i(X,\bullet)$. Recall that Bloch's cycle complex is a simplicial abelian group defined in terms of algebraic cycles. The homotopy groups of Bloch's cycle complexes, called Bloch's higher Chow groups, are a generalisation of Chow groups that are designed to generalise the relation between the $\text{K}_0$ and the Chow groups of a quasi-projective variety to higher $K$-groups. Via the Dold--Kan correspondence, we view Bloch's cycle complexes as objects of the derived category $\mathcal{D}(\Z)$. \begin{definition}[Classical motivic cohomology of smooth schemes]\label{definitionclassicalmotiviccohomology} Let $B$ be a field or a mixed characteristic Dedekind domain ({\it e.g.}, $B=\Z$), and $X$ be a smooth $B$-scheme. For any integer $i \in \Z$, the {\it classical motivic complex} $$\Z(i)^{\text{cla}}(X) \in \mathcal{D}(\Z)$$ is the shift of Bloch's cycle complex $z^i(X,\bullet)$: $$\Z(i)^{\text{cla}}(X) := z^i(X,\bullet)[-2i],$$ where $\bullet$ is the cohomological index. \end{definition} Note that, by construction, the classical motivic complexes $\Z(i)^{\text{cla}}$ vanish in degrees more than $2i$, and in all degrees for weights $i<0$. In the following definition, we use the slice filtration in stable homotopy theory, as introduced by Voevodsky \cite{voevodsky_open_2022,voevodsky_possible_2002,bachmann_norms_2021}. \begin{definition}[Motivic filtration on $K$-theory of smooth schemes]\label{definitionclassicalmotivicfiltration} Let $B$ be a field or a mixed characteristic Dedekind domain. The {\it classical motivic filtration} on algebraic $K$-theory of smooth $B$-schemes is the functor $$\text{Fil}^\star_{\text{cla}} \text{K}(-) : \text{Sm}_{B}^{\text{op}} \longrightarrow \text{FilSp}$$ defined as the image, via the mapping spectrum construction $\omega^{\infty} : \text{SH}(B) \rightarrow \text{PSh}(\text{Sm}_{B}, \text{Sp})$, of the slice filtration $f^\star \text{KGL}$ on the $K$-theory motivic spectrum $\text{KGL} \in \text{SH}(B)$. \end{definition} \begin{remark}\label{remarkclassicalBlochrelatedtoslice} The pullback of algebraic cycles being well-defined only along flat maps, it is not straightforward to prove that the classical motivic complexes $\Z(i)^{\text{cla}}$ of Definition~\ref{definitionclassicalmotiviccohomology} are functorial. Over a field, Voevodsky overcomes this technicality by proving that Bloch's cycle complexes are represented in $\text{SH}$ by the zeroth slice of the $K$-theory motivic spectrum $\text{KGL}$. Over a mixed characteristic Dedekind domain, this identification is proved by Bachmann \cite{bachmann_very_2022}. In particular, this means that Bloch's cycle complexes $z^i(-,\bullet)$, when seen as a construction taking values in the derived category $\mathcal{D}(\Z)$, is indeed functorial, and multiplicative. In terms of Definitions~\ref{definitionclassicalmotiviccohomology} and~\ref{definitionclassicalmotivicfiltration}, this implies that for every integer $i \in \Z$, there is an equivalence of $\mathcal{D}(\Z)$-valued\footnote{Every value of the functor $\text{gr}^i_{\text{cla}} \text{K}(-)[-2i]$ has a natural module structure over the $\mathbb{E}_{\infty}$-ring $\text{gr}^0_{\text{cla}} \text{K}(\Z)$, which, by \cite[Proposition~$6.1$]{spitzweck_commutative_2018} and \cite{bachmann_very_2022}, is naturally identified with the $\mathbb{E}_{\infty}$-ring $H\Z$. This implies that the spectra-valued functor $\text{gr}^i_{\text{cla}}\text{K}(-)[-2i]$ takes values in $H\Z$-linear spectra, {\it i.e.}, in the derived category $\mathcal{D}(\Z)$.} functors $$\Z(i)^{\text{cla}}(-) := \text{gr}^i_{\text{cla}} \text{K}(-) [-2i].$$ \end{remark} \begin{example}[Weight zero classical motivic cohomology]\label{exampleweightzeroclassicalmotiviccohomology} For every smooth scheme $X$ over a field or a mixed characteristic Dedekind domain, there is a natural equivalence $$\Z(0)^{\text{cla}}(X) \simeq R\Gamma_{\text{Zar}}(X,\Z)$$ in the derived category $\mathcal{D}(\Z)$ (\cite[Proposition~$6.1$]{spitzweck_commutative_2018}). \end{example} \begin{example}[Weight one classical motivic cohomology]\label{exampleweight1classicalmotiviccohomology} For every smooth scheme $X$ over a field or a mixed characteristic Dedekind domain, there is a natural equivalence $$\Z(1)^{\text{cla}}(X) \simeq R\Gamma_{\text{Zar}}(X,\mathbb{G}_m)[-1]$$ in the derived category $\mathcal{D}(\Z)$ (\cite[Theorem~$7.10$]{spitzweck_commutative_2018}). In particular, the complex $\Z(1)^{\text{cla}}(X)$ is concentrated in degrees one and two, where it is given by $$\text{H}^1(\Z(1)^{\text{cla}}(X)) \cong \mathcal{O}(X)^{\times} \quad \text{ and } \quad \text{H}^2(\Z(1)^{\text{cla}}(X)) \cong \text{Pic}(X).$$ \end{example} \subsection{The cdh-local motivic filtration}\label{subsectioncdhlocalmotivicfiltration} \vspace{-\parindent} \hspace{\parindent} Following \cite{bachmann_A^1-invariant_2024}, we review in this subsection the cdh-local motivic filtration on homotopy $K$\nobreakdash-theory $\text{KH}(-)$ of qcqs schemes (Definition~\ref{definitionmotivicfiltrationonKHtheoryofschemes}), whose shifted graded pieces $\Z(i)^{\text{cdh}}$ will serve as a building block for the definition of the motivic complexes $\Z(i)^{\text{mot}}$ (Remark~\ref{remarkmaincartesiansquareformotiviccohomology}). We first define the lisse motivic complexes $\Z(i)^{\text{lisse}}$ as an intermediate construction, and as a practical tool for later sections. The following definition is motivated by the observation of Bhatt--Lurie that connective algebraic $K$-theory $\text{K}^{\text{conn}}(-)$ is left Kan extended on animated commutative rings from smooth $\Z$-algebras \cite[Proposition~A$.0.4$]{elmanto_modules_2020}. \begin{definition}[Motivic filtration on connective $K$-theory of animated rings]\label{definitionlissemotivicfiltrationconnectiveKtheory} The {\it motivic filtration on connective algebraic $K$-theory} of animated commutative rings is the functor $$\text{Fil}^\star_{\text{lisse}} \text{K}^{\text{conn}}(-) : \text{AniRings} \longrightarrow \text{FilSp}$$ defined as the left Kan extension of the classical motivic filtration on algebraic $K$-theory of smooth $\Z$-algebras $$\text{Fil}^\star_{\text{lisse}} \text{K}^{\text{conn}}(-) := \big(L_{\text{AniRings}/\text{Sm}_{\Z}} \text{Fil}^\star_{\text{cla}} \text{K}\big)(-).$$ \end{definition} Note that connective algebraic $K$-theory is not a Zariski sheaf on commutative rings. Most of our results on the following lisse motivic complexes $\Z(i)^{\text{lisse}}$ will be formulated in the generality of local rings. \begin{definition}[Lisse motivic cohomology of animated rings]\label{definitionlissemotiviccohomology} For any integer $i \in \Z$, the {\it lisse motivic complex} $$\Z(i)^{\text{lisse}}(-) : \text{AniRings} \longrightarrow \mathcal{D}(\Z)$$ is the shifted graded piece of the motivic filtration of Definition~\ref{definitionlissemotivicfiltrationconnectiveKtheory}: $$\Z(i)^{\text{lisse}}(-) := \text{gr}^i_{\text{lisse}} \text{K}^{\text{conn}}(-) [-2i].$$ \end{definition} Note that the lisse motivic complexes $\Z(i)^{\text{lisse}}$ are the left Kan extension of the classical motivic complexes $\Z(i)^{\text{cla}}$, and in particular vanish in weights $i<0$. \begin{example}[Weight zero lisse motivic cohomology] For every local ring $R$, there is a natural equivalence $$\Z(0)^{\text{lisse}}(R) \simeq \Z[0]$$ in the derived category $\mathcal{D}(\Z)$. This is a consequence of Example~\ref{exampleweightzeroclassicalmotiviccohomology}, by using that the functor $\Z(0)^{\text{lisse}}(-)$ is left Kan extended on local rings from its restriction to local essentially smooth $\Z$\nobreakdash-algebras. \end{example} \begin{example}[Weight one lisse motivic cohomology]\label{exampleweight1lissemotiviccohomology} For every commutative ring $R$, there is a natural equivalence $$\Z(1)^{\text{lisse}}(R) \simeq \big(\tau^{\leq 1} R\Gamma_{\text{Zar}}(R,\mathbb{G}_m)\big)[-1]$$ in the derived category $\mathcal{D}(\Z)$. In particular, the complex $\Z(1)^{\text{lisse}}(R) \in \mathcal{D}(\Z)$ is concentrated in degrees one and two, where it is given by $$\text{H}^1(\Z(1)^{\text{lisse}}(R)) \cong \mathcal{O}(R)^{\times} \quad \text{ and } \quad \text{H}^2(\Z(1)^{\text{lisse}}(R)) \cong \text{Pic}(R).$$ This is a consequence of Example~\ref{exampleweight1classicalmotiviccohomology}, by using that the left Kan extension of a functor taking values in degrees at most two takes values in degrees at most two, and that the functor $\tau^{\leq 1} R\Gamma_{\text{Zar}}(-,\mathbb{G}_m)$ on commutative rings is left Kan extended from its restriction to smooth $\Z$-algebras. Here the latter left Kan extension property is a consequence of the same left Kan extension property for the functors $\mathbb{G}_m(-)$ (which is a special case of Mathew's criterion \cite[Proposition~A$.0.4$]{elmanto_modules_2020}) and $\text{Pic}(-)$ (which is a consequence of rigidity, see \cite[Lemma~$7.6$]{elmanto_motivic_2023}). \end{example} \begin{definition}[Cdh-local motivic filtration on $KH$-theory of schemes]\label{definitionmotivicfiltrationonKHtheoryofschemes} The {\it cdh-local motivic filtration on homotopy $K$-theory} of qcqs schemes is the functor $$\text{Fil}^\star_{\text{cdh}} \text{KH}(-) : \text{Sch}^{\text{qcqs,op}} \longrightarrow \text{FilSp}$$ defined as $$\text{Fil}^\star_{\text{cdh}} \text{KH}(-) := \big(L_{\text{cdh}} \text{Fil}^\star_{\text{lisse}} \text{K}^{\text{conn}}\big)(-) = \big(L_{\text{cdh}} L_{\text{Sch}^{\text{qcqs,op}}/\text{Sm}_{\Z}^{\text{op}}} \text{Fil}^\star_{\text{cla}} \text{K}\big)(-).$$ \end{definition} \begin{remark}\label{remarkclassicalcdhlocalmotivicfiltrationcomparisonmap} By construction of the cdh-local motivic filtration (Definition~\ref{definitionmotivicfiltrationonKHtheoryofschemes}), there is a natural comparison map of presheaves $$\text{Fil}^\star_{\text{cla}} \text{K}(-) \longrightarrow \text{Fil}^\star_{\text{cdh}} \text{KH}(-)$$ on smooth $\Z$-schemes. \end{remark} \begin{definition}[Cdh-local motivic cohomology of schemes] For any integer $i \in \Z$, the {\it cdh-local motivic complex} $$\Z(i)^{\text{cdh}}(-) : \text{Sch}^{\text{qcqs,op}} \longrightarrow \mathcal{D}(\Z)$$ is the shifted graded piece of the motivic filtration of Definition~\ref{definitionmotivicfiltrationonKHtheoryofschemes}: $$\Z(i)^{\text{cdh}}(-) := \text{gr}^i_{\text{cdh}} \text{KH}(-)[-2i].$$ \end{definition} Although we will refer throughout the text to \cite{bachmann_A^1-invariant_2024} for the properties of these cdh\nobreakdash-local motivic complexes $\Z(i)^{\text{cdh}}$, we already mention the following result, as it will play an important role to establish the completeness of the motivic filtration on algebraic $K$-theory. \begin{proposition}[Completeness of the cdh-local motivic filtration, after \cite{bachmann_A^1-invariant_2024}]\label{propositioncompletenessofcdhlocalmotivicfiltration} Let $d \geq 0$ be an integer, and $X$ be a qcqs scheme of valuative dimension at most $d$. Then for every integer $i \in \Z$, the spectrum $\emph{Fil}^i_{\emph{cdh}} \emph{KH}(X)$ is in cohomological degrees at most $-i+d$. In particular, the filtration $\emph{Fil}^\star_{\emph{cdh}} \emph{KH}(X)$, and its rationalisation $\emph{Fil}^\star_{\emph{cdh}} \emph{KH}(X;\Q)$, are complete. \end{proposition} \subsection{Definition of motivic cohomology}\label{subsectiondefinitionmotiviccohomology} \vspace{-\parindent} \hspace{\parindent} In this subsection, we introduce the motivic filtration on algebraic $K$-theory of qcqs derived schemes (Definitions~\ref{definitionmotivicfiltrationonKtheoryofschemes} and \ref{definitionmotivicfiltrationonKtheoryofderivedschemes}), by constructing a filtered extension of the cartesian square of Theorem~\ref{theoremKST+LT}. To do so, we first define a filtered cdh-local cyclotomic trace map $\text{Fil}^\star_{\text{cdh}} \text{KH}(-) \rightarrow \text{Fil}^\star_{\text{mot}} L_{\text{cdh}} \text{TC}(-)$ (Construction~\ref{constructionfilteredcdhlocalcyclotomictrace}). \begin{theorem}[Filtered cyclotomic trace in the smooth case]\label{theoremfilteredcyclotomictraceinthesmoothcase} The cyclotomic trace map $$\emph{K}(-) \longrightarrow \emph{TC}(-),$$ viewed as a map of spectra-valued presheaves on the category of smooth $\Z$-schemes, admits a unique, multiplicative extension to a map $$\emph{Fil}^\star_{\emph{cla}} \emph{K}(-) \longrightarrow \emph{Fil}^\star_{\emph{mot}} \emph{TC}(-)$$ of filtered presheaves of spectra. \end{theorem} \begin{proof} By Definition~\ref{definitionmotivicfiltrationonTC}, the natural cartesian square $$\begin{tikzcd} \text{TC}(-) \ar[r] \ar[d] & \text{HC}^-(-) \ar[d] \\ \prod_{p \in \mathbb{P}} \text{TC}(-;\Z_p) \ar[r] & \prod_{p \in \mathbb{P}} \text{HC}^-(-;\Z_p) \end{tikzcd}$$ admits a multiplicative filtered extension $$\begin{tikzcd} \text{Fil}^\star_{\text{mot}} \text{TC}(-) \ar[r] \ar[d] & \text{Fil}^\star_{\text{HKR}} \text{HC}^-(-) \ar[d] \\ \prod_{p \in \mathbb{P}} \text{Fil}^\star_{\text{BMS}} \text{TC}(-;\Z_p) \ar[r] & \prod_{p \in \mathbb{P}} \text{Fil}^\star_{\text{HKR}} \text{HC}^-(-;\Z_p) \end{tikzcd}$$ on qcqs derived schemes. Let $p$ be a prime number. It then suffices to prove that the natural maps $$\text{K}(-) \longrightarrow \text{HC}^-(-), \text{ } \text{K}(-) \longrightarrow \text{HC}^-(-;\Z_p), \text{ and }\text{K}(-) \longrightarrow \text{TC}(-;\Z_p),$$ viewed as maps of spectra-valued presheaves on the category of smooth $\Z$-schemes, admit unique multiplicative filtered extensions to maps of filtered presheaves of spectra $$\text{Fil}^\star_{\text{cla}} \text{K}(-) \longrightarrow \text{Fil}^\star_{\text{HKR}} \text{HC}^-(-), \quad \text{Fil}^\star_{\text{cla}} \text{K}(-) \longrightarrow \text{Fil}^\star_{\text{HKR}} \text{HC}^-(-;\Z_p), \text{ and }$$ $$ \text{Fil}^\star_{\text{cla}} \text{K}(-) \longrightarrow \text{Fil}^\star_{\text{BMS}} \text{TC}(-;\Z_p)$$ respectively. The proof of \cite[Proposition~$4.6$]{elmanto_motivic_2023}, which is stated over a quasismooth map of commutative rings $k_0 \rightarrow k$ such that $k$ is a field, applies readily to the case where $k_0=k=\Z$. More precisely, we use in this proof the Gersten conjecture for classical motivic cohomology on smooth $\Z$-schemes, which is \cite[Theorem~$1.1$]{geisser_motivic_2004}. In particular, the natural map $\text{K}(-) \rightarrow \text{HC}^-(-)$, viewed as a map of spectra-valued presheaves on the category of smooth $\Z$-schemes, admits a unique multiplicative filtered extension to a map of filtered presheaves of spectra $\text{Fil}^\star_{\text{cla}} \text{K}(-) \rightarrow \text{Fil}^\star_{\text{HKR}} \text{HC}^-(-)$. Similarly, the $p$-completed cotangent complex $(\mathbb{L}_{R/\Z})^\wedge_p$ of a smooth $\Z$-algebra $R$ is concentrated in degree zero, given by the $p$-flat $R$-module $(\Omega^1_{R/\Z})^\wedge_p$. So for every integer $i \in \Z$, this implies that the object $\text{gr}^i_{\text{HKR}} \text{HC}^-(R;\Z_p) \in \mathcal{D}(\Z)$ is in cohomological degrees less than~$-i$. The proof of \cite[Proposition~$4.6$]{elmanto_motivic_2023} then adapts readily to prove that the natural map $\text{K}(-) \rightarrow \text{HC}^-(-;\Z_p)$, viewed as a map of spectra-valued presheaves on the category of smooth $\Z$-schemes, admits a unique multiplicative extension to a map of filtered presheaves of spectra $\text{Fil}^\star_{\text{cla}} \text{K}(-) \rightarrow \text{Fil}^\star_{\text{HKR}} \text{HC}^-(-;\Z_p)$. Finally, the natural map $\text{K}(-) \rightarrow \text{TC}(-;\Z_p)$, viewed as a map of spectra-valued presheaves on the category of smooth $\Z$-schemes, admits a unique multiplicative extension to a map of filtered presheaves of spectra $\text{Fil}^\star_{\text{cla}} \text{K}(-) \rightarrow \text{Fil}^\star_{\text{BMS}} \text{TC}(-;\Z_p)$, by \cite[Proposition~$6.12$]{annala_atiyah_2024}. \end{proof} \begin{remark}\label{remarkBMSfiltrationonTCcanbereconstructedfromclassicalmotivicfiltration} For every prime number $p$, the BMS filtration $\text{Fil}^\star_{\text{BMS}} \text{TC}(-;\Z_p)$ is determined by its $p$-quasisyntomic-local values (Theorem~\ref{theoremBMSfiltrationonTCpcompletedsatisfiesquasisyntomicdescentandisLKEfrompolynomialalgebras}). By the proof of \cite[Lemma~$6.10$]{annala_atiyah_2024}, the $p$-completed left Kan extension of the functor $\text{Fil}^\star_{\text{cla}} \text{K}(-)$, from smooth $\Z$-algebras to $p$\nobreakdash-quasisyntomic rings, is $p$\nobreakdash-quasisyntomic-locally identified, via the map induced by \cite[Proposition~$6.12$]{annala_atiyah_2024}, with the functor $\text{Fil}^\star_{\text{BMS}} \text{TC}(-;\Z_p)$. In particular, one can reconstruct the BMS filtration $\text{Fil}^\star_{\text{BMS}} \text{TC}(-;\Z_p)$ on qcqs derived schemes (Definition~\ref{definitionBMSfiltrationonTCpcompletedofqcqsderivedschemes}) from the classical motivic filtration $\text{Fil}^\star_{\text{cla}} \text{K}(-)$ on smooth $\Z$-schemes (Definition~\ref{definitionclassicalmotivicfiltration}). This will be used in Section~\ref{sectionratonialstructure} to construct Adams operations on the BMS filtration $\text{Fil}^\star_{\text{BMS}} \text{TC}(-;\Z_p)$. \end{remark} \begin{construction}[Filtered cdh-local cyclotomic trace]\label{constructionfilteredcdhlocalcyclotomictrace} The {\it filtered cdh-local cyclotomic trace map} is the map $$\text{Fil}^\star_{\text{cdh}} \text{KH}(-) \longrightarrow \text{Fil}^\star_{\text{mot}} L_{\text{cdh}} \text{TC}(-)$$ of functors from (the opposite category of) qcqs schemes to the category of multiplicative filtered spectra defined as the cdh sheafification of the composite $$\big(L_{\text{Sch}^{\text{qcqs,op}}/\text{Sm}_{\Z}^{\text{op}}} \text{Fil}^\star_{\text{cla}} \text{K}\big)(-) \longrightarrow \big(L_{\text{Sch}^{\text{qcqs,op}}/\text{Sm}_{\Z}^{\text{op}}} \text{Fil}^\star_{\text{mot}} \text{TC}\big)(-) \longrightarrow \text{Fil}^\star_{\text{mot}} \text{TC}(-),$$ where the first map is the map induced by Theorem~\ref{theoremfilteredcyclotomictraceinthesmoothcase} and the second map is the canonical map. Note here that sheafification is a multiplicative operation, and that the compatibility between left Kan extension and multiplicative structures is ensured by \cite[Corollary~$3.2.3.2$]{lurie_higher_2017} (see also \cite[$2.3.2$]{elmanto_motivic_2023}). \end{construction} \needspace{5\baselineskip} \begin{definition}[Motivic filtration on $K$-theory of schemes]\label{definitionmotivicfiltrationonKtheoryofschemes} The {\it motivic filtration on non-connective algebraic $K$-theory} of qcqs schemes is the functor $$\text{Fil}^\star_{\text{mot}} \text{K}(-) : \text{Sch}^{\text{qcqs,op}} \longrightarrow \text{FilSp}$$ defined by the cartesian square of functors of multiplicative filtered spectra $$\begin{tikzcd} \text{Fil}^\star_{\text{mot}} \text{K}(-) \ar[r] \ar[d] & \text{Fil}^\star_{\text{mot}} \text{TC}(-) \ar[d] \\ \text{Fil}^\star_{\text{cdh}} \text{KH}(-) \ar[r] & \text{Fil}^\star_{\text{mot}} L_{\text{cdh}} \text{TC}(-), \end{tikzcd}$$ where the bottom horizontal map is the map of Construction \ref{constructionfilteredcdhlocalcyclotomictrace}, and the right vertical map is cdh sheafification. \end{definition} \begin{definition}[Motivic filtration on $K$-theory of derived schemes]\label{definitionmotivicfiltrationonKtheoryofderivedschemes} The {\it motivic filtration on non-connective algebraic $K$-theory} of qcqs derived schemes is the functor $$\text{Fil}^\star_{\text{mot}} \text{K}(-) : \text{dSch}^{\text{qcqs,op}} \longrightarrow \text{FilSp}$$ defined by the cartesian square of functors of multiplicative filtered spectra $$\begin{tikzcd} \text{Fil}^\star_{\text{mot}} \text{K}(-) \ar[r] \ar[d] & \text{Fil}^\star_{\text{mot}} \text{TC}(-) \ar[d] \\ \text{Fil}^\star_{\text{mot}} \text{K}(\pi_0(-)) \ar[r] & \text{Fil}^\star_{\text{mot}} \text{TC}(\pi_0(-)) \end{tikzcd}$$ where $\pi_0(-) : \text{dSch} \rightarrow \text{Sch}$ is restriction to the classical locus, the filtration on $\text{K}(\pi_0(-))$ is given by Definition~\ref{definitionmotivicfiltrationonKtheoryofschemes}, and the filtrations on $\text{TC}(-)$ and $\text{TC}(\pi_0(-))$ are given by Definition~\ref{definitionmotivicfiltrationonTC}. \end{definition} \begin{definition}[Motivic cohomology of derived schemes]\label{definitionmotiviccohomologyofderivedschemes} For any integer $i \in \Z$, the {\it motivic complex} $$\Z(i)^{\text{mot}} : \text{dSch}^{\text{qcqs,op}} \longrightarrow \mathcal{D}(\Z)$$ is the shifted graded piece of the motivic filtration of Definition~\ref{definitionmotivicfiltrationonKtheoryofderivedschemes}: $$\Z(i)^{\text{mot}}(-) := \text{gr}^i_{\text{mot}} \text{K}(-)[-2i].$$ For every qcqs derived scheme $X$, also denote by $$\text{H}^n_{\text{mot}}(X,\Z(i)) := \text{H}^n(\Z(i)^{\text{mot}}(X)) \quad n\in \Z$$ the {\it motivic cohomology groups} of $X$. \end{definition} \begin{remark}[Motivic cohomology of schemes]\label{remarkmaincartesiansquareformotiviccohomology} Let $X$ be a qcqs scheme, and $i \in \Z$ be an integer. By Definition \ref{definitionmotivicfiltrationonKtheoryofderivedschemes}, there is a natural cartesian square $$\begin{tikzcd} \Z(i)^{\text{mot}}(X) \arrow{r} \arrow{d} & \Z(i)^{\text{TC}}(X) \ar[d] \\ \Z(i)^{\text{cdh}}(X) \arrow{r} & \big(L_{\text{cdh}} \Z(i)^{\text{TC}}\big)(X) \end{tikzcd}$$ in the derived category $\mathcal{D}(\Z)$, where the bottom horizontal map is induced by Construction~\ref{constructionfilteredcdhlocalcyclotomictrace} and the right vertical map is cdh sheafification. This cartesian square can serve as a definition for the motivic cohomology of the scheme $X$. \end{remark} We now construct, for later use, a comparison map from classical motivic cohomology to the motivic cohomology of Definition~\ref{definitionmotiviccohomologyofderivedschemes}. \begin{definition}[Filtered classical-motivic comparison map]\label{definitionfilteredclassicalmotiviccomparisonmap} The {\it filtered classical-motivic comparison map} is the map of presheaves $$\text{Fil}^\star_{\text{cla}} \text{K}(-) \longrightarrow \text{Fil}^\star_{\text{mot}} \text{K}(-)$$ on smooth $\Z$-schemes induced by the maps Remark~\ref{remarkclassicalcdhlocalmotivicfiltrationcomparisonmap} and Theorem~\ref{theoremfilteredcyclotomictraceinthesmoothcase}. Note here that the compatibility between these two maps and Definition~\ref{definitionmotivicfiltrationonKtheoryofschemes} is automatic by Construction~\ref{constructionfilteredcdhlocalcyclotomictrace}. \end{definition} \begin{definition}[Classical-motivic comparison map]\label{definitionclassicalmotiviccomparisonmap} For any integer $i \in \Z$, the {\it classical-motivic comparison map} is the map of $\mathcal{D}(\Z)$-valued presheaves $$\Z(i)^{\text{cla}}(-) \longrightarrow \Z(i)^{\text{mot}}(-)$$ on smooth $\Z$-schemes induced by taking the $i^{\text{th}}$ shifted graded piece of the filtered map of Definition~\ref{definitionfilteredclassicalmotiviccomparisonmap}. \end{definition} In the rest of this section, we discuss some of the first properties of the motivic filtration. \begin{remark}[Comparison to cdh-local motivic cohomology]\label{remarkcomparisontocdhlocalmotiviccohomology} By construction (Definition~\ref{definitionmotivicfiltrationonKHtheoryofschemes}), the cdh-local motivic complex $$\Z(i)^{\text{cdh}} : \text{Sch}^{\text{qcqs,op}} \longrightarrow \mathcal{D}(\Z)$$ is a cdh sheaf, so the common fibre of the horizontal maps in the cartesian square of Remark~\ref{remarkmaincartesiansquareformotiviccohomology} is also a cdh sheaf. In particular, for every qcqs scheme $X$, the left vertical map of this cartesian square exhibits cdh-local motivic cohomology as the cdh sheafification of motivic cohomology: $$\Z(i)^{\text{cdh}}(X) \simeq \big(L_{\text{cdh}} \Z(i)^{\text{mot}}\big)(X).$$ \end{remark} \begin{proposition}\label{propositionpadicstructuremain} Let $X$ be a qcqs scheme, and $p$ be a prime number. Then for every integer $k \geq 1$, there is a natural cartesian square $$\begin{tikzcd} \emph{Fil}^\star_{\emph{mot}} \emph{K}(X;\Z/p^k) \arrow{r} \arrow{d} & \emph{Fil}^\star_{\emph{BMS}} \emph{TC}(X;\Z/p^k) \ar[d] \\ \emph{Fil}^\star_{\emph{cdh}} \emph{KH}(X;\Z/p^k) \arrow{r} & \emph{Fil}^\star_{\emph{BMS}} L_{\emph{cdh}} \emph{TC}(X;\Z/p^k) \end{tikzcd}$$ of filtered spectra. \end{proposition} \begin{proof} This is a consequence of Proposition~\ref{propositionpcompletionofmotiviconTCisBMS} and Definition~\ref{definitionmotivicfiltrationonKtheoryofderivedschemes}. \end{proof} \begin{corollary}\label{corollarymainpadicstructureongradeds} Let $X$ be a qcqs scheme, and $p$ be a prime number. Then for any integers $i \in \Z$ and $k \geq 1$, there is natural cartesian square $$\begin{tikzcd} \Z/p^k(i)^{\emph{mot}}(X) \arrow{r} \arrow{d} & \Z/p^k(i)^{\emph{BMS}}(X) \ar[d] \\ \Z/p^k(i)^{\emph{cdh}}(X) \arrow{r} & \big(L_{\emph{cdh}} \Z/p^k(i)^{\emph{BMS}}\big)(X) \end{tikzcd}$$ in the derived category $\mathcal{D}(\Z/p^k)$. \end{corollary} \begin{proof} This is a direct consequence of Proposition~\ref{propositionpadicstructuremain}. \end{proof} \begin{remark}[$\l$-adic motivic cohomology]\label{remarkladicmotiviccohomology} For any prime number $p$ and integer $k \geq 1$, the filtered presheaf $\text{Fil}^\star_{\text{BMS}} \text{TC}(-;\Z/p^k)$ and its cdh sheafification vanish on qcqs $\Z[\tfrac{1}{p}]$-schemes. In particular, Proposition~\ref{propositionpadicstructuremain} implies that for every qcqs $\Z[\tfrac{1}{p}]$-scheme $X$, the natural map $$\text{Fil}^\star_{\text{mot}} \text{K}(X;\Z/p^k) \longrightarrow \text{Fil}^\star_{\text{cdh}} \text{KH}(X;\Z/p^k)$$ is an equivalence of filtered spectra. \end{remark} \begin{remark}[Completeness and exhaustivity of $\text{Fil}^\star_{\text{mot}} \text{K}$] The filtration $\text{Fil}^\star_{\text{mot}} \text{K}(X)$ of Definition~\ref{definitionmotivicfiltrationonKtheoryofderivedschemes} will be proved to be $\N$-indexed, hence exhaustive, on general qcqs derived schemes $X$ (Proposition~\ref{propositionmotivicfiltrationisexhaustive}), and complete on qcqs schemes of finite valuative dimension (Proposition~\ref{propositionmotivicfiltrationiscompleteonqcqsschemesoffinitevaluativedimension}). Note that these results can already be proved modulo any prime number~$p$, as a formal consequence of Proposition~\ref{propositionpadicstructuremain} and Section~\ref{sectionmotivicfiltrationonTC}. \end{remark} The following result is a filtered version of the classical Dundas--Goodwillie--McCarthy theorem (\cite[Theorem~$7.0.0.2$]{dundas_local_2013}). \begin{proposition}\label{theoremfilteredDGMtheorem} Let $A \rightarrow B$ be a map of animated commutative rings such that the induced map $\pi_0(A) \rightarrow \pi_0(B)$ of commutative rings is surjective with finitely generated nilpotent kernel. Then the natural commutative diagram $$\begin{tikzcd} \emph{Fil}^\star_{\emph{mot}} \emph{K}(A) \arrow{r} \arrow{d} & \emph{Fil}^\star_{\emph{mot}} \emph{TC}(A) \ar[d] \\ \emph{Fil}^\star_{\emph{mot}} \emph{K}(B) \arrow{r} & \emph{Fil}^\star_{\emph{mot}} \emph{TC}(B) \end{tikzcd}$$ is a cartesian square of filtered spectra. \end{proposition} \begin{proof} By Definition~\ref{definitionmotivicfiltrationonKtheoryofderivedschemes}, it suffices to prove that the natural commutative diagram $$\begin{tikzcd} \text{Fil}^\star_{\text{mot}} \text{K}(\pi_0(A)) \arrow{r} \arrow{d} & \text{Fil}^\star_{\text{mot}} \text{TC}(\pi_0(A)) \ar[d] \\ \text{Fil}^\star_{\text{mot}} \text{K}(\pi_0(B)) \arrow{r} & \text{Fil}^\star_{\text{mot}} \text{TC}(\pi_0(B)) \end{tikzcd}$$ is a cartesian square of filtered spectra. For every cdh sheaf $F$ defined on qcqs schemes, the natural map $F(\pi_0(A)) \rightarrow F(\pi_0(B))$ is an equivalence. The result is then a consequence of Definition~\ref{definitionmotivicfiltrationonKtheoryofschemes}. \end{proof} \newpage \section{Rational structure of motivic cohomology}\label{sectionratonialstructure} \vspace{-\parindent} \hspace{\parindent} In this section, we prove some fundamental properties of the motivic filtration that we introduced in the previous section: namely, that it is always exhaustive and finitary, and complete on schemes of finite valuative dimension. These properties modulo a prime number $p$ are a formal consequence of the analogous properties for the BMS filtration on $p$-completed topological cyclic homology. Proving these results integrally will however require more understanding of the rational part of this motivic filtration. Our two main results on rational motivic cohomology are as follows. The first main result is the following generalisation of the rational splitting of the classical motivic filtration. \begin{theorem}[The motivic filtration is rationally split]\label{theorem21'rationalmotivicfiltrationsplits} Let $X$ be a qcqs derived scheme. Then there exists a natural multiplicative equivalence of filtered spectra $$\emph{Fil}^\star_{\emph{mot}} \emph{K}(X;\Q) \simeq \bigoplus_{j \geq \star} \Q(j)^{\emph{mot}}(X)[2j].$$ \end{theorem} As for the classical motivic filtration, this splitting is induced by suitable Adams operations, which we construct in the generality of qcqs derived schemes in the first section. This result is however not enough to prove the exhaustivity, completeness, or finitariness of the motivic filtration. Instead, these will be proved along the way to the following second main result on rational motivic cohomology. \begin{theorem}\label{theoremrationalstructuremain} Let $X$ be a qcqs scheme. Then there is a natural fibre sequence of filtered spectra $$\emph{Fil}^\star_{\emph{mot}} \emph{K}(X;\Q) \rightarrow \emph{Fil}^\star_{\emph{cdh}} \emph{KH}(X;\Q) \rightarrow \emph{cofib}\big( \emph{Fil}^{\star - 1}_{\emph{HKR}} \emph{HC}(X_{\Q}/\Q) \rightarrow \emph{Fil}^{\star - 1}_{\emph{HKR}} L_{\emph{cdh}} \emph{HC}(X_{\Q}/\Q)\big)[1].$$ \end{theorem} To prove this result, we use a classical argument of Weibel at the level of $K$-theory (Lemma~\ref{lemma2WeibelargumentrationalKtheory}) and the rational splitting Theorem~\ref{theorem21'rationalmotivicfiltrationsplits} to reduce the statement to the case of characteristic zero, where the result is essentially \cite[Theorem~$4.10\,(3)$]{elmanto_motivic_2023}. The key step in this argument, in order to pass from a statement at the level of $K$\nobreakdash-theory to a filtered statement, is to prove beforehand that the motivic filtration $\text{Fil}^\star_{\text{mot}} \text{K}$ is $\N$\nobreakdash-indexed (Proposition~\ref{propositionmotivicfiltrationisexhaustive}). The strategy to prove this is as follows. We first introduce a new rigid-analytic variant of the HKR filtrations in the generality of qcqs derived schemes, whose graded pieces are rigid-analytic variants of derived de Rham cohomology. We then adapt a theorem of Goodwillie --stating in modern language that periodic cyclic homology is truncating in characteristic zero-- to this rigid-analytic variant of periodic cyclic homology. This rigid-analytic Goodwillie theorem implies, by the work of Land--Tamme on truncating invariants, that the rigid-analytic variant of periodic cyclic homology is a cdh sheaf on qcqs schemes. A filtered consequence of this cdh descent result then formally implies the desired result, {\it i.e.}, that the motivic filtration $\text{Fil}^\star_{\text{mot}} \text{K}$ is $\N$\nobreakdash-indexed. \subsection{Adams operations}\label{subsectionAdamsoperations} \vspace{-\parindent} \hspace{\parindent} In this subsection we construct Adams operations on filtered algebraic $K$-theory of qcqs derived schemes (Construction~\ref{constuction22'AdamsonKtheory}). In \cite[Appendix~B]{elmanto_motivic_2023}, Elmanto--Morrow construct Adams operations $\psi^m$ on the $m$\nobreakdash-perio\-dic filtered $K$-theory $\text{Fil}^\star_{\text{cla}} \text{K}(X)[\tfrac{1}{m}]$ of smooth $\Z$-schemes $X$, acting on the $i^{\text{th}}$ graded piece as multiplication by $m^i$. Using this construction, Bachmann--Elmanto--Morrow construct the following Adams operations $\psi^m$ on the filtered $KH$-theory of arbitrary qcqs schemes $X$. \begin{proposition}[Adams operations on filtered $KH$-theory, \cite{bachmann_A^1-invariant_2024}]\label{proposition34AdamsoperationsonfilteredKHtheory} Let $m \geq 2$ be an integer, and $X$ be a qcqs scheme. Then there exists a natural automorphism $\psi^m$ of the filtered spectrum $\emph{Fil}^\star_{\emph{cdh}} \emph{KH}(X)[\tfrac{1}{m}]$ such that for every integer $i \in \Z$, the induced automorphism on the $i^{\emph{th}}$ graded piece $\Z[\tfrac{1}{m}](i)^{\emph{cdh}}(X)[2i]$ is multiplication by $m^i$. \end{proposition} We now use the Adams operations $\psi^m$ on the $m$-periodic filtered $K$-theory $\text{Fil}^\star_{\text{cla}} \text{K}(X)[\tfrac{1}{m}]$ of smooth $\Z$-schemes $X$ (\cite[Appendix~$B$]{elmanto_motivic_2023}) to construct Adams operations $\psi^m$ on the filtered $p$-completed topological cyclic homology $\text{Fil}^\star_{\text{BMS}} \text{TC}(X;\Z_p)$ of qcqs $\Z[\tfrac{1}{m}]$-schemes $X$. \begin{proposition}[Adams operations on filtered $\text{TC}(-;\Z_p)$]\label{proposition24AdamsoprationsonfilteredTCpcompleted} Let $m \geq 2$ be an integer, $X$ be a qcqs derived $\Z[\tfrac{1}{m}]$-scheme, and $p$ be a prime number. Then there exists a natural multiplicative automorphism $\psi^m$ of the filtered spectrum $\emph{Fil}^\star_{\emph{BMS}} \emph{TC}(X;\Z_p)$ such that for every integer $i \in \Z$, the induced automorphism on the $i^{\emph{th}}$ graded piece $\Z_p(i)^{\emph{BMS}}(X)[2i]$ is multiplication by $m^i$. Moreover, this automorphism $\psi^m$ is uniquely determined by its naturality and the fact that on smooth $\Z[\tfrac{1}{m}]$-schemes $X$, the diagram of filtered spectra $$\begin{tikzcd} \emph{Fil}^\star_{\emph{cla}} \emph{K}(X)[\tfrac{1}{m}] \arrow{r} \arrow[d,"\psi^m"] & \emph{Fil}^\star_{\emph{BMS}} \emph{TC}(X;\Z_p) \arrow[d,"\psi^m"] \\ \emph{Fil}^\star_{\emph{cla}} \emph{K}(X)[\tfrac{1}{m}] \arrow{r} & \emph{Fil}^\star_{\emph{BMS}} \emph{TC}(X;\Z_p) \end{tikzcd}$$ where the horizontal maps are induced by Remark~\ref{remarkBMSfiltrationonTCcanbereconstructedfromclassicalmotivicfiltration}, and the left vertical map is defined in \cite[Construction~$\emph{B}.4$]{elmanto_motivic_2023}, is commutative. \end{proposition} \begin{proof} The functor $\text{Fil}^\star_{\text{BMS}} \text{TC}(-;\Z_p)$ on $p$-quasisyn\-tomic rings is $p$-quasisyn\-tomic-locally identified with the $p$-completed left Kan extension of the functor $\text{Fil}^\star_{\text{cla}} \text{K}(-)$ from smooth $\Z$-algebras to $p$\nobreakdash-quasisyn\-tomic rings (Remark~\ref{remarkBMSfiltrationonTCcanbereconstructedfromclassicalmotivicfiltration}). The functor $\text{Fil}^\star_{\text{BMS}} \text{TC}(-;\Z_p)$ on $p$-quasisyntomic rings moreover satisfies $p$-quasisyntomic descent (Theorem~\ref{theoremBMSfiltrationonTCpcompletedsatisfiesquasisyntomicdescentandisLKEfrompolynomialalgebras}\,$(1)$), so the Adams operation $\psi^m$ on the functor $\text{Fil}^\star_{\text{cla}} \text{K}(-)[\tfrac{1}{m}]$ (\cite[Appendix~B]{elmanto_motivic_2023}) induces a natural automorphism $\psi^m$ of the presheaf $\text{Fil}^\star_{\text{BMS}} \text{TC}(-;\Z_p)$, acting as multiplication by $m^i$ on the $i^{\text{th}}$ graded piece $\Z_p(i)^{\text{BMS}}(-)[2i]$. The same result then applies to animated commutative $\Z[\tfrac{1}{m}]$-algebras by left Kan extending the result on polynomial $\Z[\tfrac{1}{m}]$-algebras (Theorem~\ref{theoremBMSfiltrationonTCpcompletedsatisfiesquasisyntomicdescentandisLKEfrompolynomialalgebras}\,$(2)$), and to general qcqs derived $\Z[\tfrac{1}{m}]$-schemes by Zariski sheafifying the result on animated commutative $\Z[\tfrac{1}{m}]$\nobreakdash-algebras. \end{proof} The following result is \cite[Construction~$6.4.8$ and Proposition~$6.4.12$]{raksit_hochschild_2020}. Note that if $X$ is a qcqs derived $\Z[\tfrac{1}{m}]$-scheme, then the filtered spectrum $\text{Fil}^\star_{\text{HKR}} \text{HC}^-(X)$ is naturally $\Z[\tfrac{1}{m}]$-linear. \begin{proposition}[Adams operations on filtered $\text{HC}^-$, \cite{raksit_hochschild_2020}]\label{proposition23raksitAdamsoperationsonHC-withmintertibleinthescheme} Let $m \geq 2$ be an integer, and $X$ be qcqs derived $\Z[\tfrac{1}{m}]$-scheme. Then there exists a natural multiplicative automorphism~$\psi^m$ of the filtered spectrum $\emph{Fil}^\star_{\emph{HKR}} \emph{HC}^-(X)$ such that for every integer $i \in \Z$, the induced automorphism on the $i^{\emph{th}}$ graded piece $\widehat{\mathbb{L}\Omega}^{\geq i}_{X/\Z[\tfrac{1}{m}]}[2i]$ is multiplication by $m^i$. \end{proposition} \begin{lemma}\label{lemma35compatibilityAdamsTCpcompletedandHC-pcompleted} Let $m \geq 2$ be an integer, $X$ be a qcqs derived $\Z[\tfrac{1}{m}]$-scheme, and $p$ be a prime number. Then the natural diagram of filtered spectra $$\begin{tikzcd} \emph{Fil}^\star_{\emph{BMS}} \emph{TC}(X;\Z_p) \arrow{r} \arrow[d,"\psi^m"] & \emph{Fil}^\star_{\emph{HKR}} \emph{HC}^-(X;\Z_p) \arrow[d,"\psi^m"] \\ \emph{Fil}^\star_{\emph{BMS}} \emph{TC}(X;\Z_p) \arrow{r} & \emph{Fil}^\star_{\emph{HKR}} \emph{HC}^-(X;\Z_p) \end{tikzcd}$$ where the horizontal maps are defined in Construction~\ref{constructionfilteredmapTCpcompletedtoHC-pcompleted}, the left map is the map defined in Proposition~\ref{proposition24AdamsoprationsonfilteredTCpcompleted}, and the right map is the map induced by Proposition~\ref{proposition23raksitAdamsoperationsonHC-withmintertibleinthescheme}, is commutative. \end{lemma} \begin{proof} By \cite[Lemma~B.$8$]{elmanto_motivic_2023}, the natural diagram of filtered spectra $$\begin{tikzcd} \text{Fil}^\star_{\text{cla}} \text{K}(X)[\frac{1}{m}] \arrow{r} \arrow[d,"\psi^m"] & \text{Fil}^\star_{\text{HKR}} \text{HC}^-(X) \arrow[d,"\psi^m"] \\ \text{Fil}^\star_{\text{cla}} \text{K}(X)[\tfrac{1}{m}] \arrow{r} & \text{Fil}^\star_{\text{HKR}} \text{HC}^-(X) \end{tikzcd}$$ is commutative for every smooth $\Z[\tfrac{1}{m}]$-scheme $X$. The result is then a consequence of Proposition~\ref{proposition24AdamsoprationsonfilteredTCpcompleted}, where the compatibility between the filtered maps is a consequence of the proof of Theorem~\ref{theoremfilteredcyclotomictraceinthesmoothcase}. \end{proof} \begin{construction}[Adams operations on filtered TC]\label{construction31'AdamsonTC} Let $m \geq 2$ be an integer, and $X$ be a qcqs derived $\Z[\tfrac{1}{m}]$-scheme. The automorphism $\psi^m$ of the filtered spectrum $\text{Fil}^\star_{\text{mot}} \text{TC}(X)$ is the automorphism defined by pullback along the natural cartesian square of filtered spectra $$\begin{tikzcd} \text{Fil}^\star_{\text{mot}} \text{TC}(X) \arrow{r} \arrow{d} & \text{Fil}^\star_{\text{HKR}} \text{HC}^-(X) \ar[d] \\ \prod_{p \in \mathbb{P}} \text{Fil}^\star_{\text{BMS}} \text{TC}(X;\Z_p) \arrow{r} & \prod_{p \in \mathbb{P}} \text{Fil}^\star_{\text{HKR}} \text{HC}^-(X;\Z_p), \end{tikzcd}$$ where the automorphism $\psi^m$ of $\text{Fil}^\star_{\text{HKR}} \text{HC}^-(X)$ is the automorphism of Proposition~\ref{proposition23raksitAdamsoperationsonHC-withmintertibleinthescheme}, the automorphism $\psi^m$ of $\prod_{p \in \mathbb{P}} \text{Fil}^\star_{\text{HKR}} \text{HC}^-(X;\Z_p)$ is induced by the automorphism $\psi^m$ of $\text{Fil}^\star_{\text{HKR}} \text{HC}^-(X)$, and the automorphism $\psi^m$ of $\prod_{p \in \mathbb{P}} \text{Fil}^\star_{\text{BMS}} \text{TC}(X;\Z_p)$ is the automorphism of Proposition~\ref{proposition24AdamsoprationsonfilteredTCpcompleted}. Note here that the compatibility between the automorphisms $\psi^m$ and the bottom map is given by Lemma~\ref{lemma35compatibilityAdamsTCpcompletedandHC-pcompleted}. \end{construction} Note in the following result that if $X$ is a qcqs derived $\Z[\tfrac{1}{m}]$-scheme, then the filtered spectrum $\text{Fil}^\star_{\text{mot}} \text{TC}(X)$ is naturally $\Z[\tfrac{1}{m}]$-linear. \begin{corollary}\label{corollary31''AdamsoperationsongradedpiecesoffilteredTC} Let $m \geq 2$ be an integer, and $X$ be a qcqs derived $\Z[\tfrac{1}{m}]$-scheme. Then for every integer $i \in \Z$, the automorphism $\psi^m$ induced on the $i^{\emph{th}}$ graded piece $\Z(i)^{\emph{TC}}(X)[2i]$ of the filtered spectrum $\emph{Fil}^\star_{\emph{mot}} \emph{TC}(X)$ is multiplication by $m^i$. Moreover, if $X$ is smooth over $\Z[\tfrac{1}{m}]$, then the natural diagram of filtered spectra $$\begin{tikzcd} \emph{Fil}^\star_{\emph{cla}} \emph{K}(X)[\tfrac{1}{m}] \arrow{r} \arrow[d,"\psi^m"] & \emph{Fil}^\star_{\emph{mot}} \emph{TC}(X) \arrow[d,"\psi^m"] \\ \emph{Fil}^\star_{\emph{cla}} \emph{K}(X)[\tfrac{1}{m}] \arrow{r} & \emph{Fil}^\star_{\emph{mot}} \emph{TC}(X) \end{tikzcd}$$ where the horizontal maps are defined in Theorem~\ref{theoremfilteredcyclotomictraceinthesmoothcase}, and the left vertical map is defined in \cite[Construction $\emph{B}.4$]{elmanto_motivic_2023}, is commutative. \end{corollary} \begin{proof} The identification of the automorphism $\psi^m$ on the graded pieces is a consequence of Propositions~\ref{proposition24AdamsoprationsonfilteredTCpcompleted} and \ref{proposition23raksitAdamsoperationsonHC-withmintertibleinthescheme}. The second statement is a consequence of the analogous compatibilities for $\prod_{p \in \mathbb{P}} \text{Fil}^\star_{\text{BMS}} \text{TC}(X;\Z_p)$ (Proposition~\ref{proposition24AdamsoprationsonfilteredTCpcompleted}) and for $\text{Fil}^\star_{\text{HKR}} \text{HC}^-(X)$ (\cite[Lemma~B.$8$]{elmanto_motivic_2023}), and of Lem\-ma~\ref{lemma35compatibilityAdamsTCpcompletedandHC-pcompleted}. \end{proof} \begin{lemma}\label{lemma36compatibilityAdamsoperationsonKHandLcdhTC} Let $m \geq 2$ be an integer, and $X$ be a qcqs $\Z[\tfrac{1}{m}]$-scheme. Then the natural diagram of filtered spectra $$\begin{tikzcd} \emph{Fil}^\star_{\emph{cdh}} \emph{KH}(X)[\tfrac{1}{m}] \arrow{r} \arrow[d,"\psi^m"] & \emph{Fil}^\star_{\emph{mot}} L_{\emph{cdh}} \emph{TC}(X) \arrow[d,"\psi^m"] \\ \emph{Fil}^\star_{\emph{cdh}} \emph{KH}(X)[\frac{1}{m}] \arrow{r} & \emph{Fil}^\star_{\emph{mot}} L_{\emph{cdh}} \emph{TC}(X) \end{tikzcd}$$ where the horizontal maps are defined in Construction~\ref{constructionfilteredcdhlocalcyclotomictrace}, the left map is the map of Proposition~\ref{proposition34AdamsoperationsonfilteredKHtheory}, and the right map is the map induced by Construction~\ref{construction31'AdamsonTC}, is commutative. \end{lemma} \begin{proof} The left map $\psi^m$ of this diagram is defined by cdh sheafifying the left Kan extension from smooth $\Z[\tfrac{1}{m}]$-schemes to qcqs $\Z[\tfrac{1}{m}]$-schemes of the automorphism $\psi^m$ on $\text{Fil}^\star_{\text{cla}} \text{K}(-)[\tfrac{1}{m}]$ (\cite[Appendix~B]{elmanto_motivic_2023}). The result then follows from Construction~\ref{constructionfilteredcdhlocalcyclotomictrace} and Corollary~\ref{corollary31''AdamsoperationsongradedpiecesoffilteredTC}. \end{proof} \begin{construction}[Adams operations on filtered $K$-theory]\label{constuction22'AdamsonKtheory} Let $m \geq 2$ be an integer. Following Definition~\ref{definitionmotivicfiltrationonKtheoryofschemes}, if $X$ is a qcqs $\Z[\tfrac{1}{m}]$-scheme, the automorphism $\psi^m$ of the filtered spectrum $\text{Fil}^\star_{\text{mot}} \text{K}(X)[\frac{1}{m}]$ is the automorphism defined by pullback along the natural cartesian square of filtered spectra $$\begin{tikzcd} \text{Fil}^\star_{\text{mot}} \text{K}(X)[\tfrac{1}{m}] \arrow{r} \arrow{d} & \text{Fil}^\star_{\text{mot}} \text{TC}(X) \ar[d] \\ \text{Fil}^\star_{\text{cdh}} \text{KH}(X)[\tfrac{1}{m}] \arrow{r} & \text{Fil}^\star_{\text{mot}} L_{\text{cdh}} \text{TC}(X), \end{tikzcd}$$ where the automorphism $\psi^m$ of $\text{Fil}^\star_{\text{mot}} \text{TC}(X)$ is the automorphism of Construction~\ref{construction31'AdamsonTC}, the automorphism $\psi^m$ of $\text{Fil}^\star_{\text{mot}} L_{\text{cdh}} \text{TC}(X)$ is defined by cdh sheafifying the automorphism $\psi^m$ of $\text{Fil}^\star_{\text{mot}} \text{TC}(-)$, and the automorphism $\psi^m$ of $\text{Fil}^\star_{\text{cdh}} \text{KH}(X)[\tfrac{1}{m}]$ is the automorphism of Proposition~\ref{proposition34AdamsoperationsonfilteredKHtheory}. Note here that the compatibility between the automorphisms $\psi^m$ and the bottom map is given by Lemma~\ref{lemma36compatibilityAdamsoperationsonKHandLcdhTC}. Following Definition~\ref{definitionmotivicfiltrationonKtheoryofderivedschemes}, if $X$ is a qcqs derived $\Z[\tfrac{1}{m}]$-scheme, the automorphism $\psi^m$ of the filtered spectrum $\text{Fil}^\star_{\text{mot}} \text{K}(X)[\tfrac{1}{m}]$ is the automorphism defined by pullback along the natural cartesian square of filtered spectra $$\begin{tikzcd} \text{Fil}^\star_{\text{mot}} \text{K}(X)[\tfrac{1}{m}] \ar[r] \ar[d] & \text{Fil}^\star_{\text{mot}} \text{TC}(X) \ar[d] \\ \text{Fil}^\star_{\text{mot}} \text{K}(\pi_0(X))[\tfrac{1}{m}] \ar[r] & \text{Fil}^\star_{\text{mot}} \text{TC}(\pi_0(X)), \end{tikzcd}$$ where the automorphisms $\psi^m$ of $\text{Fil}^\star_{\text{mot}} \text{TC}(X)$ and $\text{Fil}^\star_{\text{mot}} \text{TC}(\pi_0(X))$ are defined in Construction~\ref{construction31'AdamsonTC}, and the automorphism $\psi^m$ of $\text{Fil}^\star_{\text{mot}} \text{K}(\pi_0(X))[\tfrac{1}{m}]$ is the automorphism of the previous paragraph. Note here that the compatibility between the automorphisms $\psi^m$ is automatic by construction. \end{construction} \begin{corollary}\label{corollary22''AdamsoperationsongradedpiecesoffilteredKtheory} Let $m \geq 2$ be an integer, and $X$ be a qcqs derived $\Z[\tfrac{1}{m}]$-scheme. Then for every integer $i \in \Z$, the automorphism $\psi^m$ induced on the $i^{\emph{th}}$ graded piece $\Z[\tfrac{1}{m}](i)^{\emph{mot}}(X)[2i]$ of the filtered spectrum $\emph{Fil}^\star_{\emph{mot}} \emph{K}(X)[\tfrac{1}{m}]$ is multiplication by $m^i$. \end{corollary} \begin{proof} This is a consequence of Proposition~\ref{proposition34AdamsoperationsonfilteredKHtheory} and~Corollary~\ref{corollary31''AdamsoperationsongradedpiecesoffilteredTC}. \end{proof} The following lemma explains how to use Adams operations to deduce splitting results on the rationalisation of certain filtrations. \begin{lemma}\label{lemma29howtouseAdamsoperations} Let $$\emph{Fil}^\star F(-) : \emph{dSch}^{\emph{qcqs,op}} \longrightarrow \emph{FilSp}$$ be a Zariski sheaf of filtered spectra. For each integer $m \geq 2$, let $\psi^m$ be a natural multiplicative automorphism of the filtered spectrum $\emph{Fil}^\star F(X)$ on qcqs derived $\Z[\tfrac{1}{m}]$-schemes $X$, satisfying the following properties: \begin{enumerate}[label=(\roman*)] \item for every qcqs derived scheme $X$, the rationalised filtration $\emph{Fil}^\star F(X;\Q)$ is complete; \item for any integers $i \in \Z$ and $m \geq 2$, and every qcqs derived $\Z[\tfrac{1}{m}]$-scheme $X$, the induced automorphism on the $i^{\emph{th}}$ graded piece $\emph{gr}^i F(X)$ is multiplication by $m^i$; \item for any integers $m, m' \geq 2$, and every qcqs derived $\Z[\tfrac{1}{mm'}]$-scheme $X$, the space of natural transformations from $\psi^m \circ \psi^{m'}$ to $\psi^{mm'}$, as endomorphisms of the filtered spectrum $\emph{Fil}^\star F(X)$, is contractible. \end{enumerate} Then for every qcqs derived scheme $X$, and any integers $i,k \in \Z$ such that $k \geq i$, there exists a natural equivalence of spectra $$\emph{Fil}^i F(X;\Q) \simeq \big(\bigoplus_{i \leq j < k} \emph{gr}^j F(X;\Q)\big) \oplus \emph{Fil}^k F(X;\Q).$$ \end{lemma} \begin{proof} For every spectrum $C$ equipped with a map $F : C \rightarrow C$, denote by $C^F$ the homotopy fibre of the map $F$. Let $m \geq 2$ be an integer, $i,k \in \Z$ be integers such that $k \geq i$, and $X$ be a qcqs derived $\Z[\tfrac{1}{m}]$-scheme, for which we first construct the desired equivalence of spectra $$\text{Fil}^i F(X;\Q) \simeq \big(\bigoplus_{i \leq j < k} \text{gr}^j F(X;\Q)\big) \oplus \text{Fil}^k F(X;\Q).$$ We first prove that the spectrum $\big(\text{Fil}^{i+1} F(X;\Q)\big)^{\psi^m-m^i}$ is zero. The filtration $\text{Fil}^{i+1+\star} F(X;\Q)$ induced on the spectrum $\text{Fil}^{i+1} F(X;\Q)$ is complete (hypothesis~$(i)$), so it suffices to prove that the natural map $$\psi^m - m^i : \text{gr}^j F(X;\Q) \longrightarrow \text{gr}^j F(X;\Q)$$ is an equivalence of spectra for every integer $j \geq i+1$. For every integer $j \geq i+1$, this map can be identified with multiplication by the nonzero integer $m^i(m^{j-i}-1)$ on the $\Q$-linear spectrum $\text{gr}^j F(X;\Q)$ (hypothesis ($ii$)), and is thus an equivalence. Taking the fibre of the natural map $\psi^m - m^i$ on the fibre sequence of spectra $$\text{Fil}^{i+1} F(X;\Q) \longrightarrow \text{Fil}^i F(X;\Q) \longrightarrow \text{gr}^i F(X;\Q),$$ this implies that the natural map $$\big(\text{Fil}^i F(X;\Q)\big)^{\psi^m-m^i} \longrightarrow \big(\text{gr}^i F(X;\Q)\big)^{\psi^m-m^i}$$ is an equivalence of spectra. The spectrum $\big(\text{gr}^i F(X;\Q)\big)^{\psi^m-m^i}$ can be naturally identified with the spectrum $\text{gr}^i F(X;\Q) \oplus \text{gr}^i F(X;\Q)[-1]$ (hypothesis ($ii$)), and the induced composite map $$\text{gr}^i F(X;\Q) \longrightarrow \big(\text{gr}^i F(X;\Q)\big)^{\psi^m-m^i} \xlongrightarrow{\sim} \big(\text{Fil}^i F(X;\Q)\big)^{\psi^m-m^i} \xlongrightarrow{\text{can}} \text{Fil}^i F(X;\Q)$$ induces a natural splitting of spectra $$\text{Fil}^i F(X;\Q) \simeq \text{gr}^i F(X;\Q) \oplus \text{Fil}^{i+1} F(X;\Q).$$ By induction, this implies that for every integer $k \geq i$, there is a natural equivalence of spectra $$\text{Fil}^i F(X;\Q) \simeq \big(\bigoplus_{i \leq j < k} \text{gr}^j F(X;\Q)\big) \oplus \text{Fil}^k F(X;\Q).$$ We now prove the desired equivalence of spectra for a general qcqs derived scheme~$X$. The presheaf $\text{Fil}^\star F(-)$ being a Zariski sheaf of filtered spectra, the presheaves $$\text{Fil}^i F(-;\Q) \text{ and } \big(\bigoplus_{i \leq j < k} \text{gr}^j F(-;\Q)\big) \oplus \text{Fil}^k F(-;\Q)$$ are Zariski sheaves of spectra. It thus suffices to construct compatible equivalences of spectra $$\text{Fil}^i F(X_{\Z[\tfrac{1}{m}]};\Q) \simeq \big(\bigoplus_{i \leq j < k} \text{gr}^j F(X_{\Z[\tfrac{1}{m}]};\Q)\big) \oplus \text{Fil}^k F(X_{\Z[\tfrac{1}{m}]};\Q)$$ for all integers $m \geq 2$. The construction of this equivalence for each integer $m \geq 2$, which depends on the map $\psi^m : \text{Fil}^\star F(X_{\Z[\tfrac{1}{m}]}) \rightarrow \text{Fil}^\star F(X_{\Z[\tfrac{1}{m}]})$, is the first part of this proof. Let $m, m' \geq 2$ be integers. The compatibility between the constructions of the equivalences for $m$, $m'$, and $mm'$ only depend on the choices, for every integer $i \in \Z$, of the identification of the maps $\psi^m$, $\psi^{m'}$, and $\psi^{mm'}$ on the $i^{\text{th}}$ graded piece with multiplication by $m^i$, $(m')^i$, and $(mm')^i$ respectively. These choices are compatible up to homotopy (hypothesis ($iii$)), which concludes the proof. \end{proof} \begin{remark}\label{remarklemmahowtouseAdamsonclassicalschemes} Lemma~\ref{lemma29howtouseAdamsoperations} can also be proved, with the same proof, for Zariski sheaves of filtered spectra $\text{Fil}^\star F(-)$ that are defined on qcqs schemes, on qcqs schemes of finite valuative dimension, on noetherian schemes of finite dimension, or on smooth schemes over a given commutative ring. \end{remark} \begin{remark} Hypothesis ($iii$) in Lemma~\ref{lemma29howtouseAdamsoperations} follows from the construction of the Adams operations on the filtrations $\text{Fil}^\star_{\text{cla}} \text{K}(-)$ and $\text{Fil}^\star_{\text{HKR}} \text{HC}^-(-)$, when these are defined. This hypothesis ($iii$) then follows formally for all the other filtrations considered in this subsection. \end{remark} \begin{proposition}\label{propositionhowtouseAdamsoperations} Let $$\emph{Fil}^\star F(-) : \emph{Sch}^{\emph{qcqs,op}} \longrightarrow \emph{FilSp}$$ be a finitary Zariski sheaf of filtered spectra. For each integer $m \geq 2$, let $\psi^m$ be a natural multiplicative automorphism of the filtered spectrum $\emph{Fil}^\star F(X)$ on qcqs $\Z[\tfrac{1}{m}]$-schemes $X$, satisfying the following properties: \begin{enumerate}[label=(\roman*)] \item for every noetherian scheme $X$ of finite dimension, there exists an integer $d \in \Z$ such that for every integer $i \in \Z$, the spectrum $\emph{Fil}^i F(X;\Q)$ is in cohomological degrees at most $-i+d$; \item for any integers $i \in \Z$ and $m \geq 2$, and every qcqs $\Z[\tfrac{1}{m}]$-scheme $X$, the induced automorphism on the $i^{\emph{th}}$ graded piece $\emph{gr}^i F(X)$ is multiplication by $m^i$; \item for any integers $m, m' \geq 2$, and every qcqs $\Z[\tfrac{1}{mm'}]$-scheme $X$, the space of natural transformations from $\psi^m \circ \psi^{m'}$ to $\psi^{mm'}$, as endomorphisms of the filtered spectrum $\emph{Fil}^\star F(X)$, is contractible. \end{enumerate} Then for every qcqs scheme $X$, there exists a natural multiplicative equivalence of filtered spectra $$\emph{Fil}^\star F(X;\Q) \simeq \bigoplus_{j \geq \star} \emph{gr}^j F(X;\Q).$$ \end{proposition} \begin{proof} By finitariness, it suffices to prove the result on noetherian schemes of finite dimension. Hypothesis $(i)$ implies that the filtration $\text{Fil}^\star F(X;\Q)$ is complete on such schemes $X$. Lemma~\ref{lemma29howtouseAdamsoperations} and Remark~\ref{remarklemmahowtouseAdamsonclassicalschemes} then imply that there exist natural equivalences of spectra $$\text{Fil}^i F(X;\Q) \simeq \big(\oplus_{i \leq j < k} \text{gr}^j F(X;\Q)\big) \oplus \text{Fil}^k F(X;\Q)$$ for all integers $i,k \in \Z$ such that $k \geq i$. Again using completeness, taking the limit over $k \rightarrow +\infty$ induces a natural equivalence of spectra $$\text{Fil}^i F(X;\Q) \simeq \prod_{j \geq i} \text{gr}^j F(X;\Q).$$ Hypothesis $(i)$ then implies that, at each cohomological degree, only a finite number of terms in the previous product are nonzero. In particular, the previous equivalence can be rewritten as a natural equivalence of spectra $$\text{Fil}^i F(X;\Q) \simeq \bigoplus_{j \geq i} \text{gr}^j F(X;\Q),$$ which induces the desired multiplicative equivalence of filtered spectra. \end{proof} \begin{remark} Let $X$ be a qcqs derived scheme. The argument of Proposition~\ref{propositionhowtouseAdamsoperations}, where the necessary hypotheses are satisfied by Corollary~\ref{corollary31''AdamsoperationsongradedpiecesoffilteredTC} and Proposition~\ref{propositionfilteredTCandrationalfilteredTCarecomplete}, implies that there is a natural multiplicative equivalence of filtered spectra $$\text{Fil}^\star_{\text{mot}} \text{TC}(X;\Q) \simeq \prod_{j \geq \star} \Q(j)^{\text{TC}}(X)[2i].$$ \end{remark} \subsection{Rigid-analytic HKR filtrations}\label{sectionrigidanalyticdR} \vspace{-\parindent} \hspace{\parindent} In this subsection, we define variants, which we call rigid-analytic, of the HKR filtrations. We start by explaining the relevant objects at the level of Hochschild homology. For every qcqs derived scheme $X$, there is a natural $\text{S}^1$-equivariant arithmetic fracture square \begin{equation}\label{equationarithmeticfracturesquareforHH} \begin{tikzcd} \text{HH}(X) \ar[r] \ar[d] & \text{HH}(X_{\Q}/\Q) \ar[d] \\ \prod_{p \in \mathbb{P}} \text{HH}(X;\Z_p) \ar[r] & \prod'_{p \in \mathbb{P}} \text{HH}(X;\Q_p) \end{tikzcd} \end{equation} in the derived category $\mathcal{D}(\Z)$, where we use base change for Hochschild homology for the top right corner, and the convention adopted in the Notation part for the bottom left and bottom right corners. Applying homotopy fixed points $(-)^{h\text{S}^1}$ for the $\text{S}^1$-action induces a natural cartesian square $$\begin{tikzcd} \text{HC}^-(X) \ar[r] \ar[d] & \text{HC}^-(X_{\Q}/\Q) \ar[d] \\ \prod_{p \in \mathbb{P}} \text{HC}^-(X;\Z_p) \ar[r] & \big(\prod'_{p \in \mathbb{P}} \text{HH}(X;\Q_p)\big)^{h\text{S}^1} \end{tikzcd}$$ in the derived category $\mathcal{D}(\Z)$, where we use that taking homotopy fixed points $(-)^{h\text{S}^1}$ commutes with limits for the bottom left corner. We call {\it rigid-analytic variant of Hochschild homology and of negative cyclic homology} the bottom right corners of the previous two cartesian squares, respectively. This terminology should find some justification in Section~\ref{subsectionrigidanalyticGoodwillietheorem}. Following Section~\ref{subsectionHKRfiltrations}, we use the previous cartesian square to introduce a new HKR filtration on this rigid-analytic variant of negative cyclic homology $\big(\prod'_{p \in \mathbb{P}} \text{HH}(X;\Q_p)\big)^{h\text{S}^1}$. \begin{definition}[HKR filtration on rigid-analytic $\text{HC}^-$]\label{definition11HKRfiltrationonHC-solid} The {\it HKR filtration on rigid-analytic negative cyclic homology} of qcqs derived schemes is the functor $$\text{Fil}^\star_{\text{HKR}} \big(\prod_{p \in \mathbb{P}} {}^{'} \, \text{HH}(-;\Q_p)\big)^{h\text{S}^1} : \text{dSch}^{\text{qcqs,op}} \longrightarrow \text{FilSp}$$ defined by the cocartesian square $$\begin{tikzcd} \text{Fil}^\star_{\text{HKR}} \text{HC}^-(-) \ar[d] \ar[r] & \text{Fil}^\star_{\text{HKR}} \text{HC}^-(-_{\Q}/\Q) \ar[d] \\ \prod_{p \in \mathbb{P}} \text{Fil}^\star_{\text{HKR}} \text{HC}^-(-;\Z_p) \ar[r] & \text{Fil}^\star_{\text{HKR}} \big(\prod'_{p \in \mathbb{P}} \text{HH}(-;\Q_p)\big)^{h\text{S}^1}. \end{tikzcd}$$ \end{definition} \begin{remark}\label{remark12HKRfiltrationonHC-solid} Let $p$ be a prime number, and $X$ be a qcqs derived scheme over $\Z_{(p)}$. The natural map $$\prod_{\l \in \mathbb{P}} \text{Fil}^\star_{\text{HKR}} \text{HC}^-(X;\Z_{\l}) \longrightarrow \text{Fil}^\star_{\text{HKR}} \text{HC}^-(X;\Z_p)$$ is then an equivalence of filtered spectra, and we then denote by $\text{Fil}^\star_{\text{HKR}} \text{HH}(X;\Q_p)^{h\text{S}^1}$ the filtered spectrum $\text{Fil}^\star_{\text{HKR}} \prod'_{\l \in \mathbb{P}} \text{HH}(X;\Q_{\l})^{h\text{S}^1}$. In particular, the natural commutative diagram $$\begin{tikzcd} \text{Fil}^\star_{\text{HKR}} \text{HC}^-(X) \arrow{r} \arrow{d} & \text{Fil}^\star_{\text{HKR}} \text{HC}^-(X_{\Q}/\Q) \ar[d] \\ \text{Fil}^\star_{\text{HKR}} \text{HC}^-(X;\Z_p) \arrow{r} & \text{Fil}^\star_{\text{HKR}} \text{HH}(X;\Q_p)^{h\text{S}^1} \end{tikzcd}$$ is a cartesian square of filtered spectra. \end{remark} Similarly, one can apply homotopy orbits $(-)_{h\text{S}^1}$ for the $\text{S}^1$-action to the arithmetic fracture square for Hochschild homology (\ref{equationarithmeticfracturesquareforHH}). The functor $(-)_{h\text{S}^1}$ does not commute with limits, but the $\text{S}^1$-action on the product $\prod_{p \in \mathbb{P}} \text{HH}(X;\Z_p)$ is diagonal. Using the natural fibre sequence $$\text{HH}(X;\Z_p) \longrightarrow \text{HH}(X;\Z_p)_{h\text{S}^1} \longrightarrow \text{HH}(X;\Z_p)_{h\text{S}^1}[2]$$ in the derived category $\mathcal{D}(\Z)$ and the fact that the functors $\text{HH}(-;\Z_p)$ and $\text{HH}(-;\Z_p)_{h\text{S}^1}$ are in non-positive cohomological degrees on animated commutative rings, one can prove that the complex \hbox{$\text{HH}(X;\Z_p)_{h\text{S}^1} \in \mathcal{D}(\Z)$} is derived $p$-complete, hence the natural map $$\text{HC}(X;\Z_p) \longrightarrow \text{HH}(X;\Z_p)_{h\text{S}^1}$$ is an equivalence in the derived category $\mathcal{D}(\Z)$. In particular, applying homotopy orbits $(-)_{h\text{S}^1}$ to the arithmetic fracture square for Hochschild homology (\ref{equationarithmeticfracturesquareforHH}) induces a natural cartesian square $$\begin{tikzcd} \text{HC}(X) \ar[r] \ar[d] & \text{HC}(X_{\Q}/\Q) \ar[d] \\ \prod_{p \in \mathbb{P}} \text{HC}(X;\Z_p) \ar[r] & \big(\prod'_{p \in \mathbb{P}} \text{HH}(X;\Q_p)\big)_{h\text{S}^1} \end{tikzcd}$$ in the derived category $\mathcal{D}(\Z)$. We use this cartesian square to introduce the following HKR filtration on the bottom right corner. \begin{definition}[HKR filtration on rigid-analytic HC]\label{definition13HKRfiltrationonHCsolid} The {\it HKR filtration on rigid-analytic cyclic homology} of qcqs derived schemes is the functor $$\text{Fil}^\star_{\text{HKR}} \big(\prod_{p \in \mathbb{P}} {}^{'} \, \text{HH}(-;\Q_p)\big)_{h\text{S}^1} : \text{dSch}^{\text{qcqs,op}} \longrightarrow \text{FilSp}$$ defined by the cocartesian square $$\begin{tikzcd} \text{Fil}^\star_{\text{HKR}} \text{HC}(-) \ar[d] \ar[r] & \text{Fil}^\star_{\text{HKR}} \text{HC}(-_{\Q}/\Q) \ar[d] \\ \prod_{p \in \mathbb{P}} \text{Fil}^\star_{\text{HKR}} \text{HC}(-;\Z_p) \ar[r] & \text{Fil}^\star_{\text{HKR}} \big(\prod'_{p \in \mathbb{P}} \text{HH}(-;\Q_p)\big)_{h\text{S}^1}. \end{tikzcd}$$ \end{definition} \begin{remark}\label{remark15'HKRfiltrationonHCsolidallprimesprestrictedproduct} Taking homotopy orbits $(-)_{h\text{S}^1}$ commutes with colimits, so the natural map $$\text{Fil}^\star_{\text{HKR}} \text{HC}(X;\Q) \longrightarrow \text{Fil}^\star_{\text{HKR}} \text{HC}(X_{\Q}/\Q)$$ is an equivalence in the filtered derived category $\mathcal{DF}(\Q)$. Upon applying rationalisation to the natural cartesian square $$\begin{tikzcd} \text{Fil}^\star_{\text{HKR}} \text{HC}(X) \arrow{r} \arrow{d} & \text{Fil}^\star_{\text{HKR}} \text{HC}(X_{\Q}/\Q) \ar[d] \\ \prod_{p \in \mathbb{P}} \text{Fil}^\star_{\text{HKR}} \text{HC}(X;\Z_p) \arrow{r} & \text{Fil}^\star_{\text{HKR}} \big(\prod'_{p \in \mathbb{P}} \text{HH}(X;\Q_p)\big)_{h\text{S}^1}, \end{tikzcd}$$ this implies that the natural map $$\prod_{p \in \mathbb{P}} {}^{'} \, \text{Fil}^\star_{\text{HKR}} \text{HC}(X;\Q_p) := \big(\prod_{p \in \mathbb{P}} \text{Fil}^\star_{\text{HKR}} \text{HC}(X;\Z_p)\big)_{\Q} \longrightarrow \text{Fil}^\star_{\text{HKR}} \big(\prod_{p \in \mathbb{P}} {}^{'} \, \text{HH}(X;\Q_p)\big)_{h\text{S}^1}$$ is an equivalence in the filtered derived category $\mathcal{DF}(\Q)$. \end{remark} \begin{remark}\label{remark15''HKRfiltrationonHCsolid} Let $p$ be a prime number, and $X$ be a qcqs derived scheme over $\Z_{(p)}$. As in Remark~\ref{remark12HKRfiltrationonHC-solid}, we denote the filtered complex $\text{Fil}^\star_{\text{HKR}} \big(\prod'_{\l \in \mathbb{P}} \text{HH}(X;\Q_{\l})\big)_{h\text{S}^1}$ by $$\text{Fil}^\star_{\text{HKR}} \text{HH}(X;\Q_p)_{h\text{S}^1} \in \mathcal{DF}(\Q).$$ In particular, the natural map $$\text{Fil}^\star_{\text{HKR}} \text{HC}(X;\Q_p) \longrightarrow \text{Fil}^\star_{\text{HKR}} \text{HH}(X;\Q_p)_{h\text{S}^1}$$ is an equivalence in the filtered derived category $\mathcal{DF}(\Q)$ by Remark~\ref{remark15'HKRfiltrationonHCsolidallprimesprestrictedproduct}. \end{remark} The Tate construction $(-)^{t\text{S}^1}$ is by definition the cofibre of the norm map $$(-)_{h\text{S}^1}[1] \rightarrow (-)^{h\text{S}^1}.$$ Applying the Tate construction $(-)^{t\text{S}^1}$ to the arithmetic fracture square for Hochschild homology (\ref{equationarithmeticfracturesquareforHH}) then induces a natural cartesian square $$\begin{tikzcd} \text{HP}(X) \ar[r] \ar[d] & \text{HP}(X_{\Q}/\Q) \ar[d] \\ \prod_{p \in \mathbb{P}} \text{HP}(X;\Z_p) \ar[r] & \big(\prod'_{p \in \mathbb{P}} \text{HH}(X;\Q_p)\big)^{t\text{S}^1} \end{tikzcd}$$ in the derived category $\mathcal{D}(\Z)$, where we use the analogous cartesian squares for $\text{HC}^-$ and $\text{HC}$ to identify the bottom left corner. \begin{definition}[HKR filtration on rigid-analytic HP]\label{definition13HKRfiltrationonHPsolid} The {\it HKR filtration on rigid-analytic periodic cyclic homology} of qcqs derived schemes is the functor $$\text{Fil}^\star_{\text{HKR}} \big(\prod_{p \in \mathbb{P}} {}^{'} \, \text{HH}(-;\Q_p)\big)^{t\text{S}^1} : \text{dSch}^{\text{qcqs,op}} \longrightarrow \text{FilSp}$$ defined by the cocartesian square $$\begin{tikzcd} \text{Fil}^\star_{\text{HKR}} \text{HP}(-) \ar[d] \ar[r] & \text{Fil}^\star_{\text{HKR}} \text{HP}(-_{\Q}/\Q) \ar[d] \\ \prod_{p \in \mathbb{P}} \text{Fil}^\star_{\text{HKR}} \text{HP}(-;\Z_p) \ar[r] & \text{Fil}^\star_{\text{HKR}} \big(\prod'_{p \in \mathbb{P}} \text{HH}(-;\Q_p)\big)^{t\text{S}^1}. \end{tikzcd}$$ \end{definition} \begin{remark}\label{remark14HKRfiltrationonHPsolid} Let $p$ be a prime number, and $X$ be a qcqs derived scheme over $\Z_{(p)}$. As in Remarks~\ref{remark12HKRfiltrationonHC-solid} and \ref{remark15''HKRfiltrationonHCsolid}, we denote the filtered complex $\text{Fil}^\star_{\text{HKR}} \big(\prod_{\l \in \mathbb{P}} \text{HH}(X;\Q_{\l})\big)^{t\text{S}^1}$ by $$\text{Fil}^\star_{\text{HKR}} \text{HH}(X;\Q_p)^{t\text{S}^1} \in \mathcal{DF}(\Q).$$ In particular, the natural commutative diagram $$\begin{tikzcd} \text{Fil}^\star_{\text{HKR}} \text{HP}(X) \arrow{r} \arrow{d} & \text{Fil}^\star_{\text{HKR}} \text{HP}(X_{\Q}/\Q) \ar[d] \\ \text{Fil}^\star_{\text{HKR}} \text{HP}(X;\Z_p) \arrow{r} & \text{Fil}^\star_{\text{HKR}}\text{HH}(X;\Q_p)^{t\text{S}^1} \end{tikzcd}$$ is a cartesian square of filtered spectra. \end{remark} \begin{lemma}\label{lemma9reductionfromHC-HPtoHC-HPsolid} Let $X$ be a qcqs derived scheme. Then the natural commutative diagram $$\begin{tikzcd} \prod'_{p \in \mathbb{P}} \emph{Fil}^\star_{\emph{HKR}} \emph{HC}^-(X;\Q_p) \arrow{r} \arrow{d} & \emph{Fil}^\star_{\emph{HKR}} \big(\prod'_{p \in \mathbb{P}} \emph{HH}(X;\Q_p)\big)^{h\emph{S}^1} \ar[d] \\ \prod'_{p \in \mathbb{P}} \emph{Fil}^\star_{\emph{HKR}} \emph{HP}(X;\Q_p) \arrow{r} & \emph{Fil}^\star_{\emph{HKR}} \big(\prod'_{p \in \mathbb{P}} \emph{HH}(X;\Q_p)\big)^{t\emph{S}^1} \end{tikzcd}$$ is a cartesian square of filtered spectra. \end{lemma} \begin{proof} There is a natural commutative diagram of filtered spectra $$\hspace*{-.5cm}\begin{tikzcd}[sep=tiny] \prod'_{p \in \mathbb{P}} \text{Fil}^\star_{\text{HKR}} \text{HC}^-(X;\Q_p) \arrow{r} \arrow{d} & \prod'_{p \in \mathbb{P}} \text{Fil}^\star_{\text{HKR}} \text{HP}(X;\Q_p) \ar[d] \ar[r] & \prod'_{p \in \mathbb{P}} \text{Fil}^{\star-1}_{\text{HKR}} \text{HC}(X;\Q_p)[2] \arrow{d} \\ \text{Fil}^\star_{\text{HKR}}\big(\prod'_{p \in \mathbb{P}} \text{HH}(X;\Q_p)\big)^{h\text{S}^1} \arrow{r} & \text{Fil}^\star_{\text{HKR}} \big(\prod'_{p \in \mathbb{P}} \text{HH}(X;\Q_p)\big)^{t\text{S}^1} \ar[r] & \text{Fil}^{\star-1}_{\text{HKR}} \big(\prod'_{p \in \mathbb{P}} \text{HH}(X;\Q_p)\big)_{h\text{S}^1}[2] \end{tikzcd}$$ where, by definition of the bottom terms (Definitions~\ref{definition11HKRfiltrationonHC-solid}, \ref{definition13HKRfiltrationonHPsolid}, and \ref{definition13HKRfiltrationonHCsolid}), the horizontal lines are fibre sequences. In this diagram, the right vertical map is an equivalence (Remark~\ref{remark15'HKRfiltrationonHCsolidallprimesprestrictedproduct}), so the left square is a cartesian square. \end{proof} \begin{lemma}\label{lemmaHKRfiltrationHC-solidproductallprimesiscomplete} Let $X$ be a qcqs derived scheme. Then the filtrations $$\prod_{p \in \mathbb{P}}{}^{'} \big(\emph{Fil}^\star_{\emph{HKR}} \emph{HH}(X;\Q_p)\big)^{h\emph{S}^1}, \text{ } \prod_{p \in \mathbb{P}}{}^{'} \big(\emph{Fil}^\star_{\emph{HKR}} \emph{HH}(X;\Q_p)\big)_{h\emph{S}^1}, \text{ and } \prod_{p \in \mathbb{P}}{}^{'} \big(\emph{Fil}^\star_{\emph{HKR}} \emph{HH}(X;\Q_p)\big)^{t\emph{S}^1}$$ are complete. \end{lemma} \begin{proof} The HKR filtrations on $\text{HC}^-(X)$, $\text{HC}(X)$, and $\text{HP}(X)$ are complete by Lemma~\ref{lemmaHKRfiltrationonHC-isalwayscomplete}. The product on prime numbers $p$ of the $p$-completions of these filtrations are also complete, since $p$-completions and products commute with limits. Similarly, the HKR filtrations on $\text{HC}^-(X_{\Q}/\Q)$, $\text{HC}(X_{\Q}/\Q)$, and $\text{HP}(X_{\Q}/\Q)$ are complete by \cite[Theorem~$1.1$]{antieau_periodic_2019}. The rigid-analytic HKR filtrations on $\text{HC}^-$, $\text{HC}$, and $\text{HP}$ are thus also complete, as pushouts of three complete filtrations. \end{proof} We now describe the graded pieces of these rigid-analytic HKR filtrations, by analogy with the classical HKR filtrations. \begin{definition}[Rigid-analytic Hodge-complete derived de Rham cohomology]\label{definition16rigidanalyticHodgecompletederiveddeRhamHC-} For every integer $i \in \Z$, the functor $$R\Gamma_{\text{Zar}}\Big(-,\prod_{p \in \mathbb{P}} {}^{'} \,\widehat{\underline{\mathbb{L}\Omega}}^{\geq i}_{-_{\Q_p}/\Q_p}\Big) : \text{dSch}^{\text{qcqs,op}} \longrightarrow \mathcal{D}(\Q)$$ is defined as the shifted graded piece of the HKR filtration on $\prod'_{p \in \mathbb{P}} \text{HH}(-;\Q_p)^{h\text{S}^1}$: $$R\Gamma_{\text{Zar}}\Big(-,\prod_{p \in \mathbb{P}} {}^{'} \,\widehat{\underline{\mathbb{L}\Omega}}^{\geq i}_{-_{\Q_p}/\Q_p}\Big) := \text{gr}^i_{\text{HKR}} \big(\prod_{p \in \mathbb{P}} {}^{'} \, \text{HH}(-;\Q_p)\big)^{h\text{S}^1}[-2i].$$ \end{definition} \begin{remark}\label{remark18cartesiansquaredefiningsolidderiveddeRhamcohomology} Let $X$ be a qcqs derived scheme, and $i \in \Z$ be an integer. By Definition~\ref{definition11HKRfiltrationonHC-solid}, there is a natural cartesian square $$\begin{tikzcd} R\Gamma_{\text{Zar}}\Big(X,\widehat{\mathbb{L}\Omega}^{\geq i}_{-/\Z}\Big) \arrow{r} \arrow{d} & R\Gamma_{\text{Zar}}\Big(X,\widehat{\mathbb{L}\Omega}^{\geq i}_{-_{\Q}/\Q}\Big) \ar[d] \\ R\Gamma_{\text{Zar}}\Big(X,\prod_{p \in \mathbb{P}} \big(\widehat{\mathbb{L}\Omega}^{\geq i}_{-/\Z}\big)^\wedge_p\Big) \arrow{r} & R\Gamma_{\text{Zar}}\Big(X,\prod'_{p \in \mathbb{P}} \,\widehat{\underline{\mathbb{L}\Omega}}^{\geq i}_{-_{\Q_p}/\Q_p}\Big) \end{tikzcd}$$ in the derived category $\mathcal{D}(\Z)$, which can serve as an alternative definition for the bottom right term. \end{remark} \begin{remark}\label{remark17fibreseuquenceforsolidderiveddeRhamcohomology} Let $X$ be a qcqs derived scheme, and $i \in \Z$ be an integer. The complexes $$R\Gamma_{\text{Zar}}\Big(X,\prod_{p \in \mathbb{P}}{}^{'} \,\widehat{\underline{\mathbb{L}\Omega}}_{-_{\Q_p}/\Q_p}\Big) \quad \text{and} \quad R\Gamma_{\text{Zar}}\Big(X,\prod_{p \in \mathbb{P}}{}^{'} \,\underline{\mathbb{L}\Omega}^{\leq i}_{-_{\Q_p}/\Q_p}\Big)$$ are defined as in Definition~\ref{definition16rigidanalyticHodgecompletederiveddeRhamHC-}, where $(-)^{h\text{S}^1}$ is replaced by $(-)^{t\text{S}^1}$ and $(-)_{h\text{S}^1}$ respectively. In particular, the natural fibre sequence $$\text{Fil}^\star_{\text{HKR}} \big(\prod_{p \in \mathbb{P}} {}^{'} \, \text{HH}(X;\Q_p)\big)^{h\text{S}^1} \rightarrow \text{Fil}^\star_{\text{HKR}} \big(\prod_{p \in \mathbb{P}} {}^{'} \, \text{HH}(X;\Q_p)\big)^{t\text{S}^1} \rightarrow \text{Fil}^{\star-1}_{\text{HKR}} \big(\prod_{p \in \mathbb{P}} {}^{'} \, \text{HH}(X;\Q_p)\big)_{h\text{S}^1}[2]$$ induces a natural fibre sequence $$R\Gamma_{\text{Zar}}\Big(X,\prod_{p \in \mathbb{P}}{}^{'} \,\widehat{\underline{\mathbb{L}\Omega}}^{\geq i}_{-_{\Q_p}/\Q_p}\Big) \longrightarrow R\Gamma_{\text{Zar}}\Big(X,\prod_{p \in \mathbb{P}}{}^{'} \,\widehat{\underline{\mathbb{L}\Omega}}_{-_{\Q_p}/\Q_p}\Big) \longrightarrow R\Gamma_{\text{Zar}}\Big(X,\prod_{p \in \mathbb{P}}{}^{'} \,\underline{\mathbb{L}\Omega}^{<i}_{-_{\Q_p}/\Q_p}\Big),$$ where the right term is naturally identified with the complex $$\Big(\prod_{p \in \mathbb{P}} R\Gamma_{\text{Zar}}\big(X,\big(L\Omega^{<i}_{-/\Z}\big)^\wedge_p\big)\Big)_{\Q} \in \mathcal{D}(\Q)$$ by Remark~\ref{remark15'HKRfiltrationonHCsolidallprimesprestrictedproduct}. \end{remark} \begin{remark}\label{remark17''solidderiveddeRhamcohomologyoneprimeatatime} Let $p$ be a prime number, $X$ be a qcqs derived scheme over $\Z_{(p)}$, and $i \in \Z$ be an integer. Following Remarks~\ref{remark12HKRfiltrationonHC-solid}, \ref{remark15''HKRfiltrationonHCsolid}, and~\ref{remark14HKRfiltrationonHPsolid}, we denote the complexes $$R\Gamma_{\text{Zar}}\Big(X,\prod_{\l \in \mathbb{P}}{}^{'} \widehat{\underline{\mathbb{L}\Omega}}^{\geq i}_{-_{\Q_{\l}}/\Q_{\l}}\Big), \text{ } R\Gamma_{\text{Zar}}\Big(X,\prod_{\l \in \mathbb{P}}{}^{'} \widehat{\underline{\mathbb{L}\Omega}}_{-_{\Q_{\l}}/\Q_{\l}}\Big), \text{ and } R\Gamma_{\text{Zar}}\Big(X,\prod_{\l \in \mathbb{P}}{}^{'} \underline{\mathbb{L}\Omega}^{< i}_{-_{\Q_{\l}}/\Q_{\l}}\Big)$$ by $$R\Gamma_{\text{Zar}}\big(X,\widehat{\underline{\mathbb{L}\Omega}}^{\geq i}_{-_{\Q_p}/\Q_p}\big), \text{ } R\Gamma_{\text{Zar}}\big(X,\widehat{\underline{\mathbb{L}\Omega}}_{-_{\Q_p}/\Q_p}\big), \text{ and } R\Gamma_{\text{Zar}}\big(X,\underline{\mathbb{L}\Omega}^{< i}_{-_{\Q_p}/\Q_p}\big)$$ respectively. In particular, by Remark~\ref{remark17fibreseuquenceforsolidderiveddeRhamcohomology}, there is a natural fibre sequence $$R\Gamma_{\text{Zar}}\big(X,\widehat{\underline{\mathbb{L}\Omega}}^{\geq i}_{-_{\Q_p}/\Q_p}\big) \longrightarrow R\Gamma_{\text{Zar}}\big(X,\widehat{\underline{\mathbb{L}\Omega}}_{-_{\Q_p}/\Q_p}\big) \longrightarrow R\Gamma_{\text{Zar}}\big(X,\underline{\mathbb{L}\Omega}^{< i}_{-_{\Q_p}/\Q_p}\big)$$ in the derived category $\mathcal{D}(\Q_p)$, where the right term is naturally identified with the complex $$R\Gamma_{\text{Zar}}\big(X,\big(\mathbb{L}\Omega^{<i}_{-/\Z}\big)^\wedge_p[\tfrac{1}{p}]\big) \in \mathcal{D}(\Q_p).$$ \end{remark} In the following result, we reformulate the motivic filtration on topological cyclic homology of Definition~\ref{definitionmotivicfiltrationonTC} in terms of the rigid-analytic HKR filtration on negative cyclic homology. This can be interpreted as a filtered arithmetic fracture square for topological cyclic homology. \begin{proposition}\label{propositionmainconsequenceBFSwithfiltrations} Let $X$ be qcqs derived scheme. Then the natural commutative diagram $$\begin{tikzcd} \emph{Fil}^\star_{\emph{mot}} \emph{TC}(X) \arrow{r} \arrow{d} & \emph{Fil}^\star_{\emph{HKR}} \emph{HC}^-(X_{\Q}/\Q) \ar[d] \\ \prod_{p \in \mathbb{P}} \emph{Fil}^\star_{\emph{BMS}} \emph{TC}(X;\Z_p) \arrow{r} & \emph{Fil}^\star_{\emph{HKR}} \Big(\prod'_{p \in \mathbb{P}} \emph{HH}(X;\Q_p)\Big)^{h\emph{S}^1} \end{tikzcd}$$ is a cartesian square of filtered spectra. \end{proposition} \begin{proof} This is a consequence of Definitions~\ref{definitionmotivicfiltrationonTC} and~\ref{definition11HKRfiltrationonHC-solid}. \end{proof} \begin{corollary}\label{corollaryusefulBFSafterLcdh} Let $X$ be a qcqs scheme. Then the natural commutative diagram $$\begin{tikzcd} \big(L_{\emph{cdh}}\, \emph{Fil}^\star_{\emph{mot}} \emph{TC}(-)\big)(X) \arrow{r} \arrow{d} & \big(L_{\emph{cdh}}\, \emph{Fil}^\star_{\emph{HKR}} \emph{HC}^-(-_{\Q}/\Q)\big)(X) \ar[d] \\ \Big(L_{\emph{cdh}} \prod_{p \in \mathbb{P}} \emph{Fil}^\star_{\emph{BMS}} \emph{TC}(-;\Z_p)\Big)(X) \arrow{r} & \Big(L_{\emph{cdh}}\, \emph{Fil}^\star_{\emph{HKR}} \big(\prod'_{p \in \mathbb{P}} \emph{HH}(-;\Q_p)\big)^{h\emph{S}^1}\Big)(X), \end{tikzcd}$$ is a cartesian square of filtered spectra. \end{corollary} \begin{proof} This is a consequence of Proposition~\ref{propositionmainconsequenceBFSwithfiltrations}, which we restrict to qcqs schemes and then sheafify for the cdh topology. \end{proof} \begin{corollary}\label{corollarymainconsequenceBFSongradedpieces} Let $X$ be a qcqs derived scheme. Then for every integer $i \in \Z$, the natural commutative diagram $$\begin{tikzcd} \Z(i)^{\emph{TC}}(X) \arrow{r} \arrow{d} & R\Gamma_{\emph{Zar}}\Big(X,\widehat{\mathbb{L}\Omega}^{\geq i}_{-_{\Q}/\Q}\Big) \ar[d] \\ \prod_{p \in \mathbb{P}} \Z_p(i)^{\emph{BMS}}(X) \arrow{r} & R\Gamma_{\emph{Zar}}\Big(X,\prod'_{p \in \mathbb{P}} \underline{\widehat{\mathbb{L}\Omega}}^{\geq i}_{-_{\Q_p}/\Q_p}\Big), \end{tikzcd}$$ is a cartesian square in the derived category $\mathcal{D}(\Z)$. \end{corollary} \begin{proof} This is a direct consequence of Proposition~\ref{propositionmainconsequenceBFSwithfiltrations}. \end{proof} \subsection{A rigid-analytic Goodwillie theorem}\label{subsectionrigidanalyticGoodwillietheorem} \vspace{-\parindent} \hspace{\parindent} In this subsection, we prove that the rigid-analytic version of periodic cyclic homology is a truncating invariant (Theorem~\ref{theoremtruncatinginvariantHPsolid}). We first recall the definition of Hochschild, cyclic, negative cyclic, and periodic cyclic homologies of a general cyclic object. \begin{definition}[Cyclic object]\label{definitioncyclicobject} Let $\mathcal{C}$ be a category, or an $\infty$-category. The {\it cyclic category} $\Lambda$ is the category with objects $[n]$ indexed by non-negative integers $n$, and morphisms $[m] \rightarrow [n]$ given by homotopy classes of degree one increasing maps from $\text{S}^1$ to itself that map the subgroup $\Z/(m+1)$ to $\Z/(n+1)$. A {\it cyclic object} in $\mathcal{C}$ is then a contravariant functor from the cyclic category $\Lambda$ to $\mathcal{C}$. \end{definition} \begin{notation}[Hochschild homology of a cyclic object]\label{notationmapSforcyclicobjects} Let $\mathcal{A}$ be an abelian category with exact infinite products, and~$X_{\bullet}$ be a cyclic object in $\mathcal{A}$. Following \cite[Section~II]{goodwillie_cyclic_1985} (see also \cite[Section~$2.2$]{morrow_aws_2018}), one can define the Hochschild homology $\text{HH}(X_{\bullet})$ of $X_{\bullet}$, as an object of the derived category $\mathcal{D}(\mathcal{A})$ equipped with a natural $\text{S}^1$-action. The cyclic, negative cyclic, and periodic cyclic homologies of the cyclic object $X_{\bullet}$ are then defined by $$\text{HC}(X_{\bullet}) := \text{HH}(X_{\bullet})_{h\text{S}^1}, \text{ } \text{HC}^-(X_{\bullet}) := \text{HH}(X_{\bullet})^{h\text{S}^1}, \text{ and } \text{HP}(X_{\bullet}) := \text{HH}(X_{\bullet})^{t\text{S}^1}.$$ In this context, there is moreover a natural map $$s : \text{HC}(X_\bullet)[-2] \longrightarrow \text{HC}(X_\bullet)$$ in the derived category $\mathcal{D}(\mathcal{A})$, from which periodic cyclic homology $\text{HP}(X_\bullet) \in \mathcal{D}(\mathcal{A})$ can be recovered by the formula $$\text{HP}(X_\bullet) \simeq \lim\limits_{\longleftarrow} \Big(\cdots \xlongrightarrow{s} \text{HC}(X_\bullet)[-4] \xlongrightarrow{s} \text{HC}(X_\bullet)[-2] \xlongrightarrow{s} \text{HC}(X_\bullet)\Big).$$ \end{notation} We now apply the previous general construction to define Hochschild homology and its variants on solid associative derived algebras over a solid commutative ring. \begin{definition}[Solid Hochschild homology]\label{definitionHHofsolid} Let $k$ be a solid commutative ring, and $R$ be a connective solid $k$-$\E_1$-algebra. The simplicial object $$\cdots \hspace{1mm} \substack{\longrightarrow\\[-0.9em] \longrightarrow\\[-0.9em] \longrightarrow\\[-0.9em] \longrightarrow} \hspace{1mm} R \otimes_{k} R \otimes_{k} R \hspace{1mm} \substack{\longrightarrow\\[-0.9em] \longrightarrow\\[-0.9em] \longrightarrow} \hspace{1mm} R\otimes_{k} R \hspace{1mm} \substack{\longrightarrow\\[-0.9em] \longrightarrow} \hspace{1mm} R$$ has a natural structure of cyclic object in the derived $\infty$-category of solid $k$-modules (Definition~\ref{definitioncyclicobject}), induced by permutation of the tensor factors. We write $\underline{\text{HH}}(R/k)$, $\underline{\text{HC}}(R/k)$, $\underline{\text{HC}^-}(R/k)$ and $\underline{\text{HP}}(R/k)$ for the Hochschild, cyclic, negative cyclic and periodic cyclic homologies of this cyclic object (Notation~\ref{notationmapSforcyclicobjects}). If $k=\Z$, we simply denote these by $\underline{\text{HH}}(R)$, $\underline{\text{HC}}(R)$, $\underline{\text{HC}^-}(R)$ and $\underline{\text{HP}}(R)$. \end{definition} For $f : R \rightarrow R'$ a map of $\Z$-$\E_1$-algebras and $F : \E_1\text{-Alg}_{\Z} \rightarrow \mathcal{D}(\Z)$ a functor, we denote by $F(f) \in \mathcal{D}(\Z)$ the cofibre of the map $F(R) \rightarrow F(R')$. More generally, for $f : R \rightarrow R'$ a map of solid $\Z$-$\mathbb{E}_1$-algebras and $F$ a $\mathcal{D}(\text{Solid})$-valued functor on solid $\Z$-$\mathbb{E}_1$-algebras, denote by $F(f) \in \mathcal{D}(\text{Solid})$ the cofibre of the map $F(R) \rightarrow F(R')$. The following result is \cite[Theorem~IV.$2.6$]{goodwillie_cyclic_1985}. More precisely, this result of Goodwillie is for maps of simplicial $\Z$-algebras, and the underlying $\infty$-category of simplicial $\Z$-algebras is naturally identified with the $\infty$-category of connective $\Z$-$\mathbb{E}_1$-algebras, by the monoidal Dold--Kan correspondence and \cite[Proposition~$7.1.4.6$]{lurie_higher_2017}. \begin{theorem}[\cite{goodwillie_cyclic_1985}]\label{theoremGoodwillieIV26} Let $f : R \rightarrow R'$ be a $1$-connected map of connective $\Z$-$\E_1$-algebras. For every integer $n \geq 0$, the natural map $$n!s^n : \emph{HC}_{\ast+2n}(f) \longrightarrow \emph{HC}_\ast(f)$$ is the zero map for $\ast \leq n-1$. \end{theorem} Goodwillie's proof of Theorem~\ref{theoremGoodwillieIV26}, although stated with respect to the abelian category of $\Z$\nobreakdash-modules (or $k$-modules, for $k$ an arbitrary discrete commutative ring), is valid for any abelian symmetric monoidal category with exact infinite products (see Notation~\ref{notationmapSforcyclicobjects} and Definition~\ref{definitionHHofsolid}). One can thus prove the following generalisation of the previous result. \begin{theorem}\label{theoremGoodwillieIV26solid} Let $f : R \rightarrow R'$ be a $1$-connected map of connective solid $\Z$-$\E_1$-algebras. Then for every integer $n \geq 0$, the natural map $$n!s^n : \underline{\emph{HC}}(f)[-2n] \longrightarrow \underline{\emph{HC}}(f),$$ in the derived category $\mathcal{D}(\emph{Solid})$, is the zero map on cohomology groups\footnote{By this, we mean that it is the zero map as a map of solid abelian groups, {\it i.e.}, that it factors through the zero object of the abelian category of solid abelian groups. Note that the underlying abelian group of a nonzero solid abelian group can be zero ({\it e.g.}, the quotient of $\Z_p$ with the $p$-adic topology by $\Z_p$ with the discrete topology). In particular, being zero for a map of solid abelian groups cannot be detected on the underlying map of abelian groups.} in degrees at least $-n+1$. \end{theorem} \begin{proof} To prove the result for simplicial solid $\Z$-algebras, it suffices to prove that the abelian category of solid abelian groups is symmetric monoidal, and has exact infinite products. The first claim is \cite[Theorem~$6.2$\,(i)]{clausen_condensed_2019}. The second claim is a consequence of the fact that the abelian category of condensed abelian groups has exact arbitrary products (\cite[Theorem~$1.10$]{clausen_condensed_2019}), and that the category of solid abelian groups, as a subcategory of the abelian category of condensed abelian groups, is stable under all limits (\cite[Theorem~$5.8\,(i)$]{clausen_condensed_2019}). We omit the proof of the analogue of \cite[Proposition~$7.1.4.6$]{lurie_higher_2017} to pass from simplicial solid $\Z$-algebras to connective solid $\Z$-$\mathbb{E}_1$-algebras. \end{proof} For the rest of this section, and following the convention of condensed mathematics, a condensed object is called {\it discrete} if its condensed structure is trivial. Given a classical object $X$, we denote by $\underline{X}$ the associated discrete condensed object. \begin{lemma}\label{lemmasolidHHisHHpcompleted} Let $R$ be a connective $\Z$-$\mathbb{E}_1$-algebra, and $p$ be a prime number. Then the natural map $$\underline{\emph{HH}(R)} \longrightarrow \underline{\emph{HH}}(\underline{R}^\wedge_p),$$ in the derived category $\mathcal{D}(\emph{Solid})$, exhibits the target as the $p$-completion of the source. In particular, there is a natural equivalence $$\underline{\emph{HH}(R)}^\wedge_p[\tfrac{1}{p}] \xlongrightarrow{\sim} \underline{\emph{HH}}(\underline{R}^\wedge_p[\tfrac{1}{p}]/\Q_p)$$ in the derived category $\mathcal{D}(\emph{Solid})$. \end{lemma} \begin{proof} The solid tensor product of $p$-complete solid connective $\Z$-$\mathbb{E}_1$-algebras is $p$-complete, and so is the totalisation of a complex of $p$-complete solid connective $\Z$-modules (\cite{clausen_witt_2021}). In particular, the complex $\underline{\text{HH}}(\underline{R}^\wedge_p)$, as a totalisation of tensor powers of the $p$-complete solid connective $\Z$-$\mathbb{E}_1$-algebra~$\underline{R}^\wedge_p$, is $p$-complete. By the derived Nakayama lemma (\cite[091N]{stacks_project_authors_stacks_2019}, see also \cite{clausen_witt_2021}), it thus suffices to prove the first statement modulo $p$. By base change for Hochschild homology (resp. solid Hochschild homology), this is equivalent to proving that the natural map $$\underline{\text{HH}((R/p)/\F_p)} \longrightarrow \underline{\text{HH}}((\underline{R}/p)/\underline{\F_p})$$ is an equivalence in the derived category $\mathcal{D}(\text{Solid})$. The desired equivalence is then a consequence of the fact that reduction modulo $p$ and tensor products commute with the functor~$\underline{(-)}$ from $\Z$-$\mathbb{E}_1$-algebras to solid $\Z$-$\mathbb{E}_1$-algebras. The second statement follows from the fact that rationalisation commutes with the functor $$\underline{(-)} : \mathcal{D}(\Z) \longrightarrow \mathcal{D}(\text{Solid}),$$ and base change for solid Hochschild homology. \end{proof} \begin{proposition}\label{propositionsolidHCisHCpcompleted} Let $R$ be a connective $\Z$-$\mathbb{E}_1$-algebra, and $p$ be a prime number. Then the natural map $$\underline{\emph{HC}(R)} \longrightarrow \underline{\emph{HC}}(\underline{R}^\wedge_p),$$ in the derived category $\mathcal{D}(\emph{Solid})$, exhibits the target as the $p$-completion of the source. In particular, there is a natural equivalence $$\underline{\emph{HC}(R)}^\wedge_p[\tfrac{1}{p}] \xlongrightarrow{\sim} \underline{\emph{HC}}(\underline{R}^\wedge_p[\tfrac{1}{p}]/\Q_p)$$ in the derived category $\mathcal{D}(\emph{Solid})$. \end{proposition} \begin{proof} The first statement is a consequence of Lemma~\ref{lemmasolidHHisHHpcompleted}, and the description \cite[Definition~$2.19$]{morrow_aws_2018} of (solid) cyclic homology in terms of the cyclic object $\text{HH}(R)$ (resp. $\underline{\text{HH}}(\underline{R}^\wedge_p)$) in the stable $\infty$-category $\mathcal{D}(\Z)$ (resp. $\mathcal{D}(\text{Solid})$). The second statement follows from the fact that rationalisation commutes direct sums (or equivalently, with the functor $(-)_{h\text{S}^1}$) and with the functor $\underline{(-)}$ from the derived category $\mathcal{D}(\Z)$ to the derived category $\mathcal{D}(\text{Solid})$. \end{proof} \begin{corollary}\label{corollaryadaptationoftheoremGoodwillieIV26} Let $f : R \rightarrow R'$ a $1$-connected map of connective $\Z$-$\E_1$-algebras. Then for every integer $n \geq 0$, the map $$n! s^n : \prod_{p \in \mathbb{P}} {}^{'} \, \emph{HC}(f;\Q_p)[-2n] \longrightarrow \prod_{p \in \mathbb{P}} {}^{'} \, \emph{HC}(f;\Q_p),$$ in the derived category $\mathcal{D}(\Q)$, is the zero map on cohomology groups in degrees at least $-n+1$. \end{corollary} \begin{proof} For every prime number $p$, the natural map $$n! s^n : \underline{\text{HC}}(\underline{f}^\wedge_p)[-2n] \longrightarrow \underline{\text{HC}}(\underline{f}^\wedge_p),$$ in the derived category $\mathcal{D}(\text{Solid})$, is zero in cohomological degrees at least $-n+1$ (Theorem~\ref{theoremGoodwillieIV26solid}). Forgetting the condensed structure and taking the product over all primes $p$, this implies that the natural map $$n! s^n : \prod_{p \in \mathbb{P}} \text{HC}(f;\Z_p)[-2n] \longrightarrow \prod_{p \in \mathbb{P}} \text{HC}(f;\Z_p),$$ in the derived category $\mathcal{D}(\Z)$, is zero in cohomological degrees at least $-n+1$ (Proposition~\ref{propositionsolidHCisHCpcompleted}). Taking rationalisation then implies the desired result. \end{proof} \begin{theorem}[Rigid-analytic HP is truncating]\label{theoremtruncatinginvariantHPsolid} The construction $$R \longmapsto \Big(\prod_{p \in \mathbb{P}} {}^{'} \, \emph{HH}(R;\Q_p)\Big)^{t\emph{S}^1} := \Big(\Big(\prod_{p \in \mathbb{P}} \emph{HH}(R;\Z_p)\Big)_{\Q}\Big)^{t\emph{S}^1},$$ from connective $\Z$-$\E_1$-algebras $R$ to the derived category $\mathcal{D}(\Q)$, is truncating. More precisely, there exists a truncating invariant \hbox{$E : \emph{Cat}^{\emph{perf}}_{\infty} \rightarrow \mathcal{D}(\Q)$} such that the previous construction is the composite $R \mapsto \emph{Perf}(R) \mapsto E(\emph{Perf}(R))$. \end{theorem} \begin{proof} Let $f : R \rightarrow R'$ be a $1$-connected map of connective $\Z$-$\E_1$-algebras. We want to prove that the natural map $$\Big(\prod_{p \in \mathbb{P}} {}^{'} \, \text{HH}(R;\Q_p)\Big)^{t\text{S}^1} \longrightarrow \Big(\prod_{p \in \mathbb{P}} {}^{'} \, \text{HH}(R';\Q_p)\Big)^{t\text{S}^1}$$ is an equivalence, or equivalently that its homotopy cofibre $\big(\prod'_{p \in \mathbb{P}} \text{HH}(f;\Q_p)\big)^{t\text{S}^1}$ vanishes in the derived category $\mathcal{D}(\Q)$. For every integer $n \geq 0$, the natural map $$n!s^n : \prod_{p \in \mathbb{P}} {}^{'} \, \text{HC}(f;\Q_p)[-2n] \longrightarrow \prod_{p \in \mathbb{P}} {}^{'} \, \text{HC}(f;\Q_p)$$ is the zero map in cohomological degrees at least $-n+1$ (Corollary~\ref{corollaryadaptationoftheoremGoodwillieIV26}). This map is $\Q$-linear, so the map $$s^n : \prod_{p \in \mathbb{P}} {}^{'} \, \text{HC}(f;\Q_p)[-2n] \longrightarrow \prod_{p \in \mathbb{P}} {}^{'} \, \text{HC}(f;\Q_p)$$ is also the zero map in cohomological degrees at least $-n+1$. Taking the inverse limit over integers $n \geq 0$ and using the equivalence at the end of Notation~\ref{notationmapSforcyclicobjects} then implies that the complex $\big(\prod'_{p \in \mathbb{P}}\text{HH}(f;\Q_p)\big)^{t\text{S}^1}$ is zero in the derived category $\mathcal{D}(\Q)$. \end{proof} \subsection{Rigid-analytic derived de Rham cohomology is a cdh sheaf} \vspace{-\parindent} \hspace{\parindent} The aim of this section is to prove the cdh descent result Corollary~\ref{corollaryHPsolidallprimespisacdhsheafwithfiltration}, which is a rigid-analytic analogue of the following result, used in \cite{elmanto_motivic_2023} to prove Theorem~\ref{theoremrationalstructuremain} in characteristic zero. \begin{proposition}[\cite{cortinas_cyclic_2008,bals_periodic_2023}]\label{propositionfilteredHPisacdhsheafinchar0} For every integer $i \in \Z$, the presheaf $$\emph{Fil}^i_{\emph{HKR}} \emph{HP}(-_{\Q}/\Q) : \emph{Sch}^{\emph{qcqs,op}} \longrightarrow \mathcal{D}(\Q)$$ is a cdh sheaf. \end{proposition} \begin{proof} This is a consequence of \cite[Corollary~A.$6$]{land_k-theory_2019} and \cite[Theorem~$1.3$]{bals_periodic_2023}. \end{proof} We first extract the following cdh descent result from Theorem~\ref{theoremtruncatinginvariantHPsolid}. Note that this argument uses the theory of truncating invariants, as developed by Land--Tamme \cite{land_k-theory_2019}, in a crucial way. \begin{corollary}\label{corollaryHPsolidallprimespisacdhsheafwithoutfiltration} The presheaf $$\Big(\prod_{p \in \mathbb{P}} {}^{'} \, \emph{HH}(-_;\Q_p)\Big)^{t\emph{S}^1} : \emph{Sch}^{\emph{qcqs,op}} \longrightarrow \mathcal{D}(\Q)$$ is a cdh sheaf. \end{corollary} \begin{proof} This is a consequence of Theorem~\ref{theoremtruncatinginvariantHPsolid} and \cite[Theorem~E]{land_k-theory_2019}. \end{proof} We then adapt the main splitting result of \cite{bals_periodic_2023} on periodic cyclic homology over $\Q$ to our rigid-analytic setting. \begin{proposition}\label{propositionsplittingofHPsolid} Let $X$ be a qcqs derived scheme. Then there exists a natural equivalence $$\Big( \prod_{p \in \mathbb{P}} {}^{'} \, \emph{HH}(X;\Q_p)\Big)^{t\emph{S}^1} \simeq \prod_{i \in \Z} R\Gamma_{\emph{Zar}}\Big(X,\prod_{p \in \mathbb{P}} {}^{'} \, \underline{\widehat{\mathbb{L}\Omega}}_{-_{\Q_p}/\Q_p}\Big)[2i]$$ in the derived category $\mathcal{D}(\Q)$. \end{proposition} \begin{proof} We adapt the proof of \cite[Theorem~$3.4$]{bals_periodic_2023}, which proves a similar decomposition for periodic cyclic homology over $\Q$. The crucial point to adapt this proof is to note that there is a natural equivalence $$\prod_{p \in \mathbb{P}} {}^{'} \, \text{HH}(X;\Q_p) \simeq \bigoplus_{i \in \N} R\Gamma_{\text{Zar}}\Big(X,\big(\prod_{p \in \mathbb{P}} (\mathbb{L}^i_{-/\Z})^\wedge_p\big)_{\Q}\Big)[i]$$ in the derived category $\mathcal{D}(\Q)$ (see \cite[Remark~$2.8$]{bals_periodic_2023}). This is the rigid-analytic analogue of \cite[Proposition~$2.7$]{bals_periodic_2023}, and it is for instance a consequence of Lemma~\ref{lemma29howtouseAdamsoperations} applied to the $\N$-indexed filtration $\text{Fil}^\star_{\text{HKR}} \prod'_{p \in \mathbb{P}} \text{HH}(-;\Q_p)$. By \cite[Theorem~$3.2$]{bals_periodic_2023}, this implies that there is a natural equivalence $$\Big(\text{Fil}^\star_{\text{H}} R\Gamma_{\text{Zar}}\big(X,\prod_{p \in \mathbb{P}} {}^{'} \underline{\widehat{\mathbb{L}\Omega}}_{-_{\Q_p}/\Q_p}\big) \otimes \text{Fil}^\star_{\text{T}} \big(\prod_{p \in \mathbb{P}} {}^{'} \Q_p\big)^{t\text{S}^1}\Big)^\wedge \xlongrightarrow{\sim} \text{Fil}^\star_{\text{T}} \Big(\prod_{p \in \mathbb{P}} {}^{'} \, \text{HH}(X;\Q_p)\Big)^{t\text{S}^1}$$ in the filtered derived category $\mathcal{DF}(\Q)$, where the tensor product $\otimes$ is the Day convolution tensor product, $\text{Fil}^\star_{\text{H}}$ is the Hodge filtration on derived de Rham cohomology, and $(-)^\wedge$ is the completion with respect to the associated filtration. By a degree argument explained in \cite[proof of Theorem~$3.4$]{bals_periodic_2023}, the filtered object $\text{Fil}^\star_{\text{T}} \big(\prod'_{p \in \mathbb{P}} \Q_p\big)^{t\text{S}^1}$ carries a canonical splitting, which induces an equivalence $$\prod_{i \in \Z} \Big(\text{Fil}^{\star+i}_{\text{H}} R\Gamma_{\text{Zar}}\big(X,\prod_{p \in \mathbb{P}} {}^{'} \underline{\widehat{\mathbb{L}\Omega}}_{-_{\Q_p}/\Q_p}\big)\Big)[2i] \xlongrightarrow{\sim} \text{Fil}^\star_{\text{T}} \Big(\prod_{p \in \mathbb{P}} {}^{'} \, \text{HH}(X;\Q_p)\Big)^{t\text{S}^1}$$ in the filtered derived category $\mathcal{DF}(\Q)$. It then suffices to prove that the desired result is indeed obtained by taking the colimit over $\star \rightarrow -\infty$ of the previous equivalence. Following \cite[proof of Theorem~$3.4$]{bals_periodic_2023}, the result for the source is a formal consequence of the connectivity estimate for the functor $$R\Gamma_{\text{Zar}}\Big(-,\prod_{p \in \mathbb{P}}{}^{'}\underline{\mathbb{L}\Omega}^{\leq i}_{-_{\Q_p}/\Q_p}\Big)$$ on animated commutative ring, which is itself a consequence of Remark~\ref{remark15'HKRfiltrationonHCsolidallprimesprestrictedproduct}, and of the classical connectivity estimate for the HKR filtration on cyclic homology (Proposition~\ref{propositionHKRfiltrationonHCrationalisfinitary}). The result for the target is a consequence of the fact that the Tate filtration is always exhaustive (\cite[Proposition~B.$6$]{bals_periodic_2023}). \end{proof} \begin{corollary}\label{corollaryHPsolidallprimespisacdhsheafongradedpieces} The presheaf $$R\Gamma_{\emph{Zar}}\Big(-,\prod_{p \in \mathbb{P}} {}^{'} \, \underline{\widehat{\mathbb{L}\Omega}}_{-_{\Q_p}/\Q_p}\Big) : \emph{Sch}^{\emph{qcqs,op}} \longrightarrow \mathcal{D}(\Q)$$ is a cdh sheaf. \end{corollary} \begin{proof} This is a consequence of the natural splitting Proposition~\ref{propositionsplittingofHPsolid}, and of the cdh descent result Corollary~\ref{corollaryHPsolidallprimespisacdhsheafwithoutfiltration}. \end{proof} \begin{corollary}\label{corollaryHPsolidallprimespisacdhsheafwithfiltration} For every integer $i \in \Z$, the presheaf $$\emph{Fil}^i_{\emph{HKR}} \Big(\prod_{p \in \mathbb{P}} {}^{'} \, \emph{HH}(-_;\Q_p)\Big)^{t\emph{S}^1} : \emph{Sch}^{\emph{qcqs,op}} \longrightarrow \mathcal{D}(\Q)$$ is a cdh sheaf. \end{corollary} \begin{proof} The HKR filtration on $\big(\prod'_{p \in \mathbb{P}} \text{HH}(-;\Q_p)\big)^{t\text{S}^1}$ is complete by Lemma~\ref{lemmaHKRfiltrationHC-solidproductallprimesiscomplete}, so it suffices to prove the result on graded pieces. The result is then Corollary~\ref{corollaryHPsolidallprimespisacdhsheafongradedpieces}. \end{proof} \subsection{The Atiyah--Hirzebruch spectral sequence}\label{subsectionAHSS} \vspace{-\parindent} \hspace{\parindent} In this subsection, we use the results of the previous sections to prove Theorem~\ref{theoremintroAHSSandAdams}. \begin{proposition}[The motivic filtration is $\N$-indexed]\label{propositionmotivicfiltrationisexhaustive} Let $X$ be a qcqs derived scheme. Then for every integer $i \leq 0$, the natural map $$\emph{Fil}^i_{\emph{mot}} \emph{K}(X) \longrightarrow \emph{K}(X)$$ is an equivalence of spectra. In particular, the motivic filtration $\emph{Fil}^\star_{\emph{mot}} \emph{K}(X)$ on $\emph{K}(X)$ is exhaustive. \end{proposition} \begin{proof} First assume that $X$ is a qcqs classical scheme. The filtration $\text{Fil}^\star_{\text{cdh}} \text{KH}(X)$ is $\N$-indexed by \cite{bachmann_A^1-invariant_2024}, so it suffices to prove that the filtration $$\text{cofib}\Big(\text{Fil}^\star_{\text{mot}} \text{TC}(X) \longrightarrow \big(L_{\text{cdh}} \text{Fil}^\star_{\text{mot}} \text{TC}\big)(X)\Big)$$ is $\N$-indexed (Definition~\ref{definitionmotivicfiltrationonKtheoryofschemes}). To prove this, we use Proposition~\ref{propositionmainconsequenceBFSwithfiltrations} and Corollary~\ref{corollaryusefulBFSafterLcdh}. For every prime number $p$, the filtration $\text{Fil}^\star_{\text{BMS}} \text{TC}(-;\Z_p)$ is $\N$-indexed on qcqs schemes (\cite[Theorem~$7.2\,(1)$]{bhatt_topological_2019} and Theorem~\ref{theoremBMSfiltrationonTCpcompletedsatisfiesquasisyntomicdescentandisLKEfrompolynomialalgebras}\,$(2)$). In particular, the filtration $$\text{cofib}\Big(\prod_{p \in \mathbb{P}} \text{Fil}^\star_{\text{BMS}} \text{TC}(X;\Z_p) \longrightarrow \big(L_{\text{cdh}} \prod_{p \in \mathbb{P}} \text{Fil}^\star_{\text{BMS}} \text{TC}(-;\Z_p)\big)(X)\Big)$$ is $\N$-indexed. The presheaf $\text{Fil}^\star_{\text{HKR}} \text{HP}(-_{\Q}/\Q)$ is a cdh sheaf on qcqs schemes (Proposition~\ref{propositionfilteredHPisacdhsheafinchar0}), and the filtration $\text{Fil}^\star_{\text{HKR}} \text{HC}(X_{\Q}/\Q)$ is $\N$-indexed by construction. In particular, the filtration $$\text{cofib}\Big(\text{Fil}^\star_{\text{HKR}} \text{HC}^-(X_{\Q}/\Q) \longrightarrow \big(L_{\text{cdh}} \text{Fil}^\star_{\text{HKR}} \text{HC}^-(-_{\Q}/\Q)\big)(X)\Big)$$ is naturally identified with the filtration $$\text{cofib}\Big(\text{Fil}^{\star-1}_{\text{HKR}} \text{HC}(X_{\Q}/\Q) \longrightarrow \big(L_{\text{cdh}} \text{Fil}^{\star-1}_{\text{HKR}} \text{HC}(-_{\Q}/\Q)\big)(X)\Big)[1],$$ which is $\N$-indexed. Similarly, the presheaf $\text{Fil}^\star_{\text{HKR}} \big(\prod'_{p \in \mathbb{P}} \text{HH}(-;\Q_p)\big)^{t\text{S}^1}$ is a cdh sheaf on qcqs schemes (Corollary~\ref{corollaryHPsolidallprimespisacdhsheafwithfiltration}), and the filtration $\text{Fil}^\star_{\text{HKR}} \big(\prod'_{p \in \mathbb{P}} \text{HH}(-;\Q_p)\big)_{h\text{S}^1}$ is $\N$-indexed (Remark~\ref{remark15'HKRfiltrationonHCsolidallprimesprestrictedproduct}). In particular, the filtration $$\text{cofib}\Big(\text{Fil}^\star_{\text{HKR}} \big(\prod_{p \in \mathbb{P}}{}^{'} \text{HH}(X;\Q_p)\big)^{h\text{S}^1} \longrightarrow \big(L_{\text{cdh}} \text{Fil}^\star_{\text{HKR}} \big(\prod_{p \in \mathbb{P}}{}^{'} \text{HH}(-;\Q_p)\big)^{t\text{S}^1}\big)(X)\Big)$$ is naturally identified with the filtration $$\text{cofib}\Big(\text{Fil}^{\star-1}_{\text{HKR}} \big(\prod_{p \in \mathbb{P}}{}^{'} \text{HH}(X;\Q_p)\big)_{h\text{S}^1} \longrightarrow \big(L_{\text{cdh}} \text{Fil}^{\star-1}_{\text{HKR}} \big(\prod_{p \in \mathbb{P}}{}^{'} \text{HH}(-;\Q_p)\big)_{h\text{S}^1}\big)(X)\Big)[1],$$ which is $\N$-indexed. Assume now that $X$ is a general qcqs derived scheme. By Definition~\ref{definitionmotivicfiltrationonKtheoryofderivedschemes} and the previous paragraph, it suffices to prove that the filtration $$\text{cofib}\Big(\text{Fil}^\star_{\text{mot}} \text{TC}(X) \longrightarrow \text{Fil}^\star_{\text{mot}} \text{TC}(\pi_0(X))\Big)$$ is $\N$-indexed. To prove this, we use Proposition~\ref{propositionmainconsequenceBFSwithfiltrations}. For every prime number $p$, the filtration $\text{Fil}^\star_{\text{BMS}} \text{TC}(X;\Z_p)$ is $\N$-indexed on polynomial $\Z$-algebras by the previous paragraph, and hence on general qcqs derived schemes by Zariski descent and Theorem~\ref{theoremBMSfiltrationonTCpcompletedsatisfiesquasisyntomicdescentandisLKEfrompolynomialalgebras}\,$(2)$. In particular, the filtration $$\text{cofib}\Big(\prod_{p \in \mathbb{P}} \text{Fil}^\star_{\text{BMS}} \text{TC}(X;\Z_p) \longrightarrow \prod_{p \in \mathbb{P}} \text{Fil}^\star_{\text{BMS}} \text{TC}(\pi_0(X);\Z_p)\Big)$$ is $\N$-indexed. The filtration $$\text{cofib}\Big(\text{Fil}^\star_{\text{HKR}} \text{HC}^-(X_{\Q}/\Q) \longrightarrow \text{Fil}^\star_{\text{HKR}} \text{HC}^-(\pi_0(X)_{\Q}/\Q)\Big)$$ is $\N$-indexed by \cite[Theorem~$4.39$]{elmanto_motivic_2023}. Similarly, using Theorem~\ref{theoremtruncatinginvariantHPsolid} and Proposition~\ref{propositionsplittingofHPsolid} as in the proof of Corollary~\ref{corollaryHPsolidallprimespisacdhsheafwithfiltration}, the natural map $$\text{Fil}^\star_{\text{HKR}} \Big(\prod_{p \in \mathbb{P}}{}^{'} \text{HH}(X;\Q_p)\Big)^{t\text{S}^1} \longrightarrow \text{Fil}^\star_{\text{HKR}} \Big(\prod_{p \in \mathbb{P}}{}^{'} \text{HH}(\pi_0(X);\Q_p)\Big)^{t\text{S}^1}$$ is an equivalence of filtered spectra. In particular, the filtration $$\text{cofib}\Big(\text{Fil}^\star_{\text{HKR}} \Big(\prod_{p \in \mathbb{P}}{}^{'} \text{HH}(X;\Q_p)\Big)^{h\text{S}^1} \longrightarrow \text{Fil}^\star_{\text{HKR}} \Big(\prod_{p \in \mathbb{P}}{}^{'} \text{HH}(\pi_0(X);\Q_p)\Big)^{h\text{S}^1}\Big)$$ is naturally identified with the filtration $$\text{cofib}\Big(\text{Fil}^\star_{\text{HKR}} \Big(\prod_{p \in \mathbb{P}}{}^{'} \text{HH}(X;\Q_p)\Big)_{h\text{S}^1} \longrightarrow \text{Fil}^\star_{\text{HKR}} \Big(\prod_{p \in \mathbb{P}}{}^{'} \text{HH}(\pi_0(X);\Q_p)\Big)_{h\text{S}^1}\Big)[1],$$ which is $\N$-indexed by Remark~\ref{remark15'HKRfiltrationonHCsolidallprimesprestrictedproduct}. \end{proof} \begin{corollary}\label{corollarymotiviccomplexiszerofornegativeweight} Let $X$ be a qcqs scheme. Then for every integer $i<0$, the motivic complex $\Z(i)^{\emph{mot}}(X) \in \mathcal{D}(X)$ is zero. \end{corollary} \begin{proof} This is a direct consequence of Proposition~\ref{propositionmotivicfiltrationisexhaustive}. \end{proof} \begin{proposition}\label{propositionfibrefilteredTCandLcdhTCiscomplete} Let $d \geq 0$ be an integer, and $X$ be a qcqs scheme of valuative dimension at most~$d$. Then for every integer $i \in \Z$, the fibre of the natural map of spectra $$\emph{Fil}^i_{\emph{mot}} \emph{TC}(X) \longrightarrow \emph{Fil}^i_{\emph{mot}} L_{\emph{cdh}} \emph{TC}(X)$$ is in cohomological degrees at most $-i+d+2$. In particular, the filtration formed by these spectra for all integers $i \in \Z$, and the rationalisation of this filtration, are complete. \end{proposition} \begin{proof} The last statement is a consequence of the first, as the connectivity bound for a given filtration induces the same connectivity bound for its rationalisation. For the connectivity bound, we use Proposition~\ref{propositionmainconsequenceBFSwithfiltrations} and Corollary~\ref{corollaryusefulBFSafterLcdh} to compare the spectra $\text{Fil}^i_{\text{mot}} \text{TC}(X)$ and $\text{Fil}^i_{\text{mot}} L_{\text{cdh}} \text{TC}(X)$. The presheaf $\prod_{p \in \mathbb{P}} \text{Fil}^i_{\text{BMS}} \text{TC}(X;\Z_p)$ takes values in cohomological degrees at most $-i+1$ on affine schemes (Lemma~\ref{lemmaBMSfiltrationproductallprimesiscomplete}). In particular, the fibre of the natural map of spectra $$\prod_{p \in \mathbb{P}} \text{Fil}^i_{\text{BMS}} \text{TC}(X;\Z_p) \longrightarrow \big(L_{\text{cdh}} \prod_{p \in \mathbb{P}} \text{Fil}^i_{\text{BMS}} \text{TC}(-;\Z_p)\big)(X)$$ is in cohomological degrees at most $-i+d+2$, as each term is in cohomological degrees at most $-i+d+1$ (\cite[Theorem~$3.12$]{clausen_hyperdescent_2021} and \cite[Theorem~$2.4.15$]{elmanto_cdh_2021}). By Proposition~\ref{propositionfilteredHPisacdhsheafinchar0}, the fibre of the natural map of spectra $$\text{Fil}^i_{\text{HKR}} \text{HC}^-(X_{\Q}/\Q) \longrightarrow \big(L_{\text{cdh}} \text{Fil}^i_{\text{HKR}} \text{HC}^-(-_{\Q}/\Q)\big)(X)$$ is naturally identified with the fibre of the natural map of spectra $$\text{Fil}^{i-1}_{\text{HKR}} \text{HC}(X_{\Q}/\Q)[1] \longrightarrow \big(L_{\text{cdh}} \text{Fil}^{i-1}_{\text{HKR}} \text{HC}(-_{\Q}/\Q)[1]\big)(X).$$ The presheaf $\text{Fil}^{i-1}_{\text{HKR}} \text{HC}(-_{\Q}/\Q)$ takes values in cohomological degrees at most $-i-1$ ({\it e.g.},~by Lemma~\ref{lemmaHKRfiltrationonHC-isalwayscomplete} and the description of the graded pieces Remark~\ref{remarkgradedpiecesoftheHKRfiltrations}), so the previous fibre is in cohomological degrees at most $-i+d-1$ (\cite[Theorem~$3.12$]{clausen_hyperdescent_2021} and \cite[Theorem~$2.4.15$]{elmanto_cdh_2021}). Similarly, by Corollary~\ref{corollaryHPsolidallprimespisacdhsheafwithfiltration}, the fibre of the natural map of spectra $$\text{Fil}^i_{\text{HKR}}\big(\prod_{p \in \mathbb{P}}{}^{'} \text{HH}(X;\Q_p)\big)^{h\text{S}^1} \longrightarrow \Big(L_{\text{cdh}} \text{Fil}^i_{\text{HKR}} \big(\prod_{p \in \mathbb{P}}{}^{'} \text{HH}(-;\Q_p)\big)^{h\text{S}^1}\Big)(X)$$ is naturally identified with the fibre of the natural map of spectra $$\text{Fil}^{i-1}_{\text{HKR}}\big(\prod_{p \in \mathbb{P}}{}^{'} \text{HH}(X;\Q_p)\big)_{h\text{S}^1}[1] \longrightarrow \Big(L_{\text{cdh}} \text{Fil}^{i-1}_{\text{HKR}} \big(\prod_{p \in \mathbb{P}}{}^{'} \text{HH}(-;\Q_p)\big)_{h\text{S}^1}[1]\Big)(X)$$ The presheaf $\text{Fil}^{i-1}_{\text{HKR}}\big(\prod'_{p \in \mathbb{P}} \text{HH}(-;\Q_p)\big)_{h\text{S}^1}$ takes values in cohomological degrees at most $-i-1$ (Remark~\ref{remark15'HKRfiltrationonHCsolidallprimesprestrictedproduct}), so the previous fibre is in cohomological degrees at most $-i+d-1$ (\cite[Theorem~$3.12$]{clausen_hyperdescent_2021} and \cite[Theorem~$2.4.15$]{elmanto_cdh_2021}). The previous three connectivity results imply the desired result. \end{proof} \begin{proposition}[Completeness of the motivic filtration]\label{propositionmotivicfiltrationiscompleteonqcqsschemesoffinitevaluativedimension} Let $d \geq 0$ be an integer, and $X$~be a qcqs scheme of valuative dimension at most $d$. Then for every integer $i \in \Z$, the spectrum $\emph{Fil}^i_{\emph{mot}} \emph{K}(X)$ is in cohomological degrees at most $-i+d+2$. In particular, the motivic filtration $\emph{Fil}^\star_{\emph{mot}} \emph{K}(X)$, and its rationalisation $\emph{Fil}^\star_{\emph{mot}} \emph{K}(X;\Q)$, are complete. \end{proposition} \begin{proof} As in the proof of Proposition~\ref{propositionfibrefilteredTCandLcdhTCiscomplete}, the last statement is a consequence of the first. By Definition~\ref{definitionmotivicfiltrationonKtheoryofschemes}, there is a natural fibre sequence of spectra $$\text{fib}\Big(\text{Fil}^i_{\text{mot}}\text{TC}(X) \longrightarrow \text{Fil}^i_{\text{mot}} L_{\text{cdh}} \text{TC}(X)\Big) \longrightarrow \text{Fil}^i_{\text{mot}} \text{K}(X) \longrightarrow \text{Fil}^\star_{\text{cdh}} \text{KH}(X).$$ The left term of this fibre sequence is in cohomological degrees at most $-i+d+2$ by Proposition~\ref{propositionfibrefilteredTCandLcdhTCiscomplete}, and the right term is in cohomological degrees at most $-i+d$ by \cite{bachmann_A^1-invariant_2024}, hence the desired result. \end{proof} \begin{remark}[Non-noetherian Weibel vanishing] Let $X$ be a qcqs scheme of valuative dimension at most $d$. The same argument as in Proposition~\ref{propositionmotivicfiltrationiscompleteonqcqsschemesoffinitevaluativedimension} implies that the negative $K$-groups $\text{K}_{-n}(X)$ vanish for integers $n > d+2$. This is a weak form of Weibel vanishing in the non-noetherian case. \end{remark} \begin{remark}[Motivic Weibel vanishing]\label{remarkweakconnectivityformotiviccomplexes} Let $X$ be a qcqs scheme of dimension at most $d$. Proposition~\ref{propositionmotivicfiltrationiscompleteonqcqsschemesoffinitevaluativedimension} implies that for every integer $i \in \Z$, the motivic complex $\Z(i)^{\text{mot}}(X) \in \mathcal{D}(\Z)$ is in degrees at most~\hbox{$i+d+2$}. When $X$ is noetherian (in which case the valuative and Krull dimensions coincide), we will prove that it is even in degrees at most $i+d$ (Theorem~\ref{theoremmotivicWeibelvanishing}). \end{remark} \begin{remark}\label{remark28statementsongradedpiecesimplystatementonfiltration} A map of finitary presheaves of filtered spectra on qcqs schemes, which are filtration-complete on finite-dimensional noetherian schemes, is an equivalence if and only if it is an equivalence on graded pieces. In light of Propositions~\ref{propositionCmotivicfiltrationfinitaryNisnevichsheaf} and~\ref{propositionmotivicfiltrationiscompleteonqcqsschemesoffinitevaluativedimension}, we will formulate most of our results at the level of motivic cohomology, although they can often be promoted to results on the associated filtered spectra. \end{remark} \begin{corollary}[Completeness of $\text{Fil}^\star_{\text{mot}} L_{\text{cdh}} \text{TC}$] Let $X$ be a qcqs scheme of finite valuative dimension. Then the filtrations $\emph{Fil}^\star_{\emph{mot}} L_{\emph{cdh}} \emph{TC}(X)$ and $\emph{Fil}^\star_{\emph{mot}} L_{\emph{cdh}} \emph{TC}(X;\Q)$ are complete. \end{corollary} \begin{proof} This is a consequence of Propositions~\ref{propositionfilteredTCandrationalfilteredTCarecomplete} and \ref{propositionfibrefilteredTCandLcdhTCiscomplete}. \end{proof} \begin{remark} The filtrations $\text{Fil}^\star_{\text{mot}} \text{TC}(X)$ and $\text{Fil}^\star_{\text{mot}} L_{\text{cdh}} \text{TC}(X)$ do not satisfy separately a connectivity bound similar to that of Proposition~\ref{propositionfibrefilteredTCandLcdhTCiscomplete}. \end{remark} \begin{corollary}[Atiyah--Hirzebruch spectral sequence]\label{corollaryAHSS} Let $X$ be a qcqs derived scheme. The motivic filtration $\emph{Fil}^\star_{\emph{mot}} \emph{K}(X)$ on non-connective algebraic $K$-theory $\emph{K}(X)$ induces a natural Atiyah--Hirzebruch spectral sequence $$E^{i,j}_2 = \emph{H}^{i-j}_{\emph{mot}}(X,\Z(-j)) \Longrightarrow \emph{K}_{-i-j}(X).$$ If $X$ is a qcqs classical scheme of finite valuative dimension, then this Atiyah--Hirzebruch spectral sequence is convergent. \end{corollary} \begin{proof} The first statement is a consequence of the fact the motivic filtration $\text{Fil}^\star_{\text{mot}} \text{K}(X)$ is $\N$-indexed (Proposition~\ref{propositionmotivicfiltrationisexhaustive}). The second statement is a consequence of the connectivity bound for this motivic filtration (Proposition~\ref{propositionmotivicfiltrationiscompleteonqcqsschemesoffinitevaluativedimension}). \end{proof} The main consequence of Propositions~\ref{propositionmotivicfiltrationisexhaustive} and~\ref{propositionmotivicfiltrationiscompleteonqcqsschemesoffinitevaluativedimension} that we will use is the following result. \begin{corollary}\label{corollaryKtheorysplitsrationally} Let $X$ be a qcqs derived scheme. Then for every integer $i \geq 0$, there exists a natural equivalence of spectra $$\emph{K}(X;\Q) \simeq \big(\bigoplus_{0 \leq j < i} \Q(i)^{\emph{mot}}(X)[2i]\big) \oplus \emph{Fil}^i_{\emph{mot}} \emph{K}(X;\Q).$$ \end{corollary} \begin{proof} The motivic filtration $\text{Fil}^\star_{\text{mot}} \text{K}(X;\Q)$ is $\N$-indexed by Proposition~\ref{propositionmotivicfiltrationisexhaustive}. The result on qcqs classical schemes is then a consequence of Lemma~\ref{lemma29howtouseAdamsoperations} and Remark~\ref{remarklemmahowtouseAdamsonclassicalschemes}, where the necessary hypotheses are satisfied by Proposition~\ref{propositionmotivicfiltrationiscompleteonqcqsschemesoffinitevaluativedimension} and Corollary~\ref{corollary22''AdamsoperationsongradedpiecesoffilteredKtheory}. Assume now that $X$ is a general qcqs derived scheme. Again by Lemma~\ref{lemma29howtouseAdamsoperations}, where the necessary hypotheses are satisfied for the filtration $\text{Fil}^\star_{\text{mot}} \text{TC}(-)$ by Proposition~\ref{propositionfilteredTCandrationalfilteredTCarecomplete} and Corollary~\ref{corollary31''AdamsoperationsongradedpiecesoffilteredTC}, there is a natural equivalence of spectra $$\text{Fil}^0_{\text{mot}} \text{TC}(X;\Q) \simeq \big(\bigoplus_{0 \leq j < i} \Q(j)^{\text{TC}}(X)[2j]\big) \oplus \text{Fil}^i_{\text{mot}} \text{TC}(X;\Q).$$ The result is then a consequence of Definition~\ref{definitionmotivicfiltrationonKtheoryofderivedschemes} and the previous case, where the equivalences $$\text{Fil}^0_{\text{mot}} \text{K}(\pi_0(X);\Q) \simeq \bigoplus_{0 \leq j < i} \Q(j)^{\text{mot}}(\pi_0(X))[2j]\big) \oplus \text{Fil}^i_{\text{mot}} \text{K}(\pi_0(X);\Q)$$ and $$\text{Fil}^0_{\text{mot}} \text{TC}(\pi_0(X);\Q) \simeq \big(\bigoplus_{0 \leq j < i} \Q(j)^{\text{TC}}(\pi_0(X))[2j]\big) \oplus \text{Fil}^i_{\text{mot}} \text{TC}(X;\Q)$$ are compatible, by construction, with the natural map $$\text{Fil}^\star_{\text{mot}} \text{K}(\pi_0(X);\Q) \longrightarrow \text{Fil}^\star_{\text{mot}} \text{TC}(\pi_0(X);\Q)$$ of Definition~\ref{definitionmotivicfiltrationonKtheoryofschemes}. \end{proof} \begin{lemma}\label{lemmacdhsheafificationcommuteswithfilteredcolimits} Let $\tau$ be the Zariski, Nisnevich, or cdh topology, and $(F_i)_{i \in I}$ be a direct system of presheaves. Then the natural map of presheaves $$\lim\limits_{\longrightarrow i} L_{\tau}\, F_i \longrightarrow L_{\tau} \lim\limits_{\longrightarrow i} F_i$$ is an equivalence. In particular, the sheafification functor $L_{\tau}$ sends finitary presheaves to finitary $\tau$ sheaves. \end{lemma} \begin{proof} As a left adjoint, the sheafification functor $L_{\tau}$, from presheaves to $\tau$ sheaves, commutes with all colimits. Being a sheaf for the topology $\tau$ is detected using only finite limits, so the inclusion functor from $\tau$ sheaves to presheaves commutes with filtered colimits. Composing the previous two functors then implies that the functor $L_{\tau}$, from presheaves to presheaves, commutes with filtered colimits. To prove the second statement, let $F$ be a finitary presheaf, and $(R_i)_{i \in I}$ be a direct system of commutative rings. The fact that the natural map $$\lim\limits_{\longrightarrow i} L_{\tau}\, F(R_i) \longrightarrow L_{\tau} F(\lim\limits_{\longrightarrow i} R_i)$$ is an equivalence is a consequence of the finitariness of $F$, and of the first statement applied to the direct system of presheaves $\big(F(-_{R_i})\big)_{i \in I}$. \end{proof} \begin{proposition}[The motivic filtration is finitary]\label{propositionCmotivicfiltrationfinitaryNisnevichsheaf} Let $i \in \Z$ be an integer. Then the presheaf $$\emph{Fil}^i_{\emph{mot}} \emph{K}(-) : \emph{dSch}^{\emph{qcqs,op}} \longrightarrow \emph{Sp}$$ is a finitary Nisnevich sheaf, \emph{i.e.}, it satisfies descent for the Nisnevich topology and commutes with filtered colimits of rings. \end{proposition} \begin{proof} It suffices to prove the result modulo $p$ for every prime number $p$, and rationally. Algebraic $K$-theory is a finitary Nisnevich sheaf on qcqs derived schemes (\cite[Proposition~A.$15$]{clausen_descent_2020}). The presheaf $\text{Fil}^i_{\text{mot}} \text{K}(-;\Q)$ is a natural direct summand of rationalised algebraic $K$-theory (Corollary~\ref{corollaryKtheorysplitsrationally}), so it also is a finitary Nisnevich sheaf. To prove the result modulo a prime number $p$, note that the presheaf $\text{Fil}^i_{\text{cdh}} \text{KH}(-)$ is a finitary cdh sheaf (\cite{bachmann_A^1-invariant_2024}), and in particular a finitary Nisnevich sheaf. By Theorem~\ref{theoremBMSfiltrationonTCpcompletedsatisfiesquasisyntomicdescentandisLKEfrompolynomialalgebras}, the presheaf $\text{Fil}^i_{\text{BMS}} \text{TC}(-;\F_p)$ is a finitary Nisnevich sheaf. By Lemma~\ref{lemmacdhsheafificationcommuteswithfilteredcolimits}, this implies that the presheaf $L_{\text{cdh}} \text{Fil}^i_{\text{BMS}} \text{TC}(-;\F_p)$ is a finitary Nisnevich sheaf, and the result modulo $p$ is then a consequence of Proposition~\ref{propositionpadicstructuremain}. \end{proof} \begin{corollary}\label{corollarymotiviccomplexesarefinitary} Let $i \in \Z$ be an integer. Then the presheaf $$\Z(i)^{\emph{mot}}(-) : \emph{dSch}^{\emph{qcqs,op}} \longrightarrow \mathcal{D}(\Z)$$ is a finitary Nisnevich sheaf. \end{corollary} \begin{proof} This is a direct consequence of Proposition~\ref{propositionCmotivicfiltrationfinitaryNisnevichsheaf}. \end{proof} \begin{proof}[Proof of Theorem~\ref{theorem21'rationalmotivicfiltrationsplits}] The motivic filtration $\text{Fil}^\star_{\text{mot}} \text{K}(-)$ is finitary on qcqs schemes (Proposition~\ref{propositionCmotivicfiltrationfinitaryNisnevichsheaf}), and, for every noetherian scheme $X$ of finite dimension $d$ and every integer $i \in \Z$, the spectrum $\text{Fil}^i_{\text{mot}} \text{K}(X;\Q)$ is in cohomological degrees at most $-i+d+2$ (Proposition~\ref{propositionmotivicfiltrationiscompleteonqcqsschemesoffinitevaluativedimension}). Proposition~\ref{propositionhowtouseAdamsoperations} then implies that there exists a natural multiplicative equivalence of filtered spectra $$\text{Fil}^\star_{\text{mot}} \text{K}(X;\Q) \simeq \bigoplus_{j \geq \star} \Q(j)^{\text{mot}}(X)[2j].$$ The same argument as in Corollary~\ref{corollaryKtheorysplitsrationally} then implies the result for general qcqs derived schemes. \end{proof} \begin{corollary}\label{corollaryKtheorysplitsrationally2} Let $X$ be a qcqs derived scheme. Then there exists a natural equivalence of spectra $$\emph{K}(X;\Q) \simeq \bigoplus_{i \geq 0} \Q(i)^{\emph{mot}}(X)[2i].$$ \end{corollary} \begin{proof} The motivic filtration $\text{Fil}^\star_{\text{mot}} \text{K}(X;\Q)$ is $\N$-indexed by Proposition~\ref{propositionmotivicfiltrationisexhaustive}. The result is then a consequence of the rational splitting Theorem~\ref{theorem21'rationalmotivicfiltrationsplits}. \end{proof} \subsection{Rational structure of motivic cohomology}\label{subsectionrationalstructure} \vspace{-\parindent} \hspace{\parindent} In this subsection, we finish the proof of Theorem~\ref{theoremrationalstructuremain}. We first use an argument of Weibel to prove the following result at the level of $K$-theory. We then use the rational splitting Corollary~\ref{corollaryKtheorysplitsrationally} to prove a filtered version of this result, which reduces the proof of Theorem~\ref{theoremrationalstructuremain} to the case of characteristic zero. \begin{lemma}\label{lemma2WeibelargumentrationalKtheory} Let $X$ be a qcqs scheme. Then the natural commutative diagram $$\begin{tikzcd} \emph{K}(X;\Q) \arrow{r} \arrow{d} & \emph{K}(X_{\Q};\Q) \ar[d] \\ \emph{KH}(X;\Q) \arrow{r} & \emph{KH}(X_{\Q};\Q) \end{tikzcd}$$ is a cartesian square of spectra. \end{lemma} \begin{proof} By Zariski descent, it suffices to prove the result for affine schemes $X=\text{Spec}(R)$. For every integer $n \in \Z$, let $$\text{NK}_n(R) := \text{coker}\big(\text{K}_n(R) \longrightarrow \text{K}_n(R[T])\big)$$ be the $n^{\text{th}}$ $NK$-group of $R$. By \cite[Corollary~$6.4$]{weibel_meyer_1981} (see also \cite[Exercise~$9.12$]{thomason_higher_1990}, where some unnecessary hypotheses in Weibel's result are removed), there is a natural isomorphism of abelian groups $$\text{NK}_n(R)\otimes_{\Z} \Q \xlongrightarrow{\cong} \text{NK}_n(R\otimes_{\Z} \Q)$$ for every integer $n \in \Z$ and every commutative ring $R$. For every commutative ring $R$, the homotopy groups of the fibre $\text{K}^{\text{W}}(R)$ of the map of spectra $\text{K}(R) \rightarrow \text{KH}(R)$ have a natural exhaustive complete filtration with graded pieces given by the iterated $NK$-groups of $R$. In particular, for every integer $n \in \Z$ and every commutative ring $R$, the natural map $$\text{K}^{\text{W}}(R;\Q) \longrightarrow \text{K}^{\text{W}}(R\otimes_{\Z} \Q;\Q)$$ is an equivalence of spectra, which implies the desired result. \end{proof} \begin{corollary}\label{corollaryTCrationalwithcdhandHC-withoutfiltration} Let $X$ be a qcqs scheme. Then the natural commutative diagram $$\begin{tikzcd} \emph{K}(X;\Q) \arrow{r} \arrow{d} & \emph{HC}^-(X_{\Q}/\Q) \ar[d] \\ \emph{KH}(X;\Q) \arrow[r] & L_{\emph{cdh}} \emph{HC}^-(-_{\Q}/\Q)(X) \end{tikzcd}$$ is a cartesian square of spectra. \end{corollary} \begin{proof} The result for qcqs $\Q$-schemes $X$ is due to Corti\~{n}as--Haesemeyer--Schlichting--Weibel \cite{cortinas_cyclic_2008,cortinas_K-regularity_2008} (see also Theorem~\ref{theoremKST+LT}). The general result is then a consequence of Lemma~\ref{lemma2WeibelargumentrationalKtheory}. \end{proof} We record for completeness the following result, which is well-known in characteristic zero (\cite{cortinas_cyclic_2008,cortinas_K-regularity_2008}). \begin{corollary}\label{corollaryrationalKtheoryintermsofHC} Let $X$ be a qcqs scheme. Then there is a natural fibre sequence of spectra $$\emph{K}(X;\Q) \longrightarrow \emph{KH}(X;\Q) \longrightarrow \emph{cofib}\big(\emph{HC}(X_{\Q}/\Q) \longrightarrow L_{\emph{cdh}} \emph{HC}(-_{\Q}/\Q)(X)\big)[1].$$ \end{corollary} \begin{proof} By Theorem~\ref{theoremKST+LT} and Lemma~\ref{lemma2WeibelargumentrationalKtheory}, the natural commutative diagram $$\begin{tikzcd} \text{K}(X;\Q) \arrow{r} \arrow{d} & \text{HC}^-(X_{\Q}/\Q) \ar[d] \\ \text{KH}(X;\Q) \arrow{r} & L_{\text{cdh}} \text{HC}^-(-_{\Q}/\Q)(X) \end{tikzcd}$$ is a cartesian square of spectra. By construction, there is moreover a natural commutative diagram of spectra $$\begin{tikzcd} \text{HC}^-(X_{\Q}/\Q) \arrow{r} \arrow{d} & \text{HP}(X_{\Q}/\Q) \ar[d] \ar[r] & \text{HC}(X_{\Q}/\Q)[2] \ar[d] \\ L_{\text{cdh}} \text{HC}^-(-_{\Q}/\Q)(X) \arrow{r} & L_{\text{cdh}} \text{HP}(-_{\Q}/\Q)(X) \arrow{r} & L_{\text{cdh}} \text{HC}(-_{\Q}/\Q)(X)[2], \end{tikzcd}$$ where the horizontal lines are fibre sequences. The middle vertical line of this diagram is an equivalence (\cite[Corollary~$3.13$]{cortinas_cyclic_2008}, see also \cite[Corollary~A.$6$]{land_k-theory_2019}), so the cofibre of the left vertical map is naturally identified with the spectrum $$\text{cofib}\big(\text{HC}(X_{\Q}/\Q) \longrightarrow L_{\text{cdh}} \text{HC}(-_{\Q}/\Q)(X)\big)[1].$$ \end{proof} The following result is a filtered refinement of Lemma~\ref{lemma2WeibelargumentrationalKtheory}. \begin{corollary}\label{corollaryfilteredLemma2onKandKHwithrational} Let $X$ be a qcqs scheme. Then the natural commutative diagram $$\begin{tikzcd} \emph{Fil}^\star_{\emph{mot}} \emph{K}(X;\Q) \arrow{r} \arrow{d} & \emph{Fil}^\star_{\emph{mot}} \emph{K}(X_{\Q};\Q) \ar[d] \\ \emph{Fil}^\star_{\emph{cdh}} \emph{KH}(X;\Q) \arrow{r} & \emph{Fil}^\star_{\emph{cdh}} \emph{KH}(X_{\Q};\Q) \end{tikzcd}$$ is a cartesian square of filtered spectra. \end{corollary} \begin{proof} By Corollary~\ref{corollaryKtheorysplitsrationally}, for every integer $i \geq 0$, the spectrum $\text{Fil}^i_{\text{mot}} \text{K}(X;\Q)$ is naturally a direct summand of the spectrum $\text{K}(X;\Q)$. The same applies to the other three filtrations of the cartesian square, by noting that the motivic filtration $\text{Fil}^\star_{\text{cdh}} \text{KH}(-)$ also is $\N$-indexed (\cite{bachmann_A^1-invariant_2024}). The compatibility between the several filtrations is automatic from the construction of the splittings. So the result is a consequence of Lemma~\ref{lemma2WeibelargumentrationalKtheory}. \end{proof} The following result is a filtered refinement of Corollary~\ref{corollaryTCrationalwithcdhandHC-withoutfiltration}. \begin{theorem}\label{theoremTCrationalwithcdhandHC-filtered} Let $X$ be a qcqs scheme. Then the natural commutative diagram $$\begin{tikzcd} \emph{Fil}^\star_{\emph{mot}} \emph{K}(X;\Q) \arrow{r} \arrow{d} & \emph{Fil}^\star_{\emph{HKR}} \emph{HC}^-(X_{\Q}/\Q) \ar[d] \\ \emph{Fil}^\star_{\emph{cdh}} \emph{KH}(X;\Q) \arrow[r] & L_{\emph{cdh}} \emph{Fil}^\star_{\emph{HKR}} \emph{HC}^-(-_{\Q}/\Q)(X) \end{tikzcd}$$ is a cartesian square of filtered spectra. \end{theorem} \begin{proof} This is a consequence of Corollary~\ref{corollaryfilteredLemma2onKandKHwithrational}, where the filtration $\text{Fil}^\star_{\text{mot}} \text{TC}(-)$ of qcqs $\Q$-schemes is naturally identified with the filtration $\text{Fil}^\star_{\text{HKR}} \text{HC}^-(-/\Q)$ (Remark~\ref{remarkcomparisontoEMfiltrationonTCoverafield}). \end{proof} The following result is the rational part of Theorem~\ref{theoremintromaincartesiansquares}. \begin{corollary}\label{corollarycartesiansquarerational} Let $X$ be a qcqs scheme. Then for every integer $i \in \Z$, the natural commutative diagram $$\begin{tikzcd} \Q(i)^{\emph{mot}}(X) \ar[r] \ar[d] & R\Gamma_{\emph{Zar}}\big(X,\widehat{\mathbb{L}\Omega}^{\geq i}_{-_{\Q}/\Q}\big) \ar[d] \\ \Q(i)^{\emph{cdh}}(X) \ar[r] & R\Gamma_{\emph{cdh}}\big(X,\widehat{\mathbb{L}\Omega}^{\geq i}_{-_{\Q}/\Q}\big) \end{tikzcd}$$ is a cartesian square in the derived category $\mathcal{D}(\Q)$. \end{corollary} \begin{proof} This is a direct consequence of Theorem~\ref{theoremTCrationalwithcdhandHC-filtered}. \end{proof} \begin{proof}[Proof of Theorem~\ref{theoremrationalstructuremain}] By Theorem~\ref{theoremTCrationalwithcdhandHC-filtered}, the natural commutative diagram $$\begin{tikzcd} \text{Fil}^\star_{\text{mot}} \text{K}(X;\Q) \arrow{r} \arrow{d} & \text{Fil}^\star_{\text{HKR}} \text{HC}^-(X_{\Q}/\Q) \ar[d] \\ \text{Fil}^\star_{\text{cdh}} \text{KH}(X;\Q) \arrow{r} & L_{\text{cdh}} \text{Fil}^\star_{\text{HKR}} \text{HC}^-(-_{\Q}/\Q)(X) \end{tikzcd}$$ is a cartesian square of filtered spectra. By Definition~\ref{definitionHKRfiltrationonHC} and Remark~\ref{remarkHHandvariantsrelativetoQ}, there is a natural commutative diagram of filtered spectra $$\hspace*{-.25cm}\begin{tikzcd}[sep=small] \text{Fil}^\star_{\text{HKR}} \text{HC}^-(X_{\Q}/\Q) \arrow{r} \arrow{d} & \text{Fil}^\star_{\text{HKR}} \text{HP}(X_{\Q}/\Q) \ar[d] \ar[r] & \text{Fil}^{\star-1}_{\text{HKR}} \text{HC}(X_{\Q}/\Q)[2] \ar[d] \\ L_{\text{cdh}} \text{Fil}^\star_{\text{HKR}} \text{HC}^-(-_{\Q}/\Q)(X) \arrow{r} & L_{\text{cdh}} \text{Fil}^\star_{\text{HKR}} \text{HP}(-_{\Q}/\Q)(X) \arrow{r} & L_{\text{cdh}} \text{Fil}^{\star-1}_{\text{HKR}} \text{HC}(-_{\Q}/\Q)(X)[2] \end{tikzcd}$$ where the horizontal lines are fibre sequences. The middle vertical map of this diagram is an equivalence (Proposition~\ref{propositionfilteredHPisacdhsheafinchar0}), so the cofibre of the left vertical map is naturally identified with the filtered spectrum $$\text{cofib}\big(\text{Fil}^{\star-1}_{\text{HKR}} \text{HC}(X_{\Q}/\Q) \longrightarrow L_{\text{cdh}} \text{Fil}^{\star-1}_{\text{HKR}} \text{HC}(-_{\Q}/\Q)(X)\big) [1].$$ \end{proof} The following result was proved by Elmanto--Morrow \cite{elmanto_motivic_2023} for qcqs schemes $X$ over $\Q$. \begin{corollary}[Rational motivic cohomology]\label{corollaryrationalmainresultongradedpieces} Let $X$ be qcqs scheme. Then for every integer $i \in \Z$, there is a natural fibre sequence $$\Q(i)^{\emph{mot}}(X) \longrightarrow \Q(i)^{\emph{cdh}}(X) \longrightarrow \emph{cofib}\Big(R\Gamma_{\emph{Zar}}\big(X,\mathbb{L}\Omega^{<i}_{-_{\Q}/\Q}\big) \longrightarrow R\Gamma_{\emph{cdh}}\big(X,\Omega^{<i}_{-_{\Q}/\Q}\big)\Big)[-1]$$ in the derived category $\mathcal{D}(\Q)$. \end{corollary} \begin{proof} For every valuation ring extension $V$ of $\Q$, the natural map $$\mathbb{L}\Omega^{<i}_{V/\Q} \longrightarrow \Omega^{<i}_{V/\Q}$$ is an equivalence in the derived category $\mathcal{D}(\Q)$ by results of Gabber--Ramero (\cite[Theorem~$6.5.8\,(ii)$ and Corollary~$6.5.21$]{gabber_almost_2003}). The presheaves $R\Gamma_{\text{cdh}}(-, \mathbb{L}\Omega^{<i}_{-_{\Q}/\Q})$ and $R\Gamma_{\text{cdh}}(-,\Omega^{<i}_{-_{\Q}/\Q})$ are finitary cdh sheaves on qcqs schemes, so the natural map $$R\Gamma_{\text{cdh}}\big(-,\mathbb{L}\Omega^{<i}_{-_{\Q}/\Q}\big) \longrightarrow R\Gamma_{\text{cdh}}\big(-, \Omega^{<i}_{-_{\Q}/\Q}\big)$$ is an equivalence (\cite[Corollary~$2.4.19$]{elmanto_cdh_2021}). The result then follows from Theorem~\ref{theoremrationalstructuremain}. \end{proof} \begin{example}[Weight zero motivic cohomology]\label{exampleweightzeromotiviccohomology} For every qcqs scheme $X$, the natural map $$\Z(0)^{\text{mot}}(X) \longrightarrow \Z(0)^{\text{cdh}}(X) \simeq R\Gamma_{\text{cdh}}(X,\Z),$$ where the last idenfication is \cite{bachmann_A^1-invariant_2024}, is an equivalence in the derived category $\mathcal{D}(\Z)$. Indeed, it suffices to prove the result rationally, and modulo $p$ for every prime number $p$. The result rationally is a consequence of Corollary~\ref{corollaryrationalmainresultongradedpieces}. For every prime number $p$, the presheaf $\F_p(0)^{\text{BMS}}$ is naturally identified with the presheaf $R\Gamma_{\text{ét}}(-,\F_p)$ (\cite[Proposition~$7.16$]{bhatt_topological_2019}) which is a cdh sheaf on qcqs schemes (\cite[Theorem~$5.4$]{bhatt_arc-topology_2021}), so the result modulo $p$ is a consequence of Corollary~\ref{corollarymainpadicstructureongradeds}. \end{example} \subsection{A global Beilinson fibre square} \vspace{-\parindent} \hspace{\parindent} In this subsection we prove Theorem~\ref{theoremBFSglobalwithsolid}, which is a rewriting of the Beilinson fibre square of Antieau--Mathew--Morrow--Nikolaus \cite[Theorem~$6.17$]{antieau_beilinson_2020} in terms of the rigid-analytic derived de Rham cohomology of Section~\ref{sectionrigidanalyticdR}. Note that our statements are formulated in the generality of derived schemes, and that the functor $-_{\F_p}$ is then the derived base change from~$\Z$ to $\F_p$. The results in \cite{antieau_beilinson_2020} are stated in the generality of $p$-torsionfree commutative rings, on which derived and classical reduction modulo $p$ coincide. \begin{construction}[The map $\chi$]\label{constructionthemapchi} Let $i \in \Z$ be an integer. Following \cite{antieau_beilinson_2020}, we construct for every qcqs derived scheme $X$ a natural map $$\chi : \Big(\prod_{p \in \mathbb{P}} \Z_p(i)^{\text{BMS}}(X_{\F_p})\Big)_{\Q} \longrightarrow \Big(\prod_{p \in \mathbb{P}} R\Gamma_{\text{Zar}}\big(X,(\mathbb{L}\Omega_{-/\Z})^\wedge_p\big)\Big)_{\Q}$$ in the derived category $\mathcal{D}(\Q)$. \begin{enumerate} \item ($p \leq i+1$) Let $p$ be a prime number. By \cite[Theorem~$6.17$]{antieau_beilinson_2020}, there exists an integer $N \geq 0$ depending only on $i$ and a natural map $$\chi : \Z_p(i)^{\text{BMS}}(R/p) \longrightarrow \frac{1}{p^N} \big(\mathbb{L}\Omega_{R/\Z}\big)^\wedge_p$$ on $p$-torsionfree $p$-quasisyntomic rings $R$, and in particular on polynomial $\Z$\nobreakdash-algebras $R$. The functors $\Z_p(i)^{\text{BMS}}(-/p)$ and $\tfrac{1}{p^N}(\mathbb{L}\Omega_{-/\Z})^\wedge_p$, as functors from animated commutative rings to $p$\nobreakdash-complete objects in the derived category $\mathcal{D}(\Z)$, are left Kan extended from polynomial $\Z$\nobreakdash-alge\-bras (Corollary~\ref{corollaryBMSsyntomiccohomologyhasquasisyntomicdescentandLKEfrompolynomialZalgebras}\,$(2)$ and by construction, respectively). Left Kan extending the previous map then induces a natural map $$\chi : \Z_p(i)^{\text{BMS}}(R/p) \longrightarrow \frac{1}{p^N} \big(\mathbb{L}\Omega_{R/\Z}\big)^\wedge_p$$ on animated commutative rings $R$, where the reduction modulo $p$ is the derived one. Inverting~$p$ and Zariski sheafifying induces a natural map $$\chi : \Q_p(i)^{\text{BMS}}(X_{\F_p}) \longrightarrow R\Gamma_{\text{Zar}}\big(X,(\mathbb{L}\Omega_{-/\Z})^\wedge_p[\tfrac{1}{p}]\big)$$ in the derived category $\mathcal{D}(\Q)$, on general qcqs derived schemes $X$. \item ($p \geq i+2$) For prime numbers $p$ such that $p \geq i+2$, there actually exists a natural map $$\chi : \Z_p(i)^{\text{BMS}}(R/p) \longrightarrow \big(\mathbb{L}\Omega_{R/\Z}\big)^\wedge_p$$ on $p$-torsionfree $p$-quasisyntomic rings $R$ (\cite[Theorem~$6.17$]{antieau_beilinson_2020}), and in particular on polynomial $\Z$-algebras $R$. Left Kan extending this map again induces a natural map $$\chi : \Z_p(i)^{\text{BMS}}(R/p) \longrightarrow \big(\mathbb{L}\Omega_{R/\Z}\big)^\wedge_p$$ on animated commutative rings $R$. Taking the product over all primes $p \geq i+2$, and then rationalisation and Zariski sheafification, induces a natural map $$\chi : \Big(\prod_{p \in \mathbb{P}_{\geq i+2}} \Z_p(i)^{\text{BMS}}(X_{\F_p})\Big)_{\Q} \longrightarrow \Big(\prod_{p \in \mathbb{P}_{\geq i+2}} R\Gamma_{\text{Zar}}\big(X,(\mathbb{L}\Omega_{-/\Z})^\wedge_p\big)\Big)_{\Q}$$ in the derived category $\mathcal{D}(\Q)$, on general qcqs derived schemes $X$. \item (general construction) For every qcqs derived scheme $X$, define the desired natural map~$\chi$ as the product of the map $\chi$ of $(2)$ with the finite product over prime numbers $p \leq i+1$ of the map $\chi$ of $(1)$. \end{enumerate} \end{construction} \begin{theorem}[Beilinson fibre square, after \cite{antieau_beilinson_2020}]\label{theoremBFSasinAMMN} Let $X$ be a qcqs derived scheme. Then for every integer $i \in \Z$, the natural diagram $$\begin{tikzcd} \Big(\prod_{p \in \mathbb{P}} \Z_p(i)^{\emph{BMS}}(X)\Big)_{\Q} \ar[r] \ar[d] & \Big(\prod_{p \in \mathbb{P}} R\Gamma_{\emph{Zar}}\big(X,(\mathbb{L}\Omega^{\geq i}_{-/\Z})^\wedge_p\big)\Big)_{\Q} \ar[d] \\ \Big(\prod_{p \in \mathbb{P}} \Z_p(i)^{\emph{BMS}}(X_{\F_p})\Big)_{\Q} \arrow[r,"\chi"] & \Big(\prod_{p \in \mathbb{P}} R\Gamma_{\emph{Zar}}\big(X,(\mathbb{L}\Omega_{-/\Z})^\wedge_p\big)\Big)_{\Q} \end{tikzcd}$$ in the derived category $\mathcal{D}(\Q)$ is commutative, with total cofibre naturally identified with the complex $$\Big(\prod_{p \in \mathbb{P}} R\Gamma_{\emph{Zar}}\big(X,\mathbb{L}\Omega^{<i}_{-_{\F_p}/\Z}\big)\Big)_{\Q} \in \mathcal{D}(\Q).$$ \end{theorem} \begin{proof} This is a consequence of Construction~\ref{constructionthemapchi} and \cite[Theorem~$6.17$]{antieau_beilinson_2020}. \end{proof} In the following result, the map $\underline{\widehat{\chi}}$ is defined as the composite of the map $\chi$ of Construction~\ref{constructionthemapchi} with the natural maps $$\big(\prod_{p \in \mathbb{P}} R\Gamma_{\text{Zar}}\big(X,(\mathbb{L}\Omega_{-/\Z})^\wedge_p\big)\big)_{\Q} \rightarrow \big(\prod_{p \in \mathbb{P}} R\Gamma_{\text{Zar}}\big(X,(\widehat{\mathbb{L}\Omega}_{-/\Z})^\wedge_p\big)\big)_{\Q} \rightarrow R\Gamma_{\text{Zar}}\big(X,\prod_{p \in \mathbb{P}}{}^{'} \underline{\widehat{\mathbb{L}\Omega}}_{-_{\Q_p}/\Q_p}\big)$$ induced by Hodge-completion and Remark~\ref{remark18cartesiansquaredefiningsolidderiveddeRhamcohomology}. \begin{theorem}\label{theoremBFSglobalwithsolid} Let $X$ be a qcqs derived scheme. Then for every integer $i \in \Z$, there exists a natural commutative square $$\begin{tikzcd} \Q(i)^{\emph{TC}}(X) \ar[r] \ar[d] & R\Gamma_{\emph{Zar}}\big(X,\widehat{\mathbb{L}\Omega}^{\geq i}_{-_{\Q}/\Q}\big) \ar[d] \\ \big(\prod_{p \in \mathbb{P}} \Z_p(i)^{\emph{BMS}}(X_{\F_p})\big)_{\Q} \arrow[r,"\underline{\widehat{\chi}}"] & R\Gamma_{\emph{Zar}}\big(X,\prod'_{p \in \mathbb{P}} \underline{\widehat{\mathbb{L}\Omega}}_{-_{\Q_p}/\Q_p}\big) \end{tikzcd}$$ in the derived category $\mathcal{D}(\Q)$, whose total cofibre is naturally identified with the complex $$\Big(\prod_{p \in \mathbb{P}} R\Gamma_{\emph{Zar}}\big(X,\mathbb{L}\Omega^{<i}_{-_{\F_p}/\Z}\big)\Big)_{\Q} \in \mathcal{D}(\Q).$$ \end{theorem} \begin{proof} By construction, there exists a natural commutative diagram $$\hspace*{-.2cm}\begin{tikzcd}[sep=small] \Q(i)^{\text{TC}}(X) \ar[r] \ar[d] & R\Gamma_{\text{Zar}}\big(X,(\widehat{\mathbb{L}\Omega}^{\geq i}_{-/\Z})_{\Q}\big) \ar[r] \ar[d] & R\Gamma_{\text{Zar}}\big(X,\widehat{\mathbb{L}\Omega}^{\geq i}_{-_{\Q}/\Q}\big) \ar[d] \\ \Big(\prod_{p \in \mathbb{P}} \Z_p(i)^{\text{BMS}}(X)\Big)_{\Q} \ar[r] \ar[d] & \Big(\prod_{p \in \mathbb{P}} R\Gamma_{\text{Zar}}\big(X,(\widehat{\mathbb{L}\Omega}^{\geq i}_{-/\Z})^\wedge_p\big)\Big)_{\Q} \ar[r] \ar[d] & R\Gamma_{\text{Zar}}\big(X,\prod'_{p \in \mathbb{P}} \underline{\widehat{\mathbb{L}\Omega}}^{\geq i}_{-_{\Q_p}/\Q_p}\big) \ar[d] \\ \Big(\prod_{p \in \mathbb{P}} \Z_p(i)^{\text{BMS}}(X_{\F_p})\Big)_{\Q} \arrow[r] & \Big(\prod_{p \in \mathbb{P}} R\Gamma_{\text{Zar}}\big(X,(\widehat{\mathbb{L}\Omega}_{-/\Z})^\wedge_p\big)\Big)_{\Q} \ar[r] & R\Gamma_{\text{Zar}}\big(X,\prod'_{p \in \mathbb{P}} \underline{\widehat{\mathbb{L}\Omega}}_{-_{\Q_p}/\Q_p}\big) \end{tikzcd}$$ in the derived category $\mathcal{D}(\Q)$, where all the inner squares are cartesian expect the left bottom one, and where the commutativity for the left bottom square is part of Theorem~\ref{theoremBFSasinAMMN}. The desired total cofibre is then naturally identified with the total cofibre of the left bottom square, and the result is then a consequence of Theorem~\ref{theoremBFSasinAMMN}. \end{proof} Theorem~\ref{theoremBFSglobalwithsolid} means in particular that the complex $\Q(i)^{\text{TC}}(X)$ can be expressed purely in terms of characteristic zero, characteristic $p$, and rigid-analytic information. \begin{corollary}\label{corollaryBFSonlyonep} Let $p$ be a prime number, and $X$ be a qcqs derived $\Z_{(p)}$-scheme. Then for every integer $i \in \Z$, there exists a natural cartesian square $$\begin{tikzcd} \Q(i)^{\emph{TC}}(X) \ar[r] \ar[d] & R\Gamma_{\emph{Zar}}\big(X,\widehat{\mathbb{L}\Omega}^{\geq i}_{-_{\Q}/\Q}\big) \ar[d] \\ \Q_p(i)^{\emph{BMS}}(X_{\F_p}) \arrow[r,"\underline{\widehat{\chi}}"] & R\Gamma_{\emph{Zar}}\big(X,\underline{\widehat{\mathbb{L}\Omega}}_{-_{\Q_p}/\Q_p}\big) \end{tikzcd}$$ in the derived category $\mathcal{D}(\Q)$. \end{corollary} \begin{proof} The base change $X_{\F_{\l}}$ is zero for every prime number $\l$ different from $p$. The complex~$\mathbb{L}\Omega^{<i}_{X_{\F_p}/\Z}$ is $\F_p$-linear, so its rationalisation vanishes. The result is then a consequence of Theorem~\ref{theoremBFSglobalwithsolid} and Remark~\ref{remark14HKRfiltrationonHPsolid}. \end{proof} The following result is an analogue of Theorem~\ref{theoremBFSglobalwithsolid} at the level of filtered objects. \begin{theorem}\label{theoremmainconsequenceBFSwithfiltrations} Let $X$ be qcqs derived scheme. Then there exists a natural commutative square of filtered spectra $$\begin{tikzcd} \emph{Fil}^\star_{\emph{mot}} \emph{TC}(X;\Q) \arrow{r} \arrow{d} & \emph{Fil}^\star_{\emph{HKR}} \emph{HC}^-(X_{\Q}/\Q) \ar[d] \\ \Big(\prod_{p \in \mathbb{P}} \emph{Fil}^\star_{\emph{BMS}} \emph{TC}(X_{\F_p})\Big)_{\Q} \arrow[r,"\underline{\widehat{\chi}}"] & \emph{Fil}^\star_{\emph{HKR}} \Big(\prod'_{p \in \mathbb{P}} \emph{HH}(X;\Q_p)\Big)^{t\emph{S}^1}, \end{tikzcd}$$ whose total cofibre is naturally identified with the filtered spectrum $\big(\prod_{p \in \mathbb{P}} \emph{Fil}^{\star-1}_{\emph{HKR}} \emph{HC}(X_{\F_p})\big)_{\Q}[2]$. \end{theorem} \begin{proof} The construction of the map $\chi$ in the proof of \cite[Theorem~$6.17$]{antieau_beilinson_2020} adapts readily to define a map at the filtered level instead of graded pieces. The proof is then the same as in Construction~\ref{constructionthemapchi} and Theorem~\ref{theoremBFSglobalwithsolid}. \end{proof} \begin{question} Given the results of the previous sections, and in particular Corollary~\ref{corollaryHPsolidallprimespisacdhsheafwithfiltration} and Theorem~\ref{theoremmainconsequenceBFSwithfiltrations}, it is a natural question --to which we do not know the answer-- to ask whether the presheaf $$\Big(\prod_{p \in \mathbb{P}} \text{Fil}^\star_{\text{BMS}} \text{TC}(-_{\F_p})\Big)_{\Q} : \text{Sch}^{\text{qcqs,op}} \longrightarrow \text{FilSp}$$ is a cdh sheaf, where $-_{\F_p}$ again means derived base change from $\Z$ to $\F_p$. \end{question} \newpage \section{\texorpdfstring{$p$}{TEXT}-adic structure of motivic cohomology} \vspace{-\parindent} \hspace{\parindent} In this section, we give a description of motivic cohomology with finite coefficients in terms of Bhatt--Lurie's syntomic cohomology (Theorem~\ref{theorempadicmotiviccohomologyintermsofsyntomicohomology}). \subsection{Comparison to étale cohomology} \vspace{-\parindent} \hspace{\parindent} In this subsection, we construct a natural comparison map, called the Beilinson--Lichtenbaum comparison map, from the motivic cohomology of a scheme to the étale cohomology of its generic fibre (Definition~\ref{definitionBeilinsonLichtenbaumcomparisonmap}). We then use this comparison map to establish a complete description of $\l$-adic motivic cohomology in terms of étale cohomology (Theorem~\ref{theoremladicmotiviccohomology}). We use the following important result of Deligne to construct the Beilinson--Lichtenbaum comparison map. \begin{theorem}[\cite{bhatt_arc-topology_2021}]\label{theoremBMcdhdescentforétalecohomology} Let $p$ be a prime number, and $k \geq 1$ be an integer. Then for every integer~\hbox{$i \geq 0$}, the presheaf $$R\Gamma_{\emph{ét}}(-[\tfrac{1}{p}],\mu_{p^k}^{\otimes i}) : \emph{Sch}^{\emph{qcqs,op}} \longrightarrow \mathcal{D}(\Z/p^k)$$ is a cdh sheaf. \end{theorem} \begin{proof} By \cite[Theorem~$5.4$]{bhatt_arc-topology_2021}, the presheaf $R\Gamma_{\text{ét}}(-[\tfrac{1}{p}],\mu_{p^k}^{\otimes i})$ is an arc sheaf on qcqs schemes, and the arc topology is finer than the cdh topology.\footnote{More precisely, the arc topology is finer than the $v$ topology, which is finer than the h topology, which is finer than the cdh topology (see \cite{bhatt_arc-topology_2021,elmanto_cdh_2021}).} \end{proof} \begin{definition}[Cdh-local Beilinson--Lichtenbaum comparison map]\label{definitioncdhlocalBeilinsonLichtenbaumcomparisonmap} Let $p$ be a prime number, and $k \geq 1$ be an integer. For any integer $i \geq 0$, the {\it cdh-local Beilinson--Lichtenbaum comparison map} is the map $$\Z/p^k(i)^{\text{cdh}}(-) \longrightarrow R\Gamma_{\text{ét}}(-[\tfrac{1}{p}], \mu_{p^k}^{\otimes i})$$ of functors from (the opposite category of) qcqs derived schemes to the derived category $\mathcal{D}(\Z/p^k)$ defined as the composite $$\Z/p^k(i)^{\text{cdh}}(-) \longrightarrow \Z/p^k(i)^{\text{cdh}}(-[\tfrac{1}{p}]) \simeq \big(L_{\text{cdh}} \tau^{\leq i} R\Gamma_{\text{ét}}(-,\mu_{p^k}^{\otimes i})\big)(-[\tfrac{1}{p}]) \longrightarrow R\Gamma_{\text{ét}}(-[\tfrac{1}{p}],\mu_{p^k}^{\otimes i})$$ where the first map is induced by base change from $\Z$ to $\Z[\tfrac{1}{p}]$, the equivalence is \cite{bachmann_A^1-invariant_2024}, and the last map is induced by the natural transformation $\tau^{\leq i} \rightarrow \text{id}$ and Theorem~\ref{theoremBMcdhdescentforétalecohomology}. \end{definition} \begin{definition}[Beilinson--Lichtenbaum comparison map]\label{definitionBeilinsonLichtenbaumcomparisonmap} Let $p$ be a prime number, and $k \geq 1$ be an integer. For any integer $i \geq 0$, the {\it Beilinson--Lichtenbaum comparison map} (or {\it motivic-étale comparison map}) is the map $$\Z/p^k(i)^{\text{mot}}(-) \longrightarrow R\Gamma_{\text{ét}}(-[\tfrac{1}{p}], \mu_{p^k}^{\otimes i})$$ of functors from (the opposite category of) qcqs schemes to the category $\mathcal{D}(\Z/p^k)$ defined as the composite $$\Z/p^k(i)^{\text{mot}}(-) \longrightarrow \Z/p^k(i)^{\text{cdh}}(-) \longrightarrow R\Gamma_{\text{ét}}(-[\tfrac{1}{p}],\mu_{p^k}^{\otimes i})$$ where the first map is cdh sheafification and the second map is the cdh-local Beilinson--Lichtenbaum comparison map of Definition~\ref{definitioncdhlocalBeilinsonLichtenbaumcomparisonmap}. \end{definition} \begin{remark}\label{remarkcommutativediagramtodefinemotivictosyntomiccomparisonmap} Let $p$ be a prime number, $k \geq 1$ be an integer, $R$ be a commutative ring, and $R^h_p$ be the $p$-henselisation of $R$. Then for every integer $i \geq 0$, the natural diagram $$\begin{tikzcd} \Z/p^k(i)^{\text{mot}}(\text{Spec}(R)) \arrow{r} \arrow{d} & \Z/p^k(i)^{\text{cdh}}(\text{Spec}(R)) \ar[d] \ar[r] & R\Gamma_{\text{ét}}\big(\text{Spec}(R[\tfrac{1}{p}]),\mu_{p^k}^{\otimes i}\big) \ar[d] \\ \Z/p^k(i)^{\text{mot}}(\text{Spec}(R^h_p)) \ar[d] \arrow{r} & \Z/p^k(i)^{\text{cdh}}(\text{Spec}(R^h_p)) \ar[r] \ar[d] & R\Gamma_{\text{ét}}\big(\text{Spec}(R^h_p[\tfrac{1}{p}]),\mu_{p^k}^{\otimes i}\big) \arrow[d,"\text{id}"] \\ \Z/p^k(i)^{\text{BMS}}(\text{Spec}(R^h_p)) \ar[r] & \big(L_{\text{cdh}} \Z/p^k(i)^{\text{BMS}}\big)(\text{Spec}(R^h_p)) \ar[r] & R\Gamma_{\text{ét}}\big(\text{Spec}(R^h_p[\tfrac{1}{p}]),\mu_{p^k}^{\otimes i}\big), \end{tikzcd}$$ is a commutative diagram in the derived category $\mathcal{D}(\Z/p^k)$, where the top horizontal right map and the middle horizontal right map are given by Definition~\ref{definitioncdhlocalBeilinsonLichtenbaumcomparisonmap}, and the bottom horizontal right map is induced by \cite[Theorem~$8.3.1$]{bhatt_absolute_2022}. This statement is a consequence of the naturality of the constructions, except for the commutativity of the bottom right square, which is proved in \cite{bachmann_A^1-invariant_2024}. \end{remark} \begin{theorem}[$\l$-adic motivic cohomology]\label{theoremladicmotiviccohomology} Let $p$ be a prime number, $X$ be a qcqs scheme over $\Z[\tfrac{1}{p}]$, and $k \geq 1$ be an integer. Then for every integer $i \geq 0$, the Beilinson--Lichtenbaum comparison for classical motivic cohomology induces a natural equivalence $$\Z/p^k(i)^{\emph{mot}}(X) \simeq \big(L_{\emph{cdh}} \tau^{\leq i} R\Gamma_{\emph{ét}}(-,\mu_{p^k}^{\otimes i})\big)(X)$$ in the derived category $\mathcal{D}(\Z/p^k)$. \end{theorem} \begin{proof} The syntomic complex $\Z/p^k(i)^{\text{BMS}}$ and its cdh sheafification $L_{\text{cdh}} \Z/p^k(i)^{\text{BMS}}$ vanish on qcqs derived $\Z[\tfrac{1}{p}]$-schemes. In particular, the natural map $$\Z/p^k(i)^{\text{mot}}(X) \longrightarrow \Z/p^k(i)^{\text{cdh}}(X)$$ is an equivalence in the derived category $\mathcal{D}(\Z/p^k)$ (Proposition~\ref{propositionpadicstructuremain}). The Beilinson--Lichtenbaum comparison for classical motivic cohomology induces a natural equivalence $$\Z/p^k(i)^{\text{cdh}}(X) \xlongleftarrow{\sim} \big(L_{\text{cdh}} \tau^{\leq i} R\Gamma_{\text{ét}}(-,\mu_{p^k}^{\otimes i})\big)(X)$$ in the derived category $\mathcal{D}(\Z/p^k)$ (\cite{bachmann_A^1-invariant_2024}). The desired equivalence is the composite of the previous two equivalences. \end{proof} \begin{corollary}\label{corollaryladicmotivcohomologylowdegreesisétalecohomology} Let $p$ be a prime number, $X$ be a qcqs $\Z[\tfrac{1}{p}]$-scheme, and $k \geq 1$ be an integer. Then for every integer $i \geq 0$, there is a natural equivalence $$\tau^{\leq i} \Z/p^k(i)^{\emph{mot}}(X) \simeq \tau^{\leq i} R\Gamma_{\emph{ét}}(X,\mu_{p^k}^{\otimes i})$$ in the derived category $\mathcal{D}(\Z/p^k)$. \end{corollary} \begin{proof} The natural map $$\tau^{\leq i} \big(L_{\text{cdh}} \tau^{\leq i} R\Gamma_{\text{ét}}(-,\mu_{p^k}^{\otimes i})\big)(X) \longrightarrow \tau^{\leq i} \big(L_{\text{cdh}} R\Gamma_{\text{ét}}(-,\mu_{p^k}^{\otimes i})\big)(X)$$ is an equivalence in the derived category $\mathcal{D}(\Z/p^k)$. The result is then a consequence of Theorems~\ref{theoremBMcdhdescentforétalecohomology} and \ref{theoremladicmotiviccohomology}. \end{proof} \subsection{Comparison to syntomic cohomology} \vspace{-\parindent} \hspace{\parindent} In this subsection, we study motivic cohomology with finite coefficients. Our main result is a computation of $p$-adic motivic cohomology in terms of syntomic cohomology (Theorem~\ref{theorempadicmotiviccohomologyintermsofsyntomicohomology}). \begin{notation}[Syntomic cohomology of derived scheme, after Bhatt--Lurie \cite{bhatt_absolute_2022}]\label{notationsyntomiccohomology} Let $X$ be a qcqs derived scheme, $p$ be a prime number, and $i \in \Z$ be an integer. We denote by $$\Z_p(i)^{\text{syn}}(X) \in \mathcal{D}(\Z_p)$$ the {\it syntomic cohomology} of $X$, as defined in \cite[Section~$8.4$]{bhatt_absolute_2022}. For every integer $k \geq 1$, we also denote by $\Z/p^k(i)^{\text{syn}}(X)$ the derived reduction modulo $p^k$ of the previous complex. In particular, the presheaf $\Z/p^k(i)^{\text{syn}}$ is a Zariski sheaf, whose restriction to animated commutative rings is left Kan extended from smooth $\Z$-algebras, and such that on classical commutative rings $R$, there is, by definition, a natural cartesian square $$\begin{tikzcd} \Z/p^k(i)^{\text{syn}}(\text{Spec}(R)) \arrow{r} \arrow{d} & R\Gamma_{\text{ét}}(\text{Spec}(R[\frac{1}{p}]),\mu_{p^k}^{\otimes i}) \ar[d] \\ \Z/p^k(i)^{\text{BMS}}(\text{Spec}(R^h_p)) \arrow{r} & R\Gamma_{\text{ét}}(\text{Spec}(R^h_p[\tfrac{1}{p}]),\mu_{p^k}^{\otimes i}) \end{tikzcd}$$ in the derived category $\mathcal{D}(\Z/p^k)$, where $R^h_p$ is the $p$-henselisation of the commutative ring~$R$, and the bottom map is the map of \cite[Theorem~$8.3.1$]{bhatt_absolute_2022}. \end{notation} \begin{construction}[Motivic-syntomic comparison map]\label{constructionmotivicsyntomiccomparisonmap} Let $p$ be a prime number, and $k \geq 1$ be an integer. For any integer $i \geq 0$, the {\it motivic-syntomic comparison map} is the map $$\Z/p^k(i)^{\text{mot}}(-) \longrightarrow \Z/p^k(i)^{\text{syn}}(-)$$ of functors from (the opposite category of) qcqs schemes to the derived category $\mathcal{D}(\Z/p^k)$ defined as the Zariski sheafification of the map on commutative rings $R$ induced by the natural commutative diagram $$\begin{tikzcd} \Z/p^k(i)^{\text{mot}}(\text{Spec}(R)) \ar[d] \ar[r] & R\Gamma_{\text{ét}}\big(\text{Spec}(R[\tfrac{1}{p}]),\mu_{p^k}^{\otimes i}\big) \ar[d] \\ \Z/p^k(i)^{\text{BMS}}(\text{Spec}(R^h_p)) \ar[r] & R\Gamma_{\text{ét}}\big(\text{Spec}(R^h_p[\tfrac{1}{p}]),\mu_{p^k}^{\otimes i}\big) \end{tikzcd}$$ of Remark~\ref{remarkcommutativediagramtodefinemotivictosyntomiccomparisonmap}. \end{construction} The following cartesian square, where the bottom horizontal map was described independently in \cite{bachmann_A^1-invariant_2024}, can be seen as an alternative definition of $p$-adic motivic cohomology of qcqs schemes (see Corollary~\ref{corollarymainpadicstructureongradeds}). \begin{proposition}\label{propositioncartesiansquaremotivicsyntomic} Let $X$ be a qcqs scheme, $p$ be a prime number, and $k \geq 1$ be an integer. Then for every integer $i \geq 0$, the commutative diagram $$\begin{tikzcd} \Z/p^k(i)^{\emph{mot}}(X) \ar[r] \ar[d] & \Z/p^k(i)^{\emph{syn}}(X) \ar[d]\\ \Z/p^k(i)^{\emph{cdh}}(X) \ar[r] & \big(L_{\emph{cdh}} \Z/p^k(i)^{\emph{syn}}\big)(X) \end{tikzcd}$$ where the top horizontal map is the motivic-syntomic comparison map of Construction~\ref{constructionmotivicsyntomiccomparisonmap} and the vertical maps are cdh sheafification, is a cartesian square in the derived category $\mathcal{D}(\Z/p^k)$. \end{proposition} \begin{proof} By \cite[Remark~$8.4.4$]{bhatt_absolute_2022}, there is a natural fibre sequence $$R\Gamma_{\text{ét}}(X,j_!\mu_{p^k}^{\otimes i}) \longrightarrow \Z/p^k(i)^{\text{syn}}(X) \longrightarrow \Z/p^k(i)^{\text{BMS}}(X)$$ in the derived category $\mathcal{D}(\Z/p^k)$. The first term of this fibre sequence satisfies arc descent by \cite[Theorem~$5.4$]{bhatt_arc-topology_2021}, hence in particular cdh descent. The result is then a consequence of Corollary~\ref{corollarymainpadicstructureongradeds}. \end{proof} The following result is a mixed characteristic generalisation of Elmanto--Morrow's fundamental fibre sequence for motivic cohomology of characteristic $p$ schemes (\cite[Corollary~$4.32$]{elmanto_motivic_2023}). \begin{theorem}[$p$-adic motivic cohomology]\label{theorempadicmotiviccohomologyintermsofsyntomicohomology} Let $X$ be a qcqs scheme, $p$ be a prime number, and $k \geq 1$ be an integer. Then for every integer $i \geq 0$, there is a natural fibre sequence $$\Z/p^k(i)^{\emph{mot}}(X) \longrightarrow \Z/p^k(i)^{\emph{syn}}(X) \longrightarrow \big(L_{\emph{cdh}} \tau^{>i} \Z/p^k(i)^{\emph{syn}}\big)(X)$$ in the derived category $\mathcal{D}(\Z/p^k)$. In particular, the fibre of the motivic-syntomic comparison map is in degrees at least $i+2$. \end{theorem} \begin{proof} By \cite{bachmann_A^1-invariant_2024}, there is a natural equivalence $$\Z/p^k(i)^{\text{cdh}}(X) \simeq \big(L_{\text{cdh}} \tau^{\leq i} \Z/p^k(i)^{\text{syn}}\big)(X)$$ in the derived category $\mathcal{D}(\Z/p^k)$. The result is then a consequence of Proposition~\ref{propositioncartesiansquaremotivicsyntomic}. \end{proof} \begin{corollary}\label{corollarypadiccomparisoninsmalldegreessyntomiccoho} Let $X$ be a qcqs scheme, $p$ be a prime number, and $k \geq 1$ be an integer. Then for every integer $i \geq 0$, the motivic-syntomic comparison map induces a natural equivalence $$\tau^{\leq i} \Z/p^k(i)^{\emph{mot}}(X) \xlongrightarrow{\sim} \tau^{\leq i} \Z/p^k(i)^{\emph{syn}}(X)$$ in the derived category $\mathcal{D}(\Z/p^k)$. \end{corollary} \begin{proof} This is a consequence of Theorem~\ref{theorempadicmotiviccohomologyintermsofsyntomicohomology}. \end{proof} Algebraic $K$-theory being $\mathbb{A}^1$-invariant on regular schemes, one could expect the classical $\mathbb{A}^1$\nobreakdash-inva\-riant motivic cohomology to be a good theory for general regular schemes, not only schemes that are smooth over a Dedekind domain. However, most of the results on classical motivic cohomology in mixed characteristic are proved only in the smooth case, as consequences of the Gersten conjecture proved by Geisser \cite{geisser_motivic_2004}. This is the case for the Beilinson--Lichtenbaum conjecture, which compares motivic cohomology with finite coefficients to étale cohomology. Combined with Theorem~\ref{theoremintrocohomology}\,$(6)$, the following result extends the analogous result for classical motivic cohomology to the regular case. \begin{corollary}[Beilinson--Lichtenbaum conjecture for $F$-smooth schemes]\label{corollaryFsmoothnessBeilinsonLichtenbaumcomparison} Let $p$ be a prime number, $X$ be a $p$-torsionfree $F$-smooth scheme ({\it e.g.}, a regular scheme flat over $\Z$), and $k \geq 1$ be an integer. Then for every integer $i \geq 0$, the fibre of the Beilinson--Lichtenbaum comparison map $$\Z/p^k(i)^{\emph{mot}}(X) \longrightarrow R\Gamma_{\emph{ét}}\big(X[\tfrac{1}{p}],\mu_{p^k}^{\otimes i}\big)$$ is in degrees at least $i+1$. \end{corollary} \begin{proof} By \cite[Theorem~$1.8$]{bhatt_syntomic_2023}, the fibre of the natural map $$\Z/p^k(i)^{\text{syn}}(X) \longrightarrow R\Gamma_{\text{ét}}\big(X[\tfrac{1}{p}],\mu_{p^k}^{\otimes i}\big)$$ is in degrees at least $i+1$. The result is then a consequence of Theorem~\ref{theorempadicmotiviccohomologyintermsofsyntomicohomology}. \end{proof} \begin{corollary}\label{corollarycomparisonpadicmotiviccohomologyofFsmooththingswithmotiviccohoofgenericfibre} Let $p$ be a prime number, $X$ be a $p$-torsionfree $F$-smooth scheme, and $k \geq 1$ be an integer. Then the fibre of the natural map $$\Z/p^k(i)^{\emph{mot}}(X) \longrightarrow \Z/p^k(i)^{\emph{mot}}\big(X[\tfrac{1}{p}]\big)$$ is in degrees at least $i+1$. \end{corollary} \begin{proof} By construction, the Beilinson--Lichtenbaum comparison map $$\Z/p^k(i)^{\text{mot}}(X) \longrightarrow R\Gamma_{\text{ét}}\big(X[\tfrac{1}{p}],\mu_{p^k}^{\otimes i}\big)$$ naturally factors as the composite $$\Z/p^k(i)^{\text{mot}}(X) \longrightarrow \Z/p^k(i)^{\text{mot}}\big(X[\tfrac{1}{p}]\big) \longrightarrow R\Gamma_{\text{ét}}\big(X[\tfrac{1}{p}],\mu_{p^k}^{\otimes i}\big).$$ The fibre of the second map is in degrees at least $i+1$ by Corollary~\ref{corollaryladicmotivcohomologylowdegreesisétalecohomology}, and the fibre of the composite is in degrees at least $i+1$ by Corollary~\ref{corollaryFsmoothnessBeilinsonLichtenbaumcomparison}, so the fibre of the first map is in degrees at least $i+1$. \end{proof} \newpage \section{Comparison to classical motivic cohomology}\label{sectionclassical} \vspace{-\parindent} \hspace{\parindent} For smooth schemes over a field, Elmanto--Morrow proved that the motivic complexes $\Z(i)^{\text{mot}}$ coincide with the classical motivic complexes $\Z(i)^{\text{cla}}$ (\cite[Corollary~$6.4$]{elmanto_motivic_2023}). Their proof uses Gersten injectivity and the projective bundle formula to reduce the statement to the case of fields, and relies on Gabber's presentation lemma (\cite[Theorem~$3.1.1$]{colliot-thelene_bloch_1997}), which is unknown in mixed characteristic. In this section, we prove partial results comparing the complexes $\Z(i)^{\text{mot}}$ and $\Z(i)^{\text{cla}}$ in mixed characteristic. \subsection{Comparison to classical motivic cohomology in low degrees} \vspace{-\parindent} \hspace{\parindent} In this subsection, we prove that the classical-motivic comparison map $\Z(i)^{\text{cla}} \rightarrow \Z(i)^{\text{mot}}$ (Definition~\ref{definitionclassicalmotiviccomparisonmap}) is an equivalence with rational or $\l$-adic coefficients, and integrally in degrees at most~$i+1$. \begin{proposition}\label{propositionpadiccomparisonclassicalmotivicinsmalldegrees} Let $p$ be a prime number, $B$ be a Dedekind domain such that every characteristic~$p$ residue field of $B$ is perfect, and $X$ be a smooth scheme over $B$. Then for any integers $i \geq 0$ and $k \geq 1$, the fibre of the classical-motivic comparison map $$\Z/p^k(i)^{\emph{cla}}(X) \longrightarrow \Z/p^k(i)^{\emph{mot}}(X)$$ is in degrees at least $i+2$ in the derived category $\mathcal{D}(\Z/p^k)$. If $p$ is moreover invertible in $X$, then this fibre vanishes, {\it i.e.}, the previous classical-motivic comparison map is an equivalence. \end{proposition} \begin{proof} If $p$ is invertible in the scheme $X$, then the composite map $$\Z/p^k(i)^{\text{cla}}(X) \longrightarrow \Z/p^k(i)^{\text{mot}}(X) \longrightarrow \Z/p^k(i)^{\text{cdh}}(X)$$ is an equivalence in the derived category $\mathcal{D}(\Z/p^k)$ by \cite{bachmann_A^1-invariant_2024}. The right map is also an equivalence by Remark~\ref{remarkladicmotiviccohomology}, so the left map is an equivalence. In general, consider the natural commutative diagram $$\begin{tikzcd} \Z/p^k(i)^{\text{cla}}(X) \ar[r] \ar[d] & \big(L_{\text{Nis}} \tau^{\leq i} \Z/p^k(i)^{\text{syn}}\big)(X) \ar[d] \\ \Z/p^k(i)^{\text{mot}}(X) \ar[r] & \Z/p^k(i)^{\text{syn}}(X) \end{tikzcd}$$ in the derived category $\mathcal{D}(\Z/p^k)$. The top horizontal map is an equivalence by \cite[Theorems~$1.2\,(2)$ and $1.3$]{geisser_motivic_2004} and \cite[Theorem~$5.8$]{bhatt_syntomic_2023}. The fibre of the right vertical map is naturally identified with $\big(L_{\text{Nis}} \tau^{>i} \Z/p^k(i)^{\text{syn}})(X)[-1]$, and is thus in degrees at least $i+2$. The fibre of the bottom horizontal map is in degrees at least $i+2$ by Theorem~\ref{theorempadicmotiviccohomologyintermsofsyntomicohomology}. So the fibre of the left vertical map is in degrees at least $i+2$. \end{proof} \begin{proposition}\label{propositionrationalcomparisonclassicalmotivic} Let $B$ be a mixed characteristic Dedekind domain, and $X$ be a smooth scheme over~$B$. Then for every integer $i \geq 0$, the classical-motivic comparison map $$\Q(i)^{\emph{cla}}(X) \longrightarrow \Q(i)^{\emph{mot}}(X)$$ is an equivalence in the derived category $\mathcal{D}(\Q)$. \end{proposition} \begin{proof} This is a consequence of the rational splitting of algebraic $K$-theory induced by Adams operations. More precisely, we use the splitting induced by Lemma~\ref{lemma29howtouseAdamsoperations} for the filtrations $\text{Fil}^\star_{\text{cla}} \text{K}(-;\Q)$ (which is $\N$-indexed by construction) and $\text{Fil}^\star_{\text{mot}} \text{K}(-;\Q)$ (which is $\N$-indexed by Proposition~\ref{propositionmotivicfiltrationisexhaustive}). These decompositions are compatible with the classical-motivic comparison map because of the compatibility between the associated Adams operations (Section~\ref{subsectionAdamsoperations}). \end{proof} \begin{theorem}[Comparison to classical motivic cohomology]\label{theoremcomparisonclassical-motivicforsmoothoverDedekinddomain} Let $B$ be a mixed characteristic Dedekind domain such that every residue field of $B$ is perfect, and $X$ be a smooth scheme over $B$. Then for every integer $i \geq 0$, the fibre of the classical-motivic comparison map $$\Z(i)^{\emph{cla}}(X) \longrightarrow \Z(i)^{\emph{mot}}(X)$$ is in degrees at least $i+3$. \end{theorem} \begin{proof} Let $F(X) \in \mathcal{D}(\Z)$ be the fibre of the classical-motivic comparison map $$\Z(i)^{\text{cla}}(X) \longrightarrow \Z(i)^{\text{mot}}(X).$$ We want to prove that for every integer $k \leq i+2$, the abelian group $\text{H}^k(F(X))$ is zero. By Proposition~\ref{propositionrationalcomparisonclassicalmotivic}, the abelian group $\text{H}^k(F(X))$ is torsion for every integer $k \in \Z$. For every prime number $p$ and every integer $k \in \Z$, there is a natural short exact sequence of abelian groups $$0 \longrightarrow \text{H}^k(F(X))/p \longrightarrow \text{H}^k(F(X)/p) \longrightarrow \text{H}^{k+1}(F(X))[p] \longrightarrow 0.$$ By Proposition~\ref{propositionpadiccomparisonclassicalmotivicinsmalldegrees}, for every prime number $p$, the abelian group $\text{H}^k(F(X)/p)$ is zero if $k \leq i+1$, hence the abelian group $\text{H}^k(F(X))[p]$ is zero for every integer $k \leq i+2$. This implies the desired result. \end{proof} \begin{corollary}\label{corollarycomparisonclassicalmotivic} Let $B$ be a mixed characteristic Dedekind domain such that every residue field of $B$ is perfect. Then for every integer $i \geq 0$, the classical-motivic comparison map induces an equivalence of $\mathcal{D}(\Z)$-valued presheaves $$\Z(i)^{\emph{cla}}(-) \longrightarrow \big(L_{\emph{Zar}} \tau^{\leq i} \Z(i)^{\emph{mot}}\big)(-)$$ on essentially smooth $B$-schemes. \end{corollary} \begin{proof} The classical motivic complex $\Z(i)^{\text{cla}}$ is a Zariski sheaf which is locally supported in degrees at most $i$ (\cite[Corollary~$4.4$]{geisser_motivic_2004}), so the result is a consequence of Theorem~\ref{theoremcomparisonclassical-motivicforsmoothoverDedekinddomain}. \end{proof} \subsection{Comparison to classical motivic cohomology in low dimensions} \vspace{-\parindent} \hspace{\parindent} In this subsection, we study in more detail the defect for the classical-motivic comparison map to be an equivalence (Theorem~\ref{theoremclassicalmotiviccomparisonfibreintermsofétale}), and prove that this defect vanishes on smooth schemes of dimension less than or equal to one over a mixed characteristic Dedekind domain (Corollary~\ref{corollaryclassicalmotiviccomparisonsmalldimension}). \begin{lemma}\label{lemmaDcdhsheafificationofrigidisweaklyrigid} Let $B$ be a commutative ring, $\pi$ be an element of $B$, $\mathcal{C}$ be a presentable $\infty$-category, and $F : \emph{Alg}_{B} \rightarrow \mathcal{C}$ be a finitary and rigid functor. If the functor $F$ is zero on $B[\tfrac{1}{\pi}]$-algebas, then for every qcqs $B$-scheme $X$, the natural map $$\big(L_{\emph{cdh}} F\big)(X) \longrightarrow \big(L_{\emph{cdh}} F\big)(X_{B/\pi})$$ is an equivalence in the $\infty$-category $\mathcal{C}$. \end{lemma} \begin{proof} Covers in a site are stable under base change, and the cdh sheafification of a finitary presheaf is finitary, so the presheaf $$(L_{\text{cdh}} F)(-_{B/\pi})$$ is a finitary cdh sheaf on qcqs $B$-schemes. It then suffices to prove that for every henselian valuation ring $V$ which is a $B$-algebra, the natural map $$F(V) \longrightarrow \big(L_{\text{cdh}} F\big)(V/\pi)$$ is an equivalence in the $\infty$-category $\mathcal{C}$. The presheaf $L_{\text{cdh}} F$ is a finitary cdh sheaf, so it is invariant under nilpotent extensions. In particular, the natural map $$\big(L_{\text{cdh}} F\big)(V/\pi) \longrightarrow \big(L_{\text{cdh}} F\big)(V/\sqrt{(\pi)})$$ is an equivalence in the $\infty$-category $\mathcal{C}$. The quotient of a henselian valuation ring by one of its prime ideals is a henselian valuation ring, so the target of the previous map is naturally identified with the object $F(V/\sqrt{(\pi)}) \in \mathcal{C}$. We finally prove that the natural map $$F(V) \longrightarrow F(V/\sqrt{(\pi)})$$ is an equivalence in the $\infty$-category $\mathcal{C}$. If $\pi$ is invertible in the henselian valuation ring $V$, then both terms are zero by hypothesis on the functor $F$. And if $\pi$ is not invertible in $V$, then in particular the henselian local ring $V$ is $\pi$-henselian, and the result is a consequence of rigidity for the functor $F$. \end{proof} \begin{corollary}\label{corollaryEhighdegreecdhBMSsyntomiccohoisweaklyrigid} Let $p$ be a prime number, $B$ be a discrete valuation ring of mixed characteristic $(0,p)$, $\pi$ be a uniformizer of $B$, and $X$ be a qcqs $B$-scheme. Then for any integers $i \geq 0$ and $k \geq 1$, the natural map $$\big(L_{\emph{cdh}} \tau^{>i} \Z/p^k(i)^{\emph{BMS}}\big)(X) \longrightarrow \big(L_{\emph{cdh}} \tau^{>i} \Z/p^k(i)^{\emph{BMS}}\big)(X_{B/\pi})$$ is an equivalence in the derived category $\mathcal{D}(\Z/p^k)$. \end{corollary} \begin{proof} The functor $$\tau^{>i} \Z/p^k(i)^{\text{BMS}} : \text{Alg}_{B} \longrightarrow \mathcal{D}(\Z/p^k)$$ is finitary (Theorem~\ref{theoremBMSfiltrationonTCpcompletedsatisfiesquasisyntomicdescentandisLKEfrompolynomialalgebras}\,$(2)$) and rigid (Theorem~\ref{theoremAMMNrigidity}). The functor $\Z/p^k(i)^{\text{BMS}}$ is moreover zero on $\Z[\tfrac{1}{p}]$-algebras (Remark~\ref{remarkderivedpcompletionwithBMS}), so the result is a consequence of Lemma~\ref{lemmaDcdhsheafificationofrigidisweaklyrigid}. \end{proof} \begin{remark}\label{remarkhighdegreecdhsheafifiedsyntomicisweaklyrigidgeneralised} One can prove similarly that Corollary~\ref{corollaryEhighdegreecdhBMSsyntomiccohoisweaklyrigid} holds for $B$ a general valuation ring of mixed characteristic $(0,p)$, where the base change to the characteristic $p$ field $B/\pi$ is replaced by the base change to the characteristic $p$ valuation ring $B/\sqrt{(p)}$. \end{remark} \begin{proposition}\label{propositionclassicalhenselianpadic} Let $p$ be a prime number, $B$ be a discrete valuation ring of mixed characteristic~$(0,p)$, and $R$ be a henselian local ind-smooth $B$-algebra of residue characteristic $p$. Then for any integers $i \geq 0$ and $k \geq 1$, the natural map $$\tau^{>i} \Z/p^k(i)^{\emph{BMS}}(R) \longrightarrow \big(L_{\emph{cdh}} \tau^{>i} \Z/p^k(i)^{\emph{BMS}}\big)(R)$$ is an equivalence in the derived category $\mathcal{D}(\Z/p^k)$. \end{proposition} \begin{proof} Let $\pi$ be a uniformizer of the discrete valuation ring $B$, and consider the commutative diagram $$\begin{tikzcd} \tau^{>i} \Z/p^k(i)^{\text{BMS}}(R) \ar[r] \ar[d] & \big(L_{\text{cdh}} \tau^{>i} \Z/p^k(i)^{\text{BMS}}\big)(R) \ar[d] \\ \tau^{>i} \Z/p^k(i)^{\text{BMS}}(R/\pi) \ar[r] & \big(L_{\text{cdh}} \tau^{>i} \Z/p^k(i)^{\text{BMS}}\big)(R/\pi) \end{tikzcd}$$ in the derived category $\mathcal{D}(\Z/p^k)$. The commutative ring $R$ is $p$-henselian, hence $\pi$-henselian, so the left vertical map is an equivalence by Theorem~\ref{theoremAMMNrigidity}. The right vertical map is an equivalence by Corollary~\ref{corollaryEhighdegreecdhBMSsyntomiccohoisweaklyrigid}. We prove now that the bottom horizontal map is an equivalence. The commutative ring $R/\pi$ is a henselian local ind-smooth algebra over the field~$B/\pi$, so the classical-motivic comparison map $$\Z/p^k(i)^{\text{cla}}(R/\pi) \longrightarrow \Z/p^k(i)^{\text{mot}}(R/\pi)$$ is an equivalence in the derived category $\mathcal{D}(\Z/p^k)$ (\cite[Corollary~$6.4$]{elmanto_motivic_2023}). The complex $\Z/p^k(i)^{\text{cla}}(R) \in \mathcal{D}(\Z/p^k)$ is moreover in degrees at most $i$ since the commutative ring $R$ is henselian local, so this is equivalent to the fact that the natural map $$\tau^{>i} \Z/p^k(i)^{\text{BMS}}(R/\pi) \longrightarrow \big(L_{\text{cdh}} \tau^{>i} \Z/p^k(i)^{\text{BMS}}\big)(R/\pi)$$ is an equivalence in the derived $\mathcal{D}(\Z/p^k)$ (Theorem~\ref{theorempadicmotiviccohomologyintermsofsyntomicohomology}). \end{proof} \begin{theorem}\label{theoremclassicalmotiviccomparisonfibreintermsofétale} Let $p$ be a prime number, $B$ be a mixed characteristic Dedekind domain such that every characteristic $p$ residue field of $B$ is perfect, and $R$ be a henselian local ind-smooth $B$-algebra of residue characteristic $p$. Then for any integers $i \geq 0$ and $k \geq 1$, the fibre of the classical-motivic comparison map $$\Z/p^k(i)^{\emph{cla}}(R) \longrightarrow \Z/p^k(i)^{\emph{mot}}(R)$$ is in degrees at least $i+2$, given by the complex $$\big(L_{\emph{cdh}} \tau^{\leq i} R\Gamma_{\emph{ét}}(-,j_! \mu_{p^k}^{\otimes i})\big)(R)[-1] \in \mathcal{D}(\Z/p^k).$$ \end{theorem} \begin{proof} The fact that the fibre of the classical-motivic comparison map is in degrees at least $i+2$ is a special case of Proposition~\ref{propositionpadiccomparisonclassicalmotivicinsmalldegrees}. The henselian local ring $R$ is local for the Nisnevich topology, so the proof of Proposition~\ref{propositionpadiccomparisonclassicalmotivicinsmalldegrees} implies that the complex $\Z/p^k(i)^{\text{cla}}(R) \in \mathcal{D}(\Z/p^k)$ is moreover in degrees at most $i$, and this fibre is naturally identified with the complex $\big(\tau^{>i} \Z/p^k(i)^{\text{mot}}(R)\big)[-1] \in \mathcal{D}(\Z/p^k)$. By Theorem~\ref{theorempadicmotiviccohomologyintermsofsyntomicohomology}, this complex is in turn naturally identified with the complex $$\text{fib}\big(\tau^{>i} \Z/p^k(i)^{\text{syn}}(R) \longrightarrow \big(L_{\text{cdh}} \tau^{>i} \Z/p^k(i)^{\text{syn}}(R)\big)[-1] \in \mathcal{D}(\Z/p^k).$$ Consider the commutative diagram $$\begin{tikzcd} \tau^{>i} \Z/p^k(i)^{\text{syn}}(R) \ar[r] \ar[d] & \big(L_{\text{cdh}} \tau^{>i} \Z/p^k(i)^{\text{syn}}\big)(R) \ar[d] \\ \tau^{>i} \Z/p^k(i)^{\text{BMS}}(R) \ar[r] & \big(L_{\text{cdh}} \tau^{>i} \Z/p^k(i)^{\text{BMS}}\big)(R) \end{tikzcd}$$ in the derived category $\mathcal{D}(\Z/p^k)$. The commutative ring $R$ is $p$-henselian, so the left vertical map is an equivalence (Notation~\ref{notationsyntomiccohomology}). The commutative ring $R$ is local, so there exists a prime ideal $\mathfrak{p}$ of the Dedekind domain $B$ such that $p \in \mathfrak{p}$ and $R$ is essentially smooth over the localisation $B_{\mathfrak{p}}$. The local ring $B_{\mathfrak{p}}$ is a discrete valuation ring of mixed characteristic $(0,p)$, so the bottom horizontal map is an equivalence (Proposition~\ref{propositionclassicalhenselianpadic}). The fibre of the top horizontal map is then naturally identified with the $(-1)$-cohomological shift of the fibre of the right vertical map. By Lemma~\ref{lemmaGfibresequencecdhhighdegreesyntomiccohomologywithétaleandBMS} (applied for $j=i$ and after cdh sheafification), this fibre is naturally identified with the complex $$\big(L_{\text{cdh}} \tau^{>i} R\Gamma_{\text{ét}}(-,j_! \mu_{p^k}^{\otimes i})\big)(R) \in \mathcal{D}(\Z/p^k).$$ The functor $R\Gamma_{\text{ét}}(-,j_! \mu_{p^k}^{\otimes i})$ is rigid (\cite{gabber_affine_1994}, see also \cite[Corollary~$1.18\,(1)$]{bhatt_arc-topology_2021}) and satisfies cdh descent (\cite[Theorem~$5.4$]{bhatt_arc-topology_2021}), so the object $$\big(L_{\text{cdh}} R\Gamma_{\text{ét}}(-,j_!\mu_{p^k}^{\otimes i})\big)(R)$$ is zero in the derived category $\mathcal{D}(\Z/p^k)$ for the $p$-henselian commutative ring $R$. In particular, there is a natural equivalence $$\big(L_{\text{cdh}} \tau^{>i} R\Gamma_{\text{ét}}(-,j_! \mu_{p^k}^{\otimes i})\big)(R) \simeq \big(L_{\text{cdh}} \tau^{\leq i} R\Gamma_{\text{ét}}(-,j_! \mu_{p^k}^{\otimes i})\big)(R)[1]$$ in the derived category $\mathcal{D}(\Z/p^k)$, which implies the desired equivalence. \end{proof} \begin{corollary}\label{corollaryclassicalmotiviccomparisonsmalldimension} Let $B$ be a mixed characteristic Dedekind domain such that every residue field of $B$ is perfect, and $X$ be a smooth $B$-scheme of dimension less than or equal to one over $B$. Then for every integer $i \geq 0$, the classical-motivic comparison map $$\Z(i)^{\emph{cla}}(X) \longrightarrow \Z(i)^{\emph{mot}}(X)$$ is an equivalence in the derived category $\mathcal{D}(\Z)$. \end{corollary} \begin{proof} The presheaves $\Z(i)^{\text{cla}}$ and $\Z(i)^{\text{mot}}$ are finitary Nisnevich sheaves, hence it suffices to prove the result on the henselisation of every local ring of the scheme $X$. Let $R$ be such a henselian local ring, which is then a henselian local ind-smooth $B$-algebra of Krull dimension at most three. The result rationally is a special case of Proposition~\ref{propositionrationalcomparisonclassicalmotivic}, so it suffices to prove the result modulo $p$, for every prime number $p$. Let $p$ be a prime number. If $p$ is invertible in the local ring $R$, this is Proposition~\ref{propositionpadiccomparisonclassicalmotivicinsmalldegrees}. Assume now that $p$ is not invertible in the henselian local ring $R$, and in particular that the ring $R$ is $p$-henselian. By Theorem~\ref{theoremclassicalmotiviccomparisonfibreintermsofétale}, the fibre of the classical-motivic comparison map $$\F_p(i)^{\text{cla}}(R) \longrightarrow \F_p(i)^{\text{mot}}(R)$$ is in degrees at least $i+2$, given by the complex $$\big(L_{\text{cdh}} \tau^{\leq i} R\Gamma_{\text{ét}}(-,j_!\mu_p^{\otimes i})\big)(R)[-1] \in \mathcal{D}(\F_p).$$ The commutative ring $R$ is a noetherian ring of Krull dimension at most two, hence of valuative dimension at most two. So this complex is also in degrees at most $i+3$ (\cite[Theorem~$2.4.15$]{elmanto_cdh_2021}), and it is thus only in degrees $i+2$ and $i+3$. The complexes $\F_p(i)^{\text{cla}}(R)$ and $\F_p(i)^{\text{mot}}(R)$ (for $i \geq 0$) are the graded pieces of the $\N$-indexed complete filtrations $\text{Fil}^\star_{\text{cla}} \text{K}(R;\F_p)$ and $\text{Fil}^\star_{\text{mot}} \text{K}(X;\F_p)$ on the algebraic $K$-theory of $R$. In particular, there is natural spectral sequence $$E_2^{i,j} = \text{H}^{i-j}\big(\big(L_{\text{cdh}} \tau^{\leq -j} R\Gamma_{\text{ét}}(-,j_! \mu_p^{\otimes (-j)})\big)(R)\big) \Longrightarrow 0.$$ The previous cohomological bound implies that all the differentials in this spectral sequence are zero, so this spectral sequence degenerates. This implies the desired equivalence. \end{proof} \newpage \section{Comparison to Milnor \texorpdfstring{$K$}{TEXT}-theory and lisse motivic cohomology} \vspace{-\parindent} \hspace{\parindent} In this section, we study the motivic complexes $\Z(i)^{\text{mot}}$ on local rings. We prove that these are left Kan extended, in degrees at most $i$, from local essentially smooth $\Z$-algebras (Theorem~\ref{theoremmotiviccohomologyisleftKanextendedonlocalringsinsmalldegrees}). Via the comparison to classical motivic cohomology in the smooth case (Theorem~\ref{theoremcomparisonclassical-motivicforsmoothoverDedekinddomain}), this means that the motivic complexes~$\Z(i)^{\text{mot}}$ are controlled, up to degree~$i$, by classical motivic cohomology (Corollary~\ref{corollarylissemotivicmaincomparisontheorem}). We use the latter result to construct a comparison map from the $i^{\text{th}}$ (improved) Milnor $K$-group of a general local ring $A$ to the motivic cohomology group $\text{H}^i_{\text{mot}}(A,\Z(i))$, and prove that this map is an isomorphism with finite coefficients (Theorem~\ref{theoremcomparisontoMilnorKtheory}). \subsection{Comparison to lisse motivic cohomology} \vspace{-\parindent} \hspace{\parindent} In this subsection, we prove a comparison between motivic cohomology and lisse motivic cohomology on general local rings (Corollary~\ref{corollarylissemotivicmaincomparisontheorem}), which generalises to mixed characteristic the analogous comparison result of Elmanto--Morrow over a field (\cite[Theorem~$7.7$]{elmanto_motivic_2023}). To do so, we use the following comparison map. \begin{definition}[Lisse-motivic comparison map]\label{definitionlissemotiviccomparisonmap} For every integer $i \in \Z$, the {\it lisse-motivic comparison map} is the map $$\Z(i)^{\text{lisse}}(-) \longrightarrow \Z(i)^{\text{mot}}(-)$$ of functors from animated commutative rings to the derived category $\mathcal{D}(\Z)$ defined as the composite $$\big(L_{\text{AniRings}/\text{Sm}_{\Z}} \Z(i)^{\text{cla}}\big)(-) \longrightarrow \big(L_{\text{AniRings}/\text{Sm}_{\Z}} \Z(i)^{\text{mot}}\big)(-) \longrightarrow \Z(i)^{\text{mot}}(-),$$ where the first map is the map induced by Definition~\ref{definitionclassicalmotiviccomparisonmap} and the second map is the canonical map. \end{definition} \begin{lemma}\label{lemmarationalmotiviccohoisLKEindegreesatmost2i} For every integer $i \geq 0$, the functor $$\tau^{\leq 2i} \Q(i)^{\emph{mot}}(-) : \emph{AniRings} \longrightarrow \mathcal{D}(\Q)$$ is left Kan extended from smooth $\Z$-algebras. \end{lemma} \begin{proof} By \cite[Example~$1.0.6$]{elmanto_modules_2020}, connective algebraic $K$-theory $$\tau^{\leq 0} \text{K}(-;\Q) : \text{AniRings} \longrightarrow \mathcal{D}(\Q)$$ is left Kan extended from smooth $\Z$-algebras. By Corollary~\ref{corollaryKtheorysplitsrationally}, this implies that the functor $$\bigoplus_{i \geq 0} \tau^{\leq 0}\big(\Q(i)^{\text{mot}}(-)[2i]\big) : \text{AniRings} \longrightarrow \mathcal{D}(\Q)$$ is left Kan extended from smooth $\Z$-algebras, which is equivalent to the desired result. \end{proof} \begin{corollary}\label{corollaryrationallissemotiviccomparison} Let $R$ be an animated commutative ring. Then for every integer $i \geq 0$, the lisse-motivic comparison map induces a natural equivalence $$\Q(i)^{\emph{lisse}}(R) \xlongrightarrow{\sim} \tau^{\leq 2i} \Q(i)^{\emph{mot}}(R)$$ in the derived category $\mathcal{D}(\Q)$. \end{corollary} \begin{proof} If $R$ is a smooth $\Z$-algebra, the result is a consequence of Proposition~\ref{propositionrationalcomparisonclassicalmotivic} and the fact that, by construction, the classical motivic complex $\Z(i)^{\text{cla}}(R) \in \mathcal{D}(\Z)$ is in degrees at most~$2i$. In general, this is then a consequence of Lemma~\ref{lemmarationalmotiviccohoisLKEindegreesatmost2i}. \end{proof} \begin{proposition}\label{propositionrationalcomparisonlissemotivicforlocalrings} Let $R$ be a local ring. Then for every integer $i \geq 0$, the lisse-motivic comparison map induces a natural equivalence $$\Q(i)^{\emph{lisse}}(R) \longrightarrow \tau^{\leq i} \Q(i)^{\emph{mot}}(R)$$ in the derived category $\mathcal{D}(\Q)$. Moreover, the motivic cohomology group $\emph{H}^j_{\emph{mot}}(R,\Q(i))$ is zero for \hbox{$i < j \leq 2i$}. \end{proposition} \begin{proof} The classical motivic complex $\Z(i)^{\text{cla}}(-)$ is Zariski-locally in degrees at most $i$ (\cite[Corollary~$4.4$]{geisser_motivic_2004}). By taking left Kan extension, this implies that the lisse motivic complex $\Z(i)^{\text{lisse}}(-)$ is also Zariski-locally in degrees at most $i$. In particular, the lisse motivic complex $\Q(i)^{\text{lisse}}(R)$ is in degrees at most $i$. The result is then a consequence of Corollary~\ref{corollaryrationallissemotiviccomparison}. \end{proof} \begin{remark}\label{remarkDrinfeldtheoremK-1onhenselianlocalrings} By Drinfeld's theorem (\cite[Theorem~$3.7$]{drinfeld_infinite_2006}), the $K$-group $\text{K}_{-1}(R)$ vanishes for every henselian local ring $R$. By Corollary~\ref{corollaryKtheorysplitsrationally}, this implies that for every integer $i \geq 0$, the motivic cohomology group $\text{H}^{2i+1}_{\text{mot}}(R,\Q(i))$ is zero, {\it i.e.}, that the motivic cohomology group $\text{H}^{2i+1}_{\text{mot}}(R,\Z(i))$ is torsion. \end{remark} \begin{corollary}\label{corollaryrationalmotiviccohomologyisleftKanextendedinsmalldegrees} For every integer $i \geq 0$, the functor $\tau^{\leq i} \Q(i)^{\emph{mot}}$, from local rings to the derived category $\mathcal{D}(\Q)$, is left Kan extended from local essentially smooth $\Z$-algebras. \end{corollary} \begin{proof} This is a consequence of Lemma~\ref{lemmarationalmotiviccohoisLKEindegreesatmost2i} and Proposition~\ref{propositionrationalcomparisonlissemotivicforlocalrings}. \end{proof} \begin{proposition}\label{propositionmodpmotiviccohoisleftKanextendedinsmalldegrees} Let $p$ be a prime number, and $k$ be an integer. Then for every integer~\hbox{$i \geq 0$}, the functor $$\tau^{\leq i} \Z/p^k(i)^{\emph{mot}}(-) : \emph{Rings} \longrightarrow \mathcal{D}(\Z/p^k)$$ is left Kan extended from smooth $\Z$-algebras. \end{proposition} \begin{proof} By Theorem~\ref{theorempadicmotiviccohomologyintermsofsyntomicohomology}, this is equivalent to the fact that the functor $\tau^{\leq i} \Z/p^k(i)^{\text{syn}}(-)$ on commutative rings is left Kan extended from smooth $\Z$-algebras. The functor $\Z/p^k(i)^{\text{syn}}(-)$ is left Kan extended from smooth $\Z$-algebras (Notation~\ref{notationsyntomiccohomology}), so this is equivalent to the fact that the functor $\tau^{>i} \Z/p^k(i)^{\text{syn}}(-)$ on commutative rings is left Kan extended from smooth $\Z$-algebras. By \cite[Lemma~$7.6$]{elmanto_motivic_2023}, it then suffices to prove that the functor $\tau^{>i} \Z/p^k(i)^{\text{syn}}(-)$ is rigid. To prove this, consider the fibre sequence of $\mathcal{D}(\Z/p^k)$-valued functors $$R\Gamma_{\text{ét}}(-,j_! \mu_{p^k}^{\otimes i}) \longrightarrow \Z/p^k(i)^{\text{syn}}(-) \longrightarrow \Z/p^k(i)^{\text{BMS}}(-)$$ on commutative rings (\cite[Remark~$8.4.4$]{bhatt_absolute_2022}). By rigidity for étale cohomology (\cite{gabber_affine_1994}, see also \cite[Corollary~$1.18\,(1)$]{bhatt_arc-topology_2021}), the first term of this fibre sequence is rigid. The desired result is then a consequence of Theorem~\ref{theoremAMMNrigidity}. \end{proof} \begin{corollary}\label{corollarymodpmotiviccohomologyisleftKanextendedinsmalldegreesonlocalrings} Let $p$ be a prime number, and $k \geq 1$ be an integer. Then for every integer~\hbox{$i \geq 0$}, the functor $\tau^{\leq i} \Z/p^k(i)^{\emph{mot}}$, from local rings to the derived category $\mathcal{D}(\Z/p^k)$, is left Kan extended from local essentially smooth $\Z$-algebras. \end{corollary} \begin{proof} This is a consequence of Proposition~\ref{propositionmodpmotiviccohoisleftKanextendedinsmalldegrees}. \end{proof} \begin{lemma}\label{lemmatopdegreeintegraltomodpissurjective} Let $R$ be a local ring, $p$ be a prime number, and $k \geq 1$ be an integer. Then for every integer $i \geq 0$, the natural map of abelian groups $$\emph{H}^i_{\emph{mot}}(R,\Z(i)) \longrightarrow \emph{H}^i_{\emph{mot}}(R,\Z/p^k(i))$$ is surjective. \end{lemma} \begin{proof} Let $P \rightarrow R$ be a henselian surjection, where $P$ is a local ind-smooth $\Z$-algebra. By Corollary~\ref{corollarymodpmotiviccohomologyisleftKanextendedinsmalldegreesonlocalrings}, the functor $\tau^{\leq i} \Z/p^k(i)^{\text{mot}}$ is left Kan extended on local rings from local essentially smooth $\Z$-algebras, so the natural map of abelian groups $$\text{H}^i_{\text{mot}}(P,\Z/p^k(i)) \longrightarrow \text{H}^i_{\text{mot}}(R,\Z/p^k(i))$$ is surjective. That is, the right vertical map in the commutative diagram of abelian groups $$\begin{tikzcd} \text{H}^i_{\text{mot}}(P,\Z(i)) \ar[r] \ar[d] & \text{H}^i_{\text{mot}}(P,\Z/p^k(i)) \ar[d] \\ \text{H}^i_{\text{mot}}(R,\Z(i)) \ar[r] & \text{H}^i_{\text{mot}}(R,\Z/p^k(i)) \end{tikzcd}$$ is surjective. To prove that the bottom horizontal map is surjective, it thus suffices to prove that the top vertical map is surjective. The local ring $P$ is a filtered colimit of local essentially smooth $\Z$-algebras, so it suffices to prove that this top vertical map is surjective for local essentially smooth $\Z$-algebras. To prove this, it suffices to prove that the motivic complex $\Z(i)^{\text{mot}}(-)$ is zero in degree~$i+1$ on local essentially smooth $\Z$-algebras. The classical motivic complex $\Z(i)^{\text{cla}}(-)$ is Zariski-locally in degrees at most $i$ (\cite[Corollary~$4.4$]{geisser_motivic_2004}), so this is a consequence of Theorem~\ref{theoremcomparisonclassical-motivicforsmoothoverDedekinddomain}. \end{proof} \begin{corollary}\label{corollaryHilbert90} Let $R$ be a local ring. Then for every integer $i \geq 1$, the motivic cohomology group $\emph{H}^{i+1}_{\emph{mot}}(R,\Z(i))$ is zero. If the local ring $R$ is moreover henselian, then the motivic cohomology group $\emph{H}^1_{\emph{mot}}(R,\Z(0))$ is zero. \end{corollary} \begin{proof} By Lemma~\ref{lemmatopdegreeintegraltomodpissurjective} and the short exact sequence of abelian groups $$0 \longrightarrow \text{H}^i_{\text{mot}}(R,\Z(i))/p \longrightarrow \text{H}^i_{\text{mot}}(R,\F_p(i)) \longrightarrow \text{H}^{i+1}_{\text{mot}}(R,\Z(i))[p] \longrightarrow 0$$ for every prime number $p$ and every integer $i \geq 0$, the abelian group $\text{H}^{i+1}_{\text{mot}}(R,\Z(i))$ is torsionfree. By Proposition~\ref{propositionrationalcomparisonlissemotivicforlocalrings} if $i \geq 1$, and by Remark~\ref{remarkDrinfeldtheoremK-1onhenselianlocalrings} if $i=0$ and $R$ is henselian, it is also torsion, so it is zero. \end{proof} \begin{theorem}\label{theoremmotiviccohomologyisleftKanextendedonlocalringsinsmalldegrees} For every integer $i \geq 0$, the functor $\tau^{\leq i} \Z(i)^{\emph{mot}}$, from local rings to the derived category $\mathcal{D}(\Z)$, is left Kan extended from local essentially smooth $\Z$-algebras. \end{theorem} \begin{proof} It suffices to prove the result rationally, and modulo $p$ for every prime number $p$. The result rationally is Corollary~\ref{corollaryrationalmotiviccohomologyisleftKanextendedinsmalldegrees}. Let $p$ be a prime number. For every local ring $R$, the natural map of abelian groups $$\text{H}^i_{\text{mot}}(R,\Z(i)) \longrightarrow \text{H}^i_{\text{mot}}(R,\F_p(i))$$ is surjective by Lemma~\ref{lemmatopdegreeintegraltomodpissurjective}, so the natural map $$\big(\tau^{\leq i} \Z(i)^{\text{mot}}(R)\big)/p \longrightarrow \tau^{\leq i} \F_p(i)^{\text{mot}}(R)$$ is an equivalence in the derived category $\mathcal{D}(\F_p)$. The result modulo $p$ is then Corollary~\ref{corollarymodpmotiviccohomologyisleftKanextendedinsmalldegreesonlocalrings}. \end{proof} Note that the proof of Theorem~\ref{theoremmotiviccohomologyisleftKanextendedonlocalringsinsmalldegrees} is similar to the proof of Elmanto--Morrow in equicharacteristic. The following consequence, however, uses the comparison to classical motivic cohomology Theorem~\ref{theoremcomparisonclassical-motivicforsmoothoverDedekinddomain}. The proof of the latter, in any characteristic, is somehow simpler than the proof of Elmanto--Morrow's (stronger) comparison result to classical motivic cohomology: in particular, it does not use a presentation lemma, or the projective bundle formula. The proof of Corollary~\ref{corollarylissemotivicmaincomparisontheorem} then provides an alternative argument to the proof of \cite[Theorem~$7.7$]{elmanto_motivic_2023}. \begin{corollary}[Comparison to lisse motivic cohomology]\label{corollarylissemotivicmaincomparisontheorem} Let $R$ be a local ring. Then for every integer $i \geq 0$, the lisse-motivic comparison map induces a natural equivalence $$\Z(i)^{\emph{lisse}}(R) \xlongrightarrow{\sim} \tau^{\leq i} \Z(i)^{\emph{mot}}(R)$$ in the derived category $\mathcal{D}(\Z)$. \end{corollary} \begin{proof} The classical motivic complex $\Z(i)^{\text{cla}}(-)$ is Zariski-locally in degrees at most $i$ (\cite[Corollary~$4.4$]{geisser_motivic_2004}). The result is then a consequence of Theorems~\ref{theoremcomparisonclassical-motivicforsmoothoverDedekinddomain} and~\ref{theoremmotiviccohomologyisleftKanextendedonlocalringsinsmalldegrees}. \end{proof} In the rest of this section, we restrict our attention to henselian local rings, in order to describe the motivic cohomology group $\text{H}^2_{\text{mot}}(-,\Z(1))$. \begin{lemma}\label{lemmavanishingpadici+1} Let $R$ be a henselian local ring, and $p$ be a prime number. Then for any integers $i \geq 0$ and $k \geq 1$, the motivic cohomology group $\emph{H}^{i+1}_{\emph{mot}}(R,\Z/p^k(i))$ is zero. \end{lemma} \begin{proof} By Theorem~\ref{theorempadicmotiviccohomologyintermsofsyntomicohomology}, the motivic cohomology group $\text{H}^{i+1}_{\text{mot}}(R,\Z/p^k(i))$ is naturally identified with the kernel of the natural map of abelian groups $$\text{H}^{i+1}(\Z/p^k(i)^{\text{syn}}(R)) \longrightarrow \text{H}^{i+1}((L_{\text{cdh}} \tau^{>i} \Z/p^k(i)^{\text{syn}})(R))$$ for every commutative ring $R$. If $R$ is henselian local, let $\mathfrak{m}$ be its maximal ideal, and consider the natural commutative diagram $$\begin{tikzcd} \text{H}^{i+1}(\Z/p^k(i)^{\text{syn}}(R)) \ar[r] \ar[d] & \text{H}^{i+1}((L_{\text{cdh}} \tau^{>i} \Z/p^k(i)^{\text{syn}})(R)) \ar[d] \\ \text{H}^{i+1}(\Z/p^k(i)^{\text{syn}}(R/\mathfrak{m})) \ar[r] & \text{H}^{i+1}((L_{\text{cdh}} \tau^{>i} \Z/p^k(i)^{\text{syn}})(R/\mathfrak{m})) \end{tikzcd}$$ of abelian groups. The functor $\tau^{>i} \Z/p^k(i)^{\text{syn}}$ is rigid (proof of Proposition~\ref{propositionmodpmotiviccohoisleftKanextendedinsmalldegrees}), so the left vertical map is an isomorphism. The field $R/\mathfrak{m}$ is a local ring for the cdh topology, so the bottom horizontal map is an isomorphism. In particular, the top horizontal map is injective. \end{proof} \begin{proposition}\label{propositionweightoneH3(1)vanishes} Let $R$ be a henselian local ring. Then for every integer $i \geq 1$, the motivic cohomology group $\emph{H}^{i+2}_{\emph{mot}}(R,\Z(i))$ is zero. \end{proposition} \begin{proof} By Lemma~\ref{lemmavanishingpadici+1} and the short exact sequence of abelian groups $$0 \longrightarrow \text{H}^{i+1}_{\text{mot}}(R,\Z(i))/p \longrightarrow \text{H}^{i+1}_{\text{mot}}(R,\F_p(i)) \longrightarrow \text{H}^{i+2}_{\text{mot}}(R,\Z(i))[p] \longrightarrow 0$$ for every prime number $p$, the abelian group $\text{H}^{i+2}_{\text{mot}}(R,\Z(i))$ is torsionfree. By Proposition~\ref{propositionrationalcomparisonlissemotivicforlocalrings} if $i \geq 2$, and by Remark~\ref{remarkDrinfeldtheoremK-1onhenselianlocalrings} if $i=1$, it is also torsion, so it is zero. \end{proof} The following example is a consequence of Example~\ref{exampleweight1lissemotiviccohomology}, Corollary~\ref{corollarylissemotivicmaincomparisontheorem}, and Proposition~\ref{propositionweightoneH3(1)vanishes}. \begin{example}\label{exampleweightonemotiviccohomology} For every qcqs scheme $X$, the natural map $$R\Gamma_{\text{Nis}}(X,\mathbb{G}_m)[-1] \longrightarrow \Z(1)^{\text{mot}}(X),$$ defined as the Nisnevich sheafification of the lisse-motivic comparison map (see also Definition~\ref{definitionmotivicfirstChernclass}), is an isomorphism in degrees at most three. That is, the motivic complex $\Z(1)^{\text{mot}}(X)$ vanishes in degrees at most zero, and there are natural isomorphisms of abelian groups $$\text{H}^1_{\text{mot}}(X,\Z(1)) \cong \mathcal{O}(X)^{\times}, \quad \text{H}^2_{\text{mot}}(X,\Z(1)) \cong \text{Pic}(X), \quad \text{H}^3_{\text{mot}}(X,\Z(1)) \cong \text{H}^2_{\text{Nis}}(X,\mathbb{G}_m).$$ \end{example} \subsection{Comparison to Milnor \texorpdfstring{$K$}{TEXT}-theory} \vspace{-\parindent} \hspace{\parindent} In this subsection, we construct, for every integer $i \geq 1$, a symbol map $$\text{K}^{\text{M}}_i(A) \longrightarrow \text{H}^i_{\text{mot}}(A,\Z(i))$$ for local rings $A$ (Definition~\ref{definitionsymbolmap}), through which we compare the Milnor $K$-groups to motivic cohomology (Theorem~\ref{theoremcomparisontoMilnorKtheory}). Note that the arguments in Lemmas~\ref{lemmasymbolmapmilnorKtheorywelldefined} and \ref{lemmasymbolmapfactorsthoughimprovedMilnorKgroup} are very similar to that of \cite[Section~$7$]{elmanto_motivic_2023}, except for the Gersten injectivity for classical motivic cohomology, which is unknown integrally in mixed characteristic. For every commutative ring $A$, the lisse-motivic comparison map (Definition~\ref{definitionlissemotiviccomparisonmap} and Example~\ref{exampleweight1lissemotiviccohomology}) induces on $\text{H}^1$ a natural isomorphism of abelian groups $$A^\times \xlongrightarrow{\cong} \text{H}^1_{\text{mot}}(A,\Z(1)).$$ By multiplicativity of the motivic complexes, this induces, for every integer $i \geq 0$, a symbol map of abelian groups $$(A^{\times})^{\otimes i} \longrightarrow \text{H}^i_{\text{mot}}(A,\Z(i)).$$ \begin{lemma}\label{lemmaMilnorK_2} For every local essentially smooth $\Z$-algebra $A$, there is a natural isomorphism $$\emph{H}^2_{\emph{mot}}(A,\Z(2)) \cong \emph{K}_2(A)$$ of abelian groups. \end{lemma} \begin{proof} By Theorem~\ref{theoremcomparisonclassical-motivicforsmoothoverDedekinddomain}, the classical-motivic comparison map $$\text{H}^2_{\text{cla}}(A,\Z(2)) \longrightarrow \text{H}^2_{\text{mot}}(A,\Z(2))$$ is an isomorphism of abelian groups. The result is then a consequence of the Atiyah--Hirzebruch spectral sequence for classical motivic cohomology (Remark~\ref{remarkclassicalBlochrelatedtoslice}), where we use that the classical motivic complex $\Z(1)^{\text{cla}}(A) \in \mathcal{D}(\Z)$ is concentrated in degree one (Example~\ref{exampleweight1classicalmotiviccohomology} and \cite[Corollary~$4.4$]{geisser_motivic_2004}). \end{proof} \begin{lemma}\label{lemmasymbolmapmilnorKtheorywelldefined} Let $A$ be a local ring. Then for every integer $i \geq 0$, the natural map of abelian groups $$(A^{\times})^{\otimes i} \longrightarrow \emph{H}^i_{\emph{mot}}(A,\Z(i))$$ induced by the lisse-motivic comparison map factors through the Milnor $K$-group $\emph{K}^{\emph{M}}_i(A)$. \end{lemma} \begin{proof} By definition of the Milnor $K$-groups, it suffices to prove that the symbol map respects the Steinberg relations. Let $a \in A$ be an element such that $a$ and $1-a$ are units in $A$. By multiplicativity of the motivic complexes, it suffices to consider the case $i=2$ and to prove that $a \otimes (1-a)$ is sent to zero via the symbol map. Let $\Z[t] \rightarrow A$ be the ring homomorphism sending $t$ to $a$, and let $\mathfrak{p} \subset \Z[t]$ be the prime ideal defined as the inverse image of the maximal ideal of $A$ via this ring homomorphism. By naturality of the symbol map, the diagram of abelian groups $$\begin{tikzcd} (\Z[t]_{\mathfrak{p}})^{\times} \otimes_{\Z} (\Z[t]_{\mathfrak{p}})^{\times} \ar[r] \ar[d] & \text{H}^2_{\text{mot}}(\Z[t]_{\mathfrak{p}},\Z(2)) \ar[d] \\ A^{\times} \otimes_{\Z} A^{\times} \ar[r] & \text{H}^2_{\text{mot}}(A,\Z(2)) \end{tikzcd}$$ is commutative. It then suffices to prove that the top horizontal arrow of this diagram sends $t \otimes (1-t)$ to zero. The local ring $\Z[t]_{\mathfrak{p}}$ is essentially smooth over $\Z$, so the right vertical map of the commutative diagram of abelian groups $$\begin{tikzcd} (\Z[t]_{\mathfrak{p}})^{\times} \otimes_{\Z} (\Z[t]_{\mathfrak{p}})^{\times} \ar[r] \ar[d] & \text{H}^2_{\text{mot}}(\Z[t]_{\mathfrak{p}},\Z(2)) \ar[d] \\ (\text{Frac}(\Z[t]_{\mathfrak{p}}))^{\times} \otimes_{\Z} (\text{Frac}(\Z[t]_{\mathfrak{p}}))^{\times} \ar[r] & \text{H}^2_{\text{mot}}(\text{Frac}(\Z[t]_{\mathfrak{p}}),\Z(2)) \end{tikzcd}$$ is injective. Indeed, by Lemma~\ref{lemmaMilnorK_2} this is equivalent to the fact that the natural map $$\text{K}_2(A) \longrightarrow \text{K}_2(\text{Frac}(A))$$ is injective, and this is the Gersten injectivity for $\text{K}_2$ (\cite[Corollary~$6$]{gillet_relative_1987} and \cite[Theorem~$2.2$]{dennis_K2_1975}). It then suffices to prove that the bottom horizontal map of this diagram sends $t \otimes (1-t)$ to zero. This is a consequence of the fact that the symbol map to classical motivic cohomology respects the Steinberg relation for fields (\cite{nesterenko_homology_1989}, see also \cite{totaro_milnor_1992}). \end{proof} \begin{definition}[Symbol map]\label{definitionsymbolmap} Let $A$ be a local ring. For every integer $i \geq 0$, the {\it symbol map} $$\text{K}^{\text{M}}_{\text{i}}(A) \longrightarrow \text{H}^i_{\text{mot}}(A,\Z(i))$$ is the natural map of abelian groups of Lemma~\ref{lemmasymbolmapmilnorKtheorywelldefined}. \end{definition} Following \cite{kerz_milnor_2010}, for $A$ a local ring and $i \geq 0$ an integer, we denote by $\widehat{\text{K}}{}^{\text{M}}_i(A)$ the $i^{\text{th}}$ {\it improved Milnor $K$-group} of $A$. \begin{lemma}\label{lemmasymbolmapfactorsthoughimprovedMilnorKgroup} Let $A$ be a local ring. Then for every integer $i \geq 0$, the symbol map $$\emph{K}^{\emph{M}}_i(A) \longrightarrow \emph{H}^i_{\emph{mot}}(A,\Z(i))$$ factors through the improved Milnor $K$-group $\widehat{\emph{K}}{}^{\emph{M}}_i(A)$. \end{lemma} \begin{proof} Let $M_i \geq 1$ be the integer defined in \cite{kerz_milnor_2010}. If the residue field of the local ring $A$ has at least $M_i$ elements, then the natural map $$\text{K}^{\text{M}}_i(A) \longrightarrow \widehat{\text{K}}{}^{\text{M}}_i(A)$$ is an isomorphism of abelian groups (\cite[Proposition~$10\,(5)$]{kerz_milnor_2010}). Assume now that the residue field of the local ring $A$ has less than $M_i$ elements. We want to prove that the symbol map $$\text{K}{}^{\text{M}}_i(A) \longrightarrow \text{H}^i_{\text{mot}}(A,\Z(i))$$ factors through the surjective map $\text{K}{}^{\text{M}}_i(A) \rightarrow \widehat{\text{K}}{}^{\text{M}}_i(A)$, {\it i.e.}, that every element of the abelian group $\text{ker}(\text{K}{}^{\text{M}}_i(A) \rightarrow \widehat{\text{K}}{}^{\text{M}}_i(A))$ is sent to zero by the previous symbol map. Let $\mathfrak{m}$ be the maximal ideal of the local ring $A$, and $p$ be its residue characteristic. The residue field~$A/\mathfrak{m}$ of the local ring $A$ is isomorphic to a finite extension $\F_q$ of $\F_p$. Let $\l \geq 1$ be an integer which is coprime to the degree of this extension, and such that $p^{\l} \geq M_i$. As a tensor product of finite field extensions of coprime degree, the commutative ring $\F_q \otimes_{\F_p} \F_{p^{\l}}$ is a field. Let $V$ be the finite étale extension of $\Z_{(p)}$ corresponding to the field extension $\F_{p^\l}$ of $\F_p$. The commutative ring $A' := A \otimes_{\Z_{(p)}} V$ is finite over the local ring $A$, and the quotient $A'/\mathfrak{m}A'$ is a field, so the commutative ring $A'$ is a local $A$-algebra, whose residue field has at least $M_i$ elements. Let $P_{\bullet} \rightarrow A$ be a simplicial resolution of the local ring $A$ where each term $P_m$ is a local ind-smooth $\Z$-algebra, and each face map $P_{m+1} \rightarrow P_m$ is a henselian surjection. By Theorem~\ref{theoremmotiviccohomologyisleftKanextendedonlocalringsinsmalldegrees}, there is then a natural equivalence $$\underset{m}{\text{colim}}\, \tau^{\leq i} \Z(i)^{\text{mot}}(P_m) \xlongrightarrow{\sim} \tau^{\leq i} \Z(i)^{\text{mot}}(A)$$ in the derived category $\mathcal{D}(\Z)$. In particular, this equivalence induces a natural isomorphism $$\text{coeq}\big(\text{H}^i_{\text{mot}}(P_1,\Z(i)) \hspace{1mm} \substack{\longrightarrow\\[-0.9em] \longrightarrow} \hspace{1mm} \text{H}^i_{\text{mot}}(P_0,\Z(i))\big) \xlongrightarrow{\cong} \text{H}^i_{\text{mot}}(A,\Z(i))$$ of abelian groups, where the motivic cohomology groups in the left term are naturally identified with classical motivic cohomology groups by Theorem~\ref{theoremcomparisonclassical-motivicforsmoothoverDedekinddomain}. Similarly, $P_{\bullet} \otimes_{\Z_{(p)}} V \rightarrow A'$ is a simplicial resolution of the local ring $A'$ where each term $P_m \otimes_{\Z_{(p)}} V$ is an ind-smooth $V$\nobreakdash-algebra, and each face map $P_{m+1} \otimes_{\Z_{(p)}} V \rightarrow P_m \otimes_{\Z_{(p)}} V$ is a henselian surjection, so there is a natural isomorphism $$\text{coeq}\big(\text{H}^i_{\text{mot}}(P_1 \otimes_{\Z_{(p)}} V,\Z(i)) \hspace{1mm} \substack{\longrightarrow\\[-0.9em] \longrightarrow} \hspace{1mm} \text{H}^i_{\text{mot}}(P_0 \otimes_{\Z_{(p)}} V,\Z(i))\big) \xlongrightarrow{\cong} \text{H}^i_{\text{mot}}(A',\Z(i))$$ of abelian groups, where the motivic cohomology groups of the left term are naturally identified with classical motivic cohomology groups. Classical motivic cohomology of smooth schemes over a mixed characteristic Dedekind domain admits functorial transfer maps along finite étale morphisms, so the previous two isomorphisms induce a transfer map $$N_{\l} : \text{H}^i_{\text{mot}}(A',\Z(i)) \longrightarrow \text{H}^i_{\text{mot}}(A,\Z(i))$$ such that pre-composition with the natural map $\text{H}^i_{\text{mot}}(A,\Z(i)) \rightarrow \text{H}^i_{\text{mot}}(A',\Z(i))$ is multiplication by~$\l$. In particular, the kernel of the natural map $\text{H}^i_{\text{mot}}(A,\Z(i)) \rightarrow \text{H}^i_{\text{mot}}(A',\Z(i))$ is $\l$-torsion. Consider the commutative diagram $$\begin{tikzcd} \text{K}{}^{\text{M}}_i(A) \ar[r] \ar[d] & \text{K}{}^{\text{M}}_i(A') \ar[d] \\ \text{H}^i_{\text{mot}}(A,\Z(i)) \ar[r] & \text{H}^i_{\text{mot}}(A',\Z(i)) \end{tikzcd}$$ of abelian groups, and let $x$ be an element of the abelian group $\text{ker}(\text{K}{}^{\text{M}}_i(A) \rightarrow \widehat{\text{K}}{}^{\text{M}}_i(A))$. The residue field of the local ring $A'$ has at least $M_i$ elements, so the natural map $\text{K}{}^{\text{M}}_i(A') \rightarrow \widehat{\text{K}}{}^{\text{M}}_i(A')$ is an isomorphism (\cite[Proposition~$10\,(5)$]{kerz_milnor_2010}), and $x$ is sent to zero by the top horizontal map. In particular, the image of $x$ by the left vertical map is in the kernel of the bottom horizontal map, and is thus $\l$-torsion by the previous paragraph. Let $\l' \geq 1$ be an integer which is coprime to $\l$ and to the degree of $\F_q$ over~$\F_p$, and such that $p^{\l'} \geq M_i$. The previous argument for this integer $\l'$ implies that the image of $x$ by the left vertical map is also $\l'$-torsion, hence it is zero. \end{proof} \begin{conjecture}\label{conjecturecomparisonMilnorKtheoryandmotiviccohomology} Let $A$ be a local ring. Then for every integer $i \geq 0$, the natural map $$\widehat{\text{K}}{}^{\text{M}}_i(A) \longrightarrow \text{H}^i_{\text{mot}}(A,\Z(i))$$ induced by Lemma~\ref{lemmasymbolmapfactorsthoughimprovedMilnorKgroup} is an isomorphism of abelian groups. \end{conjecture} The previous conjecture was proved by Elmanto--Morrow for equicharacteristic local rings (\cite[Theorem~$7.12$]{elmanto_motivic_2023}). Their proof uses as an input the analogous result in the smooth case for classical motivic cohomology, which is unknown in mixed characteristic (see Remark~\ref{remarksmoothcaseimpliesgeneralcaseMilnorcomparison}). \begin{theorem}[Singular Nesterenko--Suslin isomorphism with finite coefficients]\label{theoremcomparisontoMilnorKtheory} Let $A$ be a henselian local ring. Then for any integers $i \geq 0$ and $n \geq 1$, the natural map $$\widehat{\emph{K}}{}_i^{\emph{M}}(A)/n \longrightarrow \emph{H}^i_{\emph{mot}}(A,\Z(i))/n$$ is an isomorphism of abelian groups. \end{theorem} \begin{proof} If the local ring $A$ contains a field, then the natural map $$\widehat{\text{K}}{}^{\text{M}}_i(A) \longrightarrow \text{H}^i_{\text{mot}}(A,\Z(i))$$ is an isomorphism of abelian groups (\cite[Theorem~$7.12$]{elmanto_motivic_2023}). Otherwise, $A$ is a henselian local ring of mixed characteristic $(0,p)$ for some prime number $p$. In particular, the local ring~$A$ is $p$-henselian. If $p$ does not divide the integer $n$, then consider the commutative diagram $$\begin{tikzcd} \widehat{\text{K}}{}^{\text{M}}_i(A)/n \ar[r] \ar[d] & \text{H}^i_{\text{mot}}(A,\Z(i))/n \ar[d] \\ \widehat{\text{K}}{}^{\text{M}}_i(A/p)/n \ar[r] & \text{H}^i_{\text{mot}}(A/p,\Z(i))/n \end{tikzcd}$$ of abelian groups. The left vertical map is an isomorphism by \cite[Proposition~$10\,(7)$]{kerz_milnor_2010}. By Lemma~\ref{lemmatopdegreeintegraltomodpissurjective} and Corollary~\ref{corollaryladicmotivcohomologylowdegreesisétalecohomology}, the right vertical map is naturally identified with the natural map of abelian groups $$\text{H}^i_{\text{ét}}(A,\mu_n^{\otimes i}) \longrightarrow \text{H}^i_{\text{ét}}(A/p,\mu_n^{\otimes i}),$$ which is an isomorphism by rigidity of étale cohomology (\cite{gabber_affine_1994}, see also \cite[Corollary~$1.18\,(1)$]{bhatt_arc-topology_2021}). The local ring $A/p$ is an $\F_p$-algebra, so the bottom horizontal map is an isomorphism (\cite[Theorem~$7.12$]{elmanto_motivic_2023}), and the result is true in this case. It then suffices to prove that for every integer $k \geq 1$, the natural map $$\widehat{\text{K}}{}^{\text{M}}_i(A)/p^k \longrightarrow \text{H}^i_{\text{mot}}(A,\Z(i))/p^k$$ is an isomorphism of abelian groups. By Lemma~\ref{lemmatopdegreeintegraltomodpissurjective} and Corollary~\ref{corollarypadiccomparisoninsmalldegreessyntomiccoho}, the natural map $$\text{H}^i_{\text{mot}}(A,\Z(i))/p^k \longrightarrow \text{H}^i(\Z/p^k(i)^{\text{BMS}}(A))$$ is an isomorphism of abelian groups. By \cite[Theorem~$3.1$]{luders_milnor_2023}, the composite map $$\widehat{\text{K}}{}^{\text{M}}_i(A)/p^k \longrightarrow \text{H}^i_{\text{mot}}(A,\Z(i))/p^k \longrightarrow \text{H}^i(\Z/p^k(i)^{\text{BMS}}(A))$$ is an isomorphism of abelian groups, hence the desired result. \end{proof} \begin{remark}\label{remarksmoothcaseimpliesgeneralcaseMilnorcomparison} If Conjecture~\ref{conjecturecomparisonMilnorKtheoryandmotiviccohomology} is true for local ind-smooth $\Z$-algebras, then the left Kan extension properties \cite[Proposition~$1.17$]{luders_milnor_2023} and Theorem~\ref{theoremmotiviccohomologyisleftKanextendedonlocalringsinsmalldegrees} imply that Conjecture~\ref{conjecturecomparisonMilnorKtheoryandmotiviccohomology} is true for all local rings. See the proof of \cite[Theorem~$7.12$]{elmanto_motivic_2023} for more details. \end{remark} \begin{remark}\label{remarkMilnorcomparisonisinjectiveiffGersteninjectivitytrueforMilnorKgroupssmoothcase} Let $A$ be a local essentially smooth $\Z$-algebra, $i \geq 0$ be an integer, and consider the commutative diagram $$\begin{tikzcd} \widehat{\text{K}}{}^{\text{M}}_i(A) \ar[r] \ar[d] & \text{H}^i_{\text{mot}}(A,\Z(i)) \ar[d] \\ \widehat{\text{K}}{}^{\text{M}}_i(\text{Frac}(A)) \ar[r] & \text{H}^i_{\text{mot}}(\text{Frac}(A),\Z(i)) \end{tikzcd}$$ of abelian groups. The bottom horizontal map is an isomorphism by the Nesterenko--Suslin isomorphism for fields (\cite{nesterenko_homology_1989}, see also \cite[Theorem~$7.12$]{elmanto_motivic_2023}). The left vertical being injective then implies that the top horizontal map is injective. That is, the Gersten injectivity conjecture for the improved Milnor $K$-groups would imply the injectivity part of Conjecture~\ref{conjecturecomparisonMilnorKtheoryandmotiviccohomology}. Knowing the Gersten injectivity conjecture for the motivic cohomology group $\text{H}^i_{\text{mot}}(-,\Z(i))$ would imply that these two facts are equivalent. See \cite{luders_relative_2022} for related results on the Gersten conjecture for improved Milnor $K$-groups. \end{remark} \newpage \section{The projective bundle formula} \vspace{-\parindent} \hspace{\parindent} In this section, we prove that the motivic complexes $\Z(i)^{\text{mot}}$ satisfy the projective bundle formula (Theorem~\ref{theoremprojectivebundleformula}) and regular blowup excision (Theorem~\ref{theoremregularblowupformula}). This implies in particular that the presheaves $\Z(i)^{\text{mot}}$ fit within the recent theory of non-$\mathbb{A}^1$-invariant motives of Annala--Iwasa \cite{annala_motivic_2023} and Annala--Hoyois--Iwasa \cite{annala_algebraic_2023,annala_atiyah_2024}. \subsection{First Chern classes}\label{subsectionfirstChernclasses} \vspace{-\parindent} \hspace{\parindent} In this subsection, we construct the motivic first Chern class (Definition~\ref{definitionmotivicfirstChernclass}) in order to formulate the projective bundle formula (Theorem~\ref{theoremprojectivebundleformula}). \begin{definition}[Motivic first Chern class]\label{definitionmotivicfirstChernclass} Let $X$ be a qcqs derived scheme. The {\it motivic first Chern class} is the natural map $$c_1^{\text{mot}} : R\Gamma_{\text{Nis}}(X,\mathbb{G}_m)[-1] \longrightarrow \Z(1)^{\text{mot}}(X),$$ in the derived category $\mathcal{D}(\Z)$, defined as the Nisnevich sheafification of the natural map of presheaves $$\big(\tau^{\leq 1} R\Gamma_{\text{Zar}}(-,\mathbb{G}_m)\big)[-1] \longrightarrow \Z(1)^{\text{mot}}(-)$$ induced by Definition~\ref{definitionlissemotiviccomparisonmap} and Example~\ref{exampleweight1lissemotiviccohomology}. We also denote by $$c_1^{\text{mot}} : \text{Pic}(X) \longrightarrow \text{H}^2_{\text{mot}}(X,\Z(1))$$ the map induced on $\text{H}^2$. \end{definition} \begin{remark}\label{remarkunicityfirstChernclassfromsmoothcase} The motivic first Chern class of Definition~\ref{definitionmotivicfirstChernclass} is uniquely determined by its naturality, and the fact that it is given by the map of Definition~\ref{definitionclassicalmotiviccomparisonmap} on smooth $\Z$-schemes. \end{remark} For every qcqs scheme $X$, the line bundle $\mathcal{O}(1) \in \text{Pic}(\mathbb{P}^1_X)$, via the multiplicative structure of the motivic complexes $\Z(i)^{\text{mot}}$, induces, for every integer $i \in \Z$, a natural map $$c_1^{\text{mot}}(\mathcal{O}(1)) : \Z(i-1)^{\text{mot}}(\mathbb{P}^1_X)[-2] \longrightarrow \Z(i)^{\text{mot}}(\mathbb{P}^1_X)$$ in the derived category $\mathcal{D}(\Z)$. If $\pi : \mathbb{P}^1_X \rightarrow X$ is the canonical projection map, this in turn induces a natural map \begin{equation}\label{equationmapforP^1bundleformule} \pi^\ast \oplus c_1^{\text{mot}}(\mathcal{O}(1))\pi^\ast : \Z(i)^{\text{mot}}(X) \oplus \Z(i-1)^{\text{mot}}(X)[-2] \longrightarrow \Z(i)^{\text{mot}}(\mathbb{P}^1_X) \end{equation} in the derived category $\mathcal{D}(\Z)$. The aim of the following section is to prove that this map is an equivalence (Theorem~\ref{theoremP^1bundleformulaformotiviccohomology}). To prove such an equivalence, we will need compatibilities with other first Chern classes. \begin{construction}[$\mathbb{P}^1$-bundle formula for additive invariants] Following \cite[Section~$5.1$]{elmanto_motivic_2023}, every additive invariant $E$ of $\Z$-linear $\infty$-categories in the sense of \cite[Definition~$5.11$]{hoyois_higher_2017} admits a natural first Chern class, inducing a natural map of spectra $$\pi^\ast \oplus (1-c_1)(\mathcal{O}(-1))\pi^\ast : E(X) \oplus E(X) \longrightarrow E(\mathbb{P}^1_X).$$ For every additive invariant of $\Z$-linear $\infty$-categories, this map is an equivalence (\cite[Lemma~$5.6$]{elmanto_motivic_2023}). \end{construction} \begin{remark}[Compatibility with filtrations]\label{remarkfilteredadditiveinvariants} If the additive invariant $E$ of $\Z$-linear $\infty$-categories, seen as a presheaf of spectra on qcqs schemes, admits a multiplicative filtered refinement $\text{Fil}^\star E$ which is a multiplicative filtered module over the lisse motivic filtration $\text{Fil}^\star_{\text{lisse}} \text{K}^{\text{conn}}$ (Definition~\ref{definitionlissemotiviccohomology}),\footnote{The lisse motivic filtration $\text{Fil}^\star_{\text{lisse}} \text{K}^{\text{conn}}$ is usually defined only on affine schemes. Here the argument works if $\text{Fil}^\star E$ is a Zariski sheaf of filtered spectra on qcqs schemes, and if its restriction to affine schemes is a multiplicative filtered module over the lisse motivic filtration $\text{Fil}^\star_{\text{lisse}} \text{K}^{\text{conn}}$.} then this map has a natural filtered refinement $$\pi^\ast \oplus (1-c_1)(\mathcal{O}(-1)) \pi^\ast : \text{Fil}^\star E(X) \oplus \text{Fil}^{\star - 1} E(X) \longrightarrow \text{Fil}^\star E(\mathbb{P}^1_X)$$ by \cite[Construction~$5.11$ and Lemma~$5.12$]{elmanto_motivic_2023}. If $E$ is algebraic $K$-theory, equipped with the motivic filtration $\text{Fil}^\star_{\text{mot}} \text{K}$ (Definition~\ref{definitionmotiviccohomologyofderivedschemes}), the argument of \cite[Lemma~$5.12$]{elmanto_motivic_2023} and Remark~\ref{remarkunicityfirstChernclassfromsmoothcase} imply that this map recovers, up to a shift, the map $(\ref{equationmapforP^1bundleformule})$ on graded pieces. \end{remark} \begin{example}[Compatibility with cdh-local motivic cohomology] The multiplicative cdh-local filtration $\text{Fil}^\star_{\text{cdh}} \text{KH}$ (Definition~\ref{definitionmotivicfiltrationonKHtheoryofschemes}) is naturally a module over the multiplicative filtration $\text{Fil}^\star_{\text{mot}} \text{K}$ ({\it e.g.},~because cdh sheafification preserves multiplicative structures), so the first Chern class for the cdh\nobreakdash-local motivic complexes of \cite{bachmann_A^1-invariant_2024} is compatible with the motivic first Chern class of Definition~\ref{definitionmotivicfirstChernclass}. \end{example} \begin{example}[Compatibility with syntomic cohomology]\label{exampleSelmerKtheory} Let $X$ be a qcqs scheme, and $p$ be a prime number. By \cite{bachmann_A^1-invariant_2024}, the syntomic first Chern class of \cite[Section~$7$]{bhatt_absolute_2022} is compatible with the motivic first Chern class of Definition~\ref{definitionmotivicfirstChernclass} via the motivic-syntomic comparison map (Construction~\ref{constructionmotivicsyntomiccomparisonmap}). Note here that the motivic-syntomic comparison map can be seen as the map induced on graded pieces from a multiplicative map of filtered spectra $$\text{Fil}^{\star}_{\text{mot}} \text{K}(X;\Z_p) \longrightarrow \text{Fil}^\star_{\text{mot}} \text{K}^{\text{Sel}}(X;\Z_p),$$ where the target is the filtration on $p$-completed Selmer $K$-theory (\cite[Remark~$8.4.3$]{bhatt_absolute_2022}). These motivic and syntomic first Chern classes then coincide with the first Chern classes coming from the additive invariants $\text{K}(-;\Z_p)$ and $\text{K}^{\text{Sel}}(-;\Z_p)$ (Remark~\ref{remarkfilteredadditiveinvariants}). \end{example} \subsection{\texorpdfstring{$\mathbb{P}^1$}{TEXT}-bundle formula} \vspace{-\parindent} \hspace{\parindent} In this subsection, we prove that the motivic complexes $\Z(i)^{\text{mot}}$ satisfy the $\mathbb{P}^1$-bundle formula on qcqs schemes (Theorem~\ref{theoremP^1bundleformulaformotiviccohomology}). Note that the $\mathbb{P}^1$-bundle formula is unknown for the cdh-local motivic complexes $\Z(i)^{\text{cdh}}$ on general qcqs schemes.\footnote{More precisely, Bachmann--Elmanto--Morrow proved in \cite{bachmann_A^1-invariant_2024} that if a qcqs scheme $X$ satisfies the condition $\text{Val}(X)$ (Section~\ref{subsectionconditionVal}), then the cdh-local motivic complexes $\Z(i)^{\text{cdh}}$ satisfy the $\mathbb{P}^1$-bundle formula at $X$. In particular, the $\mathbb{P}^1$-bundle formula is known for these complexes over a field, and over a mixed characteristic perfectoid valuation ring by the results of \cite{bouis_cartier_2023}, but not on general qcqs schemes.} The cartesian square of Remark~\ref{remarkmaincartesiansquareformotiviccohomology} thus cannot be used directly to prove the $\mathbb{P}^1$-bundle formula for the motivic complexes $\Z(i)^{\text{mot}}$, as was done by Elmanto--Morrow in equicharacteristic (\cite[Section~$5$]{elmanto_motivic_2023}). Instead, we use in a crucial way our main result on $p$-adic motivic cohomology (Theorem~\ref{theorempadicmotiviccohomologyintermsofsyntomicohomology}), and a degeneration argument using Selmer $K$-theory. \begin{theorem}[$\mathbb{P}^1$-bundle formula]\label{theoremP^1bundleformulaformotiviccohomology} Let $X$ be a qcqs scheme, and $\pi : \mathbb{P}^1_X \rightarrow X$ be the canonical projection map. Then for every integer $i \in \Z$, the natural map $$\pi^\ast \oplus c_1^{\emph{mot}}(\mathcal{O}(1)) \pi^\ast : \Z(i)^{\emph{mot}}(X) \oplus \Z(i-1)^{\emph{mot}}(X)[-2] \longrightarrow \Z(i)^{\emph{mot}}(\mathbb{P}^1_X)$$ is an equivalence in the derived category $\mathcal{D}(\Z)$. \end{theorem} Using Theorem~\ref{theorempadicmotiviccohomologyintermsofsyntomicohomology}, the proof of Theorem~\ref{theoremP^1bundleformulaformotiviccohomology} will reduce to the proof of a similar equivalence for the cdh sheaves $\big(L_{\text{cdh}} \tau^{>i} \F_p(i)^{\text{syn}}\big)(-)$ (Proposition~\ref{propositionP^1bundleformulaforcdhlocalhighdegreesyntomiccohomology}). Most of this section is devoted to the study of these cdh sheaves. \begin{lemma}\label{lemmaGfibresequencecdhhighdegreesyntomiccohomologywithétaleandBMS} Let $p$ be a prime number. Then for any integers $i,j \geq 0$ and $k \geq 1$, the natural sequence $$L_{\emph{Nis}} \tau^{>j} R\Gamma_{\emph{ét}}(-,j_!\mu_{p^k}^{\otimes i}) \longrightarrow \big(L_{\emph{Nis}} \tau^{>j} \Z/p^k(i)^{\emph{syn}}\big)(-) \longrightarrow \big(L_{\emph{Nis}} \tau^{>j} \Z/p^k(i)^{\emph{BMS}}\big)(-)$$ is a fibre sequence of $\mathcal{D}(\Z/p^k)$-valued presheaves on qcqs schemes. \end{lemma} \begin{proof} The three presheaves are finitary Nisnevich sheaves, so it suffices to prove the result on henselian local rings (\cite[Corollary~$3.27$ and Example~$4.31$]{clausen_hyperdescent_2021}). Let $A$ be a henselian local ring. By \cite[Remark~$8.4.4$]{bhatt_absolute_2022}, there is a natural fibre sequence $$R\Gamma_{\text{ét}}(A,j_!\mu_{p^k}^{\otimes i}) \longrightarrow \Z/p^k(i)^{\text{syn}}(A) \longrightarrow \Z/p^k(i)^{\text{BMS}}(A)$$ in the derived category $\mathcal{D}(\Z/p^k)$, so it suffices to prove that the natural map $$\Z/p^k(i)^{\text{syn}}(A) \longrightarrow \Z/p^k(i)^{\text{BMS}}(A)$$ is surjective in degree $j$. If $p$ is invertible in the henselian valuation ring $A$, the target of this map is zero. If $p$ is not invertible in $A$, then the valuation ring $A$ is $p$-henselian, and this map is an equivalence (Notation~\ref{notationsyntomiccohomology}). \end{proof} \begin{lemma}\label{lemmaFweirdétalecohomologygroupisconcentratedincohomologicaldegreei+1} Let $p$ be a prime number, and $V$ be a rank one henselian valuation ring of mixed characteristic $(0,p)$. Then for any integers $i \geq 0$ and $k \geq 1$, the complex $$\big(L_{\emph{cdh}} \tau^{>i} R\Gamma_{\emph{ét}}(-,j_!\mu_{p^k}^{\otimes i})\big)(\mathbb{P}^1_V) \in \mathcal{D}(\Z/p^k)$$ is concentrated in degree $i+1$.\footnote{We will prove, at the end of Proposition~\ref{propositionCP1bundleformulaformixedcharvaluationringrank1}, that this complex is actually zero.} \end{lemma} \begin{proof} The presheaf $L_{\text{cdh}} \tau^{>i} R\Gamma_{\text{ét}}(-,j_!\mu_{p^k}^{\otimes i})$ is the cdh sheafification of a presheaf taking values in degrees at least $i+1$, so it takes values in degrees at least $i+1$. To prove that the complex $$\big(L_{\text{cdh}} \tau^{>i} R\Gamma_{\text{ét}}(-,j_!\mu_{p^k}^{\otimes i})\big)(\mathbb{P}^1_V) \in \mathcal{D}(\Z/p^k)$$ is in degrees at most $i+1$, consider the fibre sequence $$\big(L_{\text{cdh}} \tau^{\leq i} R\Gamma_{\text{ét}}(-,j_!\mu_{p^k}^{\otimes i})\big)(\mathbb{P}^1_V) \longrightarrow R\Gamma_{\text{ét}}(\mathbb{P}^1_V,j_!\mu_{p^k}^{\otimes i}) \longrightarrow \big(L_{\text{cdh}} \tau^{>i} R\Gamma_{\text{ét}}(-,j_!\mu_{p^k}^{\otimes i})\big)(\mathbb{P}^1_V)$$ in the derived category $\mathcal{D}(\Z/p^k)$, which is a consequence of arc descent for the presheaf $R\Gamma_{\text{ét}}(-,j_!\mu_{p^k}^{\otimes i})$ (\cite[Theorem~$1.8$]{bhatt_arc-topology_2021}). The scheme $\mathbb{P}^1_V$ has valuative dimension two, so the complex $$\big(L_{\text{cdh}} \tau^{\leq i} R\Gamma_{\text{ét}}(-,j_!\mu_{p^k}^{\otimes i})\big)(\mathbb{P}^1_V) \in \mathcal{D}(\Z/p^k)$$ is in degrees at most $i+2$ (\cite[Theorem~$2.4.15$]{elmanto_cdh_2021}). By the $\mathbb{P}^1$-bundle formula for the presheaves $R\Gamma_{\text{ét}}(-,j_!\mu_{p^k}^{\otimes i})$ (\cite[proof of Theorem~$9.1.1$]{bhatt_absolute_2022}), there is a natural equivalence $$R\Gamma_{\text{ét}}(V,j_!\mu_{p^k}^{\otimes i}) \oplus R\Gamma_{\text{ét}}(V,j_!\mu_{p^k}^{\otimes (i-1)})[-2] \longrightarrow R\Gamma_{\text{ét}}(\mathbb{P}^1_V,j_!\mu_{p^k}^{\otimes i})$$ in the derived category $\mathcal{D}(\Z/p^k)$. The functors $R\Gamma_{\text{ét}}(-,j_!\mu_{p^k}^{\otimes i})$ and $R\Gamma_{\text{ét}}(-,j_!\mu_{p^k}^{\otimes (i-1)})$ are moreover rigid (\cite{gabber_affine_1994}, see also \cite[Corollary~$1.18\,(1)$]{bhatt_arc-topology_2021}) and the valuation ring $V$ is $p$\nobreakdash-henselian, so the complex $R\Gamma_{\text{ét}}(\mathbb{P}^1_V,j_!\mu_{p^k}^{\otimes i}) \in \mathcal{D}(\Z/p^k)$ is zero. This implies that the complex $\big(L_{\text{cdh}} \tau^{>i} R\Gamma_{\text{ét}}(-,j_!\mu_{p^k}^{\otimes i})\big)(\mathbb{P}^1_V)$ is naturally identified with the complex $$\big(L_{\text{cdh}} \tau^{\leq i} R\Gamma_{\text{ét}}(-,j_!\mu_{p^k}^{\otimes i})\big)(\mathbb{P}^1_V)[1] \in \mathcal{D}(\Z/p^k),$$ and is thus in degrees at most~\hbox{$i+1$}. \end{proof} Following \cite{elmanto_cdh_2021}, we say that a $\mathcal{D}(\Z)$-valued presheaf on qcqs schemes satisfies {\it henselian $v$-excision} if for every henselian valuation ring $V$ and every prime ideal $\mathfrak{p}$ of $V$, this presheaf sends the bicartesian square of commutative rings $$\begin{tikzcd} V \ar[r] \ar[d] & V_{\mathfrak{p}} \ar[d] \\ V/\mathfrak{p} \ar[r] & V_{\mathfrak{p}} / \mathfrak{p} V_{\mathfrak{p}} \end{tikzcd}$$ to a cartesian square. Note that in the previous bicartesian square, all the commutative rings are henselian valuation rings by \cite[Lemma~$3.3.5$]{elmanto_cdh_2021}. The following lemma explains how to use henselian $v$-excision to prove that a map of cdh sheaves is an equivalence. \begin{lemma}\label{lemmaBhvexcisivefinitarycdhsheavesimplysufficientonrankatmost1henselianvaluationrings} Let $S$ be a qcqs scheme of finite valuative dimension, $\mathcal{C}$ be a $\infty$-category which is compactly generated by cotruncated objects, and $F,G : \emph{Sch}^{\emph{qcqs,op}}_S \rightarrow \mathcal{C}$ be finitary cdh sheaves satisfying henselian $v$-excision. Then a map of presheaves $F \rightarrow G$ is an equivalence of presheaves if and only if the map $F(V) \rightarrow G(V)$ is an equivalence in $\mathcal{C}$ for every henselian valuation ring $V$ of rank at most one with a map $\emph{Spec}(V) \rightarrow S$. \end{lemma} \begin{proof} By \cite[Proposition~$3.1.8\,(2)$]{elmanto_cdh_2021}, a map $F \rightarrow G$ is an equivalence of presheaves if and only if it is an equivalence on henselian valuation rings over $S$. The presheaves $F$ and $G$ being finitary, this is equivalent to the fact that it is an equivalence on henselian valuation rings of finite rank over $S$. By induction, and using henselian $v$-excision, this is in turn equivalent to the fact that it is an equivalence on henselian valuation rings of rank at most one over $S$. \end{proof} \begin{lemma}\label{lemmaAdescentresultforcdhhighdegreesyntomiccohomology} Let $p$ be a prime number. Then for any integers $i \geq 0$ and $k \geq 1$, the $\mathcal{D}(\Z/p^k)$-valued presheaves $\big(L_{\emph{cdh}} \tau^{>i} \Z/p^k(i)^{\emph{syn}}\big)(-)$ and $\big(L_{\emph{cdh}} \tau^{>i} \Z/p^k(i)^{\emph{syn}}\big)(\mathbb{P}^1_{-})$ are finitary cdh sheaves on qcqs schemes, and satisfy henselian $v$-excision. \end{lemma} \begin{proof} The presheaf $\Z/p^k(i)^{\text{syn}}$ is finitary, and the cdh sheafification of a finitary presheaf is a finitary cdh sheaf (Lemma~\ref{lemmacdhsheafificationcommuteswithfilteredcolimits}), so the presheaf $\big(L_{\text{cdh}} \tau^{>i} \Z/p^k(i)^{\text{syn}}\big)(-)$ is a finitary cdh sheaf. Covers in a site are stable under base change, so the presheaf $$\big(L_{\text{cdh}} \tau^{>i} \Z/p^k(i)^{\text{syn}}\big)(\mathbb{P}^1_{-})$$ is also a finitary cdh sheaf. Henselian valuation rings are local rings for the cdh topology, and the presheaves $\tau^{>i} R\Gamma_{\text{ét}}(-,j_!\mu_{p^k}^{\otimes i})$ and $\tau^{>i} \Z/p^k(i)^{\text{BMS}}$ are rigid (\cite{gabber_affine_1994} and Theorem~\ref{theoremAMMNrigidity}), so the presheaves $$\big(L_{\text{cdh}} \tau^{>i} R\Gamma_{\text{ét}}(-,j_!\mu_{p^k}^{\otimes i})\big)(-) \quad \text{and} \quad \big(L_{\text{cdh}} \tau^{>i} \Z/p^k(i)^{\text{BMS}}\big)(-)$$ satisfy henselian $v$-excision. By Lemma~\ref{lemmaGfibresequencecdhhighdegreesyntomiccohomologywithétaleandBMS} (applied for $j=i$ and after cdh sheafification), the presheaf $\big(L_{\text{cdh}} \tau^{>i} \Z/p^k(i)^{\text{syn}}\big)(-)$ then satisfies henselian $v$-excision. Finally, the presheaf $$\big(L_{\text{cdh}} \tau^{>i} \Z/p^k(i)^{\text{syn}}\big)(\mathbb{P}^1_{-})$$ satisfies henselian $v$\nobreakdash-excision, as a consequence of \cite[Lemma~$3.3.7$]{elmanto_cdh_2021}, and henselian $v$-excision for the presheaf $\big(L_{\text{cdh}} \tau^{>i} \Z/p^k(i)^{\text{syn}}\big)(-)$. \end{proof} For every qcqs scheme $X$, the compatibilities between the motivic and syntomic first Chern classes of Section~\ref{subsectionfirstChernclasses} imply that the natural diagram $$\begin{tikzcd}[column sep = huge] \Z/p^k(i)^{\text{mot}}(X) \oplus \Z/p^k(i-1)^{\text{mot}}(X)[-2] \text{ } \ar[r,"\pi^\ast \oplus c_1^{\text{mot}}(\mathcal{O}(1))\pi^\ast"] \ar[d] & \Z/p^k(i)^{\text{mot}}(\mathbb{P}^1_X) \ar[d] \\ \Z/p^k(i)^{\text{syn}}(X) \oplus \Z/p^k(i-1)^{\text{syn}}(X)[-2] \text{ } \ar[r,"\pi^\ast \oplus c_1^{\text{syn}}(\mathcal{O}(1))\pi^\ast"] & \Z/p^k(i)^{\text{syn}}(\mathbb{P}^1_X) \end{tikzcd}$$ is commutative. We define the natural map $$\big(L_{\text{cdh}} \tau^{>i} \Z/p^k(i)^{\text{syn}}\big)(X) \oplus \big(L_{\text{cdh}} \tau^{>i-1} \Z/p^k(i-1)^{\text{syn}}\big)(X)[-2] \longrightarrow \big(L_{\text{cdh}} \tau^{>i} \Z/p^k(i)^{\text{syn}}\big)(\mathbb{P}^1_X)$$ in the derived category $\mathcal{D}(\Z/p^k)$ as the map induced, via Theorem~\ref{theorempadicmotiviccohomologyintermsofsyntomicohomology}, by taking cofibres along the vertical maps of this commutative diagram. \begin{proposition}\label{propositionCP1bundleformulaformixedcharvaluationringrank1} Let $p$ be a prime number, and $V$ be a rank one henselian valuation ring of mixed characteristic $(0,p)$. Then for any integers $i \geq 0$ and $k \geq 1$, the natural map $$\tau^{>i} \Z/p^k(i)^{\emph{syn}}(V) \oplus \big(\tau^{>i-1} \Z/p^k(i-1)^{\emph{syn}}(V)\big)[-2] \longrightarrow \big(L_{\emph{cdh}} \tau^{>i} \Z/p^k(i)^{\emph{syn}}\big)(\mathbb{P}^1_V)$$ is an equivalence in the derived category $\mathcal{D}(\Z/p^k)$. \end{proposition} \begin{proof} The valuation ring $V$ is $p$-henselian, so the natural maps $$\tau^{>i} \Z/p^k(i)^{\text{syn}}(V) \longrightarrow \tau^{>i} \Z/p^k(i)^{\text{BMS}}(V)$$ and $$\big(\tau^{>i-1}\Z/p^k(i-1)^{\text{syn}}(V)\big)[-2] \longrightarrow \big(\tau^{>i-1} \Z/p^k(i)^{\text{BMS}}(V)\big)[-2]$$ are equivalences in the derived category $\mathcal{D}(\Z/p^k)$. We first prove that the induced map $$\tau^{>i} \Z/p^k(i)^{\text{BMS}}(V) \oplus \big(\tau^{>i-1} \Z/p^k(i-1)^{\text{BMS}}(V)\big)[-2] \longrightarrow \big(L_{\text{cdh}} \tau^{>i} \Z/p^k(i)^{\text{BMS}}\big)(\mathbb{P}^1_V)$$ is an equivalence in the derived category $\mathcal{D}(\Z/p^k)$. Let $\kappa$ be the residue field of $V$. By the rigidity property Theorem~\ref{theoremAMMNrigidity}, the natural maps $$\tau^{>i} \Z/p^k(i)^{\text{BMS}}(V) \longrightarrow \tau^{>i} \Z/p^k(i)^{\text{BMS}}(\kappa)$$ and $$\big(\tau^{>i-1} \Z/p^k(i-1)^{\text{BMS}}(V)\big)[-2] \longrightarrow \big(\tau^{>i-1} \Z/p^k(i-1)^{\text{BMS}}(\kappa)\big)[-2]$$ are equivalences in the derived category $\mathcal{D}(\Z/p^k)$. By Corollary~\ref{corollaryEhighdegreecdhBMSsyntomiccohoisweaklyrigid}, the natural map $$\big(L_{\text{cdh}} \tau^{>i} \Z/p^k(i)^{\text{BMS}}\big)(\mathbb{P}^1_V) \longrightarrow \big(L_{\text{cdh}} \tau^{>i} \Z/p^k(i)^{\text{BMS}}\big)(\mathbb{P}^1_{V/p})$$ is an equivalence in the derived category $\mathcal{D}(\Z/p^k)$. The presheaf $(L_{\text{cdh}} \tau^{>i} \Z/p^k(i)^{\text{BMS}})(\mathbb{P}^1_{-})$ is moreover a finitary cdh sheaf, so it is invariant under nilpotent extensions. In particular, the natural map $$\big(L_{\text{cdh}} \tau^{>i} \Z/p^k(i)^{\text{BMS}}\big)(\mathbb{P}^1_{V/p}) \longrightarrow \big(L_{\text{cdh}} \tau^{>i} \Z/p^k(i)^{\text{BMS}}\big)(\mathbb{P}^1_{\kappa})$$ is an equivalence in the derived category $\mathcal{D}(\Z/p^k)$. It then suffices to prove that the natural map $$\tau^{>i} \Z/p^k(i)^{\text{BMS}}(\kappa) \oplus \big(\tau^{>i-1} \Z/p^k(i-1)^{\text{BMS}}(\kappa)\big)[-2] \longrightarrow \big(L_{\text{cdh}} \tau^{>i} \Z/p^k(i)^{\text{BMS}}\big)(\mathbb{P}^1_{\kappa})$$ is an equivalence in the derived category $\mathcal{D}(\Z/p^k)$, and this is a consequence of the $\mathbb{P}^1$-bundle formula on characteristic $p$ fields for the presheaves $\Z/p^k(i)^{\text{cdh}}$ (\cite{bachmann_A^1-invariant_2024}) and $L_{\text{cdh}} \Z/p^k(i)^{\text{BMS}}$ (\cite[Lemma~$5.17$]{elmanto_motivic_2023}). We prove now that the natural map $$\tau^{>i} \Z/p^k(i)^{\text{syn}}(V) \oplus \big(\tau^{>i-1} \Z/p^k(i-1)^{\text{syn}}(V)\big)[-2] \longrightarrow \big(L_{\text{cdh}} \tau^{>i} \Z/p^k(i)^{\text{syn}}\big)(\mathbb{P}^1_V)$$ is an equivalence in the derived category $\mathcal{D}(\Z/p^k)$. By Lemma~\ref{lemmaGfibresequencecdhhighdegreesyntomiccohomologywithétaleandBMS} (applied for $j=i$ and after cdh sheafification), we just proved that the cofibre of this map is naturally identified with the complex $$\big(L_{\text{cdh}} \tau^{>i} R\Gamma_{\text{ét}}(-,j_!\mu_{p^k}^{\otimes i})\big)(\mathbb{P}^1_V) \in \mathcal{D}(\Z/p^k).$$ By Example~\ref{exampleSelmerKtheory}, these complexes, indexed by integers $i \geq 0$, form the graded pieces of the filtered spectrum defined as the cofibre of the natural map of filtered spectra $$\text{Fil}^\star_{\text{mot}} \text{K}(X;\Z/p^k) \longrightarrow \text{Fil}^\star_{\text{mot}} \text{K}^{\text{Sel}}(X;\Z/p^k).$$ The cofibre of the natural map $\text{K}(-;\Z/p^k) \rightarrow \text{K}^{\text{Sel}}(-;\Z/p^k)$, as a cofibre of two additive invariants of $\Z$-linear $\infty$-categories, is an additive invariant of $\Z$-linear $\infty$-categories and, as such, satisfies the $\mathbb{P}^1$-bundle formula (\cite[Lemma~$5.6$]{elmanto_motivic_2023}). This filtration then induces a spectral sequence $$E_2^{i,j} = \text{H}^{i-j}\big(\big(L_{\text{cdh}} \tau^{>-j} R\Gamma_{\text{ét}}(-,j_!\mu_{p^k}^{\otimes (-j)})\big)(\mathbb{P}^1_V)\big) \Longrightarrow 0.$$ By Lemma~\ref{lemmaFweirdétalecohomologygroupisconcentratedincohomologicaldegreei+1}, for every integer $i \geq 0$, the complex $\big(L_{\text{cdh}} \tau^{>i} R\Gamma_{\text{ét}}(-,j_!\mu_{p^k}^{\otimes i})\big)(\mathbb{P}^1_V) \in \mathcal{D}(\Z/p^k)$ is concentrated in degree $i+1$, so this spectral sequence degenerates. This implies the desired equivalence. \end{proof} \begin{remark}\label{remarkP^1bundleformulaforfields} The proof of Proposition~\ref{propositionCP1bundleformulaformixedcharvaluationringrank1} uses a reduction to the case of fields of characteristic $p$, where the result is a consequence of the $\mathbb{P}^1$-bundle formula for the presheaves $\Z/p^k(i)^{\text{cdh}}$ (\cite{bachmann_A^1-invariant_2024}) and $L_{\text{cdh}} \Z/p^k(i)^{\text{BMS}}$ (\cite[Lemma~$5.17$]{elmanto_motivic_2023}). It is however possible to bypass these two results and prove directly the $\mathbb{P}^1$-bundle formula on fields of characteristic~$p$ for the presheaves $L_{\text{cdh}} \tau^{>i} \Z/p^k(i)^{\text{BMS}}$, by imitating the degeneration argument of \cite[Lemma~$5.17$]{elmanto_motivic_2023}. \end{remark} \begin{proposition}\label{propositionP^1bundleformulaforcdhlocalhighdegreesyntomiccohomology} Let $X$ be a qcqs scheme, and $p$ be a prime number. Then for any integers $i \geq 0$ and $k \geq 1$, the natural map $$\big(L_{\emph{cdh}} \tau^{>i} \Z/p^k(i)^{\emph{syn}}\big)(X) \oplus \big(L_{\emph{cdh}} \tau^{>i-1} \Z/p^k(i-1)^{\emph{syn}}\big)(X)[-2] \longrightarrow \big(L_{\emph{cdh}} \tau^{>i} \Z/p^k(i)^{\emph{syn}}\big)(\mathbb{P}^1_X)$$ is an equivalence in the derived category $\mathcal{D}(\Z/p^k)$. \end{proposition} \begin{proof} The presheaves $L_{\text{cdh}} \tau^{>i} \Z/p^k(i)^{\text{syn}}$ and $\big(L_{\text{cdh}} \tau^{>i} \Z/p^k(i)^{\text{syn}}\big)(\mathbb{P}^1_{-})$ are finitary cdh shea\-ves on qcqs schemes, and satisfy henselian $v$-excision (Lemma~\ref{lemmaAdescentresultforcdhhighdegreesyntomiccohomology}). It then suffices to prove the desired equivalence for henselian valuation rings of rank at most one (Lemma~\ref{lemmaBhvexcisivefinitarycdhsheavesimplysufficientonrankatmost1henselianvaluationrings}). Let~$V$ be a henselian valuation ring of rank at most one. If $p$ is invertible in the valuation ring $V$, then this is equivalent to proving that the natural map $$\tau^{>i} R\Gamma_{\text{ét}}(V,\mu_{p^k}^{\otimes i}) \oplus \big(\tau^{>i-1} R\Gamma_{\text{ét}}(V,\mu_{p^k}^{\otimes (i-1)})\big)[-2] \longrightarrow \big(L_{\text{cdh}} \tau^{>i} R\Gamma_{\text{ét}}(-,\mu_{p^k}^{\otimes i})\big)(\mathbb{P}^1_V)$$ is an equivalence in the derived category $\mathcal{D}(\Z/p^k)$. For every integer $i \geq 0$, there is a fibre sequence of $\mathcal{D}(\Z/p^k)$-valued presheaves $$\Z/p^k(i)^{\text{cdh}}(-) \longrightarrow R\Gamma_{\text{ét}}(-,\mu_{p^k}^{\otimes i}) \longrightarrow L_{\text{cdh}} \tau^{>i} R\Gamma_{\text{ét}}(-,\mu_{p^k}^{\otimes i})$$ on qcqs $\Z[\tfrac{1}{p}]$-schemes (\cite{bachmann_A^1-invariant_2024}). The desired equivalence is then a consequence of the $\mathbb{P}^1$\nobreakdash-bundle formula on qcqs $\Z[\tfrac{1}{p}]$-schemes for the presheaf $\Z/p^k(i)^{\text{cdh}}$ (\cite{bachmann_A^1-invariant_2024}) and the presheaf $R\Gamma_{\text{ét}}(-,\mu_{p^k}^{\otimes i})$ (\cite[proof of Theorem~$9.1.1$]{bhatt_absolute_2022}). If $p$ is zero in the valuation ring $V$, then this is a consequence of the $\mathbb{P}^1$-bundle formula on qcqs $\F_p$-schemes for the presheaves $\Z/p^k(i)^{\text{cdh}}$ (\cite{bachmann_A^1-invariant_2024}) and $L_{\text{cdh}} \Z/p^k(i)^{\text{BMS}}$ (\cite[Theorem~$5.14$]{elmanto_motivic_2023}). If $p$ is neither invertible nor zero in the valuation ring $V$, then $V$ is a rank one henselian valuation ring of mixed characteristic $(0,p)$, and the result is Proposition~\ref{propositionCP1bundleformulaformixedcharvaluationringrank1}. \end{proof} \begin{proof}[Proof of Theorem~\ref{theoremP^1bundleformulaformotiviccohomology}] It suffices to prove the result rationally, and modulo $p$ for every prime number~$p$. Rationally, the Atiyah--Hirzebruch spectral sequence degenerates (Theorem~\ref{theorem21'rationalmotivicfiltrationsplits}), so the result is a consequence of the $\mathbb{P}^1$-bundle formula for algebraic $K$-theory (Section~\ref{subsectionfirstChernclasses}). Let $p$ be a prime number. By Theorem~\ref{theorempadicmotiviccohomologyintermsofsyntomicohomology}, for every integer $i \in \Z$, there is a fibre sequence of $\mathcal{D}(\F_p)$-valued presheaves on qcqs schemes $$\F_p(i)^{\text{mot}}(-) \longrightarrow \F_p(i)^{\text{syn}}(-) \longrightarrow \big(L_{\text{cdh}} \tau^{>i} \F_p(i)^{\text{syn}}\big)(-).$$ By \cite[Theorem~$9.1.1$]{bhatt_absolute_2022}, the natural map $$\pi^\ast \oplus c_1^{\text{syn}}(\mathcal{O}(1))\pi^\ast : \F_p(i)^{\text{syn}}(X) \oplus \F_p(i-1)^{\text{syn}}(X)[-2] \longrightarrow \F_p(i)^{\text{syn}}(\mathbb{P}^1_X)$$ is an equivalence in the derived category $\mathcal{D}(\F_p)$. The result modulo $p$ is then a consequence of Proposition~\ref{propositionP^1bundleformulaforcdhlocalhighdegreesyntomiccohomology}. \end{proof} \subsection{Regular blowup and projective bundle formulae} \vspace{-\parindent} \hspace{\parindent} In this subsection, we prove the regular blowup formula for the motivic complexes $\Z(i)^{\text{mot}}$ (Theorem~\ref{theoremregularblowupformula}). By an argument of Annala--Iwasa, this and the $\mathbb{P}^1$-bundle formula imply the general projective bundle formula for the motivic complexes $\Z(i)^{\text{mot}}$ (Theorem~\ref{theoremprojectivebundleformula}). \begin{theorem}[Regular blowup formula]\label{theoremregularblowupformula} Let $Y \rightarrow Z$ be a regular closed immersion of qcqs schemes.\footnote{A morphism $Y \rightarrow Z$ is a regular closed immersion if it is a closed immersion, and if $Z$ admits an affine open cover such that $Y$ is defined by a regular sequence on each of the corresponding affine schemes.} Then for every integer $i \geq 0$, the commutative diagram $$\begin{tikzcd} \Z(i)^{\emph{mot}}(Z) \ar[r] \ar[d] & \Z(i)^{\emph{mot}}(Y) \ar[d] \\ \Z(i)^{\emph{mot}}(\emph{Bl}_Y(Z)) \ar[r] & \Z(i)^{\emph{mot}}(\emph{Bl}_Y(Z) \times_Z Y) \end{tikzcd}$$ is a cartesian square in the derived category $\mathcal{D}(\Z)$. \end{theorem} \begin{proof} It suffices to prove the result rationally, and modulo $p$ for every prime number $p$. By definition, a cdh sheaf sends an abstract blowup square to a cartesian square, and in particular satisfies the regular blowup formula. By Corollary~\ref{corollaryrationalmainresultongradedpieces}, the regular blowup formula for the presheaf $\Q(i)^{\text{mot}}$ is then equivalent to the regular blowup formula for the presheaf $R\Gamma_{\text{Zar}}(-,\mathbb{L}\Omega^{<i}_{-_{\Q}/\Q})$. And the regular blowup formula for the presheaf $R\Gamma_{\text{Zar}}(-,\mathbb{L}\Omega^{<i}_{-_{\Q}/\Q})$ is a consequence of the fact that for every integer $j \geq 0$, the presheaf $R\Gamma_{\text{Zar}}(-,\mathbb{L}^j_{-_/\Z} \otimes_{\Z} \Q)$ satisfies the regular blowup formula (\cite[Lemma~$9.4.3$]{bhatt_absolute_2022}). Let $p$ be a prime number. Similarly, Corollary~\ref{corollarymainpadicstructureongradeds} implies that the regular blowup formula for the presheaf $\F_p(i)^{\text{mot}}$ is equivalent to the regular blowup formula for the presheaf $\F_p(i)^{\text{BMS}}$. By \cite[Corollary~$5.31$]{antieau_beilinson_2020}, there exists an integer $m \geq 0$ and an equivalence of presheaves $$\F_p(i)^{\text{BMS}}(-) \xlongrightarrow{\sim} \text{fib} \Big(\text{can}-\phi_i : (\mathcal{N}^{\geq i} \Prism_{-}\{i\}/\mathcal{N}^{\geq i+m} \Prism_{-}\{i\})/p \longrightarrow (\Prism_{-}\{i\}/\mathcal{N}^{\geq i+m} \Prism_{-}\{i\})/p\Big).$$ In particular, it suffices to prove that for every integer $j \geq 0$, the presheaf $\mathcal{N}^j \Prism_{-}/p$ satisfies the regular blowup formula. By \cite[Remark~$5.5.8$ and Example~$4.7.8$]{bhatt_absolute_2022}, there is a fibre sequence of presheaves $$\mathcal{N}^j \Prism_{-} \{i\}/p \longrightarrow \text{Fil}^{\text{conj}}_j \overline{\Prism}_{-/\Z_p\llbracket \widetilde{p} \rrbracket}/p \xlongrightarrow{\Theta + j} \text{Fil}^{\text{conj}}_{j-1} \overline{\Prism}_{-/\Z_p\llbracket \widetilde{p} \rrbracket}/p.$$ The presheaves $\text{Fil}^{\text{conj}}_j \overline{\Prism}_{-/\Z_p\llbracket \widetilde{p} \rrbracket}/p$ and $\text{Fil}^{\text{conj}}_{j-1} \overline{\Prism}_{-/\Z_p\llbracket \widetilde{p} \rrbracket}/p$ have finite filtrations with graded pieces given by modulo $p$ powers of the cotangent complex, and the result is then a consequence of the regular blowup formula for powers of the cotangent complex (\cite[Lemma~9.4.3]{bhatt_absolute_2022}). \end{proof} \begin{theorem}[Projective bundle formula]\label{theoremprojectivebundleformula} Let $X$ be a qcqs scheme, $r \geq 1$ be an integer, $\mathcal{E}$~be a vector bundle of rank $r+1$ on $X$, and $\pi : \mathbb{P}_X(\mathcal{E}) \rightarrow X$ be the projectivisation of $\mathcal{E}$. Then for every integer $i \in \Z$, the natural map $$\sum_{j=0}^r c_1^{\emph{mot}}(\mathcal{O}(1))^j \pi^{\ast} : \bigoplus_{j=0}^r \Z(i-j)^{\emph{mot}}(X)[-2j] \longrightarrow \Z(i)^{\emph{mot}}(\mathbb{P}_X(\mathcal{E}))$$ is an equivalence in the derived category $\mathcal{D}(\Z)$. \end{theorem} \begin{proof} By Zariski descent, it suffices to consider the case where the vector bundle $\mathcal{E}$ is given by $\mathbb{A}^{r+1}_X$, {\it i.e.}, to prove that the natural map $$\sum_{j=0}^r c_1^{\text{mot}}(\mathcal{O}(1))^j \pi^{\ast} : \bigoplus_{j=0}^r \Z(i-j)^{\text{mot}}(X)[-2j] \longrightarrow \Z(i)^{\text{mot}}(\mathbb{P}^r_X)$$ is an equivalence in the derived category $\mathcal{D}(\Z)$. The presheaves $\Z(i)^{\text{mot}}$ satisfy the $\mathbb{P}^1$-bundle formula (Theorem~\ref{theoremP^1bundleformulaformotiviccohomology}). Moreover, for every qcqs scheme $X$ and every integer $m \geq 0$, they send the blowup square $$\begin{tikzcd} \mathbb{P}^m_X \ar[r] \ar[d] & \text{Bl}_X(\mathbb{A}^{m+1}_X) \ar[d] \\ X \arrow[r,"0"] & \mathbb{A}^{m+1}_X \end{tikzcd}$$ to a cartesian square in the derived category $\mathcal{D}(\Z)$ (Theorem~\ref{theoremregularblowupformula}, in the special case where the regular closed immersion $Y \rightarrow Z$ is the zero section $X \rightarrow \mathbb{A}^{m+1}_X$). By the argument of \cite[Lemma~$3.3.5$]{annala_motivic_2023}, these two properties imply, by induction, the desired projective bundle formula. \end{proof} In the following result, denote by $\Z(i)^{\text{mot}}_X : \text{Sm}^{\text{op}}_X \rightarrow \mathcal{D}(\Z)$ the Zariski sheaves on smooth schemes over $X$ induced by restriction of the motivic complexes $\Z(i)^{\text{mot}}$. \begin{corollary}[Motivic cohomology is represented in motivic spectra]\label{corollaryP1motivicspectra} For every qcqs scheme $X$, the motivic complexes $\{\Z(i)^{\emph{mot}}_X\}_{i \in \Z}$ are represented by a $\mathbb{P}^1$-motivic spectrum in the sense of \cite{annala_motivic_2023}. \end{corollary} \begin{proof} By definition of $\mathbb{P}^1$-motivic spectra, this is a consequence of elementary blowup excision (which is a special case of Theorem~\ref{theoremregularblowupformula}) and the $\mathbb{P}^1$-bundle formula (Theorem~\ref{theoremP^1bundleformulaformotiviccohomology}). \end{proof} \newpage \section{Motivic Weibel vanishing and pro cdh descent} \vspace{-\parindent} \hspace{\parindent} In this section, we study the motivic complexes $\Z(i)^{\text{mot}}$ on noetherian schemes. We prove a general vanishing result which refines Weibel's vanishing conjecture on negative $K$-groups (Theorem~\ref{theoremmotivicWeibelvanishing}), and prove that they coincide with Kelly--Saito's pro cdh motivic complexes $\Z(i)^{\text{procdh}}$ (Theorem~\ref{theoremcomparisonprocdhmotivic}). The key input for both these results is the fact that the motivic complexes $\Z(i)^{\text{mot}}$ satisfy pro cdh excision (Theorem~\ref{theoremprocdhdescentformotiviccohomology}), {\it i.e.}, that they send abstract blowup squares to pro cartesian squares. \begin{notation}[Abstract blowup square]\label{notationabstractblowupsquare} An {\it abstract blowup square} (of noetherian schemes) is a cartesian square \begin{equation}\label{equationabstractblowupsquare} \begin{tikzcd} Y' \ar[r] \ar[d] & X' \ar[d] \\ Y \ar[r] & X \end{tikzcd} \end{equation} of qcqs schemes (resp. of noetherian schemes) such that $X' \rightarrow X$ is proper and finitely presented, \hbox{$Y \rightarrow X$} is a finitely presented closed immersion, and the induced map \hbox{$X'\setminus Y' \rightarrow X \setminus Y$} is an isomorphism. In this context, we also denote, for every integer $r \geq 0$, by $rY$ (resp.~$rY'$) the $r-1^{\text{st}}$ infinitesimal thickening of $Y$ inside $X$ (resp. of $Y'$ inside $X'$). \end{notation} We use Kelly--Saito's recent definition in \cite{kelly_procdh_2024} of the pro cdh topology to encode the fact that a Nisnevich sheaf ({\it e.g.}, the motivic complex $\Z(i)^{\text{mot}}$) satisfies pro cdh excision. Kelly--Saito proved in particular that if $S$ has finite valuative dimension and noetherian topological space, then the pro cdh topos of $S$ is hypercomplete and has enough points. For our purposes, the following definition will be used only for noetherian schemes $S$. \begin{definition}[Pro cdh descent, after \cite{kelly_procdh_2024}]\label{definitionprocdhsheaf} Let $S$ be a qcqs scheme. A {\it pro cdh sheaf} on finitely presented $S$-schemes is a presheaf $$F : \text{Sch}_S^{\text{fp,op}} \longrightarrow \mathcal{D}(\Z)$$ satisfying Nisnevich descent, and such that for every abstract blowup square of finitely presented $S$-schemes $(\ref{equationabstractblowupsquare})$, the natural commutative diagram $$\begin{tikzcd} F(X) \ar[r] \ar[d] & F(X') \ar[d] \\ \{F(rY)\}_r \ar[r] & \{F(rY')\}_r \end{tikzcd}$$ is a weakly cartesian square of pro objects in the derived category $\mathcal{D}(\Z)$.\footnote{By this, we mean that all the cohomology groups of the total fibre of this commutative square are zero as pro abelian groups. All the presheaves $F$ that we will consider (most importantly, the presheaves $\Z(i)^{\text{mot}}$) are bounded above on noetherian schemes, by a constant depending only on the dimension of their input (for the motivic complexes $\Z(i)^{\text{mot}}$, this is Proposition~\ref{propositionmotivicfiltrationiscompleteonqcqsschemesoffinitevaluativedimension}); this definition of weakly cartesian square will then be equivalent to being weakly cartesian in the stable $\infty$-category of pro objects in the derived category $\mathcal{D}(\Z)$, in the sense of \cite[Definition~$2.27$]{land_k-theory_2019}.} \end{definition} \subsection{Pro cdh descent for the cotangent complex} \vspace{-\parindent} \hspace{\parindent} In this subsection, we review the pro cdh descent for powers of the cotangent complex on noetherian schemes (Proposition~\ref{propositionprocdhdescentforcotangentcomplex}). On finite-dimensional noetherian schemes, this is \cite[Theorem~$2.10$]{morrow_pro_2016}. On general noetherian schemes, the proof follows the sketch presented in \cite[proof of Lemma~$8.5$]{elmanto_motivic_2023}. In particular, the arguments are exactly as in \cite{morrow_pro_2016}, except for the following generalisation of Grothendieck's formal functions theorem (\cite[Corollary~$4.1.7$]{grothendieck_elements_1961}), where the finite dimensionality hypothesis is removed. We give some details for the sake of completeness. For every commutative ring $A$, recall that a pro $A$-module $\{M_r\}_r$ is zero if for every index~$r$, there exists an index $r' \geq r$ such that the map $M_{r'} \rightarrow M_r$ is the zero map. Similarly, a map $\{M_r\}_r \rightarrow \{N_r\}_r$ of pro $A$-modules is an isomorphism if its kernel and cokernel are zero pro $A$-modules. We say that a pro object $\{C_r\}_r$ in the derived category $\mathcal{D}(A)$ is weakly zero if all its cohomology groups are zero pro $A$-modules. Note that all the pro complexes that we will consider are uniformly bounded above, so this definition is equivalent to being weakly zero in the stable $\infty$-category of pro objects in the derived category $\mathcal{D}(A)$ (\cite[Definition~$2.27$]{land_k-theory_2019}). Similarly, we say that a map $\{C_r\}_r \rightarrow \{C_r'\}_r$ of pro objects in the derived category $\mathcal{D}(A)$ is a weak equivalence if its fibre is weakly zero as a pro object in the derived category $\mathcal{D}(A)$. \begin{lemma}[Formal functions theorem, after Lurie \cite{lurie_spectral_2019}]\label{lemmaformalfunctionstheoremLurie} Let $A$ be a noetherian commutative ring, $I$ be an ideal of $A$, $X$ be a proper scheme over $\emph{Spec}(A)$, and $X^\wedge_I$ be the formal completion of $X$ along the vanishing locus of $I$. Then for every coherent sheaf $\mathcal{F}$ over $X$, the natural map $$R\Gamma_{\emph{Zar}}(X,\mathcal{F}) \longrightarrow R\Gamma_{\emph{Zar}}(X^\wedge_I,\mathcal{F}^\wedge_I)$$ where $\mathcal{F}^\wedge_I$ is the pullback of $\mathcal{F}$ along the natural map $X^\wedge_I \rightarrow X$, exhibits the target as the $I$-adic completion\footnote{The cohomology groups of these coherent sheaves are finitely generated $A$-modules (because $X$ is proper over $\text{Spec}(A)$), so the derived $I$-adic completion and the classical $I$-adic completion coincide in this context.} of the source in the derived category $\mathcal{D}(A)$. More precisely, the natural map $$\{R\Gamma_{\emph{Zar}}(X,\mathcal{F})/I^r\}_r \longrightarrow \{R\Gamma_{\emph{Zar}}(X \times_{\emph{Spec}(A)} \emph{Spec}(A/I^r), \mathcal{F} \otimes^{\mathbb{L}}_{\mathcal{O}_X} \mathcal{O}_X/I^r\mathcal{O}_X)\}_r$$ is a weak equivalence of pro objects in the derived category $\mathcal{D}(A)$. \end{lemma} \begin{proof} The first statement is a special case of \cite[Lemma~$8.5.1.1$]{lurie_spectral_2019}. The second statement, although {\it a priori} stronger, follows by an examination of the previous proof (and in particular, the proof of \cite[Lemma~$8.1.2.3$]{lurie_spectral_2019}). \end{proof} \begin{lemma}\label{lemmacohomologygroupsofcotangentcomplexarefinitelygenerated} Let $A$ be a noetherian commutative ring, and $X$ be a proper scheme over $\emph{Spec}(A)$. Then for any integers $j \geq 0$ and $n \in \Z$, the $A$-module $\emph{H}^n_{\emph{Zar}}(X,\mathbb{L}^j_{-/A})$ is finitely generated. \end{lemma} \begin{proof} The scheme $X$ is of finite type over $\text{Spec}(A)$, so the $\mathcal{O}_X$-module $\mathcal{H}^n_{\text{Zar}}(-,\mathbb{L}^j_{-/A})$ is coherent. Because $X$ is proper over $\text{Spec}(A)$, its cohomology groups are thus finitely generated $A$-modules. \end{proof} \begin{corollary}\label{corollaryM16Theorem2.4(iv)} Let $A$ be a noetherian commutative ring, $I$ be an ideal of $A$, and $X$ be a noetherian scheme which is proper over $\emph{Spec}(A)$. Then for any integers $j \geq 0$ and $n \in \Z$, the natural map $$\{ \emph{H}^n_{\emph{Zar}}(X,\mathbb{L}^j_{-/A})/I^r\}_r \longrightarrow \{\emph{H}^n_{\emph{Zar}}(X,\mathbb{L}^j_{-/A} \otimes^{\mathbb{L}}_{\mathcal{O}_X}\mathcal{O}_X / I^r \mathcal{O}_X) \}_r$$ is an isomorphism of pro $A$-modules. \end{corollary} \begin{proof} By Lemma~\ref{lemmacohomologygroupsofcotangentcomplexarefinitelygenerated} and its proof, all the terms in the hypercohomology spectral sequence $$E_2^{p,q} = \text{H}^p_{\text{Zar}}(X,\mathcal{H}^q_{\text{Zar}}(-,\mathbb{L}^j_{-/R})) \Longrightarrow \text{H}^{p+q}_{\text{Zar}}(X,\mathbb{L}^j_{-/A})$$ are finitely generated $A$-modules. The functor $\{-\otimes_A A/I^r\}_r$ is exact on the category of finitely generated $A$-modules (\cite[Theorem~$1.1$\,(ii)]{morrow_pro_2016}), so it induces a spectral sequence of pro $A$-modules $$E_2^{p,q} = \{\text{H}^p_{\text{Zar}}(X,\mathcal{H}^q_{\text{Zar}}(-,\mathbb{L}^j_{-/A}))/I^r\}_r \Longrightarrow \{\text{H}^{p+q}_{\text{Zar}}(X,\mathbb{L}^j_{-/A})/I^r\}_r.$$ It then suffices to prove that the natural map of pro $A$-modules $$\{\text{H}^p_{\text{Zar}}(X,\mathcal{H}^q_{\text{Zar}}(-,\mathbb{L}^j_{-/A}))/I^r\}_r \longrightarrow \{\text{H}^p_{\text{Zar}}(X,\mathcal{H}^q_{\text{Zar}}(-,\mathbb{L}^j_{-/A} \otimes^{\mathbb{L}}_{\mathcal{O}_X} \mathcal{O}_X/I^r\mathcal{O}_X))\}_r$$ is an isomorphism for all integers $p,q \geq 0$. The natural map $$\{\text{H}^p_{\text{Zar}}(X,\mathcal{H}^q_{\text{Zar}}(-,\mathbb{L}^j_{-/A}))/I^r\}_r \longrightarrow \{\text{H}^p_{\text{Zar}}(X,\mathcal{H}^q_{\text{Zar}}(-,\mathbb{L}^j_{-/A}) \otimes_A^{\mathbb{L}} A/I^r)\}_r$$ is an isomorphism by Lemma~\ref{lemmaformalfunctionstheoremLurie} applied to the coherent sheaf $\mathcal{H}^q_{\text{Zar}}(-,\mathbb{L}_{-/A})$ on $X$, and the natural map $$\{\text{H}^p_{\text{Zar}}(X,\mathcal{H}^q_{\text{Zar}}(-,\mathbb{L}^j_{-/A}) \otimes_A^{\mathbb{L}} A/I^r)\}_r \longrightarrow \{\text{H}^p_{\text{Zar}}(X,\mathcal{H}^q_{\text{Zar}}(-,\mathbb{L}^j_{-/A} \otimes^{\mathbb{L}}_{\mathcal{O}_X} \mathcal{O}_X/I^r\mathcal{O}_X))\}_r$$ is an isomorphism by \cite[Lemma~$2.3$]{morrow_pro_2016}. \end{proof} \begin{lemma}\label{lemmaLemma2.8(i)} Let $A$ be a noetherian commutative ring, $I$ be an ideal of $A$, and $X$ be a proper scheme over $\emph{Spec}(A)$ such that the induced map $X \setminus V(I\mathcal{O}_X) \rightarrow \emph{Spec}(A) \setminus V(I)$ is an isomorphism. \begin{enumerate} \item For any integers $j \geq 0$ and $n \in \Z$, the natural map $$\{\emph{H}^n_{\emph{Zar}}(X,\mathbb{L}^j_{-/A} \otimes^{\mathbb{L}}_{\mathcal{O}_X} I^r \mathcal{O}_X)\}_r \longrightarrow \{I^r\emph{H}^n_{\emph{Zar}}(X,\mathbb{L}^j_{-/A})\}_r$$ is an isomorphism of pro $A$-modules. \item For any integers $j \geq 0$ and $n \in \Z$ such that $(j,n) \neq (0,0)$, the $A$-module $\emph{H}^n_{\emph{Zar}}(X,\mathbb{L}^j_{-/A})$ is killed by a power of $I$; in particular, the pro $A$-module $\{I^r\emph{H}^n_{\emph{Zar}}(X,\mathbb{L}^j_{-/A})\}_r$ is zero. \end{enumerate} \end{lemma} \begin{proof} $(1)$ The short exact sequence $0 \rightarrow \{I^r \mathcal{O}_X \}_r \rightarrow \mathcal{O}_X \rightarrow \{\mathcal{O}_X / I^r \mathcal{O}_X \}_r \rightarrow 0$ of pro $\mathcal{O}_X$-modules induces a long exact sequence $$\cdots \rightarrow \text{H}^n_{\text{Zar}}(X,\mathbb{L}^j_{-/A} \otimes^{\mathbb{L}}_{\mathcal{O}_X} I^r \mathcal{O}_X)\}_r \rightarrow \text{H}^n_{\text{Zar}}(X,\mathbb{L}^j_{-/A}) \rightarrow \{\text{H}^n_{\text{Zar}}(X,\mathbb{L}^j_{-/A} \otimes^{\mathbb{L}}_{\mathcal{O}_X} \mathcal{O}_X/I^r\mathcal{O}_X)\}_r \rightarrow \cdots$$ of pro $A$-modules. By Corollary~\ref{corollaryM16Theorem2.4(iv)}, the boundary maps of this long exact sequence vanish, hence the natural map $$\{\text{H}^n_{\text{Zar}}(X,\mathbb{L}^j_{-/A} \otimes^{\mathbb{L}}_{\mathcal{O}_X} I^r \mathcal{O}_X)\}_r \longrightarrow \{I^r\text{H}^n_{\text{Zar}}(X,\mathbb{L}^j_{-/A})\}_r$$ is an isomorphism of pro $A$-modules. \\$(2)$ By Lemma~\ref{lemmacohomologygroupsofcotangentcomplexarefinitelygenerated}, the $A$-module $\text{H}^n_{\text{Zar}}(X,\mathbb{L}^j_{-/A})$ is finitely generated. Because the map $$X \setminus V(I\mathcal{O}_X) \rightarrow \text{Spec}(A) \setminus V(I)$$ is an isomorphism, this $A$-module is moreover supported on $V(I)$ if $(j,n) \neq (0,0)$. If $(j,n) \neq (0,0)$, this implies that the $A$-module $\text{H}^n_{\text{Zar}}(X,\mathbb{L}^j_{-/A})$ is killed by a power of $I$. \end{proof} \begin{corollary}\label{corollaryM16lemma2.8(ii)} Let $A$ be a noetherian commutative ring, $I$ be an ideal of $A$, and $X$ be a proper scheme over $\emph{Spec}(A)$ such that the induced map $X \setminus V(I\mathcal{O}_X) \rightarrow \emph{Spec}(A) \setminus V(I)$ is an isomorphism. Then for every integer $j \geq 0$, the natural map $$\{ \mathbb{L}^j_{A/\Z} \otimes^{\mathbb{L}}_A I^r \}_r \longrightarrow \{ R\Gamma_{\emph{Zar}}(X,\mathbb{L}^j_{-/\Z} \otimes^{\mathbb{L}}_{\mathcal{O}_X} I^r \mathcal{O}_X) \}_r$$ is a weak equivalence of pro objects in the derived category $\mathcal{D}(A)$. \end{corollary} \begin{proof} By Lemma~\ref{lemmaLemma2.8(i)}, and for any integers $n,a,b \in \Z$, the pro $A$-module $$\{\text{H}^n(\mathbb{L}^j_{A/\Z} \otimes^{\mathbb{L}}_A \text{H}^a_{\text{Zar}}(X,\mathbb{L}^b_{-/A} \otimes^{\mathbb{L}}_{\mathcal{O}_X} I^r \mathcal{O}_X))\}_r$$ is zero, except if $(a,b)=(0,0)$. By transitivity for the powers of the cotangent complex (see the proof of \cite[Lemma~$2.8\,(ii)$]{morrow_pro_2016} for more details), this implies that the natural map $$\{\text{H}^n(\mathbb{L}^j_{A/\Z} \otimes^{\mathbb{L}}_A \text{H}^0_{\text{Zar}}(X,I^r\mathcal{O}_X))\}_r \longrightarrow \{\text{H}^n_{\text{Zar}}(X,\mathbb{L}^j_{-/\Z} \otimes^{\mathbb{L}}_{\mathcal{O}_X} I^r \mathcal{O}_X)\}_r$$ is an isomorphism of pro $A$-modules. Let $B$ be the $A$-algebra $\text{H}^0_{\text{Zar}}(X,\mathcal{O}_X)$. Applying Lemma~\ref{lemmaLemma2.8(i)}\,$(1)$ for $j=n=0$, it then suffices to prove that the natural map $\{I^r\}_r \rightarrow \{I^r B\}_r$ is an isomorphism of pro $A$-modules. The $A$-algebra $B$ is finite and isomorphic to $A$ away from the vanishing locus of $I$, so the kernel and cokernel of the structure map $A \rightarrow B$ are killed by a power of $I$. The result is then a formal consequence of \cite[Theorem~$1.1$\,(ii)]{morrow_pro_2016}. \end{proof} \begin{lemma}\label{lemmaM16Lemma2.4.(i)} Let $Y \rightarrow X$ be a closed immersion of noetherian schemes, and $\mathcal{I}$ be the associated ideal sheaf on $X$. Then for every integer $j \geq 0$, the natural map $$\{ R\Gamma_{\emph{Zar}}(X,\mathbb{L}^j_{-/\Z} \otimes^{\mathbb{L}}_{\mathcal{O}_X} \mathcal{O}_X/\mathcal{I}^r) \}_r \longrightarrow \{ R\Gamma_{\emph{Zar}}(rY,\mathbb{L}^j_{-/\Z}) \}_r$$ is a weak equivalence of pro objects in the derived category $\mathcal{D}(A)$. \end{lemma} \begin{proof} The scheme $X$ is noetherian, hence quasi-separated, so we may assume by induction that $X$ is affine. In this case, the result is \cite[Corollary~$4.5\,(ii)$]{morrow_pro_2018}. \end{proof} \begin{proposition}\label{propositionprocdhdescentforcotangentcomplex} Let $j \geq 0$ be an integer. Then for every abstract blowup square of noetherian schemes (\ref{equationabstractblowupsquare}), the natural commutative diagram $$\begin{tikzcd} R\Gamma_{\emph{Zar}}(X,\mathbb{L}^j_{-/\Z}) \ar[r] \ar[d] & R\Gamma_{\emph{Zar}}(X',\mathbb{L}^j_{-/\Z}) \ar[d] \\ \{R\Gamma_{\emph{Zar}}(rY,\mathbb{L}^j_{-/\Z})\}_r \ar[r] & \{R\Gamma_{\emph{Zar}}(rY',\mathbb{L}^j_{-/\Z})\}_r \end{tikzcd}$$ is a weakly cartesian square of pro objects in the derived category $\mathcal{D}(\Z)$. In particular, the presheaf $R\Gamma_{\emph{Zar}}(-,\mathbb{L}^j_{-/\Z})$ is a pro cdh sheaf on noetherian schemes. \end{proposition} \begin{proof} The scheme $X$ is noetherian, hence quasi-separated, so we may assume by induction that $X$ is affine, given by the spectrum of a noetherian commutative ring $A$. Let $I$ be the ideal of $A$ defining the closed subscheme $Y$ of $\text{Spec}(A)$. By Lemma~\ref{lemmaM16Lemma2.4.(i)}, the desired statement is equivalent to the fact that the commutative diagram $$\begin{tikzcd} \mathbb{L}^j_{A/\Z} \ar[r] \ar[d] & R\Gamma_{\text{Zar}}(X',\mathbb{L}^j_{-/\Z}) \ar[d] \\ \{\mathbb{L}^j_{A/\Z} \otimes^{\mathbb{L}}_A A/I^r\}_r \ar[r] & \{R\Gamma_{\text{Zar}}(X',\mathbb{L}^j_{-/\Z} \otimes^{\mathbb{L}}_{\mathcal{O}_{X'}} \mathcal{O}_{X'}/I^r\mathcal{O}_{X'})\}_r \end{tikzcd}$$ is a weakly cartesian square of pro objects in the derived category $\mathcal{D}(\Z)$. Taking fibres along the vertical maps, this is exactly Corollary~\ref{corollaryM16lemma2.8(ii)}. \end{proof} We now use Proposition~\ref{propositionprocdhdescentforcotangentcomplex} to prove pro cdh descent for variants of the cotangent complex (Corollary~\ref{corollaryprocdhdescentforcotangentcomplexandvariants}). In the following two lemmas, we consider inverse systems of objects in the derived category~$\mathcal{D}(\Z)$. We say that an inverse system $(C_r)_r$ in the derived category $\mathcal{D}(\Z)$ is {\it essentially zero} if for every index $r$ and every integer $n \in \Z$, there exists an index $r' \geq r$ such that the map $\text{H}^n(C_{r'}) \rightarrow \text{H}^n(C_r)$ is the zero map. In particular, an inverse system $(C_r)_r$ in the derived category $\mathcal{D}(\Z)$ is essentially zero if and only if the associated pro object $\{C_r\}_r$ in the derived category $\mathcal{D}(\Z)$ is weakly zero. \begin{lemma}\label{lemmaprozeroimpliesprozeromodp} Let $(C_r)_r$ be an inverse system in the derived category $\mathcal{D}(\Z)$. If $(C_r)_r$ is essentially zero, then $\big(\prod_{p \in \mathbb{P}} C_r/p\big)_r$ is essentially zero. \end{lemma} \begin{proof} Assume that the inverse system $(C_r)_r$ is essentially zero. Let $r_0$ be an index of this inverse system, $n \in \Z$ be an integer, and $p$ be a prime number. We will use repeatedly that for every index $r$, there is a natural short exact sequence $$0 \longrightarrow \text{H}^n(C_r)/p \longrightarrow \text{H}^n(C_r/p) \longrightarrow \text{H}^{n+1}(C_r)[p] \longrightarrow 0$$ of abelian groups. Let $r_1 \geq r_0$ be an index such that the map $\text{H}^n(C_{r_1}) \rightarrow \text{H}^n(C_{r_0})$ is the zero map. Then for every index $r \geq r_1$, the map $\text{H}^n(C_r)/p \rightarrow \text{H}^n(C_{r_0})/p$ is the zero map, and the map $\text{H}^n(C_r/p) \rightarrow \text{H}^n(C_{r_0}/p)$ thus factors through the map $\text{H}^n(C_r/p) \rightarrow \text{H}^{n+1}(C_r)[p]$. Let~\hbox{$r_2 \geq r_1$} be an index such that the map $\text{H}^{n+1}(C_{r_2}) \rightarrow \text{H}^{n+1}(C_{r_1})$ is the zero map. Then the map $$\text{H}^{n+1}(C_{r_2})[p] \rightarrow \text{H}^{n+1}(C_{r_1})[p]$$ is the zero map. By construction, the map $$\text{H}^n(C_{r_2}/p) \longrightarrow \text{H}^n(C_{r_0}/p)$$ factors as $$\text{H}^n(C_{r_2}/p) \longrightarrow \text{H}^{n+1}(C_{r_2})[p] \xlongrightarrow{0} \text{H}^{n+1}(C_{r_1})[p] \longrightarrow \text{H}^n(C_{r_0}/p),$$ and is thus also the zero map. The index $r_2$ does not depend on the prime number $p$, so the map $$\prod_{p \in \mathbb{P}} \text{H}^n(C_{r_2}/p) \longrightarrow \prod_{p \in \mathbb{P}} \text{H}^n(C_{r_0}/p)$$ is the zero map, and the inverse system $\big(\prod_{p \in \mathbb{P}} C_r/p\big)_r$ is essentially zero. \end{proof} \begin{lemma}\label{lemmaprozeroimpliesprozeropcompletion} Let $(C_r)_r$ be an inverse system in the derived category $\mathcal{D}(\Z)$. If $(C_r)_r$ is essentially zero, then $\big(\prod_{p \in \mathbb{P}} (C_r)^\wedge_p\big)_r$ is essentially zero. \end{lemma} \begin{proof} Assume that the inverse system $(C_r)_r$ is essentially zero. Let $r_0$ be an index of this inverse system, $n \in \Z$ be an integer, and $p$ be a prime number. We will use repeatedly that for every index $r \geq 0$, there is a short exact sequence $$0 \longrightarrow \text{Ext}^1_{\Z_p}(\Q_p/\Z_p, \text{H}^n(C_r)) \longrightarrow \text{H}^n((C_r)^\wedge_p) \longrightarrow \text{Hom}_{\Z_p}(\Q_p/\Z_p, \text{H}^{n+1}(C_r)) \longrightarrow 0$$ of abelian groups. Let $r_1 \geq r_0$ be an index such that the map $\text{H}^n(C_{r_1}) \rightarrow \text{H}^n(C_{r_0})$ is the zero map. Then for every index $r \geq r_1$, the map $\text{Ext}^1_{\Z_p}(\Q_p/\Z_p, \text{H}^n(C_r)) \rightarrow \text{Ext}^1_{\Z_p}(\Q_p/\Z_p, \text{H}^n(C_{r_0}))$ is the zero map, and the map $\text{H}^n((C_r)^\wedge_p) \rightarrow \text{H}^n((C_{r_0})^\wedge_p)$ thus factors through the map $$\text{H}^n((C_r)^\wedge_p) \longrightarrow \text{Hom}_{\Z_p}(\Q_p/\Z_p, \text{H}^{n+1}(C_r)).$$ Let $r_2 \geq r_1$ be an index such that the map $\text{H}^{n+1}(C_{r_2}) \rightarrow \text{H}^{n+1}(C_{r_1})$ is the zero map. Then the map $\text{Hom}_{\Z_p}(\Q_p/\Z_p, \text{H}^{n+1}(C_{r_2})) \rightarrow \text{Hom}_{\Z_p}(\Q_p/\Z_p, \text{H}^{n+1}(C_{r_1}))$ is the zero map. By construction, the map $$\text{H}^n((C_{r_2})^\wedge_p) \longrightarrow \text{H}^n((C_{r_0})^\wedge_p)$$ factors as $$\text{H}^n((C_{r_2})^\wedge_p) \longrightarrow \text{Hom}_{\Z_p}(\Q_p/\Z_p, \text{H}^{n+1}(C_{r_2})) \xlongrightarrow{0} \text{Hom}_{\Z_p}(\Q_p/\Z_p, \text{H}^{n+1}(C_{r_1})) \longrightarrow \text{H}^n((C_{r_0})^\wedge_p),$$ and is thus the zero map. The index $r_2$ does not depend on the prime number $p$, so the map $$\prod_{p \in \mathbb{P}} \text{H}^n((C_{r_2})^\wedge_p) \longrightarrow \prod_{p \in \mathbb{P}} \text{H}^n((C_{r_0})^\wedge_p)$$ is the zero map, and the inverse system $\big(\prod_{p \in \mathbb{P}} (C_r)^\wedge_p\big)_r$ is essentially zero. \end{proof} \begin{corollary}\label{corollaryprocdhdescentforcotangentcomplexandvariants} Let $j \geq 0$ be an integer, and let $F$ be one of the presheaves $$R\Gamma_{\emph{Zar}}\big(-,\mathbb{L}^j_{-/\Z}\big),\text{ } R\Gamma_{\emph{Zar}}\big(-,\prod_{p \in \mathbb{P}} \mathbb{L}^j_{-_{\F_p}/\F_p}\big), \text{ } R\Gamma_{\emph{Zar}}\big(-,\prod_{p \in \mathbb{P}}(\mathbb{L}^j_{-/\Z})^\wedge_p\big), \text{ and } R\Gamma_{\emph{Zar}}\big(-,\mathbb{L}^j_{-_{\Q}/\Q}\big),$$ where $-_{\F_p}$ is the derived base change from $\Z$ to $\F_p$. Then the presheaf $F$ is a pro cdh sheaf on noetherian schemes. \end{corollary} \begin{proof} The presheaf $F$ is a Nisnevich sheaf, so the result is equivalent to proving that $F$ sends an abstract blowup square of noetherian schemes to a weakly cartesian square of pro objects in the derived category~$\mathcal{D}(\Z)$. For $R\Gamma_{\text{Zar}}\big(-,\mathbb{L}^j_{-/\Z}\big)$, this is Proposition~\ref{propositionprocdhdescentforcotangentcomplex}. For $R\Gamma_{\text{Zar}}\big(-,\prod_{p \in \mathbb{P}} \mathbb{L}^j_{-_{\F_p}/\F_p}\big)$, this is a formal consequence of Proposition~\ref{propositionprocdhdescentforcotangentcomplex} and Lemma~\ref{lemmaprozeroimpliesprozeromodp}. For $R\Gamma_{\text{Zar}}\big(-,\prod_{p \in \mathbb{P}} (\mathbb{L}^j_{-/\Z})^\wedge_p\big)$, this is similarly a formal consequence of Proposition~\ref{propositionprocdhdescentforcotangentcomplex} and Lemma~\ref{lemmaprozeroimpliesprozeropcompletion}. And for $R\Gamma_{\text{Zar}}\big(-,\mathbb{L}^j_{-_{\Q}/\Q}\big)$, this is a consequence of Proposition~\ref{propositionprocdhdescentforcotangentcomplex} and the fact that the rationalisation of a zero pro system of abelian groups is zero. \end{proof} \subsection{Pro cdh descent for motivic cohomology} \vspace{-\parindent} \hspace{\parindent} In this subsection, we prove pro cdh descent for the motivic complexes $\Z(i)^{\text{mot}}$ (Theo\-rem \ref{theoremprocdhdescentformotiviccohomology}). We use Corollary~\ref{corollarymainconsequenceBFSongradedpieces} to decompose the proof into several steps, which ultimately all rely on Corollary~\ref{corollaryprocdhdescentforcotangentcomplexandvariants}. We start with the following rational results. \begin{proposition}[\cite{elmanto_motivic_2023}]\label{propositionprocdhdescentforderiveddeRhamchar0} For every integer $i \geq 0$, the presheaf $R\Gamma_{\emph{Zar}}\big(-,\widehat{\mathbb{L}\Omega}^{\geq i}_{-_{\Q}/\Q}\big)$ is a pro cdh sheaf on noetherian schemes. \end{proposition} \begin{proof} This is a part of \cite[proof of Theorem~$8.2$]{elmanto_motivic_2023}. More precisely, one uses Proposition~\ref{propositionfilteredHPisacdhsheafinchar0} to reduce the proof to a finite number of powers of the cotangent complex relative to $\Q$, where this is Corollary~\ref{corollaryprocdhdescentforcotangentcomplexandvariants}. \end{proof} The following result is a rigid-analytic variant of Proposition~\ref{propositionprocdhdescentforderiveddeRhamchar0}. \begin{proposition}\label{propositionprocdhdescentforrigidanalyticdR} For every integer $i \geq 0$, the presheaf $R\Gamma_{\emph{Zar}}\big(-,\prod'_{p \in \mathbb{P}} \underline{\widehat{\mathbb{L}\Omega}}^{\geq i}_{-_{\Q_p}/\Q_p}\big)$ is a pro cdh sheaf on noetherian schemes. \end{proposition} \begin{proof} By Remark~\ref{remark17fibreseuquenceforsolidderiveddeRhamcohomology}, there is a fibre sequence of presheaves $$R\Gamma_{\text{Zar}}\Big(-,\prod_{p \in \mathbb{P}}{}^{'} \underline{\widehat{\mathbb{L}\Omega}}^{\geq i}_{-_{\Q_p}/\Q_p}\Big) \longrightarrow R\Gamma_{\text{Zar}}\Big(-,\prod_{p \in \mathbb{P}}{}^{'} \underline{\widehat{\mathbb{L}\Omega}}_{-_{\Q_p}/\Q_p}\Big) \longrightarrow R\Gamma_{\text{Zar}}\Big(-,\big(\prod_{p \in \mathbb{P}} \big(\mathbb{L}\Omega^{<i}_{-/\Z}\big)^\wedge_p\big)_{\Q}\Big)$$ on qcqs derived schemes, and in particular on noetherian schemes. By Corollary~\ref{corollaryHPsolidallprimespisacdhsheafwithoutfiltration}, the presheaf $R\Gamma_{\text{Zar}}\big(-,\prod'_{p \in \mathbb{P}} \widehat{\underline{\mathbb{L}\Omega}}_{-_{\Q_p}/\Q_p}\big)$ is a cdh sheaf on noetherian schemes, so it is a pro cdh sheaf on noetherian schemes. The presheaf $R\Gamma_{\text{Zar}}\big(-,\big(\prod_{p \in \mathbb{P}} \big(\mathbb{L}\Omega^{<i}_{-/\Z}\big)^\wedge_p\big)_{\Q}\big)$ has a finite filtration with graded pieces given by the presheaves $R\Gamma_{\text{Zar}}\big(-,\big(\prod_{p \in \mathbb{P}} \big(\mathbb{L}^j_{-/\Z}\big)^\wedge_p\big)_{\Q}\big)$ ($0 \leq j < i$). These presheaves are pro cdh sheaves on noetherian schemes by Corollary~\ref{corollaryprocdhdescentforcotangentcomplexandvariants}, so the presheaf $R\Gamma_{\text{Zar}}\big(-,\big(\prod_{p \in \mathbb{P}} \big(\mathbb{L}\Omega^{<i}_{-/\Z}\big)^\wedge_p\big)_{\Q}\big)$ is a pro cdh sheaf on noetherian schemes. This implies that the presheaf $R\Gamma_{\text{Zar}}\big(-,\prod'_{p \in \mathbb{P}} \underline{\widehat{\mathbb{L}\Omega}}^{\geq i}_{-_{\Q_p}/\Q_p}\big)$ is a pro cdh sheaf on noetherian schemes. \end{proof} \begin{proposition}\label{propositionprocdhdescentforQ(i)TC} For every integer $i \geq 0$, the presheaf $\Q(i)^{\emph{mot}}$ is a pro cdh sheaf on noetherian schemes. \end{proposition} \begin{proof} By Corollary~\ref{corollaryrationalmainresultongradedpieces}, there is a fibre sequence of presheaves $$\Q(i)^{\text{mot}}(-) \longrightarrow \Q(i)^{\text{cdh}}(-) \longrightarrow \text{cofib}\Big(R\Gamma_{\text{Zar}}\big(-,\mathbb{L}\Omega^{<i}_{-_{\Q}/\Q}\big) \longrightarrow R\Gamma_{\text{cdh}}\big(-,\Omega^{<i}_{-_{\Q}/\Q}\big)\Big)[-1]$$ on qcqs derived schemes, and in particular on noetherian schemes. Cdh sheaves are in particular pro cdh sheaves, so it suffices to prove that the presheaf $R\Gamma_{\text{Zar}}\big(X,\mathbb{L}\Omega^{<i}_{-_{\Q}/\Q}\big)$ is a pro cdh sheaf on noetherian schemes. This presheaf has a finite filtration with graded pieces given by the presheaves $R\Gamma_{\text{Zar}}\big(-,\mathbb{L}^j_{-_{\Q}/\Q}\big)$ ($0 \leq j < i$), so the result is a consequence of Corollary~\ref{corollaryprocdhdescentforcotangentcomplexandvariants}. Alternatively, one can prove this result by using Corollary~\ref{corollaryKtheorysplitsrationally} and pro cdh descent for algebraic $K$-theory (\cite[Theorem~A]{kerz_algebraic_2018}). \end{proof} \begin{corollary}\label{corollaryprocdhdescentforQ(i)TC} For every integer $i \geq 0$, the presheaf $\Q(i)^{\emph{TC}}$ is a pro cdh sheaf on noetherian schemes. \end{corollary} \begin{proof} By Remark~\ref{remarkmaincartesiansquareformotiviccohomology}, the presheaf $\Q(i)^{\text{TC}}$ is a pro cdh sheaf on noetherian schemes if and only if the presheaf $\Q(i)^{\text{mot}}$ is a pro cdh sheaf on noetherian schemes. The result is then a consequence of Proposition~\ref{propositionprocdhdescentforQ(i)TC}. \end{proof} By Remark~\ref{remarkmaincartesiansquareformotiviccohomology}, the presheaf $\Q(i)^{\text{TC}}$ is a pro cdh sheaf on noetherian schemes if and only if the presheaf $\Q(i)^{\text{mot}}$ is a pro cdh sheaf on noetherian schemes. One can then prove Proposition~\ref{propositionprocdhdescentforQ(i)TC} alternatively by using Corollary~\ref{corollaryKtheorysplitsrationally} and pro cdh descent for algebraic $K$-theory (\cite[Theorem~A]{kerz_algebraic_2018}). We now turn our attention to Bhatt--Morrow--Scholze's syntomic complexes $\Z_p(i)^{\text{BMS}}$. \begin{corollary}\label{corollaryprocdhdescentforrationalisedBMSforallp} For every integer $i \geq 0$, the presheaf $\big(\prod_{p \in \mathbb{P}} \Z_p(i)^{\emph{BMS}}\big)_{\Q}$ is a pro cdh sheaf on noetherian schemes. \end{corollary} \begin{proof} Rationalising the cartesian square of Corollary~\ref{corollarymainconsequenceBFSongradedpieces} yields a cartesian square of presheaves $$\begin{tikzcd} \Q(i)^{\text{TC}}(-) \arrow{r} \arrow{d} & R\Gamma_{\text{Zar}}\Big(-,\widehat{\mathbb{L}\Omega}^{\geq i}_{-_{\Q}/\Q}\Big) \ar[d] \\ \big(\prod_{p \in \mathbb{P}} \Z_p(i)^{\text{BMS}}(-)\big)_{\Q} \arrow{r} & R\Gamma_{\text{Zar}}\Big(-,\prod'_{p \in \mathbb{P}} \underline{\widehat{\mathbb{L}\Omega}}^{\geq i}_{-_{\Q_p}/\Q_p}\Big) \end{tikzcd}$$ on qcqs derived schemes, and in particular on noetherian schemes. The other three presheaves of this cartesian square being pro cdh sheaves on noetherian schemes (Propositions~\ref{propositionprocdhdescentforderiveddeRhamchar0}, \ref{propositionprocdhdescentforrigidanalyticdR}, and \ref{propositionprocdhdescentforQ(i)TC}), the bottom left presheaf is also a pro cdh sheaf on noetherian schemes. \end{proof} \begin{lemma}\label{lemmaprocdhdescentformodsyntomiccohomology} Let $p$ be a prime number. Then for every integer $i \geq 0$, the presheaf $\F_p(i)^{\emph{BMS}}$ is a pro cdh sheaf on noetherian schemes. \end{lemma} \begin{proof} By \cite[Corollary~$5.31$]{antieau_beilinson_2020}, there exists an integer $m \geq 0$ and an equivalence of presheaves\footnote{Prismatic cohomology was first defined on $p$-complete $p$-quasisyntomic rings (\cite{bhatt_topological_2019,bhatt_prisms_2022}), and then generalised to arbitrary animated commutative rings by taking the left Kan extension from polynomial $\Z$-algebras, and imposing that it depends only on the derived $p$-completion of its input (\cite{antieau_beilinson_2020,bhatt_absolute_2022}). On noetherian rings $R$, the derived and classical $p$-completions agree, so the prismatic cohomology of $R$ is naturally identified with the prismatic cohomology of the classical $p$-completion of $R$.} $$\F_p(i)^{\text{BMS}}(-) \xlongrightarrow{\sim} \text{fib} \Big(\text{can}-\phi_i : (\mathcal{N}^{\geq i} \Prism_{-}\{i\}/\mathcal{N}^{\geq i+m} \Prism_{-}\{i\})/p \longrightarrow (\Prism_{-}\{i\}/\mathcal{N}^{\geq i+m} \Prism_{-}\{i\})/p\Big).$$ In particular, it suffices to prove that for every integer $j \geq 0$, the presheaf $\mathcal{N}^j \Prism_{-}/p$ is a pro cdh sheaf on noetherian schemes. By \cite[Remark~$5.5.8$ and Example~$4.7.8$]{bhatt_absolute_2022}, there is a fibre sequence of presheaves $$\mathcal{N}^j \Prism_{-} \{i\}/p \longrightarrow \text{Fil}^{\text{conj}}_j \overline{\Prism}_{-/\Z_p\llbracket \widetilde{p} \rrbracket}/p \xlongrightarrow{\Theta + j} \text{Fil}^{\text{conj}}_{j-1} \overline{\Prism}_{-/\Z_p\llbracket \widetilde{p} \rrbracket}/p.$$ The presheaves $\text{Fil}^{\text{conj}}_j \overline{\Prism}_{-/\Z_p\llbracket \widetilde{p} \rrbracket}/p$ and $\text{Fil}^{\text{conj}}_{j-1} \overline{\Prism}_{-/\Z_p\llbracket \widetilde{p} \rrbracket}/p$ have finite filtrations with graded pieces given by modulo $p$ powers of the cotangent complex. The result is then a consequence of Corollary~\ref{corollaryprocdhdescentforcotangentcomplexandvariants}. \end{proof} \begin{lemma}\label{lemmatorsionimpliesboundedtorsion} Let $A$ be an abelian group of the form $A=\prod_{p \in \mathbb{P}} A_p$, where $A_p$ is a derived $p$-complete abelian group. If $A$ is torsion, then $A$ is bounded torsion ({\it i.e.}, there exists an integer $N \geq 1$ such that $A$ is $N$-torsion). \end{lemma} \begin{proof} Assume that the abelian group $A$ is torsion. Then for every prime number $p$, the abelian group $A_p$ is torsion and derived $p$-complete, hence it is bounded $p$-power torsion by \cite[Theorem~$1.1$]{bhatt_torsion_2019}. Let $S$ be the set of prime numbers $p$ such that $A_p$ is not the zero group. Then there exists an inclusion of abelian groups $\prod_{p \in S} \F_p \subseteq A$, and, if $S$ is infinite, then $\prod_{p \in S} \F_p$ is not torsion. So $S$ is finite, and, as a finite product of bounded torsion abelian groups, the abelian group $A$ is bounded torsion. \end{proof} \begin{proposition}\label{propositionprocdhdescentforprodZp(i)syntomic} For every integer $i \geq 0$, the presheaf $\prod_{p \in \mathbb{P}} \Z_p(i)^{\emph{BMS}}$ is a pro cdh sheaf on noetherian schemes. \end{proposition} \begin{proof} Fix an abstract blowup square of noetherian schemes ($\ref{equationabstractblowupsquare}$). Let $\{C_r\}_r$ be the pro object in the derived category $\mathcal{D}(\Z)$ defined as the total fibre of the commutative square obtained by applying the presheaf $\prod_{p \in \mathbb{P}} \Z_p(i)^{\text{BMS}}$ to this abstract blowup square. We want to prove that $\{C_r\}_r$ is weakly zero. By Corollary~\ref{corollaryprocdhdescentforrationalisedBMSforallp}, its rationalisation $\{C_r \otimes_{\Z} \Q\}_r$ is weakly zero. Let $r_0 \geq 0$ and $n \in \Z$ be integers. Let $r_1 \geq r_0$ be an integer such that the map $$\text{H}^n(C_{r_1}) \otimes_{\Z} \Q \longrightarrow \text{H}^n(C_{r_0}) \otimes_{\Z} \Q$$ is the zero map. We now construct an integer $r_2 \geq r_1$ such that the map $$\text{H}^n(C_{r_2}) \longrightarrow \text{H}^n(C_{r_0})$$ is the zero map. By Lemma~\ref{lemmaprocdhdescentformodsyntomiccohomology}, and for every prime number $p$, the pro abelian group $\{\text{H}^n(C_r/p)\}_r$ is zero, which implies that the pro abelian group $\{\text{H}^n(C_r)/p\}_r$ is zero. By induction, this implies that for every integer $N \geq 1$, the pro abelian group $\{\text{H}^n(C_r)/N\}_r$ is zero. By construction, the cohomology groups $\text{H}^n(C_r)$ ($r \geq 0$) are naturally products, indexed by prime numbers $p$, of derived $p$-complete abelian groups. The kernel and cokernel of a map of derived $p$-complete abelian groups are derived $p$-complete abelian groups. So the image $A_{r_0}$ of the map $\text{H}^n(C_{r_1}) \rightarrow \text{H}^n(C_{r_0})$ is a product, indexed by prime numbers $p$, of derived $p$-complete abelian groups. This abelian group $A_{r_0}$ is also torsion by definition of the integer~$r_1$, so Lemma~\ref{lemmatorsionimpliesboundedtorsion} implies that there exists an integer $N \geq 1$ such that $A_{r_0}$ is $N$-torsion. Let~$r_2 \geq r_1$ be an integer such that the map $\text{H}^n(C_{r_2})/N \rightarrow \text{H}^n(C_{r_1})/N$ is the zero map. Then the map $$\text{H}^n(C_{r_2}) \longrightarrow \text{H}^n(C_{r_0})$$ factors as $$\text{H}^n(C_{r_2}) \longrightarrow \text{H}^n(C_{r_2})/N \xlongrightarrow{0} \text{H}^n(C_{r_1})/N \longrightarrow A_{r_0} \subseteq \text{H}^n(C_{r_0}),$$ and is thus the zero map, which concludes the proof. \end{proof} \begin{corollary}\label{corollaryprocdhdescentforsyntomiccomplexes} Let $p$ be a prime number. Then for every integer $i \geq 0$, the presheaf $\Z_p(i)^{\emph{BMS}}$ is a pro cdh sheaf on noetherian schemes. \end{corollary} \begin{proof} The presheaf $\Z_p(i)^{\text{BMS}}$ is a direct summand of the presheaf $\prod_{\l \in \mathbb{P}} \Z_{\l}(i)^{\text{BMS}}$, so the result is a consequence of Proposition~\ref{propositionprocdhdescentforprodZp(i)syntomic}. \end{proof} \begin{proposition}\label{propositionprocdhdescentforZ(i)TC} For every integer $i \geq 0$, the presheaf $\Z(i)^{\emph{TC}}$ is a pro cdh sheaf on noetherian schemes. \end{proposition} \begin{proof} By Corollary~\ref{corollarymainconsequenceBFSongradedpieces}, there is a cartesian square of presheaves $$\begin{tikzcd} \Z(i)^{\text{TC}} \ar[d] \ar[r] & R\Gamma_{\text{Zar}}\big(-,\widehat{\mathbb{L}\Omega}^{\geq i}_{-_{\Q}/\Q}\big) \ar[d] \\ \prod_{p \in \mathbb{P}} \Z_p(i)^{\text{BMS}} \ar[r] & R\Gamma_{\text{Zar}}(-,\prod'_{p \in \mathbb{P}} \underline{\widehat{\mathbb{L}\Omega}}^{\geq i}_{-_{\Q_p}/\Q_p}\big) \end{tikzcd}$$ on qcqs derived schemes, and in particular on noetherian schemes. The presheaves $$R\Gamma_{\text{Zar}}\big(-,\widehat{\mathbb{L}\Omega}^{\geq i}_{-_{\Q}/\Q}\big), \text{ } R\Gamma_{\text{Zar}}\big(-,\prod_{p \in \mathbb{P}}{}^{'} \underline{\widehat{\mathbb{L}\Omega}}^{\geq i}_{-_{\Q_p}/\Q_p}\big), \text{ and } \prod_{p \in \mathbb{P}} \Z_p(i)^{\text{BMS}}$$ are pro cdh sheaves on noetherian schemes by Propositions~\ref{propositionprocdhdescentforderiveddeRhamchar0}, \ref{propositionprocdhdescentforrigidanalyticdR}, and \ref{propositionprocdhdescentforprodZp(i)syntomic} respectively. So the presheaf $\Z(i)^{\text{TC}}$ is a pro cdh sheaf on noetherian schemes. \end{proof} The following result was proved on noetherian schemes over a field by Elmanto--Morrow \cite{elmanto_motivic_2023}. \begin{theorem}[Pro cdh descent]\label{theoremprocdhdescentformotiviccohomology} For every integer $i \geq 0$, the motivic complex $\Z(i)^{\emph{mot}}$ is a pro cdh sheaf on noetherian schemes. \end{theorem} \begin{proof} By Remark~\ref{remarkmaincartesiansquareformotiviccohomology}, there is a cartesian square of presheaves $$\begin{tikzcd} \Z(i)^{\text{mot}} \ar[d] \ar[r] & \Z(i)^{\text{TC}} \ar[d] \\ \Z(i)^{\text{cdh}} \ar[r] & L_{\text{cdh}} \Z(i)^{\text{TC}} \end{tikzcd}$$ on qcqs schemes, and in particular on noetherian schemes. The presheaf $\Z(i)^{\text{TC}}$ is a pro cdh sheaf on noetherian schemes by Proposition~\ref{propositionprocdhdescentforZ(i)TC}. The presheaves $\Z(i)^{\text{cdh}}$ and $L_{\text{cdh}} \Z(i)^{\text{TC}}$ are cdh sheaves on noetherian schemes by construction, hence pro cdh sheaves on noetherian schemes. So the presheaf $\Z(i)^{\text{mot}}$ is a pro cdh sheaf. \end{proof} \begin{remark}[Pro cdh descent for algebraic $K$-theory]\label{remarkprocdhdescentforKtheory} The arguments to prove Theorem~\ref{theoremprocdhdescentformotiviccohomology} can be adapted to give a new proof of the pro cdh descent for algebraic $K$-theory of Kerz--Strunk--Tamme \cite{kerz_algebraic_2018}. More precisely, by Theorem~\ref{theoremKST+LT}, pro cdh descent for algebraic $K$-theory is equivalent to pro cdh descent for TC. By Corollary~\ref{corollaryrationalKtheoryintermsofHC}, the result rationally reduces to the pro cdh descent for HC, which is proved by Morrow (\cite[Theorem~$0.2$]{morrow_pro_2016}). The result mod $p$ is similar to that of Lemma~\ref{lemmaprocdhdescentformodsyntomiccohomology}, where the Nygaard filtration and the relative prismatic cohomology are replaced by the Tate filtration and by relative THH; the pro cdh descent for relative THH then reduces to the pro cdh descent for powers of the cotangent complex by \cite[Section~$5.2$]{antieau_beilinson_2020}. Following Section~\ref{sectionrigidanalyticdR}, there is a natural cartesian square $$\begin{tikzcd} \text{TC}(-) \ar[r] \ar[d] & \text{HC}^-(-_{\Q}/\Q) \ar[d] \\ \prod_{p \in \mathbb{P}} \text{TC}(-;\Z_p) \ar[r] & \Big(\prod'_{p \in \mathbb{P}} \text{HH}(-;\Q_p)\Big)^{h\text{S}^1}. \end{tikzcd}$$ Using the cdh descent for the presheaves $\text{HP}(-_{\Q}/\Q)$ (\cite{land_k-theory_2019}) and $\big(\prod'_{p \in \mathbb{P}} \text{HH}(-;\Q_p)\big)^{t\text{S}^1}$ (Corollary~\ref{corollaryHPsolidallprimespisacdhsheafwithoutfiltration}), the pro cdh descent of the two right terms reduces to the pro cdh descent for HC. The integral statement is then similarly a consequence of Lemma~\ref{lemmatorsionimpliesboundedtorsion}. \end{remark} \subsection{Motivic Weibel vanishing} \vspace{-\parindent} \hspace{\parindent} In this subsection, we prove Theorem~\ref{theoremmotivicWeibelvanishing}, which is a motivic refinement of Weibel's vanishing conjecture on negative $K$-groups (\cite[Theorem~B\,$(i)$]{kerz_algebraic_2018}). \begin{lemma}\label{lemmamotiviccohomologyofhenselianvaluationringisindegreesatmosti} Let $V$ be a henselian valuation ring. Then for every integer $i \geq 0$, the motivic complex $\Z(i)^{\emph{mot}}(V) \in \mathcal{D}(\Z)$ is in degrees at most $i$. \end{lemma} \begin{proof} Henselian valuation rings are local rings for the cdh topology, so the natural maps $$\Z(i)^{\text{mot}}(V) \longrightarrow \Z(i)^{\text{cdh}}(V) \longleftarrow \Z(i)^{\text{lisse}}(V)$$ are equivalences in the derived category $\mathcal{D}(\Z)$ (Remark~\ref{remarkcomparisontocdhlocalmotiviccohomology} and Definition~\ref{definitionmotivicfiltrationonKHtheoryofschemes}). \end{proof} \begin{lemma}\label{lemmanilfibreofmotiviccohomologyisindegreesatmosti} Let $A$ be a local ring, and $I$ be a nil ideal of $A$. Then for every integer $i \geq 0$, the fibre of the natural map $$\Z(i)^{\emph{mot}}(A) \longrightarrow \Z(i)^{\emph{mot}}(A/I)$$ is in degrees at most $i$. \end{lemma} \begin{proof} We first prove the result rationally, and modulo $p$ for every prime number $p$. Any finitary cdh sheaf is invariant under nil extensions. By Corollary~\ref{corollaryrationalmainresultongradedpieces}, the result after rationalisation is thus equivalent to the fact that the fibre of the natural map $$\mathbb{L}\Omega^{<i}_{(A_{\Q})/\Q}[-1] \longrightarrow \mathbb{L}\Omega^{<i}_{((A/I)_{\Q})/\Q}[-1]$$ is in degrees at most $i$. Both terms of this map are in degrees at most $i$. In degree $i$, this map is given by the natural map $$\Omega^{i-1}_{(A_{\Q})/\Q} \longrightarrow \Omega^{i-1}_{((A/I)_{\Q})/\Q},$$ which is surjective as the $\Q$-algebra $(A/I)_{\Q}$ is a quotient of the $\Q$-algebra $A_{\Q}$. Let $p$ be a prime number. By Corollary~\ref{corollarymainpadicstructureongradeds}, the result modulo $p$ is equivalent to the fact that the fibre of the natural map $$\F_p(i)^{\text{BMS}}(A) \longrightarrow \F_p(i)^{\text{BMS}}(A/I)$$ is in degrees at most $i$. The pair $(A,I)$ is henselian, so this is a consequence of Theorem~\ref{theoremAMMNrigidity}. By the previous rational statement, the fibre $F \in \mathcal{D}(\Z)$ of the natural map $$\Z(i)^{\text{mot}}(A) \longrightarrow \Z(i)^{\text{mot}}(A/I)$$ has torsion cohomology groups in degrees at least $i+1$. By the short exact sequence of abelian groups $$0 \longrightarrow \text{H}^j(F)/p \longrightarrow \text{H}^j(F/p) \longrightarrow \text{H}^{j+1}(F)[p] \longrightarrow 0$$ for every prime number $p$ and every integer $j \geq i+1$, the previous torsion statement implies that these cohomology groups are also torsionfree, hence zero, in degrees at least $i+2$. It then remains to prove that the abelian group $\text{H}^{i+1}(F)$ is zero. By Corollary~\ref{corollaryHilbert90} and its proof, the abelian group $\text{H}^{i+1}_{\text{mot}}(A,\Z(i))$ is torsionfree, so it suffices to prove that the natural map of abelian groups $\text{H}^i_{\text{mot}}(A,\Z(i)) \rightarrow \text{H}^i_{\text{mot}}(A/I,\Z(i))$ is surjective. Let $P$ be a local ind-smooth $\Z$-algebra with a surjective map $P \rightarrow A$. By Theorem~\ref{theoremmotiviccohomologyisleftKanextendedonlocalringsinsmalldegrees} (see also Lemma~\ref{lemmasymbolmapfactorsthoughimprovedMilnorKgroup} for a related argument), and because $P \rightarrow A/I$ is also a surjection from a local ind-smooth $\Z$-algebra, the composite map of abelian groups $$\text{H}^i_{\text{mot}}(P,\Z(i)) \longrightarrow \text{H}^i_{\text{mot}}(A,\Z(i)) \longrightarrow \text{H}^i_{\text{mot}}(A/I,\Z(i))$$ is surjective, so the right map is surjective, as desired. \end{proof} \begin{theorem}[Motivic Weibel vanishing]\label{theoremmotivicWeibelvanishing} Let $d \geq 0$ be an integer, and $X$ be a noetherian scheme of dimension at most $d$. Then for every integer $i \geq 0$, the motivic complex $\Z(i)^{\emph{mot}}(X) \in \mathcal{D}(\Z)$ is in degrees at most $i+d$. \end{theorem} \begin{proof} The presheaf $\Z(i)^{\text{mot}} : \text{Sch}^{\text{qcqs,op}} \rightarrow \mathcal{D}(\Z)$ satisfies the following properties: \begin{enumerate} \item it is finitary (Corollary~\ref{corollarymotiviccomplexesarefinitary}); \item it satisfies pro cdh descent on noetherian schemes (Theorem~\ref{theoremprocdhdescentformotiviccohomology}); \item for every henselian valuation ring $V$, the complex $\Z(i)^{\text{mot}}(V)$ is in degrees at most~$i$ (Lemma~\ref{lemmamotiviccohomologyofhenselianvaluationringisindegreesatmosti}); \item for every noetherian local ring $A$ and every nilpotent ideal $I$ of $A$, the fibre of the natural map $\Z(i)^{\text{mot}}(A) \rightarrow \Z(i)^{\text{mot}}(A/I)$ is in degrees at most $i$ (Lemma~\ref{lemmanilfibreofmotiviccohomologyisindegreesatmosti}). \end{enumerate} By \cite{bachmann_A^1-invariant_2024}, the presheaf $\Z(i)^{\text{cdh}} : \text{Sch}^{\text{qcqs,op}} \rightarrow \mathcal{D}(\Z)$ is a finitary cdh sheaf which is in degrees at most $i$ on henselian valuation rings, hence it also satisfies the previous properties. By \cite[Proposition~$8.10$]{elmanto_motivic_2023} applied to the presheaf $\text{fib}\big(\Z(i)^{\text{mot}} \rightarrow \Z(i)^{\text{cdh}}\big)[i]$, this implies that for every noetherian scheme $X$ of dimension at most $d$, the complex $$\text{fib}\big(\Z(i)^{\text{mot}}(X) \longrightarrow \Z(i)^{\text{cdh}}(X)\big)$$ is in degrees at most $i+d$. The complex $\Z(i)^{\text{cdh}}(X)$ is also in degrees at most $i+d$ (\cite{bachmann_A^1-invariant_2024}), so the complex $\Z(i)^{\text{mot}}(X)$ is in degrees at most $i+d$. \end{proof} \begin{remark}[Relation to Weibel's $K$-theoretic vanishing conjecture]\label{remarkrelationWeibelKtheoreticconjecture} Let $X$ be a noetherian scheme of dimension at most $d$. Theorem~\ref{theoremmotivicWeibelvanishing} states that the Atiyah--Hirzebruch spectral sequence $$E_2^{i,j} = \text{H}^{i-j}_{\text{mot}}(X,\Z(-j)) \Longrightarrow \text{K}_{-i-j}(X)$$ is supported in the left half plane $x \leq d$: see the following representation of the $E_2$ page, where $\text{H}^j(i)$ denotes the motivic cohomology group $\text{H}^j_{\text{mot}}(X,\Z(i))$. \medskip \begin{equation*} \begin{tikzcd}[sep=tiny] \cdots & 0 & 0 & 0 & 0 & \cdots & 0 & 0 & 0 & 0 \\ \cdots & 0 & 0 & \text{H}^0(0) & \text{H}^1(0) & \cdots & \text{H}^{d-2}(0) \ar[rrd] & \text{H}^{d-1}(0) & \text{H}^d(0) & 0 \\ \cdots & 0 & \text{H}^0(1) \ar[rrd] & \text{H}^1(1) & \text{H}^2(1) & \cdots & \text{H}^{d-1}(1) \ar[rrd] & \text{H}^d(1) & \text{H}^{d+1}(1) & 0 \\ \cdots & \text{H}^0(2) & \text{H}^1(2) & \text{H}^2(2) & \text{H}^3(2) & \cdots & \text{H}^d(2) & \text{H}^{d+1}(2) & \text{H}^{d+2}(2) & 0 \\ & \vdots & \vdots & \vdots & \vdots & \vdots & \vdots & \vdots & \vdots & \vdots \end{tikzcd} \end{equation*} \medskip In particular, the negative $K$-groups $\text{K}_{-i-j}(X)$ vanish for $-i-j<-d$ (this is Weibel's vanishing conjecture on algebraic $K$-theory), and there is a natural edge map isomorphism $$\text{K}_{-d}(X) \cong \text{H}^d_{\text{mot}}(X,\Z(0))$$ of abelian groups. Using the description of weight zero motivic cohomology (Example~\ref{exampleweightzeromotiviccohomology}), the latter result recovers the known description of $\text{K}_{-d}(X)$ (\cite[Corollary~D]{kerz_algebraic_2018}). Note that Theorem~\ref{theoremmotivicWeibelvanishing} is however not a new proof of these results of Kerz--Strunk--Tamme, as our Atiyah--Hirzebruch spectral sequence relating motivic cohomology and algebraic $K$-theory relies on Theorem~\ref{theoremKST+LT}, which itself relies on the results in \cite{kerz_algebraic_2018}. \end{remark} \begin{remark}\label{remarkdescriptionofK_-d} Let $X$ be a noetherian scheme of dimension at most $d$. Then for every integer~$i \geq 0$, the proof of Theorem~\ref{theoremmotivicWeibelvanishing} also implies that the natural map $$\Z(i)^{\text{mot}}(X) \longrightarrow \Z(i)^{\text{cdh}}(X)$$ is surjective on $\text{H}^{i+d}$. For $i=0$, this map is even an isomorphism on $\text{H}^d$ (actually on all cohomology groups, by Example~\ref{exampleweightzeromotiviccohomology}), thus recovering Weibel's conjecture that the map $\text{K}_{-d}(X) \rightarrow \text{KH}_{-d}(X)$ is an isomorphism \cite{weibel_K-theory_1980,kerz_algebraic_2018}. \end{remark} The following result is a description of the group $\text{K}_{-d+1}$, similar to the description of the group $\text{K}_{-d}$ predicted by Weibel (Remark~\ref{remarkrelationWeibelKtheoreticconjecture}). \begin{corollary} Let $d \geq 0$ be an integer, and $X$ be a noetherian scheme of dimension at most $d$. Then there is a natural exact sequence $$\emph{H}^{d-2}_{\emph{cdh}}(X,\Z) \xlongrightarrow{\delta} \emph{H}^{d+1}_{\emph{mot}}(X,\Z(1)) \longrightarrow \emph{K}_{-d+1}(X) \longrightarrow \emph{H}^{d-1}_{\emph{cdh}}(X,\Z) \longrightarrow 0$$ of abelian groups, where $\delta$ is the differential map coming from the $E_2$-page of the Atiyah--Hirzebruch spectral sequence (Corollary~\ref{corollaryAHSS}). Moreover, for every integer $m \geq 2$, if $m$ is invertible in $X$, then the image of the map $(m-1)\delta$ is $m$-power torsion. \end{corollary} \begin{proof} The motivic complex $\Z(i) ^{\text{mot}}(X)$ is zero for $i<0$ (Corollary~\ref{corollarymotiviccomplexiszerofornegativeweight}), and is naturally identified with the complex $R\Gamma_{\text{cdh}}(X,\Z)$ for $i=0$ (Example~\ref{exampleweightzeromotiviccohomology}). The first statement is then a consequence of the Atiyah--Hirzebruch spectral sequence (Corollary~\ref{corollaryAHSS}) and of motivic Weibel's vanishing (Theorem~\ref{theoremmotivicWeibelvanishing}). The second statement is a consequence of the compatibility of the map $\delta$ with the Adams operation $\psi^m$ (Construction~\ref{constuction22'AdamsonKtheory}). More precisely, Corollary~\ref{corollary22''AdamsoperationsongradedpiecesoffilteredKtheory} implies that the induced map $$\delta : \text{H}^{d-2}_{\text{cdh}}(X,\Z)[\tfrac{1}{m}] \longrightarrow \text{H}^{d+1}_{\text{mot}}(X,\Z(1))[\tfrac{1}{m}]$$ satisfies the equation $\delta = m\delta$, {\it i.e.}, $(m-1)\delta=0$, which implies the desired result. \end{proof} \subsection{Comparison to pro cdh motivic cohomology} \vspace{-\parindent} \hspace{\parindent} In this subsection, we compare the motivic complexes $\Z(i)^{\text{mot}}$ to Kelly--Saito's pro cdh motivic complexes $\Z(i)^{\text{procdh}}$ (Theorem~\ref{theoremcomparisonprocdhmotivic}). In equicharacteristic, this is \cite[Corollary~$1.11$]{kelly_procdh_2024} (see also \cite[Theorem~$1.15$]{elmanto_motivic_2023}). Our proof is structurally the same, although finitariness and pro cdh descent in mixed characteristic rely on the main results of Section~\ref{sectionratonialstructure}, and our proof of the comparison to lisse motivic cohomology is different in mixed characteristic (see comment before Corollary~\ref{corollarylissemotivicmaincomparisontheorem}). \begin{lemma}\label{lemmanilextensionhenselianvaluationringlisse=mot} Let $R$ be a nil extension of a henselian valuation ring, {\it i.e.}, a commutative ring $R$ whose quotient $R/I$ by its ideal of nilpotent elements $I$ is a henselian valuation ring. Then for every integer $i \geq 0$, the lisse-motivic comparison map $$\Z(i)^{\emph{lisse}}(R) \longrightarrow \Z(i)^{\emph{mot}}(R)$$ is an equivalence in the derived category $\mathcal{D}(\Z)$. \end{lemma} \begin{proof} By Corollary~\ref{corollarylissemotivicmaincomparisontheorem}, it suffices to prove that the complex $\Z(i)^{\text{mot}}(R) \in \mathcal{D}(\Z)$ is in degrees at most~$i$. Let $I$ be the ideal of nilpotent elements of the commutative ring $R$. Using the natural fibre sequence $$\text{fib}\big(\Z(i)^{\text{mot}}(R) \longrightarrow \Z(i)^{\text{mot}}(R/I)\big) \longrightarrow \Z(i)^{\text{mot}}(R) \longrightarrow \Z(i)^{\text{mot}}(R/I)$$ in the derived category $\mathcal{D}(\Z)$, the result is then a consequence of Lemmas~\ref{lemmamotiviccohomologyofhenselianvaluationringisindegreesatmosti} and~\ref{lemmanilfibreofmotiviccohomologyisindegreesatmosti}. \end{proof} \begin{theorem}[Comparison to pro cdh motivic cohomology]\label{theoremcomparisonprocdhmotivic} Let $X$ be a noetherian scheme. Then for every integer $i \geq 0$, the lisse-motivic comparison map induces a natural equivalence $$\Z(i)^{\emph{procdh}}(X) \xlongrightarrow{\sim} \Z(i)^{\emph{mot}}(X)$$ in the derived category $\mathcal{D}(\Z)$. \end{theorem} \begin{proof} The presheaf $\Z(i)^{\text{mot}} : \text{Sch}^{\text{qcqs,op}} \rightarrow \mathcal{D}(\Z)$ satisfies the following properties: \begin{enumerate} \item it is finitary (Corollary~\ref{corollarymotiviccomplexesarefinitary}); \item it satisfies pro cdh descent on noetherian schemes (Theorem~\ref{theoremprocdhdescentformotiviccohomology}); \item for every pro cdh local ring $R$, the lisse-motivic comparison map $$\Z(i)^{\text{lisse}}(R) \longrightarrow \Z(i)^{\text{mot}}(R)$$ is an equivalence in the derived category $\mathcal{D}(\Z)$ (Lemma~\ref{lemmanilextensionhenselianvaluationringlisse=mot} and \cite[Proposition~$1.7$]{kelly_procdh_2024}). \end{enumerate} By \cite[Theorem~$9.7$]{kelly_procdh_2024},\footnote{This theorem is stated for schemes over a field, but the proof works over any noetherian commutative ring.} this implies that for every noetherian scheme $X$, the lisse-motivic comparison map induces a natural equivalence $$\Z(i)^{\text{procdh}}(X) \xlongrightarrow{\sim} \Z(i)^{\text{mot}}(X)$$ in the derived category $\mathcal{D}(\Z)$. \end{proof} \newpage \section{\texorpdfstring{$\mathbb{A}^1$}{TEXT}-invariant motivic cohomology-invariant motivic cohomology}\label{sectionA1invariantmotiviccohomology} \vspace{-\parindent} \hspace{\parindent} The theory of classical motivic cohomology of smooth schemes over a mixed characteristic Dedekind domain \cite{bloch_algebraic_1986,levine_techniques_2001,geisser_motivic_2004}, as a theory of $\mathbb{A}^1$-invariant motivic cohomology, admits a natural generalisation to general qcqs schemes. More precisely, Spitzweck constructed in \cite{spitzweck_commutative_2018}, for every qcqs scheme $X$, an $\mathbb{A}^1$-motivic spectrum $H\Z^{\text{Spi}} \in \text{SH}(\Z)$, which represents Bloch's cycle complexes on smooth $\Z$-schemes, and whose pullback to $\text{SH}(B)$, for $B$ a field or a mixed characteristic Dedekind domain, still represents Bloch's cycle complexes on smooth $B$-schemes. Bachmann then proved in \cite{bachmann_very_2022} that Spitzweck's construction coincides with the zeroth slice of the homotopy $K$-theory motivic spectrum $\text{KGL} \in \text{SH}(\Z)$. Finally, Bachmann--Elmanto--Morrow recently proved in \cite{bachmann_A^1-invariant_2024} that the slice filtration is compatible with arbitrary pullbacks, thus defining a well-behaved $\mathbb{A}^1$-motivic spectrum $H\Z_X \in \text{SH}(X)$ for arbitrary qcqs schemes $X$. The associated $\mathbb{A}^1$-invariant motivic complexes $$\Z(i)^{\mathbb{A}^1}(X) \in \mathcal{D}(\Z)$$ are related to the homotopy $K$-theory $\text{KH}(X)$ by an $\mathbb{A}^1$-invariant Atiyah--Hirzebruch spectral sequence. For our purposes, we will only use that there is a natural map $$\Z(i)^{\text{cdh}}(X) \longrightarrow \Z(i)^{\mathbb{A}^1}(X)$$ which exhibits the target as the $\mathbb{A}^1$-localisation of the source, and which is an equivalence if the qcqs scheme $X$ satisfies the condition $\text{Val}(X)$ (see Definition~\ref{definitionconditionVal} and Theorem~\ref{theoremBEMmotivicregularity}). \subsection{Comparison to \texorpdfstring{$\mathbb{A}^1$}{TEXT}-invariant motivic cohomology} \vspace{-\parindent} \hspace{\parindent} In this subsection, we prove that the $\mathbb{A}^1$-localisation of the motivic complexes $\Z(i)^{\text{mot}}$ recover Bachmann--Elmanto--Morrow's $\mathbb{A}^1$-invariant motivic complexes $\Z(i)^{\mathbb{A}^1}$ (Theorem~\ref{theoremA1localmotiviccohomologymain}). This is a motivic refinement of \cite[Theorem~$1.0.1$]{elmanto_thh_2021}, and implies that the motivic complexes $\Z(i)^{\text{mot}}$ recover the classical motivic complexes $\Z(i)^{\text{cla}}$ on smooth schemes over a mixed characteristic Dedekind domain after $\mathbb{A}^1$-localisation (Corollary~\ref{corollary26A1localmotiviccohomologyisclassicalmotiviccohomology}). \begin{lemma}\label{lemmaA1cdh=A1cdhA1} Let $\mathcal{C}$ be a presentable stable $\infty$-category, and $F : \emph{Sch}^{\emph{qcqs,op}} \rightarrow \mathcal{C}$ be a presheaf. Then the natural map $$L_{\mathbb{A}^1} L_{\emph{cdh}} F \longrightarrow L_{\mathbb{A}^1} L_{\emph{cdh}} L_{\mathbb{A}^1} F$$ induced by $\mathbb{A}^1$-localisation is an equivalence of $\mathcal{C}$-valued presheaves. \end{lemma} \begin{proof} A filtered colimit of cdh sheaves is a cdh sheaf (Lemma~\ref{lemmacdhsheafificationcommuteswithfilteredcolimits}). In particular, the $\mathbb{A}^1$-localisation of a cdh sheaf is a cdh sheaf, so the natural composites $$L_{\mathbb{A}^1} L_{\text{cdh}} F \longrightarrow L_{\mathbb{A}^1} L_{\text{cdh}} L_{\mathbb{A}^1} F \longrightarrow L_{\mathbb{A}^1} L_{\text{cdh}} L_{\mathbb{A}^1} L_{\text{cdh}} F$$ and $$L_{\mathbb{A}^1} L_{\text{cdh}} L_{\mathbb{A}^1} F \longrightarrow L_{\mathbb{A}^1} L_{\text{cdh}} L_{\mathbb{A}^1} L_{\text{cdh}} F \longrightarrow L_{\mathbb{A}^1} L_{\text{cdh}} L_{\mathbb{A}^1} L_{\text{cdh}} L_{\mathbb{A}^1} F$$ are equivalences in the $\infty$-category $\mathcal{C}$. This implies the desired result. \end{proof} \begin{lemma}\label{lemmaA1finitedeRhamiszerochar0} For every integer $i \geq 0$, the $\mathbb{A}^1$-localisation of the presheaf $$R\Gamma_{\emph{Zar}}\big(-,\mathbb{L}\Omega^{<i}_{-_{\Q}/\Q}\big) : \emph{dSch}^{\emph{qcqs,op}} \longrightarrow \mathcal{D}(\Q)$$ is zero. \end{lemma} \begin{proof} By Zariski descent, it suffices to prove the result on animated commutative rings. The functor $\mathbb{L}\Omega^{<i}_{-_{\Q}/\Q}$, from animated commutative rings to the derived category $\mathcal{D}(\Q)$, is left Kan extended from polynomial $\Z$-algebras. Equivalently, it commutes with sifted colimits. This property being preserved by $\mathbb{A}^1$-localisation, the functor $L_{\mathbb{A}^1} \mathbb{L}\Omega^{<i}_{-_{\Q}/\Q}$ is also left Kan extended from polynomial $\Z$-algebras. As it is also constant on polynomial $\Z$-algebras, and zero on the zero ring, it is the zero functor. \end{proof} \begin{corollary} Let $X$ be a qcqs derived scheme. Then for every integer $i \geq 0$, the natural map $$\big(L_{\mathbb{A}^1} \widehat{\mathbb{L}\Omega}^{\geq i}_{-_{\Q}/\Q}\big)(X) \longrightarrow \big(L_{\mathbb{A}^1} \widehat{\mathbb{L}\Omega}_{-_{\Q}/\Q}\big)(X)$$ is an equivalence in the derived category $\mathcal{D}(\Q)$. \end{corollary} \begin{proof} There is a natural fibre sequence $$\big(L_{\mathbb{A}^1} \widehat{\mathbb{L}\Omega}^{\geq i}_{-_{\Q}/\Q}\big)(X) \longrightarrow \big(L_{\mathbb{A}^1} \widehat{\mathbb{L}\Omega}_{-_{\Q}/\Q}\big)(X) \longrightarrow \big(L_{\mathbb{A}^1} \mathbb{L}\Omega^{<i}_{-_{\Q}/\Q}\big)$$ in the derived category $\mathcal{D}(\Q)$. The result is then a consequence of Lemma~\ref{lemmaA1finitedeRhamiszerochar0}. \end{proof} \begin{lemma}\label{lemmaA1syntomiccohoiszero} Let $p$ be a prime number. Then for every integer $i \geq 0$, the $\mathbb{A}^1$-localisation of the presheaf $$\F_p(i)^{\emph{BMS}}(-) : \emph{dSch}^{\emph{qcqs,op}} \longrightarrow \mathcal{D}(\F_p)$$ is zero. \end{lemma} \begin{proof} The presheaf $\F_p(i)^{\text{BMS}}$ is a Zariski sheaf, and its restriction to animated commutative rings is left Kan extended from polynomial $\Z$-algebras (Corollary~\ref{corollaryBMSsyntomiccohomologyhasquasisyntomicdescentandLKEfrompolynomialZalgebras}). The result then follows by the same argument as in Lemma~\ref{lemmaA1finitedeRhamiszerochar0}. \end{proof} \begin{theorem}\label{theoremA1localmotiviccohomologymain} Let $X$ be a qcqs scheme. Then for every integer $i \geq 0$, the natural map $$\big(L_{\mathbb{A}^1} \Z(i)^{\emph{mot}}\big)(X) \longrightarrow \big(L_{\mathbb{A}^1} \Z(i)^{\emph{cdh}}\big)(X) \simeq \Z(i)^{\mathbb{A}^1}(X)$$ is an equivalence in the derived category $\mathcal{D}(\Z)$. \end{theorem} \begin{proof} It suffices to prove the result rationally, and modulo $p$ for every prime number $p$. By Lemma~\ref{lemmaA1finitedeRhamiszerochar0}, the object $$\big(L_{\mathbb{A}^1} R\Gamma_{\text{Zar}}\big(X,\mathbb{L}\Omega^{<i}_{-_{\Q}/\Q}\big)\big)(X)$$ is zero in the derived category $\mathcal{D}(\Q)$. Lemma~\ref{lemmaA1cdh=A1cdhA1} then implies that the object $$\big(L_{\mathbb{A}^1} R\Gamma_{\text{cdh}}(-,\mathbb{L}\Omega^{<i}_{-_{\Q}/\Q}\big)\big)(X)$$ is zero in the derived category $\mathcal{D}(\Q)$. In particular, the natural map $$\big(L_{\mathbb{A}^1} R\Gamma_{\text{Zar}}\big(-, \mathbb{L}\Omega^{<i}_{-_{\Q}/\Q}\big)\big)(X) \longrightarrow \big(L_{\mathbb{A}^1} R\Gamma_{\text{cdh}}(-,\mathbb{L}\Omega^{<i}_{-_{\Q}/\Q}\big)\big)(X)$$ is an equivalence in the derived category $\mathcal{D}(\Q)$, which implies the desired result rationally by Corollary~\ref{corollaryrationalmainresultongradedpieces}. Similarly, for every prime number $p$, the syntomic complex $\F_p(i)^{\text{BMS}}$ vanishes after $\mathbb{A}^1$\nobreakdash-locali\-sation by Lemma~\ref{lemmaA1syntomiccohoiszero}, which implies the desired result modulo $p$ by Lemma~\ref{lemmaA1cdh=A1cdhA1} and Corollary~\ref{corollarymainpadicstructureongradeds}. \end{proof} \begin{remark} Let $X$ be a qcqs derived scheme. One can prove, using similar arguments and Corollary~\ref{corollarymainconsequenceBFSongradedpieces}, that there is a natural fibre sequence $$\big(L_{\mathbb{A}^1} \Z(i)^{\text{TC}}\big)(X) \longrightarrow R\Gamma_{\text{Zar}}\big(X,\widehat{\mathbb{L}\Omega}_{-_{\Q}/\Q}\big) \longrightarrow R\Gamma_{\text{Zar}}\big(X,\prod_{p \in \mathbb{P}} {}^{'} \underline{\widehat{\mathbb{L}\Omega}}_{-_{\Q_p}/\Q_p}\big)$$ in the derived category $\mathcal{D}(\Z)$. Note that this implies Theorem~\ref{theoremA1localmotiviccohomologymain} on qcqs classical schemes, via the cdh descent results \cite[Lemma~$4.5$]{elmanto_motivic_2023} and Corollary~\ref{corollaryHPsolidallprimespisacdhsheafongradedpieces}. \end{remark} \begin{theorem}\label{theorem25A1localmotivicfiltrationisclassicalfiltration} Let $X$ be a smooth scheme over a mixed characteristic Dedekind domain. Then the natural map $$\emph{Fil}^\star_{\emph{cla}} \emph{K}(X) \longrightarrow \big(L_{\mathbb{A}^1} \emph{Fil}^\star_{\emph{mot}} \emph{K}\big)(X)$$ is an equivalence of filtered spectra. \end{theorem} \begin{proof} By Proposition~\ref{propositionrationalcomparisonclassicalmotivic} and its proof, this natural map is an equivalence rationally, so it suffices to prove that it is an equivalence modulo every prime number $p$. Let $p$ be a prime number. By \cite{bachmann_A^1-invariant_2024}, the natural map $$\text{Fil}^\star_{\text{cla}} \text{K}(X) \longrightarrow \big(L_{\mathbb{A}^1} \text{Fil}^\star_{\text{cdh}} \text{KH}\big)(X)$$ is an equivalence of filtered spectra, so it suffices to prove that the natural map $$\big(L_{\mathbb{A}^1} \text{Fil}^\star_{\text{mot}} \text{K}\big)(X)/p \longrightarrow \big(L_{\mathbb{A}^1} \text{Fil}^\star_{\text{cdh}} \text{KH}\big)(X)/p$$ is an equivalence of filtered spectra. By Proposition~\ref{propositionpadicstructuremain}, this is equivalent to the fact that the natural map $$\big(L_{\mathbb{A}^1} \text{Fil}^\star_{\text{BMS}} \text{TC}(-;\F_p)\big)(X) \longrightarrow \big(L_{\text{A}^1} L_{\text{cdh}} \text{Fil}^\star_{\text{BMS}} \text{TC}(-;\F_p)\big)(X)$$ is an equivalence of filtered spectra. The filtered spectrum $\big(L_{\mathbb{A}^1} \text{Fil}^\star_{\text{BMS}} \text{TC}(-;\F_p)\big)(X)$ is complete by the connectivity bound of Lemma~\ref{lemmaBMSfiltrationproductallprimesiscomplete}, and its graded pieces are zero by Lemma~\ref{lemmaA1syntomiccohoiszero}, so it is zero. By Lemma~\ref{lemmaA1cdh=A1cdhA1}, the target $\big(L_{\text{A}^1} L_{\text{cdh}} \text{Fil}^\star_{\text{BMS}} \text{TC}(-;\F_p)\big)(X)$ of the previous map is then also zero, and this map is in particular an equivalence. \end{proof} \begin{remark}\label{remark27A1localisationshouldnotbenecessary} We expect that the $\mathbb{A}^1$-localisation in Theorem~\ref{theorem25A1localmotivicfiltrationisclassicalfiltration} is not necessary, {\it i.e.}, that the natural map $$\text{Fil}^\star_{\text{mot}} \text{K}(X) \longrightarrow \big(L_{\mathbb{A}^1} \text{Fil}^\star_{\text{mot}} \text{K}\big)(X)$$ is an equivalence of filtered spectra for smooth schemes $X$ over a torsionfree Dedekind domain. By Theorem~\ref{theoremA1localmotiviccohomologymain}, this is equivalent to the fact that the composite $$\text{Fil}^\star_{\text{mot}} \text{K}(X) \xlongrightarrow{L_{\text{cdh}}} \text{Fil}^\star_{\text{cdh}} \text{KH}(X) \xlongrightarrow{L_{\mathbb{A}^1}} \big(L_{\mathbb{A}^1} \text{Fil}^\star_{\text{cdh}} \text{KH}\big)(X)$$ is an equivalence of filtered spectra, where the second map is expected to be an equivalence for every qcqs scheme $X$ (\cite{bachmann_A^1-invariant_2024}). \end{remark} \begin{corollary}\label{corollary26A1localmotiviccohomologyisclassicalmotiviccohomology} Let $X$ be a smooth scheme over a mixed characteristic Dedekind domain. Then for every integer $i \geq 0$, there is a natural equivalence $$z^i(X,\bullet)[-2i] \simeq \big(L_{\mathbb{A}^1} \Z(i)^{\emph{mot}}\big)(X)$$ in the derived category $\mathcal{D}(\Z)$. \end{corollary} \begin{proof} This is a consequence of Theorem~\ref{theorem25A1localmotivicfiltrationisclassicalfiltration}. \end{proof} \subsection{\texorpdfstring{$F$}{TEXT}-smoothness of valuation rings}\label{subsectionconditionVal} \vspace{-\parindent} \hspace{\parindent} In this subsection, we formulate the key hypothesis used in the work of Bachmann--Elmanto--Morrow \cite{bachmann_A^1-invariant_2024} on $\mathbb{A}^1$-invariant motivic cohomology (Definition~\ref{definitionconditionVal}). This hypothesis relies on the following conjecture, implicit in the work of Bhatt--Mathew \cite{bhatt_syntomic_2023} on $F$\nobreakdash-smoothness. \begin{conjecture}[Bhatt--Mathew]\label{conjecturevaluationringsareFsmooth} Every valuation ring is $F$-smooth. \end{conjecture} The following theorem summarizes the known cases of Conjecture~\ref{conjecturevaluationringsareFsmooth}. Note that the notion of $F$-smoothness implicitly depends on a fixed prime number $p$, and that a commutative ring with bounded $p$-power torsion is $F$-smooth if and only if its $p$-completion is $F$-smooth. \begin{theorem}\label{theoremknowncasesofvaluationringsbeingFsmooth} Let $V$ be a valuation ring, and $p$ be a prime number. \begin{enumerate} \item (Bhatt--Mathew \cite{bhatt_syntomic_2023}) If $V$ is a discrete valuation ring, then $V$ is $F$-smooth. \item (Gabber--Ramero \cite{gabber_almost_2003}, Gabber \cite{kerz_towards_2021}, Kelly--Morrow \cite{kelly_k-theory_2021}) If $V$ is a valuation ring extension of $\F_p$, then $V$ is $F$-smooth. \item (Bouis \cite{bouis_cartier_2023}) If $V$ is a valuation ring extension of a $p$-torsionfree perfectoid valuation ring of mixed characteristic $(0,p)$, then $V$ is $F$-smooth. \end{enumerate} \end{theorem} \begin{definition}\label{definitionconditionVal} \begin{enumerate} \item Let $p$ be a prime number (on which the notion of $F$-smoothness implicitly depends). A qcqs scheme $X$ {\it satisfies the condition} $\text{Val}(X,p)$ if every valuation ring $V$ with a map $\text{Spec}(V) \rightarrow X$ is $F$-smooth. \item A qcqs scheme $X$ {\it satisfies the condition} $\text{Val}(X)$ if it satisfies the condition $\text{Val}(X,p)$ for every prime number $p$. \end{enumerate} \end{definition} \begin{examples}\label{examplesconditionVal} Let $p$ be a prime number. \begin{enumerate} \item Every qcqs $\Z[\tfrac{1}{p}]$-scheme satisfies the condition $\text{Val}(X,p)$ because every valuation ring in which $p$ is invertible is vacuously $F$-smooth. Consequently, every qcqs $\Q$-scheme satisfies the condition $\text{Val}(X)$. \item Every qcqs $\F_p$-scheme satisfies the condition $\text{Val}(X)$ (Theorem~\ref{theoremknowncasesofvaluationringsbeingFsmooth}$\,(2)$). More generally, for every integer $N \geq 1$, every qcqs $\Z/N$-scheme satisfies the condition $\text{Val}(X)$. \item Every qcqs $V$-scheme, where $V$ is a commutative ring with bounded $p$-power torsion and whose $p$\nobreakdash-completion is a perfectoid valuation ring of mixed characteristic $(0,p)$, satisfies the condition $\text{Val}(X)$ (Theorem~\ref{theoremknowncasesofvaluationringsbeingFsmooth}$\,(3)$ for the condition $\text{Val}(X,p)$, and $(1)$ for the condition $\text{Val}(X,\l)$ at all other primes $\l$). \end{enumerate} \end{examples} \begin{remark} The results in \cite{bachmann_A^1-invariant_2024} that are conditional on some hypothesis on valuation rings actually assume that valuation rings satisfy a suitable syntomic-étale comparison theorem. Such a condition is weaker than that of Definition~\ref{definitionconditionVal}, as this syntomic-étale comparison is a consequence of $F$-smoothness (\cite[Theorem~$1.8$]{bhatt_syntomic_2023}). Note that all known cases of this syntomic-étale comparison for mixed characteristic valuation rings are proved as consequences of $F$\nobreakdash-smoothness. \end{remark} \subsection{Motivic regularity} \vspace{-\parindent} \hspace{\parindent} By Quillen's fundamental theorem of algebraic $K$-theory, the algebraic $K$-theory of a regular noetherian ring is $\mathbb{A}^1$-invariant. Vorst conjectured a partial converse of this result, {\it i.e.}, that an essentially finite type algebra over a field whose $K$-groups are $\mathbb{A}^1$-invariant is regular (\cite{vorst_localization_1979}). The conjecture was proved in characteristic zero by Corti$\tilde{\text{n}}$as--Haesemeyer--Weibel (\cite{cortinas_K-regularity_2008}). Kerz--Strunk--Tamme then proved a variant of the conjecture in positive characteristic (\cite[Theorem~A]{kerz_towards_2021}), and asked if Vorst's conjecture holds for general excellent noetherian rings (\cite[Question~D]{kerz_towards_2021}). In this subsection, we study the extent to which these questions have a natural analogue in motivic cohomology. More precisely, given a qcqs scheme $X$ and an integer $i \geq 0$, and in light of Theorem~\ref{theoremA1localmotiviccohomologymain}, we will be interested in when the natural map $$\Z(i)^{\text{mot}}(X) \longrightarrow \Z(i)^{\mathbb{A}^1}(X)$$ is an equivalence in the derived category $\mathcal{D}(\Z)$. We will use repeatedly the following result of \cite{bachmann_A^1-invariant_2024}. \begin{theorem}[\cite{bachmann_A^1-invariant_2024}]\label{theoremBEMmotivicregularity} Let $i \geq 0$ be an integer. \begin{enumerate} \item For every qcqs scheme $X$, the natural map $$\Q(i)^{\emph{cdh}}(X) \longrightarrow \Q(i)^{\mathbb{A}^1}(X)$$ is an equivalence in the derived category $\mathcal{D}(\Q)$. \item For every prime number $p$ and every qcqs $\Z[\tfrac{1}{p}]$-scheme $X$, the natural map $$\F_p(i)^{\emph{cdh}}(X) \longrightarrow \F_p(i)^{\mathbb{A}^1}(X)$$ is an equivalence in the derived category $\mathcal{D}(\F_p)$. \item For every qcqs scheme $X$ satisfying the condition $\emph{Val}(X)$ (Definition~\ref{definitionconditionVal}), the natural map $$\Z(i)^{\emph{cdh}}(X) \longrightarrow \Z(i)^{\mathbb{A}^1}(X)$$ is an equivalence in the derived category $\mathcal{D}(\Z)$. \end{enumerate} \end{theorem} \begin{theorem}[Rational motivic regularity]\label{theoremrationalmotivicregularity} Let $X$ be a qcqs scheme. Then for every integer $i \geq 0$, the following are equivalent: \begin{enumerate} \item the natural map $\Q(i)^{\emph{mot}}(X) \rightarrow \Q(i)^{\mathbb{A}^1}(X)$ is an equivalence in the derived category $\mathcal{D}(\Q)$; \item the natural map $\Q(i)^{\emph{mot}}(X_{\Q}) \rightarrow \Q(i)^{\mathbb{A}^1}(X_{\Q})$ is an equivalence in the derived category $\mathcal{D}(\Q)$. \end{enumerate} Moreover, $(1)$ or $(2)$ for all integers $i \geq 0$ is equivalent to each of the following statements: \begin{enumerate}\setcounter{enumi}{2} \item for every integer $j \geq 0$, the natural map $R\Gamma_{\emph{Zar}}(X,\mathbb{L}^j_{-_{\Q}/\Q}) \rightarrow R\Gamma_{\emph{cdh}}(X,\Omega^j_{-_{\Q}/\Q})$ is an equivalence in the derived category $\mathcal{D}(\Q)$; \item the natural map $\emph{K}(X;\Q) \rightarrow \emph{KH}(X;\Q)$ is an equivalence of spectra; \item the natural map $\emph{HC}(X_{\Q}/\Q) \rightarrow L_{\emph{cdh}} \emph{HC}(-_{\Q}/\Q)(X)$ is an equivalence of spectra. \end{enumerate} \end{theorem} \begin{proof} $(4)$ and $(5)$ are equivalent by Corollary~\ref{corollaryrationalmainresultongradedpieces}. Theorem~\ref{theoremBEMmotivicregularity}\,$(1)$ and the Adams decompositions Corollary~\ref{corollaryKtheorysplitsrationally} and $$\text{KH}(X;\Q) \simeq \bigoplus_{i \geq 0} \Q(i)^{\text{cdh}}(X)$$ then imply that $(1)$ for all integers $i \geq 0$ and $(4)$ are equivalent. By Corollary~\ref{corollaryrationalmainresultongradedpieces}, $(1)$ is equivalent to the fact that the natural map $$R\Gamma_{\text{Zar}}\big(X,\mathbb{L}\Omega^{<i}_{-_{\Q}/\Q}\big) \longrightarrow R\Gamma_{\text{cdh}}\big(X,\Omega^{<i}_{-_{\Q}/\Q}\big)$$ is an equivalence in the derived category $\mathcal{D}(\Q)$, which is in turn equivalent to $(2)$, and implies that $(1)$ for all integers $i \geq 0$ is equivalent to $(3)$. \end{proof} \begin{corollary}[Motivic regularity in characteristic zero]\label{corollarymotivicregularityinchar0} Let $X$ be a qcqs $\Q$-scheme. Then the following are equivalent: \begin{enumerate} \item for every integer $i \geq 0$, the natural map $\Z(i)^{\emph{mot}}(X) \rightarrow \Z(i)^{\mathbb{A}^1}(X)$ is an equivalence in the derived category $\mathcal{D}(\Z)$; \item for every integer $j \geq 0$, the natural map $R\Gamma_{\emph{Zar}}(X,\mathbb{L}^j_{-/\Q}) \rightarrow R\Gamma_{\emph{cdh}}(X,\Omega^j_{-/\Q})$ is an equivalence in the derived category $\mathcal{D}(\Q)$; \item the natural map $\emph{K}(X) \rightarrow \emph{KH}(X)$ is an equivalence of spectra; \item the natural map $\emph{HC}(X/\Q) \rightarrow L_{\emph{cdh}} \emph{HC}(-/\Q)(X)$ is an equivalence of spectra. \end{enumerate} \end{corollary} \begin{proof} For every prime number $p$, the natural map $$\text{K}(X;\F_p) \longrightarrow \text{KH}(X;\F_p)$$ is an equivalence of spectra on qcqs $\Q$-schemes (\cite[Proposition~$1.6$]{weibel_homotopy_1989}). Similarly, for any prime number $p$ and integer $i \geq 0$, the natural map $$\F_p(i)^{\text{mot}}(X) \longrightarrow \F_p(i)^{\text{cdh}}(X)$$ is an equivalence in the derived category $\mathcal{D}(\F_p)$ (Remark~\ref{remarkladicmotiviccohomology}). In particular, the natural map $$\text{K}(X) \longrightarrow \text{KH}(X) \quad \text{(resp.} \text{ } \Z(i)^{\text{mot}}(X) \longrightarrow \Z(i)^{\text{cdh}}(X)\text{)}$$ is an equivalence of spectra (resp. in the derived category $\mathcal{D}(\Z)$) if and only if the natural map $$\text{K}(X;\Q) \longrightarrow \text{KH}(X;\Q) \quad \text{(resp.} \text{ } \Q(i)^{\text{mot}}(X) \longrightarrow \Q(i)^{\text{cdh}}(X)\text{)}$$ is an equivalence of spectra (resp. in the derived category $\mathcal{D}(\Q)$). The result then follows from Theorems~\ref{theoremBEMmotivicregularity}\,$(1)$ and~\ref{theoremrationalmotivicregularity}. \end{proof} \begin{proposition}[$\l$-adic motivic regularity]\label{propositionladicmotivicregularity} Let $\l$ be a prime number, and $X$ be a qcqs $\Z[\tfrac{1}{\l}]$-scheme. Then for any integers $i \geq 0$ and $k \geq 1$, the natural map $$\Z/\l^k(i)^{\emph{mot}}(X) \longrightarrow \Z/\l^k(i)^{\mathbb{A}^1}(X)$$ is an equivalence in the derived category $\mathcal{D}(\Z/\l^k)$. \end{proposition} \begin{proof} This is a consequence of Remark~\ref{remarkladicmotiviccohomology} and Theorem~\ref{theoremBEMmotivicregularity}\,$(2)$. \end{proof} Recall the following known result at the level of $K$-theory in positive characteristic. \begin{corollary}\label{corollaryKregularityoverZ/N} Let $N \geq 1$ be an integer, and $X$ be a qcqs $\Z/N$-scheme. Then the natural map $\emph{K}(X) \rightarrow \emph{KH}(X)$ is an equivalence of spectra if and only if for every prime number $p$ dividing $N$, the natural map $\emph{TC}(X;\F_p) \rightarrow L_{\emph{cdh}} \emph{TC}(-;\F_p)(X)$ is an equivalence of spectra. \end{corollary} \begin{proof} For every prime number $p$, the functor $\text{TC}(-;\F_p)$ is zero on qcqs $\Z[\tfrac{1}{p}]$-schemes. The natural map $\text{K}(X) \rightarrow \text{KH}(X)$ is an equivalence if and only if it is an equivalence rationally, and modulo $p$ for every prime number $p$. The result is then a consequence of Theorems~\ref{theoremKST+LT} and \ref{theoremrationalmotivicregularity}\,$(4)$-$(5)$. \end{proof} The following result is a motivic analogue of Corollary~\ref{corollaryKregularityoverZ/N}. \begin{corollary}[Motivic regularity in positive characteristic]\label{corollarymotivicregularityoverZ/N} Let $N \geq 1$ and $i \geq 0$ be integers, and $X$ be a qcqs $\Z/N$-scheme. Then the natural map $\Z(i)^{\emph{mot}}(X) \rightarrow \Z(i)^{\mathbb{A}^1}(X)$ is an equivalence in the derived category $\mathcal{D}(\Z)$ if and only if for every prime number $p$ dividing $N$, the natural map $\F_p(i)^{\emph{BMS}}(X) \rightarrow (L_{\emph{cdh}} \F_p(i)^{\emph{BMS}})(X)$ is an equivalence in the derived category~$\mathcal{D}(\F_p)$. \end{corollary} \begin{proof} By Theorem~\ref{theoremBEMmotivicregularity}\,$(3)$ and Example~\ref{examplesconditionVal}\,$(2)$, the natural map $$\Z(i)^{\text{mot}}(X) \longrightarrow \Z(i)^{\mathbb{A}^1}(X)$$ is an equivalence if and only if the natural map $\Z(i)^{\text{mot}}(X) \rightarrow \Z(i)^{\text{cdh}}(X)$ is an equivalence. The natural map $\Z(i)^{\text{mot}}(X) \rightarrow \Z(i)^{\text{cdh}}(X)$ is an equivalence if and only if it is an equivalence rationally, and modulo $p$ for every prime number $p$. For every prime number $p$, the functor $\F_p(i)^{\text{BMS}}$ is zero on qcqs $\Z[\tfrac{1}{p}]$-schemes. The result is then a consequence of Corollary~\ref{corollarymainpadicstructureongradeds} and Theorem~\ref{theoremrationalmotivicregularity}\,\hbox{$(1)$-$(2)$}. \end{proof} The following result is \cite[Theorem~$6.1\,(2)$]{elmanto_motivic_2023}, where we use Theorem~\ref{theoremBEMmotivicregularity} and Example~\ref{examplesconditionVal}\,$(2)$ to identify cdh-local and $\mathbb{A}^1$-invariant motivic cohomologies in characteristic~$p$. \begin{theorem}[\cite{elmanto_motivic_2023}]\label{theoremelmantomorrowmotiviccohomologyisA1invariantonregularFpschemes} Let $p$ be a prime number, and $X$ be an ind-regular $\F_p$-scheme. Then for every integer $i \geq 0$, the natural map $$\Z(i)^{\emph{mot}}(X) \longrightarrow \Z(i)^{\mathbb{A}^1}(X)$$ is an equivalence in the derived category $\mathcal{D}(\Z)$. \end{theorem} \begin{corollary}\label{corollaryA1invarianceforsmoothschemesoverperfectvaluationringcharacteristicp} Let $p$ be a prime number, $V$ be a characteristic $p$ perfect valuation ring, and $X$ be an ind-smooth scheme over $V$. Then for every integer $i \geq 0$, the natural map $$\Z(i)^{\emph{mot}}(X) \longrightarrow \Z(i)^{\mathbb{A}^1}(X)$$ is an equivalence in the derived category $\mathcal{D}(\Z)$. \end{corollary} \begin{proof} By \cite[Proposition~$4.1.1$]{antieau_valuation_2021}, the perfect valuation ring $V$ is the filtered colimit of its smooth $\F_p$-subalgebras. So the scheme $X$ is in particular ind-smooth over $\F_p$, and the result is a consequence of Theorem~\ref{theoremelmantomorrowmotiviccohomologyisA1invariantonregularFpschemes}. \end{proof} We now study motivic regularity in mixed characteristic. \begin{proposition}[Motivic regularity of valuation rings] Let $V$ be a henselian valuation ring. If $V$ satisfies the condition $\emph{Val}(V)$ ({\it e.g.}, if $V$ is an extension of a perfectoid valuation ring), then for every integer $i \geq 0$, the natural map $$\Z(i)^{\emph{mot}}(V) \longrightarrow \Z(i)^{\mathbb{A}^1}(V)$$ is an equivalence in the derived category $\mathcal{D}(\Z)$. \end{proposition} \begin{proof} If the ring $V$ satisfies the condition $\text{Val}(V)$, the natural map $$\Z(i)^{\text{mot}}(V) \longrightarrow \Z(i)^{\mathbb{A}^1}(V)$$ is naturally identified with the natural map $$\Z(i)^{\text{mot}}(V) \longrightarrow \Z(i)^{\text{cdh}}(V)$$ in the derived category $\mathcal{D}(\Z)$ (Theorem~\ref{theoremBEMmotivicregularity}\,$(3)$). This latter map is an equivalence by Remark~\ref{remarkcomparisontocdhlocalmotiviccohomology}, and because henselian valuation rings are local for the cdh topology. \end{proof} \begin{proposition}[\cite{bachmann_A^1-invariant_2024}]\label{propositionBEMBeilinsonLichtenbaumcomparison} Let $p$ be a prime number, $S$ be a qcqs scheme of finite valuative dimension and satisfying the condition $\emph{Val}(S,p)$, and $X$ be a qcqs $S$-scheme. Then for any integers $i \geq 0$ and $k \geq 1$, the fibre of the cdh-local Beilinson--Lichtenbaum comparison map (Definition~\ref{definitioncdhlocalBeilinsonLichtenbaumcomparisonmap}) $$\Z/p^k(i)^{\emph{cdh}}(X) \longrightarrow R\Gamma_{\emph{ét}}(X[\tfrac{1}{p}],\mu_{p^k}^{\otimes i})$$ in the derived category $\mathcal{D}(\Z/p^k)$ is in degrees at least $i$. \end{proposition} \begin{proof} The presheaves $\Z/p^k(i)^{\text{cdh}}(-)$ and $R\Gamma_{\text{ét}}(-[\tfrac{1}{p}],\mu_{p^k}^{\otimes i})$ are finitary cdh sheaves on qcqs $S$\nobreakdash-schemes (\cite{bachmann_A^1-invariant_2024} and Theorem~\ref{theoremBMcdhdescentforétalecohomology}), so it suffices to prove the result on henselian valuation rings $V$ with a map $\text{Spec}(V) \rightarrow S$ (\cite[Corollary~$2.4.19$]{elmanto_cdh_2021}). If the henselian valuation ring $V$ is $p$-torsionfree, the condition $\text{Val}(S,p)$ and Corollary~\ref{corollaryFsmoothnessBeilinsonLichtenbaumcomparison} imply that the fibre of the Beilinson--Lichtenbaum comparison map $$\Z/p^k(i)^{\text{mot}}(V) \longrightarrow R\Gamma_{\text{ét}}(V[\tfrac{1}{p}],\mu_{p^k}^{\otimes i})$$ is in degrees at least $i+1$. If the henselian valuation ring $V$ is not $p$-torsionfree, then it is an $\F_p$-algebra, and it is $F$-smooth (Theorem~\ref{theoremknowncasesofvaluationringsbeingFsmooth}$\,(2)$). In particular, there is a natural equivalence $$\Z/p^k(i)^{\text{BMS}}(V) \xlongrightarrow{\sim} R\Gamma_{\text{ét}}(V,W_k\Omega^i_{\text{log}})[-i]$$ in the derived category $\mathcal{D}(\Z/p^k)$ (\cite[Proposition~$5.1\,(ii)$]{luders_milnor_2023}), and Theorem~\ref{theorempadicmotiviccohomologyintermsofsyntomicohomology} implies that the fibre of the Beilinson--Lichtenbaum comparison map is in degrees at least $i$. \end{proof} \begin{theorem}\label{theoremA1regularitypadicmain} Let $p$ be a prime number, $V$ be a valuation ring whose $p$-completion is perfectoid, and $X$ be a $p$-torsionfree $F$-smooth scheme over $\emph{Spec}(V)$. Then for any integers $i \geq 0$ and $k \geq 1$, the fibre of the natural map $$\Z/p^k(i)^{\emph{mot}}(X) \longrightarrow \Z/p^k(i)^{\mathbb{A}^1}(X)$$ in the derived category $\mathcal{D}(\Z/p^k)$ is in degrees at least $i+1$. \end{theorem} \begin{proof} By Theorem~\ref{theoremBEMmotivicregularity}\,$(3)$ and Example~\ref{examplesconditionVal}\,$(3)$, this is equivalent to the fact that the fibre of the left vertical map in the natural commutative diagram $$\begin{tikzcd} \Z/p^k(i)^{\text{mot}}(X) \ar[r] \ar[d] & R\Gamma_{\text{ét}}(X[\tfrac{1}{p}],\mu_{p^k}^{\otimes i}) \ar[d] \\ \Z/p^k(i)^{\text{cdh}}(X) \ar[r] & \big(L_{\text{cdh}} R\Gamma_{\text{ét}}(-[\tfrac{1}{p}],\mu_{p^k}^{\otimes i})\big)(X) \end{tikzcd}$$ is in degrees at least $i+1$, {\it i.e.}, that it is an isomorphism on cohomology groups in degrees less than or equal to~$i-1$, and injective in degree $i$. The right vertical map is an equivalence (Theorem~\ref{theoremBMcdhdescentforétalecohomology}). The fibre of the top horizontal map is in degrees at least $i+1$ (Corollary~\ref{corollaryFsmoothnessBeilinsonLichtenbaumcomparison}). The fibre of the bottom horizontal map is in degrees at least $i$ (Proposition~\ref{propositionBEMBeilinsonLichtenbaumcomparison} and Theorem~\ref{theoremknowncasesofvaluationringsbeingFsmooth}\,$(3)$). This implies the desired result, by the following elementary argument. On cohomology groups in degrees at most $i-2$, all but the left vertical map are isomorphisms, so the left vertical map is an isomorphism. In degree $i-1$, the top horizontal and right vertical maps are isomorphisms, so the left vertical map is injective; the bottom horizontal map is moreover injective, so the left vertical map is an isomorphism. In degree $i$, the top horizontal map is injective and the right vertical map is an isomorphism, so the left vertical map is injective. \end{proof} \begin{corollary}\label{corollaryA1invarianceinsmalldegreessmoothoverniceperfectoidvaluationring} Let $V$ be a valuation ring whose $p$-completion is perfectoid for every prime number~$p$,\footnote{Note that this condition is vacuously true for every prime number $p$ different from the residue characteristic of the valuation ring $V$.} and $X$ be an ind-smooth $V$-scheme. Then for every integer $i \geq 0$, the fibre of the natural map $$\Z(i)^{\emph{mot}}(X) \longrightarrow \Z(i)^{\mathbb{A}^1}(X)$$ in the derived category $\mathcal{D}(\Z)$ is in degrees at least $i+2$. \end{corollary} \begin{proof} By \cite[Corollary~$2.3$ and Proposition~$2.4.2$]{antieau_K-theory_2022}, the natural map $$\text{K}(X) \longrightarrow \text{KH}(X)$$ is an equivalence of spectra for every ind-smooth scheme over a general valuation ring $V$. By Theorem~$10.1$, this implies that the natural map $$\Q(i)^{\text{mot}}(X) \longrightarrow \Q(i)^{\mathbb{A}^1}(X)$$ is an equivalence in the derived category $\mathcal{D}(\Q)$. That is, the cohomology groups of the fibre of the natural map $$\Z(i)^{\text{mot}}(X) \longrightarrow \Z(i)^{\mathbb{A}^1}(X)$$ are torsion. In degrees less than or equal to $i+2$, and if the $p$-completion of the valuation ring $V$ is perfectoid for every prime number $p$, then these cohomology groups are also $p$-torsionfree for every prime number $p$ by Theorem~\ref{theoremA1regularitypadicmain}, and are thus zero. \end{proof} \newpage \section{Examples} \vspace{-\parindent} \hspace{\parindent} In this subsection, we revisit certain known results on algebraic $K$-theory in terms of the motivic complexes $\Z(i)^{\text{mot}}$. \subsection{Perfect and semiperfect rings} \vspace{-\parindent} \hspace{\parindent} Let $p$ be a prime number. It was proved by Kratzer \cite[Corollary~$5.5$]{kratzer_lambda_1980} that for every perfect $\F_p$-algebra $R$ and every integer $n \geq 1$, the $K$-group $\text{K}_n(R)$ is uniquely $p$-divisible (see also \cite{antieau_K-theory_2022} for a mixed characteristic generalisation). It was also proved by Kelly--Morrow that for every $\F_p$-algebra~$R$ with perfection $R_{\text{perf}}$, the natural map $\text{K}(R) \rightarrow \text{K}(R_{\text{perf}})$ is an equivalence after inverting $p$ (\cite[Lemma~$4.1$]{kelly_k-theory_2021}, see also \cite[Example~$2.1.11$]{elmanto_perfection_2020} and \cite[Theorem~$3.1.2$ and Proposition~$3.3.1$]{coulembier_K-theory_2023} for different proofs). The following result is a motivic refinement of these two facts. \begin{theorem}[Motivic cohomology of perfect $\F_p$-schemes, after \cite{elmanto_motivic_2023}]\label{theoremmotiviccohomologyofperfectschemes} Let $X$ be a qcqs $\F_p$-scheme. \begin{enumerate} \item For every integer $i \geq 0$, the natural map $$\Z(i)^{\emph{mot}}(X)[\tfrac{1}{p}] \longrightarrow \Z(i)^{\emph{mot}}(X_{\emph{perf}})[\tfrac{1}{p}]$$ is an equivalence in the derived category $\mathcal{D}(\Z[\tfrac{1}{p}])$. \item For every integer $i \geq 1$, the natural map $$\Z(i)^{\emph{mot}}(X_{\emph{perf}}) \longrightarrow \Z(i)^{\emph{mot}}(X_{\emph{perf}})[\tfrac{1}{p}]$$ is an equivalence in the derived category $\mathcal{D}(\Z)$. \end{enumerate} \end{theorem} \begin{proof} By \cite[Theorem~$4.24\,(5)$]{elmanto_motivic_2023},\footnote{This result is proved as a consequence of the same result in classical motivic cohomology \cite{geisser_k-theory_2000} and in syntomic cohomology \cite{antieau_beilinson_2020}, and ultimately goes back to the fact that the Frobenius acts by multiplication by $p^i$ on the logarithmic de Rham--Witt sheaf $W\Omega^i_{\text{log}}$.} for every integer $i \geq 0$, the natural map $$\phi_X^\ast : \Z(i)^{\text{mot}}(X) \longrightarrow \Z(i)^{\text{mot}}(X)$$ induced by the absolute Frobenius $\phi_X : X \rightarrow X$ of a qcqs $\F_p$-scheme $X$ is multiplication by $p^i$. In particular, this natural map is an equivalence after inverting $p$, and $(1)$ is a consequence of this and the fact that the presheaf $\Z(i)^{\text{mot}}$ is finitary (\cite[Theorem~$4.24\,(4)$]{elmanto_motivic_2023}). Similarly, the same result applied to the perfect $\F_p$-scheme $X_{\text{perf}}$ implies that multiplication by $p^i$ on the complex $\Z(i)^{\text{mot}}(X_{\text{perf}}) \in \mathcal{D}(\Z)$ is an equivalence. If $i \geq 1$, this is equivalent to the fact that the natural map $$\Z(i)^{\text{mot}}(X_{\text{perf}}) \longrightarrow \Z(i)^{\text{mot}}(X_{\text{perf}})[\tfrac{1}{p}]$$ is an equivalence in the derived category $\mathcal{D}(\Z)$. \end{proof} \begin{remark}[Negative $K$-groups of perfect $\F_p$-algebras] It is possible to construct examples of perfect $\F_p$-algebras whose negative $K$-groups are not $p$-divisible (\cite[Section~$3.3$]{coulembier_K-theory_2023}). Theorem~\ref{theoremmotiviccohomologyofperfectschemes}\,$(2)$ states that the only non-$p$-divisible information in the negative $K$-groups of a perfect $\F_p$-algebra $R$ actually come from weight zero motivic cohomology, {\it i.e.}, from the complex $R\Gamma_{\text{cdh}}(R,\Z)$ (Example~\ref{exampleweightzeromotiviccohomology}). \end{remark} Recall that a $\F_p$-algebra is {\it semiperfect} if its Frobenius is surjective. \begin{corollary}[Motivic cohomology of semiperfect $\F_p$-algebras] Let $S$ be a semiperfect $\F_p$-algebra. Then for every integer $i \geq 1$, the natural commutative diagram $$\begin{tikzcd} \Z(i)^{\emph{mot}}(S) \ar[r] \ar[d] & \Z_p(i)^{\emph{syn}}(S) \ar[d] \\ \Z(i)^{\emph{mot}}(S_{\emph{perf}}) \ar[r] & \Z_p(i)^{\emph{syn}}(S_{\emph{perf}}) \end{tikzcd}$$ is a cartesian square in the derived category $\mathcal{D}(\Z)$. \end{corollary} \begin{proof} It suffices to prove the result modulo $p$, and after inverting $p$. After inverting $p$, the vertical maps become equivalences by Theorem~\ref{theoremmotiviccohomologyofperfectschemes}\,$(1)$ (and the same argument for syntomic cohomology). We prove now the result modulo $p$. By Theorem~\ref{theoremmotiviccohomologyofperfectschemes}\,$(2)$ (and the same argument for syntomic cohomology), the bottom terms of the commutative diagram are zero modulo $p$, so it suffices to prove that the natural map $$\F_p(i)^{\text{mot}}(S) \longrightarrow \F_p(i)^{\text{syn}}(S)$$ is an equivalence in the derived category $\mathcal{D}(\F_p)$. By \cite[Corollary~$4.32$]{elmanto_motivic_2023} (see also Theorem~\ref{theorempadicmotiviccohomologyintermsofsyntomicohomology} for a mixed characteristic generalisation), this is equivalent to the fact that $$R\Gamma_{\text{cdh}}(S,\widetilde{\nu}(i))[-i-1] \simeq 0$$ in the derived category $\mathcal{D}(\F_p)$. By definition, the Frobenius map $\phi_S : S \rightarrow S$ is surjective, and has nilpotent kernel. The presheaf $R\Gamma_{\text{cdh}}(-,\widetilde{\nu}(i))[-i-1]$ is a finitary cdh sheaf, so the natural map $$R\Gamma_{\text{cdh}}(S,\widetilde{\nu}(i))[-i-1] \longrightarrow R\Gamma_{\text{cdh}}(S_{\text{perf}},\widetilde{\nu}(i))[-i-1]$$ is then an equivalence in the derived category $\mathcal{D}(\F_p)$. The target of this map is zero by Theorem~\ref{theoremmotiviccohomologyofperfectschemes}\,$(2)$ (where we use that $i \geq 1$, and the same argument for syntomic cohomology), and applying \cite[Corollary~$4.32$]{elmanto_motivic_2023} to the perfect $\F_p$-algebra $S_{\text{perf}}$. \end{proof} \subsection{Finite chain rings} \vspace{-\parindent} \hspace{\parindent} Finite chain rings are commutative rings $\mathcal{O}_K/\pi^n$, where $\mathcal{O}_K$ is a mixed characteristic discrete valuation ring with finite residue field, $\pi$ is a uniformizer of $\mathcal{O}_K$, and $n \geq 1$ is an integer. Examples of finite chain rings thus include finite fields, rings of the form $\Z/p^n$, and truncated polynomials over a finite field. \begin{lemma}\label{lemmafinitechainringsdecomposition} Let $\mathcal{O}_K$ be a discrete valuation ring of mixed characteristic $(0,p)$ and with finite residue field $\F_q$, $\pi$ be a uniformizer of $\mathcal{O}_K$, and $n \geq 1$ be an integer. Then for every integer $i \geq 0$, there is a natural equivalence $$\Z(i)^{\emph{mot}}(\mathcal{O}_K/\pi^n) \simeq \left\{ \begin{array}{ll} \Z[0] & \emph{if } i=0\\ \Z_p(i)^{\emph{BMS}}(\mathcal{O}_K/\pi^n) \oplus \Z(i)^{\emph{mot}}(\F_q)[\tfrac{1}{p}] & \emph{if } i \geq 1 \end{array} \right.$$ in the derived category $\mathcal{D}(\Z)$. \end{lemma} \begin{proof} The result for $i=0$ follows from the equivalences $$\Z(0)^{\text{mot}}(\mathcal{O}_K/\pi^n) \simeq R\Gamma_{\text{cdh}}(\mathcal{O}_K/\pi^n,\Z) \simeq R\Gamma_{\text{cdh}}(\F_q,\Z) \simeq \Z[0]$$ in the derived category $\mathcal{D}(\Z)$, the first equivalence being Example~\ref{exampleweightzeromotiviccohomology}, the second equivalence being nilpotent invariance of cdh sheaves, and the last equivalence being a consequence of the fact that fields are local for the cdh topology. For every integer $i \geq 0$, the commutative diagram $$\begin{tikzcd} \Z(i)^{\text{mot}}(\mathcal{O}_K/\pi^n) \ar[r] \ar[d] & \Z_p(i)^{\text{BMS}}(\mathcal{O}_K/\pi^n) \ar[d] \\ \Z(i)^{\text{mot}}(\F_q) \ar[r] & \Z_p(i)^{\text{BMS}}(\F_q) \end{tikzcd}$$ is a cartesian square in the derived category $\mathcal{D}(\Z)$ (Theorem~\ref{theoremfilteredDGMtheorem}). If $i \geq 1$, the bottom right term vanishes (use for instance the description of Bhatt--Morrow--Scholze's syntomic cohomology in characteristic $p$ in terms of logarithmic de Rham--Witt forms), and there is a natural equivalence $$\Z(i)^{\text{mot}}(\F_q) \xlongrightarrow{\sim} \Z(i)^{\text{mot}}(\F_q)[\tfrac{1}{p}]$$ in the derived category $\mathcal{D}(\Z)$ (by a classical result in motivic cohomology, see also Theorem~\ref{theoremmotiviccohomologyofperfectschemes}\,$(2)$ for a more general statement), hence the desired result. \end{proof} \begin{proposition} Let $\mathcal{O}_K$ be a mixed characteristic discrete valuation ring with finite residue field, $\pi$ be a uniformizer of~$\mathcal{O}_K$, and $n \geq 1$ be an integer. Then for every integer $m \in \Z$, there is a natural isomorphism $$\emph{K}_m(\mathcal{O}_K/\pi^n) \cong \left\{ \begin{array}{llll} \Z & \emph{if } m=0\\ \emph{H}^1_{\emph{mot}}(\mathcal{O}_K/\pi^n,\Z(i)) & \emph{if } m=2i-1, \emph{ } i \geq 1\\ \emph{H}^2_{\emph{mot}}(\mathcal{O}_K/\pi^n,\Z(i)) & \emph{if } m=2i-2, \emph{ } i \geq 2\\ 0 & \emph{if } m<0 \end{array} \right.$$ of abelian groups. \end{proposition} \begin{proof} Let $p$ be the residue characteristic of the discrete valuation ring $\mathcal{O}_K$. The result with $p$-adic coefficients is \cite[Corollary~$2.16$]{antieau_K-theory_2024}. The result with $\Z[\tfrac{1}{p}]$-coefficients reduces to the case $n=1$, where the result follows from the description of the (classical) motivic cohomology of finite fields. The integral result is then a consequence of Lemma~\ref{lemmafinitechainringsdecomposition}. \end{proof} \begin{theorem}[Motivic cohomology of finite chain rings, after \cite{antieau_K-theory_2024}] Let $\mathcal{O}_K$ be a discrete valuation ring of mixed characteristic $(0,p)$ and with finite residue field $\F_q$, $\pi$ be a uniformizer of $\mathcal{O}_K$, and $n \geq 1$ be an integer. Then for every integer $i \geq 4p^n$,\footnote{Note that this is not an optimal lower bound on the integer $i$. See \cite[Theorem~$1.4$]{antieau_K-theory_2024} for a more precise result, in terms of the ramification index of $\mathcal{O}_K$.} the motivic complex $$\Z(i)^{\emph{mot}}(\mathcal{O}_K/\pi^n) \in \mathcal{D}(\Z)$$ is concentrated in degree one, where it is given by a group of order $(q^i-1)q^{i(n-1)}$. \end{theorem} \begin{proof} This is a consequence of Lemma~\ref{lemmafinitechainringsdecomposition}, the classical computation of the motivic cohomology of~$\F_q$, and \cite[Theorem~$1.4$ and Proposition~$1.5$]{antieau_K-theory_2024}. \end{proof} \begin{remark}[Nilpotence of $v_1$] Antieau--Krause--Nikolaus also determine the nilpotence degree of the element $v_1$ in the mod $p$ syntomic cohomology of $\Z/p^n$ (\cite[Theorem~$1.8$]{antieau_K-theory_2024}). This is a refinement of the key result in the study of $K(1)$-local $K$-theory of Bhatt--Clausen--Mathew \cite{bhatt_remarks_2020}. Note that this result on the nilpotence degree of $v_1$ can be reformulated, via Lemma~\ref{lemmafinitechainringsdecomposition}, as a statement on the mod $p$ motivic cohomology of $\Z/p^n$. \end{remark} \subsection{Valuation rings} \vspace{-\parindent} \hspace{\parindent} Recall that a valuation ring is an integral domain $V$ such that for any elements $f$ and $g$ in $V$, either $f \in gV$ or $g \in fV$. In recent years, valuation rings have been used as a way to bypass resolution of singularities, in order to adapt arguments from characteristic zero to more general contexts \cite{kerz_towards_2021,kelly_k-theory_2021,bouis_cartier_2023,bachmann_A^1-invariant_2024}. In this subsection, we describe the motivic cohomology of valuation rings (Theorems~\ref{theoremvaluationringslisse=mot} and~\ref{theoremvaluationringsmotiviccohomologyfinitecoefficients}). We start with the following result, stating that the motivic complexes~$\Z(i)^{\text{mot}}$, on henselian valuation rings, have a description purely in terms of algebraic cycles. See \cite[Section~9]{elmanto_motivic_2023} for related results over a field. \begin{theorem}\label{theoremvaluationringslisse=mot} Let $V$ be a henselian valuation ring. Then for every integer $i \geq 0$, the motivic complex $\Z(i)^{\emph{mot}}(V) \in \mathcal{D}(\Z)$ is in degrees at most $i$, and the lisse-motivic comparison map (Definition~\ref{definitionlissemotiviccomparisonmap}) $$\Z(i)^{\emph{lisse}}(V) \longrightarrow \Z(i)^{\emph{mot}}(V)$$ is an equivalence in the derived category $\mathcal{D}(\Z)$. \end{theorem} \begin{proof} The second statement already appears in the proof of Lemma~\ref{lemmamotiviccohomologyofhenselianvaluationringisindegreesatmosti}. As in Lemma~\ref{lemmamotiviccohomologyofhenselianvaluationringisindegreesatmosti} or Corollary~\ref{corollarylissemotivicmaincomparisontheorem}, the first statement is then a consequence of \cite[Corollary~$4.4$]{geisser_motivic_2004}. \end{proof} \begin{example} Let $V$ be a henselian valuation ring. By Example~\ref{exampleweightzeromotiviccohomology}, there is a natural equivalence $$\Z(0)^{\text{mot}}(V) \simeq \Z[0]$$ in the derived category $\mathcal{D}(\Z)$. Similarly, Theorem~\ref{theoremvaluationringslisse=mot}, Example~\ref{exampleweight1lissemotiviccohomology}, and the fact that the Picard group of a local ring is zero, imply that the motivic complex $\Z(1)^{\text{mot}}(V) \in \mathcal{D}(\Z)$ is concentrated in degree one, where it is given by $$\text{H}^1_{\text{mot}}(V,\Z(1)) \cong V^{\times}.$$ \end{example} We now apply the results of the previous sections to give an alternative description of the motivic cohomology of valuation rings with finite coefficients. The following proposition will be used to reformulate the results of \cite{bouis_cartier_2023} on syntomic cohomology in terms of motivic cohomology. \begin{proposition}\label{propositionmotiviccohomologyvaluationringsmodpisindegreesatmosti} Let $p$ be a prime number, and $V$ be a henselian valuation ring. Then for any integers $i \geq 0$ and $k \geq 1$, there is a natural equivalence $$\Z/p^k(i)^{\emph{mot}}(V) \xlongrightarrow{\sim} \tau^{\leq i} \Z/p^k(i)^{\emph{syn}}(V)$$ in the derived category $\mathcal{D}(\Z/p^k)$. \end{proposition} \begin{proof} Henselian valuation rings are local rings for the cdh topology, so this is a consequence of Theorem~\ref{theorempadicmotiviccohomologyintermsofsyntomicohomology}. \end{proof} The following result is an analogue for valuation rings of Geisser--Levine's description of motivic cohomology of smooth $\F_p$-algebras \cite{geisser_k-theory_2000}. It can be deduced from the results of Kelly--Morrow \cite{kelly_k-theory_2021} and Elmanto--Morrow \cite{elmanto_motivic_2023}. \begin{theorem}\label{theoremmotiviccohomologyofcharpvaluationrings} Let $p$ be a prime number, and $V$ be a henselian valuation ring of characteristic $p$. Then for any integers $i \geq 0$ and $k \geq 1$, there is a natural equivalence $$\Z/p^k(i)^{\emph{mot}}(V) \xlongrightarrow{\sim} W_k\Omega^i_{V,\emph{log}}[-i]$$ in the derived category $\mathcal{D}(\Z/p^k)$. \end{theorem} \begin{proof} Valuation rings of characteristic $p$ are Cartier smooth over $\F_p$ by results of Gabber--Ramero and Gabber (\cite[Theorem~$3.4$]{bouis_cartier_2023}), so this is a consequence of \cite[Proposition~$5.1\,(ii)$]{luders_milnor_2023} and Proposition~\ref{propositionmotiviccohomologyvaluationringsmodpisindegreesatmosti}. \end{proof} We then prove a mixed characteristic version of Theorem~\ref{theoremmotiviccohomologyofcharpvaluationrings}, starting with the following $\l$-adic general result. \begin{proposition}\label{propositionladicmotiviccohomologyofvaluationrings} Let $p$ be a prime number, and $V$ be a henselian valuation ring such that $p$ is invertible in $V$. Then for any integers $i \geq 0$ and $k \geq 1$, the Beilinson--Lichtenbaum comparison map (Definition~\ref{definitionBeilinsonLichtenbaumcomparisonmap}) naturally factors through an equivalence $$\Z/p^k(i)^{\emph{mot}}(V) \xlongrightarrow{\sim} \tau^{\leq i} R\Gamma_{\emph{ét}}(\emph{Spec}(V),\mu_{p^k}^{\otimes i})$$ in the derived category $\mathcal{D}(\Z/p^k)$. \end{proposition} \begin{proof} By Proposition~\ref{propositionmotiviccohomologyvaluationringsmodpisindegreesatmosti}, the motivic complex $\Z/p^k(i)^{\text{mot}}(V) \in \mathcal{D}(\Z/p^k)$ is in degrees at most $i$, so the result is a consequence of Corollary~\ref{corollaryladicmotivcohomologylowdegreesisétalecohomology}. \end{proof} The following result generalises Proposition~\ref{propositionladicmotiviccohomologyofvaluationrings} when $p$ is not necessarily invertible in the valuation ring $V$, at least over a perfectoid base. \begin{theorem}[Motivic cohomology of valuation rings with finite coefficients]\label{theoremvaluationringsmotiviccohomologyfinitecoefficients} Let $p$ be a prime number, $V_0$ be a $p$-torsionfree valuation ring whose $p$\nobreakdash-completion is a perfectoid ring, and $V$ be a henselian valuation ring extension of $V_0$. Then for any integers $i \geq 0$ and $k \geq 1$, the Beilinson--Lichtenbaum comparison map (Definition~\ref{definitionBeilinsonLichtenbaumcomparisonmap}) induces a natural map $$\Z/p^k(i)^{\emph{mot}}(V) \longrightarrow \tau^{\leq i} R\Gamma_{\emph{ét}}(\emph{Spec}(V[\tfrac{1}{p}]),\mu_{p^k}^{\otimes i})$$ in the derived category $\mathcal{D}(\Z/p^k)$, which is an isomorphism in degrees less than or equal to $i-1$. On~$\emph{H}^i$, this map is injective, with image generated by symbols, via the symbol map $$(V^\times)^{\otimes i} \rightarrow \emph{H}^i_{\emph{ét}}(\emph{Spec}(V[\tfrac{1}{p}]),\mu_{p^k}^{\otimes i}).$$ \end{theorem} \begin{proof} The fact that the Beilinson--Lichtenbaum comparison map factors through the complex $$\tau^{\leq i} R\Gamma_{\text{ét}}(\text{Spec}(V[\tfrac{1}{p}]),\mu_{p^k}^{\otimes i}) \in \mathcal{D}(\Z/p^k)$$ is a consequence of Proposition~\ref{propositionmotiviccohomologyvaluationringsmodpisindegreesatmosti}. The isomorphism in degrees less than or equal to $i-1$ and the injectivity in degree $i$ of this map are then a consequence of \cite[Theorems~$3.1$ and~$4.12$]{bouis_cartier_2023}. The last statement is a consequence of the isomorphism $$\widehat{\text{K}}{}^{\text{M}}_i(V)/p^k \xlongrightarrow{\cong} \text{H}^i_{\text{mot}}(V,\Z/p^k(i))$$ of abelian groups (Theorem~\ref{theoremcomparisontoMilnorKtheory} and Corollary~\ref{corollaryHilbert90}). \end{proof} \begin{remark} The generation by symbols appearing in Theorem~\ref{theoremvaluationringsmotiviccohomologyfinitecoefficients} was also studied in the context of syntomic cohomology of general $p$-torsionfree $F$-smooth schemes by Bhatt--Mathew \cite{bhatt_syntomic_2023}. Note that all valuation rings are conjecturally $F$-smooth (see Conjecture~\ref{conjecturevaluationringsareFsmooth}), and that the proof of Theorem~\ref{theoremvaluationringsmotiviccohomologyfinitecoefficients} adapts more generally to any henselian $F$-smooth valuation ring. \end{remark} \subsection{\texorpdfstring{$\C^{\star}$}{TEXT}-algebras} \vspace{-\parindent} \hspace{\parindent} By Gelfand representation theorem, the commutative $\C^\star$-algebras are exactly the algebras of continuous complex-valued functions $\mathscr{C}(X;\C)$ on a compact Hausdorff space $X$. An important theorem of Corti\~{n}as--Thom states that commutative $\C^\star$-algebras are $K$-regular (\cite[Theorem~$1.5$]{cortinas_algebraic_2012}). This result was further generalised recently by Aoki to all smooth algebras over commutative $\C^\star$-algebras, and over a general local field (\cite[Theorem~$8.7$]{aoki_K-theory_2024}). The following result is a motivic analogue of the latter result. \begin{theorem}[$\C^\star$-algebras are motivically regular, after \cite{cortinas_algebraic_2012,aoki_K-theory_2024}] Let $X$ be a compact Hausdorff space, $F$ be a characteristic zero local field, and $A$ be a smooth $\mathscr{C}(X;F)$\nobreakdash-algebra. Then for any integers $i \geq 0$ and $n \geq 0$, the natural map $$\Z(i)^{\emph{mot}}(A) \longrightarrow \Z(i)^{\emph{mot}}(A[T_1,\dots,T_n])$$ is an equivalence in the derived category $\mathcal{D}(\Z)$. \end{theorem} \begin{proof} By \cite[Theorem~$8.7\,(2)$]{aoki_K-theory_2024}, the natural map $$\text{K}(A[T_1,\dots,T_n]) \longrightarrow \text{KH}(A[T_1,\dots,T_n])$$ is an equivalence of spectra for every integer $n \geq 0$. By Corollary~\ref{corollarymotivicregularityinchar0}, this implies that the vertical maps in the commutative diagram $$\begin{tikzcd} \Z(i)^{\text{mot}}(A) \ar[r] \ar[d] & \Z(i)^{\text{mot}}(A[T_1,\dots,T_n]) \ar[d] \\ \Z(i)^{\mathbb{A}^1}(A) \ar[r] & \Z(i)^{\mathbb{A}^1}(A[T_1,\dots,T_n]) \end{tikzcd}$$ are equivalences in the derived category $\mathcal{D}(\Z)$. The bottom horizontal map is an equivalence in the derived category $\mathcal{D}(\Z)$ by definition of the presheaf $\Z(i)^{\mathbb{A}^1}$ (\cite{bachmann_A^1-invariant_2024}, see also Section~\ref{sectionA1invariantmotiviccohomology}). So the top horizontal map is an equivalence in the derived category $\mathcal{D}(\Z)$. \end{proof} \subsection{Truncated polynomials} \vspace{-\parindent} \hspace{\parindent} In this subsection, we study the motivic cohomology of truncated polynomials, {\it i.e.}, the motivic cohomology of commutative rings of the form $R[x]/(x^e)$. Given a $\mathcal{D}(\Z)$-valued functor $F(-)$, a commutative ring $R$, and an integer $e \geq 1$, we use the notation $$F(R[x]/(x^e),(x)) := \text{fib}(F(R[x]/(x^e)) \longrightarrow F(R)),$$ where the map is induced by the canonical projection $R[x]/(x^e) \rightarrow R$. The relative $K$-theory $\text{K}(k[x]/(x^e),(x))$ of truncated polynomials over a perfect field $k$ of positive characteristic was computed by Hesselholt--Madsen \cite{hesselholt_K-theory_1997,hesselholt_cyclic_1997}, using topological restriction homology. Their calculation was reproved by Speirs \cite{speirs_K-theory_2020} using Nikolaus--Scholze's approach to topological cyclic homology \cite{nikolaus_topological_2018}, and by Mathew \cite{mathew_recent_2022} and Sulyma \cite{sulyma_floor_2023} using Bhatt--Morrow--Scholze's filtration on topological cyclic homology \cite{bhatt_topological_2019}. This last approach was then extended to mixed characteristic by Riggenbach \cite{riggenbach_K-theory_2022}. More precisely, Riggenbach used computations in prismatic cohomology to extend the previous result to a computation of the $p$-adic relative $K$-theory $\text{K}(R[x]/(x^e),(x);\Z_p)$ of perfectoid rings $R$, and also reproved the $p$-adic part of the known description of $\text{K}(\Z[x]/(x^e),(x))$, originally due to Angeltveit--Gerhardt--Hesselholt \cite{angeltveit_K-theory_2009}. This recent progress would seem to indicate that $K$-theory calculations using equivariant stable homotopy may be pushed further by using cohomological techniques. Note however that the calculations in \cite{mathew_recent_2022,sulyma_floor_2023,riggenbach_K-theory_2022} are purely $p$-adic ones, as they rely on (instances of) prismatic cohomology. In fact, all of the previous integral calculations in mixed characteristic ({\it i.e.}, for $R$ the ring of integers of a number field) rely on a rational result of Soulé \cite{soule_rational_1980} and Staffeldt \cite{staffeldt_rational_1985}, who compute the ranks of the associated relative $K$-groups using equivariant homotopy theory. In this subsection, we revisit and extend this rational computation, and discuss some natural motivic refinements of the previous results. All of the above calculations use trace methods, via the Dundas--Goodwillie--McCarthy theorem. We first state the corresponding results at the level of cohomology theories. \begin{lemma}\label{lemmatruncatedpolynomialmotTC} Let $R$ be a commutative ring, and $e \geq 1$ be an integer. Then for every integer~$i \geq 0$, the natural map $$\Z(i)^{\emph{mot}}(R[x]/(x^e),(x)) \longrightarrow \Z(i)^{\emph{TC}}(R[x]/(x^e),(x))$$ is an equivalence in the derived category $\mathcal{D}(\Z)$. \end{lemma} \begin{proof} This is a direct consequence of Remark~\ref{remarkmaincartesiansquareformotiviccohomology}, and the fact that cdh sheaves are invariant under nilpotent extensions. \end{proof} \begin{corollary}\label{corollarytruncatedpolynomialpadicBMS} Let $R$ be a commutative ring, $e \geq 1$ be an integer, and $p$ be a prime number. Then for every integer $i \geq 0$, the natural map $$\Z_p(i)^{\emph{mot}}(R[x]/(x^e),(x)) \longrightarrow \Z_p(i)^{\emph{BMS}}(R[x]/(x^e),(x))$$ is an equivalence in the derived category $\mathcal{D}(\Z_p)$. \end{corollary} \begin{proof} This is a consequence of Lemma~\ref{lemmatruncatedpolynomialmotTC}. \end{proof} \begin{corollary}\label{corollarytruncatedpolynomialsmotdeRhamrational} Let $R$ be a commutative ring, and $e \geq 1$ be an integer. Then for every integer $i \geq 0$, there is a natural equivalence $$\Q(i)^{\emph{mot}}(R[x]/(x^e),(x)) \simeq \mathbb{L}\Omega^{<i}_{(R[x]/(x^e),(x))_{\Q}/\Q}[-1]$$ in the derived category $\mathcal{D}(\Q)$. \end{corollary} \begin{proof} This is a consequence of Lemma~\ref{lemmatruncatedpolynomialmotTC} and cdh descent for the presheaf $\widehat{\mathbb{L}\Omega}_{-/\Q}$ on commutative $\Q$-algebras (\cite[Lemma~$4.5$]{elmanto_motivic_2023}). \end{proof} \begin{lemma} For every commutative ring $R$ and integer $e \geq 1$, the object $$\Z(0)^{\emph{mot}}\big(R[x]/(x^e),(x)\big)$$ is zero in the derived category $\mathcal{D}(\Z)$. \end{lemma} \begin{proof} This is a consequence of the fact that the motivic complex $\Z(0)^{\text{mot}}$ is a cdh sheaf (Example~\ref{exampleweightzeromotiviccohomology}). \end{proof} \begin{lemma}\label{lemmatruncatedpolynomiale-1forQ} For any integers $e \geq 1$ and $i \geq 0$, the complex $$\mathbb{L}\Omega^{\leq i}_{(\Q[x]/(x^e),(x))/\Q} \in \mathcal{D}(\Q)$$ is concentrated in degree zero, given by a $\Q$-vector space of dimension $e-1$. \end{lemma} \begin{proof} This follows from a standard argument using the natural grading of the $\Q$-algebra $\Q[x]/(x^e)$ and the $\Q$-linear derivation $d : \Q[x]/(x^e) \rightarrow \Q[x]/(x^e)$ given by $d(x^j) = jx^j$; see for instance the proof of \cite[Proposition~$5$]{staffeldt_rational_1985}. \end{proof} \begin{theorem}\label{theoremtruncatedpolynomialmain} Let $R$ be a commutative ring such that the cotangent complex $\mathbb{L}_{(R\otimes_{\Z} \Q)/\Q}$ vanishes ({\it e.g.}, if $R \otimes_{\Z} \Q$ is ind-étale over $\Q$),\footnote{See \cite{mondal_ind-etale_2022} for more on this condition.} and $e \geq 1$ be an integer. Then for every integer~$i \geq 1$, there is a natural equivalence $$\Q(i)^{\emph{mot}}(R[x]/(x^e),(x)) \simeq (R \otimes_{\Z} \Q)^{e-1}[-1]$$ in the derived category $\mathcal{D}(\Q)$. \end{theorem} \begin{proof} By Corollary~\ref{corollarytruncatedpolynomialsmotdeRhamrational}, there is a natural equivalence $$\Q(i)^{\text{mot}}(R[x]/(x^e),(x)) \simeq \mathbb{L}\Omega^{<i}_{(R[x]/(x^e),(x))_{\Q}/\Q}[-1]$$ in the derived category $\mathcal{D}(\Q)$. By the Künneth formula for derived de Rham cohomology, and because all the positive powers of the cotangent complex $\mathbb{L}_{(R\otimes_{\Z} \Q)/\Q}$ vanish, there is a natural equivalence $$\mathbb{L}\Omega^{<i}_{(R[x]/(x^e),(x))_{\Q}/\Q} \simeq \mathbb{L}\Omega^{<i}_{(\Q[x]/(x^e),(x))/\Q} \otimes_{\Q} R$$ in the derived category $\mathcal{D}(\Q)$. The result is then a consequence of Lemma~\ref{lemmatruncatedpolynomiale-1forQ}. \end{proof} When $R$ is the ring of integers of a number field, the following result is due to Soulé \cite{soule_rational_1980} when $e=2$, and to Staffeldt \cite{staffeldt_rational_1985} for $e \geq 2$ a general integer. Their proof uses rational homotopy theory, and ultimately reduces to a computation in cyclic homology. \begin{corollary}\label{corollarytruncatedpolynomialrationalKtheory} Let $R$ be a commutative ring such that the cotangent complex $\mathbb{L}_{(R\otimes_{\Z} \Q)/\Q}$ vanishes, and $e \geq 1$ be an integer. Then for every integer $n \in \Z$, there is a natural isomorphism $$\emph{K}_n(R[x]/(x^e),(x);\Q) \cong \left\{ \begin{array}{ll} (R \otimes_{\Z} \Q)^{e-1} & \emph{if } n \emph{ is odd and } n \geq 1 \\ 0 & \emph{otherwise} \end{array} \right.$$ of abelian groups. \end{corollary} \begin{proof} This is a consequence of Theorem~\ref{theoremtruncatedpolynomialmain} and Corollary~\ref{corollaryKtheorysplitsrationally}. \end{proof} \begin{remark} Let $K$ be a number field, $\mathcal{O}_K$ be its ring of integers, and $e \geq 1$ be an integer. The orders in the torsion part of the relative $K$-theory $\text{K}(\mathcal{O}_K[x]/(x^e),(x))$ were completely determined in \cite[Remark~$1.8$]{riggenbach_K-theory_2022}. It would be interesting to use this result and Theorem~\ref{theoremtruncatedpolynomialmain} to obtain an integral description of the relative motivic complexes $\Z(i)^{\text{mot}}(\mathcal{O}_K[x]/(x^e),(x))$ for all $i \geq 0$. This would in particular reprove and generalise the result for $K=\Q$ of Angeltveit--Gerhardt--Hesselholt \cite{angeltveit_K-theory_2009}. \end{remark} We also deduce from the work of Riggenbach the following motivic interpretation of the analogous result in $K$-theory (\cite[Theorem~$1.1$]{riggenbach_K-theory_2022}). \begin{theorem}[Truncated polynomials over perfectoids, after \cite{riggenbach_K-theory_2022}] Let $R$ be a perfectoid ring, and $e \geq 1$ be an integer. Then for every integer $i \geq 1$, there is a natural equivalence $$\Z_p(i)^{\emph{mot}}(R[x]/(x^e),(x)) \simeq \mathbb{W}_{ei}(R)/V_e\mathbb{W}_i(R)[-1]$$ in the derived category $\mathcal{D}(\Z_p)$, where $\mathbb{W}(R)$ denotes the big Witt vectors of $R$, and $V$ the associated Verschiebung operator. \end{theorem} \begin{proof} This is a consequence of \cite[proof of Corollary~$6.5$]{riggenbach_K-theory_2022} and Corollary~\ref{corollarytruncatedpolynomialpadicBMS}. \end{proof} \begin{remark}[Cuspidal curves] The algebraic $K$-theory of cuspidal curves ({\it i.e.}, curves that are defined by an equation of the form $y^a-x^b$, for $a,b\geq 2$ coprime integers) was completely determined over a perfect $\F_p$-algebra by Hesselholt--Nikolaus \cite{hesselholt_algebraic_2020}, using Nikolaus--Scholze's approach \cite{nikolaus_topological_2018} to topological cyclic homology. This result was then generalised to mixed characteristic perfectoid rings by Riggenbach \cite{riggenbach_K-theory_2023}, ultimately relying on computations in relative topological Hochschild homology. It would seem that the associated Atiyah--Hirzebruch spectral sequence should degenerate in this context, thus providing a similar computation of the motivic cohomology of cuspidal curves. An interesting question would be whether these results can be reproved, or even extended to more general base rings, using techniques from prismatic cohomology and derived de Rham cohomology. \end{remark} \newpage \bibliographystyle{alpha} \bibliography{motivic.bib} \end{document}
2412.06595v2
http://arxiv.org/abs/2412.06595v2
Totally nonnegative matrices, chain enumeration and zeros of polynomials
\documentclass[reqno]{amsart} \usepackage{amsmath,amscd,amsfonts,amsthm,amssymb,latexsym,mathrsfs,mathtools,caption} \usepackage{tikz,hyperref,cleveref} \usetikzlibrary{matrix,arrows,patterns,decorations.markings} \usepackage{tikz,float,comment} \usepackage{shuffle} \numberwithin{equation}{section} \usepackage[T1]{fontenc} \usepackage{lmodern} \usepackage{textcomp} \usepackage{hyperref} \hypersetup{ bookmarks=true, unicode=false, pdftoolbar=true, pdfmenubar=true, pdffitwindow=false, pdfstartview={FitH}, pdftitle={}, pdfauthor={Author}, pdfcreator={Creator}, pdfproducer={Producer}, pdfkeywords={keyword1} {key2} {key3}, pdfnewwindow=true, colorlinks=true, linkcolor=blue, citecolor=black, filecolor=magenta, urlcolor=black } \theoremstyle{plain} \newtheorem{theorem}{Theorem}[section] \newtheorem{proposition}[theorem]{Proposition} \newtheorem{lemma}[theorem]{Lemma} \newtheorem{corollary}[theorem]{Corollary} \newtheorem{problem}[theorem]{Problem} \newtheorem{conjecture}[theorem]{Conjecture} \newtheorem{question}[theorem]{Question} \theoremstyle{definition} \newtheorem{definition}[theorem]{Definition} \newtheorem{example}[theorem]{Example} \newtheorem{exercise}[theorem]{Exercise} \newtheorem{notation}[theorem]{Notation} \theoremstyle{remark} \newtheorem{remark}[theorem]{Remark} \newcommand{\NN}{\mathbb{N}} \newcommand{\zz}{\mathbf{z}} \newcommand{\ww}{\mathbf{w}} \newcommand{\xx}{\mathbf{x}} \newcommand{\yy}{\mathbf{y}} \newcommand{\uu}{\mathbf{u}} \newcommand{\vv}{\mathbf{v}} \newcommand{\ee}{\mathbf{e}} \renewcommand{\aa}{\mathbf{a}} \newcommand{\one}{\hat{1}} \newcommand{\zero}{\hat{0}} \newcommand{\LL}{\mathscr{L}} \newcommand{\SSS}{\mathscr{S}} \newcommand{\MM}{\mathrm{M}} \newcommand{\PO}{\mathbb{P}} \newcommand{\FQ}{\text{FQSym}} \newcommand{\HH}{\mathcal{H}} \newcommand{\BB}{\mathcal{B}} \newcommand{\VV}{\mathcal{V}} \newcommand{\A}{\mathcal{A}} \newcommand{\EE}{\mathcal{E}} \newcommand{\FF}{\mathcal{F}} \newcommand{\FFF}{\mathbb{F}} \newcommand{\PP}{\mathcal{P}} \newcommand{\TN}{\mathrm{TN}} \newcommand{\GG}{\mathcal{G}} \newcommand{\HHH}{\overline{\mathcal{H}}} \newcommand{\ZZ}{\mathbb{Z}} \newcommand{\QQ}{\mathbb{Q}} \newcommand{\RR}{\mathbb{R}} \newcommand{\CC}{\mathbb{C}} \newcommand{\RRR}{\mathcal{R}} \newcommand{\CCC}{\mathcal{C}} \newcommand{\PR}{\mathbb{R}\{t\}} \newcommand{\PC}{\mathbb{C}\{t\}} \newcommand{\SNN}{\text{SNN}} \newcommand{\CM}{\mathbb{C}_{ {MA}}} \newcommand{\CMS}{\mathbb{C}_{ {MAS}}} \newcommand{\sym}{\mathfrak{S}} \newcommand{\ps}{\text{ps}_q^1} \renewcommand{\mod}{\mathop{\rm \ mod}} \renewcommand{\Im}{{\rm Im}} \renewcommand{\Re}{{\rm Re}} \newcommand{\FQS}{\text{FQSym}} \newcommand{\PF}{\mathrm{PF}} \newcommand{\reff}[2]{\ref*{#1}(\ref*{#2})} \def\newop#1{\expandafter\def\csname #1\endcsname{\mathop{\rm #1}\nolimits}} \def\shuff#1#2{\mathbin{ \hbox{\vbox{ \hbox{\vrule \hskip#2 \vrule height#1 width 0pt }\hrule}\vbox{ \hbox{\vrule \hskip#2 \vrule height#1 width 0pt \vrule }\hrule}}}} \newop{diag} \newop{AB} \newop{DB} \newop{B} \newop{AT} \newop{DT} \newop{peak} \newop{rank} \newop{des} \newop{Sym} \newop{inv} \newop{Orb} \newop{Hom} \newop{Re} \author[P.~Br\"and\'en]{Petter Br\"and\'en} \address{Department of Mathematics, KTH Royal Institute of Technology, SE-100 44 Stockholm, Sweden} \email{[email protected], [email protected]} \author[L.~Saud]{Leonardo Saud Maia Leite} \title[TN-matrices, chain enumeration and zeros of polynomials]{Totally nonnegative matrices, chain enumeration and zeros of polynomials} \begin{document} \definecolor{ududff}{rgb}{0.30196078431372547,0.30196078431372547,1.} \definecolor{xdxdff}{rgb}{0.49019607843137253,0.49019607843137253,1.} \definecolor{red}{rgb}{1,0,0} \definecolor{ffqqff}{rgb}{1,0,1} \definecolor{zzttqq}{rgb}{0.6,0.2,0} \definecolor{ududff}{rgb}{0.30196078431372547,0.30196078431372547,1} \definecolor{cqcqcq}{rgb}{0.7529411764705882,0.7529411764705882,0.7529411764705882} \definecolor{black}{rgb}{0,0,0} \begin{abstract} We prove that any lower triangular and totally nonnegative matrix whose diagonal entries are all equal to one gives rise to a family of polynomials with only real zeros. This has consequences to problems in several areas of mathematics. It is used to develop a general theory for chain enumeration in posets and zeros chain polynomials. The results obtained extend and unify results of the first author, Brenti, Welker and Athanasiadis. In the process we define a notion of $h$-vectors for a large class of posets which generalize the notions of $h$-vectors associated to simplicial and cubical complexes. A consequence of our methods is a characterization of the convex hull of all characteristic polynomials of hyperplane arrangements of fixed dimension and over a fixed finite field. This may be seen as a refinement of the critical problem of Crapo and Rota. We also use the methods developed to answer an open problem posed by Forg\'acs and Tran on the real-rootedness of polynomials arising from certain bivariate rational functions. \end{abstract} \subjclass[2020]{06A07, 05E45, 15B48, 26C10} \keywords{Totally nonnegative matrix, real-rooted polynomial, P\'olya frequency sequence, chain polynomial, barycentric subdivision, shellability, $q$-matroid, $r$-cubical poset, binomial poset, upper homogeneous poset, the critical problem} \thanks{PB is a Wallenberg Academy Scholar supported by the Knut and Alice Wallenberg Foundation, and the G\"oran Gustafsson foundation.} \maketitle \thispagestyle{empty} \tableofcontents \section{Introduction} Chain enumeration in partially ordered sets (posets) is a central topic in enumerative and algebraic combinatorics \cite[Chapter 3]{stanley2011enumerative}, and the zeros of polynomials associated to chain enumeration has been studied frequently in the literature. Particular attention has been given to the problem of determining if chain polynomials of posets are real-rooted for various classes of posets. For distributive lattices, this problem is equivalent to the poset conjecture (Neggers-Stanley conjecture) for natural labelings, which was stated by Neggers \cite{neggers1978representations} in the seventies and disproved by Stembridge in \cite{stembridge2007counterexamples}. Brenti and Welker \cite{brenti2008f} proved that the chain polynomials of the face lattices of simplicial polytopes are real-rooted, and conjectured that the same is true for any polytope. Athanasiadis \cite{athanasiadis2021face} proved Brenti and Welker's conjecture for cubical polytopes. More recently, Athanasiadis, Kalampogia-Evangelinou and Douvropoulos \cite{athanasiadis2023two,athanasiadis2022chain} proved that the chain polynomials of subspace lattices, partition lattices of types $A$ and $B$, the lattices of flats of near-pencils and uniform matroids, rank-selected subposets of Cohen-Macaulay simplicial posets and noncrossing partition lattices associated to finite Coxeter groups are real-rooted. We approach the problem of real-rooted chain polynomials through total positivity. We prove a general theorem, Theorem~\ref{maint}, stating that any lower triangular totally nonnegative matrix whose diagonal entries are all equal to one gives rise to a family real-rooted polynomials. For the special case when the matrix is a lower triangular Toeplitz matrix, Theorem~\ref{maint} implies that any P\'olya frequency sequence gives rise to an infinite family of real-rooted polynomials, see Theorem~\ref{PFT}. This is used to answer an open problem posed by Forg\'acs and Tran \cite[Problem 13]{forgacs2016polynomials} on the real-rootedness of polynomials arising from certain bivariate rational functions, see Theorem~\ref{FTT}. We also apply Theorem~\ref{PFT} to chain enumeration in upper homogeneous (upho) posets, a class of posets recently introduced by Stanley \cite{stanley2020upho, stanley2024theorems}. We prove that a family of chain polynomials associated to any Schur-positive upho poset \cite{fu2024monoid} are real-rooted, Theorem~\ref{uphot}. In Section \ref{sec-tn} we identify a class of posets that we call $\mathrm{TN}$-posets, to which Theorem~\ref{maint} applies to prove that such posets have real-rooted chain polynomials. The class of $\mathrm{TN}$-posets is closed under rank selection. Hence we get for free that the chain polynomials associated to a rank selected sub-poset of a $\mathrm{TN}$-poset are real-rooted. Examples of $\mathrm{TN}$-posets are Boolean algebras, lattices of subspaces of a finite dimensional vector space, $r$-cubical lattices and dual partition lattices. Given a $\mathrm{TN}$-poset $P$ we introduce, in Section \ref{section-subdivision}, a notion of $h$-vectors relative to $P$. For Boolean complexes this notion reduces to the usual definition of $h$-vectors, and for cubical complexes our definition reduces to Adin's cubical $h$-vector \cite{adin1996new}. We prove in Theorem~\ref{rpossel} that if the rank generating polynomial of a $P$-poset $Q$ (for example an order ideal of $P$) has nonnegative $h$-vector, then the chain polynomial of $Q$ is real-rooted. This is a vast generalization of a theorem of Brenti and Welker \cite{brenti2008f} who proved that if a Boolean cell complex $\Delta$ has nonnegative $h$-vector, then the $f$-polynomial of the barycentric subdivision of $\Delta$ is real-rooted. Equivalently, the chain polynomial of $\Delta$ is real-rooted. In Section \ref{sec-critical} we apply the theory developed in previous sections to $q$-posets, a $q$-analog of simplicial complexes first considered by Rota \cite{Rota71}, and more extensively studied by Alder \cite{alder2010q}. These are posets for which every principal order ideal is isomorphic to the lattice of subspaces of a finite dimensional vector space over $\mathbb{F}_q$. We introduce an $h$-vector for $q$-posets, based on the theory developed in Section~\ref{section-subdivision}, and thus answer an open problem posed by Alder \cite{alder2010q}. We prove that the rank generating polynomial of every shellable $q$-poset has a nonnegative $h$-vector. In the process we provide a characterization of the convex hull of all characteristic polynomials of hyperplane arrangements of fixed dimension over a fixed finite field, Corollary \ref{ch-cor}. This may be seen as a refinement of the critical problem of Crapo and Rota \cite{crapo1970foundations}. In Theorem~\ref{qmrr} we prove that the chain polynomial of any $q$-matroid is real-rooted. In Section \ref{sec-r-cubical} we consider $r$-cubical posets. We introduce notions of $h$-vectors and shellability for $r$-cubical posets which generalize the corresponding notions for cubical complexes. We prove that each shellable $r$-cubical poset has a nonnegative $h$-vector, and that the chain polynomial of any $r$-cubical poset with a nonnegative $h$-vector is real-rooted. This generalizes Athanasiadis' theorem for the case of cubical complexes \cite{athanasiadis2021face}. In Section \ref{sec-part} we study the infinite dual partition lattice. We introduce notions of $h$-vectors and shellability for so called partition posets, and prove that chain polynomials of shellable partition posets are real-rooted. In a sequel \cite{BL2} to this paper we apply the theory developed here to chain enumeration in geometric lattices. For example we prove that the chain polynomials of Dowling lattices and the lattices of flats of paving matroids are real-rooted. \section{Totally nonnegative and resolvable matrices} \label{sec-tnm} In this section we will prove that for lower triangular matrices $R$ whose diagonal entries are all equal to one, total nonnegativity is equivalent to the resolvability of the row generating polynomials of $R$. This will enable us to prove in Section \ref{sec-interlacing} that such matrices give rise to a family of real-rooted polynomials. For $N \in \NN\cup\{\infty\}$, let $\Gamma_N$ be the directed graph on $\{ (i,j) \in \NN^2: j\leq i \leq N\}$ with edges $$ (i,j) \to (i,j+1) \ \ \ \mbox{ and } \ \ \ (i+1,j) \to (i,j), $$ and let $\lambda=(\lambda_{i,j})_{0\leq j \leq i < N}$ be an array of nonnegative numbers. \begin{figure} \centering \begin{tikzpicture}[line cap=round,line join=round,>=triangle 45,x=1cm,y=1cm,scale=1.4] \draw [color=cqcqcq,, xstep=0.5cm,ystep=0.5cm]; \clip(-1.1480869706791401,-6.160766230552031) rectangle (7,0.7001462190018983); \draw [->,line width=0.3pt] (0,0) -- (5.5,0); \draw [->,line width=0.3pt] (0,0) -- (0,-5.5); \draw [line width=0.3pt] (0,-1) -- (1,-1); \draw [line width=0.3pt] (0,-2) -- (2,-2); \draw [line width=0.3pt] (0,-3) -- (3,-3); \draw [line width=0.3pt] (0,-4) -- (4,-4); \draw [line width=0.3pt] (0,-5) -- (5,-5); \draw [line width=0.3pt] (1,-5) -- (1,-1); \draw [line width=0.3pt] (2,-5) -- (2,-2); \draw [line width=0.3pt] (3,-5) -- (3,-3); \draw [line width=0.3pt] (4,-5) -- (4,-4); \draw [line width=0.3pt,domain=0:5.5] plot(\x,{(-0-2*\x)/2}); \draw [->,-latex] (0,-1) -- (0,-0.4); \draw [->,-latex] (0,-1) -- (0.6,-1); \draw [->,-latex] (0,-2) -- (0,-1.4); \draw [->,-latex] (0,-2) -- (0.6,-2); \draw [->,-latex] (1,-2) -- (1,-1.4); \draw [->,-latex] (1,-2) -- (1.6,-2); \draw [->,-latex] (0,-3) -- (0,-2.4); \draw [->,-latex] (0,-3) -- (0.6,-3); \draw [->,-latex] (1,-3) -- (1,-2.4); \draw [->,-latex] (1,-3) -- (1.6,-3); \draw [->,-latex] (2,-3) -- (2,-2.4); \draw [->,-latex] (2,-3) -- (2.6,-3); \draw [->,-latex] (0,-4) -- (0,-3.4); \draw [->,-latex] (0,-5) -- (0,-4.4); \draw [->,-latex] (0,-4) -- (0.6,-4); \draw [->,-latex] (0,-5) -- (0.6,-5); \draw [->,-latex] (1,-4) -- (1,-3.5); \draw [->,-latex] (1,-4) -- (1.6,-4); \draw [->,-latex] (2,-4) -- (2,-3.4); \draw [->,-latex] (2,-4) -- (2.6,-4); \draw [->,-latex] (3,-4) -- (3,-3.4); \draw [->,-latex] (3,-4) -- (3.6,-4); \draw [->,-latex] (1,-5) -- (1,-4.4); \draw [->,-latex] (1,-5) -- (1.6,-5); \draw [->,-latex] (2,-5) -- (2,-4.4); \draw [->,-latex] (2,-5) -- (2.6,-5); \draw [->,-latex] (3,-5) -- (3,-4.4); \draw [->,-latex] (3,-5) -- (3.6,-5); \draw [->,-latex] (4,-5) -- (4,-4.4); \draw [->,-latex] (4,-5) -- (4.6,-5); \draw (-0.05404958007457205,0.42200111969565796) node[anchor=north west] {0}; \draw (0.9658157840483304,-0.4773347013945192) node[anchor=north west] {1}; \draw (1.9671381415508162,-1.4508425489663608) node[anchor=north west] {2}; \draw (2.9777320023635103,-2.350178370056538) node[anchor=north west] {3}; \draw (3.96051135324558,-3.4164012507304595) node[anchor=north west] {4}; \draw (4.9525622074378575,-4.389909098302301) node[anchor=north west] {5}; \draw (-0.4434527191033166,0.11604151045879355) node[anchor=north west] {0}; \draw (-0.4805387323441494,-0.9038238536640878) node[anchor=north west] {1}; \draw (-0.4712672290339412,-1.9236892177869693) node[anchor=north west] {2}; \draw (-0.4527242224135248,-2.925011575289435) node[anchor=north west] {3}; \draw (-0.4527242224135248,-3.898519422861276) node[anchor=north west] {4}; \draw (-0.4712672290339412,-4.918384786984157) node[anchor=north west] {5}; \draw (-0.5,-0.3) node[anchor=north west] {$\lambda_{00}$}; \draw (-0.5,-1.3) node[anchor=north west] {$\lambda_{10}$}; \draw (-0.5,-2.3) node[anchor=north west] {$\lambda_{20}$}; \draw (-0.5,-3.3) node[anchor=north west] {$\lambda_{30}$}; \draw (-0.5,-4.3) node[anchor=north west] {$\lambda_{40}$}; \draw (0.5,-1.3) node[anchor=north west] {$\lambda_{11}$}; \draw (0.5,-2.3) node[anchor=north west] {$\lambda_{21}$}; \draw (0.5,-3.3) node[anchor=north west] {$\lambda_{31}$}; \draw (0.5,-4.3) node[anchor=north west] {$\lambda_{41}$}; \draw (1.5,-4.3) node[anchor=north west] {$\lambda_{42}$}; \draw (2.5,-4.3) node[anchor=north west] {$\lambda_{43}$}; \draw (3.5,-4.3) node[anchor=north west] {$\lambda_{44}$}; \draw (1.5,-3.3) node[anchor=north west] {$\lambda_{32}$}; \draw (2.5,-3.3) node[anchor=north west] {$\lambda_{33}$}; \draw (1.5,-2.3) node[anchor=north west] {$\lambda_{22}$}; \draw (5.6479249557034725,0.2365710534914977) node[anchor=north west] {\Large $j$}; \draw (-0.07259258669498846,-5.446860475666013) node[anchor=north west] {\Large $i$}; \begin{scriptsize} \draw [fill=black] (0,0) circle (1.0pt); \draw [fill=black] (2,-2) circle (1.0pt); \draw [fill=black] (0,-0.9888560254288775) circle (1.0pt); \draw [fill=black] (0,-2.0005086637613667) circle (1.0pt); \draw [fill=black] (1,-2) circle (1.0pt); \draw [fill=black] (0,-3) circle (1.0pt); \draw [fill=black] (1,-3) circle (1.0pt); \draw [fill=black] (2,-3) circle (1.0pt); \draw [fill=black] (0,-4) circle (1.0pt); \draw [fill=black] (0,-5) circle (1.0pt); \draw [fill=black] (1,-4) circle (1.0pt); \draw [fill=black] (2,-4) circle (1.0pt); \draw [fill=black] (3,-4) circle (1.0pt); \draw [fill=black] (1,-5) circle (1.0pt); \draw [fill=black] (2,-5) circle (1.0pt); \draw [fill=black] (3,-5) circle (1.0pt); \draw [fill=black] (4,-5) circle (1.0pt); \draw [fill=black] (1,-1) circle (1.0pt); \draw [fill=black] (3,-3) circle (1.0pt); \draw [fill=black] (4,-4) circle (1.0pt); \draw [fill=black] (5,-5) circle (1.0pt); \end{scriptsize} \end{tikzpicture} \caption{The directed graph $\Gamma_N$ with its weights.} \label{fig:Gamma} \end{figure} Attach the weight $\lambda_{i,j}$ to the vertical edge $(i+1,j) \to (i,j)$, and attach the weight $1$ to each horizontal edge, see Fig. \ref{fig:Gamma}. Let further $r_{n,k}(\Gamma_N, \lambda)$ be the weighted sum of all paths from $(n,0)$ to $(k,k)$, where the weight of a path is the product of the weights of the edges used in the path. Recall that a matrix with real entries is \emph{totally nonnegative} (TN) if all of its minors are nonnegative. By the Lindstr\"om-Gessel-Viennot Lemma~\cite[Thm. 7.16.1]{stanley2011enumerative}, the matrix $R(\Gamma_N, \lambda)= (r_{n,k}(\Gamma_N, \lambda))_{n,k=0}^N$ is totally nonnegative. By Whitney's reduction Theorem~\cite{whitney1952reduction}, the converse is also true. \begin{theorem}\label{WL} Let $R=(r_{n,k})_{n,k=0}^N$, $N \in \NN\cup\{\infty\}$, be a lower triangular matrix whose diagonal entries are all equal to one. Then $R$ is totally nonnegative if and only if there is an array of nonnegative numbers $\lambda=(\lambda_{i,j})_{0\leq j \leq i < N}$ such that $R= R(\Gamma_N, \lambda)$. Moreover we may choose $\lambda$ so that \begin{equation}\label{limp} \lambda_{n,k}=0 \ \ \ \mbox{ implies } \ \ \ \lambda_{n+1,k}=0, \end{equation} for all $0\leq k \leq n <N-1$, and then $\lambda$ is unique. \end{theorem} \begin{proof} It suffices to prove the theorem for finite $N$. One direction is the Lindstr\"om-Gessel-Viennot lemma. We prove the existence of $\Gamma_N$, $N \in \NN$, and $\lambda$ by induction on $N$. Let $R=(r_{n,k})_{n,k=0}^N$ be a lower triangular matrix with nonnegative entries and with all diagonal entries equal to one, and let $m+1$ be the first index for which $r_{m+1,0}=0$. Whitney's reduction theorem says that $R$ is $\mathrm{TN}$ if and only if $r_{j,0}=0$ for each $j>m$, and the matrix $\tilde{R}= (\tilde{r}_{n,k})_{n,k=0}^{N-1}$, where $\tilde{r}_{n,k}= r_{n+1,k+1}- \mu_n r_{n,k+1}$ and $$ \mu_n= \begin{cases} r_{n+1,0}/r_{n,0} &\mbox{ if } n \leq m \\ 0 &\mbox{ otherwise} \end{cases} $$ is $\mathrm{TN}$. If $\lambda$ satisfies \eqref{limp}, then $$ r_{n+1,k+1}(\Gamma_N, \lambda)= \lambda_{n,0} r_{n,k+1}(\Gamma_N, \lambda)+ r_{n,k}(\Gamma_{N-1}, \tilde{\lambda}), $$ where $\tilde{\lambda}_{n,k}=\lambda_{n+1,k+1}$, and $$ \lambda_{n,0} = \begin{cases} r_{n+1,0}(\Gamma_N, \lambda)/r_{n,0}(\Gamma_N, \lambda) &\mbox{ if } n \leq m \\ 0 &\mbox{ otherwise} \end{cases}. $$ Suppose $R$ is $\mathrm{TN}$. By induction there exist $\gamma$ satisfying \eqref{limp} such that $\tilde{r}_{n,k}= r_{n,k}(\Gamma_{N-1}, \gamma)$. But then $r_{n,k}= r_{n,k}(\Gamma_{N}, \lambda)$, where $\lambda$ is obtained from $\gamma$ by adding a first column equal to $\mu$ to the left of $\gamma$. The uniqueness of $\lambda$ follows by induction. \end{proof} \begin{definition} \label{resolv} Let $R=(r_{n,k})_{n,k=0}^N$, $N \in \NN \cup\{\infty\}$, be a lower triangular matrix whose diagonal entries are all equal to one, and let $R_n(t) = \sum_{k=0}^n r_{n,k} t^k$ be the generating polynomial of the $n$th row. The matrix $R$ (and the sequence $\{R_n(t)\}_{n=0}^N$) is called \emph{resolvable} if there is a matrix $(\lambda_{n,k})_{0\leq k\leq n <N}$ of nonnegative numbers, and an array of monic polynomials $(R_{n,k}(t))_{0\leq k \leq n \leq N}$ such that \begin{itemize} \item $R_{n,0}(t)=R_n(t)$ and $R_{n,n}(t) =t^n$ for all $0\leq n \leq N$, \item $t^k$ divides $R_{n,k}(t)$ for all $0 \leq k \leq n \leq N$, and \item if $0\leq k \leq n <N$, then \begin{equation}\label{r-pasc} R_{n+1,k}(t)=R_{n+1,k+1}(t)+ \lambda_{n,k} R_{n,k}(t). \end{equation} \end{itemize} \end{definition} If the matrix $R$ is resolvable, then we say that the polynomials $(R_{n,k}(t))_{0\leq k \leq n \leq N}$ \emph{resolve} $R$. Notice that $(\lambda_{n,k})_{0\leq k\leq n <N}$ uniquely determines $(R_{n,k}(t))_{0\leq k \leq n \leq N}$. \begin{example} The identity matrix is resolvable with $R_{n,k}(t)=t^n$ and $\lambda_{n,k}=0$ for all $0 \leq k \leq n$. \end{example} \begin{example}\label{forex} Let $(x_0, x_1, x_2, \ldots)$ be a sequence of real numbers such that $x_i \geq 1$ for all $i$, and consider the matrix $R = (r_{n,k})_{n,k=0}^\infty$ where \begin{equation} \label{maximal-matrix} r_{n,k} = \begin{cases} x_k &\mbox{ if } k<n, \\ 1 &\mbox{ if } k=n, \\ 0 &\mbox{ if } k > n. \end{cases} \end{equation} Then $R$ is resolvable with $$ R_{n,k} (t) = \begin{cases} R_n (t) &\mbox{ if } k=0, \\ t^n + (x_n -1) t^{n-1} &\mbox{ if } 1 \leq k \leq n-1, \\ t^n &\mbox{ if } k = n, \end{cases} $$ and $$ \lambda_{n,k} = \begin{cases} 1 &\mbox{ if } k=0, \\ 0 &\mbox{ if } 1 \leq k \leq n-1, \\ x_n - 1 &\mbox{ if } k=n. \end{cases} $$ \end{example} \begin{example} The matrix $\left(\binom n k \right)_{n,k=0}^\infty$ is resolvable with $R_{n,k}(t)= t^k(1+t)^{n-k}$ and $\lambda_{n,k}=1$ for all $0 \leq k \leq n$. \end{example} Notice that, by \eqref{r-pasc}, \begin{equation} \label{r-sum} R_{n+1,k}(t)= t^{n+1}+ \sum_{j \geq k} \lambda_{n,j}R_{n,j}(t), \ \ \ \ 0\leq k \leq n<N, \end{equation} if $\{R_n(t)\}_{n=0}^N$ is resolvable. The next theorem characterizes resolvability in terms of totally nonnegative matrices and ``quantum'' real-rooted polynomials. \begin{theorem}\label{eqcon} Let $R=(r_{n,k})_{n,k=0}^N$ be a lower triangular matrix with all diagonal entries equal to one. The following are equivalent: \begin{enumerate} \item $R$ is resolvable, \item There are linear diagonal operators $\alpha_i : \RR[t] \to \RR[t]$, $1 \leq i \leq N$, such that $$ \alpha_i(t^k)= \alpha_{i,k}t^k, \mbox{ where } \alpha_{i,k} \geq 0 \mbox{ for all } i,k, $$ and $$ R_n(t) = (t+ \alpha_1)(t+\alpha_2) \cdots (t+\alpha_n) 1. $$ \item $R$ is $\mathrm{TN}$. \end{enumerate} Moreover if (2) is satisfied, then $R_{n,k}(t) = (t+\alpha_1)\cdots (t+\alpha_{n-k})t^k$ and $\lambda_{n,k}= \alpha_{n+1-k,k}$. \end{theorem} \begin{proof} Suppose (2) holds, and let $R_{n,k}(t) = (t+\alpha_1)\cdots (t+\alpha_{n-k})t^k$. Then \begin{align*} R_{n+1,k}(t)-R_{n+1,k+1}(t) &= (t+\alpha_1)\cdots (t+\alpha_{n-k})( (t+\alpha_{n+1-k})t^k-t^{k+1})\\ &=\alpha_{n+1-k,k}R_{n,k}. \end{align*} Hence $R$ satisfies (1) with $\lambda_{n,k}= \alpha_{n+1-k,k}$. Conversely if $R$ satisfies (1), then $R$ satisfies (2) with $\alpha_{i,k}=\lambda_{k+i-1,k}$. Now, suppose $R$ is a $\mathrm{TN}$-matrix. Then, by Theorem~\ref{WL}, $R=R(\Gamma_N, \lambda)$ for some nonnegative array $\lambda$. Define $(R_{n,k}(t))_{0\leq k \leq n \leq N}$ as follows. Let $$R_{n,k}(t)=\sum_{j=0}^n r_{n,j}^k(\lambda) t^j,$$ where $r_{n,j}^k(\lambda)$ is the $\lambda$-weighted sum over of paths from $(n,k)$ to $(j,j)$ in $\Gamma$. Then $R_{n,k}(t)$ satisfy Definition \ref{resolv} by construction. Conversely if the polynomials $R_{n,k}(t)$ and $\lambda$ satisfy Definition \ref{resolv}, then $R=R(\Gamma_N, \lambda)$ since $\lambda$ uniquely determines $R_{n,k}(t)$. \end{proof} \begin{example} Let $x_1,x_2, \ldots$ be nonnegative real numbers, and consider the matrix $R=(e_k(x_1,\ldots, x_n))_{n,k=0}^\infty$, where $e_k(x_1,\ldots,x_n)$ is the $k$th \emph{elementary symmetric polynomial} in the variables $x_1,\ldots, x_n$. Then $R_n(t) = (t+ \alpha_1)(t+\alpha_2) \cdots (t+\alpha_n) 1$, where $\alpha_i(t^k)= x_i t^k$ for all $i,k$. Hence $R$ is $\mathrm{TN}$ and $R_{n,k}(t)= t^k (t+x_1) \cdots (t+x_{n-k})$. \end{example} \begin{remark}\label{thethe} Henceforth we will always assume that the numbers $\lambda_{n,k}$ satisfy \eqref{limp}, so that the polynomials $R_{n,k}(t)$ associated to a resolvable matrix $R$ are uniquely determined. \end{remark} \section{Interlacing sequences of polynomials from resolvable matrices} \label{sec-interlacing} In this section we will use resolvability to a prove a general theorem, Theorem~\ref{mainUTP}, which for any lower triangular $\mathrm{TN}$-matrix with all diagonal entries equal to one produces a family of real-rooted and mutually interlacing polynomials. This will be our main tool in forthcoming sections. Suppose $f(t), g(t) \in \RR[t]$ are real-rooted polynomials with positive leading coefficients, and that $$ \cdots \leq \alpha_3 \leq \alpha_2 \leq \alpha_1 \ \ \mbox{ and } \ \ \cdots \leq \beta_3 \leq \beta_2 \leq \beta_1 $$ are the zeros of $f(t)$ and $g(t)$, respectively. We say that the zeros of $f(t)$ \emph{interlace} those of $g(t)$ if $$ \cdots \leq \alpha_3 \leq \beta_3 \leq \alpha_2 \leq \beta_2 \leq \alpha_1 \leq \beta_1, $$ and we write $f(t) \prec g(t)$. In particular the degrees of $f(t)$ and $g(t)$ differ by at most one. By convention we also write $0 \prec f(t)$ and $f(t) \prec 0$ for any real-rooted polynomial $f(t)$. A sequence $\{f_i(t)\}_{i=0}^n$ of real-rooted polynomials with nonnegative coefficients is said to be an \emph{interlacing sequence} if $f_i(t) \prec f_j(t)$ for all $i<j$. Let $\mathcal{P}_n$ be the set of all interlacing sequences $\{f_i(t)\}_{i=0}^n$ of polynomials. \begin{lemma}\label{basint} Let $\{f_i(t)\}_{i=0}^n \in \mathcal{P}_n$ be an interlacing sequence of polynomials. \begin{enumerate} \item If $\{\lambda_i\}_{i=0}^n \in \RR_{\geq 0}^{n+1}$, then $$ f_0(t) \prec \lambda_0 f_0(t) + \lambda_1 f_1(t)+\cdots + \lambda_n f_n(t) \prec f_n(t). $$ \item If the $f_i(t)$ is of degree $d$ and its zeros all lie in $[-1,0]$ for each $0\leq i \leq n$, then the sequence $\{g_i(t)\}_{i=0}^{n+1}$ defined by $$ g_k(t) = t\sum_{j=0}^{k-1} f_{j}(t) + (1+t) \sum_{j = k}^n f_{j}(t) $$ is an interlacing sequence of polynomials whose zeros all lie in $[-1,0]$. \end{enumerate} \end{lemma} \begin{proof} The first result is standard, see e.g. \cite[Lemma 2.6]{borcea2010multivariate}. For the second result, let $h_j(t)= (1-t)^df_j(t/(1-t))$ and $r_j(t) = (1-t)^{d+1}g_j(t/(1-t))$. Then $\{h_j(t)\}_{j=0}^n$ is an interlacing sequence of polynomials with nonnegative coefficients, since the zeros of $f_j(t)$ all lie in $[-1,0]$. Moreover, $$ r_k(t) = t\sum_{j=0}^{k-1}h_j(t)+ \sum_{j=k}^nh_j(t), $$ and hence $\{r_j(t)\}_{j=0}^{n+1}$ is interlacing by e.g. \cite[Corollary 7.8.7]{branden2015unimodality}. Since $g_j(t)= (1+t)^{d+1}r_j(t/(1+t))$, the result follows. \end{proof} For $N \in \NN\cup \{\infty\}$, let $\RR_N[t]$ be the linear space of all polynomials in $\RR[t]$ of degree at most $N$. \begin{definition}\label{subop} Let $R=(r_{n,k})_{n,k=0}^N$, where $N \in \NN\cup \{\infty\}$, be a lower triangular matrix with all diagonal entries equal to one. The \emph{chain polynomials} associated to $R$ are the polynomials in $\{p_n(t)\}_{n=0}^N$ defined by $p_0(t)=1$, and \begin{equation}\label{polreceq} p_n(t)= t \sum_{k=0}^{n-1}r_{n,k}p_k(t), \ \ \ 0<n \leq N. \end{equation} The \emph{subdivision operator} associated to $R$ is the linear map $\EE : \RR_N[t] \to \RR_N[t]$ defined by $\EE(t^n)= p_n(t)$ for each $0 \leq n \leq N$. \end{definition} Notice that the subdivision operator may alternatively be defined by $\EE(1)= 1$, and \begin{equation} \label{fundE} \EE(t^n) = t\EE(R_n(t) -t^{n}), \ \ \ \mbox{ if } 0<n \leq N. \end{equation} \begin{example}\label{binomet} The subdivision operator for $\left(\binom n k \right)_{n,k=0}^\infty$ was considered in \cite{branden2015unimodality,brenti2008f}. This operator has the property that it maps the $f$-polynomial of a simplicial complex $\Delta$ to the $f$-polynomials of the barycentric subdivision of $\Delta$, see \cite[Ch. 7.3.3]{branden2015unimodality}. The chain polynomials in this case are given by \begin{equation}\label{snkk} p_n(t) = \sum_{k=1}^n k! S(n,k) t^k, \end{equation} where $S(n,k)$ is a Stirling number of the second kind \cite{stanley2011enumerative}. \end{example} \begin{example}\label{forex2} Let $x_0,x_1, \ldots$ be real numbers such that $x_i \geq 1$ for all $i$, and consider the matrix $R$ in Example~\ref{forex}. It follows that $$ p_n(t) =x_0 (1+x_1t) (1+x_2t)\cdots (1+x_{n-1}t), \ \ \ n \geq 1. $$ \end{example} \begin{proposition}\label{chainform} Let $\mathsf{p}=(p_n(t))_{n=0}^N$ be the vector of chain polynomials associated to $R$. Then $$ \mathsf{p}= R(t)^{-1}(1,0,0,\ldots)^T, $$ where $R(t)= I-t(R-I)$, and $I$ is the identity matrix. Moreover $$ p_n(t) = (-1)^{n}\det \big(R(t) [\{1,2,\ldots, n\}, \{0,1,\ldots, n-1\}]\big), $$ i.e., $(-1)^{n}$ times the minor of $R(t)$ whose rows are indexed by $\{1,2,\ldots, n\}$ and columns by $\{0,1,\ldots, n-1\}$. \end{proposition} \begin{proof} The recursion \eqref{polreceq} translates as $(1+t)\mathsf{p}-tR\mathsf{p}=(1,0,0,\ldots)^T$, which proves the first statement. The second statement follows from the first by Cramer's rule. \end{proof} The next theorem is the most general theorem on real-rootedness in this paper. It produces a family of real-rooted polynomials to any totally nonnegative lower triangular matrix with ones on the diagonal. The case when $R=\left(\binom n k \right)_{n,k=0}^\infty$ was proved by the first author in \cite{branden2006linear}. \begin{theorem}\label{mainUTP} Let $R=(r_{n,k})_{n,k=0}^N$ be a lower triangular $\mathrm{TN}$-matrix with all diagonal entries equal to one, and let $0 \leq n \leq N$. Let further $\{R_{n,k}(t)\}_{k=0}^n$ be the family of polynomials afforded by Definition \ref{resolv} and Theorem~\ref{eqcon}. Then $\{\EE(R_{n,k}(t))\}_{k=0}^n$ is an interlacing sequence of polynomials whose zeros all lie in the interval $[-1,0]$. \end{theorem} \begin{proof} The proof is by induction over $n$, the case $n=0$ being trivial. Assume $\{\EE(R_{n,k}(t))\}_{k =0}^n$ is an interlacing sequence of polynomials whose zeros lie in $[-1,0]$. Then by \eqref{r-sum} and \eqref{fundE}, \begin{align*} \EE(R_{n+1,k}(t)) &= \EE(t^{n+1})+ \sum_{j \geq k} \lambda_{n,j}\EE(R_{n,j}(t)) \\ &= t\sum_{j \geq 0} \lambda_{n,j}\EE(R_{n,j}(t))+ \sum_{j \geq k} \lambda_{n,j} \EE(R_{n,j}(t)) \\ &= t \sum_{j =0 }^{k-1} \lambda_{n,j} \EE(R_{n,j}(t)) + (1+t) \sum_{j \geq k} \lambda_{n,j} \EE(R_{n,j}(t)) \smash{\text{\quad\raisebox{2.0\baselineskip}{.}}} \end{align*} The proof now follows from Lemma~\ref{basint} (2), since the sequence $\{\lambda_{n,j}\EE(R_{n,j}(t))\}_{j=0}^n$ is interlacing. \end{proof} \begin{theorem} \label{maint} Let $R=(r_{n,k})_{n,k=0}^N$, where $N \in \NN\cup \{\infty\}$, be a lower triangular matrix with all diagonal entries equal to one. If $R$ is totally nonnegative, then for each $n \in \NN$, the zeros of the chain polynomial $p_n(t)$ defined by \eqref{polreceq} are real and located in the interval $[-1,0]$. Moreover, the zeros of $p_n(t)$ interlace those of $p_{n+1}(t)$ for each $0 \leq n <N$. \end{theorem} \begin{proof} By Theorem~\ref{mainUTP} it remains to prove that $p_{n}(t) \prec p_{n+1}(t)$ for $0 \leq n <N$. By the recursion in the proof of Theorem~\ref{mainUTP}, $$ p_{n+1}(t)= \EE(R_{n+1,n+1}(t))= t \sum_{j =0 }^{n} \lambda_{n,j} \EE(R_{n,j}(t)). $$ Since the sequence $\{\EE(R_{n,k}(t))\}_{k \geq 0}$ is interlacing, Lemma~\ref{basint} (2) implies $p_{n+1}(t) \prec t p_{n}(t)$, which implies $p_{n}(t) \prec p_{n+1}(t)$. \end{proof} \section{P\'olya frequency sequences} \label{sec-polya} Recall that a sequence $\{a_i\}_{i=0}^\infty$ of real numbers is a \emph{P\'olya frequency sequence} if the Toeplitz matrix $(a_{i-j})_{i,j=0}^\infty$ is $\mathrm{TN}$, where $a_k=0$ if $k<0$. P\'olya frequency sequences were characterized by Aissen, Schoenberg, Whitney and Edrei \cite{aissen1951generating} as follows. \begin{theorem}\label{ASWE} A sequence $(a_i)_{i=0}^\infty$ of real numbers is a P\'olya frequency sequence if and only if its generating function is of the form \begin{equation} \label{PFS} \sum_{n=0}^\infty a_n x^n = C x^N e^{\gamma x} \cdot \frac {{\prod_{i=1}^\infty (1+ \alpha_ix)}} {{\prod_{i=1}^\infty (1- \beta_ix)}}, \end{equation} where $C, \gamma, \alpha_i, \beta_i$ are nonnegative real numbers, $N \in \NN$, and $\sum_{i=1}^\infty (\alpha_i+\beta_i)<\infty$. \end{theorem} \begin{remark}\label{polpf} Let $P : \NN \rightarrow \RR$. Then $P$ is a polynomial of degree $d$ if and only if $$ \sum_{n=0}^\infty P(n) x^n = \frac {h(x)} {(1-x)^{d+1}}, $$ where $h(x)$ is a polynomial of degree at most $d$ for which $h(1) \neq 0$, see e.g. \cite[Corollary 4.3.1]{stanley2011enumerative}. Hence, by Theorem~\ref{ASWE}, the sequence $\{P(n)\}_{n =0}^\infty$, where $P$ is a polynomial of degree $d$, is a P\'olya frequency sequence if and only if all zeros of $h(x)$ are real and non-positive. \end{remark} If $R$ is a lower triangular matrix and $i,j \in \NN$, consider the map $Z_{ij}^R : \NN \to \RR$ defined by $$ n \longmapsto (R^n)_{ij}, $$ that is, $Z_{ij}^R(n)$ is the $(i,j)$-entry of $R^n$. \begin{theorem}\label{Z-pos} Suppose $R$ is a lower triangular matrix with all diagonal entries equal to one, and let $i\geq j$ be nonnegative integers. \begin{itemize} \item[(a)] $Z_{ij}^R(n)$ is a polynomial in $n$ of degree $i-j$. \item[(b)] If $R$ is $\mathrm{TN}$, then $\{Z_{ij}^R(n)\}_{n=0}^\infty$ is a P\'olya frequency sequence. \end{itemize} \end{theorem} \begin{proof} The case when $j>0$ follows from the case when $j=0$ by observing that $(R^n)_{ij}= (S^n)_{i-j,0}$, where $S$ is the matrix obtained from $R$ by deleting the $j$ initial rows and columns of $R$. Suppose $j=0$. Let $$ f_i(t)= \sum_{n=0}^\infty Z_{i0}^R(n)t^n = \big( (I-tR)^{-1}\big)_{i0}. $$ Since $$ I-tR= (1-t) \left( \left(1+ \frac t {1-t} \right) I - \frac t {1-t} R \right), $$ it follows from Proposition~\ref{chainform} that $$ f_i(t)= \frac 1 {1-t} p_i\left(\frac t {1-t}\right), $$ where $p_i(t)$ is the polynomial defined by \eqref{polreceq}. Consider the polynomial $h_i(t)=(1-t)^i p_i(t/(1-t))$. Then $h_i(t)$ has degree at most $i$, and $$ f_i(t)= \frac {h_i(t)}{(1-t)^{i+1}}, $$ so that (a) follows from Remark \ref{polpf}. If $R$ is $\mathrm{TN}$, then $p_i(t)$ is a polynomial of degree $i$ whose zeros all lie in $[-1,0]$ by Theorem~\ref{maint}. Hence all zeros of $h_i(t)$ are real and nonpositive, from which (b) follows by Remark \ref{polpf}. \end{proof} Let $g(x)=\sum_{n=1}^\infty b_n x^n \in \RR[[x]]$ be a formal power series with constant term equal to one, and consider the expansion $$ \frac 1 {1-tg(x)}= \sum_{n=0}^\infty g_n(t) x^n \in \RR[[x,t]]. $$ Then $g_n(t)$ is a polynomial of degree at most $n$ for each $n \in \NN$. We will now prove that for an important class of series $g(x)$, the polynomials $g_n(t)$ are all real-rooted. \begin{theorem} \label{PFT} Let $f(x)$ be as in \eqref{PFS}, and consider the formal power series \begin{equation}\label{pnp} \frac 1 {1-t(f(x)-f(0))} = \sum_{n=0}^\infty r_n(t) x^n \in \RR[t][[x]]. \end{equation} Then $r_n(t)$ is a real-rooted polynomial for each $n \in \NN$, and the zeros of $r_n(t)$ interlace those of $r_{n+1}(t)$. Moreover, if $f(0) \neq 0$, then all zeros of $r_n(t)$ lie in the interval $[-1/f(0),0]$. \end{theorem} \begin{proof} Suppose first that $f(0) \neq 0$, and let $r_n(f;t)$ be the polynomial $r_n(t)$ defined by \eqref{pnp}. Since $$ \frac 1 {1-t(f(x)-f(0))} = \frac 1 {1-tf(0) \left(\frac {f(x)}{f(0)}-1\right)}, $$ $r_n(f;t)= r_n(f/f(0); tf(0))$. Hence we may assume $f(0) =1$. The matrix $R=(a_{i-j})_{i,j=0}^\infty$ is $\mathrm{TN}$, and hence the polynomials $p_n(t)$ defined by \eqref{polreceq} are $[-1,0]$-rooted polynomials by Theorem~\ref{maint}. Moreover $p_n(t) \prec p_{n+1}(t)$. Clearly, $p_0(t)=1$ and $ p_n(t)= -tp_n(t)+t\sum_{k=0}^{n}a_{n-k} p_k(t)$ for $n \geq 1$, from which we deduce $$ \frac 1 {1-t(f(x)-f(0)))} = \sum_{n=0}^\infty p_n(t) x^n. $$ Hence $r_n(t) = p_n(t)$, which proves the case when $f(0) \neq 0$ by Theorem~\ref{maint}. If $f(x)$ is as in \eqref{PFS}, with $f(0)=0$, then consider $$ f_{\epsilon}(x)= \frac {(x+\epsilon)^N}{x^N} f(x), $$ where $\epsilon \geq 0$. Then, for fixed $n$, the coefficients of the degree $n$ polynomial $r_n(f_\epsilon;t)$ depend continuously on $\epsilon$. The conclusion follows from Hurwitz's theorem on the continuity of zeros. \end{proof} \subsection{A problem of Forg\'acs and Tran} The next theorem solves an open problem stated by Forg\'acs and Tran \cite[Problem 13]{forgacs2016polynomials}. \begin{theorem}\label{FTT} Let $Q(x)$, where $Q(0)>0$, be a polynomial whose zeros are all real and positive, and let $r$ be a positive integer\footnote{Notice that Problem 13 is stated for all $r\geq 0$ in \cite{forgacs2016polynomials}. However for $r=0$, $q_n(t)$ fails to be a polynomial.}. Consider the power series $$ \frac 1 {Q(x)-tx^r} = \sum_{n=0}^\infty q_n(t) x^n. $$ Then $q_n(t)$ is a polynomial whose zeros are real and negative for each $n \in \NN$. Moreover, the zeros of $q_n(t)$ interlace those of $q_{n+1}(t)$ for each $n \in \NN$. \end{theorem} \begin{proof} The series $ f(x) = {x^r}/{Q(x)} $ is of the form \eqref{PFS}. Hence the polynomials $r_n(t)$ in $$ \frac 1 {1-tf(x)} = \sum_{n=0}^\infty r_n(t) x^n $$ are all real-rooted, and the zeros of consecutive polynomials interlace by Theorem~\ref{PFT}. Since $$ \sum_{n=1}^\infty r_n(t) x^n = \frac {tx^r} {Q(x)-tx^r}, $$ we see that $q_n(t) = t^{-1}r_{n+r}(t)$, from which the theorem follows. \end{proof} \section{\texorpdfstring{$\mathrm{TN}$}--posets} \label{sec-tn} For undefined poset terminology we refer to \cite{stanley2011enumerative}. In this section we study chain polynomials associated to what we call quasi-rank uniform posets. Suppose $P$ is a locally finite poset. Define a \emph{quasi-rank function} $\rho : P \to \NN$ by $$ \rho(x) = \max\{ k : x_0 <x_1 < \cdots <x_k=x\}. $$ The \emph{quasi-rank} of $P$ is $\rho(P)= \sup\{ \rho(x) : x \in P\} \in \NN\cup\{\infty\}$. \begin{definition} \label{rank uniform} Let $P$ be a locally finite poset with a least element $\zero$. $P$ is called \emph{quasi-rank uniform} if for any $x, y\in P$ with $\rho(x)=\rho(y)$, $$ |\{ z \in \langle x \rangle : \rho(z) = k \}| = |\{ z \in \langle x \rangle : \rho(z) = k \}|, \ \ \mbox{ for all } 0 \leq k \leq \rho(x), $$ where $ \langle x \rangle = \{z \in P : z \leq x\}$. If in addition $\rho(y) = \rho(x)+1$ whenever $y$ covers $x$ in $P$, i.e., when $\rho$ is the rank function of $P$, then we say that $P$ is \emph{rank uniform}. \end{definition} \begin{figure}[H] \centering \begin{tikzpicture}[line cap=round,line join=round,>=triangle 45,x=1cm,y=1cm,scale=0.5] \clip(-5,-6) rectangle (10.339115239601984,3.395947613421145); \draw [line width=2pt] (-1,-3)-- (2,-5); \draw [line width=2pt] (1,-3)-- (2,-5); \draw [line width=2pt] (2,-5)-- (3,-3); \draw [line width=2pt] (2,-5)-- (5,-3); \draw [line width=2pt] (1,-3)-- (1,-1); \draw [line width=2pt] (1,-1)-- (0,1); \draw [line width=2pt] (-1,-3)-- (0,1); \draw [line width=2pt] (5,-3)-- (5,-1); \draw [line width=2pt] (5,-1)-- (4,1); \draw [line width=2pt] (3,-3)-- (4,1); \draw [line width=2pt] (0,3)-- (0,1); \draw [line width=2pt] (0,3)-- (4,1); \draw [line width=2pt] (4,1)-- (4,3); \draw [line width=2pt] (4,3)-- (0,1); \begin{scriptsize} \draw [fill=ududff] (2,-5) circle (5.0pt); \draw (1.8,-5.142979430587969) node[anchor=north west] {$0$}; \draw [fill=ududff] (-1,-3) circle (5.0pt); \draw (-1.5,-3.046320446741683) node[anchor=north west] {$1$}; \draw [fill=ududff] (1,-3) circle (5.0pt); \draw (0.6,-3.1384034426538507) node[anchor=north west] {$1$}; \draw [fill=ududff] (1,-1) circle (5.0pt); \draw (1.1662321852744877,-0.9638280776511151) node[anchor=north west] {$2$}; \draw [fill=ududff] (0,1) circle (5.0pt); \draw (-0.8,1.0407479102830028) node[anchor=north west] {$3$}; \draw [fill=ududff] (3,-3) circle (5.0pt); \draw (3.1495582510750277,-3.0959035983866965) node[anchor=north west] {$1$}; \draw [fill=ududff] (5,-3) circle (5.0pt); \draw (5.154134239009145,-3.06757036887526) node[anchor=north west] {$1$}; \draw [fill=ududff] (5,-1) circle (5.0pt); \draw (5.18955077589844,-0.956744770273256) node[anchor=north west] {$2$}; \draw [fill=ududff] (4,1) circle (5.0pt); \draw (4.204971050376029,1.0832477545501573) node[anchor=north west] {$3$}; \draw [fill=ududff] (0,3) circle (5.0pt); \draw (-0.8,3.1940733531521617) node[anchor=north west] {$4$}; \draw [fill=ududff] (4,3) circle (5.0pt); \draw (4.162471206108875,3.20823996790788) node[anchor=north west] {$4$}; \end{scriptsize} \end{tikzpicture} \caption{A quasi-rank uniform poset, and the corresponding quasi-rank function.} \label{fig:quasi-rank-uniform} \end{figure} We will primarily be interested in rank uniform posets, but find it more convenient working in the more general setting of quasi-rank uniform posets. Let $P$ be quasi-rank uniform. If $x \in P$, $\rho(x)=n$ and $0 \leq k \leq n$, let $$ r_{n,k}= r_{n,k}(P)=|\{ z \in \langle x \rangle : \rho(z) = k \}|, $$ and let $R= R(P)= (r_{n,k})_{n,k=0}^N$. \begin{proposition} Let $P$ be a quasi-rank uniform poset, and let $n$ be a positive integer. The chain polynomial $p_n(t)$, see \eqref{polreceq}, associated to the matrix $R(P)$ may be expressed as \begin{equation}\label{pnchain} p_n(t) = \sum_{j \geq 1} |\{x_0 < x_1<\cdots < x_j=x : \rho(x_0)=0\}|t^j, \end{equation} where $x$ is fixed element in $P$ for which $\rho(x)=n$. \end{proposition} \begin{proof} Let $P$ be quasi-rank uniform. Define polynomials $p_x(t)$, $x \in P$, by $p_{x}(t)=1$ if $x$ is a minimal element, and $$ p_x(t)= \sum_{j \geq 1} |\{x_0 < x_1<\cdots < x_j=x : \rho(x_0)=0\}|t^j, \ \ \ \mbox{ if } \rho(x) > 0. $$ If $\rho(x)> 0$, then $$ p_x(t)=t \sum_{y<x} p_y(t), $$ from which the proof follows by induction on $\rho(x)$. \end{proof} We call the polynomial \eqref{pnchain} the $n$th \emph{chain polynomial}\footnote{If $P$ has a least element $\zero$, then the chain polynomial of the interval $[\zero, x]$ in $P$ is commonly defined as $t^{-1}(1+t)^{2}p_n(t)$.} of $P$. \begin{remark}\label{mob} Let $P$ be a quasi-rank uniform poset with a least element $\zero$, and suppose $x \in P$ with $\rho(x)=n$. By \eqref{pnchain} and Philip Hall's Theorem~\cite[Prop. 3.8.5]{stanley2011enumerative}, $p_n(-1)= \mu_P(\zero,x)$, where $\mu_P$ is the M\"obius function of $P$. \end{remark} \begin{definition} \label{TN-poset} A quasi-rank uniform poset $P$ is a \emph{$\mathrm{TN}$-poset} if the matrix $R(P)$ is totally nonnegative. \end{definition} From Theorem~\ref{maint} and \eqref{pnchain} we deduce \begin{theorem} \label{TN-poset-chain} If $P$ is a $\mathrm{TN}$-poset, then for each $n\in \NN$ the zeros of the chain polynomial $p_n(t)$ are real and located in the interval $[-1,0]$. Moreover $p_{n}(t) \prec p_{n+1}(t)$. \end{theorem} \begin{example} Let $x_0,x_1,\ldots$ be positive integers with $x_0=1$, and recall the matrix $R$ considered in Examples \ref{forex} and \ref{forex2}. Let $P$ be the rank uniform poset that has exactly $x_i$ elements of rank $i$ for all $i$, and is such that $x<y$ whenever the rank of $x$ is smaller than the rank of $y$. Then $R(P)=R$, and $P$ is $\mathrm{TN}$ by Example \ref{forex}. Moreover $$ p_n(t) = t(1+x_1t) (1+x_2t)\cdots (1+x_{n-1}t), \ \ \ n \geq 1, $$ by Example \ref{forex2}. \end{example} \begin{remark} Suppose $P$ is a quasi-rank uniform poset with a least element $\zero$, and let $x \in P$, $\rho(x)=k$. Then $$ Z_{k0}^R(n)= \left(R^n\right)_{k0}= |\{ \zero = x_0 \leq x_1 \leq \cdots \leq x_n =x\}|. $$ Hence $Z_{k0}^R(n)$ is equal to the \emph{zeta polynomial} $Z_{[\zero, x]}(n)$ of the interval $[\zero, x]$, see \cite[Section 3.12]{stanley2011enumerative}. Hence if $P$ is $\mathrm{TN}$, then $\{Z_{[\zero, x]}(n)\}_{n=0}^\infty$ is a P\'olya frequency sequence by Theorem~\ref{Z-pos}. \end{remark} If $S=\{s_0<s_1<\cdots\}$ is a subset of $\NN$, and $P$ is quasi-rank uniform let $$ P_S= \{ x \in P: \rho(x) \in S\}, $$ be the \emph{(quasi-)rank selected} subposet induced by $S$. Define $\rho_S : P_S \to \NN$ by $\rho_S(x)= k$ if $\rho(x)=s_k$. \begin{lemma}\label{rightr} Let $P$ be a locally finite poset with quasi-rank function $\rho$, and let $S=\{s_0<s_1<\cdots\}$ be a subset of $\NN$. Then the quasi-rank function associated to $P_S$ is equal to $\rho_S$. \end{lemma} \begin{proof} Let $\rho'$ be the quasi-rank function of $P_S$, and let $x \in P_S$ be such that $\rho(x)=s_k$. Let $x_0 < x_1<\cdots < x_{s_k} =x$ be a chain of maximal length in $\langle x \rangle$. Then $\rho(x_j)=j$ for all $0\leq j \leq s_k$, since otherwise $\rho(x_j)>j$ for some $j$ and then we could find a longer chain in $\langle x \rangle$. Hence $x_{s_0} < x_{s_1} <\cdots < x_{s_k} =x$ is a chain in $P_S$, which proves $\rho'(x) \geq k = \rho_S(x)$. On the other hand if $\rho'(x)= j$ and $\rho_S(x)=k$, then there exists a chain $$x_{s_{i_0}} < x_{s_{i_1}} <\cdots < x_{s_{i_j}} =x,$$ in $P$ where $\rho(x_{s_{i_\ell}}) = s_{i_\ell} \in S$ for each $1 \leq \ell \leq j$. Moreover $i_j= k$, and hence $\{i_1, i_2,\ldots, i_j \} \subseteq \{1,2,\ldots, k\}$, which implies $j\leq k$. \end{proof} Since submatrices of $\mathrm{TN}$-matrices are $\mathrm{TN}$, we deduce by Lemma~\ref{rightr}, \begin{proposition} \label{propranksel} Let $P$ be a $\mathrm{TN}$-poset, and let $S$ be a set of nonnegative integers. Then $P_S$ is a $\mathrm{TN}$-poset. \end{proposition} Let $P$ be a quasi-rank uniform poset with a least element $\zero$. Following \cite{stanley1976binomial}, we define $\mu_S(n)=\mu_{P_S}(\zero, x)$ where $\rho_S(x)=n$, and $\mu_{P_S}$ is the M\"obius function of $P_S$. If $S=\NN$, then we write $\mu_S(n)=\mu(n)$. The next theorem generalizes \cite[Theorem~2.1]{stanley1976binomial}. \begin{theorem}\label{mobius} Let $P$ be a quasi-rank uniform poset with a least element $\zero$, and let $S=\{s_0<s_1< \cdots \}$ a set of nonnegative integers containing $0$. Then $$ \mu_S(n) = (-1)^n \det(R[\{s_1,\ldots, s_n\}, \{s_0, \ldots, s_{n-1}\}]). $$ \end{theorem} \begin{proof} It suffices to prove the theorem for $S=\NN$, since rank selection corresponds to taking principal submatrices of $R$. By Remark \ref{mob}, $p_n(-1)= \mu(n)$. The theorem now follows from Proposition~\ref{chainform}. \end{proof} Theorem~\ref{mobius} provides a necessary condition for a poset to be $\mathrm{TN}$, namely that the M\"obius function alternates in sign. \begin{corollary}\label{altmu} If $P$ is a $\mathrm{TN}$-poset with a least element $\zero$, then $(-1)^n \mu_S(n) \geq 0$ for all $S$ and $n$. \end{corollary} Let us formulate Theorem~\ref{mobius} and Corollary \ref{altmu} in terms of flag $h$-vectors of quasi-rank uniform $\mathrm{TN}$-posets. Recall that a poset is bounded if it contains a $\zero$ and a $\one$. Let $P$ be a finite and bounded poset of rank $n$. The \emph{flag $f$-vector} of $P$ is the function $\alpha_P : 2^{[n-1]} \to \ZZ$ defined by $\alpha_P(S)$ is equal to the number of maximal chains of $P_S$. The \emph{flag $h$-vector} of $P$ is the function $\beta_P : 2^{[n-1]} \to \ZZ$ defined by $$ \beta_P(S) = \sum_{T \subseteq S} (-1)^{|S \setminus T|}\alpha_P(T). $$ It is known that the flag $h$-vectors of Cohen-Macaulay posets are nonnegative, see \cite{BjornerCM}. The same is true for bounded $\mathrm{TN}$-posets. \begin{theorem} Let $P$ be a quasi-rank uniform and bounded poset of quasi-rank $n$, and let $S=\{s_1,\ldots, s_k\} \subseteq [n-1]$. Then $$ \beta_P(S) = \det(R[\{s_1,\ldots, s_k, n\}, \{0,s_1, \ldots, s_{k}\}]), $$ where $R=R(P)$. Hence if $P$ is $\mathrm{TN}$, then $\beta_P(S) \geq 0$ for all $S \subseteq [n-1]$. \end{theorem} \begin{proof} By Philip Hall's theorem, $\beta_P(S) = (-1)^{n}\mu_{P_{S\cup\{0,n\}}}(\zero, \one)$. The theorem now follows from Theorem~\ref{mobius}. \end{proof} \subsection{$\TN$-posets from $\TN$-matrices} Suppose $P$ is a quasi-rank uniform poset with $R(P)= (r_{n,k})_{k,n=0}^N$. Then all diagonal entries of $R(P)$ are equal to one, and $r_{n,k} \leq r_{n+1,k}$ for all $n<N$. The latter is true since if $\rho(x)=n+1$, then there exists $x' < x$ with $\rho(x')=n$. We shall now see that the converse is true. \begin{theorem}\label{TNTTNP} Let $N \in \NN \cup\{\infty\}$. Suppose $R=(r_{n,k})_{k,n=0}^N$ is a lower triangular matrix with nonnegative integer entries and all diagonal entries equal to one. If $r_{n,k} \leq r_{n+1,k}$ for all $k\leq n <N$, then there exists a quasi-rank uniform poset $P$, with a largest element $\one$, for which $R=R(P)$. \end{theorem} \begin{proof} We prove the theorem for $N \in \NN$ first. We prove by induction on $N \in \NN$ that there exists a poset $P_N$ with a greatest element $\one$ for which $R(P_N)=R$. The case when $N=0$ is trivial. Suppose true for some $N \geq 0$. Consider a matrix $R_{N+1}= (r_{n,k})_{k,n=0}^{N+1}$ satisfying the hypothesis in the statement of the theorem. By induction there is a quasi-rank uniform poset $P_N$ on $X$ with $R(P_N)= (r_{n,k})_{k,n=0}^{N}$. For each $1 \leq n \leq N$, let $x_n \in P_N$ be a specified element of quasi-rank $n$. Construct a poset $P_{N+1}$ on the disjunct union $X \cup Y_0 \cup Y_1 \cup \cdots \cup Y_{N+1}$, where $|Y_k|=r_{N+1,k}-r_{N,k}$ as follows. \begin{enumerate} \item Order $X$ as in $P_N$. \item Let $Y_{N+1}$ consist of a single element which is declared to be the new largest element $\one_{N+1}$ in $P_{N+1}$. \item Let $Y_0$ be a set of minimal elements in $P_{N+1}$ which are only related to $\one_{N+1}$. \item For $1 \leq n \leq N$, let $Y_n$ be an anti-chain in $P_{N+1}$. If $y \in Y_n$, then $y < \one_{N+1}$ and $x < y$ for all $x < x_n$ are the relations involving $y$. \end{enumerate} It follows that $P_{N+1}$ is quasi-rank uniform and $R(P_{N+1})= R_{N+1}$. The case when $N = \infty$ follows from the construction above. Clearly $P_N$ is an order ideal of $P_{N+1}$ for all finite $N$. Hence $P= \cup_{N=0}^\infty P_N$ has the desired properties. \end{proof} \subsection{Binomial and Sheffer posets} Binomial and Sheffer posets are rank uniform posets that have been studied frequently in combinatorics \cite{doubilet1972foundations,ehrenborg1995sheffer,reiner1993upper}. \emph{Binomial posets} are rank uniform posets $P$ with a least element $\zero$ for which any two intervals of the same length contain the same number of maximal chains. The \emph{factorial function} of $P$ is the function $B : \NN \to \NN$, where $B(n)$ is the number of maximal chains in an interval of length $n$ in $P$. More generally a \emph{Sheffer poset} is a rank uniform poset $P$ for which \begin{itemize} \item if $\rho(x)=\rho(y)=n$, then $[\zero, x]$ and $[\zero, y]$ have the same number $D(n)$ of maximal chains, \item if $[x,u]$ and $[y,v]$, $\zero \not \in \{x,y\}$, are two intervals of the same length $n$, then they have the same number $B(n)$ of maximal chains. \end{itemize} In particular, binomial posets are Sheffer posets, with $B=D$. \begin{example} The infinite Boolean algebra of finite subsets of $\NN$ is binomial with factorial function $B(n)=n!$. \end{example} \begin{example} Let $q$ be a prime power and let $\mathbb{B}(q)$ be the lattice of all finite subspaces of a vector space of infinite dimension over $\FFF_q$. Then $\mathbb{B}(q)$ is a binomial poset with $$B(n)= \mathbf{n} != (1 + q + \cdots + q^{n-1})\cdot (1 + q + \cdots + q^{n-2})\cdots (1+q)\cdot 1. $$ \end{example} \begin{example} \label{def-r-cube} The infinite $r$-cubical lattice, $r \in \mathbb{Z}_{>0}$, is a Sheffer poset see \cite{ehrenborg1996r}. It is defined as the infinite Cartesian product (with a least element $\hat{0}$ adjoined) $$ \mathbf{C}_r= \left( \prod_{i \geq 1} M_r \right) \sqcup \{ \hat{0} \}, $$ of copies of $M_r$, where $M_r$ is the poset formed from an antichain on $[r]=\{1,\ldots,r\}$ and a largest element denoted $z$, see Fig. \ref{fig:M_r}. The factorial functions for $\mathbf{C}_r$ are $B(n) = n!$ and $D(n)= r^{n-1}(n-1)!$, see \cite{ehrenborg1996r}. \begin{figure}[H] \centering \begin{tikzpicture}[line cap=round,line join=round,>=triangle 45,x=1cm,y=1cm] \clip(3.0,-1) rectangle (6.6,3); \draw [line width=2pt] (5,2)-- (4.5,0); \draw [line width=2pt] (5,2)-- (4,0); \draw [line width=2pt] (5,2)-- (6,0); \begin{scriptsize} \draw [fill=xdxdff] (4.5,0) circle (2.5pt); \draw [fill=xdxdff] (4,0) circle (2.5pt); \draw (4.9,0.2) node[anchor=north west] {$\cdots$}; \draw [fill=xdxdff] (6,0) circle (2.5pt); \draw [fill=ududff] (5,2) circle (2.5pt); \draw (4.8,2.5) node[anchor=north west] {$z$}; \draw (3.8,-0.3) node[anchor=north west] {$1$}; \draw (4.3,-0.3) node[anchor=north west] {$2$}; \draw (5.8,-0.3) node[anchor=north west] {$r$}; \draw (6.4,1) node[anchor=north west] {.}; \end{scriptsize} \end{tikzpicture} \caption{The Hasse diagram of $M_r$.} \label{fig:M_r} \end{figure} \end{example} \begin{proposition} \label{bin-PF} Let $P$ be a binomial poset of rank $N \in \NN \cup \{\infty\}$, with factorial function $B$. Then $P$ is $\mathrm{TN}$ if and only if the matrix $(1/B(n-k))_{n,k=0}^N$ is $\mathrm{TN}$, where $(1/B)(k) := 0$ if $k<0$. Hence if $N=\infty$, then $P$ is $\mathrm{TN}$ if and only if $\{1/B(n)\}_{n=0}^\infty$is a P\'olya frequency sequence. \end{proposition} \begin{proof} The numbers $r_{n,k}$ are given by $$ r_{n,k}= \frac {B(n)}{B(k) B(n-k)}, $$ see \cite{ehrenborg1995sheffer}. Since total nonnegativity is closed under multiplying by diagonal matrices with positive entries on the diagonals, it follows that $(r_{n,k})_{n,k=0}^\infty$ is $\mathrm{TN}$ if and only if $(1/B(n-k))_{n,k=0}^N$ is $\mathrm{TN}$. \end{proof} \begin{example} The sequence $\{1/n!\}_{n=0}^\infty$ is a P\'olya frequency sequence, since $$ \sum_{n=0}^\infty \frac{1}{n!} x^n = e^x. $$ Hence, the infinite Boolean algebra is a $\mathrm{TN}$-poset, and the matrix $\left( \binom n k \right)_{n,k=0}^\infty$ is $\mathrm{TN}$. From \eqref{pnchain} it follows that $$ p_n(t) = \sum_{k=1}^n k! S(n,k) t^k $$ as claimed in Example \ref{binomet}. \end{example} \subsection{Chain polynomials of (weak) upper homogeneous posets} Next we will apply Theorem~\ref{maint} to chain enumeration in upper homogeneous posets, a class of posets recently introduced by Stanley \cite{stanley2020upho,stanley2024theorems}. This will give a combinatorial interpretation for the polynomials in Theorem~\ref{PFT} for all integer P\'olya frequency sequencies $(a_i)_{i=0}^\infty$ for which $a_0=1$. A poset $P$ is called \emph{upper homogeneous} (upho) if it is graded, has a least element $\zero$, has finitely many elements of each rank, and is such that each upper order ideal is isomorphic to the poset itself. For example, $\NN^n$ is an upho poset. We take the opportunity to generalize this construction. Recall the definition of the quasi-rank function $\rho$ associated to a poset. A poset $P$ is called \emph{weak upper homogeneous} (wupho) if it has finitely many elements of each quasi-rank, and if for each $x,y \in P$ with $\rho(x)=\rho(y)$ $$ |\{z \in P : x \leq z \mbox{ and } \rho(z)=n \}| = |\{z \in P : y \leq z \mbox{ and } \rho(z)=n \}|, $$ for each $n \geq \rho(x)$. Clearly any upho poset is wupho. If $P$ is wupho, define a matrix $\overline{R}=\overline{R}(P)= (\overline{r}_{n,k}(P))_{n,k=0}^N$ by $$ \overline{r}_{n,k}(P) = |\{z \in P : x \leq z \mbox{ and } \rho(z)=n \}|, $$ where $x \in P$ is a fixed element of rank $k$. \begin{proposition} Let $P$ be a wupho poset, and let $n$ be a positive integer. Then the $n$th chain polynomial of $\overline{R}(P)$, see \eqref{polreceq}, is equal to \begin{equation}\label{pnupho} p_n(t)=\sum_{j \geq 1} |\{ x=x_0<x_1<\cdots<x_{j-1}<x_j : \rho(x_j) =n\}|t^j, \end{equation} where $x$ is a fixed minimal element of $P$. We call $p_n(t)$ the $n$th \emph{chain polynomial} of $P$. \end{proposition} \begin{proof} Let $f_0(t)=1$ and let $f_n(t)$, $n\geq 1$, be the polynomial on the right hand side of \eqref{pnupho}. Then, by conditioning on $\rho(x_{j-1})=k$, $$ f_n(t) = t\sum_{k=0}^{n-1}\overline{r}_{n,k}(P) f_k(t), $$ and hence $(f_n(t))$ satisfies the same recursion as $(p_n(t))$. \end{proof} \begin{definition} We say that a wupho poset $P$ is $\mathrm{TN}$ if the matrix $\overline{R}(P)$ is totally nonnegative. \end{definition} Hence an upho poset is $\mathrm{TN}$ if and only if it is Schur positive as defined in \cite{fu2024monoid}. In this case $\overline{R}(P)$ is a Toeplitz matrix $\overline{R}(P)= (r_{n-k}(P))_{n,k=0}^\infty$, where $r_n(P)= |\{x \in P: \rho(x)=n\}|$. Theorem~\ref{maint} immediately gives. \begin{theorem}\label{uphot} Let $P$ be a $\mathrm{TN}$ poset. Then, for each $n \geq 0$, all the zeros of the $n$th chain polynomial $p_n(t)$ are real and located in the interval $[-1,0]$. Moreover $p_n(t) \prec p_{n+1}(t)$. \end{theorem} Hopkins \cite{hopkins2024upho} constructs upho lattices from certain supersolvable lattices. These are all Schur positive. It was proved in \cite{fu2024monoid} that any P\'olya frequency sequence $(r_n)_{n=0}^\infty$ of positive integers such that $r_0=1$ is the rank sequence $(r_n(P))_{n=0}^\infty$ of an upho posets $P$. Does this extend to wupho posets? \begin{problem}\label{TNTUTNP} Suppose $\overline{R}= (\overline{r}_{n,k})_{n,k=0}^N$ is a matrix with nonnegative integer entries and all diagonal entries equal to one. If $\overline{R}$ is $\TN$ and $\overline{r}_{n, k} \leq \overline{r}_{n, k+1}$ for all $0\leq k <n\leq N$, then does there exist a wupho poset $P$ for which $\overline{R}(P)=\overline{R}$? \end{problem} The analog of Problem \ref{TNTUTNP} for quasi-rank uniform posets is true (even without the $\TN$-assumption) by Theorem \ref{TNTTNP}. \section{Rank uniform posets with real-rooted chain polynomials} \label{section-subdivision} For simplicial complexes, it is often convenient to study the $h$-vector instead of the $f$-vector of the simplicial complex. Let $\Delta$ be an abstract simplicial complex (we consider the empty set a face of $\Delta$). The $f$-\emph{polynomial} of $\Delta$ is defined by $$ f_\Delta(t) = \sum_{S \in \Delta}t^{\# S}. $$ If $\Delta$ has dimension $n-1$, then the $h$-\emph{vector} of $\Delta$ may be defined as the sequence $(h_0(\Delta), h_1(\Delta), \ldots, h_n(\Delta))$ for which $$ f_\Delta(t) = \sum_{k=0}^n h_k(\Delta) t^k(1+t)^{n-k}. $$ Hence the polynomials $t^k(1+t)^{n-k}$ play a crucial role in the study of chain polynomials of simplicial complexes. Brenti and Welker \cite{brenti2008f} proved that if the $h$-vector of a simplicial complex $\Delta$ is nonnegative, then the chain polynomial of $\Delta$ has only real zeros. Notice that the polynomials $t^k(1+t)^{n-k}$ are the polynomials $R_{n,k}(t)$ associated to the $\mathrm{TN}$ matrix $\left( \binom n k \right)_{n,k=0}^\infty$, which is the matrix $R(\mathbb{B})$ associated to the $\mathrm{TN}$-poset $\mathbb{B}$. In this section we will prove a vast generalization of Brenti and Welker's result, and in the process define natural analogs of $h$-vectors for all $\mathrm{TN}$-posets. In this section all posets are assumed to have a least element $\zero$. \begin{definition}Let $P$ be a $\mathrm{TN}$-poset and let $(R_{n,k}(t))_{0\leq k \leq n \leq N}$ be the array of polynomials associated to the matrix $R= R(P)$, see Definition \ref{resolv} and Remark~\ref{thethe}. Let $Q$ be a quasi-rank uniform poset of rank $r$. \begin{itemize} \item $Q$ is a \emph{$P$-poset} if for each $x \in Q$ there is a $y \in P$ such that $\{z \in Q : z \leq x\}$ is isomorphic to $\{z \in P : z \leq y\}$. \item $Q$ is a \emph{combinatorial $P$-poset} if for each $y \in Q$, \begin{equation}\label{QaR} \sum_{x \leq_Q y} t^{\nu(x)}= R_{m,0}(t), \ \ \ \ m=\nu(y), \end{equation} where $\nu$ is the quasi-rank function of $Q$. \item A combinatorial $P$-poset $Q$ of rank $r$ is said to be \emph{$P$-positive} if its rank generating polynomial, $$ f_{Q}(t)= \sum_{x \in Q} t^{\nu(x)}, $$ has a nonnegative expansion in the polynomials $\{R_{r,k}(t)\}_{k=0}^{r}$, i.e., $$ f_{Q}(t) = \sum_{k=0}^r h_k(Q) \cdot R_{r,k}(t), $$ where $h_k(Q) \geq 0$ for all $0 \leq k \leq r$. The \emph{$h$-vector} of $Q$ is the vector $(h_0(Q), h_1(Q), \ldots, h_r(Q))$. \end{itemize} \end{definition} \begin{remark} Clearly any $P$-poset is a combinatorial $P$-poset. Notice that a combinatorial $P$-poset may fail to have an $h$-vector, the reason being that $\{R_{r,k}(t)\}_{k=0}^r$ does not have to be a basis for the space of polynomials of degree at most $r$. However, for the $\mathrm{TN}$-posets considered in this paper it will be a basis. \end{remark} \begin{example} Examples of $\mathbb{B}$-positive posets are boolean cell complexes with nonnegative $h$-vectors, for example Cohen-Macaulay simplicial complexes. \end{example} If $P$ is a finite poset, let $\hat{P}$ be the poset obtained by adding a new largest element $\one$ to $P$. If $P$ is quasi-rank uniform, then so is $\hat{P}$. \begin{theorem}\label{TN-Ppos} Suppose $P$ is a $\mathrm{TN}$-poset, and that $Q$ is a (combinatorial) $P$-poset. Then $Q$ is $P$-positive if and only if $\hat{Q}$ is $\mathrm{TN}$. \end{theorem} \begin{proof} Suppose $\hat{Q}$ is $\mathrm{TN}$ of rank $n+1$. Then the matrix $R(\hat{Q})$ is resolvable by Theorem~\ref{eqcon}. Let $\{\hat{R}_{k,j}(t)\}$ be the polynomials that resolve $R(\hat{Q})$, and $\{R_{k,j}(t)\}_{k,j}$ the polynomials that resolve $R(P)$. Then $\hat{R}_{k,j}(t)= R_{k,j}(t)$ whenever $k \leq n$, and $$ f_{Q}(t)= \hat{R}_{n+1,0}(t)- t^{n+1}= \sum_{j=0}^n \lambda_{n,j}(\hat{Q}) \cdot R_{n,j}(t), $$ by \eqref{r-sum}. Hence $Q$ is $P$-positive. Conversely if $Q$ is $P$-positive, then $$ f_{Q}(t)= \sum_{j=0}^n h_j \cdot R_{n,j}(t), $$ for nonnegative numbers $h_j$. Define $\lambda_{k,j}(\hat{Q}) = \lambda_{k,j}(P)$ for $0\leq j \leq k \leq n-1$, and $\lambda_{n,j}(\hat{Q})=h_j$ for $0 \leq j \leq n$. Define further $\hat{R}_{k,j}(t)=R_{k,j}(t)$ for $0\leq j \leq k \leq n$, and $$ \hat{R}_{n+1,k}(t)= t^{n+1}+ \sum_{j \geq k} h_jR_{n,j}(t). \ \ \ \ 0\leq k \leq n. $$ It follows that $R(\hat{Q})$ is resolvable. \end{proof} \begin{example} Suppose $P$ is a quasi-rank uniform $\mathrm{TN}$-poset and $y \in P$ has quasi-rank $n$. For $d \leq n$ consider the poset $$ Q=\{x \leq y : \rho(x) \leq d\}. $$ Then $Q$ is $P$-positive by Theorem~\ref{TN-Ppos} and Proposition~\ref{propranksel}. \end{example} The \emph{chain polynomial} of a finite poset $Q$ is the generating polynomial $$ c_Q(t)= \sum_{C}t^{|C|} $$ of the collection of all chains $C$ in $Q$. Notice that if $Q$ is quasi-rank uniform of rank $n$, and has a least element $\zero$, then $$ c_Q(t)= t^{-1}(1+t) p_{n+1}(t) $$ where $p_{n+1}(t)$ is the $(n+1)$st chain polynomial of $\hat{Q}$. Theorem~\ref{rposp} below is a vast generalization of the theorem of Brenti and Welker \cite{brenti2008f} who proved the analog of Theorem~\ref{rposp} for the case when $P= \mathbb{B}$. \begin{theorem} \label{rposp} Let $P$ be a $\mathrm{TN}$-poset with a least element $\zero$. All zeros of the chain polynomial of any $P$-positive poset $Q$ are located in the interval $[-1,0]$. \end{theorem} \begin{proof} Follows from Theorems \ref{TN-poset-chain} and \ref{TN-Ppos}. \end{proof} The next theorem is an immediate consequence of Theorems \ref{propranksel} and \ref{TN-Ppos}. \begin{theorem} \label{rpossel} Let $P$ be a $\mathrm{TN}$-poset with a least element $\zero$, and let $S$ be an infinite set of nonnegative integers containing $0$. If $Q$ is a $P$-positive poset, then $Q_S$ is $P_S$-positive. \end{theorem} Hence if $P$ is a quasi-rank uniform $\mathrm{TN}$-poset with a least element $\zero$ and $Q$ is a $P$-positive poset, then the chain polynomial of any rank-selected sub-poset of $Q$ is real-rooted. \begin{example} For the case when $P = \mathbb{B}$, Theorems \ref{rposp} and \ref{rpossel} imply that if $Q$ is a boolean poset (or a simplicial complex) with nonnegative $h$-vector, then the chain polynomial of any rank selected sub-poset of $Q$ is real-rooted. This was first proved in \cite{athanasiadis2023two}. \end{example} \section{Subspace lattices and the critical problem of Crapo and Rota} \label{sec-critical} \newcommand{\LLL}{\mathcal{L}} Let $\FFF_q$ be a finite field, and let $\mathbb{B}_d(q)$ be the lattice of all subspaces of $\FFF_q^d$. Let further $\mathbb{B}(q)$ be the lattice of all finite dimensional subspaces of $\FFF_q^\NN$. Then $\mathbb{B}(q)$ is a binomial poset, and if $y \in \mathbb{B}(q)$ has rank (dimension) $n$, then $$ |\{ x \leq y : \rho(x)=k \}| = {\binom n k}_{\! \! \! q}, $$ where ${\binom n k}_{\! q}$ is a $q$-binomial number \cite[Chapter 1.7]{stanley2011enumerative}. Rota introduced \emph{$q$-posets} in \cite{Rota71}, and Alder studied them further in~\cite{alder2010q}. In our terminology $q$-posets are the same as $\mathbb{B}(q)$-posets. Let $P$ be a $q$-poset. If $\rho(y)=n$, then $$ R^q_n(t)= \sum_{x \leq y} t^{\rho(x)} = \sum_{k=0}^n {\binom n k}_{\! \! \! q} t^k. $$ These polynomials are known as the Rogers-Szeg\H{o} polynomials \cite[Chapter 3]{szego1926beitrag}, and they satisfy the recursion ${R}^q_0 (t) =1$, and $$ {R}_{n+1}^q(t) = t{R}_n^q(t)+ {R}_n^q(qt), \ \ \ \ \mbox{ for } n \geq 0. $$ Let $\alpha \colon \RR[t] \to \RR[t]$ be the diagonal linear operator defined by $ \alpha(f)(t) = f(qt). $ Hence $ {R}_n^q(t) = (t+\alpha)^n 1, $ which by Lemma~\ref{eqcon} proves the well known fact that the matrix $\left({\binom n k}_{\! q}\right)_{n,k=0}^\infty$ is $\mathrm{TN}$ for $q>0$. Recall that $ R^q_{n,k}(t)= (t+\alpha)^{n-k}t^k 1, $ and that by Lemma~\ref{eqcon}, \begin{proposition} \label{SSRrec} If $0\leq k < n+1$, then \begin{equation}\label{RS1} R^q_{n+1,k+1}(t)=R^q_{n+1,k}(t) - q^kR^q_{n,k}(t). \end{equation} \end{proposition} \begin{remark} It is not hard to prove using \eqref{RS1} that $R^q_{n,k}(t)= q^{k(n-k)} t^k R^q_{n-k}(q^{-k}t)$, but since we will not use it here we leave the proof to the reader. \end{remark} Denote by $\EE_q$, the subdivision operator associated to $({\binom n k}_{\! q})_{n,k=0}^\infty$, see Definition~\ref{subop}. Notice that the definition of $R^q_{n,k}(t)$ makes sense for any nonnegative real number $q$. \begin{theorem} \label{mainssl} Let $q$ be a nonnegative real number. The sequence $\{R^q_{n}(t)\}_{n=0}^\infty$ is resolvable. In particular, for each $n \geq 0$, $\{\EE_q(R^q_{n,k}(t))\}_{k=0}^n$ is an interlacing sequence of polynomials whose zeros lie in $[-1,0]$. \end{theorem} \begin{proof} The theorem now follows from Theorem~\ref{mainUTP} since $({\binom n k}_{\! q})_{n,k=0}^\infty$ is $\mathrm{TN}$. \end{proof} \subsection{The critical problem and expansions of characteristic polynomials} To apply Definition \ref{resolv} and Theorem~\ref{mainssl}, we want to have an understanding for when $q$-posets are $\mathbb{B}(q)$-positive. It turns out that this is closely related to positivity questions regarding characteristic polynomial of hyperplane arrangements in $\FFF_q^n$, and the \emph{critical problem} of Crapo and Rota \cite[Chapter 16]{crapo1970foundations}. Let $\HH = \{H_1,\ldots, H_m\}$ be a collection (list) of hyperplanes in $\mathbb{B}_n(q)$. Recall that the \emph{characteristic polynomial} of $\HH$ is $$ \chi_\HH(t) = \sum_{A \subseteq \{1,\ldots,m\}} (-1)^{|A|} t^{\dim\left( \cap_{i \in A} H_i\right)}. $$ For $0\leq k \leq n$ and $q \in \RR$, define polynomials $\chi^q_{n,k}(t)$ by $\chi^q_{n,0}(t)=t^n$, and $$ \chi^q_{n,k}(t)= t^{n-k}(t-1)(t-q)\cdots (t-q^{k-1}), $$ otherwise. The reason for why these polynomials are important for us is the next lemma. Define a bijective linear operator $R^q : \RR[t] \to \RR[t]$ by $R^q(t^n)=R_n^q(t)$ for each $n \in \NN$. \begin{lemma}\label{chitor} For each $0 \leq k \leq n$, $ R^q(\chi_{n,k})(t) =R^q_{n,k}(t). $ \end{lemma} \begin{proof} A direct computation proves the recursion $$ \chi^q_{n,k+1}(t)=\chi^q_{n,k}(t)-q^k\chi^q_{n-1,k}(t), $$ from which the lemma follows from Proposition~\ref{SSRrec} by induction on $k$. \end{proof} In light of Definition \ref{TN-poset} and Lemma~\ref{chitor} one might ask for which hyperplane arrangements $\HH$ in $\FFF_q^n$, the characteristic polynomial $\chi_\HH$(t) has a nonnegative expansion in the polynomials $\{\chi^q_{n,k}(t)\}_{k=0}^n$. We will prove that all do. In particular the following theorem holds for all chromatic polynomials of graphs. \begin{theorem}\label{posexpchi}Let $\HH$ be a hyperplane arrangement in $\FFF_q^n$. The characteristic polynomial of $\HH$ has a unique expansion $$ \chi_\HH(t)= \sum_{k=0}^{n} \theta_k(\HH) \cdot \chi^q_{n,k}(t), $$ where $\theta_k(\HH) \geq 0$ for all $k$, and $\theta_0(\HH)+\theta_1(\HH)+\cdots+\theta_{n}(\HH)=1$. \end{theorem} Theorem~\ref{posexpchi} has the following consequence. \begin{corollary}\label{ch-cor} The convex hull of the set of all characteristic polynomials of hyperplane arrangements in $\FFF_q^n$ is equal to the simplex $$ \left\{ \sum_{k=0}^{n} \theta_k \cdot \chi^q_{n,k}(t) : \theta_k \geq 0 \mbox{ for all } k, \mbox{ and } \theta_0 + \theta_1+ \cdots +\theta_{n}=1\right\}. $$ \end{corollary} \begin{proof} By Theorem~\ref{posexpchi} the convex hull of the characteristic polynomials is a subset of the simplex. The converse follows by the fact that if $U$ is a subspace of $\FFF_q^n$ of dimension $k$, then the characteristic polynomial of the arrangement $$ \HH= \{H : \mbox{ the normal of $H$ is in $U$}\} $$ is equal to $\chi^q_{n,k}(t)$. This may be proved using the critical problem, see Theorem~\ref{crita}. \end{proof} To prove Theorem~\ref{posexpchi} we recall the critical problem of Crapo and Rota. Recall that the \emph{matroid} associated to the hyperplane arrangement $\HH=\{H_1,\ldots, H_m\}$ is the (ordered) set $\MM= \{ n_1,\ldots, n_m\} \subseteq \FFF_q^n \setminus \{(0)\}$, where $n_i$ is a normal vector of $H_i$ for each $i$. We write $\chi_\MM(t)$ for $\chi_\HH(t)$. \begin{theorem}\cite[Thm 1 in Ch. 16]{crapo1970foundations} \label{crita} Let $\HH$ be a hyperplane arrangement in $\FFF_q^n$, and let $m \in \NN$. Then $$ \chi_{\MM}(q^m) = |\{ \varphi \in \Hom(\FFF_q^n, \FFF_q^m) : \ker(\varphi)\cap \MM= \varnothing\}|, $$ Moreover if $F$ is a flat of $\MM$, then \begin{equation}\label{q-flats} \chi_{\MM/F}(q^m) = |\{ \varphi \in \Hom(\FFF_q^n, \FFF_q^m) : \ker(\varphi)\cap \MM= F\}|, \end{equation} where $\MM/F$ is considered a subset of $\FFF_q^n / \langle F\rangle \cong \FFF_q^{n-r(F)}$. \end{theorem} For $n \in \NN$, let $\mathsf{D}_n \colon \RR[t] \to \RR[t]$ be the linear operator defined by $$ (\mathsf{D}_n f)(t) = f(qt)-q^nf(t), $$ and notice that $\mathsf{D}_n \chi^q_{n,0}(t) \equiv 0$, and $$ \mathsf{D}_n \chi^q_{n,k}(t)= q^{n-1}(q^k-1)\chi^q_{n-1,k-1}(t), \ \ \ \mbox{ for } 1\leq k \leq n. $$ Also, $$ \chi_\HH(1)= \theta_0(\HH)= \begin{cases} 1 &\mbox{ if } \HH = \varnothing, \mbox{ and } \\ 0 &\mbox{ otherwise} \end{cases} . $$ Hence to prove that $\chi_\HH(t)$ has a nonnegative expansion in the polynomials $\chi^q_{n,k}(t)$ it suffices to prove that $\mathsf{D}_n \chi_\HH(t)$ may be expressed as a nonnegative sum of characteristic polynomials $\chi_{\HH'}(t)$ of hyperplane arrangements $\HH'$ in $\FFF_q^{n-1}$. Hence we want to prove \begin{theorem}\label{fundn}Let $\MM \subseteq \FFF_q^n \setminus \{(0)\}$ be nonempty. Then $$ \mathsf{D}_n\chi_\MM(t) = \sum_{\MM'} \lambda(\MM') \cdot \chi_{\MM'}(t), $$ where $\MM' \subseteq \FFF_q^{n-1} \setminus \{(0)\}$ and $\lambda(\MM') \geq 0$. \end{theorem} \begin{lemma}\label{Dn-form}Let $\MM \subseteq \FFF_q^n \setminus \{(0)\}$ be nonempty. Then $$ \mathsf{D}_n\chi_\MM(t)= \sum_{F} \chi_{\MM/F}(q)\cdot \left( \chi_F(t) -\chi_\MM(t)\right), $$ where the sum is over all flats $F$ of $\MM$. \end{lemma} \begin{proof} Let $C_\MM(n,m)= \{ \varphi \in \Hom(\FFF_q^n, \FFF_q^m) : \ker(\varphi)\cap \MM= \varnothing\}$ and consider the injective map $$ \Phi : C_\MM(n,m)\times \Hom(\FFF_q^n , \FFF_q) \to C_\MM(n,m+1) $$ given by $$ \Phi(\varphi, \lambda)(x)= \varphi(x)+ \lambda(x)e_{m+1}, $$ where $e_{m+1}$ is the $(m+1)$st standard basis vector of $\FFF_q^{m+1}$. Then the number of elements in $C_\MM(n,m+1)$ which are not in the image of $\Phi$ is $$ \chi_\MM(q^{m+1})- q^n \chi_\MM(q^{m}) = (D_n\chi_\MM)(q^m), $$ by Theorem~\ref{crita}. Hence $D_n\chi_\MM(q^m)$ is equal to the number of pairs $(\varphi,\lambda)$, where $\varphi \in \Hom(\FFF_q^n, \FFF_q^m)$ and $\lambda \in \Hom(\FFF_q^n, \FFF_q)$ are such that \begin{itemize} \item $\ker(\varphi) \cap \MM \neq \varnothing$, and \item $\ker(\lambda) \cap \ker(\varphi) \cap \MM = \varnothing$. \end{itemize} Let us count the number of such $(\varphi,\lambda)$. Suppose $\ker(\lambda) \cap \MM= F \neq \MM$. The number of such $\lambda$ is $\chi_{\MM/F}(q)$ by Theorem~\ref{crita}. For $\varphi$ we have $\varnothing \neq \ker(\varphi)\cap \MM \subseteq \MM \setminus F$. Thus the number of such $\varphi$ is $\chi_{F}(q^m)- \chi_\MM(q^m)$, and hence $$ \mathsf{D}_n\chi_\MM(q^m)= \sum_{F < \MM} \chi_{\MM/F}(q)\cdot \left( \chi_F(q^m) -\chi_\MM(q^m)\right), $$ from which the lemma follows since the identity holds for all positive $m$. \end{proof} \begin{lemma}\label{dccon} If $A \subseteq B \subseteq \FFF_q^n \setminus \{(0)\}$, then $$ \chi_A(t)-\chi_B(t) = \sum_{C} \chi_C(t), $$ where the sum is over a collection of $C \subseteq \FFF_q^{n-1} \setminus \{(0)\}$. \end{lemma} \begin{proof} Let $e \in B \setminus A$. By deletion/contraction, $$ \chi_B(t)= \chi_{B \setminus e}(t)-\chi_{B/e}(t), $$ where $\chi_{B/e}(t)$ is the characteristic polynomial of an arrangement in $\FFF_q^{n-1} \setminus \{(0)\}$. Hence $$ \chi_A(t)-\chi_B(t) = (\chi_A(t)-\chi_{B \setminus e}(t))+\chi_{B/e}(t), $$ from which the lemma follows by iterating. \end{proof} Theorem~\ref{fundn} now follows from Lemma~\ref{Dn-form} and Lemma~\ref{dccon}. \subsection{Shellable \texorpdfstring{$q$}--posets} \newcommand{\UU}{\mathcal{U}} The next definition is a straightforward generalization of shellable simplicial complexes to $q$-posets. It was studied first by Alder \cite{alder2010q}. A finite ranked poset $P$ is \emph{pure} if all maximal elements of $P$ have the same rank. We call the maximal elements of $P$ \emph{facets}. For $y \in P$, let $\langle y \rangle = \{x \in P : x\leq y\}$. \begin{definition} Let $P$ be a pure $q$-poset of rank $n$. Suppose the facets of $P$ admit an ordering $F_1,\ldots, F_m$ such that for each $k>1$, $$ \left(\langle F_1\rangle \cup \cdots \cup \langle F_{k-1} \rangle \right) \cap \langle F_k \rangle $$ is pure of rank $n-1$. Then we say that $P$ is \emph{shellable}, and that the ordering is a \emph{shelling}. \end{definition} For example if $H_1, \ldots, H_m$ are distinct hyperplanes of $\mathbb{B}_{n+1}(q)$, then $$P= \langle H_1\rangle \cup \cdots \cup \langle H_{\ell} \rangle$$ is shellable, and any ordering of the hyperplanes is a shelling. This is because the intersection of any two hyperplanes is a subspace of dimension $n-1$. \begin{theorem}\label{expU} If $P$ is a shellable $q$-poset, then $P$ is $\mathbb{B}(q)$-positive. In particular the chain polynomials of $P$ and its rank selected sub-posets are real-rooted. \end{theorem} \begin{proof} Suppose $P$ has rank $n$, and let $F_1,F_2,\ldots, F_m$ be a shelling of $P$. Let $P_k= \langle F_1\rangle \cup \cdots \cup \langle F_{k} \rangle$. Then $f_{P_1}(t)=R_n(t)$. With the notation above, we may write $P_k$ as the disjoint union $$ P_k= P_{k-1}\cup Q_k, $$ where $$ Q_k= \langle F_k \rangle \setminus \big(\langle H_1\rangle \cup \cdots \cup \langle H_{\ell} \rangle\big), $$ and where $H_i$ is a facet of $F_k$ for each $i$. We prove by induction on $k$ that $P_k$ is $\mathbb{B}(q)$-positive, the case when $k=1$ being obvious. Let $k>1$. Since $\langle F_k \rangle \cong \mathbb{B}_n(q)$ we may assume $F_k=\mathbb{B}_n(q)$ in the following computation. By Inclusion-Exclusion, $$ \sum_{x \in Q_k} t^{\dim(x)}= \sum_{S \subseteq [m]} (-1)^{|S|}\left(\sum_{x \in \cap_{i \in S}\langle H_i \rangle }t^{\dim(x)} \right)= R_q(\chi_\HH)(t). $$ Hence $$ f_{P_k}(t)=f_{P_{k-1}}(t)+ R(\chi_\HH)(t), $$ and the theorem follows from Theorem~\ref{posexpchi}, Lemma~\ref{chitor} and Theorem~\ref{rpossel}. \end{proof} In \cite{alder2010q}, Alder raised the question of finding a suitable notion of $h$-vectors for $q$-posets $P$. In particular it should have the property that the $h$-vector is nonnegative whenever $P$ is shellable, and it should specialize to the common $h$-vector for simplicial complexes when $q=1$. Theorem~\ref{expU} offers such a notion: if $P$ is a $q$-poset of rank $n$, then the $h$-vector of $P$ is defined as the unique vector $(h_0(P), h_1(P), \ldots, h_n(P))$ for which $$ f_P(t) = \sum_{k=0}^n h_k(P) \cdot R^q_{n,k}(t). $$ An interesting class of shellable $q$-posets are related to $q$-matroids \cite{jurrius2018defining}. \begin{definition} A $q$-\emph{matroid} on $\mathbb{B}_n(q)$ is a function $\varphi : \mathbb{B}_n(q) \to \NN$ satisfying \begin{enumerate} \item $\varphi(x) \leq \dim(x)$, for all $x \in \mathbb{B}_n(q)$, and \item $\varphi(x) \leq \varphi(y)$, for all $x \leq y \in \mathbb{B}_n(q)$, and \item $\varphi(x\vee y)+\varphi(x \wedge y) \leq \varphi(x) + \varphi(y)$, for all $x,y \in \mathbb{B}_n(q)$. \end{enumerate} \end{definition} For example if $0 \leq r \leq n$, then $\varphi : \mathbb{B}_n(q) \to \NN$ defined by $$ \varphi(x)= \min(\dim(x),r) $$ is a $q$-matroid. The \emph{set of independent spaces} of a $q$-matroid on $\mathbb{B}_n(q)$ is $$ P(\varphi)= \{x \in \mathbb{B}_n(q) : \varphi(x)= \dim(x)\}. $$ This poset is a pure order ideal of $\mathbb{B}_n(q)$, and hence a pure $q$-poset. Moreover \begin{theorem}[\cite{Ghorpade}] \label{qmth} If $\varphi$ is a $q$-matroid, then $P(\varphi)$ is shellable. \end{theorem} The rank of a $q$-matroid $\varphi$ on $\mathbb{B}_n(q)$ is $\max\{ \varphi(x) : x \in \mathbb{B}_n(q)\}$. From Theorems \ref{expU}, \ref{qmth} and \ref{rpossel} we deduce \begin{theorem}\label{qmrr} If $\varphi$ is a $q$-matroid, then $P(\varphi)$ is $\mathbb{B}(q)$-positive. In particular the chain polynomials of $P(\varphi)$ and its rank selected sub-posets are real-rooted. \end{theorem} \section{\texorpdfstring{$r$}--cubical posets} \label{sec-r-cubical} In this section we study $r$-cubical lattices and posets, see \cite{ehrenborg1996r}. Let $r$ be a positive integer. Recall the definition of $r$-cubical lattice $\mathbf{C}_r$ in Example \ref{def-r-cube}. If $x \in \mathbf{C}_r$ has rank $n$, then we call the interval $[\zero, x]$ an $r$-\emph{cube} of rank $n$ and denote it by $C_{n,r}$. The rank generating polynomials of $\mathbf{C}_r$ are $ R_n (t) = 1 + t(r+t)^{n-1}. $ Define \begin{equation} \label{rec-r-cube} R_{n,k}(t) = \begin{cases} R_n(t) &\mbox{ if } k = 0, \\ (r-1+t)t^k(r+t)^{n-k-1} &\mbox{ if } 0 < k < n, \\ t^n &\mbox{ if } k =n. \end{cases}\, \end{equation} The next lemma proves that $\mathbf{C}_r$ is a $\mathrm{TN}$-poset. \begin{lemma} \label{matrix-r-cube} The matrix $R(\mathbf{C}_r)$ is totally nonnegative. Moreover $$ R_{n+1,k}(t)=R_{n+1,k+1}(t)+ \lambda_{n,k} R_{n,k}(t), \ \ \ \ 0 \leq k \leq n, $$ where $$ \lambda_{n,k} = \begin{cases} 1 &\mbox{ if } k=0, \\ r &\mbox{ if } 1 \leq k < n, \\ r-1 &\mbox{ if } k = n. \end{cases}\, $$ \end{lemma} \begin{proof} It is straightforward to check that the polynomials $R_{n,k}(t)$ satisfy the stated recursion, and hence $R(\mathbf{C}_r)$ is resolvable. The lemma now follows from Theorem~\ref{eqcon}. \end{proof} We call $\mathbf{C}_r$-posets \emph{$r$-cubical posets}. A $2$-cubical poset is called \emph{cubical}. In \cite{adin1996new} Adin introduced an $h$-vector for cubical posets. Let $f_P(t)$ be the rank generating polynomial of a cubical poset $P$ of rank $n$, i.e., $$ f_P(t)= \sum_{x \in P} t ^{\rho(x)}. $$ The (normalized) cubical $h$-\emph{vector} \cite{adin1996new, athanasiadis2021face, ehrenborg2000flags} of $P$ may be defined as the vector $(h_0(P), h_1(P), \ldots, h_n(P))$ for which \begin{equation} \label{q-h-vector} \begin{split} (1+t) h_P(t) = &(-1)^{n} f_P(-1)t^{n+1} + f_P(0) \left( 1 - \left( \frac{1-t}{2} \right)^{n} \right) + \\ &\left( \frac{1-t}{2} \right)^{n} f_P \left( \frac{2t}{1-t} \right), \end{split} \end{equation} where $h_P(t)= \sum_{i=0}^nh_i(P)t^i$. \begin{theorem}\label{q-h-pos} Let $P$ be cubical poset. Then $P$ has nonnegative cubical $h$-vector if and only if $P$ is $\mathbf{C}_2$-positive. \end{theorem} \begin{proof} Notice that \eqref{q-h-vector} defines a unique bijective linear operator $H : \RR_n[t] \to \RR_n[t]$ satisfying $H(f_P)(t) = h_P(t)$ for all cubical poset $P$ of rank $n$. It is straightforward to check that $$ H(R_{n,k})(t)= \begin{cases} t^k/2, &\mbox{ if } 0<k<n, \\ t^k, &\mbox{ otherwise,} \end{cases} $$ from which the theorem follows. \end{proof} Adin \cite{adin1996new} proved that shellable cubical complexes have nonnegative cubical $h$-vectors. Athanasiadis \cite{athanasiadis2021face} proved that the $f$-polynomial of the barycentric subdivision of any cubical complex with nonnegative cubical $h$-vector is real-rooted, thus confirming a conjecture of Brenti, Mohammadi and Welker \cite{brenti2008f}. An alternative proof of Athanasiadis' result now follows directly from \begin{theorem} If $P$ is a cubical poset with nonnegative cubical $h$-vector, then the chain polynomial of $P$ is real-rooted. Moreover if $S$ is a set of positive integers, then the chain polynomial of $P_S$ is real-rooted. \end{theorem} \begin{proof} Apply Theorems \ref{rposp}, \ref{rpossel} and \ref{q-h-pos}. \end{proof} Lemma~\ref{matrix-r-cube} and Theorem~\ref{q-h-pos} suggest how to define a suitable $r$-cubical $h$-vector. \begin{definition} \label{qh-def} Let $P$ be an $r$-cubical poset of rank $n$. The $r$-cubical $h$-vector of $P$ is the unique vector $(h_0(P),h_1(P), \ldots, h_n(P))$ for which $$ f_P(t) = \sum_{k=0}^n h_k(P)\cdot R_{n,k}(t), $$ where $R_{n,k}(t)$ is defined by \eqref{rec-r-cube} for the matrix $R=R(\mathbf{C}_r)$. \end{definition} By the proof of Theorem~\ref{q-h-pos} we see that for $r=2$, the entries of the cubical $h$-vector \eqref{q-h-vector} and Definition \ref{qh-def} agree up to a factor of $2$. In light of the Definition \ref{qh-def}, we want to extend the definition of shellable cubical complexes to pure $r$-cubical posets. Let $P$ be a pure $r$-cubical poset of rank $n$, and let $F_1, \ldots, F_m$ be a total ordering of the facets of $P$. Let $P_k = \langle F_1 \rangle \cup \langle F_2 \rangle \cup \cdots \cup \langle F_k \rangle$. A condition that is required in all sensible definitions of shellings is \begin{itemize} \item[$(\mathrm{S}_1)$] For each $k \geq 2$, $P_{k-1} \cap \langle F_k \rangle $ is pure of rank $n-1$. \end{itemize} Identify $\langle F_k \rangle$ with $C_{n,r}$. Each facet of $P_{k-1} \cap \langle F_k \rangle$ is of the form $(z,\ldots, z, \alpha_i, z,\ldots, z)$ where $\alpha_i \in [r]$ is in the $i$th position for some $1\leq i \leq n$. The \emph{type} of the $k$th step is the vector $(a_0,a_1, \ldots, a_r)$ where \begin{align*} a_j = &\mbox{ the number of $i \in [n]$ for which there are exactly $j$ facets of } \\ &\mbox{ $P_{k-1} \cap \langle F_k \rangle$ with a ``non $z$'' in the $i$th position.} \end{align*} The following definition extends the definition of shellable cubical complexes, see \cite{adin1996new}, to $r$-cubical posets. \begin{definition} Let $P$ be a pure $r$-cubical poset of rank $n$. The ordering $F_1, \ldots, F_m$ is a \emph{shelling} if $(\mathrm{S}_1)$ is satisfied, and \begin{itemize} \item[$(\mathrm{S}_2)$] For each $1< k \leq m$ the type of the $k$th step either satisfies $a_1 \geq 1$ or $a_r=n$. \end{itemize} Then we say that $P$ is \emph{shellable}. \end{definition} Notice that $a_0+a_1+\cdots + a_r=n$. \begin{theorem} Let $P$ be a pure $r$-cubical poset of rank $n$. If $P$ is shellable, then $P$ has a nonnegative $r$-cubical $h$-vector. In particular, $P$ is $\mathbf{C}_r$-positive. \end{theorem} \begin{proof} Let $F_1,\ldots, F_m$ be a shelling of $P$. We prove that the $r$-cubical $h$-vector of $P_k$ is nonnegative by induction over $k$. If $k=1$, then $f_{P_k}(t)= R_{n,0}(t)$ and hence the $h$-vector is $(1,0,0,\ldots)$. Let $k \geq 2$. Then $$ f_{P_k}(t)= f_{P_{k-1}}(t)+ \sum_{x \in \langle F_k \rangle \setminus (P_{k-1} \cap \langle F_k \rangle)} t^{\rho(x)}. $$ By induction $f_{P_{k-1}}(t)$ has a nonnegative expansion in the polynomials $R_{n,k}(t)$. It remains to prove that the second summand, which we name $S_2(t)$, also does. Identify $\langle F_k \rangle$ with $C_{n,r}$. For $1 \leq i \leq n$, let $A_i$ be the set of all $a \in [r]$ for which there is a maximal element of $P_{k-1} \cap \langle F_k \rangle$ that have an $a$ in the $i$th coordinate. Let $x=(x_1,\ldots, x_n) \in \langle F_k \rangle$. Then $x \in \langle F_k \rangle \setminus (P_{k-1} \cap \langle F_k \rangle)$ if and only if $x_i \in \{z\} \cup [r] \setminus A_i$ for all $i$. Hence $$ S_2(t)= \prod_{i=1}^n (t+ |[r]\setminus A_i|)= \prod_{i=0}^r(t+r-i)^{a_i}, $$ where $(a_0,\ldots, a_r)$ is the type of the $k$th step. If $a_r=n$, then the second summand is $t^n = R_{n,n}(t)$. Suppose $a_1 \geq 1$. Since $\zero \in P_{k-1} \cap \langle F_k \rangle$ we may write $S_2(t)= t(t+r-1)g(t)$, where $g(t)$ is a product of factors of the form $t+i$, where $0 \leq i \leq r$. Since $$ t+i = \frac {r-i} r t + \frac i r (t+r), $$ $g(t)$ has a nonnegative expansion in $t^j (r+t)^{n-2-j}$, and thus $S_2(t)$ has a nonnegative expansion in $R_{n,k}(t)$. \end{proof} \section{Dual partition lattices} \label{sec-part} Recall that the dual partition lattice of rank $n$, $\Pi_{n+1}'$, is the poset of all partitions of $[n+1]$, with $\pi \leq \sigma$ if every block of $\sigma$ is contained in a block of $\pi$. The \emph{infinite dual partition lattice} $\Pi'$ is the set of all partitions of $\{1,2,\ldots \}$ into finitely many blocks, with $\pi \leq \sigma$ if every block of $\sigma$ is contained in a block of $\pi$. Hence $\Pi'$ is rank uniform with $R=R(\Pi')= (S(n+1,k+1))_{n,k=0}^\infty$. It is well known that $R$ is $\mathrm{TN}$. We will now find a combinatorial description of the polynomials $R_{n,k}(t)$, which gives another proof of the total nonnegativity of $R$. Recall that the \emph{chromatic polynomial} of a simple graph $G=(V,E)$ may be expressed as $$ \chi_G(t)= \sum_{k=1}^n S_G(k) \cdot (t)_k, \ \ \ |V|=n, $$ where $(t)_k = t(t-1)\cdots (t-k+1)$, and $S_G(k)$ is the number of partitions of $V$ such that no block contains an edge from $E$. The \emph{$\sigma$-polynomial} of $G$ is defined by $$ \sigma_G(t)= \sum_{k=1}^n S_G(k) \cdot t^k $$ Hence $\sigma_G(t)=\mathcal{S}(\chi_G(t))$, where $ \mathcal{S} : \RR[t] \to \RR[t]$ is the invertible linear map defined by $ \mathcal{S}((t)_k) = t^k$ for all $k \in \NN$. Let $B_{n,k}$ be the graph on $[n]$ with edges $\{i, j\}$, $1\leq i<j \leq k$, and notice that $\chi_{B_{n+1,k+1}}(t)= t^{n-k}(t)_{k+1}$. \begin{figure}[H] \centering \begin{tikzpicture}[line cap=round,line join=round,>=triangle 45,x=1cm,y=1cm] \clip(-9,-2) rectangle (2.5,2); \draw [line width=2pt] (-8,1)-- (-6,-1); \draw [line width=2pt] (-6,-1)-- (-6,1); \draw [line width=2pt] (-6,1)-- (-8,1); \draw [line width=2pt] (-8,1)-- (-8,-1); \draw [line width=2pt] (-8,-1)-- (-6,1); \draw [line width=2pt] (-8,-1)-- (-6,-1); \draw [line width=2pt] (0,1)-- (-1,-1); \draw [line width=2pt] (-1,-1)-- (1,-1); \draw [line width=2pt] (1,-1)-- (0,1); \draw (-8.5,1.4) node[anchor=north west] {1}; \draw (-6,1.4) node[anchor=north west] {2}; \draw (-8.5,-1.1) node[anchor=north west] {3}; \draw (-6,-1.1) node[anchor=north west] {4}; \draw (-5.072039820114968,-0.2858820632730748) node[anchor=north west] {5}; \draw (-4.052339710155575,-0.2749175459616835) node[anchor=north west] {6}; \draw (-3.0545686348189647,-0.23105947671611815) node[anchor=north west] {7}; \draw (-0.06125540880913355,1.5013342584837122) node[anchor=north west] {1}; \draw (-1.3550684515533096,-1.1) node[anchor=north west] {2}; \draw (1,-1.1) node[anchor=north west] {3}; \draw (1.9891093284210437,-0.2749175459616835) node[anchor=north west] {4}; \begin{scriptsize} \draw [fill=ududff] (-8,1) circle (2.5pt); \draw [fill=ududff] (-8,-1) circle (2.5pt); \draw [fill=ududff] (-6,1) circle (2.5pt); \draw [fill=ududff] (-6,-1) circle (2.5pt); \draw [fill=xdxdff] (-5,0) circle (2.5pt); \draw [fill=xdxdff] (-4,0) circle (2.5pt); \draw [fill=xdxdff] (-3,0) circle (2.5pt); \draw [fill=ududff] (-1,-1) circle (2.5pt); \draw [fill=xdxdff] (0,1) circle (2.5pt); \draw [fill=ududff] (1,-1) circle (2.5pt); \draw [fill=xdxdff] (2,0) circle (2.5pt); \end{scriptsize} \end{tikzpicture} \caption{The graphs of $B_{7,4}$ and $B_{4,3}$, respectively.} \label{fig:partition-hyperplanes} \end{figure} \begin{theorem}\label{partnk} If $0\leq k \leq n$, then $$ R_{n,k}(t) = \sigma_{B_{n+1,k+1}}(t)/t= \mathcal{S}\left(t^{n-k}(t)_{k+1}\right)/t. $$ Moreover $$ R_{n+1,k}(t)= R_{n+1,k+1}(t)+ (k+1) R_{n,k}(t), \ \ \ 0\leq k \leq n. $$ \end{theorem} \begin{proof} For $k=0$, $$ \sigma_{B_{n+1,1}}(t)/t = \sum_{k=0}^n S(n+1,k+1)t^k = R_{n,0}(t). $$ It remains to prove the identity $$ \sigma_{B_{n+2,k+1}}(t)= \sigma_{B_{n+2,k+2}}(t)+ (k+1)\sigma_{B_{n+1,k+1}}(t), $$ or equivalently, $$ t^{n+1-k}(t)_{k+1}= t^{n-k}(t)_{k+2} + (k+1) t^{n-k}(t)_{k+1}, $$ which is trivially true. \end{proof} We call $\Pi'$-posets \emph{partition posets}. We shall now try to come up with a reasonable notion of shellings for partition posets. The $h$-vector of a partition poset $P$ of rank $n$ is the vector $(h_0(P), h_1(P), \ldots, h_n(P))$ for which $$ f_P(t)= \sum_{x \in P}t^{\rho(x)} = \sum_{k=0}^n h_k(P) \cdot R_{n,k}(t). $$ Let $F_1, \ldots, F_m$ be an ordering of the maximal elements of a rank $n$ pure partition poset $P$. Let $P_k = \langle F_1 \rangle \cup \langle F_2 \rangle \cup \cdots \cup \langle F_k \rangle$. Again, a condition that is required in all sensible definitions of shellings is \begin{itemize} \item[$(\mathrm{S}_1)$] For each $k \geq 2$, $P_{k-1} \cap \langle F_k \rangle $ is pure of rank $n-1$. \end{itemize} Suppose $(\mathrm{S}_1)$ is satisfied, and identify $\langle F_k \rangle$ with $\Pi_{n+1}'$. An element $H_e \in \Pi_{n+1}'$ of rank $n-1$ has precisely $n-1$ blocks with one element each, and a single block $\{ i, j \}$. Hence we may identify $P_{k-1} \cap \langle F_k\rangle$ with a simple graph $G=G(F_k)=([n+1], E)$. Express the rank generating polynomial of $P_k$ as $$ f_{P_k}(t) = f_{P_{k-1}}(t)+ \sum_{x \in Q(G)} t^{\rho(x)}, $$ where $$ Q(G) =\langle F_k \rangle \setminus \left(P_{k-1} \cap \langle F_k \rangle\right) = \langle F_k \rangle \setminus \cup_{e \in E} \langle H_{e} \rangle. $$ Now $\pi \in Q(G)$ if and only if no block of $\pi$ contains an edge in $E$. Hence $$ \sum_{x \in Q(G)} t^{\rho(x)} = \sigma_G(t)/t = \mathcal{S}(\chi_G(t))/t. $$ The second property of a shelling that we want is that $h$-vectors of shellable partition posets are nonnegative. In light of Theorem~\ref{partnk} a reasonable condition is then that $\chi_G(t)$ has a nonnegative expansion in $\{ t^{n-k}(t)_{k+1} \}_{k=0}^n$. This is true for all graphs with at most $10$ vertices. This led us to ask if all graphs have a nonnegative expansion in $\{ t^{n-k}(t)_{k+1} \}_{k=0}^n$. However G. Royle and E. Gioan found that the complete bipartite $K_{7,7}$ fails to have a nonnegative expansion \cite{Royle}. \emph{Chordal graphs} may be defined recursively as follows. The empty graph (the graph with no vertices or edges) is chordal. A nonempty graph $G=(V,E)$ is chordal if and only if there exists a vertex $v \in V$ such that the neighbors of $v$ form a clique and $G \setminus v$ is chordal. \begin{definition} \label{partshelling} Let $P$ be a pure partition poset. A total order $F_1, \ldots, F_m$ on the facets of $P$ is called a \emph{shelling} if $(\mathrm{S}_1)$ is satisfied, and \begin{itemize} \item[(C)] for all $1<i \leq m$ the graph $G(F_i)$ defined above is chordal. \end{itemize} \end{definition} \begin{lemma} \label{chordal-nonnegative} Let $G$ be a chordal graph on $n$ vertices. Then the chromatic polynomial of $G$ has a nonnegative expansion in $\{ t^{n-k}(t)_{k} \}_{k=1}^n$. \end{lemma} \begin{proof} The proof is by induction on the number $n$ of vertices of $G$. The case when $n=1$ is clear. Suppose $n \geq 2$. By definition $G$ is obtained from a chordal graph $H$ on $n-1$ vertices by adding a new vertex $v$ whose neighbors $A$ form a clique. Hence $ \chi_G(t) = (t-a)\chi_H(t), $ where $|A|=a$. By induction, $$ \chi_H(t) = \sum_{k=1}^{n-1} \theta_k \cdot t^{n-1-k}(t)_{k}, \ \ \mbox{ where } \theta_k \geq 0 \mbox{ for all } k. $$ Since $\chi_H(k)=0$ for all $0 \leq k \leq a-1$, it follows that $\theta_k=0$ for all $0 \leq k \leq a-1$. Now $$ (t-a)t^{n-1-k}(t)_k= \left(1- \frac a k \right) t^{n-k}(t)_k + \frac a k t^{n-1-k}(t)_{k+1}, $$ which proves the lemma. \end{proof} From Lemma~\ref{chordal-nonnegative} and the discussion above it we deduce: \begin{corollary}\label{parpossh} Let $P$ be a pure partition poset. If $P$ is shellable, then it has a nonnegative $h$-vector, and hence the chain polynomials of $P$ and its rank selected sub-posets are real-rooted. \end{corollary} Define a linear order on the set $\Pi_{n}^k$ of partitions of $[n]$ into $k$ blocks as follows. For Definition \ref{partshelling} to make sense, $\langle \Pi_{n}^k \rangle$ should be shellable for all $k \leq n$. We shall now prove this. Order the blocks $v_1, \ldots, v_k$ of $\pi \in \Pi_{n}^k$ so that $\max(v_1) < \cdots < \max(v_k)$. Order the elements within the blocks in decreasing order, and consider the permutation $\sigma(\pi)= v_1v_2 \cdots v_k$. If $\pi, \pi' \in \Pi_{n}^k$ we write $\pi <_\ell \pi'$ if $\sigma(\pi)$ comes before $\sigma(\pi')$ in the lexicographical ordering. Hence $<_\ell$ is a total order. For $\pi \in \Pi_{n}^k$, define a graph $G(\pi)=(V,E)$, where $V=\{v_1,\ldots, v_k\}$ are the blocks of $\pi$, and $E$ is the set of all $\{v_i,v_j\}$ for which there exists a partition $\pi' \in \Pi_{n}^k$ for which $\pi' <_\ell \pi$ and $\pi \wedge \pi'$ is equal to the partition $\pi[v_i,v_j]$ obtained from $\pi$ by merging the blocks $v_i$ and $v_j$. \begin{lemma}\label{vivj} Let $\pi \in \Pi_{n}^k$. Then $\{v_i,v_j\}$, where $i<j$, is not an edge in $G(\pi)$ if and only if \begin{itemize} \item[(a)] $|v_1| = \cdots =|v_i|=1$, and \item[(b)] if $i < m \leq k$, then $x<y$ for all $x \in v_i$ and $y \in v_m$. \end{itemize} \end{lemma} \begin{proof} We first prove that if (a) or (b) fails, then $\{v_i,v_j\}$ is an edge of $G(\pi)$. Suppose $|v_s|>1$ for some $s<i$ and consider the partition $\pi'$ obtained from $\pi$ by splitting $v_s$ into two blocks, and merging $v_i$ with $v_j$. Then $\pi' <_\ell \pi$ and $\pi \wedge \pi' = \pi[v_i,v_j]$. Hence $\{v_i,v_j\}$ is an edge. Similarly if $|v_i|>1$, then let $\pi'$ be the partition obtained from $\pi$ by moving the largest element of $v_i$ to $v_j$. Then $\pi' <_\ell \pi$ and $\pi \wedge \pi' = \pi[v_i,v_j]$. Hence $\{v_i,v_j\}$ is an edge. Suppose (a) holds and (b) fails for some $m$. Let $\pi'$ be the partition obtained from $\pi$ by merging $v_i$ and $v_j$ and splitting $v_m$ into two blocks by making the smallest element of $v_m$ the only element in one of the blocks. Then $\pi' <_\ell \pi$ and $\pi \wedge \pi' = \pi[v_i,v_j]$. Hence $\{v_i,v_j\}$ is an edge. Conversely if (a) and (b) hold, then $\{v_i,v_j\}$ is not an edge. \end{proof} \begin{remark}\label{G-form} Let $s$ be the maximal $i$ for which $v_i$ is not a vertex of an edge in $G(\pi)$. Then, by Lemma~\ref{vivj}, $v_j= \{j\}$ for all $1 \leq j \leq s$, and the graph $G(\pi)$ is isomorphic to $B_{k,k-s}$. \end{remark} \begin{theorem}\label{pi-shell} The order $<_\ell$ defines a shelling of the facets of $\langle \Pi_{n}^k \rangle$. \end{theorem} \begin{proof} We need to prove that if $\pi' <_\ell \pi \in \Pi_{n}^k$, then $\pi' \wedge \pi \leq \pi[v_s,v_t]$ for some edge $\{v_s,v_t\}$ in $G(\pi)$. If $G(\pi)$ is the complete graph, then there is nothing to prove. Otherwise $\pi$ satisfies (a) and (b) in Lemma~\ref{vivj} for some maximal $i \geq 1$. Since $\pi' <_\ell \pi$, the blocks $\{1\}, \ldots, \{i\}$ of $\pi$ are also blocks of $\pi'$. Hence $\pi' \wedge \pi \leq \pi[v_s,v_t]$ for some $s>t >i$. Since $v_{i+1}, \ldots, v_k$ form a clique in $G(\pi)$, the proof follows. \end{proof} \begin{corollary} Let $0\leq k <n$. The $h$-vector of $\langle \Pi_{n+1}^{k+1} \rangle$ is equal to $$\big( (i+1) S(n-k+i,i+1)\big)_{i=0}^k.$$ \end{corollary} \begin{proof} By Theorem~\ref{pi-shell}, the order $<_\ell$ defines a shelling. By Remark \ref{G-form}, $h_{k-i}$ is equal to the number of partitions in $\Pi_{n+1}^{k+1}$ for which $\{j\}$ is a block of $\pi$ for all $1\leq j \leq i$, but $\{i+1\}$ is not a block. Hence $$ h_{k-i}= S(n-i+1, k-i+1)-S(n-i,k-i)= (k-i+1)S(n-i,k-i+1), $$ which proves the lemma. \end{proof} \bibliographystyle{abbrv} \bibliography{references} \end{document}
2412.06659v1
http://arxiv.org/abs/2412.06659v1
Ischebeck's formula, Grade and quasi-homological dimensions
\documentclass[12pt]{amsart} \usepackage{amssymb} \usepackage{amsthm} \usepackage{amscd} \usepackage{color} \usepackage{hyperref} \usepackage{tikz-cd} \usepackage[utf8]{inputenc} \usepackage[T1]{fontenc} \usepackage{mathpazo} \usepackage[scaled=.95]{helvet} \usepackage{courier} \swapnumbers \textwidth=16.00cm \textheight=22cm \topmargin=0.00cm \oddsidemargin=0.00cm \evensidemargin=0.00cm \headheight=14.4pt \headsep=1cm \numberwithin{equation}{section} \hyphenation{semi-stable} \emergencystretch=10pt \usepackage[colorinlistoftodos]{todonotes} \usepackage[all]{xy} \newcommand{\C}{\mathbb{C}} \newcommand{\A}{\mathbb{A}} \newtheorem{theorem}{Theorem}[section] \newtheorem{lemma}[theorem]{Lemma} \newtheorem{lem}[theorem]{Lemma} \newtheorem{proposition}[theorem]{Proposition} \newtheorem{prop}[theorem]{Proposition} \newtheorem{corollary}[theorem]{Corollary} \newtheorem{cor}[theorem]{Corollary} \newtheorem{BRconstruction}[theorem]{Brodmann and Rotthaus Construction} \newtheorem{thm}[theorem]{Theorem} \newcommand{\OO}{\mathcal{O}} \theoremstyle{definition} \newtheorem{notation}[theorem]{Notation} \newtheorem{nota}[theorem]{Notation} \newtheorem{defn}[theorem]{Definition} \newtheorem{definition}[theorem]{Definition} \newtheorem{definitions}[theorem]{Definitions} \newtheorem{example}[theorem]{Example} \newtheorem{Example}[theorem]{Example} \newtheorem{ex}[theorem]{Example} \newtheorem{examples}[theorem]{Examples} \newtheorem{xca}[theorem]{Exercise} \newtheorem{discussion}[theorem]{Discussion} \newtheorem{remark}[theorem]{Remark} \newtheorem{rem}[theorem]{Remark} \newtheorem{conj}[theorem]{Conjecture} \newtheorem{question}[theorem]{Question} \newtheorem{Question}[theorem]{Question} \newtheorem{answer}[theorem]{Answer} \newtheorem{questions}[theorem]{Questions} \newtheorem{claim}[theorem]{Claim} \newtheorem{subclm}[theorem]{Sublaim} \newtheorem{notes}[theorem]{Notes} \newtheorem{Note}[theorem]{Note} \newtheorem{Notes}[theorem]{Notes} \newtheorem{ting}[theorem]{ting} \newtheorem{tingnotation}[theorem]{Setting} \newtheorem{setnot}[theorem]{Setting} \numberwithin{equation}{section} \newtheorem{gexample}[theorem]{General Example} \newtheorem{remark/questions}[theorem]{Remark and Questions} \newtheorem{fact}[theorem]{Fact} \newtheorem{facts}[theorem]{Facts} \newtheorem{remarks}[theorem]{Remarks} \newtheorem{remarksq}[theorem]{Remarks and Questions} \DeclareMathOperator{\D}{D} \newtheorem*{agra}{Acknowlegment}\newcommand{\rd}{\mbox{rdim}}\newcommand{\N}{\mathbb{N}}\newcommand{\Z}{\mathbb{Z}}\newcommand{\CC}{\mathbb{C}} \newcommand{\PP}{\mathbb{P}} \newtheorem*{fund}{Funding} \long\def\alert#1{\smallskip\line{\hskip\parindent\vrule \vbox{\advance\hsize-2\parindent\hrule\smallskip\parindent.4\parindent \narrower\noindent#1\smallskip\hrule}\vrule\hfill}\smallskip} \newtheorem{definitionsremarks}[theorem]{Definitions and Remarks} \newtheorem{exampleremarks}[theorem]{Example and Remarks} \newtheorem{setup}[theorem]{Setup} \newtheorem{ConvNot}[theorem]{Conventions and Notations} \newtheorem{conjecture}[theorem]{Conjecture} \newtheorem{quesremark}[theorem]{Question and Remarks} \newtheorem{newremark}[theorem]{Remarks} \newtheorem{newquestion}[theorem]{Question} \newtheorem{newexample}[theorem]{Example} \newtheorem{newclaim}[theorem]{} \newtheorem{Endpiecenotation}[theorem]{Endpiece Notation} \newtheorem{note}[theorem]{Note} \newtheorem{introduction}[theorem]{Introduction} \newtheorem{insidert}[theorem]{Insider Technique} \newtheorem{notee}[theorem]{} \newtheorem{construction}[theorem]{Construction} \newtheorem{Construction}[theorem]{Construction of Example} \newtheorem{Approximation}[theorem]{Approximation Technique} \newtheorem{Comment}[theorem]{Comments} \newtheorem{Discussion}[theorem]{Discussion} \newtheorem{insider}[theorem]{Insider Construction Details} \newtheorem{abf}[theorem]{Auslander-Buchsbaum Formula} \newcommand{\m}{\mathfrak{m}} \newcommand{\n}{\mathfrak{n}} \newcommand{\ia}{\mathfrak{a}} \newcommand{\ib}{\mathfrak{b}} \newcommand{\ic}{\mathfrak{c}} \newcommand\Der{\operatorname{Der}} \newcommand\Syz{\operatorname{Syz}} \newcommand{\p}{\mathfrak{p}} \newcommand{\q}{\mathfrak{q}} \DeclareMathOperator{\pd}{pd} \newcommand{\Fa}{\operatorname{\mathfrak{F}}_\mathfrak{a}} \DeclareMathOperator{\Rad}{{{Rad}}} \DeclareMathOperator{\width}{width} \DeclareMathOperator{\Tor}{Tor} \DeclareMathOperator{\tor}{Tor} \DeclareMathOperator{\ndim}{N-dim} \DeclareMathOperator{\Hom}{Hom} \DeclareMathOperator{\Ext}{Ext} \DeclareMathOperator{\ext}{Ext} \DeclareMathOperator\Att{Att} \DeclareMathOperator\Ass{Ass} \DeclareMathOperator\Assh{Assh} \DeclareMathOperator\ass{Ass} \DeclareMathOperator{\pdim}{pd} \DeclareMathOperator{\id}{id} \DeclareMathOperator{\qpd}{qpd} \DeclareMathOperator{\Supp}{Supp} \DeclareMathOperator{\Spec}{Spec} \DeclareMathOperator{\depth}{depth} \newcommand\cdim{\operatorname{CI-dim}} \def\dim{\mathop{\rm dim}} \def \N{{\mathbb N}} \def\Ann{\operatorname{Ann}} \numberwithin{equation}{section} \begin{document} \dedicatory{} \title[]{Ischebeck's formula, Grade and quasi-homological dimensions} \author{V. H. Jorge-P\'erez} \author{Paulo Martins} \author{V. D. Mendoza-Rubio} \address{Universidade de S{\~a}o Paulo - ICMC, Caixa Postal 668, 13560-970, S{\~a}o Carlos-SP, Brazil} \email{[email protected]} \address{Universidade de S{\~a}o Paulo - ICMC, Caixa Postal 668, 13560-970, S{\~a}o Carlos-SP, Brazil} \email{[email protected]} \address{Universidade de S{\~a}o Paulo - ICMC, Caixa Postal 668, 13560-970, S{\~a}o Carlos-SP, Brazil} \email{[email protected]} \keywords{quasi-projective dimension, quasi-injective dimension, grade, vanishing of Ext, Ischebeck's formula, quasi-perfect module, Gorenstein dimension} \subjclass[2020]{13D05, 13D07,13D02} \thanks{Corresponding author: Victor D. Mendoza Rubio} \begin{abstract} The quasi-projective dimension and quasi-injective dimension are recently introduced homological invariants that generalize the classical notions of projective dimension and injective dimension, respectively. For a local ring $R$ and finitely generated $R$-modules $M$ and $N$, we provide conditions involving quasi-homological dimensions where the equality $\sup \{i\geq 0: \Ext_R^i(M,N)\not=0\}=\depth R-\depth M$, which we call Ischebeck's formula, holds. One of the results in this direction generalizes a well-known result of Ischebeck concerning modules of finite injective dimension, considering the quasi-injective dimension. On the other hand, we establish an inequality relating the quasi-projective dimension of a finitely generated module to its grade and introduce the concept of a quasi-perfect module as a natural generalization of a perfect module. We prove several results for this new concept similar to the classical results. Additionally, we provide a formula for the grade of finitely generated modules with finite quasi-injective dimension over a local ring, as well as grade inequalities for modules of finite quasi-projective dimension. In our study, Cohen-Macaulayness criteria are also obtained. \end{abstract} \maketitle \section{Introduction} Throughout this paper, we assume that \( R \) is a commutative Noetherian ring and that all \( R \)-modules are finitely generated. The study of homological dimensions of modules is an important topic in commutative algebra, closely connected to the resolution of central problems in this area, as evidenced by several works such as \cite{ArayaYoshino,ArayaTakahashi2022,GhoshTakahashi2024,CelikbasIimaSadeghiTakahashi2018,GJT,Gheibi}. For any non-zero $R$-modules $M$ and $N$, we define: \begin{align*} \operatorname{P}_R(M,N)&:=\sup \lbrace i \geq 0 : \operatorname{Ext}^i_R(M,N) \neq 0 \rbrace, \\ \operatorname{grade}(M,N)&:=\inf \lbrace i\geq 0: \Ext_R^i(M,N)\not=0 \rbrace. \end{align*} These values have been widely studied, especially when at least one of the modules $M $ or $N $ has some finite homological dimension. For instance, regarding to the $\operatorname{P}_R(M,N)$, when $R$ is local the equality \begin{equation}\label{Fisch} \operatorname{P}_R(M,N)=\depth R-\depth M \end{equation} was proved in several cases involving some homological dimension. One of the more known cases is when $N$ has finite injective dimension, due to Ischebeck \cite[2.6]{isch}, letting thus the authors motivated to refer to \eqref{Fisch} as the \textit{Ischebeck formula}. In the same paper, Ischebeck showed that \eqref{Fisch} is true when $\operatorname{pd}_R M<\infty$. Other cases where the equality has been proved include: (1) $M$ has finite complete intersection dimension and $\operatorname{P}_R(M,N)<\infty$, by Araya and Yoshino \cite[Theorem 4.2]{ArayaYoshino}. (2) $M$ has finite injective dimension and $N$ has finite Gorenstein injective dimension, due to Sazeedeh \cite[Theorem 2.10]{saz}. (3) $M$ has finite AB-dimension by Araya \cite[Lemma 2.5]{Araya2}. (4) $M$ has finite redu\-cing complete intersection dimension with $\operatorname{depth} M \leq \operatorname{depth} R$ and $\operatorname{P}_R(M,N)<\infty$, by Celikbas et al. \cite[Proposition 4.9]{Celikbaset}. On the other hand, the study of the grade involving certain homological dimensions can be evidenced in works like \cite{ArayaYoshino,Yoshida,Yassemi,GLCMHGD,new,Foxby1,khatami}. The study of \textit{perfect modules}, that is, modules satisfying $\operatorname{grade} M := \operatorname{grade}(M, R) = \operatorname{pd}_R M$, has received particular attention in this context. This concept was generalized by Foxby \cite{Foxby1}, who replaced projective dimension with Gorenstein dimension, introducing a new class of modules called $G$--perfect modules. This new class has been further explored in works such as \cite{Yassemi, GhoshPuth}, among others. Recently, Gheibi, Jorgensen and Takahashi \cite{GJT} introduced a new homological dimension called quasi-projective dimension (qpd) which is a finer invariant than projective dimension, in the sense that for any $R$-module $M$, there is always an inequality $\operatorname{qpd}_R M \leq \operatorname{pd}_R M$ and equality holds when $\operatorname{pd}_R M$ is finite. Following the work \cite{GJT}, Gheibi \cite{Gheibi} introduced another new homological dimension called quasi-injective dimension (qid) as a generalization of the injective dimension. We refer to these new dimensions as \textit{quasi-homological dimensions}. The aim of this paper is to study the values $\operatorname{P}_R(M,N)$ and $\operatorname{grade}(M,N)$ when at least one between $M$ and $N$ has a finite quasi-homological dimension. The following theorem presents our main results concerning the study of \( \operatorname{P}_R(-,-) \) and quasi-homological dimensions, and is motivated by {\cite[Theorems 6.2(4) and 6.12]{GJT} and \cite[Corollary 3.5]{Gheibi}}. In addition, it provides new cases in that the Ischebeck formula holds under certain conditions of the finiteness of quasi-homological dimension on $M$ or $N$, obtaining thus not only improvements of \cite[Corollary 3.5]{Gheibi} and \cite[Lemma 3.5(1)]{MCMtensorproductsandvanishingofExtmodules}, but also a generalization of the well-known result of Ischebeck concerning to finitely generated modules of finite injective dimension \cite[2.6]{isch} mentioned previously. \begin{theorem}[See Theorems \ref{teo:qproj}, \ref{g-dim} and \ref{isch}]\label{qw88} Let $R$ be a Noetherian local ring, and let $M$ and $N$ be non-zero $R$-modules. Suppose that $\operatorname{P}_R(M,N)<\infty$. Then the equality $$\operatorname{P}_R(M,N)=\operatorname{depth} R-\depth M$$ holds in each one of the following cases: \begin{enumerate} \item $M$ has finite quasi-projective dimension. \item $M$ has finite Gorenstein dimension and $N$ has finite quasi-projective dimension. \item $N$ has finite quasi-injective dimension. \end{enumerate} \end{theorem} On the other hand, in our study of the grade of two modules $M$ and $N$ and quasi-homological dimension, we give special attention in the case where $N=R$. We prove a relation of inequality between the grade and quasi-projective dimension, similarly as occurs with the projective dimension and Gorenstein dimension. \begin{theorem}[See Theorem \ref{theo4.5}]\label{to12} Let $M$ be a non-zero $R$-module. One then has the inequality $\operatorname{grade} M \leq \operatorname{qpd}_R M$. \end{theorem} It is worth noting that, unlike the case of projective dimension and Gorenstein dimension, the inequality in the theorem above does not follow trivially. The motivation for establishing Theorem \ref{to12} is to introduce the notion of quasi-perfect modules as a natural generalization of perfect modules. More specifically, we define an $R$-module $M$ to be \textit{quasi-perfect} if $\operatorname{qpd}_R M<\infty$ and $\operatorname{grade} M=\operatorname{qpd}_R M$. This new definition allows us to generalize several well-known results for perfect modules, where Theorem \ref{to12} plays a crucial role. For instance, we generalize a well-known result stating that: A (finitely generated) $R$-module $M$ over a regular local ring is Cohen-Macaulay if and only if it is perfect (see e.g. \cite[Corollary 2.2.10]{BH}). Namely, we prove that an $R$-module with finite quasi-projective dimension is Cohen Macaylay if it is quasi-perfect, and that the converse holds when $R$ is Cohen-Macaulay (see Proposition \ref{prop:eqv}). A ce\-lebrated application of (Peskine-Szpiro) intersection theorem is that: A local ring is Cohen-Macaulay if it admits a nonzero Cohen-Macaulay module of finite projective dimension (see \cite[Corollary 9.6.2]{BH}). However, the same does not work with quasi-projective dimension in place of projective dimension. As an application of our results, we obtain a Cohen-Macaulayness criterion for a local ring in terms of the existence of modules with finite quasi-projective dimension (see Corollary \ref{pros1}). Concerning the study of grade and quasi-injective dimension, we obtain a formula for the grade of finitely generated modules with finite quasi-injective dimension over a local ring, which improves \cite[Theorem 3.7]{Gheibi}. In fact, this theorem says that when $R$ is local and $M$ is a non-zero $R$-module of finite quasi-injective dimension, one then has the inequality $\dim M\leq \operatorname{depth} R$. However, the theorem does not show how many is missing from $\dim M$ to be equal to $\depth R$. The formula that we obtain shows that the missing quantify is exactly $\operatorname{grade} M$. \begin{theorem}[See Theorem \ref{teo:forinj}]\label{12aqs} Let $R$ be a local ring and let $M$ be a non-zero $R$-module with $\operatorname{qid}_R M <\infty$. Then $$\operatorname{dim} M=\depth R-\operatorname{grade} M.$$ \end{theorem} The above theorem can be compared to \cite[Theorem 4.2]{new}. Motivated by Bass's Conjecture (see e.g. \cite[Corollary 9.6.2]{BH}), Takahashi \cite[Theorem 3.5(1)]{Takahashi} proved that a local ring \( R \) with a dualizing complex is Cohen Macaulay if it admits a non-zero \( R \)-module of finite Gorenstein injective dimension and maximal Krull dimension. Moreover, Gheibi \cite[Corollary 3.8]{Gheibi} proved an analogous to Takahashi's result with quasi-injective dimension. As an application of Theorem \ref{12aqs}, we derive the following Cohen-Macaulayness criterion. \begin{corollary}[See Corollary \ref{corol:criteria}] Let $R$ be a local ring. If there exists a non-zero $R$-module $M$ such that $\operatorname{qid}_R M < \infty$ and $\dim R = \dim M + \operatorname{grade} M$, then $R$ is Cohen-Macaulay. \end{corollary} Now, we briefly describe the structure of this paper. In Section \ref{section2}, we provide de\-finitions, notations, and some results that are considered in this paper. In Section \ref{section3}, we prove items (1) and (2) of Theorem \ref{qw88} and explore some of their consequences. In Section \ref{section4}, we prove item (3) of Theorem \ref{qw88} obtaining a generalization of a celebrated result by Ischebeck. In Section \ref{section5}, we prove results concerning the grade and the quasi-homological dimensions, we prove Theorems \ref{to12} and \ref{12aqs}, introduce the definition of quasi-perfect module and explore some applications obtaining Cohen-Macaulayness criteria for rings and modules. Finally, in the last section, we apply some results of Section 5 to obtain some grade inequalities for modules with finite quasi-projective dimension, with Theorem \ref{genary} being the main result of this section. \section{Conventions and Background}\label{section2} In this section, we introduce fundamental definitions and facts, such as quasi-projective dimension, quasi-injective dimension and Gorenstein dimension. These concepts will be essential throughout the rest of the paper. \begin{newclaim} Let $R$ be a local ring, let $M$ be an $R$-module and consider a minimal free resolution \begin{align*} \cdots \rightarrow F_i \xrightarrow{\varphi_i} F_{i-1} \rightarrow \cdots \rightarrow F_1 \xrightarrow{\varphi_1} F_0 \xrightarrow{\varphi_0} M \rightarrow 0 \end{align*} of $M$. For $i \geq 1$, the $i$-\textit{syzygy} of $M$, denoted by $\Omega^i(M)$, is defined as the kernel of the map $\varphi_{i-1}$. When $i=0$, we set $\Omega^0(M)=M$. For $i \geq 0$, the modules $\Omega^i(M)$ are defined uniquely up to isomorphism. \end{newclaim} \begin{newclaim} In this work, we adopt the convention that $\sup \emptyset=-\infty$ and that $\inf \emptyset = \infty$. \end{newclaim} \begin{newclaim} Let $M$ and $N$ be non-zero $R$-module. Recall the following notations: \begin{align*} \operatorname{P}_R(M,N)&=\sup \{ i \geq 0: \Ext_R^i(M,N)\not=0\},\\ \operatorname{grade}(M,N)&=\inf\{i \geq 0: \Ext_R^i(M,N)\not=0\}. \end{align*} Note that $\operatorname{grade}(M,N)$ can be infinite (e.g., let $\mathfrak{m} \neq \mathfrak{n}$ be two maximal ideals of $R$, $M=R/\mathfrak{m}$ and $N= R/\mathfrak{n}$). \end{newclaim} Let $M$ be an $R$-module and let $I$ be an ideal of $R$ such that $IM \neq M$. We denote by $\operatorname{depth}(I,M)$ the common length of a maximal $M$-regular sequence in $I$ (see \cite[Definition 1.2.6]{BH}). \begin{newclaim}\label{claim2.3} Let $R$ be a local ring, and let $M$ and $N$ be non-zero $R$-modules. One then has $\operatorname{grade}(M,N)<\infty,$ and the following chain of inequalities holds: \begin{align*} 0 \leq \operatorname{grade}(M,N) \leq \operatorname{P}_R(M,N) \leq \infty. \end{align*} \end{newclaim} \begin{proof} We only need to show that $\operatorname{grade}(M,N)<\infty$. Set $I= \operatorname{ann}(M)$. Note that $I$ is a proper ideal of $R$, as $M \neq 0$. Since $N \neq 0 $, by Nakayama's Lemma, we have $IN \neq N$. Thus, by \cite[Proposition 1.2.10(e)]{BH}, $\operatorname{grade}(M,N) = \inf \lbrace i \geq 0 : \operatorname{Ext}_R ^i(M,N) \neq 0 \rbrace = \operatorname{depth}(I,M)$, whence $\operatorname{grade}(M,N) < \infty$. \end{proof} \begin{facts}\label{facts3} Let $M$ and $N$ be non zero $R$-modules. \begin{enumerate} \item (\cite[Theorem 2.1]{Yassemi}) The following inequalities hold: \begin{itemize} \item[(a)] \(\operatorname{depth} N - \dim M \leq \operatorname{grade}(M, N)\); \item[(b)] If \(\operatorname{Supp} M \subseteq \operatorname{Supp} N\), then \(\operatorname{grade}(M, N) \leq \dim N - \dim M\). \end{itemize} \item By item (1), we always have $\operatorname{depth} R \leq \operatorname{grade} M + \dim M \leq \dim R$. \item (\cite[Proposition 1.2.10(a) and (e)]{BH}) The following equalitie holds: \[ \begin{array}{lll} \text{grade}(M, N) & =\inf \{ \operatorname{depth} \, N_{\mathfrak{p}} \mid \mathfrak{p} \in \text{Supp} \, M \} \\ &=\inf \{ \operatorname{depth} \, N_{\mathfrak{p}} \mid \mathfrak{p} \in \text{Supp} \, M \cap \text{Supp} \, N \}. \end{array} \] \end{enumerate} \end{facts} \begin{newclaim} For a complex $$X: \cdots \stackrel{\partial_{i+2}}{\longrightarrow} X_{i+1} \stackrel{\partial_{i+1}}{\longrightarrow } X_i \stackrel{\partial_{i}}{\longrightarrow } X_{i-1} \longrightarrow \cdots $$ of $R$-modules, we set for each integer $i$, $\operatorname{Z}_i(X)=\ker \partial_i$ and $\operatorname{B}_i(X)= \operatorname{Im} \partial_{i+1}$ and $\operatorname{H}_i(X)=\operatorname{Z}_i(X)/\operatorname{B}_i(X)$. Moreover, we set: \begin{align*} &\left\{ \begin{aligned} \sup X &= \sup \{ i \in \mathbb{Z} \mid X_i \neq 0 \},\\ \inf X &= \inf \{ i \in \mathbb{Z} \mid X_i \neq 0 \}, \end{aligned} \right. \quad \left\{ \begin{aligned} \text{hsup } X &= \sup \{ i \in \mathbb{Z} \mid \operatorname{H}_i(X) \neq 0 \},\\ \text{hinf } X &= \inf \{ i \in \mathbb{Z} \mid \operatorname{H}_i(X) \neq 0 \}. \end{aligned} \right. \end{align*} The \textit{lenght} of $X$ is defined to be $\operatorname{lenght} X= \sup X - \inf X$. We say that $X$ is \textit{bounded}, if $\operatorname{lenght} X < \infty$. We say that $X$ is \textit{bounded below} if $\inf X > -\infty$ and $X$ is \textit{bounded above} if $\sup X < \infty$. \end{newclaim} \subsection{Quasi-homological dimensions.}The definition of quasi-projective dimension (resp. quasi-injective dimension) was introduced by Gheibi, Jorgensen and Takahashi \cite{GJT} (resp. Gheibi \cite{Gheibi}) as a generalization of the standard definition of projective dimension (resp. injective dimension). \begin{definition} Let $M$ be an $R$-module. \begin{enumerate} \item A \textit{quasi-projective resolution} of $M$ is a bounded below complex $P$ of projective $R$-modules such that for all $i \geq \inf P$ there exist non-negative integers $a_i$, not all zero, such that $H_i(P) \cong M^{\oplus a_i}$. We define the \textit{quasi-projective dimension of $M$} by \begin{align*} \operatorname{qpd}_R M = \inf \lbrace \sup P - \operatorname{hsup} P \mid P \text{ is a bounded quasi-projective resolution of } M \}, \end{align*} if $M\not=0$, and $\operatorname{qpd}_R M=-\infty$ if $M=0$. \item A \textit{quasi-injective resolution} of $M$ is a bounded above complex $I$ of injective $R$-modules such that for all $i \leq \sup I$ there exist non-negative integers $b_i$, not all zero, such that $H_i(I) \cong M^{\oplus b_i}$. We define the quasi-injective dimension of $M$ by \begin{align*} \operatorname{qid}_R M = \inf \lbrace \operatorname{hinf} I - \operatorname{inf} I \mid I \text{ is a bounded quasi-injective resolution of } M \}, \end{align*} if $M\not=0$, and $\operatorname{qid}_R M=-\infty$ if $M=0$. \end{enumerate} One has $\text{qpd}_R M = \infty$ (resp. $\operatorname{qid}_R M=\infty$) if and only if $M$ does not admit a bounded quasi-projective resolution (resp. quasi-injective resolution). We remark that $\operatorname{qpd}_R M$ is finer than projective dimension, in the sense that there is always an inequality $\operatorname{qpd}_R M \leq \operatorname{pd}_R M$ and equality holds when $\operatorname{pd}_R M$ is finite (see \cite[Corollary 4.10]{GJT}). The same works for the quasi-injective dimension, that is, $\operatorname{qid}_R M \leq \operatorname{id}_R M$ and equality holds when $\operatorname{id}_R M$ is finite, under the assumption that $R$ is local (see \cite[Corollary 3.3]{Gheibi}) \end{definition} We observe that results concerning the quasi-projective and quasi-injective dimensions can be found in \cite{GJT} and \cite{Gheibi}. Bellow, we state some very useful results related to the quasi-homological dimensions to be used in this paper. \begin{newclaim}\label{prop41} Let $R$ be a local ring. A complex $(X,\partial)$ of free $R$-modules of finite rank is called \textit{minimal} if $\partial_i \otimes_R k=0$ for all $i$, where $k$ is the residue field of $R$. If $M$ is a non-zero $R$-module with $\operatorname{qpd}_R M<\infty,$ then there exists a finite minimal quasi-projective resolution $F$ of $M$ such that $\mathrm{qpd}_R M = \sup F - \mathrm{hsup} F$ (see \cite[Proposition 4.1]{GJT}) \end{newclaim} \begin{theorem}\cite[Theorem 4.4]{GJT}\label{rem:ABF} Let $R$ be a local ring, and let $M$ be an $R$-module of finite quasi-projective dimension. Then \[ \operatorname{qpd}_R M = \operatorname{depth} R - \operatorname{depth} M. \] In particular, if $M\not=0$, then $\operatorname{depth} M\leq \operatorname{depth} R$ and $\operatorname{qpd}_R M\leq \operatorname{depth} R.$ \end{theorem} \begin{theorem}\cite[Theorem 4.11]{GJT}\label{depthformula} Let $R$ be a local ring, and let $M$ and $N$ be $R$-modules. Suppose that $M$ has finite quasi-projective dimension and $\operatorname{Tor}_{i}^R(M,N)=0$ for all $i>0$. Then \[ \operatorname{depth} M + \operatorname{depth} N = \operatorname{depth} R + \operatorname{depth} (M \otimes_R N). \] \end{theorem} \begin{theorem}\cite[Theorem 3.2]{Gheibi}\label{inj} Let $R$ be a local ring and let $M$ be a non-zero $R$-module. If $\operatorname{qid}_R M < \infty$, then $\operatorname{qid}_R M = \operatorname{depth} R$. \end{theorem} \subsection{Gorenstein dimension} The notion of Gorenstein dimension was introduced by Auslander \cite{Auslander1967} and deve\-loped by Auslander and Bridger in \cite{AuBr}. For an $R$-module $M$, set $M^\ast=\Hom_R(M,R)$. \begin{definition} Let $M$ be an $R$-module. \begin{enumerate} \item We say that $M$ is \textit{totally reflexive} if $M$ is reflexive and $\operatorname{Ext}^i_R(M, R) = 0 = \operatorname{Ext}^i_R(M^*, R)$ for all $i > 0$. \item The \textit{Gorenstein dimension} of $M$, denoted by $\operatorname{G-dim}_R M$, is defined to be the infimum of all non-negative integers $k$ such that there exists an exact sequence $$0 \to G_k \to \cdots \to G_0 \to M \to 0$$ where each $G_i$ is totally reflexive. \end{enumerate} \end{definition} We can observe that $\operatorname{G-dim}_R M =0$ if and only if $M$ is totally reflexive. \subsection{Complete intersection dimension} The notion of complete intersection dimension was introduced by Avramov, Gasharov and Peeva \cite{avramov2}. \begin{definition} Let $R$ be a local ring. A diagram of local ring maps $R \to R' \twoheadleftarrow S$ is called a \textit{quasi-deformation} if $R \to R'$ is flat and the kernel of the surjection $R' \twoheadleftarrow S$ is generated by a regular sequence on $S$. The \textit{complete intersection dimension} of an $R$-module $M$ is defined as follows: \begin{align*} \cdim_R M = \inf \lbrace \pd_S (M \otimes_R R')- \operatorname{pd}_S R' : R \rightarrow R' \twoheadleftarrow S \text{ is a quasi-deformation}\rbrace. \end{align*} \end{definition} \section{Quasi-projective dimension and Ischebeck's formula}\label{section3} In \cite[Theorems 6.2(4) and 6.12]{GJT}, the authors studied the vanishing of the $\operatorname{Ext}$-modules $\operatorname{Ext}_R^i(M,N)$ in two scenarios: when \( M \) has finite quasi-projective dimension, and when \( M \) has finite Gorenstein dimension while \( N \) has finite quasi-projective dimension. Inspired by these results and \cite[Theorem 4.2]{ArayaYoshino}, we investigate the value of \( \operatorname{P}_R(M,N) \) in these cases, assuming that \( \operatorname{P}_R(M,N) < \infty \) and that $R$ is local. More precisely, the goal of this section is to prove that for non-zero $R$-modules $M$ and $N$ such that $\operatorname{P}_R(M, N)<\infty$ the equality \begin{equation}\label{Isck} \operatorname{P}_R(M,N)=\depth R-\depth M \end{equation} holds in each one of the following cases: \begin{enumerate} \item $M$ has finite quasi-projective dimension. \item $M$ has finite Gorenstein dimension and $N$ has finite quasi-projective dimension. \end{enumerate} \subsection{The first case} In this subsection, we prove the equality \eqref{Isck} considering that $\operatorname{P}_R(M,N)<\infty$ holds in the case (1). The following lemma will be needed. \begin{lemma}\label{lemmavanishing} Let $R$ be a local ring, and let $M$ and $N$ be non-zero $R$-modules such that $\operatorname{P}_R(M,N)<\infty$. One then has the inequality $\operatorname{P}_R(M,N) \leq \operatorname{qpd}_R M$. \end{lemma} \begin{proof} We may assume that $\operatorname{qpd}_R M < \infty$. Then, by \cite[Corollary 6.4]{GJT}, we have that $\operatorname{Ext}_R^i(M,N)=0$ for all $i \geq \operatorname{qpd}_R M +1$ and consequently $\operatorname{P}_R(M,N) \leq \operatorname{qpd}_R M$. \end{proof} \begin{theorem}\label{teo:qproj} Let $R$ be a local ring, and let $M$ and $N$ be non-zero $R$-modules such that $\operatorname{P}_R(M,N)<\infty$. If $\operatorname{qpd}_R M<\infty$, then $$\operatorname{P}_R(M,N)=\operatorname{qpd}_R M=\operatorname{depth} R - \operatorname{depth} M.$$ \end{theorem} \begin{proof} The second equality is just the Auslander-Buchsbaum formula for quasi-projective dimension (Theorem \ref{rem:ABF}). Since $M$ has finite quasi-projective dimension, by \ref{prop41}, there exists a minimal quasi-projective resolution $$F=(0 \rightarrow F_s \rightarrow \cdots \rightarrow F_h \xrightarrow{\partial_h} \cdots )$$ of $M$, where $s=\sup F,$ $h=\operatorname{hsup} F$ and $r:=\operatorname{qpd}_R M=s-h$. Letting $C=\operatorname{Coker}(\partial_{h+1})$, we obtain an exact sequence $$0 \rightarrow F_s \xrightarrow{\partial_s} \cdots \rightarrow F_h \rightarrow C \rightarrow 0,$$ which is a minimal free resolution of $C$, so that $\operatorname{pd}_R C=s-h=:r$. For all $i \in \mathbb{Z}$, setting $Z_i=Z_i(F)$ and $B_i=B_i(F)$, note that there are exact sequences \begin{align*} 0 \rightarrow Z_i \rightarrow F_i \rightarrow B_{i-1} \rightarrow 0 \text{ and } 0 \rightarrow B_i \rightarrow Z_i \rightarrow \operatorname{H}_i(F) \rightarrow 0. \end{align*} Then we see that \[ \operatorname{Ext}^j_R(B_i, N) \cong \operatorname{Ext}^j_R(Z_i, N) \cong \operatorname{Ext}^{j+1}_R(B_{i-1}, N) \cong \cdots \cong \operatorname{Ext}^{j+i+1-\inf F}_R(B_{\inf F - 1}, N) = 0 \] for all \( i \geq \inf F \) and \( j > p:=\operatorname{P}_R(M,N) \), where the last equality is due to \( B_{\inf F-1} = 0 \). Thus, $\operatorname{Ext}^j_R(B_i, N)=0$ for all $i$ and $j>p$. Since $h=\operatorname{hsup} F$, then there exists a short exact sequence \begin{equation*} 0 \longrightarrow M^{\oplus a_h} \longrightarrow C \longrightarrow \operatorname{B}_{h-1} \longrightarrow 0 \end{equation*} for a positive integer $a_h$. Therefore, as $\operatorname{Ext}^j_R(B_{h-1}, N)=0=\Ext_R^j(M,N)$ for all $j>p$, then $\Ext_R^j(C,N)=0$ for all $j>p$. Thus, if $r > p$, then $\Ext_R^r(C,N)=0$, which contradicts to \cite[p. 154, Lemma 1(iii)]{Matsu} as $\operatorname{pd}_R C=r$. Therefore $r\leq p$, and it follows from Lemma \ref{lemmavanishing} that $r=p$. \end{proof} Araya and Yoshino \cite[Theorem 4.2]{ArayaYoshino} proved the above theorem with complete intersection dimension instead of quasi-projective dimension. As an application of Theorem \ref{teo:qproj}, we recover this result. We observe that for a local ring $(R,\mathfrak{m},k)$, one always has that $\operatorname{qpd}_R k < \infty$ (see \cite[Proposition 3.6(1)]{GJT}). However, $\operatorname{CI-dim}_R k< \infty$ if and only if $R$ is a complete intersection ring. \begin{corollary}\cite[Theorem 4.2]{ArayaYoshino} Let $R$ be a local ring, and let $M$ and $N$ be non-zero $R$-modules such that $\operatorname{P}_R(M,N)< \infty$. If $\operatorname{CI-dim}_R M < \infty$, then \begin{align*} \operatorname{P}_R(M,N)=\operatorname{depth} R -\operatorname{depth} M. \end{align*} \end{corollary} \begin{proof} Since $\cdim_R M < \infty$, there exists a quasi-deformation $R \rightarrow R' \twoheadleftarrow S$ such that $\operatorname{pd}_S (M \otimes_R R')< \infty$. Set $(-)^\prime=(-) \otimes_R R'$. By the flatness of $R \to R'$, we have $\operatorname{P}_R(M,N)=\operatorname{P}_{R'}(M',N')$ and $\operatorname{depth} R - \operatorname{depth} M = \operatorname{depth} R' - \operatorname{depth} M'$. Thus, we may assume that $R=R'$, i.e. $R = S/(\boldsymbol{x})$, where $\boldsymbol{x}$ is a regular sequence on $S$. Since $\operatorname{pd}_S M < \infty$, then $\operatorname{qpd}_R M < \infty$, by \cite[Propostion 3.7]{GJT}. Thus, the desired equality follows directly by Theorem \ref{teo:qproj}. \end{proof} The New Intersection Theorem (that was proved by Peskine and Szpiro \cite{PS} for some cases, and by Hochster \cite{Hoch} in the equal characteristic case and finally by Roberts \cite{Roberts} for unequal characteristic case), shows that if \( M \) and \( N \) are \( R \)-modules, one has the inequality \( \dim N \leq \operatorname{pd}_R M + \dim (M \otimes_R N) \). However, the version with quasi-projective dimension of this inequality, generally does not hold (see \cite[Example 4.7]{arxiv}). The following proposition provides, as a corollary, a context in which this is true. \begin{proposition}\label{prop:int} Let $R$ be a local ring, and let $M$ and $N$ be non-zero $R$-modules such that $\operatorname{P}_R(M,N)< \infty$. Then we have the inequality \[ \dim \operatorname{Ext}^i_R(M, N) + i \leq \operatorname{qpd}_R M + \dim(M \otimes_R N) \quad \text{for all} \ i. \] \end{proposition} \begin{proof} We may assume $\operatorname{qpd}_RM < \infty$. Since \[ \operatorname{Supp} \operatorname{Ext}^i_R(M, N) \subseteq \operatorname{Supp} M \cap \operatorname{Supp} N = \operatorname{Supp}(M \otimes_R N) \quad \text{for all} \ i, \] we have \begin{equation*} \dim \operatorname{Ext}^i_R(M, N) \leq \dim(M \otimes_R N) \quad \text{for all} \ i. \end{equation*} Now, since $\operatorname{qpd}_R M < \infty$, for integers $i$ such that $\operatorname{Ext}_R^i(M,N) \neq 0 $ note that $i \leq \operatorname{qpd}_R M$ by Theorem \ref{teo:qproj}. Then we obtain the desired inequality. \end{proof} \begin{corollary}\label{intersection} Let $R$ be a local ring, and let $M$ and $N$ be non-zero $R$-modules such that $\operatorname{P}_R(M,N)< \infty$. If $N$ is Cohen-Macaulay and $\operatorname{pd}_RN < \infty$, then \begin{align*} \dim N \leq \operatorname{qpd}_R M + \dim M \otimes_R N. \end{align*} \end{corollary} \begin{proof} Since $N$ is Cohen-Macaulay and $\operatorname{pd}_R N < \infty$, it follows from \cite[Proposition 3.5]{Foxby1979} and \cite[Lemma 2.16] {Yassemi1} that $\dim N \leq \dim \operatorname{Ext}_R ^i(M,N)+i$ for some $i$. Hence, the desired inequality follows from Proposition \ref{prop:int}. \end{proof} \subsection{The second case.} Now, we are going to prove that $\operatorname{P}_R(M,N)= \depth M - \depth R$, provided that $\operatorname{P}_R(M,N)<\infty$ holds in the case (2). \begin{lemma}\label{lemm2} Let $R$ be a local ring, and let $M$ and $N$ be non-zero $R$-modules such that $\operatorname{P}_R(M,N)< \infty$ and $\operatorname{qpd}_R N < \infty$. One then has the inequality $\operatorname{P}_R(M,N)\leq \operatorname{G-dim}_RM$. \end{lemma} \begin{proof} We may assume that $\operatorname{G-dim}_R M < \infty$. By contradiction, assume that $p:=\operatorname{P}_R(M,N)>\operatorname{G-dim}_R M$. Since $\operatorname{Ext}_R^i(M,N)=0$ for all $i>p$, then \cite[Lemma 6.11(2)]{GJT} implies that $\operatorname{Ext}_R^p(M,N)=0$. This is a contradiction to the definition of $p$. \end{proof} \begin{fact}[{\cite[Remark 3.3]{ChristensenIyengar2007})}]\label{fact9} Let $R$ be an $R$-module with $\operatorname{G-dim}_RM < \infty$. Then, there exists a short exact sequence $ 0 \to M \to H \to X \to 0$, with $X$ totally reflexive and $\operatorname{pd}_R H = \operatorname{G-dim}_R M$. \end{fact} \begin{theorem}\label{g-dim} Let $R$ be a local ring, and let $M$ and $N$ be non-zero $R$-modules such that $\operatorname{P}_R(M,N)<\infty$. If $\operatorname{G-dim}_R M<\infty$ and $\operatorname{qpd}_R N<\infty$, then $$\operatorname{P}_R(M,N)=\operatorname{G-dim}_R M= \operatorname{depth} R -\operatorname{depth} M.$$ \end{theorem} \begin{proof} Let $p=\operatorname{P}_R(M,N)$. The second equality is just the classical Auslander-Bridger formula (\cite[Theorem 29]{masiek}). We will prove the first equality by induction on $p$. Suppose $p=0$, that is $\Ext_R^i(M,N)=0$ for all $i>0$. Since $\operatorname{G-dim}_R M< \infty$, we can use a Gorenstein dimension approximation of $M$ (see Fact \ref{fact9}), that is, a short exact sequence \begin{equation}\label{q1zp} 0 \to M \to H \to X \to 0, \end{equation} where $X$ is totally reflexive and $\operatorname{pd}_R H=\operatorname{G-dim}_R M$. Since $\operatorname{pd}_R H<\infty$, then $\operatorname{P}_R(H,N)<\infty$. Hence, using the long sequence $\Ext(-, N)$ induced from \eqref{q1zp}, it follows that $\operatorname{P}_R(X, N)<\infty$. Thus, applying Lemma \ref{lemm2}, we have that $\Ext_R^i(X,N)=0$ for all $i>0$. Then using again the long sequence $\Ext(-, N)$ induced from \eqref{q1zp}, it follows that $\Ext_R^i(H,N)=0$ for all $i>0$. Therefore, by \cite[p. 154, Lemma 1(iii)]{Matsu}, $H$ is free. Consequently, the equality $\operatorname{pd}_R H=\operatorname{G-dim}_R M$ says that $\operatorname{G-dim}_R M=0=p$. Now, suppose that $p>0$. We see that $\operatorname{P}_R(\Omega^1M,N)=p-1$ and, by Lemma \ref{lemm2}, $M$ is not totally reflexive. We also have that $\operatorname{G-dim}_R( \Omega^1 M)=\operatorname{G-dim}_R M -1$. By induction, we obtain that $\operatorname{P}_R(\Omega^1M, N)=\operatorname{G-dim}_R(\Omega^1 M)$. Thus, $p-1=\operatorname{G-dim}_R M-1,$ whence $p=\operatorname{G-dim}_R M$, as desired. \end{proof} As an application, we obtain the next proposition that provides a generalization of \cite[Theorem 1.4]{Yassemi1}. \begin{proposition} Let $R$ be a local ring, and let $M$ and $N$ be non-zero $R$-modules such that $\operatorname{P}_R(M,N)<\infty$ and $\operatorname{qpd}_R N < \infty$. One then has the inequality \[ \dim \operatorname{Ext}^i_R(M, N) + i \leq \operatorname{G-dim}_R M + \dim(M \otimes_R N) \quad \text{for all} \ i. \] \end{proposition} \begin{proof} The proof follows the same lines as the proof of Proposition \ref{prop:int}, substituting \(\operatorname{G-dim}_R M\) with \(\operatorname{qpd}_R M\) and Theorem \ref{g-dim} with Theorem \ref{teo:qproj}. \end{proof} \section{A generalization of a result by Ischebeck}\label{section4} Let $R$ be a local ring, and let $M$ and $N$ be non-zero $R$-modules. Ischebeck \cite[2.6]{isch} proved that \( \operatorname{P}_R(M, N) = \operatorname{depth} R - \operatorname{depth} M \), assuming that \( N \) has finite injective dimension. A similar result was established by Sazeedeh \cite{saz}, when \( M \) has finite injective dimension and \( N \) has finite Gorenstein injective dimension. In the main result of this section (Theorem \ref{isch}), we generalize Ischebeck's result considering quasi-injective dimension in place of injective dimension and adding the condition that $\operatorname{P}_R(M,N) < \infty$. \begin{lemma}\label{lemmadepthzero} Let $R$ be a local ring, and let $M$ and $N$ be non-zero $R$-modules such that $\operatorname{P}_R(M,N) < \infty$. If $\operatorname{qid}_R N < \infty$, then $\operatorname{depth} M \leq \operatorname{depth} R$. \end{lemma} \begin{proof} Let $\boldsymbol{x}$ be a maximal $M$-regular sequence. By induction on the length of $M$ and using the long exact sequence $\operatorname{Ext}_R(-,N)$, one can see that $\operatorname{P}_R(M/\textbf{\textit{x}}M,N)<\infty$ and $\operatorname{P}_R(M/\textbf{\textit{x}}M,N)=\operatorname{P}_R(M,N)+\operatorname{depth} M$. By \cite[Corollary 3.5]{Gheibi}, we have $\operatorname{P}_R(M/\textbf{\textit{x}}M,N) \leq \operatorname{depth} R$. Combining this with the previous equality, we see that $\operatorname{P}_R(M,N)+\depth M \leq \depth R$. Thus, since $\operatorname{P}_R(M,N)\geq 0$ (see \ref{claim2.3}), we obtain that $\operatorname{depth} M \leq \operatorname{depth} R$. \end{proof} \begin{lemma}\label{lema:notation} Let $(R,\mathfrak{m},k)$ be a local ring, and let $M$ and $N$ be non-zero $R$-modules with $\operatorname{P}_R(M,N)< \infty$ and $\operatorname{qid}_R N < \infty$. Let $$I = ( 0 \rightarrow I_0 \xrightarrow{\partial_0} I_{-1} \xrightarrow{\partial_{-1}} I_{-2} \rightarrow \cdots ) $$ be a bounded quasi-injective resolution of $N$ such that $\operatorname{qid}_R N = \operatorname{hinf} I - \operatorname{inf} I$ and $\operatorname{sup} I= \operatorname{hsup} I=0$ (such $I$ always exists, by \cite[Remark 2.3(3)]{Gheibi}). Set $s=\operatorname{hinf} I$, $Z_i=Z_i(I)$ and $B_i=B_i(I)$, for all $i \in \mathbb{Z}$. Then \begin{enumerate} \item $\operatorname{P}_R(M,N)=\operatorname{P}_R(M,Z_s)$. \item If $t=\operatorname{depth} R >0$, then $\operatorname{Ext}_R^t(k,Z_s) \neq 0$. \end{enumerate} \end{lemma} \begin{proof} (1) Set $p=\operatorname{P}_R(M,N)$ and $p_s=\operatorname{P}_R(M,Z_s)$. For all $i \in \mathbb{Z}$, there are exact sequences \begin{align}\label{seqs} 0 \rightarrow Z_i \rightarrow I_i \rightarrow B_{i-1} \rightarrow 0 \text{ and } 0 \rightarrow B_i \rightarrow Z_i \rightarrow H_i(I) \rightarrow 0. \end{align} We claim that $\Ext_R^{i>p}(M,B_{-j})=0$ and $\Ext^{i>p}_R(M,Z_{-j})=0$ for all $j \geq 0$. Indeed, it is clear that $\Ext_R^{i > p} (M,B_0)=0$ since $B_0=0$. By using the exact sequence $0 \rightarrow Z_0 \rightarrow I_0 \rightarrow B_{-1} \rightarrow 0 $ and noting that $Z_0 \cong \oplus^{b_0} N$ for some positive integer $b_0$, the fact that $\Ext^{i>p}_R (M,N)=0$ allow us to obtain that $\Ext^{i>p}_R(M,B_{-1})=0$. Now, considering the exact sequence $0 \rightarrow B_{-1} \rightarrow Z_{-1} \rightarrow H_{-1}(I) \rightarrow 0$, we can see that $\Ext_R^{i>p}(M,Z_{-1}) =0$. Therefore, using the exact sequences of \eqref{seqs}, we conclude inductively the vanishing of desired Ext-modules. In particular, we have $\operatorname{Ext}_R^{i>p}(M,Z_s)=0$ and thus $p_s \leq p$. To prove the desired equality, suppose by contradiction that $p_s < p$. Then $\operatorname{Ext}_R^p(M,Z_s)=0$. Since $\operatorname{H}_s(I)\cong \oplus^{b_s} N$ for some positive integer $b_s$, there exists a short exact sequence $$0 \rightarrow B_s \rightarrow Z_s \rightarrow \oplus^{b_s} N \rightarrow 0.$$ Thus from its long exact sequence $\operatorname{Ext}_R(M,-)$, using the fact that $\operatorname{Ext}_R^{p+1}(M,B_s)=0$, we see that $\Ext_R^p(M,N)=0$, a contradiction to definition of $p$. Hence, $p=p_s$. (2) We have that $t=\operatorname{qid}_R N = \operatorname{id}_R Z_s$, by Theorem \ref{inj}. For a prime ideal $\mathfrak{p} \neq \mathfrak{m}$ of $R$ and $x \in \mathfrak{m} \backslash \mathfrak{p}$, consider the exact sequence $0 \longrightarrow R / \mathfrak{p} \xrightarrow{x} R / \mathfrak{p}$. This yields an exact sequence $$ \operatorname{Ext}_R^t\left(R / \mathfrak{p}, Z_s\right) \xrightarrow{x} \operatorname{Ext}_R^t\left(R / \mathfrak{p}, Z_s\right) \longrightarrow 0 . $$ Since $t>0$, then \cite[Lemma 3.1]{Gheibi} says that $\operatorname{Ext}_R^t(R/\mathfrak{p},Z_s)$ is finitely generated, and therefore, by Nakayama's Lemma, we have that $\operatorname{Ext}_R^t\left(R / \mathfrak{p}, Z_s\right)=0$. Since $\operatorname{id}_R Z_s=t$, we must then have $\operatorname{Ext}_R^t\left(k, Z_s\right) \neq 0$ by \cite[Corollary 3.1.12]{BH}. \end{proof} Actually, item (2) is deduced from \cite[proof of Theorem 3.2]{Gheibi}. \begin{theorem}\label{isch} Let $(R,\mathfrak{m},k)$ be a local ring, and let $M$ and $N$ be non-zero $R$-modules with $\operatorname{P}_R(M,N)<\infty$. If $\operatorname{qid}_R N < \infty$, then \begin{align*} \operatorname{P}_R(M,N)=\operatorname{depth} R- \operatorname{depth} M. \end{align*} \end{theorem} \begin{proof} If $\operatorname{depth} R =0$, Lemma \ref{lemmadepthzero} yields that $\operatorname{depth} M=0$ and $\operatorname{P}_R(M,N)=0$ by \cite[Corollary 3.5]{Gheibi}. Thus, the desired equality holds. So, we may assume $t=\operatorname{depth} >0$. Consider the notation introduced in Lemma \ref{lema:notation}. By Theorem \ref{inj}, we have that $t=\operatorname{qid}_R N = \operatorname{id}_RZ_s$. By Lemma \ref{lema:notation}(1), it is enough to prove that $$\operatorname{P}_R(M,Z_s)= \operatorname{depth} R - \operatorname{depth} M.$$ We prove it by induction on $\depth M$. If $\operatorname{depth} M=0$, then there exists an exact sequence $$0 \rightarrow k \rightarrow M \rightarrow C \rightarrow 0.$$ This induces an exact sequence $$\Ext_R^t(M, Z_s) \rightarrow \Ext_R^t(k, Z_s) \rightarrow \Ext_R^{t+1}(C,Z_s)=0.$$ Then $\Ext_R^t(M,Z_s)\not=0$ since $\operatorname{Ext}_R^t(k,Z_s) \neq 0$ by Lemma \ref{lema:notation}(2). Additionally, we have that $\Ext_R^i(M, Z_s)=0$ for $i>t$ as $t=\operatorname{id}_R Z_s$. Thus, $\operatorname{P}_R(M,Z_s)=t=\depth R$, as desired. Now, assume that $r:=\depth M>0$. Let $x \in \mathfrak{m}$ be an $M$-regular element. Consider the exact sequence \begin{align}\label{seq5} 0 \rightarrow M \xrightarrow{x} M \rightarrow M/xM \rightarrow 0. \end{align} Since $\operatorname{P}_R(M,N)<\infty$, using its long exact sequence $\Ext_R(-,N)$, we see that \linebreak $\operatorname{P}_R(M/xM, N)<\infty$. Then, by induction, $\operatorname{P}_R(M/xM,Z_s)=t-r+1$. The exact sequence \eqref{seq5} induces for all $i\geq 0$ an exact sequence \begin{equation}\label{p9q8} \Ext_R^i(M, Z_s) \xrightarrow{x} \Ext_R^i(M, Z_s) \rightarrow \Ext_R^{i+1}(M/xM,Z_s) \rightarrow \operatorname{Ext}_R^{i+1} (M,Z_s) \end{equation} which consist of finitely generated $R$-modules when $i>0$ by \cite[Lemma 3.1]{Gheibi}. By Lemma \ref{lemmadepthzero}, we have that $t-r \geq 0 $. If we consider $i>t-r$ in \eqref{p9q8}, then $\operatorname{Ext}_R^{i+1}(M/xM,Z_s)=0$ as $\operatorname{P}_R(M/xM,Z_s)=t-r+1$, whence, by Nakayama's Lemma, $\Ext_R^{i}(M, Z_s)=0$. If we take $i=t-r$ in \eqref{p9q8}, then $\operatorname{Ext}_R^{t-r}(M,Z_s)\not=0$ since $\Ext_R^{t-r+1}(M/xM,Z_s)\not=0=\Ext_R^{t-r+1}(M,Z_s)$. Therefore, $\operatorname{P}_R(M,Z_s)=t-r$. \end{proof} The following example of Khatami and Yassemi \cite{Khatami1} demonstrates that the condition \( \operatorname{P}_R(M, N) < \infty \) in Theorems \ref{teo:qproj}, \ref{g-dim} and \ref{isch} cannot be omitted. \begin{example} Let $(R,\mathfrak{m},k)$ be a Gorenstein local ring which is not regular. The residue field $k$ has finite quasi-projective, quasi-injective and Gorenstein dimensions (see \cite[Proposition 3.6(1)]{GJT}, \cite[Proposition 2.8(1)]{Gheibi} and \cite[Theorem 17]{masiek}, respectively). On the other hand, since $R$ is non-regular, we must have that $\operatorname{pd}_R k = \infty$ and $\operatorname{P}_R(k,k)=\infty$. \end{example} \section{Grade and quasi-homological dimensions}\label{section5} In this section, we present results concerning the relation between the grade and the quasi-homological dimensions. \subsection{Grade, quasi-projective dimension and quasi-perfect modules} In this subsection, we provide a relation between grade and quasi-projective dimension, and introduce a new class of modules, called the quasi-perfect modules. It is well known that for an $R$-module $M$ the following chain of inequalities holds: $\operatorname{grade} M \leq \operatorname{G-dim}_R M \leq \operatorname{pd}_R M$. Then the following question arises naturally. \begin{question} Let $M$ be an $R$-module. Does the inequality $\operatorname{grade} M \leq \operatorname{qpd}_R M$ hold? \end{question} The next theorem says that the answer to this question is affirmative. \begin{theorem}\label{theo4.5} Let $M$ be a non-zero $R$-module. One then has the inequality $\operatorname{grade} M \leq \operatorname{qpd}_R M$. \end{theorem} \begin{proof} For each $\mathfrak{p} \in \operatorname{Supp} M$, we have that $\operatorname{grade} M \leq \operatorname{grade}_{R_\mathfrak{p}}M_{\mathfrak{p}}$ and, by \cite[Proposition 3.5(1)]{GJT}, $\operatorname{qpd}_{R_{\mathfrak{p}}}M_{\mathfrak{p}} \leq \operatorname{qpd}_R M$. In view of this, we may assume that $R$ is local. We will proceed by induction on $\operatorname{grade} M$. If $\operatorname{grade} M=0$, it is clear that the inequality holds. Suppose $\operatorname{grade} M > 0$. Hence there exists an $R$-regular element $x$ such that $xM=0$ by \cite[p. 129, Theorem 16.6]{Matsu}. We may assume that $\operatorname{qpd}_R M<\infty$. Then, by \cite[Proposition 2.11(1)]{Gheibi}, there exists a positive integer $n$ such that $\operatorname{qpd}_{R/(x^n)} M < \infty$. Then we have \begin{align*} \operatorname{qpd}_{R/(x^n)} M &= \depth_{R/(x^n)} R/(x^n)-\depth_{R/xR} M\\&=\depth R-1-\depth_R M \\&=\operatorname{qpd}_R M -1, \end{align*} where the first and third equalities follow from the Auslander-Buchsbaum formula for quasi-projective dimension, while the second equality is due to \cite[Exercise 1.2.26(b)]{BH}. Moreover since $\operatorname{grade} M>0$, from from \cite[p. 140, Lemma 2(i)]{Matsu}, one can derived $ \operatorname{grade}_{R/(x^n)} M=\operatorname{grade} M -1$. Hence, by induction, $\operatorname{grade}_{R/(x^n)} M \leq \operatorname{qpd}_{R/(x^n)} M$. Consequently, $\operatorname{grade} M \leq \operatorname{qpd}_R M$. \end{proof} \begin{definition} Let $M$ be an $R$-module. We define the {\it Cohen-Macaulay defect} of \( M \), denoted by \( \operatorname{cmd} M\), as the difference \(\dim M - \operatorname{depth} M\). \end{definition} It is clear that $M$ is Cohen-Macaulay as an $R$-module if and only if $\operatorname{cmd} M =0$. The next theorem relates the grade of two modules of finite quasi-projective dimension with their quasi-projective dimensions. \begin{proposition}\label{Proposition5.6} Let \( R \) be a local ring, and let \( M \) and \( N \) be non-zero \( R \)-modules of finite quasi-projective dimensions. Then \[ \qpd_R M -\operatorname{grade}(M, N) \leq \qpd_R N + \operatorname{cmd} M. \] \end{proposition} \begin{proof} By Facts \ref{facts3}(1), we have the inequality $ -\operatorname{grade}(M, N) \leq \dim M-\operatorname{depth} N$. Thus $$\depth R-\depth M -\operatorname{grade}(M, N) \leq \dim M-\operatorname{depth} N+\depth R-\depth M,$$ and by Auslander-Buchsbaum formula for quasi-projective dimension, we get: \[ \qpd_R M -\operatorname{grade}(M, N) \leq \qpd_R N + \operatorname{cmd} M. \] \end{proof} Let \( M \) be an \( R \)-module. The chain of inequalities \[ \operatorname{grade} M \leq \operatorname{qpd}_R M \leq \operatorname{pd}_R M \] motivates introducing a new concept as a natural generalization of a perfect module considering the quasi-projective dimension. \begin{definition} Let \( M \) and \( N \) be \( R \)-modules with \(\operatorname{qpd}_R M < \infty\). We say that \( M \) is \( N \)-\textit{quasi-perfect} if \(\operatorname{qpd}_R M = \operatorname{grade}(M, N)\). In particular, if \( N = R \), then \( M \) is called \emph{quasi-perfect} if \(\operatorname{qpd}_R M = \operatorname{grade} M\). \end{definition} \begin{remark} (1) If $(R, \mathfrak{m},k)$ is a local ring, then $k$ is a quasi-perfect module (see \cite[Proposition 3.6]{GJT}). More generally, let $R$ be a ring and let $I$ be an ideal of $R$ such that the Koszul complex with respect to a system of generators of $I$ is a quasi-projective resolution of $R/I$ (e.g., when $I$ is a complete intersection or, more broadly, a quasi-complete intersection), then the $R$-module $R/I$ is always quasi-perfect (see \cite[Theorem 7.4(a)]{GJT}). (2) There are quasi-perfect modules that are not perfect neither $G$-perfects. For instance, the residue field $k$ is quasi-perfect but is not perfect (resp. $G$-perfect) unless that $R$ is regular (resp. Gorenstein). (3) Let $R$ be a local ring. If $p^R(M,R)<\infty$ and $M$ is quasi-perfect, then $M$ is $\operatorname{G-}$perfect. As shown using \cite[Proposition 6.14]{GJT}, the classical Auslander-Bridger formula, and the Auslander-Buchsbaum formula for quasi-projective dimension. \end{remark} Next, we establish some results for quasi-perfect modules similar to the classic results for perfect modules. We start presenting a relation between Cohen-Macaulayness and quasi-perfectness. To motivate this, we remind a well-known fact: If $R$ is a Cohen-Macaulay local ring, and $M$ is an $R$-module of finite projective dimension, then $M$ is a Cohen-Macaulay $R$-module if and only if $M$ is perfect (see e.g \cite[Theorem 2.1.5]{BH}). Next proposition generalizes this fact in the context of quasi-perfect $R$-modules. \begin{proposition}\label{prop:eqv} Let $R$ be a local ring, and let $M$ be a non-zero $R$-module with $\operatorname{qpd}_R M < \infty$. Then the following statements hold: \begin{enumerate} \item If $M$ is Cohen-Macaulay, then $M$ is quasi-perfect. \item If $R$ is Cohen-Macaulay and $M$ is quasi-perfect, then $M$ is Cohen-Macaulay. \end{enumerate} \end{proposition} \begin{proof} \begin{enumerate} \item By Theorem \ref{theo4.5}, we have that $\operatorname{grade} M \leq \operatorname{qpd}_R M$ and, since $M$ is Cohen-Macaulay, we have that $\operatorname{qpd}_R M \leq \operatorname{grade} M$ by Proposition \ref{Proposition5.6}. Thus $\operatorname{grade} M = \operatorname{qpd}_R M $, as desired. \item Since $R$ is Cohen-Macaulay and $M$ is quasi-perfect, we have \[ \begin{array}{lll} \dim M &= \operatorname{depth} R - \operatorname{grade} M \quad &\text{(by Facts \ref{facts3} (2))} \\ &= \operatorname{depth} R - \text{qpd}_R M \quad &\text{(by hypothesis)} \\ &= \operatorname{depth} M \quad & \text{(by Theorem \ref{rem:ABF})}. \end{array} \] \end{enumerate} \end{proof} The notion of quasi-perfect ideal was introduced in \cite{Gheibi} as follows. An ideal $I$ of $R$ is said to be \textit{quasi-perfect} if $\operatorname{grade} I:= \operatorname{grade} R/I=\operatorname{qpd}_R R/I$. Thus, observe that an ideal \( I \) of \( R \) is quasi-perfect in the sense of \cite{Gheibi} if and only if the \( R \)-module \( R/I \) is a quasi-perfect \( R \)-module. The following corollary generalizes \cite[Corollary 7.6]{GJT}. \begin{corollary} Let $R$ be a Cohen-Macaulay local ring, and let $I$ be a quasi-perfect ideal of $R$. Then $I$ is a Cohen-Macaulay ideal of $R$. \end{corollary} Let $M$ and $N$ be $R$-modules. We set $\operatorname{q}^R(M,N):=\sup \lbrace i \geq 0 : \operatorname{Tor}_i^R(M,N) \neq 0 \rbrace$. \begin{theorem} \label{theorem1.8} Let $R$ be a local ring. Let $M$ and $N$ be non-zero $R$-modules such that $N$ Cohen-Macaulay and $\operatorname{qpd}_R M < \infty$. If $\operatorname{q}^R(M,N)=0$, then \( M \otimes_R N \) is Cohen-Macaulay if and only if \( M \) is \( N \)-quasi-perfect. \end{theorem} \begin{proof} Since \(\operatorname{Supp}(M \otimes_R N) = \operatorname{Supp} M \cap \operatorname{Supp} N\), from Facts \ref{facts3}(1) and the fact that \( N \) is Cohen-Macaulay, we have $$\operatorname{depth} N - \dim (M \otimes_R N) = \operatorname{grade}(M \otimes_R N, N).$$ Adding \(-\operatorname{depth} (M \otimes_R N)\) to the equality, we get $${\rm cmd}(M \otimes_R N) = \operatorname{depth} N - \operatorname{depth} (M \otimes_R N) - \operatorname{grade}(M \otimes_R N, N).$$ On the other hand, \[ \begin{array}{lll} \operatorname{grade}(M \otimes_R N, N) &=& \inf \{ \operatorname{depth} N_{\mathfrak{p}} \mid \mathfrak{p} \in \operatorname{Supp}(M \otimes_R N) \}\\ &=& \inf \{ \operatorname{depth} N_{\mathfrak{p}} \mid \mathfrak{p} \in \operatorname{Supp} M \cap \operatorname{Supp} N \}\\ &=& \operatorname{grade}(M, N). \end{array} \] Therefore, we have $${\rm cmd}(M \otimes_R N) = \operatorname{depth} N - \operatorname{depth} (M \otimes_R N) - \operatorname{grade}(M, N).$$ Now, by the Auslander-Buchsbaum formula and the depth formula both for quasi-projective dimension (see Theorems \ref{rem:ABF} and \ref{depthformula}), we have $${\rm cmd}(M \otimes_R N) = \operatorname{qpd}_R M - \operatorname{grade}(M, N).$$ The equality shows that \( M \otimes_R N \) is Cohen-Macaulay if and only if \( M \) is \( N \)-quasi-perfect. \end{proof} \begin{proposition}\label{prop:mixed} Let $M$ be a quasi-perfect $R$-module. For any prime ideal $\mathfrak{p} \in \operatorname{Supp} M$ the following are equivalent: \begin{enumerate} \item $\mathfrak{p} \in \operatorname{Ass} M$. \item $\operatorname{depth} R_{\mathfrak{p}}= \operatorname{grade} M$. \end{enumerate} Moreover, for every $\mathfrak{p} \in \operatorname{Ass}(M)$, we have $\operatorname{grade} R/\mathfrak{p} = \operatorname{grade}M$. \end{proposition} \begin{proof} Let $\mathfrak{p} \in \operatorname{Supp} M$. Then \cite[Proposition 3.5(1)]{GJT} and Theorem \ref{theo4.5} yield the following sequence of inequalities: \begin{align}\label{in} \operatorname{grade}M \leq \operatorname{grade}M_{\mathfrak{p}} \leq \operatorname{qpd}_{R_{\mathfrak{p}}}M_{\mathfrak{p}} \leq \operatorname{qpd}_R M. \end{align} Since $M$ is quasi-perfect, all the inequalities of (\ref{in}) become equalities. In particular, $\operatorname{qpd}_{R_\mathfrak{p}} M_\mathfrak{p}=\operatorname{grade} M_\mathfrak{p}$. On the other hand, by the Auslander-Buchsbaum formula for quasi-projective dimension, we get $\operatorname{qpd}_{R_{\mathfrak{p}}}M_{\mathfrak{p}} + \operatorname{depth}M_{\mathfrak{p}} = \operatorname{depth}R_{\mathfrak{p}}$. Then it follows from the equalities that $\operatorname{depth} M_{\mathfrak{p}} = 0$ if and only if $\operatorname{grade} M = \operatorname{depth}R_{\mathfrak{p}}$. Therefore, the equivalence holds. For the second part, assume that $\mathfrak{p} \in \operatorname{Ass}(M)$. Then $ \operatorname{ann}M \subseteq \mathfrak{p} $ and hence, $\operatorname{grade}M=\operatorname{depth}( \operatorname{ann} M, R)\leq \operatorname{depth}(\mathfrak{p}, R)=\operatorname{grade} R/\mathfrak{p}. $ For the other inequality, note that according to what was proved above, we have $\operatorname{grade} M = \operatorname{depth} R_{\mathfrak{p}}$. Since $\operatorname{grade} R/\mathfrak{p}\leq \operatorname{depth} R_\mathfrak{p}$, then $\operatorname{grade} R/\mathfrak{p} \leq \operatorname{grade} M$. \end{proof} It is well-known that a local ring is Cohen-Macaulay if and only if it admits a non-zero Cohen-Macaulay module with finite projective dimension. However, this statement does not hold true when quasi-projective dimension is considered in place of projective dimension. In fact, the residue field of every local ring satisfies these conditions. The next corollary provides criteria for the base ring \( R \) to be Cohen-Macaulay, based on the existence of a Cohen-Macaulay module with finite quasi-projective dimension with an additional hypo\-thesis. \begin{corollary}\label{pros1} Let $R$ be a local ring. The following are equivalent: \begin{enumerate} \item $R$ is Cohen-Macaulay. \item There exists a non-zero Cohen-Macaulay $R$-module $M$ with $\operatorname{qpd}_R M < \infty$ and $\dim M= \operatorname{dim} R - \operatorname{grade}M$. \item There exists a non-zero Cohen-Macaulay $R$-module $M$ with $\operatorname{qpd}_R M < \infty$ and such that $\dim R/\mathfrak{p}+\operatorname{depth} R_{\mathfrak{p}}=\dim R$ for some $\mathfrak{p} \in \operatorname{Ass} M$. \end{enumerate} \end{corollary} \begin{proof} The assertions (1) $\Rightarrow$ (2) and (1) $\Rightarrow$ (3) are trivial, by setting $M=R$. \begin{itemize} \item[(2) $\Rightarrow$ (1)] Since $M$ is Cohen-Macaulay, then $M$ is quasi-perfect, by Proposition \ref{prop:eqv}(1), that is, $\operatorname{grade}M =\operatorname{qpd}_R M$. Therefore, using the Auslander-Buchsbaum formula for quasi-projective dimension and assumption, we see that $\operatorname{dim} R = \operatorname{depth} R$, that is $R$ is Cohen-Macaulay. \item[(3) $\Rightarrow$ (2)] As before, since $M$ is Cohen-Macaulay, then $M$ is quasi-perfect. Proposition \ref{prop:mixed} and \cite[Theorem 2.1.2(a)]{BH} give $\operatorname{depth} R_{\mathfrak{p}}= \operatorname{grade} M$ and $\operatorname{dim} M= \dim R/\mathfrak{p}$, respectively. Substituting these in the equality given by assumption, we obtain $\dim M = \dim R - \operatorname{grade} M$. \end{itemize} \end{proof} \subsection{Grade and quasi-injective dimension} In the main theorem of this subsection, we prove a formula for the grade of finitely generated modules of finite quasi-injective dimension over a local ring. This theorem improves \cite[Theorem 3.7]{Gheibi}. \begin{theorem}\label{teo:forinj} Let $R$ be a local ring and let $M$ be a non-zero $R$-module with $\operatorname{qid}_R M <\infty$. Then $$\operatorname{dim} M=\depth R-\operatorname{grade} M.$$ \end{theorem} \begin{proof} We proceed by induction on $\operatorname{grade} M.$ If $\operatorname{grade} M=0$, from Facts \ref{facts3}(2), we have that $\operatorname{depth}R\leq \operatorname{dim}M$. The opposite inequality is also valid by \cite[Theorem 3.7]{Gheibi}. Thus, we have \(\operatorname{depth} R = \operatorname{dim} M\), and the desired equality holds. Now, assume that $\operatorname{grade} M>0$. Then, by \cite[p. 129, Theorem 16.6]{Matsu}, there exists an $R$-regular element $x$ with $xM=0$. Since $\operatorname{qid}_R M<\infty$, then $\operatorname{qid}_{R/(x^n)} M<\infty$ for some $n>0$ by \cite[Proposition 2.11(2)]{Gheibi}. As $\operatorname{grade}(M)>0$, we can observe from \cite[p. 140, Lemma 2(ii)]{Matsu} that $\operatorname{grade}_{R/(y)} M=\operatorname{grade} M-1$. Therefore, by induction, we have: $$\operatorname{dim}_{R/(x^n)} M=\depth_{R/(x^n)} R/(y)-\operatorname{grade}_{R/(x^n)} M. $$ Thus, $$\operatorname{dim} M=\depth R-1-(\operatorname{grade} M-1),$$ whence the desired equality follows. \end{proof} \begin{remark} One can observe that the equality in the previous theorem can be rewritten as \( \operatorname{qid}_R M = \dim M + \operatorname{grade} M \), using Theorem \ref{inj}. \end{remark} By Bass's Conjecture, a local ring is Cohen-Macaulay if it admits a non-zero Cohen-Macaulay module with finite injective dimension. Again, this statement is not true when quasi-injective dimension is considered in place of injective dimension. In \cite[Corollary 3.8]{Gheibi}, Gheibi proved the following: If there exists a non-zero \( R \)-module \( M \) with maximal Krull dimension and finite quasi-injective dimension. As a corollary, we derive a more general sufficient condition for \( R \) to be Cohen-Macaulay. \begin{corollary}\label{corol:criteria} Let $R$ be a local ring. If there exists a non-zero $R$-module $M$ such that $\operatorname{qid}_R M < \infty$ and $\dim R = \dim M + \operatorname{grade} M$, then $R$ is Cohen-Macaulay. \end{corollary} \begin{corollary} Let $R$ be a local ring, and let $M$ be a non-zero $R$-module. If $M$ is quasi-perfect and $\operatorname{qid}_R M < \infty$, then $M$ is Cohen-Macaulay. \end{corollary} \begin{proof} We have $$\depth R- \depth M=\operatorname{grade} M=\depth R-\dim M,$$ where the first equality is due to $M$ being quasi-perfect and to the Auslander-Buchs\-baum formula for quasi-projective dimension, while the other is because of Theorem \ref{teo:forinj}. Thus, $\depth M=\dim M$, and therefore, $M$ is Cohen-Macaulay. \end{proof} \section{Grade inequalities for modules with finite quasi-projective dimension}\label{section6} In \cite{Yassemi}, the respective authors established some grade inequalities for modules with finite projective dimension or finite Gorenstein dimension. In this section, using similar arguments to those developed in \cite{Yassemi}, we provide some grade inequalities for mo\-dules of finite quasi-projective dimension. The inequality $\operatorname{grade} M \leq \operatorname{qpd}_R M$, established in Theorem \ref{theo4.5}, will play a crucial role in this section. \begin{theorem}\label{teo:grade} Let $M$, $N$ and $L$ be non-zero $R$-modules such that $\operatorname{qpd}_R N< \infty$ and $\operatorname{qpd}_R L < \infty$. If $\operatorname{Supp} M \subseteq \operatorname{Supp} L$, then $$\operatorname{grade} L + \operatorname{grade}(M,L) \leq \operatorname{grade}(M,N)+\operatorname{qpd}_R N.$$ \end{theorem} \begin{proof} Choose $\mathfrak{p} \in \operatorname{Supp} M$ such that $\operatorname{grade}(M,N)=\operatorname{depth}N_{\mathfrak{p}}$. Note that $\operatorname{qpd}_{R_{\mathfrak{p}}} N_{\mathfrak{p}} \leq \operatorname{qpd}_R N < \infty$, by \cite[Proposition 3.5(1)]{GJT}. Thus, by the Auslander-Buchsbaum formula for quasi-projective dimension, we have: \begin{align*} \operatorname{grade}(M,N) & = \operatorname{depth} R_{\mathfrak{p}}-\operatorname{qpd}_{R_{\mathfrak{p}}} N_{\mathfrak{p}} \\ & = \operatorname{depth} L_{\mathfrak{p}} + \operatorname{qpd}_{R_{\mathfrak{p}}} L_{\mathfrak{p}} - \operatorname{qpd}_{R_{\mathfrak{p}}} N_{\mathfrak{p}} \\ & \geq \operatorname{grade}(M,L) + \operatorname{grade} L_{\mathfrak{p}}-\operatorname{qpd}_R N \,\,\, \text{ (by Theorem \ref{theo4.5})} \\ & \geq \operatorname{grade}(M,L) + \operatorname{grade} L-\operatorname{qpd}_R N. \end{align*} \end{proof} \begin{corollary} Let $M$ and $L$ be non-zero $R$-modules such that $\operatorname{qpd}_R L < \infty$. If $\operatorname{Supp} M \subseteq \operatorname{Supp} L$, then \begin{align*} \operatorname{grade}(M,L) + \operatorname{grade} L \leq \operatorname{grade} M. \end{align*} \end{corollary} \begin{proof} It follows directly by Theorem \ref{teo:grade}, setting $N=R$. \end{proof} \begin{corollary} Let $M$ and $N$ be non-zero $R$-modules such that $\operatorname{qpd}_R N < \infty$. One then has the inequality \begin{align*} \operatorname{grade} M \leq \operatorname{grade}(M,N) + \operatorname{qpd}_R N. \end{align*} \end{corollary} \begin{proof} It follows directly by Theorem \ref{teo:grade}, setting $L=R$. \end{proof} \begin{corollary} Let $M$ and $N$ be non-zero $R$-modules such that $\operatorname{qpd}_R N < \infty$ and $\operatorname{Supp} M \subseteq \operatorname{Supp} N$. Then \begin{align*} \operatorname{grade} (M,N) + \operatorname{grade} N \leq \operatorname{grade} M \leq \operatorname{grade}(M,N)+ \operatorname{qpd}_R N. \end{align*} In particular, if $N$ is quasi-perfect, then $\operatorname{grade}(M,N)=\operatorname{grade} M - \operatorname{grade}N$. \end{corollary} \begin{theorem} Let $R$ be a local ring, and let $M$ and $N$ be non-zero $R$-modules such that $\operatorname{qpd}_R N < \infty$ and $\operatorname{P}_R (M,N)=0$. Then, for any $R$-module $L$, we have: \begin{enumerate} \item $\operatorname{grade}(L,\operatorname{Hom}_R(M,N))+ \operatorname{qpd}_R N \geq \operatorname{grade} L$. \item If $\operatorname{Supp} L \subseteq \operatorname{Supp}(\operatorname{Hom}_R (M,N))$, then $\operatorname{grade}L \geq \operatorname{grade}(L,\operatorname{Hom}_R (M,N))+\operatorname{grade}N$. \end{enumerate} In particular, if $\operatorname{Supp} L \subseteq \operatorname{Supp}(\operatorname{Hom}_R (M,N))$ and $N$ is quasi-perfect, then the equalities hold. \end{theorem} \begin{proof} \begin{enumerate} \item Choose $\mathfrak{p} \in \operatorname{Supp}L$ such that $$\operatorname{grade}(L,\operatorname{Hom}_R (M,N))=\operatorname{depth}( \operatorname{Hom}_{R_\mathfrak{p}}(M_\mathfrak{p},N_\mathfrak{p})).$$ Since $\operatorname{P}_R(M,N)=0$, then it is easy to see that $\operatorname{P}_{R_{\mathfrak{p}}}(M_{\mathfrak{p}},N_{\mathfrak{p}})=0$. Thus, by \cite[Lemma 4.1]{ArayaYoshino}, we have that $$\operatorname{grade}(L,\operatorname{Hom}_R (M,N))=\operatorname{depth} (\operatorname{Hom}_{R_\mathfrak{p}}(M_\mathfrak{p},N_\mathfrak{p}))=\operatorname{depth} N_\mathfrak{p}.$$ Again, note that $\operatorname{qpd}_{R_{\mathfrak{p}}} N_{\mathfrak{p}} \leq \operatorname{qpd}_R N < \infty$ by \cite[Proposition 3.5(1)]{GJT}. Therefore, by the Auslander-Buchsbaum formula for quasi-projective dimension, we have: \begin{align*} \operatorname{grade}(L,\operatorname{Hom}_R (M,N)) & = \operatorname{depth} R_{\mathfrak{p}}- \operatorname{qpd}_{R_\mathfrak{p}} N_{\mathfrak{p}} \\ & \geq \operatorname{grade} L - \operatorname{qpd}_R N. \end{align*} \item Choose $\mathfrak{p} \in \operatorname{Supp} L$ such that $\operatorname{grade} L = \operatorname{depth} R_{\mathfrak{p}}$. Again, since $\operatorname{P}_R(M,N)=0$, we have that $\operatorname{depth} (\operatorname{Hom}_{R_\mathfrak{p}}(M_\mathfrak{p},N_\mathfrak{p}))=\operatorname{depth} N_\mathfrak{p}$. Therefore, by the Auslander-Buchsbaum formula for quasi-projective dimension, we have: \begin{align*} \operatorname{grade} L & = \operatorname{depth} N_{\mathfrak{p}} + \operatorname{qpd}_\mathfrak{p} N_\mathfrak{p} \\ & = \operatorname{depth}( \operatorname{Hom}_{R_\mathfrak{p}}(M_\mathfrak{p},N_\mathfrak{p})) + \operatorname{qpd}_{R_{\mathfrak{p}}} N_{\mathfrak{p}} \\ & \geq \operatorname{grade}(L,\operatorname{Hom}_R(M,N)) + \operatorname{grade} N_{\mathfrak{p}} \; \text{ (by Theorem \ref{theo4.5})} \\ & \geq \operatorname{grade}(L,\operatorname{Hom}_R(M,N)) + \operatorname{grade} N. \end{align*} \end{enumerate} \end{proof} In the next theorem, which is the main result of this section, we present a refined version of the second inequality of \cite[Theorem 3.1]{ArayaYoshino}, replacing the projective dimension with the quasi-projective dimension. Furthermore, we recover the first inequality by introducing an additional hypothesis. \begin{theorem}\label{genary} Let $R$ be a local ring, and let $M$ and $N$ be non-zero $R$-modules such that $\operatorname{qpd}_R N< \infty $ and $\operatorname{q}^R(M,N)=0$. Then, for any $R$-module $L$, we have: \begin{enumerate} \item $\operatorname{grade}(L,M) \leq \operatorname{grade}(L,M \otimes_R N)+ \operatorname{qpd}_R N $. \item If $\operatorname{Supp} L \subseteq \operatorname{Supp} N$, then $\operatorname{grade}(L,M \otimes_R N) + \operatorname{grade} N \leq \operatorname{grade}(L,M)$. \end{enumerate} In particular, if $\operatorname{Supp} L \subseteq \operatorname{Supp} N$ and $N$ is quasi-perfect, then the equalities hold. \end{theorem} \begin{proof} \begin{enumerate} \item Choose $\mathfrak{p} \in \operatorname{Supp} L$ such that $\operatorname{grade}(L,M \otimes_R N) =\operatorname{depth}(M \otimes_R N)_{\mathfrak{p}}$. As $\operatorname{q}^R(M,N)=0$, then it is easy to see that $\operatorname{q}^{R_{\mathfrak{p}}}(M_{\mathfrak{p}},N_{\mathfrak{p}})=0$. Therefore, by the Auslander-Buchsbaum formula and the depth formula for quasi-projective dimension (see Theorems \ref{depthformula} and \ref{rem:ABF}) we have that: \begin{align*} \operatorname{grade}(L,M \otimes_R N) & = \operatorname{depth}(M \otimes_R N)_{\mathfrak{p}} \\ & = \operatorname{depth}(M_{\mathfrak{p}} \otimes_{R_{\mathfrak{p}}} N_{\mathfrak{p}})\\ & = \operatorname{depth} M_{\mathfrak{p}}-\operatorname{qpd}_{R_{\mathfrak{p}}} N_{\mathfrak{p}} \\ & \geq \operatorname{grade}(L,M)- \operatorname{qpd}_R N. \end{align*} \item Choose $\mathfrak{p} \in \operatorname{Supp} L $ such that $\operatorname{grade}(L,M) = \operatorname{depth} M_{\mathfrak{p}}$. Again, we have that as $\operatorname{q}^R(M,N)=0$, then we see that $\operatorname{q}^{R_{\mathfrak{p}}}(M_{\mathfrak{p}},N_{\mathfrak{p}})=0$. Another time, by the Auslander-Buchsbaum formula and the depth formula for quasi-projective dimension, we have that: \begin{align*} \operatorname{grade}(L,M) & = \operatorname{depth} (M_{\mathfrak{p}} \otimes_{R_{\mathfrak{p}}} N_{\mathfrak{p}}) + \operatorname{qpd}_{R_{\mathfrak{p}}} N_{\mathfrak{p}}.\\ & \geq \operatorname{grade}(L,M \otimes_R N)+ \operatorname{grade} N_{\mathfrak{p}} \; \text{ (by Theorem \ref{theo4.5})} \\ & \geq \operatorname{grade}(L,M\otimes_R N) + \operatorname{grade}N . \end{align*} \end{enumerate} \end{proof} \begin{fund} The first author was supported by S\~ao Paulo Research Foundation (FAPESP) under grant 2019/21181-0. The second author was supported by S\~ao Paulo Research Foundation (FAPESP) under grant 2022/12114-0. The third author was supported by S\~ao Paulo Research Foundation (FAPESP) under grants 2022/03372-5 and 2023/15733-5 \end{fund} \begin{agra} Part of this work has been done when the third author was visiting the University of Nebraska-Lincoln. He is grateful for the hospitality. He thanks Roger and Sylvia Wiegand for some discussions. \end{agra} \begin{thebibliography}{PTW02} \bibitem{Araya2} Araya, T. (2019). A homological dimension related to AB rings. \textit{Beitr. Algebra Geom.} 60:225--231. \bibitem{ArayaTakahashi2022} Araya, T., Takahashi, R. (2022). On reducing homological dimensions over noetherian rings. \textit{Proc. Amer. Math. Soc.} 150(2):469--480. \bibitem{ArayaYoshino} Araya, T., Yoshino, Y. (1998). Remarks on a depth formula, a grade inequality and a conjecture of Auslander. \textit{Commun. Algebra} 11:3793--3806. \bibitem{Auslander1967} Auslander, M. (1967). \textit{Anneaux de Gorenstein et torsion en algèbre commutative.} Séminaire d'algèbre commutative 1966/67, notes by M. Mangeneÿ, C. Peskine, L. Szpiro. Paris: École Normale Supérieure de Jeunes Filles. \bibitem{AuBr} Auslander, M., Bridger, M. (1969). \textit{Stable Module Theory.} Province, RI: American Mathematical Society. \bibitem{avramov2} Avramov, L. L., Gasharov, V. N., Peeva, I. V. (1997). Complete intersection dimension. \textit{Publ. Math. I.H.E.S.} 86:67--114. \bibitem{BH} Bruns, W., Herzog, J. (1998). \textit{Cohen-Macaulay Rings}. Cambridge Studies in Advanced Mathematics, 39. Rev. ed. Cambridge: Cambridge University Press. \bibitem{CelikbasIimaSadeghiTakahashi2018} Celikbas, O., Iima, K. I., Sadeghi, A., Takahashi, R. (2018). On the ideal case of a conjecture of Auslander and Reiten. \textit{Bull. Sci. Math.} 142:94--107. \bibitem{Celikbaset} Celikbas, O. et al. (2024). Depth formula for modules of finite reducing projective dimension. \textit{Ann. Mat. Pura Appl.} :1--20. \bibitem{ChristensenIyengar2007} Christensen, L. W., Iyengar, S. (2007). Gorenstein dimension of modules over homomorphisms. \textit{J. Pure Appl. Algebra} 208(1):177--188. \bibitem{GLCMHGD} Divaani-Aazar, K., Hajikarimi, A. (2011). Generalized Local Cohomology Modules and Homological Gorenstein Dimensions. \textit{Commun. Algebra}, 39(6):2051--2067. \bibitem{new} Divaani-Aazar, K., Aghjeh Mashhad, F. M., Tousi, M. (2013). On the Existence of Certain Mo\-dules of Finite Gorenstein Homological Dimensions. \textit{Commun. Algebra} 42(4):1630–1643. \bibitem{Foxby1} Foxby, H. B. (1975). Quasi-perfect modules over Cohen-Macaulay rings. \textit{Math. Nachr.} 66:103--110. \bibitem{Foxby1979} Foxby, H. B. (1979). Bounded complexes of flat modules. \textit{J. Pure Appl. Algebra} 15(2):149--172. \bibitem{GJT} Gheibi, M., Jorgensen, D. A., Takahashi, R. (2021). Quasi-projective dimension. \textit{Pacific J. Math.} 312(1):113--147. \bibitem{Gheibi} Gheibi, M. (2024). Quasi-injective dimension. \textit{J. Pure Appl. Algebra} 228(2):107468. \bibitem{GhoshPuth} Ghosh, D., Puthenpurakal, T. J. (2024). Gorenstein Rings via Homological Dimensions, and Symmetry in Vanishing of Ext and Tate Cohomology. \textit{Algebr. Represent. Theory} 27:639–653. \bibitem{GhoshTakahashi2024} Ghosh, D., Takahashi, R. (2024). Auslander--Reiten conjecture and finite injective dimension of Hom. \textit{Kyoto J. Math.} 64(1):229--243. \bibitem{Hoch} Hochster, M. (1975). \textit{Topics in the Homological Theory of Modules over Commutative Rings.} Amer. Math. Soc. CBMS Regional Conference Series, Vol. 24. \bibitem{isch} Ischebeck, F. (1969). Eine dualitat zwischen den Funktoren Ext und Tor. \textit{J. Algebra} 11:510--531. \bibitem{arxiv} Jorge-Pérez, V. H., Martins, P., Mendoza-Rubio, V. D. (2024). A generalized depth formula for modules of finite quasi-projective dimension. arXiv:2409.08996v2. \bibitem{khatami} Khatami, L., Yassemi, S. (2004). Cohen-Macaulayness of tensor products. \textit{Rocky Mountain J. Math.} 34(1):205--213. \bibitem{Khatami1} Khatami, L., Yassemi, S. (2007). A Bass Formula for Gorenstein Injective Dimension. \textit{Commun. Algebra} 35(6):1882-–1889. \bibitem{MCMtensorproductsandvanishingofExtmodules} Kimura, K., Otake, Y., Takahashi, R. (2022). Maximal Cohen-Macaulay tensor products and vanishing of Ext modules. \textit{Bull. Lond. Math. Soc.} 54(6):2456--2468. \bibitem{masiek} Maşiek, V. (2000). Gorenstein dimension and torsion of modules over commutative Noetherian rings. \textit{Commun. Algebra} 28(12):5783--5811. \bibitem{Matsu} Matsumura, H. (1989). \textit{Commutative ring theory}. Cambridge Studies in Advanced Mathematics, 8. Cambridge: Cambridge University Press. \bibitem{PS} Peskine, C., Szpiro, L. (1973). Dimension projective finie et cohomologie locale. \textit{Inst. Hautes Études Sci. Publ. Math.} 42:47--119. \bibitem{Roberts} Roberts, P. (1987). Le théorème d’intersection. \textit{C. R. Acad. Sci. Paris Sér. I Math.} 304:177–180. \bibitem{VanishingOfCohomologyOverCompleteIntersectionRings} {Sadeghi, A.} (2015). Vanishing of cohomology over complete intersection rings. \textit{Glasg. Math. J.} 57:445--455. \bibitem{saz} Sazeedeh, R. (2013). Gorenstein injective modules and a generalization of Ischebeck formula. \textit{J. Algebra Appl.} 12(4):1250197. \bibitem{Takahashi} Takahashi, R. (2006). The Existence of Finitely Generated Modules of Finite Gorenstein Injective Dimension. \textit{Proc. Amer. Math. Soc.} 134(11):3115--3121. \bibitem{Yassemi1} Yassemi, S. (1995). G-dimension. \textit{Math. Scand.} 77:161--174. \bibitem{Yassemi} Yassemi, S., Khatami, L., Sharif, T. (2001). Grade and Gorenstein dimension. \textit{Commun. Algebra} 29:5085--5094. \bibitem{Yoshida} Yoshida, K. (1998). Tensor products of perfect modules and maximal surjective Buchsbaum modules. \textit{J. Pure Appl. Algebra} 123(1-3):313--326. \end{thebibliography} \end{document} \bibitem{AuBr} M. Auslander, M. Bridger, \emph{Stable module theory}, Mem. Amer. Math. Soc, \textbf{94}, American Mathematical Society, Providence, R.I, 1969. \bibitem{tachikawa} Luchezar L. Avramov, Ragnar-Olaf Buchweitz, and Liana M. \c{S}ega, \textit{Extensions of a dualizing complex by its ring: commutative versions of a conjecture of Tachikawa}, Journal of Pure and Applied Algebra, vol. 201, no. 1-3, (2005) 218--239 \bibitem{GJT} Gheibi, Mohsen; Jorgensen, David A.; Takahashi, Ryo. Quasi-projective dimension. Pacific J. Math. 312 (2021), no. 1, 113--147. \bibitem{BH} W. Bruns and J. Herzog, \emph{Cohen-Macaulay rings}, Cambridge Stud. Adv. Math. \textbf{39}, Cambridge Univ. Press, Cambridge, 1993. \bibitem{pdimtensor} O. Celikbas, S. Dey, and T. Kobayashi, \emph{On the projective dimension of tensor products of modules}, arXiv preprint arXiv:2304.04490, \bibitem{HyperhomologicalAlgebra} L.W. Christensen and H. B. Foxby, \emph{Hyperhomological Algebra with Applications to Commutative Rings}, 2006, available at \url{https://www.math.ttu.edu/~lchriste/download/918-final.pdf}. \bibitem{methods} L.W. Christensen, H.-B. Foxby, H. Holm, Derived category methods in commutative algebra, \url{https://www.math.ttu.edu/~lchriste/download/dcmca.pdf}. \bibitem{FiniteHomologicalDimensionOfHomv3} S. Dey and D. Ghosh, Finite homological dimension of {H}om and vanishing of {E}xt, arXiv preprint arXiv:2310.10607v3, 2023. \bibitem{HomGaGP} S. Dey, R. Holanda, and C.B. Miranda-Neto. Homological dimensions, the Gorenstein property, and special cases of some conjectures, arXiv preprint arXiv:2405.00152v1. \bibitem{ghoshsamanta} D. Ghosh and M. Samanta, \textit{Auslander-Reiten conjecture for modules whose (self) dual has finite complete intersection dimension}, arXiv preprint arXiv:2405.01497v2. \bibitem{injectivedimensiontakahashi} { D. Ghosh, R. Takahashi}, Auslander-Reiten conjecture and finite injective dimension of Hom, Kyoto J. Math. \textbf{64} (2024), 229--243. \bibitem{mendozarubio2023} V. D. Mendoza-Rubio and V. H. Jorge-Pérez, \emph{Auslander-Reiten Conjecture, Finite $C$-Injective Dimension of $\operatorname{Hom}$, and vanishing of $\operatorname{Ext}$}, arXiv preprint arXiv:2312.05914. \bibitem{DualGdim} V. D. Mendoza Rubio and V. H. Jorge-Pérez, \emph{On modules whose dual is of finite Gorenstein dimension: The $k$-torsion, homological criteria, and applications in modules of differentials of order $n$}, arXiv preprint arXiv:2312.06124, 2024. \bibitem{Matsu}{\sc H. Matsumura}, {\it Commutative ring theory}, Cambridge University Press, 1986. \bibitem{Rotman79} J. Rotman, An Introduction to Homological Algebra, Academic Press, New York, 1979. \bibitem{Yassemi} S. Yassemi, \emph{G-dimension}, Math. Scand. \textbf{77} (1995), no. 2, 161--174. \bibitem{Strooker} J.R. Strooker, \textit{Homological Questions in Local Algebra}, Cambridge University Press, Cambridge, 1990. \bibitem{Gheibi} M. Gheibe, Quasi-injective dimension, Journal of Pure and Applied Algebra Volume 228, Issue 2, February 2024, 107468 \bibitem{Yassemi} S. Yassemi, L. Khatami and T. Sharif, Grade and Gorenstein dimension, Comm. Algebra, 29, (2001), 5085--5094.
2412.06705v3
http://arxiv.org/abs/2412.06705v3
Matroids and amplitudes
\documentclass{amsart} \usepackage{amssymb,mathtools} \usepackage[margin=0.9in]{geometry} \usepackage{graphics} \usepackage{hyperref} \usepackage[capitalize]{cleveref} \usepackage{tikz} \usepackage{tikz-cd} \usepackage{bm} \usetikzlibrary{decorations.markings} \usepackage{nccmath} \makeatletter \def\part{ \@startsection{part} {0} {\z@} {\linespacing\@plus\linespacing} {.5\linespacing} {\let\@secnumfont\relax\normalfont\Large\bfseries\raggedright}} \makeatother \newtheorem{conjecture}{Conjecture} \newtheorem{theorem}[conjecture]{Theorem} \newtheorem{lemma}[conjecture]{Lemma} \newtheorem{proposition}[conjecture]{Proposition} \newtheorem{corollary}[conjecture]{Corollary} \newtheorem{definition}[conjecture]{Definition} \newtheorem{defn}[conjecture]{Definition} \newtheorem{exercise}[conjecture]{Exercise} \newtheorem{problem}[conjecture]{Problem} \newtheorem{assumption}[conjecture]{Assumption} \newtheorem{remark}[conjecture]{Remark} \newtheorem{question}[conjecture]{Question} \newtheorem{example}[conjecture]{Example} \def\bT{{\overline{\T}}} \def\dL{{\mathfrak L}} \def\tC{{\tilde C}} \def\tSigma{\Sigma'} \def\one{{\bf 1}} \def\Trop{{\rm Trop}} \def\O{{\mathcal{O}}} \def\Res{{\rm Res}} \def\res{{\rm res}} \def\P{{\mathbb P}} \def\Q{{\mathbb Q}} \def\Z{{\mathbb Z}} \def\C{{\mathbb C}} \def\ZZ{{\mathbb Z}} \def\build{{\mathcal{G}}} \def\Gr{{\rm Gr}} \def\R{{\mathbb R}} \def\Cone{{\rm Cone}} \def\A{{\mathcal{A}}} \def\bA{{\bar {\mathcal{A}}}} \def\M{{\mathcal{M}}} \def\F{{\mathcal{F}}} \def\bU{{\bar U}} \def\N{N} \def\0{{\hat 0}} \def\S{{\mathcal{S}}} \def\T{{\mathcal{T}}} \def\Vol{{\rm Vol}} \def\B{{\mathcal{B}}} \def\tilM{{\widetilde{M}}} \def\tM{{\widetilde\M}} \def\L{{\mathcal{L}}} \def\Frac{{\rm Frac}} \def\dlog{{\rm dlog}} \def\reg{{\rm reg}} \def\dR{{\rm dR}} \def\Spec{{\rm Spec}} \def\GL{{\rm GL}} \def\I{{\mathcal{I}}} \def\balpha{{\bm{\alpha}}} \def\tE{{\widetilde E}} \def\conv{{\rm conv}} \def\Ker{{\rm Ker}} \def\sp{{\rm span}} \def\rk{{\rm rk}} \def\del{\kern-0.8pt{\setminus}\kern-0.8pt} \def\v{{\mathbf{v}}} \def\Fl{{\rm Fl}} \def\A{{\mathcal{A}}} \def\rOS{{\bar\A}} \def\B{{\mathcal{B}}} \def\I{{\mathcal{I}}} \newcommand\ip[1]{\langle #1 \rangle} \def\hi{b} \def\x{{\mathbf{x}}} \def\y{{\mathbf{y}}} \def\v{{\mathbf{v}}} \def\u{{\mathbf{u}}} \def\bchi{{\bar \chi}} \def\ba{{\bar a}} \def\nbc{{\textbf{nbc}}} \def\a{{\mathbf{a}}} \def\b{{\mathbf{b}}} \def\bL{{\bar L}} \def\bE{{\bar E}} \def\bM{{\overline{\M}}} \def\be{{\bar e}} \def\OS{{\rm OS}} \def\lf{{\rm lf}} \def\bOmega{{\overline{\Omega}}} \def\bP{{\overline{P}}} \def\bQ{{\overline{Q}}} \def\tb{{\tilde b}} \def\top{{\rm top}} \def\ty{{\tilde y}} \def\sign{{\rm sign}} \def\codim{{\rm codim}} \def\tT{{\tilde \T}} \def\bG{{\overline{G}}} \def\rOS{{\bar A}} \def\OS{A} \def\Int{{\rm Int}} \def\an{{\rm an}} \def\At{{\mathfrak{A}}} \def\atom{c} \def\Hom{{\rm Hom}} \def\be{{\bar e}} \def\bomega{{\bar \omega}} \def\minL{\hat {\mathfrak o}} \def\pFl{{\Delta}} \def\sep{{\rm sep}} \def\flip{\tilde} \def\emptyflag{\varnothing} \def\tP{{\tilde P}} \def\tQ{{\tilde Q}} \def\image{{\rm image}} \def\z{{\mathbf{z}}} \def\F{{\mathcal{F}}} \def\tF{{\tilde \F}} \newcommand\arxiv[1]{{\tt arXiv:#1}} \newcommand\bip[1]{{\overline{\langle #1 \rangle}}} \newcommand\gBip[1]{{\langle #1 \rangle}_{\L}} \newcommand\gDBip[1]{{\langle #1 \rangle}^{\L}} \newcommand\tipB[1]{{\langle #1 \rangle}'_B} \newcommand\dRip[1]{\langle #1 \rangle^{\dR}} \newcommand\gdRip[1]{\langle #1 \rangle^{\nabla}} \newcommand\gDdRip[1]{\langle #1 \rangle_{\nabla}} \newcommand\dRipp[1]{\langle #1 \rangle^{\dR'}} \newcommand\bdRip[1]{\overline{\langle #1 \rangle}^{\dR}} \newcommand\DdRip[1]{\langle #1 \rangle_{\dR}} \newcommand\tip[1]{{{\langle #1 \rangle'}}} \newcommand\fullip[1]{\langle #1 \rangle^\T} \newcommand\halfip[1]{\langle #1 \rangle} \newcommand\TL[1]{{\bf *** #1 ***}} \numberwithin{conjecture}{section} \numberwithin{equation}{section} \renewcommand{\thepart}{\Roman{part}} \author{Thomas Lam} \address{Department of Mathematics, University of Michigan, 2074 East Hall, 530 Church Street, Ann Arbor, MI 48109-1043, USA} \email{\href{mailto:[email protected]}{[email protected]}} \begin{document} \begin{abstract} In the 1990s, Kita--Yoshida and Cho--Matsumoto introduced intersection forms on the twisted (co)homologies of hyperplane arrangement complements. We give a closed combinatorial formula for these intersection pairings. We show that these intersection pairings are obtained from (continuous and discrete) Laplace transforms of subfans of the Bergman fan of the associated matroid. We compute inverses of these intersection pairings, allowing us to identify (variants of) these intersection forms with the contravariant form of Schechtman--Varchenko, and the bilinear form of Varchenko. Building on parallel joint work with C. Eur, we define a notion of scattering amplitudes for matroids. We show that matroid amplitudes satisfy locality and unitarity, and recover biadjoint scalar amplitudes in the case of the complete graphic matroid. We apply our formulae for twisted intersection forms to deduce old and new formulae for scattering amplitudes. \end{abstract} \title{Matroids and amplitudes} \maketitle \setcounter{tocdepth}{1} \tableofcontents \section{Introduction} The theory of hyperplane arrangements is one of the central topics in algebraic combinatorics and combinatorial algebraic geometry. Let $\bA = \{H_1,H_2,\ldots,H_n\} \subset \P^d$ denote an arrangement of hyperplanes in complex projective space, and let $\bU:= \P^d \setminus \bA$ denote the hyperplane arrangement complement. Brieskorn \cite{Brie}, following ideas of Arnold, showed that the cohomology ring $H^*(\bU)$ is generated by the classes of the 1-forms $df_j/f_j$, where $f_j$ is a linear function cutting out the hyperplane $H_j$. Orlik and Solomon \cite{OS} subsequently described the ring $H^*(\bU)$ by generators and relations, defining the \emph{Orlik-Solomon algebra} $\OS^\bullet(M)$, where $M$ denotes the matroid of $\bA$. Motivated by connections to the theory of hypergeometric functions, attention turned to twisted cohomologies of hyperplane arrangement complements. Gauss's hypergeometric function is distinguished by being the solution to a second-order linear differential equation with three regular singular points. The Aomoto-Gelfand generalized hypergeometric functions \cite{Aom,Gel} generalize Gauss's hypergeometric function by allowing singularities along hyperplanes in $\P^d$. These generalized hypergeometric functions can be viewed as twisted periods, pairings between algebraic deRham twisted cohomology $H^*(\bU, \nabla_\a)$ and Betti twisted homology $H^*(\bU,\L_\a)$ groups. Esnault, Schechtman, and Viehweg \cite{ESV} and Schechtman, Terao, and Varchenko \cite{STV} showed that under a genericity hypothesis \eqref{eq:Mon}, elements of the twisted cohomologies $H^*(\bU, \nabla_\a)$ could again be represented by global algebraic logarithmic forms. Thus $H^*(\bU, \nabla_\a)$ can be identified with the cohomology of the \emph{Aomoto complex} $(\OS^\bullet(M),\omega)$; see \eqref{eq:Aomotocomplex}. In the 1990s, Cho and Matsumoto \cite{CM} and Kita and Yoshida \cite{KY} introduced intersection pairings on these twisted (co)homologies, which we denote \begin{align*} \gdRip{\cdot,\cdot}&: H^*(\bU, \nabla_\a) \otimes H^*(\bU, \nabla_{-\a}) \to \C, \\ \gBip{\cdot,\cdot}&: H_*(\bU, \L_\a) \otimes H_*(\bU, \L_{-\a}) \to \C. \end{align*} The first goal of this work is to give a closed formula for these intersection pairings, which we call the \emph{(twisted) deRham cohomology} (resp. \emph{(twisted) Betti homology}) intersection forms, following the terminology of \cite{BD}. Explicit formulae for these pairings were previously known, for example, in the one-dimensional case \cite{CM,KY}, the case of a generic arrangement \cite{Matgen}, and the braid arrangement \cite{MHhom, Miz}. A general method to compute $\gBip{\cdot,\cdot}$ is given in \cite{KY2}, and this approach is further studied in \cite{Tog}. Our explicit formulae for $\gdRip{\cdot,\cdot}$ and $\gBip{\cdot,\cdot}$ reveal new connections between existing constructions. The Bergman fan $\Sigma_{\bU}$ of $\A$ is a polyhedral fan \cite{Bergman,FS,AK}, in modern language the \emph{tropical variety} associated to the very affine variety $\bU$. First, we show that $\gdRip{\cdot,\cdot}$ and $\gBip{\cdot,\cdot}$ can be expressed as a Laplace transform and a discrete Laplace transform of various subfans of the Bergman fan. In particular, this gives an interpretation of the Cho-Matsumoto twisted period relations as interpolating between continuous and discrete Laplace transforms. Second, we prove that the twisted deRham cohomology intersection form $\gdRip{\cdot,\cdot}$ is essentially equal to the ``contravariant form" of Schechtman and Varchenko \cite{SV}, and the Betti homology intersection form $\gBip{\cdot,\cdot}$ is essentially equal to the inverse of Varchenko's bilinear form on a real hyperplane arrangement \cite{Var}. Our work is heavily motivated by the theory of scattering amplitudes from physics. Cachazo-He-Yuan \cite{CHYarbitrary} introduced around a decade ago a new approach to tree-level scattering amplitudes in various theories: biadjoint scalar, Yang-Mills, gravity, ... This approach relies on the \emph{scattering equations} on the configuration space $M_{0,n+1}$ of $n+1$ points on $\P^1$ to produce rational functions on kinematic space. Mizera \cite{Miz} first observed that the Cachazo-He-Yuan formalism could be interpreted in terms of the twisted intersection forms of \cite{CM,KY} for the hyperplane arrangement complement $\bU = M_{0,n+1}$, and he showed that $\gdRip{\cdot,\cdot}$ and $\gBip{\cdot,\cdot}$ described \emph{biadjoint scalar amplitudes} and the \emph{inverse string theory KLT kernel} respectively. Scattering potentials and scattering equations had appeared earlier in the mathematical literature, especially in work of Varchenko \cite{Varcrit,Varbook,VarBethe,Varquantum} where they were called \emph{master functions} and critical point equations. One of the starting points of our work is to replace the space $M_{0,n+1}$ with an arbitrary (oriented) matroid. We rely on the concurrent parallel work \cite{EL} joint with C. Eur, where we construct \emph{canonical forms for oriented matroids}. The work \cite{EL} shows that topes of oriented matroids can be viewed as positive geometries \cite{ABL,LamPosGeom}, and in the current work we use their canonical forms as an input to the CHY construction of scattering amplitudes. More precisely, the construction of \cite{EL} replaces the \emph{Parke-Taylor forms} from physics, allowing us to systematically use the formalism of matroids in our theory. An eventual goal of this work is to understand the twisted periods of hyperplane arrangement complements in matroid-theoretic terms \cite{Lamstring}. In the last part of this work, we give some immediate applications of our results to physics: a new formula for biadjoint scalar amplitudes using \emph{temporal Feynman diagrams}, a construction of scattering forms for matroids, and new formulae for various determinants of amplitudes. Further applications to physics will be pursued in separate future work. \section{Main results} Let $M$ be the matroid associated to the hyperplane arrangement $\bA$, defined on the ground set $E$, and let $\M$ be the corresponding oriented matroid. Thus $M$ has rank $r = d+1$ where $d$ is the dimension of the projective hyperplane arrangement $\bA$. The lattice of flats of $M$ is denoted $L(M)$, with minimum $\hat 0$ and maximum $\hat 1$. Let $\OS^\bullet(M)$ denote the Orlik-Solomon algebra of $M$, and $\rOS^\bullet(M)$ the reduced Orlik-Solomon algebra; see \cref{sec:OS}. Thus $\rOS^\bullet(M)$ is isomorphic to the cohomology ring $H^*(\bU)$ of the projective hyperplane arrangement complement $\bU$. We always assume that the hyperplane arrangement $\bA$ is essential. We let $\OS(M) = \OS^r(M)$ denote the top homogeneous component of $\OS^\bullet(M)$. Fix $0 \in E$. Let $\T, \T^+, \T^\star, \T^0$ denote the set of topes, the set of topes $P$ satisfying $P(0) = +$, the set of topes bounded with respect to a general extension $\star$, and the set of bounded topes with respect to $0$, respectively. See \cref{sec:matroids}. \subsection{Canonical forms for oriented matroids} A \emph{positive geometry} is a semialgebraic subset $X_{\geq 0}$ of a projective algebraic variety $X$ \cite{ABL,LamPosGeom} satisfying certain axioms. Any positive geometry is equipped (by definition) with a rational top-form $\Omega(X_{\geq 0})$ on $X$, called the \emph{canonical form} of the positive geometry $X_{\geq 0}$. We will not need the full definition of positive geometry in this work. Instead, we note that every full-dimensional (oriented) projective polytope $P \subset \P^d$ is a positive geometry and is thus equipped with a distinguished top-form $\Omega_P$, satisfying the recursion: \noindent (a) if $P$ is a point then $\Omega_P = \pm 1$ depending on orientation, and \noindent (b) if $\dim(P) > 0$, then all the poles of $\Omega_P$ are simple and along facet hyperplanes, and we have the recursion $\Res_F \Omega_P = \Omega_F$, for any facet $F$ of $P$. In \cite{EL}, Eur and the author generalize canonical forms to oriented matroids, showing the existence of distinguished elements in the Orlik-Solomon algebra that play the role of canonical forms. \begin{theorem}[see \cref{thm:EL}]\label{thm:ELintro} To each tope $P \in \T$, there is a distinguished canonical form $\Omega_P \in \OS(M)$, satisfying the recursions of canonical forms. Furthermore, the collection $\{\Omega_P \mid P \in \T^\star\}$ give a basis of $\OS(M)$. \end{theorem} For the case that $P$ is a chamber of a real hyperplane arrangement, the canonical form $\Omega_P$ is the usual one associated to a projective polytope. Canonical forms play a special role in our computations: we will compute our intersection pairings with respect to the basis of \cref{thm:ELintro}. \subsection{Matroid intersection forms} Let $R := \Z[\a] = \Z[a_e \mid e \in E]$ and $S := \Z[\b] = \Z[b_e \mid e \in E]$ be the polynomial rings in variables $a_e$ (resp. $b_e$), and let $Q = \Frac(R)$ and $K = \Frac(S)$ be their fraction fields. When the parameters are specialized to complex numbers, the variables $a_e,b_e$ are related by $b_e = \exp(- \pi i a_e)$ (see \cref{sec:twistedco}). Our main objects of study are two combinatorially defined bilinear forms \begin{align*} \dRip{\cdot,\cdot}&: \OS(M) \otimes \OS(M) \to Q, \\ \halfip{\cdot,\cdot}_B&: \Z^{\T^+} \otimes \Z^{\T^+} \to K, \end{align*} called the \emph{deRham cohomology twisted intersection form} and \emph{Betti homology twisted intersection form} respectively. We remark that $\dRip{\cdot,\cdot}$ is defined for an arbitrary matroid while $\halfip{\cdot,\cdot}_B$ is only defined in the setting of an oriented matroid. The bilinear form $\dRip{\cdot,\cdot}$ is defined (\cref{def:dR}) by using \emph{residue maps} on the Orlik-Solomon algebra, and the bilinear form $\halfip{\cdot,\cdot}_B$ is defined (\cref{def:Bettipair}) directly using the combinatorics of the Las Vergnas lattice of flats. \subsection{Laplace transforms of Bergman fan} We explain the combinatorics of $\dRip{\cdot,\cdot}$ and $\halfip{\cdot,\cdot}_B$ in the language of \emph{Bergman fans}. Bergman \cite{Ber} defined the logarithmic limit-set of an algebraic variety, with the aim of studying the behavior of the variety at infinity. We view Bergman's construction as a \emph{tropical variety}: the set of valuations of points of the variety defined over the field of Puiseux series. When the variety is a linear space, the Bergman fan depends only on the matroid of that linear space. The Bergman fan $\Sigma_M$ of a matroid $M$ was further studied by Ardila and Klivans \cite{AK} and Feichtner and Sturmfels \cite{FS}. We shall consider a particular fan structure on $\Sigma_M$: the maximal cones $C_{F_\bullet}$ are $d$-dimensional cones indexed by $F_\bullet \in \Fl(M)$, where $\Fl(M)$ denotes the set of complete flags of flats of $M$. Other \emph{nested fan structures} on $\Sigma_M$ are considered in \cref{sec:building}. Associated to a tope $P \in \T$, the \emph{Bergman fan of $P$}, $\Sigma_M(P)$, is the subfan of $\Sigma_M$ consisting of all cones $C_{F_\bullet}$ where $F_\bullet \in \Fl(P)$; see \cite{AKW}. In \cref{prop:noover}, we introduce a canonical decomposition of the intersection of positive Bergman fans: for $P,Q \in \T$, we introduce a collection $G^{\pm}(P,Q)$ of partial flags of lattices, and we have $$ \Sigma_M(P) \cap \Sigma_M(Q) = \bigsqcup_{G_\bullet \in G^{\pm}(P,Q)} \Sigma_M(P,G_\bullet), $$ where both sides of the equality are viewed as collections of $d$-dimensional cones. In \cref{sec:Bergman}, we introduce two integral operators $\L$ and $\dL$ called the \emph{continuous Laplace transform} and \emph{discrete Laplace transform} respectively. These operators are defined as an integral and as a sum over lattice points respectively, and produce rational functions in $\a$ and $\b$ respectively when applied to subfans of $\Sigma_M$. \begin{theorem}[\cref{thm:deRhamfan} and \cref{thm:Bettifan}] \label{thm:fan} Let $P,Q \in \T$ be topes. Then \begin{align*} \dRip{\Omega_P,\Omega_Q} &= \sum_{G_\bullet \in G^{\pm}(P,Q)} (\pm)^r (-1)^{\sum_{i=1}^s \rk(G_i)} \L(\Sigma_M(P,G_\bullet)) \\ \halfip{P,Q}_B&= (-1)^d \sum_{G_\bullet \in G^{\pm}(P,Q)} (\pm)^r b(G_\bullet) \dL(\Sigma_M(P,G_\bullet)). \end{align*} In particular, $\dRip{\Omega_P,\Omega_P} = \L(\Sigma_M(P))$ and $\halfip{P,P}_B = (-1)^d \dL(\Sigma_M(P,G_\bullet))$. \end{theorem} The sign $(\pm)^r$ is explained in \cref{thm:deRhamfan}, and the quantity $b(G_\bullet)$ is a signed monomial in the $b$-variables, defined in \cref{def:Bettipair}. We show in \cref{prop:nondeg} and \cref{thm:Bettinondeg} that two bilinear forms are non-degenerate. In \cref{sec:building}, we show that \cref{thm:fan} is compatible with other \emph{nested fan structures} on $\Sigma_M$. \begin{example}\label{ex:3pt} Let $\A$ be the arrangement of three points $\{z_1,z_2,z_3\}$ in $\P^1(\R)$. Thus $M = M(\A) = U_{2,3}$ is the uniform matroid of rank $2$ on three elements $E = \{1,2,3\}$. The Bergman fan $\Sigma_M$ consists of three rays (see \cref{fig:posBerg}), which we draw in $\R^E/\one$. Let $P,Q,R$ be the three topes (modulo negation) given by the intervals $P = [z_1,z_2]$, $Q = [z_2,z_3]$, and $R = [z_3,z_1]$. The intersection $\Sigma_M(P) \cap \Sigma_M(R)$ consists of the single cone $C_{F_\bullet}$ where $F_\bullet = (\hat 0 \subset \{1\} \subset \hat 1)$. By \cref{thm:fan}, we have $$ \dRip{\Omega_P,\Omega_R} = - \frac{1}{a_1}, \qquad \halfip{P,R}_B = -\frac{b_1}{b_1^2-1} = b_1(1+b_1^2 + b_1^4 + \cdots). $$ On the other hand, $\Sigma_M(P)$ is the union of two cones, $C_{F_\bullet}$ and $C_{F'_\bullet}$ where $F'_\bullet = (\hat 0 \subset \{2\} \subset \hat 1)$. By \cref{thm:fan}, we have $$ \dRip{\Omega_P,\Omega_P} = \frac{1}{a_1} + \frac{1}{a_2}, \qquad \halfip{P,P}_B = 1 + \frac{1}{b_1^2-1} + \frac{1}{b_2^2-1} = -\left(1 + (b_1^2 + b_1^4 + \cdots) + (b_2^2 + b_2^4+ \cdots) \right). $$ \begin{figure} \begin{center} \begin{tikzpicture} \draw[->] (0:0) -- (0:1); \node (A1) at (0:1.1) {$1$}; \draw[->] (0:0) -- (90:1); \node (A2) at (90:1.15) {$2$}; \draw[->] (0:0) -- (225:1); \node (A3) at (225:1.15) {$3$}; \node (AA) at (270:1.2) {$\Sigma_M$}; \begin{scope}[shift={(3,0)}] \draw[->] (0:0) -- (0:1); \node (A1) at (0:1.1) {$1$}; \draw[->] (0:0) -- (90:1); \node (A2) at (90:1.15) {$2$}; \node (AA) at (270:1.2) {$\Sigma_M(P)$}; \end{scope} \begin{scope}[shift={(6,0)}] \draw[->] (0:0) -- (90:1); \node (A2) at (90:1.15) {$2$}; \draw[->] (0:0) -- (225:1); \node (A3) at (225:1.15) {$3$}; \node (AA) at (270:1.2) {$\Sigma_M(Q)$}; \end{scope} \begin{scope}[shift={(9,0)}] \draw[->] (0:0) -- (0:1); \node (A1) at (0:1.1) {$1$}; \draw[->] (0:0) -- (225:1); \node (A3) at (225:1.15) {$3$}; \node (AA) at (270:1.2) {$\Sigma_M(R)$}; \end{scope} \begin{scope}[shift={(12,0)}] \draw[->] (0:0) -- (0:1); \node (A1) at (0:1.1) {$1$}; \node (AA) at (270:1.2) {$\Sigma_M(P) \cap \Sigma_M(R)$}; \end{scope} \end{tikzpicture} \end{center} \caption{Positive Bergman fans and their intersections.} \label{fig:posBerg} \end{figure} \end{example} \subsection{Twisted intersection forms} We recall the definition of the intersection forms on twisted (co)homology due to Cho and Matsumoto \cite{CM} and Kita and Yoshida \cite{KY}. For more details, see \cref{sec:twistedco}. Let $\bA$ be a projective hyperplane arrangement, and let $E$ be the indexing set for hyperplanes given by $\{f_e = 0\}$, with $0 \in E$ the hyperplane at infinity. Let $a_e$, $e \in E$ be complex parameters. Consider the meromorphic 1-form $$ \omega = \omega_\a = \sum_e a_e \dlog f_e = \sum_{e \in E \setminus 0} a_e \dlog(f_e/f_0) \in \Omega^1(\bU) $$ on $\bU$, where we assume that $\sum_{e \in E} a_e = 0$, or equivalently, $a_0 = - \sum_{e \in E \setminus 0} a_e$. We have a logarithmic connection $(\O_\bU,\nabla_\a := d + \omega \wedge)$ on the trivial rank one vector bundle $\O_\bU$ on $\bU$. The flat (analytic) sections of $\nabla_\a$ define a complex rank one local system $\L_\a$ on $\bU$. Up to isomorphism, the local system $\L_\a$ is determined by a representation of the fundamental group $\pi_1(\bU)$; the natural generators $\gamma_e, e \in E$ of $\pi_1(\bU)$ are sent to the monodromy values $b_e = \exp(-\pi i a_e)$. When the genericity hypothesis \begin{equation}\label{eq:Mon} a_F = \sum_{e \in F} a_e \notin \Z \mbox{ for all connected }F \in L(M) \setminus \{ \hat 0, \hat 1\} \end{equation} is satisfied, a theorem of Kohno \cite{Koh} (see \cref{thm:Koh}) states that we have \emph{regularization} isomorphisms $$ \reg: H^{\lf}_k(\bU,\L_\a) \stackrel{\cong}{\longrightarrow} H_k(\bU,\L_\a), \qquad \reg: H^k(\bU,\nabla_\a) \stackrel{\cong}{\longrightarrow} H^k_c(\bU,\nabla_\a) $$ between locally-finite (or Borel-Moore) twisted homology and usual twisted homology, and between twisted cohomology and compactly supported twisted cohomology. These isomorphisms are inverse to the natural maps between these (co)homologies. The intersection forms $\gdRip{\cdot,\cdot}$ and $\gBip{\cdot,\cdot}$ are defined by composing the Poincar\'e-Verdier duality pairings with the regularization isomorphism: \begin{align*} \gdRip{\cdot,\cdot}&: H^d(\bU,\nabla^\vee_\a) \otimes H^d(\bU,\nabla_\a) \xrightarrow{{\rm id} \otimes \reg} H^d(\bU,\nabla^\vee_\a) \otimes H^d_c(\bU,\nabla_\a) \xrightarrow{\text{Poincar\'e-Verdier}} \C, \\ \gBip{\cdot,\cdot}&: H^{\lf}_d(\bU,\L^\vee_\a) \otimes H^{\lf}_d(\bU,\L_\a) \xrightarrow{{\rm id} \otimes \reg}H^{\lf}_d(\bU,\L^\vee_\a) \otimes H_d(\bU, \L_\a) \xrightarrow{\text{Poincar\'e-Verdier}} \C. \end{align*} In the deRham case $\gdRip{\cdot,\cdot}$, we view this as a bilinear form on the Aomoto cohomology $\rOS(M,\omega)$ of the Orlik-Solomon algebra, using the result \cref{thm:ESV} of Esnault--Schechtman--Viehweg \cite{ESV}. In the Betti case $\gBip{\cdot,\cdot}$, we choose a basis of twisted cycles with the \emph{standard loading}, and obtain a bilinear form on $\Z^{\T^0}$. In both cases, somewhat surprisingly, the bilinear form turns out to be symmetric. It has long been expected that the intersection forms $\gdRip{\cdot,\cdot}$ and $\gBip{\cdot,\cdot}$ have explicit combinatorial formulae. For instance, we may quote Matsumoto and Yoshida \cite[p. 228]{MYrecent}: ``We expect that these intersection numbers can be expressed combinatorially in a closed form." In \cref{thm:dRpairmain} and \cref{thm:Bettipairmain} we resolve this question in the affirmative. \begin{theorem}\label{thm:combgeom} In the case of a projective hyperplane arrangement, the geometrically defined intersection forms $\gdRip{\cdot,\cdot}$ and $\gBip{\cdot,\cdot}$ agree with the combinatorially defined intersection forms $\dRip{\cdot,\cdot}$ and $\halfip{\cdot,\cdot}_B$ when the parameters satisfy $\sum_{e\in E} a_e =0$ (resp. $\prod_{e \in E} b_e =1$). \end{theorem} The basic approach to the computation of the intersection forms is the same as in the original works \cite{CM,KY}, and carried out in various cases in, for example, \cite{MOY,MY,Goto,Tog,MHcoh,MHhom}. Our key novelty lies in the systematic use of the wonderful compactification $X_{\max}$ of $\bU$ associated to the maximal building set. \begin{remark}\label{rem:descent} For generic parameters, the bilinear form $\dRip{\cdot,\cdot}$ is non-degenerate on $\OS(M)$, but in \cref{sec:Aomoto} we show that when $\sum_e a_e = 0$ is satisfied, the bilinear form $\dRip{\cdot,\cdot}$ descends to the Aomoto cohomology $\rOS(M,\omega)$. Similarly, for generic parameters the bilinear form $\halfip{\cdot,\cdot}_B$ is non-degenerate on $\Z^{\T^+}$, but when $\prod_{e \in E} b_e =1$, the rank drops, and it restricts to a non-degenerate bilinear form on $\Z^{\T^0}$ (see \cref{thm:Bettinondeg}). We view the bilinear forms $\dRip{\cdot,\cdot}$ and $\halfip{\cdot,\cdot}_B$ with generic parameters as the ``correct" combinatorial objects, as they lead to the most elegant combinatorics. We expect these bilinear forms can be geometrically interpreted as {\bf local} twisted intersection forms for the corresponding central hyperplane arrangement. \end{remark} Recall that a very affine variety $U$ is a closed subvariety of a complex torus. The description of the intersection forms in terms of the Bergman fan (\cref{thm:fan}) is especially attractive because of the following natural problem. \begin{problem}\label{prob:Bergman} Generalize \cref{thm:combgeom} to arbitrary very affine varieties $U$ by replacing the Bergman fan $\Sigma_M$ with the tropicalization $\Trop(U)$. \end{problem} We point the reader to \cite[Section 6]{LamModuli} for more discussion in this direction. In the case that $U$ is the uniform matroid stratum of the Grassmannian $\Gr(k,n)$, \cref{prob:Bergman} is related to the study of the generalized biadjoint scalar amplitudes of Cachazo-Early-Guevara-Mizera \cite{CEGM,CEZ,CEZ24}. \subsection{deRham homology intersection form} For a subset $B \subseteq E$, denote $$a^B:= \prod_{b \in B} a_b.$$ For two bounded topes $P,Q \in \T^\star$, we define in \cref{def:DdR} the set $\B(P,Q)$, consisting of all bases $B \in \B(M)$ such that both topes $P$ and $Q$ belong to the \emph{bounded simplex} cut out by $B$. The \emph{deRham homology intersection form} on $\Z^{\T^\star}$ is defined to be $$ \DdRip{P,Q} := \sum_{B \in \B(P,Q)} a^B. $$ \begin{theorem} The bilinear form $\frac{1}{a_E}\DdRip{\cdot,\cdot}$ is the inverse of the bilinear form $\dRip{\cdot,\cdot}$ with respect to the basis $\{\Omega_P \mid P \in \T^\star\}$. \end{theorem} \begin{figure} \begin{center} $$ \begin{tikzpicture}[extended line/.style={shorten >=-#1,shorten <=-#1}, extended line/.default=1cm] \draw[fill=none,dashed](0,0) circle (3.8); \draw[extended line] (90:3) -- (210:3); \draw[extended line] (90:3) -- (330:3); \draw[extended line] (330:3) -- (210:3); \draw[extended line] (90:3) -- (270:3); \draw[extended line] (210:3) -- (30:3); \draw[extended line] (330:3) -- (150:3); \node[color=blue] at (100:4.1) {$(13)$}; \node[color=blue] at (90:4.1) {$(23)$}; \node[color=blue] at (80:4.1) {$(12)$}; \node[color=blue] at (330:4.2) {$(34)$}; \node[color=blue] at (337:4.1) {$(14)$}; \node[color=blue] at (30:4.2) {$(24)$}; \node[color=blue] at (180:4) {$\star$}; \node[color=red] at (120:1) {$1234$}; \node[color=red] at (60:1) {$1324$}; \node[color=red] at (0:1) {$1342$}; \node[color=red] at (-60:1) {$1432$}; \node[color=red] at (-120:1) {$1423$}; \node[color=red] at (-180:1) {$1243$}; \end{tikzpicture} $$ \end{center} \caption{The configuration space of $5$ point on $\P^1$, drawn with a general extension $\star$ at infinity.} \label{fig:M05star} \end{figure} \begin{example}\label{ex:KLTexample} In \cref{fig:M05star} we have drawn the hyperplane arrangement associated to the configuration space $M_{0,5}$, with a general extension $\star$ drawn as the ``circle at infinity". The set $\T^\star$ consists of the six labeled regions bounded with respect to $\star$ and which are labeled by the permutations $w \in S_4$ satisfying $w(1) = 1$. Two simplices contain both $1234$ and $1342$, namely $B = \{(12),(13),(14)\}$ and $\{(12),(13),(34)\}$. One additional simplex $B = \{(12),(13),(24)\}$ contains both $1234$ and $1324$. We obtain $$ \DdRip{1234,1342} = a_{12}a_{13}( a_{14} + a_{34}), \qquad \DdRip{1234,1324} = a_{12}a_{13}( a_{14} +a_{24}+ a_{34}). $$ \end{example} \begin{remark} The elegance of the deRham homology intersection form, and in particular the fact that it is positive, suggests that there is a direct geometric interpretation of this form, without relying on the duality with the deRham cohomology intersection form. \end{remark} \subsection{Betti cohomology intersection form} Given $P,Q \in \T^+$, define the \emph{separating set} $$ \sep(P,Q) := \{ e \in E \setminus 0 \mid P(e) \neq Q(e)\} \subset E. $$ In the case of an affine hyperplane arrangement, these are the set of hyperplanes, not including the plane at infinity, that separate $P$ from $Q$. The Betti cohomology intersection form on $\Z^{\T^+}$ is defined to be $$ \ip{P,Q}^B := b_{\sep(P,Q) }+ (-1)^r b_{E \setminus \sep(P,Q)} = \ip{Q,P}^B $$ for $P,Q \in \T^+$. In fact, $\ip{P,Q}^B$ is actually defined for $P,Q \in \T$, and $\ip{P,Q}^B= (-1)^r \ip{P,-Q}^B$. The following result is \cref{thm:Bettiinverse}. \begin{theorem} The $\T^+ \times \T^+$ matrices $(-1)^{r-1}(1- b_E)^{-1}\ip{\cdot,\cdot}^B_{\T^+}$ and $\ip{\cdot,\cdot}^{\T^+}_B$ are inverse. \end{theorem} \begin{example} Consider the hyperplane arrangement of \cref{fig:M05star} and take $0$ to be the hyperplane $(12)$. Then we have $\halfip{1234,1324}^B = b_{23} - b_{12}b_{13}b_{14}b_{24}b_{34}$ and $\halfip{1234,1423}^B = b_{24}b_{34}-b_{12}b_{13}b_{14}b_{23}$. \end{example} \begin{remark} The elegance of the Betti cohomology intersection form suggests that there is a direct geometric interpretation of this form, without relying on the duality with the Betti homology intersection form. \end{remark} \subsection{Relation to the bilinear forms of Schechtman--Varchenko and Varchenko} In \cite{SV}, motivated by the study of Knizhnik-Zamolodchikov equations, Schechtman and Varchenko introduced a \emph{contravariant form} $\ip{\cdot,\cdot}^{SV}$ on the Orlik-Solomon algebra $\rOS(M)$ of a hyperplane arrangement. Their bilinear form is an analogue of the Shapovalov form of a highest weight representation of a Kac-Moody algebra. The contravariant form is generalized to an arbitrary matroid by Brylawski and Varchenko \cite{BV}, and the restriction of the form to ``singular vectors" (corresponding to the Aomoto cohomology of the Orlik-Solomon algebra) was studied by Falk and Varchenko \cite{FalkVar}. The following result is proved as \cref{cor:SVform}; see also \cref{rem:a0infinity}. \begin{corollary}\label{cor:SV} The Schechtman--Varchenko contravariant form $\ip{\cdot,\cdot}^{SV}$ for a central hyperplane arrangement is equal to the deRham intersection form $\dRip{\cdot,\cdot}$ up to an overall factor of $a_E$. For an affine arrangement, the Schechtman--Varchenko contravariant form $\ip{\cdot,\cdot}^{SV}$ is obtained from the deRham intersection form $\dRip{\cdot,\cdot}$ by evaluating at $a_0 = \infty$. \end{corollary} Schechtman and Varchenko \cite[(4.7.4)]{SV} relate the contravariant form to twisted (co)homology via an asymptotic formula. As described in \cref{rem:BBM} below, Belkale, Brosnan, and Mukhopadhyay \cite{BBM} show that the twisted deRham cohomology intersection form $\gdRip{\cdot,\cdot}$ can be obtained from $\ip{\cdot,\cdot}^{SV}$. This should be compared to our \cref{thm:combgeom} and \cref{cor:SV}. \begin{remark}\label{rem:BBM} Let $\bU$ be a projective hyperplane arrangement with matroid $M$, and let the $a_e$ be generic. View the Schechtman--Varchenko contravariant form as a map $S:\rOS(M)^* \to \rOS(M)$ (\cref{prop:Fk} and \eqref{eq:RS}). Then \cite[(2.7)]{BBM} show that the composition \begin{equation}\label{eq:BBM} \rOS(M, \omega)^* \to \rOS(M)^* \stackrel{S}{\longrightarrow} \rOS(M) \longrightarrow \rOS(M, \omega) \end{equation} can be identified with $\gdRip{\cdot,\cdot}$, after composing with the isomorphism $\rOS(M,\omega) \cong H^*(\bU, \nabla_\a)$. Note that in \eqref{eq:BBM} the bilinear form $\ip{\cdot,\cdot}^{SV}$ (giving rise to the map $S:\rOS(M)^* \to \rOS(M)$) has full rank on $\rOS(M)$, in contrast to our description of $\gdRip{\cdot,\cdot}$ (\cref{rem:descent}). We thank Prakash Belkale for explaining the results of \cite{BBM} to us. \end{remark} In \cite{Var}, Varchenko introduces a bilinear form $\ip{\cdot,\cdot}^V$ on a real configuration of hyperplanes. As Varchenko observes, the contravariant form $\ip{\cdot,\cdot}^{SV}$ is the quasiclassical limit of $\ip{\cdot,\cdot}^V$. The bilinear form $\ip{\cdot,\cdot}^V$ was generalized to the setting of oriented matroids in \cite{HV,Ran}. \begin{corollary}[{\cref{cor:Var}}] Varchenko's bilinear form $\ip{\cdot,\cdot}^V$ is obtainted from the Betti cohomology intersection form $\ip{\cdot,\cdot}^B$ by evaluating at $b_0 = 0$. Equivalently, Varchenko's bilinear form is the inverse of the Betti homology intersection form $\halfip{\cdot,\cdot}_B$, after evaluating at $b_0 = 0$. \end{corollary} This appears to be the first geometric interpretation of Varchenko's bilinear form $\ip{\cdot,\cdot}^V$. Among the deep properties of their contravariant form $\ip{\cdot,\cdot}^{SV}$, Schechtman--Varchenko \cite{SV} proved a formula for its determinant (recalled in \cref{thm:SVdet}), and an analogous determinant for $\ip{\cdot,\cdot}^V$ is given in \cite{Var}. We give variants of these results: in \cref{thm:Aomotodet} we compute the determinant of $\bdRip{\cdot,\cdot}$ on Aomoto cohomology, and in \cref{thm:Bettihomdet}, we compute the determinant of $\halfip{\cdot,\cdot}_B$ on the lattice $\Z^{\T^+}$. \subsection{Scattering amplitudes} Our work is motivated by the theory of scattering amplitudes in physics, and especially the scattering equations of Cahcazo-He-Yuan \cite{CHYarbitrary}. For a survey intended for mathematicians, we refer the reader to \cite{LamModuli}. In the CHY formalism for the scattering of $n+1$ particles, \emph{kinematic space} $K_{n+1}$ (roughly, the space of momentum vectors of $n$ particles) is coupled with the \emph{worldsheet}, the moduli space $M_{0,n+1}$ by \emph{scattering equations} (S.E.). Various scattering amplitudes can then be obtained via the CHY ansatz: $$ {\rm amplitude} = \sum_{\text{solns } p \text{ to S.E.}} f(p) $$ where $f(p)$ is a rational function on $M_{0,n+1}$ evaluated at the solution $p$ to the scattering equations. The choice of function $f(p)$ depends on the specific quantum field theory: biadjoint scalar, Yang-Mills, gravity, and so on. As explained in \cite{LamModuli} and reviewed in \cref{sec:veryaffine}, the \emph{biadjoint scalar} amplitudes can be viewed as functions $A(\Omega,\Omega')$ that depend on the choice of two rational top-forms $\Omega,\Omega'$, and this definition extends the CHY formalism to the setting of very affine varieties. Here, the very affine variety $U$ takes the role of the worldsheet, replacing the moduli space $M_{0,n+1}$. In \cref{sec:amplitude}, we define amplitudes for matroids using the deRham intersection form $\dRip{\cdot,\cdot}$ and the canonical forms of \cref{thm:EL}. We show in \cref{thm:AP} the basic properties of ``locality" and ``unitarity" for matroid amplitudes. This result exposes a surprising parallel between the dichotomy of deletion-contraction in matroid theory and factorization phenomena in quantum field theory. In the case of $U = M_{0,n+1}$, the relationship between twisted cohomology and CHY amplitudes was first observed by Mizera \cite{Miz}, and this equality was proven in a general setting by Matsubara-Heo \cite[Corollary 2.7]{MHcoh}. In \cref{sec:scatform}, we give a new proof of this equality in the case that $U$ is a hyperplane arrangement complement. Our approach relies on the definition of a scattering correspondence \cref{def:scatcorr}, which has appeared in the setting of hyperplane arrangements \cite{CDFV} and in likelihood geometry \cite{Huh,HS}. In \cref{sec:M0n}, we spell out some of our results in the case $U = M_{0,n+1}$, which is the case of the complete graphic matroid $M = M(K_n)$. We obtain a new formula (\cref{thm:temporal}) for biadjoint scalar amplitudes in terms of objects we coin \emph{temporal Feynman diagrams}. We show (\cref{thm:Frost}) that the celebrated field-theory KLT (Kawai-Lewellen-Tye) matrix \cite{BDSV} can be obtained from our results in a form that is different to the existing literature. In \cref{cor:det1} and \cref{cor:det2}, we give new formulae for determinants of matrices of partial amplitudes. We summarize the basic analogies between matroids and quantum field theory in the following table. \begin{center} \begin{tabular}{|c|c|} \hline worldsheet & matroid \\ \hline kinematic space & dual of Lie algebra of intrinsic torus \\ \hline \# of solutions to scattering equations & beta invariant \\ \hline Parke-Taylor form & canonical form of a tope \\ \hline biadjoint scalar partial amplitude & Laplace transform of Bergman fan \\ \hline inverse string KLT matrix & discrete Laplace transform of Bergman fan\\ \hline physical poles & connected flats \\ \hline factorization & deletion-contraction \\ \hline Feynman diagram & flag of flats \\ \hline \end{tabular} \end{center} \subsection{Matroids and motives} We have largely excluded from this work a discussion of the generalized hypergeometric functions \begin{equation}\label{eq:AG} \int_{[P]} \varphi_P \; \Omega \end{equation} studied by Aomoto \cite{Aom} and Gelfand \cite{Gel}. These integral functions are a main motivation for the study of twisted (co)homologies of hyperplane arrangement complements. Indeed, the integrals \eqref{eq:AG} are given by pairings between twisted cocycles $[\Omega] \in H^d(U,\nabla_\a)$ and twisted cycles $[P \otimes \varphi_P] \in H_d(U,\L^\vee_\a)$. As noted in the original work of Cho and Matsumoto \cite{CM}, the computation of the intersection forms $\ip{\cdot,\cdot}^\nabla$ and $\ip{\cdot,\cdot}_{\L}$ leads to explicit period relations for the twisted periods \eqref{eq:AG}. See for example \cite{MOY,MY,Goto}. We briefly discuss twisted period relations in \cref{sec:beta}. The relation to scattering amplitudes suggests one to focus on the special case when $\Omega = \Omega_P$ is a canonical form in \eqref{eq:AG}. The resulting integral functions, which we call \emph{string amplitudes for hyperplane arrangements}, will be studied in the work \cite{Lamstring}. In the special case that $U = M_{0,n+1}$, these functions are the open string theory amplitudes at tree-level; see \cite{AHLstringy,BD,Miz}. Let us explicitly articulate one of the main directions that our work opens up. \begin{problem}\label{prop:motives} For an oriented matroid $\M$, define and study the space of all twisted period matrices $\mathbf{P}^\a$ (as in \cref{sec:beta}) compatible with $\M$. \end{problem} We view \cref{prop:motives} as a step towards \emph{(twisted) motives} for matroids. We have seen that the intersection forms $\dRip{\cdot,\cdot}, \DdRip{\cdot,\cdot}, \halfip{\cdot,\cdot}^B,\halfip{\cdot,\cdot}_B$ exist even for matroids not arising from hyperplane arrangements. A fundamental tension is the question: do the twisted period matrices $\mathbf{P}^\a$ exist when $M$ is a nonrealizable matroid? \subsection*{Acknowledgements} We acknowledge support from the National Science Foundation under grants DMS-1953852 and DMS-2348799. We thank the Simons Foundation for support under a Simons Fellowship. We are grateful to the Institute for Advanced Study, Princeton for supporting a visit during which part of this manuscript was completed. We thank Chris Eur for our parallel joint work on canonical forms for matroids. We thank Hadleigh Frost, June Huh, Sebastian Mizera, Oliver Schlotterer, Bernd Sturmfels, and Simon Telen for stimulating discussions. We thank Prakash Belkale, Nick Early, and Alexander Varchenko for helpful comments on an earlier version of this manuscript. \part{Combinatorics} \section{Matroids}\label{sec:matroids} We denote $[n]:=\{1,2,\ldots,n\}$. \subsection{Conventions for matroids} Let $M$ be a matroid of rank $r = d+1$ with ground set $E$. We use the notation \begin{align*} \rk = \rk_M &= \mbox{rank function of $M$,} \\ \B(M) &= \mbox{set of bases of $M$,} \\ \I_k(M) &= \mbox{$k$-element independent sets of $M$.} \end{align*} An element $e \in E$ is a \emph{loop} if it belongs to no bases, and a \emph{coloop} if it belongs to all bases. Two elements $e, e' \in E$ are called parallel if they belong to the same bases. An element $e \in E$ is in \emph{general position} if $\rk(S \cup e) = \min(\rk(S) + 1,r)$ for any $S \subseteq E \setminus e$. A matroid $M$ is called \emph{simple} if it has no loops and no parallel elements. If $M,M'$ are matroids on the ground sets $E,E'$ with ranks $r, r'$, then the \emph{direct sum} $M\oplus M'$ is the rank $(r+r')$ matroid on the ground set $E \sqcup E'$ with bases $\B(M\oplus M') = \{B \sqcup B' \mid B \in \B(M), B' \in \B(M')\}$. A matroid $M$ is called \emph{connected} or \emph{indecomposable} if it cannot be expressed as a non-trivial direct sum $M = M|_{E_1} \bigoplus M|_{E_2}$ where $E = E_1 \sqcup E_2$. Let $L(M)$ denote the lattice of flats of $M$, and let $L^k(M)$ denote the set of flats of rank $k$. Each flat $F \in L(M)$ is viewed as a subset of $E$. By convention $L(M)$ has minimal element $\hat 0$ (consisting of all the loops) and maximal element $\hat 1 = E$. We use $\vee$ and $\wedge$ to denote the join and meet operations of $L(M)$. A flat $F$ is called \emph{connected} if the restriction $M^F$ (see \cref{ssec:extensions}) is connected. An atom $a \in L(M)$ is a flat of rank one and we let $\At(M)$ denote the set of atoms of $M$. An atom in a loopless matroid consists of an equivalence class of parallel elements of $M$. We say that an atom $a\in \At(M)$ is a coloop if any of the elements in $a$ is a coloop. For an example of $L(M)$, see \cref{fig:5line}. An \emph{affine matroid} $(M,0)$ is a matroid $M$ together with a distinguished element $0 \in E$. In terms of hyperplane arrangements, $0$ indexes the hyperplane at infinity. We say that an affine matroid $(M,0)$ is generic at infinity if $0 \in E$ is in general position. \subsection{Some invariants} We will be interested in the following invariants of a matroid $M$: \begin{align*} \chi_M(t) &= \mbox{characteristic polynomial}\\ \bchi_M(t) &= \mbox{reduced characteristic polynomial}\\ \mu^+(M) &= \mbox{unsigned M\"obius invariant} \\ \beta(M) &= \mbox{beta invariant} \\ w_\Sigma(M) = |\bchi_M(-1)| &= \mbox{(reduced) total Whitney invariant} \end{align*} Let $\mu = \mu_{L(M)}(x,y)$ denote the Mobius function of $L(M)$, where $[x,y]$ is an interval in $L$. For $x \in L$, we set $\mu(x) := \mu(\hat 0, x)$. Let $\mu(M):= \mu(\hat 1)$ denote the \emph{Mobius invariant} of $M$, and let $\mu^+(M) = |\mu(M)|$ denote the unsigned Mobius invariant. Let $\chi_M(t)$ (resp. $\bchi_M(t)$) denote the \emph{characteristic polynomial} (resp. reduced characteristic polynomial) of $M$, given by $$ \chi_M(t):= \sum_{F \in L(M)} t^{r - \rk(F)} \mu(F), \qquad \text{and} \qquad \bchi_M(t) := \chi_M(t)/(t-1). $$ The \emph{beta invariant} $\beta(M)$ of $M$ is given by $$ \beta(M) := (-1)^{r+1} \left.\frac{d}{dt} \chi_M(t) \right|_{t=1}. $$ If $e \in E$ is neither a loop nor a coloop, then we have the recursion \begin{equation}\label{eq:betaeq} \beta(M) = \beta(M/e) + \beta(M\setminus e) \end{equation} We have $\beta(M) = 0$ if and only if $M$ is disconnected, or a loop, or empty ($|E|=0$). \subsection{Extensions and liftings}\label{ssec:extensions} For a flat $F \in L(M)$, we have the matroids \begin{align*} M^F &:= \text{restriction of $M$ to $F$} = \text{deletion of $E \setminus F$ from $M$} \\ M_F&:= \text{contraction of $M$ by $F$}. \end{align*} The lattice $L(M^F)$ of flats of $M^F$ (resp. $L(M_F)$ of flats of $M_F$) is isomorphic to the lower order ideal $[\hat 0, F] \subset L(M)$ (resp. upper order ideal $[F, \hat 1]\subset L(M)$). For an element $e \in E$, we denote by $M\backslash e $ the deletion of $e$, and by $M/e = M_e$ the contraction of $M$ by $e$. We call $(M, M' = M\backslash e, M'' = M/e)$ a deletion-contraction triple. More generally, we have a deletion-contraction triple for any atom $a \in \At(M)$. An \emph{extension} (resp. \emph{lifting}) $\tilM$ of $M$ is a matroid $\tilM$ on $\tE = E \cup \star$ such that the deletion $\tilM \backslash\star$ (resp. contraction $\tilM/\star$) is equal to $M$. The extension or lifting $\tilM$ is called general if the element $\star$ is in general position in $\tilM$. Given a matroid $M$ on $E$ , we often let $(\tilM, \star)$ denote an affine matroid on $\tilde E = E \cup \star$ which is a general extension of $M$ by an element $\star$. \begin{lemma}\label{lem:betageneric} Suppose that $(\tilM,\star)$ is a general extension of a non-loop matroid $M$. Then $\mu^+(M) = \beta(\tilM)$. \end{lemma} \begin{proof} We may assume that $M$ is simple. Then we have $$ \chi_M(t) = \sum_{A \subset E} (-1)^{|A|} t^{r- \rk(A)}, \qquad \chi_{\tilM}(t) = \sum_{A \subset E \cup \star} (-1)^{|A|} t^{r- \rk(A)}. $$ By genericity, if $\star \notin A$ and $\rk(A) < d$ then $\rk(A \cup \star) = \rk(A) + 1$. Also, if $\rk(A) = r$ then $\rk(A \cup \star) = r$. It follows that $$ \chi_{\tilM}(t) = (\chi_M(t)-\chi_M(0)) (1 - 1/t). $$ Thus, $$ (-1)^{r+1} \beta(\tilM) = \left. \frac{d}{dt} \chi_{\tilM}(t) \right|_{t=1} =\left(\chi'_M(t)(1-1/t)-(\chi_M(t)-\chi_M(0))(1/t^2)\right)|_{t=1} = \chi_M(0)-\chi_M(1) = \mu(M), $$ where for the last equality we have used $\chi_M(1) = 0$. \end{proof} \begin{lemma}\label{lem:genericlift} Suppose that $\overline{M}$ is a general lifting of a matroid $M$. Then $\chi_{\overline{M}}(t) = (t-1) \chi_M(t)$. \end{lemma} \begin{proof} We may assume that $M$ is simple and of rank $r$. Then we have \begin{align*} \chi_{\bM}(t) &= \sum_{A \subset E \cup \star} (-1)^{|A|} t^{r+1- \rk_{\overline{M}}(A)} \\ &= \sum_{A \subset E}(-1)^{|A|} t^{r+1- \rk_{M}(A)} + \sum_{A \cup \star \subset E \cup \star}(-1)^{|A|+1} t^{r+1- \rk_{M}(A)-1} \\ &= t \chi_M(t) - \chi_M(t) = (t-1)\chi_M(t). \qedhere \end{align*} \end{proof} \subsection{Flags of flats} The order complex $\Delta(Q)$ of a poset $Q$ is the simplicial complex whose vertices are the elements of $Q$ and whose simplices are the chains of $Q$. Define $$\Delta(M) := \Delta(L(M)-\{\hat 0, \hat 1\}),$$ the order complex of the (reduced) lattice of flats in $M$. The faces $E_\bullet \in \Delta(M)$ can be identified with partial flags of flats $$ E_\bullet = \{\hat 0 = E_0 \subset E_1 \subset E_2 \subset \cdots \subset E_{s} \subset E_{s+1}= E = \hat 1\} $$ which start at $\hat 0$ and end at $\hat 1 = E$, and have $s = s(E_\bullet)$ intermediate flats. The facets, or maximal simplices, of $\Delta(M)$ can be identified with complete flags of flats $$ F_\bullet = \{\hat 0 = F_0 \subset F_1 \subset F_2 \subset \cdots \subset F_{r-1} \subset F_r = E = \hat 1\} $$ where $F_i$ is a flat of rank $i$. We denote by $\Fl(M)$ the set of complete flags in $L(M)$, or equivalently, the set of facets of $\Delta(M)$. Let $\Fl^k$ denote the set of saturated flags $F_\bullet = \{\hat 0 = F_0 \subset F_1 \subset \cdots \subset F_k \mid \rk(F_i) = i\}$ of length $k$ starting at $\hat 0$. Let $\Fl^\bullet(M) = \bigcup_k \Fl^k(M)$ denote the set of all saturated flags in $L(M)$ starting at $\hat 0$. \subsection{Oriented matroids}\label{sec:OM} Let $\M$ be an oriented matroid with underlying matroid $M$. We typically view $\M$ as a collection of \emph{signed covectors}, certain sign sequences $X: E \to \{+,0,-\}$ satisfying a collection of axioms \cite{OMbook}. For a signed covector $X$, the zero set $X_0 \subset E$ is given by $X_0:= \{e \in E \mid X(e) = 0\} \in L(M)$, and is a flat. The negative $-X$ of a signed covector $X$ is always a signed covector. Given two signed covectors $X,Y$ of $\M$, the composition $X \circ Y$ is also a signed covector of $\M$ and is defined by \begin{equation}\label{eq:compo} (X \circ Y)(e) = \begin{cases} X(e) & \mbox{if $X(e) \neq 0$} \\ Y(e) & \mbox{if $X(e) = 0$.} \end{cases} \end{equation} Oriented matroids can also be axiomatized using \emph{chirotopes}: a function $\chi: \B(M) \to \{+,-\}$ satisfying a collection of axioms. The choice of $\M$ is equivalent to the choice of a pair $\chi,-\chi$ of opposite chirotopes. We typically assume that a choice of chirotope has been fixed, omitting it from the notation. Let $\L = \L(\M)$ denote the lattice of signed covectors of $\M$. We have $X \leq Y$ in $\L(M)$ if $Y$ is obtained from $X$ by setting some entries to 0. By convention, $\L$ has a minimal element $\minL$ and a maximal element $\hat 1 = (0,0,\ldots,0)$. In the poset $\L \setminus \{\minL,\hat 1\}$, the maximal elements are signed cocircuits, and the minimal elements are \emph{topes}. We let $\T = \T(\M)$ denote the set of topes of $\M$. The oriented matroid $\M$ is \emph{acyclic} if there is a tope $P \in \T$ with $P(e) = +$ for all $e \in E$. There is a surjective map of posets $$ \phi: \L(\M)\setminus \minL \to L(M), \qquad X \mapsto X_0 = \{e \in E \mid X(e) = 0\} $$ sending a signed covector to its zero set. The rank $\rk(X)$ is defined to be $\rk(X) = \rk(\phi(X))$. For a tope $P \in \T$, we let $\L(P):=[P,\hat 1]$ denote the closed interval between $P$ and $\hat 1$. The lattice $\L(P)$ is known as the \emph{Las Vergnas face lattice}. The restriction of $\phi$ to $\L(P)$ is injective, with image equal to $L(P) \subset L(M)$. We will often identify $\L(P)$ and $L(P)$ via this map. The elements of $\L(P)$ or $L(P)$ are called the \emph{faces} of $P$. Rank one faces are called \emph{facets}. Corank one faces are called \emph{vertices}. If $\M$ is acyclic and $P$ is the positive tope, then $L(P)$ is the set of zero sets of the nonnegative signed covectors of $\M$. We let $\Fl(P) \subset \Fl(M)$ be the set of flags of flats that belong to $L(P)$. Similarly, define $\Delta(P):= \Delta(L(P) - \{\hat 0,\hat 1\})$ to be the order complex of the reduced part of $L(P)$. \subsection{Affine oriented matroids}\label{sec:AOM} An \emph{affine oriented matroid} is a pair $(\M,0)$ where $0 \in E$ is a distinguished element. We let $\T^+= \T^+(\M)$ denote the set of topes $P\in \T$ satisfying $P(0) = +$. Thus, $\T^+$ can be identified with the orbits of $\T$ under negation. \begin{defn} Given an affine oriented matroid $(\M,0)$, we define the \emph{bounded complex} by $$ \L^0 := \{\minL\} \cup \{X \in \L \setminus \minL \mid Y(0) = + \text{ for all } Y \geq X\} \subset \L. $$ The set of bounded topes $\T^0(\M)$ of $(\M,0)$ are the minimal elements of $\L^0 \setminus \minL$. \end{defn} By definition, we have $\T^0 \subset \T^+$. Now let $\tM$ be an extension of $\M$ by an element labelled $\star$. Given a sign sequence $X: E \to \{+,0,-\}$, we denote by $(\epsilon, X)$ the sign sequence $\widetilde X$ on $\widetilde E = \{\star\} \sqcup E$ defined by $\widetilde X(\star) = \epsilon$ and $\widetilde X(e) = X(e)$ for all $e\in E$. The pair $(\tM, \star)$ is an affine oriented matroid, and we let $$ \T^\star = \T^\star(\widetilde\M) := \{\mbox{topes $P \in \T(\M)$ such that $(+,P)$ is bounded in }(\tM,\star)\}. $$ If $\tM$ is a general extension of $\M$, then there is a simpler description of the set of bounded topes, not requiring one to check all faces of $P$. \begin{lemma} Suppose that $\tM$ is a general extension of $\M$. Then we have \begin{equation}\label{eq:Tstar} \T^\star = \T^\star(\widetilde\M) = \{P \in \T(M) \mid (+,P) \text{ is a tope of $\tM$ but $(-,P)$ is not}\}. \end{equation} \end{lemma} The following result appears in classical work of Greene, Las Vergnas, Zaslavsky \cite{GZ, LV}. \begin{proposition}\label{prop:numbertopes} Let $(\M,0)$ be an affine matroid and let $(\tM,\star)$ be a general extension of $\M$. We have \begin{align*} |\T^+| &= w_\Sigma(M), \qquad |\T^\star| = \mu^+(M), \qquad |\T^0| = \beta(M). \end{align*} \end{proposition} \section{Orlik-Solomon algebra and canonical forms}\label{sec:OS} Let $M$ be a matroid on ground set $E$. \subsection{Orlik-Solomon algebra} Let $\Lambda^\bullet(E)$ denote the exterior algebra over $\Z$ generated by elements $e \in E$. If $S = \{s_1,\ldots,s_k\} \subset E$ is an ordered set, then we write $e_S := e_{s_k} \wedge \cdots \wedge e_{s_1}$. (We caution the reader that this convention is the reverse of that of \cite{EL}.) Define the linear map $\partial: \Lambda^\bullet(E) \to \Lambda^{\bullet-1}(E)$ by $$ \partial(e_{1} \wedge e_2 \wedge \cdots \wedge e_k) = \sum_{i=1}^k (-1)^{k-i} e_1 \wedge \cdots \wedge \widehat{e_i} \wedge \cdots \wedge e_k. $$ We have $\partial^2 = 0$. \begin{definition} The \emph{Orlik-Solomon algebra} $\OS^\bullet(M)$ is the quotient of the exterior algebra $\Lambda^\bullet(E)$ over $\Z$ by the ideal $$ I = (\partial e_S \mid S \subseteq E \text{ is dependent}). $$ \end{definition} When $E = [n]$, we denote the generators of $\OS(M)$ by $e_1,e_2,\ldots,e_n$ for clarity. The Orlik-Solomon algebra is supported in degrees $0,1,\ldots,r$. \begin{proposition}[\cite{OS,OTbook,SV}] For each $k = 0,1,\ldots,r$, $\OS^k(M)$ is a free $\Z$-module with rank equal to the absolute value of the coefficient of $t^k$ in the characteristic polynomial $\chi_M(t)$. \end{proposition} In particular, $\OS(M)$ is a free $\Z$-module with rank $\mu^+(M)$. We have $\OS^0(M) \cong \Z$, the isomorphism given by identifying the basis element $e_\emptyset \in \OS^0(M)$ with $1 \in \Z$. Let \begin{align*} \OS(M) &:= \OS^r(M) \mbox{ denote the top degree component of the Orlik-Solomon algebra.} \end{align*} By convention, for the empty matroid $M_\emptyset$, we have $\OS(M_\emptyset) = \OS^0(M_\emptyset) \cong \Z$. For a flat $F \in L(M)$ of rank $k$, define the subspace $\OS_F(M) \subset \OS^k(M)$ by $$ \OS_F(M) = {\rm span}(e_S \mid S \in \I_k(M) \text{ and } \overline{S} = F) \cong \OS(M^F). $$ \begin{proposition}[\cite{OTbook,SV}]\label{prop:OSsum} We have a direct sum decomposition $$ \OS^\bullet(M) = \bigoplus_{F \in L(M)} \OS_F(M). $$ \end{proposition} \subsection{Broken circuits}\label{sec:nbc} A basis of $\OS^\bullet(M)$ can be constructed from the broken circuit complex, dating back to work of Wilf and Brylawski. Fix a total ordering $\prec$ on $E$. A broken circuit is a set $C' = C \setminus \min(C)$ where $C$ is a circuit, and the minimum $\min(C)$ is taken with respect to $\prec$. An independent set $S \subset E$ is called \nbc~if it does not contain any broken circuits. A basis $B \in \B(M)$ is called a \nbc-basis if it does not contain any broken circuits. For the following, see \cite{OTbook, Yuz}. \begin{theorem} The set $\{e_S \mid \mbox{S is \nbc~and } S \in \I_k(M)\}$ is a basis of $\OS^k(M)$. \end{theorem} \subsection{Reduced Orlik-Solomon algebra} We assume now that we have an affine matroid $(M,0)$. For clarity, the element of $\Lambda^\bullet(E)$ that corresponds to $0 \in E$ is denoted $e_0$. Since $\partial^2 = 0$, the map $\partial$ descends to a map $\partial: \OS^\bullet(M) \to \OS^{\bullet-1}(M)$. We let $$ \rOS^\bullet(M) := \partial(\OS^{\bullet}(M)) \subset \OS^\bullet(M) $$ denote the \emph{reduced Orlik-Solomon algebra}. The subalgebra $\rOS^\bullet(M)$ is generated by $\be:= e- e_0$ for $e \in E \setminus 0$, and it is also equal to the kernel of $\partial$ on $\OS^\bullet(M)$. The reduced Orlik-Solomon algebra is supported in degrees $0,1,\ldots,r-1$. For the next results, see \cite[Section 2.7]{Yuz01} and \cite[Proposition 3.2]{Dim}. \begin{proposition} For each $k = 0,1,\ldots,r-1$, $\rOS^k(M)$ is a free $\Z$-module with rank equal to the absolute value of the coefficient of $t^k$ in the reduced characteristic polynomial $\bchi_M(t)$. \end{proposition} Let $d:= r-1$ and \begin{align*} \rOS(M)&:= \rOS^d(M) \mbox{ denote the top degree component of the reduced Orlik-Solomon algebra.} \end{align*} \begin{proposition}\label{prop:OSrOS} Let $M$ be a matroid with rank $r \geq 1$. We have an isomorphism $\partial: \OS(M) \stackrel{\cong}{\longrightarrow} \rOS(M)$. \end{proposition} \subsection{Canonical forms} In this section we assume that an orientation $\M$ of $M$, together with a general extension $(\tM,\star)$ of $\M$, has been given. Let $B \in \B(M)$ be a basis. We now define topes that are in the bounded part of $B$; see \cite{EL}. Let $C_B$ be the signed fundamental circuit on $B \cup \star$ of $\tM$ with $C_B(\star) = -$. The circuit necessarily has support $B\cup \star$ by genericity of the extension $\tM$. Note that $(+, C_B|_B)$, i.e.\ the sign sequence on $B\cup \star$ with $+$ at $\star$ and $C_B(i)$ at $i\in B$, is a tope in the restriction $\widetilde\M|_{B\cup \star}$ but $(-, C_B|_B)$ is not. We say that a tope $P \in \T$ is in the \emph{bounded part} of $B$ if we have $P|_B = C_B|_B$. Write $$ \T^{B} = \{P \in \T \mid \mbox{$P$ is in the bounded part of $B$}\}. $$ \begin{lemma} For any basis $B$, we have $\T^B \subset \T^\star$. \end{lemma} \begin{proof} If a tope $(+,P)$ satisfies $P|_B = C_B|_B$ for some basis $B$ of $\M$, then $P$ is automatically bounded in $\tM$ since $(-,P)$ cannot be orthogonal to $C_B$. \end{proof} For an \emph{unordered} basis $B \in M$, we say that an ordering $(b_1,b_2,\ldots,b_r)$ of $B$ is positive if $\chi(b_r,b_{r-1},\ldots,b_1) = +$, where $\chi$ is the chirotope of $\M$. We define an element $$ e_B := \chi(b_r,b_{r-1},\ldots,b_1) e_{b_r} \wedge \cdots \wedge e_{b_1} \in A(M)$$ where $(b_1,b_2,\ldots,b_r)$ is any ordering of $B$. In the following result we will use the residue maps between Orlik-Solomon algebras, reviewed in \cref{sec:residue}; see \cite{EL} for further details. \begin{theorem}[{\cite[Theorem 2.10]{EL}}] \label{thm:EL} For each $P \in \T(\M)$, there exists a distinguished element $\Omega_P \in \OS(M)$ satisfying the following properties: \begin{enumerate} \item The \emph{canonical form} $\Omega_P$ is invariant under simplification of matroids, satisfies $\Omega_{-P} = (-1)^r \Omega_P$, and is uniquely characterized by the following recursion. If $\M$ is the rank $0$ empty matroid with chirotope $\chi$, then $\Omega_P = \chi(\emptyset) \in \OS^0(M)$. If $r \geq 1$, then for any atom $\atom \in \At(M)$, we have $$ \Res_\atom \Omega_P = \begin{cases} P(e)\, \Omega_{P/\atom} \in \OS(M/\atom) &\mbox{if $\atom \in L(P)$,} \\ 0 & \mbox{otherwise.} \end{cases} $$ Here, $P/\atom = P_\atom \in \T(\M/\atom)$ is the tope given by $P/\atom = P|_{E \setminus \atom}$, and the chirotope of $\M/\atom$ is fixed by choosing $e \in \atom$ and setting $\chi_{\M/\atom}(e_1,\ldots,e_{r-1}) := \chi_{\M}( e_1,\ldots,e_{r-1},e)$. \item For a general extension $(\tM, \star)$ of $\tM$, the elements $$ \{\Omega_P \mid P \in \T^\star\} $$ form a basis of $\OS(M)$, and for any basis $B \in \B(M)$, we have \begin{equation}\label{eq:cone} (-1)^{|C^{-1}_B(-)|-1}e_B = \sum_{P \in \T^B} \Omega_P. \end{equation} \end{enumerate} \end{theorem} By \cref{prop:OSrOS}, the set $\{\bOmega_P := \partial \Omega_P \mid P \in \T^\star\}$ is a basis of $\rOS(M)$. \begin{example} Let $\M$ be the oriented matroid of rank 2 associated to the arrangement of three points on $\P^1$, as in \cref{ex:3pt}. Then $\OS^\bullet(M)$ is generated by $e_1,e_2,e_3$ with the relation $e_2 e_1 - e_3 e_1 + e_3e_2 = 0$. The canonical forms of $P,Q,R$ are $\Omega_P = e_2 e_1, \Omega_Q = e_3 e_2, \Omega_R = e_1 e_3$. Any two of these give a basis of $\OS(M)$. The reduced canonical forms are $\partial \Omega_P = e_2 - e_1, \partial \Omega_Q = e_3 - e_2, \partial \Omega_R = e_1 - e_3$. Any two of these give a basis of $\rOS(M)$. \end{example} \begin{remark} In the case that $M$ arises from a real hyperplane arrangement $\bA$, the canonical forms of \cref{thm:EL}, are the usual canonical forms of polytopes \cite{ABL,LamPosGeom}. These forms have also appeared in the work of Yoshinaga \cite{Yos} where they are referred to as the ``chamber basis". \end{remark} \section{DeRham cohomology intersection form} \subsection{Residue maps}\label{sec:residue} \begin{proposition}[{\cite[Proposition 2.2]{EL}}]\label{prop:OSexact} For every atom $\atom \in \At(M)$, we have a short exact sequence \[ 0\longrightarrow \OS^\bullet(M\backslash \atom) \overset{\iota_{\atom}}\longrightarrow \OS^\bullet(M) \overset{\Res_{\atom}}\longrightarrow \OS^{\bullet-1}(M/\atom) \to 0 \] where $\iota_{\atom}(e_I) = e_I$ for $I \subseteq E \setminus c$, and $\Res_{\atom}(e_I) = e_{I\setminus e}$ if $I = (e \in \atom,i_1, \dots, i_{k-1})$ and $\Res_{\atom}(e_I) = 0$ if $I \cap \atom = \emptyset$. These maps restrict to give the short exact sequence \[ 0\longrightarrow \rOS^\bullet(M\backslash \atom) \longrightarrow \rOS^\bullet(M) \longrightarrow \rOS^{\bullet-1}(M/\atom) \to 0. \] \end{proposition} Now let $F_\bullet = (\hat 0 = F_0 \subset F_1 \subset \cdots \subset F_k) \in \Fl^k(M)$ be a saturated flag of flats. Then $F_1$ is an atom in $L(M)$, and for each $i = 1,2,\ldots,k-1$, we have that the contraction $F_{i+1}/F_i$ of $F_{i+1}$ is an atom in the lattice of flats $L(M/F_i)$ of the contraction $M/F_i$. Thus the following definition makes sense. \begin{definition} Let $F_\bullet = (\hat 0 = F_0 \subset F_1 \subset \cdots \subset F_k) \in \Fl^k(M)$ be a saturated flag of length $k$. The \emph{residue map} $\Res_{F_\bullet}: \OS^\bullet(M) \to \OS^{\bullet - k}(M/F_k)$ of the flag $F_\bullet$ is the $k$-fold composition $$ \Res_{F_\bullet} = \Res_{F_k/F_{k-1}} \circ \cdots \circ \Res_{F_2/F_1} \circ \Res_{F_1}: \OS^\bullet(M) \to \OS^{\bullet-k}(M/F_k). $$ For an element $x \in \OS^k(M)$, we view the residue $\Res_{F_\bullet}(x)$ of $x$ at $F_\bullet$ as an integer via the identification $\OS^0(M/F_k) \cong \Z$. \end{definition} By \cref{prop:OSexact}, $\Res_{F_\bullet}$ restricts to a residue map $\Res_{F_\bullet}: \rOS^\bullet(M) \to \rOS^{\bullet - k}(M/F_k)$. \begin{example} Let $M = U_{2,3}$ be the uniform matroid of rank 2 on $\{e_1,e_2,e_3\}$. Let $F_\bullet = (\hat 0 \subset \{e_1\} \subset \hat 1)$. Then $$ \Res_{F_\bullet} e_2 \wedge e_1 = \Res_{F_\bullet} e_3 \wedge e_1 = 1, \qquad \text{and} \qquad \Res_{F_\bullet} e_3 \wedge e_2 = 0. $$ This is consistent with the relation $e_2e_1 - e_3e_1 + e_3 e_2 = 0$ in $\OS(M) = \OS^2(M)$. \end{example} For $S \subset E$, let $L(S) \subseteq L(M)$ be the sublattice of $L(M)$ generated by the atoms in $S$. Equivalently, $L(S) = L(M \backslash S)$ where $M \backslash S$ is the matroid obtained by deleting all elements not in $S$. Now let $S \in \I_k(M)$ be an independent set of size $k$ and let $F_\bullet \in \Fl^k$ be a saturated flag of length $k$. We say that $F_\bullet$ is \emph{generated} by $S$ if $F_\bullet$ is a maximal chain in $L(S)$. In other words, each $F_i$ is spanned by a subset of $S$. Given a pair $(S,F_\bullet)$ where $F_\bullet$ is generated by an \emph{ordered} independent set $S=(s_1,s_2,\ldots,s_k)$, we define a permutation $\sigma = \sigma(S,F_\bullet) \in S_k$ by \begin{equation}\label{eq:sigma} F_i = {\rm span}(s_{\sigma(1)},s_{\sigma(2)},\ldots,s_{\sigma(i)}), \qquad \text{for } i = 1, 2, \ldots,k. \end{equation} \begin{definition}\label{def:rSF} Let $F_\bullet \in \Fl^k(M)$ be a saturated flag and $S = (s_1,\ldots,s_k)$ be an ordered independent set. Define the \emph{residue $r(S, F_\bullet) \in \{0,1,-1\}$ of $S$ at $F_\bullet$} as follows. If $F_\bullet$ is not generated by $S$ then we set $r(S,F_\bullet)= 0$. If $F_\bullet$ is generated by $S$, then we set $r(S,F_\bullet) = (-1)^{\sigma(S,F_\bullet)}$ to be the sign of the permutation $\sigma(S,F_\bullet)$. \end{definition} The following comparison follows immediately from the definitions. \begin{lemma}\label{lem:rSF} Let $F_\bullet \in \Fl^k(M)$ be a saturated flag and $S = (s_1,\ldots,s_k)$ be an ordered independent set. Then $$ \Res_{F_\bullet}(e_S) = r(S, F_\bullet). $$ \end{lemma} \subsection{Definition of intersection form} Let $R := \Z[\a]= \Z[a_e: e \in E]$ be the polynomial ring in variables $a_e$ indexed by $e$ and let $Q = \Frac(A) = \Q(a_e: e \in E)$ be the fraction field of rational functions. For a subset $S \subset E$, define $$ a_S:= \sum_{e \in S} a_e. $$ For a flag $F_\bullet \in \Fl^k(M)$, define $$ \frac{1}{a_{F_\bullet}} := \prod_{i=1}^{k-1} \frac{1}{a_{F_i}} \in Q, \qquad \frac{1}{a'_{F_\bullet}} := \prod_{i=1}^{k} \frac{1}{a_{F_i}} \in Q. $$ \begin{definition}\label{def:dR} The $Q$-valued \emph{deRham cohomology intersection form} on $\OS^k(M)$ is given by $$ \dRip{x, y}:= \sum_{F_\bullet \in \Fl^k(M)} \Res_{F_\bullet}(x) \frac{1}{a_{F_\bullet}} \Res_{F_\bullet}(y). $$ \end{definition} We shall also use the slight modification $$ \dRipp{x,y} := \sum_{F_\bullet \in \Fl^k(M)} \Res_{F_\bullet}(x) \frac{1}{a'_{F_\bullet}} \Res_{F_\bullet}(y). $$ It is clear from the definition that $\dRip{\cdot,\cdot}$ is a symmetric bilinear form. We view $\dRip{\cdot,\cdot}$ both as a $Q$-valued form on $\OS^k(M)$, and as a $Q$-valued form on $\OS^k(M)_Q := \OS^k(M) \otimes_\Z Q$. \begin{proposition}\label{prop:dRind} Let $S, S'$ be two ordered independent sets of size $k$. Then $$ \dRip{e_S,e_{S'}}= \sum_{F_\bullet \in \Fl^k(M)} r(S, F_\bullet) \frac{1}{a_{F_\bullet}} r(S', F_\bullet) . $$ \end{proposition} \begin{proof} Follows immediately from \cref{lem:rSF}. \end{proof} The formula in \cref{prop:dRind} will be improved in \cref{thm:localBF}. \begin{example}\label{ex:boolean} Let $M$ be the boolean matroid of rank $d$ on $E = \{e_1,\ldots,e_d\}$. The flats of $M$ consists of all the subsets of $E$. The complete flags of flats $F_\bullet$ are in bijection with saturated chains of subsets $F_\bullet(w) = \{ \emptyset \subset \{e_{w_1}\} \subset \{e_{w_1},e_{w_2}\} \subset \cdots \}$, or equivalently with permutations $w = w_1w_2 \cdots w_d$ of $\{1,2,\ldots, d\}$. The only basis is $E$ and $\OS(M)$ is one-dimensional, spanned by $e_E$. We have $$ \dRip{e_E, e_E} = \sum_{w \in S_d} \frac{1}{a_{F_\bullet(w)}} = \sum_{w \in S_d} \prod_{i=1}^{d-1} \frac{1}{a_{w_1} + \cdots + a_{w_d}} = \frac{a_E}{a_1 \cdots a_d}. $$ \end{example} \begin{proposition}\label{prop:dRdirectsum} The bilinear form $\dRip{\cdot,\cdot}$ on $\OS^k(M)$ is compatible with the direct sum decomposition $\OS^k(M) = \bigoplus_{F \in L^k(M)} \OS_F(M)$ of \cref{prop:OSsum}. That is, for distinct $F,F' \in L^k(M)$ and $x \in \OS_F(M)$, $x' \in \OS_{F'}(M)$, we have $\dRip{x,x'} =0$. \end{proposition} \begin{proof} We may assume that $x = e_S$ and $x' = e_{S'}$ where $\overline{S} = F$ and $\overline{S'} = F'$. Let $F_\bullet \in \Fl^k(M)$. We have $r(S,F_\bullet) = 0$ unless $F_k = F$, and $r(S',F_\bullet) = 0$ unless $F_k = F'$. Thus $r(S,F_\bullet) r(S', F_\bullet) = 0$ for all $F_\bullet \in \Fl^k(M)$, and hence $\dRip{x,x'}=0$. \end{proof} The following result states that $\dRip{\cdot,\cdot}$ is compatible with restriction to flats. \begin{proposition}\label{prop:restrictF} Let $F \in L(M)$. The restriction of $\dRip{\cdot,\cdot}$ to $\OS_F(M)$ is equal to $\dRip{\cdot,\cdot}$ for $\OS(M^F)$. \end{proposition} \begin{proof} The interval $[\hat 0, F]$ in $L(M)$ is isomorphic to $L(M^F)$. \end{proof} \subsection{Bilinear form on reduced Orlik-Solomon algebra} \begin{proposition}\label{prop:dRpartial} Let $x,y \in \OS_F(M)$. Then $$ \dRip{x,y} = \dRipp{\partial x, \partial y}. $$ \end{proposition} \begin{proof} Let $x = e_S$ and $y = e_T$ for ordered independent sets $S = (s_1,\ldots,s_k),T = (t_1,\ldots,t_k)$ such that $\bar S = \bar T = F$ for some flat $F$. Let $F_\bullet \in \Fl^k(M)$. Since $x,y \in \OS_F(M)$, we have $\Res_{F_\bullet}(x) = \Res_{F_\bullet}(y) = 0$ unless $F_k = F$. We calculate \begin{align*} \dRipp{\partial x, \partial y} &= \sum_{F_\bullet \in \Fl^{k-1}(M)} \Res_{F_\bullet}(\partial x) \frac{1}{a'_{F_\bullet}} \Res_{F_\bullet}(\partial y) \\ &= \sum_{i,j=1}^k (-1)^{i-1}(-1)^{j-1} \sum_{F_\bullet \in \Fl^{k-1}(M) \mid F_{k-1} = \overline{S \setminus i} = \overline{T \setminus j}} r(S \setminus s_i, F_\bullet) \frac{1}{a'_{F_\bullet}} r(T \setminus t_j, F_\bullet) \\ &= \sum_{G_\bullet \in \Fl^k(M) \mid G_k = F} r(S, G_\bullet) \frac{1}{a_{G_\bullet}} r(T, G_\bullet) = \dRip{x,y}. \qedhere \end{align*} \end{proof} Recall the reduced Orlik-Solomon algebra $\rOS^\bullet(M) \subset \OS^\bullet(M)$ from \cref{sec:OS}. \begin{corollary}\label{cor:same} The bilinear form $\dRipp{\cdot,\cdot}$ on $\rOS(M)$ agrees with the bilinear form $\dRip{\cdot,\cdot}$ on $\OS(M)$ under the isomorphism $\partial: \OS(M) \to \rOS(M)$ of \cref{prop:OSrOS}. \end{corollary} \subsection{Intersection form on topes}\label{sec:pFl} For a tope $P \in \T(\M)$ and a flag $F_\bullet \in \Fl(M)$, define $$ r(P, F_\bullet) := \Res_{F_\bullet}(\Omega_P). $$ \begin{lemma} For any $P \in \T(\M)$ and $F_\bullet \in \Fl(M)$, we have $r(P,F_\bullet) \in \{-1,0,1\}$. \end{lemma} \begin{proof} By \cref{thm:EL}, the residue $\Res_{F_1} \Omega_P$ is either 0, or it equals to another canonical form $\Omega_{P/F_1}$. The result then follows from induction on the rank $r$, with the case $r = 1$ being trivial. \end{proof} Recall that $$ \pFl(M) := \{E_\bullet = (\hat 0 \subset E_1 \subset \cdots \subset E_s \subset E = \hat 1)\} $$ denotes the set of partial flags of flats in $L(M)$. We always assume that a partial flag starts at $\hat 0$ and ends at $\hat 1$. We let $s = s(E_\bullet)$ denote the number of flats in $E_\bullet$ that belong to the proper part $L(M) \setminus \{\hat0,\hat1\}$. Let $L(P)$ denote the Las Vergnas face lattice of a tope $P \in \T(\M)$ (see \cref{sec:OM}), viewed as a subposet of $L(M)$. Note that $L(P) = L(-P)$. A \emph{wonderful face} of $P$ is a partial flag $G_\bullet = \{\hat 0 \subset G_1 \subset G_2 \cdots \subset G_s \subset \hat 1\}$ where $G_i \in L(P)$. We let $\pFl(P) = \Delta(L(P) - \{\hat 0,\hat1\})$ denote the set of wonderful faces of $P$, viewed as a subcomplex of $\pFl(M)$. The closure $\bG_\bullet \subset \pFl(P)$ of a wonderful face $G_\bullet$ is the set of all partial flags $G'_\bullet$ of wonderful faces that refine $G_\bullet$. A \emph{wonderful vertex} of $P$ is a complete flag $F_\bullet \in \Fl(P)$. Equivalently, $F_\bullet$ is a facet of $\pFl(P)$. In particular, a wonderful vertex $F_\bullet$ is contained in the closure of a wonderful face $G_\bullet$ if every flat in $G_\bullet$ also appears in $F_\bullet$. We endow $\pFl(P)$ with the poset structure $G'_\bullet \leq G_\bullet$ if and only $G'_\bullet \in \bG_\bullet$. Write $\emptyflag \in \pFl(P)$ for the trivial flag $\{\hat 0 < \hat 1\}$. The relation between $\pFl(P)$ and the wonderful compactification is explained in \cref{prop:wonderfulface}. \begin{lemma}\label{lem:fliptope} Let $P$ be a tope and $F \in L(P)$. Then there is a unique tope $P_{\flip F}$ on the antipodal side of $F$. More precisely, we have $$ P_{\flip F}(e) = \begin{cases} - P(e) & \mbox{if $e \in F$,} \\ P(e) & \mbox{if $e \notin F$.} \end{cases} $$ \end{lemma} \begin{proof} Viewing $F$ as a signed covector, the tope $P_{\flip F}$ is given by the composition $F \circ (-P)$ (see \eqref{eq:compo}). \end{proof} \begin{proposition}\label{prop:flipflag} Let $G_\bullet \in \pFl(P)$. Then there exists a tope $P_{\flip G_\bullet} \in \T$ satisfying \begin{equation}\label{eq:flip} P_{\flip G_\bullet}(e) = P(e) (-1)^{\#\{1 \leq i \leq s \mid e \in G_i\}} \end{equation} for all $e \in E$. We have $(P_{\flip G_\bullet})_{\flip G_\bullet} = P$. \end{proposition} \begin{proof} Apply \cref{lem:fliptope} to $P$ and the flat $G_1 \in L(P)$ to obtain $P_{\flip G_1}$. We have $G_2 \in L(P_{\flip G_1})$ since $G_1 \subset G_2$, so we may apply \cref{lem:fliptope} again to $P_{\flip G_1}$ and the flat $G_2 \in L(P_{\flip G_1})$. Continuing in this manner, we obtain the tope $P_{\flip G_\bullet}$. \end{proof} For $P,Q \in \T$, define $$ G(P,Q):= \{G_\bullet \in \pFl(P) \mid Q = P_{\flip G_\bullet}\}, \qquad \text{and} \qquad G^{\pm}(P,Q):= G(P,Q) \cup G(P,-Q) . $$ \begin{lemma}\label{lem:closurePQ} Suppose that $G_\bullet \in G(P,Q)$. Then the closure $\bG_\bullet \subset \pFl(M)$ is the same regardless of whether it is taken in $\pFl(P)$ or $\pFl(Q)$. \end{lemma} \begin{proof} Let $E_\bullet \in \bG_\bullet$, where the closure is taken in $\pFl(P)$. For each $E \in E_\bullet \setminus G_\bullet$, let $X \in \L(P)$ be a signed covector lifting $E$. Similarly to \cref{prop:flipflag}, the formula $X_{\flip G_\bullet}(e) = X(e) (-1)^{\#\{1 \leq i \leq s \mid e \in G_i\}}$ determines a signed covector $X_{\flip G_\bullet}$, and $X_{\flip G_\bullet} \in \L(Q)$. It follows that $E \in L(Q)$, and thus $E_\bullet \in \pFl(Q)$. \end{proof} \begin{proposition}\label{prop:noover} \ \begin{enumerate} \item We have $G(P,P) = \{\emptyflag\}$ consisting only of the trivial flag, and $G(P,-P) = \emptyset$. \item We have $G(P,Q) = G(Q,P)$ and $G(-P,-Q) = G(P,Q)$. We have $G^{\pm}(P,Q) = G^{\pm}(Q,P)$. \item For distinct $G_\bullet, G'_\bullet \in G^{\pm}(P,Q)$, we have $\bG_\bullet \cap \overline{G'_\bullet} = \emptyset$. \item We have $\bigsqcup_{G_\bullet \in G^{\pm}(P,Q)} \{F_\bullet \in (\bG_\bullet \cap \Fl(M))\} = \Fl(P) \cap \Fl(Q)$. \end{enumerate} \end{proposition} \begin{proof} (1) is clear from the definitions. For (2), the equality $G(P,Q)= G(Q,P)$ follows from the last statement of \cref{prop:flipflag} and the equality $G(-P,-Q) = G(P,Q)$ is clear from the definitions. The last equality $G^{\pm}(P,Q) = G^{\pm}(Q,P)$ also follows. For (3), suppose that $F_\bullet \in \bG_\bullet \cap \bG'_\bullet$ for some wonderful vertex $F_\bullet$ and $G_\bullet \neq G'_\bullet$. Then $P_{\flip G_\bullet}$ and $P_{\flip G'_\bullet}$ are both obtained from $P$ by flipping the signs of some subset of $\{F_1,\ldots, F_{r-1}\}$. Suppose that $P_{\flip G_\bullet} = P_{\flip G'_\bullet}$. Then \eqref{eq:flip} shows that $\{e \in E \mid P(e) = P_{\flip G_\bullet}(e)\}$ uniquely determines $G_\bullet$ (once $F_\bullet$ has been fixed), forcing the contradiction $G_\bullet = G'_\bullet$. However, it is not possible to have $P_{\flip G_\bullet} = Q$ and $P_{\flip G'_\bullet} = -Q$ because $P_{\flip G_\bullet}(e) = P_{\flip G'_\bullet}(e)$ for any $e \in E \setminus F_{r-1}$. It follows that if $G_\bullet \neq G'_\bullet$ then $P_{\flip G_\bullet} \neq P_{\flip G'_\bullet}$. For (4), the union is disjoint by (3). The inclusion $\subseteq$ is clear from \cref{lem:closurePQ}. To prove the inclusion $\supseteq$, we proceed by induction. Assume that $r > 1$, and let $F_\bullet \in \Fl(P) \cap \Fl(Q)$. Then by induction, $F_\bullet/F_1 \in \bG'_\bullet \cap \Fl(M_{F_1})$ for some $G'_\bullet \in G^{\pm}(P_{F_1},Q_{F_1})$, where $P_{F_1} = P|_{E \setminus F_1}$ and $Q_{F_1} = Q|_{E \setminus F_1}$. After possibly replacing $Q$ by $-Q$, we may suppose that $(P_{F_1})_{\flip G'_\bullet} = Q_{F_1}$. If $P|_{F_1} = Q|_{F_1}$, then $P_{\flip G_\bullet} = Q$ for $G_\bullet$ the natural lift of $G'_\bullet$ (adding no additional flats so that $s(G_\bullet) = s(G'_\bullet)$). If $P|_{F_1} = -Q|_{F_1}$, then instead we lift $G'_\bullet$ to a partial flag in $\pFl(M)$ and then add $F_1$ to it to obtain $G_\bullet$ (so that $s(G_\bullet) = s(G'_\bullet)+1$). In both cases, we have shown that $F_\bullet \in \bG_\bullet$ for some $G_\bullet \in G^{\pm}(P,Q)$. \end{proof} In ``big" examples, we typically have $|G^{\pm}(P,Q)| \in \{0,1\}.$ \begin{example}\label{ex:3pttope} We give an example where $|G^{\pm}(P,Q)| > 1$. Consider the two-dimensional arrangement of two lines $\ell_1,\ell_2$ in $\R^2$, and let $\ell_0$ denote the line at infinity. $$ \begin{tikzpicture} \draw (0:1.5)--(180:1.5); \draw (90:1.5)--(270:1.5); \draw (0,0) circle (1.5); \node[color=blue] at (45:1.65) {$0$}; \node[color=blue] at (7:1.2) {$1$}; \node[color=blue] at (95:1.2) {$2$}; \node[color=red] at (45:0.75) {\scriptsize $+++$}; \node[color=red] at (135:0.75) {\scriptsize$++-$}; \node[color=red] at (225:0.75) {\scriptsize$+--$}; \node[color=red] at (-45:0.75) {\scriptsize$+-+$}; \end{tikzpicture} $$ The corresponding matroid $M$ is the boolean matroid of rank three on three elements $E = \{0,1,2\}$. The set $\T^+$ consists of four topes: $(+,+,+),(+,-,+),(+,-,-),(+,+,-)$. Then \begin{align*} G^{\pm}((+,+,+),(+,+,+)) &= \{(\hat 0 \subset \hat 1)\}, \\ G^{\pm}((+,+,+),(+,-,+)) &= \{(\hat 0 \subset \{1\} \subset \hat 1), (\hat 0 \subset \{2\} \subset \{1,2\} \subset \hat 1), (\hat 0 \subset \{0\} \subset \{0,1\} \subset \hat 1), (\hat 0 \subset \{0,2\} \subset \hat 1)\}, \\ G^{\pm}((+,+,+),(+,-,-)) &= \{(\hat 0 \subset \{1,2\} \subset \hat 1), (\hat 0 \subset \{0\} \subset \hat 1), (\hat 0 \subset \{1\} \subset \{0,1\} \subset \hat 1), (\hat 0 \subset \{2\}\subset \{0,2\} \subset \hat 1)\}, \\ G^{\pm}((+,+,+),(+,+,-)) &= \{(\hat 0 \subset \{2\} \subset \hat 1), (\hat 0 \subset \{1\} \subset \{1,2\} \subset \hat 1), (\hat 0 \subset \{0\} \subset \{0,2\} \subset \hat 1), (\hat 0 \subset \{0,1\} \subset \hat 1)\}. \end{align*} \end{example} \begin{lemma}\label{lem:FlP} Let $F_\bullet \in \Fl(M)$ and $P \in \T$. We have $r(P,F_\bullet) \neq 0$ if and only if $F_\bullet \in \Fl(P)$. \end{lemma} \begin{proof} By \cref{thm:EL}, we have $\Res_{F_1}(\Omega_P) \neq 0$ if and only if $F_1 \in L(P)$ is a facet of $P$. In this case, $\Res_{F_1}(\Omega_P) = \Omega_{P/F_1}$, and $L(P/F_1)$ is isomorphic to the interval $[F_1, \hat 1] \subset L(P)$. The result then follows by induction. \end{proof} \begin{lemma}\label{lem:Gsign} Let $P, Q \in \T$ and $G_\bullet \in G(P,Q)$. Suppose $F_\bullet \in \bG_\bullet \cap \Fl(M)$. Then $$ r(P,F_\bullet) r(Q,F_\bullet) = (-1)^{\sum_{i=1}^s \rk(G_i)}. $$ \end{lemma} \begin{proof} We proceed by induction on $s$. If $s = 0$ then $G_\bullet = \emptyflag$ and $P = Q$ and the claim is clear. Suppose $s \geq 1$, and let $p = \rk(G_1)$. Pick $f_1,f_2,\ldots,f_p$ so that $F_i = \sp(f_1,\ldots,f_i)$ and fix the chirotope of $\M_{G_1}$ by $$ \chi_{\M_{G_1}}(e_1,\ldots,e_{r-p}) := \chi_{\M}(e_1,\ldots,e_{r-p},f_p,f_{p-1},\ldots,f_1). $$ Then by \cref{thm:EL}, we have $$ \Res_{F_p = G_1} \circ \cdots \circ \Res_{F_1} \Omega_P = \prod_{i=1}^p P(f_i) \Omega_{P_{G_1}}, $$ where $P_{G_1} = P|_{E \setminus G_1} \in \T(\M_{G_1})$, and similarly for $Q$. It follows from the definitions that $G_\bullet/G_1 = (\hat 0 = G_1/G_1,G_2/G_1,\ldots,) \in G(P_{G_1},Q_{G_1})$. By the inductive hypothesis, we have $$ r(P_{G_1},F_\bullet/G_1) r(Q_{G_1},F_\bullet/G_1) = (-1)^{\sum_{i=2}^s \rk(G_i) - \rk(G_1)}. $$ By \cref{prop:flipflag}, we have $\prod_{i=1}^p P(f_i) Q(f_i) = (-1)^{sp}$. Thus \begin{align*} r(P,F_\bullet) r(Q,F_\bullet) &= (-1)^{\sum_{i=2}^s \rk(G_i) - p} \prod_{i=1}^p P(f_i) Q(f_i) = (-1)^{\sum_{i=2}^s (\rk(G_i) - p) + sp } = (-1)^{\sum_{i=1}^s \rk(G_i)}. \qedhere \end{align*} \end{proof} \begin{theorem}\label{thm:dRtope} Let $P,Q \in \T$. Then $$ \dRip{\Omega_P,\Omega_Q} = \sum_{G_\bullet \in G^{\pm}(P,Q)} (\pm)^r (-1)^{\sum_{i=1}^s \rk(G_i)} \sum_{F_\bullet \in \bG_\bullet \cap \Fl(M)} \frac{1}{a_{F_\bullet}}, $$ where the sign $(\pm)^r$ is equal to $1$ or $(-1)^r$ depending on whether $G_\bullet$ belongs to $G(P,Q)$ or $G(P,-Q)$. In particular, $$ \dRip{\Omega_P,\Omega_P} = \sum_{F_\bullet \in \Fl(P)} \frac{1}{a_{F_\bullet}}. $$ \end{theorem} \begin{proof} By \cref{lem:FlP} and \cref{prop:noover}(3), $$ \dRip{\Omega_P,\Omega_Q} = \sum_{F_\bullet \in \Fl(P) \cap \Fl(Q)} \pm \frac{1}{a_{F_\bullet}} = \sum_{G_\bullet \in G^{\pm}(P,Q)} \sum_{F_\bullet \in \bG_\bullet \cap \Fl(M)} \pm \frac{1}{a_{F_\bullet}}. $$ Since $\Omega_{-Q} = (-1)^r \Omega_Q$, by \cref{lem:Gsign}, the sign $\pm$ is equal to $(\pm)^r (-1)^{\sum_{i=1}^s \rk(G_i)}$. The last statement follows from \cref{prop:noover}(1). \end{proof} \begin{example} Continue \cref{ex:3pttope}. We have \begin{align*} \dRip{\Omega_{(+,+,+)},\Omega_{(+,+,+)}} &= \frac{1}{a_1 a_2} + \frac{1}{a_0 a_1} + \frac{1}{a_0 a_2}, \\ \dRip{\Omega_{(+,+,+)},\Omega_{(+,-,-)}} &= \frac{1}{a_1 a_2} + \frac{1}{a_0 a_1} + \frac{1}{a_0 a_2}, \\ \dRip{\Omega_{(+,+,+)},\Omega_{(+,+,-)}} &= -\frac{1}{a_1 a_2} - \frac{1}{a_0 a_1} - \frac{1}{a_0 a_2}. \end{align*} \end{example} \section{DeRham homology intersection form} By \cref{prop:dRdirectsum} and \cref{prop:restrictF}, to understand the bilinear form $\dRip{\cdot,\cdot}$ it suffices to consider the form on the top homogeneous component $\OS(M) = \OS^r(M)$ of the Orlik-Solomon algebra. We henceforth focus on this case. In this section, we investigate the dual $\DdRip{\cdot,\cdot}$ of the symmetric bilinear form $\dRip{\cdot,\cdot}$. We discover remarkable combinatorics when we compute $\DdRip{\cdot,\cdot}$ on the basis dual to the canonical forms in \cref{thm:EL}. \subsection{Non-degeneracy} \begin{proposition}\label{prop:nondeg} The symmetric bilinear form $\dRip{\cdot,\cdot}$ is non-degenerate on $\OS(M)_Q := \OS(M) \otimes_{\Z} Q$. \end{proposition} \cref{prop:nondeg} can also be deduced from the results of \cite{SV}. In \cref{thm:dRmain}, we will sharpen \cref{prop:nondeg} by explicitly inverting the bilinear form matrix. We prove \cref{prop:nondeg} using residue maps, which will be useful in the sequel. Assume that $M$ is a simple matroid. Let $\atom \in \At(M)$ be an atom, which we view as both an element of $L(M)$ and as an element of $\OS^1(M)$. Let $R_{\atom} := R/(a_\atom)$. Let $M' = M \backslash \atom = M^{E \backslash \atom}$ and $M'' = M/\atom = M_\atom$. Let $\theta_{\atom}: R \to R_{\atom}$ be the quotient map that sends $a_{\atom}$ to $0$. \begin{lemma}\label{lem:deleteform} For $x,y \in A(M')$, we have $$ \dRip{x,y}_{M'} = \theta_{\atom} \dRip{\iota_{\atom} x, \iota_{\atom} y}_{M}. $$ \end{lemma} \begin{proof} It suffices to show that for two bases $B,B' \in \B(M')$, we have $$ \dRip{e_B, e_{B'}}_{M'} = \theta_{\atom} \dRip{\iota_{\atom} e_B, \iota_{\atom} e_{B'}}_{M}. $$ For any flag $F_\bullet \in \Fl(M')$, by \cref{def:rSF} we have that the residues $r(B,F_\bullet)$ and $r(B',F_\bullet)$ are the same regardless of whether they are calculated inside $M$ or $M'$. Let $F'_{\bullet}$ be a flag in $L(M')$ generated by $B$. Since we have an injection $\iota: L(M') \hookrightarrow L(M)$, the flag $F'_\bullet$ can also be identified with a flag $F_\bullet = \iota_{\atom}(F'_\bullet)$ in $L(M)$ generated by $B$. We have $$ \frac{1}{a_{F'_\bullet}} =\theta_{\atom} \frac{1}{a_{F_\bullet}} $$ and the result follows from \cref{prop:dRind}. \end{proof} \begin{lemma}\label{lem:contractform} For $x,y \in \OS(M'')$, we have $$ \res_{\atom=0} \dRip{x \wedge \atom, y \wedge \atom}_{M} = \dRip{x,y}_{M''}, $$ where $\res_{\atom=0}: Q \to Q_\atom = \Frac(R_\atom)$ is the map that sends $f(x)$ to $\theta_{\atom}(x_{\atom} f(x))$, if this is well-defined. \end{lemma} \begin{proof} It suffices to show that for two bases $B,B' \in \B(M)$, we have $$ \res_{\atom=0} \dRip{e_B\wedge \atom, e_{B'}\wedge \atom}_{M} = \dRip{e_B,e_{B'}}_{M''}. $$ For a flag $F_\bullet \in \Fl(M)$ with $F_1 = \atom$, we let $(F/\atom)_\bullet \in \Fl(M'')$ be the flag defined by $(F/\atom)_i = F_{i+1} \backslash \atom$. The pairing $ \dRip{e_B\wedge \atom, e_{B'}\wedge \atom}_{M}$ is a sum of terms $\pm \frac{1}{a_{F_\bullet}}$ for various flags $F_\bullet$. We have $$ \res_{\atom = 0} \frac{1}{a_{F_\bullet}} = \begin{cases} \frac{1}{a_{(F/\atom)_\bullet}} & \mbox{if $F_1 = \atom$,} \\ 0 & \mbox{otherwise.} \end{cases} $$ Thus $\Res_{\atom = 0} \dRip{e_B \wedge \atom, e_{B'}\wedge \atom}_{M}$ can be expressed as a sum over flags in $\Fl(M'')$, and comparing with \cref{prop:dRind} we see that it equals to $\dRip{e_B,e_{B'}}_{M''}$. \end{proof} \begin{lemma}\label{lem:Resa0} For any $x \in \iota_{\atom}(\OS(M'))$ and $y \in \OS^{r-1}(M)$, we have $\res_{\atom=0} \dRip{x,y \wedge \atom}= 0$. \end{lemma} \begin{proof} The operation $\res_{\atom=0}$ will annihilate $\dRip{x, y \wedge \atom}$ unless there are terms that involve $1/a_\atom$. These terms appear in the summands of \cref{prop:dRind} for flags $F_\bullet$ with $F_1 = \atom$. But if $F_\bullet$ is a flag with $F_1 = \atom$, then $\Res_{F_1}(\iota_{\atom}(\OS(M'))) = 0$, so $\Res_{F_\bullet}(\iota_{\atom}(\OS(M'))) = 0$. It follows that $\res_{\atom=0} \dRip{x, y \wedge \atom}= 0$. \end{proof} \begin{proof}[{Proof of \cref{prop:nondeg}}] The statement reduces to the case that $M$ is simple which we assume. Suppose that $0 \neq \eta \in \OS(M)_Q$ belongs to the kernel of $\dRip{\cdot,\cdot}$. By clearing denominators, we may assume that $\eta \in \OS(M)_R:=\OS(M) \otimes_\Z R$. Since the pairing $\dRip{\cdot,\cdot}$ is homogeneous of degree $-d$, we may assume that $\eta$ is a homogeneous element, that is $\eta =\sum_{B\in M} p_B(\a) e_B$ where $p_B(\a) \in R$ all have the same degree. We assume that $\eta \neq 0$ has been chosen to have minimal degree. Pick an atom $\atom$. Write $$ \eta = \eta' + \eta'' \wedge \atom $$ for $\eta'$ and $\eta''$ not depending on $\atom$. Note that $\eta'$ and $\eta''$ are not uniquely determined by $\eta$. For example, if $e_1,e_2,e_3$ are dependent, then $e_2 e_1 - e_3 e_1 + e_3 e_2 = 0$, so $(e_2-e_3)e_1= - e_3 e_2$, where both $e_2-e_3$ and $-e_3e_2$ do not involve $e_1$. The map $\Res_{\atom}: \OS(M) \to \OS(M'')$ can be extended to a map $\Res_{\atom}:\OS(M)_R \to \OS(M'')_{R}$. The map $\theta_{\atom}: R \to R_{\atom}$ can be applied to coefficients to give a map $\theta_{\atom}:\OS(M)_R \to \OS(M)_{R_{\atom}}$. By composition we obtain a map $\theta_{\atom} \Res_{\atom}: \OS(M)_R \to \OS(M'')_{R_{\atom}}$. Consider $\theta_{\atom} \Res_{\atom} \eta = \theta_{\atom} \eta''$. By \cref{lem:contractform}, we deduce that $$ \dRip{\theta_{\atom} \eta'', \tau''}_{M''} = \res_{\atom=0} \dRip{\theta_{\atom} \eta'' \wedge \atom, \tau'' \wedge \atom}_M= \res_{\atom=0} \dRip{\eta, \tau'' \wedge \atom}_M =0 $$ for any $\tau'' \in \OS(M'')$. In the second equality, we used $\res_{\atom=0} \dRip{\eta', \tau'' \wedge \atom}_M = 0$ which holds by \cref{lem:Resa0}, and $\res_{\atom=0} \dRip{\eta''' \wedge \atom, \tau'' \wedge \atom} = 0$ if $\eta''' \in \Ker(\theta_{\atom})$ allowing us to replace $\theta_{\atom} \eta''$ by $\eta''$. By induction we may assume that $\dRip{\cdot,\cdot}_{M''}$ is non-degenerate, and so we have $\theta_{\atom} \eta'' = 0$ inside $\OS(M'')_{R_{\atom}}$, or equivalently, $\Res_{\atom}(\eta) = \eta'' \in \Ker(\theta_{\atom})$ as an element of $\OS(M'')_R$. Thus, $$ \eta \in \iota_{\atom}(\OS(M')_R) + \Ker(\theta_{\atom}). $$ Let $\eta = \iota_{\atom}(\nu) \mod \Ker(\theta_{\atom})$. Then by \cref{lem:deleteform}, we deduce that $ \dRip{\theta_{\atom} \nu, \OS(M')}_{M'} = \theta_{\atom}\dRip{ \nu, \OS(M')}_{M'} =0$ and by induction, we must have $\theta_{\atom}\nu = 0$. Thus $\eta \in \Ker(\theta_{\atom})$, or equivalently, $\eta = a_\atom \mu$ for some homogeneous element $\mu \in \OS(M)_R$. This contradicts our assumption that $\eta$ was chosen to have minimal degree. \end{proof} \subsection{deRham homology pairing} In this section we work with a general extension $(\tM, \star)$ of $\M$ by $\star$, viewed as an affine oriented matroid. Let $\T^\star$ denote the corresponding set of bounded topes \eqref{eq:Tstar}. We have $\tE = E \cup \star$. We define a symmetric bilinear form $\DdRip{\cdot,\cdot}$ on $\Z^{\T^\star}$ with values in $R = \Z[\a] = \Z[a_e \mid e \in E]$. For $P \in \T^\star$, we write $P$ to also denote the corresponding basis element of $\Z^{\T^\star}$. Denote $$a^B:= \prod_{b \in B} a_b, \qquad \mbox{for $B \subseteq E$.}$$ \begin{definition}\label{def:DdR} For two bounded topes $P,Q \in \T^\star$, define $$ \B(P,Q) = \{B \in \B(M) \mid P, Q \in \T^{B}\}. $$ Define the $R$-valued \emph{deRham homology intersection form} on $\Z^{\T^\star}$ by $$ \DdRip{P,Q} := \sum_{B \in \B(P,Q)} a^B. $$ \end{definition} By definition $\DdRip{\cdot,\cdot}$ is a symmetric bilinear form, homogeneous of degree $r$. \begin{theorem}\label{thm:dRmain} The bilinear form $\frac{1}{a_E}\DdRip{\cdot,\cdot}$ (resp. $\DdRip{\cdot,\cdot}$) is the inverse of the bilinear form $\dRip{\cdot,\cdot}$ (resp. $\dRipp{\cdot,\cdot}$) with respect to the basis $\{\Omega_P \mid P \in \T^\star\}$ of $\OS(M)$. \end{theorem} \begin{corollary} Viewing the $a_e$ as complex parameters, the bilinear form $\dRip{\cdot,\cdot}$ on $\OS(M)$ is non-degenerate when $a_E \neq 0$ and \eqref{eq:Mon} is satisfied. \end{corollary} \begin{proof} In \cref{cor:denom}, we will show that the matrix entries of $\dRip{\cdot,\cdot}$ only have the linear forms $a_F$ in the denominator, where $F$ varies over connected flats. Since $\DdRip{\cdot,\cdot}$ has polynomial entries, we obtain the stated result from \cref{thm:dRmain}. \end{proof} \begin{example} Consider the line arrangement with five lines labeled $E = \{a,b,c,d,e\}$ and five regions labeled $1,2,3,4,5$ as in \cref{fig:5line}. We use the five parameters $a,b,c,d,e$ in place of $a_e, e \in E$. \begin{figure} \begin{center} $$ \begin{tikzpicture}[extended line/.style={shorten >=-#1,shorten <=-#1}, extended line/.default=1cm] \useasboundingbox (0,-0.3) rectangle (12,2); \draw (0,0) -- (5,0); \draw[extended line] (1,0) --(3,1); \draw[extended line=0.4cm] (1.5,-0.3) --(3,2); \draw[extended line=0.7cm] (3,0) --(3,1.6); \draw[extended line] (4,0) --(3,1); \node[color=blue] at (-0.2,0) {$a$}; \node[color=blue] at (0,-0.5) {$b$}; \node[color=blue] at (1.5,-0.5) {$c$}; \node[color=blue] at (3.2,-0.5) {$d$}; \node[color=blue] at (4.7,-0.5) {$e$}; \node[color=red] at (2.85,1.45) {$1$}; \node[color=red] at (2.55,1.02) {$2$}; \node[color=red] at (1.6,0.15) {$5$}; \node[color=red] at (2.5,0.35) {$3$}; \node[color=red] at (3.3,0.3) {$4$}; \begin{scope}[shift={(7,0.5)}] \node (h0) at (2,-0.8) {$\hat 0$}; \node (a) at (0,0) {$a$}; \node (b) at (1,0) {$b$}; \node (c) at (2,0) {$c$}; \node (d) at (3,0) {$d$}; \node (e) at (4,0) {$e$}; \node (ab) at (-1.5,1) {$ab$}; \node (ac) at (-0.5,1) {$ac$}; \node (ad) at (0.5,1) {$ad$}; \node (ae) at (1.5,1) {$ae$}; \node (bc) at (2.5,1) {$bc$}; \node (bde) at (3.5,1) {$bde$}; \node (cd) at (4.5,1) {$cd$}; \node (ce) at (5.5,1) {$ce$}; \node (abcde) at (2,1.8) {$abcde$}; \draw (a)--(ab)--(b); \draw (a)--(ac)--(c); \draw (a)--(ad)--(d); \draw (a)--(ae)--(e); \draw (b)--(bc)--(c); \draw (b)--(bde)--(d); \draw (e)--(bde); \draw (c)--(cd)--(d); \draw (c)--(ce)--(e); \draw (h0)--(a); \draw (h0)--(b); \draw (h0)--(c); \draw (h0)--(d); \draw (h0)--(e); \draw (abcde)--(ab); \draw (abcde)--(ac); \draw (abcde)--(ad); \draw (abcde)--(ae); \draw (abcde)--(bc); \draw (abcde)--(bde); \draw (abcde)--(cd); \draw (abcde)--(ce); \end{scope} \end{tikzpicture} $$ \end{center} \caption{Left: a line arrangement in $\P^2$ consisting of 5 lines. The line at infinity is the general extension $\star$ and not one of the hyperplanes of the arrangement. Right: the lattice of flats $L(M)$.} \label{fig:5line} \end{figure} \noindent The deRham cohomology intersection form $\dRip{\cdot,\cdot}$ is given by \scalebox{0.75}{\hspace*{-0.8cm} $ \begin{bmatrix} \frac{1}{d (b+d+e)}+\frac{1}{e (b+d+e)}+\frac{1}{c d}+\frac{1}{c e} & -\frac{1}{e (b+d+e)}-\frac{1}{c e} & -\frac{1}{d (b+d+e)} & \frac{1}{e (b+d+e)}+\frac{1}{d (b+d+e)} & 0 \\ -\frac{1}{e (b+d+e)}-\frac{1}{c e} & \frac{1}{b c}+\frac{1}{b (b+d+e)}+\frac{1}{e (b+d+e)}+\frac{1}{c e} & -\frac{1}{b c}-\frac{1}{b (b+d+e)} & -\frac{1}{e (b+d+e)} & \frac{1}{b c} \\ -\frac{1}{d (b+d+e)} & -\frac{1}{b c}-\frac{1}{b (b+d+e)} & \frac{1}{a c}+\frac{1}{a d}+\frac{1}{b c}+\frac{1}{b (b+d+e)}+\frac{1}{d (b+d+e)} & -\frac{1}{a d}-\frac{1}{d (b+d+e)} & -\frac{1}{a c}-\frac{1}{b c} \\ \frac{1}{e (b+d+e)}+\frac{1}{d (b+d+e)} & -\frac{1}{e (b+d+e)} & -\frac{1}{a d}-\frac{1}{d (b+d+e)} & \frac{1}{a d}+\frac{1}{a e}+\frac{1}{d (b+d+e)}+\frac{1}{e (b+d+e)} & 0 \\ 0 & \frac{1}{b c} & -\frac{1}{a c}-\frac{1}{b c} & 0 & \frac{1}{a b}+\frac{1}{a c}+\frac{1}{b c} \end{bmatrix}. $} For example, the $(1,3)$-entry is equal to $-1/(d (b+d+e))$ because there is a single flag $F_\bullet = (\hat 0 \subset \{d\} \subset \{b,d,e\} \subset \hat 1)$ for which both residues $\Res_{F_\bullet} \bOmega_{P_1}$ and $\Res_{F_\bullet} \bOmega_{P_3}$ are non-zero. This can be deduced from \cref{thm:EL}. \noindent The deRham homology intersection form $\DdRip{\cdot,\cdot}$ is given by $$ \begin{bmatrix} a c d+b c d+c d e & a c d+b c d & a c d & 0 & 0 \\ a c d+b c d & a c d+a c e+b c d+b c e & a c d+a c e & a c e & 0 \\ a c d & a c d+a c e & a b d+a b e+a c d+a c e & a b e+a c e & a b d+a b e \\ 0 & a c e & a b e+a c e & a b e+a c e+a d e & a b e \\ 0 & 0 & a b d+a b e & a b e & a b c+a b d+a b e \end{bmatrix}. $$ For example, the $(1,2)$-entry is equal to $acd+bcd$ because the two simplices bounded by $a,c,d$ and $b,c,d$ contain both of the chambers $1$ and $2$. \end{example} \cref{thm:dRmain} can be proven by induction in a direct combinatorial manner. We instead proceed indirectly, using the flag space of \cite{SV}. This has the advantage of directly connecting our constructions to \cite{SV}. \subsection{Flag space}\label{sec:flagspace} Let $\tF^k$ denote the free abelian group on elements $[F_\bullet]$ for $F_\bullet \in \Fl^k$. Let $G_\bullet = (G_0 \subset G_1 \subset \cdots \subset G_{j-1} \subset G_{j+1} \subset \cdots \subset G_k)$ be a partial flag with a single jump, where $\rk(G_i) = i$. For $L \in L(M)$ satisfying $G_{j-1} < L <G_{j+1}$, let $G^L_\bullet := (G_0 \subset \cdots \subset G_{j-1} \subset L \subset G_{j+1} \subset \cdots \subset G_k) \in \Fl^k$. \begin{definition} The \emph{flag space} $\F^k$ is the quotient of $\tF^k$ by the submodule generated by the elements $$ \sum_{L \in (G_{j-1},G_{j+1})} [G^L_\bullet] $$ for all $0 < j < k$ and all partial flags $G_\bullet$ with a single jump. \end{definition} Define a map $\eta: \tF^k \to \OS^k(M)^* = \Hom(\OS^k(M),\Z)$ by the formula \begin{equation}\label{eq:etadef} (\eta([F_\bullet]), x) = \Res_{F_\bullet} x \end{equation} for $x \in \OS^k(M)$. Abusing notation, we may also write $\Res_y: \OS^k(M) \to \OS^k(M)$ for an arbitrary $y \in \tF^k$. \begin{lemma}\label{lem:Resdes} The action of $\tF^k$ descends to $\F^k$. \end{lemma} \begin{proof} We need to show that for any partial flag $G_\bullet$ with a single jump, we have that $\sum_L \Res_{G^L_\bullet}$ acts by zero on $\OS^k(M)$. Since $\Res_{G^L_\bullet}$ is a composition of residue maps, we reduce immediately to the case $j = 1$. We may further assume that $M$ is simple. Let $e_S \in \OS^k(M)$ for $S \subset E$. If $|S \cap G_2| < 2$ then $ \Res_{G^L_\bullet} e_S = 0$ for any $L$. If $|S \cap G_2| > 2$ then $S$ is not independent and $e_S = 0$. If $S \cap G_2 = \{e,e'\}$, then \begin{equation*} \sum_L \Res_{G^L_\bullet} e_S = \Res_{G_p} \cdots \Res_{G_3} (\Res_{e'} \Res_{e} e_S + \Res_{e} \Res_{e'} e_S) = 0. \qedhere \end{equation*} \end{proof} Suppose that $k = r$. By \cref{thm:EL}, $\OS(M)$ has basis $\{\Omega_P \mid P \in \T^\star\}$. Let $\{\delta_P \mid P \in \T^\star\}$ denote the dual basis of $\OS(M)^*$. In this basis, the homomorphism $\eta: \F^r \to \OS(M)^*$ is given by \begin{equation}\label{eq:etadeltaP} \eta([F_\bullet]) = \sum_{P \in \T^\star} r(P,F_\bullet) \delta_P. \end{equation} Let $\delta_{F_\bullet} \in (\tF^k)^*$ be the linear functional taking the value $1$ on $[F_\bullet]$ and $0$ on all other flags. Define a map $\nu: \OS^k(M) \to (\tF^k)^*$ by $$ \nu(x) := \sum_{F_\bullet \in \Fl^k} \Res_{F_\bullet}(x) \delta_{F_\bullet}. $$ By the proof of \cref{lem:Resdes}, $\nu$ has image in the subspace $(\F^k)^* \subset (\tF^k)^*$. \begin{proposition} The two maps $\eta: \F^k \to \OS^k(M)^*$ and $\nu:\OS^k(M) \to (\F^k)^*$ are transpose to each other. \end{proposition} \begin{proof} Let $S \in \I_k(M)$ and $F_\bullet \in \Fl^k$. We have \begin{align*} ([F_\bullet], \nu(e_S)) &= ([F_\bullet], \sum_{F'_\bullet \in \Fl^k} \Res_{F'_\bullet}(e_S) \delta_{F'_\bullet}) = \Res_{F_\bullet}(e_S) \stackrel{\eqref{eq:etadef}}{=} (\eta([F_\bullet]), e_S). \qedhere \end{align*} \end{proof} A fundamental property of the flag space $\F^k$ is the duality with $\OS^k(M)$. \begin{proposition}[{\cite[Theorem 2.4]{SV}}] \label{prop:Fk} The maps $\eta: \F^k \to \OS^k(M)^*$ and $\nu: \OS^k(M) \to (\F^k)^*$ are isomorphisms. \end{proposition} \begin{remark} The flag spaces $\F^k$ form a complex $(\F^\bullet, d)$ where the differential $d$ is defined in \cite[(2.2.1)]{SV}. The cohomology of this complex is naturally isomorphic to the reduced cohomology of the order complex of $L(M) \setminus \{\hat 0, \hat 1\}$. See \cite[Remark 3.8]{FT}. \end{remark} \subsection{Proof of \cref{thm:dRmain}} In this section, we extend coefficients of $\OS^\bullet(M)$ and $\F^\bullet$ from $\Z$ to $Q$. Following \cite{SV}, define linear maps $R^k: \OS^k(M)_Q \to \F^k_Q$ and $S^k: \F^k_Q \to \OS^k(M)_Q$ by \begin{align}\label{eq:RS} \begin{split} R^k(x) &:=\sum_{F_\bullet \in \Fl^k} \Res_{F_\bullet}(x) \frac{1}{a'_{F_\bullet}} [F_\bullet], \\ S^k([F_\bullet]) &:= \sum_{S \in \I_k(M)} r(S,F_\bullet) a^S e_S. \end{split} \end{align} \begin{proposition}[{\cite[Lemma 3.4.4]{SV}}]\label{prop:SVinverse} For any $k$, we have $S^k \circ R^k = {\rm id}$. \end{proposition} \begin{proof} Proceed by induction on $k$. The case $k=1$ is straightforward. Let $S = \{s_1,\ldots,s_k\}$ be an ordered independent set with closure $F:=\bar S $. Then for $e \in F \setminus S$, the set $S \cup e$ is dependent, giving $$ e \wedge (\sum^k_{i=1} (-1)^{k-i} e_{s_k} \wedge \cdots \widehat{e_{s_i}} \cdots \wedge e_{s_1}) = e_S. $$ Thus we have \begin{equation}\label{eq:SV} (\sum_{e \in F} a_e e) \wedge(\sum^k_{i=1} (-1)^{k-i} e_{s_k} \wedge \cdots \widehat{e_{s_i}} \cdots \wedge e_{s_1})= a_F e_S. \end{equation} Fix an independent set $T \in \I_k$ and let $F=\bar T \in L^k(M)$. We have $$ S^k \circ R^k(e_T) = \sum_{F_\bullet} \frac{1}{a'_{F_\bullet}} r(T,F_\bullet) \sum_{Z} r(Z,F_\bullet) a^Z e_{Z} = \sum_{Z,F_\bullet} \frac{a^Z}{a'_{F_\bullet}} r(T,F_\bullet) r(Z,F_\bullet) e_Z $$ where the summation can be restricted to pairs $(Z, F_\bullet) \in \I_k \times \Fl^k$ such that both $Z$ and $T$ generate $F_\bullet$, and in particular $F_k = F$. For each such pair $(Z,F_\bullet)$, there exists a unique $ b\in Z$ such that $b \notin F_{k-1}$ and a unique $t_i \in T = \{t_1,t_2,\ldots,t_k\}$ such that $t_i \notin F_{k-1}$. We may rewrite the sum as $$ S^k\circ R^k(e_T) = \frac{1}{a_F}\sum_{b \in F} \sum_{i =1}^k (-1)^{k-i} \frac{a_b}{a_G} e_b \left(\sum_{Z^-, F^-_\bullet} \frac{1}{a_{F^-_\bullet}} r(Z^-, F^-_\bullet) r(T^-, F^-_\bullet) a^{Z^-} e_{Z^-} \right) $$ where $G = \overline{T \setminus t_i}$, and $Z^- = Z \setminus b$, and $T^- = T \setminus t_i$, and $F^-_\bullet \in \Fl^{k-1}$ is obtained by dropping $F_k$ from $F_\bullet$. We compute, using the inductive hypothesis, \begin{align*} S^k \circ R^k (e_T) &= \frac{1}{a_F} \left( \sum_{b\in F}a_b e_b \right)\sum_{i =1}^k (-1)^{k-i} \left((S^{k-1} \circ R^{k-1})(e_{T \setminus t_i})\right)\\ &= \frac{1}{a_F} \left( \sum_{b\in F}a_b e_b \right) \wedge \left(\sum_{i =1}^k (-1)^{k-i} e_{T \setminus t_i} \right) & \mbox{by inductive hypothesis}\\ & = e_T &\mbox{by \eqref{eq:SV}.} & \qedhere \end{align*} \end{proof} Define two $\T^\star \times \T^\star$ matrices $$ V(P,Q):=\frac{1}{a_E} \dRip{\Omega_P,\Omega_Q} = \dRipp{\Omega_P,\Omega_Q}, \qquad W(P,Q):= \DdRip{P,Q}. $$ \begin{lemma}\label{lem:V} The matrix $V$ is the matrix of $R^r: \OS(M)_Q \to \F^r_Q$ with respect to the basis $\{\Omega_P \mid P \in \T^\star\}$ of $\OS(M)$ and $\{\delta_P \mid P \in \T^\star\}$ of $\eta:\F^r \cong \OS(M)^*$. \end{lemma} \begin{proof} Follows from the definitions. \end{proof} \begin{lemma}\label{lem:W} The matrix $W$ is the matrix of the linear map $S^r: \F^r \to \OS(M)$ with respect to the basis $\{\delta_P \mid P \in \T^\star\}$ of $\F^r \cong \OS(M)^*$ and $\{\Omega_P \mid P \in \T^\star\}$ of $\OS(M)$. \end{lemma} \begin{proof} Define $S'(\delta_P) = \sum_{Q \in \T^\star} W(P,Q) \Omega_Q$. Then \begin{align*} S'([F_\bullet]) &= S'(\sum_{P \in \T^\star} r(P,F_\bullet) \delta_P) \\ &= \sum_{P \in \T^\star} r(P,F_\bullet) \sum_{Q \in \T^\star} \Omega_Q \sum_{B \in \B(P,Q)} a^B & \mbox{by \cref{def:DdR}} \\ &= \sum_B a^B \left(\sum_{P \in \T^B} \Res_{F_\bullet}(\Omega_P) \right) \left(\sum_{Q \in \T^B} \Omega_Q\right) \\ &= \sum_B a^B r(B,F_\bullet) e_B & \mbox{by \eqref{eq:cone}}. \end{align*} Comparing with the definition of $S^r$, we find that $S' = S^r$. \end{proof} \cref{thm:dRmain} is equivalent to the matrix identity $VW = {\rm Id}$, which follows from \cref{prop:SVinverse}, \cref{lem:V} and \cref{lem:W}. \subsection{Comparison to Schechtman--Varchenko contravariant form} The following result compares our definition with the ``contravariant form'' of Schechtman and Varchenko \cite{SV} defined in the setting of affine hyperplane arrangements. This form is extended to the setting of matroids by Brylawski and Varchenko \cite{BV}. Let $\ip{\cdot,\cdot}_{SV}$ be the form on $\OS^k(M)$ induced by the map $R^k: \OS^k(M) \to (\F^k)^*$. More precisely, $$ \ip{x,y}_{SV} := (\eta(R^k(x)), y), $$ where $(\cdot,\cdot)$ is the natural evaluation map on $\OS^k(M)^* \otimes \OS^k(M)$. \begin{corollary}\label{cor:SVform} Suppose that $x, y \in \OS^k(M)$. Then $$ \dRipp{x,y} = \ip{x,y}_{SV} =\ip{y,x}_{SV}. $$ \end{corollary} \begin{proof} For two independent sets $S,S' \in \I_k(M)$, we compute: \begin{align*} \ip{e_S,e_{S'}}_{SV} &= (\eta(R^k(e_S)), e_{S'}) = \sum_{F_\bullet} r(S,F_\bullet) \frac{1}{a'_{F_\bullet}} (\eta([F_\bullet]), e_{S'}) = \sum_{F_\bullet} r(S,F_\bullet) \frac{1}{a'_{F_\bullet}} r(S',F_\bullet) = \dRipp{e_S, e_{S'}}. \qedhere \end{align*} \end{proof} \begin{remark}\label{rem:a0infinity} Our symmetric bilinear form $\dRip{\cdot,\cdot}$ agrees with that of \cite{SV} in the case of a central hyperplane arrangement, and to that of \cite{BV}. In the case of an affine hyperplane arrangement $\A$, the symmetric bilinear form $\ip{\cdot,\cdot}_{SV,\A}$ of \cite{SV} is obtained from our $\dRip{\cdot,\cdot}$ by ``removing contributions from infinity". More precisely, for an affine matroid $(M,0)$ associated to an affine arrangement $\A$, we have $$ \ip{\cdot,\cdot}_{SV,\A} = \dRip{\cdot,\cdot}|_{a_0 = \infty}. $$ The substitution $a_0 = \infty$ sends $1/a_F$ to 0 for any flat $F \ni 0$ containing $0$. \end{remark} \subsection{Schechtman-Varchenko determinant} The main result of Schechtman and Varchenko \cite{SV} (in the hyperplane arrangement case) and Brylawski and Varchenko \cite{BV} (in the general matroid case) is the following determinantal formula. \begin{theorem}[\cite{SV,BV}]\label{thm:SVdet} The determinant of the form $\dRipp{\cdot,\cdot}$ on the free $\Z$-module $\OS(M)$ is equal to $$ \Delta' = \frac{1}{\prod_{F \in L(M)\setminus \hat 0} a_F^{\beta(M^F) \mu^+(M_F)}}. $$ The determinant of the form $\dRip{\cdot,\cdot}$ on $\OS(M)$ is equal to $$ \Delta = \frac{a_E^{\mu^+(M)-\beta(M)}}{\prod_{F \in L(M)\setminus \{\hat 0,\hat 1\}} a_F^{\beta(M^F) \mu^+(M_F)}}. $$ \end{theorem} For $F$ an atom, we have $\beta(M^F) = 1$, so the exponent $\beta(M^F) \mu^+(M_F)$ is equal to $\mu^+(M_F)$. For $F = E$, we have $\mu^+(M_F) = 1$, so the exponent $\beta(M^F) \mu^+(M_F)$ is equal to $\beta(M)$. \section{Aomoto complex intersection form} In this section, we consider an affine oriented matroid $(\M,0)$, and study the situation when the parameters $a_e \in \C$ are specialized to complex numbers satisfying \begin{equation}\label{eq:sumto0} a_E = \sum_{e \in E} a_e = 0, \end{equation} or equivalently, $a_0 = - \sum_{e \in E \setminus 0} a_e$. In this section, we always assume that \eqref{eq:Mon} is satisfied. By \cref{cor:denom}, $\dRip{\cdot,\cdot}$ is defined when \eqref{eq:Mon} is satisfied. \begin{remark} Falk and Varchenko \cite{FalkVar} study the Schechtman-Varchenko contravariant form on the \emph{subspace of singular vectors} within the flag space $\F^r$, which is dual to the setting of this section. \end{remark} \begin{remark} Instead of taking $a_e, e \in E$ to be complex parameters, we could alternatively work in the ring $R_0 = R/(a_E)$ and its fraction field $Q_0 = \Frac(F_0)$. \end{remark} \subsection{Aomoto complex}\label{sec:Aomoto} Let $a_e$, $e \in E$ be complex parameters. Consider the element $$ \omega = \sum_e a_e e \in \OS^1(M) \otimes_{\Z} \C. $$ Since $\omega \wedge \omega = 0$, multiplication by $\omega$ gives a chain complex, the \emph{Aomoto complex}: \begin{equation}\label{eq:Aomotocomplex} \OS^0(M) \otimes_\Z \C \stackrel{\omega}{\longrightarrow} \OS^1(M) \otimes_\Z \C \stackrel{\omega}{\longrightarrow} \cdots \stackrel{\omega}{\longrightarrow} \OS^r(M) \otimes_\Z \C, \end{equation} denoted $(\OS^\bullet(M), \omega)$. When $\sum_e a_e = 0$, we have $\omega \in \rOS^1(M)$, and we obtain a subcomplex $(\rOS^\bullet(M), \omega) \subset (\OS^\bullet(M),\omega)$. We let $\OS^\bullet(M,\omega)$ (resp. $\rOS^\bullet(M,\omega)$) denote the cohomologies of the Aomoto complex. The cohomology of the Aomoto complex was initially considered in the study of the topology of hyperplane arrangement complements; see \cref{sec:twistedco}. Yuzvinsky \cite{Yuz} studied the cohomology from the abstract perspective of the Orlik-Solomon algebra. \begin{theorem}[{\cite[Proposition 2.1 and Theorem 4.1]{Yuz}}]\label{thm:Yuz}\ \begin{enumerate} \item Suppose that $\sum_e a_e \neq 0$. Then we have $\OS^\bullet(M,\omega) = 0$. \item Suppose that \eqref{eq:sumto0} and \eqref{eq:Mon} hold. Then we have $\rOS^k(M,\omega) = 0$ unless $k = d$, and $ \dim \rOS^{d}(M,\omega) = \beta(M)$. \end{enumerate} \end{theorem} Denote $\rOS(M,\omega):= \rOS^{d}(M,\omega)$ for the non-vanishing cohomology group of the complex $(\rOS^\bullet(M),\omega)$. Henceforth, we always assume that $\sum_e a_e = 0$ when considering the cohomology $\rOS(M,\omega)$. We have the following comparison (cf. \cite[Theorem 4.1]{Yuz}). \begin{proposition}\label{prop:OSrOStwisted} Suppose that \eqref{eq:sumto0} and \eqref{eq:Mon} hold. The isomorphism $\partial: \OS^r(M) \otimes \C \to \rOS^{r-1}(M) \otimes \C$ of \cref{prop:OSrOS} descends to an isomorphism $\partial: \OS^r(M,\omega) \to \rOS^{r-1}(M,\omega) = \rOS(M,\omega)$. \end{proposition} \begin{proof} For any two elements $\alpha, \beta$ of $A^\bullet$, we have the Leibniz rule: $$ \partial( \alpha \wedge \beta) = \pm \partial(\alpha) \wedge \beta + \alpha \wedge \partial(\beta) $$ which holds generally for the contraction of a differential form $\alpha \wedge \beta$ against a vector field $\partial$. Now, let $\alpha = \omega$ and $\beta \in A^\bullet(M)$. Then $\partial(\omega) = \sum_{e \in E} a_e = 0$, so \begin{equation}\label{eq:partialomega} \partial( \omega \wedge \beta) =\omega \wedge \partial(\beta). \end{equation} It follows that $\partial$ sends the subspace $\omega \OS^{r-1}(M) \subset \OS^r(M)$ isomorphically to the subspace $\omega \rOS^{r-2}(M) \subset \rOS^{r-1}(M)$. Thus $\partial$ descends to an isomorphism $\partial: \OS^r(M,\omega) \cong \rOS^{r-1}(M,\omega)$. \end{proof} \begin{lemma}\label{lem:AMgeneric} Let $(\tilM,\star)$ be a general extension of $M$ by $\star$. Then $\rOS(M)_\C := \rOS(M)\otimes_\Z \C \cong \rOS(\tilM, \omega)$. \end{lemma} \begin{proof} Let $\tE = E \cup \star$. There is an inclusion $\iota_0: \rOS(M)_\C \to \rOS(\tilM)_\C$, and therefore a map $\kappa: \rOS(M)_\C \to \rOS(\tilM, \omega)$. We show that this map is surjective. Clearly any $\partial e_B$ where $B \in \B(M)$ is in the image of $\kappa$. Suppose that $\star \cup B' \in \B(\tilM)$. Let us consider $\partial(\star \wedge e_{B'} )\in \rOS(\tilM, \omega)$. By \eqref{eq:partialomega}, we have $$ \omega \wedge \partial \left( \frac{1}{a_\star} e_{B'} \right) = \partial \left(\frac{1}{a_\star}\omega \wedge e_{B'} \right)= \partial(\star \wedge e_{B'}) + \text{ terms in the image of } \kappa, $$ so $\partial(\star \wedge e_{B'} )$ lies in the image of $\kappa$ and we conclude that the map $\kappa$ is surjective. However, by \cref{lem:betageneric}, we have $|\mu(M)| = \beta(\tilM)$, so $\kappa$ is an isomorphism. \end{proof} \subsection{Canonical forms for Aomoto cohomology} For $P \in \T$, the \emph{reduced canonical form} $\bOmega_P \in \rOS(M)$ is $$ \bOmega_P:= \partial \Omega_P, $$ where $\Omega_P$ is the canonical form of \cref{thm:EL}. Recall that $\T^0 \subset \T(\M)$ denotes the set of topes bounded with respect to $0 \in E$. \begin{theorem}[\cite{EL}]\label{thm:ELtwisted} Assume that the $a_e \in \C$ are generic, and \eqref{eq:sumto0}. The canonical forms $$ \{\Omega_P \mid P \in \T^0\}, \qquad \text{and} \qquad \{\bOmega_P \mid P \in \T^0\} $$ give bases of $\OS(M,\omega)$ and $\rOS(M,\omega)$ respectively. \end{theorem} In \cref{cor:Aomotobasis} below, we shall strengthen \cref{thm:ELtwisted} by weakening the genericity assumption. \subsection{Descent of intersection form}\label{sec:descent} According to \cref{thm:SVdet}, when $a_E = 0$, the symmetric form $\dRip{\cdot, \cdot}$ is degenerate. \begin{theorem}\label{thm:descent} Suppose \eqref{eq:sumto0} holds. The symmetric pairing $\dRip{\cdot,\cdot}$ on $\OS(M)_\C$ descends to a symmetric pairing $\bdRip{\cdot, \cdot}$ on $\OS(M,\omega)$. \end{theorem} \begin{proof} Let $B \in \B(M)$ be a basis, and $\tau\in \I_{r-1}(M)$ be an independent set of size $r-1$. We shall check that $$ \dRip{ e_\tau \wedge \omega, e_B}= 0. $$ Let $F_\bullet = (F_0 \subset F_1 \subset \cdots \subset F_r)$ be generated by $B$. Let $L(\tau) \subset L$ be the sublattice of the lattice of flags generated by $\tau$. Since $F_r \notin L(\tau)$, there is a minimal $\alpha = \alpha(F_\bullet)$ such that $F_\alpha \notin L(\tau)$. We say that $F_\bullet$ is \emph{nearly generated} by $\tau$ if $F_\bullet$ is generated by $B' = \tau \cup f$ for some $f \in E$. Let $$ F(\tau,B) := \{F_\bullet \mid F_\bullet \mbox{ is generated by } B \mbox{ and nearly generated by } \tau\}. $$ We define a simple graph $\Gamma(\tau,B)$ with vertex set $F(\tau,B)$. For $i = 1,2,\ldots,r-1$, let $\mu_i(F_\bullet) = \mu_{i,B}(F_\bullet) = (F_0 \subset F_1 \subset \cdots \subset F'_i \subset \cdots \subset F_r)$ be the unique flag differing from $F_\bullet$ in the $i$-th position and such that $\mu_{i,B}(F_\bullet)$ is still generated by $B$. If $B = \{b_1,\ldots,b_r\}$ is ordered so that $F_k = b_{1} \vee \cdots \vee b_{k}$ then we have the explicit formula $$ F'_i = b_{1} \vee \cdots \vee b_{{i-1}} \vee b_{{i+1}}. $$ Let $F_\bullet \in F(\tau,B)$ and $\alpha = \alpha(F_\bullet)$. Then $F_{\alpha-1} \in L(\tau)$ and $F_{\alpha} = F_{\alpha-1} \vee b \notin L(\tau)$ for some $b \in B$. Since $F_\bullet$ is nearly generated by $\tau$, it follows that $F_\bullet$ is generated by the basis $B_{F_\bullet} := \tau \cup b$. We note that if $\alpha > 1$, then $$ B_{\mu_{\alpha-1}(F_\bullet)} = B_{F_\bullet} \qquad \text{and} \qquad \alpha(\mu_{\alpha-1}(F_\bullet)) = \alpha-1 $$ and if $\alpha < r$ then $\alpha(\mu_\alpha(F_\bullet))\in \{\alpha,\alpha+1\}$ (using that $F_\bullet$ is generated by $\tau \cup b$), and $$ B_{\mu_\alpha(F_\bullet)} = \begin{cases} B_{F_\bullet} &\mbox{if $\alpha(\mu_\alpha(F_\bullet)) = \alpha+1$,}\\ B_{F_\bullet}\cup b' - b \text{ for some } b' \in B& \mbox{if $\alpha(\mu_\alpha(F_\bullet)) = \alpha$.}\ \end{cases} $$ It follows that both $\mu_{\alpha-1}(F_\bullet)$ and $\mu_{\alpha}(F_\bullet)$ belong to $F(\tau,B)$. For each $F_\bullet$, we add the edge $(F_\bullet, \mu_{\alpha(F_\bullet)}(F_\bullet))$ whenever $\alpha < r$, and add the edge $(F_\bullet, \mu_{\alpha(F_\bullet)-1}(F_\bullet))$ whenever $\alpha > 1$. (If $\alpha = 1$, we only add $(F_\bullet,\mu_1(F_\bullet))$, and if $\alpha = r$, we only add $(F_\bullet,\mu_{r-1}(F_\bullet))$.). This defines the graph $\Gamma(\tau,B)$. For $F_\bullet \in F(\tau,B)$, define $$ E(F_\bullet) := \{f \mid \tau \cup f \text{ generates } F_\bullet\} = F_{\alpha}\setminus F_{\alpha-1} \subset E. $$ We compute that \begin{align*} \dRip{e_\tau \wedge \omega, e_B}&= \sum_{F_\bullet \in F(\tau,B)} h(F_\bullet) \prod_{i=1}^{r-1} \frac{1}{a_{F_i}} \sum_{E(F_\bullet)} a_f \\ &=\sum_{F_\bullet \in F(\tau,B)}h(F_\bullet)\prod_{i=1}^{r-1} \frac{1}{a_{F_i}} \sum_{F_{\alpha}\setminus F_{\alpha-1}} a_f \\ &=\sum_{F_\bullet \in F(\tau,B)} h(F_\bullet) \prod_{i=1}^{r-1} \frac{1}{a_{F_i}} (a_{F_\alpha} - a_{F_{\alpha-1}})\\ &=\sum_{F_\bullet \in F(\tau,B)} h(F_\bullet) \left(\prod_{i \neq \alpha} \frac{1}{a_{F_i}} - \prod_{i \neq \alpha-1} \frac{1}{a_{F_i}}\right) \end{align*} where the first term is omitted if $\alpha = r$ (using $a_E = 0$), and the second term is omitted if $\alpha = 1$. The sign $h(F_\bullet) \in \{+,-\}$ is given by the formula $$ h(F_\bullet) = r(B,F_\bullet) r(\{f,\tau_{1},\ldots,\tau_{d-1}\},F_\bullet), $$ where $e_\tau = e_{\tau_{d-1}} \wedge \cdots \wedge e_{\tau_{1}}$ and $f$ is any element of $E(F_\bullet)$. Let $(F_\bullet,F'_\bullet)$ be an edge of $\Gamma(\tau,B)$. In the case $\alpha(F_\bullet) \neq \alpha(F'_\bullet)$, we have $r(B,F_\bullet) =- r(B,F'_\bullet) $ and the factor $r(\{f,\tau_1,\ldots,\tau_{r-1}\},F_\bullet)$ changes sign, so we have $h(F_\bullet) = h(F'_\bullet)$. In the case $\alpha(F_\bullet) = \alpha(F'_\bullet)$, we have $r(B,F_\bullet) = - r(B,F'_\bullet)$ but the factor $r(\{f,\tau_1,\ldots,\tau_{r-1}\},F_\bullet)$ does not change sign, so we have $h(F_\bullet) = -h(F'_\bullet)$. The (at most) two terms in the $F_\bullet$ summand cancel out with the corresponding terms (depending on whether $\alpha$ changes) for $F'_\bullet$ and $F''_\bullet$ where the (at most) two edges incident to $F_\bullet$ in $\Gamma(\tau,B)$ are $(F_\bullet,F'_\bullet = \mu_{\alpha(F_\bullet)}(F_\bullet))$ and $(F_\bullet,F''_\bullet = \mu_{\alpha(F_\bullet)-1}(F_\bullet))$. We conclude that $\dRip{ e_\tau \wedge \omega, e_B } = 0$. \end{proof} \begin{example} Let $U_{2,n}$ denote the uniform matroid of rank $2$ on $[n]$. Let $\tau = \{1\}$ and $B = \{i,j\}$. Then $e_\tau \wedge \omega= \sum_{k=2}^n a_k e_1 \wedge e_k$. The flags that potentially contribute to $\dRip{ e_\tau \wedge \omega, e_B = e_i \wedge e_j}$ are $(\hat 0 \subset \{i\} \subset \hat 1)$ and $(\hat 0 \subset \{j\} \subset \hat 1)$, and we obtain $$ \dRip{ e_\tau \wedge \omega, e_B} = \begin{cases} \frac{1}{a_i} a_i - \frac{1}{a_j} a_j = 0 & \mbox{if $i,j \neq 1$,} \\ \frac{1}{a_1} \left(- \sum_{k=2}^n a_k \right) - \frac{1}{a_i} a_i= 0&\mbox{if $i = 1$ and $j >1$,} \end{cases} $$ using \eqref{eq:sumto0}. \end{example} By \cref{prop:dRpartial} and \cref{prop:OSrOStwisted}, the symmetric form $\dRipp{\cdot, \cdot}$ on $\rOS(M)$ also descends to a symmetric form on $\rOS(M,\omega)$, and we use $\bdRip{\cdot, \cdot}$ to denote the symmetric forms on both $\OS(M,\omega) = \OS^r(M,\omega)$ and $\rOS(M,\omega) = \rOS^{r-1}(M,\omega)$. The assumption \eqref{eq:sumto0} is always in place when we use the notation $\bdRip{\cdot, \cdot}$. \subsection{$\beta$\nbc-basis} \def\Bnbc{\B_{\mathbf{nbc}}} We continue to assume that $(M,0)$ is an affine matroid. Recall that in \cref{sec:nbc} we have defined \nbc-bases with respect to a fixed total order $\prec$ on $E$. We assume that $0$ is the minimum of $\prec$. Then every $\nbc$-basis $B$ of $(M,0, \prec)$ contains the element $0$. \begin{definition} A \nbc-basis $B$ is called a $\beta$\nbc-basis if for any $i \in B \setminus 0$ there exists $j \prec i$ such that $B \setminus i \cup j \in \B(M)$. \end{definition} Let $\Bnbc = \Bnbc(\M)$ denote the set of $\beta$\nbc-bases $B$, where we always assume that $B = (b_1 \succ b_{2} \succ \cdots \succ b_r)$ is reversely ordered according to $\prec$. \begin{theorem}[\cite{Zie}]\label{thm:Bnbc} The cardinality of $\Bnbc$ is equal to $\beta(M)$. \end{theorem} $\beta$\nbc-bases behave well with respect to deletion-contraction of the largest element. Suppose that $e_\prec = \max_\prec E$, and consider the deletion-contraction triple $(M,M' = M\backslash e_\prec,M'' =M/e_\prec)$. \begin{proposition}[{\cite[Theorem 1.5]{Zie}}]\label{prop:Zie} Suppose that $e_\prec = \max_\prec E$ is not a loop and $|E| > 1$. Then $$ \Bnbc(M) = \Bnbc(M') \sqcup \{(B \sqcup e_\prec) \mid B \in \Bnbc(M'')\}. $$ \end{proposition} For each ordered basis $B \in \Bnbc$, we define a flag $$ F_\bullet(B) := (\hat 0 \subset \sp(b_1) \subset \sp(b_1,b_2) \subset \cdots \subset \sp(b_1,\ldots,b_{r-1})) \in \Fl^{r-1}(M). $$ \subsection{$\beta$\nbc-basis determinant} We now assume that an orientation $\M$ of $M$ has been fixed. Let $(F^{(1)}_{\bullet},\ldots,F^{(\beta)}_{\bullet})$ be an ordering of $\{F_\bullet(B) \mid B \in \Bnbc\}$, and let $(P_1,\ldots,P_\beta)$ be an ordering of the set $\T^0(\M)$ of bounded topes. Both sets have cardinality $\beta(M)$. In the following, we declare that a $0 \times 0$ matrix has determinant $1$. \begin{proposition}\label{prop:detnbc} The $\beta(M) \times \beta(M)$ matrix $$ Z = \left(\Res_{F^{(i)}_{\bullet}} \bOmega_{P_j}\right)^{\beta(M)}_{i,j=1} $$ has determinant $\pm 1$. \end{proposition} \begin{proof} We may suppose that $M$ is simple. Let $D_M := \det(Z)$ denote the determinant. We prove the statement by a deletion-contraction induction. If $|E| = 0$ we have $\beta(M) = 0$ and if $|E| = 1$ we have $\beta(M) = 1$, and in both cases the claim is clear. Let $e = e_\prec = \max_\prec(M)$, and consider the deletion-contraction triple $(M,M' = M\backslash e,M'' =M/e)$. The set of flags $\Fl_\nbc = \{F_\bullet(B) \mid B \in \Bnbc\}$ decomposes into a disjoint union $\Fl'_\nbc \sqcup \Fl''_\nbc$ as in \cref{prop:Zie}. On the other hand, let us write $\T^0(\M) = \T_1 \sqcup \T_2 \sqcup \T_3$ where \begin{align*} \T_1 &= \mbox{topes in $\T^0(\M)$ that are also topes of $\T^0(\M')$} \\ \T_2 &= \mbox{topes in $\T^0(\M)$ that are cut into two topes in $\T^0(\M')$} \\ \T_3 &= \mbox{topes in $\T^0(\M)$ whose restriction to $E'$ do not belong to $\T^0(\M')$}. \end{align*} Each term of the determinant $D_M$ corresponds to a bijection $\tau: \Fl_\nbc \to \T^0(\M)$ between flags and topes. Suppose that $\tau$ maps two distinct flags $F^{(a)}_{\bullet}, F^{(b)}_{\bullet} \in \Fl''_\nbc$ to two topes $P, P' \in \T_2$ respectively, where $P,P'$ are divided by $e$, i.e. $P(f) = P'(f)$ for all $f \in E \setminus e$ and $P(e) = - P'(e)$. Then we obtain another bijection $\tau'$ by swapping $P,P'$, and since $\Res_e \bOmega_P = - \Res_e \bOmega_{P'}$, the contribution of $\tau$ and $\tau'$ to the determinant cancels out. Furthermore, if $\tau(F_\bullet) \in \T_1$ for $F_\bullet \in \Fl''_\nbc$ then $\Res_{F_\bullet}(\tau(F_\bullet)) = 0$. Let $\Z/2\Z$ act involutively on $\T_2$ by sending a tope $P$ to the adjacent tope on the other side of $e$. Since $|\T_2/(\Z/2\Z)| + |\T_3| = |\T^0(\M'')| = \beta(M'')$, we reduce to summing over bijections $\tau$ that induce a bijection between $\Fl''_\nbc$ and $\T_2/(\Z/2\Z) \sqcup \T_3$. For such $\tau$, we may restrict $\tau$ to $\Fl'_\nbc$ and obtain a bijection $\tau': \Fl'_\nbc \to \T^0(\M')$ by composing with the map that sends each tope in $\T_1 \cup \T_2$ to $\T^0(\M')$ by restricting topes to $E'$. For $F_\bullet \in \Fl'_\nbc$ and $P, P' \in \T_2$ divided by $e$, we have that at least one of $\Res_{F_\bullet}(\bOmega_P), \Res_{F_\bullet}(\bOmega_{P'})$ vanishes, and the sum is equal to $\Res_{F_\bullet}(\bOmega_P+\bOmega_{P'})$. It follows that for each non-vanishing term $\tau': \Fl'_\nbc \to \T^0(\M')$ in the determinant $D_{M'}$, there is a unique corresponding $\tau|_{\Fl'_{\nbc}}$ that gives rise to it. So viewing $\tau|_{\Fl'_{\nbc}}$ as a bijection $\tau|_{\Fl'_{\nbc}}: \Fl'_{\nbc} \to \T^0(\M')$ and $\tau|_{\Fl''_\nbc}$ as a bijection $\tau|_{\Fl''_\nbc}: \Fl''_\nbc \to \T^0(\M'')$, we have a bijection \begin{equation}\label{eq:tautau} \tau \mapsto (\tau' = \tau|_{\Fl'_{\nbc}}, \tau'' = \tau|_{\Fl''_\nbc}) \end{equation} that sends non-zero terms of the determinant $D_M$ to pairs of non-zero terms of the determinants $D_{M'}$ and $D_{M''}$. It remains to show that the signs are correct. Let $P,P'$ be divided by $e$. If we swap $P$ and $P'$ in $\tau$ then $(-1)^\tau$ acquires a sign $(-1)$. However, this is compensated for by the sign-change $\Res_e(\bOmega_P) = - \Res_e(\bOmega_{P'})$ (\cref{thm:EL}). It follows that up to a single global sign, the map \eqref{eq:tautau} sends a term in $D_M$ to a product of terms in $D_{M'} D_{M''}$. By induction, we conclude that $D_M = \pm D_{M'} D_{M''} = \pm 1$. \end{proof} For $F \subset E \setminus 0$, define $$ \bomega(F) := \sum_{e \in F} a_e (e - e_0) \in \rOS^1(M). $$ and $$ S(F_\bullet(B)):= \bomega(F_{r-1}) \wedge \bomega(F_{r-2}) \wedge \cdots \wedge \bomega(F_{1}). $$ \begin{lemma}\label{lem:Resnbc} For $B \in \Bnbc$ and $P \in \T^0(\M)$, we have $\bdRip{S(F_\bullet(B)), \bOmega_P} = \Res_{F_\bullet(B)} \bOmega_P$. \end{lemma} \begin{proof} Recall the isomorphism $S^k: \F^k \to \OS^k(M)$ from \eqref{eq:RS}. We have $$ S^{r-1}(F_\bullet(B))=\omega(F_{r-1}) \wedge \omega(F_{r-2}) \wedge \cdots \wedge \omega(F_{1}), \qquad \omega(F):= \sum_{e \in F} a_e e. $$ Thus $$ S(F_\bullet(B)) = S^{r-1}(F_\bullet(B)) \mod e_0 \OS^\bullet(M). $$ Since $P \in \T^0(\M)$ is bounded, we have that $\Res_{F_\bullet} \bOmega_P = 0$ for any $F_\bullet \in \Fl^{r-1}(M)$ such that $0 \in F_{r-1}$. It follows that $\bdRip{S(F_\bullet(B)), \bOmega_P} =\bdRip{S^{r-1}(F_\bullet(B)), \bOmega_P}$. By \cref{cor:SVform}, \cref{prop:SVinverse} and \eqref{eq:etadef}, we have for $y \in \OS^{r-1}(M)$, $$ \bdRip{S^{r-1}(F_\bullet(B)), y} = (\eta((R^{r-1} \circ S^{r-1})(F_\bullet(B))), y) = (\eta(F_\bullet(B)),y) = \Res_{F_\bullet(B)} y. $$ Thus $\bdRip{S(F_\bullet(B)), \bOmega_P} = \Res_{F_\bullet(B)} \bOmega_P$. \end{proof} By \cref{lem:Resnbc}, \cref{prop:detnbc} calculates the determinant of the $\bdRip{\cdot,\cdot}$-pairing between the two sets $\{S(F_\bullet(B)) \mid B \in \Bnbc(M)\}$ and $\{\bOmega_{P} \mid P \in \T^0(\M)\}$ in $\rOS(M,\omega)$. \begin{corollary}\label{cor:Aomotobasis} When \eqref{eq:Mon} is satisfied, the two sets $\{S(F_\bullet(B)) \mid B \in \Bnbc(M)\}$ and $\{\bOmega_{P} \mid P \in \T^0(\M)\}$ generate dual spanning lattices of $\rOS(M,\omega)$. In particular, both $\{S(F_\bullet(B)) \mid B \in \Bnbc(M)\}$ and $\{\bOmega_{P} \mid P \in \T^0(\M)\}$ form bases of $\rOS(M,\omega)$. \end{corollary} The basis $\{S(F_\bullet(B)) \mid B \in \Bnbc(M)\}$ of $\rOS(M,\omega)$ was studied by Falk and Terao \cite{FT}. \begin{corollary}\label{cor:twistnondeg} When \eqref{eq:Mon} is satisfied, $\bdRip{\cdot,\cdot}$ is a non-degenerate symmetric bilinear form on $\rOS(M,\omega)$. \end{corollary} Note that \cref{cor:twistnondeg} is only proven in the case that an orientation $\M$ of $M$ exists, though it is likely that it always holds. \begin{example}\label{ex:npoint} Let $(\M,0)$ be the affine oriented matroid of the arrangement $\bA$ of $n$ real points $1,2,\ldots,n$ in order on the real affine line, with $0$ the point at infinity. Then the underlying matroid $M$ is isomorphic to the uniform matroid $U_{2,n+1}$ of rank 2 on the set $E = [n+1]$. The space $\rOS(M) = \rOS^1(M)$ has basis $\be_1,\be_2,\ldots,\be_n$, where $\be_i = e_i - e_0$. We take the total order on $E$ to be $0 \prec 1 \prec 2 \prec \cdots \prec n$, so that the $\beta$\nbc-basis is $$ \Bnbc = \{20,30,\ldots,n0\}. $$ and $\beta(M) = n-1$. We have $$ \{S(F_\bullet(B)) \mid B \in \Bnbc(M)\} = \{a_2 \be_2, a_3 \be_3,\ldots, a_n \be_n\} $$ and $$ \{\bOmega_{P} \mid P \in \T^0(\M)\} = \{\be_2 - \be_1,\be_3- \be_2, \ldots, \be_n- \be_{n-1}\}. $$ The matrix $Z$ of \cref{prop:detnbc} is given by $$ \begin{bmatrix} 1 & -1 & 0 & \cdots & 0 \\ 0 &1 & -1 & \cdots & 0 \\ \vdots & \ddots & \ddots & \ddots & \vdots \\ 0 & 0 & 0 & \cdots &-1\\ 0 & 0 & 0 & \cdots &1 \end{bmatrix} $$ which has determinant 1. Let $\theta_i = \be_i- \be_1$ for $i =2,\ldots,n$, so that $\{\theta_2,\ldots,\theta_n\}$ generate the same lattice as $\{\bOmega_{P} \mid P \in \T^0(\M)\}$. Then using the relation $\omega = 0$, we have $$ a_i (e_i - e_0) = a_i e_i + \frac{1}{a_0}(\sum_{i=1}^n a_i e_i) = \frac{a_i}{a_0} \left( a_2 \theta_2 + \cdots + (a_i+a_0) \theta_i + \cdots + a_n \theta_n\right). $$ The transition matrix from $\{S(F_\bullet(B)) \mid B \in \Bnbc(M)\}$ to $\{\theta_2,\ldots,\theta_n\}$ is, after multiplying the rows by $\frac{a_i}{a_0}$, $$ \begin{bmatrix} a_0+a_2 & a_3 & a_4 & \cdots & a_n \\ a_2 &a_0+a_3 & a_4 & \cdots & a_n \\ \vdots & \ddots & \ddots & \ddots & \vdots \\ a_2 & a_3 & a_4 & \cdots &a_n\\ a_2 & a_3 & a_4 & \cdots &a_0+a_n \end{bmatrix} $$ which has determinant $-a^{n-2}_0 a_1$. So the transition matrix between $\{S(F_\bullet(B)) \mid B \in \Bnbc(M)\}$ and $\{\bOmega_{P} \mid P \in \T^0(\M)\}$ has determinant $$ \det = \pm \frac{a_1a_2\cdots a_n}{a_0} =\pm R_M(\a)^{-1}, \mbox{where $R_M(\a)$ is defined in \cref{def:RM}.} $$ \end{example} \subsection{Determinant on bounded chambers} Let $L_0 \subset L(M)$ consist of those flats containing $0$. Thus $L_0 \cong L(M_0)$. In the following, for $F \in L$ or $F \in L_0$, we write $\beta(F)$ to refer to the beta invariant of $M^F$. \begin{definition}\label{def:RM} Let $(M,0)$ be a simple affine matroid. Define $$ R_M(\a):= \frac{\prod_{F \in L_0 \setminus \hat 1} a_{F}^{\beta(F) \beta(M_F)}}{\prod_{F \in L \setminus (L_0 \cup \hat 0)} a_F^{\beta(F) \beta(M_F)}}. $$ \end{definition} The following result is a variant of \cref{thm:SVdet} for $\rOS(M,\omega)$. \begin{theorem}\label{thm:Aomotodet} The determinant of $\bdRip{\cdot, \cdot}$ on the lattice spanned by $\{\bOmega_{P} \mid P \in \T^0(\M)\}$ is equal to $$ \det \bdRip{\cdot, \cdot}_{\T^0}= \pm R_M(\a). $$ \end{theorem} If $\beta(M) = 0$ then the determinant is defined to be 1. \begin{corollary} The transition matrix between the two bases $\{S(F_\bullet(B)) \mid B \in \Bnbc(M)\}$ and $\{\bOmega_{P} \mid P \in \T^0(\M)\}$ has determinant equal to $\pm R_M(\a)^{\pm 1}$. \end{corollary} \begin{proof} The determinant in question is equal to the ratio of the determinants in \cref{thm:Aomotodet} with \cref{prop:detnbc}. \end{proof} \begin{corollary}\label{cor:bnbcdet} The determinant of $\bdRip{\cdot, \cdot}$ on the lattice spanned by $\{S(F_\bullet(B)) \mid B \in \Bnbc(M)\}$ is equal to $\pm R_M(\a)^{-1}$. \end{corollary} Our proof of \cref{cor:bnbcdet} depends on the existence of an orientation $\M$ of $M$, though it is likely that the result holds without this assumption. \begin{example} Consider the affine hyperplane arrangement $\bA$ in $\R^2$, pictured below. $$ \begin{tikzpicture} \draw (0:1.5) -- (180:1.5); \draw (60:1.5) -- (240:1.5); \draw (-60:1.5) -- (-240:1.5); \draw (-1.5,-0.6)--(1.5,-0.6); \node[color=blue] at (-1.6,0) {$1$}; \node[color=blue] at (-1.6,-0.6) {$2$}; \node[color=blue] at (240:1.65) {$3$}; \node[color=blue] at (-60:1.65) {$4$}; \end{tikzpicture} $$ Let $(M,0)$ be the affine matroid of $\A$, with ground set $E = \{0,1,2,3,4\}$. The characteristic polynomial of $\A$ (or the reduced characteristic polynomial of $M$) is $\bchi(t) = t^2- 4t + 4$. The rank $2$ flats are $134,23,24,012,03,04$, of which $134$ and $012$ are connected. The reduced Orlik-Solomon algebra $\rOS(M)$ is the exterior algebra on $\be_1,\be_2,\be_3,\be_4$ modulo the relations $\be_2 \be_1 = 0$, $\be_3 \be_1 - \be_4 \be_1 + \be_4 \be_3 =0$, and all cubic monomials vanish. Thus $\dim(\rOS^2(M)) = 4$ with \nbc~basis $$ \be_3 \be_1, \be_4 \be_1, \be_3 \be_2, \be_4 \be_2. $$ The intersection form $\dRip{\cdot,\cdot}$ on the \nbc~basis is \scalebox{0.75}{\hspace*{-0.5cm} $\begin{bmatrix} \frac{1}{a_1 a_{012}}+\frac{1}{a_0 a_{012}}+\frac{1}{a_0 a_3}+\frac{1}{a_1 a_{134}}+\frac{1}{a_3 a_{134}} & \frac{1}{a_1 a_{012}}+\frac{1}{a_0 a_{012}}+\frac{1}{a_1 a_{134}} & \frac{1}{a_0 a_{012}}+\frac{1}{a_0 a_3} & \frac{1}{a_0 a_{012}} \\ \frac{1}{a_1 a_{012}}+\frac{1}{a_0 a_{012}}+\frac{1}{a_1 a_{134}} & \frac{1}{a_1 a_{012}}+\frac{1}{a_0 a_{012}}+\frac{1}{a_0 a_4}+\frac{1}{a_1 a_{134}}+\frac{1}{a_4 a_{134}} & \frac{1}{a_0 a_{012}} & \frac{1}{a_0 a_{012}}+\frac{1}{a_0 a_4} \\ \frac{1}{a_0 a_{012}}+\frac{1}{a_0 a_3} & \frac{1}{a_0 a_{012}} & \frac{1}{a_2 a_{012}}+\frac{1}{a_0 a_{012}}+\frac{1}{a_0 a_3}+\frac{1}{a_2 a_3} & \frac{1}{a_2 a_{012}}+\frac{1}{a_0 a_{012}} \\ \frac{1}{a_0 a_{012}} & \frac{1}{a_0 a_{012}}+\frac{1}{a_0 a_4} & \frac{1}{a_2 a_{012}}+\frac{1}{a_0 a_{012}} & \frac{1}{a_2 a_{012}}+\frac{1}{a_0 a_{012}}+\frac{1}{a_0 a_4}+\frac{1}{a_2 a_4} \\ \end{bmatrix} $ } \noindent with determinant $$ \frac{a_{01234}^3}{a_0^2 a_1 a_2^2a_3^2 a_4^2 a_{012}a_{134}}, $$ agreeing with \cref{thm:SVdet}. Taking $a_0 \to \infty$, we get $$ \begin{bmatrix} \frac{1}{a_3 a_{134}}+\frac{1}{a_1 a_{134}} & \frac{1}{a_1 a_{134}}& 0 & 0 \\ \frac{1}{a_1 a_{134}} &\frac{1}{a_1 a_{134}}+\frac{1}{a_4 a_{134}}& 0 & 0 \\ 0 & 0 & \frac{1}{a_2a_3} & 0 \\ 0 & 0 & 0 & \frac{1}{a_2a_4} \\ \end{bmatrix} \qquad \mbox{with determinant} \qquad \frac{1}{a_1 a_2^2 a_3^2 a_4^2 a_{134}}. $$ This is the matrix of the Schechtman-Varchenko contravariant form \cite{SV}. Now, let us consider bounded chambers. We have $\beta(M) = \bchi(1) = 1$. The bilinear form $\bdRip{\cdot,\cdot}$ on the basis $\{\bOmega_P \mid P \in \T^0(\M)\}$ is the single entry $$ \frac{1}{a_2a_3} + \frac{1}{a_2a_4} + \frac{1}{a_3(a_1+a_3+a_4)} + \frac{1}{a_4(a_1+a_3+a_4)} = \frac{(a_3+a_4)(a_1+a_2+a_3+a_4)}{a_2a_3a_4 a_{134}}. $$ The factors in the numerator are, up to sign, equal to $a_{012}$ and $a_0$, with $\{0,1,2\}$ and $\{0\}$ the connected flats in $L_0$, agreeing with \cref{thm:Aomotodet}. \end{example} \subsection{Proof of \cref{thm:Aomotodet}} For $F \subset E$, define $\kappa_F := \beta(F) \beta(M_F)$ if $F$ is a flat and $0$ otherwise. Let $e \in E$ be neither a loop or a coloop, and let $\kappa'_F, \kappa''_F$ and $\beta',\beta''$ be the corresponding functions for $M', M''$. \begin{lemma}\label{lem:kappaF} For $F \subset E \setminus e$, we have $\kappa_F + \kappa_{F \cup e} = \kappa'_F + \kappa''_F$. \end{lemma} \begin{proof} We may assume that $M$ is simple. If $F$ and $F\cup e$ are both non-flats, we have $0 = 0$. If both are flats then $F \cup e$ is decomposable, so $\kappa_{F \cup e} = 0$, and we have $$ \kappa_F = \beta(F) \beta(M_F) = \beta(F) (\beta(M/F \backslash e) + \beta(M/(F\cup e))) = \kappa'_F + \kappa''_F, $$ where we have used \eqref{eq:betaeq} and the fact that $e$ is not a loop or coloop in $M_F$. If $F$ is a flat and $F \cup e$ is not, then $\kappa_{F \cup e} = \kappa''_F = 0$, and $$ \kappa_F = \beta(F) \beta(M_F) = \beta(F) \beta(M'_F) = \kappa'_F, $$ because $M/F$ and $(M/F)\backslash e$ have the same lattice of flats (the element $e$ belongs to a non-trivial parallel class in $M_F$). If $F$ is not a flat but $F\cup e$ is, then $\kappa_F = 0$ and $$ \kappa_{F\cup e} = \beta(F \cup e) \beta(M_{F \cup e}) = \beta'(F) \beta(M'_{F }) + \beta''(F) \beta(M''_{F}) = \kappa'_{F} + \kappa''_F, $$ where in the second equality we have used \eqref{eq:betaeq} for $\beta(F \cup e)$ and the isomorphism $L(M_{F \cup e}) = L(M'_F)$. \end{proof} The statement of \cref{thm:Aomotodet} reduces to the case that $M$ is simple, which we assume. We proceed by deletion-contraction induction. When $\rk(M) = 1$, we have $\beta(M) = 1$, and the determinant is equal to $1$. We henceforth assume that $\rk(M) > 1$. If $M$ is not connected, then $\beta(M) = 0$, and the result holds by our convention. We thus assume that $M$ is connected, and in particular has no coloops, and apply deletion-contraction to an element $e \in E \setminus 0$. Since $a_E = 0$, we have $a_0 = - \sum_{e \in E \setminus 0} a_e$. We use this substitution to work within the ring of rational functions in $a_e$, $e \in E \setminus 0$. To begin the proof of the theorem, we note that by \cref{cor:denom}, all the pairings $\bdRip{\bOmega_P, \bOmega_{Q}}$ have denominators belonging to $\{a_F \mid F \text{ connected}\}$. Also, according to \cref{cor:Aomotobasis}, $\{\bOmega_{P} \mid P \in \T^0(\M)\}$ is a basis of $\OS(M,\omega)$ when \eqref{eq:Mon} is satisfied, and thus the determinant in question can only vanish when one of the $a_F$ vanishes. We thus have \begin{lemma} The determinant is of the form \begin{equation}\label{eq:gammaF} D(\M) = C(\M) \cdot \prod_{F \text{ connected }\in L \setminus \{\hat 0,\hat 1\}} a_F^{\gamma_F} \end{equation} where $C(\M)$ is a constant and $\gamma_F \in \Z$. \end{lemma} The assumption that $M$ is connected implies that $F$ and $E \setminus F$ cannot simultaneously be flats. It follows that there are no repetitions (even up to sign) among the linear forms in the product \eqref{eq:gammaF}. In particular, the integers $\gamma_F$ are uniquely determined. Recall the decomposition $\T^0(\M) = \T_1 \sqcup \T_2 \sqcup \T_3$ from the proof of \cref{prop:detnbc}. Let $\T_2' \subset \T_2$ be a choice of a tope $P$ for each pair of topes $(P,P')$ divided by $e$. Define \begin{align*} Z_1 &= \{\bOmega_P \mid P \in \T^0(\M')\} = \{ \bOmega_P \mid P \in \T_1\} \sqcup \{\partial (\Omega_P + \Omega_{P'}) \mid P,P' \in \T_2 \text{ divided by } e\}\\ Z_2 &= \{\bOmega_P \mid P \in \T'_2 \} \sqcup \{ \bOmega_P \mid P \in \T_3\}. \end{align*} It is easy to see that $Z_1 \sqcup Z_2$ is again a basis of of $\rOS(M,\omega)$ and spans the same lattice as $\{\bOmega_{P} \mid P \in \T^0(\M)\}$. We compute the determinant with respect to $Z_1 \sqcup Z_2$, ordering $Z_1$ before $Z_2$. \begin{lemma} Let $Y'$ be the matrix of $\bdRip{\cdot,\cdot}$ with respect to the basis $Z_1 \sqcup Z_2$ and let $Y$ be obtained from $Y'$ by multiplying the rows indexed by $Z_2$ by $a_e$, and then substituting $a_e = 0$ in the whole matrix. Then $Y$ has the form $$ Y = \begin{bmatrix} A & B \\ 0 & D \end{bmatrix} $$ where $A$ is a matrix representing $\bdRip{\cdot,\cdot}_{M'}$ and $D$ is a matrix representing $\bdRip{\cdot,\cdot}_{M''}$. \end{lemma} \begin{proof} The statement regarding $A$ follows immediately from \cref{lem:deleteform}. The statement concerning $D$ follows from \cref{lem:contractform}. Finally, we need to show that the bottom-left block of $Y$ is the zero matrix. Similarly to the proof of \cref{lem:deleteform}, for $x \in Z_1$, we have $\Res_e(x) = 0$. Thus for $x \in Z_1$ and $y \in Z_2$, none of the terms contributing to $\bdRip{x,y}$ have $a_e$ in the denominator. It follows that those entries become $0$ after multiplying by $a_e$ and setting $a_e$ to $0$. \end{proof} The cardinality of $Z_2$ is equal to $\beta(M'')$. It follows that $$ \left.\left(a_{e}^{\beta(M'')} D(\M)\right)\right|_{a_{e}=0} = \pm D(\M') D(\M''). $$ We immediately obtain that the constant $C(\M)$ in \eqref{eq:gammaF} is equal to $\pm 1$. We also deduce that for the flat $F = \{e\}$, the integer $\gamma_F$ is equal to $\beta(M'')$. For this flat, $M_F = M''$ and $\beta(F) = 1$, so $\gamma_F = \kappa_F$. Now let $F \subset E \setminus e$. Then $a_F|_{a_e = 0} = a_{F \cup e}|_{a_e = 0}$. So comparing the coefficient of $a_F|_{a_e = 0}$ on both sides and using the inductive hypothesis for $\M',\M''$ and \cref{lem:kappaF}, we see that it is consistent with $$ \gamma_F = \begin{cases} - \kappa_F & \mbox{ if $F \in L \setminus (L_0 \cup \hat 0)$} \\ \kappa_F & \mbox{if $F \in L_0$.} \end{cases} $$ Note that in the case that $F$ and $F\cup e$ are both flats, the latter is decomposable and $\gamma_{F \cup e} = 0$. However, there is one possible ambiguity. It is possible for $a_F|_{a_e=0}$ to equal $-a_{F'}|_{a_e=0}$. This occurs in two situations: (a) when $F$ and $F'$ are flats such that $F \cup F' = E$ and $F \cap F' = \{e\}$, or (b) when $F$ and $F'$ are flats such that $F \cup F' = E \setminus e$ and $F \cap F' = \emptyset$. Call such pairs of flats $(F,F')$ \emph{$e$-special pairs}. Note that the situation $F \cup F' = E$ and $F \cap F' = \emptyset$ does not appear since in this case $M$ is not connected. Thus for $F \notin \{\hat 0, e\}$ and connected, the integer $\gamma_F$ in $D(\M)$ is equal to the product of the corresponding exponents in $D(\M')$ and $D(\M'')$, except for flats belonging to $e$-special pairs. For a special pair $(F,F')$, the integer $\gamma_F + \gamma_{F'} = \kappa_F + \kappa_{F'}$ is determined. Since $\rk(M) \geq 2$ and $M$ is connected, we have $|E| \geq 3$, and thus there is $e' \in E \setminus \{0, e\}$. Repeating the deletion-contraction argument with $e'$ uniquely determines $\gamma_F$ and $\gamma_{F'}$ for $e$-special pairs $(F,F')$. This completes the proof of \cref{thm:Aomotodet}. \subsection{Inverse} It would be interesting to compute the inverse of the matrix $\bdRip{\bOmega_P,\bOmega_Q}$ for $P,Q \in \T^0$. In the case that $0$ is generic, this follows from \cref{thm:dRmain}. \begin{theorem} Suppose that the affine matroid $(M,0)$ is generic at infinity. Then the inverse of the matrix $\bdRip{\bOmega_P,\bOmega_Q}$ with $P,Q \in \T^0$ is given by the matrix $\DdRip{P,Q} := \sum_{B \in \B(P,Q)} a^B$ of \cref{def:DdR}. \end{theorem} \begin{proof} Let $\M' = \M \setminus 0$, and identify $(\M')^\star = \M$ and $\star = 0$. With these choices, $\T^\star(\M') = \T^0(\M)$. Furthermore, the calculation of $\bdRip{\bOmega_P,\bOmega_Q}_{M}$ only involves flags of flats that do not contain $0$ (and only uses $a_e$, $e \in E \setminus 0$), so we have $\bdRip{\bOmega_P,\bOmega_Q}_{M} = \dRip{\Omega_P,\Omega_Q}_{M'}$. The result follows by applying \cref{thm:dRmain} to $\M'$. \end{proof} \section{Betti homology intersection form} In this section we consider an affine oriented matroid $(\M,0)$. We use the notation and results in \cref{sec:pFl}. \subsection{Definition of Betti homology intersection form} Let $S := \Z[\b] = \Z[b_e \mid e \in E]$ and $K = \Frac(S)$. When we specialize the parameters $b_e$ to complex numbers, they are related to the parameters $a_e$ by the formula \begin{equation}\label{eq:ba} b_e = \exp(-\pi i a_e), \end{equation} to be explained in \cref{ssec:twisted}. For $e \in E$ and $S \subset E$, define $$\tb_e:= b_e^2-1, \qquad b_S := \prod_{e \in S} b_e, \qquad \tb_S := b_S^2 - 1.$$ \begin{lemma} We have $\tb_S = \sum_{\emptyset \subsetneq S' \subseteq S} \prod_{e \in S'} \tb_e$. \end{lemma} Recall that $\T^+$ denotes the set of topes $P \in \T$ satisfying $P(0) = +$. Let $\Z^{\T^+}$ denote the free abelian group with basis $\{P \mid P \in \T(\M)\}$. For clarity, we sometimes also write $[P] \in \Z^{\T^+}$ for the basis element indexed by $P$. For $P \in \T^+$, we define $[-P] := (-1)^r [P] \in \Z^{\T^+}$, so that all topes $P \in \T$ index elements of $\Z^{\T^+}$. We shall define a $K$-valued bilinear pairing on $\Z^{\T^+}$, $$ \halfip{\cdot,\cdot}_B:\Z^{\T^+} \otimes \Z^{\T^+} \to K. $$ For $E_\bullet \in \pFl(P)$, define $$ \frac{1}{\tb_{E_\bullet}} := \prod_{i=1}^s \frac{1}{\tb_{E_i}} = \prod_{i=1}^s \frac{1}{b_{E_i}^2 - 1}. $$ \begin{definition}\label{def:Bettipair} For $G_\bullet = \{\hat 0 \subset G_1 \subset G_2 \cdots \subset G_s \subset E\} \in \pFl(P)$, define $$ \ip{G_\bullet}_B := b(G_\bullet) \sum_{E_\bullet \in \bG_\bullet} \prod_{i=1}^{s(E_\bullet)} \frac{1}{b_{E_i}^2 -1} = b(G_\bullet) \sum_{E_\bullet \in \bG_\bullet} \prod^{s(E_\bullet)}_{i=1} \frac{1}{\tb_{E_i}} = b(G_\bullet) \sum_{E_\bullet \in \bG_\bullet} \frac{1}{\tb_{E_\bullet}} $$ where $$ b(G_\bullet):= \prod_{i=1}^s (-1)^{\rk(G_i)} b_{G_i}. $$ Define the Betti homology intersection form on $\Z^{\T^+}$ by \begin{equation}\label{eq:halfPQ} \halfip{P,Q}_B := \sum_{G_\bullet \in G^{\pm}(P,Q)} (\pm)^r \ip{G_\bullet}_B, \end{equation} where the sign $(\pm)^r$ is equal to $1$ or $(-1)^r$ depending on whether $G$ belongs to $G(P,Q)$ or $G(P,-Q)$, and $P,Q \in \T$. We write $\bip{P,Q}_B$ when we work with coefficients satisfying $b_E = \prod_{e\in E} b_e = 1$. \end{definition} If we consider $\ip{P,Q}_B$ as a rational function in $\{b_e \mid e \in E\}$, then $\bip{P,Q}_B$ is the image of that rational function in the fraction field of the ring $\Z[b_e \mid e \in E]/(\prod b_e = 1)$. It follows from the definitions that \eqref{eq:halfPQ} is consistent with $[P] = (-1)^r[-P]$. \begin{remark} Since the formula for $\ip{G_\bullet}_B$ uses $\bG_\bullet$, the expression $\ip{G_\bullet}_B$ depends on $\pFl(P)$ and thus on $P$. However, if $G_\bullet \in G(P,Q)$, then $\ip{G_\bullet}_B$ is the same whether we consider $G_\bullet \in \pFl(P)$ or $G_\bullet \in \pFl(Q)$; see \cref{lem:closurePQ}. \end{remark} \begin{remark} While the deRham cohomology intersection form $\dRip{\cdot,\cdot}$ is defined for an unoriented matroid $M$, the Betti homology intersection form $\halfip{\cdot,\cdot}_B$ is defined with a choice of orientation $\M$ of $M$. It would be interesting to define $\halfip{\cdot,\cdot}_B$ without choosing an orientation. \end{remark} By \cref{prop:noover}, each term $\frac{1}{\tb_{E_\bullet}}$ appears at most once in $\halfip{P,Q}_B$. \begin{proposition}\label{prop:ipneg} We have $\halfip{P,Q}_B = \halfip{Q,P}_B = \halfip{-P,-Q}_B = \halfip{-Q,-P}_B$. \end{proposition} \begin{proof} Follows from \cref{prop:noover}(2). \end{proof} \begin{example}\label{ex:3pttopeB} We calculate $\halfip{\cdot,\cdot}_B$ for the arrangement in \cref{ex:3pttope}. We order $$\T^+ = \{(+,+,+), (+,-,+), (+,-,-), (+,+,-)\}.$$ We have the $4 \times 4$ matrix: $$\halfip{\cdot,\cdot}^{\T^+}_B= \begin{bmatrix} \frac{(b_1 b_2 b_0-1) (b_1 b_2 b_0+1)}{\left(b_1^2-1\right) \left(b_2^2-1\right) \left(b_0^2-1\right)} & -\frac{(b_1+b_2 b_0) (b_1 b_2 b_0-1)}{\left(b_1^2-1\right) \left(b_2^2-1\right) \left(b_0^2-1\right)} & \frac{(b_1 b_2+b_0) (b_1 b_2 b_0-1)}{\left(b_1^2-1\right) \left(b_2^2-1\right) \left(b_0^2-1\right)} & -\frac{(b_1 b_0+b_2) (b_1 b_2 b_0-1)}{\left(b_1^2-1\right) \left(b_2^2-1\right) \left(b_0^2-1\right)} \\ -\frac{(b_1+b_2 b_0) (b_1 b_2 b_0-1)}{\left(b_1^2-1\right) \left(b_2^2-1\right) \left(b_0^2-1\right)} & \frac{(b_1 b_2 b_0-1) (b_1 b_2 b_0+1)}{\left(b_1^2-1\right) \left(b_2^2-1\right) \left(b_0^2-1\right)} & -\frac{(b_1 b_0+b_2) (b_1 b_2 b_0-1)}{\left(b_1^2-1\right) \left(b_2^2-1\right) \left(b_0^2-1\right)} & \frac{(b_1 b_2+b_0) (b_1 b_2 b_0-1)}{\left(b_1^2-1\right) \left(b_2^2-1\right) \left(b_0^2-1\right)} \\ \frac{(b_1 b_2+b_0) (b_1 b_2 b_0-1)}{\left(b_1^2-1\right) \left(b_2^2-1\right) \left(b_0^2-1\right)} & -\frac{(b_1 b_0+b_2) (b_1 b_2 b_0-1)}{\left(b_1^2-1\right) \left(b_2^2-1\right) \left(b_0^2-1\right)} & \frac{(b_1 b_2 b_0-1) (b_1 b_2 b_0+1)}{\left(b_1^2-1\right) \left(b_2^2-1\right) \left(b_0^2-1\right)} & -\frac{(b_1+b_2 b_0) (b_1 b_2 b_0-1)}{\left(b_1^2-1\right) \left(b_2^2-1\right) \left(b_0^2-1\right)} \\ -\frac{(b_1 b_0+b_2) (b_1 b_2 b_0-1)}{\left(b_1^2-1\right) \left(b_2^2-1\right) \left(b_0^2-1\right)} & \frac{(b_1 b_2+b_0) (b_1 b_2 b_0-1)}{\left(b_1^2-1\right) \left(b_2^2-1\right) \left(b_0^2-1\right)} & -\frac{(b_1+b_2 b_0) (b_1 b_2 b_0-1)}{\left(b_1^2-1\right) \left(b_2^2-1\right) \left(b_0^2-1\right)} & \frac{(b_1 b_2 b_0-1) (b_1 b_2 b_0+1)}{\left(b_1^2-1\right) \left(b_2^2-1\right) \left(b_0^2-1\right)} \\ \end{bmatrix} $$ For instance, the $(1,2)$-entry is \begin{align*} \halfip{(+,+,+), (+,-,+)}_B = &- \frac{b_1}{b_1^2-1} \left(1 + \frac{1}{b_1^2b_2^2-1} + \frac{1}{b_1^2b_0^2-1}\right) - \frac{b_1b_2^2}{(b_2^2-1)(b_1^2b_2^2-1)}\\ &- \frac{b_1b_0^2}{(b_0^2-1)(b_1^2b_0^2-1)} - \frac{b_2b_0}{b_2^2b_0^2-1} \left( 1 +\frac{1}{b_2^2-1}+ \frac{1}{b_0^2-1}\right) , \end{align*} the four terms corresponding to the four elements of $$ G^{\pm}((+,+,+),(+,-,+)) = \{(\hat 0 \subset \{1\} \subset \hat 1), (\hat 0 \subset \{2\} \subset \{1,2\} \subset \hat 1), (\hat 0 \subset \{0\} \subset \{0,1\} \subset \hat 1), (\hat 0 \subset \{0,2\} \subset \hat 1)\}. $$ Note that the $4 \times 4$ matrix $\halfip{\cdot,\cdot}^{\T^+}_B$ has rank 4. The corresponding matrix $\dRip{\cdot,\cdot}_{\T^+}$ has rank 1. \end{example} \subsection{Limit} For $P,Q \in \T$, the intersection pairing $\dRip{\Omega_P,\Omega_Q}$ can be obtained from $\halfip{P,Q}_B$ by taking a limit. In the following result, we view the intersection forms as rational functions in $\a$ and $\b$ respectively. \begin{theorem}\label{thm:limit} For $P,Q \in \T$, we have $$ \dRip{\Omega_P,\Omega_Q} = \lim_{\alpha \to 0} \alpha^d \left.\halfip{P,Q}_B \right|_{b_e \to 1 + \alpha a_e/2}. $$ \end{theorem} \begin{proof} With $b_e = 1 + \alpha a_e/2$, we have $b_e^2 = 1 + \alpha a_e + O(\alpha^2)$, and $\tb_e = \alpha a_e + O(\alpha^2)$. Let $G_\bullet \in \pFl(M)$ and $E_\bullet \in \bG_\bullet$. Then $$ \lim_{\alpha \to 0} \alpha^d \left. b(G_\bullet) \frac{1}{\tb_{E_\bullet}} \right|_{b_e \to 1 + \alpha a_e/2} = \lim_{\alpha \to 0} \alpha^d \prod_{i=1}^{s(E_\bullet)} \frac{1}{\alpha a_{E_i}} = \lim_{\alpha \to 0} \alpha^{d-s} \frac{1}{a_{E_\bullet}}. $$ So in the limit $\lim_{\alpha \to 0}$ only full flags $E_\bullet \in \bG_\bullet \cap \Fl(M)$ (with $d = s$) contribute. The result follows from comparing \cref{thm:dRtope} with \cref{def:Bettipair}. \end{proof} \subsection{Non-degeneracy} In this section, we consider specializations of the parameters $b_e$ to complex numbers. We consider the following genericity assumption: \begin{equation}\label{eq:bMon} \tb_F = b_F^2-1 \neq 0 \mbox{ for all connected }F \in L(M) \setminus \{ \hat 0, \hat 1\} \end{equation} This assumption is implied by \eqref{eq:Mon} when $\a$ and $\b$ are related by \eqref{eq:ba}. Recall from \cref{prop:numbertopes} that $|\T^+| = w_\Sigma(M)$ and $|\T^0| = \beta(M)$. \begin{theorem}\label{thm:Bettinondeg}\ \begin{enumerate} \item Suppose that \eqref{eq:bMon} holds and $b^2_E \neq 1$. Then $\halfip{\cdot,\cdot}_B$ is non-degenerate on $\Z^{\T^+}$. \item Suppose that \eqref{eq:bMon} holds and $b_E = 1$. Then the restriction of $\halfip{\cdot,\cdot}_B$ to $\Z^{\T^0}$ is non-degenerate. \end{enumerate} \end{theorem} \cref{thm:Bettinondeg}(1) follows from \cref{thm:Bettihomdet} below. \begin{proof}[Proof of \cref{thm:Bettinondeg}(2)] It suffices to show that the $\T^0 \times \T^0$ matrix $\halfip{P,Q}_B$ is non-degenerate. Applying \cref{thm:limit} to this matrix, we obtain the $\T^0 \times \T^0$ matrix in \cref{thm:Aomotodet}. This matrix has non-vanishing determinant whenever \eqref{eq:sumto0} and \eqref{eq:Mon} are satisfied. These two conditions follow from taking the limit of $b_E = 1$ and \eqref{eq:bMon} respectively. It follows that the $\T^0 \times \T^0$ matrix $\halfip{P,Q}_B$ has a non-vanishing determinant. \end{proof} \subsection{Determinant} \begin{theorem}\label{thm:Bettihomdet} The determinant of $\halfip{\cdot,\cdot}_B$ on $\Z^{\T^+}$ is equal to $$ \det \halfip{\cdot,\cdot}_B^{\T^+} = (-1)^{(r-1)w_\Sigma(M)}\frac{(1-b_E)^{w_\Sigma(M)-\beta(M)}}{\prod_{F \in L(M) \setminus \{\hat0,\hat1\}} (1-b_F^2)^{\beta(F)w_\Sigma(M_F)}}. $$ \end{theorem} \cref{thm:Bettihomdet} will be proved in \cref{sec:Betticohomdet}. \begin{conjecture}\label{conj:Bettidet} The determinant of the $\T^0 \times \T^0$ matrix $\bip{\cdot,\cdot}_B$ is equal to $$ \frac{ \prod_{F \in L_0 \setminus \hat 1} (1-b_{E \setminus F}^2)^{\beta(F)\beta(M_F)} }{\prod_{F \in L \setminus (L_0 \cup \hat 0)} (1-b_F^2)^{\beta(F) \beta(M_F)}}. $$ \end{conjecture} \begin{example}\label{ex:5pt1} Let us consider the arrangement $\bA$ in \cref{ex:npoint} with $n = 4$, taking the points to be $z_1,z_2,z_3,z_4 \in \R$. We have $\beta(M) = n-1= 3$ and $w_\Sigma(M) = n+1 = 5$. The bounded topes $\T^0$ consists of the $3$ intervals $$ P_1 = [z_1,z_2], \qquad P_2 = [z_2,z_3], \qquad P_{3}= [z_3,z_4]. $$ We write down the intersection forms. The intersection matrix $\halfip{P,Q}_B$ restricted to $(P,Q) \in \T^0 \times \T^0$ is $$ \halfip{\cdot,\cdot}^{\T^0}_B = \begin{bmatrix} \frac{1}{b^2_1-1} + \frac{1}{b^2_2-1} + 1 & -\frac{b_2}{b^2_2-1} & 0 \\ -\frac{b_2}{b^2_2-1} & \frac{1}{b^2_2-1} + \frac{1}{b^2_3-1} + 1 & -\frac{b_3}{b^2_3-1} \\ 0 & -\frac{b_3}{b^2_3-1} & \frac{1}{b^2_3-1} + \frac{1}{b^2_4-1} +1\end{bmatrix} $$ with determinant $$ \det \halfip{\cdot,\cdot}_B^{\T^0} = \frac{\tb_{1234}}{ \tb_1 \tb_2 \tb_3 \tb_4} = - \frac{1- (b_1b_2b_3b_4)^2}{(1-b_1^2)(1-b_2^2)(1-b_3^2)(1-b_4^2)}. $$ If we consider $\T^+$, we have two additional topes $P_4 = [z_4,\infty]$ and $P_5 = [-\infty,z_1]$. We have $$ \halfip{\cdot,\cdot}^{\T^+}_B = \begin{bmatrix} \frac{1}{b^2_1-1} + \frac{1}{b^2_2-1} + 1 & -\frac{b_2}{b^2_2-1} & 0 & 0 & -\frac{b_1}{b^2_1-1} \\ -\frac{b_2}{b^2_2-1} & \frac{1}{b^2_2-1} + \frac{1}{b^2_3-1} + 1 & -\frac{b_3}{b^2_3-1} &0 &0 \\ 0 & -\frac{b_3}{b^2_3-1} & \frac{1}{b^2_3-1} + \frac{1}{b^2_4-1} +1 & -\frac{b_4}{b^2_4+1} &0 \\ 0 &0 & -\frac{b_4}{b^2_4-1} &\frac{1}{b^2_4-1} + \frac{1}{b^2_0-1} +1 &-\frac{b_0}{b^2_0-1} \\ -\frac{b_1}{b^2_1-1} &0 &0 & -\frac{b_0}{b^2_0-1} &\frac{1}{b^2_0-1} + \frac{1}{b^2_1-1} +1 \\ \end{bmatrix} $$ with determinant $$ \det \halfip{\cdot,\cdot}_B^{\T^+}= \frac{(b_E-1)^2}{\tb_0 \tb_1 \tb_2 \tb_3 \tb_4} = - \frac{ (1 - b_0b_1b_2b_3b_4)^2}{(1-b_0^2)(1-b_1^2)(1-b_2^2)(1-b_3^2)(1-b_4^2)}. $$ When $b_E = b_0b_1b_2b_3b_4 = 1$, $\halfip{\cdot,\cdot}_B$ is degenerate on $\Z^{\T^+}$ with a two-dimensional kernel. However, it restricts to a non-degenerate symmetric bilinear form on $\Z^{\T^0}$. \end{example} \section{Betti cohomology intersection form} \def\id{{\rm id}} In this section, we assume that $(\M,0)$ is an affine oriented matroid. \subsection{Definition of Betti cohomology intersection form} Given $P,Q \in \T$, define the \emph{separating set} $$ \sep(P,Q) := \{ e \in E \mid P(e) \neq Q(e)\} \subset E. $$ If $P, Q \in \T^+$, then $\sep(P,Q) \subseteq E \setminus 0$. \begin{definition}\label{def:Betticohpair} The $S$-valued Betti cohomology intersection form on $\Z^{\T^+}$ is given by $$ \ip{P,Q}^B := b_{\sep(P,Q) }+(-1)^r b_{E \setminus \sep(P,Q)} = \ip{Q,P}^B $$ for $P,Q \in \T^+$. \end{definition} Note that $\ip{P,Q}^B$ can be extended to $P, Q \in \T$, with $\ip{P,Q}^B= (-1)^r \ip{P,-Q}^B$. The sign $(-1)^r$ is parallel to the signs in \cref{def:Bettipair} and \cref{thm:EL}(1). The main result of this section is the following. \begin{theorem}\label{thm:Bettiinverse} The $\T^+ \times \T^+$ matrices $(-1)^{r-1}(1- b_E )^{-1}\ip{\cdot,\cdot}^B_{\T^+}$ and $\ip{\cdot,\cdot}^{\T^+}_B$ are inverse. \end{theorem} \subsection{Varchenko's bilinear form} Define \emph{Varchenko's bilinear form} \cite{Var} on $\Z^{\T^+}$ by $$ \ip{P,Q}^V := b_{\sep(P,Q)}. $$ It is immediate from the definition that $\ip{P,Q}^V= \ip{P,Q}^B_0 := \ip{P,Q}^B|_{b_0 = 0}$. Evaluating \cref{thm:Bettiinverse} at $b_0 = 0$ gives the following corollary. \begin{corollary}\label{cor:Var} The inverse of the $\T^+ \times \T^+$ matrix $\halfip{\cdot, \cdot}_{0,B}:=\halfip{\cdot,\cdot}_B|_{b_0 = 0}$ is equal to $(-1)^{r-1} \ip{\cdot,\cdot}^V$. \end{corollary} \cref{cor:Var} generalizes \cite[Theorem 5.2]{Var} which describes the possible denominators that can appear in the inverse of $\ip{\cdot,\cdot}^V$. \begin{remark} Our bilinear form $\ip{P,Q}^{B}$ can be obtained from Varchenko's bilinear form as follows. Let $(\bM,\star)$ be a general lifting of $\M$ by a new element $\star$. Then $\bT^+ = \T^+(\bM)$ is naturally in bijection with $\T$. Let $\bP,\bQ \in \bT^+$ lift the topes $P,Q \in \T$ . Then we have the equality $\ip{\bP,\bQ}_{\bM}^B = \ip{P,Q}^V + \ip{P,-Q}^V$. \end{remark} \subsection{Determinant}\label{sec:Betticohomdet} \begin{theorem}\label{thm:Bettidet} The bilinear form $\ip{\cdot,\cdot}^B$ on $\Z^{\T^+}$ has determinant $$ \det \ip{\cdot,\cdot}^B_{\T^+} = (1 - b_E )^{\beta(M)} \prod_{F \in L(M) \setminus \{\hat0,\hat1\}} (1-b_F^2)^{\beta(F)w_\Sigma(M_F)}. $$ \end{theorem} \begin{proof}[Proof of \cref{thm:Bettihomdet}] By \cref{prop:numbertopes}, we have $|\T^+| = w_\Sigma(M)$. Combine \cref{thm:Bettidet} and \cref{thm:Bettiinverse} to obtain \begin{align*} \det \halfip{\cdot,\cdot}_B^{\T^+} &= (-1)^{(r-1) w_\Sigma(M)} (1- b_E)^{w_\Sigma(M)}\frac{1}{\det \ip{\cdot,\cdot}^B_{\T^+}}. \qedhere \end{align*} \end{proof} \begin{example}\label{ex:5pt2} The inverse of the matrix $\halfip{\cdot,\cdot}_B^{\T^+}$ from \cref{ex:5pt1} is $1/(b_E- 1)$ times $$ \ip{\cdot,\cdot}^B_{\T^+} =\begin{bmatrix} b_0 b_1 b_2 b_3 b_4+1 & b_0 b_1 b_3 b_4+b_2 & b_0 b_1 b_4+b_2 b_3 & b_0 b_1+b_2 b_3 b_4 & b_0 b_2 b_3 b_4+b_1 \\ b_0 b_1 b_3 b_4+b_2 & b_0 b_1 b_2 b_3 b_4+1 & b_0 b_1 b_2 b_4+b_3 & b_0 b_1 b_2+b_3 b_4 & b_0 b_3 b_4+b_1 b_2 \\ b_0 b_1 b_4+b_2 b_3 & b_0 b_1 b_2 b_4+b_3 & b_0 b_1 b_2 b_3 b_4+1 & b_0 b_1 b_2 b_3+b_4 & b_0 b_4+b_1 b_2 b_3 \\ b_0 b_1+b_2 b_3 b_4 & b_0 b_1 b_2+b_3 b_4 & b_0 b_1 b_2 b_3+b_4 & b_0 b_1 b_2 b_3 b_4+1 & b_0+b_1 b_2 b_3 b_4 \\ b_0 b_2 b_3 b_4+b_1 & b_0 b_3 b_4+b_1 b_2 & b_0 b_4+b_1 b_2 b_3 & b_0+b_1 b_2 b_3 b_4 & b_0 b_1 b_2 b_3 b_4+1 \end{bmatrix}, $$ which has determinant $$\det \ip{\cdot,\cdot}^B_{\T^+} = (1-b_0^2)(1-b_1^2)(1-b_2^2)(1-b_3^2)(1-b^2_4) (1 - b_0b_1b_2b_3b_4)^3.$$ Setting $b_0 = 0$, we obtain Varchenko's matrix $$ \ip{\cdot,\cdot}^V_{\T^+} = \begin{bmatrix} 1 & b_2 & b_2 b_3 & b_2 b_3 b_4 & b_1 \\ b_2 & 1 & b_3 & b_3 b_4 & b_1 b_2 \\ b_2 b_3 & b_3 & 1 & b_4 & b_1 b_2 b_3 \\ b_2 b_3 b_4 & b_3 b_4 & b_4 & 1 & b_1 b_2 b_3 b_4 \\ b_1 & b_1 b_2 &b_1 b_2 b_3 & b_1 b_2 b_3 b_4 & 1 \end{bmatrix} $$ which has determinant $$\det \ip{\cdot,\cdot}^V_{\T^+} = (1-b_1^2)(1-b_2^2)(1-b_3^2)(1-b_4)^2.$$ \end{example} \begin{example} We continue \cref{ex:3pttopeB}. The inverse of the matrix $\halfip{\cdot,\cdot}^{\T^+}_B$ is equal to $1/(1-b_0b_1b_2)$ times the matrix $$ \ip{\cdot,\cdot}^B_{\T^+} = \begin{bmatrix} 1-b_0 b_1 b_2 & b_1-b_0 b_2 & b_1 b_2-b_0 & b_2-b_0 b_1 \\ b_1-b_0 b_2 & 1-b_0 b_1 b_2 & b_2-b_0 b_1 & b_1 b_2-b_0 \\ b_1 b_2-b_0 & b_2-b_0 b_1 & 1-b_0 b_1 b_2 & b_1-b_0 b_2 \\ b_2-b_0 b_1 & b_1 b_2-b_0 & b_1-b_0 b_2 & 1-b_0 b_1 b_2 \\ \end{bmatrix} $$ which has determinant $\det \ip{\cdot,\cdot}^B_{\T^+} = (1-b_0^2)^2 (1-b_1^2)^2 (1-b_2^2)^2$. Note that in this case $\beta(M) = 0$. \end{example} \begin{example} Consider the line arrangement $\bA$ in $\P^2$, pictured below. The parameters $b_e, e \in E$ are taken to be $a,b,c,d$, where $d = b_0$ corresponds to the line at infinity. $$ \begin{tikzpicture} \draw (0.1,-0.1) -- (2.4,2.9); \draw (-0.5,0.8) -- (3.5,0.8); \draw (2.9,-0.1) -- (0.6,2.9); \draw (1.5,1.25) circle (1.7); \node[color=blue] at (0,-0.1) {$b$}; \node[color=blue] at (-0.6,0.8) {$a$}; \node[color=blue] at (3,-0.15) {$c$}; \node[color=blue] at (1.5,-0.6) {$d$}; \node[color=red] at (0.2,0.55) {$1$}; \node[color=red] at (0.5,1.5) {$2$}; \node[color=red] at (1.5,2.2) {$3$}; \node[color=red] at (2.5,1.5) {$4$}; \node[color=red] at (2.8,0.55) {$5$}; \node[color=red] at (1.5,0.4) {$6$}; \node[color=red] at (1.5,1.15) {$7$}; \end{tikzpicture} $$ The intersection matrices are $$ \ip{\cdot,\cdot}^B_{\T^+} =\begin{bmatrix} 1-a b c d & a-b c d & a b-c d & a b c-d & b c-a d & c-a b d & a c-b d \\ a-b c d & 1-a b c d & b-a c d & b c-a d & a b c-d & a c-b d & c-a b d \\ a b-c d & b-a c d & 1-a b c d & c-a b d & a c-b d & a b c-d & b c-a d \\ a b c-d & b c-a d & c-a b d & 1-a b c d & a-b c d & a b-c d & b-a c d \\ b c-a d & a b c-d & a c-b d & a-b c d & 1-a b c d & b-a c d & a b-c d \\ c-a b d & a c-b d & a b c-d & a b-c d & b-a c d & 1-a b c d & a-b c d \\ a c-b d & c-a b d & b c-a d & b-a c d & a b-c d & a-b c d & 1-a b c d \\ \end{bmatrix} $$ with determinant $\det \ip{\cdot,\cdot}^B_{\T^+} = (1-a^2)^3 (1-b^2)^3 (1-c^2)^3 (1-d^2)^3 (1-abcd)$ and \def\acd{[acd]} \def\bcd{[bcd]} \def\abc{[abc]} \def\abd{[abd]} \def\cd{[cd]} \def\ac{[ac]} \def\ad{[ad]} \def\ab{[ab]} \def\bd{[bd]} \def\bc{[bc]} $$\halfip{\cdot,\cdot}^{\T^+}_B = \begin{bmatrix} \frac{\acd}{\tilde a \tilde c \tilde d} & - \frac{a \cd}{\tilde a \tilde c \tilde d} & -\frac{c d}{\tilde c \tilde d} & \frac{d \ac}{\tilde a \tilde c \tilde d} & -\frac{a d}{\tilde a \tilde d} & -\frac{c \ad}{\tilde a \tilde c \tilde d} & \frac{a c}{\tilde a \tilde c} \\ -\frac{a \cd}{\tilde a \tilde c \tilde d} & \frac{\ab \cd}{\tilde a \tilde b \tilde c \tilde d} &- \frac{b \cd}{\tilde b \tilde c \tilde d} & \frac{\tilde a b c \tilde d-a b^2 \tilde c d+a \tilde c d}{\tilde a \tilde b \tilde c \tilde d} & \frac{d \ab}{\tilde a \tilde b \tilde d} & \frac{-a^2 b \tilde c d+a \tilde b c \tilde d+b \tilde c d}{\tilde a \tilde b \tilde c \tilde d} & -\frac{c \ab}{\tilde a \tilde b \tilde c} \\ -\frac{c d}{\tilde c \tilde d} & -\frac{b \cd}{\tilde b \tilde c \tilde d} & \frac{\bcd}{\tilde b \tilde c \tilde d} & -\frac{c\bd}{\tilde b \tilde c \tilde d} & -\frac{b d}{\tilde b \tilde d} & \frac{d \bc}{\tilde b \tilde c \tilde d} & \frac{b c}{\tilde b \tilde c} \\ \frac{d \ac }{\tilde a \tilde c \tilde d} & \frac{\tilde a b c \tilde d-a b^2 \tilde c d+a \tilde c d}{\tilde a \tilde b \tilde c \tilde d} & -\frac{c\bd}{\tilde b \tilde c \tilde d} & \frac{\ac \bd}{\tilde a \tilde b \tilde c \tilde d} & -\frac{a\bd}{\tilde a \tilde b \tilde d} & \frac{-a^2 \tilde b c d+a b \tilde c \tilde d+\tilde b c d}{\tilde a \tilde b \tilde c \tilde d} & -\frac{b\ac}{\tilde a \tilde b \tilde c} \\ -\frac{a d}{\tilde a \tilde d} & \frac{d \ab }{\tilde a \tilde b \tilde d} & -\frac{b d}{\tilde b \tilde d} & -\frac{a\bd}{\tilde a \tilde b \tilde d} & \frac{\abd}{\tilde a \tilde b \tilde d} & -\frac{b\ad}{\tilde a \tilde b \tilde d} & \frac{a b}{\tilde a \tilde b} \\ -\frac{c\ad}{\tilde a \tilde c \tilde d} & \frac{-a^2 b \tilde c d+a \tilde b c \tilde d+b \tilde c d}{\tilde a \tilde b \tilde c \tilde d} & \frac{d \bc}{\tilde b \tilde c \tilde d} & \frac{-a^2 \tilde b c d+a b \tilde c \tilde d+\tilde b c d}{\tilde a \tilde b \tilde c \tilde d} & -\frac{b \ad}{\tilde a \tilde b \tilde d} & \frac{\ad\bc}{\tilde a \tilde b \tilde c \tilde d} & -\frac{a\bc}{\tilde a \tilde b \tilde c} \\ \frac{a c}{\tilde a \tilde c} & -\frac{c\ab}{\tilde a \tilde b \tilde c} & \frac{b c}{\tilde b \tilde c} & -\frac{b\ac}{\tilde a \tilde b \tilde c} & \frac{a b}{\tilde a \tilde b} & -\frac{a\bc}{\tilde a \tilde b \tilde c} & \frac{\abc}{\tilde a \tilde b \tilde c} \\ \end{bmatrix} $$ where for clarity we have used the notation $\tilde a := a^2-1$, $[acd] := a^2c^2d^2 -1$, and so on. We have $\beta(M) = 1$ and $w_\Sigma(M) = 7$, and $$\det \halfip{\cdot,\cdot}^{\T^+}_B = \frac{(1 - a b c d)^6}{(1 - a^2)^3 (1 - b^2)^3 (1 - c^2)^3 (1 - d^2)^3}.$$ \end{example} \subsection{Proof of \cref{thm:Bettidet}} \begin{lemma}\label{lem:Bettifactor} The determinant $\det \ip{\cdot,\cdot}^B_{\T^+}$ is a constant times a rational function whose irreducible factors belong to $$ \{\tb_F \mid F \in L(M) \setminus \{\hat 0,\hat 1\} \} \cup \{(1 - b_E)\}. $$ \end{lemma} \begin{proof} Follows from the form of $\halfip{\cdot,\cdot}_B$ and \cref{thm:Bettiinverse}. \end{proof} We deduce \cref{thm:Bettidet} from the following theorem of Varchenko \cite{Var}, generalized to the setting of oriented matroids in \cite{HV,Ran}. Recall that $L_0 \subset L(M)$ denotes the subposet of flats $F$ that contain $0$. \begin{theorem}[\cite{Var,HV,Ran}]\label{thm:BettiVar} The $\T^+ \times \T^+$ matrix $\ip{\cdot,\cdot}^V_{\T^+}$ has determinant $$ \det \ip{P,Q}^V_{\T^+} = \prod_{F \in L(M) \setminus \{L_0 \cup \hat 0\}} (-\tb_F)^{\beta(F)w_\Sigma(M_F)} =\prod_{F \in L(M) \setminus \{L_0 \cup \hat 0\}} (1-b_F^2)^{\beta(F)w_\Sigma(M_F)}. $$ \end{theorem} We apply \cref{thm:BettiVar} to the affine matroid $(\bM,\star)$, a general lifting of $\M$. We denote the groundset of $\bM$ by $\bE = E \sqcup \star$. We consider the topes $\bT^+$ of $\bM$ positive with respect to $\star$. Then $\bT^+$ is in bijection with $\T = \T(\M)$ under the map $\bP \mapsto \bP|_E$. For $P \in \T$, write $\bP \in \bT^+$ for the corresponding positive tope. We have $$ \ip{\bP,\bQ}^V = \begin{cases} b_{\sep(P,Q)} &\mbox{ if $P,Q \in \T^+$,} \\ b_{\sep(-P,-Q)} & \mbox{ if $-P,-Q \in \T^+$,} \\ b_{E \setminus \sep(P,-Q)} &\mbox{ if $P \in \T^+$ and $-Q \in \T^+$,} \\ b_{E \setminus \sep(-P,Q)} & \mbox{ if $-P \in \T^+$ and $Q \in \T^+$.} \end{cases} $$ Here, $\ip{\cdot,\cdot}^V$ is calculated with respect to the affine matroid $(\bM,\star)$ while $\sep(\cdot,\cdot)$ is calculated with respect to $(\M,0)$. Note that $b_\star$ does not appear in these formulae. We extend the bilinear form $\ip{\cdot,\cdot}^V$ on $\Z^{\bT^+}$ to $\R^{\bT^+}$, and calculate the determinant with respect to the basis $$ \left\{ \frac{1}{\sqrt{2}}([\bP]+[-\bP]) \mid P \in \T^+ \right\} \cup \left\{ \frac{1}{\sqrt{2}}([\bP]-[-\bP]) \mid P \in \T^+ \right\}. $$ We have that \begin{align*} \ip{\frac{1}{\sqrt{2}}([\bP]+[-\bP]), \frac{1}{\sqrt{2}}([\bQ]+[-\bQ])}^V &= b_{\sep(P,Q)}+ b_{E \setminus \sep(P,Q)} \\ \ip{\frac{1}{\sqrt{2}}([\bP]+[-\bP]), \frac{1}{\sqrt{2}}([\bQ]-[-\bQ])}^V&=0 \\ \ip{\frac{1}{\sqrt{2}}([\bP]-[-\bP]), \frac{1}{\sqrt{2}}([\bQ]-[-\bQ])}^V&= b_{\sep(P,Q)}- b_{E \setminus \sep(P,Q)}. \end{align*} Thus the intersection matrix of $\ip{\cdot,\cdot}^V$ with respect to this basis is block diagonal. One block is identical to $\ip{\cdot,\cdot}^B$ on the basis $\T^+$ and the other is identical to $ \ip{\cdot,\cdot}^B|_{b_0 \mapsto -b_0}$ on the same basis. \begin{lemma}\label{lem:wsigma} Let $F \in L(\overline{M}) \setminus \hat 0$ be such that $\star \notin F$. Then $$ w_\Sigma(\overline{M}_F) = \begin{cases} 1 & \mbox{if $F = E$} \\ 2 w_\Sigma(M_F) & \mbox{if $F \subsetneq E$.} \end{cases} $$ \end{lemma} \begin{proof} When $F = E$, the matroid $\overline{M}_F$ is a rank 1 matroid on the single element $\star$. The reduced characteristic polynomial is equal to $1$, so $w_\Sigma = 1$. When $F \subsetneq E$, the matroid $\overline{M}_F$ is a general lifting of $M_F$. By \cref{lem:genericlift}, we have $\chi_{\overline{M}_F}(t) = (t-1)\chi_{M_F}(t)$ and it follows that $w_\Sigma(\overline{M}_F) = 2w_{\Sigma}(M_F)$. \end{proof} Using \cref{lem:wsigma}, we conclude that \begin{align*} \det \ip{\cdot,\cdot}^B_{\T^+} (\det \ip{\cdot,\cdot}^B_{\T^+}) |_{b_0 \mapsto -b_0} &= \pm \prod_{F \in L(\overline{M}) \setminus \{L_\star \cup \hat 0\}} (-\tb_F)^{\beta(F)w_\Sigma(\overline{M}_F)} \\ &= \pm \prod_{F \in L(M) \setminus \{\hat 0, \hat 1\}}(1-b_F^2)^{2\beta(F)w_\Sigma(M_F)}(1-b_E^2)^{\beta(M)}. \end{align*} Comparing this with \cref{lem:Bettifactor}, we deduce that for $L \neq \hat 1$, the factor $(1-b_F^2)^{2\beta(F)w_\Sigma(M_F)}$ factors up to sign as $(1-b_F^2)^{\beta(F)w_\Sigma(M_F)} (1-b_F^2)^{\beta(F)w_\Sigma(M_F)}$ in $\det \ip{\cdot,\cdot}^B_{\T^+} (\det \ip{\cdot,\cdot}^B_{\T^+}) |_{b_0 \mapsto -b_0} $. However, the factor $(1-b_E^2)^{\beta(M)}$ factors as $(1-b_E)^{\beta(M)} (1+b_E)^{\beta(M)}$. This proves \cref{thm:Bettidet} up to sign. To fix the sign, we substitute $b_0 = 0$ into the intersection matrix of $\ip{\cdot,\cdot}^B$, obtaining an instance of $\ip{\cdot,\cdot}^V$ for $(\M,0)$. Applying \cref{thm:BettiVar} we see that the sign gives the stated formula. \subsection{Proof of \cref{thm:Bettiinverse}} Let $P \in \T$ and $L(P) \subset L(M)$ denote the face lattice of the tope $\T$. By convention, $L(P)$ has minimal element $\hat 0 = \emptyset$ and maximal element $\hat 1 = E$. For a graded poset $L$, we let $\chi(L):= \sum_{x \in L} (-1)^{\rk(x)}$ be its Euler characteristic. \begin{theorem}[see {\cite[Theorem 4.3.5]{OMbook}}] \label{thm:sphere} The lattice $L(P)$ is the (augmented with a $\hat 0$ and $\hat 1$) face lattice of regular cell decomposition of a $(d-1)$-dimensional sphere. In particular, $L(P)$ is a graded poset with Euler characteristic equal to 0. The subposet $L(P) \setminus \hat 1$ has Euler characteristic $(-1)^{d}$. \end{theorem} We shall show that $$ \sum_{Q \in \T^+} \ip{P,Q}_B \ip{Q,R}^B = (-1)^{r-1}(1 - b_E) \delta_{P,R}. $$ For $P, R \in \T^+$, let us define \begin{equation}\label{eq:U} U = U(P,R):= \sum_{G_\bullet \in \pFl(P)} \ip{G_\bullet}_B b_{\sep(R,Q_{G_\bullet})}, \qquad V = V(P,R):=(-1)^r \sum_{G_\bullet \in \pFl(P)} \ip{G_\bullet}_B b_{\sep(R,(-Q)_{G_\bullet})}. \end{equation} Note that if $Q_{G_\bullet} \in \T^+$ then $\sep(R,Q_{G_\bullet}) \subseteq E \setminus 0$, but if $(-Q)_{G_\bullet} \in \T^+$, then $0 \in \sep(R,Q_{G_\bullet})$. The following identity follows from the definitions. \begin{lemma} We have $$ \sum_{Q \in \T^+} \ip{P,Q}_B \ip{Q,R}^B = U(P,R) + V(P,R). $$ \end{lemma} \begin{proof} Follows from the definitions. The sign $(\pm)^r$ in \cref{def:Bettipair} of $\ip{P,Q}_B$ cancels out with the sign $(-1)^r$ in \cref{def:Betticohpair}. \end{proof} \begin{example} We continue \cref{ex:5pt1} and \cref{ex:5pt2}. Let $P = P_1 = R$. Then \begin{align*} U(P,R) &= \left(1 + \frac{1}{b_1^2-1} + \frac{1}{b_2^2-1} \right) \cdot 1 + \left( - \frac{b_2}{b_2^2-1} \right) \cdot b_2 + \left( - \frac{b_1}{b_1^2-1} \right) \cdot b_1 = (-1)^1, \\ V(P,R) &= \left(1 + \frac{1}{b_1^2-1} + \frac{1}{b_2^2-1} \right) \cdot b_0b_1b_2b_3b_4b_5 + \left( - \frac{b_2}{b_2^2-1} \right) \cdot b_0b_1b_3b_4b_5 + \left( - \frac{b_1}{b_1^2-1} \right) \cdot b_0b_2b_3b_4b_5 = b_E. \end{align*} \end{example} We shall show that $U(P,R) = (-1)^{r-1} \delta_{P,R}$. The equality $V(P,R) = (-1)^r b_E \delta_{P,R}$ follows similarly. We note that $$ \sep(P,G_\bullet) := \sep(P,Q_{G_\bullet}) = (G_s \setminus G_{s-1}) \sqcup (G_{s-2} \setminus G_{s-3}) \sqcup \cdots. $$ \subsubsection{The case $P=R$} Let us first assume that $P = R$. Let $G_\bullet = \{\hat 0 < G_1 < G_2 < \cdots < G_s < \hat 1\}$. Then \begin{equation}\label{eq:sepG} b(G_\bullet) b_{\sep(P,Q_{G_\bullet})} = (-1)^{\sum_i \rk(G_i)} b_{G_s}^2 b_{G_{s-2}}^2 \cdots, \end{equation} where $\sum_i \rk(G_i) := \sum_{i=1}^{s(G_\bullet)} \rk(G_i)$. Thus with $\beta(G_i) = 1/\tb_{G_i}$ and $\beta(G_\bullet) = \prod_{i=1}^s \beta(G_i)$, we have \begin{align}\label{eq:bipG} \begin{split} \ip{G_\bullet}_B b_{\sep(P,Q_{G_\bullet})} &= (-1)^{\sum_i \rk(G_i)} \prod_{i \equiv s} \frac{b_{G_i}^2}{b_{G_i}^2-1} \prod_{j \not \equiv s} \frac{1}{b_{G_j}^2-1} \sum_{E_\bullet \in \overline{G}_\bullet} \prod_{E_k \notin G_\bullet} \frac{1}{\tb_{E_k}} \\ &= (-1)^{\sum_i \rk(G_i)} \prod_{i \equiv s}(1+\beta(G_i)) \prod_{j \not \equiv s} \beta(G_j) \sum_{E_\bullet \in \overline{G}_\bullet} \prod_{ E_k \notin G_\bullet} \beta(E_k). \end{split} \end{align} Here, $i \equiv r$ means equal parity. Expanding the factors $(1+\beta(G_i))$ in \eqref{eq:bipG} and summing over $G_\bullet \in \pFl(P)$, we may write $U=U(P,P)$ as a sum $$ U= \sum_{E_\bullet} u_{E_\bullet} \beta(E_\bullet) $$ for some integer coefficients $u_{E_\bullet}$. \def\co{\kappa} Let us compute the coefficients $u_{E_\bullet}$. We say that $E_\bullet$ is \emph{compatible} with $G_\bullet$ if $\beta(E_\bullet)$ appears in the expansion \eqref{eq:bipG} for $G_\bullet$. Write $\co(E_\bullet) \subseteq \pFl(P)$ for the set of $G_\bullet$ such that $E_\bullet$ is compatible with $G_\bullet$. Then we have $$ u_{E_\bullet} = \sum_{G_\bullet \in \co(E_\bullet)} (-1)^{\sum_i \rk(G_i)}. $$ \begin{lemma}\label{lem:compat} $E_\bullet$ is compatible with $G_\bullet$ if and only if \begin{enumerate} \item Every pair $(E_i,G_j)$ is comparable, that is, $E_i \leq G_j$ or $G_j \leq E_i$. \item If $G_i$ is missing from $E_\bullet$ then $i \equiv s(G_\bullet)$. \end{enumerate} \end{lemma} Now, let us first consider the case $E_\bullet = \emptyflag$. Then by \cref{lem:compat}, $E_\bullet$ is compatible with $G_\bullet$ if and only if $s(G_\bullet) \in \{0,1\}$. By \cref{thm:sphere}, we have $\chi(L(P)\setminus\hat 1) = (-1)^{r-1}$, so we conclude that the coefficient of $u_{\emptyflag}$ is $(-1)^{r-1}$. Now suppose that $E_\bullet \neq \emptyflag$ is compatible with $G_\bullet \in \co(E_\bullet)$. We have that $E_1 \subseteq G_2$. Consider a non-empty subset $\co' \subset \co(E_\bullet)$ where $$ \co' := \{G'_\bullet \in \co(E_\bullet) \mid G'_{s'-i} = G_{s-i} \text{ for all $i$ such that } E_1 \subsetneq G_{s-i}\}. $$ In other words, the higher rank flats in $G'_\bullet$ are the same as those of $G_\bullet$. Then $\co'$ is in bijection with the lower interval $[\hat0,E_1]$. The Euler characteristic of this lower interval is equal to 0 by \cref{thm:sphere} applied to the restriction $\M^{E_1}$. It follows that the contribution of $\co'$ to the coefficient of $\beta(E_\bullet)$ in $ \ip{G_\bullet}_B b_{\sep(P,Q_{G_\bullet})}$ is equal to $0$. We conclude that the coefficient of $u_{E_\bullet}$ for $E_\bullet \neq \emptyflag$ is equal to $0$. This completes the proof for the case $P =R$. \subsubsection{The case $P\neq R$} Now we move on to the case that $P \neq R$. We will proceed by induction on the rank $r$ of $\M$ and number of elements $|E|$ of the ground set. Let $S := \sep(P,R)$. In the following, for the empty flag $G_\bullet = \emptyflag$, we denote $G_s = \emptyset$. Divide $\pFl(P)$ into three subsets: \begin{align*} G^1(P)&:= \{G_\bullet \mid G_s \cap S = \emptyset\} \\ G^2(P)&:= \{G_\bullet \mid \emptyset \neq G_s \cap S \subsetneq G_s\} \\ G^3(P)&:= \{G_\bullet \mid \emptyset \neq G_s \subseteq S\}. \end{align*} For a fixed $F \in L(P)$, write $G^F(P) := \{G_\bullet \in \pFl(P) \mid G_s = F\}$. For $G_\bullet \in \pFl(P)$, define $$\sep_-:= \sep(P,G_\bullet) \cap \sep(P,R), \qquad \text{and} \qquad \sep_+:= \sep(P,R) \setminus \sep(P,G_\bullet).$$ \begin{lemma}\label{lem:G2} The contribution of $G_\bullet \in G^2(P)$ to the summation \eqref{eq:U} is 0. \end{lemma} \begin{proof} Consider $G^F(P)$ for a fixed $F \in L(P)$ such that $\emptyset \neq F \cap S \subsetneq F$. Then $G^F(P)$ is in natural bijection with $G(P^F)$ where $P^F \in \T(\M^F)$ is the tope $P^F=P|_{ F}$ in the restriction of $\M$ to $F$. Furthermore, $\sep(R^F,(G_\bullet)^F) = \sep(R,G_\bullet)\cap F$, and by assumption that $F \not \subset S$, we have $R^F \neq P^F$. By the inductive hypothesis applied to the two topes $P^F, R^F \in \T(\M^F)$ we deduce that the contribution sums to $0$. \end{proof} \begin{lemma}\label{lem:G3} Suppose that $\emptyset \neq F \subseteq S$. Then the contribution of $G^F(P) \subset G^3(P)$ to the summation \eqref{eq:U} is equal to $$ \sum_{G_\bullet \in G^F(P)} \ip{G_\bullet}_B b_{\sep(R,Q_{G_\bullet})} = - b_S \beta(F)\sum_{E_\bullet \in \pFl(\M_F)} \beta(E_\bullet). $$ \end{lemma} \begin{proof} Let $G_\bullet \in G^F(P)$. We have $\sep(P,G_\bullet) \subset F \subset S$. Thus $\sep(R,G_\bullet) = S \setminus \sep(P,G_\bullet)$. We write $$ \frac{\sep_+}{\sep_-} b_{G_s}^2 b_{G_{s-2}}^2 \cdots = \frac{b_S}{b_{\sep(P,G_\bullet)}^2} b_F^2 b_{G_{s-2}}^2 \cdots = b_Sb_{G_{s-1}}^2 b_{G_{s-3}}^2 \cdots, $$ where in the last equality we used \eqref{eq:sepG}. We may thus reduce the calculation of the contribution of $G^F(P)$ to a sum over $G(P^F)$. We deduce that the contribution is equal to $(-1)^{\rk(F)} b_S U(P^F,P^F) \beta(F)\sum_{E_\bullet \in \pFl(\M_F)} \beta(E_\bullet)$, where $U(P^F,P^F)$ is calculated in $\M^F$. Since we showed the previous case that $U(P^F,P^F)= (-1)^{\rk(F)-1}$, our result holds. \end{proof} By \cref{lem:G3}, the contribution of $G^3(P)$ is \begin{equation}\label{eq:G3} \sum_{G_\bullet \in G^3(P)} \ip{G_\bullet}_B b_{\sep(R,Q_{G_\bullet})} = - b_S \sum_{F \subseteq S} \sum_{E_\bullet \mid E_1 = F} \beta(E_\bullet) = -\sum_{E_\bullet \mid E_1 \subseteq S} b_S \beta(E_\bullet). \end{equation} \begin{lemma}\label{lem:G1} The coefficient of $\beta(E_\bullet)$ in the summation $\sum_{G_\bullet \in G^1(P)} \ip{G_\bullet}_B b_{\sep(R,Q_{G_\bullet})}$ vanishes if $E_1 \not \subset S$. \end{lemma} \begin{proof} We have $$ \sum_{G_\bullet \in G^1(P)} \ip{G_\bullet}_B b_{\sep(R,G_\bullet)} = b_S \sum_{G_\bullet \in G^1(P)} \ip{G_\bullet}_B b_{\sep(P,G_\bullet)}. $$ Suppose that $E_1 \not \subset S$. Let $D = E_1 \setminus S$. We can fix the rest of $G_\bullet$ and let $G_1$ vary over $[\hat 0, D]$. (The case $G_1 = \hat 0$ means that we do not include any $G_1$ in the interval $(\hat 0, D]$.). As in the proof of \cref{lem:G2}, these contributions cancel out. \end{proof} Now, fix $E_\bullet$ satisfying $\emptyset \neq E_1 \subseteq S$ and consider the coefficient of $\beta(E_\bullet)$. The contribution from $G^3(P)$ is given by \eqref{eq:G3}, and is simply $-b_S$. For $\emptyflag \neq G_\bullet \in G^1(P)$, we have $G_1 \cap S = \emptyset$ so $G_1 \cap E_1 = \emptyset$, and thus $\ip{G_\bullet}_B b_{\sep(R,G_\bullet)}$ does not contribute to the coefficient of $\beta(E_\bullet)$. However, for the term $\ip{\emptyflag}_B b_{\sep(R,\emptyflag)} = \ip{\emptyflag}_B b_S$ we get a coefficient of $b_S$. The contributions cancel and the coefficient of $\beta(E_\bullet)$ vanishes. Finally, let us consider the coefficient of $\beta(E_\bullet)$ where $E_\bullet = \emptyflag$. There is no contribution from $G^3(P)$. The contribution from $G^1(P)$ is equal to $$ b_S \sum_{G_1 \in L(P) \mid G_1 \cap S = \emptyset} (-1)^{\rk(G_1)}. $$ The subposet $X(S) = \{G_1 \in L(P) \mid G_1 \cap S = \emptyset\} \subset L(P)$ is obtained from the ball $L(P) \setminus \hat 1$ as follows. Let $H \subset E$ denote the facets of $P$. We remove from $L(P) \setminus \hat 1$ the upper order ideal generated by $H \cap S = H \cap \sep(P,R)$. We show that the Euler characteristic of $X(S)$ is $0$ by using standard results on shellings of $L(P)$. We find a shelling order on the facets $H$ of $P$ by considering a shortest path from $P$ to the negative tope $-P$. By picking this path to pass through $R$, we can arrange the facets belonging to $H \cap \sep(P,R)$ to come first. Finally, it is known that the union of any proper initial subset of facets in this shelling order is a contractible subcomplex; see \cite[Proposition 4.3.1 and Lemma 4.7.28]{OMbook}. Since $X(S)$ is the complement of a contractible complex in a contractible complex, it has Euler characteristic $0$. We have shown that $U(P,R) = 0$. This completes the inductive step, and the proof of the theorem. \section{Bergman fan and Laplace transform}\label{sec:Bergman} \subsection{Bergman fan} Let $\R^E$ be the vector space with basis $\epsilon_e$, $e \in E$. For $S \subset E$, let $\epsilon_S \in \R^E$ denote the vector $\epsilon_S:= \sum_{s \in S} \epsilon_s$. Then $\epsilon_E = \one$, the all $1$-s vector. For each partial flag $G_\bullet \in \pFl(M)$, let $C'_{G_\bullet}$ denote the simplicial $s+1$-dimensional cone $$ C'_{G_\bullet}:= \sp_{\R_{\geq 0}}(\epsilon_{G_1}, \epsilon_{G_2}, \ldots, \epsilon_{G_s}, \epsilon_{G_{s+1}}=\one), $$ and let $C_{G_\bullet}$ denote the simplicial $s$-dimensional cone $$ C_{G_\bullet}:= \sp_{\R_{\geq 0}}(\epsilon_{G_1}, \epsilon_{G_2}, \ldots, \epsilon_{G_{s}}). $$ Each cone $C'_{G_\bullet}$ is the direct product of $C_{G_\bullet}$ with $\R_{\geq 0} \cdot \one$. When $G_\bullet = F_\bullet \in \Fl(M)$ is a complete flag, the cone $C'_{G_\bullet}$ is $r$-dimensional, and the cone $C_{F_\bullet}$ is $d = (r-1)$-dimensional. \begin{remark} In the literature, $C_{G_\bullet}$ is usually defined as the image of $C'_{G_\bullet}$ in the quotient space $\R^E/\one$. The result is that the Laplace transform (resp. discrete Laplace transform) of $C_{G_\bullet}$ below would be defined modulo $\sum_e a_e = 0$ (resp. $\prod_e b_e = 1$). \cref{thm:deRhamfan} and \cref{thm:Bettifan} would then give the forms $\bdRip{\cdot,\cdot}$ and $\bip{\cdot,\cdot}_B$ (instead of $\dRip{\cdot,\cdot}$ and $\ip{\cdot,\cdot}_B$). \end{remark} \begin{definition} The \emph{Bergman fan} $\tSigma_M$ (resp. $\Sigma_M$) is the $r$-dimensional (resp. $(r-1)$-dimensional) fan in $\R^E$ given by the union of the cones $C'_{G_\bullet}$ (resp. $C_{G_\bullet}$) for $G_\bullet \in \pFl(M)$. We denote by $|\tSigma_M|$ (resp. $|\Sigma_M|$) the support of the Bergman fan. \end{definition} The maximal cones of $\tSigma_M$ (resp. $\Sigma_M$) are exactly the cones $C'_{F_\bullet}$ (resp. $C_{F_\bullet}$) for $F_\bullet \in \Fl(M)$. There are other, coarser, fan structures on $|\Sigma_M|$ which we discuss in \cref{sec:building}; see \cite{FS}. A simplicial cone $C \subset \R^E$ is \emph{unimodular} if the primitive integer vector generators of the cone can be extended to a $\Z$-basis of $\Z^E$. The following result is easy to see directly. \begin{lemma}\label{lem:unimodular} For any $G_\bullet \in \pFl(M)$, the simplicial cone $C'_{G_\bullet}$ (resp. $C_{G_\bullet}$) is unimodular. \end{lemma} \subsection{Laplace transform} The \emph{Laplace transform} is the integral transform defined as follows. Given a function $f(\x)$ on $\R^n$, the Laplace transform $\L(f) = \L(f)(\y)$ is the function on $\R^n$ $$ \L(f)(\y) = \int_{\R_{>0}^n} f(\x) \exp(-\x \cdot \y) d^n \x, $$ which is defined on the domain $\Gamma \subset \R^n$ of convergence of the integral. Recall that a cone $C$ is \emph{pointed} if it does not contain a line. If $C = \sp_{\R_{\geq 0}}(\v_1,\ldots,\v_m)$ is a pointed cone in $\R^n$, we consider the analogous integral transform of the characteristic function of $C$: \begin{equation}\label{eq:Lapint} \L(C)(\y) = \int_C \exp(-\x \cdot \y) d^n \x. \end{equation} Define the dual cone $$ C^*:= \{\y \in \R^n \mid \x \cdot \y \geq 0 \text{ for all } \x \in \R^n\}. $$ The integral \eqref{eq:Lapint} converges absolutely for $\y \in \Int(C^*)$ in the interior of the dual cone $C^*$ to a rational function. We declare this rational function to be the Laplace transform $\L(C)$ of $C$, ignoring the domain of convergence of the integral. If $C$ has dimension less than $n$, then we instead set $\L(C)(\y) = 0$. \begin{lemma}\label{lem:Lsimplicial} Suppose that $C = \sp_{\R_{\geq 0}}(\v_1,\ldots,\v_n)$ is a $n$-dimensional simplicial cone in $\R^n$. Then the Laplace transform of $C$ is the rational function $$ \L(C)(\y) = \prod_{i=1}^n \frac{|\det(\v_1,\ldots,\v_n)|}{\v_i \cdot \y}, $$ and the integral \eqref{eq:Lapint} converges absolutely for $\y \in \Int(C^*)$. \end{lemma} \begin{proof} By rescaling, we may assume that $\det(\v_1,\ldots,\v_n) = 1$. Let $\{\u_1,\ldots,\u_n\}$ be the dual basis. Then we may write $$ \x = z_1 \v_1 + z_2 \v_2 + \cdots + z_n \v_n, \qquad \y = w_1 \u_1 + \cdots + w_n \u_n $$ and $$ \int_C \exp(-\x \cdot \y) d^n \x = \int_{\R_{>0}^n} \exp(-(z_1w_1+\cdots +z_nw_n)) dz_1 \cdots dz_n = \prod_{i=1}^n \int_0^\infty \exp(-z_iw_i) dz_i = \prod_{i=1}^n \frac{1}{w_n}, $$ where for the last equality we have assumed that $w_i >0$, or equivalently, $\y \in \Int(C^*)$. Finally, we note that $w_i = \v_i \cdot \y$. \end{proof} \begin{lemma} Let $C$ be a $n$-dimensional pointed cone in $\R^n$. Then the integral \eqref{eq:Lapint} converges absolutely for $\y \in \Int(C^*)$ to a rational function. \end{lemma} \begin{proof} Triangulate $C$ with simplicial cones $C_1,C_2,\ldots,C_k$. Since $C_i \subset C$, it follows that $C^* = \bigcap C_i^*$, so by \cref{lem:Lsimplicial}, the integral converges absolutely when $\y \in \Int(C^*)$, and equals the rational function $\L(C) = \sum_{i=1}^k \L(C_i)$. \end{proof} Now, if $C$ is a cone that contains a line then we define $\L(C):=0$. These definitions are consistent in the following sense. \begin{proposition}[{\cite[p.341]{Bar}}]\label{prop:Lval} Suppose that $C_1,\ldots,C_k \subset \R^n$ are polyhedral cones, and we have the identity \begin{equation}\label{eq:indicator} \sum_i \alpha_i [C_i] = 0 \end{equation} of indicator functions $[C_i]$. Then $$ \sum_i \alpha_i \L(C_i) = 0. $$ \end{proposition} In other words, $\L$ is a valuation on the algebra of indicator functions of cones. Note that $\L(C_i) = 0$ if $C_i$ is not full-dimensional, so in \eqref{eq:indicator}, we can replace indicator functions by measures of cones. See also \cite{GLX} for a study of the closely related dual volume rational functions. \subsection{Discrete Laplace transform} The discrete Laplace transform is defined as follows. Given a closed rational polyhedral cone $C$, we define $$ \dL(C) := \sum_{\y \in C \cap \Z^n} \b^{2\y}, $$ where $\b^{2\y} = b_1^{2y_1} b_2^{2y_2} \cdots b_n^{2y_n}$. Note the unusual factor of $2$, which is used to match geometric parameters that will be introduced later. We similarly define $\dL(C^\circ)$ for a (relatively) open rational polyhedral cone $C^\circ$. Then $\dL$ can be viewed as a linear operator on the indicator functions of the cones. In particular, $$ \dL(C^\circ) = \dL(C) + \sum_{F} (-1)^{\codim(F)} \dL(F) $$ where the summation is over all proper faces of $C$, including the origin. For example, for $C$ equal to a cone over a quadrilateral, we would have $$ \dL(C^\circ) = \dL(C) - (\dL(F_1)+\dL(F_2)+\dL(F_3)+\dL(F_4)) +(\dL(F_{12})+\dL(F_{23})+\dL(F_{34})+\dL(F_{14})) - \dL(0), $$ where $F_1,F_2,F_3,F_4$ are the four facets of $C$, and $F_{ij} = F_i \cap F_j$ are the codimension two faces. \begin{lemma}\label{lem:discLap} Suppose that $C$ is a unimodular simplicial cone generated by primitive integer vectors $\v_1,\v_2,\ldots,\v_n$. Then \begin{align*} \dL(C) &= \prod_{i=1}^n \frac{1}{1 - \b^{2\v_i}}, \qquad \dL(C^\circ) = \b^{2(\v_1+\v_2+\cdots+\v_n)} \prod_{i=1}^n \frac{1}{1 - \b^{2\v_i}}. \end{align*} We have the identity $$ \dL(C^\circ) = \sum_{I \subseteq [n]} (-1)^{n-|I|}\prod_{i\in I}^n \frac{1}{1 - \b^{2\v_i}} = (-1)^n \sum_{I \subseteq [n]} \prod_{i\in I}^n \frac{1}{\b^{2\v_i}-1}. $$ \end{lemma} \begin{proof} The set $C \cap \Z^n$ is equal to $\{\alpha_1 \v_1 + \cdots + \alpha_n \v_n \mid \alpha_i \in \Z_{\geq 0}\}$. We have $$ \dL(C) = \sum_{\alpha \in \Z_{\geq 0}^n} \b^{2 (\alpha_1 \v_1 + \cdots + \alpha_n \v_n)} = \prod_{i=1}^n (1+ \b^{2\v_i} + \b^{4\v_i} + \cdots) = \prod_{i=1}^n \frac{1}{1 - \b^{2\v_i}}. $$ The map $\y \mapsto \y + \v_1+ \v_2 + \cdots + \v_n$ sends $C \cap \Z^n$ bijectively to $C^\circ \cap \Z^n$, giving the second equation. \end{proof} For an arbitrary rational polyhedral cone $C$, we may define $\dL(C)$ by writing $[C]$ in terms of indicator functions of unimodular simplicial cones. Again, we have $\dL(C) = 0$ if $C$ contains a line, but unlike the case of $\L(C)$, the rational function $\dL(C)$ does not vanish when $C$ is not full-dimensional. We have the following analogue of \cref{prop:Lval}; see \cite[Theorem 3.3]{Bar}. \begin{proposition}\label{prop:dLval} Suppose that $C_1,\ldots,C_k \subset \R^n$ are rational polyhedral cones, and we have the identity \eqref{eq:indicator} of indicator functions. Then $$ \sum_i \alpha_i \dL(C_i) = 0. $$ \end{proposition} In other words, $\dL$ is a valuation on the algebra of indicator functions of cones. We will only need the following result for unimodular simplicial cones, but we state it in the natural generality. \begin{proposition} The Laplace transform and discrete Laplace transform are related by $$ \L(C) = (-1)^n \lim_{\alpha \to 0} \alpha^n \dL(C)|_{b_i \mapsto 1 + \alpha a_i/2}, $$ where $C$ is a rational polyhedral cone in $n$-dimensions. \end{proposition} \begin{proof} Suppose first that $C$ has dimension less than $n$. Then $\L(C)=0$ and it is not hard to see that the limit vanishes as well. Next suppose that $C$ is a unimodular simplicial full-dimensional cone. Then substituting $1- \b^{2\v}|_{b_i \mapsto 1 + \alpha a_i/2} = \alpha(\v \cdot \a) + O(\alpha^2)$ into \cref{lem:Lap} and \cref{lem:discLap}, we obtain the stated formula. Now let $C$ be an arbitrary full-dimensional rational polyhedral cone. We may write (the indicator function of) $C$ as an alternating sum of unimodular simplicial cones; see \cite{BaPo}. The stated formula then follows from the result for unimodular simplicial cones and \cref{prop:Lval,prop:dLval}. \end{proof} \subsection{Laplace transforms of the Bergman fan} In the following, we shall utilize the $k$-dimensional Laplace transform of $k$-dimensional subfans of the Bergman fan $\tSigma_M$ or $\Sigma_M$. The lattice $\Z^E \subset \R^E$ defines a measure in $\R^E$ by declaring that the unit cube of the lattice $\Z^E \subset \R^E$ has volume $1$. A $k$-dimensional subspace $V \subset \R^E$ is \emph{unimodular} if the abelian group $V \cap \Z^E$ has a $\Z$-basis that can be extended to a $\Z$-basis of $\Z^E$. We define a measure in $V$ by declaring that the unit cube in $V \cap \Z^E$ has volume $1$. Let $C$ be a $k$-dimensional cone spanning a unimodular subspace $V \subset \R^E$. We define the Laplace transform $\L(C) = \L^k(C)$ by using this measure. It follows from \cref{lem:unimodular} that the subspace spanned by any cone $C \subset \Sigma_M$ (resp. $C' \subset \tSigma_M$) of the Bergman fan is a unimodular subspace. \begin{lemma}\label{lem:Lap} Let $F_\bullet = \{F_0 \subset F_{1} \subset F_{2} \subset \cdots \subset F_{r-1} \subset F_r = E\}$ be a complete flag. Let $C'_{F_\bullet}$ (resp. $C_{F_\bullet}$) be the corresponding cone of the Bergman fan. Then the Laplace transform of $C'_{F_\bullet}$ (resp. $C_{F_\bullet}$) is equal to $$ \L(C') = \prod_{i=1}^r \frac{1}{a_{F_i}} = \frac{1}{a'_{F_\bullet}}, \qquad \L(C) = \prod_{i={1}}^{r-1} \frac{1}{a_{F_i}} = \frac{1}{a_{F_\bullet}}. $$ \end{lemma} \begin{lemma}\label{lem:dLap} Let $E_\bullet = \{\emptyset = E_0 \subset E_{1} \subset E_{2} \subset \cdots \subset E_{s} \subset E_{s+1} = E\}$ be a partial flag. Let $C'_{E_\bullet}$ (resp. $C_{E_\bullet}$) be the corresponding cone of the Bergman fan. Then the discrete Laplace transform of $C'_{E_\bullet}$ (resp. $C_{E_\bullet}$) is equal to $$ \dL(C') = (-1)^{s+1} \prod_{i=1}^{s+1} \frac{1}{\tb_{E_i}}, \qquad \dL(C) = (-1)^s \prod_{i=1}^s \frac{1}{\tb_{E_i}}= (-1)^s \frac{1}{\tb_{E_\bullet}}. $$ \end{lemma} \subsection{Local Bergman fan}\label{sec:localBF} Henceforth, we only work with the $(r-1)$-dimensional Bergman fan $\Sigma_M$. For a basis $B \in \B(M)$, the \emph{local Bergman fan} $\Sigma_M(B) \subset \Sigma_M$ is the subfan given by the union of cones $C_{F_\bullet}$ where $F_\bullet$ is generated by $B$. The support $|\Sigma_M(B)|$ is equal to the intersection of $|\Sigma_M|$ with the normal cone $C(e_B)$ to the vertex $e_B$ in the normal fan of the \emph{matroid polytope} $P_M$ \cite[Section 4]{FS}. For $B,B' \in \B(M)$, define $$ \Sigma_M(B,B') := \Sigma_M(B) \cap \Sigma_M(B') \subset \Sigma_M $$ to be the intersection of the two subfans $\Sigma_M(B)$ and $\Sigma_M(B')$. \begin{proposition} The support $|\Sigma_M(B,B')|$ of the fan $\Sigma_M(B,B')$ is the intersection of $|\Sigma_M|$ with the normal cone $C(G)$ where $G$ is the smallest face of $P_M$ containing $e_B$ and $e_{B'}$. \end{proposition} \begin{proof} We have $$ |\Sigma_M(B,B')| = |\Sigma_M(B)| \cap |\Sigma_M(B')| = (|\Sigma_M| \cap C(e_B)) \cap (|\Sigma_M| \cap C(e_{B'})) = |\Sigma_M| \cap C(e_B) \cap C(e_{B'}).$$ The two cones $C(e_B)$ and $C(e_{B'})$ are (maximal) cones in the normal fan $\N(P_M)$ of the matroid polytope $P_M$ of $M$. The intersection $C(e_B) \cap C(e_{B'})$ is again a cone. It is equal to $C(G)$, where $G$ is the smallest face of $P_M$ containing both vertices $e_B$ and $e_{B'}$. \end{proof} The fan $\Sigma_M(B,B')$ is a subfan of the pure $(r-1)$-dimensional fan $\Sigma_M$. The $(r-1)$-dimensional part of $\Sigma_M(B,B')$ is the union of the cones $C_{F_\bullet}$ where $F_\bullet \in \Fl(M)$ is generated by both $B$ and $B'$. Note that $|\Sigma_M(B,B')|$ is always non-empty since it contains the origin. However, it may have dimension less than $r-1$. Recall the definition of the permutation $\sigma(B,F_\bullet)$ from \eqref{eq:sigma}. \begin{proposition}[{\cite[Proposition 4.5]{BV}}] \label{prop:BV} Suppose that $F_\bullet \in \Fl(M)$ is generated by both $B$ and $B'$. Then the permutation $\sigma(B,F_\bullet)\sigma(B',F_\bullet)^{-1}$ depends only on $B,B'$. \end{proposition} We define $$ (-1)^{B,B'} := (-1)^{\sigma(B,F_\bullet)} (-1)^{\sigma(B',F_\bullet)} \in \{+1,-1\}. $$ By \cref{prop:BV}, $(-1)^{B,B'}$ does not depend on the choice of $F_\bullet$, as long as it is generated by both $B$ and $B'$. The following result gives a tropical interpretation of the symmetric bilinear form $\dRip{\cdot, \cdot}$. \begin{theorem}\label{thm:localBF} We have $\dRip{e_B,e_{B'}} = (-1)^{B,B'} \L(|\Sigma_M(B,B')|) = (-1)^{B,B'} \sum_{F_\bullet} \frac{1}{a_{F_\bullet}}$, where the summation is over flags $F_\bullet$ generated by both $B$ and $B'$. \end{theorem} \begin{proof} If no flags $F_\bullet$ are generated by both $B$ and $B'$, then both sides are $0$. Suppose otherwise. The Laplace transform of a union of $(r-1)$-dimensional cones, intersecting only in lower-dimensional cones, is equal to the sum of the Laplace transform of the corresponding $(r-1)$-dimensional cones. We thus have $\L(|\Sigma_M(B,B')|) = \sum_{F_\bullet} \frac{1}{a_{F_\bullet}}$, summed over $F_\bullet \in \Fl(M)$ generated by both $B$ and $B'$. Comparing with \cref{prop:dRind}, we see that the result follows from the equalites $$ r(B,F_\bullet) r(B',F_\bullet) = (-1)^{\sigma(B,F_\bullet)} (-1)^{\sigma(B',F_\bullet)} = (-1)^{B,B'}, $$ for any $F_\bullet$ generated by both $B$ and $B'$. \end{proof} \subsection{Positive Bergman fan}\label{sec:posBF} In \cref{sec:localBF}, we only considered the matroid $M$. We now work with an oriented matroid $\M$ lifting $M$, and let $P \in \T(\M)$ be a tope. Recall that we have defined the Las Vergnas face lattice $L(P) \subset L(M)$ of the tope $P$, and $\Fl(P)$ denotes the set of complete flags of lattices belonging to $L(P)$. The \emph{positive Bergman fan} \cite{AKW} $\Sigma_M(P)$ of the tope $P$ is the subfan of $\Sigma_M$ obtained by taking the union of all cones $C_{F_\bullet}$ for $F_\bullet \in \Fl(P)$, together with all the faces of these cones. Recall that for a tope $P$, we denote by $\pFl(P)$ the set of all partial flags of flats belonging to $L(P)$. \begin{definition} Let $G_\bullet \in \pFl(P)$. Define $$ \Sigma_M(P,G_\bullet):= \bigcup_{F_\bullet \in \bG_\bullet \cap \Fl(M)} C_{F_\bullet}, $$ to be the union of maximal cones (together with all subcones) in the Bergman fan that are indexed by complete flags in the closure of $G_\bullet$. \end{definition} See \cref{fig:posBerg} for an example. The cones in $\Sigma_M(P,G_\bullet)$ are exactly the cones of the Bergman fan that have the cone $C_{G_\bullet}$ as a face. In other words, $\Sigma_M(P,G_\bullet)$ is the star of $C_{G_\bullet}$ inside $\Sigma_M(P)$. Let $P, Q \in \T$. Define $\Sigma_M(P,Q)$ to be the subfan of $\Sigma_M$ that consists of all maximal cones $C_{F_\bullet}$ that belong to both $\Sigma_M(P)$ and $\Sigma_M(Q)$, and all faces of these cones. \begin{proposition} The subfan $\Sigma_M(P,Q) \subset \Sigma_M$ is given by $$ \Sigma_M(P,Q) = \bigsqcup_{G_\bullet \in G^{\pm}(P,Q)} \Sigma_M(P,G_\bullet) = \bigsqcup_{G_\bullet \in G^{\pm}(P,Q)} \Sigma_M(Q,G_\bullet). $$ \end{proposition} \begin{proof} Follows from \cref{prop:noover}(3),(4). \end{proof} \begin{theorem}\label{thm:deRhamfan} Let $P,Q \in \T$ be topes. Then \begin{equation}\label{eq:deRhamfan} \dRip{\Omega_P,\Omega_Q}= \sum_{G_\bullet \in G^{\pm}(P,Q)} (\pm)^r (-1)^{\sum_{i=1}^s \rk(G_i)} \L(\Sigma_M(P,G_\bullet)), \end{equation} where the sign $(\pm)^r$ is as in \cref{thm:dRtope}. In particular, we have $$\dRip{P,P} = \L(\Sigma_M(P)).$$ \end{theorem} \begin{proof} Apply \cref{lem:Lap} to the right hand side \eqref{eq:deRhamfan}, and compare with \cref{thm:dRtope}. \end{proof} \begin{theorem}\label{thm:Bettifan} Let $P,Q \in \T$ be topes. For any $G_\bullet \in \pFl(P)$, we have \begin{equation}\label{eq:Bettifan1} \halfip{G_\bullet}_B = (-1)^d b(G_\bullet) \dL(\Sigma_M(P,G_\bullet)), \end{equation} and thus \begin{equation}\label{eq:Bettifan2} \halfip{P,Q}_B= (-1)^d \sum_{G_\bullet \in G^{\pm}(P,Q)} (\pm)^r b(G_\bullet) \dL(\Sigma_M(P,G_\bullet)). \end{equation} In particular, we have $$\halfip{P,P}_B = (-1)^d \dL(\Sigma_M(P)).$$ \end{theorem} \begin{proof} Since by definition $\halfip{P,Q}_B = \sum_{G_\bullet \in G^{\pm}(P,Q)} (\pm)^r \halfip{G_\bullet}$, we have that \eqref{eq:Bettifan1} implies \eqref{eq:Bettifan2}. For \eqref{eq:Bettifan1}, we need to show that $$ (-1)^d \sum_{\y \in \Sigma_M(P,G_\bullet) \cap \Z^{E}/\Z} \b^{2\y}= \sum_{E_\bullet \in \bG_\bullet} \frac{1}{\tb_{E_\bullet}}. $$ By \cref{lem:dLap}, we have \begin{equation}\label{eq:yE} \frac{1}{\tb_{E_\bullet}} = (-1)^s \dL(C_{E_\bullet}) = (-1)^s\sum_{ \y \in C_{E_\bullet} \cap \Z^E} \b^{2\y}. \end{equation} Let $\y$ be a point in the relatively open cone $C^\circ_{E_\bullet}$. Then $\b^{2\y}$ appears in the summation of \eqref{eq:yE} for every cone $C_{E'_{\bullet}} \supseteq C_{E_\bullet}$, or equivalently, for every partial flag $E'_\bullet$ that refines $E_\bullet$. Viewing $\pFl(P)$ as a poset, the coefficient of $\b^{2\y}$ in $\sum_{E_\bullet \in \bG_\bullet} \frac{1}{\tb_{E_\bullet}}$ is up to sign equal to the Euler characteristic of the upper order ideal $\{E'_\bullet \geq E_\bullet\} \subset \pFl(P)$. By \cref{thm:sphere}, $L(P)$ is the face lattice of a regular cell decomposition of a $d$-dimensional ball. It follows that the upper order ideal of $E_\bullet$ in $\pFl(P) \cup \hat 1$ is also the face lattice of a regular cell decomposition of a ball. After removing $\hat 1$, we see that the Euler characteristic is equal to $\pm 1$, depending on dimension. We deduce that the coefficient of $\b^{2\y}$ in $\sum_{E_\bullet \in \bG_\bullet} \frac{1}{\tb_{E_\bullet}}$ is equal to $(-1)^d$. \end{proof} \subsection{Building sets and nested triangulation}\label{sec:building} \def\bT{{\bar T}} \def\dec{{\rm dec}} We assume that $M$ is simple in this section. A subset $\build \subseteq L(M) \setminus \hat0$ is a \emph{building set} if $\hat 1 \in \build$ and for any $F \in L(M)-\hat 0$ and $\max \build_{\leq F}= \{F_1,\ldots,F_k\}$, there is an isomorphism of posets \begin{equation}\label{eq:facF} \prod_{i=1}^k [\0, F_i] \longrightarrow [ \0, F], \qquad (x_1,\ldots,x_k) \mapsto x_1 \vee \cdots \vee x_k, \end{equation} obtained by taking joins. This isomorphism holds if and only if we have a disjoint union of flats $F = \bigsqcup_i F_i$. For $F \in L - \hat 0$, we define $\dec(F) = \{F_1,F_2,\ldots, F_k\}$ via \eqref{eq:facF}. There is a maximal choice of building set, which is $ \build_{\max} = L \setminus \hat 0$. There is also the minimal building set \begin{equation}\label{eq:Bmin} \build_{\min} := \{F \in L \setminus \{\hat 0,\hat 1\} \mid F \mbox{ is connected}\} \cup \{\hat 1\}. \end{equation} A subset $T_\bullet$ of a building set $\build \setminus \{\hat 1\}$ is called \emph{nested} if for any $k \geq 2$ incomparable elements $T_1,\ldots,T_k$ of $T_\bullet$, the join $T_1 \vee T_2 \vee \cdots \vee T_k$ does not belong to $\build$. The $\build$-nested collections form a simplicial complex $N(\build)$, called the \emph{nested set complex}. The subset of maximal (under inclusion) nested sets are denoted $N_{\max}(\build)$. Every maximal nested collection $S_\bullet \in N_{\max}(\build)$ contains exactly $d=r-1$ elements. For a nested collection $T_\bullet$, let $|T_\bullet| = \bigcup_i T_i$ be the union of all elements in $T_\bullet$. If $T_\bullet$ has a unique maximum then the maximum is equal to $|T_\bullet|$, otherwise $|T_\bullet|$ is a flat that does not belong to $\build$. Define the closure $\bT_\bullet \subset N(\build)$ to be the set of all nested collections that contain $T_\bullet$. In the case that $\build = \build_{\max}$, a subset $T_\bullet \subset \build_{\max}$ is nested if and only if it is a flag of flats. We have $N(\build_{\max}) = \pFl(M)$. Now let $\build$ be an arbitrary building set. For a nested set $T_\bullet = \{T_1,\ldots, T_s\} \in N(\build)$, define the cone $$ C_{T_\bullet} := \sp(\epsilon_{T_1},\ldots,\epsilon_{T_s}). $$ The cones $C_{S_\bullet}$ where $S_\bullet \in N_{\max}(\build)$ form a triangulation of $\Sigma_M$; see \cite{FS}. \begin{lemma}\label{lem:SF} Let $S_\bullet \in N_{\max}(\build)$ and $F_\bullet \in \Fl(M)$. Then the cone $C_{S_\bullet}$ contains $C_{F_\bullet}$ if and only if every $F_i, i=1,2,\ldots,d$ is a disjoint union of some of the $S_j \in S_\bullet$. \end{lemma} \begin{proof} The ``if" part is clear. For the ``only if" part, suppose that $C_{F_\bullet} \subseteq C_{S_\bullet}$. Since $C_{S_\bullet}$ is simplicial, each $\epsilon_{F_i}$ has a unique expression as a linear combination of the generators of $C_{S_\bullet}$, and this expression must be a nonnegative linear combination. It follows from this that $\epsilon_{F_i}$ is the sum of $\epsilon_{S_j}$ over all $S_j \in S_\bullet$ such that $S_j \subseteq F_i$ and $S_j$ is maximal under inclusion with this property. In particular, we deduce that $F_i$ is the disjoint union of these $S_j$. \end{proof} The cones $F_\bullet$ described by \cref{lem:SF} are in bijection with total orderings of $S_\bullet$ (compatible with the usual ordering by inclusion). We now repeat the constructions from \cref{sec:pFl} in the nested setting. Let $P \in \T$. Define $N(P,\build) \subset N(\build)$ to be the nested collections where every set belongs to $L(P) \cap \build$. For $T_\bullet \in N(P,\build)$, define $P_{\flip T_\bullet} \in \T$ by (see \cref{prop:flipflag}) $$ P_{\flip T_\bullet}(e) = (-1)^{\#\{i \mid e \in T_i\}}P(e). $$ For $P,Q \in \T$, we define $T(P,Q) \subset N(P,\build)$ by $$ T(P,Q) := \{T_\bullet \in N(P,\build) \mid P_{\flip T_\bullet} \in \{Q\} \} = \{T_\bullet \in N(Q,\build) \mid Q_{\flip T_\bullet} \in \{P\}\}, $$ and $T^{\pm}(P,Q) := T(P,Q) \cup T(P,-Q)$. For $T_\bullet \in N(\build)$, define $$b(T_\bullet):= (-1)^{\sum_i \rk(T_i)} \prod_{T_i \in T_\bullet} b_{T_i}.$$ Let $G_\bullet \in \pFl(M)$. The \emph{decomposition} $\dec(G_\bullet)$ of $G_\bullet$ (with respect to $\build$) is $$ \dec(G_\bullet) := \{F \in \build \mid F \text{ appears in } \dec(G_1),\dec(G_2),\ldots,\dec(G_s) \text{ an odd number of times}\}. $$ \begin{lemma}\label{lem:GT} Let $G_\bullet \in \pFl(M)$ and $T_\bullet = \dec(G_\bullet)$. \begin{enumerate} \item We have $\dec(G_\bullet) \in N(\build)$. \item If $G_\bullet \in \pFl(P)$, then $T_\bullet \in N(P,\build)$. \item For $G_\bullet \in \pFl(P)$, we have $P_{\flip G_\bullet} = P_{\flip T_\bullet}$. \item We have $(-1)^{\sum_i \rk(T_i)} = (-1)^{\sum_i \rk(G_i)}$ and $b(G_\bullet) = b(T_\bullet) \b^{2\y}$ for some $\y \in \Z^E_{\geq 0} \setminus {\bf 0}$. \end{enumerate} \end{lemma} \begin{proof} For (1), suppose that $F \subseteq F'$. Then the factorization \eqref{eq:facF} for $[\hat 0, F']$ induces one for $[\hat 0, F]$. Thus for each $E_i \in \dec(F)$ we have $E_i \subseteq E'_j$ for some $E'_j \in \dec(F')$, and $E \cap E'_{\ell} = \emptyset$ for $\ell \neq j$. Thus the union $\bigcup_i \dec(G_i)$ is a nested collection, and (1) follows. For (2), suppose that $F \in L(P)$. Then the decomposition \eqref{eq:facF} induces a decomposition $L(P^F) = L(P^{E_1}) \times \cdots \times L(P^{E_k})$. In particular, $E_i \in L(P)$, from which (2) follows. (3) and (4) follow from the definitions. \end{proof} Note, however, that the decomposition map $G_\bullet \mapsto \dec(G_\bullet)$ is not injective. \begin{definition} Let $T_\bullet \in N(P,\build)$. Define $$ \Sigma_M(P,T_\bullet):= \bigcup_{S_\bullet \in \bT_\bullet \cap N_{\max}(P,\build)} C_{S_\bullet}, $$ to be the union of maximal cones (together with all subcones) in the nested Bergman fan that are indexed by nested collections in the closure of $T_\bullet$. \end{definition} \begin{lemma}\label{lem:ETcompat} Let $E_\bullet \in \pFl(P)$ and $T_\bullet \in N(P,\build)$. Suppose that $C_{E_\bullet} \subset \Sigma_M(P,T_\bullet)$. Then for all $E_i \in E_\bullet$ and a collection $\{T_{j_1}, \ldots, T_{j_k}\} \subset T_\bullet$ that are pairwise disjoint and not comparable with $E_i$, we have $E_i \cap \bigsqcup_{a} T_{j_a} = \emptyset$ and $E_i \vee \bigsqcup_{a} T_{j_a} \notin \build$. In particular, $E_i \vee \bigsqcup_{a} T_{j_a} = E_i \cup \bigsqcup_{a} T_{j_a} $. \end{lemma} \begin{proof} Suppose that $C_{E_\bullet} \subset C_{S_\bullet}$ where $S_\bullet \in \bT_\bullet \cap N_{\max}(P,\build)$. Then $C_{E_\bullet}$ is a face of some $C_{F_\bullet} \subset C_{S_\bullet}$. Every $F_i \in F_\bullet$ is a disjoint union of some subset of $S_\bullet$. Thus any $F_i \in F_\bullet$ and $S_j \in S_\bullet$ are nested. In particular, any $E_i \in E_\bullet$ and $T_j \in T_\bullet$ are nested. This implies the stated result. \end{proof} \begin{lemma}\label{lem:Gdivide} For each $E_\bullet \in \pFl(P)$ such that $C_{E_\bullet} \subset \Sigma_M(P,T_\bullet)$, there is a distinguished $G_\bullet$ satisfying $\dec(G_\bullet) = T_\bullet$ such that $C_{E_\bullet} \subset \Sigma_M(P, G_\bullet)$. This $G_\bullet$ satisfies $b(G_\bullet)/b(T_\bullet) = \b^{2\y}$ where $\y$ is a multiplicity-free sum of the cone generators $\epsilon_{E_i}$ and is minimal in the following sense. For any other $G'_\bullet$ satisfying $\dec(G'_\bullet) = T_\bullet$ such that $C_{E_\bullet} \subset \Sigma_M(P, G'_\bullet)$, we have $b(G'_\bullet) = b(G_\bullet) \b^{2\y}$, where $\y \in \Z_{\geq 0}^E/{\bf 0}$ does not lie in the cone $C_{E_\bullet}$. \end{lemma} \begin{proof} We have $C_{E_\bullet} \subset \Sigma_M(P, G_\bullet)$ if and only if any pair $(E_i \in E_\bullet, G_j \in G_\bullet)$ is comparable. We construct the minimal $G_\bullet$ algorithmically. Let $E_s$ be the largest proper flat in $E_\bullet$. Let $s' = s(G_\bullet)$ so that $G_{s'}$ is the largest proper flat in $G_\bullet$. \medskip \noindent \emph{Case 1.} Suppose that $|T_\bullet| \not \subseteq E_s$ and $E_s \cap T_\bullet = \emptyset$. Then for \cref{lem:GT}(4) to hold, $G_\bullet$ must contain a flat that contains $E_s \vee |T_\bullet|$. The minimal choice is $G_{s'}= E_s \vee |T_\bullet| = E_s \cup |T_\bullet|$, using \cref{lem:ETcompat}. We also pick $G_{s' -1} = E_s$. Alter $T_\bullet$ by removing the maximal (under inclusion) elements of $T_\bullet$, and we remove $E_s$ from $E_\bullet$. \noindent \emph{Case 2.} Suppose that $|T_\bullet| \not \subseteq E_s$ and $E_s \cap T_\bullet \neq \emptyset$. Then as in Case 1, we set $G_{s'}= E_s \vee |T_\bullet| = E_s \cup |T_\bullet|$. Alter $T_\bullet$ by removing the maximal (under inclusions) elements of $T_\bullet$ and adding $\dec(E_s \setminus |T_\bullet|)$ to the resulting nested collection, and we remove $E_s$ from $E_\bullet$. \noindent \emph{Case 3.} Suppose that $|T_\bullet| = E_s$. Then we take $G_{s'} = E_s$. Alter $T_\bullet$ by removing the maximal elements, and we remove $E_s$ from $E_\bullet$. \noindent \emph{Case 4.} Suppose that $|T_\bullet| \subsetneq E_s$. Then we remove $E_s$ from $E_\bullet$ but leave $T_\bullet$ unchanged. \medskip In all four cases, we continue by running the algorithm for the new $T_\bullet$ and $E_\bullet$, working inside the matroid $M^{E_s}$. This produces the desired $G_\bullet$. One step of the algorithm produces all the flats in $G_\bullet$ that contain $E_s$ (and then $E_s$ is removed). Each step contributes one or no factors of $b^2_{E_s}$ to the ratio $b(G_\bullet)/b(T_\bullet)$. This proves the statement about $b(G_\bullet)/b(T_\bullet)$. We observe that any other choice of $G'_\bullet$ for a particular step would involve strictly more subspaces that contain $E_s$, and involve additional elements that are not contained in $E_s$. The claimed property $b(G'_\bullet) = b(G_\bullet) \b^{2\y}$ follows. \end{proof} \begin{theorem}\label{thm:deRhamfannested} Let $P,Q \in \T$ be topes. Then $$ \dRip{\Omega_P,\Omega_Q}= \sum_{T_\bullet \in T^{\pm}(P,Q)} (\pm)^r (-1)^{\sum_{i=1}^s \rk(T_i)} \L(\Sigma_M(P,T_\bullet)), $$ where the sign $(\pm)^r$ is equal to $1$ or $(-1)^r$ depending on whether $T_\bullet$ belongs to $T(P,Q)$ or $T(P,-Q)$. \end{theorem} \begin{proof} By \cref{prop:noover} and \cref{lem:Gdivide} applied to maximal cones, we have $$ \bigcup_{G_\bullet \mid \dec(G_\bullet) = T_\bullet} \Sigma_M(P,G_\bullet) = \Sigma_M(P,T_\bullet) $$ and the overlaps of the union are of dimension lower than $r-1$. It follows that $$\L(\Sigma_M(P,T_\bullet)) = \sum_{G_\bullet \mid \dec(G_\bullet)=T_\bullet} \L(\Sigma_M(P,G_\bullet)).$$ Finally, we use \cref{lem:GT}(4) and substitute into \cref{thm:deRhamfan}. \end{proof} By definition, $\dRip{\cdot,\cdot}$ takes values in $R[a_F^{-1} \mid F \in L(M)]$. From \cref{thm:deRhamfannested}, we have the following improvement. \begin{corollary}\label{cor:denom} The bilinear form $\dRip{\cdot,\cdot}$ on $\OS(M)$ takes values in $R[a^{-1}_F \mid F \in L \setminus \{\hat 0, \hat 1\} \text{ is connected }]$. \end{corollary} \begin{theorem}\label{thm:Bettifannested} Let $P,Q \in \T$ be topes. Then \begin{equation}\label{eq:Tlattice} \halfip{P,Q}_B= (-1)^d \sum_{T_\bullet \in T^{\pm}(P,Q)}(\pm)^r b(T_\bullet) \dL(\Sigma_M(P,T_\bullet)). \end{equation} \end{theorem} \begin{proof} We claim that \begin{equation}\label{eq:latticepoint} \dL(\Sigma_M(P,T_\bullet)) = \sum_{G_\bullet \mid \dec(G_\bullet)=T_\bullet} \frac{b(G_\bullet)}{b(T_\bullet)} \dL(\Sigma_M(P,G_\bullet)). \end{equation} We view $ \frac{b(G_\bullet)}{b(T_\bullet)} \dL(\Sigma_M(P,G_\bullet))$ as the generating function of the lattice points in $|\Sigma_M(P,G_\bullet)|$ translated by the vector $\y$, where $ \frac{b(G_\bullet)}{b(T_\bullet)} = \b^{2\y}$; see \cref{lem:GT}(4). Denote the translation $|\Sigma_M(P,G_\bullet)| + \y$ by $|\Sigma_M(P,G_\bullet)|'$. Note that the vector $\y$ lies in $C_{G_\bullet}$, so $|\Sigma_M(P,G_\bullet)|' \subset |\Sigma_M(P,G_\bullet)|$. Consider $E_\bullet \in \pFl(P)$ such that $C_{E_\bullet} \subset \Sigma_M(P,T_\bullet)$. The translation vector for the minimal $G_\bullet$ from \cref{lem:Gdivide} sends the cone $C_{E_\bullet}$ to itself, surjective on lattice points in the interior. For any other $G'_\bullet$ from \cref{lem:Gdivide}, the translation vector maps the cone $C_{E_\bullet}$ out of itself. It follows that the lattice points in the interior of $C_{E_\bullet}$ appear in $|\Sigma_M(P,G_\bullet)|'$ and not in any other $|\Sigma_M(P,G'_\bullet)|'$. We have proved the equality \eqref{eq:latticepoint}. Substituting \eqref{eq:latticepoint} into \eqref{eq:Tlattice}, we obtain \cref{thm:Bettifan}, proving the theorem. \end{proof} By definition, $\halfip{\cdot,\cdot}_B$ takes values in $S[\tb_F^{-1} \mid F \in L(M)]$, where $S = \Z[\b]$. From \cref{thm:Bettifannested}, we have the following improvement. \begin{corollary}The bilinear form $\halfip{\cdot,\cdot}_B$ on $\OS(M)$ takes values in $S[\tb^{-1}_F \mid F \in L \setminus \{\hat 0, \hat 1\} \text{ is connected}\;]$. \end{corollary} \begin{example} Let $M$ be the boolean matroid on $E = \{1,2,3\}$ and $P$ be the positive tope. Let $\build = \{ \{1\},\{2\},\{3\}\}$. Let $T_\bullet = \{\{1\}\}$. Let $$ G_\bullet = \{ \hat 0 \subset \{1\} \subset \hat 1\}, \qquad G'_\bullet = \{\hat 0 \subset \{2\} \subset \{1,2\} \subset \hat 1\}. $$ Then we have $\dec(G_\bullet) = \dec(G'_\bullet) = T_\bullet$, and $b(G_\bullet) = b(T_\bullet)$, and $b(G'_\bullet) = b_2^2 b(T_\bullet)$. Now consider \begin{align*} E^{(1)}_\bullet &= \{ \hat 0 \subset \{1\} \subset \hat 1\}, \qquad E^{(2)}_\bullet = \{\hat 0 \subset \{1\} \subset \{1,2\} \subset \hat 1\}, \qquad E^{(3)}_\bullet = \{\hat 0 \subset \{2\} \subset \{1,2\} \subset \hat 1\}, \\\qquad E^{(4)}_\bullet &= \{\hat 0 \subset \{2\}\subset \hat 1\}, \qquad E^{(5)}_\bullet = \{\hat 0 \subset \{1,2\} \subset \hat 1\}, \qquad E^{(6)}_\bullet = \{\hat 0 \subset \hat 1\}. \end{align*} We have $C_{E^{(i)}_\bullet} \subset C_{T_\bullet}$ for each $i = 1,2,3,4,5$. Let $G^{(i)}_\bullet \in \pFl(P)$ be the partial flag of \cref{lem:Gdivide} applied to $E^{(i)}_\bullet$. We have $$ G^{(1)}_\bullet = G^{(2)}_\bullet = G^{(5)}_\bullet = G^{(6)}_\bullet = G_\bullet, \qquad G^{(3)}_\bullet = G^{(4)}_\bullet = G'_\bullet $$ and we may verify that $\epsilon_2 \notin C_{E^{(i)}_\bullet}$ for $i=1,2,5,6$. The proof of \cref{thm:Bettifannested}, restricted to lattice points in $\sp(\epsilon_1,\epsilon_2)$, is the statement that $$ \{(x,y) \in \Z_{\geq 0}^2\} = (\sp(\epsilon_1,\epsilon_1+\epsilon_2) \cap \Z_{\geq 0}^2) \sqcup (\sp(\epsilon_2,\epsilon_1+\epsilon_2) \cap \Z_{\geq 0}^2 + \epsilon_2), $$ where the first term are lattice points in $|\Sigma_M(P,G_\bullet)|$, while the second term are the lattice points in the translation $|\Sigma_M(P,G'_\bullet)| + \epsilon_2$, in both cases restricting to lattice points in $\Z_{\geq 0}^2$. \end{example} \subsection{Nested deRham cohomology intersection form} We give one more description of $\dRip{\cdot,\cdot}$ using nested combinatorics. Let $\build$ be a building set and $N(\build)$ denote the nested set complex and $N_{\max}(\build)$ denote the maximal nested collections. Let $S_\bullet = (S_1,\ldots,S_{r-1}) \in N_{\max}(\build)$ be a maximal nested set. For a basis $B \in \B(M)$, we say that $S_\bullet$ is generated by $B$ if $S_i \in L(B)$ for $i =1,2,\ldots, r-1$. Pick an ordering of $S_\bullet$ and $B$. Then there exists a permutation $\sigma$ such that $$ b_{\sigma(i)} \in S_i \setminus \bigcup_{j \mid S_j \subsetneq S_i} S_j, \qquad \mbox{for $i =1,2,\ldots,r-1$.} $$ In other words, $b_{\sigma(i)}$ belongs to $S_i$ but not to any smaller set in the nested collection. Define $r(B,S_\bullet) := (-1)^{\sigma}$, which depends on both the orderings of $B$ and of $S_\bullet$. Also define \begin{equation}\label{eq:aS} \frac{1}{a_{S_\bullet}} := \prod_{i=1}^{r-1} \frac{1}{a_{S_i}}. \end{equation} The following is the nested version of \cref{prop:dRind}, and follows in a similar way to \cref{thm:deRhamfannested}. \begin{proposition}\label{prop:nesteddR} We have $\dRip{e_B,e_{B'}} = \sum_{S_\bullet \in N_{\max}(\build)} r(B, S_\bullet) \frac{1}{a_{S_\bullet}} r(B',S_\bullet)$. \end{proposition} While $r(B, S_\bullet)$ depends on the ordering of $S_\bullet$, the product $r(B, S_\bullet)r(B', S_\bullet)$ does not. \part{Geometry} \section{Hyperplane arrangement complements} \def\comp{{\rm comp}} In this section, we work with central hyperplane arrangements in $\C^r$, or projective hyperplane arrangements in $\P^d$, or affine hyperplane arrangements in $\C^d$, where $d = r-1$. \subsection{Hyperplane arrangements} We denote by $\A = \{H_e \mid e \in E\} \subset \C^r$ a central hyperplane arrangement in $\C^r$. Whenever we discuss chambers of $\A$, or the oriented matroid of $\A$, we assume that $\A$ is defined over the reals. We call $\A$ \emph{essential} if $\bigcap_e H_e = (0)$. Unless otherwise specified, the hyperplane arrangements we consider are assumed to be essential and non-empty. We let $M = M(\A)$ denote the rank $r$ matroid associated to $\A$ and $\M = \M(\A)$ a choice of oriented matroid associated to $\A$. We let $f_e$ be a linear function cutting out $H_e$. Let $\bA \subset \P^{d}$ denote the projective hyperplane arrangement associated to $\A$. The hyperplanes of $\bA$ are still denoted by $H_e$, $e \in E$, and the corresponding matroid $M(\bA) = M(\A)$ is the same as for $\A$. Once a hyperplane at infinity $H_0$ has been chosen, we obtain an affine hyperplane arrangement $(\bA,0) = \{H_e \mid e \in E \setminus 0\} \subset \C^{d}$. Thus we work with three types of hyperplane arrangements: central $\A$, projective $\bA$, and affine $(\bA,0)$, and the corresponding matroids are denoted $M$, $M$, and $(M,0)$. Let $\bU = \P^d\setminus \bA = \C^d \setminus (\bA,0)$ denote the hyperplane arrangement complement, which is the same for $\bA$ and $(\bA,0)$. We use matroid terminology to refer to various hyperplane arrangement notions. For example, the lattice of flats $L(M)$ is the lattice of all intersections of hyperplanes in $\bA$, with $\hat 0 = \P^d$ and $\hat 1 = \emptyset$ the intersection of all hyperplanes (empty, since we are assuming that $\bA$ is essential). The connected components of $\bU(\R)$ are called chambers and are identified with the positive topes $\T^+$. The bounded chambers are identified with the bounded topes $\T^0$. We use $P$ to both denote a tope of the oriented matroid, or a chamber of $\bU(\R)$. \subsection{Wonderful compactifications}\label{sec:WC} The compactification $\P^d$ of $\bU$ typically does not have a normal-crossing boundary divisor. However, it can be blown up in various ways to obtain a smooth compactification with normal-crossing boundary divisor. Recall the notion of building sets from \cref{sec:building}. For a building set $\build$ of $L(M)$, let $X_{\build}$ be the corresponding De Concini-Procesi wonderful compactification \cite{DP}. The wonderful compactification $X_{\build}$ is a smooth projective compactification of $\bU$ with a normal crossing boundary divisor. Let $F_1,F_2,\ldots,F_s$ be any ordering of $\build$ such that $F_j > F_i$ implies that $j < i$. Then $X_{\build}$ is obtained from $\P^d$ by first blowing up $F_1$, then blowing up the proper transform of $F_2$, and so on. Geometrically, such an ordering can be obtained by blowing up low-dimensional flats before high-dimensional ones. We write $X_{\max}:= X_{\build_{\max}}$ and $X_{\min}:= X_{\build_{\min}}$ for short. The wonderful compactification $X_{\build}$ has a stratification induced by the intersection of boundary divisors (\cite[Theorem 3.2]{DP}). The boundary divisors $D_F$ of $X_{\build}$ are indexed by flats $F \in \build$. Intersections of the divisors $D_F$ endows $X_{\build}$ a stratification. For $S_\bullet = (S_1,\ldots,S_k)$ the intersection $$ X_{S_\bullet} := D_{S_1} \cap \cdots \cap D_{S_k} $$ is non-empty if and only if $S_\bullet \in \N(\build)$ is a nested collection, and if so, the intersection is transversal and irreducible. In other words, the stratification of $X_{\build}$ has the combinatorics of the dual of the nested set complex. In particular, the vertex strata of $X_{\build}$ are indexed by maximal nested subsets $S_\bullet \in N_{\max}(\build)$. In the case of $\build = \build_{\max}$, the strata $X_{G_\bullet}$ of the compactification $X_{\max}$ are labeled by partial flags $G_\bullet \in \pFl(M)$. We let $$\mathring{X}_{G_\bullet} := X_{G_\bullet} \setminus \bigcup_{G'_\bullet \leq G_\bullet} X_{G'_\bullet} $$ denote the relatively open strata, so that $X_{G_\bullet} = \bigsqcup_{G'_\bullet \leq G_\bullet} \mathring{X}_{G'_\bullet}$. Given a tope $P \in \T^+$, the (analytic) closure $\bP \subset X_{\max}(\R)$ is stratified by the intersection with the strata $X_{G_\bullet}$. Recall that $\pFl(P)$ denotes the set of wonderful faces of $P$, or equivalently, the set of partial flags of flats that belong to $L(P)$. \begin{proposition}[{\cite[Theorem 4.5]{BEPV}}]\label{prop:wonderfulface} A stratum $X_{G_\bullet} \cap \bP$ of $\bP$ in the wonderful compactification $X_{\max}$ is non-empty if and only if $G_\bullet \in \pFl(P)$ is a wonderful face of $P$. \end{proposition} \section{Twisted (co)homology}\label{sec:twistedco} In this section $\bU$ denotes a projective hyperplane arrangement complement of an essential hyperplane arrangement. \subsection{Cohomology} We write $H^\bullet_\dR(\bU,\C)$ to denote the algebraic deRham cohomology of $\bU$, and let $H^\bullet(\bU,\Z)$ and $H_{\bullet}(\bU,\Z)$ denote the Betti (co)homology groups. Brieskorn showed that $H^\bullet_\dR(\bU,\C)$ is generated by the forms $\dlog f_e = \frac{df_e}{f_e}$. \begin{theorem}[\cite{Brie, OS}]\label{thm:Bri} Let $\A \subset \C^d$ be a central and essential hyperplane arrangement with matroid $M$ and complement $U:= \C^d \setminus \A$. We have an isomorphism $$ \OS^\bullet(M) \otimes_\Z \C \cong H_{\dR}^\bullet(U,\C), \qquad e \mapsto [\dlog f_e]. $$ Let $\bA \subset \P^{d}$ be an essential projective hyperplane arrangement with matroid $M$. We have an isomorphism $$ \rOS^\bullet(M) \otimes_\Z \C \cong H_{\dR}^\bullet(\bU,\C), \qquad (e - e') \mapsto [\dlog (f_e/f_{e'})]. $$ \end{theorem} The lattice $\rOS^\bullet(M) \subset \rOS^\bullet(M) \otimes_\Z \C$ spans a lattice inside $H^\bullet_\dR(\bU,\C)$. Under the comparison isomorphism $$ \comp: H^\bullet_\dR(\bU,\C) \to H^\bullet_B(\bU,\Z) \otimes_\Z \C, $$ the lattice $\rOS^\bullet(M)$ is identified with the lattice $H^\bullet_B(\bU,2\pi i \Z) \subset H^\bullet_B(\bU,\Z) \otimes_\Z \C$. \subsection{Twisted (co)homology}\label{ssec:twisted} Henceforth, we assume we are in the case of a projective arrangement, or an affine arrangement. Let $a_e$, $e \in E$ be complex parameters. Consider the meromorphic 1-form \begin{equation}\label{eq:omega} \omega = \omega_\a = \sum_e a_e \dlog f_e = \sum_{e \in E \setminus 0} a_e \dlog(f_e/f_0) \in \Omega^1(\bU) \end{equation} on $\bU$, where we assume that $\sum_{e \in E} a_e = 0$, or equivalently, $a_0 = - \sum_{e \in E \setminus 0} a_e$. We may also view $\omega$ as an element $$ \omega = \sum_e a_e e \in \rOS^1(M).$$ The formula $$ \nabla_\a := d + \omega \wedge $$ defines a logarithmic connection $(\O_\bU,\nabla_\a)$ on the trivial rank one vector bundle $\O_\bU$ on $\bU$. The flat (analytic) sections of $\nabla_\a$ define a complex rank one local system $\L_\a$ on $\bU$. The local sections of $\L_\a$ are branches of the multi-valued function \begin{equation}\label{eq:varphi} \varphi^{-1}:= \prod_e f_e^{-a_e} = \prod_{e \in E \setminus 0} (f_e/f_0)^{-a_e}, \end{equation} which satisfies $\nabla_\a \varphi^{-1} = 0$. Fixing a basepoint $u_0 \in \bU$, the local system $\L_\a$ determines a homomorphism $$ \rho: \pi_1(\bU,u_0) \to \C^\times $$ which determines the isomorphism class of the local system $\L_\a$. Let $\gamma_e$ be a loop starting and ending at $u_0$ that goes once around $H_e$. Then the monodromy of $\L_\a$ around $H_e$ is given by $$ \rho(\gamma_e) = \exp(-2\pi i a_e) = b_e^2, \qquad \text{where} \qquad b_e := \exp(-\pi i a_e). $$ The monodromies satisfy $1 = \prod_e \rho(\gamma_e) = \prod_e b_e^2$ which is implied by $\sum_e a_e =0$. The dual local system $\L^\vee_\a = \L_{-\a}$ has local sections given by the branches of $\varphi = \prod_e f_e^{a_e}$ with $\b$ parameters given by $b^\vee_e = b_e^{-1}$. We have $\dlog \varphi= \omega$. Let $\pi: X_{\build} \to \P^d$ be a wonderful compactification of $\bU$. Recall that for a proper non-trivial flat $F$ belonging to the building set $\build$, we denote by $D_F$ the corresponding divisor of $X_{\build}$. If $\rk(F) = 1$ then $D_F$ is the pre-image of a hyperplane $H= H_e \subset \P^d$ and maps birationally to $H$. Otherwise, $D_F$ is the exceptional divisor of the blowup of (the proper transform of) the flat $F$. \begin{lemma}[see \cite{ESV}]\label{lem:monod}\ \begin{enumerate} \item The residue of the connection $\pi^* \nabla_\a$ along $D_F$ is equal to $a_F$. \item The monodromy of $\L_\a$ once anticlockwise around $D_F$ is equal to $b^2_F$. \end{enumerate} \end{lemma} \begin{proof} We prove (a). This implies (b) by \cite[II.Th\'eor\`eme 1.17]{Del}. If $\rk(F) =1$, then the result is clear since $\pi$ is an isomorphism locally near $D_F$ and by definition \eqref{eq:omega} the residue of $\nabla_\a$ along $H_e$ is $a_e$. Otherwise, let $t$ be the local equation of $D_F$ in the blowup. For $e \in F$, let the local equation of the proper transform of $H_e$ near the general point of $D_F$ be $g_e$. Then $\pi^*(f_e) = t g_e$ for $e \in F$. The pullback of $\omega = \sum_e a_e \dlog f_e$ to $X$ is thus locally given by $$ \pi^*\omega = \sum_{e \in F} a_e (\dlog g_e+\dlog t) + \sum_{e \notin F} a_e \dlog f_e. $$ Since $\dlog f_e$ for $e \notin F$ and $\dlog g_e$ for $e \in F$ have no pole along $D_F$, the residue of $\pi^* \nabla_\a = d + \pi^*\omega$ along $D_F$, is equal to the residue of $(\sum_{e \in F}a_e)\dlog t$ at $t = 0$, which is equal to $a_F = \sum_{e \in F} a_e$. \end{proof} We consider the four twisted Betti (co)homology groups \begin{align*} H_k(\bU,\L_\a) & = \mbox{homology with coefficients in the local system $\L_\a$,} \\ H^{\lf}_k(\bU,\L_\a) &= \mbox{locally-finite homology with coefficients in the local system $\L_\a$,} \\ H^k(\bU,\L_\a) &= \mbox{cohomology with coefficients in the local system $\L_\a$,}\\ H^k_c(\bU,\L_\a) &= \mbox{compactly supported cohomology with coefficients in the local system $\L_\a$}. \end{align*} The locally-finite homology group is also often called Borel-Moore homology. Duality between homology and cohomology gives canonical isomorphisms \begin{equation}\label{eq:4dual} H_k(\bU,\L^\vee_\a) \cong H^k(\bU, \L_\a)^\vee, \qquad H^{\lf}_k(\bU,\L^\vee_\a) \cong H^k_c(\bU, \L_\a)^\vee. \end{equation} The following well-known result is proved in \cite[Theorem 1]{Koh} with a stronger assumption. See also \cite{MHcoh,EV,CDO}. \begin{theorem}[see \cite{Koh}]\label{thm:Koh} Under the assumption \eqref{eq:Mon}, the natural morphisms induce isomorphisms \begin{align*} H_k(\bU,\L_\a) &\stackrel{\cong}{\longrightarrow} H^{\lf}_k(\bU,\L_\a) \\ H^k_c(\bU,\L_\a) &\stackrel{\cong}{\longrightarrow} H^k(\bU,\L_\a) \end{align*} for each $k$. Furthermore, we have the vanishing $$ H_k(\bU,\L_\a)= H^{\lf}_k(\bU,\L_\a)= H^k_c(\bU,\L_\a) = H^k(\bU,\L_\a) = 0 \qquad \mbox{when $k \neq d$}. $$ \end{theorem} \begin{proof} Our argument is the same as that of \cite{Koh}. See also \cite[Proposition 6.5]{BD} for a similar argument. Let $Y \to \P^d$ be the minimal wonderful compactification of $\bA$ (see \cref{sec:WC}). Thus $Y$ is a smooth compactification of $\bU$ with a normal-crossing boundary divisor $D:=Y \setminus U$, obtained by blowing up all the connected flats $F$. Let $\iota: \bU \to Y$ be the inclusion. To prove the first statement, we show that $\iota_! \L_\a \cong R\iota_* \L_\a$ in the derived category on $Y$. It suffices to check that the stalk of $R\iota_* \L_\a$ vanishes on $D$. Since by construction $D$ is a normal crossing divisor, it suffices to check that $\L_\a$ has non-trivial monodromy around every irreducible component of $D$. The irreducible components $D_F$ of $D$ are labeled by the connected flats $F \in L(M)$. The divisor $D_F$ is the exceptional divisor obtained when (the proper transform of) $F$ is blown up. By \cref{lem:monod}, the monodromy of $\L_\a$ around $D_F$ is given by $b^2_F = \prod_{e \in F} b^2_e = \exp(-2\pi i a_F)$. By assumption \eqref{eq:Mon}, this is not equal to 1, so $\L_\a$ has non-trivial monodromy around $D_F$. We conclude that $\iota_! \L_\a \cong R\iota_* \L_\a$. Taking derived global sections we obtain the isomorphism $H^k_c(\bU,\L_\a) \cong H^k(\bU,\L_\a)$, and by duality \eqref{eq:4dual} we obtain $H_k(\bU,\L_\a) \cong H^{\lf}_k(\bU,\L_\a)$. Now we prove the last statement. Since $\bU$ is affine, by Artin vanishing we have $H^k(\bU,\L_\a) = 0$ for $k > d$ and by Poincar\'e-Verdier duality we have $H^k_c(\bU,\L_\a) = 0$ for $k < d$. Combining with $H^k_c(\bU,\L_\a) \cong H^k(\bU,\L_\a)$ and \eqref{eq:4dual}, we obtain the vanishing of all four groups when $k \neq d$. \end{proof} \subsection{Logarithmic description of twisted cohomology} The analogue of Brieskorn's theorem (\cref{thm:Bri}) for twisted cohomology is due to Esnault--Schechtman--Viehweg \cite{ESV}, and was extended by Schechtman--Terao--Varchenko \cite{STV}. Let $(\O^\an_\bU, \nabla^\an_\a)$ denote the analytification of $(\O_\bU,\nabla_\a)$. Let $(\Omega^\bullet_\bU,\nabla_\a)$ (resp. $(\Omega^{\bullet,\an}_\bU, \nabla^\an_\a)$) denote the complex of algebraic (resp. analytic) differential forms on $\bU$, equipped with the differential $\nabla_\a$ (resp. $\nabla_\a^\an$). The \emph{algebraic deRham cohomology} (resp. \emph{analytic deRham cohomology}) of the connection $\nabla_\a$ is $$ H^\bullet(\bU, \nabla_\a) := H^\bullet(\Gamma(\bU, \Omega_\bU^\bullet),\nabla_\a), \qquad H^\bullet(\bU, \nabla^\an_\a) := H^\bullet(\Gamma(\bU, \Omega_\bU^{\bullet,\an}),\nabla^\an_\a). $$ In both cases, the cohomology groups are usually defined as hypercohomologies, and we may replace by global sections since $\bU$ is affine (resp. Stein); see \cite[II.6]{Del}. We have a GAGA-type isomorphism between algebraic and analytic deRham cohomology of the connection $$ H^\bullet(\bU, \nabla_\a) \cong H^\bullet(\bU, \nabla^\an_\a), $$ see \cite[II Th\'eor\`eme 6.2]{Del}. We may thus freely switch between algebraic and analytic deRham cohomologies. The identification of flat sections of $\nabla_\a$ with the local system $\L_\a$ gives a comparison isomorphism $$ \comp_\a: H^\bullet(\bU,\nabla_\a) \stackrel{\cong}{\longrightarrow} H^\bullet(\bU, \L_\a). $$ Let $(\rOS^\bullet,\omega)$ denote the Aomoto complex of \cref{sec:Aomoto}. Since $d$ acts trivially on $\rOS^\bullet$, we have $\nabla_\a = \omega \wedge$ on $\rOS^\bullet$, and thus $(\rOS^\bullet,\omega)$ is a subcomplex of $\Gamma(\bU, (\Omega_\bU^\bullet,\nabla_\a))$. \begin{theorem}[\cite{ESV,STV}]\label{thm:ESV} Under the assumption \eqref{eq:Mon}, the natural inclusion $$ (\rOS^\bullet, \omega) \hookrightarrow \Gamma(\bU, (\Omega_\bU^\bullet,\nabla_\a)) $$ is a quasi-isomorphism. In particular, we have $$ H^\bullet(\bU,\L_\a) \cong H^\bullet(\bU, \nabla_\a) \cong H^\bullet(\rOS^\bullet,\omega). $$ \end{theorem} \begin{proof} We work in the minimal wonderful compactification $\pi: Y = X_{\min} \to \P^d$. Let $D:= Y \setminus \bU$. By \cref{lem:monod} and assumption \eqref{eq:Mon}, the monodromy of the pullback $\pi^*\L_\a$ is non-trivial around any of the boundary divisors $D_F \subset Y$. By \cite[II.3.13, 3.14]{Del}, this implies that in the calculation of $H^\bullet(\bU,\nabla_\a)$ we may replace the complex $(\Omega_\bU^\bullet,\nabla_\a)$ (or $(\Omega_\bU^{\bullet,\an},\nabla^\an_\a)$) by the complex $(\Omega^\bullet_Y(\log D), \nabla_\a)$ of logarithmic differential forms on $Y$, equipped with the differential $\nabla_\a$. Namely, we have an isomorphism \begin{equation}\label{eq:Lnabla} H^\bullet(\bU,\L_\a) \cong {\mathbb H}^\bullet(\Omega^\bullet_Y(\log D),\nabla_\a). \end{equation} By \cite{Del}, the spectral sequence $$ E_1^{p,q} = H^q(Y, \Omega^p_Y(\log D)) \implies H^{p+q}(\bU,\C) $$ degenerates at $E_1$ and by Brieskorn's theorem (\cref{thm:Bri}) we have an isomorphism $H^\bullet(\bU,\C) \cong \rOS^\bullet \cong \Gamma(Y,\Omega^\bullet_Y(\log D))$. It follows that the higher cohomologies of the sheaves $\Omega^p_Y(\log D)$ vanish. Combining with \eqref{eq:Lnabla}, the higher cohomology vanishing gives an isomorphism $H^\bullet(\bU,\L_\a) \cong H^\bullet(\Gamma(Y,\Omega^\bullet_Y(\log D)),\nabla_\a)$. Composing with the isomorphism $(\rOS^\bullet,\omega) \stackrel{\cong}{\longrightarrow} (\Gamma(Y,\Omega^\bullet_Y(\log D)), \nabla_\a)$, we obtain the stated quasi-isomorphism. \end{proof} It follows from \cref{thm:Koh} and \cref{thm:ESV} that in this case $\rOS^k(M,\omega)$ vanishes unless $k =d$; compare with \cref{thm:Yuz}. Furthermore, we have $\dim H^d(\bU,\L_\a) = \dim H^d(\bU,\nabla_\a) = \beta(M)$. \subsection{Betti homology twisted intersection form} The intersection form in Betti homology was first defined by Kita and Yoshida \cite{KY}. \begin{definition} Assume \eqref{eq:Mon}. The \emph{Betti homology intersection form} $$ \gBip{\cdot, \cdot}: H^{\lf}_d(\bU,\L^\vee_\a) \otimes H^{\lf}_d(\bU, \L_\a) \to \C $$ is the composition of the the regularization isomorphism $H^{\lf}_d(\bU,\L_\a) \cong H_d(\bU,\L_\a)$ of \cref{thm:Koh} with the Poincar\'e-Verdier duality perfect pairing $$ H^{\lf}_d(\bU,\L^\vee_\a) \otimes H_d(\bU, \L_\a) \to \C. $$ The \emph{Betti cohomology intersection form} $\gDBip{\cdot,\cdot}$ is similarly defined. \end{definition} The Betti homology and Betti cohomology intersection forms are related by duality. Namely, the perfect pairing $\gBip{\cdot,\cdot}$ together with \eqref{eq:4dual} induces isomorphisms $\gamma: H_\bullet(\bU,\L_\a) \cong H^\bullet(\bU, \L_\a^\vee)$ and $\delta: H_\bullet(\bU,\L^\vee_\a) \cong H^\bullet(\bU, \L_\a)$, and we have $$ \gDBip{a,b} = \gBip{\gamma^{-1}(b),\delta^{-1}(a)} $$ for $a \in H^d(\bU, \L_\a)$ and $b \in H^d(\bU,\L^\vee_\a)$. We now describe the twisted homology group $H_p(\bU,\L_\a)$ explicitly. The $p$-th chain group $C_p(\bU,\L_\a)$ is spanned by twisted $p$-chains $[\Gamma \otimes \varphi^{-1}_\Gamma]$ where $\Gamma$ is a $p$-chain, that is, a map of the $p$-dimensional simplex $\Delta^p$ into $\bU$, and $\varphi^{-1}_\Gamma$ is a choice of a branch of $\varphi^{-1}$ from \eqref{eq:varphi} on $\Gamma$. The boundary operator $\partial: C_p(\bU,\L_\a) \to C_{p-1}(\bU,\L_\a)$ is given by $$ \partial [\Gamma \otimes \varphi^{-1}_\Gamma] = \sum_i (-1)^i [\partial_i \Gamma \otimes (\varphi^{-1}_\Gamma)|_{\partial_i \Gamma}] $$ where we write $\sum_i$ for the summation over the boundary components of a $p$-simplex. The twisted $p$-chains in the kernel of $\partial$ are called \emph{twisted $p$-cycles}, and the twisted homology group is defined as $$ H_p(\bU,\L_\a) := \frac{\ker( \partial: C_p(\bU,\L_a) \to C_{p-1}(\bU,\L_\a))}{\image ( \partial: C_{p+1}(\bU,\L_a) \to C_{p}(\bU,\L_\a))}. $$ The twisted locally-finite homology group $H^{\lf}_p(\bU,\L_\a)$ is defined similarly, now allowing formal linear combinations of $[\Gamma \otimes \varphi^{-1}]$ that are locally-finite. Twisted cycles are also called \emph{loaded} cycles in the literature. \begin{definition}\label{def:standard} Let $P \in \T$ be a tope of $\bA$. The \emph{standard twisted $d$-cycle} $[P \otimes \varphi^{-1}_P] \in C_d^{\lf}(\bU,\L_\a)$ is the locally-finite $d$-dimensional cycle where $\varphi^{-1}_P$ is the scalar multiple of the branch of $\varphi^{-1}$ that takes positive real values on $P$ when the $\a$ are real. \end{definition} For each $e$, there is a ``real" branch of $f_e^{-a_e}$ that for $a_e \in \R$ takes real values on $\bU(\R) \cap \{f_e > 0\}$. We have \begin{equation}\label{eq:standardb} \varphi^{-1}_P = b_P \varphi^{-1}, \qquad b_P := \prod_{e: P(e) = -} b_e, \end{equation} where $\varphi^{-1}$ denotes the product of the real branches of $f^{-a_e}_e$. The prefactor $b_P$ is chosen so that $\varphi^{-1}_P$ takes positive real values on $P$. Indeed, $$ b_P \varphi^{-1} = \prod_{e \neq 0} b_e (f_e/f_0)^{-a_e} = \prod_{e \neq 0} \exp(-\pi i a_e) (f_e/f_0)^{-a_e} = \prod_{e \neq 0}(-f_e/f_0)^{-a_e}. $$ We remark that formally speaking $[P \otimes \varphi^{-1}_P]$ should be expressed as an infinite sum of chains that head towards the boundary of $P$. Recall from \cref{thm:Koh} that under assumption \eqref{eq:Mon}, $H_\bullet^{\lf}(\bU,\L_\a) \cong H_\bullet(\bU,\L_\a)$ vanishes unless $\bullet = d$. We now assume that we are working with an affine hyperplane arrangement $\bA \subset \C^d$, and let $\T^0$ denote the set of bounded topes (or chambers). By \cref{prop:numbertopes}, we have $|\T^0| = \beta(M) = \dim H_d(\bU,\L_a)$. The following result is due to Kohno \cite{Koh}; see also \cite[Proposition 3.14]{DT}. It will also follow indirectly from our results (\cref{thm:Bettinondeg} and \cref{thm:Bettipairmain}). \begin{proposition} Assume \eqref{eq:Mon}. The twisted cycles $C(P) := [P \otimes \varphi^{-1}_P]$ (resp. $C^\vee(P) := [P \otimes \varphi_P]$) for $P \in \T^0$ form a basis of $H_d^{\lf}(\bU,\L_\a)$ (resp. $H_d^{\lf}(\bU,\L^\vee_\a)$). The regularized twisted cycles $\reg(C(P))$ (resp. $\reg(C^\vee(P))$) for $P \in \T^0$ form a basis of $H_d(\bU,\L_\a)$ (resp. $H_d(\bU,\L^\vee_\a)$). \end{proposition} We take $\{[P \otimes \varphi^{-1}_P]\}$ as a basis for $H_d^{\lf}(\bU,\L_\a)$, and $\{[P \otimes \varphi_P]\}$ as a basis for $H_d^{\lf}(\bU,\L^\vee_\a)$. \begin{proposition}\label{prop:isoB} There is a natural isomorphism $ H^{\lf}_\bullet(\bU,\L_\a) \cong H^{\lf}_\bullet(\bU, \L_\a^\vee)$ given by $[P \otimes \varphi^{-1}_P] \mapsto [P \otimes \varphi_P]$. \end{proposition} Compositing with the isomorphism of \cref{prop:isoB}, $\ip{\cdot,\cdot}_B$ can be viewed as a bilinear form on $ H^{\lf}_\bullet(\bU,\L_\a) $. The pairing $\gBip{C^\vee, C}$ is given by the formula \begin{equation}\label{eq:CC} \gBip{C^\vee, C} = \langle C^\vee, \reg(C) \rangle := \sum_{p \in \Gamma^\vee \cap \Gamma} \varphi_{\Gamma^\vee}(p) \langle\Gamma^\vee,\Gamma \rangle_p \varphi^{-1}_\Gamma(p). \end{equation} where $\reg(C) = [\Gamma \otimes \varphi^{-1}_\Gamma] \in H_d(\bU,\L_\a)$ is a regularization of $C$ in general position with respect to $C^\vee = [\Gamma^\vee \otimes \varphi_{\Gamma^\vee}]$, and $\langle\Gamma^\vee,\Gamma \rangle_p$ denotes the topological intersection number of the transverse intersection $\Gamma^\vee \cap \Gamma$ at $p$. With all the above notation defined, we can state our main theorem for the Betti intersection pairing. \begin{theorem}\label{thm:Bettipairmain} For $P,Q \in \T^+$, we have $\gBip{[P \otimes \varphi_P],[Q \otimes \varphi^{-1}_Q]}= \bip{P,Q}_B$, as defined in \cref{def:Bettipair}. \end{theorem} The proof of \cref{thm:Bettipairmain} will be given in \cref{sec:Bettipairmain}. Kita and Yoshida initially studied the Betti homology twisted intersection form in \cite{KY,KY2} and Yoshida \cite{Yos} studied determinantal formulae for the intersection form. The intersection pairing is studied for the braid arrangement \cite{MOY}, for arrangements appearing in conformal field theory \cite{MY}, and in relation to Lauricella's hypergeometric function \cite{Goto}, and Togi \cite{Tog} gave some closed formulae. In \cite{GM}, Goto and Matsubara-Heo study the intersection form for GKZ type systems, which involve arrangements of hypersurfaces of higher degree. \begin{remark} The symmetry of the bilinear form $\gBip{\cdot,\cdot}$ is specific to the choice of isomorphism in \cref{prop:isoB}, and the choice of standard loading in \cref{def:standard}. Indeed, the original bilinear form computed in \cite{KY} did not have the symmetry property. \end{remark} \subsection{deRham cohomology twisted intersection form} The intersection form in deRham cohomology was introduced and studied by Cho and Matsumoto \cite{CM}. The dual connection $\nabla^\vee_\a$ to $\nabla_\a$ is given by the connection $\nabla_{-\a}$. \begin{definition} Assume \eqref{eq:Mon}. The \emph{deRham cohomology intersection form} $$ \gdRip{\cdot, \cdot}: H^d(\bU,\nabla^\vee_\a) \otimes H^d(\bU, \nabla_\a) \to \C $$ is the composition of the the regularization isomorphism $\reg: H^d(\bU,\nabla_\a) \cong H_c^d(\bU,\nabla_\a)$ of \cref{thm:Koh} with the Poincar\'e-Verdier duality perfect pairing $$ H^d(\bU,\nabla^\vee_\a) \otimes H^d_c(\bU, \nabla_\a) \to \C. $$ The \emph{deRham homology intersection form} $\gDdRip{\cdot, \cdot}$ is similarly defined. \end{definition} Just as in the Betti case, the deRham cohomology and deRham homology intersection forms are related by duality. Let $[\theta^\vee] \in H^d(\bU,\nabla^\vee_\a)$ and $[\theta] \in H^d(\bU,\nabla_\a)$ be cohomology classes represented by twisted $d$-forms $\theta^\vee,\theta$. To compute $\dRip{\theta^\vee,\theta}$, we first find a cohomologous compactly supported form $\reg(\theta)$, representing a class in $[\reg(\theta)] \in H^d_c(\bU, \nabla_\a)$. The Poincar\'e-Verdier duality pairing is then given by \begin{equation}\label{eq:PV} \gdRip{\theta^\vee,\theta} = \ip{\theta^\vee,\reg(\theta)} = \frac{1}{(2\pi i)^d} \int_\bU \theta^\vee \wedge \reg(\theta), \end{equation} where the integral is over all (complex) points of $\bU$. We caution that the factor of $(2\pi i)^{-d}$ is not always included in the literature. We note that $\omega_{-\a} = - \omega_{\a}$ and $\rOS(M,\omega) \cong \rOS(M,-\omega)$. By \cref{thm:ESV}, the natural maps \begin{equation}\label{eq:rOSaa} \rOS(M,\omega) \longrightarrow H^\bullet(\bU,\nabla^\vee_\a), \qquad \text{and} \qquad \rOS(M,\omega) \longrightarrow H^\bullet(\bU,\nabla_\a) \end{equation} are isomorphisms, so we may view the deRham cohomology intersection form as a bilinear form $$ \gdRip{\cdot,\cdot}: \rOS(M,\omega) \otimes \rOS(M,\omega) \to \C. $$ We can now state our main theorem for the deRham intersection pairing. \begin{theorem}\label{thm:dRpairmain} For $x,y \in \rOS(M,\omega)$, we have $\gdRip{x,y} = \bdRip{x,y}$, as defined in \cref{sec:descent}. \end{theorem} The proof of \cref{thm:dRpairmain} will be given in \cref{sec:dRpairmain}. Cho and Matsumoto \cite{CM} first studied the deRham cohomology twisted intersection pairing in the one-dimensional case. Matsumoto \cite{Matgen} gave an explicit formula for the intersection pairing in the case of a generic hyperplane arrangement, and specific arrangements were considered, for instance, in \cite{Goto, MOY, MY}. Goto and Matsubara-Heo \cite{GM} study the intersection form for GKZ type systems. \begin{remark} The symmetry of the bilinear form $\ip{\cdot,\cdot}^{\dR}$ has been apparent from the beginning \cite{CM}. A formula due to Matsubara-Heo \cite[Theorem 2.2]{MHcoh} exhibits this symmetry in a general setting. \end{remark} \section{Proof of deRham cohomology pairing}\label{sec:dRpairmain} We prove \cref{thm:dRpairmain}, following \cite{CM,Matgen}. Let us first consider the one-dimensional case $\bU = \P^1 \setminus \{z_1,z_2,\ldots,z_t\}$ and let $\theta$ be a holomorphic $1$-form on $\bU$. Let $g_i$ denote a smooth function that takes value $1$ in a small neighborhood $V_i \ni z_i$ and takes value $0$ outside a larger neighborhood $W_i \supset V_i$. Also, let $\psi_i$ be a smooth function that is a solution to $\nabla_\a \psi_i = \theta$ in $\overline{W_i}$. The existence of this solution follows from the assumption \eqref{eq:Mon} that the monodromy of $\L_\a$ is non-trivial around $z_i$, which implies \begin{equation} \label{eq:DM} H^\bullet(W_i \setminus z_i, \L_\a) \cong H^\bullet(S^1, \L_\a) = 0. \end{equation} See the comprehensive discussion in \cite[Section 2]{DM}. We then take $$ \reg(\theta):= \theta - \nabla_\a(\sum_i \psi_i g_i) = \theta - \sum_i (\psi_i d g_i - g_i \theta) = \theta( 1- \sum_i g_i) - \sum_i \psi_i dg_i. $$ The first formula shows that $\reg(\theta)$ is $\nabla_\a$-cohomologous to $\theta$, and the rightmost expression shows that $\reg(\theta)$ is compactly supported. Now, for another holomorphic $1$-form $\theta^\vee$, we have $$ \gdRip{\theta^\vee,\theta} = \frac{1}{2\pi i} \int_\bU \theta^\vee \wedge \reg(\theta) = - \frac{1}{2\pi i} \sum_i \int_\bU \theta^\vee \wedge \psi_i dg_i $$ since $\theta^\vee \wedge \theta = 0$ as there are no holomorphic $2$-forms on $\bU$. Since $dg_i$ is supported on a small annulus $W_i \setminus V_i$ around $z_i$, the integral localizes to each of these points. By a limiting procedure, one may replace $g_i$ by a step function, and $dg_i$ is a delta-function on a circle, that is $$ - \frac{1}{2\pi i} \sum_i \int_\bU \theta^\vee \wedge \psi_i dg_i = \sum_i \frac{1}{2\pi i} \int_{|z-z_i| = \epsilon} \psi_i \theta^\vee = \sum_i \Res_{z_i} (\psi \theta^\vee). $$ Matsumoto \cite[Lemma 4.1]{Matgen} shows that \begin{equation}\label{eq:Mat} \Res_{z_i}(\psi \theta^\vee) = \Res_{z_i} (\theta^\vee) \frac{1}{a_i} \Res_{z_i}(\theta), \qquad \text{giving} \qquad \gdRip{\theta^\vee, \theta} = \sum_i \Res_{z_i} (\theta^\vee) \frac{1}{a_i} \Res_{z_i}(\theta), \end{equation} and further extends this calculation to higher-dimensional cases of generically intersecting collection of hyperplanes. In that case, the integral localizes to the vertices of the arrangement, and since the hyperplanes are normal crossing at each vertex, the existence of solutions to $\nabla_\a \psi = \theta$ can be obtained by applying the Kunneth formula to \eqref{eq:DM}. Since the computation is local, it can be applied whenever there is a compactification with normal-crossing divisors. Matsumoto's local formula gives: \begin{proposition}\label{prop:dRresidue} Let $\pi: X_\build \to \P^d$ be a wonderful compactification of $\bU$. Then for $x, y \in \rOS(M,\omega)$, we have $$ \gdRip{x,y} = \sum_{S_\bullet \in N_{\max}(\build)} \Res_{S_\bullet}(x) \frac{1}{a_{S_\bullet}} \Res_{S_\bullet}(y), $$ where the summation is over the maximal nested collections (see \cref{sec:building} and \eqref{eq:aS}), and $\Res_{S_\bullet}$ is the residue at a normal-crossing vertex, obtained as the integral over a multi-dimensional torus $(S^1)^d$: \begin{equation}\label{eq:ResS} \Res_{S_\bullet}(x) = \int_{|t_d|=\epsilon} \cdots \int_{|t_1|=\epsilon} x, \end{equation} where $t_1,t_2,\ldots,t_d$ are local coordinates cutting out the normal-crossing vertex $X_{S_\bullet}$. \end{proposition} The residue map $\Res_{S_\bullet}$ is defined up to a sign which appears twice and cancels out. \begin{proof} Let $S_\bullet = (S_1,\ldots,S_d)$. The vertex $X_{S_\bullet}$ of $X_\build$ is the transverse intersection of $D_{S_1},\ldots,D_{S_d}$. According to \cref{lem:monod}, the residue of $\pi^* \nabla_\a$ around $D_{S_i}$ is equal to $a_{S_i}$. By Matsumoto's theorem \cite[Theorem 2.1]{Matgen}, or by applying the Kunneth formula to \eqref{eq:Mat}, we see that the local contribution around $X_{S_\bullet}$ is given by $\Res_{S_\bullet}(x) \frac{1}{a_{S_\bullet}} \Res_{S_\bullet}(y)$. \end{proof} Matsumoto's approach is generalized significantly by Matsubara-Heo \cite[Theorem 2.2]{MHcoh} from which \cref{prop:dRresidue} could also be obtained directly. In the case that $X_\build = X_{\max}$, we have $\N(\build) = \Fl(M)$, so comparing with \cref{def:dR}, the proof of \cref{thm:dRpairmain} is completed by the following. \begin{lemma}\label{lem:ResF} The residue $\Res_{F_\bullet}(x)$ agrees with the residue defined in \cref{sec:residue}. \end{lemma} \begin{proof} The formula \eqref{eq:ResS} can be written as the composition of one-dimensional residue integrals. We proceed by induction on the number of integrals $d$. When $d=1$, we have $X_{\max} = \P^1$, so the result is clear. Suppose $d > 1$ and let $F_1 = \{e\}$. The divisor $D_{F_1} \subset X_{\max}$ is isomorphic to the maximal wonderful compactification $X''_{\max}$ of the contraction $\bA'' = \bA \cap H_e$, where the local equation of $H_e$ is $t_1 = 0$. The following diagram commutes: $$ \begin{tikzcd} \rOS(M)\otimes \C \arrow[r, "\Res_e"] \arrow[d] & \rOS(M/e)\otimes \C \arrow[d] \\ \Omega^d(\bU) \arrow[r, "\Res_{H_e}"] & \Omega^{d-1}(\P^{d-1} \setminus \bA'') \end{tikzcd} $$ where $\Res_{H_e}$ can be computed by a one-dimensional integral $\int_{|t_1|=\epsilon}$. The result follows by induction on dimension. \end{proof} The analogue of \cref{lem:ResF} also holds for other building sets (see \cref{sec:building}), but the recursive structure in the proof is simpler for $X_{\max}$. \section{Proof of Betti homology pairing}\label{sec:Bettipairmain} \def\neg{{\rm neg}} We work in an affine chart where $f_0$ is positive. Then $\bU$ (resp. $\bU(\R)$) is identified with an open subset of $\C^d$ (resp. $\R^d$). Each connected component $P$ of $\bU(\R)$ inherits an orientation from the ambient orientation of $\R^d$. Viewing each $P$ as a signed covector, our assumption that $f_0$ is positive is that $P(0) = +$. \subsection{Regularization} We now discuss the regularization of our twisted cycles $C(P) = [P \otimes \varphi^{-1}_P]$, following \cite{KY,KY2}. The construction is local. We first discuss the one-dimensional case. The regularization of a single interval $C = [[0,1] \otimes \psi]$ is taken to be the following finite sum of twisted $1$-chains: $$ C_\reg = \frac{1}{b_0^2-1}[S(0,\epsilon)\otimes \psi] + [[\epsilon,1-\epsilon] \otimes \psi]- \frac{1}{b_1^2-1}[S(1,1-\epsilon)\otimes \psi], $$ pictured below in the punctured complex plane $\C^1 \setminus \{0,1\}$: $$ \begin{tikzpicture} \draw[thick,decoration={markings, mark=at position 0.9 with {\arrow{>}}},postaction={decorate}] ([shift=(3:1)]0,0) arc (3:360:1); \draw[thick,decoration={markings, mark=at position 0.5 with {\arrow{>}}},postaction={decorate}] (1,0) -- (4,0); \draw[thick,decoration={markings, mark=at position 0.9 with {\arrow{>}}},postaction={decorate}] (4,0) arc (-180:177:1); lldraw (0,0) circle (1pt); lldraw (5,0) circle (1pt); \node[color=blue] at (0,-0.2) {$0$}; \node[color=blue] at (5,-0.2) {$1$}; \end{tikzpicture} $$ We assume that the analytic continuation of $\psi$ once anticlockwise around $0$ (resp. $1$) produces a factor of $b^2_0$ (resp. $b^2_1$). For instance, if $\varphi = z^{-a_0}(1-z)^{-a_1}$ then we would have $b_i = \exp(-\pi i a_i)$. Here, $S(0,\epsilon)$ (resp. $S(1,1-\epsilon)$ denotes an oriented circle starting at the point $\epsilon$ (resp. $1-\epsilon$) and going once around the point $0$ (resp. $1$) in the anti-clockwise direction. (Strictly speaking, we should express $S(0,\epsilon)$ and $S(1,1-\epsilon)$ as the sum of two 1-chains, each of which is a half circle.). See \cite{KY,AKbook,Mat} for this formula. That the factor $b_0^2-1$ is correct follows from the calculation: $$ \partial [S(0,\epsilon) \otimes \psi] = [\epsilon \otimes b^2_0\psi(\epsilon)] - [\epsilon \otimes \psi(\epsilon)]= (b_0^2-1)[\epsilon \otimes \psi(\epsilon)], $$ where $b_0^2$ (resp. $1$) is the value of $\psi$ at the end (resp. start) of the curve $S(0,\epsilon)$. To show that $C_\reg$ is a regularization of $C$, we observe that the difference $$ C - C_\reg = \sum_{i= 0,1} \partial(\text{punctured disk $D_i$ analytically continued anticlockwise from the interval}) $$ is the boundary of a twisted locally-finite $2$-chain. In higher-dimensions, for a region $P$ where every vertex is a transversal intersection, we use a product construction. More precisely, let us consider the locally-finite chain $C = [(0,1]^d \otimes \psi]$ in $\R^d$, equipped with the standard orientation of $\R^d$. We imagine that locally $P$ is the product of intervals $[0,1]^d$, and the point $(0,0,\ldots,0)$ is a vertex of $P$, a transverse intersection of the hyperplanes $z_i = 0$. Then we define the regularization $\reg(C)$ by \begin{equation}\label{eq:regC} \reg(C) = \left(\prod_{i=1}^d \frac{1}{b_i^2-1} S_i(0,\epsilon)+ [\epsilon,1]_i \right) \otimes \psi \end{equation} where the $i$-th factor of the product belongs to coordinate $z_i$ of $\C^d$. That \eqref{eq:regC} is a regularization of $C$ follows by taking the product of the $1$-dimensional case. Note that in \eqref{eq:regC}, $C$ and $\reg(C)$ are twisted $d$-chains and not cycles. Gluing together such local regularizations, we obtain the regularization $\reg([P \otimes \varphi^{-1}_P])$. \subsection{Intersection numbers} Now let $C^\vee = C^\vee(Q)$ and $C= C(P)$. In the following, we calculate the intersection number $\ip{C^\vee,C}$ from \eqref{eq:CC} by replacing $C$ with the regularization $\reg(C)$, and $C^\vee$ by a slight deformation. Our calculation follows the local strategy of \cite{KY,KY2}. Embed $\bU$ into the maximal wonderful compactification $X = X_{\max}$. \begin{proposition}\label{prop:Bettiint} Let $P,Q \in \T^+$. We have the formula $$ \gBip{[Q \otimes \varphi_Q], \reg([P \otimes \varphi^{-1}_P])} = \sum_{E_\bullet} (\pm)^r \prod_{i \in I} (-1)^{\rk(E_i)} \frac{b_{E_i}}{\tb_{E_i}} \prod_{j \in J} \frac{1}{\tb_{E_j}} = \sum_{E_\bullet} (\pm)^r \prod_{i \in I} (-1)^{\rk(E_i)} b_{E_i} \prod_{\ell \in I \cup J} \frac{1}{\tb_{E_\ell}} $$ where the sum is over wonderful faces $E_\bullet \in \Delta(P)$ such that $\mathring{X}_{E_\bullet}$ intersects $\overline{P} \cap \overline{Q}$, and the first (resp. second) product over $i\in I$ (resp. $j \in J$) is over wonderful facets for which $P$ and $Q$ are on the opposite (resp. same) sides of. The sign $(\pm)^r$ is equal to $(-1)^r$ if the line at infinity $0 \in E$ is crossed when going from $P$ to $Q$; otherwise, the sign is taken to be $1$. \end{proposition} \begin{proof} Since $X$ is smooth, it is locally isomorphic to $\C^n$ with $X(\R)$ identified with $\R^d \subset \C^d$. We have the decomposition $\C^d = \R^d \oplus i \R^d$, where $i \R^d$ can be identified with the fiber of the normal bundle of the manifold $X(\R)$ in $X(\C)$. Let $\bQ$ be the (analytic) closure of $Q$ in the wonderful compactification $X_{\max}(\R)$. Recall that the faces of $\bQ$ are defined to be the non--empty intersections of $\bQ$ with various strata $X_{E_\bullet}$ of $X$. Fix a barycenter $\tau_K$ in the relative interior of each face $K$ of $\bQ$. Define a continuous vector field $Z$ on $\bQ$ that such that if $p$ lies in the relative interior of a face $K$ of $\bQ$, then $Z(p)$ points towards the barycenter $\tau_K \in K$. This is possible since each face $K$ of $\bQ$ is contractible. By construction, the vector field $Z$ vanishes exactly at the points $\tau_K$. If $f: W \to Q$ is a parametrization of $Q$, then we obtain a new (locally-finite) cycle $Q_Z$ by the parametrization $$ f_Y(w) = f(w) + i Z(f(w)) \in X(\C). $$ (Formally, we should write $Q$ as the sum of infinitely many elementary chains, and apply this formula to each of them.). Let $F_\bullet$ be a wonderful vertex of $\bQ$. Locally near $X_{F_\bullet}$ we have that $\bQ$ can be identified with a positive orthant, and $Q_Z$ is the product of the picture $$ \begin{tikzpicture} \draw[thick,decoration={markings, mark=at position 0.5 with {\arrow{>}}},postaction={decorate}] (0,0) -- (3.75,0); \draw[dashed] (3.75,0) -- (5,0); \draw[thick,color=red,decoration={markings, mark=at position 0.5 with {\arrow{>}}},postaction={decorate}] (0,0) to [bend left = 45] (2.5,0); \begin{scope} \clip (2.5,1) rectangle (3.75,-1); \draw[thick,color=red,decoration={markings, mark=at position 0.5 with {\arrow{>}}},postaction={decorate}] (2.5,0) to [bend right = 45] (5,0); \end{scope} \begin{scope} \clip (3.75,-1) rectangle (5.75,1); \draw[dashed,color = red] (2.5,0) to [bend right = 45] (5,0); \end{scope} lldraw (0,0) circle (1pt); lldraw (2.5,0) circle (1pt); \node[color=blue] at (2.5,-0.2) {$\tau_{E_\bullet}$}; \node[color=blue] at (0,-0.2) {$\tau_{F_\bullet}$}; \node[color=black] at (1.5,-0.2) {$Q$}; \node[color=red] at (1.5,0.75) {$Q_Z$}; \end{tikzpicture} $$ in each coordinate. We replace the locally-finite twisted cycle $C^\vee(Q) = [Q \otimes \varphi_Q]$ with the homologous locally-finite twisted cycle $$ C^\vee(Q_Z) = [Q_Z \otimes \varphi_Q] $$ and $[P \otimes \varphi_P^{-1}]$ by its regularization, which locally near any vertex $X_{F_\bullet}$ of $X$ has the form \eqref{eq:regC}. Consider a term in the expansion of \eqref{eq:regC} where we take the factor $S_\ell(0,\epsilon)$ for $\ell \in L$ and $[\epsilon,1]_m$ for $m \in M$, and $L \sqcup M = [d]$. For sufficiently small $\epsilon$, any intersection of $Q_Y$ with such a chain will occur at a point $p \in \bU \subset X$ where $$ p|_{M} = \tau_M, \qquad \text{and} \qquad p_\ell \in S_\ell(0,\epsilon) \mbox{ for $\ell \in L$}, $$ and $\tau_M$ is the center of the face indexed by $M$. (Here, we have written the coordinates of $p = (p_1,\ldots,p_d)$ using the same local coordinates as in \eqref{eq:regC}.) Thus the intersection $Q_Z \cap \reg(C)$ consists of isolated transverse points, one close to each of the barycenters $\tau_M$. We now calculate the intersection number $\varphi_{\Gamma^\vee}(p) \langle\Gamma^\vee,\Gamma \rangle_p \varphi^{-1}_\Gamma(p)$ at the point $p$ close to $\tau_M$. We get a factor of $$ \prod_{\ell \in L}\frac{1}{\tb_\ell} $$ from the prefactors of $S_\ell(0,\epsilon)$, and a factor of $$ \varphi_{Q}(p) \varphi^{-1}_P(p) = \prod_{i \in I} b_i $$ resulting from analytic continuation of $\varphi^{-1}_P$ along half circles (cf. the definition \eqref{eq:standardb} of the standard loading), where $I \subset L$ denotes the set of indices where $P$ and $Q$ are on the opposite sides of in the neighborhood of $X_{F_\bullet}$. The set of indices where $P$ and $Q$ are on the same side of is $J = L \setminus I$. In other words, $L = I \sqcup J$. Finally, we calculate the sign. We obtain a sign $(-1)^{|I|}$ from comparing the orientations of $P$ and $Q$ at the intersection point. However, if we pass through the line at infinity to get from $P$ to $Q$, we additionally obtain a sign of $(\pm)^r$ since the analytic continuation of the natural orientation of $\R^d$ past the line at infinity gains a sign of $(-1)^{d+1} = (-1)^r$. (This sign is the reason that $\P^d(\R)$ is an orientable manifold if and only if $d$ is odd.). There is an additional sign coming from an orientation change in an even-codimensional blowup; see \cite[p.12]{MHhom} for details. In our language, this contributes a factor $\prod_i (-1)^{\rk(F_i)+1}$, and together we get the sign $(\pm)^r (-1)^{|I|}\prod_i (-1)^{\rk(F_i)+1} = (\pm)^r \prod_i (-1)^{\rk(F_i)}$. Putting everything together, we obtain the stated formula. \end{proof} \begin{example} Let $\bU = \C \setminus \{0,1\}$ and $P = Q = (0,1)$. Then $C^\vee(Q_Z)$ and $\reg(C(P))$ are pictured below: $$ \begin{tikzpicture} \draw[thick,decoration={markings, mark=at position 0.9 with {\arrow{>}}},postaction={decorate}] ([shift=(3:1)]0,0) arc (3:360:1); \draw[thick,decoration={markings, mark=at position 0.5 with {\arrow{>}}},postaction={decorate}] (1,0) -- (4,0); \draw[thick,decoration={markings, mark=at position 0.9 with {\arrow{>}}},postaction={decorate}] (4,0) arc (-180:177:1); lldraw (0,0) circle (1pt); lldraw (5,0) circle (1pt); \node[color=blue] at (0,-0.2) {$0$}; \node[color=blue] at (5,-0.2) {$1$}; \draw[thick,color=red,decoration={markings, mark=at position 0.5 with {\arrow{>}}},postaction={decorate}] (0,0) to [bend left = 45] (2.5,0); \draw[thick,color=red,decoration={markings, mark=at position 0.5 with {\arrow{>}}},postaction={decorate}] (2.5,0) to [bend right = 45] (5,0); \node[color=red] at (1.5,0.75) {$C^\vee(Q_Z)$}; \node[color=black] at (1.7,-0.5) {$\reg(C(P))$}; \end{tikzpicture} $$ The three intersection points (from left to right) give the three terms in $$ \gBip{C^\vee(Q), C(P)} = \frac{1}{b_0^2 -1} + 1 + \frac{1}{b_1^2 - 1}. $$ \end{example} \begin{remark} \cref{prop:Bettiint} also holds for any wonderful compactification $X = X_{\build}$. \end{remark} \subsection{Proof of \cref{thm:Bettipairmain}} The faces in the intersection $\bP \cap \bQ$ in the maximal wonderful compactification $X$ is the union of $\bG_\bullet$ for $G_\bullet \in G(P,Q)$. By \cref{prop:noover}(3), there is no overlap in $\bG_\bullet$ for $G_\bullet \in G(P,Q)$. By definition, we have $Q = Q_{G_\bullet}$, so $P$ and $Q$ are on opposite sides of each of the facets $G_i$. For any proper face $E_\bullet \in \bG_\bullet$, we have $Q \neq Q_{E_\bullet}$, so we must have that $P$ and $Q$ are on the same side of all the $E_i$ not belonging to $G_\bullet$. Thus the set $I$ in \cref{prop:Bettiint} corresponds to $G_\bullet$ and the set $L= I \cup J$ corresponds to $E_\bullet$. Comparing \cref{prop:Bettiint} with \cref{def:Bettipair}, we obtain \cref{thm:Bettipairmain}. \section{Twisted period relations}\label{sec:beta} \def\PP{{\bf P}} We continue to assume \eqref{eq:Mon}. Let $\comp_\a: H^\bullet(\bU,\nabla_\a) \to H^\bullet(\bU,\L_\a)$ denote the comparison map. Then for $x \in H^\bullet(\bU,\nabla_\a)$ and $y \in H^\bullet(\bU,\nabla_{-\a})$, we have \begin{equation}\label{eq:compip} (2\pi i)^{d} \gdRip{x,y}= \gDBip{\comp_\a(x), \comp_{-\a}(y)}. \end{equation} The factor of $(2\pi i )^d$ comes from the choice of normalization in \eqref{eq:PV}, and agrees, for instance, with the choice in \cite{BD}. Twisted periods are the pairings $$ (\comp_\a(\eta), [\Gamma \otimes \varphi_\Gamma]) = \int_\Gamma \varphi_\gamma \eta $$ between $\eta \in H^\bullet(\bU,\nabla_\a)$ and $[\Gamma \otimes \varphi_\Gamma] \in H_\bullet(\bU,\L^\vee_\a)$ obtained via the comparison map, and the natural duality $(\cdot, \cdot)$ between $H^\bullet(\bU,\L_\a)$ and $H_\bullet(\bU,\L^\vee_\a)$. Fixing bases of $H^\bullet(\bU,\nabla_\a)$ and $H_\bullet(\bU,\L^\vee_\a)$, we may express \eqref{eq:compip} as a matrix identity. For concreteness, we assume that we take the basis $\{\bOmega_P \mid P \in \T^0\}$ of $H^\bullet(\bU,\nabla_\a)$ and $\{C^\vee(P) \mid P \in \T^0\}$ of $H_\bullet(\bU,\L^\vee_\a)$. We furthermore identify $H^\bullet(\bU,\nabla_\a)$ with $H^\bullet(\bU,\nabla^\vee_\a)$ via \eqref{eq:rOSaa} and $H_\bullet(\bU,\L^\vee_\a)$ with $H_\bullet(\bU,\L_\a)$ via \cref{prop:isoB}. Let $\PP^\a$ denote the $\T^0 \times \T^0$ period matrix $$ \PP^\a_{P,Q} = \int_Q \varphi_Q \Omega_P. $$ Note that with our conventions $\PP^{-\a}$ is obtained from $\PP^\a$ by simply substituting $\a \mapsto -\a$. Substituting into \eqref{eq:compip}, we obtain the beautiful twisted period relations of Cho and Matsumoto~\cite{CM}: \begin{equation}\label{eq:CM} (2\pi i)^d \dRip{\cdot, \cdot}_{\T^0} = \PP^\a \ip{\cdot, \cdot}^B_{\T^0} (\PP^{-\a})^T = \PP^{-\a} \ip{\cdot, \cdot}^B_{\T^0} (\PP^{\a})^T, \end{equation} where $\ip{\cdot, \cdot}^B_{\T^0}$ is simply the inverse matrix to $\ip{\cdot, \cdot}^{\T^0}_B$, and we have used that both $ \dRip{\cdot, \cdot}_{\T^0}$ and $\ip{\cdot, \cdot}^B_{\T^0}$ are symmetric matrices. Let $$ \beta(s,t):= \int_0^1 x^s (1-x)^t \frac{dx}{x(1-x)} = \frac{\Gamma(s) \Gamma(t)}{\Gamma(s+t)} $$ denote the beta function. For the case of $\bU = \C \setminus \{0,1\}$, and $\T^0 = \{P = [0,1]\}$, we have $$ \gdRip{\bOmega_P,\bOmega_P} = \frac{1}{a_0} + \frac{1}{a_1}, \qquad \text{and} \qquad \gBip{P,P}= 1 + \frac{1}{b_0^2-1} + \frac{1}{b_1^2-1} = \frac{b^2_0b^2_1-1}{(b^2_0-1)(b^2_1-1)}, $$ $$ \PP^\a_{P,P} = \beta(a_0,a_1), \qquad \text{and} \qquad \PP^{-\a}_{P,P} = \beta(-a_0,-a_1) $$ and we obtain from \eqref{eq:CM} the quadratic relation for the beta function: $$ 2 \pi i (\frac{1}{a_0} + \frac{1}{a_1}) = \beta(-a_0,-a_1) \beta(a_0,a_1) \frac{2}{i} \frac{\sin(\pi a_0) \sin(\pi a_1)}{\sin(\pi(a_0+a_1))}, $$ where we have used that $$ \frac{b}{b^2-1} = \frac{i}{2}\frac{1}{\sin(\pi a)} \qquad \text{ when } \qquad b = \exp(-\pi i a). $$ Combining all our main theorems (\cref{thm:deRhamfan,thm:Bettifan,thm:dRpairmain,thm:Bettipairmain}), we obtain the following result. \begin{corollary} The twisted period relations of Cho and Matsumoto give an exact formula for the continuous Laplace transform in terms of the discrete Laplace transform (or for the discrete Laplace transform in terms of the continuous Laplace transform), with coefficients given by the period matrix (or its inverse). \end{corollary} Note that we also have the limit formula \cref{thm:limit} relating the deRham and Betti intersection forms, but the twisted period relations are of a different nature. \begin{remark} It would be interesting to understand the precise relation with the Todd class formulae relating lattice points in a polytope to the volume function; see for instance \cite[Theorem 8.9]{BaPo}. \end{remark} \part{Physics} \def\kin{{\Lambda_\C}} \def\Crit{{\rm Crit}} \section{Scattering amplitudes on very affine varieties}\label{sec:veryaffine} We refer the reader to \cite{LamModuli} for a more leisurely exposition of the material in this section. A very affine variety $U$ is a closed subvariety of an algebraic torus. The group $\C[U]^\times/\C^\times$ of units modulo scalars in the coordinate ring of $U$ is isomorphic to a lattice $\Lambda = \Lambda_U \cong \Z^{c}$. The \emph{intrinsic torus} $T = T_U$ is defined to be the torus with character lattice $\Lambda$. The very affine variety $U$ can always be embedded as a closed subvariety $U \hookrightarrow T$ of the intrinsic torus. We identify $\kin= \Lambda \otimes_\Z \C$ with the complex Lie algebra of the dual torus $T^\vee$. Choose a natural embedding $T \hookrightarrow \P^{c}$, and let $\{f_e \mid e \in E\}$ be a choice of homogeneous coordinates on $\P^c$, where $|E| = c+1$. A point $\a \in \kin$ is given by a collection of complex numbers $\{a_e, e\in E\}$ satisfying $\sum_{e} a_e =0$. For $\a \in \kin$, we have the multi-valued function $$ \varphi := f^\a = \prod_{e \in E} f_e^{a_e} $$ on $T$, which we may pullback to a multi-valued function on $U$. In the case of a projective hyperplane arrangement complement $\bU$ with hyperplanes $\{H_e \mid e \in E\}$, we make choices so that $f_e$ is a linear form vanishing on $H_e$. While the function $\varphi = f^\a$ is multi-valued, the 1-form $$ \omega = \dlog f^\a = \sum_{e\in E} a_e \dlog f_e = \sum_{e \in E} a_e \frac{d f_e}{f_e} $$ is a well-defined algebraic $d$-form on $U$. \begin{definition} The \emph{scattering equations} for $U$ are the critical point equations $$ \omega = \sum_{e \in E} a_e \frac{d f_e}{f_e} = 0 $$ on $U$. Let $\Crit(\omega) = \{\omega = 0\} \subset U$ denote the critical point locus. \end{definition} In the case of a hyperplane arrangement complement, the multi-valued function $f$ is known as the \emph{master function} by Varchenko \cite{Varbook}. When $\a \in \Lambda_\C$ is generic and $U$ is smooth, Huh \cite{Huh} showed that the critical point equations consist of $|\chi(U)|$ reduced points. For hyperplane arrangement complements, this was established by Varchenko \cite{Varcrit} in a special case, and by Orlik and Terao \cite{OT} in general. The following approach to scattering amplitudes was pioneered by Cachazo--He--Yuan \cite{CHYarbitrary}, who considered the case $U = M_{0,n+1}$, the configuration space of $n+1$ points on $\P^1$. Relations to likelihood geometry were studied in \cite{ST}. \begin{definition}\label{def:CHY} Let $U$ be a $d$-dimensional very affine variety and $\a$ be generic. Let $x_1,\ldots,x_d$ be local coordinates on $U$, and let $$ \Omega = h(x_1,\ldots,x_d) d^d \x ,\qquad \Omega' = h'(x_1,\ldots,x_d) d^d \x $$ be two rational $d$-forms on $U$. Then the $(\Omega,\Omega')$-amplitude of $U$ is defined to be $$ \A_U(\Omega,\Omega'):= \sum_p h(p) \det\left(\frac{\partial^2 \log \varphi}{\partial x_i \partial x_j}\right)_p^{-1}h'(p) $$ where the summation is over the critical points $p \in \Crit(\omega)$. \end{definition} While \cref{def:CHY} applies only when the parameters $\a$ are generic, the formula produces a rational function in $\a$ which we view as the amplitude. \cref{def:CHY} is a sum over critical points, and sometimes called a \emph{stationary phase formula}. For an arbitrary very affine variety $U$, we do not (currently) know of a natural candidate for rational forms $\Omega$ to use in \cref{def:CHY}. However, for a hyperplane arrangement complement, the canonical forms $\bOmega_P$ are natural choices. In the case $U = M_{0,n+1}$, the canonical forms $\bOmega_P$ are known as \emph{Parke-Taylor} forms; see \cite{LamModuli,AHLstringy,BD}. For higher configuration spaces, see \cite{CEZ,CEZ24}. \section{Amplitudes for matroids}\label{sec:amplitude} Let $\M$ be an oriented matroid. Recall that $R = \Z[\a] = \Z[a_e \mid e \in E]$ and $Q = \Frac(R)$. We let $R_0 = R/(a_E)$ and $Q_0 = \Frac(R_0)$. In this section, we view the deRham intersection forms as taking values in rational functions. \begin{definition}\label{def:matroidamp} For a tope $P \in \T$, we define the \emph{amplitude} $$ \A(P):= \bdRip{\Omega_P,\Omega_P} \in Q_0 $$ and for a pair of topes $P, Q \in \T$, we define the \emph{partial amplitude} $$ \A(P,Q):= \bdRip{\Omega_P,\Omega_Q} \in Q_0, $$ which is symmetric in $P$ and $Q$. \end{definition} \begin{theorem}\label{thm:CHY} When $\M$ is the oriented matroid of a projective hyperplane arrangement and $U$ is the hyperplane arrangement complement, \cref{def:CHY} and \cref{def:matroidamp} agree, that is, $\A(P,Q) = \A(\bOmega_P,\bOmega_Q)$. \end{theorem} In the scattering amplitudes literature, the relation between the stationary phase formula \cref{def:CHY} and twisted cohomology was first observed by Mizera \cite{Miz}. The formula was established rigorously in a general setting by Matsubara-Heo \cite{MHcoh}. Thus \cref{thm:CHY} can be deduced from \cref{thm:dRpairmain} and the results of \cite{MHcoh}. \cref{thm:CHY} can also be deduced from the results of Varchenko; for instance by using \cite[Theorem 3.1]{VarBethe}. \begin{remark} It would be interesting to develop an analogue of \cref{def:CHY} for the other intersection forms $\DdRip{\cdot,\cdot}, \halfip{\cdot,\cdot}_B, \halfip{\cdot,\cdot}^B$. \end{remark} For $F \in L(M)$ and $q \in Q_0$, let $$ \res_{a_F}(q) := (a_F q)|_{a_F = 0}, $$ if it is defined. When $\res_F(q)$ is defined, it belongs to $\Frac(R_0/(a_F))$. Let $P \in \T$ be a tope and let $\A(P)$ be the amplitude. We discuss some properties of the rational function $\A(P)$. Recall that $L(P) \subset L(M)$ is the face lattice of the tope $P$. For $F \in L(P)$, the tope $P$ restricts to the tope $P^F :=P|_F \in \T(\M^F)$, and contracts to the tope $P_F:=P|_{E \setminus F} \in \T(\M_F)$. \begin{theorem}\label{thm:AP} If $M$ is decomposable then $\A(P) = 0$ for all topes $P \in \T(\M)$. Otherwise, the poles of the rational function $\A(P)$ are all simple and can only be along $\{a_F = 0\}$ for each connected $F \in L(P) - \{\hat 0, \hat 1\}$. We have the recursion $$ \res_{a_F} \A(P)= \A(P|_F) \A(P|_{E \setminus F}). $$ \end{theorem} \begin{proof} If $M$ is decomposable, then $\beta(M) = 0$, and $\dim \OS(M,\omega) = 0$. Thus $\bdRip{\cdot,\cdot}$ is the $0$ form, and $\A(P) = 0$. Now, suppose that $M$ is connected. By \cref{thm:dRtope}, $ \A(P) = \sum_{F_\bullet \in \Fl(P)} \frac{1}{a_{F_\bullet}}. $ It is clear that the only possible poles of $\A(P)$ are along $a_F = 0$ for $F \in L(P)$. Suppose $F \in L(P) - \{\hat 0, \hat 1\}$. Then in each term of this sum, $\frac{1}{a_F}$ appears at most once as a factor. Thus $\res_{a_F} \A(P)$ is defined and given by $$ \res_{a_F} \A(P) = \sum_{F_\bullet \in \Fl(P) \mid F \in F_\bullet} \prod_{F_i \neq F} \frac{1}{a_{F_i}} = \sum_{F'_\bullet \in \Fl([\hat 0, F]), F''_\bullet \in \Fl([F,\hat 1])} \frac{1}{a_{F'_\bullet}} \frac{1}{a_{F''_\bullet}} = \A(P|_F) \A(P|_{E \setminus F}). $$ If $F$ is decomposable then $\A(P|_F)$ vanishes so $\res_{a_F} \A(P) =0$ and $\A(P)$ did not have a pole along $a_F$. \end{proof} \begin{remark} We do not know if $a_F$ is a pole of $\A(P)$ for every connected flat $F \in L(P)$. \end{remark} \begin{remark} The location of the poles of $\A(P)$ described in \cref{thm:AP} is called ``locality" in the theory of scattering amplitudes. The residue formula for $\A(P)$ is called ``unitarity". Thus \cref{thm:AP} states that matroid amplitudes satisfy locality and unitarity. \end{remark} \section{Scattering forms for matroids}\label{sec:scatform} In this section, we prove \cref{thm:CHY} using \emph{scattering forms}. Let $M$ be a matroid of rank $r$ on $E$. \subsection{Scattering forms on kinematic space} Let $\Lambda := \Spec(R_0)$ be the free abelian group generated by $\{a_e \mid e \in E\}$, with the relation $\sum_e a_e = 0$. In the case that $M$ is the matroid of a projective hyperplane arrangement complement $\bU$, this definition agrees with those in \cref{sec:veryaffine}. \begin{definition}\label{def:kinematic} The \emph{kinematic space} of $M$ is the complex vector space $\kin := \Lambda \otimes_\Z \C$. \end{definition} The vector space $\kin$ has dimension $|E|-1$. For an ordered basis $B= \{b_1,b_2,\ldots,b_r\}$, let $\iota_B: W_B \hookrightarrow \kin$ be a $(r-1)$-dimensional \emph{affine} subspace satisfying $da_e = 0$ for $e \notin B$. In other words, the functions $a_e$, $e \notin B$ are constant on $W_B$. Thus the function $a_B = \sum_{b \in B} a_e$ is also constant on $W_B$. The $1$-forms $\{da_b \mid b \in B\}$ span the cotangent space at every point of $W_B$, satisfying the single relation $\sum_{b \in B} da_b = 0$. The volume form of $W_B$ is the $d=(r-1)$-form, given by $$ \mu_{W_B}:= da_{b_d} \wedge \cdots \wedge da_{b_{1}}. $$ The volume form $\mu_{W_B}$ changes sign when $B$ is reordered. (The $b_i$ here should not be confused with the parameters $\b = (b_e, e \in E)$.) For a flag $F_\bullet \in \Fl(M)$, define a differential form $\eta_{F_\bullet}$ on $\kin$ by $$ \eta_{F_\bullet}:= \bigwedge_{i=1}^{r-1} \dlog a_F = \bigwedge_{i=1}^{r-1} \frac{d(a_{F_i})}{a_{F_i}} = \frac{d(a_{F_{r-1}})}{a_{F_{r-1}}} \wedge \cdots \wedge \frac{d(a_{F_1})}{a_{F_1}} . $$ \begin{definition} For an element $x \in \rOS(M)$, the \emph{scattering form} $\eta_x$ on $\kin$ is defined to be $$ \eta_x:= \sum_{F_\bullet \in \Fl(M)} \Res_{F^-_\bullet}(x) \eta_{F_\bullet}$$ where $F^-_\bullet \in \Fl^d(M)$ is obtained from $F_\bullet$ by ignoring the last step of the flag. \end{definition} \begin{proposition}\label{prop:iotaB} For any ordered basis $B$, the pullback $\iota_B^*(\eta_{x})$ is equal to $\dRipp{x, \partial e_{B}} \mu_{W_B}$. \end{proposition} \begin{proof} First we calculate the pullback $\iota_B^* \eta_{F_\bullet}$, which is a top-form on the $d$-dimensional affine space $W_B$. Since $da_e = 0$ for $e \notin B$, we have $$ da_{F_d} \wedge da_{F_{d-1}} \wedge \cdots \wedge da_{F_1} = da_{b_{\sigma(d-1)}} \wedge da_{b_{\sigma(d-2)}} \wedge \cdots \wedge da_{b_{\sigma(1)} }= r(B,F_\bullet) da_{b_{d-1}} \wedge \cdots \wedge da_{b_{1}} $$ where $\sigma$ is the permutation satisfying $F_i = \sp\{b_{\sigma(1)},\ldots,b_{\sigma(i)}\}$, and in the second equality we have also used $\sum_{b_i \in B} db_i = 0$. Thus $\iota_B^*(\eta_{F_\bullet}) = r(B,F_\bullet)\frac{1}{a_{F_\bullet}} \mu_{W_B}$. Suppose that $x = \partial e_{B'}$. Then using \cref{prop:dRind} and \cref{prop:dRpartial}, we have \begin{align*} \iota_B^*(\eta_{\partial e_{B'}}) &= \sum_{F_\bullet \in \Fl(M)} r(B',F_\bullet) \iota_B^*(\eta_{F_\bullet}) = \sum_{F_\bullet \in \Fl(M)} r(B',F_\bullet) \frac{1}{a_{F_\bullet}} r(B,F_\bullet) \mu_{W_B} = \dRipp{\partial e_{B'}, \partial e_{B}} \mu_{W_B}.\qedhere \end{align*} \end{proof} \subsection{Scattering forms for topes} Let $\M$ be an oriented matroid lifting $M$, and let $P \in \T(\M)$ be a tope. Then we have $$ \eta_P:= \eta(\bOmega_P) = \sum_{ F_\bullet \in \Fl(M)} r(P,F_\bullet) \eta_{F_\bullet}. $$ \begin{lemma}\label{lem:etaP} Let $P \in \T(\M)$ be a tope. Then the poles of $\eta_P$ are (only) along $\{a_F = 0\}$ for connected flats $F \in L(P) \setminus \{\hat 0, \hat 1\}$, and we have $$ \Res_{a_F = 0} \eta_P = (-1)^{\rk(F)} \eta_{P_F} \wedge \eta_{P^F} $$ as forms on $\{a_F = 0\}$. \end{lemma} \begin{proof} By definition, the only possible poles in $\eta_P$ are along $\{a_F = 0\}$ for flats $F \in L(P)$. For a flag $F_\bullet$ passing through $F$, write $F_\bullet = (F'_\bullet < F < F''_\bullet)$ for the parts of the flag before and after $F$. We have $$ \Res_{a_F = 0} \eta_{F_\bullet} = \begin{cases} (-1)^{\rk(F)} \eta_{F''_\bullet} \wedge \eta_{F'_\bullet} & \mbox{if $F_\bullet$ passes through $F$,} \\ 0 & \mbox{otherwise.} \end{cases} $$ The formula for $\Res_{a_F = 0} \eta_P$ now follows in the same way as in the proof of \cref{thm:AP}. \end{proof} A tope $P \in \T(\M)$ is called \emph{simplex-like} if we have $\bOmega_P = \partial e_B$ for some basis $B \in \B(M)$. Applying \cref{prop:iotaB} to $x = \bOmega_Q$, we have the following result. We have used \cref{prop:dRpartial} to express the answer in terms of the canonical forms $\Omega_P$ instead of $\bOmega_P$. \begin{corollary}\label{cor:simplexscat} For a tope $Q$ and a simplex-like tope $P$ satisfying $\bOmega_P = \partial e_B$, we have $$ \iota^*_B(\eta_Q) = \bdRip{\Omega_P,\Omega_Q}\mu_{W_B} $$ for any subspace $\iota_B: W_B \hookrightarrow \kin$ satisfying $da_e = 0$ for $e \notin B$. In particular, we have $$ \iota^*_B(\eta_P) = \bdRip{\Omega_P,\Omega_P}\mu_{W_B}. $$ \end{corollary} It would be interesting to remove the simplex-like condition in \cref{cor:simplexscat}. \begin{remark} The terminology ``kinematic space" comes from the special case of the moduli space $M_{0,n+1}$ of $(n+1)$-pointed rational curves, which corresponds to the graphic matroid $M(K_n)$ on the complete graph with $n$ vertices. We refer the reader to \cite{LamModuli} for an exposition targeted at mathematicians. Scattering forms were first defined in the $M_{0,n+1}$ setting in \cite{ABHY}, and studied further in \cite{AHLstringy}. \end{remark} \subsection{Scattering correspondence} We continue to assume that $\sum_e a_e = 0$. Let $\bU$ be a projective hyperplane arrangement complement with matroid $M$. The space $\Lambda_\C$ in \cref{sec:veryaffine} can be identified with the kinematic space in \cref{def:kinematic}. On the variety $\bU \times \kin$ we have the twist $1$-form $$ \omega = \sum_{e \in E} a_e \dlog f_e $$ which we now view as depending on both the coordinates $a_e$ on $\kin$, and the functions $f_e$ on $\bU$. \begin{definition}\label{def:scatcorr} The \emph{scattering variety} or \emph{critical point variety} $\I$ is the vanishing set in $\bU \times \kin$ of the twist $1$-form $\omega$ and fits into the scattering correspondence diagram \begin{equation}\label{eq:scatteringmap} \begin{tikzcd} &\I \arrow{rd}{q} \arrow[swap]{ld}{p} & \\\bU && \kin \end{tikzcd}. \end{equation} \end{definition} The space $\I$ was first studied in \cite{OT}. In \cite[Proposition 2.5]{CDFV}, it is shown that $p: \I \to \bU$ is a vector bundle. The scattering variety was studied in the setting of very affine varieties in \cite{HS, Huh}. On $\bU$ we have the canonical form $\bOmega_P$ for a tope $P \in \T^+(\M)$. In the following, we will use the notion of \emph{pushforward} of a rational form along a rational map $f: X \to Y$ of relative dimension $0$ between complex algebraic varieties. We refer the reader to \cite[Section 3.9]{LamModuli} for more details on pushforwards. The following result generalizes \cite[Theorem 3.26]{LamModuli}. \begin{theorem}\label{thm:etaP} We have the equality $ q_* p^* \bOmega_P = \eta_{P} $ of rational $d$-forms on $\kin$. \end{theorem} \begin{proof} \def\bI{{\overline{\I}}} \def\bp{\bar p} \def\bq{\bar q} We proceed by induction on $d$ and $|E|$. The claim reduces to that for simple matroids. The base case is the rank $1$ matroid with a single element. In this case, $\kin$ is a single point (a 0-dimensional vector space), and the result is trivial. Now suppose $d \geq 1$. Consider the maximal wonderful compactification $\pi: X= X_{\max} \to \P^d$ of $\bU$ and view $X \times \kin$ as a vector bundle over $X$. Let $Z = X \setminus \bU$. There is an evaluation map $$ \Psi: X \times \kin \to \Omega^1_X(\log Z), \qquad \Psi: (x,\a) \longmapsto \omega_\a(x), $$ sending $X \times \kin$ to the vector bundle $\Omega^1_X(\log Z)$ of logarithmic one-forms on $X$. Note that the statement that $\Omega^1_X(\log Z)$ is a vector bundle requires that $X$ is smooth with normal-crossing boundary divisor. By \cite[Proof of Theorem 3.8]{Huh}, the kernel $\bI:= \ker \Psi$ is a vector bundle on $X$ that coincides with the closure of the scattering correspondence $\I \subset \bU \times \kin \subset X \times \kin$. The diagram \eqref{eq:scatteringmap} sits inside the diagram \[ \begin{tikzcd} &\bI \arrow{rd}{\bq} \arrow[swap]{ld}{\bp} & \\ X && \kin \end{tikzcd} \] and it suffices to show that $\bq_* \bp^* \bOmega_P = \eta_P$. Let $\Theta:= \bp^* \bOmega_P$. Since $\bI$ is a vector bundle over $X$, the poles of $\Theta$ are the divisors $\bI|_D \subset \bI$ for each polar divisor $D \subset X$ of $\bOmega_P$. According to \cite{BEPV}, the poles of $\bOmega_P$ on $X$ are all simple and along the divisors $X_F$ for each flat $F \in L(P)$. The image $\bq(X_F)$ is contained in the subspace $\{a_F = 0\} \subset \kin$. Indeed, for the case $F = \{e\}$ is a single element, this is part of the ``geometric deletion-restriction formula" of Denham--Garrousian--Schulze \cite[Theorem 3.1]{DGS}. For a non-atom flat, the result follows in the same way after using \cref{lem:monod}. The map $\bq: \bI \to \kin$ is proper and surjective. According to the ``residue commutes with pushforward" result of Khesin and Rosly \cite[Proposition 2.5]{KR}, we have $$ \bq_* \Res_{\bI|_D} \Theta = \Res_{a_F = 0} \bq_* \Theta. $$ Brauner-Eur-Pratt-Vlad \cite{BEPV} show that the (analytic) closure $\bP$ of $P$ in $X$ is a positive geometry \cite{ABL} with canonical form $\Omega(\bP) :=\pi^*\bOmega_P$. The intersection of $X_F$ with $\bP$ is the product of corresponding (closures of) topes $ \bP_F \times \bP^F$ and has canonical form the product $ \Omega(\bP_F) \wedge \Omega(\bP^F)$ of the canonical forms, and \cite[Theorem 4.5]{BEPV} show that $\Res_{\bI|_D} \Theta = \bp^* \left(\bOmega_{P_F} \wedge \bOmega_{P^F}\right)$. We have an isomorphism of bundles over $X_F = X(M_F) \times X(M^F)$: $$ \bI|_{D_F} \cong \bI(M_F) \times \bI(M^F) . $$ (Note that even if $M$ is simple, the matroid $M_F$ may not be simple, and the kinematic space $\kin(M_F)$ is typically of a higher dimension than the kinematic space of its simplification.) By the inductive hypothesis applied to $M^F$ and $M_F$, we calculate \begin{align*} \bq_* \Res_{\bI|_D} \Theta &= \bq_* \bp^* \left( \bOmega_{P_F} \wedge \bOmega_{P^F} \right)= \pm \eta_{P_F} \wedge \eta_{P^F}, \end{align*} where $\eta_{P^F}$ (resp. $\eta_{P_F}$) is a differential form on the subspace $\kin|_{F}$ (resp. $\kin|_{E \setminus F}$), direct summands of the vector space $\{a_F = 0\} \subset \kin$. The sign $\pm$ can be calculated to be $(-1)^{\rk(F)}$ and is the same sign appearing in the proof of \cref{prop:Bettiint}. These are all the possible poles of $\bq_* \bp^* \bOmega_P$. By \cref{lem:etaP}, the poles and residues of $\bq_* \bp^* \bOmega_P$ and $\eta_P$ agree and it follows that they are equal since both forms are pullbacks of meromorphic forms on the projective space $\P(\kin)$. This proves the induction step, and the theorem. \end{proof} \begin{example} Let $d = 1$ and suppose $\bU = \C \setminus \{z_1,z_2,\ldots,z_n\}$, so that $E = \{1,2,\ldots,n\} \cup \{0\}$. The canonical form for $P = [z_e,z_{e+1}]$ is $\bOmega_P = \dlog (z-z_e) - \dlog (z-z_{e+1})$. We work in affine coordinates, using $a_1,a_2,\ldots,a_n$ as coordinates on $\kin$, eliminating $a_0 = -(a_1+\cdots+a_n)$. Then $$ \omega = \sum_{e=1}^n a_e \frac{dz}{z-z_e}, \qquad \text{and} \qquad \I = \{(z,\a) \mid p(z) := \sum_{e=1}^n \frac{a_e}{z-z_e} = 0\}. $$ Write $(z-z_e) p(z) = a_e + q(z)$, so that on $\I$ we have $$ 0 = da_e + q'(z) dz + \mbox{terms involving other $da_{e'}$}. $$ Thus $dz = -da_e/q'(z) + \text{other terms}$, and the definition of pushforward gives $$ q_*p^* \frac{dz}{z-z_f} = - \sum_{z_* \in \Crit(\omega_\a)} \frac{1}{q'(z_*) (z_*-z_f)} da_e + \mbox{terms involving other $da_{e'}$}. $$ Let $u(z):= (z-z_f)(a_e+q(z))$ which has zeroes at $z= z_f, \infty$ and $z= z_* \in \Crit(\omega_\a)$. The global residue theorem\footnote{We thank Simon Telen for explaining to us the use of global residue theorems for computing pushforwards.} for the rational function $1/u(z)$ states that the sum of the residues vanishes, which is the identity $$ \frac{1}{u'(z_f)} + \text{ residue at }\infty + \sum_{z_* \in \Crit(\omega_\a)} \frac{1}{u'(z_*)} = 0. $$ Now, the residue $r_\infty$ at $\infty$ does not depend on the choice of $f \in E \setminus 0$. Since $u'(z) = (a_e+q(z)) + (z-z_f)q'(z)$, we have $$ r_\infty + \frac{1}{a_e+q(z_f)} + \sum_{z_* \text{ roots }} \frac{1}{(z_*-z_f)q'(z_*)} = 0, $$ giving $$ \qquad - \sum_{z_* \text{ roots }} \frac{1}{(z_*-z_f)q'(z_*)} = r_\infty + \frac{1}{a_e + q(z_f)} = r_\infty + \delta_{e,f} \frac{1}{a_f}, $$ where $\delta_{e,f}$ is the Kronecker delta function. Thus the pushforward of $dz/(z-z_f)-dz/(z-z_g)$ is equal to $da_f/a_f -da_g/a_g$, agreeing with \cref{thm:etaP}. \end{example} \begin{remark} \cref{thm:etaP} is a variant of the results of \cite{ABL,AHLstringy} where the pushforward of the canonical form along the algebraic moment map of a toric variety is computed. \end{remark} \subsection{Proof of \cref{thm:CHY}} Recall the description of $p: \I \to \bU$ as a (trivial) vector bundle from \cite[Proposition 2.5]{CDFV}. Let $z_1,\ldots,z_d$ be coordinates on $\C^d$, and write $$ \omega = \sum_{i=1}^d h_i(\z) dz_i, \qquad \text{where} \qquad h_i(\z) = \sum_{e \in E\setminus 0} h_i^e(\z) a_e, $$ and $h_i^e(\z)$ are rational functions in $\z$, well-defined on $\bU$. The fiber $p^{-1}(\z)$ is the codimension $d$ subspace of $\kin$ cut out by the linear equations $h_1(\z) =0, h_2(\z)=0,\ldots,h_d(\z) = 0$. Now let $W \subset \kin$ be an affine subspace of dimension $d$. The condition for $W \cap p^{-1}(\z)$ to have positive dimension is the zero-set of a rational function in $\z$. Thus for $\z$ belonging to a dense subset $\bU' \subset \bU$, the intersection $W \cap p^{-1}(\z)$ will be a single point. We conclude that $q: \I \to \kin$ restricts to a rational map $q_W := q|_W: \bU \to W$. For a generic $W \subset \kin$, the rational map $q_W$ has degree equal to $\beta(M)$. This is the \emph{scattering map}; see \cite[Section 3.10]{LamModuli}. Since pushforwards and pullbacks of rational forms can be calculated on dense subsets, for a rational top-form $\Omega$ on $\bU$, we have $$ \iota_W^* q_* p^* \Omega = (q_W)_* p^* \Omega = (q_W)_* \Omega, \qquad \text{where} \qquad q_W : \bU \to W $$ and $\iota_W: W \hookrightarrow \kin$ denotes the inclusion. We work in affine coordinates. Let $B = \{b_1,b_2,\ldots,b_d,0\} \in \B(M)$ be a basis containing $0 \in E$, so that $\partial e_B = \be_{b_d} \wedge \be_{b_{d-1}} \wedge \cdots \wedge \be_{b_1}$. Let $W = W_B$ be a generic $d$-dimensional affine subspace such that $da_e = 0$ for $e \notin B$. We claim that \begin{equation}\label{eq:pushres} (q_W)_* \Omega = \A_\bU(\Omega,\partial e_B) \mu_{W_B} = \A_\bU(\Omega,\partial e_B) da_{b_d} \wedge \cdots \wedge da_{b_1}, \end{equation} where $\A_\bU(\Omega,\partial e_B)$ is defined in \cref{def:CHY}. At a point $\a \in W$, the left hand side is defined as the sum over pre-images $q_W^{-1}(\a) \in \bU$, which are exactly the critical points of $\omega_\a$ that the right hand side is defined as a sum over. The contribution of each critical point $\z \in q_W^{-1}(\z)$ to the coefficient of $\mu_{W_B}$ is the evaluation of a rational function. By a direct calculation, for both sides this rational function is equal to $$ \det\left(\frac{\partial^2 \log \varphi}{\partial f_{b_i} \partial f_{b_j}}\right)^{-1} \times \frac{\Omega}{\partial e_B}. $$ See also \cite[Lemma 3.19 and Proposition 2.23]{LamModuli}. Combining \eqref{eq:pushres} with \cref{cor:simplexscat} and \cref{thm:etaP}, we obtain $\bdRip{\Omega_P, e_B} = \A_{\bU}(\bOmega_P,\partial e_B)$ for any tope $P$ and any basis $B$. Extending by linearity, we see that $\A(P,Q) = \A_\bU(\bOmega_P,\bOmega_Q)$, so the two definitions of amplitudes agree. \section{Configuration space of $n+1$ points on $\P^1$}\label{sec:M0n} \subsection{Complete graphic matroid} Let $M$ be the graphic matroid of the complete graph $K_n$, on the ground set $E = \{(i,j) \mid 1 \leq i < j < n\}$ of edges, where we identify $(j,i)$ with $(i,j)$. The lattice of flats $L(M)$ is the partition lattice $\Pi_n$. The elements of $\Pi_n$ are the set partitions of $[n]$. For two set partitions $\pi, \pi'$, we have $\pi \leq\pi'$ if $\pi$ refines $\pi'$. As a flat, the set partition $\pi$ corresponds to the following subset of $E$: $$ \pi = \{(i,j) \in E \mid i \text{ and } j \text{ belong to the same block of }\pi\}. $$ For example, the set partition $\pi = (145|26|3)$ of $\{1,2,3,4,5,6\}$ can be viewed as the collection of edges $\{(1,4),(1,5),(4,5),(2,6)\}$. Let $\M$ be the oriented graphic matroid associated to the orientation where $(i,j)$ is oriented $i \to j$ for $i < j$. The oriented matroid $\M$ arises from the braid arrangement $\B_n$, consisting of the hyperplanes $H_{i,j} = \{z_i - z_j = 0 \mid 1 \leq i < j < n\}$ in $\R^n$. The braid arrangement is not essential, and instead we prefer to consider $\M$ as arising from the affine graphic hyperplane arrangement $\bA \subset \R^{n-2} = \{(z_2,\ldots,z_{n-1})\}$ with hyperplanes $$ \{z_i = 0 \mid i = 2,3,\ldots,n-1\} \cup \{z_i = 1 \mid i=2,3,\ldots,n-1\} \cup \{z_i - z_j = 0 \mid 2 \leq i < j \leq n-1\}. $$ See \cref{fig:M05}. In other words, the coordinate $z_1$ has been set to $0$ and the coordinate $z_n$ has been set to $1$. The hyperplane at infinity should be thought of as $\{z_1 - z_n = 0\}$. We still denote the $\binom{n}{2}$ hyperplanes as $H_{ij}$. The parameters $a_e$ will be taken to be $\{a_{(i,j)} \mid 1 \leq i < j \leq n\}$. The following result gives a standard description of $M_{0,n+1}$, the \emph{configuration space of $n+1$ points on $\P^1$}, also known as the \emph{moduli space of rational curves with $n+1$ marked points}; see for instance \cite{LamModuli}. \begin{figure} \begin{center} $$ \begin{tikzpicture}[scale=1.5,extended line/.style={shorten >=-#1,shorten <=-#1}, extended line/.default=1cm] \draw (0,-1) -- (0,3); \draw (1,-1) -- (1,3); \draw (-1,0) -- (3,0); \draw (-1,1) -- (3,1); \draw(-1,-1) -- (3,3); \node at (0,3.2) {$z_2 = 0$}; \node at (1,3.2) {$z_2 = 1$}; \node at (3,0.15) {$z_3 = 0$}; \node at (3,1.15) {$z_3 = 1$}; \node at (3,3.2) {$z_2 = z_3$}; \node[color=blue] at (0.2,2.8) { $(12)$}; \node[color=blue] at (1.2,2.8) { $(14)$}; \node[color=blue] at (3.1,2.8) { $(23)$}; \node[color=blue] at (2.8,-0.15) { $(13)$}; \node[color=blue] at (2.8,.85) { $(34)$}; \node[color=red] at (-0.5,2) {$2143$}; \node[color=red] at (0.5,2) {$1243$}; \node[color=red] at (1.5,2) {$1423$}; \node[color=red] at (2.5,2) {$1432$}; \node[color=red] at (2,0.5) {$1342$}; \node[color=red] at (0.6,0.25) {$1324$}; \node[color=red] at (0.3,0.65) {$1234$}; \node[color=red] at (-0.5,0.5) {$2134$}; \node[color=red] at (2,-0.5) {$3142$}; \node[color=red] at (0.5,-0.5) {$3124$}; \node[color=red] at (-0.4,-0.75) {$3214$}; \node[color=red] at (-0.7,-0.3) {$2314$}; \end{tikzpicture} $$ \end{center} \caption{The configuration space $M_{0,5}$ of $5$ points on $\P^1$ as a hyperplane arrangement complement.} \label{fig:M05} \end{figure} \begin{proposition} For the affine graphic arrangement $\bA$, the complement $\bU$ is isomorphic to $M_{0,n+1}$. \end{proposition} The set $\T^+$ of positive topes has cardinality $n!/2$ and can be indexed by permutations $\sigma \in S_n$ satisfying $\sigma^{-1}(1) < \sigma^{-1}(n)$. For example, for $n = 4$, we have 12 chambers, as shown in \cref{fig:M05}. For a permutation $\sigma = \sigma(1) \sigma(2) \cdots \sigma(n)$, the tope $P_\sigma$ satisfies $$ P_{\sigma}((i,j)) = \begin{cases} + & \mbox{if $\sigma^{-1}(i) < \sigma^{-1}(j)$,} \\ - & \mbox{if $\sigma^{-1}(i) > \sigma^{-1}(j)$.} \end{cases} $$ Geometrically, $P_\sigma$ is the region in $\R^{n-2}$ satisfying $$ z_{\sigma(1)} < z_{\sigma(2)} < \cdots < z_{\sigma(n)}, \qquad \mbox{where $z_1 = 0$ and $z_n = 1$.} $$ The facets of the tope $P_{\sigma}$ are the $n-1$ hyperplanes $H_{(\sigma(i),\sigma(i+1))}$, $i=1,2,\ldots,n-1$. The lattice of positive flats $L(P)$ associated to $P_\sigma$ is isomorphic to the boolean lattice, and consists of the set partitions \begin{equation}\label{eq:pisigma} \pi_{c_1,c_2,\ldots, c_{r-1}} = \{\sigma(1),\ldots,\sigma(c_1)| \sigma(c_1+1),\ldots,\sigma(c_2)| \cdots | \sigma(c_{r-1}+1),\ldots,\sigma(n)\} \end{equation} for $1 \leq c_1 < c_2 < \cdots < c_{r-1} < n$. \begin{lemma} The set of bounded topes $\T^0$ has cardinality $(n-2)!$ and are indexed by permutations $\sigma$ satisfying $\sigma(1) = 1$ and $\sigma(n) = n$. \end{lemma} \begin{proof} It follows from \eqref{eq:pisigma} that the only way for $(1,n)$ not to belong to any of the flats in $L(P)\setminus \hat 1$ is to have $\sigma(1) = 1$ and $\sigma(n) = n$. \end{proof} Let us also pick a general extension $\star$, given by the hyperplane $H_\star = \{z_2 + z_3 + \cdots + z_{n-1} = - \epsilon < 0\}$, pictured in \cref{fig:M05star}. With this choice of extension, we have \begin{lemma}\label{lem:Knstar} The set of bounded topes $\T^\star$ has cardinality $(n-1)!$ and is indexed by permutations $\sigma$ satisfying $\sigma(1) = 1$. \end{lemma} \begin{proof} Suppose that $\sigma(1) = 1$. Then in $P_\sigma$ we have $z_i > z_1 = 0$ for $i=2,\ldots,n-1$. It follows that $P_\sigma$ does not intersect $H_\star$ and belongs to $\T^\star$. Conversely, if $\sigma(1) > 1$, then there is clearly a point $z^+ \in P_\sigma$ such that $z^+_2 + z^+_3 + \cdots + z^+_{n-1} > - \epsilon$. By decreasing $z^*_{\sigma(1)}$ a large amount, there is also a point $z^- \in P_\sigma$ such that $z^+_2 + z^+_3 + \cdots + z^+_{n-1} < - \epsilon$. Thus $P_\sigma \notin \T^\star$ if $\sigma(1) > 1$. \end{proof} \subsection{Temporal Feynman diagrams} We shall use a reindexing of variables that is common in physics: for $A \subset [n]$, write $$ s_A := \sum_{(i<j) \in A} a_{(i,j)} $$ so that, for example, $s_{ij} = a_{(i,j)}$ and $s_{ijk} = a_{(i,j)} + a_{(i,k)} + a_{(j,k)}$. A \emph{planar tree} on $[n+1]$ is a planarly embedded tree $T$ with leaves labeled $1,2,\ldots,n+1$ in cyclic clockwise order. A planar tree $T$ is called \emph{cubic} if all non-leaf vertices have degree three. An edge $e$ of $T$ is called \emph{internal} if it is not incident to any of the leaves. Let $\prec$ be the partial order on the internal edges $I(T)$ of $T$ such that $e \preceq e'$ if and only if the path from $e$ to $n+1$ passes through $e'$. Denote by $\T_{n+1}$ the set of planarly embedded trees on $[n+1]$ and $\T^{(3)}_{n+1}$ the subset of cubic planar trees. Define a partial order $<$ on $\T_{n+1}$ by $T \leq T'$ if $T$ can be obtained from $T'$ be contracting internal edges. Then the maximal elements of $\T_{n+1}$ are the cubic planar trees and $\T_{n+1}$ is isomorphic to the dual of the face poset of the $(n-2)$-dimensional associahedron. A \emph{temporal Feynman diagram} is a pair $(T,\leq)$ consisting of a cubic planar tree $T$ together with a total ordering $<$ of the internal edges such that if $e \preceq e'$ then $e \leq e'$. It is easy to see that the number of temporal Feynman diagrams is equal to $(n-1)!$. To each edge $e$ of a cubic planar tree let $S_e \subset E = \{1,2,\ldots,n\}$ denote the set of leaves in the component of $T \setminus e$ \emph{not} containing $n+1$. Note that $S_e$ is always an interval in $[n]$. Define $$ F(S_e) = \{(i,j) \mid i < j \text{ and } i,j \in S_e\}. $$ To each temporal Feynman diagram $(T,\leq)$ and internal edge $e \in I(T)$ of $T$, let $I_{\leq e} := \{e' \in I \mid e' \leq e\}$ and $$ D(e):= \{e' \in I_{\leq e} \mid e' \text{ is } \preceq \text{ maximal in } I_{\leq e}\}. $$ Define \begin{equation}\label{eq:Fe} F_\leq(e) := \bigcup_{e' \in D(e)} F(S_{e'}). \end{equation} It is easy to see that $F(e')$ for $e' \in D(e)$ are disjoint intervals, so that \eqref{eq:Fe} is a disjoint union. For an internal edge $e$, we define the (usual) \emph{propagator} $X_e$ and the \emph{temporal propagator} $Y_e$ by $$ X_e := \sum_{(i,j) \in F(S_e)} a_{(i,j)} = s_{S_e}, \qquad \text{ and } \qquad Y_e := \sum_{(i,j) \in F_\leq(e)} a_{(i,j)} = \sum_{e' \in D(e)} X_{e'} = \sum_{e' \in D(e)} s_{S_{e'}}. $$ \begin{remark} We may interpret temporal Feynman diagrams as a particle scattering process. Let $(T, \leq)$ be a temporal Feynman diagram. We orient every edge of $T$ towards the leaf $n+1$, viewing $T$ as a scattering process where $n$ incoming particles $1,2,\ldots,n$ produce a single outgoing particle $n+1$. At each cubic vertex of $T$, two incoming particles collide to produce a single outgoing particle. The data of $\leq$ is a total ordering on when these cubic collisions occur, illustrated in \cref{fig:temporal}. \begin{figure} \begin{center} \begin{tikzpicture} \begin{scope}[decoration={ markings, mark=at position 0.5 with {\arrow{>}}} ] \draw[decoration={markings, mark=at position 0.5 with {\arrow{>}}},postaction={decorate}] (0,5) -- (1,4.5); \draw[decoration={markings, mark=at position 0.5 with {\arrow{>}}},postaction={decorate}] (0,4) -- (1,4.5); \draw[decoration={markings, mark=at position 0.5 with {\arrow{>}}},postaction={decorate}] (1,4.5) -- (3,3.5); \draw[decoration={markings, mark=at position 0.5 with {\arrow{>}}},postaction={decorate}] (0,3) -- (3,3.5); \draw[decoration={markings, mark=at position 0.5 with {\arrow{>}}},postaction={decorate}] (3,3.5) -- (4,3); \draw[decoration={markings, mark=at position 0.5 with {\arrow{>}}},postaction={decorate}] (4,3) -- (5,3); \draw[decoration={markings, mark=at position 0.5 with {\arrow{>}}},postaction={decorate}] (0,2) -- (2,2); \draw[decoration={markings, mark=at position 0.5 with {\arrow{>}}},postaction={decorate}] (0,1) -- (2,2); \draw[decoration={markings, mark=at position 0.5 with {\arrow{>}}},postaction={decorate}] (2,2) -- (4,3); \node[color=blue] at (-0.2,1) {$1$}; \node[color=blue] at (-0.2,2) {$2$}; \node[color=blue] at (-0.2,3) {$3$}; \node[color=blue] at (-0.2,4) {$4$}; \node[color=blue] at (-0.2,5) {$5$}; \node[color=blue] at (5.2,3) {$6$}; \node[color=red] at (2.1,4.15) {$a$}; \node[color=red] at (3,2.2) {$b$}; \node[color=red] at (3.5,3.4) {$c$}; \draw[dashed,color=purple] (3.3, 5)--(3.3,1); \node[text width=7.5cm] at (10,4.2) {The three internal edges are ordered $a < b < c$ according to the order the corresponding particle is produced.}; \node[text width=7.5cm] at (10,1.8) {The dashed vertical line intersects the two internal edges $b$ and $c$. The temporal propagator $Y_c$ is equal to $X_b+ X_c$.}; \end{scope} \end{tikzpicture} \end{center} \caption{A temporal Fenyman diagram can be viewed as a recording of a particle scattering process.} \label{fig:temporal} \end{figure} The usual propagator $X(e)$ has the physical interpretation as the square of the momentum along the internal edge $e$. The temporal propagator $Y(e)$ is the sum of squares of momenta traveling along internal edges immediately after the particle traveling along $e$ has been produced. \end{remark} Let $P_{\id}$ denote the chamber indexed by the identity permutation. \begin{definition} The $(n+1)$-point planar $\phi^3$-amplitude is defined to be $A_{n+1}^{\phi^3} = \A(P_\id)$. \end{definition} The amplitude $\A(P_\id)$ is also commonly called the \emph{biadjoint scalar amplitude}. By \cref{thm:CHY}, it can be computed using the formalism of scattering equations (\cref{def:CHY}), and is the simplest and most fundamental of the class of amplitudes originally considered by Cachazo-He-Yuan. \begin{theorem}\label{thm:temporal} The $(n+1)$-point planar $\phi^3$-amplitude is given by $$ A_{n+1}^{\phi^3} = \sum_{(T,\leq)} \prod_{e \in I(T)} \frac{1}{Y_e}, $$ summed over temporal Feynman diagrams $(T,\leq)$ on $[n+1]$. \end{theorem} \begin{proof} We apply \cref{thm:dRtope} to \cref{def:matroidamp}. The Las Vergnas face lattice $L(P_\id) \subset \Pi_n$ is the subposet consisting of set partitions $\pi$ where every block is an interval. These are exactly the set partitions that can be written by inserting dividers into the identity permutation. A flag of flats $F_\bullet \in \Fl(P_\id)$ then corresponds to a temporal Feynman diagram in a natural way: each step in the flag $F_\bullet$ produces a cubic vertex. For example, the temporal Feynman diagram of \cref{fig:temporal} corresponds to the flag \begin{equation*} (1|2|3|4|5) < (1|2|3|45) < (12|3|45)< (12|345) < (12345). \qedhere \end{equation*} \end{proof} \begin{example} Consider $n=4$, corresponding to $M_{0,5}$. The six temporal Feynman diagrams are illustrated here,, where ${\bf 1}, {\bf 2}$ are used to indicate orderings on internal edges: $$ \begin{tikzpicture}[scale=0.7] \coordinate (A) at (0,0); \coordinate (B) at (1,0); \coordinate (C) at (1.5,-0.866); \node (L1) at (-0.5,-0.866) {$1$}; \node (L2) at (-0.5,+0.866) {$2$}; \node (L3) at (1.5,+0.866) {$3$}; \node (L4) at (2.5,-0.866) {$4$}; \node (L5) at (1,-2*0.866) {$5$}; \draw[thick] (A) --(B)--(C)--(L4); \draw[thick] (L1)--(A)--(L2); \draw[thick] (L3)--(B); \draw[thick] (C)--(L5); \begin{scope}[shift={(4,-0.866)}] \coordinate (A) at (0,0); \coordinate (B) at (1,0); \coordinate (C) at (1.5,+0.866); \node (L1) at (-0.5,-0.866) {$1$}; \node (L2) at (-0.5,+0.866) {$2$}; \node (L3) at (1,1.732) {$3$}; \node (L4) at (2.5,+0.866) {$4$}; \node (L5) at (1.5,-0.866) {$5$}; \draw[thick] (A) --(B)--(C)--(L4); \draw[thick] (L1)--(A)--(L2); \draw[thick] (L3)--(C); \draw[thick] (B)--(L5); \node[align=left] at (0.5,-0.2) {$\bf 1$}; \node[align=left] at (1.4,0.3) {$\bf 2$}; \end{scope} \begin{scope}[shift={(8,-0.866)}] \coordinate (A) at (0,0); \coordinate (B) at (1,0); \coordinate (C) at (1.5,+0.866); \node (L1) at (-0.5,-0.866) {$1$}; \node (L2) at (-0.5,+0.866) {$2$}; \node (L3) at (1,1.732) {$3$}; \node (L4) at (2.5,+0.866) {$4$}; \node (L5) at (1.5,-0.866) {$5$}; \draw[thick] (A) --(B)--(C)--(L4); \draw[thick] (L1)--(A)--(L2); \draw[thick] (L3)--(C); \draw[thick] (B)--(L5); \node[align=left] at (0.5,-0.2) {$\bf 2$}; \node[align=left] at (1.4,0.3) {$\bf 1$}; \end{scope} \begin{scope}[shift={(12,0)}] \coordinate (A) at (0,0); \coordinate (B) at (1,0); \coordinate (C) at (1.5,-0.866); \node (L5) at (-0.5,-0.866) {$5$}; \node (L1) at (-0.5,+0.866) {$1$}; \node (L2) at (1.5,+0.866) {$2$}; \node (L3) at (2.5,-0.866) {$3$}; \node (L4) at (1,-2*0.866) {$4$}; \draw[thick] (A) --(B)--(C)--(L3); \draw[thick] (L5)--(A)--(L1); \draw[thick] (L2)--(B); \draw[thick] (C)--(L4); \end{scope} \begin{scope}[shift={(16,-0.866)}] \coordinate (A) at (0,0); \coordinate (B) at (1,0); \coordinate (C) at (1.5,+0.866); \node (L5) at (-0.5,-0.866) {$5$}; \node (L1) at (-0.5,+0.866) {$1$}; \node (L2) at (1,1.732) {$2$}; \node (L3) at (2.5,+0.866) {$3$}; \node (L4) at (1.5,-0.866) {$4$}; \draw[thick] (A) --(B)--(C)--(L3); \draw[thick] (L5)--(A)--(L1); \draw[thick] (L2)--(C); \draw[thick] (B)--(L4); \end{scope} \begin{scope}[shift={(20,0)}] \coordinate (A) at (0,0); \coordinate (B) at (1,0); \coordinate (C) at (1.5,-0.866); \node (L4) at (-0.5,-0.866) {$4$}; \node (L5) at (-0.5,+0.866) {$5$}; \node (L1) at (1.5,+0.866) {$1$}; \node (L2) at (2.5,-0.866) {$2$}; \node (L3) at (1,-2*0.866) {$3$}; \draw[thick] (A) --(B)--(C)--(L2); \draw[thick] (L5)--(A)--(L4); \draw[thick] (L1)--(B); \draw[thick] (C)--(L3); \end{scope} \end{tikzpicture} $$ The six temporal Feynman diagrams give the expression \begin{align*} A_5^{\phi^3} &= \frac{1}{a_{12}(a_{12}+a_{13}+a_{23})} + \frac{1}{a_{12}(a_{12}+a_{34})} + \frac{1}{a_{34}(a_{12}+a_{34})} \\ &\;\;+ \frac{1}{a_{34}(a_{23}+a_{24}+a_{34})} + \frac{1}{a_{23}(a_{23}+a_{24}+a_{34})} + \frac{1}{a_{23}(a_{12}+a_{13}+a_{23})} \\ &= \frac{1}{s_{12}s_{123}} + \frac{1}{s_{12}(s_{12}+s_{34})} + \frac{1}{s_{34}(s_{12}+s_{34})} + \frac{1}{s_{34}s_{234}}+ \frac{1}{s_{23}s_{234}} + \frac{1}{s_{23}s_{123}} \end{align*} which equals the usual sum over five Feynman diagrams (or cubic planar trees) \begin{align*} A_5^{\phi^3} &= \frac{1}{a_{12}(a_{12}+a_{13}+a_{23})} + \frac{1}{a_{12}a_{34}} + \frac{1}{a_{23}(a_{23}+a_{24}+a_{34})} + \frac{1}{a_{23}(a_{12}+a_{13}+a_{23})} + \frac{1}{a_{12}(a_{12}+a_{13}+a_{23})} \\ &= \frac{1}{s_{12}s_{123}} + \frac{1}{s_{12}s_{34}} + \frac{1}{s_{34}s_{234}}+ \frac{1}{s_{23}s_{234}} + \frac{1}{s_{23}s_{123}} \end{align*} \end{example} \begin{lemma} A flat $\pi$ of $L(M(K_n))$ is connected exactly when it has one non-singleton block. \end{lemma} Applying \cref{thm:deRhamfannested} with the choice of minimal building set (see \cref{sec:building}), we find that $\A(P_\id)$ is also given by a sum over cubic planar trees, giving the alternative formula $$ A_{n+1}^{\phi^3} = \sum_{T \in \T^{(3)}_{n+1}} \prod_{e \in I(T)} \frac{1}{X_e}. $$ This formula is the usual definition of the planar $\phi^3$-amplitude. For brevity, we do not discuss the partial amplitudes $\A(P,Q)$. \subsection{KLT matrix} The KLT (Kawai-Lewellen-Tye) relation is an important identity in the physics of scattering amplitudes. In its original formulation in \cite{KLT}, it expresses closed string amplitudes in terms of open string amplitudes via a \emph{string theory KLT kernel}. Later, a \emph{field theory KLT kernel} was studied, relating gravity amplitudes to gauge theory amplitudes. We show that our formula (\cref{thm:dRmain}) for the inverse of $\dRip{\cdot,\cdot}$ recovers the field theory KLT kernel in its formulation in \cite[(2.8)]{Frost}; see also \cite{BDSV,FMM}. \begin{theorem}\label{thm:Frost} Let $\id = 1 2 \cdots (n-1) n$ and $\sigma = 1 \sigma(2) \cdots \sigma'(n-1) \sigma(n)$. With the choices in \cref{lem:Knstar}, we have \begin{equation}\label{eq:Frost} \DdRip{P_\id,P_{\sigma}} = \prod_{j = 2}^{n} (a_{(1,j)} + \sum_{k \in [2,j-1] \mid \sigma^{-1}(k) < \sigma^{-1}(j)} a_{(k,j)}). \end{equation} \end{theorem} Each basis $B \in \B(M)$ can be identified with a spanning tree $T(B)$ on $[n]$ whose edges are the elements of $B$. The following observation is straightforward. \begin{lemma}\label{lem:treeB} With the choices in \cref{lem:Knstar}, we have $P_\sigma \in \T^B$ if and only if for each distinct $i, j \in [2,n-1]$ such that $i$ belongs to the path from $j$ to $1$ in $T(B)$ we have $\sigma^{-1}(i) < \sigma^{-1}(j)$. \end{lemma} \begin{proof}[Proof of \cref{thm:Frost}] Expanding the right hand side of \eqref{eq:Frost} each term is a monomial in $a_{(i,j)}$ of degree $n-1$. Since each index $i \in [n]$ appears, this monomial corresponds to a base $B \in \B(M)$ and a tree $T(B)$. By \cref{def:DdR}, we need to show that these are exactly the trees $T(B)$ such that $P_\id,P_{\sigma} \in \T^B$. By \cref{lem:treeB} applied to $P_\id$, any such tree has the property that if the path from $j$ to $1$ contains $i$ then $i < j$. Each such tree has the property that every vertex $i \in [2,n]$ is connected to exactly one vertex $j < i$, and therefore correspond exactly to the expansion of $$ \prod_{j = 2}^{n} (a_{(1,j)} + \sum_{k \in [2,j-1] } a_{(k,j)}). $$ Of these trees $T(B)$, the ones satisfying $P_{\sigma} \in \T^B$ are given by the right hand side of \eqref{eq:Frost}, again by \cref{lem:treeB}. \end{proof} Our \cref{thm:dRmain} recovers the well-known relation between partial scalar amplitudes and the KLT kernel; see for example \cite[Proposition 2.2]{Frost} or \cite{MizKLT}. See \cref{ex:KLTexample}. \subsection{String theory KLT matrix and $\alpha'$-corrected amplitudes} The Betti homology intersection pairings $\ip{P,Q}_B$ are called \emph{$\alpha'$-corrected amplitudes} in \cite{MizKLT}. They were studied from a mathematical point of view in \cite{MHhom}. A \emph{temporal planar tree} is a pair $(T, \leq)$ where $T \in \T_{n+1}$ is a planar tree on $[n+1]$ and $\leq$ is a total ordering of the internal edges. Let $\T_{n+1,\leq}$ denote the set of temporal planar trees on $[n+1]$. Our formula \cref{def:Bettipair} for $\ip{P,P}_B$ is likely novel, and gives: $$ \ip{P,P}_B = \sum_{(T,\leq) \in \T_{n+1,\leq}}\prod_{e \in I(T)} \frac{1}{b^2_{F_\leq(e)}-1}. $$ The formula in \cite{MizKLT,MHhom} is obtained by taking the minimal building set for $L(M(K_n))$: $$ \ip{P,P}_B = \sum_{T \in \T_{n+1}}\prod_{e \in I(T)} \frac{1}{a_{F_\leq(e)}} = \sum_{T \in \T_{n+1}}\prod_{e \in I(T)} \frac{1}{b^2_{F(e)}-1}. $$ Frost \cite{FroAss} interpreted this in terms of a sum over lattice points, analogous to our \cref{thm:Bettifan}. The Betti cohomology intersection pairing $\bip{\cdot,\cdot}^B$ is known as the \emph{string theory KLT matrix}. The intersection form matrix $\ip{\cdot,\cdot}^B_{\T^+}$ of \cref{thm:Bettiinverse} has dimensions $n!/2 \times n!/2$, and as such differs from the dimensions of the usual KLT matrix in the literature (which has dimension $(n-1)!$ or $(n-2)!$). Our matrix $\ip{\cdot,\cdot}^B_{\T^+}$, for a one-dimensional arrangement, appeared in \cite[Section 6.4.3]{BMRS} under the name of ``overcomplete form of KLT relations". \subsection{Determinants} We list some of the determinantal formulae obtained for various amplitudes. We have $\mu(M(K_n)) =(n-1)!$ and $\beta(M(K_n)) = (n-2)!$. A set partition $\pi$ with multiple non-singleton blocks is a decomposable flat and $\beta(\pi) = 0$. For a set partition $\pi_S$ with a one non-singleton block $S$, the interval $[\hat 0, \pi_S]$ is isomorphic to $\Pi_{|S|}$, so $\beta(\pi) = (|S|-2)!$. The interval $[\pi_S,\hat 1] \subset \Pi_n$ is isomorphic to $\Pi_{n-|S|+1}$, so $\mu(M_{\pi_S}) = (n-|S|)!$. From \cref{thm:Aomotodet} and \cref{thm:SVdet} we obtain the following determinantal formulae. \begin{corollary}\label{cor:det2} The determinant of the $(n-2)! \times (n-2)!$ matrix $\A(P_\sigma,P_{\sigma'})$ where $\sigma,\sigma'$ satisfy $\sigma(1) =\sigma'(1) = 1$ and $\sigma(n) =\sigma'(n) = n$ is given by $$ \pm \frac{\prod_{\{1,n\} \subset S \subsetneq [n]}{a_{F(S)}^{(n-|S|-1)! (|S|-2)!}}}{\prod_{\substack{S \subseteq [n] \\ |S| \geq 2 \text{ and } |S \cap \{1,n\}|\leq 1}}{a_{F(S)}^{(n-|S|-1)!(|S|-2)!}}}. $$ \end{corollary} \begin{corollary}\label{cor:det1} The determinant of the $(n-1)! \times (n-1)!$ matrix $\dRip{\Omega_{P_\sigma},\Omega_{P_{\sigma'}}}$ where $\sigma,\sigma'$ satisfy $\sigma(1) =\sigma'(1) = 1$ is given by $$ \frac{a^{(n-1)!-(n-2)!}_{F([n])} }{\prod_{S \subsetneq [n] \mid |S| \geq 2} a_{F(S)}^{(n-|S|)! (|S|-2)!}}. $$ \end{corollary} \begin{example} Let $n = 4$, corresponding to $M_{0,5}$. Then \cref{cor:det2} and \cref{cor:det1} give the determinants of a $2 \times 2$ and a $6 \times 6$ matrix. $$ \begin{bmatrix} \frac{1}{s_{12} s_{123}}+\frac{1}{s_{12} s_{34}}+\frac{1}{s_{123} s_{23}}+\frac{1}{s_{23} s_{234}}+\frac{1}{s_{234} s_{34}} & -\frac{1}{s_{123} s_{23}}-\frac{1}{s_{23} s_{234}} \\ -\frac{1}{s_{123} s_{23}}-\frac{1}{s_{23} s_{234}} & \frac{1}{s_{123} s_{13}}+\frac{1}{s_{123} s_{23}}+\frac{1}{s_{13} s_{24}}+\frac{1}{s_{23} s_{234}}+\frac{1}{s_{234} s_{24}} \\ \end{bmatrix} $$ has determinant $$ \frac{s_{134}s_{124}s_{14}}{s_{12}s_{13}s_{23}s_{24}s_{34}s_{123}s_{234}}. $$ {\tiny \begin{multline*} \hspace{-25pt} \left[ \begin{matrix} \frac{1}{s_{12} s_{123}}+\frac{1}{s_{12} s_{34}}+\frac{1}{s_{123} s_{23}}+\frac{1}{s_{23} s_{234}}+\frac{1}{s_{234} s_{34}} & -\frac{1}{s_{123} s_{23}}-\frac{1}{s_{23} s_{234}} & -\frac{1}{s_{234} s_{34}} \\ -\frac{1}{s_{123} s_{23}}-\frac{1}{s_{23} s_{234}} & \frac{1}{s_{123} s_{13}}+\frac{1}{s_{123} s_{23}}+\frac{1}{s_{13} s_{24}}+\frac{1}{s_{23} s_{234}}+\frac{1}{s_{234} s_{24}} & -\frac{1}{s_{13} s_{24}}-\frac{1}{s_{234} s_{24}} \\ -\frac{1}{s_{234} s_{34}} & -\frac{1}{s_{13} s_{24}}-\frac{1}{s_{234} s_{24}} & \frac{1}{s_{13} s_{134}}+\frac{1}{s_{13} s_{24}}+\frac{1}{s_{134} s_{34}}+\frac{1}{s_{234} s_{24}}+\frac{1}{s_{234} s_{34}} \\ \frac{1}{s_{23} s_{234}}+\frac{1}{s_{234} s_{34}} & -\frac{1}{s_{23} s_{234}} & -\frac{1}{s_{134} s_{34}}-\frac{1}{s_{234} s_{34}} \\ -\frac{1}{s_{23} s_{234}} & \frac{1}{s_{23} s_{234}}+\frac{1}{s_{234} s_{24}} & -\frac{1}{s_{234} s_{24}} \\ -\frac{1}{s_{12} s_{34}}-\frac{1}{s_{234} s_{34}} & -\frac{1}{s_{234} s_{24}} & \frac{1}{s_{234} s_{24}}+\frac{1}{s_{234} s_{34}}\\ \end{matrix} \right. \\ \hspace{-25pt} \left. \begin{matrix} \frac{1}{s_{23} s_{234}}+\frac{1}{s_{234} s_{34}} & -\frac{1}{s_{23} s_{234}} & -\frac{1}{s_{12} s_{34}}-\frac{1}{s_{234} s_{34}} \\ -\frac{1}{s_{23} s_{234}} & \frac{1}{s_{23} s_{234}}+\frac{1}{s_{234} s_{24}} & -\frac{1}{s_{234} s_{24}} \\ -\frac{1}{s_{134} s_{34}}-\frac{1}{s_{234} s_{34}} & -\frac{1}{s_{234} s_{24}} & \frac{1}{s_{234} s_{24}}+\frac{1}{s_{234} s_{34}} \\ \frac{1}{s_{134} s_{14}}+\frac{1}{s_{134} s_{34}}+\frac{1}{s_{14} s_{23}}+\frac{1}{s_{23} s_{234}}+\frac{1}{s_{234} s_{34}} & -\frac{1}{s_{14} s_{23}}-\frac{1}{s_{23} s_{234}} & -\frac{1}{s_{234} s_{34}} \\ -\frac{1}{s_{14} s_{23}}-\frac{1}{s_{23} s_{234}} & \frac{1}{s_{124} s_{14}}+\frac{1}{s_{124} s_{24}}+\frac{1}{s_{14} s_{23}}+\frac{1}{s_{23} s_{234}}+\frac{1}{s_{234} s_{24}} & -\frac{1}{s_{124} s_{24}}-\frac{1}{s_{234} s_{24}} \\ -\frac{1}{s_{234} s_{34}} & -\frac{1}{s_{124} s_{24}}-\frac{1}{s_{234} s_{24}} & \frac{1}{s_{12} s_{124}}+\frac{1}{s_{12} s_{34}}+\frac{1}{s_{124} s_{24}}+\frac{1}{s_{234} s_{24}}+\frac{1}{s_{234} s_{34}} \end{matrix} \right] \end{multline*} } has determinant $$ \frac{s_{1234}^4}{s_{12}^2 s_{13}^2 s_{14}^2 s_{23}^2 s_{24}^2 s_{34}^2s_{123}s_{124}s_{134}s_{234}}. $$ \end{example} Other applications and directions to the physics of amplitudes and correlators will be explored in future work. \bibliographystyle{alpha} \bibliography{refs} \end{document}
2412.07054v1
http://arxiv.org/abs/2412.07054v1
Transcendence of ${}_3F_2(1)$ Hypergeometric Series and $L$-values of Modular Forms
\documentclass[12pt]{amsart} \usepackage{amsfonts,latexsym,amsthm,amssymb,amsmath,amscd,euscript} \usepackage{tikz-cd} \usepackage[margin = 2cm]{geometry} \usepackage[shortlabels]{enumitem} \usepackage[utf8]{inputenc} \usepackage{hyperref} \usepackage{arydshln} \usepackage{dirtytalk} \usepackage{float} \usepackage{multirow} \usepackage[normalem]{ulem} \hypersetup{ colorlinks=true, linkcolor=red, filecolor=red, urlcolor=red, } \urlstyle{same} \definecolor{purple}{rgb}{0.59, 0.44, 0.84} \newcommand{\ft}[1]{{\color{purple} #1}} \renewcommand{\bar}{\overline} \newtheorem{theorem}{Theorem}[section] \newtheorem{proposition}[theorem]{Proposition} \newtheorem{Lemma}[theorem]{Lemma} \newtheorem{corollary}[theorem]{Corollary} \newtheorem*{exercise}{Exercise} \newtheorem{Conjecture}[theorem]{Conjecture} \theoremstyle{definition} \newtheorem{definition}[theorem]{Definition} \theoremstyle{remark} \newtheorem{remark}{Remark} \newtheorem*{notation}{Notation} \usepackage{fancyhdr} \usepackage{caption} \captionsetup[table]{skip=5pt} \usepackage{thmtools} \usepackage[framemethod=TikZ]{mdframed} \mdfdefinestyle{mdrecbox} { linewidth=0.5pt, skipabove=12pt, frametitleaboveskip=5pt, frametitlebelowskip=0pt, skipbelow=2pt, frametitlefont=\bfseries, innertopmargin=4pt, innerbottommargin=8pt, nobreak=true, } \declaretheoremstyle [ headfont=\bfseries, mdframed={style=mdrecbox}, headpunct={\\[3pt]}, postheadspace={0pt}, ] {thmrecbox} \declaretheorem[style=thmrecbox,name=Example, numberlike=theorem]{examplebox} \newenvironment{example} { \begin{examplebox} \leavevmode \begin{enumerate}} { \end{enumerate} \end{examplebox} } \newcommand{\nc}{\newcommand} \nc{\R}{\mathbb R} \nc{\C}{\mathbb C} \nc{\F}{\mathbb F} \nc{\Q}{\mathbb Q} \nc{\Z}{\mathbb Z} \renewcommand{\H}{\mathbb H} \nc{\N}{\mathbb N} \nc{\B}{\mathbb B} \nc\scalemath[2]{\scalebox{#1}{\mbox{\ensuremath{\displaystyle #2}}}} \nc{\limin}{\underline{\lim}} \nc{\limsu}{\overline{\lim}} \nc{\bl}{\color{blue}} \newtheorem*{theorem*}{Theorem} \nc{\cT}{\mathcal T} \nc{\cP}{\mathcal P} \nc{\cM}{\mathcal M} \nc{\cC}{\mathcal C} \nc{\cB}{\mathcal B} \nc{\cO}{\mathcal O} \nc{\quat}{\left(\frac{a,b}{F}\right)} \nc{\cS}{\mathcal S} \nc{\e}{\mathbf{e}} \nc{\w}{\mathbf{w}} \renewcommand{\v}{\mathbf{v }} \renewcommand{\u}{\mathbf{u }} \nc{\f}{\mathbf{f}} \nc{\s}{\text{ }} \nc{\Mod}{\operatorname{Mod}} \nc{\Aut}{\operatorname{Aut}} \nc{\del}{\partial} \nc{\inter}{\mathrm{o}} \nc{\close}[1]{\overline{#1}} \nc{\pderiv}[2]{\frac{\partial #1}{\partial #2}} \nc{\tr}{\operatorname{tr}} \nc{\disc}{\text{disc}} \newcommand*\pFqskip{8mu} \catcode`,\active \newcommand*\pFq{\begingroup \catcode`\,\active \def ,{\mskip\pFqskip\relax} \dopFq } \catcode`\,12 \def\dopFq#1#2#3#4#5{ {}_{#1}\mathbb{F}_{#2}\biggl(\genfrac..{0pt}{}{#3}{#4};#5\biggr) \endgroup } \newcommand{\hypcoeff}[1]{\frac{(\ba)_{#1}}{(\bbeta)_{#1}}} \newcommand*\HYPERskip{&} \catcode`,\active \newcommand*\pfq{ \begingroup \catcode`\,\active \def ,{\HYPERskip} \doHyper } \catcode`\,12 \def\doHyper#1#2#3#4#5{ \, _{#1}F_{#2}\left[\begin{matrix}#3 \smallskip \\ #4\end{matrix} \; ; \; #5\right] \endgroup } \newcommand*\pPqskip{8mu} \catcode`,\active \newcommand*\pPq{\begingroup \catcode`\,\active \def ,{\mskip\pPqskip\relax} \dopPq } \catcode`\,12 \def\dopPq#1#2#3#4#5{ {}_{#1}\mathbb{P}_{#2}\biggl(\genfrac..{0pt}{}{#3}{#4};#5\biggr) \endgroup } \renewcommand{\epsilon}{\varepsilon} \renewcommand{\Im}{\operatorname{Im}} \title{Transcendence of ${}_3F_2(1)$ Hypergeometric Series and \textit{L}-values of Modular Forms} \author{Esme Rosen } \date{} \begin{document} \begin{abstract} We examine the families of hypergeometric series associated to a special class of modular hypergeometric Galois representations recently studied by Allen, Grove, Long, and Tu. The underlying motives allow us to express a special $L$-value of the corresponding modular form in terms of these hypergeometric series. We use extra symmetries coming from the underlying modular forms to study the transcendence of families of periods, which have the form ${}_3F_2(1)$, on hypergeometric surfaces. For several hypergeometric surfaces with four periods, we show that the vector space generated by these elements has dimension at most 2. As an application of our results, we utilize a classical formula of ${}_3F_2(1)$ series due to Kummer to find relations between $L$-values of modular forms that differ by an even twist, as predicted in a theorem of Shimura. \end{abstract} \maketitle \tableofcontents \section{Introduction} There is a long history of studying the transcendence of important numbers such as $\pi$ and $e$. Some well-known examples are studied in \cite{piagm}. Kontsevich and Zagier \cite{kontsevichzagier} introduced the concept of \textit{periods}, which are integrals of differentials on complex varieties. These periods generate an intermediate ring between the complex numbers and the algebraic numbers, which gives rise to interest in studying their transcendence properties; for example, the most famous period, $\pi$, is known to be transcendental. In general, understanding the transcendence of periods is very difficult. A class of examples of a period in number theory are the special values of (automorphic) $L$-functions, which are expected to be transcendental when nonzero \cite{waldschmidt}. However, proving these transcendence results is very hard. For example, many questions remain regarding the transcendence of special values of the Riemann zeta function at odd integers, the best studied automorphic $L$-function. So far, only the transcendence of $\zeta(3)$ is definitively known due to a famous result of Ap\'ery, which has also been interpreted in terms of modular forms by Beukers \cite{beukers}. Some progress has been made by Zudilin and his collaborators; for example, in \cite{zudilin5} he shows that one of $\zeta(5), \zeta(7)$, $\zeta(9)$, and $\zeta(11)$ is irrational. However, the proof is nonconstructive, and so proving transcendence for a given odd zeta value other than 3 remains open. Interestingly, transcendence results about the zeta function can be related to certain integrals of hypergeometric type, see \cite{zudilin5} as well as \cite{rhinviola} by Rhin and Viola. Many of the other great successes of transcendence theory in the past century involve the theory of modular forms and abelian varieties. One well-known example is Nesterenko's \cite{nesterenko} proof that $\pi, e^\pi$, and $\Gamma(1/4)$ are algebraically independent over $\Q$. This was a corollary to a theorem that stated at least three of $e^{\pi \tau}, E_2(\tau), E_4(\tau)$, and $E_6(\tau)$ for $\tau$ in the upper half plane are algebraically independent over $\Q$. Here $E_n(\tau)$ is the classical Eisenstein series, and the desired result follows when $\tau=i$. The motivating result for this paper is a theorem of W\"ustholz \cite{wustholz}, which is stated in terms of general algebraic groups (see \cite{cohen} for a formulation in terms of abelian varieties). \begin{theorem}[W\"ustholz]\label{wust} Let $\Lambda$ denote the set of periods of differentials of the first and second kind on a simple abelian surface, $A$ over a number field $K$. The vector space generated by $1, 2\pi i$, and $\Lambda$ has dimension 4 if $A$ has complex multiplication (CM), dimension 6 if $A$ has quaternionic multiplication (QM), and dimension 10 otherwise. \end{theorem} Recall roughly speaking, an abelian variety has complex multiplication if the endomorphism algebra contains a CM number field, and has quaternionic multiplication if the endomorphism algebra of the variety contains a quaternion algebra. In other words, if an abelian variety has extra endomorphisms, then this extra structure reduces the dimension of the vector space above. This is a central theme of our results. W\"ustholz's theorem is central to many other transcendence results, some of which are discussed in the survey paper \cite{waldschmidt} by Waldschmidt. This paper will be focused on a special class of periods called hypergeometric series. The classical Gaussian hypergeometric series for rational parameters $a$, $b$, and $c$ and a complex number $z$ can be written as a period via the integral formula $$P(a,b,c; z):=B(b,c-b)\pfq{2}{1}{a,b}{,c}{z}=\int_0^1x^{b-1}(1-x)^{c-b-1}(1-z x)^{-a}\text{d}x$$ when $b>c>0$ and $B(x,y)$ is the beta function. There is a well-developed theory of the curves on which these periods are defined, which are called generalized Legendre curves; see \cite{archinard} for the definition. Deines, Fuselier, Long, Swisher, and Tu \cite{glc} determine whether a family of 2-dimensional abelian varieties constructed from the Jacobian of a given generalized Legendre curve has QM by studying if a certain quotient of beta values is algebraic. In the process, they use that when the Jacobian has QM, the first cohomology decomposes into isomorphic 2-dimensional pieces over a suitable number field $K$. There are \say{hypergeometric varieties} for which generalized hypergeometric series are periods (see \cite{deinesetal2} and \cite{robertsvillegas}), but because these have higher dimensions, the result of W\"ustholz no longer applies, and the transcendence degree of their periods is less well understood. The work of Otsubo and his collaborators addresses some specific cases, but mainly when the underlying variety has a CM structure. In \cite{otsubo}, he describes the Beilinson regulator in terms of ${}_3F_2(1)$ values with certain parameters. A few of the hypergeometric series in this paper also appear in Table \ref{tab}, specifically in the imprimitive cases, which are classes 2-4. A similar method allows him to write certain different ${}_3F_2(1)$ as a linear combination of logarithms for algebraic numbers in \cite{otsubo2}, which are known by Baker's theorem \cite{baker} to be transcendental. He also proves geometric results about the Ceresa cycles of Fermat curves in \cite{otsubo3}, which relies on the linear independence of several hypergeometric series. All of the papers listed above involve Fermat curves, whose zeta functions are known to be the product of Hecke $L$-functions. This implies that if there is an underlying modular form, it has to be CM. The goal of this paper is to understand some explicit examples where there is extra structure, like CM and the analog of QM examples given below, and find relations between the periods. We replicate the QM situation above in the following way. Let $$HD=\{\{a_1,...,a_n\},\{b_1,...,b_n\}, \{\lambda \}\}$$ for $a_i,b_i\in \Q.$ If $M=\text{lcd}(a_1,...,a_n,b_1,...,b_n)$ where lcd denotes the least common denominator, $M$ is called the \textit{level} of $HD$, and we will assume $\lambda\in \Q(\zeta_M)\setminus\{0\}$. Then Katz \cite{katz} proved that there is a Galois representation $\sigma_{HD}$ associated to $HD$ defined over the absolute Galois group $G_{\Q(\zeta_M)}$, unramified at all but finitely many primes of $\Q(\zeta_M)$, whose trace of Frobenius at unramified primes is related to a finite field analogue of hypergeometric functions. This representation is $n$-dimensional if $\lambda\neq 0,1$, and is $n-1$-dimensional if $\lambda=1$. Suppose the induction \begin{equation}\label{ind} \rho_{HD}:=\text{Ind}_{G_{\Q(\zeta_N)}}^{G_\Q}\sigma_{HD} \end{equation}satisfies $$\rho|_{G_K} =\sigma_1\oplus\sigma_2\oplus ....\oplus \sigma_{\varphi(N)},$$ where $\rho|_{G_K}$ denotes the restriction to the absolute Galois group $G_K$ of some number field $K$ and the $\sigma_i$ are 2-dimensional and isomorphic to each other. In this case, we say $\sigma_{HD}$ is \textit{potentially 2-isotypic} (see \cite{lilongliu}). In the ${}_2F_1$ cases in \cite{glc}, the representations arising from QM abelian surfaces are potentially 2-isotypic due to the decomposition of the cohomology discussed above, so we may view this as a generalization of the 2-dimensional abelian varieties admitting QM. Recently, Allen, Grove, Long, and Tu \cite{aglt} studied the 2-dimensional Katz representation associated to $$HD(r,s):=\{\{1/2,1/2,r\},\{1,1,s\},\{1\}\}.$$ They showed that the period associated to $HD(r,s)$ is related to an explicit weight 3 modular form denoted $\mathbb{K}_2(r,s)$ to be defined below (Equation \ref{eq:K2}). Namely, these are constructed so that the critical $L$-values $L(\mathbb{K}_2(r,s),1)$ and $\pi^{-1}L(\mathbb{K}_2(r,s),2)$ can be written as an algebraic multiple of $B(r,s-r)\s_3F_2(HD(r,s),1)$. The $\mathbb{K}_2(r_i,s_i)$ are sorted into explicit families using a geometric background (Definition \ref{conj}), and if each of the $\mathbb{K}_2(r_i,s_i)$ in a given family lies in the same Hecke orbit as modular forms, then the family is said to be \textit{Galois} (Definition \ref{galois}). Notably, in these cases the induced representation of $\sigma_{HD(r,s)}$, as given in (\ref{ind}), is potentially 2-isotypic. This provides easily computable examples to test the transcendence degree of the periods on certain hypergeometric surfaces. First, we write down all the Galois families. \begin{theorem}\label{classification} There are exactly 29 Galois families, and a complete list is given in Table \ref{tab} below. These 29 families correspond to 17 distinct Hecke eigenforms, 5 of which are CM. \end{theorem} For the Galois families, we prove that the corresponding Hecke eigenform is a linear combination of $\mathbb{K}_2(r,s)$ functions using the action of the Hecke operators (Theorem \ref{complete}). Correspondingly, the exact $L$-values at 1 and 2 for the eigenform can be written as a linear combination of $B(r,s-r)\s_3F_2(HD(r,s),1)$ (Corollary \ref{lval2}). In Section \ref{class}, we also discuss how the 29 Galois families can be divided into 11 classes using the Coxeter group interpretation of ${}_3F_2(1)$ identities due to Beyer et al. \cite{coxeter}. Our main transcendence result is the following:. \begin{theorem}\label{main} Assume $M=\text{lcd}(r,s,1/2)$ as above, and that the associated family as in Definition \ref{conj} is Galois. If the family is in the Hecke orbit of a CM modular form, the vector space for periods of differentials of the first kind over $\Bar{\Q}$ has dimension 1. For the non-CM cases satisfying $\varphi(M)\le 4$, the vector space for periods of differentials of the first kind over $\Bar{\Q}$ has dimension less than or equal to $2$. \end{theorem} For the Galois families associated to CM modular forms, we expect the vector space given in Theorem \ref{wust} by W\"ustholz to be generated by $1,2\pi i$, and the periods of one differential of the first kind and one differential of the second kind. When the modular form is non-CM, because of the potentially 2-isotypic Galois representation in the background, we anticipate the vector space to be generated by $1,2\pi i$, and the periods of two differentials of the first kind and two differentials of the second kind. Therefore, Theorem \ref{main} is consistent with W\"ustholz's theorem. For example, if \begin{equation}\label{gi} f_i=B(1/2,i/8)\pfq{3}{2}{1/2,1/2,i/8}{,1,i/8+1/2}{1}, \quad i=1,3,5,7 \end{equation} then the vector space of over $\Bar{\Q}$ generated by $f_1,f_3,f_5,f_7$ is less than or equal to 2, and a basis is $f_3$ and $f_5$. We anticipate this is equal to 2. Actually, we expect the transcendence degree of $\Bar{\Q}(f_1,f_3,f_5,f_7)$ to be 2, but this is an even stronger result than predicted by W\"ustholz. We will also be interested in special values of $L$-values of newforms in terms of hypergeometric series. This connection goes back to Zagier's \cite{zagierL} result, \begin{equation}\label{zagier} L(\eta(2\tau)^4\eta(4\tau)^4,2)=\frac{\pi^2}{16}\cdot \pfq{4}{3}{1/2,1/2,1/2,1/2}{,1,1,1}{1}. \end{equation} Often, due to the underlying hypergeometric motives, results like this also come with corresponding identities of finite field hypergeometric series. In this case, Ahlgren and Ono \cite{ahlgrenono} showed that for a finite field hypergeometric series originally defined by \cite{greene}, we have for each odd prime $$\pfq{4}{3}{\phi,\phi,\phi,\phi}{,\epsilon,\epsilon,\epsilon}{1}_p= a_p(\eta(2\tau)^4\eta(4\tau)^4) +p$$ where $\phi$ is the quadratic character mod $p$ and $\epsilon$ is the trivial character mod $p$, which in Greene's dictionary correspond to the parameters in Equation (\ref{zagier}). In the formula above, we use a slightly different normalization of Greene's finite field hypergeometric function due to Beukers, Cohen, and Mellit \cite{bcm}, often denoted $H_p$. There are several other examples of $L$-values expressed in terms of hypergeometric series in the literature, e.g. \cite{asairec}, \cite{lilongliu}, \cite{osburnstraub}, \cite{zudlinlval}, and \cite{zudilin3}, some of which are still conjectural. An application of our transcendence results \say{extends} identities like the following to identities of $L$-functions. \begin{theorem}[Kummer, 1836]\label{kummer} When both series converge $$ \pfq{3}{2}{1/2,1/2,r}{,1,s}{1}=\frac{\Gamma(s)\Gamma(s-r)}{\Gamma(s-1/2)\Gamma(s-r+1/2)}\pfq{3}{2}{1/2,1/2,1-r}{,1,1/2-r+s}{1}.$$ \end{theorem} The version here is a special case of the more general theorem; see Andrews, Askey, and Roy \cite{aar} Cor. 3.3.5. The notion of extending classical identities to obtain new results is motivated by papers such as \cite{whipple}, \cite{chenchu} and \cite{zudilin2}. The modular form version of this theorem is stated in terms of the Fourier expansion of the modular form at $i\infty$, which is usually written using $q=e^{2\pi i\tau}$. For example, there is a striking relationship between the families $\{(1/4,1), (3/4,1)\}$ and $\{(1/4,3/4),(3/4,5/4)\}$: the Fourier coefficients are \begin{align*} \mathbb{K}_2(1/4,1)=q - 2 q^5 - 7 q^9 + 14 q^{13} + 18 q^{17} - 32 q^{21} - 21 q^{25} + 14 q^{29}+O(q^{30})\\ \mathbb{K}_2 (1/4,3/4)=q + 2 q^5 - 7 q^9 - 14 q^{13} + 18 q^{17} + 32 q^{21} - 21 q^{25} - 14 q^{29}+O(q^{30}), \end{align*} that is, they differ by a sign. The pairs $(1/4,1)$ and $(3/4,5/4)$ are related by Theorem \ref{kummer} stated above. This general twisting property is our first main theorem, stated more precisely in Theorem \ref{twisting}. \begin{theorem}\label{twistingpre} Let $h=h(r,s)=r-s +3/2$. If $0<h(r,s)<3/2$, then for any positive integer $n$, the $n$th Fourier coefficients of $\mathbb{K}_2(r,s)(\tau)$ and $\mathbb{K}_2(r,h)(\tau)$ are the same up to a sign. \end{theorem} Assume for now that $\mathbb{K}_2(r,s)$ belongs to a Galois family, so that a linear combination of this $\mathbb{K}_2(r,s)$ and several other \say{conjugate} modular forms equals a newform $f$, which \say{completes} the $\mathbb{K}_2(r,s)$ function. Applying this theorem to all the $\mathbb{K}_2(r,s)$ making up $f$, in many cases we obtain $f\otimes \chi$, where $\chi$ is a finite order character. Now we use the discussion above to write $L(f,1)$ as a linear combination of ${}_3F_2(1)$ hypergeometric series. We may apply the Kummer transformation, Theorem \ref{kummer}, to each hypergeometric series. Then using the linear relations needed to prove Theorem \ref{main}, we can realize $L(f,1)$ as a multiple of $L(f\otimes \chi,1)$, which is our extension of Kummer to the setting of $L$-functions. When $\chi$ is an even character, a theorem of Shimura \cite{shimurazeta}, Theorem \ref{shimura} implies that this multiple must be algebraic. For example, we will show that \begin{equation}\label{L} L(f_{64.3.d.a},1)=-\frac{1}{2}\zeta_{48}L(f_{256.3.c.g},1). \end{equation} This extends the Kummer transformation applied to the family in (\ref{gi}) to a relation of $L$-values. Here, $f_{256.3.c.g}$ denotes the newform associated to 256.3.c.g in the L-functions and Modular Forms Database (LMFDB). The theorem of Shimura and our main transcendence results in Theorem \ref{main} turn out to be beautifully intertwined. As noted above, we require the transcendence results to prove relations such as (\ref{L}) arising from Theorem \ref{twistingpre}. However, the proof of these transcendence results also uses the theorem of Shimura in a different way. Namely, we make use of the fact that we can write the $L$-values of $f$ and of $f\otimes \chi$ in terms of our hypergeometric series when $\chi$ is a quadratic character to prove linear relations among the hypergeometric series. The paper is organized as follows. First, we discuss the necessary background in Section \ref{prelim}. Section \ref{modcox} is focused on classifying all the possible families, and we also set up the necessary tools to study the $L$-functions of newforms using the method described above. In particular, we prove Theorem \ref{twistingpre} and Theorem \ref{classification}, as well as establishing a few other helpful preliminary results. In Section \ref{lvall}, we prove Theorem \ref{main} in several parts on a case-by-case basis. The relations between $L$-values are crucial for the non-CM cases, and in the process we show several relations between $L$-values arising from the Kummer transformation and Theorem \ref{twistingpre}. After discussing some further directions, we provide a table of all Hecke eigenforms obtained using our method in the Appendix. \bigskip \subsection*{Acknowledgements} The author would like to express her gratitude to Ling Long and Fang-Ting Tu for suggesting this problem based on their recent work, and for their helpful guidance and suggestions along the way. She would also like to thank Hasan Saad and Wadim Zudilin for their helpful comments on an earlier version of this paper. The author is supported by a summer research assistantship from the Louisiana State University Department of Mathematics. \bigskip \section{Preliminaries}\label{prelim} In this section, we introduce the $\mathbb{K}_2(r,s)$ functions and recall some of their important properties from \cite{aglt}. We also briefly explain how we can frame our discussion throughout the paper using a finite Coxeter group. \subsection{The $\mathbb{K}_2(r,s)$ Functions} For rational parameters $a_i$ and $b_i$ and $\lambda\in \C\setminus\{0,1\}$, let $(a)_k=a(a+1)...(a+k-1)$. Then define the generalized hypergeometric series by $$\pfq{n}{n-1}{a_1,a_2,...,a_n}{b_1,b_2,...,b_{n-1}}{\lambda}=\sum_{k=0}^\infty\frac{(a_1)_k(a_2)_k...(a_n)_k}{(b_1)_k(b_2)_k...(b_{n-1})_k}\frac{\lambda^k}{k!}.$$ It is well known (see e.g. \cite{yang}) that when $\lambda$ is the modular lambda function, the hypergeometric function $$\pfq{2}{1}{1/2,1/2}{,1}{\lambda}$$ is a weight 1 modular form on $\Gamma(2)$. Let $q=e^{2\pi i\tau}$. Multiplying by $d/dq\log\lambda=d\lambda/(2\pi i\lambda)$ then yields a weight 3 weakly holomorphic modular form, which can be made holomorphic by multiplying appropriate powers of $\lambda$ and $1-\lambda$. Using this idea, Allen, Grove, Long, and Tu \cite{aglt} defined the function \begin{equation}\label{eq:K2} \mathbb{K}_2(r,s)(\tau)= \frac{\eta(\tau/2)^{16s-8r-12}\eta(2\tau)^{8r+8s-12}}{\eta(\tau)^{24s-30}}. \end{equation} These are closely related to hypergeometric functions by the following Lemma. \begin{Lemma}[\cite{aglt}]\label{aglt} The function $\mathbb{K}_2(r,s)$ is a weight 3 cusp form for some finite index subgroup $\Gamma$ of $SL_2(\Z)$ if and only if $0<r<s<3/2$, and $$\mathbb{K}_2(r,s)(\tau)=2^{1-4r}\lambda^r(1-\lambda)^{s-r-1}\pfq{2}{1}{1/2,1/2}{,1}{\lambda}\frac{d}{dq}\log \lambda.$$ Moreover, $\Gamma$ is congruence if and only if the exponents in the eta product \eqref{eq:K2} are integral, and the level is $$\frac{48}{\gcd(24r,24)}\frac{48}{\gcd(24(s-r),24)}.$$ \end{Lemma} \noindent For the remainder of this paper, we will only work with $\mathbb{K}_2(r,s)$ defined on congruence subgroups. It is well established that such modular forms have a Fourier expansion in terms of the variable $q$. The $q$-expansion of $\mathbb{K}_2(r,s)(\tau)$ should a priori have local uniformizer $e^{2\pi i\tau/N}$, where \begin{equation}\label{N} N=\frac{48}{\gcd(24r,24)}. \end{equation} However, it turns out we can lift all of our congruence $\mathbb{K}_2(r,s)(\tau)$ to $\Gamma_1(\mathcal{N})$ by using $\mathbb{K}_2(r,s)(N\tau)$, for which the local uniformizer is $q=e^{2\pi i\tau}$ as above. Despite this, $N$ appears in several formulas later. Note that by construction, if $r=a/b$ where $a$ and $b$ are coprime, then $N=2b$. We will use this fact several times. By a slight abuse of notation, when we write $\mathbb{K}_2(r,s)$ without a variable, we mean $\mathbb{K}_2(r,s)(N\tau)$. The modular forms $\mathbb{K}_2(r,s)$ are also related to generalized hypergeometric series via their $L$-values. More precisely, the integral representation of hypergeometric functions gives that \begin{align} 2^{1-4r}\int_0^1\lambda^r(1-\lambda)^{s-r-1}\pfq{2}{1}{1/2,1/2}{\hspace{.8cm}1}{\lambda}\frac{d}{dq}\log \lambda=-2\pi i\int_{0}^{i\infty} \mathbb{K}_2(r,s)(N\tau)d\tau\\=\frac{2^{1-4r}B(r,s-r)}{N}\pfq{3}{2}{1/2\hspace{.3cm}1/2\hspace{.3cm}r}{ \hspace{1.1cm}1\hspace{.5cm}s}{1}. \end{align} We also have the integral representation for the special $L$-values attached to $\mathbb{K}_2(r,s)$, \begin{equation}\label{lval} L(\mathbb{K}_2(r,s),1)=-2\pi i\int_{0}^{i\infty} \mathbb{K}_2(r,s)(N\tau)d\tau=\frac{2^{1-4r}B(r,s-r)}{N}\pfq{3}{2}{1/2,1/2,r}{,1,s}{1}. \end{equation} For convenience, we will establish the notation $$F(r,s):=\frac{2^{1-4r}B(r,s-r)}{N}\pfq{3}{2}{1/2,1/2,r}{,1,s}{1}.$$ \noindent\textit{Remark}: Our normalization is non-standard and is chosen so that the computations with $L$-values are more convenient. Note that it differs from the $P(r,s)$ function used in \cite{aglt} as follows $$\pi F(r,s)=N2^{1-4r}P(r,s).$$ The only notable difference is the multiple of $\pi$ from a transcendence perspective. From our point of view, the factor of $\pi$ does not impact our main results. As alluded to above, $\pi F(r,s)$ is a period on a hypergeometric surface. If $F(r,s)$ and $F(r',s')$ are \textit{conjugate}, then they can be defined as periods on the same surfaces. This reduces to the following definition. \noindent As above, $M=\text{lcd}(1/2,r,s)$, where we assume $r$ and $s$ are simplified into lowest terms. \begin{definition}\label{conj} Two pairs $(r,s)$ and $(r',s')$ are \textit{conjugate} if there exists an integer $c$ coprime to $M$ so that $r-cr'$ and $s-cs'$ are both integers. \end{definition} \noindent \textbf{Example}: The pairs $(1/8,5/8)$, $(3/8,7/8)$, $(5/8,9/8)$, and $(7/8,11/8)$ are conjugate, e.g., $3/8+5\cdot(1/8)=1$ and $7/8+5\cdot(5/8)=4$ are integers (here $c=-5$). Note that all four of these have the form $(n/8,n/8+1/2)$ for $n$ coprime to 8. The associated variety is the projective desingularization of the \begin{equation}\label{eq:C} C(n/8,n/8+1/2):\s y^8=x_1^4(1-x_1)^4x_2^{8-n}(1-x_2)^4(1-x_1x_2)^4 \end{equation} where $1\leq n\leq 7$ (see \cite{deinesetal2} or \cite{robertsvillegas}). On each of these surfaces there is a differential 2-form of the first kind $$\omega_n=\frac{1}{\sqrt[8]{x_1^4(1-x_1)^4x_2^{n}(1-x_2)^4(1-x_1x_2)^4}}dx_1dx_2,$$ so that the period $$\tau_n=\int_0^1\int_0^1\omega_n=2^{4r-1}\pi F(n/8,n/8+1/2).$$ Moreover, $$\frac{x_1^3(1-x_1)^3x_2^{n-1}(1-x_2)^3(1-x_1x_2)^3}{(\sqrt[8]{x_1^4(1-x_1)^4x_2^{n}(1-x_2)^4(1-x_1x_2)^4})^7}dx_1dx_2$$ is also a differential form on $C(n/8,n/8+1/2)$. This simplifies to $\omega_{8-n}$, demonstrating that the periods $\tau_n$ and $\tau_{8-n}$ are defined on the same surface. A similar process allows us to view all 4 periods as on the same surface. For example, $$\frac{x_1^2(1-x_1)^2(1-x_2)^2(1-x_1x_2)^2}{(\sqrt[8]{x_1^4(1-x_1)^4x_2(1-x_2)^4(1-x_1x_2)^4})^5}dx_1dx_2$$ is by definition a differential on $C(1/8,5/8)$, but is the same as $\omega_5$, a differential on $C(5/8,9/8)$. Let $X$ denote a smooth projective model of \eqref{eq:C}, which has singularities, where all $\omega_n$ are defined. We have the action of $\mu_8$, the 8th roots of unity, on the space of regular 2-forms, $H^2_{dR}(X,\C)$. Throughout, we will use the notation $\zeta_n=e^{2\pi i/n}$. As discussed in \cite{archinard}, the action is given by $(x_1,x_2,y)\mapsto (x_1,x_2,\zeta_8^{-1}y)$ on the affine equation. This decomposes the cohomology into pieces: $$H^2_{dR}(X,\C)=\bigoplus_{n=0}^7V_n.$$ We only want to consider the cases where $(n,8)=1$, that is, $V_1\oplus V_3\oplus V_5\oplus V_7$. Here, each $V_n$ corresponds to the differential $\omega_n$. This is parallel to how the induction $\rho$ decomposes on the Galois representation side due to the 2-isotypic property. While the conjugates put the $\mathbb{K}_2(r,s)$ functions into families based on the underlying hypergeometric series, we can also use information about them in modular forms. \begin{definition}[\cite{aglt}]\label{galois} A family of conjugate pairs $(r_i,s_i)$ is \textit{Galois} if each $\mathbb{K}_2(r_i,s_i)$ is in the same Hecke orbit for all $i$. \end{definition} As mentioned in the introduction, the Galois condition is crucial since it ensures that the induction $\rho$ of $\sigma_{HD}$ given in (\ref{ind}) is potentially 2-isotpyic. This is reflected in the additional symmetries among the periods, which will be our focus later. The example above is Galois, because for $p=3,5,7$, and $T_p$ the $p$th Hecke operator, \begin{equation}\label{hecke3} T_p\mathbb{K}_2(1/8,n/8+1/2)=C_p\mathbb{K}_2(p/8,p/8+1/2) \end{equation}for some integer $C_p$, so these conjugates are in the same Hecke orbit. A similar property holds for all Galois families (see Lemma \ref{hecke2} below). There are non-Galois cases as well: for example, $(1/12,1/6)$ and $(7/12,7/6)$ are conjugate, but the former is in the Hecke orbit of $f_{576.3.g.c}$ and the latter in the Hecke orbit of $f_{576.3.g.f}$. Identifying the Hecke orbit of the modular forms in the LMFDB is possible since we know the level from Lemma \ref{aglt}, and so this is a finite check. \subsection{Coxeter Group Interpretation} There is a long history of studying hypergeometric transformations in terms of symmetries of a finite Coxeter group going back to at least Bailey \cite{bailey}. A finite Coxeter group is really just a special kind of finite group, which for us will typically be $S_n$, the symmetric group on $n$ variables, or $D_n$, the dihedral group of order $2n$. For ${}_3F_2(1)$ series 1, this has been studied in great depth by Beyer et al. \cite{coxeter}, and there are similar results for certain ${}_4F_3(1)$ \cite{formichella}, \cite{green} as well. Using the finite symmetry group is very helpful for our classification of $\mathbb{K}_2(r,s)$ families and for stating other identities. This is because the Coxeter group provides a uniform way to state identities that come from the hypergeometric side (such as analytic continuation formulas) and the modular forms side (such as the Atkin-Lehner involution). Based on the work of Bailey, Beyer et al. show that for ${}_3F_2(1)$ series, the invariance group for a properly normalized series is $S_5$, and they use this to classify all two term identities between the hypergeometric series, such as the Kummer transformation. Typically, the action of $S_5$ on a hypergeometric multiset $\{\{a,b,c\},\{d,e\}\}$ is expressed by writing the multiset as a column vector $(a,b,c,d,e)^t$. Denote such a vector by $\mathfrak{a}$. Then we write $S_5$ as an appropriate subgroup of $GL_5(\Z)$ so that our action on $\mathfrak{a}$ is the usual multiplication of a matrix and a vector. From the point of view of the Coxeter groups, the initial multiset $HD$ is not so important, but we will assume that $a=b=1/2,d=1$ for convenience throughout. Thus, we may write $\mathfrak{a}=(r,s)$ in our setting, with $a,b$ and $d$ implicit, so $F(r,s)=F(\mathfrak{a})$. The Kummer transformation \ref{kummer} in our new notation is $$F(\mathfrak{a})=C(r,s)F(K\mathfrak{a})$$ for $K$ the $5\times5$ matrix in the proof of Lemma \ref{cox} and $C(r,s)$ a gamma quotient given explicitly in (\ref{crs}) below. Beyer et al. \cite{coxeter} also study \say{three-term identities}, originate in the work of Thomae \cite{thomae} and Bailey \cite{bailey}. A typical three-term identity formulated in the language of Coxeter groups by \cite{coxeter} is a theorem of Thomae, $${}_3F_2( \mathfrak{a})(1)=\alpha{}_3F_2(m_1\mathfrak{a})(1)+\alpha'{}_3F_2(m_2\mathfrak{a})(1)$$ where $\alpha,\alpha'$ are constants and $m_1$ and $m_2$ are certain $5\times5$ matrices specified in \cite{coxeter}. A more explicit realization of this result is given in Proposition \ref{thomae} below. They also show that for three-term identities, the invariance group is $S_6\times C_2$, where $C_2$ is the group of order 2, but the process of producing identities is more complicated. We will discuss three-term identities in our setting in Section \ref{3term}. Allen et al. \cite{aglt} showed there are only finitely many $(r,s)$ which correspond to holomorphic congruence modular forms, and the set of all of these pairs is $$\mathbb{S}_2':=\{(r,s)\s|\s 0<r<s<3/2, \s 24s, 8(r+s)\in \Z\}.$$ Excluding the \say{imprimitive} cases when $s=1/2$ or $r=1$, there are exactly 167 such pairs; however, we want to include these examples, and so our definition is slightly different from \cite{aglt}. There are two natural actions on the modular forms $\mathbb{K}_2(r,s)$, the Kummer transformation (Theorem \ref{kummer}) discussed above and the Atkin-Lehner involution. We will only use $W_2$, which is defined by the map $W_2:\tau\mapsto -1/2\tau$. Recall if $r=a/b$ is reduced, then $N=2b$. The action of $W_2$ is $$W_2\mathbb{K}_2(r,s)(2b\tau)=\tau^{-3}\s \mathbb{K}_2(r,s)(-b/\tau)=2^{8s-16r}(-i)^3\mathbb{K}_2(s-r,s)(b\tau).$$ This is straightforward to prove using the well-known formula $\eta(-1/\tau)=\sqrt{-i\tau}\eta(\tau)$. We fix the identity embedding of $\sqrt{-1}$ into $\C$, which we denote by $i$, for the remainder of this paper. The action of $W_2$ and the Kummer transformation on $F(r,s)$ are given by the operators \begin{equation}\label{eq:A} A: (r,s)\mapsto (s-r,s) \end{equation} and \begin{equation}\label{eq:K} K: (r,s)\mapsto (1-r,1/2-r+s). \end{equation} \begin{proposition}\label{cox} The group $G$ generated by $A$ and $K$ is isomorphic to $D_6$, the dihedral group of order 12. \end{proposition} \begin{proof} Both $A$ and $K$ are involutions, i.e., they have order two. The general version of Theorem \ref{kummer} given in Corollary 3.3.5 of \cite{aar} gives $$K((a,b,c,d,e)^t)=(a,d-b,d-c,d,d+e-b-c)^t. $$ Thus, with respect to the standard basis for $\Q^6$, the matrix for $K$ is $$K=\begin{pmatrix} 1& 0& 0& 0& 0\\ 0& -1& 0& 1& 0\\ 0& 0& -1& 1& 0\\ 0& 0& 0& 1& 0\\ 0& -1& -1& 1& 1 \end{pmatrix}.$$ The matrix for $A$ is defined similarly. We also compute that $$AK=\begin{pmatrix} 1&0&0&0&0\\ 0&-1&0&1&0\\ 0&-1&0&0&1\\ 0&0&0&1&0\\ 0&-1&-1&1&a \end{pmatrix}.$$ Using linear algebra, it is then easy to check that $A$ and $K$ have order 2 and $AK$ has order 6, and so the group is isomorphic to the dihedral group of order 12. \end{proof} Note $D_6$ is a subgroup of the group $S_6\times C_2$ given in \cite{coxeter} for three term identities. We also point out that $D_6$ as a group is a subgroup pf $S_5$ as well; however, using the normalization of \cite{coxeter}, denoted $\hat{F}(r,s)$, we should not view our group $G$ as a subgroup of the invariance group $S_5$. We write $\hat{F}(r,s)$ in terms of $F(r,s)$ in equation \ref{fhat} below. Note that $\hat{F}(K(r,s))=\hat{F}(r,s)$ from \cite{coxeter}, and so $K$ is in the invariance group of $\hat{F}(r,s)$. However, $\hat{F}(A(r,s))\neq \hat{F}(r,s)$ in general. For example, we can see $A(1/8,5/8)=(1/2,5/8)$, and it is easy to check numerically that $\hat{F}(1/8,5/8)\neq\hat{F}(1/2,5/8)$. This implies $A$ is not an element of the invariance group. As a result, besides the matrix $K$ arising from the Kummer transformation, we will use the group $G$ exclusively for three-term identities. \section{Symmetries from Modular Forms and Coxeter Groups}\label{modcox} In this section, we first show the technical but crucial fact that every Galois family of conjugates can be completed to a Hecke eigenform. This allows us to compute exact $L$-values of the modular forms and construct Table \ref{tab:my_label} in the Appendix. Next, we prove Theorem \ref{twistingpre} and Theorem \ref{classification}. Finally, we explain the types of identities that will be used to prove Theorem \ref{main} and provide a few simple examples. \subsection{Building Eigenforms from the $\mathbb{K}_2$ Functions} \subsubsection{Hecke Operators} First, we prove that the Hecke operators can be used to construct Hecke eigenforms out of Galois families. Denote the newform for the Hecke orbit of our Galois family by $f$, its character by $\chi$, and its Hecke eigenvalue field by $K_f$. Since Definition \ref{conj} of conjugates excludes $c$ dividing the level $M$, it is apparent that if $(r_i,s_i)$ and $(r_j,s_j)$ are conjugate, then $r_i$ and $r_j$ have the same denominator. In Table \ref{tab}, we organize families using an element of the form $(1/b,s'/m)$. The following lemma guarantees that this exists. As above, and throughout, we will assume that $a$ and $b$ are coprime. \begin{Lemma} Assume $b\geq 2$ is an integer and $a$ is an integer coprime to $b$. Then every $\mathbb{K}_2(a/b,s/m)$ defined on a congruence subgroup for $a,b,$ and $s$ integers is conjugate to $\mathbb{K}_2(1/b,s'/m)$ for some integer $1/b<s'/m<3/2$. \end{Lemma} \begin{proof} The case $b=2$ is trivial, since the only possible $a/2$ lying between $0$ and $3/2$ is 1/2. So assume $b>2$. Note $(1/b)(b-a)+a/b$ equals 1, an integer. Also observe that $b-a$ is coprime to $M$, since $(a,b)=1$. So we only need to prove there is some $s'$ so that $(b-a)s'/m+s/m$ is also an integer, that is, $(b-a)s'+s\equiv 0\mod m$. For the pair $(a/b,s/m)$ to correspond to a modular form on a congruence group, $m$ has to be either 1, 2, 4, 6, 8, 12, or 24, all of which can be written as products consisting of powers of 2 and 3. Therefore, $(b-a, m)=1$, and it follows that the linear congruence \begin{equation}\label{cong} (b-a)s'+s\equiv 0\mod m \end{equation} has a unique solution modulo $m$. We choose a solution $x$ of (\ref{cong}) so that $x\leq m$. If $x/m\geq 1/b$ we only need $x/m<3/2$ so that the conjugate $(1/b,x/m)$ constructed above is holomorphic. Because we assumed $x\leq m$, we have $x/m\leq 1<3/2$. Therefore, we can take $s'=x$. When $x/m<1/b$, since $b>2$, we have $x/m<1/2$, and it follows that $1/n<x/m+1<3/2$. Moreover, clearly $x+m$ satisfies the congruence (\ref{cong}) as well, so we take $s'=x+m$. \end{proof} \noindent\textit{Remark}: Each $\mathbb{K}_2(a_i/b,s_i)$ has nonzero $n$th Fourier coefficients only if $n\equiv a_i\mod b$. This can be checked by directly expanding the eta product definition of $\mathbb{K}_2(a_i/b,s_i)(N\tau)$. \begin{definition} We say a family $\{\mathbb{K}_2(r_i,s_i)\}$ is of \textit{multiplicative type} if for $i\neq j$, $r_i\neq r_j$ for all $i$ and $j$. \end{definition} The motivation for this definition is that in families of multiplicative type, $\mathbb{K}_1(1/b,s_1)$ has multiplicative coefficients. In this case, we can write our family as $$S_{i/b,s_i}:=\{\mathbb{K}_2(i/b,s_i)\}_{i\in (\Z/b\Z)^\times}.$$ \begin{Lemma}\label{hecke2} Let $\{\mathbb{K}_2(i/b,s_i)\}_{i\in (\Z/b\Z)^\times}$ be a Galois family of multiplicative type. Let $p$ be a prime so that $p\equiv i\mod b$. Then there exists a nonzero integer $C(p,i)$ so that for any $j\in (\Z/b\Z)^\times$, \begin{equation*} T_p\mathbb{K}_2(j/b,s_j)=C(p,j) \mathbb{K}_2(k/b,s_k), \end{equation*} where $ij=k$ as elements of $(\Z/b\Z)^\times$. In other words, the action of the Hecke operators is governed by the group structure of $(\Z/b\Z)^\times$. \end{Lemma} \noindent\textit{Remark}: Lemma \ref{hecke2} is a more general version of an example given in Corollary 3.1 of \cite{aglt} for the family $(1/8,1)$. They can use the Dwork dash operator since $s=1$, which has the same action. In this sense, Lemma \ref{hecke2} tells us that we can use the Hecke operators to generalize the Dwork dash operator, which does not apply when $s\neq 1$, to families of multiplicative type. \begin{proof} Note that $S_{i/b,s_i}$ generates a vector space with dimension larger than the dimension of $K_f$. This is because the set spans the Hecke orbit and the $q$-coefficients are all integers. In particular, the set spans the Hecke orbit, which implies $T_p\mathbb{K}_2(i/b,s_i)$ can be written as a linear combination of $\mathbb{K}_2(h/b,s_h)$ functions. Next, denote the $n$th Fourier coefficient of $\mathbb{K}_2(i/b,s_i)$ and $T_p\mathbb{K}_2(i/b,s_i)$ by $A_n$ and $B_n$ respectively. Then we have the formula for $T_p$ on the Fourier coefficients $$B_n=A_{np}+p^2 \chi(p)A_{n/p},$$ where we assume $A_{n/p}$ is zero if $n/p$ is not an integer. Then, $A_{np}$ is nonzero only if $n\equiv k\mod b$. To see this, note that for $\mathbb{K}_2(j/b,s_j)$ to be defined on a congruence group, the only possibilities for $b$ are $b=3,4,8,12,24$ based on Lemma \ref{aglt}. In all of these cases, $(\Z/b\Z)^\times$ is isomorphic to a finite product of $\Z/2\Z$ with itself, and so each element has order 2. Recall $A_n$ is nonzero only if $n\equiv j\mod b$. Then for $A_{np}$ to be nonzero, we need $np\equiv j\mod b$ where $p\equiv i$. By assumption $ij=k$ in $(\Z/b\Z)^\times$, and since each element is its own inverse, this implies $j=ik$. Since $p\equiv i\mod b$, we have $np\equiv j\mod b$ if and only if $n\equiv k\mod b$. We now consider $A_{n/p}$. Analogous to above, since $p\equiv i\mod b$ and $i$ is its own inverse, $1/p$ is equivalent to $i$ in $(\Z/b\Z)^\times$. Thus, for $n/p$ to be congruent to $j\mod b$, we must have $n\equiv ij=k\mod b$ as well. To conclude, by the first paragraph, $T_p\mathbb{K}_2(j/b,s_j)$ is a linear combination of $\mathbb{K}_2(h/b,s_h)$ functions, and since $\mathbb{K}_2(k/b,s_k)$ is the only member of its family with nonzero coefficients for $n\equiv k\mod b$, we have proved the lemma. \end{proof} We establish the notation $$K_i:=\mathbb{K}_2(i/b,s_i),$$ since the Lemma shows the action of the Hecke operators on a family of multiplicative type depends only on the index $i\in (\Z/b\Z)^\times$. As a corollary of Lemma \ref{hecke2}, we can formulate the actions of the Hecke operators on this family $S_{r_m,s_m}$ as follows. \begin{corollary}\label{hecke4} Assume $S_{r_m,s_m}=\{K_i\}_{i\in (\Z/b\Z)^\times}$ is a set of conjugates satisfying the hypotheses of Lemma \ref{hecke2}. For any primes $p$ and $\ell$ coprime to $N$, $$ T_\ell T_p= T_pT_\ell =D(p,\ell)T_{[p\ell]},\quad D(p,\ell) \in \Z, $$ where $[p\ell] \equiv p\ell \mod b$ is defined so that $0<[p\ell]<b$. Therefore, the Hecke algebra on this set is determined by $T_p$, $p\in (\Z/b\Z)^\times$. Moreover, for a prime $p\in (\Z/b\Z)^\times$, after a suitable reordering of the basis $S_{r_i,s_i}$, the action of $T_p$ is given by $$ \begin{pmatrix} T_{p,1} &0& \cdots & 0\\ 0& T_{p,2} & \ddots & 0\\ 0&\ddots & \ddots & 0\\ 0&\cdots& 0& T_{p,\frac{\phi(b)}2} \end{pmatrix}, \quad T_{p,i}= \begin{pmatrix} 0 &a_i\\ b_i& 0\\ \end{pmatrix}, \quad a_ib_i = D(p,p). $$ Also, when $p\equiv 1\mod b$ is a prime, the action of $T_p$ on $S_{r_m,s_m}$ is $C(p,1)\text{Id}$, where Id is the identity matrix. \end{corollary} \begin{proof} Note $$T_pT_\ell K_j=C(\ell,j)C(p,[j\ell])K_{[p \ell j]}$$ by standard properties of the Hecke operators and Lemma \ref{hecke2}. On the other hand, $$T_{[p\ell]}K_j=C([p\ell],j)K_{[[p\ell]j]},$$ since every integer less than $b$ and coprime to $b$ happens to be a prime number in all of our cases. But $[[p\ell]j]=[p\ell j]$ in $(\Z/b\Z)^\times$, and so we have $$T_{\ell}T_pK_j=\frac{C(\ell,j)C(p,[j\ell])}{C([p\ell],j)}T_{[p\ell]}K_j.$$ This constant does not depend on $j$ because the Hecke algebra is commutative, and it is by definition $D(p,\ell)$ as stated in the Corollary. The remainder of the corollary follows immediately from Lemma \ref{hecke2}. \end{proof} \begin{theorem}\label{complete} Assume $\{\mathbb{K}_2(i/b,s_i)\}_{i\in (\Z/b\Z)^\times}$ is a Galois family of multiplicative type in the Hecke orbit of the newform $f$. Then there exist algebraic numbers $\beta_i$ integral over a quadratic number field depending on $i$ so that $$\sum_{i\in (\Z/b\Z)^\times}\beta_i \mathbb{K}_2(i/b,s_i)$$ is an eigenvector of $T_p$ for all $p$ coprime to 6. Moreover, any Hecke eigenform constructed as above can be written as $f\otimes \phi$, where $\phi$ is some trivial or quadratic character with conductor dividing $b$. \end{theorem} \begin{proof} By Corollary \ref{hecke4}, the eigenvalues of $T_p$ are $\pm \sqrt{D(p,p)}$ for each $p$. The only possible combination of $\mathbb{K}_2(1/b,s_1)$ and $\mathbb{K}_2(i/b,s_i)$ that is an eigenvector of $T_i$ with eigenvalue $\pm\sqrt{D(p,p)}$ is $$\mathbb{K}_2(1/b,s_1)\pm \sqrt{D(i,i)}\mathbb{K}_2(i/b,s_i).$$ Because the matrices for $T_i$ are simultaneously diagnolizable by the spectral theorem, there is a basis for $S_{i/b,s_i}$ which are eigenvectors for all the $T_i$. By the discussion above, each such eigenvector must have the form $$\sum_{i\in (\Z/b\Z)^\times}\pm \sqrt{D(i,i)} \mathbb{K}_2(i/b,s_i).$$ The coefficients $\beta_i$ in the statement are thus $\sqrt{D(i,i)}$ up to a sign. However, we cannot choose the sign arbitrarily according to Corollary \ref{hecke4}; we must have that if $ij=k$ in $\Z$, then $\beta_i\beta_j=\beta_k$. We arbitrarily choose signs for the $\beta_i$ so that this is true. The only ambiguity in constructing our $\beta_i$ is the choice of sign, and so any other Hecke eigenform constructed using the family $\{\mathbb{K}_2(i/b,s_i)\}$ must have the same Fourier coefficients up to a sign. For the coefficients to remain multiplicative, the only way we can adjust this sign is equivalent to tensoring the coefficients by a quadratic character. In our linear combination, we only have control over the coefficients modulo $b$, and so the resulting character has to be on $(\Z/b\Z)^\times$, that is, the conductor divides $b$ \end{proof} Because of equation (\ref{lval}), integrating both sides produces an exact $L$-value of the Hecke eigenform. \begin{corollary}\label{lval2} Assume $\{\mathbb{K}_2(i/b,s_i)\}_{i\in (\Z/b\Z)^\times}$ is a family of multiplicative type. Then there are algebraic numbers integral over a quadratic number field depending on $i$, $\beta_i$, so that for some quadratic character $\phi$ with conductor dividing $b$, $$L(f\otimes \phi,1)=\sum_{i\in (\Z/b\Z)^\times}\beta_i F(i/b,s_i).$$ \end{corollary} Using the functional equation for a weight 3 Hecke eigenform, we can also obtain the special $L$-value at 2 from this formula as well. \smallskip \noindent \textit{Remark}: For a given newform $f$ with LMFDB label $xx.3.x.x$, note if $f$ has Hecke eigenvalue field larger than $\Q$, then the choice of $L$-function depends on which (inner) twist we choose in Corollary \ref{lval2}. We establish our choice of twist in Table \ref{tab:my_label}, and henceforth, when we write $L(f_{xx.3.x.x},1)$, we are referring to the twist given in that table unless noted otherwise. \smallskip \subsubsection{An Example of Constructing Hecke Eigenforms} To illustrate the Theorem above, we use the recurring family that includes the pair $(1/8,5/8)$. Equation (\ref{hecke3}) shows the family is Galois, and the conjugates have the form $(a/8,a/8+1/2)$ for $a=1,3,5,7$. Using the level and character provided in \cite{aglt} and carefully studying the Fourier coefficients, we determine that all four of these are in the Hecke orbit of $f_{64.3.d.a}$, which agrees with Table \ref{tab} below. To find the linear combination of the conjugates, it suffices to determine the action of the Hecke operators $T_p\mathbb{K}_2(1/8,5/8)$ for $p=3,5,7$. A computation on the Fourier coefficients reveals the following action. $$\begin{array}{c||c|c|c|c} &f_1&f_3&f_5&f_7\\\hline T_3&12f_3&f_1&-4f_7&-3f_5\\ T_5&-48f_5&16f_7&f_1&-3f_3\\ T_7&-64f_7&16f_5&-4f_3&f_1 \end{array} $$ From the table, we see that $T_3^2=12$, $T_5^2=-48$, and $T_3T_5=-3T_7$. We conclude $\beta_1=2\sqrt{3}$, $\beta_2=4i\sqrt{3}$ and because of the multiplicative nature of the coefficients, $\beta_3=-\beta_1\beta_2/3=-8i$. In practice, we can also find the $\beta_i$ using the LMFDB database. Recall that the first term of the Fourier expansion of $\mathbb{K}_2(a/8,a/8+1/2)$ is $q^a$. We can match this with the coefficient for $q^a$ on the LMFDB page for $f_{64.3.d.a}$ For example, when $a=5$, in the LMFDB, the coefficient of $q^5$ for $f_{64.3.d.a}$ is labeled $\beta_2$ and equals $4i\sqrt{3}$. This agrees with our computation above, but we also can find the $\beta_i$ in this way without needing to compute the action of the Hecke operators. Either way, we find that $$f_{64.3.d.a}=\mathbb{K}_2(1/8,5/8)+2\sqrt{3}\mathbb{K}_2(3/8,7/8)+4\sqrt{3}i\mathbb{K}_2(5/8,9/8)-8i\mathbb{K}_2(7/8,11/8).$$ By Corollary \ref{lval2}, the above formula also immediately transforms into the special $L$-value at 1 if we replace $\mathbb{K}_2$ with $F$. That is, $$L(f_{64.3.d.a},1)=F(1/8,5/8)-2\sqrt{3}F(3/8,7/8)- 4\sqrt{3}iF(5/8,9/8)-8iF(7/8,11/8).$$ We use this method repeatedly in the rest of the paper, especially when working with $L$-values. For convenience, the correct linear combination of conjugates for all the Galois cases is listed in the Appendix. \subsection{Twisting and the Kummer Transformation} Recall from the introduction that we have the following twisting property between families of conjugates. \begin{align*} \mathbb{K}_2(1/4,1)=q - 2 q^5 - 7 q^9 + 14 q^{13} + 18 q^{17} - 32 q^{21} - 21 q^{25} + 14 q^{29}+O(q^{30})\\ \mathbb{K}_2 (1/4,3/4)=q + 2 q^5 - 7 q^9 - 14 q^{13} + 18 q^{17} + 32 q^{21} - 21 q^{25} - 14 q^{29}+O(q^{30}). \end{align*} This appears to be the modular forms version of Theorem \ref{kummer} above in terms of families of conjugates. The relationship is summarized the diagram below. \begin{center} \includegraphics[scale=.4]{twistingdiagram.png} \end{center} In the diagram, the dotted lines denote relations between modular forms (Theorem \ref{twisting} below), the dashed lines are relations between differentials on a hypergeometric surface (Definition \ref{conj}), and the solid lines are transcendence relations of hypergeometric series (Theorem \ref{kummer}). Our application to $L$-values of newforms for some special cases arises because of a combination of all three of these aspects. In this section, we focus on the twisting. This is explained by symmetries of the hypergeometric functions underlying $\mathbb{K}_2(r,s)$. \begin{theorem}\label{twisting} Let $h=h(r,s)=r-s+3/2$. Then if $0<h(r,s)<3/2$, for any positive integer $n$, the $n$th Fourier coefficients of $\mathbb{K}_2(r,s)(\tau)$ and $\mathbb{K}_2(r,h)(\tau)$ are the same up to a sign. Moreover, if $a$ is the numerator of $r$, the $n$th coefficients are equal if and only if $n\equiv a\mod N$, where $N$ is as in (\ref{N}). \end{theorem} We will say that $(r,s)$ and $(r,h)$ are \textit{Kummer twists} of each other in this case, which are labeled \say{twisting} in the figure. Note this is a slightly more specific version of Theorem \ref{twistingpre} from the introduction. \begin{proof} Apply the Pfaff transformation (see \cite{aar}), $$\pfq{2}{1}{1/2,1/2}{,1}{x}=(1-x)^{-1/2}\pfq{2}{1}{1/2,1/2}{,1}{\frac{x}{x-1}},$$ to $$\mathbb{K}_2(r,s)(\tau)d\tau=\lambda^r(1-\lambda)^{s-r-1}\pfq{2}{1}{1/2,1/2}{,1}{\lambda}\frac{d\lambda}{2\pi i\lambda}.$$ This gives us $$\lambda^{r}(1-\lambda)^{s-r-3/2}\pfq{2}{1}{1/2,1/2}{,1}{\frac{\lambda}{\lambda-1}}\frac{d\lambda}{2\pi i\lambda}.$$ Making a change of variable, $t=\lambda/(\lambda-1)$, and using that $\lambda(\tau+1)=\lambda/(\lambda-1)$ (see e.g. \cite{yoshida}), we compute \begin{align*} \mathbb{K}_2(r,s)(\tau)d\tau=&\left(\frac{t}{t-1}\right)^r\left(\frac{1}{t-1}\right)^{s-r-3/2}\pfq{2}{1}{1/2&1/2}{&1}{t}\frac{dt}{2\pi it(t-1)}\\&=(-1)^r(1-t)^{1/2-s}t^r\pfq{2}{1}{1/2&1/2}{&1}{t}\frac{dt}{2\pi it}=(-1)^r\mathbb{K}_2(r,r-s+3/2)(\tau+1)d\tau, \end{align*} invoking that the linear fractional transformation $\lambda/(\lambda-1)$ is its own inverse repeatedly. Because of this, the $q$-expansions of $\mathbb{K}_2(r,s)(\tau)$ and $\mathbb{K}_2(r,h)(\tau)$ differ by a root of unity, and as the $\mathbb{K}_2(r,s)$ functions always have rational coefficients, this means the Fourier coefficients must differ by a sign. Let $r=a/b$ for $a$ and $b$ coprime integers. For the second part, the key observation is that $b$ is always equal to $N/2$, as noted after equation (\ref{N}). Also, note that $(-1)^r=e^{-a\pi i/b}=e^{-2a\pi i/N}.$ We temporarily (only for the remainder of this proof) use the local uniformizer $q_N=e^{2\pi i\tau/N}$. The map $\tau\mapsto\tau+1$ sends $$e^{2\pi i\tau/N}\mapsto e^{2\pi i\tau/N}e^{2\pi i/N},$$ and so $q_N^n\mapsto e^{2n\pi i/N}q_N^n$. Multiplying $e^{-2a\pi i/N}$ through, we see the $n$th Fourier coefficient for $\mathbb{K}_2(r,s)(\tau)$ is the same as the $n$th Fourier coefficient of $\mathbb{K}_2(r,h)(\tau)$ up to $e^{2(n-a)\pi i/N}$. Again writing out the $q_N$-expansion, we see the first term in the Fourier expansion of $\mathbb{K}_2(r,s)(\tau)$ is $q_N^a$, and all other non-zero coefficients are equivalent to $a$ modulo $b$. Since $b=N/2$, this implies $n=a+mN/2$ for some integer $m$. This divides the $n$ into two congruence classes mod $N$, equivalent to $a$ when $m$ is even and $a+N/2$ when $m$ is odd. Evidently, the signs do not switch if and only if $m$ is even, so this proves the claim. \end{proof} Note that this doesn't even require our modular form to be defined on a congruence group, and the twist could be the same as the original modular form. However, the phenomenon is called twisting because in all but one of the Galois cases that are of multiplicative type, the completed two $q$-series differ by a twist of a character. For example, the newform \begin{equation}\label{1/4} f_{64.3.c.b}=\mathbb{K}_2(1/4,1)+4i\mathbb{K}_2(3/4,1) \end{equation} is related by Kummer to $$f_{32.3.c.a}=\mathbb{K}_2(1/4,3/4)+4i \mathbb{K}_2(3/4,5/4)$$ and $f_{64.3.c.b}=\chi_{8.b} \otimes f_{128.3.c.a}$, where $\chi_{8.b}$ is also given by its LMFDB label. The one exception is Class 3 in Table \ref{tab}, which occurs because one of the corresponding Hecke eigenforms is an oldform for the level specified in Lemma \ref{aglt}. Besides Class 5, all Galois families that are not of multiplicative type are self-twisted in the following sense. \begin{definition} We say a family of conjugates $(r_i,s_i)$ is \textit{self-twisted} if the Kummer twist of $(r_i,s_i)$ equals $(r_j,s_j)$ for some $j$ in the same family. \end{definition} All the self-twisted families outside of Class 5 arise from applying the Atkin-Lehner involution $W_2$ to a family of multiplicative type. For example, the Atkin-Lehner involution maps $\mathbb{K}_2(a/8,a/8+1/2)$ to $\mathbb{K}_2(1/2,a/8+1/2)$, up to a constant. We can then easily check that the latter family is self-twisted. Each of these families has the same number of conjugates, so there should be a relation between them. This is discussed more in Section \ref{3term}, and the specific relation for the family $(a/8,a/8+1/2)$ is Proposition \ref{fivterm}. This allows us to compute the action of the Hecke operators indirectly. \subsection{Classification of Conjugate Families}\label{class} In this section, we prove Theorem \ref{classification} by enumerating all the possible families. Understanding our situation in terms of the group action of $D_6:=\langle A,K\rangle$ on $\mathbb{S}_2'$ allows us to classify our modular forms in a natural way. We demonstrate this pictorially by constructing a graph where the nodes are the elements of $\mathbb{S}_2'$. The edges are the pairs related by $A$ (colored red) or by $K$ (colored black), and a black dotted line if two vertices are in the same Hecke orbit. An example is given in Figure \ref{fig:galois} below. Generally, if $(r,s)$ is in $\mathbb{S}_2'$, that does not imply $K$ applied to $(r,s)$ is in $\mathbb{S}_2$ (in particular, it may not be holomorphic at the cusps), so in this case we mark them in a gray color as seen in Figure \ref{fam11}. For example, $(1/4,5/4)$ corresponds to a level 16 holomorphic modular form. However, applying $K$ to this produces $(3/4,3/2)$, which is not in $\mathbb{S}_2'$. The figure shows first a specific connected component of the graph, and then a more general connected component. \begin{figure}[htbp] \centering \includegraphics[width=0.8\linewidth]{class6graph.png} \caption{The graph associated to class 6 on the left, with the Atkin-Lehner involution in red, the Kummer twist in black, and elements in the same Hecke orbit dotted. The graph on the right is associated to a generic non-CM four-term class} \label{fig:galois} \end{figure} \subsubsection{List of all Galois Classes} The main result of this section is Table \ref{tab} below. Each row of the table corresponds to a connected component of the graph described above, including all three types of edges. As a result of the table, we are able to find all connected components of the graph consisting of Galois families. This allows us to prove Theorem \ref{classification}, as well as classify the connected components of the graph. \begin{theorem} There are exactly 29 Galois families, falling into 11 connected components in the graph of Galois families. \end{theorem} From here on, when we refer to a \say{family} of pairs, we mean the set of conjugates for a given pair, and when we say a \say{class} of pairs, we mean all pairs in a connected component of the graph, i.e. a row in Table \ref{tab}. We only label the classes in Table \ref{tab}. Individual families are assigned labels in Table \ref{tab:my_label} of the Appendix. \begin{definition} For a representative pair $(r,s)$ in a given family of conjugates, we denote its stabilizer subgroup in $D_6$ by $\text{Stab}(r,s)$. \end{definition} Note this is stabilized within the family. For example, we would consider $A$ to be in $\text{Stab}(1/8,1)$ because $A(1/8,1)=(7/8,1)$, which is conjugate to $(1/8,1)$. We can classify the stabilizer as follows. \begin{proposition} For every pair $(r,s)$ in a Galois family outside of Classes 1 and 5, we have $$\text{Stab}(r,s)\cong \Z/2\Z\times \Z/2\Z. $$ If $(r,s)$ is in Class 1 or 5, then $$\text{Stab}(r,s)\cong D_6. $$ \end{proposition} Thus, in most Galois cases, $$|D_6/\text{Stab}(r,s)|=3,$$ and so Table \ref{tab} is divided into three columns. For classes 1 and 5, the stabilizer is all of $D_6$, which implies there is only one family of conjugates. Therefore, we put a star $(*)$ on the second and third columns, denoting that this family is the same as the previous one. Namely, the three columns for these classes denote the action of the Kummer twist and the Atkin-Lehner involution, respectively. Class 1 has already been studied in \cite{ahlgrenono} and \cite{osburnstraub} as mentioned in the introduction. We will discuss Class 5 in depth in Section \ref{class5}. \begin{table}[ht] \begin{center} \begin{tabular}{c|cc|cc|cc|c} Label & Data & LMFDB & K-Data & K-LMFDB&AL-Data&AL-K-Data& CM? \\\hline 1 & $1/2,1$ & 16.3.c.a & $1/2,1\s (*)$ & 16.3.c.a &$1/2,1\s (*)$&$1/2,1$& yes\\\hline 2& $1/4,5/4$ & 16.3.c.a & $1/4,1/2$ & 64.3.c.a & $1,5/4$ & -& yes\\\hline 3& $1/3,7/6$ & 36.3.d.a &$1/3,2/3$ & $\begin{array}{c} \text{36.3.d.a}\\\text{36.3.d.b} \end{array}$ & $1/6, 5/6$ & $1/6,5/6$ & yes\\\hline 4& $1/8,9/8$ & 32.3.d.a & $1/8,1/2$ & 256.3.c.e & $1,9/8$ & $1,11/8$ & yes\\\hline 5 & $1/6,4/3$ & 144.3.g.c & $1/6,1/3 \s (*)$ & 144.3.g.c & $7/6,4/3\s (*)$ & -& yes\\\hline 6 & $1/4,3/4$ & 32.3.c.a & $1/4,1$ & 64.3.c.b& $1/2,3/4$ & $1/2,5/4$& no \\\hline 7& $1/8,5/8$ & 64.3.d.a & $1/8,1$ & 256.3.c.g & $1/2,5/8$ & $1/2,11/8$& no \\\hline 8& $1/8,7/8$ & 128.3.d.c & $1/8,3/4$ & $\begin{array}{c} \text{256.3.c.c}\\\text{256.3.c.f} \end{array}$& $1/4,5/8$ & $1/4,9/8$& no\\\hline 9& $1/12,11/12$ & $\begin{array}{c} \text{288.3.g.a}\\\text{288.3.g.c} \end{array}$ & $1/12,2/3$ & $\begin{array}{c} \text{576.3.g.d}\\\text{576.3.g.h} \end{array}$& $1/6,7/12$ & $1/6,13/12$& no\\\hline 10& $1/24, 17/24$ & 288.3.b.c & $1/24, 5/6$ & 2304.3.g.y& $2/3,17/24$ & $2/3,25/24$ & no\\\hline 11& $1/24,23/24$ & 1152.3.b.i & $1/24,7/12$ & $\begin{array}{c} \text{2304.3.g.p}\\\text{2304.3.g.w} \end{array}$ & $1/12,13/24$ & $1/12,25/24$ & no\\ \end{tabular} \end{center} \caption{Classes of Galois Families.} \label{tab} \end{table} The K-Data and K-LMFDB in Table \ref{tab} signify the Kummer twist of the initial pair (\textbf{not} the action of the matrix $K$ or the Kummer transformation (Theorem \ref{kummer}), and the corresponding LMFDB label. By Theorem \ref{complete}, we can construct any twist by a quadratic character of our initial modular form if $b$ divides the conductor. When this is an inner twist, the LMFDB label is the same, but when it is not an inner twist, the label is different. For completeness, we list both labels in these cases. A complete list of all the conjugates in each Galois family and how to combine them into a Hecke eigenform is provided in the Appendix. All families of multiplicative type are Galois because we can check that Lemma \ref{hecke2} holds. The first and second columns for classes 1-4 and 6-11 are of multiplicative type. The third column corresponds to the action of the Atkin-Lehner involution on the first column. \begin{Lemma} $\mathbb{K}_2(r_i,s_i)$ is a Galois family if and only if $\mathbb{K}_2(s_i-r_i, s_i)$ is as well. \end{Lemma} \begin{proof} Because the Atkin-Lehner involution commutes with the Hecke operators, the Atkin-Lehner involution maps forms in the same Hecke orbit to forms in the same Hecke orbit, that is, if the family of $\mathbb{K}_2(r,s)$ is Galois, then the family of $\mathbb{K}_2(s-r,s)$ is as well. \end{proof} Therefore, the third column for Classes 1-4 and 6-11 are Galois. Class 5 is a special case. We will show separately that there are simple relations among the three pairs in Class 5 in Section \ref{class5}, which proves the family is Galois. To prove that Table \ref{tab} encompasses all possible cases, we simply enumerated every other family and showed they are not Galois. \subsubsection{List of non-Galois classes} We write out the non-Galois cases below, which are organized similar to above. Note that all the underlying Hecke eigenforms here are CM. We do not have an analog of Theorem \ref{complete} here, and so the table is less clean. \begin{table}[ht] \centering \begin{tabular}{ccccccccc} Label & Data & LMFDB 1 & LMFDB 2 & K-Data & K-LMFDB 1 & K-LMFDB 2\\ 12 & $1/24,5/24$ & - & - & $1/24,1/3$ &- &-\\ 13 & $1/12,17/12$ &- & 144.3 g.c & $1/12,1/6$ & 576.3.g.c & 576.3.g.f \\ 14 & $1/8,11/8$ & 128.3.d.a & 128.3.d.b & $1/8,1/4$ & 256.3.c.b & 256.3.c.d\\ 15 & $1/24,35/24$ & 1152.3.b.b & - & $1/24,1/12$ &2304.3.g.e & -\\ \end{tabular} \caption{Non-Galois Classes} \label{nongalois} \end{table} In the columns LMFDB 1 and K-LMFDB 1, we are able to construct an entire Hecke eigenform from a subset of our family of conjugates. For the label LMFDB 2, we only obtain Fourier coefficients for certain congruence classes of primes mod $b$, and so the dimension is not large enough to construct a completed Hecke eigenform. If nothing is listed, the modular forms are related to multiple Hecke eigenforms. A geometric heuristic is given for this in Section \ref{ngall}. The behavior of the Atkin-Lehner involution is very similar to the Galois case. As in the Galois case, the right-hand side of the above table is mapped by the Atkin-Lehner involution to itself. We record the Atkin-Lehner involutions of the left-hand side in a separate table. \begin{center} \begin{tabular}[ht]{ccccc} Label & Data 1 & LMFDB 1 & Data 2 & LMFDB 2 \\ AL-12 & $1/6,5/24$ & - & $1/6,35/24$ & - \\ AL-13 & $1/3,5/12$ & - & $1/3,17/12$ & - \\ AL-14 & $1/4,7/8$ & 128.3.d.a & $3/4,9/8$ & 128.3.d.b \\ AL-15 & $1/12,19/24$ & 1152.3.b.b & $1/12,7/24$ & 1152.3.b.f \\ \end{tabular} \end{center} Here is a graph for a non-Galois case. As above, if a circle is gray, the corresponding $(r,s)$ is not in $\mathbb{S}_2'$. Note that because there are multiple Hecke orbits, there are multiple connected components in each class. The only reason we sort them into classes like this is to be consistent with the Galois case. \begin{figure} \centering \includegraphics[scale=.4]{nongaloisgraph} \caption{A Graph for a non-Galois family} \label{fig:non-galois} \end{figure} \begin{center} \end{center} \subsection{Three-Term Hypergeometric Identities}\label{3term} According to Beyer et al. Section 2.2 \cite{coxeter}, three-term identities arise from the cosets of a certain normal subgroup of $S_6\times C_2$. In our case, it appears that one relevant subgroup is the commutator $D_6'=[D_6,D_6]$ in $D_6$. We have $$D_6/D_6'\cong \Z/2\Z\times \Z/2\Z=\langle M_1,M_2 \rangle,$$ where \begin{equation}\label{eq:M&N} M_1:=AKA, \quad M_2:=KAK. \end{equation} Using the matrix form of $A$ and $K$, it is straightforward to verify that these are coset representatives and $M_2,$ $M_1$, and $M_1M_2$ are indeed involutions. Note that the Kummer twist fixes (within the family) the right-hand side of Table \ref{tab}, and the Atkin-Lehner involution fixes the left-hand column of Table \ref{tab}. This implies $M_1$ and $M_2$ fix the conjugates of the left-hand column of Table \ref{tab}. Therefore, these operators correspond to switching to a different conjugate pair in these cases. The main result of the next section is the following. \begin{theorem}\label{mainpar} Let $\mathfrak{a}$ be the hypergeometric data $HD(r,s)$ associated to the left-hand column of families 4, 5 and 9 with minimal or maximal value of $r$. Then there are algebraic constants $\alpha_1,\alpha_2$ so that \begin{equation}\label{eq:3-term} F(\mathfrak{a})=\alpha_1 F(M_1\mathfrak{a})+\alpha_2F(M_2\mathfrak{a}). \end{equation} \end{theorem} From a transcendence point of view, it is especially notable that the constants are algebraic - this is not typically guaranteed for any three-term identity. For the remainder of this section, we prove a few similar identities that come directly from the $q$-expansions of modular forms. These identities essentially allow us to interchange transcendence bases. They are less crucial for proving Theorem \ref{main} than Theorem \ref{mainpar}, but are still useful at times. To demonstrate this, consider the family $$\{\mathbb{K}_2(1/2,3/4), \mathbb{K}_2(1/2,5/4)\},$$ which is self-twisted. This family is obtained by applying the Atkin-Lehner involution $W_2$ to $$\{\mathbb{K}_2(1/4,3/4),\mathbb{K}_2(3/4,5/4)\}.$$ Since both should span the space of of the same Hecke eigenform, we expect there to be relations between them. Using the Fourier coefficients, we can check that \begin{align*} f_{32.3.c.a}&=\frac{\mathbb{K}_2(1/2,3/4)(4\tau)+\mathbb{K}_2(1/2,5/4)(4\tau)}{2}+i\frac{\mathbb{K}_2(1/2,3/4)(4\tau)-\mathbb{K}_2(1/2,5/4)(4\tau)}{2}\\& =\mathbb{K}_2(1/4,3/4)(8\tau)+2i\mathbb{K}_2(3/4,5/4)(8\tau) \end{align*} Since the real and imaginary parts are the same, we obtain the following identity of eta products. \begin{proposition}\label{eta} Let $\tau\in \mathcal{H}$. Then \begin{align*} \mathbb{K}_2(1/4,3/4)(8\tau)=\frac{\eta(8\tau)^{12}}{\eta(4\tau)^2\eta(16\tau)^4}&=\frac{1}{2}\left(\eta(2\tau)^4\eta(8\tau)^2+\frac{\eta(4\tau)^{12}}{\eta(2\tau)^2\eta(8\tau)^4}\right)\\&=\frac{\mathbb{K}_2(1/2,3/4)(4\tau)+\mathbb{K}_2(1/2,5/4)(4\tau)}{2}. \end{align*} \end{proposition} Integrating both sides, we find the three-term identity $$F(1/4,3/4)/2={F(1/2,3/4)+F(1/2,5/4)}.$$ The more general statement in terms of Coxeter groups is the following. \begin{theorem}\label{threeterm} For $r=a/b$, $b=4,8,12,24$ and $a$ any integer coprime to $b$ and less than $b/2$, we have $$F(r,1-r)/2=F(AM_1(r,1-r))+F(A M_2M_1(r,1-r)).$$ Note that $AM_1(r,1-r)=(2r,r+1/2)$ and $A M_2M_1(r,1-r)=(2r,r+1).$ \end{theorem} \begin{proof} Set $s=1-r$. We will use that if $r=a/b$ where $a$ and $b$ are in lowest terms, then $N=2b$. First, note $$A(r,s)=(s-r,r)=(1-2r,r).$$ Also, Theorem \ref{kummer} by Kummer maps $$K(1-2r,r)=(2r,r+1/2)$$. The Fourier expansions of $\mathbb{K}_2(2r,r+1/2)(b\tau)$ and $\mathbb{K}_2(r,1-r)(2b\tau)$ both have initial term $q$, and they are in the same Hecke orbit. We can then check in each case that $\mathbb{K}_2(2r,r+1/2)(b\tau)$ and $\mathbb{K}_2(2r,r+1)(b\tau)$ are Kummer twists of each other, and their sum is equal to $\mathbb{K}_2(r,1-r)(2b\tau)$. Also, $$F(r,s)=-2\pi i\int_0^{i\infty}\mathbb{K}_2(r,s)(2b\tau)d\tau.$$ If we change the variable of $\mathbb{K}_2(2r,r+1/2)(b\tau)$ to $\mathbb{K}_2(2r,r+1/2)(2b\tau)$, that multiplies the integral by $2$. Multiplying through by $-2\pi i$ and integrating both sides of the above equality gives \begin{align*} & F(r,1-r)=-2\pi i\int_{0}^{i\infty} \mathbb{K}_2(r,1-r)(2b\tau)d\tau\\& =-2\pi i\int_0^{i\infty}\frac{\mathbb{K}_2(2r,r+1/2)+\mathbb{K}_2(2r,r+1)}{2}(2b\tau)d(2\tau)=2(F(2r,r+1/2)+F(2r,r+1)). \end{align*} \end{proof} Numerical evidence suggests Theorem \ref{threeterm} holds for other positive integers $b$ in addition to the ones listed, but a new proof would be needed. \smallskip \noindent \textit{Remark}: There are also versions of these identities for pairs $(r',s')$ conjugate to $(1-r,r)$ where $r'>1/2$ found using the two bases for the corresponding Hecke eigenform as above. We call these \textit{sister identities}. They have a less uniform shape than the identity above, and so we will only state them when we need them. \smallskip We also have the following associated to class 7. \begin{proposition}\label{fivterm} $$2F(1/8,5/8)=F(1/2,5/8)+F(1/2,11/8)+3F(1/2,7/8)+3F(1/2,9/8)$$ \end{proposition} There are also a few identities similar to the shape of Theorem \ref{threeterm} coming from certain non-Galois self-twists, namely 19 and 20. We record these 3 here as well. \begin{proposition} For $(r,s)=(5/4,11/8),(13/12,31/24)$, and $(17/12,35/24)$, we have $$16F(r,s) =F(1-r,s-1)-F(1-r,s).$$ \end{proposition} These are also related to the Atkin-Lehner involution, but this situation is somewhat more complicated then the Galois case. For now, we note that these identities can be proved by matching Fourier coefficients properly. Finally, there is a CM example which will be useful in the next section. \begin{proposition}\label{cmal} $$F(1/8,9/8)+2F(3/8,11/8) =4(F(1,9/8)+F(1,11/8)).$$ \end{proposition} This again can be proved by checking the Fourier expansions directly. \section{\textit{L}-Values of Newforms and the Proof of Theorem \ref{main}}\label{lvall} In this section, we prove Theorem \ref{main} in cases. Along the way, we discuss several applications. We divide the proof into CM cases and non-CM cases, which use very different techniques. \subsection{CM Examples}\label{cmex} CM modular forms, like CM abelian varieties, are better understood because of the structure given by the extra endomorphisms. The transcendental part is expected to have a closed form as a gamma quotient because each CM modular form is associated to a CM abelian variety (see \cite[Chapter 10]{stein} for example), and the $L$-function of a CM abelian variety is the product of two $L$-functions each corresponding to a Hecke Gr\"ossencharacter based on the work of Tanimaya \cite{tanimaya}, and Shimura \cite{shimuragross}. When $(r,s)$ satisfies either $r=1$ or $s=1/2$, the hypergeometric series reduces to a ${}_2F_1(1)$, which has the Gauss evaluation formula, $$\pfq{2}{1}{a,b}{,c}{1}=\frac{\Gamma(c)\Gamma(c-a-b)}{\Gamma(c-a)\Gamma(c-b)},$$ for $a,b,c$ rational numbers and $c>a+b$, (see \cite{aar} for the proof). For the other cases, we may be able to use evaluation formulas such as Dixon's formula, which are listed in Chapter 3 of \cite{aar}. In the case of weight 2 modular forms, the Gamma quotient mentioned above is the Chowla-Selberg period, so named because it is also a period on a CM elliptic curve. If $-D$ be a fundamental discriminant of an imaginary quadratic field, the Chowla-Selberg period is defined as $$ \Omega_{-D}:=\sqrt{\pi}\left(\prod_{i=1}^{D-1}\Gamma(i/D)^{\chi_D(i)}\right)^{1/2h'(-D)}, $$ where $\chi_D$ is the unique primitive quadratic character mod $D$, and $h'(-D)$ is $1/2,1/3$ for $D=-4,-3$ and is the class number $h(-D)$ otherwise (see, for example, \cite{zagier} or \cite{lilongliuCM}). A well-known result of Nesterenko \cite{nesterenko} mentioned in the introduction says that $\pi$ and $\Omega_{-D}$ are algebraically independent. Therefore, it is useful from a transcendence perspective if we can write our $L$-values in terms of a Chowla-Selberg period, perhaps up to multiplication by $\pi$. In Proposition \ref{chowlll}, we show that for all of our CM newforms $f$ besides possibly one, we have $L(f,1)$ is a multiple of $\pi^{-1}$ multiplied by a Chowla-Selberg period squared. When working with Gamma quotients, we will use the following fundamental facts about the Gamma function constantly: for any real number $x\not \in \Z$ \begin{equation}\label{gamma} \Gamma(x+1)=x\Gamma(x)\quad \text{ and } \quad \Gamma(1-x)\Gamma(x)=\frac{\pi}{\sin(\pi x)}. \end{equation} These are called the functional equation and the reflection formula. The other useful formula is the multiplication formula, for any real number $x$ and positive integer $m$ $$\prod_{k=0}^{m-1}\Gamma(z+k/m)=\pi^{\frac{m-1}{2}}m^{1/2-mz}\Gamma(mz).$$ See \cite{steinshakarchi} for proofs. Whenever we suddenly simplify a Gamma quotient below, we are implicitly using these three properties. We start this section by briefly discussing some properties of non-Galois cases that we will use in stating Proposition \ref{chowlll}. Then we prove Proposition \ref{chowlll}, and in the final section, address the unusual example that is Class 5. By then, we will have proved the CM part of Theorem \ref{main} and then make several other related observations as well. \subsubsection{Non-Galois Cases}\label{ngall} All the non-Galois cases are related to CM modular forms. However, we do not expect the space of differentials of the first kind to have dimension 1 as in Theorem \ref{main}. In the classical case of abelian varieties, the non-Galois setting is comparable to the product of two CM abelian varieties with different CM discriminant. In this case, the more general version of W\"ustholz's theorem given in \cite{cohen} predicts the periods of differentials of the first kind should have dimension 2. Understanding the $\mathbb{K}_2(r,s)$ functions is also harder in these cases. Using the same rough analogy as above, $\mathbb{K}_2(r,s)$ may be a differential on one CM abelian variety or the other, or it may be a linear combination of differentials on both. As a result, sometimes $\mathbb{K}_2(r,s)$ is in the Hecke orbit of a single modular form, and sometimes it is in the Hecke orbit of multiple modular forms. In the extreme case of Class 12, all the $\mathbb{K}_2(r,s)$ are linear combinations of multiple modular forms. We will illustrate with a representative example. Our example will be class 14. The holomorphic conjugates are $$\hat{S}:=\{\mathbb{K}_2(1/8,11/8),\s\mathbb{K}_2(5/8,7/8),\s\mathbb{K}_2(3/8,9/8),\s\mathbb{K}_2(1/8,3/8),\s\mathbb{K}_2(9/8,11/8)\}.$$ \begin{Lemma}\label{ngal} We have $F(1/8,11/8)$ and $F(5/8,7/8)$ are in $\Bar{\Q}\cdot \pi^{-1}\Omega_{-4}^2$ and $F(3/8,9/8)\in \Bar{\Q}\cdot \pi^{-1} \Omega_{-8}^2$. Moreover, $$F(1/8,3/8)=F(1/8,11/8)+16F(9/8,11/8).$$ \end{Lemma} \smallskip \noindent\textit{Remark}: The existence of the second identity has an explanation in terms of hypergeometric series. In particular, note that the parameters of the three hypergeometric series are the same modulo $\Z$. In the classical theory of hypergeometric functions, this implies there are \textit{contiguous relations} between them of the above shape (see \cite{ebisu}). \smallskip \begin{proof} The first three hypergeometric series can be evaluated as a gamma quotient using Dixon's formula. Then we can prove the relation with the Chowla-Selberg periods directly using (\ref{gamma}). For example, it is straightforward to check that $$2\sqrt{-1+\sqrt{2}}F(1/8,11/8)=\pi^{-1} \Omega_{-4}^2.$$ The final identity is proved by comparing the Fourier expansions of the underlying $\mathbb{K}_2(r,s)$ functions. \end{proof} The two Chowla-Selberg periods appearing in the Lemma correspond to the two modular forms 128.3.d.a and 128.3.d.b listed in the table for non-Galois classes, which have CM discriminant $-4$ and $-8$ respectively. Also, note that as a period on a hypergeometric surface, $F(r,s)$ should be replaced with $\pi 2^{4r-1}/N F(r,s)$. This makes the Chowla-Selberg period squared the transcendental part of our hypergeometric period. However, we retain the normalization $F(r,s)$ throughout for consistency. \begin{corollary} The vector space over $\Bar{\Q}$ generated by $1,\pi i$, and the vector space generated by the periods of the differentials in $\hat{S}$ has dimension greater than or equal to 4. \end{corollary} \begin{proof} By a well-known result of Nesterenko \cite{nesterenko}, $\pi$, $\Omega_{-4}$, and $\Omega_{-8}$ are algebraically independent. The result then follows from Lemma \ref{ngal}. \end{proof} \noindent\textit{Remark}: If we could express $F(1/8,3/8)$ or $F(9/8,11/8)$ in terms of one of the Chowla-Selberg periods above, this would be an equality. We expect this to be the case. However, there are no evaluation formulas we are aware of for these cases. \smallskip Despite the global situation being more complicated, we can still use the modular forms in $\hat{S}$ to construct a Hecke eigenform. Specifically, a computation shows that \begin{corollary}\label{local} $$L(f_{128.3.d.a},1)=F(1/8,11/8)+8iF(5/8,7/8)=\frac{1}{4} \sqrt{-\frac{1}{2} + \frac{i}{2}}\pi^{-1}\Omega_{-4}^2.$$ \end{corollary} All the modular forms listed in the columns LMFDB 1 or K-LMFDB 1 can be constructed locally in this way. In the next section, we will discuss the special $L$-values of these modular forms uniformly across the Galois and non-Galois cases using this idea. \subsubsection{The Transcendental Part of CM Examples} Above, we computed the exact $L$-value of \noindent $L(f_{128.3.d.a},1)$ as an algebraic multiple of $\pi^{-1}\Omega_{-4}^2$. A simpler example is Class 1, where it is already known from \cite{osburnstraub} that $$L(f_{16.3.c.a},1)=F(1/2,1)=\frac{1}{16}\pi^{-1}\Omega_{-4}^2.$$ We notice a similar pattern for the transcendental part of the other CM weight 3 modular forms. \begin{proposition}\label{chowlll} Assume $f$ is a CM newform of discriminant $-D$ listed either in Table \ref{tab}, classes 1-4, or Table \ref{nongalois}, in the column labeled LMFDB 1 or K-LMFDB 1 when a modular form is listed. Let $\hat{S}=\{\mathbb{K}_2(r_i,s_i)\}.$ Then the vector space over $\Bar{\Q}$ generated by the periods of $\hat{S}$ has dimension 1. Moreover, $$L(f,1)\in \Bar{\Q}\cdot \pi^{-1}\Omega_{-D}^2.$$ \end{proposition} \begin{proof} The proof is on a case by case basis. It would be very interesting, but potentially difficult, to find a geometric proof. We use Gauss evaluation, Dixon's Theorem (see \cite{aar}, Theorem 3.4.1), and Theorem 3.5.5 of \cite{aar} to evaluate the hypergeometric series. This is possible for all the series for the families listed above except for family 4.a of Class 4. The key is to use Proposition \ref{cmal}. The left-hand side of the proposition is precisely $L(f_{32.3.d.a},1)$, so we get $$L(f_{32.3.d.a},1)=4(F(1,9/8)+F(1,11/8)).$$ Now, the right-hand side can be evaluated using Gauss summation, and we obtain $$L(f_{32.3.d.a},1)=2\sqrt{2}\pi^{-1}\Omega_{-8}^2.$$ We can also evaluate family 4.b using Gauss evaluation directly, and obtain a similar result. Moreover, we check that each $F(r,s)$ in the family is itself a multiple of $\pi^{-1}\Omega_{-8}^2$. We have already covered Classes 1 and 2, and Class 3 is addressed in \cite{aglt}, Corollary 6.4 of Part II. Therefore, we are finished with the Galois cases. We also already discussed Class 14 with Lemma \ref{ngal}. For all the classes up to this point, the desired properties are easily computed symbolically by a program such as Mathematica or Maple. We have also verified the computations by hand. The remaining cases are Class 13 and Class 15. For these, Mathematica can compute the linear dependence between the periods of $\hat{S}$ efficiently, but has a harder time showing that these periods are multiples of $\pi^{-1}\Omega_{-D}$. Both cases rely on a careful application of multiplication by 3. We will provide the relations for convenience. We have $$F(1/12,1/6)=\sqrt{\frac{1}{8}+\frac{1}{4\sqrt{3}}}\pi^{-1}\Omega_{-4}^2; \quad F(1/24,35/24)=\sqrt{2}3^{3/4} \csc(\pi/8)^2 \sin(\pi/24)\pi^{-1}\Omega_{-4}^2.$$ \end{proof} \subsubsection{Class 5}\label{class5} As mentioned earlier, Class 5 is a special case. One reason is we are able to study differentials of the second kind in this class. We expect the differentials of the second kind for the other Galois CM examples to behave similarly. More importantly, we will also see below that it produces a modular form with an unexpected transcendental part to its $L$-value, based on Proposition \ref{chowlll} To start, the structure of the conjugates is very different from any other Galois family. There are three pairs in the class, all of which are conjugate: $\mathbb{K}_2(1/6,1/3), \mathbb{K}_2(1/6,4/3), \mathbb{K}_2(7/6,4/3)$. In particular, $\text{Stab}(1/6,4/3)=D_6$. The relation between this is \begin{equation*} \mathbb{K}_2(1/6,1/3) = \mathbb{K}_2(1/6,4/3)+ 16\mathbb{K}_2(7/6,4/3). \end{equation*} As an identity of hypergeometric functions recalling $M_2=KAK$, this is \begin{equation}\label{contiguous} -F(1/6,1/3)+F(A(1/6,1/3))+16F(M_2(7/6,4/3))=0. \end{equation} This is a contiguous identity, as in Lemma \ref{ngal}. Below, we provide the graph for the class. \begin{figure}[htbp] \centering \includegraphics[scale=.4]{class5graph.png} \caption{The graph for class 5 in Table \ref{tab}} \label{fam11} \end{figure} It turns out that the non-holomorphic conjugates arising from the Kummer transformation given above are periods of differentials of the second kind. Theorem \ref{wust} in \cite{wustholz} gives the transcendence degree of differentials of the first and second kind of abelian varieties. All the periods addressed in this paper so far involve periods of differentials of the first kind. Generally, there is not an easy description of a basis for periods of differentials of the second kind for our surfaces. This makes Class 5 especially nice. Explicitly, this class consists of three holomorphic conjugates: set $$f_1:=2^{-1/3}\pi F(1/6,1/3), f_2:=2^{-1/3}\pi F(1/6,4/3), \text{ and } f_3:=2^{11/3}\pi F(7/6,4/3),$$ and denote the set of all of them by $S$. For $f_2$ and $f_3$, the Kummer transformation maps to the period of a differential with a simple pole at one of the cusps; that is a differential of the second kind. For $f_1$, the Kummer transformation maps to something that isn't convergent, $F(5/6,2/3)$, but we can replace this with $F(-1/6,2/3)$. Then we get $S'$, consisting of the elements $$ f_1':=2^{-5/3}\pi F(-1/6,2/3), \quad f_2':=2^{7/3}\pi F(5/6,5/3), \quad f_3':=2^{-5/3}\pi F(-1/6,5/3). $$ The normalization is so that the values are the actual periods of the hypergeometric surface over $\Bar{\Q}$. \begin{proposition}\label{cmwus} The dimension of the vector space generated by $S$ and $S'$ over $\Bar{\Q}$ is 2. \end{proposition} \begin{proof} We can evaluate $f_1$ using Theorem 3.5.5 of \cite{aar} and $f_2$ using Dixon's formula (see \cite{aar} Theorem 3.4.1): $$f_1=\frac{2 \pi^2 \Gamma(1/6)^2}{\Gamma(5/12)^2 \Gamma(3/4)^2} \quad \text{ and } \quad f_2=\frac{\Gamma(1/6) \Gamma(1/4) \Gamma(7/12)^2}{\sqrt{2} \Gamma(3/4) \Gamma(5/6)}.$$ Using Theorem \ref{kummer}, we can evaluate $f_1'$ and $f_2'$ as well. Moreover, we can easily check that $f_1/f_2=2-\sqrt{3}$ by Equation \eqref{gamma}, and similarly that $f_1'/f_2'$ is an algebraic number. We can also check that $f_3'$ can be evaluated using Dixon's formula and that $f_3'/f_2'$ is algebraic. This proves that all three elements of $S'$ are linearly dependent over $\Bar{\Q}$. There is not a formula in \cite{aar} to evaluate $f_3$, but we may use formula \ref{contiguous} to evaluate $f_3$ as a linear combination of $f_1$ and $f_2$. This proves that all three elements of $S$ are linearly dependent over $\Bar{\Q}$ as well. Finally, we need to show the remaining two elements are linearly independent over $\Bar{\Q}$. It suffices to consider $f_2$ and $f_2'$. After some simplification, we find the transcendental part of $f_2$ is $B(1/4, 1/6)^2$ and the transcendental part of $f_2'$ is $B(1/4, 5/6)^2$. Any $B(r,s)$ for $r,s\in \Q$ is known to be transcendental (see \cite{waldschmidt}), and we can see that they are linearly independent of each other because if $aB(1/4,1/6)^2+bB(1/4,5/6)^2=0$, then $a/b=-B(1/4,5/6)^2/B(1/4,1/6)^2$ is algebraic. But the transcendental part of $B(1/4,5/6)/B(1/4,1/6)$ is $\Gamma(5/6)^6/\pi$, which is transcendental because $\Gamma(5/6)$ is known to be transcendental and algebraically independent of $\pi$. Therefore, this is impossible. \end{proof} W\"ustholz's result predicts for CM abelian varieties that the vector space generated by $1,\pi, S$ and $S'$ has dimension 4, so this is a partial result. Almost certainly, the analogue of W\"ustholz's result holds in our case, but actually proving it may require a geometric argument outside the scope of this paper. We state it as a conjecture. \begin{Conjecture} The vector space generated by $1, 2\pi i$, and the differentials of the first and second kind on the hypergeometric surface associated to $(1/4,1/6)$ has dimension 4, with a basis $1,\pi,$ $B(1/4,1/6)^2$, and $B(1/4,5/6)^2$. \end{Conjecture} A theorem of Wolfart and W\"ustholz says that there are no linear relations among beta values besides those coming from the Deligne-Koblitz-Ogus relations (see \cite{waldschmidt}), which suggests this Conjecture may be true but seems to be insufficient to prove it because the beta values are squared. For elliptic curves, we know the product of periods of the first and second kind is a multiple of $\pi$. We have an analogous result in our setting. \begin{proposition}\label{alg} We have $f_2f_2'=24\pi^2(-3+2\sqrt{3})$. \end{proposition} \begin{proof} Using the evaluations above and the reflection formula to get $$f_2f_2'=\frac{9(-2+\sqrt{3})^2\Gamma(1/4)^4\Gamma(7/12)^2\Gamma(11/12)^2}{2\pi^2}.$$ Then use multiplication by 3 on $\Gamma(1/4)^2\Gamma(7/12)^2\Gamma(11/12)^2$ to get a multiple of $\Gamma(3/4)^2$, and then simplify using reflection to get the result. \end{proof} We also note that we can complete a Hecke eigenform using class 5. Specifically, $$f_{144.3.g.c}=\frac{\mathbb{K}_1(1/6,1/3)+\mathbb{K}_1(1/6,4/3)}{2}+8\sqrt{-3}\mathbb{K}_1(7/6,4/3).$$ As an application of formula \ref{lval} and the transcendence result above, we know $L(f_{144.3.g.c},1)\in \pi^{-1}B(1/4,5/6)^2\Bar{\Q}$. \begin{corollary} We have $$L(f_{144.3.g.c},1)=\frac{(1 + i)\sqrt {3}}{(1 + \sqrt{3})12} \pi^{-1} B(1/4,1/6)^2.$$ \end{corollary} The $\pi^{-1}$ appears because the $L$-values are in terms of $F(1/6,1/3)$. A priori, we are not guaranteed to be able to relate the transcendental part of the $L$-value to the Chowla-Selberg formula, because these are not periods on an elliptic curve as in the weight 2 case (see e.g. \cite{lilongliu} or \cite{rosen}). However, based on Proposition \ref{chowlll}, our expectation is that $B(1/4,1/6)^2$ should be a multiple of $\Omega_{-4}^2$. Strangely, this does not appear to be the case. Note $$B(1/4,1/6)^2/\Omega_{-4}^2=\frac{\Gamma(1/6)^2\Gamma(3/4)^2}{\pi \Gamma(5/12)^2}.$$ There does not seem to be any multiplication formula that can reduce this to an algebraic number. The Gross-Deligne period conjecture discussed in \cite{otsubo} may also be relevant here, but we have not checked the relevant geometric properties of the sub-Hodge structure. \subsection{From Kummer to Shimura} In this section, we will give the proof of Theorem \ref{main} in the non-CM cases. Similar to above, this is on a case by case basis, but the techniques used to find relations between the periods is more involved than simply using well-known properties of the Gamma function. A fundamental tool is the following Theorem. \begin{theorem}[Shimura \cite{shimurazeta}]\label{shimura} Suppose $f$ is a newform of weight $k>2$, $0<m<k$ is an integer, and $\chi$ a finite order character so that $\chi(-1)^{m-1}=1$. Then $L(f,1)=A\cdot (2\pi i)^{m-1} g(\chi)^{-1} L(f\otimes \chi,m)$, where $g(\chi)$ is a Gauss sum and $A=A(f,m,\chi)$ is an algebraic number depending on $f,m,\chi$. Moreover, if $K_f$ is the field generated by the Hecke eigenvalues of $f$, then $A$ lies in $K_f$, and if $\sigma$ is a nontrivial automorphism of $K_f$, then $$A^\sigma=L(f^\sigma,1)/[(2\pi i)^{m-1} g(\chi^\sigma)^{-1} L(f^\sigma\otimes \chi^\sigma,m)].$$ \end{theorem} We rely on Theorem \ref{shimura} where $\chi$ is a quadratic character to prove Theorem \ref{main}. Remarkably, the identities resulting from Theorem \ref{main} then allow us to explicitly write down the algebraic constant guaranteed by Theorem \ref{shimura} for some non-quadratic twists that arise from the Kummer twist, Theorem \ref{twisting}. These formulas, like equation (\ref{L}) from the introduction, interrelate our transcendence results (Theorem \ref{main}), the Kummer transformation and Theorem \ref{twisting}, and the Shimura's Theorem \ref{shimura} in an attractive way. The general approach to finding $L$-value relations arising from the Kummer twist Theorem \ref{twisting} in the non-CM setting is to use the Kummer Transformation \ref{kummer} by Kummer from earlier, which states $$F(r,s)=2^{4-8r}C(r,s)F(1-r,1/2+q-r)$$ in terms of $F(r,s)$, where \begin{equation}\label{crs} C(r,s)=\frac{\Gamma(r) \Gamma(s - r)^2}{\Gamma(1 - r) \Gamma(s-1/2)^2 }. \end{equation} In most cases, $C(r,s)$ does not appear to be algebraic. Among the congruence examples, the only times it is algebraic can be summarized by the fact $$F(r,1)=2^{-4+8r}\csc(\pi r)F(1-r,3/2-r)$$ and $$F(r,1-r)=2^{-4 r} \csc(\pi r)F(1-r,3/2-2r),$$ for any rational $r$. In terms of transcendence, the algebraic condition means that the families, e.g., $F(r,1)$ and $F(r,r+1/2)$ give two different transcendence bases over $\Bar{\Q}$ for the space of periods. There are only five classes in Table \ref{tab} where $C(r,s)$ is algebraic, namely 6, 7, 8, 9, and 10, excluding degenerate cases where one of the beta factors is 0 (this happens when $s=1/2$). Notably, these 5 classes are all non-CM. The simplest non-CM example is class 6, which is already addressed in \cite{aglt}. Using Corollary \ref{lval2}, we have $$L(f_{32.3.c.a},1)=F(1/4,3/4)+4iF(3/4,5/4)$$ and $$L(f_{64.3.c.b},1)=F(1/4,1)+4iF(3/4,1).$$ In this case, Theorem \ref{kummer} takes the form \begin{equation}\label{eq:16} \begin{split} F(1/4,3/4)&=\sin(\pi/4)F(3/4,1)\\ F(3/4,5/4)&=\sin(3\pi/4)F(1/4,1). \end{split} \end{equation} The critical fact is then that $\sin(\pi/4)=\sin(3\pi/4)$ and so we can factor out $\sin(\pi/4)$, proving that \begin{equation}\label{eq:17} L(f_{32.3.c.a},1)=\sin(\pi/4)[4F(3/4,1)+iF(1/4,1)]=\frac{\sqrt{2}}{2}L( f_{64.3.c.b},1). \end{equation} The situation is exceptionally nice because we expect the transcendence degree of the space of differentials over $\Bar{\Q}(\pi i)$ to be 2, and we only have two periods, $F(1/4,3/4)$ and $F(3/4,5/4)$, which we anticipate to be algebraically independent; see the discussion under Section \ref{trans}, Transcendence. In other words, we \say{extend} the identities \eqref{eq:16} to an identity of $L$-functions, \eqref{eq:17}, $$L(f_{32.3.c.a},1)=\sin(\pi/4)L(f_{64.3.c.b},1).$$ However, when the denominator of $r$ is e.g. 8 (classes 7 and 8), $\sin(\pi/8)$ does not equal $\sin(3\pi/8)$, and so we cannot use this method directly. To generalize this to classes 7, 8, and 9, we use the (conjectural) transcendence basis over $\Bar{\Q}$ from Theorem \ref{main} and express the remaining two periods in terms of it. These families have extra structure from the underlying potentially 2-isotypic Galois representations, which explains the existence of the extra relations. After doing this, the Kummer transformation almost magically extends to an identity of $L$-functions as predicted by Shimura. This occurs because of the deep motivic meaning of the modularity result in \cite{aglt} relating the two hypergeometric series to the two newforms, translated from the \'Etale setting into the complex (de Rham) setting. \subsection{Non-CM Examples} We will address families 4, 5, and 9. Our approach heavily relies on Shimura's Theorem \ref{shimura} . \subsubsection{Class 7} In the following four sections, we derive $L$-value relations similar to those above for several non-CM cases. \begin{theorem}\label{f5} $L(f_{64.3.d.a},1)=-\frac{1}{2}\zeta_{48}L(f_{256.3.c.g},1).$ \end{theorem} The core of the proof is the following Lemma. Despite outwardly being a transcendence result about hypergeometric series, the proof of the Lemma relies on a surprising application of Theorem \ref{shimura}. \begin{Lemma}\label{l1} $$F(1/8,5/8)-8iF(7/8,11/8)=(1-\sqrt{-2})(2F(3/8,7/8)+4iF(5/8,9/8)).$$ \end{Lemma} \begin{proof} The newform $f_{64.3.d.a}$ has one even inner twist $\chi_{8.b}$, given by the quadratic Dirichlet character 8.b in the LMFDB. For this proof, take $f=f_{64.3.d.a}$. Then, since the twist is even, we have by Shimura's result that $L(f\otimes \chi_{8.b},1)$ and $L(f,1)$ differ by an algebraic number. The Gauss sum of the Dirichlet character is $\sqrt{8}$, and so Shimura further predicts that $\sqrt{8}\alpha$ belongs to the Hecke eigenvalue field of $f$, which is $\Q(\zeta_{12})$. We may write $L(f\otimes \chi_{8.b},1)$ and $L(f,1)$ in terms of hypergeometric series using Corollary \ref{lval2}, as $\chi_{8.b}$ is a quadratic character with conductor dividing 16. Then, rearranging the relation $L(f,1)=\alpha L(f\otimes \chi_{8.b},1)$, we deduce $$ F(1/8,5/8)-8iF(7/8,11/8)=\frac{\sqrt{3}(1-\alpha)}{1+\alpha}(2F(3/8,7/8)+4iF(5/8,9/8)).$$ Now, let $\sigma$ denote the automorphism of $\C$ obtained by lifting the unique nontrivial automorphism of $\Q(\sqrt{3})$. By construction, $f^\sigma=f\otimes \chi_{8.b}$. The second part of Shimura's theorem implies $$(\sqrt{8}L(f,1)/L(f\otimes \chi_{8.b},1))^\sigma=(\sqrt{8}\alpha)^\sigma=\sqrt{8} L(f^\sigma,1)/L(f,1)=8/(\sqrt{8}\alpha).$$ Let $\alpha':=\sqrt{8}\alpha$, an element of $\Q(\zeta_{12})$ by Shimura. In general, we may write elements of $\Q(\zeta_{12})$ as $\alpha'=a+b\sqrt{3}$ for $a,b\in \Q(i)$, and then $$1/\alpha'=\frac{a-b\sqrt{3}}{a^2-3b^2}.$$ However, from above $\sigma(\alpha')=a-b\sqrt{3}=8/\alpha'$, and so $a^2-3b^2=8$. This implies the norm $N_{\Q(\zeta_{12})/\Q(i)}(\alpha')=8$, and so the absolute norm is 64. Up to multiplication by a unit in the ring of integers, the only element of norm 64 in $\Q(\zeta_{12})$ is $2+2 i$. This is because 2 a divisor of 64, so 2 is ramified in $\Q(\zeta_{12})$ and factors as $(2)=(1+i)^2$. Moreover, the unit group of $\Q(\zeta_{12})$ has rank 1 and is generated by $\zeta_{12}-1$. Combining these two facts, we get that $[\alpha'/(2+2i)]^{12}=(\zeta_{12}-1)^n$ for some integer $n$. The elements of the unit group lie in a lattice in Minkowski space, so we should be able to determine them numerically. In particular, let $B=|\zeta_{12}-1|$, where the bars denote complex modulus. Then we have $$\log_B|[\alpha'/(2+2i)]^{12}|=n.$$ Since $n$ is an integer and the integers are discrete in the reals, there is an open ball of radius, say, 1/4, around $n$ so that $n$ is the only integer in this interval. We can compute $\log_B|[\alpha'/(2+2i)]^{12}|$ numerically to a high enough level of precision that we can determine it is within the ball of radius 1/4 around 12, and so we conclude $n=12$. Therefore, we have $\alpha'/(2+2i)=(\zeta_{12}-1)\epsilon$, where $\epsilon$ is a 12th root of unity. The 12th roots of unity can be distinguished numerically, and so we check that $\epsilon=-\zeta_{12}^{10}$. In conclusion, $\alpha'=-(2+2i)(\zeta_{12}-1)\zeta_{12}^{10}=-2 i (-1 + \sqrt{3})$. From this, we can unravel the desired identity. \end{proof} An immediate byproduct of the proof is the following relation between $f$ and the even inner twist $\chi_{8.b}$. \begin{corollary} $L(f_{64.3.d.a},1)=-i\sqrt{2-\sqrt{3}}L(f_{64.3.d.a}\otimes \chi_{8.b},1)$ \end{corollary} \begin{proof} We showed above that $L(f_{64.3.d.a},1)=-2\sqrt{8} i (-1 + \sqrt{3})=L(f_{64.3.d.a}\otimes \chi_{8.b},1)$, and \noindent $-2\sqrt{8} i (-1 + \sqrt{3})=-i\sqrt{2-\sqrt{3}}$. \end{proof} Taking the real and imaginary part of the identity in Lemma \ref{l1} shows all four conjugates can be expressed in as a $\Bar{\Q}$-linear combination of $F(3/8,7/8)$ and $F(5/8,9/8)$, and so this identity, and the similar identities in the remainder of this section, prove Theorem \ref{main} for this particular case. There is a similar identity to Lemma \ref{l1} for the twisted family. Luckily, we can easily derive it from the Lemma. \begin{Lemma}\label{l2} $F(1/8,1)+8iF(7/8,1)=-2(1 - \sqrt{-2}) (1 + \sqrt{2})(F(3/8, 1) - 2 i F(5/8, 1)).$ \end{Lemma} \begin{proof} Apply the Kummer transformation to each side of the above Lemma, which in this case has the form $F(1-r,3/2-r)=\sin(\pi r)2^{4-8r}F(r,1),$ so we get $$\sin(\pi/8)(8F(7/8,1)+iF(1/8,1))=2\cos(\pi/8)(1-\sqrt{-2})(2F(5/8,1)+iF(3/8,1)).$$ Multiply though by $i$ and note that $\cot(\pi/8)=1+\sqrt{2}$ and we arrive at the desired identity. \end{proof} With these two identities in hand, the proof of Theorem \ref{f5} is an intricate but straightforward computation. \begin{proof}[Proof of Theorem \ref{f5}] Using Lemma \ref{l1}, compute \begin{align*} & L(f_{64.3.d.a},1)\\&=F(1/8, 5/8) - 2 \sqrt{3}F(3/8, 7/8) - 4 \sqrt{-3} F(5/8, 9/8) - 8 i F(7/8, 11/8)\\& =(F(1/8, 5/8)- 8 i F(7/8, 11/8)) -2 \sqrt{3}(F(3/8, 7/8) + 2i F(5/8, 9/8))\\& =2(1-\sqrt{-2})(F(3/8,7/8)+2iF(5/8,9/8))-2 \sqrt{3}(F(3/8, 7/8) + 2i F(5/8, 9/8))\\& =2(1-\sqrt{-2}-\sqrt{3})(F(3/8,7/8)+2iF(5/8,9/8)). \end{align*} Then apply the Kummer transformation to get \begin{align*} & 2(1-\sqrt{-2}-\sqrt{3})(F(3/8,7/8)+2iF(5/8,9/8))\\& =2(1-\sqrt{-2}-\sqrt{3})(2 \cos(\pi/8) F(5/8, 1)+i\cos(\pi/8)F(3/8,1))\\& =2i\cos(\pi/8)(1-\sqrt{-2}-\sqrt{3})(F(3/8,1)-2iF(5/8,1)). \end{align*} We now need to do the first half of this process in reverse using Lemma \ref{l2} to get the desired identity. \begin{align*} & L(f_{256.3.c.g},1)\\&=F(1/8, 1) - 2 \sqrt{-3} F(3/8, 1) - 4 \sqrt{3} F(5/8, 1) + 8 i F(7/8, 1)\\& =(F(1/8, 1)+ 8 i F(7/8, 1)) -2 \sqrt{-3}(F(3/8, 1) - 2i F(5/8, 1))\\& =-2(1 - \sqrt{-2}) (1 + \sqrt{2})(F(3/8,1)-2iF(5/8,1))-2 \sqrt{-3}(F(3/8, 1) - 2i F(5/8, 1))\\& =-2((1 - \sqrt{-2}) (1 + \sqrt{2})+\sqrt{-3} )(F(3/8,1)-2iF(5/8,1)). \end{align*} Putting the two computations together, we get the relation $$L(f_{64.3.d.a},1)=\frac{2i\cos(\pi/8)(1-\sqrt{-2}-\sqrt{3})}{-2((1 - \sqrt{-2}) (1 + \sqrt{2})+\sqrt{-3})}L(f_{256.3.c.g},1).$$ Simplifying this constant and dividing to the other side of the equation gives us the desired result. \end{proof} \subsubsection{Class 8} The situation for this class is very similar to the previous case. \begin{theorem}\label{f4} $L(f_{256.3.c.f},1)=\sqrt{2 - \sqrt{2} - 4 i \sqrt{3 - 2 \sqrt{2}}}L(f_{128.3.d.c},1)$. \end{theorem} Like the previous example, there is the following key lemma. \begin{Lemma}\label{l3} $$F(1/8, 7/8) + 8i\sqrt{2} F(7/8, 9/8) = \zeta_8 (2\sqrt{2} F(3/8, 5/8) + 4i F(5/8, 11/8))$$ \end{Lemma} \begin{proof} Similar to before, we twist $f_{128.3.d.c}$ by the character $\chi_{8.b}$, which is an even twist, so will give us an algebraic relation between the $L$-values. We have $g(\chi_{8.b})=\sqrt{8}$, and again let $\alpha$ be the constant relating the two $L$-values, and $\alpha'=\sqrt{8}\alpha$. Take $\sigma$ to be the automorphism of $\Q(\zeta_8)=\Q(\sqrt{2},i)$ which sends $\sqrt{2}\mapsto-\sqrt{2}$, $i\mapsto-i$. Then $f^\sigma=f\otimes \chi_{8.b}$ as above. From this, we can deduce that $\sigma(\alpha')=8/\alpha'$, and by the same argument, it follows that $\alpha'$ has norm 64. We can take $2+2i$ again as the generator of the unique ideal of norm 64 in $\Q(\zeta_8)$, since again, 2 is ramified. A fundamental unit is $u:=\zeta_8^2+\zeta_8+1$, and take $B=|u|$. Then we can compute $\log_B(|\alpha'/(2+2i)|^8)=8$ and using another numerical calculation, determine that $\alpha'/(2+2i)=-u$. Simplifying, we get $\alpha'=-2 i (2 + \sqrt{2})$, and then by rearranging the resulting equality of $L$-values we arrive at the identity above.\end{proof} We also have the same two corollaries as above, which are proved in an identical manner. \begin{corollary} $L(f_{128.3.d.c},1)=-i\sqrt{3+\sqrt{2}}L(f_{128.3.d.c}\otimes \chi_{8.b},1)$ \end{corollary} \begin{Lemma}\label{l4} $F(1/8,3/4))-8\sqrt{-2}F(7/8, 5/4) = \zeta_8^3(1-\sqrt{2}) (4F(5/8, 11/8) + 2\sqrt{-2} F(3/8, 5/8))$. \end{Lemma} \begin{proof} Apply Kummer's Theorem \ref{kummer} to Lemma \ref{l3} to get $$\csc(\pi/8)(4\sqrt{2}F(7/8, 5/4) + 1/2 F(1/8,3/4)) = \sec(\pi/8) \zeta_8 (2F(5/8, 11/8) + \frac{2i}{\sqrt{2}} F(3/8, 5/8)).$$ Multiplying both sides by 2 and factoring out $i$ from the left-hand side, we arrive at the desired identity after simplifying. \end{proof} \begin{proof}[Proof of Theorem \ref{f4}] This is similar to the proof of the previous theorem. Using Lemma \ref{l3}, compute \begin{align*} & 16L(f_{128.3.d.c},1) =(\zeta_8+1)(2\sqrt{2}F(3/8, 5/8) + 4i F(5/8, 11/8)). \end{align*} Then apply the Kummer transformation to get \begin{align*} L(f_{128.3.d.c},1) & =(\zeta_8+1)\frac{\sec(\pi/8)}{\sqrt{2}} (\sqrt{2}F(5/8, 3/4)+2iF(3/8,5/4))\\& =1/2(\zeta_8+1)\sec(\pi/8) (4F(5/8, 3/4)+2\sqrt{-2}F(3/8,5/4)) \end{align*} We now need to do the first half of this process in reverse using Lemma \ref{l4} to get the desired identity. \begin{align*} & L(f_{256.3.c.f},1) =(\zeta_8^3(1-\sqrt{2})+1)(2 \sqrt{-2} F(3/8, 5/4) + 4 F(5/8, 3/4)). \end{align*} Putting the two computations together, we get the relation $$L(f_{128.3.d.c},1)=\frac{1/2(\zeta_8+1)\sec(\pi/8)}{(\zeta_8^3(1-\sqrt{2})+1)}L(f_{256.3.c.f},1).$$ Simplifying this constant gives us the desired result. \end{proof} \subsubsection{Class 9} \begin{theorem}\label{f9} $L(f_{576.3.g.d},1)=\sqrt{3}\zeta_8^7L(f_{288.3.d.a},1)$. \end{theorem} The proof of the key lemma in this case requires several more steps, because there is no even inner twist of $f_{288.3.g.a}$. We still have a twist switching the signs of $5$ and $7$, but it is not an inner twist. Crucially, this means that we cannot use the second part of Shimura's result, so the proof will be slightly different. We rely on a result of \cite{thomae}, also given in \cite{bailey} and \cite{coxeter} to handle this. We also rely on the following conjecture. \begin{Conjecture}\label{con} $F(5/12,7/12)/F(7/12,17/12)$ is not an element of $\Q(\sqrt{3})$. \end{Conjecture} As discussed below, we expect that these two should even be algebraically independent, so this is not too surprising of a claim, but attempting to prove it is outside of the scope of this paper. \begin{proposition}[Thomae]\label{thomae} For any rational $r$ and $s$, we have that $$F(r,s)=-2^{6-4s-4r}\frac{\cos(\pi s)}{\cos\pi(r-1/2)} F(3/2-s,3/2-r)-2^{4s-8r+1}\frac{\sin(\pi r)}{\sin\pi(r-1/2)}F(s-r,s).$$ \end{proposition} \begin{proof} The identity is derived from a special case of a result of \cite{bailey}: for a normalized ${}_3F_2$ we will denote $\hat{F}(a,b,c,d,e)$ as defined in Beyer et al. \cite{coxeter}, $$\hat{F}(1/2,1/2,r,1,s)=\alpha_1\hat{F}(1/2,1/2,3/2-s,1,3/2-r)+\alpha_2\hat{F}({{r}, {1 + r - s}, {r}, {1/2 + r}, {1/2 + r},})$$ for explicit constants $\alpha_1$ and $\alpha_2$. Then we use a different result of Thomae, (see \cite{aar} Corollary 3.3.6), which for the normalized $\hat{F}$ directly tells us $$\hat{F}({{r}, {1 + r - s}, {r}, {1/2 + r}, {1/2 + r}})=\hat{F}({{1/2}, {1/2}, {s-r}, {1 }, {s}}).$$ We also have the comparison \begin{equation}\label{fhat} \hat{F}(1/2,1/2,r,1,s)=\frac{2^{4r-1}N}{\Gamma(r)\Gamma(s-r)^2}F(r,s). \end{equation} Finally, we compute that $$\alpha_1=\frac{\pi \csc(\pi(r-1/2))}{\Gamma(r) \Gamma(s-1/2)},\s\alpha_2=\frac{-\csc(\pi(r-1/2))\pi}{\Gamma(1-r)\Gamma(s-r)}.$$ Putting all the pieces together, we get \begin{align*} \frac{2^{4r-1}}{\Gamma(r)\Gamma(s-r)^2}F(r,s)&=\frac{2^{4s-4}}{\Gamma(3/2-s)\Gamma(s-r)^2}\frac{\pi \csc(\pi(r-1/2))}{\Gamma(r) \Gamma(s-1/2)}F(3/2-s,3/2-r)\\&+\frac{2^{4(s-r)}}{\Gamma(s-r)\Gamma(r)^2}\frac{-\csc(\pi(r-1/2))\pi}{\Gamma(1-r)\Gamma(s-r)}F(s-r,s). \end{align*} Simplifying all the Gamma factors, we get to the identity. \end{proof} Notably, this identity becomes much nicer when $s=1-r$. \begin{corollary}\label{cor} If $b$ is the denominator of $r$ and $r<b/2$, we have $$F(r,1-r)=-4F(r+1/2,3/2-r)+2^{5-12r}\tan(\pi r)F(1-2r,1-r).$$ \end{corollary} With Corollary \ref{cor}, we are able to prove the key lemma of this section. \begin{Lemma}\label{l5} $F(1/12, 11/12) + 16 i F(11/12, 13/12) = \zeta_3^2 (-4 F(5/12, 7/12) - 4 i F(7/12, 17/12))$ \end{Lemma} \begin{proof} The twist switching $5$ and $7$ in this case is $\chi_{12.b}$ in the LMFDB, and we have $f_{288.3.g.a}\otimes\chi_{12.b}=f_{288.3.g.c}$. As before, if $L(f_{288.3.d.a},1)=\alpha L(f_{288.3.d.c},1)$, we can conclude $$F(1/12, 11/12) + 16 i F(11/12, 13/12) = \frac{1-\alpha}{1+\alpha} (-4 F(5/12, 7/12) - 4 i F(7/12, 17/12)).$$ We do know that $\sqrt{12}\alpha$ is in $\Q(i)$, which means $\alpha\in \Q(\sqrt{-3})=\Q(\zeta_3)$. Since applying the Atkin-Lehner involution to $F(1-2r,1-r)$ gives $F(r,1-r)$, we have already shown three term identities in our cases in Theorem \ref{threeterm}. This means for any specific $r$, this can be rewritten as a 4-term identity. In our case, $r=1/12$, from Theorem \ref{threeterm} we have $$F(5/12,7/12)/2=F(5/6,11/12)+F(5/6,17/12)$$ as well as its sister identity $$2F(11/12,13/12)=F(5/6,11/12)-F(5/6,17/12).$$ Adding these up we get that $$2F(5/6,11/12)=F(5/12,7/12)+4F(11/12,13/12).$$ Combined with Corollary \ref{cor}, this produces the 4-term identity, \begin{equation}\label{4term} F(1/12,11/12)=-4F(7/12,17/12)+8\tan(\pi/12)[F(5/12,7/12)+4F(11/12,13/12)]. \end{equation} We know from above that $$F(1/12, 11/12) + 16 i F(11/12, 13/12) = (a+b\sqrt{-3}) (-4 F(5/12, 7/12) - 4 i F(7/12, 17/12))$$ for some rational $a$ and $b$. Taking the real and imaginary part of both sides, we obtain that $$F(1/12,11/12)=4aF(5/12,7/12)-4b\sqrt{3}F(7/12,17/12)$$ and $$16F(11/12,13/12)=4aF(7/12,17/12)+4b\sqrt{3}F(5/12,7/12).$$ If we put these into the identity (\ref{4term}), we get an equation only involving $F(5/12,7/12)$ and $F(7/12,17/12)$, \begin{align*} &-4aF(5/12,7/12)+4b\sqrt{3}F(7/12,17/12)=-4F(7/12,17/12)+\\&4\tan(\pi/12)[F(5/12,7/12)-16\sqrt{3}aF(7/12,17/12)-16bF(5/12,7/12)]. \end{align*} This simplifies to $$4 (-2 + \sqrt{3} - a + (-3 + 2 \sqrt{3}) b)F(5/12,7/12)=4 (1 - (-2 + \sqrt{3}) a + \sqrt{3} b)F(7/12,17/12).$$ If both of these constants are nonzero, that implies $F(5/12,7/12)=CF(7/12,17/12)$ for some constant $C$ contained in $\Q(\sqrt{3})$. Assuming Conjecture \ref{con}, this cannot be true, so both of these constants must be zero. This gives us a system of two equations in two variables, which by linear algebra has a unique solution, specifically $a=b=-1/2$. Since $-1/2-\sqrt{-3}/2=\zeta_3^2$, we are done. \end{proof} \begin{corollary} $L(f_{288.3.d.c},1)=-\sqrt{-3}L(f_{288.3.d.a},1)$. \end{corollary} \begin{proof} This is obtained by solving the equation $(1-\alpha)/(1+\alpha)=\zeta_3^2$. \end{proof} \begin{Lemma}\label{l6} $$ F(1/12,2/3)-16F(11/12,4/3) = 1/4 ((-4 + 6 i) + (2 - 4 i) \sqrt{3})[-4 F( 5/12,4/3)+4iF(7/12,2/3) ]$$ \end{Lemma} \begin{proof} As before, apply Theorem \ref{kummer} to Lemma \ref{l1} to get $$\frac{\sqrt{2}}{-1+\sqrt{3}}[16F(11/12,4/3) + iF(1/12,2/3)] = \frac{\sqrt{2}\zeta_3^2}{1+\sqrt{3}} [-4F(7/12,2/3) -4i F( 5/12,4/3)].$$ Simplifying this gives the result. \end{proof} \begin{proof}[Proof of Theorem \ref{f9}] Using Lemma \ref{l5}, compute \begin{align*} & L(f_{288.3.d.a},1) =(-4\zeta_3-4)(F(5/12, 7/12) + i F(7/12, 17/12)) \end{align*} Then apply the Kummer transformation to get \begin{align*} &L(f_{288.3.d.a},1)=\frac{\sqrt{2}(-4\zeta_3-4)}{1+\sqrt{3}}(-F(5/12, 4/3) + i F(7/12, 2/3)). \end{align*} We now need to do the first half of this process in reverse using Lemma \ref{l6} to get the desired identity. \begin{align*} & L(f_{576.3.g.d},1)= ((-4 + 6 i) + (2 - 4 i) \sqrt{3})+4)[-F( 5/12,4/3)+iF(7/12,2/3) ] \end{align*} Taking the quotient of these and simplifying gives the identity. \end{proof} \subsection{Transcendence}\label{trans} For classical ${}_2F_1(z)$ functions, the transcendence of special values is fairly well understood. When the hypergeometric series can be written as an algebraic function of $z$, the transcendence is straightforward: ${}_2F_1(z)$ is algebraic if $z$ is algebraic and transcendental if $z$ is not. This is equivalent to the local system having finite monodromy, and these cases are classified in Beukers and Heckman \cite{beukersheckman}, including for generalized hypergeometric series. The question of transcendence is harder for hypergeometric systems with infinite monodromy. In this case, it is well known that the monodromy group is isomorphic to a triangle group (see, e.g., \cite{yoshida}). The set of algebraic numbers $z$ for a fixed datum so that ${}_2F_1(z)$ is algebraic is called the \textit{exceptional set}, denoted $\mathcal{E}$. Then Wolfart \cite{wolfart} and Cohen-W\"ustholz \cite{Cohen-Wustholz} showed that when the monodromy group is an arithmetic triangle group, as classified by Takuechi \cite{takeuchi}, then the exceptional set is infinite; otherwise, it is finite. Wolfart's proof uses the work of W\"ustholz discussed in the introduction, and work of Cohen and W\"ustholz relies on a special case of the Andre-Oort conjecture, which was proven by Edixhoven and Yafaev \cite{edixhoven-yafaev}. This construction associates each point element of the exceptional set to a particular abelian variety, which necessarily has CM type. However, the process is still rather theoretical, and so explicit elements of the exceptional set are nontrivial. For example, Yang in \cite{yanghecke} proved an algebraic evaluation of a ${}_{2}F_1$ at a CM point using the action of the Hecke operators. Generally speaking, the transcendence of the $F(r,s)$ and more generally of periods on the same (non-abelian) variety of complex dimension higher than 1 is poorly understood. We notice in the discussion above that the proof relied on two very deep theorems: W\"ustholz's theorem and a special case of the Andre-Oort conjecture. Moving up in dimension, we expect the geometry to become even more difficult, and so proving the transcendence of $F(r,s)$ seems out of reach with our current knowledge. There are some results in the function field setting, e.g., \cite{changpapanikolas}, where the cohomology is nicer. However, transcendence results over $\Bar{\Q}$ are generally very hard and require deep geometric theorems to prove. For example, $\Gamma(1/4)$ and $\Gamma(1/3)$ are known to be transcendental by Chudnovsky \cite{chudnovsky} using CM theory, but $\Gamma(1/5)$ is still unknown. A much stronger conjecture attributed to Rohrlich in \cite{waldschmidt} states that the only additive relations over $\Bar{\Q}(\pi)$ between $$\log\Gamma(j/5), \s j=1,2,3,4$$ arise from the well-known multiplicative formulas for the gamma function, specifically those stated in (\ref{gamma}) earlier. In the same spirit, we conjecture the following. \begin{Conjecture} Let $f_1,f_2,f_3,f_4$ be any of the four conjugate hypergeometric series used in the six families from classes 7, 8, and 9 above corresponding to a modular form with multiplicative coefficients. Then the only linear relations over $\Bar{\Q}$ between these 4 can be derived from those given in Lemmas \ref{l1}, \ref{l2}, \ref{l3}, \ref{l4}, \ref{l5}, and \ref{l6} respectively. Namely, the dimension of the vector space over $\Bar{\Q}$ generated by $f_1,f_2,f_3,f_4$ is 2. \end{Conjecture} For example, if $$f_1=F(1/12,11/12), f_2=F(5/12,7/12), f_3=F(7/12,17/12), f_4=F(11/12,13/12),$$ then taking the real and imaginary parts of Lemma \ref{l2} implies $f_1$ and $f_4$ can be written as a $\Bar{\Q}$-linear combination of $f_2$ and $f_3$. Note that this implies Conjecture \ref{con} above is true, though the current version is slightly stronger. However, as noted above, even proving that $f_2$ and $f_3$ are transcendental is likely to be very difficult, much less their linear independence. The best we can do is Theorem \ref{main}, which applies to the families in classes 6, 7, 8, and 9. We also have a slightly stronger statement for families 1, 5, and 9. \begin{theorem} Let $\Lambda$ denote the set of all hypergeometric $L$-values contained in any family of classes 6, 7, or 9. Then the dimension of the vector space generated by $\Lambda$ over $\Bar{\Q}$ is less than or equal to 2. \end{theorem} \begin{proof} In Table \ref{tab}, the left-hand two columns are related by the Kummer transformation, and in these three cases we have already established that $C(r,s)$ is algebraic at the very beginning of this section. Then, Theorem \ref{threeterm} allows us to express the first column in terms of the third column and vice versa. Then applying Theorem \ref{main} gives the result. \end{proof} For class 8, likely the same result holds, but the equivalent of Theorem \ref{threeterm}, Proposition \ref{fivterm}, is not nearly as nice, so some extra work would be required. To illustrate, in class 9, $\Lambda$ consists of all the $f_i$ listed above: $F(1/12,2/3)$, $F(5/12,4/3)$, $F(7/12,2/3)$, $F(11/12,4/3)$, and $F(5/6,11/12)$, $F(5/6,17/12)$, $F(1/6,7/12)$, $F(1/6,13/12)$. The first 4 come from the Kummer transformation, and the next 4 from the Atkin-Lehner involution. \section{Further Directions} We covered all the non-CM classes except 10 and 11, which both have degree 8 Hecke eigenvalue fields, so are harder to work with; however, we expect the transcendence results to be comparable. Another direction would be to study transcendence results in the family $(1-r,r)$ for any rational $r$. It is rare even for non-congruence cases to have $C(r,s)$ algebraic, which suggests there may be some extra structure under these families. We also conjecturally have Theorem \ref{threeterm} still as well as Corollary \ref{cor}, which strongly suggest the transcendence degree could be reduced. However, the underlying non-congruence modular forms lack the nice properties we used, so completely different proofs would be required in these cases. There is also a chance that some of these series are related to a Hilbert modular form or another related automorphic object as well. Another subject we did not address in depth is differentials of the second kind. Theorem \ref{wust} by W\"ustholz \cite{wustholz} mentioned in the introduction gives the transcendence degree over $\Bar{\Q}$ of 1, $\pi$, and the differentials of the first and second kind. A natural question is, what are the differentials of the second kind, and what is their transcendence degree? In Theorem \ref{cmwus}, we computed the differentials of the second kind for one CM Galois family and confirmed in this case the transcendence degree is consistent with W\"ustholz. However, our method of finding differentials of the second kind does not work for any other families. A further direction would be to compute the differentials of the second kind for the other CM examples and see if we still get a transcendence degree of 4. One could also look at the non-CM examples, where we anticipate the transcendence degree to be 6, but this would be much harder since we lack evaluation formulas in these cases. Looking further afield, there are other hypergeometric data with which our methods could apply. Namely, ${}_2F_1(z)$ hypergeometric functions are often modular forms on their underlying monodromy group, and then we can use the integral formula to obtain special $L$-values in the same manner as above. The data $HD_3:=\{\{1/3,2/3\},\{1,1\}\}$ is the most approachable of these, since ${}_2F_1(HD_3,z)$ is known to be a weight 1 modular form on $\Gamma_0(3)$, and has a classical expression as an eta quotient (see Borwein and Borwein \cite{piagm}). The corresponding $\mathbb{K}_3$ function is mentioned in \cite{aglt}, and its further properties will be studied in a forthcoming paper of Grove \cite{grove}. More generally, Yang \cite{yang} has shown that modular forms on arithmetic triangle groups can be written as a combination of ${}_2F_1(z)$ functions. However, since in general there are no cusps and thus no $q$-expansions or eta products, these cases likely would be much harder. \section{Appendix: the table of Hecke Eigenforms built from $\mathbb{K}_2$} \label{append} In the appendix, we list all the completed modular forms we obtain using this method. Note 6 of these are already given in \cite{aglt}, but we repeat them here for completeness. In cases where we are unable to express a full Hecke eigenform as a linear combination of the conjugates, we simply list the pairs. Observe also that changing the $\mathbb{K}_2(r,s)$ in this table to $F(r,s)$ will also give $N$ times the $L$-value of the eigenform, and so the table also gives us the special $L$-values obtained in the Galois families, though in some cases these can be simplified. We omit classes 1, 2 and 5 because all the conjugates are already listed in Table \ref{tab}. \fontsize{9}{14}\selectfont \begin{table}[ht] \centering \begin{tabular}{c|c} Label & Eigenform \\\hline 3.a & $f_{36.3.d.a}=\mathbb{K}_2(1/3,7/6)$+2 $\mathbb{K}_2(2/3,5/6)$ \\\hline AL-3 & $f_{36.3.d.a}=\mathbb{K}_2(1/6,5/6)+8\mathbb{K}_2(5/6,7/6)$ \\\hline 3.b & $f_{36.3.d.a}=\mathbb{K}_2(1/3,2/3)+2\mathbb{K}_2(2/3,4/3)$\\\hline 4.a & $f_{32.3.d.a}=\mathbb{K}_2(1/8,9/8)+2\mathbb{K}_2(2/8,11/8)$\\\hline AL-4 & $f_{32.3.d.a}=\mathbb{K}_2(1,9/8)/2+\mathbb{K}_2(1,11/8)/2$\\\hline 4.b & $f_{256.3.c.e}=\mathbb{K}_2(1/8,1/2)+2\mathbb{K}_2(3/8,1/2)$\\\hline 6.a & $f_{32.3.c.a}=\mathbb{K}_2(1/4, 3/4) + 4 i\mathbb{K}_2(3/4, 5/4)$ \\\hline AL-6 & $f_{32.3.c.a}=\mathbb{K}_2(1/2,3/4)/2+\mathbb{K}_2(1/2,5/4)/2$+$2i(\mathbb{K}_2(1/2,3/4)-\mathbb{K}_2(1/2,5/4))$\\\hline 6.b & $f_{64.3.c.b}=\mathbb{K}_2(1/4, 1) + 4 i\mathbb{K}_2(3/4, 1)$ \\\hline 7.a & $f_{64.3.d.a}=\mathbb{K}_2(1/8, 5/8) - 2 \sqrt{3} \mathbb{K}_2(3/8, 7/8) - 4 \sqrt{-3} \mathbb{K}_2(5/8, 9/8) - 8 i \mathbb{K}_2(7/8, 11/8)$ \\\hline AL-7 & $f_{ 64.3.d.a }=1/8 ((-i + 1 - \sqrt{3} - i \sqrt{3}) \mathbb{K}_2(1/2, 5/8) + (3 i + 3 + \sqrt{-3} - \sqrt{3}) \mathbb{K}_2(1/2, 7/8)$\\&$ + (-3 i + 3 + \sqrt{-3} + \sqrt{3}) \mathbb{K}_2(1/2, 9/8) + (i + 1 - \sqrt{-3} + \sqrt{3}) \mathbb{K}_2(1/2, 11/8))$ \\\hline 7.b & $f_{256.3.c.g}=\mathbb{K}_2(1/8, 1) - 2 \sqrt{-3} \mathbb{K}_2(3/8, 1])- 4 \sqrt{3} \mathbb{K}_2(5/8, 1) + 8 i \mathbb{K}_2(7/8, 1)$ \\\hline 8.a & $f_{128.3.d.c}=\mathbb{K}_2(1/8, 7/8) + 2\sqrt{2} \mathbb{K}_2(3/8, 5/8) + 4i \mathbb{K}_2(5/8, 11/8) + 8\sqrt{-2} \mathbb{K}_2(7/8, 9/8)$ \\\hline AL-8 & $f_{128.3.d.c}=\mathbb{K}_2(1/4, 5/8)/2+\mathbb{K}_2(1/4, 9/8)/2 + \sqrt{2} (\mathbb{K}_2(3/4, 7/8)+\mathbb{K}_2(3/4, 11/8)) $\\&$+ 2i \mathbb{K}_2(1/4, 5/8)-\mathbb{K}_2(1/4, 9/8) + 4\sqrt{-2} (\mathbb{K}_2(3/4, 7/8)-\mathbb{K}_2(3/4, 11/8))$ \\\hline 8.b & $f_{256.3.c.c}=\mathbb{K}_2(1/8, 3/4) + 2\sqrt{-2} \mathbb{K}_2(3/8, 5/4) + 4 \mathbb{K}_2(5/8, 3/4) - 8\sqrt{-2} \mathbb{K}_2(7/8, 5/4)$\\\hline 9.a & $f_{288.3.g.a}=\mathbb{K}_2(1/12, 11/12) - 4 \mathbb{K}_2(5/12, 7/12) - 4 i \mathbb{K}_2(7/12, 17/12) + 16 i \mathbb{K}_2(11/12, 13/12)$ \\\hline AL-9& $f_{288.3.g.a}=1/2(\mathbb{K}_2(1/6, 13/12) + \mathbb{K}_2(1/6, 7/12)) + i/2 (\mathbb{K}_2(1/6, 13/12) - \mathbb{K}_2(1/6, 7/12)) $\\&$- 2 (\mathbb{K}_2(5/6, 11/12) + \mathbb{K}_2(5/6, 17/12)) + 2i (\mathbb{K}_2(5/6, 11/12) - \mathbb{K}_2(5/6, 17/12)) $\\\hline 9.b & $f_{576.g.d}=\mathbb{K}_2(1/12, 11/12) + 4 \mathbb{K}_2(5/12, 7/12) + 4 i \mathbb{K}_2(7/12, 17/12) + 16 i \mathbb{K}_2(11/12, 13/12)$\\\hline 10.a & $f_{1152.3.b.i}=-16 \sqrt{-2} \mathbb{K}_2(23/24, 25/24) + 8 \sqrt{14} \mathbb{K}_2(19/24, 29/24) + 8 \sqrt{7} \mathbb{K}_2(17/24, 31/24) - 4 i \mathbb{K}_2(13/24, 35/24) $\\& $ + 4 \sqrt{2} \mathbb{K}_2(11/24, 13/24) + 2 \sqrt{-14} \mathbb{K}_2(7/24, 17/24) - 2 \sqrt{-7} \mathbb{K}_2(5/24, 19/24) + \mathbb{K}_2(1/24, 23/24)$\\\hline AL-10 & $f_{1152.3.b.i}=1/2(\mathbb{K}_2(1/12, 13/24) + \mathbb{K}_2(1/12, 25/24)) - i/2 (\mathbb{K}_2(1/12, 13/24) - \mathbb{K}_2(1/12, 25/24)) $\\&$ - 2\sqrt{2} (\mathbb{K}_2(11/12, 23/24) + \mathbb{K}_2(11/12, 35/24)) - 2 \sqrt{-2} (\mathbb{K}_2(11/12, 23/24) - \mathbb{K}_2(11/12, 35/24)) $\\&$- \sqrt{-14} (\mathbb{K}_2(7/12, 31/24) + \mathbb{K}_2(7/12, 19/24)) + \sqrt{14} (\mathbb{K}_2(7/12, 31/24) - \mathbb{K}_2(7/12, 19/24)) $\\&$ - \sqrt{-7} (\mathbb{K}_2(5/12, 29/24) + \mathbb{K}_2(5/12, 17/24)) + \sqrt{7}(\mathbb{K}_2(5/12, 29/24) - \mathbb{K}_2(5/12, 17/24)) $\\\hline 10.b & $f_{2304.g.p}=16 \sqrt{-2} \mathbb{K}_2(23/24, 17/12) + 8 \sqrt{-14} \mathbb{K}_2(19/24, 13/12) + 8 \sqrt{7} \mathbb{K}_2(17/24, 11/12) - 4 i \mathbb{K}_2(13/24, 7/12) $\\& $ + 4 \sqrt{-2} \mathbb{K}_2(11/24, 17/12) + 2 \sqrt{-14} \mathbb{K}_2(7/24, 13/12) - 2 \sqrt{7} \mathbb{K}_2(5/24, 11/12) + \mathbb{K}_2(1/24, 7/12)$\\\hline 11.a & $f_{288.3.b.c}=\mathbb{K}_2(1/24, 17/24) - 2 \sqrt{6} \mathbb{K}_2(5/24, 13/24) - 2 \sqrt{-15} \mathbb{K}_2(7/24, 23/24) - 4 \sqrt{10}\mathbb{K}_2(11/24, 19/24) $\\&$ + 4 \sqrt{-15} \mathbb{K}_2(13/24, 29/24) - 8 \sqrt{10} \mathbb{K}_2(17/24, 25/24) - 8 \mathbb{K}_2(19/24, 35/24) - 16 \sqrt{-6} \mathbb{K}_2(23/24, 31/24)$\\\hline AL-11 & $f_{288.3.b.c}=\mathbb{K}_2(1/3, 19/24), \mathbb{K}_2(2/3, 23/24),\mathbb{K}_2(2/3, 29/24),\mathbb{K}_2(1/3, 25/24), $\\&$\mathbb{K}_2(1/3, 31/24),\mathbb{K}_2(2/3, 35/24),\mathbb{K}_2(1/3, 13/24),\mathbb{K}_2(2/3, 17/24)$\\\hline 11.b & $f_{2304.3.g.y}=\mathbb{K}_2(1/24, 5/6) - 2 \sqrt{6} \mathbb{K}_2(5/24, 7/6) + 2 \sqrt{-15} \mathbb{K}_2(7/24, 5/6) + 4 \sqrt{-10} \mathbb{K}_2(11/24, 7/6) $\\&$ + 4 \sqrt{15} \mathbb{K}_2(13/24, 5/6) + 8 \sqrt{10} \mathbb{K}_2(17/24, 7/6) +8 i \mathbb{K}_2(19/24, 5/6) -16 \sqrt{-6} \mathbb{K}_2(23/24, 7/6)$\\\hline \end{tabular} \caption{Hecke Eigenforms constructed from Galois families} \label{tab:my_label} \end{table} \fontsize{12}{14}\selectfont Note for 3.a, we actually get an oldform of level 72, so the coefficients that are divisible by 2 are not consistent with the LMFDB, but after applying a Hecke operator $T_p$ with $p$ coprime to the level, they become the same. Also note that the formula for AL-3 only gives $\chi_{-4}f_{36.3.d.a}$, so some of the coefficients divisible by 2 are missing. Because this differs by an even twist, the $L$-value obtained from AL-3 and 3.a or 3.b differ by an algebraic number by Shimura's theorem, which actually is an integer. Finally, for AL-11, there may be some way to combine these conjugates to get $f_{288.3.b.c}$, parallel to the difficult construction for AL-7, which is related to Proposition \ref{fivterm}. However, this is tedious, and we leave it to the interested reader to find such a relation.\newpage \begin{thebibliography}{99} \bibitem{aglt} Allen, Michael, Brian Grove, Ling Long, and Fang-Ting Tu. \textit{The Hypergeometric Modularity Method I and II.} arXiv:2404.00711 and arXiv:2411.15116, 2024. \bibitem{ahlgrenono} Ahlgren, Scott, Ken Ono, and David Penniston. \textit{A Gaussian hypergeometric series evaluation and Apery number congruences.} J. Reine Angew. Math., \textbf{518} (2000) p.187–212. \bibitem{aar} Andrews, Geoorge, Richard Askey, and Ranjan Roy. \textit{Special Functions}. Cambridge University Press, 1999. \bibitem{archinard} Archinard, Natalia. \textit{Hypergeometric abelian Varieties}. Canad. J. Math. Vol. 55 (5) (2003), p. 897–932. \bibitem{arima} Arima, Satoshi. \textit{Differential forms of the first and second type on algebraic varieties}. Math. Soc. Japan \textbf{16}, no. 2 (1964), p. 102-108. \bibitem{otsubo} Asakura, Masanori and Noriyuki Otsubo. A Functional Logarithmic Forumula for the Hypergeometric Function $_{3}F_{2}$. Nagoya Mathematical Journal, \textbf{236} (2019). p. 29-46. \bibitem{otsubo2} Asakura, Masanori, Noriyuki Otsubo, and Tomohide Terasoma. \textit{An Algebro-Geometric Study of Special Values of Hypergeometric Functions $_{3}F_{2}$}. Nagoya Mathematical Journal. \textbf{236}, (2019). p. 47-62. \bibitem{atkinliulong} Atkin, A.O.L., Wen-Ching Winnie Li, Tong Liu, and Ling Long. \textit{Galois Representations with Quaternion Multiplication associated to Noncongruence Modular Forms}. Transactions of the AMS, \textbf{365} no. 12 (2013), p. 6217–6242. \bibitem{bailey} Bailey, Wilfrid. \textit{Generalized Hypergeometric Series}. Cambridge University Press: Cambridge, 1935. \bibitem{baker} Baker, Alan, \textit{Linear forms in the logarithms of algebraic numbers. III}. Mathematika, \textbf{14} no. 2 (1967). p. 220–228. \bibitem{beukers} Beukers, Fritz. \textit{Irrationality proofs using Modular Forms}. Astérisque, \textbf{147-148} (1987), p. 271-283. \bibitem{bcm} Beukers, Fritz, Henri Cohen, and Anton Mellit. \textit{Finite hypergeometric functions}. Pure Appl. Math. Q., \textbf{11}, no. 4 (2015). p. 559–589. \bibitem{beukersheckman} Beukers, Frits, and Gert Heckman. \textit{Monodromy for hypergeometric ${}_nF_{n-1}$ functions}. Invent. math. \textbf{95} (1989), p. 325-354. \bibitem{coxeter} Beyer, W.A., J. D. Louck, P. R. Stein. \textit{Group theoretical basis of some identities for the generalized hypergeometric series}. \textit{Journal of Mathematical Physics}, \textbf{28} (1987), p. 497-508. \bibitem{piagm} Borwein, Jonathan, and Peter Borwein. \textit{Pi and the AGM}. John Wiley and Sons: New York, 1987. \bibitem{chenchu} Chen, Marta, and Wenchang Chu. \textit{Yabu’s formulae for hypergeometric ${}_3F_2$-series through Whipple’s quadratic transformations} AIMS Mathematics, \textbf{9}, no. 8 (2024), p. 21799–21815. \bibitem{changpapanikolas} Chang, Chieh-Yu, and Matthew Papanikolas. \textit{Algebraic Relations Among Periods and Logarithms of Ranks 2 Drinfeld Modules}. American Journal of Mathematics, \textbf{133}, no. 2 (2011), p. 359-391. \bibitem{chudnovsky} Chudnovsky, \textit{Algebraic independence of constants connected with the exponential and the elliptic functions}, Dokl. Akad. Nauk Ukrain. SSR Ser. A, \textbf{8} (1976), p. 698–701. \bibitem{cohen} Cohen, Paula. Appendix to \textit{Triangle Fucshian Differential Equations with Apparent Singularities} by Hironori Shiga, Toru Tsutsui, and Jurgen Wolfart. Osaka J. Math. \textbf{41} (2004), 625–658. \bibitem{Cohen-Wustholz} Paula~B. Cohen and Gisbert W\"ustholz. \newblock Application of the {A}ndr\'e-{O}ort conjecture to some questions in transcendence. \newblock In {\em A panorama of number theory or the view from {B}aker's garden ({Z}\"urich, 1999)}, p. 89-106. Cambridge Univ. Press, Cambridge, 2002. \bibitem{glc} Deines, Alyson, Jenny Fuselier, Ling Long, Holly Swisher, and Fang-Ting Tu. \textit{Generalized Legendre Curves and quaternionic multiplication}. Journal of Number Theory \textbf{161} (2016), p. 175–203. \bibitem{deinesetal2} Deines, Alyson, Jenny Fuselier, Ling Long, Holly Swisher, and Fang-Ting Tu. \textit{Hypergeometric Series, Truncated Hypergeometric Series, and Gaussian Hypergeometric Functions}. In: Eischen, E., Long, L., Pries, R., Stange, K. (eds) \textit{Directions in Number Theory}. Association for Women in Mathematics Series, vol 3 (2016). \bibitem{asairec} Dembele, Lassina, Alexei Panchiskin, John Voight, and Wadim Zudilin. \textit{Special Hypergeometric Motives and their $L$-Functions: Asai Recognition}. Experimental Mathematics, \textbf{31} no. 4, (2020), p. 1278–1290. \bibitem{ebisu} Ebisu, A and K. Iwasaki. \textit{Three-term relations for ${}_3F_2(1)$}. Journal of mathematical analysis and applications, \textbf{463} no. 2 (2018), p. 593-610 \bibitem{edixhoven-yafaev} Edixhoven, Bas and Andrei Yafaev. \newblock Subvarieties of {S}himura varieties. \newblock {\em Ann. of Math. (2)}, \textbf{157}, no.2 (2003),p. 621-645. \bibitem{formichella} Formichella, Marc, R.M. Stadem and Eric Stade. \textit{Coxeter group actions on ${}_4F_3(1)$ hypergeometric series}. Ramanujan J, \textbf{24} (2008), p. 93-128. \bibitem{green} Green, Richard, Ilia D. Mishev, Eric Stade. \textit{Coxeter group actions and limits of hypergeometric series}. Ramanujan J \textbf{53} (2020), p. 607–651. \bibitem{greene} Greene, John. \textit{Hypergeometric functions over finite fields}. Trans. Amer. Math. Soc., \textbf{301}no.1 (1987) p. 77–101. \bibitem{grove} Grove, Brian. \textit{On the Hypergeometric Modularity Conjectures of Dawsey and McCarthy}, in preparation. \bibitem{katz} Katz, Nicholas. \textit{Exponential Sums and Differential Equations}. Princeton University Press: Princeton, 1990. \bibitem{kontsevichzagier} Kontsevich, Maxim, and Don Zagier. \textit{Periods}. IHES Proceedings (2001). \bibitem{lilongliu} Li, Wen-Ching Winnie, Tong Liu, and Ling Long. \textit{Potentially $GL_2$-Type Galois Representations associated to Noncongruence Modular Forms}. Transactions of the AMS, \textbf{371} no. 8 (2019), p. 5341–5377. \bibitem{lilongliuCM} Li, Wen-Ching Winnie, Tong Liu, and Ling Long. \textit{Computing Special L-Values of Certain Modular Forms with Complex Multiplication}. SIGMA \textbf{14} (2018), p. 5341–5377. \bibitem{whipple} Li, Wen-Ching Winnie, Ling Long, and Fang-Ting Tu. \textit{A Whipple ${}_7F_6$ formula revisited}. La Matematica, \textbf{1} (2022), p. 480–530. \bibitem{nesterenko} Nesterenko, Yuri. \textit{Modular functions and transcendence.} Math. Sb. \textbf{187} no. 9, (1996) p. 65–96. \bibitem{osburnstraub} Osburn, Robert, and Armin Straub. \textit{Interpolated sequences and critical L-values of modular forms}. In: Elliptic Integrals, Elliptic Functions and Modular Forms in Quantum Field Theory, eds. Blümlein, J., Schneider, C., Paule, P. Texts and Monographs in Symbolic Computation. Springer, 2019. \bibitem{otsubo3} Otsubo, Noriyuki. \textit{On the Abel–Jacobi maps of Fermat Jacobians}. Math. Z. \textbf{270} (2012). p. 423–444. \bibitem{rhinviola} Rhin, Georges, and Carlos Viola. \textit{On a permutation group related to $\zeta(2)$}. Acta Arithmetica \textbf{LXXVII} no. 1 (1996). p. 23-56. \bibitem{robertsvillegas} Roberts, David, and Fernando Rodriguez-Villegas. \textit{Hypergeometric Motives}. Notices of the AMS \textbf{69} no. 6 (2022), p. 914-929. \bibitem{rosen} Rosen, Esme. \textit{Modular Forms and Certain ${}_2F_1(1)$ Hypergeometric Series}. In preparation. \bibitem{shimuragross} Shimura, Goro. \textit{On the Zeta-Function of an abelian Variety with Complex Multiplication}. Annals of Mathematics, \textbf{94}, no. 3 (1971), p. 504-533. \bibitem{shimurazeta} Shimura, Goro. \textit{The special values of the zeta functions associated with cusp forms}. Communications on Pure and Applied Math, vol. XXIX (1976), p. 783-804. \bibitem{stein} Stein, William. \textit{Modular Forms: A Computational Approach}. American Mathematical Society: Providence Rhode Island, 2007. \bibitem{steinshakarchi} Stein, Elias and Rami Shakarchi. \textit{Complex Analysis}. Princeton Lectures in Analysis No. 2, Princeton University Press: Princeton, 2003. \bibitem{stiller} Stiller, Peter. \textit{Classical Automorphic Forms and Hypergeometric Functions}. Journal of Number Theory \textbf{28} (1988), p. 219-232. \bibitem{tanimaya} Tanimaya, Yukata. \textit{$L$-functions of number fields and zeta functions of abelian varieties.} Journal of the Mathematical Society of Japan \textbf{9}, no. 3 (1957), p. 330-336. \bibitem{takeuchi} Takeuchi, Kisso \textit{Commensurability Classes of Arithmetic Triangle Groups}. Lecture Notes, 2 August 2006. \bibitem{thomae} Thomae, Carl Johannes \textit{Ueber die functionen welche durch Reihen ... ,} J. Reine Angew. Math. \textbf{87}, vol. 26 (1879). \bibitem{waldschmidt} Waldschmidt, Michel. \textit{Transcendence of Periods: The State of the Art}. Pure and Applied Mathematics Quarterly \textbf{2}, No. 2 (2006), p. 435-463. \bibitem{wolfart} Wolfart, Jurgen. \textit{Fonctions hypergeometriques, arguments exceptionels et groupes de monodromie}. Publ. Math. Univ. P. et M .Curie, Problemes Diophantiens \textbf{79}(1985–86), p. 1–24. \bibitem{wustholz} W\"ustholz, Gisbert. \textit{Algebraic Groups, Hodge Theory, and Transcendence}, Proc. ICM Berkley \textbf{1} (1986), p. 476–483. \bibitem{yang} Yang, Yifan. \textit{On Differential Equations Satsified by Modular Forms}. Math. Z. \textbf{246} (2004). p. 1–19. \bibitem{yanghecke} Yang, Yifan. \textit{Schwarzian Differential Equations and Hecke Eigenforms on Shimura Curves}. Compositio Math. \textbf{149} (2013). p. 1–31. \bibitem{yoshida} Yoshida, Masaaki \textit{Hypergeometric Functions, my love}. Aspects of Mathematics, 1997. \bibitem{zagier} Zagier, Don. \textit{Elliptic Modular Forms and Their Applications}. In 123s of Modular Forms, eds. Ranested, K., D. Harder and D. Zagier. Springer: Berlin, 2008. \bibitem{zagierL} Zagier, Don. \say{The arithmetic and topology of differential equations}. In \textit{European Congress of Mathematics: Berlin, July 18–22, 2016} p. 717-776, Volker Mehrmann and Martin Skutella, eds. EMS Press, Helsinki, 2016. \bibitem{zudlinlval} Zudilin, Wadim. \textit{Period(d)ness of L-Values}. Springer Proceedings in Mathematics and Statistics. \textbf{43} (2013). \bibitem{zudilin2} Zudilin, Wadim. \textit{Two Hypergeometric Tales and a New Irrationality Measure of $\zeta(2)$}. Ann. Math. Québec \textbf{38} (2014), p. 101–117. \bibitem{zudilin3} Zudilin, Wadim. \textit{A Hypergeometric Version of the Modularity of Rigid Calabi–Yau Manifolds}. SIGMA \textbf{14} (2018). \bibitem{zudilin5} Zudilin, Wadim. \textit{Arithmetic of linear forms involving odd zeta values}. Journal de théorie des nombres de Bordeaux, \textbf{16} no. 1,(2004). p. 251-291. \end{thebibliography} \end{document}
2412.07092v2
http://arxiv.org/abs/2412.07092v2
Linear and Sublinear Diversities
\documentclass[11pt]{article} \usepackage{amsmath,amssymb,amsthm,color} \usepackage{mathtools} \usepackage[shortlabels]{enumitem} \usepackage{graphicx} \usepackage{xcolor} \newtheorem{theorem}{Theorem} \newtheorem{proposition}[theorem]{Proposition} \newtheorem{lemma}[theorem]{Lemma} \newtheorem{corollary}[theorem]{Corollary} \newcommand{\Sp}{\mathrm{span}} \newcommand{\sH}{\mathcal{H}} \newcommand{\bbS}{\mathbb{S}} \newcommand{\dc}{\delta_{C}} \newcommand{\sK}{\mathcal{K}} \newcommand{\sU}{\mathcal{U}} \newcommand{\conv}{\mathrm{conv}} \newcommand{\Pf}{\mathcal{P}_f} \renewcommand{\Re}{\mathbb{R}} \newcommand{\bx}{\mathbf{x}} \newcommand{\bone}{\mathbf{1}} \newcommand{\bzero}{\mathbf{0}} \newcommand{\sB}{\mathcal{B}} \newcommand{\td}{\widetilde{\delta}} \renewcommand{\vert}{\mathrm{Vert}} \newcommand{\D}{\mathbf{D}} \newcommand{\deltaneg}{\delta_{\mbox{\footnotesize{neg}}}} \newcommand{\supp}{\mathrm{supp}} \newcommand{\mat}[1]{\left[ \begin{matrix} #1 \end{matrix} \right]} \newcommand{\tdelta}{\widetilde{\delta}} \newcommand{\nul}{\mathrm{null}} \DeclarePairedDelimiterX{\setnorm}[1] {\lvert\mkern-2mu\lvert\mkern-2mu\lvert}{\rvert\mkern-2mu\rvert\mkern-2mu\rvert}{#1} \linespread{1.5} \usepackage{fullpage} \newenvironment{bproof}{\begin{proof} \color{blue} }{\end{proof} } \title{Linear and Sublinear Diversities} \author{David Bryant and Paul Tupper} \date{\today} \begin{document} \maketitle \begin{abstract} Diversities are an extension of the concept of a metric space, where a non-negative value is assigned to every finite set of points, rather than just pairs. A general theory of diversities has been developed which exhibits many deep analogies to metric space theory but also veers off in new directions. Just as many of the most important aspects of metric space theory involve metrics defined on $\Re^k$, many applications of diversity theory require a specialized theory for diversities defined on $\Re^k$, as we develop here. We focus on two fundamental classes of diversities defined on $\Re^k$: those that are Minkowski linear and those that are Minkowski sublinear. Many well-known functions in convex analysis belong to these classes, including diameter, circumradius and mean width. We derive surprising characterizations of these classes, and establish elegant connections between them. Motivated by classical results in metric geometry, and connections with combinatorial optimization, we then examine embeddability of finite diversities into $\Re^k$. We prove that a finite diversity can be embedded into a linear diversity exactly when it has negative type and that it can be embedded into a sublinear diversity exactly when it corresponds to a generalized circumradius. \end{abstract} \section{Introduction} A diversity \cite{BryantTupper12} is a pair $(X,\delta)$ where $X$ is a set and $\delta$ is a non-negative function defined on finite subsets of $X$ satisfying \begin{quotation} \noindent (D1) $\delta(A)\geq 0$ and $\delta(A)=0$ if and only if $|A|\leq 1$,\\ (D2) $\delta(A \cup C) \leq \delta(A \cup B) + \delta(B \cup C)$ whenever $B \neq \emptyset$. \end{quotation} \noindent As such, diversities are set-based analogues of metric spaces, and in fact the restriction of a diversity to pairs is a metric space \cite{BryantTupper12}. Properties (D1) and (D2) are equivalent to (D1) together with monotonicity \begin{quotation}\noindent (D3) $\delta(A) \leq \delta(B)$ whenever $A \subseteq B$ \end{quotation} and subadditivity on intersecting sets \begin{quotation} \noindent (D4) $\delta(A \cup B) \leq \delta(A)+ \delta(B)$ when $A \cap B \neq \emptyset$. \end{quotation} \noindent We say $(X,\delta)$ is a {\em semidiversity} if (D1) is relaxed to \begin{quotation}\noindent (D1$'$) $\delta(A)\geq 0$ and $\delta(A)=0$ if $|A|\leq 1$.\end{quotation} That is, sets with two or more points may have zero diversity. This terminology is analogous to at least some of the definitions of semimetrics. Many well-known set functions are diversities: the diameter of a set; the length of a connecting Steiner tree; the circumradius; the length of a minimal traveling salesperson tour; the mean width; the size of a smallest enclosing zonotope. Two set functions which fail to be diversities are {\em genetic diversity} \[\pi(A) = \mbox{$\binom{|A|}{2}^{-1}$} \sum_{a,b \in A} d(a,b),\] which is not monotonic (D3), and volume of convex hull, which fails (D2) and (D4). There are broad classes of diversities just like there are broad classes of metrics. The $\ell_1$ metrics have the form \[d_1(a,b) = \sum_i |a_i - b_i|,\] while {\em $\ell_1$ diversities} \cite{BryantTupper14} have the form \[\delta_1(A) = \sum_i \max_{a,b \in A} |a_i - b_i|.\] Negative-type metrics satisfy \[\sum_{a,b} x_a x_b \,d(a,b) \leq 0\] for all zero-sum vectors $x$ while {\em negative type diversities} \cite{WuBryantEtal19} satisfy \[ \sum_{A,B} x_A x_B\, \delta(A \cup B) \leq 0 \] for all zero-sum vectors $x$ with $x_\emptyset=0$. The theory of diversities sometimes runs in parallel with that of metric spaces and other times veers off in new directions. In the first paper on diversities \cite{BryantTupper12} we explored how concepts of hyperconvexity, injectivity and the tight span extended, and to an extent enriched, the analogous metric concepts. In \cite{BryantTupper14} we showed that the `geometry of graphs' \cite{LinialLondonEtal95} linking metric embeddings to approximation algorithms on graphs has a parallel `geometry of hypergraphs' linking diversity embeddings to approximation algorithms on hypergraphs. Jozefiak and Shephard \cite{jozefiak2023diversity} use this approach to obtain the best known approximation algorithms for several hypergraph optimization problems. Diversities turn out to be an exemplary class of metric structures, exhibiting fascinating connections with model theory and Urysohn's universal space \cite{BryantNiesEtal17,BryantNiesEtal21,Hallback20,hallback2020automorphism}. Other directions that have been pursued are a diversity analogue of ultrametric and normed spaces \cite{HaghmaramNourouzi20,MehrabaniNourouzi20,haghmaram2022diversity} and new diversity-based results in fixed point theory \cite{espinola2014diversities,Piatek14}.\\ Our focus here is on the intersection of diversity theory, geometry and convex analysis. Recall that the Minkowski sum of two subsets $A, B \subseteq \Re^k$ is given by \( A+ B = \{ a + b : a \in A, b \in B\}. \) We investigate diversities defined on $\Re^k$ which are {\em (Minkowski) linear} \cite{Schneider14} \begin{quotation} \noindent (D5) $\delta(\lambda A) = \lambda \delta(A) \mbox{ and } \delta(A+B) = \delta(A) + \delta(B)$ \end{quotation} and those which are {\em (Minkowski) sublinear} \begin{quotation} \noindent (D6) $\delta(\lambda A) = \lambda \delta(A) \mbox{ and } \delta(A+B) \leq \delta(A) + \delta(B),$ \end{quotation} for $\lambda \geq 0$ and $A,B$ nonempty finite subsets of $\Re^k$. Many familiar diversities defined on $\Re^k$ are Minkowski linear or sublinear (see below). We explore their properties and characterization. As per usual, we make repeated use of support functions when dealing with convex bodies and functions defined on them. The {\em support function} of a nonempty bounded set $A$ is defined \[h_A:\Re^k \rightarrow \Re:x \mapsto \sup\{a\cdot x:a \in A\}.\] Here $a \cdot x$ denotes the usual dot product in $\Re^k$, \[a \cdot x =\sum_{i=1}^k a_i x_i.\] We note that a set has the same support function as both its closure and convex hull. We make use of the following properties of the support function, see \cite[Chapter 1]{Schneider14} for further details. \begin{enumerate} \item $h_{A+B} = h_A + h_B$ and $h_{\lambda A} = \lambda h_A$ for for non-empty, bounded $A,B$ and $\lambda \geq 0.$ \item If $A,B$ are nonempty convex compact sets then $A \subseteq B$ if and only $h_A(x) \leq h_B(x)$ for all $x \in \Re^k$ \item A function $h:\Re^k \rightarrow \Re$ is the support function for some bounded nonempty set if and only if $h(x+y) \leq h(x) + h(y)$ and $h(\lambda x) = \lambda h(x)$ for all $x,y \in \Re^k$ and $\lambda \geq 0$ (that is, $h$ is sublinear). \end{enumerate} We often consider support functions restricted to $\bbS^{k-1}$, the unit sphere in $\Re^k$, noting that a support function is determined everywhere by its values on $\bbS^{k-1}$. We note that the support function restricted to $\bbS^{k-1}$ of a nonempty set is bounded if and only if the set is bounded. Our main results for diversities and semidiversities $(\Re^k,\delta)$ are: \begin{enumerate} \item (Theorem~\ref{thm:linear_characterize}) Linear diversities and semidiversities are exactly those which can be written in the form \[ \delta(A) = \int_{\bbS^{k-1}} h_A(x) \mathrm{d}\nu(x) \] for a Borel measure $\nu$ on the sphere $\bbS^{k-1}$ satisfying \begin{equation} \int_{\bbS^{k-1}} x \, \mathrm{d}\nu(x) = 0. \end{equation} \item (Theorem~\ref{thm:extremal_linear}) The extremal linear semidiversities are those where the support of $\nu$ is a finite, affinely independent set, which in turn correspond to a generalized circumradius (a Minkowski semidiversity) based on the simplex. \item (Theorem~\ref{linearSimplex}) A diversity or semidiversity is sublinear if and only if it is the maximum of linear semidiversities (just like a function is convex if and only if it is the maximum of linear functions). \end{enumerate} We then shift to studying the embeddings of finite diversities into linear or sublinear diversities. Questions regarding embeddings and approximate embeddings of metrics in normed spaces are central to metric geometry and its applications. Consider, for example, Menger's characterizations of when a metric can be embedded in Euclidean space, or the vast literature applying metric embeddings to combinatorial optimizations (reviewed in \cite{matousek2013lectures,DezaLaurent97} and \cite{indyk20178}). For finite diversities $(X,\delta)$ we show: \begin{enumerate} \item (Theorem~\ref{thm:linearEmbed}) A finite diversity can be embedded in a linear diversity if and only if it has {\em negative type}, meaning that \[\sum_{A,B} x_A x_B \delta(A \cup B) \leq 0\] for all vectors $x$ with zero sum and $x_\emptyset=0$. \item (Theorem~\ref{thm:sublinearEmbed}) A finite diversity can be embedded as a sublinear diversity if and only if it can be embedded in a Minkowski diversity (that is, a generalized circumradius) if and only if it is the maximum of a collection of negative type diversities. \end{enumerate} \section{Linear and sublinear diversities} In this section we establish basic properties and characterizations for linear and sublinear diversities. \subsection{Examples of Linear and Sublinear Diversities} We start with examples of diversities which are linear or sublinear. Note that for all diversities $(X,\delta)$ we have $\delta(\emptyset) = 0$, even if that is not stated explicitly below. \begin{enumerate} \item Let $\| \cdot \|$ be any norm on $\Re^k$. The {\em diameter diversity} is given by \[\delta(A) = \max_{a,b \in A} \|a-b\|\] for finite $A \subseteq \Re^k$. The diameter diversity is sublinear \cite[pg 49]{Schneider14}. \item The $\ell_1$ diversity $(\Re^k,\delta_1)$ is \[ \delta_1(A) = \sum_{i=1}^k \max_{a,b \in A} (a_i - b_i). \] for finite $A\subseteq \Re^k$ \cite{BryantTupper14}. For finite $A,B$ and $\lambda \geq 0$ we have \begin{align*} \delta_1(\lambda A+B) & = \sum_{i=1}^k \max\left\{ \left((\lambda a+b)_i - (\lambda a' + b')_i\right): a,a' \in A, \, b,b' \in B \right\} \\ & = \sum_{i=1}^k \max\{\lambda(a_i - a_i') + (b_i - b_i') : a,a' \in A, \, b,b' \in B \} \\ & =\lambda \delta_1(A) + \delta_1(B). \end{align*} so $(\Re^k,\delta_1)$ is a linear diversity. \item The {\em circumradius} of finite $A \subset \Re^k$ with respect to the unit ball $\sB$ is \[\delta(A) = \min\{\lambda \geq 0: A \subseteq \lambda \sB + x \mbox{ for some $x \in \Re^k$} \}. \] More generally, the {\em Minkowski diversity} $(\Re^k,\delta_K)$ with kernel $K$ is equal to the generalized circumradius \[ \delta_K(A) = \inf \{\lambda \geq 0 : A \subseteq \lambda K +x \mbox{ for some } x \in \Re^k\}, \] for finite $A \subseteq \Re^k$. Minkowski diversities are sublinear \cite{bryant2023diversities} but are not, in general, linear. For example, consider the circumradius diversity $(\Re^2,\delta)$. If $A = \{(0,0),(1,0)\}$ and $B = \{(0,0),(0,1)\}$ then $\delta(A+B) < \delta(A) + \delta(B)$. We assume that $K$ is closed, convex and has non-empty interior. We have elsewhere required that the kernel $K$ be bounded, however in this paper we will not require this. Note that if $K$ is an unbounded, $(\Re^k,\delta_K)$ is a semidiversity rather than a diversity. \item The {\em mean-width diversity} $(\Re^k,\delta_w)$ is \[\delta_w(A) = \frac{2}{\omega_k} \int_{\bbS^{k-1}} h_A(x) \, d \nu(x) \] where $\nu(x)$ is the (uniform) Haar measure on the sphere and $\omega_k = \int_{\bbS^{k-1}} \, d \nu(x)$. Equivalently, $\delta_w(A)$ is the mean-width of the convex hull of $A$. Mean-width diversities are linear \cite[pg 50]{Schneider14}. Let $w_A(x) = \max\{x \cdot (a-b):a,b\in A\}$ denote the width of $A$ in direction $x$, so $w_A(x) = h_{A-A}(x) = h_A(x) + h_A(-x)$. Then \[ \delta_w(A) = \frac{1}{\omega_k} \int_{\bbS^{k-1}} w_A(x) \, d \nu(x). \] For $1 \leq p < \infty$ we define \[ \delta^{(p)}_w(A) = \frac{1}{\omega_k} \left[\int_{\bbS^{k-1}} |w_A(x)|^p \, d \nu(x) \right]^{1/p}. \] That this is a sublinear diversity follows from the Minkowski inequality. See \cite{HaghmaramNourouzi20} Proposition 2.4, or \cite{BryantCioica-LichtEtal21} Proposition 10 in the case that $p=2$. \item A {\em zonotope} $Z$ is a Minkowski sum of line segments and the {\em length} $\ell(Z)$ of the zonotope equals the sum of the length of the line segments. We define the {\em zonotope diversity} $(\Re^k,\delta_z)$ where $\delta_z(A)$ is the minimum length of a zonotope containing $A$. We show that zonotope diversities are sublinear in Proposition~\ref{prop:zonotope}. \end{enumerate} In a Euclidean space $\Re^k$, any non-negative linear combination of sublinear semidiversities is sublinear, and any non-negative linear combination of linear semidiversities is linear. Hence the set of sublinear semidiversities forms a convex cone, as does the set of linear semidiversities. \subsection{Properties of linear and sublinear diversities} We establish some basic properties of sublinear diversities (and hence of linear diversities). This includes the continuous extension of sublinear diversities from finite sets to bounded sets. \begin{proposition} \label{prop:sublinearProperties} Let $\delta$ be a function on finite subsets of $\Re^k$ which satisfies \textnormal{(D1)}, monotonicity \textnormal{(D3)} and sublinearity \textnormal{(D6)}. \begin{enumerate} \item $\delta$ is translation invariant: $\delta(A + x) = \delta(A)$ for all finite $A \subseteq \Re^k$ and $x \in \Re$. \item $(\Re^k,\delta)$ is a diversity. \item If $\conv(A) = \conv(B)$ then $\delta(A) = \delta(B)$. \item The map $N:\Re^k \rightarrow \Re$ given by $N(x) = \delta(\{0,x\})$ is a norm on $\Re^k$. \item For all finite $A \subseteq \Re^k$ with $|A| > 2$ we have \[\delta(A) \leq \mbox{ $\frac{|A|-1}{|A|(|A|-2)}$}\sum_{a \in A} \delta(A \setminus \{a\}).\] \end{enumerate} If $\delta$ satisfies (D1$'$) rather than (D1) then 1-5 still hold except that $(\Re^k,\delta)$ is a semidiversity and $N$ is a seminorm. \end{proposition} \begin{proof} \begin{enumerate} \item By sublinearity (D6) and (D1), we have \begin{align*} \delta(A+x) &\leq \delta(A) + \delta(\{x\}) = \delta(A), \mbox{ and } \\ \delta(A)& \leq \delta(A+x) + \delta(\{-x\}) = \delta(A+x). \end{align*} \item As $\delta$ is monotonic and $\delta(\emptyset) = 0$, $\delta$ is non-negative, and by part 1. $\delta$ is translation invariant. We show that $(X,\delta)$ satisfies (D4). Suppose that $x \in A \cap B$. Then $0 \in (A-x) \cap (B-x)$ and so $(A-x) \cup (B-x) \subseteq (A-x)+(B-x)$ and \begin{align*} \delta(A \cup B) &= \delta\Big( (A -x) \cup (B -x ) \Big) \\&\leq \delta \Big( (A -x) + (B -x ) \Big) \\ &\leq \delta(A-x) + \delta(B-x) \\ &= \delta(A) + \delta(B).\end{align*} Hence $(\Re^k,\delta)$ satisfies (D1), (D3) and (D4). \item Proposition 2.2b in \cite{bryant2023diversities}. \item By (D5) we have $N(x+y) = \delta(\{0,x+y\}) \leq \delta(\{0,x\}) + \delta(\{0,y\}) = N(x) +N(y)$. If $\lambda>0$ then $N(\lambda x) = \delta(\{0,\lambda x\}) = \lambda \delta(0,x) = |\lambda| N(x)$, while if $\lambda<0$ we have \[N(\lambda x) = \delta(\{\lambda x,0\}) = \delta(\{0,-\lambda x\}) = |\lambda| N(x).\] Also, $N(0)=\delta(\{0\})=0$ if and only if $\delta(\{x,0\}) = 0$ if and only if $x=0$. \item This follows from sublinearity and the following observation; see the proof of \cite[Theorem 4.1]{BrandenbergKonig13}. By sublinearity we may assume $\sum_{a \in A} a =0$. So for each $a \in A$ \[ -\frac{1}{|A|-1} a= \frac{1}{|A|-1} \sum_{a' \neq a} a' \in \conv ( A \setminus \{a\}). \] We also have $a \in \conv(A \setminus a')$ for $a \neq a'$. So for all $a \in A$ \[ \frac{(|A|-2)|A|}{|A|-1} a = (|A|-1) a - \frac{1}{|A|-1} a \in \sum_{b \in A} \conv( A \setminus \{b\}). \] This gives \[A \subseteq \frac{|A|-1}{|A|(|A|-2)} \sum_{a \in A} \conv(A \setminus \{a\})\] and applying sublinearity gives the result. \end{enumerate} \end{proof} It is now straightforward to show that the {\em zonotope diversity} introduced above is in fact a sublinear diversity. \begin{proposition} \label{prop:zonotope} The zonotope diversity $(\Re^k,\delta_z)$ is a sublinear diversity. \end{proposition} \begin{proof} Recall that $\delta_z(A)$ is the shortest length of a zonotope containing $A$. The function $\delta_z(A)$ is clearly monotonic, vanishes when $|A| \leq 1$, and is strictly positive when $|A|>1$. Given finite $A,B$, let $Z_A$ and $Z_B$ the the minimum length zonotopes containing $A$ and $B$ respectively. Then $Z_A + Z_B$ is a zonotope containing $A+B$ with length $\ell(Z_A) + \ell(Z_B)$. By Proposition~\ref{prop:sublinearProperties} part 2, $(\Re^k,\delta_z)$ is a sublinear diversity. \end{proof} The zonotope diversity is not linear: let $A = \{(0,0),(1,0),(0,1)\}$ and $B = -A$. Then $\delta_z(A) = \delta_z(B) = 2$ but $\delta_z(A+B) = 2+\sqrt{2}$. In a semidiversity, (D1) is replaced by (D1$'$), and sets with more than one element can have diversity zero. When the semidiversity is sublinear, the sets with zero diversity are highly structured. Define the null set of a semidiversity $(\Re^k,\delta)$ to be the set \[\nul(\delta) = \Big\{x : \delta(\{0,x\}) = 0 \Big\}\] and $\nul(\delta)^\perp = \{x \in \Re^k: x \cdot y = 0 \mbox{ for all } y \in \nul(\delta)\}$. \begin{proposition} Let $(\Re^k,\delta)$ be a sublinear semidiversity. \begin{enumerate} \item $\nul(\delta)$ is a linear subspace of $\Re^k$ \item $\delta$ restricted to $\nul(\delta)^\perp$ is a diversity \item If $P$ is the projection operator for $\nul(\delta)^\perp$ then $\delta(A) = \delta(PA)$ for all finite $A \subseteq \Re^k$. \end{enumerate} \end{proposition} \begin{proof} \begin{enumerate} \item For $x,y \in \nul(\delta)$ and $\alpha>0$ we have $\delta(\{0,x+y\}) \leq \delta(\{0,x\}) + \delta(\{0,y\})=0$ and $\delta(\{0,\alpha x\}) = \alpha \delta(\{0,x\}) = 0$ so $x+y \in \nul(\delta)$ and $\alpha x \in \nul(\delta)$. By translation invariance, $\delta(\{0,-x\}) = \delta(\{x,0\}) = 0$ and $-x \in \nul(\delta)$. \item Suppose $x,y \in \nul(\delta)^\perp$ and $\delta(\{x,y\})=0$. We have that $x-y \in \nul(\delta)^\perp$ by part 1. By translation invariance $\delta(\{0,x-y\})=\delta(\{x,y\}) =0$ which implies $x-y \in \nul(\delta)$. Hence $x-y$ is both in a subspace and its orthogonal complement, and so $x=y$. \item For all finite $A \subseteq \Re^k$ we have $A \subseteq PA + B$ and $PA \subseteq A + C$ for some $B,C \subset \nul(\delta)$. We have $\delta(B) = 0$ since \[0 \leq \delta(B) \leq \sum_{b \in B} \delta(\{0,b\}) = 0,\] and, likewise, $\delta(C)=0$. By sublinearity, $\delta(A) = \delta(PA)$. \end{enumerate} \end{proof} Let $\| \cdot \|$ be a norm on $\Re^k$ with associated metric $d(x,y) = \|x-y\|$ and unit ball $\sB = \{x : \|x\| \leq 1\}$. The {\em Hausdorff distance} between two nonempty closed bounded sets $K$ and $L$ can be defined by \cite[p.\ 61]{Schneider14} : \[d_H(K,L) = \min \{\lambda: K \subseteq L + \lambda \sB \mbox{ and } L \subseteq K + \lambda \sB\}. \] For bounded $K \subseteq \Re^k$ define \begin{equation} \delta^*(K) = \sup\{\delta(A) : A \subseteq K \mbox{ finite} \}. \label{eq:deltastar} \end{equation} \begin{proposition} \label{prop:dstar} Let $(\Re^k,\delta)$ be a sublinear semidiversity. \begin{enumerate} \item For all bounded $K \subseteq \Re^k$, $\delta^*(K) < \infty$. \item For all finite $A \subseteq \Re^k$ we have \[ \delta^*(\conv(A)) = \delta(A). \] \item For all bounded $K,L \subset \Re^k$ and $\lambda \geq 0$ \[ \delta^*(K+L) \leq \delta^*(K) + \delta^*(L) \] and \[ \delta^*(\lambda K) = \lambda \delta^*(K).\] \item If $(\Re^k,\delta)$ is linear then the restriction of $\delta^*$ to the set of nonempty compact convex subsets of $\Re^k$ is a valuation. That is, \[\delta^*(K \cap L) + \delta^*(K \cup L) = \delta^*(K) + \delta^*(L)\] for all nonempty compact convex bodies $K,L$ such that $K \cap L$ and $K \cup L$ are non-empty and convex. \item The restriction of $\delta^*$ to the set of nonempty compact convex subsets of $\Re^k$ is Lipschitz continuous with respect to the Hausdorff metric, with Lipschitz constant $\delta^*(\sB)$. \end{enumerate} \end{proposition} \begin{proof} \begin{enumerate} \item By equivalency of norms on $\Re^k$ we have that $K$ is bounded with respect to metric $d$ if and only if it is bounded with respect to the induced metric of $\delta$. Let $V$ be the set of vertices of some polytope (e.g. a cube) containing $K$. For all finite $A \subseteq K$ we have by monotonicity and part 2 that \[\delta(A) \leq \delta(A \cup V) = \delta(V)\] so that $\delta^*(K) \leq \delta(V) < \infty.$ \item Let $A$ be a finite subset of $\Re^k$ and let $K = \conv(A)$. For any $A' \subseteq K$ we have $\conv(A \cup A') = \conv(A)$ so $\delta(A') \leq \delta(A \cup A') = \delta(A)$ by Proposition~\ref{prop:sublinearProperties} (ii). Hence \[\delta(A) \leq \sup\{\delta(A'): \mbox{ finite } A' \subseteq K ) \} \leq \delta(A).\] \item Fix $\epsilon>0$ and suppose that $C$ is a finite subset of $K+L$ such that $\delta(C) > \delta^*(K+L) - \epsilon$. For each $c \in C$ there is $a_c \in K$ and $b_c \in L$ such that $c = a_c + b_c$. Let $A = \{a_c:c \in C\} \subseteq K$ and $B = \{b_c:c \in C\} \subseteq L$ so that $C \subseteq A+B$. It follows that \[\delta^*(K+L) - \epsilon < \delta(C) \leq \delta(A+B) \leq \delta(A) + \delta(B) \leq \delta^*(K) + \delta^*(L).\] Taking $\epsilon$ to zero gives the result. Let $A$ be a finite subset of $K$ such that $\delta(A) > \delta^*(K) - \epsilon$. As $\lambda A \subseteq \lambda K$ we have \[\lambda (\delta^*(K) - \epsilon) < \lambda \delta(A) = \delta( \lambda A) \leq \delta^*(\lambda K).\] Hence $\delta^*(\lambda K) \geq \lambda \delta^*(K)$ from which equality follows by symmetry. \item By Lemma 3.1.1 of \cite{Schneider14} we have that if $K,L,K\cup L$ and $K\cap L$ are nonempty compact convex subsets then \[(K \cup L) + (K \cap L) = K+L. \] By linearity, \[\delta^*(K \cup L) + \delta^*(K \cap L) = \delta^*(K) + \delta^*(L).\] \item Suppose that $K,L$ are bounded nonempty subsets satisfying $d_H(K,L) = \lambda$. For any $\epsilon>0$ there is a finite $A \subseteq K$ such that $\delta(A) \leq \delta^*(K) < \delta(A) + \epsilon$. We also have $A \subseteq K \subseteq L + \lambda \sB$ so there is finite $B \subseteq L$ and $C \subseteq \sB$ such that $A \subseteq B + \lambda C$. Hence \[ \delta^*(K) < \delta(A) + \epsilon \leq \delta(B) + \lambda \delta(C) + \epsilon \leq \delta^*(L) + \lambda \delta^*(\sB) + \epsilon.\] By a symmetric argument, \[ \delta^*(L) < \delta^*(K) + \lambda \delta^*(\sB) + \epsilon.\] Taking $\epsilon$ to zero, we have \[ |\delta^*(K) - \delta^*(L) | \leq \delta^*(\sB) d_H(K,L).\] The bound is tight, as can be seen by letting $K = \sB$ and $L = 2 \sB$. Then $d_H(K,L) = 1$ and $|\delta^*(K) - \delta^*(L) | = \delta^*(\sB)$. \end{enumerate} \end{proof} Bryant et al.~\cite{bryant2023diversities} also describe an extension of Minkowski diversities from finite sets to bounded sets. They define $\tdelta(P) = \delta(\mathrm{vert}(P))$ for any polytope with vertex set $\mathrm{vert}(P)$, and extend that to general bounded convex sets $K$ by defining $\tdelta(K) = \lim_{n \rightarrow \infty} \tdelta(P_n)$ for any sequence of polytopes $P_1,P_2,\ldots$ converging to $K$. Proposition~\ref{prop:dstar} part 2. gives that $\delta^*(P) = \tdelta(P)$ for any polytope, while from Proposition~\ref{prop:dstar} part 5 we have that $\delta^*(K) = \tdelta(K)$. Hence $\delta^*$ coincides with $\tdelta$ for Minkowski diversities. \subsection{Characterization of linear diversities} The following characterization of linear diversities is essentially contained in the proof of the main theorem in Firey \cite{firey1976functional}; see also \cite{meyer1995convex}. \begin{theorem}\label{thm:linear_characterize} Let $\delta$ be a function defined on finite subsets of $\Re^k$. Then $(X,\delta)$ is a linear semidiversity if and only if there is a positive finite Borel measure $\nu$ on the unit sphere $\bbS^{k-1} = \{x \in \Re^k: \|x\|_2 = 1\}$ such that \begin{equation} \label{eq:centroid} \int_{\bbS^{k-1}} x \, \mathrm{d}\nu(x) = 0 \end{equation} and \begin{equation} \delta(A) = \int_{\bbS^{k-1}} h_A(x) \, \mathrm{d}\nu(x) \label{eq:linearInt} \end{equation} for all finite $A \subseteq \Re^k$. Such a measure is unique. \end{theorem} \begin{proof} First we show that $\delta$ given by \eqref{eq:linearInt} is a linear semidiversity. For $a \in \Re^k$ and finite $A \subseteq B \subseteq \Re^k$ we have \begin{align*} \delta(\{a\}) & = \int_{\bbS^{k-1}} h_{\{a\}}(x) \, \mathrm{d}\nu(x) = \int_{\bbS^{k-1}} a \cdot x \, \mathrm{d}\nu(x) = a \cdot \int_{\bbS^{k-1}} x \, \mathrm{d}\nu(x) = 0 \end{align*} and, since $h_A(x) \leq h_B(x)$ and $h_{A+B}(x) = h_A(x)+h_B(x)$ for all $x$ we have $\delta(A) \leq \delta(B)$ and $\delta(A+B) = \delta(A) + \delta(B)$. By Proposition~\ref{prop:sublinearProperties}, $(\Re^k,\delta)$ is a linear semidiversity. For the converse, let $(\Re^k,\delta)$ be a linear semidiversity and define $\delta^*$ as in \eqref{eq:deltastar}. By Proposition~\ref{prop:dstar} the restriction of $\delta^*$ to nonempty compact convex subsets is Minkowski linear, monotonic and vanishes on singletons. From the proof of the main theorem in \cite{firey1976functional}, we have that, for all compact convex sets $K$, \[ \delta^*(K) = \int_{\bbS^{k-1}} h_K(x) \, \mathrm{d}\nu (x) \] for some positive finite Borel measure $\nu$ satisfying \eqref{eq:centroid}, and $\nu$ is the unique such measure. (See \cite[Thm 2.14]{rudinrealandcomplex} for details on the use of the Riesz Theorem in this case.) Now for any nonempty finite $A$, let $K = \conv(A)$. Since $\delta(A)=\delta^*(K)$ and $h_A = h_K$, the result follows for all nonempty finite $A$. \end{proof} \begin{figure}[ht] \centering \includegraphics[width=0.8\textwidth]{LinearMeasures.eps} \caption{\label{fig:linearMeasures} Support of measures corresponding to (a) mean width; (b) the $L_1$ diversity; and (c) a Minkowski diversity with a simplex kernel. } \end{figure} In Figure~\ref{fig:linearMeasures} we depict the support for the measures corresponding to mean width (uniform on the unit circle), the $L_1$ diversity ($\pm e_i$), and the Minkowski diversity for a simplex kernel. The first two of these are easy enough to demonstrate. We prove the third example below, after we have a characterization of extremal linear diversities. \subsection{Extremal linear diversities} The set of linear semidiversities on $\Re^k$ forms a cone. A non-zero semidiversity $\delta$ is {\em extremal} (or lies on an extremal ray) if it cannot be expressed as the convex combination of two linear semidiversities which are not its scale copies. We make use of Theorem~\ref{thm:linear_characterize} to characterize the extremal linear diversities and semidiversities. First we prove a technical result simplifying evaluation of the Minkowski diversity for a simplex. \begin{lemma} \label{lem:linear_prog_simplex} Let $v_0,\ldots,v_j \in \Re^k$ be affinely independent with $\sum_{\ell=0}^j c_\ell v_\ell =0$, for some $c_\ell \geq 0$, $\sum_\ell c_\ell=1$. Define the polyhedron $K = \{ y : v_\ell \cdot y \leq 1, \, \text{for all } \ell \}$. Let $\delta_K$ be the Minkowski semidiversity given by $K$. Then \[ \delta_K(A) = \sum_{\ell=0}^j c_\ell h_A(v_\ell). \] for all finite $A \subseteq \Re^k$. \end{lemma} \begin{proof} Let $A = \{a_i\}_{i=1,\ldots,|A|}$. We express $\delta_K(A)$ as the solution to a linear program. Recall that $\delta_K(A)$ is the minimum $\lambda$ such that there is some $x \in \Re^k$ such that $a_i - x \in \lambda K$ for all $i$. We can rewrite this constraint as $v_\ell \cdot (a_i -x) \leq \lambda$ for all $i,\ell$. If we take $\lambda$ and $x$ to be our primal variables we get the following linear program: \begin{align*} \text{minimize} \quad & \lambda = (1,0) \cdot (\lambda, x) \\ \text{subject to} \quad & \lambda + v_\ell \cdot x \geq v_\ell \cdot a_i, \mbox{for all }i \mbox{ and }\ell. \end{align*} The dual linear program with dual variables $y_{i\ell}$ is \begin{align*} \text{maximize} \quad & \sum_{i\ell} (v_\ell \cdot a_i) y_{ij} \\ \text{subject to} \quad & y_{i\ell} \geq 0, \mbox{for all }i \mbox{ and }\ell, \\ & \sum_{i\ell} y_{i\ell} = 1, \\ & \sum_{i\ell} v_\ell y_{i\ell} = 0. \end{align*} Let $\bar{y}_\ell = \sum_i y_{i\ell}$. Then our dual constraints are equivalent to \[ \sum_{\ell} \bar{y}_\ell = 1, \ \ \ \sum_\ell \bar{y}_\ell v_\ell = 0. \] Since the $v_\ell$ are affinely independent and $\sum_\ell c_\ell v_\ell=0$, $\sum_\ell c_\ell=1$, there is a unique solution given by $\bar{y}_\ell=c_\ell$ for all $\ell$. Now it remains to determine for each $\ell$ the value of $y_{i\ell}$ for each $i$. We need to maximize $\sum_{i\ell} (v_\ell \cdot a_i) y_{i\ell}$ given $y_{i\ell}\geq 0$ and $\sum_i y_{i\ell}=c_\ell$. For each $\ell$, the solution is to let $y_{i\ell}=c_\ell$ for the $i$ that maximizes $(v_\ell \cdot a_i)$, and $0$ otherwise. This gives for the solution to the dual problem \[ \sum_\ell c_\ell \max_{a \in A} v_\ell \cdot a = \sum_\ell c_\ell h_A(v_\ell). \] \end{proof} The following theorem identifies extremal linear semidiversities as Minkowski diversities $\delta_K$ with $K$ equal to a simplex or a simplex plus a subspace. \begin{theorem} \label{thm:extremal_linear} The following are equivalent for a semidiversity $(\Re^k,\delta)$: \begin{enumerate} \item[(i)] $(\Re^k,\delta)$ is extremal in the class of linear semidiversities. \item[(ii)] $(\Re^k,\delta)$ satisfies \[\delta(A) = \int_{\bbS^{k-1}} h_A(x) \, \mathrm{d}\nu(x)\] for all finite $A \subseteq \Re^k$, where $\nu$ is a measure on $\bbS^{k-1}$ with $\int_{\bbS^{k-1}} x \, \mathrm{d} \nu(x) = 0$, such that the support of $\nu$ is a finite, affinely independent set. \item[(iii)] $(\Re^k,\delta)$ is a Minkowski semidiversity with kernel $K$ of the form \[K = \conv(W) + H^{\perp},\] where $W$ is an affinely independent set of points, $H$ is the affine closure of $W$,and $H^{\perp}$ is the orthogonal space to $H$. \end{enumerate} \end{theorem} \begin{proof} (i) $\Rightarrow$ (ii). Suppose $\delta$ is extremal and the support of $\nu$ is not affinely independent. Let $H$ be the affine hull of the support of $\nu$, with $\dim H=j$, and let $S=H \cap \bbS^{k-1}$. Affine dependence implies $\nu$ is not supported on only $j+1$ points or fewer. Therefore we can partition $S$ into $S_1,\ldots,S_{j+2}$, each with $\nu(S_i)>0$. Let $m_i = \int_{S_i} \, \mathrm{d}\nu(x) = \nu(S_i)>0$. Then \[ m = \int_S \, \mathrm{d}\nu(x) = \sum_i \int_{S_i} d\nu(x) = \sum_i m_i \] and \[ 0 = \int_S x \, \mathrm{d}\nu(x) = \sum_j \int_{S_i} x \, \mathrm{d}\nu(x) = \sum_i m_i x_i \] where $x_i = (\int_{S_i} x \, \mathrm{d}\nu(x))/m_i$. Choose a subset of the $x_i$ with $k+1$ points so that 0 is in the convex hull of them. Let's say they are $x_1, \ldots, x_{k+1}$. Find $\mu_i$ for $i=1,...,k+1$ such that $\sum_i \mu_i x_i = 0$, and $\mu_i < m_i$. Let $\mu_{k+2} = 0$. Now define $\nu'$ by $\nu'(A) = (\mu_j/m_j) \nu(A)$ for $A \subseteq S_i, j=1...k+1$, and zero otherwise. Then $\nu' \leq \nu$, and $\nu'$ has smaller support, because $m_{k+2}>0$ but $\mu_{k+2} = 0$. Also \[ \int x \, \mathrm{d}\nu'(x) = \sum_i \int_{S_i} x \frac{\mu_j}{m_j} d\nu(x) = \sum_i \frac{\mu_i}{m_j} m_i x_i = \sum_i \mu_i x_i = 0. \] We can now write $\nu = (\nu-\nu')+ \nu'$ where $\nu-\nu'$ and $\nu$ are not scale copies, so $\delta$ is not extremal in the cone of linear semidiversities. \\ (ii) $\Rightarrow$ (i). Suppose that \[\delta(A) = \int_{\bbS^{k-1}} h_A(x) \, \mathrm{d}\nu(x)\] for all finite $A \subseteq \Re^k$ and some measure $\nu$ on $\bbS^{k-1}$ with affinely independent support. Let $\delta_1$ and $\delta_2$ be linear semidiversities with corresponding measures $\nu_1$ and $\nu_2$. If $\supp(\nu_1) \subseteq \supp(\nu)$ then affine independence and the constraint that $\int_{\bbS^{k-1}} x \, \mathrm{d}\nu_1(x) = 0$ implies that $\nu_1$ is a scaled version of $\nu$. Likewise for $\nu_2$. Hence if $\nu = \lambda \nu_1 + (1-\lambda)\nu_2$ for $\lambda \in [0,1]$ we have that $\supp(\nu_1) \cup \supp(\nu_2) \subseteq \supp(\nu)$ and both $\nu_1$ and $\nu_2$ are scale versions of $\nu$. This shows that $\delta$ is an extremal linear diversity.\\ (ii) $\Rightarrow$ (iii). Let the support of $\nu$ be the points $u_0,\ldots,u_j$ with weights $m_\ell>0$ such that $\sum_\ell m_\ell u_\ell=0$. Let $m=\sum_\ell m_\ell$, $c_\ell=m_\ell/m$, and $v_\ell= m u_\ell$, so that $\delta(A) = \sum_\ell c_\ell h_A (v_\ell)$, $\sum_\ell c_\ell v_\ell=0$, and $\sum_\ell c_\ell =1$. Let $V=\{v_0,\ldots, v_j\}$, which is affinely independent because the $u_\ell$ are. Let $H$ be the span of $V$ and $H^\perp$ its orthogonal complement. By Lemma~\ref{lem:linear_prog_simplex}, $\delta=\delta_K$, the Minkowski semidiversity for the set $K = \{ y : y \cdot v_\ell \leq 1, \mbox{ for all } \ell \}$. The intersection of $K$ with $H$ is a simplex; let it have vertices $W=\{ w_0,\ldots,w_j\}$. Then $K = \conv(V)+ H^\perp$ as required. \\ (iii) $\Rightarrow$ (ii). By translating $K$ if necessary, we may assume $0$ is in the relative interior of $\conv(W)$. We can write $K= \{ y : v_\ell \cdot y \leq 1\}$ for some affinely independent $V=\{v_0,\ldots,v_j\}$. Because $\conv(W)$ is bounded, $0 \in \conv(V)$. So there are $c_\ell \geq 0$ with $\sum_{\ell} c_\ell v_\ell = 0$ and $\sum_\ell c_\ell=1$. By Lemma~\ref{lem:linear_prog_simplex} we have \[ \delta_K(A) = \sum_\ell c_\ell h_A(v_\ell) \] for all finite $A$. Let $m_\ell= c_\ell |v_\ell|$ and $u_\ell=v_\ell/|v_\ell|$. Then the $u_\ell$ are also affinely independent. Let $\nu$ be the measure that assigns mass $m_\ell$ to each $u_\ell$. \end{proof} Points in a finite dimensional convex cone can always be written as convex combinations of extremal points. The cone of linear semidiversities has infinite dimensional, so proving that linear semidiversities are in the convex hull of extremal diversities requires a little more work. \begin{theorem} \label{linearSimplex} A semidiversity $(\Re^k, \delta)$ is linear if and only if $\delta$ is a convex combination of extremal linear semidiversity functions. \end{theorem} \begin{proof} Since a weighted average of linear semidiversities is a linear semidiversity, one way is immediate. For the other, suppose that $\delta$ is a linear semidiversity. By Theorem~\ref{thm:linear_characterize}, there is a Borel measure $\nu$ on $\bbS^{k-1}$ such that $\int x \, \mathrm{d}\nu(x)=0$ and $\delta(A) = \int h_A(x) \, \mathrm{d}\nu(x)=0$ for all finite $A$. Let $m =\int \, \mathrm{d}\nu(x)$. Let $E$ be the set of all signed Borel measures on $\bbS^{k-1}$, which is a Hausdorff locally convex set \cite[p.\ 134]{voigt2020course}. The space $C$ of measures on $\bbS^{k-1}$ with $\int x \, \mathrm{d}\nu(x)=m$ and $\int x \, \mathrm{d}\nu(x)=0$ is compact by the Banach-Alaoglu theorem \cite[p.\ 114]{voigt2020course}, and is convex. We claim that the set of extremal points of $C$ is closed. Let $\nu_n$, $n \geq 1$ be a sequence of extremal measures that converges in the vague topology, so that $\int f \, \mathrm{d}\nu_n$ converges to $\int f \, \mathrm{d}\nu$ for some $\nu \in C$ for all continuous bounded $f$. By repeatedly taking subsequences, we can obtain a subsequence $\nu_{n_k} = \sum_{i=1}^j \mu_{i,k} \delta_{x_{i,k}}$ (where $\delta_{x}$ is a unit mass measure at $x$) where $x_{i,k} \rightarrow x_i$ and $\mu_{i,k} \rightarrow \mu_i$ for some $x_i \in \mathbb{S}^{k-1}$ and $\mu_i \geq 0$. Since $\int f \, \mathrm{d}\nu_{n_k} \rightarrow \int f \, \mathrm{d}\nu$ as $k \rightarrow \infty$, we must have $\nu = \sum_i \mu_i \delta_{x_i}$, showing that the limit is also an extremal point in $C$. Hence the set of extremal measures is closed. We can apply a version of the Krein-Milman Theorem (\cite[Corollary 17.7]{voigt2020course}) to obtain that $\delta$ is a weighted average of members of the closure of the extreme points of $C$. Since the set of extremal points of $C$ is closed, the result follows. \end{proof} \subsection{Characterization of sublinear diversities} We now turn our attention to sublinear diversities. We will show that the relationship between sublinear and linear diversities parallels that between convex and linear functions. Just as every convex function is the supremum of linear functions, every sublinear diversity is the supremum of linear diversities (Theorem~\ref{thm:sublinearSup}). In fact, in our case, the supremum is attained for each set, so the value of every sublinear diversity on a set is the maximum of the value of a family of linear diversities on the set. Our proof relies heavily on the `Sandwich Theorem' (Theorem 1.2.5) of \cite{FuchssteinerLusky81convex}. \begin{theorem} \label{thm:sublinearSup} Let $\delta$ be a function on finite subsets of $\Re^k$. If $(\Re^k,\delta)$ is a sublinear diversity or semidiversity then there is a collection $\{(\Re^k,\delta_\gamma)\}_{\gamma \in \Gamma}$ of linear semidiversities such that \[\delta(A) = \max\{\delta_\gamma(A):\gamma \in \Gamma\}.\] Conversely, for any collection $\{(\Re^k,\delta_\gamma)\}_{\gamma \in \Gamma}$ of linear semidiversities and $\delta$ defined by $\delta(A) = \sup_{\gamma \in \Gamma} \delta_\gamma(A)$, $(\Re^k, \delta)$ is a sublinear semidiversity. \end{theorem} \begin{proof} Suppose that $\{(\Re^k,\delta_\gamma)\}_{\gamma \in \Gamma}$ are linear semidiversities and \[\delta(A) = \sup\{\delta_\gamma(A):\gamma \in \Gamma\}\] for all finite $A \subseteq \Re^k$. Note that $\delta$ vanishes on singletons and is monotonic since each $\delta_\gamma$ has these properties. Suppose that $A,B$ are finite subsets of $\Re^k$ and $\lambda \geq 0$. Then \[ \delta(\lambda A) = \sup\{\delta_\gamma(\lambda A):\gamma \in \Gamma\} = \sup\{\lambda \delta_\gamma(A):\gamma \in \Gamma\} = \lambda \delta(A)\] and \[\delta(A+B) = \sup\{\delta_\gamma(A+B):\gamma \in \Gamma\} = \sup\{\delta_\gamma(A) + \delta_\gamma(B):\gamma \in \Gamma\} \leq \delta(A) + \delta(B).\] So $\delta$ is sublinear. By Proposition~\ref{prop:sublinearProperties}, $\delta$ is a sublinear semidiversity. For the converse, suppose that $(\Re^k,\delta)$ is sublinear. Define $\sH$ to be the set of all support functions $h_A$ for nonempty finite $A \subseteq \Re^k$. Define $p$ on the convex cone $\sH$ by $p(h_A) = \delta(A)$ for all finite sets $A$. The function $p$ is sublinear (and convex in the terminology of \cite{stonyakin2016analogue}), as for any finite $A,B$, \[ p(h_A + h_B)= p(h_{A+B}) = \delta(A + B) \leq \delta(A) + \delta(B) = p(h_A)+p(h_B), \] and $p(\lambda h_A)=\lambda p(h_A)$ for $\lambda \geq 0$. Fix finite $B \subseteq \Re^k$. Define $q_B$ on $\sH$ by \[q_B(h_A) = \sup\{\lambda: \lambda B +x \subseteq \conv(A) \mbox{ for some $x \in \Re^k$}\}.\] That is, $q_B(h_A)$ is the largest we can scale $B$ so that a translate is contained in $\conv(A)$. Note that $q_B(h_B)=1$. This tells us that $p(h_B)=\delta(B) = \delta(B) q_B(h_B)$. We show that $q_B$ is superlinear. For all $\alpha \geq 0$ we have $q_B(\alpha h_A) = q_B(h_{\alpha A}) = \alpha q_B(h_A)$. Now suppose that $A_1,A_2$ are finite and non-empty subsets of $\Re^k$. Given $\epsilon>0$ there are $\lambda_1 > q_B(h_{A_1}) - \epsilon/2$, $\lambda_2 > q_B(h_{A_2}) - \epsilon/2$, $x_1,x_2 \in \Re^k$ such that \begin{align*} \lambda_1 B + x_1 &\subseteq \conv(A_1) \\ \lambda_2 B + x_2 &\subseteq \conv(A_2) \intertext{ and hence} (\lambda_1 + \lambda_2) B + (x_1 + x_2) & \subseteq \conv(A_1) + \conv(A_2) \\ & = \conv(A_1 + A_2), \end{align*} so that $q_B(h_{A_1 + A_2}) \geq (\lambda_1 + \lambda_2) > q_B(h_{A_1}) + q_B(h_{A_2}) - \epsilon$. Taking $\epsilon \rightarrow 0$ gives superlinearity. We now have that $p$ is monotonic and sublinear and that $q_B$ is superlinear. Furthermore, for any finite $A \subseteq \Re^k$ and $\epsilon > 0$ there is $\lambda$ such that $q_B(h_A) - \epsilon < \lambda \leq q_B(h_A)$ and $x \in \Re^k$ such that $\lambda B + x \subseteq \conv (A)$, and so \begin{align*} p(h_A) & \geq p(h_{\lambda B}) \\ & = \lambda p(h_B) \\ & > (q_B(h_A) - \epsilon) \delta(B). \end{align*} Taking $\epsilon \rightarrow 0$ we conclude that $q_B(h_A) \delta(B) \leq p(h_A)$ for all $h_A \in \sH$. Recall that $q_B(h_B) \delta(B)= p(h_B)$. For each finite $B$ we have now satisfied the conditions for Theorem 1.2.5 of \cite{FuchssteinerLusky81convex}: \begin{quotation} \noindent Let $F$ be a pre-ordered cone and let $p:F \rightarrow \overline{\Re}$ be monotone and sublinear, $q:F \rightarrow \overline{\Re}$ superlinear with $q \leq p$. Then there is a monotone linear $\mu:F \rightarrow \overline{\Re}$ with $q \leq \mu \leq p$. \end{quotation} In our example $F$ is the cone $\sH$ of support functions of finite sets. Let $q(h)=\delta(B) q_B(h)$. Let $\mu_B \colon \sH \rightarrow \Re$ be the linear map given by the theorem. It is monotone, linear, and \[ \delta(B) q_B(h) \leq \mu_B(h) \leq p(h). \] Since by definition $p(h_{\{a\}})=\delta(\{a\})=0$ for all $a \in \Re^k$, $\mu_B(h_{\{a\}})=0$ for all $a \in \Re^k$. Now define $\delta_B$ by $\delta_B(A)=\mu_B(h_A)$ for all finite $A$. Then $\delta_B$ vanishes on singletons, it is monotone, linear, and hence also sublinear. By Proposition~\ref{prop:sublinearProperties} $(\Re^k,\delta_B)$ is a linear semidiversity. Because $\delta(B) q_B(h_B)=p(h_B)$, we have that \[\delta_B(B)=\mu_B(h_B) = \delta(B) q_B(B)=\delta(B)\] and for general finite $A$ we have \[\delta_B(A) =\mu_B(h_A) \leq p(h_A) = \delta(A).\] Repeating this process for all finite $B \subseteq \Re^k$ we obtain a set of linear semidiversities $\{\delta_B\}_{\text{finite } B \subseteq \Re^k}$ such that $\delta_B \leq \delta$ and $\delta_B(B) = \delta(B)$ for all finite $B \subseteq \Re^k$. So for all finite $A \subseteq \Re^k$, \[\delta(A) = \sup\{\delta_B(A) : \mbox{ finite } B \subseteq \Re^k \} = \max\{\delta_B(A) : \mbox{ finite } B \subseteq \Re^k \}, \] since the supremum is actually attained when $B=A$. \end{proof} \section{Embedding into linear and sublinear diversities} We now turn our attention from linear and sublinear diversities to the questions of when finite diversities can be isometrically embedded within linear or sublinear diversities. Questions about embedding of metric spaces have, of course, been central to metric geometry and its applications, particularly after Linial et al.\ \cite{LinialLondonEtal95} demonstrated the link between approximate embeddings and combinatorial optimization algorithms on graphs. We showed in \cite{BryantTupper14} that an analogous link holds between approximate embeddings of diversities and combinatorial optimization algorithms on hypergraphs. Here we only consider embeddings without distortion, that is, exact rather than approximate embeddings. A map $f:X_1 \mapsto X_2$ between two diversities $(X_1,\delta_1)$ and $(X_2,\delta_2)$ is an {\em isometric embedding} if $\delta_2(f(A)) = \delta_1(A)$ for all finite $A \subseteq X_1$. We say that a finite diversity $(X,\delta)$ is {\em linear-embeddable} if there is an isometric embedding from $(X,\delta)$ to a linear diversity on $\Re^k$ for some $k$ and {\em sublinear-embeddable} if there is an isometric embedding to some sublinear diversity on $\mathbb{R}^k$, for some $k$. At this stage we allow the dimension $k$ to be arbitrary. Theorem~\ref{thm:linearEmbed} gives a characterization of linear-embeddability while Theorem~\ref{thm:sublinearEmbed} gives a characterization of sublinear-embeddability. Minkowski diversities and negative type diversities were reviewed earlier. We first establish a lemma on finite diversities that are embeddable in extremal linear diversities. \begin{lemma} \label{lem:extremal_means_simplex_minkowski} If $(\Re^k,\delta)$ is an extremal linear semidiversity, and $X \subseteq \Re_k$ is finite then $(X,\delta)$ is Minkowski embeddable with a simplex kernel. \end{lemma} \begin{proof} By Theorem~\ref{thm:extremal_linear} $(\Re^k,\delta)$ is the Minkowski semidiversity with kernel $K = \conv(W) + H^{\perp}$, where $W$ is a set of affinely independent vectors lying in a subspace $H$. Let $T$ be an orthogonal matrix so that $TH = \Sp(\{e_1,\ldots, e_j\})$ and $TH^{\perp} = \Sp(\{e_{j+1},\ldots,e_j\})$. Let $T_H$ be the first $j$ rows of $T$, so that $T_H H = \Re^j$, $T_H H^\perp=\{0\}$ and $T_H K = \conv(T_H W)$ is a full-dimensional simplex in $\Re^j$. Then for all $\lambda$, $A + x \subset \lambda K$ for some $x \in \Re^k$ if and only if $T_H A + y \subset \lambda \conv(T_H W)$ for some $y \in \Re^j$. So $\delta(A) = \delta_{ \conv(T_H W)}(T_H A)$ for all finite $A$, as required. \end{proof} \begin{theorem} \label{thm:linearEmbed} Let $(X,\delta)$ be a finite diversity. The following are equivalent: \begin{enumerate} \item[(i)]$(X,\delta)$ is linear-embeddable. \item[(ii)] $(X,\delta)$ has negative type. \item[(iii)] $(X,\delta)$ can be embedded into a Minkowski diversity $(\Re^k,\delta_K)$ with kernel equal to a simplex $K \subseteq \Re^k$. \end{enumerate} \end{theorem} \begin{proof} (i)$\Rightarrow$(ii) Without loss of generality assume $X \subseteq \Re^k$ where $(\Re^k,\delta)$ is a linear diversity. From Theorem~\ref{linearSimplex} we have that any linear diversity can be expressed as a convex combination of extremal linear semidiversities. By Lemma~\ref{lem:extremal_means_simplex_minkowski} each of these extremal linear semidiversities can be expressed as a Minkowski diversity with a simplex, each of which has negative type by Theorem 17 in \cite{bryant2023diversities}. As the set of negative type diversities forms a convex cone, $(X,\delta)$ also has negative type.\\ (ii)$\Leftrightarrow$(iii) This is Theorem 17 in \cite{bryant2023diversities}.\\ (iii)$\Rightarrow$(i) Theorem~\ref{linearSimplex} shows that any Minkowski diversity with a simplex kernel (being a trivial example of a weighted average of such diversities) is linear. Therefore, if $(X,\delta)$ is embeddable in a Minkowski diversity with a simplex kernel, it also embeddable in a linear diversity. \end{proof} \begin{theorem} \label{thm:sublinearEmbed} Let $(X,\delta)$ be a finite diversity. The following are equivalent: \begin{enumerate} \item[(i)]$(X,\delta)$ is sublinear-embeddable. \item[(ii)] $(X,\delta)$ can be embedded into a Minkowski diversity. \item[(iii)] $(X,\delta)$ is the maximum of a collection of negative type diversities. \end{enumerate} \end{theorem} \begin{proof} (ii)$\Rightarrow$(i) If $(X,\delta)$ is embeddable as a Minkowski diversity, then it is sublinear-embeddable, since by Theorem 2.4 of \cite{bryant2023diversities} all Minkowski diversities are sublinear. \\ (i)$\Rightarrow$(ii) Let $(X,\delta)$ be a sublinear-embeddable. We may assume $X$ is a subset of $\Re^k$ where $(\Re^k,\delta)$ is a sublinear diversity. By Theorem~\ref{thm:sublinearSup} there is a family of linear semidiversities $\delta_\gamma$ for $\gamma \in \Gamma$ such that $\delta(A) = \max \delta_\gamma(A)$. Since $X$ is finite, it has a finite number of subsets, and so we may assume that $\Gamma$ is finite. By Proposition 4.1 (a) in \cite{bryant2023diversities} if two finite diversities are Minkowski embeddable, then so is their maximum, and hence the same is true of any finite number of finite Minkowski embeddable diversities. Therefore $(X,\delta)$ is Minkowski embeddable. \\ (i)$\Rightarrow$(iii) We may assume $X \subseteq \Re^k$ where $(\Re^k,\delta)$ is a sublinear diversity. By Theorem~\ref{thm:sublinearSup} there is a family of linear semidiversities $\delta_\gamma$ for $\gamma \in \Gamma$ such that $\delta(A) = \max \delta_\gamma(A)$. Since $X$ is finite, it has a finite number of subsets, and so we may assume that $\Gamma$ is finite. By Theorem \ref{thm:linearEmbed}, each of $(X,\delta_\gamma)$ is negative type, and therefore $(X,\delta)$ is the maximum of a collection of negative type diversities.\\ (iii)$\Rightarrow$(ii) Suppose $(X,\delta)$ is the maximum of a collection of negative type diversities. By Theorem~\ref{thm:linearEmbed} $(X,\delta)$ can then be represented as the maximum of a collection of Minkowski diversities, and since $X$ is finite, we may assume the collection is finite. By Proposition 4.1 (a) in \cite{bryant2023diversities} the maximum of a finite collection of Minkowski embeddable diversities is Minkowski embeddable. \end{proof} \bibliographystyle{plain} \begin{thebibliography}{10} \bibitem{BrandenbergKonig13} Ren{\'{e}} Brandenberg and Stefan K\"{o}nig. \newblock No dimension-independent core-sets for containment under homothetics. \newblock {\em Discrete and Computational Geometry}, 49:3--21, 2013. \bibitem{BryantCioica-LichtEtal21} David Bryant, Petru Cioica-Licht, Lisa~Orloff Clark, and Rachael Young. \newblock Inner products for convex bodies. \newblock {\em Journal of Convex Analysis}, 28(4):1249--1264, 2021. \bibitem{bryant2023diversities} David Bryant, Katharina~T Huber, Vincent Moulton, and Paul~F Tupper. \newblock Diversities and the generalized circumradius. \newblock {\em Discrete \& Computational Geometry}, pages 1--22, 2023. \bibitem{BryantNiesEtal17} David Bryant, Andr\'{e} Nies, and Paul~F. Tupper. \newblock A universal separable diversity. \newblock {\em Analysis and geometry in metric spaces}, 5(1):138--151, 2017. \bibitem{BryantNiesEtal21} David Bryant, Andr\'{e} Nies, and Paul~F. Tupper. \newblock Fra\"{i}ss\'{e} limits for relational metric structures. \newblock {\em Journal of Symbolic Logic}, 86(3):913--934, 2021. \bibitem{BryantTupper12} David Bryant and Paul~F Tupper. \newblock Hyperconvexity and tight-span theory for diversities. \newblock {\em Advances in Mathematics}, 231(6):3172--3198, 2012. \bibitem{BryantTupper14} David Bryant and Paul~F. Tupper. \newblock Diversities and the geometry of hypergraphs. \newblock {\em Discrete Mathematics and Theoretical Computer Science}, 16(2):1--20, 2014. \bibitem{DezaLaurent97} Michel~Marie Deza and Monique Laurent. \newblock {\em Geometry of Cuts and Metrics}, volume~15 of {\em Algorithms and Combinatorics}. \newblock Springer-Verlag, Berlin, 1997. \bibitem{espinola2014diversities} Rafa Esp{\'\i}nola and Bo{\.z}ena Piatek. \newblock Diversities, hyperconvexity and fixed points. \newblock {\em Nonlinear Analysis: Theory, Methods \& Applications}, 95:229--245, 2014. \bibitem{firey1976functional} William~J Firey. \newblock A functional characterization of certain mixed volumes. \newblock {\em Israel Journal of Mathematics}, 24:274--281, 1976. \bibitem{FuchssteinerLusky81convex} Benno Fuchssteiner and Wolfgang Lusky. \newblock {\em Convex cones}. \newblock North-Holland, Amsterdam, 1981. \bibitem{haghmaram2022diversity} Pouya Haghmaram, Shohreh~Golpaigani Fard, and Kourosh Nourouzi. \newblock Diversity-normed spaces and diversity embeddings. \newblock {\em Studia Mathematica}, 267:19--35, 2022. \bibitem{HaghmaramNourouzi20} Pouya Haghmaram and Kourosh Nourouzi. \newblock Ultradiversification of diversities. \newblock {\em Analysis and Geometry in Metric Spaces}, 8:410--417, 2020. \bibitem{hallback2020automorphism} Andreas Hallb{\"a}ck. \newblock Automorphism groups of universal diversities. \newblock {\em Topology and its Applications}, 285:107381, 2020. \bibitem{Hallback20} Andreas Hallb\"{a}ck. \newblock {\em Metric model theory, {P}olish groups \& diversities}. \newblock PhD thesis, Universit{\'e} de Paris, 2020. \bibitem{indyk20178} Piotr Indyk, Ji{\v{r}}{\'\i} Matou{\v{s}}ek, and Anastasios Sidiropoulos. \newblock Low-distortion embeddings of finite metric spaces. \newblock In {\em Handbook of discrete and computational geometry}, pages 211--231. Chapman and Hall/CRC, 2017. \bibitem{jozefiak2023diversity} Adam~D Jozefiak and F~Bruce Shepherd. \newblock Diversity embeddings and the hypergraph sparsest cut. \newblock {\em arXiv preprint arXiv:2303.04199}, 2023. \bibitem{LinialLondonEtal95} Nathan Linial, Eran London, and Yuri Rabinovich. \newblock The geometry of graphs and some of its algorithmic applications. \newblock {\em Combinatorica}, 15(2):215--245, 1995. \bibitem{matousek2013lectures} Jir{\i} Matou{\v{s}}ek. \newblock {\em Lectures on discrete geometry}. \newblock Springer Science \& Business Media, 2013. \bibitem{MehrabaniNourouzi20} Gholamreza~H. Mehrabani and Kourosh Nourouzi. \newblock Ultradiversities and their spherical completeness. \newblock {\em Journal of Applied Analysis}, 26(2):231--240, 2020. \bibitem{meyer1995convex} M~Meyer, G~Mokobodzki, and M~Rogalski. \newblock Convex bodies and concave functions. \newblock {\em Proceedings of the American Mathematical Society}, 123(2):477--484, 1995. \bibitem{Piatek14} B.~Pi\c{a}tek. \newblock On the gluing of hyperconvex metrics and diversities. \newblock {\em Annales Universitatis Paedagogicae Cracoviensis. 149, Studia Mathematica}, 13(1):65--76, 2014. \bibitem{rudinrealandcomplex} Walter Rudin. \newblock {\em Real and complex analysis}. \newblock McGraw-Hill, 3rd edition, 1987. \bibitem{Schneider14} Rolf Schneider. \newblock {\em Convex bodies: the {Brunn}--{Minkowski} theory}. \newblock Encyclopedia of Mathematics and It Applications. Cambridge University Press, Cambridge, 2nd edition, 2014. \bibitem{stonyakin2016analogue} Fedor~Sergeevich Stonyakin. \newblock An analogue of the {H}ahn--{B}anach theorem for functionals on abstract convex cones. \newblock {\em Eurasian Mathematical Journal}, 7(3):89--99, 2016. \bibitem{voigt2020course} J{\"u}rgen Voigt. \newblock {\em A course on topological vector spaces}. \newblock Springer, 2020. \bibitem{WuBryantEtal19} Pei Wu, David Bryant, and Paul~F. Tupper. \newblock Negative-type diversities, a multi-dimensional analogue of negative-type metrics. \newblock {\em The Journal of Geometric Analysis}, 31:1703--1720, 2021. \end{thebibliography} \end{document}
2412.07090v1
http://arxiv.org/abs/2412.07090v1
The maximum sturdiness of intersecting families
\documentclass[11pt,a4paper]{article} \usepackage{amsfonts,amsgen,amstext,amsbsy,amsopn,amsfonts,amssymb,amscd} \usepackage[leqno]{amsmath} \usepackage[amsmath,amsthm,thmmarks]{ntheorem} \usepackage{epsf,epsfig} \usepackage{float} \usepackage{dsfont} \usepackage{ebezier,eepic} \usepackage{color} \usepackage{tikz} \usepackage{multirow} \usepackage{mathrsfs} \usepackage{graphicx} \usepackage{subfigure} \usepackage{epstopdf} \setlength{\textwidth}{150mm} \setlength{\oddsidemargin}{7mm} \setlength{\evensidemargin}{7mm} \setlength{\topmargin}{-5mm} \setlength{\textheight}{245mm} \topmargin -18mm \newtheorem{thm}{Theorem}[section] \newtheorem{theoremA}{Theorem} \newtheorem{theoremB}{Theorem} \newtheorem{prop}[thm]{Proposition} \newtheorem{prob}[thm]{Problem} \newtheorem{lem}[thm]{Lemma} \newtheorem{example}[thm]{Example} \newtheorem{false statement}{False statement} \newtheorem{cor}[thm]{Corollary} \newtheorem{fact}[thm]{Fact} \newtheorem{assumption}{Assumption} \theoremstyle{definition} \newtheorem{defn}[thm]{Definition} \newtheorem{claim}[thm]{Claim} \newtheorem{remark}[thm]{Remark} \newtheorem{conj}[thm]{Conjecture} \newtheorem{corollary}[thm]{Corollary} \newtheorem{problem}{Problem} \newtheorem{case}{Case} \newtheorem{subcase}{Case}[case] \newtheorem{subsubcase}{Case}[subcase] \newtheorem{lemmaA}{Lemma} \newtheorem{factA}[lemmaA]{Fact} \renewcommand{\thetheoremA}{\Alph{theoremA}} \renewcommand{\thelemmaA}{\Alph{lemmaA}} \renewcommand{\thefactA}{\Alph{factA}} \newcommand{\Mod}[1]{\mathrm{mod}\ #1} \def\theequation{\thesection.\arabic{equation}} \makeatletter \@addtoreset{equation}{section} \baselineskip 15pt \renewcommand{\baselinestretch}{1.1} \newcommand{\de}{{\rm def}} \newcommand{\ex}{{\rm ex}} \def\hh{\mathcal{H}} \def\hm{\mathcal{M}} \def\hl{\mathcal{L}} \def\hht{\mathcal{T}} \def\he{\mathcal{E}} \def\hf{\mathcal{F}} \def\hg{\mathcal{G}} \def\hk{\mathcal{K}} \def\ha{\mathcal{A}} \def\hb{\mathcal{B}} \def\hs{\mathcal{S}} \def\hr{\mathcal{R}} \def\hc{\mathcal{C}} \def\hi{\mathcal{I}} \def\hj{\mathcal{J}} \def\hp{\mathcal{P}} \def\ex{\mathbb{E}} \begin{document} \title{\bf\Large The maximum sturdiness of intersecting families} \date{} \author{Peter Frankl$^1$, Jian Wang$^2$\\[10pt] $^{1}$R\'{e}nyi Institute, Budapest, Hungary\\[6pt] $^{2}$Department of Mathematics\\ Taiyuan University of Technology\\ Taiyuan 030024, P. R. China\\[6pt] E-mail: $^[email protected], $^[email protected] } \maketitle \begin{abstract} Given a family $\hf\subset 2^{[n]}$ and $1\leq i\neq j\leq n$, we use $\hf(\bar{i},j)$ to denote the family $\{F\setminus \{j\}\colon F\in \hf,\ F\cap \{i,j\}=\{j\}\}$. The sturdiness of $\hf$ is defined as the minimum $|\hf(\bar{i},j)|$ over all $i,j\in [n]$ with $i\neq j$. It has a very natural algebraic definition as well. In the present paper, we consider the maximum sturdiness of $k$-uniform intersecting families, $k$-uniform $t$-intersecting families and non-uniform $t$-intersecting families. One of the main results shows that for $n\geq 36(k+6)$, an intersecting family $\hf\subset \binom{[n]}{k}$ has sturdiness at most $\binom{n-4}{k-3}$, which is best possible. \end{abstract} \section{Introduction} Let $[n]$ be the standard $n$-set $\{1,2,\ldots,n\}$. We use $2^{[n]}$ to denote the power set of $[n]$ and $\binom{[n]}{k}$ to denote the collection of all $k$-subsets of $[n]$. For a family $\hf\subset 2^{[n]}$ , let $\hf^c$ be the family of complements, $\{[n]\setminus F\colon F\in \hf\}$. For definiteness let $\hf=\{F_1,F_2,\ldots,F_m\}$. Let $A(\hf)$ be the {\it incidence matrix} of $\hf$, i.e., the $n$ by $|\hf|$ (i.e., $n$ by $m$) 0-1-matrix with general entry \[ a_{i\ell} =\left\{\begin{array}{ll} 1, & \mbox{ if } i\in F_\ell; \\[5pt] 0, & \hbox{ if } i\notin F_\ell. \end{array} \right. \] Note that $A(\hf)+A(\hf^c)$ is the $n$ by $m$ all 1 matrix. Consider the product $B(\hf) = A(\hf)A(\hf^c)^T$. The general entry of $B=B(\hf)$ is \begin{align}\label{ineq-1} b_{ij} =\sum_{1\leq \ell\leq m} a_{i\ell} (1-a_{j\ell}). \end{align} For $\hf\subset 2^{[n]}$ and $i,j\in [n]$, let \[ \hf(i) =\{F\setminus \{i\}\colon i\in F\in \hf\},\ \hf(\bar{i}) =\{F\colon i\notin F\in \hf\}. \] Let $\hf(i,\bar{j})$ denote the family $\{F\setminus \{i\}\colon F\in \hf,\ F\cap \{i,j\}=\{i\}\}$. From \eqref{ineq-1}, $b_{ii}=0$ is obvious. On the other hand $b_{ij}$ for $i\neq j$ counts the number of $F\in \hf$ with $i\in F$, $j\notin F$. Hence \begin{align}\label{ineq-2} b_{ij} = |\hf(i,\bar{j})|. \end{align} \begin{defn} For $\hf\subset 2^{[n]}$ define the {\it sturdiness} $\beta(\hf)$ as $\min_{1\leq i\neq j \leq n} b_{ij}$. \end{defn} It should be clear that for $\hf\subset\tilde{\hf} \subset 2^{[n]}$, \[ b_{ij}(\hf)\leq b_{ij}(\tilde{\hf}) \mbox{ for all }i,j. \] Hence \begin{align}\label{ineq-3} \beta(\hf)\leq \beta(\tilde{\hf}). \end{align} \begin{prop}[Duality] \begin{align}\label{ineq-1.1} \beta(\hf)=\beta(\hf^c). \end{align} \end{prop} \begin{proof} Let us show that $|\hf(i,\bar{j})|=|\hf^c(\bar{i},j)|$. Let $X=[n]\setminus \{i,j\}$. For any $F\in \hf(i,\bar{j})$, $F\cup \{i\}\in \hf$. Then $(X\setminus F)\cup \{j\}\in \hf^c$, implying $X\setminus F\in \hf^c(\bar{i},j)$. It is easy to check that $\sigma \colon \hf(i,\bar{j})\mapsto \hf^c(\bar{i},j)$ defined by $\sigma(F)=X\setminus F$ is a bijection. Thus $|\hf(i,\bar{j})|=|\hf^c(\bar{i},j)|$ and \eqref{ineq-1.1} follows. \end{proof} For $\hf\subset 2^{[n]}$, $b_{ij}(\hf)=2^{n-2}$ for $i\neq j$, i.e., \begin{align} \beta(2^{[n]}) =2^{n-2}. \end{align} Similarly, \begin{align} \beta\left(\binom{[n]}{k}\right) =\binom{n-2}{k-1}. \end{align} A family $\hf\subset 2^{[n]}$ is called {\it $t$-intersecting} if $|F\cap F'|\geq t$ for all $F,F'\in \hf$. For $t=1$, we simply say that $\hf$ is intersecting. One of the most important results in extremal set theory is the following: \begin{thm}[The Erd\H{o}s-Ko-Rado Theorem \cite{ekr}]\label{thm-ekr} Suppose that $n\geq n_0(k,t)$ and $\hf\subset \binom{[n]}{k}$ is $t$-intersecting. Then \begin{align}\label{ineq-ekr2} |\hf| \leq \binom{n-t}{k-t}. \end{align} \end{thm} {\noindent\bf Remark.} For $t=1$ the exact value $n_0(k,t)=(k-t+1)(t+1)$ was proved in \cite{ekr}. For $t\geq 15$ it is due to \cite{F78}. Finally Wilson \cite{W84} closed the gap $2\leq t\leq 14$ with a proof valid for all $t$. The corresponding problem for {\it non-uniform} families was solved by Katona. \begin{thm}[The Katona Theorem \cite{Katona}]\label{thm-katona} Let $n>t>0$ and suppose that $\hf\subset 2^{[n]}$ is $t$-intersecting. Then (i) or (ii) holds. \begin{itemize} \item[(i)] $n-t$ is even and \begin{align}\label{ineq-katona1} |\hf| \leq \sum_{i\geq \frac{n+t}{2}}\binom{n}{i}. \end{align} \item[(ii)] $n-t$ is odd and \begin{align}\label{ineq-katona2} |\hf| \leq \binom{n-1}{\frac{n+t-1}{2}-1}+\sum_{ i\geq \frac{n+t+1}{2}}\binom{n}{i}. \end{align} \end{itemize} \end{thm} Our main results determine the maximum sturdiness of intersecting and $t$-intersecting families. \begin{thm}\label{thm-main} Let $n\geq 36(k+6)$ and $\hf\subset \binom{[n]}{k}$ be an intersecting family. Then \begin{align} \beta(\hf) \leq \binom{n-4}{k-3}. \end{align} \end{thm} Define the {\it triangle family} as \[ \hht=\hht(n,k)\colon = \left\{F\in \binom{[n]}{k}\colon |F\cap [3]|\geq 2\right\} \] The next claim shows that Theorem \ref{thm-main} is best possible. \begin{claim}\label{claim-1.6} For $n\geq 2k$, $\beta(\hht) =\binom{n-4}{k-3}$. \end{claim} For $t$-intersecting families, we establish the corresponding best possible result for $n\geq 2(t+3)^2k^2$. \begin{thm}\label{thm-main-1} Let $\hf\subset \binom{[n]}{k}$ be a $t$-intersecting family. If $n\geq 2(t+3)^2k^2$ then \[ \beta(\hf) \leq \binom{n-t-3}{k-t-2}. \] \end{thm} The next two results determine the maximum sturdiness of a non-uniform $t$-intersecting family. \begin{thm}\label{thm-2} If $\hf\subset 2^{[n]}$ is intersecting then $\beta(\hf)\leq 2^{n-3}$. \end{thm} \begin{thm}\label{thm-main-3} Suppose that $\hf\subset 2^{[n]}$ is $t$-intersecting. \begin{itemize} \item[(i)] If $n-t=2s$ then $\beta(\hf) \leq \sum\limits_{ 0\leq j\leq s-1} \binom{n-2}{j}$; \item[(ii)]If $n-t=2s+1$ and $n\geq \max\{4(s+2)^2, 36(s+7)\}$, then $\beta(\hf) \leq \binom{n-4}{s-2}+\sum\limits_{ 0\leq j\leq s-1} \binom{n-2}{j}$. \end{itemize} \end{thm} Let us recall a convenient notation. For $\hf\subset \binom{[n]}{k}$ and $A\subset B\subset [n]$, define \[ \hf(A,B) =\left\{F\setminus B\colon F\in \hf,\ F\cap B=A \right\}. \] \section{Shifting and proof of Claim \ref{claim-1.6}} The most important operation on intersecting families is shifting (cf. \cite{F87} for some basic properties). For $\hf\subset \binom{[n]}{k}$ and $1\leq i<j\leq n$, define the shift $$S_{ij}(\hf)=\left\{S_{ij}(F)\colon F\in\hf\right\},$$ where $$S_{ij}(F)=\left\{ \begin{array}{ll} F':= (F\setminus\{j\})\cup\{i\}, & \mbox{ if } j\in F, i\notin F \text{ and } F'\notin \hf; \\[5pt] F, & \hbox{otherwise.} \end{array} \right. $$ A family $\hf\subset \binom{[n]}{k}$ is called {\it shifted} if $S_{ij}(\hf)=\hf$ holds for all $1\leq i<j\leq n$. It is easy to show (cf. \cite{F87}) that every $k$-graph can be transformed into a shifted $k$-graph with the same number of edges by repeated shifting. Let us define the {\it shifting partial order} $\prec$. For two $k$-sets $A$ and $B$ where $A=\{a_1,\ldots,a_k\}$, $a_1<\ldots<a_k$ and $B=\{b_1,\ldots,b_k\}$, $b_1<\ldots<b_k$ we say that $A$ {\it precedes} $B$ and denote it by $A\prec B$ if $a_i\leq b_i$ for all $1\leq i\leq k$. \begin{prop}[\cite{F87}] If $\hf\subset \binom{[n]}{k}$ is shifted, then $A\prec B$ and $B\in \hf$ imply $A\in \hf$. \end{prop} Recall that $\gamma(\hf)=\min_{y\in [n]} |\hf(\bar{y})|$ is called the {\it diversity} of the family. For shifted families both $\gamma(\hf)$ and $\beta(\hf)$ are easy to compute: \begin{align} & \gamma(\hf) = |\hf(\bar{1})|,\label{ineq-2.1}\\[3pt] & \beta(\hf) =|\hf(\bar{1},n)|. \label{ineq-2.2} \end{align} A family $\hf\subset 2^{[n]}$ is called {\it $r$-wise $t$-intersecting} if $|F_1\cap F_2\cap \ldots\cap F_r|\geq t$ for all $F_1,F_2,\ldots,F_r\in \hf$. For $r=2$, we simply say that $\hf$ is $t$-intersecting. The first important property of shifted families, proved by Erd\H{o}s, Ko and Rado \cite{ekr} is \begin{flalign} &\mbox{If $\hf\subset\binom{[n]}{k}$ and $n\geq 2k$ then $\hf(n)\subset \binom{[n-1]}{k-1}$ is intersecting as well.}\\[3pt] &\mbox{(cf. \cite{F78}) If $\hf$ is shifted and intersecting then $\hf(\bar{1})$ is 2-intersecting.}\label{ineq-new2.4} \end{flalign} More generally, \begin{flalign}\label{ineq-new2.5} &\mbox{(\cite{F87}) If $\hf$ is shifted and $r$-wise $t$-intersecting then $\hf(\bar{1})$ is $(t+r-1)$-intersecting.} \end{flalign} \begin{cor}\label{cor-key} If $n\geq (t+r)(k-t-r+2)+2$ and $\hf\subset \binom{[n]}{k}$ is $r$-wise $t$-intersecting and shifted, then \begin{align} \beta(\hf) \leq \binom{n-t-r-1}{k-t-r}. \end{align} \end{cor} \begin{proof} Since $\hf$ is shifted, by \eqref{ineq-2.2} we infer that $\beta(\hf) =\hf(\bar{1},n)$. By \eqref{ineq-new2.5}, $\hf(\bar{1})\subset \binom{[2,n]}{k-1}$ is $(t+r-1)$-intersecting. We claim that $\hf(\bar{1},n)$ is also $(t+r-1)$-intersecting. Indeed, if there exist $E_1,E_2\in \hf(\bar{1},n)$ such that $|E_1\cap E_2|\leq t+r-2$ then $E_1\cup \{n\},E_2\cup\{n\}\in \hf(\bar{1})$ and $|E_1\cap E_2|= t+r-2$ follows. Since $|E_1\cup E_2|=2(k-1)-(t+r-2) =2k-t-r<n-2$, there exists $x\in [2,n-1]\setminus E_1\cup E_2$. By shiftedness, $E_2\cup \{x\} \in \hf(\bar{1})$. However, $|(E_1\cup \{n\})\cap (E_2\cup \{x\})|=t+r-2$, contradicting the fact $\hf(\bar{1})$ is $(t+r-1)$-intersecting. Thus $\hf(\bar{1},n)$ is $(t+r-1)$-intersecting. By \eqref{ineq-ekr2} and $n-2\geq (t+r)(k-t-r+2)$, $|\hf(1,\bar{n})|\leq \binom{(n-2)-(t+r-1)}{(k-1)-(t+r-1)}= \binom{n-t-r-1}{k-t-r}$. \end{proof} \begin{proof}[Proof of Claim \ref{claim-1.6}] For $3<i\neq j\leq n$, \[ b_{ij} = 3\binom{n-5}{k-3}+\binom{n-5}{k-4}= \binom{n-4}{k-3} +2\binom{n-5}{k-3}. \] For $1\leq i\leq 3$ and $3<j\leq n$, \[ b_{ij} =2\binom{n-4}{k-2}+\binom{n-4}{k-3}>\binom{n-4}{k-3}. \] For $1\leq j\leq 3$ and $3<i\leq n$, \[ b_{ij} =\binom{n-4}{k-3}. \] For $1\leq i\neq j\leq 3$, \[ b_{ij} =\binom{n-3}{k-2}>\binom{n-4}{k-3}. \] Thus for $n\geq 2k$, \[ \beta(\hht) =\binom{n-4}{k-3}. \] \end{proof} \section{Proof of Theorem \ref{thm-main}} \begin{prop} Let $\hf\subset \binom{[n]}{k}$ be an arbitrary family. Then \begin{align}\label{ineq-key} \beta(\hf) \leq \frac{k}{n-1} \gamma(\hf). \end{align} \end{prop} \begin{proof} Fix $y$ with $\hf(\bar{y}) =\gamma(\hf)$. Then \[ \sum_{x\in [n]\setminus \{y\}} |\hf(x,\bar{y})|= k|\hf(\bar{y})|. \] Hence there exists $x\neq y$ with \[ |\hf(x,\bar{y})| \leq \frac{k}{n-1}|\hf(\bar{y})| =\frac{k}{n-1} \gamma(\hf). \] \end{proof} \begin{cor}\label{cor-1} If $\gamma(\hf)<\frac{n-1}{k}\binom{n-4}{k-3} = \frac{n-1}{n-3} \frac{k-2}{k} \binom{n-3}{k-2}$, then $\beta(\hf)<\binom{n-4}{k-3}$. \end{cor} For $\{u,v,w\}\subset [n]$, let us introduce the notation: \[ \hht_{uvw} =\left\{F\in \binom{[n]}{k}\colon |F\cap \{u,v,w\}|\geq 2\right\}. \] We need the following two results. \begin{thm}[\cite{FW2022}]\label{thm-fw-2022} Let $n>36k$. Suppose that $\hf\subset \binom{[n]}{k}$ is intersecting. Then \begin{align} \gamma(\hf) \leq \binom{n-3}{k-2}. \end{align} \end{thm} \begin{thm}[\cite{FW2024}]\label{thm-fw2024} Let $\hf\subset \binom{[n]}{k}$ be an intersecting family with $k\geq 3$. Define $\alpha$ by $\gamma(\hf)=\left(1-\alpha\right)\binom{n-3}{k-2}$. If $0\leq \alpha<1$ and $n\geq \frac{36k}{1-\alpha}$, then there exists $\{u,v,w\}\subset [n]$ such that $\hf(\bar{u})=\gamma(\hf)$ and \begin{align*} &|\hf\setminus \hht_{uvw}| \leq 18\alpha \binom{n-33}{k-33}. \end{align*} \end{thm} \begin{proof}[Proof of Theorem \ref{thm-main}] If $\gamma(\hf)<\frac{k-2}{k} \binom{n-3}{k-2}$, then by Corollary \ref{cor-1} the theorem follows. Thus we may assume that $\gamma(\hf)=(1-\alpha) \binom{n-3}{k-2}$ with $\alpha\leq \frac{2}{k}$. For $k\geq 3$, $(k+6)(k-2)\geq k^2$. Hence $n\geq 36(k+6)$ and $\alpha\leq \frac{2}{k}$ imply $n\geq \frac{36k}{1-\alpha}$. By Theorem \ref{thm-fw2024} there exists $\{u,v,w\}\subset [n]$ such that $\hf(\bar{u})=\gamma(\hf)$ and \begin{align}\label{ineq4-1} &|\hf\setminus \hht_{uvw}| \leq 18\alpha \binom{n-33}{k-33}. \end{align} Set $T=\{u,v,w\}$. Let \begin{align*} &|\hf(\{u,v\},T)|=f_{uv},\ |\hf(\{u,w\},T)|=f_{uw},\ |\hf(\{v,w\},T)|=f_{vw} \end{align*} and \begin{align*} &|\hf(\{u\},T)|=g_u,\ |\hf(\{v\},T)|=g_v,\ |\hf(\{w\},T)|=g_w\ \mbox{and } |\hf(\emptyset,T)|=h. \end{align*} Then we have \begin{align*} &|\hf(\bar{u})| = f_{vw} +g_v+g_w+h =(1-\alpha) \binom{n-3}{k-2},\\[5pt] &|\hf(\bar{v})| = f_{uw} +g_u+g_w+h \leq \binom{n-3}{k-2}+g_u+g_w+h,\\[3pt] &|\hf(\bar{w})| = f_{uv} +g_u+g_v+h \leq \binom{n-3}{k-2}+g_u+g_v+h. \end{align*} It follows that \begin{align}\label{ineq-2.4} f_{vw}+f_{uw}+f_{uv}+2(g_u+g_v+g_w)+3h &\leq (3-\alpha) \binom{n-3}{k-2} +2g_u+g_v+g_w+2h \nonumber \\[3pt] &\leq (3-\alpha) \binom{n-3}{k-2}+2|\hf\setminus \hf_{uvw}^*|. \end{align} For any $x\in [n]\setminus \{u,v,w\}$, let $S=T\cup\{x\}$ and \[ |\hf(\{x,u,v\},S)|=f_{xuv},\ |\hf(\{x,u\},S)|=g_{xu}, |\hf(\{x\},S)|=h_x. \] Define $f_{xuw},f_{xvw},g_{xv},g_{xw}$ similarly. Suppose for contradiction that $\beta(\hf)>\binom{n-4}{k-3}$. That is, $|\hf(a,\bar{b})|>\binom{n-4}{k-3}$ for all $a,b\in [n]$. Then for any $x\in [n]\setminus \{u,v,w\}$, \begin{align*} &|\hf(x,\bar{u})| = f_{xvw}+g_{xv}+g_{xw}+h_x >\binom{n-4}{k-3},\\[3pt] &|\hf(x,\bar{v})| = f_{xuw}+g_{xu}+g_{xw}+h_x >\binom{n-4}{k-3},\\[3pt] &|\hf(x,\bar{w})| = f_{xuv}+g_{xu}+g_{xv}+h_x >\binom{n-4}{k-3}. \end{align*} Adding the above three inequalities, we get \[ (f_{xvw}+f_{xuw}+f_{xuv})+2(g_{xv}+g_{xu}+g_{xw})+3h_x>3\binom{n-4}{k-3}. \] Summing it over all $x\in [n]\setminus \{u,v,w\}$, we get \[ (k-2)(f_{vw}+f_{uw}+f_{uv})+2(k-1)(g_u+g_v+g_w)+3k h>3(n-3)\binom{n-4}{k-3}. \] It follows that \[ (f_{vw}+f_{uw}+f_{uv})+2\frac{k-1}{k-2}(g_u+g_v+g_w)+3\frac{k}{k-2} h>3\binom{n-3}{k-2}. \] Using \eqref{ineq-2.4}, we get \[ 3\binom{n-3}{k-2}<\frac{2}{k-2}(g_u+g_v+g_w)+\frac{6}{k-2} h+ (3-\alpha) \binom{n-3}{k-2}+2|\hf\setminus \hht_{uvw}|. \] By simplifying, \[ (k-2)\alpha \binom{n-3}{k-2} < 2(g_u+g_v+g_w)+6 h+2(k-2)|\hf\setminus \hht_{uvw}|. \] Since $g_u+g_v+g_w+h\leq |\hf\setminus \hht_{uvw}|$, \[ (k-2)\alpha \binom{n-3}{k-2} < 2(k+1)|\hf\setminus \hht_{uvw}|. \] Using \eqref{ineq4-1} we obtain that \[ (k-2)\alpha \binom{n-3}{k-2} <2(k+1)18\alpha \binom{n-33}{k-33}. \] If $\alpha=0$ or $k\leq 32$, then by Theorem \ref{thm-fw2024} $\hf\subset \hht_{uvw}$ and $\beta(\hf)\leq \binom{n-4}{k-3}$, contradicting our assumption. Thus $\alpha \neq 0$ and $k\geq 33$. Then \[ (k-2) \binom{n-3}{k-2} <36(k+1) \binom{n-33}{k-33}<36(k+1) \binom{n-33}{k-32}. \] Consequently, \[ \left(\frac{n-3}{k-2}\right)^{30} < 36\frac{k+1}{k-2}\leq 144, \] contradicting $n\geq 36(k+6)$. Thus the theorem holds. \end{proof} For $n=2k$ Erd\H{o}s, Ko and Rado noted that there are many different intersecting families $\hf\subset \binom{[n]}{k}$ satisfying $|\hf| =\frac{1}{2}\binom{n}{k}=\binom{n-1}{k-1}$. In particular, for $n=6$ there is an intersecting 3-graph $\hg_0\subset\binom{[6]}{3}$ with $|\hg_0|=10$ and $\hg_0$ regular with degree 5. Based on $\hg_0$, Huang \cite{huang} proved that Theorem \ref{thm-fw-2022} does not hold in the range $2k\leq n<(2+\sqrt{3})k$. Set $\hf_0=\{F\in \binom{[n]}{k}\colon \mbox{ there exists }G\in \hg_0,\ G\subset F\}$. \begin{thm}[\cite{huang}]\label{huang} For $k$ sufficiently large and $3k<n<(2+\sqrt{3})k$, \[ \gamma(\hf_0) > \binom{n-3}{k-2}. \] \end{thm} For $\hh\subset \binom{[n]}{k}$, a set $T\subset [n]$ is called a {\it transversal} of $\hh$ if $T\cap H\neq \emptyset$ for all $H\in \hh$. The {\it transversal number} $\tau(\hh)$ of $\hh$ is defined as the minimum size of a transversal of $\hh$. \begin{prop}\label{prop-1} Suppose that $\hh$ is a regular, intersecting 3-graph on 6 vertices and $|\hh|=10$. Then $\tau(\hh)=3$. \end{prop} \begin{proof} By assumption $\hh$ is regular of degree $(10\times 3)/6=5$. On the other hand, if $P$ is a transversal of $\hh$ of size 2, then all four of its supersets are in $\hh$. The remaining $10-4=6$ edges of $\hh$ all contain (exactly) one of the vertices of $P$. Hence at least one of them has degree at least 7, a contradiction. \end{proof} We need the following well-known fact. \begin{fact} Suppose that $\alpha \in (0,1)$ is fixed. Then for fixed $t,\ell$, let $k,n\rightarrow \infty$ with $k/n\rightarrow \alpha$, \begin{align}\label{ineq-2.5} \binom{n-t}{k-\ell}/\binom{n}{k} \rightarrow \alpha^\ell(1-\alpha)^{t-\ell}. \end{align} \end{fact} Let us show that for essentially the same range, $\beta(\hf_0)>\binom{n-4}{k-3}$ holds as well. \begin{prop} For $k$ sufficiently large and $2k<n<(2+\sqrt{3})k$, \[ \beta(\hf_0)>\binom{n-4}{k-3}. \] \end{prop} \begin{proof} Note that $\binom{n-4}{k-3} \rightarrow \alpha^3(1-\alpha) \binom{n}{k}$. For $6<i\neq j\leq n$, \begin{align*} |\hf_0(\bar{i},j)| &= 10\binom{n-8}{k-4}+16\binom{n-8}{k-5}+6\binom{n-8}{k-6}+\binom{n-8}{k-7}\\[3pt] &\rightarrow \left(10\alpha^4(1-\alpha)^4+16\alpha^5(1-\alpha)^3+6\alpha^6(1-\alpha)^2+\alpha^7(1-\alpha)\right) \binom{n}{k}\\[3pt] &>\alpha^3(1-\alpha) \binom{n}{k} \mbox{ for } \alpha\in (0.12,0.8). \end{align*} For $1\leq i\leq 6$ and $6<j\leq n$, \begin{align*} |\hf_0(\bar{i},j)| &= 5\binom{n-7}{k-4}+5\binom{n-7}{k-5}+\binom{n-7}{k-6}\\[3pt] &\rightarrow \left(5\alpha^4(1-\alpha)^3+5\alpha^5(1-\alpha)^2+\alpha^6(1-\alpha)\right) \binom{n}{k}\\[3pt] &>\alpha^3(1-\alpha) \binom{n}{k} \mbox{ for } \alpha\in \left(2-\sqrt{3},1\right). \end{align*} For $1\leq j\leq 6$ and $6<i\leq n$, \begin{align*} |\hf_0(\bar{i},j)|&= 5\binom{n-7}{k-3}+10\binom{n-7}{k-4}+5\binom{n-7}{k-5}+\binom{n-7}{k-6}\\[3pt] &\rightarrow \left(5\alpha^3(1-\alpha)^4+10\alpha^4(1-\alpha)^3+5\alpha^5(1-\alpha)^2+\alpha^6(1-\alpha)\right) \binom{n}{k}\\[3pt] &>\alpha^3(1-\alpha) \binom{n}{k} \mbox{ for } \alpha\in (0,1). \end{align*} For $1\leq i\neq j\leq 6$, \begin{align*} |\hf_0(\bar{i},j)|&= 3\binom{n-6}{k-3}+ 4\binom{n-6}{k-4}+ \binom{n-6}{k-5}\\[3pt] &\rightarrow \left(3\alpha^3(1-\alpha)^3+4\alpha^4(1-\alpha)^2+\alpha^5(1-\alpha)\right) \binom{n}{k}\\[3pt] &>\alpha^3(1-\alpha) \binom{n}{k} \mbox{ for } \alpha\in (0,1). \end{align*} Thus $\beta(\hf_0)> \binom{n-4}{k-3}$ for $2k<n<(2+\sqrt{3})k$ and $k$ sufficiently large. \end{proof} \section{Proof of Theorem \ref{thm-main-1}} In this section, we determine the maximum of the sturdiness of a $t$-intersecting family. A $t$-intersecting family $\hf\subset \binom{[n]}{k}$ is called {\it saturated} if $\hf$ ceases to be $t$-intersecting by the addition of any further $k$-sets. By \eqref{ineq-3} we may always assume that $\hf$ is saturated. For $\hf\subset \binom{[n]}{k}$, the {\it minimum degree} $\delta(\hf)$ is defined as the minimum of $|\hf(i)|$ over all $i\in [n]$. Let us recall a fundamental result of Huang and Zhao. \begin{thm}[\cite{HZ}]\label{thm-hz} For $n>2k$, if $\ha,\hb\subset \binom{[n]}{k}$ are cross-intersecting, then \[ \delta(\ha)\delta(\hb) \leq \binom{n-2}{k-2}^2. \] \end{thm} Let us use it to estimate sturdiness. For $\hf\subset \binom{[n]}{k}$, a set $T\subset [n]$ is called a {\it $t$-transversal} of $\hf$ if $|T\cap F|\geq t$ for all $F\in \hf$. Define the {\it $t$-transversal number} $\tau_t(\hf)$ as the minimum size of a $t$-transversal of $\hf$. \begin{thm}\label{thm-4.2} Let $\hf\subset \binom{[n]}{k}$ be a $t$-intersecting family. If $\tau_t(\hf)=t+1$ and $n\geq 2k-t+2$, then \[ \beta(\hf) \leq \binom{n-t-3}{k-t-2}. \] \end{thm} \begin{proof} Fix a $t$-transversal $S=\{y_1,y_2,\ldots,y_{t+1}\}$ of size $t+1$. Let \[ \hg_i=\hf(S\setminus \{y_i\},S) \subset \binom{[n]\setminus S}{k-t}, \ i=1,2,\ldots,t+1. \] Since $\hf$ is $t$-intersecting, $\hg_1,\hg_2,\ldots,\hg_{t+1}$ are pairwise cross-intersecting. By saturatedness, all supersets of $S$ are in $\hf$. Thus, \[ \hf=\hg_1\cup \hg_2\cup\ldots\cup \hg_{t+1}\cup \{F\in \hf\colon S\subset F\}. \] By Theorem \ref{thm-hz}, $\delta(\hg_1)\delta(\hg_2) \leq \binom{n-t-3}{k-t-2}^2$. By symmetry assume $\delta(\hg_1) \leq \binom{n-t-3}{k-t-2}$. That is, for some $x\in [n]\setminus S$, $|\hg_1(x)|\leq \binom{n-t-3}{k-t-2}$. Then \[ \beta(\hf) \leq |\hf(x,\overline{y_1})|=|\hg_1(x)| =\delta(\hg_1) \leq \binom{n-t-3}{k-t-2}. \] \end{proof} Recall the definition of the {\it Frankl family}: \[ \ha_i:=\ha_i(n,k,t)=\left\{F\in \binom{[n]}{k}\colon |F\cap [t+2i]|\geq t+i\right\} \mbox{ for }i=1,2,\ldots, k-t. \] One can check that $\beta(\ha_1)=\binom{n-t-3}{k-t-2}$. Moreover, \[ \beta(\ha_2)=(t+3)\binom{n-t-5}{k-t-3}+\binom{n-t-5}{k-t-4}=\left((t+3) \frac{n-k-1}{k-t-3}+1\right)\binom{n-t-5}{k-t-4}. \] Let $n-t-4= c(k-t-3)$. Then for $1\leq c<t+2$, \begin{align*} \frac{\beta(\ha_2)}{\beta(\ha_1)}&=\left((t+3) \frac{n-k-1}{k-t-3}+1\right) \frac{(k-t-2)(k-t-3)}{(n-t-3)(n-t-4)}\\[3pt] &=\left((t+3) \frac{n-t-4-(k-t-3)}{k-t-3}+1\right) \frac{(k-t-3)^2}{(n-t-4)^2}\\[3pt] &=\left((t+3)(c-1)+1\right) \frac{1}{c^2}\\[3pt] &= \frac{t+3}{c}- \frac{t+2}{c^2}>1. \end{align*} Thus for $n< (t+2)(k-t-2)+2$, $\beta(\hf)\leq \binom{n-t-3}{k-t-2}$ does not necessarily hold for a $t$-intersecting family $\hf\subset \binom{[n]}{k}$. Let \[ \tilde{\ha}_1:=\tilde{\ha}_1(n,k,t)=\left\{F\in \binom{[n]}{k}\colon |F\cap [t+2]|= t+1\right\}. \] It is easy to see that $\beta(\ha)=\beta(\ha_1)$ for all $\tilde{\ha}_1\subset \ha\subset \ha_1$. By Corollary \ref{cor-key} we infer that $\beta(\ha_1)$ is maximal among shifted $t$-intersecting families for $n\geq (t+2)(k-t)$. For $n$ sufficiently large the maximal diversity is known. \begin{thm}[\cite{F17}] Suppose that $\hf\subset \binom{[n]}{k}$ is a $t$-intersecting family, $k>t>0$ and $n\geq 2(t+3)^2k^2$.Then \[ \gamma(\hf) \leq \binom{n-t-2}{k-t-1}. \] \end{thm} We need the notion of basis for saturated $t$-intersecting families. Let $\hht_t(\hf) $ be the family of all $t$-transversals of $\hf$ of sizes at most $k$. Define the {\it basis} $\hb=\hb(\hf)$ of $\hf$ as the collection of containment minimal members in $\hht_t(\hf)$. The next lemma is easy to prove. \begin{lem}[\cite{FW2022-0}]\label{lem4-1} Suppose that $\hf\subset \binom{[n]}{k}$ is a saturated $t$-intersecting family, $n\geq 2k$. Then (i) and (ii) hold. \begin{itemize} \item[(i)] $\hb$ is a $t$-intersecting antichain, \item[(ii)] $\hf=\left\{H\in \binom{[n]}{k}\colon \exists B\in \hb, B\subset H\right\}$. \end{itemize} \end{lem} For any $\ell$ with $\tau_t(\hf)\leq \ell \leq k$, let \[ \hb^{(\ell)} = \left\{B\in \hb\colon |B|=\ell\right\},\ \hb^{(\leq \ell)} = \bigcup_{i=\tau_t(\hf)}^\ell\hb^{(i)} \mbox{ and }\hb^{(\geq \ell)} = \bigcup_{i=\ell}^k\hb^{(i)} \] \begin{lem}[\cite{FW2022-0}]\label{lem4-2} Suppose that $\hf\subset \binom{[n]}{k}$ is a saturated $t$-intersecting family with $\tau_t(\hf)\geq t+1$ and $\hb=\hb_t(\hf)$. Let $r$ be the smallest integer such that $\tau_t(\hb^{(\leq r)})\geq t+1$. Then \begin{align}\label{ineq-thb-1} \sum_{r\leq \ell \leq k} \left(\binom{\ell}{t}\ell k^{\ell-t-1}\right)^{-1}|\hb^{(\ell)}|\leq 1. \end{align} \end{lem} \begin{lem}\label{lem4-3} Let $\hf\subset \binom{[n]}{k}$ be a saturated $t$-intersecting family with $\tau_t(\hf)= t+2$ and $\hb=\hb_t(\hf)$. Let $y\in B_0\in \hb^{(t+2)}$. If $\tau_t(\hb^{(t+2)})\geq t+1$, then \[ \left|\hb^{(t+2)}(\bar{y})\right| \leq 4(t+1)(k-t+2). \] \end{lem} \begin{proof} We prove the lemma by a branching process. During the proof {\it a sequence} $S=(x_1,x_2,\ldots,x_\ell)$ is an ordered sequence of distinct elements of $X$ and we use $\widehat{S}$ to denote the underlying unordered set $\{x_1,x_2,\ldots,x_\ell\}$. In the first stage, for each of the $t+1$ $t$-subsets $\{x_1,x_2,\ldots,x_t\}\subset B_0\setminus \{y\}$, define a sequence $(x_1,x_2,\ldots,x_t)$. In the second stage, for each sequence $S_t=(x_1,\ldots,x_t)$ of length $t$, by $\tau_t(\hb^{(t+2)})\geq t+1$ there exists $B_1\in \hb^{(t+2)}$ such that $|\widehat{S_t}\cap B_1|<t$. Then replace $S_t$ by the $|B_1\setminus \widehat{S_t}|$ sequences $(x_1,\ldots,x_t,x_{t+1})$ with $x_{t+1}\in B_1\setminus \widehat{S}_t$. In the third stage, for each sequence $S_{t+1}=(x_1,\ldots,x_t,x_{t+1})$ of length $t+1$, by $\tau_t(\hf)\geq t+2$ there exists $F\in \hf$ such that $|\widehat{S_{t+1}}\cap F|<t$. Then replace $S_{t+1}$ by the $|F\setminus \widehat{S_{t+1}}|$ sequences $(x_1,\ldots,x_t,x_{t+1},x_{t+2})$ with $x_{t+2}\in F\setminus \widehat{S_{t+1}}$. Note that there are $\binom{|B_0\setminus \{y\}|}{t}=\binom{t+1}{t}=t+1$ choices for $(x_1,x_2,\ldots,x_t)$. Since $|B_0\cap B_1|\geq t$ and $\widehat{S_t}\subset B_0$, we have $|\widehat{S_t}\cap B_1|\geq t-2$. Thus there are $|B_1\setminus \widehat{S_t}|\leq 4$ choices for $x_{t+1}$. Similarly $|B_0\cap F|\geq t$ and $\widehat{S_t}\subset B_0$ imply $|\widehat{S_t}\cap F|\geq t-2$. Then there are $|F\setminus \widehat{S_{t+1}}|\leq k-t+2$ choices for $x_{t+2}$. Thus after three stages there are at most $4(t+1)(k-t+2)$ sequences of length $t+2$. Let $\hs$ be the collection of all these sequences of length $t+2$. We are left to show that for any $B\in \hb^{(t+2)}(\bar{y})$, there is some $S\in \hs$ such that $B=\widehat{S}$. Since $|B\cap (B_0\setminus \{y\})|\geq t$, there is some sequence $S_t=(x_1,x_2,\ldots,x_t)$ with $\widehat{S_t}\subset B$ in the first stage. In the second stage, since $|B\cap B_1|\geq t$ and $|\widehat{S_t}\cap B_1|<t$, there is some $x_{t+1}\in (B_1\cap B)\setminus \widehat{S_t}$. Then there is a sequence $S_{t+1}=(x_1,x_2,\ldots,x_t,x_{t+1})$ with $\widehat{S_{t+1}}\subset B$. At the third stage, since $|B\cap F|\geq t$ and $|\widehat{S_{t+1}}\cap F|<t$, there is some $x_{t+2}\in (F\cap B)\setminus \widehat{S_{t+1}}$. Thus there is a sequence $S_{t+2}=(x_1,x_2,\ldots,x_t,x_{t+1},x_{t+2})\in \hs$ with $\widehat{S_{t+2}}= B$. Therefore, \[ \left|\hb^{(t+2)}(\bar{y})\right|\leq |\hs| \leq 4(t+1)(k-t+2). \] \end{proof} Let $\binom{[n]}{\leq k}$ denote the collection of all subsets of $[n]$ of size at most $k$. For $\hg \subset \binom{[n]}{\leq k}$, let \[ \langle \hg \rangle =\left\{F\in \binom{[n]}{k}\colon \mbox{ there exists }G \in \hg \mbox{ such that }G\subset F\right\}. \] \begin{proof}[Proof of Theorem \ref{thm-main-1}] Let $\hf$ be a saturated $t$-intersecting family and let $\hb=\hb(\hf)$ be its basis. By Theorem \ref{thm-4.2} we may assume $\tau_t(\hf)\geq t+2$. Let $r$ be the smallest integer such that $\tau_t(\hb^{(\leq r)})\geq t+1$. \vspace{3pt} {\bf Case 1. } $r\geq t+3$. \vspace{3pt} Then there exists $T\in \binom{[n]}{t}$ such that $T$ is a $t$-transversal of $\hb^{(\leq r-1)}$. By \ref{lem4-1} (i), $\hb$ is $t$-intersecting. Thus $T\subset B$ for all $B\in \hb^{(\leq r-1)}$. Fix a $y\in T$. By \eqref{ineq-thb-1} and $r\geq t+3$, we have \begin{align*} |\hf(\bar{y})| \leq \sum_{r\leq \ell\leq k} |\hb^{(\ell)}|\binom{n-\ell}{k-\ell}\leq \max_{t+3\leq \ell\leq k} \binom{\ell}{t}\ell k^{\ell-t-1}\binom{n-\ell}{k-\ell}. \end{align*} Let $f(n,k,\ell,t)=\binom{\ell}{t}\ell k^{\ell-t-1}\binom{n-\ell}{k-\ell}$. Then $n\geq (t+3)^2k^2$ and $\ell\geq t+3$ imply \[ \frac{f(n,k,\ell+1,t)}{f(n,k,\ell,t)} = \frac{\ell+1}{\ell+1-t} \frac{\ell+1}{\ell} k\frac{k-\ell}{n-\ell} \leq \frac{(t+4)^2}{4(t+3)} \frac{k^2}{n}\leq 1. \] It follows that \begin{align}\label{ineq-4.3} |\hf(\bar{y})| \leq \binom{t+3}{t}(t+3) k^2\binom{n-t-3}{k-t-3}. \end{align} Note that \[ \sum_{x\in [n]\setminus\{y\}} |\hf(x,\bar{y})| =k |\hf(\bar{y})|. \] By \eqref{ineq-4.3} there exists $x\in [n]\setminus \{y\}$ such that \[ |\hf(x, \bar{y})| \leq \frac{k}{n-1}|\hf(\bar{y})| \leq \frac{k}{n-1}\binom{t+3}{t}(t+3) k^2\binom{n-t-3}{k-t-3}<\frac{(t+3)^4k^4}{6n(n-k)} \binom{n-t-3}{k-t-2}. \] For $n\geq (t+3)^2k^2$, we obtain that \[ \beta(\hf) \leq |\hf(x, \bar{y})| < \frac{1}{3}\binom{n-t-3}{k-t-2}. \] \vspace{3pt} {\bf Case 2. } $r=t+2$. \vspace{3pt} Let $S\in \hb^{(t+2)}$ and fix a $y\in S$. Since $\tau_t(\hb^{(t+2)})\geq t+1$ and $\tau_t(\hf)\geq t+2$, by Lemma \ref{lem4-3} we have \[ \left|\hb^{(t+2)}(\bar{y})\right| \leq 4(t+1)(k-t+2). \] Let $X=\cup \hb^{(t+2)}(\bar{y})$. Then \[ |X|\leq |\hb^{(t+2)}(\bar{y})|(t+2)\leq 4(t+1)(t+2)(k-t+2)<\frac{n}{2}-1. \] By $n\geq 8(t+1)k^2$ we infer that \begin{align}\label{ineq-4.6} |\hb^{(t+2)}(\bar{y})|\binom{n-t-3}{k-t-3} \leq 4(t+1)(k-t+2) \frac{k-t-2}{n-k} \binom{n-t-3}{k-t-2} < \frac{1}{2}\binom{n-t-3}{k-t-2}. \end{align} Let $\hf' =\langle \hb^{(\geq t+3)}\rangle$. By \eqref{ineq-thb-1} and $n\geq tk^2$ we have \begin{align*} |\hf'(\bar{y})| \leq \sum_{t+3\leq \ell\leq k} |\hb^{(\ell)}|\binom{n-\ell-1}{k-\ell}&\leq \max_{t+3\leq \ell\leq k} \binom{\ell}{t}\ell k^{\ell-t-1}\binom{n-\ell-1}{k-\ell} \\[3pt] &\leq \binom{t+3}{t}(t+3) k^2\binom{n-t-4}{k-t-3}\\[3pt] &\leq \frac{(t+3)^4 k^2}{6}\binom{n-t-4}{k-t-3}. \end{align*} Note that \[ \sum_{x\in [n]\setminus (X\cup \{y\})} |\hf'(x, \bar{y})| \leq \sum_{x\in [n]\setminus \{y\}} |\hf'(x, \bar{y})| = k |\hf'(\bar{y})|. \] Then there exists $x\in [n]\setminus (X\cup \{y\})$ such that for $n\geq 2(t+3)^2k^2$, \begin{align}\label{ineq-4.7} |\hf'(x, \bar{y})| \leq \frac{k}{n-|X|-1}|\hf'(\bar{y})| \leq \frac{2k}{n}|\hf'(\bar{y})| \leq \frac{(t+3)^4k^4}{3n^2}\binom{n-t-3}{k-t-2}<\frac{1}{2}\binom{n-t-3}{k-t-2}. \end{align} Adding \eqref{ineq-4.6} and \eqref{ineq-4.7}, we obtain that \begin{align*} \beta(\hf) \leq |\hf(x,\bar{y})|<\binom{n-t-3}{k-t-2}. \end{align*} \end{proof} Recall that $\hf\subset \binom{[n]}{k}$ is called $r$-wise $t$-intersecting if $|F_1\cap F_2\cap \ldots\cap F_r|\geq t$ for all $F_1,F_2,\ldots,F_r\in \hf$. If an $r$-wise $t$-intersecting family $\hf$ is not a star, then $\hf$ is $(t+r-2)$-intersecting. Thus we have the following corollary. \begin{cor} Let $\hf\subset \binom{[n]}{k}$ be an $r$-wise $t$-intersecting family. If $n\geq 2(t+r+1)^2k^2$ then \begin{align}\label{ineq-4.8} \beta(\hf) \leq \binom{n-t-r-1}{k-t-r}. \end{align} \end{cor} Let us note that $\ha_1$ shows that \eqref{ineq-4.8} is best possible. \section{Proof of Theorems \ref{thm-2} and \ref{thm-main-3}} \begin{fact} Let $\hf\subset 2^{[n]}$. Then \begin{align} \beta(\hf) \leq \frac{n}{4(n-1)}|\hf|. \end{align} \end{fact} \begin{proof} For every subset $F\in \hf$ there are $|F|(n-|F|)$ choices $x\in F$, $y\notin F$ such that $F$ contributes 1 to $\hf(x,\bar{y})$. As $a(n-a)\leq \frac{n^2}{4}$, \begin{align}\label{ineq-4.1} \sum_{x\in [n], y\in [n]\setminus \{x\}} |\hf(x,\bar{y})| \leq \frac{n^2}{4}|\hf|, \end{align} i.e., the average of $|\hf(x,\bar{y})|$ is at most $\frac{n^2|\hf|}{4n(n-1)}=\frac{n}{4(n-1)}|\hf|$. Note that equality holds iff $n$ is even and $\hf\subset \binom{[n]}{n/2}$. For $n=2\ell+1$ the same proof yields \begin{align*} \beta(\hf) \leq \frac{\ell(\ell+1)}{(2\ell+1)2\ell} |\hf| =\frac{\ell+1}{2(2\ell+1)} |\hf|. \end{align*} \end{proof} \begin{proof}[Proof of Theorem \ref{thm-2}] By \eqref{ineq-3}, we may assume $|\hf|=2^{n-1}$. Hence $\hf$ is a filter, that is, $F\subset G\subset [n]$ and $F\in \hf$ imply $G\in \hf$. Since for each complementary pair $(H,[n]\setminus H)$ exactly one of them is in $\hf$ we can compute \eqref{ineq-4.1} explicitly \[ \sum_{F\in \hf} |F|(n-|F|) =\frac{1}{2} \sum_{F\in 2^{[n]}} |F|(n-|F|) =\frac{1}{2} \sum_{x\in [n],y\in [n]\setminus \{x\}} |2^{[n]}(x,\bar{y})|=\frac{1}{2} n(n-1)2^{n-2}. \] Thus the average is exactly $2^{n-3}$. \end{proof} For $A,B\subset [n]$, define the symmetric difference $A\Delta B$ as $(A\setminus B)\cup (B\setminus A)$ and define the distance between $A$ and $B$ to be \[ d(A,B)=|A\Delta B|. \] The Hamming ball of center $C\subset [n]$ and radius $r$ is \[ \hb_r(C) =\left\{B\subset [n] \colon d(B,C) \leq r\right\}. \] A family $\ha\subset 2^{[n]}$ is called a Hamming ball of center $C\subset [n]$ and radius $r$ iff \[ \hb_r(C)\subseteq \ha \subseteq \hb_{r+1}(C). \] For $\ha,\hb\subset 2^{[n]}$, define \[ d(\ha,\hb) =\min\left\{d(A,B)\colon A\in \ha,\ B\in \hb\right\}. \] For $\ha\subset 2^{[n]}$, the $d$-neighbourhood of $\ha$ is defined as \[ \Gamma_d \ha =\left\{F\subset [n]\colon d(F, \ha) \leq d\right\}. \] Let us recall Harper's theorem \cite{harper}, see \cite{FF81} for a short proof. \begin{thm}[Harper's Theorem \cite{harper}] Let $\ha\subset 2^{[n]}$ be a non-empty family. Then there exists a Hamming ball $\ha_0$ such that $|\ha_0|=|\ha|$ and $|\Gamma_d \ha |\geq | \Gamma_d \ha_0|$. \end{thm} Using Harper's Theorem, Ahlswede and Katona \cite{AK} proved the following. \begin{thm}[\cite{AK}]\label{thm-3.4} Let $1\leq N\leq 2^n$ and let $\ha,\hb\subset 2^{[n]}$ be two families satisfying $|\ha|=N$, $|A\Delta B|\leq w$ for all $A\in \ha$, $B\in \hb$. Let $\ha_0$ be a Hamming ball of center $\emptyset$ with $|\ha_0|=N$. Then \[ |\hb| \leq \left|\left\{B\subset [n]\colon |B\Delta A| \leq w \mbox{ for all }A\in \ha_0 \right\}\right|. \] \end{thm} One can derive the following theorem from Theorem \ref{thm-3.4}. \begin{thm}[\cite{AK}]\label{thm-3.5} Let $\ha,\hb\subset 2^{[n]}$ be families satisfying $|A\Delta B|\leq w$ for all $A\in \ha$, $B\in \hb$. \begin{itemize} \item[(i)] If $w=2s$ then \[ \min\{|\ha|,|\hb|\} \leq \sum\limits_{0\leq j\leq s} \binom{n}{j}. \] \item[(ii)]If $w=2s+1$ then \[ \min\{|\ha|,|\hb|\} \leq \binom{n-1}{s}+\sum\limits_{0\leq j\leq s} \binom{n}{j}. \] \end{itemize} \end{thm} Note that the special case $\ha=\hb$ of Theorem \ref{thm-3.5} is the classical Kleitman's Diameter Theorem (\cite{kleitman}). By applying Theorem \ref{thm-3.5} we obtain following result. \begin{thm}\label{thm-main2} Suppose that $\hf\subset 2^{[n]}$ is a family with $|F\Delta F'|\leq w$ for all $F,F'\in \hf$. \begin{itemize} \item[(i)] If $w=2s$ then $\beta(\hf) \leq \sum\limits_{ 0\leq j\leq s-1} \binom{n-2}{j}$; \item[(ii)]If $w=2s+1$ then $\beta(\hf) \leq \sum\limits_{ 0\leq j\leq s-1} \binom{n-2}{j}+\binom{n-3}{s-1}$. \end{itemize} \end{thm} \begin{proof} Let $\ha =\hf(1,\bar{2})$ and $\hb=\hf(\bar{1},2)$. Then $|A\Delta B|\leq w-2$ for all $A\in \ha$, $B\in \hb$. Then by Theorem \ref{thm-3.5} we infer that for $w=2s$, \[ \beta(\hf) \leq \min\{|\ha|,|\hb|\}\leq \sum\limits_{ 0\leq j\leq s-1} \binom{n-2}{j}. \] For $w=2s-1$, \[ \beta(\hf) \leq \min\{|\ha|,|\hb|\}\leq\sum\limits_{ 0\leq j\leq s-1} \binom{n-2}{j}+ \binom{n-3}{s-1}. \] \end{proof} The Hamming ball $\hb_s(\emptyset)$ shows that the bound on $\beta(\hf)$ in Theorem \ref{thm-main2} (i) is best possible. Let $\hf=\hb_s(\emptyset)\cup \hht(n,s+1)$. Then $|F\Delta F'|\leq 2s+1$ for all $F,F'\in \hf$ and \[ \beta(\hf) = \sum\limits_{0\leq j\leq s-1} \binom{n-2}{j}+\binom{n-4}{s-2}. \] \begin{conj} Suppose that $\hf\subset 2^{[n]}$ is a family with $|F\Delta F'|\leq 2s+1$ for all $F,F'\in \hf$. Then for $n\geq 4(s+1)$, \[ \beta(\hf) \leq \sum\limits_{0\leq j\leq s-1} \binom{n-2}{j}+\binom{n-4}{s-2}. \] \end{conj} A family $\hg\subset 2^{[n]}$ is called a $u$-union family if $|G\cup G'|\leq u$ for $G,G'\in \hg$. It is easy to see that $\hg$ is $u$-union iff $\hg^c$ is $(n-u)$-intersecting. Thus the classical Katona Theorem (Theorem \ref{thm-katona}) determines the maximum size of a $u$-union family as well. Let us give a relatively general example of a $u$-union family for $u=2s+1$, $s\geq 1$. \begin{example}\label{example-1} Fix an intersecting family $\hg\subset \binom{[n]}{s+1}$ and define $\hf=\hg\cup \{F\subset [n]\colon |F|\leq s\}$. For $n\geq 2s+2$ the Erd\H{o}s-Ko-Rado Theorem (Theorem \ref{thm-ekr}) implies \begin{align} |\hf| \leq \binom{n-1}{s} +\sum_{0\leq i\leq s}\binom{n}{i} =2\sum_{0\leq i\leq s} \binom{n-1}{i}, \end{align} in line with the Katona Theorem (Theorem \ref{thm-katona}). On the other hand, \[ \gamma(\hf) =\gamma(\hg) +\sum_{0\leq i\leq s}\binom{n-1}{i} \mbox{ and } \beta(\hf) =\beta(\hg) +\sum_{0\leq i\leq s-1}\binom{n-2}{i}. \] \end{example} Thus, \begin{align}\label{ineq-5.7} \beta(\hf) \leq \binom{n-4}{s-2}+\sum_{0\leq i\leq s-1}\binom{n-2}{i} \mbox{ fails unless } \beta(\hg)\leq \binom{n-4}{s-2}. \end{align} By Theorem \ref{thm-main}, \eqref{ineq-5.7} holds for Example \ref{example-1} if $n\geq 36(s+7)$ or more generally if the $(2s+1)$-union family contains no members of size exceeding $s+1$. For the proof of Theorem \ref{thm-u-uninon}, we need the following two results of the first author. \begin{thm}[\cite{F16}]\label{thm-frankl1} Let $n,k,t$ be non-negative integers with $n\geq 2k+t$. Suppose that $\hf\subset \binom{[n]}{k+t}$, $\hg\subset \binom{[n]}{k}$ are cross-intersecting. If $\hf$ is $(t+1)$-intersecting and non-empty then \[ |\hf|+|\hg| \leq 1+\binom{n}{k} -\binom{n-k-t}{k}. \] \end{thm} \begin{thm}[\cite{F78}]\label{thm-frankl2} Let $\hf\subset \binom{[n]}{k}$ be a $t$-intersecting family with $n>2k-t$. Then \[ |\hf| \leq \binom{n}{k-t}. \] \end{thm} \begin{thm}\label{thm-u-uninon} Suppose that $\hf\subset 2^{[n]}$ is $u$-union. \begin{itemize} \item[(i)] If $u=2s$ then $\beta(\hf) \leq \sum\limits_{ 0\leq j\leq s-1} \binom{n-2}{j}$; \item[(ii)]If $u=2s+1$ and $n\geq \max\{4(s+2)^2, 36(s+7)\}$, then $\beta(\hf) \leq \sum\limits_{ 0\leq j\leq s-1} \binom{n-2}{j}+\binom{n-4}{s-2}$. \end{itemize} \end{thm} \begin{proof} Clearly (i) follows from Theorem \ref{thm-main2} (i). It suffices to prove (ii). Let $\hf\subset 2^{[n]}$ be a $(2s+1)$-union family with $\beta(\hf)$ maximal. If $\hf$ contains no members of size exceeding $s+1$, then by Theorem \ref{thm-main} and $n\geq 36(s+7)$ we have $\beta(\hf) \leq \sum\limits_{ 0\leq j\leq s-1} \binom{n-2}{j}+\binom{n-4}{s-2}$. Thus we may assume $\max\{|F|\colon F\in \hf\}>s+1$. By \eqref{ineq-3}, without loss of generality we may also assume that $\hf$ is a down set whence $\hf^{(s+2)}\neq \emptyset$. Note that the $(2s+1)$-union property implies that $\hf^{(s+2)}$, $\hf^{(s)}$ are cross-intersecting and $\hf^{(s+2)}$ is 3-intersecting. By Theorem \ref{thm-frankl1} and $n\geq 2s+2$ we obtain that \begin{align}\label{ineq-3.8} |\hf^{(s+2)}|+|\hf^{(s)}| \leq 1+\binom{n}{s}-\binom{n-s-2}{s} <2(s+2)\binom{n-2}{s-1}. \end{align} For $3\leq \ell\leq s$, $\hf^{(s+\ell)}$ is $2\ell-1$-intersecting, by Theorem \ref{thm-frankl2} we have \[ |\hf^{(s+\ell)}| \leq \binom{n}{(s+\ell) -(2\ell-1)} = \binom{n}{s-\ell+1}. \] Thus, \[ \sum_{3\leq \ell\leq s} |\hf^{(s+\ell)}| \leq \binom{n}{s-2}+\binom{n}{s-3} +\ldots +\binom{n}{0}. \] Since $n\geq 3s$ implies \[ \frac{\binom{n}{s-i}}{\binom{n}{s-i-1}} = \frac{n-s+i+1}{s-i} \geq 2, \] by $n\geq 12s$ we have \begin{align}\label{ineq-3.9} \sum_{3\leq \ell\leq s} |\hf^{(s+\ell)}| \leq 2\binom{n}{s-2} \leq \left(\frac{n}{n-s}\right)^2\frac{2s}{n-s} \binom{n-2}{s-1}< \frac{1}{4}\binom{n-2}{s-1}. \end{align} Set $\hh = \hf^{(s)}\cup \hf^{(s+1)}\cup \hf^{(s+2)}$. We choose distinct vertices $x$ and $y$ uniformly at random. Let $X_i=|\hf^{(s+i)}(x,\bar{y})|$ for $i=0,1,2$. Then \begin{align*} \ex (X_i)= |\hf^{(s+i)}| \frac{(s+i)(n-s-i)}{n(n-1)}&<|\hf^{(s+i)}| \frac{(s+2)(n-s-2)}{n(n-1)}\\[3pt] &<|\hf^{(s+i)}| \frac{s+2}{n}. \end{align*} By \eqref{ineq-3.8} and $n\geq 4(s+2)^2$ we have \[ \ex (X_0+X_2) < \frac{s+2}{n}\left(|\hf^{(s)}|+|\hf^{(s+2)}|\right) \leq \frac{1}{2} \binom{n-2}{s-1}. \] Note that $\hf^{(s+1)}$ is intersecting and $\hf^{(s)},\hf^{(s+2)}$ are cross 2-intersecting. If $\hf^{(s+1)}$ is non-trivial intersecting, then by Hilton-Milner Theorem we have \[ |\hf^{(s+1)}| \leq \binom{n-1}{s} -\binom{n-s-2}{s}+1<(s+1)\binom{n-2}{s-1}. \] If $\hf^{(s+1)}$ is a star with center $z$, then $\hf^{(s+1)}(z), \hf^{(s+2)}$ is cross-intersecting. Since $\hf^{(s+2)}$ is non-empty, we infer that \[ |\hf^{(s+1)}| \leq \binom{n-1}{s} -\binom{n-1-(s+2)}{s-1}<(s+2)\binom{n-2}{s-1}. \] In both cases, we have $|\hf^{(s+1)}|<(s+2)\binom{n-2}{s-1}$. By $n\geq 4(s+2)^2$ It follows that \[ \ex X_1 \leq \frac{s+2}{n}|\hf^{(s+1)}| \leq \frac{1}{4} \binom{n-2}{s-1}. \] Thus $\ex X< \frac{3}{4} \binom{n-2}{s-1}$. This implies that there exist $x,y\in [n]$ such that \begin{align}\label{ineq-3.10} \hh(x,\bar{y})< \frac{3}{4} \binom{n-2}{s-1}. \end{align} Adding \eqref{ineq-3.9}, \eqref{ineq-3.10} and $|\hf^{(i)}(x,\bar{y})|\leq \binom{n-2}{i-1}$ for $i=1,2,\ldots,s-1$, we conclude that \[ \beta(\hf) \leq |\hf(x,\bar{y})|\leq \sum_{0\leq i\leq s-1}\binom{n-2}{i}< \binom{n-4}{s-2}+\sum_{0\leq i\leq s-1}\binom{n-2}{i}. \] \end{proof} Recall that if $\hf$ is $t$-intersecting then $\hf^c$ is $(n-t)$-union. By \eqref{ineq-1.1}, Theorem \ref{thm-main-3} follows from Theorem \ref{thm-u-uninon}. \section{Concluding Remarks} In the present paper, we mainly considered the maximum sturdiness of $k$-uniform intersecting families, $k$-uniform $t$-intersecting families and non-uniform $t$-intersecting families. Let $\hf\subset 2^{[n]}$ be a $(2s+1)$-union family. In Theorem \ref{thm-u-uninon} (ii), we determine the maximum sturdiness of $\hf$ for $n\geq \max\{4(s+2)^2, 36(s+7)\}$. By Example \ref{example-1} and Theorem \ref{huang}, the same does not hold for the range $2(s+1)<n<(2+\sqrt{3})(s+1)$. \begin{conj} Suppose that $\hf\subset 2^{[n]}$ is $(2s+1)$-union. Then for $n\geq 4(s+1)$, \[ \beta(\hf) \leq \sum_{ 0\leq j\leq s-1} \binom{n-2}{j}+\binom{n-4}{s-2}. \] \end{conj} \begin{conj} Suppose that $\hf\subset 2^{[n]}$ is a family with $|F\Delta F'|\leq 2s+1$ for all $F,F'\in \hf$. Then for $n\geq 4(s+1)$, \[ \beta(\hf) \leq \sum_{ 0\leq j\leq s-1} \binom{n-2}{j}+\binom{n-4}{s-2}. \] \end{conj} A related problem is to consider the maximum sturdiness of an IU-family. A family $\hg\subset 2^{[n]}$ is called an IU-family if $\hg$ and $\hg^c$ are both intersecting. Equivalently, an IU-family $\hg\subset 2^{[n]}$ is both intersecting and 1-union. \begin{conj} If $\hg\subset 2^{[n]}$ is an IU-family, then \begin{align}\label{ineq-4.2} \beta(\hg)\leq 2^{n-4}. \end{align} \end{conj} For $\hg\subset 2^{[n]}$ and $X\subset [n]$, let $\hg_{\mid X}$ denote the family $\{G\cap X\colon G\in \hg\}$. \begin{fact} Let $\hg\subset 2^{[n]}$ be an IU-family. If there exists partition $[n]=X\cup Y$ such that $\hg_{\mid X}$ is intersecting, $\hg_{\mid Y}$ is union, then $\beta(\hg)\leq 2^{n-4}$. \end{fact} \begin{proof} By \eqref{ineq-3} we may assume first that all $H\subset [n]$ with $H\cap X$ containing a member of $\hg_{\mid X}$ and $H\cap Y$ contained in a member of $\hg_{\mid Y}$ are in $\hg$. Then $\hg_{\mid X}$ is an upset (filter) and $\hg_{\mid Y}$ is a down-set (complex). Hence for all $x\in X$, $y\in Y$, \[ |\hg(\bar{x},y)| \leq \left(\frac{1}{2} 2^{|X|-1}\right)\left(\frac{1}{2} 2^{|Y|-1}\right) = \frac{1}{16} 2^n =2^{n-4}, \mbox{ proving }\eqref{ineq-4.2}. \] \end{proof} \begin{thebibliography}{10} \bibitem{AK} R. Ahlswede, G.O.H. Katona, Contributions to the geometry of hamming spaces, Discrete Mathematics 17 (1977), 1--22. \bibitem{ekr} P. Erd\H{o}s, C. Ko, R. Rado, Intersection theorems for systems of finite sets, Quart. J. Math. Oxford Ser. 12 (1961), 313--320. \bibitem{F78} P. Frankl, The Erd\H{o}s-Ko-Rado theorem is true for $n = ckt$, Coll. Math. Soc. J. Bolyai 18 (1978), 365--375. \bibitem{F78-2} P. Frankl, On intersecting families of finite sets, Journal of Combinatorial Theory, Series A. 24 (1978), 146--161. \bibitem{F87} P. Frankl, The shifting technique in extremal set theory, Surveys in Combinatorics 123 (1987), 81--110. \bibitem{F16} P. Frankl, New inequalities for cross-intersecting families, Moscow Journal of Combinatorics and Number Theory 6 (2016), 27--32. \bibitem{F17} P. Frankl, Antichains of fixed diameter, Moscow Journal of Combinatorics and Number Theory 7 (2017), 3--33. \bibitem{F2022} P. Frankl, Pseudo sunflowers, Eur. J. Combin. 104 (2022), 103553. \bibitem{FF81} P. Frankl, Z. F\H{u}redi, A short proof for a theorem of Harper about Hamming-spheres, Discrete Mathematics 34(3) (1981) 311--313. \bibitem{FW2022-0} P. Frankl, J. Wang, Intersections and distinct intersections in cross-intersecting families, Eur. J. Comb. 110 (2023), 103665. \bibitem{FW2022} P. Frankl, J. Wang, Improved bounds on the maximum diversity of intersecting families, Eur. J. Comb. 118 (2024), 103885. \bibitem{FW2024} P. Frankl, J. Wang, On the $C$-diversity of intersecting hypergraphs, arXiv:2308.14028. \bibitem{Fu80} Z. F\"{u}redi, On maximal intersecting families of finite sets, J. Combin. Theory Ser. A 28 (1980) 282--289. \bibitem{harper} K.H. Harper, Optimal numberings and insperimetric problems on graphs, J. CombinatoriaI Theory 1 (1966) 385--393. \bibitem{huang} H. Huang, Two extremal problems on intersecting families, Eur. J. Comb. 76 (2019), 1--9. \bibitem{HZ} H. Huang and Y. Zhao, Degree versions of the Erd\H{o}s-Ko-Rado Theorem and Erd\H{o}s hypergraph matching conjecture, Journal of Combinatorial Theory, Ser. A, 150 (2017), 233--247. \bibitem{Katona} G.O.H. Katona, Intersection theorems for systems of finite sets, Acta Math. Acad. Sci. Hung. 15 (1964), 329--337. \bibitem{kleitman} D. J. Kleitman, On a combinatorial conjecture of Erd\H{o}s, J. Combinatorial Theory, 1:209--214, 1966. \bibitem{Ku1} A. Kupavskii, Diversity of uniform intersecting families, Eur. J. Comb. 74 (2018), 39--47. \bibitem{W84} R. M. Wilson, The exact bound in the Erd\H{o}s-Ko-Rado theorem, Combinatorica 4 (1984), 247--257. \end{thebibliography} \end{document}
2412.07164v1
http://arxiv.org/abs/2412.07164v1
Order Polytopes of Dimension $\leq 13$ are Ehrhart Positive
\documentclass[reqno, 12pt]{article} \pdfoutput=1 \def\comment#1{{\bf Comment:}\textsf{\sl#1}} \newcommand\commentL[2]{\textcolor{red}{#1},(\textcolor{blue}{You suggest #2})} \usepackage{enumerate} \usepackage{latexsym} \usepackage[centertags]{amsmath} \usepackage{amsfonts} \usepackage{amssymb} \usepackage{amsthm} \usepackage{newlfont} \usepackage{graphics} \usepackage{color} \usepackage{float} \usepackage{diagbox} \textwidth 480pt \hoffset -60pt \textheight 9in \voffset -30pt \parindent 8mm \parskip 2mm \usepackage{hyperref} \usepackage{longtable} \usepackage{rotating} \usepackage{multirow} \usepackage{extarrows} \usepackage[sort,compress,numbers]{natbib} \usepackage[utf8]{inputenc} \newtheorem{thm}{Theorem}[section] \newtheorem{cor}[thm]{Corollary} \newtheorem{lem}[thm]{Lemma} \newtheorem{prop}[thm]{Proposition} \newtheorem{fig}{Figure} \newtheorem{cnj}[thm]{Conjecture} \theoremstyle{mydefinition} \newtheorem{dfn}[thm]{Definition} \theoremstyle{myremark} \newtheorem{rem}[thm]{Remark} \newtheorem{exa}[thm]{Example} \newtheorem{prob}[thm]{Open Problem} \allowdisplaybreaks[4] \def\n{\mathfrak{n}} \def\N{\mathbb{N}} \def\Z{\mathbb{Z}} \def\CT{\mathop{\mathrm{CT}}} \newcommand{\ZZ}{{\mathbb{Z}}} \newcommand{\A}{{\mathcal{A}}} \newcommand{\B}{{\mathcal{B}}} \newcommand{\C}{{\mathcal{C}}} \newcommand{\G}{{\mathcal{G}}} \newcommand{\I}{{\mathcal{I}}} \renewcommand{\H}{{\mathcal{H}}} \newcommand{\M}{{\mathcal{M}}} \newcommand{\V}{{\mathcal{V}}} \newcommand{\U}{{\textsf{U}}} \newcommand{\DD}{{\textsf{D}}} \newcommand{\E}{{\textsf{E}}} \renewcommand{\L}{{\textsf{L}}} \renewcommand{\P}{{\mathbb{P}}} \newcommand{\F}{{\mathcal{F}}} \newcommand{\R}{{\mathbb{R}}} \newcommand{\T}{{\mathcal{T}}} \newcommand{\Q}{{\mathbb{Q}}} \def\Res{\mathop{\mathrm{Res}}} \newcommand\scalar[2]{\langle #1, \; #2 \rangle} \title{Order Polytopes of Dimension $\leq 13$ are Ehrhart Positive} \author{Feihu Liu$^{1}$, Guoce Xin$^{2,}$\thanks{This work is partially supported by the National Natural Science Foundation of China (No.12071311).},\ and Zihao Zhang$^{3}$ \\[2mm] {\small $^{1, 2, 3}$ School of Mathematical Sciences,}\\[-0.8ex] {\small Capital Normal University, Beijing, 100048, P.R.~China}\\ {\small $^1$ Email address: [email protected]}\\ {\small $^2$ Email address: guoce\[email protected]}\\ {\small $^3$ Email address: [email protected]} } \date{December 10, 2024} \begin{document} \maketitle \begin{abstract} The order polytopes arising from the finite poset were first introduced and studied by Stanley. For any positive integer $d\geq 14$, Liu and Tsuchiya proved that there exists a non-Ehrhart positive order polytope of dimension $d$. They also proved that any order polytope of dimension $d\leq 11$ is Ehrhart positive. We confirm that any order polytope of dimension $12$ or $13$ is Ehrhart positive. This solves an open problem proposed by Liu and Tsuchiya. Besides, we also verify that any $h^{*}$-polynomial of order polytope of dimension $d\leq 13$ is real-rooted. \end{abstract} \noindent \begin{small} \emph{Mathematic subject classification}: Primary 05A15; Secondary 06A07, 68R05. \end{small} \noindent \begin{small} \emph{Keywords}: Order polytope; Ehrhart polynomial; Ehrhart positive; Real-rooted polynomial. \end{small} \section{Introduction} A convex polytope is called \emph{integral} if any of its vertices has integer coordinates. Let $\mathcal{P}$ represent a $d$-dimensional integral convex polytope in $\mathbb{R}^{n}$. The function $$\mathrm{ehr}(\mathcal{P},t)=|t\mathcal{P}\cap \mathbb{Z}^n|,\ \ \ \ \ t=1,2,\ldots,$$ counts the integer points within $t\mathcal{P}$, where $t\mathcal{P}=\{t\alpha : \alpha \in \mathcal{P}\}$ denotes the $t$-th dilation of $\mathcal{P}$. Ehrhart \cite{Ehrhart62} proved that $\mathrm{ehr}(\mathcal{P},t)$ is a polynomial in $t$ of degree $d$. This polynomial is referred to as the \emph{Ehrhart polynomial} of $\mathcal{P}$. Moreover, the coefficient of $t^d$ in $\mathrm{ehr}(\mathcal{P},t)$ equals the (relative) volume of $\mathcal{P}$, the coefficient of $t^{d-1}$ is half of the boundary volume of $\mathcal{P}$, and the constant term is always $1$ (see \cite{BeckRobins}). The remaining coefficients are complicated to describe \cite{McMullen77}. An integral convex polytope $\mathcal{P}$ is said to be \emph{Ehrhart positive} (or have \emph{Ehrhart positivity}) if all the coefficients of $\mathrm{ehr}(\mathcal{P},t)$ are non-negative. In the case of dimension $3$, a well-known non-Ehrhart positive example is Reeve's tetrahedron \cite[Example 3.22]{BeckRobins}. For a comprehensive introduction to Ehrhart positivity, please refer to Liu's survey \cite{FuLiu19}. A recent development in Ehrhart positivity is presented in \cite{HibiHTY19}. The generating function $$\mathrm{Ehr}(\mathcal{P},x)=1+\sum_{t\geq 1}\mathrm{ehr}(\mathcal{P},t)x^t$$ is known as the \emph{Ehrhart series} of $\mathcal{P}$. It is of the form $$\mathrm{Ehr}(\mathcal{P},x)=\frac{h^{*}(x)}{(1-x)^{d+1}},$$ where $\dim\mathcal{P}=d$ and $h^{*}(x)$ is a polynomial in $x$ of degree $\deg h^{*}(x)\leq d$ (as detailed in \cite{BeckRobins}). The $h^{*}(x)$ is usually referred to as the \emph{$h^{*}$-polynomial} of $\mathcal{P}$. Stanley \cite{StanleyADM80} proved that the coefficients of the $h^{*}$-polynomial are nonnegative integers. Subsequently, plenty of work has been done on various properties of the $h^{*}$-polynomial of an integral polytope. Let $f(x)=a_mx^m+a_{m-1}x^{m-1}+\cdots+a_1x+a_0$ be a polynomial with nonnegative real coefficients. The polynomial $f(x)$ is said to be \emph{unimodal} if its coefficients satisfy $a_0\leq \cdots \leq a_{i-1}\leq a_i\geq a_{i+1} \geq \cdots \geq a_m$ for some $0\leq i\leq m$. It is called \emph{log-concave} if $a_i^2\geq a_{i-1}a_{i+1}$ for $1\leq i\leq m-1$. It is said to be \emph{real-rooted} if all roots of $f(x)$ are real numbers. If $f(x)$ is real-rooted, then it is log-concave; If $f(x)$ is log-concave with $a_i>0$ for all $i$, then it is unimodal. See \cite[Section 5]{RP.StanleyAC} or \cite{StanleyLog-concave}. This paper focuses on the families of order polytopes arising from finite partially ordered sets (or poset for short), which were initially introduced and studied by Stanley. See \cite{StanleyDCG} for detailed definitions. \begin{dfn}[Stanley \cite{StanleyDCG}]\label{Definitorderp} Given a poset $P$ on the set $[p]:=\{1,2,\ldots,p\}$, we associate a polytope $\mathcal{O}(P)$, called the \emph{order polytope} of $P$. It is defined to be the convex polytope consisting of those $(x_1,x_2,\ldots,x_p)\in \mathbb{R}^p$ such that \begin{align*} &x_i\leq x_j \ \ \ \text{if}\ \ \ i\prec_{P} j; \\& 0\leq x_i\leq 1 \ \ \ \text{for}\ \ \ 1\leq i\leq p. \end{align*} \end{dfn} The order polytope is an integral polytope of dimension $\dim \mathcal{O}(P)=p$ \cite{StanleyDCG}. Regarding the Ehrhart positivity of order polytopes, Stanley provided an example of a non-Ehrhart-positive order polytope of dimension $21$ in \cite{StanleyMathOverfolw} (or \cite{Alexandersson}). Subsequently, Liu and Tsuchiya \cite{LiuTsuchiya19} proved that for any positive integer $d\geq 14$, there exists a non-Ehrhart-positive order polytope of dimension $d$. They also confirmed that any order polytope of dimension $d\leq 11$ is Ehrhart positive. Naturally, they proposed the following open question. \begin{prob}[Liu--Tsuchiya, \cite{LiuTsuchiya19}]\label{Open-Ehrhart} For $d=12$ or $13$, does there exist an order polytope of dimension $d$ such that its Ehrhart polynomial has a negative coefficient? \end{prob} Our first contribution in this paper is to provide a negative solution to the above problem. That is, we confirm that any order polytope of dimension $d\leq 13$ is Ehrhart positive. Now the $h^{*}$-polynomial of order polytope $\mathcal{O}(P)$ is denoted by $h^{*}(x;P)$. For any graded poset $P$, Reiner and Welker \cite{Reiner05} proved that the $h^{*}(x;P)$ is unimodal. Br\"and\'en \cite{Branden05} showed that the $h^{*}(x;P)$ is not only unimodal but also symmetric. Recently, a comprehensive research development of $h^{*}$-polynomials was presented by Ferroni and Higashitani in \cite{Ferroni23}. They also raised the following open questions concerning order polytopes. \begin{prob}{\em \cite[Section 6]{Ferroni23} or \cite[Section 7]{Stembridge09}} For any graded poset $P$, prove that the $h^{*}$-polynomial $h^{*}(x;P)$ is real-rooted. \end{prob} There exists an order polytope whose $h^{*}$-polynomial is not real-rooted. Such an example was given by Stembridge in \cite{Stembridge09}. Similarly, Ferroni and Higashitani also raised the following open questions. \begin{prob}{\em \cite[Section 6]{Ferroni23} or \cite[Section 7]{Stembridge09}} For a general poset $P$, prove that the $h^{*}$-polynomial $h^{*}(x;P)$ is log-concave (or even unimodal). \end{prob} We verify that the $h^{*}$-polynomial of order polytopes $\mathcal{O}(P)$ of dimension $d\leq 13$ is real-rooted. Consequently, these $h^{*}$-polynomials are log-concave and unimodal. This paper is organized as follows. In Section 2, we will briefly introduce how to compute the Ehrhart polynomial by Stembridge's \texttt{posets} package \cite{Stempackage}. In Section 3, we first introduce how to generate posets. Then we study the Ehrhart positivity of order polytopes of dimensions $12$ and $13$. We also verify the real-rootedness of the $h^{*}(x;P)$ of $\#P\leq 13$. \section{Ehrhart Polynomial and $h^{*}$-polynomial} Let $P$ be a finite poset on $[p]$. The $h^{*}$-polynomial of the order polytope $\mathcal{O}(P)$ is denoted by $$h^{*}(x;P)=h_px^p+h_{p-1}x^{p-1}+\cdots+h_1x+h_0.$$ Equivalently, the Ehrhart polynomial of $\mathcal{O}(P)$ is given by $$\mathrm{ehr}(\mathcal{O}(P),t)=\sum_{i=0}^p h_i\binom{t+p-i}{p}.$$ Stembridge's \texttt{posets} package \cite{Stempackage} can be used to compute the Ehrhart polynomial of $\mathcal{O}(P)$, but not directly. Denoted by $\mathbf{t}$ the $t$-element chain on the set $[t]$. The number of order-preserving maps from $P$ to $\mathbf{t}$ turns out to be a polynomial in $t$ of degree $p$. This polynomial is referred to as the \emph{order polynomial} of $P$, and denoted $\Omega_{P}(t)$. Stanley \cite{StanleyDCG} showed that $$\mathrm{ehr}(\mathcal{O}(P),t)=\Omega_{P}(t+1).$$ The command \texttt{omega} in the \texttt{posets} package computes the order polynomial of $P$ directly, and hence can be used to compute the Ehrhart polynomial. The package \texttt{posets} is available at: https://dept.math.lsa.umich.edu/$\sim$jrs/. There are other algorithms to compute the Ehrhart polynomial or the $h^*$ polynomial of a given order polytope. Stembridge's \texttt{posets} package seems to be the best for order polytopes of dimension no more than $13$ elements. Indeed, the command \texttt{omega} for computing the order polynomial has two options: ``\texttt{linear}" and ``\texttt{ideals}". The \texttt{linear} algorithm has minimal space requirements and a running time that is linear in the number of linear extensions of $P$; The \texttt{ideals} algorithm is more appropriate for larger posets, and has worst-case time and space requirements that are quadratic in the number of order ideals of $P$. If neither algorithm is specified, the default is to use the \texttt{linear} option for posets with $\leq 6$ vertices, and the \texttt{ideals} option otherwise. \section{Main Results} To attack Open Problem \ref{Open-Ehrhart}, one needs only to generate posets $P$ of the desired dimension, compute the Ehrhart polynomial $\mathrm{ehr}(\mathcal{O}(P),t)$ and check its Ehrhart positivity. This turns out to take plenty of computation time. We first describe how to efficiently generate the posets, and then report our computation data. \subsection{Generating Posets} Brinkmann and Mckay \cite{Brinkmann02} determined the number of posets on up to $16$ points. Table \ref{TabPoset} lists the number of posets for up to $13$ points. The {notation $\text{poset}(p)$ refers} to the number of posets on the set $[p]$. \begin{tiny} \begin{table}[htbp] \centering \caption{The number of posets up to $13$ points} \begin{tabular}{c||c|c|c|c|c|c|c|c} \hline \hline $p$ & 1 & 2 & 3 & 4 & 5 & 6 & 7 & 8 \\ \hline $\text{poset}(p)$ & 1 & 2 & 5 & 16 & 63 & 318 & 2045 & 16999 \\ \hline $p$ & 9 & 10& 11 & 12 & 13 & --- & --- & --- \\ \hline $\text{poset}(p)$ & 183231 & 2567284 & 46749427 & 1104891746 & 33823827452 & --- & --- & --- \\ \hline \end{tabular}\label{TabPoset} \end{table} \end{tiny} \texttt{SageMath }\cite{SageMath} can generate all posets with a specified number of elements, but is not efficient. We use the algorithm \texttt{nauty} in \cite{McKay13} for generating posets. The C-package \texttt{nauty} \cite{Mckay2024} is available at: https://users.cecs.anu.edu.au/$\sim$bdm/nauty/. One can use the procedure ``genposetg" to generate all posets with $p$ elements. The timings spent for $p=11, 12, 13$ are approximately 2 seconds, 35 seconds, and 15 minutes, respectively. Note that this requires a large amount of storage space, as the number of posets is very large when $p=11,12,13$ (see Table \ref{TabPoset}). \subsection{Ehrhart Positivity} Stembridge's \texttt{posets} package contains all posets of at most $8$ elements. Stanley observed in \cite{StanleyMathOverfolw} that the order polytope of any poset with at most $8$ elements is Ehrhart positive. Liu and Tsuchiya \cite{LiuTsuchiya19} confirmed that the order polytope of any poset with at most $11$ elements is Ehrhart positive by using \texttt{SageMath} and Stembridge's \texttt{posets} package. It took Liu and Tsuchiya about three weeks to complete the computation for all posets of $11$ elements. Through communication with Tsuchiya, we learned that most of the time was spent for generating the posets. We use C-package \texttt{nauty} to generate the posets of $12$ elements and $13$ elements. Then we transform the output posets into their Maple format. Finally, we use the Maple-package \texttt{posets} to verify the Ehrhart positivity of order polytopes of dimension $12$ and $13$. Based on \cite[Theorem 1.7]{LiuTsuchiya19}, we summarize our findings as follows. \begin{thm}\label{Ehrhart13-14} Any order polytope of dimension $d\leq 13$ is Ehrhart positive. For any positive integer $d\geq 14$, there exists a non-Ehrhart positive order polytope of dimension $d$. \end{thm} \subsection{Real-Rootedness} A poset is defined as \emph{narrow} if its vertices can be partitioned into two chains. According to Dilworth's Theorem, a poset is narrow if and only if it contains no antichain of $3$ elements. Stembridge \cite{Stembridge09} provided an example of a narrow poset with $17$ vertices whose $h^{*}$-polynomial has nonreal zeros. This disproves Neggers' conjecture \cite{Neggers} that the $h^{*}$-polynomial of $\mathcal{O}(P)$ have all real zeros. Stembridge also proved that the $h^{*}$-polynomial of the order polytope of a narrow poset with at most $16$ vertices is real rooted. In \cite{Stembridge09}, Stembridge verified that if $P$ is a poset on at most $10$ vertices, then its $h^*$ polynomial $h^{*}(x;P)$ is real rooted. We employ Sturm's Theorem (for example, see \cite[Page 419]{Knuth81} or \cite{Sturm29}) to calculate the number of real roots of a polynomial. Let $f(x)=a_mx^m+a_{m-1}x^{m-1}+\cdots+a_1x+a_0$ be a polynomial of degree $m$ with real coefficients. The \emph{Sturm sequence} of the polynomial $f(x)$ is the sequence of polynomials $(f_0(x), f_1(x),\ldots,)$ recursively defined by $$f_0(x)=f(x),\ \ \ f_1(x)=f^{\prime}(x),\ \ \ f_{i+1}(x)=-\texttt{rem}(f_{i-1}(x),f_i(x)),$$ for $i\geq 1$, where $f^{\prime}(x)$ is the derivative of $f(x)$, and $\texttt{rem}(f_{i-1}(x),f_i(x))$ is the remainder of $f_{i-1}(x)$ when divided by $f_i(x)$. The length of the Sturm sequence is at most the degree of $f(x)$. The number of sign variations at $c\in\mathbb{R}$ of the Sturm sequence of $f(x)$ is the number of sign changes (ignoring zeros) in the sequence of real numbers $f_0(c), f_1(c), f_2(c),\ldots$. The number of sign variations is denoted by $V(c)$. \begin{thm}{\em \cite{Sturm29}}\label{Sturm-Theorem} Let $f(x)$ be a univariate polynomial with real coefficients. Let $a,b\in \mathbb{R}$ with $a<b$, and $f(a),f(b)\neq 0$. If $f(x)$ has distinct roots (i.e., $\gcd(f(x), f^{\prime}(x))=1$), then the number of real roots of $f(x)$ in $(a,b]$ is $V(a)-V(b)$. \end{thm} Letting $a\to -\infty$ and $b\to \infty$ give what we want. Clearly, the sign of $f(\infty)$ is the sign of its leading coefficient $a_m$, and the sign of $f(-\infty)$ is the sign of $a_m$ if $m$ is even and the sign of $-a_m$ if $m$ is odd. We apply Theorem \ref{Sturm-Theorem} to verify the real rootedness of the $h^{*}$-polynomial of the order polytope $\mathcal{O}(P)$ for dimension $d\leq 13$. This result was validated using \texttt{Maple}. We summarize the results as follows. \begin{thm} The $h^{*}$-polynomial of any order polytope of dimension $d\leq 13$ is real-rooted, and is hence log-concave and unimodal. \end{thm} \subsection{Timing} In this paper, we employ \texttt{Maple 2021} \cite{Maple} to confirm the above results. That is, the Ehrhart positivity and the real-rootedness of $h^{*}(x;P)$ of order polytopes of dimension $\leq 13$. Utilizing a CPU with a frequency of $2.7$ GHz, the verification of Ehrhart positivity and real-rootedness of $h^{*}(x;P)$ for dimensions $12$ and $13$ required $3160$ and $168669$ CPU hours, respectively. To expedite these computations, we used a parallel processing approach with $192$ CPUs. The actual running time for the verifications is approximately $37$ days. For the reader's convenience, we provide all the details of our calculations in the supplementary electronic material. The \texttt{Maple} program can be freely obtained at the first author'swebsite:\\ https://github.com/TygerLiu/TygerLiu.github.io/tree/main/Procedure/orderpolytope. \noindent {\small \textbf{Acknowledgements:} We are grateful to Prof. Fu Liu and Prof. Akiyoshi Tsuchiya for many useful discussions. We also are indebted to Prof. Brendan D. Mckay for helpful advice about the package \textbf{nauty}. The authors would like to thank the anonymous referee for valuable suggestions for improving the presentation. This work is partially supported by the National Natural Science Foundation of China [12071311]. \begin{thebibliography}{99} \bibitem{Alexandersson} P. Alexandersson, \emph{Polytopes and large counterexamples}, Exp. Math. 28(1) (2019), 115--120. \bibitem{BeckRobins} M. Beck and S. Robins, \emph{Computing the continuous discretely, in: Integer-Point Enumeration in Polyhedra}, second edition, Undergraduate Texts in Mathematics. Springer, New York, 2015. \bibitem{Branden05} P. Br\"and\'en, \emph{Sign-graded posets, unimodality of W-polynomials and the Charney-Davis Conjecture}, Electron. J. Combin. 11(2) (2005), R9. \bibitem{Brinkmann02} G. Brinkmann and B. D. Mckay, \emph{Posets on up to $16$ points}, Order. 19 (2002), 147--179. \bibitem{Ehrhart62} E. Ehrhart, \emph{Sur les polyh\'edres rationnels homoth\'etiques \'a $n$ dimensions}, C. R. Acad Sci. Paris. 254 (1962), 616--618. \bibitem{Ferroni23} L. Ferroni and A. Higashitani, \emph{Examples and counterexamples in Ehrhart Theory}, EMS Surv. Math. Sci. (2024). \bibitem{HibiHTY19} T. Hibi, A. Higashitani, A. Tsuchiya, and K. Yoshida, \emph{Ehrhart polynomials with negative coefficients}, Graphs Combin. 35 (2019), 363--371. \bibitem{Knuth81} D. E. Knuth, \emph{The Art of Computer Programming, Vol. 2: Seminumerical Algorithms}, 2nd ed. Addison-Wesley, Reading MA (1981). \bibitem{FuLiu19} F. Liu, \emph{On positivity of Ehrhart polynomials}, Recent trends in algebraic combinatorics, Assoc. Women Math Ser., vol. 16, Springer, Cham, (2019), 189--237. \bibitem{LiuTsuchiya19} F. Liu and A. Tsuchiya, \emph{Stanley's non-Ehrhart-positive order polytopes}, Adv. Appl. Math. 108 (2019), 1--10. \bibitem{Maple} Maple, \emph{Mathematics Software}, https://cn.maplesoft.com/products/maple/index.aspx. \bibitem{Mckay2024} B. D. McKay, \emph{Nauty user's guide}, http://users.cecs.anu.edu.au/$\sim$bdm/nauty, (2024). \bibitem{McKay13} B. D. McKay and A. Piperno, \emph{Practical graph isomorphism II}, J. Symbolic Computation. 60 (2013), 94--112. \bibitem{McMullen77} P. McMullen, \emph{Valuations and Euler-type relations on certain classes of convex polytopes}, Proc. London Math. Soc. 3(35) (1977), 113--135. \bibitem{Neggers} J. Neggers, \emph{Representations of finite partially orderly sets}, J. Combin. Inform. System Sci. 3 (1978), 113--133. \bibitem{Reiner05} V. Reiner and V. Welker, \emph{On the Charney-Davis and Neggers-Stanley conjectures}, J. Combin. Theory Ser. A. 109 (2005), 247--280. \bibitem{SageMath} SageMath, \emph{Open Source Mathematics Software}, http://www.sagemath.org/index.html. \bibitem{StanleyADM80} R. P. Stanley, \emph{Decompositions of rational convex polytopes}, Ann. Discrete Math. 6 (1980), 333--342. \bibitem{StanleyDCG} R. P. Stanley, \emph{Two poset polytopes}, Discrete Comput. Geom. 1 (1986), 9--23. \bibitem{StanleyLog-concave} R. P. Stanley, \emph{Log-concave and unimodal sequences in algebra, combinatorics, and geometry}, Ann. New York Acad Sci. 576 (1989), 500--535. \bibitem{RP.Stanley} R. P. Stanley, \emph{Enumerative Combinatorics (volume 1)}, Cambridge Studies in Advanced Mathematics, vol. 49, Cambridge University Press, (2012). \bibitem{RP.StanleyAC} R. P. Stanley, \emph{Algebraic Combinatorics: Walks, Trees, Tableaux, and More}, second edition, Undergraduate Texts in Mathematics. Springer, (2018). \bibitem{StanleyMathOverfolw} R. P. Stanley, \emph{Positivity of Ehrhart polynomial coefficients}, MathOverflow, https://mathoverflow.net/q/200574. \bibitem{Stempackage} J. Stembridge, \emph{A Maple Package for Posets}, https://www.math.lsa.umich.edu/$\sim$jrs/. \bibitem{Stembridge09} J. Stembridge, \emph{Counterexamples to the poset conjectures of Neggers, Stanley, and Stembridge}, Trans. Amer. Math. Soc. 359 (2007), 1115--1128. \bibitem{Sturm29} J. C. F. Sturm, \emph{M\'emoire sur la r\'esolution des \'equations num\'eriques}, Bulletin des Sciences de F\'erussac. 11 (1829), 419--425. \end{thebibliography} \end{document}
2412.07314v1
http://arxiv.org/abs/2412.07314v1
A measure with small support and p-summable Fourier transform
\documentclass[a4paper,10pt]{article} \usepackage{mathtext} \usepackage[T1,T2A]{fontenc} \usepackage[cp1251]{inputenc} \usepackage[english]{babel} \usepackage{amsmath} \usepackage{amsfonts} \usepackage{amssymb} \usepackage{mathrsfs} \usepackage{amsthm} \usepackage{enumerate} \usepackage{graphicx} \graphicspath{} \DeclareGraphicsExtensions{.pdf,.png,.jpg} \usepackage{color} \usepackage{euscript} \usepackage{cite} \textwidth=16cm \oddsidemargin=0pt \topmargin=0pt \newtheorem{Le}{Lemma}[section] \newtheorem{Def}{Definition}[section] \newtheorem{St}[Le]{Proposition} \newtheorem{Th}{Theorem}[section] \newtheorem{Cor}[Le]{Corollary} \newtheorem{Rem}[Le]{Remark} \newtheorem{Conj}{Conjecture} \newtheorem{Que}{Question} \newtheorem{Ex}{Example}[section] \numberwithin{equation}{section} \newcommand{\R}{\mathbb{R}} \newcommand{\Co}{\mathbb{C}} \newcommand{\N}{\mathbb{N}} \newcommand{\Z}{\mathbb{Z}} \newcommand{\ZZ}{\boldsymbol{\mathrm{Z}}} \newcommand{\eps}{\varepsilon} \newcommand{\eq}[1]{\begin{equation}{#1}\end{equation}} \newcommand{\mlt}[1]{\begin{multline}{#1}\end{multline}} \newcommand{\alg}[1]{\begin{align}{#1}\end{align}} \newcommand{\set}[2]{\{{#1}\mid{#2}\}} \newcommand{\Set}[2]{\Big\{\ {#1}\ \,\Big|\;{#2}\Big\}} \newcommand{\scalprod}[2]{\langle{#1},{#2}\rangle} \newcommand{\fdot}{\,\cdot\,} \newcommand{\Leqref}[1]{\stackrel{\scriptscriptstyle{\eqref{#1}}}{\leq}} \newcommand{\Lseqref}[1]{\stackrel{\scriptscriptstyle{\eqref{#1}}}{\lesssim}} \newcommand{\LeqrefTwo}[2]{\stackrel{\scriptscriptstyle{\eqref{#1},\eqref{#2}}}{\leq}} \newcommand{\LseqrefTwo}[2]{\stackrel{\scriptscriptstyle{\eqref{#1},\eqref{#2}}}{\lesssim}} \newcommand{\Lref}[1]{\stackrel{#1}{\leq}} \newcommand{\Lsref}[1]{\stackrel{#1}{\lesssim}} \DeclareMathOperator{\BV}{BV} \DeclareMathOperator{\I}{I} \DeclareMathOperator{\spec}{spec} \DeclareMathOperator{\supp}{supp} \DeclareMathOperator{\Heat}{H} \DeclareMathOperator{\loc}{loc} \DeclareMathOperator{\s}{s} \DeclareMathOperator{\Lip}{Lip} \newcommand{\Disp}{\mathbb{D}} \newcommand{\E}{\mathbb{E}} \newcommand{\T}{\mathcal{T}} \DeclareMathOperator{\Conv}{Conv} \DeclareMathOperator{\Fl}{Fl} \DeclareMathOperator{\diam}{diam} \DeclareMathOperator{\dist}{dist} \title{A measure with small support and $p$-summable Fourier transform.} \author{Nikita Dobronravov\footnote{Supported by Theoretical Physics and Mathematics Advancement Foundation "BASIS" grant Junior Leader (Math) 21-7-2-12-2 and by the Ministry of Science and Higher Education of the Russian Federation (agreement no. 075-15-2022-287).}} \begin{document} \maketitle \begin{abstract} We construct a probability measure $\mu$ supported on a set of zero $2d/p$\! -Hausdorff measure such that $\hat{\mu}\in L_{p}(\mathbb{R}^d)$. \end{abstract} \section{Introducion} The Uncertainty Principle (UP) in mathematical analysis is a family of facts that state: both function and its Fourier transform cannot be simultaneously small (see~\cite{HavJor}). The following theorem is yet another manifestation of the Uncertainty Principle. Denote the $\alpha$-Hausdorff measure by $\mathcal{H}_\alpha$. \begin{Th}\label{nonend} Let $S\subset \mathbb{R}^d$ be a compact set such that $\mathcal{H}_{\alpha}(S)<\infty$. Let $\zeta$ be a tempered distribution such that $\supp(\zeta)\subset S$ and $\hat{\zeta}\in L_p(\mathbb{R}^d)$ for $p<\frac{2d}{\alpha}$. Then $\zeta=0$. \end{Th} The case of Theorem~\ref{nonend} $d=1$ and $\zeta$ being a measure was considered by Salem. He also proved that for $d=1$ Theorem~\ref{nonend} cannot be strengthened to any $p>\frac{2}{\alpha}$ (see~\cite{Salem}). Beurling obtained a result that implies Theorem~\ref{nonend} in the case $d=1$ (see~\cite{Beurling}). Edgar and Rosenblatt proved Theorem~\ref{nonend} in the case $d-1\leqslant \alpha$ (see~\cite{EdgRos}). Kahane obtained the following result (see~\cite{Kah}). \begin{Th}[Kahane \cite{Kah}]\label{Kahan} Let $S\subset \mathbb{R}^d$ be a compact set such that $\mathcal{H}_{\alpha}(S)<\infty$. Let $\zeta$ be a distribution such that $\supp(\zeta)\subset S$ and $\zeta\in W_2^{\frac{\alpha-d}{2}}(\mathbb{R}^d)$. Then $\zeta=0$. \end{Th} Here $W^{\alpha}_p(\mathbb{R}^d)$ is the potential Sobolev space, defined for $p\in(1,\infty)$ by the following formula: \begin{equation} W^{\alpha}_p(\mathbb{R}^d)=\{f| \ (1-\Delta)^{\frac{\alpha}{2}}f\in L_p(\mathbb{R}^d)\}. \end{equation} We use the notation $(1-\Delta)^{\frac{\alpha}{2}}$ for \begin{equation} ((1-\Delta)^{t}f)^{\widehat{}}(\xi)=|\hat{f}(\xi)|^2(1+|2\pi \xi|^2)^{t}. \end{equation} Theorem~\ref{nonend} can be deduced from Theorem~\ref{Kahan} because the Fourier transform maps $L_p(\mathbb{R}^d)$ to $W_2^{\frac{d}{p}-\frac{d}{2}-\varepsilon}(\mathbb{R}^d)$ for $p>2$ and $\varepsilon>0$: \begin{multline} \|f\|^2_{W_2^{\frac{d}{p}-\frac{d}{2}-\varepsilon}(\mathbb{R}^d)}= \int_{\mathbb{R}^d}|\hat{f}(\xi)|^2(1+|2\pi \xi|^2)^{\frac{d}{p}-\frac{d}{2}-\varepsilon}d\xi\leqslant\\ \left(\int_{\mathbb{R}^d}|\hat{f}(\xi)|^pd\xi\right)^{\frac{2}{p}}\left(\int_{\mathbb{R}^d}(1+|2\pi \xi|^2)^{-\frac{d}{2}-\frac{\varepsilon p}{p-2}}d\xi\right)^{\frac{p-2}{p}}\lesssim\|f\|_{L_p}^2. \end{multline} Here and in what follows $A\lesssim B$ means there exists $C$ such that $A\leqslant CB$ and $C$ is uniform in certain sense. Adams and Polking proved a result that implies Theorem~\ref{nonend} (see~\cite[Theorem A]{AdPo}). However, they did not formulate anything resembling Theorem~\ref{nonend}. The limit case ($\alpha=2d/p$) was still open in the generality of Theorem~\ref{nonend}: it was unknown whether there exists an non zero measure $\mu$ supported on a set $S$, $\mathcal{H}_\alpha(S)<\infty$, and such that $\hat{\mu}\in L_\frac{2d}{\alpha}(\mathbb{R}^d)$. If one makes additional regularity assumptions on $S$, the UP holds true. \begin{Th}[Rosenblatt \cite{Ros}]\label{Rosenblatt} Let $S\subset \mathbb{R}^d$ be a $(d-1)$ dimensional smooth surface. Let $\zeta$ be a distribution such that $\supp(\zeta)\subset S$ and $\hat{\zeta}\in L_{\frac{2d}{d-1}}(\mathbb{R}^d)$. Then $\zeta=0$. \end{Th} \begin{Th}[Agranovskiy, Narayanan \cite{ArgNar}]\label{AN} Let $S\subset \mathbb{R}^d$ be a $C^1$ surface of dimension $k$. Let $\zeta$ be a distribution such that $\supp(\zeta)\subset S$ and $\hat{\zeta}\in L_{\frac{2d}{k}}(\mathbb{R}^d)$. Then $\zeta=0$. \end{Th} \begin{Th}[Raani \cite{Raani}]\label{Raani} Let $S\subset \mathbb{R}^d$ be a compact set such that $\mathcal{P}_{\alpha}(S)<\infty$. Let $\zeta$ be a distribution such that $\supp(\zeta)\subset S$ and $\hat{\zeta}\in L_{\frac{2d}{\alpha}}(\mathbb{R}^d)$. Then $\zeta=0$. \end{Th} Here $\mathcal{P}_{\alpha}$ is the $\alpha$-packing measure (see~\cite[Chapter~5]{Mattila} for more information about packing measures). Theorem~\ref{Raani} is a generalization of Theorems~\ref{Rosenblatt} and~\ref{AN}. Observe $\mathcal{P}_{\alpha}(\cdot)\geqslant\mathcal{H}_\alpha(\cdot)$. This inequality says that packing measure "sees" smaller sets than the Hausdorff measure. The main result of the paper shows this UP does not hold at the endpoint without additional structural assumptions. \begin{Th}\label{res} Let $2<p<\infty$. There exists a compact set $S\subset \mathbb{R}^d$ and a probability measure $\mu$ such that $\supp(\mu)\subset S$, $\hat{\mu}\in L_{p}$, and $\mathcal{H}_{\frac{2d}{p}}(S)=0$. \end{Th} In particular, one cannot generalise Theorem~\ref{Raani} to Hausdorff measures. The note is devoted to the proof of Theorem~\ref{res}, which is an explicit construction of a specific random Cantor-type set. Other constructions of random Cantor sets were used by Salem (see~\cite{Salem}) and Bluhm (see~\cite{Bluhm}). Our construction is more complicated and the Cantor set develops differently along different branches of the corresponding tree. This allows to distinguish the Lorentz spaces. Though these spaces play the pivotal role, we prefer to omit them in our notation. Considerations from which we construct $\mu$ and $S$ in Theorem~\ref{res} are also related with Netrusov--Hausdorff capacities. This paper does not use them as well to make things simpler. An article with the exact form of the Uncertainty Principle for Lorentz spases and Netrusov--Hausdorff capacities will appear elsewhere. In Section~\ref{Constr}, we give a construction of $S$ and $\mu$. In Section~\ref{ESC}, we prove that our construction satisfies the conditions of Theorem~\ref{res}. In Section~\ref{AL}, we prove some auxiliary lemmas. {\bf Acknowledgment.} I am grateful to my scientific adviser D. M. Stolyarov for statement of the problem and attention to my work. I also wish to thank M. K. Dospolova and A. S. Tselishchev for reading my work and advice on article formatting. \section{Construction}\label{Constr} \subsection{General construction} Let $\mathcal{M}={M_0,M_1,...}$ be an infinite sequence of natural numbers to be specified later. We start with the construction of an infinite tree $\mathcal{T}$. We will inductively construct its subtrees $\mathcal{T}_k$ such that $\mathcal{T}_k\subset\mathcal{T}_{k+1}$ and $\mathcal{T}=\cup\mathcal{T}_k$. Let $V(G)$ be the set of vertices of the graph $G$ and let $E(G)$ be the set of edges. Set $V(\mathcal{T}_0)=\{Q_0,Q_1,\dots,Q_{M_0}\}$ and $E(\mathcal{T}_0)=\{(Q_0,Q_1),(Q_0,Q_2),\dots,(Q_0,Q_{M_0})\}$. Assume we have concrusted $\mathcal{T}_{k-1}$. To build $\mathcal{T}_k$ we add to $\mathcal{T}_{k-1}$ $M_k$ new vertices connect them with $Q_k$: \begin{equation} V(\mathcal{T}_k)=V(\mathcal{T}_{k-1})\cup\{Q_{M_0+\dots+M_{k-1}+1},Q_{M_0+\dots+M_{k-1}+2},\dots ,Q_{M_0+\dots+M_{k-1}+M_k}\}, \end{equation} \begin{equation} E(\mathcal{T}_k)=E(\mathcal{T}_{k-1})\cup\{(Q_k,Q_{M_0+\dots+M_{k-1}+1}),(Q_k,Q_{M_0+\dots+M_{k-1}+2}),\dots ,(Q_k,Q_{M_0+\dots+M_{k-1}+M_k})\}. \end{equation} We will say that $Q_k$ is a paren of those new vertices and that they are kids of $Q_k$. \begin{figure}[h] \center{\includegraphics[scale=0.5]{tree.pdf}} \caption{Tree $\mathcal{T}_2$ for $M_0=3$, $M_1=4$, $M_2=6$.}\label{pic2} \end{figure} We define the weight of the vertex $Q_i$ by the formula \begin{equation} b(Q_i)=\!\!\!\!\underset{\text{ancestor of }Q_i}{\prod\limits_{Q_j\text{ is an}}}\!\!\!\! M_j^{-1}. \end{equation} We say that $Q_k$ belongs to the $n^{\text{th}}$ layer if it has exactly $n$ ancestors. We have bound $b(Q_k)\leqslant 2^{-n}$, if $Q_k$ belongs to the $n^{\text{th}}$ layer. The notation $n(Q_k)$ means the number of the layer of $Q_k$. Clearly, \begin{equation}\label{probsum} \sum\limits_{n(Q_k)=n}b(Q_k)=1. \end{equation} Now let $Q_k$ (the vertices) be cubes in $\mathbb{R}^d$ and let $l(Q)$ the side length of cube $Q$. \begin{Def} The cube sequence $\{Q_0,Q_1,\cdots\}$ corresponds to the tree $\mathcal{T}$ if it satisfies requirements: \begin{enumerate}[1)] \item $Q_0=[0,1]^d$; \item if $Q_i$ is a parent of $Q_j$, then $Q_j\subset Q_i$; \item if $Q_j$ is a son of $Q_k$, then \begin{equation}\label{soot} l(Q_j)=r_k=\frac{M_k^{-\frac{p}{2d}}b(Q_k)^{\frac{p}{2d}}}{n(Q_k)+1}. \end{equation} \end{enumerate} \end{Def} \begin{Def} Assume $\{Q_0,Q_1,\cdots\}$ corresponds to $\mathcal{T}$. The set $C_n$ is defined by the formula \begin{equation} C_n=\underset{n(Q_i)=n}{\bigcup}Q_i. \end{equation} We call the set $C=\cap_{n=1}^\infty C_n$ the Cantor-type set corresponding to $\mathcal{T}$. \end{Def} Note that different songs of a cube may intersect. \begin{St}\label{H=0} Let $C$ be a Cantor-type set corresponding to $\mathcal{T}$. Then $\mathcal{H}_{\frac{2d}{p}}(C)=0$. \end{St} \begin{proof} The cubes of $n^\text{th}$ layer provide a covering of $C$. We use those coverings to estimate the Hausdorff measure of $C$: \begin{equation} \sum\limits_{n(Q_k)=n}\diam(Q_k)^{\frac{2d}{p}}\lesssim \sum\limits_{n(Q_k)=n}l(Q_k)^{\frac{2d}{p}}= \sum\limits_{n(Q_k)=n-1}M_kr_k^{\frac{2d}{p}}\overset{\text{\eqref{soot}}}{=} \sum\limits_{n(Q_k)=n-1}\frac{b(Q_k)}{n^{\frac{2d}{p}}}\ \overset{\eqref{probsum}}{=}\ \frac{1}{n^{\frac{2d}{p}}}. \end{equation} \end{proof} There is a family of Cantor-type sets corresponding to the tree $\mathcal{T}$. To each Cantor-type set $C$, we will assign a probability measure $\mu$ such that $\supp(\mu)\subset C$. Denote by $\lambda_Q$ the Lebesgue probability measure on the cube $Q$. Denote the measure $\mu_k$ by formula: \begin{equation} \mu_k=\sum\limits_{i=k+1}^{M_0+\dots+M_{k}}b(Q_i)\lambda_{Q_i}. \end{equation} These measures satisfy recurrence relations: \begin{equation} \mu_0=\lambda_{[0,1]^d}, \end{equation} \begin{equation}\label{nonrandomrel} \mu_k=\mu_{k-1}-b(Q_k)\lambda_{Q_k}+\frac{b(Q_k)}{M_k}\underset{\text{son of }Q_k}{\sum\limits_{Q_j \text{ is a}}}\lambda_{Q_j}. \end{equation} \subsection{Random construction} The behavior of the Fourier transform of a Cantor measure can be quite chaotic (see~\cite{Strichartz} for some examples). In this subsection we will use some randomization to choose a representative for which we can estimate $\hat{\mu}$. \begin{Def} Let $M\in\mathbb{N}$ and let $0<r<\frac{1}{2}$. Let $\mu_{M,r}$ be the random variable taking values in the set of probability measures: \begin{equation} \mu_{M,r}=\frac{1}{M}\sum\limits_{j=1}^{M}S_j\lambda_{[0,r]^d}. \end{equation} Here $\{S_j\}_{j=1}^{M}$ is a sequence of independent shifts that are uniformly distributed on the cube $[0,1-r]^d$. \end{Def} Let $f_r$ be a piecewise linear function (see Figure~\ref{pic1}) \begin{equation} f_r(t)= \begin{cases} 0, & t\in (-\infty,0]\cup[1,\infty),\\ \frac{t}{r(1-r)}, & t\in [0,r],\\ \frac{1}{1-r}, & t\in [r,1-r],\\ \frac{1-t}{r(1-r)}, & t\in [1-r,1]. \end{cases} \end{equation} \begin{figure}[h] \center{\includegraphics[scale=0.5]{f_r_grafic.pdf}} \caption{Graph of $f_r$ for $r=0.1$}\label{pic1} \end{figure} Let $F_r(x)=\prod\limits_{i=1}^df_r(x_i)$. Then, \begin{equation}\label{mathex} \mathbb{E}\mu_{M,r}=\lambda_{[0,r]^d}*\lambda_{[0,1-r]^d}=F_r(x)dx \end{equation} and \begin{equation}\label{glav} \hat{\mu}_{M,r}(x)\overset{D}{=}\frac{1}{M}\sum_{j=1}^{M}e^{-2\pi i<\beta_{r,j},x>}\hat{\lambda}_0(xr). \end{equation} Here $\lambda_0$ is the Lebesgue measure on $[0,1]^d$ and $\{\beta_{r,j}\}_{j=1}^{M}$ is the sequence of independent vectors that are uniformly distributed on the cube $[0,1-r]^d$. The notation $\overset{D}{=}$ means equality of distributions. Let $\nu_{M,r}=\mu_{M,r}(\omega)$ be the value of $\mu_{M,r}$ at some point $\omega$ of the probability space to be chosen later. By definitions, $\nu_{M,r}$ is a probability measure. We will use the measures $\nu_{M,r}$ to construct the sequences $\{Q_0,\dots,Q_{M_0+\dots+M_k}\}$ of cubes inductively. Set the cube $Q_0=[0,1]^d$. The measure $\nu_{M_0,r_0}$ corresponds to $M_0$ cubes. The sequence $\{Q_1,\dots,Q_{M_0}\}$ consists of those cubes. Assume we have constructed and the cubes $Q_0,Q_1,\dots,Q_{M_0+M_1+\dots+M_{k-1}}$. Let $\nu_{M_k,\frac{r_k}{l(Q_k)},Q_k}$ be the image of $\nu_{M_k,\frac{r_k}{l(Q_k)}}$ under the homothety that maps $[0,1]^d$ into $Q_k$. The measure $\nu_{M_k,\frac{r_k}{l(Q_k)},Q_k}$ corresponds to $M_k$ cubes (see Figure~\ref{pic}). Add them to the end of the cube sequence. Thus, we have constructed the cubes $Q_0,Q_1,\dots,Q_{M_0+M_1+\dots+M_{k}}$. \begin{figure}[h] \center{\includegraphics[scale=0.5]{Construction.pdf}} \caption{Construction}\label{pic} \end{figure} \begin{Rem} The cube sequence $\{Q_0,Q_1,\cdots\}$ corresponds to a tree $\mathcal{T}$. \end{Rem} For this cube sequence the recurrence relations~\eqref{nonrandomrel} turn into \begin{equation}\label{formz} \mu_k=\mu_{k-1}-b(Q_k)\lambda_{Q_k}+b(Q_k)\nu_{M_k,\frac{r_k}{l(Q_k)},Q_k}. \end{equation} \section{Estimates}\label{ESC} \begin{Le}\label{Ep} For all $M\in \mathbb{N}$, $r<\frac{1}{2}$ and $p>1$ the inequality \begin{equation}\label{MP} \int\limits_{\mathbb{R}^d}|\mathbb{E}\hat{\mu}_{M,r}(x)|^pdx\lesssim 1 \end{equation} is true. \end{Le} The constant depends only on $d$ and $p$. \begin{proof} By \eqref{glav}, we have \begin{equation} \mathbb{E}\hat{\mu}_{M,r}(x)=\mathbb{E}\frac{1}{M}\sum_{j=1}^{M}e^{-2\pi i<\beta_{r,j},x>}\hat{\lambda}_0(rx)= \hat{\lambda}_0(rx)\mathbb{E}e^{-2\pi i<\beta_{r,1},x>}. \end{equation} Since $\lambda_0$ is a probability measure, $|\hat{\lambda}_0|\leqslant 1$. With the notation $h(t)=\frac{1-e^{it}}{t}$, we have \begin{multline} |\mathbb{E}\hat{\mu}_{M,r}(x)|= \left|\hat{\lambda}_0(rx)\mathbb{E}e^{-2\pi i<\beta_{r,1},x>}\right|\leqslant |\mathbb{E}e^{-2\pi i<\beta_{r,1},x>}|=\\ \left|\frac{1}{(1-r)^d}\int\limits_{[0,1-r]^d}e^{-2\pi i<\beta_{r,1},x>}d\beta_{r,1}\right|= \left|\prod\limits_{j=1}^dh\left(2\pi (1-r)x_j\right)\right|\lesssim\\ \prod\limits_{j=1}^{d}\min\left(1,\frac{1}{|x_j|}\right). \end{multline} The latter estimate implies \eqref{MP}. \end{proof} \begin{Le}\label{Np} For all $M\in \mathbb{N}$, $r<\frac{1}{2}$ and $p>1$ the inequality \begin{equation} \int\limits_{\mathbb{R}^d}\mathbb{E}|\hat{\mu}_{M,r}(x)-\mathbb{E}\hat{\mu}_{M,r}(x)|^pdx\lesssim M^{-\frac{p}{2}}r^{-d} \end{equation} is true. \end{Le} \begin{proof} By~\eqref{glav} we have \begin{equation} \hat{\mu}_{M,r}(x)-\mathbb{E}\hat{\mu}_{M,r}(x)\overset{D}{=}\frac{1}{M}\sum_{j=1}^{M}\big(e^{-2\pi i<\beta_{r,j},x>}\hat{\lambda}_0(xr)-\mathbb{E}\hat{\mu}_{M,r}(x)\big). \end{equation} We use Lemma~\ref{mz} below: \begin{multline}\label{glav1} \mathbb{E}|\hat{\mu}_{M,r}(x)-\mathbb{E}\hat{\mu}_{M,r}(x)|^p= \mathbb{E}\left|\frac{1}{M}\sum_{j=1}^{M}\left(e^{-2\pi i<\beta_{r,j},x>}\hat{\lambda}_0(xr)-\mathbb{E}\hat{\mu}_{M,r}(x)\right)\right|^p\leqslant\\ C_pM^{-p}M^{\frac{p}{2}}\mathbb{E}\left|e^{-2\pi i<\beta_{r,1},x>}\hat{\lambda}_0(xr)-\mathbb{E}\hat{\mu}_{M,r}(x)\right|^p= C_pM^{-\frac{p}{2}}\mathbb{E}\left|e^{-2\pi i<\beta_{r,1},x>}\hat{\lambda}_0(xr)-\mathbb{E}\hat{\mu}_{M,r}(x)\right|^p\leqslant\\ C_pM^{-\frac{p}{2}}2^{p-1}\left(\left|\hat{\lambda}_0(xr)\right|^p+\left|\mathbb{E}\hat{\mu}_{M,r}(x)\right|^p\right). \end{multline} We integrate \eqref{glav1} to complete the proof: \begin{equation} \int\limits_{\mathbb{R}^d}\mathbb{E}|\hat{\mu}_{M,r}(x)-\mathbb{E}\hat{\mu}_{M,r}(x)|^pdx\leqslant \int\limits_{\mathbb{R}^d} C_pM^{-\frac{p}{2}}2^{p-1}\left(\left|\hat{\lambda_0}(xr)\right|^p+\left|\mathbb{E}\hat{\mu}_{M,r}(x)\right|^p\right)dx\lesssim M^{-\frac{p}{2}}r^{-d}. \end{equation} In final step, we have used Lemma~\ref{Ep}. \end{proof} Fix $p_1$ such that $p_1>p$. \begin{Cor}\label{PrE} For all $M\in\mathbb{N}$ and $0<r<\frac{1}{2}$ we can choose measures $\nu_{M,r}=\mu_{M,r}(\omega)$ such that \begin{equation} \int\limits_{\mathbb{R}^d}|\hat{\nu}_{M,r}(x)-\mathbb{E}\hat{\mu}_{M,r}(x)|^{p_1}dx\lesssim M^{-\frac{p_1}{2}}r^{-d}, \end{equation} \begin{equation} \int\limits_{\mathbb{R}^d}|\hat{\nu}_{M,r}(x)-\mathbb{E}\hat{\mu}_{M,r}(x)|^{p}dx\lesssim M^{-\frac{p}{2}}r^{-d}. \end{equation} \end{Cor} \begin{Le}\label{ooo} Let $\lambda_0$ be the Lebesgue measure on $[0,1]^d$. Then for $p>2$ the inequality \begin{equation} \|\hat{\lambda}_0-\mathbb{E}\hat{\mu}_{M,r}\|_{L_{p}}\lesssim r^{\frac{1}{p'}} \end{equation} is true. \end{Le} \begin{proof} One may see from~\eqref{mathex} that $\|\lambda_0-\mathbb{E}\mu_{M,r}\|_{L_{p'}}\lesssim r^{\frac{1}{p'}}$. The Hausdorff--Young inequality finishes the proof. \end{proof} \begin{Cor}\label{cor1} We have the inequalities \begin{equation} \|\hat{\nu}_{M,r}(x)-\hat{\lambda}_0(x)\|^p_{L_{p}}\lesssim M^{-\frac{p}{2}}r^{-d} + r^{\frac{p}{p'}}, \end{equation} \begin{equation} \|\hat{\nu}_{M,r}(x)-\hat{\lambda}_0(x)\|^{p_1}_{L_{p_1}}\lesssim M^{-\frac{p_1}{2}}r^{-d}+r^{\frac{p_1}{p_1'}}. \end{equation} Here $\nu_{M,r}$ is defined by the choice of $\omega$ in Corollary~\ref{PrE}. \end{Cor} \begin{Le}\label{Ras} Let $M_0,M_1,M_2,\dots,M_{k-1}$ be fixed and let $M_k$ tends to infinity. Then, the following inequalities \begin{equation} \varlimsup\limits_{M_k\rightarrow\infty}\|\hat{\mu}_k-\hat{\mu}_{k-1}\|^p_{L_{p}}\lesssim b(Q_k)^{\frac{p}{2}}(n(Q_k)+1)^d, \end{equation} \begin{equation} \lim\limits_{M_k\rightarrow\infty}\|\hat{\mu}_k-\hat{\mu}_{k-1}\|_{L_{p_1}}=0 \end{equation} are true. \end{Le} \begin{proof} We can write estimates \begin{multline} \|\hat{\mu}_k-\hat{\mu}_{k-1}\|^p_{L_{p}}\overset{\text{\eqref{formz}}}{=} \|b(Q_k)(\hat{\nu}_{M_k,\frac{r_k}{l(Q_k)},Q_k}-\hat{\lambda}_{Q_k})\|^p_{L_{p}}= b(Q_k)^p\|\hat{\nu}_{M_k,\frac{r_k}{l(Q_k)},Q_k}-\hat{\lambda}_{Q_k}\|^p_{L_{p}}=\\ b(Q_k)^pl(Q_k)^{-d}\|\hat{\nu}_{M_k,\frac{r_k}{l(Q_k)}}-\hat{\lambda}_0\|^p_{L_{p}}\overset{\text{Cor~\ref{cor1}}}{\lesssim} b(Q_k)^pl(Q_k)^{-d}\left(M_k^{-\frac{p}{2}}\left(\frac{r_k}{l(Q_k)}\right)^{-d} + \left(\frac{r_k}{l(Q_k)}\right)^{\frac{p}{p'}}\right)\overset{\text{\eqref{soot}}}{=}\\ b(Q_k)^{\frac{p}{2}}(n(Q_k)+1)^d+b(Q_k)^{p}l(Q_k)^{-d-\frac{p}{p'}}r_k^{\frac{p}{p'}}\overset{M_k\rightarrow\infty}{\rightarrow} b(Q_k)^{\frac{p}{2}}(n(Q_k)+1)^d \end{multline} and in the same way we have \begin{equation} \|\hat{\mu}_k-\hat{\mu}_{k-1}\|^{p_1}_{L_{p_1}}\lesssim b(Q_k)^{p_1-\frac{p}{2}}M_k^{-\frac{p_1-p}{2}}(n(Q_k)+1)^d+ b(Q_k)^{p_1}l(Q_k)^{-d}\left(\frac{r_k}{l(Q_k)}\right)^{\frac{p_1}{p_1'}} \overset{M_k\rightarrow\infty}{\rightarrow}0. \end{equation} Here $b(Q_k)$, $n(Q_k)$ and $l(Q_k)$ are fixed and $r_k\rightarrow0$ while $M_k\rightarrow\infty$ (see equation~\eqref{soot}). \end{proof} \begin{Cor} There exists a constant $C>0$ such that inequality \begin{equation} \|\hat{\mu}_{k}\|^p_{L_{p}}\leqslant \|\hat{\mu}_{k-1}\|^p_{L_{p}}+Cb(Q_k)^{\frac{p}{2}}(n(Q_k)+1)^d \end{equation} is true, provided $M_k$ is sufficiently large. \end{Cor} This corollary is a combination of the previous lemma and Lemma~\ref{p+p} below. Let $\{M_0,M_2,\dots\}$ be rapidly growing infinite sequence. Let $\mu_{\infty}$ be a weak* limit of $\mu_{k}$. The rapid grouth of $\mathcal{M}_\infty$ provides us with the following inequality \begin{multline} \|\hat{\mu}_{\mathcal{M}_k}\|^p_{L_p}\leqslant \|\hat{\lambda}_0\|^p+C\sum\limits_{j=0}^{k}b(Q_j)^{\frac{p}{2}}n(Q_j)^d\lesssim \sum\limits_{j=0}^{\infty}b(Q_j)^{\frac{p}{2}}(n(Q_j)+1)^d= \sum\limits_{n=0}^{\infty}\sum\limits_{n(Q_j)=n}b(Q_j)^{\frac{p}{2}}(n+1)^d\leqslant\\ \sum\limits_{n=0}^{\infty}\sum\limits_{n(Q_j)=n}b(Q_j)2^{-n(\frac{p}{2}-1)}(n+1)^d\ \overset{\eqref{probsum}}{=}\ \sum\limits_{n=0}^{\infty} 2^{-n(\frac{p}{2}-1)}(n+1)^d<\infty. \end{multline} Thus $\hat{\mu}_{\infty}\in L_p(\mathbb{R}^d)$. This formula and Proposition~\ref{H=0} completes the proof of Theorem~\ref{res}. \section{Auxiliary lemmas}\label{AL} \begin{Le}\label{mz} Let $\{X_j\}_{j=1}^M$ be i.i.d. random variables with zero mean. Then, for $p>2$ the inequality \begin{equation} \mathbb{E}\left|\sum_{j=1}^{M}X_j\right|^p\leqslant C_pM^{\frac{p}{2}}\mathbb{E}|X_1|^p \end{equation} is true. Here $C_p$ is a constant that does not depend on the $X_j$. \end{Le} \begin{proof} First, we use the Marcinkiewicz--Zygmund inequality (see~\cite[Chapter~10.3, Theorem 2]{ChowTeicher}) \begin{equation} \mathbb{E}\left|\sum_{j=1}^{M}X_j\right|^p\leqslant C_p\mathbb{E}\left(\sum_{j=1}^{M}|X_j|^2\right)^\frac{p}{2}. \end{equation} Second, we use H\"older's inequality to obtain: \begin{equation} \mathbb{E}\left(\sum_{j=1}^{M}|X_j|^2\right)^\frac{p}{2}\leqslant\mathbb{E} M^{\frac{p}{2}-1}\sum\limits_{j=1}^{M}|X_j|^p=M^{\frac{p}{2}}\mathbb{E}|X_1|^p. \end{equation} \end{proof} \begin{Le}\label{p+p} Let $f\in L_{p}$ be a function and let $g_j\in L_{p}$ be a sequence of functions. Assume that $\varlimsup\limits_{j\rightarrow\infty}\|g_j\|_{L_{p}}\leqslant A$ and for some $p_1>p$ we have $\lim\limits_{j\rightarrow\infty}\|g_j\|_{L_{p_1}}=0$. Then \begin{equation} \varlimsup\limits_{j\rightarrow\infty} \|f+g_j\|^p_{L_{p}}\leqslant \|f\|^p_{L_{p}}+A^p. \end{equation} \end{Le} \begin{proof} For a function $h$ and $\varepsilon>0$, let \begin{equation} h_{\geqslant \varepsilon}=\chi_{\{|h|\geqslant \varepsilon\}}h, \end{equation} \begin{equation} h_{< \varepsilon}=\chi_{\{|h|< \varepsilon\}}h. \end{equation} Clearly, $h=h_{\geqslant \varepsilon}+h_{< \varepsilon}$. Pick some $\delta>0$. Then \begin{multline}\label{texv} \|f+g_j\|_{L_p}\leqslant \|f_{\geqslant\varepsilon}+{g_j}_{<\delta}\chi_{\{f< \varepsilon\}}\|_{L_p}+\|{g_j}_{\geqslant\delta}\|_{L_p}+\|f_{<\varepsilon}\|_{L_p}+\|{g_j}_{<\delta}\chi_{\{f\geqslant \varepsilon\}}\|_{L_p}=\\ (\|f_{\geqslant\varepsilon}\|^p_{L_p}+\|{g_j}_{<\delta}\chi_{\{f< \varepsilon\}}\|^p_{L_p})^{\frac{1}{p}}+\|{g_j}_{\geqslant\delta}\|_{L_p}+\|f_{<\varepsilon}\|_{L_p}+\|{g_j}_{<\delta}\chi_{\{f\geqslant \varepsilon\}}\|_{L_p}\leqslant\\ (\|f\|^p_{L_p}+\|g_j\|^p_{L_p})^{\frac{1}{p}}+\|{g_j}_{\geqslant\delta}\|_{L_p}+\|f_{<\varepsilon}\|_{L_p}+\delta\|\chi_{\{f\geqslant \varepsilon\}}\|_{L_p}. \end{multline} The equality $\lim\limits_{j\rightarrow\infty}\|g_j\|_{L_{p_1}}=0$ implies $\lim\limits_{j\rightarrow\infty}\|{g_j}_{\geqslant\delta}\|_{L_p}=0$. Therefore, \eqref{texv} yields \begin{multline} \varlimsup\limits_{j\rightarrow\infty} \|f+g_j\|_{L_p}\leqslant (\|f\|^p_{L_p}+A^p)^{\frac{1}{p}}+\|f_{<\varepsilon}\|_{L_p}+\delta\|\chi_{\{f\geqslant \varepsilon\}}\|_{L_p}\overset{\delta\rightarrow0}{\rightarrow} (\|f\|^p_{L_p}+A^p)^{\frac{1}{p}}+\|f_{<\varepsilon}\|_{L_p}\overset{\varepsilon\rightarrow0}{\rightarrow}\\ (\|f\|^p_{L_p}+A^p)^{\frac{1}{p}}. \end{multline} \end{proof} \begin{thebibliography}{99} \bibitem{AdPo} D. R. Adams, J. C. Polking, \emph{The equivalence of two definitions of capacity}, Proc. Amer. Math. Soc. 37 (1973) 529-534. \bibitem{ArgNar}M. L. Agranovskiy, E. K. Narayanan, \emph{$L_p$ integrability, support of Fourier transform, and uniqueness theorem for convolution equations}, Journal of Fourier Analysis and Appl. 10:3 (2004), 315-324. \bibitem{Beurling} A. Beurling, \emph{ On a closure problem}, Ark. Mat. 1 (1951), 301-303. \bibitem{Bluhm} C. Bluhm, \emph{Random recursive construction of Salem sets}, Ark. Mat. 34 (1996), 51-63. \bibitem{ChowTeicher} Y. S. Chow, H. Teicher, \emph{Probability Theory: Independence, Interchangeability, Martingales}, third edition, Springer, 1997. \bibitem{EdgRos} G. A. Edgar, J. M. Rosenblatt, \emph{Difference equations over locally compact abelean groups}, Transactions AMS 253 (1979), 273-289. \bibitem{HavJor} V. Havin, B. J\"{o}ricke, \emph{The Uncertainty principle in harmonic analysis}, Springer, 1994. \bibitem{Kah} J.-P.-Kahane, \emph{Dimension capacitaire et dimension de Hausdorff} , Colloque de Theorie du Potentiel, Springer Lecture Notes in Mathematics, 393-400, 1984 (in French). \bibitem{Mattila} P. Mattila, \emph{Geometry of sets and measures in Euclidean space}, Cambridge University Press, 1995. \bibitem{Salem} R. Salem, \emph{On singular monotonic functions whose spectrum has a given Hausdorff dimension}, Ark. Mat. 1 (1950), 353-365 \bibitem{Raani} K. S. Senthil Raani, \emph{$L^p$-integrability, dimensions of supports of Fourier transforms and applications}, Journal of Fourier Analysis and Appl. 20:4 (2014), 801-815 \bibitem{Strichartz} R. S. Strichartz, \emph{Self-Similar Measures and Their Fourier Transforms} I. Indiana University Mathematics Journal, 39(3) (1990), 797817. \bibitem{Ros} J. M. Rosenblatt, \emph{Linear independence of translations}, Journ. Austral. Math. Soc. 59 (1995), 131-133. \end{thebibliography} \ \ \ {\small St. Petersburg State University Department of Leonhard Euler International Mathematical Institute;\\ e-mail:[email protected]\\ \bigskip } \end{document}
2412.07308v2
http://arxiv.org/abs/2412.07308v2
Iwasawa theory and ranks of elliptic curves in quadratic twist families
\documentclass[table]{amsart} \newcounter{dummy} \makeatletter \newcommand{\mylabel}[2]{#2\def\@currentlabel{#2}\label{#1}} \makeatother \usepackage{amssymb,stmaryrd,mathrsfs} \usepackage{comment} \usepackage{float} \usepackage{longtable} \usepackage{pdflscape} \usepackage{orcidlink} \usepackage{booktabs} \usepackage{hyperref} \definecolor{vegasgold}{rgb}{0.77, 0.7, 0.35} \definecolor{darkgoldenrod}{rgb}{0.72, 0.53, 0.04} \definecolor{gold(metallic)}{rgb}{0.83, 0.69, 0.22} \hypersetup{ colorlinks=true, linkcolor=darkgoldenrod, filecolor=brown, urlcolor=gold(metallic), citecolor=darkgoldenrod, pdftitle={Iwasawa theory and ranks of elliptic curves in quadratic twist families}, } \usepackage[all,cmtip]{xy} \usepackage[margin=1.2 5in]{geometry} \usepackage{tikz} \usetikzlibrary{shapes.geometric} \tikzset{every loop/.style={min distance=10mm,looseness=10}} \DeclareFontFamily{U}{wncy}{} \DeclareFontShape{U}{wncy}{m}{n}{<->wncyr10}{} \DeclareSymbolFont{mcy}{U}{wncy}{m}{n} \DeclareMathSymbol{\Sh}{\mathord}{mcy}{"58} \usepackage[T2A,T1]{fontenc} \usepackage[OT2,T1]{fontenc} \newcommand{\Zhe}{\mbox{\usefont{T2A}{\rmdefault}{m}{n}\CYRZH}} \newtheorem{theorem}{Theorem}[section] \newtheorem{lemma}[theorem]{Lemma} \newtheorem{question}[theorem]{Question} \newtheorem{conj}[theorem]{Conjecture} \newtheorem{heuristic}[theorem]{Heuristic} \newtheorem{proposition}[theorem]{Proposition} \newtheorem{corollary}[theorem]{Corollary} \newtheorem{definition}[theorem]{Definition} \newtheorem{lthm}{\color{darkgoldenrod} \textbf{Theorem}} \renewcommand{\thelthm}{\Alph{lthm}} \newtheorem{fact}[theorem]{Fact} \numberwithin{equation}{section} \newcommand{\ralg}{\mathbf{r}_{\op{alg}}} \newcommand{\ran}{\mathbf{r}_{\op{an}}} \theoremstyle{remark} \newtheorem{remark}[theorem]{Remark} \newtheorem{example}[theorem]{Example} \newcommand{\op}[1]{\operatorname{#1}} \newcommand{\Y}{\mathcal{Y}} \newcommand{\G}{\mathfrak{G}} \newcommand{\tr}{\operatorname{tr}} \newcommand{\Frob}{\operatorname{Frob}} \newcommand{\Gal}{\operatorname{Gal}} \newcommand{\Fil}{\operatorname{Fil}} \newcommand{\cor}{\operatorname{cor}} \newcommand{\ac}{\operatorname{ac}} \newcommand{\ns}{\operatorname{ns}} \newcommand{\bE}{\mathbf{E}} \newcommand{\Aut}{\operatorname{Aut}} \newcommand{\DD}{\mathbb{D}} \newcommand{\BB}{\mathbb{B}} \newcommand{\NN}{\mathbb{N}} \newcommand{\fE}{\mathfrak{E}} \newcommand{\cE}{\mathcal{E}} \newcommand{\fL}{\mathfrak{L}} \newcommand{\fA}{\mathfrak{A}} \newcommand{\fd}{\mathfrak{d}} \newcommand{\Qp}{\mathbb{Q}_p} \newcommand{\Zp}{\mathbb{Z}_p} \newcommand{\HH}{\mathbb{H}} \newcommand{\FF}{\mathbb{F}} \newcommand{\sH}{\mathscr{H}} \newcommand{\g}{\mathbf{g}} \newcommand{\Frac}{\mathrm{Frac}} \newcommand{\ord}{\mathrm{ord}} \newcommand{\GL}{\mathrm{GL}} \newcommand{\cG}{\mathcal{G}} \newcommand{\cF}{\mathcal{F}} \newcommand{\cH}{\mathcal{H}} \newcommand{\Z}{\mathbb{Z}} \newcommand{\p}{\mathfrak{p}} \newcommand{\Q}{\mathbb{Q}} \newcommand{\F}{\mathbb{F}} \newcommand{\D}{\mathfrak{D}} \newcommand{\fM}{\mathfrak{M}} \newcommand{\Lp}{\mathbf{L}_p} \newcommand{\vp}{\varphi} \newcommand{\cL}{\mathcal{L}} \newcommand{\cC}{\mathcal{C}} \newcommand{\cK}{\mathcal{K}} \newcommand{\cY}{\mathcal{Y}} \newcommand{\cO}{\mathcal{O}} \newcommand{\cS}{\mathcal{S}} \newcommand{\rk}{\mathrm{rk}} \newcommand{\T}{\mathcal{T}} \newcommand{\fv}{\mathfrak{v}} \newcommand{\cc}{\mathfrak{c}} \newcommand{\ff}{\mathfrak{f}} \newcommand{\fI}{\mathfrak{I}} \newcommand{\Qpn}{\QQ_{p,n}} \newcommand{\Hom}{\mathrm{Hom}} \newcommand{\new}{\mathrm{new}} \newcommand{\base}{\mathrm{base}} \newcommand{\Sel}{\mathrm{Sel}} \newcommand{\Char}{\mathrm{char}} \newcommand{\Spec}{\mathrm{Spec}\ } \newcommand{\Gr}{\mathrm{Gr}} \newcommand{\Reg}{\mathrm{Reg}} \newcommand{\len}{\mathrm{len}} \newcommand{\coker}{\mathrm{coker}} \newcommand{\pr}{\mathrm{pr}} \newcommand{\Tw}{\mathrm{Tw}} \renewcommand{\Col}{\mathrm{Col}} \newcommand{\rank}{\mathrm{rank}} \renewcommand{\div}{\mathrm{div}} \newcommand{\corank}{\mathrm{corank}} \newcommand{\ur}{\mathrm{ur}} \newcommand{\N}{\mathbb{N}} \newcommand{\Qinf}{\QQ_\infty} \newcommand{\Qpinf}{\QQ_{\infty,p}} \newcommand{\Qvinf}{\QQ_{\infty,v}} \newcommand{\Kinf}{K_\infty} \newcommand{\Kvinf}{K_{\infty,v}} \newcommand{\Kpinf}{K_{\infty,p}} \newcommand{\vpi}{\varpi} \newcommand{\Afs}{A_{f,s}} \newcommand{\Ags}{A_{g,s}} \newcommand{\Tfs}{\T_{f,s}} \newcommand{\Vfs}{\mathcal{V}_{f,s}} \newcommand{\As}{\mathcal{A}_{f,s}} \newcommand{\Kwinf}{K_{\infty,w}} \newcommand{\Klinf}{_{\infty,\ell}} \newcommand{\TT}{\mathbf{T}} \renewcommand{\AA}{\mathbf{A}} \newcommand{\X}{\mathcal X} \newcommand{\tor}{\mathrm{tors}} \newcommand{\cyc}{\mathrm{cyc}} \newcommand{\A}{\mathcal{A}} \newcommand{\V}{\mathcal{V}} \newcommand{\Hc}{\mathcal{H}} \newcommand{\W}{\mathcal{W}} \renewcommand{\labelitemii}{$\diamond$} \newcommand{\AR}[1]{\textcolor{red}{#1}} \newcommand\mtx[4] { \left( {\begin{array}{cc} #1 & #2 \\ #3 & #4 \\ \end{array} } \right)} \begin{document} \title[Iwasawa Theory and Ranks of Elliptic Curves in Quadratic Twist Families]{Iwasawa Theory and Ranks of Elliptic Curves in Quadratic Twist Families} \author[J.~Hatley]{Jeffrey Hatley\, \orcidlink{0000-0002-6883-1316}} \address[Hatley]{ Department of Mathematics\\ Union College\\ Bailey Hall 202\\ Schenectady, NY 12308\\ USA} \email{[email protected]} \author[A.~Ray]{Anwesh Ray\, \orcidlink{0000-0001-6946-1559}} \address[Ray]{Chennai Mathematical Institute, H1, SIPCOT IT Park, Kelambakkam, Siruseri, Tamil Nadu 603103, India} \email{[email protected]} \begin{abstract} We study the distribution of ranks of elliptic curves in quadratic twist families using Iwasawa-theoretic methods, contributing to the understanding of Goldfeld's conjecture. Given an elliptic curve $ E/\mathbb{Q} $ with good ordinary reduction at $ 2 $ and $\lambda$-invariant $ \lambda_2(E/\mathbb{Q}) = 0 $, we use Matsuno's Kida-type formula to construct quadratic twists $ E^{(d)} $ such that $ \lambda_2(E^{(d)}/\mathbb{Q}) $ remains unchanged or increases by $ 2 $. When the root number of $E^{(d)}$ is $-1$ and the Tate-Shafarevich group $ \Sh(E^{(d)}/\mathbb{Q})[2^\infty] $ is finite, this yields quadratic twists with Mordell--Weil rank $ 1 $. These results support the conjectural expectation that, on average, half of the quadratic twists in a family have rank $ 0 $ and half have rank $ 1 $. In the cases we consider we obtain asymptotic lower bounds for the number of twists by squarefree numbers $d\leq X$ which match with the conjectured value up to an explicit power of $\log X$. Our results also apply to twists by a given prime number and are effective. They complement recent groundbreaking results of Smith on Goldfeld's conjecture. \end{abstract} \subjclass[2010]{11G05, 11R23 (primary); 11R45 (secondary).} \keywords{Kida's formula, Iwasawa theory, Selmer groups, elliptic curves.} \maketitle \section{Introduction} \label{section:intro} \par Given an elliptic curve $E/\Q$, the Mordell-Weil theorem assures that the group of rational points $E(\Q)$ is finitely generated, so that \[ E(\Q) \simeq \Z^{\ralg(E)} \oplus T \] for some finite group $T$ and some non-negative integer $\ralg(E)$, called the (algebraic) rank of $E$. The distribution of the ranks $\ralg(E)$ as $E$ varies over all elliptic curves has motivated an enormous amount of research over the past few decades, yet many of their basic properties are still not well understood. For instance, until very recently, a folklore conjecture held that the ranks of elliptic curves over $\Q$ are unbounded; however, recent heuristics have challenged this belief~\cite{bklhpr,alr-ranks}. While the existence of an upper bound for ranks has become highly-debated, mathematicians still largely believe that, whether elliptic curves of arbitrarily large rank exist, they are extremely rare. More precisely, a conjecture originating with Katz and Sarnak~\cite{katz-sarnak} asserts that, asymptotically, half of all elliptic curves over $\Q$ have rank 0 and half have rank 1. It is thus reasonable to say that, conjecturally, the \textit{average rank} of all elliptic curves over $\Q$ is $1/2$. Recent progress on this conjecture has been encouraging; for instance, Bhargava and Shankar~\cite{bhargava-shankar} prove that the average rank is indeed bounded, and that it is bounded by at most $3/2$. One may also study the average ranks of elliptic curves in restricted families. For instance, if $E:y^2=f(x)$ is an elliptic curve defined over $\Q$, and if $d$ is a squarefree integer, then the corresponding \textit{quadratic twist} $E^{(d)}:dy^2=f(x)$ is also an elliptic curve defined over $\Q$, and there is an isomorphism $E_{/K} \simeq E^{(d)}_{/K}$ upon base change to $K=\Q(\sqrt{d})$. It is natural to study the variation of arithmetic invariants in quadratic twist families, and a conjecture due to Goldfeld~\cite{goldfeld} predicts that the average rank in quadratic twist families is again $1/2$. (See Section~\ref{sec:goldfeld} for a precise statement of Goldfeld's conjecture.) As with the Katz-Sarnak conjecture, progress on Goldfeld's conjecture has been modest. Many authors have instead studied the weaker conjecture that \textit{some positive proportion} of elliptic curves in a quadratic twist family have rank 0, and a positive proportion have rank 1. In the notation of Section~\ref{sec:goldfeld}, this is the assertion that \[ n_{E,r}^*(X)\gg X \] for $r\in\{0,1\}.$ When $r=0$, Ono and Skinner~\cite{OnoSkinner} prove that \[ n_{E,0}^\ast(X)\gg \frac{X}{\log X}, \] and when $E(\Q)[2]=0$, Ono~\cite{OnoCrelle} improves this to \[ n_{E,0}^\ast(X)\gg \frac{X}{\left(\log X\right)^{1-\alpha}} \] for some $\alpha \in (0,1)$ depending on $E$. (See Theorems~\ref{Ono Skinner thm} and ~\ref{thm:ono} for the precise statements.) \par Kriz and Li~\cite{krizli} have established positive density results for both the rank $0$ and rank $1$ subsets in cases where $E(\mathbb{Q})[2] = 0$. However, their results hinge on certain technical conditions related to the properties of Heegner points that arise in $E(K)$ (see Theorem~\ref{thm:kriz-li} for a detailed statement). These conditions, while theoretically significant, pose challenges when attempting to determine their prevalence in practice. For rank $0$ quadratic twists of elliptic curves, Kriz and Nordentoft~\cite{kriz2023horizontal} have achieved unconditional positive density results by leveraging analytic techniques involving $p$-adic $L$-functions. Despite this progress, the extent to which their assumptions hold in general remains unclear in practical scenarios. \par In addition to these approaches, there are several notable conditional results. For instance, Alexander Smith~\cite{smith-selmer} has produced positive density results for elliptic curves with full rational $2$-torsion, contingent on the Birch and Swinnerton-Dyer conjecture. Earlier, Heath-Brown~\cite{heath-brown} obtained related results under the assumption of the generalized Riemann Hypothesis (GRH). More recently, Smith's work~\cite{smith2022distribution} has advanced our understanding of Goldfeld's conjecture, contributing significant breakthroughs to this area. It is worth noting, however, that the methods used in these works differ substantially from ours. Our approach yields effective results that are qualitatively distinct and our methods are different. \par In the present paper, we contribute to this body of research using a novel Iwasawa-theoretic approach. Kenkichi Iwasawa initiated the study of arithmetic objects over $\Z_p$-extensions of number fields, obtaining striking results on the uniform growth of ideal class groups along such towers; namely, that if $K$ is a number field and \[ K=K_0\subset K_1\subset \dots \subset K_n \subset K_{n+1}\subset \dots \subset K_\infty=\bigcup_n K_n \] is a $\Z_p$-extension for some prime $p$, then there exist constants $c$, $\mu$ and $\lambda$ (depending only on $K_\infty / K)$ such that the $p$-adic valuation of the class number of $K_n$ is given by \[ \mu p^n + \lambda n + c \] for $n\gg0$. This perspective was generalized to the study of Selmer groups of elliptic curves by Mazur \cite{mazurinventiones}. The details are recalled in Section~\ref{sec:iw-thy-ell}, but we sketch the main ideas here. Given a number field $K$ and a $\Z_p$-extension $K_\infty/K$ as above, one obtains a Selmer group $\op{Sel}_{p^\infty}(E/K_\infty)$ fitting into an exact sequence \begin{equation}\label{eq:intro-selmer} 0\rightarrow E(K_\infty)\otimes \Q_p/\Z_p\rightarrow \op{Sel}_{p^\infty}(E/K_\infty)\rightarrow \lim_{n\rightarrow \infty}\Sh(E/K_n)[p^\infty]\rightarrow 0. \end{equation} Here $\Sh(E/K_n)$ is the Tate-Shafarevich group defined in \eqref{eq:ts}. Moreover, $\op{Sel}_{p^\infty}(E/K_\infty)$ cofinitely generated as a $\Lambda:=\Z_p\llbracket\Gal(K_\infty/K)\rrbracket$-module. If it is furthermore $\Lambda$-cotorsion, then it has associated invariants $\mu_p(E/K)$ and $\lambda_p(E/K)$ which play a role analogous to the constants $\mu$ and $\lambda$ in Iwasawa's results on class groups. In particular, the $\lambda$-invariant gives information on the growth of the rank along the $\Z_p$-extension. In particular, there is a chain of inequalities \[\ralg(E)\leq \op{corank}_{\Z_p} \op{Sel}_{p^\infty}(E/\Q)\leq \lambda_p(E/\Q).\] In fact, if $\Sh(E/\Q)[2^\infty]$ is finite, then the first inequality is actually an equality. Our strategy is to study the variation of the $2$-adic invariants $\lambda_2(E^{(d)}/\Q)$ in quadratic twist families via a Kida-type formula due to Matsuno~\cite{Matsuno-2}. In our context, it follows from results of Kato \cite{kato} and Rubin \cite{RubinBSDCM} that $\op{Sel}_{2^\infty}(E^{(d)}/\Q_\infty)$ is cotorsion over $\Lambda$, cf. Lemma \ref{lemma on Selmer corank and lambda} for further details. Fix an elliptic curve $E/\Q$ with good ordinary reduction at $2$ for which $\lambda_2(E/\Q)=0$. Matsuno's formula (cf. Theorem \ref{thm:kida-matusno-2}) allows us to construct squarefree integers $d$ such that \[ \lambda_2(E^{(d)}/\Q)=\lambda_2(E/\Q), \] from which it follows that $\ralg(E^{(d)})=0$. Similarly, we can construct squarefree integers $d$ such that \[ \lambda_2(E^{(d)}/\Q)-\lambda_2(E/\Q)=2; \] further assumptions on the prime divisors of $d$ then allow us to invoke known cases of the parity conjecture to deduce that $\op{corank}_{\Z_p} \op{Sel}_{p^\infty}(E/\Q)$ is odd. In particular, if $\Sh(E^{(d)}/\Q)[2^\infty]$ is finite, then $\ralg(E^{(d)})=1.$ We also give explicit density results on the sets of admissable squarefree integers $d$ in these constructions. Our results may be summarized as follows. We note that our results on $\lambda$-invariants and Selmer coranks are unconditional, and that they yield results on ranks (in the direction of Goldfeld's conjecture) when the Shafarevich-Tate group is finite. For $X>0$, let $n_{E, k}'(X)$ denote the number of positive squarefree integers $d<X$ such that $\op{corank}_{\Z_2}\op{Sel}_{2^\infty}(E^{(d)}/\Q)=k$. Denote by $\omega(E)$ the sign of the functional equation of $E$. \begin{lthm}[Theorem \ref{n'E,1}]\label{thm A} Let $E$ be an elliptic curve over $\Q$ and assume that the following conditions are satisfied: \begin{enumerate} \item $E$ has good ordinary reduction at $2$ with squarefree conductor $N_E$, \item $\omega(E)=-1$, \item $E(\Q)[2]=0$, \item $\mu_2(E/\Q)=0$ and $\lambda_2(E/\Q)\leq 2$. \end{enumerate} Then we find that \[n_{E, 1}'(X)\gg \frac{X}{(\log X)^{\frac{11}{12}}}.\] \end{lthm} In fact, we are able to obtain an effective version of the above result, see Proposition \ref{corank 1 lemma}. This allows us to pin down squarefree products $d=\ell_1\dots \ell_k$ in terms of Chebotarev conditions on the primes $\ell_i$. In order to demonstrate the effectiveness of this approach, one can also specialize to twist families where $\ell$ is a prime number. We obtain the following result. \begin{lthm}[Theorem \ref{them prime twist 3.13}]\label{thm a'} Let $E$ be an elliptic curve satisfying the conditions of Theorem \ref{thm A}. Let $\mathcal{M}$ be the set of prime numbers $\ell$ such that \begin{itemize} \item $\ell\equiv 1\pmod{4}$, \item $\widetilde{E}(\F_\ell)[2]=0$ (where $\widetilde{E}$ denotes the reduced curve at $\ell$), \item $\ell$ splits completely in $\Q(i, \sqrt{-N_E})$. \end{itemize} Then $\mathcal{M}$ has density $\geq \frac{1}{12}$ and $\op{corank}_{\Z_2}\left( \op{Sel}_{2^\infty}(E^{(\ell)}/\Q)\right)=1$. \end{lthm} It is thus natural to wonder how often the conditions of Theorems \ref{thm A} and \ref{thm a'} are satisfied. In order to make sense of this question, we note that any elliptic curve $ E_{/\Q} $ has a unique globally minimal Weierstrass equation. That is, $ E $ is isomorphic to a unique curve $ E_{A,B} $ with minimal Weierstrass equation \[ E_{A, B}: y^2 = x^3 + Ax + B, \] where $ A, B \in \Z $ and $\gcd(A^3, B^2)$ is not divisible by $ d^{12} $ for any $ d > 1 $. Let $ \mathcal{C} $ be the set of all such equations. The height of $ E $ is defined as $ \operatorname{ht}(E) := \max\{|A|^3, B^2\} $. For $ X > 0 $, let $ \mathcal{C}(X) $ be the set of all $ E_{A,B} $ with $ \operatorname{ht}(E_{A,B}) \leq X $. Given a set of (isomorphism classes of) elliptic curves $\mathcal{S}\subset \cC$, say that $\mathcal{S}$ has positive density if the limit \[\liminf_{X\rightarrow \infty} \frac{\# \{E_{A, B}\in \mathcal{S}\mid \operatorname{ht}(E_{A,B}) \leq X \}}{\# \cC(X)}>0.\] It is natural to expect that the conditions of Theorem \ref{thm A} are satisfied for a positive density of elliptic curves. Indeed: \begin{description} \item[Condition (1)] depends purely on local properties at all primes. The density of elliptic curves for which $ N_E $ is squarefree is positive; see, for example, \cite{cremonasadek}. \item[Condition (2)] pertains to the rank distribution of elliptic curves. By the rank distribution conjecture of Katz and Sarnak, it is expected that half of all elliptic curves satisfy this condition. \item[Condition (3)] holds for almost all elliptic curves $ E/\mathbb{Q} $, as demonstrated by a result of Duke \cite{Dukeexceptional}. \item[Condition (4)] is more subtle and involves analyzing the truncated Euler characteristic of the Selmer group. Specifically, if this Euler characteristic is not divisible by $ p $, the condition holds. Based on heuristic arguments, it is reasonable to expect that condition (4) is satisfied for a proportion of $(1 - 1/p)$ of all elliptic curves $ E/\mathbb{Q} $. However, establishing this rigorously remains an open problem, even for a positive density of curves. Further details on this matter can be found in \cite{kunduraystats1}. In this context, it was shown by the second author \cite{ray2024statisticsIII} that: \begin{itemize} \item $E$ has good ordinary reduction at $5$, \item $\mu_5(E/\Q)=0$ and $\lambda_5(E/\Q)$. \end{itemize} \end{description} \par Our next result applies to elliptic curves for which $E(\Q)[2]\neq 0$. \begin{lthm}[Theorem \ref{pos density of primes}]\label{thm B} Let $E/\Q$ be an elliptic curve satisfying the following conditions: \begin{enumerate} \item $E$ has good ordinary reduction at $2$ with squarefree conductor $N_E$, \item $\omega(E)=+1$, \item $E(\Q)[2]\neq 0$, \item $\mu_2(E/\Q)=0$ and $\lambda_2(E/\Q)=0$. \end{enumerate} Let $\ell$ be a prime satisfying the following conditions: \begin{itemize} \item $\ell\nmid 2 N_E$, \item $\ell$ is inert in $K:=\Q(\sqrt{-N_E})$, \item $\ell\equiv 3, 5\pmod{8}$. \end{itemize} Then we have that $\op{corank}_{\Z_2} \op{Sel}_{2^\infty}(E^{(\ell)}/\Q)=1$. The set of primes satisfying the above conditions have density $\frac{1}{4}$. \end{lthm} We also study the distribution of $2$-adic $\lambda$-invariants in quadratic twist families, the following result is proven in section \ref{s 4}. Let $ E $ be an elliptic curve defined over $ \mathbb{Q} $ with good ordinary reduction at $ 2 $ and squarefree conductor $ N_E $. Assume that $ E(\mathbb{Q})[2] = 0 $. Consider the family of quadratic twists $ E^{(d)} $, where $ d > 0 $ is a squarefree integer. Define $ m_{E, N}(X) $ to be the number of such $ d $ (with $ d \leq X $) for which the $ \lambda $-invariant of $ E^{(d)} $ over $ \mathbb{Q} $ is $ N $. \begin{lthm}[Theorem \ref{last thm}]\label{Thm D} Assume that $\mu_2(E/\Q)=0$. If $N$ is any integer such that $N\geq \lambda_2(E/\Q)$ and $N\equiv \lambda_2(E/\Q)\pmod{2}$, then we have that \[m_{E, N}(X)\gg X/(\log X)^{\alpha},\] where $\alpha=\begin{cases} \frac{1}{3} & \text{ if }\op{Gal}(\Q(E[2])/\Q)\simeq \Z/3\Z;\\ \frac{2}{3}& \text{ if }\op{Gal}(\Q(E[2])/\Q)\simeq S_3. \end{cases}$ \end{lthm} \par It is natural to consider extending our approach to elliptic curves with supersingular reduction at $2$. In this setting, the standard tools used for ordinary reduction are no longer directly applicable since the Selmer group over the cyclotomic $\Z_2$-extension is not $\Lambda$-cotorsion. As a result, understanding the growth of Selmer groups in the cyclotomic $\mathbb{Z}_2$-extension requires alternative frameworks. The $\sharp$ and $\flat$ Selmer groups introduced by Sprung \cite{sprungsharpflat} provide a refined approach to tackling these challenges. However, the theory of $\sharp$ and $\flat$ Selmer groups for $p = 2$ remains underdeveloped compared to the case of odd primes. Important questions about the structure, growth properties, and $2$-adic $L$-functions associated with these Selmer groups are yet to be fully explored. We expect that our work could invigorate interest in such themes. The interplay between the arithmetic of the elliptic curve and the structure of these Selmer groups might also yield deeper insights into $2$-adic families of modular forms and Galois representations. Given the rich structure inherent in supersingular reduction, one can be optimistic that this area will produce intriguing discoveries that connect with broader themes in number theory, such as $p$-adic Hodge theory, Euler systems, and the Birch and Swinnerton-Dyer conjecture in the $2$-adic context. \section*{Statements and Declarations} \subsection*{Conflict of interest} The authors report there are no conflict of interest to declare. \subsection*{Data Availability} There is no data associated to the results of this manuscript. \section*{Acknowledgements} Partial support for this research was provided by an AMS-Simons Research Enhancement Grant for Primarily Undergraduate Institution Faculty. The second author thanks Union college for hosting him during his visit in October of 2024. The authors thank Tristan Phillips and Asbj\o rn Christian Nordentoft for their helpful comments. \section{Preliminary notions} \subsection{Iwasawa theory of elliptic curves}\label{sec:iw-thy-ell} \par In this section, we recall preliminary notions pertaining to the Iwasawa theory of elliptic curves. First, we must set up some notation. For a field $F$ of characteristic zero, $\op{G}_F$ will denote the absolute Galois group $\op{Gal}(\bar{F}/F)$. Given a prime number $\ell$, choose an embedding $\iota: \bar{\Q}\hookrightarrow \bar{\Q}_\ell$ and let $\op{G}_\ell$ denote the absolute Galois group $\op{Gal}(\bar{\Q}_\ell/\Q_\ell)$. Set $\op{I}_\ell$ to be the inertia subgroup of $\op{G}_\ell$ and let $\op{Frob}_\ell\in \op{G}_\ell/\op{I}_\ell$ be a Frobenius element. The embedding $\iota_\ell$ induces an inclusion $\iota_\ell^*: \op{G}_\ell\hookrightarrow \op{G}_{\Q}$. Throughout, $p$ will be a fixed prime number and $E:y^2=f(x)$ an elliptic curve over $\Q$. Let $\chi$ denote the $p$-adic cyclotomic character and $\omega$ the mod-$p$ reduction of $\chi$. We shall assume throughout that $E$ has potentially good ordinary reduction at $p$. In other words, there is a finite extension $\cK/\Q_p$ such that the base change $E_{/\cK}$ has good ordinary reduction. Given a prime number $\ell$, take $\F_\ell$ to be the field with $\ell$ elements. If $E$ has good reduction at $\ell$, we will denote by $\widetilde{E}(\F_\ell)$ the $\F_\ell$-rational points of the reduction of $E$ at $\ell$. We shall set $E[p^k]$ to denote the $p^k$ torsion points of $E(\bar{\Q})$. Set $E[p^\infty]$ to be the $p$-divisible group of $p$-primary torsion points. Note that $E[p^\infty]$ comes equipped with a natural action of $\op{G}_{\Q}$. Set $\mathbb{T}_p(E)$ to denote the Tate-module, i.e. the inverse limit $\varprojlim_n E[p^n]$ where the inverse limit is taken with respect to multiplication by $p$ maps $\times p: E[p^{n+1}]\rightarrow E[p^n]$. Choose an isomorphism $\mathbb{T}_p(E)\xrightarrow{\sim} \Z_p\oplus \Z_p$, and let \[\rho_{E,p}: \op{G}_{\Q}\rightarrow \op{Aut}_{\Z_p}(\mathbb{T}_p(E))\xrightarrow{\sim} \op{GL}_2(\Z_p)\] be the associated representation. The mod-$p$ reduction of $\rho_{E,p}$ is denoted \[\bar{\rho}_{E,p}:\op{G}_{\Q}\rightarrow \op{GL}_2(\F_p)\] and is identified with the Galois representation on $E[p]$. The condition that $E$ has potentially good ordinary reduction at $p$ implies that there is a finite extension $\cK/\Q_p$ such that $\rho_{E,p}$ restricts to the inertia subgroup of $\op{G}_{\cK}$ to a representation of the form $\mtx{\chi}{\ast}{0}{1}$. Let $d$ be a squarefree integer, denote by $E^{(d)}$ the quadratic twist given by $dy^2=f(x)$. Given a number field $K$, let $E(K)$ denote the Mordell--Weil group of $E$ over $K$. Let $\Omega_K^f$ be the set of finite primes of $K$ and $\Omega_K^\infty$ the archimedian places. We let $\Omega_K$ be the set of all places of $K$, i.e., $\Omega_K:=\Omega_K^f\cup \Omega_K^\infty$. We shall use $H^i(L, \cdot):=H^i(\op{G}_L, \cdot)$ where $L$ denotes $K$ or $K_v$. The Tate--Shafarevich group $\Sh(E/K)$ is defined as \begin{equation}\label{eq:ts} \Sh(E/K):=\op{ker}\left\{H^1(K, E)\rightarrow \prod_{v\in \Omega_K} H^1(K_v, E)\right\}.\end{equation} It is conjectured that the Tate--Shafarevich group $\Sh(E/K)$ is finite. This is known in some special cases. Rubin \cite{RubinShafinite} proved that if $E$ is a CM elliptic curve with analytic rank at most $1$, then $\Sh(E/\Q)$ is finite. Kolyvagin \cite{Kolyvagin} extended this to all elliptic curves $E_{/\Q}$ of analytic rank at most $1$. \par Mazur \cite{mazurinventiones} initiated the Iwasawa theory of elliptic curves by studying growth properties of Selmer groups in cyclotomic $\Z_p$-extensions. Selmer groups consist of Galois cohomology classes which satisfy local conditions at all primes. Let us recall their definition and properties in some detail. From the Kummer sequence of $\op{G}_{L}$-modules \[0\rightarrow E(\bar{L})[p^n]\rightarrow E(\bar{L})[p^\infty]\xrightarrow{\times p^n} E(\bar{L})[p^\infty]\rightarrow 0,\] we obtain the Kummer map \[\kappa_{E,L}^{(n)}: \frac{E(L)}{p^n E(L)}\hookrightarrow H^1(L, E[p^n]). \] Passing to the direct limit in $n$, one obtains: \begin{equation}\label{kapps E L defn}\kappa_{E,L}: E(L)\otimes \Q_p/\Z_p\hookrightarrow H^1(L, E[p^\infty]).\end{equation} \begin{definition} The $p$-primary Selmer group $\op{Sel}_{p^\infty}(E/K)$ consists of all classes $\alpha\in H^1(K, E[p^\infty])$ such that $\alpha_{|v}\in \op{image}\kappa_{E, K_v}$ for all primes $v\in \Omega_K$. \end{definition} The Selmer group $\op{Sel}_{p^\infty}(E/K)$ is cofinitely generated as a $\Z_p$-module, and it fits into a natural short exact sequence \begin{equation}\label{ses} 0\rightarrow E(K)\otimes \Q_p/\Z_p\xrightarrow{\kappa_{E,K}} \op{Sel}_{p^\infty}(E/K)\rightarrow \Sh(E/K)[p^\infty]\rightarrow 0.\end{equation} Let $K_\infty$ denote the cyclotomic $\Z_p$-extension of $K$, i.e., the unique extension contained in $K(\mu_{p^\infty})$ for which $\op{Gal}(K_\infty/K)\xrightarrow{\sim} \Z_p$ as a topological group. We set $\Gamma:=\op{Gal}(K_\infty/K)$ and choose a topological generator $\gamma\in \Gamma$. Let $K_n$ be the $n$-th layer of $K_\infty$, defined to be the subextension of $K_\infty$ such that $[K_n:K]=p^n$. View \[K=K_0\subset K_1\subset \dots \subset K_n \subset K_{n+1}\subset \dots \subset K_\infty=\bigcup_n K_n\] as a tower of extensions and identify $\op{Gal}(K_n/K)$ with $\Gamma_n:=\Gamma/\Gamma^{p^n}$.\begin{definition} With respect to notation above, the Iwasawa algebra is then taken to be the completed group ring $\Lambda:=\varprojlim_n \Z_p[\Gamma/\Gamma^{p^n}]$. \end{definition} Setting $T:=(\gamma-1)\in \Lambda$, we identify $\Lambda$ with the formal power series ring $\Z_p\llbracket T\rrbracket$. A monic polynomial $f(T)\in \Z_p\llbracket T\rrbracket$ is said to be a \emph{distinguished polynomial} if its nonleading coefficients are all divisible by $p$. Let $M$ be a module over $\Lambda$, set $M^\vee:=\op{Hom}_{\Z_p}(M, \Q_p/\Z_p)$ to be the Pontryagin dual of $M$. Then $M$ is said to be \emph{cofinitely generated} (resp. \emph{cotorsion}) over $\Lambda$ if $M^\vee$ is finitely generated (resp. torsion) over $\Lambda$. Let $M$ and $M'$ be cofinitely generated and cotorsion $\Lambda$-modules. Then, $M$ and $M'$ are said to be \emph{pseudo-isomorphic} if there is a map $\phi: M\rightarrow M'$ of $\Lambda$-modules, whose kernel and cokernel are finite. By the structure theory of finitely generated and torsion $\Lambda$-modules, any cofinitely generated and cotorsion $\Lambda$-module $M$ is pseudo-isomorphic to a module $M'$ whose Pontryagin dual is given as a direct sum of cyclic torsion $\Lambda$-modules \begin{equation}\label{cyclic isomorphism}(M')^\vee\simeq \left(\bigoplus_{i=1}^s \frac{\Lambda}{(p^{n_i})}\right)\oplus \left(\bigoplus_{j=1}^t \frac{\Lambda}{(f_j(T))}\right).\end{equation} Here, $s$ and $t$ are natural numbers (possibly $0$), $n_i \in \Z_{\geq 1}$ and $f_j(T)$ are distinguished polynomials. Then, one defines the Iwasawa $\mu$- and $\lambda$-invariants by \[\mu_p(M):=\sum_{i=1}^s n_i \quad \text{ and } \quad \lambda_p(M):= \sum_{j=1}^t \op{deg}f_j.\] Here, if $s=0$ (resp. $t=0$), the sum $\sum_{i=1}^s n_i$ (resp. $\sum_{j=1}^t \op{deg}f_j$) is interpreted as being equal to $0$. \begin{lemma} Let $M$ be a cofinitely generated and cotorsion $\Lambda$-module. Then $\mu_p(M)=0$ if and only if $M$ is cofinitely generated as a $\Z_p$-module, i.e., $M\simeq (\Q_p/\Z_p)^\lambda\oplus T$ where $T$ is finite. The quantity $\lambda$ is the $\Z_p$-corank of $M$ and is given by $\lambda=\lambda_p(M)$. \end{lemma} \begin{proof} The result above is an easy consequence of the structural decomposition \eqref{cyclic isomorphism} and its proof is omitted. \end{proof} \par The Selmer group $\op{Sel}_{p^\infty}(E/K_\infty)$ is defined to be the direct limit $\varinjlim_n \op{Sel}_{p^\infty} (E/K_n)$ and is a cofinitely generated module over $\Lambda$. Moreover, when $E$ has good ordinary reduction at $p$ and $K/\Q$ is an abelian extension, it is known due to results of Kato \cite{kato} and Rubin \cite{RubinBSDCM} that $\op{Sel}_{p^\infty}(E/K_\infty)$ is cotorsion over $\Lambda$. \begin{definition}If $E$ is an elliptic curve over $\Q$ and $K$ is a number field for which $\op{Sel}_{p^\infty}(E/K_\infty)$ is cotorsion over $\Lambda$, we write $\mu_p(E/K)$ and $\lambda_p(E/K)$ to denote the associated $\mu$-invariant (resp. $\lambda$-invariant). \end{definition} It is conjectured by Greenberg \cite[Conjecture 1.11]{GreenbergITEC} that if $E_{/\Q}$ is an elliptic curve with good ordinary reduction at $p$ such that $\rho_{E, p}$ is irreducible, then $\mu_p(E/\Q)=0$. We end this subsection with a criterion due to Greenberg for the entire Selmer group $\op{Sel}_{p^\infty}(E/\Q_\infty)$ to vanish. At a prime number $\ell$, let $c_\ell(E)$ be the \emph{Tamagawa number} defined as $c_\ell(E):=[\mathscr{E}(\Q_\ell): \mathscr{E}_0(\Q_\ell)]$, where $\mathscr{E}_0$ is the connected component of the Neron model $\mathscr{E}$ associated to $E$ over $\Q_\ell$. \begin{proposition}[Greenberg]\label{main prop} Let $E$ be an elliptic curve and $p$ be a prime number. Assume that: \begin{enumerate} \item $E$ has good ordinary reduction at $p$, \item $\op{Sel}_p(E/\Q)=0$, \item $p\nmid \# \widetilde{E}(\F_p)$. \item $p\nmid c_\ell(E)$ for all primes $\ell\neq p$. \end{enumerate} Then $\op{Sel}_{p^\infty}(E/\Q_{\infty})=0$, and in particular, \[\mu_p(E/\Q)=0\text{ and }\lambda_p(E/\Q)=0. \] \end{proposition} \begin{proof} The result follows from \cite[remark following Proposition 3.8, p.80, ll. 19--26]{GreenbergITEC}. \end{proof} \subsection{A formula of Hachimori and Matsuno} \par Hachimori and Matsuno \cite{hachimori-matsuno} proved a formula for the growth of Iwasawa invariants in a finite $p$-extension, generalizing Kida's formula from classical Iwasawa theory. Let $L/K$ be a finite Galois extension whose Galois group $G:=\op{Gal}(L/K)$ is a $p$-group. Let $S_{\op{add}}$ be the set of primes of $K$ at which $E$ has additive reduction. Following Hachimori and Matsuno, in this subsection we temporarily make the following additional assumptions: \begin{itemize} \item[(\mylabel{hyp:Hyp1}{\textbf{Hyp1}})] If $p=2$, then $K$ is totally imaginary. \end{itemize} \begin{itemize} \item[(\mylabel{hyp:Hyp2}{\textbf{Hyp2}})] $E$ has additive reduction at all primes of $L_\infty$ lying above $S_{\op{add}}$. \end{itemize} We shall briefly recall their result now. \begin{theorem}[Hachimori--Matsuno]\label{thm:kida-hachimori-matsuno} With respect to the notation above, assume that the following conditions are satisfied: \begin{enumerate} \item $E$ has good ordinary reduction at $p$ and $K/\Q$ an abelian extension. \item The conditions \eqref{hyp:Hyp1} and \eqref{hyp:Hyp2} hold, \item $\mu_p(E/K)=0$. \end{enumerate} The following statements hold: \begin{enumerate} \item The Selmer group $\operatorname{Sel}_{p^\infty}(E/L_\infty)$ is cofinitely generated and cotorsion over $\Lambda$ with $\mu_p(E/L)=0$. \item We have the equation: \[ \lambda_p(E/L)=[L_\infty:K_\infty] \lambda_p(E/K)+\sum_{w\in P_1(E)} \left(e(w)-1\right)+2 \sum_{w\in P_2(E)} \left(e(w)-1\right). \] Here $P_1(E)$ and $P_2(E)$ are sets of primes in $L_\infty$ defined as: \[ \begin{split} & P_1(E) := \{w \mid w \nmid p, \text{ $E$ has split multiplicative reduction at } w\}, \\ & P_2(E) := \{w \mid w \nmid p, \text{ $E$ has good reduction at } w \text{ and } E(L_{\infty, w}) \text{ has a point of order } p\}, \end{split} \] \end{enumerate} and $e(w)=e_{L_\infty/K_\infty}(w)$ is the ramification index of $w$ over $K_\infty$ \end{theorem} \subsection{Goldfeld's conjecture}\label{sec:goldfeld} \par Given an elliptic curve $E:y^2=f(x)$ over $\Q$, denote by $\ralg(E)$ the rank of the Mordell--Weil group $E(\Q)$. On the other hand, set $\ran(E)$ to denote the analytic rank, i.e., the order of the zero of the Hasse--Weil L-function $L_E(s)$ at $s=1$. The weak form of the Birch and Swinnerton--Dyer conjecture predicts that $\ralg(E)=\ran(E)$. By the theorems of Gross--Zagier and Kolyvagin, this conjecture is known whenever $\ran(E)\in \{0, 1\}$. It is natural to study the distribution of $\ralg(E)$ and $\ran(E)$ in families of elliptic curves $E_{/\Q}$. A natural family of elliptic curves is the quadratic twist family, which is defined for any elliptic curve $E_{/\Q}$. Given a squarefree integer $d$, let $E^{(d)}$ denote the quadratic twist of $E$ by $d$, given by $E^{(d)}:dy^2=f(x)$. The family of twists $\{E^{(d)}\}$ is ordered according to $|d|$. The parity of $\ran(E)$ is determined by the root number, and as a general principal, one expects that on average, $\ran(E)$ is as small as possible. For $X>0$ and $r\in \Z_{\geq 0}$, set $n_{E,r}^{\ast}(X):=\#\{d\mid \ast(E^{(d)})=r, |d|< X\}$ where $*\in \{\op{alg}, \op{an}\}$ and $d$ refers to a squarefree integer. \begin{conj}[Goldfeld's conjecture] With respect to notation above for $r\in \{0,1\}$, \[n_{E,r}^{\ast}(X)\sim \frac{1}{2} \sum_{|d|<X} 1,\] where the sum runs over fundamental discriminants $d$. \end{conj} The conjecture implies that for $r\geq 2$, we have that $n_{E,r}^*(X)=o(X)$. Moreover, it implies that for $r \in \{0,1\}$, we have $n_{E,r}^*(X)\gg X$. We note that the conjecture for $\ast=\op{an}$ implies the conjecture for $\ast=\op{alg}$. We now recall some of the strongest results shedding light on the conjecture due to Ono--Skinner \cite{OnoSkinner} and Ono \cite{OnoCrelle} which hold for elliptic curves in general. \begin{theorem}[Ono--Skinner]\label{Ono Skinner thm} Let $E_{/\Q}$ be an elliptic curve, then the following assertions hold: \begin{enumerate} \item $n_{E,0}^\ast(X)\gg \frac{X}{\log X}$ for $\ast\in \{\op{an}, \op{alg}\}$. \item If the conductor of $E$ is $\leq 100$ then there is a positive density of primes $p$ for which the twist $E^{(p)}$ or $E^{(-p)}$ has rank $0$. \end{enumerate} \begin{proof} Part (1) for $\ast=\op{an}$ is \cite[Corollary 3]{OnoSkinner}. The result for $\ast=\op{alg}$ follows from that for $\ast=\op{an}$. Part (2) is \cite[Corollary 2]{OnoSkinner}. \end{proof} \end{theorem} \begin{theorem}[Ono]\label{thm:ono} Let $E$ be an elliptic curve over $\Q$ such that $E(\Q)[2]=0$. Then there is $\alpha(E)\in (0, 1)$ such that $n_{E,0}^\ast(X)\gg \frac{X}{(\log X)^{1-\alpha(E)}}$ for $\ast\in \{\op{an}, \op{alg}\}$. \end{theorem} \begin{proof} This result is \cite[Corollary 3]{OnoCrelle}. \end{proof} Results of a different flavor due to Kriz--Li \cite{krizli} hold for a special class of elliptic curves. Let $E$ be an elliptic curve over $\Q$ with conductor $N$ and $K/\Q$ be an imaginary quadratic field satisfying the Heegner hypothesis. This means that each prime $\ell|N$ splits in $K$. Let $P\in E(K)$ be a Heegner point and $\pi_E: X_0(N)\rightarrow E$ be the modular parametrization. Let $\omega_E\in H^0(E, \Omega^1)$ be such that $\pi_E^*(\omega_E)= f_E(q) \frac{d q}{q}$ where $f_E(q)$ is the modular form associated to $E$. Let $\log \omega_E$ denote the formal logarithm associated to $\omega_E$. Take $\Q(E[2])$ to be the field fixed by the kernel of $\bar{\rho}_{E,2}$ and identify $\op{Gal}(\Q(E[2])/\Q)$ with the image of $\bar{\rho}_{E,2}$. \begin{theorem}[Kriz--Li]\label{thm:kriz-li} With respect to the notation above, assume that: \begin{enumerate} \item $2$ splits in $K$, \item $(E, K)$ satisfies the Heegner hypothesis, \item $\frac{\# \widetilde{E}^{\op{ns}}(\F_2) \cdot \log_{\omega_E}(P)}{2}\not\equiv 0\pmod{2}$ where $\widetilde{E}$ denotes the reduction of $E$ at $2$. \end{enumerate} Then for $r\in \{0, 1\}$ and $*\in \{\op{an}, \op{alg}\}$ we have that \[n_{E,r}^*(X)\gg \begin{cases} & \frac{X}{(\log X)^{5/6}} \text{ if }\op{Gal}(\Q(E[2])/\Q)\simeq S_3;\\ & \frac{X}{(\log X)^{2/3}} \text{ if }\op{Gal}(\Q(E[2])/\Q)\simeq \Z/3\Z. \end{cases}\] \end{theorem} \begin{proof} The result is \cite[Theorem 1.12]{krizli}. \end{proof} \subsection{Analytic ingredients} \par In this section, we discuss tauberian theorems and consequences which will come in handy in proving our density results. Let $F(X)$ and $G(X)$ be non-negative functions of a variable $X\in \mathbb{R}_{\geq 0}$. Assume that $G(X)>0$ for large enough values of $X$, Then we write $F(X)\sim G(X)$ to mean that $\lim_{X\rightarrow \infty} \frac{F(X)}{G(X)}=1$. On the other hand, write $F(X)\gg G(X)$ to mean that $F(X)>c G(X)$ for some positive constant $c$ which is independent of $X$. Let us begin with the standard Delange's tauberian theorem. \begin{theorem}[Delange's tauberian theorem]\label{delange} Let $f(s):=\sum_{n=1}^\infty a_n n^{-s}$ be a Dirichlet series with non-negative coefficients and $a>0$ be a real number. Assume that $f(s)$ converges for $\op{Re}(s)>a$ and has a meromorphic continuation to a neighbourhood $U$ of $\op{Re}(s)\geq a$. For $X>0$, we set $g(X):=\sum_{n\leq X} a_n$. Assume that the only pole of $f(s)$ is at $s=a$ and the order of this pole is $b\in \mathbb{R}_{>0}$, i.e., \[f(s)=\frac{1}{(s-a)^b} h(s)\] for some holomorphic function $h(s)$ defined on $U$. Then, there is a positive constant $c>0$ such that, as $X\rightarrow \infty$, we have \[g(X)\sim c X^a (\log X)^{b-1}.\] \end{theorem} A set of prime numbers $\Omega$ is said to have a natural density $\mathfrak{d}(\Omega)$ if the following limit exists: \[\mathfrak{d}(\Omega):=\lim_{X\rightarrow\infty} \frac{\#\{\ell\in \Omega\mid \ell\leq X\}}{\pi(X)}.\] On the other hand, $\Omega$ is said to have Dirichlet density if the following limit exists \[\mathfrak{d}'(\Omega):=\lim_{s\rightarrow 1^+}\frac{\sum_{\ell\in \Omega}\ell^{-s}}{\sum_{\ell}\ell^{-s}},\] where in the denominator, the sum is over all prime numbers. If the natural density exists, then so does the Dirichlet density and moreover, $\mathfrak{d}(\Omega)=\mathfrak{d}'(\Omega)$. In this article, the word density for a set of primes shall refer to their natural density. Associated to $\Omega$ is the set of square-free natural numbers $N_\Omega$ consisting of $d$ divisible only by primes $\ell\notin \Omega$. We set $N_\Omega(X):=N_\Omega \cap [1,X]$ and $n_\Omega(X):=\# N_\Omega(X)$. \begin{proposition}\label{density propn for n_Omega} Let $\Omega$ be a set of primes with positive density $\alpha\in (0, 1)$, then $n_\Omega(X)\sim c X/(\log X)^{\alpha}$, where $c>0$ is a constant. \end{proposition} \begin{proof} The result follows from \cite[Theorem 2.4, p.5 line -3 to p.6 line -10.]{serredivisibilite}. We sketch the proof here, and for further details, we refer to \emph{loc. cit.} Let $\Omega^c$ be the complement of $\Omega$ and \[\begin{split}f(s) :=\sum_{n\in N_\Omega} n^{-s} & =\sum_{T\subset \Omega^c} \left(\prod_{\ell \in T} \ell\right)^{-s} \\ & = \prod_{\ell\notin \Omega}\left(1+\ell^{-s}\right).\end{split}\] It is easy to see that \[\log f(s)=\sum_{\ell\notin \Omega} \ell^{-s}+k_1(s),\] where $k_1(s)$ is holomorphic on $\op{Re}(s)\geq 1$; and as a consequence, \[\log f(s) =(1-\alpha) \log\left(\frac{1}{s-1}\right)+k_2(s), \] where $k_2(s)$ is holomorphic on $\op{Re}(s)\geq 1$. Thus, we deduce that \[f(s)=(s-1)^{\alpha-1}h(s),\] where $h(s)$ is a non-zero holomorphic function on $\op{Re}(s)\geq 1$. It follows from the Theorem \ref{delange} that \[n_\Omega(X)\sim c X (\log X)^{-\alpha},\] where $c>0$ is a constant that does not depend on $X$. \end{proof} \section{Selmer coranks in quadratic twist families} As in Section~\ref{sec:goldfeld}, let $E$ be an elliptic curve defined over $\Q$, and for any squarefree integer $d$, let $E^{(d)}$ denote its quadratic twist by $d$. In this section, we seek to study the Iwasawa invariants of $E^{(d)}$ in terms of those of $E$. \begin{lemma}\label{lemma on Selmer corank and lambda} Let $E$ be an elliptic curve with good ordinary reduction at $2$ and $d$ be a squarefree integer. Then the following assertions hold: \begin{enumerate} \item $\op{Sel}_{2^\infty}(E^{(d)}/\Q_\infty)$ is cotorsion over $\Lambda$. Consequently the Iwasawa invariants $\mu_2(E^{(d)}/\Q)$ and $\lambda_2(E^{(d)}/\Q)$ are well defined. \item $\op{corank}_{\Z_2} \op{Sel}_{2^\infty}(E^{(d)}/\Q)\leq \lambda_2(E^{(d)}/\Q)$. \end{enumerate} \end{lemma} \begin{proof} We note that $E^{(d)}$ is isomorphic to $E$ over the quadratic extension $K:=\Q(\sqrt{d})$. Since $E$ has good ordinary reduction at $2$, the same is true of $E^{(d)}_{/K}$. It then follows that $\op{Sel}_{2^\infty}(E/K_\infty)$ is cotorsion over $\Lambda$. Equivalently, $\op{Sel}_{2^\infty}(E^{(d)}/K_\infty)$ has the same property. In particular, $\op{Sel}_{2^\infty}(E^{(d)}/\Q_\infty)$ is cotorsion over $\Lambda$, thus completing the proof of (1). \par It follows from \cite[Theorem 1.1]{Howson} that \[\begin{split}& \op{corank}_{\Z_2} H^1(\Gamma, E^{(d)}(\Q_\infty)[2^\infty])\\ =& \op{corank}_{\Z_2} H^0(\Gamma, E^{(d)}(\Q_\infty)[2^\infty]) \\ =& \op{corank}_{\Z_2}E^{(d)}(\Q)[2^\infty]=0.\end{split}\] Therefore, $H^1(\Gamma, E^{(d)}(\Q_\infty)[p^\infty])$ is finite. The natural restriction map \[\op{Sel}_{2^\infty}(E^{(d)}/ \Q)\rightarrow \op{Sel}_{2^\infty}(E^{(d)}/\Q_\infty)\] has finite kernel since $H^1(\Gamma, E^{(d)}(\Q_\infty)[p^\infty])$ is finite. This proves part (2). \end{proof} We note that part (2) of Lemma \ref{lemma on Selmer corank and lambda} implies that $\ralg(E^{(d)})\leq \lambda_2(E^{(d)}/\Q)$. If the $2$-primary part of $\Sh(E^{(d)}/\Q)$ is finite then $\ralg(E)=\op{corank}_{\Z_2} \op{Sel}_{2^\infty}(E^{(d)}/\Q_\infty)$. By aforementioned results of Kato and Rubin, this is known if $E^{(d)}$ has analytic rank at most $1$. We now recall a formula of Matsuno which relaxes the hypotheses of Theorem~\ref{thm:kida-hachimori-matsuno} when $p=2$~\cite[Theorem 5.1]{Matsuno-2}. Although Matsuno's result can be stated more generally, we will use a specialized version where $d$ is assumed to be coprime to the conductor $N_E$ of the elliptic curve $E$. Since we are primarily interested in proving distribution results, this condition on $d$ does not make a significant difference. \begin{theorem}[Matsuno]\label{thm:kida-matusno-2}Let $E/\Q$ be an elliptic curve with good ordinary reduction at $p=2$ with squarefree conductor $N_E$. Assume that $\mu_2(E/\Q)=0$. Let $d>0$ be a squarefree integer coprime to $N_E$ and let $E^{(d)}$ be the corresponding quadratic twist. Then \begin{equation}\label{eq:kida-matsuno-formula} \lambda_2(E^{(d)}/\Q) = \lambda_2(E/\Q) + \sum_{\substack{\ell \mid d \\ 2 \mid \# \tilde{E}(\F_2)}} 2^{n_\ell + 1} \end{equation} where the sum runs over the odd prime divisors of $d$ and where $n_\ell = \mathrm{ord}_2 \left( \frac{\ell^2 - 1}{8} \right)$. \end{theorem} We recall from Lemma \ref{lemma on Selmer corank and lambda} that \begin{equation}\label{effective bounds after Matsuno} \ralg(E) \leq \operatorname{corank}_{\mathbb{Z}_2} \operatorname{Sel}_{2^\infty}(E^{(d)}/\mathbb{Q}) \leq \lambda_2(E^{(d)}/\mathbb{Q}). \end{equation} Our strategy will be to control the parity of the middle term for a specific subfamily of twists $d$. As a consequence of Theorem \ref{thm:kida-matusno-2}, we can choose these twists such that $\lambda_2(E^{(d)}/\mathbb{Q})$ is effectively bounded. Let $\omega(E)$ denote the sign of the functional equation of the Hasse–Weil $L$-function of $E$ over $\mathbb{Q}$. The $p$-parity conjecture over $\Q$ is known due to Dokchitser and Dokchitser \cite[Theorem 1.4]{dok-dok}. This will be central to our approach. \begin{theorem}[Dokchitser--Dokchitser]\label{dok-dok thm} Let $p$ be a prime number. Then the quantity $s_p(E):=\op{corank}_{\Z_p}\op{Sel}_{p^\infty}(E/\Q)$ is even if and only if $\omega(E)=+1$. \end{theorem} As a result for $E^{(d)}$, $\op{corank}_{\Z_2} \op{Sel}_{2^\infty}(E/\Q)=1$ follows as a consequence of bounding $\lambda_2(E^{(d)}/\Q)$ by $2$ and the parity of the root number $\omega(E)$. \begin{corollary} Suppose that $E^{(d)}$ is a twist of $E$ such that: \begin{enumerate} \item $\lambda_2(E^{(d)}/\Q)\leq 2$, \item $\omega(E^{(d)})=-1$. \end{enumerate} Then we find that $\op{corank}_{\Z_2} \op{Sel}_{2^\infty}(E/\Q)=1$. \end{corollary} \begin{proof} Note that Theorem \ref{dok-dok thm} implies that $s_2(E)$ is odd. On the other hand, Lemma \ref{lemma on Selmer corank and lambda} asserts that \[s_2(E)\leq \lambda_2(E^{(d)}/\Q)\leq 2.\] We thus deduce that $s_2(E)=1$. \end{proof} Throughout the rest of this section, fix an elliptic curve $E_{/\Q}$ which satisfies the following conditions: \begin{itemize} \item $E$ has good ordinary reduction at $p=2$, \item the conductor $N_E$ is squarefree, \item $\mu_2(E/\Q)=0$. \end{itemize} Note that since $E$ is assumed to have good ordinary reduction, the $\mu$ and $\lambda$-invariants are indeed well-defined. \begin{definition} Let $\Omega=\Omega_E$ be the set of rational primes $\ell$ such that $\ell\nmid 2N_E$ and $\widetilde{E}(\F_\ell)[2]\neq 0$. \end{definition} Given a prime $\ell\nmid 2N_E$, we find that $\ell\in \Omega$ if and only if $\op{trace}\left(\bar{\rho}_{E,2}(\op{Frob}_\ell)\right)$ is even. Thus by the Chebotarev density theorem, the set $\Omega$ has positive density determined by the image of $\bar{\rho}_{E,2}$. This image is determined as follows. We note that $\op{GL}_2(\F_2)\simeq S_3$ and up to conjugacy, there are only three proper subgroups of $\op{GL}_2(\F_2)$: \begin{itemize} \item $G_1=\left\{\mathbf{1}\right\}$, \item $G_2=\left\{\mathbf{1}, \mtx{1}{1}{0}{1}\right\}$, \item $G_3=\left\{\mathbf{1}, \mtx{1}{1}{1}{0}, \mtx{0}{1}{1}{1}\right\}$. \end{itemize} Note that $E(\Q)[2]\neq 0$ if and only if $\op{image}\bar{\rho}_{E,2}$ is conjugate to $G_1$ or $G_2$. \begin{theorem}[Zywina] Set \[ J_1 = 256 \frac{(t^2+t+1)^3}{t^2(t+1)^2}, \quad \quad J_2(t)=256 \frac{(t+1)^3}{t}, \quad \quad J_3(t) = t^2 + 1728, \] and let $E_{/\Q}$ be an elliptic curve without complex multiplication. Then the image of $\bar{\rho}_{E,2}$ is conjugate to a subgroup of $G_i$ if and only its $j$-invariant $j(E)$ is of the form $J_i(t)$ for some $t\in \Q$. \end{theorem} \begin{remark}\label{trivial remark} There are 3 cases to consider: \begin{itemize} \item if $E(\Q)[2]\neq 0$, then all primes $\ell\nmid 2 N_E$ are contained in $\Omega$. \item If the image of $\bar{\rho}_{E, 2}$ is conjugate to $G_3$, then a prime $\ell\nmid 2 N_E$ is contained in $\Omega$ if and only if $\bar{\rho}_{E, 2}(\op{Frob}_\ell)=\op{Id}$. \item If $\bar{\rho}_{E,2}$ is surjective then $\ell\in \Omega$ if and only if $\bar{\rho}_{E, 2}(\op{Frob}_\ell)\notin \left\{\mtx{1}{1}{1}{0}, \mtx{0}{1}{1}{1}\right\}$. \end{itemize} \end{remark} \begin{corollary}\label{cor:omega-density-p-2} Let $E/\Q$ be an elliptic curve without complex multiplication, and let $j(E)$ denote its $j$-invariant. If $\bar{\rho}_{E,2}$ is not surjective, let $i \in \{1, 2, 3\}$ be the smallest value such that $j(E)=J_i(t)$ for some $t \in \Q$. Then the density of $\Omega$ is given by \[ \mathfrak{d}(\Omega)=\begin{cases} 1, & \text{if}\ i=1,2\\ \frac{1}{3}, & \text{if i=3} \end{cases}. \] On the other hand, if $\bar{\rho}_{E,2}$ is surjective, $\mathfrak{d}(\Omega)=\frac{2}{3}$. \end{corollary} \begin{proof} For a prime $\ell \nmid N_E$, let $\mathrm{Frob}_\ell$ denote the corresponding Frobenius element. Then we have \[ a_\ell(E) \equiv \mathrm{tr}\ \bar{\rho}_{E,2}\left( \mathrm{Frob}_\ell\right) \mod 2. \] As the trace is conjugacy invariant, we can thus investigate the frequency that $E(\F_\ell)[2]\neq 0$ in terms of the $G_i$. The result follows as a consequence of Remark \ref{trivial remark} and the Chebotarev density theorem. \end{proof} \begin{corollary}\label{cor 3.9} Let $d$ be a positive squarefree integer and write $d=\ell_1\dots \ell_k$. Assume that for all $i=1,\ldots,k$ we have $\ell_i\nmid 2 N_E$ and $\ell_i\notin \Omega$. Then we find that $\lambda_2(E^{(d)}/\Q)=\lambda_2(E/\Q)$. Moreover, we find that \[\op{corank}_{\Z_2} \op{Sel}_{2^\infty}(E^{(d)}/\Q)\leq \lambda_2(E/\Q).\] \end{corollary} \begin{proof} The result is a direct consequence of Theorem \ref{thm:kida-matusno-2} and Lemma \ref{lemma on Selmer corank and lambda}. \end{proof} Let $\Omega'$ be the set of primes $\ell \nmid 2 N_E$ such that $\ell\notin \Omega$. According to Remark \ref{trivial remark}, when $E(\Q)[2]\neq 0$, the set $\Omega'$ is finite. On the other hand, by Corollary~\ref{cor 3.9}, if $E(\Q)[2]=0$ then the density of $\Omega'$ is $2/3$ (resp. $1/3$) if the image of $\bar{\rho}_{E,2}$ is conjugate to $G_3$ (resp. $\op{GL}_2(\F_2)$). Given a natural number $r$, let $n^{\op{alg}}_{E,\leq r}(X)$ be the number of squarefree numbers $d>0$ such that $\textbf{r}_{\op{alg}}(E^{(d))})\leq r$ and $d<X$. \begin{theorem} Let $E$ be an elliptic curve over $\Q$ with good ordinary reduction at $2$. Assume that $N_E$ is squarefree and that $E(\Q)[2]=0$. Setting $\lambda:=\lambda_2(E/\Q)$, we find that \[n_{E, \leq \lambda}^{\op{alg}}(X)\gg \frac{X}{(\log X)^\delta}\] where $\delta:=\mathfrak{d}(\Omega)=\begin{cases} \frac{1}{3}\text{ if }\op{image}\bar{\rho}_{E, 2}\simeq G_3,\\ \frac{2}{3}\text{ if }\op{image}\bar{\rho}_{E, 2}=\op{GL}_2(\F_2).\\ \end{cases}$. \end{theorem} \begin{proof} According to Corollary \ref{cor 3.9}, if $d>0$ is a product of primes $\ell_i\in \Omega'$ then \[\ralg(E^{(d)})\leq \op{corank}_{\Z_2} \op{Sel}_{2^\infty}(E^{(d)}/\Q)\leq \lambda_2(E/\Q).\] Let $\widetilde{\Omega}$ be the complement of $\Omega'$, i.e. it is the union of $\Omega$ and the primes $\ell|2 N_E$. Clearly, $\widetilde{\Omega}$ has the same density as $\Omega$. It then follows from Proposition \ref{density propn for n_Omega} that \[n_{E, \leq \lambda}^{\op{alg}}(X)\gg n_{\widetilde{\Omega}}(X)\gg \frac{X}{(\log X)^\delta}.\] This completes the proof. \end{proof} \begin{proposition}\label{corank 1 lemma} Suppose that $E$ is an elliptic curve over $\Q$ and assume that the following assertions are satisfied: \begin{enumerate} \item $E$ has good ordinary reduction at $2$, \item $E(\Q)[2]=0$, \item $\mu_2(E/\Q)=0$ and $\lambda_2(E/\Q)\leq 2$. \item There is a finite set of primes $\ell_1, \dots, \ell_k$ coprime to $2N_E$ such that \begin{itemize} \item $\ell_1, \dots, \ell_k\notin \Omega$, \item $d=\ell_1\dots \ell_k\equiv 1\pmod{4}$. \item If $\omega(E)=+1$ (resp. $\omega(E)=-1$) then an odd (resp. even) number of the primes $\ell_i$ are inert in $\Q(\sqrt{-N_E})$ and the rest are split in $\Q(\sqrt{-N_E})$. \end{itemize} \end{enumerate} Then we have that \begin{enumerate} \item $\mu_2(E^{(d)}/\Q)=0$ and $\lambda_2(E^{(d)}/\Q)=1$, \item $\omega(E^{(d)})=-1$, \item $\op{corank}_{\Z_2}\left( \op{Sel}_{2^\infty}(E^{(d)}/\Q)\right)=1$. \end{enumerate} \end{proposition} \begin{proof} The relationship between $\omega(E)$ and $\omega(E^{(d)})$ is given by \[\omega(E^{(d)})=\chi_d(-N_E) \omega(E),\] cf. \cite[section 4]{rubinsilverberg}. The character $\chi_d(-N_E)$ is given by the Kronecker symbol $\left(\frac{d}{-N_E}\right)$. Then we have that \[\left(\frac{\ell_1\dots\ell_k}{-N_E}\right)=(-1)^{(\frac{d-1}{2})(\frac{N_E-1}{2})}\left(\frac{-N_E}{\ell_1\dots\ell_k}\right)=\left(\frac{-N_E}{\ell_1\dots\ell_k}\right)=\left(\frac{-N_E}{\ell_1}\right)\cdots \left(\frac{-N_E}{\ell_k}\right).\] Consequently, we have that $\omega(E^{(d)})=-1$. \par We note that $E(\Q)[2]=0$ is a condition equivalent to requiring that $\rho_{E, 2}(\op{G}_{\Q})$ is either $G_3$ or all of $\op{GL}_2(\F_2)$. On the other hand, since $\Q(\sqrt{d})$ is disjoint from $\Q(E[2])$ since the primes $\ell_i$ are ramified in $\Q(\sqrt{d})$ and unramified in $\Q(E[2])$. It follows that $\rho_{E, 2}(\op{G}_{\Q(\sqrt{d})})=\rho_{E, 2}(\op{G}_{\Q})$. Thus, we find that $\rho_{E, 2}(\op{G}_{\Q(\sqrt{d})})$ is either $G_3$ or all of $\op{GL}_2(\F_2)$ and deduce that $E(\Q(\sqrt{d}))[2]=0$. In particular, it follows that $E^{(d)}(\Q)[2]=0$. Then Theorem \ref{dok-dok thm} of Dokchitser and Dokchitser applies to $E^{(d)}$ and implies that $\op{corank}_{\Z_2}\op{Sel}_{2^\infty}(E^{(d)}/\Q)$ is odd. Since $\ell_i\notin \Omega$ for $i\geq 2$, we find that \[\op{corank}_{\Z_2}\op{Sel}_{2^\infty}(E^{(d)}/\Q)\leq \lambda_2(E^{(d)}/\Q)=\lambda_2(E/\Q)\leq 2.\] This in particular implies that $\op{corank}_{\Z_2}\op{Sel}_{2^\infty}(E^{(d)}/\Q)=1$. \end{proof} \begin{theorem}\label{n'E,1} Let $E$ be an elliptic curve over $\Q$ and assume that the following conditions are satisfied: \begin{enumerate} \item $E$ has good ordinary reduction at $2$ with squarefree conductor $N_E$, \item $\omega(E)=-1$, \item $E(\Q)[2]=0$, \item $\mu_2(E/\Q)=0$ and $\lambda_2(E/\Q)\leq 2$. \end{enumerate} Then we find that \[n_{E, 1}'(X)\gg \frac{X}{(\log X)^{\frac{11}{12}}}.\] \end{theorem} \begin{proof} Let $\mathcal{M}$ consist of primes $\ell$ such that $\ell\nmid 2 N_E$, $\ell\notin \Omega$, and $\ell$ splits completely in $F:=\Q(i, \sqrt{-N_E})$. Suppose that $d=\ell_1\dots \ell_s$ is a product of distinct primes $\ell_1, \dots, \ell_s\in \mathcal{M}$. Since each of the primes $\ell_i\equiv 1\pmod{4}$ it follows that $d\equiv 1\pmod{4}$. It then follows from Proposition \ref{corank 1 lemma} that $\op{corank}_{\Z_2}\left( \op{Sel}_{2^\infty}(E^{(d)}/\Q)\right)=1$. Let $\mathcal{M}^c$ be the complement of $\mathcal{M}$ and recall that $n_{\mathcal{M}^c}(X)$ is the number of squarefree $d<0$ that are products of primes $\ell\in \mathcal{M}$. We find that $n_{E, 1}'(X)\geq n_{\mathcal{M}^c}(X)$. On the other hand, Proposition \ref{corank 1 lemma} implies that \[n_{\mathcal{M}^c}(X)\gg \frac{X}{(\log X)^{1-\mathfrak{d}(\mathcal{M})}}.\] \par Thus what remains is to estimate $\mathfrak{d}(\mathcal{M})$. Let $k\subset \Q(E[2])$ be the field over which $\Q(E[2])/k$ is a cubic extension. Thus, if the residual image is conjugate to $G_3$, $k=\Q$. According to Remark \ref{trivial remark} a prime $\ell\nmid 2 N_E$ is not contained in $\Omega$ if it splits in $k$ and the primes above it are nonsplit in the extension $\Q(E[2])/k$. We show that this is independent of the condition that $\ell$ splits completely in the biquadratic extension $F$. We shall set $F(E[2])$ to denote the composite $F\cdot \Q(E[2])$. Since $[F\cdot k:k]$ is prime to $3$, we find that $\Q(E[2])\cap F\cdot k=k$. Consequently, we find that $\op{Gal}(F(E[2])/F\cdot k)\simeq \op{Gal}(\Q(E[2])/k)$. Let $\mathcal{S}$ denote the two nontrivial elements of $\op{Gal}(F(E[2])/F\cdot k)$ and view $\mathcal{S}$ as a subset of $\op{Gal}(F(E[2])/\Q)$. We find that $\ell\in \mathcal{M}$ if and only if $\op{Frob}_\ell\in \mathcal{S}$. Thus by the Chebotarev density theorem, \[\mathfrak{d}(\mathcal{M})=\frac{\# \mathcal{S}}{[F(E[2]):\Q]}\geq \frac{2}{[\Q(E[2]):\Q][F:\Q]}\geq \frac{1}{12}.\] Thus we find that $1-\mathfrak{d}(\mathcal{M})\leq \frac{11}{12}$, which proves the result. \end{proof} The proof of the above result also shows that there is an explicit set of prime numbers $\mathcal{M}$ of density $\geq \frac{1}{12}$ such that for all $\ell\in \mathcal{M}$, $\op{corank}_{\Z_2}\left( \op{Sel}_{2^\infty}(E^{(\ell)}/\Q)\right)=1$. \begin{theorem}\label{them prime twist 3.13} Let $E$ be an elliptic curve satisfying the conditions of Theorem \ref{n'E,1}. Let $\mathcal{M}$ be the set of prime numbers $\ell$ such that \begin{itemize} \item $\ell\equiv 1\pmod{4}$, \item $\widetilde{E}(\F_\ell)[2]=0$, \item $\ell$ splits completely in $\Q(i, \sqrt{-N_E})$. \end{itemize} Then $\mathcal{M}$ has density $\geq \frac{1}{12}$ and $\op{corank}_{\Z_2}\left( \op{Sel}_{2^\infty}(E^{(\ell)}/\Q)\right)=1$. \end{theorem} \begin{proof} The result follows from the proof of Theorem \ref{n'E,1}. \end{proof} \par Before moving forward, let's illustrate Theorem \ref{n'E,1} through an explicit example. \begin{example} Consider the elliptic curve with Cremona label \href{https://www.lmfdb.org/EllipticCurve/Q/53/a/1}{53a1} defined by \[E:y^2=x^3+405x+16038.\] The data on LMFDB tells us that $E$ has good ordinary reduction at $2$ and $E(\Q)[2]=0$ with \[\mu_2(E/\Q)=0\text{ and }\lambda_2(E/\Q)=1.\] This curve has Mordell--Weil rank $1$ and its root number is $-1$. Therefore Theorem \ref{n'E,1} applies to $E$ to give us that \[n_{E, 1}'(X)\gg \frac{X}{(\log X)^{\frac{11}{12}}}.\] Proposition \ref{corank 1 lemma} asserts that if $\ell\equiv 1\mod{4}$ is a prime number which splits completely in $\Q(\sqrt{-53})$ and $2\nmid a_\ell(E)$, then $\op{corank}_{\Z_2}\left( \op{Sel}_{2^\infty}(E^{(d)}/\Q)\right)=1$. For instance, the primes $\ell=13, 17,$ and $29$ each satisfy these conditions. The same is true for the product $d:=13\times 17\times 29$. \end{example} We also prove a surprising result for twists by prime numbers for elliptic curves $E/\Q$ for which $E(\Q)[2]\neq 0$. \begin{theorem}\label{pos density of primes} Let $E/\Q$ be an elliptic curve satisfying the following conditions: \begin{enumerate} \item $E$ has squarefree conductor $N_E$, \item $E$ has good ordinary reduction at $2$ \item $\omega(E)=+1$, \item $\mu_2(E/\Q)=0$ and $\lambda_2(E/\Q)=0$, \item $E(\Q)[2]\neq 0$. \end{enumerate} Let $\ell$ be a prime satisfying the following conditions: \begin{itemize} \item $\ell\nmid 2 N_E$, \item $\ell$ is inert in $K:=\Q(\sqrt{-N_E})$, \item $\ell\equiv 3, 5\pmod{8}$. \end{itemize} Then we have that $\op{corank}_{\Z_2} \op{Sel}_{2^\infty}(E^{(\ell)}/\Q)=1$. The set of primes satisfying the above conditions have density $\frac{1}{4}$. \end{theorem} \begin{proof} Since $\ell\nmid 2 N_E$, we find that \[\omega(E^{(d)})=\chi_d(-N_E) \omega(E)=-\omega(E)=-1.\] Theorem \ref{dok-dok thm} of Dokchitser and Dokchitser asserts that $s_2(E)=\op{corank}_{\Z_2}\op{Sel}_{2^\infty}(E^{(\ell)}/\Q)$ is odd. Note that $E(\Q)[2]\neq 0$ implies that $\ell\in \Omega$, see Remark \ref{trivial remark}. Since $\ell\equiv 3, 5, 11, 13\pmod{16}$, we find that $n_\ell=\mathrm{ord}_2 \left( \frac{\ell^2 - 1}{8} \right)=0$ (in accordance with notation in the statement of Theorem \ref{thm:kida-matusno-2}). Thus it follows from Theorem \ref{thm:kida-matusno-2} that \[\lambda_2(E^{(d)}/\Q) = \lambda_2(E/\Q) + 2^{n_\ell + 1}=2.\] It then follows from \eqref{effective bounds after Matsuno} that $\op{corank}_{\Z_2}\op{Sel}_{2^\infty}(E^{(\ell)}/\Q)\leq 2$. Thus, we have shown that \[\op{corank}_{\Z_2}\op{Sel}_{2^\infty}(E^{(\ell)}/\Q)=1.\] Note that there is no elliptic curve with conductor $2$. Since $N_E$ is squarefree, it must be therefore be divisible by some odd prime number. Thus, $K$ is linearly disjoint from $\Q(\sqrt{2})$. The condition that $\ell\equiv 3,5\pmod{8}$ is equivalent to $\ell$ being inert in $\Q(\sqrt{2})$. That the set of primes $\ell$ has density $\frac{1}{4}$ follows from the Chebotarev density theorem. \end{proof} \begin{example} Consider the elliptic curve with Cremona label \href{https://www.lmfdb.org/EllipticCurve/Q/15/a/4}{15a7} defined by $E:y^2=x^3-103707x+12854646$. The conditions of Theorem \ref{pos density of primes} are satisfied for $E$. The table below lists the ranks of the twists $E^{(\ell)}$ for the primes $7 \leq \ell \leq 47$, and the highlighted rows correspond to those $\ell$ which satisfy the bulleted conditions of Theorem~\ref{pos density of primes}. \begin{table}[h!] \begin{tabular}{|c|c|c|c|} \hline $\ell$ & $\ell$ mod $8$ & decomposition in $K=\Q(\sqrt{-15})$ & $\ralg(E^{(\ell)})$ \\ \hline \hline $7$ & $7$ & inert & $1$ \\ \hline \cellcolor{green!25}$11$ & \cellcolor{green!25}$3$ & \cellcolor{green!25}inert & \cellcolor{green!25}$1$ \\ \hline \cellcolor{green!25}$13$ & \cellcolor{green!25}$5$ & \cellcolor{green!25}inert & \cellcolor{green!25}$1$ \\ \hline $17$ & $1$ & split & $0$ \\ \hline $19$ & $3$ & split & $0$ \\ \hline $23$ & $7$ & split & $0$ \\ \hline \cellcolor{green!25}$29$ & \cellcolor{green!25}$5$ & \cellcolor{green!25}inert & \cellcolor{green!25}$1$ \\ \hline $31$ & $7$ & split & $0$ \\ \hline \cellcolor{green!25}$37$ & \cellcolor{green!25}$5$ & \cellcolor{green!25}inert & \cellcolor{green!25}$1$ \\ \hline $41$ & $1$ & inert & $1$ \\ \hline \cellcolor{green!25}$43$ & \cellcolor{green!25}$3$ & \cellcolor{green!25}inert & \cellcolor{green!25}$1$ \\ \hline $47$ & $7$ & split & $0$ \\ \hline \end{tabular} \label{table: rank 1 examples} \end{table} \end{example} \section{Prescribed $\lambda$-invariants}\label{s 4} \par In this short section, we prove a result about the distribution of $\lambda$-invariants in quadratic twist families. We fix an elliptic curve $E_{/\Q}$ with good ordinary reduction at $2$ with squarefree conductor $N_E$. Moreover we shall assume that $E(\Q)[2]=0$. Consider the family of quadratic twists $E^{(d)}$ where $d>0$ is a squarefree integer and let $m_{E, N}(X)$ count the number of positive squarefree $d\leq X$ such that $\lambda(E^{(d)}/\Q)=N$. \begin{theorem}\label{last thm} Assume that $\mu_2(E/\Q)=0$. If $N$ is any integer such that $N\geq \lambda_2(E/\Q)$ and $N\equiv \lambda_2(E/\Q)\pmod{2}$, then we have that \[m_{E, N}(X)\gg X/(\log X)^{\alpha},\] where $\alpha=\begin{cases} \frac{1}{3} & \text{ if }\op{Gal}(\Q(E[2])/\Q)\simeq \Z/3\Z;\\ \frac{2}{3}& \text{ if }\op{Gal}(\Q(E[2])/\Q)\simeq S_3. \end{cases}$ \end{theorem} \begin{proof} Let $\mathcal{Q}$ consist of primes $\ell \nmid 2 N_E$ such that: \begin{itemize} \item $\ell\equiv 3, 5\pmod{8}$, \item $\ell\in \Omega$, i.e., $\widetilde{E}(\F_\ell)[2]\neq 0$. \end{itemize} Let $k$ be the subfield of $\Q(E[2])$ such that $\Q(E[2])/k$ is a cubic extension. Suppose that $\ell\nmid 2 N_E$ is a prime which is inert in $\Q(\sqrt{2})$ and any prime $v$ of $k$ that lies above $\ell$ is completely split in $\Q(E[2])$. Then, $\ell\in \mathcal{Q}$ and thus $\mathcal{Q}$ has positive density. Write $N=\lambda_2(E/\Q)+2k$ and pick $k$ primes $q_1, \dots, q_k\in \mathcal{Q}$. Since $q_i\equiv 3,5 \pmod{8}$ we find that $n_{q_i}=0$. Let $d':=q_1\dots q_k$ and let $\ell_1,\dots ,\ell_s$ be primes such that $\ell_i\nmid 2N_E$ and $\ell_i\notin \Omega$. We then set $d:=d' d''$, where $d'':=\ell_1\dots\ell_s$. From \eqref{thm:kida-matusno-2} we deduce that \[\lambda_2(E^{(d)}/\Q) = \lambda_2(E/\Q) + \sum_{\substack{\ell \mid d \\ 2 \mid \# \tilde{E}(\F_2)}} 2^{n_\ell + 1}=\lambda_2(E/\Q) +2k=N.\] Keeping $d'$ fixed, the number of choices for $d''$ such that $d''\leq \frac{X}{d'}$ is $n_{\Omega}(X/d')$. By Proposition \ref{density propn for n_Omega}, there is a constant $c>0$ such that \[n_{\Omega}(X/d')\sim \frac{c(X/d')}{\left(\log (X/d')\right)^{\mathfrak{d}(\Omega)}}.\] Relabeling $c$, we find that \[n_{\Omega}(X/d')\sim c X/(\log X)^{\mathfrak{d}(\Omega)}.\] Thus \[m_{E,N}(X)\geq n_{\Omega}(X/d')\gg X/(\log X)^{\delta},\] where by Corollary \ref{cor:omega-density-p-2}, \[\alpha:=\mathfrak{d}(\Omega)=\begin{cases} \frac{1}{3} & \text{ if }\op{Gal}(\Q(E[2])/\Q)\simeq \Z/3\Z;\\ \frac{2}{3}& \text{ if }\op{Gal}(\Q(E[2])/\Q)\simeq S_3. \end{cases}\] \end{proof} \begin{example} Consider the elliptic curve with Cremona label \href{https://www.lmfdb.org/EllipticCurve/Q/53/a/1}{53a1} defined by \[E:y^2=x^3+405x+16038.\] From the data in LMFDB, we find that $\mu_2(E/\Q)=0$ and $\lambda_2(E/\Q)=1$. Moreover, the representation $\bar{\rho}_{E,2}$ is surjective. Theorem \ref{last thm} asserts that for any odd integer $N\geq 1$, \[m_{E, N}(X)\gg X/(\log X)^{2/3}.\] \end{example} \begin{example} Consider the elliptic curve \href{https://www.lmfdb.org/EllipticCurve/Q/17/a/4}{17a4} defined by \[E: y^2=x^3-11x+6.\] According to LMFDB, $\mu_2(E/\Q)=0$ and $\lambda_2(E/\Q)=0$. The representation $\bar{\rho}_{E,2}$ is surjective. Theorem \ref{last thm} asserts that \[m_{E, N}(X)\gg X/(\log X)^{2/3}\] for any even integer $N\geq 0$. \end{example} \bibliographystyle{abbrv} \bibliography{references} \end{document}
2412.07410v1
http://arxiv.org/abs/2412.07410v1
KMS states on quantum Cuntz-Krieger algebras
\documentclass[a4paper,reqno]{amsart} \setlength{\textheight}{54\baselineskip} \setlength{\textwidth}{15cm} \setlength{\voffset}{-3\baselineskip} \setlength{\oddsidemargin}{21pt} \setlength{\evensidemargin}{21pt} \usepackage{amsmath, amsfonts, amssymb, amsxtra, multicol, physics} \usepackage[utf8]{inputenc} \usepackage[english]{babel} \usepackage{hyperref} \usepackage[mathscr]{eucal} \usepackage{tikz-cd} \usepackage{comment} \usepackage{enumitem} \usepackage{seqsplit} \usepackage{dsfont} \usepackage{bm} \hypersetup{colorlinks=true,linkcolor=red, anchorcolor=green, citecolor=cyan, urlcolor=red, filecolor=magenta, pdftoolbar=true} \numberwithin{equation}{section} \newtheorem{theorem}{Theorem} \numberwithin{theorem}{section} \newtheorem{proposition}[theorem]{Proposition} \newtheorem{lemma}[theorem]{Lemma} \newtheorem{corollary}[theorem]{Corollary} \newtheorem{question}[theorem]{Question} \newtheorem{conjecture}[theorem]{Conjecture} \newtheorem{introtheorem}{Theorem} \renewcommand\theintrotheorem{\Alph{introtheorem}} \theoremstyle{definition}\newtheorem{definition}[theorem]{Definition} \theoremstyle{definition}\newtheorem{remark}[theorem]{Remark} \theoremstyle{definition}\newtheorem{example}[theorem]{Example} \theoremstyle{definition}\newtheorem*{notation*}{Notation} \theoremstyle{definition}\newtheorem*{convention*}{Convention} \theoremstyle{definition}\newtheorem{observation}{Observation} \theoremstyle{definition}\newtheorem*{acknowledgment*}{Acknowledgments} \newcommand{\N}{\mathbb{N}} \newcommand{\R}{\mathbb{R}} \newcommand{\C}{\mathbb{C}} \newcommand{\A}{\mathsf{A}} \renewcommand{\H}{\mathsf{H}} \newcommand{\B}{\mathsf{B}} \newcommand{\G}{\mathcal{G}} \newcommand{\bh}{\op{B}(\mathsf{H})} \newcommand{\cst}{C^*} \newcommand{\range}{ran} \newcommand{\tx}{\mathcal{T}_X} \newcommand{\ox}{\mathcal{O}_X} \renewcommand{\op}[1]{\operatorname{#1}} \DeclareMathOperator{\diag}{diag} \DeclareMathOperator{\Aut}{Aut} \DeclareMathOperator{\Iso}{Iso} \DeclareMathOperator{\Span}{span} \begin{document} \title{KMS states on quantum Cuntz-Krieger algebras} \begin{abstract} We study the KMS states on local quantum Cuntz-Krieger algebras associated to quantum graphs. Using their isomorphism to the Cuntz-Pimsner algebra of the quantum edge correspondence, we show that the general criteria for KMS states can be translated into statements about the underlying quantum adjacency operator, somewhat analogously to the case of classical Cuntz-Krieger algebras. We study some examples of gauge actions, for which a complete classification of KMS states can be obtained. \end{abstract} \author[M.\,Kumar]{Manish Kumar} \email{[email protected]} \address{KU Leuven, Department of Mathematics, Celestijnenlaan 200B, 3001 Leuven, Belgium} \author[M.\,Wasilewski]{Mateusz Wasilewski} \email{[email protected]} \address{Institute of Mathematics of the Polish Academy of Sciences, ul.~\'Sniadeckich 8, 00--656 Warsaw, Poland} \subjclass[2020]{46L55, 46L08} \keywords{Quantum graphs, Quantum Cuntz-Krieger algebras, KMS states} \maketitle \section{Introduction} The study of KMS states has a long history, both in mathematical physics and operator algebra theory. In this article we study these objects for $C^{\ast}$-algebras associated to quantum graphs, namely the \emph{local quantum Cuntz-Krieger algebras} (see \cite{BHINW}). We choose to work with these rather than the original quantum Cuntz-Krieger algebras (see \cite{BEVW}) because the local versions can be identified with certain Cuntz-Pimsner algebras (see \cite{Pim}), for which one can find a general condition for existence of KMS states in \cite{LN}. Classical Cuntz-Krieger algebras (\cite{CK}) can also be realized as Cuntz-Pimsner algebras of the edge $\cst$-correspondence and the general result on gauge actions and their KMS states then reduces to the condition that can be stated purely in terms of the adjacency matrix of the graph (see \cite{EFW}, and also \cite{OP} for the special case of Cuntz algebras). It turns out that in the case when the adjacency matrix is irreducible there is a unique equilibrium inverse temperature $\beta$ and it is equal to the logarithm of the spectral radius of the matrix. The aim of our paper is to perform a similar reduction in the case of quantum graphs. Quantum graphs are a non-commutative generalization of classical finite graphs without multiple edges, where we mean a triple $(\B, A, \psi)$ consisting of a finite-dimensional $\cst$-algebra $\B$, a state $\psi$ on $\B$ and a quantum adjacency operator $A$ (see Subsection \ref{sub:quantum graph}). We begin with the case of the usual gauge action, where the generators are multiplied by the number $e^{it}$. In this case we obtain the following result. \begin{introtheorem} Let $\mathcal{G}:=(\B, A, \psi)$ be a quantum graph, where $\B\simeq \bigoplus_{a=1}^{d} M_{n_a}$ and let $(\gamma_t)$ be the gauge action on the associated local quantum Cuntz-Krieger algebra $\mathcal{O}_{E_{\mathcal{G}}}$. Then the KMS states on $\mathcal{O}_{E_{\mathcal{G}}}$ are in one-to-one correspondence with positive eigenvectors of a certain integer-valued matrix $D \in M_{d}(\mathbb{N})$ built from $A$. In particular, if the matrix $D$ is irreducible then the KMS state is unique. \end{introtheorem} This shows that identifying criteria for KMS states with respect to gauge actions ultimately boils down to understanding certain classical graphs with multiple edges. We prove this theorem by computing explicitly the induced trace from Theorem \ref{Thm:LN}, which is a crucial step in actually classifying the KMS states. Because of the concrete form, for other type of actions we are also able to derive a condition for KMS states, provided that we understand the generator of the action well enough. In Subsection \ref{subsec:orth} we take a closer look on the edge correspondences. The quantum edge correspondence, denoted $E_\G$ and introduced in \cite{BHINW}, is a cyclic sub-$\cst$-correspondence of $\B\otimes_\psi\B$ and generalizes the usual edge correspondece for a classical graph. \begin{introtheorem} Let $\mathcal{G}:=(\B, A, \psi)$ be a quantum graph and let $E_{\mathcal{G}}$ be its edge correspondence. Then we can construct an orthonormal basis of $E_{\mathcal{G}}$ given a nice Kraus decomposition of $A$. \end{introtheorem} This result allows us to reprove the previous results about KMS states, because having an explicit orthonormal basis is another way to compute the induced trace. We also handle the case of a more general gauge action. In the last part of the paper we provide an example of a gauge action on $\mathcal{O}_{E_{\mathcal{G}}}$, whose restriction to $\B$ is non-trivial, using the modular group. In this case we no longer work with induced traces but with more general functionals. In this case once again the KMS states are governed by a certain matrix $D \in M_d(\mathbb{R})$, but this time it is more complicated, e.g. it depends on the inverse temperature $\beta$. In the case of the complete quantum quantum graph the situation simplifies and a more satisfactory answer can be obtained. \section{Preliminaries} \subsection{Cuntz-Pimsner algebras} Let $\B$ be a $\cst$-algebra. A {\em $\cst$-correspondence} over $\B$ is a right Hilbert $\B$-module $X$ together with a $\B$-valued inner product $\langle\cdot,\cdot\rangle$ and a $\ast$-homomorphism $\phi_X:\B\to\op{B}(X)$, where $\op{B}(X)$ is the algebra of all adjointable operators on $X$. We simply write $b\xi$ for $\phi_X(b)\xi$, $b\in\B,\xi\in X$. We follow the convention of linearity in the second coordinate of the inner product. The {\em Toeplitz-Pimsner algebra} $\mathcal{T}_X$ is the universal $\cst$-algebra generated by elements $\pi(b)$ and $T_\xi$ with $b\in\B$ and $\xi\in X$ such that $\pi:\B\to\tx$ is a $\ast$-homomorphism, $T_{a\xi b}=\pi(a)T_\xi\pi(b)$ and $T_{\xi}^*T_\eta=\pi(\langle\xi,\eta\rangle)$ for $a,b\in\B$ and $\xi,\eta\in X$. A concrete way of constructing such algebras is as follows: Let $\mathcal{F}(X)=\B\oplus\bigoplus_{n\geq 1}X^{\otimes_\B n}$ where $X^{\otimes_\B n}$ denotes the $n$-times (internal) tensor product of the $\cst$-correspondence $X$. Consider the $\ast$-homomorphism $\pi:\B\to\op{B}({\mathcal{F}(X)})$ and the left creation operators $T_\xi\in \op{B}({\mathcal{F}(X)})$, $\xi\in X$ given by $\pi(a)\eta=a\eta$ and $T_\xi(\eta)=\xi\otimes\eta$ for $a\in\B$, $\xi\in \mathcal{F}(X)$. Then $\tx$ is the $\cst$-subalgebra of $\op{B}(\mathcal{F}(X))$ generated by $\pi(a), T_\xi$. The {\em Cuntz-Pimsner algebra} $\mathcal{O}_X$ is the quotient of $\tx$ by the ideal generated by elements of the form $\pi(b)-j_X(\phi_X(b))$ for $b\in I_X$ where \[I_X=\{a\in \B; \phi_X(a)\in \mathcal{K}(X) \mbox{ and } ab=0\;\forall b\in \ker\phi_X\}.\] Here $\mathcal{K}(X)$ is the space of compact operators on $X$ generated by $|\xi\rangle\langle\eta|\in \op{B}(X)$, $\xi,\eta\in X$, where $|\xi\rangle\langle\eta|(\zeta)=\xi\langle\eta,\zeta\rangle$ for $\zeta\in X$, and $j_X:\mathcal{K}(X)\to\tx$ is the homomorphism given by $j_X(|\xi\rangle\langle\eta|)=T_\xi T_\eta^*$ for $\xi,\eta\in X$. Now let $\sigma:\R\to \Aut (\B)$ be a one-parameter group of automorphisms of $\B$, and let $U:\R\to \Iso(X)$ be a one parameter group of isometries on $X$ such that \[U_t (a\xi)=\sigma_t(a)U_t\xi\;\;\;\mbox{ and }\;\;\langle U_t\xi, U_t\eta\rangle=\sigma_t(\langle\xi,\eta\rangle).\] Moreover, both are assumed to be strongly continuous i.e. $t\mapsto\sigma_t(a)$ and $t\mapsto U_t\xi$ are continuous for all $a\in \B$ and $\xi\in X$. By the universal property of the Toeplitz-Pimsner algebra, there exists a (unique) automorphism $\gamma_t$ on $\tx$ such that $\gamma_t(\pi(a))=\pi(\sigma_t(a))$ and $\gamma_t(T_\xi)=T_{U_t\xi}$ for all $a\in \B$ and $\xi\in X$. It is immediate that $t\mapsto\gamma_t$ is a strongly continuous one parameter automorphism group of $\tx$. Further note that the expression $U_t(a\xi)=\sigma_t(a)U_t\xi$ for all $\xi\in X$ precisely means that $\phi_X(\sigma_t(a))=U_t\phi_X(a)U_t^*$. This implies that $a\in \ker\phi_X$ iff $\sigma_t(a)\in \ker\phi_X$, and $\phi_X(a)\in \mathcal{K}(X)$ iff $\phi_X(\sigma_t(a))\in \mathcal{K}(X)$ such that $\gamma_t(j_X(\phi_X(a)))=j_X(U_t\phi_X(a)U_t^*)=j_X(\phi_X(\sigma_t(a)))$. This shows that $a\in I_X$ iff $\sigma_t(a)\in I_X$ and $\gamma_t(I_X)=I_X$. Hence $\gamma_t$ further induces a strongly continuous one-parameter group of automorphisms on the quotient Cuntz-Pimsner algebra $\ox$. \subsection{Quantum graphs}\label{sub:quantum graph} Let $\B$ be a finite dimensional $\cst$-algebra equipped with a faithful state $\psi:\B\to\C$. We denote by $L^2(\B,\psi)$ or simply by $L^2(\B)$ the corresponding GNS Hilbert space, which as a set is nothing but $\B$ itself. Let $m:L^2(\B)\otimes L^2(\B)\to L^2(\B)$ be the multiplication map, and $m^*:L^2(\B)\to L^2(\B)\otimes L^2(\B)$ be its adjoint. For $\delta>0$, the state $\psi:\B\to\C$ is called a {\em $\delta$-form} if $mm^*=\delta^2 i$. \begin{definition} A {\em quantum graph} $\G$ is a triple $(\B, A,\psi)$ where $\B$ is a finite-dimensional $\cst$-algebra, $\psi:\B\to\C$ is a $\delta$-form, and $A:\B\to\B$ is a {\em quantum adjacency operator} i.e. it satisfies $m(A\otimes A)m^*=\delta^2 A$ and is $\ast$-preserving (or, equivalently, completely positive). \end{definition} Let $\G=(\B, A, \psi)$ be a quantum graph, where $\psi:\B\to\C$ is a $\delta$-form. Write \[\B=\bigoplus_{a=1}^dM_{n_a}\] and \[\psi=\bigoplus_{a=1}^d\Tr(\cdot\rho_a)\] for some natural numbers $d, n_a$ and positive invertible operators $\rho_a\in M_{n_a}$ such that $\sum_{a=1}^d\Tr(\rho_a)=1$. Here $\Tr$ denotes the (non-normalized) traces on $M_{n_a}$. We assume that each $\rho_a$ is a diagonal operator with diagonal entries equal to $\psi(e_{11}^a), \ldots, \psi(e_{n_an_a}^a)$. Here $e_{ij}^a$ denotes the matrix units of $M_{n_a}$. That $\psi$ is a $\delta$-form then means that $\Tr(\rho_a^{-1})=\delta^2=\sum_{k=1}^{n_a}\psi(e_{kk}^a)^{-1}$ for all $1\leq a\leq d.$ Set \[f_{ij}^a=\frac{e_{ij}^a}{\psi(e_{ii}^a)^{1/2}\psi(e_{jj}^a)^{1/2}}.\] Then we have \begin{align*} &m^*(f_{ij}^a)=\sum_{k=1}^{n_a}f_{ik}^a\otimes f_{kj}^a\\ & f_{ij}^af_{rs}^b=\delta_{ab}\delta_{jr}\frac{f_{is}^a}{\psi(e_{jj}^a)} \end{align*} for $1\leq a,b\leq d$ and $1\leq i,j\leq n_a, 1\leq r,s\leq n_b$. Here $\delta_{ab}$ denotes the Kronecker delta. \subsection{Quantum Edge Correspondence} Let $\G=(\B, A, \psi)$ be a quantum graph such that $\psi$ is a $\delta$-form. Consider the $\cst$-correspondence $\B\otimes_\psi\B$ which as a vector space is equal to $\B\otimes\B$ as $\psi$ is faithful; the $\B$-valued inner product is defined by $\langle a\otimes b, c\otimes d\rangle:= \psi(a^{\ast}c)b^{\ast} d$. We now consider a sub $\cst$-correspondence of $\B\otimes_\psi\B$ as follows. Consider the {\em quantum edge indicator} $\varepsilon_\G$ given by \[\varepsilon_\G=\frac{1}{\delta^2}(i\otimes A)m^*(1)\in \B\otimes_\psi\B.\] The {\em quantum edge correspondence} $E_\G$ is the cyclic bimodule generated by $\varepsilon_\G$ i.e. \[E_\G=\B\varepsilon_\G\B=\Span\{a\varepsilon_\G b; a,b\in \B\}.\] Recall from \cite[Theorem 2.9]{BHINW} that $E_\G$ is faithful (i.e. the left action is a faithful homomorphism) iff $\ker A$ does not contain a central summand of $\B$, and $E_\G$ is full (i.e. $\Span\langle E_\G,E_\G\rangle=\B$) iff range of $A$ is not orthogonal to a central summand of $\B$. \subsection{Quantum Cuntz-Krieger algebras} We now recall the definition of quantum Cuntz-Krieger algebras as defined in \cite{BEVW} (note that the third condition does not appear there, but it is a natural unitality condition that we adopt, see also \cite[Definition 3.1]{BHINW}). \begin{definition} Let $\G=(\B, A, \psi)$ be a quantum graph. The {\em quantum Cuntz-Krieger algebra} associated to $\G$ is defined to be the universal unital $\cst$-algebra $\mathcal{O}(\G)$ generated by the elements $S(b), b\in \B$ where $S:\B\to\mathcal{O}(\G)$ is a linear map satisfying the following: \begin{enumerate} \item $\mu(\mu\otimes i)(S\otimes S^*\otimes S)(m^*\otimes i)m^*=S$ \item $\mu(S^*\otimes S)m^*=\mu(S\otimes S^*)m^*A$ \item $\mu(S\otimes S^*)m^*(1)=\frac{1}{\delta^2}i$. \end{enumerate} Here $S^*$ is the map $S^*:\B\to\mathcal{O}(\G)$ given by $S^*(b)=S(b^*)^*$ and $\mu:\mathcal{O}(\G)\otimes\mathcal{O}(\G)\to\mathcal{O}(\G)$ is the multiplication operator. \end{definition} We will impose formally stronger relations, called the local Cuntz-Krieger relations in \cite[Definition 3.4]{BHINW}, namely the first two equalities are replaced by $\mu(\mu\otimes i)(S\otimes S^*\otimes S)(m^*\otimes i)=\frac{1}{\delta^2}Sm$ and $\mu(S^*\otimes S)=\frac{1}{\delta^2}\mu(S\otimes S^*)m^* Am$. They ensure that we obtain a $C^{\ast}$-algebra isomorphic to the Cuntz-Pimsner algebra associated to the quantum edge correspondence under mild conditions (that the quantum edge correspondence is faithful). In cases that both versions of quantum Cuntz-Krieger algebras are understood, they are isomorphic. We recall the following results: \begin{theorem}[{\cite[Theorem 3.6]{BHINW}}]\label{thm:local CKA and CPA} Let $\G=(\B, A,\psi)$ be a quantum graph such that $\psi$ is a $\delta$-form. Let $(\pi_\G, t_\G)$ denote the universal covariant representation of the quantum edge correspondence $E_\G$. Assume that $E_\G$ is faithful. Then $\mathcal{O}_{E_\G}$ is the local quantum Cuntz-Krieger algebra of $\G$ with the associated local quantum Cuntz-Krieger family $S:\B\to \mathcal{O}_{E_\G}$ given by $S(b)=\frac{1}{\delta}t_{\G}(b\varepsilon_\G)$ for $b\in \B$. \end{theorem} In this work we deal exclusively with the KMS states on Cuntz-Pimsner algebras associated to the quantum edge correspondence and then we use the above theorem to induce KMS states on the associated (local) quantum Cuntz-Krieger algebras. We recall the following definition. \begin{definition} Let $\mathsf{D}$ be a $\cst$-algebra, and let $\sigma=(\sigma_t)_{t\in\R}$ be a one-parameter group of automorphisms on $\mathsf{D}$. Let $\beta\in\R$. A state $\varphi:\mathsf{D}\to\C$ is said to be a $(\sigma,\beta)$-{\em KMS state} (or a KMS state at the inverse temperature point $\beta$) if $\varphi$ is $\sigma_t$ invariant and satisfies : \[\varphi(ab)=\varphi(b\sigma_{i\beta}(a))\] for all $b\in \mathsf{D}$ and entire vectors $a\in \mathsf{D}$. \end{definition} \section{KMS states (tracial setting)} We want to determine the KMS states of quantum Cuntz-Pimsner algebras with respect to some natural dynamical systems. Let $\G=(\B, A, \psi)$ be a quantum graph such that $\psi$ is a $\delta$-form. Write \[\B=\bigoplus_{a=1}^dM_{n_a}\;\;\mbox{and }\psi=\bigoplus_{a=1}^d\Tr(\rho_a\cdot)\] as above. Let $\sigma=(\sigma_t)_{t\in \R}$ be a one-parameter group of automorphisms on $\B$ and let $(U_t)_{t\in \R}$ be a one parameter group of invertible isometries on $E_\G$ such that $U_t(b\xi)=\sigma_t(b)U_t\xi$ and $\langle U_t\xi, U_t\eta\rangle=\sigma_t(\langle\xi,\eta\rangle)$. Let $\gamma=(\gamma_t)_{t\in\R}$ be the induced one-parameter groups of automorphisms on $\mathcal{O}_{E_\G}$. We first assume that $\sigma$ is trivial so that the $\beta$-KMS states on $\B$ are tracial states. In this case we will use the following theorem (slightly simplified because we are working in the finite dimensional setting, we also did not include the positive energy condition in the statement, which will be satisfied in all of the examples we consider). \begin{theorem}[{\cite[Theorems 2.1, 2.5]{LN}}]\label{Thm:LN} Let $X$ be a $\B$-correspondence and let $(U_t)_{t\in\R}$ be a one parameter group of bimodular isometries on $X$. We will write $U_{t} = \exp(itD)$, where $D$ is the generator. Let $\gamma$ be the induced action on $\mathcal{O}_X$, which is trivial on $\B$. Then $\phi: \mathcal{O}_{X} \to \mathbb{C}$ is a $(\gamma,\beta)$ KMS state iff its restriction $\tau:=\phi_{|\B}$ is a trace such that $\op{Tr}_{\tau}(b \exp(-\beta D)) = \tau(b)$, where $\op{Tr}_{\tau}$ is the trace on $\op{B}(X)$ given by $\op{Tr}(|\xi\rangle\langle \xi|):= \tau(\langle \xi, \xi\rangle)$ (see \cite[Theorem 1.1.]{LN}). Moreover, the state $\phi$ is determined as: \begin{align*} \phi(T_{\xi_1}\cdots T_{\xi_n}T_{\eta_m}\cdots T_{\eta_1}^*)=\delta_{n,m}\tau(\langle\eta_1\otimes\cdots\eta_n, e^{-\beta D}\xi_1\otimes\cdots \otimes e^{-\beta D}\xi_n\rangle) \end{align*} for $\xi_i,\eta_j\in X$. \end{theorem} \begin{remark} 1. We are first interested in the $\beta$-KMS states on $\mathcal{O}_{E_\G}$ with respect to the {\em gauge action } i.e. when $U_t=e^{it}$ for all $t\in \R$. On the other hand, there is a natural gauge action $(\Tilde{\gamma}_t)_{t\in \R}$ on the (local) quantum Cuntz-Krieger algebra given by $\tilde{\gamma}_t(S(b))=e^{it}S(b)$. If $E_\G$ is faithful, by the universal property, it is clear that the isomorphism between $\mathcal{O}_{E_\G}$ and the local quantum Cuntz-Krieger algebra (as described in Theorem \ref{thm:local CKA and CPA} above) is an equivarient map under the gauge actions. Thus there is a one-to-one correspondence between the KMS states on the two algebras with respect to the gauge actions. Moreover, such KMS state $\phi$ on the local Cuntz-Krieger algebra will be determined by \begin{align*} \phi(S(a_1)\cdots S(a_n)S(b_m)^*\cdots S(b_1)^*)&= \frac{1}{\delta^{2n}}\delta_{n,m} e^{-n\beta}\tau(\langle b_1\varepsilon_\G\otimes\cdots \otimes b_n\varepsilon_\G, a_1\varepsilon_\G\otimes\cdots \otimes a_n\varepsilon_\G\rangle)\\ &=\frac{1}{\delta^{2n}}\delta_{n,m} e^{-n\beta}\tau(A(b_n^*A(b_{n-1}^*\cdots A(b_2^*A(b_1^*a_1)a_2)\cdots a_{n-1})a_n)). \end{align*} for $a_i, b_j\in \B$. 2. Since $\mathcal{O}_{E_\G}$ is a quotient of the quantum Cuntz-Krieger algebra (see \cite[Corollary 3.7]{BHINW}) and the quotient map is clearly gauge action equivarient, we will also obtain a KMS state on the quantum Cuntz-Krieger algebra. \end{remark} Let us now concretely obtain a criterion for the existence of $\beta$-KMS states on $\mathcal{O}_{E_\G}$ with respect to the gauge action. Let $\tau:=\bigoplus_{a=1}^d\lambda_a\Tr(\cdot)$ be a tracial state on $\B$, where $\lambda_a\geq0$ and $\sum_{a=1}^d\lambda_a n_a=1$. Note that for any $X=\oplus_{a=1}^{d}X_a\in\B$ with $X_a\in M_{n_a}$, we have \begin{align*} \tau(X)&=\sum_{a=1}^d\lambda_a\Tr(X_a)=\sum_{a=1}^d\lambda_a\psi(\rho_{a}^{-1}X_a) =\sum_{a=1}^d\lambda_a\langle\rho_{a}^{-1},X_a\rangle_\psi \\&=\sum_{a=1}^d\lambda_a\langle\rho_a^{-1}, X\rangle_\psi=\langle\oplus_{a=1}^d\lambda_a\rho_a^{-1}, X\rangle_\psi. \end{align*} The left action of $f_{ij}^a$ on $E_\G$ is given by \[f_{ij}^a\eta=\sum_{k=1}^{n_a}|f_{ik}^a\varepsilon_\G\rangle\langle f_{jk}^a\xi_G|(\eta)=\sum_{k=1}^{n_a}f_{ik}^a\varepsilon_\G\langle f_{jk}^a\varepsilon_\G,\eta\rangle,\;\;\;\;\eta\in E_\G.\] (see \cite[Theorem 2.12]{BHINW}). Consider the trace $\Tr_\tau$ defined on $\op{B}(E_\G)$ as in \cite[Theorem 1.1]{LN}. By using the same theorem from \cite{LN} and \cite[Theorem 2.5]{BHINW}, we calculate \begin{align*} \Tr_\tau(f_{ij}^a)&=\sum_{k=1}^{n_a}\Tr_\tau(|f_{ik}^a\varepsilon_\G\rangle\langle f_{jk}^a\varepsilon_\G|)=\sum_{k=1}^{n_a}\tau(\langle f_{jk}^a\varepsilon_\G, f_{ik}^a\varepsilon_\G\rangle)\\ &=\sum_{k=1}^{n_a}\frac{1}{\delta^2}\tau(A((f_{jk}^a)^*f_{ik}^a))=\frac{1}{\delta^2}\sum_{k=1}^{n_a}\frac{1}{\psi(e_{ii}^a)}\tau(A(\delta_{ij}f_{kk}^a))\\ &=\frac{\delta_{ij}}{\psi(e_{ii}^a)}\frac{1}{\delta^2}\tau(A(\rho_a^{-1})) \end{align*} where we use the expression $\rho_a^{-1}=\sum_{k=1}^{n_a}\psi(e_{kk}^a)^{-1}e_{kk}^a=\sum_{k=1}^{n_a}f_{kk}^a$ as each $\rho_a$ is a diagonal matrix. Thus we get \[\Tr_\tau(e_{ij}^a)=\delta_{ij}\frac{1}{\delta^2}\tau(A(\rho_a^{-1})).\] On the other hand, we have \begin{align*} \tau(e_{ij}^a)=\lambda_a\Tr(e_{ij}^a)=\lambda_a\delta_{ij}. \end{align*} Since $\beta$ is a KMS temperature point for a state on $\mathcal{O}_{E_\G}$ if and only if $\Tr_\tau(be^{-\beta})=\tau(b)$ for all $b\in \B$, the calculations above show that the last condition is equivalent to $\frac{1}{\delta^2}\tau(A(\rho_a^{-1}))=e^\beta\lambda_a,\forall 1\leq a\leq d$ that is, \begin{align*} \frac{1}{\delta^2}\sum_{b=1}^d\lambda_b\langle\rho_b^{-1},A(\rho_a^{-1})\rangle_\psi=e^\beta\lambda_a,\;\;\;\forall 1\leq a\leq d. \end{align*} If we consider the matrix $D=[D_{ab}]_{1\leq a,b\leq d}$ in $M_d$ given by $D_{ab}=\langle\rho_b^{-1}, A(\rho_{a}^{-1})\rangle_\psi$, $\lambda\in \C^d$ is the vector given by $\lambda=(\lambda_1,\ldots,\lambda_d)$, and $p_a$ is the canonical projections in $\C^d$, then above condition is equivalent to the following: \begin{align*} \frac{1}{\delta^2}\langle p_a, D\lambda \rangle_{\mathbb{C}^d}=e^\beta\langle p_a,\lambda\rangle_{\C^d},\;\;\;\forall1\leq a\leq d \end{align*} i.e $D\lambda=\delta^2e^\beta\lambda$. \begin{remark} 1. Since $A$ is a positive map, $A(\rho_b^{-1})\geq 0$ in $\B$. As $\langle\rho_b^{-1},A(\rho_a^{-1})\rangle_\psi$ is nothing but the trace of $b$th component of $A(\rho_a^{-1})$, it follows that $\langle\rho_b^{-1},A(\rho_a^{-1})\rangle_\psi\geq 0$ i.e. $D$ is a matrix with non-negative entries. 2. If we demand $\tau=\oplus_{a=1}^d\lambda_a\Tr(\cdot)$ to be faithful, then $\lambda_a>0$ for all $a.$ In this case, the equality $D\lambda=\delta^2e^\beta\lambda $ can only occur if $\delta^2 e^\beta=r(D)$, the spectral radius of $D$ (see \cite[Corollary 8.1.30]{HJ}). In particular, the only possible value of $\beta$ is $\log(\frac{1}{\delta^2}r(D))$. %\textcolor{blue}{Why is $D$ self-adjoint? We don't and shouldn't assume that the graph is symmetric.} 3. If the matrix $D$ is irreducible, then the only possible eigenvectors with non-negative entries correspond to the eigenvalue $r(D)$; in such cases the eigenspace corresponding to $r(D)$ is one-dimensional. Hence, we will have only one choice of $\beta$ and $\lambda_a's$ and hence the $\beta$-KMS state is unique. \end{remark} We summarize the above discussion in the following theorem. \begin{theorem}\label{Thm:KMS} Let $\B=\bigoplus_{a=1}^d M_{n_a}$, and let $\G=(\B, A,\psi)$ be a quantum graph. Let $\tau=\bigoplus_{a=1}^d\lambda_a\Tr(\cdot)$ for $\lambda_a\geq 0$ with $\sum_{a=1}^d\lambda_a n_a=1$. Then $\varphi$ is a $\beta$-KMS state on $\mathcal{O}_{E_\G}$ with respect to the gauge action such that $\varphi_{|_\B}=\tau$ iff $\lambda=(\lambda_1,\ldots,\lambda_d)\in\C^d$ is an eigenvector of the matrix $D:=[\langle\rho_a^{-1}, A(\rho_{b}^{-1})\rangle_\psi]$ with the eigenvalue $\delta^2e^\beta$. Moreover, the KMS state is unique if the matrix $D$ is irreducible. \end{theorem} Let us now calculate $\beta$ in some special cases for the KMS states with respect to the gauge action. \\ \\ 1. \textbf{(The classical case):} $\B=\C^d$, $\psi(x)=\frac{x_i}{d}$ for $x=(x_1,\ldots,x_d)\in \C^d$ so that $\psi$ is a $\delta$-form for $\delta=d$, and $A:\C^d\to\C^d$ is an adjacency matrix. Let $\langle\cdot,\cdot\rangle_{\C^d}$ denote the usual inner product on $\C^d$ so that $\langle\cdot,\cdot\rangle_{\C^d}=d\langle\cdot,\cdot\rangle_\psi$. For $1\leq a\leq d$, we have $\rho_a=\frac{p_a}{d}$ so that $\rho_a^{-1}=dp_a$ where $\{p_a\}_{1\leq a\leq d}$ are the canonical projections in $\C^d$. Hence \[\langle\rho_b^{-1},A\rho_a^{-1}\rangle_\psi=\frac{1}{d}\langle\rho_b^{-1},A(\rho_a^{-1})\rangle_{\C^d}=d\langle p_b,A(p_a)\rangle_{\C^d}=dA_{ba}\] where $A=[A_{ab}]$ is written in the matrix form with the canonical basis of $\C^d$. Thus $D=dA^t$ where $A^t$ denotes the transpose of $A$. If $A$ is irreducible (i.e. the graph is connected) then the only possible value of $\beta$ is \[\beta=\log(\frac{1}{\delta^2}r(D))=\log \frac{1}{d}dr(A^t)=\log r(A)\] and the KMS state is unique. Moreover, if the graph has no source (recall that a source is a vertex with no edges into it) then $E_\G$ is faithful and the KMS state $\phi$ on the corresponding Cuntz-Krieger algebra is given by \begin{align*} \phi(S_{i_1}\cdots S_{i_n}S_{j_m}^*\cdots S_{j_1}^*)&=\delta_{m,n}\frac{1}{d^2}e^{-n\beta}\tau(\langle p_{j_1}\varepsilon_\G\otimes\cdots p_{j_n}\varepsilon_\G, p_{i_1}\varepsilon_\G\otimes\cdots \otimes p_{j_n}\varepsilon_\G\rangle)\\ &=\frac{1}{d^2(r(A))^n}\delta_{m,n}\delta_{i_1,j_1}\cdots\delta_{i_n,j_n}A_{j_2j_1} A_{j_3j_2}\cdots A_{j_nj_{n-1}}\lambda_{j_m} \end{align*} where $(\lambda_1,\cdots,\lambda_d)$ is an eigenvector of $A$ corresponding to the eigenvalue $r(A)$, and $S_i=S(p_i)$ for $1\leq i\leq d$. \\ \\ 2. \textbf{(Complete quantum graph): }$\B=\oplus_{a=1}^d{M_{n_a}}$, $\psi:\B\to\C$ a $\delta$-form and $A(\cdot)=\delta^2\psi(\cdot)1_\B$. Then, for $1\leq a,b\leq d$, we have \[\langle\rho_b^{-1},A(\rho_a^{-1})\rangle_\psi=\delta^2\langle\rho_b^{-1}, \psi(\rho_a^{-1})1_\B\rangle_\psi=\delta^2\psi(\rho_a^{-1})\psi(\rho_{b}^{-1})=\delta^2 n_an_b\] so that $D=\delta^2[n_an_b]$. Hence $D$ is an irreducible matrix with $r(D)=\delta^2(\sum_{a=1}^dn_a^2)$. So we get \[\beta=\log(\sum_{a=1}^dn_a^2)\] and the KMS state is unique. Clearly $E_\G$ is faithful and the KMS state $\phi$ on the (local) quantum Cuntz-Krieger algebra is determined by \begin{align*} \phi(S(a_1)\cdots S(a_n)S(b_m)^*\cdots S(b_1)^*)=\frac{\delta_{m,n}}{\delta^{2n}}e^{-n\beta}\psi(b_1^*a_1)\cdots\psi(b_n^*a_n). \end{align*} for $a_i, b_j\in \B$. \\ \\ 3. (\textbf{Rank-one quantum graph}): $\B=\oplus_{a=1}^d M_{n_a}$, $\psi$ a $\delta$-form and $A(x):=TxT^*$ for some $T\in \B$ such that $\Tr(\rho_a^{-1}T^*T)=\delta^2$ for $1\leq a\leq d$. We have \[\langle \rho_b^{-1}, A(\rho_a^{-1})\rangle_\psi=\delta_{ab}\psi(\rho_a^{-1}T\rho_a^{-1}T^*)=\delta_{ab}\Tr(\rho_a^{-1}T^*T)=\delta_{ab}\delta^2\] so that $D=\delta^2 id$. Hence, any vector is an eigenvector of $D$ with the eigenvalue $\delta^2$. So we get $\beta=0$ and each trace on $\B$ gives a tracial KMS state on $\mathcal{O}_{E_{\G}}$. This is not surprising, since by \cite[Proposition 4.10]{BHINW} the local quantum Cuntz-Krieger algebra of a rank-one quantum graph is the same as for the trivial quantum graph. Again $E_\G$ is faithful and the KMS state $\phi$ is determined as: \[ \phi(S(a_1)\cdots S(a_n)S(b_m)^*\cdots S(b_1)^*)=\frac{\delta_{m,n}}{\delta^{2n}}\tau(Tb_n^*T\cdots Tb_2Tb_1^*a_1T^*a_2T^*\cdots T^*a_nT^*)\] for $a_i,b_j\in \B$. \\ \\ 4. (\textbf{Single matrix block}): $\B=M_n, \tau=\frac{\Tr(\cdot)}{n}$, $\psi=\Tr(\rho\cdot)$ so that $\delta^2=\Tr(\rho^{-1})$. In this case, $D=\langle\rho^{-1}, A(\rho^{-1})\rangle_\psi=\Tr(A(\rho^{-1}))$. The KMS state is always unique with inverse temperature point $\beta=\log \frac{1}{\delta^2}\Tr(A(\rho^{-1}))$. \subsection{A classical graph with multiple edges}\label{subsec:int} Out next aim is to show that the entries of the matrix $\frac{1}{\delta^2}D$ are always non-negative integers. For $1\leq b\leq d$ and $1\leq i,j\leq n_b$, write \[Af_{ij}^b=\oplus_{a=1}^d X_{ij}^{ab},\;\;\mbox{ for }X_{ij}^{ab}\in M_{n_a}.\] Set $X^{ab}=[X_{ij}^{ab}]_{1\leq i,j\leq n_b}\in M_{n_b}(M_{n_a})\subseteq M_{n_b}(\B)$. We claim that $\frac{1}{\delta^2}X^{ab}$ is a projection in $M_{n_b}(\B)$. Indeed, since $A$ is $\ast$-preserving, we have \[Af_{ij}^{b}=A((f_{ji}^{b})^*)=(Af_{ji}^{b})^*,\] that is \[X_{ij}^{ab}=(X_{ji}^{ab})^*,\;\;\;1\leq i,j\leq n_b\] so that $X^{ab}=(X^{ab})^*$. We will now show that $\delta^2 X^{ab}=(X^{ab})^2$. We first note that \begin{align*} \delta^2 A(f_{{ij}}^{b})=m(A\otimes A)m^*(f_{ij}^{b})=m(A\otimes A)\left(\sum_{k=1}^{n_b}f_{ik}^b\otimes f_{kj}^b\right)=\sum_{k=1}^{n_b}A(f_{ik}^b)A(f_{kj}^b) \end{align*} so we get \begin{align*} &\delta^2\bigoplus_{a=1}^dX_{ij}^{ab}=\bigoplus_{a=1}^d\sum_{k=1}^{n_b}X_{ik}^{ab}X_{kj}^{ab} \end{align*} that is \begin{align*} \delta^2 X_{ij}^{ab}=\sum_{k=1}^{n_b}X_{ik}^{ab}X_{kj}^{ab}. \end{align*} This shows that $\delta^2 X^{ab}= (X_{ab})^2$ so that $\frac{1}{\delta^2}X^{ab}=(\frac{1}{\delta^2}X^{ab})^2$. Hence $\frac{1}{\delta^2}X^{ab}$ is a projection in $M_{n_b}(\B)$, which implies that its (non-normalized) trace is a natural number. We thus have the following: \begin{align*} \frac{1}{\delta^2}\langle\rho_a^{-1},A(\rho_b^{-1})\rangle_\psi&= \frac{1}{\delta^2}\sum_{k=1}^{n_b}\langle\rho_a^{-1}, A(f_{kk}^b)\rangle_\psi\\ &= \frac{1}{\delta^2}\sum_{k=1}^{n_b}\langle\rho_a^{-1}, \bigoplus_{c=1}^dX_{kk}^{cb}\rangle_\psi\\ &= \frac{1}{\delta^2}\sum_{k=1}^{n_b}\langle\rho_a^{-1}, X_{kk}^{ab}\rangle_\psi\\ &= \frac{1}{\delta^2}\sum_{k=1}^{n_b}\Tr_{M_{n_a}}(X_{kk}^{ab}). \end{align*} The last quantity is nothing but the sum of the trace of the diagonal entries of the matrix $ \frac{1}{\delta^2}X^{ab}$; hence it must be a positive integer, as was claimed. The above calculations mean that the task of finding the KMS states reduces to studying classical graphs with multiple edges. In the next subsection we will introduce an orthonormal basis of the edge correspondence, which will help us in reproving the results about KMS states and will provide an interpretation to the fact that the entries of $\frac{1}{\delta^2} D$ are integers. \subsection{An orthonormal basis of the edge correspondence}\label{subsec:orth} Let us start with a single matrix block, i.e. $\B \simeq M_n$. We have a state defined by $\psi(x):= \op{Tr}(\rho x)$ and we assume that the density matrix $\rho$ is diagonal. Recall that $m^{\ast}(e_{ij}) = \sum_{k=1}^n e_{ik}\rho^{-1} \otimes e_{kj}$, so a quantum adjacency matrix satisfies \[ \sum_{k}A(e_{ik} \rho^{-1}) A(e_{kj}) = \delta^2 A(e_{ij}), \] where $\delta^2= \op{Tr}(\rho^{-1})$. If $A$ has a Kraus decomposition $Ax = \sum_{r} V_{r} x V_{r}^{\ast}$ for some $V_r\in M_n$ then the condition becomes \[ \sum_{r,q,k} V_{r} e_{ik} \rho^{-1} V_{r}^{\ast} V_{q} e_{kj} V_{q}^{\ast} = \delta^2 \sum_{r} V_{r} e_{ij} V_{r}^{\ast}. \] If we note that $\sum_{k} e_{ik} \rho^{-1} V_{r}^{\ast} V_{q} e_{kj} = \op{Tr}(\rho^{-1} V_{r}^{\ast} V_{q}) e_{ij}$, then we see that the condition $\op{Tr}(\rho^{-1} V_{r}^{\ast} V_{q}) = \delta^2 \delta_{rq}$ is sufficient to satisfy the above equality. In fact we can and we will choose Kraus operators to satisfy this condition, using the correspondence between quantum adjacency matrices and operator spaces from \cite[Proposition 3.30]{Was}\footnote{One just has to translate the statement about the KMS inner product from there to the setting of the GNS inner product that is used here.}. We fix the quantum graph $\mathcal{G}=(M_n, A,\psi)$ and we will denote its quantum edge indicator as $\varepsilon$. It is equal to \[ \varepsilon = \frac{1}{\delta^2} \sum_{ij,r} e_{ij}\rho^{-1} \otimes V_{r} e_{ji} V_{r}^{\ast}. \] The $M_n$-valued inner product becomes $\langle a\otimes b, c\otimes d\rangle:= \psi(a^{\ast}c) b^{\ast}d$ and we also get $\langle x\cdot \varepsilon, y\cdot \varepsilon \rangle = \frac{1}{\delta^2} A(x^{\ast}y)$ (see \cite[Theorem 2.5 ]{BHINW}). By a direct computation we check that \[ \varepsilon = \frac{1}{\delta^2} \sum_{i,j,r} V_{r}^{\ast} e_{ij} \rho^{-1} \otimes V_{r} e_{ji}. \] Note that elements of the form $e_{st}\varepsilon = \frac{1}{\delta^2} \sum_{r,j} e_{sj} \otimes V_{r} e_{jt} V_{r}^{\ast}$ span the edge correspondence as a right $M_n$-module. For $1\leq a\leq n$ and appropriate $r$ we define \[ \psi_{ar}:= \sum_{k} e_{ak} V_{r} \rho^{-1} \varepsilon e_{k1} = \frac{1}{\delta^2}\sum_{k,i,j,q} e_{ak} V_{r} \rho^{-1} V_{q}^{\ast} e_{ij} \rho^{-1} \otimes V_{q} e_{ji}e_{k1} \in E_\G. \] Because $e_{ji} e_{k1} = \delta_{ik} e_{j1}$, the left leg of the tensor product becomes $\op{Tr}(V_r \rho^{-1} V_{q}^{\ast}) e_{aj} \rho^{-1} = \delta^2 \delta_{rq} e_{aj} \rho^{-1}$. It follows that \[ \psi_{ar} = \sum_{j} e_{aj}\rho^{-1} \otimes V_{r} e_{j1}. \] Let us check that these elements span the edge correspondence as a right $M_n$-module. We have \[ \sum_{r} \psi_{ar} e_{1t} V_{r}^{\ast} = \delta^2 e_{at} \varepsilon, \] and we know that these elements span the edge correspondence. We will now check that the set $(\psi_{ar})_{a,r}$ is orthogonal: \begin{align*} \langle \psi_{a_1 r_1}, \psi_{a_2 r_2}\rangle &= \sum_{j_1, j_2} \psi(\rho^{-1} e_{j_1 a_1} e_{a_2 j_2} \rho^{-1})e_{1j_1} V_{r_1}^{\ast} V_{r_2} e_{j_2 1} \\ &= \sum_{j_1, j_2} \delta_{a_1 a_2} \delta_{j_1 j_2} (\rho^{-1})_{j_1} e_{1j_1} V_{r_1}^{\ast} V_{r_2} e_{j_2 1}, \end{align*} where we used the fact that $\rho$ is a diagonal matrix. Using it once again, we note that $e_{1j} (\rho^{-1})_{j} = e_{1j} \rho^{-1}$, hence \[ \langle \psi_{a_1 r_1}, \psi_{a_2 r_2}\rangle= \delta_{a_1 a_2} e_{11} \op{Tr}(\rho^{-1} V_{r_1}^{\ast} V_{r_2}) = \delta^2 \delta_{a_1 a_2} \delta_{r_1 r_2} e_{11}. \] If we consider $\phi_{ar}:= \frac{1}{\delta} \psi_{ar}$, then we get an orthogonal set such that $\langle \phi_{ar}, \phi_{ar}\rangle = e_{11}$ is a projection, so we get a (quasi-)orthonormal basis of the edge correspondence. It follows from \cite[Theorem 1.1.]{LN} that \[\op{Tr}_{\tau} (x) = \sum_{a,r} \tau(\langle \phi_{ar}, x \phi_{ar}\rangle),\] where $\tau$ is the normalized trace on $M_n$. We can compute this expression explicitly: \begin{align*} \sum_{a,r} \langle \phi_{ar}, x\phi_{ar}\rangle &= \frac{1}{\delta^2}\sum_{j_1, j_2, a, r} \op{Tr}(\rho \rho^{-1} e_{j_1 a} x e_{a j_2} \rho^{-1}) e_{1 j_1} V_{r}^{\ast} V_{r} e_{j_2 1} \\ &= \op{Tr}(x) \frac{1}{\delta^2} \sum_{j,r} e_{11} \op{Tr}(\rho^{-1} V_{r}^{\ast} V_{r}) = \op{Tr}(x) e_{11} \op{dim}\op{span}\{V_r\}. \end{align*} After applying the normalized trace we obtain \[ \op{Tr}_{\tau}(x) = \tau(x) \op{dim}\op{span}\{V_r\}. \] Note that we have $\langle \rho^{-1}, A(\rho^{-1})\rangle = \sum_{r} \op{Tr}(\rho \rho^{-1} V_{r} \rho^{-1} V_{r}^{\ast}) = \sum_{r} \op{Tr}(\rho^{-1} V_{r}^{\ast} V_{r}) = \delta^2 \op{dim}\op{span}\{V_r\}$, so we can easily express this dimension using the quantum adjacency matrix and we see how it is related to the matrix $D$. We will now consider the case where $\B$ is a multi-matrix algebra. We have $\B:= \bigoplus_{a=1}^{d} M_{n_{a}}$, so the edge correspondence is a sub-bimodule of $\B\otimes \B = \bigoplus_{a,b=1}^{d} M_{n_{a}} \otimes M_{n_{b}}$. Because of that, the edge correspondence $E_\G$ naturally splits into a direct sum $E_{ab}$ of $M_{n_{a}}$-$M_{n_b}$-bimodules that are mutually orthogonal. We will use that to construct an orthonormal basis of $E_\G$. Let us denote by $A_{ba}$ the part of the quantum adjacency matrix mapping $M_{n_{a}}$ to $M_{n_b}$, i.e. we restrict $A$ to $M_{n_a}$ and then project onto $M_{n_b}$. We have a Kraus decomposition $A_{ba}(x) := \sum_{r} V_{ba}^{r} x (V_{ba}^{r})^{\ast}$, where the Kraus operators $V_{ba}^r$ are $n_b\times n_a$ matrices and satisfy \[\op{Tr}(\rho_{a}^{-1} (V_{ba}^{r})^{\ast} V_{ba}^{q}) = \op{Tr}(V_{ba}^{q} \rho_{a}^{-1}(V_{ba}^{r})^{\ast} )=\delta^2 \delta_{rq}.\] The edge indicator is given by \begin{align*} \varepsilon_\G = \frac{1}{\delta^2} \sum_{a,b,i,j,r} e_{ij}^a\rho_a^{-1} \otimes V_{ba}^r e_{ji}^a (V_{ba}^r)^{\ast} \end{align*} which can further be written as follows: \begin{align*} \varepsilon_\G = \frac{1}{\delta^2} \sum_{a,b,i,j,r} (V_{ba}^r)^*e_{ij}^{ba}\rho_a^{-1} \otimes V_{ba}^r e_{ji}^{ab}. \end{align*} where $e_{k\ell}^{ab}$ denotes the matrix unit of $n_a\times n_b$ matrices with $1\leq k\leq n_{a}$ and $1\leq \ell \leq n_{b}$. One can obtain the above expression by writing $V_{ba}^r=\sum_{k,\ell} (V_{ba}^r)_{k\ell}e_{k\ell}^{ba}$ and noting that $e_{ji}^{ab}e_{k\ell}^{bc}=\delta_{ik}e^{ac}_{j\ell}$. We define elements \[ \phi_{ab}^{ir}:= \frac{1}{\delta} \sum_{k} e_{ik}^{a} \rho_{a}^{-1} \otimes V_{ba}^{r} e_{k1}^{ab}, \] so that $\phi_{ab}^{ir} \in M_{n_a} \otimes M_{n_b}$. As above, one can show that $\phi_{ab}^{ir}\in E_\G$. Indeed, a direct calculation using $\Tr(V_{ba}^q\rho_a^{-1}(V_{ba}^r)^*)=\delta^2\delta_{rq}$ and the expressions above for $\varepsilon_\G$ shows that \begin{align*} \phi_{ab}^{ir}= \frac{1}{\delta}\sum_{k} e_{ ik}^{ab} V_{ba}^r\rho_a^{-1}\varepsilon_\G e_{k1}^b\in E_\G. \end{align*} It is clear that if the indices $a$ or $b$ are different then we get orthogonal elements, because the products are zero, so it suffices to see what happens in a given block. We compute \begin{align*} \langle \phi_{ab}^{i_1 r_1}, \phi_{ab}^{i_2 r_2}\rangle &= \frac{1}{\delta^2} \sum_{k_1, k_2} \op{Tr}(\rho_{a} \rho_{a}^{-1} e_{k_1 i_1} e_{i_2 k_2} \rho_{a}^{-1}) e_{1k_{1}}^{ba} (V_{ba}^{r_1})^{\ast} V_{ba}^{r_2} e_{k_{2}1} \\ &= \delta_{i_1 i_2} \delta_{r_1 r_2} e_{11}^{b}, \end{align*} where the computation is exactly the same as in the single block case. This leads us to the next proposition. \begin{proposition} Let $\G=(\B, A, \psi)$ be a quantum graph and let $E_{\G}$ be its edge correspondence. Suppose that $\B\simeq \bigoplus_{a=1}^{d} M_{n_{a}}$ and for each pair $(a,b) \in \{1,\ldots,d\}^2$ let $(V_{ba}^{r})_{r}$ be Kraus operators of $A_{ba}: M_{n_{a}} \to M_{n_b}$. Define $\phi_{ab}^{ir}:= \frac{1}{\delta}\sum_{k} e_{ik}^{a} \rho_{a}^{-1}\otimes V_{ba}^{r}e_{k1}^{ab}$. Then the family $(\phi_{ab}^{ir})_{a,b,i,r}$ is an orthonormal basis of the edge correspondence $E_{\G}$. \end{proposition} We can now use this orthonormal basis to find KMS states. Any tracial state on $\B$ is given by $\tau_{\bm{\lambda}}:= \bigoplus_{a=1}^{d} \lambda_{a} \Tr_{n_a}$, where $\bm{\lambda}:= (\lambda_1, \dots, \lambda_n)$ satisfies $\sum_{a=1}^d \lambda_a n_a = 1$. For any $x=\oplus_{a=1}^d x_a\in\B$ we have \begin{align*} \op{Tr}_{\tau_{\bm{\lambda}}}(x) &= \sum_{a,b,i,r}\tau_{\bm{\lambda}} (\langle \phi_{ab}^{ir}, x\phi_{ab}^{ir}\rangle) \\ &= \frac{1}{\delta^2} \sum_{k_1, k_2, a, b, i, r} \op{Tr}(\rho_{a} \rho_{a}^{-1} e_{k_1 i}^{a} x e_{i k_2} \rho_{a}^{-1})\tau_{\bm{\lambda}} (e_{1 k_1}^{ba} (V_{ba}^{r})^{\ast} V_{ba}^{r} e_{k_2 1}^{ab}) \\ &= \sum_{a,b} \op{Tr}_{n_a}(x_{a}) \op{dim}\op{span}\{V_{ba}^{r}: r\} \lambda_b \end{align*} The KMS condition becomes \[ \sum_{a,b} \op{Tr}_{n_a}(x_{a}) \op{dim}\op{span}\{V_{ba}^{r}:r\} \lambda_b = e^{\beta} \sum_{a} \lambda_{a} \op{Tr}_{n_a}(x_{a}) \] Given the fact that the numbers $\op{Tr}_{n_{a}} (x_{a})$ are arbitrary, we conclude that the vector $\bm{\lambda}$ is a (non-negative) eigenvector of the matrix $[T_{ab}]_{1\leq a,b\leq d}$ where $T_{ab}:= \op{dim}\op{span}\{V_{ba}^{r}\}$ with the eigenvalue $e^{\beta}$. We also note that $T_{ab} = \frac{1}{\delta^2} \langle \rho_{b}^{-1}, A\rho_{a}^{-1}\rangle = \frac{1}{\delta^2} D_{ab}$, which shows why the entries of $\frac{1}{\delta^2} D$ are integers. This reproves Theorem \ref{Thm:KMS} and provides an interpretation for the calculations carried out in Subsection \ref{subsec:int}. \begin{example} We can also handle KMS states for more general actions. Since the edge correspondence $E_{\G}$ naturally splits into a direct sum $\oplus_{a,b=1}^d E_{ab}$ of orthogonal $M_{n_a}$-$M_{n_b}$ bimodules, for each array of numbers $(N_{ab})_{a,b}$ with $N_{ab}>1$ we can consider the action $\Phi_t$ on $E_{\G}$, where $\Phi_t$ acts by multiplication by $N_{ab}^{it}$ on the $(a,b)$ component $E_{ab}$. The computations are very similar to the standard gauge action and the condition we find is that \[ \sum_{b} T_{ab} N_{ab}^{-\beta}\lambda_{b} = \lambda_{a}, \] i.e. the vector $(\lambda_{1},\dots, \lambda_d)$ is an eigenvector of the matrix $(T_{ab} N_{ab}^{-\beta})_{a,b}$ with eigenvalue $1$. \end{example} \section{KMS states (non-tracial cases)} In the non-tracial case one can use \cite[Theorem 3.2 and Theorem 3.5]{LN} to find a condition for KMS states. Once again there is a procedure to induce a weight $\kappa_{\varphi}$ on $\B(X)$ from a state $\varphi$ on $\B$ and the KMS condition is that $\kappa_{\varphi}$ restricted to $\B$ (where $\B$ acts on $X$ from the left) is equal to $\varphi$. Because we have a formula for the left action of $\B$ as compact operators on $X$, one can explicitly write down a condition for KMS states and in some simple cases we will use it. Once again, let $\G=(\B,A,\psi)$ be a quantum graph and let $E_{\G}$ be its edge correspondence. We define the isometry group on $E_{\G}$ as $U_t := e^{it} (\sigma_{-t} \otimes \sigma_{-t})$\footnote{We added the minus sign so that $\psi$ is a KMS state with $\beta=1$, not $\beta=-1$.}, where $\sigma_{t}$ is the modular group of $\psi$. Since we want to assume that $E_{\G}$ is preserved by $U_{t}$, we will assume that $A$ commutes with the modular group. Suppose now that $\varphi$ is a $\beta$-KMS state for the action $\sigma_{-t}$ on $\B$. Consider first the case $\B \simeq M_n$. Then $\psi(x) =\op{Tr}(\rho x)$ and $\varphi(x) = \op{Tr}(\sigma x)$. The $(\sigma_{-t},\beta)$-KMS condition for $\varphi$ means that for all $a,b\in M_n$ we have \[ \op{Tr}(\sigma a b) = \op{Tr}(\sigma b \rho^{\beta} a \rho^{-\beta}). \] By traciality this is equivalent to \[ \op{Tr}(\sigma a b) = \op{Tr}(\rho^{\beta} a \rho^{-\beta} \sigma b), \] so $\sigma a = \rho^{\beta} a \rho^{-\beta}\sigma$, as it happens for all matrices $b$. It follows that $\sigma$ is a scalar multiple of $\rho^{\beta}$; the scalar is unique as $\sigma$ is positive and of trace one. It follows that in the case of a multimatrix algebra on each block we have only one choice for the state and we can just vary the weights, just like in the tracial case. So any $\beta$-KMS $\varphi$ on $\B$ is of the form $\bigoplus_{a=1}^{d} \lambda_{a} \op{Tr}(\rho_{a}^{\beta} \cdot)$, where $\sum_{a=1}^{d} \lambda_a \op{Tr}(\rho_a^{\beta}) = 1$. Fix such a $\varphi$ and look at the induced functional $\kappa_{\varphi}$ on $B(E_\G)$, which on rank one operators is given by $\kappa_{\varphi}(|\xi\rangle\langle \eta|) = \varphi(\langle U_{\frac{i\beta}{2}}\eta, U_{\frac{i \beta}{2}} \xi\rangle)$. By \cite[Theorem 2.5 and Theorem 2.12]{BHINW} we obtain \begin{align*} \kappa_{\varphi}(f_{ij}^{a}) &= \sum_{k=1}^{n_{a}} \varphi(\langle U_{\frac{i\beta}{2}} f_{jk}^{a}\varepsilon_{\G}, U_{\frac{i\beta}{2}}f_{ik}\varepsilon_{\G}\rangle) \\ &= \sum_{k} e^{-\beta} \varphi(\langle \sigma_{-\frac{i\beta}{2}}(f_{jk}^{a}) \varepsilon_{\G}, \sigma_{-\frac{i \beta}{2}}(f_{ik}^{a}) \varepsilon_{\G}\rangle) \\ &= \frac{1}{\delta^2}e^{-\beta} \sum_{k} \varphi(A( (\sigma_{-\frac{i\beta}{2}}(f_{jk}^{a}))^{\ast} \sigma_{-\frac{i \beta}{2}}(f_{ik}^{a}))). \end{align*} We assume that the density matrix of $\psi$ is diagonal, hence we can compute \[\sigma_{-\frac{i\beta}{2}}(f_{jk}^{a}) = \rho^{\frac{\beta}{2}}f_{jk}^{a}\rho^{-\frac{\beta}{2}}=(\psi(e_{jj}^{a}))^{\frac{\beta}{2}} (\psi(e_{kk}^{a}))^{-\frac{\beta}{2}}f_{jk}^{a},\] so we arrive at the expression \[ \frac{e^{-\beta}}{\delta^2} \sum_{k} (\psi(e_{jj}^{a}) \psi(e_{ii}^{a}))^{\frac{\beta}{2}} (\psi(e_{kk}^{a}))^{-\beta} \frac{1}{\psi(e_{ii}^{a})} \varphi(A(\delta_{ij} f_{kk}^{a})). \] This, in turn, is equal to \[ \frac{e^{-\beta}}{\delta^2} \delta_{ij} (\psi(e_{ii}^{a}))^{\beta-1} \sum_{k} (\psi(e_{kk}^{a}))^{-\beta} \varphi(A(f_{kk}^{a})). \] Now note that $\sum_{k} \frac{f_{kk}^{a}}{\psi(e_{kk}^{a})^{\beta}} = \rho_{a}^{-\beta-1}$, so we finally obtain \[ \kappa_{\varphi}(f_{ij}^{a}) = \delta_{ij} \frac{e^{-\beta}}{\delta^2} (\psi(e_{ii}^{a})^{\beta-1} \varphi(A(\rho_{a}^{-\beta-1})). \] On the other hand $\varphi(f_{ij}^{a}) = \delta_{ij}\lambda_{a} (\psi(e_{ii}^{a}))^{\beta-1}$. By noting that $\varphi(X)=\sum_{b=1}^d\lambda_b\langle\rho_b^{\beta-1},X\rangle_\psi$ for all $X\in \B$, the KMS condition gives us the equation \[ \frac{e^{-\beta}}{\delta^2} \sum_{b} \lambda_{b} \langle \rho_b^{\beta-1}, A (\rho_a^{-\beta-1})\rangle_{\psi} = \lambda_{a}, \] which means that $(\lambda_1,\dots,\lambda_d)$ is an eigenvector of the matrix $[D_{ab}]$ where $D_{ab}= \langle \rho_b^{\beta-1}, A (\rho_a^{-\beta-1})\rangle_{\psi}$. This matrix has non-negative entries, because they can be expressed as traces of products of two positive matrices. Note that, unlike the tracial case, $\beta$ not only appears in the eigenvalue but also in the matrix itself. Let us see what happens in the case of a complete quantum graph, i.e. $Ax = \delta^2 \psi(x) \mathds{1}$. Then the left-hand side is equal to \[ e^{-\beta} \sum_{b} \lambda_b \op{Tr}(\rho_b^{\beta}) \op{Tr}(\rho_a^{-\beta}) = e^{-\beta} \op{Tr} (\rho_a^{-\beta}), \] because $\sum_{a=1}^{d} \lambda_a \op{Tr}(\rho_a^{\beta}) = 1$. It follows that $\lambda_a = e^{-\beta} \op{Tr}(\rho_{a}^{-\beta})$. But from the normalization condition for $\lambda_a$'s we obtain the equation for $\beta$: \[ \sum_{a} \op{Tr}(\rho_a^{\beta})\op{Tr}(\rho_a^{-\beta}) = e^{\beta}, \] which is a nonlinear equation and might in general not have a solution, even in the case $d=1$, i.e. a single matrix block. Indeed, consider $M_2$ equipped with a diagonal density matrix (with entries $t$ and $1-t$). Then the equation becomes \[ 2 + (\frac{t}{1-t})^{\beta} + (\frac{1-t}{t})^{\beta} = e^{\beta}. \] For $\beta=0$ the left-hand side is bigger, so if any of the ratios $\frac{1-t}{t}$ or $\frac{t}{1-t}$ are at least equal to $e$ then the left-hand side will be larger for any $\beta>0$, so there will not be a solution. On the other hand, if both ratios are strictly smaller than $e$, then eventually the right-hand side will become larger, so there will be some solution by continuity. \section*{Acknowledgments} The first-named author was partially supported by the Research Foundation - Flanders (FWO) through a Postdoctoral fellowship (1221025N). The second-named author was partially supported by the National Science Center, Poland (NCN) grant no. 2021/43/D/ST1/01446. The project is co-financed by the Polish National Agency for Academic Exchange within the Polish Returns Programme. \vspace{5 pt} \includegraphics[scale=0.5]{logoNAWA.png} \begin{thebibliography}{99999999} \bibitem[BEVW]{BEVW} M.\,Brannan, K.\,Eifler, C.\,Voigt, and M.\,Weber, {\em Quantum Cuntz-Krieger algebras}, Trans.\,Amer.\,Math.\,Soc.\,Ser.\,B \textbf{9} (2022), 782–826. \bibitem[BHINW]{BHINW} M.\,Brannan, M.\,Hamidi, L.\,Ismert, B.\,Nelson, and M.\,Wasilewski, {\em Quantum edge correspondences and quantum Cuntz-Krieger algebras}, J. Lond. Math. Soc. (2) \textbf{107} (2023), no.3, 886–913. \bibitem[CK]{CK}J.~Cuntz and W.~Krieger, \emph{A class of $C^*$-algebras and topological Markov chains,} Invent. Math. \textbf{56} (1980), no. 3, 251–268. \bibitem[EFW]{EFW} M.~Enomoto, M.~Fujii, and Y.~Watatani, \emph{KMS states for gauge action on $\mathcal{O}_{A}$}, Math. Japon. \textbf{29} (1984), 607–619. \bibitem[HJ]{HJ}R.\,A.\,Horn and C.\,R.\,Johnson, {\em Matrix analysis}, Second edition, Cambridge University Press, Cambridge, 2013. \bibitem[LN]{LN}M.\,Laca and S.\,Neshveyev, {\em KMS states of quasi-free dynamics on Pimsner algebras}, J.\,Funct.\,Anal. \textbf{211} (2004), no.2, 457–482. \bibitem[OP]{OP} D.~Olesen and G.K.~Pedersen, \emph{Some $C^{\ast}$-algebras with a single KMS-state}, Math. Scand. \textbf{42} (1978), 111-118. \bibitem[Pim]{Pim} M.V.~Pimsner, \emph{A class of $C^{\ast}$-algebras generalizing both Cuntz-Krieger algebras an crossed products by $\mathbb{Z}$}, in: \emph{Free probability theory}, 189–212, Fields Inst. Commun. \textbf{12}, Amer. Math. Soc., Providence, 1997. \bibitem[Was]{Was} M.~Wasilewski, \emph{On quantum Cayley graphs}, Doc. Math. \textbf{29} (2024), no. 6, 1281-1317. \end{thebibliography} \end{document}
2412.07396v1
http://arxiv.org/abs/2412.07396v1
Processus aléatoires et applications -- Algorithmes MCMC et vitesse de convergence
\documentclass[11pt,a4paper,titlepage,twoside ]{book} \usepackage[utf8x]{inputenc} \usepackage[T1]{fontenc} \usepackage{kpfonts} \usepackage[a4paper,includeheadfoot,pdftex,textwidth=16cm,textheight=24cm, bottom=3.6cm]{geometry} \usepackage[svgnames]{xcolor}\usepackage{graphicx} \usepackage[bookmarks=true, pdfborder={0 0 1},colorlinks=true,urlcolor=blue,citecolor=Purple, linkcolor=NavyBlue,hypertexnames=false]{hyperref} \usepackage{enumitem} \setlist{parsep=0pt} \setlist[itemize,enumerate]{nolistsep,itemsep=3pt,topsep=5pt} \setlist{leftmargin=5mm} \usepackage{fancybox} \usepackage[Lenny]{fncychap} \usepackage{fancyhdr} \setlength{\headheight}{80pt} \usepackage{amsmath} \usepackage{amsfonts} \usepackage{amssymb} \usepackage{amsthm} \usepackage{ upgreek } \usepackage{bbm} \usepackage{mathtools}\usepackage{mdframed} \usepackage{tikz} \usetikzlibrary{matrix,arrows,calc} \usepgflibrary{shapes} \usepgflibrary{fpu} \usepackage{chessboard} \usepackage[margin=10pt,font=small,labelfont=bf, labelsep=endash]{caption} \newcommand{\myrulewidth}{0pt} \definecolor{ThmColor}{rgb}{0.93,0.93,0.995} \definecolor{DefColor}{rgb}{0.92,0.96,0.985} \definecolor{RemColor}{rgb}{0.96,0.93,0.96} \definecolor{ExoColor}{rgb}{0.905,0.995,0.905} \mdfdefinestyle{thmstyle}{backgroundcolor=ThmColor,nobreak,innertopmargin=0pt} \mdfdefinestyle{defstyle}{backgroundcolor=DefColor,nobreak,innertopmargin=0pt} \mdfdefinestyle{remstyle}{backgroundcolor=RemColor,innertopmargin=0pt} \mdfdefinestyle{exostyle}{backgroundcolor=ExoColor,innertopmargin=0pt} \mdtheorem[style=thmstyle]{theorem}{Th\'eor\`eme}[section] \mdtheorem[style=thmstyle]{proposition}[theorem]{Proposition}[section] \mdtheorem[ntheorem,style=thmstyle]{corollary}[theorem]{Corollaire}[section] \mdtheorem[ntheorem,style=thmstyle]{lemma}[theorem]{Lemme}[section] \mdtheorem[ntheorem,style=defstyle]{definition}[theorem]{D\'efinition}[section] \mdtheorem[ntheorem,style=defstyle]{notation}[theorem]{Notation}[section] \mdtheorem[ntheorem,style=defstyle]{assumption}[theorem]{hypoth\`ese}[section] \mdtheorem[ntheorem,style=remstyle]{example}[theorem]{Exemple}[section] \mdtheorem[ntheorem,style=remstyle]{remark}[theorem]{Remarque}[section] \mdtheorem[ntheorem,style=exostyle]{exercise}[theorem]{Exercice}[section] }, \newcommand{\CM}{cha\^ine de Markov} \newcommand{\CCM}{Cha\^ine de Markov} \newcommand{\CMs}{cha\^ines de Markov} \newcommand{\reaches}{\rightsquigarrow} \newcommand{\Tc}{T_{\text{c}}} \newcommand{\myquote}[1]{\guillemotleft\;#1\;\guillemotright} \usepackage{cleveref} \crefname{exercise}{exercise}{exercises} \usepackage{autonum} \tikzset{myxshift/.style = {shift = {(#1, 0)}}} \tikzset{myyshift/.style = {shift = {(0, #1)}}} \newcommand{\pos}[2]{ \def\posx{{#1}} \def\posy{{#2}} } \newcommand{\urntikz} { \begin{scope}[myxshift = \posx] \begin{scope}[myyshift = \posy] \draw[thick,-] (-1.1,1.0) -- (-1.1,0.2) (-1.1,0.2) arc (180:270:0.2) (-0.9,0.0) -- (-0.3,0.0) (-0.3,0.0) arc (-90:0:0.2) (-0.1,0.2) -- (-0.1,1.0) ; \end{scope} \end{scope} } \input{sarajevo.sty} \renewcommand{\partname}{Partie} \renewcommand{\chaptername}{Chapitre} \renewcommand{\proofname}{D\'emonstration} \renewcommand{\bibname}{Bibliographie} \renewcommand{\contentsname}{Table des mati\`eres} \DeclareMathOperator{\pgcd}{pgcd} \newcommand{\vone}{\mathbf{1}} \newcommand{\myvrule}[3]{\vrule height #1 depth #2 width #3} \begin{document} \pagestyle{empty} \newgeometry{margin=1in} \hypersetup{pageanchor=false} \thispagestyle{empty} \vspace*{1cm} \begin{center} {\Huge\bfseries\scshape Processus al\'eatoires et applications \\[1mm] -- Algorithmes MCMC et vitesse de convergence \\[1mm] } \vspace*{12mm} {\large Nils Berglund}\\[2mm] {\large Institut Denis Poisson -- UMR 7013}\\[2mm] {\large Universit\'e d'Orl\'eans, Universit\'e de Tours, CNRS} \vspace*{12mm} {\Large Notes de cours}\\[4mm] \vspace*{12mm} \vspace*{27mm} --- Version du 9 d\'ecembre 2024 ---\\[2mm] \end{center} \hypersetup{pageanchor=true} \cleardoublepage \pagestyle{fancy} \fancyhead[RO,LE]{\thepage} \fancyhead[LO]{\nouppercase{\rightmark}} \fancyhead[RE]{\nouppercase{\leftmark}} \cfoot{} \setcounter{page}{1} \pagenumbering{roman} \restoregeometry \tableofcontents \cleardoublepage \setcounter{page}{1} \pagenumbering{arabic} \part[Cha\^ines de Markov \`a espace d\'enombrable]{Cha\^ines de Markov\\ \`a espace d\'enombrable} \label{part:cm_denombrable} \chapter{Exemples de cha\^ines de Markov} \label{chap:cm_exemple} \section{Textes al\'eatoires} \label{sec:ex_textes} Les \CMs\ ont \'et\'e introduites au d\'ebut du vingti\`eme si\`ecle par le math\'ematicien russe Andrey Markov, dans le but d'\'etudier des suites de variables al\'eatoires non ind\'ependantes. L'une des premi\`ere applications \'etait l'analyse de la distribution de voyelles dans des romans. Dans un \'etat d'esprit similaire, voici trois \myquote{textes}\ g\'en\'er\'es de mani\`ere al\'eatoire~: \begin{enumerate} \item[A.] \begin{mdframed}[innerleftmargin=7mm,innertopmargin=10pt,innerbottommargin=10pt] {\sf YxUV,luUqHCLvE?,MRiKaoiWjyhg nEYKrMFD!rUFUy.qvW;e:FflN.udbBdo!, \\ ZpGwTEOFcA;;RrSMvPjA'Xtn.vP?JNZA;xWP, Cm?;i'MzLqVsAnlqHyk,ghDT \\ :PwSwrnJojRhVjSe?dFkoVRN!MTfiFeemBXITdj m.h d'ea;Jkjx,XvHIBPfFT \\ s I'SLcSX;'X!S, ODjX.eMoLnQttneLnNE!qGRgCJ:BuYAauJXoOCCsQkLcyPO \\ MulKLRtSm;PNpFfp'PfgvIJNrUr t l aXtlA?;TPhPxU:,ZmVGr,,'DIjqZDBY \\ DrkPRiKDYRknDhivt;, LYXDuxNKpjegMvrtfz:JpNTDj'LFmHzXxotRM u.iya \\ UUrgZRcA QmCZffwsNWhddBUPAhJIFJvs.CkKFLJoXef;kCnXrv'uWNcpULYsnl \\ Kg OURmysAnxFjHawwsSpM H;PWPsMaFYLMFyvRWOjbdPlLQIaaspNZkuO'Ns.l \\ jEXO,lxQ'GS;n;H:DH:VWJN :t'JMTUVpKCkVZ'NyKJMGiIbQFXEgDEcWxMBiyo \\ ybRIWIAC deMJnnL;SBAZ?:.UuGnC:B.!lBUT,pT?tyHHLlCvN, mKZgwlMJOJd \\ HHobua;KU.;kADVM?jr'v.SCq:hZLR;lqkmLkhn:ajhBM,gKexDAro,HlczWTv \\ cFmNPt.MudUWPO, sTrWlJdgjoiJd.:d;CpJkJCW;FIRnpMGa;umFysOMAqQtmT \\ pPaYZKtOFYppeE.KFX?SuvcbaDrQ XECelD;cfoQKf?'jCTUaISS;fV:gqoWfSq \\ k:Tf!YuPBANtKhewiNg'ImOFs:UhcExmBjsAaMhBf UVP, 'dcFk;gxJMQGyXI; \\ nVwwfWxS:YXQMELEIObTJiilUYSlOsg.gCqlrN:nEU:irHM'nOLXWUbJLTU re' \\ kk vAwMgt'KgWSxwxqJe,z'OBCrnoIshSCDlZirla,rWNPkc?UgZm GOBX.QylY \\ jOtuF } \end{mdframed} \item[B.] \begin{mdframed}[innerleftmargin=7mm,innertopmargin=10pt,innerbottommargin=10pt] {\sf nsunragetnetelpnlac. pieln tJmends d e.imnqu caa aneezsconns re.tc oml d e c, paeisfuaul irt ssna l df.ieulat a ese t hre edn ro m eeel slsplotasstp etuoMeiiseeaenemzeaeuqpeer enuoco sfehnnir p ts 'mpisu qrd iraLp nFetesa,opQeey rieeaduset Mu\-uisecG il e m ru daeiafasousfnircot i eeedracev ever.nsn iaeulu!,mtel lpa rdbjdide tolr'murunlr bteaaua ieasilureseuavrmoce ntvqm qnurnaunsa.mraayVarinanr eumsu cnponf ciuo .pssre elreeY snrrq aani psu oqoddaiaaomrssloe'avia,loei va eroltrsurdeduuoe ffusir 'th'niIt has,slluoooe tee ?eoxaea slsii i u edtvsear e,Mesatnd o o rvdocaeagiua apugiqn rclt smtee.te, gceade etsn e v in eag ent so ra te, oi seGndd i eeet!dii e ese nanu d sp ul afeen aqelonens ssisaaoe cs eectadegotuudlru i 'c, uuuuts 'tt , dir atermdmuciqedn esovsioieieerxdroie mqso,es rrvteen,r dtei xcalrionuaae e vtmplsz miuqa u aboir br gmcdexptedn pEua't vm vnic eeren ereaa,eegeta u rss nlmxomas ea nsbnt s,eEpeteae teiasbo cd ee tu em ue quee en, sd eeneepeot } \end{mdframed} \item[C.] \begin{mdframed}[innerleftmargin=7mm,innertopmargin=10pt,innerbottommargin=10pt] {\sf cesalu'act, bouleuivoie melarous die ndant leuvoiblue poit pesois deuntaciroverchu llie e lle s r lerchar, laisueuayaissabes vet s cuetr i as, rdetite se d'iretie, de.. nendoules, le pablur e d ! copomouns ppait limmix a r aux urars laie Le r lercret ce c. n'are four nsirepapole pa vr s, nte le efit. itesit, le faun e ju estatusuet usoin prcilaisanonnout ssss l tosesace cole sientt, dent pontrtires. e, l mentoufssss chat Laneus c Chontrouc Ce e. Et deses j'ecci uleus mmon s mauit paga lanse l cont ciquner e c Cha s l'a Jes des s'erattrlunt es de sacouen erends. ve e quns som'a aisajouraite eux lala pour ! a levionible plaint n ss, danetrc ponce con du lez, l danoit, dirvecs'u ce ga vesai : chleme eesanl Pa chiontotes anent fomberie vaud'untitez e esonsan t a ! bondesal'is Ilaies, vapa e ! Lers jestsiee celesu unallas, t. ces. ta ce aielironi mmmileue cecoupe et dennt vanen A la ajole quieet, scemmu tomtemotit me aisontouimmet Le s Prage ges peavoneuse ! blec douffomurrd ntis.. rur, ns ablain i pouilait lertoipr ape. leus icoitth me e e, poiroia s. ! atuepout somise e la as } \end{mdframed} \end{enumerate} Il est clair qu'aucun de ces textes n'a de signification. Toutefois, le texte B.\ semble moins arbitraire que le texte A., et C.\ para\^\i t moins \'eloign\'e d'un texte fran\c cais que B. Il suffit pour cela d'essayer de lire les textes \`a haute voix. Voici comment ces textes ont \'et\'e g\'en\'er\'es. Dans les trois cas, on utilise le m\^eme alphabet de 60 lettres (les 26 minuscules et majuscules, quelques signes de ponctuation et l'espace). \begin{enumerate} \item Pour le premier texte, on a simplement tir\'e au hasard, de mani\`ere ind\'ependante et avec la loi uniforme, des lettres de l'alphabet. \item Pour le second texte, on a tir\'e les lettres de mani\`ere ind\'ependante, mais pas avec la loi uniforme. Les probabilit\'es des diff\'erentes lettres correspondent aux fr\'equences de ces lettres dans un texte de r\'ef\'erence fran\c cais (en l’occurrence, un extrait du {\sl Colonel Chabert}\/ de Balzac). Les fr\'equences des diff\'erentes lettres du texte al\'eatoire sont donc plus naturelles, par exemple la lettre {\sf e} appara\^\i t plus fr\'equemment (dans $13\%$ des cas) que la lettre {\sf z} ($0.2\%$). \item Pour le dernier texte, enfin, les lettres n'ont pas \'et\'e tir\'ees de mani\`ere ind\'ependante, mais d\'ependant de la lettre pr\'ec\'edente. Dans le m\^eme texte de r\'ef\'erence que pr\'e\-c\'edemment, on a d\'etermin\'e avec quelle fr\'equence la lettre {\sf a} est suivie de {\sf a} (jamais), {\sf b} (dans $3\%$ des cas), et ainsi de suite, et de m\^eme pour toutes les autres lettres. Ces fr\'equences ont ensuite \'et\'e choisies comme probabilit\'es de transition lors de la g\'en\'eration du texte. \end{enumerate} Ce proc\'ed\'e peut facilement \^etre am\'elior\'e, par exemple en faisant d\'ependre chaque nouvelle lettre de plusieurs lettres pr\'ec\'edentes. Mais m\^eme avec une seule lettre pr\'ec\'edente, il est remarquable que les textes engendr\'es permettent assez facilement de reconna\^\i tre la langue du texte de r\'ef\'erence, comme en t\'emoignent ces deux exemples: \begin{enumerate} \item[D.] \begin{mdframed}[innerleftmargin=7mm,innertopmargin=10pt,innerbottommargin=10pt] {\sf deser Eld s at heve tee opears s cof shan; os wikey coure tstheevons irads; Uneer I tomul moove t nendoot Heilotetateloreagis his ud ang l ars thine br, we tinond end cksile: hersest tear, Sove Whey tht in t ce tloour ld t as my aruswend Ne t nere es alte s ubrk, t r s; penchike sowo Spotoucthistey psushen, ron icoowe l Whese's oft Aneds t aneiksanging t ungl o whommade bome, ghe; s, ne. torththilinen's, peny. d llloine's anets but whsto a It hoo tspinds l nafr Aneve powit tof f I afatichif m as tres, ime h but a wrove Les des wined orr; t he ff teas be hende pith hty ll ven bube. g Bube d hitorend tr, Mand nd nklichis okers r whindandy, Sovede brk f Wheye o edsucoure, thatovigh ld Annaix; an eer, andst Sowery looublyereis isthalle Base whon ey h herotan wict of les, h tou dends m'dys h Wh on'swerossictendoro whaloclocotolfrrovatel aled ouph rtrsspok, ear'sustithimiovelime From alshis ffad, Spake's wen ee: hoves aloorth erthis n t Spagovekl stat hetubr tes, Thuthiss oud s hind t s potrearall's ts dofe }\footnote{Texte de r\'ef\'erence: Quelques sonnets de Shakespeare.} \end{mdframed} \item[E.] \begin{mdframed}[innerleftmargin=7mm,innertopmargin=10pt,innerbottommargin=10pt] {\sf dendewoch wich iere Daf' lacht zuerckrech, st, Gebr d, Bes. jenditerullacht, keie Un! etot' in To sendenus scht, ubteinraben Qun Jue die m arun dilesch d e Denuherelererufein ien. seurdan s ire Zein. es min? dest, in. maur as s san Gedein it Ziend en desckruschn kt vontimelan. in, No Wimmmschrstich vom delst, esichm ispr jencht sch Nende Buchichtannnlin Sphrr s Klldiche dichwieichst. ser Bollesilenztoprs uferm e mierchlls aner, d Spph! wuck e ing Erenich n sach Men. Sin s Gllaser zege schteun d, Gehrstren ite Spe Kun h Umischr Ihngertt, ms ie. es, bs de! ieichtt f; Ginns Ihe d aftalt veine im t'seir; He Zicknerssolanust, fllll. mmichnennd wigeirdie h Zierewithennd, wast naun Wag, autonbe Wehn eietichank We dessonindeuchein ltichlich bsch n, Ichritienstam Lich uchodigem Din eieiers die it f tlo nensseicichenko Mechtarzaunuchrtzubuch aldert; l von. fteschan nn ih geier Schich Geitelten Deichst Fager Zule fer in vischtrn; Schtih Un Hit ach, dit? at ichuch Eihra! Hich g ure vollle Est unvochtelirn An }\footnote{Texte de r\'ef\'erence: Un extrait du {\sl Faust}\/ de Goethe.} \end{mdframed} \end{enumerate} Cela donne, inversement, une m\'ethode assez \'economique permettant \`a une machine de d\'eterminer automatiquement dans quelle langue un texte est \'ecrit. C'est un exemple tr\`es simplifi\'e d'intelligence artificielle, ex\'ecutant une t\^ache d'apprentissage profond. \section{Mod\`ele d'urnes d'Ehrenfest} \label{sec:ex_Ehrenfest} Ce mod\`ele d'urnes a \'et\'e introduit en 1907 par Paul et Tatjana Ehrenfest, dans le but de comprendre le \myquote{paradoxe}\ de l'irr\'eversibilit\'e. Il s'agit du probl\`eme suivant. Un syst\`eme microscopique, constitu\'e de mol\'ecules qui s'en\-tre\-choquent, ob\'eit, du moins en m\'ecanique classique, aux lois de Newton. Ces lois sont \emph{r\'eversibles}, ce qui signifie que si l'on parvenait \`a filmer les mol\'ecules pendant un intervalle de temps, et qu'on passait le film \`a l'envers, cette \'evolution renvers\'ee ob\'eirait encore aux lois de Newton. Par cons\'equent, rien ne permettrait de dire quel film est pass\'e \`a l'endroit ou \`a l'envers. Dans notre monde macroscopique, en revanche, les ph\'enom\`enes sont pour la plupart \emph{irr\'eversibles}. Un verre qui tombe se brise, mais on n'observe jamais des morceaux de verre s'assembler spontan\'ement. Une goutte de colorant dans de l'eau se dilue au cours du temps, mais on ne voit jamais le colorant dilu\'e se concentrer en un point. Comment se peut-il qu'un syst\`eme r\'eversible \`a l'\'echelle microscopique se comporte de mani\`ere irr\'eversible \`a notre \'echel\-le macroscopique~? \`A un niveau un peu moins macroscopique, consid\'erons deux r\'ecipients, l'un rempli d'un gaz, et l'autre vide. Les deux r\'ecipients sont mis en contact, et au temps $0$, on ouvre une vanne permettant aux mol\'ecules du gaz de se r\'epartir entre les deux r\'ecipients. On observe alors la pression du gaz s'\'equilibrer entre les deux r\'ecipients, mais on ne s'attend pas \`a voir toutes les mol\'ecules spontan\'ement revenir dans un r\'ecipient. \begin{figure} \vspace{-3mm} \begin{center} \begin{tikzpicture}[->,>=stealth',auto,scale=0.9,node distance=3.0cm, thick,main node/.style={circle,scale=0.7,minimum size=0.4cm, fill=green!50,draw,font=\sffamily}] \pos{0}{0} \urntikz \pos{1.2}{0} \urntikz \node[main node] at(0.35,0.2) {}; \node[main node] at(0.85,0.2) {}; \node[main node] at(0.6,0.4) {}; \pos{4}{0} \urntikz \pos{5.2}{0} \urntikz \node[main node] at(4.35,0.2) {}; \node[main node] at(4.85,0.2) {}; \node[main node] at(3.4,0.2) {}; \pos{8}{0} \urntikz \pos{9.2}{0} \urntikz \node[main node] at(7.15,0.2) {}; \node[main node] at(7.65,0.2) {}; \node[main node] at(8.6,0.2) {}; \pos{12}{0} \urntikz \pos{13.2}{0} \urntikz \node[main node] at(11.15,0.2) {}; \node[main node] at(11.65,0.2) {}; \node[main node] at(11.4,0.4) {}; \node[minimum size=2.2cm] (0) at (0.1,0.5) {}; \node[minimum size=2.2cm] (1) at (4.1,0.5) {}; \node[minimum size=2.2cm] (2) at (8.1,0.5) {}; \node[minimum size=2.2cm] (3) at (12.1,0.5) {}; \path[shorten >=.3cm,shorten <=.3cm,every node/.style={font=\sffamily\footnotesize}] (0) edge [bend left,above] node {$1$} (1) (1) edge [bend left,above] node {$2/3$} (2) (2) edge [bend left,above] node {$1/3$} (3) (3) edge [bend left,below] node {$1$} (2) (2) edge [bend left,below] node {$2/3$} (1) (1) edge [bend left,below] node {$1/3$} (0) ; \end{tikzpicture} \end{center} \vspace{-7mm} \caption[]{Le mod\`ele d'urnes d'Ehrenfest, dans le cas de $3$ boules.} \label{fig_ehrenfest} \end{figure} Le mod\`ele des urnes d'Ehrenfest est un mod\`ele al\'eatoire repr\'esentant cette situation. On consid\`ere $N$ boules r\'eparties sur deux urnes. \`A chaque pas de temps, on choisit l'une des $N$ boules uniform\'ement au hasard, et on gref{fig_ehrenfest}). Soit $X_n$ le nombre de boules dans l'urne de gauche au $n$i\`eme pas de temps. On a alors \begin{equation} X_{n+1} = \begin{cases} X_n + 1 & \text{avec probabilit\'e $1 - \frac{X_n}{n}$\;,} \\ X_n - 1 & \text{avec probabilit\'e $\frac{X_n}{n}$\;.} \end{cases} \end{equation} La probabilit\'e de cette transition ne d\'epend que de $X_n$, pas des \'etats aux temps pr\'ec\'edents, et est ind\'ependante des transitions pr\'ec\'edentes. Il s'agit d'un exemple de \CM\ sur $\set{0,1,\dots,N}$, qui a des propri\'et\'es garantissant que la loi de $X_n$ converge vers une loi limite (qui s'av\`ere \^etre une loi binomiale). De plus, on peut calculer le \defwd{temps de r\'ecurrence moyen} vers l'\'etat de d\'epart, $X_0 = N$~: il est \'egal \`a $2^N$. Ceci donne une r\'eponse au paradoxe de l'irr\'eversibilit\'e~: s'il est effectivement possible qu'un \'ev\'enement qui contredit cette irr\'eversibilit\'e arrive (toutes les boules retournent dans l'urne de d\'epart), le temps n\'ecessaire pour l'observer est extr\^emement grand. D\'ej\`a pour $N=1000$, on a \begin{equation} 2^N = 2^{1000} = (2^{10})^{100} > (10^3)^{100} = 10^{300}\;. \end{equation} M\^eme pour un pas de temps d'une nanoseconde ($10^{-9}$ secondes), ce temps est de $10^{291}$ secondes. Une ann\'ee comporte environ $3\cdot 10^7$ secondes, donc il faudra attendre en moyenne plus de $10^{283}$ ans pour voir toutes les mol\'ecules dans le r\'ecipient de gauche, ce qui est largement sup\'erieur \`a l'\^age estim\'e de notre univers. Si $N$ est comparable au nombre d'Avogadro, ce temps de r\'ecurrence est encore beaucoup plus grand. \section{Marches al\'eatoires} \label{sec:ex_MA} Les marches al\'eatoires constituent un exemple relativement simple, et n\'eanmoins tr\`es important de \CMs\ sur un ensemble d\'enombrable infini. Dans ce cas, en effet, $\cX=\Z^d$ est un r\'eseau infini, de dimension $d\in\N^*$. Souvent, on consid\`ere que la \CM\ d\'emarre en $X_0=0$. Ensuite, elle choisit \`a chaque instant l'un des $2d$ sites voisins, selon une loi fix\'ee d'avance. Une \defwd{marche al\'eatoire}\/ sur $\Z^d$ est donc une \CM\ \`a valeurs dans $\Z^d$, de distribution initiale telle que $\prob{X_0 = 0} = 1$, et de probabilit\'es de transition satisfaisant \begin{equation} \label{rw1} \pcond{X_{n+1} = y}{X_n = x} = 0 \qquad \text{si $x=y$ ou $\norm{x-y}>1$\;.} \end{equation} La marche est dite \defwd{sym\'etrique}\/ si \begin{equation} \label{rw2} \pcond{X_{n+1} = y}{X_n = x} = \frac1{2d} \qquad \text{pour $\norm{x-y}=1$\;.} \end{equation} Les trajectoires de la marche al\'eatoire sont des suites de points de $\Z^d$ \`a distance $1$, qu'on a coutume d'identifier \`a la ligne gref{fig_rw2d}). \begin{figure} \begin{center} \begin{tikzpicture}[-,scale=0.5,auto,node distance=1.0cm, thick,main node/.style={draw,circle,fill=white,minimum size=3pt,inner sep=0pt}] \path[->,>=stealth'] (-1,0) edge (13,0) (0,-3) edge (0,3) ; \node at (12.0,0.5) {$n$}; \node at (-1.0,2.5) {$X_n$}; \draw (0,0) node[main node] {} -- (1,1) node[main node] {} -- (2,0) node[main node] {} -- (3,1) node[main node] {} -- (4,2) node[main node] {} -- (5,1) node[main node] {} -- (6,0) node[main node] {} -- (7,-1) node[main node] {} -- (8,0) node[main node] {} -- (9,-1) node[main node] {} -- (10,-2) node[main node] {} -- (11,-1) node[main node] {} ; \end{tikzpicture} \end{center} \vspace{-5mm} \caption[]{Une r\'ealisation d'une marche al\'eatoire unidimensionnelle.} \label{fig_marche1} \end{figure} Notons que $X_n$ est la somme de $n$ variables al\'eatoires ind\'ependantes, de m\^eme loi uniforme sur les $2d$ voisins de $0$ dans $\Z^d$. Ceci permet d'appliquer des th\'eor\`emes limites tels que le th\'eor\`eme central limite \`a l'\'etude de $X_n$ pour $n$ grand. En particulier, l'esp\'erance de $X_n$ est nulle pour tout $n$, et sa variance est proporionnelle \`a $n$. \begin{figure} \begin{center} \begin{tikzpicture}[-,scale=0.5,auto,node distance=1.0cm, thick,main node/.style={draw,circle,fill=white,minimum size=3pt,inner sep=0pt}] \path[->,>=stealth'] (-4,0) edge (8,0) (0,-5) edge (0,3) ; \draw[very thick] (0,0) node[main node,thick] {} -- (0,1) node[main node,thick] {} -- (1,1) node[main node,thick] {} -- (1,0) node[main node,thick] {} -- (2,0) node[main node,thick] {} -- (2,-1) node[main node,thick] {} -- (1,-1) node[main node,thick] {} -- (1,-2) node[main node,thick] {} -- (2,-2) node[main node,thick] {} -- (2,-3) node[main node,thick] {} -- (1,-3) node[main node,thick] {} -- (0,-3) node[main node,thick] {} -- (-1,-3) node[main node,thick] {} -- (-2,-3) node[main node,thick] {} -- (-2,-2) node[main node,thick] {} -- (-1,-2) node[main node,thick] {} -- (-1,-3) node[main node,thick] {} -- (-1,-4) node[main node,thick] {} -- (0,-4) node[main node,thick] {} -- (0,-3) node[main node,thick] {} -- (1,-3) node[main node,thick] {} -- (1,-4) node[main node,thick] {} -- (2,-4) node[main node,thick] {} -- (3,-4) node[main node,thick] {} -- (4,-4) node[main node,thick] {} -- (5,-4) node[main node,thick] {} -- (5,-3) node[main node,thick] {} -- (5,-2) node[main node,thick] {} -- (4,-2) node[main node,thick] {} -- (4,-3) node[main node,thick] {} -- (5,-3) node[main node,thick] {} -- (6,-3) node[main node,thick] {} ; \end{tikzpicture} \end{center} \vspace{-5mm} \caption[]{Une trajectoire d'une marche al\'eatoire en dimension $d=2$.} \label{fig_rw2d} \end{figure} Par exemple, en dimension $d=1$, on trouve \begin{equation} \prob{X_n = x} = \frac1{2^n}\binom{n}{\frac{n+x}2} \qquad \forall x\in\set{-n,-n+2,\dots,n-2,n}\;. \end{equation} \`A une transformation affine pr\`es, $X_n$ suit une loi binomiale (plus pr\'ecis\'ement, $(X_n + n)/2$ suit une loi binomiale). Son esp\'erance est nulle, et sa variance est \'egale \`a $n$. Ceci implique en particulier que la marche va finir par atteindre n'importe quel point de $\Z$ si l'on attend assez longtemps. Par ailleurs, $\prob{X_n = x}$ tend vers $0$ lorsque $n$ tend vers l'infini, pour tout $x$ fix\'e. La loi de $X_n$ n'admet donc pas de loi limite. Des propri\'et\'es similaires sont vraies pour la marche al\'eatoire sym\'etrique sur $\Z^d$. \section{Mod\`ele d'Ising} \label{sec:ex_Ising} Le mod\`ele d'Ising (ou de Lenz--Ising), fut introduit en 1920 par le physicien Wilhelm Lenz, et \'etudi\'e en dimension $1$ par son \'etudiant Ernst Ising. Comme le mod\`ele d'Ehrenfest, ce mod\`ele vient de la physique, plus particuli\`erement de la physique statistique. Il est cens\'e d\'ecrire un ferro-aimant, qui a la propri\'et\'e de s'aimanter spontan\'ement \`a temp\'erature suffisamment basse. On consid\`ere une partie (connexe) $\Lambda$ du r\'eseau $\Z^d$ ($d$ \'etant la dimension du syst\`eme, par exemple $3$), contenant $N$ sites. A chaque site, on attache un \myquote{spin}\ (une sorte d'aimant \'el\'ementaire), prenant valeurs $+1$ ou $-1$. Un choix d'orientations de tous les spins s'appelle une configuration, c'est donc un \'el\'ement de l'espace de configuration gref{fig_ising}). A une configuration $x\in\cX$, on associe l'\'energie \begin{equation} \label{intro1} H(x) = -\sum_{\langle i,j\rangle\in\Lambda} x_ix_j - h \sum_{i\in\Lambda}x_i\;. \end{equation} Ici, la notation $\langle i,j\rangle$ indique que l'on ne somme que sur les paires de spins plus proches voisins du r\'eseau, c'est--\`a--dire \`a une distance $1$. Le premier terme est donc d'autant plus grand qu'il y a de spins voisins diff\'erents. Le second terme d\'ecrit l'interaction avec un champ magn\'etique ext\'erieur $h$. Il est d'autant plus grand qu'il y a de spins oppos\'es au champ magn\'etique. \begin{figure} \begin{center} \begin{tikzpicture}[thick,auto,node distance=0.5cm,every node/.style={font=\sffamily\LARGE}] \draw [fill=yellow!30] (-0.3,-0.3) rectangle (3.8,2.3); \node[blue] (00) {$-$}; \node[red] (10) [right of=00] {$+$}; \node[red] (20) [right of=10] {$+$}; \node[blue] (30) [right of=20] {$-$}; \node[blue] (40) [right of=30] {$-$}; \node[blue] (50) [right of=40] {$-$}; \node[blue] (60) [right of=50] {$-$}; \node[red] (70) [right of=60] {$+$}; \node[red] (01) [above of=00] {$+$}; \node[blue] (11) [right of=01] {$-$}; \node[blue] (21) [right of=11] {$-$}; \node[red] (31) [right of=21] {$+$}; \node[blue] (41) [right of=31] {$-$}; \node[red] (51) [right of=41] {$+$}; \node[blue] (61) [right of=51] {$-$}; \node[red] (71) [right of=61] {$+$}; \node[blue] (02) [above of=01] {$-$}; \node[blue] (12) [right of=02] {$-$}; \node[red] (22) [right of=12] {$+$}; \node[blue] (32) [right of=22] {$-$}; \node[red] (42) [right of=32] {$+$}; \node[red] (52) [right of=42] {$+$}; \node[blue] (62) [right of=52] {$-$}; \node[red] (72) [right of=62] {$+$}; \node[red] (03) [above of=02] {$+$}; \node[blue] (13) [right of=03] {$-$}; \node[red] (23) [right of=13] {$+$}; \node[red] (33) [right of=23] {$+$}; \node[blue] (43) [right of=33] {$-$}; \node[blue] (53) [right of=43] {$-$}; \node[blue] (63) [right of=53] {$-$}; \node[red] (73) [right of=63] {$+$}; \node[blue] (04) [above of=03] {$-$}; \node[red] (14) [right of=04] {$+$}; \node[blue] (24) [right of=14] {$-$}; \node[red] (34) [right of=24] {$+$}; \node[red] (44) [right of=34] {$+$}; \node[blue] (54) [right of=44] {$-$}; \node[red] (64) [right of=54] {$+$}; \node[blue] (74) [right of=64] {$-$}; \end{tikzpicture} \end{center} \vspace{-5mm} \caption[]{Une configuration du mod\`ele d'Ising en dimension $d=2$.} \label{fig_ising} \end{figure} Un principe de base de la physique statistique dit que si un syst\`eme est en \'equilibre thermique \`a temp\'erature $T$, alors il se trouve dans la configuration $x$ avec probabilit\'e proportionnelle \`a $\e^{-\beta H(x)}$ (appel\'ee \defwd{mesure de Gibbs}), o\`u $\beta=1/(k_{\text{B}}T)$, avec $k_{\text{B}}$ une constante physique appel\'ee \defwd{constante de Boltzmann}. A temp\'erature faible, le syst\`eme privil\'egie les configurations de basse \'energie, alors que lorsque la temp\'erature tend vers l'infini, toutes les configurations deviennent \'equiprobables. \begin{figure} \begin{center} \begin{tikzpicture}[>=stealth',main node/.style={circle,minimum size=3pt,inner sep=0pt,fill=white,draw},x=3cm,y=1.7cm, declare function={m(\x) = tanh(2*\x); mm(\x) = tanh(2*\x +0.7);}] \draw[->,semithick] (-1,0) -> (1,0); \draw[->,semithick] (0,-1.1) -> (0,1.2); \draw[blue,very thick,-,smooth,domain=0.0:0.9,samples=50,/pgf/fpu, /pgf/fpu/output format=fixed] plot (\x, {mm(\x)}); \draw[blue,very thick,-,smooth,domain=0.0:0.9,samples=50,/pgf/fpu, /pgf/fpu/output format=fixed] plot (-\x, {-mm(\x)}); \node[] at (0.9,0.15) {$h$}; \node[] at (0.1,1.0) {$m$}; \node[main node] at (0.0, {mm(0)}) {}; \node[main node] at (0.0, {-mm(0)}) {}; \node[] at (-0.23,{mm(0)}) {$m^*(T)$}; \node[] at (0.28,{-mm(0)}) {$-m^*(T)$}; \node[] at (-0.8,0.9) {$T < \Tc$}; \end{tikzpicture} \hspace{5mm} \begin{tikzpicture}[>=stealth',main node/.style={circle,minimum size=0.25cm,fill=blue!20,draw},x=3cm,y=1.7cm, declare function={m(\x) = tanh(2*\x); mm(\x) = tanh(2*\x +0.7);}] \draw[->,semithick] (-1,0) -> (1,0); \draw[->,semithick] (0,-1.1) -> (0,1.2); \draw[blue,very thick,-,smooth,domain=-0.9:0.9,samples=100,/pgf/fpu, /pgf/fpu/output format=fixed] plot (\x, {m(\x)}); \node[] at (0.9,0.15) {$h$}; \node[] at (0.1,1.0) {$m$}; \node[] at (-0.8,0.9) {$T > \Tc$}; \end{tikzpicture} \end{center} \vspace{-5mm} \caption[]{Aimantation du mod\`ele d'Ising en fonction du champ magn\'etique ext\'erieur $h$, \`a gauche pour $T < \Tc$, et \`a droite pour $T > \Tc$.} \label{fig_ising2} \end{figure} L'\defwd{aimantation totale} de l'\'echantillon est donn\'ee par la variable al\'eatoire \begin{equation} \label{intro2} m(x) = \sum_{i\in\Lambda} x_i\;, \end{equation} et son esp\'erance vaut \begin{equation} \label{intro3} \expec m = \dfrac{\displaystyle\sum_{x\in\cX} m(x) \e^{-\beta H(x)}} {\displaystyle\sum_{x\in\cX}\e^{-\beta H(x)}}\;. \end{equation} L'int\'er\^et du mod\`ele d'Ising est qu'on peut montrer l'existence d'une \defwd{transition de phase}, en dimension $d$ sup\'erieure ou \'egale \`a $2$. Dans ce cas il existe une \defwd{temp\'erature critique} $\Tc$ en-dessous de laquelle l'aimantation varie de mani\`ere discontinue en fonction de $h$ dans la limite $N\to\infty$. Pour des temp\'eratures sup\'erieures \`a la valeur gref{fig_ising2}), \begin{itemize} \item l'aimantation est toujours strictement positive si $h > 0$, et strictement n\'egative si $h < 0$; \item si $T \geqs \Tc$, alors l'aimantation tend vers $0$ lorsque $h \to 0$, que ce soit par valeurs positives ou n\'egatives; \item en revanche, si $T < \Tc$, l'aimantation tend vers une valeur strictement positive $m^*(T)$ lorsque $h$ tend vers $0$ par valeurs positives, et vers $-m^*(T)$ lorsque $h$ tend vers $0$ par valeurs n\'egatives. \end{itemize} La quantit\'e $m^*(T)$ s'appelle l'\defwd{aimantation spontan\'ee} du syst\`eme. Elle tend contin\^ument vers $0$ lorsque $T$ tend vers $\Tc$ par la gauche. L'existence de l'aimantation spontan\'ee est importante pour de nombreux dispositifs de stockage de donn\'ees (disques durs, m\'emoires flash). Lorsque des donn\'ees sont sauvegard\'ees sur un tel dispositif, un champ magn\'etique est appliqu\'e localement afin de cr\'eer une aimantation, qui persiste lorsque le champ retombe \`a z\'ero. Des donn\'ees sous forme binaire sont ainsi repr\'esent\'ees par des domaines d'aimantation diff\'erentes, et cette information peut \^etre r\'ecup\'er\'ee par la suite, tant que l'aimant n'est pas port\'e \`a une temp\'erature d\'epassant $\Tc$. \begin{figure} \centerline{ \includegraphics*[clip=true,width=70mm]{figs/glauber060150} \hspace{0.1mm} \includegraphics*[clip=true,width=70mm]{figs/glauber060300} } \vspace{2mm} \centerline{ \includegraphics*[clip=true,width=70mm]{figs/glauber060450} \hspace{0.1mm} \includegraphics*[clip=true,width=70mm]{figs/glauber060600} } \caption[]{Exemple de simulation d'une dynamique de Glauber. Evolution au cours du temps pour $h=1$ et $\beta=0.6$, avec tous les spins initialement \'egaux \`a $-1$ (bleu). Le champ $h$ positif favorise les spins \'egaux \`a $+1$ (jaunes).} \label{fig_glauber} \end{figure} Si l'on veut d\'eterminer num\'eriquement l'aimantation, il suffit en principe de calculer la somme~\eqref{intro3}. Toutefois, cette somme comprend $2^N$ termes, ce qui cro\^it tr\`es rapidement avec la taille du syst\`eme. Par exemple pour un cube de $10\times10\times10$ spins, le nombre de termes vaut $2^{1000}$, ce qui est de l'ordre de $10^{300}$. Un ordinateur calculant $10^{10}$ termes par seconde mettrait beaucoup plus que l'\^age de l'univers \`a calculer la somme. Une alternative est d'utiliser un algorithme dit de Metropolis. Au lieu de parcourir toutes les configurations possibles de $\cX$, on n'en parcourt qu'un nombre limit\'e, de mani\`ere bien choisie, \`a l'aide d'une \CM. Pour cela, on part d'une configuration initiale $x$, puis on transforme cette configuration en retournant un spin choisi au hasard. Plus pr\'ecis\'ement, on n'op\`ere cette transition qu'avec une certaine probabilit\'e, qui d\'epend de la diff\'erence d'\'energie entre les configurations de d\'epart et d'arriv\'ee. L'id\'ee est que si les probabilit\'es de transition sont bien choisies, alors la \CM\ va \'echantillonner l'espace de configuration de telle mani\`ere qu'il suffira de lui faire parcourir une petite fraction de toutes les configurations possibles pour obtenir une bonne approximation de l'aimantation $\expec{m}$. Les questions sont alors \begin{enumerate} \item De quelle mani\`ere choisir ces probabilit\'es de transition~? \item Combien de pas faut-il effectuer pour approcher $\expec{m}$ avec une pr\'ecision donn\'ee~? \end{enumerate} R\'epondre \`a ces deux questions est l'un des objectifs principaux de ce cours. \chapter{Rappels sur les cha\^ines de Markov} \label{chap:cm_rappels} Nous rappelons dans ce chapitre quelques notions de base de la th\'eorie des \CMs, souvent sans d\'emonstration. La plupart des d\'emonstrations peuvent se trouver dans n'im\-por\-te quel bon cours sur les \CMs, comme par exemple~\cite{Durrett1}. \section{D\'efinitions, notations} \label{sec:rap_notation} Soit $\cX$ un ensemble d\'enombrable, fini ou infini. \begin{definition}[Mesure de probabilit\'e, matrice stochastique] \label{def:matrice_stoch} \begin{itemize} \item Une mesure de probabilit\'e $\nu$ sur $\cX$ est un ensemble $(\nu(x))_{x\in\cX}$ de nombres r\'eels positifs ou nuls satisfaisant \begin{equation} \label{eq:mproba} \sum_{x\in\cX} \nu(x) = 1\;. \end{equation} \item Une \defwd{matrice stochastique} sur $\cX$ est un ensemble $P = (p_{xy})_{x,y\in\cX}$ de nombres r\'eels positifs ou nuls satisfaisant \begin{equation} \label{eq:mstoch} \sum_{y\in\cX} p_{xy} = 1 \qquad \forall x\in\cX\;. \end{equation} \end{itemize} \end{definition} Remarquons que puisque les $\nu(x)$ sont positifs ou nuls, la condition~\eqref{eq:mproba} implique qu'ils sont n\'ecessairement tous dans l'intervalle $[0,1]$. Il en va de m\^eme pour les $p_{xy}$. \begin{definition}[Cha\^ine de Markov] On se donne une matrice stochastique $P$ sur $\cX$, et une mesure de probabilit\'e $\nu$ sur $\cX$. Une \defwd{\CM} (homog\`ene en temps) sur $\cX$, de loi initiale $\nu$ et de matrice de transition $P$, est une suite $(X_n)_{n\geqs0}$ de variables al\'eatoires \`a valeurs dans $\cX$, telles que $\prob{X_0 = x} = \nu(x)$ pour tout $x\in\cX$, et satisfaisant la \defwd{propri\'et\'e de Markov} \begin{align} \pcond{X_n = y}{X_0 = x_0, X_1 = x_1, \dots, X_{n-1} = x_{n-1}} &= \pcond{X_n = y}{X_{n-1} = x_{n-1}} \\ &= p_{x_{n-1}y} \end{align} pour tout $n\geqs1$ et tout choix de $x_0, \dots, x_{n-1}, y\in\cX$. \end{definition} Une cons\'equence imm\'ediate de cette d\'efinition est la suivante. \begin{proposition}[Probabilit\'e de trajectoires et loi de $X_n$] \label{prop:proba_traj} Soit $(X_n)_{n\geqs0}$ une \CM\ de loi initiale $\nu$ et de matrice de transition $P$. Alors, pour tout $n\geqs0$ et tout choix de $x_0, \dots, x_n\in\cX$, \begin{equation} \label{eq:proba_traj} \prob{X_0 = x_0, X_1 = x_1, \dots, X_n = x_n} = \nu(x_0)p_{x_0x_1} \dots p_{x_{n-1}x_n}\;. \end{equation} De plus, pour tout $n\geqs1$ et tout $y\in\cX$, on a \begin{equation} \label{eq:proba_nu_y} \prob{X_n = y} = \sum_{x_0\in\cX} \dots \sum_{x_{n-1}\in\cX} \nu(x_0)p_{x_0x_1} \dots p_{x_{n-2}x_{n-1}}p_{x_{n-1}y}\;. \end{equation} \end{proposition} Dans la suite, les notations suivantes vont s'av\'erer pratiques. \begin{itemize} \item On \'ecrira $\probin{\nu}{X_n = y}$ au lieu de $\prob{X_n = y}$ pour insister sur le fait que la loi initiale est $\nu$. \item De mani\`ere similaire, on \'ecrira $\expecin{\nu}{X_n}$ pour l'esp\'erance de $X_n$, partant de la loi $\nu$. \item Soit $\delta_x$ la mesure de probabilit\'e sur $\cX$ donn\'ee par \begin{equation} \delta_x(y) = \begin{cases} 1 & \text{si $y = x$\;,}\\ 0 & \text{sinon\;.} \end{cases} \end{equation} Alors, on \'ecrira souvent $\probin{x}{\cdot}$ et $\expecin{x}{\cdot}$ au lieu de $\probin{\delta_x}{\cdot}$ et $\expecin{\delta_x}{\cdot}$. \item Il sera pratique de voir les mesures de probabilit\'e sur $\cX$ comme des vecteurs ligne. De cette fa\c con, \eqref{eq:proba_nu_y} peut s\'ecrire \begin{equation} \probin{\nu}{X_n = y} = \bigpar{\nu P^n}_y\;. \end{equation} \end{itemize} \begin{definition}[\CCM\ r\'eversible] La \CM\ est dite \defwd{r\'eversible} s'il existe une application $\alpha:\cX\to[0,\infty)$, non identiquement nulle, telle que \begin{equation} \alpha(x) p_{xy} = \alpha(y)p_{yx} \qquad \forall x,y\in\cX\;. \end{equation} Dans ce cas, $\alpha = (\alpha_x)_{x\in\cX}$ est appel\'e un \defwd{vecteur r\'eversible}. \end{definition} Le nom r\'eversible vient de la propri\'et\'e suivante. \begin{proposition}[Renversement du temps] Supposons la \CM\ r\'eversible, pour un vecteur r\'eversible $\alpha$ qui est une mesure de probabilit\'e. Alors \begin{equation} \probin{\alpha}{X_0 = x_0, X_1 = x_1, \dots, X_n = x_n} = \probin{\alpha}{X_0 = x_n, X_1 = x_{n-1}, \dots, X_n = x_0} \end{equation} pour tout $n\in\N$, et tout choix de $x_0, x_1, \dots, x_n\in \cX$. \end{proposition} \begin{proof} Il suit de~\eqref{eq:proba_traj} que \begin{align} \probin{\alpha}{X_0 = x_0, X_1 = x_1, \dots, X_n = x_n} &= \alpha(x_0)p_{x_0 x_1}p_{x_1x_2} \dots p_{x_{n-1}x_n} \\ &= p_{x_1 x_0}\alpha(x_1)p_{x_1x_2} \dots p_{x_{n-1}x_n} \\ &= \dots \\ &= p_{x_1 x_0}p_{x_2x_1} \dots p_{x_nx_{n-1}} \alpha(x_n) \\ &= \alpha(x_n)p_{x_nx_{n-1}}\dots p_{x_2x_1} p_{x_1 x_0}\;. \end{align} ce qui est bien \'egal \`a $\probin{\alpha}{X_0 = x_n, X_1 = x_{n-1}, \dots, X_n = x_0}$. \end{proof} \section{Cha\^ines de Markov irr\'eductibles} \label{sec:rap_irred} \begin{definition}[\'Etat accessible, \CM\ irr\'eductible] \begin{itemize} \item On dit qu'un \'etat $y\in\cX$ est \defwd{accessible} depuis $x\in\cX$ s'il existe $n\geqs0$ tel que \begin{equation} \probin{x}{X_n = y} > 0\;. \end{equation} Dans ce cas, on \'ecrira $x \reaches y$. \item On dit que les \'etats $x$ et $y$ \defwd{communiquent} et on \'ecrit $x \sim y$, si on a \`a la fois $x\reaches y$ et $y\reaches x$. \item La \CM\ est \defwd{irr\'eductible} si $x \sim y$ pour tout $x, y\in\cX$. \end{itemize} \end{definition} On v\'erifie facilement que la relation $\reaches$ est \defwd{r\'eflexive} et \defwd{transitive}~: on a toujours $x\reaches x$, et si $x\reaches y$ et $y\reaches z$, alors on a $x\reaches z$. La relation $\sim$ est r\'eflexive, transitive et \defwd{sym\'etrique}~: si $x \sim y$, alors $y \sim x$. C'est donc une \defwd{relation d'\'equivalence}. On a donc une partition de $\cX$ en \defwd{classes d'\'equivalence}~: \begin{equation} \cX = \bigsqcup_{k\geqs 0} \cX_k\;, \end{equation} o\`u $\sqcup$ signifie la r\'eunion disjointe, et $x \sim y$ si et seulement si $x$ et $y$ appartiennent \`a la m\^eme classe. En particulier, la \CM\ est irr\'eductible si et seulement si elle admet une unique classe d'\'equivalence. On peut associer \`a une \CM\ un graphe orient\'e, dont les sommets sont les \'el\'ements de $\cX$, et dont les ar\^etes sont les couples $(x,y)$ tels que $p_{xy} > 0$ (avec $y\neq x$). Si $\cX$ est fini, une mani\`ere de montrer que la \CM\ est irr\'eductible est d'exhiber un chemin ferm\'e dans ce graphe, c'est-\`a dire une suite $(x_1, \dots, x_m, x_{m+1} = x_1)$, contenant tous les \'elements de $\cX$ au moins une fois, et telle que $p_{x_i x_{i+1}} > 0$ pour tout $i\in\set{1,\dots,m}$. \begin{example}[Marche al\'eatoire sym\'etrique sur $\Z^d$] La marche al\'eatoire sym\'etrique sur $\Z^d$ est irr\'eductible. En effet, pour tout $x, y\in\Z^d$, il existe un chemin reliant $x$ \`a $y$. Ce chemin peut \^etre construit en changeant chaque composante de $x$, par \'etapes successives, d'une unit\'e \`a la fois, jusqu'\`a atteindre $y$. \end{example} \begin{remark}[Classes ouvertes et ferm\'ees] Si la \CM\ n'est pas irr\'eductible, alors une classe $\cX_k$ est \defwd{ferm\'ee} si pour tout $x\in \cX_k$ et tout $y\notin\cX_k$, $y$ n'est pas accessible depuis $x$. Dans ce cas, la restriction de la \CM\ \`a $\cX_k$ est irr\'eductible. Une classe qui n'est pas ferm\'ee est dite \defwd{ouverte}. \end{remark} \section{R\'ecurrence} \label{sec:rap_rec} \begin{definition}[Temps de passage] Soit $x\in\cX$. Le \defwd{temps de passage} (ou \defwd{temps de premier passage}) de la \CM\ en $x$ est la variable al\'eatoire \begin{equation} \tau_x = \inf\setsuch{n\geqs1}{X_n = x}\;, \end{equation} avec la convention $\tau_x = \infty$ si $X_n \neq x$ pour tout $n\geqs1$. Dans le cas particulier o\`u la mesure initiale est $\delta_x$, $\tau_x$ s'appelle \'egalement \defwd{temps de retour} en $x$. \end{definition} Dans la suite, on \'ecrira \begin{equation} \probin{\nu}{\tau_x < \infty} = \lim_{n\to\infty} \probin{x}{\tau_x < n} = 1 - \probin{\nu}{\tau_x = \infty}\;. \end{equation} Attention, par convention la limite lorsque $n\to\infty$ ne comprend \emph{jamais} le terme $n = \infty$. \begin{definition}[R\'ecurrence et transience] \begin{itemize} \item Un \'etat $x\in\cX$ est dit \defwd{r\'ecurrent} si $\probin{x}{\tau_x < \infty} = 1$. \item Un \'etat non r\'ecurrent est dit \defwd{transient}. \item La \CM\ est dite \defwd{r\'ecurrente} si tous ses \'etats sont r\'ecurrents, et \defwd{transiente} si tous ses \'etats sont transients. \end{itemize} \end{definition} Le crit\`ere suivant permet de ramener la question de la r\'ecurrence d'une \CM\ \`a celle d'un petit nombre d'\'etats. \begin{proposition}[R\'ecurrence et communication] Si les \'etats $x$ et $y$ communiquent, alors $y$ est r\'ecurrent si et seulement si $x$ est r\'ecurrent. Par cons\'equent, \begin{itemize} \item si un \'etat d'une classe $\cX_k$ est r\'ecurrent (respectivement transient), alors tous les \'etats de la classe sont r\'ecurrents (respectivement transients); on dit alors que la classe est r\'ecurrente (respectivement transiente); \item si la \CM\ est irr\'eductible, et poss\`ede un \'etant r\'ecurrent (respectivement transient), alors la \CM\ est r\'ecurrente (respectivement transiente). \end{itemize} \end{proposition} \begin{proof}[\textit{D\'emonstration partielle}] Nous allons montrer que si $x$ et $y$ sont dans la m\^eme classe r\'ecurrente, alors \begin{equation} \label{rt8} \probin{x}{\tau_y<\infty} = \probin{y}{\tau_x<\infty} = 1\;. \end{equation} Soit $A_M = \bigcup_{m=1}^M \set{X_m=y}$ l'\'ev\'enement \myquote{la \CM\ visite le site $y$ lors des $M$ premiers pas}. Alors \begin{equation} \label{rt8:1} \lim_{M\to\infty} \fP^x(A_M) = \sum_{m=1}^\infty \probin{y}{\tau_y=m} = 1\;. \end{equation} Soit $n_0$ le plus petit entier tel que $\probin{y}{X_{n_0}=x}>0$. Alors pour tout $M>n_0$, \begin{align} \nonumber \fP^y\Bigpar{A_M\cap\set{X_{n_0}=x}} &= \sum_{n=1}^{M-n_0} \probin{y}{X_{n_0}=x, \tau_y=n_0+n} \\ \nonumber &= \sum_{n=1}^{M-n_0} \probin{y}{X_{n_0}=x, X_1\neq y, \dots, X_{n_0}\neq y} \probin{x}{\tau_y=n} \\ &\leqs \probin{y}{X_{n_0}=x} \sum_{n=1}^{M-n_0}\probin{x}{\tau_y=n}\;. \label{rt8:2} \end{align} La premi\`ere \'egalit\'e suit du fait que la \CM\ ne peut pas retourner en $y$ avant $n_0$ et visiter $x$ au temps $n_0$, par d\'efinition de $n_0$. Nous faisons maintenant tendre $M$ vers l'infini des deux c\^ot\'es de l'in\'egalit\'e. Le membre de gauche tend vers $\probin{y}{X_{n_0}=x}$ en vertu de~\eqref{rt8:1}. Il vient donc \begin{equation} \label{tr8:3} \probin{y}{X_{n_0}=x} \leqs \probin{y}{X_{n_0}=x} \probin{x}{\tau_y<\infty}\;. \end{equation} Comme $\probin{y}{X_{n_0}=x}\neq 0$ et $\probin{x}{\tau_y<\infty}\leqs 1$, on a n\'ecessairement $\probin{x}{\tau_y<\infty}=1$. \end{proof} Pour montrer qu'un \'etat est r\'ecurrent, le cit\`ere suivant est souvent utile en pratique. \begin{theorem}[Crit\`ere de r\'ecurrence] \label{thm:critere_rec} Un \'etat $x\in\cX$ est r\'ecurrent si et seulement si \begin{equation} \sum_{n=0}^\infty \probin{x}{X_n = x} = \infty\;. \end{equation} \end{theorem} La d\'emonstration de ce r\'esultat est bas\'ee sur la relation suivante. \begin{proposition}[\'Equation de renouvellement] \label{prop_rt1} Pour tout $x, y\in\cX$ et tout temps $n\in\N$ on a la relation \begin{equation} \label{rt3} \probin{x}{X_n=y} = \sum_{m=1}^n \probin{x}{\tau_y=m} \probin{y}{X_{n-m}=y}\;. \end{equation} \end{proposition} \begin{proof} En d\'ecomposant sur les temps de premier passage en $y$, il vient \begin{align} \nonumber \probin{x}{X_n=y} &= \sum_{m=1}^n \probin{x}{X_1\neq y, \dots, X_{m-1}\neq y,X_m=y,X_n=y} \\ &= \sum_{m=1}^n \underbrace{\pcondin{x}{X_n=y}{X_1\neq y, \dots, X_{m-1}\neq y,X_m=y}}_{=\pcondin{x}{X_n=y}{X_m=y}=\probin{y}{X_{n-m}=y}} \underbrace{\probin{x}{X_1\neq y, \dots, X_{m-1}\neq y,X_m=y}}_{=\probin{x}{\tau_y=m}}\;, \label{rt3:1} \end{align} o\`u nous avons utilis\'e la propri\'et\'e des incr\'ements ind\'ependants. \end{proof} \begin{proof}[\textit{D\'emonstration du Th\'eor\`eme~\ref{thm:critere_rec}}] \hfill \begin{itemize}[leftmargin=7mm] \item[$\Rightarrow$:] L'\'equation de renouvellement~\eqref{rt3} permet d'\'ecrire \begin{align} \nonumber S\defby \sum_{n=0}^\infty \probin{x}{X_n=x} &= 1 + \sum_{n=1}^\infty \probin{x}{X_n=x} \\ \nonumber &= 1 + \sum_{n=1}^\infty \sum_{m=1}^n \probin{x}{\tau_x=m} \probin{x}{X_{n-m}=x} \\ \nonumber &= 1 + \sum_{m=1}^\infty \probin{x}{\tau_x=m} \sum_{n=m}^\infty \probin{x}{X_{n-m}=x} \\ &= 1 + \underbrace{\sum_{m=1}^\infty \probin{x}{\tau_x=m}}_{=1} \sum_{n=0}^\infty \probin{x}{X_n=x} = 1+S\;. \label{rt4:1} \end{align} Comme $S\in[0,\infty]$, l'\'egalit\'e $S=1+S$ implique n\'ecessairement $S=+\infty$. \item[$\Leftarrow$:] On ne peut pas directement inverser les implications ci-dessus. Cependant, on peut montrer la contrapos\'ee en d\'efinissant pour tout $0<s<1$ les s\'eries enti\`eres \begin{align} \psi(s) &= \sum_{n=0}^\infty \probin{x}{X_n=x} s^n\;, \\ \phi(s) &= \sum_{n=1}^\infty \probin{x}{\tau_x=n} s^n = \expecin{x}{s^{\tau_x}}\;. \label{rt4:2} \end{align} Ces s\'eries ont un rayon de convergence sup\'erieur ou \'egal \`a $1$ car leurs coefficients sont inf\'erieurs ou \'egaux \`a $1$. Un calcul analogue au calcul~\eqref{rt4:1} ci-dessus donne alors \begin{align} \psi(s) &= 1 + \sum_{m=1}^\infty \probin{x}{\tau_x=m} \sum_{n=m}^\infty \probin{x}{X_{n-m}=x}s^n \\ &= 1 + \sum_{m=1}^\infty \probin{x}{\tau_x=m}s^m \sum_{n=0}^\infty \probin{x}{X_n=x}s^{n} = 1 + \psi(s)\phi(s)\;, \label{rt4:3} \end{align} d'o\`u \begin{equation} \label{rt4:4} \psi(s) = \frac{1}{1-\phi(s)}\;. \end{equation} Par cons\'equent, si $\probin{x}{\tau_i<\infty}=\phi(1)<1$, alors on obtient, en prenant la limite $s\nearrow1$, \begin{equation} \label{rt4:5} \sum_{n=0}^\infty \probin{x}{X_n=x} = \lim_{s\nearrow1}\psi(s) = \frac{1}{1-\phi(1)} < \infty\;, \end{equation} ce qui conclut la d\'emonstration. \qed \end{itemize} \renewcommand{\qed}{} \end{proof} \section{R\'ecurrence positive, probabilit\'e invariante} \label{sec:rap_rec_pos} \begin{definition}[R\'ecurrence positive] Un \'etat r\'ecurrent $x\in\cX$ est dit \defwd{r\'ecurrent positif} si \begin{equation} \expecin{x}{\tau_x} < \infty\;. \end{equation} Sinon, l'\'etat est appel\'e \defwd{r\'ecurrent nul}. Une \CM\ r\'ecurrente est dite \defwd{r\'ecurrente positive} si tous ses \'etats sont r\'ecurrents positifs, et \defwd{r\'ecurrente nulle} sinon. \end{definition} La r\'ecurrence positive est \`a nouveau une propri\'et\'e de classe. \begin{proposition}[R\'ecurrence positive et communication] Si les \'etats $x$ et $y$ communiquent, alors $y$ est r\'ecurrent positif si et seulement si $x$ est r\'ecurrent positif. En particulier, si la \CM\ est irr\'eductible et admet un \'etat r\'ecurrent positif, alors la \CM\ est r\'ecurrente positive. \end{proposition} \begin{remark}[Cas d'un $\cX$ fini] \label{rem:rec_Xfini} Si $\cX$ est fini et la \CM\ est irr\'eductible, alors elle est n\'ecessairement r\'ecurrente positive. En effet, l'irr\'eductibilit\'e montre que pour tout $x\in\cX$, on peut trouver un entier fini $m$ tel que \begin{equation} p = \max_{y\in\cX} \probin{y}{\tau_x > m} < 1\;. \end{equation} La propri\'et\'e de Markov implique alors que pour tout $k\geqs1$, on a \begin{equation} \probin{x}{\tau_x > km} \leqs p^k\;. \end{equation} La d\'ecroissance exponentielle des queues de la loi de $\tau_x$ implique que $\expecin{x}{\tau_x} < \infty$. \end{remark} Voici un r\'esultat de r\'ecurrence/transience tr\`es classique, qui se d\'emontre \`a l'aide du Th\'eo\-r\`eme~\ref{thm:rec_pos}. \begin{theorem}[R\'ecurrence/transience de marches al\'eatoires sym\'etriques] La marche al\'eatoire sym\'etrique sur $\Z^d$ est r\'ecurrente nulle si $d\in\set{1,2}$ et transiente si $d\geqs3$. \end{theorem} L'int\'er\^et principal de la d\'efinition de r\'ecurrence positive est li\'e \`a l'existence de probabilit\'es invariantes. \begin{definition}[Mesures et probabilit\'es invariantes] Une mesure sur $\cX$ (c'est-\`a-dire une application $\mu:\cX\to\R_+=[0,\infty)$) est dite \defwd{invariante} si \begin{equation} \label{eq:invariant} \sum_{x\in\cX} \mu(x) p_{xy} = \mu(y) \qquad \forall y\in\cX\;. \end{equation} Si $\mu$ est une mesure de probabilit\'e, on dit que c'est une \defwd{probabilit\'e invariante}. On la notera alors souvent $\pi$. \end{definition} La relation~\eqref{eq:invariant} s'\'ecrit matriciellement \begin{equation} \mu P = \mu\;, \end{equation} c'est-\`a-dire que le vecteur ligne $\mu$ est vecteur propre \`a gauche de $P$, pour la valeur propre $1$. Si $\pi$ est une probabilit\'e invariante, alors \begin{equation} \probin{\pi}{X_n = x} = \pi(x) \qquad \forall x\in\cX\;, \forall n\geqs0\;. \end{equation} \begin{example} Soit $\mu$ une mesure uniforme sur $\Z^d$, c'est-\`a-dire qu'il existe une constante $c\in\R$ telle que $\mu(x) = c$ pour tout $x\in\Z^d$. Alors $\mu$ est une mesure invariante pour la marche al\'eatoire sym\'etrique sur $\Z^d$. Toutefois, $\mu$ n'est pas une mesure de probabilit\'e, car on ne peut pas la normaliser (la somme des $\mu(x)$ vaut soit $0$, si $c=0$, soit est infinie, si $c\neq0$). \end{example} \begin{example} On v\'erifie que la loi binomiale de param\`etres $n$ et $\frac12$ est une probabilit\'e invariante du mod\`ele d'Ehrenfest \`a $n$ boules (voir Exercice~\ref{exo:Ehrenfest}). \end{example} \goodbreak Le lien entre r\'ecurrence positive et probabilit\'e invariante est mis en \'evidence par le r\'esultat suivant. \begin{theorem}[R\'ecurrence positive et probabilit\'e invariante] \label{thm:rec_pos_pi} Soit $(X_n)_{n\geqs0}$ une \CM\ irr\'eductible sur $\cX$. Alors les conditions suivantes sont \'equivalentes~: \begin{enumerate} \item La \CM\ admet une probabilit\'e invariante. \item La \CM\ admet un \'etat r\'ecurrent positif. \item Tous les \'etats $x\in\cX$ sont r\'ecurrents positifs. \end{enumerate} De plus, si ces propri\'et\'es sont v\'erifi\'ees, alors la probabilit\'e invariante est unique, et satisfait \begin{equation} \label{eq:piEtau} \pi(x) = \frac{1}{\expecin{x}{\tau_x}} \qquad \forall x\in\cX\;. \end{equation} \end{theorem} Une mani\`ere de d\'emontrer ce r\'esultat est de fixer un \'etat $z\in\cX$, et de consid\'erer la mesure $\gamma^{(z)}$, d\'efinie par \begin{equation} \label{eq:gamma(y)} \gamma^{(z)}(x) = \biggexpecin{z}{\sum_{n=1}^{\tau_z} \indicator{X_n = x}}\;, \end{equation} qui mesure le nombre moyen de passages en $x$ entre deux passages en $z$. On a alors les propri\'et\'es suivantes. \begin{proposition} \label{prop_stat1} Supposons la \CM\ irr\'eductible et r\'ecurrente. Alors on a pour tout $z\in\cX$~: \begin{enumerate} \item $\smash{\gamma^{(z)}(z)} = 1$; \item $\smash{\gamma^{(z)}}$ est une mesure invariante; \item Pour tout $x\in\cX$, on a $0<\smash{\gamma^{(z)}(x)}<\infty$; \item $\smash{\gamma^{(y)}}$ est l'unique mesure invariante telle que $\smash{\gamma^{(z)}(z)} = 1$. \end{enumerate} \end{proposition} \begin{proof} \hfill \begin{enumerate} \item \'Evident, puisque $\tau_z$ est fini presque s\^urement, $X_{\tau_z}=z$ et $X_n\neq z$ pour $1\leqs n<\tau_z$. \item Nous avons \begin{align} \nonumber \gamma^{(z)}(x) &= \Bigexpecin{z}{\sum_{n=1}^\infty \indexfct{X_n=x,n\leqs\tau_z}} = \sum_{n=1}^\infty \probin{z}{X_n=x,n\leqs\tau_z} \\ \nonumber &= \sum_{y\in\cX} \sum_{n=1}^\infty \probin{z}{X_{n-1}=y,n\leqs\tau_z}p_{yx} \\ &= \sum_{y\in\cX} p_{yx} \sum_{m=0}^\infty \probin{z}{X_m=y,m\leqs\tau_z-1}\;. \label{stat3:1} \end{align} Or la seconde somme dans cette expression peut s'\'ecrire \begin{equation} \label{stat3:2} \Bigexpecin{z}{\sum_{m=0}^{\tau_z-1} \indexfct{X_m=y}} = \Bigexpecin{z}{\sum_{m=1}^{\tau_z} \indexfct{X_m=y}} = \gamma^{(z)}(y)\;, \end{equation} vu que $\probin{z}{X_0=y}=\delta_{zy}=\probin{z}{X_{\tau_z}=y}$. Ceci prouve l'invariance de la mesure $\smash{\gamma^{(z)}}$. \item L'invariance de la mesure implique que pour tout $n\geqs0$, \begin{equation} \label{stat3:3} \gamma^{(z)}(x) = \sum_{y\in\cX}\gamma^{(z)}(y) \probin{y}{X_n=x}\;. \end{equation} En particulier, $1=\gamma^{(z)}(z)\geqs \gamma^{(z)}(y) \probin{y}{X_n=z}$ pour tout $y$. Comme par irr\'eductibilit\'e, il existe un $n$ tel que $\probin{y}{X_n=z}>0$, on en d\'eduit que $\smash{\gamma^{(z)}(y)}<\infty$ pour tout $y$. D'autre part, on a aussi $\smash{\gamma^{(z)}(x)} \geqs \probin{z}{X_n=x}$, qui est strictement positif pour au moins un $n$. \item Soit $\lambda$ une mesure invariante telle que $\lambda(z)=1$. Alors pour tout $y$ on a \begin{equation} \label{stat3:4} \lambda(y) = \sum_{x\neq z} \lambda(x) p_{xy} + p_{zy} \geqs p_{zy}\;. \end{equation} Il vient alors, en minorant $\lambda(x)$ par $p_{zx}$ dans l'expression ci-dessus, \begin{align} \nonumber \lambda(y) &\geqs \sum_{x\neq z} p_{zx}p_{xy} + p_{zy}\\ &= \probin{z}{X_2=y,\tau_z\geqs 2} + \probin{z}{X_1=y,\tau_z\geqs 1} \label{stat3:5} \end{align} Par r\'ecurrence, on trouve donc pour tout $n\geqs1$ ($a\wedge b$ d\'esigne le minimum de $a$ et $b$) \begin{equation} \lambda(y) \geqs \sum_{m=1}^{n+1} \probin{z}{X_m=y,\tau_z\geqs m} = \biggexpecin{z}{\sum_{m=1}^{(n+1)\wedge\tau_k}\indexfct{X_m=y}}\;. \label{stat3:6} \end{equation} Lorsque $n$ tend vers l'infini, le membre de droite tend vers $\smash{\gamma^{(z)}(y)}$. On a donc $\lambda(y)\geqs \smash{\gamma^{(z)}(y)}$ pour tout $y$. Par cons\'equent, $\mu=\lambda-\smash{\gamma^{(z)}}$ est une mesure invariante, satisfaisant $\mu(z)=0$. Comme $\mu(z)=\sum_y\mu(y)\probin{y}{X_n=z}$ pour tout $n$, l'irr\'eductibilit\'e implique $\mu(y)=0$ $\forall y$, donc n\'ecessairement $\lambda=\smash{\gamma^{(z)}}$. \qed \end{enumerate} \renewcommand{\qed}{} \end{proof} \begin{proof}[\textit{D\'emonstration du Th\'eor\`eme~\ref{thm:rec_pos_pi}}] \hfill \begin{itemize}[leftmargin=14mm] \item[{$2\Rightarrow 1:$}] Si $\mu(z)<\infty$ alors $z$ est r\'ecurrent, donc la \CM, \'etant irr\'eductible, est r\'ecurrente. Par la proposition pr\'ec\'edente, $\smash{\gamma^{(z)}}$ est l'unique mesure invariante prenant valeur $1$ en $z$. Or nous avons \begin{equation} \label{stat4:1} \sum_{y\in\cX}\gamma^{(z)}(y) = \biggexpecin{z}{\sum_{n=1}^{\tau_z} \underbrace{\sum_{y\in\cX}\indexfct{X_n=y}}_{=1}} = \expecin{z}{\tau_z} = \mu(z) < \infty\;. \end{equation} Par cons\'equent, la mesure $\pi$ d\'efinie par $\pi(y)=\gamma^{(z)}(y)/\mu(z)$ est une probabilit\'e invariante. \item[{$1\Rightarrow 3:$}] Soit $\pi$ une probabilit\'e invariante, et $z\in\cX$. Alors $\hat\gamma$ d\'efini par $\hat\gamma(y)=\pi(y)/\pi(z)$ est une mesure invariante telle que $\hat\gamma(z)=1$. Par la proposition pr\'ec\'edente, on a n\'ecessairement $\hat\gamma=\smash{\gamma^{(z)}}$. Il suit par le m\^eme calcul que ci-dessus \begin{equation} \label{stat4:2} \expecin{z}{\tau_z} = \sum_{y\in\cX} \hat\gamma(y) = \frac{1}{\pi(z)}\sum_{y\in\cX}\pi(y) = \frac1{\pi(z)} < \infty\;. \end{equation} \item[{$3\Rightarrow 2:$}] \'Evident. \end{itemize} Dans ce cas, l'unicit\'e de la mesure suit de celle de $\gamma^{(z)}$, et la relation~\eqref{eq:piEtau} suit de~\eqref{stat4:2}. \end{proof} Dans le cas particulier d'une \CM\ r\'eversible, la probabilit\'e invariante peut \^etre d\'eduite imm\'ediatement d'un vecteur r\'eversible. \begin{proposition}[Probabilit\'es invariante d'une \CM\ r\'eversible] Soit $(X_n)_{n\geqs0}$ une \CM\ r\'eversible, de vecteur r\'eversible $\alpha$. Alors, si \begin{equation} \cN = \sum_{x\in\cX} \alpha(x) < \infty\;, \end{equation} la \CM\ admet une probabilit\'e invariante, donn\'ee par \begin{equation} \pi(x) = \frac{1}{\cN} \alpha(x) \qquad \forall x\in\cX\;. \end{equation} \end{proposition} \begin{proof} Pour tout $x\in\cX$, on a \begin{equation} \sum_{y\in\cX} \pi(y) p_{yx} = \frac{1}{\cN}\sum_{y\in\cX} \alpha(y) p_{yx} = \frac{1}{\cN}\sum_{y\in\cX} p_{xy} \alpha(x) = \frac{1}{\cN} \alpha(x) = \pi(x)\;. \end{equation} De plus, $\pi$ est bien une mesure de probabilit\'e, puisque la somme des $\pi(x)$ vaut $1$. \end{proof} \begin{figure} \begin{center} \vspace{-5mm} \chessboard[smallboard, boardfontsize=14.4pt, setwhite={nd4},showmover=false, color=red, padding=-0.2em, pgfstyle=circle, markfields={b3,b5,c2,c6,e2,e6,f3,f5} ] \hspace{10mm} \setchessboard{ blackfieldcolor=black!30, setfontcolors} \chessboard[smallboard, showmover=false, boardfontsize=14.4pt, pgfstyle=text, color=blue, text=$8$\bfseries\sffamily, markregion=c3-c3, markregion=d3-d3, markregion=e3-e3, markregion=f3-f3, markregion=c4-c4, markregion=d4-d4, markregion=e4-e4, markregion=f4-f4, markregion=c5-c5, markregion=d5-d5, markregion=e5-e5, markregion=f5-f5, markregion=c6-c6, markregion=d6-d6, markregion=e6-e6, markregion=f6-f6, color=blue!80, text=$6$\bfseries\sffamily, markregion=c2-c2, markregion=d2-d2, markregion=e2-e2, markregion=f2-f2, markregion=c7-c7, markregion=d7-d7, markregion=e7-e7, markregion=f7-f7, markregion=b3-b3, markregion=b4-b4, markregion=b5-b5, markregion=b6-b6, markregion=g3-g3, markregion=g4-g4, markregion=g5-g5, markregion=g6-g6, color=blue!70, text=$4$\bfseries\sffamily, markregion=c1-c1, markregion=d1-d1, markregion=e1-e1, markregion=f1-f1, markregion=c8-c8, markregion=d8-d8, markregion=e8-e8, markregion=f8-f8, markregion=a3-a3, markregion=a4-a4, markregion=a5-a5, markregion=a6-a6, markregion=h3-h3, markregion=h4-h4, markregion=h5-h5, markregion=h6-h6, markregion=b2-b2, markregion=g2-g2, markregion=b7-b7, markregion=g7-g7, color=blue!60, text=$3$\bfseries\sffamily, markregion=b1-b1, markregion=a2-a2, markregion=g1-g1, markregion=h2-h2, markregion=b8-b8, markregion=a7-a7, markregion=g8-g8, markregion=h7-h7, color=blue!50, text=$2$\bfseries\sffamily, markregion=a1-a1, markregion=h1-h1, markregion=a8-a8, markregion=h8-h8 ] \end{center} \vspace{-5mm} \caption[]{Mouvements permis du cavalier sur l'\'echiquier. Nombre de mouvements possibles \`a partir de chaque case.} \label{fig_echecs} \end{figure} \begin{example}[Le cavalier fou] Un cavalier se d\'eplace sur un \'echiquier standard (de $64$ cases), en choisissant \`a chaque pas l'un des mouvements permis par les r\`egles du jeu des \'echecs, uniform\'ement gref{fig_echecs}). La position du cavalier est d\'ecrite par une \CM\ sur l'ensemble $\cX$ des $64$ cases de l'\'echiquier. Si $\alpha(x)$ d\'esigne le nombre de mouvements permis en partant de la case $x$, alors les probabilit\'es de transition sont donn\'ees par \begin{equation} p_{xy} = \begin{cases} \frac{1}{\alpha(x)} & \text{si le mouvement de $x$ vers $y$ est permis\;,}\\ 0 & \text{sinon\;.} \end{cases} \end{equation} On v\'erifie que $\alpha$ est un vecteur r\'eversible, et que $\cN = \sum_{x\in\cX} \alpha(x) = 336$ gref{fig_echecs}). La \CM\ est donc r\'eversible, et admet la probabilit\'e invariante $\pi$ donn\'ee par \begin{equation} \pi(x) = \frac{\alpha(x)}{336}\;. \end{equation} Le Th\'eor\`eme~\ref{thm:rec_pos_pi} permet alors de calculer le temps de r\'ecurrence moyen vers n'importe quel \'etat. Celui-ci vaut \begin{equation} \expecin{x}{\tau_x} = \frac{1}{\pi(x)} = \frac{336}{\alpha(x)}\;. \end{equation} \end{example} \section{Ap\'eriodicit\'e, convergence vers la probabilit\'e invariante} \label{sec:rap_conv} \begin{definition}[P\'eriode] La \defwd{p\'eriode} d'un \'etat $x\in\cX$ est le nombre \begin{equation} d_x = \pgcd\bigsetsuch{n\geqs1}{\probin{x}{X_n = i} > 0}\;. \end{equation} Si $d_x = 1$, alors on dit que $x$ est \defwd{ap\'eriodique}. Si tout $x\in\cX$ est ap\'eriodique, on dit que la \CM\ est ap\'eriodique. \end{definition} La p\'eriode est \`a nouveau un propri\'et\'e de classe. \begin{proposition}[P\'eriode et communication] Si $x \sim y$, alors $d_x = d_y$. Par cons\'equent, si la \CM\ est irr\'eductible et admet un \'etat ap\'eriodique, alors la \CM\ est ap\'eriodique. \end{proposition} \begin{example}[Marche al\'eatoire sym\'etrique sur $\Z^d$] Pour la marche al\'eatoire sym\'etrique sur $\Z^d$, la p\'eriode de l'\'etat $0$ vaut $d_0 = 2$. En effet, partant de $0$, la marche ne peut retourner en $0$ qu'au temps pairs. Par cons\'equent, la marche n'est pas ap\'eriodique (tous les \'etats sont de p\'eriode $2$). \end{example} L'importance de la notion d'ap\'eriodicit\'e vient du r\'esultat crucial suivant. \begin{theorem}[Convergence vers la probabilit\'e invariante] \label{thm:convergence_aperiodique} Soit $(X_n)_{n\geqs0}$ une \CM\ irr\'eductible, ap\'eriodique et r\'ecurrente positive, et soit $\pi$ son unique probabilit\'e invariante. Alors pour toute loi initiale $\nu$ et tout $x\in\cX$, on a \begin{equation} \lim_{n\to\infty} \probin{\nu}{X_n = x} = \pi(x)\;. \end{equation} \end{theorem} Nous allons esquisser l'id\'ee principale d'une d\'emonstration de ce th\'eor\`eme, due \`a Wolfgang Doeblin. Consid\'erons deux \CMs\ ind\'ependantes, $(X_n)_{n\geqs0}$ et $(Y_n)_{n\geqs0}$, ayant les deux la m\^eme matrice de transition $P$, mais la premi\`ere partant de $\nu$, alors que la seconde part de $\pi$. Le couple $(X_n,Y_n)$ est une \CM\ sur $\cX\times\cX$, de probabilit\'es de transition \begin{equation} p^\star_{(x,y),(u,v)} = p_{xu}p_{yv}\;, \end{equation} et de loi initiale $\rho = \nu\otimes\pi$, d\'efinie par \begin{equation} \rho(x,y) = \nu(x)\pi(y)\;. \end{equation} On montre alors (\`a l'aide du th\'eor\`eme de B\'ezout) que cette \CM\ est encore irr\'eductible et ap\'eriodique. Comme elle admet la probabilit\'e invariante $\pi\otimes\pi$, elle est aussi r\'ecurrente positive. Soit alors \begin{equation} \label{eq:tau_Delta} \tau_\Delta = \inf\bigsetsuch{n\geqs0}{X_n = Y_n} \end{equation} le temps de passage sur la \defwd{diagonale} $\Delta = \setsuch{(x,x)}{x\in\cX}$. On d\'eduit de la r\'ecurrence positive que $\tau_\Delta$ est presque s\^urement fini. Introduisons alors le processus $(Z_n)_{n\geqs0}$, d\'efini par \begin{equation} Z_n = \begin{cases} X_n & \text{si $n<\tau_\Delta$\;,}\\ Y_n & \text{si $n\geqs\tau_\Delta$\;.} \end{cases} \end{equation} Il suit de l'expression~\eqref{eq:proba_traj} de la probabilit\'e d'une trajectoire que $(Z_n)_{n\geqs0}$ est une \CM\ de loi initiale $\nu$ et de matrice de transition $P$. Par cons\'equent, $Z_n$ est \'egal en loi \`a $X_n$ pour tout $n\geqs0$. Ceci implique que pour tout $n\in\N$ et tout $x\in\cX$, on a \begin{equation} \label{eq:proof_conv_Doeblin} \probin{\rho}{X_n = x,\tau_\Delta \leqs n} = \probin{\rho}{Z_n = x,\tau_\Delta \leqs n} = \probin{\rho}{Y_n = x,\tau_\Delta \leqs n}\;. \end{equation} La premi\`ere \'egalit\'e suit de l'\'egalit\'e en loi de $X_n$ et $Y_n$, alors que la seconde vient du fait que $Z_n = Y_n$ pour $\tau_\Delta \leqs n$. On observe maintenant que pour tout $n\in\N$ et tout $x\in\cX$, on a \begin{align} \probin{\nu}{X_n = x} &= \probin{\rho}{X_n = x, \tau_\Delta \leqs n} + \probin{\rho}{X_n = x, \tau_\Delta > n}\;, \\ \pi(x) = \probin{\pi}{Y_n = x} &= \probin{\rho}{Y_n = x, \tau_\Delta \leqs n} + \probin{\rho}{Y_n = x, \tau_\Delta > n}\;. \end{align} En prenant la diff\'erence et en utilisant~\eqref{eq:proof_conv_Doeblin}, on obtient \begin{equation} \bigabs{\probin{\nu}{X_n = x} - \pi(x)} \leqs \bigabs{\probin{\rho}{X_n = x, \tau_\Delta > n} - \probin{\rho}{Y_n = x, \tau_\Delta > n}} \leqs 2 \probin{\rho}{\tau_\Delta > n}\;. \end{equation} La \CM\ $(X_n,Y_n)_{n\geqs0}$ \'etant r\'ecurrente positive, cette quantit\'e tend vers $0$ lorsque $n$ tend vers l'infini, ce qui prouve le th\'eor\`eme. En fait, on a m\^eme obtenu un peu mieux~: pour tout $n\geqs0$, on a \begin{equation} \label{eq:majo_couplage} \sum_{x\in\cX} \bigabs{\probin{\nu}{X_n = x} - \pi(x)} \leqs 2 \probin{\rho}{\tau_\Delta > n}\;. \end{equation} Si on arrive \`a majorer la probabilit\'e $\probin{\rho}{\tau_\Delta > n}$, on obtient donc une majoration d'une distance entre la loi de $X_n$ et $\pi$ (il s'agit d'une distance du type $\ell^1$). C'est un exemple de ce qu'on appelle un \defwd{argument de couplage}. \section{Exercices} \label{sec:rap_exo} \begin{exercise} \label{exo:Ehrenfest} On consid\`ere le mod\`ele des urnes d'Ehrenfest \`a $N$ boules, c'est-\`a-dire la \CM\ sur l'ensemble $\cX = \set{0,1,\dots N}$ de probabilit\'es de transition \[ p_{xy} = \begin{cases} \frac{x}{N} & \text{si $y=x-1$\;,} \\ 1-\frac{x}{N} & \text{si $y=x+1$\;,} \\ 0 & \text{sinon\;.} \end{cases} \] \begin{enumerate} \item Montrer que cette cha\^ine de Markov est irr\'eductible. Est-elle ap\'eriodique\,? \item Montrer que la distribution de probabilit\'e invariante de cette cha\^ine de Markov suit une loi bin\^omiale, dont on pr\'ecisera les param\`etres. \end{enumerate} \end{exercise} \begin{exercise} Soit $\cG=(V,E)$ un graphe non orient\'e connexe fini. Soit $(X_n)_{n\geqs0}$ la \CM\ sur $V$ construite en choisissant pour $X_{n+1}$, de mani\`ere \'equiprobable, l'un des sommets adjacents \`a $X_n$. \begin{enumerate} \item Montrer que le nombre de voisins de chaque site forme un vecteur r\'eversible. \item En d\'eduire une expression pour la probabilit\'e invariante de la \CM. \end{enumerate} \end{exercise} \begin{exercise} Soit $p\in[0,1]$. On consid\`ere la \CM\ suivante sur $\cX=\N$: \begin{center} \begin{tikzpicture}[->,>=stealth',shorten >=2pt,shorten <=2pt,auto,node distance=3.0cm, thick,main node/.style={circle,scale=0.7,minimum size=1.1cm, fill=blue!20,draw,font=\sffamily\Large}] \node[main node] (0) {$0$}; \node[main node] (1) [right of=0] {$1$}; \node[main node] (2) [right of=1] {$2$}; \node[main node] (3) [right of=2] {$3$}; \node[node distance=2cm] (4) [right of=3] {$\dots$}; \path[every node/.style={font=\sffamily\small}] (0) edge [loop left,left,distance=1.5cm,out=-150,in=150] node {$1-p$} (0) (0) edge [bend left,above] node {$p$} (1) (1) edge [bend left,above] node {$p$} (2) (2) edge [bend left,above] node {$p$} (3) (3) edge [bend left,above] node {$p$} (4) (1) edge [bend left,below] node {$1-p$} (0) (2) edge [bend left,below] node {$1-p$} (1) (3) edge [bend left,below] node {$1-p$} (2) (4) edge [bend left,below] node {$1-p$} (3) ; \end{tikzpicture} \end{center} \begin{enumerate} \item Pour quelles valeurs de $p$ la \CM\ est-elle irr\'eductible? On suppose dans la suite que $p$ est tel que la \CM\ soit irr\'eductible. \item La \CM\ est-elle ap\'eriodique? \item On suppose que la \CM\ est r\'eversible, et soit $\alpha$ un vecteur r\'eversible. Ecrire une relation de r\'ecurrence pour les composantes de $\alpha$, et en d\'eduire $\alpha_n$ en fonction de $\alpha_0$. \item Pour quelles valeurs de $p$ la \CM\ admet-elle une probabilit\'e invariante $\pi$? D\'eter\-miner $\pi$ pour ces valeurs de $p$. \item Pour quelles valeurs de $p$ la \CM\ est-elle r\'ecurrente? R\'ecurrente positive? \item D\'eterminer le temps de r\'ecurrence moyen $\expecin{0}{\tau_0}$. \item Calculer la position moyenne $\expecin{\pi}{X_n}$ pour les valeurs de $p$ telles que $\pi$ existe. \end{enumerate} \end{exercise} \begin{exercise} On consid\`ere une marche al\'eatoire unidimensionnelle sym\'etrique sur l'en\-semble $\cX = \set{0,1,\dots,N}$ avec conditions aux bords absorbantes, c'est-\`a-dire que l'on suppose que $p_{00} = p_{NN} = 1$. Soit \[ \tau = \tau_0 \wedge \tau_N = \inf\bigsetsuch{n\geqs0}{X_n\in\set{0,N}} \] le temps d'absorption, et soit \[ p(x) = \probin{i}{X_\tau=N}\;. \] \begin{enumerate} \item D\'eterminer $p(0)$ et $p(N)$. \item Montrer que pour tout $x\in\set{1,\dots,N-1}$, on a \[ p(x) = \frac12 \bigbrak{p(x-1)+p(x+1)}\;. \] Une fonction $f:\Z\supset A\to\R$ telle que $f(x) = \frac12 \brak{f(x-1)+f(x+1)}$ pour tout $x\in A$ est appel\'ee \emph{harmonique}\/ (discr\`ete). \item Montrer (par l'absurde) le \emph{principe du maximum}: Une fonction harmonique sur $A$ ne peut atteindre son minimum et son maximum qu'au bord de $A$ (on pourra supposer $A$ de la forme $A=\set{a,a+1,\dots,b-1,b}$, dans ce cas son bord est $\partial A=\set{a,b}$). \item Montrer que si $f$ et $g$ sont deux fonctions harmoniques sur $A$, alors toute combinaison lin\'eaire de $f$ et $g$ est encore harmonique. \item Montrer que si $f$ et $g$ sont deux fonctions harmoniques sur $A$, qui co\"\i ncident sur le bord de $A$, alors elles sont \'egales partout dans $A$ (consid\'erer $f-g$). \item Montrer que toute fonction lin\'eaire $f(x)=cx+h$ est harmonique. \item En utilisant les points 1., 2., 5.~et 6., d\'eterminer la fonction $p$. \end{enumerate} \end{exercise} \begin{exercise} On consid\`ere une marche al\'eatoire sym\'etrique sur $\cX=\set{0,1,\dots,N}$, avec conditions au bord absorbantes, c'est-\`a-dire que d\`es que la marche atteint l'un des \'etats $0$ ou $N$, elle y reste ind\'efiniment. Soit \[ \tau = \inf\setsuch{n\geqs 0}{X_n\in\set{0,N}} \] le temps d'absorption. Par convention, $\tau=0$ si $X_0\in\set{0,N}$. Pour $\lambda\in\R$ et $i\in\cX$ on pose \[ f(x,\lambda) = \bigexpecin{x}{\e^{-\lambda\tau}\indexfct{X_\tau=N}} = \begin{cases} \bigexpecin{x}{\e^{-\lambda\tau}} & \text{si $X_\tau=N$\;,} \\ 0 & \text{sinon\;.} \end{cases} \] \begin{enumerate} \item Que valent $f(0,\lambda)$ et $f(N,\lambda)$? \item Montrer que pour tout $x\in\set{1,\dots,N-1}$, \[ \probin{x}{\tau=n} = \frac12 \bigbrak{\probin{x-1}{\tau=n-1} + \probin{x+1}{\tau=n-1}}\;. \] \item Montrer que pour tout $x\in\set{1,\dots,N-1}$, \[ f(x,\lambda) = \frac12\e^{-\lambda} \bigbrak{f(x-1,\lambda) + f(x+1,\lambda)}\;. \] \item Trouver une relation entre $c$ et $\lambda$ telle que l'\'equation ci-dessus pour $f$ admette des solutions de la forme $f(x,\lambda)=\e^{cx}$. Montrer \`a l'aide d'un d\'eveloppement limit\'e que \[ c^2 = 2\lambda + \Order{\lambda^2}\;. \] \item D\'eterminer des constantes $a$ et $b$ telles que \[ \bigexpecin{x}{\e^{-\lambda\tau}\indexfct{X_\tau=N}} = a \e^{cx} + b \e^{-cx}\;. \] \item Effectuer un d\'eveloppement limit\'e au premier ordre en $\lambda$ de l'\'egalit\'e ci-dessus. En d\'eduire \[ \probin{x}{X_\tau=N}\;. \] \item Calculer \[ \bigexpecin{x}{\tau \indexfct{X_\tau=N}}\;. \] \item Sans faire les calculs, indiquer comment proc\'eder pour d\'eterminer la variance de la variable al\'eatoire $\tau \indexfct{X_\tau=N}$ et l'esp\'erance et la variance de $\tau$. \end{enumerate} On rappelle les d\'eveloppements limit\'es suivants: \begin{align} \cosh(x) &= \frac{\e^x+\e^{-x}}{2} = 1 + \frac{1}{2!}x^2 + \Order{x^4}\;, \\ \sinh(x) &= \frac{\e^x-\e^{-x}}{2} = x + \frac{1}{3!}x^3 + \Order{x^5}\;. \end{align} \end{exercise} \chapter{Th\'eorie spectrale et vitesse de convergence} \label{chap:cm_spectrale} Dans ce chapitre et le suivant, nous allons consid\'erer des \CMs\ $(X_n)_{n\geqs0}$ irr\'eductibles, r\'ecurrentes positives et ap\'eriodiques sur un ensemble d\'enombrable $\cX$. Soit $f:\cX\to\R$ une fonction born\'ee, et soit $\pi$ la probabilit\'e invariante de la \CM. Le but est d'estimer la quantit\'e \begin{equation} \expecin{\pi}{f} = \sum_{x\in\cX} \pi(x) f(x)\;. \end{equation} Nous savons par le Th\'eor\`eme~\ref{thm:convergence_aperiodique} que l'on a \begin{equation} \expecin{\pi}{f} = \lim_{n\to\infty} \sum_{x\in\cX} \probin{\nu}{X_n = x}f(x) = \lim_{n\to\infty} \expecin{\nu}{f(X_n)}\;, \end{equation} pour toute loi initiale $\nu$. Notre but est maintenant de majorer l'erreur \begin{equation} \label{eq:erreur_expecf} \bigabs{\expecin{\nu}{f(X_n)} - \expecin{\pi}{f}}\;. \end{equation} Une premi\`ere mani\`ere de le faire est la suivante. \begin{lemma}[Couplage et vitesse de convergence] Si la \CM\ est ap\'eriodique, alors \begin{equation} \bigabs{\expecin{\nu}{f(X_n)} - \expecin{\pi}{f}} \leqs 2 \probin{\nu\otimes\pi}{\tau_\Delta > n} \sup_{x\in\cX} \abs{f(x)}\;, \end{equation} o\`u $\tau_\Delta$ est d\'efini dans~\eqref{eq:tau_Delta}. \end{lemma} \begin{proof} On a \begin{equation} \expecin{\nu}{f(X_n)} - \expecin{\pi}{f} = \sum_{x\in\cX} \bigbrak{\probin{\nu}{X_n = x} - \pi(x)} f(x)\;. \end{equation} Le r\'esultat suit donc de~\eqref{eq:majo_couplage}. \end{proof} Si l'on arrive \`a contr\^oler $\probin{\rho}{\tau_\Delta > n}$, on obtient donc la majoration souhait\'ee. Toutefois, cela n'est pas toujours possible, et on doit alors avoir recours \`a d'autres approches. Dans ce chapitre, nous allons discuter comment l'erreur~\eqref{eq:erreur_expecf} d\'epend de quantit\'es li\'ees aux valeurs propres et vecteurs propres de la matrice de transition $P$. Une autre approche, plus robuste, bas\'ee sur les fonctions de Lyapounov, sera discut\'ee dans le chapitre suivant. \section{Quelques exemples simples} \label{sec:spec_exemples} \begin{example} Consid\'erons la matrice stochastique \begin{equation} P = \begin{pmatrix} 0 & 1 \\ 1 & 0 \end{pmatrix}\;. \end{equation} La \CM\ sur $\cX=\set{1,2}$ associ\'ee est irr\'eductible, r\'ecurrente positive, mais pas ap\'eriodique~: sa p\'eriode est \'egale \`a $2$. En fait, on a \begin{equation} P^n = \begin{cases} P & \text{si $n$ est impair\;,}\\ \one & \text{si $n$ est pair\;,} \end{cases} \end{equation} o\`u $\one$ d\'enote la matrice identit\'e. Par cons\'equent, \begin{equation} \expecin{\nu}{f(X_n)} = \nu P^n f = \begin{cases} \nu(1)f(2) + \nu(2)f(1) & \text{si $n$ est impair\;,}\\ \nu(1)f(1) + \nu(2)f(2) & \text{si $n$ est pair\;,} \end{cases} \end{equation} D'un autre c\^ot\'e, la \CM\ \'etant r\'ecurrente positive, elle admet une unique probabilit\'e invariante $\pi$, satisfaisant $\pi P = \pi$. On trouve facilement que $\pi = (\frac12, \frac12)$, ce qui implique \begin{equation} \expecin{\pi}{f(X_n)} = \pi f = \frac12 (f(1) + f(2))\;. \end{equation} On s'aper\c coit que si $\nu \neq \pi$, alors $\expecin{\nu}{f}$ ne converge pas vers $\expecin{\pi}{f}$, sauf dans le cas parti\-culier $f(1) = f(2)$. Les valeurs propres de $P$ sont $1$ et $-1$. Des vecteurs propres \`a gauche associ\'es sont $\pi$ et $(1, -1)$. La valeur propre $-1$ est associ\'ee au fait que la \CM\ est $2$-p\'eriodique. \end{example} \begin{example} On peut facilement g\'en\'eraliser cet exemple \`a des p\'eriodes sup\'erieures. Par exemple, la matrice stochastique \begin{equation} P = \begin{pmatrix} 0 & 1 & 0 \\ 0 & 0 & 1 \\ 1 & 0 & 0 \end{pmatrix} \end{equation} satisfait $P^3 = \one$. Ses valeurs propres sont les trois racines cubiques de $1$, \`a savoir $1$ et $\e^{\pm\icx 2\pi/3}$. La \CM\ associ\'ee est irr\'eductible, r\'ecurrente positive, et de p\'eriode $3$. Elle admet l'unique probabilit\'e invariante $\pi = (\frac13,\frac13,\frac13)$. \`A nouveau, si $\nu \neq \pi$, alors $\expecin{\nu}{f}$ ne converge pas vers $\expecin{\pi}{f}$, sauf dans le cas particulier o\`u $f$ est constante. \end{example} \begin{example} Par contraste, consid\'erons la matrice stochastique \begin{equation} P = \begin{pmatrix} \frac13 & \frac23 \\[3pt] \frac23 & \frac13 \end{pmatrix}\;. \end{equation} La \CM\ associ\'ee est irr\'eductible, r\'ecurrente positive, et ap\'eriodique (car, par exemple, on a $\probin{1}{X_1 = 1} = \frac13 > 0$ et $\probin{1}{X_2 = 1} = \frac59 > 0$). Les valeurs propres de $P$ sont $\lambda_0 = 1$ et $\lambda_1 = -\frac13$. Une mani\`ere de calculer $P^n$ est d'utiliser la \defwd{d\'ecomposition de Dunford} (que nous rappellerons \`a la section~\ref{sec:spec_Dunford}) \begin{equation} P = \lambda_0 \Pi_0 + \lambda_1 \Pi_1\;, \qquad \Pi_0 = \begin{pmatrix} \frac12 & \frac12 \\[3pt] \frac12 & \frac12 \end{pmatrix}\;, \qquad \Pi_0 = \begin{pmatrix} \frac12 & -\frac12 \\[3pt] -\frac12 & \frac12 \end{pmatrix}\;. \end{equation} Les matrices $\Pi_0$ et $\Pi_1$ sont des \defwd{projecteurs}~: elles satisfont $\Pi_0^2 = \Pi_0$, et $\Pi_1^2 = \Pi_1$. Elles sont obtenues chacune en multipliant un vecteur propre \`a droite et un vecteur propre \`a gauche de $P$, proprement normalis\'es. De plus, on v\'erifie que $\Pi_0\Pi_1 = \Pi_1\Pi_0 = 0$. Ceci implique, par la formule du bin\^ome de Newton, que \begin{equation} P^n = \lambda_0^n \Pi_0 + \lambda_1^n \Pi_1 = \Pi_0 + \biggpar{-\frac13}^n \Pi_1\;. \end{equation} Par cons\'equent, nous avons \begin{equation} \nu P^n f = \frac12 \bigpar{f(1) + f(2)} + \frac12 \biggpar{-\frac13}^n \bigpar{\nu(1) - \nu(2)} \bigpar{f(1) - f(2)}\;. \end{equation} Comme par ailleurs, $\pi = (\frac12, \frac12)$, on a \begin{equation} \expecin{\pi}{f} = \pi f = \frac12 \bigpar{f(1) + f(2)}\;. \end{equation} Par cons\'equent, $\expecin{\nu}{f(X_n)}$ converge exponentiellement vite vers $\expecin{\pi}{f}$, avec une diff\'erence d'ordre $3^{-n}$. \end{example} Ces exemples sugg\`erent que \begin{itemize} \item si la \CM\ est p\'eriodique, alors $P$ admet plusieurs valeurs propres diff\'erentes de module $1$, $P^n$ ne converge pas lorsque $n\to\infty$, et $\expecin{\nu}{f(X_n)}$ ne converge pas vers $\expecin{\pi}{f}$ si $\nu\neq\pi$, sauf pour des $f$ tr\`es particuliers; \item si la \CM\ est ap\'eriodique, alors $P$ admet $1$ comme valeur propre simple, toutes les autres valeurs propres de $P$ sont strictement inf\'erieures \`a $1$ en module, et $\expecin{\nu}{f(X_n)}$ converge vers $\expecin{\pi}{f}$ si $\nu\neq\pi$. \end{itemize} Nous allons voir dans les sections suivantes que ceci est effectivement le cas. \section{Normes de vecteurs et de matrices} \label{sec:spec_norm} Soit $P$ la matrice de transition d'une \CM\ irr\'eductible et r\'ecurrente positive. Nous savons que $P$ admet la valeur propre $\lambda_0 = 1$. Un vecteur propre \`a gauche associ\'e est $\pi$, alors qu'un vecteur propre \`a droite est le vecteur \begin{equation} \vone = \begin{pmatrix} 1 \\ 1 \\ \vdots \\ 1 \end{pmatrix}\;. \end{equation} En effet, la propri\'et\'e~\eqref{eq:mstoch} d'une matrice stochastique \'equivaut \`a $P\vone = \vone$. Dans la suite, il sera naturel de travailler avec les normes suivantes. \begin{definition}[Normes de vecteurs] La \defwd{norme $\ell^1$} d'un vecteur ligne $\mu$ est d\'efinie par \begin{equation} \norm{\mu}_1 = \sum_{x\in\cX} \abs{\mu(x)}\;. \end{equation} La \defwd{norme $\ell^\infty$} (ou \defwd{norme sup}) d'un vecteur colonne est d\'efinie par \begin{equation} \norm{v}_\infty = \sup_{x\in\cX} \abs{v(x)}\;. \end{equation} \end{definition} Dans la suite, nous utiliserons souvent la majoration \'el\'ementaire \begin{equation} \label{eq:l1_linfty} \bigabs{\mu v} = \biggabs{\sum_{x\in\cX} \mu(x)v(x)} \leqs \sum_{x\in\cX} \abs{\mu(x)v(x)} \leqs \norm{\mu}_1 \norm{v}_\infty\;. \end{equation} \begin{lemma}[Normes et matrice stochastique] Pour une matrice stochastique $P$, et tout vecteur ligne $\mu$ et vecteur colonne $v$ de dimension ad\'equate, on a \begin{equation} \norm{Pv}_\infty \leqs \norm{v}_\infty \qquad\text{et}\qquad \norm{\mu P}_1 \leqs \norm{\mu}_1\;. \end{equation} De plus, il existe des vecteurs $\mu$ et $v$ non nuls tels que $\norm{Pv}_\infty = \norm{v}_\infty$ et $\norm{\mu P}_1 = \norm{\mu}_1$. \end{lemma} \begin{proof} On a \begin{equation} \norm{Pv}_\infty = \sup_{x\in\cX} \biggabs{\sum_{y\in\cX}p_{xy}v(y)} \leqs \sup_{x\in\cX} \biggbrak{\norm{v}_\infty \sum_{y\in\cX}p_{xy}} = \norm{v}_\infty\;, \end{equation} et \begin{equation} \norm{\mu P}_1 = \sum_{x\in\cX} \biggabs{\sum_{y\in\cX} \mu(y) p_{yx}} \leqs \sum_{y\in\cX} \abs{\mu(y)} \sum_{x\in\cX} p_{yx} = \norm{\mu}_1\;. \end{equation} Pour avoir \'egalit\'e, il suffit de prendre $v=\vone$ et $\mu=\pi$. \end{proof} \begin{remark}[Norme subordonn\'ee] \label{rem:norme_subordonnee} On peut associer \`a $P$ une \defwd{norme subordonn\'ee} $\norm{P}$, correspondant \`a la norme $\norm{\cdot}_1$ pour la multiplication \`a gauche et \`a la norme $\norm{\cdot}_\infty$ pour la multiplication \`a droite, satisfaisant \begin{equation} \norm{P} := \sup_{v\neq0} \frac{\norm{Pv}_\infty}{\norm{v}_\infty} = \sup_{\mu\neq0} \frac{\norm{\mu P}_1}{\norm{\mu}_1} = 1\;. \end{equation} \end{remark} \begin{corollary}[Module des valeurs propres] Toute valeur propre $\lambda$ d'une matrice stochastique $P$ satisfait $\abs{\lambda} \leqs 1$. \end{corollary} \begin{proof} Soit $\lambda$ une valeur propre de $P$, et $v$ un vecteur propre \`a droite associ\'e. Alors \begin{equation} \abs{\lambda}\norm{v}_\infty = \norm{\lambda v}_\infty = \norm{Pv}_\infty \leqs \norm{v}_\infty\;, \end{equation} d'o\`u le r\'esultat, car on peut diviser des deux c\^ot\'es par $\norm{v}_\infty > 0$. \end{proof} \section{Th\'eor\`eme de Perron--Frobenius et trou spectral} \label{sec:spec_perron-Frobenius} Le r\'esultat suivant est un cas particulier du th\'eor\`eme de Perron--Frobenius (ce th\'eor\`eme est plus g\'en\'eral, car il admet des versions s'appliquant \`a des matrices non stochastiques, \`a condition que tous leurs \'el\'ements soient r\'eels non n\'egatifs). \begin{theorem}[Perron--Frobenius] Soit $P$ une matrice stochastique irr\'eductible. Alors \begin{itemize} \item $P$ admet $\lambda_0 = 1$ comme valeur propre \defwd{simple} (de multiplicit\'e alg\'ebrique $1$); \item si $P$ est ap\'eriodique, alors toutes ses valeurs propres autres que $\lambda_0$ sont de module strictement inf\'erieur \`a $1$; \item si $P$ est p\'eriodique, de p\'eriode $p$, alors elle admet exactement $p$ valeurs propres de module $1$, qui sont des racines $p$i\`emes de $1$. \end{itemize} \end{theorem} Nous admettrons ce r\'esultat. Voici toutefois quelques indications sur sa d\'emonstration. \begin{itemize} \item Si la valeur propre $\lambda_0 = 1$ n\'etait pas de multiplicit\'e $1$, on pourrait trouver au moins deux vecteurs lignes $\pi$ et $\mu$, lin\'eairement ind\'ependants, tels que $\pi P = \pi$ et $\mu P = \mu$ (dans le cas diagonalisable, sinon l'argument est un peu plus compliqu\'e). Le vecteur $\mu$ n'est pas n\'ecessairement une mesure de probabilit\'e. Mais on peut trouver $\theta\in[0,1]$ tel que la combinaison convexe \begin{equation} \nu = \theta \mu + (1-\theta)\pi \end{equation} soit une mesure de probabilit\'e. Dans le cas diagonalisable, on trouve \begin{equation} \nu P^n = \nu \qquad \forall n\geqs 0\;. \end{equation} Mais ceci contredit l'unicit\'e de la probabilit\'e invariante. \item Si $P$ est ap\'eriodique, supposons par l'absurde que $P$ admet une valeur propre $\lambda$ de module $1$, diff\'erente de $1$. Si $\lambda$ est r\'eelle, pour un vecteur propre \`a gauche $\mu$, on peut proc\'eder comme au point pr\'ec\'edent, pour construire une mesure de probabilit\'e $\nu$ satisfaisant \begin{equation} \nu P^n = \theta \lambda^n \mu + (1-\theta)\pi\;. \end{equation} Mais alors $\nu P^n$ ne converge pas vers $\pi$ lorsque $n$ tend vers l'infini, ce qui contredit le Th\'eor\`eme~\ref{thm:convergence_aperiodique}. Si $\lambda$ est complexe, alors $\bar\lambda$ est \'egalement valeur propre, de vecteur propre $\bar\mu$, et on peut appliquer un argument analogue avec le vecteur r\'eel $\mu + \bar\mu$. \item Si $P$ est p\'eriodique de p\'eriode $P$, l'id\'ee de base est que $P^p$ admet $p$ sous-espaces invariants suppl\'ementaires. La restriction de $P$ \`a chacun de ces sous-espaces doit admettre la valeur propre $1$, ce qui correspond \`a une valeur propre racine $p$i\`eme de l'unit\'e de $P$. \end{itemize} Concentrons-nous maintenant sur le cas o\`u $P$ est ap\'eriodique. \begin{lemma}[Limite de $P^n$] Si $P$ est ap\'eriodique, alors \begin{equation} \label{eq:convergence_Pn} \lim_{n\to\infty} P^n = \Pi_0 = \vone \pi\;. \end{equation} La matrice $\Pi_0$ est un \defwd{projecteur}, c'est-\`a-dire qu'elle satisfait $\Pi_0^2 = \Pi_0$. \end{lemma} \begin{proof} Le th\'eor\`eme~\ref{thm:convergence_aperiodique} implique que $\nu P^n$ converge vers $\pi$ pour toute loi initiale $\nu$. La relation~\eqref{eq:convergence_Pn} s'obtient en appliquant ceci \`a $\delta_x$ pour tout $x\in\cX$. La relation $\Pi_0^2 = \Pi_0$ suit du fait que $\pi\vone = 1$, en vertu de~\eqref{eq:mproba}. \end{proof} \begin{remark} La matrice $\Pi_0$ est une matrice dont toutes les lignes sont \'egales. En particulier, si $\cX$ est fini, de cardinal $N$, alors \begin{equation} \Pi_0 = \begin{pmatrix} \pi(1) & \dots & \pi(N) \\ \vdots & & \vdots \\ \pi(1) & \dots & \pi(N) \end{pmatrix}\;. \end{equation} \end{remark} \begin{definition}[Rayon spectral et trou spectral] Soit $P$ une matrice stochastique irr\'eductible et ap\'eriodique, et soit $P_\perp = P - \Pi_0$. Alors le \defwd{rayon spectral} de $P_\perp$ est \begin{align} \rho &= \sup\Bigsetsuch{\abs{\lambda_j}}{\text{$\lambda_j$ est valeur propre de $P_\perp$}} \\ &= \sup\Bigsetsuch{\abs{\lambda_j}}{\text{$\lambda_j$ est valeur propre de $P$}, \lambda \neq 1}\;. \end{align} Le \defwd{trou spectral} de $P$ est par d\'efinition $1 - \rho$. \end{definition} Le th\'eor\`eme de Perron--Frobenius implique que $0 \leqs \rho < 1$, donc que $1-\rho > 0$. L'int\'er\^et de cette d\'efinition est li\'e \`a l'observation suivante. \begin{proposition}[Vitesse de convergence et trou spectral] On a \begin{equation} \expecin{\nu}{f(X_n)} - \expecin{\pi}{f} = (\nu - \pi)P_\perp^n f\;. \end{equation} \end{proposition} \begin{proof} On a une d\'ecomposition de l'espace des mesures en deux sous-espace suppl\'ementaires, invariants par $P$, l'un associ\'e \`a $\Pi_0$, et l'autre associ\'e \`a $P_\perp$. Le premier est simplement le sous-espace vectoriel de dimension $1$ engendr\'e par $\pi$, alors que le second est \begin{equation} \vone_\perp = \Bigsetsuch{\mu:\cX\to\R}{\mu \vone = 0} = \biggsetsuch{\mu:\cX\to\R}{\sum_{x\in\cX}\mu(x) = 0}\;. \end{equation} En effet, si $\mu\in\vone_\perp$, alors \begin{equation} \mu P \vone = \mu \vone = 0\;, \end{equation} ce qui implique que $\mu P\in\vone_\perp$, ou encore $\vone_\perp P \subset \vone_\perp$. De plus, on a \begin{align} \mu P_\perp &= \mu P - \mu\Pi_0 = \mu P \\ \pi P_\perp &= \pi P - \pi\Pi_0 = \pi - \pi\vone \pi = 0 \label{eq:invarianceP} \end{align} puisque $\mu\Pi_0 = \mu\vone\pi = 0$ et $\pi\vone = 1$. D\'ecomposons alors $\nu$ en $\nu = \pi + \mu$. On a $\mu\in\vone_\perp$, puisque $\mu\vone = \nu\vone - \pi\vone = 1 - 1 = 0$. Il suit de~\eqref{eq:invarianceP} que pour tout $n\geqs0$, \begin{equation} \nu P^n = (\pi + \mu)P^n = \pi + \mu P_\perp^n\;. \end{equation} Par cons\'equent, \begin{equation} \expecin{\nu}{f(X_n)} = \nu P^n f = \pi f + \mu P_\perp^n f\;, \end{equation} d'o\`u le r\'esultat. \end{proof} Par la majoration~\eqref{eq:l1_linfty}, on a \begin{equation} \label{eq:decroissance_EfXn} \bigabs{\expecin{\nu}{f(X_n)} - \expecin{\pi}{f}} \leqs \norm{\nu-\pi}_1 \norm{P_\perp^n f}\infty\;. \end{equation} On s'attend \`a avoir \begin{equation} \label{eq:borne_Pperp} \norm{P_\perp^n f}_\infty \leqs C\rho^n\norm{f}_\infty \end{equation} pour une constante $C$ \`a d\'eterminer. Si c'est bien le cas, alors on aura montr\'e que $\expecin{\nu}{f(X_n)}$ converge exponentiellement vite vers $\expecin{\pi}{f}$, avec une erreur qui d\'ecro\^it comme $\rho^n$. \section{Diagonalisation et d\'ecomposition de Dunford} \label{sec:spec_Dunford} Notre objectif est maintenant de v\'erifier~\eqref{eq:borne_Pperp}. Nous supposons pour l'instant que $\cX$ est fini, de cardinal $N$. Consid\'erons d'abord le cas o\`u $P_\perp$ est diagonalisable. Alors il existe une matrice non singuli\`ere $S$ telle que \begin{equation} S^{-1}P_\perp S = \Lambda_\perp = \begin{pmatrix} 0 & 0 & \dots & \dots & 0 \\ 0 & \lambda_1 & & & \vdots \\ \vdots & & \ddots & & \vdots \\ \vdots & & & \lambda_{N-2} & 0 \\ 0 & \dots & \dots & 0 & \lambda_{N-1} \end{pmatrix}\;. \end{equation} En effet, la premi\`ere valeur propre de $P_\perp$ est nulle, puisque $\pi P_\perp = 0$, cf.~\eqref{eq:invarianceP}. On a alors $P_\perp = S\Lambda_\perp S^{-1}$, et \begin{equation} P_\perp^n = S\Lambda_\perp^n S^{-1} \qquad \forall n\geqs 0\;. \end{equation} On remarque que $\norm{\Lambda_\perp^n g}_\infty \leqs \rho^n \norm{g}_\infty$ par d\'efinition du rayon spectral, et que par cons\'equent \begin{equation} \norm{P_\perp^n}_\infty \leqs \norm{S} \, \norm{\Lambda_\perp^n S^{-1}f}_\infty \leqs \rho^n \norm{S}\,\norm{S^{-1}}\, \norm{f}_\infty\;, \end{equation} o\`u les normes de $S$ et $S^{-1}$ sont des normes subordonn\'ees, comme d\'efinies dans la remarque~\ref{rem:norme_subordonnee}. On conclut donc que~\eqref{eq:borne_Pperp} est v\'erifi\'e, avec $C = \norm{S}\,\norm{S^{-1}}$. Si $P_\perp$ n'est pas diagonalisable, on a \begin{equation} S^{-1}P_\perp S = T_\perp\;, \end{equation} o\`u $T_\perp$ est une matrice triangulaire, diagonale par blocs, o\`u les blocs sont des \defwd{blocs de Jordan} de la forme $B(\lambda_j,b_j)$, avec \begin{equation} B(\lambda,b) = \begin{pmatrix} \lambda & 1 & 0 & \dots & 0 \\ 0 & \lambda & 1 & & \vdots \\ \vdots & & \ddots & \ddots & \\ \vdots & & & \lambda & 1 \\ 0 & \dots & \dots & 0 & \lambda \end{pmatrix} \in \C^{b\times b}\;. \end{equation} La dimension $b_j$ de $B(\lambda_j,b_j)$ d\'epend de la diff\'erence entre la \defwd{multiplicit\'e alg\'ebrique} de $\lambda_j$ (sa multiplicit\'e en tant que racine du polyn\^ome caract\'eristique), et sa \defwd{multiplicit\'e g\'eom\'etrique} (la dimension du noyau de $P - \lambda_j\one$). Dans ce cas, on a \begin{equation} P_\perp^n = ST_\perp^n S^{-1} \qquad \forall n\geqs 0\;. \end{equation} On pourrait alors essayer de majorer $\norm{T_\perp^n g}_\infty$ par une constante fois $\rho^n \norm{g}_\infty$. Il est toutefois plus commode de passer par la \defwd{d\'ecomposition de Dunford}, que nous rappelons ici. \begin{proposition}[D\'ecomposition de Dunford] Soit $P$ une matrice, admettant les valeurs propres diff\'erentes $\lambda_0, \dots, \lambda_{k}$. On note $m_i$ la multiplicit\'e alg\'ebrique de $\lambda_i$, et $g_i$ sa multiplicit\'e g\'eom\'etrique (on rappelle que $1\leqs g_i\leqs m_i$). Alors on a la d\'ecomposition \begin{equation} P = \sum_{i=0}^k \bigpar{\lambda_i \Pi_i + N_i}\;, \end{equation} o\`u \begin{itemize} \item les $\Pi_i$ sont des projecteurs, satisfaisant $\Pi_i\Pi_j = \delta_{ij}\Pi_i$; \item les $N_i$ sont nilpotentes~: elles satisfont $N_i^{m_i-g_i} = 0$; \item on a $N_iN_j = 0$ si $i\neq j$ et $P_i N_j = N_j P_i = \delta_{ij}N_i$. \end{itemize} \end{proposition} Il suit de la derni\`ere propri\'et\'e que \begin{equation} P_\perp^n = \sum_{i=1}^k \bigpar{\lambda_i \Pi_i + N_i}^n\;, \end{equation} et la formule du bin\^ome de Newton implique \begin{equation} \bigpar{\lambda_i \Pi_i + N_i}^n = \Pi_i \sum_{p=0}^{m_i - g_i - 1} \lambda_i^{n-p} \binom{n}{p} N_i^p\;. \end{equation} En effet, le fait que $N_i^{m_i-g_i} = 0$ implique que tous les termes avec $p \geqs m_i - g_i$ sont nuls. Le point important ici est que puisque $m_i - g_i$ est born\'e, $\norm{P_\perp^n f}_\infty$ d\'ecro\^it toujours comme $\rho^n$, m\^eme si ce terme est multipli\'e par une constante qui d\'epend de mani\`ere plus compliqu\'ee de $P_\perp$ (mais pas de $n$). Ainsi,~\eqref{eq:borne_Pperp} reste vrai, avec un $C$ d\'ependant des termes de la d\'ecomposition de Dunford. Nous avons suppos\'e jusqu'ici que $\cX$ \'etait fini. Si $\cX$ est infini, la matrice stochastique d\'efinit un op\'erateur lin\'eaire dit \defwd{compact}, ce qui signifie essentiellement qu'il applique des ensembles compacts sur des ensembles born\'es (dont la fermeture est compacte). Pour ces op\'erateurs, la notion de valeur propre est encore bien d\'efinie. En particulier, on sait que toute valeur propre non nulle de $P$ est de multiplicit\'e finie. Par cons\'equent, on a encore une d\'ecomposition de Dunford. Toutefois, il est moins clair que la constante $C$ dans~\eqref{eq:borne_Pperp} est toujours finie. \section{Cas r\'eversible} \label{sec:spec_reversible} Les \CMs\ r\'eversibles se pr\^etent mieux \`a une \'etude spectrale que les \CMs\ non r\'eversibles. Pour le voir, supposons la \CM\ irr\'eductible et r\'ecurrente positive, de distribution stationnaire $\pi$, et introduisons le produit scalaire \begin{equation} \label{rev6} \pscal fg_\pi = \sum_{x\in\cX} \pi(x) \cc{f(x)} g(x)\;, \end{equation} o\`u $f, g\in\C^{\cX}$ sont des vecteurs colonne. On d\'enote par $\ell^2(\C,\pi)$ l'ensemble des vecteurs $f$ tels que $\pscal{f}{f}_\pi < \infty$. C'est un espace de Hilbert. \begin{lemma}[Caract\`ere autoadjoint de $P$] L'op\'erateur lin\'eaire $P$ est autoadjoint dans l'espace de Hilbert $\cH = \ell^2(\C,\pi)$, c'est-\`a-dire \begin{equation} \pscal f{Pg}_\pi = \pscal {Pf}g_\pi \qquad \forall f, g \in\cH\;. \end{equation} \end{lemma} \begin{proof} On a \begin{equation} \pscal f{Pg}_\pi = \sum_{x\in\cX} \pi(x) \cc{f(x)} \sum_{y\in\cX} p_{xy}g(y) = \sum_{y\in\cX} \pi(y) \sum_{x\in\cX} p_{yx} \cc{f(x)} g(y) = \pscal {Pf}g_\pi\;, \end{equation} o\`u on a utilis\'e la r\'eversibilit\'e dans la deuxi\`eme \'egalit\'e. \end{proof} Rappelons un r\'esultat classique de la th\'eorie des espaces de Hilbert. \begin{proposition}[Th\'eor\`eme spectral] Soit $P$ un op\'erateur autoadjoint compact dans un espace de Hilbert $\cH$. Alors toutes les valeurs propres de $P$ sont r\'eelles, et les espaces propres associ\'es sont orthogonaux. De plus, $\cH$ admet une base orthonorm\'ee de vecteurs propres, dans laquelle $P$ est diagonale. \end{proposition} \begin{proof} Soient $v_1$ et $v_2$ deux vecteurs propres \`a droite de $P$, de valeurs propres respectives $\lambda_1$ et $\lambda_2$. Alors \begin{equation} \label{rev8} (\cc\lambda_1 - \lambda_2) \pscal{v_1}{v_2}_\pi = \pscal{\lambda_1v_1}{v_2}_\pi - \pscal{v_1}{\lambda_2v_2}_\pi = \pscal{Pv_1}{v_2}_\pi - \pscal{v_1}{Pv_2}_\pi = 0\;. \end{equation} D'une part, prenant $v_1=v_2$, on obtient que $\lambda_1$ est r\'eelle. D'autre part, si $\lambda_1\neq\lambda_2$, on obtient l'orthogonalit\'e de $v_1$ et $v_2$. Le fait que $P$ est diagonalisable se montre par r\'ecurrence. On sait que $P$ admet au moins une valeur propre complexe, avec vecteur propre associ\'e $v$. On montre alors que le compl\'ement orthogonal $v_\perp = \setsuch{w\in\cH}{\pscal{w}{v}_\pi = 0}$ est invariant par $P$. La restriction $P_\perp$ de $P$ \`a $v_\perp$ admet \`a nouveau une valeur propre, ce qui permet d'\'etablir l'h\'er\'edit\'e (si $P$ est de dimension finie, la r\'ecurrence s'arr\^ete lorsque le compl\'ement orthogonal est $\set{0}$). \end{proof} On a \'egalement un lien explicite entre vecteurs propres \`a gauche et \`a droite. \begin{lemma}[Vecteurs propres \`a droite et \`a gauche] Si $v$ est un vecteur propre \`a droite de l'op\'erateur autoadjoint $P$, alors $\mu$ d\'efini par \begin{equation} \mu(x) = \pi(x) v(x) \qquad \forall x\in\cX \end{equation} est un vecteur propre \`a gauche, pour la m\^eme valeur propre. \end{lemma} \begin{proof} Soit $v$ un vecteur colonne tel que $Pv = \lambda v$. Pour tout $x\in\cX$, on a \begin{equation} \bigpar{\mu P}_x = \sum_{y\in\cX} \mu(y)p_{yx} = \sum_{y\in\cX} v(y) \pi(y) p_{yx} = \pi(x) \sum_{y\in\cX} p_{xy} v(y) = \pi(x) \bigpar{Pv}_x = \lambda \pi(x) v(x) = \lambda \mu(x)\;. \end{equation} Par cons\'equent, $\mu P = \lambda\mu$. \end{proof} Une premi\`ere cons\'equence du caract\`ere autoadjoint de $P$ est une repr\'esentation variationnelle du trou spectral. \begin{proposition}[Principe min-max] Le trou spectral de $P$ satisfait \begin{equation} \label{rev9} \rho = \sup_{v \colon \pscal{v}{\vone}_\pi=0} \frac{\abs{\pscal{v}{Pv}_\pi}}{\pscal{v}{v}_\pi}\;. \end{equation} \end{proposition} \begin{proof} Soit $(v_k)_{k\geqs0}$ une base orthonorm\'ee de vecteurs propres \`a droite de $P$. Alors tout $v\in\cH$ s'\'ecrit \begin{equation} v = \sum_{k\geqs0} c_k v_k\;, \qquad\text{ o\`u } c_k = \pscal{v_k}{v}_\pi\;. \end{equation} On obtient alors \begin{align} \pscal{v}{v}_\pi &= \sum_{k,\ell\geqs0} \cc{c}_k c_\ell \pscal{v_k}{v_\ell}_\pi = \sum_{k\geqs0} \abs{c_k}^2\;, \\ \pscal{v}{Pv}_\pi &= \sum_{k,\ell\geqs0} \cc{c}_k c_\ell \pscal{v_k}{Pv_\ell}_\pi = \sum_{k\geqs0} \lambda_k\abs{c_k}^2\;. \end{align} La premi\`ere relation n'est autre que la relation de Parseval. Par cons\'equent, \begin{equation} \frac{\abs{\pscal{v}{Pv}_\pi}}{\pscal{v}{v}_\pi} \leqs \frac{\sum_{k\geqs0} \abs{\lambda_k}\abs{c_k}^2}{\sum_{k\geqs0} \abs{c_k}^2}\;. \end{equation} Si $\pscal{v}{\vone}_\pi = 0$, alors $c_0 = 0$, de sorte que cette quantit\'e est born\'ee par $\rho$. L'\'egalit\'e a lieu dans le cas $v = v_1$, si on a num\'erot\'e les valeurs propres de mani\`ere que $\abs{\lambda_1} = \rho$. \end{proof} Il est \'egalement possible d'obtenir une majoration analogue \`a~\eqref{eq:decroissance_EfXn}. M\^eme si elle ne peut pas sembler optimale, elle a le m\'erite d'\^etre explicite. \begin{proposition}[Vitesse de convergence dans le cas r\'eversible] Si la \CM\ est r\'eversible, on a la majoration \begin{equation} \bigabs{\expecin{\nu}{f(X_n)} - \expecin{\pi}{f}} \leqs \rho^n \norm{f}_\infty \norm{\nu-\pi}_1^{1/2} \sup_{x\in\cX} \biggabs{\frac{\nu(x)}{\pi(x)}-1}^{1/2}\;. \end{equation} \end{proposition} \begin{proof} Il s'agit de majorer $\abs{(\nu-\pi)P_\perp^n f}$. La d\'ecomposition de Dunford s'\'ecrit \begin{equation} P_\perp^n = \sum_{k\geqs1} \lambda_k \Pi_k\;, \end{equation} o\`u le projecteur $\Pi_k$ peut s'\'ecrire $\Pi_k = v_k \mu_k$. En effet, $\Pi_k$ projette bien sur $v_k$ par action \`a droite, et sur $\mu_k$ par action \`a gauche. De plus, $\Pi_k^2 = v_k (\mu_k v_k) \mu_k = \Pi_k$, puisque \begin{equation} \mu_k v_k = \sum_{x\in\cX} \mu_k(x) v_k(x) = \sum_{x\in\cX} \pi(x)v_k(x) v_k(x) = \pscal{v_k}{v_k}_\pi = 1\;. \end{equation} Nous avons donc \begin{equation} \label{eq:proof_nupif} (\nu-\pi)P_\perp^n f = \sum_{k\geqs1} \lambda_k (\nu-\pi)v_k \mu_k f = \sum_{k\geqs1} \lambda_k a_k b_k\;, \end{equation} o\`u nous avons pos\'e \begin{equation} a_k = \mu_k f = \sum_{x\in\cX} \mu_k(x)f(x) = \sum_{x\in\cX} \pi(x)v_k(x)f(x) = \pscal{v_k}{f}_\pi\;, \end{equation} et \begin{equation} b_k = (\nu-\pi)v_k = \sum_{x\in\cX} (\nu(x)-\pi(x))v_k(x) = \pscal{g}{v_k}_\pi\;, \end{equation} o\`u $g$ est le vecteur colonne de composantes $g(x) = (\nu(x)-\pi(x))/\pi(x)$. Il suit alors de~\eqref{eq:proof_nupif} et de l'in\'egalit\'e de Cauchy--Schwarz que \begin{equation} \bigabs{(\nu-\pi)P_\perp^n f} \leqs \rho \sum_{k\geqs1} \abs{a_k b_k} \leqs \rho \biggpar{\sum_{k\geqs1} a_k^2}^{1/2} \biggpar{\sum_{k\geqs1} b_k^2}^{1/2}\;. \end{equation} Or, par la relation de Parseval, \begin{equation} \sum_{k\geqs1} a_k^2 \leqs \pscal{f}{f}_\pi = \sum_{x\in\cX} \pi(x) f(x)^2 \leqs\norm{f}_\infty^2\;. \end{equation} D'autre part, \begin{equation} \sum_{k\geqs1} b_k^2 \leqs \pscal{g}{g}_\pi = \sum_{x\in\cX} \pi(x)g(x)^2 \leqs \sup_{x\in\cX} \abs{g(x)} \, \norm{\pi g}_1\;. \end{equation} Comme $\norm{\pi g}_1 = \norm{\nu - \pi}_1$, le r\'esultat est prouv\'e. \end{proof} Le facteur $\norm{\nu - \pi}_1$ ne pose pas de probl\`eme, car on peut toujours le majorer par $\norm{\nu}_1 + \norm{\pi}_1 = 2$. Pour que le supremum sur $x$ soit petit, il faut que $\nu(x)$ ne soit pas trop diff\'erent de $\pi(x)$, du moins si $\pi(x)$ est petit. Une possibilit\'e est de choisir pour $\nu$ la probabilit\'e uniforme sur un ensemble probable sous $\pi$, et sur lequel $\pi$ ne varie pas trop. \begin{proposition}[Cas d'un $\nu$ uniforme] Soit $\cX_0 \subset \cX$ un ensemble fini, tel que \begin{equation} \pi(X_0^c) := \sum_{x\notin X_0} \pi(x) = \delta \qquad \text{et} \qquad \max_{x\in\cX_0} \pi(x) \leqs (1+c) \min_{x\in\cX_0} \pi(x)\;. \end{equation} Soit $\nu$ la loi uniforme sur $\cX_0$. Alors \begin{equation} \norm{\nu-\pi}_1 \leqs 2\delta + c \qquad \text{et} \qquad \sup_{x\in\cX} \biggabs{\frac{\nu(x)}{\pi(x)}-1} \leqs \max\biggset{1, \frac{c(1 + \delta)}{(1+c)(1-\delta)}}\;. \end{equation} \end{proposition} \begin{proof} Soit \begin{equation} m = \min_{x\in\cX_0} \pi(x)\;, \qquad M = \max_{x\in\cX_0} \pi(x)\;. \end{equation} Alors on a $M \leqs (1+c) m$ et \begin{equation} m \abs{\cX_0} \leqs \pi(\cX_0) = 1-\delta \leqs M \abs{\cX_0}\;. \end{equation} En combinant ces in\'egalit\'es, on obtient \begin{equation} M \leqs \frac{(1+c)(1-\delta)}{\abs{\cX_0}} \qquad\text{et}\qquad m \geqs \frac{1-\delta}{(1+c)\abs{\cX_0}}\;. \end{equation} On a \begin{equation} \norm{\nu-\pi}_1 = \sum_{x\in\cX_0} \biggabs{\frac{1}{\abs{\cX_0}} - \pi(x)} + \sum_{x\in\cX_0^c} \pi(x)\;. \end{equation} La seconde somme vaut $\delta$, alors qu'en utilisant le fait que $m\leqs\pi(x)\leqs M$ dans la premi\`ere somme, on obtient, en simplifiant l'expression obtenue, que celle-ci est toujours inf\'erieure \`a $\delta+c$. Ceci prouve la majoration de $\norm{\nu-\pi}_1$. Pour la seconde majoration, on utilise le fait que \begin{equation} \sup_{x\in\cX} \biggabs{\frac{\nu(x)}{\pi(x)}-1} = \max\biggset{\sup_{x\in\cX_0} \biggabs{\frac{\nu(x)}{\pi(x)}-1}, 1}\;, \end{equation} et on borne la premi\`ere somme \`a nouveau \`a l'aide de l'encadrement $m\leqs\pi(x)\leqs M$. \end{proof} Le message essentiel \`a retenir de ce chapitre est que la th\'eorie spectrale permet de montrer que $\expecin{\pi_0}{f(X_n)}$ converge exponentiellement vite vers $\expecin{\pi}{f}$, avec un exposant d\'etermin\'e par le trou spectral, et une constante proportionnelle \`a $\norm{f}_\infty$. Toutefois, si $\cX$ est grand ou infini, il n'est pas facile de d\'eterminer explicitement le trou spectral, ainsi que la constante. C'est pour cette raison que nous allons introduire une autre approche, bas\'ee sur des fonctions de Lyapounov, qui est plus flexible et a l'avantage de fournir des valeurs explicites de l'exposant et de la constante. \section{Exercices} \label{sec:spectral_exo} \begin{exercise} On consid\`ere la marche al\'eatoire sym\'etrique sur le cercle discret \`a $N$ sites~: \[ p_{xy} = \begin{cases} \frac12 & \text{si $y = x+1$\;,} \\ \frac12 & \text{si $y = x-1$\;,} \\ 0 & \text{sinon\;,} \end{cases} \] avec l'identification modulo $N$\,: $N+1 = 1$, $0 = N$. \begin{enumerate} \item Quelle est la matrice de transition de cette \CM\ ? \item Par un argument de sym\'etrie, trouver la probabilit\'e invariante de la cha\^ine. \item Soit $\omega = \e^{2\pi\icx/N}$. Montrer que pour tout $k\in\set{0,\dots,N-1}$, le vecteur $v_k$ de composantes \[ v_{k,x} = \omega^{k(x-1)}\;, \qquad x\in\set{1,\dots,N} \] est un vecteur propre de $P$. En d\'eduire les valeurs propres de $P$. \item D\'eterminer le rayon spectral $\rho$ de $P$ (sa valeur propre diff\'erente de $1$ de plus grand module). Distinguer les cas $N$ pair et $N$ impair. \item Par un d\'eveloppement limit\'e, d\'eterminer le trou spectral $1-\rho$ \`a l'ordre dominant en $N$. \end{enumerate} \end{exercise} \begin{exercise} Soit $p\in]0,1[$ et $q = 1 - p$. On consid\`ere la marche al\'eatoire asym\'etrique sur le cercle discret \`a $N$ sites~: \[ p_{xy} = \begin{cases} p & \text{si $y = x+1$\;,} \\ q & \text{si $y = x-1$\;,} \\ 0 & \text{sinon\;.} \end{cases} \] Par la m\^eme m\'ethode qu'\`a l'exercice pr\'ec\'edent, d\'eterminer, en fonction de $p$, le rayon spectral $\rho$ de $P$, ainsi que le trou spectral $1-\rho$ \`a l'ordre dominant en $N$. \end{exercise} \chapter{Fonctions de Lyapounov et vitesse de convergence} \label{chap:cm_Lyapounov} Dans ce chapitre, nous consid\'erons \`a nouveau des \CMs\ $(X_n)_{n\geqs0}$ irr\'eductibles, r\'ecurrentes positives et ap\'eriodiques sur un ensemble d\'enombrable $\cX$. Soit $f:\cX\to\R$ une fonction born\'ee, et soit $\pi$ la probabilit\'e invariante de la \CM. Le but est \`a nouveau de majorer l'erreur \begin{equation} \bigabs{\expecin{\nu}{f(X_n)} - \expecin{\pi}{f}}\;. \end{equation} Au lieu d'utiliser des informations sur les valeurs propres de la matrice de transition $P$, nous allons ici baser l'analyse sur des propri\'et\'es de fonctions dites de Lyapounov. Si les estimations fournies par ces fonctions ne sont pas toujours aussi pr\'ecises que celles provenant de l'analyse spectrale, la m\'ethode est plus robuste, et donne souvent des bornes explicites. \section{Notations -- formalisme des g\'en\'erateurs} \label{sec:generateurs} Commen\c cons par pr\'eciser quelques d\'efinitions li\'ees aux mesures et aux fonctions tests. \begin{definition}[Mesures sign\'ees] \label{def:mesure} Une \defwd{mesure sign\'ee finie} sur $\cX$ est une application $\mu:\cX\to\R$ telle que \begin{equation} \norm{\mu}_1 := \sum_{x\in\cX} \abs{\mu(x)} < \infty\;. \end{equation} On notera $\cE_1$ l'espace de Banach des mesures sign\'ees finies. \noindent Si $\mu:\cX\to[0,1]$, et $\norm{\mu}_1 = 1$, alors $\mu$ est une \defwd{mesure de probabilit\'e}. \end{definition} Notons que la somme de deux mesures de probabilit\'e n'est pas une mesure de probabilit\'e. Le sous-ensemble des mesures de probabilit\'e n'est donc pas un sous-espace de $\cE_1$. Cependant, la combinaison convexe de deux mesures de probabilit\'e est une mesure de probabilit\'e. \begin{definition}[Fonctions test] \label{def:fct_test} Une \defwd{fonction test} (ou \defwd{observable}) sur $\cX$ est une application $f:\cX\to\R$ telle que \begin{equation} \norm{f}_\infty := \sup_{x\in\cX} \abs{f(x)} < \infty\;. \end{equation} On notera $\cE_\infty$ l'espace de Banach des fonctions test. \end{definition} Les notations suivantes, en parties d\'ej\`a introduites, vont s'av\'erer utiles. \begin{itemize} \item Pour une mesure sign\'ee finie $\mu$ et une fonction test $f$, nous \'ecrirons \begin{equation} \mu(f) = \sum_{x\in\cX} \mu(x) f(x)\;. \end{equation} Cette quantit\'e est bien d\'efinie, car \begin{equation} \abs{\mu(f)} \leqs \sum_{x\in\cX} \abs{\mu(x)} \abs{f(x)} \leqs \sup_{x\in\cX} \abs{f(x)} \sum_{x\in\cX} \abs{\mu(x)} = \norm{f}_\infty \norm{\mu}_1 < \infty\;. \end{equation} \item Si $\mu$ est une mesure de probabilit\'e, nous \'ecrirons aussi $\mu(f) = \expecin{\mu}{f}$. \item Si $\delta_x$ d\'enote la mesure de Dirac en $x$ (c'est-\`a-dire que $\delta_x(x) = 1$ et $\delta_x(y) = 0$ si $y\neq x$), on abr\`ege $\expecin{\delta_x}{f}$ par $\expecin{x}{f}$. \item Pour $A\subset\cX$, on \'ecrit \begin{equation} \mu(A) = \mu(\indicator{A}) = \sum_{x\in A} \mu(x)\;. \end{equation} \item Si $\mu$ est une mesure de probabilit\'e, alors $\mu(A)$ est aussi la probabilit\'e de $A$. \item Pour une mesure de probabilit\'e $\mu$ et une fonction test $f$, on \'ecrira \begin{equation} \expecin{\mu}{f(X_n)} = \mu P^n f = \sum_{x\in\cX} \sum_{y\in\cX} \mu(x) (P^n)_{xy} f(y)\;, \end{equation} o\`u $(P^n)_{xy}$ est l'\'el\'ement de matrice $(x,y)$ de $P^n$. \end{itemize} \begin{definition}[Distance en variation totale] La \defwd{distance en variation totale} entre deux mesures $\mu,\nu\in\cE_1$ est \begin{equation} \normTV{\mu-\nu} = 2 \sup\bigsetsuch{\abs{\mu(A) - \nu(A)}}{A \subset X}\;. \end{equation} \end{definition} Intuitivement, deux mesures sont d'autant plus proches en variation totale qu'elles donnent des probabilit\'es proches aux \'ev\'enements. Pour des mesures de probabilit\'e, le r\'esultat suivant montre que la distance en variation totale est en fait \'equivalente \`a la norme $\ell^1$. \begin{lemma}[\'Equivalence des distances] \label{lem:TV} Si $\mu$ et $\nu$ sont deux mesures de probabilit\'e, alors \begin{equation} \normTV{\mu - \nu} = \sum_{x\in\cX} \abs{\mu(x) - \nu(x)} = \norm{\mu - \nu}_1\;. \end{equation} \end{lemma} \begin{proof} Soit $B = \setsuch{x\in\cX}{\mu(x) > \nu(x)}$. Alors on a \begin{equation} \label{eq:equiv_proof1} 0 \leqs \mu(B) - \nu(B) = 1 - \mu(B^c) + (1 - \nu(B^c)) = \nu(B^c) - \mu(B^c)\;, \end{equation} ce qui implique \begin{align} \sum_{x\in\cX} \abs{\mu(x) - \nu(x)} &= \sum_{x\in B} (\mu(x) - \nu(x)) + \sum_{x\in B^c} (\nu(x) - \mu(x)) \\ &= \mu(B) - \nu(B) + \nu(B^c) - \mu(B^c) \\ &= 2 \bigbrak{\mu(B) - \nu(B)} \label{eq:equiv_proof2} \end{align} par~\eqref{eq:equiv_proof1}. De plus, pour tout $A \subset \cX$, \begin{equation} \mu(A) - \nu(A) \leqs \sum_{x\in A\cap B} (\mu(x) - \nu(x)) \leqs \sum_{x\in B} (\mu(x) - \nu(x)) = \mu(B) - \nu(B)\;, \end{equation} o\`u nous avons utilis\'e \`a deux reprises le fait que $\mu(x) \leqs \nu(x)$ sur $A\cap B^c$. De m\^eme, \begin{equation} \nu(A) - \mu(A) \leqs \sum_{x\in A\cap B^c} (\nu(x) - \mu(x)) \leqs \nu(B^c) - \mu(B^c) = \mu(B) - \nu(B)\;. \end{equation} Il suit de~\eqref{eq:equiv_proof2} que \begin{equation} \abs{\mu(A) - \nu(A)} \leqs \mu(B) - \nu(B) = \frac12\norm{\mu-\nu}_1\;. \end{equation} De plus, si $A=B$, on a \'egalit\'e. \end{proof} \begin{definition}[G\'en\'erateur] Soit $P$ la matrice de transition d'une \CM\ sur un ensemble d\'enombrable $\cX$. Le \defwd{g\'en\'erateur} de la \CM\ est l'application $\cL:\cE_\infty\to\cE_\infty$ donn\'ee par \begin{equation} \label{eq:def_gen} (\cL f)(x) = \sum_{y\in \cX} p_{xy} \bigbrak{f(y) - f(x)}\;. \end{equation} \end{definition} Remarquons que comme $ \sum_{y\in \cX} p_{xy} = 1$, on a l'expression \'equivalente \begin{equation} (\cL f)(x) = \biggbrak{\sum_{y\in \cX} p_{xy}f(y)} - f(x) = \expecin{x}{f(X_1)} - f(x)\;. \end{equation} On peut donc \'ecrire $\cL = P - \one$, o\`u $\one$ d\'enote la matrice identit\'e. \section{Fonctions de Lyapounov} \label{sec:Lyap} Dans la suite, nous supposons que $P$ est la matrice de transition d'une \CM\ \defwd{irr\'eductible} sur $\cX$. De plus, nous supposons que $\cX$ est \'equip\'e d'une norme $\norm{\cdot}$. Par exemple, si $\cX \subset \Z$, on peut prendre $\norm{x} = \abs{x}$. Si $\cX \subset \Z^d$, on peut prendre la norme Euclidienne (ou toute autre norme \'equivalente). \begin{definition}[Fonction de Lyapounov] Une \defwd{fonction de Lyapounov} est une fonction $V: \cX\to \R_+ = [0,\infty[$ satisfaisant \begin{equation} \label{eq:gen} V(x) \to +\infty \qquad \text{pour $\norm{x}\to\infty$\;.} \end{equation} \end{definition} \begin{proposition}[Formule de Dynkin] \label{prop:Dynkin} Pour toute fonction de Lyapounov $V$, on a \begin{equation} \label{eq:Dynkin} \bigexpecin{x}{V(X_n)} = V(x) + \biggexpecin{x}{\sum_{m=0}^{n-1} (\cL V)(X_m)}\;. \end{equation} De plus, si $\tau$ est un temps d'arr\^et tel que $\expecin{x}{\tau} < \infty$, alors \begin{equation} \bigexpecin{x}{V(X_\tau)} = V(x) + \biggexpecin{x}{\sum_{m=0}^{\tau-1} (\cL V)(X_m)}\;. \end{equation} \end{proposition} \begin{proof} Montrons~\eqref{eq:Dynkin}. On proc\`ede par r\'ecurrence sur $n$. L'initialisation se fait pour $n=1$, o\`u la d\'efinition~\eqref{eq:def_gen} du g\'en\'erateur implique \begin{equation} \bigexpecin{x}{V(X_1)} = V(x) + (\cL V)(x)\;. \end{equation} Pour v\'erifier l'h\'er\'edit\'e, une premi\`ere fa\c con de proc\'eder est d'\'ecrire \begin{align} \bigexpecin{x}{V(X_{n+1})} &= \sum_{y\in\cX} V(y) \probin{x}{X_{n+1} = y} \\ &= \sum_{y\in\cX} V(y) \sum_{z\in\cX} \underbrace{\pcondin{x}{X_{n+1}=y}{X_n=z}}_{=p_{zy}} \bigprobin{x}{X_n = z} \\ &= \sum_{z\in\cX} \bigprobin{x}{X_n = z} \underbrace{\sum_{y\in\cX} V(y) p_{zy}}_{=(\cL V)(z) + V(z)} \\ &= \biggexpecin{x}{\sum_{z\in\cX}\indicator{X_n=z}(\cL V)(z)} + \sum_{z\in\cX} \bigprobin{x}{X_n = z}V(z) \\ &= \bigexpecin{x}{(\cL V)(X_n)} + \bigexpecin{x}{V(X_n)}\;. \end{align} Une autre mani\`ere de proc\'eder est d'utiliser le formalisme des esp\'erances conditionnelles, en \'ecrivant \begin{equation} \bigexpecin{x}{V(X_{n+1})} = \bigexpecin{x}{V(X_n)} + \bigexpecin{x}{V(X_{n+1}) - V(X_n)}\;. \end{equation} Or, si $\cF_n$ d\'enote la tribu engendr\'ee par $(X_0, X_1, \dots, X_n)$, on a \begin{align} \bigexpecin{x}{V(X_{n+1}) - V(X_n)} &= \bigexpecin{x}{\bigecondin{x}{V(X_{n+1}) - V(X_n)}{\cF_n}} \\ &= \bigexpecin{x}{\bigexpecin{X_n}{V(X_{n+1}) - V(X_n)}} = \bigexpecin{x}{(\cL V)(X_n)}\;. \end{align} Avec l'hypoth\`ese de r\'ecurrence, ceci conclut la d\'emonstration. \end{proof} \begin{theorem}[Croissance sous-exponentielle] \label{thm:sous_exp} Supposons qu'il existe une fonction de Lyapounov $V$ et $c > 0$, $d\geqs0$ tels que \begin{equation} (\cL V)(x) \leqs c V(x) + d \qquad \forall x\in\cX\;. \end{equation} Alors on a \begin{equation} \bigexpecin{x}{V(X_n)} \leqs (1+c)^n V(x) + \frac{(1+c)^n-1}{c}d \end{equation} pour tout $n\in\N$ et tout $x\in\cX$. \end{theorem} \begin{proof} Commen\c cons par consid\'erer le cas $d = 0$. Notons $f_n(x) = \expecin{x}{V(X_n)}$. Alors la formule de Dynkin implique \begin{align} f_n(x) &= V(x) + \biggexpecin{x}{\sum_{m=0}^{n-1} (\cL V)(X_m)} \\ &\leqs V(x) + c \sum_{m=0}^{n-1} f_m(x)\;. \end{align} En utilisant $f_0(x) = V(x)$ comme initialisation, on obtient facilement par r\'ecurrence sur $n$ que $f_n(x) \leqs (1+c)^n V(x)$. Dans le cas $d > 0$, la relation de r\'ecurrence devient \begin{equation} f_n(x) \leqs V(x) + c \sum_{m=0}^{n-1} f_m(x) + nd\;. \end{equation} En posant \begin{equation} f_n(x) \leqs (1+c)^n V(x) + k_n d\;, \end{equation} on obtient pour $k_n$ la relation de r\'ecurrence \begin{equation} k_n = c\sum_{m=0}^{n-1} k_m + n\;, \qquad k_0 = 0\;. \end{equation} On v\'erifie par r\'ecurrence que ceci \'equivaut \`a \begin{equation} k_n = \frac1c \Bigpar{(1+c)^n - 1}\;, \end{equation} d'o\`u le r\'esultat. \end{proof} \begin{theorem}[Non-explosion] \label{thm:non_explosion} Supposons qu'il existe $d \geqs 0$ et un ensemble born\'e $K\subset\cX$ tel que pour tout $x\in\cX$, on ait \begin{equation} (\cL V)(x) \leqs d \indicator{K}(x) = \begin{cases} d & \text{si $x\in K$\;,} \\ 0 & \text{sinon\;.} \end{cases} \end{equation} Alors \begin{equation} \biggprobin{x}{\lim_{n\to\infty} \norm{X_n} = \infty} = 0 \qquad \forall x\in\cX\;. \end{equation} \end{theorem} \begin{proof}[D\'emonstration (id\'ee)] Soit $\Omega_1$ l'\'ev\'enement \begin{equation} \Omega_1 = \biggsetsuch{\omega}{\lim_{n\to\infty} \norm{X_n(\omega)} = \infty}\;. \end{equation} Consid\'erons le cas $d=0$. Alors la formule de Dynkin implique \begin{equation} \bigexpecin{x}{V(X_n)} \leqs V(x)\;. \end{equation} Or, on a aussi \begin{equation} \bigexpecin{x}{V(X_n)} = \underbrace{\bigexpecin{x}{V(X_n) \indicator{\Omega_1^c}}}_{\geqs0} + \bigexpecin{x}{V(X_n)\indicator{\Omega_1}} \end{equation} Par cons\'equent, $\bigexpecin{x}{V(X_n)\indicator{\Omega_1}} \leqs \bigexpecin{x}{V(X_n)}\leqs V(x)$. Comme $V(X_n)$ tend vers l'infini sur $\Omega_1$, ceci n'est possible que si $\bigexpecin{x}{\indicator{\Omega_1}} = \probin{x}{\Omega_1} = 0$. \end{proof} \begin{theorem}[R\'ecurrence positive] \label{thm:rec_pos} Soit $f: \cX\to[1,\infty[$ et $V$ une fonction de Lyapounov telle que \begin{equation} (\cL V)(x) \leqs -cf(x) + d\indicator{K}(x) \qquad \forall x\in \cX\;, \end{equation} pour un ensemble born\'e $K\subset \cX$ et des constantes $c>0$ et $d\geqs0$. Supposons de plus qu'il existe $\delta > 0$ tel que $K$ satisfait \begin{equation} \label{eq:proba_lb} p_{xy} \geqs \delta \qquad \forall x, y\in K\;. \end{equation} Alors la \CM\ est r\'ecurrente positive, et admet donc une mesure de probabilit\'e invariante $\pi$. De plus, \begin{equation} \pi(f) < \infty\;. \end{equation} \end{theorem} \begin{proof} Commen\c cons par consid\'erer le cas o\`u $K = \set{x_0}$ contient un seul point. Nous voulons montrer que $x_0$ est r\'ecurrent positif, car dans ce cas, la \CM\ \'etant irr\'eductible, elle sera r\'ecurrente positive. Soit donc \begin{equation} \tau_K = \tau_{x_0} = \inf\setsuch{n\geqs1}{X_n\in K}\;. \end{equation} Pour tout $T\in\N$, posons $\tau_K\wedge T = \min\set{\tau_K,T}$. Par la formule de Dynkin, qui s'applique puisque $\tau_K\wedge T$ est un temps d'arr\^et fini presque s\^urement, donc d'esp\'erance finie, nous avons \begin{align} 0 \leqs \bigexpecin{x_0}{V(X_{\tau_K\wedge T)}} &= V(x_0) + \biggexpecin{x_0}{\sum_{m=0}^{\tau_K\wedge T-1} (\cL V)(X_m)} \\ \label{eq:proof_recpos1} &\leqs V(x_0) - c \biggexpecin{x_0}{\sum_{m=0}^{\tau_K\wedge T-1} f(X_m)} + d \\ &\leqs V(x_0) - c\bigexpecin{x_0}{\tau_K\wedge T} + d\;. \end{align} Il suit que \begin{equation} \bigexpecin{x_0}{\tau_{x_0}\wedge T} = \bigexpecin{x_0}{\tau_K\wedge T} \leqs \frac{V(x_0)+d}{c}\;, \end{equation} ce qui est fini. Comme la quantit\'e de droite est ind\'ependante de $T$, on peut prendre la limite lorsque $T$ tend vers l'infini, pour laquelles $\tau_K\wedge T$ tend vers $\tau_K$. Ceci implique que la \CM\ est r\'ecurrente positive. Dans le cas o\`u $K$ contient au moins deux points, nous savons d\'ej\`a par le raisonnement ci-dessus que $\bigexpecin{x_0}{\tau_K}$ est fini pour tout $x_0\in K$. Il nous faut montrer que $\bigexpecin{x_0}{\tau_{x_0}}$ est \'egalement fini pour tout $x_0\in K$. Soit $\tau_{K,n}$ le temps du $n$i\`eme passage de la cha\^ine en $K$, et soit \begin{equation} Y_n = X_{\tau_{K,n}}\;. \end{equation} (La \CM\ $(Y_n)_{n\geqs0}$ est appel\'ee la \defwd{trace} de $(X_n)_{n\geqs0}$ sur $K$.) On peut d\'eduire de l'hypoth\`ese~\eqref{eq:proba_lb} que $Y_n$ atteint $x_0$ en un temps d'esp\'erance finie. Ceci suit du fait qu'une cha\^ine r\'ecurrente sur un ensemble fini est r\'ecurrente positive. Comme le temps de retour vers $K$ est born\'e, il suit qu'on a bien $\bigexpecin{x_0}{\tau_{x_0}} < \infty$. La \CM\ \'etant r\'ecurrente positive, elle admet une unique probabilit\'e invariante $\pi$. Il reste \`a montrer que $\pi(f)$ est fini. En fait (voir~\eqref{eq:gamma(y)}), la mesure $\mu$ donn\'ee par \begin{equation} \mu(y) = \biggexpecin{x_0}{\sum_{n=1}^{\tau_{x_0}} \indicator{X_n = y}} \end{equation} est invariante pour tout $x_0\in\cX$. La probabilit\'e invariante $\pi$ est obtenue en normalisant $\mu$. Comme \begin{equation} \sum_{y\in\cX} \mu(y) = \biggexpecin{x_0}{\sum_{n=1}^{\tau_{x_0}} \underbrace{\sum_{y\in\cX}\indicator{X_n = y}}_{=1}} = \bigexpecin{x_0}{\tau_{x_0}}\;, \end{equation} on conclut que \begin{equation} \pi(y) = \frac{1}{\bigexpecin{x_0}{\tau_{x_0}}} \biggexpecin{x_0}{\sum_{n=1}^{\tau_{x_0}} \indicator{X_n = y}}\;. \end{equation} Il suit que \begin{equation} \pi(f) = \frac{1}{\bigexpecin{x_0}{\tau_{x_0}}} \biggexpecin{x_0}{\sum_{n=1}^{\tau_{x_0}} \underbrace{\sum_{y\in\cX} f(y)\indicator{X_n = y}}_{=f(X_n)}} = \frac{1}{\bigexpecin{x_0}{\tau_{x_0}}} \biggexpecin{x_0}{\sum_{n=1}^{\tau_{x_0}} f(X_n)}\;. \end{equation} Si $K=\set{x_0}$~\eqref{eq:proof_recpos1} implique que pour tout $T\in\N$, \begin{equation} \biggexpecin{x_0}{\sum_{n=1}^{\tau_{x_0}\wedge T} f(X_n)} = \biggexpecin{x_0}{\sum_{n=0}^{\tau_{x_0}\wedge T-1} f(X_n)} \leqs \frac{V(x_0)+d}{c}\;, \end{equation} ce qui permet de majorer $\pi(f)$, en prenant le limite $T\to\infty$. Si $K$ contient deux points ou plus, on a une majoration analogue, mais avec $d$ multipli\'e par l'esp\'erance du nombre de points de $K$ visit\'es avant d'atteindre $x_0$, qui est finie. \end{proof} \begin{remark} On peut affaiblir l'hypoth\`ese~\eqref{eq:proba_lb} sur $K$ de plusieurs mani\`eres. \begin{itemize} \item Il suffit de supposer qu'il existe un $k\geqs1$ tel que $(P^k)_{xy} \geqs \delta$ pour tout $x,y\in K$ et un $\delta>0$. \item Une condition suffisante encore plus faible est qu'il existe des r\'eels $a_1, a_2, \dots \geqs 0$, de somme \'egale \`a $1$, tels que \begin{equation} \sum_{k=1}^\infty a_k (P^k)_{xy} \geqs \delta \end{equation} pour tout $x,y\in K$ et un $\delta>0$. Des ensembles $K$ satisfaisant ce crit\`ere sont appel\'es \defwd{petits} (\myquote{petite set}\ en anglais). \end{itemize} \end{remark} \section{Normes \`a poids} \label{sec:normes_poids} Pour obtenir des r\'esultats de convergence de la loi de $X_n$ vers $\pi$, il est plus utile de travailler avec des normes \`a poids. \begin{definition}[Norme \`a poids sur les fonctions test] \label{def:norme_poids_fct_test} Soit $W: \cX\to [1,\infty[$. La \defwd{norme \`a poids $W$} d'une fonction test est d\'efinie comme \begin{equation} \norm{f}_W = \sup_{x\in\cX} \frac{\abs{f(x)}}{W(x)}\;. \end{equation} On notera $\cE_\infty^W$ l'espace de Banach des fonctions test $f$ telles que $\norm{f}_W < \infty$. \end{definition} Notons les propri\'et\'es suivantes\,: \begin{itemize} \item Pour tout $x\in\cX$, on a $\abs{f(x)} \leqs \norm{f}_W W(x)$. \item On a $\norm{f}_W \leqs \norm{f}_\infty$. Par cons\'equent $\cE_\infty \subset \cE_\infty^W$. \item Plus g\'en\'eralement, si $W_1$ et $W_2$ sont deux poids tels que $W_1(x) \leqs W_2(x)$ pour tout $x\in\cX$, alors $\norm{f}_{W_2} \leqs \norm{f}_{W_1}$. Par cons\'equent $\cE_\infty^{W_1} \subset \cE_\infty^{W_2}$. \end{itemize} On peut \'egalement d\'efinir une m\'etrique duale \`a $\norm{\cdot}_W$ entre mesures sign\'ees finies de la mani\`ere suivante. \begin{definition}[Distance \`a poids entre mesures] Pour une fonction poids $W: \cX\to [1,\infty[$ et deux mesures sign\'ees finies $\mu, \nu$, on pose \begin{align} \label{eq:def_dist_W} \rho_W(\mu, \nu) &= \sup_{f \colon \norm{f}_W \leqs 1} \sum_{x\in\cX} f(x) \abs{\mu(x) - \nu(x)} \\ &= \sup_{f \colon \norm{f}_W \neq 0} \frac{1}{\norm{f}_W} \sum_{x\in\cX} f(x) \abs{\mu(x) - \nu(x)}\;. \end{align} \end{definition} On a alors les propri\'et\'es suivantes\,: \begin{itemize} \item On peut remplacer $f(x)$ par $\abs{f(x)}$ dans la d\'efinition~\eqref{eq:def_dist_W}. En effet, cette transformation ne change pas $\norm{f}_W$. \item On peut \'egalement remplacer $\abs{\mu(x) - \nu(x)}$ par $\mu(x) - \nu(x)$. En effet, il suffit de changer le signe de $f(x)$ selon le signe de $\mu(x) - \nu(x)$ pour trouver le m\^eme r\'esultat. \item Si $W(x) = 1$ pour tout $x\in\cX$, alors on a \begin{equation} \rho_W(\mu, \nu) = \normTV{\mu - \nu}\;. \end{equation} En effet, le supremum dans~\eqref{eq:def_dist_W} est alors atteint pour la fonction test $f$ valant $1$ partout. \item Pour un poids $W$ g\'en\'eral, on a \begin{equation} \label{eq:rhoW} \rho_W(\mu, \nu) = \sum_{x\in\cX} W(x) \abs{\mu(x) - \nu(x)}\;. \end{equation} En effet, le supremum dans~\eqref{eq:def_dist_W} est atteint pour $f(x) = W(x)$ pour tout $x\in\cX$. \item On a la majoration \begin{equation} \label{eq:majo_munuf} \bigabs{(\mu - \nu)(f)} \leqs \norm{f}_W \rho_W(\mu,\nu)\;. \end{equation} En effet, \begin{equation} \bigabs{(\mu - \nu)(f)} \leqs \sum_{x\in\cX} \abs{\mu(x) - \nu(x)} \, \abs{f(x)} \leqs \norm{f}_W \sum_{x\in\cX} \abs{\mu(x) - \nu(x)} W(x)\;, \end{equation} d'o\`u le r\'esultat, par~\eqref{eq:rhoW}. \end{itemize} \section{Un crit\`ere de convergence} \label{sec:convergence} Les th\'eor\`emes de la section~\ref{sec:Lyap} sont d\^us \`a Meyn et Tweedie~\cite{Meyn_Tweedie_92}. Leurs travaux fournissent \'egalement un r\'esultat de convergence de $\expecin{x}{f(X_n)}$ vers $\pi(f)$, mais les hypoth\`eses sont assez difficiles \`a v\'erifier (notamment, tous les ensembles born\'es doivent \^etre petits), et les bornes obtenues ne sont pas explicites. Le r\'esultat suivant est d\^u \`a Hairer et Mattingly~\cite{Hairer_Mattingly_11}. Les hypoth\`eses sont plus faciles \`a v\'erifier en pratique, et la majoration obtenue pour $\expecin{x}{f(X_n)} - \pi(f)$ a l'avantage de faire intervenir des constantes explicites. \begin{theorem}[Crit\`ere de convergence pour esp\'erances] \label{thm:convergence} Supposons que les deux conditions suivantes soient satisfaites. \begin{enumerate} \item \textbf{Condition de d\'erive g\'eom\'etrique\,:} Il existe $d\geqs0$, $c>0$ et une fonction de Lyapounov $V$ tels que \begin{equation} \label{eq:derive_geom} (\cL V)(x) \leqs -c V(x) + d \qquad \forall x\in\cX\;. \end{equation} \item \textbf{Condition de minoration\,:} Pour un $R > 2d/c$, soit $K = \setsuch{x\in\cX}{V(x) < R}$. Alors il existe $\alpha\in]0,1[$ et une mesure de probabilit\'e $\nu$ telle que \begin{equation} \label{eq:minoration} \inf_{x\in K} p_{xy} = \inf_{x\in K} \bigprobin{x}{X_1 = y} \geqs \alpha \nu(y) \qquad \forall y\in\cX\;. \end{equation} \end{enumerate} \noindent Alors il existe des constantes $M>0$ et $\bar\gamma < 1$ telles que \begin{equation} \label{eq:borne_cv_expec} \norm{\expecin{\cdot}{f(X_n)} - \pi(f)}_{1+V} \leqs M\bar\gamma^n \norm{f - \pi(f)}_{1+V} \end{equation} pour toute fonction test $f\in\cE_\infty^{1+V}$. \end{theorem} Pr\'ecisons qu'on a \begin{equation} \norm{\expecin{\cdot}{f(X_n)} - \pi(f)}_{1+V} = \sup_{x\in\cX} \frac{\abs{\expecin{x}{f(X_n)} - \pi(f)}}{1+V(x)}\;. \end{equation} La majoration~\eqref{eq:borne_cv_expec} peut donc s'\'ecrire \begin{equation} \bigabs{\expecin{x}{f(X_n)} - \pi(f)} \leqs (1+V(x)) M\bar\gamma^n \norm{f - \pi(f)}_{1+V} \qquad \forall x\in\cX\;. \end{equation} Comme $\bar\gamma < 1$, on a donc convergence exponentielle de $\expecin{x}{f(X_n)}$ vers $\pi(f)$. Comme en pratique, on peut souvent choisir $x$ tel que $V(x)$ ne soit pas trop grand, la d\'ependance en $V(x)$ ne pose pas de probl\`eme. La pr\'esence de $\norm{f - \pi(f)}_{1+V}$ n'est pas vraiment restrictive non plus. Si par exemple $f\in\cE_\infty$, ou si $f$ est \`a support compact (c'est-\`a-dire nulle en-dehors d'un ensemble compact), cette quantit\'e est finie. La condition de minoration~\eqref{eq:minoration} est un peu plus faible que la condition~\eqref{eq:proba_lb} du Th\'eo\-r\`eme~\ref{thm:rec_pos}. Le point crucial est que l'on ait une borne inf\'erieure sur les probabilit\'es de transition qui soit ind\'ependante du point de d\'epart dans $K$. Dans la suite, nous utiliserons les notations \begin{align} (\mu\cP)(y) &= \probin{\mu}{X_1 = y} = \sum_{x\in\cX} \mu(x) p_{xy}\;, \\ (\cP f)(x) &= \expecin{x}{f(X_1)} = \sum_{y\in\cX} p_{xy} f(y)\;. \end{align} La condition de d\'erive g\'eom\'etrique~\eqref{eq:derive_geom} est \'equivalente \`a \begin{equation} (\cP V)(x) \leqs \gamma V(x) + d \qquad \forall x\in\cX\;. \end{equation} avec $\gamma = 1-c$. L'ingr\'edient essentiel de la d\'emonstation du Th\'eor\`eme~\ref{thm:convergence} est la borne suivante. \begin{proposition}[L'application $\cP$ est contractante pour la distance $\rho_{1+\beta V}$] \label{prop:contraction} Il existe $\bar\gamma\in]0,1[$ et $\beta>0$ tels que \begin{equation} \rho_{1+\beta V}(\mu\cP,\nu\cP) \leqs \bar\gamma \rho_{1+\beta V}(\mu,\nu)\;. \end{equation} \end{proposition} En fait, la proposition donne des expressions explicites pour les constantes $\beta$ et $\bar\gamma$\,: pour tout choix de $\alpha_0$ et $\gamma_0$ tels que \begin{equation} \label{eq:cond_gamma0} 0 < \alpha_0 < \alpha \qquad\text{et}\qquad \gamma + \frac{2d}{R} < \gamma_0 < 1\;, \end{equation} on peut prendre \begin{align} \beta &= \frac{\alpha_0}{d}\;,\\ \bar\gamma &= \max\biggset{1 - (\alpha-\alpha_0), \frac{2+R\beta\gamma_0}{2+R\beta}} = 1 - \min\biggset{\alpha - \alpha_0, \frac{R\beta(1-\gamma_0)}{2 + R\beta}}\;. \label{eq:beta_gammabar} \end{align} De plus, nous verrons ci-dessous que l'on a \begin{equation} \label{eq:M} M = \frac{1}{1-\bar\gamma} \sup_{x\in\cX} \frac{2 + \beta\brak{(1+\gamma)V(x)+d}}{1 + \beta V(x)} \leqs\frac{\max\set{1+\gamma,2+\beta d}}{1-\bar\gamma}\;. \end{equation} Ces expressions ne sont par particuli\`erement \'el\'egantes, mais elles ont le m\'erite d'\^etre explicites, ce qui peut servir dans les applications. Notons que si $d$ tend vers $0$, alors $\beta$ tend vers l'infini, et on peut prendre $\bar\gamma$ arbitrairement proche de $\gamma$. Nous allons d'abord montrer que la Proposition~\ref{prop:contraction} implique bien le Th\'eor\`eme~\ref{thm:convergence}. \begin{proof}[\textit{D\'emonstration du Th\'eor\`eme~\ref{thm:convergence}}] Nous donnons tout d'abord une d\'emonstation de l'existence de $\pi$, m\^eme si celle-ci suit en fait du Th\'eor\`eme~\ref{thm:rec_pos}. C'est une application du th\'eor\`eme du point fixe de Banach. Fixons $x_0\in\cX$, et soit $\mu_0^{x_0} = \delta_{x_0}$ la mesure de Dirac en $x_0$. Soit $\mu^{x_0}_n = \mu_0^{x_0}\cP^n$. La proposition implique \begin{equation} \rho_{1+\beta V}(\mu^{x_0}_{n+1},\mu^{x_0}_n) \leqs \bar\gamma \rho_{1+\beta V}(\mu^{x_0}_n,\mu^{x_0}_{n-1}) \leqs \dots \leqs \bar\gamma^n \rho_{1+\beta V}(\mu^{x_0}_1,\mu^{x_0}_0)\;. \end{equation} On a donc une suite de Cauchy, et comme on sait que la distance en variation totale est compl\`ete, donc a fortiori la distance $\rho_{1+\beta V}$, on en conclut que la suite des $\mu^{x_0}_n$ converge vers une mesure $\pi$ en variation totale. De plus, $\pi(1+V)$ est finie, car $\rho_{1+\beta V}(\pi, \mu_0^{x_0})$ l'est. Pour montrer que $\pi$ est invariante, il suffit d'observer que \begin{equation} \pi\cP = \lim_{n\to\infty} \mu^{x_0}_0\cP^{n+1} = \lim_{n\to\infty} \mu^{x_0}_0\cP^n = \pi\;. \end{equation} Afin de d\'emontrer~\eqref{eq:borne_cv_expec}, il est utile de centrer $f$. Posons donc \begin{equation} \hat f(x) = f(x) - \pi(f) \qquad \forall x\in\cX\;. \end{equation} Alors $\pi(\hat f) = \pi(f) - \pi(\pi(f)) = 0$, puisque $\pi(\pi(f)) = \pi(f)$. Ainsi \begin{equation} \cP^n f - \pi(f) = \cP^n \hat f + \cP^n \pi(f) - \pi(\hat f) - \pi(\pi(f)) = \cP^n \hat f\;. \end{equation} Ceci permet d'\'ecrire \begin{equation} \norm{\expecin{\cdot}{f(X_n)} - \pi(f)}_{1+V} = \norm{\cP^n f - \pi(f)}_{1+V} = \norm{\cP^n \hat f}_{1+V}\;. \end{equation} Il s'agit donc de montrer qu'il existe une constante $M$ telle que pour toute fonction test $\hat f$ satisfaisant $\pi(\hat f) = 0$, on ait \begin{equation} \norm{\cP^n \hat f}_{1+V} \leqs M\bar\gamma^n \norm{\hat f}_{1+V}\;. \end{equation} Or, comme $(\cP^n \hat f)(x) = \delta_x(\cP^n \hat f) = \mu^x_n(\hat f)$ et $\pi(\hat f) = 0$, on a \begin{equation} \norm{\cP^n \hat f}_{1+\beta V} = \sup_{x\in\cX} \frac{\abs{(\mu^x_n - \pi)(\hat f)}}{1 + \beta V(x)} \leqs \norm{\hat f}_{1+\beta V} \sup_{x\in\cX} \frac{\rho_{1+\beta V}(\mu^x_n,\pi)}{1 + \beta V(x)} \end{equation} en vertu de~\eqref{eq:majo_munuf}. Observons que \begin{align} \rho_{1+\beta V}(\mu^x_n,\pi) &= \rho_{1+\beta V}(\mu^x_n,\mu^x_{n+1}) + \rho_{1+\beta V}(\mu^x_{n+1},\mu^x_{n+2}) + \dots \\ &\leqs \bigbrak{\bar\gamma^n + \bar\gamma^{n+1} + \dots} \rho_{1+\beta V}(\mu^x_1,\mu^x_0) \\ &= \frac{\bar\gamma^n}{1 - \bar\gamma} \rho_{1+\beta V}(\mu^x_1,\mu^x_0)\;. \end{align} De plus, \begin{align} \rho_{1+\beta V}(\mu^x_1,\mu^x_0) &= \sum_{y\in\cX} (1 + \beta V(y)) \abs{\mu^x_1(y) - \mu^x_0(y)} \\ &\leqs \sum_{y\in\cX} (1 + \beta V(y)) \mu^x_1(y) + 1 + \beta V(x) \\ &= (1 + \beta (\cP V))(x) + 1 + \beta V(x)\;. \end{align} Comme $(\cP V)(x) = (\cL V)(x) + V(x) \leqs \gamma V(x) + d$ par l'hypoth\`ese de d\'erive g\'eom\'etrique~\eqref{eq:derive_geom}, on obtient finalement \begin{equation} \norm{\cP^n \hat f}_{1+\beta V} \leqs \norm{\hat f}_{1+\beta V} \frac{\bar\gamma^n}{1 - \bar\gamma} \sup_{x\in\cX} \frac{2 + \beta[V(x) + \gamma V(x) + d]}{1 + \beta V(x)} =: M \bar\gamma^n \norm{\hat f}_{1+\beta V}\;. \end{equation} En particulier, la borne est vraie pour $\beta = 1$, ce qui conclut la d\'emonstation. \end{proof} Il nous reste \`a d\'emontrer la Proposition~\ref{prop:contraction}. L'id\'ee est de travailler avec une d\'efinition alternative de la distance $\rho_\beta$. On introduit sur $\cX$ la distance \begin{equation} d_\beta(x,y) = \begin{cases} 0 & \text{si $x=y$\;,} \\ 2 + \beta V(x) + \beta V(y) & \text{si $x\neq y$\;,} \end{cases} \end{equation} (on v\'erifie facilement que $d_\beta$ satisfait bien la d\'efinition d'une distance), et la semi-norme de Lipschitz \begin{equation} \label{def:seminormf} \normDgamma{f}_\beta = \sup_{x\neq y} \frac{\abs{f(x)-f(y)}}{d_\beta(x,y)}\;, \end{equation} C'est une semi-norme, et non une norme, car $\normDgamma{f}_\beta = 0$ n'implique pas $f = 0$ (mais seulement que $f$ est constante). On a alors le r\'esultat suivant. \begin{lemma}[\'Equivalence des distances] \label{lem:equiv_distances} On a \begin{equation} \label{eq:lem_equiv_distances} \rho_{1+\beta V}(\mu,\nu) = \rho^*_{1+\beta V}(\mu,\nu) := \sup_{f: \normDgamma{f}_\beta \leqs 1} \sum_{x\in\cX} f(x) (\mu(x) - \nu(x))\;. \end{equation} \end{lemma} Notons que la seule diff\'erence entre la d\'efinition~\eqref{eq:def_dist_W} de $\rho_{1+\beta V}(\mu,\nu)$ et cette \'egalit\'e est l'ensemble des $f$ sur lequel on prend le supremum. Montrons tout d'abord que ce lemme implique la proposition. \begin{proof}[\textit{D\'emonstration de la Proposition~\ref{prop:contraction}}] L'id\'ee est de montrer que $\cP$ est contractante dans la seminorme $\normDgamma{\cdot}_\beta$, \`a savoir \begin{equation} \label{eq:contraction_seminorm} \normDgamma{\cP f}_\beta \leqs \bar\gamma \normDgamma{f}_\beta\;. \end{equation} En effet, ceci implique \begin{align} \rho_{1+\beta V}(\mu\cP, \nu\cP) &= \sup_{f\colon \normDgamma{f}_\beta \neq 0} \frac{1}{\normDgamma{f}_\beta} (\mu\cP - \nu\cP)(f) \\ &= \sup_{f\colon \normDgamma{f}_\beta \neq 0} \frac{1}{\normDgamma{f}_\beta} (\mu - \nu)(\cP f) \\ &\leqs \sup_{f_1\colon \normDgamma{f_1}_\beta \neq 0} \frac{\bar\gamma}{\normDgamma{f_1}_\beta} (\mu - \nu)(f_1) \\ &= \bar\gamma\rho_{1+\beta V}(\mu, \nu)\;. \end{align} Afin de d\'emontrer~\eqref{eq:contraction_seminorm}, nous fixons un $f$ tel que $\norm{f}_{1+\beta V} \leqs 1$ et $\normDgamma{f}_\beta\leqs1$. Il s'agit de montrer que \begin{equation} \bigabs{(\cP f)(x) - (\cP f)(y)} \leqs \bar\gamma d_\beta(x,y) \end{equation} pour tout $x, y\in\cX$. La relation est vraie pour $x=y$, donc nous supposons $x\neq y$. Nous consid\'erons deux cas s\'epar\'ement. \begin{itemize} \item \textbf{Cas 1:} $V(x) + V(y) \geqs R$. Dans ce cas, on a \begin{equation} \label{eq:borne_cas1} \bigabs{(\cP f)(x)} = \biggabs{\sum_{y\in\cX} p_{xy} f(y)} \leqs \underbrace{\norm{f}_{1+\beta V}}_{\leqs 1} \sum_{y\in\cX} (1+\beta V(y)) p_{xy} \leqs 1 + \beta (\cP V)(x)\;, \end{equation} d'o\`u, par la condition~\eqref{eq:derive_geom} de d\'erive g\'eom\'etrique, \begin{align} \bigabs{(\cP f)(x) - (\cP f)(y)} &\leqs 2 + \beta (\cP V)(x) + \beta (\cP V)(y) \\ &\leqs 2 + \beta\gamma V(x) + \beta\gamma V(y) + 2\beta d\\ &\leqs 2 + \beta\gamma_0 [V(x) + V(y)] \end{align} pour tout $\gamma_0$ satisfaisant~\eqref{eq:cond_gamma0}. Un calcul \'el\'ementaire montre alors qu'en posant \begin{equation} \gamma_1 = \frac{2 + \beta R\gamma_0}{2+\beta R}\;, \end{equation} on obtient bien la majoration requise \begin{equation} \bigabs{(\cP f)(x) - (\cP f)(y)} \leqs \gamma_1 \bigpar{2 + V(x) + V(y)} = \gamma_1 d_\beta(x,y)\;. \end{equation} \item \textbf{Cas 2:} $V(x) + V(y) < R$. Dans ce cas, on a $x, y\in K$. La matrice $\widetilde\cP$ d'\'el\'ements \begin{equation} \tilde p_{xy} = \frac{1}{1-\alpha} p_{xy} - \frac{\alpha}{1-\alpha} \nu(y) \end{equation} est une matrice stochastique. En effet, la condition de minoration~\eqref{eq:minoration} montre que ces \'el\'ements sont tous positifs ou nuls, et on v\'erifie imm\'ediatement que la somme sur $y$ des $\tilde p_{xy}$ vaut $1$. De plus, on a \begin{equation} (\cP f)(x) = (1-\alpha) (\widetilde\cP f)(x) + \alpha \nu(f)\;, \end{equation} d'o\`u, par un calcul analogue \`a~\eqref{eq:borne_cas1}, \begin{align} \bigabs{(\cP f)(x) - (\cP f)(y)} &= (1-\alpha) \bigabs{(\widetilde\cP f)(x) - (\widetilde\cP f)(y)} \\ &\leqs (1-\alpha) \bigbrak{2 + \beta (\widetilde\cP V)(x) + \beta (\widetilde\cP V)(y)}\\ &\leqs (1-\alpha) + \beta (\cP V)(x) + \beta (\cP V)(y)\\ &\leqs 2(1-\alpha) + \beta\gamma [V(x) + V(y)] + 2\beta d\\ &\leqs \gamma_2 d_\beta(x,y)\;, \end{align} si $\beta = \frac{\alpha_0}{d}$ est donn\'e par~\eqref{eq:beta_gammabar} et $\gamma_2 = \max\set{\gamma, 1 - (\alpha-\alpha_0)}$. La troisi\`eme ligne suit du fait que \begin{equation} (\widetilde \cP V)(x) \leqs \frac{1}{1-\alpha} (\cP V)(x)\;. \end{equation} \end{itemize} Le r\'esultat suit, avec $\bar\gamma = \max\set{\gamma_1,\gamma_2}$. \end{proof} Il nous reste \`a d\'emontrer le Lemme~\ref{lem:equiv_distances}. \begin{proof}[\textit{D\'emonstration du Lemme~\ref{lem:equiv_distances}}] D\'efinissons les boules \begin{equation} \cB = \setsuch{f\in\cE_\infty}{\norm{f}_{1+\beta V}\leqs 1}\;, \qquad \cB^* = \setsuch{f\in\cE_\infty}{\normDgamma{f}_{\beta}\leqs 1}\;. \end{equation} Alors on peut \'ecrire \begin{equation} \rho_{1+\beta V}(\mu,\nu) = \sup_{f\in\cB} f(\mu - \nu)\;, \qquad \rho^*_{1+\beta V}(\mu,\nu) = \sup_{f\in\cB^*} f(\mu - \nu)\;. \end{equation} L'observation cruciale est la suivante. Pour $c\in\R$, soit $f+c$ la fonction translat\'ee de $c$, d\'efinie par $(f+c)(x) = f(x) + c$ pour tout $x\in\cX$. Alors on a \begin{equation} (f+c)(\mu-\nu) = \sum_{x\in\cX}(f(x)+c)(\mu(x)-\nu(x)) = f(\mu-\nu) + c\sum_{x\in\cX}\mu(x) - c\sum_{x\in\cX}\nu(x) = f(\mu-\nu)\;, \end{equation} puisque $\mu$ et $\nu$ sont des mesures de probabilit\'e. Par cons\'equent, on a aussi \begin{equation} \rho_{1+\beta V}(\mu,\nu) = \sup_{f\in\widehat\cB} f(\mu - \nu)\;, \qquad \rho^*_{1+\beta V}(\mu,\nu) = \sup_{f\in\widehat\cB^*} f(\mu - \nu)\;, \end{equation} o\`u on a d\'efini les \myquote{cylindres}\ \begin{equation} \widehat\cB = \setsuch{f+c}{f\in\cB, c\in\R}\;, \qquad \widehat\cB^* = \setsuch{f+c}{f\in\cB^*, c\in\R}\;. \end{equation} Ces cylindres sont obtenus en \'etendant les boules $\cB$ et $\cB^*$ dans la direction des fonctions constantes. Nous allons montrer ci-dessous que \begin{equation} \label{eq:triple_norm_inf} \normDgamma{f}_\beta = \inf_{c\in\R} \norm{f+c}_{1+\beta V}\;. \end{equation} Cela impliquera que $\widehat\cB = \widehat\cB^*$, et par cons\'equent que $\rho_{1+\beta V}(\mu,\nu) = \rho^*_{1+\beta V}(\mu,\nu)$, qui est l'\'egalit\'e voulue. Comme remarqu\'e juste apr\`es la D\'efinition~\ref{def:norme_poids_fct_test}, on a, pour tout $x\in\cX$, \begin{equation} \abs{f(x)} \leqs \norm{f}_{1+\beta V}(1 + \beta V(x))\;. \end{equation} Par cons\'equent, on a pour tout $x\neq y\in\cX$ \begin{equation} \frac{\abs{f(x)-f(y)}}{d_\beta(x,y)} \leqs \frac{\abs{f(x)} + \abs{f(y)}}{2 + \beta V(x) + \beta V(y)} \leqs \norm{f}_{1+\beta V}\;. \end{equation} Ceci implique \begin{equation} \normDgamma{f}_\beta \leqs \norm{f}_{1+\beta V}\;. \end{equation} Comme de plus la d\'efinition de $\normDgamma{f}_\beta$ ne d\'epend que de diff\'erences de $f$ en des points diff\'erents, on a $\normDgamma{f+c}_\beta = \normDgamma{f}_\beta$ pour tout $c\in\R$. On a donc obtenu \begin{equation} \normDgamma{f}_\beta \leqs \inf_{c\in\R} \norm{f+c}_{1+\beta V}\;. \end{equation} Pour montrer l'in\'egalit\'e inverse, posons \begin{equation} c^* = \inf_{x\in\cX} \Bigpar{1 + \beta V(x) - f(x)}\;. \end{equation} Commen\c cons par montrer que $\abs{c^*} < \infty$. Comme $c^* \leqs \bigpar{1 + \beta V(x_0) - f(x_0)}$ pour tout $x_0\in\cX$, il suffit pour cela de montrer que $c^*$ est born\'ee inf\'erieurement. Cela suit du fait que pour tout $x,y\in\cX$, on a \begin{equation} f(x) \leqs \abs{f(y)} + \abs{f(x)-f(y)} \leqs \abs{f(y)} + 2 + \beta V(x) + \beta V(y)\;, \end{equation} qui implique \begin{equation} 1 + \beta V(x) - f(x) \geqs -1 -\beta V(y) - \abs{f(y)}\;. \end{equation} Comme $V(y)$ est finie pour au moins un $y\in\cX$, on a bien la minoration voulue. On observe maintenant que, d'une part, \begin{equation} \label{eq:ff_bornesup} f(x) + c^* \leqs f(x) + 1 + \beta V(x) - f(x) = 1 + \beta V(x)\;, \end{equation} alors que d'autre part, \begin{align} \label{eq:ff_borneinf} f(x) + c^* &= \inf_{y\in\cX} \bigpar{f(x) + 1 + \beta V(y) - f(y)} \\ &\geqs \inf_{y\in\cX} \bigpar{1 + \beta V(y) - \underbrace{\normDgamma{f}_\beta}_{= 1} d_\beta(x,y)} \\ &\geqs \inf_{y\in\cX} \bigpar{1 + \beta V(y) - 2 - \beta V(x) - \beta V(y)} \\ &\geqs -1 - \beta V(x)\;. \end{align} Il suit de~\eqref{eq:ff_bornesup} et~\eqref{eq:ff_borneinf} que \begin{equation} \abs{f(x) + c^*} \leqs 1+\beta V(x)\;, \end{equation} d'o\`u l'in\'egalit\'e inverse \begin{equation} \inf_{c\in\R} \norm{f+c}_{1+\beta V} \leqs \norm{f+c^*}_{1+\beta V} \leqs 1 = \normDgamma{f}_\beta\;. \end{equation} Nous avons donc d\'emontr\'e~\eqref{eq:triple_norm_inf}, et par suite aussi~\eqref{eq:lem_equiv_distances}. \end{proof} \section{Exercices} \label{sec:Lyapounov_exo} \begin{exercise} On consid\`ere la marche al\'eatoire sym\'etrique sur $\Z$. \begin{enumerate} \item Calculer $(\cL V)(x)$ pour la fonction de Lyapounov $V(x) = x^2$. Expliciter la formule de Dynkin pour un temps $n$ d\'eterministe. Appliquer, si possible, les th\'eor\`emes de croissance sous-exponentielle, non-explosion, r\'ecurrence positive et de convergence. \item Calculer $(\cL V)(x)$ pour la fonction de Lyapounov $V(x) = \abs{x}$. Comme au point pr\'ec\'edent, expliciter la formule de Dynkin, et appliquer, si possible les diff\'erents th\'eor\`emes. \item Que se passe-t-il pour la marche al\'eatoire asym\'etrique~? \end{enumerate} \end{exercise} \begin{exercise} Soit $p\in]0,1[$ et $q = 1 - p$. On consid\`ere la marche al\'eatoire sur $\N$ de probabilit\'es de transition \[ p_{xy} = \begin{cases} p & \text{si $y = x+1$\;,} \\ q & \text{si $y = 0$\;,} \\ 0 & \text{sinon\;.} \end{cases} \] Calculer $(\cL V)(x)$ pour la fonction de Lyapounov $V(x) = \abs{x}$. Que peut-on en d\'eduire~? \end{exercise} \begin{exercise} Pour $x\in\N$, on d\'enote par \[ \intpart{x} = \max\setsuch{y\in\N}{y\leqs x} \] la partie enti\`ere de $x$. Soit $p\in[0,1]$. On consid\`ere la cha\^ine de Markov sur $\N=\set{0,1,2,\dots}$ de probabilit\'es de transition \[ p_{xy} = \begin{cases} p & \text{si $y = x + 1$\;,}\\ 1 - p & \text{si $y = \intpart{x/2}$\;,}\\ 0 & \text{sinon\;.} \end{cases} \] Pour $\alpha\in\R$, on pose $V(x) = \e^{\alpha x}$. \begin{enumerate} \item Pour quelles valeurs de $\alpha$ la fonction $V$ est-elle une fonction de Lyapounov~? On dira que ces valeurs de $\alpha$ sont \emph{admissibles}. \item Calculer $(\cL V)(x)$, o\`u $\cL$ est le g\'en\'erateur de la cha\^ine de Markov. On distinguera les cas $x$ pair et $x$ impair. \item Pour quels $p$ la cha\^ine est-elle \`a croissance sous-exponentielle~? \item D\'eterminer une fonction $f_p(\alpha)$ telle que \[ (\cL V)(x) \leqs - f_p(\alpha) V(x) \] pour tout $x\in\N^* = \set{1,2,\dots}$. \item \'Etudier la fonction $\alpha\mapsto f_p(\alpha)$ pour les valeurs de $\alpha$ admissibles~: comportement aux bords du domaine, croissance/d\'ecroissance, convexit\'e. \item Pour quelles valeurs de $p$ existe-t-il un $\alpha$ admissible tel que $f_p(\alpha) > 0$~? \item Pour quelles valeurs de $p$ peut-on affirmer l'existence d'une unique probabilit\'e invariante $\pi$ telle que la loi de $X_n$ converge exponentiellement vite vers $\pi$~? \end{enumerate} \end{exercise} \chapter{Algorithmes MCMC} \label{chap:cm_MCMC} \section{M\'ethodes Monte Carlo} \label{sec:MC} On appelle \defwd{m\'ethodes Monte Carlo}\/ un ensemble d'algorithmes stochastiques, introduits dans les ann\'ees 1940 par le mathématicien polonais Stanis\l{}aw Ulam, permettant d'estimer num\'eriquement des grandeurs pouvant \^etre consid\'er\'ees comme des esp\'erances. En voici pour commencer un exemple tr\`es simple. \begin{example}[Calcul d'un volume] On aimerait calculer num\'eriquement le volume $\abs{V}$ d'un sous-ensemble compact $V$ de $\R^N$. On suppose que $V$ est donn\'e par un certain nombre $M$ d'in\'egalit\'es: \begin{equation} \label{mcmc1} V = \bigsetsuch{x\in\R^N}{f_1(x)\geqs0, \dots, f_M(x)\geqs0}\;. \end{equation} Par exemple, si $M=1$ et $f_1(x)=1-\norm{x}^2$, alors $V$ est une boule. Dans ce cas, bien s\^ur, le volume de $V$ est connu explicitement. On s'int\'eresse \`a des cas o\`u $V$ a une forme plus compliqu\'ee, par exemple une intersection d'un grand nombre de boules et de demi-espaces. Dans la suite nous supposerons, sans limiter la g\'en\'eralit\'e, que $V$ est contenu dans le cube unit\'e $[0,1]^N$. \end{example} Une premi\`ere m\'ethode de calcul num\'erique du volume consiste \`a discr\'etiser l'espace. Divisons le cube $[0,1]^N$ en cubes de cot\'e $\eps$ (avec $\eps$ de la forme $1/K$, $K\in\N$). Le nombre total de ces cubes est \'egal \`a $1/\eps^N=K^N$. On compte alors le nombre $n$ de cubes dont le centre est contenu dans $V$, et le volume de $V$ est approximativement \'egal \`a $n\eps^N$. Plus pr\'ecis\'ement, on peut encadrer $\abs{V}$ par $n_-\eps^N$ et $n_+\eps^N$, o\`u $n_-$ est le nombre de cubes enti\`erement contenus dans $V$, et $n_+$ est le nombre de cubes dont l'intersection avec $V$ est non vide (Figure~\ref{fig:discretisation}). Toutefois, effectuer ces tests n'est en g\'en\'eral pas ais\'e num\'eriquement. Quelle est la pr\'ecision de l'algorithme~? Si le bord $\partial V$ est raisonnablement lisse, l'erreur faite pour $\eps$ petit est de l'ordre de la mesure $\abs{\partial V}$ du bord fois $\eps$. Pour calculer $\abs{V}$ avec une pr\'ecision donn\'ee $\delta$, il faut donc choisir $\eps$ d'ordre $\delta/\abs{\partial V}$. Cela revient \`a un nombre de cubes d'ordre \begin{equation} \label{mcmc2} \biggpar{\frac{\abs{\partial V}}{\delta}}^N\;, \end{equation} ou encore, comme on effectue $M$ tests pour chaque cube, \`a un nombre de calculs d'ordre $(M\abs{\partial V}/\delta)^N$. Ce nombre ne pose pas de probl\`eme pour les petites dimensions ($N=1,2$ ou $3$ par exemple), mais cro\^\i t vite avec la dimension $N$. C'est ce qu'on appelle le \defwd{fl\'eau de la dimension}. \begin{figure} \vspace{-3mm} \begin{center} \scalebox{0.8}{ \begin{tikzpicture}[->,>=stealth',shorten >=2pt,shorten <=2pt,auto,node distance=3.0cm,thick, cont/.style={thick,rectangle,scale=1,minimum size=1cm, fill=green!80!blue!50,draw,font=\sffamily\Large}, int/.style={thick,,rectangle,scale=1,minimum size=1cm, fill=red!40!blue!50,draw,font=\sffamily\Large}] \node[int] at (-5,0) {}; \node[int] at (-4,0) {}; \node[int] at (2,0) {}; \node[int] at (-5,-1) {}; \node[int] at (2,-1) {}; \node[int] at (3,-1) {}; \node[int] at (4,-1) {}; \node[int] at (-5,-2) {}; \node[int] at (4,-2) {}; \node[int] at (5,-2) {}; \node[int] at (-5,-3) {}; \node[int] at (-4,-3) {}; \node[int] at (4,-3) {}; \node[int] at (3,-3) {}; \node[int] at (5,-3) {}; \node[int] at (-3,-4) {}; \node[int] at (-2,-4) {}; \node[int] at (-1,-4) {}; \node[int] at (0,-4) {}; \node[int] at (1,-4) {}; \node[int] at (2,-4) {}; \node[int] at (-4,1) {}; \node[int] at (-3,1) {}; \node[int] at (2,1) {}; \node[int] at (3,1) {}; \node[int] at (4,1) {}; \node[int] at (-3,2) {}; \node[int] at (-2,2) {}; \node[int] at (-1,2) {}; \node[int] at (4,2) {}; \node[int] at (5,2) {}; \node[int] at (-1,3) {}; \node[int] at (0,3) {}; \node[int] at (1,3) {}; \node[int] at (2,3) {}; \node[int] at (3,3) {}; \node[int] at (4,3) {}; \node[int] at (5,3) {}; \draw[blue,ultra thick] plot[smooth cycle,tension=0.8] coordinates{(-5,-3) (5,-3) (2,0) (5,2.5) (-2,2)}; \node[cont] at (-3,0) {}; \node[cont] at (-2,0) {}; \node[cont] at (-1,0) {}; \node[cont] at (0,0) {}; \node[cont] at (1,0) {}; \node[cont] at (-2,1) {}; \node[cont] at (-1,1) {}; \node[cont] at (0,1) {}; \node[cont] at (1,1) {}; \node[cont] at (0,2) {}; \node[cont] at (1,2) {}; \node[cont] at (2,2) {}; \node[cont] at (3,2) {}; \node[cont] at (-4,-1) {}; \node[cont] at (-3,-1) {}; \node[cont] at (-2,-1) {}; \node[cont] at (-1,-1) {}; \node[cont] at (0,-1) {}; \node[cont] at (1,-1) {}; \node[cont] at (-4,-2) {}; \node[cont] at (-3,-2) {}; \node[cont] at (-2,-2) {}; \node[cont] at (-1,-2) {}; \node[cont] at (0,-2) {}; \node[cont] at (1,-2) {}; \node[cont] at (2,-2) {}; \node[cont] at (3,-2) {}; \node[cont] at (-3,-3) {}; \node[cont] at (-2,-3) {}; \node[cont] at (-1,-3) {}; \node[cont] at (0,-3) {}; \node[cont] at (1,-3) {}; \node[cont] at (2,-3) {}; \end{tikzpicture} } \end{center} \vspace{-4mm} \caption[]{Calcul d'une aire par discr\'etisation. L'aire est encadr\'ee par $n_-\eps^2$ et $n_+\eps^2$, o\`u $n_-$ est le nombre de carr\'es de c\^ot\'e $\eps$ enti\`erement contenus dans le domaine, et $n_+$ est le nombre de carr\'es intersectant le domaine.} \label{fig:discretisation} \end{figure} Une alternative int\'eressante pour les $N$ grands est fournie par l'\emph{algorithme de Monte Carlo}. Dans ce cas, on g\'en\`ere une suite $X_1, X_2, \dots, X_n, \dots$ de variables al\'eatoires ind\'epen\-dantes, identiquement distribu\'ees (i.i.d.), de loi uniforme sur $[0,1]^N$. Ceci est facile \`a r\'ealiser num\'eriquement, car on dispose de g\'en\'erateurs de nombres (pseudo-)al\'eatoires distribu\'es uniform\'ement sur $[0,1]$ (ou plut\^ot sur $\set{0,1,\dots,n_{\text{max}}}$ o\`u $n_{\text{max}}$ est un entier du genre $2^{31}-1$, mais en divisant ces nombres par $n_{\text{max}}$, on obtient de bonnes approximations de variables uniformes sur $[0,1]$). Il suffit alors de consid\'erer des $N$-uplets de tels nombres. Consid\'erons alors les variables al\'eatoires i.i.d. \begin{equation} \label{mcmc3} Y_n = \indicator{X_n\in V}\;, \qquad n = 1,2,\dots\;. \end{equation} On aura \begin{equation} \label{mcmc4} \expec{Y_n} = \bigprob{X_n\in V} = \abs{V}\;. \end{equation} Les moyennes empiriques \begin{equation} \label{mcmc5} S_n = \frac1n \sum_{m=1}^n Y_m \end{equation} ont esp\'erance $\expec{S_n}=\abs{V}$ et variance $\variance{S_n}=\variance{Y_1}/n$. La loi faible des grands nombres implique que \begin{equation} \label{mcmc6} \lim_{n\to\infty} \Bigprob{\bigabs{S_n-\expec{S_n}} > \delta} = 0 \end{equation} pour tout $\delta>0$. Par cons\'equent, $S_n$ devrait donner une bonne approximation du volume $\abs{V}$ lorsque $n$ est suffisamment grand.\footnote{La loi forte des grands nombres affirme par ailleurs que $S_n$ tend vers $\abs{V}$ presque s\^urement, c'est-\`a-dire que $S_n$ n'est plus vraiment al\'eatoire \`a la limite.} Pour savoir comment choisir $n$ en fonction de la pr\'ecision d\'esir\'ee, il nous faut estimer la probabilit\'e que $\abs{S_n-\abs{V}} > \delta$, pour $n$ grand mais fini. Une premi\`ere estimation est fournie par l'in\'egalit\'e de Bienaym\'e--Chebychev, qui affirme que \begin{equation} \label{mcmc7} \Bigprob{\bigabs{S_n-\expec{S_n}} > \delta} \leqs \frac{\variance(S_n)}{\delta^2} = \frac{\variance(Y_1)}{\delta^2 n} < \frac{1}{\delta^2 n}\;, \end{equation} o\`u nous avons utilis\'e le fait que $\variance(Y_1)\leqs\expec{Y_1^2}\leqs 1$. On peut donc affirmer que pour que la probabilit\'e de faire une erreur sup\'erieure \`a $\delta$ soit inf\'erieure \`a $\eps$, il suffit de choisir \begin{equation} \label{mcmc8} n > \frac1{\delta^2\eps}\;. \end{equation} Comme pour chaque $m$, il faut g\'en\'erer $N$ variables al\'eatoires, et effectuer $M$ tests, le nombre de calculs n\'ecessaires est d'ordre $MN/(\delta^2\eps)$. L'avantage de cette m\'ethode est que ce nombre ne cro\^\i t que lin\'eairement avec $N$, par opposition \`a la croissance exponentielle dans le cas de la discr\'etisation. On notera toutefois que contrairement \`a la discr\'etisation, qui donne un r\'esultat certain aux erreurs pr\`es, la m\'ethode de Monte Carlo ne fournit que des r\'esultats vrais avec tr\`es grande probabilit\'e (d'o\`u son nom). \begin{remark}[Estimation d'erreur am\'elior\'ee] L'estimation~\eqref{mcmc7} est assez pessimiste, et peut \^etre consid\'erablement am\'elior\'ee. Par exemple, le th\'eor\`eme central limite montre que \begin{equation} \label{mcmc9} \lim_{n\to\infty} \Biggprob{\frac{(S_n-\expec{S_n})^2}{\variance(S_n)} > \eta^2} = \int_{\abs{x}>\eta} \frac{\e^{-x^2/2}}{\sqrt{2\pi}} \6x\;, \end{equation} dont le membre de droite d\'ecro\^\i t comme $\e^{-\eta^2/2}$ pour $\eta$ grand. Cela indique que pour $n$ grand, \begin{equation} \label{mcmc10} \Bigprob{\abs{S_n-\abs{V}} > \delta} \simeq \e^{-n\delta^2/2\variance(Y_1)}\;. \end{equation} Ceci permet d'am\'eliorer le crit\`ere~\eqref{mcmc8} en \begin{equation} \label{mcmc11} n > \const \frac{\log(1/\eps)}{\delta^2}\;, \end{equation} d'o\`u un nombre de calculs d'ordre $NM\log(1/\eps)/\delta^2$. Cette estimation n'est pas une borne rigoureuse, contrairement \`a~\eqref{mcmc8}, parce qu'on n'a pas tenu compte de la vitesse de convergence dans~\eqref{mcmc9}, qui par ailleurs ne s'applique que pour $\eta$ ind\'ependant de $\eps$. On devrait plut\^ot utiliser des estimations provenant de la th\'eorie des grandes d\'eviations, que nous n'aborderons pas ici. Les r\'esultats sont toutefois qualitativement corrects. \end{remark} \begin{example}[Estimation d'un volume] A titre d'illustration, supposons qu'on veuille d\'eterminer le volume d'un domaine de dimension $N=1000$, d\'efini par $M=10$ in\'egalit\'es, avec une pr\'ecision de $\delta=10^{-4}$. La m\'ethode de discr\'etisation n\'ecessite un nombre de calculs d'ordre $10^{5000}$, ce qui est irr\'ealisable avec les ordinateurs actuels. La m\'ethode de Monte Carlo, en revanche, fournit un r\'esultat de la m\^eme pr\'ecision, s\^ur avec probabilit\'e $1-10^{-6}$, avec un nombre de calculs d'ordre $\log(10^6)\cdot 10^{12} \simeq 10^{13}$, ce qui ne prend que quelques minutes sur un PC. La Table~\ref{tab:MC} compare des co\^uts pour diff\'erentes valeurs de $N$. \end{example} \begin{table}[ht] \begin{center} \begin{tabular}{|r|r|r|r|} \hline \myvrule{13pt}{5pt}{0pt} $N$ & Discr\'etisation & Monte Carlo BC & Monte Carlo TCL \\ \hline \myvrule{13pt}{5pt}{0pt} $1$ & $10^5$ & $10^{15}$ & $1,\!4\cdot 10^{10}$ \\ $2$ & $10^9$ & $2\cdot10^{15}$ & $2,\!8\cdot 10^{10}$ \\ $3$ & $10^{13}$ & $3\cdot10^{15}$ & $4,\!2\cdot 10^{10}$ \\ $10$ & $10^{41}$ & $10^{16}$ & $1,\!4\cdot 10^{11}$ \\ $100$ & $10^{401}$ & $10^{17}$ & $1,\!4\cdot 10^{12}$ \\ $1000$ & $10^{4001}$ & $10^{18}$ & $1,\!4\cdot 10^{13}$ \\ \hline \end{tabular} \end{center} \vspace{-3mm} \caption[]{Comparaison, pour diff\'erentes dimensions $N$ et un nombre $M=10$ d'in\'egalit\'es, des co\^uts de calcul d'un volume avec pr\'ecision $\delta = 10^{-4}$, par discr\'etisation ($10^{4N+1}$), par la m\'ethode de Monte Carlo avec $\eps=10^{-6}$ avec l'estimation de Bienaym\'e--Chebychev ($10^{15} N$), et par la m\^eme m\'ethode avec l'estimation bas\'ee sur le th\'eor\`eme central limite ($\log(10^6)\cdot 10^9 N$).} \label{tab:MC} \end{table} La m\'ethode de Monte Carlo se g\'en\'eralise facilement \`a d'autres probl\`emes que des calculs de volume. Supposons par exemple donn\'e un espace probabilis\'e $(\Omega,\cF,\pi)$, et une variable al\'eatoire $Y:\Omega\to\R$. On voudrait estimer l'esp\'erance de $Y$. Pour cela, l'algorithme de Monte Carlo consiste \`a g\'en\'erer des variables al\'eatoires ind\'ependantes $Y_1, Y_2, \dots, Y_n, \dots$, toutes de loi $\pi Y^{-1}$, et de calculer leur moyenne. Cette moyenne doit converger vers l'esp\'erance cherch\'ee (pourvu que $Y$ soit int\'egrable). Cet algorithme n'est toutefois efficace que si l'on arrive \`a g\'en\'erer les variables al\'eatoires $Y_i$ de mani\`ere efficace. Une fois de plus, ceci est relativement facile en dimension faible, mais devient rapidement difficile lorsque la dimension cro\^\i t. \begin{remark}[Cas unidimensionnel] \label{rem:mcmc2} Une variable al\'eatoire uni\-dimensionnelle $Y$ s'ob\-tient facilement \`a partir d'une variable de loi uniforme. Soit en effet $U$ une variable uniforme sur $[0,1]$. Sa fonction de r\'epartition est donn\'ee par \begin{equation} \label{mcmc12} F_U(u) = \prob{U\leqs u} = u \qquad \text{pour $0\leqs u\leqs 1$.} \end{equation} On cherche une fonction $\varphi$ telle que la variable $Y=\varphi(u)$ admette la fonction de r\'eparti\-tion prescrite $F_Y(y)$. Or on a \begin{equation} \label{mcmc13} F_Y(y) = \prob{Y\leqs y} = \prob{\varphi(U)\leqs y} = \prob{U \leqs \varphi^{-1}(y)} = \varphi^{-1}(y)\;. \end{equation} Il suffit donc de prendre $Y = F_Y^{-1}(U)$. \end{remark} \begin{example}[Loi exponentielle] Par exemple, pour g\'en\'erer une variable de loi exponentielle, dont la fonction de r\'eparti\-tion est donn\'ee par $F_Y(y)=1-\e^{-\lambda y}$, il suffit de prendre \begin{equation} \label{mcmc14} Y = -\frac1\lambda \log(1-U)\;. \end{equation} \end{example} \begin{example}[Loi normale -- Algorithme de Box--Muller] Pour la loi normale, cette m\'ethode n\'ecessiterait le calcul approch\'e de sa fonction de r\'epartition, ce qui est num\'eriquement peu efficace. Il existe toutefois une alternative permettant d'\'eviter ce calcul. Soient en effet $U$ et $V$ deux variables al\'eatoires ind\'ependantes, de loi uniforme sur $[0,1]$. On introduit successivement les variables \begin{align} \nonumber R &= \sqrt{-2\log(1-U)}\;, & Y_1 &= R\cos\Phi\;, \\ \Phi &= 2\pi V\;, & Y_2 &= R\sin\Phi\;. \label{mcmc15} \end{align} Alors $Y_1$ et $Y_2$ sont des variables al\'eatoires ind\'ependantes, de loi normale centr\'ee r\'eduite. Pour le voir, on v\'erifie d'abord que $R$ a la fonction de r\'epartition $1-\e^{-r^2/2}$, donc la densit\'e $r\e^{-r^2/2}$. Le couple $(R,\Phi)$ a donc la densit\'e jointe $r\e^{-r^2/2}/(2\pi)$, et la formule de changement de variable montre que le couple $(Y_1,Y_2)$ a la densit\'e jointe $\e^{-(y_1^2+y_2^2)/2}/(2\pi)$, qui est bien celle d'un couple de variables normales centr\'ees r\'eduites ind\'ependantes. \end{example} Bien entendu, les esp\'erances de variables al\'eatoires de loi unidimensionnelle sont soit connues explicitement, soit calculables num\'eriquement par la simple estimation d'une int\'egrale. Nous nous int\'eressons ici \`a des situations o\`u la loi de $Y$ n'est pas aussi simple \`a repr\'esenter. \section{M\'ethodes Monte Carlo par cha\^ines de Markov} \label{sec:MCMC} Consid\'erons le cas d'un espace probabilis\'e discret $(\cX,\cP(\cX),\pi)$, o\`u $\cX$ est un ensemble d\'enombrable, mais tr\`es grand. Par exemple, dans le cas du mod\`ele d'Ising (voir la section~\ref{sec:ex_Ising}), $\cX=\set{-1,1}^N$ est de cardinal $2^N$, et on s'int\'eresse \`a des $N$ grands, par exemple d'ordre $1000$. La mesure de probabilit\'e $\pi$ est dans ce cas une application de $\cX$ vers $[0,1]$ telle que la somme des $\pi(x)$ vaut $1$. On voudrait estimer l'esp\'erance d'une variable al\'eatoire $Y:\cX\to\R$, comme par exemple l'aimantation dans le cas du mod\`ele d'Ising~: \begin{equation} \label{mcmc16} \expecin{\pi}{Y} = \sum_{x\in\cX} Y(x)\pi(x)\;. \end{equation} La m\'ethode de Monte Carlo usuelle consiste alors \`a g\'en\'erer une suite de variables al\'eatoires $X_0, X_1, \dots$ sur $\cX$, ind\'ependantes et de loi $\pi$, puis de calculer la moyenne des $Y(X_m)$. Pour g\'en\'erer ces $X_m$, on pourrait envisager de proc\'eder comme suit~: on d\'efinit un ordre total sur $\cX$, et on d\'etermine la fonction de r\'epartition \begin{equation} \label{mcmc17} x \mapsto F_\pi(x) = \sum_{y\leqs x} \pi(y)\;. \end{equation} Si $U$ est une variable de loi uniforme sur $[0,1]$, alors $F_\pi^{-1}(U)$ suit la loi $\pi$. Toutefois, en proc\'edant de cette mani\`ere, on ne gagne rien, car le calcul des sommes~\eqref{mcmc17} est aussi long que le calcul de la somme~\eqref{mcmc16}, que l'on voulait pr\'ecis\'ement \'eviter~! Les m\'ethodes MCMC (pour \emph{Monte Carlo Markov Chain}\/) \'evitent cet inconv\'enient. L'id\'ee est de simuler \emph{en m\^eme temps}\/ la loi $\pi$ et la variable al\'eatoire $Y$, \`a l'aide d'une \CM\ sur $\cX$, de probabilit\'e invariante $\pi$. Soit donc $(X_n)_{n\in\N}$ une telle cha\^ine, suppos\'ee irr\'eductible, r\'ecurrente positive, ap\'eriodique, et de loi initiale $\nu$ arbitraire. On lui associe une suite $Y_n=Y(X_n)$ de variables al\'eatoires. Celles-ci peuvent se d\'ecomposer comme suit~: \begin{equation} \label{mcmc18} Y_n = \sum_{x\in\cX} Y(x) \indicator{X_n=x}\;. \end{equation} Consid\'erons les moyennes empiriques \begin{equation} \label{mcmc19} S_n = \frac1n \sum_{m=0}^{n-1} Y_m\;. \end{equation} Le Th\'eor\`eme~\ref{thm:convergence_aperiodique} permet d'\'ecrire \begin{align} \nonumber \lim_{n\to\infty} \bigexpecin{\nu}{S_n} &= \lim_{n\to\infty} \frac1n \biggexpecin{\nu}{\sum_{m=0}^{n-1}Y_m} \\ \nonumber &= \sum_{x\in\cX} Y(x) \lim_{n\to\infty} \frac1n \biggexpecin{\nu}{\sum_{m=0}^{n-1}\indicator{X_m=x}} \\ \nonumber &= \sum_{x\in\cX} Y(x) \pi(x) \\ &= \expecin{\pi}{Y}\;. \label{mcmc20} \end{align} L'esp\'erance de $S_n$ converge bien vers l'esp\'erance cherch\'ee. Pour pouvoir appliquer l'id\'ee de la m\'ethode Monte Carlo, il nous faut plus, \`a savoir la convergence (au moins) en probabilit\'e de $S_n$ vers $\expec{Y}$. On ne peut pas invoquer directement la loi des grands nombres, ni le th\'eor\`eme central limite, car les $Y_n$ ne sont plus ind\'ependants. Mais il s'av\`ere que des r\'esultats analogues restent vrais dans le cas de cha\^ines de Markov. \begin{theorem}[R\'eduction de variance partant de la probabilit\'e invariante] Supposons la cha\^ine\ r\'eversible, et de loi initiale \'egale \`a sa probabilit\'e invariante. Soit $\rho$ le rayon spectral associ\'e \`a la cha\^ine. Alors \begin{equation} \label{mcmc21} \variance(S_n) \leqs \frac1n \Biggpar{\frac{1+\rho}{1-\rho}} \variance^\pi(Y)\;. \end{equation} \end{theorem} \begin{proof} Comme la cha\^ine\ d\'emarre dans la probabilit\'e invariante $\pi$, tous les $Y_i$ ont m\^eme loi $\pi Y^{-1}$, m\^eme s'ils ne sont pas ind\'ependants. Il suit que \begin{align} \nonumber \variance(S_n) &= \frac1{n^2} \Biggbrak{\sum_{m=0}^{n-1}\variance(Y_m) + 2\sum_{0\leqs p<q<n} \cov(Y_p,Y_q)} \\ &= \frac1n \variance^\pi(Y) + \frac2{n^2} \sum_{m=1}^{n-1} (n-m) \cov(Y_0,Y_m)\;, \label{mcmc22} \end{align} en vertu du fait que $(Y_p,Y_q)$ a la m\^eme loi que $(Y_0,Y_{q-p})$. Or si $\vone=\smash{\transpose{(1,1,\dots,1)}}$ on a \begin{align} \nonumber \cov(Y_0,Y_m) &= \Bigexpecin{\pi}{(Y_0-\expecin{\pi}{Y_0}) (Y_m-\expecin{\pi}{Y_m})} \\ \nonumber &= \sum_{x\in\cX} \sum_{y\in\cX} \bigpar{Y(x)-\expecin{\pi}{Y}} \bigpar{Y(y)-\expecin{\pi}{Y}} \underbrace{\probin{\pi}{X_0=x,X_m=y}}_{=\pi(x)(P^m)_{xy}}\\ \nonumber &= \sum_{x\in\cX} \pi(x)\bigpar{Y(x)-\expecin{\pi}{Y}} \bigbrak{P^m(Y-\expecin{\pi}{Y}\vone)}_x\\ \nonumber &= \pscal{Y-\expecin{\pi}{Y}\vone}{P^m(Y-\expecin{\pi}{Y}\vone)}_\pi\\ &\leqs \rho^m \pscal{Y-\expecin{\pi}{Y}\vone}{Y-\expecin{\pi}{Y}\vone}_\pi = \rho^m \variance^\pi(Y)\;. \label{mcmc23} \end{align} Dans l'in\'egalit\'e \`a la derni\`ere ligne, nous avons utilis\'e le fait que $Y-\expecin{\pi}{Y}\vone\in \vone_\perp$ puisque la somme des $\pi(x)(Y(x)-\expecin{\pi}{Y})$ est nulle, et que par cons\'equent ce vecteur se trouve dans le sous-espace compl\'ementaire au vecteur propre $\vone$. Le r\'esultat suit alors en rempla\c cant dans~\eqref{mcmc22}, en majorant $n-m$ par $n$ et en sommant une s\'erie g\'eom\'etrique. \end{proof} Il suit de cette estimation et de l'in\'egalit\'e de Bienaym\'e--Chebychev que pour calculer $\expecin{\pi}{Y}$ avec une pr\'ecision $\delta$ et avec probabilit\'e $1-\eps$, il faut choisir \begin{equation} \label{mcmc24} n \geqs \frac{\variance^\pi(Y)}{\delta^2\eps} \biggpar{\frac{1+\rho}{1-\rho}}\;. \end{equation} En pratique, on ne peut pas faire d\'emarrer la cha\^ine\ exactement avec la probabilit\'e invariante. Ceci conduit \`a une convergence un peu plus lente, mais du m\^eme ordre de grandeur puisque la loi des $Y_n$ converge exponentiellement vite vers $\pi Y^{-1}$. Les r\'esultats sont bien s\^ur meilleurs si on choisit bien la condition initiale, c'est-\`a-dire de mani\`ere \`a ce que la loi des $Y_n$ converge rapidement. \section{Algorithmes de type Metropolis} \label{sec:Metropolis} Nous avons vu comment estimer l'esp\'erance d'une variable al\'eatoire $Y$ \`a l'aide d'une cha\^ine de Markov de probabilit\'e invariante donn\'ee par la loi de $Y$. Pour que cet algorithme soit efficace, il faut encore que l'on puisse trouver facilement, en fonction de cette loi, une matrice de transition donnant la probabilit\'e invariante souhait\'ee. Une m\'ethode pour le faire a \'et\'e d\'evelopp\'ee \`a Los Alamos au d\'ebut des ann\'ees 1950, par Nicholas Metropolis, Arianna Rosenbluth, Marshall Rosenbluth, Augusta Teller et Edward Teller (plus connu pour le d\'eveloppement de la bombe \`a hydrog\`ene)\footnote{Il semble par ailleurs que la contribution principale de Nicholas Metropolis ait \'et\'e de mettre \`a disposition du temps de calcul sur l'ordinateur MANIAC qu'il g\'erait.}. L'algorithme devrait donc \^etre appel\'e Metropolis--Rosenbluth--Rosenbluth--Teller--Teller, mais il est plus connu sous le nom d'algorithme de Metropolis, ou de Metropolis--Hastings, pour une forme plus g\'en\'erale d\'evelopp\'ee par la suite par Wilfred Keith Hastings. Le but de cet algorithme est d'\'echantillonner une \defwd{mesure de Gibbs}, de la forme \begin{equation} \label{metro1} \pi(x) = \frac{\e^{-\beta H(x)}}{Z_\beta}\;, \qquad \text{o\`u } Z_\beta = \sum_{x\in\cX}\e^{-\beta H(x)}\;. \end{equation} Le param\`etre $\beta$ d\'esigne la temp\'erature inverse du syst\`eme, et la fonction $H: \cX\to\R$ associe \`a toute configuration $x\in\cX$ son \'energie. Nous en avons vu un exemple dans la section~\ref{sec:ex_Ising} avec le mod\`ele d'Ising. Il s'agit donc de construire une \CM\ sur $\cX$ admettant $\pi$ comme probabilit\'e invariante. Une mani\`ere simple d'approcher ce probl\`eme est de chercher une \CM\ r\'eversible. On cherche donc une matrice de transition $P$ sur $\cX$ dont les \'el\'ements satisfont \begin{equation} \label{metro4} \pi(x)p_{xy} = \pi(y)p_{yx} \end{equation} pour toute paire $(x,y)\in\cX\times\cX$. Cela revient \`a imposer que \begin{equation} \label{metro5} \frac{p_{xy}}{p_{yx}} = \e^{-\beta\Delta H(x,y)}\;, \end{equation} o\`u \begin{equation} \Delta H(x,y) = H(y) - H(x) \end{equation} est la diff\'erence d'\'energie entre les \'etats $y$ et $x$. On notera que cette condition ne fait pas intervenir la constante de normalisation $Z_\beta$, ce qui est souhaitable, car le calcul de cette constante est aussi co\^uteux que celui de $\expecin{\pi}{Y}$. L'algorithme de Metropolis consiste dans un premier temps \`a d\'efinir un ensemble de transitions permises, c'est-\`a-dire une relation sym\'etrique $\sim$ sur $\cX$ (on supposera toujours que $x\not\sim x$). Une fois la relation $\sim$ fix\'ee, on choisit des probabilit\'es de transition telles que \begin{equation} \label{metro6} p_{xy} = \begin{cases} \myvrule{10pt}{14pt}{0pt} p_{yx} \e^{-\beta\Delta H(x,y)} &\text{si $x\sim y$\;,}\\ \displaystyle 1 - \sum_{z\sim x} p_{xz} &\text{si $x=y$\;,}\\ 0 &\text{autrement\;.} \end{cases} \end{equation} On remarque que la \CM\ est irr\'eductible \`a condition que la relation $\sim$ le soit (deux \'etats quelconques de $\cX$ peuvent \^etre reli\'es par un chemin d'\'etats \'equivalents par $\sim$). De plus, la \CM\ est ap\'eriodique si $p_{xx} > 0$ pour tout $x\in\cX$. Si $\cX$ est fini, la cha\^ine est automatiquement r\'ecurrente positive. Pour satisfaire la condition de r\'eversibilit\'e~\eqref{metro4} lorsque $x\sim y$, une possibilit\'e est de prendre \begin{equation} \label{metro7} p_{xy} = \begin{cases} q &\text{si $H(y)\leqs H(x)$\;,}\\ q \e^{-\beta\Delta H(x,y)} &\text{si $H(y)> H(x)$\;,} \end{cases} \end{equation} o\`u $q\in]0,1]$ est une constante qui contr\^ole la vitesse de l'algorithme. Elle doit \^etre choisie assez petite pour que $p_{xx}$ soit positif. Ce choix revient \`a effectuer la transition avec probabilit\'e $q$ si elle d\'ecro\^\i t l'\'energie, et de ne l'effectuer qu'avec probabilit\'e $q\e^{-\beta\Delta H(x,y)}$ si elle fait cro\^\i tre l'\'energie. Une autre possibilit\'e est de choisir \begin{equation} \label{metro8} p_{xy} = \frac{q}{1+\e^{\beta\Delta H(x,y)}}\;. \end{equation} \begin{remark}[$q$ non constant] Au lieu de choisir un $q$ constant, on peut \'egalement choisir des coefficients $q_{xy}$ d\'ependant de $x$ et $y$, satisfaisant $q_{xy} = q_{yx}$, et avec $\sum_{y\sim x}q_{xy}$ assez petit pour avoir $p_{xx} > 0$. Cela peut permettre, dans certains cas, d'acc\'el\'erer la convergence de l'algorithme. \end{remark} Nous allons illustrer cette m\'ethode dans le cas du mod\`ele d'Ising, mais on voit facilement comment la g\'en\'eraliser \`a d'autres syst\`emes. Rappelons que dans le cas du mod\`ele d'Ising (voir la section~\ref{sec:ex_Ising}), l'univers est donn\'e par $\cX=\set{-1,1}^\Lambda$, o\`u $\Lambda$ est un sous-ensemble (suppos\'e ici de cardinal fini $N$) de $\Z^d$. L'\'energie est donn\'ee par \begin{equation} \label{metro2} H(x) = -\sum_{i,j\in\Lambda\colon\norm{i-j}=1}x_ix_j - h \sum_{i\in\Lambda} x_i\;, \end{equation} o\`u $h\in\R$ est le champ magn\'etique. L'objectif est de calculer l'esp\'erance de la variable aimantation, donn\'ee par \begin{equation} \label{metro3} m(x) = \sum_{i\in\Lambda} x_i\;. \end{equation} Le choix de la relation sym\'etrique $\sim$ sur $\cX$ d\'epend de la physique que l'on souhaite mod\'eliser. Les deux choix les plus courants sont \begin{itemize} \item la \defwd{dynamique de Glauber}\/, qui consiste \`a choisir $x\sim y$ si et seulement si les deux configurations $x$ et $y$ diff\`erent en exactement un point de $\Lambda$; on parle de dynamique de renversement de spin; \item la \defwd{dynamique de Kawasaki}\/, qui consiste \`a choisir $x\sim y$ si et seulement si $y$ est obtenue en intervertissant deux composantes de $x$; on parle de dynamique d'\'echange de spin. Dans ce cas, la cha\^ine n'est pas irr\'eductible sur $\cX$, car elle conserve le nombre total de spins $+1$ et $-1$ : elle est en fait irr\'eductible sur chaque sous-ensemble de configurations \`a nombre fix\'e de spins de chaque signe. \end{itemize} Remarquons que le calcul de la diff\'erence d'\'energie $\Delta H$ est particuli\`erement simple dans le cas de la dynamique de Glauber, car seuls le spin que l'on renverse et ses voisins entrent en compte. Ainsi, si $R_k(x)$ d\'enote la configuration obtenue en renversant le spin num\'ero $k$ de $x$, on aura \begin{equation} \label{metro9} \Delta H(x,R_k(x)) = 2x_k \Biggbrak{\sum_{\;j\colon\norm{j-k}=1}x_j + h}\;, \end{equation} qui est une somme de $2d+1$ termes pour un r\'eseau $\Lambda\subset\Z^d$. Concr\`etement, l'algorithme de Metropolis avec dynamique de Glauber s'impl\'emente de la mani\`ere suivante (avec $q = 1/N$)~: \begin{mdframed}[innerleftmargin=7mm,innertopmargin=10pt,innerbottommargin=10pt] \begin{enumerate} \item {\bf \'Etape d'initialisation~:} \begin{itemize} \item choisir une configuration initiale $X_0$ (de pr\'ef\'erence telle que $\pi(X_0)$ ne soit pas trop petit); \item calculer $m_0=m(X_0)$ (n\'ecessite $N$ calculs); \item calculer $H(X_0)$ (n\'ecessite de l'ordre de $dN$ calculs); \item poser $S = m_0$. \end{itemize} \item {\bf Etape d'it\'eration~:} Pour $n=0, 1, \dots, n_{\max}-1$, \begin{itemize} \item choisir un spin $k$ au hasard uniform\'ement dans $\Lambda$; \item calculer $\Delta H(X_n,y)$, o\`u $y=R_k(X_n)$ est obtenu en renversant le spin choisi; \item si $\Delta H(X_n,y) \leqs 0$, poser $X_{n+1} = y$; \item si $\Delta H(X_n,y) > 0$, soit $B_n$ une variable de Bernoulli de param\`etre $\e^{-\beta\Delta H(X_n,y)}$, qui est ind\'ependante des autres al\'eas; poser $X_{n+1}=y$ si $B_n=1$, et $X_{n+1}=X_n$ si $B_n=0$; \item si $B_n=1$ (on a renvers\'e le spin $k$), alors $m_{n+1}=m_n+2(X_n)_k$, sinon $m_{n+1}=m_n$; \item ajouter $m_{n+1}$ \`a $S$. \end{itemize} \end{enumerate} \end{mdframed} Le quotient $S/(n+1)$ converge alors vers $\expecin{\pi}{m}$, avec une vitesse d\'etermin\'ee par~\eqref{mcmc21}. La seule quantit\'e difficile \`a estimer est le trou spectral $1-\rho$. Nous allons donner une exemple de son estimation \`a l'aide de la m\'ethode des fonctions de Lyapounov dans la section suivante. \section{Mod\`ele d'Ising sur le cercle discret} \label{sec:MCMC_conv} Nous donnons dans cette section quelques exemples d'estimations de vitesse de convergence pour le mod\`ele d'Ising sur $\Lambda = \Z/N\Z$, pour la dynamique de Glauber et une matrice de transition d\'efinie par~\eqref{metro7}. Cela signifie que l'on consid\`ere $N$ spins align\'es, avec conditions aux bords p\'eriodiques~: on identifie $i=0$ avec $i=N$ et $i=-1$ avec $i=N-1$, de mani\`ere que chaque spin $i\in\Lambda$ ait exactement deux voisins $i-1$ et $i+1$. Soit $N_\pm(x)$ le nombre de spins de la configuration $x$ valant $\pm1$. Alors on a \begin{equation} \label{eq:Nplus_Nminus} N_+(x) + N_-(x) = N\;, \qquad N_+(x) - N_-(x) = m(x)\;, \end{equation} ce qui implique $m(x) = N - 2N_-(x)$. Soit par ailleurs \begin{equation} I(x) = \bigabs{\bigsetsuch{i\in\Lambda}{x_{i+1}\neq x_i}} \end{equation} le nombre d'\myquote{interfaces}\ de $x$, c'est-\`a-dire le nombre de fois que la fonction $i\mapsto x_i$ change de signe en faisant le tour du cercle discret. Alors on a \begin{equation} \sum_{i,j\in\Lambda\colon\norm{i-j}=1}x_ix_j = \sum_{i\in\Lambda} x_ix_{i+1} = N - 2I(x)\;. \end{equation} Par cons\'equent, l'\'energie du mod\`ele d'Ising peut \'egalement s'\'ecrire \begin{align} H(x) &= 2I(x) - hm(x) - N\\ &= 2I(x) + 2hN_-(x) - N(1+h)\;. \end{align} Les constantes $-N$ et $-N(1+h)$ n'ont pas d'incidence sur la dynamique de Glauber, qui ne fait intervenir que des diff\'erences d'\'energie entre configurations. D\'enotons par $\boxplus$ la configuration dont tous les spins valent $+1$, et par $\boxminus$ celle dont tous les spins valent $-1$. Alors on a \begin{equation} H(\boxplus) = -N(1+h)\;, \qquad H(\boxminus) = -N(1-h)\;. \end{equation} Dans la suite, on supposera que $N$ est pair, et que $0 < h\leqs 1$. Dans ce cas, la configuration d'\'energie minimale est $\boxplus$, et on v\'erifie qu'on a deux configurations d'\'energie maximale \'egale \`a $N$, donn\'ees par \begin{equation} (1,-1,1,-1,\dots) \qquad \text{et} \qquad (-1,1,-1,1,\dots)\;. \end{equation} Afin de pouvoir appliquer l'appro\-che par fonctions de Lyapounov, nous commen\c cons par d\'eterminer le g\'en\'erateur. \begin{proposition}[G\'en\'erateur pour la dynamique de Glauber] \label{prop:generateur_Glauber} Pour tout $x\in\cX$, notons \begin{align} A_+(x) &= \bigsetsuch{y\in\cX}{y\sim x, H(y) > H(x)}\;, \\ A_0(x) &= \bigsetsuch{y\in\cX}{y\sim x, H(y) = H(x)}\;, \\ A_-(x) &= \bigsetsuch{y\in\cX}{y\sim x, H(y) < H(x)}\;. \end{align} Alors pour toute fonction $V:\cX\to\R$, on a \begin{equation} \label{eq:LV(x)} (\cL V)(x) = q \Biggbrak{\sum_{y\in A_-(x)\cup A_0(x)} \bigpar{V(y)-V(x)} + \sum_{y\in A_+(x)}\bigpar{V(y)-V(x)} \e^{-\beta \Delta H(x,y)}}\;. \end{equation} \end{proposition} \begin{proof} Il suit de~\eqref{metro6} et~\eqref{metro7} que \begin{align} (\cL V)(x) &= \sum_{y\in A_-\cup A_0} p_{xy} V(y) + \sum_{y\in A_+} p_{xy} V(y) + p_{xx} V(x) - V(x) \\ &= q \sum_{y\in A_-\cup A_0} V(y) + q \sum_{y\in A_+} \e^{-\beta\Delta H(x,y)} V(y) - q \Biggbrak{\sum_{y\in A_-\cup A_0} 1 + \sum_{y\in A_+} \e^{-\beta\Delta H(x,y)}}V(x)\;, \end{align} d'o\`u le r\'esultat, en regroupant les termes. \end{proof} Commen\c cons par \'etudier le cas particulier $\beta = 0$, qui correspond \`a une temp\'erature infinie. Dans ce cas, toutes les transitions permises ont la m\^eme probabilit\'e $q = 1/N$, et le syst\`eme effectue une marche al\'eatoire sym\'etrique sur l'hypercube de dimension $N$. La probabilit\'e invariante est simplement la mesure uniforme. Remarquons que $m(X_n)$ satisfait \begin{equation} \label{eq:dynamique_m} m(X_{n+1}) = \begin{cases} m(X_n) + 2 & \text{avec probabilit\'e $\dfrac{N_-(X_n)}{N} = \dfrac12 - \dfrac{m(X_n)}{2N}$\;,} \\[4mm] m(X_n) - 2 & \text{avec probabilit\'e $\dfrac{N_+(X_n)}{N} = \dfrac12 + \dfrac{m(X_n)}{2N}$\;.} \end{cases} \end{equation} Comme ces probabilit\'es de transition ne d\'ependent que de $m(X_n)$, la suite des $M_n = m(X_n)$ est une \CM\ sur $\cM = \set{-N,-N+2,\dots,N-2,N}$. En fait, $(M_n + N)/2$ n'est autre que le mod\`ele d'Ehrenfest \`a $N$ boules, qui admet la loi binomiale de param\`etres $(N,\frac12)$ comme probabilit\'e invariante. Ceci donne l'id\'ee d'utiliser $V(x) = m(x)^2$ comme fonction de Lyapounov. En effet, les valeurs de $m$ proches de $0$ sont plus probables que les valeurs proches de $\pm N$. On obtient facilement la condition de d\'erive g\'eom\'etrique suivante. \begin{proposition}[Condition de d\'erive g\'eom\'etrique pour $V = m^2$] Si $\beta = 0$, la fonction de Lyapounov $V(x) = m(x)^2$ satisfait \begin{equation} \label{eq:LV_m2} (\cL V)(x) = -c V(x) + d\;, \qquad \text{avec $c = \dfrac{4}{N}$ et $d = 4$\;.} \end{equation} \end{proposition} \begin{proof} Par la Proposition~\ref{prop:generateur_Glauber} avec $\beta = 0$, on a \begin{align} (\cL V)(x) &= q \sum_{y\sim x} \Bigbrak{m(y)^2 - m(x)^2} \\ &= q \Bigbrak{N_-(x)\Bigpar{(m(x)+2)^2 - m(x)^2} + N_+(x)\Bigpar{(m(x)-2)^2 - m(x)^2}} \\ &= 4q \Bigbrak{N - V(x)} \end{align} en vertu de~\eqref{eq:Nplus_Nminus}. Le r\'esultat suit du fait que $q=\frac1N$. \end{proof} Pour appliquer la condition de minoration~\eqref{eq:minoration} du Th\'eor\`eme~\ref{thm:convergence}, il nous faut choisir $R > 2d/c = 2N$, donc par exemple $R = 4N$. Ainsi, on aura \begin{equation} \label{eq:K_Ising_beta0} K = \Bigsetsuch{x\in\cX}{m(x)^2 < 4N} = \Bigsetsuch{x\in\cX}{\abs{m(x)} < 2\sqrt{N}}\;. \end{equation} On constate que la condition de minoration ne peut pas \^etre satisfaite, car $p_{xy}$ est non nulle seulement si $x\sim y$, ce qui n'est pas le cas pour tous les $x,y\in K$. Une mani\`ere de r\'esoudre ce probl\`eme est de consid\'erer une puissance de la matrice de transision. \begin{lemma}[Condition de d\'erive pour processus acc\'el\'er\'e] \label{lem:derive_itere} Soit $(X_n)_{n\geqs0}$ une \CM\ dont le g\'en\'erateur satisfait \begin{equation} (\cL V)(x) \leqs -c V(x) + d \qquad \forall x\in\cX \end{equation} pour une fonction de Lyapounov $V$ et des constantes $c > 0$ et $d\geqs 0$. Alors pour tout $T\in\N^*$, le g\'en\'erateur $\cL^T$ du processus acc\'el\'er\'e $(X_{nT})_{n\geqs0}$ satisfait \begin{equation} (\cL^T V)(x) \leqs -\bigbrak{1 - (1-c)^T} V(x) + \bigbrak{1 - (1-c)^T} \frac{d}{c} \qquad \forall x\in\cX\;. \end{equation} \end{lemma} \begin{proof} Pour $x\in\cX$ et $n\in\N$, soit \begin{equation} g_n(x) = (\cP^n V)(x) = \bigexpecin{x}{V(X_n)}\;. \end{equation} Alors on obtient, comme dans la d\'emonstration de la formule de Dynkin, \begin{equation} g_{n+1}(x) \leqs -(1-c) g_n(x) + d\;. \end{equation} En utilisant $g_0(x) = V(x)$ comme condition initiale, on obtient facilement par r\'ecurrence sur $n$ \begin{equation} g_n(x) \leqs -(1-c)^n V(x) + \bigbrak{1 - (1-c)^n} \frac{d}{c} \qquad \forall n\in\N^*\;. \end{equation} Le r\'esultat suit alors du fait que $(\cL^T V)(x) = g_T(x) - V(x)$. \end{proof} On peut simplifier l'\'etude en ne consid\'erant que la \CM\ $(M_n)_{n\geqs0}$. Notons $\hat\cL$ son g\'en\'erateur, et $\hat\pi$ sa probabilit\'e invariante, qui se d\'eduit de la loi binomiale. Alors~\eqref{eq:LV_m2} implique que si $V(m) = m^2$, on a \begin{equation} (\hat\cL V)(m) = -c V(m) + d\;, \qquad \text{avec $c = \dfrac{4}{N}$ et $d = 4$\;.} \end{equation} Cette relation peut aussi \^etre d\'eduite directement de~\eqref{eq:dynamique_m}. On obtient alors le r\'esultat de convergence suivant, en appliquant le Th\'eor\`eme~\ref{thm:convergence} a une puissance convenablement choisie de la matrice de transition de $(M_n)_{n\geqs0}$. \begin{proposition}[Convergence pour fonctions de l'aimantation lorsque $\beta=0$] Il existe des constants $C > 0$ et $\bar\gamma < 1$, ind\'ependantes de $N$, telles que pour toute fonction test $f:\cM\to\R$, on ait \begin{equation} \label{eq:convergence_aimantation} \bigabs{\expecin{m}{f(M_n)} - \hat\pi(f)} \leqs C(1+m^2) \bar\gamma^{n/N} \norm{f - \hat\pi(f)}_{1+m^2}\;. \end{equation} \end{proposition} \begin{proof} Consid\'erons le processus $(M^T_n)_{n\geqs0}$ acc\'el\'er\'e d'un facteur $T\in\N^*$, d\'efini comme $M^{T}_n = M_{Tn}$. En proc\'edant comme dans la d\'emonstration du Lemme~\ref{lem:derive_itere}, on obtient \begin{equation} (\hat\cL^T V)(m) = -c_T V(m) + d_T\;, \qquad \text{avec $c_T = 1-(1-c)^T$ et $d_T = c_T\frac{d}{c}$\;.} \end{equation} On remarque que $2d_T/c_T = 2d/c = 2N$. On peut donc utiliser la m\^eme valeur $4N$ pour $R$, et \begin{equation} K = \Bigsetsuch{m\in\cM}{\abs{m} < 2\sqrt{N}\,}\;. \end{equation} En revanche, on a gagn\'e en ce qui concerne la condition de minoration. En effet, pour $m$ d'ordre $\sqrt{N}$, l'\'evolution de $M_n$ est essentiellement une marche al\'eatoire sym\'etrique (plus pr\'ecis\'ement, les probabilit\'es de transitions sont minor\'ees par celles d'une marche al\'eatoire de param\`etre $\frac12 - \Order{N^{-1/2}}$). La variance d'une telle marche al\'eatoire au temps $T$ est d'ordre $T$. Il suit du th\'eor\`eme central limite (ou du th\'eor\`eme de de Moivre--Laplace) que \begin{equation} \bigprobin{m}{M_T = p} \geqs \frac{a}{\sqrt{T}} \qquad \forall m,p\in K \end{equation} pour une constante $a$ ind\'ependante de $T$. Choisissons alors pour $\nu$ la probabilit\'e uniforme sur $K$. Prenons $T = bN$ pour un $b$ positif. Comme le cardinal de $K$ est d'ordre $\sqrt{N}$, on peut choisir $b$ de telle mani\`ere que \begin{equation} \inf_{m\in K} \bigprobin{m}{M_T = p} \geqs \frac12 \nu(p) \qquad \forall p\in K\;. \end{equation} Comme $\nu(m) = 0$ pour tout $m\not\in K$, la condition de minoration~\eqref{eq:minoration} est satisfaite avec $\alpha = \frac12$. On v\'erifie alors que les choix \begin{equation} \alpha_0 = \frac14\;, \qquad \gamma_0 = 1 - \frac{c_T}4 \end{equation} satisfont~\eqref{eq:cond_gamma0}, avec $\gamma = 1 - c_T$. On trouve ensuite \`a l'aide de~\eqref{eq:beta_gammabar} \begin{equation} \beta = \frac{1}{4Nc_T}\;, \qquad R\beta = \frac{1}{c_T}\;, \qquad \bar\gamma = \max\biggset{\frac14, 1 - \frac{c_T}{4(1+2c_T)}}\;. \end{equation} Comme de plus \begin{equation} \log(1-c_T) = T\log\biggpar{1-\frac{4}{N}} = -\frac{4T}{N} + \biggOrder{\frac{T}{N^2}} = -4b + \biggOrder{\frac{1}{N}}\;, \end{equation} $c_T$ converge vers une limite ind\'ependante de $N$ lorsque $N\to\infty$ (qui vaut $1-\e^{-4b}$). Par le Th\'eor\`eme~\ref{thm:convergence}, l'esp\'erance de $f(M^{T}_n)$ converge vers sa limite $\hat\pi(f)$ exponentiellement vite, avec un taux $\bar\gamma^n$. Le r\'esultat suit en revenant au temps non acc\'el\'er\'e, quitte \`a remplacer $\bar\gamma$ par $\bar\gamma^{1/b}$. \end{proof} La borne~\eqref{eq:convergence_aimantation} montre que pour approcher $\hat\pi(f)$ \`a une distance d'ordre $\delta$, il faut choisir un $n$ d'ordre $N\log(1/\delta)$. Cette convergence est relativement rapide. Bien entendu, comme la probabilit\'e invariante $\hat\pi$ est connue explicitement, on n'a pas besoin d'estimer $\hat\pi(f)$, on peut la calculer directement avec un co\^ut $N$. On notera aussi que ce r\'esultat ne marche que pour des fonctions de l'aimantation. Il n'affirme rien, par exemple, sur des fonctions qui d\'ependraient de $I(x)$. Le second cas particulier que nous allons consid\'erer est celui o\`u $0 < h \leqs 1$ et $\beta$ est assez grand (dans un sens \`a pr\'eciser plus bas), donc o\`u la temp\'erature est assez basse. Pour $h > 0$, l'\'etat d'\'energie minimale est $\boxplus$. Un candidat pour une fonction de Lyapounov est la diff\'erence d'\'energie \begin{equation} \label{eq:Lyapounov_H} V(x) = H(x) - H(\boxplus) = 2I(x) + 2h N_-(x)\;, \end{equation} qui est bien positive ou nulle. Nous commen\c cons par donner une majoration g\'en\'erale de $\cL H$ (comme $\cL c = 0$ pour toute constante $c$, cela fournit \'egalement une majoration de $\cL V$). \begin{lemma}[Majoration de $\cL H$] \label{lem:majo_LH} Supposons $\beta \geqs \frac1{2h}$. Alors pour tout $x\in\cX$, on a \begin{equation} \label{eq:majoration_LH} (\cL H)(x) \leqs -2qh\abs{A_-(x)} + 2qh \e^{-2\beta h}\abs{A_+(x)}\;. \end{equation} \end{lemma} \begin{proof} Il suit de l'expression~\eqref{metro9} de $\Delta H$ que \begin{equation} \Delta H(x,y) \neq 0 \qquad \Rightarrow \qquad \abs{\Delta H(x,y)} \geqs 2h\;. \end{equation} En effet, la somme des $x_j$ est n\'ecessairement un entier pair, et le minimum de $\abs{\Delta H(x,y)}$ est atteint lorsque cette somme vaut $0$. Il suit que la premi\`ere somme dans~\eqref{eq:LV(x)} est major\'ee par $-2h\abs{A_-(x)}$, puisque les termes avec $y\in A_0(x)$ sont nuls. La seconde somme s'\'ecrit \begin{equation} \sum_{y\in A_+(x)}\Delta H(x,y) \e^{-\beta \Delta H(x,y)}\;. \end{equation} Or la fonction $u\mapsto f(u) = u\e^{-\beta u}$ est croissante sur $[0,1/\beta]$ et d\'ecroissante sur $[1/\beta, \infty[$. Comme $\Delta H(x,y)$ est minor\'e par $2h$ dans cette somme, et que $2h \geqs 1/\beta$, celle-ci est inf\'erieure ou \'egale \`a $2h\e^{-2\beta h}\abs{A_+}$. \end{proof} La majoration~\eqref{eq:majoration_LH} exprime le fait que si $\beta$ est assez grand, alors l'\'energie a tendance \`a diminuer, sauf dans le cas particulier o\`u $\abs{A_-} = 0$, c'est-\`a-dire si $x$ n'a pas de configuration voisine d'\'energie inf\'erieure. Cela n'arrive que si $x \in \set{\boxplus, \boxminus}$. \begin{proposition}[Condition de d\'erive g\'eom\'etrique pour $V = H - H(\boxplus)$] Si $\beta \geqs \frac1{2h}$, la fonction de Lyapounov~\eqref{eq:Lyapounov_H} satisfait la condition de d\'erive g\'eom\'etrique \begin{equation} \label{eq:majo_LV1} (\cL V)(x) \leqs -c V(x) + d \qquad \text{avec $c = \dfrac{h}{2N}$ et $d = 2h\e^{-2\beta h} + h^2$\;.} \end{equation} \end{proposition} \begin{proof} Nous allons consid\'erer s\'epar\'ement les cas $x=\boxplus$, $x=\boxminus$, et $x\in\cX\setminus\set{\boxplus,\boxminus}$. \begin{itemize} \item Si $x = \boxplus$, alors $\abs{A_-(x)} = 0$, puisque retourner un spin augmente toujours l'\'energie. Par cons\'equent, \eqref{eq:majoration_LH} implique \begin{equation} (\cL V)(\boxplus) \leqs 2h\e^{-2\beta h}\;, \end{equation} o\`u nous avons utilis\'e $\abs{A_+(\boxplus)} = N$ et $q = 1/N$. Comme $V(\boxplus) = 0$, la borne~\eqref{eq:majo_LV1} est bien v\'erifi\'ee. \item Si $x = \boxminus$, alors on a \begin{equation} (\cL V)(\boxminus) \leqs 2h\e^{-2\beta h}\;, \qquad V(\boxminus) = 2hN\;. \end{equation} Par cons\'equent,~\eqref{eq:majo_LV1} est v\'erifi\'e puisque $d = 2h\e^{-2\beta h} + 2hNc$. \item Pour tous les autres $x$, on a toujours $\abs{A_-(x)} \geqs \frac12 I(x)$. En effet, pour chaque interface, changer le spin $-1$ qui se trouve d'un c\^ot\'e de l'interface en $+1$ diminue l'\'energie de la configuration. Le facteur $\frac12$ vient du fait que ce spin $-1$ peut \^etre compris entre deux interfaces. Ainsi, \begin{align} (\cL V)(x) &\leqs -qh I(x) + 2h\e^{-2\beta h}\\ &= -\frac12 qhV(x) + qh^2N_-(x) + 2h\e^{-2\beta h}\\ &\leqs -\frac12 qhV(x) + qh^2N + 2h\e^{-2\beta h}\;. \end{align} Ceci montre que~\eqref{eq:majo_LV1} est bien v\'erifi\'e, puisque $q = 1/N$. \qed \end{itemize} \renewcommand{\qed}{} \end{proof} Si nous supposons $\beta \geqs \frac{1}{2h}\log(\frac2h)$, alors on peut prendre $d = 2h^2$. Dans la condition de minoration, il faut donc prendre $R > 8hN$, de sorte que $K = \setsuch{x\in\cX}{V(x)<R}$ contient beaucoup d'\'etats (selon la valeur de $h$, il peut m\^eme arriver que $K = \cX$). Il nous faut donc \`a nouveau acc\'elerer le temps afin de pouvoir appliquer le Th\'eor\`eme~\ref{thm:convergence}. Nous n'allons pas donner une analyse d\'etaill\'ee, mais un argument heuristique. Si l'on prend $\nu = \delta_{\boxplus}$, on aura, pour la \CM\ acc\'el\'er\'ee d'un facteur $T$, \begin{equation} \alpha = \inf_{x\in K} \probin{x}{X_T = \boxplus}\;. \end{equation} La question est de savoir comment choisir $T$ pour que $\alpha$ soit d'ordre $1$, disons $\alpha = \frac12$. On s'attend \`a ce que la transition la plus difficile soit celle de $\boxminus$ vers $\boxplus$. La mani\`ere la plus \'economique de faire cette transition est de renverser d'abord un spin quelconque, puis de renverser des spins adjacents, un par un, jusqu'\`a atteindre $\boxplus$ (Figure~\ref{fig:Ising_optimal_transition}). On v\'erifie que seule la premi\`ere transition fait augmenter l'\'energie. Quitte \`a augmenter encore $\beta$, on peut mod\'eliser la transition en n\'egligeant tout renversement de spin non optimal, faisant augmenter l'\'energie plus que n\'ecessaire. On aboutit alors \`a la \CM\ de la Figure~\ref{fig:Ising_transition}. En effet, la probabilit\'e de la premi\`ere transition est de $Nq\e^{-\beta\Delta H(\boxplus,R_k(\boxplus))}$, o\`u $\Delta H(\boxplus,R_k(\boxplus)) = 4-2h$ ne d\'epend pas de $k$, puisqu'on peut retourner n'importe lequel des $N$ spins. Toutes les transitions suivantes on la m\^eme probabilit\'e $2q$, car on peut choisir de quel c\^ot\'e la goutte cro\^it. \begin{figure} \vspace{-3mm} \begin{center} \scalebox{0.6}{ \begin{tikzpicture}[->,>=stealth',shorten >=2pt,shorten <=2pt,auto,node distance=3.0cm,thick, minus spin/.style={ultra thick,rectangle,scale=1,minimum size=1cm, fill=blue!50,draw,font=\sffamily\Large}, plus spin/.style={ultra thick,,rectangle,scale=1,minimum size=1cm, fill=yellow!50,draw,font=\sffamily\Large}] \node[minus spin] at (0,0) {$-$}; \node[minus spin] at (0,1) {$-$}; \node[minus spin] at (0,2) {$-$}; \node[minus spin] at (0,3) {$-$}; \node[minus spin] at (0,4) {$-$}; \node[minus spin] at (0,5) {$-$}; \node[minus spin] at (1.5,0) {$-$}; \node[minus spin] at (1.5,1) {$-$}; \node[plus spin] at (1.5,2) {$+$}; \node[minus spin] at (1.5,3) {$-$}; \node[minus spin] at (1.5,4) {$-$}; \node[minus spin] at (1.5,5) {$-$}; \node[minus spin] at (3,0) {$-$}; \node[minus spin] at (3,1) {$-$}; \node[plus spin] at (3,2) {$+$}; \node[plus spin] at (3,3) {$+$}; \node[minus spin] at (3,4) {$-$}; \node[minus spin] at (3,5) {$-$}; \node[minus spin] at (4.5,0) {$-$}; \node[minus spin] at (4.5,1) {$-$}; \node[plus spin] at (4.5,2) {$+$}; \node[plus spin] at (4.5,3) {$+$}; \node[plus spin] at (4.5,4) {$+$}; \node[minus spin] at (4.5,5) {$-$}; \node[minus spin] at (6,0) {$-$}; \node[plus spin] at (6,1) {$+$}; \node[plus spin] at (6,2) {$+$}; \node[plus spin] at (6,3) {$+$}; \node[plus spin] at (6,4) {$+$}; \node[minus spin] at (6,5) {$-$}; \node[minus spin] at (7.5,0) {$-$}; \node[plus spin] at (7.5,1) {$+$}; \node[plus spin] at (7.5,2) {$+$}; \node[plus spin] at (7.5,3) {$+$}; \node[plus spin] at (7.5,4) {$+$}; \node[plus spin] at (7.5,5) {$+$}; \node[plus spin] at (9,0) {$+$}; \node[plus spin] at (9,1) {$+$}; \node[plus spin] at (9,2) {$+$}; \node[plus spin] at (9,3) {$+$}; \node[plus spin] at (9,4) {$+$}; \node[plus spin] at (9,5) {$+$}; \path[->,>=stealth', semithick] (-0.5,-1) edge (10,-1); \node at (9,-1.5) {\Large temps}; \end{tikzpicture} } \hspace{2mm} \begin{tikzpicture}[-,scale=0.5,auto,node distance=1.0cm, thick,main node/.style={draw,circle,fill=white,minimum size=3pt,inner sep=0pt}, yscale=0.7, xscale=1.8] \path[->,>=stealth'] (-0.5,0) edge (7.2,0) (0,-0.5) edge (0,10) ; \node at (6.7,0.6) {\small $N_+$}; \node at (1.2,8.7) {\small $H(x) - H(\boxplus)$}; \draw[semithick,dashed] (0,7) -- (1,7); \draw[semithick,dashed] (0,3) -- (5,3); \foreach \i in {1,...,6} \draw[semithick] (\i,-0.3) -- (\i,0.3); \draw (0,6) node[main node] {} -- (1,7) node[main node] {} -- (2,6) node[main node] {} -- (3,5) node[main node] {} -- (4,4) node[main node] {} -- (5,3) node[main node] {} -- (6,0) node[main node] {} ; \node[] at (-0.6,6) {\small $2hN$}; \node[] at (-0.7,3) {\small $4+2h$}; \node[] at (-1.3,7) {\small $4+2h(N-1)$}; \node[] at (0,-1.2) {\small $\boxminus$}; \node[] at (6,-1.2) {\small $\boxplus$}; \end{tikzpicture} \end{center} \vspace{-4mm} \caption[]{\`A gauche, exemple d'une transition optimale de l'\'etat $\boxminus$ vers l'\'etat $\boxplus$ par croissance d'une goutte, pour $N=6$. \`A droite, valeur de la diff\'erence d'\'energie $H(x) - H(\boxplus)$ en fonction de $N_+$.} \label{fig:Ising_optimal_transition} \end{figure} Soit alors $f(y) = \expecin{y}{\tau_N}$. Pour $y\in\set{2,\dots,N-1}$, cette fonction satisfait \begin{equation} f(y) = 2q f(y+1) + (1-2q)f(y) + 1\;. \end{equation} Avec la condition initiale $f(y) = 0$, on trouve \begin{equation} \label{eq:f(2)} \expecin{2}{\tau_N} = \frac{N-2}{2q} = \frac{N(N-2)}{2}\;. \end{equation} Par ailleurs, pour $y\in\set{0,1}$ on obtient les \'equations \begin{align} f(0) &= \e^{-\beta\Delta H} f(1) + (1 - \e^{-\beta\Delta H}) f(0) + 1 \\ f(1) &= qf(0) + (1-3q)f(1) + 2qf(2) + 1\;. \end{align} En r\'esolvant ce syst\`eme pour $f(0)$ et $f(1)$ (ce qui revient \`a calculer la matrice fondamentale de la \CM\ absorb\'ee en $2$), on obtient \begin{equation} f(0) = \frac32 \e^{\beta\Delta H} + \frac1{2q} + f(2)\;. \end{equation} En combinant ceci avec~\eqref{eq:f(2)}, on aboutit finalement, dans cette approximation, \`a \begin{equation} \expecin{\boxminus}{\tau_{\boxplus}} \simeq \frac32\e^{2\beta(2-h)} + \frac{N(N-1)}{2}\;. \end{equation} L'in\'egalit\'e de Markov implique alors \begin{equation} \probin{\boxminus}{\tau_{\boxplus} \geqs k } \leqs \frac{\expecin{\boxminus}{\tau_{\boxplus}}}{k}\;, \end{equation} Par cons\'equent, en choisissant $T = 2 \expecin{\boxminus}{\tau_{\boxplus}}$, on aura $\alpha = \frac12$. Soient alors $c_T$ et $d_T$ les constantes donn\'ees par le Lemme~\ref{lem:derive_itere}. Comme \begin{equation} \log(1-c_T) = T\log\biggpar{1 - \frac{h}{2N}} = -\frac{hT}{2N} \biggpar{1 + \biggOrder{\frac{h^2}{N}}}\;, \end{equation} on a $c_T = 1 - \Order{\e^{-hT/(2N)}}$. Un choix possible de param\`etres est \begin{equation} \alpha_0 = \frac14\;, \qquad R = 16hN\;, \qquad \gamma_0 = 1 - \frac{1}{4}c_T\;, \end{equation} ce qui conduit \`a $R\beta = 1/c_T$ et \begin{equation} \bar\gamma = \frac{11}{12} + \Order{\e^{-hT/(2N)}}\;. \end{equation} Le point important est que $1-\bar\gamma$ est minor\'e par une quantit\'e ind\'ependante de $N$. On s'attend donc \`a une convergence de la forme \begin{equation} \label{eq:convergence_Ising2} \bigabs{\expecin{x}{f(X_n)} - \pi(f)} \leqs C(1+V(x)) \bar\gamma^{n/T} \norm{f - \pi(f)}_{1+V} \end{equation} avec $T = N(N-1) + 2\e^{\beta(2-h)}$. Pour atteindre une pr\'ecision $\delta$, il faut choisir $n$ d'ordre $T\log(1/\delta)$. Si $\beta$ n'est pas trop grand, ce temps varie comme $N^2$. Toutefois, si $\e^{\beta(2-h)}$ d\'epasse $N^2$, c'est ce terme qui d\'etermine le temps de convergence. L'algorithme converge donc moins rapidement \`a tr\`es faible temp\'erature, en raison du temps n\'ecessaire \`a renverser le premier spin de la configuration $\boxminus$. \begin{figure} \vspace{-3mm} \begin{center} \begin{tikzpicture}[->,>=stealth',shorten >=2pt,shorten <=2pt,auto,node distance=3.0cm, thick,main node/.style={circle,scale=0.7,minimum size=1.2cm, fill=blue!20,draw,font=\sffamily\Large}] \node[main node] (0) {$0$}; \node[main node] (1) [right of=0] {$1$}; \node[main node] (2) [right of=1] {$2$}; \node[main node] (3) [right of=2] {$3$}; \node[circle,scale=0.7,minimum size=1.2cm] (dots) [right of=3,distance=2cm] {\dots}; \node[main node] (N-1) [right of=dots] {\small $N-1$}; \node[main node] (N) [right of=N-1] {$N$}; \path[every node/.style={font=\sffamily\small}] (0) edge [bend left, above] node {$\e^{-\beta\Delta H}$} (1) (1) edge [bend left, above] node {$2q$} (2) (1) edge [bend left, below] node {$q$} (0) (2) edge [bend left, above] node {$2q$} (3) (3) edge [bend left, above] node {$2q$} (dots) (dots) edge [bend left, above] node {$2q$} (N-1) (N-1) edge [bend left, above] node {$2q$} (N) (0) edge [loop right, above,distance=1.5cm,out=120,in=60] node {$1 - \e^{-\beta\Delta H}$} (0) (1) edge [loop right, above,distance=1.5cm,out=120,in=60] node {$1-3q$} (1) (2) edge [loop right, above,distance=1.5cm,out=120,in=60] node {$1-2q$} (2) (3) edge [loop right, above,distance=1.5cm,out=120,in=60] node {$1-2q$} (3) (N-1) edge [loop right, above,distance=1.5cm,out=120,in=60] node {$1-2q$} (N-1) (N) edge [loop right, above,distance=1.5cm,out=120,in=60] node {$1$} (N) ; \end{tikzpicture} \end{center} \vspace{-2mm} \caption[]{\CCM\ mod\'elisant une transition optimale de l'\'etat $\boxplus$ vers l'\'etat $\boxminus$, par croissance d'une goutte de spins $+1$. La valeur de $\Delta H$ pour la transition entre les \'etats $0$ et $1$ est $\Delta H(\boxplus, R_k(\boxplus)) = 4 - 2h$. Les \'etats sont num\'erot\'es selon le nombre $N_+$ de spins valant $+1$.} \label{fig:Ising_transition} \end{figure} Dans le cas du mod\`ele d'Ising sur $\Lambda\subset\Z^2$, la situation est moins favorable. En effet, en partant de la configuration $\boxminus$, il faut d'abord cr\'eer une goutte de spins $+1$ d'une certaine taille avant que l'\'energie se mette \`a diminuer en approchant $\boxplus$. Dans ce cas, il existe des algorithmes alternatifs, tels que l'algorithme dit de Swendsen--Wang, qui convergent beaucoup mieux. Au lieu de retourner un seul spin \`a la fois, cet algorithme retourne des groupes de spins bien choisis. \part[Cha\^ines de Markov \`a espace continu]{Cha\^ines de Markov\\ \`a espace continu} \label{part:cm_continu} \chapter{D\'efinition et exemples de \CMs\ \`a espace continu} \label{chap:cont_ex} Dans ce chapitre, nous examinons comment on peut \'etendre la th\'eorie des \CMs\ sur un ensemble $\cX$ d\'enombrable \`a des ensembles infinis non d\'enombrables, plus pr\'ecis\'ement des sous-ensembles ouverts de $\R^d$. Une grande partie des concepts du cas discret (\'evolution de la loi de $X_n$, probabilit\'e invariante) peuvent \^etre transpos\'es \`a cette situation de mani\`ere assez directe, essentiellement en \myquote{rempla\c cant les sommes par des int\'egrales}. Il faut \^etre un peu prudent, toutefois, en g\'en\'eralisant les notions de r\'ecurrence et de r\'ecurrence positive. Nous aborderons cette question dans le chapitre suivant. \section{D\'efinitions et notations} \label{sec:cont_def} Soit $\cX\subset\R^d$ un ouvert. Cet ensemble est muni de la \defwd{tribu des bor\'eliens}, qui contient en particulier tous les ouverts de $\cX$. Voici d'abord la g\'en\'eralisation de concept de matrice stochastique \`a cette situation. \begin{definition}[Densit\'e de probabilit\'e, noyau markovien \`a densit\'e] \label{def:noyau_markovien} \begin{itemize} \item Une \defwd{densit\'e de probabilit\'e} $\nu$ sur $\cX$ est une application $\nu:\cX\to\R_+ = [0,\infty[$, continue par morceaux, et satisfaisant \begin{equation} \label{eq:dproba} \int_{\cX} \nu(x)\6x = 1\;. \end{equation} \item Un \defwd{noyau markovien \`a densit\'e} sur $\cX$ est une application $p:\cX\times\cX\to\R_+$, continue par morceaux, satisfaisant \begin{equation} \label{eq:dstoch} \int_{\cX} p(x,y)\6y = 1 \qquad \forall x\in\cX\;. \end{equation} \end{itemize} \end{definition} Dans la suite, nous utiliserons la m\^eme notation pour la mesure de probabilit\'e associ\'ee \`a la densit\'e $\nu$. Cela revient \`a poser \begin{equation} \nu(A) = \int_A \nu(x)\6x \end{equation} pour tout bor\'elien $A\subset\cX$. La g\'en\'eralisation naturelle de la notion de \CM\ est alors la suivante. \begin{definition}[Cha\^ine de Markov sur un ouvert $\cX$ de $\R^d$] \label{def:CM_continu} Soit $\nu$ une densit\'e de probabilit\'e sur $\cX$, et $p$ un noyau markovien \`a densit\'e. Une \defwd{\CM} (homog\`ene en temps) sur $\cX$, de loi initiale $\nu$ et de noyau de transition $p$, est une suite $(X_n)_{n\geqs0}$ de variables al\'eatoires \`a valeurs dans $\cX$, telles que $\prob{X_0 \in A} = \nu(A)$ pour tout bor\'elien $A\subset\cX$, et satisfaisant la \defwd{propri\'et\'e de Markov} \begin{align} \pcond{X_n \in A}{X_0 = x_0, X_1 = x_1, \dots, X_{n-1} = x_{n-1}} &= \pcond{X_n \in A}{X_{n-1} = x_{n-1}} \\ &= \int_A p(x_{n-1},x_n) \6x_n \label{eq:Markov_cont} \end{align} pour tout $n\geqs1$, tout choix de $x_0, \dots, x_{n-1}\in\cX$, et tout bor\'elien $A\subset \cX$. \end{definition} Comme la probabilit\'e qu'une variable al\'eatoire \`a densit\'e prenne une valeur particuli\`ere vaut $0$, il n'est pas imm\'ediatement \'evident que les probabilit\'es conditionnelles dans~\eqref{eq:Markov_cont} sont bien d\'efinies. Il faut en fait les interpr\'eter \`a l'aide de densit\'es conditionnelles. Pour ce faire, soit \begin{equation} \cB_\eps(x_0) = \bigsetsuch{x\in\cX}{\norm{x-x_0} < \eps} \end{equation} la boule ouverte de centre $x_0$ et de rayon $\eps$ (o\`u $\norm{\cdot}$ est la norme Euclidienne). On d\'efinit alors \begin{align} \bigpcond{X_1 \in A}{X_0 = x_0} &= \lim_{\eps\to0} \bigpcond{X_1 \in A}{X_0 \in\cB_\eps(x_0)} \\ &= \lim_{\eps\to0} \frac{\bigprob{X_1\in A, X_0 \in\cB_\eps(x_0)}} {\bigprob{X_0 \in\cB_\eps(x_0)}}\;. \end{align} Si $f(x_0,x_1)$ d\'esigne la densit\'e jointe de $X_0$ et $X_1$, alors on a \begin{align} \bigprob{X_0 \in\cB_\eps(x_0)} &= \int_{\cB_\eps(x_0)} \nu(x)\6x\;, \\ \bigprob{X_1\in A, X_0 \in\cB_\eps(x_0)} &= \int_A \int_{\cB_\eps(x_0)} f(x,x_1) \6x\6x_1\;, \end{align} de sorte que \begin{equation} \bigpcond{X_1 \in A}{X_0 = x_0} = \int_A \lim_{\eps\to0} \frac{\displaystyle\int_{\cB_\eps(x_0)} f(x,x_1) \6x} {\displaystyle\int_{\cB_\eps(x_0)} \nu(x)\6x} \6x_1 = \int_A \frac{f(x_0,x_1)}{\nu(x_0)} \6x_1\;. \end{equation} La derni\`ere \'egalit\'e suit du th\'eor\`eme de la valeur moyenne, qui montre que \begin{equation} \lim_{\eps\to0} \frac{1}{\abs{\cB_\eps(x_0)}} \int_{\cB_\eps(x_0)} \nu(x) \6x = \nu(x_0)\;, \qquad \lim_{\eps\to0} \frac{1}{\abs{\cB_\eps(x_0)}} \int_{\cB_\eps(x_0)} f(x,x_1) \6x = f(x_0,x_1)\;. \end{equation} En comparant avec~\eqref{eq:Markov_cont} pour $n=1$, il vient \begin{equation} \label{eq:f_x0x1} \frac{f(x_0,x_1)}{\nu(x_0)} = p(x_0,x_1)\;. \end{equation} Le noyau markovien $p(x_0,x_1)$ s'interpr\`ete donc comme la \defwd{densit\'e conditionnelle de $X_1$ sachant que $X_0 = x_0$}. Par un raisonnement analogue, pour tout $n\in\N^*$, la densit\'e jointe de $(X_0,\dots,X_n)$ vaut \begin{equation} \label{eq:f_x0xn} f(x_0,\dots,x_n) = \nu(x_0)p(x_0,x_1)\dots p(x_{n-1},x_n)\;. \end{equation} C'est l'analogue continu de la relation~\eqref{eq:proba_traj} pour la probabilit\'e des trajectoires dans le cas discret. La loi de chaque $X_n$ est obtenue en calculant la marginale ad\'equate de la loi jointe. Ainsi, \eqref{eq:f_x0x1} implique \begin{equation} \bigprob{X_1\in A} = \int_A \int_{\cX} f(x_0,x_1) \6x_0 \6x_1 = \int_A \int_{\cX} \nu(x_0)p(x_0,x_1) \6x_0 \6x_1\;. \end{equation} De mani\`ere analogue, \eqref{eq:f_x0xn} montre que \begin{align} \bigprob{X_2\in A} &= \int_A \int_{\cX} \int_{\cX} \nu(x_0)p(x_0,x_1)p(x_1,x_2) \6x_0 \6x_1 \6x_2 \\ &= \int_A \int_{\cX} \nu(x_0)p^2(x_0,x_2)\6x_0\6x_2\;, \end{align} o\`u $p^2$ est un noyau markovien d\'efini par \begin{equation} p^2(x_0,x_2) = \int_{\cX} p(x_0,x_1)p(x_1,x_2)\6x_1\;. \end{equation} Plus g\'en\'eralement, pour tout $n\geqs2$ on a \begin{equation} \prob{X_n\in A} = \int_A \int_{\cX} \nu(x_0)p^n(x_0,x_n)\6x_0\6x_n\;, \end{equation} o\`u $p^n$ est un noyau markovien d\'efini par r\'ecurrence par \begin{equation} \label{eq:Chapman-Kolmogorov} p^n(x_0,x_n) = \int_{\cX} p^{n-1}(x_0,x_{n-1})p(x_{n-1},x_n)\6x_{n-1}\;, \end{equation} avec $p^1 = p$. Cette relation est appel\'ee \defwd{relation de Chapman--Kolmogorov}. Il sera commode d'utiliser les notations suivantes, o\`u $A\subset\cX$ est un bor\'elien, et $f:\cX\to\R$~: \begin{align} \probin{\nu}{X_1\in A} &= (\nu\cP)(A) := \int_A\int_{\cX} \nu(x_0)p(x_0,x_1)\6x_0\6x_1\;, \\ \expecin{x_0}{f(X_1)} &= (\cP f)(x_0) := \int_{\cX} p(x_0,x_1) f(x_1)\6x_1\;, \\ \expecin{\nu}{f(X_1)} &= (\nu\cP)(f) := \nu(\cP f) = \int_{\cX} \int_{\cX} \nu(x_0) p(x_0,x_1) f(x_1)\6x_0\6x_1\;. \label{eq:P_cont} \end{align} La relation de Chapman--Kolmogorov permet de d\'efinir $\cP^n$ pour tout $n\geqs1$ en rempla\c cant $p$ par $p^n$ dans~\eqref{eq:P_cont}. On a \'egalement des concepts de mesure sign\'ee et fonction test tout \`a fait analogues \`a ceux du cas discret. \begin{definition}[Mesures sign\'ees finies \`a densit\'e] \label{def:mesure_cont} Soit $\mu:\cX\to\R$ une application continue par morceaux telle que \begin{equation} \norm{\mu}_1 := \int_{\cX} \abs{\mu(x)}\6x < \infty\;. \end{equation} Elle d\'efinit une \defwd{mesure sign\'ee finie \`a densit\'e}, qui associe \`a tout bor\'elien $A$ le nombre \begin{equation} \mu(A) := \int_A \mu(x)\6x\;. \end{equation} On notera $\cE_1$ l'espace de Banach des mesures sign\'ees finies \`a densit\'e. Si $\mu:\cX\to\R_+$ et $\norm{\mu}_1 = 1$, alors $\mu$ est une mesure de probabilit\'e. \end{definition} \begin{definition}[Fonctions test] \label{def:fct_test_cont} Une \defwd{fonction test} sur $\cX$ est une application $f:\cX\to\R$, continue par morceaux, telle que \begin{equation} \norm{f}_\infty := \sup_{x\in\cX} \abs{f(x)} < \infty\;. \end{equation} On notera $\cE_\infty$ l'espace de Banach des fonctions test. \end{definition} De mani\`ere analogue au cas discret, nous utiliserons la notation \begin{equation} \mu(f) = \int_{\cX} \mu(x)f(x)\6x\;. \end{equation} Cette int\'egrale est bien d\'efinie pour tout $\mu\in\cE_1$ et tout $f\in\cE_\infty$, et on a \begin{equation} \abs{\mu(f)} \leqs \int_{\cX} \abs{\mu(x)} \abs{f(x)} \6x \leqs \norm{\mu}_1 \norm{f}_\infty\;. \end{equation} \begin{remark}[Continuit\'e par morceaux] L'hypoth\`ese de continuit\'e par morceaux n'est pas vraiment n\'ecessaire. Toutes les int\'egrales ci-dessus peuvent \^etre interpr\'et\'ees comme des int\'egrales de Lebesgue, et alors on peut remplacer \myquote{continue par morceaux}\ par \myquote{mesurable}. Toutefois, quand nous \'etudierons les question de r\'ecurrence, l'hypoth\`ese de continuit\'e par morceaux simplifiera nettement la th\'eorie. Cette hypoth\`ese est amplement suffisante pour les applications. \end{remark} \section{Exemples de \CMs\ \`a espace continu} \label{sec:cont_ex} Voici quelques exemples simples de \CMs\ \`a espace d'\'etats continu. \begin{example}[Variables i.i.d.] Soit $\mu$ une densit\'e de probabilit\'e sur $\cX$, et soit \begin{equation} p(x,y) = \mu(y) \qquad \forall x, y\in\cX\;. \end{equation} Il est imm\'ediat de v\'erifier que $p$ est un noyau markovien \`a densit\'e. Pour tout $x\in\cX$ et tout bor\'elien $A\subset\cX$, on a \begin{equation} \bigprobin{x}{X_1\in A} = \int_A \mu(x_1)\6x_1 = \mu(A)\;, \end{equation} ce qui montre que $X_1$ a la densit\'e $\mu$. De plus, on trouve \begin{equation} p^2(x,x_2) = \int_\cX p(x,x_1)p(x_1,x_2)\6x_1 = \int_\cX \mu(x_1)\mu(y) \6x_1 = \mu(y)\;. \end{equation} Plus g\'en\'eralement, on v\'erifie par r\'ecurrence que pour tout $n\geqs1$, on a \begin{equation} p^n(x,x_n) = \mu(x_n) \qquad \forall x, x_n\in\cX\;. \end{equation} Par cons\'equent, les variables al\'eatoires $X_1, X_2, \dots$ ont toutes la m\^eme loi, de densit\'e $\mu$. De plus, on v\'erifie facilement que \begin{equation} \bigprobin{x}{X_1\in A_1,\dots, X_n\in A_n} = \mu(A_1)\dots \mu(A_n) \end{equation} pour tout $n\geqs1$, et tout choix de bor\'eliens $A_1$, \dots, $A_n$. Les variables $X_1, X_2, \dots$ sont donc ind\'ependantes. \end{example} \begin{example}[Marche al\'eatoire \`a pas Gaussiens] Prenons $\cX = \R$. Supposons que $X_0 = 0$ et que \begin{equation} X_{n+1} = X_n + Y_{n+1} \qquad \forall n\in\N\;, \end{equation} o\`u les $Y_n$ sont i.i.d., de loi normale centr\'ee et de variance $\sigma^2$. En d'autres termes, on a \begin{equation} X_n = \sum_{m=1}^n Y_m \qquad \forall n\in\N\;. \end{equation} On dit que $(X_n)_{n\geqs0}$ est une \defwd{marche al\'eatoire \`a pas Gaussiens sur $\R$}. Montrons que c'est une \CM\ sur $\R$. La propri\'et\'e de Markov suit de l'ind\'ependance des $Y_n$. Les probabilit\'es de transition sont donn\'ees par \begin{align} \bigpcond{X_{n+1}\in A}{X_n = x} &= \bigpcond{X_n + Y_{n+1}\in A}{X_n = x} \\ &= \bigpcond{x + Y_{n+1}\in A}{X_n = x} \\ &= \bigprob{Y_{n+1}\in A - x} \\ &= \int_{A-x} \frac{\e^{-y^2/(2\sigma^2)}}{\sqrt{2\pi\sigma^2}} \6y \\ &= \int_{A} \frac{\e^{-(z-x)^2/(2\sigma^2)}}{\sqrt{2\pi\sigma^2}} \6z\;, \end{align} o\`u $A-x = \setsuch{y-x}{y\in A}$. Il suit que le noyau de transition de la \CM\ est donn\'e par \begin{equation} p(x,y) = \frac{\e^{-(y-x)^2/(2\sigma^2)}}{\sqrt{2\pi\sigma^2}}\;. \end{equation} Plus g\'en\'eralement, si les $Y_n$ ont une densit\'e $\mu$, alors $p(x,y) = \mu(y-x)$. \end{example} \begin{example}[Mod\`ele auto-r\'egressif AR(1)] \label{ex:AR1} Les \defwd{mod\`eles auto-r\'egressifs} sont couramment utilis\'es en statistiques, en \'econom\'etrie et en traitement du signal. Le mod\`ele AR(1) en est un cas particulier, o\`u le param\`etre $1$, appel\'e \defwd{ordre}, d\'esigne le temps de m\'emoire. Il est d\'efini par la relation de r\'ecurrence \begin{equation} X_{n+1} = aX_n + Y_{n+1}\;, \end{equation} o\`u $a\in\R$, et les $Y_n$ sont i.i.d., de loi normale centr\'ee et de variance $\sigma^2$. Il suit alors d'un calcul analogue \`a celui de l'exemple pr\'ec\'edent que \begin{align} \bigpcond{X_{n+1}\in A}{X_n = x} &= \bigprob{Y_{n+1}\in A - ax} \\ &= \int_{A} \frac{\e^{-(z-ax)^2/(2\sigma^2)}}{\sqrt{2\pi\sigma^2}} \6z\;. \end{align} Le noyau de transition du mod\`ele AR(1) est donc donn\'e par \begin{equation} p(x,y) = \frac{\e^{-(y-ax)^2/(2\sigma^2)}}{\sqrt{2\pi\sigma^2}}\;. \end{equation} Plus g\'en\'eralement, le mod\`ele autor\'egressif d'ordre $p$, AR($p$), est d\'efini par \begin{equation} X_{n+1} = \sum_{i=1}^p a_iX_{n-i} + Y_{n+1}\;. \end{equation} Si $p\geqs2$, la suite des $X_n$ n'est pas une \CM, puisque la valeur de $X_{n+1}$ d\'epend des valeurs \`a $p$ temps pr\'ec\'edents. Toutefois, les vecteurs $Z_n = (X_n, X_{n+1}, \dots, X_{n+p-1})$ d\'efinissent une \CM\ sur $\R^p$. \end{example} \begin{example}[Applications it\'er\'ees bruit\'ees] Une autre g\'en\'eralisation du mod\`ele AR(1) est donn\'ee par la relation de r\'ecurrence \begin{equation} X_{n+1} = F(X_n) + Y_{n+1}\;, \end{equation} o\`u $F:\R\to\R$, et les $Y_n$ sont \`a nouveau i.i.d., de loi normale centr\'ee et de variance $\sigma^2$. Il s'agit d'une \CM\ de noyau de transition \begin{equation} p(x,y) = \frac{\e^{-(y-F(x))^2/(2\sigma^2)}}{\sqrt{2\pi\sigma^2}}\;. \end{equation} On peut \'evidemment consid\'erer d'autres lois pour les $Y_n$ que la loi normale. Ce genre de mod\`ele appara\^it par exemple en dynamique des populations, ou en \'epid\'emiologie. Sa dynamique d\'epend fortement des propri\'et\'es de $F$ (points fixes, stabilit\'e). \end{example} \chapter{Probabilit\'es invariantes et vitesse de convergence} \label{chap:cont_conv} La principale difficult\'e des \CMs\ \`a espace continu, par rapport aux \CMs\ \`a espace d\'enombrable, est que l'on a $\probin{x}{X_n = y} = 0$ pour tout choix de $x, y\in\cX$ et de $n\in\N^*$. Par cons\'equent, l'esp\'erance du temps de passage en un point diff\'erent du point de d\'epart est en g\'en\'eral infinie. La solution consiste \`a ne pas consid\'erer les temps de premier passage en des points, mais en des ensembles ouverts. C'est ce que nous \'etudierons plus en d\'etail dans la Section~\ref{sec:cont_rec}. Avec cette modification, la th\'eorie des fonctions de Lyapounov s'applique sans grandes modifications, comme nous allons le voir dans la Section~\ref{sec:cont_conv}. \section{Irr\'eductibilit\'e et r\'ecurrence de Harris} \label{sec:cont_rec} Nous consid\'erons dans cette section une \CM\ $(X_n)_{n\geqs0}$ sur un ouvert $\cX$ de $\R^d$, de noyau de transition \`a densit\'e $p$. Le d\'efinition du temps de premier passage est la m\^eme que dans le cas d\'enombrable, mais nous la rappelons n\'eanmoins ici. \begin{definition}[Temps de premier passage] Soit $A\subset\cX$ un bor\'elien. Alors le \defwd{temps de premier passage en $A$} de la \CM\ $(X_n)_{n\geqs0}$ est la variable al\'eatoire \begin{equation} \tau_A = \inf\setsuch{n\geqs1}{X_n \in A} \in\N^*\cup\set{\infty}\;. \end{equation} \end{definition} La d\'efinition de l'irr\'eductibilit\'e est en revanche l\'eg\`erement diff\'erente de celle du cas discret, en raison du fait que les probabilit\'es de transition vers des points sont nulles. \begin{definition}[Irr\'eductibilit\'e d'une \CM\ \`a espace continu] La \CM\ $(X_n)_{n\geqs0}$ est dite \defwd{irr\'eductible} si pour tout $x\in\cX$ et tout ouvert $A\subset\cX$, il existe un $n\in\N^*$ tel que $\probin{x}{X_n\in A} > 0$. De mani\`ere \'equivalente, pour tout $x\in\cX$ et $A\subset\cX$ ouvert, il existe un $n\in\N^*$ tel que $\probin{x}{\tau_A \leqs n} > 0$. \end{definition} Remarquons que si $p(x,y) > 0$ pour tout $x,y\in\cX$, alors la \CM\ est irr\'eductible. C'est le cas pour tous les exemples du chapitre pr\'ec\'edent faisant intervenir des variables Gaussiennes. Nous pouvons maintenant donner les analogues continus des d\'efinitions de r\'ecurrence et de r\'ecurrence positive. \begin{definition}[R\'ecurrence (positive) au sens de Harris] \begin{itemize} \item La \CM\ $(X_n)_{n\geqs0}$ est \defwd{Harris--r\'ecurrente} si \begin{equation} \bigprobin{x}{\tau_A < \infty} = 1 \end{equation} pour tout $x\in\cX$ et tout ouvert $A\subset\cX$. \item La \CM\ $(X_n)_{n\geqs0}$ est \defwd{Harris--r\'ecurrente positive} si de plus \begin{equation} \expecin{x}{\tau_A} < \infty \end{equation} pour tout $x\in\cX$ et tout ouvert $A\subset\cX$. \end{itemize} \end{definition} Remarquons que contrairement au cas discret, la d\'efinition fait intervenir le temps de passage en tout ensemble ouvert $A$. Par cons\'equent, une \CM\ Harris--r\'ecurrente est automatiquement irr\'e\-ducti\-ble, puisque $\bigprobin{x}{\tau_A < \infty} = 1$ implique $\probin{x}{\tau_A \leqs n} > 0$ pour un $n$ fini. L'int\'er\^et principal de cette d\'efinition est li\'e aux mesures et probabilit\'es invariantes, d\'efinies comme suit. \begin{definition}[Mesure et probabilit\'e invariantes] Une mesure $\mu$ sur $\cX$ est \defwd{invariante} si $\mu\cP = \mu$, c'est-\`a-dire si \begin{equation} \int_{\cX} \mu(x) p(x,y) \6x = \mu(y) \qquad \forall y\in\cX\;. \label{def:proba_inv_cont} \end{equation} Si $\mu$ est une mesure de probabilit\'e, alors on dit que c'est une \defwd{probabilit\'e invariante}. \end{definition} \begin{theorem}[R\'ecurrence, mesures invariantes et probabilit\'es invariantes] \label{thm:Harris} Si la \CM\ $(X_n)_{n\geqs0}$ est Harris--r\'ecurrente, alors elle admet une mesure invariante $\mu$. Si elle est de plus Harris--r\'ecurrente positive, alors elle admet une probabilit\'e invariante $\pi$. De plus, $\pi$ est essentiellement unique, c'est-\`a-dire que si $\pi'$ est une autre probabilit\'e invariante, alors $\pi'(A) = \pi(A)$ pour tout ouvert $A\subset\cX$. \end{theorem} Afin de pr\'eparer la d\'emonstration de ce r\'esultat, nous introduisons la notion de processus tu\'e en touchant un sous-ensemble de $\cX$. \begin{definition}[Noyau du processus tu\'e en touchant $B\subset\cX$] Soit $B$ un bor\'elien de $\cX$, $B^c = \cX\setminus B$, et soit $p^\dagger = p^\dagger_B$ la fonction d\'efinie par \begin{equation} p^\dagger(x,y) = p^\dagger_B(x,y) = \begin{cases} p(x,y) & \text{si $y\in B^c$\;,} \\ 0 & \text{si $y\in B$\;.} \end{cases} \label{eq:def_pkilled} \end{equation} On d\'efinit par r\'ecurrence des noyaux $p^\dagger_n$ par $p^\dagger_1 = p^\dagger$ et \begin{equation} p^\dagger_{n+1}(x,y) = \int_{B^c} p^\dagger_n(x,z) p^\dagger(z,y) \6z \qquad \forall n\in\N^*\;. \label{eq:def_pkilledn} \end{equation} \end{definition} Notons que les noyaux $p^\dagger_n$ ne sont pas en g\'en\'eral markoviens, car leur int\'egrale par rapport \`a $y$ est en g\'en\'eral strictement inf\'erieure \`a $1$. On dit que ce sont des noyaux \defwd{sous-markoviens}. Leur int\'er\^et pour nous est le lemme suivant. \begin{lemma}[Processus tu\'e et loi de $\tau_B$] \label{lem:processus_tue} Pour tout $n\geqs1$, tout $x\in\cX$, et tout bor\'elien $A\subset \cX$ tel que $A\cap B = \varnothing$, on a \begin{equation} \bigprobin{x}{X_n\in A, \tau_B > n} = \int_A p^\dagger_n(x,y)\6y\;. \end{equation} \end{lemma} \begin{proof} Cela suit du fait que \begin{align} \bigprobin{x}{X_n\in A, \tau_B > n} &= \bigprobin{x}{X_1\notin B, X_2\notin B, \dots, X_{n-1}\notin B, X_n\in A\setminus B} \\ &= \int_{B^c} \int_{B^c} \dots \int_{B^c} \int_{A\setminus B} p(x,x_1) p(x_1,x_2) \dots p(x_{n-1},x_n) \6x_n \6x_{n-1} \dots \6x_2 \6x_1 \\ &= \int_{\cX} \int_{\cX} \dots \int_{\cX} \int_A p^\dagger(x,x_1) p^\dagger(x_1,x_2) \dots p^\dagger(x_{n-1},x_n) \6x_n \6x_{n-1} \dots \6x_2 \6x_1 \end{align} en vertu de~\eqref{eq:def_pkilled}, et puisque $A\setminus B = A$. Par~\eqref{eq:def_pkilledn} et une r\'ecurrence sur $n$, ceci est bien \'egal \`a l'int\'egrale sur $A$ de $p^\dagger_n$ par rapport \`a sa seconde variable. \end{proof} \begin{remark}[Processus de Markov tu\'e] On peut associer \`a $(X_n)_{n\geqs0}$ un \defwd{processus tu\'e en touchant $B$}, not\'e $(X^\dagger_n)_{n\geqs0}$, de la mani\`ere suivante. On ajoute \`a $\cX$ un \defwd{\'etat cimeti\`ere} $\dagger$, qui est absobant, et on pose \begin{equation} X^\dagger_n = \begin{cases} X_n & \text{si $n<\tau_B$\;,}\\ \dagger & \text {si $n\geqs \tau_B$\;.} \end{cases} \end{equation} Son noyau restreint \`a $B^c$ est alors $p^\dagger$, et on a $\bigprobin{x}{X_n\in A, \tau_B > n} = \bigprobin{x}{X_n^\dagger\in A}$ si $A\cap B = \varnothing$. \end{remark} Un objet important li\'e au processus tu\'e est le noyau de potentiel, qui joue un r\^ole similaire \`a celui de la matrice fondamentale d'une \CM\ absorbante. \begin{definition}[Noyau de potentiel] Soit $B\subset\cX$ un ouvert. Le \defwd{noyau de potentiel} de la \CM\ $(X_n)_{n\geqs0}$ relatif \`a $B$ est l'application qui associe \`a chaque $x\in\cX$ et chaque bor\'elien $A\subset\cX$ le nombre \begin{equation} \label{eq:def_GB} G_B(x,A) = \biggexpecin{x}{\sum_{n=0}^{\tau_B-1}\indicator{X_n\in A}} \in [0,\infty]\;. \end{equation} \end{definition} Le lien entre noyau de potentiel et processus tu\'e est le suivant. \begin{proposition}[Densit\'e du noyau de potentiel] Pour $x\in B$, le noyau de potentiel $G_B(x,\cdot)$ est une mesure (pas n\'ecessairement finie), qui admet sur $B^c$ la densit\'e \begin{equation} \label{eq:densite_GB} g_B(x,y) = \sum_{n=1}^\infty p_n^\dagger(x,y)\;, \end{equation} pour tous les $y$ tels que cette s\'erie converge. \end{proposition} \begin{proof} Notons tout d'abord que $\indicator{X_n\in A_1\cup A_2} \leqs \indicator{X_n\in A_1} + \indicator{X_n\in A_2}$, avec \'egalit\'e si $A_1\cap A_2 = \varnothing$. Ceci montre que $G_B(x,\cdot)$ est une mesure, puisque $G_B(x,A_1 \cup A_2) \leqs G_B(x,A_1) + G_B(x,A_2)$, avec \'egalit\'e si $A_1\cap A_2 = \varnothing$. Soit maintenent $A\subset\cX$ un bor\'elien tel que $A\cap B = \varnothing$. Alors on a \begin{align} \int_A g_B(x,y) \6y &= \sum_{n=1}^\infty \int_A p_n^\dagger(x,y) \6y \\ &= \sum_{n=1}^\infty \bigprobin{x}{X_n\in A, \tau_B > n} \\ &= \biggexpecin{x}{\sum_{n=1}^\infty \indicator{X_n\in A, \tau_B > n}} \\ &= \biggexpecin{x}{\sum_{n=1}^{\tau_B-1} \indicator{X_n\in A}} = G_B(x,A)\;. \label{eq:demo_densite_muB} \end{align} Pour obtenir la derni\`ere ligne, nous avons utilis\'e le fait que $A\cap B = \varnothing$ (et donc $x\notin A$). Ceci montre que $g_B(x,y)$ est bien la densit\'e de $G_B(x,\cdot)$, du moins sur $B^c$. \end{proof} Nous sommes maintenant en mesure de donner une d\'emonstration (au moins partielle) du Th\'eor\`eme~\ref{thm:Harris} (nous admettrons l'unicit\'e essentielle). \begin{proof}[\textit{D\'emonstration du Th\'eor\`eme~\ref{thm:Harris}}] Le d\'emonstration est inspir\'ee de la construction du cas discret, reposant sur les mesures donn\'ees par~\eqref{eq:gamma(y)}. Fixons un $x\in\cX$, et soit $B_\eps = \cB_\eps(x)$ la boule de centre $x$ et de rayon $\eps>0$. Pour tout bor\'elien $A\subset\cX$, nous posons \begin{equation} \label{eq:mu_eps} \mu_\eps(A) = G_{B_\eps}(x,A)\;. \end{equation} Notre but est de montrer que $\mu_\eps$ converge vers une mesure invariante lorsque $\eps\to0$. Commen\c cons par remarquer que \begin{equation} \mu_\eps(B_\eps) = 1\;. \end{equation} En effet, l'hypoth\`ese de r\'ecurrence de Harris implique que $\probin{x}{\tau_{B_\eps} < \infty} = 1$, et par cons\'equent la somme~\eqref{eq:def_GB} a presque s\^urement un nombre fini de termes, dont seul le dernier contribue \`a $\mu_\eps(B_\eps)$. D'autre part, $\mu_\eps$ admet sur $B^c$ la densit\'e donn\'ee par~\eqref{eq:densite_GB}. Nous observons maintenant que pour tout $y\in B_\eps^c$, on a \begin{align} \mu_\eps(y) &= p^\dagger(x,y) + \sum_{n=2}^\infty p_n^\dagger(x,y) \\ &= p(x,y) + \sum_{n=2}^\infty \int_{B_\eps^c} p_{n-1}^\dagger(x,z) p^\dagger(z,y) \6z \\ &= p(x,y) + \int_{B_\eps^c} \sum_{m=1}^\infty p_{m}^\dagger(x,z) p(z,y) \6z \\ &= p(x,y) + \int_{B_\eps^c} \mu_\eps(z) p(z,y) \6z\;. \end{align} Nous avons utilis\'e \`a deux reprises le fait que $p^\dagger(z,y) = p(z,y)$ pour tout $z\in\cX$, puisque $y\in B_\eps^c$. Il suit que \begin{equation} \mu_\eps(y) - \int_{\cX} \mu_\eps(z) p(z,y) \6z = p(x,y) - \int_{B_\eps} \mu_\eps(z) p(z,y) \6z\;. \end{equation} Or, comme $\mu_\eps(B_\eps) = 1$, $\mu_\eps$ est une mesure de probabilit\'e sur $B$. Par cons\'equent, \begin{equation} \inf_{z\in B_\eps} p(z,y) \leqs \int_{B_\eps} \mu_\eps(z) p(z,y) \6z \leqs \sup_{z\in B_\eps} p(z,y)\;. \end{equation} Il suit que pour tout $x$ en lequel $x\mapsto p(x,y)$ est continue, on a \begin{equation} \lim_{\eps\to0} \int_{B_\eps} \mu_\eps(z) p(z,y) \6z = p(x,z)\;. \end{equation} Par cons\'equent, on a pour ces $x$ \begin{equation} \lim_{\eps\to0} \biggbrak{\mu_\eps(y) - \int_{\cX} \mu_\eps(z) p(z,y) \6z} = 0\;. \end{equation} Ceci montre que la limite de $\mu_\eps$ lorsque $\eps\to0$ est invariante pour presque tout $x$ (en tout point de continuit\'e de $p$, mais la valeur de $\mu_\eps$ en des points isol\'es n'influe pas sur les probabilit\'es). Consid\'erons finalement le cas Harris--r\'ecurrent positif. Alors on a \begin{equation} \mu_\eps(\cX) = \biggexpecin{x}{\sum_{n=0}^{\tau_{B_\eps}-1} \indicator{X_n\in\cX}} = \bigexpecin{x}{\tau_{B_\eps}}\;. \end{equation} On peut alors prendre \begin{equation} \pi_\eps(y) = \frac{1}{\bigexpecin{x}{\tau_{B_\eps}}} \mu_\eps(x)\;. \end{equation} C'est la densit\'e d'une mesure de probabilit\'e, qui converge vers une probabilit\'e invariante lorsque $\eps\to0$. \end{proof} \begin{remark}[Hypoth\`eses de r\'ecurrence] \label{rem:hypo_recurrence} Dans la d\'emonstration, nous n'avons pas utilis\'e les hypoth\`eses de r\'ecurrence (positive) de Harris dans toute leur g\'en\'eralit\'e. En fait, nous avons seulement suppos\'e qu'il existe un point particulier $x\in\cX$ et un $\eps_0 > 0$ tels que pour tout $\eps\in]0,\eps_0[$, le temps de passage dans la boule de rayon $\eps$ centr\'ee en $x$, partant de $x$, est presque s\^urement fini, respectivement d'esp\'erance finie. \end{remark} Le r\'esultat suivant permet d'exprimer des esp\'erances sous la probabilit\'e invariante en termes d'excursions vers un ensemble fix\'e $B$. \begin{proposition}[Esp\'erance de fonctions test] \label{prop:Nummelin} Soit $(X_n)_{n\geqs0}$ une \CM\ Harris--r\'ecurrente positive, $\pi$ son unique probabilit\'e invariante, et $f:\cX\to\R_+$. Alors pour tout ouvert $B\subset\cX$, on a \begin{equation} \label{eq:Nummelin} \pi(f) = \int_B \pi(x) \biggexpecin{x}{\sum_{n=0}^{\tau_B-1} f(X_n)} \6x = \int_B \pi(x) \biggexpecin{x}{\sum_{n=1}^{\tau_B} f(X_n)} \6x \;. \end{equation} \end{proposition} \begin{proof} Montrons par r\'ecurrence que pour tout $N\in\N$, on a \begin{equation} \pi(x) = \pi(x)\indicator{x\in B} + \sum_{n=1}^N \int_B \pi(y) p_n^\dagger(y,x)\6y + \int_{\cX} \pi(y) p^\dagger_{N+1}(y,x)\6y\;. \label{eq:rec_pi} \end{equation} L'initialisation suit de la d\'ecomposition $\pi(x) = \pi(x)\indicator{x\in B} + \pi(x)\indicator{x\in B^c}$ et du fait que \begin{equation} \pi(x)\indicator{x\in B^c} = (\pi\cP)(x)\indicator{x\in B^c} = \int_\cX \pi(y)p(y,x)\indicator{x\in B^c} \6y = \int_\cX \pi(y)p^\dagger(y,x) \6y\;. \end{equation} L'h\'er\'edit\'e vient de \begin{align} \int_\cX \pi(y) p^\dagger_{N+1}(y,x)\6y &= \int_\cX \biggbrak{\pi(y)\indicator{y\in B} + \int_\cX \pi(z)p^\dagger(z,y)\6z} p^\dagger_{N+1}(y,x)\6y \\ &= \int_B \pi(y) p_{N+1}^\dagger(y,x)\6y + \int_{\cX} \pi(y) p^\dagger_{N+2}(y,x)\6y\;. \end{align} Faisons alors tendre $N$ vers l'infini dans~\eqref{eq:rec_pi}. Le Lemme~\ref{lem:processus_tue} montre que \begin{equation} \lim_{N\to\infty} \int_A p^\dagger_{N+1}(y,x)\6x = \lim_{N\to\infty} \bigprobin{y}{X_{N+1}\in A, \tau_B > N+1} = 0 \end{equation} pour tout bor\'elien $A$, par r\'ecurrence du processus. Il suit que $p^\dagger_{N+1}(y,x)$ tend vers $0$, d'o\`u \begin{equation} \pi(x) = \pi(x)\indicator{x\in B} + \int_B \pi(y) g_B(y,x) \6y\;, \end{equation} o\`u $g_B$ est la densit\'e~\eqref{eq:densite_GB} du noyau de potentiel $G_B$. En int\'egrant cette relation contre $f$, il vient, en permutant les variables $x$ et $y$, \begin{align} \pi(f) &= \int_B \pi(x)f(x) \6x + \int_{B^c} \int_B \pi(y) g_B(y,x)\6y f(x) \6x \\ &= \int_B \pi(x) \biggbrak{f(x) + \int_{B^c} g_B(x,y) f(y)\6y} \6x\;. \end{align} Or on a \begin{align} f(x) + \int_{B^c} g_B(x,y) f(y)\6y = \bigexpecin{x}{f(X_0)} + \biggexpecin{x}{\sum_{n=1}^{\tau_B-1}f(X_n)}\;. \end{align} Ceci montre la premi\`ere \'egalit\'e dans~\eqref{eq:Nummelin}. La seconde \'egalit\'e vient du fait que le terme $n=0$ de la premi\`ere somme est \'egal au terme $n=\tau_B$ de la seconde. \end{proof} \begin{remark}[Lien entre probabilit\'e invariante et temps de r\'ecurrence moyen] En prenant $f = 1$ dans~\eqref{eq:Nummelin} avec $B = B_\eps = \cB_\eps(x)$, il vient \begin{equation} \int_B \pi(x) \bigexpecin{x}{\tau_B} \6x = 1\;. \end{equation} Il suit de~\eqref{def:proba_inv_cont} que $\pi$ est continue presque partout (en tout point de continuit\'e de $y\mapsto p(x,y)$). En ces points, le th\'eor\`eme de la valeur moyenne implique \begin{equation} \pi(x) = \lim_{\eps\to0} \frac{1}{\abs{\cB_\eps(x)}\bigexpecin{x}{\tau_{B_\eps}}}\;. \end{equation} C'est l'analogue de la relation ~\eqref{eq:piEtau} du cas discret. \end{remark} \begin{example}[Processus auto-r\'egressif AR(1)] Nous avons d\'ej\`a observ\'e que le processus AR(1) \'etait Harris--r\'ecurrent, puisque sa densit\'e de transition est minor\'ee par une constante strictement positive sur tout compact. Comme les $Y_n$ sont Gaussiennes, et que toute somme de Gaussiennes est encore Gaussienne, on s'attend \`a avoir une probabilit\'e invariante Gaussienne. En fait, on a la relation de r\'ecurrence \begin{equation} \variance(X_{n+1}) = a^2 \variance(X_n) + \sigma^2\;. \end{equation} On v\'erifie par r\'ecurrence que \begin{equation} \variance(X_n) = a^{2n}\variance(X_0) + \frac{\sigma^2}{1-a^2}\;. \end{equation} Ainsi, si $\abs{a} < 1$, la loi de $X_n$ converge vers une loi normale centr\'ee de variance $\sigma^2/(1-a^2)$. C'est aussi la probabilit\'e invariante. On notera que dans le cas $a=1$, on obtient la marche al\'eatoire \`a pas Gaussiens. Dans ce cas, la loi de $X_n$ ne converge pas, et la \CM\ n'est pas Harris--r\'ecurrente positive. \end{example} \section{Fonctions de Lyapounov et vitesse de convergence} \label{sec:cont_conv} Dans cette section, nous consid\'erons une \CM\ sur $\cX\subset\R^d$, admettant une densit\'e de transition $p$ continue par morceaux. Par simplicit\'e, nous la supposerons \'egalement irr\'eductible, m\^eme si certains r\'esultats peuvent \^etre \'etendus \`a des situations plus g\'en\'erales. Il s'av\`ere que l'approche par fonctions de Lyapounov \`a l'\'etude de propri\'et\'es de r\'ecurrence et de convergence des lois se transpose assez facilement au cas d'un espace continu. La d\'efinition de fonction de Lyapounov est la m\^eme que dans le cas discret. \begin{definition}[Fonction de Lyapounov] Une \defwd{fonction de Lyapounov} est une fonction $V: \cX\to \R_+ = [0,\infty[$ satisfaisant \begin{equation} \label{eq:gen} V(x) \to +\infty \qquad \text{pour $\norm{x}\to\infty$\;.} \end{equation} \end{definition} La d\'efinition de g\'en\'erateur s'adapte aussi tr\`es facilement au cas continu. \begin{definition}[G\'en\'erateur] Le \defwd{g\'en\'erateur} $\cL$ d'une \CM\ sur un ensemble ouvert $\cX\subset\R^d$ est d\'efini, pour toute fonction $f:\cX\to\R$, par \begin{equation} (\cL f)(x) = (\cP f)(x) - f(x) = \int_{\cX} p(x,y)f(y)\6y - f(x)\;. \end{equation} \end{definition} Les trois r\'esultats suivants, concernant la formule de Dynkin, la croissance sous-exponen\-tielle et la non-explosion, restent inchang\'es par rapport au cas discret, avec essentiellement les m\^emes d\'emonstrations. Nous r\'ep\'etons donc simplement ici leurs \'enonc\'es. \begin{proposition}[Formule de Dynkin] \label{prop:Dynkin_cont} Pour toute fonction de Lyapounov $V$, on a \begin{equation} \label{eq:Dynkin_cont} \bigexpecin{x}{V(X_n)} = V(x) + \biggexpecin{x}{\sum_{m=0}^{n-1} (\cL V)(X_m)}\;, \end{equation} De plus, si $\tau$ est un temps d'arr\^et tel que $\expecin{x}{\tau} < \infty$, alors \begin{equation} \bigexpecin{x}{V(X_\tau)} = V(x) + \biggexpecin{x}{\sum_{m=0}^{\tau-1} (\cL V)(X_m)}\;. \end{equation} \end{proposition} \begin{theorem}[Croissance sous-exponentielle] \label{thm:sous_exp_cont} Supposons qu'il existe une fonction de Lyapounov $V$ et $c > 0$, $d\geqs0$ tels que \begin{equation} (\cL V)(x) \leqs c V(x) + d \qquad \forall x\in\cX\;. \end{equation} Alors on a \begin{equation} \bigexpecin{x}{V(X_n)} \leqs (1+c)^n V(x) + \frac{(1+c)^n-1}{c}d \end{equation} pour tout $n\in\N$ et tout $x\in\cX$. \end{theorem} \begin{theorem}[Non-explosion] \label{thm:non_explosion_cont} Supposons qu'il existe $d \geqs 0$ et un ensemble born\'e $K\subset\cX$ tel que pour tout $x\in\cX$, on ait \begin{equation} (\cL V)(x) \leqs d \indicator{K}(x) = \begin{cases} d & \text{si $x\in K$\;,} \\ 0 & \text{sinon\;.} \end{cases} \end{equation} Alors \begin{equation} \biggprobin{x}{\lim_{n\to\infty} \norm{X_n} = \infty} = 0 \qquad \forall x\in\cX\;. \end{equation} \end{theorem} Le r\'esultat de r\'ecurrence positive et sa d\'emonstration doivent \^etre tr\`es l\'eg\`erement adapt\'es. La principale diff\'erence est que l'ensemble $K$ doit \^etre un ouvert born\'e. On pourrait \'egalement prendre un compact d'int\'erieur non vide, le point important \'etant que $K$ doit \^etre born\'e et contenir un ensemble ouvert, afin de pouvoir appliquer la Harris--r\'ecurrence. \begin{theorem}[R\'ecurrence positive] \label{thm:rec_pos_cont} Soit $f: \cX\to[1,\infty[$ et $V$ une fonction de Lyapounov telle que \begin{equation} (\cL V)(x) \leqs -cf(x) + d\indicator{K}(x) \qquad \forall x\in \cX\;, \end{equation} pour un ouvert born\'e $K\subset \cX$ et des constantes $c>0$ et $d\geqs0$. Supposons de plus qu'il existe $\delta > 0$ tels que \begin{equation} \label{eq:proba_lb_cont} p(x,y) \geqs \delta \qquad \forall x, y\in K\;. \end{equation} Alors la \CM\ est Harris--r\'ecurrente positive, et admet donc une mesure de probabilit\'e invariante $\pi$. De plus, \begin{equation} \pi(f) < \infty\;. \end{equation} \end{theorem} \begin{proof} Nous allons consid\'erer d'abord le passage en $K$, puis celui en $A\subset K$, puis celui en un $A$ g\'en\'eral. \begin{enumerate} \item Fixons $x\in\cX$, et soit $T\in\N^*$. Nous noterons $\tau_K\wedge T = \min\set{\tau_K, T}$. Alors la formule de Dynkin implique \begin{align} 0 \leqs \bigexpecin{x}{V(X_{\tau_K\wedge T})} &= V(x) + \biggexpecin{x}{\sum_{m=0}^{\tau_K\wedge T -1} (\cL V)(X_m)} \\ \label{eq:EVXfm} &\leqs V(x) - c \biggexpecin{x}{\sum_{m=0}^{\tau_K\wedge T -1} f(X_m)} + d \\ &\leqs V(x) - c \bigexpecin{x}{\tau_K\wedge T} + d\;. \end{align} Par cons\'equent, on a \begin{equation} \bigexpecin{x}{\tau_K\wedge T} \leqs \frac{V(x)+d}{c} \end{equation} pour tout $T\in\N^*$. Comme le membre de droite ne d\'epend pas de $T$, on obtient, en faisant tendre $T$ vers l'infini, \begin{equation} \label{eq:EtauK} \bigexpecin{x}{\tau_K} \leqs \frac{V(x)+d}{c} \qquad\forall x\in\cX\;. \end{equation} \item Soit $\tau_{K,n}$ le temps du $n$i\`eme passage de la \CM\ en $K$, qui est d'esp\'erance finie en vertu de~\eqref{eq:EtauK}. Soit $(Y_n)_{n\geqs0}$ la \CM\ trace sur $K$, d\'efinie par $Y_n = X_{\tau_{K,n}}$. Alors, pour tout $x_0\in K$ et tout ouvert $A\subset K$, on a \begin{equation} \bigprobin{x_0}{Y_1 \notin A} = 1 - \bigprobin{x_0}{Y_1 \in A} \leqs 1 - \bigprobin{x_0}{X_1\in A} \leqs 1 - \delta\abs{A} \end{equation} par l'hypoth\`ese~\eqref{eq:proba_lb_cont}. Notons que cette borne est ind\'ependante du $x_0\in K$ choisi. Si $\hat\tau_A = \inf\setsuch{n\geqs1}{Y_n\in A}$ est le temps du premier passage de $Y_n$ en $A$, alors on a pour tout $n\in\N$ \begin{equation} \bigprobin{x_0}{\hat \tau_A \geqs n+1} = \bigprobin{x_0}{\hat \tau_A \geqs n, Y_n\notin A} \leqs \bigpar{1 - \delta\abs{A}} \bigprobin{x_0}{\hat\tau_A \geqs n}\;. \end{equation} Par r\'ecurrence sur $n$, on obtient alors \begin{equation} \bigprobin{x_0}{\hat\tau_A \geqs n} \leqs \bigpar{1 - \delta\abs{A}}^n \qquad \forall n\in\N\;. \end{equation} Il suit que \begin{equation} \bigexpecin{x_0}{\hat \tau_A} = \sum_{n\geqs0} n \bigprobin{x_0}{\hat\tau_A \geqs n} \leqs \sum_{n\geqs0} n \bigpar{1 - \delta\abs{A}}^n < \infty\;. \end{equation} Par cons\'equent, $\bigexpecin{x_0}{\tau_A < \infty}$, en vertu de~\eqref{eq:EtauK} et du fait que $\expecin{x_0}{V(X_1)}$ est born\'e par la formule de Dynkin. Ici, nous pouvons invoquer la Remarque~\ref{rem:hypo_recurrence} pour conclure que la \CM\ admet une probabilit\'e invariante $\pi$. En effet, il suffit d'appliquer le r\'esultat que nous venons d'obtenir aux $A$ donn\'es par des boules de centre $x_0$ et de rayon $\eps$ assez petit. \item Afin de montrer que la \CM\ est Harris--r\'ecurrente positive, il faut encore v\'erifier que $\bigexpecin{x}{\tau_A}$ est fini pour tout $x\in\cX$ et tout ouvert $A\subset\cX$. La majoration~\eqref{eq:EtauK} nous permet de nous limiter aux $x\in K$. On peut alors adapter l'argument du point pr\'ec\'edent. Soit $(Z_n)_{n\geqs0}$ le processus trace sur $K\cup A$. Il suit de l'hypoth\`ese d'irr\'eductibilit\'e que $Z_n$ va visiter $A$ avec probabilit\'e strictement positive au bout d'un temps assez long. Comme $K$ est born\'e, on peut trouver un entier $n_1$ tel que $\probin{x}{\hat\tau_A > n_1}$ soit major\'e par un $p<1$ pour tout $x\in K$. On peut alors proc\'eder comme au point pr\'ec\'edent pour montrer que $\bigexpecin{x}{\tau_A}$ est fini. \item Afin de majorer $\pi(f)$, nous utilisons le Lemme~\ref{lem:majo_LH} avec $B = K$ et~\eqref{eq:EVXfm} pour obtenir \begin{equation} \pi(f) = \int_K \pi(x) \biggexpecin{x}{\sum_{n=0}^{\tau_K-1} f(X_n)\6x} \leqs \pi(K) \sup_{x\in K}\frac{V(x)+d}{c}\;, \end{equation} ce qui est fini puisque $K$ est born\'e. \qed \end{enumerate} \renewcommand{\qed}{} \end{proof} Afin de formuler un r\'esultat de convergence, nous travaillerons \`a nouveau avec des normes et des distances \`a poids. Les d\'efinitions suivantes sont des adaptations naturelles de celles du cas discret. \begin{definition}[Normes et distances \`a poids] \begin{itemize} \item Un \defwd{poids} sur $\cX$ est une application $W:\cX\to[1,\infty[$. \item La norme \`a poids d'une fonction test $f$ est d\'efinie par \begin{equation} \norm{f}_W = \sup_{x\in\cX} \frac{\abs{f(x)}}{W(x)}\;. \end{equation} On note $\cE_\infty^W$ l'espace de Banach des fonctions test de norme $\norm{f}_W$ finie. \item Pour deux mesures sign\'ees finies \`a densit\'e, on d\'efinit \begin{align} \rho_W(\mu,\nu) &= \sup_{f\colon\norm{f}_W\leqs 1} \int_\cX f(x) \abs{\mu(x)-\nu(x)} \6x \\ &= \sup_{f\colon\norm{f}_W\neq 0} \frac{1}{\norm{f}_W} \int_\cX f(x) \abs{\mu(x)-\nu(x)} \6x \\ &= \int_x W(x) \abs{\mu(x)-\nu(x)} \6x\;. \end{align} \end{itemize} \end{definition} L'analogue continu du Th\'eor\`eme~\ref{thm:convergence} prend alors la forme suivante. Sa d\'emonstration est la m\^eme que celle du cas discret. \begin{theorem}[Ergodicit\'e g\'eom\'etrique] \label{thm:convergence_cont} Supposons que les deux conditions suivantes soient satisfaites. \begin{enumerate} \item \textbf{Condition de d\'erive g\'eom\'etrique\,:} Il existe $d\geqs0$, $c>0$ et une fonction de Lyapounov $V$ tels que \begin{equation} \label{eq:derive_geom_cont} (\cL V)(x) \leqs -c V(x) + d \qquad \forall x\in\cX\;. \end{equation} \item \textbf{Condition de minoration\,:} Pour un $R > 2d/c$, soit $K = \setsuch{x\in\cX}{V(x) < R}$. Alors il existe $\alpha\in]0,1[$ et une mesure de probabilit\'e $\nu$ telle que \begin{equation} \label{eq:minoration_cont} \inf_{x\in K} p(x,y) = \inf_{x\in K} \bigprobin{x}{X_1 = y} \geqs \alpha \nu(y) \qquad \forall y\in\cX\;. \end{equation} \end{enumerate} \noindent Alors il existe des constantes $M>0$ et $\bar\gamma < 1$ telles que \begin{equation} \label{eq:borne_cv_expec_cont} \norm{\expecin{\cdot}{f(X_n)} - \pi(f)}_{1+V} \leqs M\bar\gamma^n \norm{f - \pi(f)}_{1+V} \end{equation} pour toute fonction test $f\in\cE_\infty^{1+V}$. \end{theorem} Les constantes $\bar\gamma$ et $M$ sont \`a nouveau d\'etermin\'ees par les relations~\eqref{eq:cond_gamma0}, \eqref{eq:beta_gammabar} et \eqref{eq:M}. \begin{example}[Processus auto-r\'egressif AR(1)] Nous avons vu dans l'Exemple~\ref{ex:AR1} que le noyau de transition du mod\`ele AR(1) \'etait donn\'e par \begin{equation} p(x,y) = \frac{\e^{-(y-ax)^2/(2\sigma^2)}}{\sqrt{2\pi\sigma^2}}\;. \end{equation} Prenons comme fonction de Lyapounov $V(x) = x^2$. Alors on a \begin{align} (\cL V)(x) &= \frac{1}{\sqrt{2\pi\sigma^2}} \int_{-\infty}^{\infty} y^2 \e^{-(y-ax)^2/(2\sigma^2)} \6y - V(x) \\ &= \frac{1}{\sqrt{2\pi}} \int_{-\infty}^{\infty} (ax+\sigma z)^2 \e^{-z^2/2} \6z - x^2 \\ &= -(1-a^2)x^2 + \sigma^2\;. \end{align} Pour obtenir la deuxi\`eme ligne, nous avons utilis\'e le changement de variables $y=ax+\sigma z$. La derni\`ere ligne suit des propri\`et\'es de la densit\'e d'une loi gaussienne standard. La condition de d\'erive g\'eom\'etrique~\eqref{eq:derive_geom_cont} est donc v\'erifi\'ee avec $c = 1 - a^2$ et $d = \sigma^2$. Pour v\'erifier la condition de minoration, nous devons choisir un $R > 2d/c = 2\sigma^2/(1-a^2)$. Alors nous avons $K=[-\sqrt{R},\sqrt{R}]$ et \begin{equation} \inf_{x\in K} p(x,y) = \inf_{\abs{x}<\sqrt{R}} \frac{\e^{-(y-ax)^2/(2\sigma^2)}}{\sqrt{2\pi\sigma^2}} = \frac{\e^{-(\abs{y}+a\sqrt{R})^2/(2\sigma^2)}}{\sqrt{2\pi\sigma^2}}\;, \end{equation} l'infimum \'etant atteint pour $x=\pm \sqrt{R}$, selon le signe de $y$. Ceci sugg\`ere de prendre pour $\nu$ la mesure de densit\'e \begin{equation} \nu(y) = \frac{\e^{-(\abs{y}+a\sqrt{R})^2/(2\sigma^2)}}{\cN \sqrt{2\pi\sigma^2}} \indicator{\abs{y}\leqs \sqrt{R}}\;, \end{equation} o\`u \begin{equation} \cN = \frac{1}{\sqrt{2\pi\sigma^2}}\int_{-\sqrt{R}}^{\sqrt{R}} \e^{-(\abs{y}+a\sqrt{R})^2/(2\sigma^2)} \6y = \frac{1}{\sqrt{2\pi}} \int_{-\sqrt{R}/\sigma}^{\sqrt{R}/\sigma} \e^{-(\abs{z}+a\sqrt{R}/\sigma)^2/2} \6y \end{equation} est la constante de normalisation assurant que $\nu$ soit une mesure de probabilit\'e (nous avons pos\'e $y=\sigma z$ pour obtenir la seconde in\'egalit\'e). En effet, la condition de minoration~\eqref{eq:minoration_cont} est alors satisfaite en prenant \begin{equation} \alpha = \cN\;. \end{equation} En choisissant $R=4\sigma^2/(1-a^2)$, on obtient un $\alpha$ ind\'ependant de $\sigma$. On v\'erifie alors que cela donne un taux de convergence $1-\bar\gamma$ strictement positif si $a^2 < 1$, mais qui tend vers $0$ lorsque $a^2$ tend vers $1$. \end{example} \section{Exercices} \label{sec:Lyapounov_cont_exo} \begin{exercise} On consid\`ere la \CM\ sur $\R$ donn\'ee par \[ X_{n+1} = aX_n + Y_{n+1} \] avec $a\in[0,\infty[$, les $Y_n$ \'etant ind\'ependantes, identiquement distribu\'ees, de loi uniforme sur $[-\delta,\delta]$ pour un $\delta > 0$. \begin{enumerate} \item Donner les probabilit\'es de transition $p(x,y)$ de la cha\^ine. \item Soit la fonction de Lyapounov $V(x) = x^2$. Calculer $(\cL V)(x)$. \item Pour quelles valeurs de $a$ la cha\^ine est-elle \`a croissance sous-exponentielle~? \item \`A l'aide de la formule de Dynkin, calculer la variance de $X_n$ lorsque $a=1$. \item Pour quelles valeurs de $a$ la cha\^ine satisfait-elle une condition de d\'erive g\'eom\'etrique~? Quels en sont les param\`etres~? \item Lorsque la condition de d\'erive g\'eom\'etrique est satisfaite, trouver $\alpha\in]0,1[$, une mesure de probabilit\'e $\nu$, et une condition sur $p$ tels que la condition de minoration soit satisfaite. Que peut-on en d\'eduire~? \end{enumerate} \end{exercise} \begin{exercise} On consid\`ere la cha\^ine de Markov sur $\R$ donn\'ee par \[ X_{n+1} = aX_n + Y_{n+1} \] avec $a\in[0,\infty[$, les $Y_n$ \'etant ind\'ependantes, identiquement distribu\'ees, de loi de Cauchy de param\`etre $c>0$. Soit la fonction de Lyapounov $V(x) = \abs{x}^\beta$. Pour quelles valeurs de $\beta>0$ la quantit\'e $(\cL V)(x)$ est-elle finie~? \end{exercise} \begin{exercise} On consid\`ere la cha\^ine de Markov sur $\R_+ = [0,\infty[$ donn\'ee par \[ X_{n+1} = aX_n + Y_{n+1} \] avec $a\in[0,\infty[$, les $Y_n$ \'etant ind\'ependantes, identiquement distribu\'ees, de loi exponentielle de param\`etre $\lambda>0$. \begin{enumerate} \item Donner les probabilit\'es de transition $p(x,y)$ de la cha\^ine. \item Calculer \[ \int_0^\infty x^k\e^{-\lambda x}\6x \] pour $k\in\set{0,1}$. \item Soit la fonction de Lyapounov $V(x) = x$. Calculer $(\cL V)(x)$. \item Pour quelles valeurs de $a$ la cha\^ine est-elle \`a croissance sous-exponentielle~? \item \`A l'aide de la formule de Dynkin, calculer l'esp\'erance de $X_n$ lorsque $a=1$. Que se passe-t-il lorsque $n\to\infty$~? \item Pour quelles valeurs de $a$ la cha\^ine satisfait-elle une condition de d\'erive g\'eom\'etrique~? Quels en sont les param\`etres~? \item Lorsque la condition de d\'erive g\'eom\'etrique est satisfaite, trouver $\alpha\in]0,1[$ et une mesure $\nu$ tels que la condition de minoration soit satisfaite. Que peut-on en d\'eduire~? \end{enumerate} \end{exercise} \bibliographystyle{plain} \bibliographystyle{alpha} \bibliography{KESM} \nocite{Nummelin84,Durrett1} \vfill \bigskip\bigskip\noindent {\small Nils Berglund \\ Institut Denis Poisson (IDP) \\ Universite d'Orleans, Universite de Tours, CNRS -- UMR 7013 \\ B\^atiment de Mathematiques, B.P. 6759\\ 45067~Orleans Cedex 2, France \\ {\it E-mail address: } {\tt [email protected]} \\ {\tt https://www.idpoisson.fr/berglund} \end{document}
2412.07394v2
http://arxiv.org/abs/2412.07394v2
Numerical approximations for a hyperbolic integrodifferential equation with a non-positive variable-sign kernel and nonlinear-nonlocal damping
\documentclass[11pt]{article} \usepackage[margin=2.4cm]{geometry} \usepackage{graphicx} \graphicspath{{./images}} \usepackage{amsmath,amsthm,latexsym,amsfonts,amssymb,mathrsfs} \usepackage[usenames]{color} \usepackage{tikz} \usetikzlibrary{math} \usepackage{cancel,soul,ulem} \usepackage{float} \usepackage{verbatim} \usepackage{booktabs} \newcommand{\pe}{\psi} \def\d{\delta} \def\dsp{\displaystyle} \def\e{{\epsilon}} \def\eb{\bar{\eta}} \def\enorm#1{\|#1\|_2} \def\Fp{F^\prime} shpack{{FISHPACK}} \def\fortran{{FORTRAN}} \def\gmres{{GMRES}} \def\gmresm{{\rm GMRES($m$)}} \def\Kc{{\cal K}} \def\norm#1{\|#1\|} \def\wb{{\bar w}} \def\zb{{\bar z}} \def\bfE{\mbox{\boldmath$E$}} \def\bfG{\mbox{\boldmath$G$}} \numberwithin{equation}{section} \newcommand{\ud}{\mathrm{d}} \newcommand{\diam}{\mathrm{diam}} \newtheorem{theorem}{Theorem}[section] \newtheorem{prop}[theorem]{Proposition}\newtheorem{lemma}[theorem]{Lemma} \newtheorem{corollary}[theorem]{Corollary} \newtheorem{proposition}[theorem]{Proposition} \newtheorem{assumption}{Assumption} \newtheorem{definition}{Definition}[section] \newtheorem{remark}{Remark}[section] \title{Numerical approximations for a hyperbolic integrodifferential equation with a non-positive variable-sign kernel and nonlinear-nonlocal damping} \author{ Wenlin Qiu\thanks{School of Mathematics, Shandong University, Jinan 250100, China. (Email: [email protected])} \and Xiangcheng Zheng\thanks{School of Mathematics, Shandong University, Jinan 250100, China. (Email: [email protected])} \and Kassem Mustapha\thanks{Corresponding author. School of Mathematics and Statistics, University of New South Wales, Sydney, Australia. (Email: [email protected])}} \begin{document} \maketitle \begin{abstract} This work considers the Galerkin approximation and analysis for a hyperbolic integrodifferential equation, where the non-positive variable-sign kernel and nonlinear-nonlocal damping with both the weak and viscous damping effects are involved. We derive the long-time stability of the solution and its finite-time uniqueness. For the semi-discrete-in-space Galerkin scheme, we derive the long-time stability of the semi-discrete numerical solution and its finite-time error estimate by technical splitting of intricate terms. Then we further apply the centering difference method and the interpolating quadrature to construct a fully discrete Galerkin scheme and prove the long-time stability of the numerical solution and its finite-time error estimate by designing a new semi-norm. Numerical experiments are performed to verify the theoretical findings. \end{abstract} \section{Introduction} This work considers the numerical approximation and analysis of a hyperbolic integrodifferential equation \cite{Cannarsa1,Cannarsa} which models viscoelastic systems with memory in various fields, see e.g. \cite{Alabau-Boussouira,Appleby,Cannarsa1,Georgiev,Lebon,Renardy} \begin{equation}\label{eq1.1} \begin{split} u''(x, t) + q(t)u'(x, t) -\Delta u(x,t) + \int_0^{t}\beta(t-s)\Delta u(x,s)ds = f(x, t), \end{split} \end{equation} for $ (x,t)\in\Omega \times (0,T]$, subject to homogeneous Dirichlet boundary condition $u =0$ on $\partial\Omega \times (0,T]$. Here, $u'=\frac{\partial u}{\partial t}$, $u''=\frac{\partial^2 u}{\partial t^2}$, $T$ may be finite or infinite that will be specified in different cases, $\Omega$ is a bounded domain in $\mathbb{R}^d$ ($d=1,2,3$) with smooth boundary $\partial\Omega$, $\Delta$ is the Laplace operator, and $f$ is the forcing function. This model involves the nonlinear-nonlocal damping term with weak and viscous damping coefficients $\mu_1$ and $\mu_2$, respectively (cf. \cite[Equation 6.2]{Cannarsa} and \cite{Emm}) \begin{equation}\label{eq1.2} \begin{split} q(t) = G\left( \int_{\Omega}\mu_1| u(x,t)|^2+\mu_2|\nabla u(x,t)|^2dx \right), \quad \mu_1, \mu_2 \geq 0, \quad \mu_1^2+ \mu_2^2 > 0, \end{split} \end{equation} and initial conditions \begin{equation}\label{eq1.3} u(x,0)=u_0(x), \quad u'(x,0)=u_1(x), \quad x\in \Omega, \end{equation} where $\nabla$ is the gradient operator, the function $G:\mathbb{R}^+\rightarrow \mathbb{R}^+$, and $u_0$ and $u_1$ are given initial values. Moreover, the variable-sign convolution kernel $\beta\in L^1(0,\infty)$ is selected as \cite{Cannarsa1,Cannarsa}: \begin{equation}\label{eq1.5} \begin{split} & \beta(t) = \frac{e^{-\sigma t}t^{\alpha-1}\cos(\gamma t)}{\Gamma(\alpha)}, \quad \text{satisfying the following standard conditions} \\ & \text{(i)} \; \alpha = 1, \quad \sigma> 1, \quad 0\leq \gamma \leq \sigma, \\ & \text{(ii)} \; \alpha = 1/2, \quad \sigma> 1, \quad 0\leq \gamma \leq \sqrt{3}\sigma. \end{split} \end{equation} See \cite{Cannarsa1,Cannarsa,Messaoudi1,Messaoudi2} for the well-posedness of certain nonlinear hyperbolic integrodifferential equations. For the numerical aspect, Yanik and Fairweather \cite{Yanik} developed a Galerkin finite element method for a variant of (\ref{eq1.1}) where the nonlinearity appears in the convolution term. Cannon et al. \cite{Cannon} established the stability and $L^2$-error estimate of the Galerkin approximation for the linear version of (\ref{eq1.1}) with smooth kernel. Allegretto and Lin \cite{Allegretto} extended the results to the linear case with the positive-type or monotonic kernel. Fairweather \cite{Fairweather} formulated the orthogonal spline collocation methods and derived error estimates. Larsson and Saedpanah developed the continuous Galerkin method for the linear case of (\ref{eq1.1}) with weakly singular kernels \cite{Larsson,Saedpanah}. Karaa et al. \cite{Karaa} proved a priori hp-estimate under the case of smooth kernels. Xu \cite{Xu2} deduced the long-time decay properties of the numerical solutions with tempered weakly singular kernels. Then, Xu \cite{Xu4} proved the long-time $L^{\infty}$ error estimate for a homogeneous viscoelastic rod. Recently, Baker et al. considered the convolution quadrature method for the wave equation with linear or nonlinear time-fractional damping terms \cite{Baker1,Baker2}. Despite the significant progress on the linear case of (\ref{eq1.1}), there are rare studies considering the hyperbolic integrodifferential equation with variable-sign kernel and nonlinear-nonlocal damping coefficient. Due to the variability of the sign and the non-positivity of the kernel, many commonly-used quadrature rules for convolution with a completely monotonic kernel such as the rectangle rule are not applicable. Furthermore, the nonlinear-nonlocal damping makes the numerical analysis more intricate, especially in the case of long-time estimates that have significant applications such as the long-time performance tests of viscoelastic materials. This work accommodates these issues to propose and analyze a formally second-order accurate numerical approximation for problem \eqref{eq1.1}-\eqref{eq1.5}. Specifically, the main contributions are enumerated as follows: \begin{itemize} \item We introduce a transformed kernel to convert the original model with a non-positive kernel into another hyperbolic integrodifferential equation with a positive type kernel, and overcome the difficulties caused by the variable-sign kernel and the nonlinear-nonlocal damping to prove the long-time stability and finite-time uniqueness of the solutions of problem \eqref{eq1.1}-\eqref{eq1.5}, the methods of which could assist the subsequent analysis of numerical solutions. \item We construct a spatial semi-discrete Galerkin scheme for solving \eqref{eq1.1}-\eqref{eq1.5} and derive the long-time stability of numerical solutions and the finite-time uniqueness and error estimate. A key ingredient of the proof lies in estimating the difference between nonlinear-nonlocal terms in (\ref{eq5.2}). Instead of splitting this difference into several pieces by introducing intermediate terms, which is difficult due to the nonlinearity and nonlocality and may deteriorate the accuracy, we introduce a quantity $\varphi_\vartheta$ to rewrite this difference into a more integrating form. By this means, a novel splitting of the difference (cf. (\ref{eq5.3})--(\ref{eq5.4})) appears to facilitate the analysis, cf. the estimates among (\ref{eq5.5})--(\ref{eq5.14}). \item We employ the centering difference method and the interpolating quadrature to construct a fully discrete Galerkin scheme. We deduce the long-time stability of numerical solutions and the finite-time uniqueness and error estimate. Apart from the techniques in studying the semi-discrete scheme, we define a new semi-norm by (\ref{eq6.3}) to clarify the structure of the numerical scheme and the error equation to support the error estimate. \end{itemize} The rest of the work is organized as follows: Section \ref{sec2} addresses some well-posedness issues of the model. Section \ref{sec3} gives long-time stability estimates and finite-time error estimates for the spatial semi-discrete scheme. The fully-discrete scheme is constructed and analyzed in Section \ref{sec4}. Section \ref{sec5} provides numerical experiments to verify the theoretical results, and we address concluding remarks in the last section. \section{Well-posedness issue} \label{sec2} Let $L^2(\Omega)$, $H^j(\Omega)$ ($j \ge 1$) and $H^1_0(\Omega)$ denote usual Sobolev spaces on $\Omega$. The notation $(\cdot, \cdot)$ indicates the $L^2(\Omega)$-inner product on $\Omega$ and $\|\cdot\|$ is the associated norm, and $\|\cdot\|_j$ denotes the norm in $H^j(\Omega)$ for $j\ge 1.$ For $T>0$, the $L^{1}(0,T; L^2(\Omega))$ indicates the space of measurable functions $g: [0,T] \rightarrow L^2(\Omega)$ satisfying $\int_{0}^{T}\|g(t)\|dt< \infty$. For the normed linear space $\mathcal{X}$ endowed with the norm $\|\cdot\|_{\mathcal{X}}$, the $ {\mathcal C}^{j}([0, T]; \mathcal{X})$ ($j=0,1$) represents the space of continuous or continuously differentiable functions from $[0, T]$ to $\mathcal{X}$, respectively. In particular, we set $ {\mathcal C}([0,T]; \mathcal{X})= {\mathcal C}^{0}([0,T]; \mathcal{X})$. Throughout the work, we use $C$ to denote a generic positive constant independent of the mesh sizes and may differ at different occurrences. To prove the main results, the following lemma for the variable-sign kernels in \eqref{eq1.5} is critical (the proof could be found in \cite{Cannarsa1} for the case (i) in \eqref{eq1.5} and in \cite{Zhao} for the case (ii) in \eqref{eq1.5}). \begin{lemma}\label{lemma1.1} The kernel $K(t)=\int_t^{\infty}\beta(s)ds$ is of positive type with $K(\infty)=0$ and $K(0):=K_0<1$. \end{lemma} We impose the following assumptions on the function $G$ in \eqref{eq1.2}: for $z\ge 0,$ ($\mathbf{A1}$) $G(z)\geq g_0$ for some positive constant $g_0$, \par ($\mathbf{A2}$) $G$ is continuously differentiable with $0\leq G'(z)\leq \mathcal{L}$, where $\mathcal{L}$ is the Lipschitz constant. Examples of $G$ include $G(z)=1+z$ or $G(z)=\sqrt{1+z}$. Note that the assumption $(\mathbf{A2})$ implies that for any $0\leq z\leq k$ with $k>0$, $G(z)=G(0)+G'(\xi)z\leq G(0)+\mathcal L k=:g_1(k)$. In subsequent analysis, we write $g_1(k)$ as $g_1$ for simplicity if $k$ depends on the solutions or given data. For each $t \in [0,T],$ the weak solution $u$ of \eqref{eq1.1} is defined as: \begin{multline}\label{eq2.3} (u''(t),v) + G\left( \mu_1\| u(t)\|^2+\mu_2\|\nabla u(t)\|^2 \right) (u'(t),v) + (\nabla u(t), \nabla v) \\ - \int_0^{t}\beta(t-s)(\nabla u(s), \nabla v)ds = (f(t), v), \end{multline} for any $v \in H^1_0(\Omega)$ and any $t\in (0,T]$. We then apply the energy method, see e.g. \cite[Corollary 4.7]{Cannarsa}, to perform mathematical analysis for problem \eqref{eq1.1}-\eqref{eq1.5}. \subsection{Long-time stability} \begin{theorem}\label{theorem2.1} Under assumption $(\mathbf{A2})$, for $u_0\in H^1(\Omega)$, $u_1\in L^2(\Omega)$ and $f\in L^{1}(0,T;L^2(\Omega))$, the weak solution $u\in {\mathcal C}^1([0,T]; L^{2}(\Omega))\cap {\mathcal C}([0,T]; H^1_0(\Omega))$ is unique and satisfies the regularity estimate: \begin{equation}\label{eq2.5} \|u'(t)\| + \|\nabla u(t)\| \leq C\left( \|u_1\|+ \|\nabla u_0\| + \int_0^{t} \|f(s)\|ds \right), \quad {\rm for}~~~ 0<t\leq T. \end{equation} Furthermore, if $u_0\in H^2_0(\Omega)$, $u_1\in H^1(\Omega)$ and $f'\in L^{1}(0,T;L^2(\Omega))$, then $u(t) \in H^2(\Omega)$ for each $t\in (0,T],$ and \begin{equation}\label{eq2.5n} \|\nabla u'(t)\| \leq C\left( \|\nabla u_1\| + \|\Delta u_0\| + \|f(0)\| + \int_{0}^{t}(\|f(s)\|+\|f'(s)\|)ds \right),\quad {\rm for}~~~ 0<t\leq T. \end{equation} \end{theorem} \begin{proof} In \eqref{eq2.3}, we choose $v=u'(t)$ to get \begin{multline*} \frac{1}{2}\frac{d}{dt}\|u'(t)\|^2 + G\left( \mu_1\| u(t)\|^2+\mu_2\|\nabla u(t)\|^2 \right) \|u'(t)\|^2 + \frac{1}{2}\frac{d}{dt}\|\nabla u(t)\|^2 \\ - \int_0^{t}\beta(t-s)(\nabla u(s), \nabla u'(t))ds = (f(t), u'(t)). \end{multline*} Hence, multiply through by $2$, integrate over the time interval $(0,t)$ and use the assumption ($\mathbf{A1}$), $K'(t)=-\beta(t)$ and the Cauchy-Schwarz inequality to get \begin{multline} \label{eq4.4} \|u'(t)\|^2 + 2g_0 \int_0^{t}\|u'(s)\|^2ds +\|\nabla u(t)\|^2 \\ + 2\int_0^{t}\int_0^{s}K'(s-z)(\nabla u(z), \nabla u'(s))dzds \le \|u_1\|^2 + \|\nabla u_0\|^2 + 2\int_0^{t} \|f(s)\| \| u'(s)\|ds. \end{multline} Integration by parts yields \begin{multline*} \Phi(t;u) := \int_{0}^{t}\int_0^sK'(s-z)(\nabla u(z), \nabla u'(s))dzds \\ = \left( \nabla u_{0}, \int_{0}^{t}K(s)\nabla u'(s)ds \right) - \frac{K_0}{2}\left[\|\nabla u(t)\|^2 - \|\nabla u_{0}\|^2\right]\\ + \int_0^{t}\int_0^{s}K(s-z)(\nabla u'(q), \nabla u'(s))dzds. \end{multline*} Since the last term is nonnegative (by Lemma \ref{lemma1.1}) and since \[\int_{0}^{t}K(s)\nabla u'(s)ds = K(t)\nabla u(t) - K_0\nabla u_0 + \int_{0}^{t}\beta(s)\nabla u(s)ds,\] \begin{equation}\label{eq4.8} \Phi(t; u) \geq K(t)\left( \nabla u_{0}, \nabla u(t) \right) + \int_{0}^{t}\beta(s)(\nabla u_0,\nabla u(s)) ds - \frac{K_0}{2}\left[\|\nabla u(t)\|^2 + \|\nabla u_{0}\|^2\right]. \end{equation} Inserting this in \eqref{eq4.4} and using again the Cauchy-Schwarz inequality, we have \begin{multline}\label{eq4.9} \|u'(t)\|^2 + 2g_0 \int_0^{t}\|u'(s)\|^2ds + \mu_0\|\nabla u(t)\|^2 \leq \|u_1\|^2 + (1+K_0)\|\nabla u_0\|^2 \\ + 2\int_0^{t} \|f(s)\| \| u'(s)\|ds + 2K(t)\| \nabla u_{0}\| \|\nabla u(t) \| + 2\int_{0}^{t}\beta(s)\|\nabla u_0\| \|\nabla u(s)\| ds. \end{multline} Applying the Young's inequality and using $K(t)\leq c_0$, then the last three terms on the right-hand side are bounded by \begin{multline*} 2\left(\int_0^{t} \|f(s)\|ds\right)^2 + \frac{1}{2}\|u'(\Bar{t})\|^2+ \frac{\mu_0}{2} \|\nabla u(t) \|^2 + \frac{2c_0^2}{\mu_0} \| \nabla u_{0}\|^2, \\ +2\int_{0}^{t}\beta(s)ds \|\nabla u_0\|^2 + \frac{1}{2}\int_{0}^{t}\beta(s) \|\nabla u(s)\|^2 ds, \end{multline*} where $\|u'(\Bar{t})\|=\sup\limits_{0\leq s \leq t}\|u'(s)\|$ for some $0\le \Bar{t}\le t$. Therefore, with $c_1:=\max\limits_{0\leq t \leq \infty}\int_{0}^{t}|\beta(s)|ds $, we have \begin{multline}\label{eq4.11} \|u'(t)\|^2 + \frac{\mu_0}{2}\|\nabla u(t)\|^2 \leq \|u_1\|^2+ \\ \left(1+c_0+2c_1 + \frac{2c_0^2}{\mu_0} \right)\|\nabla u_0\|^2 + \frac{1}{2}\int_{0}^{t}\beta(s) \|\nabla u(s)\|^2 ds + 2\left(\int_0^{t} \|f(s)\|ds\right)^2 + \frac{1}{2}\|u'(\Bar{t})\|^2. \end{multline} Choosing $t=\Bar{t}$ yields after some simple simplifications \[ \frac{1}{2}\|u'(\Bar{t})\|^2 \leq \|u_1\|^2 + \left(1+c_0+2c_1 + \frac{2c_0^2}{\mu_0} \right)\|\nabla u_0\|^2 + \frac{1}{2}\int_{0}^{\Bar{t}}\beta(t) \|\nabla u(t)\|^2 dt + 2\left(\int_0^{\Bar{t}} \|f(s)\|ds\right)^2.\] Substituting this into the right-hand side of \eqref{eq4.11}, we obtain \[\|u'(t)\|^2 + \frac{\mu_0}{2}\|\nabla u(t)\|^2 \leq C(\|u_1\|^2 +\|\nabla u_0\|^2) + 4\left(\int_0^{t} \|f(s)\|ds\right)^2 + \int_{0}^{t}\beta(s) \|\nabla u(s)\|^2 ds.\] Since $\int_{0}^{\infty}|\beta(t)| dt \leq c_1$, the application of Gr\"{o}nwall's lemma yields \eqref{eq2.5}. From \cite[Proposition 4.4]{Cannarsa}, we conclude that $u(t) \in H^2(\Omega)$ for each $t\in (0,T].$ To show the estimate in \eqref{eq2.5n}, we choose $v=-\Delta u'(t)$ then follow the above derivation and use the identity \[\int_0^t ( f(s), \Delta u'(s))ds=( f(t), \Delta u(t))-( f(0), \Delta u(0))-\int_0^t ( f'(s), \Delta u(s))\,ds,\] in addition to the first achieved estimate. \end{proof} \subsection{Finite-time uniqueness} Let $\hat{u},\tilde{u}$ be two solutions of \eqref{eq2.3}. We need to show that the difference $Y(t):=\hat{u}-\tilde{u}=0$. From \eqref{eq2.3}, \begin{multline} (Y''(t), v) + G_0(\hat{u},\nabla \hat{u})(Y'(t), v) + (\nabla Y(t), \nabla v) \\ - \int_0^{t}\beta(t-s)(\nabla Y(s), \nabla v)ds = - [G_0(\hat{u},\nabla \hat{u})-G_0(\tilde{u},\nabla \tilde{u})] (\tilde{u}'(t), v),\label{eq4.14} \end{multline} where \begin{equation}\label{eq4.16} \begin{split} G_0(u(t),\nabla u(t)):=G\left( \mu_1\| u(t)\|^2+\mu_2\|\nabla u(t)\|^2 \right)\geq g_0. \end{split} \end{equation} Choose $v=Y'(t)$ in \eqref{eq4.14} to get \begin{multline}\label{eq4.17} \frac{1}{2}\frac{d}{dt}\|Y'(t)\|^2 + g_0 \|Y'(t)\|^2 + \frac{1}{2}\frac{d}{dt}\|\nabla Y(t)\|^2 + \int_0^{t}K'(t-s)(\nabla Y(s), \nabla Y'(t))ds \\ \leq \left|G_0(\hat{u},\nabla \hat{u}) - G_0(\tilde{u},\nabla \tilde{u})\right| \|\tilde{u}'(t)\| \|Y'(t)\|. \end{multline} By \eqref{eq2.5}, the assumption ($\mathbf{A2}$) and Poincar\'{e} inequality, we obtain \begin{equation}\label{eq4.18} \begin{split} \left|G_0(\hat{u},\nabla \hat{u}) - G_0(\tilde{u},\nabla \tilde{u})\right| \|\tilde{u}'(t)\| \leq C( \| Y(t)\| + \|\nabla Y(t)\| ) \leq C\|\nabla Y(t)\|. \end{split} \end{equation} Thus, integrate \eqref{eq4.17} over the time interval $(0,t_{*})$, and then, use $Y'(0)=Y(0)=0$, \eqref{eq4.8} and \eqref{eq4.18}, we get \begin{equation*} \begin{split} \|Y'(t_*)\|^2 & + 2 g_0 \int_0^{t_*}\|Y'(t)\|^2dt + \mu_0\|\nabla Y(t_*)\|^2 \leq C \int_0^{t_*}\|\nabla Y(t)\| \|Y'(t)\| dt \\ & \leq g_0 \int_0^{t_*}\| Y'(t)\|^2 dt + C\int_0^{t_*}\|\nabla Y(t)\|^2 dt. \end{split} \end{equation*} Canceling the similar terms yields \[ \|Y'(t_*)\|^2 + g_0 \int_0^{t_*}\|Y'(t)\|^2dt + \mu_0\|\nabla Y(t_*)\|^2 \leq C\int_0^{t_*}\|\nabla Y(t)\|^2 dt.\] We finally apply the Gr\"{o}nwall's lemma to complete the proof. \section{Spatial semi-discrete scheme}\label{sec3} Given a quasiuniform partition of $\Omega$ with the diameter $h$ and let $\mathcal{S}_h\subset H^1_0(\Omega)$ be a finite-dimensional space with respect to this partition, with the following approximation property \cite{ McLean1, Mustapha, Thomee} \begin{equation*} \begin{array}{ll} \inf\limits_{\psi\in \mathcal{S}_h}\{ \|v-\psi\| + h\|\nabla(v-\psi)\| \} \leq Ch^2 \|v\|_2, \quad v\in H^2(\Omega)\cap H^1_0(\Omega). \end{array} \end{equation*} For $t \in [0,T],$ the semi-discrete Galerkin finite element solution $u_h(t)\in \mathcal{S}_h$ is determined by \begin{align}\label{eq2.6} (u''_h(t),v_h) & + G\left( \mu_1\left\|u_h(t)\right\|^2+\mu_2\left\|\nabla u_h(t)\right\|^2 \right) (u'_h(t),v_h) + (\nabla u_h(t),\nabla v_h) \\ &\nonumber- \int_0^{t}\beta(t-s)(\nabla u_h(s),\nabla v_h)ds = (f(t), v_h), \quad {\rm for~any}~~v_h\in \mathcal{S}_h, \end{align} with $u_h(0) = u_{0h}:= w_h(0)$ and $ u'_h(0) = u_{1h}:=w_h'(0)$, where for each $t\in (0,T],$ the Ritz projection $w_h(t): H^1_0(\Omega)\rightarrow \mathcal{S}_h$ is defined by \begin{equation}\label{eq2.9} (\nabla \eta(t), \nabla v_h)=0, \quad \text{for any} \quad v_h\in \mathcal{S}_h, ~~{\rm where}~~\eta(t) = u(t) - w_h(t). \end{equation} Following the proof of the regularity property in Theorem \ref{theorem2.1}, we obtain the stability of the semi-discrete numerical solution in the next theorem. \begin{theorem}\label{theorem2.2} Under the assumptions of Theorem \ref{theorem2.1}, the solution $u_h$ of \eqref{eq2.6} satisfies \begin{equation*} \begin{array}{ll} \|u'_h(t)\| + \|\nabla u_h(t)\| \leq C\left( \|u_{1h}\|+ \|\nabla u_{0h}\| + \int_0^{t} \|f(s)\|ds\right), \text{ for } 0<t\leq T. \end{array} \end{equation*} \end{theorem} In the next theorem, we show the convergence of the semi-discrete Galerkin finite element solution. For convenience, let $E(t)=u(t)-u_h(t)$. \begin{theorem}\label{theorem2.3} Let $u_0\in H^2(\Omega)$, $u_1\in H^1(\Omega)$, and $f,f'\in L^1(0,T;L^2(\Omega))$. Under the assumptions $(\mathbf{A1})$-$(\mathbf{A2})$, and for $0<t\leq T$, we have \begin{equation*} \left\|E'(t)\right\|^2 + \left\|\nabla E(t)\right\|^2 \leq Ch^2\int_0^{t} \left( \|u''(s)\|^2_1 + \| u'(s)\|^2_1 + \| u(s)\|^2_2 \right)ds +Ch^2\|u(t)\|_1^2. \end{equation*} \end{theorem} \begin{proof} First, recall the definition of $\eta$ in \eqref{eq2.9}. The following Ritz projection error estimate is well known \cite{Thomee}: for $r=0,1$, and with $\partial_t$ denotes the partial time derivative, \begin{equation}\label{eq2.10} \begin{array}{ll} \dsp \|\partial_t^\theta\eta(t)\|+h\| \nabla(\partial_t^\theta\eta)(t)\| \leq Ch^{r+1}\|\partial_t^\theta u(t)\|_{H^{r+1}(\Omega)}, \quad \theta=0,1,2. \end{array} \end{equation} To derive the error bound in Theorem \ref{theorem2.3}, we decompose $E$ as: \begin{equation}\label{eq2.11} \begin{split} & E(t)=u(t)-u_h(t)=[u(t)-w_h(t)]-[u_h(t)-w_h(t)]:=\eta(t)-\xi(t), \end{split} \end{equation} and the main task is to estimate $\xi(t)$. We use \eqref{eq2.6}, \eqref{eq2.3} and \eqref{eq2.9} to get \begin{align*} (\xi''(t), v_h) &+ \left( G_0(u_h, \nabla u_h)u_h'(t) - G_0(w_h, \nabla w_h) w_h'(t), v_h \right) \nonumber\\ & + (\nabla \xi (t), \nabla v_h) - \int_{0}^{t}\beta(t-s)(\nabla \xi(s), \nabla v_h) ds \\ \nonumber& = (\eta''(t), v_h) + \left( G_0(u, \nabla u)u'(t) - G_0(w_h, \nabla w_h) w_h'(t), v_h \right), \end{align*} for any $v_h \in \mathcal{S}_h$, where $G_0(y,z)$ is defined in \eqref{eq4.16}. Choosing $v_h=\xi'(t)$ gives \begin{equation}\label{eq5.1} \begin{split} \frac{1}{2}\frac{d}{dt}& \|\xi'(t)\|^2 + \left( G_0(u_h, \nabla u_h)u_h'(t) - G_0(w_h, \nabla w_h) w_h'(t), \xi'(t) \right) \\ & + \frac{1}{2}\frac{d}{dt}\|\nabla \xi (t)\|^2 + \int_{0}^{t}K'(t-s)(\nabla \xi(s), \nabla \xi'(t)) ds \\ & = (\eta'', \xi'(t))+ \left( G_0(u, \nabla u)u'(t) - G_0(w_h, \nabla w_h) w_h'(t), \xi'(t) \right). \end{split} \end{equation} Denoting \begin{equation}\label{qqq1} \varphi_{\vartheta}(t):= \mu_1\| \xi(t) \vartheta + w_h(t)\|^2+\mu_2\|\nabla \xi(t)\vartheta+\nabla w_h(t)\|^2, \end{equation} and so, the second term on the left-hand side of \eqref{eq5.1} can be rewritten as: \begin{equation}\label{eq5.2} \begin{split} B(t):&=\big( G_0(u_h, \nabla u_h)u_h'(t) - G_0(w_h, \nabla w_h) w_h'(t), \xi'(t) \big) \\ & = \left(\int_0^1 \frac{d}{d\vartheta} \left[G(\varphi_{\vartheta}(t))(\vartheta\xi'(t) + w_h'(t) )\right] d\vartheta, \xi'(t)\right) \\ & = \int_0^1 G'(\varphi_{\vartheta}(t)) \frac{d \varphi_{\vartheta}(t)}{d \vartheta} \left[ \vartheta\|\xi'(t)\|^2 + (w_h', \xi'(t))\right] d\vartheta + \int_0^1 G(\varphi_{\vartheta}(t)) d\vartheta \|\xi'(t)\|^2, \end{split} \end{equation} where \begin{equation*} \frac{d \varphi_{\vartheta}(t)}{d \vartheta}=2\vartheta \left(\mu_1\| \xi(t)\|^2+\mu_2\|\nabla\xi(t)\|^2 \right) + 2\left[ \mu_1(w_h(t), \xi(t)) + \mu_2(\nabla w_h(t), \nabla \xi(t))\right], \end{equation*} and $ \int_0^1 G(\varphi_{\vartheta}(t)) d\vartheta \|\xi'(t)\|^2 \geq g_0 \|\xi'(t)\|^2$. Then we have \begin{equation}\label{eq5.3} \begin{split} B(t) \geq 2(J_1(t; \vartheta, w_h)+J_2(t; \vartheta, w_h)) + g_0 \|\xi'(t)\|^2, \quad t>0, \end{split} \end{equation} where \begin{equation}\label{eq5.4} \begin{split} & J_1(t; \vartheta, w_h) = \int_0^1 G'_{\vartheta} \left(\mu_1\| \xi(t)\|^2+\mu_2\|\nabla\xi(t)\|^2 \right) [\vartheta^2\|\xi'(t)\|^2+ \vartheta(w_h'(t), \xi'(t))] d\vartheta, \\ & J_2(t; \vartheta, w_h) = \int_0^1 G'_{\vartheta}\Big( \mu_1(w_h(t), \xi(t)) + \mu_2(\nabla w_h(t), \nabla\xi(t))\Big)[\vartheta \|\xi'(t)\|^2 +(w_h'(t), \xi'(t))] d\vartheta, \end{split} \end{equation} with $G'_{\vartheta}=G'(\varphi_{\vartheta}(t))$. Substitute \eqref{eq5.2} and \eqref{eq5.3} into \eqref{eq5.1}, and use the inequality \begin{equation}\label{eq5.5} (\eta'', \xi'(t)) \leq \|\eta''\| \|\xi'(t)\| \leq \frac{g_0}{8}\|\xi'(t)\|^2 + \frac{2}{g_0}\|\eta''\|^2, \end{equation} we conclude that \begin{multline}\label{eq5.6} \frac{1}{2} \frac{d}{dt}\|\xi'(t)\|^2 + \frac{7g_0}{8} \|\xi'(t)\|^2 + \frac{1}{2}\frac{d}{dt}\|\nabla \xi (t)\|^2 \\ + \int_{0}^{t}K'(t-s)(\nabla \xi(s), \nabla \xi'(t)) ds\leq \frac{2}{g_0}\|\eta''\|^2 + 2\sum\limits_{q=1}^3 \left|J_q(t; \vartheta, w_h)\right|, \end{multline} where $$J_3(t; \vartheta, w_h)=\left( G_0(u, \nabla u)u'(t) - G_0(w_h, \nabla w_h) w_h'(t), \xi'(t) \right).$$ The next task is to estimate the terms $\left|J_q(t; \vartheta, w_h)\right|$ for $q=1,2,3$. From the definition of $w_h$, $\|\nabla w_h'(t)\|\le \|\nabla u'(t)\|$, and so, using the Poincar\'{e} inequality and \eqref{eq2.5n}, we get \begin{equation}\label{eq5.7} \|w_h'(t)\| \le C\|\nabla u'(t)\| \leq C\left( \|\nabla u_1\| + \|\Delta u_0\| + \|f(0)\| + \int_{0}^{t}(\|f(s)\|+\|f'(s)\|)ds \right), \end{equation} which, together with Theorem \ref{theorem2.2} and the Poincar\'{e} inequality, gives \begin{equation}\label{eq5.8} \|\xi'(t)\| \leq \|w_h'(t)\| + \|u_h'(t)\| \leq C\left( \|\nabla u_1\| + \|\Delta u_0\| + \|f(0)\| + \int_0^t (\|f(s)\|+\|f'(s)\|)ds \right). \end{equation} Using \eqref{eq5.7}, \eqref{eq5.8}, and the Poincar\'{e} inequality, we obtain \begin{equation}\label{eq5.9} \left|J_1(t; \vartheta, w_h)\right| \leq C(\| \xi(t)\|^2 + \|\nabla \xi(t)\|^2)\leq C\|\nabla \xi(t)\|^2. \end{equation} From the definition of $w_h$, $\|\nabla w_h(t)\|\le \|\nabla u(t)\|$, and thus, using \eqref{eq2.5}, we notice that \begin{equation}\label{eq5.11} \|\nabla w_h(t)\| \leq C\left( \|\nabla u_1\|+ \|\nabla u_0\| + \int_{0}^{t}\|f(s)\|ds \right), \end{equation} and hence, using this, in addition to the estimates in \eqref{eq5.7} and \eqref{eq5.8}, and \eqref{eq5.11}, and the Young's inequality yield \begin{equation*} \left|J_2(t; \vartheta, w_h)\right| \leq C(\|\nabla \xi(t)\| + \|\xi(t)\|) \|\xi'(t)\| \leq C\|\nabla \xi(t)\| \|\xi'\| \leq \frac{g_0}{4}\|\xi'(t)\|^2 + C\|\nabla \xi(t)\|^2. \end{equation*} To bound $\left|J_3(t; \vartheta, w_h)\right|$, we apply the assumptions ($\mathbf{A1}$)-($\mathbf{A2}$) and the Cauchy-Schwarz inequality to get \begin{equation*} \begin{split} J_3(t; \vartheta, w_h) &= G_0(u,\nabla u) (u'- w_h', \xi') + ( [G_0(u,\nabla u)-G_0(w_h, \nabla w_h)]w_h', \xi') \\ & \leq g_1 \|\eta'\| \|\xi'(t)\| + \mathcal{L} \left| \mathcal{Q}[u,\nabla u](t) - \mathcal{Q}[w_h,\nabla w_h](t) \right| \|w_h'\| \|\xi'(t)\|, \end{split} \end{equation*} where $\mathcal{Q}[y,z](t):=\mu_1\|y(t)\|^2+\mu_2\|z(t)\|^2$. Then we utilize the inequality $\|y\|^2-\|z\|^2\leq (\|y\|+\|z\|)\|y-z\|$, \eqref{eq5.7}, \eqref{eq5.11} and Young's inequality to get \begin{equation}\label{eq5.14} \begin{split} |J_3(t; \vartheta, w_h)| & \leq \frac{g_0}{8}\|\xi'(t)\|^2 + \frac{2g_1}{g_0}\|\eta'\|^2 + C( \|\eta(t)\|^2 + \|\nabla\eta(t)\|^2). \end{split} \end{equation} Now we invoke \eqref{eq5.9}--\eqref{eq5.14} in \eqref{eq5.6} to obtain \begin{multline}\label{eq5.15} \frac{1}{2}\frac{d}{dt}\|\xi'(t)\|^2 + \frac{g_0}{2} \|\xi'(t)\|^2 + \frac{1}{2}\frac{d}{dt}\|\nabla \xi (t)\|^2 + \int_{0}^{t}K'(t-s)(\nabla \xi(s), \nabla \xi'(t)) ds \\ \leq \frac{2}{g_0}\|\eta''\|^2 + \frac{2g_1}{g_0}\|\eta'\|^2 + C \|\nabla\eta\|^2 + C\|\nabla \xi(t)\|^2. \end{multline} We incorporate $\xi'(0) = \xi(0)=0$ to integrate \eqref{eq5.15} over the time interval $(0,t)$ and apply \begin{multline*} \Phi(t; \xi) \geq K(t)\left( \nabla \xi(0), \nabla \xi(t) \right) + \int_{0}^{t}\beta(s)(\nabla \xi(0),\nabla \xi(s)) ds \\ - \frac{K_0}{2}\left[\|\nabla \xi(t)\|^2 + \|\nabla \xi(0)\|^2\right] = -\frac{K_0}{2}\|\nabla \xi(t)\|^2 \end{multline*} to get \[ \frac{1}{2}\|\xi'(t)\|^2 + \frac{g_0}{2} \int_0^{t}\|\xi'\|^2ds + \frac{\mu_0}{2}\|\nabla \xi (t)\|^2 \leq C \int_0^{t} \left(\|\eta''\|^2 + \|\eta'\|^2 + \|\nabla\eta\|^2 \right)ds + C \int_0^{t}\|\nabla \xi\|^2 ds.\] Apply the Gr\"{o}nwall's lemma to get \[ \|\xi'(t)\|^2 + g_0\int_0^{t}\|\xi'\|^2ds + \mu_0\|\nabla \xi (t)\|^2 \leq C \int_0^{t} \left(\|\eta''\|^2 + \|\eta'\|^2 + \|\nabla\eta\|^2 \right)ds,\] which, together with \eqref{eq2.10}, gives \[ \|\xi'(t)\|^2 + \mu_0\|\nabla \xi (t)\|^2 \leq Ch^2 \int_0^{t} \left( \|u''\|^2_1 + \| u'\|^2_1 + \| u\|^2_2 \right)ds. \] We combine this and \eqref{eq2.10} to complete the proof. \end{proof} \section{Fully-discrete scheme} \label{sec4} In this section, we shall establish and analyze a fully discrete scheme. Using $K'(t)=-\beta(t)$ and denoting $\mu_0:=1-K_0$, we have \begin{equation*} \int_0^t K'(t-s)\Delta u(s)ds = K(t)\Delta u_0 - K_0 \Delta u(t) + \int_0^t K(t-s)\Delta u'(s) ds. \end{equation*} Thus we reformulate \eqref{eq1.1} as \begin{align} & u''(t) + q(t)\,u'(t) - \mu_0 \Delta u(t) - \int_0^t K(t-s)\Delta u'(s) ds = f(t) + K(t)\Delta u_0. \label{ModelA} \end{align} The task now is to propose and analyze a fully discrete scheme for solving problem \eqref{ModelA}. Let $\tau$ be the temporal step size and $U^n\in \mathcal{S}_h$ with $0\leq n\in\mathbb N$ be the approximation of $u(t_n)$ with $t_n=n\tau$. Let $\delta_tU^n=(U^n-U^{n-1})/\tau$, $\bar{\delta}_tU^n=(U^{n+1}-U^{n-1})/(2\tau)$, $\delta_t^2 U^n=\delta_t(\delta_tU^{n+1})$, and $\widetilde{U}^n=(U^{n+1}+U^{n-1})/2$, $n\geq 1$. Using a linear polynomial interpolation \begin{align*} \varphi(s)\approx \mathcal{N}_1(s):=\varphi(t_{p-1})+\frac{s-t_{p-1}}{\tau}[\varphi(t_{p})-\varphi(t_{p-1})],\quad{\rm for}~~s \in [t_{p-1},t_p]\,, \end{align*} we approximate the integral term $\int_{0}^{t_n}K(t_n-s)\varphi(s)ds$ by \begin{equation}\label{eq2.14} Q_n(\varphi) = \sum\limits_{p=1}^{n}\int_{t_{p-1}}^{t_p}K(t_n-s)\mathcal{N}_1(s)ds= \sum\limits_{p=0}^{n}\widetilde{\kappa}_{np}\varphi(t_p), \quad n\geq 1, \end{equation} where \begin{equation}\label{coeff} \begin{split} \widetilde{\kappa}_{np} = \int_{-\min(\tau, t_p)}^{\min(\tau, t_{n-p})} K(t_n-s) \max\left( 1-\left| \frac{s}{\tau} \right|, 0 \right) dt, \quad 0\leq p \leq n. \end{split} \end{equation} Using \eqref{eq2.14} and the notations $u^n:=u(t_n)$ and $f^n:=f(t_n)$, we write \eqref{ModelA} at $t_n$ as \begin{align} &\delta_t^2 u^n + q(t_n)\bar{\delta}_tu^n - \mu_0 \Delta \widetilde{u}^n - \sum\limits_{p=0}^{n}\widetilde{\kappa}_{np}\Delta \bar{\delta}_t u^p = f^n + K(t_n)\Delta u_0 + \mathcal{R}^n, \label{ModelC}\\ & \delta_t u^1 = u_1 + \frac{\tau}{2}u_2 + \mathcal{R}^0, \quad u^0 = u_0=u(0),~u_1=u'(0),~u_2=u''(0), \label{ModelE} \end{align} in which $\bar{\delta}_t u^0 = u_1$, $\mathcal{R}^n:=\sum\limits_{l=1}^{4}R_l^n$ with $n\geq 1$, $u_2=u''(0)=-q(0)u_1 + \Delta u_0 + f^0$, and \begin{align} &R_1^n = \delta_t^2 u^n - u''(t_n),\quad R_2^n = q(t_n)\left[ \bar{\delta}_tu^n - u'(t_n) \right],\quad R_3^n = \mu_0\left[ \Delta \widetilde{u}^n - \Delta u^n \right], \label{qq1} \\ & R_4^n = \int_0^{t_n} K(t_n-s)\Delta u'(s) ds - \sum\limits_{p=0}^{n}\widetilde{\kappa}_{np}\Delta u'(t_p) + \sum\limits_{p=1}^{n}\widetilde{\kappa}_{np}\Delta (u'(t_p) - \bar{\delta}_tu^p), \label{qq4} \\ & \mathcal{R}^0 = \frac{1}{2\tau}\int_0^{\tau} (\tau - s)^2 u'''(s)dt. \label{qq5} \end{align} The weak form of \eqref{ModelC} is defined as: for any $\psi\in H_0^1(\Omega)$ and for $n\ge 1,$ \begin{multline}\label{qq6} (\delta_t^2u^n,\psi) + G\left( \mu_1\|u^n\|^2+\mu_2\|\nabla u^n\|^2 \right) (\bar{\delta}_tu^n,\psi) + \mu_0(\nabla \widetilde{u}^n,\nabla \psi) \\ + \sum\limits_{p=0}^{n}\widetilde{\kappa}_{np}(\nabla \bar{\delta}_t u^p,\nabla \psi) = (f^n, \psi) + K(t_n)(\nabla u^0, \nabla\psi) + (\mathcal{R}^n, \psi). \end{multline} We drop the truncation errors and define then the fully discrete Galerkin scheme as: for $n\ge 1,$ \begin{multline}\label{eq2.15} (\delta_t^2U^n,\psi_h) + G\left( \mu_1\|U^n\|^2+\mu_2\|\nabla U^n\|^2 \right) (\bar{\delta}_tU^n,\psi_h) + \mu_0(\nabla \widetilde{U}^n,\nabla \psi_h)\\ + \sum\limits_{p=0}^{n}\widetilde{\kappa}_{np}(\nabla \bar{\delta}_t U^p,\nabla \psi_h) = (f^n, \psi_h) + K(t_n)(\nabla U^0, \nabla\psi_h), \end{multline} for any $\psi_h\in \mathcal{S}_h$, given the initial values \begin{equation}\label{eq2.16} U^0=u_{0h}, \quad U^1=u_{0h}+ \tau u_{1h}+\frac{\tau^2}{2}u_{2h}, \end{equation} where $u_{jh}$, $j=0,1,2$ are suitable approximations of $u_j$ in $\mathcal{S}_h$. \subsection{Stability analysis} We show the stability of the fully discrete solution. \begin{theorem}\label{theorem2.4} Suppose that $\beta(t)$ is given by \eqref{eq1.5} and the assumptions $(\mathbf{A1})$-$(\mathbf{A2})$ hold with $\tau\sum_{i=0}^n\|f^i\|<\infty$ for $0<T< \infty$. Then for any $1\leq m\leq T/\tau$, it holds \begin{equation*} \left\|U^m\right\|_A \leq \left\|U^0\right\|_A + C\left( \|\nabla u_{0h}\| + \|\nabla u_{1h}\| + \tau\sum_{i=0}^n\|f^i\| \right), \end{equation*} where $C$ is independent from $T$, $m$ and $\tau$, and the semi-norm $\|\cdot\|_A$ is defined as \begin{equation}\label{eq6.3} \begin{split} \|U^m\|_A = \sqrt{ \|\delta_tU^{m+1}\|^2 + \frac{\mu_0}{2} \left(\|\nabla U^{m+1}\|^2+\|\nabla U^{m}\|^2 \right) }. \end{split} \end{equation} Consequently, the fully-discrete scheme (\ref{eq2.15}) has a unique solution. \end{theorem} \begin{proof} Choose $\psi=\bar{\delta}_tU^n$ in \eqref{eq2.15} to obtain \begin{multline}\label{eq6.1} (\delta_t^2U^n,\bar{\delta}_tU^n) + G\left( \mu_1\|U^n\|^2+\mu_2\|\nabla U^n\|^2 \right) (\bar{\delta}_tU^n,\bar{\delta}_tU^n) + \mu_0(\nabla \widetilde{U}^n,\nabla \bar{\delta}_tU^n)\\ + \sum\limits_{p=0}^{n}\widetilde{\kappa}_{np}(\nabla \bar{\delta}_t U^p,\nabla \bar{\delta}_tU^n) = (f^n, \bar{\delta}_tU^n) + K(t_n)(\nabla U^0, \nabla\bar{\delta}_tU^n), \quad n\geq 1, \end{multline} where, by direct calculations, \begin{equation}\label{eq6.2} \begin{split} (\delta_t^2U^n,\bar{\delta}_tU^n) &= \frac{1}{2 \tau} \left( \|\delta_tU^{n+1}\|^2 -\|\delta_tU^{n}\|^2 \right), \\ (\nabla \widetilde{U}^n,\nabla \bar{\delta}_tU^n) &= \frac{1}{4\tau} \left(\|\nabla U^{n+1}\|^2+\|\nabla U^{n}\|^2 \right) - \left(\|\nabla U^{n}\|^2+\|\nabla U^{n-1}\|^2 \right). \end{split} \end{equation} We sum \eqref{eq6.1} multiplied by $2\tau$ from 1 to $m$ and apply the new norm $\|\cdot\|_A$ defined by \eqref{eq6.3}, the assumption ($\mathbf{A1}$) and \eqref{eq6.2} to get \begin{multline}\label{eq6.4} \|U^m\|_A^2 + 2g_0\tau \sum\limits_{n=1}^{m}\|\bar{\delta}_tU^n\|^2 + 2 \tau \sum\limits_{n=1}^{m}\sum\limits_{p=0}^{n}\widetilde{\kappa}_{np}(\nabla \bar{\delta}_tU^p,\nabla \bar{\delta}_tU^n) \\ \leq \|U^0\|_A^2 + 2 \tau \sum\limits_{n=1}^{m}K(t_n)(\nabla U^0, \nabla \bar{\delta}_tU^n) + 2 \tau \sum\limits_{n=1}^{m}(f^n, \bar{\delta}_tU^n). \end{multline} Since \begin{equation*} \begin{split} 2 \tau \sum\limits_{n=1}^{m}K(t_n)\nabla \bar{\delta}_tU^n & = \sum\limits_{n=1}^{m}K(t_n)\left[ (\nabla U^{n+1}+\nabla U^{n}) - (\nabla U^{n} + \nabla U^{n-1}) \right] \\ & = K(t_m)\left(\nabla U^{m+1}+\nabla U^{m}\right) - K(t_1)(\nabla U^{1}+\nabla U^{0}) \\ & \quad + \sum\limits_{n=1}^{m-1}\left[ K(t_n)-K(t_{n+1}) \right] (\nabla U^{n+1}+\nabla U^{n}), \end{split} \end{equation*} \[2 \tau \sum\limits_{n=1}^{m}K(t_n)(\nabla U^0, \nabla \bar{\delta}_tU^n) \leq C\|\nabla U^0\|\left( \left\|U^0\right\|_A + \left\|U^m\right\|_A \right) + C\|\nabla U^0\|\sum\limits_{n=1}^{m-1} \int_{t_{n}}^{t_{n+1}} |\beta(s)|ds \left\|U^n\right\|_A.\] Following \cite[51--52]{McLean}, it holds \[ \tau \sum\limits_{n=1}^{m}\sum\limits_{p=0}^{n}\widetilde{\kappa}_{np}(\nabla \bar{\delta}_tU^p,\nabla \bar{\delta}_tU^n) \geq \tau \sum\limits_{n=1}^{m} \widetilde{\kappa}_{n0} (\nabla u_{1h},\nabla \bar{\delta}_tU^n). \] Furthermore, recall the definition of $\|\cdot\|_A$ in \eqref{eq6.3}, thus we have \begin{align} \|\bar{\delta}_tU^n\| &\leq \frac{\|U^n\|_A+\|U^{n-1}\|_A}{2}, \nonumber \\ \left| \tau (\nabla u_{1h},\nabla \bar{\delta}_tU^n) \right| &\leq \sqrt{\frac{2}{\mu_0}} \frac{(\|U^n\|_A+\|U^{n-1}\|_A)}{2} \|\nabla u_{1h}\|. \label{eq6.7} \end{align} We substitute the above contribution into \eqref{eq6.4} to get \begin{equation*} \begin{split} \|U^m\|_A^2 &\leq \|U^0\|_A^2 + 2\sqrt{\frac{2}{\mu_0}} \sum\limits_{n=1}^{m} \widetilde{\kappa}_{n0} \frac{(\|U^n\|_A+\|U^{n-1}\|_A)}{2} \|\nabla u_{1h}\| \\ & \quad + C\|\nabla U^0\|\left( \left\|U^0\right\|_A + \left\|U^m\right\|_A \right) + C\|\nabla U^0\|\sum\limits_{n=1}^{m-1} \int_{t_{n}}^{t_{n+1}} |\beta(s)|ds \left\|U^n\right\|_A \\ & \quad + 2 \tau \sum\limits_{n=1}^{m}\|f^n\| \frac{\|U^n\|_A+\|U^{n-1}\|_A}{2}. \end{split} \end{equation*} We choose a suitable $\ell$ such that $\|U^\ell\|_A = \max\limits_{0\leq n \leq m}\|U^n\|_A$, which, together with $\beta\in L_{1}(0,\infty)$, yields \begin{equation}\label{eq6.9} \begin{split} \|U^\ell\|_A & \leq \|U^0\|_A + C\left[ \sum\limits_{n=1}^{m} \widetilde{\kappa}_{n0} \|\nabla u_{1h}\| + \tau \sum\limits_{n=1}^{m}\|f^n\| + \|\nabla u_{0h}\| \right]. \end{split} \end{equation} By \eqref{coeff} and the monotonicity of $s^{\alpha-1}$, we have for $m\geq 1$ and $\sigma \geq 1$ \begin{equation}\label{eq6.10} \begin{split} \sum\limits_{n=1}^{m} \widetilde{\kappa}_{n0} & \leq \sum\limits_{n=1}^{m} \int_{t_{n-1}}^{t_n} K(t) dt = \int_{0}^{t_m} K(t) dt \leq \int_{0}^{t_m} \int_t^{\infty }|\beta(s)| ds dt \\ & \leq \int_0^{\infty} \int_t^{\infty}\frac{s^{\alpha-1}}{\Gamma(\alpha)} e^{-\sigma s}dsdt \leq \int_0^{\infty} \frac{t^{\alpha-1}}{\Gamma(\alpha)} \int_t^{\infty}e^{-\sigma s}dsdt \\ &= \int_0^{\infty} \frac{t^{\alpha-1}}{\Gamma(\alpha)} \frac{e^{-\sigma t}}{\sigma}dt \leq \int_0^{\infty} \frac{e^{-t}t^{\alpha-1}}{\Gamma(\alpha)}dt = 1. \end{split} \end{equation} Combine \eqref{eq6.9} and \eqref{eq6.10} to complete the proof. \end{proof} \subsection{Error estimate} We derive error estimate of the fully discrete Galerkin scheme. \begin{theorem}\label{theorem2.5} Suppose that $\beta(t)$ is given by \eqref{eq1.5} and the assumptions $(\mathbf{A1})$-$(\mathbf{A2})$ hold with $u_0\in H^2(\Omega)$, $u_1\in H^1(\Omega)$, and $f,f'\in L^1(0,T; L^2(\Omega))$ for $0<T<\infty$. Then the following error estimate holds for any $1\le m\leq T/\tau$: \begin{align} \left\|\nabla \left(U^{m} - u(t_{m})\right) \right\| &\leq Ch \left( \|u'\|_{L^{\infty}(H^1) } + \|u_1\|_1 +\tau \|u_0\|_2 \right) \nonumber \\ & + Ch\left(\|u\|_{L^{\infty}(H^2)}+\|u'\|_{L^{\infty}(H^1)}+\|u''\|_{L^{\infty}(H^1)}\right) \nonumber \\ & + C\tau \int_0^{2\tau} \|u'''(t)\|dt + C\tau^2 \int_{\tau}^{t_{m+1}} \|u^{(4)}(t)\|dt \nonumber \\ & + C\tau^2 \int_0^{t_{m+1} }\|u'''(t)\|dt + C\tau^2 \int_0^{t_{m+1} }\|\Delta u''(t)\|dt \nonumber \\ & + C \tau \int_0^{\tau} \|\Delta u''(t)\|dt + C\tau^2 \int_{\tau}^{t_{m+1}} \|\Delta u'''(t)\|dt. \label{eq2.20} \end{align} \end{theorem} \begin{proof} We split $u(t_n)-U^n=[u(t_n)-w_h(t_n)]-[U^n-w_h(t_n)]:=\eta^n-\xi^n$ for $n\geq 1$ where $w_h(t)$ is the elliptic projection of $u(t_n)$ in \eqref{eq2.9}, and it suffices to bound $\|\xi^n\|$. We subtract \eqref{qq6} from \eqref{eq2.15} to get \begin{equation*} \begin{split} (\delta_t^2\xi^n,\psi) &+ \left[G_0\left( U^n, \nabla U^n\right) (\bar{\delta}_tU^n,\psi) - G_0\left( w_h(t_n),\nabla w_h(t_n)\right) (\bar{\delta}_tw_h(t_n),\psi) \right] \\ &+ \mu_0 (\nabla \widetilde{\xi}_1^n,\nabla \psi) + \sum\limits_{p=1}^{n}\widetilde{\kappa}_{np}(\nabla \bar{\delta}_t\xi^p,\nabla \psi) \\ & = (\delta_t^2\eta^n,\psi) + \left(G_0\left(u^n,\nabla u^n\right) \bar{\delta}_tu^n - G_0\left( w_h(t_n),\nabla w_h(t_n)\right) \bar{\delta}_tw_h(t_n), \psi\right) \\ & + ( G_0\left(u^n,\nabla u^n\right)(u'(t_n) - \bar{\delta}_tu^n) , \psi) + \sum\limits_{j=1}^{4} ( R_j^n, \psi), \quad n\geq 1, \end{split} \end{equation*} for $\psi\in \mathcal{S}_h$. Choosing $\psi=\bar{\delta}_t\xi^n$ gives \begin{equation}\label{eq7.1} \begin{split} (\delta_t^2\xi^n,\bar{\delta}_t\xi^n) &+ \left[G_0\left( U^n, \nabla U^n\right) (\bar{\delta}_tU^n,\bar{\delta}_t\xi^n) - G_0\left( w_h(t_n),\nabla w_h(t_n)\right) (\bar{\delta}_tw_h(t_n),\bar{\delta}_t\xi^n) \right] \\ &+ \mu_0 (\nabla \widetilde{\xi}_1^n,\nabla \bar{\delta}_t\xi^n) + \sum\limits_{p=1}^{n}\widetilde{\kappa}_{np}(\nabla \bar{\delta}_t\xi^p,\nabla \bar{\delta}_t\xi^n) \\ & = (\delta_t^2\eta^n,\bar{\delta}_t\xi^n) + \left(G_0\left(u^n,\nabla u^n\right) \bar{\delta}_tu^n - G_0\left( w_h(t_n),\nabla w_h(t_n)\right) \bar{\delta}_tw_h(t_n), \bar{\delta}_t\xi^n \right) \\ & + ( G_0\left(u^n,\nabla u^n\right)(u'(t_n) - \bar{\delta}_tu^n), \bar{\delta}_t\xi^n) + \sum\limits_{j=1}^{4} ( R_j^n, \bar{\delta}_t\xi^n ), \quad n\geq 2, \end{split} \end{equation} We first find a lower bound of the second term of the left-hand side of \eqref{eq7.1}. By the Newton-Leibniz formula, we rewrite this term as \begin{equation}\label{eq7.2} \begin{split} B_0(n; \tau):&=\left( G_0(U^n, \nabla U^n)\bar{\delta}_tU^n - G_0(w_h(t_n), \nabla w_h(t_n)) \bar{\delta}_tw_h(t_n), \bar{\delta}_t\xi^n \right) \\ & = \left(\int_0^1 \frac{d}{d\vartheta} \left[G(\varphi_{\vartheta}(t_n))(\vartheta \bar{\delta}_t\xi^n + \bar{\delta}_tu_{h}^{*}(t_n) )\right] d\vartheta, \bar{\delta}_t\xi^n \right) \\ & = \int_0^1 G'(\varphi_{\vartheta}(t_n)) \frac{d \varphi_{\vartheta}(t_n)}{d \vartheta} \left[ \vartheta\|\bar{\delta}_t\xi^n\|^2 + \left(\bar{\delta}_tw_h(t_n), \bar{\delta}_t\xi^n\right)\right] d\vartheta \\ &+ \int_0^1 G(\varphi_{\vartheta}(t_n)) d\vartheta \|\bar{\delta}_t\xi^n\|^2, \end{split} \end{equation} where $\varphi_{\vartheta}(t)$ is given by \eqref{qqq1} and $$\frac{d \varphi_{\vartheta}(t_n)}{d \vartheta}:=2\vartheta \left[\mu_1\| \xi^n\|^2+\mu_2\|\nabla\xi^n\|^2 \right] + 2\left[ \mu_1(w_h(t_n), \xi^n) + \mu_2(\nabla w_h(t_n), \nabla\xi^n) \right].$$ We then use $ \int_0^1 G(\varphi_{\vartheta}(t_n)) d\vartheta \|\bar{\delta}_t\xi^n\|^2 \geq g_0 \|\bar{\delta}_t\xi^n\|^2$ to get \begin{equation*} \begin{split} B_0(n; \tau) \geq 2\sum\limits_{q=1}^4 \hat{J}_q(n; \vartheta, \tau) + g_0 \|\bar{\delta}_t\xi^n\|^2, \quad t>0, \end{split} \end{equation*} where \begin{equation}\label{eq7.4} \begin{split} & \hat{J}_1(n; \vartheta, \tau) = \int_0^1 G'_{\vartheta n} \left(\mu_1\| \xi^n\|^2+\mu_2\|\nabla\xi^n\|^2 \right) [\vartheta^2\|\bar{\delta}_t\xi^n\|^2+\vartheta(\bar{\delta}_tw_h(t_n), \bar{\delta}_t\xi^n)] d\vartheta, \\ & \hat{J}_2(n; \vartheta, \tau) = \int_0^1 G'_{\vartheta n}\Big( \mu_1(w_h(t_n), \xi^n) + \mu_2(\nabla w_h(t_n), \xi^n)\Big)[\vartheta \|\bar{\delta}_t\xi^n\|^2 +\bar{\delta}_tw_h(t_n), \bar{\delta}_t\xi^n)] d\vartheta, \end{split} \end{equation} with $G'_{\vartheta n}=G'(\varphi_{\vartheta}(t_n))$. Below, we shall analyze the terms $\hat{J}_1$ and $\hat{J}_2$ in \eqref{eq7.4}. First, we apply Theorem \ref{theorem2.4} to obtain $\|\bar{\delta}_tU^n\|\leq C$, and \eqref{eq5.7} gives \begin{equation}\label{eq7.5} \begin{split} \|\bar{\delta}_t w_h(t_n)\| \leq \Big\| \frac{1}{2\tau} \int_{t_{n-1}}^{t_{n+1}} w_h'(t)dt \Big\| \leq \frac{C}{\tau} \int_{t_{n-1}}^{t_{n+1}}\|w_h'(t)\|dt \leq C. \end{split} \end{equation} Then we employ the triangle inequality to get \begin{equation}\label{eq7.6} \begin{split} \|\bar{\delta}_t\xi^n\|\leq \|\bar{\delta}_tU^n\|+\|\bar{\delta}_t w_h(t_n)\| \leq C, \quad n\geq 2. \end{split} \end{equation} Based on \eqref{eq7.5}--\eqref{eq7.6} and the assumption ($\mathbf{A2}$), we have for $n\geq 2$ \[\|\hat{J}_1(n; \vartheta, \tau)\| \leq C\left(\| \xi^n\|^2+\|\nabla \xi^n\|^2 \right) \leq C\|\nabla \xi^n\|^2.\] Then we use \eqref{eq7.5}--\eqref{eq7.6}, Poincar\'{e} inequality and Young inequality to get \[\|\hat{J}_2(n; \vartheta, \tau)\| \leq C\left(\| \nabla \xi^n\|+\|\xi^n\| \right) \|\bar{\delta}_t\xi^n\| \leq C\| \nabla \xi^n\| \|\bar{\delta}_t\xi^n\| \leq \frac{g_0}{8}\|\bar{\delta}_t\xi^n\|^2 + C\| \nabla \xi^n\|^2.\] We further estimate the second term of the right-hand side of \eqref{eq7.1}. We apply \eqref{eq6.3} to get \begin{equation}\label{eq7.11} \begin{split} &\left(G_0\left(u^n,\nabla u^n\right) \bar{\delta}_tu^n - G_0\left( w_h(t_n),\nabla w_h(t_n)\right) \bar{\delta}_tw_h(t_n), \bar{\delta}_t\xi^n \right)\\ &\leq \left| \left( G_0\left(u^n,\nabla u^n\right) [\bar{\delta}_tu^n - \bar{\delta}_tw_h(t_n)], \bar{\delta}_t\xi^n\right) \right| \\ & + \left| \left( [G_0\left(u^n,\nabla u^n\right) - G_0\left( w_h(t_n),\nabla w_h(t_n)\right)] \bar{\delta}_tw_h(t_n), \bar{\delta}_t\xi^n\right) \right| \\ & \leq \mathcal{L}\left[ \mu_1(\|u^n\|^2-\|w_h(t_n)\|^2)+\mu_2(\|\nabla u^n\|^2-\|\nabla w_h(t_n)\|^2)\right]\\ & \qquad \times \|\bar{\delta}_tw_h(t_n)\|\|\bar{\delta}_t\xi^n\| + g_1 \left\|\bar{\delta}_t\eta^n\right\|\|\bar{\delta}_t\xi^n\| \leq C\left(\|\nabla \eta^n\| + \left\|\bar{\delta}_t\eta^n\right\| \right) \frac{\|\xi^n\|_A+\|\xi^{n-1}\|_A}{2}. \end{split} \end{equation} We invoke \eqref{eq7.2}--\eqref{eq7.11} in \eqref{eq7.1}, sum the resulting equation multiplied by $2\tau$ for $n$ from 1 to $m$ and use \eqref{eq6.2}--\eqref{eq6.7} to get \begin{multline*} \|\xi^m\|^2_A \leq \|\xi^0\|^2_A + C \tau \sum\limits_{n=1}^{m} \Big(\|\xi^n\|_A^2 + \|\delta_t^2 \eta^n\| \frac{\|\xi^n\|_A+\|\xi^{n-1}\|_A}{2} \\ + \left\|u'(t_n)-\bar{\delta}_tu^n\right\| \frac{\|\xi^n\|_A+\|\xi^{n-1}\|_A}{2} + \sum\limits_{j=1}^{4}\|R_j^n\| \frac{\|\xi^n\|_A+\|\xi^{n-1}\|_A}{2} \\ + \left(\|\nabla \eta^n\| + \left\|\bar{\delta}_t\eta^n\right\| \right) \frac{\|\xi^n\|_A+\|\xi^{n-1}\|_A}{2}\Big). \end{multline*} Choose a suitable $\ell$ such that $\|\xi^\ell \|_A = \max\limits_{0\leq n \leq m}\left\|\xi^n \right\|_A$, we have for $j\le m,$ \begin{multline}\label{eq7.13} \|\xi^\ell\|_A \leq C \tau \sum\limits_{n=1}^\ell \|\xi^n\|_A + \left\|\xi^0 \right\|_A + C \tau \sum\limits_{n=1}^\ell \|\delta_t^2 \eta^n\| \\ + C \tau \sum\limits_{n=1}^\ell\Big( \left\|u'(t_n)-\bar{\delta}_tu^n\right\| + \sum\limits_{j=1}^{4}\|R_j^n\| + \left(\|\nabla \eta^n\| +\left\|\bar{\delta}_t\eta^n\right\| \right)\Big). \end{multline} Next, we will estimate the terms of the right-hand side of \eqref{eq7.13}. First, we get \begin{align} & \tau \sum\limits_{n=1}^{m} \left\|\delta_t^{2}\eta^n \right\|\leq C \tau \sum\limits_{n=1}^{m} \|\eta''_1(\kappa_n)\|\leq Ch \|u''\|_{L^{\infty}(H^1)}, \;\; \kappa_n\in [t_{n-1}, t_{n+1}], \label{eq7.14} \\ & \tau \sum\limits_{n=1}^{m} \left\|u'(t_n)-\bar{\delta}_tu^n\right\| \leq \frac{\tau^2}{2}\int_0^{t_{m+1}}\|u'''(s)\|ds,\nonumber\\ & \tau \sum\limits_{n=1}^{m} \left(\|\nabla \eta^n\| +\left\|\bar{\delta}_t\eta^n\right\| \right) \leq Ch\left(\|u\|_{L^{\infty}(H^2)}+\|u'\|_{L^{\infty}(H^1)}\right). \nonumber \end{align} Then we discuss the terms $R_j^n$ with $1\leq j\leq 4$. First, we bound the term on $R_1^n$ in \eqref{qq1}. We apply the Taylor expansion with integral remainder to arrive at \begin{equation*} \begin{split} & u''(t_n) - \delta_t^{2}u^n = \frac{-1}{6\tau^2} \left[ \int_{t_n}^{t_{n+1}}(t_{n+1}-t)^3u^{(4)}(t)dt + \int_{t_{n-1}}^{t_{n}}(t-t_{n-1})^3u^{(4)}(t)dt\right], \\ & n\geq 2, \quad u''(t_1) - \delta_t^{2}u^1 = \frac{-1}{2\tau^2} \left[ \int_{t_1}^{t_{2}}(t_{2}-t)^2 u'''(t)dt + \int_{0}^{t_{1}}t^2u'''(t)dt\right], \end{split} \end{equation*} which gives \begin{equation*} \begin{split} \tau \sum\limits_{n=1}^{m} \|R_1^n\| \leq \tau \int_0^{2\tau} \|u'''(t)\|dt + \tau^2 \int_{\tau}^{t_{m+1}} \|u^{(4)}(t)\|dt. \end{split} \end{equation*} To estimate $R_2^n$ in \eqref{qq1}, we use the identity \begin{equation*} \begin{split} u'(t_n)-\Bar{\delta}_tu^n &= \frac{-1}{4\tau} \left[ \int_{t_n}^{t_{n+1}}(t_{n+1}-t)^2u'''(t)dt + \int_{t_{n-1}}^{t_{n}}(t-t_{n-1})^2u'''(t)dt\right], \end{split} \end{equation*} and obtain \begin{equation*} \begin{split} \tau \sum\limits_{n=1}^{m} \left\|R_2^n \right\|\leq \frac{g_1}{2} \tau^2 \int_0^{t_{m+1} }\|u'''(t)\|dt. \end{split} \end{equation*} However, to estimate $R_3^n$ in \eqref{qq1}, we use \begin{equation*} \begin{split} \Delta u(t_n) - \Delta\widetilde{u}^n & = \frac{-1}{2} \Bigg[ \int_{t_{n}}^{t_{n+1}} (t_{n+1}-t) \Delta u''(t)dt + \int_{t_{n-1}}^{t_{n}} (t-t_{n-1}) \Delta u''(t) dt \Bigg], \end{split} \end{equation*} and get \begin{equation*} \begin{split} \tau \sum\limits_{n=1}^{m} \|R_3^n\| \leq \tau^2 \int_0^{t_{m+1} }\|\Delta u''(t)\|dt. \end{split} \end{equation*} To estimate $R_4^n$ in \eqref{qq4}, we follow the procedure in \cite[52]{McLean} with $\hat{\mu}_j = \int_{t_j}^{t_{j+1}}K(s)ds$ to obtain \begin{equation*} \begin{split} \tau \sum\limits_{n=1}^{m} \|R_4^n\| & \leq \hat{\mu}_0 \tau \int_0^{\tau} \|\Delta u''(t)\|dt + \tau^2 \sum\limits_{p=2}^{m}\left(\sum\limits_{n=p}^{m} \hat{\mu}_{n-p} \right) \int_{t_{p-1}}^{t_p} \|\Delta u'''(t)\|dt \\ & + \tau \sum\limits_{p=1}^{m} \Delta\left(u'(t_p)-\bar{\delta}_tu^p \right) \left( \sum\limits_{n=p}^{m} \widetilde{\kappa}_{np} \right). \end{split} \end{equation*} Thus, since $\sum\limits_{n=p}^{m} \hat{\mu}_{n-p} \leq \int_0^{t_m}K(t)dt \leq C$ and since $\sum\limits_{n=p}^{m} \widetilde{\kappa}_{np} \leq C\tau \sum\limits_{n=p}^{m} \leq C$ (which is due to \eqref{coeff}), \begin{equation*} \begin{split} \tau \sum\limits_{n=1}^{m} \|R_4^n\| & \leq C \tau \int_0^{\tau} \|\Delta u''(t)\|dt + C\tau^2 \int_{\tau}^{t_{m+1}} \|\Delta u'''(t)\|dt. \end{split} \end{equation*} Finally, to estimate the first term of the right-hand side of \eqref{eq7.13}, i.e., $\|\xi^0\|_A$. we subtract \eqref{ModelE} from \eqref{eq2.16} and use \eqref{qq5} to get \begin{equation*} \begin{split} & \delta_t \xi^1 = \delta_t \eta^1 + (u_{1h}-u_1) + \frac{\tau}{2}(u_{2h}-u_2) - \mathcal{R}^0, \\ & \nabla \xi^1 = \tau \left[ \delta_t \nabla\eta^1 + (\nabla u_{1h}-\nabla u_1) + \frac{\tau}{2}(\nabla u_{2h}-\nabla u_2) - \nabla\mathcal{R}^0 \right], \end{split} \end{equation*} which, together with \eqref{eq6.3}, leads to \begin{equation}\label{eq7.22} \begin{split} \left\|\xi^0 \right\|_A & \leq Ch \left( \|u'\|_{L^{\infty}(H^1) } + \|u_1\|_1 +\tau \|u_0\|_2 \right) + C\tau \int_0^{\tau} \| u'''(t)\|dt. \end{split} \end{equation} We invoke \eqref{eq7.14}--\eqref{eq7.22} in \eqref{eq7.13} and use $\|\nabla \eta(t_m)\|\leq Ch\max\limits_{0\leq t \leq t_m}\|u(t)\|_2$ and the discrete Gr\"{o}nwall's lemma to obtain \eqref{eq2.20}. This completes the proof. \end{proof} \subsection{Discussion on temporal accuracy} Since the solution of \eqref{eq1.1}-\eqref{eq1.3} may exhibit initial singularity and is in general smooth away from the initial time under certain regularity assumptions on the given data, we focus the attention near the initial time and thus consider \eqref{eq1.1}-\eqref{eq1.3} with $q(t)= q(0):=q_0$ for simplicity. Hence, to consider the temporal solution regularity, one could apply the eigenpairs of $-\Delta$ to decompose the linear version of \eqref{eq1.1}-\eqref{eq1.3} as ordinary differential equations of the following form for some $\lambda>0$: for $t>0$, \begin{equation}\label{eq7.36} u'' + q_0 u' +\lambda u -\lambda \int_0^{t}\beta(t-s) u(s)ds = f(t), ~~{\rm with}~~ u(0) = u_0~{\rm and}~u'(0) = u_1. \end{equation} Since \[ u'(t)= u_1 + \int_0^t u''(s) ds~{\rm and}~ u(t)= u_0 + tu_1 + \int_0^t (t-s)u''(s) ds, \quad t>0,\] \begin{equation*} \begin{split} u''(t) = f(t) & - q_0 \left[u_1 + \int_0^t u''(s) ds\right] - \lambda \left[u_0 + tu_1 + \int_0^t (t-s)u''(s) ds\right] \\ & + \lambda \int_0^{t}\beta(t-s) \left[u_0 + su_1 + \int_0^s (s-z)u''(z) dz \right]ds, \quad t> 0. \end{split} \end{equation*} Thus, for the case (i) in (\ref{eq1.5}), we have the asymptotic behaviour of $u''$ and $u'''$ as follows \begin{align*} u''(t)& \simeq f(t) - q_0 u_1 - \lambda u_0 + \frac{\lambda u_0}{\Gamma(1+\alpha)}t^{\alpha} + O(t^{\alpha}), \quad t\rightarrow 0^{+},\\ u'''(t)& \simeq f'(t) - q_0\, u''(0) - \lambda u_1 + \frac{\lambda u_0}{\Gamma(\alpha)}t^{\alpha-1} + O(t^{\alpha-1}), \quad t\rightarrow 0^{+}, \end{align*} which implies $|u''(t)| \leq C$ and $|u'''(t)| \leq C(1 + t^{\alpha-1})$. Similarly we could get $|u^{(4)}(t)| \leq C(1 + t^{\alpha-2})$. For the case (ii) in (\ref{eq1.5}), where there is no singularity in the kernel, the solutions could be smooth under smooth data. Thus, it is reasonable to assume that the solutions to \eqref{eq1.1}-\eqref{eq1.3} satisfy \begin{equation}\label{regul} \begin{split} &t^{\alpha-1}\|\Delta u''(t)\| +\|\Delta u'''(t)\| \leq C(1+t^{\alpha-1}), \quad \|u^{(4)}(t)\| \leq C(1+t^{\alpha-2})\text{ for case } (i), \\ & \|\Delta u''(t)\| + \|\Delta u'''(t)\| + \|u^{(4)}(t)\| \leq C\text{ for case } (ii). \end{split} \end{equation} Then the last three terms on the right-hand side of \eqref{eq2.20} could be further bounded as \begin{equation*} \begin{split} & \int_0^{2\tau} \|u'''(t)\|dt + \tau \int_{\tau}^{t_{m+1}} \|u^{(4)}(t)\|dt \leq C\tau^\alpha, \\ & \int_0^{t_{m+1} }\Big(\|u'''(t)\| + \|\Delta u''(t)\|\Big)dt \leq C, \\ & \int_0^{\tau} \|\Delta u''(t)\|dt + C\tau \int_{\tau}^{t_{m+1}} \|\Delta u'''(t)\|dt \leq C\tau, \end{split} \end{equation*} which results in $O(\tau^{1+\alpha})$ temporal accuracy with $\alpha=1$ or $\alpha=\frac{1}{2}$. \begin{remark}\label{rem2.1} Motivated by the above analysis, Theorem \ref{theorem2.5} indicates the $O(h+\tau^{1+\alpha})$ accuracy of the $H^1$-norm error with $\alpha=1$ or $\alpha=\frac{1}{2}$. \end{remark} \section{Numerical experiment} \label{sec5} \subsection{One-dimensional case} Let $\Omega=(0,1)$, $u_0(x)=\sin(\pi x)$, $u_1(x)=\sin(2\pi x)$ and $f(x,t)=t^{\alpha}e^{-\sigma t} \cos(\gamma t) \sin(\pi x)$. Let $G(z)=\sqrt{1+z}$ with $\mu_1=\mu_2=1$ in \eqref{eq1.2} and $h=\frac{1}{M}$ for some $M>0$. Since the exact solution is unknown, to illustrate numerically the achieved $H^1$-norm convergence rates in Theorem \ref{theorem2.5}, we define \begin{align*} {E_t(M,N)}& =\sqrt{h\sum\limits_{j=1}^{M-1}\left|V_j^{N+1}-V_j^{2N+1} \right|^2}, ~ {E_s(M,N)}=\sqrt{h\sum\limits_{j=1}^{M-1}\left|V_j^{N+1}-V_{2j}^{N+1} \right|^2},~V_j^n = \frac{U_j^n-U_{j-1}^n}{h}\,.\end{align*} Here $U_j^n$ approximates $u(x_j,t_n)$ and hence, $V_j^n$ approximates $\nabla u (x_j,t_n)$, $E_t$ represents the difference between gradients of numerical solutions at time $T$ computed under the time step sizes $\tau=T/N$ and $\tau=T/(2N)$, and $E_s$ could be interpreted similarly. The convergence rates in time and space are accordingly defined as \begin{align*} CR_t =\log_{2}\left(\frac{E_t(M,N)}{E_t(M,2N)}\right), \quad CR_s=\log_{2}\left(\frac{E_s(M,N)}{E_s(2M,N)}\right). \end{align*} We fix $T=1$ to evaluate the $H^1$ errors and convergence rates of the fully-discrete Galerkin scheme in Table \ref{tab1}, which demonstrate the $O(\tau^{1+\alpha} + h)$ accuracy as predicted in Theorem \ref{theorem2.5}. \begin{table}[H] \center \footnotesize \caption{ $H^1$ errors and time-space convergence rates under $T=1$.} \label{tab1} \vskip 2mm \begin{tabular}{cccccccccccc} \toprule & & & \multicolumn{2}{c}{$N=32$, $\gamma=3\sqrt{3}$, $\sigma=3$} & &\multicolumn{2}{c}{$M=32$, $\gamma=1.0$, $\sigma=2.0$}\\ \cmidrule{4-5} \cmidrule{7-8} $\alpha=0.5$ &$M$ & & {$E_s$} & {$CR_s$} & $N$ & {$E_t$} & {$CR_t$} \\ \midrule & $32$ & & $2.4232 \times 10^{-2}$ & * & $128$ & $2.3145 \times 10^{-3}$ & * \\ & $64$ & & $1.2798 \times 10^{-2}$ & 0.92 & $256$ & $7.2638 \times 10^{-4}$ & 1.67 \\ & $256$ & & $6.5733 \times 10^{-3}$ & 0.96 & $512$ & $2.3469 \times 10^{-4}$ & 1.63 \\ & $512$ & & $3.3306 \times 10^{-3}$ & 0.98 & $1024$ & $7.7536 \times 10^{-5}$ & 1.60 \\ \text{Predict} & & & & 1.00 & & & 1.50 \\ \midrule & & & \multicolumn{2}{c}{$N=32$, $\gamma=2$, $\sigma=2$} & &\multicolumn{2}{c}{$M=32$, $\gamma=0.5$, $\sigma=1.1$}\\ \cmidrule{4-5} \cmidrule{7-8} $\alpha=1.0$ &$M$ & & {$E_s$} & {$CR_s$} & $N$ & {$E_t$} & {$CR_t$} \\ \midrule & $16$ & & $3.6308 \times 10^{-2}$ & * & $16$ & $1.2054 \times 10^{-1}$ & * \\ & $32$ & & $1.8470 \times 10^{-2}$ & 0.98 & $32$ & $2.8221 \times 10^{-2}$ & 2.09 \\ & $64$ & & $9.5854 \times 10^{-3}$ & 0.95 & $64$ & $6.8974 \times 10^{-3}$ & 2.03 \\ & $128$ & & $4.9102 \times 10^{-3}$ & 0.97 & $128$ & $1.7148 \times 10^{-3}$ & 2.01 \\ \text{Predict} & & & & 1.00 & & & 2.00 \\ \bottomrule \end{tabular} \end{table} Next, we test the possible energy dissipation of the proposed model. It is shown in \cite[Page 498]{Cavalcanti1} that the Euler-Bernoulli viscoelastic model, which coincides with the form of model (\ref{eq1.1}) with $-\Delta$ replaced by $\Delta^2$, admits the energy dissipation with the corresponding energy \begin{equation}\label{mh1} \hat E(t):=\frac{1}{2} \|u'(t)\|^2 + \frac{1}{2} \|\Delta u(t)\|^2 \end{equation} Motivated by this definition, we define the following energy for problem \eqref{eq1.1} with $f=0$ by \begin{equation}\label{energy1} \begin{split} \widetilde{E}(t) = \frac{1}{2} \|u'(t)\|^2 + \frac{1}{2} \|\nabla u(t)\|^2, \quad t\geq 0, \end{split} \end{equation} and we numerically evaluate this energy as follows \begin{equation*} \begin{split} \widetilde{E}^n = \frac{1}{2} \|\bar{\delta}_tU^n\|^2 + \frac{1}{2} \|\nabla U^n\|^2, \quad n\geq 1, \quad \widetilde{E}^0 = \frac{1}{2} \|u_1\|^2 + \frac{1}{2} \|\nabla u_0\|^2. \end{split} \end{equation*} Numerical results are presented in Figure \ref{fig1}, which indicate the dissipative property of the energy for $\alpha=0.5$ or $1$. \begin{figure}[H] \centering \includegraphics[width=3.8in]{Graph1.eps} \caption{Plots of the energy under $f=0$, $\sigma=3$, $\gamma=3\sqrt{3}$, and $N=M=32$.} \label{fig1} \end{figure} \subsection{Two-dimensional case} Here, let $\Omega=(0,1)\times (0,1)$, $u_0(x,y)=\sin(\pi x)\sin(\pi y)$, $u_1(x,y)=\sin(2\pi x)\sin(2\pi y)$, and $f(x,y,t)=0$. Let $G(z)=\sqrt{1+z}$ with $\mu_1=\mu_2=1$ in \eqref{eq1.2} and $h_x=h_y=h=\frac{1}{M}$ for some $M>0$. In two-dimensional case, to illustrate numerically the achieved $H^1$-norm convergence rates in Theorem \ref{theorem2.5}, we denote \begin{align*} {E_t(M,N)}& =h\,\sqrt{\sum\limits_{i=1}^{M-1}\sum\limits_{j=1}^{M-1}\left|W_{i,j}^{N+1}-W_{i,j}^{2(N+1)} \right|^2}, \quad {E_s(M,N)}=h\,\sqrt{\sum\limits_{i=1}^{M-1}\sum\limits_{j=1}^{M-1}\left|W_{i,j}^{N+1}-W_{2i,2j}^{N+1} \right|^2}, \\ W_{i,j}^n & = \sqrt{\left(\frac{U_{i,j}^n-U_{i-1,j}^n}{h}\right)^2+\left(\frac{U_{i,j}^n-U_{i,j-1}^n}{h}\right)^2}. \end{align*} In Table \ref{tab3-1}, we test the $H^1$ errors and convergence orders of the fully discrete Galerkin scheme for the two-dimensional case. We observe that the numerical results are consistent with our theoretical analysis (see Remark \ref{rem2.1}). \begin{table} \center \footnotesize \caption{ $H^1$ errors and time-space convergence rates under $T=1/2$.} \label{tab3-1} \vskip 2mm \begin{tabular}{cccccccccccc} \toprule & & & \multicolumn{2}{c}{$N=16$, $\gamma=3\sqrt{3}$, $\sigma=3$} & &\multicolumn{2}{c}{$M=64$, $\gamma=0.5$, $\sigma=1.5$}\\ \cmidrule{4-5} \cmidrule{7-8} $\alpha=0.5$ &$M$ & & {$E_s$} & {$CR_s$} & $N$ & {$E_t$} & {$CR_t$} \\ \midrule & $64$ & & $6.9769 \times 10^{-3}$ & * & $64$ & $4.2265 \times 10^{-3}$ & * \\ & $128$ & & $3.3304 \times 10^{-3}$ & 1.07 & $128$ & $1.3363 \times 10^{-3}$ & 1.66 \\ & $256$ & & $1.6259 \times 10^{-3}$ & 1.03 & $256$ & $4.3567 \times 10^{-4}$ & 1.62 \\ & $512$ & & $8.0321 \times 10^{-4}$ & 1.02 & $512$ & $1.4597 \times 10^{-4}$ & 1.58 \\ \text{Predict} & & & & 1.00 & & & 1.50 \\ \midrule & & & \multicolumn{2}{c}{$N=16$, $\gamma=2$, $\sigma=2$} & &\multicolumn{2}{c}{$M=64$, $\gamma=0.5$, $\sigma=1.1$}\\ \cmidrule{4-5} \cmidrule{7-8} $\alpha=1.0$ &$M$ & & {$E_s$} & {$CR_s$} & $N$ & {$E_t$} & {$CR_t$} \\ \midrule & $64$ & & $8.9036 \times 10^{-3}$ & * & $32$ & $9.9870 \times 10^{-3}$ & * \\ & $128$ & & $4.3094 \times 10^{-3}$ & 1.05 & $64$ & $2.4006 \times 10^{-3}$ & 2.06 \\ & $256$ & & $2.1189 \times 10^{-3}$ & 1.02 & $128$ & $5.1903 \times 10^{-4}$ & 2.21 \\ & $512$ & & $1.0505 \times 10^{-3}$ & 1.01 & $256$ & $1.2184 \times 10^{-4}$ & 2.09 \\ \text{Predict} & & & & 1.00 & & & 2.00 \\ \bottomrule \end{tabular} \end{table} \section{Concluding remarks} \label{sec6} This work performs numerical analysis for a hyperbolic integrodifferential equation. The developed techniques in this work overcome the difficulties caused by the non-positive variable-sign kernel and nonlinear-nonlocal damping, which could also be extended to investigate other related problems such as the Euler-Bernoulli viscoelastic problem in \cite{Cavalcanti1} that coincides with the form of model (\ref{eq1.1}) with $-\Delta$ replaced by $\Delta^2$, or (\ref{eq1.1}) with a different damping coefficient \cite{Xu3} \begin{equation*} q(t) = G\left( \int_0^t \int_{\Omega}\mu_1| u(x,s)|^2+\mu_2|\nabla u(x,s)|^2dxds \right), \;\; \mu_1, \mu_2 \geq 0, \;\; \mu_1^2+ \mu_2^2 \neq 0. \end{equation*} In particular, it is shown in \cite[Theorem 2.1]{Cavalcanti1} that if the kernel function $\beta(t)$ satisfies the assumption ($\mathbf{A1}$) and \begin{align} &\qquad\qquad \beta(0)>0, \quad 1-\int_{0}^{\infty}\beta(s)ds = \ell >0, \label{ass1} \\ & -c_1\beta(t) \leq \beta'(t) \leq -c_2 \beta(t), \quad 0\leq \beta''(t) \leq c_3 \beta(t) \quad \forall t\geq 0, \nonumber \end{align} where $c_1,c_2,c_3$ are positive constants, the energy (\ref{mh1}) decays exponentially in time. However, since the kernel $\beta(t)$ in this work has a variable sign and initial singularity, it does not satisfy the conditions in \eqref{ass1} such that it is difficult to follow the derivations in \cite{Cavalcanti1} to derive the energy decay. Nevertheless, the numerical experiment suggests the energy decay as shown in Figure \ref{fig1}, which motivates us to perform a further investigation for this issue. Finally, in the current work we only prove the error estimate in the $H^1$ norm. Due to the existence of the gradient term in $q(t)$, which may limit the improvement of the convergence order in deriving the $L^2$ error, it is not straightforward to obtain the optimal $L^2$ error estimate in the current circumstance. We will investigate this interesting question in the near future. \begin{thebibliography}{99} \small \bibitem{Alabau-Boussouira} F. Alabau-Boussouira, P. Cannarsa and D. Sforza, Decay estimates for second order evolution equations with memory, J. Funct. Anal., 254 (2008), 1342--1372. \bibitem {Allegretto} W. Allegretto and Y. Lin, Numerical solutions for a class of differential equations in linear viscoelasticity, Calcolo, 30 (1993), 69--88. \bibitem {Appleby} J. A. Appleby, M. Fabrizio, B. Lazzari and D. W. Reynolds, On exponential asymptotic stability in linear viscoelasticity, Math. Models Methods Appl. Sci., 16 (2006), 1677--1694. \bibitem {Baker1} K. Baker and L. Banjai, Numerical analysis of a wave equation for lossy media obeying a frequency power law, IMA J. Numer. Anal., 42 (2022), 2083--2117. \bibitem {Baker2} K. Baker, L. Banjai and M. Ptashnyk, Numerical analysis of a time-stepping method for the Westervelt equation with time-fractional damping, Math. Comp., 93 (2024), 2711--2743. \bibitem {Cavalcanti1} M. M. Cavalcanti, V. N. Domingos Cavalcanti and T. F. Ma, Exponential decay of the viscoelastic Euler-Bernoulli equation with a nonlocal dissipation in general domains, Differ. Integral Equ., 17 (2004), 495--510. \bibitem {Cannarsa1} P. Cannarsa and D. Sforza, Integro-differential equations of hyperbolic type with positive definite kernels, J. Differential Equ., 250 (2011), 4289--4335. \bibitem {Cannarsa} P. Cannarsa and D. Sforza, A stability result for a class of nonlinear integrodifferential equations with $L^1$ kernels, Appl. Math., 35 (2008), 395--430. \bibitem {Cannon} J. R. Cannon, Y. Lin and C. Xie, Galerkin Methods and $L^2$-error estimates for hyperbolic integrodifferential equations, Calcolo, 26 (1989), 197--207. \bibitem{Emm} E. Emmrich and M. Thalhammer, A class of integrodifferential equations incorporating nonlinear and nonlocal damping with applications in nonlinear elastodynamics: Existence via time discretization, Nonlinearity, 24 (2011), 2523--2546. \bibitem {Fairweather} G. Fairweather, Spline collocation methods for a class of hyperbolic partial integrodifferential equations, SIAM J. Numer. Anal., 31 (1994), 444--460. \bibitem {Georgiev} V. Georgiev, B. Rubino and R. Sampalmieri, Global existence for elastic waves with memory, Arch. Rational Mech. Anal., 176 (2005), 303--330. \bibitem {Karaa} S. Karaa, A. K. Pani and S. Yadav, A priori hp-estimates for discontinuous Galerkin approximations to linear hyperbolic integrodifferential equations, Appl. Numer. Math., 96 (2015), 1--23. \bibitem {Larsson} S. Larsson and F. Saedpanah, The continuous Galerkin method for an integrodifferential equation modeling dynamic fractional order viscoelasticity, IMA J. Numer. Anal., 30 (2010), 964--986. \bibitem {Lebon} G. Lebon, C. Perez-Garcia and J. Casas-Vazquez, On the thermodynamic foundations of viscoelasticity, J. Chem. Phys., 88 (1988), 5068--5075. \bibitem {McLean} W. McLean and V. Thom\'{e}e, Numerical solution of an evolution equation with a positive-type memory term, J. Austral. Math. Soc. Ser. B, 35 (1993), 23--70. \bibitem {McLean1} W. McLean and K. Mustapha, A second-order accurate numerical method for a fractional wave equation, Numer. Math., 105 (2007), 481--510. \bibitem {Messaoudi1} S. A. Messaoudi and B. Said-Houari, Global nonexistence of positive initial-energy solutions of a system of nonlinear viscoelastic wave equations with damping and source terms, J. Math. Anal. Appl., 365 (2010), 277--287. \bibitem {Messaoudi2} S. A. Messaoudi, Blow up and global existence in a nonlinear viscoelastic wave equation, Math. Nach., 260 (2003), 58--66. \bibitem {Mustapha} K. Mustapha and H. Mustapha, A second-order accurate numerical method for a semilinear integrodifferential equation with a weakly singular kernel, IMA J. Numer. Anal., 30 (2010), 555--578. \bibitem {Renardy} M. Renardy, W. J. Hrusa and J. A. Nohel, Mathematics Problems in Viscoelasticity, in: Pitman Monographs Pure Appl. Math., vol. 35, Longman Sci. Tech., Harlow, Essex, 1988. \bibitem {Saedpanah} F. Saedpanah, Continuous Galerkin finite element methods for hyperbolic integro-differetial equations, IMA J. Numer. Anal., 35 (2015), 885--908. \bibitem {Thomee} V. Thom\'{e}e, Galerkin finite element methods for parabolic problems, Springer, 2007. \bibitem {Xu2} D. Xu, Decay properties for the numerical solutions of a partial differential equation with memory, J. Sci. Comput., 62 (2015), 146--178. \bibitem {Xu3} D. Xu, Analytical and numerical solutions of a class of nonlinear integrodifferential equations with L1 kernels, Nonlinear Anal.-Real, 51 (2020), 103002. \bibitem {Xu4} D. Xu, Application of the Crank–Nicolson time integrator to viscoelastic wave equations with boundary feedback damping, IMA J. Numer. Anal., 42 (2022), 487--514. \bibitem {Xu5} D. Xu, Finite element methods of the two nonlinear integrodifferential equations, Appl. Math. Comput., 58 (1993), 241--273. \bibitem {Yanik} E. G. Yanik and G. Fairweather, Finite element methods for parabolic and hyperbolic partial integrodifferential equations, Nonlinear Anal., 12 (1988), 785--809. \bibitem {Zhao} M. Zhao and W. Qiu, Finite element approximation and analysis of damped viscoelastic hyperbolic integrodifferential equations with $L^1$ kernel, Appl. Math. Lett., 151 (2024), 108993. \end{thebibliography} \end{document}
2412.07452v1
http://arxiv.org/abs/2412.07452v1
On the fundamental group of steady gradient Ricci solitons with nonnegative sectional curvature
\documentclass[12pt,twoside]{article} \usepackage{geometry} \geometry{left=3.5cm,right=3.5cm,top=3.5cm,bottom=3.5cm} \usepackage{graphicx,subcaption} \usepackage{amssymb} \usepackage{indentfirst} \usepackage{amsmath} \usepackage{amsthm} \usepackage{bm} \usepackage{changepage} \usepackage{lineno} \usepackage{setspace} \usepackage{booktabs,multirow} \usepackage{authblk} \usepackage{graphicx} \usepackage{float} \usepackage[flushleft]{threeparttable} \usepackage{tikz-cd} \usepackage{cite} \usepackage{url} \usepackage{lipsum} \usepackage[marginal]{footmisc}\usepackage{fancyhdr} \usepackage[colorlinks = true, linkcolor = blue, urlcolor = blue, citecolor = blue, anchorcolor = blue]{hyperref} \renewenvironment{abstract}{ \setlength\textwidth{3in} \begin{quote} }{ \end{quote}}\renewcommand{\thefootnote}{} \numberwithin{equation}{section} \newtheorem{theorem}{Theorem}[section] \newtheorem{proposition}[theorem]{Proposition} \newtheorem{definition}[theorem]{Definition} \newtheorem{lemma}[theorem]{Lemma} \newtheorem{corollary}[theorem]{Corollary} \newtheorem{claim}{Claim} \newtheorem{remark}[theorem]{Remark} \newcommand\keywords[1]{\textbf{ Keywords:}#1} \title{On the fundamental group of steady gradient Ricci solitons with nonnegative sectional curvature} \author{Yuxing $\text{Deng}^*$ and~~Yuehan~Hao } \date{} \begin{document} \maketitle \pagestyle{fancy} \fancyhf{} \fancyhead[OC]{\small FUNDAMENTAL GROUP OF STEADY GRADIENT RICCI SOLITONS} \fancyhead[EC]{\small YUXING DEND~AND~YUEHAN~HAO} \renewcommand{\headrulewidth}{0pt} \fancyfoot[c]{\small \thepage} \footnote{ \keywords{ Ricci flow, Ricci soliton, fundamental group}\\ $^{*}$Supported by National Key R$\&$D Program of China 2022YFA1007600. } \begin{abstract} ABSTRACT. In this paper, we study the fundamental group of the complete steady gradient Ricci soliton with nonnegative sectional curvature. We prove that the fundamental group of such a Ricci soliton is either trivial or infinite. As a corollary, we show that an $n$-dimensional complete $\kappa$-noncollapsed steady gradient Ricci soliton with nonnegative sectional curvature must be diffeomorphic to $\mathbb{R}^n$. \end{abstract} \section{Introduction} \subsection{Introduction} A \textbf{Ricci soliton} is a quadruple $(M ,g,X,\lambda)$ consisting of a smooth manifold $M $, a Riemannian metric $g$, a smooth vector field $X$ and a real constant $\lambda$, if these variables satisfy the equation \begin{equation} \label{ricci soliton definition} {\rm Ric}+\mathcal{L}_Xg=\frac{\lambda}{2}g \end{equation} on $M$, where ${\rm Ric}$ denotes the Ricci tensor of $g$, and $\mathcal{L}$ denotes the Lie derivative. For $\lambda =0$ the Ricci soliton is \textbf{steady}, for $\lambda >0$ it is \textbf{shrinking} and for $\lambda <0$ \textbf{expanding}. Up to scaling, we can normalize $\lambda = 0,1,$ or $-1$, respectively. If the vector field $X= \nabla f$ for some smooth function $f$ on $M $, then the Ricci soliton is called a \textbf{gradient Ricci soliton}. For such a soliton, equation (\ref{ricci soliton definition}) simplifies to \begin{equation} \label{ric+downtri f=} {\rm Ric} +\nabla^2f=\frac{\lambda}{2} g, \end{equation} since $\mathcal{L}_{\nabla f} g = 2 \nabla^2 f$. Here $\nabla^2$ denotes the Hessian of $f$. In this paper, We will use the notation $(M,g,f, \lambda)$ to denote a gradient Ricci soliton. If we can determine the \textbf{scale} $\lambda$ and the \textbf{potential function} $f$ from the context, then the underlying manifold $(M,g)$ will be often referred to as the Ricci soliton. In addition to certain metric $g$ of $M$, we simplify the notation to $M$. Since a Ricci soliton that arises as the blow-up limit of a compact Ricci flow is $\kappa$-noncollpased. We are interested in $\kappa$-noncollapsed Ricci solitons. The notion of $\kappa$-noncollapsing is defined as follows. \begin{definition}[Noncollapsed manifolds] \label{noncollapsed} Let $\kappa, \rho>0 $. We say that a Riemannian manifold $\left(\mathcal{M}, g\right)$ is $\mathbf{\kappa }$\textbf{-noncollapsed on the scale} $\rho$ if for any $ x \in \mathcal{M}$ and $ r \in(0, \rho)$ satisfying $R \leq r^{-2} $ in $B_{r}(x) $, we have \begin{equation} {\rm Vol } B_{r}(x) \geq \kappa r^{n}, \end{equation} where $B_r(x)$ is the geodesic ball with the radius $r$ based at $x$. If $\left(\mathcal{M} , g\right) $ is $\kappa $-noncollapsed on the scale $\rho$ for all $ \rho \in(0, \infty) $, we say that $ \left(\mathcal{M} , g\right) $ is $\kappa $\textbf{-noncollapsed on all scales}. \end{definition} Ricci solitons play an important role in the singularity analysis of the Ricci flow. Ricci flows generated by shrinking and steady Ricci solitons are special ancient solutions to the Ricci flow. $\kappa$-solutions are $\kappa$-noncollapsed ancient solutions. The classification of $\kappa$-solutions with positive curvature may rely on the classification of shrinking and steady gradient Ricci with positive curvature, especially the rotational symmetry of positively curved steady gradient Ricci solitons. For recent progress on the classification of $\kappa$-solutions to the Ricci flow, see \cite{Cho-Li-3023,Li-2020-arXiv,Brendle-2020,Brendle-Daskalopoupos-Sesum-2021,Brendle_singularity_models_2022,Brendle_Daskalopoulos_Naff_Sesum_2023,Brendle_Naff_2023,Deng-Zhu-PAMS-2020,Li-Zhang-2022,cao-xie-2024-arxiv}. In dimension 3, the uniqueness of $\kappa$-noncollapsed and non-flat steady gradient Ricci solitons was claimed by Perelman in \cite{perelman_entropy_2002}. Perelman's claim was confirmed by Brendle in \cite{brendle_rotational_2013}. There are also many works on the higher dimensional $\kappa$-noncollapsed steady gradient Ricci solitons, such as \cite{Deng-Zhu-2018,Cao_chen_2012,brendle_rotational_2014,Deng-Zhu-Mathann-2020,Deng-Zhu-SCM-2020,Deng-Zhu-JEMS-2020,Bamler-Chow-Deng-Ma-2022,Chow-Deng-Ma-2022,chan_dimension_2023,zhao-zhu-arxiv-2022,zhao-zhu-arxiv-2024,ma-Mahmoudian-Sesum-arxiv-2023}. Many new examples of steady gradient Ricci solitons have also been found, see \cite{Lai-JDG-2024,appleton_family_2022,conlon-Deruelle-arxiv-2020,buttsworth-2021-arxiv,Buzano-Dancer-Wang-2015,wink-2023} and references therein. Due to the steady Ricci solitons constructed by Yi Lai \cite{Lai-JDG-2024}, 4-dimensional $\kappa$-noncollapsed steady gradient Ricci soliton with nonnegative sectional curvature and positive Ricci curvature is not unique. How to classify these Ricci solitons is still an open problem. Even the topology of 4-dimensional $\kappa$-noncollapsed steady gradient Ricci soliton with nonnegative sectional curvature has not been classified yet. It is well-known that the topology of manifolds with nonnegative sectional curvature has been studied by Cheeger and Gromoll in \cite{Cheeger-Gromoll-1972}. A manifold with nonnegative sectional curvature must be a normal bundle over a soul\cite{Cheeger-Gromoll-1972,sarafutdinov-1979}. For the expanding case, an expanding gradient Ricci soliton with nonnegative Ricci curvature must be diffeomorphic to the Euclidean space, see Lemma 5.5 in \cite{Cao-Catino-Chen-Mantegazza-Mazzieri-2014} or Lemma 2.2 in \cite{Chen-Deruelle-2015}. For the shrinking case, the topology of a shrinking gradient Ricci soliton with nonnegative sectional curvature is still unknown. It is open whether a 4-dimensional compact shrinking gradient Ricci soliton with positive sectional curvature is diffeomorphic to a quotient of $\mathbb{S}^4$ or $\mathbb{CP}^2$. There are also many nontrivial noncompact shrinking gradient Ricci solitons with nonnegative sectional curvature and finite fundamental groups, such as $(\mathbb{S}^2\times \mathbb{R})/\mathbb{Z}_2$. In this paper, we study the topology of steady gradient Ricci solitons with nonnegative sectional curvature. We get the following result on the fundamental group of steady gradient Ricci solitons. \begin{theorem} \label{MAIN} Suppose that $(M,g,f)$ is an $n$-dimensional complete steady gradient Ricci soliton with nonnegative sectional curvature. Then the fundamental group $\pi_1(M)$ of $M$ is either trivial or infinite. Moreover, if $\pi_1(M)$ is trivial, then $M$ is diffeomorphic to $\mathbb{R}^n$. \end{theorem} Bamler\cite{bamler_fundamental_2021} has proved that any $\kappa$-noncollapsed ancient soliton to the Ricci flow must have a finite fundamental group. \begin{theorem}[Theorem 1.1. \cite{bamler_fundamental_2021}] \label{bamler_noncollapsed finite fundamental group_theorem} Let $(M,(g_t)_{t \le 0})$ be a $\kappa$-noncollapsed ancient Ricci flow that has complete time-slices and bounded curvature on compact time-intervals. Then the fundamental group $\pi_1(M)$ is finite. \end{theorem} As an application of Bamler's finiteness theorem and Theorem \ref{MAIN}. We have \begin{theorem}\label{MAIN-2} An $n$-dimensional $\kappa$-noncollapsed complete steady gradient Ricci soliton with nonnegative sectional curvature is diffeomorphic to $\mathbb{R}^n$. \end{theorem} By Theorem \ref{MAIN-2}, we have the following refinement of Corollary 1.2 by Cao and Xie in \cite{cao-xie-2024-arxiv}. \begin{corollary} Let $(M^4,g,f)$ be a $4$-dimensional complete noncompact, non-flat, $\kappa$-noncollapsed steady gradient Ricci soliton with nonnegative isotropic curvature. Then, either (i) $(M^4,g,f)$ has positive isotropic curvature and positive Ricci curvature, or (ii) $(M^4,g,f)$ is isometric to the product $N^3\times\mathbb{R}$ of the 3D Bryant soliton with the real line. \end{corollary} \subsection{Outline} We now outline the main steps involved in the proof of the main result. In Section 2, we briefly review group actions and the geometry of the universal Riemannian covering of manifolds. Particularly, each compact subgroup of the affine group acting on affine spaces has a fixed point in this space\cite{wolf_spaces_2011}. Moreover, covering automorphism groups, consisting of fixed-point-free isometries, are closely associated with the fundamental groups \cite{lee_introduction_2011, lee_introduction_2012,petersen_riemannian_2006}. In Section 3, we prove that the isometry groups of universal covers based on gradient Ricci solitons with nonnegative sectional curvature preserve the splitting property argued in \cite{guan_rigidity_nodate}. In Section 4, in addition to positive Ricci curvature, we argue that such solitons are simply connected. Furthermore, these solitons do not have non-trivial finite quotient manifolds. In Section 5, we present the proof of Theorem \ref{MAIN}. Suppose that $M$ is a steady gradient Ricci soliton with nonnegative sectional curvature. Applying the splitting theorem in \cite{guan_rigidity_nodate}, there is a universal Riemannian covering map $\pi :N^k \times \mathbb{R}^{n-k} \rightarrow M$, where the Ricci curvature on $N^k$ is strictly positive. Then we analyze the behaviour of components of a covering automorphism acting on $N^k$ and $\mathbb{R}^{n-k}$ separately. We shall show that if its covering automorphism group is not trivial, then it must be infinite. \section{Preliminaries} In this section, we recall some results concerning the group actions on manifolds and Riemannian universal coverings. \subsection{Group actions on manifolds} \begin{definition} If $G$ is a group and $M$ is a set, a \textbf{left action of $\mathbf{G}$ on $\mathbf{M}$} is a map $G \times M \rightarrow M$, often written as $(g,x) \mapsto g \cdot x$, which satisfies \begin{itemize} \item[(i)] $g_1\cdot( g_2 \cdot x )= (g_1g_2) \cdot x$ for all $g_1, g_2 \in G$ and $x \in M$; and \item[(ii)] $e \cdot x =x$ for all $x \in M$. \end{itemize} \end{definition} For any $x \in M$, the set of images of $x$ under all elements of the group $G \cdot x = \{g\cdot x : g \in G\} \subseteq M$ is called the \textbf{orbit} of $x$. A \textbf{topological group} is a group $G$ with a topology such that the multiplication $ G \times G \rightarrow G$, $(x,y) \mapsto xy$ and the inverse $ G \rightarrow G$, $x \mapsto x^{-1}$ are continuous. A \textbf{Lie group} is a smooth manifold equipped with a group structure such that the multiplication and inversion maps are both smooth. The following terminology regarding group actions are standard, see\cite{lee_introduction_2011,lee_introduction_2012,lee_introduction_2018}. \begin{definition} \label{group terminology} Suppose that there is a left action of a group $G$ on a set $M$. \begin{itemize} \item The action is said to be \textbf{free} if the only element of $G$ that fixes any point in $M$ is the identity; that is, $g \cdot x \not= x$ for $g \in G $ and $x \in M $ except $g = e$, where $e$ is the identity element of $G$. \item If $M$ is a smooth manifold, $G$ is a Lie group, and the defining map $G \times M \rightarrow M$ is smooth, then the action is said to be a \textbf{smooth action}. \item A continuous left action of a Lie group $G$ on a smooth manifold $M$ is said to be a \textbf{proper action} if the map $G \times M \rightarrow M \times M$ given by $(g,x) \mapsto (g\cdot x, x)$ is a proper map; that is, the pre-image of each compact set is compact. \item If in addition $M$ is equipped with a Riemannian metric, the action is said to be an \textbf{isometric action} if the map $x \mapsto \varphi \cdot x$ is an isometry for each $\varphi \in G$. \end{itemize} \end{definition} We consider the Lie groups with the properties above due to the following theorem. \begin{theorem}[Theorem 21.10. \cite{lee_introduction_2012}]\label{theorem-manifold/action is a manifold} Suppose $G$ is a Lie group acting smoothly, freely and properly on a smooth manifold $M$. Then the orbit space $M/G$ is a topological manifold of dimension equal to ${\rm dim}M- {\rm dim}G$ and has a unique smooth structure with the property that the quotient map $\pi: M \rightarrow M/G$ is a smooth submersion. \end{theorem} The following lemma gives two alternative characterizations of proper actions. \begin{lemma}[Proposition 21.5. \cite{lee_introduction_2012}] \label{proper_equivalence} Let $M$ be a smooth manifold, and let $G$ be a Lie group acting continuously on $M$. The following are equivalent: \begin{itemize} \item[(a)] The action is proper. \item[(b)] If $\{x_i\}$ is a sequence in $M$ and $\{g_i\}$ is a sequence in $G$ such that both $\{x_i\}$ and $\{ g_i \cdot x_i \}$ converge, then a subsequence of $\{g_i\}$ converges. \item[(c)] For every compact subset $K \subseteq M$, the set $G_K=\{g \in G: (g \cdot K) \cap K \not= \emptyset \}$ is compact. \end{itemize} \end{lemma} By Lemma \ref{proper_equivalence}, it is easy to check that if $G$ acts properly on $M$, then a finite subgroup of $G$ also acts properly on $M$. Hence, we have the following corollary. \begin{corollary} \label{corollary-finite subgroup inherited smooth proper free} Suppose a Lie group $G$ acts smoothly, freely and properly on a smooth manifold $M$. If $H$ is a finite subgroup of $G$, then $H$ also acts smoothly, freely and properly on $M$. \end{corollary} \begin{proof} By definition, it is easy to check that $H$ acts freely and smoothly on $M$. Since $H$ is finite, any subset of $H$ is also finite. For any compact set $K \subseteq M$, $H_K\subseteq H$ is finite and therefore is compact. By Lemma \ref{proper_equivalence} (c), $H$ also acts properly. \end{proof} In the end of this subsection, we recall from \cite{wolf_spaces_2011} the characterization of compact group actions on affine spaces. \begin{lemma}[Lemma 3.1.2 \cite{wolf_spaces_2011}] \label{constant_compact affine subgroup has a fixed point_lemma} Let $A(n)$ be the affine group acting on affine spaces $A^n$, and let $B$ be a compact subgroup of $A(n)$. Then $B$ has a fixed point on $A^n$. \end{lemma} \subsection{Riemannian covering} Given a Riemannian covering map $\pi: (\tilde{M}, \tilde{g}) \rightarrow (M,g)$. There is an important Lie group which acts on $\tilde{M}$ and is closely related to $\pi$. \begin{definition}[Covering automorphism groups of Riemannian covering maps] \label{Covering Automorphism Groups} Let $(\tilde{M}, \tilde{g})$ and $(M, g )$ be Riemannian manifolds and $\pi: \tilde{M} \rightarrow M$ be a Riemannian covering map. A \textbf{covering automorphism} (or \textbf{deck transformation}) of $\pi$ is an isomorphism $\varphi: \tilde{M} \rightarrow \tilde{M}$ such that $\pi \circ \varphi = \pi$; the set of all covering automorphisms is a group acting on $\tilde{M}$ on the left, under composition of isomorphisms, called the \textbf{covering automorphism group} (or \textbf{deck transformation group}) and denoted by ${\rm Aut}_{\pi}(\tilde{M})$. \end{definition} Let $(M,g)$ be a Riemannian manifold. An isometry from $(M,g)$ to itself is called an \textbf{isometry} of $(M,g)$. The set of all isometries of $(M,g)$ is a group under compositions, called the \textbf{isometry group } of $(M,g)$, denoted by ${\rm Iso}(M,g)$. We may also denote ${\rm Iso}(M,g)$ by ${\rm Iso}(M)$ for convenience if it will not cause misunderstanding. Given a Riemannian covering map $\pi: (\tilde{M},\tilde{g}) \rightarrow (M,g)$ with $\tilde{g}=\pi^{\ast}g$. Obviously, the covering automorphism group ${\rm Aut}_{\pi}(\tilde{M})$ is a subgroup of the isometry group ${\rm Iso}(\tilde{M},\tilde{g})$. In general, a subgroup of ${\rm Iso}(\tilde{M},\tilde{g})$ may not act smoothly, freely and properly on $\tilde{M}$. However, ${\rm Aut}_{\pi}(\tilde{M})$ does. \begin{proposition}[\cite{lee_introduction_2012,petersen_riemannian_2006}] \label{Aut:smooth+free+proper+isometrical proposition} Let $\pi: (\tilde{M}, \tilde{g}) \rightarrow (M , g )$ be a Riemannian covering map over a Riemannian manifold $M$. ${\rm Aut}_{\pi}(\tilde{M})$ is a discrete Lie group acting smoothly, freely, properly and isometrically on $\tilde{M}$. \end{proposition} ${\rm Aut}_{\pi}(\tilde{M})$ is closely related to the fundamental groups of $\tilde{M}$ and $M$. See the covering automorphism group structure theorem below. \begin{theorem}[Theorem 12.7. \cite{lee_introduction_2011}] \label{Covering Automorphism Group Structure Theorem} Suppose that $\pi : \tilde{M} \rightarrow M$ is a covering map, $\tilde{x} \in \tilde{M}$, and $x= \pi(\tilde{x})$. Then, we have: \begin{equation}\label{formula-structure theorem} {\rm Aut}_{\pi}(\tilde{M}) \cong \frac{ {\rm N}_{\pi_1(M,x)}(\pi_*\pi_1(\tilde{M},\tilde{x})) }{ \pi_*\pi_1(\tilde{M},\tilde{x})}, \end{equation} where $\pi_* : \pi_1(\tilde{M} , \tilde{x}) \rightarrow \pi_1(M, x)$ denotes the induced homomorphism by $\pi$. \end{theorem} ${\rm N}_{\pi_1(M,x)}(\pi_*\pi_1(\tilde{M},\tilde{x}))$ in (\ref{formula-structure theorem}) is the normalizer of $\pi_*\pi_1(\tilde{M},\tilde{x})$ in $\pi_1(M,x)$. If $\pi_1(\tilde{M},\tilde{x})$ is trivial, then ${\rm N}_{\pi_1(M,x)}(\pi_*\pi_1(\tilde{M},\tilde{x}))=\pi_1(M,x)$. Consequently, \begin{theorem}[Corollary 12.9. \cite{lee_introduction_2011}] \label{theorem-structure theorem on universal covering map} Suppose that $\pi : \tilde{M} \rightarrow M$ is a universal covering map, $\tilde{x} \in \tilde{M}$, and $x= \pi(\tilde{x})$. Then, we have: \begin{equation} {\rm Aut}_{\pi}(\tilde{M}) \cong \pi_1(M,x).\notag \end{equation} \end{theorem} As a result of Proposition \ref{Aut:smooth+free+proper+isometrical proposition} and Theorem \ref{theorem-structure theorem on universal covering map}, the following holds. \begin{proposition}[Corollary 2.33. \cite{lee_introduction_2018}]\label{propositon-quotients isometric to universal cover module deck group} Suppose $(M,g)$ and $(\tilde{M}, \tilde{g})$ are Riemannian manifolds, and $\pi : \tilde{M} \rightarrow M$ is a universal Riemannian covering map. Then $M$ is isometric to $\tilde{M}/{\rm Aut}_{\pi}(\tilde{M})$. \end{proposition} \section{The isometry group of the universal cover} Given a steady gradient Ricci soliton $(M,g,f)$. Suppose $(\tilde{M},\tilde{g})$ is the universal cover of $(M,g)$. By Theorem \ref{theorem-structure theorem on universal covering map}, \begin{align} \pi_1(M,x) \cong {\rm Aut}_{\pi}(\tilde{M})\subseteq {\rm Iso}(\tilde{M},\tilde{g})\notag \end{align} So, to figure out the structure of $\pi_1(M,x)$, we need to study ${\rm Iso}(\tilde{M},\tilde{g})$. In \cite{guan_rigidity_nodate}, Guan-Lv-Xu prove a splitting theorem for gradient Ricci solitons with nonnegative sectional curvature. \begin{theorem}[Theorem 1.1. \cite{guan_rigidity_nodate}] \label{split theorem_universal} Let $(M^n ,g ,f)$ be a gradient Ricci soliton. If $g$ has nonnegative sectional curvature, then the rank of the Ricci curvature is constant. Thus, either the Ricci curvature is strictly positive or the universal covering $(\tilde{M}, \tilde{g}) = (N^k,h) \times \mathbb{R}^{n-k}$ splits isometrically and $(N^k, h)$ has strictly positive Ricci curvature. \end{theorem} We need to study the relations of ${\rm Iso}(N,g)$, ${\rm Iso}(\mathbb{R}^k)$ and ${\rm Iso}(\tilde{M} , \tilde{g})$. We first recall the following notion of holonomy group. \begin{definition}[Riemannian holonomy groups\cite{joyce_riemannian_nodate}] \label{holonomy_group} Let $(M,g)$ be a Riemannian manifold of dimension $n$, $TM$ be the tangent bundle over $M$ and $\nabla$ be the Levi-Civita connection of $g$ on $TM$. Fix a point $x \in M$. Given a loop $\gamma$ based at $x$ (i.e. a piecewise smooth path $\gamma: [0,1]\rightarrow M$ with $\gamma(0 ) =\gamma(1) =x$), we let $P_{\gamma}$ denote the parallel transportation of $\nabla$ along $\gamma$ and $T_xM$ denote the fiber over $x$. Then, $P_{\gamma} : T_xM \rightarrow T_xM$ is an invertible linear transformation, hence an element of $GL(T_xM) \cong GL(n, \mathbb{R})$. The \textbf{Riemannian holonomy group of $\mathbf{(M,g)}$ based at $\mathbf{x}$} is defined by \begin{equation} {\rm Hol}_x(g) := \{ P_{\gamma} : \gamma\text{ is a loop based at }x \} \subseteq GL(T_xM). \end{equation} \end{definition} The definition of canonical decompositions of the tangent space $T_xM$\cite{kobayashi_foundations_1996} is given as follows. \begin{definition}[Canonical decompositions] With the condition of Definition \ref{holonomy_group}, let $T^{(0)}_x$ be the set of elements in $T_xM$ which are left fixed by ${\rm Hol}_x(g)$. It is the maximal linear subspace of $T_xM$ on which ${\rm Hol}_x(g)$ acts trivially. Let $T_x'$ be the orthonormal complement of $T^{(0)}_x$ in $T_xM$. It is invariant by ${\rm Hol}_x(g)$ and can be decomposed into a direct sum $T_x' = \sum_{i=1}^{k} T^{(i)}_x$ of mutually orthogonal, invariant and irreducible subspaces. We shall call $T_xM= \sum_{i=0}^{k} T_x^{(i)}$ a \textbf{canonical decomposition} (or \textbf{de Rham decomposition}) of $T_xM$. \end{definition} Let $T^{(i)}$ be the involutive distribution on $M$ obtained by parallel transformation of $T_x^{(i)}$ for each $i=1,2,\cdots,k$. Theorem 5.4 of Chapter IV in \cite{kobayashi_foundations_1996} states that the maximal integral manifold $M_0$ of $T^{(0)}$ through a fixed point is locally Euclidean. Moreover, if $M$ is simply connected, then a canonical decomposition $T_xM= \sum_{i=0}^{k} T_x^{(i)}$ is unique up to an order. Subsequently, the de Rham decomposition theorem shows that if $M$ is simply connected and complete, then $M$ is isometric to the direct product of the maximal integral manifolds of the parallel distributions of $T^{(i)},i=1,2,\cdots,k$, respectively, see \cite{de_rham_sur_1952,kobayashi_foundations_1996}. \begin{lemma}[de Rham] Let $(M,g)$ be a simply connected and complete Riemannian manifold. Then, up to isometries and permutations of factors, there exists a unique decomposition as a direct product \begin{equation} (M,g) = \prod_{i=0}^{k}(M_i,g_i), \end{equation} where $(M_0, g_0)$ is a Euclidean space (possibly a point) and all $(M_i, g_i)$, $i=1,\cdots,k,$ are simply connected, complete and irreducible Riemannian manifolds. \label{sdecomposition} \end{lemma} Given the de Rham decomposition of $(M,g)$, Hano has proved the following relation between the isometry group of $M$ and those of $M_i$. \begin{theorem}[Theorem 1 \cite{Hano-1955}]\label{theorem-decomposition of isometry group-identity components} Let $(M,g)$ be a simply connected complete Riemannian manifold and $(M,g)=(M_1,g_1)\times(M_2,g_2)\times\cdots\times(M_k,g_k)$ be the de Rham decomposition of $(M,g)$. Then, \begin{align}\label{decomposition of isometry group} {\rm Iso}_{0}(M,g)\cong {\rm Iso}_{0}(M_1,g_1)\times {\rm Iso}_{0}(M_2,g_2)\times\cdots\times {\rm Iso}_{0}(M_k,g_k), \end{align} where ${\rm Iso}_{0}(M,g)$ and ${\rm Iso}_{0}(M_i,g_i)$ are the connected components of the identity in ${\rm Iso}(M,g)$ and ${\rm Iso}(M_i,g_i)$ respectively. \end{theorem} In general, ${\rm Iso}_0(M,g)$ and ${\rm Iso}_0(M_i,g_i)$ in (\ref{decomposition of isometry group}) can not be replaced by ${\rm Iso}(M,g)$ and ${\rm Iso}(M_i,g_i)$. For example, given any irreducible Riemannian manifold $(M,g)$, ${\rm Iso}(M,g)\times {\rm Iso}(M,g)$ is a proper subgroup of ${\rm Iso}(M\times M,g\oplus g)$. In view of Theorem \ref{split theorem_universal}, we can obtain the following decomposition result due to the difference between the curvatures on $N^k$ and $\mathbb{R}^{n-k}$. \begin{proposition} Suppose that $N$ is a simply connected complete Riemannian manifold with positive Ricci curvature. Then, we have ${\rm Iso} (N \times \mathbb{R}^n) ={\rm Iso} ( N ) \times {\rm Iso} ( \mathbb{R}^n)$ for $n\in \mathbb{N}$. \label{proposition-isometry separately N R^n} \end{proposition} \begin{proof} Let $\varphi: N \times \mathbb{R}^n \rightarrow N \times \mathbb{R}^n $ be an isometry of $N \times \mathbb{R}^n $. Fix a point $ (x, y) \in N \times \mathbb{R}^n $ and consider the differential of $\varphi$: \begin{equation} d \varphi_{(x, y)}: T_{(x, y)}(N \times \mathbb{R}^n ) \cong T_{x} N \oplus T_{y} \mathbb{R}^n \rightarrow T_{\left(x^{\prime}, y^{\prime}\right)}(N \times \mathbb{R}^n ) \cong T_{x^{\prime}} N \oplus T_{y^{\prime}} \mathbb{R}^n, \end{equation} where $\left(x^{\prime}, y^{\prime}\right)=\varphi(x, y) $. \begin{claim} $d\varphi$ sends $T_{x} N $ to $T_{x'} N $ and $T_{y} \mathbb{R}^n $ to $T_{y'} \mathbb{R}^n $. \label{isometrysep} \end{claim} \begin{proof}[Proof of claim \ref{isometrysep}] Recall from Proposition 2.5 of Chapter IV in \cite{kobayashi_foundations_1996} that if $\varphi$ is an isometry of $N \times \mathbb{R}^n$, then the differential of $\varphi$ commutes with parallel transports. More precisely, if $\gamma $ is a loop based at $(x,y)$ in $N \times \mathbb{R}^n$, then the following diagram is commutative: \begin{equation} \begin{tikzcd} T_{(x,y)}(N \times \mathbb{R}^n) \arrow[d,"d\varphi"] \arrow[r,"\gamma"] & T_{(x,y)}(N \times \mathbb{R}^n) \arrow[d,"d\varphi"]\\ T_{(x',y')}(N \times \mathbb{R}^n) \arrow[r,"\gamma'"] & T_{(x',y')}(N \times \mathbb{R}^n), \end{tikzcd} \end{equation} where $\gamma' = \varphi(\gamma)$. Regarding this, $d\varphi$ sends a canonical decomposition of $T_{(x,y)}( N \times \mathbb{R}^n)$ to a canonical decomposition of $T_{\varphi(x,y)}( N \times \mathbb{R}^n)$. Note that sectional curvatures are invariant under isometries. Since $N$ has positive Ricci curvature and lemma \ref{sdecomposition}, $d\varphi$ sends $T_{x} N $ to $T_{x'} N $ and $T_{y} \mathbb{R}^n $ to $T_{y'} \mathbb{R}^n $ separately. \end{proof} As an isometry maps geodesics to geodesics, $\varphi$ sends totally geodesic submanifolds $N \times\{y\} $ to $ N \times\left\{y^{\prime}\right\} $ and $\{x\} \times \mathbb{R}^n$ to $\left\{x^{\prime}\right\} \times \mathbb{R}^n $. Suppose $\pi_{N}$ and $\pi_{\mathbb{R}^n}$ are the projection from $N \times \mathbb{R}^n$ onto $N$ and $\mathbb{R}^n$, respectively. We define $\varphi_1 : N \rightarrow N$ by $\varphi_1(u)= \pi_N \circ \varphi(u,y)$ and $\varphi_2 : \mathbb{R}^n \rightarrow \mathbb{R}^n$ by $\varphi_2(v)= \pi_{\mathbb{R}^n} \circ \varphi(x,v)$, where $u \in N$ and $v \in \mathbb{R}^n$. Considering that $\varphi_{1} \in {\rm Iso} (N)$ and $\varphi_{2} \in {\rm Iso} (\mathbb{R}^n) $, $ \left(\varphi_{1}, \varphi_{2}\right)$ is a product isometry of $N \times \mathbb{R}^n $. Isometries $\varphi$ and $\left(\varphi_{1}, \varphi_{2}\right)$ satisfy $\varphi(x,y) = \left(\varphi_{1}, \varphi_{2}\right)(x,y)$ and $d\varphi_{(x,y)} = d\left(\varphi_{1}, \varphi_{2}\right)_{(x,y)}$. Hence they coincide globally and we complete the proof. \end{proof} \section{Steady gradient Ricci solitons with nonnegative sectional curvature and positive Ricci curvature} In this section, we will study steady gradient Ricci solitons with nonnegative sectional curvature and positive Ricci curvature. If the scalar curvature attains its maximum and the Ricci curvature is positive, then $M^n$ is diffeomorphic to $\mathbb{R}^n$\cite{Cao_chen_2012}. On a steady gradient Ricci soliton, the scalar curvature attains its maximum if and only if the potential $f$ has a critical point $x_0$, i.e., $|\nabla f|(x_0)=0$. It is still unknown whether there exists a critical point on $M$, even if we assume the sectional curvature is nonnegative. Given a steady gradient Ricci soliton $(M^n,g,f)$, denote the level set of a function $f$ on $M$, by $M_s :=\{ x\in M: f(x) =s\}$. When $M_s$ is compact, we show the existence of the critical point in subsection \ref{subsection-compact}. When $M_s$ is noncompact, we will show that the level set is diffeomorphic to $\mathbb{R}^{n-1}$ in subsection \ref{subsection-noncompact}. We will also show that a steady gradient Ricci soliton with nonnegative sectional curvature and positive Ricci curvature has only trivial quotients in subsection \ref{subsection-quotient}. \subsection{Compact level sets of \textit{f}}\label{subsection-compact} Counting the ends of a noncompact manifold is an important problem in the study of noncompact manifolds. For the definition of ends, one may refer to\cite{chow_ricci_2023}. In \cite{gromoll_complete_1969}, Gromoll and Meyer have proved that a complete open manifold $M$ of positive Ricci curvature has only one end. Munteanu and Wang generalize Gromoll and Meyer's results on steady gradient Ricci solitons. They prove that a steady gradient Ricci soliton is either connected at infinity or isometric to $N\times \mathbb{R}$, where $N$ is a compact Ricci flat manifold (See Theorem 4.2 in \cite{Munteanu_wang_2011}). This result gives a strong restriction of the topology of steady gradient Ricci solitons. \begin{lemma}\label{lemma-existence of critical point} Suppose $(M,g,f)$ is an $n$-dimensional complete steady gradient Ricci soliton. Assume that $f$ has a compact level set $M_s$ for some $s \in f(M)$. Then, there exists a point $x \in M$, such that $|\nabla f|(x) = 0$. \end{lemma} \begin{proof} We prove this by contradiction. Suppose $|\nabla f|(x)\neq0$, $\forall~x\in M$. Then, $f$ must be an open map. Hence, $f(M)$ is a connected and open subset of $\mathbb{R}$. We may assume that $f(M)=(a,b)$, where $-\infty\le a<b\le +\infty$. Note that $s\in (a,b)$ and $|\nabla f|$ is not vanishing everywhere. Then, $M$ is diffeomorphic to $(a,b)\times M_s$. Hence, $M$ has two ends when $M_s$ is compact. It contradicts Theorem 4.2 in \cite{Munteanu_wang_2011}. \end{proof} By Lemma \ref{lemma-existence of critical point}, we can show that $M$ is diffeomorphic to the Euclidean space when the level set is compact. \begin{theorem}\label{theorem-level set compact} Suppose $(M,g,f)$ is an $n$-dimensional complete steady gradient Ricci soliton with positive Ricci curvature. If $M_s$ is compact for some $s \in f(M)$, then, $M$ is diffeomorphic to $\mathbb{R}^n$. \end{theorem} \begin{proof} Hamilton\cite{hamilton_formation_1995} has proved that a steady gradient Ricci soliton satisfies \begin{equation}\label{identity-hamilton} R+|\nabla f|^2 = C, \end{equation} where $C$ is a constant. Here $R$ denotes the scalar curvature. By lemma \ref{lemma-existence of critical point}, $f$ has a critical point $x_0$. So, the scalar curvature attains its maximum at point $x_0$ by (\ref{identity-hamilton}). By Proposition 2.3 in \cite{Cao_chen_2012}, $M$ is diffeomorphic to $\mathbb{R}^n$. \end{proof} \subsection{Noncompact level sets of \textit{f}}\label{subsection-noncompact} According to the above, if an $n$-dimensional complete steady gradient Ricci soliton $(M,g,f)$ with positive Ricci curvature has no critical point, then all level sets of $f$ are noncompact. \begin{theorem}\label{theorem-noncompact level set} Suppose $(M,g,f)$ is an $n$-dimensional complete steady gradient Ricci soliton with nonnegative sectional curvature and positive Ricci curvature. If $M_s$ is noncompact for some $s \in f(M)$, then $M$ is diffeomorphic to $\mathbb{R}^n$. \end{theorem} \begin{proof} If there is a point $x_0 \in M$ such that $|\nabla f|(x_0)=0$, then we can show that $M$ is diffeomorphic to $\mathbb{R}^n$ by the same argument as in the proof of Theorem \ref{theorem-level set compact}. Now, we assume that $|\nabla f|$ is not vanishing everywhere. Note that $M_s$ is a smooth manifold due to the regular value theorem for any $s \in f(M)$. Then, we follow the computation in the Lemma 2.1 of \cite{chan_dimension_2023}. The second fundamental form ${\rm II}$ of $M_s$ is determined by \begin{equation} {\rm II}(X, Y ) := <\nabla_X {\rm n},Y> = \frac{1 }{|\nabla f |}{\rm Ric}(X, Y ) \label{II>0} \end{equation} for any $X, Y$ tangent to $M_s$, where ${\rm n}= -\frac{\nabla f}{|\nabla f|}$. Then ${\rm II}>0$. Moreover, \begin{equation} {\rm sec}_{M_s} (X, Y ) = {\rm sec}_{M}(X, Y ) + {\rm II}(X, X){\rm II}(Y, Y )-({\rm II}(X, Y ))^2 > 0, \end{equation} where ${\rm sec}$ denotes sectional curvature, and $X,Y$ are unit vectors tangent to $M_s$ with $X \perp Y$. More precisely, $M_s$ is an $(n-1)$-manifold with positive sectional curvature. Since Lemma \ref{lemma-existence of critical point} implies that all level sets of $f$ are noncompact, $M_s$ is diffeomorphic to an Euclidean space. Hence, we obtain that $M$ is diffeomorphic to $\mathbb{R}^n$ followed by the same argument as in the proof of Lemma \ref{lemma-existence of critical point}. \end{proof} As a corollary of Theorem \ref{theorem-level set compact} and Theorem \ref{theorem-noncompact level set}, we obtain the following main result of this section. \begin{corollary}\label{corollary-Ricci positive case is simply connected} Suppose $(M,g,f)$ is an $n$-dimensional complete steady gradient Ricci soliton with nonnegative sectional curvature and positive Ricci curvature. Then, $M$ is diffeomorphic to $\mathbb{R}^n$. \end{corollary} \begin{remark} For $n=4$, Theorem \ref{theorem-noncompact level set} and Corollary \ref{corollary-Ricci positive case is simply connected} still hold if we only assume $(M,g,f)$ has positive Ricci curvature. Note that any $3$-dimensional noncompact manifold with positive Ricci curvature is diffeomorphic to $\mathbb{R}^3$(cf.\cite{Schoen-Yau-1982,Liu-2013} ). So, the argument in Theorem \ref{theorem-noncompact level set} and Corollary \ref{corollary-Ricci positive case is simply connected} remains true. \end{remark} \subsection{Quotients of steady gradient Ricci solitons}\label{subsection-quotient} To prove Theorem \ref{MAIN}, we still need to study the quotients of steady gradient Ricci soltions with nonnegative sectional curvature and positive Ricci curvature. \begin{theorem}\label{theorem-symmetry of f} Let $(M,g,f)$ be a complete gradient Ricci soliton. Suppose $G$ is a finite subgroup of ${\rm Iso}(M,g)$. Then, one of the following holds: (1) $(M,g)$ locally splits off a line; (2) $f(x)=f(\phi(x))$ for all $x\in M$ and $\phi\in G$. \end{theorem} \begin{proof} Let $\phi\in G$ and $V_1,V_2\in T_xM$ for some $x\in M$. Assume $y=\phi(x)$. By the Ricci soliton equation and $\phi\in {\rm Iso}(M,g)$, we have \begin{align} -\nabla_{V_1}\nabla_{V_2}f|_x=&{\rm Ric}(V_1,V_2)|_x-\frac{\lambda}{2}g(V_1,V_2)|_x\notag\\ =&{\rm Ric}_{\phi^{\ast}g}(V_1,V_2)|_x-\frac{\lambda}{2}g_{\phi^{\ast}g}(V_1,V_2)|_x\notag\\ =&{\rm Ric}(\phi_{\ast}(V_1),\phi_{\ast}(V_2))|_y-\frac{\lambda}{2}g(\phi_{\ast}(V_1),\phi_{\ast}(V_2))|_y\notag\\ =&-\nabla_{\phi_{\ast}(V_1)}\nabla_{\phi_{\ast}(V_2)}f|_y\notag\\ =&-\bar{\nabla}_{V_1}\bar{\nabla}_{V_2}(f\circ\phi)|_x\notag\\ =&-\nabla_{V_1}\nabla_{V_2}(f\circ\phi)|_x\label{Parallel-vector-field-1} \end{align} where $\bar{\nabla}$ is the gradient with respect to the metric $\phi^{\ast}g$. By $(\ref{Parallel-vector-field-1})$, we have \begin{align} \nabla(\nabla f-\nabla (f\circ \phi))\equiv0.\notag \end{align} Let $V=\nabla f-\nabla (f\circ \phi)$. Then, $V$ is a parallel vector field by the equation above. If $V$ is nonzero, then $(M,g)$ locally splits off a line. If $V\equiv0$, then there is a constant $c$ such that \begin{align} f(x)-f(\phi(x))=c,~\forall~x\in M.\label{Parallel-vector-field-2} \end{align} Since $\phi\in G$ and $G$ is a finite group, then there is a positive integer $k$ such that $\phi^{k}=Id$. Hence, for any fixed point $x\in M$, \begin{align} kc=\sum_{i=0}^{k-1}[f(\phi^{i}(x))-f(\phi^{i+1}(x))]=f(x)-f(\phi^{k}(x))=0.\notag \end{align} It follows that \begin{align} c=0.\label{Parallel-vector-field-3} \end{align} By (\ref{Parallel-vector-field-2}) and (\ref{Parallel-vector-field-3}), we get \begin{align} f(x)=f(\phi(x)),~\forall~x\in M. \end{align} We complete the proof. \end{proof} \begin{remark} In Theorem \ref{theorem-symmetry of f}, we assume that $G$ is a finite subgroup of ${\rm Iso}(M,g)$. If the isomorphism $\phi$ is related to a killing vector field, one may refer to \cite{petersen-gradient-2009}. \end{remark} As an application of Theorem \ref{theorem-symmetry of f} and Corollary \ref{corollary-Ricci positive case is simply connected}, we show that steady gradient Ricci solitons with nonnegative sectional curvature and positive Ricci curvature have no finite quotients. \begin{corollary} \label{corollary-sec>=0+Ric>0+finite =>simplyconnected} Let $(M,g,f)$ be a complete steady gradient Ricci soliton with nonnegative sectional curvature and positive Ricci curvature. Suppose $(N,h)$ is a finite quotient of $(M,g)$. Then, $N$ is simply connected, i.e., $(N,h)$ is isometric to $(M,g)$. \end{corollary} \begin{proof} By Corollary \ref{corollary-Ricci positive case is simply connected}, $M$ is simply connected. Hence, $M$ is a universal cover of $N$. Let $\pi: M\rightarrow N$ be the covering map. By Proposition \ref{propositon-quotients isometric to universal cover module deck group}, $N$ is isometric to $M/{\rm Aut}_{\pi}(M)$. Note that ${\rm Aut}_{\pi}(M)$ is a finite group by assumption. Recall that $(M,g,f)$ can not locally split off a line by Theorem \ref{split theorem_universal}. Therefore, for any $\phi\in {\rm Aut}_{\pi}(M)$, we have $f=f\circ\phi$ on $M$ by Theorem \ref{theorem-symmetry of f}. Hence, we can define a smooth function $\bar{f}$ on $N$ such that $f=\pi\circ \bar{f}$. Since $\pi$ is a local isomorphism, one can check that $(N,h,\bar{f})$ is a steady gradient Ricci soliton with nonnegative sectional curvature and positive Ricci curvature. By Corollary \ref{corollary-Ricci positive case is simply connected}, $N$ is simply connected. Hence, we complete the proof. \end{proof} \section{The Proof of Theorem \ref{MAIN}} Now we prove the main theorem. \begin{proof}[Proof of Theorem \ref{MAIN}] It suffices to consider the case that $\pi_1(M)$ is finite. Let $(\tilde{M} , \tilde{g},\tilde{f})$ be the universal cover of $(M,g,f)$ and $\pi : (\tilde{M} , \tilde{g}) \rightarrow (M,g)$ be the covering map. By Theorem \ref{split theorem_universal}, either $(M,g,f)$ has strictly positive Ricci curvature or the universal cover $(\tilde{M}, \tilde{g}) = (N ,h ) \times \mathbb{R}^k$ splits isometrically, where $(N , h)$ has strictly positive Ricci curvature and $k \ge 1$. Due to Lemma 2.1 in \cite{petersen-gradient-2009}, $f$ can also split so that $N$ and $\mathbb{R}^k$ are both steady gradient Ricci solitons. If $M$ has strictly positive Ricci curvature, then $\pi_1(M)$ is trivial due to Corollary \ref{corollary-Ricci positive case is simply connected}. Now. it remains to consider the case $k\ge 1$. By Proposition \ref{proposition-isometry separately N R^n}, \begin{align} {\rm Aut}_{\pi} (\tilde{M})\subseteq {\rm Iso} (N \times \mathbb{R}^k) = {\rm Iso} ( N ) \times {\rm Iso} ( \mathbb{R}^k)\notag \end{align} For any $\phi \in {\rm Aut}_{\pi} (\tilde{M})$, we can assume that $\phi = (\phi_1 ,\phi_2)$, where $\phi_1 \in {\rm Iso}(N)$ and $\phi_2 \in {\rm Iso}(\mathbb{R}^k)$. By Theorem \ref{theorem-structure theorem on universal covering map}, the fundamental group $\pi_1(M) \cong {\rm Aut}_{\pi} (\tilde{M})$. Since $\pi_1(M)$ is finite, we see that ${\rm Aut}_{\pi} (\tilde{M})$ is also finite. It follows that $\phi$, $\phi_1$ and $\phi_2$ all have finite orders. Let $G_{\phi}=\{\phi^i|i\in\mathbb{N}\}$. Then, $G_{\phi}$ is a finite subgroup of ${\rm Aut}_{\pi} (\tilde{M})$. By Corollary \ref{corollary-finite subgroup inherited smooth proper free} and Proposition \ref{Aut:smooth+free+proper+isometrical proposition}, $G_{\phi}$ acts smoothly, freely, properly and isometrically on $\tilde{M}$. Similarly, let $G_{\phi_2}=\{\phi_2^i|i\in\mathbb{N}\}$. Since $\phi_2$ has a finite order, $G_{\phi_2}$ is a finite subgroup of ${\rm Iso}(\mathbb{R}^k)$. So, $G_{\phi_2}$ is also compact. By Lemma \ref{constant_compact affine subgroup has a fixed point_lemma}, $G_{\phi_2}$ has at least one fixed point. Therefore, $\phi_2^i$ has at least one fixed point for all $i\in\mathbb{N}$. We also define $G_{\phi_1}=\{\phi_1^i|i\in\mathbb{N}\}$. Then, $G_{\phi_1}$ is a finite subgroup of ${\rm Iso}(N)$. Let the order of $\phi$ be $r$. We want to show that $r=1$. Suppose $r\ge 2$. For $0< i<r$, $\phi^i$ has no fixed point on $N\times \mathbb{R}^k$ since $G_{\phi}$ is a free action. Note that $\phi^i=(\phi^i_1,\phi_2^i)$ and $\phi_2^i$ has at least a fixed point on $\mathbb{R}^k$. So, $\phi^i_1$ has no fixed point on $N$ for all $0<i<r$. Let the order of $\phi_1$ be $r_1$. Then, $r_1|r$ and therefore $r_1\le r$. Note that $\phi_1^i$ has no fixed point and cannot be the identity map for $0<i<r$. Hence, $r=r_1$. It follows that $\phi^i_1$ has no fixed point on $N$ for all $0<i<r_1$. Then, $G_{\phi_1}$ is a free action on $N$ by definition. By Corollary \ref{corollary-finite subgroup inherited smooth proper free}, $G_{\phi_1}$ acts freely and properly on $N$. By Theorem \ref{theorem-manifold/action is a manifold}, $N/G_{\phi_1}$ is a quotient of $N$. However, $G_{\phi_1}$ is trivial by Corollary \ref{corollary-sec>=0+Ric>0+finite =>simplyconnected}. It contradicts the fact that $\phi_1^i$ has no fixed point for $0<i<r$. Hence, $r=1$ and $\phi$ is the identity map. Since $\phi$ is an arbitrary isomorphism in ${\rm Aut}_{\pi} (\tilde{M})$, we obtain that $\pi_1(M)\cong {\rm Aut}_{\pi} (\tilde{M})=\{{\rm Id}_{\tilde{M}}\}$. Now we are left to show that $M$ is diffeomorphic to $\mathbb{R}^n$. We note that $(N,h)$ is a steady gradient Ricci soliton with nonnegative sectional curvature and positive Ricci curvature. Hence, $N$ is diffeomorphic to $\mathbb{R}^{n-k}$. Then, $\tilde{M}$ is diffeomorphic to $\mathbb{R}^n$. Since $\pi_1(M)$ is trivial, $M$ is diffeomorphic to $\tilde{M}$ and therefore is also diffeomorphic to $\mathbb{R}^n$. Hence we complete the proof. \end{proof} \begin{spacing}{1.0} \bibliographystyle{plain} \bibliography{simplyconnected_Reference.bib} \end{spacing} {\footnotesize YUXING DENG, SCHOOL OF MATHEMATICS AND STATISTICS, BEIJING INSTITUTE OF TECHNOLOGY, BEIJING,100081, CHINA, [email protected]} {\footnotesize YUEHAN HAO, SCHOOL OF MATHEMATICS AND STATISTICS, BEIJING INSTITUTE OF TECHNOLOGY, BEIJING,100081, CHINA, [email protected]} \end{document}
2412.07510v1
http://arxiv.org/abs/2412.07510v1
Roman domination number of zero-divisor graphs over commutative rings
\begin{filecontents*}{example.eps} gsave newpath 20 20 moveto 20 220 lineto 220 220 lineto 220 20 lineto closepath 2 setlinewidth gsave .4 setgray fill grestore stroke grestore \end{filecontents*} \RequirePackage{fix-cm} \documentclass[11pt]{svjour3} \usepackage{amssymb} \usepackage{amsmath, mathtools} \usepackage[11pt]{extsizes} \smartqed \usepackage{booktabs,caption} \usepackage{tikz} \usepackage{calc} \usetikzlibrary{decorations.markings} \tikzstyle{vertex}=[circle, draw, inner sep=0pt, minimum size=1pt] \newcommand{\vertex}{\node[vertex]} \newcounter{Angle} \usepackage{graphicx} \usepackage{pdflscape} \usepackage{geometry} \geometry{ a4paper, total={210mm,297mm}, left=30mm, right=30mm, top=25mm, bottom=25mm, } \numberwithin{equation}{section} \usepackage{amsfonts} \usepackage{xcolor} \newcommand{\overbar}[1]{\mkern 1.5mu\overline{\mkern-1.5mu#1\mkern-1.5mu}\mkern 1.5mu} \journalname{Indian J. Pure Appl. Math.} \begin{document} \title{Roman domination number of zero-divisor graphs over commutative rings} \titlerunning{Roman domination number of zero-divisor graphs over commutative rings} \author{Ravindra Kumar$^1$\and Om Prakash$^{*1}$ } \authorrunning{R. Kumar and O. Prakash} \institute{\at $^1$ Department of Mathematics\\ Indian Institute of Technology Patna, Patna 801 106, India \\ \email{[email protected](*corresponding author), [email protected] } } \date{Received: date / Accepted: date} \maketitle \begin{abstract} For a graph $G= (V, E)$, a Roman dominating function is a map $f : V \rightarrow \{0, 1, 2\}$ satisfies the property that if $f(v) = 0$, then $v$ must have adjacent to at least one vertex $u$ such that $f(u)= 2$. The weight of a Roman dominating function $f$ is the value $f(V)= \Sigma_{u \in V} f(u)$, and the minimum weight of a Roman dominating function on $G$ is called the Roman domination number of $G$, denoted by $\gamma_R(G)$. The main focus of this paper is to study the Roman domination number of zero-divisor graph $\Gamma(R)$ and find the bounds of the Roman domination number of $T(\Gamma(R))$. \keywords{Commutative ring \and Roman domination number \and Total graph \and Zero divisor graph.} \subclass{13M99 \and 05C25} \end{abstract} \section{Introduction} Let $R$ be a commutative ring with unity and $Z(R)$ be the set of zero-divisors of $R$. The zero-divisor graph of $R$, denoted by $\Gamma(R)$, is a graph with set of vertices $Z(R)- \{0\}$ such that there is an edge (undirected) between the vertices $x, y \in V(\Gamma(R))$ if and only if $xy = 0$. It is noted that $\Gamma(R)$ is an empty graph if and only if $R$ is an integral domain.\\ \indent The concept of the zero-divisor graph was introduced by Beck in \cite{beck} in 1988. Later, Anderson and Livingston \cite{ander} redefined Beck's definition in 1999 and established several fundamental results on $\Gamma(R)$. Consequently, in the last four decades, plenty of works have been reported by several researchers, a few are \cite{akbari1,akbari,ander,beck,kumar,kumar1}. Further, in $2002$, Redmond \cite{redmond} extended the study of zero-divisor graph for noncommutative rings. He defined an undirected zero-divisor graph $\Gamma(R)$ of a noncommutative ring $R$ with set of vertices $Z(R)^* = Z(R) \setminus \{0\}$ and for distinct vertices $a$ and $b$, there is an edge between them if and only if either $ab= 0$ or $ba= 0$. \par On the other hand, the concept of the Roman domination was motivated by the defence strategies used to defend the Roman empire during the reign of Emperor Constantine the great $274-337$ AD. There were mainly eight region from Asia minor to Britain of Roman empire at the time of Constantine. To defend all the region by the four groups of legions, he imposed the certain rules. He ordered that for all cities of the Roman empire, at most two group of legions should be stationed under following conditions. \begin{itemize} \item {A region is securable if a group of legion can be moved to it in a single step from an adjacent region.} \item {At least two group of legions must occupy a region before a group of legion can move out of it (i.e., at least one group of legion must remain behind).} \end{itemize} Based on the above conditions of the Roman Empire, presently we have the mathematical concept of Roman domination. It is initially defined and discussed by Stewart \cite{stewart} in $1999$, and later by ReVelle and Rosing \cite{revelle} in $2000$. The proper definition of Roman domination was introduced by Cockayne et al. \cite{cockayne} in 2004. After that several works have been reported on various aspects of Roman domination in the graph, including generalizations such as weak Roman domination \cite{henning}, double Roman domination \cite{ahangar, beeler}. \par A Roman dominating function on a graph $G= (V, E)$ is a function $f : V \rightarrow \{0, 1, 2\}$ with the property that every vertex $u \in V$ for which $f(u) = 0$ is adjacent to at least one vertex $v \in V$ for which $f(v)= 2$. The weight of a Roman dominating function is the value $f(V)= \Sigma_{u \in V} f(u)$. The Roman domination number of a graph $G$, denoted by $\gamma_R(G)$, is the minimum weight of an Roman dominating function on a graph $G$. Further, let $G= (V, E)$ be a graph with $f : V \rightarrow \{0, 1, 2\},$ a function and $V_0, V_1, V_2$ be the ordered partition of $V$ induced by $f$, where $V_i= \{v \in V \vert ~ f(v)= i \}$ and $\lvert V_i \rvert = n_i$, for $i = 0, 1, 2$. It is noted that there exists a one-one correspondence between the function $f : V \rightarrow \{0, 1, 2\}$ and the ordered partition $V_0, V_1, V_2$ of $V$. Therefore, it can be represented as $f = (V_0, V_1, V_2)$. A function $f = (V_0, V_1, V_2)$ is a Roman dominating function (RDF) if the set $V_2$ dominates the set $V_0,$ i.e., $V_0 \subseteq N[V_2]$. A function $f = (V_0, V_1, V_2)$ is said to be a $\gamma_R$-function if it is an RDF and $f(V)= \gamma_R(G)$. \par Now, we recall some definitions and notations that will be used throughout this paper. Let $G = (V, E)$ be a graph of order $n$. The open neighbourhood of any vertex $v \in V$ is the set $N(v) = \{u \in V \vert uv \in E\}$ and closed neighbourhood is the set $N[V] = N(v) \bigcup \{v\}$. The open neighbourhood of a subset $S$ of $V$ is $N(S) = \bigcup_{v \in S}N(v)$ and the closed neighbourhood is $N[S] = N(S) \bigcup S$. A set $S \subseteq V$ is called a dominating set if every vertex of $V$ is either in $S$ or adjacent to at least one vertex in $S$. The domination number $\gamma(G)$ of a graph $G$ is the minimum cardinality among the dominating sets of $G$. A graph $G$ of order $n$ is said to be complete if every vertex in $G$ is adjacent to every other vertex in $G$ and it is denoted by $K_n$. A graph is said to be regular or $k$-regular if all its vertices have the same degree $k$. Also, a graph $G=(V, E)$ is called a bipartite graph if its vertex set $V$ can be partitioned into two subsets $V_1$ and $V_2$ such that each edge of $G$ has one end vertex in $V_1$ and another end vertex in $V_2$. It is denoted by $K_{m,n}$ where $m$ and $n$ are the numbers of vertices in $V_1$ and $V_2$, respectively. A complete bipartite graph of the form $K_{1,n}$ is called a star graph. For more basic definitions and results on graph theory, we may refer \cite{bala}. \par Section $2$ contains some basic results on Roman domination graph. In section $3$, we present Roman domination number of a zero-divisor graph $\Gamma(R)$ for $R = R_1 \times R_2$ for different diameters of $R_1$ and $R_2$ and later we generalized it for $R= R_1 \times R_2 \times ... \times R_n$. In section $4$, we present lower and upper bounds for the Roman domination number of $T(\Gamma(R))$. Section 5 concludes the work. \section{Basic Results} We start this section with several classes of graphs with well-known Roman domination numbers and their straightforward calculations. \par It is easy to see that for a complete graph $K_n$, $\gamma_R(K_n) = 2$. Let $G$ be a complete $r-$ partite graph $(r\geq 2)$ with partite set $V_1, V_2,...,V_r$ such that $\lvert V_i \rvert > 2$ for $1\leq i\leq r$. Then $\gamma_R(G) = 4$. If $\lvert V_i \rvert = 2$ for some $i$, then $\gamma_R(G) = 3$ because one vertex of that set assigned $2$ and another vertex is assigned $1$. If $\lvert V_i \rvert = 1$ for some $i$, then $\gamma_R(G) = 2$. Hence, we can say that Roman domination number of any star graph is $2$ and bistar graph is $3$.\\ \begin{example} Consider a ring $R = \mathbb{Z}_{25}$. The graph of $\Gamma(\mathbb{Z}_{25})$ is shown in figure $1$. \[\begin{tikzpicture} \vertex (A) at (-1.5,1.5) [label=left:${5}$]{}; \vertex (B) at (0.5,1.5) [label=right:${10}$]{}; \vertex (C) at (0.5,-0.5) [label=right:${15}$]{}; \vertex (D) at (-1.5,-0.5) [label=left:${20}$]{}; \path (A) edge (B) (B) edge (C) (C) edge (D) (D) edge (A) (A) edge (C) (B) edge (D) ; \end{tikzpicture}\] \hspace{6.6cm} \textbf{Figure 1} \\ \end{example} In this case, the graph $\Gamma(\mathbb{Z}_{25})$ is a complete graph of $4$ vertices i.e.$K_4$. Now, we define a function $g: V(\Gamma(\mathbb{Z}_{25})) \longrightarrow \{0, 1, 2\}$ in a way such that $g(5) = 0$, $g(10) = 0$, $g(15) = 0$ and $g(20) = 2$. Clearly, by the definition, $g$ is an RDF with weight $g(V) = \sum_{u\in V} f(u) =2$. Since, this weight is minimum, so $\gamma_R(\Gamma(\mathbb{Z}_{25})) = 2$ or, $\gamma_R(K_n) = 2$. \par Moreover, some results on Roman domination number given by Cockayne et al. in \cite{cockayne} are given below. \begin{proposition} For the classes of paths $P_n$ and cycles $C_n$, \par $\gamma_R(P_n) = \gamma_R(C_n) = \lceil \frac{2n}{3} \rceil$.\\ \end{proposition} Also, they have proposed a relation between domination number and Roman domination number of a graph as follows. \begin{proposition} For any graph $G$,\\ \par $\gamma(G) \leq \gamma_R(G) \leq 2\gamma(G)$. \end{proposition} \begin{proposition} For any graph $G$ of order $n$, $\gamma(G) = \gamma_R(G)$ if and only if $G = \overline{K_n}$. \end{proposition} \section{Main Results} \begin{theorem} Let $S$ be a finite principal ideal local ring. Then $\gamma_R(\Gamma(S)) = 2$. \end{theorem} \begin{proof} Let $M$ be a maximal ideal of the finite principal ideal local ring $S$. Suppose $a \in S$ such that $M = <a>$, then $M= aS$. Let the set of unit elements of $S$ be $U= \{ u_1, u_2,..., u_m \}$. Since $S$ finite, there exists a positive integer $n$ such that $a^n = 0$ and $a^{n-1} \neq 0$. Then the element of $\Gamma(S)$ is of the form $u_i a^j$ where $i \leq m, ~ j \leq n$. Then $M = \{ u_i a^j : i \leq m, ~ j \leq n\}$. Since, $a^{n-1}$ is adjacent to all vertex of $M$. So, we define Roman dominating function $f = (V_0, V_1, V_2)$ such that $V_0 = M\backslash \{a^{n-1}\}, ~ V_1= \phi$ and $V_2= a^{n-1}$. Hence, every element $x$ of $V_0$ for which $f(x)= 0$ is adjacent to element of $V_2$. Thus, the Roman dominating number $\gamma_R(\Gamma(S)) = \sum_{u\in M}f(u)= \sum_{u_0\in V_0}f(u_0)+ \sum_{u_1\in V_1}f(u_1)+ \sum_{u_2\in V_2}f(u_2)= 0+0+2= 2$. \end{proof} \begin{theorem} Let $R= R_1 \times R_2$ be a ring such that $diam(\Gamma (R_1))= diam(\Gamma (R_2)) = 0$ and $\lvert R_1 \rvert \geq 5 ~\& ~\lvert R_2 \rvert \geq 5$. Then $\gamma_R(\Gamma(R)) = 4$. \end{theorem} \begin{proof} Let $R= R_1 \times R_2$ be a ring such that $diam(\Gamma (R_1))= diam(\Gamma (R_2)) = 0$. Then we have three cases. \textbf{Case 1:} $Z(R_1)= \{0, a\}$ and $Z(R_2)= \{0, b\}$ and let $Reg(R_1)= \{x_1, x_2,..., x_n\}$ and $Reg(R_2)= \{y_1, y_2,..., y_m\}$. Now, we are going to construct a graph for this case. \[\begin{tikzpicture} \vertex (A) at (-3,2) [label=below:${(x_i, b)}$]{}; \vertex (B) at (0,2) [label=above:${(0,b)}$]{}; \vertex (C) at (3,2) [label=right:${(x_i,0)}$]{}; \vertex (D) at (0,0.4) [label=right:${(a,0)}$]{}; \vertex (E) at (-3,-0.5) [label=below:${(a,b)}$]{}; \vertex (F) at (3,-0.5) [label=below:${(0,y_i)}$]{}; \vertex (G) at (0,-1.2) [label=below:${(a,y_i)}$]{}; \path (A) edge (B) (B) edge (C) (B) edge (D) (B) edge (E) (C) edge (F) (D) edge (E) (D) edge (G) (D) edge (F) ; \end{tikzpicture}\] \hspace{6.6cm} \textbf{Figure 2} \\ Also, we define a function $g : V(\Gamma(R)) \longrightarrow \{0,1,2\}$ by \[ g(x, y) = \left\{ \begin{array}{ll} 2 & if~ (x, y) = (0, b) ~and ~(x,y)= (a,0) \\ 0 & otherwise \\ \end{array} \right. \] Here, it is easily seen that $g$ is a Roman dominating function such that $g(v)= 2+2=4$. Hence, $\gamma_R(\Gamma(R)) = 4$. \textbf{Case 2:} Suppose $R_1$ is an integral domain and $Z(R_2)= \{0,b\}$, then we have the following induced subgraph. \[\begin{tikzpicture} \vertex (A) at (-1.5,1.5) [label=left:${(x_i, b)}$]{}; \vertex (B) at (0.5,1.5) [label=right:${(0,b)}$]{}; \vertex (C) at (0.5,-0.5) [label=right:${(x_i,0)}$]{}; \vertex (D) at (-1.5,-0.5) [label=left:${(0,y_i)}$]{}; \path (A) edge (B) (B) edge (C) (C) edge (D) ; \end{tikzpicture}\] \hspace{6.6cm} \textbf{Figure 3} \\ Again, we define a function $g$ as follows: \[ g(x, y) = \left\{ \begin{array}{ll} 2 & if~ (x, y) = (0, b) ~and ~(x,y)= (x_i,0)~ for~ a~ fixed~ i\\ 0 & otherwise \\ \end{array} \right. \] Clearly, $g$ is a RDF with $g(v)= 2+2=4$. Therefore, $\gamma_R(\Gamma(R)) = 4$. \textbf{Case 3:} Now, we suppose $R_1$ and $R_2$ are integral domains. In this case, $\Gamma(R)$ is a complete bipartite graph and $\lvert R_1 \rvert \geq 5 ~\& ~\lvert R_2 \rvert \geq 5$. Therefore, $\gamma_R(\Gamma(R)) = 4$. \end{proof} \begin{theorem} Let $R= R_1 \times R_2$ be a ring such that $diam(\Gamma (R_1))= 0$ and $diam(\Gamma (R_2)) = 1$. Then $\gamma_R(\Gamma(R)) = 4$. \end{theorem} \begin{proof} Since $diam(\Gamma (R_1))= 0$ and $diam(\Gamma (R_2)) = 1$. Then we have two cases for the ring $R_1$.\\ \textbf{Case 1:} Let $Z(R_1)= \{0,a\}$. Then $Reg(R_1)= \{x_1,x_2,...,x_n\}$, $Reg(R_2)= \{y_1,y_2,...,y_m\}$. Suppose $Z(R_2)= \{0,z_1,z_2,...,z_k\}$ such that $z_i z_j = 0$ for all $i,j \leq k$. Now, we are going to construct a graph for this condition. \[\begin{tikzpicture} \vertex (A) at (1,4) [label=above:${(a, z_j)}$]{}; \vertex (B) at (-1,2) [label=above:${(0,z_j)}$]{}; \vertex (C) at (3,2) [label=above:${(a,0)}$]{}; \vertex (D) at (0,0) [label=left:${(x_i,0)}$]{}; \vertex (E) at (2,0) [label=right:${(0,y_j)}$]{}; \vertex (F) at (5,2) [label=below:${(a,y_i)}$]{}; \vertex (G) at (-3,2) [label=below:${(x_i,z_j)}$]{}; \path (A) edge (B) (A) edge (C) (B) edge (G) (B) edge (D) (B) edge (C) (C) edge (F) (C) edge (E) (D) edge (E) ; \end{tikzpicture}\] \hspace{6.6cm} \textbf{Figure 4} \\ Also, we define a function $g$ as follows: \[ g(x, y) = \left\{ \begin{array}{ll} 2 & if~ (x, y) = (a, 0) ~and ~(x,y)= (0,z_j) for~ j=1 \\ 0 & otherwise \\ \end{array} \right. \] It has been easily seen that $g$ is an RDF. Therefore, $g(v)= 2+2= 4$ and hence $\gamma_R(\Gamma(R)) = 4$. \textbf{Case 2:} Let $R_1$ be an integral domain. Then we have an induced subgraph given in fig $4$. \[\begin{tikzpicture} \vertex (A) at (-1,0) [label=left:${(x_i, z_j)}$]{}; \vertex (B) at (0,0) [label=right:${(0,z_j)}$]{}; \vertex (C) at (0,1) [label=right:${(x_i,0)}$]{}; \vertex (D) at (0,2) [label=left:${(0,y_j)}$]{}; \path (A) edge (B) (B) edge (C) (C) edge (D) ; \end{tikzpicture}\] \hspace{6.6cm} \textbf{Figure 5} \\ Again, we define a function $g$ as follows. \[ g(x, y) = \left\{ \begin{array}{ll} 2 & if~ (x, y) = (x_i, 0) ~and ~(x,y)= (0,z_j) for~ i=j=1 \\ 0 & otherwise \\ \end{array} \right. \] It can be easily verify that $g$ is an RDF. Then $g(v)= 2+2= 4$ and hence $\gamma_R(\Gamma(R)) = 4$. \end{proof} \begin{theorem} Let $R= R_1 \times R_2$ be a ring such that $diam(\Gamma (R_1))= diam(\Gamma (R_2)) = 1$. Then $\gamma_R(\Gamma(R)) = 4$. \end{theorem} \begin{proof} The proof is the same as the proof of the Theorem $3.3$. \end{proof} \begin{theorem} Let $R= R_1 \times R_2$ be a ring such that $diam(\Gamma (R_1))= 0$ and $diam(\Gamma (R_2)) = 2$. Then $\gamma_R(\Gamma(R)) = 4$. \end{theorem} \begin{proof} Let $R= R_1 \times R_2$ be a ring and $R_2$ be a finite local ring generated by $x$, say, $Z(R_2)= xR_2$ with $x^l=0$ and $x^{l-1}\neq 0$. Now, we have two cases. \\ \textbf{Case 1:} Suppose $Z(R_1)= \{0,a\}$, $Reg(R_1)= \{u_1,u_2,...,u_n\}$, $Reg(R_2)= \{v_1,v_2,...,v_m\}$ and $Z(R_2)= \{0,v_1x,v_2x,...,v_mx^{l-1}\}$ such that two vertices $v_ix^j$ and $v_sx^r$ of $\Gamma (R)$ are adjacent if and only if $j+r \geq l$. Now, we define the RDF $g$ on $V(\Gamma(R))$ as follows. For any one value of $m$, $g(0, v_mx^{l-1}) = 2$ and $g(a,0)=2$ and for the remaining vertices $x,y$, let $g(x,y)=0$. It is easily seen that $g$ is an RDF and $g(v)=2+2=4$. \\ \textbf{Case 2:} Let $R_1$ be an integral domain. Then $\Gamma(R)$ is an induced subgraph after deleting the vertices $(a,0), (a,v_j), (a, v_ix^j)$ for each $i~\& ~j$ from $case 1$. Now, defining RDF $g$ as $g(u_i,0)=2$ for any one of $i's,$ say, $i=1$ and $g(0, v_mx^{l-1})=2$ for $m=1$ and for the remaining vertices $(x,y)$, let $g(x,y)=0$. Then $g(v)=2+2=4$. Hence, in both cases, $\gamma_R(\Gamma(R)) = 4$. \end{proof} \begin{theorem} Let $R= R_1 \times R_2$ be a ring such that $diam(\Gamma (R_1))= 1~ or ~ 2$ and $diam(\Gamma (R_2)) = 2$. Then $\gamma_R(\Gamma(R)) = 4$. \end{theorem} \begin{proof} The proof is the same as given in Theorem $3.5$. \end{proof} \begin{remark} Let $R$ be a finite commutative ring with unity. If $R$ is a product of two local rings with diameters less than equal to $2$. Then Roman domination number is $4$. \end{remark} Let $G$ and $H$ be a graph. We define the Cartesian product of $G$ and $H$ to be the graph $G \Box H$ such that the vertex set of $G \Box H$ is $V(G) \times V(H)$, i.e., $\{(x,y)\vert x\in G, y\in H\}$. Also, two vertices $(x_1,y_1)$ and $(x_2,y_2)$ are adjacent in $G \Box H$ if and only if one of the following is true: \begin{itemize} \item $x_1 = x_2$ and $y_1$ is adjacent to $y_2$ in $H$, or \item $y_1 = y_2$ and $x_1$ is adjacent to $x_2$ in $G$. \end{itemize} \begin{proposition} Let $R_1$ and $R_2$ be two rings such that $\lvert \Gamma(R_1) \rvert = m$ and $\lvert \Gamma(R_2) \rvert = n$ and having $\Delta (\Gamma(R_1))= r_1$, $\Delta (\Gamma(R_2))= r_2$. Then $\gamma_R(\Gamma(R_1) \Box \Gamma(R_2)) \leq mn-r_1-r_2+1$. \end{proposition} \begin{proof} Suppose $R_1$ and $R_2$ be two rings and $\Delta (\Gamma(R_1))= r_1$, $\Delta (\Gamma(R_2))= r_2$ with $\lvert \Gamma(R_1) \rvert = m$ and $\lvert \Gamma(R_2) \rvert = n$. Now, we know from the definition of Cartesian product of two graphs, $V(\Gamma(R_1) \Box \Gamma(R_2)) = mn$. Therefore, there exists a vertex $v$ in $\Gamma(R_1) \Box \Gamma(R_2)$ such that $deg(v)= r_1+r_2$. If $V_2$= \{v\}, $V_1= V- N[v]$ and $V_0= V-V_1-V_2$, then $f= (V_0, V_1, V_2)$ is a Roman dominating function with $f(V)= \lvert V_1 \rvert + 2\lvert V_2 \rvert = mn-(r_1+r_2+1)+2 = mn-r_1-r_2+1$. Hence, the weight of the function $f$ is $mn-r_1-r_2+1$ and $\gamma_R(\Gamma(R_1) \Box \Gamma(R_2)) \leq mn-r_1-r_2+1$. \end{proof} \begin{corollary} Suppose that total number of non-zero zero-divisor in a ring $R_1$ is $1$, say $\lvert Z(R_1)^*\rvert = 1$ and $\lvert Z(R_2) \rvert \geq 2$, then $\gamma_R(\Gamma(R_1) \Box \Gamma(R_2)) = \gamma_R(\Gamma(R_2))$, since $\Gamma(R_1) \Box \Gamma(R_2) \cong \Gamma(R_2)$. \end{corollary} Now, we give some examples. \begin{example} Any graph $G$ has a Roman domination number equal to $2$, then a vertex of graph $G$ is adjacent to every other vertex of $G$. In paper \cite[Theorem 2.5]{ander}, it is proved that for a commutative ring $R$, there is a vertex of $\Gamma(R)$ which is adjacent to every other vertex if and only if either $R \equiv \mathbb{Z}_2 \times A$ where $A$ is an integral domain, or $Z(R)$ is an annihilator ideal (and hence is a prime). \end{example} \begin{example} \textbf{(a)} In \cite{akbari}. it is proved that for any finite ring $R$, if $\Gamma(R)$ is a regular graph of degree $m$, then $\Gamma(R)$ is a complete graph $K_m$ or a complete bipartite graph $K_{m,m}$. In this case, $\gamma_R(\Gamma(R)) = 2~or~ 4$, provided $m\geq 3$. \\ \textbf{(b)} In \cite[Theorem 9]{akbari}, let $R$ be a finite principal ideal ring. If $\Gamma(R)$ is a Hamiltonian graph, then it is either a complete graph or complete bipartite graph. Thus $\gamma_R(\Gamma(R)) = 2~or~ 4$.\\ \textbf{(c)} In \cite[Theorem 8]{akbari} , let $R$ be a finite decomposable ring. If $\Gamma(R)$ is a Hamiltonian graph, then $\Gamma(R) \equiv K_{n,n}$ for some natural number $n$. Consequently, $\gamma_R(\Gamma(R)) = 4$. \end{example} \begin{corollary} In \cite[Corollary 1]{akbari}, the graph $\Gamma(\mathbb{Z}_n)$ is a Hamiltonian graph if and only if $n= p^2$ where $p$ is a prime greater than $3$ and in this case, $\Gamma(\mathbb{Z}_n) \equiv K_{p-1}$. Thus, Roman domination number of $\Gamma(R)$ is $2$. \end{corollary} \begin{theorem} Let $R= R_1 \times R_2 \times ...\times R_n$, for a fixed integer $n\geq 3$ and $R_i$ be an integral domain for each $i= 1,2,...,n$. Then $\gamma_R(\Gamma(R)) = 2n$. \end{theorem} \begin{proof} Let $R= R_1 \times R_2 \times ...\times R_n$ be a ring and each $R_i$ be an integral domain for $i= 1,2,...,n$. Then the set $S= \{(1,0,0,...,0), (0,1,0,...,0),...,(0,0,0,...,1)\}$ upto $n$ terms is a dominating set and no subset $T$ of $R_1 \times R_2 \times ...\times R_n$ with cardinality less than $n$ can be a dominating set. Now, define a $\gamma_R$- function $g$ in such a way that $g(1,0,0,...,0)= g(0,1,0,...,0)= g(0,0,1,...,0)= ...= g(0,0,0,...,1)= 2$ and $g(u)= 0$ for rest of the vertices of $\Gamma(R)$ where $u$ is a vertex of $n$-tuples of $R_1 \times R_2 \times ...\times R_n$. Therefore, $g(V)= 2+2+...+2$ upto $n$ terms, it follows that $g(V)= 2n$ and hence $\gamma_R(\Gamma(R)) = 2n$. \end{proof} It is known from \cite[Theorem 8.7]{atiyah} that a finite Artinian ring $R$ is a product of local rings. Based on this result, we have the following: \begin{theorem} Let $R$ be a finite commutative Artinian ring and $R= R_1 \times R_2 \times ...\times R_n$ where each $R_i$ is a local ring for $i= 1,2,...,n$. Then $\gamma_R(\Gamma(R)) = 2n$ for $n\geq 3$. \end{theorem} \begin{proof} The proof of this Theorem is the same as above. \end{proof} \begin{theorem} Let $n= p_1^{m_1} p_2^{m_2}...p_k^{m_k}$ for any fixed integer $k\geq 1$ and for distinct primes $p_1,..., p_k$ and positive integers $m_1,..., m_k$. Then the following results hold. \end{theorem} \noindent \textbf{(a)} $\gamma_R(\Gamma(\mathbb{Z}_n)) = 2k,$ if $n= p_1^{m_1}... p_k^{m_k}$ and $ k\geq 3$.\\ \noindent \textbf{(b)} $\gamma_R(\Gamma(\mathbb{Z}_n)) = 4,$ if $n= p_1^{m_1} p_2^{m_2}$ where either $m_1 \geq 2~ or ~ m_2\geq 2~ \text{or}~ m_1= m_2=1$ and $p_1, p_2 \geq 3$.\\ \noindent \textbf{(c)} $\gamma_R(\Gamma(\mathbb{Z}_n)) = 2,$ if $n= p_1^{m_1}$ and $m_1 \geq 2$ or $n= p_1 p_2$ where either $p_1 = 2, p_2 \geq 3 ~or~ p_1 \geq 3, p_2 = 2$. \begin{proof} It is known that $\mathbb{Z}_n \cong \mathbb{Z}_{p_1}^{m_1} \times ... \times \mathbb{Z}_{p_k}^{m_k}$ where $n= p_1^{m_1}...p_k^{m_k}$. To prove part (a), from \cite[Proposition 1]{cockayne}, $\gamma(G)\leq \gamma_R(G) \leq 2\gamma(G)$. We suppose that $k \geq 3$ and $p_1, p_2,...,p_k$ are distinct primes and $m_1, m_2,..., m_k$ are positive integers. Now, we construct a set $T= \{(p_1^{m_1-1},0,0,...,0), (0, p_2^{m_2-1},0,...,0),.$ $..,(0,0,0,...,p_k^{m_k-1})\}$ and define a function $f= (V_0, V_1, V_2)$ such that $V_0 = V(\Gamma(\mathbb{Z}_n)) \setminus T$, $V_1= \phi$, $V_2 = T$. Then the function $f$ is a Roman dominating function with $f(V)= 2\lvert T\rvert = 2k$ and hence $\gamma_R(\Gamma(\mathbb{Z}_n)) = 2k$. \\ Proof of (b) is same as part (a). To prove (c), it is noted that $p_1^{m_1-1}$ is a vertex adjacent to all vertices in $\Gamma(\mathbb{Z}_n)$ and whenever $n= p_1p_2$, where $p_1 = 2, p_2 \geq 3 ~or~ p_1 \geq 3, p_2 = 2,$ then $\Gamma(\mathbb{Z}_n)$ is a star graph. Thus $\gamma_R(\Gamma(\mathbb{Z}_n)) = 2$. \end{proof} \begin{theorem} If $R$ is a commutative ring with unity that contains at least one prime ideal, then $\gamma_R(\Gamma(R)) \leq 2(\lvert p\rvert -1)$ where $p$ is a prime ideal. \end{theorem} \begin{proof} Suppose $R$ is a commutative ring with unity that contains at least one prime ideal $p$, then every edge in $\Gamma(R)$ has at least one end vertex in $p$. Let $S= p\setminus \{0\}$. Then $S$ is a dominating set of $\Gamma(R)$. Since every edge has at least one vertex in $S$, we define a function $g= (V_0, V_1, V_2)$ such that $V_0= V(\Gamma(R))\setminus S$, $V_1= \phi$, $V_2= S$. Also, $p$ is a prime ideal whose cardinality is minimal among the cardinalities of all prime ideals of $R$. Hence, we can conclude $g(V) \leq 2\lvert S\rvert = 2(\lvert p\rvert -1)$. Thus, $\gamma_R(\Gamma(R)) \leq 2(\lvert p\rvert -1)$. \end{proof} \section{Roman domination number of total graph} Let $R$ be a commutative ring with unity and $Z(R)$ be the set of zero-divisors of $R$. The total graph of a ring $R$, denoted by $T(\Gamma(R))$, is the undirected graph with vertex set $R$ and two vertices $x, y \in R$ are adjacent if and only if $x+y \in Z(R)$. This concept was introduced and studied by Anderson and Badawi \cite{anderson}. Here, $Z(\Gamma(R))$ and $Reg(\Gamma(R))$ are disjoint induced subgraphs of $T(\Gamma(R))$. It is observed that if $Z(R)$ is an ideal with $\lvert Z(R) \rvert = \beta$ and $2\in Z(R)$, then $deg(v) = \beta -1$ for every $v \in V(T(\Gamma(R)))$ and if $2 \notin Z(R)$, then $deg(v) = \beta -1$ for each $v \in Z(R)$ and $deg(v) = \beta$ for every $v \notin Z(R)$. Hence, there is no vertex in $T(\Gamma(R))$ which has degree $\lvert R \rvert - 1$. In this section, we obtain bounds for Roman domination number of $T(\Gamma(R))$.\\ To obtain the bound for the Roman domination number of $T(\Gamma(R))$, we first state the following result of Anderson and Badawi \cite{anderson}. \begin{lemma}[\cite{anderson}, Theorem 2.1] Let $R$ be a commutative ring such that $Z(R)$ is an ideal of $R$. Then $Z(\Gamma(R))$ is a complete (induced) subgraph of $T(\Gamma(R))$ and $Z(\Gamma(R))$ is disjoint from $Reg(\Gamma(R))$. \end{lemma} \begin{theorem} Let $R$ be a commutative ring (not necessarily finite) with identity and $Z(R)$ be its ideal with $\lvert R/Z(R) \rvert = \alpha$. Then $3 \leq \gamma_R(T(\Gamma(R))) \leq 2\alpha$. \end{theorem} \begin{proof} It is noted that $T(\Gamma(R))$ has no vertex of degree $\lvert R \rvert -1$, therefore, $\gamma_R(T(\Gamma(R))) \geq 3$. Suppose $\lvert Z(R) \rvert = \beta$ and $Z(R)$ is an ideal of $R$. Then, by Theorem $2.2$ of \cite{anderson}, there are two cases arise: $(i)$ If $2 \in Z(R)$, then each coset $x+Z(R)$ is a complete graph $K_{\beta}$ and $(ii)$ if $2 \notin Z(R)$, then coset $(x+Z(R)) \bigcup (-x+Z(R))$ is a complete bipartite graph $K_{\beta, \beta}$. Hence, \[ T(\Gamma(R))= \begin{cases} K_{\beta}\cup \underbracket{K_{\beta} \cup K_{\beta} \cup ... \cup K_{\beta}}_{(\alpha - 1) copies},& \text{if } 2 \in Z(R)\\ K_{\beta}\cup \underbracket{K_{\beta, \beta} \cup K_{\beta, \beta} \cup ... \cup K_{\beta, \beta}}_{(\frac{\alpha - 1}{2}) copies},& \text{if } 2 \notin Z(R) \end{cases} \] Note that each of the component in case $(i)$ has Roman domination number $2$ as the definition of complete graph. Therefore, $\gamma_R(T(\Gamma(R)))= 2\alpha$. In case $(ii)$, each complete bipartite graph has maximum Roman domination number $4$. Hence, $\gamma_R(T(\Gamma(R))) \leq 2+ (\frac{\alpha -1}{2}) 4 = 2\alpha$. Thus, in both cases, $3 \leq \gamma_R(T(\Gamma(R))) \leq 2\alpha$. \end{proof} \section{Conclusion} The main objective of this paper was to study the Roman domination number of zero-divisor graph of a commutative ring $R$. Here, we have first calculated the Roman domination number of $\Gamma(R)$ where $R = R_1 \times R_2$, for different diameters of $\Gamma(R_1)$ and $\Gamma(R_2)$ and then we generalized it for $R= R_1 \times R_2 \times ... \times R_n$. At the end of this paper, we have discussed the upper and lower bounds of Roman domination number of total graph $T(\Gamma(R))$. \begin{thebibliography}{20} \bibitem{ahangar} Ahangar, HA., Chellali, M., Sheikholeslami, SM.: On the double Roman domination in graphs. Discrete Appl. Math. \textbf{232}, 1-7 (2017). \vspace{.3 cm} \bibitem{akbari1} Akbari, S., Maimani, HR., Yassemi, S.: When a zero-divisor graph is planer or a complete r-partite graph. J. Algebra. \textbf{270}, 169-180 (2003). \vspace{.3 cm} \bibitem{akbari} Akbari, S., Mohammadian, A.: On the zero-divisor graph of a commutative ring. J. Algebra. \textbf{274}, 847-855 (2004). \vspace{.3 cm} \bibitem{anderson} Anderson, DF., Badawi, A.: The total graph of a commutative ring. J. Algebra. \textbf{320}, 2706-2719 (2008). \bibitem{ander} Anderson, DF., Livingston, PS.: The zero-divisor graph of a commutative ring, J. Algebra. \textbf{217}, 434-447 (1999). \vspace{.2 cm} \bibitem{atiyah} Atiyah, MF., MacDonald, IG.: Introduction to commutative algebra. Reading, MA: Addison-Wesley (1969). \vspace{.3 cm} \bibitem{bala} Balakrishnan, R., Ranganathan, K.: A Textbook of Graph Theory, (2nd edition), Springer New York Heidelberg Dordrecht London (2012). \vspace{.3 cm} \bibitem{beck} Beck, I.: Coloring of commutative ring. J. Algebra. \textbf{116}, 208-226 (1988). \vspace{.3 cm} \bibitem{beeler} Beeler, RA., Haynes, TW., Hedetniemi, ST.: Double Roman domination, Discrete Appl. Math. \textbf{211}, 23-29 (2016). \vspace{.3 cm} \bibitem{cockayne} Cockayne, EJ., Dreyer, Jr PA., Hedetniemi, SM., Hedetniemi, ST.: Roman domination in graphs. Discrete Math. \textbf{278}, 11-22 (2004). \vspace{.3 cm} \bibitem{henning} Henning, MA., Hedetniemi, ST.: Defending the roman empire- a new strategy. Discrete Math. \textbf{266}, 239-251 (2003). \vspace{.3 cm} \bibitem{kumar} Kumar, R., Prakash, O.: Divisor graph of the complement of $\Gamma(R)$. Asian-Eur. J. Math. \textbf{12}(4), DOI: 10.1142/S1793557119500578 (2019). \vspace{.3 cm} \bibitem{kumar1} Kumar, R., Prakash, O.: Pancyclic zero divisor graph over the ring $\mathbb{Z}_n[i]$. Discrete Math. Algorithms Appl. DOI: 10.1142/S1793830922500495 (2022). \vspace{.3 cm} \bibitem{redmond} Redmond, SP.: The zero-divisor graph of a non-commutative ring. Int. J. Commutative Ring. \textbf{1}, 203-211 (2002). \vspace{.3 cm} \bibitem{revelle} ReVelle, CS., Rosing, KE.: Defendens imperium romanum: a classical problem in military strategy, Amer. Math. Monthly. \textbf{107}(7), 585-594 (2000). \vspace{.3 cm} \bibitem{stewart} Stewart, I.: Defend the Roman empire!. Sci. Amer. \textbf{281}(6), 136-139 (1999). \vspace{.3 cm} \end{thebibliography} \end{document}
2412.07483v1
http://arxiv.org/abs/2412.07483v1
Lines, Twisted Cubics on Cubic Fourfolds, and the Monodromy of the Voisin Map
\documentclass[a4paper,11pt]{amsart} \usepackage[a4paper,top=3cm,bottom=3cm, left=3cm,right=3cm,marginparwidth=60pt]{geometry} \usepackage{anysize} \marginsize{1.3in}{1.3in}{1in}{1in} \usepackage{comment} \usepackage{xcolor} \usepackage{amsmath} \usepackage{mathtools} \usepackage[all]{xy} \usepackage[utf8]{inputenc} \usepackage{varioref} \usepackage[normalem]{ulem} \usepackage{amsfonts} \usepackage{amssymb} \usepackage{bbm} \usepackage{esint} \usepackage{graphicx} \usepackage{tikz} \usepackage{empheq} \usepackage{enumitem} \usepackage{tikz-cd} \usepackage[font=small,labelfont=bf]{caption} \usepackage{subcaption} \usetikzlibrary{matrix,arrows,decorations.pathmorphing} \usepackage{mathrsfs} \usepackage[hypertexnames=false,backref=page,pdftex, pdfpagemode=UseNone, breaklinks=true, extension=pdf, colorlinks=true, linkcolor=blue, citecolor=red, urlcolor=blue, ]{hyperref} \renewcommand*{\backref}[1]{{-- cited on p.~#1}} \usepackage{cleveref} \usepackage{soul} \def\franco#1{{\color{cyan}(#1)}} \definecolor{brickred}{rgb}{0.8, 0.25, 0.33} \def\luca#1{{\color{brickred}#1}} \usepackage[textsize=small]{todonotes} \newcommand\Luca[1]{\todo[color=yellow!40]{#1}} \newcommand\Lucaline[1]{\todo[inline,color=yellow!40]{#1}} \newcommand{\scrG}{{\mathscr G}} \def\Mon{\operatorname{Mon}} \def\sing{\operatorname{sing}} \def\Ram{\operatorname{Ram}} \def\Branch{\operatorname{Branch}} \def\II{\operatorname{II}} \renewcommand{\P}{{\mathbb P}} \newcommand{\bbR}{{\mathbb R}} \newcommand{\sO}{\mathcal O} \newcommand{\wh}[1]{{\widehat{#1}}} \newcommand{\wt}[1]{{\widetilde{#1}}} \newcommand{\Bl}{\operatorname{Bl}} \newcommand{\Gr}{\operatorname{Gr}} \newcommand{\Hilb}{\operatorname{Hilb}} \newcommand{\Sym}{\operatorname{Sym}} \newcommand{\x}{\times} \newcommand{\ord}{{\mathrm{ord}}} \newcommand{\et}{{\mathrm{et}}} \newcommand{\flatt}{{\mathrm{flat}}} \theoremstyle{plain} \newtheorem{satz}[subsection]{Satz} \newtheorem{theorem}[subsection]{Theorem} \newtheorem{definition}[subsection]{Definition} \newtheorem{question}[subsection]{Question} \newtheorem{lemma}[subsection]{Lemma} \newtheorem{corollary}[subsection]{Corollary} \newtheorem{assumption}[subsection]{Assumption} \newtheorem{set}[subsection]{Setting} \newtheorem{bigthm}{Theorem} \renewcommand{\thebigthm}{\Alph{bigthm}} \newtheorem{proposition}[subsection]{Proposition} \theoremstyle{remark} \newtheorem{example}[subsection]{Example} \newtheorem{remark}[subsection]{Remark} \title[Monodromy of the Voisin map]{Lines, Twisted Cubics on Cubic Fourfolds, and the Monodromy of the Voisin Map} \author[]{Franco Giovenzana} \address[F. Giovenzana]{Laboratoire de Math\'ematiques d’Orsay\\ Universit\'e Paris-Saclay\\Rue Michel Magat, B\^at. 307, 91405\\ Orsay, France} \email{[email protected]} \author[Franco and Luca Giovenzana]{Luca Giovenzana} \address[L. Giovenzana]{Department of Pure Mathematics\\ University of Sheffield\\ Hicks Building, Hounsfield Road\\ Sheffield, S3 7RH\\ UK} \email{[email protected]} \usepackage{framed} \begin{document} \thispagestyle{empty} \begin{abstract} For a cubic fourfold \( Y \) with associated Fano variety of lines \( F \), we establish several properties of the finite-degree 16 self-rational map \( \psi \colon F \dashrightarrow F \) introduced by Voisin. We begin by analyzing the singularities of the nodal quintic with 16 nodes associated with a general line under the specialization to a line in the branch locus of \( \psi \). This approach reveals that the ramification of the natural resolution of indeterminacy of \( \psi \) is simple. The main part of the paper focuses on the intriguing interplay between \( \psi \) and the fixed locus of the antisymplectic involution on the LLSvS variety \( Z \), examined via the degree 6 Voisin map \( F \times F \dashrightarrow Z \). As an application, we show that the monodromy of \( \psi \) is maximal. \end{abstract} \makeatletter \@namedef{subjclassname@2020}{ \textup{2020} Mathematics Subject Classification} \makeatother \subjclass[2020]{32J27 (primary), 32S15 (secondary).} \keywords{Irreducible symplectic varieties, cubic fourfolds} \maketitle \setlength{\parindent}{1em} \setcounter{tocdepth}{1} \section{Introduction} Let $Y$ be a smooth cubic fourfold and $F$ be its (Fano) variety of lines. The rich geometry of these varieties has attracted great attention from the mathematical community for several reasons. Notably, the Fano variety $F$ is one of the earliest examples of a locally complete family of projective hyperkähler manifolds, and the binome $Y-F$ between a Fano variety and a hyperk\"ahler manifold has inspired numerous other constructions. One distinguishing feature of $F$ is the degree 16 self-rational Voisin map $\psi\colon F\dashrightarrow F$. Remarkably, $\psi$ is the only known finite, non-birational self-map defined on a locally complete family of projective hyperkähler manifolds. Finite maps such as $\psi$ are particularly subtle to study as, unlike birational maps, they cannot be detected through their action on the second cohomology group and lack a comprehensive classification result as Hodge Torelli theorem that would allow precise control over their behavior. Since its construction \cite{voisin-map-F}, this map has continued to attract significant interest in the mathematical community, as demonstrated by the numerous works on the subject\cite{Amerik,GK-invariants, GK-monodromy, GK-lines}. In this paper, we focus on the monodromy of the Voisin map $\psi$. The monodromy group is a discrete invariant that encodes the symmetries and intrinsic geometry of finite maps. Determining the monodromy group of branched coverings is a classical problem, originating with Jordan in the 1870s. This topic was revitalized by Harris, who provided a modern framework by proving that the monodromy and Galois groups coincide, and developed tools to establish when the monodromy group is maximal. Significant progress was made by Vakil, who introduced innovative techniques to study monodromy in Schubert problems on Grassmannians \cite{vakil}. Building on these developments, we take a step further by investigating linear spaces on a cubic hypersurface of dimension 4. Our main result shows that the monodromy group of the Voisin map $\psi$ is maximal, meaning that it is the full symmetric group on 16 elements. To achieve this, we leverage another Voisin map involving twisted cubics on the cubic fourfold, which unveils a fascinating connection with the fixed locus of the natural anti-symplectic involution on the LLSvS variety. \bigskip We now introduce the necessary notation and recall some useful results in order to state our main theorems. Lines on a cubic fourfold fall into two cases: For a general line $L$ the linear space \[ \Lambda_\ell := \bigcap_{p\in L} T_p Y \] is 2-dimensional and in this case the line is said to be of \textit{type I}. For special lines, called of \textit{type II}, the dimension of $\Lambda_\ell$ is 3. For the general line we have $\Lambda_\ell \cap Y = 2\ell + r$ for a line $r\in F$, and one sets $\psi(\ell) := r$. This map has been studied in detail in \cite{Amerik}, its indeterminacy locus consists of the lines of type $\II$, which for a general cubic fourfold form a smooth surface $S_{\II}$. Blowing up $S_{\II}$ provides a resolution of the indeterminacy \[ \xymatrix{ &\wh F\ar[ld]\ar[rd]^{\wh \psi}\\ F\ar@{-->}[rr]^{\psi} && F. } \] Gounelas and Kouvidakis recently computed that the restricion of $\wh \psi$ to the exceptional divisor $E$ of the blow-up morphism, which coincides with the ramification divisor of $\wh\psi$, is birational onto the image and they posed the question of whether the ramification of $\wh\psi$ is simple \cite[Theorem~B, Remark~3.13]{GK-monodromy}. In Section \ref{sec:ramification} we study the ramification of $\wh\psi$ by consider the projection of $Y$ from a general line $R$ in $Y$. Its resolution is a conic bundle over $\mathbb P^3$ with discriminant locus a quintic surface $S_r$ with exactly 16 nodes. These nodes correspond to the preimage of $r$ under $\wh\psi$. Nodal quintic surfaces have been classically studied by Beauville and Catanese \cite{Beauville-nodal, Catanese} and more recently studied in \cite{HUY-nodal-quintics,7auth, catanese-new}. Our first attention is to study the singularity of $S_r$ for a special line $r$, which lies in the branch locus of $\wh\psi$. \begin{theorem}[see Theorem~\ref{thm:A3-singularity-quintic}] Let $r\in F$ be a general line in the branch divisor of $\wh\psi$. Then the quintic surface $S_r$ has one singularity of type $A_3$ corresponding to the unique line $L$ of type $\II$ over $r$. \end{theorem} As a result of our analysis we affirmatively answer Gounelas and Kouvidakis' question: \begin{corollary}[{see Corollary~\ref{cor:ram-simple}}] The ramification at the general point of the ramification locus of $\wh\psi$ is simple. \end{corollary} In Section~\ref{sec:variety-P} we shift our focus to the geometry of twisted cubics. With any smooth cubic fourfold $Y$ not containing a plane Lehn, Lehn, Sorger, and van Straten associated an eight-dimensional hyperkähler variety $Z$ parametrizing families of twisted cubics and their flat degenerations. The variety $Z$ is equipped with a natural antisymplectic involution $\tau$ \cite{Lehn-oberwolfach}. Its fixed locus is a smooth Lagrangian submanifold with 2 connected components: one is isomorphic to the cubic fourfold $Y$, the other one, $W$, is of general type \cite{FMSOG-II} and remains somewhat mysterious. Using a the degree 6 rational map $\varphi\colon F\times F \dashrightarrow Z$, constructed by Voisin \cite{Voisin-map-varphi}, we offer an alternative description of $W$. We define the variety $P$ as the closure in $F\times F$ of \[ \{ (\ell_1,\ell_2)\in F\times F : \ell_i \text{ are of type I,\quad $\ell_1\not = \ell_2$,\quad and }\quad \psi(\ell_1)=\psi(\ell_2) \}, \] which is birational to an irreducible component of the self-product of $\wh F$ over $F$. \begin{theorem}[see~Theorem~\ref{thm: P->W}] The variety $P$ is mapped onto $W$ under the Voisin map $\varphi\colon F\times F \dashrightarrow Z$. \end{theorem} In Section~\ref{sec:monodromy} after revising the basic notions of monodromy we tackle the study of $\wh \psi$. Despite extensive study and numerous works concerning $\wh\psi$, for example about its entropy \cite{Amerik}, many of its properties remain elusive. Through an investigation of the restriction of the map $\varphi$ to the variety $P$ we prove the following \begin{theorem}[see Theorem~\ref{thm:monodromy-maximal}] The monodromy group of $\psi$ is the entire symmetric group on 16 elements. \end{theorem} \subsection*{Acknowledgments} This project began long time ago, and over the years we benefited from conversations with many people. It is our pleasure to thank everybody who expressed interest and shared their point of view, especially Enrico Fatighenti, Frank Gounelas, Christian Lehn, Emanuele Macrì, Giovanni Mongardi, Alan Thompson and Yilong Zhang. Franco Giovenzana was funded by Deutsche Forschungsgemeinschaft (DFG, German Research Foundation) Projektnummer 509501007, and partially supported by the European Research Council (ERC) under the European Union’s Horizon 2020 research and innovation programme (ERC-2020- SyG-854361- HyperK). All authors are members of INdAM GNSAGA. \section{The ramification of the Voisin map is simple}\label{sec:ramification} In this section we recall basic facts abour lines on cubic fourfolds and various properties of the Voisin map on $F$. Then we move on to prove that the ramification of the Voisin map is simple. Recall that the Gauss map associates to any point of the smooth cubic fourfold $Y\subset \P(V)$ its projective tangent space: \[ \mathscr G\colon Y \to \P(V^\vee),\ P\mapsto T_P Y \simeq \P^4. \] Clemens and Griffiths distinguished lines on cubic hypersurfaces into two types, we recall here the definition for cubic fourfolds \cite{griffiths-clemens}. \begin{proposition} Given a line $L$ on a smooth cubic fourfold $Y$, either the following equivalent conditions hold: \begin{enumerate} \item $N_{L|Y} \simeq \sO_L^{\oplus 2}\oplus \sO_L(1)$, \item $\mathscr G|_L\colon L\to \mathscr G (L)$ is 1:1, \item $\mathscr G (L)$ is a smooth conic in $\P(V^\vee)$, \item $\bigcap_{P\in L} T_P Y$ is isomorphic to $\P^2$; \end{enumerate} or the following equivalent conditions hold: \begin{enumerate} \item[(5)] $N_{L|Y} \simeq \sO_L(-1)\oplus \sO_L(1)^{\oplus 2}$, \item[(6)] $\mathscr G|_L\colon L\to \mathscr G (L)$ is 2:1, \item[(7)] $\mathscr G (L)$ is a line in $\P(V^\vee)$, \item[(8)] $\bigcap_{P\in L} T_P Y$ is isomorphic to $\P^3$. \end{enumerate} \end{proposition} \begin{definition}[{\cite[Definition~6.6]{griffiths-clemens}}] Given a line $L$, we say that $L$ is a line of type I if the equivalent conditions $(1)-(4)$ hold, whereas if $(5)-(8)$ hold we say that $L$ is a line of type II. We set $\Lambda_L:= \cap_{P\in L} T_P Y$. \end{definition} We record here these elementary observations for future reference. \begin{remark} In case $\ell\in F$ is of type $\II$, then the line $\mathscr G(L) \subset \P(V^\vee)$ is the projective dual of $ \Lambda_L \subset \P(V)$. In case of a line of type I, $\mathscr G(L)$ spans a $\P^2$ in $\P(V^\vee)$ which is dual to $\Lambda_L$. \end{remark} \begin{remark}\cite[Remark~2.2.2]{HuyBookCubics}\label{huy-derivatives} Let $Y = V(F)\subset \P^5$ be a smooth cubic fourfold, let $L$ be a line in $Y$, then $L$ is of type II if and only if the partial derivatives $\partial_0 F|_L,...,\partial_6 F|_L$ span a vector space of dimension 2 in $H^0(L,\sO_{\mathbb P^5}(2))$. \end{remark} Voisin observed that if $L$ is a line of type I, then $\Lambda_L\cap Y$ is a cubic curve, which, as it contains $L$ with multiplicity 2, consists of $L$ and a residual line $R$. As the general line is of type I, one defines the rational map \begin{align*} \psi\colon F \dashrightarrow F, \quad \ell \mapsto r. \end{align*} This map has been studied in \cite{voisin-map-F} and \cite{Amerik}, where it is proven to be finite of degree 16. Lines of type II form a surface $S_{\II}$ in $F$, and blowing up $F$ in this surface resolves the indeterminacy of $\psi$: \[ \xymatrix{ &\wh F\ar[ld]\ar[rd]^{\wh \psi}\\ F\ar@{-->}[rr]^{\psi} && F. } \] The blow up $\wh F$ can be identified as the closure of the graph \cite[Lemma 4.1]{GK-lines},\cite[Remark 2.2.19]{HuyBookCubics}. \begin{align*} F \dashrightarrow \Gr(3,6), \ \ell \mapsto \Lambda_\ell. \end{align*} Elements in the exceptional locus are then just pairs $(\ell,\Xi)$, where $L$ is a line of type II and $\Xi$ is a projective plane such that $L \subset \Xi \subset \Lambda_\ell$. For such 2-dimensional spaces $\Xi$ we have that $\Xi \cap Y = 2\ell + r$ and we shall say that $r$ is \textit{residual} to $\ell$. Given a general line $r\in F$, we consider the diagram \[\xymatrix{ &\wt Y:=\Bl_R Y\ar[ld]^p\ar[rd]^{\wt \pi}\\ Y \ar@{-->}[rr]^{\pi_R} && \P^3 } \] where $\pi_R$ is the projection from $R$ and $\wt Y$ is the blow-up of $Y$ in $R$. The morphism $\wt \pi$ is a conic bundle with discriminant a quintic surface $S_r$, whose singular locus consists of 16 nodes. The 16 nodes correspond to the preimage of $R$ under $\wh\psi$. Indeed, if $p_i$ denote the nodes of $S_R$ for $i=1,..,16$, then $L_i:=p(\wt \pi^{-1}(p_i))$ are the 16 lines for which $\psi(L_i) = R$ (see \cite[\S~6.4.5]{HuyBookCubics} for an account on the various results in the literature about this surface). Let $E$ be the exceptional divisor of the blow-up morphism $\wh F \to F$. As $F$ has trivial canonical bundle, the divisor $E$ coincide with the ramification locus of the map $\wh \psi$. \begin{theorem}[Gounelas-Kouvidakis,\cite{GK-lines}]\label{GK-RamBirational} The restriction $\wh \psi|_E \colon E \to F$ is generically 1-to-1 onto the image. \end{theorem} In other words, over the general point $r$ in the branch divisor of $\wh \psi$, there is exactly one point of ramification, i.e. there exists exactly one line $L$ of type II with residual $R$, meaning that $\wh\psi (\ell, \langle L,R \rangle) = r$. \begin{theorem}\label{thm:A3-singularity-quintic} Let $r\in F$ be a general line in the branch divisor of $\wh \psi$. Then the quintic surface $S_r$ has exactly 14 singularities of type $A_1$ and exactly 1 singularity of type $A_3$ corresponding to the unique line $L$ of type $\II$ with residual line $R$. \end{theorem} \begin{proof} Let $r$ be a general line in the branch locus of $\wh\psi$ and let $\ell$ be the unique line of type $\II$ with residual $r$. We may assume that $R$ is of type I and that is given as the zero set $V(x_2,x_3,x_4,x_5)$. Following Clemens and Griffiths \cite[equation~6.9]{griffiths-clemens} we write the equation of the cubic fourfold as \begin{equation} x_4x_0^2 + x_5x_0x_1 + x_3x_1^2 + x_0Q_0 + x_1Q_1 + P = 0, \end{equation} where $Q_0,Q_1$ and $P$ are homogeneous polynomials in the variables $x_2,x_3,x_4,x_5$ of degree 2, 2, and 3 respectively. The quintic surface $S_r\subset V(x_0,x_1) \simeq \mathbb P^3$ is given by the determinant of the matrix \[ M:=\begin{pmatrix} P & Q_0 & Q_1\\ Q_0 & x_4 & x_5 \\ Q_1 & x_5 & x_3 \end{pmatrix}. \] We assume that the line $L$ is given by $V(x_1,x_2,x_4,x_5)$\footnote{Notice that with this assumption the line $L$ is not contained in the plane tangent to $R$ is $\Lambda_r = V(x_3,x_4,x_5)$, would have been in contradiction with $R$ being residual to $L$.}, then $p := (1:0:0:0:0:0)$ is the intersection of $L$ and $R$, and $q:=(0:0:0:1:0:0)$ is the point of intersection with $\mathbb P^3$. We translate these assumptions in conditions on the coefficients of $Q_0,\ Q_1$, and $P$. For this, we introduce the notation $Q_0 = \sum a_I\underline{x}^I$, $Q_1 = \sum b_I\underline x^I$ and $P = \sum c_I\underline x^I$. The cubic fourfold $Y$ contains the line $L$ if $F(x_0,0,0,x_3,0,0,0)$ is the zero form in $\mathbb C[x_0,x_3]$, hence we get the conditon \begin{align}\label{eq: cond1} a_{33} = c_{333} = 0. \end{align} The fact that the line $R$ is residual to $L$, i.e. $\langle L, R\rangle \cap Y = 2L+R$, translates into the further condition \begin{align}\label{eq: cond2} b_{33} = 0. \end{align} By Remark~\ref{huy-derivatives} the line $L$ is of type II can be rephrased in terms of the derivatives $\partial_i F|_L$, and we get the additional condition \begin{align}\label{eq: cond3} a_{23}c_{335} - a_{35}c_{233} = 0. \end{align} To conclude the proof we check with the aid of Macaulay2 that the surface defined by det$(M)$ for polynomials $Q_0,\ Q_1,$ and $P$ with coefficients satisfying conditions~\eqref{eq: cond1},~\eqref{eq: cond2}, and \eqref{eq: cond3} has a singularity of type $A_3$ at the point $q$. See ancillary file. \end{proof} \begin{corollary}\label{cor:ram-simple} The ramification at the general point of the ramification locus of $\wh\psi$ is simple. \end{corollary} This answers the natural question raised in \cite[Remark~3.13]{GK-monodromy}. \begin{proof} Let $b$ be a general point in the branch divisor of $\psi$. Then by Theorem \ref{GK-RamBirational} the preimage $\wh\psi^{-1}(b)$ consists of a finite number of points $p_1,\ldots p_k$ where only $p_1$ is of ramification, while $\wh\psi$ is \'etale at the points $p_2,\ldots p_k$. All the points $p_i$ correspond to singularities of the quintic surface $S_b$: in particular $p_2,\ldots p_k$ are singularities of type $A_1$, in contrast $p_1$ is a singular point of type $A_3$ by Theorem~\ref{thm:A3-singularity-quintic}. The surface $S_b$ is specialization of the surface $S_r$ for the general line $r\in F$, which has exactly 16 nodes. Thus we deduce that 2 points among these 16 specialize to the singularity of type $A_3$, whereas the remaining 14 specialize to the $p_2,..,p_k$, deducing that $S_b$ has 14 singularities of type $A_1$ apart from the $A_3$ singularity. \end{proof} Let $r$ be any line on the cubic fourfold, we denote by $F_r$ the locus of lines meeting $r$ \[ F_r :=\lbrace \ell \in F: L\cap R \not = \emptyset \rbrace. \] If $r$ is general, then $F_r$ is a smooth surface \cite[\S 3~ Lemma~1]{voisin-torelli}. As an application of our analysis we determine the singularities of $F_r$ for a general point $r$ in the branch locus of $\wh\psi$. \begin{corollary} Let $r\in F$ be a general line in the branch locus of $\wh \psi$, then the surface $F_r$ has one singularity of type $A_1$ at the point $\ell$ corresponding to the unique line $L$ of type $\II$ with residual line $R$. \end{corollary} \begin{proof} Let $\pi_R\colon Y \dashrightarrow \P^3$ be the projection from $R$. Then there is a map \[f\colon F_r \dashrightarrow S_r,\quad \ell \mapsto \pi_R(L).\] The map clearly extends on the point $r$ by mapping $r$ to $\pi_r(\Lambda_r)$, when the line $r$ is of type $I$. The map $f$ is 2-to-1 and can be regarded as the quotient of an involution, which, in case $r$ is general, it has 16 fixed points mapped to the 16 singular points of $S_r$. If $r$ is as in the hypothesis, then $S_r$ is has 14 singularities of type $A_1$ and one singularity of type $A_3$ corresponding to $\ell$. All the nodes get resolved in the 2-to-1 cover $F_r$, while the $A_3$ singularity gives an $A_1$ singularity, hence the claim. \end{proof} \section{On a lagrangian submanifold of $Z$ and singular cubic surfaces}\label{sec:variety-P} Given a smooth cubic fourfold $Y$, we denote by $Z=Z(Y)$ the associated LLSvS variety. It carries the action of an antisymplectic involution which has been studied in \cite{FMSOG-II} whose fixed locus consists of two connected components, one isomorphic to the cubic fourfold, which we hence denote $Y$, and a second one which we denote $W$. Goal of this section is to study the less understood variety $W$. In order to do that we introduce a subvariety $P$ of $F\times F$ which is related to $W$ via the Voisin map $\varphi: F\times F\dashrightarrow Z$. A key tool in our strategy is to reduce statements to the study of cubic surfaces $S$ which are given by intersecting the cubic $Y$ with a projective space $\mathbb P^3$ and consider lines and twisted cubics on $S$. \subsection{The LLSvS variety $Z$} Given a smooth cubic fourfold $Y$ not containing any plane, Lehn, Lehn, Sorger and van Straten constructed in \cite{LLSvS} a hyperk\"ahler variety $Z$ of dimension 8, which parametrizes families of twisted cubics on $Y$ and their flat degenerations. As any twisted cubic has $3t+1$ as Hilbert polynomial, the starting point is to consider the compactification $\Hilb^{gtc}(Y)$ of the space of twisted cubics inside the Hilbert scheme $\Hilb^{3t+1}(Y)$. Curves parametrized by this variety are called \textit{generalised twisted cubics}. The variety $\Hilb^{gtc}(Y)$ is smooth, of dimension 10 and admits a morphism to the Grassmannian $G:=\Gr(4,6)$, which maps any generalised twisted cubic to its linear span. This morphism factors through a $\P^2$-fibration onto a variety $Z'$ which has a generically finite morphism to the Grassmannian. The hyperk\"ahler projective manifold $Z$ is obtained by $Z'$ by contracting the divisor parametrising linear systems of curves which are not arithmetically Cohen-Macaulay. It comes with the natural polarisation $\mathcal{L}$ induced by the map to the Grassmannian $G$. It has dimension 8 and the image of the divisor under the contraction is a Lagrangian subvariety isomorphic to the cubic fourfold $Y$. \begin{remark} The construction above of the variety $Z'$ is performed relatively over the Grassmannian $G$, so that can be carried out for any cubic surface $S$, thus we shall and will use the variety $Z'(S)$ for any cubic surface $S$. It clearly comes equipped with the $\P^2$-fibration $\mathrm{Hilb}^{gtc}(S)\to Z'(S)$. \end{remark} The hyperk\"ahler manifold $Z$ has a biregular antisymplectic involution $\tau\colon Z \to Z$ as pointed out in \cite{Lehn-oberwolfach}. It can be defined as follows: Let $C\subset Y$ be a generalised twisted cubic and let $Q$ be any quadric (surface) in the linear span $\langle C \rangle$, which contains $C$. The intersection $Q\cap Y$ is a sextic which contains $C$, the residual cubic is another generalised twisted cubic $C'$. Antisymplectic involutions of K3$^{[n]}$-type manifolds and this particular example have been studied in \cite{antisymplectic, FMSOG-II}. \begin{theorem}[{\cite[Theorem 1.4]{FMSOG-II}}] The fixed locus of the antisymplectic involution has two connected components, one of which is the lagrangian subvariety $Y$. The other component $W$ is of general type, in particular $\omega_{W} = 3 \mathcal{L}|_{W}$, where $\mathcal L$ is the primitive polarization on $Z$. \end{theorem} Voisin constructed a degree 6 rational map $\varphi\colon F\times F \dashrightarrow Z$ in this way \cite{Voisin-map-varphi}. Given a general pair of skew lines $(\ell_1, \ell_2)$ and a point $p\in L_1$, then the union of $\ell_1$ and the quadric, which is residual to $\ell_1$ in the intersection $\langle L_1,\ p\rangle$, is a generalized twisted cubic. The indeterminacy locus, the resolution of it and the branch divisor of its resolution have been studied in \cite{Muratore, chen, mio}. The involution $\tau$ fits in the following commutative diagram: \[\xymatrix{ F\x F \ar[d]^\sigma \ar@{-->}[r]^{\varphi} & Z\ar[d]^\tau\\ F\x F \ar@{-->}[r]^{\varphi} & Z, } \] where $\sigma\colon (\ell_1,\ell_2)\mapsto (\ell_2,\ell_1)$ switches the factors. \subsection{A variety dominating $W$} We denote by $P$ the closure in $F\times F$ of \[ \{ (\ell_1,\ell_2)\in F\times F : \ell_i \text{ are of type I, $\ell_1\not = \ell_2$ and } \psi(\ell_1)=\psi(\ell_2) \}. \] It turns out to have a central r\^ole in our study: The following intriguing lemma is the starting point which allows us to relate properties of $\psi$ and $\phi$. \begin{lemma}\label{lem: fundamental} Let $(\ell_1, \ell_2)\in P$ be a general pair of lines, satisfying the conditions \begin{enumerate} \item $\ell_1$ and $\ell_2$ are skew, \item $\ell_1$ and $\ell_2$ are both of type I, \item and $\psi(\ell_1) = \psi(\ell_2)$. \end{enumerate} Then the linear span of $L_1$ and $L_2$ intersects $Y$ in a Cayley cubic surface. \end{lemma} \begin{proof} We claim that the linear span $\Lambda:=\langle L_1, L_2\rangle$ intersects $Y$ in a cubic surface with two singular points on $L_1$. We denote by $r$ the residual line $\psi(\ell_1)$, then $R$ and $L_1$ are coplanar and we consider the line $T$ in $\P(V^*)$ of hyperplanes which contain $\Lambda$. As both $T$ and $\scrG (L_1)$ lie in the 2 dimensional linear space $\langle L_1, R \rangle^* \subset \P(V^*)$, the line $T$ intersects the smooth conic $\scrG (L_1)$ in two points, which correspond to the image under the Gauss map of two singular points of $S:=\Lambda\cap Y$ lying on the line $L_1$. Repeating the argument for $L_2$ we find two singular points of $S$ on $L_2$ and we deduce that the cubic surface $S$ is Cayley by \cite[Corollary~9.2.3]{CAG}. \end{proof} Following the construction of the Voisin map, one gets the following \begin{lemma}\label{fundamental lemma} Let $L_1$ and $L_2$ be two lines that satisfy the general conditions: \begin{enumerate}\label{general-conditions} \item $L_1$ and $L_2$ are of type I, \item $L_1$ and $L_2$ are skew. \end{enumerate} If $\psi(\ell_1)= \psi(\ell_2)=:r$ then $\varphi(\ell_1, \ell_2) = \varphi(\ell_2, \ell_1)$. \end{lemma} We report the following theorem which is a key result for our study, but we defer its proof to later, because it relies on some observations conerning Cayley cubics. \begin{theorem}\label{thm: P->W} The Voisin map $\varphi\colon F\times F \dashrightarrow Z$ restricts to a dominant rational map $\varphi|_P\colon P\dashrightarrow W$. The regular map $W\to G$ is birational onto the image, which consists of the projective spaces that intersect $Y$ in a Cayley cubic or a degeneration of it. \end{theorem} In the following we analyze the maps $P\dashrightarrow W \to G$ in greater detail. In order to do so, we describe singular cubic surfaces with focussing on lines on them. \subsection{Cayley cubics and their degenerations} The stratification of the space of cubic surfaces given by the type of singularities has been studied by Bruce in \cite{bruce}. Here we are interested only in cubic surfaces with exactly 4 singular points of type $A_1$, called Cayley cubics, and their degenerations. From his analysis degenerations of Cayley cubics can be summed up by the following diagram: \[ 4A_1\ \succeq\ 2A_1A_3\ \succeq\ A_1A_5\ \succeq\ \wt E_6\ \succeq X_6\ \succeq X_7\ \succeq X_8\ \succeq\ X_9 \] where $\wt E_6$ denotes the cubics with one simple elliptic singularity and $X_i$ are non-normal cubic surfaces. In the following we focus on lines and twisted cubic on singular cubic surfaces to study the maps $\varphi$ and $\psi$. We notice that the map $\psi$ is in general not defined on $Z(S)$ for a singular cubic surface $S$ because a line $L$ need not have a plane tangent at every point. However, if $\pi\subset \langle S\rangle$ is tangent at every point of $L\subset S$ with $\pi \cap S = 2L + R$ for a line $R$, we say that the line $R$ is \textit{residual}, and that $L$ is \textit{triple} if $R=L$. Generalised twisted cubics on singular cubic surfaces have already been carefully analysed in \cite[\S~2]{LLSvS}, which we closely follow. Every cubic surface $S$ with at most double rational points has a minimal resolution $\wt S\to S$ with $\wt S$ isomorphic to the blow-up of $\mathbb P^2$ in six points. The orthogonal $\Gamma:=K_{\wt S}^{\perp}\subset H^2(\wt S, \mathbb Z)$ is isomorphic to the negative definite root lattice $E_6$. We denote by $R_0$ the roots in $\Gamma$ which are effective, they correspond to irreducible components of exceptional curves for the resolution. We also denote by $W(R_0)$ the subgroup of the Weyl group of $E_6$ generated by reflections in the effective roots $R_0$. The moduli space of generalised twisted cubics on $S$ with reduced structure is then given by a union of projective planes \cite[Theorem~2.1]{LLSvS}: \begin{align*} \mathrm{Hilb}^{gtc}(S)_{red} = (R/W(R_0)) \times \P^2. \end{align*} In particular, linear systems of generalised twisted cubics on $S$ are in bijection with the orbits of $R$ under the action of $W(R_0)$. These correspond to the set $Z'(S):=a^{-1}(E)$ where $E\in G$ is the linear span $\langle S \rangle$. Along this bijection families of arithmetically Cohen-Macaulay (aCM) curves correspond to orbits containing an effective root. By the definition of $\tau$ it is clear that it restricts to an action on twisted cubics contained on a cuibic surface, and indeed admits a nice description in this setting: As observed in \cite{Lehn-oberwolfach} it is indeed given by the association $\alpha \mapsto -\alpha$ for the roots $\alpha \in R$. We defer to Appendix~\ref{appendix:cubic-surfaces} details about these singular surfaces, and recall here only the main facts we need to analyse the the Voisin maps. We also introduce a piece of notation. We denote by $G_{4A1}$ the closure in the Grassmanian $G$ of \[ \{ E\in G: E\cap Y \mbox{ is a singular cubic surface of type }4A_1 \}. \] Analogously, we define the loci $G_{2A_1A_3}$, $G_{A_1A_5}$, $G_{\wt E_6}$, and for the non-normal surfaces $G_{X_6}$, $G_{X_7}$, $G_{X_8}$, $G_{X_9}$. \bigskip \subsubsection*{Singular cubic surfaces of type $4A_1$} Any Cayley cubic, i.e. a surface with exactly 4 $A_1$ singular points, contains exactly 9 lines whose schematic representation resembles a tetrahedron with vertices at the singular points. We name \textit{edges} the 6 of them connecting 2 singular points. In particular, if $P_1, P_2, P_3, P_4$ are the singular points, we denote $E_{ij}$ the line containing $P_i$ and $P_j$. Each of the other 3 lines intersect exactly 2 edges, we denote $J_{ij,kl}$ the line joining the edges $E_{ij}$ and $E_{kl}$ for $\{i,j,k,l\} = \{1,2,3,4\}$. See Figure~\ref{fig:4A1}. \begin{figure}[h!] \centering \captionsetup[subfigure]{justification=centering} \begin{subfigure}[t]{0.45\textwidth} \centering \begin{tikzpicture}[x=0.75pt,y=0.75pt,yscale=-1,xscale=1] \draw (313,38.47) -- (450.5,262.04) -- (175.5,262.04) -- cycle ; \draw (313,38.47) -- (317.03,178.12) ; \draw (317.03,178.12) -- (175.5,262.04) ; \draw (317.03,178.12) -- (450.5,262.04) ; \draw [draw opacity=0] (269.15,197.77) .. controls (268.26,193.46) and (267.8,189.03) .. (267.8,184.5) .. controls (267.8,155.17) and (287.19,129.69) .. (315.66,116.85) -- (357.65,184.5) -- cycle ; \draw [color={rgb, 255:red, 208; green, 2; blue, 27 } ,draw opacity=1 ] (269.15,197.77) .. controls (268.26,193.46) and (267.8,189.03) .. (267.8,184.5) .. controls (267.8,155.17) and (287.19,129.69) .. (315.66,116.85) ; \draw [draw opacity=0] (356.5,262) .. controls (356.5,262) and (356.5,262) .. (356.5,262) .. controls (317.84,262) and (285.01,240.33) .. (273.19,210.21) -- (356.5,185.5) -- cycle ; \draw [color={rgb, 255:red, 208; green, 2; blue, 27 } ,draw opacity=1 ] (356.5,262) .. controls (356.5,262) and (356.5,262) .. (356.5,262) .. controls (317.84,262) and (285.01,240.33) .. (273.19,210.21) ; \draw [draw opacity=0] (361.11,117.19) .. controls (371.99,142.82) and (368.74,171.71) .. (353.78,193.75) -- (287.72,147.9) -- cycle ; \draw [color={rgb, 255:red, 5; green, 26; blue, 245 } ,draw opacity=1 ] (361.11,117.19) .. controls (371.99,142.82) and (368.74,171.71) .. (353.78,193.75) ; \draw [draw opacity=0] (347.39,202.4) .. controls (342.83,207.39) and (337.52,211.81) .. (331.48,215.48) .. controls (307.03,230.36) and (277.09,229.15) .. (253.14,214.97) -- (289.6,146.64) -- cycle ; \draw [color={rgb, 255:red, 5; green, 26; blue, 245 } ,draw opacity=1 ] (347.39,202.4) .. controls (342.83,207.39) and (337.52,211.81) .. (331.48,215.48) .. controls (307.03,230.36) and (277.09,229.15) .. (253.14,214.97) ; \draw [draw opacity=0] (322.73,130.27) .. controls (327,131.57) and (331.13,133.39) .. (335.04,135.74) .. controls (357.76,149.43) and (367.69,177.83) .. (363.55,208.66) -- (285.92,217.34) -- cycle ; \draw [color={rgb, 255:red, 126; green, 211; blue, 33 } ,draw opacity=1 ] (322.73,130.27) .. controls (327,131.57) and (331.13,133.39) .. (335.04,135.74) .. controls (357.76,149.43) and (367.69,177.83) .. (363.55,208.66) ; \draw [draw opacity=0] (238.81,158.65) .. controls (259.05,137.86) and (285.3,127.04) .. (309.5,129.02) -- (288.16,218.74) -- cycle ; \draw [color={rgb, 255:red, 126; green, 211; blue, 33 } ,draw opacity=1 ] (238.81,158.65) .. controls (259.05,137.86) and (285.3,127.04) .. (309.5,129.02) ; \draw (308.75,266.88) node [anchor=north west][inner sep=0.75pt] [align=left] {E\textsubscript{12}}; \draw (208.67,135.62) node [anchor=north west][inner sep=0.75pt] [align=left] {E\textsubscript{02}}; \draw (395.13,128.85) node [anchor=north west][inner sep=0.75pt] [align=left] {E\textsubscript{01}}; \draw (291.27,89.62) node [anchor=north west][inner sep=0.75pt] [align=left] {E\textsubscript{03}}; \draw (358.87,220.02) node [anchor=north west][inner sep=0.75pt] [align=left] {E\textsubscript{13}}; \draw (221.47,233.42) node [anchor=north west][inner sep=0.75pt] [align=left] {E\textsubscript{23}}; \draw (272,158.2) node [anchor=north west][inner sep=0.75pt] [font=\footnotesize,xslant=0] {$J_{03,12}$}; \draw (298.4,206.6) node [anchor=north west][inner sep=0.75pt] [font=\footnotesize] {$J_{01,23}$}; \draw (330.4,154) node [anchor=north west][inner sep=0.75pt] [font=\footnotesize] {$J_{02,13}$}; \draw (314,15.8) node [anchor=north west][inner sep=0.75pt] {$P_{0}$}; \draw (459.2,257.4) node [anchor=north west][inner sep=0.75pt] {$P_{1}$}; \draw (147.6,256.6) node [anchor=north west][inner sep=0.75pt] {$P_{2}$}; \end{tikzpicture} \caption{Lines on a singular cubic surface of type $4A_1$} \label{fig:4A1} \end{subfigure} \hfill \begin{subfigure}[t]{0.45\textwidth} \centering \begin{tikzpicture}[x=0.75pt,y=0.75pt,yscale=-1,xscale=1] \draw (279,63) -- (356,204) ; \draw (212,206) -- (279,63) ; \draw (212,206) -- (356,204) ; \draw [draw opacity=0][line width=1.5] (260.19,24.83) .. controls (270.77,40.35) and (280.91,66.22) .. (287.21,96.41) .. controls (294.55,131.62) and (294.76,163.54) .. (288.93,181.22) -- (257.84,102.54) -- cycle ; \draw [color={rgb, 255:red, 10; green, 15; blue, 219 } ,draw opacity=1 ][line width=1.5] (260.19,24.83) .. controls (270.77,40.35) and (280.91,66.22) .. (287.21,96.41) .. controls (294.55,131.62) and (294.76,163.54) .. (288.93,181.22) ; \draw [draw opacity=0][line width=1.5] (273.85,127.49) .. controls (287.42,135.81) and (300.95,164.16) .. (306.8,199.29) .. controls (309.84,217.54) and (310.32,234.74) .. (308.66,248.77) -- (277.21,204.21) -- cycle ; \draw [color={rgb, 255:red, 9; green, 9; blue, 207 } ,draw opacity=1 ][line width=1.5] (273.85,127.49) .. controls (287.42,135.81) and (300.95,164.16) .. (306.8,199.29) .. controls (309.84,217.54) and (310.32,234.74) .. (308.66,248.77) ; \draw (272,14) node [anchor=north west][inner sep=0.75pt] [align=left] {L\textsubscript{1}}; \draw (314,237) node [anchor=north west][inner sep=0.75pt] [align=left] {L\textsubscript{2}}; \draw (244,211.4) node [anchor=north west][inner sep=0.75pt] {$R_{03}$}; \draw (208,125) node [anchor=north west][inner sep=0.75pt] [align=left] {R\textsubscript{02}}; \draw (325,119) node [anchor=north west][inner sep=0.75pt] [align=left] {R\textsubscript{23}}; \draw (288,50) node [anchor=north west][inner sep=0.75pt] [align=left] {P\textsubscript{2}}; \draw (187,194) node [anchor=north west][inner sep=0.75pt] [align=left] {P\textsubscript{0}}; \draw (360,197) node [anchor=north west][inner sep=0.75pt] [align=left] {P\textsubscript{3}}; \draw (277,147) node [anchor=north west][inner sep=0.75pt] [align=left] {P\textsubscript{1}}; \end{tikzpicture} \caption{Lines on a $2A_1A_3$ surface} \label{fig:2A1A3} \end{subfigure} \caption{} \label{fig:figures} \end{figure} We denote by $K$ the linear system of generalised twisted cubics containing the reducible elements given by the lines through a singular point \[ K = [E_{ij}\cup E_{ik} \cup E_{il}] = [E_{ji}\cup E_{jk} \cup E_{jl}] \in Z'_{aCM}(S). \] \begin{lemma}\label{lem: opposite edges} Opposite edges map to $K$ under $\varphi$, that is: \begin{align*} \varphi(E_{ij}, E_{kl}) = K\quad \text{ for indices such that } \{i,j,k,l\}=\{1,2,3,4\}. \end{align*} \end{lemma} \begin{lemma}\label{lem: 4A_1 LLSvS} Let $S$ be a Cayley cubic. Then \begin{align*} \# Z'_{aCM}(S)=13\quad \mbox{ and }\quad \#Z'(S)^{\tau} = 1. \end{align*} \end{lemma} \begin{proof} The first equality follows from \cite[Table~1]{LLSvS}. The second one can be deduced by the description of $\tau$ in terms of roots. \end{proof} \medskip \begin{proof}(of Theorem~\ref{thm: P->W}) By Lemma~\ref{lem: fundamental} the general point $(\ell_1,\ell_2)$ in $P$ corresponds to opposite edges on a Cayley cubic surface. By Lemma~\ref{lem: opposite edges} its image $\varphi((\ell_1, \ell_2))$ is an aCM twisted cubic fixed by the involution on $Z$, i.e. a point in $W$. In particular, $\varphi$ restricts to a dominant map $\varphi: P\dashrightarrow W$. Finally, the morphism $W\to G$ is 1:1 because by Lemma~\ref{lem: 4A_1 LLSvS} given a projective space $\Lambda \simeq \mathbb P^3$ which intersects $Y$ in a Cayley cubic $S$, $K$ is the unique linear system of $aCM$ generalised twisted cubics in $S$ which is fixed by $\tau$. \end{proof} \subsubsection*{Singular cubic surfaces of type $2A_1A_3$} Let $S$ be a cubic surface with a singular point $P_2$ of type $A_3$, and 2 singular points $P_0,P_3$ of type $A_1$. It contains exactly 5 lines as depicted in Figure~\ref{fig:2A1A3}: the line $R_{01}$ through $P_0,P_1$; the line $R_{02}$ through $P_0,P_2$; the line $R_{03}$ through $P_{03}$; the line $L_1$ containing the only singular point $P_2$, and the line $L_2$ contained in the smooth locus. \begin{proposition}\label{prop:2A1A3-residual} Let $S$ be a cubic surface of type $2A_1A_3$. Then: \begin{align*} L_1\mbox{ is residual to }R_{23},\qquad L_1 \mbox{ is residual to }R_{02} ,\\ L_2\mbox{ is residual to }R_{03},\qquad L_2\mbox{ is residual to } L_1. \end{align*} Moreover, there is no plane in $\langle S\rangle\simeq \mathbb P^3$ tangent to $S$ at every point of $L_2$. In particular, there are no triple lines on a surface with $2A_1A_3$ singularities. \end{proposition} \begin{remark} If $S$ is a cubic surface of type $2A_1A_3$ given by intersecting a smooth cubic fourfold $Y$ with a $\mathbb P^3$, then one can conclude there are no triple lines on S. Indeed, if all the lines are of type I, Proposition~\ref{prop:2A1A3-residual} can be restated as: \begin{align*} \psi(R_{23})=\psi(R_{02}) = L_1, \text{ and } \psi(R_{03})= \psi(L_1) = L_2. \end{align*} If a line, e.g. $R_{23}$, is of type II one can conclude that $\wh\psi(R_{23}, \pi)=L_1$ where $\pi=\langle R_{23}, L_1\rangle$. \end{remark} \begin{lemma} Let $S$ be a cubic surface of type $2A_1A_3$. Then \begin{align*} \# Z'_{aCM}(S)=5\quad \mbox{ and }\quad \#Z_{aCM}'(S)^{\tau} = 1. \end{align*} \end{lemma} \begin{proof} The first equality follows from \cite[Table~1]{LLSvS}. The second one can be deduced by the description of $\tau$ in terms of roots. \end{proof} \subsubsection*{Singular cubic surfaces of type $A_1A_5$}\label{ssec:A1A5} Let $S$ be a cubic surface with an $A_1$ and an $A_5$ singularity. It contains exactly two lines, say $L_1$ passing through both singular points, and $L_2$ which intersects the singular locus in the $A_5$ point. \begin{proposition}\label{prop: lines-A1A5} Let $S$ be a cubic surface of type $A_1A_5$. Then \begin{align*} L_2 \text{ is triple, and } L_2 \text{ is residual to } L_1. \end{align*} \end{proposition} \begin{lemma}\label{lem:gtc-on-A1A5} Let $S$ be a cubic surface of type $A_1A_5$. Then \begin{align*} \# Z'_{aCM}(S)= \#Z_{aCM}'(S)^{\tau} = 1. \end{align*} \end{lemma} \begin{proof} Clear from \cite[Table~1]{LLSvS}. \end{proof} \subsection{Simple elliptic cubic surfaces} Any cubic surface $S$ with a simple elliptic singularity is a cone over a planar elliptic curve $E$. Let $v$ be the vertex. The Fano variety $F(S)$ of $S$ can be naturally identified with the embedded elliptic curve $E$, indeed to any point $P$ of $E$ corresponds the line through $P$ and $v$. The scheme of generalised twisted cubics is isomorphic to $\Sym^3(E)$ and the summation map to $E$ is a $\P^2$-bundle giving the fibration $\Hilb^{gtc}(S)\to Z'(S)$ \cite[Proposition~2.7]{LLSvS}. A generalised twisted cubic corresponds to the union of 3 lines, which join the vertex with 3 points on $E$. If these are collinear, then they are the support of a nCM curve with an embedded point at the vertex. Via the isomorphism $F(S)\simeq E$, the residual line can be expressed in terms of the group law of $E$, and the analogue of $P$ for lines on the surface $S$ \begin{align*} P(S):= \{(p_1,p_2)\in F(S)\times F(S) \mid \mbox{ $p_1$ and $p_2$ have the same residual}\} \end{align*} admits a nice description: \begin{proposition} Let $S$ be a simple elliptic cubic surface. Then \begin{itemize} \item the residual to the line $p\in E\simeq F(S)$ is given by $-2p$; \item $P(S) =\{(p,p+\xi) \mid \xi\in E[2]\}$; \item the triple lines on $S$ are given by the 3-torsion points $E[3]\subset E$. In particular, every simple elliptic cubic surface has 9 triple lines. \end{itemize} \end{proposition} \begin{proposition}\label{cor: A1A5-E6} Let $Y$ be a general cubic fourfold, and let $\ell_1$ and $\ell_2$ be general lines of type $I$ such that $\psi(\ell_1) = \psi(\ell_2) = \ell_2$. Then $\langle E_{\ell_1}, E_{\ell_2} \rangle$ is a 3-dimensional space which cuts $Y$ in a cubic surface of type either $A_1A_5$, or $\wt E_6$. \end{proposition} \begin{proof} By Lemma \ref{lem: fundamental} a general point of $P$ consists of a pair of lines whose linear span intersects $Y$ in a Cayley cubic. Thus, if $\ell_1$ and $\ell_2$ are as in the hypothesis, then the 3 dimensional linear space $\langle E_{\ell_1},E_{\ell_2}\rangle$ intersects $Y$ in a cubic surface $S$ which is a degeneration of a Cayley cubic and contains $\ell_2$ as a triple line on the surface. By the analysis of lines on cubic surfaces, $S$ is either of type $A_1A_5$, $\wt E_6$ or non-normal. As by assumption the lines $\ell_1$ and $\ell_2$ are not of type $\II,$ we can exclude the non-normal cases. \end{proof} We explore now some connection of our study of singular cubic surfaces with results of Gounelas and Kouvidakis \cite{GK-lines}. As a consequence, we deduce the existence of a surface of type $A_1A_5$, which is a fundamental little ingredient for the main theorem. Let $Y$ be a smooth cubic fourfold. The natural map $\gamma\colon \mathbb L \to Y$ from the universal family of lines to the cubic fourfold is a fibration in (2,3)-complete intersection curves. Given a point $y\in Y$, its fiber $\gamma^{-1}(y)$ consists of the lines trough the point $y$. Gounelas and Kouvidakis studied the $g^1_3$ of these curves coming from the rulings of the corresponding quadric and their relation with the Voisin map. \begin{corollary}\label{cor:existence A1A5} Let $Y$ be a general cubic fourfold. Then there exists a $\Lambda \in G$ such that $\Lambda\cap Y$ is a cubic surface of type $A_1A_5$. \end{corollary} \begin{proof} Given a pair of lines $\ell_1$ and $\ell_2$ both of type I, with same residual and such that one is triple, then these are naturally contained in the cubic surface $\langle E_{\ell_1}, E_{\ell_2} \rangle\cap Y$, which is either of type $A_1A_5$ or $\wt E_6$. To discern the 2 cases, we reason as follows. A triple line through the point $y\in Y$ corresponds to a ramification point of the $g^1_3$ of the curves $C_y$ \cite[Lemma 3.10]{GK-lines}. As the general triple ramification point of one of the possibly two $g^1_3$ of the curve $C_y$ occurs when $C_y$ is smooth, we deduce that the general $\langle E_{\ell_1}, E_{\ell_2} \rangle$ cuts $Y$ in a cubic surface of type $A_1A_5$. \end{proof} \begin{corollary}\label{cor:lineSecTyp} Let $Y$ be a general cubic fourfold. \begin{enumerate} \item For any $\ell\in S_{\II}$ the intersection $E_\ell\cap Y$ is a non-normal singular cubic surface. \item If $\ell\in V\cap S_{\II}$, then $E_\ell\cap Y$ is of type $X_7$ or worse. \item If $\ell\in (V\cap S_{\II})_{\sing}$ is a node, then $E_\ell\cap Y$ is of type $X_8$ or worse. In particular, there are 3780 projective spaces of dimension 3 intersecting $Y$ in a singular surface of type $X_8$. \item For any $\ell\in S_{\II}$ the surface $E_\ell\cap Y$ is not of type $X_9$. \end{enumerate} \end{corollary} \begin{proof} Given a line $L$ of type $\II$, the cubic surface $E_\ell \cap Y$ is singular along the entire $L$. By the study of Gounelas and Kouvidakis a line as in item (2) corresponds to a triple type two line, and one as in item (3) corresponds to a type two line with two triple tangent 2-planes \cite[comments right above Remark~4.20]{GK-lines}. The distinction in cases comes from an elementary analysis of lines on non-normal cubic surfaces (see Appendix~\ref{appendix:cubic-surfaces}). The second statement in item (3) follows from \cite[Theorem~B]{GK-lines}. For the last item, let $L$ be a line of type $\II$ such that $E_L\cap Y$ is a singular cubic surface of type $X_9$, that is a cone over a cuspidal cubic curve. Then the lines passing trough the vertex of the cone form a curve, which contains a cuspidal cubic. This is in contradiction with \cite[Proposition 3.5]{GK-lines}. \end{proof} \section{The monodromy group of $\psi$ is maximal}\label{sec:monodromy} In this section we prove that the monodromy group of $\wh\psi$ is maximal. In order to do so, we use that the ramification of $\wh\psi$ is simple, as proven in Section~\ref{sec:ramification} and the results of Section~\ref{sec:variety-P} about the variety $P$. Given a generically finite dominant morphism $f\colon X\to Y$ of degree $d$ between irreducible varieties (necessarily of the same dimension), we obtain a degree d extension of function fields $k(X)/k(Y)$, and taking the Galois closure $K/k(Y)$ of this extension, we denote by $\mathrm{Gal}_f = \mathrm{Gal}(K/k(Y))$. This agrees with the usual monodromy group $\mathrm{Mon}_f$ (see \cite{Har,sottile-yahl}), which is defined as the image in $S_d$ of the group of deck transformations of the unramified (i.e., topological) cover $X\setminus f^{-1}(\mathrm{Branch}(f)) \to U$, where $U := Y \setminus\mathrm{Branch}(f)$. Recall the following classical results for $f\colon X\to Y$ and $U$ as above. \begin{proposition}[{\cite[p.698]{Har}}]\label{prop:transposition} If there exists a fibre of $f$ with a unique point of ramification index two and all other points unramified, then $\mathrm{Mon}_f \subset S_d$ contains a transposition. \end{proposition} Let $X^{[s]}_U$ be the complement of the big diagonal in the fibre product of $X_U$ $s$-times with itself over $U$. In other words, the fiber of the natural morphism $X^{[s]}_U \to U$ consists of $s$ distinct points in the fibre of $f$. \begin{lemma}[{\cite[ Proposition 2]{sottile-yahl}}] \label{lem:2-transitive} $X^{(s)}_U$ is irreducible if and only if $\mathrm{Mon}_f$ is an s-transitive subgroup of $S_d$. \end{lemma} Thanks to Corollary \ref{cor:ram-simple} we can readily apply Proposition \ref{prop:transposition} and deduce that the monodromy group $\mathrm{Mon}_{\wh\psi}$ contains a transposition. In order to apply Lemma \ref{lem:2-transitive} we are led to study the irreducibility of $\wh F^{(2)}_U$ which is strictly related to the variety $P$, we introduced in the previous section. The rest of the section is then devoted to prove the following which is the crucial ingredient to conclude the maximality of monodromy. \begin{proposition}\label{prop:irreducible} The variety $P$ is irreducible. \end{proposition} We postpone the proof of it and illustrate right away how to get the maximality of monodromy. \begin{theorem}\label{thm:monodromy-maximal} The monodromy of $\wh\psi$ is maximal, i.e. $\Mon_{\wh\psi} = S_{16}$. \end{theorem} \begin{proof} From Corollary \ref{cor:ram-simple} and Proposition \ref{prop:transposition} $\Mon_{\wh\psi}$ contains a transposition and thanks to Proposition \ref{prop:irreducible} and \ref{lem:2-transitive} the action of $\Mon_{\wh \psi}$ is $2$-transitive. Thus $\Mon_{\wh\psi}$ contains all transpositions: Indeed up to renumbering we can assume the transposition $(1,2)\in \Mon_{\wh \psi}$. By 2-transitivity there exists $\sigma\in\Mon_{\wh\psi}$ such that $\sigma(1)=a$ and $\sigma(2)=b$ for any $a$ and $b$ and we have \[ \sigma \circ (1,2) \circ \sigma^{-1} = (a,b). \] Hence $\Mon_\wh\psi$ contains any transposition and coincides with the entire symmetric group of 16 elements. \end{proof} In order to prove the irreducibility of $P$ we take advantage of the map $\varphi \colon P \dashrightarrow W$ and use its various properties we have showed in Section \ref{sec:variety-P}, to compute the ramification of $\varphi$ at some special points. Clearly there is an open subset $U_{et}\subset W$ for which the base change $P_{U_{et}} \to U_{et}$ is étale of degree 6 and proper, but as we are interested in the ramification of $\varphi$ we need to study a slightly bigger open subset, thus we provide the following technical proposition. \begin{proposition}\label{prop:flat-open} There is an open $U_{\mathrm{flat}}\subset W$ such that \begin{enumerate} \item The pullback $\varphi\colon P_{U_\flatt} \to U_\flatt$ is a flat finite morphism. \item The open set $U_\flatt$ contains a point in $w \in W$ such that the 3-dimensional linear space $a(w)$ intersects $Y$ in a cubic surface of type $A_1A_5$. \end{enumerate} \end{proposition} \begin{proof} Let $P \xleftarrow{\pi}\wh P \xrightarrow{\wh\varphi} W$ be any resolution of indeterminacy of $\varphi$ and $\wh P \xrightarrow{\varphi_1} P_{\mathrm{Stein}} \xrightarrow{\varphi_2} W$ be its Stein factorization. We denote by $E_\pi$ and $E_1$ the exceptional loci of $\pi$ and $\varphi_1$ respectively. \[ \wh{P}\setminus \wh \varphi^{-1}\left(\wh \varphi \left( E_1\cup E_\pi \right)\right) \to Y \setminus \wh \varphi (E_1 \cup E_\pi) =:U \] is proper, as base change of the proper map $\wh \varphi$, and with finite fibers, as we removed the exceptional divisor $E_1$). Thus it is a finite morphism. Having removed $E_\pi$ the source space is actually a subset of $P$. Hence, we have proven that the restriction of $\varphi$ to $P_U \to U$ is a finite morphism. Moreover, the further restriction of $\varphi$ to a regular locus of $P_U$ is a finite morphism of non singular varieties, hence flat by \cite[Chapter~4, Remark~3.11]{Liu}. This open set satisfies then point (1) of the claim.\medskip We now construct explicitly such an open set, for which we show point (2) of the claim. In order to do so, we consider the map $\varphi$ and the auxiliary map $\varphi_G$ \[ \begin{tikzcd}[row sep=tiny] P \ar[r,dashed, "\varphi"]\ar[rr, dashed, bend left, "\varphi_G"] & W\ar[r,"a"] & G. \end{tikzcd} \] Their graph closures are going to serve as an explicit resolution of indeterminacy. \begin{align*} \wh P:=\mathrm{closure}\{((\ell_1,\ell_2),w)\in P\times W \mid \varphi(\ell_1,\ell_2) = w \};\\ \wh P_G:=\mathrm{closure}\{((\ell_1,\ell_2),g)\in P\times G \mid \varphi_G(\ell_1,\ell_2) = g \}. \end{align*} As $\varphi_G = a\circ \varphi$, by continuity we have a natural map $\wh P \to \wh P_G$ given by \begin{align}\label{map-P--PG} \wh P \longrightarrow \wh P_G, \quad \left( \ell_1, \ell_2, w \right) \longmapsto \left( \ell_1, \ell_2, a(w) \right). \end{align} As before, we factor the resolution of indeterminacy $\wh \varphi \colon \wh P \to W$ in $\wh\varphi = \varphi_2\circ \varphi_1$ using Stein and we denote by $E_1$ the exceptional divisor of $\varphi_1$ and $E_\pi$ the exceptional divisor of $\pi\colon \wh P \to P$. By Corollary~\ref{cor:existence A1A5} there exists a point $w\in W$ such that $a(w)\in G$ is a point that intersects $Y$ in a surface $S$ with singularities $A_1A_5$. By Proposition~\ref{prop: lines-A1A5} $S$ contains exactly 2 lines $\ell_1$ and $\ell_2$, that by generality are of type $I$, such that $\psi(\ell_1) = \ell_2 = \psi (\ell_2)$. By the very definition of $\wh P_G$, its points consist of a pair of lines and a 3-dimensional linear subspace, which contains the 2 lines, hence \[ \wh\varphi_G^{-1} \big( a(w) \big) = \left\lbrace \Big( \big( \ell_1,\ell_2 \big), a(w) \Big),\ \Big( \big( \ell_2,\ell_1 \big), a(w) \Big) \right\rbrace. \] Now, as there is just one linear system of arithmetically Cohen-Macaulay generalized twisted cubics on a cubic surface of type $A_1A_5$ by Proposition~\ref{lem:gtc-on-A1A5} we get \[ \wh\varphi^{-1} \big( w \big) = \left\lbrace \Big( \big( \ell_1,\ell_2 \big), w \Big),\ \Big( \big( \ell_2,\ell_1 \big), w \Big) \right\rbrace. \] As $\wh\varphi^{-1}(w)$ is finite, we have $w\not \in \wh\varphi (E_1)$. Notice that $\varphi_G|_P:P\dashrightarrow G$ is defined by the association $(\ell_1,\ell_2)\mapsto \langle L_1,L_2\rangle = \langle L_1,L_2, R\rangle$ where $r = \psi(\ell_1) = \psi(\ell_2)$. Moreover, if both $\ell_1$ and $\ell_2$ are of type I, then we have $\varphi_G((\ell_1,\ell_2)) = \langle E_{\ell_1}, E_{\ell_2}\rangle$, hence pairs of distinct lines of type I in $P$ are not in the indeterminacy locus of $\varphi$. Since $Z_{aCM}(S)$ is a singleton for a cubic surface $S$ of type $A_1A_5$ by Lemma~\ref{lem:gtc-on-A1A5}, the map $\varphi$ is defined at the point $(\ell_1,\ell_2)$, thus we conclude that $w\not \in \wh\varphi(E_\pi)$. Now we are left to prove flatness of $\varphi$ at the points $(\ell_1, \ell_2)$ and $(\ell_2, \ell_1)$ in $P$. To do so we observe that $(\ell_1, \ell_2)$ is a regular point of $P$ as it follows from the lemma below. \end{proof} \begin{lemma}\label{lem:singular-locus-P} The singular locus of $\wh F \times_{\wh\psi} \wh F$ is contained in $E \times_{\wh\psi} E$. \end{lemma} \begin{proof} Let $(f_1,f_2)\in \wh F \times_{\wh\psi} \wh F$ be a point with $\wh \psi(f_1) = \wh \psi(f_2) = r$. Then its tangent space is the fiber product over the diagram \[ \begin{tikzcd}[row sep=tiny] T_{f_1} \wh F \ar[rd, "d\wh\psi"] && T_{f_2} \wh F \ar[ld,"d\wh\psi", swap]\\ & T_r F \end{tikzcd} \] where $T_{f_1} \wh F$ and $T_{f_2} \wh F$ have dimension 4 as $\wh F$ is a non-singular fourfold. If one among $f_1$ and $f_2$ is not in the ramification of $\wh \psi$, which coincide with the exceptional divisor $E$, then $d\wh\psi$ is an isomorphism at that point and the fiber product is a vector space of dimension 4, hence the claim. \end{proof} \begin{proof}(of Proposition~\ref{prop:irreducible}) First we are going to prove that $P$ modulo the switching involution $\sigma$ is irreducible. In order to do so, we start with a point $q\in U_{\mathrm{flat}}\cap W_{A_1A_5}$ which exists by Proposition~\ref{prop:flat-open}. As exactly 2 lines lie on a surface with singularities $A_1A_5$ (see Proposition~\ref{prop: lines-A1A5}), the preimage $\varphi^{-1}(q)$ consists of two points, say: \[ (\ell_1,\ell_2) \quad \text{and} \quad (\ell_2, \ell_1). \] Using the involution $\sigma$ on $F\times F$ switching the factors, which clearly restrict to an involution of $P$, we see that $P$ at $(\ell_1, \ell_2)$ is locally isomorphic to $P$ at $\sigma((\ell_1,\ell_2)) = (\ell_2, \ell_1)$. As $\varphi$ is flat, proper and of degree 6 at the point $q$, we conclude that the schematic preimage $\varphi^{-1}(q)$ is a finite scheme with multiplicity 3 both at $(\ell_1,\ell_2)$ and $(\ell_2,\ell_1)$. We pick now a point $p\in U_{\mathrm{et}}$ close enough to $q$ and focus on the preimages $\varphi^{-1}(p) = \{p_1,\cdots, p_6\}$: If we let $p$ tend to $q$, then 3 points out of the 6 in $\varphi^{-1}(p)$, say $p_1, p_2 $ and $p_3$, tend to $(\ell_1,\ell_2)$ and the other 3 points, i.e. $p_4,p_5$ and $p_6$, tend to $(\ell_2,\ell_1)$. Moreover, $\sigma$ maps the first triple to the second one, i.e. $\sigma(\{p_1,p_2,p_3\}) = \{p_4,p_5,p_6\}$, thus $p_1,p_2$ and $p_3$ lie in the same connected component of $P$, in other words the quotient of $P$ by the switching involution is path-connected. By Lemma \ref{lem:singular-locus-P} we deduce that it is irreducible too. Hence, it is now sufficient to show that for some point $p_1\in P_{U_{et}}$, the points $p_1$ and $\sigma(p_1)$ lie in the same connected component of $P_{U_{et}}$. To do so, we consider one transposition $\tau$ in $\Mon_\psi$, whose existence is granted by Proposition~\ref{prop:transposition} and Corollary~\ref{cor:ram-simple}. If we consider it as an element of deck transformations, this means that there exists a point $r\in F\setminus \Branch(\wh \psi)$, and a loop $\gamma$ centered at $r$ such that $\gamma$ acts on the fiber of $r$ exchanging two points, say $\ell_1\mapsto \ell_2$ and $\ell_2\mapsto \ell_1$, and as identity on the remaining other points of the fiber. We now consider the action under deck transformation of $\gamma$ for the natural map $P \dashrightarrow F$ defined by $\psi$. A moment's thought tells us that under the action of $\gamma$ the point $p=(\ell_1,\ell_2)\in P$, with $\psi(\ell_1)=\psi(\ell_2)=r$, is mapped to $(\ell_2,\ell_1)$. Thus $P$ is irreducible as we claimed. \end{proof} \section{Appendix}\label{appendix:cubic-surfaces} In this appendix we study singular cubic surfaces that are degenerations of a cubic surface with exactly 4 singular points of type $A_1$. These surfaces are called \textit{Cayley}. The stratification of the space of cubic surfaces given by the type of singularities has been studied by Bruce in \cite{bruce}. From his analysis Cayley cubics can degenerate to a singular surface of type $2A_1A_3$, which in turn degenerates to a singular surface $A_1A_5$, which in turn degenerates to surfaces with simple elliptic singularities and then to non-normal ones. Here is the diagram of degenerations: \[ 4A_1 \succeq 2A_1A_3 \succeq A_1A_5 \succeq \wt E_6 \succeq X_6 \succeq X_7 \succeq X_8 \succeq X_9 \] where $\wt E_6$ denotes the cubics with one simple elliptic singularity and $X_i$ are non-normal cubic surfaces. We discuss them in details in the subsequent subsections, focussing on lines and generalised twisted cubics and the restrictions of the Voisin maps $\varphi$, and $\psi$ to a cubic surface. \subsection*{Singular cubic surfaces of type $4A_1$} Under a suitable change of coordinate every cubic surface with 4 singular points of type $A_1$ can be expressed as zero locus of the following polynomial: \begin{align*} F = t_0t_1t_2 + t_0t_1t_3 + t_0t_2t_3 + t_1t_2t_3 \end{align*} Its singular locus consists of the coordinates points \begin{align*} p_0=(1:0:0:0),\quad p_1=(0:1:0:0),\quad p_2=(0:0:1:0),\quad p_3=(0:0:0:1). \end{align*} Such a surface can be obtained as the anticanonical model of the blow-up of $\mathbb P^2$ in special configuration of points. Indeed, let $\widetilde S$ be the blow-up of $P^2$ in the six points given by the intersection of 4 general lines $L_0, L_1,...,L_3$. Then the anticanonical linear system of $\widetilde S$ maps to a cubic surfaces $S$ and contracts the strict tranforms of $L_i$ to 4 singular $A_1$ points. The exceptional divisors map to lines in a tetrahedron-like disposition, we refer to them as edges, and denote $E_{ij}$ the line connecting the $i$-th and the $j$-th coordinate point. In particular, the edge $E_{ij}$ is cut by the equations $(t_k=t_l=0)$ where $\{i,j,k,l\}=\{1,2,3,4\}$. Besides the edges there are 3 more lines on $\bar S$ Joining opposite edges; we denote by $J_{ij,kl}$ the line intersecting the edges $E_{ij}$ and $E_{kl}$, where $\{i,j,k,l\}=\{1,2,3,4\}$. These are images of the strict transforms of lines through the blown-up points corresponding to the edges it intersects. For example, the line $J_{01,23}$ is the zero locus $(t_0+t_1 = t_2+t_3=0)$. Generalised twisted cubics on singular cubic surfaces have been studied in \cite[\S~2]{LLSvS}. Any generalised twisted cubic on an integral normal cubic surface moves in a 2-dimensional linear system and can be described as the image of the pull-back of a line in $\mathbb P^2$ along the diagram of a determinantal representation of $S$. The determinantal representation for the Cayley cubic described above defines $K\in Z'(S)$ which contains as reducible elements of the family, the union of all the edges passing through a fixed vertex of the tetrahedron: \[ K = [E_{ij}\cup E_{ik} \cup E_{il}] \in Z'(S). \] \begin{lemma} Opposite edges map to $K$ under $\varphi$, that is: \begin{align*} \varphi(E_{ij}, E_{kl}) = K\quad \text{ for indices such that } \{i,j,k,l\}=\{1,2,3,4\}. \end{align*} \end{lemma} \begin{proof} The proof is elementary and can be checked directly by the definition of the Voisin map $\varphi$. For example to compute $\varphi(E_{02}, E_{13})$, consider the plane $\pi=(t_1=0)$ containing the line $E_{02}$ and the point $p_\in E_{13}$. Its intersection with $S$ is given by $\pi\cap S=E_{02}\cup E_{03}\cup E_{23}$, hence $\varphi(E_{02}, E_{13}) = \left[E_{03}\cup E_{13} \cup E_{23}\right]$ and the claim is shown. The other cases are completely similar. \end{proof} \subsection*{Singular cubic surfaces $2A_1A_3$}\phantom. Let $S$ be a cubic surface with a singular point $P_2$ of type $A_3$, and 2 singular points $P_0,P_3$ of type $A_1$. It contains exactly 5 lines as depicted in Figure~\ref{fig:2A1A3}: the line $R_{01}$ through $P_0,P_1$; the line $R_{02}$ through $P_0,P_2$; the line $R_{03}$ through $P_{03}$; the line $L_1$ containing the only singular point $P_2$, and the line $L_2$ contained in the smooth locus. Such a surface is given in an opportune coordinate system by the equation \[ t_0t_2t_3 - t_1^2t_3-t_0t_1^2 = 0 \] Its singular locus consists of the point $P_2:=(0:0:1:0)$ of type $A_3$ and of the points $P_0:=(1:0:0:0)$, $P_3:=(0:0:0:1)$ of type $A_1$.\\ The lines are then described by the equations $R_{23}=\{t_0=t_1=0\},R_{02}=\{t_1=t_3=0\}$ and $R_{03}=\{t_1=t_2=0\}$, $L_1=(t_0=t_3=0)$, and $L_2=(t_2=t_0+t_3=0)$.\\ \begin{proposition} Let $S$ be a cubic surface of type $2A_1A_3$. Then: \begin{align*} L_1\mbox{ is residual to }R_{23},\qquad L_1 \mbox{ is residual to }R_{02} ,\\ L_2\mbox{ is residual to }R_{03},\qquad L_2\mbox{ is residual to } L_1. \end{align*} Moreover, there is no plane in $\langle S\rangle\simeq \mathbb P^3$ tangent to $S$ at every point of $L_2$. In particular, there are no triple lines on a surface with $2A_1A_3$ singularities. \end{proposition} \begin{proof} The proof can be checked through straightforward computations. For example, intersecting the surface $S$ with the plane $\pi=(t_0=0)$, one gets: \begin{align*} \pi\cap S &= (t_0=0)\cap (t_0t_2t_3 - t_1^2t_3-t_0t_1^2 = 0) \\ &= (t_0=0)\cap(t_1^2t_3=0) = 2R_{23} + L_1, \end{align*} which shows that $L_1$ is residual to $R_{23}.$ \end{proof} \subsection*{Singular cubic surfaces $A_1A_5$} A singular cubic surface $S$ with exactly one $A_1$ and one $A_5$ singularities is given in an opportune of coordinate system by the equation \[ t_3(t_0t_2-t_1^2)+t_0^3 = 0 \] whose singular locus consists of the 2 points $P_1:=(0:0:0:1)$ of type $A_5$ and $P_2:=(0:0:1:0)$ of type $A_1$. There are exactly 2 lines on $S$ and they have equations $L_1=\{t_0=t_1=0\}$ and $L_2= \{t_0=t_3=0\}$. \begin{proposition} Let $S$ be a cubic surface of type $A_1A_5$. Then \begin{align*} L_2 \text{ is triple, and } L_2 \text{ is residual to } L_1. \end{align*} \end{proposition} \begin{proof} Straightforward computation: The intersection \[ \{t_3=0\}\cap S = 3L_2. \] shows the first statement, while \[ \{t_0=0\}\cap S = L_1+2L_2 \] shows the second one. \end{proof} \subsection*{The cubic surface $X_6$} In a suitable system of coordinates, the surface $X_6$ is given by the equation \begin{align*} t_0^2t_2 + t_1^2t_3 = 0. \end{align*} It is a non-normal surface whose singular locus consists of the line $L=(t_0=t_1=0)$. We record only the following which is necessary for the proof of Corollary~\ref{cor:lineSecTyp} \begin{proposition} The line $L$ is not triple on $X_6$. \end{proposition} \begin{proof} The proof is an elementary computation: Each plane containing $L$ is of the form $\pi=(at_0 + bt_1=0)$ with $(a:b)\in \mathbb P^1$. By considering the intersection with such a plane $\pi$ one has: \begin{align*} X_6 \cap (-t_0 + bt_1 = 0) = (-t_0 + bt_1 = t_1^2(b^2t_2 + t_3) =0) = 2L + R \end{align*} with $R= b^2t_2 + t_3 = bt_1 + t_0 = 0$ different from $L$, and \begin{align*} X_6 \cap (t_1=0) = (t_1 = t_0^2t_2=0) =2L + R \end{align*} with $R= (t_1 = t_2 =0)$. This shows that $L$ is not triple. \end{proof} \subsection*{The cubic surface $X_7$} In a suitable system of coordinates, the surface $X_7$ is given by the equation \begin{align*} t_1^3 + t_2^3 + t_1t_2t_3. \end{align*} It is a non-normal surface isomorphic to the cone over a nodal planar cubic whose singular locus consists of the line $L=(t_1=t_2=0)$. We record only the following which is necessary for the proof of Corollary~\ref{cor:lineSecTyp} \begin{proposition} There is a unique plane $\pi$ making $L$ triple, i.e. such that $\pi\cap X_7 = 3L$. \end{proposition} \begin{proof} The proof is an elementary computation: Each plane containing $L$ is of the form $\pi =(at_1+bt_2=0)$ with $(a:b)\in \mathbb P^1$. By considering the intersection with such a plane $\pi$ one has: \begin{align*} X_7 \cap (t_0 - bt_1 = 0) = (t_0 - bt_1 = t_1^2(bt_2 + b^2t_3 + t_1) = 0) = 2L + R. \end{align*} with $R = (t_0 - bt_1 = t_1 + bt_2 + b^2t_3=0)$. Also, one has: \begin{align*} X_7 \cap (t_0 = 0) = (t_0 = t_1^3=0) = 3L. \end{align*} This shows the claim. \end{proof} \subsection*{The cubic surface $X_8$} In a suitable system of coordinates, the surface $X_8$ is given by the equation \begin{align*} t_0t_1t_2 + t_0^2t_3 + t_1^3 = 0. \end{align*} It is a non-normal surface whose singular locus consists of the line $L=(t_0=t_1=0)$. We record only the following which is necessary for the proof of Corollary~\ref{cor:lineSecTyp} \begin{proposition} There are 2 planes $\pi$ and $\pi'$ making $L$ triple, i.e. such that $\pi\cap X_8 = \pi'\cap X_8 = 3L$. \end{proposition} \begin{proof} The proof is an elementary computation: Each plane containing $L$ is of the form $\pi =(at_1+bt_2=0)$ with $(a:b)\in \mathbb P^1$. By considering the intersection with such a plane $\pi$ one has: \begin{align*} X_8 \cap (t_1 - bt_2 = 0) = (t_1 - bt_2 = t_2^2(b^3t_2 + t_2 + bt_3) = 0) = 2L + R. \end{align*} with $R = (t_0 - bt_1 = t_1 + bt_2 + b^2t_3=0)$. Also, one has: \begin{align*} X_8 \cap (t_2 = 0) = (t_1^3 = t_2=0) = 3L. \end{align*} The planes $\pi = (t_1=0)$ and $\pi'= (t_2=0)$ are hence the only planes as in the claim. \end{proof} \bibliography{literatur} \bibliographystyle{alpha} \end{document}
2412.07561v1
http://arxiv.org/abs/2412.07561v1
The $L_q$ Minkowski problem for $\mathbf{p}$-harmonic measure
\documentclass[12pt]{article} \usepackage{amsmath, graphicx, amsfonts,amssymb, calrsfs} \usepackage{amsfonts,mathrsfs, color, amsthm} \addtolength{\topmargin}{-0.1\textheight} \addtolength{\textheight}{0.2\textheight} \addtolength{\textwidth}{0.2\textwidth} \setlength{\oddsidemargin}{-0.08in} \setlength{\evensidemargin}{-0.08in} \pretolerance=5000 \usepackage{accents} \usepackage{enumitem} \usepackage{indentfirst} \usepackage{fancyhdr} \def\cK{\mathcal{K}} \def\sphere{\mathbb{S}^{n-1}} \def\N{\mathbb{N}} \def\Rn{{\mathbb R^n}} \def\R{\mathbb{R}} \def\cH{\mathcal{H}} \def\dV{\,d \widetilde{V}_K(u)} \def\theequation{\thesection.\arabic{equation}} \newtheorem{theorem}{Theorem}[section] \newtheorem{lemma}{Lemma}[section] \newtheorem{remark}{Remark}[section] \newtheorem{proposition}{Proposition}[section] \newtheorem{corollary}{Corollary}[section] \newtheorem{example}{Example}[section] \newtheorem{definition}{Definition}[section] \def\cC{\mathcal{C}} \def\bpf{\begin{proof}} \def\epf{\end{proof}} \def\be{\begin{equation}} \def\ee{\end{equation}} \def\bea{\begin{eqnarray}} \def\eea{\end{eqnarray}} \def\bt{\begin{theorem}} \def\et{\end{theorem}} \def\bl{\begin{lemma}} \def\el{\end{lemma}} \def\br{\begin{remark}} \def\er{\end{remark}} \def\bc{\begin{corollary}} \def\ec{\end{corollary}} \def\bd{\begin{definition}} \def\ed{\end{definition}} \def\bp{\begin{proposition}} \def\ep{\end{proposition}} \numberwithin{equation}{section} \begin{document} \title{The $L_q$ Minkowski problem for $\mathbf{p}$-harmonic measure \footnote{Keywords: Minkowski problem, convex body, $\mathbf{p}$-harmonic measure, $\mathbf{p}$-Laplacian. These authors contributed equally: Hai Li, Longyu Wu. $^\ddagger$Corresponding author: email: [email protected]}} \author{Hai Li$^\dagger$, Longyu Wu$^\dagger$, Baocheng Zhu$^{\dagger,\ \ddagger}$} \date{\quad} \maketitle \begin{abstract} In this paper, we consider an extremal problem associated with the solution to a boundary value problem. Our main focus is on establishing a variational formula for a functional related to the $\mathbf{p}$-harmonic measure, from which a new measure is derived. This further motivates us to study the Minkowski problem for this new measure. As a main result, we prove the existence of solutions to the $L_q$ Minkowski problem associated with the $\mathbf{p}$-harmonic measure for $0<q<1$ and $1<\mathbf{p}\ne n+1$. \vskip 2mm 2020 Mathematics Subject Classification: 31B05, 35J25, 42B37, 52A20, 52A40. \end{abstract} \section{Introduction}\label{sect:1} The $L_q$ Minkowski problem is one of the most important contents in convex geometry. It can be stated as: For any given $q\in \R$ and a finite nonzero Borel measure $\mu$ on the unit sphere $\sphere$ in $\R^n$, whether there exists a convex body whose $L_q$ surface area measure is the given measure $\mu$. When $q=1$, the $L_q$ Minkowski problem reduces to the classical one, which dates back to the early works by Minkowski and was developed further by Aleksandrov, Fenchel and Jessen. The $L_q$ Minkowski problem for $q>1$ was first studied by Lutwak \cite{L93}. Since then, this problem has received significant attention, leading to remarkable progress (see e.g., \cite{HS04, HZ05, LZ04, U03}). When $q<1$, the problem is more challenging (see e.g., \cite{C06, CW06, DZ12, JZ16, LW13, Z15}). Particularly for $q=0$, it becomes the logarithmic Minkowski problem (see e.g., \cite{BZ13, CL22, LX24, S02, S03, TX23, Z14}). For more progress on the $L_q$ Minkowski problem, we refer to \cite{CL20, HX15, M24} and the references therein. It is well known that the solutions to the $L_q$ Minkowski problem are key ingredients in the rapidly developing $L_q$ Brunn-Minkowski theory of convex bodies. For instance, they have played an important role in establishing affine Sobolev inequalities (see e.g., \cite{CZ09, HS09, LZ02, Z99}). Along with the rapid development of the Brunn-Minkowski theory, the Minkowski problem has been greatly enriched. Examples include the Minkowski problem for the dual curvature measure \cite{HZ16, LW20}, the Gaussian surface area measure \cite{CZ23, FX23, HZ21}, the chord measure \cite{GZ24, LZ24++, XZ23}, and the Minkowski problem for unbounded closed convex sets \cite{LZ24+, S18, S24, YZ23}, as well as for log-concave functions \cite{CK15, FY22, R22}. These problems are well-known for their close relationships among convex geometry, integral geometry, differential geometry, and PDEs. Jerison systematically integrated the Brunn-Minkowski theory with potential theory and the regularity theory of fully nonlinear equations. In his earlier works \cite{J89, J91}, he first studied the Minkowski problem for harmonic measure. Later, in another paper \cite{J96}, he examined a similar problem for electrostatic capacity. Jerison's contributions sparked significant research into Minkowski problems. A notable example of ongoing research is the study of the Minkowski problem for $\mathbf{p}$-capacity by Colesanti et al. \cite{CZ15}. Recently, this problem has been extended to the $L_q$ case \cite{ZX20}. In fact, such kind of Minkowski problem is closely related to a boundary value problem. More examples of Minkowski problems associated with the boundary value problems include those for capacity \cite{AV22, HZ18, LH23, X20, XX19} and for torsional rigidity \cite{CF10, HZ23, LZ20}. Let $K$ be a bounded convex domain with boundary $\partial K$ and $N$ be a neighborhood of $\partial K$. In this paper, we consider the following boundary value problem \begin{equation}\label{1.1} \left\{ \begin{aligned} &\text{div}\left({{\left|{\nabla u}\right|}^{\mathbf{p}-2} \nabla u}\right)=0&&\text{in}\ K\cap N,\\ &u>0&&\text{in}\ K,\\ &u=0&&\text{on}\ \partial K. \end{aligned} \right. \end{equation} Here, $N$ is chosen so that the solution $u_K$ satisfies $\left\|u_K\right\|_{L^\infty\left(\bar N\cap K\right)} +\left\|\nabla u_K\right\|_{L^\infty\left(\bar N\cap K\right)}<\infty$ and $\left|{\nabla u_K}\right|\ne0$ in $K\cap N$, where ${\left\|\cdot\right\|_{L^\infty}}$ is the ${L^\infty}$ norm, $\nabla$ is the gradient operator and $\bar N$ is the closure of $N$. Throughout this paper, we assume that $\partial N$ is of class $C^{\infty}$. Let $W^{1,\mathbf{p}}$ denote the usual Sobolev space with $1<\mathbf{p}<\infty$. Following Akman-Mukherjee \cite{AM24}, the $\mathbf{p}$-harmonic function $u_K\in W^{1,\mathbf{p}}\left(K\cap N\right)$ can be used to define the measure $\omega_\mathbf{p} =\left|\nabla u_K\right|^{\mathbf{p}-1} \mathcal{H}^{n-1}\llcorner_{\partial K}$. Moreover, the $\mathbf{p}$-harmonic measure $\mu_K$ is defined by $\mu_K=(g_K)_*\omega_\mathbf{p}$, that is, \begin{equation}\label{1.2} \mu_K\left(E\right) =\int_{g_K^{-1}\left(E\right)} {\left|\nabla u_K\right|}^{\mathbf{p}-1} d{\mathcal H}^{n-1} \end{equation} for any Borel set $E$ on the unit sphere $\mathbb{S}^{n-1}$, where $g_K:\partial K\to\mathbb{S}^{n-1}$ is the Gauss map and $\mathcal{H}^{n-1}$ is the $(n-1)$-dimensional Hausdorff measure. According to Akman-Mukherjee \cite{AM24}, the definition \eqref{1.2} is valid for any convex set, and the $\mathbf{p}$-harmonic measure is of variation meaning. In fact, the $\mathbf{p}$-harmonic measure has been studied by Lewis et al. \cite{L06, L13}, and Jerison's work \cite{J91} on harmonic measure has been nontrivially extended to the $\mathbf{p}$-harmonic measure setting by Akman-Mukherjee \cite{AM24}. By studying the discrete measure case and using the approximation arguments, Akman-Mukherjee \cite{AM24} demonstrated the solvability of the Minkowski problem for $\mathbf{p}$-harmonic measure, provided that the given measure is not concentrated on any great subsphere and its centroid is at the origin. Recently, smooth solutions have been established by using the Gauss curvature flow \cite{LZ24}. Detailed discussions on the relationships among the Minkowski problem for $\mathbf{p}$-harmonic measure, harmonic measure \cite{J91}, and $\mathbf{p}$-capacitary measure \cite{CZ15} can be found on page 13 of \cite{AM24}. In this paper, we focus on the following problem concerning the $\mathbf{p}$-harmonic measure, where $1<\mathbf{p}<\infty$, unless specified otherwise. \vskip.2cm \textbf{$L_q$ Minkowski problem for $\mathbf{p}$-harmonic measure.} {\it Let $q\in\mathbb{R}$ and $\mu$ be a finite Borel measure on $\mathbb{S}^{n-1}$. What are the necessary and sufficient conditions for $\mu$ such that there exists a convex body $\Omega$ satisfying $\mu=h_{\Omega}^{1-q}\mu_\Omega$? Here $h_{\Omega}$ is the support function of $\Omega$. } \vskip.2cm Actually, the measure $h_{\Omega}^{1-q}\mu_\Omega=\mu_{\Omega,q}$ in the above problem can be derived from our new variational formula (see Theorem \ref{th:3.1} below), and we call it the $L_q$ $\mathbf{p}$-harmonic measure. As mentioned above, the $L_1$ Minkowski problem for $\mathbf{p}$-harmonic measure was recently studied by Akman-Mukherjee \cite{AM24}. By studying an extremal problem for a functional related to the $\mathbf{p}$-harmonic measure, we can obtain a solution to the $L_q$ Minkowski problem for $\mathbf{p}$-harmonic measure for $0<q<1$. This can be stated as main result of this paper as follows. \begin{theorem}\label{th:1.1} Let $0<q<1$, $1<\mathbf{p}\ne n+1$, and $\mu$ be a finite Borel measure on $\mathbb{S}^{n-1}$. If $\mu$ is not concentrated on any closed hemisphere, there exists a convex body $\Omega$ containing the origin in its interior so that $\mu=c\mu_{\Omega,q}$, where $c$ is a positive explicit constant. In particular $c=1$, if $\mathbf{p}\ne n+1-q$. \end{theorem} This paper is organized as follows. In Section \ref{sect:2}, we review some necessary notations and background on convex sets, $\mathbf{p}$-harmonic functions and $\mathbf{p}$-harmonic measures. In Section \ref{sect:3}, after establishing a variational formula associated with the $\mathbf{p}$-harmonic measure, we further introduce the $L_q$ $\mathbf{p}$-harmonic measure for $q\in\mathbb{R}$ and prove its weak convergence. In Section \ref{sect:4}, we complete the proof of Theorem \ref{th:1.1}. \section{Preliminaries}\label{sect:2} \subsection{Background for convex sets}\label{subsect:2.1} In this subsection, we collect the necessary background, notations and preliminaries. More details on convex sets can be found in \cite{G06, G07, S14}. Let $K\subset \mathbb{R}^{n}$ be a convex set with boundary $\partial K$, one can define the multi-valued Gauss map $g_K:\partial K\to\mathbb{S}^{n-1}$ by \begin{equation}\label{2.1} {g_K}\left(x\right) =\left\{{\xi\in {\mathbb{S}^{n-1}}: \left\langle{y-x,\xi }\right\rangle< 0\ \text{for all}\ y\in K}\right\}, \end{equation} i.e., the set of all unit outward normal vectors at $x\in \partial K$, where $\left\langle{\cdot, \cdot}\right\rangle$ is the standard inner product on $\mathbb{R}^{n}$. The set defined in \eqref{2.1} is a singleton for $\mathcal{H}^{n-1}$-a.e. $x\in\partial K$. For a measurable subset $E\subset\mathbb{S}^{n-1}$, let $g_K^{-1}(E):=\{{x\in\partial K:g_K(x)\cap E\ne\emptyset}\}$ be the inverse image of $g_K$, and ${\left(g_K\right)_*}$ be the push forward of $g_K$ given by \[\left({{\left(g_K\right)}_*}\mu\right)\left(E\right) =\mu\left({g_K^{-1}\left(E\right)}\right),\] where $\mu$ is a measure defined on any measurable subsets of $\partial K$. If $E$ is a Borel subset of $\mathbb{S}^{n-1}$, $g_K^{-1}\left(E\right)$ is $\mathcal{H}^{n-1}$-measurable. For a compact convex set $K\subset\mathbb{R}^{n}$ and nonzero $x\in\mathbb{R}^{n}$, the support function of $K$ is defined by $h_K\left(x\right) =\max\limits_{y\in K}\left\langle {x,y}\right\rangle$, and the support hyperplane of $K$ is given by $${H_K}(x) =\left\{{y\in {\mathbb{R}^n}:\left\langle{x,y} \right\rangle ={h_K}(x)}\right\}.$$ If $K\cap{H_K}\left( x \right)$ consists of only a single point for all $x$, then $K$ is strictly convex. In particular, a convex and compact subset in $\Rn$ with nonempty interior is called a convex body. A convex set $K$ is said to be of class $C_+^2$ (resp. $C_ +^{2,\alpha }$ for $\alpha\in\left({0,1}\right]$) if $\partial K$ is of class $C_+^2$ (resp. $C_+^{2,\alpha}$) and the Gauss map $g_K: \partial K\to\mathbb{S}^{n-1}$ is a diffeomorphism. For any convex set $K$ of class $C_+^{2}$, we have $K\cap {H_K}\left( {{g_K}\left( x \right)} \right) =\left\{ x \right\}$, where $x\in \partial K$. Moreover, the support function is differentiable and \[\nabla {h_K}\left( {{g_K}\left( x \right)} \right) = x,\] where $\nabla $ is the gradient operator on $\mathbb{R}^{n}$. For $\xi \in \mathbb{S}^{n-1}$, there exists an orthonormal basis $\left\{ {{e^1}, \ldots ,{e^{n - 1}},\xi } \right\}$ of $\mathbb{R}^{n}$, where $\left\{ {e^i}\right\}$ spans the tangent space ${T_\xi }\left(\mathbb{S}^{n-1}\right)$. Then, for any $x\in \mathbb{R}^{n}$, we have the decomposition \begin{equation}\nonumber x=\sum\limits_{i=1}^{n-1}x^ie^i +\left\langle{x,\xi}\right\rangle\xi\ \ \text{with}\ \ x^i=\left\langle x,e^i\right\rangle. \end{equation} Let $\xi ={g_K}\left( x \right)$ for any $x\in\partial K$, then we have \begin{equation}\label{2.2} \nabla {h_K}\left(\xi\right) =\sum\limits_{i = 1}^{n - 1} {{\bigtriangledown_i}{h_K}\left( \xi \right){e^i}} +\left\langle {\nabla {h_K}\left( \xi \right),\xi } \right\rangle \xi, \end{equation} where ${\bigtriangledown _i}{h_K}\left(\xi\right) =\left\langle {\nabla {h_K}\left(\xi\right),{e^i}}\right\rangle$. Let $\mathcal{A}_+^{2,\alpha}$ be the set of all compact convex sets that are of class $C_+^{2,\alpha}$. For a sequence of compact convex sets $\left\{\Omega_j\right\}_{j=0}^{\infty}$, we say that $\Omega_j$ converges to $\Omega_0$ and denote it as $\Omega_j\to \Omega_{0}$, if the Hausdorff distance $d_{\mathcal H}\left({\partial \Omega_j,\partial\Omega_0}\right)$ between ${\Omega_j}$ and $\Omega_{0}$ converges to $0$ as $j \to\infty$. According to Theorem 2.46 of \cite{AM24}, for any compact convex set $\Omega$ with Gaussian curvature $\kappa$, there exists a sequence $\left\{\Omega_j \right\}_{j=1}^\infty\subset\mathcal{A}_+^{2,\alpha}$ with Gaussian curvature $\kappa_{j}$ such that $\Omega_{j}\to \Omega$, and for any continuous function $f$ defined on the unit sphere $\mathbb{S}^{n-1}$, \begin{equation}\nonumber \int_{\mathbb{S}^{n-1}}\frac{f\left(\xi\right)}{\kappa_j\left({g_{\Omega_j}^{-1}\left(\xi\right)}\right)}d\xi \to \int_{\mathbb{S}^{n-1}}\frac{f\left(\xi\right)} {\kappa\left({g_\Omega^{-1}\left(\xi\right)}\right)}d\xi, \end{equation} as $j\to\infty$. Let $C\left(E\right)$ denote the set of all continuous functions defined on subset $E\subset\mathbb{S}^{n-1}$ and let $C_{+}\left(E\right)\subset C\left(E\right)$ denote the set of all strictly positive functions. The Wulff shape $K_f$ associated with a nonnegative function $f\in C\left(E\right)$ is defined by \begin{equation}\nonumber {K_f} =\left\{{x\in\mathbb{R}^{n}:\left\langle {x,\xi}\right\rangle \le f\left(u\right)}\ \text{for all}\ \xi\in E\right\}. \end{equation} Let $\mathcal K_o^n$ be the set of convex bodies containing the origin $o$ in their interiors. A well-known fact is that $K_f\in\mathcal K_o^n$ if $f\in C_{+}\left(\mathbb{S}^{n-1}\right)$, and $h_{K_f}=f$ almost everywhere with respect to the surface area measure of $K_f$. Schneider \cite{S14} proved that if $\{f_j\}_{j=1}^\infty\subset C_{+}\left(\mathbb{S}^{n-1}\right)$ converges to $f\in C_{+}\left(\mathbb{S}^{n-1}\right)$ uniformly as $j\to\infty$, then the sequence $\{K_{f_j}\}$ is also convergent in the sense of the Hausdorff metric, i.e., \begin{equation}\label{2.3} K_{f_j}\to K_f,\ \text{as}\ j\to\infty. \end{equation} \subsection{The $\mathbf{p}$-harmonic functions and $\mathbf{p}$-harmonic measures}\label{subsect:2.2} We now review some properties of the $\mathbf{p}$-harmonic function, which are also referenced in \cite{AM24} for more details. The $\mathbf{p}$-harmonic functions minimize the $\mathbf{p}$-Dirichlet energy $\int_{K}{\left|{\nabla u}\right|}^\mathbf{p}dx$ and are weak solutions to the $\mathbf{p}$-Laplacian equation $\Delta_\mathbf{p}u=\text{div}\left({{\left|{\nabla u}\right|}^{\mathbf{p}-2} \nabla u}\right)=0$ in a convex domain $K$. The existence of a weak solution $u_K\in W^{1,\mathbf{p}}\left(K\right)$ to $\Delta_\mathbf{p}u=0$ in $K$, with boundary condition $u=f$ on $\partial K$, is known. The uniqueness of the weak solution follows directly from the comparison principle, while the regularity theory presents more complex challenges. Let $K\in \mathcal{A}_+^{2,\alpha}$ and $f\in C^{1,\alpha}\left(\partial K\right)$, it follows from \cite{L88} that $u_K\in C^{1,\beta}\left(\bar{K}\right)$ for some $\beta(n,\mathbf{p},\alpha)\in(0,1)$. Tolksdorf \cite{T84} has proved that the weak solutions to $\Delta_\mathbf{p}u = 0$ in $K$ are locally $C^{1,\beta}$ for some $\beta(n,\mathbf{p})\in \left(0,1\right)$. This shows that for any compact subset $K^\prime\subset \subset K$, the weak solutions are continuously differentiable on $K^\prime$ and their first derivatives are H\"older continuous. Hence, the weak solution $u$ to \eqref{1.1} belongs to $C^{1,\beta}(\bar K\cap N)$. Since $\left|{\nabla u}\right|\ne0$ in $K\cap N$, the $\mathbf{p}$-Laplacian operator is uniformly elliptic in $K\cap N$. It follows from the boundary Schauder estimates \cite{GT01} that the Hessian matrix $D^{2}u$ is well-defined on $\partial K$. Let $u_{K_j}$ be the weak solution to \eqref{1.1} for $K_j$. Then, by Proposition 3.65 of \cite{AM24}, $\nabla u_{K_j}\to\nabla u_K$ uniformly in $N$, if $K_j\to K$. For the $\mathbf{p}$-harmonic function, we provide two important lemmas. The first one can be stated as follows. \begin{lemma}\label{lem:2.1} Let $K$ be a bounded convex domain containing the origin and $u$ be the solution to \eqref{1.1}, there exists a constant $M>0$, independent of $K$, such that \begin{equation}\nonumber \left|{\nabla u}\right|\le M\ \mathrm{on}\ \partial K. \end{equation} \end{lemma} \begin{proof} By Theorem 2.46 of \cite{AM24}, for any convex domain $K$, there exists a sequence of convex domains $\{K_j\}\subset\mathcal{A}_+^{2,\alpha}$ that converges to $K$ as $j\to\infty$. Thus, we only need to consider the case that $K\in \mathcal{A}_+^{2,\alpha}$. Let $u$ be a solution to the boundary value problem \begin{equation}\label{2.4} \left\{ \begin{aligned} &\text{div}\left({{\left|{\nabla u}\right|}^{\mathbf{p}-2} \nabla u}\right)=0&&\text{in}\ K\setminus \bar\Omega_0,\\ &u>0&&\text{in}\ K,\\ &u=0&&\text{on}\ \partial K,\\ \end{aligned} \right. \end{equation} where $\bar\Omega_0:=K\setminus N$. If $u=1$ in $\bar\Omega_0$, it follows from page 204 of \cite{L77} that $u$ is a $\mathbf{p}$-capacity function of $K\setminus\bar\Omega_0$. By Theorem 2 of \cite{CS03}, we conclude that $u\in C^{\infty}\left(K \setminus \bar\Omega_0\right) \cap C\left(K \setminus \Omega_0\right)$, $0<u<1$ in $K \setminus\bar\Omega_0$ and $K_s= \left\{ {x\in K:u(x)\ge s} \right\}$ is convex for $0\le s\le 1$. Since $\left| {\nabla u\left( x \right)} \right| > 0$ in $K \setminus \bar \Omega_0$, by Theorem 4 of \cite{CS03}, we obtain \begin{equation}\label{2.5} - \frac{{\partial {h_{{K_s}}}\left( {{{-\nabla u\left( x \right)} \mathord{\left/ {\vphantom {{\nabla u\left( x \right)} {\left| {\nabla u\left( x \right)} \right|}}} \right. \kern-\nulldelimiterspace} {\left| {\nabla u\left( x \right)} \right|}}} \right)}}{{\partial s}} =\frac{1}{{\left| {\nabla u\left( x \right)} \right|}}, \end{equation} for all $x\in\partial K_s$. By applying Proposition 1 of \cite{CS03}, we further have \[\frac{{\partial {h^2_{{K_s}}}\left( {{{-\nabla u\left( x \right)} \mathord{\left/ {\vphantom {{\nabla u\left( x \right)} {\left| {\nabla u\left( x \right)} \right|}}} \right. \kern-\nulldelimiterspace} {\left| {\nabla u\left( x \right)} \right|}}} \right)}}{{\partial s^{2}}} \ge 0,\] thus $\frac{{\partial {h_{{K_s}}}\left( {{{-\nabla u\left( x \right)} \mathord{\left/ {\vphantom {{\nabla u\left( x \right)} {\left| {\nabla u\left( x \right)} \right|}}} \right. \kern-\nulldelimiterspace} {\left| {\nabla u\left( x \right)} \right|}}} \right)}}{{\partial s}} $ is non-decreasing for every fixed $x$. This, together with \eqref{2.5}, shows that $\left|{\nabla u\left(x\right)}\right|$ attains its maximum on $\partial\bar\Omega_0$. Let $B_r$ be a ball with radius $r$ included in $\bar\Omega_0$ and internally tangent to $\partial\bar\Omega_0$ at $x\in\partial\bar\Omega_0$, and let $v$ be a solution to the equation \eqref{2.4} with $\bar \Omega_0$ replaced by $B_r$. As $B_{r}\subset \bar \Omega_0$, we have $K \setminus \bar \Omega_0 \subset K \setminus B_{r}$, thus \begin{equation*} \left\{ \begin{aligned} &\Delta_{\mathbf{p}}u= \Delta _{\mathbf{p}}v && \text{in}\ K \setminus\bar\Omega_0,\\ &u=v=0\ &&\text{on}\ \partial K,\\ &v\le u\ &&\text{on}\ \partial \Omega_0.\\ \end{aligned} \right. \end{equation*} Then, by the comparison principle (cf. Theorem 2.1 of \cite{G13}), $v\le u$ on $K \setminus\bar\Omega_0$. This, combined with $u(x)=v(x)$, implies that $\left|{\nabla u\left(x\right)}\right|\le\left|{\nabla v\left(x\right)} \right|$ for $x\in\partial\bar\Omega_0$. Then, we can calculate the value of $\left|{\nabla v\left(x\right)}\right|$ and obtain a positive constant $m$ depending on $r$ and $n$ such that \begin{equation}\label{2.6} \left|{\nabla u}\right|\le m \end{equation} in $K\setminus\bar\Omega_0$. Moreover, since $u\in C^{1,\beta}\left({\bar K\cap N}\right)$ with $\beta=\beta(n,\mathbf{p},\alpha)$, it follows that $\nabla u$ is $\beta$-H\"older continuous. Then, there exists a constant $\Lambda>0$ such that $$ \left| {\nabla u\left( y \right)-\nabla u\left(z\right)}\right|\le \Lambda{\left|{y-z} \right|^\beta} $$ for $y,z\in {\bar K\cap N}$. Thus, we have \[\left| {\nabla u\left(z \right)} \right| \le \Lambda {\left| {y-z} \right|^\beta } + \left| {\nabla u\left(y\right)} \right|\] for any $z\in \partial K$ and $y\in K\cap N$. This, together with \eqref{2.6} and the boundedness of ${\bar K\cap N}$, shows that there exists a finite positive constant $M$, independent of $K$, such that $$|{\nabla u\left(z\right)}|\le M$$ for all $z\in\partial K$. This completes the proof of Lemma \ref{lem:2.1}. \end{proof} The second order covariant derivative of $h_K:\mathbb{S}^{n-1}\to\mathbb{R}$ is locally given by $${\bigtriangledown}^2{h_K} =\sum\limits_{{i,j= 1}}^{n-1}(\bigtriangledown_{i,j}h_K) e^i\otimes e^j,$$ where $\bigtriangledown_{i,j}h_K(x)=\partial_{i,j}(h_K\circ\varphi^{-1})(\varphi(x))$ with $U\subset\mathbb{S}^{n-1}$ and $\varphi:U\to V\subset\mathbb{R}^{n-1}$ being a coordinate chart. Let $\mathbb{I}$ be the unit matrix of order $(n-1)$ and $C[\bigtriangledown^2h_K+h_K\mathbb{I}]$ be the cofactor matrix of $\left({{\bigtriangledown^2}{h_K}+{h_K}{\mathbb{I}}}\right)$ with element ${C_{i,j}}\left[\cdot\right] =\left\langle{C\left[\cdot\right]{e^j},{e^i}}\right\rangle$. The following lemma directly follows from Lemma 3.44 of \cite{AM24}. \begin{lemma}\label{lem:2.2} Let $\left\{{e^1,\ldots,e^{n-1},\xi}\right\}$ be an orthonormal basis of $\mathbb{R}^{n}$, and let $u$ be the solution to \eqref{1.1} for a convex domain $K$ that is of class $C_+^{2,\alpha}$. Then we have \begin{enumerate}[label=\upshape(\roman*)] \item $\left\langle {{D^2}u\left( {\nabla {h_K}\left( \xi \right)} \right)\xi ,\xi } \right\rangle = \frac{1}{{\mathbf{p}-1}}\kappa \left( {\nabla {h_K}\left( \xi \right)} \right)\left| {\nabla u\left( {\nabla {h_K} \left( \xi \right)} \right)} \right|{\rm{Tr}}\left( {C\left[ {{\bigtriangledown ^2}{h_K} + {h_K}{\mathbb{I}}} \right]} \right)$, \item $\left\langle {{D^2}u\left( {\nabla {h_K}\left( \xi \right)} \right){e^i},\xi } \right\rangle =-\kappa \left( {\nabla {h_K}\left( \xi \right)} \right) \sum\limits_{j = 1}^{n - 1} {{C_{i,j}}\left[ {{\bigtriangledown ^2}{h_K} + {h_K}{\mathbb{I}}} \right]} {\bigtriangledown _j} \left( {\left| {\nabla u\left( {\nabla {h_K}\left( \xi \right)} \right)} \right|} \right)$. \end{enumerate} \end{lemma} At the end of this subsection, we review the weak convergence of the $\mathbf{p}$-harmonic measure. Let $u\in W^{1,\mathbf{p}}\left(K\cap N\right)$ be a $\mathbf{p}$-harmonic function, a solution to \eqref{1.1} in $K\cap N$. Following Akman-Mukherjee \cite{AM24}, one can define the $\mathbf{p}$-harmonic measure \begin{equation}\nonumber {\mu_{\Bar K}}\left(E\right) ={\mu_K}\left(E\right) =\int_{g_K^{-1}\left(E\right)}{\left| {\nabla u\left(x\right)} \right|}^{\mathbf{p}-1}d{\mathcal{H}}^{n-1}\left(x\right), \end{equation} where $E\subset\mathbb{S}^{n-1}$ is a Borel subset. If $K\in \mathcal{A}_+^{2,\alpha }$, we have $\nabla h_K\left(\xi\right)=g_K^{-1}\left(\xi\right)$, and we can use the transformation rule of the Jacobian (cf. page 8 of \cite{AM24}) to obtain \begin{equation}\label{2.7} (g_K)_*\mathcal{H}^{n- 1}\llcorner_{\partial K} =|\det\left({\bigtriangledown}^2h_K+h_K{\mathbb{I}}\right)| \mathcal{H}^{n- 1}\llcorner_{\mathbb{S}^{n-1}} =\frac{1}{\left(\kappa\circ g_K^{-1}\right)} \mathcal{H}^{n- 1}\llcorner_{\mathbb{S}^{n-1}}. \end{equation} Therefore, \begin{equation}\nonumber \begin{split} d{\mu _K} = {\left| {\nabla u\left( {\nabla {h_K}\left(\xi\right)} \right)} \right|^{\mathbf{p}-1}}d{\mathcal{H}^{n - 1}} \llcorner_{\partial K}={\left| {\nabla u\left( \nabla {h_{{K}}}\left(\xi\right) \right)} \right|^{\mathbf{p}-1}}\det \left( {{\bigtriangledown ^2}{h_K} + {h_K}\mathbb{I}} \right)d\xi. \end{split} \end{equation} For a compact convex set $K$ and a sequence of compact convex sets $\left\{K_{j}\right\}$ with $K_{j}\to K$ as $j\to\infty$, Akman-Mukherjee \cite{AM24} proved that \begin{equation}\label{2.8} \mathop{\lim}\limits_{j\to\infty } \int_{\mathbb{S}^{n-1}}{f\left(\xi\right)}d\mu_{K_j}\left(\xi\right) =\int_{\mathbb{S}^{n-1}}{f\left(\xi\right)} d{\mu_K}\left(\xi\right) \end{equation} for any $f \in C\left(\mathbb{S}^{n-1}\right)$. This shows that the $\mathbf{p}$-harmonic measure is weakly convergent. Moreover, it can be checked that the centroid of the $\mathbf{p}$-harmonic measure is at the origin. \begin{lemma}\label{lem:2.3} Let $K$ be a bounded convex domain, then for any $x_0\in \mathbb{R}^{n}$, $$ \int_{{\mathbb{S}^{n - 1}}} {\left\langle {{x_0},\xi } \right\rangle } d{\mu_K}(\xi)=0. $$ \end{lemma} \begin{proof} Let $u_K$ be a weak solution to the $\mathbf{p}$-Laplace equation in $K \cap N$, or equivalently, \begin{equation}\label{2.9} \int_{K\cap N} {{{\left|\nabla u_K(x)\right|}^{\mathbf{p}-2}} \left\langle{\nabla u_K(x),\nabla\phi(x)}\right\rangle}dx =0 \end{equation} for any smooth function $\phi$ defined in $K\cap N$ with compact support. Consider the boundary value problem \eqref{1.1} and let $f$ be a function in $C^\infty\left(\overline{K\cap N}\right)$ such that $f=u_K$ on $\partial N\cap K$ and $f=1$ on $\partial K$. Notice that $$g_K(x)=-\frac{\nabla u_K(x)}{\left|{\nabla u_K(x)}\right|},$$ then for any $x_0\in \mathbb{R}^{n}$, we have the following calculation: \begin{equation*} \begin{split} &\int_{{\mathbb{S}^{n - 1}}} {\left\langle {{x_0},\xi } \right\rangle } d{\mu_K}\left( \xi \right)\\ =&\int_{{\mathbb{S}^{n - 1}}} {\left\langle {{x_0},\xi } \right\rangle } {\left| {\nabla {u_K} \left( {g_K^{ - 1}\left( \xi \right)} \right)} \right|^{{\mathbf{p}} - 1}}d{S_K}\left( \xi \right)\\ =&\int_{\partial K} {{{\left| {\nabla {u_K}\left( x \right)} \right|}^{{\mathbf{p}} - 1}} \left\langle {{x_0}, g_K(x)} \right\rangle } d{\mathcal{H}^{n - 1}}\\ =&\int_{\partial K} {{{\left| {\nabla {u_K}\left( x \right)} \right|}^{{\mathbf{p}} - 2}}} \left\langle {\nabla {u_K}\left( x \right),{g_K}\left( x \right)} \right\rangle \left\langle {{x_0}, g_K(x)} \right\rangle \left( {u_K\left( x \right)-f\left( x \right) } \right)d{\mathcal{H}^{n - 1}}\\ &+\int_{\partial N \cap K} {{{\left| {\nabla {u_K}\left( x \right)} \right|}^{{\mathbf{p}} - 2}}} \left\langle {\nabla {u_K}\left( x \right),{\nu _{\partial N \cap K}}\left( x \right)} \right\rangle \left\langle {{x_0}, g_K(x)} \right\rangle \left( {u_K\left( x \right)-f\left( x \right) } \right)d{\mathcal{H}^{n - 1}}\\ =&\int_{\partial \left( {K \cap N} \right)} {{{\left| {\nabla {u_K}\left( x \right)} \right|}^{{\mathbf{p}} - 2}}\left\langle {\nabla {u_K}\left( x \right),{\nu _{\partial \left( {K \cap N} \right)}}\left( x \right)} \right\rangle \left\langle {{x_0}, g_K(x)} \right\rangle \left( {u_K\left( x \right) - f\left( x \right)} \right)} d{\mathcal{H}^{n - 1}}\\ =&\int_{K \cap N} {\text{div}\left( {{{\left| {\nabla {u_K}\left( x \right)} \right|}^{{\mathbf{p}} - 2}}\nabla {u_K}\left( x \right)\left\langle {{x_0}, g_K(x)} \right\rangle \left( {u_K\left( x \right) - f\left( x \right)} \right)} \right)} dx\\ =& 0,\\ \end{split} \end{equation*} where we have used the divergence theorem and \eqref{2.9}. This proves the desired property. \end{proof} \section{The variational formula associated with $\mathbf{p}$-harmonic measure}\label{sect:3} Associated with the $\mathbf{p}$-harmonic measure $\mu_K$ of a compact convex set $K\subset\R^n$, Akman-Mukherjee \cite{AM24} introduced a continuous functional \begin{equation}\label{3.1} \Gamma\left(K\right) =\int_{\mathbb{S}^{n-1}}h_K\left(\xi\right) d{\mu_K}\left(\xi\right). \end{equation} By Lemma \ref{lem:2.3}, it can be verified that the functional $\Gamma(\cdot)$ is translation invariant. That is, for any $x_0\in \mathbb{R}^{n}$, \begin{equation}\label{3.2} \Gamma \left({K+x_0}\right)=\Gamma\left(K\right). \end{equation} In the following part of this section, we will focus on calculating the variation of $\Gamma\left(K\right)$ with respect to the $q$-sum for $q>0$ and introduce the $L_q$ $\mathbf{p}$-harmonic measure. To do so, we will briefly review the concept of the $q$-sum. Let $K$ and $L$ be two compact convex sets containing the origin. For $q\ge1$ and $t\ge 0$, Firey's $q$-sum $K^t$ can be defined by $h_{K^t}^q=h_K^q+th_L^q$ on $\mathbb{S}^{n-1}$. Following B\"or\"oczky et al. \cite{BZ12}, the $q$-sum $K^t$ for $0<q<1$ can be defined as the Wulff shape of the function $\left(h_K^q+ th_L^q\right)^{\frac{1}{q}}$, that is \begin{equation}\label{3.3} {K^t}=\left\{{x\in \mathbb{R}^{n}:\left\langle {x,\xi}\right\rangle \le{\left( {h_K^q\left(\xi\right) +th_L^q\left(\xi\right)} \right)}^{\frac{1}{q}}}\ \text{for all}\ \xi\in\mathbb{S}^{n-1}\right\}. \end{equation} In this case, $h_{K^t}^q=h_K^q+th_L^q$ holds almost everywhere on $\mathbb{S}^{n-1}$ with respect to the surface area measure $S_{K^t}$ of $K^t$. Thus, we have $S_{K^t}\left(\omega_t\right)=0$, where \[{\omega_t}=\left\{\xi\in {\mathbb{S}^{n- 1}}:h_{{K^t}}^q(\xi)\ne h_K^q (\xi)+th_L^q(\xi)\right\}.\] Let $K,L\in \mathcal{A}_+^{2,\alpha}$ and $q>0$. We take a small enough \begin{equation}\label{3.4} \tau :=\tau\left(d_{\mathcal H}\left({\partial K,\partial N}\right), d_{\mathcal H}\left({\partial L,\partial N}\right), \left\|u\right\|_{W^{1,\mathbf{p}}\left(N\right)}\right) >0, \end{equation} where $u$ is the solution to \eqref{1.1}, such that ${K^t}\in\mathcal{A}_+^{2,\alpha}$, $\partial K^{t}\subset N$, and $K^{t}\cap\partial N=K\cap\partial N$ for all $\left|t\right|\le\tau$. With this choice, we conclude that $g_{K^t}:\partial K^t\to\mathbb{S}^{n-1}$ is a diffeomorphism. It follows that ${\mathcal{H}^{n-1}}\left({\omega _t}\right)=0$ and \[\int_{\mathbb{S}^{n-1}}h_{K^t}^qd\xi =\int_{\mathbb{S}^{n-1}}{(h_K^q+th_L^q)}d\xi.\] Next, we consider the $\mathbf{p}$-harmonic measure corresponding to $u(\cdot,t)\in W^{1,\mathbf{p}}(K^{t}\cap N)$, which is a weak solution to the Dirichlet problem \begin{equation}\label{3.5} \left\{ \begin{aligned} &\text{div}\left({{\left|{\nabla u\left(x,t\right)}\right|}^{\mathbf{p}-2} \nabla u\left(x,t\right)}\right)=0&&x\in K^t\cap N,\\ &u\left(x,t\right)=0&&x\in\partial K^t,\\ &u\left(x,t\right) = u\left(\frac{x}{\left(1+t\right)^{\frac{1}{q}}}\right)&&x\in\partial N\cap K^t, \end{aligned} \right. \end{equation} where $\left| t \right|$ is small enough so that upon zero extension, $u\left(x,t\right) \in {W^{1,\mathbf{p}}}\left( N \right)$. By defining \begin{equation}\label{3.6} \mathcal{F}\left[h_{K^t}\right]\left(\xi\right) :={\left|{\nabla u\left({\nabla{h_{K^t}} \left(\xi\right),t}\right)}\right|^{\mathbf{p}-1}}\det \left({{\bigtriangledown^2}{h_{{K^t}}}+h_{K^t}\mathbb I}\right), \end{equation} we obtain \begin{equation*} d{\mu_{{K^t}}} ={\left|{\nabla u\left({\nabla{h_{{K^t}}}\left(\xi\right),t} \right)}\right|^{\mathbf{p}-1}} d{\cal H}^{n-1}{\llcorner_{\partial K^t}} =\mathcal{F}\left[h_{K^t}\right]\left(\xi\right)d\xi, \end{equation*} and \begin{equation}\label{3.7} \Gamma\left(K^t\right) =\int_{\mathbb{S}^{n-1}}{{h_{K^t}}\left(\xi\right)} d{\mu_{K^t}}\left(\xi\right) =\int_{{\mathbb{S}^{n-1}}}h_{K^t} \left(\xi\right)\mathcal{F}\left[h_{K^t}\right] \left(\xi\right)d\xi. \end{equation} \begin{lemma}\label{lem:3.1} Let $1<\mathbf{p}<\infty$ and $q>0$, and let $\mathcal{F}$ be given by \eqref{3.6}. Then we have \begin{equation}\label{3.8} \mathcal{F}\left[ {\left( {1 + t} \right)^{\frac{1}{q}}{h_K}} \right]\left( \xi \right) = {\left( {1 + t} \right)^{\frac{n-\mathbf{p}}{q}}}\mathcal{F}\left[ {{h_K}} \right]\left( \xi \right), \end{equation} for all $\left| t \right|\le\tau$. Here $\tau$ is given in \eqref{3.4}. \end{lemma} \begin{proof} The proof is similar to that of Lemma 3.12 in \cite{AM24}. For completeness, we provide a proof as follows. We first deal with the case that $0<q<1$. By setting $L=K$ in \eqref{3.3}, we obtain that $K^{t}=\lambda K$ is the Wulff shape of the support function $\lambda h_{K}$, where $\lambda=\left(1+t\right)^{\frac{1}{q}}$. Let $u_{\lambda}\left(\cdot\right):=u\left(\cdot,\lambda^{q}-1\right)$ be the weak solution to the Dirichlet problem \begin{equation}\label{3.9} \left\{ \begin{aligned} &\text{div}\left( {{{\left| {\nabla u_{\lambda}\left( x \right)} \right|}^{\mathbf{p}-2}} \nabla u_{\lambda}\left( x \right)} \right) = 0&&x\in\lambda K\cap N,\\ &u_{\lambda}\left(x\right)=0&&x\in\partial(\lambda K),\\ &u_{\lambda}\left(x\right)= u\left(\frac{x}{\lambda}\right)&&x\in\partial N\cap\lambda K, \end{aligned} \right. \end{equation} for $\left| {{\lambda ^q} - 1} \right| \le \tau $. Then we have \begin{equation}\label{3.10} \begin{split} \mathcal{F}\left[ {{\lambda h_{K}}} \right]\left( \xi \right) &={\left| {\nabla u_{\lambda}\left( {\lambda \nabla {h_{K}}\left( \xi \right)} \right)} \right|^{\mathbf{p}-1}}\lambda^{n-1}\det \left( {{\bigtriangledown ^2}{h_{K}}+{h_{K}}\mathbb{I}} \right)\\ &= \left({\frac{\left|{\nabla {u_\lambda }\left( {\lambda \nabla {h_K}\left( \xi \right)} \right)}\right|}{\left| {\nabla u\left( {\nabla {h_K}\left( \xi \right)} \right)} \right|}}\right)^{\mathbf{p} - 1} {\lambda ^{n - 1}}\mathcal{F} \left[ {{ h_{K}}} \right]\left( \xi \right). \end{split} \end{equation} As $u$ is the solution to \eqref{1.1}, we have that $u\left(\frac{x}{\lambda}\right)$ is also the solution to \eqref{3.9} in $\lambda K$. By the uniqueness of the solution to \eqref{3.9}, $u_{\lambda}\left(x\right)=u\left(\frac{x}{\lambda}\right)$ in $\lambda K$. It follows that $\nabla {u_\lambda }\left( x \right) = \frac{1}{\lambda }\nabla u\left( {\frac{x}{\lambda }} \right)$, thus \eqref{3.10} gives \[\mathcal{F}\left[ {\lambda {h_K}} \right]\left( \xi \right) = {\lambda^{n-\mathbf{p}}}\mathcal{F}\left[ {{h_K}} \right]\left( \xi \right)\] for $\left| {{\lambda ^q} - 1} \right| \le \tau $. This proves the case $0<q<1$. Note that the $q$-sum $K^t$ for $q\ge1$ can also be given by \eqref{3.3}, and the argument for the case $q\ge1$ follows along the same lines. Therefore, the remaining case of the proof is omitted. \end{proof} We define $\dot u\left(x\right) ={{{\left. {\frac{\partial}{{\partial t}}}\right|}_{t= 0}}u\left( {x,t} \right)}$ and present a differentiability lemma as follows. \begin{lemma}\label{lem:3.2} Let $1<\mathbf{p}<\infty$ and $q>0$, and let $K, L\in \mathcal{A}_+^{2,\alpha}$ be two compact convex sets containing the origin. If $u\left(\cdot,t\right)\in W^{1,\mathbf{p}}\left(K^{t} \cap N\right)$ is the solution to \eqref{3.5}, the following holds: \begin{enumerate}[label=\upshape (\roman*)] \item The map $t\mapsto u\left({x,t} \right)$ is differentiable at $t=0$ for all $x\in\bar K\cap N$, and $\dot u\in C^{2,\beta}\left(\overline{K\cap N}\right)$ with $\beta=\beta(n,\mathbf{p},\alpha)$; \item For $x\in\partial K$ and $q\ge1$, $\dot u(x)=\left| {\nabla u\left(x\right)}\right| \left({\frac{1}{q}h_K^{1-q}\left(g_K\left(x\right)\right)h_L^q \left( {{g_K}\left(x\right)}\right)}\right)$. If $0<q<1$, this equality holds almost everywhere with respect to $S_K$. \end{enumerate} \end{lemma} \begin{proof} Part (i) comes from Proposition 3.20 of \cite{AM24}. Here, we provide a brief proof of (ii) for the case $0<q<1$; the case $q\ge1$ follows similarly. Define $\omega\left(x,t\right)=\frac{u\left(x,t\right)-u\left(x,0\right)}{t}$ for $t\neq0$. According to (3.23) in \cite{AM24}, there exists a sequence $\{t_k\}$ such that $t_k\to 0$ as $k\to\infty$, and the limit \begin{equation*} \lim\limits_{k\to\infty }\omega\left(x,{t_k}\right) =\lim\limits_{k\to\infty } \frac{u\left({x,{t_k}} \right)-u\left({x,0}\right)}{t_k} =:\omega\left(x\right) \end{equation*} exists for all $x\in K\cap N$. Moreover, for $x\in \partial K$, there exists a sequence $\left\{x_j\right\}\subset \text{int}K$ such that $x_j\to x$ as $j\to\infty$, and \begin{equation*} \omega\left(x\right) =\lim\limits_{j\to\infty }\omega\left(x_j\right) =\lim\limits_{j\to\infty }\lim \limits_{k\to\infty}\omega\left({x_j},{t_k}\right) =\lim\limits_{k\to\infty }\frac{u\left(x,t_k\right)-u\left(x,0\right)}{t_k}, \end{equation*} for any $x\in\partial K$. Hence, the function $t\to u\left({\cdot,t}\right)$ is differentiable at $t=0$ for all $x\in\bar K\cap N$. It follows from (3.26) and (3.27) of \cite{AM24} that $\dot u\in C^{2,\beta} \left(\overline{K\cap N}\right)$, and \[\left| {\omega \left( {{x_k},{t_k}} \right) -\omega\left( {{x_k},0}\right)}\right|\leqslant\Lambda\left|{x_k-x}\right|\] for $\Lambda>0$ and any $x_k\in\partial K^{t_k}$. Thus, \begin{equation*} \omega\left(x\right) =\lim\limits_{k\to\infty}\omega\left(x_k,t_k\right) =\lim\limits_{k\to\infty}\frac{u\left(x_k,t_k\right)-u\left(x_k,0\right)}{t_k} =\lim\limits_{k\to\infty}\frac{u\left(x\right)-u\left(x_k,0\right)}{t_k} \end{equation*} for any $x\in\partial K$. For $\xi\in\mathbb{S}^{n-1}$, there exists $x\in\partial K$ and $x_k\in\partial K^{t_k}$ so that $x=\nabla h_{K}\left(\xi\right)$, $x_{k}=\nabla h_{K^{t_k}}\left(\xi\right)$. Then, we compute: \begin{equation*} \begin{split} \nabla h_{K^{t_k}} &=\nabla {\left({h_K^q+t_kh_L^q}\right)^{\frac{1}{q}}}\\ &= {\left({h_K^q+t_kh_L^q}\right)^{\frac{{1-q}}{q}}}h_K^{q - 1}\nabla {h_K} +t_k{\left( {h_K^q + t_kh_L^q} \right)^{\frac{{1 - q}}{q}}}h_L^{q - 1}\nabla {h_L}\\ &={\left({1+t_kh_L^qh_K^{-q}} \right)^{\frac{{1 - q}}{q}}}\nabla {h_K} +t_k{\left( {{{\left( {h_L^qh_K^{-q}}\right)}^{-1}}+t_k} \right)^{\frac{{1 - q}}{q}}}\nabla {h_L}\\ &=\nabla {h_K}+\left( {{{\left( {1 +t_kh_L^qh_K^{ - q}} \right)}^{\frac{{1 - q}}{q}}} - 1} \right)\nabla {h_K} + t_kh_L^{q - 1}h_K^{1 - q}{\left( {1 + t_kh_L^qh_K^{ - q}} \right)^{\frac{{1 - q}}{q}}}\nabla {h_L}, \end{split} \end{equation*} $S_{K^{t_k}}$-almost everywhere. Taking the limit as $k\to \infty$, we obtain: \begin{equation*} \begin{split} \mathop {\lim }\limits_{k \to \infty} \frac{{{x_k} - x}}{t_k} &= \mathop {\lim }\limits_{k \to \infty} \frac{{\left( {{{\left( {1 + t_kh_L^qh_K^{ - q}} \right)}^{\frac{{1 - q}}{q}}} - 1} \right)\nabla {h_K} + t_kh_L^{q - 1}h_K^{1 - q}{{\left( {1 +t_kh_L^qh_K^{ - q}} \right)}^{\frac{{1 - q}}{q}}}\nabla {h_L}}}{t_k}\\ &= \frac{{1 - q}}{q}h_L^qh_K^{ - q}\nabla {h_K} + h_L^{q - 1}h_K^{1 - q}\nabla {h_L}\\ &= \nabla \left( {\frac{1}{q}h_K^{1 - q}h_L^q} \right), \end{split}. \end{equation*} $S_{K}$-almost everywhere. Thus, \begin{equation*} \begin{split} \omega \left( x \right) =\mathop {\lim }\limits_{k \to \infty } \frac{{u\left( x \right) - u\left( {{x_k},0} \right)}}{{{t_k}}} =-\left\langle {\nabla u\left( x \right),\nabla \left( {\frac{1}{q}h_K^{1 - q}h_L^q} \right)} \right\rangle, \end{split} \end{equation*} $S_{K}$-almost everywhere for all $x\in \partial K$. Notice that $\xi=-\frac{{\nabla u\left( x \right)}}{{\left| {\nabla u\left( x \right)} \right|}}$ and \[\frac{1}{q}h_K^{1 - q}\left( \xi \right)h_L^q\left( \xi \right) =\left\langle {\xi ,\nabla \left( {\frac{1}{q}h_K^{1 - q} \left(\xi\right)h_L^q\left(\xi\right)} \right)} \right\rangle,\] due to the Euler's homogeneous function theorem. We can conclude that \[\omega \left( x \right) = \left| {\nabla u\left( x \right)} \right|\left( {\frac{1}{q}h_K^{1 - q}\left( {{g_K}\left( x \right)} \right)h_L^q\left( {{g_K}\left( x \right)} \right)} \right).\] This completes the proof of the second assertion for the case $0<q<1$. \end{proof} In the following, we prove two lemmas which are critical for establishing the variational formula of $\Gamma\left(K \right)$ with respect to the $q$-sum. The first one can be stated as follows. \begin{lemma}\label{lem:3.3} Let $1<\mathbf{p}<\infty$, and let $K, L\in \mathcal{A}_+^{2,\alpha}$ be two compact convex sets containing the origin. Then, for the Wulff shape $K^t$ with $\left| t \right|\le\tau$ (where $\tau$ is given in \eqref{3.4}), if $0<q<1$, we have \begin{equation*} \begin{split} {\left.{\frac{d}{{dt}}}\right|_{t = 0}}\mathcal{F}\left[ {{h_{{K^t}}}} \right]\left(\xi\right) =&\sum\limits_{i,j=1}^{n-1} {{\bigtriangledown _j} \left({{C_{i,j}}\left[ {{\bigtriangledown ^2}{h_K}+{h_K}{\mathbb I}}\right] {{\left|{\nabla u\left({\nabla {h_{{K}}}\left(\xi\right)}\right)}\right|}^{\mathbf{p}-1}} {\bigtriangledown _i}\left( {\frac{1}{q}h_K^{1-q}h_L^q} \right)}\right)}\\ &-\left( {\mathbf{p}-1}\right) {\left| {\nabla u\left( {\nabla {h_{{K}}}\left(\xi\right)}\right)}\right|^{\mathbf{p}-2}} \det\left( {{\bigtriangledown ^2}{h_K} + {h_K}{\mathbb I}}\right) \left\langle{\nabla\dot u\left({\nabla{h_K}\left(\xi\right)}\right),\xi}\right\rangle \end{split} \end{equation*} $S_K$-almost everywhere on $\mathbb{S}^{n-1}$. If $q\ge1$, this equality always holds on $\mathbb{S}^{n-1}$. \end{lemma} \begin{proof} Since the proof for the case $q\ge1$ is similar to that for the case $0<q<1$, we will focus only on the latter. According to \eqref{3.6}, we have the following calculation \begin{equation}\label{3.11} \begin{split} &{\left.{\frac{d}{{dt}}}\right|_{t = 0}}\mathcal{F}\left[ {{h_{{K^t}}}}\right]\left(\xi\right)\\ =&{\left. {\frac{d}{{dt}}} \right|_{t = 0}}\left( {{{\left| {\nabla u\left( {\nabla {h_{{K^t}}}\left(\xi\right),t} \right)} \right|}^{\mathbf{p}-1}}\det \left( {{\bigtriangledown ^2}{h_{K^t}} + {h_{K^t}}{\mathbb I}} \right)} \right)\\ =&\left( {\mathbf{p}-1} \right){\left| {\nabla u\left( {\nabla {h_K}\left( \xi \right)} \right)} \right|^{\mathbf{p}-2}}{\left. {\det \left( {{\bigtriangledown ^2}{h_K} + {h_K}{\mathbb I}} \right)\frac{d}{{dt}}} \right|_{t = 0}}\left| {\nabla u\left( {\nabla {h_{{K^t}}}\left( \xi \right),t} \right)} \right| \\ &+{\left| {\nabla u\left( {\nabla {h_K}\left( \xi \right)} \right)} \right|^{\mathbf{p}-1}}{\left. {\frac{d}{{dt}}} \right|_{t = 0}}\det \left( {{\bigtriangledown ^2}{h_{{K^t}}} + {h_{{K^t}}}{\mathbb I}} \right). \end{split} \end{equation} Notice that \[\int_{{\mathbb{S}^{n-1}}} {\left( {{\bigtriangledown ^2}{h_{{K^t}}} + {h_{{K^t}}}{\mathbb{I}}} \right)} d{S_{{K^t}}} = \int_{{\mathbb{S}^{n-1}}} \left({\bigtriangledown ^2}{{\left( {h_K^q + th_L^q} \right)}^{\frac{1}{q}}} + {{\left( {h_K^q + th_L^q} \right)}^{\frac{1}{q}}}{\mathbb{I}}\right) d{S_{{K^t}}},\] we differentiate both sides with respect to $t$ at $t=0$ and obtain \begin{equation*} \begin{split} &\int_{{\mathbb{S}^{n-1}}}{{{\left. {\frac{d}{{dt}}} \right|}_{t = 0}}\left( {{\bigtriangledown ^2}{h_{{K^t}}} + {h_{{K^t}}}{\mathbb{I}}} \right)} d{S_K} +\int_{{\mathbb{S}^{n-1}}} \left({{\bigtriangledown ^2}{h_{{K}}} + {h_{{K}}}{\mathbb{I}}}\right) {\left. {\frac{d}{{dt}}} \right|_{t = 0}}d{S_{{K^t}}}\\ =&\int_{{\mathbb{S}^{n-1}}} {{{\left. {\frac{d}{{dt}}} \right|}_{t = 0}} \left( {{\bigtriangledown ^2}{{\left( {h_K^q + th_L^q} \right)}^{\frac{1}{q}}} + {{\left( {h_K^q + th_L^q} \right)}^{\frac{1}{q}}}{\mathbb{I}}} \right)} d{S_K} +\int_{{\mathbb{S}^{n-1}}} \left({{\bigtriangledown ^2}{{ {h_K}}} + { {h_K} }{\mathbb{I}}}\right) {\left. {\frac{d}{{dt}}} \right|_{t = 0}}d{S_{{K^t}}}. \end{split} \end{equation*} This implies that \[\int_{{\mathbb{S}^{n-1}}} {{{\left. {\frac{d}{{dt}}} \right|}_{t = 0}}\left( {{\bigtriangledown ^2}{h_{{K^t}}} + {h_{{K^t}}}{\mathbb{I}}} \right)} d{S_K} = \int_{{\mathbb{S}^{n-1}}} {{{\left. {\frac{d}{{dt}}} \right|}_{t = 0}}\left( {{\bigtriangledown ^2}{{\left( {h_K^q + th_L^q} \right)}^{\frac{1}{q}}} + {{\left( {h_K^q + th_L^q} \right)}^{\frac{1}{q}}}{\mathbb{I}}} \right)} d{S_K}.\] Therefore, \[{{{\left. {\frac{d}{{dt}}} \right|}_{t = 0}}\left( {{\bigtriangledown ^2}{h_{{K^t}}} + {h_{{K^t}}}{\mathbb{I}}} \right) = {{\left. {\frac{d}{{dt}}} \right|}_{t = 0}}\left( {{\bigtriangledown ^2}{{\left( {h_K^q + th_L^q} \right)}^{\frac{1}{q}}} + {{\left( {h_K^q + th_L^q} \right)}^{\frac{1}{q}}}{\mathbb{I}}} \right)}\] $S_K$-almost everywhere. Hence, \begin{equation}\label{3.12} \begin{split} &{\left. {\frac{d}{{dt}}} \right|_{t = 0}}\det \left( {{\bigtriangledown ^2}{h_{{K^t}}} + {h_{{K^t}}}{\mathbb I}} \right)\\ =&{\rm{Tr}}\left( {C\left[ {{\bigtriangledown ^2}{h_K} + {h_K}{\mathbb I}} \right]{{\left. {\frac{d}{{dt}}} \right|}_{t = 0}}\left( {{\bigtriangledown ^2}{h_{{K^t}}} + {h_{{K^t}}}{\mathbb I}} \right)} \right)\\ =&{\rm{Tr}}\left( {C\left[ {{\bigtriangledown ^2}{h_K} + {h_K}{\mathbb I}} \right]{{\left. {\frac{d}{{dt}}} \right|}_{t = 0}}\left( {{\bigtriangledown ^2}{{\left( {h_K^q + th_L^q} \right)}^{\frac{1}{q}}} + {{\left( {h_K^q + th_L^q} \right)}^{\frac{1}{q}}}{\mathbb I}} \right)} \right)\\ =&{\rm{Tr}}\left( {C\left[ {{\bigtriangledown ^2}{h_K} + {h_K}{\mathbb I}} \right]\left( {{\bigtriangledown ^2}\left( {\frac{1}{q}h_K^{1 - q}h_L^q} \right) + \left( {\frac{1}{q}h_K^{1 - q}h_L^q} \right){\mathbb I}} \right)} \right). \end{split} \end{equation} $S_K$-almost everywhere. As the unit outer normal $\xi$ of $K^t$ satisfies the identity \begin{equation*} \xi = - \frac{{\nabla u\left( {\nabla {h_{{K^t}}}\left( \xi \right),t} \right)}}{{\left| {\nabla u\left( {\nabla {h_{{K^t}}}\left( \xi \right),t} \right)} \right|}}, \end{equation*} then $\left| {\nabla u\left( {\nabla {h_{{K^t}}}\left( \xi \right),t} \right)} \right| = - \left\langle {\nabla u\left( {\nabla {h_{{K^t}}}\left( \xi \right),t} \right),\xi } \right\rangle$, and we have the following calculation \begin{equation*} \begin{split} &{\left. {\frac{d}{{dt}}} \right|_{t = 0}}\left| {\nabla u\left( {\nabla {h_{{K^t}}}\left( \xi \right),t} \right)} \right|\\ =&-{\left. {\frac{d}{{dt}}} \right|_{t = 0}}\left\langle {\nabla u\left( {\nabla {h_{{K^t}}}\left( \xi \right),t} \right),\xi } \right\rangle \\ =&-\left( {\left\langle {{D^2}u\left( {\nabla {h_K}\left( \xi \right)} \right){{\left. {\frac{d}{{dt}}} \right|}_{t = 0}}\nabla {h_{{K^t}}}\left( \xi \right),\xi } \right\rangle + \left\langle {\nabla \dot u\left( {\nabla {h_K}\left( \xi \right)} \right),\xi } \right\rangle } \right)\\ =&-\left( {\left\langle {{D^2}u\left( {\nabla {h_K}\left( \xi \right)} \right)\nabla \left( {{{\left. {\frac{d}{{dt}}} \right|}_{t = 0}}{{\left( {h_K^q + th_L^q} \right)}^{\frac{1}{q}}}} \right),\xi } \right\rangle + \left\langle {\nabla \dot u\left( {\nabla {h_K}\left( \xi \right)} \right),\xi } \right\rangle } \right)\\ =&-\left( {\left\langle {{D^2}u\left( {\nabla {h_K}\left( \xi \right)} \right)\nabla \left( {\frac{1}{q}h_K^{1 - q}h_L^q} \right),\xi } \right\rangle + \left\langle {\nabla \dot u\left( {\nabla {h_K}\left( \xi \right)} \right),\xi } \right\rangle } \right)\\ =&-\left( {{J_1} + {J_2}} \right), \end{split} \end{equation*} $S_K$-almost everywhere. Since $$ \nabla {h_K}\left( \xi \right) = {h_K}\left( \xi \right)\xi + \sum\limits_{i = 1}^{n - 1} {{\bigtriangledown _i}{h_K}\left( \xi \right){e^i}} $$ and $$ \nabla {h_L}\left( \xi \right) = {h_L}\left( \xi \right)\xi + \sum\limits_{i = 1}^{n - 1} {{\bigtriangledown _i}{h_L}\left( \xi \right){e^i}}, $$ we have \begin{equation}\label{3.13} \begin{split} \nabla \left( {\frac{1}{q}h_K^{1-q}(\xi)h_L^q(\xi)} \right) =\left( {\frac{1}{q}h_K^{1-q}(\xi)h_L^q(\xi)} \right)\xi +\sum\limits_{i=1}^{n-1} {{\bigtriangledown_i} \left({\frac{1}{q}h_K^{1-q}(\xi)h_L^q(\xi)}\right){e^i}}. \end{split} \end{equation} This, together with Lemma \ref{lem:2.2}, yields that \begin{equation*} \begin{split} {J_1}=&\left\langle {{D^2}u\left( {\nabla {h_K}\left( \xi \right)} \right)\nabla \left( {\frac{1}{q}h_K^{1-q}h_L^q} \right),\xi } \right\rangle\\ =&\left\langle {{D^2}u\left( {\nabla {h_K}\left( \xi \right)} \right)\xi ,\xi } \right\rangle \left( {\frac{1}{q}h_K^{1 - q}h_L^q} \right) +\sum\limits_{i = 1}^{n - 1} {\left\langle {{D^2}u\left( {\nabla {h_K}\left(\xi\right)} \right){e^i},\xi} \right\rangle {\bigtriangledown _i} \left( {\frac{1}{q}h_K^{1 - q}h_L^q} \right)} \\ =&\frac{1}{{\mathbf{p}-1}}\kappa \left( {\nabla {h_K}\left( \xi \right)} \right) \left|{\nabla u\left( {\nabla {h_K}\left( \xi \right)} \right)} \right| {\rm{Tr}}\left( {C\left[ {{\bigtriangledown ^2}{h_K}+{h_K}{\mathbb I}} \right]} \right) \left( {\frac{1}{q}h_K^{1 - q}h_L^q} \right)\\ &-\sum\limits_{i = 1}^{n - 1} {\kappa \left( {\nabla {h_K}\left(\xi\right)} \right)\sum\limits_{j = 1}^{n - 1} {{C_{i,j}}\left[ {{\bigtriangledown ^2}{h_K} + {h_K}{\mathbb I}} \right]} {\bigtriangledown _j} \left( {\left| {\nabla u\left( {\nabla {h_K}\left( \xi \right)} \right)} \right|} \right){\bigtriangledown _i}\left( {\frac{1}{q}h_K^{1 - q}h_L^q} \right)} \\ =&\frac{1}{{\mathbf{p}-1}}\kappa \left( {\nabla {h_K}\left( \xi \right)} \right) \left| {\nabla u\left( {\nabla {h_K}\left( \xi \right)} \right)} \right| {\rm{Tr}}\left( {C\left[ {{\bigtriangledown ^2}{h_K}+{h_K}{\mathbb I}} \right]} \right) \left( {\frac{1}{q}h_K^{1 - q}h_L^q} \right)\\ &-\kappa \left( {\nabla {h_K}\left(\xi\right)} \right)\sum\limits_{i,j = 1}^{n - 1} {{C_{i,j}}\left[ {{\bigtriangledown ^2}{h_K} + {h_K}{\mathbb I}} \right]} {\bigtriangledown _j} \left( {\left| {\nabla u\left( {\nabla {h_K} \left( \xi \right)} \right)} \right|} \right){\bigtriangledown _i} \left( {\frac{1}{q}h_K^{1 - q}h_L^q} \right). \end{split} \end{equation*} Then, using $\sum\limits_{j=1}^{n-1} {{\bigtriangledown _j}{C_{i,j}}\left[ {{\bigtriangledown ^2}{h_K} + {h_K}{\mathbb I}} \right]}=0$ (cf. (4.3) of \cite{CY76}), we have \begin{equation*} \begin{split} {J_1} =& \frac{1}{{\mathbf{p}-1}}\kappa \left( {\nabla {h_K}\left( \xi \right)} \right)\left| {\nabla u\left( {\nabla {h_K}\left( \xi \right)} \right)} \right|{\rm{Tr}}\left( {C\left[ {{\bigtriangledown ^2}{h_K} + {h_K}{\mathbb I}} \right]} \right)\left( {\frac{1}{q}h_K^{1 - q}h_L^q} \right)\\ &- \kappa \left( {\nabla {h_K}\left( \xi \right)} \right)\sum\limits_{i,j = 1}^{n - 1} {{\bigtriangledown _j}\left( {{C_{i,j}}\left[ {{\bigtriangledown ^2}{h_K} + {h_K}{\mathbb I}} \right]{\left| {\nabla u\left( {\nabla {h_K}\left( \xi \right)} \right)} \right|}} \right){\bigtriangledown _i}\left( {\frac{1}{q}h_K^{1 - q}h_L^q} \right)}. \end{split} \end{equation*} Hence, \begin{equation}\label{3.14} \begin{split} &{\left. {\frac{d}{{dt}}} \right|_{t = 0}}\left| {\nabla u\left( {\nabla {h_{{K^t}}}\left( \xi \right),t} \right)} \right| \\ =&\kappa \left( {\nabla {h_K}\left( \xi \right)} \right)\sum\limits_{i,j = 1}^{n - 1} {{\bigtriangledown _j}\left( {{C_{i,j}}\left[ {{\bigtriangledown ^2}{h_K} + {h_K}{\mathbb I}} \right]\left( {\left| {\nabla u\left( {\nabla {h_K}\left( \xi \right)} \right)} \right|} \right)} \right){\bigtriangledown _i}\left( {\frac{1}{q}h_K^{1 - q}h_L^q} \right)} \\ &-\frac{1}{{\mathbf{p}-1}}\kappa \left( {\nabla {h_K}\left( \xi \right)} \right)\left| {\nabla u\left( {\nabla {h_K}\left( \xi \right)} \right)} \right|{\rm{Tr}}\left( {C\left[ {{\bigtriangledown ^2}{h_K} + {h_K}{\mathbb I}} \right]} \right)\left( {\frac{1}{q}h_K^{1 - q}h_L^q} \right) \\ &-\left\langle {\nabla \dot u\left( {\nabla {h_K}\left( \xi \right)} \right),\xi } \right\rangle, \end{split} \end{equation} $S_K$-almost everywhere. Applying \eqref{2.7} and substituting both \eqref{3.14} and \eqref{3.12} into \eqref{3.11}, we obtain that \begin{equation*} \begin{split} &{\left. {\frac{d}{{dt}}} \right|_{t = 0}}\mathcal{F}\left[ {{h_{{K^t}}}} \right]\left( \xi\right)\\ =&\left( {\mathbf{p}-1} \right){\left| {\nabla u\left( {\nabla {h_K}\left( \xi \right)} \right)} \right|^{\mathbf{p}-2}}\sum\limits_{i,j = 1}^{n - 1} {{\bigtriangledown _j}\left( {{C_{i,j}}\left[ {{\bigtriangledown ^2}{h_K} + {h_K}{\mathbb I}} \right]\left( {\left| {\nabla u\left( {\nabla {h_K}\left( \xi \right)} \right)} \right|} \right)} \right){\bigtriangledown _i}\left( {\frac{1}{q}h_K^{1 - q}h_L^q} \right)}\\ &-{\left| {\nabla u\left( {\nabla {h_K}\left( \xi \right)} \right)} \right|^{\mathbf{p}-1}}{\rm{Tr}}\left( {C\left[ {{\bigtriangledown ^2}{h_K} + {h_K}{\mathbb I}} \right]} \right)\left( {\frac{1}{q}h_K^{1 - q}h_L^q} \right) \\ &-\left( {\mathbf{p}-1} \right)\frac{{{{\left| {\nabla u\left( {\nabla {h_K}\left( \xi \right)} \right)} \right|}^{\mathbf{p}-2}}}}{{\kappa \left( {\nabla {h_K}\left( \xi \right)} \right)}}\left\langle {\nabla \dot u\left( {\nabla {h_K}\left( \xi \right)} \right),\xi } \right\rangle \\ &+{\left| {\nabla u\left( {\nabla {h_K}\left( \xi \right)} \right)} \right|^{\mathbf{p}-1}}{\rm{Tr}}\left( {C\left[ {{\bigtriangledown ^2}{h_K} + {h_K}{\mathbb I}} \right]\left( {{\bigtriangledown ^2}\left( {\frac{1}{q}h_K^{1 - q}h_L^q} \right) + \left( {\frac{1}{q}h_K^{1 - q}h_L^q} \right){\mathbb I}} \right)} \right)\\ =&\left( {\mathbf{p}-1} \right){\left| {\nabla u\left( {\nabla {h_K}\left( \xi \right)} \right)} \right|^{\mathbf{p}-2}}\sum\limits_{i,j = 1}^{n - 1} {{\bigtriangledown _j}\left( {{C_{i,j}}\left[ {{\bigtriangledown ^2}{h_K} + {h_K}{\mathbb I}} \right]\left( {\left| {\nabla u\left( {\nabla {h_K}\left( \xi \right)} \right)} \right|} \right)} \right){\bigtriangledown _i}\left( {\frac{1}{q}h_K^{1 - q}h_L^q} \right)} \\ &-\left( {\mathbf{p}-1} \right)\frac{{{{\left| {\nabla u\left( {\nabla {h_K}\left( \xi \right)} \right)} \right|}^{\mathbf{p}-2}}}}{{\kappa \left( {\nabla {h_K}\left( \xi \right)} \right)}}\left\langle {\nabla \dot u\left( {\nabla {h_K}\left( \xi \right)} \right),\xi } \right\rangle \\ &+ {\left| {\nabla u\left( {\nabla {h_K}\left( \xi \right)} \right)} \right|^{\mathbf{p}-1}}{\rm{Tr}}\left( {C\left[ {{\bigtriangledown ^2}{h_K} + {h_K}{\mathbb I}} \right]\left( {{\bigtriangledown ^2}\left( {\frac{1}{q}h_K^{1 - q}h_L^q} \right)} \right)} \right), \end{split} \end{equation*} $S_K$-almost everywhere. Since \begin{equation*} \begin{split} &\sum\limits_{i,j = 1}^{n - 1}{{\bigtriangledown _j}\left( {{C_{i,j}}\left[ {{\bigtriangledown ^2}{h_K} + {h_K}{\mathbb I}} \right]{{\left| {\nabla u\left( {\nabla {h_K}\left( \xi \right)} \right)} \right|}^{\mathbf{p}-1}}{\bigtriangledown _i}\left( {\frac{1}{q}h_K^{1 - q}h_L^q} \right)} \right)}\\ =&\sum\limits_{i,j = 1}^{n - 1} {{\bigtriangledown _j}\left( {{C_{i,j}}\left[ {{\bigtriangledown ^2}{h_K} + {h_K}{\mathbb I}} \right]{{\left| {\nabla u\left( {\nabla {h_K}\left( \xi \right)} \right)} \right|}^{\mathbf{p}-1}}} \right)} {\bigtriangledown _i}\left( {\frac{1}{q}h_K^{1 - q}h_L^q} \right)\\ &+\sum\limits_{i,j = 1}^{n - 1} {{C_{i,j}}\left[ {{\bigtriangledown ^2}{h_K} + {h_K}{\mathbb I}} \right]{{\left| {\nabla u\left( {\nabla {h_K}\left( \xi \right)} \right)} \right|}^{\mathbf{p}-1}}} {\bigtriangledown _{j,i}}\left( {\frac{1}{q}h_K^{1 - q}h_L^q} \right)\\ =&\left( {\mathbf{p}-1} \right){\left| {\nabla u\left( {\nabla {h_K}\left( \xi \right)} \right)} \right|^{\mathbf{p}-2}}\sum\limits_{i,j = 1}^{n - 1} {{\bigtriangledown _j}\left( {{C_{i,j}}\left[ {{\bigtriangledown ^2}{h_K} + {h_K}{\mathbb I}} \right]\left( {\left| {\nabla u\left( {\nabla {h_K}\left( \xi \right)} \right)} \right|} \right)} \right){\bigtriangledown _i}\left( {\frac{1}{q}h_K^{1 - q}h_L^q} \right)}\\ &+ {\left| {\nabla u\left( {\nabla {h_K}\left( \xi \right)} \right)} \right|^{\mathbf{p}-1}}{\rm{Tr}}\left( {C\left[ {{\bigtriangledown ^2}{h_K} + {h_K}{\mathbb I}} \right]{\bigtriangledown ^2}\left( {\frac{1}{q}h_K^{1 - q}h_L^q} \right)} \right). \end{split} \end{equation*} Hence, \begin{equation*} \begin{split} {\left. {\frac{d}{{dt}}} \right|_{t = 0}}\mathcal{F}\left[ {{h_{{K^t}}}} \right]\left( \xi \right) =& \sum\limits_{i,j = 1}^{n - 1} {{\bigtriangledown _j}\left( {{C_{i,j}}\left[ {{\bigtriangledown ^2}{h_K} + {h_K}{\mathbb I}} \right]{{\left| {\nabla u\left( {\nabla {h_K}\left( \xi \right)} \right)} \right|}^{\mathbf{p}-1}}{\bigtriangledown _i}\left( {\frac{1}{q}h_K^{1 - q}h_L^q} \right)} \right)} \\ &-\left( {\mathbf{p}-1} \right)\frac{{{{\left| {\nabla u\left( {\nabla {h_K}\left( \xi \right)} \right)} \right|}^{\mathbf{p}-2}}}}{{\kappa \left( {\nabla {h_K}\left( \xi \right)} \right)}}\left\langle {\nabla \dot u\left( {\nabla {h_K}\left( \xi \right)} \right),\xi } \right\rangle, \end{split} \end{equation*} $S_K$-almost everywhere. \end{proof} Lemmas \ref{lem:3.2} and \ref{lem:3.3} can be employed to prove the following result. \begin{lemma}\label{lem:3.4} Let $1<\mathbf{p}<\infty$ and $q>0$, and let $K, L\in \mathcal{A}_+^{2,\alpha}$ be two compact convex sets containing the origin. Then, for the Wulff shape $K^t$ with $\left| t \right|\le\tau$ (where $\tau$ is given in \eqref{3.4}), we have \begin{equation}\label{3.15} \int_{{\mathbb{S}^{n-1}}} {{h_K}{{\left. {\frac{d}{{dt}}} \right|}_{t = 0}}\mathcal{F}\left[ {{h_{{K^t}}}} \right]\left( \xi \right)} d\xi = \int_{{\mathbb{S}^{n-1}}} {h_K^{1 - q}h_L^q{{\left. {\frac{d}{{dt}}} \right|}_{t = 0}}\mathcal{F}\left[ {{{\left( {1 + t} \right)}^{\frac{1}{q}}}{h_K}} \right]\left( \xi \right)} d\xi. \end{equation} \end{lemma} \begin{proof} Since $K\in\mathcal{A}_+^{2,\alpha}$, by Lemma \ref{lem:3.3}, we have \begin{equation}\label{3.16} \begin{split} &\int_{{\mathbb{S}^{n-1}}}{{h_K}{{\left. {\frac{d}{{dt}}} \right|}_{t = 0}}\mathcal{F}\left[ {{h_{{K^t}}}} \right]\left( \xi \right)} d\xi \\ =&\int_{{\mathbb{S}^{n-1}}} {{h_K}\sum\limits_{i,j = 1}^{n - 1} {{\bigtriangledown _j}\left( {{C_{i,j}}\left[ {{\bigtriangledown ^2}{h_K} + {h_K}{\mathbb I}} \right]{{\left| {\nabla u\left( {\nabla {h_K}\left( \xi \right)} \right)} \right|}^{\mathbf{p}-1}}{\bigtriangledown _i}\left( {\frac{1}{q}h_K^{1 - q}h_L^q} \right)} \right)} } d\xi \\ &-\int_{{\mathbb{S}^{n-1}}} {{h_K}\left( {\mathbf{p}-1} \right)\frac{{{{\left| {\nabla u\left( {\nabla {h_K}\left( \xi \right)} \right)} \right|}^{\mathbf{p}-2}}}}{{\kappa \left( {\nabla {h_K}\left( \xi \right)} \right)}}\left\langle {\nabla \dot u\left( {\nabla {h_K}\left( \xi \right)} \right),\xi } \right\rangle } d\xi \\ =&I_1-I_2. \end{split} \end{equation} Then, by repeatedly applying Stokes's theorem for a compact manifold without boundary, we can calculate the term $I_1$ as follows. \begin{equation}\label{3.17} \begin{split} I_1 & = \int_{{\mathbb{S}^{n-1}}} {\sum\limits_{i,j = 1}^{n - 1} {{h_K}{\bigtriangledown _j}\left( {{C_{i,j}}\left[ {{\bigtriangledown ^2}{h_K} + {h_K}{\mathbb{I}}} \right]{{\left| {\nabla u\left( {\nabla {h_K}\left( \xi \right)} \right)} \right|}^{\mathbf{p}-1}}{\bigtriangledown _i}\left( {\frac{1}{q}h_K^{1 - q}h_L^q} \right)} \right)} } d\xi \\ &=-\int_{{\mathbb{S}^{n-1}}} {\sum\limits_{i,j = 1}^{n - 1} {{C_{i,j}}\left[ {{\bigtriangledown^2}{h_K} + {h_K}{\mathbb{I}}} \right]{{\left| {\nabla u\left( {\nabla {h_K}\left( \xi \right)} \right)} \right|}^{\mathbf{p}-1}}{\bigtriangledown _i}\left( {\frac{1}{q}h_K^{1 - q}h_L^q} \right){\bigtriangledown _j}} } {h_K}d\xi\\ &=\int_{{\mathbb{S}^{n-1}}} {\sum\limits_{i,j = 1}^{n - 1} {h_K^{1-q}h_L^q{\bigtriangledown _j}\left( {{C_{i,j}}\left[ {{\bigtriangledown ^2}{h_K} + {h_K}{\mathbb{I}}} \right]{{\left| {\nabla u\left( {\nabla {h_K}\left( \xi \right)} \right)} \right|}^{\mathbf{p}-1}}{\bigtriangledown _i}\left( {\frac{1}{q}{h_K}} \right)} \right)} } d\xi. \end{split} \end{equation} By using (ii) of Lemma \ref{lem:3.2}, along with the formulas \eqref{3.13} and \eqref{2.2}, we can calculate \begin{equation*} \begin{split} \frac{1}{\mathbf{p}-1}{I_2} =&\int_{{\mathbb{S}^{n-1}}} {{h_K}\frac{{{{\left| {\nabla u\left( {\nabla {h_K}\left( \xi \right)} \right)} \right|}^{\mathbf{p}-2}}}}{{\kappa \left( {\nabla {h_K}\left( \xi \right)} \right)}}\left\langle {\nabla \dot u\left( {\nabla {h_K}\left( \xi \right)} \right),\xi } \right\rangle } d\xi \\ =&\int_{\partial K} {{{\left| {\nabla u} \right|}^{\mathbf{p}-2}}{h_K} \circ {g_K}\left\langle {\nabla \left( {\left| {\nabla u} \right|\left( {\frac{1}{q}{{\left( {{h_K} \circ {g_K}} \right)}^{1 - q}}{{\left( {{h_L} \circ {g_K}} \right)}^q}} \right)} \right),{g_K}} \right\rangle } d{\mathcal{H}^{n - 1}}\\ =&\int_{\partial K} {{{\left| {\nabla u} \right|}^{\mathbf{p}-2}}{h_K} \circ {g_K}\left\langle {\nabla \left( {\left| {\nabla u} \right|} \right)\left( {\frac{1}{q}{{\left( {{h_K} \circ {g_K}} \right)}^{1 - q}}{{\left( {{h_L} \circ {g_K}} \right)}^q}} \right),{g_K}} \right\rangle } d{\mathcal{H}^{n - 1}}\\ &+\int_{\partial K} {{{\left| {\nabla u} \right|}^{\mathbf{p}-2}}{h_K} \circ {g_K}\left| {\nabla u} \right|\frac{1}{q}{{\left( {{h_K} \circ {g_K}} \right)}^{1 - q}}{{\left( {{h_L} \circ {g_K}} \right)}^q}} d{\mathcal{H}^{n - 1}}\\ =&\int_{\partial K} {{{\left| {\nabla u} \right|}^{\mathbf{p}-2}}{{\left( {{h_K} \circ {g_K}} \right)}^{1 - q}}{{\left( {{h_L} \circ {g_K}} \right)}^q}\left\langle {\nabla \left( {\left| {\nabla u} \right|} \right)\frac{1}{q}{h_K} \circ {g_K},{g_K}} \right\rangle } d{\mathcal{H}^{n - 1}}\\ &+\int_{\partial K} {{{\left| {\nabla u} \right|}^{\mathbf{p}-2}}{{\left( {{h_K} \circ {g_K}} \right)}^{1 - q}}{{\left( {{h_L} \circ {g_K}} \right)}^q}\left\langle {\left| {\nabla u} \right|\nabla \left( {\frac{1}{q}{h_K} \circ {g_K}} \right),{g_K}} \right\rangle } d{\mathcal{H}^{n - 1}}\\ =&\int_{{\mathbb{S}^{n-1}}} {h_K^{1 - q}h_L^q\frac{{{{\left| {\nabla u\left( {\nabla {h_K}\left( \xi \right)} \right)} \right|}^{\mathbf{p}-2}}}}{{\kappa \left( {\nabla {h_K}\left( \xi \right)} \right)}}\left\langle {\nabla \left( {\left| {\nabla u\left( {\nabla {h_K}\left( \xi \right)} \right)} \right|\left( {\frac{1}{q}{h_K}} \right)} \right),\xi } \right\rangle } d\xi. \end{split} \end{equation*} This, together with \eqref{3.17} and \eqref{3.16}, yields that \begin{equation}\label{3.18} \begin{split} &\int_{{\mathbb{S}^{n-1}}}{{h_K}{{\left. {\frac{d}{{dt}}} \right|}_{t = 0}}\mathcal{F}\left[ {{h_{{K^t}}}} \right]\left( \xi \right)} d\xi\\ =& \int_{{\mathbb{S}^{n-1}}} {h_K^{1 - q}h_L^q\sum\limits_{i,j = 1}^{n - 1} {{\bigtriangledown _j}\left( {{C_{i,j}}\left[ {{\bigtriangledown ^2}{h_K} + {h_K}{\mathbb{I}}} \right]{{\left| {\nabla u\left( {\nabla {h_K}\left( \xi \right)} \right)} \right|}^{\mathbf{p}-1}}{\bigtriangledown _i}\left( {\frac{1}{q}{h_K}} \right)} \right)} d\xi } \\ &- \left( {\mathbf{p}-1} \right)\int_{{\mathbb{S}^{n-1}}} {h_K^{1 - q}h_L^q\frac{{{{\left| {\nabla u\left( {\nabla {h_K}\left( \xi \right)} \right)} \right|}^{\mathbf{p}-2}}}}{{\kappa \left( {\nabla {h_K}\left( \xi \right)} \right)}}\left\langle {\nabla \left( {\left| {\nabla u\left( {\nabla {h_K}\left( \xi \right)} \right)} \right|\left( {\frac{1}{q}{h_K}} \right)} \right),\xi } \right\rangle } d\xi. \end{split} \end{equation} On the other hand, by Lemma \ref{lem:3.2} and Lemma \ref{lem:3.3} with $L=K$, we have \begin{equation*} \begin{split} &{\left. {\frac{d}{{dt}}} \right|_{t = 0}}{\cal F}\left[ {{{\left( {1 + t} \right)}^{\frac{1}{q}}}{h_K}} \right]\left( \xi \right)\\ =& \sum\limits_{i,j = 1}^{n - 1} {{\bigtriangledown _j}\left( {{C_{i,j}}\left[ {{\bigtriangledown ^2}{h_K} + {h_K}{\mathbb{I}}} \right]{{\left| {\nabla u\left( {\nabla {h_K}\left( \xi \right)} \right)} \right|}^{\mathbf{p}-1}}{\bigtriangledown _i}\left( {\frac{1}{q}h_K} \right)} \right)} \\ &-\left( {\mathbf{p}-1} \right)\frac{{{{\left| {\nabla u\left( {\nabla {h_K}\left( \xi \right)} \right)} \right|}^{\mathbf{p}-2}}}}{{\kappa \left( {\nabla {h_K}\left( \xi \right)} \right)}}\left\langle {\nabla \left( {\left| {\nabla u\left( {\nabla {h_K}\left( \xi \right)} \right)} \right|\left( {\frac{1}{q}{h_K}} \right)} \right),\xi } \right\rangle, \end{split} \end{equation*} for $q\ge1$. Note that the above equality holds almost everywhere with respect to $S_K$ if $0<q<1$, then by substituting it into \eqref{3.18}, we can obtain \eqref{3.15}. \end{proof} Now, the main result of this section can be stated as follows. \begin{theorem}\label{th:3.1} Let $1<\mathbf{p}<\infty$, $q>0$, $K\in\mathcal K_o^n$ and $L\subset \mathbb{R}^{n}$ be a compact convex set containing the origin. Then, for the Wulff shape $K^t$ with $\left| t \right|\le\tau$ (where $\tau$ is given in \eqref{3.4}), we have \begin{equation}\label{3.19} \begin{split} {\left. {\frac{d}{{dt}}} \right|_{t = 0}}\Gamma \left( {{K^t}} \right) = \frac{{n-\mathbf{p} + 1}}{q}\int_{{\mathbb{S}^{n-1}}} {h_L^q\left( \xi \right)h_K^{1 - q}\left( \xi \right)} d\mu_{K}\left(\xi\right) . \end{split} \end{equation} \end{theorem} \begin{proof} Let $K\in\mathcal K_o^n$ and $L\subset \mathbb{R}^{n}$ be a compact convex set containing the origin. We first prove the case that $K, L\in \mathcal{A}_+^{2,\alpha}$. Then, by formula \eqref{3.7} and Lemmas \ref{lem:3.4} and \ref{lem:3.1}, we have \begin{equation*} \begin{split} &{\left. {\frac{d}{{dt}}} \right|_{t = 0}}\Gamma \left( {{K^t}} \right) \\ =& {\left. {\frac{d}{{dt}}} \right|_{t = 0}}\int_{{\mathbb{S}^{n-1}}} {{{\left( {h_K^q\left( \xi \right) + th_L^q\left( \xi \right)} \right)}^{\frac{1}{q}}}\mathcal{F} \left[ {{h_{{K^t}}}} \right]\left( \xi \right)} d\xi \\ =& \int_{{\mathbb{S}^{n-1}}} {{{\left. {\frac{d}{{dt}}} \right|}_{t = 0}}{{\left( {h_K^q\left( \xi \right) + th_L^q\left( \xi \right)} \right)}^{\frac{1}{q}}}\mathcal{F} \left[ {{h_{{K^t}}}} \right]\left( \xi \right)} d\xi + \int_{{\mathbb{S}^{n-1}}} {{h_K}\left( \xi \right){{\left. {\frac{d}{{dt}}} \right|}_{t = 0}}\mathcal{F} \left[ {{h_{{K^t}}}} \right]\left( \xi \right)} d\xi \\ =& \frac{1}{q}\int_{{\mathbb{S}^{n-1}}} {h_L^qh_K^{1-q}\mathcal{F} \left[ {{h_K}} \right]\left( \xi \right)} d\xi +\int_{{\mathbb{S}^{n-1}}} {h_L^q h_K^{1 - q}{{\left. {\frac{d}{{dt}}} \right|}_{t = 0}}\mathcal{F} \left[ {{{\left( {1 + t} \right)}^{\frac{1}{q}}}{h_K}} \right]\left( \xi \right)} d\xi \\ =&\frac{1}{q}\int_{{\mathbb{S}^{n-1}}} {h_L^qh_K^{1-q}\mathcal{F} \left[ {{h_K}} \right]\left( \xi \right)} d\xi +\frac{{n-\mathbf{p}}}{q}\int_{{\mathbb{S}^{n-1}}} {h_L^qh_K^{1-q}\mathcal{F}\left[ {{h_K}} \right]\left( \xi \right)} d\xi \\ =&\frac{{n-\mathbf{p} + 1}}{q}\int_{{\mathbb{S}^{n-1}}} {h_L^qh_K^{1 - q}\mathcal{F} \left[ {{h_K}} \right]\left( \xi \right)}d\xi \\ =&\frac{{n-\mathbf{p} + 1}}{q}\int_{{\mathbb{S}^{n-1}}} {h_L^qh_K^{1-q}}d\mu_{K}. \end{split} \end{equation*} This proves \eqref{3.19} for the case that $K, L\in \mathcal{A}_+^{2,\alpha}$. For $K\in\mathcal K_o^n$ and a compact convex set $L \subset \mathbb{R}^{n}$ containing the origin, we can respectively choose two sequences $\{K_j\}_{j=1}^\infty$ and $\{L_j\}_{j=1}^\infty$ in $\mathcal{A}_+^{2,\alpha}$, such that $K_j\to K$ and $L_j\to L$ as $j\to\infty$. It follows that $h_{K_j}\to h_K$ and $h_{L_j}\to h_L$ uniformly. Then, by \eqref{2.3}, the continuity of the functional $\Gamma$ on compact convex sets and the weak convergence \eqref{2.8}, we can verify the desired \eqref{3.19}. \end{proof} In view of the variational formula \eqref{3.19}, one can generalize the $\mathbf{p}$-harmonic measure and introduce the following $L_{q}~\mathbf{p}$-harmonic measure. \begin{definition}\label{def:3.1} Let $q\in\mathbb{R}$, $1<\mathbf{p}<\infty$, and $K\in\mathcal K_o^n$. We define the $L_q$ $\mathbf{p}$-harmonic measure $\mu _{K,q}$ for each Borel $E\subset \mathbb{S}^{n-1}$ as \begin{equation}\nonumber {\mu _{K,q}}\left(E\right) = \int_E {h_K^{1 - q}\left( \xi \right)} d{\mu _K}\left(\xi\right). \end{equation} \end{definition} The weak convergence of the $L_q$ $\mathbf{p}$-harmonic measure is critical and can be stated as follows. \begin{lemma}\label{lem:3.5} Let $q\in\mathbb{R}$, $1<\mathbf{p}<\infty$, and $K\in\mathcal K_o^n$. Then for any sequence of convex bodies $\{K_j\}$ in $\mathcal K_o^n$, if $K_{j}\to K$ as $j\to\infty$, then $\mu_{K_j,q}$ converges to $\mu_{K,q}$ weakly, as $j\to\infty$. \end{lemma} \begin{proof} It follows from \eqref{2.8} that the $\mathbf{p}$-harmonic measure is convergent weakly. Then, by Definition \ref{def:3.1} and $K_{j}\to K$ as $j\to\infty$, for any function $f\in C\left(\mathbb{S}^{n-1}\right)$, we have \begin{equation}\nonumber \lim\limits_{j\to\infty}\int_{\mathbb{S}^{n-1}}fd\mu_{K_j,q} =\lim\limits_{j\to\infty} \int_{\mathbb{S}^{n-1}}fh_{K_j}^{1-q}d\mu_{K_j} =\int_{\mathbb{S}^{n-1}}fh_K^{1-q}d\mu_K =\int_{\mathbb{S}^{n-1}}fd\mu_{K,q}. \end{equation} Thus, the desired weak convergence follows. \end{proof} \section{The proof of Theorem \ref{th:1.1}}\label{sect:4} In this section, we study the $L_q$ Minkowski problem associated with $\mathbf{p}$-harmonic measure for $0<q<1$ and $1<\mathbf{p}\ne n+1$. By introducing an appropriate functional and studying a related extremal problem as well as the existence of a solution, we can finally prove Theorem \ref{th:1.1} via the variation method. To begin with, we prove the following lemma, which is critical for our later approximation argument. \begin{lemma}\label{lem:4.1} Let $0<q<1$. If $f:\mathbb{S}^{n-1}\to \mathbb{R}$ is a positive, twice continuously differentiable function, there exists a convex body $L$ containing the origin in its interior and a constant $r>0$ such that $$f^q=h_L^q-h^q_{rB_2^n},$$ where $B_2^n$ is the standard unit ball in $\mathbb{R}^n$. \end{lemma} \begin{proof} We extend the function $f$ to $\mathbb{R}^{n}\setminus\left\{o\right\}$ by defining $F\left(x\right):=\left|x\right|f\left(\frac{x}{\left|x\right| }\right)$ and we define $G\left(x\right):=\left|x\right|$ for $x\in\mathbb{R}^n$. Then, we can verify that the function $\left(F^q+r^qG^q\right)^{\frac{1}{q}}$ is positively homogeneous of degree one, where $r>0$. According to Euler's homogeneous function theorem, \[\left\langle {x,\nabla{{{\left( {{F^q} + {r^q}{G^q}} \right)}^{\frac{1}{q}}}}} \right\rangle = {\left( {{F^q} + {r^q}{G^q}} \right)^{\frac{1}{q}}},\] we then take the first derivative with respect to each component $x_j$ of $x$ and obtain \[\sum\limits_{i = 1}^n {\left( {\frac{{\partial {x_i}}}{{\partial {x_j}}} \frac{{\partial \left( {{{\left( {{F^q} + {r^q}{G^q}} \right)}^{\frac{1}{q}}}} \right)}}{{\partial {x_i}}} + {x_i}\frac{{{\partial ^2}\left( {{{\left( {{F^q} + {r^q}{G^q}} \right)}^{\frac{1}{q}}}} \right)}} {{\partial {x_i}\partial {x_j}}}} \right)} = \frac{{\partial \left( {{{\left( {{F^q}+{r^q}{G^q}} \right)}^{\frac{1}{q}}}} \right)}}{{\partial {x_j}}},\] where $j=1,\ldots,n$. Thus, we have \begin{equation}\label{4.1} \sum\limits_{i = 1}^n {\left( {{x_i}\frac{{{\partial ^2}\left( {{{\left( {{F^q} + {r^q}{G^q}} \right)}^{\frac{1}{q}}}} \right)}}{{\partial {x_i}\partial {x_j}}}} \right)} =0, \end{equation} for all $j=1,\ldots,n$. Let $D^2_{x}\left(\left(F^q+r^qG^q\right)^{\frac{1}{q}}\right)$ be the second differential of function ${F^q}+{r^q}{G^q}$ at $x$, that is \[D_x^2\left( {{{\left( {{F^q} + {r^q}{G^q}} \right)}^{\frac{1}{q}}}} \right) = {\left( {\frac{{{\partial ^2}\left( {{{\left( {{F^q} + {r^q}{G^q}} \right)}^{\frac{1}{q}}}} \right)}}{{\partial {x_i}\partial {x_j}}}} \right)_{ij}}.\] It follows from \eqref{4.1} that \begin{equation}\label{4.2} xD^2_{x}\left(\left(F^q+r^qG^q\right)^{\frac{1}{q}}\right)z^\intercal=0, \end{equation} where $z^\intercal$ is the transpose of $z\in\mathbb{R}^{n}$. For any two vectors $x,y\in \mathbb{S}^{n-1}$ with $x\perp y$, we can verify \[yD_x^2\left( {{F^q}+{r^q}{G^q}}\right)y^\intercal =yD_x^2\left( {{F^q}} \right)y^\intercal+q{r^q}.\] Since the second differential $D_x^2\left( {{F^q}} \right)$ of function $F^q$ is continuous on $\mathbb{S}^{n-1}$, and $yD_x^2\left({{F^q}} \right)y^\intercal$ has a minimum, we can choose a suitable $r>0$ so that \begin{equation}\label{4.3} yD_x^2\left( {{ {{F^q} + {r^q}{G^q}}}} \right)y^\intercal \ge 0. \end{equation} Let $x\in \mathbb{S}^{n-1}$. Then for any nonzero $z\in\mathbb{R}^{n}$, there exists $\alpha_1,\alpha_2\in\mathbb{R}$ such that $z=\alpha_1x+\alpha_2x'$, where $x'\perp x$ and $x'\in \mathbb{S}^{n-1}$. Since \begin{equation*} \begin{split} &D_x^2\left( {{{\left( {{F^q} + {r^q}{G^q}} \right)}^{\frac{1}{q}}}} \right)\\ =&\frac{1}{q}\left( {\frac{1}{q} - 1} \right){\left( {{F^q} + {r^q}{G^q}} \right)^{\frac{1}{q}-2}}|{\nabla}\left( {{F^q}+{r^q}{G^q}} \right)|^2\mathrm{I} +\frac{1}{q}{\left( {{F^q} + {r^q}{G^q}} \right)^{\frac{1}{q} - 1}}D_x^2\left( {{F^q} + {r^q}{G^q}} \right), \end{split} \end{equation*} where $\mathrm{I}$ is the unit matrix of order $n$. This, together with \eqref{4.2} and \eqref{4.3}, shows that \begin{equation}\nonumber zD^2_{x}\left(\left(F^q+r^qG^q\right)^{\frac{1}{q}}\right)z^\intercal \ge 0, \end{equation} for any nonzero $z\in\mathbb{R}^{n}$ and $x\in\mathbb{S}^{n-1}$. It follows that the matrix $D_x^2\left( {{{\left( {{F^q} + {r^q}{G^q}} \right)}^{\frac{1}{q}}}} \right)$ is positive semi-definite for any nonzero $x\in\mathbb{R}^{n}$. Then, by Theorem 1.5.13 of \cite{S14}, we can verify that the function ${{{\left( {{F^q} + {r^q}{G^q}} \right)}^{\frac{1}{q}}}}$ is sublinear. The existence of the convex body $L$ directly follows from Theorem 1.7.1 of \cite{S14}. \end{proof} Let $Q$ be a compact convex set, $\mu$ be a finite Borel measure on $\mathbb{S}^{n-1}$, and $0<q<1$. We define the functional $\Phi_Q:Q\to\mathbb{R}$ as follows: \begin{equation}\label{4.4} {\Phi_Q}\left(\zeta\right)=\int_{{\mathbb{S}^{n-1}}} {{{\left( {{h_Q}\left(\xi\right)-\left\langle {\zeta,\xi}\right\rangle}\right)}^{q}}}d\mu\left(\xi\right). \end{equation} Next, we proceed to prove two necessary lemmas concerning the functional $\Phi_Q$. \begin{lemma}\label{lem:4.2} Let $0<q<1$ and $Q$ be a compact convex set, there exists a unique $\zeta\left(Q\right)\in \rm{int} Q$ such that \[{\Phi_Q}\left( {\zeta \left( Q \right)}\right) =\mathop {\sup }\limits_{\zeta\in Q} {\Phi_Q} \left(\zeta\right),\] and for any $x_0\in\mathbb{R}^{n}$, we have ${\zeta\left(Q+x_0\right)}={\zeta\left(Q\right)}+x_0$. \end{lemma} \begin{proof} Let $0<\lambda<1$ and $\zeta_{1},\zeta_{2}\in Q$. From equality \eqref{4.4} and the concavity of the function $s^q$ with $s\ge0$ and $0<q<1$, we obtain that \begin{equation*} \begin{split} &\lambda {\Phi_Q}\left( {{\zeta_1}} \right) +\left( {1-\lambda } \right){\Phi_Q}\left( {{\zeta _2}} \right)\\ =&\int_{{\mathbb{S}^{n-1}}} {\lambda {{\left( {{h_Q}\left( \xi \right) - \left\langle {{\zeta _1},\xi } \right\rangle } \right)}^q} + } \left( {1 - \lambda } \right){\left( {{h_Q}\left( \xi \right) - \left\langle {{\zeta _2},\xi } \right\rangle } \right)^q}d\mu \left( \xi \right)\\ \le&{\int_{{\mathbb{S}^{n-1}}} {\left( {{h_Q}\left( \xi \right) - \left( {\lambda \left\langle {{\zeta _1},\xi } \right\rangle + \left( {1 - \lambda } \right)\left\langle {{\zeta _2},\xi } \right\rangle } \right)} \right)} ^q}d\mu \left( \xi \right)\\ =&{\Phi_Q}\left( {\lambda {\zeta _1} + \left( {1 - \lambda } \right){\zeta _2}} \right), \end{split} \end{equation*} where the equality holds if and only if $\left\langle{{\zeta_1},\xi}\right\rangle =\left\langle{{\zeta_2},\xi}\right\rangle$ for all ${\xi\in \mathbb{S}^{n-1}}$, implying ${{\zeta _1} = {\zeta _2}}$. Therefore, ${\Phi_Q}$ is strictly concave on $Q$, it follows that there exists a unique point ${\zeta\left( Q \right)}\in Q$ such that ${\Phi_Q}\left({\zeta\left(Q\right)}\right) =\sup\limits_{\zeta\in Q}{\Phi_Q}\left(\zeta\right)$. Next, we prove $\zeta\left(Q\right)\in \text{int}Q$. Suppose to the contrary that $\zeta\left(Q\right)\in \partial Q$, and let $\omega$ be the set of all unit outward normal vectors at $\zeta\left(Q\right)$: \[\omega = \left\{ {\left. {\xi \in {\mathbb{S}^{n-1}}} \right|{h_Q}\left( \xi \right) =\left\langle {\zeta \left( Q \right),\xi } \right\rangle } \right\}.\] Take $x_{0}\in \text{int}Q$ and define \[{\xi _0} := \frac{{{x_0} - \zeta \left( Q \right)}}{{\left| {{x_0} - \zeta \left( Q \right)} \right|}}.\] It can be verified that ${\left\langle {{\xi_0},\xi} \right\rangle }<0$ for $\xi \in \omega$. Define \begin{equation}\nonumber \omega_+ :=\left\{\left.{\xi \in{\mathbb{S}^{n-1}}\setminus\omega}\right|\left\langle{{\xi _0},\xi }\right\rangle \ge 0\right\} \ \text{and}\ \omega_- :=\left\{\left.{\xi \in{\mathbb{S}^{n-1}}\setminus\omega}\right|\left\langle{{\xi _0},\xi }\right\rangle < 0\right\}, \end{equation} then for $\xi\in\omega_+$, there exists a $\epsilon>0$ such that ${{h_Q}\left(\xi\right)-\left\langle{\zeta\left(Q \right),\xi} \right\rangle} \ge\epsilon$. Choose $0<\delta<\frac{\epsilon}{2}$ small enough so that $\zeta\left(Q\right)+\delta \xi_0\in\text{int}Q$, which further gives \begin{equation}\nonumber {h_Q}\left(\xi\right)-\left\langle {\zeta\left(Q \right)+\delta{\xi _0},\xi}\right\rangle >\frac{{\epsilon}}{2}, \end{equation} for $\xi\in\omega_+$. These, together with \eqref{4.4} and the Lagrange mean value theorem, imply that \begin{equation}\nonumber \begin{split} &{\Phi_Q}\left({\zeta\left(Q\right)+\delta{\xi_0}}\right)- {\Phi_Q}\left({\zeta\left(Q\right)}\right)\\ =&\int_{{\mathbb{S}^{n-1}}} {{{\left( {{h_Q}\left( \xi \right)-\left\langle{\zeta\left(Q\right)+\delta{\xi_0},\xi } \right\rangle}\right)}^q}}d\mu \left(\xi\right) - \int_{{\mathbb{S}^{n-1}}} {{{\left({{h_Q}\left(\xi\right)- \left\langle {\zeta \left(Q\right),\xi}\right\rangle } \right)}^q}}d\mu\left(\xi\right)\\ =&\int_\omega{{{\left({-\left\langle {\delta {\xi_0},\xi }\right\rangle }\right)}^q}} d\mu \left(\xi \right)+\int_{{\mathbb{S}^{n-1}}\setminus\omega} {{{\left( {{h_Q}\left(\xi\right)-\left\langle {\zeta \left(Q \right) +\delta {\xi_0},\xi } \right\rangle } \right)}^q} - {{\left({{h_Q}\left(\xi\right)-\left\langle {\zeta\left( Q \right),\xi}\right\rangle}\right)}^q}} d\mu \left(\xi\right)\\ \ge&\int_\omega{{{\left({-\left\langle {\delta{\xi_0},\xi } \right\rangle}\right)}^q}} d\mu \left(\xi \right) -\int_{{\omega_+}}{{{\left({{h_Q}\left(\xi \right)-\left\langle{\zeta \left(Q\right),\xi} \right\rangle}\right)}^q} -{{\left( {{h_Q}\left(\xi \right)- \left\langle{\zeta \left( Q \right)+\delta {\xi _0},\xi } \right\rangle}\right)}^q}} d\mu\left(\xi \right)\\ >&\int_\omega{{{\left({-\left\langle{\delta {\xi _0},\xi } \right\rangle}\right)}^q}} d\mu \left(\xi \right)-\int_{{\omega_+}}{q{{\left({\frac{\epsilon}{2}} \right)}^{q-1}}\left\langle{\delta{\xi_0},\xi } \right\rangle }d\mu\left(\xi\right). \end{split} \end{equation} Notice that $\lim\limits_{\delta \to 0^{+}}{\delta^{1-q}}=0$. Hence, there exists a small enough $\delta_{0}>0$ such that ${\Phi _Q}\left( {\zeta \left( Q \right) + \delta {\xi _0}} \right) > {\Phi _Q}\left( {\zeta \left( Q \right)} \right)$, which leads to a contradiction, as $\zeta(Q)$ was chosen such that $\Phi_Q\left( \zeta(Q) \right) = \sup\limits_{\zeta \in Q} \Phi_Q(\zeta)$. Therefore, we conclude that $\zeta\left(Q\right)\in \text{int}Q$. Thus, for any $x_0\in \mathbb{R}^{n}$, we have \begin{equation*} \begin{split} {\Phi _{Q + x_0}}\left( {\zeta \left( {Q + x_0} \right)} \right) &=\mathop {\sup }\limits_{\zeta \in Q + x_0} \int_{{\mathbb{S}^{n-1}}} {{{\left( {{h_{Q + x_0}} \left( \xi \right) - \left\langle {\zeta,\xi } \right\rangle } \right)}^q}} d\mu \left( \xi \right)\\ &=\mathop {\sup }\limits_{\zeta \in Q} \int_{{\mathbb{S}^{n-1}}} {{{\left( {{h_Q}\left( \xi \right) - \left\langle {\zeta ,\xi } \right\rangle } \right)}^q}} d\mu \left( \xi \right)\\ &={\Phi _Q}\left( {\zeta \left( Q \right)} \right)\\ &=\int_{{\mathbb{S}^{n-1}}} {{{\left( {{h_{Q + x_0}}\left( \xi \right) -\left\langle {{\zeta(Q)+ x_0},\xi } \right\rangle } \right)}^q}} d\mu \left( \xi \right)\\ &={\Phi _{Q + x_0}}\left( {\zeta(Q)+ x_0} \right). \end{split} \end{equation*} Therefore, by the uniqueness of the extreme point $\zeta\left(Q+x_0\right)$, we conclude that ${\zeta\left(Q+x_0\right)}={\zeta\left(Q\right)}+x_0$. \end{proof} \begin{lemma}\label{lem:4.3} Let $0<q<1$, $\mu$ be a finite Borel measure on $\mathbb{S}^{n-1}$, and $\left\{Q_{j}\right\}^\infty_{j=1}$ be a sequence of compact convex sets. If $Q_j$ converges to a compact convex set $Q$ as $j\to\infty$, then we have $\lim\limits_{j\to \infty}\zeta\left(Q_{j}\right)=\zeta\left(Q\right)$ and $\mathop {\lim }\limits_{j \to \infty }\Phi_{Q_{j}}\left(\zeta \left(Q_{j}\right)\right)=\Phi_{Q}\left(\zeta \left(Q\right)\right)$. \end{lemma} \begin{proof} Since the sequence $\{\zeta(Q_j)\}$ is bounded, there exists a convergent subsequence (still denoted by $\{\zeta(Q_j)\}$) that converges to some $\zeta_{0}\in Q$. Next, we prove that $\zeta_{0}=\zeta(Q)$. If otherwise, by using \eqref{4.4} and Lemma \ref{lem:4.2}, we have \[\mathop {\lim }\limits_{j \to \infty } {\Phi _{{Q_j}}}\left( {\zeta \left( {{Q_j}} \right)} \right) = {\Phi _Q}\left( {{\zeta _0}} \right)<{\Phi _Q}\left( {{\zeta \left(Q\right)}}\right) =\mathop {\lim }\limits_{j \to \infty } {\Phi _{{Q_j}}}\left( {\zeta \left( {{Q}} \right)} \right).\] On the other hand, since ${\zeta \left( {{Q}} \right)}\in \text{int}Q_{j}$ for sufficiently large $j$, it follows that ${\Phi _{{Q_j}}}\left( {\zeta \left( {{Q_j}} \right)} \right) >{\Phi _{{Q_j}}}\left( {\zeta \left( Q \right)} \right)$ for sufficiently large $j$. This contradiction implies that $\zeta_{0}=\zeta\left( {{Q}} \right)$. Using \eqref{4.4} again, we can verify that $\mathop {\lim }\limits_{j \to \infty }\Phi_{Q_{j}}\left(\zeta \left(Q_{j}\right)\right) =\Phi_{Q}\left(\zeta \left(Q\right)\right)$. \end{proof} Now, we are able to prove the Theorem \ref{th:1.1} as follows. \begin{proof}[Proof of Theorem \ref{th:1.1}] Recall that the Wulff shape $K_f$ associated with a function $f\in C_+\left(\mathbb{S}^{n-1}\right)$ is given by \begin{equation}\nonumber {K_f} =\left\{{x\in\mathbb{R}^{n}:\left\langle {x,u}\right\rangle \le f\left(u\right)}\ \text{for all}\ u\in\mathbb{S}^{n-1}\right\}. \end{equation} Then for $0<q<1$, $f\in C_+\left(\mathbb{S}^{n-1}\right)$, and a finite Borel measure $\mu$ on $\mathbb{S}^{n-1}$, we introduce a functional $\Phi_f:K_f\to\mathbb{R}$ by \begin{equation}\label{4.5} {\Phi_f}\left(\zeta\right) =\int_{{\mathbb{S}^{n-1}}} {{{\left( {f\left(\xi\right)-\left\langle {\zeta,\xi}\right\rangle}\right)}^{q}}}d\mu\left(\xi\right), \end{equation} for $\zeta\in K_f$. We then construct the following minimization problem: \begin{equation}\label{4.6} \mathop {\inf }\limits_{f \in {C_+} \left( {{\mathbb{S}^{n-1}}} \right)} \left\{ {\mathop {\sup }\limits_{\zeta \in {K_f}} {\Phi _{f}} \left( \zeta \right):\Gamma \left( K_{f} \right)= \Gamma \left( B_2^n \right)} \right\}. \end{equation} Since $h_{K_f}\le f$ and $K_{h_{K_f}}=K_f\in \mathcal K_o^n$ for any $f\in C_+\left(\mathbb{S}^{n-1}\right)$, by \eqref{4.4} and \eqref{4.5}, we obtain that $$\Phi_{K_f}\left(\zeta\right)=\Phi_{h_{K_f}}\left(\zeta\right)\le{\Phi_f}\left(\zeta\right),$$ where $\zeta\in{K_f}$. It follows that $\mathop{\sup}\limits_{\zeta\in{K_f}} {\Phi_{K_f}}\left(\zeta\right) \le\mathop{\sup}\limits_{\zeta\in{K_f}} {\Phi_f}\left(\zeta\right).$ Therefore, we can search for the minimum for \eqref{4.6} among the support functions of convex bodies that contain the origin in their interiors, and we can verify that $h_K$ is a solution to \eqref{4.6} if and only if $K$ is a solution to the problem \begin{equation}\label{4.7} \mathop {\inf }\limits_{Q \in \mathcal K_o^n} \left\{ {\mathop {\sup }\limits_{\zeta \in Q} {\Phi_Q} \left( \zeta \right):\Gamma \left( Q \right)=\Gamma \left(B_2^n\right)} \right\}. \end{equation} Let $\left\{{Q_j}\right\}_{j = 1}^\infty $ be a minimizing sequence for the problem \eqref{4.7}. That is, $\Gamma\left(Q_j \right)=\Gamma\left(B_2^n \right)$ and $$\mathop {\lim}\limits_{j\to\infty} {\Phi _{Q_j}} \left( {\zeta \left( {Q_j} \right)} \right) = \mathop {\inf }\limits_{Q \in \mathcal K_o^n} \left\{ {\Phi_Q\left(\zeta \left( {Q} \right)\right):\Gamma \left( Q \right) = \Gamma \left( B_2^n \right)} \right\}.$$ According to Lemma \ref{lem:4.2}, we can suitably translate each $Q_j$ to obtain a sequence $\left\{K_j\right\}_{j=1}^\infty$ in $\mathcal K_o^n$ such that $\zeta\left(K_j\right)=o$ and $\Gamma \left( {{K_j}} \right)=\Gamma \left( B_2^n \right)$ by \eqref{3.2}. Therefore, $\left\{ {{K_j}} \right\}_{j = 1}^\infty$ is also the minimizing sequence for the problem \eqref{4.7}, and ${\Phi _{K_j}}\left(o\right)$ converges to $$\mathop {\inf }\limits_{Q \in {\mathcal{K}_o^n}} \left\{ {{\Phi_Q}\left( {\zeta \left( Q \right)} \right):\Gamma \left( Q \right) = \Gamma \left( B_2^n \right)} \right\},$$ as $j\to\infty$. We now prove that the sequence $\left\{{K_j} \right\}$ is uniformly bounded. To do so, we let $R_j:=\mathop {\max }\limits_{\xi \in {\mathbb{S}^{n-1}}} {h_{{K_j}}}\left(\xi\right)$ and assume that the maximum can be achieved by some $\xi_0\in \mathbb{S}^{n-1}$. Then, we have $$R_{j}{\left\langle {{\xi _0},\xi } \right\rangle _ + } \le {h_{{K_j}}}\left( \xi \right)$$ for all $j$ and $\xi\in\mathbb{S}^{n-1}$, and hence \begin{equation}\label{4.8} \int_{\mathbb{S}^{n-1}} {{\left({R_{j}{{\left\langle{{\xi _0},\xi} \right\rangle}_+}}\right)}^q}d\mu \left(\xi\right) \le\int_{\mathbb{S}^{n-1}}{{\left( {{h_{{K_j}}}\left( \xi \right)} \right)}^q} d\mu \left( \xi\right) ={\Phi_{K_j}}\left(o\right). \end{equation} On the other hand, for sufficiently large $j$, we have \begin{equation}\nonumber {\Phi_{K_j}}\left(o\right) \le\Phi_{B_2^n-\zeta\left(B_2^n\right)}\left(o\right) =\int_{\mathbb{S}^{n-1}}{{\left(1-\left\langle{\zeta\left(B_2^n\right),\xi} \right\rangle\right)}^q} d\mu \left( \xi\right). \end{equation} This, together with \eqref{4.8}, implies that $\{R_j\}$ is uniformly bounded, where we have used the fact that the measure $\mu$ is finite and not concentrated on any closed hemisphere. Therefore, the boundedness of the sequence $\left\{{K_j}\right\}$ follows. By the Blaschke selection theorem, there exists a subsequence (still denoted by $\left\{ {{K_j}} \right\}$) that converges to some compact convex set $\Omega$ as $j \to \infty$. In the following, we prove that $\dim (\Omega)=n$. If $\dim (\Omega)<n-1$, then $\mathcal{H}^{n-1}\left(\Omega\right)=0=\mathcal{H}^{n-1}\left(\partial \Omega\right)$. It follows from definition \eqref{3.1} and Lemma \ref{lem:2.1} that $\Gamma \left( \Omega \right)=0$, which contradicts to the following \begin{equation}\label{4.9} \Gamma\left(\Omega\right) =\lim\limits_{j\to\infty}\Gamma\left(K_j \right) =\Gamma\left(B_2^n\right) >0. \end{equation} If $\dim (\Omega) = n - 1$, there are at least two half-spaces containing $\Omega$ that share a common boundary, and $\Omega$ degenerates to a $1$-codimensional subset of a hyperplane. By Lemma \ref{lem:2.1} again, \begin{equation}\nonumber \left| {\nabla u} \right|\le M, \end{equation} thus we obtain that \begin{equation}\nonumber \Gamma\left(\Omega\right) =\int_{\mathbb{S}^{n-1}}{h_\Omega}d{\mu_\Omega} \le{M^{\mathbf{p}-1}}\int_{\mathbb{S}^{n-1}} {{h_\Omega}} d{S_\Omega}\left(\xi\right) =0, \end{equation} which again contradicts to \eqref{4.9}. Therefore, $\text{dim}(\Omega)=n$ and $\Omega$ is indeed a convex body. By Lemma \ref{lem:4.3}, we have ${\zeta\left(\Omega\right)=o}$ and \begin{equation}\label{4.10} {\Phi _{{h_\Omega}}}\left( o \right) =\mathop {\inf }\limits_{f \in {C^ + }\left( {{\mathbb{S}^{n-1}}} \right)} \left\{ {\mathop {\sup }\limits_{\zeta\in{K_f}} {\Phi _f}\left( \zeta \right): \Gamma \left( {{K_f}} \right) = \Gamma \left( B_2^n \right)} \right\}. \end{equation} Let $\Omega_1$ be a compact convex set containing the origin and $\Omega^t$ be the Wulff shape of ${\left({h_\Omega^q+t{h_{\Omega_1}^q}}\right)}^{\frac{1}{q}}$ for a small enough $t$, where $$\lambda\left(t\right) :={\left( {\frac{{\Gamma \left( B_2^n \right)}}{\Gamma \left(\Omega^t\right)}}\right)^{\frac{1}{n-\mathbf{p}+1}}}.$$ Here, we have used the condition that $\mathbf{p}\neq n+1$. Then, by equalities \eqref{3.1} and \eqref{3.8}, we can verify that $\Gamma\left({\lambda\left(t\right){\Omega^t}}\right)=\Gamma\left(B_2^n\right)$. In the following, we prove that $\zeta\left(t\right):=\zeta\left({\lambda\left(t\right)\Omega^t}\right)$ is differentiable at $t=0$. Let $\zeta=(\zeta_1,\zeta_2,\ldots,\zeta_n)$ and $F=(F_1,F_2,\ldots,F_n)$ be a vector-value function from an open neighbourhood of the origin $\left({0,0,0,\ldots,0}\right)$ in $\mathbb{R}^{n+1}$ to $\mathbb{R}^{n}$, where $${F_i}\left( {t,{\zeta _1},{\zeta _2}, \ldots ,{\zeta _n}} \right) =\int_{{\mathbb{S}^{n-1}}} {\frac{{{\xi _i}}}{{{{\left( {\lambda \left( t \right){h_{{\Omega^t}}} \left( \xi \right) - \left( {{\zeta _1}{\xi _1}+{\zeta _2}{\xi _2}+ \cdots +{\zeta _n}{\xi _n}} \right)} \right)}^{1 - q}}}}} d\mu \left( \xi \right)$$ for $i=1,2,\ldots,n$. As $\zeta(t)$ is an extreme point of $\Phi_{\lambda(t)\Omega^t}\left(\zeta\right)$ for $\zeta\in\lambda(t)\Omega^t$, it follows that $F_i(t,\zeta(t))=0$. Then, two functions both \begin{equation}\nonumber {\left. {\frac{{\partial {F_i}}}{{\partial t}}} \right|_{\left( {t,{\zeta _1},{\zeta _2}, \ldots ,{\zeta _n}} \right)}} = \int_{{\mathbb{S}^{n-1}}} {\frac{{\left( {q - 1} \right){\xi _i}\left( {\lambda '\left( t \right){h_{\Omega^t}}\left( \xi \right) +\lambda\left(t\right){h_{\Omega^t}'}\left( \xi \right)} \right)}}{{{{\left( {\lambda \left( t \right){h_{{\Omega^t}}}\left( \xi \right) - \left( {{\zeta _1}{\xi _1}+{\zeta _2}{\xi _2}+ \cdots +{\zeta _n}{\xi_n}} \right)} \right)}^{2-q}}}}} d\mu \left( \xi \right) \end{equation} and \begin{equation}\nonumber {\left. {\frac{{\partial {F_i}}}{{\partial {\zeta _j}}}} \right|_{\left( {t,{\zeta _1},{\zeta _2}, \ldots ,{\zeta _n}} \right)}} = \int_{{\mathbb{S}^{n-1}}} {\frac{{\left( {1 - q} \right){\xi _i}{\xi _j}}}{{{{\left( {\lambda \left( t \right){h_{{\Omega^t}}}\left( \xi \right) - \left( {{\zeta _1}{\xi _1}+{\zeta _2}{\xi _2}+ \cdots +{\zeta _n}{\xi _n}} \right)} \right)}^{2-q}}}}} d\mu \left( \xi \right) \end{equation} are all continuous on a small neighbourhood of $\left(0,0,0,\ldots,0\right)$, and \begin{equation}\label{4.11} {\left( {{{\left. {\frac{{\partial F}}{{\partial \zeta }}} \right|}_{\left( {0,0,0, \ldots ,0} \right)}}} \right)_{n \times n}} = \int_{{\mathbb{S}^{n-1}}} {\frac{{\left( {1 - q} \right)\xi^\intercal \xi}}{{h_\Omega^{2 - q}\left( \xi \right)}}} d\mu \left( \xi \right), \end{equation} where ${{\xi ^\intercal}\xi }$ is an $(n\times n)$ matrix. As the measure $\mu$ is not concentrated on any closed hemisphere, for any nonzero $x\in\mathbb{R}^{n}$, we have \begin{equation}\nonumber {x}{\left( {{{\left. {\frac{{\partial F}}{{\partial \zeta }}} \right|}_{\left( {0,0,0, \ldots ,0} \right)}}} \right)_{n \times n}}x^\intercal = \int_{{\mathbb{S}^{n-1}}} {\frac{{\left( {1 - q} \right){{\left\langle {x,\xi } \right\rangle }^2}}}{{h_\Omega^{2 - q}\left( \xi \right)}}} d\mu \left( \xi \right) >0. \end{equation} It follows that the matrix in \eqref{4.11} is positive definite. Then, by ${F_i}\left({0,0,0,\ldots,0}\right)=0$ and the continuity of ${\partial {F_i}} \mathord{\left/{\vphantom{\partial{F_i} {\partial{\zeta _j}}}}\right. \kern-\nulldelimiterspace} {\partial {\zeta _j}}$ on a neighbourhood of $\left({0,0,0,\ldots ,0}\right)$, one can use the implicit function theorem to obtain that $\zeta\left( t \right)$ is continuously differentiable on a small neighbourhood of $\left({0,0,0,\ldots,0}\right)$. Hence, the derivative $\zeta'\left(0\right)$ of $\zeta \left(t\right)$ at $t=0$ exists. Put $\Phi\left(t\right): =\Phi_{\lambda \left( t \right){{\left(h_\Omega^q + th_{\Omega_1}^q \right)}^{\frac{1}{q}}}} \left( {\zeta\left( t \right)} \right)$, then \eqref{4.10} shows that $\Phi\left(t\right)$ attains the minimal value at $t=0$. Thus by \eqref{4.10} and \begin{equation}\nonumber \lambda'\left( 0 \right) = - \frac{1}{\left( {n-\mathbf{p} + 1}\right)\Gamma \left(B_2^n\right)} {\left.{\frac{d}{dt}}\right|_{t = 0}}\Gamma\left({\Omega^t}\right), \end{equation} we have the following calculation: \begin{equation}\label{4.12} \begin{split} 0=&{\left.{\frac{d}{dt}}\right|_{t = 0}}\Phi \left( t \right)\\ =&{\left.{\frac{d}{dt}}\right|_{t = 0}}\int_{{\mathbb{S}^{n-1}}} {{\left({\lambda\left(t\right){{\left( {h_\Omega^q + t{h_{\Omega_1}^q}}\right)}^{\frac{1}{q}}} -\left\langle{\zeta \left( t \right),\xi } \right\rangle} \right)}^{q}} d\mu\left(\xi\right)\\ =&q\int_{{\mathbb{S}^{n-1}}} {h_\Omega^{q - 1}\left( {\lambda '\left( 0 \right){h_\Omega} + \frac{1}{q}h_\Omega^{1 - q}h_{\Omega_1}^q -\left\langle {\zeta'\left(0\right),\xi } \right\rangle } \right)} d\mu \left( \xi \right)\\ =&q\int_{{\mathbb{S}^{n-1}}} {h_\Omega^{q - 1}\left( { - \frac{{{h_\Omega}}}{{\left( {n-\mathbf{p} + 1} \right) \Gamma \left( B_2^n \right)}}{{\left. {\frac{d}{{dt}}} \right|}_{t = 0}}\Gamma \left({\Omega^t} \right) +\frac{1}{q}h_\Omega^{1 - q}h_{\Omega_1}^q} \right)} d\mu \left( \xi\right)\\ &-q\int_{{\mathbb{S}^{n-1}}} {\left\langle {\zeta'\left(0\right),h_\Omega^{{q} - 1}\xi } \right\rangle } d\mu \left( \xi \right)\\ =&-\frac{q}{{n-\mathbf{p} + 1}}\int_{{\mathbb{S}^{n-1}}} {\frac{{h_\Omega^q}}{{\Gamma \left( B_2^n\right)}} {{\left. {\frac{d}{{dt}}}\right|}_{t = 0}}\Gamma \left(\Omega^t\right)} d\mu\left( \xi \right) +\int_{{\mathbb{S}^{n-1}}} {h_{\Omega_1}^q} d\mu \left( \xi \right)\\ &-q\left\langle {\zeta'\left(0\right),\int_{{\mathbb{S}^{n-1}}} {h_\Omega^{{p}-1}\xi } d\mu \left( \xi \right)}\right\rangle. \end{split} \end{equation} Since ${\zeta\left(\Omega\right)=o}$ is an extreme point of $\Phi_\Omega\left(\zeta\right)$ for $\zeta\in \Omega$, we have \begin{equation}\nonumber \int_{{\mathbb{S}^{n-1}}} h_\Omega^{q-1}\xi d\mu(\xi)=o. \end{equation} This, together with \eqref{4.12} and Theorem \ref{th:3.1}, gives that \begin{equation}\label{4.13} \begin{split} \int_{{\mathbb{S}^{n-1}}} {{h_{\Omega_1}^q}} d\mu =&\frac{q}{{n-\mathbf{p} + 1}} \int_{{\mathbb{S}^{n-1}}} {\frac{{h_\Omega^q}}{{\Gamma \left( B_2^n \right)}} {{\left. {\frac{d}{{dt}}} \right|}_{t = 0}}\Gamma \left( {\Omega^t} \right)} d\mu \\ =&\int_{{\mathbb{S}^{n-1}}} {\frac{{h_\Omega^q}}{{\Gamma \left( B_2^n \right)}} \int_{{\mathbb{S}^{n-1}}} {{h_{\Omega_1}^q}} d{\mu _{\Omega,q}}} d\mu \\ =&\int_{{\mathbb{S}^{n-1}}} {{h_{\Omega_1}^q} \int_{{\mathbb{S}^{n-1}}} {\frac{{h_\Omega^q}}{{\Gamma \left( B_2^n \right)}}d\mu }} d{\mu_{\Omega,q}}. \end{split} \end{equation} For any $f\in C_{+}\left(\mathbb{S}^{n-1} \right)$, there exists a sequence of positive twice continuously differentiable functions $\left\{ {{f_j}} \right\}_{j = 1}^\infty$ that converges to $f$. Then for each $f_{j}$, Lemma \ref{lem:4.1} shows that there exists a convex body $L_j$ containing the origin in its interior and a constant $r_j>0$, such that $f_j^q=h^q_{L_j}-h^q_{r_jB_2^n}$. Hence, by \eqref{4.13}, we have \begin{equation}\label{4.14} \int_{{\mathbb{S}^{n-1}}} {{h_{L_j}^q}} d\mu =\int_{{\mathbb{S}^{n-1}}} {{h_{L_j}^q}\int_{{\mathbb{S}^{n-1}}} {\frac{{h_\Omega^q}}{{\Gamma \left( B_2^n \right)}}d\mu }} d{\mu _{\Omega,q}}, \end{equation} and similarly, \begin{equation}\label{4.15} \int_{{\mathbb{S}^{n-1}}} {{h_{r_jB_2^n}^q}} d\mu =\int_{{\mathbb{S}^{n-1}}} {{h_{r_jB_2^n}^q}\int_{{\mathbb{S}^{n-1}}} {\frac{{h_\Omega^q}}{{\Gamma \left( B_2^n \right)}}d\mu }} d{\mu _{\Omega,q}}. \end{equation} By subtracting \eqref{4.15} from \eqref{4.14} and using the approximate argument, we conclude \begin{equation}\nonumber \int_{{\mathbb{S}^{n-1}}} {{f^q}} d\mu =c\int_{{\mathbb{S}^{n-1}}} {{f^q}} d{\mu _{\Omega,q}}, \end{equation} where $$ c=\int_{{\mathbb{S}^{n-1}}} \frac{{h_\Omega^q}}{\Gamma \left( B_2^n \right)}d\mu . $$ By the Riesz representation theorem, we have $\mu=c{\mu_{\Omega,q}}$. Furthermore, Lemma \ref{lem:3.1} and Definition \ref{def:3.1} imply that the $L_q$ $\mathbf{p}$-harmonic measure is positively homogeneous of degree $(n-\mathbf{p}+1-q)$, then there exists a convex body $\tilde{\Omega}$ so that $\mu=\mu_{\tilde{\Omega},q}$, if $\mathbf{p}\neq n+1-q$. We have completed the proof of Theorem \ref{th:1.1}. \end{proof} \vskip 5mm \noindent {\bf Acknowledgements} This paper is partially supported by the China Postdoctoral Science Foundation (No. 2024M761902), the National Natural Science Foundation of China (No. 12371060) and the Shaanxi Fundamental Science Research Project for Mathematics and Physics (No. 22JSZ012). The third author also received support from the Mathematical Sciences Institute at the Australian National University. \vskip 5mm \begin{thebibliography}{99} \addtolength{\itemsep}{-1.5ex} \bibitem{AV22} M. Akman, J. Gong, J. Hineman, J.L. Lewis and A. Vogel, {\it The Brunn-Minkowski inequality and a Minkowski problem for nonlinear capacity}, Mem. Amer. Math. Soc. {\bf275} (2022), vi+115. \bibitem{AM24} M. Akman and S. Mukherjee, {\it On the Minkowski problem for $p$-harmonic measures}, arXiv:2310.10247 (2024), https://arxiv.org/abs/2310.10247. \bibitem{BZ12} K. B\"or\"oczky, E. Lutwak, D. Yang and G. Zhang, {\it The log-Brunn-Minkowski inequality}, Adv. Math. {\bf231} (2012), 1974-1997. \bibitem{BZ13} K. B\"or\"oczky, E. Lutwak, D. Yang and G. Zhang, {\it The logarithmic Minkowski problem}, J. Amer. Math. Soc. {\bf26} (2013), 831-852. \bibitem{CL22} S. Chen, Y. Feng and W. Liu, {\it Uniqueness of solutions to the logarithmic Minkowski problem in $\mathbb{R}^3$}, Adv. Math. {\bf411} (2022), Paper No. 108782. \bibitem{CZ23} S. Chen, S. Hu, W. Liu and Y. Zhao, {\it On the planar Gaussian-Minkowski problem}, Adv. Math. {\bf435} (2023), part A, Paper No. 109351. \bibitem{CL20} S. Chen, Y. Huang, Q. Li and J. Liu, {\it The $L_p$-Brunn-Minkowski inequality for $p<1$}, Adv. Math. {\bf368} (2020), Paper No. 107166. \bibitem{C06} W. Chen, {\it $L_p$ Minkowski problem with not necessarily positive data}, Adv. Math. {\bf201} (2006), 77-89. \bibitem{CY76} S.Y. Cheng and S.T. Yau, {\it On the regularity of the solution of the $n$-dimensional Minkowski problem}, Comm. Pure Appl. Math. {\bf29} (1976), 495-516. \bibitem{CW06} K.S. Chou and X.J. Wang, {\it The $L_p$-Minkowski problem and the Minkowski problem in centroaffine geometry}, Adv. Math. {\bf205} (2006), 33-83. \bibitem{CZ09} A. Cianchi, E. Lutwak, D. Yang and G. Zhang, {\it Affine Moser-Trudinger and Morrey-Sobolev inequalities}, Calc. Var. PDE. {\bf36} (2009), 419-436. \bibitem{CF10} A. Colesanti and M. Fimiani, {\it The Minkowski problem for torsional rigidity}, Indiana Univ. Math. J. {\bf59} (2010), 1013-1039. \bibitem{CZ15} A. Colesanti, K. Nystr\"om, P. Salani, J. Xiao, D. Yang and G. Zhang, {\it The Hadamard variational formula and the Minkowski problem for $p$-capacity}, Adv. Math. {\bf285} (2015), 1511-1588. \bibitem{CS03} A. Colesanti and P. Salani, {\it The Brunn-Minkowski inequality for $p$-capacity of convex bodies}, Math. Ann. {\bf327} (2003), 459-479. \bibitem{CK15} D. Cordero-Erausquin and B. Klartag, {\it Moment measures}, J. Funct. Anal. {\bf268} (2015), 3834-3866. \bibitem{DZ12} J. Dou and M. Zhu, {\it The two dimensional $L_p$ Minkowski problem and nonlinear equations with negative exponents}, Adv. Math. {\bf230} (2012), 1209-1221. \bibitem{FY22} N. Fang, S. Xing and D. Ye, {\it Geometry of log-concave functions: the $L_p$ Asplund sum and the $L_p$ Minkowski problem}, Calc. Var. PDE. {\bf61} (2022), Paper No. 45. \bibitem{FX23} Y. Feng, S. Hu and L. Xu, {\it On the $L_p$ Gaussian Minkowski problem}, J. Differ. Equ. {\bf363} (2023), 350-390. \bibitem{G13} E.I. Galakhov, {\it Comparison principles for the $p$-Laplacian operator}, J. Math. Sci. (N.Y.) {\bf202} (2014), 825-834. \bibitem{G06} R.J. Gardner, {\it Geometric Tomography}, second ed., Cambridge Univ. Press, New York, 2006, xxii+492. \bibitem{GT01} D. Gilbarg, N.S. Trudinger, {\it Elliptic partial differential equations of second order}, Springer-Verlag, Berlin, 2001, xiv+517. \bibitem{G07} P. Gruber, {\it Convex and discrete geometry}, Springer, Berlin, 2007, xiv+578. \bibitem{GZ24} L. Guo, D. Xi and Y. Zhao, {\it The $L_p$ chord Minkowski problem in a critical interval}, Math. Ann. {\bf389} (2024), 3123-3162. \bibitem{HS09} C. Haberl and F. Schuster, {\it Asymmetric affine $L_p$ Sobolev inequalities}, J. Funct. Anal. {\bf257} (2009), 641-658. \bibitem{HZ18} H. Hong, D. Ye and N. Zhang, {\it The $p$-capacitary Orlicz-Hadamard variational formula and Orlicz-Minkowski problems}, Calc. Var. PDE. {\bf57} (2018), Paper No. 5. \bibitem{HS04} C. Hu, X. Ma and C. Shen, {\it On the Christoffel-Minkowski problem of Firey's $p$-sum}, Calc. Var. PDE. {\bf21} (2004), 137-155. \bibitem{HZ23} Z. Hu and H. Li, {\it On the existence of solutions to the Orlicz-Minkowski problem for torsional rigidity}, Arch. Math. {\bf120} (2023), 543-555. \bibitem{HX15} Y. Huang, J. Liu and L. Xu, {\it On the uniqueness of $L_p$-Minkowski problems: the constant $p$-curvature case in $\mathbb{R}^3$}, Adv. Math. {\bf281} (2015), 906-927. \bibitem{HZ16} Y. Huang, E. Lutwak, D. Yang and G. Zhang, {\it Geometric measures in the dual Brunn-Minkowski theory and their associated Minkowski problems}, Acta Math. {\bf216} (2016), 325-388. \bibitem{HZ21} Y. Huang, D. Xi and Y. Zhao, {\it The Minkowski problem in Gaussian probability space}, Adv. Math. {\bf385} (2021), Paper No. 107769. \bibitem{HZ05} D. Hug, E. Lutwak, D. Yang and G. Zhang, {\it On the $L_p$ Minkowski problem for polytopes}, Discrete Comput. Geom. {\bf33} (2005), 699-715. \bibitem{J89} D. Jerison, {\it Harmonic measure in convex domains}, Bull. Amer. Math. Soc. (N.S.) {\bf21} (1989), 255-260. \bibitem{J91} D. Jerison, {\it Prescribing harmonic measure on convex domains}, Invent. Math. {\bf105} (1991), 375-400. \bibitem{J96} D. Jerison, {\it A Minkowski problem for electrostatic capacity}, Acta Math. {\bf176} (1996), 1-47. \bibitem{JZ16} H. Jian, J. Lu and G. Zhu, {\it Mirror symmetric solutions to the centro-affine Minkowski problem}, Calc. Var. PDE. {\bf55} (2016), Paper No. 41. \bibitem{L77} J.L. Lewis, {\it Capacitary functions in convex rings}, Arch. Ration. Mech. Anal. {\bf66} (1977), 201-224. \bibitem{L06} J.L. Lewis, {\it Note on $p$-harmonic measure}, Comput. Methods Funct. Theory {\bf6} (2006), 109-144. \bibitem{L13} J.L. Lewis, K. Nystr\"om and A. Vogel, {\it On the dimension of $p$-harmonic measure in space}, J. Eur. Math. Soc. {\bf15} (2013), 2197-2256. \bibitem{LZ24} C. Li and X. Zhao, {\it Flow by Gauss curvature to the Minkowski problem of $p$-harmonic measure}, arXiv:2404.18757v2 (2024), https://arxiv.org/abs/2404.18757. \bibitem{LH23} H. Li and Z. Hu, {\it On the polar Orlicz Minkowski type problem for the general mixed $\mathfrak{p}$-capacity}, J. Math. Anal. Appl. {\bf522} (2023), Paper No. 126925. \bibitem{LZ24+} N. Li, D. Ye and B. Zhu, {\it The dual Minkowski problem for unbounded closed convex sets}, Math. Ann. {\bf388} (2024), 2001-2039. \bibitem{LZ20} N. Li and B. Zhu, {\it The Orlicz-Minkowski problem for torsional rigidity}, J. Differ. Equ. {\bf269} (2020), 8549-8572. \bibitem{LW20} Q. Li, W. Sheng and X.J. Wang, {\it Flow by Gauss curvature to the Aleksandrov and dual Minkowski problems}, J. Eur. Math. Soc. {\bf22} (2020), 893-923. \bibitem{L88} G. Lieberman, {\it Boundary regularity for solutions of degenerate elliptic equations}, Nonlinear Anal. {\bf12} (1988), 1203-1219. \bibitem{LX24} Y. Liu, X. Lu, Q. Sun and G. Xiong, {\it The logarithmic Minkowski problem in $\mathbb{R}^2$}, Pure Appl. Math. Q. {\bf20} (2024), 869-902. \bibitem{LW13} J. Lu and X.J. Wang, {\it Rotationally symmetric solutions to the $L_p$-Minkowski problem}, J. Differ. Equ. {\bf254} (2013), 983-1005. \bibitem{L93} E. Lutwak, {\it The Brunn-Minkowski-Firey theory. I. Mixed volumes and the Minkowski problem}, J. Differ. Geom. {\bf38} (1993), 131-150. \bibitem{LZ24++} E. Lutwak, D. Xi, D. Yang and G. Zhang, {\it Chord measures in integral geometry and their Minkowski problems}, Comm. Pure Appl. Math. {\bf77} (2024), 3277-3330. \bibitem {LZ02} E. Lutwak, D. Yang and G. Zhang, {\it Sharp affine $L_p$ Sobolev inequalities}, J. Differ. Geom. {\bf62} (2002), 17-38. \bibitem{LZ04} E. Lutwak, D. Yang and G. Zhang, {\it On the $L_p$-Minkowski problem}, Trans. Amer. Math. Soc. {\bf356} (2004), 4359-4370. \bibitem{M24} E. Milman, {\it A sharp centro-affine isospectral inequality of Szeg\"o-Weinberger type and the $L^p$-Minkowski problem}, J. Differ. Geom. {\bf127} (2024), 373-408. \bibitem{R22} L. Rotem, {\it A Riesz representation theorem for functionals on log-concave functions}, J. Funct. Anal. {\bf282} (2022), Paper No. 109396. \bibitem{S14} R. Schneider, {\it Convex Bodies: The Brunn-Minkowski theory}, Cambridge Univ. Press, Cambridge, 2014, xxii+736. \bibitem{S18} R. Schneider, {\it A Brunn-Minkowski theory for coconvex sets of finite volume}, Adv. Math. {\bf332} (2018), 199-234. \bibitem{S24} R. Schneider, {\it A weighted Minkowski theorem for pseudo-cones}, Adv. Math. {\bf450} (2024), Paper No. 109760. \bibitem{S02} A. Stancu, {\it The discrete planar $L_0$-Minkowski problem}, Adv. Math. {\bf167} (2002), 160-174. \bibitem{S03} A. Stancu, {\it On the number of solutions to the discrete two dimensional $L_0$-Minkowski problem}, Adv. Math. {\bf180} (2003), 290-323. \bibitem{U03} V. Umanskiy, {\it On solvability of two-dimensional $L_p$-Minkowski problem}, Adv. Math. {\bf180} (2003), 176-186. \bibitem{TX23} J. Tao, G. Xiong and J. Xiong, {\it The logarithmic Minkowski inequality for cylinders}, Proc. Amer. Math. Soc. {\bf151} (2023), 2143-2154. \bibitem{T84} P. Tolksdorf, {\it Regularity for a more general class of quasilinear elliptic equations}, J. Differ. Equ. {\bf51} (1984), 126-150. \bibitem{XZ23} D. Xi, D. Yang, G. Zhang and Y. Zhao, {\it The $L_p$ chord Minkowski problem}, Adv. Nonlinear Stud. {\bf23} (2023), Paper No. 20220041. \bibitem{X20} J. Xiao, {\it Prescribing capacitary curvature measures on planar convex domains}, J. Geom. Anal. {\bf30} (2020), 2225-2240. \bibitem{XX19} G. Xiong, J. Xiong and L. Xu, {\it The $L_p$ capacitary Minkowski problem for polytopes}, J. Funct. Anal. {\bf277} (2019), 3131-3155. \bibitem{YZ23} J. Yang, D. Ye and B. Zhu, {\it On the $L_p$ Brunn-Minkowski theory and the $L_p$ Minkowski problem for $C$-coconvex sets}, Int. Math. Res. Not. {\bf2023} (2023), 6252-6290. \bibitem {Z99} G. Zhang, {\it The affine Sobolev inequality}, J. Differ. Geom. {\bf53} (1999), 183-202. \bibitem{Z14} G. Zhu, {\it The logarithmic Minkowski problem for polytopes}, Adv. Math. {\bf262} (2014), 909-931. \bibitem{Z15} G. Zhu, {\it The $L_p$ Minkowski problem for polytopes for $0<p<1$}, J. Funct. Anal. {\bf269} (2015), 1070-1094. \bibitem{ZX20} D. Zou and G. Xiong, {\it The $L_p$ Minkowski problem for the electrostatic $\mathfrak{p}$-capacity}, J. Differ. Geom. {\bf116} (2020), 555-596. \end{thebibliography} \vskip 2mm \noindent Hai Li, {\small\tt [email protected]}\\ {Department of Mathematics and Statistics, Shaanxi Normal University, Xi'an, 710119, China} \vskip 2mm \noindent Longyu Wu, {\small\tt [email protected]}\\ {Department of Mathematics and Statistics, Shaanxi Normal University, Xi'an, 710119, China} \vskip 2mm \noindent Baocheng Zhu, {\small\tt [email protected]}\\ {Department of Mathematics and Statistics, Shaanxi Normal University, Xi'an, 710119, China} \end{document}
2412.07613v1
http://arxiv.org/abs/2412.07613v1
Discontinuous Galerkin methods for the complete stochastic Euler equations
\documentclass[10pt]{amsart} \usepackage{enumitem} \usepackage{amsmath} \usepackage{amssymb} \usepackage{amsthm} \usepackage{comment} \usepackage{hyperref} \usepackage{mathrsfs} \usepackage{breqn} \usepackage{xcolor} \usepackage{graphicx} \usepackage{stmaryrd} \usepackage{geometry} \geometry{margin= 1in} \let\vec\mathbf \usepackage{subcaption} \newtheorem{thm}{Theorem}[section] \newtheorem{ex}[thm]{Example} \newtheorem{prop}[thm]{Proposition} \newtheorem{lem}[thm]{Lemma} \newtheorem{cor}[thm]{Corollary} \newtheorem{asp}[thm]{Assumption} \newtheorem{con}[thm]{Condition} \newtheorem{dfn}[thm]{Definition} \newtheorem{rmk}[thm]{Remark} \newtheorem{cnj}[thm]{Conjecture} \newcommand{\UU}{\mathfrak{U}} \newcommand{\CC}{\mathcal{C}} \newcommand{\BB}{\mathcal{B}} \newcommand{\FF}{\mathfrak{F}} \newcommand{\XX}{\mathcal{X}} \newcommand{\LL}{\mathcal{L}} \newcommand{\PP}{\mathcal{P}} \newcommand{\Ss}{\mathcal{S}} \newcommand{\OO}{\mathcal{O}} \newcommand{\yy}{\mathcal{Y}} \newcommand{\TT}{\mathscr{T}} \newcommand{\MM}{\mathcal{M}} \newcommand{\V}{\mathcal{V}} \newcommand{\rr}{\vec{r}_t} \newcommand{\T}{\mathcal{O}} \newcommand{\R}{\mathbb{R}} \newcommand{\G}{\mathbb{G}} \newcommand{\Z}{\mathbb{Z}} \newcommand{\F}{\mathbb{F}} \newcommand{\E}{\mathbb{E}} \newcommand{\p}{\mathbb{P}} \newcommand{\I}{\mathbb{I}} \newcommand{\N}{\mathbb{N}} \newcommand{\B}{\mathscr{B}} \usepackage{authblk} \numberwithin{equation}{section} \newcommand{\dd}{\mathrm{d}} \newcommand{\dx}{\,\mathrm{d}x} \newcommand{\dt}{\,\mathrm{d}t} \newcommand{\dxt}{\,\mathrm{d}x\,\mathrm{d}t} \newcommand{\ds}{\,\mathrm{d}\sigma} \newcommand{\dxs}{\,\mathrm{d}x\,\mathrm{d} s} \newcommand{\dsx}{\,\mathrm{d}\sigma\,\mathrm{d}x} \newcommand{\dz}{\,\mathrm{d}z} \newcommand{\bxi}{\boldsymbol{\xi}} \newcommand{\bfw}{\mathbf w} \newcommand{\bfr}{\mathbf r} \newcommand{\bfU}{\mathbf U} \newcommand{\bfH}{\mathbf H} \newcommand{\bfG}{\mathbf G} \newcommand{\EE}{\mathscr E} \newcommand{\bfh}{\mathbf h} \newcommand{\ee}{\mathrm e} \newcommand{\bn}{\mathbf n} \newcommand{\VV}{\mathcal V} \newcommand{\bU}{\mathbf{U}} \newcommand{\bu}{\mathbf u} \newcommand{\bh}{\mathbf h} \newcommand{\bm}{\mathbf m} \newcommand{\bV}{\mathbf V} \newcommand{\diff}{\mathrm{d}} \newcommand{\bx}{\mathbf x} \newcommand{\bg}{\mathbf{g}} \newcommand{\diag}{\mathrm{diag}} \newcommand{\IdxM}{\mat{\mathbb I_4}} \newcommand{\bvu}{\underline{\bu}} \newcommand{\bvf}{\underline{\bf}} \newcommand{\bfnum}{\mathbf f^{\mathrm{num}}} \newcommand{\bvecfnum}{\underline{{\bf}_{e_m}^{\mathrm{num}}}} \newcommand{\CV}{\mathcal{V}_{t,x}} \usepackage[foot]{amsaddr} \newcommand{\armean}[1]{\{\{#1\}\}} \newcommand{\logmean}[1]{\{\{#1\}\}_{\log}} \newcommand{\jump}[1]{\llbracket#1\rrbracket} \newcommand{\mat}[1]{\underline{\underline{#1}}} \newcommand{\norm}[1]{\|#1\|} \newcommand{\PO}[1]{{\color{blue}#1}} \title{Discontinuous Galerkin methods for the complete stochastic Euler equations} \author{Dominic Breit} \author{Thamsanqa Castern Moyo} \author{Philipp \"Offner} \address{TU Clausthal, Institute of Mathematics, Erzstra\ss e 1, 38678 Clausthal-Zellerfeld, Germany.} \email{[email protected]} \email{[email protected]} \email{[email protected]} \begin{document} \maketitle \begin{abstract} In recent years, stochastic effects have become increasingly relevant for describing fluid behaviour, particularly in the context of turbulence. The most important model for inviscid fluids in computational fluid dynamics are the Euler equations of gas dynamics which we focus on in this paper. To take stochastic effects into account, we incorporate a stochastic forcing term in the momentum equation of the Euler system. To solve the extended system, we apply an entropy dissipative discontinuous Galerkin spectral element method including the Finite Volume setting, adjust it to the stochastic Euler equations and analyze its convergence properties. Our analysis is grounded in the concept of \textit{dissipative martingale solutions}, as recently introduced by Moyo (J. Diff. Equ. 365, 408-464, 2023). Assuming no vacuum formation and bounded total energy, we proof that our scheme converges in law to a dissipative martingale solution. During the lifespan of a pathwise strong solution, we achieve convergence of at least order 1/2, measured by the expected $L^1$ norm of the relative energy. The results built a counterpart of corresponding results in the deterministic case. In numerical simulations, we show the robustness of our scheme, visualise different stochastic realizations and analyze our theoretical findings. \end{abstract} \section{Introduction} The Euler equations of gas dynamics are one of -if not the- most studied system for describing inviscid, compressible fluid flow. They contain the three conservation laws of mass, momentum, and energy and serve as foundational tools for understanding and predicting fluid behavior in applications like aerodynamics, astrophysics, and meteorology. The system has been thoroughly investigated from an analytical and numerical perspective. Numerically, many different schemes have been developed to solve the Euler equations in a highly efficient way. The aim is always to obtain the most accurate result for real-world problems. From a modeling perspective, the inclusion of uncertainty and/or to describe turbulent effects, researchers have recently started to introduce stochastic forcing into the Euler equations (an other models as well) to describe more realistic scenarios. \\ In this paper, we follow this trend and study the complete Euler equations describing the motion of a general inviscid, compressible and heat-dependent fluid subject to a stochastic perturbation in the momentum equation. In the variables density $\varrho$, momentum $\vec m$ and specific internal energy $e$, the considered system reads then as \begin{align}\label{Euler} \begin{aligned} \dd \varrho + \mathrm{div}_{x}(\vec m)\, \dd t &=0 \quad \text{in}\, Q,\\ \dd \vec m+ \mathrm{div}_x\Big(\frac{\vec m \otimes \vec m}{\varrho}\Big)\, \dd t + \nabla_x p\,\dd t&=\varrho \phi \,\dd W\quad \text{in}\, Q,\\ \dd (\varrho e) + \mathrm{div}_x\left(\varrho e\frac{\vec m}{\varrho}\right)\dd t &=-p\,\mathrm{div}\Big(\frac{\vec m}{\varrho}\Big)\dd t \quad \text{in}\, Q,\end{aligned} \end{align} where $Q=(0,T)\times\mathcal O$ for some $T>0$ and $\mathcal O\subset\R^n$, $n=1,2,3$, bounded. Here, $W$ denotes a (cylindrical) Wiener process on a filtered probability space $(\Omega, \FF, (\FF_t)_{t\geq 0},\p)$ and $\phi$ the noise coefficient (a Hilbert-Schmidt operator\footnote{We refer to Section \ref{sec:prob} for the precise probability framework. }). The key idea behind adding the stochastic perturbation to the deterministic Euler system is to take physical, empirical or numerical uncertainties and thermodynamical fluctuations into account. Additionally, it is also commonly used to model turbulent effects, see \cite{Bi,MiRo} and the references therein. However, it is well-known that the deterministic counterpart of \eqref{Euler} is well-posed only locally in time and solutions can develop shocks \cite{Sm}. Recently, also the precise behaviour of singularities could be described \cite{BSV}. Due to the technique of convex integration, there has been, and continues to be, significant interest in constructing scenarios where the deterministic Euler equations yield multiple entropy solutions from the same initial data. See, for example, \cite{ChDeO,DeSz} for the isentropic Euler system and \cite{FKKM} for the complete Euler equations. This is not only valid in the deterministic case but also if stochastic perturbations are added, cf. \cite{BFHcv} for the isentropic Euler equations and in \cite{ChFeFl} for the complete Euler equations. The only chance to obtain globally defined solutions which obey the laws of thermodynamics in a stable way is the concept of measure-valued solutions. It was already introduced (in the deterministic case for incompressible fluids) in the 80's \cite{DiP,DiMa} but attracted a lot of attention again recently due to the weak-strong uniqueness principle available for dissipative solutions \cite{HoFeBr,Bren,FEJB,KrZa,Neu} and to demonstrate convergence of structure preserving methods \cite{feireisl2020convergence,feireisl2021numerical} within this framework. Weak-strong uniqueness results were recently extended to the stochastic case by the second author in \cite{Mo} where he showed the existence of a dissipative martingale solution to \eqref{Euler}. Such a solution is weak in the probabilistic sense which means the probability space is not a priori given but becomes an integral part of the solution. \\ Meanwhile from the numerical perspective, there has been a long history on solving the deterministic Euler equations. An instrumental method in this context is the approach pioneered by Godunov \cite{godunov1959finite}, which can be conceptualised as a finite volume (FV) technique that tackles exact Riemann problems at each cell interface. Godunov's method has served as the foundation for numerous extensions and has been employed as the groundwork for the development of higher-order methods, exemplified in works such as \cite{harten1983upstream,roe1981approximate,toro1997riemann}. Despite its historical significance, Godunov's method continues to attract attention, with recent studies showcasing its convergence through dissipative weak (DW) solutions of the Euler equations \cite{LuYu}. Additionally, novel error estimates for the multidimensional compressible Euler system have been introduced in \cite{LuSh}. Notably, the results of \cite{LuYu} build upon convergence outcomes for several first-order FV schemes, including the (local) Lax-Friedrichs method, as discussed in \cite{feireisl2020convergence}. For a comprehensive overview of recent convergence results, we direct readers to \cite{feireisl2021numerical} for further details. In parallel with low-order methods, contemporary applications involve high-order methods for solving hyperbolic conservation laws, particularly the Euler equations. Among these methods, the Discontinuous Galerkin (DG) method stands out as one of the most favoured within the hyperbolic community. Originating from the work of Reed and Hill in 1973, initially devised for solving the hyperbolic neutron transport equation in nuclear reactors \cite{ReHi}, DG has evolved over the years. Mathematical foundations were solidified in the 90s \cite{CoKa, HeWa}, and more recent advancements have focused on ensuring structure-preserving (or property-preserving) properties, as pointed out in \cite{KuHa, oeff2023} and references therein. In the context of the Euler equation, recent convergence results via dissipative weak solutions have been demonstrated for DG in \cite{LuOe}. This marked a significant milestone as the first convergence result for a high-order method within the DW framework. Later one these results were extended to other structure-preserving Finite Element (FE) based schemes in \cite{AbLu, KuLu}. All convergence results using the DW framework are made on the assumption that the numerical solution $(\varrho^h,\vec m^h,\mathcal E^h)$, usually formulated in terms of the total energy $\mathcal E^h$ rather than the internal energy, satisfies for some $K>0$ \begin{align}\label{ass:mainA} \inf_{t\in[0,T],x\in\T}\varrho^h(t,x)\geq \frac{1}{K},\quad \sup_{t\in[0,T],x\in\T}\mathcal E^h(t,x)\leq K, \end{align} uniformly in $h$. Here $h$ is the corresponding grid parameter and the superscript denotes that the quantity is the numerical approximation, cf. \cite{AbLu,feireisl2021numerical, KuLu, LuYu}. The physical interpretation of \eqref{ass:mainA} is that the density is not reaching vacuum and we have no blow up in terms of energy (and so also the speed is bounded). In the stochastic case\footnote{Here, we refer that we consider stochastic forcing inside the equations not uncertain initial/boundary data or uncertainty inside the flux where other techniques like Monte-Carlo simulations, general Polynomial chaos or stochastic collocation methods are used and heavily considered \cite{AbMi}.}, the only available result -up to the authors knowledge- is the paper \cite{ChKo}, where a finite volume scheme is considered for the barotropic Euler equations. The authors from \cite{ChKo} work under assumption \eqref{ass:mainA}. Unfortunately, this is not realistic for stochastic PDEs. Due to the Gaussian character inherited from the driving Wiener process solutions are in general not bounded in probability, not even locally in time. This is even the case for linear problems with smooth data which are globally well-posed, see, for instance, \cite{DP}. \\ Due to this we replace \eqref{ass:mainA} by the following assumption which is on the continuous level certainly satisfied by a strong solution (see Definition \ref{def:compstrong}) and thus seems much more plausible. We suppose that there is a stopping time $\mathfrak t$ with $\p(\mathfrak t>0)=1$ and a deterministic constant $K>0$ such that $\p$-a.s. \begin{align}\label{ass:mainB} \inf_{t\in[0,\mathfrak t],x\in\T}\varrho^h(t,x)\geq \frac{1}{K},\quad \sup_{t\in[0,\mathfrak t],x\in\T}\mathcal E^h(t,x)\leq K, \end{align} uniformly in $h$. Heuristically speaking, we assume that \eqref{ass:mainB} is only fulfilled up to some time $\mathfrak t$ which exists with probability $\p(\mathfrak t>0)=1$. Let us explain some heuristics behind this assumption. If the velocity gradient is bounded in space-time the standard maximum principle for the continuity equation \eqref{Euler}$_1$ implies strict positive of the density. However such a bound on the velocity field can never be uniform in probability (as the stochastic forcing is not uniformly bounded). Hence, for any given deterministic time $T>0$, any positiv deterministic bound can be deceeded with a positive probability. So, out of all possible paths the solution can attain some of them will not satisfy \eqref{ass:mainA}. One can consequently either neglect certain paths or adapt the time horizon. For any given path there is a (possibly small) time for which the movement of the noise is small. Thus \eqref{ass:mainB} holds provided the solution is sufficiently smooth in space.\\ Based on this assumption we are able to prove the consistency and convergence\footnote{Convergence means in such context: Convergence in law up to a subsequence and it is based on the stochastic compactness method employing Jakubowski's extension of the Skorokhod representation theorem from \cite{Jaku} (due to the defect measures, which appear in the nonlinear term when passing to the limit in the discrete solution, the classical version for Polish spaces does not apply). A main feature is that we have to include the stopping from \eqref{ass:mainA} in the path space which creates several technical difficulties and will be explained in all details in the following chapters.} between a DG discretization (including the FV case) of \eqref{Euler} and the dissipative martingale solution from \cite{Mo}. The convergence result can be improved in the life-space of a strong solution to \eqref{Euler} which exists locally in time up to a stopping time $\mathfrak s$ (which we expect to be significantly smaller than $\mathfrak t$ in \eqref{ass:mainB}), cf. Definition \ref{def:compstrong}, and is defined on a given stochastic basis. In this case we obtain convergence of order $1/2$, where the error is measured as the expectation of the $L^1$ norm of the relative energy between discrete and continuous variables evaluated at the final time. The result is based on the relative entropy of the discrete solution and provides a stochastic counterpart of the deterministic case. Therefore, the paper is organized as follows: In Section \ref{sec:cr}, we introduce the notation, the mathematical framework and the discontinuous Galerkin (DG) method under consideration. We give a focus on the stochastic part and how we are dealing with it. Then, in Section \ref{se_concept} we repeat the definition of dissipative measure-valued martingale solutions from \cite{Mo}, strong solutions in the probabilistic and PDE sense, and formulate the main results about consistency and convergence results. The chapter should give a detailed introduction inside the topic and provide the main results in this context. In Section \ref{mvsproof}, the consistency results are proven in all technical details where in Section \ref{se_Convergence} the focus is on the convergence result. In addition, in Section \ref{sec_numerical} we show -up to the authors knowledge- for the first time numerical results using the extended Euler system \eqref{Euler}. Furthermore, we verify our theoretical findings and discuss problems which may rise as well inside the numerical experiments. A conclusion and an outlook in Section \ref{se_outlock} finishes this paper. \section{Mathematical framework}\label{Asec} In this section, we introduce the stochastic Euler system \eqref{Euler} and describe different formulations for it. On the continuous level (for sufficiently smooth solutions) they are equivalent, but from a numerical perspective one is more favorable than the other as already can be seen in the deterministic case in \cite{abgrall2018,abgrall2010}. Next, we provide a brief introduction to the probability framework; for further details, see \cite[Chapter 2]{FrBrHo}. Finally, we introduce the discontinuous Galerkin (DG) method under consideration the well-known DG spectral element method using summation-by-parts operators and a flux differencing approach. The method is well investigated and well-used applied for the deterministic Euler equations, cf. \cite{chen2017,He,LuOe,renac_2019} and references therein. We adjust it to the stochastic Euler system. \subsection{Constitutive relations}\label{sec:cr} The fluid model is described by means of three basic state variables: the mass density $\varrho=\varrho(t,x)$, the momentum $\vec m = \vec m(t,x)$, and the (absolute) temperature $\vartheta=\vartheta(t,x)$, where $t$ is the time, $x$ is the space variable (Eulerian coordinate system). The time evolution of the fluid flow is governed by the system of partial differential equations given by \begin{align}\label{EulerB} \begin{aligned} \dd \varrho + \mathrm{div}_{x}\vec m\, \dd t &=0 \quad \text{in}\, Q,\\ \dd \vec m+ \mathrm{div}_x\Big(\frac{\vec m \otimes \vec m}{\varrho}\Big)\, \dd t + \nabla_x p\,\dd t&=\varrho \phi \,\dd W\quad \text{in}\, Q,\\ \dd e + \mathrm{div}_x\left(e\frac{\vec m}{\varrho}\right)\dd t &=-p\,\mathrm{div}\Big(\frac{\vec m}{\varrho}\Big)\dd t \quad \text{in}\, Q ,\end{aligned} \end{align} describing: the balance of mass, momentum and internal energy, respectively. Here, $p(\varrho,\vartheta)$ denotes pressure and $e=e(\varrho,\vartheta)$ is the internal energy. In the theoretical part, we focus on the two-dimensional Euler system \eqref{EulerB}. However, the same arguments apply in both one and three dimensions\footnote{In Section \ref{sec_numerical}, we demonstrate also some results in one dimension for completeness.}. The total energy is the sum of the kinetic and internal energy, respectively, that is \begin{align*} \mathcal E=\mathcal E(\varrho,\vartheta)=\frac{1}{2}\frac{|\vec m|^2}{\varrho}+\varrho e(\varrho,\vartheta). \end{align*} Applying formerly It\^{o}'s formula to the process $t\mapsto \frac{1}{2}\frac{|\vec m|^2}{\varrho}$, the internal energy equation in \eqref{EulerB} can equivalently be written in terms of the total energy as\footnote{In the case of a cylindrical Wiener process, see \eqref{Dwiener} below, one has $\|\phi\|^2_{\ell_2}:=\sum_{k\in\N}|\phi e_k|^2$. Note that this is a function depending on space.} \begin{equation}\label{eq_total} \dd \mathcal E + \mathrm{div}_x\left(\left(\mathcal E+p\right)\frac{\vec m}{\varrho}\right)\dd t =\frac{1}{2}\varrho\|\phi\|_{\ell_2}^2 \,\dd t+ \phi\cdot \vec m\,\dd W. \end{equation} For completeness, the system (\ref{EulerB}) is supplemented by a set of constitutive relations characterising the physical principles of a compressible inviscid fluid. In particular, we assume that the pressure $p(\varrho,\vartheta)$ and the internal energy $e =e(\varrho,\vartheta)$ satisfy the caloric equation of state \begin{equation}\label{caloric} p=(\gamma-1)\varrho e, \end{equation} where $\gamma>1$ is the adiabatic constant. In addition, we suppose that the absolute temperature $\vartheta$ satisfies the Boyle-Mariotte thermal equation of state: \begin{equation}\label{boyle} p =\varrho\vartheta \quad \mathrm{yielding}\quad e= c_v\vartheta, c_v =\frac{1}{\gamma-1}. \end{equation} Finally, we suppose that the pressure $p=p(\varrho, \vartheta)$, the specific internal energy $e =e(\varrho,\vartheta)$, and the specific entropy $ s = s(\varrho,\vartheta)$ are interrelated through Gibbs' relation \begin{equation}\label{gibbs} \vartheta D s (\varrho,\vartheta) = D e(\varrho,\vartheta)+ p(\varrho,\vartheta)D\left(\frac{1}{\varrho}\right). \end{equation} If $p,e,s$ satisfy (\ref{gibbs}), in the context of \textit{smooth} solutions to (\ref{Euler}), the second law of thermodynamics is enforced through the entropy balance equation \begin{equation}\label{entrB} \dd (\varrho s (\varrho,\vartheta))+\mathrm{div}_x( s(\varrho,\vartheta)\vec m)\, \dd t =0, \end{equation} where $s(\varrho,\vartheta)$ denotes the (specific) entropy and is of the form \begin{equation}\label{entropy} s(\varrho,\vartheta)=\log(\vartheta^{c_v})-\log(\varrho). \end{equation} Denoting by $$S=S(\varrho,\vartheta)=\varrho s(\varrho,\vartheta)=\varrho\log(\vartheta^{c_v})-\varrho\log(\varrho)$$ the total entropy, equation \eqref{entropy} can also be written as \begin{equation}\label{EntrB} \dd S (\varrho,\vartheta)+\mathrm{div}_x\Big(S(\varrho,\vartheta)\frac{\vec m}{\varrho}\Big)\, \dd t =0. \end{equation} For weak solutions, the equalities in (\ref{entrB}) and (\ref{EntrB}) no longer hold, the entropy balance is given as an inequality, for more details see \cite{SBGD}. Finally, system \eqref{EulerB} can be reformulated as \begin{align} \dd \varrho + \mathrm{div}_x\vec m\,\dd t &=0\qquad\,\,\text{in $Q$},\label{eq:aEuler}\\ \dd \vec m + \mathrm{div}_x\left(\frac{ \vec m \otimes \vec m}{\varrho}\right)\dd t+\nabla_x p\,\dd t &= \varrho\phi\dd W\,\, \text{in $Q$,}\label{eq:bEuler}\\ \dd S +\mathrm{div}_x\left(\frac{S\vec m}{\varrho} \right)\,\dd t &=0 \quad\qquad\text{in $Q$.}\label{eq:cEuler} \end{align} For physical relevant solutions, the problem is augmented by the total energy balance \begin{equation}\label{eq:tenergy} \dd \int_{\T}\mathcal E\, \dd x= \dd \int_{\T}\left [\frac{1}{2}\varrho|\vec u|^2 +\varrho e\right]\, \dd x= \int_{\T}\varrho\phi \cdot\vec u\,\dd x \dd W+ \frac{1}{2}\int_{\T}\|\phi\|_{\ell_2}^2\dx\,\dd t. \end{equation} Strong solutions of the system (\ref{Euler}) satisfy (\ref{eq:tenergy}), but for weak solutions it has to be added in the definition. In fact, weak solutions to not posses sufficient spatial regularity to derive \eqref{eq:tenergy} form the other equations. It needs to be derived on the approximate level and subsequently kept in the limit procedure. \begin{ex} A particular instance is given by the two-dimensional standard Wiener process \begin{align}\label{eq:14.02} \phi W=\begin{pmatrix} 1\\0\end{pmatrix}\beta_1+\begin{pmatrix} 0\\1\end{pmatrix}\beta_2 \end{align} with two independent standard one-dimensional Wiener processes $ \beta_1$ and $\beta_2$, where $\|\phi\|_{\ell^2}^2=2$. We will apply this noise in our numerical section \ref{sec_numerical} multiplied with some constant to handle the strength and so the influence of it. \end{ex} \subsection{Probability framework} \label{sec:prob} Let $(\Omega, \FF, (\FF_t)_{t\geq 0},\p)$ be a complete stochastic basis with a probability measure $\p$ on $(\Omega,\FF)$ and right-continuous filtration $(\FF_t)_{t\geq 0}$. The filtration is a family of $\sigma$-sub-algebras of $\FF$ indexed by time $t\geq0$, where each $\FF_t$ represents the information available up to time $t$. In other words, it is the collections of events that are \emph{known} at time $t$. Let $\UU$ be a separable Hilbert space with orthonormal basis $(e_k)_{k\in \N}$ (a natural choice would be $\UU=L^2(\mathcal O)$). We denote by $ L_2(\UU,L^2(\T))$ the set of Hilbert-Schmidt operators from $\UU$ to $L^2(\T)$, i.e., the set of bounded linear mappings $\Phi:\UU\rightarrow L^2(\mathcal O)$ satisfying \begin{equation}\label{eq:HSa} \sum_{k\geq 1}\|\phi(e_k)\|_{L^{2}(\T)}^2<\infty. \end{equation} The stochastic process $W$ appearing in \eqref{EulerB} is a cylindrical Wiener process $W =(W_t)_{t \geq 0}$ in $\UU$, meaning it is of the form \begin{equation} W(s) = \sum_{k\in \N}e_k \beta_k(s), \label{Dwiener} \end{equation} where $(\beta_k)_{k \in \N}$ is a sequence of independent real-valued one-dimensional Wiener processes relative to $(\FF_t)_{t\geq0}$. To identify the precise definition of the diffusion coefficient, set $\UU =\ell^2$ and consider $\varrho \in L^{1}(\T), \varrho > 0$. For a mapping $\phi \in L_2(\UU,L^2(\T))$, we set $ \phi_k=\phi(e_k)\in L^2(\T) $, $k\in\N,$ and suppose that \begin{equation}\label{eq:HS} \sum_{k\geq 1}\|\phi_k\|_{L^{\infty}(\T)}^2= \sum_{k\geq 1}\|\phi(e_k)\|_{L^{\infty}(\T)}^2<\infty. \end{equation} Consequently, since $\phi_k$ is bounded we deduce \begin{equation}\label{fyB} \|\sqrt{\varrho}\phi_k\|_{L_2(\UU, L^2(\T))}^2\lesssim c(\phi)( \|\varrho\|_{L^{1}(\T)}). \end{equation} The stochastic integral \[ \int_{0}^{\tau} \varrho \phi \,\dd W = \sum_{k\geq1}\int_{0}^{\tau}\varrho\phi(e_k)\, \dd \beta_k, \] takes values in the Banach space $C([0,T];W^{-\mathfrak k,2}(\T))$ in the sense that \begin{equation} \int_{\T}\left(\int_{0}^{\tau}\varrho\phi \,\dd W\cdot \varphi \right)\,\dd x =\sum_{k\geq1}\int_{0}^{\tau}\left(\int_{\T}\varrho\phi(e_k)\cdot \varphi \,\dd x\right)\,\dd \beta_k, \quad \varphi \in W^{\mathfrak k,2}(\T), \mathfrak k>\frac{n}{2}. \end{equation} Finally, we define the auxiliary space $\UU_0$ with $\UU \subset \UU_0$ as \begin{eqnarray} \UU_0 :&=&\Bigg\{ e = \sum_{k}\alpha_k e_k:\sum_{k}\frac{\alpha_{k}^{2}}{k^2} < \infty \Bigg\}, \nonumber\\ \| e\|_{\UU_0}^{2}:& =& \sum_{k}^{\infty}\frac{\alpha_{k}^{2}}{k^2}, \ e=\sum_{k}\alpha_ke_k, \label{auxiliary} \end{eqnarray} so that the embedding $\UU \hookrightarrow \UU_0$ is Hilbert-Schmidt, and the trajectories of $W$ belong $\p$-a.s. to the class $C([0,T];\UU_0)$ (see \cite{Prato}). \subsection{Discontinuous Galerkin schemes}\label{sec:DG} As mentioned in Subsection \ref{sec:cr}, we focus on the two-dimensional setting. The spatial domain $\T \subset \R^2$ is then discretized with a mesh of tensor-product elements, e.g., a regular quadrilateral grid, denoted by $\TT_h$. We denote the generic cell $K$ and the uniform mesh size with $h$. They are given by $$\mathcal{K}:= [x_{i-1/2,j}, x_{i+1/2,j}] \times [y_{i,j-1/2}, y_{i,j+1/2}]$$ with $h:= x_{i+1/2,j}-x_{i-1/2,j}=y_{i,j+1/2}-y_{i,j-1/2}$, for simplicity. Extensions to (unstructured) rectangular meshes with cell sizes $h_x\neq h_y$ are straightforward. With $\partial \mathcal{K}$ we denote the boundary of an element $\mathcal{K}$ and by $\EE$ the set of all interfaces of all cells $\mathcal{K} \in \TT_h$ where $\ee$ is one interface of $\partial \mathcal{K}$. Between two elements $\mathcal{K}^-$ and $\mathcal{K}^+$ we have a normal vector $\bn$. It is given either by $\bn=(n_x, 0)$ or by $\bn=(0,n_y)$ depending on the interface. Let $\mathcal{Q}^{\mathfrak p}([-1,1]^2)$ be the space of all multivariate polynomials of degree at most $\mathfrak p\in\N_0$ in each variable.On each element $\mathcal{K} \in \TT_h$, we have a linear map $T_{\mathcal{K}}:[-1,1]^2\to \mathcal{K}$ and $\mathcal{Q}^{\mathfrak p}(\mathcal{K})$ is spanned by functions $\psi \circ T_{\mathcal{K}}^{-1}$ with $\psi\in \mathcal{Q}^{\mathfrak p}([-1,1]^2)$. The DG solution space $\VV^h$ is given by \begin{equation}\label{eq:spaceA} \VV^h = \left\{ v^h\in L^1(\T) \Big| v^h|_{\mathcal{K}} \in \mathcal{Q}^{\mathfrak p}(\mathcal{K}) \text{ for all } \mathcal{K}\in \TT_h \right\}. \end{equation} Approximate solutions to the Euler equations \eqref{Euler} live in the vector version of this space, that is $\tilde{\VV}^h= [\VV^h]^4$. To describe an element of the finite dimensional space $\VV^h$, we apply a nodal Gauss-Lobatto basis. We denote by $\xi_i$ the Gauss-Lobatto points in the interval $[-1,1]$. Further, $L_i$ is the Lagrange polynomial which fulfils $L_i(\xi_j)=\delta_{i,j}$. Here, $\delta_{i,j}$ is the Kronecker delta. Lagrange polynomials form a basis for $\mathcal{Q}^{\mathfrak p}([-1,1])$ in one dimension. We obtain a basis for $\mathcal{Q}^{\mathfrak p}([-1,1]^2)$ via the tensor product of the one-dimensional basis, i.e., $L_i(x)L_j(y)$. In each element, each component of the conservative variable vector of the Euler equations is approximated by a polynomial in the reference space \eqref{eq:spaceA}. The nodal values (interpolation points) are our degrees of freedom (DOFs) where we have to calculate the time-dependent nodal coefficients for all components (density, momentum, energy) in the following. To make this point clear, our numerical solution is given by $\bU^h=(\varrho^h, \bm^h,\mathcal E^h)^\top \in \tilde{\VV}^h$ and each component is represented by a polynomial, e.g. in the reference element the approximated density is given by \begin{equation}\label{eq:approx} \varrho^h(x,y,t) = \sum_{i,j=0}^{\mathfrak p} \hat{\varrho}^h(\xi_i, \nu_j, t) L_i(x) L_j(y)= \sum_{i,j=0}^{\mathfrak p} \hat{\varrho}_{i,j}(t) L_i(x) L_j(y) \end{equation} where $ \xi$ and $\nu$ are our Gauss-Lobatto nodes in $x-$ and $y-$direction and $\hat{\varrho}_{i,j}(t)$ is the time-dependent coefficient of our polynomial presentation, i.e. $\hat{\varrho}_{i,j}(t)= \varrho^h(\xi_i, \nu_j,t)$. Analogous notation holds for the other components of $\bU^h$. \\ Next, we explain our DG discretization for the stochastic Euler system \eqref{EulerB}. To avoid the non-conservative products inside the flux formulation which can yield to problems and needs special treatment \cite{abgrall2018,abgrall2010, renac_2019}, we use equation \eqref{eq_total} instead of the internal energy equation in \eqref{EulerB}. Therefore, the third conservative variable is the total energy $\mathcal E$ and we consider the system \begin{align}\label{EulerC} \begin{aligned} \dd \varrho + \mathrm{div}_{x}(\vec m)\, \dd t &=0 \quad \text{in}\, Q,\\ \dd \vec m+ \mathrm{div}_x\Big(\frac{\vec m \otimes \vec m}{\varrho}\Big)\, \dd t + (\gamma-1)\nabla_x \bigg(\mathcal E-\frac{1}{2}\frac{|\vec m|^2}{\varrho}\bigg)\,\dd t&=\varrho \phi \,\dd W\quad \text{in}\, Q,\\ \dd \mathcal E + \mathrm{div}_x\left(\left(\mathcal E+(\gamma-1)\Big(\mathcal E-\frac{1}{2}\frac{|\vec m|^2}{\varrho}\Big)\right)\frac{\vec m}{\varrho}\right)\dd t &=\frac{1}{2}\varrho\|\phi\|_{\ell_2}^2 \,\dd t+ \phi\cdot \vec m\,\dd W\quad \text{in}\, Q.\end{aligned} \end{align} Or in shorter terms \begin{align}\label{EulerC_short} \dd\bU+\mathrm{div}(\mathbf{f}(\bU))\dt=\bh(\bU)\dt+\bg(\bU)\,\dd W, \end{align} where \begin{align*} \mathbf{f}(\bU)&=\mathbf{f}\begin{pmatrix}\varrho\\\vec m\\\mathcal E \end{pmatrix}=\begin{pmatrix}\vec m\\\frac{\vec m \otimes \vec m}{\varrho}+ (\gamma-1)\bigg(\mathcal E-\frac{1}{2}\frac{|\vec m|^2}{\varrho}\bigg)\mathbb I_{2\times 2}\\\left(\mathcal E+(\gamma-1)\Big(\mathcal E-\frac{1}{2}\frac{|\vec m|^2}{\varrho}\Big)\right)\frac{\vec m}{\varrho}\end{pmatrix},\\ \mathbf{h}(\bU)&=\mathbf{h}\begin{pmatrix}\varrho\\\vec m\\\mathcal E \end{pmatrix}=\varrho\begin{pmatrix}0\\0\\\tfrac{1}{2}\|\phi\|_{\ell_2}^2 \end{pmatrix},\quad\mathbf{g}(\bU)=\mathbf{g}\begin{pmatrix}\varrho\\\vec m\\\mathcal E \end{pmatrix}=\begin{pmatrix}0\\\varrho \phi \\\phi\cdot \vec m\end{pmatrix}. \end{align*} In the case of the two-dimensional noise from \eqref{eq:14.02} we have \begin{align}\label{eq:14.02b} \mathbf{h}(\bU)=&\varrho\begin{pmatrix}0\\0\\ 0 \\1\end{pmatrix},\quad\mathbf{g}(\bU)\dd W=\begin{pmatrix}0\\\varrho\, \dd\beta_1\\\varrho\,\dd \beta_2 \\m_1\,\dd\beta_1+m_2\,\dd\beta_2\end{pmatrix}. \end{align} If the noise is truly infinite dimensional, it has to be approximated for practical purposes as outlined in Remark \ref{rmk:noise} below. We start by defining the DG discretization for the flux term $\mathrm{div}(\mathbf{f}(\bU))$ in the Euler equation \eqref{EulerC}. Let $(\Omega, \FF, (\FF_t)_{t\geq 0},\p)$ be a complete stochastic basis with a probability measure $\p$ on $(\Omega,\FF)$ and right-continuous filtration $(\FF_t)_{t\geq 0}$, $W$ an $(\mathfrak F_t)$-adapted (cylindrical) Wiener process and $\phi\in L_2(\mathfrak U;L^2(\T))$. Multiplying by a test function $\bV^h \in \tilde{\VV}^h$ and integrate over the domain $\T$ it reads as \begin{align}\label{eq:DG_strong_analytical} \begin{aligned} \dd\int_{\T} \bU^h \cdot \bV^h \diff \bx& +\sum_{K\in \TT_h} \int_K (\mathrm{div}\,\mathbf{f} (\bU^h) )\cdot \bV^h \diff \bx\dt \\=&- \sum_{\partial K^- \in \TT_h} \int_{\partial K^-} (\bfnum(\bU^{h,-}, \bU^{h,+},\bn^-) - \mathbf{f}(\bU^h) \cdot \bn^-) \cdot \bV^{h,-}\,\dd\mathcal H^1\dt\\ &+\int_{\T}\bh(\bU^h) \cdot \bV^h \diff \bx\,\dd t+\int_{\T}\bg(\bU^h) \cdot \bV^h \diff \bx\,\dd W, \end{aligned} \end{align} where $\mathbf{f}(\bU^h)=(\mathbf{f}_1,\mathbf{f}_2)$ and for the numerical flux $\bfnum$ we choose the local Lax-Friedrichs flux given by \begin{equation}\label{eq:LLF} f^{\mathrm num}(\bU^{h,-}, \bU^{h,+}, \bn^-):= \frac{1}{2} \left(\mathbf{f}(\bU^{h,-}) + \mathbf{f}(\bU^{h,+}) \right)\cdot \bn^- - \frac{\lambda}{2} \left( \mathbf{f}(\bU^{h,+})-\mathbf{f}(\bU^{h,-}) \right), \end{equation} where $\lambda \geq \lambda_{max}$ is an upper bound for the maximal wave speed. For the Euler equations \eqref{EulerC}, it holds that $\lambda_{max}=\max\{ |\bu^h_{K^-}|+c_{K^-},|\bu^h_{K^+}|+c_{K^+} \}$ with $c=\sqrt{\gamma \frac{p^h}{\varrho^h} }$.\\ Note that an inner product denoted by $\cdot$ means that \eqref{eq:DG_strong_analytical} has to be solved for each component of the Euler equation separately. Later all the calculations in our high-order DG method are performed in a reference element $I=[-1,1]^2$. To evaluate the integrals, we proceed by choosing the Gauss-Lobatto nodes as collocated quadrature points. Finally, by this selection of a tensor-product basis and Gauss-Lobatto quadrature, the DG operators have a Kronecker-product structure as well. We denote by $\mat{M}_1$ on $[-1,1]$ the one-dimensional mass matrix. It has diagonal form with quadrature weight on the diagonal. Further, we obtain the one-dimensional differentiation matrix $\mat{D}_{1}$ by evaluating the derivatives of the basis functions at the nodal points. We use the index here to clarify that we have the one-dimensional setting and working on the reference element $I$. To clarify the setting, let $\xi_j$ be the Gauss-Lobatto quadrature points $-1=\xi_0<\xi_1<\cdots<\xi_{\mathfrak p}=1$ in $[-1,1]$ with corresponding quadrature weights $\{\omega_j\}_{j=0}^{\mathfrak p}$. The nodal Lagrangian basis is given by $L_j(\xi_l)=\delta_{jl}$ and we can define the discrete inner $\langle u,v\rangle_{\omega}:=\sum_{j=0}^{\mathfrak p} \omega_j u(\xi_j) v(\xi_j)$ in one space-dimension. Then, the above described operators are given by \begin{itemize} \item Difference matrix $\mat{D}_1$ with $\mat{D}_{1,jl} =L_l'(\xi_j)$ with $L_l'$ be the first derivative of the $l-$Lagrange polynomial, \item Mass matrix $\mat{M}_{1,jl}=\langle L_j,L_l\rangle_{\omega} =\omega_j \delta_{jl}$, so that $\mat{M}_1 = \diag \{\omega_0, \dots, \omega_{\mathfrak p}\}$. \end{itemize} Later, we need also the operators \begin{itemize} \item Stiffness matrix $\mat{Q}_{1,jl}=\langle L_j',L_l\rangle_{\omega} =\langle L_j,L_l'\rangle_{\omega}$ \item Interface matrix $\mat{B}_1=\diag(-1,0,\cdots,0,1)$ \end{itemize} Up to this point, these operators would work separately on every component of the conservative variable vector $\bU^h$, i.e. on the density $\varrho^h$, momentum $\bm^h$, and energy $\mathcal{E}^h$. To extend these operators to the Euler system, a simple Kronecker product can be used. It is \begin{equation*} \mat{\mathbf{M}}_1=\mat{M}_1 \otimes \IdxM \qquad \mat{\mathbf{D}}_1=\mat{D}_1 \otimes \IdxM, \end{equation*} where $ \IdxM$ is the $4\times4$-identy matrix.\\ Similar in two-dimension, we obtain the local mass matrix and differentiation matrix in the standard element $I$ through Kronecker products: $\mat{\mathbf{M}}_I= \mat{\mathbf{M}}_1\otimes \mat{\mathbf{M}}_1, \mat{\mathbf{D}}_{1,I}= \IdxM\otimes \mat{\mathbf{D}}_1, \mat{\mathbf{D}}_{2,I}= \mat{\mathbf{D}}_1 \otimes \IdxM.$ To solve \eqref{eq:DG_strong_analytical}, we have to evaluate the cell interface integrals. Due to the tensor structure ansatz, we evaluate at each interface the one-dimensional Gauss-Lobatto quadrature rule. Due to the Kronecker ansatz, we obtain the interface operators $ \mat{\mathbf{B}}_{\ee_1}= \mat{\mathbf{M}}_1 \otimes \mat{\mathbf{B}}_1 \text{ and } \mat{\mathbf{B}}_{\ee_2}= \mat{\mathbf{B}}_1 \otimes \mat{\mathbf{M}}_1 $ depending on the considered cell interfaces. The defined operators fulfil the summation-by-parts property meaning that they mimic discretely integration-by-parts. \\ In \eqref{eq:DG_strong_analytical}, we have to calculate the time-dependent coefficients of our polynomial representation \\ $\bU^h= (\varrho^h, \bm^h, \mathcal{E}^h) \in\tilde{\VV}^h.$ We have to distinguish in numbering between the interpolation (quadrature) points in $x$- and $y$- direction in \eqref{eq:approx}. For simplicity, we are renumbering the points. We have $n_{\mathfrak p}=({\mathfrak p}+1)^2$ quadrature/interpolation points\footnote{Alternatively, a multi-index can be used for the notation.} denoted by $\boldsymbol{\xi}_i$ in each element and $n_{b}=4{\mathfrak p}$ at the interfaces. On each face we have ${\mathfrak p}+1$ Gauss-Lobatto quadrature nodes but on the corners they intersect. The basis is given by $\mathbf{L}_i$ with $i \in \{1,n_{\mathfrak p}\}$. We denote by $\bvu$ the vector of coefficients (i.e., the nodal values) of $\bU^h$ on $K$: \begin{equation}\label{eq:coefficients} \bvu= \left(\varrho^h(\boldsymbol{\xi}_1), \bm^h(\boldsymbol{\xi}_1), \mathcal{E}^h(\boldsymbol{\xi}_1 ); \varrho^h(\boldsymbol{\xi}_2), \bm^h(\boldsymbol{\xi}_2), \mathcal{E}^h(\boldsymbol{\xi}_2 ); \dots ;\varrho^h(\boldsymbol{\xi}_{n_{\mathfrak p}}), \bm^h(\boldsymbol{\xi}_{n_{\mathfrak p}}), \mathcal{E}^h(\boldsymbol{\xi}_{n_{\mathfrak p}}) \right)^T. \end{equation} Let $ \underline{\mathbf{f}}_m$ ($m=1,2$) denote the vector of values of $\mathbf{f}_m(\bU^h)$ evaluated at the nodal points. For each cell interface $\ee \in \partial K \subset \EE$, we have to evaluate $ \mathbf{f}(\bU^h)$ at the one-dimensional Gauss-Lobatto nodes on the cell interface $\ee$ (where the trace of $\bU^h$ is taken from inside $K$ dotted with the scaled normal vector $\bn$ facing outwards from $\ee$). We denote this by $\mat{\mathbf{R}}_{\ee_m}\underline{\mathbf{f}}_{\ee_m}$. Likewise $ \underline{\mathbf{f}}^{\mathrm{num}}_{\ee_m}$ denotes the nodal values of $\bfnum(\bU^{h,-}, \bU^{h,+},\bn^-)$. With these operators, we can finally re-write the DG semidiscretization \eqref{eq:DG_strong_analytical} on the reference element as follows \begin{align}\label{eq:DG_standard} \begin{aligned} \dd \bvu &+ \big(\mat{\mathbf{D}}_{1,I} \underline{\mathbf{f}}_1 +\mat{\mathbf{D}}_{2,I} \underline{\mathbf{f}}_2\big)\dt\\&\qquad\qquad= \mat{\mathbf{M}}_I^{-1} \sum_{j \in \partial I} \mat{\mathbf{B}}_{j} \left(\mat{\mathbf{R}}_{j}\underline{\mathbf{f}}_{j}-\underline{\mathbf{f}}^{\mathrm{num}} \right)\dt+\underline\bh\,\dd t+\underline\bg\,\dd W, \end{aligned} \end{align} where $\partial I$ denotes the cell interfaces of the reference element. Equation \eqref{eq:DG_standard} describes the classical discontinuous Galerkin spectral element method (DGSEM) in two-space dimension. The method is by construction not entropy conservative/ dissipative for the deterministic Euler equations \cite{zbMATH07517718,gassner2016}. To obtain an high-order entropy dissipative DG method for the Euler equation, we apply the flux differencing approach \cite{fischer_2013,gassner2016}. To this end we replace the volume flux $\sum_{m=1}^2 \mat{\mathbf{D}}_{m,I} \bvf_m $ in equation \eqref{eq:DG_standard} above using consistent, symmetric two-point numerical fluxes. The resulting DG scheme in the reference element reads than \begin{align}\label{eq:DG_standard_2} \begin{aligned} \dd \bvu &+ 2 \left( \mat{\mathbf{D}}_{1,I} \underline{\mathbf{f}}^{\mathrm{num}}_{1,Vol}(\bvu, \bvu)+\mat{\mathbf{D}}_{2,I} \underline{\mathbf{f}}^{\mathrm{num}}_{2,Vol}(\bvu, \bvu) \right)\dt\\&\qquad\qquad= \mat{\mathbf{M}}_I^{-1} \sum_{j \in \partial I} \mat{\mathbf{B}}_{j} \left(\mat{\mathbf{R}}_{j}\bvf_{j}-\underline{\mathbf{f}}^{\mathrm{num}}_j \right)\dt+\underline\bh\,\dd t+\underline\bg\,\dd W, \end{aligned} \end{align} where $\underline{\mathbf{f}}^{\mathrm{num}}_{Vol}$ denotes the numerical volume flux working on each degree of freedom and $\underline{\mathbf{f}}^{\mathrm{num}}_j $ is the classical numerical flux at the interface. For the numerical volume flux $\mathbf{f}^{\operatorname{num}}_{m,Vol}$ with $m=1,2$ , we select the consistent, symmetric and entropy conservative two-point flux from \cite{ranocha2018}. It is defined for each component separately: \begin{equation}\label{eq:Ranocha_flux} \begin{aligned} f^{\mathrm num}_{\varrho,1} &=\{\{\varrho\}\}_{\log}\armean{u_1}, \quad f^{\mathrm num}_{\varrho u_1,1} =\armean{ u_1} f^{\mathrm num}_{\varrho,1} +\armean{ p}, \quad f^{\mathrm num}_{\varrho u_2,1}=\armean{ u_2} f^{\mathrm num}_{\varrho} \\ f^{\mathrm num, x}_{E,1}&= \left( \{\{\varrho\}\}_{\log} \left( \armean{ u_1}^2+\armean{ u_2}^2 - \frac{\armean{ u_1+u_2}^2}{2} \right) -\frac{1}{\gamma-1} \frac{\logmean{ \varrho} }{\logmean{ \varrho/p} } + \armean{ p} \right) \armean{u_1} \\ &-\frac{\jump{p} \jump{\bu}}{4}, \end{aligned} \end{equation} with $\mathbf{f}^{\operatorname{num}}_2$ defined analogously. Here, we have used the abbreviations $\armean{\varrho}=\frac{\varrho^{+}+\varrho^-}{2} $ and $\logmean{\varrho}=\frac{\varrho^{+}-\varrho^-}{\log \varrho^+-\log \varrho^-}$. \begin{rmk}[Interpretation and solvability] \begin{itemize} \item Discretization \eqref{eq:DG_standard_2} is classical entropy stable DGSEM method for the Euler equation equipped with two additional forcing terms due to the noise. The first term $\bh$ has no stochastic component and can be interpreted as a simple forcing of the total energy whereas the second part $\bg$ acts on the momentum equation and the total energy and takes the stochasticity into account. \item The well-posedness assumption regarding \eqref{eq:DG_standard_2} can be substituted with the condition that the coefficients of the nonlinear SDE system are Lipschitz continuous, as noted in \cite{KuLu} for the deterministic parts. This substitution is directly deducible from the definition of the Euler fluxes and the assumptions concerning the positive values of pressure and density. The noise coefficients are linear and thus clearly Lipschitz as well. Hence standard results yield the existence of a solution defined on a given complete stochastic basis $(\Omega, \FF, (\FF_t)_{t\geq 0},\p)$ with respect to the $(\FF_t)$-Wiener process $W$ and given initial data (which we assume for simplicity to be deterministic). \end{itemize} \end{rmk} \section{Concept of solutions and main results}\label{se_concept} \subsection{Measure-valued solutions}\label{MVS} We are now ready to introduce the concept of measure-valued martingale solutions to the \textit{complete} stochastic Euler system written in entropy conservative variables (\ref{eq:aEuler})--(\ref{eq:cEuler}). From here onward, we denote by $\mathcal{M}^+$ the space of non-negative Radon measures, and we denote by $A$ the space of ``dummy variables": \begin{equation}\label{eq:space} A = \bigg\{[\varrho', \vec m',{S'}]\bigg|\varrho'\geq 0, \vec m' \in \R^3,S'\in \R\bigg\} \end{equation} We denote by $\mathcal{P}(A)$ the space of probability measures on $A$. We follow \cite{Mo} and define: \begin{dfn}[Dissipative measure-valued martingale solution]\label{E:dfn}Let $\varrho_0\in L^{\gamma}(\T)$, $\vec m\in L^{\frac{2\gamma}{\gamma +1}}(\T)$, $S_0\in L^{\gamma}(\T)$ and $\phi \in L_2(\UU;L^2(\T))$. Then \[ ((\Omega,\FF, (\FF_t)_{t\geq 0},\p),\varrho,\vec m,S, \mathcal{R}_{\text{conv}},\mathcal{R}_{\text{press}},\mathcal{V}_{t,x},\mathfrak t, W) \] is called a dissipative measure-valued martingale solution to (\ref{eq:aEuler})--(\ref{eq:cEuler}) with initial law $\Lambda$, provided\footnote{Some of our variables are not stochastic processes in the classical sense and we interpret their adaptedness in the sense of random distributions as introduced in \cite{FrBrHo} (Chap. 2.2).}: \begin{enumerate} \item [(a)] $(\Omega,\FF, (\FF_t)_{t\geq 0},\p)$ is a stochastic basis with complete right-continuous filtration; \item[(b)] $\mathfrak t$ is an $(\FF_t)_{t\geq 0}$-stopping time with $\p(\mathfrak t>0)=1$; \item[(c)]$W$ is a $(\FF_t)_{t\geq 0}$-cylindrical Wiener process; \item[(d)] The density $\varrho$ is $(\FF_t)_{t\geq 0}$-adapted and satisfies $\p$-a.s. \[ \varrho(\cdot\wedge\mathfrak t) \in C_{\text{loc}}([0,\infty), W^{-4,2}(\T))\cap L_{\text{loc}}^{\infty}(0,\infty;L^{\gamma}(\T)); \] \item[(e)] The momentum $\vec m$ is $(\FF_t)_{t\geq 0}$-adapted and satisfies $\p$-a.s. \[ \vec m(\cdot\wedge\mathfrak t) \in C_{\text{loc}}([0,\infty), W^{-4,2}(\T))\cap L_{\text{loc}}^{\infty}(0,\infty;L^{\frac{2\gamma}{\gamma +1}}(\T)); \] \item[(f)] The total entropy $S$ is $(\FF_t)_{t\geq 0}$-adapted and satisfies $\p$-a.s. \[ S(\cdot\wedge\mathfrak t) \in L^\infty([0,\infty),L^{\gamma}(\T))\cap BV_{w,\text{loc}}(0,\infty;W^{-l,2}(\T)), l>\frac{n+2}{2}; \] \item[(g)] The parametrised measures $(\mathcal{R}_{\text{conv}},\mathcal{R}_{\text{press}},\mathcal{V})$ are $(\FF_t)_{t\geq 0}$-progressively measurable and satisfy $\p$-a.s. \begin{eqnarray}\label{eq:para} t\mapsto \mathcal{R}_{\mathrm{conv}}(t\wedge\mathfrak t) &\in &L_{\text{weak-(*)}}^{\infty}(0,\infty;\mathcal{M}^+(\T, \mathbb{R}^{3\times 3}));\\ t\mapsto \mathcal{R}_{\mathrm{press}}(t\wedge\mathfrak t) &\in &L_{\text{weak-(*)}}^{\infty}(0,\infty;\mathcal{M}^+(\T, \mathbb{R}));\\ (t,x)\mapsto \mathcal{V}_{t\wedge\mathfrak t,x} &\in &L_{\text{weak-(*)}}^{\infty}(Q;\mathcal{P}(A)); \end{eqnarray} \item[(h)] We have $\varrho(0,\cdot)=\varrho_0$, $\vec m(0,\cdot)=\vec m_0$ and $\liminf_{\tau\rightarrow0+}S(\tau,\cdot)\geq S(\cdot,0)$ $\mathbb P$-a.s.; \item[(i)]For all $\varphi \in C^{\infty}(\T)$ and all $\tau > 0$ there holds $\p$-a.s. \begin{equation}\label{eq:cont} \left[\int_{\T}\varrho\varphi\, \dd x\right]_{t=0}^{t=\tau\wedge\mathfrak t} = \int_{0}^{\tau\wedge\mathfrak t}\int_{\T} \vec m \cdot \nabla \varphi \, \dd x\dd t; \end{equation} \item[(j)]For all $\boldsymbol{\varphi} \in C^{\infty}(\T)$ with $\boldsymbol{\varphi}\cdot\mathbf n=0$ and all $\tau > 0$ there holds $\p$-a.s. \begin{eqnarray}\label{eq:mcxs} \left[\int_{\T}\vec m \cdot \boldsymbol{\varphi}\right]_{t=0}^{t=\tau\wedge\mathfrak t}&=&\int_{0}^{\tau\wedge\mathfrak t}\int_{\T}\left[\frac{\vec m \otimes\vec m}{\varrho}:\nabla\boldsymbol{\varphi}+\varrho\exp{\left(\frac{S}{c_v\varrho}\mathrm{div}\boldsymbol{\varphi}\right)}\right]\, \dd x\, \dd t\\ &&+\int_{0}^{\tau\wedge\mathfrak t}\nabla \varphi:\dd \mathcal{R}_{\text{conv}} \dd t+\int_{0}^{\tau\wedge\mathfrak t}\int_{\T}\mathrm{div} \boldsymbol{\varphi}\,\dd \mathcal{R}_{\text{press}} \dd t\nonumber\\ &&+\int_{0}^{\tau\wedge\mathfrak t}\boldsymbol{\varphi}\cdot \varrho \phi \, \dd x\, \dd W;\nonumber \end{eqnarray} \item[(k)] The total entropy holds in the sense that \begin{equation}\label{eq:entr} \int_{0}^{\tau\wedge\mathfrak t}\int_{\T}\left[\langle\mathcal{V}_{t,x};S'\rangle\partial_t \varphi+\langle\mathcal{V}_{t,x},S'\frac{\vec m'}{\varrho'}\rangle\cdot \varphi\right]\, \dd x\dd t \leq \left[\int_{\T}\langle \mathcal{V}_{t,x};S'\rangle \varphi\, \dd x\right]_{t=0}^{t=\tau\wedge\mathfrak t}, \end{equation} for any $\varphi \in C^1([0,\infty)\times \T), \varphi \geq 0, \p$-a.s. \item[(l)] The total energy satisfies \begin{equation}\label{eq:Ener} \mathcal{E}_{t\wedge\mathfrak t}=\mathcal{E}_{s\wedge\mathfrak t}+\frac{1}{2}\int_{s\wedge\mathfrak t}^{t\wedge\mathfrak t}\|\sqrt{\varrho}\phi\|_{L_2(\UU;L^2(\T))}^2\, \dd \sigma + \int_{s\wedge\mathfrak t}^{t\wedge\mathfrak t}\int_{\T}\vec m \cdot \phi \, \dd x \dd W, \end{equation} $\p$-a.s. for a.a. $0\leq s<t$, where \[ \mathcal{E}= \int_{\T}\left[\frac{1}{2}\frac{|\vec m |^2}{\varrho}+c_v\varrho^{\gamma}\exp{\left(\frac{S}{c_v\varrho}\right)}\right]\, \dd x + \frac{1}{2}\int_{\T}\dd \,\mathrm{tr}\mathcal{R}_{\text{conv}}(t) +c_v\int_{\T}\dd \,\mathrm{tr}\mathcal{R}_{\text{press}}(t) \] for $\tau \geq 0$ and \[ \mathcal E_0= \int_{\T}\left[\frac{1}{2}\frac{|\vec m_0 |^2}{\varrho_0}+c_v\varrho_0^{\gamma}\exp{\left(\frac{S_0}{c_v\varrho_0}\right)}\right]\, \dd x. \] \end{enumerate} \end{dfn} The following result is proved in \cite{Mo} (with $\mathfrak t=\infty$) and ensures that dissipative measure-valued martingale solution exists globally. \begin{thm}\label{ExMainr} Assume (\ref{eq:HS}) holds. Let $\Lambda$ be a Borel probability measure on $L^{\gamma}(\T)\times L^{\gamma}(\T)\times L^{\frac{2\gamma}{\gamma +1}}(\T)$ such that \[ \Lambda\bigg\{(\varrho,S,\vec m)\in L^{\gamma}(\T)\times L^{\gamma}(\T)\times L^{\frac{2\gamma}{\gamma +1}}(\T): 0<\underline{\varrho}<\varrho<\overline{\varrho},0<\underline{\vartheta}<\vartheta<\overline{\vartheta} \bigg\}=1, \] where $\underline{\vartheta}, \overline{\vartheta},\underline{\varrho}, \overline{\varrho} $ are deterministic constants. Moreover, the moment estimate \[ \int_{L^{\gamma}(\T)\times L^{\gamma}(\T)\times L^{\frac{2\gamma}{\gamma +1}}(\T)}\left\|\frac{1}{2}\frac{|\vec m|^2}{\varrho}+c_v\varrho^{\gamma}\exp{\left(\frac{S}{c_v\varrho}\right)}\right\|_{L^1(\T)}^p\,\dd \Lambda < \infty, \] holds for all $p\geq 1$. Then there exists a dissipative measure-valued martingale solution to the \textit{complete} stochastic Euler system (\ref{eq:aEuler})--(\ref{eq:cEuler}) in the sense of Definition \ref{E:dfn} (with $\mathfrak t=\infty$) subject to the initial law $\Lambda$. It satisfies the entropy balance also in the renormalised sense, that is we have \begin{equation}\label{eq:entrren} \int_{0}^{\tau\wedge\mathfrak t}\int_{\T}\left[\langle\mathcal{V}_{t,x};Z(S')\rangle\partial_t \varphi+\langle\mathcal{V}_{t,x},Z(S')\frac{\vec m'}{\varrho'}\rangle\cdot \varphi\right]\, \dd x\dd t \leq \left[\int_{\T}\langle \mathcal{V}_{t,x};Z(S')\rangle \varphi\, \dd x\right]_{t=0}^{t=\tau\wedge\mathfrak t}, \end{equation} for any $\varphi \in C^1([0,\infty)\times \T), \varphi \geq 0, \p$-a.s.,{and any $Z \in BC(\R)$ non-decreasing.} \end{thm} \subsection{Strong solutions} In the following, we will give the definition of a local strong pathwise solution. These solutions are strong in the probabilistic and PDE sense, at least locally in time. In particular, the Euler system (\ref{Euler}) will be satisfied pointwise in space-time on the given stochastic basis associated to the cylindrical Wiener process $W$. \begin{dfn}[Strong Solution]\label{def:compstrong} Let $(\Omega, \FF, (\FF_t)_{t\geq 0}, \p)$ be a complete stochastic basis with a right-continuous filtration, let $W$ be an $(\FF_t)_{t\geq 0}$-cylindrical Wiener process, $\phi\in L_2(\mathfrak U;L^2(\T))$ and $\ell\geq4$. The triplet $[r, \Theta, \vec v]$ and a stopping time $\mathfrak{s}$ is called a (local) strong pathwise solution to the system (\ref{Euler}) provided: \begin{itemize} \item[(a)] the density $r> 0$ $\p$-a.s., $t \mapsto r (t\wedge \mathfrak{s},\cdot)\in W^{\ell,2}(\T)$ is $(\FF_t)_{t\geq 0}$-adapted, \[ \E\left[\sup_{t\in [0,T]}\|r (t\wedge \mathfrak{s},\cdot)\|_{W^{\ell,2}(\T)}^q\right]<\infty \quad\text{for all $1\leq q <\infty$, $T>0$},; \] \item[(b)] the temperature $\Theta > 0$ $\p$-a.s., $t \mapsto \Theta (t\wedge \mathfrak{s},\cdot)\in W^{\ell,2}(\T)$ is $(\FF_t)_{t\geq 0}$-adapted, \[ \E\left[\sup_{t\in [0,T]}\|\Theta (t\wedge \mathfrak{s},\cdot)\|_{W^{\ell,2}(\T)}^q\right]<\infty \quad\text{for all $1\leq q <\infty$, $T>0$}; \] \item[(c)] the velocity $t\mapsto \vec v(t\wedge \mathfrak{s},\cdot) \in W^{\ell,2}(\T)$is $(\FF_t)_{t\geq 0}$-adapted, \[ \E\left[\sup_{t\in [0,T]}\|\vec v (t\wedge \mathfrak{s},\cdot)\|_{W^{\ell,2}(\T)}^q\right]<\infty \quad\text{for all $1\leq q <\infty$, $T>0$}; \] \item[(d)] for all $t\geq0$ there holds $\p$-a.s. \[ r(t\wedge\mathfrak{s}) = \varrho(0)- \int_{0}^{t\wedge \mathfrak{s}}\mathrm{div}_x(r\vec v)\,\dd t, \] \[ (r\vec v)(t\wedge \mathfrak{s})=(r\vec v)(0)-\int_{0}^{t\wedge}\mathrm{div}(r\vec v\otimes \vec v)\, \dd t-\int_{0}^{t\wedge\mathfrak{s}}\nabla_x p(r,\Theta)\,\dd t +\int_{0}^{t\wedge \mathfrak{s}}r\phi\, \dd W, \] \[ (rs(r,\Theta))(t\wedge\mathfrak{s}) = (rs(r,\Theta))(0)- \int_{0}^{t\wedge \mathfrak{s}}\mathrm{div}_x(rs(r,\Theta)\vec v)\,\dd t, \] where $s$ is the total entropy given by (\ref{entropy}). \end{itemize} \end{dfn} Since density and temperature are strictly positive one can rewrite the system from Definition \ref{def:compstrong} (d) equivalently in terms of $\chi:=\log(\varrho)$, the velocity field $\vec v$ and the temperature $\Theta$ obtaining \begin{align}\label{EulerD} \begin{aligned} \dd \Theta + \vec v\cdot\nabla\Theta\dd t +\Theta\,\mathrm{div}(\vec v)\dd t&=0 \quad \text{in}\, Q\\ \dd \chi + \vec v\cdot\nabla\chi+\mathrm{div}_{x}(\vec v)\, \dd t &=0 \quad \text{in}\, Q,\\ \dd \vec v+ (\vec v\cdot\nabla)\vec v\, \dd t + \Theta\nabla_x \chi\,\dd t+\,\nabla\Theta\,\dt&=\phi \,\dd W\quad \text{in}\, Q .\end{aligned} \end{align} This is a symmetric hyperbolic system with additive noise. Given some initial data $(\varrho_0,\bfU_0,\Theta_0)$ we expect the existence of a unique strong pathwise solution provided (setting $\chi_0:=\log(\varrho_0)$) \begin{align*} \chi_0,\bfU_0,\Theta_0\in W^{\ell,2}(\mathcal O),\quad\phi\in L_2(\mathfrak U;W^{\ell,2}(\mathcal O)), \end{align*} together with no flux boundary conditions for the initial data. A corresponding result for the problem on the whole space follows from \cite{Kim}. The following weak (measure-valued)-strong uniqueness principle is proved in \cite{Mo}. \begin{thm}\label{thm_a} The pathwise weak-strong uniqueness holds true for the system (\ref{eq:aEuler})--(\ref{eq:cEuler}) in the following sense. Let \[((\Omega , \FF , (\FF)_{t \geq 0}, \mathbb{P} ),\varrho,\vec m, S, \mathcal{R}_{\mathrm{conv}},\mathcal{R}_{\mathrm{press}},\mathcal{V}_{t,x},W) \] be a dissipative measure-valued martingale solution to (\ref{eq:aEuler})-(\ref{eq:cEuler}) in the sense of Definition (\ref{E:dfn}) (with $\mathfrak t=\infty$) which satisfies additionally the relative entropy balance in the renormalised sense, cf.~\eqref{eq:entrren}. Let the triplet $[r,\Theta,\vec v]$ and a stopping time $\mathfrak{s}$ be a strong solution in the sense of Definition \ref{def:compstrong} of the same problem; defined on the stochastic basis with the same Wiener process and with initial data \begin{equation}\label{Idata} \varrho(0,\cdot) =r(0,\cdot),\quad\vec u (0,\cdot)={\vec v}(0,\cdot),\qquad \vartheta (0,\cdot)=\vec \Theta(0,\cdot)\quad\p\text{-a.s.} \end{equation} Then \[ [{\varrho},{\vartheta},{\vec u}](\cdot \wedge \mathfrak{s}) \equiv [r,{\Theta},{\vec v}] (\cdot \wedge \mathfrak{s}), \] and \[ \mathcal{R}_{\mathrm{conv}}=\mathcal{R}_{\mathrm{press}}=0, \] $\p$-a.s., and for any $(t,x)\in (0,T)\times\T$ \[ \mathcal{V}_{t\wedge \mathfrak{s},x}= \delta_{ s(r,\Theta)}, \] $\p$-a.s. \end{thm} \begin{rmk} If $\mathfrak p=0$ one can argue as in \cite{feireisl2021numerical} to show that the solution obtained by our algorithm (cf. Theorem \ref{thm:main1} below) satisfies the relative entropy balance in the renormalised sense, cf.~\eqref{eq:entrren}, such that Theorem \ref{thm_a} applies. For $\mathfrak p\geq1$ this is currently unclear. Note that this point does not seem to be affected by the noise, this issue appears already in the deterministic case. \end{rmk} \subsection{Consistency and convergence} In this subsection we formulate our main results concerning the DG scheme from \eqref{eq:DG_standard_2} with solution $(\varrho^h,\vec m^h,\mathcal E^h)$ posed on a stochastic basis $(\Omega, \FF, (\FF_t)_{t\geq 0},\p)$ regarding its approximation properties for the Euler equations \eqref{Euler}. We work under the following hypothesis: There is an $(\mathfrak F_t)$-stopping time $\mathfrak t$ with $\p(\mathfrak t>0)=1$ and deterministic constants $K>0$ such that $\p$-a.s. \begin{align}\label{ass:main} \inf_{t\in[0,\mathfrak t],x\in\T}\varrho^h(t,x)\geq \frac{1}{K},\quad \sup_{t\in[0,\mathfrak t],x\in\T}\mathcal E^h(t,x)\leq K, \end{align} uniformly in $h$. This assumption is the natural counterpart of the corresponding hypothesis in the deterministic case made in various papers such as \cite{AbLu,feireisl2021numerical, KuLu, LuYu}. Since the scheme \eqref{eq:DG_standard_2} is formulated in terms of the total energy $\mathcal E^h$, while Definition \ref{E:dfn} is formulated in terms of the total entropy $S$ we need to introduce the approximate total entropy $S^h$ given by \begin{align*} S^h=c_v\varrho^h\log\bigg((\gamma-1)\bigg(\mathcal E^h-\frac{1}{2}\frac{|\vec m^h|^2}{\varrho^h}\bigg)\bigg)-(c_v+1)\varrho^h\log(\varrho^h). \end{align*} Also, the approximate temperature $\vartheta^h$ is given as \begin{align*} \vartheta^h=\frac{\gamma-1}{\varrho^h}\bigg(\mathcal E^h-\frac{1}{2}\frac{|\vec m^h|^2}{\varrho^h}\bigg). \end{align*} Now we formulate our main result concerning the consistency of the scheme \eqref{eq:DG_standard_2}. \begin{thm}\label{thm:main1} Let $(\Omega, \FF, (\FF_t)_{t\geq 0},\p)$ be a complete stochastic basis with a probability measure $\p$ on $(\Omega,\FF)$ and right-continuous filtration $(\FF_t)_{t\geq 0}$ and $W$ an $(\FF_t)$-adapted Wiener process. Suppose that $\phi\in L_2(\mathfrak U;L^2(\T))$ satisfies \eqref{eq:HS}. Let $(\varrho^h,\vec m^h,\mathcal E^h)$ be the solution to \eqref{eq:DG_standard_2}. Suppose there is an $(\mathfrak F_t)$-stopping time $\mathfrak t$ such that \eqref{ass:main} holds. Suppose that the initial data $(\varrho_0,\vec m_0,S_0)$ satisfies \begin{align*} \varrho_0,S_0\in\ L^\gamma(\T),\quad\vec m_0\in L^{\frac{2\gamma}{\gamma+1}}(\T),\quad\inf_{x\in\T}\varrho_0>K,\quad \sup_{x\in\T}\mathcal E_0<K. \end{align*} Then it there is a null-sequence $(h_m)$ such that for all $q<\infty$ \begin{equation} \begin{cases} {\varrho}^{h_m} \to \Tilde{\varrho} \,\, \text{in }\,\, C([0,\tilde{\mathfrak t}];W^{-4,2})\cap C_w([0,\Tilde{\mathfrak t}]; L^{\gamma}(\T)),\\ {S}_{h_m} \to \Tilde{S} \,\, \text{in }\,\, L^q(0,T;W^{-\mathfrak p-4,2}(\T)),\\ {S}_{h_m} \rightharpoonup^\ast \Tilde{S} \,\, \text{in }\,\, L^\infty(0,T; L^{\gamma}(\T)),\\ {\vec{m}}^{h_m} \to \Tilde{\vec m} \, \, \text{in}\, \, C([0,\tilde{\mathfrak t}];W^{-4,2}(\T))\cap C_w ([0,\Tilde{\mathfrak t}];L^{\frac{2\gamma}{\gamma +1}}(\T)),\\ \varrho^{h_m}\vartheta^{h_m} \to \Tilde{\varrho}\Tilde{\vartheta}+\Tilde{\mathcal{R}}_{\text{press}} \,\, \text{in} \,\, L_{w^*}^{\infty}(0,T;\mathcal{M}^{+}(\T,\R)),\\ \frac{{\vec m}^h\otimes {\vec m}^h}{\varrho^h} \to \frac{\Tilde{\vec m}^h\otimes \Tilde{\vec m}^h}{\Tilde\varrho^h}+\Tilde{\mathcal{R}}_{\text{conv}} \,\, \text{in} \,\, L_{w^*}^{\infty}(0,T;\mathcal{M}^{+}(\T,\R^{3\times 3})), \\ \delta_{(\varrho^h,\vec m^h,S^h)}\to \Tilde{\mathcal{V}}_{t,x} \,\, \text{in} \,\, L_{w^*}^{\infty}(Q;\mathcal{P}(A)), \end{cases} \end{equation} in law as $h\rightarrow0$, where, for a filtered probability space $(\Tilde{\Omega},\Tilde\FF, (\Tilde\FF_t)_{t\geq 0},\Tilde\p)$, an $(\Tilde\FF_t)$-adapted Wiener process $\Tilde W$ and an $(\Tilde\FF_t)$-stopping time $\Tilde{\mathfrak t}$, \[ ((\Tilde{\Omega},\Tilde\FF, (\Tilde\FF_t)_{t\geq 0},\Tilde\p),\Tilde\varrho,\Tilde{\vec m},\Tilde S, \Tilde{\mathcal{R}}_{\text{conv}},\Tilde{\mathcal{R}}_{\text{press}},\Tilde{\mathcal{V}}_{t,x},\Tilde{\mathfrak t}, \Tilde{W}) \] is a dissipative measure-valued solution to (\ref{Euler}) in the sense of Definition \ref{E:dfn} with initial law $\delta_{\varrho_0}\otimes\delta_{\vec m_0}\otimes\delta_{S_0}$. It holds $\tilde{\mathfrak t}\sim \mathfrak t$ in law. \end{thm} \begin{rmk} A general framework for the numerical approximation of martingale solutions to nonlinear stochastic PDEs is given in \cite{OPW}. However, it is rather designed for parabolic problems and does not include the possibility for the solution to develop defect measures which is crucial for hyperbolic problems as considered here. \end{rmk} If we have a strong pathwise solution (as described in Definition \ref{def:compstrong}) we can obtain a stronger result. The strong solution exists on a given stochastic basis up to the stopping time $\mathfrak s$. For the strong solution, there is no vaccum and the energy is bounded, i.e., a counterpart of \eqref{ass:main} is satisfied. Hence one may hope that $\mathfrak s$ is significantly smaller than $\mathfrak t$. For $M\in\N$ we introduce the stopping time \[ \mathfrak s_{M}=\inf\bigg \{t\in (0,\mathfrak{s})|\quad\|(r,\Theta,\vec v)(s,\cdot)\|_{W^{\mathfrak p+1,\infty}(\T)}>M\bigg\}, \] where $\mathfrak p$ is the degree of the polynomials in the DG solution space, cf. Section \ref{sec:DG}. Since $[r,\Theta,\vec v]$ is a strong solution, \[ \p\left[\lim_{M\to \infty}\mathfrak s_M=\mathfrak{s}\right]=1. \] Our main result regarding convergence is the following theorem. \begin{thm}\label{thm:main2} Let $(\Omega, \FF, (\FF_t)_{t\geq 0},\p)$ be a complete stochastic basis with a probability measure $\p$ on $(\Omega,\FF)$ and right-continuous filtration $(\FF_t)_{t\geq 0}$, $W$ an $(\FF_t)$-adapted Wiener process and $\phi\in L_2(\mathfrak U;W^{\ell,2}(\mathcal O))$ for some $\ell\geq\max\{4,\mathfrak p+3\}$. Let $\mathbf{U}^h=(\varrho^h,\vec m^h,\mathcal E^h)$ be the solution to \eqref{eq:DG_standard_2}. Suppose there is an $(\mathfrak F_t)$-stopping time $\mathfrak t$ such that \eqref{ass:main} holds. Let the triplet $[r,\Theta,\vec v]$ and a stopping time $\mathfrak{s}$ be a strong solution in the sense of Definition \ref{def:compstrong} with initial data \begin{align*} \varrho_0,\vec m_0,\Theta_0\in W^{\mathfrak p+3,2}(\T),\quad\inf_{x\in\T}\varrho_0>K,\quad \sup_{x\in\T}\mathcal E_0<K. \end{align*} \begin{enumerate} \item[(a)] Let $\mathfrak p=0$. Then it holds for all $t\in[0,T]$ and $M\in\N$ \begin{align}\label{eq:errorA} \E \Big[\Big(\|\varrho^h-r\|^2_{L^2(\T)}+\|\vec u^h-\vec v\|^2_{L^2(\T)}+\|\vartheta^h-\Theta\|^2_{L^2(\T)}\Big)(t\wedge\mathfrak t\wedge\mathfrak s_M)\Big]\leq\,c h^{\frac{1}{2}}. \end{align} \item[(b)] For $\mathfrak p\geq1$, we further assume that there is some deterministic $\mathfrak c>0$ independent of $h$ such that \begin{equation}\label{as_1} |\mathbf{U}^h (\mathbf{x}_j ,t\wedge\mathfrak t) - \mathbf{U}^h (\mathbf{x}_i ,t\wedge \mathfrak t) | \leq \mathfrak c h \qquad \forall x_j, x_i \in \mathcal{K}. \end{equation} Then, it holds for all $t\in[0,T]$ and $M\in\N$ \begin{align}\label{eq:errorB} \E \Big[\Big(\|\varrho^h-r\|^2_{L^2(\T)}+\|\vec u^h-\vec v\|^2_{L^2(\T)}+\|\vartheta^h-\Theta\|^2_{L^2(\T)}\Big)(t\wedge\mathfrak t\wedge\mathfrak s_M)\Big]\leq\,c h. \end{align} \end{enumerate} The constant $c$ above depends on $M$ and $K$ from \eqref{ass:main}. \end{thm} \begin{rmk} There is hardly any literature on the numerical approximation of local strong solutions to SPDEs. In fact, the only result which is comparable to Theorem \ref{thm:main2} we are aware of is \cite{BrDo}. In \cite{BrDo} local strong solutions to the 3D incompressible stochastic Navier--Stokes equations are considered. \end{rmk} \begin{rmk}\label{rmk:noise} In the case of a truly infinite dimensional noise, a practical implementation requires an approximation by a finite sum, i.e., replacing $W$ by $W^N=\sum_{k=1}^Ne_k\beta_k$ for some large $N\in\N$. This leads to the additional error term \begin{align*} \sum_{k=N+1}^\infty\int_0^{\tau\wedge\mathfrak s_m\wedge \mathfrak t}\int_{\T}\varrho^h|\phi e_k|^2\dx\dt\leq c(K)\sum_{k=N+1}^\infty\int_{\T}|\phi e_k|^2\dx \end{align*} in the proof of Theorem \ref{thm:main2}. Its size can be controlled by the choice of $N$. \end{rmk} \section{Consistency}\label{mvsproof} The aim of this section is to prove Theorem \ref{thm:main1}. First of all, we can use the boundedness of $\mathcal E^h$ from \eqref{ass:main}. Arguing as in \cite{HoFeBr} we can deduce the following bounds \begin{align}\label{eq:2512}\begin{aligned} \varrho^h&\in L^{\infty}([0,\mathfrak t];L^{\gamma}(\T)), \\ \vec m^h &\in L^{\infty}([0,\mathfrak t];L^{\frac{2\gamma}{\gamma +1}}(\T)),\\ \frac{\vec m^h}{\sqrt{\varrho^h}}&\in L^{\infty}([0,\mathfrak t];L^{2}(\T)),\\ \frac{\vec m^h \otimes \vec m^h}{\varrho^h}&\in L^{\infty}([0,\mathfrak t];L^{1}(\T)),\\ S^h&\in L^{\infty}([0,\mathfrak t];L^{\gamma}(\T)),\\ \frac{ S^h}{\sqrt{\varrho^h}}&\in L^{\infty}([0,\mathfrak t];L^{2\gamma}(\T)), \end{aligned} \end{align} which hold $\p$-a.s. uniformly in $h$. \subsection{The consistency formulation} Due to our shape regular mesh, cf. \cite{KuLu}, and by following line by line the proof of Theorem 4.2 in \cite{LuOe} we obtain for all $\tau \in (0,T]$: \begin{itemize} \item for all $\varphi \in C^{p+1}(\overline{\T})$: \begin{equation}\label{eq:consistency_rho} \left[ \int_{\T} \varrho^h \varphi \diff \bx \right]_{t=0}^{t=\tau} =\int_0^\tau \int_{\T} \bm^h \cdot \nabla_{\bx} \varphi \diff \bx \diff t +\int_0^\tau e_{\varrho^h} (t,\varphi) \diff t; \end{equation} \item for all $\boldsymbol{\varphi}\in C^{p+1}(\overline{\T};\R^2)$: \begin{equation}\label{eq:consistency_m} \begin{aligned} \left[ \int_{\T} \bm^h \boldsymbol{\varphi} \diff \bx \right]_{t=0}^{t=\tau} &=\int_0^\tau \int_{\T} \Big(\frac{\bm^h\otimes\bm^h}{\varrho^h} : \nabla_{\bx}\boldsymbol{\varphi} +\varrho^h\vartheta^h \mathrm{div}_{\bx} \boldsymbol{\varphi} \Big)\diff \bx \diff t\\ &+\int_0^\tau\int_{\T} \varrho^h\phi\cdot\Pi_h\boldsymbol\varphi\diff \bx \,\dd W + \int_0^\tau e_{\bm^h} (t,\boldsymbol{\varphi}) \diff t, \end{aligned} \end{equation} where $\Pi_h$ is the projection onto the solution space $\mathcal V^h$; \item for all $\psi \in C^{p+1}( \overline{\T}), \; \psi\geq 0$: \begin{equation}\label{eq:consistency_S} \left[ \int_{\T} S^h \psi \diff \bx \right]_{t=0}^{t=\tau} \leq \int_0^\tau \int_{\T} S^h \frac{\bm^h}{\varrho^h} \cdot \nabla_\bx {\psi} \diff \bx \diff t + \int_0^\tau e_{S^h} (t,\psi) \diff t; \end{equation} \item We have the energy balance \begin{align} \label{eq:consistency_E} \int_{\T} \mathcal E^h(\tau)\diff \bx =\int_{\T} \mathcal E^h_0 \diff \bx+\frac{1}{2}\int_0^\tau\|\sqrt{\varrho^h}\phi\|_{L_2}^2 \,\dd t+ \int_0^\tau\int_{\T}\phi\cdot \vec m^h\diff\bx\,\dd W \end{align} \item The error $ \mathbf e^h:=(e_{j\varrho^h},e_{\vec m^h}, e_{S^h})$ tends to zero under mesh refinement: We have $\p$-a.s. \begin{equation}\label{eq:consistency_error} \norm{\mathbf e^h(\varphi,\boldsymbol{\varphi},\psi)}_{L^1(0,\mathfrak t)} \to 0 \text{ if } h\to 0. \end{equation} \end{itemize} Note that the proof of \eqref{eq:consistency_error} in \cite{LuOe} is only based on the bounds from \eqref{ass:main}, which is the reason why it only holds up to the stopping time in our case. We also remark that, in fact, a stronger statement is proved in \cite{LuOe} for the case $\mathfrak p\geq 1$ and under the additional assumption \eqref{as_1}. In our case it can be written as \begin{equation}\label{eq:consistency_errorB} \sup_{t\in[0,\mathfrak t]}|\mathbf e^h(\varphi,\boldsymbol{\varphi},\psi)|\leq\,ch\|(\varphi,\boldsymbol{\varphi},\psi)\|_{W^{\mathfrak p+1,\infty}(\T)}, \end{equation} where $c=c(\mathcal{K})$ with $\mathcal{K}$ from \eqref{ass:main}. If $\mathfrak p=0$ instead, it is proved using the Godunov/(local) Lax-Friedrichs fluxes that \begin{equation}\label{eq:consistency_errorA} \sup_{t\in[0,\mathfrak t]}|\mathbf e^h(\varphi,\boldsymbol{\varphi},\psi)|\leq\,ch^{\frac{1}{2}}\|(\varphi,\boldsymbol{\varphi},\psi)\|_{W^{1,\infty}(\T)}. \end{equation} The latter is based on a weak BV-estimate. \begin{rmk} It should be stressed out that in the case $\mathfrak p=0$ we are in the classical FV context. The $L^1$ error in the relative energy with $1/2$ corresponds to $1/4$ in $L^2$ of the conserved (primitive) quantities. Under the additional assumption that the total variation of the numerical solution is uniformly bounded $1/2$ in the $L^2$-norm is achieved, cf. \cite{LuSh}. The study in \cite{LuSh} employs the Godunov method. Nonetheless, the findings can be applied to the Lax-Friedrichs FV method by following the same analysis. Moreover, it is important to highlight that, in the examination of consistency within the DG framework, the additional assumption \eqref{as_1} was introduced regarding the variation among the numerical solution at different nodal points within a single element and their proportional adjustments concerning the grid parameter $h$. The application of limiters was also disregarded. Including the application of limiters, a result similar to the FV setting can only be expected.\footnote{However, it is still unclear how limiters can be constructed and incorporated for the stochastic setting which will be part of future research.} This also is in line with the analysis of the flux corrected FE method approach using monolithic convex limiting and residual distribution schemes. In these contexts, one can directly follow the analysis presented in \cite{LuSh}, expecting convergence rates of $1/4$ or $1/2$ only if the total variation of the numerical solution remains uniformly bounded. Note that it is currently work in progress to develop new deterministic a priori error bounds for the DG method (as well as RD and MCL) by means of the consistency analysis. \end{rmk} In order to prepare the proof of compactness we need some time-regularity of the variables $\varrho^h,\vec m^h$ and $S^h$. For the stochastic term in the momentum equation we have using continuity of $\Pi_h$ \begin{align*} \E \left[ \left \|\int_{0}^{\cdot} \Pi_h(\varrho^h \phi )\,\dd W\right\|_{C^{\alpha}([0,T];L^2(\T))}^{q}\right] \leq\, c \,\E \left[ \int_{0}^{T}\Big\|\Pi_h(\sqrt{\varrho^h} \phi)\Big\|^{q}_{L_2(\UU,L^2(\T))} \, \dd t \right] \end{align*} for all $\alpha \in (0,1/2-1/q)$ and $q>2$, see [\cite{Brt_1}, Lemma 9.1.3. b)] or [\cite{Hofm}, Lemma 4.6]). Using continuity of $\Pi_h$ we end up with \begin{align*} \E \left[ \left \|\int_{0}^{\cdot} \Pi_h(\varrho^h \phi) \,\dd W\right\|_{C^{\alpha}([0,T];L^2(\T))}^{q}\right] \leq\, c \,\E \left[ \int_{0}^{T}\Big\|\sqrt{\varrho^h} \phi\Big\|^{q}_{L_2(\UU,L^2(\T))} \, \dd t \right] \leq\, c(\overline{\varrho},q,\phi,T). \end{align*} The final estimate follows from $\int_{\T}\varrho^h\dx=\int_{\T}\varrho_0\dx$ and \eqref{eq:HS}. Now combining the deterministic estimates from \eqref{eq:2512} and \eqref{eq:consistency_errorB} with the stochastic estimate above, and using the embeddings $W^{1,2}_t \hookrightarrow C^{\alpha}_{t}$ and $L^2_x \hookrightarrow W^{-3,2}_x$ shows \begin{equation}\label{estimate} \E \left[\big\|\vec{m}^h\big\|_{C^{\alpha}([0,\mathfrak t];W^{-\mathfrak p-3,2}(\T))} \right] \leq c(T) \end{equation} for all $\alpha < 1/2$ as a consequence of \eqref{eq:consistency_m}. Similarly, the continuity equation \eqref{eq:consistency_rho} together with \eqref{eq:2512} and \eqref{eq:consistency_errorB} yields $\partial_t \varrho_{\varepsilon} \in L^{\infty}(0,\mathfrak t;W^{-\mathfrak p-3,2}(\T))$ $\p$-a.s. In particular, we obtain \[ \E\left[\big\| \varrho^h\big\|_{C^{\alpha}([0,\mathfrak t];W^{-\mathfrak p-3,2}(\T))}\right] \leq C. \] Finally, we deduce \begin{align} \label{eq:12.01} \E\left[\| S^h\|_{\mathrm{BV}([0,\mathfrak t];W^{-\mathfrak p-3,2}(\T))}\right] \leq C \end{align} from \eqref{eq:consistency_S}. \subsection{Compactness Argument} We set the spaces (where $q<\infty$ is arbitrary): \begin{eqnarray*} \mathscr{X}_{\Vec{m}}: = C([0,T];W^{-\mathfrak p-4,2}(\T))\cap C_w ([0,T];L^{\frac{2\gamma}{\gamma +1}}(\T)),&\quad& \mathscr{X}_{W} :=C([0,T];\UU_0),\\ \mathscr{X}_{\varrho}:=C([0,T];W^{-\mathfrak p-4,2}(\T))\cap C_w([0,T]; L^{\gamma}(\T)), &\quad& \mathscr{X}_{\text{C}}:=L^{\infty}_{w*}(0,T;\mathcal{M}(\T,\R^{3\times3})),\\ \mathscr{X}_{ S}:=L^q(0,T;W^{-\mathfrak p-4,2}(\T))\cap L^\infty_{w^\ast}(0,T; L^{\gamma}(\T)), &\qquad& \mathscr{X}_{\text{Q}}:=L^{\infty}_{w*}(0,T;\mathcal{M}(\T,\R^3)),\\ \mathscr X_{\mathfrak t}=\R,\qquad\mathscr{X}_{\text{P}}:=L^{\infty}_{w*}(0,T;\mathcal{M}^{+}(\T,\R)), &\qquad& \mathscr{X}_{\text{e}}:=L^2(0,T;W^{-\mathfrak p-4}(\T)), \end{eqnarray*} with respect to weak-* topology for all spaces with $L^{\infty}(\cdot,\mathcal{M}^{\cdot}(\cdot))$. Furthermore, for $T>0$ fixed, we choose the product path space \begin{equation} \mathfrak{X}:=\mathscr{X}_{\varrho }\times \mathscr{X}_{ \Vec{m}} \times\mathscr{X}_{ S} \times\mathscr{X}_{\text{prss}} \times\mathscr{X}_{\text{conv}}\times\mathscr X_{\text{Q}}\times \mathscr{X}_{\text{e}}\times \mathscr{X}_{W}\times \mathscr{X}_{\mathfrak t}, \end{equation} where $\mathscr{X}_{\text{prss}}$, $\mathscr{X}_{\text{conv}}$ and $\mathscr X_{\text{Q}}$ are the path spaces for the nonlinear terms \[ \mathrm{P}^h:=(\varrho^h)^{\gamma} \exp{\left( \frac{ S^h}{c_v \varrho^h}\right)},\qquad\mathrm{C}^h:=\frac{\vec m^h\otimes \vec m^h}{\varrho^h},\qquad \mathrm Q^h:= S^h \frac{\vec m^h}{\varrho^h}, \] respectively. We denote by $\mathscr B_{\mathfrak X}$ the Borelian $\sigma$-algebra on $\mathfrak X$ and by $$\mathcal{L}[\varrho^h(\cdot\wedge\mathfrak t),\vec{m}^h(\cdot\wedge\mathfrak t), \mathrm S^h(\cdot\wedge\mathfrak t), P^h(\cdot\wedge\mathfrak t), \mathrm C^h(\cdot\wedge\mathfrak t), \mathrm Q^h,\mathbf e^h(\cdot\wedge\mathfrak t), W,\mathfrak t]$$ the probability law on $\mathfrak{X}$. As in \cite{Mo} we can infer that it is a sequence of tight measures on $\mathfrak{X}$. Note that the variables $\mathfrak t$ and $\mathbf e^h$ are not included in \cite{Mo}. The law of $\mathfrak t$ is clearly tight as being a Radon measure on the Polish space $\R$, while tightness of $\mu_{ \mathbf{e}^h}$ follows from \eqref{eq:consistency_errorB}. Also, tightness of the law of $S^h$ follows from \eqref{eq:12.01} and Helly’s selection theorem. In view of Jakubowksi's version of the Skorokhod representation theorem \cite{Jaku} (see also Brzezniak et al. \cite{Brzez}, and \cite[Section 2.8]{FrBrHo} for property c), we have the following proposition. \begin{prop} \label{skorokhod} There exists a nullsequence $(h_m)_{m \in \N}$, a complete probability space $(\Tilde{\Omega}, \Tilde{\FF},\Tilde{\p})$ with $(\mathfrak{X},\mathscr{B}_{\mathfrak{X}})$-valued random variables $(\Tilde{\varrho}^{h_m},\Tilde{\vec{m}}^{h_m},\Tilde{ S}^{h_m}, \Tilde{P}^{h_m}, \Tilde{C}^{h_m},\Tilde{Q}^{h_m},\Tilde{\mathbf e^{h_m}} ,\Tilde{W}^{h_m},\Tilde{\mathfrak t}^{h_m})$, $m \in \N$,\\ and $(\Tilde{\varrho},\Tilde{\vec{m}}, \Tilde{ S},\Tilde{P},\Tilde{C},\Tilde{Q},\tilde{\mathbf e}, \Tilde{W},\tilde{\mathfrak t})$ such that \begin{itemize} \item [(a)] For all $m \in \N$ the law of $$(\Tilde{\varrho}^{h_m},\Tilde{\vec{m}}^{h_m},\Tilde{ S}^{h_m}, \Tilde{P}^{h_m}, \Tilde{C}^{h_m},\Tilde{Q}^{h_m},\Tilde{\mathbf e^{h_m}} ,\Tilde{W}^{h_m},\Tilde{\mathfrak t}^{h_m})$$ on $\mathfrak{X}$ coincides with $$\mathcal{L}[\varrho^{h_m}(\cdot\wedge\mathfrak t),\vec{m}^{h_m}(\cdot\wedge\mathfrak t), \mathrm S^{h_m}(\cdot\wedge\mathfrak t), P^{h_m}(\cdot\wedge\mathfrak t), \mathrm C^{h_m}(\cdot\wedge\mathfrak t), \mathrm Q^{h_m},\mathbf e^{h_m}(\cdot\wedge\mathfrak t), W,\mathfrak t];$$ \item[(b)] The sequence $$(\Tilde{\varrho}^{h_m},\Tilde{\vec{m}}^{h_m},\Tilde{ S}^{h_m}, \Tilde{P}^{h_m}, \Tilde{C}^{h_m},\Tilde{Q}^{h_m},\Tilde{\mathbf e^{h_m}} ,\Tilde{W}^{h_m},\Tilde{\mathfrak t}^{h_m}), \,\,m \in \N,$$ converges $\Tilde{\p}$-almost surely to $$(\Tilde{\varrho},\Tilde{\vec{m}}, \Tilde{ S},\Tilde{P},\Tilde{C},\Tilde{Q},\tilde{\mathbf e}, \Tilde{W},\tilde{\mathfrak t})$$ in the topology of $\mathfrak{X}$, i.e. \begin{equation} \begin{cases} \Tilde{\varrho}^{h_m} \to \Tilde{\varrho} \,\, \text{in }\,\, C([0,T];W^{-\mathfrak p-4,2})\cap C_w([0,T]; L^{\gamma}(\T)),\\ \Tilde{S}^{h_m} \to \Tilde{S} \,\, \text{in }\,\, L^q(0,T;W^{-\mathfrak p-4,2}(\T)),\\ \Tilde{S}^{h_m} \rightharpoonup^\ast \Tilde{S} \,\, \text{in }\,\, L^\infty(0,T; L^{\gamma}(\T)),\\ \Tilde{\vec{m}}_{h_m} \to \Tilde{\vec m} \, \, \text{in}\, \, C([0,T];W^{-\mathfrak p-4,2}(\T))\cap C_w ([0,T];L^{\frac{2\gamma}{\gamma +1}}(\T)),\\ \Tilde{P}^{h_m} \to \overline{\Tilde{P}} \,\, \text{in} \,\, L_{w^*}^{\infty}(0,T;\mathcal{M}^{+}(\T,\R)),\\ \Tilde{C}^{h_m} \to \overline{\Tilde{C}} \,\, \text{in} \,\, L_{w^*}^{\infty}(0,T;\mathcal{M}^{+}(\T,\R^{3\times 3})), \\ \Tilde{Q}^{h_m} \to \overline{\Tilde{Q}} \,\, \text{in} \,\, L_{w^*}^{\infty}(0,T;\mathcal{M}(\T,\R^3)), \\ \Tilde{\vec{e}}^{h_m} \to 0 \, \, \text{in}\, \, L^2(0,T;W^{-\mathfrak p-4,2}(\T)),\\ \Tilde{W}^{h_m} \to \Tilde{W} \,\, \text{in} \,\, C([0,T];\UU_0),\\ \Tilde{\mathfrak t}^{h_m} \to \Tilde{\mathfrak t} \,\, \text{in} \,\, \R, \end{cases} \end{equation} $\Tilde{\p}$-a.s. \item[(c)] For any Carath\'eodory function $H =H(t,x,\varrho,\vec m, S)$ where $(t,x)\in (0,T)\times\T,\, \, (\varrho,\vec m,S)\in \R^5$, satisfying for some $q_1,q_2,q_3>0$ the growth condition \[ H(t,x,\varrho,\vec m, S)\lesssim 1 +|\varrho|^{q_1}+|\vec m|^{q_2}+|S|^{q_2} \] uniformly in $(t,x)$, we denote by $\overline{H(\varrho,\vec m, S)}(t,x) =\langle \mathcal{V}_{t,x},H \rangle$. Then it holds \[ H(\tilde{\varrho}^{h_m},\tilde{\vec m}^{h_m}, \tilde{S}^{h_m}) \rightharpoonup \overline{H(\tilde{\varrho},\tilde{\vec m}, \tilde{S})}\quad\text{in}\,\,L^{k}((0,T)\times\T) \] $\tilde{\p}$-a.s. as $m \to \infty$ for all $1 < k\leq\frac{\gamma+1}{q_1}\wedge \frac{2}{q_2}$. \end{itemize} \end{prop} To guarantee adaptedness of random variables and to ensure that the stochastic integral is well-defined on the new probability space we introduce filtrations for correct measurability. We simplify notation and set $ \Tilde{\mathcal{X}}:=\left[\tilde{\varrho},\tilde{\vec{m}},\tilde{ S}\right]$. Let $\tilde{\FF}_t$ and $\tilde{\FF}_{t}^{h_m}$ be the $\tilde{\p}$-augmented filtration of the correspnding random variables from Proposition \ref{skorokhod}, i.e. \begin{align*} \tilde{\FF}_t &= \sigma (\sigma(t\vee\tilde{\mathfrak t},\rr \Tilde{\mathcal{X}},\rr\tilde{W}) \cup \sigma_t( \tilde{P},\tilde{C},\tilde{Q},\tilde e)\cup\{ \mathcal{N} \in \tilde{\FF};\tilde{\p}(\mathcal{N})=0\}), t\geq 0,\\ \tilde{\FF}_{t}^{\varepsilon_m}&=\sigma(\sigma(t\vee\Tilde{\mathfrak t},\rr\Tilde{\mathcal{X}}^{\varepsilon_m},\rr\tilde{W}^{h_m})\cup \sigma_t (\tilde{P}^{h_m},\tilde{C}^{h_m},\tilde{Q}^{h_m},\tilde{\mathbf e}^{h_m})\cup\{ \mathcal{N} \in \tilde{\FF};\tilde{\p}(\mathcal{N})=0\}), t\geq 0. \end{align*} Here $\rr$ denotes the restriction operator to the interval $[0,t]$ on the path space and $\sigma_t$\footnote{The family of $\sigma$-fields $(\sigma_{t}[\vec V])_{t\geq 0}$ given as random distribution history of \begin{equation*} \sigma_t[\vec V]:= \bigcap_{s>t}\sigma\left(\bigcup_{\varphi\in C_c^{\infty}(Q;\R^3)}\{\langle \vec V, \varphi \rangle <1 \}\cup \{N\in \FF, \p(N)=0\} \right) \end{equation*} is called the history of $\vec V$. In fact, any random distribution is adapted to its history, see\cite{FrBrHo} (Chap. 2.2).} denotes the history of a random distribution. \subsection{Passage to the limit} Noticing that \eqref{eq:DG_standard_2} is a finite dimensional problem we can write in the form \begin{align}\label{eq:1406a} X_t^m&=X^m(0)+\int_0^t\mu(X^m_s)\,\dd s+\int_0^t\Sigma(X_s)\,\dd W, \end{align} where $(X_t^m)$ is an $\R^N$-valued stochastic process (in, fact $X_t^m$ corresponds to $\bvu$ from \eqref{eq:DG_standard_2} with $h=h_m$), $\mu:\R^N\rightarrow\R^N$ and $\Sigma:\R^N\rightarrow\R^{N\times N}$ are Lipschitz continuous functions in the range of $(X^m_{t\wedge \mathfrak t})$ due to \eqref{ass:main}. We have to show that $\tilde\p$-a.s. \begin{align}\label{eq:1406b} \tilde X_{t\wedge \tilde{\mathfrak t}}^m&=\tilde X^m(0)+\int_0^{t\wedge \mathfrak t}\mu(\tilde X^m_s)\,\dd s+\int_0^{t\wedge \mathfrak t}\Sigma(\tilde X^m_s)\,\dd \tilde W^m, \end{align} i.e., the equation continues to hold on the new probability space. Here $\tilde X_t^m$ relates to $(\tilde\varrho^{h_m},\Tilde{\vec{m}}^{h_m},\Tilde{\mathcal E}^{h_m})$ exactly as $X_t^m$ relates to $(\varrho^{h_m},{\vec{m}}^{h_m},{\mathcal E}^{h_m})$ via \eqref{eq:DG_standard_2}. By Proposition \ref{skorokhod} the processes $(X^m_{t\wedge\mathfrak t})$ and $(\tilde X^m_t)=(\tilde X^m_{t\wedge\tilde{\mathfrak t}})$ coincide in law on $C^0([0,T];\R^N)$ and the same holds true for $W$ and $\tilde W$. The mapping $X\mapsto \int_0^{\cdot}\mu(t,X_t)\dt$ is continuous on the $C^0([0,T];\R^N)$. However, the mapping $(X,W)\mapsto \int_s^t\Sigma(r,X_r)\,\diff W$ is not. So, we can not identify it immediately. We will make use of the fact that a martingale is uniquely determined by its quadratic variations. This can be done with the help of an elementary method by Brzezniak and Ondrej\'at \cite{BZ}. We consider the functionals \begin{align*} \mathfrak M(Y,\mathfrak r)_t&=Y_{t\wedge \mathfrak r}-Y(0)-\int_0^{t\wedge \mathfrak r}\mu(r,Y_r)\diff r,\\ \mathfrak N(Y,\mathfrak r)_t&=\int_0^{t\wedge \mathfrak r}|\Sigma(r,Y_r)|^2\diff r,\quad \mathfrak L(Y,\mathfrak r)_t=\int_0^{t\wedge \mathfrak r}\Sigma(r,Y_r)\diff r \end{align*} Obviously $\mathfrak M$, $\mathfrak N$ and $\mathfrak L$ are continuous on the pathspace. Consequently, by equality of laws, we have \begin{align*} \mathfrak M(X^m,\mathfrak t)_{t}&\sim^d \mathfrak M(\tilde X^m,\tilde{\mathfrak t})_{t},\quad \mathfrak N(X^m,\mathfrak t)_{t} \sim^d \mathfrak N(\tilde X^m,\tilde{\mathfrak t})_{t},\quad \mathfrak L(X^m,\mathfrak t)_{t}\sim^d \mathfrak L(\tilde X^m,\tilde{\mathfrak t})_{t}. \end{align*} Let $\mathfrak M(X^m,\mathfrak t)_{s,t}$ denote the increment $\mathfrak M(X^m,\mathfrak t)_{t}-\mathfrak M(X^m,\mathfrak t)_{s}$ and similarly for $\mathfrak N(X^m,\mathfrak t)_{s,t}$ and $\mathfrak N_k(X^m,\mathfrak t)_{s,t}$. Note that the proof will be complete once we show that the process $\mathfrak M(\tilde X^m, \tilde{\mathfrak t})_t$ is an $(\tilde{\FF}_t^m)_{t\geq0}$-martingale and its quadratic and cross variations satisfy, respectively, \begin{equation}\label{marta} \begin{split} \langle\langle \mathfrak M(\tilde X^m,\tilde{\mathfrak t})\rangle\rangle_{t}&=\mathfrak N(\tilde X^m,\tilde{\mathfrak t})_{t},\quad\langle\langle \mathfrak M(\tilde X^m,\tilde{\mathfrak t}),\tilde W\rangle\rangle_{t}=\mathfrak L(\tilde X^m,\tilde{\mathfrak t})_{t}. \end{split} \end{equation} Indeed, in that case we have \begin{align}\label{neu3108a} \Big\langle\Big\langle \mathfrak M(\tilde X^m,\tilde{\mathfrak t})-\int_0^{\cdot\wedge \tilde{\mathfrak t}}\Sigma(s,\tilde X^m_s)\,\dd\tilde W\Big\rangle\Big\rangle_{t}=0 \end{align} which implies the desired equation \eqref{eq:1406b}. Let us verify \eqref{marta}. To this end, we fix times $s,t\in[0,T]$ such that $s<t$ and let $$h:\R\times C^0([0,s];\R^N)\rightarrow [0,1]$$ be a continuous function. Since $\mathfrak M(X^m,\mathfrak t)$ is a square integrable $(\F_t)_{t\geq0}$-martingale, we infer that $$\big[\mathfrak M(X^m,\mathfrak t)\big]^2-\mathfrak N(X^m,\mathfrak t),\quad \mathfrak M(X^m,\mathfrak t)W-\mathfrak L(X^m,\mathfrak t),$$ are $(\FF_t)_{t\geq0}$-martingales. Let $\bfr_s$ be the restriction of a function to the interval $[0,s]$. Then it follows from the equality of laws that \begin{equation}\label{exp11} \tilde{\E}\big[\,h\big(s\vee\tilde{\mathfrak t},\bfr_s\tilde X^m,\bfr_s\tilde{W}^m\big)\mathfrak M(\tilde{\mathfrak t},\tilde X^m,\tilde{\mathfrak t})_{s,t}\big]=\E \big[\,h\big(s\vee \mathfrak t,\bfr_sX^m,\bfr_sW\big)\mathfrak M(X^m,\mathfrak t)_{s,t}\big]=0, \end{equation} \begin{equation}\label{exp21} \begin{split} &\tilde{\E}\bigg[\,h\big(s\vee\tilde{\mathfrak t},\bfr_s\tilde X^m,\bfr_s\tilde{W}^m\big)\Big([\mathfrak M(\tilde X^m,\tilde{\mathfrak t})^2]_{s,t}-\mathfrak N(\tilde X^m,\tilde{\mathfrak t})_{s,t}\Big)\bigg]\\ &=\E\bigg[\,h\big(s\vee{\mathfrak t},\bfr_sX^m,\bfr_sW^m\big)\Big([\mathfrak M(X^m,\mathfrak t)^2]_{s,t}-\mathfrak N(X^m,\mathfrak t)_{s,t}\Big)\bigg]=0, \end{split} \end{equation} \begin{equation}\label{exp31} \begin{split} &\tilde{\E}\bigg[\,h\big(s\vee\tilde{\mathfrak t},\bfr_s\tilde X^m,\bfr_s\tilde{W}^m\big)\Big([\mathfrak M(\tilde X^m,\tilde{\mathfrak t})\tilde{W}^m]_{s,t}-\mathfrak L(\tilde X^m,\tilde{\mathfrak t})_{s,t}\Big)\bigg]\\ &=\E\bigg[\,h\big(s\vee{\mathfrak t},\bfr_sX^m,\bfr_sW\big)\Big([\mathfrak M(X^m,\mathfrak t)W]_{s,t}-\mathfrak L(X^m,\mathfrak t)_{s,t}\Big)\bigg]=0. \end{split} \end{equation} So we have shown \eqref{marta} and hence (\ref{neu3108a}). This finishes the proof of \eqref{eq:1406b} and thus have shown that the equations \begin{equation}\label{eq:consistency_rhotilde} \left[ \int_{\T} \tilde\varrho^h \varphi \diff \bx \right]_{t=0}^{t=\tau\wedge\tilde{\mathfrak t}} =\int_0^{\tau\wedge\tilde{\mathfrak t}} \int_{\T} \tilde{\bm}^h \cdot \nabla_{\bx} \varphi \diff \bx \diff t +\int_0^{\tau\wedge\tilde{\mathfrak t}} \tilde e_{\varrho^h} (t,\varphi) \diff t; \end{equation} \begin{equation}\label{eq:consistency_mtilde} \begin{aligned} \left[ \int_{\T} \tilde{\bm}^h \boldsymbol{\varphi} \diff \bx \right]_{t=0}^{t=\tau\wedge\tilde{\mathfrak t}} &=\int_0^{\tau\wedge\tilde{\mathfrak t}} \int_{\T} \Big(\frac{\tilde{\bm}^h\otimes\tilde{\bm}^h}{\tilde\varrho^h} : \nabla_{\bx}\boldsymbol{\varphi} +\tilde\varrho^h\tilde\vartheta^h \mathrm{div}_{\bx} \boldsymbol{\varphi} \Big)\diff \bx \diff t\\ &+\int_0^{\tau\wedge\tilde{\mathfrak t}}\int_{\T} \tilde\varrho^h\phi\cdot\Pi_{h_m}\boldsymbol\varphi\diff \bx \,\dd \tilde W + \int_0^{\tau\wedge\tilde{\mathfrak t}} \tilde e_{\bm^h} (t,\boldsymbol{\varphi}) \diff t, \end{aligned} \end{equation} \begin{equation}\label{eq:consistency_Stilde} \left[ \int_{\T} \tilde S^h \psi \diff \bx \right]_{t=0}^{t=\tau\wedge\tilde{\mathfrak t}} \leq \int_0^{\tau\wedge\tilde{\mathfrak t}} \int_{\T} \tilde S^h \frac{\tilde\bm^h}{\tilde\varrho^h} \cdot \nabla_\bx {\psi} \diff \bx \diff t + \int_0^{\tau\wedge\tilde{\mathfrak t}} \tilde e_{S^h} (t,\psi) \diff t; \end{equation} hold $\tilde\p$-a.s. for all sufficiently smooth spatial test-functions. For the stochastic integral in we have by It\^{o}-isometry, $\phi\in L_2(\mathfrak U;L^2(\T))$ and \eqref{ass:main} \begin{align*} \tilde\E\bigg[\bigg|\int_0^{\tau\wedge\tilde{\mathfrak t}}\int_{\T} \tilde\varrho^h\phi\cdot(\Pi_{h_m}-\mathrm{id})\boldsymbol\varphi\diff \bx \,\dd \tilde W\bigg|^2\bigg]&=\tilde\E\bigg[\int_0^{\tau\wedge\tilde{\mathfrak t}}\bigg(\int_{\T} \tilde\varrho^h\phi\cdot(\Pi_{h_m}-\mathrm{id})\boldsymbol\varphi\diff \bx\bigg)^2\dt \bigg]\\ &\leq\,c(\tau,K,\phi)\|(\Pi_{h_m}-\mathrm{id})\boldsymbol\varphi\|^2_{L^2(\T)}, \end{align*} which vanishes as $m\rightarrow\infty$. By Proposition \ref{skorokhod} we can pass to the limit in the remaining terms noticing that the error terms on the right-hand side vanish as $m\rightarrow\infty$. Hence the limit object is a dissipative solution in the sense of Definition \ref{E:dfn} provided we can establish the energy balance. On the original probability space, the approximate the energy equality holds in the sense that \begin{eqnarray*} \mathfrak E_{t\wedge\mathfrak t}^{h_m}= \mathfrak E_{s\wedge\mathfrak t}^{h_m}+\frac{1}{2}\int_{s\wedge\mathfrak t}^{t\wedge\mathfrak t}\|\sqrt{\varrho^{h_m}}\phi\|_{L_2(\UU,L^2(\T))}^{2}\,\dd \sigma+ \int_{s\wedge\mathfrak t}^{t\wedge\mathfrak t}\int_{\T}\vec m^{h_m}\phi \,\dd x\dd W, \end{eqnarray*} $\p$-a.s for a.a $0\leq s<t$ (including $s=0$), where \[ \mathfrak E_{t}^{h_m} =\int_{\T}\left[\frac{1}{2}\frac{|\vec m^{h_m}|^2}{\varrho^{h_m}}+ c_v(\varrho^{h_m})^{\gamma}\exp{\left(\frac{S^{h_m}}{c_v\varrho^{h_m}}\right)} \right]\, \dd x , \] for a.a $t \geq 0$. For any fixed $s$ this is equivalent to \[ -\int_{s\wedge\mathfrak t}^{T\wedge\mathfrak t}\partial_t \varphi \mathfrak E_{t\wedge\mathfrak t}^{h_m} \, \dd t- \varphi(s)\mathfrak E_{s\wedge\mathfrak t}^{h_m} = \frac{1}{2}\int_{s\wedge\mathfrak t}^{T\wedge\mathfrak t}\varphi \|\sqrt{\varrho^{h_m}}\phi\|_{L_2(\UU,L^2(\T))}^{2}\,\dd t+ \int_{s\wedge\mathfrak t}^{T\wedge\mathfrak t}\varphi\int_{\T}\vec m^{h_m} \cdot \phi \,\dd x\dd W, \] $\p$-a.s for all $\varphi\in C_{0}^{\infty}([s,T))$. By virtue of Theorem 2.9.1 in \cite{FrBrHo}, and in view of Proposition \ref{skorokhod} the energy equality continues to hold on the new probability space and reads \[ \tilde{\mathfrak E}_{t\wedge\tilde{\mathfrak t}}^{h_m}=\tilde{\mathfrak E}_{s\wedge\tilde{\mathfrak t}}^{h_m}+\frac{1}{2}\int_{s\tilde{\wedge\mathfrak t}}^{t\wedge\tilde{\mathfrak t}}\|\sqrt{\tilde{\varrho}^{h_m}}\phi\|_{L_2(\UU,L^2(\T))}^{2}\,\dd \sigma+ \int_{s\wedge\tilde{\mathfrak t}}^{t\wedge\tilde{\mathfrak t}}\int_{\T}\tilde{\vec m}^{h_m}\phi \,\dd x\dd \tilde{W}^{h_m}, \] $\tilde{\p}$-a.s. for a.a $s$(including $s=0$) and all $t \geq s$. Averaging in $t$ and s, the above expression becomes continuous on the path space. Furthermore, fixing $s=0$ and Lemma 2.1 in \cite {Debussche}, the bounds established in Proposition \ref{skorokhod}, and higher moments to perform the limit $m\to \infty$ we obtain \begin{equation}\label{eq:newEner} \tilde{\mathfrak{E}}_{t\wedge\tilde{\mathfrak t}}\leq\tilde{\mathfrak{E}}_0+\frac{1}{2}\int_{s\wedge\tilde{\mathfrak t}}^{t\wedge\tilde{\mathfrak t}}\|\sqrt{\tilde{\varrho}}\phi\|_{L_2(\UU;L^2(\T))}^2\, \dd \sigma + \int_{s\wedge\tilde{\mathfrak t}}^{t\wedge\tilde{\mathfrak t}}\int_{\T}\tilde{\vec m} \cdot \phi \, \dd x \dd \tilde{W}, \end{equation} $\p$-a.s. for a.a. $t\in [0,T]$, where \[ \tilde{\mathfrak {E}}_t= \int_{\T}\left[\frac{1}{2}\frac{|\tilde{\vec m} |^2}{\tilde{\varrho}}+c_v\tilde{\varrho}^{\gamma}\exp{\left(\frac{\tilde{S}}{c_v\tilde{\varrho}}\right)}\right]\, \dd x + \frac{1}{2}\int_{\T}\dd\, \mathrm{tr}\tilde{\mathcal{R}}_{\text{conv}}(t) +c_v\int_{\T}\dd\, \tilde{\mathcal{R}}_{\text{press}}(t), \] and \[ \tilde{\mathfrak E}_0= \int_{\T}\left[\frac{1}{2}\frac{|{\vec m}_0 |^2}{{\varrho}_0}+c_v{\varrho}_0^{\gamma}\exp{\left(\frac{S_0}{c_v\varrho_0}\right)}\right]\, \dd x. \] Performing the limit $\varepsilon_m\to 0$ yields an energy inequality. Hence, to convert (\ref{eq:newEner}) to equality, it is sufficient to augment the term contributing to the internal energy ($\tilde{\mathcal{R}}_{\text{press}}(t)$) by $h(t)\dd x$ with A suitable spatially homogeneous $h\geq 0$. NOte that $\tilde{\mathcal{R}}_{\text{press}}(t)$ acts on $\mathrm{div}_x\varphi$ such that $\int_{\T}h(t)\mathrm{div}_x \varphi\,\dd x=0$ and hence \[ -\int_{s\wedge\tilde{\mathfrak t}}^{T\wedge\tilde{\mathfrak t}}\partial_t \varphi \tilde{\mathfrak E}_t \, \dd t- \varphi(s)\tilde{\mathfrak E}_{s\wedge\tilde{\mathfrak t}} = \frac{1}{2}\int_{s\wedge\tilde{\mathfrak t}}^{T\wedge\tilde{\mathfrak t}}\varphi \|\sqrt{\tilde\varrho}\phi\|_{L_2(\UU,L^2(\T))}^{2}\,\dd t+ \int_{s\wedge\tilde{\mathfrak t}}^{T\wedge\tilde{\mathfrak t}}\varphi\int_{\T}\tilde{\vec m}\cdot \phi \,\dd x\dd \tilde W, \] $\p$-a.s for all $\varphi\in C_{0}^{\infty}([s,T))$. This completes the proof of Theorem \ref{thm:main1}. \section{Convergence}\label{se_Convergence} In this section we prove Theorem \ref{thm:main2}. The main tool is the relative entropy, which we derive in the next subsection. \subsection{The relative entropy} We consider the \textit{relative entropy} for the discrete solution $(\varrho^h,\vec m^h, \mathcal E^h)$ measuring its distance to a smooth comparison function $(r,\Theta,\vec v)$ (which will be chosen in the next subsection as the strong pathwise solution to \eqref{Euler}). It is given by \begin{align}\label{relative} \begin{aligned} \mathcal{K}\big(\varrho^h,E^h,\vec m^h\big|r,\Theta,\vec v\big) &= \frac{1}{2}\int_{\T}\varrho^h\left|\frac{\vec m^h}{\varrho^h} -\vec v\right|^2\, \dd x-\int_{\T}\Theta\varrho^h {s}(\varrho^h,\mathcal E^h)\, \dd x-\int_{\T}\varrho^h\partial_{\varrho}H_{\Theta}(r,\Theta)\,\dd x\\&+\int_{\T}\big(\partial_{\varrho}H_{\Theta}(r,\Theta)(r)-H_{\Theta}(r,\Theta)\big)\, \dd x \end{aligned} \end{align} for $t\in[0,\mathfrak t]$. Here we used the \textit{ballistic free energy} \begin{equation}\label{ballistic} H_{{\Theta}}(\varrho,\vartheta)=\varrho e(\varrho,\vartheta)-\Theta \varrho s (\varrho,\vartheta), \end{equation} recalling the formulas for $e$ and $s$ from Section \ref{sec:cr}. We obtain the following result. \begin{prop}[Relative Entropy Inequality]\label{propA} Let $(\Omega, \FF, (\FF_t)_{t\geq 0},\p)$ be a complete stochastic basis with a probability measure $\p$ on $(\Omega,\FF)$ and right-continuous filtration $(\FF_t)_{t\geq 0}$ and $W$ an $(\FF_t)$-adapted Wiener process. Let $(\varrho^h,\vec m^h,\mathcal E^h)$ be the solution to \eqref{eq:DG_standard_2}. Suppose there is an $(\mathfrak F_t)$-stopping time $\mathfrak t$ such that \eqref{ass:main} holds. Let $(r,\Theta, \vec v)$ be a trio of $(\FF_t)_{t\geq 0}$ -adapted stochastic processes defined on $(\Omega, \FF, (\FF_t)_{t\geq 0},\p)$ such that for some $T>0$ \begin{align*} r,\Theta,\vec v \in C([0,T]; C^{1} (\T)) \quad\p \text{-a.s.,} \,\,\, \E\left[\sup_{t\in [0,T]}\|(r,\Theta,\vec v)\|_{W^{1,\infty}(\T)}\right]^q < \infty \text{ for all }\, 2\leq q < \infty, \end{align*} \[ 0<\underline{r} \leq r(t,x) \leq \overline{r}, \quad 0<\underline{\Theta} \leq \Theta(t,x) \leq \overline{\Theta} \quad \p\text{-a.s.} \] Suppose further that it holds \begin{align}\label{eq:sass} \begin{aligned} \dd r &= D_t^dr\,\dd t,\\ \dd \vec v &=D_t^d \vec v \,\dd t+ \mathbb{D}_t^s \vec v\, \dd W,\\ \dd \Theta &= D_t^d \Theta\,\dd t, \end{aligned} \end{align} where \[ D^dr,D^d \Theta,D^d\vec v \in L^{q}(\Omega; C([0,T];C^{1}(\T)))\qquad \mathbb{D}^s\vec v \in L^2(\Omega; L^2(0,T;L_2(\UU;L^2(\T))), \] \begin{equation}\label{eq:prop} \left(\sum_{k\geq 1}|\mathbb{D}^s\vec v(e_k)|^q\right)^{\frac{1}{q}} \in L^{q}(\Omega; L^q(0,T;L^q(\T))). \end{equation} Then the relative entropy inequality: \begin{align}\label{REI} \begin{aligned} \mathcal{K}\bigg(\varrho^h,\mathcal E^h,\vec m^h\bigg|r,\Theta,\vec v\bigg)&\leq\,c \,\mathcal{K}\bigg(\varrho^h,\mathcal E^h,\vec m^h\bigg|r,\Theta,\vec v\bigg)(0)+c\int_{0}^{\tau}\mathcal{Q}\bigg(\varrho^h,\vartheta^h,\vec m^h\bigg|r,\Theta,\vec v\bigg)\, \dd t+\mathbb{M}\\ &+c\,\mathbf e^h(\partial_{\varrho}H_{\Theta}(r,\Theta),\vec v,\Theta) \end{aligned} \end{align} holds $\p$-a.s for all $\tau \in (0,T\wedge\mathfrak t)$, where \begin{align*} \mathcal{Q}\bigg(\varrho^h,\vartheta^h,\vec m^h\bigg|r,\Theta,\vec v\bigg)= &\int_{\T}\varrho\left(\frac{\vec m^h}{\varrho^h}-\vec v\right)\cdot\nabla_x\vec v\cdot \left(\vec v-\frac{\vec m^h}{\varrho^h}\right)\,\dd x\nonumber\\ &+\int_{\T}[(D_t^d \vec v + \vec v\cdot\nabla_x \vec v )\cdot(\varrho\vec v- \vec m^h) -p(\varrho^h,\vartheta^h)\mathrm{div}_x\vec v] \, \dd x \\ &-\int_{0}^{\tau}\int_{\T}[\varrho^h s(\varrho^h,\vartheta^h) D_t^d \Theta + s(\varrho^h,\vartheta^h)\vec m^h\cdot \nabla_x \Theta ] \, \dd x \dd t\nonumber\\ &+\int_{0}^{T}\int_{\T}[\varrho^h s (r,\Theta) \partial_{t}\Theta+ \vec m^h s(r,\Theta)\cdot\nabla_x\Theta]\, \dd x\dd t \nonumber\\ &+\int_{\T} \left(\left(1-\frac{\varrho^h}{r}\right)\partial_{t}p(r,\Theta)-\frac{\vec m^h}{r}\cdot\nabla_x p(r,\Theta)\right)\,\dd x\nonumber,\\ &-\sum_{k\geq 1}\int_{\T}\mathbb{D}_t^s\vec v(e_k)\cdot \varrho^h\phi(e_k)\, \dd x \\ &+\frac{1}{2}\|\sqrt{{\varrho^h}}\phi\|_{L_2(\UU,L^2(\T))}^{2} +\frac{1}{2}\sum_{k\geq 1}\int_{\T}\varrho^h|\mathbb{D}_t^s\vec v(e_k)|^2\, \dd x, \end{align*} and \begin{align*} \mathbb{M}&= \int_{0}^{\tau}\int_{\T}{\vec m^h}\phi \,\dd x\dd {W} \\ & -\int_{0}^{t}\int_{\T}\bigg[\vec m^h\mathbb{D}_t^s\vec v+ \Pi_h\vec v\varrho^h \phi\bigg]\,\dd x \dd W +\int_{0}^{t}\int_{\T}\varrho^h\vec v\cdot \mathbb{D}_t^s\vec v\, \dd x\dd W. \end{align*} \end{prop} \begin{proof} By It\^{o}'s formula we obtain from \eqref{eq:consistency_m} and \eqref{eq:sass} \begin{align}\label{eq:PA} \begin{aligned} \dd \left( \int_{\T}\vec m^h\cdot\vec v \,\dd x \right)&=\left(\int_{\T}\bigg[\vec m^h\cdot D_t^d\vec v+ \left(\frac{\vec m^h\otimes\vec m^h}{\varrho^h}\right)\cdot\nabla \vec v+ p(\varrho^h, s^h)\mathrm{div}\vec v\bigg]\,\dd x \right)\, \dd t\\ &+\sum_{k\geq 1}\int_{\T}\mathbb{D}_t^s\vec v(e_k)\cdot \varrho^h\phi(e_k)\, \dd x \dd t\\ &+e_{\vec m^h}(t,\vec v)\dt+\dd M_1,\end{aligned} \end{align} where \[ M_1 =\int_{0}^{t}\int_{\T}\bigg[\vec m\mathbb{D}_t^s\vec v+ \Pi_h\vec v\varrho \phi\bigg]\,\dd x \dd W. \] Similarly to (\ref{eq:PA}), we compute \begin{align}\label{eq:PB} \begin{aligned} \dd\left(\int_{\T}\frac{1}{2}\varrho^h |\vec v|^2\,\dd x\right)&=\int_{\T}\varrho^h\vec v \cdot \nabla \vec v\cdot \vec v\, \dd x \dd t+ \int_{\T}\varrho^h\vec v\cdot D_t^d\vec v\, \dd x \dd t\\ &+\frac{1}{2}\sum_{k\geq 1}\varrho^h|\mathbb{D}_t^s\vec v(e_k)|^2\, \dd x \dd t +\dd M_2, \end{aligned} \end{align} where \[ M_2 = \int_{0}^{t}\int_{\T}\varrho^h\vec v\cdot \mathbb{D}_t^s\vec v\, \dd x\dd W. \] Testing the entropy balance \eqref{eq:consistency_S} with $\Theta$ we deduce \begin{equation}\label{eq:PD} \dd \left(\int_{\T} \varrho^h s(\varrho^h,\vartheta^h) \Theta \dd x\right) \geq\int_{\T} \vec m^h s(\varrho^h,\vartheta^h) \cdot \nabla_x \Theta\, \dd x\dd t+\int_{\T}\varrho^h s(\varrho^h,\vartheta^h) \partial_t \Theta\, \dd x\dd t+e_{S^h}(t,\Theta)\dt. \end{equation} Similarly, testing the continuity equation \eqref{eq:consistency_rho} with $\partial_{\varrho}H_{\Theta}(r,\Theta)$ yields \begin{equation}\label{eq:PF} \dd\left(\int_{\T}\varrho \partial_{\varrho}H_{\Theta}(r,\Theta)\, \dd x \right) =\int_{\T} \vec m\cdot \nabla_x(\partial_{\varrho}H_{\Theta}(r,\Theta)) \,\dd x+ \int_{\T}\varrho^h \partial_t(\partial_{\varrho}H_{\Theta}(r,\Theta))\, \dd x+e_{\varrho^h}(t,\partial_{\varrho}H_{\Theta}(r,\Theta))\dt. \end{equation} Finally, we collect and sum the resulting expressions (\ref{eq:PA})--(\ref{eq:PF}), and add the energy balance from \eqref{eq:consistency_S} to the sum. \end{proof} \subsection{Error estimates} \begin{proof}[Proof of Theorem \ref{thm:main2}] We choose now $(r,\vec v,\Theta)$ in Proposition \ref{propA} to be the local strong solution from Definition and $t\in[0,\mathfrak t\wedge \mathfrak s_M]$. The terms contained in $\mathcal Q$ corresponding exactly to those in \cite{Mo}\footnote{In fact, in \cite{Mo} also the defect measures appear which are not present for the discrete solution.} and we obtain as there \begin{align*} \mathcal{Q}\bigg(\varrho^h,\vartheta^h,\vec m^h\bigg|r,\Theta,\vec v\bigg)(\tau\wedge \mathfrak s_M\wedge \mathfrak t)\leq\,c \int_0^{\tau\wedge \mathfrak s_M\wedge \mathfrak t} \mathcal{K}\bigg(\varrho^h,\mathcal E^h,\vec m^h\bigg|r,\Theta,\vec v\bigg)\,\diff t \end{align*} $\p$-a.s. with a constant depending on $M$. The right-hand side can be handled by Gronwall's lemma. The stochastic terms in \eqref{REI} vanish after taking expectations. Finally, due to the regularity of $(r,\vec v,\Theta)$ we can use estimate \eqref{eq:consistency_errorB} and \eqref{eq:consistency_errorA} (depending on whether $\mathfrak p\geq 1$ or $\mathfrak p=0$) to control the error term by $h$. By \cite[Lemma 2.7]{LuSh} we have for some $c>0$ independent of $h$ \begin{align*} &\E\Big[\mathcal{K}\bigg(\varrho^h,\mathcal E^h,\vec m^h\bigg|r,\Theta,\vec v\bigg)(\tau\wedge \mathfrak s_M\wedge \mathfrak t)\Big]\\&\qquad\qquad\leq\,c\,\E \Big[\Big(\|\varrho^h-r\|^2_{L^2(\T)}+\|\vec u^h-\vec v\|^2_{L^2(\T)}+\|\vartheta^h-\Theta\|^2_{L^2(\T)}\Big)(t\wedge\mathfrak t\wedge\mathfrak s_M)\Big], \end{align*} where the constant depends on $K$ from \eqref{ass:main}. The proof is complete. \end{proof} \section{Numerical simulation}\label{sec_numerical} To support our theoretical analysis, we derive some simulations solving the stochastic Euler system \eqref{EulerC}. The numerical experiments are conducted using the Julia programming language \cite{bezanson2017julia}. For stochastic time integration we employ if nothing else is said the Euler–Maruyama method from the DifferentialEquations.jl package \cite{rackauckas2017differentialequations}. Spatial discretization is handled using the Trixi.jl framework \cite{ranocha2022adaptive,schlottkelakemper2021purely}. Visualization of the results is achieved through Plots.jl \cite{christ2023plots}, and Random.jl package of Julia is used for generating the noise environment. As described before, up-to-the-authors knowledge this is the first time to present simulations for this type of problems where discontinuities may rise and we lose regularity inside the path even if we start with smooth initial data. In our numerical testing, we focus therefore either on short time periods and/or weak noise strength meaning that if we apply \eqref{eq:14.02}, we multiply with constants which we choose appropriately to our testing. These numerical simulations should only give a proof of concept where further investigations in this direction together with a more efficient implementation strategy is part of future research. Meanwhile we also refer to \cite{breit2023mean} for a numerical investigation in context of the stochastic Navier-Stokes equations. \\ We are investigating two errors in detail. We are investigating the expected value of the classical discrete $L^2$ norm in terms of density and the expected value \eqref{eq:errorA} which represents the estimation of relative energy. The error behaviour in other quantities, such as momentum and energy, follows a similar pattern to that of the density in our observations and is therefore omitted from further analysis. We analyse therefore \begin{align} \mathtt{E}_1&=\E \left[ \|\varrho^h-\varrho\|_{L^2(\T)} \right]; \label{L2_error}\\ \mathtt{E}_2&=\E\Big[\Big(\|\varrho^h-r\|^2_{L^2(\T)}+\|\vec u^h-\vec v\|^2_{L^2(\T)}+\|\vartheta^h-\Theta\|^2_{L^2(\T)}\Big) \Big]. \label{relative_energy} \end{align} at the end time. Since an analytical solution is unavailable, we compute a reference solution on the finest grid using the same set-up and project the numerical solution $ \varrho$ onto this finer grid to evaluate the errors. For the spatial discretisation, we consistently use the local Lax-Friedrichs (LLF) or Rusanov flux for the surface flux, while for the volume flux, we apply the entropy-conservative and kinetic-energy-preserving flux \eqref{eq:Ranocha_flux} from \cite{ranocha2018}. With polynomial degree zero, this approach aligns with the classical cell-centered finite volume (FV) framework, where the FV method employs the LLF flux. For the DG case, we restrict ourself in the following mainly to the polynomial degree one for simplicity and efficiency. As stated before, this part is only a proof of concept, a more and detailed investigation will be left for future work together with additional tasks like the construction of limiting strategies for the stochastic time-integration method and a more efficient implementation. In the simulations, we set the number of samples to 1000 and consider one- and two-dimensional cases. \subsection{One dimension}\label{sec_one_d} We consider the one-dimensional stochastic compressible Euler equations $\gamma = 1.4$ \begin{equation}\label{eq:euler} \dd \begin{pmatrix} \varrho \\ m \\ \varrho e \end{pmatrix} + \dd_x \begin{pmatrix} m \\ m^2/\varrho + p \\ (\varrho e + p) v \end{pmatrix} \dd t = \frac{1}{2}\mu^2 \begin{pmatrix} 0 \\ 0 \\ \varrho \end{pmatrix} \dd t + \mu \begin{pmatrix} 0 \\ \varrho \\ m \end{pmatrix} \dd \beta \end{equation} where $\mu$ denotes a stochastic forcing factor and $\beta$ is a standard one-dimensional Wiener process. If nothing else is said, we select $\mu=1$ in the one dimensional tests. \subsubsection{Grid Convergence}\label{sub_sub_section_1} We consider a simple density wave \begin{equation} \varrho(0, x) = 1 + 0.5 \sin\bigl(2\cdot x \bigr), \quad v(0, x) = 0.1, \quad p(0,x)= 10, \end{equation} für $t \in [0, 0.5]$ and $x \in [-1, 1]$. In the deterministic case, we obtain a simple density wave (advection behaviour). The reference solution is calculated using $2^{12}=4096$ elements and the time-step is set to $\Delta t=10^{-5}$ where $T=0.5$.\\ In Table \ref{FV_DG_density}, the errors are plotted. We observe error behaviour for the FV methods in density around 0.8 and the estimation in relative entropy is around 1.23 which is a little bit better than from our theoretical result but beyond the accuracy estimation for classical FV methods which is one. \\ In the case of DGSEM with order $\mathfrak{p=1}$ we obtain second order for the density error (and all other quantities) and fourth order in terms of the estimation of relative energy. These results are better than the one obtained through our analysis which relies on the consistency investigation and does take the regularity of the solution into account. We stress that the results presented lies inside the behaviour which one expects for DGSEM methods applied to the deterministic Euler equations as demonstrated already in several works, for instance \cite{gassner2016,zbMATH07517718,chen2017}. {\small{ \begin{table}[h!] \centering \begin{tabular}{|c|c|c|c|c|c|c|c|c|} \hline & \multicolumn{4}{c|}{FV} & \multicolumn{4}{c|}{DGSEM ($\mathfrak{p}=1$)} \\ \cline{2-9} Elements & $\mathtt{E}_1$ & EOC($\mathtt{E}_1$) & $\mathtt{E}_2$ & EOC($\mathtt{E}_2$) & $\mathtt{E}_1$ & EOC($\mathtt{E}_1$) & $\mathtt{E}_2$ & EOC($\mathtt{E}_2$) \\ \hline 64 & 0.2492 & - & 13.6617 & - & 0.0029& - & 0.0041 & -\\ 128 & 0.1616 & 0.63& 7.4707 & 0.87& 0.0007 & 2.00 & 0.0003&3.98 \\ 256 & 0.0913& 0.82 & 3.1981 & 1.23 & 0.0002 & 2.00 & $1.63\cdot 10^{-5}$ & 4.00 \\ 512 & 0.0464 & 0.98 & 1.0426 & 1.60 & $4.48\cdot 10^{-5}$ & 2.00 & $1.01\cdot 10^{-6}$ & 4.02 \\ \hline average EOC& - & 0.81 & - & 1.23 & -& 2.0 & -& 4.00 \\ \hline \end{tabular} \caption{LLF FV and entropy stable DGSEM ($\mathfrak{p}=1$) methods - error plot using 1000 samples} \label{FV_DG_density} \end{table} }} \subsubsection{One-dimensional Riemann problem} Next, we examine one-dimensional Riemann problems in the domain \( [0,1] \). It is important to note that our Theorem \ref{thm:main2} does not apply to these test cases, as the initial conditions do not satisfy the requirement of belonging to \( W^{\mathfrak{p}+3,2} \); in fact, they are no longer even Lipschitz continuous. These types of problems are included for comparison with the results from \cite{LuSh}, where the Godunov method and a particular FV method were numerically studied in the deterministic case. The reference solutions are again calculated using \(4096\) elements. The initial data are as follows: \begin{itemize} \item Rarefaction wave: $ (\varrho, v, p) (0, x) = \begin{cases} (0.5197, - 0.7259,0.4), \qquad x <0.5,\\ (1, 0, 1),\qquad x>0.5. \end{cases}$ \item Contact discontinuity: $ (\varrho, v, p) (0, x) = \begin{cases} (0.5,0.5,5), \qquad x <0.5,\\ (1, 5,0, 5),\qquad x>0.5. \end{cases}$ \item Shock wave: $ (\varrho, v, p) (0, x) = \begin{cases} (1,0.7276,1), \qquad x <0.5,\\ (0.5313, 0, 0.4),\qquad x>0.5. \end{cases}$ \end{itemize} We use $\Delta t=10^{-5}$ and $T=0.2$ in all three test cases. The boundary conditions are outflow conditions. In Tables \ref{FV_DG_rarefaction} and \ref{FV_Contact_Shock}, the errors and experimental orders of convergence are given for different settings. To summarize the results: \begin{itemize} \item Rarefaction wave: For $\mathfrak p=0$ we recognise that convergence rates in terms of density are slightly larger than $0.5$ (in average 0.62) whereas the estimation of relative energy is around 1.28 (larger than 1). This is in agreement with the deterministic case from \cite{LuSh} where for Godunov method similar results have been presented and verified the theoretical result\footnote{In \cite{LuSh}, the assumption on the initial data was less restrictive and only an estimation in terms of projection from the initial data and the initial data itself was used.}. In the DGSEM setting with $\mathfrak{p}=1$, we recognise that due to the lack of regularity and on account of the Gibbs phenomena (projection at the beginning of the simulation), the convergence error rates are lower in both quantities. Here, additional techniques have to be used like the application of modal filters, for instance similar to the works \cite{offner2015zweidimensionale,zbMATH00036793}, to obtain better error rates. In the following, we will concentrate on the FV setting. \item For the single contact wave the convergence rate for the density is in average around $0.43$ which is slightly better than $1/4$. In terms of the estimation of the relative energy we have $0.84$ where the most accurate results are obtained on the finest grid. \item For the single shock wave the convergence rate of $\mathtt{E}_1$ is around $0.52$. The error in $\mathtt{E}_2$ is about $1.02$. This outcome is consistent with the deterministic case discussed in \cite{LuSh}. \end{itemize} {\small{ \begin{table}[h!] \centering \begin{tabular}{|c|c|c|c|c|c|c|c|c|} \hline & \multicolumn{4}{c|}{FV} & \multicolumn{4}{c|}{DGSEM ($\mathfrak{p}=1$)} \\ \cline{2-9} Elements & $\mathtt{E}_1$ & EOC($\mathtt{E}_1$) & $\mathtt{E}_2$ & EOC($\mathtt{E}_2$) & $\mathtt{E}_1$ & EOC($\mathtt{E}_1$) & $\mathtt{E}_2$ & EOC($\mathtt{E}_2$) \\ \hline 64 & 0.0321 & - & 0.0046 & - & 0.0141 & - & 0.00111 & -\\ 128 & 0.0224 & 0.51 & 0.0021 & 1.09 & 0.0098 & 0.52 & 0.00077 & 0.52\\ 256 & 0.0148 & 0.61 & 0.0009 & 1.26 & 0.0074& 0.41 & 0.00066 & 0.23 \\ 512 & 0.0089 & 0.72 & 0.0003 & 1.47 & 0.0061 & 0.27 & 0.00061 & 0.08 \\ \hline average EOC& - & 0.61 & - & 1.27 & - & 0.4 & - & 0.28 \\ \hline \end{tabular} \caption{LLF-FV and entr. dissipative DGSEM methods for the rarefaction wave - 1000 samples} \label{FV_DG_rarefaction} \end{table} }} {\small{ \begin{table}[h!] \centering \begin{tabular}{|c|c|c|c|c|c|c|c|c|} \hline & \multicolumn{4}{c|}{Contact} & \multicolumn{4}{c|}{Shock} \\ \cline{2-9} Elements & $\mathtt{E}_1$ & EOC($\mathtt{E}_1$) & $\mathtt{E}_2$ & EOC($\mathtt{E}_2$) & $\mathtt{E}_1$ & EOC($\mathtt{E}_1$) & $\mathtt{E}_2$ & EOC($\mathtt{E}_2$) \\\hline 64 & 0.0673 & - & 0.5913 & - & 0.049 & - & 0.010 & -\\ 128 & 0.0525 & 0.36& 0.3620 & 0.71 & 0.037 & 0.45 & 0.005 & 0.86 \\ 256 & 0.0395 & 0.41 & 0.2058 &0.81 & 0.025 & 0.51 & 0.003 & 1.01 \\ 512 & 0.0279 & 0.50 & 0.1031 & 1.00 & 0.017 & 0.60 & 0.001 & 1.19\\ \hline average EOC& - & 0.42 & & 0.84& -& 0.52 & - & 1.02\\ \hline \end{tabular} \caption{LLF-FV method - 1000 samples} \label{FV_Contact_Shock} \end{table} }} \subsubsection{1D SOD} This experiment is the famous SOD problem. The exact deterministic solutions contains a rarefaction, contact and shock waves. In this example, the final time is set to $T=0.15$ and the initial data is given by \begin{equation*} (\varrho, v, p) (0, x) = \begin{cases} (1,0,1), \qquad x <0.5,\\ (0.125,0, 0.1),\quad x>0.5. \end{cases} \end{equation*} We use again $4096$ elements and $\Delta t=10^{-5}$. The convergence rates can be seen in Table \ref{FV_SOD} and we recognize that even without the regulartiy of the intitial data, we obtain convergence rates which align with the result itself inside the FV setting ($\approx 0.5$ in $\mathtt{E}_1$ and $\approx 1$ in $\mathtt{E}_2$). {\small{ \begin{table}[h!] \centering \begin{tabular}{|c|c|c|c|c|} \hline Elements & $\mathtt{E}_1$ & EOC($\mathtt{E}_1)$ &$\mathtt{E}_2$ &EOC($\mathtt{E}_2)$ \\ \hline \hline 64 & 0.0378 & - & 0.0203 & - \\ 128 & 0.0287 & 0.40 & 0.0106 & 0.93 \\ 256 & 0.0200 & 0.51 & 0.0052& 1.05 \\ 512 & 0.0130 & 0.63 & 0.0022 & 1.20 \\ \hline av. order & - & 0.51 & -&1.06 \\ \hline \end{tabular} \caption{FV method for SOD - 1000 samples} \label{FV_SOD} \end{table} }} We also illustrate the impact of the $\mu$ parameter and provide examples demonstrating various noise behaviours. Figure \ref{fig:SOD} presents the density profiles for different configurations of the (stochastic) Euler equations, with a zoomed-in view around the shock wave. The simulations utilise the FV framework with $2^{12}$ elements. The solid black line represents the deterministic solution obtained via the explicit Euler method with $\Delta t = 10^{-4}$. For the stochastic equations, the Euler–Maruyama method is applied using the same $\Delta t$. In the left-hand panels, we display three simulations with distinct noise behaviors, while the right-hand panels compare two simulations with identical noise behavior but different $\mu$ values: one (blue, dotted) and two (red, dash-dotted), alongside the deterministic solution. The parameter $\mu$ significantly influences the deviation of the stochastic solution from the deterministic one, with further analysis on this effect provided in the subsequent subsection. \begin{figure}[h!] \centering \includegraphics[width=0.49\textwidth]{Figures/SOD_both.pdf} \includegraphics[width=0.49\textwidth]{Figures/SOD_real.pdf} \caption{Different noise (left) and different noise strength(right)} \label{fig:SOD} \end{figure} \subsection{Two dimension}\label{sec_two_d} We consider the two-dimensional stochastic compressible Euler equations \eqref{EulerC_short} with the right hand side \begin{equation*} \mathbf{h}(\bU)=\mu^2\varrho\begin{pmatrix}0\\0\\ 0 \\1\end{pmatrix},\quad\mathbf{g}(\bU)\dd W= \mu\begin{pmatrix}0\\\varrho\, \dd \beta_1\\\varrho\,\dd \beta_2 \\m_1\,\dd\beta_1+m_2\,\dd\beta_2\end{pmatrix}. \end{equation*} where $\mu$ denotes again a stochastic forcing factor. The domain is divided into $n\times n$ uniform quads for the elements ($n \in \N$). \subsubsection{Convergence } Similar to Section \ref{sub_sub_section_1}, we consider now the two dimensional density wave given by \begin{equation} \varrho(0, x) = 1 + 0.5 \sin\bigl(2\cdot x \bigr), \quad v_1(0, x) = 0.1, \quad v_2(0, x) = 0.1, \quad p(0,x)= 10, \end{equation} für $t \in [0, 0.1]$ and $x \in [-1, 1]^2$. Here, we use $\mu=0.1$ and calculate everything with $\Delta t=10^{-4}$. The reference solution is calculated using $256^2$ elements. In Table \ref{FV_DG_density_2d}, we see the error behaviour for the FV and DGSEM method with $\mathfrak{p}=1$. We reach first and second order of accuracy which is in line with the deterministic numerical simulations for a smooth test case, e.g. \cite{oeff2023,gassner2016,zbMATH07517718,chen2017} and references therein. {\small{ \begin{table}[h!] \centering \begin{tabular}{|c|c|c|c|c|c|c|c|c|} \hline & \multicolumn{4}{c|}{FV} & \multicolumn{4}{c|}{DGSEM ($\mathfrak{p}=1$)} \\ \cline{2-9} Elements & $\mathtt{E}_1$ & EOC($\mathtt{E}_1$) & $\mathtt{E}_2$ & EOC($\mathtt{E}_2$) & $\mathtt{E}_1$ & EOC($\mathtt{E}_1$) & $\mathtt{E}_2$ & EOC($\mathtt{E}_2$) \\ \hline $16^2$ & 0.2622 & - & 11.5589 & - & 0.0365 & - & 0.6069 & -\\ $32^2$ & 0.1816 & 0.53 & 6.6302 & 0.80 & 0.0093 & 1.97 & 0.0453 & 3.74 \\ $64^2$ & 0.0974 & 0.90 & 2.4735 & 1.42 & 0.0022 & 2.05 & 0.0027 & 4.06 \\ $128^2$ & 0.0368 & 1.41 & 0.4531 & 2.45 & 0.0001 & 2.30 & 0.0001 & 4.59 \\ \hline average EOC& - & 0.95 & - & 1.52 & - & 2.11 & - & 4.13 \\ \hline \end{tabular} \caption{LLF FV and entropy stable DGSEM ($\mathfrak{p}=1$) methods - error plot using 1000 samples} \label{FV_DG_density_2d} \end{table} }} \subsubsection{Kelvin-Helmholtz and influence of the noise } In the second test case in two-space dimensions, we consider the famous Kelvin-Helmholtz instability test. Kelvin-Helmholtz describes a shear flow of three fluid layers with different densities. The initial data are given by \begin{equation}\label{init_KH} (\varrho, v_1, v_2, p)(\bx, 0)=\begin{cases} (2,-0.5,0,2.5), \quad I_1 \leq y \leq I_2,\\ (1,0.5,0,2.5), \text{ otherwise 0}, \end{cases} \end{equation} where the interface profiles $I_j=I_j(\bx):= J_j+\epsilon Y_j(\bx),\; j=1,2$ and $0<\epsilon\ll1$, are chosen to be small perturbations around the lower $J_1=0.25$ and the upper $J_2=0.75$ interfaces, respectively. Moreover, $Y_j= \sum_{m=1}^M a_j^m \cos \left( b_j^m +2\pi m x \right)$, $j=1,2$, with $a_j^m\in [0,1]$ and $b_j^m\in[-\pi, \pi],\; j=1,2,\; m=1,\cdots, M$ are arbitrary but fixed numbers. The coefficients $a_j^m$ have been normalized such that $\sum_{m=1}^M a_j^m=1$ to guarantee that $|I_j-J_j|\leq \epsilon$ for $j=1,2.$ In the simulations, we have $M=10$, $\epsilon=0.01$ and $T=1.5$. We solve the Kelvin-Helmholtz problem on $[0,1]\times[0,1]$ with periodic boundary conditions. It is known that no grid conference can be obtained as can be seen for instance in \cite{Fjordholm2016, feireisl2020convergence,LuOe} and references therein. Vertices rise on different locations and fine scales which can also derive problems in terms of stability for entropy dissipative DG methods \cite{glaubitz2024generalized,ranocha2025robustness}.\\ However, to demonstrate now the differences driven by the noise and also by the forcing factor $\mu$, we print in Figure \ref{fig:main} different realisations using the flux differencing DGSEM method with $\mathfrak{p}=2$ on a grid with $64 \times 64$ elements. The time step is set to $\Delta t=10^{-4}$. We are using again explicit Euler in the deterministic case (for comparison) and the Euler–Maruyama in the stochastic setting. Subfigure \ref{fig:sub1} depicts the deterministic solution, whereas subfigures \ref{fig:sub2}–\ref{fig:sub3} show two different realizations of the same test case with distinct noise configurations. Subfigures \ref{fig:sub3}–\ref{fig:sub4} illustrate the same problem but with varying noise strengths. These results clearly highlight the significant impact of noise strength on the solution. We do not want to hide that if the strength is to high above $1$, we have obtained also stability issues in this test case for the tested realisations. This can possible also happen for other realisations with the smaller strength where the noise can have an stabilisation effect as well. A more detailed investigation of such context will left for future research. \begin{figure}[h!] \centering \begin{subfigure}{0.45\textwidth} \includegraphics[width=\linewidth]{Figures/Fig_DG_2_grid_6_det.pdf} \caption{Deterministic} \label{fig:sub1} \end{subfigure} \begin{subfigure}{0.45\textwidth} \includegraphics[width=\linewidth]{Figures/Fig_DG_2_grid_6.pdf} \caption{First realization with noise $\mu=0.1$} \label{fig:sub2} \end{subfigure} \begin{subfigure}{0.45\textwidth} \includegraphics[width=\linewidth]{Figures/Fig_DG_2_grid_6_2.pdf} \caption{Second realization with noise $\mu=0.1$} \label{fig:sub3} \end{subfigure} \begin{subfigure}{0.45\textwidth} \includegraphics[width=\linewidth]{Figures/Fig_DG_2_grid_6_2_strong.pdf} \caption{Second realization with noise $\mu=0.25$} \label{fig:sub4} \end{subfigure} \caption{Kelvin-Helmholtz and influence of the noise } \label{fig:main} \end{figure} \section{Conclusion and outlook}\label{se_outlock} In this work, we have extended the entropy-dissipative DGSEM methods including the FV case to solve the stochastic Euler system, demonstrating convergence and providing error estimates across various settings. Our analysis builds on the consistency framework from \cite{LuOe} and the theoretical results in \cite{Mo}, leading to a first convergence result for the stochastic Euler equations. Additionally, we implemented these methods and examined error behavior across multiple experiments as a proof of concept.\\ We have seen that the method is stable and for sufficient smooth solutions (for the deterministic case), we obtain even better convergence rates than we would have expected. This is also in line with the purely deterministic framework where several optimal error estimations for different DG settings are known as references for instance in \cite{zbMATH06698849,Jiao2022,Liu2020Liu,zbMATH07086311}. However, our analysis is relying on the consistency estimation where no regularity have been assumed. In the future, we plan to derive new error estimates using the consistency analysis and transfer it also from the purely deterministic setting to the stochastic one. Meanwhile, we have numerically analyzed the stochastic Euler equations using LLF-FV and DGSEM methods. We observe that LLF-FV methods exhibit error behavior in the stochastic setting similar to that derived in \cite{LuSh} for the deterministic case. Furthermore, our testing of the DGSEM method shows that, with smooth initial data, numerical results resemble those of deterministic case. However, for cases involving discontinuities, we observe oscillations, indicating a need for the development of limiting strategies including stochasticity to address more complex test cases where a first step in this direction was recently done in \cite{Wo}. Future research will focus on these limiting strategies, along with turbulence modeling aspects. \section*{Acknowledgement} The work of D.B. was supported by the German Research Foundation (DFG) within the framework of the priority research program SPP 2410 under the grant BR 4302/3-1 (525608987) and under the personal grant BR 4302/5-1 (543675748).\\ P.Ö. was supported by the DFG within SPP 2410, project OE 661/5-1 (525866748) and under the personal grant OE 661/4-1(520756621).\\ P.Ö. thanks Hendrik Ranocha (JGU Mainz) for his assistance in implementing the stochastic forcing term in Trixi.jl. \section*{Conflict of Interest} The authors have no conflict of interest to declare. \bibliographystyle{abbrv} \bibliography{literature} \end{document}
2412.07667v1
http://arxiv.org/abs/2412.07667v1
A symbolic computational approach to the generalized gambler's ruin problem in one and two dimensions
\documentclass[reqno]{amsart} \usepackage[utf8]{inputenc} \usepackage{xcolor,fullpage,hyperref,cleveref,enumitem} \usepackage{tikz,amsmath,amssymb,color,mathtools} \usetikzlibrary{calc} \usepackage{float} \usepackage{pgf} \usepackage[backrefs]{amsrefs} \usepackage{amsthm} \usepackage{listings} \lstset{frame=tb, language=Python, aboveskip=3mm, belowskip=3mm, showstringspaces=false, columns=flexible, basicstyle={\small\ttfamily}, numbers=none, numberstyle=\tiny\color{gray}, keywordstyle=\color{blue}, commentstyle=\color{dkgreen}, stringstyle=\color{mauve}, breaklines=true, breakatwhitespace=true, tabsize=3 } \newcommand{\norm}[1]{\left\lVert#1\right\rVert} \theoremstyle{definition} \newtheorem{theorem}{Theorem}[section] \newtheorem{corollary}{Corollary}[theorem] \newtheorem{proposition}{Proposition}[theorem] \newtheorem{problem}{Problem} \newtheorem{example}{Example} \newtheorem{exercise}{Exercise} \newtheorem{lemma}[theorem]{Lemma} \newtheorem{conjecture}[theorem]{Conjecture} \newtheorem{guess}[theorem]{Guess} \theoremstyle{definition} \newtheorem{definition}{Definition}[section] \theoremstyle{remark} \newtheorem*{remark}{Remark} \newtheorem{observation}[theorem]{Observation} \newtheorem*{question}{Question} \newcommand{\NN}{\mathbb{N}} \newcommand{\PP}{\mathbb{N}^+} \newcommand{\ZZ}{\mathbb{Z}} \DeclarePairedDelimiter\ceil{\lceil}{\rceil} \DeclarePairedDelimiter\floor{\lfloor}{\rfloor} \newcommand{\seqnum}[1]{\href{https://oeis.org/#1}{\rm \underline{#1}}} \usepackage{thmtools} \usepackage{thm-restate} \declaretheorem[name=Theorem,numberwithin=section]{thm} \newcommand{\tcr}[1]{\textcolor{red}{#1}} \newcommand{\tcb}[1]{\textcolor{blue}{#1}} \newcommand{\lucy}[1]{{\bf\color{magenta}Lucy: #1}} \title{A symbolic computational approach to the generalized gambler's ruin problem in one and two dimensions} \author[Martinez]{Lucy Martinez} \address[L.~Martinez]{Department of Mathematics, Rutgers University, Piscataway, NJ 08854} \email{\textcolor{blue}{\href{mailto:[email protected]}{[email protected]}}} \begin{document} \begin{abstract} The power of symbolic computation, as opposed to mere numerical computation, is illustrated with efficient algorithms for studying the generalized gambler's ruin problem in one and two dimensions. We also consider a new generalization of the classical gambler's ruin where we add a third step which we call the mirror step. In this scenario, we provide closed formulas for the probability and expected duration. \end{abstract} \maketitle \section{Introduction} Throughout we let $x$ be some positive integer such that $0< x < N$ where $N\in \NN=\{1,2,3,\ldots \}$. Consider a gambler who starts with $x$ dollars. At each gamble, the gambler either wins a dollar with probability $\frac{1}{2}$ or loses a dollar with probability $\frac{1}{2}$. The gambler's goal is to reach $N$ dollars without first running out of money (i.e., hitting $0$ dollars). If the gambler reaches $N$ dollars, we say that they are a \textit{winner}. The gambler continues to play until they either run out of money or win. This scenario is known as the \textit{gambler's ruin problem}, first posed by Pascal in 1656 in a letter to Fermat, as noted by Edwards \cite{Edwards1983}. In 1657, Christiaan Huygens restated the problem and published a solution for the probability of winning~\cite{Huygens}. For additional historical context, we refer the reader to a paper of Seongjoo Song and Jongwoo Song~\cite{Song}. In this paper, we begin by providing an overview of the classical gambler's ruin problem, recalling results for both the probability of winning and the expected duration of the game. We also summarize analogous results on the generalized $1$-dimensional and $2$-dimensional versions of the gambler's ruin problem. Building on the $1$-dimensional version, we introduce a new generalization of the classical gambler's ruin game that includes an additional step called the \textit{mirror step}. For the generalized model, we derive formulas for the probability of winning and expected duration of the game. The objective of this paper is to propose an approach to reduce the computational running time required to determine the probability of winning and the expected duration specifically for the generalized 1-dimensional and 2-dimensional versions. \subsection{Classical gambler's ruin problem}\label{subsec:classical} Let $f(x)$ be the probability that the gambler exits the game as a winner starting with $x$ dollars. For $0<x<N$, this probability satisfies the recurrence relation \begin{equation} \label{eq:probclass} f(x)=\frac{1}{2}f(x-1)+\frac{1}{2}f(x+1), \quad f(0)=0, f(N)=1. \end{equation} That is, if the gambler starts with $x$ dollars, then in the next round, the gambler has $x-1$ dollars or $x+1$ dollars, each with probability $\frac{1}{2}$. Using this recurrence relation and the boundary conditions, we can find the solution to be $f(x)=\frac{x}{N}$. If the gambler starts with $x$ dollars, let $g(x)$ be the expected number of steps (expected duration of the game) the gambler takes to exit the game (either with $N$ dollars or $0$ dollars). Similar to the probability, for $1<x<N$, \[g(x)=\frac{1}{2}g(x-1)+\frac{1}{2}g(x+1) +1, \quad g(0)=0, g(N)=0.\] At each round, if the gambler has $x$ dollars, then in the next round, the gambler will have either $x-1$ dollars or $x+1$ dollars, each with probability $\frac{1}{2}$. However, we add 1 to the count since the gambler has taken one extra step. Using the recurrence relation and the boundary conditions, we can find the solution to be $g(x)=x(N-x)$. Building upon the expected duration, we can obtain the probability generating function of the duration of the game. For a formal variable $t$ and $0<x<N$, \[F(x,t)=t\left(\frac{1}{2}F(x-1,t)+\frac{1}{2}F(x+1,t)\right), \quad F(0,t)=1, F(N,t)=1.\] Taking the derivative of $F(x,t)$ with respect to $t$, and evaluating at $t=1$ recovers the expected duration of the game at $x$. Consider extending the game so that the probability of losing one dollar or winning one dollar are not the same. In other words, let $p$ be the probability of winning one dollar, and $q=1-p$ be the probability of losing one dollar. Let $f(x)$ be the probability of exiting the game as a winner starting with $x$ dollars. Similarly to \Cref{eq:probclass}, we get \[ f(x)=qf(x-1)+pf(x+1), \quad f(0)=0, f(N)=1, \text{ and } p+q=1. \] Edwards gives a conjecture on how Pascal solved the above using a method of recursive formula \cite{Edwards1983}. We provide Edwards' solution to $f(x)$. Rewrite $f(x+1)-f(x)$ and observe the following \begin{equation*} f(x+1)-f(x)=\frac{q}{p}\left(f(x)-f(x-1)\right)=\frac{q^2}{p^2}\left(f(x-1)-f(x-2)\right)=\cdots= \frac{q^i}{p^i}\left(f(1)-f(0)\right). \end{equation*} Hence, \begin{align*} f(x)&= \left ( \sum_{j=0}^{x-1}\left(\frac{q}{p}\right)^j \right ) f(1). \end{align*} Since $p=1-q$ then $p\neq q$, it follows that $\frac{q}{p}\neq 1$. Therefore, by the geometric series, we obtain \begin{align*} 1=f(N)&=\frac{1-\left(\frac{q}{p}\right)^N}{1-\frac{q}{p}} \cdot f(1). \end{align*} Thus, we can recover the following \begin{align*} f(1)=\frac{1-\frac{q}{p}}{1-\left(\frac{q}{p}\right)^N} \quad \text{and} \quad f(x)=\frac{1-\left(\frac{q}{p}\right)^x}{1-\left(\frac{q}{p}\right)^N}. \end{align*} \subsection{Generalized $1$-dimensional gambler's ruin} The gambler's ruin problem can be formulated as follows: A particle starts at a point $x$ on a line of length $N$ where $0<x<N$. The particle moves to the left from $x$ to $x-1$ with probability $\frac{1}{2}$, or to the right from $x$ to $x+1$ with probability $\frac{1}{2}$. Consider extending the 1-dimensional gambler's ruin game to include more than two steps on a line of length $N$. Let $r$ be a positive integer. Let $a_1,a_2,\ldots , a_r$ be distinct integers such that $a_1<a_2<\cdots<a_r$ where $a_1<0$ and $a_r>0$. Let $p_1, p_2,\ldots,p_r$ be probabilities such that $p_1+p_2+\ldots + p_r=1$, and let $P$ be the probability table $P=[[a_1,p_1], [a_2,p_2],\ldots , [a_r,p_r]]$ where each pair $[a_i,p_i]$ represents the outcome $a_i$ occurring with probability $p_i$. The generalized 1-dimensional gambler's ruin problem states that if a particle is currently at some $x$ then the particle moves from $x$ to $x+a_1$ with probability $p_1$, or moves from $x$ to $x+a_2$ with probability $p_2$, or moves from $x$ to $x+a_3$ with probability $p_3$, and so on. Similarly to the method of solving a system of linear equations as in \Cref{subsec:classical}, we can obtain the probability of winning and the expected duration for the generalized case for any starting position. However, as $N$ grows, the computation time to solve a system of $N$ linear equations will be slower. In \Cref{sec:general 1d}, we present a faster method to reduce the computational running time by going from a system of $N-1$ linear equations with $N-1$ unknowns to a system of $a_r$ linear equations with $a_r$ unknowns where $a_r$ is the maximum of the steps in the probability table $P=[[a_1,p_1], [a_2,p_2],\ldots , [a_r,p_r]]$. Our method significantly drops the computational running time and we make comparisons between the direct approach and our strategy in \Cref{subsec:time comparison in 1d}. \subsection{$2$-dimensional gambler's ruin}\label{subsec:2 dim} Let $M$ and $N$ be positive integers. Consider a particle starting at a point $(x,y)$ in the interior of a rectangular grid of size $M\times N$, where $0<x<M$ and $0<y<N$. At each step, the particle moves in one of four directions, each with probability $\frac{1}{4}$: $(x-1,y), (x,y+1), (x+1,y),(x,y-1)$. The particle stops moving once it hits one of the four boundaries, defined by $x=0, x=M, y=0$, or $y=N$. By setting up a recurrence relation for the expected duration, we can obtain a system of $(M-1)\times(N-1)$ linear equations with $(M-1)\times(N-1)$ unknowns. Andrej Kmet and Marko Petkov\v{s}ek gave an explicit formula involving a double sum, enabling direct computation of the expected duration for the $2$-dimensional gambler's ruin game without the need to solve systems of equations or use recursion \cite{Kmet}. While Kmet and Petkov\v{s}ek's formula expresses the expected duration as a double sum, our method reduces the computational running time by going from a system of $(M-1)\times(N-1)$ linear equations with $(M-1)\times(N-1)$ unknowns to a system of $N-1$ linear equations and $N-1$ unknowns. Our method is significantly faster than the direct approach and Kmet and Petkov\v{s}ek' formula. We make comparisons in \Cref{subsec:time comparison in 2d}. One way to generalize the $2$-dimensional game is to change the probabilities of each of the four directions with probabilities $p_L, p_U, p_R,$ and $p_B$, corresponding to left, up, right and down movements, respectively where $p_L+p_U+p_R+p_B=1$. Although one can generalize the number of steps for either of the four directions, we focus on the case when the set of steps the particle can move is $\{[0,1],[0,-1],[1,0],[-1,0]\}$, and remark that one can adapt our strategy for an {\it arbitrary} (finite) set of allowed steps, and arbitrary probability distribution. \subsection{A mirror step variant of gambler's ruin}\label{subsec:mirror} We consider a new generalization of the gambler's ruin problem. A particle starts at some point $x$ on a line of length $N$ where $0< x <N$. At each step, the particle either moves from $x$ to $x-1$ with probability $q_1$, or moves from $x$ to $x+1$ with probability $q_2$, or moves from $x$ to $N-x$ with probability $p$ where $0<p<1$ and $q_1+q_2+p=1$. We call this last step the \textit{mirror step}. The particle continues to walk on the line until it reaches $0$ or $N$. In this paper, we focus on the case when $q_1=q_2=\frac{1-p}{2}$ and we call this the \textit{symmetric case}. We begin with an example where the particle starts at $x=1$ and generate data for different $p$ values with fixed $N$. \begin{example}\label{ex: x=1 prob in intro} Let $N=100$ and $x=1$. We generate data for the probability that if the particle is currently at $x=1$, the particle eventually ends at $100$. Let $p\in \{\frac{1}{2},\frac{1}{3},\frac{1}{4},\ldots, \frac{1}{10}\}$. We use the procedure \texttt{Lk(p,x,N)}, as described in \Cref{appendix:mirror} in Maple which generates the following data in about $4.765$ seconds: \begin{align*} T \coloneqq &[0.4142135624, 0.3660254038, 0.3333333333, 0.3090169944, 0.2898979486, 0.2742918852, 0.2612038750, \\ &0.2500000000,0.2402530734]. \end{align*} \noindent The sequence $T$ reads as follows. If the particle is currently at $x=1$ and $p=\frac{1}{2}$, the particle moves from $x$ to $x-1$ with probability $\frac{1-p}{2}=\frac{1}{4}$, or moves from $x$ to $x+1$ with probability $\frac{1-p}{2}=\frac{1}{4}$, or moves from $x$ to $100-x$ with probability $p=\frac{1}{2}$. Then, the probability of the particle starting at $x=1$ and ending at $100$ is $T_1=0.4142135624$. Similarly, if the particle is currently at $x=1$ and $p=\frac{1}{3}$, the particle moves from $x$ to $x-1$ with probability $\frac{1-p}{2}=\frac{1}{3}$, or moves from $x$ to $x+1$ with probability $\frac{1-p}{2}=\frac{1}{3}$, or moves from $x$ to $100-x$ with probability $p=\frac{1}{3}$. Then, the probability of the particle starting at $x=1$ and ending at $100$ is $T_2=0.3660254038$. Thus, $T_i$ is the probability of the particle starting at $x=1$ and ending at $100$ for $p=\frac{1}{i+1}$ where $1\leq i\leq 9$. We then use the function \texttt{identify} in Maple on the sequence $T$. The function \texttt{identify} is based, in part, on the continued fraction expansion of any given numerical value. Using this function on the values of $T$, we conjecture that each of the probabilities in $T$ converges to \begin{align*} M\coloneqq&\left[\sqrt{2}-1, \frac{\sqrt{3}-1}{2}, \frac{1}{3}, \frac{\sqrt{5}-1}{4}, \frac{\sqrt{6}-1}{5}, \frac{\sqrt{7}-1}{6},\frac{2 \sqrt{2}-1}{7}, \frac{1}{4}, \frac{\sqrt{10}-1}{9} \right]. \end{align*} That is, the probability of the particle starting at $x=1$ and ending at $100$ converges to $M_i$ for $p=\frac{1}{i+1}$ where $1\leq i\leq 9$. \end{example} The previous example illustrates that when the particle starts at $x=1$, the probability of ending at $N$ converges fast. Denote this probability by $f_N^{(p)}(x)$. We state the following guess for $x=1$ and in \Cref{cor:probability at infinity} we provide a proof. \begin{guess}\label{guess:x=1} If the particle starts at $x=1$, then \[\lim_{N\to\infty}f_N^{(p)}(1)=\frac{\sqrt{p}-p}{1-p}.\] \end{guess} In \Cref{sec:mirror step}, we provide other expressions for $\lim_{N\to \infty} f_N^{(p)}(x)$ when $x=2$ and $x=N-2$ with fixed $N$, and in \Cref{cor:probability at infinity} we provide the general formula for the limit. This paper is structured as follows. In Section \ref{sec:general 1d}, we present a new approach to compute the probability of winning and the expected duration of the game that reduces the computational running time for the generalized $1$-dimensional gambler's ruin \footnote{All computations were performed using Maple on a laptop with an Intel Core i7-10510U processor (4 cores, 8 logical processors) and 8 GB of RAM.}. In Section \ref{sec:2d case}, we provide the analogous approach for the generalized $2$-dimensional case and compare the computational running times to a formula provided by Andrej Kmet and Marko Petkov\v{s}ek. In Section \ref{sec:mirror step}, we consider a mirror step variant of gambler's ruin and provide closed formulas for both the probability of winning and the expected duration of the game. We conclude with future directions in Section \ref{sec:future}. \section{Generalized $1$-dimensional Gambler's Ruin}\label{sec:general 1d} In this section, we introduce the recurrence relation for the probability of winning in the generalized $1$-dimensional gambler's ruin game, and introduce symbolic variables to the recurrence equation of the probability and expected duration. Recall that $P=[[a_1,p_1], [a_2,p_2],\ldots , [a_r,p_r]]$ denotes a probability table, where each pair $[a_i,p_i]$ represents the outcome $a_i$ occurring with probability $p_i$. To set up notation, we start with an example. \begin{example}\label{ex:probability notation} Let $N=5$ and $P$ be the probability table given by $P=\left[[-2,\frac{1}{2}],[1,\frac{1}{4}],[2,\frac{1}{4}]\right]$. If the particle starts at some $x$ where $0<x<5$ on the line of length $5$, then it can move along the line as follows: from $x$ to $x-2$ with probability $\frac{1}{2}$, or from $x$ to $x+1$ with probability $\frac{1}{4}$, or from $x$ to $x+2$ with probability $\frac{1}{4}$. \end{example} \subsection{Probability}\label{subsec:prob} We now establish the recurrence relation for the probability that the particle reaches some position $\geq N$ starting from an initial position $x$. We then rewrite this recurrence and introduce new variables for the probabilities at each $x$. Define $f(x)$ as the probability that a particle starting at $x$ will eventually reach a position $\geq N$. For $0<x<N$, this probability satisfies the recurrence relation \begin{equation}\label{eq:generalprob} f(x)=\sum_{i=1}^r p_if(x+a_i), \end{equation} where $a_i\in \ZZ$, $a_1<a_2<\cdots <a_r$, $a_1<0$ and $a_r>0$. Unlike the classical gambler’s ruin problem, the generalized 1-dimensional scenario has more than two boundary conditions. Certainly $f(0)=0$ and $f(N)=1$. Since $a_1< a_2< \cdots< a_r$, the values $a_1$ and $a_r$ represent the minimum and maximum of all the integers $a_i$, respectively. For any integer $k$ such that $a_{1}+1 \leq k \leq 0$, it follows that $f(k)=0$. Indeed, if the particle is at $x=1$, it may move to $x+a_1=a_1+1$ with probability $p_1$. Given that $a_1<0$, this movement brings the particle to some position $k\leq 0$, implying that $f(k)=0$ for $k=a_1+1, a_1+2, \ldots, -1,0$. Similarly, for any integer $\ell$ such that $N \leq \ell \leq N+a_r-1$, we have $f(\ell)=1$. If the particle is at $x=N-1$, it may move to $x+a_r=N+a_r-1$ with probability $p_r$. Since $a_r>0$, this movement brings the particle to some position $\ell\geq N$, so $f(\ell)=0$ for $\ell=N,N+1,N+a_r-2,N+a_r-1$. Thus, there are $a_r-a_1$ boundary conditions. \begin{example}\label{ex:slow version N=5}(Continuing \Cref{ex:probability notation}) Recall the probability table $P=\left[[-2,\frac{1}{2}],[1,\frac{1}{4}],[2,\frac{1}{4}]\right]$ and $N=5$. If $f(x)$ denotes the probability that the particle reaches some position $\geq 5$, then \begin{equation*} f(x)=\frac{1}{2}f(x-2)+\frac{1}{4}f(x+1)+\frac{1}{4}f(x+2) \end{equation*} with initial and final conditions $f(-1)=f(0)=0$ and $f(5)=f(6)=1$. \noindent This setup results in a system of 4 linear equations for $0<x<5$, \begin{align*} f(1)&=\frac{1}{2}f(-1)+\frac{1}{4}f(2)+\frac{1}{4}f(3)\\ f(2)&=\frac{1}{2}f(0)+\frac{1}{4}f(3)+\frac{1}{4}f(4)\\ f(3)&=\frac{1}{2}f(1)+\frac{1}{4}f(4)+\frac{1}{4}f(5)\\ f(4)&=\frac{1}{2}f(2)+\frac{1}{4}f(5)+\frac{1}{4}f(6). \end{align*} Solving for the unknowns using the boundary conditions yields $f(1)= \frac{1}{5}, f(2)=\frac{13}{45}, f(3)=\frac{23}{45}, f(4)=\frac{29}{45}$. \end{example} Although this direct method works for small values of $N$, solving the system of $N-1$ equations becomes computationally expensive as $N$ grows. To address this, we rewrite the recurrence relation \Cref{eq:generalprob} as \begin{equation}\label{eq:bettergeneralprob} f(x)=\frac{1}{p_r}f(x-a_r)-\frac{1}{p_r}\sum_{i=1}^{r-1}p_if(x+a_i-a_r) \end{equation} obtained by the change of variables $x\mapsto x-a_r$. The boundary conditions remain, $f(a_1+1)= f(a_1+2)= \ldots= f(-1)=f(0)=0$ and $f(N)=f(N+1)=\ldots = f(N+a_r-2)= f(N+a_r-1)=1$. For each $1\leq j \leq a_r$ define $d_j=f(j)$ and construct the set $S=\{d_1,d_2,\ldots, d_j\}$ where $a_r$ is the maximum of the $a_i$'s. Using these variables, we express \begin{align}\label{eq:linear equations for better recurrence} f(a_r+1),f(a_r+2),\ldots, f(N+a_r-1) \end{align} as linear combinations of the elements in $S$. Observe that $a_1<a_2<\cdots<a_r$ implies $a_i-a_r<0$ for any $1\leq i \leq r-1$. Hence, $x+a_i-a_r<x\leq N+a_r-1$ for all $x\in \{a_r+1, a_r+2,\ldots, N+a_r-1\}$ since $0<a_r+1<a_r+2< \cdots< N+a_r-1$. Thus, $x+a_i-a_r< N+a_r-1$. Also, for any $1\leq i\leq r-1$, $x+a_i-a_r>x+a_1-a_r$ since $a_i> a_1$. Now, $x+a_i-a_r> x+a_1-a_r\geq a_r+1+a_1-a_r=a_1+1$ since $0<a_r+1<a_r+2<\cdots < N+a_r-1$. It follows that $x+a_i-a_r> a_1+1$. Combining the above, we get $a_1+1<x+a_i-a_r<N+a_r-1$. This implies that $f(x+a_i-a_r)$ depends only on the following terms \begin{align*} &f(a_1+1),f(a_1+2),\ldots, f(0),\\ &f(1), f(2), \ldots, f(a_r),\\ &f(a_r+1),f(a_r+2),\ldots, f(N-1)\\ &f(N), f(N+1), f(N+2), \ldots , f(N+a_r-1). \end{align*} We know $0=f(a_1+1)=f(a_1+2)=\cdots =f(0)$, $1=f(N)= f(N+1)=f(N+2)= \cdots =f(N+a_r-1)$ and $f(1)=d_1,f(2)=d_2,\ldots,f(j)=d_j$ where $j=a_r$. For any $a_r+1\leq x \leq N-1$, $f(a_r+1)$ is a linear combination of $f(1)$ and $f(a_1+1), f(a_2)+1,\ldots, f(a_{r-1}+1)$. Recall that $a_1+1\leq a_2, a_2+1\leq a_3, \ldots, a_{r-1}+1\leq a_r$. Hence, $f(a_r+1)$ depends on at most the expression $f(a_r)$, which is known. Thus, $f(a_r+1)$ is a linear combination of the elements in $S$. Since $f(x)$ is a recursive formula, for any $x>a_r+1$, $f(x)$ will be a linear combination of the variables in $S$. Simultaneously, we have $f(N)=f(N+1)=\ldots =f(N+a_r-2)=f(N+a_r-1)=1$. Therefore, $f(N), f(N+1), f(N+2), \ldots , f(N+a_r-1)$ are linear combinations of $\{d_1,d_2,\ldots,d_j\}$ where $j=a_r$ and are all equal to $1$. Hence, solving this system of $a_r$ equations with $a_r$ unknowns yields solutions for the variables in $S$ which provide solutions for the rest of the expressions, namely $f(a_r+1),f(a_r+2),\ldots, f(N-1)$. \begin{example}\label{ex:faster version N=5}(Continuing Example \ref{ex:slow version N=5}) Let $N=5$ and $P=[[-2,1/2],[1,1/4],[2,1/4]]$ as in \Cref{ex:slow version N=5}. Recall the boundary conditions: $f(-1)=f(0)=0$ and $f(5)=f(6)=1$. Note that $a_3=2$, the maximum of $\{-2,1,2\}$, introduces the new variables $\{d_1,d_2\}$ such that $f(1)=d_1$. Using the recurrence relation $f(x)=4f(x-2)-2f(x-4)-f(x-1)$, we construct the equations for $f(3), f(4) , f(5)$, and $f(6)$, with $r=3$: \begin{align*} f(-1)&=f(0)=0 \\ f(1)&=d_1\\ f(2)&=d_2 \\ f(3)&=4f(1)-2f(-1)-f(2)= 4d_1-d_2\\ f(4)&=4f(2)-2f(0)-f(3)=5d_2-4d_1 \\ f(5)&=4f(3)-2f(1)-f(4)= 18d_1-9d_1\\ f(6)&=4f(4)-2f(2)-f(5)=27d_2-34d_1. \end{align*} Since $f(5)=f(6)=1$, the system of equations $1= 18d_1-9d_1$ and $1=27d_2-34d_1$ has the solution $d_1=\frac{1}{5}$ and $d_2=\frac{13}{45}$. Substituting these values gives \[ f(1)=\frac{1}{5}, f(2)=\frac{13}{45}, f(3)=\frac{23}{45}, f(4)=\frac{29}{45}.\] These results agree with \Cref{ex:slow version N=5}. \end{example} \subsection{Expected duration}\label{subsec:exp} In the previous subsection, we considered the probability for the particle to reach some position $\geq N$. In this section, we consider the expected duration for the particle to end at a position $\leq 0$ or a position $\geq N$. Since this is analogous to the probability case, we omit the details. Define $g(x)$ as the expected number of steps that a particle starting at $x$ will eventually reach a position $\leq 0$ or a position $\geq N$. For $0<x<N$, this expected duration satisfies the recurrence relation \begin{equation}\label{eq:generalexp} g(x)=\sum_{i=1}^r p_ig(x+a_i)+1, \end{equation} where $a_i\in \ZZ$, $a_1<a_2<\cdots <a_r$, $a_1<0$, $a_r>0$ and boundary conditions $0=g(a_1+1)= g(a_1+2)=\cdots=g(-1)=g(0)$ and $0=g(N)=g(N+1)=\cdots =g(N+a_r-2)=g(N+a_r-1)$. Similar to the probability case, we rewrite \Cref{eq:generalexp} to \begin{equation}\label{eq:bettergeneralexp} g(x)=\frac{1}{p_r}g(x-a_r)-\frac{1}{p_r}\sum_{i=1}^{r-1}p_ig(x+a_i-a_r)-\frac{1}{p_r} \end{equation} where $0=g(a_1+1)= g(a_1+2)=\cdots=g(-1)=g(0)$ and $0=g(N)=g(N+1)=\cdots =g(N+a_r-2)=g(N+a_r-1)$. Using \Cref{eq:bettergeneralexp} reduces the computational running time for the expected duration of the game as we will see in the next subsection. \subsection{Comparison between the slower method and the faster method}\label{subsec:time comparison in 1d} In this subsection, we compare the computational running times in Maple between the classical approach to solving $N-1$ linear equations and the faster method introduced in the previous section. Specically, we evaluate the performance of \lstinline{GR1dLG} and \lstinline{NewGR1dLG}, as described in \Cref{appendix:ggr1d}, Using the commands \lstinline{time(GR1dLG)} and \lstinline{time(NewGR1dLG)}, we measure the execution time for varying values of $N$ with $P=[[-1,\frac{1}{3}],[1,\frac{1}{3}],[2,\frac{1}{3}]]$. The measure timed in Maple are summarized below: \begin{table}[H] \centering \begin{tabular}{|c|c|c|c|} \hline $N$ & Slow Method & Faster Method\\ &(seconds) & (seconds)\\ \hline $100$ & $0.796$ & $0.015$ \\ \hline $105$ & $11.171$ & $0.015$ \\ \hline $110$ & $21.093$ & $0.015$ \\ \hline $115$ & $500.828$ & $0.015$ \\ \hline $120$ & $2466.609$ & $0.046$ \\ \hline \end{tabular} \end{table} The data in the table suggests that introducing symbolic variables in \Cref{eq:bettergeneralexp} dramatically reduces the running time for computing the expected durations of the game starting at all starting locations. \subsection{Variance} In this subsection, we consider the variance of the duration of the generalized $1$-dimensional gambler's ruin game. The computation builds up the expected duration discussed in \Cref{subsec:exp}. Let $P=[[a_1,p_1],[a_2,p_2],\ldots,[a_r,p_r]]$ such that $p_1+\ldots+p_r=1$ and define $F(x,t)$ as the probability generating function of the duration of the generalized $1$-dimensional gambler's ruin game. For $0<x<N$, this functions satisfies the recurrence relation \begin{equation*} F(x,t)=t(p_1F(x+a_1,t)+p_2F(x+a_2,t)+\cdots+p_rF(x+a_r,t)), \end{equation*} where $F(0,t)=1$ and $F(N,t)=1$. Making the substitution $t\mapsto z+1$ yields \begin{equation}\label{eq:1d gen fun in z} F(x,z)=(z+1)(p_1F(x+a_1,z)+p_2F(x+a_2,z)+\cdots+p_rF(x+a_r,z), \end{equation} with $F(0,z)=1$ and $F(N,z)=1$. We derive an expression to estimate the second factorial moment. Expanding $F(x,z)$ as a Taylor series gives \begin{equation}\label{eq:taylor in 1d} F(x,z)=1+g(x)z+\frac{h(x)}{2!}z^2+\cdots \end{equation} where $g(x)$ is the expected duration as defined as in \Cref{subsec:exp}, and $h(x)$ represents the second factorial moment at $x$. Substituting \Cref{eq:taylor in 1d} into \Cref{eq:1d gen fun in z} results in \begin{align*} 1+g(x)z+\frac{h(x)}{2}z^2 + \dots &=(1+z)\left( p_1\left(1+g(x+a_1)z+\frac{h(x+a_1)}{2}z^2 +\cdots \right)\right)\\ &+(1+z)\left(p_2\left(1+g(x+a_2)z+\frac{h(x+a_2)}{2}z^2+\cdots\right)\right)\\ &\quad \vdots\\ &+(1+z)\left(p_r\left(1+g(x+a_r)z+\frac{h(x+a_r)}{2}z^2+\cdots\right)\right). \end{align*} \noindent From the previous expression, we extract the coefficient of $z^2$ to get an expression for $h(x)$. Hence, \begin{equation*} h(x)-(p_1h(x+a_1)+p_2h(x+a_2)+\cdots + p_rh(x+a_r))=2(p_1g(x+a_1)+p_2g(x+a_2)+\cdots+p_rg(x+a_r)) \end{equation*} where $0=g(a_1+1)= g(a_1+2)=\cdots=g(-1)=g(0)$ and $0=g(N)=g(N+1)=\cdots =g(N+a_r-2)=g(N+a_r-1)$. The sum $g(x)+h(x)$ gives the second moment for $0<x<N$. \noindent The variance at $x$, denoted $V(x)$, is computed as \[ V(x)=g(x)+h(x)-(g(x))^2\] where $g(x)$ and $h(x)$ are defined as follows \begin{align*} &g(x)=\frac{1}{p_r}g(x-a_r)-\frac{1}{p_r}\sum_{i=1}^{r-1}p_ig(x+a_i-a_r)-\frac{1}{p_r} \intertext{and} &h(x)-\sum_{i=1}^{r}p_ih(x+a_i)=2\sum_{i=1}^rp_ig(x+a_i). \end{align*} We conclude this section with a table comparing the expected duration and the standard deviation when the particle starts at $x=\frac{N}{2}$ for various $N$ and probability table $P=[[-2,\frac{1}{3}],[1,\frac{1}{3}],[2,\frac{1}{3}]]$ . \begin{table}[H] \centering \begin{tabular}{|c|c|c|c|} \hline $N$ & $x$ & Expected Duration& Standard Deviation \\ \hline $10$ & $5$ & $ 8.613479400$ & $ 6.321808669$\\\hline $20$ & $10$ & $ 25.23344696$ & $18.44137538$\\ \hline $30$ & $15$ & $42.94261730$ & $ 29.22243692$\\ \hline $40$ & $20$ & $59.58246747$ & $ 37.26482832$ \\ \hline $50$ & $25$ & $75.36543964$ & $ 43.30155080$\\ \hline $60$ & $30$ & $90.70157954$ & $48.13687964$\\ \hline $70$ & $35$ & $105.8379590$ & $52.27336865 $\\ \hline $80$ & $40$ & $120.8913756$ & $55.98253147$\\ \hline $90$ & $45$ & $135.9117966$ & $59.40593597$\\ \hline $100$ & $50$ & $150.9194653$ & $62.61931702$ \\ \hline \end{tabular} \end{table} \section{$2$-Dimensional Gambler's Ruin}\label{sec:2d case} In this section, we consider the $2$-dimensional gambler's ruin and provide analogous results to the probability, expected duration and variance discussed in \Cref{sec:general 1d}. We introduce symbolic variables to the recurrence equation of the probability and expected duration. We then compare the computational running times in Maple between our method and the formula provided by Kmet and Petkov\v{s}ek. We conclude this section with an analysis of the variance. We begin by recalling the setup for the $2$-dimensional case as described in \Cref{subsec:2 dim}. Consider a particle starting at a point $(x,y)$ in the interior of a rectangular grid of size $M\times N$, where $0<x<M$ and $0<y<N$. At each step, the particle moves in one of four directions, with probabilities $p_L, p_U, p_R,$ and $p_B$, corresponding to left, up, right and down movements, respectively. The particle stops moving once it hits one of the four boundaries, defined by $x=0, x=M, y=0$, or $y=N$. \subsection{Probability} We begin by considering the recurrence relation for the probability that the particle reaches some position at one of the four boundaries. We then rewrite this recurrence and introduce new variables for the probabilities at each $x$ and $y$. Define $f(x,y)=f_L(x,y)L+f_R(x,y)R+f_U(x,y)U+f_D(x,y)D$, where $f_L(x,y)$ is the probability that the particle, starting at $(x,y)$ will exit the rectangle on the left side, and analogously for $f_R(x,y),f_U(x,y),f_D(x,y)$, and $L,R,U,D$ are formal variables. For $0<x<M$ and $0<y<N$, this probability satisfies the recurrence relation \begin{equation}\label{eq:twodim prob} f(x,y)=p_Lf(x-1,y)+p_Uf(x,y+1)+p_Rf(x+1,y)+p_Bf(x,y-1) \end{equation} with boundary conditions $f(0,y)=L, f(x,N)=U, f(M,y)=R$ and $f(x,0)=B$ for all $0<x<M$ and $0<y<N$. Thus, the coefficients of $L, U, R,$ and $B$ in $f(x,y)$ represent the respective probabilities that a particle starting at $(x,y)$ will end on each boundary. For any $0<x<M$ and $0<y<N$, $f(x,y)$ can be determined by solving a system of $(M-1)\times(N-1)$ linear equations with $(M-1)\times(N-1)$ unknowns. In the next subsection, we will introduce symbolic variables into the recurrence relation for the expected duration, as outlined in \Cref{sec:general 1d}. This method, applied to Equation \ref{eq:twodim prob} , but the same approach works for efficient computations of probabilities. This is further detailed in \Cref{appendix:ggr2d}. \subsection{Expected duration}\label{subsec:exp in 2d} We now consider the expected duration until the particle reaches some position at one of the four boundaries. We focus on the case when $p_W=p_N=p_E=p_S=\frac{1}{4}$, and remark that one can adapt our strategy for $p_W\neq p_N\neq p_E\neq p_S$ as described in \Cref{appendix:ggr2d}. Define $g(x,y)$ as the expected number of steps that a particle starting at $(x,y)$ will eventually take to reach a position at one of the boundaries. For $0<x<M$ and $0<y<N$, this expected duration satisfies the recurrence relation \begin{equation}\label{eq:2d expected duration} g(x,y)=\frac{1}{4}g(x-1,y)+\frac{1}{4}g(x,y+1)+\frac{1}{4}g(x+1,y)+\frac{1}{4}g(x,y-1)+1 \end{equation} with boundary conditions $g(0,y)=g(M,y)=g(x,0)=g(x,N)=0$ for all $0<x<M$ and $0<y<N$. Similarly to the probability case, $g(x,y)$ can be determined by solving a system of $(M-1)\times(N-1)$ linear equations with $(M-1)\times(N-1)$ unknowns. Orr and Zeilberger provided a solution that reduces the number of linear equations from $(M-1)\times(N-1)$ to $\mathcal{O}(N+M)$. Their approach exploits symmetry by solving $g(0,y)=g(M-1,y)$ for $0<y<N$, and $g(x,1)=g(x,N-1)$ for $0<x<M$ \cite{OrrDoron}. Kmet and Petkov\v{s}ek gave an explicit solution involving a double sum, enabling direct computation of the expected duration without the need to solve systems of equations or use recursion \cite{Kmet}. For the special case where $M=N$, they established the following result. \begin{theorem}[\cite{Kmet}, Equation (11)] Consider the 2-dimensional gambler's ruin problem as stated in this section. Then, the expected duration of the game when the particle starts at $(x,y)$ is given by \begin{equation*} g(x,y)=\frac{4}{M^2}\sum_{\substack{k=1 \\ k \text{ odd}}}^{M-1} \left(\sin{\left(\frac{jk\pi}{M}\right)}\cot{\left(\frac{k\pi}{2M}\right)} \sum_{\substack{\ell=1 \\ \ell \text{ odd}}}^{M-1} \frac{\sin{(\frac{i\ell\pi}{M})}\cot{(\frac{\ell\pi}{2M})}}{\sin^2{(\frac{k\pi}{2M})}+\sin^2{(\frac{\ell \pi}{2M})}}\right) \end{equation*} for any $0\leq x,y\leq M$. \end{theorem} We propose an alternative approach to those of Orr and Zeilberger, and Kmet and Petkov\v{s}ek. By rewriting \Cref{eq:2d expected duration}, we obtain the recurrence \begin{equation}\label{eq:2d new expected recurrence} g(x,y)=4g(x-1,y)-g(x-1,y-1)-g(x-1,y+1)-g(x-2,y)-4 \end{equation} where $x\to x-1$. The boundary conditions remain $g(0,y)=g(M,y)=g(x,0)=g(x,N)=0$ for all $x,y$. For each $1\leq j \leq N-1$ define $d_j=g(1,j)$ and construct the set $S=\{d_1,d_2,\ldots, d_{N-1}\}$. Using these variables, we reduce the system of equations. Initially, $g(x,y)$ is solved by constructing a system of $(M-1)\times(N-1)$ linear equations with $(M-1)\times(N-1)$ unknowns. By using $S$, we reformulate the system into: \begin{itemize}[leftmargin=0.2 in] \item $N-1$ equations for $x=1$, \item $(M-2)\times(N-1)$ equations for $2\leq x\leq M-1$, and \item $N-1$ boundary equations for $x=M$. \end{itemize} The boundary equations, derived using \Cref{eq:2d new expected recurrence}, are expressed in terms of the variables in $S$ and reduce the system to $N-1$ boundary equations with $N-1$ unknowns. Once these $N-1$ variables are solved, the remaining $(M-2)\times(N-1)$ equations can be determined. We illustrate this process in \Cref{ex:new approach}. \begin{example}\label{ex:new approach} Let $M=N=3$. This system consists of $4$ linear equations with $2$ unknowns: $d_1=g(1,1)$ and $d_2=g(1,2)$. The boundary conditions are $g(0,y)=g(M,y)=g(x,0)=g(x,N)=0$ for all $0\leq x,y\leq 3$. Using the recurrence in \Cref{eq:2d new expected recurrence}, we derive \begin{align*} d_1&=g(1,1)\\ d_2&=g(1,2)\\ g(2,1)&=4d_1-d_2-4\\ g(2,2)&=4d_2-d_1-4\\ g(3,1)&=0=16d_1-8d_2-16\\ g(3,2)&=0=16d_2-8d_1-16. \end{align*} Solving the last two equations yields $d_1=2$ and $d_2=2$. Substituting these values into the earlier equations gives $g(1,1)=g(1,2)=g(2,1)=g(2,2)=2$. \end{example} \subsection{Comparison between Kmet and Petkov\v{s}ek formula and faster method}\label{subsec:time comparison in 2d} We compare the computational running times in Maple between Kmet and Petkov\v{s}ek's formula for the expected duration and the faster method introduced in the previous subsection. Specifically, we evaluate the performance of \lstinline{NewGR2dL} and \lstinline{KmetPetkovsek}, as described in \Cref{appendix:ggr2d}. Using the commands \lstinline{time(evalf(NewGR2dL(M,M)))} and \lstinline{time(KmetPetkovsek(M)))}, we measure the execution time for varying values of $M$. The results provide a direct comparison of the efficiency of the two methods. The measured times in Maple are summarized below: \begin{table}[H] \centering \begin{tabular}{|c|c|c|c|c|c|c|} \hline $M$ & Faster Method & Kmet and Petkov\v{s}ek & $M$ & Faster Method & Kmet and Petkov\v{s}ek \\ &(seconds) & (seconds)& &(seconds) & (seconds)\\ \hline 10 & 0.015 & 0.015 & 90 & 9.843 & 181.953 \\ \hline 20 & 0.015 & 0.500 & 100 & 14.765 & 301.906\\ \hline 30 & 0.093 & 1.859 & 110 & 21.171 & 451.625\\ \hline 40 & 0.343 & 5.703 & 120 & 36.187 & 641.609\\ \hline 50 & 0.468 & 15.171 & 130 & 57.281 & 872.015 \\ \hline 60 & 1.171 & 35.125 & 140 & 80.140 & 1155.281\\ \hline 70 & 2.640 & 63.062 & 150 & 133.109 & 1564.515\\ \hline 80 & 5.062 & 114.234 & 160 & 226.312& 2023.125\\ \hline \end{tabular} \end{table} While Kmet and Petkov\v{s}ek's formula expresses the expected duration as a double sum, it is computationally less efficient compared to our method. \subsection{Variance} In this subsection, we analyze the variance of the duration of the $2$-dimensional gambler's ruin game under the condition $p_W=p_N=p_E=p_S=\frac{1}{4}$. The computation builds upon the expected duration discussed \Cref{subsec:exp in 2d}. Define $F(x,y,t)$ as the probability generating function of the duration of the $2$-dimensional gambler's ruin game. For $0<x<M$ and $0<y<N$, this function satisfies the recurrence relation \begin{equation}\label{eq:2d variance eq in t} F(x,y,t)=\frac{t}{4}(F(x-1,y,t)+F(x+1,y,t)+F(x,y-1,t)+F(x,y+1,t)) \end{equation} where $F(0,0,t)=1$ and $F(M,N,t)=1$. Making the substitution $t\mapsto z+1$ yields \begin{equation} F(x,y,z)=\frac{1+z}{4}(F(x-1,y,z)+F(x+1,y,z)+F(x,y-1,z)+F(x,y+1,z)), \end{equation} with $F(0,0,z)=1$ and $F(M,N,z)=1$. We derive an expression to estimate the second factorial moment. Expanding $F(x,y,z)$ as a Taylor series gives \begin{equation}\label{eq:taylor in 2d} F(x,y,z)=1+g(x,y)z+\frac{h(x,y)}{2!}z^2 +\cdots \end{equation} where $g(x,y)$ is the expected duration as defined in \Cref{subsec:exp in 2d}, and $h(x,y)$ represents the second factorial moment at $(x,y)$. Substituting \Cref{eq:taylor in 2d} into \Cref{eq:2d variance eq in t} and extracting the coefficient of $z^2$ yields an expression for $h(x,y)$: \[4h(x,y)-h(x-1,y)-h(x+1,y)-h(x,y-1)-h(x,y+1)=2(g(x-1,y)+g(x+1,y)+g(x,y-1)+g(x,y+1))\] where $g(0,y)=g(M,y)=g(x,0)=g(x,N)=0$ for all $x,y$. The sum $g(x,y)+h(x,y)$ gives the second moment for $0<x<M$ and $0<y<N$. The variance at $(x,y)$, denoted by $V(x,y)$, is computed as \[ V(x,y)=g(x,y)+h(x,y)-(g(x,y))^2\] where $g(x,y)$ and $h(x,y)$ are defined by \[g(x,y)=4g(x-1,y)-g(x-1,y-1)-g(x-1,y+1)-g(x-2,y)-4\] and \[ 4h(x,y)-h(x-1,y)-h(x+1,y)-h(x,y-1)-h(x,y+1)=2(g(x-1,y)+g(x+1,y)+g(x,y-1)+g(x,y+1)).\] We conclude this section with a table comparing the expected duration and the standard deviation when the particle starts at $(x,y)=(\frac{M}{2},\frac{M}{2})$ for various $M$ under probabilities $P=[\frac{1}{6},\frac{1}{3},\frac{1}{6},\frac{1}{3}]$. \begin{table}[H] \centering \begin{tabular}{|c|c|c|c|} \hline $M$ & $(x,y)$ & Expected Duration& Standard Deviation \\ \hline $2$ & $(1,1)$ & $1$ & $0$\\\hline $4$ & $(2,2)$ & $4.470588235$ & $ 2.891342524$\\ \hline $6$ & $(3,3)$ & $10.3030$ & $7.102295958$\\ \hline $8$ & $(4,4)$ & $18.47746573 $ & $12.97689858$ \\ \hline $10$ & $(5,5)$ & $28.99020033 $ & $20.52455308$\\ \hline $12$ & $(6,6)$ & $ 41.84019550$ & $29.74741677$\\ \hline $14$ & $(7,7)$ & $ 57.02707373$ & $40.64621816$\\ \hline $16$ & $(8,8)$ & $ 74.55067222$ & $53.22126050$\\ \hline $18$ & $(9,9)$ & $94.41091165 $ & $67.47268859$\\ \hline $20$ & $(10,10)$ & $116.6077497 $ & $83.40057864$ \\ \hline \end{tabular} \end{table} \section{A Mirror Step Variant of Gambler's Ruin}\label{sec:mirror step} In this section, we begin by formulating a new generalization of the gambler's ruin problem in $1$-dimension. A particle starts at some point $x$ on a line of length $N$ where $0< x <N$. At each step, the particle moves from $x$ to $x-1$ with probability $q_1$, or moves from $x$ to $x+1$ with probability $q_2$, or moves from $x$ to $N-x$ with probability $p$ where $0<p<1$ and $q_1+q_2+p=1$. We call this last step the \textit{mirror step}. The particle continues to walk on the line until it reaches $0$ or $N$. We focus on the case when $q_1=q_2=\frac{1-p}{2}$ and we call this the \textit{symmetric case}. \subsection{Probability} Define $f(x)=f_N^{(p)}(x)$ as the probability that a particle starting at $x$ will eventually reach $N$. For $0<x<N$, this probability satisfies the recurrence relation \begin{equation} \label{eq:prob recurrence mirror} f(x)=\frac{1-p}{2}f(x-1)+\frac{1-p}{2}f(x+1)+pf(N-x) \end{equation} where $f(0)=0$ and $f(N)=1$. Before providing the solution to \Cref{eq:prob recurrence mirror}, we will try to guess the limit as $N$ goes to infinity of the probability when the particle starts at some $x$ and ends at $N$, by using a fixed, large, $N$ as discussed in \Cref{subsec:mirror}. We are interested in \begin{equation}\label{eq:limit of fN(x)} \lim_{N\to \infty} f_N^{(p)}(x) \end{equation} and we hope to get expressions for when $x=2$ and $x=N-2$. First, we describe the approach that will generate data for fixed values of $N$ and $x$. Using the data, we can make some guesses for the limit in \Cref{eq:limit of fN(x)}. \begin{itemize}[leftmargin=0.2 in] \item Fix $N$ as large as possible and use Maple to generate $N-1$ linear equations with \Cref{eq:prob recurrence mirror}. \item Solve for the $N-1$ linear equations. \item Generate data for $f_N^{(p)}(x)$ for different $p$ values with fixed $N$ and a fixed location $x$. \item We use the function \texttt{identify} in Maple to try to identify the numerical values given by $f_N^{(p)}(x)$. The function \texttt{identify} is based, in part, on the continued fraction expansion of the numerical values. As $N$ grows the numerical value will converge to some number. \item Guess a formula for the number with fixed $x$ and varying $p$. \end{itemize} \noindent We begin with an example on how to generate data in Maple using the code accompanying this article. \begin{example}\label{ex: x=1 prob} Let $N=100$ and $x=2$. We generate data for the probability that if the particle starts at $x=2$, the particle eventually ends at $100$. Let $p\in \{\frac{1}{2},\frac{1}{3},\frac{1}{4},\ldots, \frac{1}{10}\}$. We use the procedure \texttt{Lk(p,x,N)}, as described in \Cref{appendix:mirror}, in Maple which generates the following data in about $8.390$ seconds: \begin{align*} T \coloneqq &[0.48528137423857029281, 0.46410161513775458705, 0.444, 0.42705098312484227231,\\ &0.41171425595857973499, 0.39811169380648470689, 0.38595282133533513790, 0.375, \\ &0.36506306819388080622]. \end{align*} \noindent The sequence $T$ reads as follows. If the particle starts at $x=2$ and $p=\frac{1}{2}$, the particle moves from $x$ to $x-1$ with probability $\frac{1-p}{2}=\frac{1}{4}$, or moves from $x$ to $x+1$ with probability $\frac{1-p}{2}=\frac{1}{4}$, or moves from $x$ to $100-x$ with probability $p=\frac{1}{2}$. Then, the probability of the particle starting at $x=2$ and ending at $100$ is $T_1=0.48528137423857029281$. Similarly, if the particle starts at $x=2$ and $p=\frac{1}{3}$, the particle moves from $x$ to $x-1$ with probability $\frac{1-p}{2}=\frac{1}{3}$, or moves from $x$ to $x+1$ with probability $\frac{1-p}{2}=\frac{1}{3}$, or moves from $x$ to $100-x$ with probability $p=\frac{1}{3}$. Then, the probability of the particle starting at $x=2$ and ending at $100$ is $T_2=0.46410161513775458705$. Thus, $T_i$ is the probability of the particle starting at $x=2$ and ending at $100$ for $p=\frac{1}{i+1}$ where $1\leq i\leq 9$. Using \texttt{identify} in Maple for the sequence $T$, we conjecture that each of the probabilities in $T$ converge to \begin{align*} M\coloneqq&\left[-8+6 \sqrt{2}, -3+2 \sqrt{3}, \frac{4}{9}, \frac{-5+3 \sqrt{5}}{4}, \frac{-24 + 14\sqrt{6}}{25}, \frac{-7+4 \sqrt{7}}{9}, \frac{-32+36 \sqrt{2}}{49}, {\frac{3}{8}}, \frac{-40+22 \sqrt{10}}{81}\right]. \end{align*} That is the probability of the particle starting at $x=2$ and ending at $N$ as $N$ grows converges to $M_i$ for $p=\frac{1}{i+1}$ where $1\leq i\leq 9$. \end{example} The previous example illustrates that when the particle starts at $x=2$, $\{f_N^{(p)}(2)\}$ converges fast. We state the following guess for the expression of the limit when $x=2$ and in \Cref{cor:probability at infinity} we provide a proof. \begin{guess}\label{guess:x=2} If the particle starts at $x=2$, then \[\lim_{N\to\infty}f_N^{(p)}(2)=\frac{2\sqrt{p}(1+p-2\sqrt{p})}{(1-p)^2}.\] \end{guess} Using the same approach from above, we can obtain data for $x=N-2$. We guess the following expression of the limit when $x=N-2$ and provide a proof in \Cref{cor:probability at infinity}. \begin{guess}\label{guess:x=N-2} If the particle starts at $x=N-2$, then \[\lim_{N\to\infty}f_N^{(p)}(N-2)=\frac{(1+p)(1+p-2\sqrt{p})}{(1-p)^2}.\] \end{guess} We were able to guess more expressions for $\lim_{N\to\infty} f_N(x)$ with other $x$ values. After we made these guesses, we established the following key lemma which provides a relation between $f(x)$ and $f(N-x)$ for any $0\leq x\leq N$. We stress that this is only true for the symmetric case when the probability of the particle moving from $x$ to $x-1$ is the same as the probability of the particle moving from $x$ to $x+1$. \begin{lemma}\label{lem:mirror probability equal to 1} Consider the symmetric case when $f(x)=\frac{1-p}{2}f(x-1)+\frac{1-p}{2}f(x+1)+pf(N-x)$ with boundary conditions $f(0)=0, f(N)=1$ for some $0<p<1$. For any $0\leq x\leq N$, the following identity holds \[f(x) + f(N-x)=1.\] \end{lemma} \begin{proof} Call $g(x)=1-f(N-x)$. We will show that $f(x)+f(N-x)=1$ by proving that $g(x)=f(x)$. Note that $g(0)=0$ and $g(N)=1$, and $f(x)=1-g(N-x)$. Using the recurrence in \Cref{eq:prob recurrence mirror}, we substitute $1-g(N-x)$ for $f(x)$ and obtain: \begin{equation*} g(N-x)=\frac{1-p}{2}g(N-(x-1))+\frac{1-p}{2}g(N-(x+1))+pg(x). \end{equation*} That is, \begin{equation*} g(x)=\frac{1-p}{2}g(x+1))+\frac{1-p}{2}g(x-1)+pg(N-x) \end{equation*} which has boundary conditions $g(0)=0$ and $g(N)=1$. Therefore, $g(x)=f(x)$ as desired. \end{proof} Lemma \ref{lem:mirror probability equal to 1} establishes that the sums of the probabilities when the particle starts at $x$ and when the particle starts at $N-x$ equals to 1. Using this identity, we rewrite \Cref{eq:prob recurrence mirror} to \begin{equation}\label{eq:inhomogeneous eq} f(x)=\frac{p}{1+p}+\frac{1}{2}\left(\frac{1-p}{1+p}\right)f(x-1)+\frac{1}{2}\left(\frac{1-p}{1+p}\right)f(x+1), \quad f(0)=0, f(N)=1. \end{equation} We can now derive the probability of the particle ending at $N$ if it starts at some $x$, for general $N, x$ and $p$. \begin{theorem}\label{thm:probability mirror} Consider the generalization of the gambler's ruin problem when we add a mirror step. Then, the probability of ending at $N$ starting at $x$ is given by \begin{equation*} f(x)=\frac{1}{2}\frac{\left(\frac{1-\sqrt{p}}{1+\sqrt{p}}\right)^N+1}{\left(\frac{1+\sqrt{p}}{1-\sqrt{p}}\right)^N-\left(\frac{1-\sqrt{p}}{1+\sqrt{p}}\right)^N}\left(\frac{1+\sqrt{p}}{1-\sqrt{p}}\right)^x +\frac{1}{2}\frac{\left(\frac{1+\sqrt{p}}{1-\sqrt{p}}\right)^N+1}{\left(\frac{1-\sqrt{p}}{1+\sqrt{p}}\right)^N-\left(\frac{1+\sqrt{p}}{1-\sqrt{p}}\right)^N}\left(\frac{1-\sqrt{p}}{1+\sqrt{p}}\right)^x+\frac{1}{2} \end{equation*} whenever we restrict the particle moves by either moving from $x$ to $x-1$ with probability $q_1$, or from $x$ to $x+1$ with probability $q_2$, or from $x$ to $N-x$ with probability $p$ where $q_1=q_2=\frac{1-p}{2}$. \end{theorem} \begin{proof} We can solve for \Cref{eq:inhomogeneous eq} because it is an inhomogeneous recurrence relation. Hence, we find a homogeneous and an inhomogeneous solution. The general solution to the homogeneous equation $f(x)=\frac{1}{2}\left(\frac{1-p}{1+p}\right)f(x-1)+\frac{1}{2}\left(\frac{1-p}{1+p}\right)f(x+1)$ is \begin{equation*} f(x)=A\left(\frac{1+p+2\sqrt{p}}{1-p}\right)^x+B\left(\frac{1+p-2\sqrt{p}}{1-p}\right)^x \end{equation*} for some numbers $A$ and $B$. Next, we find the particular solution to the inhomogeneous relation by setting $f^*(x)=C$ for some constant $C$. Then, \begin{equation*} f^*(x)=\frac{p}{1+p}+\frac{1}{2}\left(\frac{1-p}{1+p}\right)f^*(x-1)+\frac{1}{2}\left(\frac{1-p}{1+p}\right)f^*(x+1) \end{equation*} becomes \begin{equation*} C=\frac{p}{1+p}+\frac{1}{2}\left(\frac{1-p}{1+p}\right)C+\frac{1}{2}\left(\frac{1-p}{1+p}\right)C \end{equation*} which has solution $C=\frac{1}{2}$. Therefore, $f^*(x)=\frac{1}{2}$ is the particular solution, and the general inhomogeneous solution is \begin{equation}\label{eq:general inhomogeneous sln} f(x)=A\left(\frac{1+p+2\sqrt{p}}{1-p}\right)^x+B\left(\frac{1+p-2\sqrt{p}}{1-p}\right)^x+\frac{1}{2}. \end{equation} Using Maple, we find $A$ and $B$ by using the boundary conditions to get a system of two linear equations. Namely, \begin{align*} 0&=A+B+\frac{1}{2} \intertext{and} \frac{1}{2}&=A\left(\frac{1+p+2\sqrt{p}}{1-p}\right)^N+B\left(\frac{1+p-2\sqrt{p}}{1-p}\right)^N. \end{align*} Solving for the above linear equations, we get \begin{align} A&=\frac{1}{2}\frac{\left(\frac{1+p-2\sqrt{p}}{1-p}\right)^N+1}{\left(\frac{1+p+2\sqrt{p}}{1-p}\right)^N-\left(\frac{1+p-2\sqrt{p}}{1-p}\right)^N} \label{eq: A eq}\intertext{and} B&=\frac{1}{2}\frac{\left(\frac{1+p+2\sqrt{p}}{1-p}\right)^N+1}{\left(\frac{1+p-2\sqrt{p}}{1-p}\right)^N-\left(\frac{1+p+2\sqrt{p}}{1-p}\right)^N}.\label{eq: B eq} \end{align} Rewriting Equations \ref{eq:general inhomogeneous sln}, \ref{eq: A eq} and \ref{eq: B eq} gives \begin{align*} f(x)&=A\left(\frac{1+\sqrt{p}}{1-\sqrt{p}}\right)^x+B\left(\frac{1-\sqrt{p}}{1+\sqrt{p}}\right)^x+\frac{1}{2} \end{align*} as desired. \end{proof} \Cref{thm:probability mirror} provides a formula for $\lim_{N\to\infty}f_N(x)$ for any $x$ where $0<x<N$. \begin{corollary}\label{cor:probability at infinity} If the particle starts at some $x$ where $0<x<N$, then \[\lim_{N\to \infty} f_N^{(p)}(x)=\frac{1}{2}-\frac{1}{2}\left(\frac{1-\sqrt{p}}{1+\sqrt{p}}\right)^x\] whenever we restrict the particle moves by either moving from $x$ to $x-1$ with probability $q_1$, or from $x$ to $x+1$ with probability $q_2$, or from $x$ to $N-x$ with probability $p$ where $q_1=q_2=\frac{1-p}{2}$. \end{corollary} \begin{proof} Let \begin{equation}\label{eq: fNx} f_N^{(p)}(x)=\frac{1}{2}\frac{\left(\frac{1-\sqrt{p}}{1+\sqrt{p}}\right)^N+1}{\left(\frac{1+\sqrt{p}}{1-\sqrt{p}}\right)^N-\left(\frac{1-\sqrt{p}}{1+\sqrt{p}}\right)^N}\left(\frac{1+\sqrt{p}}{1-\sqrt{p}}\right)^x +\frac{1}{2}\frac{\left(\frac{1+\sqrt{p}}{1-\sqrt{p}}\right)^N+1}{\left(\frac{1-\sqrt{p}}{1+\sqrt{p}}\right)^N-\left(\frac{1+\sqrt{p}}{1-\sqrt{p}}\right)^N}\left(\frac{1-\sqrt{p}}{1+\sqrt{p}}\right)^x+\frac{1}{2}, \end{equation} and \[g_N^{(p)}(x)=\frac{1}{2}\frac{1}{\left(\frac{1+\sqrt{p}}{1-\sqrt{p}}\right)^N}\left(\frac{1+\sqrt{p}}{1-\sqrt{p}}\right)^x+\frac{1}{2}-\frac{1}{2}\left(\frac{1-\sqrt{p}}{1+\sqrt{p}}\right)^x.\] We will show that $\lim_{N\to\infty}f_N^{(p)}(x)-g_N^{(p)}(x)=0$. Observe that \begin{align*} f_N^{(p)}(x)-g_N^{(p)}(x)&=\frac{1}{2}\frac{\left(\frac{1-\sqrt{p}}{1+\sqrt{p}}\right)^N+1}{\left(\frac{1+\sqrt{p}}{1-\sqrt{p}}\right)^N-\left(\frac{1-\sqrt{p}}{1+\sqrt{p}}\right)^N}\left(\frac{1+\sqrt{p}}{1-\sqrt{p}}\right)^x +\frac{1}{2}\frac{\left(\frac{1+\sqrt{p}}{1-\sqrt{p}}\right)^N+1}{\left(\frac{1-\sqrt{p}}{1+\sqrt{p}}\right)^N-\left(\frac{1+\sqrt{p}}{1-\sqrt{p}}\right)^N}\left(\frac{1-\sqrt{p}}{1+\sqrt{p}}\right)^x\\ & \quad - \frac{1}{2}\frac{1}{\left(\frac{1+\sqrt{p}}{1-\sqrt{p}}\right)^N}\left(\frac{1+\sqrt{p}}{1-\sqrt{p}}\right)^x+\frac{1}{2}\left(\frac{1-\sqrt{p}}{1+\sqrt{p}}\right)^x\\ &=\frac{1}{2}\left(\frac{1+\sqrt{p}}{1-\sqrt{p}}\right)^x\left(\frac{\left(\frac{1-\sqrt{p}}{1+\sqrt{p}}\right)^N+1}{\left(\frac{1+\sqrt{p}}{1-\sqrt{p}}\right)^N-\left(\frac{1-\sqrt{p}}{1+\sqrt{p}}\right)^N}-\left(\frac{1-\sqrt{p}}{1+\sqrt{p}}\right)^N\right)\\ &\quad+\frac{1}{2}\left(\frac{1-\sqrt{p}}{1+\sqrt{p}}\right)^x\left(\frac{\left(\frac{1+\sqrt{p}}{1-\sqrt{p}}\right)^N+1}{\left(\frac{1-\sqrt{p}}{1+\sqrt{p}}\right)^N-\left(\frac{1+\sqrt{p}}{1-\sqrt{p}}\right)^N}+\frac{1}{2}\right)\\ &=\frac{1}{2}\left(\frac{1+\sqrt{p}}{1-\sqrt{p}}\right)^x\left(\frac{\left(\frac{1-\sqrt{p}}{1+\sqrt{p}}\right)^N+\left(\frac{1-\sqrt{p}}{1+\sqrt{p}}\right)^{2N}}{\left(\frac{1+\sqrt{p}}{1-\sqrt{p}}\right)^N-\left(\frac{1-\sqrt{p}}{1+\sqrt{p}}\right)^N}\right)-\frac{1}{2}\left(\frac{1-\sqrt{p}}{1+\sqrt{p}}\right)^x\left(\frac{1+\left(\frac{1-\sqrt{p}}{1+\sqrt{p}}\right)^N}{\left(\frac{1+\sqrt{p}}{1-\sqrt{p}}\right)^N-\left(\frac{1-\sqrt{p}}{1+\sqrt{p}}\right)^N}\right)\\ &=\frac{1}{2}\frac{\left(1+\left(\frac{1-\sqrt{p}}{1+\sqrt{p}}\right)^N\right)\left( \left(\frac{1+\sqrt{p}}{1-\sqrt{p}}\right)^x\left(\frac{1-\sqrt{p}}{1+\sqrt{p}}\right)^N - \left(\frac{1-\sqrt{p}}{1+\sqrt{p}}\right)^x\right)}{\left(\frac{1+\sqrt{p}}{1-\sqrt{p}}\right)^N-\left(\frac{1-\sqrt{p}}{1+\sqrt{p}}\right)^N}\\ &=\frac{1}{2}\frac{\left(1+ \left(\frac{1-\sqrt{p}}{1+\sqrt{p}}\right)^{N}\right)\left(\left(\frac{1+\sqrt{p}}{1-\sqrt{p}}\right)^x-1\right)}{\left(\frac{1+\sqrt{p}}{1-\sqrt{p}}\right)^{2N}-1}. \end{align*} Since \begin{align*} \lim_{N\to \infty} \left(\frac{1-\sqrt{p}}{1+\sqrt{p}}\right)^{N}=0 \intertext{and} \lim_{N\to \infty}\frac{1}{2}\frac{\left(\left(\frac{1+\sqrt{p}}{1-\sqrt{p}}\right)^x-1\right)}{\left(\frac{1+\sqrt{p}}{1-\sqrt{p}}\right)^{2N}-1}=0, \end{align*} It follows that \[\lim_{N\to\infty}f_N^{(p)}(x)-g_N^{(p)}(x)=0.\] Thus, \[\lim_{N\to \infty} f_N^{(p)}(x) =\frac{1}{2}-\frac{1}{2}\left(\frac{1-\sqrt{p}}{1+\sqrt{p}}\right)^x.\] \end{proof} Using \Cref{cor:probability at infinity} provides a proof for \Cref{guess:x=1}. See the following example. \begin{example} Setting $x=1$, \[\lim_{N\to \infty} f_N^{(p)}(1)=\frac{1}{2}-\frac{1}{2}\left(\frac{1-\sqrt{p}}{1+\sqrt{p}}\right)=\frac{\sqrt{p}-p}{1-p}\] as expected. \end{example} \subsection{Expected duration} We now consider the expected duration of the gambler's ruin problem with a mirror step. Define $g(x)$ as the expected number of steps that a particle starting at $x$ will eventually reach a position $0$ or $N$. For $0<x<N$, this expected duration satisfies the recurrence relation \begin{equation} \label{eq:mirror exp} g(x)=\frac{1-p}{2}g(x-1)+\frac{1-p}{2}g(x+1) + pg(N-x) +1, \quad g(0)=0, g(N)=0. \end{equation} We use this recurrence relation to find a closed formula for the expected duration of the game. \begin{theorem}\label{thm:expected duration mirror} Consider the generalization of the gambler's ruin problem when we add a mirror step. Then, the expected duration of ending at $0$ or $N$ starting at $x$ is given by \begin{equation*} g(x)=\frac{1}{1-p}x(N-x) \end{equation*} whenever we restrict the particle moves by either moving from $x$ to $x-1$ with probability $q_1$, or from $x$ to $x+1$ with probability $q_2$, or jumps to $N-x$ with probability $p$ where $q_1=q_2=\frac{1-p}{2}$. \end{theorem} Remark: When $p=0$, \Cref{thm:expected duration mirror} recovers the formula for the expected duration of the classical gambler's ruin game. \begin{proof} Let $h(x)=\frac{1}{1-p}x(N-x)$ and observe that $h(0)=0, h(N)=0$. We prove that $h(x)$ satisfies the same recurrence relation as $g(x)$. Applying the recurrence from \Cref{eq:mirror exp} to $h(x)$ and simplifying yields \begin{align*} &\frac{1-p}{2}h(x-1) +\frac{1-p}{2}h(x+1)+ph(N-x)+1\\ &= \frac{1-p}{2}\left(\frac{1}{1-p}(x-1)(N-x+1)\right)+\frac{1-p}{2}\left(\frac{1}{1-p}(x+1)(N-x-1)\right)+p\left(\frac{1}{1-p}x(N-x)\right)+1\\ &=\frac{1}{1-p}x(N-x). \end{align*} Hence, $h(x)$ satisfies the following recurrence relation \begin{equation*} h(x)=\frac{1-p}{2}h(x-1) +\frac{1-p}{2}h(x+1)+ph(N-x)+1, \quad h(0)=0, h(N)=0. \end{equation*} Thus, we get that $h(x)=g(x)$ which completes the proof. \end{proof} \section{Future Work }\label{sec:future} In \Cref{sec:mirror step}, we consider a generalization of the gambler's ruin problem where the particle starts at some point $x$ on a line of length $N$ where $0< x <N$. At each step, the particle either moves to the left by one step with probability $q_1$, moves to the right by one step with probability $q_2$, or moves to $N-x$ with probability $p$ where $0<p<1$ and $q_1+q_2+p=1$. We focus on the case when $q_1=q_2=\frac{1-p}{2}$, and we provide formulas for the probability that the particle ends at $N$ and the expected number of steps to finish the game. Thus, it is an open problem to give formulas for general $q_1, q_2$ and $p$. Computational evidence suggests the following conjecture when the probability of moving from $x$ to $x-1$ is the same as the probability of moving from $x$ to $N-x$. \begin{conjecture} Consider the generalization of the gambler's ruin problem when we add a mirror step. If the particle starts at $x=1$, then \[\lim_{N\to \infty} f_N^{(p)}(1)= \frac{\sqrt{(p+1)(1-3p+4p^2)}-(1-2p)(p+1)}{2p(p+1)}\] whenever we restrict the particle moves by either moving from $x$ to $x-1$ with probability $q_1$, or from $x$ to $x+1$ with probability $p$, or from $x$ to $N-x$ with probability $q_2$ where $q_1=q_2=\frac{1-p}{2}$. \end{conjecture} \section*{Acknowledgements} The author thanks her advisor Dr. Doron Zeilberger for the introduction to the problem and feedback on an earlier draft. The author was supported by the NSF Graduate Research Fellowship Program under Grant No. 2233066. \bibliographystyle{plain} \bibliography{bibliography.bib} \appendix \section{Computational Tools for Analyzing the Gambler's Ruin Problem}\label{appendix:maple} \subsection{Method Descriptions} There are four text files accompanying this article: \lstinline{GGR.txt, GGR1d.txt, GGR2d.txt,} and \lstinline{GGR1dMirror.txt} which can be found in the GitHub repository \\ \href{https://github.com/marti310/Gamblers-Ruin}{https://github.com/marti310/Gamblers-Ruin}. In this section, we will describe the functionality of some of the main procedures. These text files should be saved in the same directory. All procedures were written and tested for Maple 20. \subsubsection{\lstinline{GGR.txt}} The \lstinline{GGR.txt} file contains the following main procedures. \begin{itemize} \item \lstinline{ProbN(N)} Returns a list $L$ of length $N-1$. This function inputs a positive integer $N$ and computes the probability of ending at $N$ for every $1\leq x \leq N-1$. Example: \begin{lstlisting} read `GGR.txt`: N:=10; ProbN(N); \end{lstlisting} Output: \lstinline!{[1/10, 1/5, 3/10, 2/5, 1/2, 3/5, 7/10, 4/5, 9/10]}! \item \lstinline{ExpN(N)} Returns a list $L$ of length $N-1$. This function inputs a positive integer $N$ and computes the expected number of steps of ending at either $0$ or $N$ for every $1\leq x \leq N-1$. \end{itemize} \subsubsection{\lstinline{GGR1d.txt}}\label{appendix:ggr1d} The \lstinline{GGR1d.txt} file contains procedures for the classical gambler's ruin game. In addition, it contains procedures for the $1$-dimensional case. We provide the main procedures. \begin{itemize} \item \lstinline{GR1dPG(N,P)} Returns a list $L$ of length $N-1$. This function inputs a positive integer $N$ and a probability table $P$ where $P=[[a_1,p_1],[a_2,p_2],...,[a_r,p_r]]$ and computes the probability of the particle ending at some position $\geq N$ for every $1\leq x \leq N-1$. Remark: This procedures uses the classical approach of solving for $N-1$ linear equations. Example: \begin{lstlisting} read `GGR1d.txt`: P:=GR1dPG(10,[[-1,1/2],[1,1/2]]); \end{lstlisting} Output: \lstinline!{[1/10, 1/5, 3/10, 2/5, 1/2, 3/5, 7/10, 4/5, 9/10]}! \item \lstinline{GR1dLG(N,P)} Returns a list $L$ of length $N-1$. This function inputs a positive integer $N$ and a probability table $P$ where $P=[[a_1,p_1],[a_2,p_2],...,[a_r,p_r]]$ and computes the expected number of steps for the particle to end at $\leq 0$ or $\geq N$ for every $1\leq x \leq N-1$. Remark: This procedures uses the classical approach of solving for $N-1$ linear equations. \item \lstinline{NewGR1dPG(N,P)} Returns a list $L$ of length $N-1$. This function inputs a positive integer $N$ and a probability table $P$ where $P=[[a_1,p_1],[a_2,p_2],...,[a_r,p_r]]$ and computes the probability for the particle to end at $\geq N$ for every $1\leq x \leq N-1$. Remark: This procedures uses the faster method. Example: \begin{lstlisting} read `GGR1d.txt`: P:=NewGR1dPG(10,[[-1,1/2],[1,1/2]]); \end{lstlisting} Output: \lstinline!{[1/10, 1/5, 3/10, 2/5, 1/2, 3/5, 7/10, 4/5, 9/10]}! \item \lstinline{NewGR1dLG(N,P)} Returns a list $L$ of length $N-1$. This function inputs a positive integer $N$ and a probability table $P$ where $P=[[a_1,p_1],[a_2,p_2],...,[a_r,p_r]]$ and computes the expected number of steps for the particle to end at $\leq 0$ or $\geq N$ for every $1\leq x \leq N-1$. Remark: This procedures uses the faster method. \end{itemize} \subsubsection{\lstinline{GGR2d.txt}}\label{appendix:ggr2d} The \lstinline{GGR2d.txt} file contains procedures for the $2$-dimensional gambler's ruin game. We provide the main procedures. \begin{itemize} \item \lstinline{GR2dP(M,N,L,U,R,B)} Returns an $(M-1) \times (N-1)$ matrix whose entries are linear combinations of L,U,R,B. This function inputs positive integers $M, N$ and symbols L,U,R,B where L is the left edge, U is the top edges, R is the right edge and B is the bottom edge of the $M\times N$ rectangle, and computes the probability of the particle starting at some point $(a,b)$ and ending on L, U, R or B for every $1\leq a \leq M-1$ and $1\leq b \leq M-1$. Remark: This procedures uses the classical approach of solving for $(M-1)\times(N-1)$ linear equations. Example: \begin{lstlisting} read `GGR2d.txt`: GR2dP(3,3,L,U,R,B); \end{lstlisting} Output: \lstinline!{[[(3L)/8 + (3B)/8 + U/8 + R/8, (3L)/8 + B/8 + (3U)/8 + R/8], [L/8 + (3B)/8 + U/8 + (3R)/8, L/8 + B/8 + (3U)/8 + (3R)/8]]}! \item \lstinline{GR2dL(M,N)} Returns an $(M-1)\times (N-1)$ matrix $M$. This function inputs positive integers $M, N$ and computes the expected number of steps of the particle starting at some point $(a,b)$ and ending on L, U, R or B for every $1\leq a \leq M-1$ and $1\leq b \leq M-1$. Remark: This procedures uses the classical approach of solving for $(M-1)\times(N-1)$ linear equations. Example: \begin{lstlisting} read `GGR2d.txt`: GR2dL(3,3); \end{lstlisting} Output: \lstinline!{[[2, 2], [2, 2]]}! \item \lstinline{NewGR2dP(M,N,L,U,R,B)} Returns an $(M-1) \times (N-1)$ matrix whose entries are linear combinations of L,U,R,B. This function inputs positive integers $M, N$ and symbols L,U,R,B where L is the left edge, U is the top edges, R is the right edge and B is the bottom edge of the $M\times N$ rectangle, and computes the probability of the particle starting at some point $(a,b)$ and ending on L, U, R or B for every $1\leq a \leq M-1$ and $1\leq b \leq M-1$. Remark: This procedures uses the faster method. Example: \begin{lstlisting} read `GGR2d.txt`: NewGR2dP(3,3,L,U,R,B); \end{lstlisting} Output: \lstinline!{[[(3L)/8 + (3B)/8 + U/8 + R/8, (3L)/8 + B/8 + (3U)/8 + R/8], [L/8 + (3B)/8 + U/8 + (3R)/8, L/8 + B/8 + (3U)/8 + (3R)/8]]}! \item \lstinline{NewGR2dL(M,N)} Returns an $(M-1)\times (N-1)$ matrix $M$. This function inputs positive integers $M, N$ and computes the expected number of steps of the particle starting at some point $(a,b)$ and ending on L, U, R or B for every $1\leq a \leq M-1$ and $1\leq b \leq M-1$. Remark: This procedures uses the faster method. Example: \begin{lstlisting} read `GGR2d.txt`: NewGR2dL(3,3); \end{lstlisting} Output: \lstinline!{[[2, 2], [2, 2]]}! \item \lstinline{NewGR2dPG(M,N,L,U,R,B,P)} Returns an $(M-1) \times (N-1)$ matrix whose entries are linear combinations of L,U,R,B. This function inputs positive integers $M, N$, symbols L,U,R,B where L is the left edge, U is the top edges, R is the right edge and B is the bottom edge of the $M\times N$ rectangle, and a probability table $P=[p_L,p_U,p_R,p_B]$ such that the particle moves left by one step with probability $p_L$, or moves up by one step with probability $p_U$, or moves right by one step with probability $p_R$, or moves down by one step with probability $p_B$, and computes the probability of the particle starting at some point $(a,b)$ and ending on L, U, R or B for every $1\leq a \leq M-1$ and $1\leq b \leq M-1$. Remark: This procedures uses the faster method. Example: \begin{lstlisting} read `GGR2d.txt`: NewGR2dPG(3,3,L,U,R,B,[1/4,1/4,1/4,1/4]); \end{lstlisting} Output: \lstinline!{[[R/8 + (3L)/8 + U/8 + (3B)/8, R/8 + (3L)/8 + (3U)/8 + B/8], [(3R)/8 + L/8 + U/8 + (3B)/8, (3R)/8 + L/8 + (3U)/8 + B/8]]}! \item \lstinline{NewGR2dLG(M,N,P)} Returns an $(M-1) \times (N-1)$ matrix. This function inputs positive integers $M, N$, and a probability table $P=[p_L,p_U,p_R,p_B]$ such that the particle moves left by one step with probability $p_L$, or moves up by one step with probability $p_U$, or moves right by one step with probability $p_R$, or moves down by one step with probability $p_B$, and computes the expected number of steps of the particle starting at some point $(a,b)$ and ending on L, U, R or B for every $1\leq a \leq M-1$ and $1\leq b \leq M-1$. Remark: This procedures uses the faster method. Example: \begin{lstlisting} read `GGR2d.txt`: NewGR2dLG(3,3,[1/4,1/4,1/4,1/4]); \end{lstlisting} Output: \lstinline!{[[2, 2], [2, 2]]}! \item \lstinline{KmetPetkovsek(N)} Returns an $(N-1) \times (N-1)$ matrix. This function inputs a positive integer $N$ and implements Kmet and Petkovsek's formula for the expected duration of the 2-dimensional gambler's ruin game for the $M=N$ case. Example: \begin{lstlisting} read `GGR2d.txt`: KmetPetkovsek(3); \end{lstlisting} Output: \lstinline!{[[2, 2], [2, 2]]}! \end{itemize} \subsubsection{\lstinline{GGR1dMirror.txt}}\label{appendix:mirror} The \lstinline{GGR1dMirror.txt} file contains the following main procedures for the new generalization of gambler's ruin where we add a third step. \begin{itemize} \item \lstinline{ProbN2(N,P)} Returns a list $L$ of length $N$. This function inputs a positive integer $N$ and a probability table $P$ where $P=[p_1,p_2,p_3]$ where $p_1+p_2+p_3=1$ and outputs a list $L$ of length $N$ such that $L[x]$ is the probability of the particle ending at $N$ when it starts at $x$ where the particle can move from $x$ to $x-1$ with probability $p_1$, or $x$ to $x+1$ with probability $p_2$, or $x$ to $N-x$ with probability $p_3$. Example: \begin{lstlisting} read `GGR1dMirror.txt`: ProbN2(5,[1/3,1/3,1/3]); \end{lstlisting} Output: \lstinline!{[7/19, 9/19, 10/19, 12/19, 1]}! \item \lstinline{ExpN2(N,P)} Returns a list $L$ of length $N$. This function inputs a positive integer $N$ and a probability table $P$ where $P=[p_1,p_2,p_3]$ where $p_1+p_2+p_3=1$ and outputs a list $L$ of length $N$ such that $L[x]$ is the expected number of steps that the particle takes to end at $N$ or $0$ when it starts at $x$ where the particle can move from $x$ to $x-1$ with probability $p_1$, or $x$ to $x+1$ with probability $p_2$, or $x$ to $N-x$ with probability $p_3$. Example: \begin{lstlisting} read `GGR1dMirror.txt`: ExpN2(5,[1/3,1/3,1/3]); \end{lstlisting} Output: \lstinline!{[6, 9, 9, 6, 0]}! \item \lstinline{Lk(p,x,N)} Returns a number. This function inputs a probability value $p, 0<p<1$, and positive integers $x,N$ where $0<x<N$ and outputs the exact probability of the particle ending at $N$ when it starts at location $x$ where with probability $(1-p)/2$ the particle moves to $x-1$, with probability $(1-p)/2$ the particle moves to $x+1$ and with probability $p$ the particle moves to $N-x$. Example: \begin{lstlisting} read `GGR1dMirror.txt`: Lk(1/3,4,100); \end{lstlisting} Output: \lstinline!{0.4974226119}! \end{itemize} \end{document}
2412.07644v1
http://arxiv.org/abs/2412.07644v1
Characterizations of multidimensional compact almost automorphic functions and applications to Poissons and heat equations
\documentclass[11pt]{article} \usepackage[centertags]{amsmath} \usepackage{mathtools} \usepackage{amssymb} \usepackage{amsthm} \usepackage{graphicx} \usepackage{upquote} \usepackage{cases} \usepackage{hyperref} \usepackage{xcolor} \usepackage{multicol} \usepackage{scalerel} \usepackage{scalerel} \newcommand\reallywidehat[1]{\arraycolsep=0pt\relax\begin{array}{c} \stretchto{ \scaleto{ \scalerel*[\widthof{\ensuremath{#1}}]{\kern-.5pt\bigwedge\kern-.5pt} {\rule[-\textheight/2]{1ex}{\textheight}} }{\textheight} }{0.5ex}\\ #1\\ \rule{-1ex}{0ex} \end{array} } \textwidth=15cm \topmargin -1.5cm \textheight=22.5cm \evensidemargin 0.4cm \oddsidemargin 0.6cm \frenchspacing \newtheorem{tw}{Theorem}[section] \newtheorem{lem}[tw]{Lemma}\newtheorem{cor}[tw]{Corollary}\newtheorem{prop}[tw]{Proposition}\newtheorem{definition}{Definition}[section] \newtheorem{rem}{Remark} \newtheorem{example}{Example}[section] \renewcommand{\theequation}{\thesection.\arabic{equation}} n{{\hfill$\square$}} \def\X{{\mathcal X}} \def\G{{\mathcal G}} \def\R{{\mathcal R}} \def\Rn{{\mathbb R}^n} \def\N{{\mathbb N}} \def\t{{\mathbf t}} \def\s{{\mathbf s}} \def\L{{\pmb{\mbox{L}}}} \def\={\hspace{-3mm}&=&\hspace{-3mm}} \def\ll{\left|\hspace{-0.5mm}\left|} \def\rr{\right|\hspace{-0.5mm}\right|} \renewcommand{\baselinestretch}{1,2} \title{\texttt{ Characterizations of multidimensional compact almost automorphic functions and applications to Poisson's and heat equations}} \author{ Alan Chávez$^1$\footnote{E-mail: [email protected] (corresponding author)}~, Jolbyn Casta\~neda $^2$\footnote{E-mail: [email protected]}, Alexis R. Carranza $^3$\footnote{E-mail: [email protected]}, Kamal Khalil $^4$\footnote{E-mail: [email protected].}\\ \small{$^{1,3}$ OASIS research group, Instituto de investigaci\'on en Matem\'aticas, }\\ \small{ Departamento de Matem\'aticas, FCFYM, Universidad Nacional de Trujillo,}\\ \small{Av. Juan Pablo II S/N, Trujillo-Per\'u}\\ \small{$^{2}$ OASIS research group \& Escuela de Matem\'aticas - FCFYM, Universidad Nacional de Trujillo,}\\ \small{Av. Juan Pablo II S/N, Trujillo-Per\'u}\\ \small{$^{4}$ LMAH, University of Le Havre Normandie, FR-CNRS-3335, ISCN, Le Havre 76600, France,}\\\\ \small{TO THE MEMORY OF MANUEL PINTO}\\ } \bibliographystyle{acm} \begin{document} \maketitle \begin{abstract} Let \(\mathcal{G}\) be a non-empty subset of the Euclidean space \(\mathbb{R}^m\) (\(m \geq 1\)). This work is dedicated to further exploring the properties of \(\mathcal{G}\)-multi-almost automorphic functions defined on \(\mathbb{R}^m\) with values in a Banach space \(\mathbb{X}\). Using the theory of \(\mathcal{G}\)-multi-almost automorphic functions, we provide two new characterizations of compact almost automorphic functions. In the first characterization, \(\mathcal{G}\) corresponds to the lattice subgroup \(\mathbb{Z}^m \subset \mathbb{R}^m\); in the second, \(\mathcal{G}\) is taken to be a dense subset of \(\mathbb{R}^m\). Furthermore, we establish the invariance of the space of bounded and compactly \(\mathcal{G}\)-multi-almost automorphic functions under integral operators with Bi-almost automorphic kernels. Finally, we present applications to the analysis of the almost automorphic dynamics of Poisson's equation and the heat equation. \end{abstract} \tableofcontents \section{Introduction}\label{sec:Introduction} \setcounter{equation}{0} \color{red} \color{black} Since their introduction by S. Bochner in 1966 \cite{03,04,05}, almost automorphic functions have garnered significant attention from mathematicians. This interest arises, on one hand, because these functions are a natural generalization of periodic and almost periodic functions, making them an ideal framework for studying the dynamics of differential, integral, or integro-differential equations, as demonstrated in works such as \cite{campos2014almost, campos2020barycentric, rezoug2024asymptotically} and other related references. On the other hand, the concept of almost automorphy provides a fertile ground for investigations in abstract topological dynamical systems, as evidenced by studies like \cite{fuhrmann2020tameness, garcia2021mean, lenz2024abstract, lenz2024pure} and the references cited therein. In this work, we contribute to the ongoing study of the almost automorphic dynamics of partial differential equations, specifically to the Poisson's and heat equations. The concept of complex-valued almost automorphic functions on a topological group \( G \) was first developed by Veech \cite{18, veech1965almost}. Let \( \mathcal{X} \) be a Banach space, \( G = \mathbb{R}^m \), and let \( \mathrm{R} \) denote the collection of all sequences in \( \mathbb{R}^m \). The notion of almost automorphy for a function \( f: \mathbb{R}^m \to \mathcal{X} \) is described in the following definition: \begin{definition}(Bochner)\label{dfAA} Let \( f: \mathbb{R}^m \rightarrow \mathcal{X} \) be a continuous function. Then, \( f \) is almost automorphic if and only if, for any sequence \( \mathbf{b}_k = (b_k^1, b_k^2, \dots, b_k^m) \in \mathrm{R} \), there exist a subsequence \( \mathbf{b}_{k_l} = (b_{k_l}^1, b_{k_l}^2, \dots, b_{k_l}^m) \) of \( \mathbf{b}_k \) and a function \( f^* : \mathbb{R}^m \to \mathcal{X} \) such that the following pointwise limits hold: \[ f^*(\mathbf{t}) = \lim_{l \to \infty} f(\mathbf{t} + \mathbf{b}_{k_l}), \quad f(\mathbf{t}) = \lim_{l \to \infty} f^*(\mathbf{t} - \mathbf{b}_{k_l}). \] \end{definition} Additionally, if the previous limits are replaced by the following: \[ \lim_{l \to \infty} \sup_{\mathbf{t} \in \mathcal{K}} \| f(\mathbf{t} + \mathbf{b}_{k_l}) - f^*(\mathbf{t}) \|_{\mathcal{X}} = 0, \quad \lim_{l \to \infty} \sup_{\mathbf{t} \in \mathcal{K}} \| f^*(\mathbf{t} - \mathbf{b}_{k_l}) - f(\mathbf{t}) \|_{\mathcal{X}} = 0, \] where \( \mathcal{K} \) is any compact subset of \( \mathbb{R}^m \), then \( f \) is called compact almost automorphic. Note that in the provided definition, the sequence \( (\mathbf{b}_k)_{k \in \mathbb{N}} \) can be any sequence in the Euclidean space \( \mathbb{R}^m \). However, a new approach explored in works such as \cite{chavez2022multi, chavez2023almost} involves considering these sequences within a set \( \mathrm{R} \), which is a non-empty collection of sequences in \( \mathbb{R}^m \) (rather than the collection of all sequences in \( \mathbb{R}^m \)). This perspective led the first author and collaborators to introduce the concept of \( \mathrm{R} \)-multi-almost periodic/automorphic functions, a more general notion than the classical concept of almost periodicity/automorphy. In \cite{chavez2022multi, chavez2023almost}, the authors extensively studied the fundamental structural properties of \( (\mathrm{R}, \mathcal{B}) \)-multi-almost periodic/automorphic functions and provided applications to both partial differential equations and integral equations. Let \( \mathcal{G}\) be a non empty subset of \(\mathbb{R}^m\). In the present work, we will denote by \( \mathrm{R}_\mathcal{G} \) a non empty collection of sequences in \( \mathcal{G} \), while \( \mathrm{R}_\mathcal{G}^a \) will denote the collection of all sequences in \( \mathcal{G} \). What is involved in this work is the notion of \( \mathrm{R}_\mathcal{G} \)-multi-almost automorphic function, which is as follows (see also Definition \ref{eovako} below) \begin{definition}\label{eovako00}\index{function!(compactly) $({\mathrm{R}}_{{\mathcal{G}}},{\mathcal B})$-multi-almost automorphic} Let $F : {\mathbb R}^{m} \rightarrow \mathcal{X}$ be a continuous function. $F(\cdot)$ is ${\mathrm{R}}_{{\mathcal{G}}}$-multi-almost automorphic if, for every sequence $({\bf b}_{k}=(b_{k}^{1},b_{k}^{2},\cdot \cdot\cdot ,b_{k}^{m})) \in {\mathrm{R}}_{{\mathcal{G}}}$ there exist a subsequence $({\bf b}_{k_{l}}=(b_{k_{l}}^{1},b_{k_{l}}^{2},\cdot \cdot\cdot , b_{k_{l}}^{m}))$ of $({\bf b}_{k})$ and a function $F^{\ast} : {\mathbb R}^{m} \rightarrow \mathcal{X}$ such that \begin{align}\label{love1234567800} \lim_{l\rightarrow +\infty}F\bigl({\bf t} +(b_{k_{l}}^{1},\cdot \cdot\cdot, b_{k_{l}}^{m}) \bigr)=F^{\ast}({\bf t} ) \end{align} and \begin{align}\label{love12345678900} \lim_{l\rightarrow +\infty}F^{\ast}\bigl({\bf t} -(b_{k_{l}}^{1},\cdot \cdot\cdot, b_{k_{l}}^{m}) \bigr)=F({\bf t} ), \end{align} pointwise for ${\bf t}\in {\mathbb R}^{m}$. \end{definition} When \(\mathrm{R}_\mathcal{G}=\mathrm{R}_\mathcal{G}^a \), by way of simplifying things, \( \mathrm{R}_\mathcal{G}^a \)-multi-almost automorphic functions will be referred to as \( \mathcal{G} \)-multi-almost automorphic functions. The concept of \( \mathcal{G} \)-multi-almost automorphic functions is not entirely new. In the one-dimensional case where \( \mathcal{G} = \mathbb{Z} \) (the discrete subgroup of \( \mathbb{R} \)), the first author and collaborators introduced the class of piecewise continuous almost automorphic functions (also known as \( \mathbb{Z} \)-almost automorphic functions) between 2013 and 2014. Initially, this class of discontinuous functions was developed to study differential equations with piecewise constant arguments \cite{chavez2014discontinuous}. Later, the approach was extended to investigate differential equations with deviating arguments, as demonstrated in works such as \cite{ding2017asymptotically, qi2022piecewise}. More recently, the theory of \( \mathbb{Z} \)-almost automorphic functions with values in a general Banach space has been explored in \cite{Alanpiecewise}. With the help of this theory, it is proved that every compact almost automorphic function is \( \mathbb{Z} \)-almost automorphic and uniformly continuous, and vice versa, providing a new characterization of the compact almost automorphic function space. Additionally, in the same work, \( \mathbb{Z} \)-almost automorphic functions were utilized to offer a straightforward proof of the characterization of discrete almost automorphic functions as restrictions to \( \mathbb{Z} \) of compact almost automorphic functions. Consequently, new applications beyond the theory of differential equations for \( \mathbb{Z} \)-almost automorphic functions have emerged. In the work \cite{es2022compact} the study of the existence and uniqueness of compact almost automorphic solutions for a class of semilinear evolution equations in Banach spaces is provided. Building on the aforementioned preliminaries, in the applications of the class of \( \mathbb{Z} \)-almost automorphic functions and recognizing the significance of almost automorphic functions in the study of differential equations, this work makes two primary contributions: \begin{itemize} \item First, we present two new characterizations of compact almost automorphic functions (Section \ref{section char}), expanding the theoretical framework established in earlier studies. \item Second, using the new characterizations, we analyze the almost automorphic dynamics of Poisson's and heat equations. \end{itemize} \noindent Let us develop into more detail regarding the second point mentioned previously. We begin by considering the Poisson's equation: \begin{equation}\label{Poisson 0} \Delta u=f, \end{equation} where \( \Delta \) denotes the Laplace operator in the spatial variable \( x = (x_1, \ldots, x_m) \), defined as \( \Delta = \sum_{i=1}^m \partial^2_{x_i} \). After characterizing compact almost automorphic functions, we establish, for equation (\ref{Poisson 0}), the following result: {\it if \( f \) is bounded and \( \mathcal{G} \)-multi-almost automorphic, where \( \mathcal{G} \) is either the lattice subgroup \( \mathbb{Z}^m \) or a dense subset of \( \mathbb{R}^m \), then every bounded and continuous function that is a distributional solution of the Poisson's equation (\ref{Poisson 0}) is compact almost automorphic}. Now, le us consider the following initial value problem for the heat equation: \begin{equation}\label{Heat equation 1} \left\{ \begin{array}{lll} \partial_t u(t,x)&=&\Delta u(t,x), \quad t>0,\, x\in \mathbb{R}^m,\\ u(0,x) &= &f(x), \quad x\in \mathbb{R}^m, \end{array}\right. \end{equation} where \( \partial_t \) denotes the partial derivative with respect to time \( t \) and \(\Delta \) is the Laplace operator in space variable. Analogously to the Poisson's equation, we prove that: {\it if the initial data \( f \) in (\ref{Heat equation 1}) is \( \mathcal{G} \)-multi-almost automorphic, where \( \mathcal{G} \) is either the lattice subgroup \( \mathbb{Z}^m \) or a dense subset of \( \mathbb{R}^m \), then the solution \( u(t,x) \) of the heat equation (\ref{Heat equation 1}) is also compact almost automorphic in space variable}. In summary, our work extends and generalizes results from previous studies such as \cite{CHAKPPINTO2023, sibuya1971almost}, emphasizing the persistence of compact almost automorphic properties in solutions to the Poisson's and heat equations under weaker conditions on the source or initial data (respectively). The organization of the present work is as follows: In Section \ref{section maa}, we provide the definition and some basic facts about \( \mathcal{G} \)-multi-almost automorphic functions. In Section \ref{section dense}, we study the case in which \( \mathcal{G} \) is a dense subgroup of \( \mathbb{R}^m \). In Section \ref{section char}, we present new characterizations of compact almost automorphic functions. In Section \ref{section conv}, we prove the invariance under convolution products with compact Bi-almost automorphic kernels of the bounded and compactly \( \mathrm{R}_{\mathcal{G}} \)-multi-almost automorphic function space. In Section \ref{section appl}, we provide applications of the developed theory to Poisson's and heat equations. Finally, the appendices contain additional results and proofs related to almost automorphic functions on topological groups, as well as conditions under which a subgroup of the Euclidean space becomes dense. \section{On $\mathcal{G}$-multi-almost automorphic functions}\label{section maa} Throughout the present work, $\mathcal{X}, \mathcal{Y}$ denote Banach spaces, we assume that $m\in {\mathbb N}$, ${\mathcal B}$ is a non-empty collection of subsets of $\mathcal{X}$, \( {\mathcal{G}} \) is a non-empty subset of ${\mathbb R}^{m}$, ${\mathrm{R}}_{{\mathcal{G}}}$ is a non-empty set of sequences in $\mathcal{G}$ and ${\mathrm{R}}_{{\mathcal{G}}}^a$ is the collection of all the sequences in $\mathcal{G}$. Usually, ${\mathcal B}$ denotes the collection of all bounded subsets of $\mathcal{X}$ or all compact subsets of $\mathcal{X}$. We will always assume that for every $x\in \mathcal{X}$, there exist $B\in {\mathcal B}$ such that $x\in B.$ $BUC(\mathbb{R}^m; \X)$ is the space of bounded and uniformly continuous functions from $\mathbb{R}^m$ to $ \X$, while $ UC(\mathbb{R}^m; \X)$ is the space of uniformly continuous functions from $\mathbb{R}^m$ to $ \X$. \begin{definition}\label{eovako}\index{function!(compactly) $({\mathrm{R}}_{{\mathcal{G}}},{\mathcal B})$-multi-almost automorphic} Suppose that $F : {\mathbb R}^{m} \times \mathcal{X} \rightarrow \mathcal{Y}$ is a continuous function. Then we say that the function $F(\cdot;\cdot)$ is $({\mathrm{R}}_{{\mathcal{G}}},{\mathcal B})$-multi-almost automorphic if, for every $B\in {\mathcal B}$ and for every sequence $({\bf b}_{k}=(b_{k}^{1},b_{k}^{2},\cdot \cdot\cdot ,b_{k}^{m})) \in {\mathrm{R}}_{{\mathcal{G}}}$ there exist a subsequence $({\bf b}_{k_{l}}=(b_{k_{l}}^{1},b_{k_{l}}^{2},\cdot \cdot\cdot , b_{k_{l}}^{m}))$ of $({\bf b}_{k})$ and a function $F^{\ast} : {\mathbb R}^{m} \times \mathcal{X} \rightarrow \mathcal{Y}$ such that \begin{align}\label{love12345678} \lim_{l\rightarrow +\infty}F\bigl({\bf t} +(b_{k_{l}}^{1},\cdot \cdot\cdot, b_{k_{l}}^{m});x\bigr)=F^{\ast}({\bf t};x) \end{align} and \begin{align}\label{love123456789} \lim_{l\rightarrow +\infty}F^{\ast}\bigl({\bf t} -(b_{k_{l}}^{1},\cdot \cdot\cdot, b_{k_{l}}^{m});x\bigr)=F({\bf t};x), \end{align} pointwise for ${\bf t}\in {\mathbb R}^{m}$ and uniformly for $x\in B$. If the above limits converge uniformly on compact subsets of ${\mathbb R}^{m}$, then we say that $F(\cdot ; \cdot)$ is compactly $({\mathrm{R}}_{{\mathcal{G}}},{\mathcal B})$-multi-almost automorphic. \end{definition} In the particular case in which ${\mathrm{R}}_{{\mathcal{G}}}={\mathrm{R}}_{{\mathcal{G}}}^a$, we say that $F(\cdot ; \cdot)$ is $({\mathcal{G}},{\mathcal B})$-multi-almost automorphic (res. compactly $({\mathcal{G}},{\mathcal B})$-multi-almost automorphic). Also note that, if $\mathcal{G}=\mathbb{R}^m$, then we are in the class of $({\mathrm{R}},{\mathcal B})$-multi-almost automorphic functions, see \cite{chavez2022multi}. Also, if the function $F({\bf t};x) =F( {\bf t} )$ (i.e., $F : {\mathbb R}^{m} \rightarrow \mathcal{Y}$), then we say that $F$ is ${\mathrm{R}}_{{\mathcal{G}}}$-multi-almost automorphic (resp. compactly ${\mathrm{R}}_{{\mathcal{G}}}$-multi-almost automorphic) and that space is denoted by $ {{\mathrm{R}}_{{\mathcal{G}}}}AA({\mathbb R}^{m} ; \mathcal{Y})$ (resp. $ \mathcal{K} {\mathrm{R}}_{{\mathcal{G}}}AA({\mathbb R}^{m} ; \mathcal{Y})$). In this situation, if ${\mathrm{R}}_{{\mathcal{G}}}={\mathrm{R}}_{{\mathcal{G}}}^a$, $F(\cdot)$ will be called ${\mathcal{G}}$-multi-almost automorphic (resp. compactly ${\mathcal{G}}$-multi-almost automorphic), see Definition \ref{eovako00}; and the space is denoted by $ {{\mathcal{G}}}AA({\mathbb R}^{m} ; \mathcal{Y})$ (resp. $ \mathcal{K} {\mathcal{G}}AA({\mathbb R}^{m} ; \mathcal{Y})$). Also, $ AA({\mathbb R}^{m} ; \mathcal{Y})$ (resp. $ \mathcal{K}AA({\mathbb R}^{m} ; \mathcal{Y})$) will denote the space of (classical) almost automorphic functions (resp. compactly-almost automorphic functions). \begin{prop}\label{eovako54321}\index{function!(compactly) $({\mathrm{R}}_{{\mathcal{G}}},{\mathcal B})$-multi-almost automorphic} Suppose that $F : {\mathbb R}^{m} \times \mathcal{X} \rightarrow \mathcal{Y}$ is a continuous function. Then the function $F(\cdot;\cdot)$ is $({\mathrm{R}}_{{\mathcal{G}}},{\mathcal B})$-multi-almost automorphic if and only if for every $B\in {\mathcal B}$ and for every sequence $({\bf b}_{k}=(b_{k}^{1},b_{k}^{2},\cdot \cdot\cdot ,b_{k}^{m})) \in {\mathrm{R}}_{{\mathcal{G}}}$ there exists a subsequence $({\bf b}_{k_{l}}=(b_{k_{l}}^{1},b_{k_{l}}^{2},\cdot \cdot\cdot , b_{k_{l}}^{m}))$ of $({\bf b}_{k})$ such that \begin{align}\label{snajkamlada} \lim_{l\rightarrow +\infty}\lim_{n\rightarrow +\infty} F\Bigl({\bf t} - {\bf b}_{k_{l}}+{\bf b}_{k_{n}} ;x\Bigr)=F({\bf t};x) , \end{align} pointwise for ${\bf t}\in {\mathbb R}^{m}$ and all $x\in B$. \end{prop} \begin{proof} It is not difficult to see that if $F$ is $({\mathrm{R}}_{{\mathcal{G}}},{\mathcal B})$-multi-almost automorphic, then given any set $B\in {\mathcal B}$ and any sequence $({\bf b}_{k}=(b_{k}^{1},b_{k}^{2},\cdot \cdot\cdot ,b_{k}^{m})) \in {\mathrm{R}}_{{\mathcal{G}}}$, there exists a subsequence $({\bf b}_{k_{l}}=(b_{k_{l}}^{1},b_{k_{l}}^{2},\cdot \cdot\cdot , b_{k_{l}}^{m}))$ of $({\bf b}_{k})$ such that (\ref{snajkamlada}) holds. On the other hand, let $({\bf b}_{k}=(b_{k}^{1},b_{k}^{2},\cdot \cdot\cdot ,b_{k}^{m})) \in {\mathrm{R}}_{{\mathcal{G}}}$ and $B\in {\mathcal B}$, by hypothesis, there exists a subsequence $({\bf b}_{k_{l}}=(b_{k_{l}}^{1},b_{k_{l}}^{2},\cdot \cdot\cdot , b_{k_{l}}^{m}))$ of $({\bf b}_{k})$ such that (\ref{snajkamlada}) holds. Now, let us define, for each fixed $l$, each ${\bf t}\in \mathbb{R}^m$ and any $x\in B$, the function $$F^*({\bf t} -{\bf b}_{k_{l}};x):=\lim_{n\rightarrow +\infty} F\Bigl({\bf t} -{\bf b}_{k_{l}}+{\bf b}_{k_{n}} ;x\Bigr)\, . $$ Then, for each ${\bf t} \in \mathbb{R}^m$, since ${\bf t}={\bf t} +{\bf b}_{k_{l}} - {\bf b}_{k_{l}}$ we have $$F^*({\bf t};x)=\lim_{n\rightarrow +\infty} F\Bigl({\bf t} + {\bf b}_{k_{n}} ;x\Bigr)\, ,$$ uniformly for $x\in B$. On the other hand, we have $$ \lim_{n\rightarrow +\infty}F^*({\bf t}- {\bf b}_{k_{n}};x)=\lim_{l\rightarrow +\infty}F^*({\bf t} - {\bf b}_{k_{l}};x)=$$ $$=\lim_{l\rightarrow +\infty}\lim_{n\rightarrow +\infty} F\Bigl({\bf t} - {\bf b}_{k_{l}}+{\bf b}_{k_{n}} ;x\Bigr)=F\Bigl({\bf t } ;x\Bigr)\, ,$$ point-wisely in ${\bf t} \in \mathbb{R}^m$ and uniformy for $x\in B$. Thus, $F$ is $({\mathrm{R}}_{{\mathcal{G}}},{\mathcal B})$-multi-almost automorphic. \end{proof} \begin{definition}\label{UnifContF} Let $F : {\mathbb R}^{m} \times \mathcal{X} \rightarrow \mathcal{Y}$. Then, the function $F(\cdot;\cdot)$ is called uniformly continuous in $\mathbb{R}^m$, uniformly for any $B\in {\mathcal B}$ if, for every $B\in {\mathcal B}$ and every $\epsilon>0$, there exists $\delta =\delta (\epsilon,B)>0$, such that, if $\t,\s \in \mathbb{R}^m$ with $||\t-\s||<\delta$, then $$ \sup_{x\in B}\| F \bigl({\bf t};x\bigr) - F\bigl({\bf s};x\bigr) \|_{\mathcal{Y}}<\epsilon\, . $$ \end{definition} In this situation, we will say that $F : {\mathbb R}^{m} \times \mathcal{X} \rightarrow \mathcal{Y}$ is uniformly continuous in the first variable. In the following definition, we introduce the notion of $({\mathrm{R}}_{{\mathcal{G}}},{\mathcal B})$-uniformly continuous function. \begin{definition}\label{(R,B) uniformly continuous} Let $F : {\mathbb R}^{m} \times \mathcal{X} \rightarrow \mathcal{Y}$. Then, the function $F(\cdot;\cdot)$ is called $({\mathrm{R}}_{{\mathcal{G}}},{\mathcal B})$-uniformly continuous if, for every $B\in {\mathcal B}$ and for every two sequences $({\bf a}_{k}=(a_{k}^{1},a_{k}^{2},\cdots ,a_{k}^{m}))$, $ ({\bf b}_{k}=(b_{k}^{1},b_{k}^{2},\cdot \cdot\cdot ,b_{k}^{m}))$ $ \in {\mathrm{R}}_{{\mathcal{G}}}$ such that $ ||{\bf a}_{k}-{\bf b}_{k} || \rightarrow 0$, as $k\to +\infty$, we have \begin{align} \lim_{k\rightarrow +\infty } \sup_{x\in B}\| F \bigl({\bf t}+{\bf a}_{k};x\bigr) - F\bigl({\bf t}+{\bf b}_{k};x\bigr) \|_{\mathcal{Y}} \rightarrow 0, \end{align} pointwise for ${\bf t} \in \mathbb{R}^m$.\end{definition} Note that, if ${\mathcal{G}}=\mathbb{R}^m$ and ${\mathrm{R}}_{{\mathcal{G}}}={\mathrm{R}}_{{\mathcal{G}}}^a$, then Definition \ref{UnifContF} and Definition \ref{(R,B) uniformly continuous} are equivalent. The next theorem gives a useful consequence for functions that are compactly $({\mathrm{R}}_{{\mathcal{G}}},{\mathcal B})$-multi-almost automorphic. Its proof is inspired by \cite[Lemma 3.3]{chavez2024compact}. The theorem is used to prove, for instance, the uniform continuity of functions defined by integrals with kernels depending on two variables (see for example Theorems \ref{NewThmappl} and \ref{InvConv1}). \begin{tw}\label{ThRKAA} Let $F : {\mathbb R}^{m} \times \mathcal{X} \rightarrow \mathcal{Y}$ be bounded and continuous. If $F$ is compactly $({\mathrm{R}}_{{\mathcal{G}}},{\mathcal B})$-multi-almost automorphic, then $F$ is $({\mathrm{R}}_{{\mathcal{G}}},{\mathcal B})$-multi-almost automorphic and $({\mathrm{R}}_{{\mathcal{G}}},{\mathcal B})$-uniformly continuous. \end{tw} \begin{proof} Since \( F \) is compactly \( ({\mathrm{R}}_{{\mathcal{G}}},{\mathcal{B}}) \)-multi-almost automorphic, it follows that \( F \) is \( ({\mathrm{R}}_{{\mathcal{G}}},{\mathcal{B}}) \)-multi-almost automorphic. Now, let us prove that \( F \) is \( ({\mathrm{R}}_{{\mathcal{G}}},{\mathcal{B}}) \)-uniformly continuous. Let \( B \) be any set in \( \mathcal{B} \), and let us pick the arbitrary sequences \( {\bf a}_{k} = (a_{k}^{1}, a_{k}^{2}, \cdots , a_{k}^{m}) \) and \( {\bf b}_{k} = (b_{k}^{1}, b_{k}^{2}, \cdots , b_{k}^{m}) \in {\mathrm{R}}_{{\mathcal{G}}} \) which satisfy \( ||{\bf a}_{k} - {\bf b}_{k} || \rightarrow 0 \) as \( k \to \infty \). We must show that, for each \( {\bf t} \in \mathbb{R}^m \): \begin{align*} \lim_{k\to \infty}\sup_{x\in B}\| F \bigl({\bf t}+{\bf a}_{k};x\bigr) - F\bigl({\bf t}+{\bf b}_{k};x\bigr) \|_{\mathcal{Y}} = 0\, . \end{align*} Let us define for each $({\bf t};x)$ the uniformly bounded sequence $$ \alpha_{k}({\bf t};x):= \| F \bigl({\bf t}+{\bf a}_{k};x\bigr) - F\bigl({\bf t}+{\bf b}_{k};x\bigr) \|_{\mathcal{Y}}\, .$$ We claim that $\alpha_{k}({\bf t},x) \to 0$ as $k\to \infty$. In fact, let $( \alpha_k'({\bf t},x)) \subset (\alpha_k({\bf t},x))$ be a convergent subsequence which converges to $\psi_0({\bf t},x)$, where $$ \alpha_k'({\bf t},x)= \| F \bigl({\bf t}+{\bf a}_{k}^{'};x\bigr) - F\bigl({\bf t}+{\bf b}_{k}^{'};x\bigr) \|_{\mathcal{Y}}$$ and $ ({\bf a}_{k}^{'}) \subset ({\bf a}_{k})$, $({\bf b}_{k}^{'}) \subset ({\bf b}_{k})$ are subsequences coming from the definition of $ \alpha_k'({\bf t},x)$. Since $F$ is compactly $({\mathrm{R}}_{{\mathcal{G}}},{\mathcal B})$-multi-almost automorphic, there exist a subsequence $({\bf b}_{k_{l}}=(b_{k_{l}}^{1},b_{k_{l}}^{2},\cdot \cdot\cdot , b_{k_{l}}^{m}))$ of $({\bf b}_{k}^{'})$ (also a subsequence $({\bf a}_{k_{l}})$ of $({\bf a}_{k}^{'})$) and a continuous (in the first variable) function $F^{\ast} : {\mathbb R}^{m} \times \mathcal{X} \rightarrow \mathcal{Y}$ such that the following limits hold \begin{align}\label{love12345678} \lim_{l\rightarrow +\infty} \sup_{t\in \mathcal{E}} \| F\bigl({\bf t} +{\bf b}_{k_{l}};x\bigr)-F^{\ast}({\bf t};x) \| \end{align} and \begin{align}\label{love123456789} \lim_{l\rightarrow +\infty} \sup_{t\in \mathcal{E}} \| F^{\ast}\bigl({\bf t} -{\bf b}_{k_{l}};x\bigr)-F({\bf t};x)\|, \end{align} which are also uniformly in $x\in B$ and, $\mathcal{E}$ is any compact subset of $\mathbb{R}^m$ On the other hand, because $(||{\bf a}_{k}-{\bf b}_{k}||)$ is bounded, there exists $R>0$ such that for all $l\in \mathbb{ N}$, ${\bf c}_{k_{l}}:={\bf a}_{k_{l}}-{\bf b}_{k_{l}} \in \overline{ B(0,R)}$ (the closed ball with center $0$ and radius $R$). Therefore, we have: \begin{align*} \| F \bigl({\bf t}+{\bf a}_{k_{l}};x\bigr) &- F\bigl({\bf t}+{\bf b}_{k_{l}};x\bigr) \|_{\mathcal{Y}} \leq \\ &\leq \| F \bigl({\bf t}+{\bf c}_{k_{l}}+{\bf b}_{k_{l}};x\bigr) - F^{\ast}\bigl({\bf t}+{\bf c}_{k_{l}};x\bigr) \|_{\mathcal{Y}} +\\ &+ \|F \bigl({\bf t}+{\bf b}_{k_{l}};x\bigr)- F^{\ast}\bigl({\bf t};x\bigr) \|_{\mathcal{Y}} +\|F^{\ast} \bigl({\bf t};x\bigr)- F^{\ast}\bigl({\bf t}+{\bf c}_{k_{l}} ;x\bigr) \|_{\mathcal{Y}} \\ &\leq \sup_{{\bf z}\in \overline{ B(0,R)}+{\bf t}}\| F \bigl({\bf z}+{\bf b}_{k_{l}};x\bigr) - F^{\ast}\bigl({\bf z};x\bigr) \|_{\mathcal{Y}} +\\ &+ \|F \bigl({\bf t}+{\bf b}_{k_{l}};x\bigr)- F^{\ast}\bigl({\bf t};x\bigr) \|_{\mathcal{Y}} +\|F^{\ast} \bigl({\bf t};x\bigr)- F^{\ast}\bigl({\bf t}+{\bf c}_{k_{l}} ;x\bigr) \|_{\mathcal{Y}} . \end{align*} Now, since $F(\cdot,\cdot)$ is compactly $({\mathrm{R}}_{{\mathcal{G}}},{\mathcal B})$-multi-almost automorphy and $F^{\ast} \bigl(\cdot;x\bigr)$ is continuous, we have (using the previous inequality) $$\lim_{l \to \infty}\sup_{x\in B} \| F \bigl({\bf t}+{\bf a}_{k_{l}};x\bigr) - F\bigl({\bf t}+{\bf b}_{k_{l}};x\bigr) \|_{\mathcal{Y}} = 0\, .$$ Therefore, $\psi_0({\bf t},x)=0$. Now, since every convergent subsequence of the bounded sequence $\alpha_{k}({\bf t};x)$ converge towards zero, we conclude $$\lim_{k \to \infty} \sup_{x\in B} \| F \bigl({\bf t}+{\bf a}_{k};x\bigr) - F\bigl({\bf t}+{\bf b}_{k};x\bigr) \|_{\mathcal{Y}} =0\, .$$ \end{proof} Of course, as in the classical situation, we have the next two results \begin{prop}\label{prou} Let $F:\mathbb{R}^m \times\X \to \mathcal{Y}$ be $({\mathrm{R}}_{{\mathcal{G}}},{\mathcal B})$-multi-almost automorphic and uniformly continuous in the first variable; then, its limit function $F^*$(see definition \ref{eovako}) is also uniformly continuous in the first variable. \end{prop} \begin{proof} Let $(b_k) \in {\mathrm{R}}_{{\mathcal{G}}}$ and $B\in \mathcal{B}$, since $F$ is $({\mathrm{R}}_{{\mathcal{G}}},{\mathcal B})$-multi-almost automorphic there exist a subsequence $(b_{k_l})\subset(b_k)$ and a function $F^*:\mathbb{R}^m \times \X \to \mathcal{Y}$ such that the following limits, point-wise in ${\bf t}$ and uniformly in $x\in B$, hold $$\lim_{l\rightarrow\infty}{F(\t+b_{k_l};x)=F^*(\t;x)}\, ,$$ $$\lim_{l\rightarrow\infty}{F^*(\t-b_{k_l};x)=F(\t;x)}.$$ Now, let $\epsilon > 0$, since $F$ is uniformly continuous in the first variable, there exists $\delta > 0$ such that if $\t,\s \in \mathbb{R}^m$ with $||\t-\s||<\delta$, then $\sup_{x\in B}||F(\t;x) - F(\s ;x)||_{\mathcal{Y}}<\epsilon/2$. Therefore, taking the limit when $l\rightarrow\infty$ in the following inequality \begin{eqnarray*} ||F^*(\t;x) - F^*(\s;x)||_{\mathcal{Y}} &\leq & || F^*(\t;x) - F(\t+b_{k_l};x)||_{\mathcal{Y}} + ||F(\t+b_{k_l};x) - F(\s + b_{k_l};x)||_{\mathcal{Y}} +\\ &+& ||F(\s + b_{k_l};x)-F^*(\s ;x)||_{\mathcal{Y}}\, , \end{eqnarray*} we have: $$\sup_{x\in B}||F^*(\t,x)-F^*(\s,x)||_{\mathcal{Y}} < \epsilon\, ,$$ provided that $\t,\s \in \mathbb{R}^m$ with $||\t-\s||<\delta$. Thus, $F^*$ is uniformly continuous in the first variable. \end{proof} The proof of the following proposition is analogous to that of the previous one, so we omit its proof. \begin{prop} Suppose that \( F : {\mathbb R}^{m} \times \mathcal{X} \rightarrow \mathcal{Y} \) is an \(({\mathrm{R}}_{{\mathcal{G}}}, {\mathcal{B}})\)-multi-almost automorphic function with limit function \( F^* \) (see Definition \ref{eovako}). If for every \( B \in {\mathcal{B}} \) there exists a real number \( L_{B} > 0 \) such that, for every \( x \in B \) and \( {\bf s}, {\bf t} \in {\mathbb R}^{m} \), we have \[ \bigl\| F({\bf s}; x) - F({\bf t}; x) \bigr\|_{\mathcal{Y}} \leq L_{B} \|{\bf s} - {\bf t}\|_{\mathcal{X}}, \] then, we also have \[ \bigl\| F^*({\bf s}; x) - F^*({\bf t}; x) \bigr\|_{\mathcal{Y}} \leq L_{B} \|{\bf s} - {\bf t}\| , \] for every $x\in B$ and ${\bf s}, {\bf t} \in \mathbb{R}^m$. \end{prop} \noindent Following the work \cite{chavez2022multi}, we have: \begin{tw}\label{Follprev} The following properties hold \begin{enumerate} \item The space of bounded and ${\mathrm{R}}_{{\mathcal{G}}}$-multi-almost automorphic functions is a Banach space under the supremum norm. \item Suppose that $F : {\mathbb R}^{m} \times \mathcal{X} \rightarrow \mathcal{Y}$ is $(\mathcal{G},{\mathcal B})$-multi-almost automorphic, where ${\mathcal B}$ denotes any collection of compact subsets of $\mathcal{X}.$ If for every $B\in {\mathcal B}$ there exists a real number $L_{B}>0$ such that, for every $x,\ y\in B$ and ${\bf t}\in {\mathbb R}^{m},$ we have \begin{align}\label{elbe} \bigl\| F({\bf t};x) -F({\bf t} ;y)\bigr\|_{\mathcal{Y}}\leq L_{B}\|x-y\|_{\mathcal{X}} ; \end{align} then, for every set $B\in {\mathcal B},$ the set $\{F({\bf t},x) : {\bf t}\in \mathcal{G},\ x\in B\}$ is relatively compact in $\mathcal{Y}.$ \end{enumerate} \end{tw} In Theorem \ref{Follprev}, the proof of part {\it 1} is analogous to the classical version of almost automorphy, see for instance \cite{CHAKPPINTO2023}, while the proof of item {\it 2} is analogous to \cite[Proposition 2.5-(i)]{chavez2022multi}. With respect to the previous proposition, we clarify the following: in item {\it 1}, the completeness is of the space \( BC(\mathbb{R}^m; \mathcal{X}) \cap \mathrm{R}_{\mathcal{G}} A A(\mathbb{R}^m; \mathcal{X}) \) and not just of the space \( \mathrm{R}_{\mathcal{G}} A A(\mathbb{R}^m; \mathcal{X}) \), this is due to the fact that, in general, the elements of \( \mathrm{R}_{\mathcal{G}} A A(\mathbb{R}^m; \mathcal{X}) \) are not bounded (see the examples presented in \cite{chavez2022multi}). With respect to item {\it 2}, we clarify that we are working with all the sequences in \( \mathcal{G}\), so that when \( \mathcal{G} = \mathbb{R}^m \), we are dealing with the classical situation of almost automorphy, as in \cite[Proposition 2.5-(i)]{chavez2022multi}. In the next section we prove that if \( \mathcal{G} \) is a dense subgroup of \( \mathbb{R}^m \), then every function in \( \mathcal{G} A A(\mathbb{R}^m; \mathcal{X}) \) is bounded and has relatively compact range. \section{$\mathcal{G}$-multi-almost automorphic functions, where $\mathcal{G}$ is a dense subgroup of $\mathbb{R}^m$}\label{section dense} As quoted in the previous section, we know that an $({\mathrm R}_{\mathcal{G}},{\mathcal B})$-multi-almost automorphic function is not necessarily bounded and, therefore, does not have a relatively compact range. In the next theorem, we overcome this situation by requiring $\mathcal{G}$ to be a dense subgroup of $\mathbb{R}^m$. \begin{tw}\label{teo1} Let $\mathcal{G} \subset \mathbb{R}^m$ be a dense subgroup and $f \in \mathcal{G}AA(\mathbb{R}^m ; \mathcal{X})$, then: \begin{enumerate} \item $f$ is bounded. \item $f$ has relatively compact range. Thus, $\mathcal{R}(f):=\{f({\bf t}) : {\bf t}\in \mathbb{R}^m\}$ is relatively compact in $\mathcal{X}$. \end{enumerate} \end{tw} \begin{proof} Let us see: \begin{enumerate} \item [{\it 1.}] Let $f_r := f|_{\mathcal{G}} : \mathcal{G} \rightarrow \X$ be the restriction of $f$ to $\mathcal{G}$, which is a continuous function. We note that $f_r$ is almost automorphic on $\mathcal{G}$ and, according to Theorem \ref{pro1}, $f_r$ is bounded on $\mathcal{G}$. Consequently, due to the continuity of $f$, the boundedness of $f_r$ and the density of $\mathcal{G}$ in $\mathbb{R}^m$, it follows that $f$ is bounded on $\mathbb{R}^m$. \item [{\it 2.}] Since $\mathcal{G}$ is dense in $\mathbb{R}^m$, according to Proposition \ref{grp}, we have: \begin{equation}\label{Eqnclos} \overline{f(\mathbb{R}^m)} = \overline{f(\overline{\mathcal{G}})} = \overline{f_r(\mathcal{G})}, \end{equation} where $f_r$ is the restriction of $f$ to the group $\mathcal{G}$. Moreover, since $f_r: \mathcal{G} \rightarrow \X$ is almost automorphic, by Theorem \ref{pro2}, $f_r$ has a relatively compact range, that is $\overline{f_r(\mathcal{G})}$ is compact in $\mathcal{X}$. Therefore, from (\ref{Eqnclos}), we conclude that $\overline{f(\mathbb{R}^m)}$ is compact in $\mathcal{X}$. \end{enumerate} \end{proof} \begin{cor} Let $\mathcal{G}$ be a dense subgroup of $\mathbb{R}^m$, then \begin{enumerate} \item $\mathcal{G}AA(\mathbb{R}^m ; \mathcal{X}) \subset BC(\mathbb{R}^m ; \mathcal{X});$ where $BC(\mathbb{R}^m ; \mathcal{X})$ is the space of bounded continuous functions. \item $\mathcal{G}AA(\mathbb{R}^m ; \X)$ is a Banach space under the supremum norm. \end{enumerate} \end{cor} In the following result, we establish that if $\mathcal{G}$ is a dense subgroup of $\mathbb{R}^m$, then any $\mathcal{G}$-multi-almost automorphic function which decays asymptotically across $\mathcal{G}$ must be the null function. \begin{tw}\label{teo6} Let $\mathcal{G}$ be a dense subgroup of $\mathbb{R}^m$ and $f\in \mathcal{G}AA(\mathbb{R}^m ; \X)$ such that $$\displaystyle\lim_{\t \in \mathcal{G},\, ||\t||\rightarrow\infty}{||f(\t)||_{\X}} = 0\, ;$$ then, $f \equiv 0$. \end{tw} \begin{proof} Since $f\in \mathcal{G} AA(\mathbb{R}^m ;\mathcal{X})$, then its rectriction map $f_r:\mathcal{G}\to \X$ is almost automorphic on the group $\mathcal{G}$. On the other hand, according to Lemma \ref{lem8}, $\mathcal{G}$ is unbounded. Therefore, there exist a sequence $(b_k)_{k\in\mathbb{N}}$ in $\mathcal{G}$ which is unbounded. Now, since $f_r$ is almost automorphic, there exit a subsequence $(b_{k_l})_{l\in\mathbb{N}}\subset (b_k)_{k\in\mathbb{N}}$ and a function $f_r^*:\mathcal{G} \to \mathcal{X}$ such that for each $\t \in \mathcal{G}$, the following limits hold: \begin{equation}\label{eq1} f_r^*(\t) = \lim_{l \rightarrow \infty}{f_r(\t+b_{k_l})}\, , \end{equation} \begin{equation}\label{eq2} \lim_{l \rightarrow \infty}{f_r^*(\t-b_{k_l})} = f_r(\t)\, . \end{equation} But, $\displaystyle\lim_{\t \in \mathcal{G},\, ||\t||\rightarrow\infty}{||f(\t)||_{\X}} = 0;$ then, in light of (\ref{eq1}), we have: $$f_r^*(\t) = 0\, , \t\in \mathcal{G}\, .$$ Now using (\ref{eq2}), we have: $$f_r(\t)=\lim_{l \rightarrow \infty}{f_r^*(\t-b_{k_l})}=0\, , \t\in \mathcal{G}\, . $$ Thus, $f_r \equiv 0$ in $\mathcal{G}$. Now the conclusion follows from the continuity of $f$ and the denseness of the group $\mathcal{G}$. \end{proof} \section{New characterizations of compact almost automorphic functions}\label{section char} In this section, we present the main results of the present work. We begin with the following well-known characterization of multidimensional compact almost automorphic functions presented in \cite{CHAKPPINTO2023}. \begin{tw}\label{crt} ({\bf First characterization}) A function $f:\mathbb{R}^{m}\to \mathcal{X}$ is compact almost automorphic if and only if it is almost automorphic and uniformly continuous. Thus, $$\mathcal{K}AA(\mathbb{R}^m ; \X) = AA(\mathbb{R}^m ; \X) \cap UC(\mathbb{R}^m ; \X) .$$ \end{tw} The next characterization, where we consider $\mathcal{G}=\mathbb{Z}^m$, is a generalization to the multidimensional setting of \cite[Theorem 3.16]{Alanpiecewise}. \begin{tw} ({\bf Second characterization}) A function $f: \mathbb{R}^m \rightarrow \X$ is compact almost automorphic if and only if it is $\mathbb{Z}^m$-almost automorphic and uniformly continuous, that is: $$\mathcal{K}AA(\mathbb{R}^m ; \X) = \mathbb{Z}^mAA(\mathbb{R}^m ; \X) \cap UC(\mathbb{R}^m ; \X) .$$ \end{tw} \begin{proof} Let us see\\ \noindent i) Let $f$ be a compact almost automorphic function, then $f$ is $\mathbb{Z}^m$-almost automorphic and, according to Theorem \ref{crt}, it is uniformly continuous. Therefore we have the inclusion $\mathcal{K}AA(\mathbb{R}^m ; \X)\subset \mathbb{Z}^mAA(\mathbb{R}^m ; \X) \cap UC(\mathbb{R}^m ; \X).$ \\ \noindent ii) Let us prove the reverse inclusion. Consider $f$ a $\mathbb{Z}^m$-almost automorphic and uniformly continuous function; then, according to Theorem \ref{crt}, we need to prove that $f$ is almost automorphic.\\ Let $({\bf b}_{k}=(b_{k}^{1},b_{k}^{2},\cdot \cdot\cdot ,b_{k}^{m}))_{k\in \mathbb{N}}$ be an arbitrary sequence in $\mathbb{R}^m$. Denoting by $[|\cdot |]$ the integer part of a real umber and by $\{\cdot \}$ the decimal part, then for each $j\in \{1,2,\cdots , m\}$ we have the decomposition $b_k^j=[|b_k^j|]+\{b_k^j\}$, where $[|b_k^j|]\in \mathbb{Z}$ and $\{b_k^j\} \in [0,1)$. In this way we have \begin{eqnarray*} {\bf b}_k &=& \underbrace{([|b_k^1|],[|b_k^2|],\cdots,[|b_k^m|])}_{\in \mathbb{Z}^m} + \underbrace{(\{b_k^1\},\{b_k^2\},\cdots,\{b_k^m\})}_{\in [0,1)^m}. \end{eqnarray*} Since $f \in \mathbb{Z}^mAA(\mathbb{R}^m,\X)$, there exist a subsequence $(\alpha_k)_{k \in\mathbb{N}}$ of $([|b_k^1|],[|b_k^2|],\cdots,[|b_k^m|])_{k \in\mathbb{N}}$ and a function $f^*:\mathbb{R}^m \rightarrow \X$ such that the following pointwise limits hold $$\lim_{k\rightarrow\infty}{f(\t+\alpha_k)=f^*(\t)}\, , \t \in \mathbb{R}^m\, ,$$ $$\lim_{k\rightarrow\infty}{f^*(\t-\alpha_k)=f(\t)}\, , \t \in \mathbb{R}^m\, .$$ On the other hand, since for each $k\in \mathbb{N}$, $(\{b_k^1\},\{b_k^2\},\cdots,\{b_k^m\}) \in [0,1)^m$, then the sequence $((\{b_k^1\},\{b_k^2\},\cdots,\{b_k^m\}))_{k\in \mathbb{N}}$ is bounded. Therefore, there exists a convergent subsequence $(\beta_k=(\beta_k^1,\beta_k^2,\cdots,\beta_k^m))_{k\in \mathbb{N}}$ of $(\{b_k^1\},\{b_k^2\},\cdots,\{b_k^m\}))_{k\in \mathbb{N}}$ which converges to the point $\beta_0=(\beta_0^1,\beta_0^2,\cdots,\beta_0^m) \in [0,1]^m$. In this way, it is possible to construct a subsequence $(\eta_k)_{k \in \mathbb{N}} \subset ({\bf b}_k)_{k\in \mathbb{N}}$, defined by $$\eta_k := (\alpha_k^1,\alpha_k^2,\cdots,\alpha_k^m)+(\beta_k^1,\beta_k^2,\cdots,\beta_k^m)=\alpha_k + \beta_k \, .$$ Now, define the function $h:\mathbb{R}^m\to \X$ by $h(\t):= f^*(\t+\beta_0)$. \noindent Let $\t \in \mathbb{R}^m$ be fixed, we have \begin{eqnarray*} ||f(\t+\eta_k)-h(\t)||_{\X}\leq ||f(\t+\alpha_k+\beta_k)-f(\t+\alpha_k+\beta_0)||_{\X}+||f(\t+\alpha_k+\beta_0)-h(\t)||_{\X}\, . \end{eqnarray*} Since \(\beta_k \to \beta_0\) and \(||f(\t + \beta_0 + \alpha_k) - h(\t)||_{\mathcal{X}} \rightarrow 0\) when \(k \rightarrow \infty\) and due to the uniform continuity of \(f\), we conclude (according to the previous inequality) that: $$\lim_{k\rightarrow \infty}{f(\t+\eta_k)=h(\t)}.$$ On the other hand, according to Proposition \ref{prou}, we know that \( f^* \) is uniformly continuous. Then, arguing as before, it can be proved that $$\lim_{k\rightarrow \infty}{h(\t-\eta_k)=f(\t)}\, .$$ \end{proof} \begin{tw} ({\bf Third characterization}) Let $f: \mathbb{R}^m\to \mathcal{X}$ be a continuous function and $\mathcal{G}$ a dense subset of $\mathbb{R}^m$. Then, $f$ is compact almost automorphic if and only if it is $\mathcal{G}$-almost automorphic and uniformly continuous. Thus, $$\mathcal{K}AA(\mathbb{R}^m ; \X) = \mathcal{G}AA(\mathbb{R}^m ; \X) \cap UC(\mathbb{R}^m ; \X) .$$ \end{tw} \begin{proof} The inclusion $\mathcal{K}AA(\mathbb{R}^m ; \X) \subset \mathcal{G}AA(\mathbb{R}^m ; \X) \cap UC(\mathbb{R}^m ; \X)$ follows from Theorem \ref{crt}. Let us prove the reverse inclusion. Let $f \in \mathcal{G}AA(\mathbb{R}^m ; \X) \cap UC(\mathbb{R}^m ; \X)$, according to Theorem \ref{crt} we need to prove that $f$ is almost automorphic. Let $({\bf b}_k) \subset \mathbb{R}^m$ be an arbitrary sequence and $\epsilon>0$. For each $k\in \mathbb{N}$, let $B^*({\bf b}_k,\epsilon/2^k):=B({\bf b}_k,\epsilon/2^k)\setminus \{{\bf b_k} \}$, where $B({\bf b}_k,\epsilon/2^k)$ is the open ball with center $ {\bf b}_k$ and radius $\epsilon /2^k$. Since $\mathcal{G}$ is dense in $\mathbb{R}^m$, for each $k\in \mathbb{N}$, there exists a point $\gamma_k \in B^*({\bf b}_k,\epsilon/2^k)\cap \mathcal{G}$. Equivalently, there exists a sequence $(\gamma_{k})$ in $\mathcal{G}$ such that $\gamma_{k}\not = {\bf b}_k $ and $||\gamma_{k}-{\bf b}_k||<\epsilon/2^k$, which implies that $\lim_{k\to \infty}||\gamma_{k}-{\bf b}_k||=0$. Since $f$ is $\mathcal{G}$-almost automorphic, there exist a subsequence $(\gamma_{k_l}) \subset (\gamma_{k})$ and a function $f^*:\mathbb{R}^m \rightarrow \X$ such that the following pointwise limits hold \begin{equation}\label{EeQqNnWw} \lim_{l\rightarrow\infty}{f(\t+\gamma_{k_l})=f^*(\t)}\, , \, \, \, \lim_{l\rightarrow\infty}{f^*(\t-\gamma_{k_l})=f(\t)}\, , \, \, {\bf t} \in \mathbb{R}^m\, . \end{equation} From Proposition \ref{prou}, it follows that $f^*$ is also uniformly continuous. On the other hand, there exists a subsequence $({\bf b}_{k_l}) \subset ({\bf b}_k)$ such that $\lim_{l\to \infty}||\gamma_{k_l}-{\bf b}_{k_l}||=0$. Now, from the following inequalities \begin{eqnarray*} ||f(\t+{\bf b}_{k_l})-f^*(\t)||_{\mathcal{X}}&\leq & ||f(\t+{\bf b}_{k_l})-f(\t+\gamma_{k_l})||_{\mathcal{X}}+\\ &+& ||f(\t+\gamma_{k_l})-f^*(\t) ||_{\mathcal{X}}\, , \end{eqnarray*} \begin{eqnarray*} ||f^*(\t-{\bf b}_{k_l})-f(\t)||_{\mathcal{X}}&\leq & ||f^*(\t-{\bf b}_{k_l})-f^*(\t-\gamma_{k_l})||_{\mathcal{X}}+\\ &+& ||f^*(\t-\gamma_{k_l})-f(\t) ||_{\mathcal{X}}\, , \end{eqnarray*} the uniform continuity of $f$, of $f^*$ and the limits in (\ref{EeQqNnWw}) we conclude that $$\lim_{l\rightarrow\infty}{f(\t+ {\bf b}_{k_l})=f^*(\t)}\, ; $$ and, $$\lim_{l\rightarrow\infty}{f^*(\t-{\bf b}_{k_l})=f(\t)}.$$ Therefore $f$ is almost automorphic. \end{proof} \section{Convolution products}\label{section conv} Using the notation $\mathcal{I}_{{\bf t}}=(-\infty,t_1]\times (-\infty,t_2]\times \cdots \times (-\infty,t_m]$ for a point ${\bf t} = (t_1,t_2,\cdots , t_m) \in \mathbb{R}^m$, we have: \begin{tw}\label{TI0101} \cite{chavez2022multi} \begin{enumerate} \item Let $f:\mathbb{R}^{m}\rightarrow \X$ be a bounded ${\mathrm R}_{\mathcal{G}}$-multi-almost automorphic function. Let $(K({\bf t}))_{{\bf t}\in (0,\infty)^{m}}\subseteq L(\X, \mathcal{Y})$ be a strongly continuous operator family and $$\int_{(0,\infty)^{m}}\| K({\bf t})\|_{L(\X,\mathcal{Y})}\, d {\bf t}\, < +\infty .$$ Define \begin{equation}\label{rep01} F({\bf t}):=\int_{\mathcal{I}_{\bf t}}K({\bf t}-\eta)f(\eta)\, d\eta,\quad {\bf t}\in {\mathbb R}^{m}. \end{equation} Then $F(\cdot)$ is a bounded ${\mathrm R}_{\mathcal{G}}$-multi-almost automorphic function. \item Let $f:\mathbb{R}^{m}\rightarrow \X$ be a bounded (compactly) ${\mathrm R}_{\mathcal{G}}$-multi-almost automorphic function. Let $(K({\bf t}))_{{\bf t}\in \mathbb{R}^{m}}\subseteq L(\X, \mathcal{Y})$ be a strongly continuous operator family and $$\int_{\mathbb{R}^{m}}\| K({\bf t})\|_{L(\X,\mathcal{Y})}\, d {\bf t}\, < +\infty .$$ Define \begin{equation}\label{rep02} F({\bf t}):=\int_{\mathbb{R}^m}K({\bf t}-\eta)f(\eta)\, d\eta,\quad {\bf t}\in {\mathbb R}^{m}. \end{equation} Then $F(\cdot)$ is a bounded (compactly) ${\mathrm R}_{\mathcal{G}}$-multi-almost automorphic function. \end{enumerate} \end{tw} \begin{proof} Let us prove the second item under the hypothesis that $f$ is compactly ${\mathrm R}_{\mathcal{G}}$-multi-almost automorphic. Let $({\bf b}_{k})$ be an arbitrary sequence in ${\mathrm R}_{\mathcal{G}}$, then there exist a subsequence $({\bf b}_{k_{l}})$ of $({\bf b}_{k})$ and a function $f^{\ast} : {\mathbb R}^{m}\rightarrow \X$ such that the following limits hold\begin{align}\label{love1234567801} \lim_{l\rightarrow +\infty}\sup_{{\bf t}\in \mathcal{E}}||f\bigl({\bf t} +{\bf b}_{k_{l}})-f^{\ast}({\bf t}) ||_{\mathcal{X}}=0 \end{align} and \begin{align}\label{love12345678902} \lim_{l\rightarrow +\infty}\sup_{{\bf t}\in \mathcal{E}}||f^{\ast}\bigl({\bf t}-{\bf b}_{k_{l}})-f({\bf t}) ||_{\mathcal{X}}=0\, ; \end{align} where $\mathcal{E}$ is a compact subset of $\mathbb{R}^m$. Now, let us define the function $$ F^{\ast}({\bf t}):=\int_{\mathbb{R}^m}K({\bf t}-\eta)f^{\ast}(\eta)\, d\eta,\quad {\bf t}\in {\mathbb R}^{m}\, . $$ Take $\mathcal{E}\subset \mathbb{R}^m$ a compact set; then, for any ${\bf t} \in \mathcal{E} $ we have \begin{eqnarray*} ||F({\bf t}+{\bf b}_{k_{l}})-F^{\ast}({\bf t})||_{\mathcal{Y}} &=&|| \int_{\mathbb{R}^m}K(\eta)( f({\bf t}-\eta + {\bf b}_{k_{l}})-f^{\ast}({\bf t}-\eta))\, d\eta||_{\mathcal{Y}}\\ &\leq & \int_{\mathbb{R}^m}||K(\eta)||_{L(\mathcal{X},\mathcal{Y})} \sup_{z\in \mathcal{E}_{\eta}}|| f(z+{\bf b}_{k_{l}})-f^{\ast}(z)||_{\mathcal{X}}d\eta\, , \end{eqnarray*} where, $\mathcal{E}_{\eta}:=\mathcal{E} -{\eta}$ is a compact subset of $\mathbb{R}^m$. Using (\ref{love1234567801}) and the Lebesgue's Dominated Convergence Theorem, we conclude $$\lim_{l \to \infty}\sup_{{\bf t}\in \mathcal{E}}||F({\bf t}+{\bf b}_{k_{l}})-F^{\ast}({\bf t})||_{\mathcal{Y}}=0\, .$$ Analogously, we have $$\lim_{l \to \infty}\sup_{{\bf t}\in \mathcal{E}}||F^{\ast}({\bf t}-{\bf b}_{k_{l}})-F({\bf t})||_{\mathcal{Y}}=0\, .$$ \end{proof} Based on the characterization theorems of compactly multi-almost automorphic functions presented in Section \ref{section char}, we derive the following corollary: \begin{cor} Let $\mathcal{G}$ be either $\mathbb{Z}^m$ or a dense subset of $\mathbb{R}^m$ and $f:\mathbb{R}^{m}\rightarrow \X$ be a bounded and ${\mathcal{G}}$-multi-almost automorphic function. Let $(K({\bf t}))_{{\bf t}\in \mathbb{R}^{n}}\subseteq L(\X, \mathcal{Y})$ be a strongly continuous operator family and $$\int_{\mathbb{R}^{m}}\| K({\bf t})\|_{L(\X,\mathcal{Y})}\, d {\bf t}\, < +\infty .$$ Define \begin{equation}\label{rep021} F({\bf t}):=\int_{\mathbb{R}^m}K({\bf t}-\eta)f(\eta)\, d\eta,\quad {\bf t}\in {\mathbb R}^{m}. \end{equation} If $F$ is uniformly continuous, then $F$ is compactly multi-almost automomorphic; that is $F \in \mathcal{K} AA(\mathbb{R}^m; \mathcal{Y})$. \end{cor} In what follows, we study the invariance of the bounded and compact $\mathrm{R}_{\mathcal{G}}$-multi-almost automorphic function space in the context where the kernel \( K \) in (\ref{rep01}) or (\ref{rep02}) depends on two variables, i.e., is of the form \( K(\cdot, \cdot) \). For this purpose, we introduce the notion of a (compactly) $\mathrm{R}_{\mathcal{G}}$-multi-Bi-almost automorphic function in the subsequent definition. \begin{definition}\label{defBaa} A jointly continuous function $K:\mathbb{R}^m\times \mathbb{R}^m\times \X \to \mathcal{Y}$ is $(\mathrm{R}_{\mathcal{G}},\mathcal{B})$-multi-Bi-almost automorphic if for any $B \in \mathcal{B}$ and any sequence $(b_k)_{k \in \mathbb{N}} \in \mathrm{R}_{\mathcal{G}}$, there exist a subsequence $(b_{k_l})\subset (b_k)$ and a function $K^*:\mathbb{R}^m\times \mathbb{R}^m\times \X \to \mathcal{Y}$ such that \begin{equation}\label{EqNew01} \lim_{l \to +\infty} K({\bf t}+b_{k_l}, {\bf s}+b_{k_l},x)=K^*({\bf t}, {\bf s},x)\, , \end{equation} and \begin{equation}\label{EqNew02} \lim_{l \to +\infty}K^*({\bf t}-b_{k_l}, {\bf s}-b_{k_l},x)=K({\bf t}, {\bf s},x)\, , \end{equation} hold pointwisely for $({\bf t}, {\bf s}) \in \mathbb{R}^m \times \mathbb{R}^m$ and any $x \in B$. If the limits in (\ref{EqNew01}) and (\ref{EqNew02}) are uniform on compact subsets of $\mathbb{R}^m \times \mathbb{R}^m$, then we say that $K$ is compactly $(\mathrm{R}_{\mathcal{G}},\mathcal{B})$-multi-Bi-almost automorphic. \end{definition} The next definition, was given in \cite{chavez2021almostaut} in order to study almost automorphic type solutions to integral equations of advanced and delayed type \begin{definition} We say that a jointly continuous function $K:\mathbb{R}^m\times\mathbb{R}^m\times \X \to \mathcal{Y}$ is $\lambda$-bounded if there exists a non negative function $\lambda:\mathbb{R}^m\times\mathbb{R}^m \to \mathbb{R}$ such that for every $\tau \in \mathbb{R}^m$ we have \begin{equation}\label{Eeqqlambda} ||K(\t+\tau,\s+\tau,x)||_{\mathcal{Y}} \leq \lambda({\bf t},{\bf s})\, , \end{equation} where the inequality is for each $(\t,\s)\in \mathbb{R}^m\times\mathbb{R}^m$ and any $x\in \X$. \end{definition} As it was proved for the one-dimensional case, see \cite[Lemma 2.3]{chavez2024compact}, the inequality (\ref{Eeqqlambda}) also holds for the limit function $K^*$ of a $\lambda$-bounded and multi-Bi-almost auromorphic function $K$. That is, we have the following lemma \begin{lem}\label{lemBou} Let us suppose that $K:\mathbb{R}^m \times \mathbb{R}^m \times \X\to \mathcal{Y}$ is $(\mathrm{R}_{\mathcal{G}}, \mathcal{B})$-multi-Bi-almost automorphic and $\lambda$-bounded. Then, its limit function $K^*:\mathbb{R}^m\times \mathbb{R}^m\times \X\to \mathcal{Y}$ (see definition \ref{defBaa}) is also $\lambda$-bounded; that is, for each $(\t,\s)\in \mathbb{R}^m\times\mathbb{R}^m$ and any $x\in \X$ we have $$||K^*(\t+\tau,\s+\tau,x)||_{\mathcal{Y}} \leq \lambda({\bf t},{\bf s}),\, \, \forall \tau \in \mathbb{R}^m\, .$$ \end{lem} \noindent {\sl Property ({\bf SC})}. We say that the operator valued function \( K: \mathbb{R}^m \times \mathbb{R}^m \to \mathcal{L}(\X; \mathcal{Y}) \) satisfies the property of being \textbf{(SC)} if, for each \( x \in \X \), the function \( (\mathbf{t}, \mathbf{s}) \mapsto K(\mathbf{t}, \mathbf{s}) x \) is continuous. \begin{tw}\label{TeoComb} Let $\mathrm{R}_{\mathcal{G}}$ be the set of sequences such that all of its subsequences are also in $\mathrm{R}_{\mathcal{G}}$ and let the operator valued function $K:\mathbb{R}^m \times \mathbb{R}^m\to \mathcal{L}(\X; \mathcal{Y})$ satisfies ({\bf SC}), is compactly $\mathrm{R}_{\mathcal{G}}$-multi-Bi-almost automorphic and is $\lambda$-bounded with $\lambda({\bf t},{\bf s})=\phi({\bf t}-{\bf s})$ and $\phi \in L^1(\mathbb{R}^m)$. Then, the operator $\Gamma$, defined by $$\Gamma u({\bf t}):= \int_{\mathbb{R}^m} K({\bf t},\eta)u(\eta)d\eta\, ,$$ maps the Banach space $\mathcal{K} \mathrm{R}_{\mathcal{G}} AA(\mathbb{R}^m ; \mathcal{X})\cap BC(\mathbb{R}^m ; \mathcal{X})$ to the Banach space $\mathcal{K} \mathrm{R}_{\mathcal{G}} AA(\mathbb{R}^m ; \mathcal{X})\cap BC(\mathbb{R}^m ; \mathcal{Y})$. \end{tw} \begin{proof} Let $u \in \mathcal{K} \mathrm{R}_{\mathcal{G}} AA(\mathbb{R}^m, \X)\cap BC(\mathbb{R}^m ; \mathcal{X})$ and define $v:=\Gamma u$. Since $K(\cdot,\cdot)$ is compactly $\mathrm{R}_{\mathcal{G}}$-multi-Bi-almost automorphic and $u$ is compactly $\mathrm{R}_{\mathcal{G}}$-multi-almost automorphic we can ensure that given any sequence $(b_k)\subset \mathrm{R}_{\mathcal{G}}$ there exist a subsequence $(b_{k_l})\subseteq (b_k)$ and functions $K^*(\cdot,\cdot)$ and $u^*$ such that the following limits, uniformly on compact subsets of $\mathbb{R}^m\times \mathbb{R}^m$, hold: $$\lim\limits_{l\to +\infty} K(\t+b_{k_l},\s+b_{k_l})=K^*(\t,\s),\quad \lim\limits_{l\to +\infty} K^*(\t-b_{k_l},\s-b_{k_l})=K(\t,\s);$$ and the following limits, uniformly on compact subsets of $\mathbb{R}^m$, also hold $$\lim\limits_{l\to +\infty} u(\t+b_{k_l})=u^*(\t),\quad \lim\limits_{l\to +\infty} u^*(\t-b_{k_l})=u(\t).$$ Now, using the Lebesgue's Dominated Convergence Theorem we see that \begin{equation}\label{Eq001} \lim\limits_{l\to +\infty} v(\t+b_{k_l})=v^*(\t),\, \, \lim_{l\to \infty}v^*(\t-b_{k_l})=v(\t)\, , \end{equation} where $$v^*(\t):=\int_{\mathbb{R}^m} K^*(\t,\eta)u^*(\eta)d\eta\, .$$ That is, $v$ is $\mathrm{R}_{\mathcal{G}}$-multi-almost automorphic. In what follows we prove that the limits in (\ref{Eq001}) are uniform for ${\bf t}$ in compact subsets of $\mathbb{R}^m$. Let $E$ be a compact subset of $\mathbb{R}^m$. Since $\phi \in L^1(\mathbb{R}^m)$, there exist a strictly increasing sequence of positive real numbers $(R_n)_{n\in \mathbb{N}}$ such that: $$\lim_{n \to \infty}\int_{\mathbb{R}^m \setminus B[R_n,0]}\phi(z)dz =0\, , $$ where $B[R_n,0]$ is the closed ball of center $0$ and radii $R_n$. Note that, if $n$ is sufficiently large, then there exists a subsequence $(\tilde{R}_n)$ of $(R_n)$ such that $ E+\mathbb{R}^m \setminus B[R_n,0] \subset \mathbb{R}^m \setminus B[\tilde{R}_n,0] $. Now, let us take ${\bf t} \in E$, then we have \begin{eqnarray*} ||v(\t+{\bf b}_{k_l})-v^*(\t)||_{\mathcal{X}}&\leq & \int_{\mathbb{R}^m}||K(\t+{\bf b}_{k_l}, \eta+{\bf b}_{k_l})-K^*(\t,\eta)||\, ||u(\eta+\t_k)||_{\mathcal{X}}d\eta \\ &+& \int_{\mathbb{R}^m}||K^*({\bf t},\eta)||\, ||u(\eta+{\bf b}_{k_l})-u^*(\eta)||d\eta\\ &\leq & \int_{B[R_n,0]}||K(\t+{\bf b}_{k_l}, \eta+{\bf b}_{k_l})-K^*(\t,\eta)||\, ||u(\eta+\t_k)||_{\mathcal{X}}d\eta \\ &+& \int_{\mathbb{R}^m \setminus B[R_n,0] }||K(\t+{\bf b}_{k_l}, \eta+{\bf b}_{k_l})-K^*(\t,\eta)||\, ||u(\eta+\t_k)||_{\mathcal{X}}d\eta \\ &+& \int_{\mathbb{R}^m}||K^*({\bf t},\eta)||\, ||u(\eta+{\bf b}_{k_l})-u^*(\eta)||d\eta\\ &\leq & ||u||_{\infty} \sup_{({\bf t}, {\bf s})\in \mathcal{E}\times B[R_n,0]} ||K(\t+{\bf b}_{k_l}, \eta+{\bf b}_{k_l})-K^*(\t,\eta)||\, \mathcal{L}(B[R_n,0])\\ &+& 2||u||_{\infty} \int_{\mathbb{R}^m \setminus B[R_n,0]}\phi({\t}-\eta)d\eta\\ &+& \int_{\mathbb{R}^m} \phi(\eta) \, \sup_{z\in E_{\eta}}|| u(z+{\bf b}_{k_{l}})-u^{\ast}(z)||_{\mathcal{X}}d\eta\, ,\\ \end{eqnarray*} where, $E_{\eta}:=E -{\eta}$ is a compact subset of $\mathbb{R}^m$. From the previous inequalities, we have \begin{eqnarray*} \sup_{{\bf t}\in \mathcal{E}}||v(\t+{\bf b}_{k_l})-v^*(\t)||_{\mathcal{X}}&\leq & ||u||_{\infty} \sup_{({\bf t}, {\bf s})\in \mathcal{E}\times B[R_n,0]} ||K(\t+{\bf b}_{k_l}, \eta+{\bf b}_{k_l})-K^*(\t,\eta)||\, \mathcal{L}(B[R_n,0])\\ &+& 2||u||_{\infty} \int_{\mathbb{R}^m \setminus B[R_n,0]}\phi({\t}-\eta)d\eta\\ &+& \int_{\mathbb{R}^m} \phi(\eta) \, \sup_{z\in E_{\eta}}|| u(z+{\bf b}_{k_{l}})-u^{\ast}(z)||_{\mathcal{X}}d\eta\, . \end{eqnarray*} Now, taking the limit in the last inequality when $l\to \infty$, we obtain $$\lim_{l\to \infty}\sup_{{\bf t}\in \mathcal{E}}||v(\t+{\bf b}_{k_l})-v^*(\t)||_{\mathcal{X}} \leq 2||u||_{\infty} \int_{\mathbb{R}^m \setminus B[R_n,0]}\phi({\t}-\eta)d\eta .$$ On the other hand, the integral of the right hand side satisfies: $$\int_{\mathbb{R}^m \setminus B[R_n,0]}\phi({\t}-\eta)d\eta\leq \int_{E+\mathbb{R}^m \setminus B[R_n,0]}\phi(z)dz \leq \int_{\mathbb{R}^m \setminus B[\tilde{R}_n,0]}\phi(z)dz\, \to 0, \, \, n\to \infty\, .$$ Therefore, $$\lim_{l\to \infty}\sup_{{\bf t}\in \mathcal{E}}||v(\t+{\bf b}_{k_l})-v^*(\t)||_{\mathcal{X}}=0\, .$$ Analogously, we have $$\lim_{l\to \infty}\sup_{{\bf t}\in E}||v^*(\t-{\bf b}_{k_l})-v(\t)||_{\mathcal{X}} =0\, . $$ \end{proof} \begin{tw}\label{NewThmappl} Let $\mathrm{R}_{\mathcal{G}}$ be the set of sequences such that all of its subsequences are also in $\mathrm{R}_{\mathcal{G}}$ and let the operator valued function $K:\mathbb{R}^m \times \mathbb{R}^m\to \mathcal{L}(\X; \mathcal{Y})$ satisfies ({\bf SC}), is compactly multi-Bi-almost automorphic and is $\lambda$-bounded with $\lambda({\bf t},{\bf s})=\phi({\bf t}-{\bf s})$ and $\phi \in L^1(\mathbb{R}^m)$. Then, the operator $\Gamma$, defined by $$\Gamma u({\bf t}):= \int_{\mathbb{R}^m} K({\bf t},\eta)u(\eta)d\eta\, ,$$ maps the space $\mathrm{R}_{\mathcal{G}} AA(\mathbb{R}^m, \mathcal{X})\cap B UC(\mathbb{R}^m ; \mathcal{X})$ to the space $\mathrm{R}_{\mathcal{G}} AA(\mathbb{R}^m, \mathcal{X})\cap B UC(\mathbb{R}^m ; \mathcal{Y})$. \end{tw} \begin{proof} Using the hypothesis we can see that, if $u$ is bounded then $\Gamma u$ is also bounded. Also, if $u \in \mathrm{R}_{\mathcal{G}} AA(\mathbb{R}^m, \mathcal{X})$, then (as in the previous theorem) we conclude that $\Gamma u \in \mathrm{R}_{\mathcal{G}} AA(\mathbb{R}^m, \mathcal{X})$. It only rest to prove that $\Gamma u$ is uniformly continuous. Let $(\t_k),(\s_k)$ be two sequences in $\mathbb{R}^m$ such that $||\t_k-\s_k|| \to 0$ when $k\to +\infty$, then \begin{eqnarray*} ||v(\t_k)-v(\s_k)||&\leq & \int_{\mathbb{R}^m}||K(\t_k, \eta+\t_k)-K(\s_k,\eta+\s_k)||\, ||u(\eta+\t_k)||d\eta \\ &+& \int_{\mathbb{R}^m}||K(\s_k,\eta+\s_k)||\, ||u(\eta+\t_k)-u(\eta+\s_k)||d\eta\\ &:=&I_1(k)+I_2(k)\, . \end{eqnarray*} Now, by Lemma \ref{lemBou}, Theorem \ref{ThRKAA}, the uniform continuity of $u$ and the Lebesgue's Dominated Convergence Theorem, we conclude $$\lim_{k\to +\infty}I_1(k)=0 \, =\, \lim_{k\to +\infty}I_2(k) \, .$$ \end{proof} \begin{cor} Let the operator valued function $K:\mathbb{R}^m \times \mathbb{R}^m\to \mathcal{L}(\X; \mathcal{Y})$ satisfies ({\bf SC}), is compactly multi-Bi-almost automorphic and is $\lambda$-bounded with $\lambda({\bf t},{\bf s})=\phi({\bf t}-{\bf s})$ and $\phi \in L^1(\mathbb{R}^m)$. Then, the operator $\Gamma$, defined by $$\Gamma u({\bf t}):= \int_{\mathbb{R}^m} K({\bf t},\eta)u(\eta)d\eta\, ,$$ maps the Banach space $\mathcal{K} AA(\mathbb{R}^m, \mathcal{X})$ to the Banach space $\mathcal{K} AA(\mathbb{R}^m, \mathcal{Y})$. \end{cor} Using the previous results and the characterization theorems of the space of compactly multi-almost automorphic functions presented in section \ref{section char}, it is not difficult to prove the next corollary \begin{cor} Let $\mathcal{G}$ be either $\mathbb{Z}^m$ or a dense subset of $\mathbb{R}^m$ and let $\mathrm{R}_{\mathcal{G}}$ be the set of sequences such that all of its subsequences are also in $\mathrm{R}_{\mathcal{G}}$ and let the operator valued function $K:\mathbb{R}^m \times \mathbb{R}^m\to \mathcal{L}(\X; \mathcal{Y})$ satisfies ({\bf SC}), is $\mathrm{R}_{\mathcal{G}}$-multi-Bi-almost automorphic and is $\lambda$-bounded with $\lambda({\bf t},{\bf s})=\phi({\bf t}-{\bf s})$ and $\phi \in L^1(\mathbb{R}^m)$. Then, if $u$ is bounded and belongs to $\mathrm{R}_{\mathcal{G}} AA(\mathbb{R}^m, \mathcal{X})$, and $\Gamma u$ is uniformly continuous, where $$\Gamma u({\bf t}):= \int_{\mathbb{R}^m} K({\bf t},\eta)u(\eta)d\eta\, ;$$ then, $\Gamma u \in \mathcal{K} AA(\mathbb{R}^m, \mathcal{Y})$. \end{cor} In the proof of the next result, we use the fact that every multi-Bi-almost automorphic function is, according to Theorem \ref{ThRKAA}, ${\mathrm{R}}_{{\mathcal{G}}}$-uniformly continuous, where $\mathcal{G}$ is the set $\{({\bf t},{\bf s})\in \mathbb{R}^m \times \mathbb{R}^m\, :\, {\bf t}={\bf s} \}$. We omit the details. \begin{tw}\label{InvConv1} Let $K:\mathbb{R}^m \times \mathbb{R}^m\to \mathcal{L}(\X; \mathcal{Y})$ satisfies ({\bf SC}), is compactly multi-Bi-almost automorphic and is $\lambda$-bounded with $\lambda({\bf t},{\bf s})=\phi({\bf t}-{\bf s})$ and $\phi \in L^1([0,\infty)^m)$. Then, the operator $\Pi$ defined by \begin{equation}\label{Operator2} \Pi u ({\bf t}):=\int_{-\infty}^{{\bf t}} K({\bf t},\eta)u(\eta)d\eta\, , \end{equation} maps the Banach space $\mathcal{K}AA(\mathbb{R}^m; \mathcal{X})$ to the Banach space $\mathcal{K}AA(\mathbb{R}^m; \mathcal{Y})$. \end{tw} \color{black} Applications of Theorems \ref{TeoComb} and \ref{InvConv1} to ordinary differential equations with exponential dichotomy can be found in \cite{chavez2024compact}. \section{Applications to the Poisson's and heat equations}\label{section appl} \subsection{Compact almost automorphy of the solution to Poisson's equation} \begin{definition} Let $ f\in BC(\mathbb{R}^m,\mathbb{R}) $. A function $u:\mathbb{R}^m \to \mathbb{R}$ is solution in the sense of distributions of the Poisson's equation $$\Delta u=f,$$ if \begin{equation} \notag\int_{\mathbb{R}^{m}}u(x)\Delta\phi(x)dx=\int_{\mathbb{R}^{m}} f(x)\phi(x)dx,\hspace{0.5cm}\forall\phi\in C^{\infty}_{0}(\mathbb{R}^{m}). \end{equation} \end{definition} From the the theory of distributional solutions for the Laplace equation, we have \begin{tw} \label{Thmf01} \cite{Jost02} \begin{enumerate} \item (Weyl) If $u\in L^{1}_{loc}(\mathbb{R}^{m},\mathbb{R})$ and $u$ is a solution in the sense of distributions of $\Delta u=0$, then $u$ is harmonic, i.e., $u\in C^{\infty}(\mathbb{R}^{m})$ and $\Delta u=0$ in $\mathbb{R}^{m}$. \item (Lioville) If $u:\mathbb{R}^{m}\to \mathbb{R}$ is harmonic and bounded, then $u$ is constant. \end{enumerate} \end{tw} Theorem \ref{Thmf01} and the Arzel'a-Ascolí Theorem were used in \cite{CHAKPPINTO2023} to prove that, every bounded and continuous function with almost automorphic distributional laplacian is in fact compact almost automorphic. The next Theorem is the main result of the present subsection. \begin{tw}\label{main theorem} Let $\mathcal{G}$ be either $\mathbb{Z}^m$ or a dense subset of $\mathbb{R}^m$ and $f\in \mathcal{G}AA(\mathbb{R}^{m},\mathbb{R})$. If $u:\mathbb{R}^{m}\to \mathbb{R}$ is a bounded continuous function which is a solution in the sense of distributions of Poisson's equation: \begin{equation} \notag\Delta u=f\, , \end{equation} then $u\in \mathcal{K}AA(\mathbb{R}^{m},\mathbb{R})$. \end{tw} \begin{proof} Indeed, as demonstrated in the proof of \cite[Theorem 20]{CHAKPPINTO2023}, we establish that \( u \) is uniformly continuous on \( \mathbb{R}^m \) and compactly \( \mathcal{G} \)-multi almost automorphic. Consequently, the conclusion that \( u \) is a compact almost automorphic solution follows from either the second characterization theorem (in the case where \( \mathcal{G} = \mathbb{Z}^m \)) or the third characterization theorem (when \( \mathcal{G} \) is a dense subset of \( \mathbb{R}^m \)) presented in Section \ref{section char}. \end{proof} This theorem emphasizes that, even if the source function \( f \) is not fully almost automorphic, but belongs to \( \mathcal{G}\text{AA}(\mathbb{R}^m, \mathbb{R}) \) with \( \mathcal{G} \) as specified, its solution remains compact almost automorphic. Consequently, compact almost automorphic solutions to Poisson's equation are preserved under a broader class of source functions, extending beyond the almost periodic/automorphic case presented in \cite{sibuya1971almost,CHAKPPINTO2023}. \subsection{Compact almost automorphy of the solution to heat equation} Le us consider the heat equation: \begin{equation}\label{Heat equation} \left\{ \begin{array}{lll} \partial_t u(t,x)&=&\Delta u(t,x), \quad t>0,\, x\in \mathbb{R}^m,\\ u(0,x) &= &f(x), \quad x\in \mathbb{R}^m, \end{array}\right. \end{equation} where $ f \in BC(\mathbb{R}^m,\mathbb{R}) $. The maximal domain of the Laplace operator $ \Delta $ in the Banach space $Z= BC(\mathbb{R}^m,\mathbb{R}) $ or $ BUC(\mathbb{R}^m,\mathbb{R}) $, is given by $$ D(\Delta):=\lbrace v\in Z : \Delta v \text{ exists in } Z\rbrace . $$ \begin{definition} Let $ f\in BC(\mathbb{R}^m,\mathbb{R}) \cap \overline{D(A)}$, by a solution of equation \eqref{Heat equation} we mean a function $u: [0,+\infty)\times\mathbb{R}^m \longrightarrow \mathbb{R}$, such that $u \in C^{1}((0,+\infty),BC(\mathbb{R}^m,\mathbb{R}))$, $ u(t,\cdot) \in D(\Delta)$ for all $t>0$ and $u$ satisfies \eqref{Heat equation} pointwisely.\\ In particular, if $f \in BUC(\mathbb{R}^m,\mathbb{R}) \cap \overline{D(A)} $, the solution $u$ satisfies $u \in C([0,+\infty), BC(\mathbb{R}^m,\mathbb{R}))\cap C^{1}((0,+\infty),BC(\mathbb{R}^m,\mathbb{R}))$, $ u(t,\cdot) \in D(\Delta) $ for all $t>0$ and $u$ satisfies \eqref{Heat equation} poitnwisely. \end{definition} It is well known that, the heat equation \eqref{Heat equation} admits the following solution $$ u(t,x)=T(t)f(x), \quad t>0, \, x\in \mathbb{R}^m, \quad \text{ for } f\in \overline{D(A)}.$$ and $ u(0,x)=f(x) $, where $ (T(t))_{t\geq 0} $ is the Gaussian semigroup in $ BC(\mathbb{R}^m,\mathbb{R}) $ given by: \begin{eqnarray}\label{Gaussian semigroup} T(t)f(x)&=& \dfrac{1}{(4\pi t)^{m/2}} \int_{\mathbb{R}^m} e^{-\|x-z\|^2 /(4t)} f(z) dz= \dfrac{1}{(4\pi t)^{m/2}} \int_{\mathbb{R}^m} e^{-\|z\|^2 /(4t)} f(x-z) dz \nonumber \\ &=&(K(t,\cdot)*f)(x), \quad x\in \mathbb{R}^m\, , \end{eqnarray} for $t>0$, $x\in \mathbb{R}^m$ (see for instance \cite[Example 3.7.6]{Arendt}). The following is the main result concerning the heat equation in this work \begin{tw} Let $f\in \mathcal{G}AA(\mathbb{R}^m,\mathbb{R}) \cap \overline{D(A)}$, where $\mathcal{G}$ be either $\mathbb{Z}^m$ or a dense subset of $\mathbb{R}^m$ and $f\in \mathcal{G}AA(\mathbb{R}^{m},\mathbb{R})$. Then, equation \eqref{Heat equation} admits a unique solution $u$ such that, for each $t>0$, $u(t,\cdot) \in \mathcal{K}AA(\mathbb{R}^m,\mathbb{R})$. \end{tw} \begin{proof} Let $f \in \mathcal{G} AA(\mathbb{R}^m,\mathbb{R})$ and let $t>0$, from the invariance Theorem \ref{TI0101}, we have that $u(t,\cdot)\in \mathcal{G}AA(\mathbb{R}^m,\mathbb{R})$. Now, using the charaterization theorems in section \ref{section char}, it only rest to prove that $u(t,\cdot)$ is uniformly continuous, and this follows as in ithe proof of \cite[Theorem 25]{CHAKPPINTO2023}. In fact, let $x,y\in \mathbb{R}^m$; then \begin{align}\label{Lipch} |u(t,x)-u(t,y) | \leq \dfrac{1}{(4\pi t)^{m/2}} \int_{\mathbb{R}^m} |e^{-\|x-z\|^2 /(4t)}-e^{-\|y-z\|^2 /(4t)} |dz \, \|f\|_{\infty}. \end{align} Also, for fixed $t>0$ and $z\in \mathbb{R}^m$, the function $x\longmapsto e^{-\|x-z\|^2 /(4t)}$ is globally Lipschitz continuous, i.e., $$| e^{-\|x-z\|^2 /(4t)}-e^{-\|y-z\|^2 /(4t)}| \leq \sup_{w\in \mathbb{R}^m} \dfrac{c_m}{2t} \| w-z \|e^{-\|w-z\|^2 /(4t)} \| x-y\|, $$ where $c_m > 0$ is a constant. Note that $$\sup_{w\in \mathbb{R}^m} \| w-z \|e^{-\|w-z\|^2 /(4t)}=\| w_0-z \|e^{-\|w_0-z\|^2 /(4t)}\, \, , w_0 \in \mathcal{D}\, ,$$ where $$\mathcal{D}:=\{ w\in \mathbb{R}^m\, : \, \|w-z \| =\sqrt{2t}\}\, .$$ Now, from (\ref{Lipch}) and using invariance under translation of the Lebesgue integral, we have \begin{align*} |u(t,x)-u(t,y) | &\leq c'_m(t) \|f\|_{\infty} \|x-y\|. \end{align*} \end{proof} Thus, we have proved that the $\mathcal{G}$-multi-almost automorphy of the initial data $f$ of equation \eqref{Heat equation}, is sufficient to obtain the compact almost automorphy in space variable of the solution $u$. \appendix \section{Sequentially almost automorphic functions on topological groups} Let $\mathcal{G}$ be a topological group (commutative, locally compact). We will write the group operation on $\mathcal{G}$ additively, that is by $+$, and the inverse operation by $-$. \begin{definition}\label{Defi01} Let $f: \mathcal{G} \to \mathcal{X}$ be a continuous function. $f$ is sequentially almost automorphic, if for any sequence $( s^{\prime}_{m})$ of $\mathcal{G}$ there exist a subsequence $( s_{m})$ of $(s^{\prime}_{m})$ and a function $f^*:\mathcal{G}\to \mathcal{X}$ such that the following pointwise limits holds: \begin{equation} \notag\lim_{m\to \infty}f(t+s_{m})=f^*(t), \end{equation} and: \begin{equation} \notag\lim_{n\to \infty}f^*(t-s_{m})=f(t)\, . \end{equation} \end{definition} \noindent $AA(\G,\X)$ will denote the space of sequentially almost automorphic functions. The next two theorems are of importance in the present work. \begin{tw}\label{pro1} If $f \in AA(\G,\X)$, then $f$ is bounded. \end{tw} \begin{proof} Suppose that $f$ is not bounded, that is, there exists a sequence $(s'_m)_{m\in \mathbb{N}} \subset \G$ such that \begin{equation*} \lim_{n\rightarrow\infty}{||f(s'_m)||_{\mathcal{X}}} = \infty, \end{equation*} as $f\in AA(\G,\mathcal{X})$, there exists a subsequence $(s_m)_{m\in \mathbb{N}}\subset(s'_m)_{m\in \mathbb{N}}$ and a function $h:\G \rightarrow \mathcal{X}$ such that the following pointwise limits holds: \begin{equation*} f^*(\mathbf{t}) = \lim_{m\rightarrow \infty}f(\mathbf{t}+s_m)\,\,, \,\, f(\mathbf{t}) = \lim_{n\rightarrow \infty}f^*(\mathbf{t}-s_m). \end{equation*} If $\textbf{t} = 0$, in the first limit we have: \begin{equation*} \lim_{m\rightarrow\infty}{f(s_m)} = f^*(0)\,\,; \,\, f^*(0) \in \mathcal{X}, \end{equation*} with what we have \begin{equation*} \lim_{m\rightarrow \infty}{||f(s_m)||_{\mathcal{X}}} = ||f^*(0)||_{\mathcal{X}} < \infty, \end{equation*} which is a contradiction, therefore, $f$ is bounded. \end{proof} \begin{tw}\label{pro2} If $f \in AA(\G,\X)$, then the range of $f$ is relatively compact in $\X$. \end{tw} \begin{proof} Let $(y'_m)_{m\in \mathbb{N}} \subset \mathrm{R}(f) := \{y \in X: \exists \t \in \G \,\,\text{whit}\,\, f(\t) = y\}$ be an arbitrary sequence. By the definition of $\R(f)$ there must be a sequence $(s'_m)_{m\in \N}\subset \G$ such that $y'_m = f(s'_m)$, as $f \in AA(\G,\X)$ there exist a subsequence $(s_m)_{m\in \N} \subset (s'_m)_{m\in \N}$ and a function $f^*:\G \rightarrow \X$ such that \begin{equation*} \lim_{m\rightarrow\infty}{y_m} = \lim_{m\rightarrow\infty}{f(s_m)} = f^*(0). \end{equation*} Therefore, there exists a subsequence $(s_m)_{m\in \N} \subset (s'_m)_{m\in \N}$ wich converges to $\X$, which means that $\R(f)$ is relatively compact. \end{proof} \section{Subgroups of $\mathbb{R}^m$} Here we summarize important results on dense subgroups of the group $\mathbb{R}^m$. We start with the following assertion: \begin{lem}\label{lem8} Every nontrivial subgroup of $\mathbb{R}^m$ is unbounded. \end{lem} \begin{proof} Let $ H $ be a nontrivial subgroup of $\mathbb{R}^m $. Since $H $ is nontrivial, it contains at least one element $v$ other than the zero vector. Now, for eack $k\in \mathbb{N}$, define $ v_k := k \cdot v \in H$, where $k \cdot v =v+v+\cdots +v $ (k-times). Then, for each $ k $, we have $ \|v_k\| = \|k \cdot v\| = k \cdot \|v\| $. Therefore, the sequence $ \{v_k\} $ is unbounded. This proves that $ H $ is unbounded. \end{proof} \subsection{Dense subgroups of $\mathbb{R}^m$} Let $\mathbb{Q}$ be the rational numbers. Some dense subgroups of $\mathbb{R}^m$ are, for example, $\mathbb{Q}^m$, $\mathbb{Q}^k\times \mathbb{R}^{m-k}$, $\mathbb{Q}^{k_1}\times G^{k_2} \times \mathbb{R}^{k_3}$ where $k_1+k_2+k_3=m$ and $G^{k_2}$ is a dense subgroup of $\mathbb{R}^{k_2}$, etc. In the following Theorem and in the subsequent Proposition, it is described some conditions under which a subgroup of $\mathbb{R}^m$ becomes dense. \begin{tw}\label{ApB1} Let $\theta_1, \theta_2, \cdots,\theta_m$ real numbers. For the subgroup $$\mathbb{Z}^{m} + \mathbb{Z}(\theta_1, \theta_2, \cdots,\theta_m)= \{(s_1 + s_0\theta_1, \cdots,s_m+s_0\theta_m); \,\,(s_0,s_1,\cdots,s_m)\in \mathbb{Z}^{m+1}\}\subset \mathbb{R}^m $$ to be dense in $\mathbb{R}^m$, it is necessary and sufficient that the $m+1$ numbers $1,\theta_1, \theta_2, \cdots,\theta_m$ be linearly independent on $\mathbb{Q}$. \end{tw} \begin{prop}\label{ApB2} Let $G$ be a finitely generated subgroup of $\mathbb{R}^m$. The following conditions are equivalent: \begin{enumerate} \item $G$ is dense in $\mathbb{R}^m$. \item For any nonzero linear form $\phi: \mathbb{R}^m\rightarrow\mathbb{R}$ we have $\phi(G)\not\subset\mathbb{Z}$. \end{enumerate} \end{prop} For a proof of Theorem \ref{ApB1} and Proposition \ref{ApB2}, the reader is invited to consult the works \cite{elghaoui2015rational,waldschmidt1995topologie}. Finally, we also present the next important fact on continuous functions: \begin{prop}\label{grp} Let $f:\mathbb{R}^m \rightarrow \X$ be a continuous function and $A \subset \mathbb{R}^m$, then $$\overline{f(\overline{A})} = \overline{f(A)}.$$ \end{prop} \subsection*{Acknowledgments} The authors would like to express their gratitude to the anonymous referees for their careful reading and helpful comments. \subsection*{Funding} A. Ch\'avez and J. Casta\~neda were supported by CONCYTEC through the PROCIENCIA program under the E041-2023-01 competition, according to contract PE501082885-2023 \section*{Declarations} \subsection*{Conflict of interest} No potential conflict of interest was reported by the authors. \begin{thebibliography}{100} \bibitem{Arendt} {\sc Arendt, W., Batty, C.~J., Hieber, M., and Neubrander, F.} \newblock {\em Vector-valued Laplace Transforms and Cauchy Problems}, vol.~96. \newblock Springer Science \& Business Media, Basel, 2011. \bibitem{03} {\sc Bochner:, S.} \newblock Curvature and {B}etti numbers in real and complex vector bundles. \newblock {\em Univ. e Politec. Torino Rend. Sem. Mat. 15\/} (1955/56), 225--253. \bibitem{04} {\sc Bochner, S.} \newblock A new approach to almost periodicity. \newblock {\em Proc. Nat. Acad. Sci. U.S.A. 48\/} (1962), 2039--2043. \bibitem{05} {\sc Bochner, S.} \newblock Continuous mappings of almost automorphic and almost periodic functions. \newblock {\em Proc. Nat. Acad. Sci. U.S.A. 52\/} (1964), 907--910. \bibitem{campos2014almost} {\sc Campos, J., and Tarallo, M.} \newblock Almost automorphic linear dynamics by {F}avard theory. \newblock {\em Journal of Differential Equations 256}, 4 (2014), 1350--1367. \bibitem{campos2020barycentric} {\sc Campos, J., and Tarallo, M.} \newblock Barycentric solutions of linear almost periodic equations: {B}aire class and almost automorphy. \newblock {\em Journal of Dynamics and Differential Equations 32}, 3 (2020), 1475--1509. \bibitem{chavez2024compact} {\sc Ch{\'a}vez, A., Aragon{\'e}s, N., Pinto, M., and Zavaleta, U.} \newblock Compact almost automorphic dynamics of non-autonomous differential equations with exponential dichotomy and applications to biological models with delay. \newblock {\em arXiv preprint arXiv:2404.16758 ( https://doi.org/10.48550/arXiv.2404.16758)\/} (2024). \bibitem{chavez2014discontinuous} {\sc Ch{\'a}vez, A., Castillo, S., and Pinto, M.} \newblock Discontinuous almost automorphic functions and almost automorphic solutions of differential equations with piecewise constant argument. \newblock {\em Electronic Journal of Differential Equations 2014}, 56 (2014), 1--13. \bibitem{Alanpiecewise} {\sc Ch{\'a}vez, A., and Huatangari, L.~Q.} \newblock Vector valued piecewise continuous almost automorphic functions and some consequences. \newblock {\em Journal of Mathematical Analysis and Applications 542}, 2 (2025), 128768. \bibitem{chavez2022multi} {\sc Ch{\'a}vez, A., Khalil, K., Kosti{\'c}, M., and Pinto, M.} \newblock Multi-dimensional almost automorphic type functions and applications. \newblock {\em Bulletin of the Brazilian Mathematical Society, New Series 53}, 3 (2022), 801--851. \bibitem{chavez2023almost} {\sc Ch{\'a}vez, A., Khalil, K., Kosti{\'c}, M., and Pinto, M.} \newblock Almost periodic type functions of several variables and applications. \newblock {\em Journal of Mathematical Analysis and Applications 525}, 1 (2023), 127115. \bibitem{CHAKPPINTO2023} {\sc Ch{\'a}vez, A., Khalil, K., Pereyra, A., and Pinto, M.} \newblock Compact almost automorphic solutions to {P}oisson’s and heat equations. \newblock {\em Bulletin of the Malaysian Mathematical Sciences Society 47}, 2 (2024), 1--22. \bibitem{chavez2021almostaut} {\sc Ch{\'a}vez, A., Pinto, M., and Zavaleta, U.} \newblock On almost automorphic type solutions of abstract integral equations, a {B}ohr-{N}eugebauer type property and some applications. \newblock {\em Journal of Mathematical Analysis and Applications 494}, 1 (2021), 124395. \bibitem{ding2017asymptotically} {\sc Ding, H.-S., and Wan, S.-M.} \newblock Asymptotically almost automorphic solutions of differential equations with piecewise constant argument. \newblock {\em Open Mathematics 15}, 1 (2017), 595--610. \bibitem{elghaoui2015rational} {\sc Elghaoui, M., and Ayadi, A.} \newblock Rational criterion testing the density of additive subgroups of $\mathbb{R}^n$ and $\mathbb{C}^n$. \newblock {\em Applied general topology 16}, 2 (2015), 127--139. \bibitem{es2022compact} {\sc Es-sebbar, B., Ezzinbi, K., and Khalil, K.} \newblock Compact almost automorphic solutions for semilinear parabolic evolution equations. \newblock {\em Applicable Analysis 101}, 7 (2022), 2553--2579. \bibitem{fuhrmann2020tameness} {\sc Fuhrmann, G., and Kwietniak, D.} \newblock On tameness of almost automorphic dynamical systems for general groups. \newblock {\em Bulletin of the London Mathematical Society 52}, 1 (2020), 24--42. \bibitem{garcia2021mean} {\sc Garc{\'\i}a-Ramos, F., J{\"a}ger, T., and Ye, X.} \newblock Mean equicontinuity, almost automorphy and regularity. \newblock {\em Israel Journal of Mathematics 243}, 1 (2021), 155--183. \bibitem{Jost02} {\sc Jost, J.} \newblock {\em Partial {D}ifferential {E}quations}, vol.~2. \newblock Springer, New York, 2007. \bibitem{lenz2024abstract} {\sc Lenz, D., Spindeler, T., and Strungaru, N.} \newblock Abstract almost periodicity for group actions on uniform topological spaces. \newblock {\em Canadian Journal of Mathematics 76}, 3 (2024), 798--829. \bibitem{lenz2024pure} {\sc Lenz, D., Spindeler, T., and Strungaru, N.} \newblock Pure point spectrum for dynamical systems and mean, {B}esicovitch and {W}eyl almost periodicity. \newblock {\em Ergodic Theory and Dynamical Systems 44}, 2 (2024), 524--568. \bibitem{qi2022piecewise} {\sc Qi, L., and Yuan, R.} \newblock Piecewise continuous almost automorphic functions and {F}avard’s theorems for impulsive differential equations in honor of {R}ussell {J}ohnson. \newblock {\em Journal of Dynamics and Differential Equations\/} (2022), 1--43. \bibitem{rezoug2024asymptotically} {\sc Rezoug, N., Salim, A., and Benchohra, M.} \newblock Asymptotically almost automorphy for impulsive integrodifferential evolution equations with infinite time delay via {M}{\"o}nch fixed point. \newblock {\em Evolution Equations and Control Theory\/} (2024), 0--0. \bibitem{sibuya1971almost} {\sc Sibuya, Y.} \newblock Almost periodic solutions of {P}oisson’s equation. \newblock {\em Proceedings of the American mathematical society 28}, 1 (1971), 195--198. \bibitem{18} {\sc Veech, W.~A.} \newblock Almost automorphic functions. \newblock {\em Proceedings of the National Academy of Sciences 49}, 4 (1963), 462--464. \bibitem{veech1965almost} {\sc Veech, W.~A.} \newblock Almost automorphic functions on groups. \newblock {\em American Journal of Mathematics 87}, 3 (1965), 719--751. \bibitem{waldschmidt1995topologie} {\sc Waldschmidt, M.} \newblock {\em Topologie des points rationnels}. \newblock Univ. P. et M. Curie, 1995. \end{thebibliography} \end{document}
2412.07886v1
http://arxiv.org/abs/2412.07886v1
Convergence estimates for the Magnus expansion IE. Finite dimensional Banach algebras
\documentclass[a4paper,11pt]{amsart} \usepackage[utf8]{inputenc} \usepackage{CJKutf8} \usepackage{amsmath,amsthm,amssymb} \usepackage{graphicx} \usepackage{subcaption} \usepackage{float} \usepackage{cancel} \usepackage{slashed} \usepackage{comment} \newcommand{\mychoice}[3]{#1 } \mychoice{ \newcommand{\plabel}[1]{ \label{#1}} \newcommand{\gbibitem}[1]{ \bibitem{#1}} \newcommand{\snewpage}{} \newcommand{\scleardoublepage}{} \newcommand{\EMPTYPAGE}{} \specialcomment{commentx}{}{}\excludecomment{commentx} \specialcomment{commenty}{}{}\excludecomment{commenty} \newcommand{\rechoicecomm}[1]{} }{ \usepackage{xcolor} \newcommand{\plabel}[1]{ \label{#1}\rlap{\smash{${}^{^{[#1]}}$}}} \newcommand{\gbibitem}[1]{ \bibitem{#1}\rlap{\smash{${}^{^{[#1]}}$}}} \newcommand{\snewpage}{\newpage} \newcommand{\scleardoublepage}{\cleardoublepage} \newcommand{\EMPTYPAGE}{EMPTY PAGE} \newenvironment{commentx}{\color{magenta} }{\color{black} } \newenvironment{commenty}{\color{blue} }{\color{black} } \newcommand{\rechoicecomm}[1]{#1} }{ \usepackage{xcolor} \newcommand{\plabel}[1]{ \label{#1}} \newcommand{\gbibitem}[1]{ \bibitem{#1}} \newcommand{\snewpage}{} \newcommand{\scleardoublepage}{} \newcommand{\EMPTYPAGE}{} \specialcomment{commentx}{}{}\excludecomment{commentx} \newenvironment{commenty}{\scriptsize }{\normalsize } \newcommand{\rechoicecomm}[1]{} } \DeclareMathOperator{\Tan}{\slashed{\mathrm T}an} \DeclareMathOperator{\ad}{ad} \DeclareMathOperator{\Ad}{Ad} \DeclareMathOperator{\AC}{AC} \DeclareMathOperator{\ATT}{AT} \DeclareMathOperator{\LR}{LR} \DeclareMathOperator{\sgn}{sgn} \DeclareMathOperator{\Id}{Id} \DeclareMathOperator{\tr}{tr} \DeclareMathOperator{\des}{des} \DeclareMathOperator{\smc}{sc} \DeclareMathOperator{\artanh}{artanh} \DeclareMathOperator{\sech}{sech} \DeclareMathOperator{\csch}{csch} \DeclareMathOperator{\arcosh}{arcosh} \DeclareMathOperator{\arsinh}{arsinh} \DeclareMathOperator{\mul}{mul} \DeclareMathOperator{\res}{res} \DeclareMathOperator{\asc}{asc}\DeclareMathOperator{\conv}{conv} \DeclareMathOperator{\MP}{\mathcal M} \DeclareMathOperator{\intD}{\mathring D} \newcommand{\real}{\mathrm{real}} \newcommand{\anal}{\mathrm{an}} \DeclareMathOperator{\Dbar}{D} \DeclareMathOperator{\Rea}{Re} \DeclareMathOperator{\Ima}{Im} \DeclareMathOperator{\Rexp}{exp_{R}} \DeclareMathOperator{\Lexp}{exp_{L}} \DeclareMathOperator{\BCH}{BCH} \DeclareMathOperator{\spec}{sp} \newcommand{\qand}{\qquad\text{and}\qquad} \theoremstyle{definition} \newtheorem{point}{}[section] \newtheorem{defin}[point]{Definition} \newtheorem{chall}[point]{Challenge} \newtheorem{deco}[point]{} \newtheorem{remark}[point]{Remark} \newtheorem{example}[point]{Example} \newtheorem{discussion}[point]{Discussion} \theoremstyle{plain} \newtheorem{prop}[point]{Proposition} \newtheorem{lemma}[point]{Lemma} \newtheorem{cor}[point]{Corollary} \newtheorem{theorem}[point]{Theorem} \theoremstyle{remark} \newtheorem{itpoint}[point]{} \newcommand{\bo}{\boldsymbol} \newcommand{\leaveout}[1]{} \newcommand{\nnnewpage}{} \newcommand{\qedexer}{\renewcommand{\qedsymbol}{$\diamondsuit$}\qed\renewcommand{\qedsymbol}{$\Box$}} \newcommand{\qedremark}{\renewcommand{\qedsymbol}{$\triangle$}\qed\renewcommand{\qedsymbol}{$\Box$}} \newcommand{\qedno}{\renewcommand{\qedsymbol}{}} \newcommand{\proofnote}[1]{ \renewcommand{\qedsymbol}{$\triangle$} \begin{proof}[Note] #1 \renewcommand{\qedsymbol}{} \end{proof} \renewcommand{\qedsymbol}{$\Box$} } \newcommand{\eqed}{ \pushQED{\qed} \qedhere \popQED } \newcommand{\eqedexer}{ \renewcommand{\qedsymbol}{$\diamondsuit$} \pushQED{\qed} \qedhere \popQED \renewcommand{\qedsymbol}{$\Box$} } \newcommand{\eqedremark}{ \renewcommand{\qedsymbol}{$\triangle$} \pushQED{\qed} \qedhere \popQED \renewcommand{\qedsymbol}{$\Box$} } \newcommand{\eqedstate}{ \renewcommand{\qedsymbol}{$\Box$} \pushQED{\qed} \qedhere \popQED \renewcommand{\qedsymbol}{$\Box$} } \newcommand{\proofremarkqed}[1]{ \begin{proof}[Remark] #1 \renewcommand{\qedsymbol}{$\triangle$} \end{proof} \renewcommand{\qedsymbol}{$\Box$} } \renewcommand{\thesubsection}{{\arabic{section}.\Alph{subsection}}} \newcommand{\marginextend}[1]{ \addtolength{\oddsidemargin}{-#1} \addtolength{\evensidemargin}{-#1} \addtolength{\textwidth}{#1}\addtolength{\textwidth}{#1}} \newcommand{\updownextend}[1]{ \addtolength{\topmargin}{-#1} \addtolength{\textheight}{#1} \addtolength{\textheight}{#1}} \marginextend{1cm} \updownextend{0cm} \allowdisplaybreaks[4] \title[Convergence estimates for the Magnus expansion IE]{Convergence estimates for the Magnus expansion IE. Finite dimensional Banach algebras} \author{Gyula Lakos} \address{Alfréd Rényi Institute of Mathematics, Reáltanoda utca 13-15, Budapest, H--1053, Hungary} \email{[email protected]} \keywords{Magnus expansion, Baker--Campbell--Hausdorff expansion, convergence estimates, resolvent method} \subjclass[2020]{Primary: 46H30, Secondary: 16W80.} \begin{document} \begin{abstract} We review and provide simplified proofs related to the Magnus expansion, and improve convergence estimates. Observations and improvements concerning the Baker--Campbell--Hausdorff expansion are also made. In this Part IE, we consider the case of finite dimensional Banach algebras. We show that Magnus expansion is convergent (and works in logarithmic sense) if the cumulative norm $< 2+\varepsilon_{n}$, where $\varepsilon_{n}$ is a positive number depending on the dimension $n$ of the Banach algebra. We also show concrete finite-dimensional counterexamples of multiple Baker--Campbell--Hausdorff type for any cumulative norm $>2$ (necessarily of possibly great dimension). \end{abstract} \maketitle \section*{Introduction} This paper is a direct continuation of Part I \cite{L1}. Here we will consider finite dimensional Banach algebras but which are not far from the extremal case of Part I. \textbf{Introduction to the setting of finite dimensional Banach algebras.} Since its inception, Magnus \cite{M}, finite dimensional normed algebras, which are automatically Banach algebras, play a prominent role in the study and applications of the Magnus expansion, cf.~Blanes, Casas, Oteo, Ros \cite{BCOR}. In the case of operators on Hilbert spaces, finitely dimensionality is not a strong restriction with respect to the convergence radius in terms of the cumulative norm: The counterexamples by Sch\"affer \cite{Scha} (indirectly), Vinokurov \cite{V}, Moan \cite{Ma}, and the convergence results of Moan, Niesen \cite{MN}, Casas \cite{Ca} show that the critical case for the convergence of the Magnus expansion for Hilbert space operators is the cumulative norm $\pi$, and it is represented by concrete counterexamples over $2\times2$ real matrices. (See Part II \cite{L2} for a detailed discussion.) In the case of general, possibly infinite dimensional Banach algebras, results of Moan, Oteo \cite{MO} and Part I \cite{L1} show that the critical convergence radius in terms of the cumulative norm is $2$, and counterexamples for convergence with cumulative norm $2$ must need to be somewhat wild. It is not immediately clear that how finite dimensionality affects these results; but, as it turns out, positively. An ease in the finite dimensional case that issues of convergence do not depend on the choice of the norm, as all norms are equivalent. Selecting among the convergent or divergent measures in terms of the possible norms is a different matter, however. \snewpage \textbf{Outline of content.} In Section \ref{sec:DivFin}, we start with considering the ``Minimal Examples'', which are counterexamples to the convergence of the BCH expansion in the setting of $2\times2$ upper triangular matrix algebras; these are of cumulative norm $\pi+\varepsilon$. Next, we exhibit examples of multiple Baker--Campbell--Hausdorff type for divergence with cumulative norm $2\frac{n}{n-1}$ in some $n\times n$ matrix algebras (which are, of course, of dimension $d=n^2$), where $n\geq2$. Although examples are sometimes results of lucky choices, here will explain how these latter finite dimensional counterexamples relate to the infinite dimensional ones of Part I \cite{L1}. In Section \ref{sec:ConFin}, we show that Magnus expansion is convergent and works in logarithmic sense if the cumulative norm $< 2+\varepsilon_{n}$, where $\varepsilon_{n}$ is a positive number depending on the (real) dimension $n$ of the Banach algebra. This section draws heavily on the resolvent method introduced in the formal case by Mielnik, Pleba\'{n}ski \cite{MP}, and developed further in Part I \cite{L1}. Interestingly, methods of discrete geometry are used; notably, we use the estimates of Rogers \cite{R} on packing densities of centrally convex bodies. \textbf{Acknowledgements.} The author is grateful to Fernando Casas and Kurusch Eb\-ra\-hi\-mi-Fard for organizing venues for furthering research, and also to Pierre-Louis Giscard, Ilya Kuprov, and Stefano Pozza for inspiring conversations. \begin{commentx} \tableofcontents \end{commentx} \snewpage \section{Divergence for finite dimensional Banach algebras} \plabel{sec:DivFin} Here we will see how the higher dimensionality leads to divergent Magnus series with measures of cumulative norm approaching $2$ from above. \subsection{Low dimensional counterexamples} \plabel{ss:low} ~\\ Real and complex algebras of low dimension can be classified, cf. Study \cite{Stu}, Scheffers \cite{Sche}, Kobayashi, Shirayanagi, Takahasi, Tsukada \cite{KSTT}. It is trivial that if the dimension $d$ of the unital algebra $\mathfrak A$ is $1$ or $2$, then the algebra is commutative. Recall that in the commutative case the Magnus series is trivially convergent; the logarithmic version of Magnus formula holds if the cumulative norm is $<\pi$, and beyond that it may or may not hold (compare, say, $\mathbb R$ and $\mathbb C$). It is still easy to see that if the dimension $d$ of the unital algebra $\mathfrak A$ is $3$, then the algebra is either commutative or isomorphic to the algebra of upper triangular matrices. Now, upper triangular matrices already lead to some divergent Magnus series: \begin{lemma}\plabel{lem:rys1} Assume that $-\pi\leq\alpha\leq\pi$ and $\varepsilon\neq0$. Let \[M_1^{[\alpha,\varepsilon]}=\begin{bmatrix} \frac{\pi-\alpha}2& -\frac{\pi+\alpha}2 \varepsilon\\&- \frac{\pi-\alpha}2 \end{bmatrix} \qquad\text{and}\qquad M_2^{[\alpha,\varepsilon]}=\begin{bmatrix} \frac{\pi+\alpha}2 & \frac{\pi-\alpha}2 \varepsilon\\ &- \frac{\pi+\alpha}2 \end{bmatrix}.\] Then the Magnus (BCH) series of the ordered measure $M_1^{[\alpha,\varepsilon]}\mathbf 1_{[0,1)}\boldsymbol. M_2^{[\alpha,\varepsilon]}\mathbf 1_{[1,2)}$ is not absolutely convergent. \begin{proof} If the series $k\mapsto\mu_{k,\mathbb R}\left( M_1^{[\alpha,\varepsilon]}\mathbf 1_{[0,1)}\boldsymbol. M_2^{[\alpha,\varepsilon]}\mathbf 1_{[1,2)}\right)$ were absolute convergent, then so would be $k\mapsto\mu_{k,\mathbb R}\left(\mathrm i\cdot(M_1^{[\alpha,\varepsilon]}\mathbf 1_{[0,1)}\boldsymbol. M_2^{[\alpha,\varepsilon]}\mathbf 1_{[1,2)})\right)$ in the complex(ified) algebra. Assume that $M^{[\alpha,\varepsilon]}$ is the absolute convergent sum of the Magnus series. Then \begin{multline*}\exp M^{[\alpha,\varepsilon]}= \Rexp\left(\mathrm i\cdot(M_1^{[\alpha,\varepsilon]}\mathbf 1_{[0,1)}\boldsymbol. M_2^{[\alpha,\varepsilon]}\mathbf 1_{[1,2)})\right)=\\=\exp (\mathrm iM_1^{[\alpha,\varepsilon]})\exp (\mathrm iM_2^{[\alpha,\varepsilon]})= \begin{cases}\begin{bmatrix}-1&-\frac{ \varepsilon(\pi^2+\alpha^2)(1+\mathrm e^{-\mathrm i \alpha }) }{\pi^2- \alpha^2}\\&-1\end{bmatrix} &\text{for } -\pi < \alpha < \pi ,\\\\ \begin{bmatrix}-1&\pm\pi\mathrm i\varepsilon \\&-1\end{bmatrix} &\text{for } \alpha =\pm \pi . \end{cases} \end{multline*} In any case, the result is not diagonalizable, not even in the larger algebra of $2\times 2$ complex matrices. It implies that $M^{[\alpha,\varepsilon]}$ must have Jordan form $\begin{bmatrix}a&1 \\&a\end{bmatrix}$ in the larger algebra of $2\times 2$ complex matrices with $a\in2\pi\mathrm i\left(\mathbb Z+\frac12 \right)$. This, however, contradicts to the fact that the terms of the Magnus (BCH) series should be traceless in the larger algebra of $2\times 2$ complex matrices. (The first term is known, and the other ones are sums of commutators. Or, alternatively, this also follows directly from considering the homomorphisms given by the diagonal positions.) \end{proof} \end{lemma} In the previous proof, instead of ``proper analysis'', only linear algebra was used. The following lemma, of more analytical nature, indicates the oscillatory nature of the coefficients: \begin{lemma}\plabel{lem:rys2} In the previous lemma, for the terms of the Magnus expansion, $k\geq1$, \begin{multline*} \mu_{k,\mathrm R}\left( M_1^{[\alpha,\varepsilon]}\mathbf 1_{[0,1)}\boldsymbol. M_2^{[\alpha,\varepsilon]}\mathbf 1_{[1,2)}\right)=\begin{bmatrix}\pi& \\&-\pi\end{bmatrix}\cdot\delta_{k,1}+\\+ \begin{bmatrix}0&1 \\&0\end{bmatrix}\cdot \begin{cases}(-1)^{\frac{k }2}\cdot- \frac{2(\pi^2+ \alpha ^2)(1+\cos\alpha) \varepsilon}{\pi^2-\alpha ^2}+O(2^{-k})&\text{for $k$ even, } -\pi < \alpha < \pi ,\\\\ (-1)^{\frac{k+1}2}\cdot- \frac{2(\pi^2+ \alpha^2)(\sin\alpha)\varepsilon}{ \pi^2- \alpha^2}+O(2^{-k})&\text{for $k$ odd, } -\pi < \alpha < \pi ,\\\\ O(2^{-k})&\text{for $k$ even, } \alpha =\pm \pi ,\\\\ \pm(-1)^{\frac{k-1}2}2\pi\varepsilon+ O(2^{-k})&\text{for $k$ odd, } \alpha =\pm \pi . \end{cases} \end{multline*} In particular, the terms of the Magnus expansion limit to $\begin{bmatrix}0& \pm c^{[\alpha]}_{\mathrm{odd}}\varepsilon\\&0\end{bmatrix}$ and $\begin{bmatrix}0& \pm c^{[\alpha]}_{\mathrm{even}}\varepsilon\\&0\end{bmatrix}$ where $c^{[\alpha]}_{\mathrm{odd}},c^{[\alpha]}_{\mathrm{even}}\geq0 $, but at least one of them is nonzero. \begin{proof} Let \[M^{[\alpha,\varepsilon]}(t)=\begin{cases}\begin{bmatrix}\pi t& {\dfrac {t\varepsilon\,{\pi } \left( (\pi^2+ \alpha^2) \left( \cosh \left( t\pi\right) -\mathrm e^{ - t \alpha } \right)- 2 \alpha \pi\,\sinh \left( t\pi \right) \right) }{ \left( {\pi }^{2}- \alpha^{2} \right)\sinh \left( t\pi \right) }} \\&-\pi t\end{bmatrix} \\\hfill\text{for }-\pi < \alpha < \pi ,\\ \begin{bmatrix}\pi t&\mp\pi t\varepsilon\left(\boldsymbol\beta(\pm2\pi t) \right) \\&-\pi t\end{bmatrix} \hfill\text{for } \alpha =\pm \pi ;\\ \end{cases}\] where we used the abbreviation $\boldsymbol\beta(x)=\frac{x}{\mathrm e^x-1}$. Then $M^{[\alpha,\varepsilon]}(t)$ is meromorphic in $t$ with possible poles in $\mathrm i\mathbb Z$, and it can be checked that \[\exp M^{[\alpha,\varepsilon]}(t)=\Rexp\left(t\cdot(M_1^{[\alpha,\varepsilon]}\mathbf 1_{[0,1)}\boldsymbol. M_2^{[\alpha,\varepsilon]}\mathbf 1_{[1,2)})\right).\] In fact, $M^{[\alpha,\varepsilon]}(t)$ extends to $t=0$ holomorphically with $M^{[\alpha,\varepsilon]}(0)=\Id_2$, and the exponential identity above also holds there. As the continuation of $\log$ is unique near the identity, \[M^{[\alpha,\varepsilon]}(t)=\log\Rexp\left(t\cdot(M_1^{[\alpha,\varepsilon]}\mathbf 1_{[0,1)}\boldsymbol. M_2^{[\alpha,\varepsilon]}\mathbf 1_{[1,2)})\right) =\sum_{k=1}^\infty t^k\mu_{k,\mathrm R}\left( M_1^{[\alpha,\varepsilon]}\mathbf 1_{[0,1)}\boldsymbol. M_2^{[\alpha,\varepsilon]}\mathbf 1_{[1,2)} \right)\] holds for $t\sim 0$. Now, $\left(M^{[\alpha,\varepsilon]}(t)\right)_{1,2}$ has residue \begin{align*} &{\frac { ({\pi }^{2}+ \alpha ^2) \varepsilon\, \left( \mathrm i\right) (1+\mathrm e^{-\mathrm i\alpha}) }{{ \pi }^{2}- \alpha ^{2}}} &&\text{at } t=\mathrm i \text{ for } -\pi < \alpha < \pi ,\\ & {\frac { ({\pi }^{2}+ \alpha ^2 )\varepsilon\, \left(-\mathrm i\right) (1+\mathrm e^{\mathrm i\alpha}) }{{ \pi }^{2}- \alpha ^{2}}} &&\text{at } t=-\mathrm i \text{ for } -\pi < \alpha < \pi ,\\ & \pm \pi \varepsilon &&\text{at } t= \mathrm i \text{ for } \alpha =\pm \pi ,\\ & \pm \pi \varepsilon &&\text{at } t= -\mathrm i \text{ for } \alpha =\pm \pi . \end{align*} Then the residues at $t=\pm\mathrm i$ give the macroscopic contributions in the power series, the residues at $t=\pm 2\mathrm i$ give $O(2^{-k})$, and the rest is altogether even smaller. \end{proof} \end{lemma} \begin{example}\plabel{ex:rys} In the (proof of) the previous lemma, the cases $\alpha=0$ (the ``balanced cases'') and $\alpha=\pm \pi $ (the ``totally unbalanced cases'') are particularly instructive. Here we find for $t\sim 0$, or even for $|t|<1$, or even just symbolically in $t$, \[ \sum_{k=1}^\infty t^k\mu_{k,\mathrm R}\left( M_1^{[\alpha,\varepsilon]}\mathbf 1_{[0,1)}\boldsymbol. M_2^{[\alpha,\varepsilon]}\mathbf 1_{[1,2)}\right) =\begin{cases}\begin{bmatrix}\pi t& \varepsilon\pi t \tanh\left(\frac\pi2 t\right) \\&-\pi t\end{bmatrix} &\text{for } \alpha=0,\\\\ \begin{bmatrix}\pi t&\mp \pi t\varepsilon\left(\boldsymbol\beta(\pm2\pi t) \right) \\&-\pi t\end{bmatrix} &\text{for } \alpha =\pm \pi \\ \end{cases} \]\[ =\begin{cases}\begin{bmatrix}\pi t& \sum_{j=1}^\infty -4(-1)^j (1-2^{-2j})\zeta(2j)\varepsilon t^{2j} \\&-\pi t\end{bmatrix} &\text{for } \alpha=0,\\\\ \begin{bmatrix}\pi t&\mp \pi\varepsilon t + \pi^2 \varepsilon t^2\pm\sum_{j=1}^\infty (-1)^j2\pi\zeta(2j)\varepsilon t^{2j+1} \\&-\pi t\end{bmatrix} &\text{for } \alpha =\pm \pi . \end{cases} \] (One may recall that as $1<2j\nearrow+\infty$, one has $\zeta(2j)\searrow1$, and even $(1-2^{-2j})\zeta(2j)\searrow1$.) \end{example} The example(s) above are also instructive in illustrating that in the few cases when the Magnus expansion of $\phi$ is explicitly known, it comes typically not from the direct evaluation of the iterated integrals but from the knowledge of $\log(\Rexp(t\cdot\phi))$ for $t\sim 0$. \begin{lemma}\plabel{lem:rys3} Assume that the algebra of $2\times2$ (real or complex) upper triangular matrices is endowed by an algebra norm $\|\cdot\|$ such that $\left\| \begin{bmatrix}1&\\&-1\end{bmatrix} \right\|=1$. Then \[\|M_1^{[\alpha,\varepsilon]}\|+\|M_2^{[\alpha,\varepsilon]}\|\leq \pi+\pi|\varepsilon| \left\| \begin{bmatrix}0&1\\&0\end{bmatrix}\right\|. \] In the totally unbalanced cases, \[\|M_1^{[-\pi,\varepsilon]}\|=\pi, \qquad \|M_2^{[-\pi,\varepsilon]}\|= \pi|\varepsilon| \left\| \begin{bmatrix}0&1\\&0\end{bmatrix}\right\|,\] and \[\|M_1^{[\pi,\varepsilon]}\|=\pi|\varepsilon| \left\| \begin{bmatrix}0&1\\&0\end{bmatrix}\right\|, \qquad \|M_2^{[\pi,\varepsilon]}\|= \pi.\] \begin{proof} These are trivial norm estimates. \end{proof} \end{lemma} \snewpage The main point is, \begin{theorem}\plabel{thm:rys1} Assume that the algebra of $2\times2$ (real or complex) upper triangular matrices is endowed by an $\|\cdot\|_{\ell^p}$ norm with $1\leq p\leq+\infty$, or even just with any algebra norm $\|\cdot\|$ such that $\left\| \begin{bmatrix}1&\\&-1\end{bmatrix} \right\|=1$. Let $r>0$ and $\delta>0$. Then (a) There exist an example of type Lemma \ref{lem:rys1} such that $\|M_1^{[\alpha,\varepsilon]}\|:\|M_2^{[\alpha,\varepsilon]}\|=r$ and $\|M_1^{[\alpha,\varepsilon]}\|+\|M_2^{[\alpha,\varepsilon]}\|<\pi+\delta$. (b) There exist a counterexample of BCH type $M_1\mathbf 1_{[0,1)}\boldsymbol. M_2\mathbf 1_{[1,2)}$ to the convergence of the Magnus expansion such that $\|M_1 \|:\|M_2 \|=r$ and $\|M_1 \|+\|M_2 \|=\pi+\delta$, and the coefficients of the Magnus expansion are unbounded. \begin{proof} (a) We can choose a small $\varepsilon>0$ such that $r\in\left[ \varepsilon \left\| \begin{bmatrix}0&1\\&0\end{bmatrix}\right\|, \varepsilon^{-1} \left\| \begin{bmatrix}0&1\\&0\end{bmatrix}\right\|^{-1} \right]$ and $\pi \varepsilon\left\| \begin{bmatrix}0&1\\&0\end{bmatrix}\right\| <\delta$. According the previous lemma, the first relation implies that as we pass from one totally unbalanced case to the other, the norm ratio $r$ will be realized. The second relation implies that the cumulative norm is still small. (b) We can linearly upscale (a). \end{proof} \end{theorem} \begin{remark}\plabel{rem:rys} The $\|\cdot\|_{\ell^p}$ norms with $p=1,2,+\infty$ can be computed relatively explicitly. For example, regarding the cumulative norm \[\|M_1^{[\alpha,\varepsilon]}\|_{\ell^p}+\|M_2^{[\alpha,\varepsilon]}\|_{\ell^p}=\pi+\pi|\varepsilon|\qquad\text{if }p=1,\infty;\] and \begin{multline*} \|M_1^{[\alpha,\varepsilon]}\|_{\ell^2}+\|M_2^{[\alpha,\varepsilon]}\|_{\ell^2}=\\ =\frac12\pi|\varepsilon|+ \sqrt{\left(\frac{\pi-\alpha}2\right)^2+\left(\frac{\pi+\alpha}2\right)^2\frac{|\varepsilon|^2}4} + \sqrt{\left(\frac{\pi+\alpha}2\right)^2+\left(\frac{\pi-\alpha}2\right)^2\frac{|\varepsilon|^2}4} \\ \leq \pi+\pi|\varepsilon| .\eqedremark \end{multline*} \end{remark} \begin{remark}\plabel{rem:rys11} The counterexamples in the previous theorem fit to the line of Wei \cite{W} in using the argument of parabolicity (but we use it in a slightly more sophisticated way), of Michel \cite{Mi} in finding a whole range of balanced and unbalanced counterexamples (but we consider a different setting), and of Vinokurov \cite{V} in finding relatively simple counterexamples (but we use an even simpler algebra and exhibit a bigger range of counterexamples). \qedremark \end{remark} \snewpage I prefer to think about the construction of Lemma \ref{lem:rys1} as the ``Minimal Examples''. This is not about any mathematical minimality property but that the construction realizes counterexamples in a quite minimal setting. There are variants of it, but one cannot go below cumulative norm $\pi$: \begin{theorem}\plabel{thm:rys2} Assume that $\mathfrak A$ is the algebra of the $2\times2$ upper triangular matrices (real or complex), endowed by a Banach algebra norm $|\cdot|_{\mathfrak A}$. Assume that $\phi$ is a continuous $\mathfrak A$-valued measure of cumulative norm $\leq\pi$. Then the Magnus series of $\phi$ is (absolute) convergent. \begin{proof} Using standard Neumann series arguments, it is easy to see that the absolute value of the diagonal elements is dominated by the norm. More precisely, if $X\in \mathfrak A$, then $|(X)_{1,1}|\leq |X|_{\mathfrak A}$ and $|(X)_{2,2}|\leq |X|_{\mathfrak A}$. If $\int|\phi|_{\mathfrak A}<\pi$, then in the complex(ified) setting for $t\in\Dbar(0,1+\varepsilon)$, $M(t)=\log\Rexp(t\cdot\phi)$ exists as one can apply the definition $\log A=\int_{\lambda\in{[0,1]}} \frac{A-1}{\lambda+(1-\lambda)A}\,\mathrm d\lambda$ based on the critical behavior in the diagonals. The power series expansion of $M(t)$ around $t=0$ gives the convergent Magnus expansion for $t=1$. In fact, the same applies even if $\int|\phi|_{\mathfrak A}=\pi$ but $\left|\int(\phi)_{1,1}\right|<\pi$ and $\left|\int(\phi)_{2,2}\right|<\pi$. Let us now consider the measure $\tilde \phi$ such that $\phi=\frac12(\int(\phi)_{1,1}+\int(\phi)_{2,2})\cdot\Id_2+\tilde \phi$. I. e. $\tilde\phi$ is the traceless modification of $\phi$. The Magnus expansion of $\tilde \phi$ is equiconvergent to the Magnus expansion of $ \phi$. This traceless modification is convergent as the integrated modified diagonals get off from the boundary of $\Dbar(0,\pi)$ except if $C=\int(\phi)_{1,1}=-\int(\phi)_{2,2}$ with $|C|=\int|\phi|_{\mathfrak A}=\pi$. It is sufficient to consider this latter case. Reparametrizing the measure $\phi$ in terms of variation, and multiplying it a complex unit vector, we can assume that $\phi$ is supported on $[0,\pi)$ and $\phi=\begin{bmatrix}\mathbf 1_{[0,\pi)}&\mathbf c\\&-\mathbf 1_{[0,\pi)}\end{bmatrix}$ and $|\psi|_{\mathfrak A}=\mathbf 1_{[0,\pi)}$. Now, $\mathbf c$ may be essentially constant i. e. a scalar times $\mathbf 1_{[0,\pi)}$. In this case the Magnus expansion is convergent. If this is not so, then there are elements $\begin{bmatrix}1&a\\&-1\end{bmatrix}$ and $\begin{bmatrix}1&b\\&-1\end{bmatrix}$ of unit norm such that $a\neq b$. But then the norm of $n\begin{bmatrix}0& (b-a)\\&0\end{bmatrix} =\left(\begin{bmatrix}1&a\\&-1\end{bmatrix}\begin{bmatrix}1&b\\&-1\end{bmatrix}\right)^n-\begin{bmatrix}1&\\&1\end{bmatrix}$ would be bounded by $2$, which is an absurdum. \end{proof} \end{theorem} If dimension $4$ is allowed for an algebra $\mathfrak A$, then there are already counterexamples to cumulative $\pi$ with respect to the convergence of the Magnus expansion. Indeed, in the case of real $2\times 2$ matrices with the $\ell^2$ operator norm the Moan--Schäffer example (see Schäffer \cite{Scha}, Moan \cite{Ma}, Moan, Niesen \cite{MN}) or the Magnus critical example (see Part II \cite{L2} for a detailed discussion) are like that. Taking $\oplus\mathbb R$ and the joint maximum norm, we can always increase the dimension of the Banach algebras while leaving the cumulative norm of the counterexample the same. However, we have promised a series of counterexamples with cumulative norm $\searrow2$. \snewpage ~ \subsection{Higher dimensional counterexamples} \plabel{ss:high} ~\\ Let us start with a particular example, which we will generalize. \begin{example} \plabel{ex:div} Here we will construct an example for the divergence of Magnus expansion which is of mBCH type, using $5\times5$ real matrices of cumulative norm $5\cdot\frac2{5-1}=\frac52$. We will use the matrices \[M^{(5)}_1=\frac24\left[\begin{matrix}0&1&1&1&1\\&0&&&\\&&0&&\\&&&0&\\&&&&0\end{matrix}\right],\,\,\, M^{(5)}_2=\frac24\left[\begin{matrix}0&&&&\\-1&0&1&1&1\\&&0&&\\&&&0&\\&&&&0\end{matrix}\right],\,\,\, M^{(5)}_3=\frac24\left[\begin{matrix}0&&&&\\&0&&&\\-1&-1&0&1&1\\&&&0&\\&&&&0\end{matrix}\right],\] \[M^{(5)}_4=\frac24\left[\begin{matrix}0&&&&\\&0&&&\\&&0&&\\-1&-1&-1&0&1\\&&&&0\end{matrix}\right],\,\,\, M^{(5)}_5=\frac24\left[\begin{matrix}0&&&&\\&0&&&\\&&0&&\\&&&0&\\-1&-1&-1&-1&0\end{matrix}\right].\] The measure we will consider is $\psi_5=M^{(5)}_1\mathbf 1_{[0,1)}\boldsymbol. M^{(5)}_2\mathbf 1_{[1,2)}\boldsymbol. M^{(5)}_3\mathbf 1_{[2,3)}\boldsymbol. M^{(5)}_4\mathbf 1_{[3,4)}\boldsymbol. M^{(5)}_5\mathbf 1_{[4,5)}$. For the matrices we will use the $\ell^1$ operator norm $\|\cdot\|_{\ell^1}$. Then the cumulative norm of $\psi_5$ is \[\int\|\psi_5\|_{\ell^1}=\|M^{(5)}_1\|_{\ell^1}+\ldots+\|M^{(5)}_5\|_{\ell^1}=5\cdot\frac24=\frac52.\] On the other hand, \begin{align*} \Rexp(\psi_5)&=(\exp M^{(5)}_1)\ldots(\exp M^{(5)}_5) =(\Id_5+M^{(5)}_1)\ldots(\Id_5+M^{(5)}_5) \\ &= \left[ \begin {matrix} -{\dfrac {33}{32}}&-{\dfrac {41}{32}}&-{ \dfrac {21}{32}}&{\dfrac {9}{32}}&{\dfrac {27}{16}}\\\\ - {\dfrac {27}{16}}&-\dfrac{3}{16}&-{\dfrac {7}{16}}&\dfrac3{16}&{\dfrac {9}{8}} \\\\ -{\dfrac {9}{8}}&-{\dfrac {9}{8}}&\dfrac38&\dfrac18&\dfrac34 \\\\ -\dfrac34&-\dfrac34&-\dfrac34&\dfrac34&\dfrac12\\\\ -\dfrac12&- \dfrac12&-\dfrac12&-\dfrac12&1\end {matrix} \right] = \left[ \begin {matrix} 1&-7&-\sqrt {15}&5&-2 \\ -1&-2&2\,\sqrt {15}&5&-1\\ 1&8&0 &5&0\\ -1&-2&-2\,\sqrt {15}&5&1\\ 1&-7&\sqrt {15}&5&2\end {matrix} \right] \cdot\\&\qquad\cdot \underbrace{\left[ \begin {matrix} 1&0&0&0&0\\ 0&{\frac { 61}{64}}&-{\frac {5\,\sqrt {15}}{64}}&0&0\\ 0&{ \frac {5\,\sqrt {15}}{64}}&{\frac {61}{64}}&0&0\\ 0&0 &0&-1&1\\ 0&0&0&0&-1\end {matrix} \right]}_{=F_5} \cdot \left[ \begin {matrix} 1&-7&-\sqrt {15}&5&-2 \\ -1&-2&2\,\sqrt {15}&5&-1\\ 1&8&0 &5&0\\ -1&-2&-2\,\sqrt {15}&5&1\\ 1&-7&\sqrt {15}&5&2\end {matrix} \right]^{-1} \end{align*} informs us about the real Jordan form $F_5$ of $\Rexp(\psi_5)$. We see that the geometric multiplicity of the eigenvalue $-1$ is $1$, therefore it cannot be the exponential of a real matrix $M$. (The eigenvalue $-1$ is not forbidden for the exponential of a real matrix but its geometric multiplicity should be even.) On the other hand, the sum $M$ of the Magnus series should be exactly a matrix like that. \qedexer \end{example} That will be our general strategy: we will consider measures of real matrices with time-ordered exponential with eigenvalue $-1$ of geometric multiplicity $1$. In order to establish this behaviour we, in fact, do not have to deal with the Jordan form, a simple rank computation for $\Rexp(\phi)+\Id$ is sufficient. Now, we start the general construction. Let $Q^{(n)}_s$ be the $n\times n$ real matrix such that its elements ($i$th row, $j$th column) are given by \[\left(Q^{(n)}_u\right)_{i,j}=\delta_{i,u}\sgn(j-u)\] (where the Kronecker's delta notation is used). \begin{lemma}\plabel{lem:parprod} For any $1\leq k\leq n$, and any scalar $s$, \begin{multline*}(\exp sQ^{(n)}_1)\ldots(\exp sQ^{(n)}_k)=(\Id_n+ sQ^{(n)}_1)\ldots(\Id_n+ sQ^{(n)}_k) =\\= \left[\begin{array}{c|c} A^{(k)} (s)+B^{(k)}(s)&C^{(k,n-k)}(s)\\\hline 0_{(n-k)\times k}&\Id_{n-k} \end{array}\right]; \end{multline*} such that $A^{(k)}(s)$ is a $k\times k$ matrix whose elements ($i$th row, $j$th column) are given by \[\left(A^{(k)}(s)\right)_{i,j}= \begin{cases} (s+1)^{j-i+1}-(s+1)^{j-i-1}&\text{if }\quad i<j,\\ s+1&\text{if }\quad i=j,\\ 0&\text{if }\quad i>j; \end{cases}\] $B^{(k)}(s)$ is a $k\times k$ matrix whose elements ($i$th row, $j$th column) are given by \[\left(B^{(k)}(s)\right)_{i,j}=(s+1)^{k-i}-(s+1)^{k-i+1};\] $C^{(k,n-k)}(s)$ is a $k\times (n-k)$ matrix whose elements ($i$th row, $j$th column) are given by \[\left(C^{(k,n-k)}(s)\right)_{i,j}=-(s+1)^{k-i}+(s+1)^{k-i+1}.\] (The elements $B^{(k)}(s)$ and $C^{(k,n-k)}(s)$ depend only on the row number.) \begin{proof} One can prove this by induction in $k$. \end{proof} \end{lemma} \begin{lemma}\plabel{lem:parprod2} (a) For any $1\leq n$, and any scalar $s$, \[ P^{(n)}(s):=(\exp sQ^{(n)}_1)\ldots(\exp sQ^{(n)}_n)=(\Id_n+ sQ^{(n)}_1)\ldots(\Id_n+ sQ^{(n)}_n) =A^{(n)} (s)+B^{(n)}(s) \plabel{eq:proder} \] (b) If $s+1\neq \pm 1$, then he eigenvalues of product matrix $P^{(n)}(s)$ have geometric multiplicity at most $1$. If $s=0$, then $\lambda=s+1=1$ has geometric multiplicity $n$; if $s=2$, then $\lambda=s+1=-1$ has geometric multiplicity $n-1$. (c) If $s=2+\frac2{n-1}$, then $-1$ is an eigenvalue of $P^{(n)}(s)$. \begin{proof} (a) This is an immediate consequence of the previous lemma. (b) Let us consider the matrix $P^{(n)}(s)-\lambda\Id_n=(A^{(n)} (s)-\lambda\Id_n) +B^{(n)}(s)$. If $\lambda\neq s+1$, then $(A^{(n)} (s)-\lambda\Id_n)$ is an invertible triangular matrix of full rank, while $B^{(n)}(s)$ is rank $0$ or $1$. Therefore the rank of the sum is $n-1$ of $n$. If $\lambda= s+1\neq -1$, then (we also know that $s+1\neq1$) we have $(s+1)^2-1\neq 0$, thus the column space $(A^{(n)} (s)-\lambda\Id_n)$ contains the column vectors whose last entry is $0$, and $(A^{(n)} (s)-\lambda\Id_n)$ also has a $0$ column. Meanwhile the columns of $B^{(n)}(s)$ are uniformly a column vector, whose last entry is $1-(s+1)=-s\neq0$. Then it is easy to see that the columns of $(A^{(n)} (s)-\lambda\Id_n) +B^{(n)}(s)$ must be independent. The special cases $\lambda= s+1=\pm1$ can be checked directly. (c) using (a) one can check that the column vector with uniform entries $1$ is an eigenvector with eigenvalue $-1$. \end{proof} \end{lemma} Now it is clear what to do, set $M^{(n)}_k=\frac2{n-1}Q^{(n)}_k$, and \[\psi_n=M^{(n)}_1\mathbf 1_{[0,1)}\boldsymbol. \ldots\boldsymbol. M^{(n)}_k\mathbf 1_{[k-1,k)}\boldsymbol. \ldots\boldsymbol. M^{(n)}_n\mathbf 1_{[n-1,n)}.\] Then we obtain \begin{theorem}\plabel{th:parprod} Let $n\geq2$. The cumulative norm of $\psi_n$ in the $\ell^1$ operator norm $\|\cdot\|_{\ell^1}$ of $n\times n$ matrices is \[\int \|\psi_n\|_{\ell^1}=\frac{2n}{n-1},\] while the Magnus expansion of $\psi_n$ is not absolutely convergent. \begin{proof} The statements about the cumulative norms are straightforward. As $-1$ is an eigenvalue of geometric multiplicity $1$ of $\Rexp (\psi_n)$, it cannot an exponential of a real matrix. Meanwhile, the sum $M$ of the Magnus series should be a matrix like that. \end{proof} \end{theorem} \begin{remark}\plabel{rem:parprod} One can see that the eigenvalues of $\Rexp (\psi_n)$ are on the unit circle, with the only nontrivial Jordan block $\begin{bmatrix}-1&1\\&-1\end{bmatrix}$ in complex form (corresponding to column vectors of arithmetic progressions). Indeed, one can argue as follows. Let $\mathbf v$ be the column vector of length $n$ containing only entries $1$. Then the matrix $\Id_n-\frac1n\mathbf v\mathbf v^\top$ defines a positive semidefinite quadratic form $S_n$ on $\mathbb R^n$, but it descends to a positive definite quadratic form $\tilde S_n$ on the factor space $\mathbb R^n/\mathbb R\mathbf v=\tilde{\mathbb R}^n$. Then $P_n=\Rexp(\psi_n)$ leaves $S_n$ invariant, and it also descends to linear map $\tilde P_n $ on $\tilde{\mathbb R}^n$, which is therefore orthogonal with respect to $\tilde S_n$. Thus the eigenvalues of $\tilde P_n$ are unit complex numbers with trivial Jordan blocks. Getting back to $\mathbb R^n$, can gives one extra eigenvalue from $\mathbb R\mathbf v$ which we know to be $-1$, and it can give only one extra nontrivial Jordan block, which must be as indicated because the effect of $P_n$ is easy to check on column vectors of arithmetic progressions. \qedremark \end{remark} \snewpage \begin{commenty} \begin{example}\plabel{ex:parprod} \[\Rexp(\psi_2)=\left[ \begin {array}{cc} -3&2\\ \noalign{\medskip}-2&1\end {array}\right]= \left[ \begin {array}{cc} 2&-\frac12\\ \noalign{\medskip}2&\frac12\end {array} \right] \left[ \begin {array}{cc} -1&1\\ \noalign{\medskip}0&-1\end {array}\right] \left[ \begin {array}{cc} 2&-\frac12\\ \noalign{\medskip}2&\frac12\end {array} \right]^{-1} ;\] \[ \Rexp(\psi_3)= \left[ \begin {array}{ccc} -2&-1&2\\ \noalign{\medskip}-2&0&1 \\ \noalign{\medskip}-1&-1&1\end {array} \right] = \left[ \begin {array}{ccc} 1&3&-1\\ \noalign{\medskip}-1&3&0 \\ \noalign{\medskip}1&3&1\end {array} \right] \left[ \begin {array}{ccc} 1&0&0\\ \noalign{\medskip}0&-1&1 \\ \noalign{\medskip}0&0&-1\end {array} \right] \left[ \begin {array}{ccc} 1&3&-1\\ \noalign{\medskip}-1&3&0 \\ \noalign{\medskip}1&3&1\end {array} \right]^{-1} ;\] \begin{multline*} \Rexp(\psi_4)= \left[ \begin {array}{cccc} -{\frac {115}{81}}&-{\frac {106}{81}}&-{ \frac {10}{81}}&{\frac {50}{27}}\\ \noalign{\medskip}-{\frac {50}{27}} &-{\frac {5}{27}}&-{\frac {2}{27}}&{\frac {10}{9}} \\ \noalign{\medskip}-{\frac {10}{9}}&-{\frac {10}{9}}&\frac59&\frac23 \\ \noalign{\medskip}-\frac23&-\frac23&-\frac23&1\end {array} \right] = \left[ \begin {array}{cccc} 7&\sqrt {5}&4&-\frac32\\ \noalign{\medskip}-3 &-3\,\sqrt {5}&4&-\frac12\\ \noalign{\medskip}-3&3\,\sqrt {5}&4&\frac12 \\ \noalign{\medskip}7&-\sqrt {5}&4&\frac32\end {array} \right] \cdot\\\cdot \left[ \begin {array}{cccc} {\frac {79}{81}}&-{\frac {8\,\sqrt {5}}{ 81}}&0&0\\ \noalign{\medskip}{\frac {8\,\sqrt {5}}{81}}&{\frac {79}{81 }}&0&0\\ \noalign{\medskip}0&0&-1&1\\ \noalign{\medskip}0&0&0&-1 \end {array} \right] \left[ \begin {array}{cccc} 7&\sqrt {5}&4&-\frac32\\ \noalign{\medskip}-3 &-3\,\sqrt {5}&4&-\frac12\\ \noalign{\medskip}-3&3\,\sqrt {5}&4&\frac12 \\ \noalign{\medskip}7&-\sqrt {5}&4&\frac32\end {array} \right]^{-1}. \eqedexer \end{multline*} \end{example} \end{commenty} For $d\geq3$, let $\mathrm C^{\{\{d\}\}}_{\mathbb K}$ denote the infimum of the cumulative norms of $\mathfrak A$ valued ordered measures whose Magnus expansion is not absolutely convergent and $\mathfrak A$ is a $d$ dimensional Banach algebra over $\mathbb K$. We know that $\mathrm C^{\{\{3\}\}}_{\mathbb K}=\pi$, and $\mathrm C^{\{\{d\}\}}_{\mathbb K}$ is (possibly not strictly) decreasing in $d$, but $\geq2$. By Theorem \ref{th:parprod}, for dimension $d=n^2$ ($n\geq2$), the continuous measure $\psi_n$ provides a counterexample to the convergence of the Magnus expansion. Thus, for $d\geq4$, \begin{equation} \mathrm C^{\{\{d\}\}}_{\mathbb R}\leq 2+\frac2{ \lfloor\sqrt d \rfloor-1}. \plabel{eq:ram} \end{equation} (Meanwhile, $\mathrm C^{\{\{d\}\}}_{\mathbb R}\geq \mathrm C^{\{\{d\}\}}_{\mathbb C}\geq \mathrm C^{\{\{2d\}\}}_{\mathbb R}$ is trivial.) Note, however, that the cumulative of $\psi_2$ is $4$ and the cumulative of $\psi_3$ is $3$. So, these counterexamples get lower cumulative norms than $\pi$ only for dimensions $d\geq9$. For $4\leq\dim_{\mathbb K}\mathfrak A\leq8$, it would be interesting to see counterexamples with cumulative norm less than $\pi$, if they exist. In particular, most prominently, it is not clear how far one can go when $\mathfrak A$ is isomorphic to the algebra of real $2\times2$ matrices (allowing other norms than the $\ell^2$ operator norm). \snewpage ~ \subsection{Relationship to the free $L^1$ mBCH counterexamples} \plabel{ss:rel} ~\\ In Part I \cite{L1} we have considered counterexamples of mBCH type for the convergence of the Magnus expansion. There, the tautological measure $2\cdot\mathrm Z^1_{[0,1)}$ was replaced by \begin{multline*} \hat\psi_n=\log\Rexp(2\cdot\mathrm Z^1_{[0,\frac1n)} )\mathbf1_{[0,1)}\boldsymbol.\ldots\\ \ldots\boldsymbol. \log\Rexp(2\cdot\mathrm Z^1_{[\frac{k-1} n,\frac{k}n )})\mathbf1_{[k-1,k)}\boldsymbol.\ldots\boldsymbol. \log\Rexp(2\cdot\mathrm Z^1_{[\frac{n-1} n,\frac{n}n)} )\mathbf1_{[n-1,n)} \end{multline*} (with $n\geq2$). In that the cumulative norm of the measure increases only slightly, but the time-ordered exponentals will be the same in $\mathrm F^{1,\mathrm{loc}}([0,1))$ but not lying in $\mathrm F^{1 }([0,1))$. These counterexample can be understood in the algebra finitely generated by the $\log\Rexp(2\cdot\mathrm Z^1_{[\frac{k-1} n,\frac{k}n )})$ but which is, nevertheless infinite dimensional and not particularly manageable. Also, the cumulative norm $\int|\hat\psi_n|=n\Theta\left(\frac2n\right)=2+\frac2n+O\left(\frac1{n^2}\right) $ and the cumulative norm $\int\|\psi_n\|_{\ell^1}=2+\frac2{n-1}$ are quite comparable. According to these, the counterexamples $\psi_n$ compare preferably to the $\hat\psi_n$. Here we would like to argue that the $\psi_n$ and $\hat\psi_n$ closely related to each other. The point is that $\psi_n$ can be obtained from $\hat\psi_n$ by ``reducing'' the algebra $\mathrm F^{1 }([0,1))$. Although the following discussion can be made completely precise, for the sake of ease, we keep it informal. In particular, we will sometimes pretend that $\mathrm F^{1 }([0,1))$ is the superposition of elements $X_{t_1}\ldots X_{t_k}$ with $t_i\in[0,1)$, even if this is not so simple. The precise idea is that $\mathrm F^{1 }([0,1))$ can be subjected to contractive homomorphisms but which can be set up so that the terms of the Magnus series are kept seen relatively large. The first thing one can do is to replace $X_{t_1}\ldots X_{t_k}$ by $\mathsf{a}^{\asc(t_1,\ldots,t_k)}\mathsf{d}^{\des(t_1,\ldots,t_k)}X_{t_1,t_2}$, i.~e. the internal structure of the expression gets ignored, only the number of ascents and descents get recorded. Actually, it is sufficient to have $\mathsf{a}\equiv1$ and $\mathsf{d}\equiv-1$ here; this setup still keeps the size of the terms of the Magnus expansion. This leads to an ``abstract composition kernel approach''. What can make this more down-to-earth is to consider it as a representation. We have the representation space generated by superpositions of $Y_t$ ($t\in [0,1)$), where the representation rule is $X_{t_1} Y_{t_2}=Y_{t_1}$ for $t_1<t_2$ and $X_{t_1} Y_{t_2}=-Y_{t_1}$ for $t_1>t_2$. If the superpositions of $Y_t$ is coded as an $L^1$ function $f(t)$, then the effect of $Z_{[a,b)}$ on $f(t)$ leads to $\tilde f(t)$ where \[\tilde f(t)=\int_{s\in[0,1)} \underbrace{\chi_{[a,b)}(t) \sgn(s-t)}_{=K_{[a,b)}(s,t)}\,f(s)\,\mathrm ds.\] Thus $Z_{[a,b)}$ gets represented by kernel $K_{[a,b)}(s,t)$ ($L^1$ in $s$, $L^\infty$ in $t$). This representation is still quite strange, and indeed, there is no loss in norm with respect to terms of the Magnus expansion. \snewpage What one can make the situation tamer is the introduction of additional eliminating relations $X_{t_1}X_{t_2}=0$ for $t_1,t_2\in[\frac{k-1}n,\frac{k}n )$. This \textit{will} improve the convergence of the Magnus expansion but hopefully not so much. This is compatible to the representation process of the previous paragraph. It leads to the kernels $K^{[1/n]}_{[a,b)}(s,t)$ where the values on the squares $[\frac{k-1}n,\frac{k}n )\times [\frac{k-1}n,\frac{k}n ) $ are killed. Then, the image of the time-ordered exponential of $2\cdot\mathrm Z^1_{[0,1)}$, and of $\hat \psi_n$ and the computations with the latter one can be viewed in terms of kernels which are linear combinations of characteristic functions of $[\frac{k-1}n,\frac{k}n )\times [\frac{l-1}n,\frac{l}n )$. I. e. in terms of $n\times n$ matrices. Indeed, this is exactly the process how $\psi_n$ was obtained from $\hat \psi_n$. \snewpage \section{Convergence for finite dimensional Banach algebras} \plabel{sec:ConFin} We recall certain estimates due to Rogers, which are standard material. Let us consider any centrally symmetric compact convex body $H$ in the $n$-dimensional space $\mathbb R^n$. Let $\vartheta(H)$ be the infimum of the covering density of $\mathbb R^n$ by translates of $H$; and let $\vartheta_L(H)$ be the same with respect to lattice coverings. Then, by Rogers \cite{R}, for $n\geq3$, \begin{align} \vartheta(H)\leq \vartheta_L(H)\leq&\min_{0<\eta<1/n}(1+\eta)^n(1+n\log(1/\eta))\plabel{eq:r1}\\ &=-n{ W}_{-1} \left(-{\tfrac1n}\right) \left( 1-{\frac {1}{n { W}_{-1} \left(-{\tfrac1n}\right)} } \right) ^{n+1}\notag\\ <& \left(1+\frac1{n\log n}\right)^n(1+n\log(n\log n))\plabel{eq:r2}\\ <& n\log n +n\log\log n+2n+1 \plabel{eq:r3}\\ <& n\log n +n\log\log n+5n.\plabel{eq:r4} \end{align} Here \eqref{eq:r1} is the proper result of \cite{R} (which is expressed explicitly using the Lambert $W_{-1}$ function); \eqref{eq:r2} reflects the choice $\eta=\frac1{n\log n}$; and while, for example, \eqref{eq:r3} is still true, \eqref{eq:r4} is the much quoted estimate; see G. Fejes Tóth \cite{F} for more on this. The main point is that there is a dimension-dependent but otherwise universal quantity $\vartheta_n$ (which can be chosen as any expression on the RHS), which is nearly linear in $n$, estimating the minimal translative covering densities from above. Moreover, as it is explained in Rogers, Zong \cite{RZ}, if $0<r<1$, then $H$ can be covered by at most $(1+r^{-1})^n\vartheta(H)\leq (1+r^{-1})^n\vartheta_n$ translates of $rH$ (i. e. by homothetical copies of $H$ with ratio $r$). For a recent review on these topics, see Naszódi \cite{N}. \begin{theorem}\plabel{thm:gain2} Assume that $\mathfrak A$ is Banach algebra of finite real dimension $n$. Assume that $\phi$ is an ordered measure of cumulative norm $\int|\phi|=\omega$. Let $\lambda\in[0,1]$. Then \begin{multline} \left|\mu_{2,\mathrm R}(\phi)\right|= \left|\int_{t_1,t_2\in[0,1]} \lambda^{\asc(t_1,t_2)} (\lambda-1)^{\des(t_1,t_2)}\phi(t_1)\phi(t_2)\right| \\ \leq\frac{\omega^2}2\left(1- {\frac {{2}^{1-n}}{n} \left( {\frac {1-\frac2n}{\mathrm e}} \right) }\frac1{\vartheta_n} \min(\lambda,1-\lambda)\right). \plabel{eq:gar3} \end{multline} \begin{proof} The measure $\phi$ can be approximated by measures of mBCH type. Therefore it is sufficient to prove the statement in the case \[\phi=u_1\mathbf1_{ [\tau_0,\tau_1)}\boldsymbol.\ldots \boldsymbol.u_k\mathbf1_{\mathbf [\tau_{k-1},\tau_k)}\] such that $u_i\in\mathfrak A$, $|u_i|=1$, $0=\tau_0<\tau_1<\ldots<\tau_k=\omega$. We also write $\phi(t)=u(t)\mathbf 1_{[0,\omega)}$, where $u(t)$ is the corresponding piecewise constant function. \snewpage Let us apply the argument of Rogers, Zong \cite{RZ} where $ H$ is the closed unit ball $K$ of $\mathfrak A\simeq\mathbb R^n$. Then $K$ can be covered by at most $s= (1+r^{-1})^n\vartheta_n $ many copies of $rK$, say $K_1,\ldots, K_{\lfloor s\rfloor}$. Let $L_1=K\cap K_1$, and $L_i=(K\cap K_i )\setminus (K_1\cup \ldots \cup K_{i-1})$. Let $N_i$ be the union of those $[\tau_{p-1},\tau_p)$ such that $u_p\in L_i$. (I. e., formally, $N_i=u^{-1}(L_i)$.) Let $v_i$ be the Lebesgue measure of $N_i$. Clearly, $v_1+\ldots+v_s=\omega$. We cut $N_i$ into a lower part $N_i^-$ and lower part $N_i^+$, each of them of Lebesgue measure $v_i/2$. Let $P_i^-:[0,v_i/2)\rightarrow N_i^-$ and $P_i^+:[0,v_i/2)\rightarrow N_i^+$ be the corresponding monotone increasing measure preserving transformations. It is easy to see that $P_i^+(q)-P_i^-(q)\geq v_i/2$ for any $q$. On the other hand, $|u(P_i^+(q))-u(P_i^-(q))|\leq 2r$, by the construction of $L_i\subset K_i$. One can write \begin{multline*} \mu_{2,\mathrm R}(\phi) =\int_{t_1,t_2\in[0,\omega]} \lambda^{\asc(t_1,t_2)} (\lambda-1)^{\des(t_1,t_2)} u(t_1)u(t_2)\, \mathrm dt_1\,\mathrm dt_2\\ =\sum_i\int_{q\in [0,v_i/2),t_2\in[0,\omega]} \lambda^{\asc(P_i^-(q),t_2)} (\lambda-1)^{\des( P_i^-(q),t_2)} u(P_i^-(q))u(t_2) +\\+ \lambda^{\asc(P_i^+(q),t_2)} (\lambda-1)^{\des( P_i^+(q),t_2)} u(P_i^-(q))u(t_2) \, \mathrm dq\,\mathrm dt_2. \end{multline*} The norm of $(\lambda^{\asc(t_{11},t_2)} (\lambda-1)^{\des(t_{11},t_2)} u(t_{11})+ \lambda^{\asc(t_{12},t_2)} (\lambda-1)^{\des(t_{12},t_2)} u(t_{12}) )u(t_2)$ can be estimated $(\lambda^{\asc(t_{11},t_2)} (1-\lambda)^{\des(t_{11},t_2)}\cdot1 + \lambda^{\asc(t_{12},t_2)} (1-\lambda)^{\des(t_{12},t_2)}\cdot1 )\cdot1 ) $, but this leads only to the trivial estimate \[|\mu_{2,\mathrm R}(\phi)|\leq\frac12\omega^2.\] However, when $t_{11}<t_2<t_{12}$ or $t_{12}<t_2<t_{11}$ holds, then there is gain, as a component $\min(\lambda,1-\lambda)\cdot 2$ in the estimate can be replaced by $\min(\lambda,1-\lambda)\cdot | u(t_{11})-u(t_{12})|$. This is a gain $\min(\lambda,1-\lambda)\cdot (2-| u(t_{11})-u(t_{12})|)$ in the estimate. Therefore, we have a better overall estimate \begin{equation} |\mu_{2,\mathrm R}(\phi)|\leq\frac{\omega^2}2- \sum_i\min(\lambda,1-\lambda)\cdot(2-2r)\cdot\left(\frac{v_i}2\right)^2. \plabel{eq:gar1} \end{equation} The inequality between the arithmetic and square means implies that $\sum (v_i)^2\geq \frac{\omega^2}{\lfloor s \rfloor}\geq \frac{\omega^2}s$. Therefore \eqref{eq:gar1} implies \begin{equation} |\mu_{2,\mathrm R}(\phi)|\leq \frac{\omega^2}2\left(1- \frac{1-r}s\min(\lambda,1-\lambda)\right). \plabel{eq:gar2} \end{equation} In the previous argument the value of $0<r<1$ (and thus $s$) was unfixed. Now $r$ can be optimized to $r=\frac12\left(\sqrt{n^2+6n+1}-n-1\right)$, but, for the sake of simplicity, we take $r=1-\frac2n$. With this latter choice, \[\frac{1-r}s\frac1{\vartheta_n}= {\frac {{2}^{1-n}}{n} \left( {\frac {n-2}{n-1}} \right) ^{ n}}\frac1{\vartheta_n} > {\frac {{2}^{1-n}}{n} \left( {\frac {1-\frac2n}{\mathrm e}} \right) }\frac1{\vartheta_n} . \] Putting this into \eqref{eq:gar2}, we obtain \eqref{eq:gar3}. \end{proof} \end{theorem} \snewpage From Part I \cite{L1} we may recall: For $\lambda\in[0,1]$, the convergence radius of $\Theta^{(\lambda)}(x)$ around $x=0$ is \[{\mathrm C}_\infty^{(\lambda)}=\begin{cases} 2&\text{if }\lambda=\frac12, \\ \dfrac{2\artanh (1-2\lambda)}{1-2\lambda}=\dfrac{\log\dfrac{1-\lambda}{\lambda}}{1-2\lambda}&\text{if }\lambda\in(0,1) \setminus\{\frac12\},\\ +\infty&\text{if }\lambda\in\{0,1\}. \end{cases}\] This is a strictly convex, nonnegative function in $\lambda\in(0,1)$, symmetric for $\lambda\mapsto1-\lambda$; its minimum is ${\mathrm C}_\infty^{(1/2)}=2$. In particular, in $\lambda\in[0,1]$, it yields a $[2,+\infty]$-valued strictly convex continuous function. For $d\geq3$, let $\Lambda_d$ be the number such that $0<\Lambda_d\leq\frac12$ and ${\mathrm C}_\infty^{(\Lambda_d )}={\mathrm C}^{\{\{d\}\}}_{\mathbb R}$. We know that $\Lambda_d$ is increasing in $d$ but $\leq\frac12$. Then \[\Lambda_d\geq \Lambda_3=0.0588740902\ldots>\frac1{17}.\] For a better appreciation, asymptotically, \begin{lemma} \plabel{lem:appr} For $d\geq25$, \begin{equation} \Lambda_d\geq \frac12\left(1-\sqrt{\frac{3}{\lfloor\sqrt{d}\rfloor-1}}\right). \plabel{eq:emo} \end{equation} \begin{proof} Let $\tilde\Lambda_d$ denote the RHS of \eqref{eq:emo}. It is sufficient to prove that ${\mathrm C}_\infty^{(\tilde\Lambda_d )}\geq{\mathrm C}^{\{\{d\}\}}_{\mathbb R}$. In turn, by \eqref{eq:ram}, for that, it is sufficient to prove that ${\mathrm C}_\infty^{(\tilde\Lambda_d )}\geq2+\frac2{\lfloor\sqrt d\rfloor-1}$. On the other hand, it is not hard to prove that ${\mathrm C}_\infty^{(\lambda)}\geq2+\frac83(\lambda-1/2)^2$, leading to the desired result. \end{proof} \end{lemma} In general, however, the estimate $\Lambda_d>\frac1{17}$ may be convenient. Now we can state \begin{theorem} \plabel{thm:fin} Assume that $\mathfrak A$ is Banach algebra of finite real dimension $n$, and $\phi$ is an $\mathfrak A$ valued ordered measure such that \begin{equation} \int|\phi|<\frac{2}{1-\left( {\dfrac {{2}^{-2-n}}{n} \left( {\dfrac {1-\frac2n}{\mathrm e}} \right) }\dfrac{\Lambda_n}{\vartheta_n}\right)} . \plabel{eq:dana} \end{equation} Then $\psi$ is $M$-controlled, in particular, its Magnus expansion convergent, and the logarithmic Magnus formula holds. \begin{proof} Assume that $\phi$ is subject to \eqref{eq:dana}. Assume $\int|\phi|\geq2$. Let us decompose $\phi=\phi_1\boldsymbol.\phi_2\boldsymbol. \phi_3$, where $\int|\phi_1|=1$ and $\int|\phi_2|=2$. Let $\lambda\in[\Lambda_n,1-\Lambda_n]$. By Theorem \ref{thm:gain2}, \[\Theta^{(\lambda)}(\phi_1)\leq \Theta^{(\lambda)}( 1) -\underbrace{\frac{1}2\left( {\frac {{2}^{1-n}}{n} \left( {\frac {1-\frac2n}{\mathrm e}} \right) }\frac{\Lambda_n}{\vartheta_n}\right)}_{r:=} .\] That means that $\phi_1$ has delay \[1- \left(\Theta^{(\lambda)} \right)^{-1}(\Theta^{(\lambda)}( 1)-r)\geq 1- \left(\Theta^{(1/2)} \right)^{-1}(\Theta^{(1/2)}( 1)-r)\equiv\frac{r}{4-r}. \] (See the ``Delay estimate reduction principle'' of Part I.) The same applies to $\phi_2$, consequently $\phi_1\cdot\phi_2$ collects delay at least $\frac{2r}{4-r}$. Thus the resolvent expansion will be convergent (thus the resolvent $\mathcal R^{(\lambda)}(\Rexp \phi)$ exists) as long as $ \int|\phi_3|<\frac{2r}{4-r}$, i. e. if $\int|\phi |<2+\frac{2r}{4-r}=\frac{2}{1-\frac14r}$. But that was exactly our assumption. $\mathcal R^{(\lambda)}(\Rexp \phi)$ also exists if $\int|\phi|<2$. More generally, the statement about the existence of the resolvent for $\lambda\in[\Lambda_n,1-\Lambda_n]$ also holds if $\phi$ is replaced by $t\cdot \phi$ where $t$ is from the closed complex unit disk. Now, we claim, in general, \begin{equation} \frac{2}{1-\frac14r}\leq \mathrm C_{\mathbb R}^{\{\{n\}\}}. \plabel{eq:anto} \end{equation} Indeed, otherwise, $\frac{2}{1-\frac14r}> \mathrm C_{\mathbb R}^{\{\{n\}\}}$, and we could chose an $\delta>0$ such that \[\mathrm C_{\mathbb R}^{\{\{n\}\}}<\frac{2}{1-\frac14r(1-\delta)}.\] We also know that \[\mathrm C_{\mathbb R}^{\{\{n\}\}}=\mathrm C_\infty^{(\Lambda_n)} <\mathrm C_\infty^{(\Lambda_n(1-\delta))}.\] Finally, we could choose a counterexample $\psi$ to the convergence of the Magnus expansion such that \begin{equation}\int|\psi|<\frac{2}{1-\frac14r(1-\delta)}\plabel{eq:hora}\end{equation} and \begin{equation}\int|\psi|<\mathrm C_\infty^{(\Lambda_n(1-\delta))} .\plabel{eq:xora}\end{equation} Repeating the arguments of the previous paragraph, \eqref{eq:hora} causes $\mathcal R^{(\lambda)}(\Rexp (t\cdot\psi))$ to exist for $\lambda\in[\Lambda_n(1-\delta),1-\Lambda_n(1-\delta)]$ and $t\in\Dbar(0,1)$; while \eqref{eq:xora} causes $\mathcal R^{(\lambda)}(\Rexp (t\cdot\psi))$ to exist for $\lambda\in[0,\Lambda_n(1-\delta))$ and $\lambda\in(1-\Lambda_n(1-\delta),1]$. Ultimately, $\psi$ is $M$-controlled, and its Magnus expansion is convergent; this is a contradiction. Returning to the main statement now, \eqref{eq:anto} implies \[\int|\phi|\leq \mathrm C_{\mathbb R}^{\{\{n\}\}}\equiv \mathrm C_\infty^{(\Lambda_n)}.\] This, in turn, implies that $\mathcal R^{(\lambda)}(\Rexp (t\cdot\phi))$ to exist for $\lambda\in[0,\Lambda_n )$ and $\lambda\in(1-\Lambda_n ,1]$. Ultimately, $\phi$ is $M$-controlled. \end{proof} \end{theorem} If we ignore $M$-controlledness and the logarithmic issues, and we concentrate on the convergence of the Magnus expansion, then the previous theorem says \begin{equation} \mathrm C_{\mathbb R}^{\{\{n\}\}}\geq \frac{2}{1-\left( {\dfrac {{2}^{-2-n}}{n} \left( {\dfrac {1-\frac2n}{\mathrm e}} \right) }\dfrac{\Lambda_n}{\vartheta_n}\right)}. \plabel{eq:subdana} \end{equation} Note, however, that there is a huge gap between \eqref{eq:ram} and \eqref{eq:subdana}, which would be interesting to close. One can improve upon the cumulative convergence radius $\mathrm C_2=2.89847930\ldots$ of the BCH expansion by similar arguments. \snewpage \begin{thebibliography}{MM} \gbibitem{BCOR} Blanes, S.; Casas, F.; Oteo, J. A.; Ros, J.: The Magnus expansion and some of its applications. \textit{Phys. Rep.} \textbf{470} (2009), 151--238. \gbibitem{Ca} Casas, Fernando: Sufficient conditions for the convergence of the Magnus expansion. \textit{J. Phys. A } \textbf{40} (2007), 15001--15017. \gbibitem{F} Fejes Tóth, Gábor: A note on covering by convex bodies. \textit{Can. Math. Bull.} \textbf{52} (2009), 361--365. \gbibitem{KSTT} Kobayashi, Yuji; Shirayanagi, Kiyoshi; Takahasi, Sin-Ei; Tsukada, Makoto: A complete classification of three-dimensional algebras over $\mathbb R$ and $\mathbb C$ --- \begin{CJK}{UTF8}{gbsn} 温故知新\end{CJK} [OnKoChiShin] (visiting old, learn new). \textit{Asian-European Journal of Mathematics} \textbf{14} (2021), 2150131 (25 pages). \gbibitem{L1} Lakos, Gyula: \textit{Convergence estimates for the Magnus expansion I. Banach algebras.} arXiv:1709.01791 \gbibitem{L2} Lakos, Gyula: \textit{Convergence estimates for the Magnus expansion II. $C^*$-algebras.} arXiv:1910.03328 \gbibitem{M} Magnus, Wilhelm: On the exponential solution of differential equations for a linear operator. \textit{Comm. Pure Appl. Math.} \textbf{7} (1954), 649--673. \gbibitem{Mi} Michel, Jean: Bases des alg\`{e}bres de Lie et s\'erie de Hausdorff. \textit{S\'eminaire Dubreil. Alg\`ebre,} \textbf{27} n.1 (1973-1974), exp. n. 6, 1-9 (1974). \gbibitem{MP} Mielnik, Bogdan; Pleba\'{n}ski, Jerzy: Combinatorial approach to Baker-Campbell-Hausdorff exponents. \textit{Ann. Inst. H. Poincar\'e Sect. A (N.S.)} \textbf{12} (1970), 215--254. \gbibitem{Ma} Moan, Per Christian: \textit{On backward error analysis and Nekhoroshev stability in the numerical analysis of conservative systems of ODEs.} PhD thesis, University of Cambridge, 2002. \gbibitem{MN} Moan, Per Christian; Niesen, Jitse: Convergence of the Magnus series. \textit{Found. Comput. Math.} \textbf{8} (2008), 291--301. \gbibitem{MO} Moan, P. C.; Oteo, J. A.: Convergence of the exponential Lie series. \textit{J. Math. Phys.} \textbf{42} (2001), 501--508. \gbibitem{N} Naszódi, Márton: Flavors of translative coverings. In: \textit{New Trends in Intuitive Geometry.} Springer Verlag, New York, Berlin, Heidelberg, 2018. Pages 335--358. \gbibitem{R} Rogers, C. A.: A note on coverings.\textit{ Mathematika} \textbf{4} (1957), 1--6. \gbibitem{RZ} Rogers C. A.; Zong C.: Covering convex bodies by translates of convex bodies. \textit{Mathematika} \textbf{44} (1997), 215--218. \gbibitem{Scha} Sch\"affer, Juan Jorge: On Floquet's theorem in Hilbert spaces. \textit{Bull. Amer. Math. Soc.} \textbf{70} (1964), 243--245. \gbibitem{Sche} Scheffers, Georg: Zurückführung complexer Zahlensysteme auf typische Formen. \textit{Math. Ann.} \textbf{39} (1891), 293--390. \gbibitem{Stu} Study, E.: Über Systeme complexer Zahlen und ihre Anwendung in der Theorie der Transformationsgruppen. \textit{Monatsh. Math. Physik} \textbf{1} (1890), 283--354. \gbibitem{V} Vinokurov, V. A.: Logarithm of the solution of a linear differential equation, the Hausdorff formula and conservation laws. (In Russian) \textit{Dokl. Akad. Nauk SSSR} \textbf{319} (1991), 792--797; translation in \textit{Soviet Math. Dokl.} \textbf{44} (1992), 200--205. \gbibitem{W} Wei, James: Note on the global validity of the Baker-Hausdorff and Magnus theorems. \textit{J. Math. Phys.} \textbf{4} (1963), 1337--1341. \end{thebibliography} \end{document}
2412.07928v1
http://arxiv.org/abs/2412.07928v1
Renormalization for Bruin-Troubetzkoy ITMs
\documentclass[english]{amsart} \usepackage[T1]{fontenc} \usepackage[utf8]{inputenc} \usepackage{csquotes, xpatch} \usepackage{microtype} \usepackage{amsmath} \usepackage{amssymb} \usepackage{amsthm} \usepackage{mathrsfs} \usepackage{mathtools} \usepackage[all]{xy} \usepackage{tikz} \usepackage{caption} \usepackage{subfig} \usepackage{booktabs} \usepackage{graphicx} \usetikzlibrary{automata,positioning,calc,intersections,through,backgrounds,patterns,fit,external} \usepackage[style=alphabetic, maxnames=5, backend=biber,sorting=nyt]{biblatex} \addbibresource{biblio.bib} \renewbibmacro{in:}{\ifentrytype{article}{}{\printtext{\bibstring{in}\intitlepunct}}} \usepackage{url} \usepackage[colorlinks]{hyperref} \hypersetup{hidelinks} \usepackage{bookmark} \usepackage[nameinlink,capitalise,noabbrev]{cleveref} \theoremstyle{plain} \newtheorem{theorem}{Theorem} \newtheorem{lemma}[theorem]{Lemma} \newtheorem{proposition}[theorem]{Proposition} \newtheorem{corollary}[theorem]{Corollary} \newtheorem{conjecture}[theorem]{Conjecture} \theoremstyle{definition} \newtheorem{definition}[theorem]{Definition} \theoremstyle{remark} \newtheorem*{remark}{Remark} \newcommand{\numberset}{\mathbb} \newcommand{\CC}{\numberset{C}} \newcommand{\DD}{\numberset{D}} \newcommand{\HH}{\numberset{H}} \newcommand{\NN}{\numberset{N}} \newcommand{\PP}{\numberset{P}} \newcommand{\QQ}{\numberset{Q}} \newcommand{\RR}{\numberset{R}} \renewcommand{\SS}{\numberset{S}} \newcommand{\TT}{\numberset{T}} \newcommand{\ZZ}{\numberset{Z}} \newcommand{\cC}{\mathcal{C}} \newcommand{\cD}{\mathcal{D}} \newcommand{\cE}{\mathcal{E}} \newcommand{\cH}{\mathcal{H}} \newcommand{\cL}{\mathcal{L}} \renewcommand{\epsilon}{\varepsilon} \newcommand{\alphabet}{\mathbf{A}} \newcommand{\rauzy}{\mathcal{R}} \newcommand{\rauzygraph}{\mathcal{G}_{\rauzy}} \newcommand{\cone}{\RR_+^{\alphabet\cup\gap}} \newcommand{\gasket}{\mathbf{R}} \renewcommand{\sl}{\mathfrak{sl}} \newcommand{\bX}{\mathbf{X}} \newcommand{\sW}{\mathscr{W}} \newcommand*{\transpose}[2][-7mu]{\ensuremath{\mskip1mu\prescript{\smash{\top\mkern#1}}{}{\mathstrut#2}}}\providecommand{\norm}[2][]{#1\lVert#2#1\rVert} \providecommand{\abs}[2][]{#1\lvert#2#1\rvert} \DeclareMathOperator{\GL}{GL} \DeclareMathOperator{\SL}{SL} \DeclareMathOperator{\SO}{SO} \DeclareMathOperator{\Mat}{Mat} \DeclareMathOperator{\diam}{diam} \DeclareMathOperator{\area}{area} \begin{document} \title[Renormalization for Bruin-Troubetzkoy ITMs]{Renormalization for Bruin-Troubetzkoy ITMs} \date{\today} \author{Mauro Artigiani} \address{Universidad Nacional de Colombia - Bogotá} \email[M.~Artigiani]{[email protected]} \author{Pascal Hubert} \address{Aix-Marseille Universit\'e, CNRS\\ Institut de Math\'ematiques de Marseille, I2L, 13453, Marseille, France} \email[P.~Hubert]{[email protected]} \author{Alexandra Skripchenko} \address{Faculty of Mathematics\\ National Research University Higher School of Economics\\ Usacheva St. 6\\ 119048 Moscow, Russia} \email[A.~Skripchenko]{[email protected]} \subjclass[2020]{Primary: 37E05; Secondary: 37A05, 37A44, 11J70} \keywords{interval translation mappings, renormalization, multidimensional continued fraction algorithm, Pisot property, Hausdorff dimension} \begin{abstract} We study a class of interval translation mappings introduced by Bruin and Troubetzkoy, describing a new renormalization scheme, inspired by the classical Rauzy induction for this class. We construct a measure, invariant under the renormalization, supported on the parameters yielding infinite type interval translation mappings in this class. With respect to this measure, a.e.\ transformation is uniquely ergodic. We show that this set has Hausdorff dimension between $1.5$ and $2$, and that the Hausdorff dimension coincides with the affinity dimension. Finally, seeing our renormalization as a multidimensional continued fraction algorithm, we show that it has almost always the Pisot property. We discover an interesting phenomenon: the dynamics of this class of transformations is often (conjecturally: almost always) weak mixing, while the renormalizing algorithm typically has the Pisot property. \end{abstract} \maketitle \section{Introduction} This paper is focused on the ergodic properties of two classes of related dynamical systems. The first class we are interested in is a particular family of interval translation mappings (or ITMs, for short), the second one is a Markovian multidimensional continued fraction algorithm (MCF). ITMs were introduced in~\cite{BK} as a natural generalization of \emph{interval exchange transformations} (IETs). IETs and their ergodic properties were widely studied in the last decades, see, e.g.,~\cites{Viana:IETs, Yoccoz:IEMs} and the references therein. Typical IETs are known to be uniquely ergodic~\cites{Masur:ue, Veech:ue} and weakly mixing~\cite{AvilaForni}, while ergodic properties of certain special classes of IETs can be remarkably different (for example, Arnoux-Rauzy IETs are almost never minimal and those who are minimal are typically not weakly mixing~\cite{ACFH}). All these results were achieved by the study of the properties of the renormalization algorithm called \emph{Rauzy induction} (and variations of it). This algorithm can be seen as a representative of the class of Markovian multidimensional continued fraction algorithms. The key difference between IETs and ITMs is that the latter are not necessarily surjective: the images of the intervals do not need to form a partition, they simply form a collection of subintervals of the original interval, see~\cref{fig:BT_ITM} for an example. More formally, \begin{definition} An \emph{interval translation map} is a piecewise translation map $T$ defined on an open interval $I \subset \RR$ with values in $I$. We call $T$ a $n$-interval translation map (or $n$-ITM) if $I$ has $n$ maximal open sub-intervals to which the restriction of the $T$ is a translation. The endpoints of theses intervals are called \emph{singularities} of the map, and the endpoints of the image of the intervals are the images of the singularities. \end{definition} It was noticed already in~\cite{BK} that each ITM is either of \emph{finite} or \emph{infinite} type. This classification is based on the properties of the \emph{attractor} of the ITM. Namely, for a given mapping $T$ we consider the sequence $\Omega_n = I\cap TI\cap T^2I\cdots\cap T^{n}I$. If this sequence stabilizes for some $N\in\NN$, i.e., $\Omega_k = \Omega_{k+1}$ for all $k\ge N$, then the ITM $T$ is of \emph{finite type}. If there is no such $N$ and the limit set $\Omega =I\cap TI\cap T^2I\cdots$ is a Cantor set, then the ITM is of \emph{infinite type}, see also~\cite{ST}. Dynamics of ITMs of finite type basically coincides with the one of IETs. However, ITMs of infinite type are remarkably different. M.~Boshernitzan and I.~Kornfeld described the first example of ITM of infinite type. In the same paper, they formulated the following \begin{conjecture}\label{conjecture} The set of parameters that give rise to ITMs of infinite type has zero Lebesgue measure. \end{conjecture} To the best of our knowledge, this conjecture is currently completely open. The only known cases are for ITMs on $2$ and $3$ intervals, see~\cites{BK, Volk} respectively, and for a (very special) family of $n$-ITMs, which generalizes the one we study in this paper to an arbitrarily high number of intervals of continuity, see~\cite{Br}. Very recently, a \emph{topological} version of the conjecture has been proven in~\cite{DSvS}. In this paper, we focus on a special subclass of ITMs, which was defined by H.~Bruin and S.~Troubetzkoy in~\cite{BT} and which was, historically, the first concrete example of a family of ITMs. The class is described as follows: let $U:= \{(\alpha, \beta): 0\le\beta\le\alpha\le1 \} $ and $L:= \{(\alpha, \beta): 0\le\alpha\le\beta+1\le1 \} $ and $R:=U\cup L;$ for the internal point $(\alpha, \beta)\in U$ we define \begin{equation} T_{\alpha,\beta}(x) = \begin{cases} x+\alpha, & x\in[0,1-\alpha)\\ x+\beta, & x\in[1-\alpha, 1-\beta)\\ x+\beta-1, & x\in[ 1-\beta, 1), \end{cases} \end{equation} see~\cref{fig:BT_ITM} for an example. \begin{figure}[tb] \centering \includegraphics[width=.6\textwidth]{BT_ITM.pdf} \caption{An example of a Bruin-Troubetzkoy ITM. The intervals below are images of the ones above, color coded.} \label{fig:BT_ITM} \end{figure} The transformation $T(x) = T_{\alpha,\beta}(x)\colon [0,1)\to[0,1)$ is a $3$-ITM. By identifying the points $0$ and $1$ we get an interval translation map on a circle with two intervals. We remark that the original example of ITM in the paper by Boshernitzan and Kornfeld also belongs to this family. In their paper, Bruin and Troubetzkoy proved \cref{conjecture} for this special family of ITMs (see~\cite[Theorem~6]{BT}). They also showed that, considering the set of $3$-ITMs $T_{\alpha, \beta}$, the set $B$ of parameters that gives rise to uniquely ergodic ITMs of infinite type is a dense $G_\delta$ subset of the set $A$ of parameters that give rise to ITMs of infinite type (see~\cite[Corollary~13]{BT}). In this paper, we improve their result in the following way: \begin{theorem}\label{thm:uniquelyergodic} There exists a natural measure $\mu$, whose support set coincides with $A$, such that for $\mu$-almost all $(\alpha, \beta)\in A$, the transformation $T_{\alpha, \beta}$ is uniquely ergodic. \end{theorem} We remark that the result by Bruin and Troubetzkoy was later generalized by Bruin for a slightly more extended subclass of ITMs (see~\cite[Theorem~1]{Br}). Also, one can see Bruin-Troubetzkoy ITMs as a special type of double rotations, which were introduced in~\cite{SIA} and studied in~\cites{BC, AFHS}. Bruin and Troubetzkoy obtained their results by describing a special type of renormalization procedure (a Gauss-like map) for their class of ITMs. Using this procedure, they found a symbolic (more precisely, substitutional) presentation of the interval translation mappings they were interested in, and used it to prove their Theorem~6 and to construct the non-uniquely ergodic examples. Our strategy is quite different. In fact, we treat Bruin-Troubetzkoy family as a particular class of \emph{systems of isometries}. First, we introduce a new renormalization procedure that is based on the induction that I.~Dynnikov defined for systems of isometries (see \cite{D}). Our renormalization algorithm is a projectivization of the linear map defined by the induction procedure and can be seen as a Markovian multidimensional continued fraction (MCF) algorithm. Our \cref{thm:uniquelyergodic} is hence an immediate corollary of the general statement proved by C.~Fougeron in~\cite{Fougeron:Simplicial} for a broad class of MCF. Our approach also allows us to get another improvement for the result by Bruin and Troubetzkoy. Namely, we prove the following estimations on the Hausdorff dimension of the set $A$ mentioned above: \begin{theorem}\label{thm:hdimboundaries} Let $A$ be the set of parameters $(\alpha, \beta)$ yielding infinite type Bruin-Troubetzkoy ITMs. Then, its Hausdorff dimension can be bounded by \[ 1.5 \le \dim_H (A) < 2. \] Moreover, the Hausdorff dimension of the set $A$ is equal to its affinity dimension. \end{theorem} We refer to \cref{sec:HausdorffEstimate} for the definition of affinity dimension. In the previous result, the upper bound follows from the application of Fougeron's criterion, in a fashion similar to how we obtain \cref{thm:uniquelyergodic}. The lower bound is achieved by applying the strategy developed in~\cite{JiaoLiPanXu:Applications} for another fractal, of rather similar origin, called the Rauzy gasket. The Rauzy gasket was widely studied in the literature for several reasons, including symbolic dynamics, Arnoux-Rauzy interval exchange transformations, pseudogroups of rotations and $\mathbb{R}$-trees as well as Novikov's problem of asymptotic behavior of plane sections of triply periodic surfaces, see \cite{DHS} for more details and references. In fact, to prove that the Hausdorff dimension is equal to the affinity dimension, we follow a very recent paper by N.~Jurga who obtains a similar result for the Rauzy gasket~\cite{Jurga:Gasket}. \begin{remark} The upper bound can be slightly improved using the strategy developed by Policott and Sewell for the Rauzy gasket~\cite{PollicottSewell}, see~\cite{Zernikov:Diplom}. \end{remark} In view of~\cref{thm:hdimboundaries}, we call the set $A$ the \emph{Bruin-Troubetzkoy gasket}, see \cref{fig:BTGasket,fig:BTGasket_simplex} on \cpageref{fig:BTGasket_simplex}. The topological similarity between this fractal and the Rauzy gasket follows from the structure of the renormalization algorithm we construct, which is quite similar to well-known and well studied MCF algorithms, such as the Arnoux-Rauzy map, the Cassaigne algorithm and the Arnoux-Rauzy-Poincar\'e algorithms, see~\cite{CLL} and the references therein for the details. In order to reflect these features we give to our algorithm a special name, we call it the \emph{Arnoux-Rauzy-Cassaigne algorithm} (or ARC, for short). \begin{figure}[t] \centering \includegraphics[width=.9\textwidth]{BTgasket_square.pdf} \caption{The Bruin-Troubetzkoy gasket.} \label{fig:BTGasket} \end{figure} Using the MCF point of view, it is natural to compare the ergodic and spectral properties of our algorithm with the ones of the above mentioned algorithms, and the renormalization algorithm itself is the second dynamical system we are looking at in the current paper. Our main result about the ARC MCF algorithm is the following: \begin{theorem}\label{thm:pisot} The cocycle defined by the ARC algorithm has almost always \emph{Pisot} Lyapunov spectrum. \end{theorem} To obtain it, we follow the ideas from~\cite{CLL}. Contrary to this result, it is known that self-similar Bruin-Troubetzkoy ITMs of infinite type are very often weakly mixing \cite{Mercat, BruinRadinger}. Moreover, we believe that weak mixing for Bruin-Troubetzkoy ITMs is the typical behavior. In fact, we conjecture that: \begin{conjecture}\label{weakmixingconjecture} Almost all (with respect to the measure $\mu$ obtained in \cref{thm:uniquelyergodic}) Bruin-Troubetzkoy ITMs of infinite type are weakly mixing. \end{conjecture} Therefore, we have discovered an interesting phenomenon that did not appear before in the situations that can be used as a natural references in our context (generic IETs, Arnoux-Rauzy IETs, and so on): we have a dynamical system which is typically weakly mixing while the renormalization algorithm satisfies the Pisot condition. This seems to be an interesting phenomenon, which warrants further investigation. \subsection*{Organization of the paper} The paper is organized as follows: we start with the detailed description of the renormalization algorithm (see \cref{sec:induction}); in the same section we prove \cref{thm:uniquelyergodic} and the upper bound in \cref{thm:hdimboundaries}. \cref{sec:Pisot} is devoted to the proof of the \cref{thm:pisot}. Finally, the lower bound in the \cref{thm:hdimboundaries} is proved in the \cref{sec:HausdorffEstimate}. \subsection*{Acknowledgments} We would like to thank Sebastien Labb\'e for interesting conversations and some improvements for the first version of the paper; we are also grateful to Paul Mercat for fruitful discussions. We thank Juan Galvis, Yessica Trujillo and Juan Pablo Sierra for their help in preparing \cref{fig:BTGasket,fig:BTGasket_simplex}. \section{Renormalization}\label{sec:induction} In this section we first describe the induction procedure for Bruin-Troubetzkoy family of ITMs. Then, we apply it to get \cref{thm:uniquelyergodic}. \subsection{Notation} First, we change the notation in order to make the description of our family more homogeneous. Namely, we introduce new parameters: if $\alpha>\beta$, we have \[ \begin{split} a &= 1 - \alpha,\\ b &=\alpha -\beta,\\ c &= \beta. \end{split} \] Thus, $a + b + c = 1$ and \[ \begin{split} T([0,a)) &= [1-a,1).\\ T([a,a+b)) &= [1-b,1).\\ T([a+b,1)) &= [0, c). \end{split} \] We always assume that $a$, $b$, and $c$ are \emph{rationally independent}. Let us also enumerate the intervals of continuity of the map $T$ from the left to the right; thus, the first interval is the one of the length $a$, the second is the one of the length $b$ and the third is the one of the length $c$. The vector that codes the order in which the intervals appear at the preimage of $T$, is given by $(1,2,3).$ \subsection{The induction $\rauzy$} To define our induction, we distinguish three cases. \begin{figure}[t] \centering \subfloat[][Case 1.\label{fig:Case1}]{\includegraphics[width=.4\textwidth]{RauzyCase1.pdf}} \qquad\qquad \subfloat[][Case 3.\label{fig:Case3}]{\includegraphics[width=.4\textwidth]{RauzyCase3.pdf}} \caption{The two cases of the $\rauzy$ induction not (immediately) yielding a finite type ITM.} \label{fig:RauzyInfiniteCases} \end{figure} {\bf Case 1: $a>b+c$.}\label{case1} We consider the first return map on the subinterval $[b+c,1)$. It is an ITM in the same family with the following lengths of intervals: \[ \begin{split} a' &= a-b-c,\\ b' &= b,\\ c' &= c, \end{split} \] see \cref{fig:Case1}. We observe that the order of intervals does not change: the interval of length $a'$ is still the leftmost, the interval of length $b'$ is in the middle, while the interval of length $c'$ is the rightmost. So, the coding is again given by the vector $(1,2,3)$. {\bf Case 2: $c<a<b+c$.}\label{case2} One can check that in this case the ITM can be reduced to the ITM on 2 intervals and thus belongs to the finite case, see \cref{fig:Case2}. \begin{figure}[b] \centering \includegraphics[width=.6\textwidth]{RauzyCase2.pdf} \caption{The case of the $\rauzy$ induction inducing a finite type ITM.} \label{fig:Case2} \end{figure} {\bf Case 3: $a<c$.}\label{case3} We consider the first return map to the subinterval $[a,1)$. As result we get the following ITM: \[ \begin{split} T([a,a+b)) &= [1-b,1),\\ T([a+b,2a+b)) &= [a-1,1),\\ T([1-c+a,1)) &= [0,c-a) \end{split} \] Then, the lengths change in the following way: \[ \begin{split} a' &= a,\\ b' &= b,\\ c' &= c-a, \end{split} \] see \cref{fig:Case3}. However, this case is very different from the first case, since the \emph{order} of the intervals has changed: now the interval of length $b'$ is the leftmost one, while the interval of length $a'$ is in the middle (the third interval is always the rightmost). Note that the position of the intervals in the image does not change: the image of the rightmost interval is always in the left part and contains $0$, while the two other intervals are on the right and contain the rightmost point of the support interval. So, the coding is given by the vector $(2,1,3)$. It is easy to see that if we start with an ITM with the combinatorial coding given by the vector $(2,1,3)$ we get the symmetric picture. More precisely, if the interval labelled by $2$ is longer than half of the support interval, after the induction we still have the intervals in the order $(2,1,3)$. Whereas if the interval labelled by $3$ is longer than the rightmost interval (in our case it is interval labelled by $1$), then the resulting ITM is coded by $(1,2,3)$ again. Thus, the induction process can be seen as a Markovian multidimensional continued fraction algorithm that can be easily described in terms of simplicial systems (see~\cite{Fougeron:Simplicial}). The diagram associated with this system is presented in \cref{fig:Rauzy_graph}, where we ignore Case~2 since it corresponds to the finite case ITMs. The following lemma clearly holds: \begin{lemma}\label{infinitetype} The set $A$ of parameters that give rise to the infinite type Bruin-Troubetzkoy ITMs coincides with the set of parameters that do not enter the hole during the induction procedure. \end{lemma} We recall the terminology used for the classical Rauzy induction for IETs. In each iteration of the induction, the longest interval, i.e., the one that gets cut, will be called the \emph{winner} and the shortest one is called the \emph{loser}. As with IETs, we name the intervals using the corresponding letter. With this convention, in Case 1 the $a$-interval is the winner and the $c$-interval is the loser, whereas in Case 3 it is the opposite. We will sometimes simply say that the letter (and not the corresponding interval) is the winner or the loser. The following statement follows from \cref{infinitetype}: \begin{lemma}\label{winlose} A Bruin-Troubetzkoy ITM is of infinite type if and only if each letter wins and loses infinite number of times. \end{lemma} Thus, given that $\lambda = (a,b,c)$ is a vector of lengths of the intervals and $\lambda' = (a', b', c')$ is the vector of lengths of the intervals after the application of the induction, we have $\lambda = \mathcal{R} \lambda',$ with $\mathcal{R} =\mathcal{R}(k_1,k_2,k_3,\cdots) = A^{k_1}C_{A}B^{k_2}C_{B}A^{k_3}\cdots,$ where $k_1, k_2, \cdots \in\mathbb{N},$ \[ A = \begin{pmatrix} 1 & 1 & 1\\ 0 & 1& 0 \\ 0 & 0& 1 \end{pmatrix} \qquad \text{and} \qquad B = \begin{pmatrix} 1 & 0 & 0\\ 1 & 1& 1 \\ 0 & 0& 1 \end{pmatrix}, \] while \[ C_A = \begin{pmatrix} 1 & 0 & 0\\ 0 & 1& 0 \\ 1 & 0& 1 \end{pmatrix} \qquad \text{and} \qquad C_B = \begin{pmatrix} 1 & 0 & 0\\ 0 & 1& 0 \\ 0 & 1& 1 \end{pmatrix}. \] We remark that the matrices $A$ and $B$ of the induction coincide with the one defined by P. Arnoux and G. Rauzy in~\cite{AR}. \begin{figure}[bt] \begin{tikzpicture}[>=stealth] \node (left) [left=2cm] {$(1, 2, 3)$}; \node (right) [right=2cm] {$(2, 1, 3)$}; \draw[->,>=latex] (left) to[bend left] node[above] {$C_A$} (right); \draw[->,>=latex] (right) to[bend left] node[above] {$C_B$} (left); \draw[->,>=latex] (right) to [out=30, in=330, looseness=4] node[right] {$B$} (right); \draw[->,>=latex] (left) to [out=150, in=210, looseness=4] node[left] {$A$} (left); \end{tikzpicture} \caption{The Rauzy graph $\rauzygraph$ of the induction $\rauzy$.} \label{fig:Rauzy_graph} \end{figure} The graph if the induction $\rauzy$ is shown on \cref{fig:Rauzy_graph}. We stress that the coefficients $k_i$ can be equal to $0$. However, since applying the matrix $A$ implies cutting of $a$-intervals (and, similarly the matrix $B$ implies cutting the $b$-interval), an ITM is of infinite type if and only if we have infinitely often that even and odd $k_i$'s are \emph{strictly} positive. Now one can check that any $\rauzy$ that contains $A$ and $B$ in positive powers together with $C_A$ and $C_B$ has strictly positive entries. Therefore, the following lemma holds: \begin{lemma} There exists a special acceleration of the induction described above. \end{lemma} The definition of \emph{special acceleration} can be seen in~\cite[Remark~1]{FS}. Morally, it means a first return map to some subsimplex compactly contained in the parameter space. Exploiting the machinery of simplicial systems introduced in~\cite{Fougeron:Simplicial} and related results from~\cite{FS}, we easily obtain \cref{thm:uniquelyergodic}. \begin{proof}[Proof of \cref{thm:uniquelyergodic}] The simplicial system associate to $\rauzy$ is \emph{uniformly expanding} by~\cite[Proposition~4.1]{Fougeron:Simplicial} and therefore ergodic thanks to~\cite[Corollary~4.4]{Fougeron:Simplicial}. It is obvious that the simplicial system is \emph{quickly escaping} in the sense of \cite{Fougeron:Simplicial} and thus, by Theorem~1.1 in \cite{Fougeron:Simplicial}, we obtain the natural measure $\mu$ that induces the measure of maximal entropy on the natural suspension. Therefore the set of parameters which follow the same path (generic for $\mu$) is a single point, and so by the standard argument originated by Veech we conclude that the original ITM is uniquely ergodic. This completes the proof of \cref{thm:uniquelyergodic}. \end{proof} As a corollary of the previous result, we obtain an upper estimate on the Hausdorff dimension of the parameters yielding Bruin-Troubetzkoy ITMs of infinite type. \begin{corollary} The set $A$ of parameters that give rise to the infinite type Bruin-Troubetzkoy ITMs has Hausdorff dimension strictly smaller than $2$. \end{corollary} \begin{figure}[t] \centering \includegraphics[width=.9\textwidth]{BT_Gasket.pdf} \caption{The Bruin-Troubetzkoy gasket using the simplicial coordinates $(a, b, c)$.} \label{fig:BTGasket_simplex} \end{figure} We will obtain a lower bound, using thermodynamical formalism, in \cref{sec:HausdorffEstimate}. In \cref{fig:BTGasket_simplex}, we represent the Bruin-Troubetzkoy gasket using the parameters $(a, b, c)$ instead of $(\alpha, \beta)$ as in \cref{fig:BTGasket}. \subsection{Recovering Bruin and Troubetzkoy's Gauss map} We now show that our induction can be accelerated to recover the Gauss map of Bruin and Troubetzkoy. \begin{proposition}\label{prop:RauzyGauss} There exists an acceleration of $\rauzy$ such that, after rescaling the original interval, the induced transformation is the one obtained via the Gauss map of Bruin and Troubetzkoy. \end{proposition} \begin{proof} We begin by recalling the definition of the Gauss map. If $T_{\alpha, \beta}$ is a Bruin-Troubetzkoy ITM, then we define the ITM $T_{\alpha', \beta'}$ where \begin{equation}\label{eq:GaussMap} (\alpha', \beta') = \biggl(\frac{\beta}{\alpha}, \frac{\beta-1}{\alpha} + \biggl\lfloor \frac{1}{\alpha} \biggr\rfloor\biggr), \end{equation} where $\lfloor \cdot \rfloor$ is the (lower) integer part. Let us recall that we can recover the parameter $\alpha$ and $\beta$ from the length ones using that $\alpha = b + c$ and $\beta = c$, if the intervals are in the order $(1,2,3)$, and similarly, replacing $b$ by $a$ in the other case. We observe that, by definition of the $\rauzy$ induction, the first case is repeated $n$ times, with $n \ge 0$ given by \[ n = \biggl\lfloor \frac{a}{b+c} \biggr\rfloor = \biggl\lfloor \frac{1-\alpha}{\alpha} \biggr\rfloor = \biggl\lfloor \frac{1}{\alpha} \biggr\rfloor - 1. \] Then, we are in the third case, and we change the order of the intervals. After the above steps, the three intervals of continuity are of lengths \[ \begin{split} a' &= a - n(b+c),\\ b' &= b,\\ c' &= c - (a- n(b+c)). \end{split} \] The total length is $a'+b'+c' = b+c = \alpha$. So, if we renormalize by dividing the interval by $\alpha$, rescaling it to length $1$, we see that \[ \alpha' = \frac{a'+c'}{\alpha} = \frac{c}{\alpha} = \frac{\beta}{\alpha} \] Moreover, since \[ c' = \beta - 1 +\alpha + \biggl( \biggl\lfloor \frac{1}{\alpha} \biggr\rfloor - 1\biggr) \alpha = \beta - 1 + \biggl\lfloor \frac{1}{\alpha} \biggr\rfloor \alpha, \] we have that \[ \beta' = \frac{c'}{\alpha} = \frac{\beta - 1}{\alpha} + \biggl\lfloor \frac{1}{\alpha} \biggr\rfloor. \] The above formulas agree with~\eqref{eq:GaussMap} and so we are done. \end{proof} \section{Pisot property for the ARC algorithm}\label{sec:Pisot} \subsection{The ARC multidimensional continued fraction algorithm}\label{sec:ARC_MCF} The induction $\rauzy$ introduced in the previous section defines a multidimensional continued fraction algorithm (or MCF algorithm, for short), which we call the \emph{Arnoux-Rauzy-Cassaigne} (or ARC) MCF algorithm, as we will now explain. We can naturally act by the matrices of the $\rauzy$ induction to the standard $2$-dimensional simplex $\Delta = \Delta^2 = \{(x,y,z): x,y,z\ge 0, x+y+z=1\}$. In formulas, we have: \begin{equation}\label{eq:ARC_IFS} \begin{split} f_A (x,y,z) &= \biggl(\frac{1}{2-x}, \frac{y}{2-y}, \frac{z}{2-z}\biggr),\\ f_B (x,y,z) &= \biggl(\frac{x}{2-x}, \frac{1}{2-y}, \frac{z}{2-z}\biggr),\\ f_{C_A} (x,y,z) &= \biggl(\frac{1-y}{2-x-y}, \frac{y}{2-x-y}, \frac{z}{2-x-y}\biggr),\\ f_{C_B} (x,y,z) &= \biggl(\frac{x}{2-x-y}, \frac{1-x}{2-x-y}, \frac{z}{2-x-y}\biggr). \end{split} \end{equation} We will now show that the cocycle defined by the ARC MCF algorithm has negative second Lyapunov exponent (see below for the relevant definitions), following~\cite{CLL}; in the terminology of~\cite{Lagarias}, the MCF algorithm is \emph{strongly convergent}. This will imply that it satisfies the Pisot condition from~\cite{BST}. We remark that the measure $\mu$ obtained in the proof of \cref{thm:uniquelyergodic} induces a measure, which, slightly abusing the notation, we still denote $\mu$, on set of walks along the graph $\rauzygraph$. We will call $1$ the state corresponding to the permutation $(1, 2, 3)$ and $2$ the permutation $(2, 1, 3)$. Thus, for instance, the path $112$ corresponds to the application of the matrices $AC_A$ of the induction. As usual, let $[a_1, a_2, \dotsc, a_n]$ be the cylinder formed by all the words in $\{1, 2\}^\NN$ that begin with the letters $a_1a_2\cdots a_n$. We now recall the definition of Lyapunov exponents in the present context (along with the relevant notation). Given an infinite word $w\in\{1,2\}^\NN$, we consider the product of matrices corresponding to the path on the graph $\rauzygraph$ described by $w$: \[ X_n(w) = X_{w_0w_1} X_{w_1w_2} \cdots X_{w_{n-2}w_{n-1}}, \] for $n\ge 1$. This forms a cocycle, as \[ X_{m+n}(w) = X_m(w) X_n(\sigma^m w), \] where $\sigma\colon\{1,2\}^\NN\to\{1,2\}^\NN$ is the shift map. Since all the matrices of the $\rauzy$ induction are invertible, this cocycle is log-integrable with respect to the measure $\mu$ constructed in \cref{thm:uniquelyergodic}: \[ \int_{\{1,2\}^\NN} \log \max\bigl\{\|X_1(w)\|, \|X_1(w)^{-1}\|\bigr\} \, d\mu(w) < \infty. \] Hence there are ($\mu$-almost everywhere) well defined Lyapunov exponents: \[ \lambda_1 = \lim_{n\to\infty} \frac{\log{\|X_n(w)\|}}{n}, \qquad \lambda_1 + \lambda_2 = \lim_{n\to\infty} \frac{\log{\|\wedge^2 X_n(w)\|}}{n}, \qquad \lambda_3 = -(\lambda_1 + \lambda_2) \] where $\wedge$ is, as usual, the exterior product of matrices, and the last equality follows from the fact that the matrices of the induction have determinant $1$. Moreover, since, by unique ergodicity, the nested cones $X_1 X_2 \cdots X_n = X_{[1,n)}\RR^3_{\ge 0}$, where we have suppressed the dependence on $w$, converge for $\mu$-almost all words $w$, to a line $\RR_{0} f$, for some $f\RR^3$, we have the following characterization of the second Lyapunov exponent will be useful later on: \[ \lambda_2 = \lim_{n\to\infty} \frac{\log{\Bigl\|\transpose{X_n(w)}|_{f^\perp}\Bigr\|}}{n}, \] where $\transpose{X}$ denotes the transpose matrix. \subsection{Some technical lemmas} We begin with a general lemma about matrices and norms, whose proof can be found in~\cite[Lemma~3.5]{CLL}. \begin{lemma}\label{lemma:submultiplicative} Let $X$ and $Y$ non negative $d\times d$ real matrices, such that $XY\neq 0$. Let $\|\cdot\|$ denote any seminorm on $\RR^s$ which is a norm on every $f^\perp$, with $f\in (X\RR^d_{\ge 0} \cup Y\RR^d_{\ge 0}) \setminus \{0\}$. We have \[ \Bigl\| \transpose{XY} \Bigr\|^{XY\RR^d_{\ge 0}} \le \Bigl\| \transpose{X} \Bigr\|^{X\RR^d_{\ge 0}} \Bigl\| \transpose{Y} \Bigr\|^{Y\RR^d_{\ge 0}}. \] \end{lemma} From the definitions, we obtain: \begin{lemma}\label{lemma:products_of_M} Let $(M_n)_{n \in \NN} \in \{A, B, C_A, C_B\}^\NN$ be a sequence of matrices. If $(M_n)$ contains infinitely many occurrences of $A$, $B$, $C_A$ and $C_B$, then there exists an increasing sequence of integers $(n_m)_{m \in \NN}$, such that $n_0 = 0$ and \[ M_{[n_m, n_{m+1})} \in \{A^k C_A, B^k C_B: k\in\NN\}. \] \end{lemma} Following~\cite{CLL}, we introduce the seminorm $\|\cdot\|_D$ on $\RR^3$ given by \[ \|v\|_D = \max v -\min v = \max_{i=1,2,3} v_i - \min_{i=1,2,3} v_i. \] We remark that the seminorm is invariant under addition of constant vectors. Moreover, for any $f\in\RR^3_{\ge 0}$, the restriction of the seminorm to $f^\perp$ is a genuine norm, see~\cite[Lemma~3.7]{CLL}. The same result implies that, when restricted to $f^\perp$, for a given $f\in\RR^3_{\ge 0}$, we obtain a norm on $3\times 3$ matrices, which is invariant under addition of constant vectors, and comparable with the infinity norm. More precisely, in~\cite[Lemma~3.7]{CLL} it is proven that: \begin{equation}\label{eq:Dnorm_vs_infinitynorm} \frac{1}{2}\bigl\| M|_{f^\perp} \bigr\|_D \le \bigl\| M|_{f^\perp} \bigr\|_\infty \le 2 \, \bigl\| M|_{f^\perp} \bigr\|_D \end{equation} With this notation, we have \begin{lemma}[\protect{\cite[Lemma~3.8]{CLL}}]\label{lemma:hyperplanes} Let $M$ be a $3\times 3$ positive and invertible real matrix. Consider the set $\cH$ of hyperplanes orthogonal to some vector in \[ S = (M\cE \cup (\cE - \cE) \cup M(\cE - \cE)) \setminus \{0\}, \] where $\cE = \{e_1, e_2, e_3\}$ and $\cD$ be the finite union of one-dimensional intersections of two hyperplanes of $\cH$: \[ \cD = \bigcup_{h_1, h_2 \in \cH} h_1 \cap h_2. \] Then, the maximal value of the norm $\|\cdot\|_D$, for all restrictions orthogonal to a positive vector in the cone $M\RR^3_{\ge 0}$, is attained at some vector $v$ in $\Bigl(\cD \setminus \pm \transpose{M^{-1}} \RR^3_{>0}\Bigr) \setminus \{0\}$, i.e., \[ \Bigl\|\transpose{M}\Bigr\|_D^{M\RR^3_{\ge 0}} := \sup_{f\in M\RR^3_{\ge 0} \setminus \{0\}} \Bigl\|\transpose{M}|_{f^\perp}\Bigr\|_D = \max_{v\in \bigl(\cD \setminus \pm \transpose{M^{-1}} \RR^3_{>0}\bigr) \setminus \{0\}} \frac{\Bigl\|\transpose{M}v\Bigr\|_D}{\|v\|_D}. \] \end{lemma} We will use the next lemma to control the norm introduced above for specific products of the matrices of our MCF. \begin{lemma}\label{lemma:norm_one} For every $k\ge 0$ we have that \[ \Bigl\|\transpose{(A^kC_A)}\Bigr\|_D^{\RR_{\ge 0}^3} = 1, \qquad \Bigl\|\transpose{(B^kC_B)}\Bigr\|_D^{\RR_{\ge 0}^3} = 1. \] \end{lemma} \begin{proof} We will prove the lemma only for $A^kC_A$, the other case being similar. Choose a vector $f \in \RR^3_{\ge 0} \setminus \{0\}$ and let $v = (v_1, v_2, v_3) \in f^\perp$. Using the invariance of the norm $\|\cdot\|_D$ under addition by constant vectors, we obtain, for every $k\ge 0$ \[ \Bigl\|\transpose{(A^kC_A)}\Bigr\|_D^{\RR_{\ge 0}^3} = \left\| \begin{pmatrix} k+1 & 0 & 1 \\ k & 1 & 0 \\ k & 0 & 1 \end{pmatrix} \begin{pmatrix} v_1 \\ v_2 \\ v_3 \end{pmatrix} \right\|_D = \left\| \begin{pmatrix} 1 & 0 & 0 \\ 0 & 1 & -1 \\ 0 & 0 & 0 \end{pmatrix} \begin{pmatrix} v_1 \\ v_2 \\ v_3 \end{pmatrix} \right\|_D. \] Let $v' = (v_1, v_2 - v_3, 0)$, then \[ \min v \le \min v' \le 0 \le \max v' \le \max v. \] We remark that, since we work in $3$ dimensions, we have that $\|(v_1, v_2, v_3)\|_D = \max\{|v_1 - v_2|, |v_1 - v_3|, |v_2 - v_3|\}$, so the previous equation implies that $\|v'\|_D \le \|v\|_D$. Hence \[ \frac{\Bigl\|\transpose{(A^k C_A)}v\Bigr\|_D}{\|v\|_D} \le 1, \] for every $v\in f^\perp$ and $k\ge 0$. To obtain the equality, we observe that, since $f$ is non zero and non negative, we can take a vector $v = (a, -b, 0)$, for $a$, $b>0$ inside $f^\perp$. For this vector, the direct computation yields \[ \Bigl\|\transpose{(A^k C_A)}(a, -b, 0)\Bigr\|_D = \|(a, -b, 0)\|_D, \] as we wanted. \end{proof} We need to take care of a ``base case'' before we can do the general one. \begin{lemma}\label{lemma:AC_ABC_B} Let $M = AC_A BC_B$ or $M = BC_B AC_A$. Then \[ \Bigl\| \transpose{M} \Bigr\|_D^{M\RR^3_{\ge 0}} \le \frac{4}{5}. \] \end{lemma} \begin{proof} We will prove the result in the case $M = AC_A BC_B$, the other case is symmetric. By direct computation: \[ M = AC_ABC_B = \begin{pmatrix} 3 & 3 & 2 \\ 1 & 2 & 1 \\ 1 & 1 & 1 \end{pmatrix}, \qquad M^{-1} = \begin{pmatrix} 1 & -1 & -1 \\ 0 & 1 & -1 \\ -1 & 0 & 3 \end{pmatrix}. \] Given $z = (a, b, c)$ we can compute \[ \Bigl\|\transpose{M}z\Bigr\|_D = \Bigl\|\transpose{(3a+b+c, 3a+2b+c, 2a+b+c)}\Bigr\|_D = \Bigl\|\transpose{(0, b, a+b+c)}\Bigr\|_D. \] We now construct the set $\cH$ as in \cref{lemma:hyperplanes}. By a direct computation: \begin{align*} Me_1 &= (3, 1, 1), & e_1 - e_3 &= (1, 0, -1), & M(e_1 - e_3) &= (1, 0, 0), \\ Me_2 &= (3, 2, 1), & e_1 - e_2 &= (1, -1, 0), & M(e_1 - e_2) &= (0, -1, 0), \\ Me_3 &= (2, 1, 1), & e_2 - e_3 &= (0, 1, -1), & M(e_2 - e_3) &= (1, 1, 0). \end{align*} Hence, $\cH$ is made of nine hyperplanes. Again by \cref{lemma:hyperplanes}, we need to consider vectors $z \in \Bigl(\cD \setminus \pm \transpose{M^{-1}} \RR^3_{>0}\Bigr) \setminus \{0\}$. The relevant computations are in \cref{table:CLL}, from which the result follows. \end{proof} \begin{table}[tp] \[ \begin{array}{ccccccc} \toprule u & v & z = u \wedge v & \transpose{M}z &\|z\|_D & \|\transpose{M}z\|_D \\ \midrule Me_1 & Me_2 & (-1, 0, 3) & (0, 0, 1) & 4 & 1 \\ Me_1 & Me_3 & (0, -1, -1) & (0, -1, 0) & 2 & 1 \\ Me_1 & e_1 - e_3 & (-1, 4, -1) & (0, 4, 1) & 5 & 4 \\ Me_1 & e_1 - e_2 & (1, 1, -4) & (0, 1, -1) & 5 & 2 \\ Me_1 & e_2 - e_3 & (-2, 3, 3) & (0, 3, 2) & 5 & 3 \\ Me_1 & M(e_1 - e_2) & (1, 0, -3) & (0, 0, -1) & 4 & 1 \\ Me_1 & M(e_2 - e_3) & (-1, 1, 2) & (0, 1, 1) & 3 & 1 \\ Me_1 & M(e_1 - e_3) & (0, 1, -1) & (0, 1, 0) & 2 & 1 \\ Me_2 & Me_3 & (1, -1, -1) & (1, 0, 0) & 2 & 1 \\ Me_2 & e_1 - e_3 & (-2, 4, -2) & (-4, 0, 2) & 6 & 4 \\ Me_2 & e_1 - e_2 & (1, 1, -5) & (-1, 0, -2) & 6 & 2 \\ Me_2 & e_2 - e_3 & (-3, 3, 3) & (-3, 0, 0) & 6 & 3 \\ Me_2 & M(e_1 - e_2) & (1, 0, -3) & (0, 0, -1) & 4 & 1 \\ Me_2 & M(e_2 - e_3) & (-1, 1, 1) & (-1, 0, 0) & 2 & 1 \\ Me_2 & M(e_1 - e_3) & (0, 1, -2) & (-1, 0, -1) & 3 & 1 \\ Me_3 & e_1 - e_3 & (-1, 3, -1) & (-1, 2, 0) & 4 & 3 \\ Me_3 & e_1 - e_2 & (1, 1, -3) & (1, 2, 0) & 4 & 2 \\ Me_3 & e_2 - e_3 & (-2, 2, 2) & (-2, 0, 0) & 4 & 2 \\ Me_3 & M(e_1 - e_2) & (1, 0, -2) & (1, 1, 0) & 3 & 1 \\ Me_3 & M(e_2 - e_3) & (-1, 1, 1) & (-1, 0, 0) & 2 & 1 \\ Me_3 & M(e_1 - e_3) & (0, 1, -1) & (0, 1, 0) & 2 & 1 \\ \bottomrule \end{array} \] \caption{The computations involved in \cref{lemma:AC_ABC_B}. We only wrote the values of $u$ and $v$ which yield a $z = u \wedge v$ in $\RR^3\setminus\transpose{M}^{-1}\RR^3_{>0}$.}\label{table:CLL} \end{table} The main technical result is the following \begin{lemma}\label{lemma:measure_cylinder} Let $\mu$ the measure on $\{1,2\}^\NN$ obtained in \cref{thm:uniquelyergodic}. For every $\epsilon > 0$, there exists an $N$ such that, for every $n > N$ and $\mu$-almost every sequences $(M_n)_{n \in \NN} \in \{A, B, C_A, C_B\}^\NN$, we have \[ \Bigl\| \transpose{M}_{[0,n)}|_{f^\perp}\Bigr\|_\infty \le (n+1) \biggl(\frac{4}{5}\biggr)^{\frac{1}{8}n(\mu([112211221])-\epsilon)-\frac{1}{8}}, \] where $\bigcap_{n\in\NN} M_{[0,n)}\RR^3_{\ge 0} = \RR_{\ge 0} f$. \end{lemma} \begin{proof} We begin by recalling that, by its construction, the measure $\mu$ assigns positive measure to every cylinder. In particular, $\mu([1])$, $\mu([2])>0$. By ergodicity of $\mu$, $\mu$-almost every sequence of matrices $(M_n)_{n\in\NN}$ contains infinitely often each one of the matrices $A$, $B$, $C_A$ and $C_B$. Then, by \cref{lemma:products_of_M}, there is an increasing sequence $(n_i)_{i\in\NN}$ such that $n_0 = 0$ and \[ N_i = M_{[n_i, n_{i+1})} \in \{A^k C_A, B^k C_B: k\in\NN\} \] for all $i$. For all positive $n$, there exists a unique $m\in\NN$ such that $n_m \le n-1 < n_{m+1}$. Let $g = M_{[0,n)}^{-1} f$, then by \cref{lemma:submultiplicative}, we obtain \[ \begin{split} \Bigl\| \transpose{M}_{[0,n)}|_{f^\perp}\Bigr\|_\infty &\le \Bigl\| \transpose{M}_{[0,n)}\Bigr\|_\infty^{M_{[0,n)} \RR^3_{\ge 0}} \\ &\le \Bigl\| \transpose{M}_{[n_m,n)}\Bigr\|_\infty^{M_{[n_m, n)} \RR^3_{\ge 0}} \cdot \Bigl\| \transpose{M}_{[0,n_m)}\Bigr\|_\infty^{M_{[0, n_m)} \RR^3_{\ge 0}}\\ &\le \Bigl\| \transpose{M}_{[n_m,n)}\Bigr\|_\infty \cdot \Bigl\| \transpose{M}_{[0,n_m)}\Bigr\|_\infty^{M_{[0, n_m)} \RR^3_{\ge 0}} \end{split} \] Since $M_{[n_m,n)}$ is of the form \[ A^k = \begin{pmatrix} 1 & k & k \\ 0 & 1 & 0 \\ 0 & 0 & 1 \end{pmatrix}, \qquad B^k = \begin{pmatrix} 1 & 0 & 0 \\ k & 1 & k \\ 0 & 0 & 1 \end{pmatrix}, \] for some $k\in\NN$, and $\Bigl\| \transpose{A^k}\Bigr\|_\infty = \Bigl\| \transpose{B^k}\Bigr\|_\infty = k+1$, we have \[ \Bigl\| \transpose{M}_{[n_m,n)}|_{f^\perp}\Bigr\|_\infty \le n - n_m + 1 \le n + 1. \] Now, we deal with the second term: $\Bigl\| \transpose{M}_{[0,n_m)}\Bigr\|_\infty^{M_{[0, n_m)} \RR^3_{\ge 0}}$, with $M_{[0,n_m)} = \prod_{i=0}^{m-1} N_i = N_{[0,m)}$. Let $J_m$ be the set of indices $j\in \{0, 1, \dotsc, n_m - 8\}$ such that $M_{[j,j+8)} =(A C_A B C_B)^2$. Call $J'_m \subseteq J_m$ a subset of maximal cardinality such that \begin{equation}\label{eq:estimate_on_Jm} \min ((J'_m - J'_m) \cap \NN_{>0}) \ge 8. \end{equation} We remark that $\# J'_m \ge \frac{1}{8} \# J_m$. Now, if $j\in J'_m$ there exists a unique $i = i(j)\in\NN$ such that $n_{i} \in\{j, j+1, j+2\}$ and $N_i N_{i+1} \in \{AC_A, BC_B\}$. In particular, if $j, j' \in J'_m$ with $j\neq j'$, then $|i(j') - i(j)| \ge 2$, thanks to~\eqref{eq:estimate_on_Jm}. Denote $I_m = \{i(j), j\in J'_m\}$, so $\# I_m = \# J'_m$. Using \cref{lemma:submultiplicative} recursively together with~\eqref{eq:Dnorm_vs_infinitynorm}, \cref{lemma:norm_one} and \cref{lemma:AC_ABC_B} we obtain \[ \begin{split} \Bigl\|\transpose{M}_{[0,n_m)}\Bigr\|_\infty^{M_{[0, n_m)} \RR^3_{\ge 0}} &= \Bigl\|\transpose{N}_{[0,m)}\Bigr\|_\infty^{N_{[0, m)} \RR^3_{\ge 0}} \le 2 \Bigl\|\transpose{N}_{[0,m)}\Bigr\|_D^{N_{[0, m)} \RR^3_{\ge 0}} \\ &\le 2 \prod_{i\in I_m} \Bigl\|\transpose{(N_i N_{i+1})}\Bigr\|_D^{N_i N_{i+1} \RR^3_{\ge 0}} \cdot \prod_{\substack{i\in{0,1,\dotsc, m-1} \\ i\notin I_m, i\notin I_m+1}} \Bigl\|\transpose{N_i}\Bigr\|_D^{N_i \RR^3_{\ge 0}}\\ &\le 2 \biggl(\frac{4}{5}\biggr)^{\# I_m}. \end{split} \] We can now conclude the proof. Using Birkhoff ergodic theorem, for $\mu$-almost every $x\in\{1,2\}^\NN$, we have \[ \lim_{n\to\infty} \frac{1}{n} \sum_{k=0}^{n-8} \chi_{[112211221]} \circ S^k (x) = \lim_{n\to\infty} \frac{1}{n} \sum_{k=0}^{n-1} \chi_{[112211221]} \circ S^k (x) = \mu([112211221]). \] Hence, for $\mu$-almost every $x\in\{1,2\}^\NN$, and for all $\epsilon>0$, there exists an $N$ such that, if $n>N$, then \[ \Biggl| \frac{1}{n} \sum_{k=0}^{n-1} \chi_{[112211221]} \circ S^k (x) - \mu([112211221])\Biggr| < \epsilon, \] which implies that \[ \begin{split} \# J_m &= \sum_{k=0}^{n_m-8} \chi_{[112211221]} \circ S^k (x) \\ &\ge \sum_{k=0}^{n-8} \chi_{[112211221]} \circ S^k (x) -1 \\ &\ge n(\mu([112211221]) -\epsilon) - 1, \end{split} \] and the proof is complete (we used the definition of the cylinder $[112211221]$ and the coefficients $n_m$ for the first ineuquality). \end{proof} The previous work allows us to prove the negativity of the second Lyapunov exponent for the MCF algorithm. \begin{theorem}\label{thm:no_weak_mixing} The MCF algorithm defined by the renormalization algorithm has $\mu$-almost everywhere \emph{negative} second Lyapunov exponent. \end{theorem} \begin{proof} From the above discussion, using \cref{lemma:measure_cylinder}, for $\mu$-almost every word $w$ and every $\epsilon>0$, we have that \[ \begin{split} \lambda_2 &= \lim_{n\to\infty} \frac{\log{\Bigl\|\transpose{X_n(w)}|_{f^\perp}\Bigr\|}}{n} \\ &\le \lim_{n\to\infty} \frac{\log{ \Bigl( (n+1) \bigl(\frac{4}{5}\bigr)^{\frac{1}{8}n(\mu([112211221])-\epsilon)-\frac{1}{8}}\Bigr)}}{n} \\ &= \lim_{n\to\infty} \frac{\log{(n+1)} + \bigl(\frac{1}{8}n(\mu([112211221])-\epsilon)-\frac{1}{8}\bigr)\log{\bigl(\frac{4}{5}\bigr)}}{n} \\ &< \frac{1}{8}(\mu([112211221])-\epsilon)\log{\biggl(\frac{4}{5}\biggr)}\\ &< 0, \end{split} \] which proves the statement. \end{proof} \section{Hausdorff dimension estimates}\label{sec:HausdorffEstimate} In this section, we show that the Hausdorff dimension of the gasket defined by our MCF algorithm is the same as its affinity dimension, and prove estimates for it. Our approach follows the very recent papers~\cite{Jurga:Gasket, JiaoLiPanXu:Applications}. \subsection{Thermodynamic formalism}\label{sec:thermodynamic_formalism} In this subsection we recall some general definitions and results from~\cite{Jurga:Gasket} that we will use. Let $\bX$ be a set of matrices in $\SL(3,\RR)$. We say that $\bX$ is \emph{irreducible} if no proper linear subspace of $\RR^3$ is preserved by all the matrices in $\bX$. Given a matrix $X\in\SL(3,\RR)$, let $\alpha_1(X) \ge \alpha_2(X) \ge \alpha_3(X)$ be its singular values. Then, for $s\ge 0$, the singular value function $\phi^s\colon\SL(3,\RR)\to\RR_+$ is defined as \begin{equation}\label{eq:singular_value_f} \phi^s = \begin{cases} \Bigl(\frac{\alpha_2(X)}{\alpha_1(X)}\Bigr)^s, & \text{if $0\le s \le 1$},\\ \frac{\alpha_2(X)}{\alpha_1(X)}\Bigl(\frac{\alpha_3(X)}{\alpha_1(X)}\Bigr)^{s-1}, & \text{if $1\le s \le 2$},\\ \Bigl(\frac{\alpha_2(X)\alpha_3(X)}{\alpha_1^2(X)}\Bigr)^{s-1}, & \text{if $s \ge 2$}. \end{cases} \end{equation} The irreducibility of $\bX$ implies that the function $\phi$ is quasimultiplicative on $\bX$, see~\cite[Section~2.2]{Jurga:Gasket} for details. We now define the zeta function $\zeta_\bX\colon [0,\infty)\to[0,\infty]$ by \[ \zeta_\bX (s) = \sum_{n=1}^{\infty} \sum_{X\in\bX^n} \phi^s(X). \] Finally, the \emph{affinity dimension} $s_\bX$ is the critical exponent of the above series: \begin{equation}\label{eq:affinity_dim} s_\bX = \inf \{s\ge 0: \zeta_\bX(s)<\infty\}. \end{equation} We will use the following result, which uses the strong open set condition (SOSC), see, e.g.,~\cite[Definition~2.5]{Jurga:Gasket}. \begin{theorem}[\protect{\cite[Theorem~1.3]{Jurga:Gasket}}]\label{thm:Jurga} Suppose a finite set $\bX$ of positive matrices in $\SL(3,\RR)$ generates a semigroup $S_\bX$ which is Zariski dense in $\SL(3,\RR)$ and satisfies the SOSC. Then $\dim_H K_\bX = \min \{s_\bX, 2\}$. \end{theorem} We say that a set of positive matrices $\bX = \{X_1, \dotsc, X_n\}$ is \emph{balanced} if there exists a $c>0$ such that, for all $i=1,\dotsc,n$, \[ \frac{\min(X_i)_{j,k}}{\max (X_i)_{j,k}} \ge c. \] This implies that the singular value function $\phi$ is almost-submultiplicative on the set $\bX$, see~\cite[Proposition~2.1]{Jurga:Gasket}. Finally, we recall the definition of the pressure $P_\bX\colon[0,\infty)\to\RR$, given by \[ P_\bX(s) = \lim_{n\to\infty} \frac{1}{n} \log {\Biggl(\sum_{X\in\bX^n} \phi^s(X)\Biggr)}, \] where the limit exists since the function $\phi^s$ is almost-submultiplicative. One can see that $P_\bX$ is a continuous, strictly decreasing, convex function and that its unique root is exactly the affinity dimension $s_\bX$. \subsection{The Bruin-Troubetzkoy gasket} The graph $\rauzygraph$ in \cref{fig:Rauzy_graph} can be encoded, using the alphabet $\alphabet = \{A, C_A, B, C_B\}$ by the matrix \[ T = (t_{ij}) = \begin{pmatrix} 1 & 1 & 0 & 0 \\ 0 & 0 & 1 & 1 \\ 0 & 0 & 1 & 1 \\ 1 & 1 & 0 & 0 \end{pmatrix}. \] Considering the paths on the graph $\rauzygraph$ we define a topological Markov shift, which naturally we call the ARC topological Markov shift. Then, the set of (one-sided) infinite paths, starting from the vertex $1 = (1, 2, 3)$, on the graph corresponds to the set \[ \sW = \{w = (w_i)_{i=1}^\infty: w_1 = A, C_A, w_i \in\alphabet, t_{w_i w_{i+1}} = 1, \text{ for all } i\ge 1\}. \] A word of length $n$, $w = w_1 w_2 \cdots w_n$, is \emph{admissible} if $w_1 = A, C_A$ and \[ t_{w_1w_2} t_{w_2w_3} \cdots t_{w_{n-1}w_n} = 1. \] The set of admissible words of length $n$, which corresponds to the set of $n$ length path on the graph $\rauzygraph$, is denoted $\sW_n$. We recall that in \cref{sec:ARC_MCF} we defined the Arnoux-Rauzy-Cassaigne MCF algorithm. Bearing in mind this algorithm, the Bruin-Troubetzkoy gasket $\gasket$ is \[ \gasket = \bigcup_{w\in\sW} \bigcap_{n=1}^\infty f_{w_1} \circ \cdots \circ f_{w_n} (\Delta). \] In other words, it is the \emph{attractor} of the iterated function system driven by the paths on the graph $\rauzygraph$, using the transformations in~\eqref{eq:ARC_IFS}. In the following, we will identity the alphabet $\alphabet$ with the set of matrices bearing the same names. It is easy to see that the set of matrices $\alphabet$ is irreducible. Then, we can use the results of the previous section. \subsection{Equality between Hausdorff dimension and affinity dimension} The main result of this section is the following \begin{theorem}\label{thm:dimH_eq_affdim} The Hausdorff dimension of the Bruin-Troubetzkoy gasket $\gasket$ is equal to its affinity dimension. That is \[ \dim_H \gasket = s_{\alphabet} = \inf \Biggl\{ s\ge 0: \sum_{n=1}^{\infty} \sum_{w\in\sW_n} \frac{\alpha_2(w)}{\alpha_1(w)} \left(\frac{\alpha_3(w)}{\alpha_1(w)}\right)^{s-1}< \infty \Biggr\}. \] \end{theorem} We will closely follow the strategy used for the analogous result for the Rauzy gasket in the paper~\cite{Jurga:Gasket}, see her Theorem~1.1. Let \[ \Gamma = \{A^n C_A C_B, C_A B^n C_B, (C_A C_B)^n A\}_{n \ge 1} \subset \SL(3,\RR) \] and $\Gamma_N$ the $N$-th truncation of the set: \[ \Gamma_N = \{A^n C_A C_B, C_A B^n C_B, (C_A C_B)^n A\}_{1 \le n \le N}. \] We will denote by $S_\Gamma$ and $S_{\Gamma_N}$ the semigroups generated by $\Gamma$ and $\Gamma_N$ respectively. The results recalled in \cref{sec:thermodynamic_formalism}, applied to these semigroups allow us to define their affinity dimension as in~\eqref{eq:affinity_dim}. We will denote by $s_\Gamma$ and $s_{\Gamma_N}$ the affinity dimensions of the semigroups $\Gamma$ and $\Gamma_N$ respectively. \begin{lemma}\label{lemma:excluding_constants} If $K_\Gamma$ denotes the projective limit set of $S_\Gamma$, then $\gasket\setminus K_\Gamma$ is countable. \end{lemma} \begin{proof} The only words on $\alphabet$ that appear in $\sW$ but do not appear as combinations of elements in $\Gamma$ are the ones which are eventually constantly equal to either $A$, $B$ or $C_A C_B$. Since this set is countable, we are done. \end{proof} \begin{proposition}\label{prop:simultaneous_conjugacy} The matrices in $\Gamma$ can be simultaneously conjugated to a set of balanced matrices. \end{proposition} \begin{proof} We begin by observing that the matrices in $\Gamma$ are non-negative. Consider the matrix \[ M_\epsilon = \begin{pmatrix} 1 & -\epsilon & -\epsilon \\ -\epsilon & 1 & -\epsilon \\ -\epsilon & -\epsilon & 1 \end{pmatrix}, \] for some sufficiently small $\epsilon$. For instance, $\epsilon \le \frac{1}{5}$ is enough. A direct computation shows that the entries grow linearly with $n$. This implies that we can find two constants $0< c_1 < c_2 < \infty$, which depend only on $\Gamma$ and $\epsilon$, such that, for all $X\in\Gamma$ we have that $M_\epsilon^{-1} X M_\epsilon = X'$ satisfies that $c_1 \le \frac{X'}{n} \le c_2$. For concreteness, let us compute $A^n C_A C_B M_\epsilon$, we have \[ A^n C_A C_B M_\epsilon = \begin{pmatrix} n+1-3n\epsilon & 2n-(2n+1)\epsilon & n-(3n+1)\epsilon \\ -\epsilon & 1 & -\epsilon \\ 1-2\epsilon & 1-2\epsilon & 1-2\epsilon \end{pmatrix}. \] Multiplying by the matrix $M_\epsilon^{-1}$ we see that all the entries, once we divided by $n$, are bounded from above and below. Repeating the computation for the matrices $C_A B^n C_B$ and $(C_A C_B)^n A$, we find the constants $c_1$ and $c_2$. \end{proof} \begin{corollary}\label{cor:sup_affinity_exp} We have that $\sup_N s_{\Gamma_N} = s_\Gamma$. \end{corollary} \begin{proof} Since $\Gamma_N \subseteq \Gamma$, the exponents satisfy $s_{\Gamma_N} \le s_\Gamma$. Let $s < s_\Gamma$. By \cref{prop:simultaneous_conjugacy}, the function $\phi^s$ is almost-submultiplicative on $S_\Gamma$ and $S_N$. Then, $s_\Gamma$ and $s_{\Gamma_N}$ are the unique zeros of the respective pressure functions $P_\Gamma$ and $P_{\Gamma_N}$. Since $\phi^s$ is quasimultiplicative,~\cite[Proposition~3.2]{KaenmakiReeve} ensures that $0<P_\Gamma (s) = \sup_N P_{\Gamma_N}$, which implies that $P_{\Gamma_N}>0$ for some $N$. \end{proof} \begin{lemma}\label{lemma:equality_affinity_exp} We have that $s_\alphabet = s_\Gamma$. \end{lemma} \begin{proof} Let $S_\alphabet$ be the semigroup generated by $\sW$. Since $\Gamma \subseteq S_\alphabet$ we have that $s_\Gamma \le s_\alphabet$. We will now show the other direction. We have \[ \begin{split} \zeta_\alphabet (s) &\le \zeta_\Gamma (s) + \sum_{n=1}^{\infty}\sum_{w\in\sW_n} \sum_{k=1}^{\infty} \phi^s(w A^k) + \phi^s(w B^k) + \phi^s(w (C_AC_B)^k) \\ &\le C\zeta_\Gamma (s) + \sum_{n=1}^{\infty}\sum_{w\in\sW_n} \sum_{k=1}^{\infty} \phi^s(w A^k C_A C_B) + \phi^s(w B^kC_B) + \phi^s(w (C_AC_B)^k A) \\ &\le 2C\zeta_\Gamma(s), \end{split} \] where we used that we can find a constant $C < \infty$, that only depend on the matrices in $\alphabet$ and $s$, such that for all matrices $X\in\alphabet$ and $Y\in\SL(3,\RR)$, we have $\phi^s(Y) \le C\phi^s(YX)$, see, e.g.,~\cite[Lemma~1]{BochiGourmelon}. The above inequalities imply $s_\alphabet \le s_\Gamma$ and we are done. \end{proof} \begin{proposition}\label{prop:Zariski_density} For all sufficiently large $N$, the subgroup $S_N$ generated by $\Gamma_N$ is Zariski dense in $\SL(3,\RR)$. \end{proposition} \begin{proof} We begin by recalling that the Zariski closure of any subgroup is an algebraic group. Let $G$ be the Zariski closure of $S_\Gamma$, and $\mathfrak{g}$ its Lie algebra, which corresponds to the tangent space to the identity. It is clear that $\mathfrak{g} \subseteq \sl(3, \RR)$, where $\sl(3, \RR)$ is the Lie algebra of the Lie group $\SL(3,\RR)$ is the $8$-dimensional algebra of the $3\times 3$ matrices with zero trace and the usual matrix conmutador as Lie bracket: $[X,Y] = XY - YX$. We will show that $\mathfrak{g} = \sl(3, \RR)$, by finding $8$ linearly independent matrices in $\mathfrak{g}$. Let us consider the matrices \[ A^n (C_A C_B) = \begin{pmatrix} n+1 & 2n & n \\ 0 & 1 & 1 \\ 1 & 1 & 1 \end{pmatrix}, \] for any $n\in\NN$. Let $P$ be a polynomial that is zero on all the points of $S_\Gamma$. Then we can form the real polynomial \[ q(x) = P \left(\begin{pmatrix} x+1 & 2x & x \\ 0 & 1 & 1 \\ 1 & 1 & 1 \end{pmatrix}\right). \] Since $q(n)=0$ for all $n\in\NN$, $q\equiv 0$, which implies that the matrices \[ \gamma(x) = \begin{pmatrix} x+1 & 2x & x \\ 0 & 1 & 1 \\ 1 & 1 & 1 \end{pmatrix} \] form a curve inside the real algebraic group $G$. Then \[ X_1 = \frac{d}{dx} \gamma(0)^{-1}\gamma(x)\biggr|_{x=0} = \begin{pmatrix} 1 & 2 & 1 \\ 0 & 0 & 0 \\ -1 & -2 & -1 \end{pmatrix}\in\mathfrak{g}. \] Similarly, by considering $C_A B^n C_B$ and $(C_A C_B)^n$ respectively, we obtain that \[ X_2 = \begin{pmatrix} 0 & 0 & 0 \\ 1 & 1 & 1 \\ -1 & -1 & -1 \end{pmatrix} \qquad \text{and} \qquad X_3 = \begin{pmatrix} 0 & 0 & 0 \\ 0 & 0 & 0 \\ 1 & 1 & 0 \end{pmatrix} \] are in $\mathfrak{g}$. We now consider the conmutators \[ \begin{split} X_4 &= [X_1, X_2] = \begin{pmatrix} 1 & 1 & 1 \\ 0 & 0 & 0 \\ -1 & -1 & -1 \end{pmatrix}, \qquad X_5 = [X_1, X_3] = \begin{pmatrix} 1 & 1 & 0 \\ 0 & 0 & 0 \\ -2 & -3 & -1 \end{pmatrix},\\ X_6 &= [X_2, X_3] = \begin{pmatrix} 0 & 0 & 0 \\ 1 & 1 & 0 \\ -2 & -2 & -1 \end{pmatrix}, \qquad X_7 = [X_3, X_4] = \begin{pmatrix} -1 & -1 & 0 \\ 0 & 0 & 0 \\ -2 & -2 & 1 \end{pmatrix},\\ X_8 &= [X_2, X_5] = \begin{pmatrix} -1 & -1 & -1 \\ -1 & -2 & 0 \\ 4 & 5 & 3 \end{pmatrix}. \end{split} \] It can be checked that the set $\{X_i\}_{i=1}^8$ is a linearly independent subset of $\mathfrak{g}$, and hence $\mathfrak{g}=\sl(3,\RR)$. Thus, $S_\Gamma$ is Zariski dense inside $\SL(3,\RR)$. To conclude the proof, we remark that, since $S_\Gamma$ is a subsemigroup of $S_\alphabet$, the latter is also Zariski dense inside $\SL(3,\RR)$. Since $\SL(3,\RR)$ is a (Zariski) closed and connected subgroup of $\GL(3,\RR)$, density of $\Gamma_N$ for sufficiently large $N$ follows from~\cite[Lemma~3.7]{MorrisSert:Variational}. \end{proof} We can now prove the main result of this section. \begin{proof}[Proof of \cref{thm:dimH_eq_affdim}] From \cref{prop:simultaneous_conjugacy} and \cref{prop:Zariski_density}, for sufficiently large $N$ we have can simultaneously conjugate every $\Gamma_N$ to a subset of \emph{positive} matrices in $\SL(3,\RR)$ which satisfies the SOSC and that generate a Zariski dense subgroup of $\SL(3,\RR)$. Then, \cref{thm:Jurga}, together with \cref{cor:sup_affinity_exp} and \cref{lemma:equality_affinity_exp} yield \[ \dim_H \gasket \ge \sup_N \dim_H K_{\Gamma_N} = \sup_N s_{\Gamma_N} = s_\Gamma = s_\alphabet. \] Let us show the reverse inequality. Since $\gasket \setminus K_\Gamma$ is countable, we have that $\dim_H \gasket = \dim_H K_\Gamma$. By \cref{prop:simultaneous_conjugacy} we can simultaneously conjugate $\Gamma$ to $\Gamma_\epsilon$, a set of positive matrices in $\SL(3,\RR)$. These matrices send the positive cone into a compact subset of itself, so one can reason as in~\cite[Section~6.1.2]{Jurga:Gasket} to show that $\dim_H K_\Gamma \le s_\Gamma$. Finally, by \cref{lemma:equality_affinity_exp}, we have $s_\Gamma = s_\alphabet$. Then, $\dim_H \gasket \le s_\alphabet$. \end{proof} \subsection{A lower bound on $\dim_H \gasket$.} We recall that we denote by $\cE = \{e_1, e_2, e_3\}$ the standard base of $\RR^3$. The corresponding elements in $\PP(\RR^3)$ will be denoted by $E_i = \RR e_i$. In this section, it will be more convenient to use a different set of generators for the semigroup $S_\Gamma$. Let \[ D_1 = A, \quad D_2^n = C_A B^n C_B, \quad D_3 = C_A C_B = \begin{pmatrix} 1 & 0 & 0 \\ 0 & 1 & 0 \\ 1 & 1 & 1 \end{pmatrix}, \] for any $n \ge 1$. Then, $\{D_1, D_2^n, D_3, n\in\NN\}$ still generates the semigroup $s_\Gamma$. Since we will also use their transpose, we list them, to help the reader: \[ \transpose{D_1} = \begin{pmatrix} 1 & 0 & 0 \\ 1 & 1 & 0 \\ 1 & 0 & 1 \end{pmatrix}, \quad \transpose{D_2^n} = \begin{pmatrix} 1 & n & 1 \\ 0 & n+1 & 1 \\ 0 & n & 1 \end{pmatrix}, \quad \transpose{D_3} = \begin{pmatrix} 1 & 0 & 1 \\ 0 & 1 & 1 \\ 0 & 0 & 1 \end{pmatrix}. \] The following result follows from direct computations and will be left to the reader. \begin{lemma}\label{lemma:deltaprime} The matrices $\{D_1, D_2^n, D_3\}$ and their transpose preserve the simplex $\Delta$. Moreover, $\Bigl\{\transpose{D_1}, \transpose{D_2^n}, \transpose{D_3}\Bigr\}$ also preserves the open sub-simplex $\Delta'$ with vertices $(0:1:1)$, $(1:0:1)$ and $(1:1:0)$. \end{lemma} From this, we obtain the following useful corollary. Let us introduce some notation we will need. Let $\gamma = \gamma_1 \gamma_2 \cdots \gamma_k \in S_\Gamma$, then $\transpose{\gamma} = \transpose{\gamma_k} \cdots \transpose{\gamma_2} \transpose{\gamma_1}$. Moreover, given $1\le m \le k$, we denote $\transpose{\gamma}_{[1,m)} = \transpose{\gamma_k} \cdots \transpose{\gamma_{k-m+1}}$. We stress that, in the previous notation, we first take the transpose and then cut the product after $m$ terms. \begin{corollary}\label{cor:insidedeltaprime} For any $i=1,2,3$, for any $k \ge 1$ and any $\gamma\in S_\Gamma$ of length $k$, if $\gamma_j = i$ for some $j$, then $\transpose{\gamma} E_i \in \Delta'$. \end{corollary} \begin{proof} A direct computation shows that \[ \transpose{D_1} E_1 = \transpose{D_3} E_3 = (1:1:1)\in\Delta', \] and \[ \transpose{D_2^n} E_2 = (n:n+1:n)\in\Delta', \] as we wanted. \end{proof} \begin{proposition}\label{prop:gammainDelta} Let $\gamma\in S_\Gamma$, and assume that its last $m$ letters are not the same. Then, $\transpose{\gamma}\gamma \Delta \subset \transpose{\gamma}_{[1,m)} \Delta \cap \overline{\Delta'}$. \end{proposition} \begin{proof} The inclusion $\transpose{\gamma}\gamma \Delta \subset \transpose{\gamma}_{[1,m)} \Delta$ holds trivially. Hence, we need to show that $\transpose{\gamma}\gamma \Delta \subset \Delta'$. If all the letters appear in $\gamma$, then we can conclude by \cref{cor:insidedeltaprime}. Since not all the last $m$ letters of $\gamma$ are the same, we must have that two among $\{D_1, D_2^n, D_3\}$ appear in the last $m$ digits of $\gamma$. We consider each case separately, \textbf{Case 1: only $D_1$ and $D_2^n$ appear.} In this case, we actually have to distinguish whether $D_1$ or $D_2^n$ occurs first. If we have $D_1 D_2^n$ inside $\gamma$, then $\transpose{\gamma}$ contains $\transpose{D_2^n} \transpose{D_1}$. Since \[ \transpose{D_2^n} \transpose{D_1} = \begin{pmatrix} 1 & n & 1 \\ 0 & n+1 & 1 \\ 0 & n & 1 \end{pmatrix} \begin{pmatrix} 1 & 0 & 0 \\ 1 & 1 & 0 \\ 1 & 0 & 1 \end{pmatrix} = \begin{pmatrix} n+2 & n & 1 \\ n+2 & n+1 & 1 \\ n+1 & n & 1 \end{pmatrix}, \] we have that $\transpose{D_2^n} \transpose{D_1} E_i \in \Delta'$ for $i=1,2,3$, as we wanted. Similarly, if $D_2 D_1^n$ is contained inside $\gamma$, then $\transpose{\gamma}$ contains $\transpose{D_1} \transpose{D_2^n}$. Since \[ \transpose{D_1} \transpose{D_2^n} = \begin{pmatrix} 1 & n & 1 \\ 1 & 2n+1 & 2 \\ 1 & 2n & 2 \end{pmatrix}, \] we have that $\transpose{D_1} \transpose{D_2^n} E_i \in \Delta'$ for $i=1,2,3$, and we are done with this case. \textbf{Case 2: only $D_3$ and $D_2^n$ appear.} Let us define the set \[ \nabla_x = \{(x:y:z) \in \Delta: x \le y+z\}. \] It can be checked that this set is invariant under the action of $D_2^n$, $D_3$ and their transposes. Moreover, $D_2^n \Delta \subset \nabla_x$ and $D_3 \Delta \subset \nabla_x$. Hence, $\transpose{\gamma}\gamma \Delta \subset \nabla_x$. Now we compute \[ \transpose{D_3} \transpose{D_2^n} = \begin{pmatrix} 1 & 2n & 2 \\ 0 & 2n+1 & 2 \\ 0 & n & 1 \end{pmatrix} \qquad \text{and} \qquad \transpose{D_2^n} \transpose{D_3} = \begin{pmatrix} 1 & n & n+2 \\ 0 & n+1 & n+2 \\ 0 & n & n+1 \end{pmatrix}. \] We observe that, in both cases, the last two columns belong to $\Delta'$, while the first coordinate is invariant. In other words, $\transpose{\gamma}_{[1,m)} E_2$, $\transpose{\gamma}_{[1,m)} E_3 \in \Delta'$, whereas $\transpose{\gamma}_{[1,m)} E_1 = E_1$. Finally we have $\transpose{\gamma}\gamma \Delta \subset \transpose{\gamma}_{[1,m)} \Delta \cap \nabla_x = \transpose{\gamma}_{[1,m)} \Delta \cap \overline{\Delta'}$, and we are done. \textbf{Case 3: only $D_1$ and $D_2$ appear.} This case can be treated as the previous one, replacing $D_2^n$ by $D_1$ and $\nabla_x$ by $\nabla_z$, which is defined analogously. \end{proof} The following is the key technical result of this section. \begin{lemma}\label{lemma:lastdigitsgamma} For every $m\in\NN$, there exists an $\epsilon_m>0$ such that, for all $\gamma\in\Gamma$, if the last $m$ letters of $\gamma$ are not the same, then we have \[ \| \gamma e_i \| \ge \epsilon_m \alpha_1(\gamma), \] for $i=1,2,3$. \end{lemma} \begin{proof} Using the $KA^+K$ Cartan decomposition of $\SL(3,\RR)$, we can write every matrix $X\in\SL(3,\RR)$ as $\tilde{k}_X a_X k_X$ where $\tilde{k}_X$, $k_X\in\SO(3,\RR)$ and $a_X$ is the diagonal matrix made by the singular values $\alpha_1(X)\ge\alpha_2(X)\ge\alpha_3(X)$. By~\cite[Lemma~14.2]{BenoistQuint:RandomBook}, one has \[ \|\gamma e_i \| \ge \|\gamma\| d(E_i, H_\gamma), \] where $H_\gamma = k_\gamma^{-1} (E_i^\perp)$ is a repelling hyperplane for $\gamma$ and \[ d(\RR v, \RR w) = \frac{\|v \wedge w \|}{\|v\| \|w\|}, \] with $\|\cdot\|$ the standard Euclidean norm on $\RR^3$ and the induced one on $\wedge^2 \RR^3$. One can check that $(\tilde{k}_\gamma E_i)^\perp = (V_{\transpose{\gamma}})^\perp = H_\gamma$. Hence, it is enough to check that the angle between $E_i$ and $V_{\transpose{\gamma}}$ is bounded away from $\frac{\pi}{2}$. Since $E_i^\perp$ is the span of $E_j$, for $j\neq i$, which is an edge of the simplex $\Delta$, it is enough to show that $d(V_{\transpose{\gamma}}, \partial\Delta)$ is bounded from below by a constant that only depends on $m$, not on $\gamma$. By definition, $V_{\transpose{\gamma}}$, is the attracting fixed point of $\transpose{\gamma}\gamma$. In particular, $V_{\transpose{\gamma}}\in\transpose{\gamma}\gamma\Delta$. By \cref{prop:gammainDelta}, we have that $\transpose{\gamma}\gamma \Delta \subset \transpose{\gamma}_{[1,m)} \Delta \cap \overline{\Delta'}$. We remark that $\transpose{\gamma}_{[1,m)} \Delta \cap \overline{\Delta'}$ is a quadrilateral which does not intersect the boundary of the simplex. Thus, $d(\partial\Delta, \transpose{\gamma}_{[1,m)} \Delta \cap \overline{\Delta'}) > 0$. Since, for any given $m$, there exists only finitely many $\gamma$ of length $m$, we can find a $d_m>0$ such that \[ d(V_{\transpose{\gamma}}, \partial\Delta) \ge d\Bigl(\transpose{\gamma}\gamma\Delta, \partial\Delta\Bigr) \ge d\Bigl(\transpose{\gamma}_{[1,m)} \Delta \cap \overline{\Delta'}, \partial\Delta\Bigr) > d_m, \] which completes the proof. \end{proof} We will need an estimation of the distortion of the simplex $\Delta$ by an element $\gamma\in\Gamma$. \begin{lemma}\label{lemma:Distortion} Assume that the last two letters of $\gamma$ are not the same. Then, there exists a constant $C_2 > 1$ such that: \begin{enumerate} \item $\diam(\gamma\Delta) \le C_2 \frac{\alpha_2(\gamma)}{\alpha_1(\gamma)}$. \item $\area(\gamma\Delta) \le C_2 \alpha_1(\gamma)^{-3}$. \end{enumerate} \end{lemma} \begin{proof} We begin with the first point. It is enough to show that $d(\gamma E_i, \gamma E_j) \le C_2 \frac{\alpha_2(\gamma)}{\alpha_1(\gamma)}$, for $i, j = 1, 2, 3$. We have that \[ \begin{split} d(\gamma E_i, \gamma E_j) &= \frac{\|\gamma e_i \wedge \gamma e_j\|}{\|\gamma e_i\| \|\gamma e_j\|} \\ &\le \frac{\alpha_1(\gamma)\alpha_2(\gamma) \|e_i \wedge e_j \|}{\|\gamma e_i\| \|\gamma e_j\|} \\ &\le \frac{\alpha_1(\gamma)\alpha_2(\gamma)}{\epsilon_2^2 (\alpha_1(\gamma))^2} \\ &= \epsilon_2^{-2}\frac{\alpha_2(\gamma)}{\alpha_1(\gamma)}, \end{split} \] where we used \cref{lemma:lastdigitsgamma} in the second inequality. To prove the second point, we begin with the following elementary geometrical fact. Let $x,y,z\in\RR^3\setminus\{0\}$, then the area of the triangle $\stackrel{\triangle}{xyz}$ with vertices $x$, $y$ and $z$ is given by \[ \area(\stackrel{\triangle}{xyz}) = \frac{\|x \wedge y \wedge z\|}{2d_E(0,\stackrel{\triangle}{xyz})}, \] where $d_E$ is the distance from the origin to the plane containing the three points. Slightly abusing the notation, we identify the projective simplex $\Delta$ with the ordinary $3$-simplex in $\RR^3$. Let $x_\gamma = \gamma e_1$, $y_\gamma = \gamma e_2$ and $z_\gamma = \gamma e_3$ the three vertices of $\gamma\Delta$. By definition, \[ x_\gamma = \frac{\gamma e_1}{\|\gamma e_1\|_1}, \] where $\|\cdot\|_1$ is the $\ell^1$-norm on $\RR^3$, and similarly for the other points. Hence \[ \|x_\gamma \wedge y_\gamma \wedge z_\gamma \| = \frac{\|\gamma e_1 \wedge \gamma e_2 \wedge \gamma e_3\|}{\prod_{i=1}^3\|\gamma e_i \|_1} \] Since $\|\gamma e_1 \wedge \gamma e_2 \wedge \gamma e_3\| = \|e_1 \wedge e_2 \wedge e_3\| = 1$, and the entries of $\gamma$ are non-negative, then $\|\gamma e_i \|_1 \ge \|\gamma e_i\|$, so by \cref{lemma:lastdigitsgamma} we are done. \end{proof} We will need the following geometrical result, which follows from the definition of Hausdorff dimension and was proven in~\cite[Lemma~4.1]{PollicottSewell}. \begin{lemma}\label{lemma:PS} For every $\delta > 0$, there exists a $C_\delta > 0$ such that, for all $\gamma\in\Gamma$, there exists a finite open cover $\{B_i(\gamma)\}_{i=1,\dotsc,k}$ of $\gamma\Delta$ with $\diam(B_i(\gamma))\le\diam(\gamma\Delta)$ such that \[ \sum_{i=1}^k \diam^{1+\delta} B_i(\gamma) \le c_\delta \cdot \diam^{1-\delta} \gamma\Delta \cdot \area^\delta \gamma\Delta. \] \end{lemma} Exactly as in \cref{lemma:excluding_constants}, we can decompose the $\gasket$ as a set of \emph{nice} points $\gasket_{\text{nice}}$ whose coding is not eventually constant and a countable set. However, we remark that $\gasket_{\text{nice}} \neq K_\Gamma$, since we have switched the generating set. Let \[ \Gamma^m = \{\gamma\in\Gamma: \text{ the last two digits of $\gamma$ are different and } \diam\gamma\Delta \le 1/m\}, \] and consider the two families of coverings: \[ \mathcal{U}_m = \{B_i(\gamma), \gamma\in\Gamma^m\}, \quad \text{and} \quad \mathcal{U}'_m = \{\gamma\Delta, \gamma\in\Gamma^m\}, \] with $B_i(\gamma)$ defined by \cref{lemma:PS}. We define \[ Y = \bigcap_{m=1}^\infty \bigcup_{U\in\mathcal{U}_m} U, \quad \text{and} \quad Y' = \bigcap_{m=1}^\infty \bigcup_{U\in\mathcal{U}'_m} U. \] Then $\mathcal{U}_m$ is a Vitali cover of $Y$: for every $y\in Y$ and every $\delta>0$, there exists some $U\in\mathcal{U}_m$ such that $\diam U < \delta$ and $y\in U$. Similarly, $\mathcal{U}'_m$ is a Vitali cover of $Y'$. Moreover, by construction $Y\supset Y'$. We have \begin{lemma}\label{lemma:nice_points} We have the inclusion $\gasket_{\text{nice}} \subset Y$. \end{lemma} \begin{proof} Let $x$ be a point in $\gasket_{\text{nice}}$. Then, its coding with respect to $\{D_1, D_2, D_3\}$ is not eventually constant. In particular, there are infinitely many pairs of adjacent letters in its coding which are different one from the other. By diving the coding into subwords after each of these pairs appears, we form infinitely many words $\gamma\in\Gamma$ whose two last letters are different. Thus $x\in\gamma\Delta$. Moreover, since $\diam\gamma\Delta\to 0$ as the length of $\gamma$ increases, we see that $x\in Y' \subset Y$. \end{proof} Let $\Gamma_0 = \langle D_1, D_3 \rangle$ be the semigroup generated by the matrices $D_1 = A$ and $D_3 = C_A C_B$. Then, the arc $I = I(E_1, E_3) = \{\RR(s e_1 + t e_3), s,t\in\RR_{\ge 0}\}$ is preserved by $\Gamma_0$. \begin{lemma} There exists an $\epsilon > 0$ such that, for all $\gamma\in\Gamma_0$ having the last two digits different from each other, we have \[ \alpha_2(\gamma) \ge \epsilon, \qquad \text{and} \qquad \epsilon |\gamma I| \le \frac{1}{\alpha_1(\gamma)^2}, \] where $|\gamma I|$ is the length of the arc $\gamma I$. \end{lemma} \begin{proof} Since the matrices $D_1$ and $D_3$ preserve $I$ and their restriction to the subspace generated by $E_1$ and $E_3$ has determinant one, by \cref{lemma:lastdigitsgamma}, we have \[ |\gamma I| = d(\gamma E_1, \gamma E_3) = \frac{\|\gamma e_1 \wedge \gamma e_3\|}{\|\gamma e_1\| \|\gamma e_3\|} = \frac{1}{\|\gamma e_1\| \|\gamma e_3\|} \le \frac{1}{\alpha_1(\gamma)^2}. \] We recall that the top singular value gives the operator norm of a matrix. Hence, $\alpha_1 (\gamma) \ge \|\gamma e_i \|_1$ for $i=1$, $2$, $3$. So, as in the proof of \cref{lemma:Distortion}, we obtain that \[ \area(\gamma\Delta) = C_2 \frac{\|\gamma e_1 \wedge \gamma e_2 \wedge \gamma e_3\|}{\prod_{i=1}^{3}\|\gamma e_i\|_1} \ge C_2 \frac{1}{\alpha_1(\gamma)^3}. \] Combining the last two inequalities, we obtain \[ \max\{|\gamma I(E_1, E_2)|, |\gamma I(E_2, E_3)|\} \ge \frac{\area(\gamma\Delta)}{|\gamma I|} \ge C_2 \frac{1}{\alpha_1(\gamma)}. \] Finally, using the first part of \cref{lemma:Distortion}, we have \[ \alpha_2(\gamma) \ge C_2 \alpha_1 (\gamma )\diam(\gamma\Delta) \ge C_2 \alpha_1 (\gamma ) \max\{|\gamma I(E_1, E_2)|, |\gamma I(E_2, E_3)|\} \ge C_2^2 \frac{1}{\alpha_1(\gamma)} \] and we are done. \end{proof} We are now ready to conclude and prove the main result of this section. \begin{theorem} The Hausdorff dimension of the Bruin-Troubetzkoy gasket $\gasket$ is greater than $3/2$: \[ \dim_H \gasket = s_\alphabet \ge \frac{3}{2}. \] \end{theorem} \begin{proof} Let $\gamma\in\Gamma_0$ be an element whose last two digits are different, the singular value function~\eqref{eq:singular_value_f} we have \[ \phi^{3/2} (\gamma) = \frac{\alpha_2(\gamma)}{\alpha_1(\gamma)} \biggl(\frac{\alpha_3(\gamma)}{\alpha_1(\gamma)}\biggr)^{1/2} = \frac{\alpha_2(\gamma)^{1/2}}{\alpha_1(\gamma)^2} \ge \frac{\epsilon^{1/2}}{\alpha_1(\gamma)^2} \ge \epsilon^2 |\gamma I|. \] Then \begin{multline*} \sum_{n=1}^{\infty} \sum_{w\in\sW_n} \phi^{3/2}(w) \ge \sum_{\gamma\in\Gamma_0} \phi^{3/2} \ge \sum_{\substack{\gamma\in\Gamma_0 \\ \text{last two digits different}}} \phi^{3/2} \\ \ge \epsilon^2 \sum_{\substack{\gamma\in\Gamma_0 \\ \text{last two digits different}}} |\gamma I|. \end{multline*} We remark that, by \cref{lemma:nice_points}, every point inside $\gamma \cap \gasket_{\text{nice}}$ is covered infinitely many times by \[ \{\gamma I, \gamma\in\Gamma_0 \text{ with the last two digits different from each other}\}. \] Since $I \setminus (I\cap \gasket_{\text{nice}})$ is countable, the series $\ge \sum_{\gamma\in\Gamma_0} \phi^{3/2}$ diverges and $\dim_H \gasket \ge \frac{3}{2}$, as we wanted. \end{proof} \printbibliography \end{document} \typeout{get arXiv to do 4 passes: Label(s) may have changed. Rerun}
2412.08075v2
http://arxiv.org/abs/2412.08075v2
When entropy meets Turán: new proofs and hypergraph Turán results
\documentclass[reqno,11pt]{amsart} \usepackage{amsthm, amsmath, amssymb, stmaryrd} \usepackage[utf8]{inputenc} \usepackage[T1]{fontenc} \usepackage[hyphenbreaks]{breakurl} \usepackage[hyphens]{url} \usepackage{systeme} \usepackage[shortlabels]{enumitem} \usepackage[hidelinks]{hyperref} \usepackage{microtype} \usepackage{bm} \usepackage[margin=1in]{geometry} \usepackage[textsize=scriptsize,backgroundcolor=orange!5]{todonotes} \usepackage[noabbrev,capitalize,sort]{cleveref} \crefname{equation}{}{} \crefname{enumi}{}{} \numberwithin{equation}{section} \usepackage{mathtools} \newtheorem{theorem}{Theorem}[section] \newtheorem{proposition}[theorem]{Proposition} \newtheorem{lemma}[theorem]{Lemma} \newtheorem{claim}[theorem]{Claim} \newtheorem{corollary}[theorem]{Corollary} \newtheorem{conjecture}[theorem]{Conjecture} \newtheorem{fact}[theorem]{Fact} \theoremstyle{definition} \newtheorem{definition}[theorem]{Definition} \newtheorem{problem}[theorem]{Problem} \newtheorem{question}[theorem]{Question} \newtheorem{example}[theorem]{Example} \newtheorem{setup}[theorem]{Setup} \theoremstyle{remark} \newtheorem*{remark}{Remark} \newtheorem*{notation}{Notation} \newcommand{\abs}[1]{\left\lvert#1\right\rvert} \newcommand{\sabs}[1]{\lvert#1\rvert} \newcommand{\norm}[1]{\left\lVert#1\right\rVert} \newcommand{\snorm}[1]{\lVert#1\rVert} \newcommand{\ang}[1]{\left\langle #1 \right\rangle} \newcommand{\sang}[1]{\langle #1 \rangle} \newcommand{\floor}[1]{\left\lfloor #1 \right\rfloor} \newcommand{\ceil}[1]{\left\lceil #1 \right\rceil} \newcommand{\paren}[1]{\left( #1 \right)} \newcommand{\sqb}[1]{\left[ #1 \right]} \newcommand{\sqbb}[1]{\left\llbracket #1 \right\rrbracket} \newcommand{\set}[1]{\left\{ #1 \right\}} \newcommand{\setcond}[2]{\left\{ #1 \;\middle\vert\; #2 \right\}} \newcommand{\cond}[2]{\left( #1 \;\middle\vert\; #2 \right)} \newcommand{\sqcond}[2]{\left[ #1 \;\middle\vert\; #2 \right]} \newcommand{\one}{\mathbbm{1}} \newcommand{\wt}{\widetilde} \newcommand{\wh}{\widehat} \newcommand{\wv}{\overrightarrow} \newcommand{\avgbeta}{\beta} \newcommand{\textred}[1]{\textcolor{red}{#1}} \DeclareMathOperator{\sgn}{sgn} \DeclareMathOperator{\codim}{codim} \DeclareMathOperator{\Span}{span} \DeclareMathOperator{\mult}{mult} \DeclareMathOperator{\vol}{vol} \DeclareMathOperator{\supp}{supp} \DeclareMathOperator{\ex}{ex} \newcommand{\id}{\text{Id}} \newcommand*{\eqdef}{\stackrel{\mbox{\normalfont\tiny def}}{=}} \newcommand{\rc}{\mathrm{rc}} \newcommand{\CC}{\mathbb{C}} \newcommand{\EE}{\mathbb{E}} \newcommand{\FF}{\mathbb{F}} \newcommand{\QQ}{\mathbb{Q}} \newcommand{\RR}{\mathbb{R}} \newcommand{\NN}{\mathbb{N}} \newcommand{\ZZ}{\mathbb{Z}} \newcommand*{\PP}{\mathbb{P}} \newcommand{\cL}{\mathcal L} \newcommand{\cJ}{\mathcal J} \newcommand{\cC}{\mathcal C} \newcommand{\cP}{\mathcal P} \newcommand{\cF}{\mathcal F} \newcommand{\cV}{\mathcal V} \newcommand{\cE}{\mathcal E} \newcommand{\cD}{\mathcal D} \newcommand{\cA}{\mathcal A} \newcommand{\cG}{\mathcal G} \newcommand{\cM}{\mathcal M} \newcommand{\cB}{\mathcal B} \newcommand{\cS}{\mathcal S} \newcommand{\cSF}{\mathcal{S}_{\mathcal{F}}} \newcommand{\cSJ}{\mathcal{S}_{\mathcal{J}}} \newcommand{\bF}{\mathbf{F}} \newcommand{\bJ}{\mathbf{J}} \newcommand{\fm}{\mathfrak{m}} \newcommand{\cT}{\mathcal{T}} \newcommand{\cN}{\mathcal{N}} \newcommand{\cH}{\mathcal{H}} \newcommand{\bi}{\mathbf{i}} \newcommand{\HH}{\mathbb{H}} \newcommand{\Hasse}{\mathsf H} \newcommand{\totalB}{\mathbb B} \newcommand{\prioB}{\mathcal B} \newcommand{\totalD}{\mathbb D} \newcommand{\prioD}{\mathcal D} \newcommand{\totalT}{\mathbb T} \newcommand{\totalS}{\mathbb S} \newcommand{\totalH}{\mathbb H} \newcommand\tT{\vcenter{\hbox{\scalebox{0.6}{$T$}}}} \newlength{\hght} \newcommand{\halfscript}[2]{\settoheight{\hght}{a}{#1\!\!\:\:}\raisebox{.5\hght}{$\scriptstyle{#2}$}} \newcommand*{\arXiv}[1]{\href{http://arxiv.org/abs/#1}{arXiv:#1}} \makeatletter \newcommand\thankssymb[1]{\textsuperscript{\@fnsymbol{#1}}} \makeatother \author[Ting-Wei Chao]{Ting-Wei Chao\thankssymb{1}} \author[Hung-Hsun Hans Yu]{Hung-Hsun Hans Yu\thankssymb{2}} \thanks{\thankssymb{1}Department of Mathematics, Massachusetts Institute of Technology, Cambridge, MA, USA. Email: {\tt [email protected]}} \thanks{\thankssymb{2}Department of Mathematics, Princeton University, Princeton, NJ 08544\@. Email: {\tt [email protected]}} \title{When entropy meets Tur\'an: \linebreak new proofs and hypergraph Tur\'an results} \begin{document} \maketitle \begin{abstract} In this paper, we provide a new proof of a density version of Tur\'an's theorem. We also rephrase both the theorem and the proof using entropy. With the entropic formulation, we show that some naturally defined entropic quantity is closely connected to other common quantities such as Lagrangian and spectral radius. In addition, we also determine the Tur\'an density for a new family of hypergraphs, which we call tents. Our result can be seen as a new generalization of Mubayi's result on the extended cliques. \end{abstract} \section{Introduction} For any $k$-graph (i.e. $k$-uniform hypergraph) $F$, its \emph{Tur\'an number} $\ex(n,F)$ is the maximum number of edges in an $F$-free $k$-graph $G$ on $n$ vertices. Here, $G$ is $F$-free if it contains no subgraph (not necessarily induced) isomorphic to $F$. The study of Tur\'an numbers was initiated by Tur\'an \cite{Turan85}, who first considered the case where $k=2$ and $F$ is the complete graph $K_{r+1}$ on $(r+1)$ vertices. There, Tur\'an showed that $\ex(n,F)$ is maximized by the balanced complete $r$-partite graph $T_{n,r}$, which we now refer to as the Tur\'an graph. Tur\'an's foundational work has motivated subsequent works on related problems, driving continuing research in extremal graph theory. The general Tur\'an problem is fairly understood when $k=2$. Although the exact value of $\ex(n,F)$ is not known for general graphs $F$, the celebrated Erd\H{o}s--Stone theorem asserts that $\ex(n,F) = \left(1-\frac{1}{r}+o(1)\right)\binom{n}{2}$ if $\chi(F) = r+1$, where $T_{n,r}$ is an asymptotic extremizer. If we define the \emph{Tur\'an density} to be \[\pi(F) = \lim_{n\to\infty}\frac{\ex(n,F)}{\binom{n}{k}}\] for a $k$-graph $F$, then the Erd\H{o}s--Stone theorem can be rephrased as $\pi(F) = 1-\frac{1}{\chi(F)-1}$ when $F$ is a graph. It is worth pointing out that when $\chi(F)=2$, Erd\H{o}s--Stone gives that $\pi(F)=0$, showing that $\ex(n,F)$ is subquadratic but does not determine the asymptotic behavior of $\ex(n,F)$. Despite lots of effort, there are still many interesting open problems regarding the asymptotic behavior of $\ex(n,F)$ when $F$ is bipartite. However, in this paper, we will focus on the non-degenerate case where $\pi(F)>0$. Given how much we know about Tur\'an numbers and Tur\'an densities of graphs, it might be surprising how little we know about hypergraph Tur\'an problems. In fact, the exact value of $\pi(F)$ is still unknown even for $F=K_4^{(3)}$, the $3$-uniform clique on $4$ vertices. Tur\'an showed that $\pi(K_4^{(3)})\geq \frac{5}{9}$ and conjectured that it is actually an equality. However, proving this conjecture still seems hard to date, and the current best upper bound $\pi(F)\leq 0.561666$ was obtained by Razborov \cite{Raz10} using flag-algebraic computation, which was later verified by \cite{BT11} and \cite{F-RV13}. The difficulty comes from the fact that hypergraph Tur\'an problems have drastically different behaviors from the graph case. For example, there is a large family of constructions all showing $\pi(K_4^{(3)})\geq \frac{5}{9}$ given in \cite{Kos82} (also see \cite{F-D-F88}). In comparison, the Erd\H{o}s--Simonovits theorem states that any asymptotic extremizer of $\pi(K_{r+1})$ should be close to $T_{n,r}$. We will discuss other interesting phenomena for hypergraph Tur\'an problems in \cref{subsec:hypergraph-turan-density}. The aim of this paper is to find inspiration for new ways to approach hypergraph Tur\'an problems by examining our new proof of the density Tur\'an theorem, i.e. $\pi(K_{r+1}) = 1-\frac{1}{r}$. This leads to new hypergraph Tur\'an results regarding hypergraphs that we call ``tents'', which generalizes Mubayi's result \cite{Mub06} on the extended cliques. We will introduce our results and related work in more detail in \cref{subsec:hypergraph-turan-density}. Before diving into hypergraph Tur\'an problems, we will first give a quick overview of known proofs of Tur\'an's theorem. We will then introduce the entropy method, which we use to rephrase both the theorem statement and our proof. Then we will mention our hypergraph Tur\'an results that can be obtained using the new perspective, which can be thought of as one of our main results. \subsection{Proofs of Tur\'an's theorem} Tur\'an's original proof \cite{Turan85} works by a clever induction on the number of vertices by removing a $K_r$ from the graph. Erd\H{o}s \cite{Erdos70} later provided another proof that modified the graph step by step, maintaining the $K_{r+1}$-freeness and making the graph complete multipartite at the end. This method has the benefit that it is easier to see that the Tur\'an graph $T_{n,r}$ is the extremizer. A proof of the same spirit is a folklore proof that proceeds with symmetrization (also known now as Zykov Symmetrization as this trick was used by Zykov \cite{Zyk49,Zyk52} in his work). The proof modifies the graph by taking two non-adjacent vertices, and replacing one with another (see \cite[Chapter 41]{AZ18}). Unfortunately, all those proofs do not easily generalize to hypergraphs as they all use properties of graphs crucially. One proof that looks entirely different from the previous proofs is by applying the Caro--Wei theorem, which is due to Alon and Spencer \cite{AS00}. The Caro--Wei theorem, independently proven by Caro \cite{Caro79} and Wei \cite{Wei81}, gives a lower bound on the independence number of a graph $G$ based on its degree sequence. The standard proof of the Caro--Wei theorem is a nice probabilistic argument, which can be found in \cite{AS00}. By taking the complement and an application of Cauchy--Schwarz, the density Tur\'an theorem immediately follows from Caro--Wei. However, this argument does not generalize well to higher uniformities---although the Caro--Wei theorem can be extended to hypergraphs (see \cite{CT91}), applying the inequality on the complement no longer gives tight hypergraph Tur\'an results. Another proof that is seemingly different from all the above is a proof due to Motzkin and Straus \cite{MS65}. Their proof relies crucially on a quantity called \emph{Lagrangian}. The Lagrangian $L(G)$ of a graph $G=(V,E)$ is defined as \[\max \sum_{\{u,v\}\in E}x_ux_v \textup{ subj. to } x_v\geq 0\quad\forall v\in V\textup{ and }\sum_{v\in V}x_v=1.\] Despite its somewhat long definition, it is a natural quantity to consider in the context of Tur\'an problems. To see this, let $N$ be some large positive integers. Consider the \emph{blowup} of $G$ obtained by putting in $(x_v+o(1))N$ copies of each vertex $v\in V$ so that there are $N$ vertices in total, where $(x_v)_{v\in V}$ is the extremizer for the Lagrangian. Then there are $(L(G)+o(1))N^{2}$ edges in the blowup. On the other hand, it is clear that $\abs{E}\leq L(G)\abs{V}^2$, which shows that the density Tur\'an theorem is equivalent to that $L(G)\leq \frac{1}{2}\left(1-\frac{1}{r}\right)$ for every $K_{r+1}$-free graph $G$. Motzkin and Straus' idea is that if $u$ and $v$ are not adjacent, then there is an extremizer with either $x_u=0$ or $x_v=0$ for $L(G)$. Therefore if $G$ is $K_{r+1}$-free, then there is an extremizer with support of size at most $r$. A simple application of Cauchy--Schwarz then concludes the proof. Despite its algebraic look, this proof is actually similar to Zykov Symmetrization in spirit. It is natural to generalize graph Lagrangian to hypergraph Lagrangian. For any $k$-graph $G=(V,E)$, its \emph{hypergraph Lagrangian} $L(G)$ is defined as the maximum of $\sum_{\{u_1,\ldots, u_k\}\in E}x_{u_1}\cdots x_{u_v}$ under the same condition. As before, when each $v\in V$ is blown-up to $(x_v+o(1))N$ vertices where $(x_v)_{v\in V}$ is the extremizer for the Lagrangian, there are $(L(G)+o(1))N^k$ edges in the blowup. As we will mostly talk about the density of a hypergraph rather than the number of edges, it is convenient to define $b(G)=k!L(G)$ to be the \emph{blowup density} of $G$. Intuitively, it is the largest edge density of the blowups of $G$. As it turns out, hypergraph Lagrangian is indeed useful for some hypergraph Tur\'an problems, and we will discuss some of those later in \cref{subsec:hypergraph-turan-density} and \cref{sec:known}. A lesser-known but nonetheless interesting algebraic argument was discovered by Li and Li \cite{LL81}. There, they considered the polynomial \[f\left((x_v)_{v\in V(G)}\right) = \prod_{uv\not\in E}(x_u-x_v)\] for any graph $G$. The key observation is that if $G$ is $K_{r+1}$-free, then $f$ vanishes whenever $r+1$ of the variables $(x_v)_{v\in V(G)}$ are equal to one another. In light of this, let $I$ be the ideal of polynomials that vanish whenever $r+1$ of the variables are equal. Then $f\in I$, and Tur\'an's theorem follows from an explicit description of the generators of $I$ that Li and Li worked out. Our proof looks different from all the proofs mentioned above. For graphs, our proof can be seen as a double-counting argument that, peculiarly, counts infinitely many objects. In particular, we will lower bound the number of stars of each size, and show that $K_{r+1}$-freeness actually imposes an upper bound on the numbers. An interesting feature our proof has is that in order to get the tight bound on the Tur\'an density, it is necessary to take stars of any size into account. Despite the distinctive look of our proof, our proof is closely related to the standard probabilistic proof of the Caro--Wei theorem. In fact, if one runs the standard proof on the blowup of the graph, and take the size of the blowup to infinity, then the limit of the argument becomes our argument (we thank Maya Sankar for pointing this out to us). In spite of the similarity to the proof of the Caro--Wei theorem, our counting argument has the advantage that it can be easily rephrased in terms of entropy. This will be crucial as it will inform us how we should adapt the proof for hypergraphs. We will therefore give an introduction to the entropy method in the next subsection. \subsection{The entropy method} The concept of entropy in the context of information theory was first formulated by Shannon in his seminal work in 1948 on the noisy-channel coding theorem \cite{Sha48}. Roughly speaking, the entropy of a random variable measures how much information the random variable carries. Using entropy, Shannon determined the best efficiency of a code transmitted through a noisy channel that can be corrected with high probability. This has become the foundation of information theory, and many other definitions of entropy have been made as well. However, in this paper, we will only use Shannon's definition of entropy. The adaptation of Shannon entropy in combinatorics and outside the context of information theory came much later in comparison. Some early examples include Chung, Frankl, Graham and Shearer's work on triangle-intersecting family of graphs \cite{CGFS86} (where Shearer's inequality was introduced), Radhakrishnan's entropic proof of the Br\'egman's theorem \cite{Rad97}, and Friedgut and Kahn's theorem on the number of copies of a fixed hypergraph in another hypergraph with a given number of edges \cite{FK98}. There is nonetheless a significant growth in work using the entropy method in the past decade or two. Two recent exciting, and perhaps unexpected, examples are Gilmer's breakthrough on the union-closed set conjecture \cite{Gil22} and the work of Gowers, Green, Manners and Tao resolving Marton's conjecture (also known as the polynomial Freimann--Ruzsa conjecture over $\FF_2$) \cite{GGMT24}. In the context of extremal graph theory, the entropy method is particularly useful when dealing with counts of homomorphisms or homomorphism densities. Here, for any $F,G$ that are graphs or general $k$-graphs, a \emph{homomorphism} from $F$ to $G$ is a function $f:V(F)\to V(G)$ that sends edges of $F$ to edges of $G$. In particular, $f$ must be injective on any edge of $F$. The \emph{homomorphism density} $t(F,G)$ is the probability that a uniformly random chosen function from $V(F)\to V(G)$ is actually a homomorphism. In this terminology, a corollary of the Kruskal--Katona theorem says that $t(K_3, G)\leq t(K_2, G)^{\frac{3}{2}}$, which follows immediately from Shearer's inequality (see also \cite{CY24} for an entropic proof of a slightly stronger result). In the last decade, the entropy method has been applied to show that various bipartite graphs $F$ are \emph{Sidorenko}, i.e. $t(F,G)\geq t(K_2,G)^{e(F)}$. This was first formalized by Szegedy \cite{Sze15} building on a previous work \cite{LS11}, and this was later adapted to attack Sidorenko's conjecture \cite{Par14, CL17, CKLL18-1, CKLL18-2} and related problems \cite{Fitch18, Lee21, GLLV22, BMN24}. In fact, we will also prove some Sidorenko-type result using arguments similar to Szegedy's in our entropic proofs. Given how much the entropy method has been utilized to understand relations between homomorphism densities, it should be surprising that no entropic proof for Tur\'an's theorem was known. Indeed, an equivalent formulation of the density Tur\'an theorem is that if $t(K_{r+1},G)=0$ then $t(K_2, G)\leq 1-\frac{1}{r}$. In this paper, we give the first entropic proof of the density Tur\'an theorem. To do so, we rephrase the density Tur\'an theorem in the following way, and we will later show the equivalence between the two formulations. Below, and throughout the paper, we use $\HH(X)$ to denote the Shannon entropy of a random variable $X$ (see \cref{sec:prelim} for definitions and basic properties). \begin{theorem}[Entropic Tur\'an theorem]\label{thm:entropic-turan} Let $r$ be a positive integer, and let $G$ be a $K_{r+1}$-free graph. Let $X,Y$ be random variables distributed on $V(G)$ so that $\{X,Y\}$ is always an edge in $G$. Assume $X,Y$ are symmetric, i.e. the distribution of $(X,Y)$ and the one of $(Y,X)$ are the same. Then \[\HH(X,Y) \leq 2\HH(X)+\log_2\left(1-\frac{1}{r}\right).\] \end{theorem} We make a brief remark that the equivalence is shown via an entropic reinterpretation of blowup density and Langrangian. Indeed, it turns out that for a given graph $G$, the maximum of the quantity $\HH(X,Y)-2\HH(X)$ for symmetric $V(G)$-valued random variables $X,Y$ with $\{X,Y\}\in E(G)$ is related to the blowup density $b(G)$ of $G$. More surprisingly, the maximum of $\HH(X,Y)-\HH(X)$ is related to the spectral radius $\rho(G)$ of $G$. Those connections will be made precise and proven in \cref{sec:connection}, where we also generalize the connections to hypergraphs. One benefit is that as an immediate corollary of our entropic Tur\'an theorem, we can generalize spectral Tur\'an theorems established by Wilf \cite{Wil86} and Nikiforov \cite{Nik02,Nik06}. \begin{theorem}\label{thm:spectral-Turan-tree} Let $r\geq 2$ and $T$ be a tree with $\ell\geq 1$ vertices. For any $K_{r+1}$-free graph $G$, we have \[\rho(G)^\ell\leq \left(1-\frac{1}{r}\right)\#\{\text{homomorphisms from $T$ to $G$}\}.\] \end{theorem} To see that this is indeed a generalization of Wilf's and Nikiforov's results, we can take $T$ to be the path $P_{\ell}$ on $\ell$ vertices. Wilf's result corresponds to $\ell=1$, whereas Nikiforov's results correspond to $\ell=2$ and general $\ell$. \begin{theorem}[\cite{Wil86,Nik02,Nik06}]\label{thm:spectral-Turan} Let $r\geq 2$. For any $K_{r+1}$-free graph $G$ with $n$ vertices and $m$ edges, we have \[\rho(G)\leq \left(1-\frac{1}{r}\right)n,\] \[\rho(G)^2\leq \left(1-\frac{1}{r}\right)\cdot 2m,\] and \[\rho(G)^\ell\leq \left(1-\frac{1}{r}\right)w_\ell(G),\] where $w_\ell(G)$ denotes the number of $\ell$-walks in $G$. \end{theorem} \subsection{Hypergraph Tur\'an densities}\label{subsec:hypergraph-turan-density} Using the idea from our entropic proof of the density Tur\'an theorem, we can determine the Tur\'an densities for some new family of hypergraphs. Before presenting our results, let us first introduce some definitions and previous work that are relevant. For any family of $k$-graphs $\cF$, its Tur\'an number $\textup{ex}(n,\cF)$ is defined to be the maximum number of edges in a $k$-graph $G$ that is $F$-free for every $F\in \cF$. The Tur\'an density is defined analogously by $\pi(\cF) = \lim_{n\to\infty}\textup{ex}(n,\cF)/\binom{n}{k}$. For any family of $k$-graphs $\cF$ and a $k$-graph $G$, we say that $G$ is \emph{$\cF$-hom-free} if there does not exist any homomorphism $F\to G$ for every $F\in \cF$. A $F$-hom-free $k$-graph is simply a $k$-graph that is $\{F\}$-hom-free. It is a standard result in the field that $\pi(\cF)$ is the supremum of $b(G)$ where $G$ runs through all $\cF$-hom-free $k$-graphs (see \cite[Section 2]{Kee11} or \cite[Lemma 2.2]{San24} for example). Notice that a single edge has blowup density $k!/k^k$, showing that $b(G)\geq k!/k^k$ if $G$ is not empty. This immediately shows that either $\pi(\cF)=0$ or $\pi(\cF)\geq k!/k^k$ for any family of $k$-graphs $\cF$. We see that among the possible values of Tur\'an density, there is a ``jump'' going from $0$ to $k!/k^k$. When $k=2$, this is indeed the behavior of Tur\'an densities: the Erd\H{o}s--Stone theorem shows that all possible values are $0, \frac{1}{2}, \frac{2}{3}, \frac{3}{4},\ldots$, showing that there are only jumps in the case of graphs. However, for hypergraphs, the set of possible Tur\'an densities has a different behavior. It was first discovered by Frankl and R\"odl \cite{FR84} that for each $k\geq 3$, there are infinitely many \emph{non-jumps} $\delta$, where for every $\varepsilon>0$ there exists a family $\cF$ of $k$-graphs with $\pi(\cF)\in (\delta,\delta+\varepsilon)$. On the other hand, Baber and Talbot \cite{BT11} showed that jumps do exist above $k!/k^k$ when $k=3$. However, our understanding in jumps and non-jumps is still limited, and we do not even know whether $k!/k^k$ is a jump. A standard argument shows that $k!/k^k$ is a jump if and only if there exists a finite family $\cF$ of $k$-graph with $\pi(\cF)=k!/k^k$ and $b(F)>k!/k^k$ for each $F\in \cF$ (see \cite{FR84}). The fact that we do not know whether $k!/k^k$ is a jump can thus be seen as a result of not having sufficient understanding in the families $\cF$ with $\pi(\cF)=k!/k^k$. Indeed, known families with Tur\'an densities equal to $k!/k^k$ are so few that we can list them here. For general $k$, Mubayi \cite{Mub06} showed that the $k$-uniform extended clique $E^{(k)}_{k+1}$ of size $k+1$ has Tur\'an density $k!/k^k$. Here, the \emph{extension} of a hypergraph is another hypergraph with higher uniformity obtained by adding different vertices into the edges, and an \emph{extended clique} is an extension of a complete graph. In particular, $E^{(k)}_{k+1}$ is obtained by adding $k-2$ extra vertices to each edge of $K_{k+1}$, where no two edges share any extra vertices. This was later generalized by Mubayi and Pikhurko \cite{MP07}, who showed that the hypergraph $\Delta_{(1,1,\ldots, 1)}$ with edges \[\left\{v_1,\ldots, v_k\right\}\text{ and }\{w,v_i,u^{(i)}_1,\ldots, u^{(i)}_{k-2}\}\text{ for }i\in [k]\] also has Tur\'an density $k!/k^k$. Here, and later whenever the vertex set is not explicitly described, the vertex set consists of vertices that appear in the description of the edges. Mubayi and Pikhurko's result is indeed an improvement as $E^{(k)}_{k+1}$ is homomorphic to $\Delta_{(1,1,\ldots, 1)}$, showing that $E^{(k)}_{k+1}$-hom-free graphs are also $\Delta_{(1,1,\ldots,1)}$-hom-free and so $\pi(E^{(k)}_{k+1})\leq \pi(\Delta_{(1,1,\ldots,1)})$. We remark that both Mubayi's \cite{Mub06} and Mubayi and Pikhurko's \cite{MP07} results are stronger---the exact Tur\'an numbers were determined for sufficiently many vertices. If we only care about the Tur\'an density, then an argument of Sidorenko \cite{Sid89} based on hypergraph Lagrangian can be modified to show that $\pi(\Delta_{(1,\ldots,1)})=k!/k^k$ as well---this is an observation by Keevash \cite[Theorem 3.1]{Kee11}. For smaller $k$'s, slightly more is known. When $k=3$, Bollob\'as \cite{Bol74} showed that $\pi(\{K_4^{-},F_5\}) = \frac{2}{9}$ where $K_4^{-} = \{123,124,134\}$ and $F_5=\{123,124,345\}$. This was improved by Frankl and F\"uredi \cite{FF83}, who showed that $\pi(F_5)$ is already equal to $\frac{2}{9}$. Using flag algebra, Baber and Talbot \cite{BT12} improved this further by showing that $\pi(\{123,124,345,156\}) = \frac{2}{9}$. Finally, when $k=4$, Pikhurko \cite{Pik08} showed that $\pi(\{1234, 1235, 4567\}) = \frac{3}{32}$. As shown above, not a lot is known about families $\cF$ of $k$-graphs with $\pi(\cF)=k!/k^k$. As an application of our entropic proof of the density Tur\'an theorem, we will generalize our argument to show $\pi(\cF)=k!/k^k$ for a new family $\cF$ of $k$-graphs. Our method has a benefit that we may first come up with an argument and then see what family of $k$-graphs need to be forbidden in order for the argument to work. We believe that this advantage can help discovering more families $\cF$ with minimum positive Tur\'an densities. \begin{figure}[h]\centering\label{fig:Tent} \begin{tikzpicture}[scale=0.8] \coordinate (A) at (0,0); \coordinate (B) at (1,0); \coordinate (C) at (2,0); \coordinate (D) at (5,0); \coordinate (E) at (6,0); \coordinate (F) at (4.5,1.732/2); \coordinate (G) at (2.75,1.732*3/4); \coordinate (H) at (4,1.732); \coordinate (I) at (3.5,1.732*3/2); \draw [fill] (A) circle (1.6pt); \draw [fill] (B) circle (1.6pt); \draw [fill] (C) circle (1.6pt); \draw [fill] (D) circle (1.6pt); \draw [fill] (E) circle (1.6pt); \draw [fill] (F) circle (1.6pt); \draw [fill] (G) circle (1.6pt); \draw [fill] (H) circle (1.6pt); \draw [fill] (I) circle (1.6pt); \draw[rounded corners=8pt,black,line width=2pt] (0-0.2,0.5)--(-0.5-0.2,0)--(0-0.2,-0.5)--(6+0.2,-0.5)--(6.5+0.2,0)--(6+0.2,0.5)--cycle; \draw[rounded corners=6pt,black,line width=2pt] (0-0.2,0.3)--(-0.3-0.2,0)--(0-0.2,-0.3)--(2+0.3/1.732,-0.3)--(3.5+0.1+0.15*1.732,1.732*3/2+0.1*1.732-0.15)--(3.5+0.1+0.15,1.732*3/2+0.1*1.732+0.15*1.732)--(3.5+0.1-0.15*1.732,1.732*3/2+0.1*1.732+0.15)--(2-0.3/1.732,0.3)--cycle; \draw[rounded corners=6pt,black,line width=2pt] (6+0.2,0.3)--(6+0.3+0.2,0)--(6+0.2,-0.3)--(5-0.3/1.732,-0.3)--(3.5-0.1-0.15*1.732,1.732*3/2+0.1*1.732-0.15)--(3.5-0.1-0.15,1.732*3/2+0.1*1.732+0.15*1.732)--(3.5-0.1+0.15*1.732,1.732*3/2+0.1*1.732+0.15)--(5+0.3/1.732,0.3)--cycle; \node at (3.5,0) {Base}; \node at (4.7,1.732*3/2) {Apex}; \end{tikzpicture} \caption{$(3,2)$-tent} \end{figure} To state our result, for any partition $\lambda$ of $k$, let $\lambda = (\lambda_1,\ldots, \lambda_{\ell})$ where $\ell = \ell(\lambda)$ is the length of $\lambda$, and $\lambda_1\geq \cdots\geq \lambda_{\ell}$. We also denote $\sum_{i=1}^{\ell}\lambda_i$ by $\abs{\lambda}$ (which is equal to $k$ by definition). For any $\lambda$ with $\ell(\lambda)\geq 2$, we define the \emph{$\lambda$-tent}, denoted by $\Delta_{\lambda}$, to be the following $k$-graph. The $\lambda$-tent comes with an edge $e$ that is the \emph{base} and a vertex $v$ that is the \emph{apex}. Setting $\ell=\ell(\lambda)$ to be the length of $\lambda$, for each $i\in[\ell]$ we also have an edge $e_i$ containing $v$ such that $\abs{e_i\cap e}=\lambda_i$. Moreover, we require that $e_i\cap e_j = \{v\}$ for any $i\neq j\in [\ell]$. It is clear that this determines $\Delta_{\lambda}$ uniquely up to isomorphism---in fact, we must have $e\cap e_1,\ldots, e\cap e_{\ell}$ partition $e$. It is easy to check that this definition matches the definition of $\Delta_{(1,1,\ldots,1)}$ above, $F_5 = \Delta_{(2,1)}$ (with base $123$ and $4$ being the apex) and Pikhurko's result can be rephrased as $\pi(\Delta_{(3,1)})=\frac{3}{32}$. Our result can now be stated as follows. \begin{theorem}\label{thm:main-tent} Let $k\geq 2$ be a positive integer, and let $\cF_k$ be the family of $\lambda$-tents with $\abs{\lambda}=k$ and $\ell(\lambda)=2$. Then $\pi(\cF_k) = k!/k^k$. \end{theorem} Note that this is a stronger statement than Mubayi's and Mubayi and Pikhurko's results. In fact, $\Delta_{(1,1,\ldots, 1)}$ admits a homomorphism to $\Delta_{\lambda}$ for every $\abs{\lambda}=k$ and $\ell(\lambda)=2$, which shows that $\pi(\Delta_{(1,1,\ldots,1)})\leq \pi(\cF_k)$. Using the same argument, we can transform \cref{thm:main-tent} into a Tur\'an result of a single $k$-graph. \begin{theorem}\label{thm:one-tent} Let $k\geq 2$ be a positive integer, and let $\lambda$ be a partition of $k$ such that $\lambda_1\leq \lceil k/2\rceil$ and $\lambda_i=1$ for all $1<i\leq \ell(\lambda)$. Then $\pi(\Delta_\lambda) = k!/k^k$. \end{theorem} Although when $k=3$ and $4$, \cref{thm:one-tent} is subsumed by the known results mentioned above, this gives a new Tur\'an result for larger $k$'s. To show that this should be a nontrivial result for larger $k$'s, we prove the following result in the opposite direction. \begin{theorem}\label{thm:not-Turan} There exists a constant $\alpha<1$ so that for all sufficiently large $k\in\NN$ and any partition $\lambda$ of $k$ with $\ell(\lambda)\geq 2$, if $\lambda_1 >\alpha k$ then $\pi(\Delta_{\lambda})>k!/k^k$. \end{theorem} \cref{thm:one-tent} shows that the constant in \cref{thm:not-Turan} cannot be smaller than $1/2$, and it seems like an interesting question to determine the best possible value of $\alpha$. It might help us understand the $k$-graphs $F$ with $\pi(F)=k!/k^k$ as well. We leave this as a future direction for interested readers. Beyond showing $\pi(\cF) = k!/k^k$ for various families $\cF$ of $k$-graphs, our method also applies to some other scenarios where the extremizers are blowups of complete hypergraphs. Unfortunately, we have not been able to find an argument that proves a new and clean statement in those settings. We nonetheless include the arguments later in \cref{sec:known} in the hope that they will be enlightening for readers interested in adapting our arguments. The relevant background will also be introduced there. \subsection{Structure of the paper} We will first present our new proof of the density Tur\'an theorem in \cref{sec:counting}. We will then introduce the necessary entropic tools in \cref{sec:prelim}, which will set us up for \cref{sec:entropy}, where we rephrase our proof in terms of entropy. In \cref{sec:connection}, we will show how our entropic formulation captures quantities such as hypergrpah Lagrangian and spectral radius. We will use the connection to prove the spectral Tur\'an theorems and the equivalence between the entropic Tur\'an theorem and the density Tur\'an theorem. In \cref{sec:partial-hypergraph}, we set up some notations and propositions that will be useful in the later sections. In \cref{sec:main-proof}, we will apply the entropic argument in \cref{sec:entropy} to show \cref{thm:main-tent} in two different ways, and we will also prove \cref{thm:one-tent,thm:not-Turan}. Some further generalization of our arguments is included in \cref{sec:known}, where we also introduce some related known results. Finally, we will end with some concluding remarks in \cref{sec:conclusion}. \section{A new proof of the density Tur\'an theorem}\label{sec:counting} In this section, we give a new proof to the density Tur\'an theorem. The key idea is to lower bound the density of stars of each size in terms of edge density by their Sidorenko property. If the densities are large, then we shall find a large clique. The main difference of this proof from all the previous ones is that we consider stars of all sizes at once. \begin{proof}[Proof of the density Tur\'an theorem] For any two graphs $H,G$, let $t(H,G)$ be the homomorphism density of $H$ in $G$. That is, $t(H,G)$ is the probability that a function $f:V(H)\rightarrow V(G)$ chosen uniformly at random is a homomorphism from $H$ to $G$. We will need the following lemma about lower bounding the homomorphism density of stars in terms of edge density, which is a special case of Sidorenko's conjecture. We include the proof here since the proof is short. \begin{lemma} For $i\geq 0$, let $S_i=K_{1,i}$ be the star with $i+1$ vertices. Then \[t(S_i,G)\geq t(K_2,G)^i\] holds for any graph $G$. \end{lemma} \begin{proof} Assume $n=\abs{V(G)}$ and $m=\abs{E(G)}$. Note that $S_i$ has $i+1$ vertices, and hence \[t(S_i,G)=\frac{\sum_{v\in V(G)}\deg(v)^i}{n^{i+1}}\geq \frac{1}{n^i}\left(\frac{\sum_{v\in V(G)}\deg(v)}{n}\right)^i=\frac{(2m)^i}{n^{2i}}=t(K_2,G)^i,\] where the inequality follows from the convexity of $x^i$. \end{proof} Now we assume the graph $G$ is $K_{r+1}$-free. We sample a sequence of i.i.d.\@ random vertices $v_0,v_1,\dots$ from $V(G)$ uniformly at random. For $i\geq 0$, let $A_i$ be the event that the induced graph on vertices $v_0,\dots,v_{i-1},v_i$ contains $S_i$ as a subgraph centered at $v_i$. In particular, $A_0$ is the true event. Note that there can only be at most $r$ events happening at the same time. Otherwise, assume $A_{i_0},A_{i_1},\dots,A_{i_r}$ are all true for some $0=i_0<i_1<\dots<i_r$. Then $v_{i_0},\dots,v_{i_r}$ form an $(r+1)$-clique in $G$. Therefore, by double counting, we may conclude that \[\PP(A_0)+\PP(A_1)+\dots\leq r.\] On the other hand, we know that $\PP(A_i)=t(S_i,G)\geq t(K_2,G)^i$ for all $i$. Thus, we have \[\frac{1}{1-t(K_2,G)}\leq \PP(A_0)+\PP(A_1)+\dots\leq r.\] After rearranging, we get \[\frac{2m}{n^2}=t(K_2,G)\leq 1-\frac{1}{r},\] and we are done. \end{proof} \section{Shannon entropy}\label{sec:prelim} In this section, we introduce the definition of Shannon entropy and some of the properties we will use from the literature. We refer the readers to \cite[Section 14.6]{AS00} for a more detailed introduction. We will also prove a lemma which upper bounds the entropies of random variables by the entropy of their mixture. This lemma will be one of the key ingredients of many of the proofs in the rest of this paper. \subsection{Preliminaries} For any discrete random variable $X$, we write $p_X(x)\eqdef\PP(X=x)$. Also, we denote by $\supp(X)$ the support of $X$, i.e. the set of all $x$ such that $p_X(x)>0$. Through out this paper, the random variables we will consider are always discrete with finite support, i.e. $\abs{\supp(X)}<\infty$. For any such random variable, we may define its Shannon entropy. \begin{definition} For any random variable $X$, we define its Shannon entropy \[\HH(X)\eqdef\sum_{x\in\supp(X)}-p_X(x)\log_2p_X(x).\] For any sequence of random variables $X_1,\dots,X_n$, we use $\HH(X_1,\dots,X_n)$ to denote the entropy of the random tuple $(X_1,\dots,X_n)$. \end{definition} We also define the conditional entropy of $X$ given $Y$. \begin{definition} For any two random variables $X,Y$, the conditional entropy of $X$ given $Y$ is given by \[\HH(X\mid Y)\eqdef\HH(X,Y)-\HH(Y).\] Equivalently, we have \begin{align*} \HH(X\mid Y)=&\quad\smashoperator{\sum_{y\in\supp(Y)}}p_Y(y)\HH(X\mid Y=y)\\ =&\quad\smashoperator{\sum_{(x,y)\in \supp(X,Y)}}-p_{X,Y}(x,y)\log_2\left(\frac{p_{X,Y}(x,y)}{p_Y(y)}\right). \end{align*} \end{definition} Using the definition of conditional entropy, we have the following chain rule. \begin{proposition}[Chain rule] For any random variables $X_1,\dots,X_n$, we have \[\HH(X_1,\dots,X_n)=\HH(X_1)+\HH(X_2\mid X_1)+\dots+\HH(X_n\mid X_1,\dots,X_{n-1}).\] \end{proposition} The following proposition says that on a fixed support, the entropy is maximized by the uniform distribution on that support. \begin{proposition}[Uniform bound] For any random variable $X$, we have \[\HH(X)\leq \log_2\abs{\supp(X)},\] where the equality holds if and only if $X$ is uniform. \end{proposition} We will also need the following two propositions about entropy. \begin{proposition}[Subadditivity] For any three random variables $X,Y,Z$, we have \[\HH(X,Y)\leq \HH(X)+\HH(Y),\] \[\HH(X,Y\mid Z)\leq \HH(X\mid Z)+\HH(Y\mid Z).\] \end{proposition} \begin{proposition}[Dropping condition] For any three random variables $X,Y,Z$, we have \[\HH(X\mid Y)\leq \HH(X),\] \[\HH(X\mid Y,Z)\leq \HH(X\mid Z).\] \end{proposition} \subsection{Mixture and the mixture bound} In this subsection, the concern is what is called the \emph{mixture} of random variables. \begin{definition} For random variables $X_1,\dots,X_n$ and weights $w_1,\dots,w_n\geq 0$ with $\sum_{i=1}^n w_i=1$, we say that $Z$ is the \emph{mixture of $X_1,\dots,X_n$ with weight $w_1,\dots, w_n$} if $Z$ is obtained from the following procedure. We first pick an independent random index $\bi$ with probability $\PP(\bi =i)=w_i$. Then we set $Z=X_{\bi}$. \end{definition} In our applications, we will consider mixtures of random variables whose supports do not overlap too much. \begin{definition} Let $a$ be a positive integer. We say that the random variables $X_1,\dots,X_n$ have \emph{$(a+1)$-wise disjoint supports} if for any element $x\in \cup_{i=1}^n \supp (X_i)$, there are at most $a$ many indices $i$ such that $x\in\supp (X_i)$. \end{definition} With the definitions above, we may state our lemma about an upper bound on the entropies of random variables with $(a+1)$-wise disjoint supports, in terms of the entropy of their mixture. \begin{lemma}[Mixture bound]\label{lemma:mix} Let $X_1,\dots,X_n$ be random variables with $(a+1)$-wise disjoint supports. Then there exists a mixture of $X_1,\dots,X_n$, say $Z$, such that \[\sum_{i=1}^n 2^{\HH(X_i)}\leq a2^{\HH(Z)}.\] \end{lemma} \begin{proof} Let $s_i=2^{\HH(X_i)}$ and we define \[w_i=\frac{s_i}{\sum_{j=1}^n s_j}.\] Let $\bi$ be an independent random index with probability $\PP(\bi =i)=w_i$ and let $Z=X_{\bi}$ be the mixture. By chain rule, we have $\HH(Z,\bi)=\HH(\bi)+\HH(Z\mid \bi)=\HH(Z)+\HH(\bi\mid Z)$. Therefore, \[\HH(Z)=\HH(\bi)+\HH(Z\mid \bi)-\HH(\bi\mid Z).\] By the definition of entropy and conditional entropy, we have \[\HH(\bi)=\sum_{i=1}^n -w_i\log_2 w_i=\frac{-s_i}{\sum_{i=1}^n s_i}\log_2\bigl(\frac{s_i}{\sum_{i=1}^n s_i}\bigr)\] and \[\HH(Z\mid \bi)=\sum_{i=1}^n w_i\HH(X_i)=\frac{s_i\log_2 s_i}{\sum_{i=1}^n s_i}.\] We may upper bound $\HH(i\mid Z)$ by uniform bound. For any $x\in\cup_{i=1}^n\supp(X_i)$, when conditioning on $Z=x$, there are at most $a$ possible indices as an outcome of $\bi$. Thus, we have \[\HH(\bi\mid Z)\leq \log_2 a.\] Combining all above, we get \begin{align*} \HH(Z)\geq &\frac{-s_i}{\sum_{i=1}^n s_i}\log_2\bigl(\frac{s_i}{\sum_{i=1}^n s_i}\bigr)+\frac{s_i\log_2 s_i}{\sum_{i=1}^n s_i}-\log_2 a\\ =&\log_2\left(\sum_{i=1}^n s_i\right)-\log_2 a, \end{align*} and we are done after rearranging. \end{proof} The following example shows that \cref{lemma:mix} resembles a double counting on $(a+1)$-wise disjoint sets. Thus, the mixture bound can be viewed as an entropic version of this double counting. \begin{example} Let $a$ be an integer and let $S_1,\dots,S_n$ be some sets that are $(a+1)$-wise disjoint. Assume $X_i$ is a random element chosen from $S_i$ uniform at random for each $i\in [n]$, and let $Z$ be the mixture of $X_1,\ldots, X_n$ provided by \cref{lemma:mix}. We have $2^{\HH(X_i)}=\abs{S_i}$, and by uniform bound we have $2^{\HH(Z)}\leq \abs{\cup_{i=1}^nS_i}$. Hence, \cref{lemma:mix} implies that \[\sum_{i=1}^n\abs{S_i}\leq a2^{\HH(Z)}\leq a\abs{\bigcup_{i=1}^n S_i},\] which gives the same bound as the double counting argument on pairs $(x,i)$ with $x\in S_i$. \end{example} \section{Reformulation using the entropy method}\label{sec:entropy} In this subsection, we reformulate the proof in \cref{sec:counting} using entropy to prove \cref{thm:entropic-turan}. As expected, we shall sample the stars in the same way as Szegedy did \cite{Sze15}, and we will use \cref{lemma:mix} to replace the double counting argument. \begin{proof}[Proof of \cref{thm:entropic-turan}] Recall that we have a $K_{r+1}$-free graph $G$ and symmetric random variables $X,Y$ distributed on $V(G)$ with $\{X,Y\}\in E(G)$ always holding. We first fix an integer $N\in\NN$, and we will take $N$ goes to infinity later. \begin{claim}\label{claim:StarSido} For each $i=0,1,\dots, N$, there exists a random tuple $T_i=(v_0^{(i)},\dots,v_N^{(i)})\in V(G)^{N+1}$ such that \begin{enumerate} \item there is always an edge between $v_j^{(i)},v_i^{(i)}$ for all $j=0,\dots,i-1$, \item the marginal distributions of $v_j^{(i)}$ and $X$ are the same for all $j=0,1\dots,N$, and \item $\HH(T_i)=i\HH(Y\mid X)+(N+1-i)\HH(X)$. \end{enumerate} \end{claim} \begin{proof} For $i=0$, it is easy to check that $N+1$ i.i.d. random vertices $v_0^{(0)},\dots,v_N^{(0)}$ with the law of $X$ satisfy the condition. For $i\geq 1$, we first sample an edge $(v_0^{(i)},v_i^{(i)})$ using the law of $(X,Y)$. Next, we condition on $v_i^{(i)}$ and resample $v_0^{(i)}$ $(i-1)$ times conditionally independently to get $v_1^{(i)},\dots,v_{i-1}^{(i)}$. Finally, we sample $v_{i+1}^{(i)},\dots, v_N^{(i)}$ independently using the law of $X$. Note that the first two conditions are true from the way we sample the random variables. It remains to compute $\HH(T_i)$. Note that $\HH(T_i)=\HH(v_0^{(i)},\dots,v_i^{(i)})+(N-i)\HH(X)$ since we sampled $v_{i+1}^{(i)},\dots, v_N^{(i)}$ independently. By chain rule, we have \begin{align*} \HH(v_0^{(i)},\dots,v_i^{(i)})=&\HH(v_0^{(i)},\dots,v_{i-1}^{(i)}\mid v_i^{(i)})+\HH(v_i^{(i)})\\ =&i\HH(v_0^{(i)}\mid v_i^{(i)})+\HH(v_i^{(i)})\\ =&i\HH(Y\mid X)+\HH(X). \end{align*} Therefore, $\HH(T_i)=i\HH(Y\mid X)+(N+1-i)\HH(X)$. \end{proof} Now, we may apply \cref{lemma:mix} to the random tuples $T_0,\dots,T_N$ in \cref{claim:StarSido}. Since $G$ is $K_{r+1}$-free, similar to the proof in \Cref{sec:counting}, any tuple of $N+1$ vertices is in at most $r$ supports $\supp(T_i)$. Therefore, the supports of $T_0,\dots,T_N$ are $(r+1)$-wise disjoint. Thus, there is a mixture $T=(v_0,\dots,v_N)$ of $T_0,\dots,T_N$ such that \[\sum_{i=0}^N 2^{\HH(T_i)}\leq r2^{\HH(T)}.\] Note that the marginal distribution of $v_i$ is also the same as the marginal distribution of $X$, so we may upper bound $\HH(T)$ by $(N+1)\HH(X)$ by subadditivity. By using $\HH(T_i)=i\HH(Y\mid X)+(N+1-i)\HH(X)$, we get \[\sum_{i=0}^N x^i\leq r,\] where $x\eqdef 2^{\HH(Y\mid X)-\HH(X)}$. By taking $N$ to infinity, we conclude that $1/(1-x)\leq r$. Therefore, \[\HH(Y\mid X)-\HH(X)=\log_2x\leq \log_2\left(1-\frac{1}{r}\right).\qedhere\] \end{proof} Let $\abs{V(G)}=n$ and $\abs{E(G)}=m$. If we pick $(X,Y)$ uniformly at random from all the oriented edges, \cref{thm:entropic-turan} and the uniform bound give \[\log_2(2m)=\HH(X,Y)\leq 2\HH(X)+\log_2\left(1-\frac{1}{r}\right)\leq 2\log_2 n+\log_2\left(1-\frac{1}{r}\right).\] That is, $m\leq \left(1-\frac{1}{r}\right)\frac{n^2}{2}$, which recovers the density Tur\'an theorem. In the next section, we will see that \cref{thm:entropic-turan} is in fact equivalent to the density Tur\'an theorem by relating entropy to blowup densities. \section{Connecting entropy to Lagrangian and spectral radius}\label{sec:connection} In this section, we will show that \cref{thm:entropic-turan} is equivalent to the density Tur\'an theorem. We will actually generalize this equivalence in many ways: we will show it for hypergraphs, and we will also go much beyond Lagrangian and blowup densities. This will be useful later to draw connection to the spectral radius of graphs. We first observe that in \cref{thm:entropic-turan}, the quantity that we care about is actually the maximum of $\HH(X,Y)-2\HH(X)$ when $(X,Y)$ ranges over all possible symmetric distributions on the oriented edges of $G$. This quantity turns out to be related to the blowup density $b(G)$. To extend this to hypergraphs, we make the following definitions. \begin{definition}[Random edge with uniform ordering] Let $G$ be a $k$-graph, we say that a tuple of random vertices $(X_1,\dots,X_k)\in V(G)^k$ is a \emph{random edge with uniform ordering on $G$} if $(X_1,\dots,X_k)$ is symmetric and $\{X_1,\dots,X_k\}$ is always an edge of $G$. Here, $(X_1,\dots,X_k)$ being symmetric means the distribution of $(X_{\sigma(1)},\dots,X_{\sigma(k)})$ is always the same for any permutation $\sigma$ of $[n]$. \end{definition} \begin{definition}[Entropic density] For any $k$-graph $G$, define its \emph{entropic density} $b_{\textup{entropy}}(G)$ to be the largest possible value of $2^{\HH(X_1,\ldots, X_k)-k\HH(X_1)}$ for any random edge with uniform ordering $(X_1,\ldots, X_k)$. \end{definition} Note that $b_{\textup{entropy}}(G)$ exists as the space of random edge with uniform ordering is compact. We will show that $b_{\textup{entropy}}(G)$ is equal to $b(G)$, which immediately shows that \cref{thm:entropic-turan} is equivalent to the density Tur\'an theorem. We will actually show a stronger statement. To that end, we make the following notations. For any $k$-graph $G$, let $\vec{E}(G)$ be the set of oriented edges. For each $p>0$, let $b_p(G)$ be the maximum of $\prod_{(v_1,\ldots, v_k)\in\vec{E}(G)}x_{v_1}\cdots x_{v_k}$ for $(x_v)_{v\in V(G)}$ subject to $\norm{x_v}_{\ell^p}=1$ (the same definition was made by Keevash, Lenz and Mubayi \cite{KLM14} where they called the quantity the \emph{$p$-spectral radius}). Also let $b_{p,\textup{entropy}}(G)$ be the largest possible value of $2^{\HH(X_1,\ldots, X_k)-\frac{k}{p}\HH(X_1)}$ for any random edge with uniform ordering $(X_1,\ldots, X_k)$. Note that $b_p(G)$ and $b_{p,\textup{entropy}}(G)$ both exist by compactness. \begin{example}\label{ex:entropic-lagrangian} When $p=1$, we clearly have $b_p(G)=b(G)$ and $b_{p,\textup{entropy}}(G) = b_{\textup{entropy}}(G)$. When $G$ is a graph and $p=2$, it is not hard to see that $b_p(G)$ is the maximum \[\max\vec{x}^{\intercal}A_G\vec{x}\textup{ subject to }\norm{(x_v)_{v\in V(G)}}_{\ell^2}=1\] where $A_G$ is the adjacency matrix of $G$. It is a standard fact that this is exactly the spectral radius of $G$. In this case, $b_{2,\textup{entropy}}(G)$ is the largest possible value of $2^{\HH(X,Y)-\HH(X)} = 2^{\HH(Y\mid X)}$ for any random edge with uniform ordering $(X,Y)$. For general $k$, if $p=k$, then $b_p(G)$ corresponds to the spectral radius of the adjacency \linebreak$k$-tensor of $G$, which was proven in \cite{Qi13}. The quantity $b_{k,\textup{entropy}(G)}$ is the largest possible value of $2^{\HH(X_1,\ldots, X_k)-\HH(X_1)} = 2^{\HH(X_2,\ldots, X_k\mid X_1)}$. Once we prove $b_k(G) = b_{k,\textup{entropy}(G)}$, this would provide a nice alternative interpretation of the spectral radius for hypergraphs. \end{example} Now we will show that $b_p(G)$ and $b_{p,\textup{entropy}}(G)$ are equal to each other. The proof uses Lagrange multiplier in a crucial way. \begin{proposition}\label{prop:entropic-lagrangian} For any $k$-graph $G$ and any $p>0$, $b_{p,\textup{entropy}}(G) = b_p(G)$. \end{proposition} \begin{proof} For any $v\in V(G)$, let $\vec{L}_v(G)$ be the oriented link of $v$, i.e. the set $(v_2,\ldots, v_k)$ such that $(v,v_2,\ldots, v_k)\in \vec{E}(G)$. We start with the following claim that helps us simplify $\HH(X_1,\dots,X_k)-\frac{k}{p}\HH(X_1)$ when $(X_1,\dots,X_k)$ is in a certain form. \begin{claim}\label{claim:entropy-lag} For any tuple $(x_v)_{v\in V(G)}\in \RR_{\geq 0}^{V(G)}$, we consider a random edge with uniform ordering $(X_1,\dots,X_k)$ on $G$ given by \[\PP((X_1,\dots,X_k)=(v_1,\dots,v_k))=\frac{1}{\beta}\prod_{i=1}^kx_{v_i}, \text{ where }\beta\eqdef\sum_{(v_1,\dots,v_k)\in \vec{E}(G)}\prod_{i=1}^kx_{v_i}.\] We also define \[y_v\eqdef \PP(X_1=v)=\frac{x_v}{\beta}\sum_{(v_2,\dots,v_k)\in \vec{L}_v(G)}\prod_{i=2}^kx_{v_i}.\] Then we have \[\HH(X_1,\dots,X_k)-\frac{k}{p}\HH(X_1)=\log_2\beta-\frac{k}{p}\sum_{v\in V(G)}y_v\log_2 \left(\frac{x_v^p}{y_v}\right).\] \end{claim} \begin{proof} First, we have \begin{align*} \HH(X_1,\dots,X_k)=&\sum_{(v_1,\dots,v_k)\in \vec{E}(G)}-\frac{1}{\beta}\prod_{i=1}^kx_{v_i}\log_2\left(\frac{1}{\beta}\prod_{i=1}^kx_{v_i}\right)\\ =&\sum_{(v_1,\dots,v_k)\in \vec{E}(G)}\frac{1}{\beta}\prod_{i=1}^kx_{v_i}\left(\log_2\beta-\sum_{i=1}^k\log_2x_{v_i}\right)\\ =&\log_2\beta-k\sum_{v\in V(G)}y_v\log_2 x_v \end{align*} Combining this with $\HH(X_1)=\sum_{v\in V(G)}-y_v\log_2y_v$, we get \begin{align*} \HH(X_1,\dots,X_k)-\frac{k}{p}\HH(X_1)=&\log_2\beta-\frac{k}{p}\sum_{v\in V(G)}\left(py_v\log_2 x_v-y_v\log_2y_v\right)\\ =&\log_2\beta-\frac{k}{p}\sum_{v\in V(G)}y_v\log_2 \left(\frac{x_v^p}{y_v}\right).\qedhere \end{align*} \end{proof} Now, we may prove the proposition. We first show that $b_{p,\textup{entropy}}(G)\geq b_p(G)$. Let $(x_v)_{v\in V(G)}\in \RR_{\geq 0}^{V(G)}$ be the tuple that achieves the maximum in the definition of $b_p(G)$. Define $(X_1,\dots,X_k)$, $\beta$, and $(y_v)_{v\in V(G)}$ in the same way as in \cref{claim:entropy-lag}. Note that $\beta=b_p(G)$ and $\sum_{v\in V(G)}x_v^p=1$. From \cref{claim:entropy-lag}, we have \begin{align*} \HH(X_1,\dots,X_k)-\frac{k}{p}\HH(X_1)=&\log_2\beta-\frac{k}{p}\sum_{v\in V(G)}y_v\log_2 \left(\frac{x_v^p}{y_v}\right)\\ \geq& \log_2\beta-\frac{k}{p}\log_2 \left(\sum_{v\in V(G)}x_v^p\right)=\log_2\beta, \end{align*} where the inequality follows from the Jensen's inequality and the concavity of $\log_2x$. Therefore $b_{p,\textup{entropy}}(G)\geq b_p(G)$. For the opposite direction, let $(X_1,\ldots, X_k)$ be a random edge with uniform ordering achieving the maximum of $b_{p,\textup{entropy}}(G)$. For any unoriented edge $e\in E(G)$, let $q_e$ be the probability $\PP(\{X_1,\ldots, X_k\}=e)$. Also let $x_v = \left(\frac{1}{k}\sum_{e\ni v} q_e\right)^{1/p}$. Then \[\HH(X_1,\ldots, X_k) = \HH(X_1,\ldots, X_k\mid \{X_1,\ldots, X_k\})+\HH(\{X_1,\ldots, X_k\}) = \log_2 k! -\sum_{e\in E(G)}q_e\log_2 q_e\] and \[\HH(X_1) = \sum_{v\in V}-x_v^p\log_2 x_v^p.\] Therefore, $(q_e)_{e\in E(G)}$ is a maximizer of \[-\sum_{e\in E(G)}q_e\log_2 q_e+\frac{k}{p}\sum_{v\in V(G)}x_v^p\log_2 x_v^p\] subject to $q_e \geq 0$ for all $e\in E(G)$ and $\sum_{e\in E(G)}q_e=1$. Note that $\partial x_v^p/\partial q_e$ is nonzero only if $v\in e$, and if that is the case we have $\partial x_v^p/\partial q_e=1/k$. By Lagrange multiplier, we know that \[-\log_2 q_e-1 + \frac{1}{p}\sum_{v\in e}\left(1+\log_2 x_v^p\right)\] is constant for all $e\in E(G)$ with $q_e>0$. Therefore \[\alpha \eqdef \frac{q_e}{\prod_{v\in e}x_v}\] is the same for all $e\in E(G)$ with $q_e>0$. Notice that $\PP(X_1 = v) = x_v^p$ for any $v\in V(G)$, and for any $(v_1,\ldots, v_k)\in \vec{E}(G)$, we have \[\PP((X_1,\ldots, X_k)=(v_1,\ldots, v_k)) = \frac{q_e}{k!}=\frac{\alpha}{k!} \prod_{i=1}^{k}x_{v_i}.\] Therefore, using \Cref{claim:entropy-lag} with $\beta=k!/\alpha$, we see that \begin{align*} \HH(X_1,\dots,X_k)-\frac{k}{p}\HH(X_1)=&\log_2\beta-\frac{k}{p}\sum_{v\in V(G)}y_v\log_2 \left(\frac{x_v^p}{y_v}\right), \end{align*} where, in this case, $y_v=x_v^p$. Thus, $\HH(X_1,\dots,X_k)-\frac{k}{p}\HH(X_1)=\log_2 \beta$. Note that $\sum_{v\in V(G)}x_v^p=1$. Therefore by the fact that \[\beta = \sum_{(v_1,\ldots, v_k)\in \vec{E}(G)}\prod_{i=1}^{k}x_{v_i},\] we have $b_{p,\textup{entropy}}(G)\leq b_p(G)$. \end{proof} \begin{corollary}\label{cor:equiv-to-entropy} For any family $\cF$ of $k$-graphs, $\pi(\cF)$ is the supremum of $2^{\HH(X_1,\ldots, X_k)-k\HH(X_1)}$ for any random edge with uniform ordering $(X_1,\ldots, X_k)$ on any $\cF$-hom-free $k$-graph $G$. \end{corollary} \begin{proof} Since $\pi(\cF)$ is the supremum of $b(G)$ for all $\cF$-hom-free $k$-graphs $G$, we know that $\pi(\cF)$ is the supremum of $b_{\textup{entropy}}(G)$ for all $\cF$-hom-free $k$-graphs $G$ as well. The statement follows from the definition of entropic density $b_{\textup{entropy}}(G)$. \end{proof} \begin{corollary}\label{cor:entropy-equiv-to-Turan} The entropic Tur\'an theorem (\cref{thm:entropic-turan}) is equivalent to the density Tur\'an theorem. \end{corollary} \begin{proof} By \cref{cor:equiv-to-entropy}, it suffices to show that if $G$ is $K_{r+1}$-free, then $G$ is $K_{r+1}$-hom-free. This is clear as any homomorphic image of $K_{r+1}$ is $K_{r+1}$. \end{proof} \begin{remark} In the previous section, we showed that \cref{thm:entropic-turan} implies the density Tur\'an theorem using a simpler argument. This turns out to be the direction we care about in this paper. For all the Tur\'an-type results proven later in this paper using entropy and \cref{prop:entropic-lagrangian}, we may also avoid the use of \cref{prop:entropic-lagrangian} by a similar simpler argument. However, we think \cref{prop:entropic-lagrangian} is interesting on its own, so we establish the proposition here and will freely use it from now on. \end{remark} Setting $p=2$, we can now prove \cref{thm:spectral-Turan-tree} by combining \cref{thm:entropic-turan} and Szegedy's method of sampling a random homomorphic image of the tree $T$. \begin{proof}[Proof of \cref{thm:spectral-Turan-tree}] From \cref{prop:entropic-lagrangian} and the observation in \cref{ex:entropic-lagrangian}, there exists a random edge with uniform ordering $(X,Y)$ on $G$ such that $\log_2\rho(G)=\HH(Y\mid X)$. By \cref{thm:entropic-turan}, we have \[\ell\log_2\rho(G)=\ell\HH(Y\mid X)\leq \HH(X)+(\ell-1)\HH(Y\mid X)+\log_2\left(1-\frac{1}{r}\right).\] Let $v_1,\ldots, v_{\ell}$ be an ordering of the vertices of $T$ where for every $i\in\{2,\ldots,\ell\}$, the vertex $v_i$ is adjacent to exactly one $v_j$ with $j<i$. Now, we sample random vertices $X_1,\dots,X_\ell$ in $G$ as follows. Let $X_1$ be a random vertex sampled using the law of $X$. Assume we have already sampled $X_1,\dots,X_{i-1}$, and assume $v_j$ is the neighbor of $v_i$ with $j<i$. We sample $X_i$ conditionally independently such that $X_i\mid X_j\sim Y\mid X$. It follows that $X_1,\dots,X_\ell$ is always a homomorphic image of $T$ in $G$. Also, from the way we sample, we know that $\HH(X_1,\dots,X_\ell)=\HH(X)+(\ell-1)\HH(Y\mid X)$. Thus, we have \[\HH(X)+(\ell-1)\HH(Y\mid X)=\HH(X_1,\dots,X_\ell)\leq \log_2\#\{\text{homomorphisms from $T$ to $G$}\},\] and we are done by combining this with the previous inequality and rearranging. \end{proof} For general $p$, recall that our definition of $b_p(G)$ matches the definition of $p$-spectral radius given by Keevash, Lenz and Mubayi. Thus, by combining \cref{prop:entropic-lagrangian} with \cref{thm:entropic-turan}, we recover the following theorem for graphs by Kang and Nikiforov \cite{KN14}. \begin{theorem}[\cite{KN14}]\label{thm:p-spectral-Turan} Let $r\geq 2$ be a positive integer and $p\geq 1$ be a real number. For any $K_{r+1}$-free graph $G$ with $n$ vertices and $m$ edges, we have \[b_p(G)\leq \left(1-\frac{1}{r}\right)n^{2-2/p},\] and \[b_p(G)\leq \left(1-\frac{1}{r}\right)^{1/p}(2m)^{1-1/p}.\] \end{theorem} \begin{proof} From \cref{prop:entropic-lagrangian}, there exists a random edge with uniform ordering $(X,Y)$ on $G$ such that $\log_2b_p(G)=\HH(X,Y)-\frac{2}{p}\HH(X)$. We have \[\HH(X,Y)-\frac{2}{p}\HH(X)\leq \left(2-\frac{2}{p}\right)\HH(X)+\log_2\left(1-\frac{1}{r}\right)\leq \left(2-\frac{2}{p}\right)\log_2 n+\left(1-\frac{1}{r}\right),\] and \[\HH(X,Y)-\frac{2}{p}\HH(X)\leq \left(1-\frac{1}{p}\right)\HH(X,Y)+\frac{1}{p}\log_2\left(1-\frac{1}{r}\right)\leq \left(1-\frac{1}{p}\right)\log_2 (2m)+\frac{1}{p}\left(1-\frac{1}{r}\right).\qedhere\] \end{proof} We also remark that, by utilizing \cref{prop:entropic-lagrangian}, we can translate \cref{thm:main-entropy-ver} and also results in \cref{sec:known} into spectral results using arguments in the proofs of \cref{thm:spectral-Turan-tree} and \cref{thm:p-spectral-Turan}. \section{Partial hypergraphs}\label{sec:partial-hypergraph} In this section, we introduce some notations and an entropic lemma that will be useful in the later sections. Those notations are non-standard and are set for our own notational convenience when describing hypergraphs and homomorphisms. A \emph{partial $k$-graph} $F$ is a simplicial complex whose faces have size at most $k$. Its set of vertices is denoted by $V(F)$, and its set of faces, or partial edges, is denoted by $E(F)$. A \emph{homomorphism} from a partial $k$-graph $F$ to a $k$-graph $G$ is a map $f:V(F)\to V(G)$ such that for any partial edge $e\in E(F)$, $f$ is injective on $e$ and $f(e)$ is contained in some edge in $E(G)$. Now for any partial $k$-graph $F$, its \emph{extension} $\Tilde{F}$ is the $k$-graph obtained as follows: first let $E'$ be the set of maximal partial edges in $E(F)$, and then extend each partial edge in $E'$ to a $k$-edge by adding in extra vertices, where two different edges do not share any extra vertices. Notice that if $F$ is a simplicial complex generated by edges of some $k'$-graph $F'$ with $k'<k$, then $\Tilde{F}$ is the extension of $F'$ as defined in the introduction. \begin{example}[Definition of partial tents] In \cref{sec:main-proof}, the partial $k$-graphs and the corresponding extensions of concern would be the following. For any partition $\lambda$ of $k$ with $\ell\eqdef \ell(\lambda)\geq 2$, the \emph{partial $\lambda$-tent} $\Delta^p_\lambda$ is the partial $k$-graph obtained by taking the simplicial complex generated by $\Delta_\lambda$, and then restricting it to $e\cup \{v\}$ where $e$ is the base and $v$ is the apex. It is easy to verify that $\Delta_\lambda$ is the extension of the partial $k$-graph $\Delta^p_{\lambda}$. \end{example} \begin{figure}[h]\centering\label{fig:PartialTent} \begin{tikzpicture}[scale=0.8] \coordinate (A) at (0,0); \coordinate (B) at (1,0); \coordinate (C) at (2,0); \coordinate (D) at (5,0); \coordinate (E) at (6,0); \coordinate (F) at (4.5,1.732/2); \coordinate (G) at (2.75,1.732*3/4); \coordinate (H) at (4,1.732); \coordinate (I) at (3.5,1.732*3/2); \draw [fill] (A) circle (1.6pt); \draw [fill] (B) circle (1.6pt); \draw [fill] (C) circle (1.6pt); \draw [fill] (D) circle (1.6pt); \draw [fill] (E) circle (1.6pt); \draw [fill] (I) circle (1.6pt); \draw[rounded corners=8pt,black,line width=2pt] (0-0.2,0.5)--(-0.5-0.2,0)--(0-0.2,-0.5)--(6+0.2,-0.5)--(6.5+0.2,0)--(6+0.2,0.5)--cycle; \draw[rounded corners=6pt,black,line width=2pt] (0-0.2,0.3)--(-0.3-0.2,0)--(0-0.2,-0.3)--(2+0.3/1.732,-0.3)--(3.5+0.1+0.15*1.732,1.732*3/2+0.1*1.732-0.15)--(3.5+0.1+0.15,1.732*3/2+0.1*1.732+0.15*1.732)--(3.5+0.1-0.15*1.732,1.732*3/2+0.1*1.732+0.15)--(2-0.3/1.732,0.3)--cycle; \draw[rounded corners=6pt,black,line width=2pt] (6+0.2,0.3)--(6+0.3+0.2,0)--(6+0.2,-0.3)--(5-0.3/1.732,-0.3)--(3.5-0.1-0.15*1.732,1.732*3/2+0.1*1.732-0.15)--(3.5-0.1-0.15,1.732*3/2+0.1*1.732+0.15*1.732)--(3.5-0.1+0.15*1.732,1.732*3/2+0.1*1.732+0.15)--(5+0.3/1.732,0.3)--cycle; \draw[black,line width=2pt] (7,1.4)--(9,1.4); \draw[black,line width=2pt] (8.7,1.7)--(9,1.4); \draw[black,line width=2pt] (8.7,1.1)--(9,1.4); \node at (8,2) {extension}; \draw [fill] (9,1.4) circle (0.9pt); \draw [fill] (A)+(10,0) circle (1.6pt); \draw [fill] (B)+(10,0) circle (1.6pt); \draw [fill] (C)+(10,0) circle (1.6pt); \draw [fill] (D)+(10,0) circle (1.6pt); \draw [fill] (E)+(10,0) circle (1.6pt); \draw [fill,red] (F)+(10,0) circle (1.6pt); \draw [fill,red] (G)+(10,0) circle (1.6pt); \draw [fill,red] (H)+(10,0) circle (1.6pt); \draw [fill] (I)+(10,0) circle (1.6pt); \draw[rounded corners=8pt,black,line width=2pt] (10+0-0.2,0.5)--(10-0.5-0.2,0)--(10+0-0.2,-0.5)--(10+6+0.2,-0.5)--(10+6.5+0.2,0)--(10+6+0.2,0.5)--cycle; \draw[rounded corners=6pt,black,line width=2pt] (10+0-0.2,0.3)--(10-0.3-0.2,0)--(10+0-0.2,-0.3)--(10+2+0.3/1.732,-0.3)--(10+3.5+0.1+0.15*1.732,1.732*3/2+0.1*1.732-0.15)--(10+3.5+0.1+0.15,1.732*3/2+0.1*1.732+0.15*1.732)--(10+3.5+0.1-0.15*1.732,1.732*3/2+0.1*1.732+0.15)--(10+2-0.3/1.732,0.3)--cycle; \draw[rounded corners=6pt,black,line width=2pt] (10+6+0.2,0.3)--(10+6+0.3+0.2,0)--(10+6+0.2,-0.3)--(10+5-0.3/1.732,-0.3)--(10+3.5-0.1-0.15*1.732,1.732*3/2+0.1*1.732-0.15)--(10+3.5-0.1-0.15,1.732*3/2+0.1*1.732+0.15*1.732)--(10+3.5-0.1+0.15*1.732,1.732*3/2+0.1*1.732+0.15)--(10+5+0.3/1.732,0.3)--cycle; \end{tikzpicture} \caption{Partial $(3,2)$-tent and its extension. Note that for the partial tent, only the maximal edges are shown.} \end{figure} Those definitions are useful as for any partial $k$-graph $F$, a homomorphism $F\to G$ is essentially the same as a homomorphism $\Tilde{F}\to G$. This would be helpful later as instead of considering homomorphisms from $\Delta_{\lambda}$, we can consider homomorphisms from $\Delta^p_{\lambda}$, which are easier to describe. \begin{proposition}\label{prop:partial-tent-hom} Let $F$ be a partial $k$-graph, and let $G$ be a $k$-graph. Then there is a homomorphic image of $F$ in $G$ if and only if there is a homomorphic image of $\Tilde{F}$ in $G$. \end{proposition} \begin{proof} Let $(\Tilde{F})_{\textup{cpx}}$ be the simplicial complex generated by the edges in $\Tilde{F}$, which we will think of as a partial $k$-graph. Then $F$ is the restriction of $(\Tilde{F})_{\textup{cpx}}$ on $V(F)$. For any homomorphism $f:V(\Tilde{F})\to V(G)$ from $\Tilde{F}$ to $G$, we also have that it is an homomorphism from $(\Tilde{F})_{\textup{cpx}}$ to $G$. It is then easy to check that $f|_{V(F)}$ is a homomorphism from $F$ to $G$. Conversely, suppose that $g:V(F)\to V(G)$ is a homomorphism from $F$ to $G$. Note that for each $e\in E(\Tilde{F})$, we know that $e\cap V(F)\in E((\Tilde{F})_{\textup{cpx}})$ and so $e\cap V(F)$ is in $E(F)$ as well. By the definition, this implies that for every $e\in E(\Tilde{F})$, we have that $g$ is injective on $e\cap V(F)$ and $g(e\cap V(F))$ is contained in some edge in $G$. As any vertex in $V(\Tilde{F})\backslash V(F)$ is in exactly one edge in $E(\Tilde{F})$, it is possible to extend $g$ to $\Tilde{g}:V(\Tilde{F})\to V(G)$ so that $g(e)$ is an edge in $G$ for each $e\in E(\Tilde{F})$. The extended map $\Tilde{g}$ is indeed a homomorphism from $\Tilde{F}$ to $G$. \end{proof} Later on, as in the proof in \cref{sec:entropy}, we will need to show that we can sample random homomorphisms from some tree-like structures with high entropy. Before we can do so, we need to first describe what the tree-like structures are. \begin{definition}[Partial forest and forest sequence] For any partial $k$-graph $F$, any linear order $<$ on $V(F)$, and any vertex $v\in V(F)$, let $M_{F,<}(v)$ be the set of partial edges whose maximum vertex is $v$. A partial $k$-graph $F$ is a \emph{partial forest} with respect to a linear order $<$ on $V(F)$ if for every $v\in V(F)$, there is exactly one maximal partial edge $e_v$ in $M_{F,<}(v)$. In this case, the \emph{forest sequence} of $(F,<)$ is a sequence $(n_{1},\ldots, n_{k})$ where for each $i\in[k]$, $n_{i}$ is the number of vertices $v\in V(F)$ with $\abs{e_v}=i$. \end{definition} We also define quantities that are analogs of the quantity $2^{\HH(Y\mid X)-\HH(X)}$ we used in \cref{sec:entropy}. \begin{definition}[Ratio sequence] Let $(X_1,\dots,X_k)\in V(G)^k$ be a random edge with uniform ordering on a $k$-graph $G$. We define the \emph{ratio sequence} $0< x_1\leq\dots\leq x_k= 1$ of $(X_1,\dots,X_k)$ by $x_i=2^{\HH(X_i\mid X_{i+1},\dots,X_k)-\HH(X_i)}$ for each $i\in [k]$. \end{definition} We are now ready to apply Szegedy's argument to sample homomorphisms from partial forests with high entropy. \begin{lemma}\label{lemma:sample-tree} Let $(X_1,\ldots, X_k)$ be a random edge with uniform ordering on a $k$-graph $G$ and let $x_1,\ldots, x_k$ be its ratio sequence. For any partial forest $F$ with a linear order $<$, if $(n_1,\ldots, n_k)$ is its forest sequence, then one can sample a random homomorphism $(Y_v)_{v\in V(F)}$ from $F$ to $G$ with entropy equal to \[v(F)H(X_1)+\log_2\left(\prod_{i=1}^{k}x_i^{n_{k+1-i}}\right).\] Moreover, the random homomorphism can be sampled such that for any partial edge $e\in E(F)$, the distribution of $(Y_v)_{v\in e}$ is the same as $(X_i)_{k-\abs{e}+1\leq i\leq k}$. \end{lemma} \begin{proof} We will induct on $v(F)$. The case $v(F)=0$ is vacuously true. Now suppose that it holds for partial forest of size $v(F)-1$. Let $v_{\max}$ be the maximum vertex in $V(F)$. Then $F\backslash \{v_{\max}\}$ is also a partial forest, and so we may sample a random homomorphism $(Y_v)_{v\in V(F)\backslash \{v_{\max}\}}$ with the prescribed properties. Let $e$ be the maximal partial edge in $M_{F,<}(v_{\textup{max}})$, and let $j=k+1-\abs{e}$. By the inductive hypothesis, $(Y_v)_{v\in e\backslash v_{\max}}$ is identically distributed as $(X_i)_{j+1\leq i\leq k}$. Therefore, we may sample $Y_{v_{\max}}$ given $(Y_v)_{v\in e\backslash v_{\max}}$ conditionally independently so that $(Y_v)_{v\in e}$ is identically distributed as $(X_i)_{j\leq i\leq k}$. This way, \begin{align*} \HH\left((Y_v)_{v\in V(F)}\right)=&\HH\left((Y_v)_{v\in V(F)\backslash \{v_{\max}\}}\right)+\HH\left(Y_{v_{\max}}\mid (Y_v)_{v\in e\backslash \{v_{\max}\}}\right)\\ =&\left(v(F)-1\right)H(X_1)+\log_2\left(x_j^{-1}\prod_{i=1}^{k}x_i^{n_{k+1-i}}\right)+H(X_j\mid X_{j+1},\ldots, X_k)\\ =&v(F)H(X_1)+\log_2\left(\prod_{i=1}^{k}x_i^{n_{k+1-i}}\right) \end{align*} where we use that $\HH(X_i)= \HH(X_1)$ for any $i\in[k]$. It remains to show that for any partial edge $e'$ containing $v_{\max}$, the distribution of $(Y_v)_{v\in e'}$ is the same as $(X_i)_{k-\abs{e'}+1\leq i\leq k}$. This is true as $e'\subseteq e$ by the definition of $e$ and $v_{\max}$, and the distribution $(X_1,\ldots, X_k)$ is symmetric. \end{proof} \section{Proof of \cref{thm:main-tent,thm:one-tent}}\label{sec:main-proof} In this section, we will first give two proofs of \cref{thm:main-tent}. We will then show how \cref{thm:main-tent} implies \cref{thm:one-tent}. Finally, we will conclude this section with a proof of \cref{thm:not-Turan}. Throughout this section, we will fix a $k$-graph $G$ and a random edge with uniform ordering $(X_1,\ldots, X_k)$ on $G$. We will also set $0< x_1\leq \cdots\leq x_k=1$ to be its ratio sequence. We make an observation that to upper bound $b(G)=b_{\textup{entropy}}(G)$, it suffices to upper bound\linebreak $2^{\HH(X_1,\ldots, X_k)-\HH(X_k)}=x_1\cdots x_{k-1}$ by the chain rule. Therefore, the upper bound of \cref{thm:main-tent} follows from the following statement. \begin{theorem}\label{thm:main-entropy-ver} If $G$ is $\lambda$-tent-hom-free for every $\abs{\lambda} = k$ and $\ell(\lambda)=2$, then we have \[\HH(X_1,\dots,X_k)- k\HH(X_1)=\log_2(x_1\cdots x_k)\leq \log_2\frac{k!}{k^k}.\] \end{theorem} We first show that \cref{thm:main-tent} indeed follows from \cref{thm:main-entropy-ver}. \begin{proof}[Proof of \cref{thm:main-tent} using \cref{thm:main-entropy-ver}] First, it is clear that $\pi(\cF_k)\geq k!/k^k$ as a single edge does not contain any homomorphic image of any tents, and it has blowup density $k!/k^k$. To show the reverse inequality, if $G$ is $\cF_k$-hom-free, then by \cref{thm:main-entropy-ver}, we have $b(G)=b_{\textup{entropy}}(G)\leq k!/k^k$. This shows that $\pi(\cF_k)\leq k!/k^k$. \end{proof} \subsection{First proof of \cref{thm:main-entropy-ver}} To prove \cref{thm:main-entropy-ver}, we will apply \cref{lemma:sample-tree} and \cref{lemma:mix} to obtain several inequalities involving $x_1,\ldots,x_{k}$. Then we will solve for the maximum of $x_1\cdots x_{k-1}$ subject to the inequalities. \begin{lemma}\label{lemma:alt-tent-ineq} If $G$ is $\lambda$-tent-hom-free for every $\abs{\lambda} = k$ and $\ell(\lambda)=2$, then for any $i,j\in[k]$ with $i+j\leq k$, we have $x_i+x_j\leq x_{i+j}$. \end{lemma} \begin{proof} We will consider two partial forests $F^{(1)}$ and $F^{(2)}$ both on $V=\{v_1,\ldots, v_k,w\}$. Let $F^{(1)}$ be spanned by the two partial edges $\{v_1,\ldots, v_k\}$ and $\{v_{i+1},\ldots,v_k, w\}$. Let $F^{(2)}$ be spanned by the two partial edges $\{v_1,\ldots, v_k\}$ and $\{v_1,\ldots, v_{k-j}, w\}$. Then both partial $k$-graphs are indeed partial forests with respect to the linear order $v_1<\cdots <v_k<w$. It is clear that in $F^{(1)}$ with the forest sequence $(n_1,\ldots, n_k)$, the vertices $v_1,\ldots, v_k$ contribute one to $n_1,\ldots, n_k$ and $w$ contributes to $n_{k-i+1}$. Similarly, the forest sequence of $F^{(2)}$ is all-one except for $n_{k-j+1}=2$. Let $(Y^{(1)}_v)_{v\in V},(Y^{(2)}_v)_{v\in V}$ be the random homomorphism from $F^{(1)},F^{(2)}$ given by \cref{lemma:sample-tree}, respectively. Note that if some tuple of vertices is in the supports of both $(Y^{(1)}_v)_{v\in V}$ and $(Y^{(2)}_v)_{v\in V}$, then this tuple corresponds to a homomorphism from $F^{(1)}\cup F^{(2)}$ to $G$. As $F^{(1)}\cup F^{(2)}$ clearly contains a partial $(i,k-i)$-tent with base $\{v_1,\ldots, v_k\}$ and apex $w$, we know that the two random homomorphisms have disjoint support. Suppose that $(Z_v)_{v\in V}$ is the mixture given by \cref{lemma:mix}, then by \cref{lemma:mix,lemma:sample-tree} we know \[\left(x_1\cdots x_{k-1}\cdot x_i+x_1\cdots x_{k-1}\cdot x_j\right)2^{(k+1)\HH(X_1)}\leq 2^{\HH((Z_v)_{v\in V})}.\] Observe that both $F^{(1)}$ and $F^{(2)}$ contains the partial edges $\{v_1,\ldots, v_k\}$ and $\{v_{i+1},\ldots, v_{k-j},w\}$. Therefore $(Y^{(1)}_{v_1},\ldots, Y^{(1)}_{v_k})$ and $(Y^{(2)}_{v_1},\ldots, Y^{(2)}_{v_k})$ both have the same distributions as $(X_1,\ldots, X_k)$ by \cref{lemma:sample-tree}, which shows that $(Z_{v_1},\ldots, Z_{v_k})$ has the same distribution as $(X_1,\ldots, X_k)$ as well. Using a similar argument, we can show that $(Z_w, Z_{v_{i+1}},\ldots, Z_{v_{k-j}})$ has the same distribution as $(X_{i+j}, \ldots, X_k)$. As a consequence, \begin{align*} \HH\left((Z_v)_{v\in V}\right)\leq& \HH(Z_{v_1},\ldots, Z_{v_k})+\HH(Z_w\mid Z_{v_{i+1}},\ldots, Z_{v_{k-j}})\\ =&\HH(X_1,\ldots, X_{k})+\HH(X_{i+j}\mid X_{i+j+1},\ldots, X_{k})\\ =& (k+1)\HH(X_1)+\log_2(x_1\cdots x_{k-1}\cdot x_{i+j}). \end{align*} This shows that \[x_1\cdots x_{k-1}2^{(k+1)\HH(X_1)}(x_i+x_j)\leq x_1\cdots x_{k-1}2^{(k+1)\HH(X_1)}\cdot x_{i+j}\] and so the desired statement follows. \end{proof} Our next goal is to upper bound $ x_1\cdots x_{k-1}$. To upper bound the product, we prove the following auxiliary inequality. \begin{lemma}\label{lemma:aux-ineq} Suppose that $y_1,\ldots, y_k$ are some non-negative real numbers with $y_i+y_j\leq y_{i+j}$ for any $i,j\in[k]$ with $i+j\leq k$. Then \[y_1\cdots y_k\leq k!\left(\frac{y_1+\cdots +y_k}{\binom{k+1}{2}}\right)^k.\] \end{lemma} \begin{proof} We will prove this by induction. It clearly holds when $k=1$. Now suppose that $k\geq 2$ and the statement holds for $k-1$. Then by the inductive hypothesis, \[y_1\cdots y_k\leq (k-1)!\left(\frac{y_1+\cdots +y_{k-1}}{\binom{k}{2}}\right)^{k-1}y_k\leq k!\left(\frac{(k-1)\cdot\frac{y_1+\cdots+y_{k-1}}{\binom{k}{2}}+\frac{y_k}{k}}{k}\right)^k\] by AM-GM. Since \[y_1+\dots+y_{k-1} = \frac{1}{2}\sum_{i=1}^{k-1}(y_i+y_{k-i})\leq \frac{k-1}{2}y_k,\] we know \[(k-1)\cdot\frac{y_1+\cdots+y_{k-1}}{\binom{k}{2}}+\frac{y_k}{k} = \frac{2}{k}\left(y_1+\cdots +y_{k-1}+\frac{y_k}{2}\right)\leq \frac{2}{k}\cdot \frac{k}{k+1}\left(y_1+\cdots +y_k\right)\] and so \[y_1\cdots y_k\leq k!\left(\frac{\frac{2}{k+1}(y_1+\cdots+y_k)}{k}\right)^k =k!\left(\frac{y_1+\cdots +y_k}{\binom{k+1}{2}}\right)^k, \] as desired. \end{proof} Combining \cref{lemma:alt-tent-ineq} and \cref{lemma:aux-ineq}, we are now ready to prove \cref{thm:main-entropy-ver}. \begin{proof}[Proof of \cref{thm:main-entropy-ver}] By \cref{lemma:alt-tent-ineq}, $x_1,\ldots, x_k$ are non-negative reals satisfying the condition of \cref{lemma:aux-ineq}. We also know that $x_k=1$, so $x_1+\cdots +x_k\leq \frac{k-1}{2}+1 = \frac{k+1}{2}$. Thus by \cref{lemma:aux-ineq}, \[x_1\cdots x_{k-1} = x_1\cdots x_k\leq k!\left(\frac{\frac{k+1}{2}}{\binom{k+1}{2}}\right)^k = \frac{k!}{k^k},\] which is the desired statement \end{proof} \subsection{Second proof of \cref{thm:main-entropy-ver}} Here, we give an alternative proof using much more complicated partial forests. Although the proof is more involved, this proof would be the one we generalize later in \cref{sec:known}. \begin{lemma}\label{lemma:tent-ineq} If $G$ is $\lambda$-tent-hom-free for every $\abs{\lambda}=k$ and $\ell(\lambda)=2$, then for every $i\in[k-1]$, we have $x_j<x_{i+1}$ for each $j\leq i$ and \[\prod_{j=1}^{i}\frac{x_j}{x_{i+1}-x_j}\leq 1.\] \end{lemma} \begin{proof} We will fix $i$ throughout this proof. As in what we did in \cref{sec:entropy}, we will temporarily fix an integer $N\in\NN$ that will later be taken to infinity. For any $1=t_0<t_1<t_2<\cdots <t_i\leq N$, we will define a partial forest $F^{(\vec{t})}$ on $V=\{v_1,\ldots, v_{k-i-1}, w_1,\ldots, w_N\}.$ The partial forest $F^{(\vec{t})}$ is spanned by the partial edges $\{w_m, w_{t_{j+1}},\ldots, w_{t_i}\}\cup \{v_1,\ldots, v_{k-i-1}\}$ for every $t_j\leq m<t_{j+1}$, where $t_{i+1}$ is set to be $N+1$. This is indeed a partial forest with respect to the linear order $<$ with $v_1<\cdots<v_{k-i-1}<w_N<\cdots <w_1$. We can compute the forest sequence with respect to the linear order as follows: each $v_j$ contributes one to $n_j$ for each $j\leq k-i-1$, and each $w_m$ with $t_j\leq m<t_{j+1}$ contributes $1$ to $n_{k-j}$. Therefore the forest sequence $(n_1,\ldots, n_k)$ is $(t_1-t_0,\ldots, t_{i+1}-t_i, 1,\ldots, 1)$. Now let $(Y^{(\vec{t})}_v)_{v\in V}$ be the random homomorphism produced by \cref{lemma:sample-tree}. This gives \begin{align}\label{eq:tent-2ndproof-eq1} \HH\left((Y^{(\vec{t})}_v)_{v\in V}\right) = (N+k-i-1)\HH(X_1)+\log_2\left(x_{i+2}\cdots x_{k}\cdot \prod_{j\leq i+1}x_j^{t_{j}-t_{j-1}}\right). \end{align} We will now show that the supports of $(Y^{(\vec{t})}_v)_{v\in V}$ are disjoint for different choices of $\vec{t}$. Suppose for the sake of contradiction that for some $\vec{t}\neq \vec{t}'$ there is a tuple of vertices from $V(G)$ lying in the supports of $(Y^{(\vec{t})}_v)_{v\in V}$ and $(Y^{(\vec{t}')}_v)_{v\in V}.$ Then this tuple witnesses a homomorphism sending $F^{(\vec{t})}\cup F^{(\vec{t}')}$ to $G$. We will show a contradiction by demonstrating that $F^{(\vec{t})}\cup F^{(\vec{t}')}$ contains a homomorphic image of some partial $\lambda$-tent with $\ell(\lambda)=2$. Let $j\geq 1$ be the minimum index in which $\vec{t}$ and $\vec{t}'$ differ, and without loss of generality, suppose that $t_j'<t_j$. Then we can find partial edges $e=\{v_1,\ldots,v_{k-i-1},w_{t_0},w_{t_1},\ldots, w_{t_i}\}$,\linebreak $e_1=\{v_1,\ldots, v_{k-i-1},w_{t_j'}, w_{t_j},\ldots, w_{t_i}\}$ in $F^{(\vec{t})}$ and $e_2 = \{w_{t_0'},\ldots, w_{t_j'}\}$ in $F^{( \vec{t}')}$. By the minimality of $j$, we know $e_2=\{w_{t_0},\ldots, w_{t_{j-1}}, w_{t_j'}\}.$ Note that $e,e_1,e_2$ form a partial $(k-j,j)$-tent with base $e$ and apex $w_{t_j'}$, showing that $F^{(\vec{t})}\cup F^{(\vec{t}')}$ contains a partial $(k-j,j)$-tent, which is a contradiction. Therefore we may now apply \cref{lemma:mix} with $a=1$. Suppose that $(Z_v)_{v\in V}$ is the resulting mixture of $(Y_v^{(\vec{t})})_{v\in V}$ for all possible $\vec{t}$. By \cref{lemma:sample-tree} and the fact that $\{v_1,\ldots, v_{k-i-1},w_m\}$ is present in all partial forests we take for any $m\in[N]$, we know that $(Z_{v_1},\ldots, Z_{v_{k-i-1}},Z_{w_m})$ has the same distribution as $(X_{i+1},\ldots, X_k)$ for each $m\in[N]$. Hence \begin{align} \HH\left((Z_v)_{v\in V}\right)\leq& \HH(Z_{v_1},\ldots, Z_{v_{k-i-1}})+\sum_{m=1}^{N}\HH(Z_{w_m}\mid Z_{v_1},\ldots, Z_{v_{k-i-1}}) \nonumber\\ =& \HH(X_{i+2},\ldots,X_k)+N\HH(X_{i+1}\mid X_{i+2},\ldots,X_k) \nonumber\\ =& (N+k-i-1)\HH(X_1)+\log_2(x_{i+2}\cdots x_k\cdot x_{i+1}^N).\label{eq:tent-2ndproof-eq2} \end{align} Thus \cref{lemma:mix,eq:tent-2ndproof-eq1,eq:tent-2ndproof-eq2} now gives \[2^{(N+k-i-1)\HH(X_1)}\sum_{1=t_0<t_1<\cdots <t_{i+1}= N+1}x_{i+2}\cdots x_k\cdot \prod_{j\leq i+1}x_j^{t_{j}-t_{j-1}}\leq x_{i+2}\cdots x_k\cdot x_{i+1}^{N}\cdot 2^{(N+k-i-1)\HH(X_1)},\] and so \[\sum_{1=t_0<t_1<\cdots <t_{i+1}= N+1}\prod_{j\leq i+1}\left(\frac{x_j}{x_{i+1}}\right)^{t_j-t_{j-1}}\leq 1.\] Note that we may replace $j\leq i+1$ by $j<i+1$ in the product. This way, when we take $N$ to approach infinity, we must have $x_j<x_{i+1}$ for each $j\in[i]$ in order for the left hand side to converge. Moreover, the left hand side becomes \[\sum_{\delta_1,\ldots, \delta_{i}\in \NN}\prod_{j\leq i}\left(\frac{x_j}{x_{i+1}}\right)^{\delta_j} =\prod_{j\leq i}\frac{x_j}{x_{i+1}-x_j},\] as desired. \end{proof} Once again, to prove \cref{thm:main-entropy-ver}, we need to upper bound $x_1\cdots x_{k-1}$ given the inequalities in \cref{lemma:tent-ineq}. We will prove a slightly stronger statement, which will also be useful in the next section. \begin{lemma}\label{lemma:another-aux-ineq} Let $k$ be a positive integer. Fix real numbers $0<z_1<\cdots <z_k$. Let $0< y_1< \ldots< y_k$ be real numbers with \[\prod_{j\leq i}\frac{y_j}{y_{i+1}-y_j}\leq \prod_{j\leq i}\frac{z_j}{z_{i+1}-z_j}\] for any $i=1,\ldots, k-1$. Then \[y_1\cdots y_{k-1}\leq \frac{z_1\cdots z_{k-1}}{z_k^{k-1}}y_k^{k-1}.\] \end{lemma} \begin{proof} We will prove by induction on $k$. When $k=1$ this is clearly true. Now suppose that $k\geq 2$ and the statement is true for all smaller $k$. Then we have \[\frac{y_1\cdots y_i}{z_1\cdots z_i}\leq \frac{y_{i+1}^i}{z_{i+1}^{i}}\] for all $i<k-1$ by the inductive hypothesis. Now let \[\alpha_i =\frac{1}{i}\sum_{j\leq i} \frac{z_j}{z_k-z_j}\] for any $i\leq k-1$. Note that for any $i<k-1$, we have \[\left(\frac{y_1\cdots y_{i+1}}{z_1\cdots z_{i+1}}\right)^{\alpha_{i+1}}\leq \left(\frac{y_1\cdots y_i}{z_1\cdots z_i}\right)^{\alpha_{i}}\left(\frac{y_{i+1}^i}{z_{i+1}^i}\right)^{(\alpha_{i+1}-\alpha_i)}\left(\frac{y_{i+1}}{z_{i+1}}\right)^{\alpha_{i+1}}=\left(\frac{y_1\cdots y_i}{z_1\cdots z_i}\right)^{\alpha_{i}}\left(\frac{y_{i+1}}{z_{i+1}}\right)^{\frac{z_{i+1}}{z_k-z_{i+1}}}.\] Here, we are using that $\alpha_{i+1}-\alpha_i\geq 0$ as $\frac{z_1}{z_k-z_1}<\cdots <\frac{z_{i+1}}{z_k-z_{i+1}}$. Multiplying these up for $i=1,\ldots, k-2$, and we get \[\left(\frac{y_1\cdots y_{k-1}}{z_1\cdots z_{k-1}}\right)^{\alpha_{k-1}}\leq \left(\frac{y_1}{z_1}\right)^{\frac{z_1}{z_k-z_1}}\cdots\left(\frac{y_{k-1}}{z_{k-1}}\right)^{\frac{z_{k-1}}{z_k-z_{k-1}}}.\] Thus \begin{align*} \left(\frac{y_1\cdots y_{k-1}}{z_1\cdots z_{k-1}}\right)^{\alpha_{k-1}+1}\leq& \prod_{i=1}^{k-1}\left(\frac{y_i}{z_i}\right)^{\frac{z_i}{z_k-z_i}}\left(\frac{y_k-y_i}{z_k-z_i}\right)\\ =&\prod_{i=1}^{k-1}\left[\left(\frac{y_i}{z_i}\right)^{\frac{z_i}{z_k}}\left(\frac{y_k-y_i}{z_k-z_i}\right)^{\frac{z_k-z_i}{z_k}}\right]^{\frac{z_k}{z_k-z_i}}\\ \leq &\prod_{i=1}^{k-1}\left(\frac{z_i}{z_k}\cdot \frac{y_i}{z_i}+\frac{z_k-z_i}{z_k}\cdot \frac{y_k-y_i}{z_k-z_i}\right)^{\frac{z_k}{z_k-z_i}}&\textup{(weighted AM-GM)}\\ = &\prod_{i=1}^{k-1}\left(\frac{y_k}{z_k}\right)^{\frac{z_k}{z_{k}-z_i}} =\left(\frac{y_k}{z_k}\right)^{(k-1)(\alpha_{k-1}+1)}, \end{align*} completing the inductive step. \end{proof} \begin{proof}[Alternative Proof of \cref{thm:main-entropy-ver}] Suppose $G$ is $\cF_k$-hom-free. Set $z_i=i$ for each $i\in[k]$. By \cref{lemma:tent-ineq}, we know that \[\prod_{j\leq i}\frac{x_j}{x_{i+1}-x_j}\leq 1 = \prod_{j\leq i}\frac{j}{(i+1)-j}=\prod_{j\leq i}\frac{z_j}{z_{i+1}-z_j}.\] Therefore by \cref{lemma:another-aux-ineq} and the fact that $x_k=1$, \[x_1\cdots x_{k-1}\leq \frac{(k-1)!}{k^{k-1}}=\frac{k!}{k^k},\] as desired. \end{proof} \subsection{Proof of \cref{thm:one-tent,thm:not-Turan}} As mentioned in the introduction, \cref{thm:one-tent} is an immediate corollary of \cref{thm:main-tent}. We give a detailed argument of how \cref{thm:one-tent} follows from \cref{thm:main-tent} below. \begin{proof}[Proof of \cref{thm:one-tent}] Let $\lambda$ be a partition of $k$ with $\lambda_1\leq \lceil k/2\rceil$ and $\lambda_i = 1$ for all $1<i\leq \ell(\lambda)$. Again, it is clear that $\pi(\Delta_\lambda)\geq k!/k^k$, so it suffices to show that $\pi(\Delta_\lambda)\leq k!/k^k$. To show this, it suffices to show that any $\Delta_\lambda$-hom-free $k$-graph $G$ is also $\Delta_{\lambda'}$-hom-free for any $\lambda'$ with $\abs{\lambda'}=k$ and $\ell(\lambda')=2$. This will follow immediately if we show that $\Delta_{\lambda}$ admits a homomorphism to $\Delta_{\lambda'}$ for any such $\lambda'$. By \cref{prop:partial-tent-hom}, it is sufficient to show that $\Delta_{\lambda'}$ admits a homomorphism from $\Delta^p_{\lambda}$ for any $\lambda'$ with $\abs{\lambda'}=k$ and $\ell(\lambda')=2$. This is now simple: suppose that $\Delta_{\lambda'}$ has base $e'$ and apex $v'$, and $e_1', e_2'$ are two edges such that $\abs{e_i'\cap e'}=\lambda_i'$ for $i\in[2]$. We also suppose that $\Delta^p_{\lambda}$ has base $e$ and apex $v$, and $e_1,\ldots, e_{\ell}$ are partial edges such that $\abs{e_i\cap e}=\lambda_i$ for $i\in[\ell]$. As $\lambda_1'\geq \lceil k/2\rceil \geq \lambda_1$, we can take $f:e\cup \{v\}\to V(\Delta_{\lambda'})$ so that $f(v)=v'$, $f(e)=e'$ and $f(e\cap e_1)\subseteq e'\cap e_1'$. This is a homomorphism from $\Delta^p_{\lambda}$ to $\Delta_{\lambda'}$ as any vertex in $e'$ shares an edge with $v'$ in $\Delta_{\lambda'}$. \end{proof} Finally, we give a proof of \cref{thm:not-Turan} by demonstrating a $k$-graph $G$ that has $b(G)>k!/k^k$ and is $\Delta_\lambda$-free for large $\lambda_1$. Similar to an earlier lower-bound construction by Frankl and F\"uredi \cite{FF89} for $\Delta_{(k-1,1)}$, we will do so by constructing a $k$-graph $G$ so that the intersection of any two edges is small. \begin{proof} Let $\alpha<1$ be some constant that is close to $1$. In particular, assume that $\alpha > 1/2$. Let $G_{\textup{aux}}$ be an auxiliary graph with vertices $\binom{[2k]}{k}$, and two vertices are connected if the corresponding subsets have intersection at least $\alpha k$. Then $G_{\textup{aux}}$ is a regular graph with degree \[\sum_{i\leq (1-\alpha)k}\binom{k}{i}^2< k\binom{k}{\lfloor (1-\alpha)k\rfloor}^2 = 2^{(2h(\alpha)+o(1))k},\] where $h(\alpha) = -\alpha\log_2\alpha-(1-\alpha)\log_2\alpha$ and we use that \[\binom{k}{(1-\alpha+o(1))k}=2^{(h(\alpha)+o(1))k}\] when $\alpha>1/2$. By the Caro--Wei theorem, there exists an independent set of size \[\frac{\binom{2k}{k}}{2^{(2h(\alpha)+o(1))k}} = 2^{(2-2h(\alpha)+o(1))k}.\] This corresponds to a $k$-graph $G$ on $[2k]$ with $2^{(2-2h(\alpha)+o(1))k}$ edges so that any two edges have intersection less than $\alpha k$. Now if $G$ contains a homomorphic image of $\Delta_\lambda$ where $\lambda_1>\alpha k$, let $e$ be its base and let $e_1$ be the edge with $\abs{e\cap e_1}=\lambda_1$. Also let $f$ be a homomorphism from $\Delta_\lambda$ to $G$. Then $\abs{f(e)\cap f(e_1)}>\alpha k$, and so $f(e)=f(e_1)$. This shows if $v$ is the apex of $\Delta_\lambda$, then $f(v)=f(u)$ for some $u\in e$. However, $\{uv\}$ is contained in some edge in $\Delta_\lambda$, which is a contradiction. Thus $\pi(\Delta_\lambda)$ is at least $b(G)$, which is at least the density of $G$. The density of $G$ is \[\frac{k! \cdot 2^{(2-2h(\alpha)+o(1))k} }{(2k)^k} = 2^{(1-2h(\alpha)+o(1))k}\cdot \frac{k!}{k^k},\] which is strictly greater than $k!/k^k$ for sufficiently large $k$ as long as $h(\alpha)<1/2$. As $h$ is continuous on $[1/2, 1]$ and $h(1)=0$, this is true for $\alpha$ sufficiently close to $1$. \end{proof} The proof roughly gives $\alpha \approx 0.89$. Although our proof is not fully optimized, we believe that it would not give the correct upper bound for $\alpha$ even after being fully optimized. Therefore we do not pursue in this direction. \section{Other applications of our method}\label{sec:known} Recall from the introduction that Mubayi \cite{Mub06} showed $\pi(E^{(k)}_{k+1})=k!/k^k$ where $E^{(k)}_{k+1}$ is the extended clique of size $k+1$, and Mubayi and Pikhurko \cite{MP07} strengthened it to $\pi(\Delta_{(1,1,\ldots,1)}) = k!/k^k$. In fact they both proved more general results than this: Mubayi showed that for each $r\geq k$, \[\pi(E^{(k)}_{r+1})=b(K^{(k)}_r) = \prod_{i=1}^{k-1}\left(1-\frac{i}{r}\right)\] and Mubayi and Pikhurko strengthened it as follows: consider the partial $k$-graph $F$ on $r+1$ vertices generated by $[k]$ and all the $2$-subsets of $[r+1]$, and then take its extension $\Tilde{F}$. Then $\pi(\Tilde{F})=b(K^{(k)}_r)$ as well. Note that $E^{(k)}_{r+1}$ is the extension of $K_{r+1}$ as a partial $k$-graph, and there is a homomorphism from $K_{r+1}$ to $\Tilde{F}$. Therefore $\pi(E^{(k)}_{r+1})\leq \pi(\Tilde{F})$, and so $\pi(\Tilde{F}) = b(K^{(k)}_r)$ is indeed a stronger statement. We remark that Keevash's adaptation \cite[Theorem 3.1]{Kee11} of Sidorenko's argument \cite{Sid89} gives a much more general result than Mubayi and Pikhurko's result in this case, and we refer the readers to Keevash's survey for the statement. We are able to prove $\pi(\Tilde{F}) = b(K^{(k)}_r)$ as well, though our proof is considerably more complicated, and it seems hard to produce a clean stronger statement. We nonetheless outline the argument here for readers interested in improving our argument. \begin{theorem}\label{thm:fix-k-big-r-full-edge} Let $k,r$ be positive integers with $r\geq k$. Let $\cF$ be a family of $k$-graphs such that the following holds. For any $i=1,\ldots, k-1$, if we take the union of any $\binom{r-k+i}{i}+1$ different partial forests $F^{(\vec{t})}$ as in the proof of \cref{lemma:tent-ineq}, then its extension is not $\cF$-hom-free. Then $\pi(\cF) \leq b(K^{(k)}_r)$. \end{theorem} \begin{proof} Suppose that $G$ is $\cF$-hom-free. Let $(X_1,\ldots, X_k)$ be any random edge with uniform ordering on $G$ and let $x_1,\dots,x_k$ be its ratio sequence. We first fix some $i\in[k-1]$. Temporarily fix some large positive integer $N$ as in the proof of \cref{lemma:tent-ineq}. For any $1=t_0<t_1<\cdots <t_i\leq N$, let $(Y_v^{(\vec{t})})_{v\in V}$ be the random homomorphism from $F^{(\vec{t})}$ to $G$ sampled via \cref{lemma:sample-tree} as in the proof of \cref{lemma:tent-ineq}. Then by the assumption on $\cF$ and that $G$ is $\cF$-hom-free, we know that the supports of the random homomorphisms $(Y_v^{(\vec{t})})_{v\in V}$ are $\left(\binom{r-k+i}{i}+1\right)$-wise disjoint. Therefore, if $(Z_v)_{v\in V}$ is the mixture of the $(Y_v^{(\vec{t})})_{v\in V}$'s provided by \cref{lemma:mix}, we have \[ \sum_{1=t_0<t_1<\cdots <t_i\leq N}2^{\HH\left((Y_v^{(\vec{t})})_{v\in V}\right)}\leq\binom{r-k+i}{i}2^{\HH\left((Z_v)_{v\in V}\right)}.\] Using what we have computed in the proof of \cref{lemma:tent-ineq}, when $N$ is taken to infinity, we get \[\prod_{j\leq i}\frac{x_j}{x_{i+1}-x_j}\leq \binom{r-k+i}{i}.\] Now let $z_i = r-k+i$ for each $i=1,\ldots, k$. Then it is easy to verify that \[\binom{r-k+i}{i}= \prod_{j\leq i}\frac{z_j}{z_{i+1}-z_j}\] for each $i\in[k-1]$. Therefore, by \cref{lemma:another-aux-ineq}, we get that \[x_1\cdots x_{k-1}\leq \frac{z_1\cdots z_{k-1}}{z_k^{k-1}} = \frac{(r-k+1)\cdots (r-1)}{r^{k-1}} = \prod_{i=1}^{k-1}\left(1-\frac{i}{r}\right)=b(K_r^{(k)}).\] This shows that $b(G) = b_{\textup{entropy}}(G)\leq b(K_r^{(k)})$ for any $\cF$-hom-free $k$-graph $G$, and so we have $\pi(\cF)\leq b(K_r^{(k)})$. \end{proof} \begin{corollary}\label{cor:fix-k-big-r-full-edge} Let $F$ be the partial $k$-graph on $r+1$ vertices generated by $[k]$ and all the $2$-subsets of $[r+1]$. Let $\Tilde{F}$ be its extension. Then $\pi(\Tilde{F}) = b(K_r^{(k)})$. \end{corollary} \begin{proof} First of all, it is clear that $K_r^{(k)}$ is $F$-hom-free. Therefore, by \cref{prop:partial-tent-hom}, $K_r^{(k)}$ is also $\Tilde{F}$-hom-free, and so $\pi(\Tilde{F})\geq b(K_r^{(k)})$. To show that $\pi(\Tilde{F})\leq b(K_r^{(k)})$, it now suffices to show that the assumption of \cref{thm:fix-k-big-r-full-edge} holds for any $i\in[k-1]$. Indeed, for any collection $T$ of $\binom{r-k+i}{i}+1$ different possible $\vec{t}$'s, we may construct $S\subseteq\NN$ with size $r-k+i+1$ that satisfies the following: for each $s\in S$ there exists $\vec{t}\in T$ such that $s\in\{t_1,\ldots, t_i\}$, and there exists a $\vec{t}\in T$ with $\{t_1,\ldots, t_i\}\subseteq S$. Indeed, set $S' =\bigcup_{\vec{t}\in T}\{t_1,\ldots, t_i\}$. Then $\abs{T}\leq \binom{\abs{S'}}{i}$, which shows that $\abs{S'}\geq r-k+i+1$. Now simply take $S\subseteq S'$ of size $r-k+i+1$ while containing some $\{t_1,\ldots, t_i\}$ for some $\vec{t}\in T$. Label this $\vec{t}$ as $\vec{t^*}$. Now we need to show that there is a homomorphic image of $\Tilde{F}$ in the extension of $\bigcup_{\vec{t}\in T}F^{(\vec{t})}$. By \cref{prop:partial-tent-hom}, it suffices to construct a homomorphism from $F$ to $\bigcup_{\vec{t}\in T}F^{(\vec{t})}$. To do so, we will simply map $1,\ldots,k-i-1$ to $v_1,\ldots, v_{k-i-1}$, map $k-i,\ldots, k$ to $w_{t^*_0},\ldots, w_{t^*_i}$, and then map the rest of the vertices into $S\backslash \{t^*_1,\ldots, t^*_i\}$ bijectively. To show that this is indeed a homomorphism, notice first that $\{v_1,\ldots, v_{k-i-1},w_{t_0^*},\ldots, w_{t_i^*}\}$ is a partial edge in $F^{(\vec{t^*})}$. Therefore it remains to check that $\{w_{s_1},w_{s_2}\}$ and $\{v_{m},w_{s_1}\}$ are both in $\bigcup_{\vec{t}\in T}F^{(\vec{t})}$ for any $s_1\neq s_2\in S$ and $m\in[k-i-1]$. Indeed, if $s_1<s_2$ and $s_2 = t_j$ for some $\vec{t}\in T$, then $\{v_m, w_{s_1}, w_{s_2}\}$ is indeed a partial edge in $F^{(\vec{t})}$, which shows that both $\{w_{s_1},w_{s_2}\}$ and $\{v_{m},w_{s_1}\}$ are partial edges in $F^{(\vec{t})}$ as well. \end{proof} We remark that \cref{thm:fix-k-big-r-full-edge} seems much stronger than \cref{cor:fix-k-big-r-full-edge}, though we do not see a clean way to extract a stronger statement from \cref{thm:fix-k-big-r-full-edge}. We leave this as a potential future direction for interested readers. With a completely different method, we can improve Mubayi's result in a slightly different way, and this is closer to what Sideorenko actually did in his paper \cite{Sid89} using hypergraph Lagrangian. In that paper, Sidorenko showed that many extensions of partial $k$-graphs on $r+1$ vertices have Tur\'an density equal to $b(K_r^{(k)})$, as long as $r$ is at least some threshold $M_k$ that depends on $k$. One special case related to our result is the $k$-graph $F_{r+1}^{(k,k-1)}$ that can be obtained as follows: consider the partial $k$-graph on $[r+1]$ spanned by the edges $\{[k-1]\cup i:i=k,\ldots, r+1\}$ and all the $2$-subsets of $[r+1]$, and then take the extension of the partial $k$-graph. For example, $F_{k+1}^{(k,k-1)}$ is the tent $\Delta_{(k-1,1)}.$ Sidorenko's result is more general and relies on trees $T$ that satisfy the Erd\H{o}s--S\'os conjecture $\textup{ex}(T,n) \leq \frac{1}{2}(v(T)-2)n$, and we refer the readers to Sidorenko's original paper \cite{Sid89} for more details (also see \cite[Section 2]{Ste20} or \cite{TT22} for some families of trees where the Erd\H{o}s--S\'os conjecture is known to hold). With a slightly different choice of partial forests, we can also prove that $\pi(F^{(k,k-1)}_{r+1}) = b(K^{(k)}_r)$ for sufficiently large $r$ with respect to $k$. Our argument actually gives a more general statement: for any $s<k\leq r$, let $F^{(k,s)}_{r+1}$ be the extension of the partial $k$-graph spanned by $\{[s]\cup i:i=s+1,\ldots, r+1\}$ and all the $2$-subsets of $[r+1]$. Then we obtain a sufficient condition for $\pi(F^{(k,s)}_{r+1}) = b(K_r^{(k)})$. \begin{theorem}\label{thm:Sidorenko-type} Let $k,r,s$ be positive integers with $k\leq r$ and \begin{equation}\label{eq:krs-relation} k-s\geq \sum_{i=1}^{s-1}\frac{i}{r-i}. \end{equation} Then $\pi(F^{(k,s)}_{r+1}) = b(K_r^{(k)})$. \end{theorem} \begin{proof} It is clear that $K_r^{(k)}$ is $F^{(k,s)}_{r+1}$-hom-free. Therefore, $\pi(F^{(k,s)}_{r+1})\geq b(K_r^{(k)})$. To prove the other direction $\pi(F^{(k,s)}_{r+1})\leq b(K_r^{(k)})$, we may fix a $F^{(k,s)}_{r+1}$-hom-free $k$-graph $G$ and a random with uniform ordering $(X_1,\dots,X_k)$ on $G$. Let $x_1,\dots,x_k$ be the ratio sequence of $(X_1,\dots,X_k)$. We will solve for the maximum of $x_1\dots x_{k-1}$ under the constraints given by the following lemma. \begin{lemma}\label{lemma:sido-type-forest} For any integers $i,j$ with $i\in [k-s], i\leq j< k$, we have \[\frac{x_i}{r-k+i}\leq x_{j+1}-x_j.\] \end{lemma} \begin{proof} We will fix $i,j$ throughout this proof. As in what we did in \cref{sec:entropy}, we will temporarily fix an integer $N\in\NN$ that will later be taken to infinity. For any $t\in [N]$, we will define a partial forest $F^{(t)}$ on $V=\{v_1,\dots,v_{k-i},w_1,\dots,w_N\}$. The partial forest $F^{(t)}$ is spanned by the partial edges $\{v_1,\dots,v_{k-i},w_t\}$, $\{v_1,\dots,v_{k-j-1},w_m,w_t\}$ for every $m<t$, and $\{v_1,\dots,v_{k-j-1},w_m\}$ for every $m>t$. With the linear order $<$ given by $v_1<\dots<v_{k-i}<w_N<\cdots <w_1$, we know that $F^{(t)}$ is indeed a partial forest. We can compute the forest sequence with respect to the linear order as follows: each $v_m$ contributes one to $n_m$ for each $m\leq k-i$. For the contribution of $w_m$, if $m> t$ it contributes one to $n_{k-j}$; if $m=t$ it contributes one to $n_{k-i+1}$; otherwise it contributes one to $n_{k-j+1}$. Therefore the forest sequence $(n_1,\ldots, n_k)$ is $\vec{e}_1+\dots+\vec{e}_{k-i}+(N-t)\vec{e}_{k-j}+\vec{e}_{k-i+1}+(t-1)\vec{e}_{k-j+1}$, where $\vec{e}_1,\dots,\vec{e}_k$ are the vectors in the standard basis. Now let $(Y^{(t)}_v)_{v\in V}$ be the random homomorphism produced by \cref{lemma:sample-tree}. This gives \begin{align}\label{eq:sido-type-tree-eq1} \HH\left((Y^{(t)}_v)_{v\in V}\right) = (N+k-i)\HH(X_1)+\log_2\left(x_{i}\cdots x_{k}\cdot x_j^{t-1}x_{j+1}^{N-t}\right). \end{align} Now, we show that the random tuples $(Y^{(1)}_v)_{v\in V},\dots,(Y^{(N)}_v)_{v\in V}$ have $(r-k+i+1)$-wise disjoint supports. Note that, for any $t_1<\dots<t_{r-k+i+1}$, the extension of the union $\cup_{\ell=1}^{r-k+i+1} F^{(t_{\ell})}$ contains a homomorphic image of $F^{(k,k-i)}_{r+1}$, given by the partial edges $\{v_1,\dots,v_{k-i},w_{t_{\ell}}\}$ for $\ell\in [r-k+i+1]$ and $\{w_{t_{\ell'}},w_{t_{\ell}}\}$ for $1\leq \ell'<\ell\leq r-k+i+1$. Since $k-i\leq s$, this is also a homomorphic image of $F^{(k,s)}_{r+1}$. Thus, no sequence of vertices is in $\cap_{\ell=1}^{r-k+i+1}\supp ((Y^{(t_\ell)}_v)_{v\in V})$. Therefore we may now apply \cref{lemma:mix} with $a=r-k+i$. Suppose that $(Z_v)_{v\in V}$ is the resulting mixture of $(Y_v^{(t)})_{v\in V}$ for all $t\in [N]$. Note that the partial edge $\{v_1,\dots,v_{k-i}\}$ is present in all partial forests, so by \cref{lemma:sample-tree} we know that $(Z_{v_1},\ldots, Z_{v_{k-i}})$ has the same distribution as $(X_{i+1},\ldots, X_k)$. Similarly, for each $m\in[N]$, since the partial edge $\{v_1,\dots,v_{k-j-1},w_m\}$ is present in all partial forests, we know that $(Z_{v_1},\ldots, Z_{v_{k-j-1}},Z_{w_m})$ has the same distribution as $(X_{j+1},\ldots, X_k)$. Hence \begin{align} \HH\left((Z_v)_{v\in V}\right)\leq& \HH(Z_{v_1},\ldots, Z_{v_{k-i}})+\sum_{m=1}^{N}\HH(Z_{w_m}\mid Z_{v_1},\ldots, Z_{v_{k-j-1}}) \nonumber\\ =& \HH(X_{i+1},\ldots,X_k)+N\HH(X_{j+1}\mid X_{j+2},\ldots,X_k)\nonumber \\ =& (N+k-i)\HH(X_1)+\log_2(x_{i+1}\cdots x_k\cdot x_{j+1}^N).\label{eq:sido-type-tree-eq2} \end{align} Thus \cref{lemma:mix,eq:sido-type-tree-eq1,eq:sido-type-tree-eq2} now give \[\sum_{t=1}^N x_{i}\cdots x_k\cdot x_j^{t-1}x_{j+1}^{N-t}\cdot 2^{(N+k-i)\HH(X_1)}\leq (r-k+i)x_{i+1}\cdots x_k\cdot x_{j+1}^{N}\cdot 2^{(N+k-i)\HH(X_1)},\] and so \[\sum_{t=1}^Nx_ix_j^{t-1}x_{j+1}^{-t}\leq r-k+i.\] By rearranging and taking $N$ goes to infinity, we obtain \[\frac{x_i}{x_{j+1}}\cdot \frac{1}{1-\frac{x_j}{x_{j+1}}}=\sum_{t=1}^\infty \frac{x_i}{x_{j+1}}\left(\frac{x_j}{x_{j+1}}\right)^{t-1}\leq r-k+i,\] and the lemma follows. \end{proof} Once again, to prove \cref{thm:Sidorenko-type}, we need to upper bound $x_1\cdots x_{k-1}$ given the inequalities in \cref{lemma:sido-type-forest}. We start with the following inequality similar to \cref{lemma:aux-ineq}. \begin{lemma}\label{lemma:sido-type-aux-ineq} Suppose that $y_1,\ldots, y_t$ and $z$ are some non-negative real numbers. Then \[y_1\cdots y_t\leq \left(\sum_{i=1}^{t}\frac{y_i}{z+i}\right)^t\frac{(z+1)\cdots(z+t)}{t^t}.\] \end{lemma} \begin{proof} We will prove this by inducting on $t$. For $t=1$, the inequality is trivial. Assume the statement is true for $t-1$. From the inductive hypothesis and AM-GM inequality, we have \begin{align*} y_1\cdots y_t\leq& y_t\left(\sum_{i=1}^{t-1}\frac{y_i}{z+i}\right)^{t-1}\frac{(z+1)\cdots(z+t-1)}{(t-1)^{t-1}}\\ = &\left(\frac{t-1}{z+t}y_t\right)\left(\sum_{i=1}^{t-1}\frac{y_i}{z+i}\right)^{t-1}\frac{(z+1)\cdots(z+t)}{(t-1)^{t}}\\ \leq &\left(\frac{t-1}{t}\sum_{i=1}^{t}\frac{y_i}{z+i}\right)^{t}\frac{(z+1)\cdots(z+t)}{(t-1)^{t}}\\ = &\left(\sum_{i=1}^{t}\frac{y_i}{z+i}\right)^t\frac{(z+1)\cdots(z+t)}{t^t}.\qedhere \end{align*} \end{proof} Now, by using this lemma with $t=k-1, y_i=x_i$ and $z=r-k$, it is sufficient to upper bound right hand side using the conditions from \cref{lemma:sido-type-forest}. \begin{claim}\label{claim:sido-type-xk} We have \[\frac{x_1}{r-k+1}+\dots+\frac{x_{k-1}}{r-1}\leq \frac{k-1}{r}x_k.\] \end{claim} \begin{proof} Let $s'$ be the largest integer such that \[k-s'\geq \sum_{i=1}^{s'-1}\frac{i}{r-i}\] holds. In particular, we have $s\leq s'<k$. Set $c$ to be the real number such that \[\frac{k-1}{r}=(1-c)\frac{1}{r-s'}+\frac{1}{r-s'+1}+\dots+\frac{1}{r-1}.\] From the definition of $s'$, we have \[k-1\geq s'-1+\frac{s'-1}{r-s'+1}+\dots+\frac{1}{r-1}=\frac{r}{r-s'+1}+\dots+\frac{r}{r-1}\] and \[k-1< s'+\frac{s'}{r-s'}+\dots+\frac{1}{r-1}=\frac{r}{r-s'}+\dots+\frac{r}{r-1}.\] Therefore, $c\in (0,1]$. By replacing the coefficient of $x_k$ using the definition of $c$ and rearranging, we may rewrite the inequality we want to show as the following. \begin{align} &\frac{x_1}{r-k+1}+\dots+\frac{x_{k-s-1}}{r-s-1}+c\frac{x_{k-s}}{r-s}\nonumber\\ \leq &(1-c)\frac{s'}{r-s'}\frac{x_k-x_{k-s'}}{s'}+\frac{s'-1}{r-s'+1}\frac{x_k-x_{k-s'+1}}{s-1}+\dots+\frac{1}{r-1}\frac{x_k-x_{k-1}}{1}.\label{eq:sido-type-wts} \end{align} Note that \cref{lemma:sido-type-forest} implies that \[\frac{x_i}{r-k+i}\leq \frac{x_k-x_{k-j}}{j}\] holds for all $i\leq k-s'\leq j$. Thus, to prove \cref{eq:sido-type-wts}, it is sufficient to check \[k-s-1+c\leq (1-c)\frac{s'}{r-s'}+\frac{s'-1}{r-s'+1}+\dots+\frac{1}{r-1}.\] Actually, the equality holds because, by the choice of $c$, we have \begin{align*} k-s-1+c=&(1-c)\frac{r}{r-s'}+\frac{r}{r-s'+1}+\frac{r}{r-s'+2}+\dots+\frac{r}{r-1}-s+c\\ =&(1-c)\frac{s'}{r-s'}+\frac{s'-1}{r-s'+1}+\frac{s'-2}{r-s'+2}+\dots+\frac{1}{r-1}.\qedhere \end{align*} \end{proof} By combining \cref{lemma:sido-type-aux-ineq,claim:sido-type-xk}, we get \[x_1\dots x_{k-1}\leq \left(\frac{k-1}{r}x_k\right)^{k-1}\frac{(r-k+1)\cdots(r-1)}{(k-1)^{k-1}}=\frac{(r-k+1)\cdots(r-1)}{r^{k-1}}=b(K_r^{(k)}).\qedhere\] \end{proof} To give a sense of what the inequality in \cref{thm:Sidorenko-type} means, with some standard computation, we can show the following. If $r,k$ are growing positive integers such that $r = (C+o_{k\to\infty}(1))k$ for some $C\geq 1$, then the largest positive integer $s$ satisfying \cref{eq:krs-relation} is $(C(1-\exp(-C^{-1}))+o_{k\to\infty}(1))k$. In a different regime where $s=k-d$ for some positive integers $d$, we can get that the smallest positive integer $r$ satisfying the inequality is $((2d)^{-1}+o_{k\to\infty}(1))k^2$. We include those computations in the appendix (\cref{prop:linear-r,prop:quadratic-r}). We briefly remark that the threshold $M_k$ Sidorenko deduced on $r$ is the same as ours when $s=k-1$. However, Sidorenko's argument works for a more general family of hypergraphs. It is also possible that by modifying Sidorenko's argument appropriately, we may get a statement analogous to \cref{thm:Sidorenko-type} with the extra parameter $s$. \section{Concluding remarks}\label{sec:conclusion} \subsection{Exact result and stability} In this paper, we mostly focus on the Tur\'an density rather than the Tur\'an number. However, we believe that with more work, it is possible to extract the exact Tur\'an number for sufficiently many vertices from our density Tur\'an theorems \cref{thm:main-tent,thm:Sidorenko-type} at least when we also forbid all homomorphic images. More specifically, we believe that there is a corresponding stability result for \cref{thm:main-tent,thm:Sidorenko-type}, which is usually helpful to deduce the exact Tur\'an number for sufficiently many vertices. Indeed, many exact results were deduced using stability results in a crucial way. For some examples, we refer the readers to \cite{KS05,MP07, Pik08, Pik13, BIJ17, NY17, NY18, LMR23, San24}. \subsection{Other extremizers} All the Tur\'an results we are able to prove in this paper have blowups of $K^{(k)}_r$ as their asymptotic extremizers, and this is not a coincidence. We find it much easier to construct partial forests that would give tight inequalities on the ratio sequences $x_1,\ldots, x_{k}$ with equality holding when $(X_1,\ldots,X_k)$ is a uniform oriented edge in $K^{(k)}_r$. However, as mentioned in the introduction, many difficulties of hypergraph Tur\'an problems come from the potential complicated structures in the extremizers. It would thus be more exciting if our method can be applied to problems with extremizers not as simple as $K^{(k)}_r$. The first step would probably be to extend this to other Tur\'an problems where the extremizers are blowups of some other hypergraphs. Two candidates are the complete bipartite $3$-graph $(A\sqcup B, E)$ where $E = \binom{A}{2}\times B\cup A\times\binom{B}{2}$, and the complete oddly bipartite $k$-graph $(A\sqcup B,E)$ where $k$ is even, and $E$ is the $k$-edges $e$ such that $\abs{A\cap e}$ is odd. Although they are not formally blowups of some smaller hypergraphs, one can think of the complete bipartite $3$-graphs as the blowups of $(\{1,2\},\{\{1,1,2\},\{1,2,2\}\})$, and the completely oddly bipartite $k$-graphs are the blowups of some $2$-vertex ``degenerate'' hypergraphs as well. There are many known Tur\'an results where the two hypergraphs are (asymptotic) extremizers. For example, a classical result of De Caen and F\"uredi \cite{DCF00} shows that the complete bipartite $3$-graph is an asymptotic extremizer for the Fano plane. This was later extended by Mubayi--R\"odl \cite{MR02} and Baber--Talbot \cite{BT12}. On the other hand, Keevash and Sudakov \cite{KS05} showed that the complete oddly bipartite $k$-graph is the extremizer for expanded triangle. A very recent breakthrough of Sankar \cite{San24} showed that the complete oddly bipartite $4$-graph is an asymptotic extremizer for tight cycles of sufficiently large length not divisible by $4$. We are unable to construct any partial forests that give tight inequalities when $G$ is the complete bipartite $3$-graph. For $G$ being complete oddly $k$-graphs, it is possible to construct such partial forests following the argument in \cref{thm:entropic-turan} and Sidorenko's \cite{Sid92} and Frankl's \cite{Fra90} ideas, which used auxiliary $2$-graphs to show that the Tur\'an densities of expanded triangles are $1/2$. However, we have not found any other partial forests that use essentially different ideas. It would be interesting to see if there are ways to obtain tight inequalities for those two candidates of $G$ in the hope that they would give rise to new Tur\'an results. Let us close this discussion by mentioning that our method seems to capture a little structure in the conjectured extremizer for $K_4^{(3)-}$, the $3$-graph on $4$ vertices with $3$ edges. Let $G_1$ be a $3$-graph on $6$ vertices with $10$ edges so that any $2$-subset is in exactly $2$ edges---it turns out that $G_1$ does exist and is unique up to isomorphism. The \emph{iterated blowup} $G_m$ of $G_1$ is constructed inductively by replacing each vertex in $G_1$ with $G_{m-1}$. Then $G_m$ is $K_4^{(3)-}$-free, and by taking $m$ to infinity, we get that $\pi(K_4^{(3)-})\geq \frac{2}{7}$. This is a construction of Frankl and F\"uredi \cite{FF84}, and the construction is conjectured to be optimal. The current best upper bound $\pi(K_4^{(3)-})\leq 0.2871$ is obtained by Baber and Talbot \cite{BT11} using flag algebra. Though we cannot say anything new about the Tur\'an problem of $K_4^{(3)-}$ itself, our method seems to capture some structure in $G_1$. Indeed, by the partial forests $F^{(i)}=([4],\{[3],[4]\backslash \{i\}\})$ for $i=1,2,3$, we can show that if $G$ is $K_4^{(3)-}$-free and $(X_1,X_2,X_3)$ is a random edge with uniform ordering on $G$, then \[x_1 \eqdef 2^{\HH(X_1\mid X_2,X_3)-\HH(X_1)} \leq \frac{1}{3}.\] This is indeed achieved when $(X_1,X_2,X_3)$ is a uniformly chosen oriented edge in $G_1$. \subsection{Entropic spectral radius} In \cref{sec:connection}, we showed that for any $k$-graph $G$, its spectral radius is related to the maximum of $\HH(X_2,\ldots, X_k\mid X_1)$ for symmetric distribution $(X_1,\ldots,X_k)$ on the oriented edges of $G$. It would be interesting if this connection can be utilized to deduce some properties of spectral radius. One possible candidate is a result of Kang, Liu and Shan \cite{KLS18} that showed that \[\rho(G)\geq \left(\frac{1}{v(G)}\sum_{v\in V(G)}\deg(v)^{\frac{k}{k-1}}\right)^{\frac{k-1}{k}}\] for any $k$-graph $G$, where $\rho(G)$ is the spectral radius of $G$. \subsection{Entropic flag algebra} As one may have observed, many upper bounds on Tur\'an densities, especially for those that are still open, were obtained using flag algebra. Such upper bounds using flag algebra, roughly speaking, are obtained via carefully chosen sum-of-squares inequalities, enumeration of possible small configurations, and numerical computationg of positive semidefinite programs. See \cite{Raz13} for a more detailed discussion of the method. The inequalities obtained using our argument seem to be really different from the inequalities obtained by sum-of-squares. This suggests a possibility that maybe the flag algebra bounds can be improved with this new idea and some enumeration of possible partial forests to use in the argument. However, aside from the time complexity enumerating through the possible partial forests, there seem to be several technicalities to overcome for this to work. The first is that in most of our proofs, we need to look at infinitely many partial forests in order to get a tight bound. In addition, the inequalities we get, unlike the ones in flag-algebraic arguments, are highly non-linear. However, if we are just aiming for some numerical upper bound that is close to the truth, then hopefully finite but sufficiently many partial forests together with an approximation of the supremum of $x_1\cdots x_{k-1}$ subject to the inequalities would be enough. The most serious issue is probably that there has not been a framework for automated entropic computation. So far, the flag-algebraic tools are developed to keep track of the homomorphism densities of labeled graphs. Unfortunately, it seems that all our arguments for hypergrpah Tur\'an problems cannot be rephrased using homomorphism densities as we also crucially use the marginal distributions of the random homomorphisms sampled by \cref{lemma:sample-tree}. It would thus be necessary to come up with an ``entropic flag algebra'' framework and implement corresponding software to execute the idea in this subsection. We refer the readers to \cite{CY24-2} for another entropic argument that motivates this idea of ``entropic flag algebra''. \section*{Acknowledgement} The project was motivated when the first author was visiting Hong Liu at Institute for Basic Science, and the first author would like to thank his hospitality. We would also like to thank Ryan Alweiss and Freddie Manners for discussions during the early stage of this project, Dhruv Mubayi and Maya Sankar for pointing us to references for hypergraph Tur\'an problems, Yongtao Li for pointing us to references for spectral Tur\'an problems, and Noga Alon for pointing us to other useful references. Last but not least, we would like to thank Zeev Dvir, Xiaoyu He, Cosmin Pohoata and Maya Sankar for helpful comments on an earlier draft. \bibliographystyle{amsplain0} \bibliography{ref_joints} \appendix \section{Explicit relation between $r,s$ and $k$ in \cref{thm:Sidorenko-type}} In this appendix, we will relate positive integers $k,r,s$ with $k\leq r$ satisfying the inequality \begin{equation}\label{eq:appendix-rks} k-s\geq \sum_{i=1}^{s-1}\frac{i}{r-i}. \end{equation} We first compute the right hand side. \begin{lemma}\label{lemma:integral-approx} Suppose that $k,r,s$ are positive integers satisfying \cref{eq:appendix-rks}. Then $r(k-s) = \Omega(s^2)$, $r-s = \Omega(r)$ and \[\sum_{i=1}^{s-1}\frac{i}{r-i} = r\log\left(\frac{r-1}{r-s}\right)-(s-1)+O\left(\frac{s}{r}\right).\] \end{lemma} \begin{proof} We first show $r(k-s)=\Omega(s^2)$. This is clear as \[k-s\geq \sum_{i=1}^{s-1}\frac{i}{r-i}\geq \left\lfloor \frac{s-1}{2}\right\rfloor \frac{\left\lceil \frac{s-1}{2}\right\rceil}{r-\left\lceil \frac{s-1}{2}\right\rceil}=\Omega(s^2r^{-1}).\] Now we show that $r-s = \Omega(r)$. This is clear when $r\geq 2k$, so it suffices to check the case when $r<2k$. In this case, we have $2k(k-s)>r(k-s)\geq \Omega(s^2)$. This forces $s \leq ck$ for some constant $c<1$, and so $r-s=\Omega(r)$ as $s<k\leq r$. Now let $\cE$ be the error term defined by \[\cE =\sum_{i=1}^{s-1}\frac{i}{r-i}-\int_{1}^{s}\frac{x}{r-x}\textup{ d}x = \int_1^s\left(f(\lfloor x\rfloor)-f(x)\right)\textup{ d}x\] where we set $f(x) = x/(r-x)$. Note that $f'(x) = r(r-x)^{-2}$ is positive and increasing in $x$ when $x\in [1,s]\subseteq [1,r-1]$. Therefore \[0\geq f(\lfloor x\rfloor)-f(x)\geq (\lfloor x\rfloor -x)f'(x)>-\frac{r}{(r-x)^2}\] for any $x\in [1,s]$. This shows that \[0\geq \cE \geq -\frac{(s-1)r}{(r-s)^2},\] which shows that $\cE = O(s/r)$. Therefore \[\sum_{i=1}^{s-1}\frac{i}{r-i} =\int_{1}^{s}\frac{x}{r-x}\textup{ d}x+O\left(\frac{s}{r}\right)= r\log\left(\frac{r-1}{r-s}\right)-(s-1)+O\left(\frac{s}{r}\right).\qedhere\] \end{proof} \begin{proposition}\label{prop:linear-r} Let $r\geq k$ be a positive integer growing with $k$ so that $r = (C+o_{k\to\infty}(1))k$ for some constant $C\geq 1$. Then the largest positive integer $s$ satisfying \cref{eq:appendix-rks} also satisfies \linebreak$s=C(1-\exp(-C^{-1})+o_{k\to\infty}(1))k$. \end{proposition} \begin{proof} By the choice of $s$, we know \[k-s\geq \sum_{i=1}^{s-1}\frac{i}{r-i}\] and \[k-(s-1)< \sum_{i=1}^{s-2}\frac{i}{r-i}.\] Therefore \[k-s+O(1) = \sum_{i=1}^{s-1}\frac{i}{r-i}+O\left(\frac{s}{r-s}\right).\] By \cref{lemma:integral-approx}, we know that this implies \[k-s +O(1) = r\log\left(\frac{r-1}{r-s}\right)-(s-1)+O\left(\frac{s}{r}\right).\] Rearranging, we get \[\frac{r-1}{r-s} = \exp\left(\frac{k-1}{r}+O(r^{-1})\right),\] and so \begin{align*} s =& 1+(r-1)\left(1-\exp\left(-\frac{k-1}{r}+O(r^{-1})\right)\right)\\ =& 1+(C+o_{k\to\infty}(1))k \cdot \left(1-\exp\left(-C^{-1}+o_{k\to\infty}(1)\right)\right), \end{align*} where we use the fact that $r^{-1} = O(k^{-1})$. The desired statement thus follows. \end{proof} \begin{proposition}\label{prop:quadratic-r} Let $k,d$ be positive integers with $d<k$, and let $s=k-d$. Then the smallest positive integer $r$ satisfying \cref{eq:appendix-rks} also satisfies $r=(\frac{1}{2d}+o_{k\to\infty}(1))k^2$. \end{proposition} \begin{proof} By the choice of $r$, we also have \[d\geq \sum_{i=1}^{s-1}\frac{i}{r-i}\] and \[d<\sum_{i=1}^{s-1}\frac{i}{r-1-i}.\] Note that \[\frac{i}{r-i}-\frac{i}{r-1-i} = O(ir^{-2})\] for every $i\leq s-1$ as we know that $r-s = \Omega(r)$ by \cref{lemma:integral-approx}. Therefore \[d = \sum_{i=1}^{s-1}\frac{i}{r-i}+O(s^2r^{-2}).\] By \cref{lemma:integral-approx}, we know that $r = \Omega(d^{-1}s^2) = \Omega(s^2)$. Therefore by \cref{lemma:integral-approx}, \[d = r\log\left(\frac{r-1}{r-s}\right)-(s-1)+O\left(\frac{s}{r}\right)=r\log\left(\frac{r-1}{r-s}\right)-(s-1)+o_{k\to\infty}(1).\] Note that \begin{align*} r\log\left(\frac{r-1}{r-s}\right)=&r\log\left(1+\frac{s-1}{r}+\frac{s(s-1)}{r^2}+O\left(\frac{s^3}{r^3}\right)\right)\\ =&r\left(\frac{s-1}{r}+\frac{(s+1)(s-1)}{2r^2}+O\left(\frac{s^3}{r^3}\right)\right)\\ =&s-1+\frac{s^2}{2r}+o_{k\to\infty}(1). \end{align*} Therefore we get that \[\frac{r}{k^2} = \left(1+o_{k\to\infty}(1)\right)\frac{r}{s^2}= \frac{1}{2d}+o_{k\to\infty}(1),\] as desired. \end{proof} \end{document}
2412.08183v1
http://arxiv.org/abs/2412.08183v1
States and IR divergences in factorization algebras
\documentclass[preprint]{ptephy_v1} \preprintnumber{YITP-24-168, KUNS-3030} \usepackage{hyperref} \usepackage{bm} \usepackage[all]{xy} \usepackage{tcolorbox} \usepackage{tcolorbox} \tcbuselibrary{breakable, skins, theorems} \usepackage{mathrsfs} \newtheorem{theorem}{Theorem} \newtheorem{condition}{Condition} \newtheorem{definition}{Definition} \newtheorem{remark}{Remark} \newtheorem*{theorem*}{Theorem} \newtheorem{conjecture}{Conjecture} \numberwithin{equation}{subsection} \numberwithin{definition}{subsection} \numberwithin{theorem}{subsection} \numberwithin{remark}{subsection} \numberwithin{conjecture}{subsection} \begin{document} \title{States and IR divergences in factorization algebras} \author{Masashi Kawahira} \affil{Yukawa Institute for Theoretical Physics, Kyoto University, Kyoto 606-8502 Japan \email{[email protected]}} \author[2]{Tomohiro Shigemura} \affil{Department of Physics, Kyoto University, Kyoto 606-8502, Japan \email{[email protected]}} \begin{abstract}There are several choices of states. In factorization algebras, we often use a natural augmentation state $\langle-\rangle_{\rm aug}$. In physics, we use a state given by compactification of spacetime $\langle-\rangle_{\rm cptf}$, or a state corresponding Schwartz boundary condition $\langle-\rangle_{\rm Sch}$. At first glance, the relation between these three states is not so clear. In this paper, propose a definition of compactification method in factorization algebras and give a way to treat IR divergences in massless theory. As a result, we see the three sates are equivalent both in massive theory and massless theory. \end{abstract} \subjectindex{A12, A13, B05, B33 B34, B39} \maketitle \section{Introduction} Quantum field theory (QFT) is a central topic in theoretical physics. This is useful to explain various kinds of phenomena. And also, people have discovered the various mathematical conjectures by using QFTs. Nevertheless we have not obtain the complete understandings of quantum field theories. In the last ten years, Kevin Costello and Owen Gwilliam have developed a new formulation based on factorization algebras\cite{Costello:2016vjw}\cite{Costello:2021jvx}. This formulation works well in perturbative field theories. The advantage is that it make clear a lot of concepts of QFTs like path-integral or operator products. In this paper, we aim to make clear the concepts of IR divergences in massless free scalar theories. Since long ago, physicists have been studying IR divergences \cite{Bloch:1937pw}. And recently some physicists pay attention to it again\cite{Strominger:2017zoo}. Interestingly by using factorization algebras, we can treat the concepts of IR divergences {\it without any divergences}. In order to discuss IR divergences in a mathematical way, we sort the constructions of states in factorization algebras. Especially, we consider three states. We name them as {\it a natural augmentation state, compactification state and Schwartz state}. This paper is organized as follows. In section 2, we will see the goal of this paper. In section 3, we will review the formulation by Costello and Gwilliam. In section 4, we will review a natural augmentation state. In section 5, we will give a compactification state and a treatment of IR divergences. In section 6, we will review a Schwartz state for massive case and give a Schwartz state for massless case and a treatment of IR divergences. In section 7, we will prove the equivalence of the above sates. Section 8 is devoted for conclusion and discussion. \section{What will we see ?} In this paper, we consider observables in some region $U$ of $\mathbb{R}^d$, which we denote $\mathcal{O}_U\in{\rm Obs}(U):={\rm Sym}(C_{\rm c}^\infty(U))$. In factorization algebras, by taking the cohomology and states we obtain expectation values\footnote{In later section, we will give definitions of the cohomology and the state.}. \begin{align} H^0{\rm Obs}^{\rm cl}(U)\ni [\mathcal{O}_U]_U\mapsto \langle [\mathcal{O}_U]_U\rangle \in\mathbb{C} \end{align} We will explain the following three states. \begin{itemize} \item Natural augmentation state $\langle-\rangle_{\rm aug}$ \item Schwartz state $\langle-\rangle_{\rm Sch}$ \item Compactification state $\langle-\rangle_{\rm cptf}$ \end{itemize} \noindent{}$\langle-\rangle_{\rm aug}$ and $\langle-\rangle_{\rm Sch}$ are introduced by Costtelo-Gwilliam\cite{Costello:2016vjw}, and they showed that these are equivalent. Note that natural augmentation state $\langle-\rangle_{\rm aug}$ is well-defined in massive and massless case, while Schwartz state is defined only for massive case. This is essentially from IR divergence. In this paper, we will see how to get rid of IR divergence and define Schwartz state in massless case. And we will define compactification state $\langle-\rangle_{\rm cptf}$ which represents compactification-notion of quantum field theories. This is well-defined in massive case. In massless case, we need to remove IR divergence in order to define $\langle-\rangle_{\rm cptf}$. In addition, We will prove that three states are equivalent both in massive case and massless case. \section{Brief review of factorization algebra of free real scalar theory} \subsection{Observable algebra} In this paper, we would like to discuss path-integral-formulation in mathematical manner. In contrast to operator formulation, obeservables are not operators but ``functionals with compact support." In order to consider it, we define observable algebra as follows.\footnote{Precisely, we need to take a completion in order to make ${\rm Obs}(U)$ a topological vector space. However we omit the discussion. If you are interested in it, check \cite{Costello:2016vjw}.\label{ref:completion}} \begin{tcolorbox}[colframe=red,colback=red!3!] \begin{definition}{\rm (K. Costello, O. Gwilliam \cite{Costello:2016vjw})}\\ Observable algebra ${\rm Obs}(U)$ is defined as \begin{align} {\rm Obs}(U):={\rm Sym}(C_{\rm c}^\infty(U,\mathbb{C})) \end{align} and the elements are called observables on $U$. \end{definition} \end{tcolorbox} Motivation of the above definition is that observables can be regarded as a polynomial of functions with compact supports: $\mathcal{O}=c+f+f_1*f_2+\cdots$, where $*$ is a formal symmetric product, $c\in\mathbb{C}$ and $f,f_1,f_2,\cdots\in C_{\rm c}^\infty(U,\mathbb{C})$. And $\mathcal{O}$ acts $\Phi$ as \begin{align} \mathcal{O}:\Phi\mapsto c +\int_M f\Phi +\int_M f_1\Phi \int_M f_2\Phi +\cdots \end{align} \subsection{Classical derived observable space} In order to perform ``path-integral," we introduce derived observable algebras. Roughly, \textit{derived} means that we will add a concept of \textit{degree} to the observable space ${\rm Obs}(U)$. This formulation is originally given by Batalin and Vilkovisky\cite{Batalin:1981jr}\cite{Batalin:1983ggl}. \begin{align} &C_{\rm c}^\infty(U)^{0}:=(C_{\rm c}^\infty(U,\mathbb{C}),0),\\ &C_{\rm c}^\infty(U)^{-1}:=(C_{\rm c}^\infty(U,\mathbb{C}),-1). \end{align} The first one is a \textit{degree-0 linear observable}, and the second one is a \textit{degree-$(-1)$ linear observable} which corresponds to \textit{anti-field} in physics literature. For simplicity, we denote $(f,0)\in C_{\rm c}^\infty(U)^{0}$ and $(f,-1)\in C_{\rm c}^\infty(U)^{-1}$ as $f$ and $f^\star$. The symmetric product $*$ is defined for them as \begin{align} a*b=(-1)^{|a||b|}b*a\label{eq:*}. \end{align} And when we have degree $+1$ operator $\Delta^{\rm cl}$, we can define a classical derived observable algebra. \begin{tcolorbox}[colframe=red,colback=red!3!] \begin{definition}{\rm (K. Costello, O. Gwilliam \cite{Costello:2016vjw})}\\ Classical derived observable algebra ${\rm Obs}^{\rm cl}(U)$ is defined as \begin{align} {\rm Obs}^{\rm cl}(U) := {\rm Sym}\left(C_{\rm c}^\infty(U)^{-1}\xrightarrow{\Delta^{\rm cl}} C_{\rm c}^\infty(U)^{0}\right) \end{align} where $\Delta^{\rm cl}$ is a classical Batalin-Vilkovisky operator which is defined in Definition \ref{def:cl_BV_op}. \end{definition} \end{tcolorbox} \noindent{}The concrete form of $\Delta^{\rm cl}$ depends on what theory we want. In this paper, we are interested in free theory, thus we will defined as follows. \begin{tcolorbox}[colframe=red,colback=red!3!] \begin{definition}{\rm (K. Costello, O. Gwilliam \cite{Costello:2016vjw})}\label{def:cl_BV_op}\\ Classical Batalin-Vilkovisky operator $\Delta^{\rm cl}$ is a map \begin{align} C_{\rm c}^\infty(U)^{-1}\ni f^\star\mapsto -(-\Delta+m^2)f\in C_{\rm c}^\infty(U)^{0}. \end{align} Hence $\Delta^{\rm cl}$ has a degree $+1$. \end{definition} \end{tcolorbox} By (\ref{eq:*}), we can rewrite classical derived observable space ${\rm Obs}^{\rm cl}(U)$. \begin{align} &{\rm Obs}^{\rm cl}(U)= \notag\\ &\bigg(\cdots \xrightarrow{\Delta^{\rm cl}}\bigwedge^2 C_{\rm c}^\infty(U)^{-1}* {\rm Sym}\left(C_{\rm c}^\infty(U)^{0}\right) \xrightarrow{\Delta^{\rm cl}}C_{\rm c}^\infty(U)^{-1}* {\rm Sym}\left(C_{\rm c}^\infty(U)^{0}\right) \xrightarrow{\Delta^{\rm cl}}{\rm Sym}\left(C_{\rm c}^\infty(U)^{0}\right)\bigg) \end{align} This is a chain complex. It is called \textit{classical Batalin–Vilkovisky complex}. The cohomology of classical Batalin–Vilkovisky complex is called \textit{classical Batalin–Vilkovisky cohomology}: \begin{align} H^*\left({\rm Obs}^{\rm cl}(U)\right). \end{align} Let us consider the physical meanings of the cohomology $H^0\left({\rm Obs}^{\rm cl}(U)\right)$. We take a 0-degree observables $\mathcal{O}_1,\mathcal{O}_2$ and assume that these are same in the cohomology, i.e., $\exists X\ {\rm s.t.}$ \begin{align} \mathcal{O}_2-\mathcal{O}_1 = \Delta^{\rm cl}X. \end{align} Let $\Phi_{\rm cl}$ be a solution of the equation of motion $(-\Delta+m^2)\Phi=0$\footnote{If you want a theory with some interactions, we need to add some terms to $\Delta^{\rm cl}$.}. We have \begin{align} \mathcal{O}_2(\Phi_{\rm cl})-\mathcal{O}_1(\Phi_{\rm cl}) &= \Delta^{\rm cl}X(\Phi_{\rm cl})\notag\\ &= 0. \end{align} Hence we can see $H^0\left({\rm Obs}^{\rm cl}(U)\right)$ as the on-shell evaluation of observables. However $[\mathcal{O}_1](=[\mathcal{O}_2])$ is not a number. Then we define a map called \textit{state} $\langle-\rangle$. \begin{tcolorbox}[colframe=red,colback=red!3!] \begin{definition}{\rm (K. Costello, O. Gwilliam \cite{Costello:2016vjw})}\\ A state $\langle-\rangle$ is a smooth map: \begin{align} \langle-\rangle: H^0\left({\rm Obs}^{\rm cl}(M)\right) \to \mathbb{C}. \end{align} \end{definition} \end{tcolorbox} \section{Natural augmentation state} \subsection{Massive and massless case} By definition, ${\rm Obs}^{\rm cl}(\mathbb{R}^d)$ has a natural augmentation map. Costello-Gwilliam use it to define a state. \begin{tcolorbox}[colframe=red,colback=red!3!] \begin{definition} {\rm (K. Costello, O. Gwilliam \cite{Costello:2016vjw})}\\ We have a projection: \begin{align} \begin{array}{ccc} {\rm Obs}^{\rm cl}(\mathbb{R}^d)^{0} &\longrightarrow &\mathbb{C}\\ \rotatebox{90}{$\in$} & &\rotatebox{90}{$\in$}\\ c+f+f_1*f_2+\cdots &\longmapsto &c \end{array} \end{align} where $c\in\mathbb{R}$ and $f,f_1,f_2,\cdots\in C_{\rm c}^\infty(U)$. This induces \begin{align} \langle-\rangle_{\rm aug}: H^0({\rm Obs}^{\rm cl}(\mathbb{R}^d)) \to \mathbb{C}. \end{align} We call it a natural augmentation state. \end{definition} \end{tcolorbox} The cohomology is \begin{align} H^0({\rm Obs}^{\rm cl}(\mathbb{R}^d)) = \frac {{\rm Obs}^{\rm cl}(\mathbb{R}^d))^{0}} {\Delta^{\rm cl} ({\rm Obs}^{\rm cl}(\mathbb{R}^d)^{-1})}. \end{align} Hence, to check the well-definedness of $\langle-\rangle_{\rm aug}$, we see for any $X\in {\rm Obs}^{\rm cl}(\mathbb{R}^d)^{-1}$, \begin{align} \Delta^{\rm cl}X \notin \mathbb{C} \subset {\rm Obs}^{\rm cl}(\mathbb{R}^d)^{0}. \end{align} Write $X$ as $P*f^\star$ where $P$ is in ${\rm Sym}(C_{\rm c}^\infty(\mathbb{R}^d)^0)$ and $f^\star$ is in $C_{\rm c}^\infty(\mathbb{R}^d)^{-1}$. \begin{align} \Delta^{\rm cl}X = P*(\Delta^{\rm cl}f^\star) \end{align} In order that $\Delta^{\rm cl}X$ sits in $\mathbb{C}$, both $P$ and $\Delta^{\rm cl}f^\star$ must be constants. However, by definition \begin{align} \Delta^{\rm cl}f^\star \in C_{\rm c}^\infty(\mathbb{R}^d)^0. \end{align} Then it can not be a constant. Thus $\langle-\rangle_{\rm aug}$ is well-defined. \section{Compactification state} \subsection{Massive case} In order to consider compactification, locality plays an essential role. Hence we assume the measure of $U$ is finite: $\mu(U)\ll \infty$. We call it {\it locality condition}. Under this condition, we have an inclusion map \begin{align} i:{\rm Obs}^{\rm cl}(U)\to {\rm Obs}^{\rm cl}(T^d) \end{align} and \begin{align} i:H^*{\rm Obs}^{\rm cl}(U)\to H^*{\rm Obs}^{\rm cl}(T^d) \end{align} for sufficiently large torus $T^d$. Strictly speaking, we need to distinguish the above two maps, but we use the same letter $i$ for simplicity. \begin{tcolorbox}[colframe=blue,colback=blue!3!] \begin{theorem} {\rm (K. Costello, O. Gwilliam \cite{Costello:2016vjw})}\\ In massive case, \begin{align} H^n{\rm Obs}^{\rm cl}(T^d) = \left\{ \begin{array}{ll} \mathbb{C} & (n=0) \\ 0 & ({\rm otherwise}) \end{array} \right. \end{align} \end{theorem} \end{tcolorbox} \begin{proof} \begin{align} A:= \left(C_{\rm c}^\infty\left(T^d\right)^{-1}\xrightarrow{\Delta^{\rm cl}}C_{\rm c}^\infty\left(T^d\right)^0\right) \end{align} This is an isomorphism. Thus $H^*(A)=0$, then $H^0({\rm Sym}(A))=\mathbb{C}$ and $H^{n\le -1}({\rm Sym}(A))=0$ \end{proof} Then $i$ gives a state $\langle-\rangle_{\rm cptf}$. \footnote{${\rm cptf}$ means a ``compactification."} \begin{tcolorbox}[colframe=red,colback=red!3!] \begin{definition}\ \\ A compactification state $\langle-\rangle_{\rm cptf}$ is a smooth map: \begin{align} \langle-\rangle_{\rm cptf}: H^0{\rm Obs}^{\rm cl}(U) \to H^0{\rm Obs}^{\rm cl}(T^d)=\mathbb{C}. \end{align} \end{definition} \end{tcolorbox} \subsection{Massless case} \begin{tcolorbox}[colframe=blue,colback=blue!3!] \begin{theorem}\ \\ In massless case, \begin{align} H^*{\rm Obs}^{\rm cl}(T^d) = \mathbb{C}[q,r]/r^2 \end{align} where $q$ is a generator with degree $0$ and $r$ is a generator with degree $-1$. \end{theorem} \end{tcolorbox} \begin{proof} We show a quasi isomorphism: \begin{align} A\xrightarrow{\pi} B \end{align} where \begin{align} A=\left(C_{\rm c}^\infty\left(T^d\right)^{-1}\xrightarrow{\Delta^{\rm BV}}C_{\rm c}^\infty\left(T^d\right)^0\right),\ B=(\mathbb{C}\xrightarrow{0}\mathbb{C}). \end{align} We denote the basis of $\mathbb{R}$ as $r,q$. First of all, we will see the following commutative diagram: \begin{align} \xymatrix{ C_{\rm c}^\infty\left(T^d\right)^{-1} \ar[r]^{\Delta^{\rm cl}} \ar[d]_{\pi^{-1}} & C_{\rm c}^\infty\left(T^d\right)^0 \ar[d]_{\pi^0} \\ \mathbb{C} \ar[r]^{0} & \mathbb{C} } \end{align} where $\pi^{-1}$ is defined as \begin{align} \pi^{-1}(f^{\star}) := r\int_{T^d}{\rm d}x_1,\cdots,x_d\ f(x_1,\cdots,x_d) = r\int_{0}^{2\pi} {\rm d}x_1 \cdots \int_{0}^{2\pi} {\rm d}x_d \ f(x_1,\cdots,x_d) \end{align} for $f^{\star}\in C_{\rm c}^\infty\left(T^d\right)^{-1}$, and $\pi^{0}$ is defined as \begin{align} \pi^{0}(g) := q\int_{T^d}{\rm d}x_1\cdots{\rm d}x_d\ g(x_1,\cdots,x_d) = q\int_{0}^{2\pi} {\rm d}x_1 \cdots \int_{0}^{2\pi} {\rm d}x_d \ g(x_1,\cdots,x_d) \end{align} for $g\in C_{\rm c}^\infty\left(T^d\right)^0$. We can easily check that $\pi^0(g)=0$ holds if $g=\Delta^{\rm cl}f^{\star}.$ Second, we will see $H^0(A)=H^0(B)$. By definition, \begin{align} H^0(A) = \frac{C_{\rm c}^\infty(I)^0} {{\rm im}(\Delta^{\rm cl})}. \end{align} We will show that ${\rm im}(\Delta^{\rm cl})={\rm ker}(\pi^0)$. If it holds, $H^0(A)=H^0(B)$. Obviously, \begin{align} {\rm im}(\Delta^{\rm cl})\subset {\rm ker}(\pi^0). \end{align} Then we will check ${\rm im}(\Delta^{\rm cl})\supset {\rm ker}(\pi^0)$. Take $f\in {\rm ker}(\pi^0)$. $f$ satisfies \begin{align} \int_{T^d} {\rm d}x_1,\cdots,x_d\ f(x_1,\cdots,x_d)=0.\label{eq:int=0} \end{align} Then we define functions $\tilde{f},\tilde{\tilde{f}}$ as \begin{align} &\tilde{f}(x_1,\cdots,x_d) := \int_0^{x_1} {\rm d}y_1\cdots\int_0^{x_d} {\rm d}y_d \ f(y_1,\cdots,y_d),\\ &\tilde{\tilde{f}}(x_1,\cdots,x_d) := \tilde{f}(x_1,\cdots,x_d)-\frac{1}{(2\pi)^d}\int_{T^d} {\rm d}y_1\cdots{\rm d}y_d\ \tilde{f}(y_1,\cdots,y_d) \end{align} $\tilde{f}$ and $\tilde{\tilde{f}}(x)$ are in $C_{\rm c}^\infty(T^d)$ because of (\ref{eq:int=0}). And we have \begin{align} \int_{S^1}{\rm d}x\ \tilde{\tilde{f}}(x)=0 \label{eq:int_tilde=0} \end{align} Then we define \begin{align} F(x_1,\cdots,x_d) := \int_0^{x_1} {\rm d}y^1 \cdots \int_0^{x_d} {\rm d}y^d \ \tilde{\tilde{f}}(y_1,\cdots,y_d) \end{align} $F$ is in $C_{\rm c}^\infty(T^d)$ because of (\ref{eq:int=0}). We can see $\Delta^{\rm cl}F^\star=f$, thus $f\in {\rm im}(\Delta^{\rm cl})$ and ${\rm im}(\Delta^{\rm BV})\supset {\rm ker}(\pi^0)$. Third, we will see $H^{-1}(A)=H^{-1}(B)$. Clearly $H^{-1}(B)=\mathbb{C}$. Then let us think of \begin{align} H^{-1}(A)={\rm ker}(\Delta^{\rm cl}). \end{align} This is $\mathbb{C}$, since non-trivial solutions of \begin{align} \Delta f=0 \end{align} is $f={\rm const}\in C_{\rm c}^\infty(T^d)$. And $\pi^{-1}$ gives just a $(2\pi)^d$-multiplication. Then $H^{-1}(A)=H^{-1}(B)=\mathbb{C}$. \end{proof} Hence $i$ does NOT give a state. \begin{align} i: H^0{\rm Obs}^{\rm cl}(U) \to H^0{\rm Obs}^{\rm cl}(T^d) = \mathbb{R}[q] \end{align} However by sending $q$ to $0$, we obtain a state. \begin{tcolorbox}[colframe=red,colback=red!3!] \begin{definition}\ \\ A compactification state $\langle-\rangle_{\rm cptf}$ is defined as $j\circ i$ where $j$ sends $q$ to $0$. \begin{align} \langle-\rangle_{\rm cptf}: H^0{\rm Obs}^{\rm cl}(U) \xrightarrow{i} \mathbb{C}[q] \xrightarrow{j} \mathbb{C} \end{align} where $\mu(U)\ll\infty$. \end{definition} \end{tcolorbox} \subsection{Why does $j$ mean removal of IR divergence ?}\label{subsec:why_j_?} In order to see physical meaning of $j$, we review the following theorem in one-dimensional system. \begin{tcolorbox}[colframe=blue,colback=blue!3!] \begin{theorem}\label{thm:M=I_cl} {\rm (K. Costello, O. Gwilliam \cite{Costello:2016vjw})}\\ If $I\subset \mathbb{R}$ is an interval, \begin{align} H^*\left({\rm Obs}^{\rm cl}(I)\right) = \left\{ \begin{array}{ll} \mathbb{C}[q,p] & (n=0) \\ 0 & ({\rm otherwise}) \end{array} \right. \end{align} where $q,p$ has degree 0. \end{theorem} \end{tcolorbox} \begin{proof} We show a quasi-isomorphism: \begin{align} A\sim B \end{align} where \begin{align} A=\left(C_{\rm c}^\infty(I)^{-1}\xrightarrow{\Delta^{\rm cl}}C_{\rm c}^\infty(I)^0\right),\ B=(0\to\mathbb{R}^2). \end{align} $\mathbb{C}^2$ sits in degree 0. And we denote the basis of $\mathbb{C}^2$ as $q,p$. The cohomology of ${\rm Sym}(A)$ and ${\rm Sym}(B)$ are $H^*({\rm Obs}^{\rm cl}(I))$ and $\mathbb{C}[q,p]$ respectively. First of all, we will see the following commutative diagram: \begin{align} \xymatrix{ C_{\rm c}^\infty(I)^{-1} \ar[r] \ar[d]_{\pi^{-1}} & C_{\rm c}^\infty(I)^0 \ar[d]_{\pi^0} \\ 0 \ar[r] & \mathbb{C}^2 } \end{align} where $\pi^0$ is defined as \begin{align} \pi^0(g) :=q\int_I {\rm d}x\ g(x)\phi_q(x)+p\int_I {\rm d}x\ g(x)\phi_p(x) \end{align} for $g\in C_{\rm c}^\infty(I)^0$. The definition of $\phi_q,\phi_p\in C^\infty(\mathbb{R})$ is as follows. In case that $m>0$ we define $\phi_q,\phi_p\in C^\infty(\mathbb{R})$ as \begin{align} \phi_q(x)=\frac{1}{2}(e^{mx}+e^{-mx}),\ \phi_p(x)=\frac{1}{2m}(e^{mx}-e^{-mx}). \end{align} They form the kernel of $-\Delta+m^2$. If $m=0$, we define \begin{align} \phi_q(x)=1,\ \phi_p(x)=x. \end{align} We note that $\phi_p'(x)=\phi_q(x)$. We can easily check that $\pi^0(g)=0$ holds if $g=\Delta^{\rm cl}f^{\star}.$ Next, we will see $H^0(A)=H^0(B)$. By definition, \begin{align} H^0(A) = \frac{C_{\rm c}^\infty(I)^0} {{\rm im}(\Delta^{\rm cl})}. \end{align} We will show that ${\rm im}(\Delta^{\rm cl})={\rm ker}(\pi^0)$. If it holds, $H^0(A)=H^0(B)$. Obviously, \begin{align} {\rm im}(\Delta^{\rm cl})\subset {\rm ker}(\pi^0). \end{align} Then we will check ${\rm im}(\Delta^{\rm cl})\supset {\rm ker}(\pi^0)$. Take $f\in {\rm ker}(\pi^0)$. $f$ satisfies \begin{align} &\int_I {\rm d}x\ f(x)e^{mx}=0\ {\rm and} \int_I {\rm d}x\ f(x)e^{-mx}=0\ \ ({\rm for\ massive\ case}),\notag\\ &\int_I {\rm d}x\ f(x)x=0\ {\rm and} \int_I {\rm d}x\ f(x)=0\ \ ({\rm for\ massless\ case}) \label{eq:integral_of_f}, \end{align} because $\int_I {\rm d}x f(x)\phi_p(x)=0$ and $\int_I {\rm d}x f(x)\phi_q(x)=0$. Let $G\in C^0(\mathbb{R})$ be the Green function: \begin{align} &G(x)=\frac{1}{2m}e^{-m|x|}\ \ ({\rm for\ massive\ case}),\notag\\ &G(x)=-\frac{1}{2}|x|\ \ ({\rm for\ massless\ case}). \end{align} Then the convolution of $f$ and $G$ is \begin{align} (G\cdot f)(x):=\int_I {\rm d}y\ G(x-y)f(y). \end{align} This is in $C_{\rm c}^\infty(I)^0$ because of (\ref{eq:integral_of_f}). For $(G\cdot f)^\star\in C_{\rm c}^\infty(I)^{-1}$, \begin{align} f=\Delta^{\rm cl}(G\cdot f)^\star. \end{align} Thus $f\in {\rm im}(\Delta^{\rm cl})$, and ${\rm im}(\Delta^{\rm cl})\supset {\rm ker}(\pi^0)$. Finally, we will see $H^{-1}(A)=H^{-1}(B)$. Clearly $H^{-1}(B)=0$. Then let us think of \begin{align} H^{-1}(A)={\rm ker}(\Delta^{\rm cl}). \end{align} This is trivial since there are not non-trivial solutions of \begin{align} (-\Delta+m^2)f=0 \end{align} for $f\in C_{\rm c}^\infty(I)$. Then $H^{-1}(A)=H^{-1}(B)=0$. \end{proof} Focus on massless case. The physical meaning of $j$ is to remove the IR divergences. The origin of the generator $q$ is a constant field configuration $\phi_q(x)=1$ in the definition of $\pi^0$, in other words a long wave mode. ($\phi_p(x)$ does not live in $T^{d=1}$.) Since massive theory does not have long wave mode, we can determine the expectation values $\langle-\rangle_{\rm cptf}$ directly. On the other hand, massless theory has a long wave mode, and it occurs IR divergences. In path-integral calculation, we need to get rid of a long wave mode to decide the expectation values. Sending $q$ to $0$ is the same procedure. \section{Schwartz state} \subsection{Massive case} Although the notion of compactification state is clear, we need the locality condition $\mu(U)\ll\infty$ to define it. This condition is well-behaved in the case of local obseravables. However extended observables might violate locality condition. Schwartz state is well-defined without locality condition then this is good to apply to gauge theory which has extended observables like Wilson loop. Functions with compact supports are Schwartz functions. Then we have \begin{align} i:C_{\rm c}^\infty(\mathbb{R}^d) \hookrightarrow \mathcal{S}(\mathbb{R}^d) \end{align} where $\mathcal{S}(\mathbb{R}^d)$ is Schwartz space. We define \begin{align} {\rm Obs}^{\rm cl}_{\mathcal{S}}(\mathbb{R}^d) := {\rm Sym} \left( \mathcal{S}(\mathbb{R}^d)^{-1} \xrightarrow{\Delta^{\rm cl}} \mathcal{S}(\mathbb{R}^d)^{0} \right). \end{align} Then we have \begin{align} i: {\rm Obs}^{\rm cl}(\mathbb{R}^d) \to {\rm Obs}^{\rm cl}_{\mathcal{S}}(\mathbb{R}^d) \end{align} and \begin{align} i: H^*{\rm Obs}^{\rm cl}(\mathbb{R}^d) \to H^*{\rm Obs}^{\rm cl}_{\mathcal{S}}(\mathbb{R}^d). \end{align} Actually $i$ gives a state. To see this, we introduce the following theorem. \begin{tcolorbox}[colframe=blue,colback=blue!3!] \begin{theorem} {\rm (K. Costello, O. Gwilliam \cite{Costello:2016vjw})}\\ In massive case, \begin{align} H^n{\rm Obs}^{\rm cl}_{\mathcal{S}}(\mathbb{R}^d) = \left\{ \begin{array}{ll} \mathbb{C} & (n=0) \\ 0 & ({\rm otherwise}) \end{array} \right. \end{align} \end{theorem} \end{tcolorbox} \begin{proof} \begin{align} A:= \left(\mathcal{S} (\mathbb{R}^d)^{-1} \xrightarrow{\Delta^{\rm cl}} \mathcal{S} (\mathbb{R}^d)^0\right) \end{align} This is an isomorphism. To see this, we consider Fourier transformation: \begin{align} \begin{array}{rccc} &\mathcal{S}(\mathbb{R}^d)&\longrightarrow& \mathcal{S}(\mathbb{R}^d)\\ & \rotatebox{90}{$\in$}& & \rotatebox{90}{$\in$} \\ & f & \longmapsto & \hat{f} \end{array} \end{align} where \begin{align} \hat f(k_1,\cdots,k_d) := \int_{\mathbb{R}^d} f(x_1,\cdots,x_n)e^{ik\cdot x} {\rm d}x. \end{align} Fourier transformation gives an isomorphism $\mathcal{S}(\mathbb{R}^d)\stackrel{\sim}{\longrightarrow}\mathcal{S}(\mathbb{R}^d)$. Then we have \begin{align} \hat{A} := \left(\mathcal{S} (\mathbb{R}^d)^{-1} \xrightarrow{\hat\Delta^{\rm cl}} \mathcal{S} (\mathbb{R}^d)^0\right) \end{align} where $(\hat\Delta^{\rm cl} \hat f)(k)=-(k^2+m^2)f(k)$. Since it is massive, $\hat{\Delta}^{\rm cl}$ is an isomorphism. Then $\Delta^{\rm cl}$ is also an isomorphism. Thus $H^*(A)=0$, then $H^0({\rm Sym}(A))=\mathbb{C}$ and $H^{n\le -1}({\rm Sym}(A))=0$ \end{proof} Then $i$ gives a state $\langle-\rangle_{\rm Sch}$. \footnote{${\rm cptf}$ means a ``compactification."} \begin{tcolorbox}[colframe=red,colback=red!3!] \begin{definition} {\rm (K. Costello, O. Gwilliam \cite{Costello:2016vjw})}\\ A Schwartz state $\langle-\rangle_{\rm Sch}$ is a smooth map: \begin{align} \langle-\rangle_{\rm Sch}: H^0{\rm Obs}^{\rm cl}(\mathbb{R}^d) \to H^0{\rm Obs}^{\rm cl}_{\mathcal{S}}(\mathbb{R}^d) =\mathbb{C} \end{align} \end{definition} \end{tcolorbox} \subsection{Massless case in one-dimension} In massless case, $H^0{\rm Obs}^{\rm cl}_\mathcal{S}(\mathbb{R}^d)$ is NOT isomorphic to $\mathbb{C}$. This is because \begin{align} \hat\Delta^{\rm cl}: \mathcal{S}(\mathbb{R}^d)\ni \hat{f}(k)\mapsto -k^2 \hat{f}(k) \in\mathcal{S}(\mathbb{R}^d) \end{align} is not an isomorphism since \begin{align} ({\hat\Delta^{\rm cl}})^{-1}: \hat{f}(k)\mapsto -\frac{\hat{f}(k)}{k^2} \end{align} is not well-defined. Physicists call it IR divergence. However the IR divergence can be controlled by the following theorem. Using this, we can define Schwartz state in the massless case. \begin{tcolorbox}[colframe=blue,colback=blue!3!] \begin{theorem}\label{thm:IR_one-dim}\ \\ Let $\hat f\in \mathcal{S}(\mathbb{R})$. If $\hat f$ satisfies \begin{align} \hat f(0)=0,\\ \partial \hat{f}(0)=0, \end{align} then \begin{align} \hat h(k) := - \frac{\hat{f}(k)}{k^2} \end{align} is also a Schwartz function on $\mathbb{R}$. \end{theorem} \end{tcolorbox} \begin{proof} First of all, we will find $\hat g\in\mathcal{S}(\mathbb{R})$ satisfying \begin{align} \hat{f}(k)= k\hat{g}(k). \end{align} Since $\hat f(0)=0$, then \begin{align} \hat f(k) = \hat f(k) - \hat f(0). \end{align} We have a line integral \begin{align} \hat f(k) - \hat f(0) &= \int_0^k {\rm d}p\ (\partial\hat{f})(p) \notag\\ &= k \int_0^1 {\rm d}t\ (\partial\hat{f})(kt). \end{align} where $p:=kt$. Define \begin{align} \hat g(k) := \int_0^1 {\rm d}t\ (\partial \hat f)(kt), \end{align} then \begin{align} \hat{f}(k) = k\hat{g}(k)\label{eq:f=kg}. \end{align} $\hat{g}$ is Schwartz, since we can interchange differentiation and integration, because $\partial \hat{f}$ is Schwartz. Now $\hat{g}(0)=0$ by $\partial \hat f(0)=0$. By the same procedure as the above one, we have $\hat{h}\in\mathcal{S}(\mathbb{R})$ satisfying \begin{align} \hat{g}(k)=k\hat{h}(k)\label{eq:g=kh}. \end{align} By (\ref{eq:f=kg}) and (\ref{eq:g=kh}), \begin{align} \hat{f}(k)=k^2\hat{h}(k). \end{align} \end{proof} \begin{tcolorbox}[colframe=blue,colback=blue!3!] \begin{theorem}\label{thm:Schwatz_massless}\ \\ In massless and $d=1$ case \begin{align} H^n{\rm Obs}^{\rm cl}_{\mathcal{S}}(\mathbb{R}) = \left\{ \begin{array}{ll} \mathbb{C}[q,p] & (n=0) \\ 0 & ({\rm otherwise}) \end{array} \right. \end{align} \end{theorem} \end{tcolorbox} \begin{proof} We show a quasi-isomorphism: \begin{align} A\sim B \end{align} where \begin{align} A= \left(\mathcal{S}(\mathbb{R})^{-1}\xrightarrow{\Delta^{\rm cl}}\mathcal{S}(\mathbb{R})^0\right),\ B=(0\to\mathbb{R}^2). \end{align} $\mathbb{C}^2$ sits in degree 0. And we denote the basis of $\mathbb{R}^2$ as $q,p$. The cohomology of ${\rm Sym}(A)$ and ${\rm Sym}(B)$ are $H^*{\rm Obs}^{\rm cl}_{\mathcal{S}}(\mathbb{R})$ and $\mathbb{C}[q,p]$ respectively. First of all, we will see the following commutative diagram: \begin{align} \xymatrix{ \mathcal{S}(\mathbb{R})^{-1} \ar[r] \ar[d]_{\pi^{-1}} & \mathcal{S}(\mathbb{R})^0 \ar[d]_{\pi^0} \\ 0 \ar[r] & \mathbb{C}^2 } \end{align} where $\pi^0$ is defined as \begin{align} \pi^0(g) := q\int_{\mathbb{R}} {\rm d}x \ g(x)\phi_q(x) + p\int_{\mathbb{R}} {\rm d}x \ g(x)\phi_{p_1}(x) \label{eq:pi^0} \end{align} for $g\in \mathcal{S}(\mathbb{R})^0$. The definition of $\phi_q,\phi_{p}\in C^\infty(\mathbb{R})$ is as follows. \begin{align} \begin{cases} \phi_q(x)=1, \\ \phi_{p}(x)=x. \end{cases} \end{align} They form the kernel of $-\Delta$. We can easily check that $\pi^0(g)=0$ holds if $g=\Delta^{\rm cl}f^{\star}.$ Note that $\pi^0$ is well-defined, in other words all integrations in (\ref{eq:pi^0}) converge, since products of Schwartz function $g$ and polynomials $\phi_q,\phi_{p}$ are also Schwartz functions. Next, we will see $H^0(A)=H^0(B)$. By definition, \begin{align} H^0(A) = \frac{\mathcal{S}(\mathbb{R})^0} {{\rm im}(\Delta^{\rm cl})}. \end{align} $\pi^0$ is surjective \footnote{The proof is in the appendix.}, then by the isomorphism theorem \begin{align} H^0(B) = \frac{\mathcal{S}(\mathbb{R})^0} {{\rm ker}(\pi^0)}. \end{align} We will show that ${\rm im}(\Delta^{\rm cl})={\rm ker}(\pi^0)$. If it holds, $H^0(A)=H^0(B)$. Obviously, \begin{align} {\rm im}(\Delta^{\rm cl})\subset {\rm ker}(\pi^0). \end{align} Then we will check ${\rm im}(\Delta^{\rm cl})\supset {\rm ker}(\pi^0)$. Take $f\in {\rm ker}(\pi^0)$. $f$ satisfies \begin{align} &\int_{\mathbb{R}}f(x)\ {\rm d}x =0,\label{eq:pi^0_q}\\ &\int_{\mathbb{R}}xf(x)\ {\rm d}x =0\label{eq:pi^0_p}. \end{align} In order to make it clear, we consider Fourier transformation of $A$. \begin{align} \hat A = \left(\mathcal{S}(\mathbb{R})^{-1}\xrightarrow{\hat\Delta^{\rm cl}}\mathcal{S}(\mathbb{R})^0\right) \end{align} where $\hat\Delta^{\rm cl}:\hat f(k)\mapsto -k^2f(k)$. And the above conditions (\ref{eq:pi^0_p}) (\ref{eq:pi^0_q}) are the same as \begin{align} \hat f(0)=0,\\ \partial\hat f(0)=0. \end{align} Under these conditions \begin{align} \hat h(k):=-\frac{\hat{f}(k)}{k^2} \end{align} is in $\mathcal{S}(\mathbb{R})$ by Theorem \ref{thm:IR_one-dim}. Hence, $\hat \Delta^{\rm cl} \hat h=\hat f$ or $\Delta^{\rm cl} h=f$. Thus $f\in {\rm im}(\Delta^{\rm cl})$, and ${\rm im}(\Delta^{\rm cl})\supset {\rm ker}(\pi^0)$. Finally, we will see $H^{-1}(A)=H^{-1}(B)$. Clearly $H^{-1}(B)=0$. Then let us think of \begin{align} H^{-1}(A)={\rm ker}(\Delta^{\rm cl}). \end{align} This is trivial since the solution of \begin{align} \Delta f(x)=0 \ {\rm or} \ -k^2\hat f(k)=0 \end{align} is $f=0$ or $\hat f=0$. Then $H^{-1}(A)=H^{-1}(B)=0$. \end{proof} \begin{tcolorbox}[colframe=red,colback=red!3!] \begin{definition}\ \\ A Schwartz state $\langle-\rangle_{\rm Sch}$ is a smooth map: \begin{align} \langle-\rangle_{\rm Sch}: H^0{\rm Obs}^{\rm cl}(\mathbb{R}) \to H^0{\rm Obs}^{\rm cl}_{\mathcal{S}}(\mathbb{R}) =\mathbb{C}[q,p] \xrightarrow{j} \mathbb{C} \end{align} where $j$ is a map: $q,p\mapsto 0$. \end{definition} \end{tcolorbox} In case of compactification state, sending $q\mapsto 0$ means to get rid of IR divergences as explained in \ref{subsec:why_j_?}. However in case of Schwartz state, $j$ sends not only $q$ but also $p$ to $0$. If we translate the map $p\mapsto 0$ to ordinary physics words, it is the same as that the state $|0\rangle$ is assumed to be \begin{align} P|0\rangle=0. \end{align} In other words, it is translation invariance. The compactification state automatically has translation invariance because we take periodic boundary condition. In contrast, Schwartz state need to be assumed translation invariance. \subsection{Massless case in higher dimension } In one-dimensional case, we show \begin{align} \left(\mathcal{S}(\mathbb{R})^{-1}\xrightarrow{\Delta^{\rm cl}}\mathcal{S}(\mathbb{R})^0\right) \xrightarrow[\ \pi\ ]{\sim}{} (0\to\mathbb{C}^2). \end{align} The $\mathbb{C}^2$ is the solution space of Laplace equation $\Delta \phi=0$ in one-dimensional space. In order to generalize to higher dimensional case, we need to consider the following solution space\footnote{$\mathscr{H}$ denotes ``Harmonics."}: \begin{align} \mathscr{H} := \{\phi\in \mathcal{S}'(\mathbb{R}^d)\ |\ \Delta_{\mathcal{S}'}\phi=0\} \end{align} where $\mathcal{S}'(\mathbb{R}^d)$ is a space of tempered distributions and $\Delta_{\mathcal{S}'}$ is a Laplacian in $\mathcal{S}'(\mathbb{R}^d)$. $\mathcal{S}'(\mathbb{R}^d)$ is a natural dual space of $\mathcal{S}(\mathbb{R}^d)$, therefore the product $\langle \phi,f\rangle$ is well-defined for $f\in \mathcal{S}(\mathbb{R}^d)$ and $\phi\in\mathcal{S}'(\mathbb{R}^d)$. If $\phi$ is a function, we can represent $\langle \phi,f\rangle$ as integration: \begin{align} \langle \phi,f\rangle = \int_{\mathbb{R}^d} {\rm d}x_1\cdots{\rm d}x_d \ \overline{\phi(x_1,\cdots,x_d)}\ f(x_1,\cdots,x_d). \end{align} Later we will see a quasi-isomorphism: \begin{align} \left(\mathcal{S}(\mathbb{R}^d)^{-1}\xrightarrow{\Delta^{\rm cl}}\mathcal{S}(\mathbb{R}^d)^0\right) \xrightarrow[\ \pi\ ]{\sim}{} (0\to\mathscr{H}). \end{align} Roughly speaking, $\pi$ is defined\footnote{This is not well-defined because $\mathscr{H}$ has $\infty$-dimension. More accurately we need to consider the completion of $\mathscr{H}$ and deform the definition of $\pi$.} as \begin{align} \begin{array}{rccc} \pi^0\colon &\mathcal{S}(\mathbb{R}^d)^0&\longrightarrow& \mathscr{H}\\ & \rotatebox{90}{$\in$}& & \rotatebox{90}{$\in$} \\ & f & \longmapsto & \sum_{\phi\in B(\mathscr{H})}\phi\langle \phi,f\rangle. \end{array} \end{align} where $B(\mathscr{H})$ is a basis of $\mathscr{H}$. This is a natural generalization of one-dimensional case. In order to make clear the meaning of $B(\mathscr{H})$, we introduce the following known theorem. \begin{tcolorbox}[colframe=blue,colback=blue!3!] \begin{theorem}\ \\ Let $\phi\in \mathcal{S}'(\mathbb{R}^d)$. If $\Delta_{\mathcal{S'}}\phi=0$, then $\phi$ can be represented as a polynomial. \end{theorem} \end{tcolorbox} \noindent{} In other words, $\phi\in\mathcal{H}$ is a harmonic polynomial. For polynomials, Fisher inner product is convenient. \begin{tcolorbox}[colframe=red,colback=red!3!] \begin{definition}{\rm (Fisher inner product)} \\ Let $\phi,\psi$ be polynomials on $\mathbb{R}^d$. Fisher inner product is defined as \begin{align} (\phi|\psi) :=\left[ \overline{\phi\left(\frac{\partial}{\partial x_1}, \cdots, \frac{\partial}{\partial x_1}\right)} \psi(x_1,\cdots,x_d)\right]_{x_1=\cdots x_d=0}. \end{align} Let $\phi$ be $k$-degree and $\psi$ be $l$-degree. If $k\neq l$ then $(\phi|\psi)=0$. \end{definition} \end{tcolorbox} \noindent{}And we obtain an orthogonal decomposition of $\mathscr{H}$. \begin{align} \mathscr{H} = \bigoplus_{k=0}^\infty \mathscr{H}_k. \end{align} The dimension of $\mathscr{H}_k$ is finite as follows: \begin{align} {\rm dim}(\mathscr{H}_k) = \frac{(d+2k-2)(d+k-3)!}{(d-2)!k!}. \end{align} We take Gram–Schmidt orthonormalization and get basis $B(\mathscr{H})=\bigoplus_{k=0}^\infty B(\mathscr{H}_k)$. \begin{tcolorbox}[colframe=blue,colback=blue!3!] \begin{theorem}\label{thm:higher_dim}\ \\ In massless and higher dimensional case we have a dense map $\pi$: \begin{align} H^n{\rm Obs}^{\rm cl}_{\mathcal{S}}(\mathbb{R}) \xrightarrow{\pi} \left\{ \begin{array}{ll} {\rm Sym}(\widetilde{\mathscr{H}}\ ) & (n=0) \\ 0 & ({\rm otherwise}) \end{array} \right. \end{align} where $\widetilde{\mathscr{H}}$ is a completion of $\mathscr{H}$ in terms of Fisher inner product. \end{theorem} \end{tcolorbox} The proof is too long to write here, hence we place the details in the appendix. \begin{tcolorbox}[colframe=red,colback=red!3!] \begin{definition}\ \\ A Schwartz state $\langle-\rangle_{\rm Sch}$ is a smooth map: \begin{align} \langle-\rangle_{\rm Sch}: H^0{\rm Obs}^{\rm cl}(\mathbb{R}) \to H^0{\rm Obs}^{\rm cl}_{\mathcal{S}}(\mathbb{R}) ={\rm Sym}(\widetilde{\mathscr{H}}\ ) \xrightarrow{j} \mathbb{C} \end{align} where $j$ sends all generators to $0$. \end{definition} \end{tcolorbox} In one-dimensional case, all we assume is essentially the translation invariance: \begin{align} P|0\rangle=0. \end{align} However, in higher-dimensional case, it looks that we need to assume more invariance. \section{Equivalence} \subsection{Massive case} As said in the beginning, we will show the equivalence of a natural augmentation state, a compactification state and a Schwartz state in massive case. First of all, we introduce the following theorem shown by K. Costello and O. Gwilliam \cite{Costello:2016vjw}. \begin{tcolorbox}[colframe=blue,colback=blue!3!] \begin{theorem} {\rm (K. Costello, O. Gwilliam \cite{Costello:2016vjw})}\\ In massive case, \begin{align} \langle-\rangle_{\rm aug} = \langle-\rangle_{\rm Sch}. \end{align} \end{theorem} \end{tcolorbox} \noindent{}In addition, we will show that $\langle-\rangle_{\rm aug}$ is the same as $\langle-\rangle_{\rm cptf}$ in massive cases. \begin{tcolorbox}[colframe=blue,colback=blue!3!] \begin{theorem}\ \\ In massive case, for observables satisfies the locality condition, \begin{align} \langle-\rangle_{\rm aug} = \langle-\rangle_{\rm cptf}. \end{align} \end{theorem} \end{tcolorbox} \begin{proof} Take $\mathcal{O}\in {\rm Obs}^{\rm cl}(U)\subset {\rm Obs}^{\rm cl}(T^d)$, and calculate $\langle\mathcal{O}\rangle_{\rm cptf}$. \begin{align} \Delta^{\rm cl}_{T^d} : C_{\rm c}^\infty(T^d)^{-1} \to C_{\rm c}^\infty(T^d)^0 \end{align} is an isomorphism. Therefore \begin{align} \mathcal{O} = c+f+f_1*f_2+\cdots = c+\Delta^{\rm cl}_{T^d}(\cdots). \end{align} Hence, \begin{align} \langle\mathcal{O}\rangle_{\rm cptf}=c=\langle\mathcal{O}\rangle_{\rm aug}. \end{align} \end{proof} \subsection{Masless case} We will show the equivalence of the threer states in massless case. \begin{tcolorbox}[colframe=blue,colback=blue!3!] \begin{theorem}\ \\ In massless case, for observables satisfies the locality condition, \begin{align} \langle-\rangle_{\rm aug} = \langle-\rangle_{\rm cptf}. \end{align} \end{theorem} \end{tcolorbox} \begin{proof} Take $\mathcal{O}\in {\rm Obs}^{\rm cl}(U)$, and calculate $\langle\mathcal{O}\rangle_{\rm cptf}$. \begin{align} \langle\mathcal{O}\rangle_{\rm cptf} =j\circ\pi^0(\mathcal{O}) \end{align} $\mathcal{O}=c+f+f_1*f_2+\cdots$, then \begin{align} \pi^0(c+f+f_1*f_2+\cdots) &=c+\pi^0(f)+\pi^0(f_1)\cdot \pi^0(f_2)+\cdots\notag\\ &=c+q\times ({\rm some\ number}). \end{align} Hence, \begin{align} \langle\mathcal{O}\rangle_{\rm cptf}=c=\langle\mathcal{O}\rangle_{\rm aug}. \end{align} \end{proof} \begin{tcolorbox}[colframe=blue,colback=blue!3!] \begin{theorem}\ \\ In massless case, \begin{align} \langle-\rangle_{\rm aug} = \langle-\rangle_{\rm Sch}. \end{align} \end{theorem} \end{tcolorbox} \begin{proof} Take $\mathcal{O}\in {\rm Obs}^{\rm cl}(\mathbb{R}^d)$, and calculate $\langle\mathcal{O}\rangle_{\rm Sch}$. \begin{align} \langle\mathcal{O}\rangle_{\rm cptf} =j\circ\pi^0(\mathcal{O}) \end{align} $\mathcal{O}=c+f+f_1*f_2+\cdots$, then \begin{align} \pi^0(c+f+f_1*f_2+\cdots) &=c+\pi^0(f)+\pi^0(f_1)\cdot \pi^0(f_2)+\cdots\notag\\ &=c+({\rm some\ generator})\times ({\rm some\ number}). \end{align} Hence, \begin{align} \langle\mathcal{O}\rangle_{\rm Sch}=c=\langle\mathcal{O}\rangle_{\rm aug}. \end{align} \end{proof} \section{Conclusion and discussion} We have seen the concrete constructions of states in factorization algebras. In the case of the compactification state, \begin{align} \langle-\rangle_{\rm cptf}: {\rm Obs}^{\rm cl}(U) \to{\rm Obs}^{\rm cl}(T^d) \cong \begin{cases} \mathbb{C} & ({\rm massive}) \\ \mathbb{C}[q] \xrightarrow{j}\mathbb{C} &({\rm massless}) . \end{cases} \end{align} Here the origin of $q$ is the long wave mode of scalar field, so $j:q\mapsto 0$ means to get rid of IR divergences. And the case of the Schwartz state, \begin{align} \langle-\rangle_{\rm Sch}: {\rm Obs}^{\rm cl}(\mathbb{R}^d) \to{\rm Obs}^{\rm cl}_{\mathcal{S}}(\mathbb{R}^d) \cong \begin{cases} \mathbb{C} & ({\rm massive}) \\ {\rm Sym}(\widetilde{\mathscr{H}}\ ) \xrightarrow{j}\mathbb{C} &({\rm massless}) . \end{cases} \end{align} Especially, in one-dimensional case, ${\rm Sym}(\widetilde{\mathscr{H}}\ ) $ is just $\mathbb{C}[q,p]$. This space might imply the symmetry in asymptotic symmetries. For example, in one-dimensional case, $j:p\mapsto 0$ means the assumption of translation invariance for the state: $P|0\rangle=0$. The number of basis of ${\rm Sym}(\widetilde{\mathscr{H}}\ )$ might be uncountablly infinite. It seems to come from the uncountable label $x\in\mathbb{R}^{d-1}$ of $\Phi(x)$. In other words, generally QFTs have infinitely number of observables. We think that it is interesting to investigate ${\rm Sym}(\widetilde{\mathscr{H}}\ )$ in two-dimensional case. Generally, in order to avoid Coleman-Mermin-Wagner theorem\cite{Coleman:1973ci}\cite{Mermin:1966fe}, we need conformal symmetry (Virasoro algebra): \begin{align} L_m|0\rangle=0. \end{align} We expect that Virasoro algebra can be derived from ${\rm Sym}(\widetilde{\mathscr{H}}\ )$. \section*{Acknowledgment} We would like to thank Yuma Furuta for helpful discussions. The work of M. K. is supported by Grant-in-Aid for JSPS Fellows No. 22KJ1989. The work of T. S. was supported by JST SPRING, Grant Number JPMJSP2110. \appendix \section{About the proof of Theorem \ref{thm:Schwatz_massless}} \subsection{$\pi^0$ is surjective}\label{subsec:pi^0_surjective} \begin{tcolorbox}[colframe=blue,colback=blue!3!] \begin{theorem}\ \\ $\pi^0:\mathcal{S}(\mathbb{R})\to\mathbb{R}^2$ is surjective where $\pi^0$ is defined as \begin{align} \pi^0(g) = q\int_{\mathbb{R}} {\rm d}x \ g(x)\phi_q(x) + p\int_{\mathbb{R}} {\rm d}x \ g(x)\phi_{p}(x) \end{align} and $q,p$ is the basis of $\mathbb{R}^2$. \end{theorem} \end{tcolorbox} \begin{proof} It is enough to show that there are $Q,P\in\mathcal{S}(\mathbb{R})$ satisfying \begin{align} \pi^0(Q)=q,\ \pi^0(P)=p. \end{align} One good choice of $Q$ is a {\it smeared $\delta$-function}. \begin{align} Q(x) := \delta_{\rm smeared}(x) \end{align} where we assume that $\delta_{\rm smeared}$ is even, has a compact support and satisfies \begin{align} \int{\rm d}x \ \delta_{\rm smeared}(x)=1. \end{align} $\phi_{p}$ is odd, then we have $\pi^0(\delta_{\rm smeared})=q$. And $P_i$ is given as \begin{align} P(x) := - \frac{\partial}{\partial x} \delta_{\rm smeared}(x). \end{align} We can easily show that $\pi^0(P)=p$. Another choice of $Q$ is given by \begin{align} Q(x) := \frac{1}{\sqrt{\pi}}\exp(x). \end{align} And $P$ is given as \begin{align} P(x) := -\frac{\partial}{\partial x}Q(x) = \frac{2}{\sqrt{\pi}}x\exp(x^2). \end{align} \end{proof} \subsection{Physical meanings of $q$ and $p$} By Theorem \ref{thm:Schwatz_massless}, we have an isomorphism: \begin{align} \begin{array}{rccc} \pi^0\colon &H^0{\rm Obs}^{\rm cl}_\mathcal{S}(\mathbb{R}^d) &\stackrel{\sim}{\longrightarrow}& \mathbb{R}[q,p]\\ & \rotatebox{90}{$\in$}& & \rotatebox{90}{$\in$} \\ & [Q]_\mathcal{S} & \longmapsto & q\\ & [P]_\mathcal{S} & \longmapsto & p. \end{array} \end{align} Especially if we take $Q=\delta_{\rm smeared}$ and $P=-\partial\delta_{\rm smeared}$, $Q$ and $P$ is in $C_{\rm c}^\infty(\mathbb{R})$. Then we have an inclusion map: \begin{align} \begin{array}{rccc} i\colon&H^0{\rm Obs}^{\rm cl}(\mathbb{R}) &\hookrightarrow& H^0{\rm Obs}^{\rm cl}_\mathcal{S}(\mathbb{R}^d)\\ & \rotatebox{90}{$\in$}& & \rotatebox{90}{$\in$} \\ & [Q] & \longmapsto & [Q]_\mathcal{S}\\ & [P] & \longmapsto & [P]_\mathcal{S}. \end{array} \end{align} Therefore originally $q$ and $p$ are from observables \begin{align} Q=\delta_{\rm smeared},\ P=-\partial\delta_{\rm smeared}. \end{align} The action for the field $\Phi\in C^\infty(\mathbb{R}^d)$ is \begin{align} &Q(\Phi) =\int_{\mathbb{R}} {\rm d}x\ \delta_{\rm smeared}(x) \Phi(x) \sim \Phi(0),\\ &P(\Phi) =\int_{\mathbb{R}} {\rm d}x\ (-\partial\delta_{\rm smeared}(x)) \Phi(x) \sim \partial\Phi(0). \end{align} These are the same as the position observable and the momentum observables in physics literature. \section{Some properties of harmonic polynomials} \subsection{Hecke identities and a convenient representation of Fisher inner product} \begin{tcolorbox}[colframe=blue,colback=blue!3!] \begin{theorem}\ \\ If $\phi\in\mathscr{H}$, then we have two identities. \begin{align} \frac{1}{(2\pi)^{d/2}} \int_{\mathbb{R}^d} {\rm d}x_1\cdots{\rm d}x_d\ \phi(x_1,\cdots,x_d) e^{-\frac{1}{2}x^2} e^{ik\cdot x} = \phi(ik_1,\cdots,ik_d) e^{-\frac{1}{2}k^2},\\ \phi\left( \frac{\partial}{\partial x_1}, \cdots, \frac{\partial}{\partial x_d}\right) e^{-\frac{1}{2}x^2} = \phi(-x_1,\cdots,-x_d)e^{-\frac{1}{2}x^2}. \end{align} We call them Hecke identities. \end{theorem} \end{tcolorbox} \noindent{}By using Hecke identities, we have the following theorem. \begin{tcolorbox}[colframe=blue,colback=blue!3!] \begin{theorem}\ \\ If $\phi,\psi\in\mathscr{H}$, then \begin{align} (\phi|\psi) &= \frac{1}{(2\pi)^{d/2}} \int_{\mathbb{R}^d} {\rm d}x_1\cdots{\rm d}x_d\ \overline{\phi(x_1,\cdots,x_d)} \psi(x_1,\cdots,x_d) e^{-\frac{1}{2}x^2}\notag\\ &=\left\langle \phi,\frac{1}{(2\pi)^{d/2}}\psi e^{-\frac{1}{2}x^2}\right\rangle \end{align} \end{theorem} \end{tcolorbox} \begin{proof} $\overline{\phi\left( \frac{\partial}{\partial x_1}, \cdots, \frac{\partial}{\partial x_d}\right)} \psi\left(x_1,\cdots,x_d\right)$ is also in $\mathscr{H}$, then by the first Hecke identities, we obtain \begin{align} \frac{1}{(2\pi)^{d/2}} \int_{\mathbb{R}^d} {\rm d}x_1\cdots{\rm d}x_d\ \overline{\phi\left( \frac{\partial}{\partial x_1}, \cdots, \frac{\partial}{\partial x_d}\right)} \psi\left(x_1,\cdots,x_d\right) e^{-\frac{1}{2}x^2} e^{ik\cdot x}\notag\\ = \overline{\phi\left(-i \frac{\partial}{\partial k_1}, \cdots, -i\frac{\partial}{\partial k_d}\right)} \psi\left(k_1,\cdots,k_d\right) e^{-\frac{1}{2}k^2}. \end{align} Substitute $k_1=\cdots=k_d=0$, then \begin{align} \frac{1}{(2\pi)^{d/2}} \int_{\mathbb{R}^d} {\rm d}x_1\cdots{\rm d}x_d\ \overline{\phi\left( \frac{\partial}{\partial x_1}, \cdots, \frac{\partial}{\partial x_d}\right)} \psi\left(x_1,\cdots,x_d\right) e^{-\frac{1}{2}x^2} = (\phi|\psi). \end{align} Interate by part, \begin{align} (\phi|\psi) &= \frac{1}{(2\pi)^{d/2}} \int_{\mathbb{R}^d} {\rm d}x_1\cdots{\rm d}x_d\ \psi\left(x_1,\cdots,x_d\right) \overline{\phi\left( \frac{\partial}{\partial x_1}, \cdots, \frac{\partial}{\partial x_d}\right)} e^{-\frac{1}{2}x^2}\notag\\ &= \frac{1}{(2\pi)^{d/2}} \int_{\mathbb{R}^d} {\rm d}x_1\cdots{\rm d}x_d\ \psi(x_1,\cdots,x_d) \overline{\phi(x_1,\cdots,x_d)} e^{-\frac{1}{2}x^2}. \end{align} We used the second Hecke identity in the last line. \end{proof} \section{About the proof of Theorem \ref{thm:higher_dim}} \subsection{Outline of the proof} The outline of the proof is the same as the one of Theorem \ref{thm:Schwatz_massless}. Hence naively it seem to be enough to see the following things. \begin{itemize} \item[(1)] The accurate definition of $\pi^0$ \item[(2)] $\pi^0$ is a surjective map. \item[(3)] ${\rm ker}(\pi^0)\cong{\rm im}(\Delta^{\rm cl})$ \end{itemize} \noindent{}However, since $\mathscr{H}$ has infinitely large dimension, we rephrase (2) and (3) as \begin{itemize} \item[(2)'] $\pi^0$ is a dense map. \item[(3)'] ${\rm ker}(\pi^0)\cong\overline{{\rm im}(\Delta^{\rm cl})}$ where $\overline{{\rm im}(\Delta^{\rm cl})}$ is a closure of ${\rm im}(\Delta^{\rm cl})$. \end{itemize} Note that \begin{align} \frac{\mathcal{S}(\mathbb{R}^2)^0}{{\rm im(\Delta^{\rm cl})}} \cong \frac{\mathcal{S}(\mathbb{R}^2)^0}{{\rm \overline{im(\Delta^{\rm cl})}}}, \end{align} then it is sufficient to see ${\rm ker}(\pi^0)\cong\overline{{\rm im}(\Delta^{\rm cl})}$. We will see each of them in the later sections. \subsection{The accurate definition of $\pi^0$} So as to define $\pi^0$, we consider the sequence \begin{align} \{ \pi^0_n:\mathcal{S}\to \mathscr{H} \}_{n=0,1,\cdots} \end{align} as \begin{align} \pi^0_n(f) = \sum_{k=0}^n a_k\langle \phi_k, e^{-\frac{1}{4}x^2}f\rangle \phi_k. \end{align} Here $\phi_k$ forms the orthogonal basis $B(\mathscr{H})$ and the label $k$ is set in ascending order of the polynomial degree. Then we have $k>\deg(\phi_k)$. And $a_k$ will be defined as $\frac{1}{k^{\frac{k}{2}+1}}$ in order to make $\pi_n^0(f)$ Cauchy sequence, in other words \begin{align} ||\pi_n^0(f)-\pi^0_m(f)||\to 0\ {\rm as}\ n,m\to\infty \end{align} where $||\cdot ||$ is decided by Fisher inner product. From now, we will estimate $||\pi_n^0(f)-\pi^0_m(f)||$ for $n>m$. \begin{align} ||\pi_n^0(f)-\pi^0_m(f)||^2 &= \left( \sum_{k=m}^n a_k\langle \phi_k, e^{-\frac{1}{4}x^2}f\rangle \phi_k \middle| \sum_{\ell=m}^n a_{\ell}\langle \phi_{\ell}, e^{-\frac{1}{4}x^2}f\rangle \phi_{\ell} \right)\notag\\ &= \sum_{k=m}^n\sum_{\ell=m}^n \overline{a_k}a_\ell \overline{\langle \phi_k, e^{-\frac{1}{4}x^2}f\rangle} \langle \phi_{\ell}, e^{-\frac{1}{4}x^2}f\rangle\notag\\ &= \sum_{k=m}^n |a_k|^2 |\langle \phi_k, e^{-\frac{1}{4}x^2}f\rangle|^2 \end{align} $|a_k|$ is $\frac{1}{k^{\frac{k}{2}+1}}$ by definition and $|\langle \phi_k, e^{-\frac{1}{4}x^2}f\rangle|$ is \begin{align} |\langle \phi_k, e^{-\frac{1}{4}x^2}f\rangle| &= \left|\int_{\mathbb{R}^d} {\rm d}x_1\cdots{\rm d}x_{\rm d}\ \overline{\phi_k(x_1,\cdots,x_d)} e^{-\frac{1}{4}x^2}f(x_1,\cdots,x_d)\right|\notag\\ &< \int_{\mathbb{R}^d} {\rm d}x_1\cdots{\rm d}x_{\rm d}\ \left|\overline{\phi_k(x_1,\cdots,x_d)} e^{-\frac{1}{4}x^2}f(x_1,\cdots,x_d)\right|\notag\\ &< \sup_{x_1,\cdots,x_d} \left|\overline{\phi_k(x_1,\cdots,x_d)} e^{-\frac{1}{4}x^2}\right| \int_{\mathbb{R}^d} {\rm d}x_1\cdots{\rm d}x_{\rm d}\ \left|f(x_1,\cdots,x_d)\right|\notag\\ &= \sup_{x_1,\cdots,x_d} \left|\overline{\phi_k(x_1,\cdots,x_d)} e^{-\frac{1}{4}x^2}\right| ||f||_{L^1}. \end{align} $\phi_k(x_1,\cdots,x_d)$ can be represented as \begin{align} \phi_k(x_1,\cdots,x_d) = \sum_{\alpha_1+\cdots+\alpha_d=\deg(\phi_k)} c_{\alpha_1\cdots\alpha_d} x_1^{\alpha_1}\cdots x_d^{\alpha_d} \end{align} $|c_{\alpha_1\cdots\alpha_d}|$ can be estimated as being smaller than $1$. Therefore \begin{align} \sup_{x_1,\cdots,x_d} \left|\overline{\phi_k(x_1,\cdots,x_d)} e^{-\frac{1}{4}x^2}\right| &< 1\cdot \sup_{x_1,\cdots,x_d} \left| x_1^{\deg(\phi_k)} e^{-\frac{1}{4}x^2}\right|\notag\\ &= 1\cdot \left(\frac{2\deg(\phi_k)}{e}\right)^\frac{\deg(\phi_k)}{2}\notag\\ &< \deg(\phi_k)^\frac{\deg(\phi_k)}{2}\notag\\ &< k^{\frac{k}{2}}\ \ \ (\ \because\ \deg(\phi_k)<k\ ). \end{align} Thus \begin{align} ||\pi_n^0(f)-\pi^0_m(f)||^2 &= \sum_{k=m}^n |a_k|^2 |\langle \phi_k, e^{-\frac{1}{4}x^2}f\rangle|^2\notag\\ &< \sum_{k=m}^n |a_k|^2 \sup_{x_1,\cdots,x_d} \left|\overline{\phi_k(x_1,\cdots,x_d)} e^{-\frac{1}{4}x^2}\right|^2 ||f||_{L^1}^2\notag\\ &< {\rm const.}\times \sum_{k=m}^n \frac{1}{k^{k+2}}k^k\notag\\ &= {\rm const.}\times \sum_{k=m}^n \frac{1}{k^{2}} \end{align} It implies that $\pi_n^0(f)$ is a Cauchy sequence. We have seen $\pi_n^0(f)$ is a Cauchy sequence, then it converges in $\mathscr{H}$. \begin{align} \pi_n^0(f)\to \phi_f\ {\rm as}\ n\to\infty. \end{align} We define $\pi^0:\mathcal{S}^0(\mathbb{R}^d)\to\mathscr{H}$ as $\pi^0(f)=\phi_f$. \subsection{$\pi^0$ is a dense map.} In order to show it, we will see the existence of $\mathcal{O}_\phi\in\mathcal{S}(\mathbb{R}^d)$ satisfying \begin{align} \pi^0(\mathcal{O}_k)=\phi_k. \end{align} The basic way to check it is the same as the section \ref{subsec:pi^0_surjective}. Then we set $\mathcal{O}_\phi$ as \begin{align} \mathcal{O}_k(x_1,\cdots,x_d) &= \frac{1}{a_\ell(2\pi)^{d/2}} \phi_k(x_1,\cdots,x_d) e^{-\frac{1}{4}x^2}\notag\\ &= \frac{1}{a_\ell(2\pi)^{d/2}} \phi_k\left(-\frac{\partial}{\partial x_1},\cdots,-\frac{\partial }{\partial x_d}\right) e^{-\frac{1}{4}x^2}. \end{align} Thus \begin{align} \pi^0\left(\mathcal{O}_k\right) &= \sum_{\ell=0}^\infty a_\ell\left\langle \phi_\ell, \mathcal{O}_ke^{-\frac{1}{4}x^2}\right\rangle \phi_\ell\notag\\ &= \sum_{\ell=0}^\infty \left\langle \phi_\ell, \frac{1}{(2\pi)^{d/2}}\phi_k e^{-\frac{1}{2}x^2}\right\rangle \phi_\ell\notag\\ &= \phi_k. \end{align} Note that $\mathcal{O}_k$ is a natural generalization of $Q$ and $P$ for one-dimensional case in section \ref{subsec:pi^0_surjective}. \begin{align} Q(x) &:= \frac{1}{\sqrt{\pi}}\exp(x).,\\ P(x) &:= -\frac{\partial}{\partial x}Q(x) = \frac{2}{\sqrt{\pi}}x\exp(x^2). \end{align} Hence we can regard $\mathcal{O}_k\in \mathscr{H}$ as the observables for asymptotic state in higher-dimension. \subsection{${\rm ker}(\pi^0)\cong\overline{{\rm im}(\Delta^{\rm cl})}$} To show ${\rm ker}(\pi^0)=\overline{{\rm im}(\Delta^{\rm cl})}$, we think the following two steps. \begin{itemize} \item ${\rm ker}(\pi^0) \cong \left({\rm ker}(\Delta_{\mathcal{S}'})\right)^\perp$ \item ${\rm ker}(\Delta_{\mathcal{S}'}) \cong ({\rm im}(\Delta^{\rm cl}))^\perp$ \end{itemize} where \begin{align} \left({\rm ker}(\Delta_{\mathcal{S}'})\right)^\perp &:= \{ f\in \mathcal{S}'(\mathbb{R}^d)\ |\ \forall \phi\in {\rm ker}(\Delta_{\mathcal{S}'}),\ \langle \phi,e^{-\frac{1}{4}x^2}f\rangle=0 \},\notag\\ ({\rm im}(\Delta^{\rm cl}))^\perp &:= \{ \phi\in \mathcal{S}'(\mathbb{R}^d)\ |\ \forall f\in {\rm im}(\Delta^{\rm cl}),\ \langle \phi,e^{-\frac{1}{4}x^2}f\rangle=0 \}. \end{align} By the above equations, we have \begin{align} {\rm ker}(\pi^0) \cong (({\rm im}(\Delta^{\rm cl}))^\perp)^\perp \cong \overline{{\rm im}(\Delta^{\rm cl})}. \end{align} First of all, we see ${\rm ker}(\pi^0)\cong \left({\rm ker}(\Delta_{\mathcal{S}'})\right)^\perp$. Take $f\in {\rm ker}(\pi^0)$, then for all harmonic polynomial $\phi$ we have \begin{align} \langle \phi, e^{-\frac{1}{4}x^2}f\rangle=0. \end{align} In other words, for all $\phi\in {\rm ker}(\Delta_{\mathcal{S}'})$ \begin{align} \langle \phi, e^{-\frac{1}{4}x^2}f\rangle=0. \end{align} Therefore $f\in ({\rm ker}(\Delta_{\mathcal{S}'}))^\perp$, i.e. ${\rm ker}(\pi^0)\subset \left({\rm ker}(\Delta_{\mathcal{S}'})\right)^\perp$. Following the above discussion in reverse, we have ${\rm ker}(\pi^0)\supset \left({\rm ker}(\Delta_{\mathcal{S}'})\right)^\perp$. Hence, ${\rm ker}(\pi^0)\cong \left({\rm ker}(\Delta_{\mathcal{S}'})\right)^\perp$. Next, we see ${\rm ker}(\Delta_{\mathcal{S}'})\cong({\rm im}(\Delta^{\rm cl}))^\perp$. Take $\phi\in {\rm im}(\Delta^{\rm cl}))^\perp$, thus we have \begin{align} &\langle \phi,\Delta f \rangle=0 \ \ (\forall f\in\mathcal{S}(\mathbb{R}^d))\notag\\ &\Leftrightarrow \langle \Delta_{\mathcal{S}'} \phi,f \rangle=0 \ \ (\forall f\in\mathcal{S}(\mathbb{R}^d))\notag\\ &\Leftrightarrow \Delta_{\mathcal{S}'} \phi=0. \end{align} Therefore $\phi\in {\rm ker}(\Delta_{\mathcal{S}'})$. Hence ${\rm ker}(\Delta_{\mathcal{S}'})\supset({\rm im}(\Delta^{\rm cl}))^\perp$. By the above discussion in reverse, we have ${\rm ker}(\Delta_{\mathcal{S}'})\subset({\rm im}(\Delta^{\rm cl}))^\perp$. Then we obtain \begin{align} {\rm ker}(\Delta_{\mathcal{S}'})\cong({\rm im}(\Delta^{\rm cl}))^\perp. \end{align} \bibliographystyle{ptephy} \bibliography{sample} \end{document}
2412.08334v2
http://arxiv.org/abs/2412.08334v2
Maker-Breaker on Galton-Watson trees
\pdfminorversion=5 \documentclass{article} \usepackage{graphicx} \usepackage{epstopdf} \usepackage{amsmath} \usepackage{amssymb} \usepackage{bm} \usepackage{bbm} \usepackage{enumerate} \usepackage[T1]{fontenc} \usepackage[latin1]{inputenc} \usepackage{hyperref} \usepackage{color} \usepackage{curves} \usepackage{tikz} \usepackage[hang]{caption} \usepackage{ntheorem} \usepackage{array,float,caption,nicefrac} \usepackage{mathrsfs} \title{Maker-Breaker on Galton-Watson trees} \author{Timo Vilkas \\\normalsize Lunds Universitet } \theoremstyle{break} \newtheorem{theorem}{Theorem}[section] \newtheorem{corollary}{Corollary}[section] \newtheorem{lemma}{Lemma}[section] \newtheorem{proposition}{Proposition}[section] \theorembodyfont{\upshape} \newtheorem{definition}{Definition} \newtheorem*{remark}{Remark} \newtheorem{example}{Example} \makeatletter \let\c@proposition\c@theorem \let\c@lemma\c@theorem \let\c@corollary\c@theorem \makeatother \newenvironment{proof}{\noindent{\sc Proof:}}{\vspace{-1em}~\hfill $\square$\vspace{2em}} \newenvironment{nproof}[1]{\noindent{\sc Proof #1:}}{\vspace{-1em}~\hfill $\square$\vspace{2em}} \newenvironment{AMS}{}{} \newenvironment{keywords}{}{} \newcommand\IN{\mathbb{N}} \newcommand\IR{\mathbb{R}} \newcommand\IZ{\mathbb{Z}} \newcommand\IQ{\mathbb{Q}} \newcommand\IE{\mathbb{E}\,} \newcommand\Prob{\mathbb{P}} \renewcommand\epsilon{\varepsilon} \renewcommand\phi{\varphi} \DeclareMathOperator\supp{supp} \newcommand\En{\mathcal{E}} \newcommand\D{\mathscr{D}} \newcommand\law{\buildrel d \over\rightarrow} \definecolor{darkblue}{rgb}{0,0,.5} \hypersetup{colorlinks=true, breaklinks=true, linkcolor=blue, citecolor=darkblue, menucolor=blue, urlcolor=blue} \renewcommand{\captionfont}{\small} \begin{document} \newpage \maketitle \begin{abstract} We consider the following combinatorial two-player game: On the random tree arising from a branching process, each round one player (Breaker) deletes an edge and by that removes the descendant and all its progeny, while the other (Maker) fixates an edge to permanently secure it from deletion. Breaker has won once the tree's root is contained in a finite component, otherwise Maker wins by building an infinite path starting at the root. It will be analyzed both as a positional game (the tree is known to both players at the start) and with more restrictive levels of information (the players essentially explore the tree during the game). Reading the number of available edges for play as a random walk on $\mathbb{Z}$ allows us to derive the winning probability of Breaker via fixed point equations in three natural information regimes. These results provide new insights into combinatorial game theory and random structures, with potential applications to network theory, algorithmic game design and probability theory. \end{abstract} \noindent \textbf{Keywords:} Maker-Breaker game, Galton-Watson tree, branching process, percolation games, phase transition, random walk on $\IZ$ with drift.\\[0.5em] \textbf{MSC2020:} 05C57, 91A43, 60G50 \section{Introduction} Maker-Breaker games are a class of combinatorial games in which two players, called Maker and Breaker, compete by selecting elements from a finite or infinite structure, with opposing objectives. These games are well-studied in the context of graph theory, where Maker seeks to build a particular substructure while Breaker tries to prevent its formation. In the two most common variants of the game, either nodes or edges are played, a distinction which (depending on the objective) can become irrelevant in case the underlying graph is a tree. Historically seen, a game of this kind was reportedly first formulated and formalized by C.E.\ Shannon at mid-20th century and later coined as ``Shannon switching game'' \cite{Shannon}. In recent years, the study of Maker-Breaker games has been extended to boards given by random structures. Galton-Watson trees, initially introduced in the 19th century to study the extinction of family names, have since found wide applications in areas like probability theory, evolutionary biology, and network modeling. The intersection of these two fields, Maker-Breaker games on Galton-Watson trees, presents interesting challenges that combine random processes with adversarial strategies in combinatorial games. In this article, we explore the dynamics of Maker-Breaker games on Galton-Watson trees, examining how the probabilistic nature of the tree structure and the available information influence the winning probabilities of both players, when the objective for Breaker is to cut off the root, in the sense that it is eventually contained in a finite component. We delve into key questions such as how the offspring distribution of the tree affects the likelihood of Breaker's success and how the available information about the tree influences their tactics to account for the randomness of the game environment. Through this analysis, we shed light on the intricate interplay between randomness and strategy in these games, offering insights that could have broader implications for fields ranging from algorithm design to network theory. \subsection{Main results} Throughout the paper, we will write $g$ for the probability generating function of the offspring distribution (cf.\ equation \eqref{pgf} below) and $q$ for the corresponding extinction probability (cf.\ Theorem \ref{extinction}). The probability of Breaker succeeding with isolating the root in a finite component, when being in the favorable position to start the game, is denoted by $p$ (or else $\bar{p}$, if Maker starts instead). {\em Positional games} in general are usually characterized by a finite board and perfect information, which make the games deterministic, except for (potential) randomness of the game board. In contrast, we consider here a (potentially) infinite random tree as underlying graph, which is explored by the players during the game. Both strategies and outcome under optimal play depend on the information available to the players. Three different regimes will be analyzed in detail: First, the usual setting in which the whole tree is revealed at the start (turning the game into a deterministic matter), then the other extreme when no information is available (besides the edges incident to either the root or already played edges) and finally an intermediate case in which each vertex connected to the root (by edges Maker picked) is marked according to whether or not it is the root of an infinite subtree. In all three cases, the winning probability of Breaker $p$ is the solution to an equation involving the probability generating function $g$ and its derivative: \newtheorem*{Thm4.3}{Theorem \ref{formula}} \begin{Thm4.3} If the GW-tree is revealed to both players at the start, it holds \begin{equation*} p=g(p)+(1-p)\cdot g'(p). \end{equation*} \end{Thm4.3} \newtheorem*{Thm5.3}{Theorem \ref{i=0}} \begin{Thm5.3} Given there are no childless nodes and the players do not receive any further information, it holds \begin{equation*} p^2=g(p). \end{equation*} \end{Thm5.3} If the probability of not having any offspring is strictly positive in this regime, we are still able to determine $p$ in case the offspring distribution is separable (cf.\ Subsection \ref{separable}, in particular Theorem \ref{septhm}). \newtheorem*{Thm6.4}{Theorem \ref{tv=i}} \begin{Thm6.4} If the information carried by visible nodes is whether or not they have infinite progeny, it holds \begin{equation*} p^2\,(1-q)=g\big(p\,(1-q)+q\big)-q. \end{equation*} \end{Thm6.4} For all three regimes we try to illustrate the results with help of examples (in terms of the offspring distribution: Binomial, geometric and Poisson among others) and corresponding numerical calculations. It appears that the phase transition from an almost sure Breaker win to a non-trivial game is discontinuous only in the first regime (where the tree is revealed right away). Furthermore, in all cases it holds $\bar{p}=g(p)$, with the single exception when no additional information is given and the tree contains leaves (see Theorem \ref{arch} and Proposition \ref{ineq}). In the last section, some general observations, a few open problems and suggestions for further research are presented. \subsection{Related work} As already mentioned, games of this kind were introduced into scientific literature more than half a century ago and an abundance of variants, differing either in the specific rules of the game, the players' objectives or the underlying graphs, have been looked at and analyzed since. One of the first results \cite{Shannon} deals with the objective for Maker (called ``short'' there) to connect two given nodes on a general graph, while Breaker (called ``cut'') tries to prevent that. Other common objectives coined variants such as the ``connectivity game'', ``perfect matching game'', ``Hamiltonian game'' or ``clique game'', in which Maker tries to claim a spanning tree, a perfect matching, a Hamiltonian cycle or a clique of a given size respectively. Besides the rule that edges are picked one by one alternatingly, the more general (potentially biased) $(m,b)$-rule, according to which in each round first Maker plays $m$ edges at once, then Breaker plays $b$ edges, has been considered and analyzed in many different contexts. We will, however, stick to the basic $(1,1)$-rule here. Chvtal and Erd\H{o}s in their seminal paper \cite{erds} considered among others the biased $(1,b)$ connectivity game on the complete graph $K_n$ and found that the threshold for the bias is around $b=\frac{n}{\log n}$ as $n$ grows large (in the sense that if each round first Breaker picks $b$ edges, then Maker picks one edge, it is a Maker's win for $b$ smaller and a Breaker's win for $b$ larger than this threshold). More than 20 years later, Bednarska and \L uczak \cite{Luczak} analyzed the size of the largest component Maker is able to build in this context. Hefetz et al.\ \cite{Hefetz} later addressed the time until Maker wins the unbiased $(1,1)$-game in this setting, with different objectives (among others, perfect matching and Hamiltonian game). In the context of random boards, the Erd\H{o}s-Rnyi graph $G_{n,p}$ was unsurprisingly one of the first targets addressed. Stojakovi\'c and Szab \cite{StoSza} established for different objectives (connectivity, perfect matching, Hamiltonian and clique game) the threshold for the edge probability $p$, at which Maker wins the unbiased $(1,1)$-game with high probability as $n\to\infty$. In addition to that, they investigate the critical bias $b$ (asymptotically and depending on $p$) at which Maker wins the $(1,b)$-game. Ferber et al.\ \cite{Rndboards} later extended this line of research to further ranges of the edge probability $p$. London and Pluhr \cite{London} addressed these questions for a slightly altered Maker-Breaker game, in which Maker can only play edges incident to edges she already claimed. Clemens et al.\ \cite{CKM} extended their investigation of the threshold probability $p$ to the unbiased $(2,2)$-variant. While their results reveal interesting deviations from the answers for the original game, it is not hard to see that this additional rule is in fact no restriction in our chosen setting. Maker-Breaker games have also been considered on random geometric graphs $G(n,r)$: $n$ points are placed independently and uniformly at random on the unit square and connected by an edge if their distance does not exceed $r$. Here, the crucial characteristic for whether it is a Maker's or a Breaker's win (with high probability as $n\to\infty$) turned out to be the minimum degree. That it is a local property of the random graph should not come as a surprise, since the objectives considered by Beveridge et al.\ \cite{RndGeo} (connectivity, perfect matching and Hamilton game) do involve {\em all} vertices, in the sense that Maker attempts to build a structure including all nodes of the graph. In contrast to that, global graph properties were found to be crucial in other (in particular tree) contexts, such as the {\em branching number} governing both random walks and percolation on trees (cf.\ Lyons \cite{Lyons}) or the average number of offspring, determining whether the survival probability $1-q$ of a Galton-Watson tree is strictly positive (cf.\ Theorem \ref{extinction}). When determining the probability for a Galton-Watson tree to contain an infinite complete binary tree $T_2$, the offspring distribution's probability generating function occupies center stage (cf.\ Dekking \cite{Dek1}). We will revisit this result in our context as Theorem \ref{formula}. Also in the analysis of what Holroyd et al.\ (already in an earlier publication) named {\em percolation games} on rooted, directed Galton-Watson trees \cite{HM} (a token is moved along the directed edges and the game ends when a sink is reached) the winning (and draw) probabilities undergo a phase transition at a fixed point of an equation involving the offspring distribution's generating function. Here, three game variants are considered: normal (the player unable to move loses), misre (the player reaching a sink wins) and escape (Stopper wins when the game ends, Escaper wins if it doesn't). A similar but qualitatively different game on undirected rooted Galton-Watson trees (where nodes also can be traps and targets, but edges traversed in both directions) was recently analyzed by Karmakar et al.\ \cite{rooted} and yielded a similar kind of results. In terms of game variant and players' objectives, what is most closely related to the setting we consider here (besides the original Shannon switching game) is the work by Day and Falgas-Ravry: Both players in turns claim edges and while Maker's objective is to build a path, Breaker is trying to prevent that. In a first publication \cite{DFR1}, the authors considered the task of crossing a finite rectangular grid from left to right, a special case of Shannon's original game but extended to the general $(m,b)$-rule. In a second publication \cite{DFR2}, they consider an infinite underlying graph and Maker's task is to build an infinite path starting at a given vertex $v_0$ (the exact same variant we will adopt here, with $v_0$ being the root of the tree $\mathbf{0}$). Their main results concern both the $d$-dimensional grid $\mathbb{Z}^d$ and the infinite $d$-regular tree $T_d$. Observe, however, that in these cases the Maker-Breaker game is fully deterministic. \section{Preliminaries}\label{sec2} In order to get started, let us properly introduce the two central concepts: the (1,1) Maker-Breaker game and the branching process generating the (random) tree which will be the underlying graph in our analysis. \subsection{The Maker-Breaker game on the edge set of a graph} Within graph theory, {\em Maker-Breaker} is a combinatorial two-player game, taking place on a graph $G=(V,E)$, with one node marked as the origin $\mathbf{0}$. All edges in $E$ are available for play until either fixated by Maker or removed by Breaker. The two opposing players in turns are allowed to pick one of the remaining edges in order to either fixate (Maker) or remove it (Breaker). The objective for Maker is to create as big a component containing the origin as possible, while Breaker tries to cut it off. We will consider simple infinite graphs (i.e.\ no loops or multiple edges, $|V|=\infty$) with finite degrees and consider the game won for Breaker once the origin is contained in a finite component separated from the rest of the graph by removed edges. Maker wins if Breaker doesn't and by a simple compactness argument this results in the task of fixating an infinite path starting at the origin. We assume both Maker and Breaker to play optimally in the sense that they try to maximize their chance of winning in every move. Note that the game is deterministic on a given graph $G$ and only becomes a game of chance if played on a random graph or by integrating randomness into the players' strategies. Further, the probability of Maker winning the game is monotonous in $G$ in the sense that adding nodes and/or edges to the graph can not decrease it. As default, unless otherwise stated, we consider Breaker to start the game. As a first example, consider the two-dimensional grid $\IZ^2$ (see Figure \ref{MB} for an illustration). It takes little ingenuity to come up with a winning strategy for Maker in the $(1,1)$-game, i.e.\ each player gets to choose one edge at a time: Note first that every node $u\in\IZ^2$ is adjacent to at least two edges pointing outwards, to be more precise $e=\langle u,v\rangle\in E$, where $\lVert u \rVert<\lVert v \rVert$. If now Maker always plays an edge pointing outwards from $u$ after Breaker has done so (for the first time), in fact all vertices (in particular the origin) stay connected to each other. This kind of strategy is commonly referred to as {\em pairing strategy}. \begin{figure}[H] \centering \includegraphics[scale=1.3]{makerbreaker} \caption{Illustrating example of a game position on $\IZ^2$ after 10 moves each.\label{MB}} \end{figure} In order to make it more interesting, one can consider the more general $(m,b)$-variant of the game (Breaker removes $b$ edges, then Maker fixates $m$ edges in turns). Obviously, a winning strategy for Maker will persist when increasing $m$ (as it does for Breaker when $b$ is raised). However, already the outcome of the $(m,b)$-game on $\IZ^2$ for $b=2$ is far from trivial to determine. \subsection{Bienaym branching process and Galton-Watson tree} Here, we want to consider as underlying (random) graph for the Maker-Breaker game the (potentially infinite) family tree of a branching process introduced by probabilist Irne-Jules Bienaym and later (independently) made widely popular by natural scientist Francis Galton and mathematician Henry William Watson. In this classical branching process, each individual $v$ gives rise to a random number $\xi_v$ of offspring. The sequence $\{\xi_v\}_{v\in V}$ consists of independent and identically distributed copies of a random variable $\xi$. Each individual is represented by a node $v$ and nodes corresponding to parent and child are linked by an edge, to form what is commonly called {\em Galton-Watson family tree}. Depending on the offspring distribution (and chance), this tree can turn out to be finite or infinite. All nodes at a given (graph) distance $n\in\IN_0$ of the root $\mathbf{0}$ in the tree are considered to form a generation. Let $Z_n$ denote the number of individuals in generation $n$, i.e. \[Z_0=1,\ Z_{n+1}=\sum_{j=1}^{Z_n}\xi_{v_{nj}}\quad\text{for}\ n\in\IN_0,\] where the nodes are indexed in breadth-first-order so that $\mathbf{0}=v_{01}$ and in general $v_{nj}$ corresponds to individual $j$ in generation $n$. We will denote the probability mass function (pmf) of the offspring distribution by $p_k=\Prob(\xi=k)$, for $k\in\IN_0$, and write \begin{equation}\label{pgf} g(x)=\IE\big(x^\xi\big)=\sum_{k=0}^\infty p_k\, x^k \end{equation} for its {\em probability generating function} (pgf). Throughout, we will consider the offspring distribution to be almost surely finite. At this point it might be worth mentioning that $\mu:=\IE\xi=g'(1)$ is commonly referred to as \emph{Malthusian parameter}. The extinction probability of the corresponding Galton-Watson process, i.e.\ $q:=\lim_{n}\Prob(Z_n=0)$ can be characterized as the first intersection of $g$ with the identity function in the first quadrant (cf.\ Theorem 1 in \cite{Athreya-Ney} for instance): \begin{theorem}[Extinction probability]\label{extinction} The extinction probability $q$ of the branching process $(Z_n)_{n\in\IN_0}$ is the smallest nonnegative root of the equation $g(x)=x$. It equals 1, if $\mu\leq 1$ (and $p_1<1$) and is strictly less than 1 if $\mu> 1$ (or $p_1=1$). \end{theorem} Excluding the degenerate case $p_1=1$, a Galton-Watson process with $\mu\leq 1$ is hence called {\em subcritical}, one with $\mu> 1$ instead {\em supercritical}. The standard proof of this classical theorem is straight-forward and based on the fact that for fixed $x\in[0,1]$, the $n$-fold iterates $g\circ g\circ\dots\circ g(x)$ converge to $q$ as $n$ goes to infinity, see Figure \ref{G-W} for an illustration. \begin{figure}[H] \centering \includegraphics[scale=1]{G-W} \caption{Examples of probability generating functions corresponding to a subcritical (left) and supercritical process (right).\label{G-W}} \end{figure} For a given offspring distribution, let $T$ denote the (random, unlabelled) Galton-Watson (GW) tree and for an arbitrary node $v$ in $T$, we are going to write $T_v$ for the subtree rooted in $v$. By construction, every $T_v$ is a copy of $T$. The (1,1)-version of Maker-Breaker on $T$ amounts to Breaker cutting off a subtree $T_v$ (by disconnecting $v\neq\mathbf{0}$ from its parent) and Maker fixating an edge $\langle u,w\rangle$, hence adding the corresponding child node to the component containing the root. Observe that $T$ being a tree (i.e.\ not containing any cycles) makes it suboptimal for both players to play an edge linking two vertices that are not incident to any fixated edges (yet). We will refer to the nodes connected to the root by fixated edges as {\em internal nodes}. Nodes, corresponding to descendants in $T$ of currently internal nodes, which are neither internal nor cut off from the root, are referred to as {\em external nodes}, see Figure \ref{tree} for an illustration. \begin{figure}[H] \centering \includegraphics[scale=0.9]{tree} \caption{Illustrating example of a game position on $T$ with internal nodes black, external nodes gray and others omitted.\label{tree}} \end{figure} \subsection{Random walk on \texorpdfstring{$\IZ$}{\bf Z}} To complete this introductory section, we want to include a few basic results for random walk on integers. This topic is closely related to GW-trees for the following reason: Imagine we explore the tree node by node and keep a list of nodes logged as offspring of explored nodes, which haven't been explored yet. Then the length of this list can be seen as random walk on $\IZ$ with step size distribution $\xi-1$, started at 1. The tree is infinite if and only if this walk stays strictly positive. Some conditions chosen for the increments in this section might seem a bit arbitrary for now, but will appear natural later on, once we express the number of edges which the two players consider for play as a random walk on $\IZ$ (cf.\ \eqref{rw} below). Let us begin with a short proof of the well-known fact that random walk with upwards drift has a non-zero probability to never become negative if started at $0$. \begin{lemma}\label{drift} Let $(X_n)_{n\in\IN}$ be an i.i.d.\ sequence with $\IE(X_1)>0$. Then the infimum of the random walk $(S_n)_{n\in\IN_0}$, defined by $S_0=0,\ S_{n+1}=S_n+X_{n+1}$, is attained at $S_0=0$ with positive probability. \end{lemma} \begin{proof} By the law of large numbers it holds almost surely that \[\lim_{n\to\infty}\frac{S_n}{n} =\lim_{n\to\infty}\frac1n \sum_{k=1}^n X_k=\mathbb{E}(X_1)>0.\] Thus, there exists $n_0\in\mathbb{N}$ such that $\mathbb{P}\big(S_n >0\ \text{ for all } n\geq n_0\big)>0$. Since $n_0$ is finite, there must be some $n_1\in[0,n_0)$ such that $S_{n_1}$ is the minimum value of the walk with strictly positive probability. As a consequence, the event $\{\sum_{k=n_1+1}^n X_k\geq 0$ for all $n>n_1\}$ has non-zero probability. Due to the fact that $(X_n)_{n>n_1}$ and $(X_n)_{n\in\mathbb{N}}$ have the same distribution, the claim follows. \end{proof} Random walk on the integers with increments bounded from below by $-1$, i.e.\ $\Prob(X_n\geq -1)=1$, are commonly referred to as {\em left-continuous} or {\em skip-free to the left}. To avoid confusion with the offspring distribution $\xi$, let us write $(\pi_k)_{k\geq -1}$ for the probability mass function and \[\gamma(x)=\sum_{k=-1}^\infty \pi_k\,x^k=\frac{\pi_{-1}}{x}+\pi_0+\pi_1\,x+\pi_2\,x^2+\dots\] for the probability generating function of the corresponding increment distribution. Note that $\gamma$ is not defined in $x=0$ if $\pi_{-1}>0$ and that the sum defining $\gamma(x)$ is guaranteed to converge (absolutely) for $0<|x|\leq 1$. In the remainder of this section, we want to introduce the probabilities of different hitting times to be finite (and odd) for a left-continuous, non-monotonic random walk on $\IZ$ with positive drift starting at $0$, i.e.\ $(S_n)_{n\in\IN_0}$ defined by $S_0=0,\ S_n=S_{n-1}+X_{n}$, where $(X_n)_{n\in\IN}$ is an i.i.d.\ sequence, $\IE(X_1)>0$, $\Prob(X_1\geq-1)=1>\Prob(X_1\geq 0)$. To this end, let \begin{align*}\tau_{-1}&=\inf\{n,\;S_n=-1\}\quad\text{and}\\ \tau_0^+&=\inf\{n>0,\;S_n=0\} \end{align*} denote the hitting time of state $-1$ and the return time to starting state $0$ respectively. Further we define the following (conditional) probabilities: \begin{equation}\label{probs} \begin{gathered} \rho=\Prob(\tau_{-1}<\infty),\quad \sigma=\Prob(S_n>0\text{ for all }n\in\IN)\quad\text{and}\quad\theta=\Prob(\tau_0^+<\tau_{-1})\\ \text{as well as}\\ \rho_\mathrm{odd}=\Prob(\tau_{-1}\text{ is odd}\,|\,\tau_{-1}<\infty)\quad\text{and}\quad \theta_\mathrm{odd}=\Prob(\tau_0^+\text{ is odd}\,|\,\tau_0^+<\tau_{-1}). \end{gathered} \end{equation} To put it in words, $\rho$ is the probability that state $-1$ will eventually be visited and $\rho\cdot\rho_\mathrm{odd}$ the probability that this will happen for the first time after an odd number of steps. $\sigma$ denotes the probability that the walk never returns to starting state $0$ and $\theta$ is the probability that the walk first returns to $0$ before possibly visiting state $-1$. Finally, $\theta\cdot\theta_\mathrm{odd}$ is the probability that the walk performs a round trip started at $0$ without visiting $-1$ in an odd number of steps. For left-continuous walks, the strong Markov property immediately implies that $\rho$ equals the smallest solution in $[0,1]$ to $\gamma(x)=1$, cf.\ Lemma 2 in \cite{skipfree}. As we will consider increments of the form $\xi-2$, given $p_0=0<p_1$ and $\IE\xi>2$, we have a non-monotonic, left-continuous walk with positive drift and the corresponding probability is the unique solution in $(0,1)$ to $g(x)=x^2$, see Theorem \ref{i=0} below. In case the increments $\xi-2$ can be split in two (see Subsection \ref{separable}), the parity of the hitting times will play an important role, which is why we include the results of Lemma 3.2 and Thm.\ 3.3 in \cite{LCRW}, collected in a single lemma here: \begin{lemma}\label{calc} Let $(S_n)_{n\in\IN_0}$ be a left-continuous random walk with positive drift and $\pi_{-1}>0$, started in $S_0=0$. Then the following hold: \begin{enumerate}[(a)] \item $\pi_{-1}+\theta+\sigma=1$ \item $\sigma=\pi_{-1}\cdot \frac{1-\rho}{\rho}$ and $\theta=1-\frac{\pi_{-1}}{\rho}$ \item $\gamma\big(\rho\,(1-2\rho_\mathrm{odd})\big)=-1$ \item $\theta_\mathrm{odd}=\frac{\pi_{-1}\,(1-\rho_\mathrm{odd})}{\rho\theta\,(2\rho_\mathrm{odd}-1)}$ \end{enumerate} \end{lemma} \section{Different information regimes}\label{sec3} During the game, the tree is not necessarily fully known to the players at the start but can be gradually explored. What is visible to both Maker and Breaker -- besides the current set of internal and external nodes -- will vary, hence create different regimes in the subsequent analysis. The piece of information attached to every visible node $v$ will be denoted by $I(v)$. The strategies and winning probabilities will not only depend on the offspring distribution (as indicated above) but also the level of information, i.e.\ $I(v),\ v\in V,$ revealed to the players. Let us denote the probability of Breaker winning the game (given Breaker started it) by \[p:=\Prob(\text{Breaker succeeds to isolate the root $\mathbf{0}$ in a finite component}).\] Trivially, the probability of Breaker winning (no matter who starts) is bounded from below by $q$. In fact, writing $\bar{p}=\Prob(\text{Breaker wins}\,|\,\text{Maker starts})$ we can further conclude $p\geq\bar{p}\geq q$ due to the monotonicity in $G$ mentioned earlier. To avoid trivialities, we will therefore consider the game played on family trees corresponding to supercritical GW-processes, in other words offspring distributions with $\IE\xi>1$, hence $q<1$. Before we dig deeper into the different information regimes, let us state an overarching relationship between $p$ and $\bar{p}$, that will appear in each of the corresponding sections below: \begin{theorem}\label{arch} Irrespectively of $I(v)$, it holds $\bar{p}\geq g(p)$. In fact, in the regimes $I(v)=T_v$ and $I(v)=|T_v|$, as well as when $p_0=0$ in $I(v)=\emptyset$ equality holds. \end{theorem} The inequality follows immediately, considering the potential strategy of Breaker not to cut any edge incident to the root but to play the game on $T_v$ as a new game instead (in which Breaker begins), right after Maker fixates the edge $\langle\mathbf{0},v\rangle$. Then the Maker-Breaker game on $T$ (in which Maker starts) actually becomes a simultaneous play of $Z_1$ independent games (on independent copies of $T$, with Breaker starting). Given $Z_1=k$ nodes in the first generation, Breaker will win using this strategy if all $k$ subgames are lost for Maker, which by independence has probability $p^k$. Consequently, we get by conditioning on $Z_1$: \[\bar{p}\geq \sum_{k=0}^\infty p_k\, p^k=g(p).\] In the two regimes $I(v)=\emptyset$ and $I(v)=|T_v|$, there is complete symmetry in the edges considered for play. The strategy sketched above is therefore optimal from Breaker's point of view, as long as there are always edges not incident to the root available for play. Therefore, the inequality is actually an equality in the cases $I(v)=|T_v|$ as well as $p_0=0,\ I(v)=\emptyset$. The same holds for $I(v)=T_v$ in which the game is deterministic and Maker wins iff the root can be connected to a complete binary tree (cf.\ Lemma \ref{char} below). When Maker starts, Breaker wins iff for no $v$ in the first generation, $T_v$ contains a complete binary tree, hence by conditioning on $Z_1$: $\bar{p}=\sum_k p_kp^k$. For further details, see the corresponding results in the sections below. It might be worth mentioning that $\bar{p}=g(p)$ does not hold for $q>0$ (i.e.\ $p_0>0$, which is not covered by Theorem \ref{i=0}) in the regime $I(v)=\emptyset$, see Proposition \ref{ineq}. Finally, observe that Theorem \ref{arch} immediately implies $\bar{p}=1$ if $p=1$, since $1\geq\bar{p}\geq g(p)=g(1)=1$ then. \section{Instantly revealed tree, \texorpdfstring{$I(v)=T_v$}{I(v)=T\_v}}\label{sec:all} In the extreme case, where the whole tree $T$ is revealed to the players instantly, the game becomes deterministic (once $T$ is fixed). The possibility of either player winning the (1,1)-game then depends on whether or not $T$ contains an infinite complete binary tree rooted in $\mathbf{0}$, i.e.\ a family tree starting with progenitor $\mathbf{0}$ in which each individual has exactly two descendants. We will let $T_2$ denote the infinite complete binary tree and, for arbitrary $n\in\IN_0$, write $T_{2,n}$ for the 2-regular tree with $n$ levels, which is the full binary tree on $2^{n+1}-1$ vertices and the induced subgraph obtained when $T_2$ is cut at generation $n$. \begin{lemma}\label{char} In the Maker-Breaker game on a tree $T$ known to the players at the start, Maker wins if and only if $T$ contains a complete binary tree as (induced) subgraph rooted in $\mathbf{0}$. \end{lemma} \begin{proof} The sufficiency of this condition is easy to verify using the following simple pairing strategy: Maker chooses an infinite complete binary subtree $T_2$ of $T$ rooted in $\mathbf{0}$ and always fixates the edge connecting the (only) sibling in $T_2$ of the node Breaker in the same round disconnected from their common parent (arbitrary edges whenever Breaker picks an edge not in $T_2$ or the edge connecting the sibling to its parent is fixated already). That the condition is also necessary, we prove by induction on $n$, where $T_{2,n}$ is the smallest full binary tree that cannot be found in $T$ rooted in $\mathbf{0}$. If $n=1$, the root $\mathbf{0}$ has at most one child, which Breaker can disconnect in the first move. For $n>1$, at most one individual in the first generation can be the root of a copy of $T_{2,n-1}$ (otherwise there would be a $T_{2,n}$ rooted in $\mathbf{0}$). This direct descendant (if such exists) Breaker disconnects from the root in the first move. Any other $v$ in generation 1, that Maker can fixate is not the root of a $T_{2,n-1}$ and hence Breaker wins in $T_v$ by induction hypothesis. The general assumption that $\mathbf{0}$ has finite degree (a.s.)\ therefore concludes the argument. \end{proof} From this result (and its proof) we can immediately conclude the following: \begin{corollary} If Maker starts the Maker-Breaker game on a tree $T$ known to the players at the start, Maker wins if and only if there exists a child $v$ of $\mathbf{0}$ such that $T_v$ contains a complete binary tree as (induced) subgraph rooted in $v$. \end{corollary} Using this characterization, we can derive an equation for the winning probabilities of Maker/Breaker for general offspring distribution, which become particularly tidy if written in terms of $g$, the probability generating function of $\xi$, cf.\ \eqref{pgf}. \begin{theorem}\label{formula} For the (1,1)-game on a GW-tree $T$ arising from offspring distribution $\xi$ and revealed to both players at the start, it holds \begin{equation}\label{fp2} p=g(p)+(1-p)\cdot g'(p)\quad \text{and}\quad\bar{p}=g(p). \end{equation} \end{theorem} \begin{proof} According to Lemma \ref{char}, the probability $1-p$ of Maker winning the game equals the probability of $T$ containing a copy of $T_2$ rooted in $\mathbf{0}$. The latter can be described recursively with respect to the number of children the root has. \begin{align*} p&=\Prob(\mathbf{0} \text{ is not the root of a copy of }T_2)\\ &=\Prob(\mathbf{0} \text{ does not have two children being the root of a copy of }T_2)\\ &=\sum_{k=0}^\infty \Prob(\xi_\mathbf{0}=k)\cdot \Prob(X_k< 2), \end{align*} where $X_k\sim\mathrm{Bin}(k,1-p)$, as every node in generation 1 is the root of an independent copy of $T$ and the probability that a fixed such copy contains $T_2$ sharing the root is consequently again $1-p$. Plugging in the probabilities coming from the binomial distributions and rewriting the equation (mainly for aesthetic and computational convenience), we arrive at \[p=p_0+p_1+\sum_{k=2}^\infty p_k\cdot\big(p^k+k\,(1-p)\,p^{k-1}\big)=g(p)+(1-p)\cdot g'(p).\] In the same vein, the probability that Breaker wins when Maker starts, can be written as \[\bar{p}=\sum_{k=0}^\infty \Prob(\xi_\mathbf{0}=k)\cdot\Prob(X_k=0)=\sum_{k=0}^\infty p_k\cdot p^k=g(p),\] which establishes the second claim. \end{proof} The recursive equation for $p$, in this regime the probability that the underlying GW-tree does not contain a complete binary tree sharing the root, was already established more than 30 years ago, cf.\ Thm.\ 1 in \cite{Dek1}. It is further straightforward to verify that $p$ is in fact the smallest solution to \eqref{fp2} in the interval $[0,1]$, see the above reference for details. \subsection{A few tractable examples} With this in hand, we can tackle a few simple cases. As the equation is implicit in $p$, however, the pgf $g$ makes it too hard to solve \eqref{fp2} analytically in most cases, even for standard distributions for $\xi$. From the monotonicity in the underlying graph, we generally get a phase transition in the parameter(s) of the offspring distribution $\xi$ in the sense that $p$ equals $1$ for the stochastically smaller end of the sprectrum, followed by a jump to values smaller than one at the corresponding critical value (denoted $s_\mathrm{c}, \lambda_\mathrm{c}, r_\mathrm{c}$ in the sequel). \begin{proposition}[Geometric distribution]\label{geoprop} The probability of Breaker winning on an instantly revealed GW-tree with offspring distribution \begin{enumerate}[(a)] \item $\xi\sim\mathrm{Geo}_{\IN}(s)$, i.e.\ $p_k=s\,(1-s)^{k-1}$ for $k\geq 1$, equals \[p=\frac{1-\sqrt{1-4s}}{2\,(1-s)}\quad\text{and}\quad \bar{p}=\frac{1-2s-\sqrt{1-4s}}{2\,(1-s)}=p-\frac{s}{1-s}\] respectively (depending on who starts the game) for parameter $s\in(0,\frac14]$. Further it holds $p=\bar{p}=1$, for $s\in(\frac14,1]$. \item $\xi\sim\mathrm{Geo}_{\IN_0}(s)$, i.e.\ $p_k=s\,(1-s)^{k}$ for $k\geq 0$, equals \[p=\frac{1+s-\sqrt{(1-s)\,(1-5s)}}{2\,(1-s)}\text{ and }\ \bar{p}=\frac{1-s-\sqrt{(1-s)\,(1-5s)}}{2\,(1-s)}=p-q\] respectively (depending on who starts the game) for parameter $s\in(0,\frac15]$. Further it holds $p=\bar{p}=1$, for $s\in(\frac15,1]$. \end{enumerate} \end{proposition} \begin{proof} \begin{enumerate}[(a)] \item In the case of $\xi\sim\mathrm{Geo}_{\IN}(s)$, the corresponding pgf and equation \eqref{fp2} read \[g(x)=\frac{sx}{1-(1-s)x}\quad\text{and}\quad p=\frac{sp}{1-(1-s)p}+\frac{s(1-p)}{(1-(1-s)p)^2}\] respectively. The latter has solutions $p=1$ and if $s\leq\frac14$ additionally \[p=\frac{1\pm\sqrt{1-4s}}{2(1-s)}.\] This together with the fact that $p$ is the smallest solution to \eqref{fp2} in $[0,1]$ establishes the first part of the claim. To arrive at $\bar{p}$ we simply have to evaluate $g(p)$, cf.\ Theorem \ref{formula}: \[p=\frac{1-\sqrt{1-4s}}{2\,(1-s)}\quad\text{and}\quad \bar{p}=\frac{1-2s-\sqrt{1-4s}}{2\,(1-s)}=p-\frac{s}{1-s}\] \item In the case of $\xi\sim\mathrm{Geo}_{\IN_0}(s)$ -- number of failures until first success in a Bernoulli-trials experiment -- the pgf and \eqref{fp2} read \[g(x)=\frac{s}{1-(1-s)x}\quad\text{and}\quad p=\frac{s}{1-(1-s)p}+\frac{s(1-s)(1-p)}{(1-(1-s)p)^2}\] respectively. The latter has solutions $p=1$ and if $s\leq\frac15$ additionally \[p=\frac{1+s\pm\sqrt{(1-s)(1-5s)}}{2(1-s)}.\] Since the extinction probability equals $q(s)=\frac{s}{1-s}$ in this case, we can rewrite $\bar{p}$ given by $g(p)$ as $p-q$.\vspace{-1em} \end{enumerate} \end{proof} \vspace{-1em} \noindent Note that the difference $p-\bar{p}$ equals $\frac{s}{1-s}$ in both cases, while the extinction probability is 0 in the first case, since $\xi\sim\mathrm{Geo}_{\IN}(s)$ guarantees at least one descendant for each individual (i.e.\ $p_0=0$). Further it is worth mentioning that the phase transition at the critical values $s_\mathrm{c}=\frac14$ and $s_\mathrm{c}=\frac15$ respectively is a discontinuous one as the probabilities of Breaker winning jump to $1$, see Figure \ref{Geo} below for an illustration. The critical parameters in both cases correspond to a mean value of $\mu=4$ for the respective offspring distribution. The calculation of $1-p$ for $\xi\sim\mathrm{Geo}_{\IN_0}(s)$ was included as a special case in Section 3 of \cite{Dek2}. \begin{figure}[H] \centering \includegraphics[scale=1]{Geo} \caption{Winning probabilities for Breaker on a GW-tree, which arises from a geometric offspring distribution on $\IN$ (left) respectively $\IN_0$ (right) and is revealed at the beginning, depending on the corresponding parameter $s\in[0,1]$. Depicted in green, the corresponding extinction probability of the tree. \label{Geo}} \end{figure} In case the random number of decendants per individual follows a Poisson distribution, equation \eqref{fp2} is no longer analytically solvable. But one can still use Theorem\ \ref{formula} to numerically compute the winning probabilities. \begin{example}[Poisson distribution] For $\xi\sim\mathrm{Poi}(\lambda)$, the pgf reads $g(x)=\mathrm{e}^{\lambda(x-1)}$ and consequently, \eqref{fp2} becomes \[p=\big(1+\lambda(1-p)\big)\mathrm{e}^{\lambda(p-1)}\] or $1+\frac{x}{\lambda}=(1-x)\,\mathrm{e}^x$, where we substituted the exponent by $x$. For small $\lambda>0$, this equation has only the trivial solution $x=0$ (which translates to $p=1$). Increasing $\lambda$, a non-trivial solution enters the picture once the linear function on the left is a tangent to the graph of $x\mapsto (1-x)\,\mathrm{e}^x$ in some $x_0<0$. Using this gradient condition we find that $x_0$ is a solution to $\mathrm{e}^{-x}=1-x+x^2$, which leads to the critical parameter $\lambda_\mathrm{c}= 3.3509188715\dots$ For this mean in $\xi\sim\mathrm{Poi}(\lambda)$, the winning probability of Breaker (using again Lemma \ref{char} and Theorem\ \ref{formula}) drops from 1 to $p_\mathrm{c}=0.46483869\dots$, see Figure \ref{Poi1} below for an illustration. Having evaluated $p$ numerically, we can calculate $\bar{p}=g(p)$ as before. \begin{figure}[H] \centering \includegraphics[scale=1]{Poi1} \caption{Winning probabilities for Breaker on a GW-tree, which arises from a Poisson offspring distribution, $\xi\sim\mathrm{Poi}(\lambda)$. \label{Poi1}} \end{figure} It is worth noting that the mean in the critical case for a Poisson offspring distribution equals $\lambda_\mathrm{c}\neq 4$. Consequently, the fact that the phase transition with respect to $q$ (extinction) occurs at $\mu=1$ for all (non-constant) offspring distributions (cf.\ Theorem \ref{extinction}) does not have an equivalent, when it comes to the phase transition with respect to $p$ (absence of a complete binary subtree rooted at $\mathbf{0}$). This was already established in \cite{Dek1} by a simple counter-example, based on the fact that changing $p_0$ and $p_1$, while keeping $p_0+p_1$ constant, changes all moments $\IE(\xi^n),\ n\in\IN,$ but not $p$. \end{example} \subsection{Conditions for Maker to have a chance of winning} In \cite{Dek2} both a necessary and a sufficient condition for the existence of a complete $N$-ary subtree rooted at the ancestor $\mathbf{0}$ of a GW-tree for arbitrary offspring distribution has been established. For $N=2$ this refers to a binary tree, more specifically $p<1$ in our setting (cf.\ Lemma \ref{char}). \begin{proposition}\label{bound} Consider the $(1,1)$-Maker-Breaker game on an instantly revealed GW-tree arising from a general offspring distribution $\{\Prob(\xi=k)=p_k,\,k\in\IN_0\}$, in which Breaker starts. Then the following conditions hold: \begin{enumerate}[(a)] \item Maker has a positive chance of winning (p<1), if \begin{equation*} \IE\Big(\frac{1}{\xi+1}\Big)\leq\frac14+\frac{p_0}{2}+\frac14\,(p_0+p_1)^2. \end{equation*} \item If $g''(x)<\frac{1}{1-x}$ for all $x\in(0,1)$, the game is an a.s.\ win for Breaker (p=1). \end{enumerate} \end{proposition} This result follows directly from Thm.\ 3 in \cite{Dek2}, considering the special case $N=2$. \begin{example}[Binomial distribution] For $\xi\sim\mathrm{Bin}(n,r)$, we have $p_0=(1-r)^n$ and $p_1=nr\,(1-r)^{n-1}$ as well as $g(x)=(1-r+rx)^n$. Solving \eqref{fp2} numerically, we can find the critical values $r_\mathrm{c}(n)$ and calculate the winning probabilities for Breaker in both cases, $p(n,r)$ for Breaker starting and $\bar{p}(n,r)$ for Maker starting the game, in the same way as before, see Figure \ref{Bin2} and the table below for numerical results. \begin{figure}[H] \centering \includegraphics[scale=1]{Bin2} \caption{Winning probabilities for Breaker on an instantly revealed GW-tree, which arises from a Binomial offspring distribution, $\xi\sim\mathrm{Bin}(n,r)$, for $n=5$ (left) and $n=10$ (right). \label{Bin2}} \end{figure} \begin{center} \begin{tabular}{ m{0.2cm}||c|c|c|c|c|c|c|c|c|c @{}m{0cm}@{}} \hline $n$&1&2&3&4&5&6&7&8&9&10& \\[0.3cm] \hline $r_\mathrm{c}$&\small1&\small1&$\nicefrac89$&\small0.7248&\small0.6028&\small0.5137&\small0.4468&\small0.3949&\small0.3537&\small0.3202& \\[0.1cm] $p_\mathrm{c}$&\small1&\small1&$\nicefrac{5}{32}$&\small0.2584&\small0.3105&\small0.3418&\small0.3625&\small0.3773&\small0.3883&\small0.3969& \\[0.cm] \hline \end{tabular} \captionof{table}{Critical parameter $r_\mathrm{c}$ and the corresponding winning probability for Breaker, when $\xi\sim\mathrm{Bin}(n,r)$ and small values of $n$.} \end{center} In order to evaluate the bounds established in Proposition \ref{bound}, we use Fubini to get \[\IE\Big(\frac{1}{\xi+1}\Big)=\int_0^1\IE(x^\xi)\,dx=\int_0^1 (1-r+rx)^n\,dx=\frac{1-(1-r)^{n+1}}{r\,(n+1)}.\] So according to part (a) of the proposition, $p(n,r)<1$ holds for pairs $(n,r)$ which fulfill \[\frac{1-(1-r)^{n+1}}{r\,(n+1)}\leq\frac14+\frac12\,(1-r)^n+\frac14\,\big((1-r)^n+nr\,(1-r)^{n-1}\big)^2.\] For fixed $n$, this can be solved numerically and leads to a value $r_>(n)$, above which Maker has a positive chance of winning the game. Turning to part (b), it is easy to see that $g''(x)=0$ for $n=1$ (hence the inequality holds trivially on $(0,1)$ for arbitrary $r$) and $g''(x)=2r^2$ for $n=2$, which guarantees an a.s.\ Breaker win for $r\geq\frac{1}{\sqrt{2}}$. For $n\geq3$, the inequality \[n(n-1)r^2\,(1-r+rx)^{n-2}<\frac{1}{1-x}\] holds for $r\geq0$ sufficiently small and increasing it, the function on the left hand side coincides with the right hand side in $(0,1)$ for the first time when also the derivatives of both functions coincide at this point. Using this to simplify the condition, we find that the inequality holds for all $0<x<1$ if $r$ is less than $r_<(n)=\frac1n\big(\frac{n-1}{n-2}\big)^{n-2}$. Consequently, the critical parameter $r_\mathrm{c}(n)$, which separates the trivial regime ($p=1$) from the competitive one ($p<1$), lies in the interval $[r_<(n),r_>(n)]$. In Figure \ref{Bin1}, the critical values and the corresponding bounds are illustrated for $n\leq 15$. \begin{figure}[H]\hspace{-0.3cm} \includegraphics[scale=1]{Bin1} \caption{Depicted are the threshold $r_\mathrm{c}$ (green) plus corresponding bounds $r_<$ (red) and $r_>$ (blue), coming from Proposition \ref{bound}, for Binomial offspring distribution and small values of $n$. \label{Bin1}} \end{figure} Note that $\lim_{n\to\infty}n\,r_<(n)=\mathrm{e}$, which is consistent with the corresponding lower bound for the Poisson distribution, see below. \end{example} Evaluating the bounds from Proposition \ref{bound} also for the cases examined earlier, we get for $\xi\sim\mathrm{Geo}_{\IN}(s)$ that $p<1$ for $s\leq0.21332$ and $p=1$ for $s\geq0.29629$, as opposed to the true threshold $s_\mathrm{c}=\frac14$ (cf.\ Proposition \ref{geoprop}). With offspring distribution $\xi\sim\mathrm{Geo}_{\IN_0}(s)$ instead, the proposition guarantees $p<1$ for $s\leq0.16401$ and $p=1$ for $s\geq 0.22857$, while $s_\mathrm{c}=\frac15$. Having Poisson distributed offspring numbers, $\xi\sim\mathrm{Poi}(\lambda)$, we find that $\lambda\geq3.654328$ guarantees $p<1$ and $\lambda\leq\mathrm{e}$ ensures $p=1$, while the true threshold is $\lambda_\mathrm{c}= 3.3509188715\dots$ \section{No extra information, \texorpdfstring{$I(v)=\emptyset$}{I(v)=0}}\label{empty} In the most restrictive regime, in which no further information is revealed but the internal and external nodes, the strategies are simply totally random. As there is complete symmetry among all external nodes (being the root of an independent copy of our initial branching process) both players in turn remove/fixate an edge between an internal and external node picked arbitrarily. The number of available edges/external nodes (taken at times when it is Maker's turn) is hence a random walk on $\IZ$ with stepsize $\xi-2$ (observe that both Maker and Breaker use up one edge and a random number of $\xi$ is added during Maker's turn). Revealing the root $\mathbf{0}$, there are $\xi_\mathbf{0}$ external nodes available; hence the initial value of the corresponding random walk is either 1 (if Breaker starts) or 2 (if Maker starts). Let us formally define the \emph{embedded random walk} \begin{equation}\label{rw} S_0=1,\quad S_{n}=S_{n-1}+(\xi_n-2)\quad\text{for}\ n\in\IN, \end{equation} where $(\xi_n)_{n\in\IN}$ is an i.i.d.\ sequence of copies of $\xi$, the offspring distribution. Then the probability of Maker winning becomes $1-p=\Prob(S_n>0 \text{ for all }n\in\IN)$. Let us first consider offspring distributions concentrated on $\{1,2,3\}$, i.e.\ $\Prob(\xi\in\{1,2,3\})=1$, as this condition makes the number of external nodes available to Maker a {\em homogeneous discrete birth-and-death chain}. Due to the fact that the (lazy) simple symmetric random walk on $\IZ$ is recurrent, Breaker will obviously win the game with probability 1 if played on a GW-tree arising from any offspring distribution with \[p_1=p_3=r,\ p_2=1-2r,\quad\text{for arbitrary }0<r\leq\tfrac12,\] where $p_k=\Prob(\xi=k)$ as before. By a simple coupling argument (or monotonicity of Maker-Breaker in the underlying graph), more generally $0\neq p_1\geq p_3$ gives Breaker an almost sure win (even if Maker starts, i.e.\ $\bar{p}=p=1$). However, if $p_3>p_1$ (which makes the birth-and-death chain transient) we can conclude a first non-trivial result for this regime: \begin{proposition}\label{123} In the regime $I(v)=\emptyset$, if $p_1+p_2+p_3=1$ and $p_1<p_3$, then the probability of Breaker winning on a GW-tree arising from offspring distribution $\xi$, with $\Prob(\xi=k)=p_k$, equals \[p=\frac{p_1}{p_3}\quad\text{and}\quad \bar{p}=\left(\frac{p_1}{p_3}\right)^2\] respectively, depending on who starts the game. \end{proposition} \begin{proof} We can assume $p_2=0$, hence $p_3=1-p_1$, w.l.o.g.\ as including the possibility of loops will not change the probability of ever hitting $0$. Writing $\theta_k:=\Prob(\text{chain hits 0 after start in }k)$, we get $\theta_0=1$ and $\theta_2=\theta_1^2$, by the strong Markov property applied to the first time hitting 1 after start in 2. Conditioning on the first step, this gives the equation \[\theta_1=p_1\cdot\theta_{0}+p_3\cdot\theta_{2}=p_1+(1-p_1)\cdot\theta_{1}^{\,2},\quad\text{with solutions }\theta_1=\frac{1\pm |1-2p_1|}{2\,(1-p_1)}.\] The fact that $p_1<\frac12$ by assumption and $\theta_1<1$ by transience of the chain, leads to $\theta_1=\frac{p_1}{1-p_1}$. For further details, see e.g.\ Chap.\ 2.5 in \cite{LPW}. \end{proof} Note that $p_1<p_3$ coincides with the intuition that $\mu=2+(p_3-p_1)$ has to be larger than 2 for Maker to have a chance. The above line of argument can in fact be used to settle even the case of a geometric offspring distribution $\xi\sim\text{Geo}_{\IN}(s)$: \begin{proposition}[Geometric distribution]\label{emptygeo} When $I(v)=\emptyset$, the probability of Breaker winning on a GW-tree arising from offspring distribution $\xi\sim\mathrm{Geo}_{\IN}(s)$ equals for fixed $s\in[0,\frac12]$ \[p=\frac{s}{1-s}\quad\text{and}\quad \bar{p}=\left(\frac{s}{1-s}\right)^2\] respectively, depending on who starts the game. In addition to that, for $s\in(\frac12,1]$ it holds $p=\bar{p}=1$. \end{proposition} \begin{proof} Let $(X_k)_{k\in\IN}$ be a sequence of i.i.d.\ $\mathrm{Ber}(s)$ random variables (commonly referred to as Bernoulli trials). Set $\xi_0=0$ and $\xi_n=\min\{k\in\IN:\; X_{\xi_{n-1}+k}=1\}$ for $n\in\IN$. Then the sequence $(\xi_n)_{n\in\IN}$ is i.i.d.\ $\mathrm{Geo}_{\IN}(s)$. To recycle the idea of proof used in Proposition \ref{123} it is crucial to notice that \[\xi_1=\sum_{k=1}^{\xi_1}1=2+\sum_{k=1}^{\xi_1}(1-2X_k)\] and more generally \[\sum_{k=1}^{n}(\xi_k-2)=\sum_{k=1}^{S_n}(1-2X_k),\] where $S_n=\sum_{k=1}^n\xi_k$. As the left hand side describes a random walk on $\IZ$ with stepsize distribution $\xi-2$ and the right hand side a homogeneous discrete birth-and-death chain with birth probability $1-s$ and its steps grouped according to the sequence of down-steps (deaths), the probability of staying strictly positive is the same for both and the claim follows. \end{proof} In fact, with a similar reasoning as in the proof of Proposition \ref{123}, we can derive an equation for the winning probability of Breaker in this regime, that holds for general {\em strictly positive} offspring distribution. \begin{theorem}\label{i=0} Consider $\xi$ with $p_0=0$, where as before $p_k=\Prob(\xi=k)$, for $k\in\IN_0$, is the distribution's pmf and $g$ its pgf. Then the probability $p$ of Breaker winning on a GW-tree arising from offspring distribution $\xi$, when the nodes carry no further information is a solution to \begin{equation}\label{fp1} x^2=g(x). \end{equation} It is equal to $0$ if $p_1=0$, to $1$ if $\mu\leq 2,\ p_1>0$ and to the (unique) solution in $(0,1)$ in all non-trivial cases ($\mu>2,\ p_1>0$). When Maker begins, the probability of Breaker winning becomes $\bar{p}=g(p)=p^2$. \end{theorem} \begin{proof} Note that the condition $p_0=0$ makes the embedded random walk $(S_n)_{n\in\IN_0}$ described in \eqref{rw} left-continuous, i.e.\ its down steps are of size at most 1. Hence, we can write the probability $p=\Prob(S_n=0 \text{ for some }n\in\IN\,|\,S_0=1)$ by conditioning on the first step as \begin{align*}p&=p_1+\sum_{k=0}^\infty \Prob(\xi_1-2=k)\cdot\Prob(S_n=0 \text{ for some }n>1\,|\, S_1=k+1)\\ &=p_1+\sum_{k=2}^\infty p_k\cdot p^{k-1}=\frac{g(p)}{p}, \end{align*} where the strong Markov property was used in the second step and $p\neq0=p_0$ in the last one. Since $g(0)=p_0=0$, both $x=0$ and $x=1$ are general solutions to \eqref{fp1}. If $p_1=0$, the random walk $(S_n)_{n\in\IN_0}$ is non-decreasing and trivially Maker wins with probability 1. In case $p_1>0$, the walk $(S_n)_{n\in\IN_0}$ is not constant. If further $\mu\leq 2$, it does not have positive drift and will therefore visit state $0$ almost surely, i.e.\ $p=1$. For the case $p_1>0,\ \mu>2$, let us consider the function $h(x):=g(x)-x^2$. The sign of its derivative $h'$ can change at most twice, due to the convexity of $g'$. Since $h(0)=h(1)=0$ and $h'(0)=p_1>0,\ h'(1)=\mu-2>0$, it has exactly one zero in $(0,1)$. That in this case $(S_n)_{n\in\IN_0}$ is transient with positive drift (and therefore $0<p<1$, cf.\ Lemma \ref{drift}), completes the proof of the first part. If Maker starts the game, which translates to $S_0=2$, we get by left-continuity of the walk \begin{align*} \bar{p}&=\Prob(S_n=0 \text{ for some }n\in\IN\,|\,S_0=2)\\ &=\Prob(S_n=0 \text{ for some }n\in\IN\,|\,S_0=1)^2=p^2=g(p), \end{align*} which settles the second part of the claim. \end{proof} Noting that for $\xi\sim\mathrm{Geo}_\IN(s)$ it holds $p_0=0$ and $g(x)=\frac{sx}{1-(1-s)x}$, the result from Proposition \ref{emptygeo} can easily be recovered using the above theorem. \begin{example}\label{oom} Let us consider the Maker-Breaker game, played on a GW-tree with what was colloquially called ``one-or-many'' offspring distribution in the last section of \cite{Dek2}, i.e.\ $p_n=1-p_1=r$ for some $r\in(0,1)$ and $n\geq 2$. Then the tree is infinite and $g(x)=(1-r)\,x+rx^n$. For $r>\frac{1}{n-1}$, the game is non-trivial ($\mu>2$) and the winning probability of Breaker when starting the game is the (unique) $x\in(0,1)$ solving $x=(1-r)+rx^{n-1}$. In order to make the example a little less abstract, for the choice of e.g.\ $n=4$ this becomes \[p=\frac{\sqrt{r\,(4-3r)}-r}{2 r}\quad\text{and}\quad\bar{p}=\frac{2-r-\sqrt{r\,(4-3r)}}{2r},\] illustrated in Figure \ref{Oom} below. \begin{figure}[H] \hspace{-0.3cm} \includegraphics[scale=1]{OoM} \caption{\textbf{Left:} For $r(n-1)>1$, the function $x\mapsto(1-r)+rx^{n-1}$ intersects exactly once with the identity function on (0,1). \textbf{Right:} Winning probabilities for Breaker on a tree with one-or-many offspring distribution, where ``many'' refers to $n=4$.\label{Oom}} \end{figure} \end{example} To extend the analysis beyond the scope of Theorem \ref{i=0}, i.e.\ offspring distributions with $p_0>0$, we first verify that $\bar{p}>g(p)$ in this case. \begin{proposition}\label{ineq} Consider $\xi$ with $p_0>0$. Then the probability $\bar{p}$ of Breaker winning the game Maker starts on a GW-tree $T$ arising from offspring distribution $\xi$, when the nodes carry no further information is strictly bigger than $g(p)$ in the non-trivial case (i.e.\ $p<1$). \end{proposition} \begin{proof} The difference to the case $p_0=0$ examined in Theorem \ref{arch} is that Maker might conclude the exploration of a subtree rooted at a vertex in the first generation by fixating an edge incident to a leaf in the tree. Then Breaker will consequently erase an edge incident to the root next. To be more precise, consider the embedded random walk $(S_n)_{n\in\IN_0}$ with $S_0=2$ and increments distributed as $\xi-2$ and let $\tau_{\leq0}=\inf\{n\in\IN,\,S_n\leq0\}$ be the first time the walk exits $\IN$ (possibly $\tau_{\leq0}=\infty$). In contrast to left-continuous random walk, $S_{\tau_{\leq0}}=-1$ has positive probability when $p_0>0$, so let $\alpha=\Prob(S_{\tau_{\leq0}}=-1)$. Following a depth-first strategy is optimal for both players, by symmetry in the external nodes. Given the root $\mathbf{0}$ has $k\geq1$ children, this will amount to $1+Y$ consecutive games on independent trees distributed like $T$ in which Breaker starts, where $Y\sim\mathrm{Bin}(k-1,1-\alpha)$: Whenever Maker makes the last possible move in a subtree rooted in the first generation, which corresponds to the event $\{S_{\tau_{\leq0}}=-1\}$, Breaker erases the next one. So the probability for a win of Breaker in the game started by Maker is given by \[\Prob(\text{Breaker wins}\,|\,\text{Maker starts, }Z_1=k)=\IE(p^{Y+1})=p\big(\alpha+(1-\alpha)p\big)^{k-1}.\] If $p_0=0$ (hence $\alpha=0$), this term equals $p^k$. Given $p<1$ and $p_0>0$, however, it is strictly bigger than $p^k$ for $k>1$. This entails \[\bar{p}=p_0+\sum_{k=1}^\infty p_k\cdot p\big(\alpha+(1-\alpha)p\big)^{k-1}>\sum_{k=0}^\infty p_k\,p^k=g(p),\] where $p_0+p_1<1$ (implied by $p<1$) was used. \end{proof} \subsection{Separable offspring distributions}\label{separable} Let us now consider the case in which $\xi$ can be written as the sum of two i.i.d.\ integer-valued random variables: \begin{definition} Let us call the distribution of $\xi$ {\em separable}, if there exist independent and identically distributed integer-valued random variables $X_1$ and $X_2$, such that \begin{equation*}\label{sep} \xi\stackrel{d}{=}X_1+X_2. \end{equation*} \end{definition} At first glance, this might seem to be a fairly special case. However, both the Poisson distribution $\mathrm{Poi}(\lambda)$ and the negative binomial distribution $\mathrm{NB}(r,s)$, generalized to $r$ not necessarily being integer (occasionally referred to as {\em Plya distribution}), hence also the geometric distribution $\mathrm{Geo}_{\IN_0}(s)$, are even infinitely divisible (i.e.\ can be written as the sum of arbitrarily many i.i.d.\ random variables). Since the corresponding summands are integer-valued, \[\mathrm{Poi}(\lambda)=\mathrm{Poi}(\tfrac\lambda2)\ast\mathrm{Poi}(\tfrac\lambda2)\quad\text{resp.} \quad\mathrm{NB}(r,s)=\mathrm{NB}(\tfrac r2,s)\ast\mathrm{NB}(\tfrac r2,s),\] these distributions are also separable in the sense above. The binomial distribution $\mathrm{Bin}(n,r)$ with an even number $n=2k$ of trials is trivially separable, as $\mathrm{Bin}(k,r)\,\ast \,\mathrm{Bin}(k,r)=\mathrm{Bin}(2k,r)$, but not infinitely divisible. For odd $n$, $\mathrm{Bin}(n,r)$ can not even be the convolution of two i.i.d.\ random variables: Assuming for contradiction that it was, the (independent) summands $X_1,X_2$ would need to attain the values $0$ and $\frac n2$ with positive probability (so that both $0$ and $n$ can be attained by their sum, but neither negative numbers nor values bigger than $n$). Since $\frac n2$ in this case is not an integer, but $\Prob(X_1+X_2=\frac n2)>0$, we can conclude that $X_1+X_2\nsim\mathrm{Bin}(n,r)$, a contradiction. Important to observe is that if $\xi$ is separable, we get $\xi-2\stackrel{d}{=}(X_1-1)+(X_2-1)$ and can consider an extended walk $(\tilde{S}_n)_{n\in\IN_0}$ with increments distributed as $X_1-1$ and $\tilde{S}_0=S_0$, essentially splitting the steps of the embedded random walk in two. Then the probability of the embedded walk to eventually hit $0$ (or $-1$) can be calculated as $\Prob(\tilde{S}_{2n}\leq0\text{ for some }n\in\IN)$ using the results collected in Lemma \ref{calc}, as was done in \cite{LCRW}. In this way, using Cor.\ 4.5 in \cite{LCRW}, we can calculate $p$ (and $\bar{p}$) for the Maker-Breaker game in the regime $I(v)=\emptyset$ on GW-trees based on most standard offspring distributions, with the exception of $\xi\sim \mathrm{Bin}(n,r),$ where $n$ is odd. \begin{theorem}\label{septhm} If the offspring distribution is separable, i.e.\ $\xi\stackrel{d}{=}X_1+X_2$ with i.i.d.\ $(X_1,X_2)$, and $\IE\xi>2$ then the probabilities of Breaker winning on the corresponding GW-tree with the nodes carrying no further information is given by \begin{equation}\label{sep1} p=\rho\,\bigg(1-\frac{\sigma\cdot\rho_\mathrm{odd}}{1-\theta\cdot(1-\theta_\mathrm{odd})}\bigg) \end{equation} and \begin{equation}\label{sep2} \bar{p}=\rho^2\,\bigg(1-\frac{2\sigma\cdot\rho_\mathrm{odd}\,(1-\rho_\mathrm{odd})}{1-\theta\cdot(1-\theta_\mathrm{odd})}\bigg) \end{equation} depending on whether Breaker or Maker starts the game. The (conditional) probabilities $\rho$, $\sigma$ and $\theta$ as well as $\rho_\mathrm{odd}$ and $\theta_\mathrm{odd}$ appearing here refer to the ones introduced in \eqref{probs}, corresponding to the left-continuous random walk on $\IZ$ starting at $0$ with i.i.d.\ increments distributed as $X_1-1$. \end{theorem} \begin{proof} Cor.\ 4.5 in \cite{LCRW} applies almost perfectly to the embedded walk $(S_n)_{n\in\IN_0}$ here, just the walk there is shifted downwards by one (hitting $\IZ\setminus\IN_0$ instead of $\IZ\setminus\IN$). Thus, our winning probabilities for Breaker, $p$ and $\bar{p}$, correspond to a start in $k=0$ and $k=1$ respectively there. \end{proof} \begin{example}[Poisson distribution] Consider the Maker-Breaker game, played on a GW-tree based on a Poisson offspring distribution $\mathrm{Poi}(\lambda)$. For $\lambda> 2$, the game is non-trivial and the winning probability of Breaker in the regime $I(v)=\emptyset$ are given by \eqref{sep1} and \eqref{sep2} respectively. Choosing a numerical value for the parameter, e.g.\ $\lambda=3$, we get $X_1\sim\mathrm{Poi}(\frac32)$, hence approximately $\rho=0.417188$, $\sigma=0.311713,$ $\theta=0.465157$, $\rho_\mathrm{odd}=0.706513$ and $\theta_\mathrm{odd}=0.817032$, which result in $p=0.31699$ and $\bar{p}=0.14967>0.12886=g(p)$. The corresponding probabilities for small values of $\lambda$ are depicted in Figure \ref{Poi3}. It is worth mentioning, that the phase transition in this case is continuous as opposed to the regime $I(v)=T_v$. \begin{figure}[H] \centering \includegraphics[scale=1]{Poisep} \caption{Winning probabilities for Breaker on a GW-tree, which arises from a Poisson offspring distribution, $\xi\sim\mathrm{Poi}(\lambda)$. \label{Poi3}} \end{figure} \end{example} \begin{example}[Geometric distribution] If the game is played on a GW-tree based on a $\mathrm{Geo}_{\IN_0}(s)$ offspring distribution instead, it is non-trivial for $s<\frac13$ and we get $X_1\sim\mathrm{NB}(\frac12,s)$. As before we can calculate the corresponding (conditional) probabilities for the extended left-continuous walk using Lemma \ref{calc} after selecting a numerical value for the parameter. The choice $s=\frac14$ for example gives: \begin{equation*}\begin{split}\rho=\tfrac16\,(1+\sqrt{13}),\quad \sigma=\tfrac14\,(\sqrt{13}-3),\quad \theta=\tfrac14\,(5-\sqrt{13}),\\ \rho_\mathrm{odd}=\tfrac{1}{12}\,(13-\sqrt{13})\quad \text{and} \quad \theta_\mathrm{odd}=\tfrac{1}{12}\,(5+\sqrt{13}),\end{split} \end{equation*} which leads to $p=\frac23$ and $\bar{p}=\frac59>\frac12=g(p)$ in this case. \end{example} \subsection{Bounds by coupling} The monotonicity in the underlying graph also allows us to derive bounds for the winning probabilities of Breaker by means of comparison with the game on other GW-trees with (preferably) more tractable offspring distribution. While the binomial distribution $\mathrm{Bin}(n,r),$ where $n$ is odd, is not separable, hence eludes our analysis in the foregoing subsection, it holds \[\mathrm{Bin}(n-1,r)\preceq_{st}\mathrm{Bin}(n,r)\preceq_{st}\mathrm{Bin}(n+1,r).\] Hence the probabilities for Breaker to win on the corresponding trees have the reversed order (as switching to a subgraph favors Breaker). \begin{example}[Binomial distribution] To get bounds for $p$ and $\bar{p}$ relating to the Maker-Breaker game on the GW-tree with offspring distribution $\xi\sim\mathrm{Bin}(13,\frac14)$ for instance, one can calculate the corresponding probabilities for $\xi\sim\mathrm{Bin}(12,\frac14)$ and $\xi\sim\mathrm{Bin}(14,\frac14)$ respectively via Theorem \ref{septhm} in order to conclude $0.2482>p>0.1367$ and $0.0957>\bar{p}>0.0383$. Clearly, in this case the bounds are not particularly sharp, but for the Binomial distribution the winning probabilities for Breaker get a lot smaller, hence closer to each other as $n$ increases. As points of reference, $\bar{p}=p=1$ for offspring distributions $\mathrm{Bin}(n,\frac14)$ with $n\leq8$ (since then $\mu\leq2$), however approximately $p=0.078$ and $\bar{p}=0.017$ for $\xi\sim\mathrm{Bin}(16,\frac14)$. \end{example} \section{Total number of progeny given, \texorpdfstring{$I(v)=|T_v|$}{I(v)=|T\_v|}} If only the {\em size} of the subtree rooted in vertex $v$ is revealed, once $v$ is an internal or external node, and not the whole structure of the tree (as in Section \ref{sec:all}), the situation is very much like when no further information is given. In fact, for extinction probability $q=0$ both regimes coincide (see part (b) of Lemma \ref{skewed2} and the remark after Theorem \ref{tv=i}). While both players stay away from any edge towards a node that is the root of a finite subtree, there is complete symmetry in the nodes that are the root of an infinite subtree, i.e.\ vertices $v$ with $|T_v|=\infty$. Consequently, the number of edges towards external nodes considered for play when it is Maker's turn is again a random walk on $\IZ$. The step size, however, is now $\xi'-2$, where $\xi'$ is the offspring distribution skewed by the fact that vertices, which are the roots of finite subtrees, are disregarded: \begin{lemma}\label{skewed} In the case $I(v)=|T_v|$ and given the non-trivial situation, where $|T_\mathbf{0}|=\infty$, the number of external nodes with infinite progeny at Maker's turns can be described as the random walk \[S_0=c,\quad S_{n}=S_{n-1}+(\xi'_n-2)\quad\text{for}\ n\in\IN,\] where $(\xi'_n)_{n\in\IN}$ is an i.i.d.\ sequence with marginal probability mass function \[\Prob(\xi_1'=k)=\frac{1}{1-q}\, \sum_{n\geq k} p_n \binom{n}{k}(1-q)^kq^{n-k}\quad\text{for }k\in\IN,\] where $(p_k)_{k\in\IN_0}$ is again the pmf of the offspring distribution corresponding to the underlying supercritical GW-process. As before, $c=1$ corresponds to Breaker starting, $c=2$ corresponds to Maker starting the game. \end{lemma} \begin{proof} The only thing to prove (besides following the reasoning in Section \ref{empty} verbatim), is the form of the step-size distribution claimed here. Let us write $\xi^*_v$ for the number of individuals having infinite progeny among the offspring of $v$ and note that this is just an independent thinning of $\xi_v$ with factor $q$. Since $\{|T_v|=\infty\}=\{\xi^*_v>0\}$, we get $\Prob\big(\xi^*_v=0\,\big|\,|T_v|=\infty\big)=0$ and for any $k\in\IN$ \begin{align*} \Prob\big(\xi^*_v=k\,\big|\,|T_v|=\infty\big)&=\frac{\Prob(\xi^*_v=k)}{\Prob(|T_v|=\infty)}\\ &=\frac{1}{1-q}\,\sum_{n=1}^\infty p_n\cdot\Prob(\xi^*_v=k\,|\,\xi_v=n), \end{align*} which implies the claim, writing $\xi'$ for $\xi^*$ conditioned on $\{\xi^*>0\}$. \end{proof} When it comes to this skewed step size distribution $\xi'$, two basic facts are fairly easy to verify: \begin{lemma}\label{skewed2} Let $(p_k)_{k\in\IN_0}$ be the pmf of $\xi$ and for some $0\leq q<1$ the pmf of $\xi'$ be given by \[\Prob(\xi'=k)=\frac{1}{1-q}\, \sum_{n\geq k} p_n \binom{n}{k}(1-q)^kq^{n-k}\quad\text{for }k\in\IN.\] Then the following holds: \begin{enumerate}[(a)] \item $\IE\xi'=\IE\xi=\mu$ \item $\xi'\stackrel{d}{=}\xi$ in case $q=0$. \end{enumerate} \end{lemma} \begin{proof} While part (b) is both obvious from the construction of $\xi'$ and straightforward to check (by setting $q=0$ in its pmf), the claim that $\xi$ and $\xi'$ have the same mean requires a short calculation: \begin{align*} \IE\xi'&=\frac{1}{1-q}\,\sum_{k=1}^\infty k \,\sum_{n\geq k} p_n \binom{n}{k}(1-q)^kq^{n-k}\\ &=\frac{1}{1-q}\,\sum_{n=1}^\infty p_n \,\sum_{k=1}^n k\cdot\binom{n}{k}(1-q)^kq^{n-k}\\ &=\frac{1}{1-q}\,\sum_{n=1}^\infty p_n\cdot n(1-q) = \mu.\\[-1cm] \end{align*} \end{proof} For the remainder of this section we can assume $q\neq0$, since otherwise $|T_v|=\infty$ for all vertices $v$, which puts us back into the regime $I(v)=\emptyset$. But even for $q>0$, in light of Lemma \ref{skewed}, the probability of Breaker winning equals the probability of a random walk on $\IZ$ ever hitting $\IZ\setminus\IN$, much alike the most restrictive information regime $I(v)=\emptyset$ (cf.\ \eqref{rw} above). However, in this regime the stepsize is no longer distributed like $\xi-2$, but by default almost surely bigger or equal to $-1$. Following the line of argument in Section \ref{empty}, we can therfore use the theory of birth-and-death chains again to solve the most simple cases of offspring distributions, despite the change that every node now carries the number of its total progeny as information. For arbitrary offspring distribution $(p_k)_{k\in\IN_0}$, we write as before $g:[0,1]\to[0,1]$ for its pgf and $q$ for the corresponding extinction probability. \begin{proposition}\label{0123} Let $(p_k)_{k\in\IN_0}$ be a supercritical offspring distribution concentrated on $\{0,1,2,3\}$, i.e.\ $p_1<p_0+p_1+p_2+p_3=1<p_1+2p_2+3p_3$. If $g'(q)<p_3\,(1-q)^2$, then the probability of Breaker winning on a GW-tree arising from offspring distribution $\xi$, with $\Prob(\xi=k)=p_k$, given the regime $I(v)=|T_v|$ equals \[p=\frac{g'(q)}{p_3\,(1-q)^2}\quad\text{and}\quad \bar{p}=\left(\frac{g'(q)}{p_3\,(1-q)^2}\right)^2\] respectively, depending on who starts the game. For $g'(q)\geq p_3\,(1-q)^2$, it holds $\bar{p}=p=1$. \end{proposition} \begin{proof} In order to mimick the argument of Proposition\ \ref{123}, we conclude from Lemma \ref{skewed} that $\xi'$ has pmf $\Prob(\xi'=3)=p_3\,(1-q)^2,\ \Prob(\xi'=2)=(p_2+3p_3q)(1-q)$ and $\Prob(\xi'=1)=p_1+2p_2q+3p_3q^2=g'(q)$. Note that these probabilities sum to 1, as $q$ equals $g(q)=p_0+p_1q+p_2q^2+p_3q^3$ by Theorem \ref{extinction}. The corresponding birth-and-death chain is transient if $\Prob(\xi'=3)>\Prob(\xi'=1)$ and the probability it hits 0 given by \[p=\frac{\Prob(\xi'=1)}{\Prob(\xi'=3)}\text{ if started in 1 and by }\bar{p}=\left(\frac{\Prob(\xi'=1)}{\Prob(\xi'=3)}\right)^2\text{ if started in 2}.\] In case $\Prob(\xi'=1)=g'(q)=p_3\,(1-q)^2=\Prob(\xi'=3)$, the random walk described in Lemma \ref{skewed} is a (lazy) simple symmetric random walk on $\IZ$ that will a.s.\ hit state 0, irrespectively of its start value $c$. The statement for $g'(q)< p_3\,(1-q)^2$ then follows immediately by the monotonicity in the tree (or an appropriate coupling of the walks). \end{proof} \begin{example}[Binomial distribution] Let us consider the Maker-Breaker game on a GW-family tree with offspring distribution $\xi\sim\mathrm{Bin(3,r)}$ and $I(v)=|T_v|$. The branching process is supercritical if $r>\frac13$ and its extinction probability then given by the smallest positive root of $x=g(x)=(1+r(x-1))^3$, which is \[q(r)=1-\frac{3}{2r}+\frac{1}{2r^2}\sqrt{r\,(4-3r)}.\] Consequently, $g'(q)=\frac32\,(2-r-\sqrt{r\,(4-3r)})<r^3\,(1-q)^2$ holds when $r>\frac23$. By Proposition \ref{0123}, we can conclude that the probabilities for Breaker winning are given by \[p(r)=\begin{cases}1,& \text{for }0\leq r\leq\frac23\\ \dfrac{6r\,(2-r-\sqrt{r\,(4-3r)})}{\big(3r-\sqrt{r\,(4-3r)}\big)^2},&\text{for } \frac23<r\leq1\end{cases}\] and $\bar{p}(r)=p^2(r)$ respectively, depending on who starts the game. An illustration can be found in Figure \ref{Bin3} below. \begin{figure}[H] \includegraphics[scale=1]{Bin3} \caption{Winning probabilities for Breaker on a GW-tree arising from $\xi\sim\mathrm{Bin}(3,r)$, where node $v$ carries information $I(v)=|T_v|$. \label{Bin3}} \end{figure} \end{example} \begin{theorem}\label{tv=i} In the regime $I(v)=|T_v|$, the winning probability $p$ of Breaker is a solution to \begin{equation}\label{fp3} x^2\,(1-q)=g\big(x\,(1-q)+q\big)-q. \end{equation} To be more precise: $p=1$ in case $\mu\leq 2,\ p_2\neq1$ and $p=0$ if $\xi\geq2$ almost surely; for $\mu>2$ and $p_0+p_1>0$, $p$ equals the (unique) solution to \eqref{fp3} in $(0,1)$. \end{theorem} \begin{proof} First note that both $x=0$ and $x=1$ (due to $q=g(q)$ and $g(1)=1$) are general solutions to equation \eqref{fp3}. In the trivial case $q=1$, the equation obviously holds and $p$ equals $1$. In case $q=0$ ($\Leftrightarrow p_0=0$) the statement becomes Theorem \ref{i=0}, which leaves case $0<q<1$ left to treat. Using Lemma \ref{skewed} and conditioning on the first step of the random walk $(S_n)_{n\in\IN_0}$, we get \begin{align*} p&=\sum_{k=1}^\infty \Prob(\xi'_1=k)\cdot p^{k-1} =\frac{1}{1-q}\,\sum_{k=1}^\infty p^{k-1}\,\sum_{n\geq k} p_n \binom{n}{k}(1-q)^kq^{n-k}\\ &=\frac{1}{p\,(1-q)}\,\sum_{n=1}^\infty p_n\,\sum_{k=1}^n \binom{n}{k}p^k(1-q)^kq^{n-k}\\ &=\frac{1}{p\,(1-q)}\, \sum_{n=0}^\infty p_n \Big[\big(p(1-q)+q\big)^n-q^n\Big]\\ &=\frac{1}{p\,(1-q)}\,\Big(g\big(p(1-q)+q\big)-g(q)\Big), \end{align*} where we used that the factor of $p_0$ is 0 in the before last line. In fact we tacitly assumed $p\neq 0$ as well, but as mentioned above, $x=0$ solves equation \eqref{fp3}. To derive the second part of the claim, first note that by Lemma \ref{skewed2}, the random walk $(S_n)_{n\in\IN_0}$ with increments distributed like $\xi'-2$ has positive drift iff $\mu>2$. The case $\Prob(\xi\geq2)=1$ is included in $q=0$ (as mentioned at the start, where $\xi$ and $\xi'$ have the same distribution) and trivially gives $p=0$. In case $\mu\leq 2,\ p_2\neq1$, the random walk is neither constant nor having positive drift, hence $p=1$. Finally, for $\mu>2$ and $p_0>0$ (if $p_0=0$ we are back to $q=0$) let us consider the function \[h(x):=g\big(x\,(1-q)+q\big)-(1-q)\,x^2-q.\] In complete analogy to the reasoning in the proof of Theorem \ref{i=0}, the sign of its derivative $h'(x)=(1-q)\,\big(g'(x\,(1-q)+q)-2x\big)$ can change at most twice, due to the convexity of $g'$. Further, $\mu>2$ forces both $q<1$ and $p_0+p_1<1$, which implies that $g$ is even strictly convex on $(0,1)$. Therefore we have not only $h(0)=h(1)=0$ and $h'(1)=(1-q)\,(\mu-2)>0$, but also $h'(0)=(1-q)\,g'(q)>(1-q)\,g'(0)=(1-q)\,p_1\geq0$, where the strict inequality comes from $q>0$ and strict convexity of $g$. Thus the function $h$ has exactly one zero in $(0,1)$. That $(S_n)_{n\in\IN_0}$ in this case is transient with positive drift (and therefore $0<p<1$, cf.\ Lemma \ref{drift}) concludes the proof. \end{proof} Note that for offspring distributions concentrated on $\IN$ (i.e.\ $p_0=0$, hence $q=0$), as already indicated above, the statement of Theorems \ref{tv=i} and \ref{i=0} coincide, since then almost surely $|T_v|=\infty$ for all $v$ which renders this piece of information irrelevant. \begin{example}[Poisson distribution] If we consider $\xi\sim\mathrm{Poi}(\lambda)$ in this regime, the game is still non-trivial for $\lambda>2$. Calculating $q$ according to Theorem \ref{extinction} and solving \eqref{fp3} numerically, we can determine $p$ and calculate $\bar{p}=g(p)$, cf.\ Theorem \ref{arch}. The resulting probabilities are depicted below. \begin{figure}[H] \includegraphics[scale=1]{Poi2} \caption{Winning probabilities for Breaker on a GW-tree arising from $\xi\sim\mathrm{Poi}(\lambda)$, where node $v$ carries information $I(v)=|T_v|$. \label{Poi2}} \end{figure} \end{example} \section{Further observations and open problems} As mentioned earlier, the phase transition (from a.s.\ win for Breaker to non-degenerate outcome of the game) turned out to be discontinuous in the winning probabilities only in the case of complete information, i.e.\ $I(v)=T_v$. Further, the three information regimes considered seem to suggest that an increase in the level of information is (more) beneficial for Breaker (than for Maker), see Figure \ref{Poicomp} below illustrating a comparison of winning probabilities for $\xi\sim\mathrm{Poi}(\lambda)$ in the different regimes. This might not come as a surprise, owing to the fact that the underlying graph is a tree which attributes a more local effect to moves of Maker and a more global one to the moves of Breaker. \begin{figure}[H] \includegraphics[scale=1]{Poicompare} \caption{Winning probabilities for Breaker on a GW-tree arising from $\xi\sim\mathrm{Poi}(\lambda)$ in all three considered information regimes compared.\label{Poicomp}} \end{figure} Of course, there are many more regimes to be considered: When $v$ becomes an internal/external vertex, one could reveal the subtree $T_v$ not completely (as in Section \ref{sec:all}), but only down to a fixed level $k\in\IN$. Alternatively the node $v$ could carry only the number of offspring in a certain generation of $T_v$ as information, such as the number of its children and/or grand children for instance (neither disclosing the tree-structure of the first few levels, nor whether or not $T_v$ is finite -- if the corresponding numbers are bigger than $0$). Such regimes, however, appear to be more complex in their analysis and will be left for future work -- together with the question if there are information regimes other than $I(v)=T_v$, in which the phase transition in $p$ and $\bar{p}$ is a discontinuous one. Another possible generalization would be to break the information symmetry in the sense that what is disclosed to Maker resp.\ Breaker when $v$ appears as internal/external node differs. This makes the progression of the game and potential strategies certainly more complex as the players in that case can extract further information and learn from the chosen moves of their opponent during the game. It should be mentioned that in all three regimes considered here, information about the distribution of $\xi$ is not relevant for the players. In other regimes, for instance if $I(v)$ is the parity of $\xi_v$ and the offspring distribution of the type one-or-many (cf.\ Example \ref{oom}) with $n$ large and even, knowing the underlying distribution makes a big difference. In fact, in regimes where $I(v)$ contains the number of children $\xi_v$, the outcome of the game is insensitive to changes of $(p_0,p_1)$ as long as the sum $p_0+p_1$ is kept unchanged: There is no point for Maker in fixating the edge connecting a node $v$ with $\xi_v=1$ to the root, as Breaker can immediately disconnect $T_v\setminus{v}$. In such a regime, changing the distribution of $\xi$ from one-or-many to ``none-or-many'', i.e.\ $p_n=1-p_0=r$, will change the probability $q$ for the tree to be finite, but neither $p$ nor $\bar{p}$. \vspace*{1em} The question concerning possible cases in which $p=\bar{p}$, touched upon at the end of Section \ref{sec3}, can be settled in full generality without further ado: \begin{proposition} Irrespectively of offspring distribution and information regime, it holds \[p=\bar{p}\quad\Longleftrightarrow\quad p\in\{0,1\}.\] \end{proposition} \begin{proof} Since $0\leq g(p)\leq \bar{p}\leq p\leq 1$ (cf.\ Theorem \ref{arch}), the easy direction ``$\Leftarrow$'' follows directly from $g(1)=1$. As regards ``$\Rightarrow$'', $p\notin\{0,1\}$ implies $p_0<1$ and $p_0+p_1>0$ (otherwise either $p=1$ or $\xi\geq2$ a.s.\ forcing $p=0$). It is not difficult to convince yourself that in both cases, $0<p_0<1$ and $p_1>0$ respectively, the event that $\xi_\mathbf{0}>0$ and at most one individual $v$ in the first generation of the tree has offspring, i.e. \[A:=\{Z_1>0,\ Z_2=\xi_v\text{ for some $v$ in generation 1}\},\] has positive probability. On such a tree, Breaker wins with probability 1 if starting (disconnecting $v$) and probability $p$ if Maker starts (fixating the edge to $v$). Therefore, it follows \begin{align*}\bar{p}&=\Prob(\text{B wins}\,|\,\text{M starts}, A^\text{c})\cdot\Prob(A^\text{c})+ \Prob(\text{B wins}\,|\,\text{M starts}, A)\cdot\Prob(A)\\ &\leq \Prob(\text{B wins}\,|\,\text{B starts}, A^\text{c})\cdot\Prob(A^\text{c}) + p\cdot\Prob(A)\\ &<\Prob(\text{B wins}\,|\,\text{B starts}, A^\text{c})\cdot\Prob(A^\text{c}) + \Prob(\text{B wins}\,|\,\text{B starts}, A)\cdot\Prob(A)\\ &=p, \end{align*} which concludes the proof. \end{proof} Last but not least, in cases when Breaker wins, the question concerning the number of rounds necessary to exhaust the tree becomes interesting. Given optimal play, it directly relates to the size $|C(\mathbf{0})|$ of the component containing the root that Maker is able to fixate before there are no more edges available for play. In information regimes like $I(v)=\emptyset$ and $I(v)=|T_v|$, where the number of available visible edges can be regarded as a random walk, the time the game lasts (counted in rounds) is given by $\tau_{\leq0}$ the time until this walk exits $\IN$. \begin{thebibliography}{9} \bibitem{Athreya-Ney} {\sc K.B.\ Athreya} and {\sc P.E.\ Ney} (1972): {\em ``Branching processes''}, Springer Berlin, Heidelberg. \bibitem{Luczak} {\sc M.\ Bednarska} and {\sc T.\ \L uczak} (2001): {Biased positional games and the phase transition}, {\em Random Structures \& Algorithms}, Vol.\ 18 (2), pp.\ 141-152. \bibitem{RndGeo} {\sc A.\ Beveridge,} {\sc A.\ Dudek,} {\sc A.\ Frieze,} {\sc T.\ Mller} and {\sc M.\ Stojakovi\'c} (2014): {Maker-Breaker Games on Random Geometric Graphs}, {\em Random Structures \& Algorithms}, Vol.\ 45 (4), pp.\ 553-607. \bibitem{skipfree} {\sc M.\ Brown,} {\sc E.A.\ Pekz} and {\sc S.M.\ Ross} (2010): {Some results for skip-free random walk}, {\em Probability in the Engineering and Informational Sciences}, Vol.\ 24 (4), pp.\ 491-507. \bibitem{erds} {\sc V.\ Chvtal} and {\sc P.\ Erd\H{o}s} (1978): {Biased positional games}, {\em Annals of Discrete Mathematics}, Vol.\ 2, pp.\ 221-229. \bibitem{CKM} {\sc D.\ Clemens,} {\sc L.\ Kirsch} and {\sc Y.\ Mogge} (2021): {Connector-Breaker games on random boards}, {\em The Electronic Journal of Combinatorics}, Vol.\ 28 (3), pp.\ 1-33. \bibitem{DFR1} {\sc A.N.\ Day} and {\sc V.\ Falgas-Ravry} (2021): {Maker-Breaker percolation games I: Crossing grids}, {\em Combinatorics, Probability and Computing}, Vol.\ 30, pp.\ 200-227. \bibitem{DFR2} {\sc A.N.\ Day} and {\sc V.\ Falgas-Ravry} (2021): {Maker-Breaker percolation games II: Escaping to infinity}, {\em Journal of Combinatorial Theory (Series B)}, Vol.\ 151, pp.\ 482-508. \bibitem{Dek1} {\sc F.M.\ Dekking} (1991): {Branching processes that grow faster than binary splitting}, {\em The American Mathematical Monthly}, Vol.\ 98 (8), pp.\ 728-731. \bibitem{Rndboards} {\sc A.\ Ferber,} {\sc R.\ Glebov,} {\sc M.\ Krivelevich} and {\sc A.\ Naor} (2014): {Biased games on random boards}, {\em Random Structures \& Algorithms}, Vol.\ 46 (4), pp.\ 651-676. \bibitem{Hefetz} {\sc D.\ Hefetz,}{\sc M.\ Krivelevich,} {\sc M.\ Stojakovi\'c} and {\sc T.\ Szab} (2009): {Fast winning strategies in Maker-Breaker games}, {\em Journal of Combinatorial Theory (Series B)}, Vol.\ 99, pp.\ 39-47. \bibitem{HM} {\sc A.E.\ Holroyd} and {\sc J.B.\ Martin} (2021): {Galton-Watson games}, {\em Random Structures \& Algorithms}, Vol.\ 59 (4), pp.\ 495-521. \bibitem{rooted} {\sc S.\ Karmakar,} {\sc M.\ Podder,} {\sc S.\ Roy} and {\sc S.\ Sadhukhan} (2023+): {Phase transition in percolation games on rooted Galton-Watson trees},\\ \url{https://arxiv.org/abs/2303.11402}. \bibitem{Shannon} {\sc A.\ Lehman} (1964): {A solution of the Shannon switching game}, {\em Journal of the Society for Industrial and Applied Mathematics}, Vol.\ 12 (4), pp.\ 687-725. \bibitem{LPW} {\sc D.A.\ Levin,} {\sc Y.\ Peres} and {\sc E.L.\ Wilmer} (2017): {\em ``Markov chains and mixing times (2nd edition)''}, American Mathematical Society. \bibitem{London} {\sc A.\ London} and {\sc A.\ Pluhr} (2018): {Spanning tree game as Prim would have played}, {\em Acta Cybernetica}, Vol.\ 23, pp.\ 921-927. \bibitem{Lyons} {\sc R.\ Lyons} (1990): {Random walks and percolation on trees}, {\em The Annals of Probability}, Vol.\ 18 (3), pp.\ 931-958. \bibitem{Dek2} {\sc A.G.\ Pakes} and {\sc F.M.\ Dekking} (1991): {On family trees and subtrees of simple branching processes}, {\em Journal of Theoretical Probability}, Vol.\ 4 (2), pp.\ 353-369. \bibitem{StoSza} {\sc M.\ Stojakovi\'c} and {\sc T.\ Szab} (2005): {Positional games on random graphs}, {\em Random Structures \& Algorithms}, Vol.\ 26 (1-2), pp.\ 204-223. \bibitem{LCRW} {\sc T.\ Vilkas} (2024+): {Left-continuous random walk on $\mathbb{Z}$ and the parity of its hitting times}, submitted to {\em Electronic Communications in Probability},\\ \url{https://arxiv.org/abs/2407.06903}. \end{thebibliography} \vspace{0.5cm}\noindent {\sc \small Timo Vilkas\\ Statistiska institutionen,\\ Ekonomihgskolan vid Lunds universitet,\\ 220\,07 Lund, Sweden.}\\ [email protected]\\ \end{document}
2412.08400v1
http://arxiv.org/abs/2412.08400v1
Combinatorial Characterization of Exponential Families of Lumpable Stochastic Matrices
\documentclass[a4paper,11pt,oneside]{article} \pdfoutput=1 \RequirePackage{amsmath, amsthm, amssymb, mathtools, mathrsfs} \RequirePackage{natbib} \RequirePackage{tikz,tikz-cd} \RequirePackage{tabularray} \RequirePackage{algorithm2e} \RequirePackage{nicematrix} \NiceMatrixOptions{ code-for-first-row = \tiny , code-for-last-row = \tiny , code-for-first-col = \tiny , code-for-last-col = \tiny } \RequirePackage{float} \usetikzlibrary{patterns} \RequirePackage[margin=0.9in]{geometry} \RequirePackage{libertine} \RequirePackage{courier} \RequirePackage{mathpazo} \RequirePackage{xcolor} \RequirePackage{pifont} \RequirePackage{authblk} \RequirePackage[backref=page]{hyperref} \RequirePackage{enumerate} \RequirePackage{titlesec} \definecolor{wred}{rgb}{0.533333,0.10980,0.15686} \definecolor{wredlight}{rgb}{0.788, 0.11, 0.157} \titleformat{\section} {\color{wred}\normalfont\Large\bfseries} {\color{wred}\thesection}{1em}{} \titleformat{\subsection} {\color{wred}\normalfont\Large\bfseries} {\color{wred}\thesubsection}{1em}{} \hypersetup{ colorlinks=true, linkcolor=wredlight, citecolor=wredlight, filecolor=wredlight, urlcolor=wredlight } \renewcommand\backreftwosep{, } \renewcommand\backrefsep{, } \renewcommand*{\backrefalt}[4]{ \ifcase #1 \or (page:~#2) \else (pages:~#2) } \makeatletter \def\@fnsymbol#1{\ensuremath{\ifcase#1\or \dagger\or \ddagger\or \mathsection\or \mathparagraph\or \|\or **\or \dagger\dagger }} \makeatother \newtheorem{theorem}{Theorem}[section] \newtheorem{lemma}{Lemma}[section] \newtheorem{definition}{Definition}[section] \newtheorem{proposition}{Proposition}[section] \newtheorem{conjecture}{Conjecture}[section] \newtheorem{example}{Example}[section] \newtheorem{corollary}{Corollary}[section] \newtheorem{remark}{Remark}[section] \providecommand{\keywords}[1] { \small \textbf{\textit{Keywords---}} #1 } \DeclareMathOperator{\diag}{diag} \DeclareMathOperator*{\argmin}{arg\,min} \DeclareMathOperator*{\argmax}{arg\,max} \DeclareMathOperator{\aff}{aff} \DeclareMathOperator{\spann}{span} \DeclareMathOperator{\ehull}{e-hull} \DeclareMathOperator{\mhull}{m-hull} \DeclarePairedDelimiter\ceil{\lceil}{\rceil} \DeclarePairedDelimiter\floor{\lfloor}{\rfloor} \newcommand{\hadamard}{\odot} \newcommand{\zero}{0} \newcommand{\plus}{+} \newcommand{\set}[1]{\left\{ #1 \right\}} \newcommand{\eqdef}{\triangleq} \newcommand{\trn}{^\intercal} \newcommand{\abs}[1]{\left| #1 \right|} \newcommand{\nrm}[1]{\left\Vert #1 \right\Vert} \newcommand{\tv}[1]{\nrm{#1}_{\textup{\tiny\textsf{TV}}}} \newcommand{\at}[1]{\Bigr|_{#1}} \newcommand{\pred}[1]{\delta\left[#1\right]} \newcommand{\PR}[2][]{\mathbb{P}_{#1}\left( #2 \right)} \newcommand{\E}[2][]{\mathbb{E}_{#1}\left[ #2 \right]} \newcommand{\Var}[2][]{\mathbb{V}\mathbf{ar}_{#1}\left[ #2 \right]} \newcommand{\eps}{\varepsilon} \newcommand{\bigO}{\mathcal{O}} \newcommand{\kl}[2]{D\left(#1 \middle| \middle| #2\right)} \newcommand{\stoch}{{\mathfrak{s}}} \newcommand{\bbR}{\mathbb{R}} \newcommand{\bbN}{\mathbb{N}} \newcommand{\calN}{\mathcal{N}} \newcommand{\calR}{\mathcal{R}} \newcommand{\calS}{\mathcal{S}} \newcommand{\calT}{\mathcal{T}} \newcommand{\calB}{\mathcal{B}} \newcommand{\calX}{\mathcal{X}} \newcommand{\calY}{\mathcal{Y}} \newcommand{\calZ}{\mathcal{Z}} \newcommand{\calP}{\mathcal{P}} \newcommand{\calE}{\mathcal{E}} \newcommand{\calM}{\mathcal{M}} \newcommand{\calW}{\mathcal{W}} \newcommand{\calK}{\mathcal{K}} \newcommand{\calH}{\mathcal{H}} \newcommand{\calI}{\mathcal{I}} \newcommand{\calQ}{\mathcal{Q}} \newcommand{\calC}{\mathcal{C}} \newcommand{\calU}{\mathcal{U}} \newcommand{\calV}{\mathcal{V}} \newcommand{\calF}{\mathcal{F}} \newcommand{\calL}{\mathcal{L}} \newcommand{\calJ}{\mathcal{J}} \newcommand{\calD}{\mathcal{D}} \newcommand{\calG}{\mathcal{G}} \title{\vspace{-1.0cm} Combinatorial Characterization of \\ Exponential Families of Lumpable Stochastic Matrices} \begin{document} \author[1]{Shun Watanabe \thanks{email: [email protected].}} \author[2]{Geoffrey Wolfer \thanks{email: [email protected]. }} \affil[1]{Tokyo University of Agriculture and Technology, Tokyo} \affil[2]{Waseda University, Center for Data Science, Tokyo} \date{\today} \maketitle \begin{abstract} It is known that the set of lumpable Markov chains over a finite state space, with respect to a fixed lumping function, generally does not form an exponential family of stochastic matrices. In this work, we explore efficiently verifiable necessary and sufficient combinatorial conditions for families of lumpable transition matrices to form exponential families. \end{abstract} \keywords{Information geometry; irreducible Markov chains; lumpability; exponential families} \tableofcontents \clearpage \section{Introduction} \label{section:introduction} Exponential families (e-families) of distributions are of established importance in statistics due to their distinctive properties for inference problems. For instance, they uniquely provide sufficient statistics capable of condensing any amount of independent and identically distributed data into a fixed number of values \citep{pitman1936sufficient, koopman1936distributions, darmois1935lois}. What is more, it is known that the maximum likelihood estimator achieves the Cram\'{e}r-Rao lower bound only\footnote{Note that this fact only holds when imposing additional regularity conditions on the family.} when the family of distributions forms an exponential family \citep{wijsman1973attainment,joshi1976attainment,fabian1977cramer, muller1989attainment}. In the language of information geometry \citep{amari2007methods}, positive probability distributions are endowed with the structure of a smooth manifold with a pair of dual affine connections---the e-connection and m-connection---and statistical models are regarded as submanifolds. In this framework, being an e-family geometrically corresponds to being autoparallel with respect to the e-connection. Furthermore, deviation from being an e-family---and the subsequent breakdown of the statistical properties---can be measured in terms of curvature; this characterizes second order efficiency of estimators \citep{efron1975defining}. Recently, e-families have also been put under the spotlight in optimization since they allow for efficient natural-gradient computation \citep{amari1998natural}, which finds application in machine learning. It is possible to similarly construct a dually flat geometry on the space of irreducible stochastic matrices \citep{nagaoka2005exponential} defined over a fixed strongly connected transition digraph. Independently and identically distributed (iid) processes, which can be regarded as memoryless Markov chains, are known to form an e-family in the larger family of irreducible stochastic matrices \citep{ito1988}. The Markovian framework is consistent with the divergence rate of the corresponding Markov processes, and information projections \citep{boza1971asymptotically, csiszar1987conditional} which arise naturally from the study of large deviations \citep{moulos2019optimal} and hypothesis testing \citep{nakagawa1993converse, watanabe2017}. In this regard the Markovian framework strictly encompasses the vanilla framework for distributions, while accommodating for processes with time dependencies. However, Markov processes exhibit a significantly richer structure than their iid counterparts, with numerous properties---such as irreducibility, aperiodicity or time-reversibility--- that are not pertinent to iid processes but are well-established for Markov chains. A recently initiated research program seeks to analyze how Markov-centric properties translate into geometric features of the corresponding families of stochastic matrices. For instance, a Markov chain having a uniform stationary distribution is equivalent to being represented by a doubly stochastic transition matrix; it is well-established that the set of doubly stochastic matrices forms a mixture family \citep{hayashi2014information}. Similarly, verifying the detailed-balance equation---indicating the time-reversibility of the stochastic process---means that the transition matrix is self-adjoint in a certain Hilbert space; the set of reversible stochastic matrices is known to form both a mixture family and an exponential family \citep{wolfer2021information}. For context trees, it is known that a tree model forms an e-family if and only if it is an FSMX model \citep{takeuchi2007exponential, takeuchi2017information}. More recently, \citet{wolfer2024geometric} began analyzing lumpability of Markov chains \citep{kemeny1983finite}. Lumpable Markov chains allow for the reduction of the state space by merging symbols without losing Markovianity, making them highly practical. Specifically, they showed that although the lumpable set with respect to a fixed lumping map typically forms neither a mixture family nor an e-family, it is still possible to endow the family with the structure of a mutually dual foliated manifold, leading to a mixed coordinate system \citep[Chapter~3.7]{amari2007methods}. Their construction is centered around the concept of a Markov embedding, defined as a right inverse of the lumping operation, and which is argued to serve a similar role to \v{C}encov's statistical morphisms \citep{cencov1983statistical} in the context of Markov chains. The problem of selecting a good statistical model involves choosing one that enjoys favorable analytical properties. In this regard, both e-families and lumpable families are highly sought-after models, and practitioners may be interested in enjoying the best of both worlds. However, as previously mentioned, lumpable families do not generally form e-families. Indeed, they may or may not be e-families depending on their connection graph and the lumping map. This phenomenon contrasts with many previously analyzed classes; for instance, the set of reversible stochastic matrices, which forms an e-family for any symmetric connection graph. In this paper, we initiate the problem of characterizing the conditions under which lumpable stochastic matrices do form e-families. Since Markov embeddings demonstrably preserve e-families of stochastic matrices, they naturally generate one class of lumpable e-families. However, this approach proves to be quite restrictive. Perhaps surprisingly, it is possible to construct families that are not directly derived from the embedding of an e-family. In this work we explore some necessary and sufficient conditions for the lumpable set to form an e-family. \paragraph{Major contributions---} We summarize our main results below. \paragraph{Necessary and sufficient criteria with multi-row merging blocks.} We obtain sufficient and necessary conditions on the lumpable family $\calW_{\kappa}(\calY, \calE)$ for being an e-family in terms of so-called multi-row merging blocks (refer to Definition~\ref{definition:merging-block}). Namely, a sufficient condition (Corollary~\ref{corollary:no-multi-row-merging-block-is-sufficient}) for $\calW_\kappa(\calY, \calE)$ to be an e-family is that it exhibits no multi-row merging block, while if it exhibits a multi-row merging block which is redundant (Definition~\ref{definition:redundant-block}), this precludes the lumpable family from being exponential (Theorem~\ref{theorem:redundant-merging-block-criterion}). However, neither of the above conditions fully characterizes the property of being an e-family. We also provide an alternative sufficient criteria (Proposition~\ref{proposition:lazy-cycle-criterion}), which shows in particular that there could be an arbitrarily large number of multi-row merging blocks while still yielding an e-family. \paragraph{Dimensional criterion.} We show that when the lumpable family is exponential, the log-affine hull of lumpable functions modulo anti-shift functions\footnote{The term modulo here is understood in the context of direct sums and quotient spaces.} (refer to Definition~\ref{definition:anti-shift-functions}) has a well-understood dimension (refer to Theorem~\ref{theorem:dimensional-criterion}). As a consequence, a mismatch in dimension necessarily implies that the family is not exponential, and this can be verified using a polynomial-time algorithm. We also specialize the above result into Corollary~\ref{corollary:dimensional-criterion-simplified} to obtain a necessary condition purely based on elementary combinatorial properties of the connection graph and the lumping map. \paragraph{Monotonicity and stability.} We examine the property of e-families through basic operations on the edge set $\calE$. In particular, we exhibit a monotonicity property of e-families of lumpable stochastic matrices. Namely, we show in Theorem~\ref{theorem:monotonicity} that it is generally the case that when $\calE \subset \calE'$ if $\calW_\kappa(\calY, \calE')$ forms an e-family, then $\calW_\kappa(\calY, \calE)$ also forms an e-family. Additionally, we exhibit an operation on loops under which the property of being an e-family is stable (refer to Proposition~\ref{proposition:stability-diagonal-modification}). \section{Preliminaries} \label{section:preliminaries} \subsection{Notation} We let $(\calY, \calE)$ be a directed graph (digraph) with finite vertex set $\calY$ and edge set $\calE \subset \calY^2$. We assume that $(\calY, \calE)$ is strongly connected, that is every vertex is reachable from every other vertex by traversing edges in their proper direction. For $\{Y_t\}_{t \in \bbN}$ a time-homogeneous discrete time Markov chain (DTMC) over the space space $\calY$, we collect the transition probabilities into a row-stochastic matrix $P$. In other words, we write\footnote{ Our notation follows the applied probability literature. In the information theory literature, $P(y'|y)$ is sometimes used in lieu of $P(y, y')$.} $$P(y,y') = \PR{Y_{t+1} = y' | Y_{t} = y}.$$ When $P(y,y') > 0$ iff $(y,y') \in \calE$, we say that $(\calY, \calE)$ is a connection graph for $P$. We denote $\calW(\calY, \calE)$ the set of all irreducible row-stochastic matrices pertaining to the connection graph $(\calY, \calE)$. We additionally define $\calF(\calY, \calE) = \bbR^{\calE}$ the set of all real functions on the set of edges and $\calF_+(\calY, \calE)$ its positive subset. As it allows us to conveniently write a function $F \in \calF(\calY, \calE)$ in the form of a square matrix, we will routinely identify \begin{equation*} \begin{split} \calF(\calY, \calE) &\cong \set{ F \in \bbR^{\calY^2} \colon \forall (y,y') \not \in \calE \implies F(y,y') = 0 }, \\ \calF_+(\calY, \calE) &\cong \set{ F \in \calF(\calY, \calE) \colon \forall (y,y') \in \calE \implies F(y,y') > 0 }. \\ \end{split} \end{equation*} The Hadamard product of $A$ and $B$ in $\calF(\calY, \calE)$ is denoted $A \odot B$ and for $t \in \bbR$, $A^{\odot t}$ is defined as the function such that for any $y,y' \in \calE$, $A^{\odot t}(y,y') = A(y,y')^t$. We overload $\exp$ and $\log$ as follows, \begin{equation*} \begin{split} \exp \colon \calF(\calY, \calE) &\to \calF_+(\calY, \calE), \\ \log\colon \calF_+(\calY, \calE) &\to \calF(\calY, \calE), \end{split} \end{equation*} where for any $F \in \calF(\calY, \calE)$ and $(y,y') \in \calE$, $\exp(F)(y,y') = \exp(F(y,y'))$, and for any $F \in \calF_+(\calY, \calE$ and $(y,y') \in \calE$, $\log(F)(y,y') = \log(F(y,y'))$. \subsection{Lumpability {\citep{kemeny1983finite}}} \label{section:lumpability} One classical operation on Markov processes is lumping, which means merging symbols together and recording the observations on the reduced space. It is well known that this operation typically disrupts the Markov property \citep{burke1958markovian, rogers1981markov}. Chains for which the Markov property is preserved are called lumpable. More formally, for a surjective symbol merging map $\kappa \colon \calY \to \calX$, we say that the Markov chain $\{Y\}_{t \in \bbN}$ with transition matrix $P \in \calW(\calY, \calE)$ is $\kappa$-lumpable whenever the stochastic process $\{\kappa(Y_t)\}_{t \in \bbN}$ also forms a DTMC with transition matrix $P^\flat \in \calW(\calX, \calD)$, where the set \begin{equation*} \calD \eqdef \kappa(\calE) \eqdef \set{ (\kappa(y), \kappa(y')) \colon (y,y') \in \calE } \subset \calX^2 \end{equation*} is called the lumped edge set. Denoting $\calW_\kappa(\calY, \calE)$ the $\kappa$-lumpable subset of $\calW(\calY, \calE)$, observe that $\kappa$ induces a push-forward $\kappa_\star$ on stochastic matrices, \begin{equation*} \begin{split} \kappa_\star \colon \calW_\kappa(\calY, \calE) \to \calW(\calX, \calD) \end{split} \end{equation*} as well as a partition of the space $\calY$, which we denote by \begin{equation*} \calY = \biguplus_{x \in \calX} \calS_x, \end{equation*} where for any $x \in \calX$, we wrote $\calS_x \eqdef \kappa^{-1}(x)$. The following characterization of $\calW_\kappa(\calY, \calE)$ was provided by \citet{kemeny1983finite}. It holds that $P \in \calW_{\kappa}(\calY, \calE)$ if and only if for any $(x,x') \in \calD$ and any $y_1, y_2 \in \calS_{x}$, \begin{equation*} \sum_{y' \in \calS_{x'}} P(y_1,y') = \sum_{y' \in \calS_{x'}} P(y_2,y'). \end{equation*} We will use the notation of \citet{levin2009markov}\footnote{We note that in differential geometry $\sharp$ and $\flat$ commonly denote the musical isomorphism. However, in this paper, we use these symbols differently.} to often---albeit not always---disambiguate objects which pertain to the larger space using the superscript $\sharp$ and which pertain to the reduced space using the superscript $\flat$. Two lumpable families of stochastic matrices are said to be equivalent if they coincide upon relabeling of the state space and lumped state space. Namely, for $\kappa_1 \colon \calY_1 \to \calX_1, \kappa_2 \colon \calY_2 \to \calX_2$ two lumping maps, the lumpable families $\calW_{\kappa_1}(\calY_1, \calE_1)$ and $\calW_{\kappa_2}(\calY_2, \calE_2)$ are equivalent, which we denote \begin{equation*} \calW_{\kappa_1}(\calY_1, \calE_1) \cong \calW_{\kappa_2}(\calY_2, \calE_2), \end{equation*} whenever there exist two bijections $\phi^\sharp \colon \calY_1 \to \calY_2$ and $\phi^\flat \colon \calX_1 \to \calX_2$ such that \begin{equation*} \begin{split} \forall (y,y') \in \calY^2, (\phi^\sharp(y),\phi^\sharp(y')) \in \calE_2 &\Longleftrightarrow (y,y') \in \calE_1, \\ \forall (x,x') \in \calX^2, (\phi^\flat(x),\phi^\flat(x')) \in \calD_2 = \kappa(\calE_2), &\Longleftrightarrow (x,x') \in \calD_1 = \kappa(\calE_1), \\ \forall y \in \calY_1, \phi^\flat(\kappa_1(y)) &= \kappa_2(\phi^\sharp (y)). \end{split} \end{equation*} \subsection{Exponential families of stochastic matrices {\citep{nagaoka2005exponential}}} \begin{definition}[$\stoch$-normalization] \label{definition:s-normalization} When $(\calY, \calE)$ is strongly connected we define the mapping \begin{equation*} \begin{split} \stoch \colon \calF_+(\calY,\calE) &\to \calW(\calY,\calE) \\ F &\mapsto P \colon \calE \to \bbR_+, (y,y') \mapsto P(y,y') = \frac{F(y,y') v_F(y')}{\rho_F v_F(y)}, \end{split} \end{equation*} where $\rho_F$ and $v_F$ are respectively the Perron--Frobenius (PF) root and associated right eigenvector of $F$. Henceforth, $(\rho_F, v_F)$ will be called the right PF eigen-pair of $F$. \end{definition} The above-defined $\stoch$-normalization plays the role of the partition function in the distribution setting, in order to normalize an arbitrary non-negative irreducible matrix into a stochastic matrix \citep{miller1961convexity}. \begin{definition}[Anti-shift functions {\citep[Section~3]{nagaoka2005exponential}}] \label{definition:anti-shift-functions} It will be convenient to define \begin{equation*} \begin{split} \calN(\calY, \calE) \eqdef \bigg\{ &N \in \calF(\calY, \calE) \colon \exists (c, f) \in (\bbR, \bbR^\calY), \forall (y, y') \in \calE, N(y, y') = f(y') - f(y) + c \bigg\}. \end{split} \end{equation*} Observe that $\mathcal{N}(\calY, \calE)$ forms a $\abs{\calY}$-dimensional vector subspace. \end{definition} \begin{definition}[e-family of stochastic matrices {\citep{nagaoka2005exponential}}] \label{definition:e-family} We say that the parametric family of irreducible stochastic matrices $$\calV_e = \set{P_\theta \colon \theta = (\theta^1, \dots, \theta^d) \in \bbR^d} \subset \calW( \calY, \calE),$$ is an exponential family (e-family) of stochastic matrices with natural parameter $\theta$ and dimension $d$, when there exist a function $K \in \calF(\calY, \calE)$ and $d$ linearly independent functions $G_1, \dots, G_d \in \calG(\calY, \calE)$, such that \begin{equation*} P_\theta = \stoch \circ \exp \left(K + \sum_{i = 1}^{d} \theta^i G_i\right), \end{equation*} where $\mathcal{G}(\calY, \calE)$ is the quotient space \begin{equation*} \calG(\calY, \calE) \eqdef \calF(\calY, \calE)/\calN(\calY, \calE), \end{equation*} with $\calN(\calY, \calE)$ is introduced in Definition~\ref{definition:anti-shift-functions} and $\stoch$-normalization follows from Definition~\ref{definition:s-normalization}. \end{definition} In other words, there exists a one-to-one correspondence between linear subspaces of $\calG(\calY, \calE)$ and e-families \citep[Theorem~2]{nagaoka2005exponential} through the diffeomorphism \begin{equation*} \begin{split} \stoch \circ \exp \colon \calG(\calY, \calE) &\to \calW(\calY, \calE). \end{split} \end{equation*} Similarly, a mixture family (m-family) of stochastic matrices is induced from the affine hull of a collection of irreducible edge measures \citep{nagaoka2005exponential} (refer also to \citet[Section~4.2]{ hayashi2014information}). An m-family which is also an e-family is called an em-family. For instance, the set of all irreducible stochastic matrices $\calW(\calY, \calE)$ is known to form an em-family \citep{nagaoka2005exponential}. What is more, the reversible subset is also an em-family \citep{wolfer2021information} while the subset of bistochastic matrices forms an m-family but does not form an e-family \citep{hayashi2014information}. \subsection{Foliation on the $\kappa$-lumpable family.} Although the lumpable family $\calW_{\kappa}(\calY, \calE)$ was shown by \citet{wolfer2024geometric} to generally not form an m-family or an e-family of stochastic matrices, it is always possible to decompose it in terms of simpler mathematical structures, called a foliation\footnote{A foliation is a decomposition of a manifold into a union of connected but disjoint submanifolds, called leaves, all sharing the same dimension \citep[Chapter~19]{lee2013smooth}.}. This decomposition is facilitated by the notion of a Markov embedding \citep[Definition~4.3]{wolfer2024geometric}, which corresponds to a right inverse of the lumping operation and satisfies additional natural structural constraints. As such, Markov embeddings are the counterparts of the statistical morphisms axiomatized by \citet{cencov1983statistical} in the context of stochastic matrices. In particular, any $P \in \calW_{\kappa}(\calY, \calE)$ induces a canonical embedding \citep[Lemma~4.8]{wolfer2024geometric} denoted $\Lambda_\star^{(P)} \colon \calW(\calX, \calD) \to \calW_{\kappa}(\calY, \calE)$ satisfying $P = \Lambda_\star^{(P)} \kappa_\star P$. It was established that for any $P^\sharp_0 \in \calW(\calY, \calE)$, the embedding of the family $\calW(\calX, \calD)$ by $\Lambda_\star^{(P^\sharp_0)}$ \begin{equation*} \calJ(P^\sharp_{0}) \eqdef \set{ \Lambda_\star^{(P^\sharp_{0})}P \colon P \in \calW(\calX, \calD) }, \end{equation*} forms an e-family of stochastic matrices. Additionally, for any $P_0^\flat \in \calW(\calX, \calD)$, stochastic matrices lumping into $P^{\flat}_{0}$, \begin{equation*} \calL(P^{\flat}_{0}) \eqdef \set{ P \in \calW_\kappa(\calY, \calE) \colon \kappa_\star P = P^{\flat}_{0}}, \end{equation*} form an m-family, and the manifold $\calW_{\kappa}(\calY, \calE)$ can be endowed with the structure of an e-foliation. \begin{theorem}[Foliation on $\calW_{\kappa}(\calY, \calE)$ {\citep[Theorem~6.4]{wolfer2024geometric}}] \label{theorem:foliation-of-lumpable-kernels} For any fixed $P^\flat_0 \in \calW(\calX, \calD)$, \begin{equation*} \calW_\kappa(\calY, \calE) = \biguplus_{P \in \calL(P^\flat_0)} \calJ(P), \end{equation*} \begin{equation*} \dim \calW_\kappa(\calY, \calE) = \abs{\calE} - \sum_{(x,x') \in \calD} \abs{\calS_x} + \abs{\calD} - \abs{\calX}. \end{equation*} \end{theorem} Mutually dual foliations and mixed coordinate systems play a significant role in information geometry \citep[Section~3.7]{amari2007methods}. \paragraph{Problem statement---} Our goal is to obtain a full characterization of exponential families of lumpable stochastic matrices in terms of combinatorial properties of the connection graph $(\calY, \calE)$ and the lumping map $\kappa$. Ideally, we wish to develop necessary and sufficient conditions which are all verifiable in polynomial time. \paragraph{Motivation---} Previous studies of the geometric structure of well-known families of stochastic matrices---such as reversible, bistochastic or memoryless---have mostly established geometric structure that are valid for general edge sets. This is in stark contrast to lumpable stochastic matrices, where---perhaps surprisingly---the nature of the family seems to depend intricately on the structure of the edge set and its interplay with the lumping map. Another exception is found in \citep{takeuchi2007exponential, takeuchi2017information}, where the e-family nature of the context tree depends on some additional structural properties of the tree, which partially motivated our question. In addition, exponential families of stochastic matrices enjoy distinctive properties that may offer analytical power to the practitioner. The asymptotic variance of a function $G \in \bbR^\calE$ with respect to some irreducible stochastic matrix $K$ can be expressed using the second derivative of the potential function of the one-parameter exponential family (Definition~\ref{definition:e-family}) anchored at $K$ and tilted by $G$ \citep{hayashi2014information}. Furthermore, when $\calW_{\kappa}(\calY, \calE)$ forms an e-family, Bregman geometry yields a Pythagorean identity \citep{hayashi2014information}. Specifically, for any $P \in \calW(\calY, \calE)$ and $\overline{P} \in \calW_\kappa(\calY, \calE)$, \begin{equation*} \kl{P}{\overline{P}} = \kl{P}{P_m} + \kl{P_m}{\overline{P}} \end{equation*} where \begin{equation*} P_m \eqdef \argmin_{\widetilde{P} \in \calW_{\kappa}(\calY, \calE)} \kl{P}{\widetilde{P}} \end{equation*} is the unique m-projection (reverse information projection) of $P$ onto $\calW_{\kappa}(\calY, \calE)$. \paragraph{First approach---} As Markov embeddings are known to be e-geodesic affine \citep[Theorem~10]{wolfer2024geometric}, an immediate sufficient condition for a family $\calW_\kappa(\calY, \calE)$ to be an e-family is to find the existence of an embedding $\Lambda_\star$ satisfying $\calW_\kappa(\calY, \calE) = \Lambda_\star \calW(\calX, \calD)$. This corresponds to restricting the foliation of Theorem~\ref{theorem:foliation-of-lumpable-kernels} to a single e-leaf $\calJ$. As we will see in this paper, this condition is quite restrictive; there exist many more exponential families. \section{The lumpable cone} \label{section:lumpable-cone} In this section, we assume that $(\calY, \calE)$ is strongly connected and that $\kappa \colon \calY \to \calX$ is a surjective lumping function. Similar to \citet{wolfer2024geometric}, we define the set of lumpable functions as follows, \begin{equation*} \calF_\kappa(\calY, \calE) \eqdef \set{ F \in \calF(\calY, \calE) \colon \forall (x,x') \in \calD, \forall y_1, y_2 \in \calS_{x}, \sum_{y' \in \calS_{x'}} P(y_1,y') = \sum_{y' \in \calS_{x'}} P(y_2,y') }. \end{equation*} The positive subset $\calF^{+}(\calY, \calE)$ of $\calF(\calY, \calE)$, forms a blunt\footnote{A convex cone is called blunt if it does not contain the null vector.} convex cone, while $\calF^{+}_\kappa(\calY, \calE)$ is a subcone of $\calF^{+}(\calY, \calE)$, as depicted on Figure~\ref{figure:cone-section}. \begin{proposition}[Commutativity] \label{proposition:stochastic-rescaling-and-lumping-commute} $\stoch$-normalization preserves $\kappa$-lumpability and the following diagram commutes \[ \begin{tikzcd} \calF_\kappa^+(\calY, \calE) \arrow{r}{\kappa_\star} \arrow[swap]{d}{\stoch} & \calF^+(\calX, \calD) \arrow{d}{\stoch} \\\calW_\kappa(\calY, \calE) \arrow{r}{\kappa_\star}& \calW(\calX, \calD). \end{tikzcd} \] \end{proposition} \begin{proof} Let $F \in \calF_\kappa^{+}(\calY, \calE)$. Since $(\calY, \calE)$ is strongly connected, $\stoch$-normalization is well-defined over $F \in \calF_\kappa^{+}(\calY, \calE)$ and we first show that $\stoch(F) \in \calW_\kappa(\calY, \calE)$. By construction, $\stoch(F)$ is irreducible and shares the same support as $F$. It remains to verify lumpability. We let $F^\flat = \kappa_\star F$ denote the lumping of $F$, and $\rho^\flat$ and $v^\flat$ be respectively the Perron--Frobenius root and associated right eigenvector ---henceforth called "right PF eigen-pair"--- of $F^\flat$. For any $y \in \calY$, a classical argument of \citet{barr1977eigenvector} (see also \citet[Lemma~12.9]{levin2009markov}) yields that \begin{equation*} \begin{split} \sum_{y' \in \calY} F(y,y') v^\flat(\kappa(y')) &= \sum_{x' \in \calX} \sum_{y' \in \calS_{x'}} F(y,y') v^{\flat}(\kappa(y')) = \sum_{x' \in \calX} \left(\sum_{y' \in \calS_{x'}} F(y,y') \right) v^{\flat}(x') \\ &= \sum_{x' \in \calX} F^\flat(\kappa(y),x') v^{\flat}(x') = \rho^\flat v^{\flat}(\kappa(y)), \\ \end{split} \end{equation*} which implies that $(\rho^\flat, v^\flat \circ \kappa)$ is the right PF eigen-pair of $F$. As a consequence, for any $(y,y') \in \calY^2$, \begin{equation*} \stoch(F)(y,y') = F(y,y') \frac{v^\flat(\kappa(y'))}{\rho^\flat v^\flat(\kappa(y))}. \end{equation*} For any $x \in \calX$ and any $y \in \calS_x$, it holds that \begin{equation*} \begin{split} \sum_{y' \in \calS_{x'}}\stoch(F)(y,y') &= \sum_{y' \in \calS_{x'}} F(y,y') \frac{v^\flat(\kappa(y'))}{\rho^\flat v^\flat(\kappa(y))} = \sum_{y' \in \calS_{x'}} F(y,y') \frac{v^\flat(x')}{\rho^\flat v^\flat(x)} = F^\flat(x,x') \frac{v^\flat(x')}{\rho^\flat v^\flat(x)}, \end{split} \end{equation*} which does not depend on $y \in \calS_x$. As a result, $\stoch(F)$ is $\kappa$-lumpable, and $\kappa_\star \stoch(F) = \stoch(F^\flat)$. \end{proof} \begin{figure}[H] \begin{center} \tikzset{every picture/.style={line width=0.75pt}} \begin{tikzpicture}[x=0.52pt,y=0.52pt,yscale=-1,xscale=1] \draw [color={rgb, 255:red, 208; green, 2; blue, 27 } ,draw opacity=1 ] (198,77) .. controls (198,59.33) and (248.37,45) .. (310.5,45) .. controls (372.63,45) and (423,59.33) .. (423,77) .. controls (423,94.67) and (372.63,109) .. (310.5,109) .. controls (248.37,109) and (198,94.67) .. (198,77) -- cycle ; \draw [color={rgb, 255:red, 208; green, 2; blue, 27 } ,draw opacity=1 ] (198,79) -- (218.2,113.79) -- (310,281) ; \draw [color={rgb, 255:red, 208; green, 2; blue, 27 } ,draw opacity=1 ] (310,281) -- (422,81) ; \draw [color={rgb, 255:red, 74; green, 144; blue, 226 } ,draw opacity=1 ] (269,204) .. controls (269,194.06) and (287.58,186) .. (310.5,186) .. controls (333.42,186) and (352,194.06) .. (352,204) .. controls (352,213.94) and (333.42,222) .. (310.5,222) .. controls (287.58,222) and (269,213.94) .. (269,204) -- cycle ; \draw [color={rgb, 255:red, 208; green, 2; blue, 27 } ,draw opacity=1 ] [dash pattern={on 0.84pt off 2.51pt}] (160,9) -- (208.1,96.4) ; \draw [color={rgb, 255:red, 208; green, 2; blue, 27 } ,draw opacity=1 ] [dash pattern={on 0.84pt off 2.51pt}] (422,81) -- (461,11) ; \draw [color={rgb, 255:red, 74; green, 144; blue, 226 } ,draw opacity=1 ][dash pattern={on 4.5pt off 4.5pt}] (212.3,162.5) -- (556,162.5) -- (408.7,245.5) -- (65,245.5) -- cycle ; \draw [dash pattern={on 0.84pt off 2.51pt}] (250,9) -- (310,281) ; \draw [fill={rgb, 255:red, 255; green, 255; blue, 255 } ,fill opacity=1 ] (267,71) .. controls (267,69.34) and (265.66,68) .. (264,68) .. controls (262.34,68) and (261,69.34) .. (261,71) .. controls (261,72.66) and (262.34,74) .. (264,74) .. controls (265.66,74) and (267,72.66) .. (267,71) -- cycle ; \draw [fill={rgb, 255:red, 255; green, 255; blue, 255 } ,fill opacity=1 ] (296,204) .. controls (296,202.34) and (294.66,201) .. (293,201) .. controls (291.34,201) and (290,202.34) .. (290,204) .. controls (290,205.66) and (291.34,207) .. (293,207) .. controls (294.66,207) and (296,205.66) .. (296,204) -- cycle ; \draw [fill={rgb, 255:red, 255; green, 255; blue, 255 } ,fill opacity=1 ] (285,153) .. controls (285,151.34) and (283.66,150) .. (282,150) .. controls (280.34,150) and (279,151.34) .. (279,153) .. controls (279,154.66) and (280.34,156) .. (282,156) .. controls (283.66,156) and (285,154.66) .. (285,153) -- cycle ; \draw (418,110) node [anchor=north west][inner sep=0.75pt] [color={rgb, 255:red, 208; green, 2; blue, 27 } ,opacity=1 ] [align=left] {$\calF^+_\kappa(\calY, \calE)$}; \draw (540,170) node [anchor=north west][inner sep=0.75pt] [color={rgb, 255:red, 74; green, 144; blue, 226 } ,opacity=1 ] [align=left] {$\calW(\calY, \calE)$}; \draw (352,202) node [anchor=north west][inner sep=0.75pt] [color={rgb, 255:red, 74; green, 144; blue, 226 } ,opacity=1 ] [align=left] {$\calW_\kappa(\calY, \calE)$}; \draw (260,142) node [anchor=north west][inner sep=0.75pt] [align=left] {$F$}; \draw (300,192) node [anchor=north west][inner sep=0.75pt] [align=left] {$\stoch(F)$}; \draw (216,3) node [anchor=north west][inner sep=0.75pt] [align=left] {$[F]$}; \draw (266,63) node [anchor=north west][inner sep=0.75pt] [align=left] {$G: \stoch(G) = \stoch(F)$}; \end{tikzpicture} \end{center} \caption{We can regard $\calW_\kappa(\calY, \calE)$ as a section of the cone $\calF^+_\kappa(\calY, \calE)$.} \label{figure:cone-section} \end{figure} For $F \in \calF^+(\calY, \calE)$ the equivalence class \begin{equation*} [F] \eqdef \set{ G \in \calF^{+}(\calY, \calE) \colon \stoch(G) = \stoch(F) }, \end{equation*} can be parametrized by the $\abs{\calY}$-dimensional positive orthant, \begin{equation*} [F] = \set{ \rho \diag(v) F \diag(v)^{-1} \colon \rho \in \bbR_+, v \in \bbR_+^{\calY}}, \end{equation*} and the collection of all such rays generates the entire irreducible cone \begin{equation*} \calF^+(\calY, \calE) = \biguplus_{P \in \calW(\calY, \calE)} [P]. \end{equation*} While projecting onto stochastic matrices through $\stoch$-normalization preserves $\kappa$-lumpability, conjugation by an arbitrary diagonal matrix can disrupt the property. In fact, the following proposition holds. \begin{proposition}[Closure under similarity transform] \label{proposition:eigenvector-rescaling-constant-on-each-lumped-point} Let $v \in \bbR_+^{\calY}$. The two following statements are equivalent. \begin{enumerate}[$(i)$] \item For any irreducible lumpable matrix $F \in \calF_{ \kappa}^+(\calY, \calE)$, it holds that \begin{equation*} \diag(v) F \diag(v)^{-1} \in \calF_{\kappa}^{+}(\calY, \calE). \end{equation*} \item For any $x \in \calX$, it holds that for any $(y_1, y_2) \in \calS_{x}^2, v(y_1) = v(y_2)$. \end{enumerate} \end{proposition} \begin{proof} Let $v \in \bbR_+^{\calY}, F \in \calF_\kappa^{+}(\calY, \calE)$, write $F^\flat = \kappa_\star F $, let $(x,x') \in \calD = \kappa_2(\calE)$, and assume that $v$ takes constant values $v_x$ on $\calS_x$ and $v_{x'}$ on $\calS_{x'}$. Then for any $y \in \calS_x$, \begin{equation*} \begin{split} \sum_{y' \in \calS_{x'}} (\diag(v) F \diag(v)^{-1})(y, y') &= \frac{v_x}{v_{x'}} \sum_{y' \in \calS_{x'}} F(y, y') = \frac{v_x}{v_{x'}} F^\flat(x,x'), \end{split} \end{equation*} which does not depend on $y$, thus $\diag(v) F \diag(v)^{-1} \in \calF_\kappa^{+}(\calY, \calE)$. Conversely, let us assume that $\diag(v) F \diag(v)^{-1} \in \calF_\kappa^{+}(\calY, \calE)$. Let $\rho \in \bbR, w \in \bbR_+^{\calY}$ be the unique right PF eigen-pair of $F$, that is $\stoch(F) = \frac{1}{\rho} \diag(w)^{-1} F \diag(w)$. From the proof of Proposition~\ref{proposition:stochastic-rescaling-and-lumping-commute}, $w$ is constant on each $\calS_x$. Observe now that $$\stoch(F) = \stoch(\diag(v) F \diag(v)^{-1}) = \frac{1}{\rho} \diag(v \hadamard w)^{-1} \diag(v) F \diag(v)^{-1} \diag(v \hadamard w),$$ that is $\rho, v \hadamard w$ is the right PF eigen-pair of $\diag(v) F \diag(v)^{-1}$. It follows that $v \hadamard w$ must be constant over each $\calS_x$, and so must $v$. \end{proof} In other words, the cone $\calF_\kappa^{+}(\calY, \calE)$ is closed with respect to similarity transform with positive vectors which are constant over elements of the partition of $\calY$ induced by $\kappa$. \begin{corollary}[to Proposition~\ref{proposition:stochastic-rescaling-and-lumping-commute}] \label{corollary:geometric-intersection-interpretation} Let $F \in \calF^+(\calY, \calE)$. Then $\stoch(F) \in \calW_\kappa(\calY, \calE)$ if and only if $$[F] \cap \calF_\kappa^{+}(\calY, \calE) \neq \emptyset.$$ \end{corollary} \begin{proof} Suppose first that $\stoch(F) \in \calW_\kappa(\calY, \calE)$. Clearly, $F \in [F]$, and since $\calW_\kappa(\calY, \calE) \subset \calF_\kappa^{+}(\calY, \calE)$ it is immediate that $\stoch(F) \in [F] \cap \calF_\kappa^{+}(\calY, \calE) \neq \emptyset$. Conversely, suppose that there exists $G \in [F]$ such that $G \in \calF_\kappa^+(\calY, \calE)$. Then, by Proposition~\ref{proposition:stochastic-rescaling-and-lumping-commute}, $\stoch(G) \in \calW_\kappa(\calY, \calE)$, but $\stoch(G) = \stoch(F)$ since both are in $[F]$, thus also $\stoch(F) \in \calW_\kappa(\calY, \calE)$. \end{proof} The notion of a merging block, which we now introduce, will be instrumental to our analysis. \begin{definition}[Merging block] \label{definition:merging-block} Let $(x, x') \in \calD = \kappa_2(\calE)$ be such that for some $y \in \calS_x$ it holds that \begin{equation*} \abs{\set{(y,y') \in \calE \colon y' \in \calS_{x'}}} > 1. \end{equation*} Then we say that $(x,x')$ is a merging block of $(\calY, \calE)$ with respect to $\kappa$. Furthermore, when $\abs{\calS_x} \geq 2$, we say that the merging block is multi-row. \end{definition} \begin{theorem}[Log-affinity] \label{theorem:characterization-log-linearity} The two following statements are equivalent. \begin{enumerate}[$(i)$] \item $(\calY, \calE)$ has no multi-row merging block with respect to $\kappa$ (Definition~\ref{definition:merging-block}). \item $\calF_\kappa^{+}(\calY, \calE)$ is log-affine in the sense where for any $F_0, F_1 \in \calF_{\kappa}^{+}(\calY, \calE)$ and for any $t \in \bbR$, \begin{equation*} (1 - t)\log F_0 + t \log F_1 \in \calF_{\kappa}^{+}(\calY, \calE), \end{equation*} where the logarithm is here understood to be entry-wise. \end{enumerate} \end{theorem} \begin{proof} Let $F_0, F_1 \in \calF_\kappa^{+}(\calY, \calE)$, let $t \in \bbR$, and write $F_0^\flat = \kappa_\star F_0, F_1^\flat = \kappa_\star F_1$. For $(y,y') \in \calY^2$, we define $F_t(y,y') = F_0(y,y')^{1 - t} F_1(y,y')^t$. Let $(x,x') \in \calD$, we need to show that for any $y_1, y_2 \in \calS_{x}$, $F_t(y_1, \calS_{x'}) = F_t(y_2, \calS_{x'})$. The case $\abs{\calS_x} = 1$ is trivial and it remains to inspect the case $\abs{\calS_x} \geq 2$. Since $(x,x')$ is non-merging, for any $y \in \calS_{x}$ we denote $u'_{x'}(y)$ the unique element in $\calS_{x'}$ such that $(y,u'_{x'}(y)) \in \calE$. For any $y \in \calS_x$, it then holds that \begin{equation*} \begin{split} \sum_{y' \in \calS_{x'}} F_t(y,y') &= \sum_{y' \in \calS_{x'}} F_0(y,y')^{1 -t} F_1(y,y')^{t} = F_0(y,u'_{x'}(y))^{1-t} F_1(y,u'_{x'}(y))^{t} \\ &= \left(\sum_{y'\in \calS_{x'}} F_0(y,y') \right)^{1-t} \left( \sum_{y' \in \calS_{x'}} F_1(y,y') \right)^{t} = F_0^\flat(x,x') ^{1-t} F_1^\flat(x,x')^{t}, \end{split} \end{equation*} which does not depend on $y$, thus $F_t \in \calF_\kappa^+(\calY, \calE)$. Conversely, let us now suppose that $(\calY, \calE)$ has a multi-row merging block with respect to $\kappa$. We show the somewhat stronger claim that there exist two stochastic matrices $P_0, P_1 \in \calW_\kappa(\calY, \calE)$ and $t \in \bbR$ such that $P_0^{\hadamard (1 - t)} \hadamard P_1^{\hadamard t} \not \in \calF_{\kappa}^{+}(\calY, \calE)$. We let $(x_0,x_0') \in \calD$ be a merging block of $\calW_\kappa(\calY, \calE)$, and denote $y_0, y_\star, y'_a, y'_b \in \calS_{x_0}^2, \calS_{x_0'}^2$ a quadruplet such that $y_0 \neq y_\star$, $y'_a \neq y'_b$ and $(y_\star, y'_a), (y_\star, y'_b) \in \calE$. Let $\eta_a, \eta_b \in \bbR_+$ with $\eta_a < \eta_b < 1$ and such that $$(\eta_a + \eta_b)^{-1} = \frac{1}{2}\abs{ \set{\overline{x} \in \calX \colon (x_0, \overline{x}) \in \calD }} \abs{ \set{ \overline{y} \in \calS_{x_0'} \colon (y_\star,\overline{y}) \in \calE }}.$$ We construct, \begin{equation*} P_{a,b}(y,y') = \begin{cases} 0 &\text{when } (y, y') \not \in \calE \\ \eta_a &\text{when } (y,y') = (y_\star, y'_a) \\ \eta_b &\text{when } (y,y') = (y_\star, y'_b) \\ \cfrac{1}{\abs{ \set{\overline{x} \in \calX \colon (\kappa(y), \overline{x}) \in \calD }} \abs{ \set{ \overline{y} \in \calS_{\kappa(y')} \colon (y,\overline{y}) \in \calE }}} &\text{otherwise}. \end{cases} \end{equation*} We define $P_0, P_1 \in \calW_\kappa(\calY, \calE)$ as $P_0 = P_{a,b}$ and $P_1 = P_{b,a}$, and construct the combination $\widetilde{P} = P_0^{\hadamard 1/2} \hadamard P_1^{\hadamard 1/2}$. By the AM-GM inequality $2 \sqrt{\eta_a \eta_b} < \eta_a + \eta_b$, and it follows that \begin{equation*} \sum_{\overline{y} \in \calS_{x_0'}} \widetilde{P}(y_\star, \overline{y}) < \sum_{\overline{y} \in \calS_{x_0'}} \widetilde{P}(y_0, \overline{y}) \end{equation*} hence $\widetilde{P}_{1/2}$ is not $\kappa$-lumpable. \end{proof} \section{Classification for small state spaces} \label{section:classification} \begin{remark}[Degenerate lumping function] \label{remark:degenerate-lumping} If $\kappa \colon \calY \to \calX$ is such that $\abs{\calX} \in \set{ 1, \abs{\calY} }$, then $\calW_{\kappa}(\calY, \calE)$ forms an e-family. \end{remark} It immediately follows that for $\abs{\calY} = 2$, every lumpable family forms an e-family. We proceed to enumerate e-families for the three-state space. \begin{theorem}[Three-state space classification] \label{theorem:enumeration-three-state-space} When $\abs{\calY} = 3$, and assuming that $\calW_{\kappa}(\calY, \calE) \neq \emptyset$, the two following statements are equivalent. \begin{enumerate}[$(i)$] \item $\calW_{\kappa}(\calY, \calE)$ forms an e-family. \item Either $\kappa$ is degenerate or $\calW_{\kappa}(\calY, \calE)$ is equivalent to one of the 12 below-listed families. \end{enumerate} \begin{center} \begin{tblr}{cccc} $\left(\begin{tblr}{c|[dashed]cc} \zero & \plus & \plus\\\hline[dashed] \plus & \zero & \zero\\ \plus & \zero & \zero\\ \end{tblr}\right)$ & $\left(\begin{tblr}{c|[dashed]cc} \plus & \plus & \plus\\\hline[dashed] \plus & \zero & \zero\\ \plus & \zero & \zero\\ \end{tblr}\right)$ & $\left(\begin{tblr}{c|[dashed]cc} \zero & \zero & \plus\\\hline[dashed] \plus & \zero & \plus\\ \plus & \plus & \zero\\ \end{tblr}\right)$ & $\left(\begin{tblr}{c|[dashed]cc} \zero & \zero & \plus\\\hline[dashed] \plus & \plus & \zero\\ \plus & \plus & \zero\\ \end{tblr}\right)$ \\ $\left(\begin{tblr}{c|[dashed]cc} \plus & \zero & \plus\\\hline[dashed] \plus & \plus & \zero\\ \plus & \plus & \zero\\ \end{tblr}\right)$ & $\left(\begin{tblr}{c|[dashed]cc} \zero & \plus & \plus\\\hline[dashed] \plus & \plus & \zero\\ \plus & \plus & \zero\\ \end{tblr}\right)$ & $\left(\begin{tblr}{c|[dashed]cc} \plus & \zero & \plus\\\hline[dashed] \plus & \zero & \plus\\ \plus & \plus & \zero\\ \end{tblr}\right)$ & $\left(\begin{tblr}{c|[dashed]cc} \plus & \plus & \plus\\\hline[dashed] \plus & \plus & \zero\\ \plus & \plus & \zero\\ \end{tblr}\right)$ \\ $\left(\begin{tblr}{c|[dashed]cc} \zero & \plus & \plus\\\hline[dashed] \plus & \zero & \plus\\ \plus & \plus & \zero\\ \end{tblr}\right)$ & $\left(\begin{tblr}{c|[dashed]cc} \plus & \plus & \plus\\\hline[dashed] \plus & \zero & \plus\\ \plus & \plus & \zero\\ \end{tblr}\right)$ & $\left(\begin{tblr}{c|[dashed]cc} \zero & \plus & \plus\\\hline[dashed] \plus & \plus & \zero\\ \plus & \zero & \plus\\ \end{tblr}\right)$ & $\left(\begin{tblr}{c|[dashed]cc} \plus & \plus & \plus\\\hline[dashed] \plus & \plus & \zero\\ \plus & \zero & \plus\\ \end{tblr}\right)$ \end{tblr} \end{center} \end{theorem} \begin{proof} From Remark~\ref{remark:degenerate-lumping}, we only need to consider the case where $\abs{\calX} = 2$. Being an exponential family is a property common to the entire equivalence class of lumpable families. After removing empty lumpable families and grouping them into equivalence classes (refer to Section~\ref{section:lumpability} for the definition of equivalence classes), we obtain 26 cases. The 12 families described in $(ii)$ can be shown to forms e-families by applying Corollary~\ref{corollary:no-multi-row-merging-block-is-sufficient}, while the remaining 14 families can be shown to not be e-families using a dimensional argument introduced later in Theorem~\ref{theorem:dimensional-criterion}. \end{proof} \begin{remark} In the three-state space setting, we observe that $\calW_{\kappa}(\calY, \calE)$ forming an e-family coincides with $\calF_{\kappa}^{+}(\calY, \calE)$ being log-affine. \end{remark} \section{Characterizing of e-families of lumpable stochastic matrices} \label{section:towards-characterization} In this section, we assume that $1 < \abs{\calX} < \abs{\calY}$ and that $\calW_\kappa(\calY, \calE) \neq \emptyset$. \subsection{Sufficient conditions} \label{section:sufficient-condition} \subsubsection{No multi-row merging block criterion} The first criterion is a natural consequence of Theorem~\ref{theorem:characterization-log-linearity}. \begin{corollary}[No multi-row merging block criterion] \label{corollary:no-multi-row-merging-block-is-sufficient} If $(\calY, \calE)$ has no multi-row merging block with respect to $\kappa$, then $\calW_\kappa(\calY, \calE)$ forms an e-family. \end{corollary} \begin{proof} Let $P_0, P_1 \in \calW_\kappa(\calY, \calE)$ and $t \in \bbR$. Defining $\widetilde{P}_t = P_0^{\hadamard (1 - t)} \hadamard P_1^{\hadamard t}$, it follows from Theorem~\ref{theorem:characterization-log-linearity} that $\widetilde{P}_t \in \calF_{\kappa}^{+}(\calY, \calE)$. It is then a consequence of Proposition~\ref{proposition:stochastic-rescaling-and-lumping-commute} that $\stoch(\widetilde{P}_t) \in \calW_\kappa(\calY, \calE)$. Since $P_0, P_1$ and $t$ were arbitrary, \citet[Corollary~3]{nagaoka2005exponential} implies that $\calW_\kappa(\calY, \calE)$ forms an e-family. \end{proof} \begin{example}[Hudson expansion {\citep{kemeny1983finite}}] Let $(\calX, \calD)$ be a finite strongly connected graph. For a Markov chain $X_1, X_2, \dots$ sampled according to a transition matrix $P \in \calW(\calX, \calD)$, recall that the sliding window chain $$(X_1, X_2), (X_2, X_3), \dots, (X_t, X_{t+1}), \dots$$ is also a Markov chain with transition matrix $P^\sharp \in \calW_h(\calY, \calE)$, with state space $\calY = \calD$, edge set \begin{equation*} \calE = \set{ (e = (x_1, x_2),e' = (x_1', x_2')) \in \calD^2 \colon x_2 = x_1' }, \end{equation*} and lumping function $h \colon \calY \to \calX, (x_1, x_2) \mapsto x_2$. One can verify that $(\calY, \calE)$ has no multi-row merging block with respect to $h$. As a result of Corollary~\ref{corollary:no-multi-row-merging-block-is-sufficient}, $\calW_{h}(\calY, \calE)$ thus forms an e-family of lumpable Markov chains. We therefore recover the known fact that the Hudson expansion of the first-order Markov chains forms an e-subfamily of second-order Markov chains. Note that, since the Hudson expansion is known to be a particular case of a Markov embedding \citep{wolfer2024geometric}, and Markov embeddings are known to preserve e-families of stochastic matrices, the claim also follows from an embedding argument. \end{example} \subsubsection{Lazy-cycle criterion} However, for state spaces strictly larger than three, the above-stated condition is not necessary, as is demonstrated in the example below. \begin{example}[Lumpable e-family with two multi-row merging blocks] \label{example:e-family-two-merging-block} \begin{equation*} \left(\begin{tblr}{cc|[dashed]cc} \zero & \zero & \zero & \plus \\ \zero & \zero & \plus & \plus\\\hline[dashed] \zero & \plus & \plus & \zero \\ \plus & \plus & \zero & \plus \\ \end{tblr}\right) \end{equation*} \end{example} This is a consequence of the fact that constructing an e-geodesic further involves $\stoch$-normalization which can return the curve to the lumpable set. \begin{proposition}[Lazy cycle criterion] \label{proposition:lazy-cycle-criterion} If any of the two following equivalent conditions are satisfied, then $\calW_{\kappa}(\calY, \calE)$ forms an e-family. \begin{enumerate}[$(i)$] \item For any $P \in \calW_{\kappa}(\calY, \calE)$, there exist a pair of non-negative matrices $D$ and $\Pi$ such that \begin{equation*} P = D + \Pi, \end{equation*} where $\kappa_\star \Pi$ is a permutation matrix over $\calX$ and $D$ is diagonal. \item The graph $\left( \calX, \set{ (\kappa(y),\kappa(y')) \colon (y,y') \in \calE, y \neq y'} \right)$ is a cycle. \end{enumerate} \end{proposition} \begin{proof} The proposition states that all diagonal blocks are diagonal and that there is exactly one non-zero off-diagonal block per block-line. We first prove the statement in the special case where all diagonal blocks vanish, that is $\{ (x,x) \colon x \in \calX \} \cap \calD = \emptyset$, and when the off-diagonal blocks have full support, that is for any $(x,x') \in \calD$ with $x \neq x'$, $\calS_{x} \times \calS_{x'} \subset \calE$. An application of monotonicity (refer to Theorem~\ref{theorem:monotonicity}) generalizes the result beyond full-sport off-diagonal blocks, and stability by diagonal modifications (refer to Proposition~\ref{proposition:lazy-cycle-criterion}) yield the more general case where there are non vanishing diagonal blocks on the diagonal. We further reduce the problem by observing that since the lumped matrix is irreducible, $\Pi^\flat = \kappa_\star \Pi$ defines a cycle. Finally, it will be convenient to order states $\calX = \set{1, \dots, \abs{\calX}}$ and $\calY = \set{1, \dots, \abs{\calY}}$. As a result, upon relabelling, we henceforth assume that can the family can be represented by \begin{equation*} \left(\begin{tblr}{c|[dashed]c|[dashed]c|[dashed]c|[dashed]c} 0 & \boxplus_{\calS_{1} \times \calS_{2}} & 0 & \hdots & 0\\\hline[dashed] 0 & 0 & \boxplus_{\calS_{2} \times \calS_{3}} & \ddots & \vdots \\\hline[dashed] \vdots & & \ddots & \ddots & 0 \\\hline[dashed] 0 & & & 0 & \boxplus_{\calS_{\abs{\calX} - 1} \times \calS_{\abs{\calX}}} \\\hline[dashed] \boxplus_{\calS_{\abs{\calX}} \times \calS_{1}} & 0 & \hdots & 0 & 0 \end{tblr}\right) \end{equation*} where for $\calS, \calS' \subset \calY$, $\boxplus_{\calS \times \calS'}$ is $\abs{\calS} \times \abs{\calS'}$ matrix defined by \begin{equation*} \textstyle{\boxplus_{\calS \times \calS'}} = \begin{pmatrix} \plus & \plus & \hdots & \plus \\ \plus & \plus & \hdots & \plus \\ \vdots & \vdots & \vdots & \vdots \\ \plus & \plus & \hdots & \plus \\ \end{pmatrix}. \end{equation*} We let $P_0, P_1 \in \calW_{\kappa}(\calY, \calE)$, and for $t\in \bbR$, we denote $\widetilde{P}_t = P_0^{\hadamard (1- t)} \hadamard P_1^{\hadamard t} \in \calF^{+}(\calY, \calE)$ their log-affine combination. We define $\rho_t$ and $v_t$ the right PF pair of $\widetilde{P}_t$, whose existence follows by strong connectivity of $(\calY, \calE)$. By $\stoch$-normalization $P_t = \frac{1}{\rho_t}\diag(v)^{-1} \widetilde{P}_t \diag(v)$ is row-stochastic, and as a result, for any $(x,x') \in \calD$, it holds that for any $y \in \calS_{x}$, \begin{equation*} \sum_{y' \in \calS_{x'}} P_t(y,y') = \sum_{y' \in \calY} P_t(y,y') = 1, \end{equation*} thus $P_t$ is $\kappa$-lumpable and the claim holds. \end{proof} \begin{corollary} There exist e-families of lumpable stochastic matrices with an arbitrary number of multi-row merging blocks. \end{corollary} It is instructive to observe that neither criterion implies the other. What is more, there exist e-families that remain unexplained by the two above-mentioned criteria. Deriving a set of necessary conditions for $\calW_{\kappa}(\calY, \calE)$ to form an e-family is the topic of our next section. \subsection{Necessary conditions} \label{section:necessary-conditions} As we have seen in the previous section, absence of log-affinity does not always preclude the lumpable family from being exponential. However, the former will be a critical ingredient to show that at least some families of matrices with so called ``redundant blocks'' cannot form e-families. First, it will be convenient to consider the following notions of block removal. \begin{definition}[Sub-matrix and block removal] \label{definition:sub-matrix-block-removal} Let $F \in \calF_\kappa^{+}(\calY, \calE)$ and a non-degenerate surjective lumping function $\kappa \colon \calY \to \calX$. \begin{description} \item[Sub-matrix.] For $\calT \subset \calX$, we write \begin{equation*} \begin{split} \calY|_{\calT} \eqdef \calY \cap \left( \cup_{x \in \calT} \calS_{x} \right) , \qquad \calE|_{\calT} \eqdef \calE \cap \left( \bigcup_{(x,x') \in \calT} \calS_{x} \times \calS_{x'}\right), \end{split} \end{equation*} the resulting sub-graph is ($\calY|_{\calT}, \calE|_{\calT}$) and the sub-matrix $F|_{\calT} \in \calF_\kappa^{+}(\calY|_{\calT}, \calE|_{\calT})$ is defined such that for any $(y,y') \in \calE|_{\calT}$, \begin{equation*} F|_{\calT}(y,y') = F(y,y'). \end{equation*} \item[Block removal.] The matrix $F$ where the block $(x_0, x_0') \in \kappa_2(\calE)$ has been removed is defined as $F^{\setminus (x_0, x_0')}\in \calF_\kappa^{+}(\calY, \calE \setminus (\calS_{x_0} \times \calS_{x'_0}))$ where for any $(y,y') \in \calE \setminus (\calS_{x_0} \times \calS_{x'_0})$, \begin{equation*} F^{\setminus (x_0, x_0')}(y,y') = F(y,y'). \end{equation*} \end{description} \end{definition} \subsubsection{Redundant merging block criterion} Critically, while both operations in Definition~\ref{definition:sub-matrix-block-removal} preserve lumpability, any of the two can disrupt strong connectivity. When a block can be removed while in some sense preserving irreducibility, we say that it is redundant. \begin{definition}[Redundant block] \label{definition:redundant-block} Let $(x_0, x_0') \in \kappa_2(\calE)$. If there exists $\calT \subset \calX$ such that the following condition holds: \begin{enumerate}[$(i)$] \item The block $(x_0, x_0') \in \calT^2$. \item The sub-graph $(\calY|_{\calT}, \calE|_{\calT} \setminus (\calS_{x_0} \times \calS_{x_0'}))$ is strongly connected. \end{enumerate} Then we say that the block $(x_0, x_0')$ is redundant. \end{definition} \begin{theorem}[Redundant merging block criterion] \label{theorem:redundant-merging-block-criterion} If $(\calY, \calE)$ has a multi-row merging block with respect to $\kappa$ (Definition~\ref{definition:merging-block}) which is redundant (Definition~\ref{definition:redundant-block}), then $\calW_{\kappa}(\calY, \calE)$ does not form an e-family. \end{theorem} \begin{proof} We suppose that $(\calY, \calE)$ has a multi-row merging block $(x_0, x_0') \in \kappa_2(\calE)$, which is redundant. Recall from the proof Theorem~\ref{theorem:characterization-log-linearity} that there exist stochastic matrices $P_0, P_1 \in \calW_{\kappa}(\calY, \calE)$ such that $\widetilde{P}_{1/2} = P_0^{\hadamard 1/2} \hadamard P_1^{\hadamard 1/2} \not \in \calF_{\kappa}^{+}(\calY, \calE)$. From Corollary~\ref{corollary:geometric-intersection-interpretation}, it is enough to show that $[\widetilde{P}_{1/2}] \cap \calF_\kappa^{+}(\calY, \calE) = \emptyset$ in order to prove that $\stoch(\widetilde{P}_{1/2}) \not \in \calW_\kappa(\calE, \calY)$. In other words, it is sufficient to show that for any $v \in \bbR_+^{\calY}$, the rescaled $\diag(v) \widetilde{P}_{1/2}\diag(v)^{-1}$ is not $\kappa$-lumpable. Let us suppose for contradiction that there exists $v \in \bbR_+^{\calY}$, such that $\diag(v) \widetilde{P}_{1/2}\diag(v)^{-1} \in \calF_{\kappa}^{+}(\calY, \calE)$. Since $(x_0,x_0')$ is redundant, there exists $\calT \subset \calX$ such that $(x_0, x_0') \in \calT^2$ and $(\calY|_{\calT}, \calE|_{\calT} \setminus (\calS_{x_0} \times \calS_{x_0'}))$ is strongly connected. On one hand, by our assumption, it must hold that \begin{equation*} \diag(v) \left(\widetilde{P}_{1/2}|_{\calT}^{\setminus (x_0, x_0')}\right)\diag(v)^{-1} \in \calF_{\kappa}^{+}(\calY|_{\calT}, \calE|_{\calT} \setminus (\calS_{x_0} \times \calS_{x'_0})). \end{equation*} However, observe that by our construction in the proof of Theorem~\ref{theorem:characterization-log-linearity}, we also have \begin{equation*} \widetilde{P}_{1/2}|_{\calT}^{\setminus (x_0, x_0')} \in \calF_{\kappa}^{+}(\calY|_{\calT}, \calE|_{\calT} \setminus (\calS_{x_0} \times \calS_{x'_0})), \end{equation*} and it follows from irreducibility of $(\calY|_{\calT}, \calE|_{\calT} \setminus (\calS_{x_0} \times \calS_{x'_0}))$ and an application of Proposition~\ref{proposition:eigenvector-rescaling-constant-on-each-lumped-point} that $v$ must be constant over each $\calS_{x}$ for $x \in \calT$, and in particular for $x \in \set{ x_0, x_0'}$. However, in this case, inspecting the multi-row merging block $(x_0, x_0')$, \begin{equation*} \begin{split} \sum_{\overline{y} \in \calS_{x_0'}} (\diag(v) \widetilde{P}_{1/2} \diag(v)^{-1})(y_0, \overline{y}) &= \frac{v_{x_0}}{v_{x_0'}} \frac{1}{\abs{\overline{x} \in \calX \colon (x_0, \overline{x}) \in \calD}}, \end{split} \end{equation*} while \begin{equation*} \begin{split} \sum_{\overline{y} \in \calS_{x_0'}} (\diag(v) \widetilde{P}_{1/2} \diag(v)^{-1})(y_\star, \overline{y}) &= \frac{v_{x_0}}{v_{x_0'}} \left( \frac{1}{\abs{\overline{x} \in \calX \colon (x_0, \overline{x}) \in \calD}} + 2 \sqrt{\eta_a \eta_b} - (\eta_a + \eta_b) \right), \end{split} \end{equation*} which cannot be equal from the AM-GM inequality and the assumption that $\eta_a \neq \eta_b$. \end{proof} \begin{example} Let $\calY = \set{0,1,2,3,4,5}, \calX= \set{a,b,c}$, the lumping map $\kappa$ defined by the partition $\calS_{a} = \set{0,1}, \calS_{b} = \set{2,3}, \calS_{c} = \set{4,5}$, and consider the lumpable family \begin{equation*} \calW_{\kappa}(\calY, \calE) \sim \left(\begin{tblr}{cc|[dashed]cc|[dashed]cc} \plus & \zero & \plus & \plus & \plus & \zero \\ \zero & \plus & \plus & \plus & \zero & \plus \\\hline[dashed] \plus & \zero & \plus & \zero & \plus & \plus \\ \zero & \plus & \zero & \plus & \plus & \plus \\\hline[dashed] \plus & \plus & \plus & \zero & \plus & \zero \\ \plus & \plus & \zero & \plus & \zero & \plus \\ \end{tblr}\right). \end{equation*} Here, $(b,c) \in \calD$ is a merging block. When we remove edges pertaining to the block $(b,c)$---that is we remove $(2, 4), (2, 5), (3, 4), (3, 5)$---there exists a closed path $$1 \rightarrow 2 \rightarrow 0 \rightarrow 4 \rightarrow 2 \rightarrow 0 \rightarrow 3 \rightarrow 1 \rightarrow 5 \rightarrow 3 \rightarrow 1,$$ going through all the states in $\calY$. As a result, the block $(b, c)$ is redundant and from Theorem~\ref{theorem:redundant-merging-block-criterion}, $\calW_{\kappa}(\calY, \calE)$ does not form an e-family. \end{example} \begin{corollary}[Complete graph] When $\abs{\calY} \geq 4$, unless $\kappa$ is degenerate, $\calW_{\kappa}(\calY, \calY^2)$ does not form an e-family. \end{corollary} \begin{proof} When $\kappa$ is non-degenerate, observe that $(\calY, \calY^2)$ must have a multi-row merging block. Additionally, removing this block still yields a strongly connected graph. From a direct application of Theorem~\ref{theorem:redundant-merging-block-criterion}, it follows that $\calW_{\kappa}(\calY, \calY^2)$ does not form an e-family. \end{proof} The most intriguing scenarios occur in the intermediate range between having no multi-row merging blocks and all blocks being multi-row merging. Indeed, as the number of edges in the connection graph grows, it becomes increasingly challenging for the lumpable family to form an e-family. This intuition will be rigorously formalized in Section~\ref{section:monotonicity}. \subsubsection{Dimensional criterion} We introduce the set \begin{equation*} \calG_\kappa(\calY, \calE) \eqdef \set{ \log F \colon F \in \calF^+_\kappa(\calY, \calE) } \end{equation*} and its affine hull in $\calF(\calY, \calE)$, \begin{equation*} \aff(\calG_\kappa(\calY, \calE)) \eqdef \set{ \sum_{i = 1}^{k} \alpha_i G_i \colon k \in \bbN, \alpha \in \bbR^k, \sum_{i=1}^{k} \alpha_i = 1, G_1, \dots, G_k \in \calG_\kappa(\calY, \calE) }. \end{equation*} From Theorem~\ref{theorem:characterization-log-linearity}, we know that $\aff(\calG_\kappa(\calY, \calE)) = \calG_\kappa(\calY, \calE)$---that is $\calG_\kappa(\calY, \calE)$ is an affine space---if and only if $(\calY, \calE)$ has no multi-row merging block with respect to $\kappa$. For convenience, we define \begin{equation} \label{equation:block-type-nomenclature} \begin{split} \calM_{x,x'} &\eqdef \set{ y \in \calS_{x} \colon \exists y_1', y_2' \in \calS_{x'}, y_1' \neq y_2', (y,y_1'), (y,y_2') \in \calE} \qquad \text{for any $(x,x') \in \calD$}, \\ \calM &\eqdef \set{ (x,x') \in \calD \colon \calM_{x,x'} \neq \emptyset }, \\ \calU &\eqdef \set{(x,x') \in \calD \colon \calM_{x,x'} \neq \calS_{x}}, \\ \calR &\eqdef \bigcup_{(x_0,x_0') \in \calM } \set{ (y_0, y_0') \in \calM_{x_0, x_0'} \times \calS_{x_0'} \colon (y_0, y_0') \in \calE}. \end{split} \end{equation} Thus $\calM$ is the collection of merging blocks---not necessarily multi-row---with respect to $(\calY, \calE)$ and $\kappa$, for any block $(x,x') \in \calD$, $\calM_{x,x'} \subset \calS_{x}$ is its set of merging rows, $\calU$ is the collection of all blocks containing at least one non-merging row, and $\calR$ is the set of all edges which appear in some merging row. In particular, when $(x,x') \in \calD \setminus \calM$ it holds that $\calM_{x,x'} = \emptyset$. For each $(x,x') \in \calD$, we fix a set of $\abs{\calS_x}$ anchor edges \begin{equation} \label{eq:anchor-points} E_{x,x'} \eqdef \set{ \left(\bar{y}_1, \bar{y}_1'\right), \dots, \left(\bar{y}_{\abs{\calS_x}}, \bar{y}'_{\abs{\calS_x}}\right) \in \calE \cap \left(\calS_{x} \times \calS_{x'}\right), i \neq j \implies \bar{y}_i \neq \bar{y}_j}. \end{equation} \begin{lemma} \label{lemma:affine-span-log-lumpable-cone-is-linear} $\aff(\calG_{\kappa}(\calY, \calE))$ is a linear space\footnote{In particular, it contains the null vector.}. \end{lemma} \begin{proof} By definition $\aff(\calG_{\kappa}(\calY, \calE))$ is an affine space, so it suffices to show that it contains the null vector. It will be convenient to introduce $N \eqdef \max_{x \in \calX} \abs{\calS_{x}}$, and for $y \in \calY, x' \in \calX$, \begin{equation} \label{equation:definition-s} s_{y,x'} \eqdef \abs{ \left(\set{y} \times \calS_{x'}\right) \cap \calE }. \end{equation} For $\alpha \in (0, 1/N)$, we let $F_\alpha \in \calF(\calY, \calE)$ be such that for any $(y,y') \in \calE$, \begin{equation*} F_\alpha(y,y') = \begin{cases} \alpha &\text{ when } (y,y') \not \in E_{\kappa(y),\kappa(y')}, \\ 1 - \left( s_{y,\kappa(y')} - 1 \right)\alpha &\text{ otherwise}. \end{cases} \end{equation*} We similarly introduce $F_\beta$ for $\beta \in (0, 1/N)$ with $\beta \neq \alpha$. By construction, $F_\alpha, F_\beta \in \calF_{\kappa}^{+}(\calY, \calE)$. For $t \in \bbR$, let us inspect $G_t = t \log F_\alpha + (1 - t) \log F_\beta$. Setting \begin{equation*} t = \frac{\log \beta}{\log \left( \frac{\beta}{\alpha} \right)}, \end{equation*} we obtain \begin{equation*} G_t(y,y') = \begin{cases} 0 \qquad \qquad &\text{ when } (y,y') \not \in E_{\kappa(y),\kappa(y')}, \\ \log \left( 1 - \left(s_{y,\kappa(y')} - 1\right) \beta \right) + \cfrac{\log \beta}{\log \left( \frac{\beta}{\alpha} \right)} \log \left( \cfrac{1 - \left(s_{y,\kappa(y')} - 1\right) \alpha}{1 - \left(s_{y,\kappa(y')} - 1\right) \beta} \right) &\text{ otherwise}. \end{cases} \end{equation*} Further setting $\alpha = \delta$ and $\beta = 2 \delta$ for $\delta \in (0, 1/(2N))$, we obtain \begin{equation*} G_t(y,y') = \begin{cases} 0 \qquad \qquad &\text{ when } (y,y') \not \in E_{\kappa(y),\kappa(y')}, \\ \log \left( 1 - 2 \left(s_{y,\kappa(y')} - 1\right) \delta \right) + \cfrac{\log (2 \delta)}{\log \left( 2 \right)} \log \left( \cfrac{1 - \left(s_{y,\kappa(y')} - 1\right) \delta}{1 - 2 \left(s_{y,\kappa(y')} - 1\right) \delta} \right) &\text{ otherwise}. \end{cases} \end{equation*} For any fixed $s$, the function \begin{equation*} h \colon \delta \mapsto \log \left( 1 - 2 (s - 1) \delta \right) + \cfrac{\log (2 \delta)}{\log \left( 2 \right)} \log \left( \cfrac{1 - (s - 1) \delta}{1 - 2 (s - 1) \delta} \right) \end{equation*} is continuous, negative, decreasing, and satisfies $\lim_{\delta^+ \to 0} h(\delta) = 0$. So for any $\eps > 0$ there exists $\delta_s$ such that $h(\delta_s) < \eps$. Finally, taking $s_\star = \min \set{s_{y, \kappa(y')} \colon (y,y') \in E_{x,x'}, (x,x') \in \calD}$, for $\delta < \delta_\star$, we have that $\nrm{G_t} < \eps \sqrt{\abs{\calE}}$. In other words $0 \in \calF(\calY, \calE)$ can be characterized as an accumulation point of a sequence of elements of $\aff(\calG_\kappa(\calY, \calE))$. Since $\aff(\calG_{\kappa}(\calY, \calE))$ is an affine subspace of $\calF(\calY, \calE) = \bbR^{\calE}$, it is closed---it contains all its accumulation points, in particular the null function. \end{proof} Since they coincide, we henceforth write $\spann(\calG_{\kappa}(\calY, \calE)) = \aff(\calG_{\kappa}(\calY, \calE))$. \begin{proposition} For $(x_0,x_0') \in \calU$, let $G^{\uparrow}_{x_0,x_0'} \in \calF(\calY,\calE)$ be such that for any $(y,y') \in \calE$, \begin{equation*} G^{\uparrow}_{x_0,x_0'}(y,y') = \pred{ (y,y') \in E_{x_0,x_0'} }, \end{equation*} where $E_{x_0,x_0'}$ is introduced in \eqref{eq:anchor-points} and the sets $\calU, \calR$ are introduced in Eq.~\eqref{equation:block-type-nomenclature}. For any $(y_0, y_0') \in \calR$, let $G^{\uparrow}_{y_0, y_0'} \in \calF(\calY, \calE)$ be such that for any $(y,y') \in \calE$, \begin{equation*} G^{\uparrow}_{y_0, y_0'}(y,y') = \pred{(y,y') = (y_0, y_0')}. \end{equation*} The system of functions \begin{equation*} \calB_{\kappa}(\calY, \calE) \eqdef \set{G^{\uparrow}_{x_0,x_0'} \colon (x_0,x_0') \in \calU} \cup \set{G^{\uparrow}_{y_0, y_0'} \colon (y_0,y_0') \in \calR}, \end{equation*} forms a basis for $\spann(\calG_{\kappa}(\calY, \calE))$, and \begin{equation*} \dim \spann(\calG_\kappa(\calY, \calE)) = \abs{\calU} + \abs{\calR}. \end{equation*} \end{proposition} \begin{proof} For any $(x_0,x_0') \in \calD$, and for $b \in \set{0, 1}$, we let $F^{(b)}_{x_0,x_0'} \in \calF(\calY, \calE)$ be such that for any $(y,y') \in \calE$, \begin{equation*} F^{(b)}_{x_0,x_0'}(y,y') \eqdef \begin{cases} \frac{e^{b}}{s_{y,\kappa(y')}} &\text{ when } (\kappa(y),\kappa(y')) = (x_0,x_0'), \\ \frac{1}{s_{y,\kappa(y')}} &\text{ otherwise}, \end{cases} \end{equation*} where $s_{y, \kappa(y')}$ is defined in Eq.~\eqref{equation:definition-s}. By construction, $F^{(b)}_{x_0,x_0'} \in \calF_\kappa^{+}(\calY, \calE)$, \begin{equation*} \log F^{(1)}_{x_0,x_0'} - \log F^{(0)}_{x_0,x_0'} \in \spann(\calG_{\kappa}(\calY, \calE)), \end{equation*} where we relied on the fact that $\aff(\calG_{\kappa}(\calY, \calE)) = \spann(\calG_{\kappa}(\calY, \calE))$ (Lemma~\ref{lemma:affine-span-log-lumpable-cone-is-linear}), and when $(x_0, x_0') \not \in \calM$, we have \begin{equation*} \log F^{(1)}_{x_0,x_0'} - \log F^{(0)}_{x_0,x_0'} = G^{\uparrow}_{x_0,x_0'}. \end{equation*} Next, for $(y_0, y_0') \in \calR$, note that $s_{y_0, \kappa(y_0')} > 1$, and let us construct \begin{equation*} \widetilde{F}^{\uparrow}_{y_0, y'_0}(y,y') \eqdef \begin{cases} \frac{1}{2} &\text{ when } (y, y') = (y_0, y_0'), \\ \frac{1}{2 \left(s_{y_0,\kappa(y'_0)} - 1\right)} &\text{ when } y = y_0, \kappa(y') = \kappa(y'_0), y' \neq y_0', \\ \frac{1}{s_{y,\kappa(y')}} &\text{ otherwise}. \end{cases} \end{equation*} Observe that \begin{equation*} \left(\log \widetilde{F}^{\uparrow}_{y_0, y'_0} - \log F^{(0)}_{\kappa(y_0), \kappa(y_0')} \right)(y,y') = \begin{cases} \log\left(\frac{s_{y_0,\kappa(y'_0)}}{2}\right) &\text{ when } (y, y') = (y_0, y_0'), \\ \log\left(\frac{s_{y_0,\kappa(y'_0)}}{2\left(s_{y_0,\kappa(y'_0)} - 1\right)}\right) &\text{ when } y = y_0, \kappa(y') = \kappa(y'_0), y' \neq y_0', \\ 0 &\text{ otherwise}. \end{cases} \end{equation*} In order to construct $G_{y_0,y_0'}^{\uparrow}$ we first introduce \begin{equation*} \begin{split} G^{\uparrow}_{y_0, \kappa(y_0')} &\eqdef \frac{\sum_{\bar{y}_0' \in \calS_{\kappa(y_0')}\colon (y_0, \bar{y}_0') \in \calE} \left(\log \widetilde{F}^{\uparrow}_{y_0, \bar{y}'_0} - \log F^{(0)}_{\kappa(y_0),\kappa(y_0')} \right)}{\log\left(s_{y_0, \kappa(y_0')}\right) - \log(2) + \left(s_{y_0, \kappa(y_0')} - 1\right) \left(\log\left(s_{y_0, \kappa(y_0')}\right)- \log(2) - \log\left(s_{y_0, \kappa(y_0')} - 1\right)\right)}.\\ \end{split} \end{equation*} Observe that \begin{equation*} G^{\uparrow}_{y_0, \kappa(y_0')}(y,y') = \begin{cases} 1 &\text{ when } y = y_0 \text{ and } \kappa(y') = \kappa(y_0'), \\ 0 &\text{ otherwise}. \end{cases} \end{equation*} We then construct $G_{y_0,y_0'}^{\uparrow}$ as \begin{equation*} \begin{split} G_{y_0,y_0'}^{\uparrow} &= \left(1 - \frac{ \log\left(s_{y_0, \kappa(y_0')} - 1\right)}{\log\left( s_{y_0, \kappa(y_0')}\right) - \log(2)} \right) \left( \frac{\log \widetilde{F}^{\uparrow}_{y_0, y'_0} - \log F^{(0)}_{\kappa(y_0),\kappa(y_0')}}{\log\left(s_{y_0, \kappa(y_0')}\right) - \log(2) - \log\left(s_{y_0, \kappa(y_0')} - 1\right)} - G^{\uparrow}_{y_0, \kappa(y_0')} \right). \end{split} \end{equation*} We obtain that for $(y,y') \in \calE$, \begin{equation*} \begin{split} G^{\uparrow}_{y_0, y_0'}(y,y') &= \begin{cases} 1 &\text{ when } (y,y') = (y_0, y_0'), \\ 0 &\text{ otherwise}. \end{cases} \end{split} \end{equation*} The construction of $G^{\uparrow}_{x_0,x_0'}$ for $(x_0,x_0') \in \calU$ follows, \begin{equation*} G^{\uparrow}_{x_0,x_0'} = \log F^{(1)}_{x_0,x_0'} - \log F^{(0)}_{x_0,x_0'} - \sum_{(y_0, y_0') \in \calR \cap \left( \calS_{x_0} \times \calS_{x_0'} \right)} G^{\uparrow}_{y_0, y_0'}. \end{equation*} Generativity and linear independence of the system of function are immediate. \end{proof} \begin{example} \label{example:basis-affine-hull-log-lumpable-cone} Consider \begin{equation*} \calW_{\kappa}(\calY, \calE) \sim \left(\begin{tblr}{c|[dashed]ccc} \oplus & \oplus & \plus & \zero \\\hline[dashed] \oplus & \oplus & \plus & \plus \\ \oplus & \zero & \oplus & \zero \\ \oplus & \zero & \zero & \oplus \\ \end{tblr}\right), \end{equation*} where $\oplus$ indicates that the edge is fixed as an anchor--- that is it belongs to $E_{x,x'}$ for some $(x,x') \in \calD$. Then $\spann(\calG_\kappa(\calY, \calE))$ is spanned by \begin{equation*} \begin{split} \left(\begin{tblr}{c|[dashed]ccc} 1 & \zero & \zero & \zero \\\hline[dashed] \zero & \zero & \zero & \zero \\ \zero & \zero & \zero & \zero \\ \zero & \zero & \zero & \zero \\ \end{tblr}\right), \left(\begin{tblr}{c|[dashed]ccc} \zero & 1 & \zero & \zero \\\hline[dashed] \zero & \zero & \zero & \zero \\ \zero & \zero & \zero & \zero \\ \zero & \zero & \zero & \zero \\ \end{tblr}\right),\left(\begin{tblr}{c|[dashed]ccc} \zero & \zero & 1 & \zero \\\hline[dashed] \zero & \zero & \zero & \zero \\ \zero & \zero & \zero & \zero \\ \zero & \zero & \zero & \zero \\ \end{tblr}\right), \left(\begin{tblr}{c|[dashed]ccc} \zero & \zero & \zero & \zero \\\hline[dashed] 1 & \zero & \zero & \zero \\ 1 & \zero & \zero & \zero \\ 1 & \zero & \zero & \zero \\ \end{tblr}\right), \end{split} \end{equation*} \begin{equation*} \begin{split} \left(\begin{tblr}{c|[dashed]ccc} \zero & \zero & \zero & \zero \\\hline[dashed] \zero & 1 & \zero & \zero \\ \zero & \zero & 1 & \zero \\ \zero & \zero & \zero & 1 \\ \end{tblr}\right), \left(\begin{tblr}{c|[dashed]ccc} \zero & \zero & \zero & \zero \\\hline[dashed] \zero & 1 & \zero & \zero \\ \zero & \zero & \zero & \zero \\ \zero & \zero & \zero & \zero \\ \end{tblr}\right), \left(\begin{tblr}{c|[dashed]ccc} \zero & \zero & \zero & \zero \\\hline[dashed] \zero & \zero & 1 & \zero \\ \zero & \zero & \zero & \zero \\ \zero & \zero & \zero & \zero \\ \end{tblr}\right), \left(\begin{tblr}{c|[dashed]ccc} \zero & \zero & \zero & \zero \\\hline[dashed] \zero & \zero & \zero & 1 \\ \zero & \zero & \zero & \zero \\ \zero & \zero & \zero & \zero \\ \end{tblr}\right). \end{split} \end{equation*} \end{example} \begin{example} Consider \begin{equation*} \calW_{\kappa}(\calY, \calE) \sim \left(\begin{tblr}{cc|[dashed]cc} \oplus & \zero & \oplus & \zero \\ \zero & \oplus & \zero & \oplus \\\hline[dashed] \zero & \oplus & \oplus & \plus \\ \oplus & \zero & \zero & \oplus \\ \end{tblr}\right). \end{equation*} Then $\spann(\calG_{\kappa}(\calY, \calE))$ is spanned by \begin{equation*} \begin{split} \left(\begin{tblr}{cc|[dashed]cc} 1 & \zero & \zero & \zero \\ \zero & 1 & \zero & \zero \\\hline[dashed] \zero & \zero & \zero & \zero \\ \zero & \zero & \zero & \zero \\ \end{tblr}\right), \left(\begin{tblr}{cc|[dashed]cc} \zero & \zero & 1 & \zero \\ \zero & \zero & \zero & 1 \\\hline[dashed] \zero & \zero & \zero & \zero \\ \zero & \zero & \zero & \zero \\ \end{tblr}\right),\left(\begin{tblr}{cc|[dashed]cc} \zero & \zero & \zero & \zero \\ \zero & \zero & \zero & \zero \\\hline[dashed] \zero & 1 & \zero & \zero \\ 1 & \zero & \zero & \zero \\ \end{tblr}\right),\\ \left(\begin{tblr}{cc|[dashed]cc} \zero & \zero & \zero & \zero \\ \zero & \zero & \zero & \zero \\\hline[dashed] \zero & \zero & 1 & \zero \\ \zero & \zero & \zero & 1 \\ \end{tblr}\right),\left(\begin{tblr}{cc|[dashed]cc} \zero & \zero & \zero & \zero \\ \zero & \zero & \zero & \zero \\\hline[dashed] \zero & \zero & 1 & \zero \\ \zero & \zero & \zero & \zero \\ \end{tblr}\right), \left(\begin{tblr}{cc|[dashed]cc} \zero & \zero & \zero & \zero \\ \zero & \zero & \zero & \zero \\\hline[dashed] \zero & \zero & \zero & 1 \\ \zero & \zero & \zero & \zero \\ \end{tblr}\right). \end{split} \end{equation*} \end{example} We recall the definition of the exponential hull of a family of stochastic matrices $\calV \subset \calW(\calY, \calE)$ as the smallest exponential family which contains $\calV$. \begin{definition}[Exponential hull, {\citealp[Definition~7]{wolfer2021information}}] \label{definition:e-hull} Let $\calV$ be a sub-family of $\calW(\calY, \calE)$. \begin{equation*} \begin{split} \ehull(\mathcal{V}) \eqdef \Bigg\{ \stoch(\widetilde{P}) &\colon \log \widetilde{P} = \sum_{i = 1}^{k} \alpha_i \log P_i, k \in \bbN, \alpha \in \bbR^k, \sum_{i = 1}^{k} \alpha_i = 1, P_1, \dots P_k \in \calV \Bigg\}, \end{split} \end{equation*} where $\stoch$-normalization was introduced in Definition~\ref{definition:s-normalization}. \end{definition} In particular, $\ehull(\calV) = \calV$ if and only if $\calV$ forms an e-family. For instance, it is known that the exponential hull of symmetric stochastic matrices yields the reversible family \citep[Theorem~9]{wolfer2021information}. In our analysis, it will be convenient to further define $\calH_\kappa(\calY, \calE)$ and $\overline{\calH}_\kappa(\calY, \calE)$ as follows. \begin{equation*} \begin{split} \calH_\kappa(\calY, \calE) &\eqdef \set{ \log P \colon P \in \calW_{\kappa}(\calY, \calE)} \subset \calG_{\kappa}(\calY, \calE), \\ \overline{\calH}_\kappa(\calY, \calE) &\eqdef \set{ \log P \colon P \in \ehull(\calW_{\kappa}(\calY, \calE))}. \end{split} \end{equation*} Observe that $\overline{\calH}_\kappa(\calY, \calE)$ is isomorphic to the affine hull of $\calH_\kappa(\calY, \calE)$ in $\calF(\calY, \calE)/\calN(\calY, \calE)$. \begin{proposition} \label{proposition:log-exponential-hull-isomorphic-to-quotient} \begin{equation*} \overline{\calH}_\kappa(\calY, \calE) \cong \left(\spann(\calG_\kappa(\calY, \calE)) \oplus \calN(\calY, \calE) \right) / \calN(\calY, \calE). \end{equation*} \end{proposition} \begin{proof} Let $\overline{H} \in \overline{\calH}_\kappa(\calY, \calE)$. There exists $\overline{G} \in \spann(\calG_{\kappa}(\calY, \calE))$ such that \begin{equation*} \exp\left(\overline{H}\right) = \stoch\left( \exp \left(\overline{G}\right) \right). \end{equation*} It follows that we can write \begin{equation*} \overline{H} = \overline{G} + N, \end{equation*} for some $N \in \calN(\calY, \calE)$. As a result, \begin{equation*} \overline{H} \in \left(\spann(\calG_\kappa(\calY, \calE)) \oplus \calN(\calY, \calE) \right) / \calN(\calY, \calE). \end{equation*} Conversely, let $\overline{G} \in \spann(\calG_{\kappa}(\calY, \calE))$ and $N \in \calN(\calY, \calE)$. It holds that \begin{equation*} \stoch \left( \overline{G} + N \right) \in \ehull(\calW_{\kappa}(\calY, \calE)), \end{equation*} hence \begin{equation*} \left[\overline{G} + N \right]_{\calN(\calY, \calE)} \in \overline{\calH}_\kappa(\calY, \calE). \end{equation*} \end{proof} \begin{theorem}[Dimensional criterion] \label{theorem:dimensional-criterion} If \begin{equation*} \dim \left(\spann\left(\calG_\kappa(\calY, \calE)\right) \oplus \calN(\calY, \calE) \right) > \abs{\calE} + \abs{\calY} + \abs{\calD} - \abs{\calX} - \sum_{(x,x') \in \calD} \abs{\calS_x}, \end{equation*} then $\calW_{\kappa}(\calY, \calE)$ does not form an e-family. \end{theorem} \begin{proof} Suppose that $\calW_{\kappa}(\calY, \calE)$ forms an e-family. Then $\ehull(\calW_{\kappa}(\calY, \calE)) = \calW_{\kappa}(\calY, \calE)$ and \begin{equation*} \dim \overline{\calH}_{\kappa}(\calY, \calE) = \dim \calW_{\kappa}(\calY, \calE). \end{equation*} It follows follows then from Proposition~\ref{proposition:log-exponential-hull-isomorphic-to-quotient} that \begin{equation*} \dim \left(\spann\left(\calG_\kappa(\calY, \calE)\right) \oplus \calN(\calY, \calE) / \calN(\calY, \calE) \right) = \dim \calW_{\kappa}(\calY, \calE). \end{equation*} Recall that $\dim \calN(\calY, \calE) = \abs{\calY}$, and from Theorem~\ref{theorem:foliation-of-lumpable-kernels}, that \begin{equation*} \dim \calW_\kappa(\calY, \calE) = \abs{\calE} - \sum_{(x,x') \in \calD} \abs{\calS_x} + \abs{\calD} - \abs{\calX}, \end{equation*} whence the theorem holds. \end{proof} A basis for $\calN(\calY, \calE)$ can be constructed as follows. For $y_0 \in \calY$, we define $N_{y_0} \in \calF(\calY, \calE)$ be such that for any $(y,y') \in \calE$, \begin{equation*} N_{y_0}(y,y') = \pred{y' = y_0} - \pred{y = y_0}. \end{equation*} Let $y_\star \in \calY$ be arbitrary; then \begin{equation*} \set{C} \cup \set{ N_{y_0} \colon y_0 \in \calY \setminus \set{y_\star}}, \end{equation*} where $C \equiv 1$ is the constant unit function over $\calE$, forms a basis of $\calN(\calY, \calE)$. As a consequence, the bases for $\spann(\calG_\kappa(\calY, \calE))$ and $\calN(\calY, \calE)$ can be concatenated, and determining the rank of the resulting family can be obtained algorithmically, for instance with Gaussian elimination. Before performing this somewhat costly computation, one can also proceed with the following preliminary verification which does not require computing the rank of a family of functions. \begin{corollary} \label{corollary:dimensional-criterion-simplified} If \begin{equation*} \underbrace{\sum_{(x,x') \in \calD} \abs{\calS_x}}_{\# \text{ of rows in every block}} > \underbrace{\left( \abs{\calD} - \abs{\calU} \right)}_{\# \text{ completely merging blocks }} + \underbrace{\left(\abs{\calY} - \abs{\calX}\right)}_{\text{state space compression}} + \underbrace{\left(\abs{\calE} - \abs{\calR} \right)}_{\# \text{ of non-merging transitions}}, \end{equation*} where $\calU$ and $\calR$ are defined in Eq.~\eqref{equation:block-type-nomenclature}, then $\calW_{\kappa}(\calY, \calE)$ does not form an e-family. \end{corollary} \begin{proof} \begin{equation*} \dim \spann\left(\calG_\kappa(\calY, \calE)\right) \leq \dim \left(\spann\left(\calG_\kappa(\calY, \calE)\right) \oplus \calN(\calY, \calE) \right). \end{equation*} \end{proof} \begin{example}[Example~{\ref{example:basis-affine-hull-log-lumpable-cone}} continued] Recall the family \begin{equation*} \calW_{\kappa}(\calY, \calE) \sim \left(\begin{tblr}{c|[dashed]ccc} \oplus & \oplus & \plus & \zero \\\hline[dashed] \oplus & \oplus & \plus & \plus \\ \oplus & \zero & \oplus & \zero \\ \oplus & \zero & \zero & \oplus \\ \end{tblr}\right). \end{equation*} A basis of functions $\calB_\kappa(\calY, \calE)$ for $\calG_{\kappa}(\calY, \calE)$ is given in Example~\ref{example:basis-affine-hull-log-lumpable-cone}. The space $\calN(\calY, \calE)$ is spanned by \begin{equation*} \left(\begin{tblr}{c|[dashed]ccc} 1 & 1 & 1 & 0 \\\hline[dashed] 1 & 1 & 1 & 1 \\ 1 & \zero & 1 & \zero \\ 1 & \zero & \zero & 1 \\ \end{tblr}\right), \left(\begin{tblr}{c|[dashed]ccc} \zero & 1 & \zero & \zero \\\hline[dashed] -1 & \zero & -1 & -1 \\ \zero & \zero & \zero & \zero \\ \zero & \zero & \zero & \zero \\ \end{tblr}\right), \left(\begin{tblr}{c|[dashed]ccc} \zero & \zero & 1 & \zero \\\hline[dashed] \zero & \zero & 1 & \zero \\ -1 & \zero & \zero & \zero \\ \zero & \zero & \zero & \zero \\ \end{tblr}\right), \left(\begin{tblr}{c|[dashed]ccc} \zero & \zero & \zero & \zero \\\hline[dashed] \zero & \zero & \zero & 1 \\ \zero & \zero & \zero & \zero \\ -1 & \zero & \zero & \zero \\ \end{tblr}\right). \end{equation*} On one end, the manifold dimension of the lumpable family is given by \begin{equation*} \dim \calW_{\kappa}(\calY, \calE) = \abs{\calE} + \abs{\calD} - \abs{\calX} - \sum_{(x,x') \in \calD} \abs{\calS_{x}} = 11 + 4 - 2 - 8 = 5. \end{equation*} On the other hand, concatenating the two bases, one can algorithmically verify that \begin{equation*} \begin{split} \dim \left(\spann\left(\calG_\kappa(\calY, \calE)\right) \oplus \calN(\calY, \calE) \right) &= 10. \end{split} \end{equation*} Consequently, \begin{equation*} \dim \left(\spann\left(\calG_\kappa(\calY, \calE)\right) \oplus \calN(\calY, \calE) \right) > \dim \calW_{\kappa}(\calY, \calE) + \dim \calN(\calY, \calE), \end{equation*} and from Theorem~\ref{theorem:dimensional-criterion}, $\calW_{\kappa}(\calY, \calE)$ does not form an e-family. \end{example} \section{Monotonicity and stability} \label{section:monotonicity} In this section, we fix two state spaces $\calX, \calY$ and a surjective non-trivial lumping function $\kappa \colon \calY \to \calX$. For edge sets $\calE, \calE' \subset \calY^2$ such that $\calE \subset \calE'$, it is clear that when $(\calY, \calE)$ is strongly connected, so must be $(\calY, \calE')$, and the graph $(\calY, \calE)$ can be incrementally transformed into $(\calY, \calE')$ by adding edges, naturally leading to the construction of a monotonous sequence of consecutive elements $\calE = \calE_0 \subsetneq \calE_1 \subsetneq \dots \subsetneq \calE_{L} = \calE'$, where $L = \abs{\calE'} - \abs{\calE}$. When considering lumpability, more care is required. Indeed, even if $\calW_\kappa(\calY, \calE)$ is non-vacuous, adding a single edge can easily disrupt lumpability, leading to $\calW_\kappa(\calY, \calE') = \emptyset$. Only by adding edges in already existing blocks, or introducing new valid blocks can we guarantee the existence of lumpable chains under the new graph. We first explain how to construct a monotonous sequence of essentially consecutive families of irreducible and lumpable stochastic matrices. \begin{lemma}[Chaining] \label{lemma:chaining-edge-sets} Let $\calE, \calE' \in \calY^2$ be such that $\calE \subset \calE'$, and $\calW_{\kappa}(\calY, \calE)$ and $\calW_{\kappa}(\calY, \calE')$ are non-vacuous families of lumpable stochastic matrices. Then, there exists $L \in \bbN$ and a finite sequence of edge sets $\calE_0, \calE_1, \dots, \calE_L \subset \calY^2$, such that \begin{enumerate} \item $\calE_0 = \calE$, $\calE_L = \calE'$. \item The sequence $\{\calE_\ell \}_{\ell = 0, \dots, L }$ is strictly monotone increasing, that is, for any $\ell \in \set{0, \dots, L - 1}$, $\calE_\ell \subsetneq \calE_{\ell+1}$. \item The sequence $\{\calE_\ell \}_{\ell = 0, \dots, L }$ is consecutive, in the sense where for any $\ell \in \set{0, \dots, L -1}$, it holds that for any $\calE'' \subset \calY^2$ such that $\calE_\ell \subset \calE'' \subset \calE_{\ell+1}$ either $\calE'' = \calE_\ell, \calE'' = \calE_{\ell+1}$ or $\calW_{\kappa}(\calY ,\calE'') = \emptyset$. \item For any $\ell \in \set{0, \dots, L}$, $\calW_\kappa(\calY, \calE_\ell)$ is non-vacuous. \item For any $\ell \in \set{0, 1\dots, L - 1}$, $\calE_\ell$ and $\calE_{\ell+1}$ only differ either on an edge-link or a block-link: \begin{description} \item[Edge-link:] We say that $(y_\star, y_\star') \in \calE_{\ell+1}$ is an edge-link between $\calE_{\ell}$ and $\calE_{\ell+1}$ whenever $$\kappa_2(\calE_{\ell+1}) = \kappa_2(\calE_{\ell}), \text{ and } \calE_{\ell+1} \setminus \calE_{\ell} = \set{(y_\star, y_\star')}.$$ \item[Block-link:] We say that $(x_\star, x'_\star) \in \kappa_2(\calE_{\ell+1})$ is a block-link between $\calE_\ell$ and $\calE_{\ell+1}$ whenever $$\kappa_2(\calE_{\ell+1}) \neq \kappa_2(\calE_{\ell}), \calE_{\ell+1} \setminus \calE_{\ell} \subset \calS_{x_\star} \times \calS_{x_\star'} \text{, and} (x_\star, x'_\star) \text{ is non-merging}.$$ \end{description} \end{enumerate} \end{lemma} \begin{proof} Let us set $\calE_0 = \calE$ and let us denote $$\kappa_2(\calE') \setminus \kappa_2(\calE_0) = \set{(x_1,x'_1), \dots , (x_B, x'_B)},$$ the indexed collection of $B \in \bbN$ non-zero blocks in lumped $\calE'$ that vanish in lumped $\calE$. For such a block $b \in [B]$, we denote $s_b = \abs{\calS_{x_b}}$, and we let \begin{equation*} (y_{b,1}, y'_{b,1}), \dots, (y_{b, s_b}, y'_{b, s_b}) \in \calS_{x_b} \times \calS_{x'_b} \cap \calE' \end{equation*} be a collection of $s_b$ pairs such that for any $i,j \in [s_b]$, $i \neq j \implies y_{b,i} \neq y_{b,j}$. Note that such a collection of $s_b$ pairs must necessarily exist from our assumption that $\calW_\kappa(\calY, \calE') \neq \emptyset$. For simplicity, we write \begin{equation*} \Delta \calE_b \eqdef \set{ (y_{b,1}, y'_{b,1}), \dots, (y_{b, s_b}, y'_{b, s_b}) }, \end{equation*} and for $b \in [B]$, we then define inductively $$\calE_{b} = \calE_{b - 1} \cup \Delta \calE_b.$$ Observe that by construction, $\calW_\kappa(\calY, \calE_b) \neq \emptyset$ for any $b = 0, \dots, B$, and that by exhaustion, we obtain $\kappa_2(\calE') = \kappa_2(\calE_{B})$. As a second step, we now denote \begin{equation*} \calE' \setminus \calE_B = \set{ (y_1, y'_1), \dots, (y_{E}, y'_{E}) } \end{equation*} the set of edges in $\calE'$ which are still missing in $\calE_B$. For $e \in [E]$, defining inductively \begin{equation*} \calE_{B + e} = \calE_{B + e - 1} \cup \set{ (y_e, y'_e) }, \end{equation*} we obtain a full chain of non-vacuous lumpable families \begin{equation*} \calE_0 \subsetneq \dots \subsetneq \calE_{B} \subsetneq \calE_{B +1 } \subsetneq \dots \subsetneq \calE_{B + E}, \end{equation*} which concludes the claim with $L = B + E$. \end{proof} \begin{remark} Links offer minimal updates in as much as it is not possible to insert additional (non-vacuous) elements in between links. Moreover, while chains are not unique, observe that they all share the same length \begin{equation*} L = \sum_{(x,x') \in \kappa_{2}(\calE)} \abs{ (\calE' \setminus \calE) \cap (\calS_{x} \times \calS_{x'}) } + \sum_{(x,x') \in \kappa_2(\calE') \setminus \kappa_2(\calE)} \abs{ \calE' \cap (\calS_{x} \times \calS_{x'})} - \abs{\calS_{x}} + 1. \end{equation*} \end{remark} We now state and show a monotonicity property of e-families. \begin{theorem}[Monotonicity] \label{theorem:monotonicity} Let $\calE \subset \calE'$ such that both $\calW_\kappa(\calY, \calE)$ and $\calW_\kappa(\calY, \calE')$ are non-vacuous. If $\calW_\kappa(\calY, \calE')$ forms an e-family, then $\calW_\kappa(\calY, \calE)$ forms an e-family. \end{theorem} \begin{proof} Consider the max-norm on $\calF(\calY, \calE)$ defined by $\nrm{A} = \max_{y,y' \in \calE} \abs{A(y,y')}$. We prove the contrapositive of the claim. Let us suppose that $\calW_\kappa(\calY, \calE)$ does not form an e-family. From \citet[Corollary~3]{nagaoka2005exponential}, there exist $P_0, P_1 \in \calW_\kappa(\calY, \calE)$ and $t_\star \in \bbR_+$ such that $\gamma^{(e)}_{P_0 P_1}(t_\star) \not \in \calW_\kappa(\calY, \calE)$. Constructing a ball with respect to $\nrm{\cdot}$ around any lumpable matrix, it is easy to see that every point is a boundary point. In the boundary, we must also include stochastic matrices where some edges are missing. In fact, the boundary set of $\calW_\kappa(\calY, \calE)$ with respect to the max-norm is given by \begin{equation*} \partial \calW_\kappa(\calY, \calE) = \calW_\kappa(\calY, \calE) \cup \bigcup_{\calE'' \subsetneq \calE} \set{ F \in \calF_{\kappa}^{+}(\calY, \calE'') \colon F 1 \trn = 1 \trn }. \end{equation*} However, since the super-family $\calW(\calY, \calE)$ forms an e-family, it holds that $\gamma^{(e)}_{P_0, P_1} \subset \calW(\calY, \calE)$, thus $\gamma^{(e)}_{P_0, P_1}(t_\star) \not \in \partial \calW_\kappa(\calY, \calE)$. As a consequence, there must exist $\delta \in \bbR_+$ such that the curve at parameter $t_\star$ satisfies \begin{equation} \label{eq:parameter-time-outside} \inf_{P \in \calW_\kappa(\calY, \calE)} \nrm{P - \gamma^{(e)}_{P_0, P_1}(t_\star)} > \delta. \end{equation} We first treat the two simple cases where $\calE$ and $\calE'$ are connected by either an edge-link or a block-link. From the chaining Lemma~\ref{lemma:chaining-edge-sets}, we will then deduce the general claim. \paragraph{Edge-link case:} Let us first assume that $\kappa_2(\calE) = \kappa_2(\calE')$ and that $\calE$ and $\calE'$ differ on a single edge $(y_\star, y_\star') \in \calE' \setminus \calE$, where $(y_\star, y_\star') \in \calS_{x_\star} \times \calS_{x'_\star}$. By construction, since $\calW(\calY, \calE)$ is lumpable, there exists $\bar{y}'_\star \in \calS_{x'_\star}$ with $\bar{y}'_\star \neq y'_\star$ such that $(y_\star, \bar{y}'_\star) \in \calE$. Let $\eta \in (0, \bar{\eta}/2]$ with $\bar{\eta} \eqdef \min \{ P_0(y_\star, \bar{y}'_\star), P_1(y_\star, \bar{y}'_\star) \}$, and for $k \in \{ 0, 1\}$ and $(y,y') \in \calY^2$ set \begin{equation*} \begin{split} P'_k(y, y') &= \begin{cases} 0 &\text{ when } (y,y') \not \in \calE' \\ \eta &\text{ when } (y,y') = (y_\star, y'_\star) \\ P_k(y, y') - \eta &\text{ when } (y,y') = (y_\star, \bar{y}'_\star) \\ P_k(y, y') &\text{ otherwise.} \end{cases} \end{split} \end{equation*} The resulting $P'_0$ an $P'_1$ are irreducible and $\kappa$-lumpable stochastic matrices, that is $P'_0, P'_1 \in \calW_{\kappa}(\calY, \calE')$. We proceed to inspect the curve $\gamma^{(e)}_{P'_0, P'_1} \colon \bbR \to \calW(\calY, \calE')$ and for simplicity, denote $P'_t \eqdef \gamma^{(e)}_{P'_0, P'_1}(t)$ the transition matrix such that for any $(y,y') \in \calY^2$, \begin{equation*} P'_t(y,y') = P'_0(y,y')^{1-t} P'_1(y,y')^{t} \frac{v_{t, \eta}(y')}{\rho_{t, \eta} v_{t, \eta}(y)}, \end{equation*} where $\rho_{t, \eta}$ and $v_{t, \eta}$ are respectively the Perron--Frobenius root and associated eigenvector of $P_0'^{\hadamard (1 - t)} \hadamard P_1'^{\hadamard t}$. Observe that by unicity, for any $t \in \bbR$ it holds that $v_{t} = v_{t, 0}, \rho_{t} = \rho_{t, 0}$, and following analyticity of $\eta \mapsto v_{t, \eta}$ and $\eta \mapsto \rho_{t, \eta}$ on the closed interval $[0, \bar{\eta}/2]$ \citep{kato2013perturbation}, at time parameter $t = t_\star$ achieving \eqref{eq:parameter-time-outside}, it will be convenient to introduce \begin{equation*} \begin{split} \underline{\rho} = \min_{\eta \in [0, \bar{\eta} / 2]} \rho_{t_\star , \eta} > 0, \qquad \underline{v} = \min_{y \in \calY, \eta \in [0, \bar{\eta} / 2]} v_{t_\star , \eta}(y) > 0, \qquad \overline{v} = \max_{y \in \calY, \eta \in [0, \bar{\eta} / 2]} v_{t_\star , \eta}(y). \end{split} \end{equation*} Furthermore, there exists $\bar{\eta}_{\delta} \in \bbR_+$ such that for $\eta < \bar{\eta}_{\delta}$ \begin{equation*} \abs{\frac{v_{t_\star, \eta}(y')}{\rho_{t_\star, \eta} v_{t_\star, \eta}(y)} - \frac{v_{t_\star}(y')}{\rho_{t_\star} v_{t_\star}(y)}} \leq \delta. \end{equation*} For $(y,y') \in \calE' \setminus \set{ (y_\star, y_\star'), (y_\star, \bar{y}'_\star) }$ and any $t \in \bbR$, \begin{equation*} P'_t(y,y') - P_t(y,y') = P_0(y,y')^{1-t} P_1(y,y')^{t} \left(\frac{v_{t, \eta}(y')}{\rho_{t, \eta} v_{t, \eta}(y)} - \frac{v_{t}(y')}{\rho_{t} v_{t}(y)}\right), \end{equation*} thus, \begin{equation*} \abs{P'_{t_\star}(y,y') - P_{t_\star}(y,y')} \leq \abs{\frac{v_{t_\star, \eta}(y')}{\rho_{t_\star, \eta} v_{t_\star, \eta}(y)} - \frac{v_{t_\star}(y')}{\rho_{t_\star} v_{t_\star}(y)}}. \end{equation*} Additionally, \begin{equation*} \begin{split} P'_t(y_\star, y'_\star) - P_t(y_\star, y'_\star) &= P'_t(y_\star, y'_\star) = \eta \frac{v_{t, \eta}(y'_\star)}{\rho_{t, \eta} v_{t, \eta}(y_\star)}, \\ \end{split} \end{equation*} leading to \begin{equation*} \begin{split} \abs{P'_{t_\star}(y_\star, y'_\star) - P_{t_\star}(y_\star, y'_\star)} &\leq \eta \frac{\overline{v}}{\underline{\rho} \underline{v}}. \end{split} \end{equation*} Finally, observing that \begin{equation*} \begin{split} &P'_t(y_\star, \bar{y}'_\star) - P_t(y_\star, \bar{y}'_\star) \\ =& \left(P_0(y_\star, \bar{y}'_\star) - \eta\right)^{1 - t}\left(P_1(y_\star, \bar{y}'_\star) - \eta\right)^{t} \frac{v_{t, \eta}(\bar{y}'_\star)}{\rho_{t, \eta} v_{t, \eta}(y_\star)} - \left(P_0(y_\star, \bar{y}'_\star) - \eta\right)^{1 - t}\left(P_1(y_\star, \bar{y}'_\star) - \eta\right)^{t} \frac{v_{t}(\bar{y}'_\star)}{\rho_{t} v_{t}(y_\star)} \\ &+ \left(P_0(y_\star, \bar{y}'_\star) - \eta\right)^{1 - t}\left(P_1(y_\star, \bar{y}'_\star) - \eta\right)^{t} \frac{v_{t}(\bar{y}'_\star)}{\rho_{t} v_{t}(y_\star)} - P_0(y_\star, \bar{y}'_\star)^{1 - t} P_1(y_\star, \bar{y}'_\star)^{t} \frac{v_{t}(\bar{y}'_\star)}{\rho_{t} v_{t}(y_\star)} \\ \end{split} \end{equation*} we obtain that \begin{equation*} \begin{split} \abs{P'_{t_\star}(y_\star, \bar{y}'_\star) - P_{t_\star}(y_\star, \bar{y}'_\star)} \leq& \abs{\frac{v_{t_\star, \eta}(\bar{y}'_\star)}{\rho_{t_\star, \eta} v_{t_\star, \eta}(y_\star)} - \frac{v_{t_\star}(\bar{y}'_\star)}{\rho_{t_\star} v_{t_\star}(y_\star)}} \\ &+ \left(1 - \left(1 - \frac{\eta}{P_0(y_\star, \bar{y}'_\star)}\right)^{1 - t_\star}\left(1 - \frac{\eta}{P_1(y_\star, \bar{y}'_\star)}\right)^{t_\star} \right) \frac{\overline{v}}{\underline{\rho} \underline{v}}, \\ \end{split} \end{equation*} and for \begin{equation*} \begin{split} \eta < \min \set{ P_0(y_\star, \bar{y}'_\star) \left(1 - \left(1 - \delta \frac{\underline{\rho} \underline{v}}{4 \overline{v}} \right)^{\frac{1}{2(1 -t_\star)}}\right), P_1(y_\star, \bar{y}'_\star) \left(1 - \left(1 - \delta \frac{\underline{\rho} \underline{v}}{4 \overline{v}} \right)^{\frac{1}{2 t_\star}}\right), \bar{\eta}_{ \delta/4} }, \end{split} \end{equation*} the above is smaller than $\delta /2$. As a result, if we also assume that $\eta$ such that $\eta < \bar{\eta}_{\delta/2}$ and $\eta < \frac{\delta \underline{\rho} \underline{v}}{2 \overline{v}}$, it holds that \begin{equation*} \nrm{P_{t_\star}' - P_{t_\star}} \leq \delta / 2. \end{equation*} For any $P \in \calW_\kappa(\calY, \calE')$, it then holds from the triangle inequality that \begin{equation*} \nrm{P - P_{t_\star}} \leq \nrm{P - P'_{t_\star}} + \nrm{P'_{t_\star} - P_{t_\star}}, \end{equation*} thus, \begin{equation*} \nrm{P - P'_{t_\star}} > \nrm{P - P_{t_\star}} - \delta/2 > \delta/2, \end{equation*} and taking the infimum over $\calW_\kappa(\calY, \calE')$ finishes proving that $\gamma^{(e)}_{P'_0, P'_1}(t_\star) \not \in \calW_\kappa(\calY, \calE')$. Invoking \citet[Corollary~3]{nagaoka2005exponential}, we conclude that $\calW_\kappa(\calY, \calE')$ does not form an e-family. \paragraph{Block-link case:} Let us now assume that $\calE'$ and $\calE$ differ by a block-link, that is $\kappa_2(\calE') \setminus \kappa_2(\calE) = \{ (x_\star, x'_\star) \}$, a single non-merging block, and $\calE' \setminus \calE \subset \calS_{x_\star} \times \calS_{x'_\star}$. We denote $s = \abs{\calS_{x_\star}}$ and we enumerate $(y_1, y_1'), \dots, (y_s, y_{s}') \in \calS_{x_\star} \times \calS_{x'_\star}$ the elements which are also in $\calE' \setminus \calE$. Since elements of $\calW_\kappa(\calY, \calE)$ are stochastic matrices, there must exist a block $(x_\star, \bar{x}_\star') \in \kappa_2(\calE), $ with $ \bar{x}_\star' \neq x_\star'$. We let $\eta \in (0, \bar{\eta}/2]$ where $$\bar{\eta} \eqdef \min \{ P_k(y,y') \colon k \in \{ 0, 1 \}, (y, y') \in \calS_{x_\star} \times \calS_{\bar{x}'_\star} \}.$$ We further let $\bar{y}'_1, \dots, \bar{y}'_s \in \calS_{\bar{x}'_\star}$ be such that $(y_1, \bar{y}_1'), \dots, (y_s, \bar{y}'_{s}) \in \calS_{x_\star} \times \calS_{\bar{x}'_\star}$. For $k \in \set{0,1}$ and $(y,y') \in \calY^2$, we construct \begin{equation*} P'_{k}(y,y') = \begin{cases} 0 &\text{ when } (y,y') \not \in \calE' \\ \eta &\text{ when } (y, y' ) \in \calS_{x_\star} \times \calS_{x'_\star}\\ P_{k}(y,y') - \eta &\text{ when there exists $i \in [s]$ such that $(y,y') = (y_i, \bar{y}_i')$} \\ P_{k}(y,y') &\text{ otherwise.}\\ \end{cases} \end{equation*} Observe that $P'_0$ and $P'_1$ are both irreducible and $\kappa$-lumpable stochastic matrices. Similar to the edge-link case, we inspect the curve $\gamma^{(e)}_{P_0', P_1'} \colon \bbR \to \calW(\calY, \calE')$, and we denote $P'_t = \gamma^{(e)}_{P_0', P_1'}(t)$ the point on the curve at time parameter $t$. Using a similar analyticity argument as for the edge-link case, we show that the two curves $\gamma^{(e)}_{P_0', P_1'}$ and $\gamma^{(e)}_{P_0, P_1}$ can be made arbitrarily close point-wise, and in particular for $t = t_\star$. That is, there exists $\bar{\eta}_{\delta}$ such that for $\eta < \bar{\eta}_{ \delta}$, it holds that $\nrm{P'_{t_\star} - P_{t_\star}} \leq \delta / 2$. An application of the triangle inequality and \citet[Corollary~3]{nagaoka2005exponential} conclude the claim. \paragraph{Chaining and conclusion.} As a conclusion of the above, if $\calE$ and $\calE'$ only differ by an edge-link or a block-link, the claim holds. The general case can be obtained by inductively applying the above result to the chain of consecutive edge sets $\{\calE_\ell\}_{\ell = 0 \dots, L}$ with $\calE_0 = \calE, \calE_L = \calE', $ obtained from Lemma~\ref{lemma:chaining-edge-sets}. \end{proof} \begin{remark} Any chain constructed between two e-families can only be linked by non-merging blocks as adding a merging block immediately disrupts the property of being an e-family. Note that adding either an edge-link or a (non-merging) block-link can still disrupt the quality of being an e-family. The length of a chain between two e-families $\calW_{\kappa}(\calY, \calE)$ and $\calE_{\kappa}(\calY, \calE')$ simplifies to \begin{equation*} L = \abs{\kappa_2(\calE') \setminus \kappa_2(\calE)} + \sum_{(x,x') \in \kappa_{2}(\calE)} \abs{ (\calE' \setminus \calE) \cap (\calS_{x} \times \calS_{x'}) }. \end{equation*} \end{remark} \begin{definition}[Maximal e-families and minimal non e-families] We consider the partial order on non-vacuous lumpable and irreducible families induced from their edge sets ordered by inclusion. We call $\calW_\kappa(\calY, \calE)$ a maximal e-family, if $\calW_\kappa(\calY, \calE)$ forms an e-family, and any non-vacuous lumpable family $\calW_{\kappa}(\calY, \calE')$ such that $\calE \subsetneq \calE'$ does not form an e-family. Similarly, we call $\calW_\kappa(\calY, \calE')$ a minimal non e-family, if $\calW_\kappa(\calY, \calE')$ forms an e-family, and any non-vacuous lumpable family $\calW_{\kappa}(\calY, \calE)$ such that $\calE \subsetneq \calE'$ forms an e-family. \end{definition} \begin{remark}[Well-definedness] The family $\calW_\kappa(\calY, \calY^2)$ defined over the complete graph being lumpable for any $\kappa$, irreducible, and not forming an e-family (in non-trivial settings) it always holds that for any $\calE \subsetneq \calY^2$ such that $\calW_{\kappa}(\calY, \calE)$ forms an e-family, that there exists $\calE' \subset \calY^2$ with $\calE \subset \calE'$ such that $\calW_{\kappa}(\calY, \calE')$ does not form an e-family. Note however that for any $\calE' \subset \calE$ such that $\calW_\kappa(\calY, \calE')$ is non-vacuous and does not form an e-family, it is not always possible to extract $\calE \subset \calE'$ such that $\calW_{\kappa}(\calY, \calE)$ forms a non-vacuous e-family. The example below demonstrates this fact. \begin{equation*} \calW_{\kappa}(\calY, \calE) \sim \left(\begin{tblr}{cc|[dashed]cc} \zero & \zero & \zero & \plus \\ \zero & \zero & \plus & \zero \\\hline[dashed] \zero & \plus & \zero & \plus \\ \plus & \plus & \zero & \plus \\ \end{tblr}\right) \end{equation*} Observe that if we remove any subset of edges from the family above, it would become either reducible, or non-lumpable. The minimality statement thus only holds vacuously in this case. \end{remark} It is instructive to observe that adding or removing diagonal blocks from the diagonal does not alter the e-family property. \begin{proposition}[Stability through diagonal modification] \label{proposition:stability-diagonal-modification} Let $\calE, \calE'$ where $\calE \subset \calE' \subset \calY^2$ be such that $\calW_{\kappa}(\calY, \calE') \neq \emptyset$, and suppose additionally that $$\calE' \setminus \calE \subset \set{(y,y) \colon y \in \calY, \kappa(y) \not \in \calD = \kappa_2(\calE)}.$$ Then it holds that $\calW_\kappa(\calY, \calE')$ forms an e-family if and only if $\calW_\kappa(\calY, \calE)$ forms an e-family. \end{proposition} \begin{proof} Let us first assume that $\calE'$ and $\calE$ differ on a single diagonal block $(x_0, x_0) \in \calD'$ and suppose that $\calW_\kappa(\calY, \calE)$ forms an e-family. We let $P_0', P_1'$ be an arbitrary pair in $\calW_\kappa(\calY, \calE')$ and for $t \in \bbR_+$, we define $\widetilde{P'_t} = {P'_0}^{\hadamard (1 - t)} \hadamard {P'_1}^{\hadamard t}$ to be their log-affine combination. For $i \in \{0,1\}$, we additionally define $P_i \in \calW_{\kappa}(\calY, \calE)$ be such that \begin{equation*} P_i(y,y') = \begin{cases} P'_i(y,y') &\text{ when } \kappa(y) \neq x_0 \\ 0 &\text{ when } (\kappa(y), \kappa(y')) = (x_0, x_0) \\ \frac{P'_i(y,y')}{\sum_{x' \neq x_0} P'^{\flat}_i(x_0, x') } &\text{ otherwise}. \end{cases} \end{equation*} Since $\calW_\kappa(\calY, \calE)$ forms an e-family, it must be that $\stoch(\widetilde{P}_t) \in \calW_\kappa(\calY, \calE)$ where $\widetilde{P}_t= P_0^{\hadamard (1-t)} \hadamard P_1^{\hadamard t}$. Thus, from Corollary~\ref{corollary:geometric-intersection-interpretation}, there exists $v \in \bbR_+$ such that $\diag(v) \widetilde{P}_t \diag(v)^{-1} \in \calF_{\kappa}(\calY, \calE)$. Observe now that the same vector $v$ also satisfies $\diag(v) \widetilde{P}'_t \diag(v)^{-1} \in \calF_{\kappa}(\calY, \calE')$, thus $\stoch(\widetilde{P}'_t) \in \calW_\kappa(\calY, \calE')$, which forms an e-family. The case where $\calE$ and $\calE'$ differ by more than one diagonal block can be retrieved using chaining (refer to Lemma~\ref{lemma:chaining-edge-sets}). The converse statement follows more generally by monotonicity (refer to Theorem~\ref{theorem:monotonicity}). \end{proof} \section{Algorithmics} \label{section:algorithmics} The classification problem involves determining whether the family $\calW_{\kappa}(\calY, \calE)$ forms an exponential family, given a strongly connected graph $(\calY, \calE)$ and a lumping map $\kappa$. Specifically, we will provide a worst-case time complexity analysis of the criteria developed in Section~\ref{section:sufficient-condition} and Section~\ref{section:necessary-conditions}. \subsection{Deterministic classification} \label{section:deterministic-classification} Complexity will be typically expressed as a function of the number of vertices or edges in the graph $(\calY, \calE)$ and the lumping map $\kappa$. The space $\calY$ is represented by the integers $\set{1, \dots, \abs{\calY}}$. As is traditional in the literature, we will use the landau notation $\bigO$. We assume that the graph $(\calY, \calE)$ is represented in the machine using adjacency lists leading to a total structure of size $\bigO(\abs{\calY} + \abs{\calE})$. Time complexity is measured in terms of elementary field operations (addition, multiplication) and memory accesses. Storing the graph already requires $\bigO(\abs{\calY} + \abs{\calE})$ operations thus we will take this quantity as our lower bound and be mostly concerned with complexity which exceeds this value. Recall that the lumping map can have two representations---either partitional or functional. For the partitional representation, we store an array of size $\abs{\calX}$ corresponding to each elements of the lumped space. In each entry $x \in \calX$ of this array we store a list of elements of $\calY$ lumping into $x$. For the functional representation, we store an array of size $\abs{\calY}$ where at each entry $y \in \calY$ we store $\kappa(y)$. It is not hard to see that one representation can be constructed from the other in $\bigO(\abs{\calY})$. As a result, both representations can be interchangeably considered, and we will henceforth assume that we can both compute $\kappa(y)$ in $\bigO(1)$ and loop over elements of $\calS_x$ in $\abs{\calS_x}$. \begin{proposition}[Complexity of basic procedures] We rely on the subroutines below. \begin{enumerate}[$(i)$] \item Determine whether $(\calY, \calE)$ is strongly connected: $\bigO(\abs{\calY} + \abs{\calE})$. \item Determine whether $P \in \calW(\calY, \calE)$ is $\kappa$-lumpable: $\bigO(\abs{\calX}\abs{\calY} + \abs{\calE})$. \item Determine whether $\calW_{\kappa}(\calY, \calE)$ is non-vacuous: $\bigO(\abs{\calX}\abs{\calY} + \abs{\calE})$. \item Construct the lumped graph $(\calX, \calD)$ (as adjacency lists) in $\bigO(\abs{\calX}^2 + \abs{\calE})$. \item List merging blocks of $(\calY, \calE)$ with respect to $\kappa$: $\bigO(\abs{\calX}\abs{\calY} + \abs{\calE})$. \end{enumerate} \begin{proof} Simple algorithms yields $(ii), (iii), (iv), (v)$, while for $(i)$, listing strongly connected components can be achieved by running Tarjan's strongly connected components algorithm \citep{tarjan1972depth}, Kosaraju-Sharir's algorithm \citep{sharir1981strong}, or the path-based strong component algorithm \citep{dijkstra1976discipline} which all run in $\bigO(\abs{\calY} + \abs{\calE})$. \end{proof} \end{proposition} \begin{proposition}[Complexity of no multi-row merging block criterion (Corollary~\ref{corollary:no-multi-row-merging-block-is-sufficient})] $\bigO(\abs{\calX} \abs{\calY} + \abs{\calE})$. \end{proposition} \begin{proposition}[Complexity of dimensional criterion (Theorem~\ref{theorem:dimensional-criterion})] $\bigO(\abs{\calE}^{\omega})$ with $\omega \leq 2.371552$. \end{proposition} \begin{proof} It should be clear that the bottleneck operation is the computation of the rank of the system of matrices. Typically, computing the rank of a matrix is done by Gaussian elimination. For a system of $m$ vectors of dimension $n$, this approach can be theoretically implemented in $\bigO(m n ^{\omega - 1})$ \citep{ibarra1982generalization} where $2 \leq \omega \leq 2.371552$ is the matrix multiplication exponent \citep{williams2024new}. In our case, we have at most $\abs{\calE} + \abs{\calY}$ vectors, each of dimension $\abs{\calE}$. As a result, we obtain a worst-case time complexity of $\bigO((\abs{\calE} + \abs{\calY}) \abs{\calE}^{\omega - 1})$. \end{proof} \begin{remark} In practice---that is for most implementations---the complexity is of order $\bigO(\abs{\calE}^3)$. Note that with a parallel algorithm, it is possible to deterministically compute this rank in $\bigO(\log^2\abs{\calY})$ time \citep{mulmuley1986fast} using a polynomial number of processors. Since the rank calculation is the bottleneck, distributing this task would substantially improve the efficiency of our algorithm. \end{remark} \begin{proposition} There exists a $\bigO(\abs{\calY}^\omega)$ time verifiable witness that can be used to conclude that a lumpable family is not an e-family. \end{proposition} \begin{proof} Suppose that $\calW_{\kappa}(\calY , \calE)$ does not form an e-family. Then there exists a $P_0, P_1 \in \calW_{\kappa}(\calY , \calE)$ and $t \in \bbR$ such that $\gamma_{P_0, P_1}^{(e)}(t)$ is not $\kappa$-lumpable. The triplet $(P_0, P_1, t)$ is a witness and it remains to argue that it can be verified in polynomial time. Since constructing a point at parameter $t$ on the e-geodesic amounts to computing the Perron-Frobenius eigenpair of a Hadamard product of two matrices of size $\abs{\calY} \times \abs{\calY}$, it follows that $\gamma_{P_0, P_1}^{(e)}(t)$ can be computed in $\bigO(\abs{\calY}^\omega)$, and it can be verified that it is not $\kappa$-lumpable in $\bigO(\abs{\calX} \abs{\calY} + \abs{\calE})$. \end{proof} \bibliography{bibliography} \bibliographystyle{abbrvnat} \end{document} input
2412.08415v2
http://arxiv.org/abs/2412.08415v2
The two-boost problem and Lagrangian Rabinowitz Floer homology
\documentclass[a4paper]{amsart} \usepackage{hyperref, enumitem} \usepackage[dvipsnames]{xcolor} \usepackage{amsfonts,amsmath, amssymb,mathrsfs,mathtools,amsthm} \usepackage[doi=false, url=false, eprint=false, maxbibnames=10, firstinits=true]{biblatex} \addbibresource{bibliography.bib} \usepackage{todonotes} \usepackage[margin=0pt]{caption} \newtheorem{thm}{Theorem}[section] \newtheorem{prop}[thm]{Proposition}\newtheorem{define}[thm]{Definition}\newtheorem{lem}[thm]{Lemma}\newtheorem{cor}[thm]{Corollary}\newtheorem{rem}[thm]{Remark}\newtheorem{ex}[thm]{Example}\numberwithin{equation}{section} \newcommand{\J}{\color{MidnightBlue}} \newcommand{\A}{\mathcal{A}} \newcommand{\N}{\mathbb{N}} \newcommand{\Nn}{\mathbb{N^\ast}} \newcommand{\Q}{\mathbb{Q}} \newcommand{\Z}{\mathbb{Z}} \newcommand{\R}{\mathbb{R}} \newcommand{\C}{\mathbb{C}} \newcommand{\B}{\mathcal{B}} \newcommand{\F}{\mathscr{F}} \newcommand{\scinf}{\mathrm{sc}^\infty} \newcommand{\Crit}{\operatorname{Crit}} \newcommand{\dom}{\mathrm{dom}} \newcommand{\supp}{\operatorname{supp}} \newcommand{\Exp}{\operatorname{Exp}} \newcommand{\Id}{\operatorname{Id}} \newcommand{\reg}{\operatorname{reg}} \usepackage{verbatim} \parindent=0pt \parskip=4pt \newcommand{\kai}{\color{red}} \newcommand{\ol}{\overline} \newcommand{\wt}{\widetilde} \newcommand{\wh}{\widehat} \newcommand{\om}{\omega} \newcommand{\p}{\partial} \newcommand{\one} {{{\mathchoice \mathrm{ 1\mskip-4mu l} \mathrm{ 1\mskip-4mu l} \mathrm{ 1\mskip-4.5mu l} \mathrm{ 1\mskip-5mu l}}}} \newcommand{\LRFH}{{\rm LRFH}} \begin{document} \title{The two-boost problem and Lagrangian Rabinowitz Floer homology} \author{Kai Cieliebak} \address{Department of Mathematics, University of Augsburg} \thanks{Kai Cieliebak is funded by the Deutsche Forschungsgemeinschaft (DFG, German Research Foundation) – 541525489.} \author{Urs Frauenfelder} \address{Department of Mathematics, University of Augsburg} \thanks{ Urs Frauenfelder is funded by the Deutsche Forschungsgemeinschaft (DFG, German Research Foundation) – 541525489.} \author{Eva Miranda} \address{Laboratory of Geometry and Dynamical Systems, Department of Mathematics, Universitat Politècnica de Catalunya-IMTech \& CRM} \thanks{Eva Miranda is funded by the Catalan Institution for Research and Advanced Studies via an ICREA Academia Prize 2021 and by the Alexander Von Humboldt Foundation via a Friedrich Wilhelm Bessel Research Award. She is also supported by the Spanish State Research Agency, through the Severo Ochoa and Mar\'{\i}a de Maeztu Program for Centers and Units of Excellence in R\&D (project CEX2020-001084-M), by the AGAUR project 2021 SGR 00603 and by the project “Computational, dynamical and geometrical complexity in fluid dynamics”, Ayudas Fundación BBVA a Proyectos de Investigación Científica 2021. Eva Miranda and Jagna Wi\'sniewska are partially supported by the Spanish State Research Agency grants reference PID2019-103849GB-I00 of AEI / 10.13039/501100011033 and PID2023-146936NB-I00 funded by MICIU/AEI/ 10.13039/501100011033 and, by ERDF/EU. } \author{Jagna Wi\'sniewska} \address{Laboratory of Geometry and Dynamical Systems, Department of Mathematics, Universitat Politècnica de Catalunya}\thanks{Jagna Wi\'sniewska's postdoctoral contract is financed under the project “Computational, dynamical and geometrical complexity in fluid dynamics”, Ayudas Fundación BBVA a Proyectos de Investigación Científica 2021.} \maketitle \begin{abstract} \noindent The two-boost problem in space mission design asks whether two points of phase space can be connected with the help of two boosts of given energy. We provide a positive answer for a class of systems related to the restricted three-body problem by defining and computing its Lagrangian Rabinowitz Floer homology. The main technical work goes into dealing with the noncompactness of the corresponding energy hypersurfaces. \end{abstract} \section{Introduction} The {\em two-boost problem} goes back to the classical work of W.\,Hohmann on the attainability of heavenly bodies \cite{Hohmann}. Although this work was written more than four decades before the first human being stepped on the moon, the Hohmann transfer is still one of the crucial ingredients in space mission design, see e.g.~\cite{Vallado}. The Hohmann transfer is a transfer between two circular orbits in the Kepler problem with the help of a Kepler ellipse which is tangent to the two circles. It requires two tangential boosts, one to transfer from the first circle to the ellipse and a second one to transfer from the ellipse to the second circle. Given two points in the plane different from the origin, there always exists a conic section through the two points with focus in the origin. This means that, for the Kepler problem, two points in phase space can always be connected with the help of two boosts. The motivating question for this paper is whether this continues to hold for more general systems. The general setup for the two-boost problem is as follows: Consider the cotangent bundle $T^*Q$ of a manifold $Q$ with its canonical exact symplectic form $\om=d\lambda$, $\lambda=p\,dq$, and a Hamiltonian $H:T^*Q\to\R$. Given two points $q_0,q_1\in Q$ and an energy value $c$, the two-boost problem asks for the existence of a Hamiltonian orbit of energy $c$ connecting the cotangent fibres $T_{q_0}^*Q$ and $T_{q_1}^*Q$. Such orbits arise as critical points of the {\em Rabinowitz action functional} \begin{align*} \A^{H-c}_{q_0,q_1} & : \mathscr{H}_{q_0,q_1}\times \R \to \R,\\ \A^{H-c}_{q_0,q_1}(v,\eta) & := \int_0^1 \lambda(\partial_t v)dt - \eta \int_0^1 (H-c)(v(t))dt \end{align*} associated to the path space $$ \mathscr{H}_{q_0,q_1}:= \left\lbrace v \in W^{1,2}([0,1], T^*Q)\ \big|\ v(i)\in T^*_{q_i}Q\quad \textrm{for}\quad i=0,1\right\rbrace. $$ Thus the two-boost problem has a positive answer whenever the corresponding {\em Lagrangian Rabinowitz Floer homology} $\LRFH_*(\A^{H_0-h}_{q_0,q_1})$ is well-defined and nontrivial. As observed in~\cite{CieliebakFrauenfelder2009}, the well-definedness of (Lagrangian) Rabinowitz Floer homology requires some hypothesis on the Hamiltonian $H$. A suitable class is formed by {\em magnetic Hamiltonians} $$ H(q,p) = \frac12|p-A(q)|^2 - V(q) $$ for a magnetic potential $A\in\Omega^1(Q)$, a potential\footnote{ As is customary in celestial mechanics, our $V$ is {\em minus} the physical potential.} $V:Q\to\R$, and a Riemannian metric on $Q$. Under the assumption that the underlying manifold $Q$ is {\em compact}, the Rabinowitz Floer homology of such Hamiltonians (with possibly noncompact magnetic field) has been studied in~\cite{CieliebakFrauenfelderPaternain2010} for the periodic case and in~\cite{Merry2014} for the Lagrangian case. An important dynamical quantity associated to such a Hamiltonian is its {\em Ma\~n\'e critical value}. By a Theorem of W.\,Merry~\cite{Merry2014}, for $c$ above the Ma\~n\'e critical value the Lagrangian Rabinowitz Floer homology $\LRFH_*(\A^{H-c}_{q_0,q_1})$ is well-defined and nontrivial, hence the two-boost problem is solvable. Returning to celestial mechanics, let us consider the {\em planar circular restricted 3-body problem}. Here two heavy bodies (the primaries) move under their mutual gravitational attraction on circles around their center of mass in a plane, and the third body (of negligible mass) moves in the same plane under the gravitational attraction by the primaries. In rotating coordinates, this system is described by the autonomous Hamiltonian \begin{align*} H(q_1,q_2,p_1,p_2) &=\frac{1}{2}(p_1^2+p_2^2)+ p_1q_2-p_2q_1 - V(q_1,q_2), \end{align*} where $V$ is the sum of the (negative) Coulomb potentials of the two primaries (see e.g.~\cite{FrauenfeldervanKoert2018}). Writing it as \begin{align*} H(q_1,q_2,p_1,p_2) &= \frac{1}{2}(|p_1+q_2|^2+|p_2-q_1|^2)-V_{\rm eff}(q_1,q_2) \end{align*} with the effective potential $V_{\rm eff}(q_1,q_2)=V(q_1,q_2)+\frac12(q_1^2+q_2^2)$, we see that this is a magnetic Hamiltonian, with magnetic field generating the Coriolis force. In contrast to the previous paragraph, the configuration space $\R^2\setminus\{q_E,q_M\}$ is now noncompact due to possible collisions with the primaries at positions $q_E,q_M$ and possible escapes to infinity. Our goal is to define and compute Lagrangian Rabinowitz Floer homology in this situation. Since the noncompactness due to collisions can be removed by Moser or Levi-Civita regularization (see~\cite{FrauenfeldervanKoert2018}), we will focus our attention on the noncompactness at infinity in $\R^2$. For this, we consider the quadratic Hamiltonian \begin{equation}\label{DefH0} \begin{aligned} H_0(q_1,q_2,p_1,p_2) & :=\frac{1}{2}(p_1^2+p_2^2)+ p_1q_2-p_2q_1 = \frac{p_r^2}{2}+\frac{p_\theta^2}{2r^2}+p_\theta, \end{aligned} \end{equation} where $(r,\theta)$ are polar coordinates on the plane and $(p_r,p_\theta)$ their conjugate momenta. We introduce the following set of smooth functions constant outside a compact set: \begin{equation}\label{DefHset} \mathcal{H}:=\left\lbrace h\in C^\infty(T^*\R^2)\ \Big|\ dh\in C_c^\infty(T^*\R^2),\quad h>0,\quad h-dh(p\partial_p)> 0\right\rbrace. \end{equation} For $q_0,q_1\in \R^2$ we define the Rabinowitz action functional $\A^{H_0-h}_{q_0,q_1} : \mathscr{H}_{q_0,q_1}\times \R \to \R$ as above, with $Q=\R^2$ and $H-c$ replaced by $H_0-h$. Our first result is \begin{thm}\label{thm:LRFH} Fix $q_0,q_1\in \R^2$ and let $H_0$ be as in \eqref{DefH0}. For every $h \in \mathcal{H}$ the Lagrangian Rabinowitz Floer homology of $\A^{H_0-h}_{q_0,q_1}$ is well defined. Moreover, for any two $h_0,h_1\in \mathcal{H}$ the Lagrangian Rabinowitz Floer homology of $\A^{H_0-h_0}_{q_0,q_1}$ and $\A^{H_0-h_1}_{q_0,q_1}$ are isomorphic. \end{thm} We denote the Lagrangian Rabinowitz Floer homology of $\A^{H_0-h}_{q_0,q_1}$ by $\LRFH_*(\A^{H_0-h}_{q_0,q_1})$. Here we use coefficients in $\Z_2=\Z/2\Z$ and a half-integer grading by Maslov indices (see~\S\ref{ss:grading}). Moreover, it comes with a canonical action filtration. Our next result computes the positive action part in the case $q_0\neq q_1$. \begin{thm}\label{thm:posLRFH} For the Hamiltonian $H_0$ in \eqref{DefH0}, $h \in \mathcal{H}$, and any $q_0,q_1\in \R^2$ with $q_0\neq q_1$, the positive Lagrangian Rabinowitz homology of $\A^{H_0-h}_{q_0,q_1}$ is well defined and equal to $$ \LRFH_*^+\left(\A^{H_0-h}_{q_0,q_1}\right)= \begin{cases} \mathbb{Z}_2 & \textrm{for}\quad *=1/2,\\ 0 & \textrm{otherwise}. \end{cases} $$ \end{thm} This shows that the two-boost problem at energy $0$ is solvable for $H_0-h$ with $h\in\mathcal{H}$. Note that $h(q,p)=c+V(q)$ belongs to $\mathcal{H}$ for each constant $c>0$ and compactly supported potential $V\geq 0$, so the two-boost problem is solvable for $H_0-V$ with such $V$ at each energy $c>0$. The potential in the planar circular restricted 3-body problem (after regularisation) satisfies $V\geq 0$, but it is not compactly supported. The following result allows us to deal with certain non-compactly supported potentials. \begin{prop}\label{prop:CritH=CritH1} Let $H_0$ be the Hamiltonian in \eqref{DefH0}. Fix $q_0,q_1\in \R^2$ with $q_0\neq q_1$ and a positive constant $c>0$. Let $V: \R^2 \to \R$ be a nonnegative potential for which there exist $r_0>0$ and $(\alpha, a) \in \{2\}\times \left(0 ,\frac{c^2}{4}\right)\cup (2, +\infty) \times \R_+$, such that for $r>r_0$ we have $$ V(r,\theta)\leq \frac{a}{r^\alpha}\qquad \textrm{and} \qquad \partial_r V (r, \theta) \geq -\frac{a}{r^{\alpha+1}}. $$ Then there exists a function $V_0\in \mathcal{H}$ such that $\Crit \A^{H_0-V-c}_{q_0,q_1} = \Crit \A^{H_0-V_0}_{q_0,q_1}$. \end{prop} In view of Theorem~\ref{thm:posLRFH}, the two-boost problem is therefore solvable for $H_0-V$ at energy $c>0$ for any $(V,c)$ as in Proposition~\ref{prop:CritH=CritH1}. In particular, this holds for any $c>0$ and $V\geq 0$ which decays at infinity as $r^{-\alpha}$ with $\alpha>2$. Note that this does {\em not} cover the planar circular restricted 3-body problem, in which the potential decays as $r^{-1}$. We defer the extension to this case to future work. \begin{rem}\label{rem:contact} Note that for $h \in \mathcal{H}$ the Liouville vector field $p\partial_p$ is transverse to $(H_0-h)^{-1}(0)$, so the zero level set of $H_0-h$ is of restricted contact type. Thus the solutions of the two-boost problem resulting from Theorem~\ref{thm:posLRFH} and Proposition~\ref{prop:CritH=CritH1} can also be interpreted as solutions of Arnold's chord conjecture (existence of a Reeb chord connecting two Legendrians) for certain noncompact contact manifolds. \end{rem} \begin{rem} Work by R.\,Nicholls~\cite{Nicholls2021} provides some evidence that, if defined, the Ma\~n\'e critical value for the restricted three-body problem should be zero. It would be interesting to make this rigorous by developing a notion of Ma\~n\'e critical value for magnetic Hamiltonians over noncompact base manifolds. \end{rem} \section{Lagrangian Rabinowitz Floer homology} Lagrangian Rabinowitz Floer homology was first defined by Merry in \cite{Merry2014} for virtually exact Lagrangian submanifolds and compact hypersurfaces of virtual contact type in symplectically aspherical symplectic manifolds. In his setting, the pull-back of the symplectic form to the universal cover of the symplectic manifold had to be exact and the pre-image of the chosen hypersurface in the universal cover had to be of contact type. The symplectic manifold and the Lagrangian submanifold could be non-compact, but the chosen hypersurface of virtual contact type had to be compact. Our aim is to use Lagrangian Rabinowitz Floer homology to analyse the Planar Circular Restricted Three Body Problem. Therefore, we are interested in the setting of a cotangent bundle $T^*Q$ with its standard symplectic form and a pair of fibres $T_{q_0}^*Q, T_{q_1}^*Q$ as the Lagrangians. In this section we will explain the definition of the Lagrangian Rabinowitz Floer homology in this setting. It is a far easier setting than the one in \cite{Merry2014}, as the cotangent bundle is an exact manifold and the fibres are exact Lagrangians. Moreover, the level sets above the 4th and 5th Lagrange point of the Hamiltonian corresponding to the Planar Circular Restricted Three Body Problem are all of exact contact type (see Remark~\ref{rem:contact}). The only challenge of this setting is that all the energy level sets in the Restricted Three Body Problem are non-compact. Rabinowitz Floer homology (for periodic orbits) of noncompact energy level sets of ``tentacular Hamiltonians'' on $\R^{2n}$ has been defined in~\cite{Pasquotto2017}. However, the Hamiltonian for the Planar Circular Restricted Three Body Problem does not belong to this class, so new arguments are needed for this setting. \subsection{The Lagrangian Rabinowitz action functional} Throughout this section, $Q$ denotes a smooth oriented $n$-dimensional manifold, and $T^*Q$ its cotangent bundle equipped with the exact symplectic form $\omega=d\lambda$ for the canonical $1$-form $\lambda:=p\,dq$. Fix $q_0,q_1 \in Q$ and define the space of paths $$ \mathscr{H}_{q_0,q_1}:= \left\lbrace v \in W^{1,2}([0,1], T^*Q)\ \big|\ v(i)\in T^*_{q_i}Q\quad \textrm{for}\quad i=0,1\right\rbrace. $$ Consider a Hamiltonian $H: T^*Q \to \R$ with regular level set $H^{-1}(0)$. The {\em Lagrangian Rabinowitz action functional} $\A^H_{q_0,q_1}:\mathscr{H}_{q_0,q_1}\times \R\to\R$ associated to $H$ is defined as $$ \A^H_{q_0,q_1}(v,\eta):= \int_0^1 \lambda(\partial_t v)dt - \eta \int_0^1 H(v(t))dt. $$ \begin{rem} We have a natural bijection $\mathscr{H}_{q_0,q_1}\ni v \mapsto \ol v\in \mathscr{H}_{q_1,q_0}$ with $\ol v(t):=v(1-t)$. Then $$ \A^H_{q_0,q_1}(v, \eta) = -\A^H_{q_1,q_0}(\ol v, -\eta)\qquad\forall\ (v, \eta) \in \mathscr{H}_{q_0,q_1}\times\R. $$ \end{rem} The derivative of $\A^H_{q_0,q_1}$ in direction $(\xi, \sigma) \in T_v \mathscr{H}_{q_0,q_1}\times T_\eta \R$ equals \begin{equation}\label{eq:dAH} d\A^H_{q_0,q_1}(v,\eta)[\xi,\sigma] = \int_0^1 \omega (\xi,\partial_tv -\eta X_H) - \sigma\int_0^1 H(v)dt. \end{equation} Here $X_H$ is the Hamiltonian vector field defined by $dH=-i_{X_H}\om$. Consequently, $(v,\eta) \in \Crit (\A^H_{q_0,q_1})$ if and only if it satisfies $$ \partial_t v= \eta X_H(v)\quad \textrm{and}\quad v(t)\in H^{-1}(0)\quad \forall\ t\in [0,1]. $$ Thus we can have three types of critical points: \begin{itemize} \item $\eta>0$ and $\wt v(t):=v(t/\eta)$ is a Hamiltonian chord (i.e., an integral curve of $X_H$) on $H^{-1}(0)$ from $T^*_{q_0}Q$ to $T_{q_1}^*Q$; \item $\eta<0$ and $\wt v(t):=v(t/\eta)$ is a Hamiltonian chord on $H^{-1}(0)$ from $T^*_{q_1}Q$ to $T_{q_0}^*Q$; \item $\eta=0$ and $v$ is a constant path in $T^*_{q_0}Q\cap T_{q_1}^*Q\cap H^{-1}(0)$ (which can only occur if $q_0=q_1$). \end{itemize} In particular, if $T^*_{q_0}Q \cap H^{-1}(0)=\emptyset$ or $T_{q_1}^*Q \cap H^{-1}(0)=\emptyset$, then $ \Crit (\A^H_{q_0,q_1})=\emptyset$. Therefore, from now on we will assume that \begin{equation}\label{T*QcapHnonempty} T^*_{q_0}Q \cap H^{-1}(0)\neq\emptyset,\quad T^*_{q_1}Q \cap H^{-1}(0)\neq\emptyset,\quad\textrm{and } T^*_{q_0}Q\pitchfork H^{-1}(0) \text{ if }q_0=q_1. \end{equation} In order to construct Lagrangian Rabinowitz Floer homology we want the critical set of the action functional to be bounded in $L^\infty$. It is possible to define Lagrangian Rabinowitz Floer homology without this assumption, but the construction is much more challenging, so we postpone it for future projects. In fact, to construct Lagrangian Rabinowitz Floer homology we will need the boundedness of the critical set of the action functional to persist under compact perturbations of the Hamiltonian. We formalize this in the following definition. \begin{define} Consider a Hamiltonian $H: T^*Q \to \R$, such that $H^{-1}(0)$ is noncompact. Fix $q_0,q_1\in T^*Q$ satisfying \eqref{T*QcapHnonempty}. Let $K\subseteq T^*Q$ be a compact set and let $$ \mathcal{H}\subseteq \{ h\in C^\infty(T^*Q)\ |\ dh\in C_0^\infty(K)\}, $$ be an open neighbourhood of $0$ in $\{ h\in C^\infty(T^*Q)\ |\ dh\in C_0^\infty(K)\}$. We say that the associated Lagrangian Rabinowitz Floer functional $\A^H_{q_0,q_1}:\mathscr{H}_{q_0,q_1}\times \R\to \R$ has critical set \emph{continuously compact} in $(K,\mathcal{H})$ if $$ \forall\ h\in\mathcal{H}\quad \textrm{and}\quad\forall\ (v,\eta) \in \Crit \A^{H+h}_{q_0,q_1} \quad \textrm{we have}\quad v([0,1])\subseteq K. $$ \end{define} \subsection{Grading}\label{ss:grading} In~\cite{CieliebakFrauenfelder2009}, Rabinowitz Floer homology (for periodic orbits) is equipped with a shifted integer grading under the hypothesis that the ambient symplectic manifold has vanishing first Chern class. In this subsection we indicate how this carries over to the Lagrangian setting; see~\cite{Merry2014} for details. Consider an exact symplectic manifold $(V,\om=d\lambda)$ of dimension $2n$ and a Hamiltonian $H:V\to\R$ with regular level set $\Sigma=H^{-1}(0)$ such that $\xi=\ker(\lambda|_\Sigma)$ is a contact structure. Consider in addition two Lagrangian submanifolds $L_0,L_1\subset V$ transverse to $\Sigma$ with $\lambda|_{L_i}=0$. We assume that $L_0$ and $L_1$ are either disjoint or equal. The condition of vanishing first Chern class in~\cite{CieliebakFrauenfelder2009} gets replaced by vanishing of the relative first Chern class $c_1(V,L_0\cup L_1)$. The transversal Conley-Zehnder index in~\cite{CieliebakFrauenfelder2009} gets replaced by the {\em transverse Maslov index} defined a follows. Let $(v,\eta)$ be a critical point of the Rabinowitz action functional $\A^H$ on the space of paths from $L_0$ to $L_1$. Pick a symplectic trivialization $v^*\xi\cong [0,1]\times\R^{2n-2}$ sending $T_{v(i)}L_i\cap\xi_{v(i)}$ to $\R^{n-1}$ for $i=0,1$. In this trivialization, the image of $T_{v(i)}L_i\cap\xi_{v(i)}$ under the linearized flow of $\eta X_H$ gives a path $\Lambda:[0,1]\to\mathcal{L}_{n-1}$ in the Grassmannian $\mathcal{L}_{n-1}$ of Lagrangian subspaces of $(\R^{2n-2},\om_{\rm std})$ with $\Lambda(0)=\R^{n-1}$. Let $\mu^{\rm tr}(v,\eta)\in\frac12\Z$ be the Maslov index of the path $\Lambda$ defined by Robbin and Salamon in~\cite{Robbin1993}. By vanishing of $c_1(V,L_0\cup L_1)$ this is independent of the choices. In the case $\eta=0$ (which occurs only if $L_0=L_1$), $v$ is a critical point of an auxiliary Morse function $f$ on $L_0\cap\Sigma$. Its {\em signature index} is ${\rm ind}^\sigma_f(v,0) := -\frac12{\rm sign}\,{\rm Hess}_f(v)$, and we set it to zero if $\eta\neq 0$. According to~\cite[Appendix A]{CieliebakFrauenfelder2009}, the {\em Maslov index} $$ \mu(v,\eta) := \mu^{\rm tr}(v,\eta) + {\rm ind}^\sigma_f(v,\eta) \in\frac12\Z $$ defines a half-integer grading on the Rabinowitz Floer complex with respect to which the differential has degreee $-1$. This discussion applies to the setting in this section where $V=T^*Q$ and $L_i=T_{q_i}^*Q$ for an oriented $n$-dimensional manifold $Q$. \begin{ex}\label{ex:Maslov} Consider the free Hamiltonian $H_{\bullet}(q,p):=\frac{1}{2}|p|^2$ on $T^*\R^n$ and fix an energy $c>0$. Assume first that $q_0\neq q_1\in\R^n$. Then $\A^{H_{\bullet}-c}_{q_0,q_1}$ has exactly two critical points $(v_\pm,\eta_\pm)$ given by $$ v_\pm(t)=\Bigl((1-t)q_0+tq_1,\frac{q_1-q_0}{\eta_\pm}\Bigr),\qquad \eta_\pm=\pm\frac{\sqrt{2c}}{|q_1-q_0|}. $$ The linearized flow of $\eta_\pm X_{H_\bullet}$ along $v_\pm$ equals $\Phi_t=\begin{pmatrix}\one & t\eta_\pm\one \\ 0 & \one\end{pmatrix}$, so the corresponding path of Lagrangian subspaces $\Lambda:[0,1]\to\mathcal{L}_{n-1}$ is given by $\Lambda(t)=\Phi_t(\R^{n-1})={\rm graph}\,A(t)$ with $A(t)=t\eta_\pm\one$. Thus the (Localization) property in~\cite{Robbin1993} yields $$ \mu(v_\pm,\eta_\pm) = \mu^{\rm tr}(v_\pm,\eta_\pm) = \frac12{\rm sign}\,A(1) - \frac12{\rm sign}\,A(0) = \pm\frac{n-1}{2}. $$ In the case $q_0=q_1$, let $f$ be a Morse function on the $(n-1)$-sphere $T_{q_0}Q\cap H_\bullet^{-1}(c)$ with exaxctly two critical points, the maximum $v_+$ and the minimum $v_-$. Then $\A^{H_{\bullet}-c}_{q_0,q_0}$ has exactly two critical points $(v_\pm,0)$ of index $$ \mu(v_\pm,0) = {\rm ind}^\sigma_f(v_\pm,0) = \pm\frac{n-1}{2}. $$ For $q_0\neq q_1$ this immediately implies $$ \LRFH_*^\pm\left(H_\bullet^{-1}(c), T^*_{q_0}\R^n,T^*_{q_1}\R^n\right)= \begin{cases} \mathbb{Z}_2 & \textrm{for}\quad *=\pm\frac{n-1}{2},\\ 0 & \textrm{otherwise}. \end{cases} $$ Moreover, for $n\neq 3$ and any $q_0,q_1$ (equal or not) we obtain $$ \LRFH_*\left(H_\bullet^{-1}(c), T^*_{q_0}\R^n,T^*_{q_1}\R^n\right)= \begin{cases} \mathbb{Z}_2 & \textrm{for}\quad *=\frac{n-1}{2}\textrm{ and }*=-\frac{n-1}{2},\\ 0 & \textrm{otherwise}. \end{cases} $$ (This also holds for $n=2$, but an additional argument is needed to show that the two critical points of index $\pm1/2$ do not cancel in homology). \end{ex} \begin{rem}\label{rem:Maslov} The previous example generalizes to the free Hamiltonian $H_{\bullet}(q,p)=\frac{1}{2}|p|^2$ on the cotangent bundle of an $n$-dimensional Riemannian manifold $(Q,g)$. For $q_0\neq q_1$ and $(v,\eta)\in \Crit\A^{H_{\bullet}-c}_{q_0,q_1}$, the path $v$ projects onto a geodesic $\bar v$ from $q_0$ to $q_1$ and Proposition 6.38 in~\cite{RobbinSalamon1995} yields \begin{equation*} \mu(v,\eta) = {\rm ind}(\bar v) + {\rm sign}(\eta)\frac{n-1}{2}, \end{equation*} where ${\rm ind}(\bar v)$ is the Morse index of the geodesic $\bar v$ (i.e., the number of conjugate points along $\bar v$). \end{rem} \subsection{Floer trajectories}\label{sec:FloerTrajec} In this subsection we introduce the fundamental notion for constructing any Floer-type homology - the Floer trajectories. We start by equipping the space $\mathscr{H}_{q_0,q_1}$ with a metric. An \emph{almost complex structure} $J$ on a manifold $M$ is a bundle endomorphism $J: TM \to TM$ satisfying $J^2 = -\operatorname{Id}$. An almost complex structure $J$ on a symplectic manifold $(M, \omega)$ is called \emph{compatible} if $\omega(\ \cdot\ ,J \cdot\ )$ defines a Riemannian metric on $M$. Denote by $\mathcal{J}(M,\omega)$ the space of all compatible almost complex structures on $(M,\omega)$ with the $C^\infty$-topology. A linear algebra argument \cite[Prop. 13.1]{Silva2001} shows that $\mathcal{J}(M,\omega)$ is contractible. A smooth $2$-parameter family $\{J_{t,\eta}\}_{(t,\eta)\in[0,1]\times \R}$ of compatible almost complex structures on $(T^*Q, \omega)$ defines an $L^2$-metric on $\mathscr{H}_{q_0,q_1}\times \R$ by $$ \langle (\xi_1, \sigma_1), (\xi_2, \sigma_2)\rangle := \int_0^1 \omega(\xi_1(t),J_{t,\eta}(v(t))\xi_2(t) )+\sigma_1\sigma_2.$$ for $(\xi_i,\sigma_i)\in T_{(v,\eta)}(\mathscr{H}_{q_0,q_1}\times \R)$. The gradient of the Lagrangian Rabinowitz action functional $\A^H_{q_0,q_1}$ with respect to this metric equals $$ \nabla \A^H_{q_0,q_1} (v,\eta)= \left( \begin{array}{c} -J_{t,\eta}(v(t)) (\partial_tv -\eta X_H) \\ - \int H(v)dt. \end{array} \right) $$ Fix an open subset $\mathcal{V}\subseteq T^*Q$ and $\mathbb{J}\in \mathcal{J}(T^*Q,\omega)$. We denote by $\mathcal{J}(\mathcal{V}, \mathbb{J})$ the set of all smooth maps $$ [0,1]\times \mathbb{R}\to \mathcal{J}(T^*Q,\omega),\qquad (t,\eta)\mapsto J_{t,\eta} $$ satisfying \begin{align*} J_{t,\eta}(x) = \mathbb{J}(x)\text{ for } x\notin \mathcal{V} \quad\textrm{and}\quad \sup_{(t,\eta)\in [0,1]\times \mathbb{R}}\|J_{t,\eta}\|_{C^k}<+\infty \text{ for all } k\in \mathbb{N}. \end{align*} Fix Hamiltonians $H_\pm$ and $J_\pm\in\mathcal{J}(\mathcal{V},\mathbb{J})$. A {\em homotopy} from $(H_-,J_-)$ to $(H_+,J_+)$ is a smooth family $\Gamma = \{(H_s, J_s)\}_{s\in \R}$ of Hamiltonians and compatible almost complex structures $J_s\in \mathcal{J}(\mathcal{V},\mathbb{J})$ which agrees with $(H_-,J_-)$ for $s\leq s_-$ and with $(H_+,J_+)$ for $s\geq s_+$, with some $s_\pm\in\R$. A solution $u: \R \to \mathscr{H}_{q_0,q_1} \times \R$ to the gradient flow equation $\partial_s u = \nabla \A^{H_s}_{q_0,q_1}(u)$ is called a \emph{Floer trajectory}. In other words, a Floer trajectory $u=(v,\eta) \in W^{1,2}(\R\times[0,1], T^*Q)\times W^{1,2}(\R)$ is a solution to the equations \begin{equation}\label{FloerEq} \begin{aligned} \partial_s v(s,t) & = - J_{s, t,\eta}(v(s,t)) (\partial_sv(s,t)-\eta(s)X_{H_s}(v(s,t))),\\ \partial_s \eta (s) & = - \int_0^1 H_s\circ v(s,t)dt, \end{aligned} \end{equation} $$ v(s,0) \in T^*_{q_0}Q \quad \textrm{and} \quad v(s,1) \in T^*_{q_1}Q \quad \forall\ s\in \R. $$ For $(x_-, x_+) \in \Crit(\A^{H_-}_{q_0,q_1})\times \Crit \A^{H_+}_{q_0,q_1}$ we denote the space of Floer trajectories from $x_-$ to $x_+$ by $$ \F_\Gamma(x_-, x_+):= \left\lbrace\begin{array}{c|c} & \partial_s u = \nabla \A^{H_s}_{q_0,q_1}(u),\\ \hspace*{-0.2cm}\smash{\raisebox{.5\normalbaselineskip}{ $u: \R \to \mathscr{H}_{q_0,q_1} \times \R$}}& \lim_{s\to \pm \infty}u(s)=x_\pm. \end{array}\right\rbrace $$ In case the homotopy $\Gamma$ is constant in $s$, i.e. $H_s\equiv H$ and $J_{s,t,\eta}\equiv J \in \mathcal{J}(\mathcal{V},\mathbb{J})$ for some Hamiltonian $H$ and a compatible almost complex structure $J$, we denote $\F_{H,J}(x_-,x_+):=\F_\Gamma(x_-, x_+)$. Moreover, for every pair $(a,b)\in \R^2$ we denote \begin{equation}\label{DefM(a,b)} \mathcal{M}^\Gamma(a,b):=\left\lbrace\begin{array}{c|c} & (x_-,x_+)\in \Crit\A^{H_-}_{q_0,q_1}\times \Crit\A^{H_+}_{q_0,q_1},\\ \hspace*{-0.2cm}\smash{\raisebox{.5\normalbaselineskip}{ $u\in \F_\Gamma(x_-,x_+)$}}& \A^{H_-}_{q_0,q_1}(x_-)\geq a, \quad \A^{H_+}_{q_0,q_1}(x_+)\leq b. \end{array}\right\rbrace \end{equation} Analogously, we denote $\mathcal{M}^{H,J}(a,b):=\mathcal{M}^\Gamma(a,b)$ whenever the homotopy $\Gamma$ is constant in $s$ and equal to the pair $(H,J)$. If s homotopy $\Gamma$ is constant in $s$, then the action is increasing along Floer trajectories. However, for a nonconstant homotopy $\Gamma$ this need not be the case. To deal with this phenomenon, we introduce a condition that ensures that the action cannot decrease indefinitely along a Floer trajectory. We say a that homotopy $\Gamma:= \{(H_s, J_s)\}_{s\in \R}$ with $H_{\pm}:=\lim_{s\to \pm \infty}H_s$ satisfies the \emph{Novikov finiteness condition} if for all $a,b\in \R$ we have \begin{equation}\label{Novikov} \begin{aligned} \inf\left\lbrace\begin{array}{c|c} & y \in \Crit\A^{H_+}_{q_0,q_1}, \quad \exists\ x\in \Crit \A^{H_-}_{q_0,q_1},\\ \hspace*{-0.2cm}\smash{\raisebox{.5\normalbaselineskip}{ $\A^{H_+}_{q_0,q_1}(y)$}}& \A^{H_-}_{q_0,q_1}(x)\geq a, \quad \F_\Gamma(x,y)\neq \emptyset. \end{array}\right\rbrace & > -\infty,\\ \sup\left\lbrace\begin{array}{c|c} & x \in \Crit\A^{H_-}_{q_0,q_1}, \quad \exists\ y\in \Crit \A^{H_+}_{q_0,q_1},\\ \hspace*{-0.2cm}\smash{\raisebox{.5\normalbaselineskip}{ $\A^{H_-}_{q_0,q_1}(x)$}}& \A^{H_+}_{q_0,q_1}(x)\leq b, \quad \F_\Gamma(x,y)\neq \emptyset. \end{array}\right\rbrace &< +\infty. \end{aligned} \end{equation} \subsection{Defining Lagrangian Rabinowitz Floer homology} In this subsection we will recall the construction of the Lagrangian Rabinowitz Floer homology from \cite{Merry2014}, using standard Floer techniques introduced in Floer's seminal paper \cite{Floer1989} and the techniques typical for the setting of the Rabinowitz action functional from the first two author's paper \cite{CieliebakFrauenfelder2009}. More precisely, we will prove the following theorem. \begin{thm}\label{thm:DefLRFH} Consider a cotangent bundle $(T^*Q,\omega)$ with its standard symplectic form and a Hamiltonian $H:T^*Q\to \R$ with regular level set $H^{-1}(0)$. Fix a pair $q_0,q_1\in Q$ such that both sets $H^{-1}(0)\cap T^*_{q_0}Q$ and $H^{-1}(0)\cap T^*_{q_1}Q$ are compact and nonempty. Fix a compatible almost complex structure $\mathbb{J}\in \mathcal{J}(T^*Q,\omega)$. Assume that there exists a compact subset $K$, an open subset $\mathcal{V}$ satisfying $K\subseteq \mathcal{V}\subseteq T^*Q$, and an open neighbourhood $\mathcal{H}\subseteq \{h\in C^\infty(T^*Q)\ |\ dh\in C_0^\infty(K)\}$ of $0$ such that: \begin{enumerate} \item for all $h\in \mathcal{H}$ the Hamiltonian $H+h$ satisfies \eqref{T*QcapHnonempty}; \item the Lagrangian Rabinowitz Floer functional $\A^H_{q_0,q_1}$ has critical set continuously compact in $(K,\mathcal{H})$; \item for all $h_0,h_1 \in \mathcal{H}$ and $J_0, J_1 \in \mathcal{J}(\mathcal{V}, \mathbb{J})$ every homotopy $\Gamma=\{(H+h_s, J_s)\}_{s\in \R}$ from $(H+h_0,J_0)$ to $(H+h_1,J_1)$ satisfies the Novikov finiteness condition \eqref{Novikov} and for all $a, b \in \R$ the space of Floer trajectories $\mathcal{M}^\Gamma(a,b)$ is bounded in the $L^\infty$-norm. \end{enumerate} Then for every $h\in\mathcal{H}$ the Lagrangian Rabinowitz Floer homology $\LRFH_*(\A^{H+h}_{q_0,q_1})$ is well defined and isomorphic to $\LRFH_*(\A^{H}_{q_0,q_1})$. \end{thm} \begin{proof} We will define Lagrangian Rabinowitz Floer homology in the case $q_0\neq q_1$, and then briefly explain the case $q_0=q_1$. Therefore, we first assume $q_0\neq q_1$. The first step is to show that for $q_0\neq q_1$ and generic $h\in \mathcal{H}$ the Lagrangian Rabinowitz action functional $\A^{H+h}_{q_0,q_1}$ is Morse, i.e., its Hessian $\nabla^2\A^{H+h}_{q_0,q_1}(x)$ has trivial kernel for all $x \in \Crit \A^{H+h}_{q_0,q_1}$. Observe that for $q_0\neq q_1$ we we have $\Crit \A^{H+h}_{q_0,q_1} \cap \left( \mathscr{H}_{q_0,q_1}\times\{0\}\right)=\emptyset$. By a standard Sard-Smale argument \cite[Theorem A.51]{McDuff2012}, there exists a residual subset $\mathcal{H}^{\reg}\subseteq \mathcal{H}\subseteq C_0^\infty(K)$ such that for all $h \in \mathcal{H}^{\reg}$ we have \begin{equation*}D \phi^\eta_{H+h} (T^*_{q_0}Q) \pitchfork T^*_{q_1}Q\qquad\forall\ (v, \eta)\in \Crit\A^{H+h}_{q_0,q_1}, \end{equation*} where $\phi^t_{H+h}$ denotes the Hamiltonian flow of $X_{H+h}$. The condition is equivalent to the triviality of the kernel of the Hessian $\nabla^2 \A^{H+h}_{q_0,q_1}(x)$ for all $x \in \Crit \A^{H+h}_{q_0,q_1}$. A straightforward consequence of the Morse property of the Lagrangian Rabinowitz action functional and continuous compactness of the critical set of $\A^{H}_{q_0,q_1}$ is that the critical set of $\A^{H+h}_{q_0,q_1}$ is finite for each $h \in \mathcal{H}^{\reg}$. Fix $h\in \mathcal{H}^{\reg}$. Since $K \subseteq \mathcal{V}$, by continuous compactness of the critical set of $\A^{H}_{q_0,q_1}$ in $(K,\mathcal{H})$ for every $J\in \mathcal{J}(\mathcal{V}, \mathbb{J})$, we have \begin{equation}\label{nonEmpty} v(\R\times[0,1])\cap \mathcal{V}\neq \emptyset\quad \forall\ u=(v,\eta)\in \mathscr{F}_{H+h,J}(x,y)\quad\forall\ x,y \in \Crit\A^{H+h}_{q_0,q_1}. \end{equation} Therefore, we can apply the Sard-Smale argument in \cite[Thm. A.51]{McDuff2012} to conclude that there exists a residual set $\mathcal{J}^{\reg}_h\subseteq \mathcal{J}(T^*Q,\omega,\mathcal{V})$ such that for all $x,y\in \Crit\A^{H+h}_{q_0,q_1}$ and all $J \in \mathcal{J}^{\reg}_h$ the space of Floer trajectories $\mathscr{F}_{H+h,J}(x,y)$ is a transversely cut out smooth manifold. Let us now fix $h\in \mathcal{H}^{\reg}$ and $J\in \mathcal{J}^{\reg}_h$. For every $x,y\in \Crit\A^{H+h}_{q_0,q_1}$ there is a natural $\R$-action on $\F_{H+h,J}(x,y)$ given by $$ \F_{H+h,J}(x,y) \times \R \ni (u, s) \longmapsto u(s + \cdot) \in \F_{H+h,J}(x,y). $$ Denote the quotient space $\overline{\F}_{H+h,J}(x,y):=\F_{H+h,J}(x,y)/ \R$. By \cite[Thm. 2.23]{Merry2014}, for all $x,y\in \Crit\A^{H+h}_{q_0,q_1}$ with $x\neq y$ the quotient $\overline{\F}_{H+h,J}(x,y)$ is also a smooth manifold and its dimension is given by $$ \dim \overline{\F}_{H+h,J}(x,y)= \mu (y) - \mu (x) -1. $$ By condition 3 in Theorem~\ref{thm:DefLRFH}, for all $x,y\in \Crit\A^{H+h}_{q_0,q_1}$ the space of Floer trajectories $\mathscr{F}_{H+h,J}(x,y)$ is bounded in the $L^\infty$-norm. Now standard compactness arguments (see \cite[Prop. 3b]{Floer1989} or \cite[Thm. 9.1.7]{Audin2014}) imply that $\overline{\F}_{H+h,J}(x,y)$ is compact up to breaking in the sense of Floer. If $\mu(y)-\mu(x)=1$ this means that $\overline{\F}_{H+h,J}(x,y)$ is a finite set of points \cite[Thm. 9.2.1]{Audin2014}. Now we are ready to define Lagrangian Rabinowitz Floer homology. For $h\in \mathcal{H}^{\reg}$ and $k\in\Z$ let $CF_k(\A^{H+h}_{q_0,q_1})$ be the $\mathbb{Z}_2$-vector space of formal sums of the form $\sum_{x\in S}x$, where $S\subseteq \Crit\A^{H+h}_{q_0,q_1}$ is a (possibly infinite) set satisfying $\mu(x)=k$ for all $x\in S$ and the \emph{Novikov finiteness condition} $$ \# \{ x\in S\ |\ \A^{H+h}_{q_0,q_1}(x)>a\}<+\infty\qquad\forall\ a\in\mathbb{R}. $$ Now we fix an almost complex structure $J\in \mathcal{J}^{\reg}_h$ and turn $CF(\A^{H+h}_{q_0,q_1})$ into a chain complex. For $x,y\in \Crit\A^{H+h}_{q_0,q_1}$ with $\mu(y)-\mu(x)=1$ we set $$ n(x,y):=\# \overline{\F}_{H+h,J}(x,y) \operatorname{mod}2\in \mathbb{Z}_2. $$ The Floer boundary operator $\partial_k^J: CF_k(\A^{H+h}_{q_0,q_1})\to CF_{k-1}(\A^{H+h}_{q_0,q_1})$ is the linear map defined on generators by $$ \partial_k^J y := \sum_x n(x,y)x. $$ Compactness up to breaking implies that $\partial_k^J\circ\partial_{k+1}^J=0$, so we can define the {\em Lagrangian Rabinowitz Floer homology} $$ \LRFH_k(\A^{H+h}_{q_0,q_1};J) := \ker \partial_k^J / \operatorname{im}\partial_{k+1}^J. $$ A standard continuation argument shows that the Lagrangian Rabinowitz Floer homology does not depend on the choice of $h\in \mathcal{H}^{\reg}$ and $J\in \mathcal{J}^{\reg}_h$. For this, let $h_i\in \mathcal{H}^{\reg}$ and $J_i\in \mathcal{J}^{\reg}_{h_i}$ for $i=0,1$ be given. Pick a homotopy $\Gamma = \{(H+h_s, J_s)\}_{s\in \R}$ connecting $(H+h_0,J_0)$ to $(H+h_1,J_1)$ as in condition 3 of Theorem~\ref{thm:DefLRFH}. As before, it follows that for generic $\Gamma$ and any pair $(x,y)\in \Crit \A^{H+h_0}_{q_0,q_1} \times \Crit \A^{H+h_1}_{q_0,q_1}$ the associated space of Floer trajectories $\F_\Gamma(x,y)$ is a smooth manifold of dimension $\mu(y)-\mu(x)$ which is compact up to breaking. In particular, if $\mu(x)=\mu(y)$ then $\F_\Gamma(x,y)$ is a finite set of points and we set $m(x,y):= \# \F_\Gamma(x,y)\mod 2$. We define a linear map $\phi^\Gamma : (CF_*(\A^{H+h_1}_{q_0,q_1}),\p^{J_1})\to (CF(\A^{H+h_0}_{q_0,q_1}),\p^{J_0})$ on generators by $$ \phi^\Gamma(y) := \sum_{x\in \Crit \A^{H+h_0}_{q_0,q_1}}m(x,y)x. $$ Here the Novikov finiteness condition \eqref{Novikov} on $\Gamma$ ensures that $\phi^\Gamma\left( CF(\A^{H+h_1}_{q_0,q_1})\right)\subseteq CF(\A^{H+h_0}_{q_0,q_1})$. Compactness up to breaking implies that $\phi^\Gamma$ is a chain map, and composition with a homotopy in the opposite direction shows that $\phi^\Gamma$ induces an isomorphism on homology (see \cite[Chapter 11]{Audin2014} for details on this standard argument). Since $\LRFH_k(\A^{H+h}_{q_0,q_1};J)$ does not depend on $J\in \mathcal{J}^{\reg}_h$, we can drop $J$ from the notation and write is as $\LRFH_k(\A^{H+h}_{q_0,q_1})$. Since this homology is the same vector space for all $h\in \mathcal{H}^{\reg}$, we can unambiguously extend it as this vector space to all $h\in \mathcal{H}^{\reg}$. This proves the theorem in the case $q_0\neq q_1$. In the case $q_0=q_1$, by condition 1 for each $h\in\mathcal{H}$ we have critical points $(v,0)\in\Crit\A^{H+h}_{q_0,q_0}$ where $v$ is a constant path in the nonempty transverse intersection $S_h:=T^*_{q_0}Q\cap (H+h)^{-1}(0)$. If $S_h$ has positive dimension, then the functional $\A^{H+h}_{q_0,q_0}$ will only be Morse-Bott for generic $h\in\mathcal{H}$. This phenomenon is well known in Rabinowitz Floer theory and can be dealt with by picking a Morse function on $S_h$ and counting gradient flow lines with cascades, see~\cite{CieliebakFrauenfelder2009}. With this understood, the rest of the argument works as in the previous case. \end{proof} \begin{cor} Consider a cotangent bundle $(T^*Q,\omega)$ with its standard symplectic form and a Hamiltonian $H:T^*Q\to \R$ such that $H^{-1}(0)$ is of exact contact type. Assume that there exists an exhausting sequence of compact sets $\{K_n\}_{n\in\mathbb{N}}$, $K_n\subseteq K_{n+1}\subseteq T^*Q$, $\bigcup_{n\in\mathbb{N}}K_n=T^*Q$ and an open neighbourhood $\mathcal{H}$ of $0$ in $\{h\in C^\infty(T^*Q)\ |\ dh\in C_c^\infty(T^*Q)\}$ such that for every $n\in \mathbb{N}$ the sets $K_n$ and $\mathcal{H}_n=\{h\in\mathcal{H}\ |\ dh\in C_0^\infty(K_n)\}$ satisfy the assumptions of Theorem \ref{thm:DefLRFH}. Then for any $h\in\mathcal{H}$ the Lagrangian Rabinowitz Floer homology $\LRFH_*(\A^{H+h}_{q_0,q_1})$ is well defined and isomorphic to $\LRFH_*(\A^{H}_{q_0,q_1})$. \end{cor} \subsection{Positive Lagrangian Rabinowitz Floer homology} The action functional $\A^{H}_{q_0,q_1}$ provides an $\R$-filtration on $CF_*(\A^{H}_{q_0,q_1})$ by $$CF_*^{\leq a}\left(\A^{H}_{q_0,q_1}\right)\coloneqq \left\lbrace {\textstyle \sum_{x\in S}x}\in CF_*\left(\A^{H}_{q_0,q_1}\right) \left|\ \sup_{x\in S}\A^{H}_{q_0,q_1}(x) \leq a\right.\right\rbrace. $$Since Floer trajectories are defined by the $L^2$-gradient of the action functional, the boundary operator does not increase the action, i.e.\ \begin{equation}\label{filtration} \partial\left( CF^{\leq a}_{*+1}\left(\A^{H}_{q_0,q_1}\right)\right)\subseteq CF^{\leq a}_*\left(\A^{H}_{q_0,q_1}\right). \end{equation} The {\em positive Rabinowitz Floer homology} $\LRFH_*^+(\A^{H}_{q_0,q_1})$ is the homology of the quotient complex $$ CF_*^+\left(\A^{H}_{q_0,q_1}\right) \coloneqq CF_*\left(\A^{H}_{q_0,q_1}\right)\Big/CF_*^{\leq 0}\left(\A^{H}_{q_0,q_1}\right) $$with boundary operator $\partial^{\scriptscriptstyle +}$ induced by $\partial$ on the quotient. In order to get the required transversality in the preceding discussion, we need to replace $H$ by $H+h$ with $h\in\mathcal{H}^{reg}$ as in the previous subsection. For independence of positive Lagrangian Rabinowitz Floer homology of the choice of $h$ we need a further condition. The regular level set $H^{-1}(0)$ is said to be of {\em exact contact type} if there exists a Liouville vector field $Y$ on $T^*Q$ (satisfying $L_Y\om=\om$) such that $dH(Y)>0$ along $H^{-1}(0)$. Setting $(\xi,\sigma)=(Y,\eta)$ in equation~\eqref{eq:dAH} we get \begin{equation}\label{dA^H(Y)} \A^H (v,\eta) - d\A^H(v,\eta)[Y, \eta] = \eta\int_0^1 dH(Y)(v(t))dt. \end{equation} At critical points the second term on the left-hand side vanishes and we conclude \begin{equation}\label{Crit+} \begin{aligned} \Crit^\pm\A^{H}_{q_0,q_1} &:= \left\lbrace x\in \Crit\A^{H}_{q_0,q_1}\ |\ \pm\A^{H}_{q_0,q_1}(x) >0\right\rbrace \cr &= \left\lbrace (v,\eta)\in \Crit\A^{H}_{q_0,q_1}\ |\ \pm \eta >0\right\rbrace. \end{aligned} \end{equation} The following lemma shows that the exact contact type property provides a set of compactly supported perturbations for which the set of positive action values is bounded away from zero. \begin{lem}\label{lem:posAction} Consider a cotangent bundle $(T^*Q,\omega)$ with its standard symplectic form and a Hamiltonian $H:T^*Q\to \R$ such that $H^{-1}(0)$ is of exact contact type. Fix $q_0,q_1\in Q$ with $q_0\neq q_1$. Assume that there exists a compact subset $K\subseteq T^*Q$ and an open neighbourhood $\mathcal{H}$ of $0$ in $\{h\in C^\infty(T^*Q)\ |\ dh\in C_0^\infty(K)\}$ such that for all $h\in \mathcal{H}$ the following holds: \begin{equation} \forall\ (v,\eta) \in \Crit \A^{H+h}_{q_0,q_1} \quad \textrm{we have}\quad v([0,1]) \subseteq K.\label{PO_K} \end{equation} Then there exists an open neighbourhood $\mathcal{O}(K)$ of $0$ in $\mathcal{H}$ such that \begin{equation} \inf\left\lbrace\begin{array}{c|c} \A^{H+h}_{q_0,q_1}(x) & x\in \Crit^+ \A^{H+h}_{q_0,q_1}, \quad h\in\mathcal{O}(K)\end{array} \right\rbrace >0.\label{infA+} \end{equation} \end{lem} \begin{proof} By assumption there exists a Liouville vector field $Y$ such that $dH(Y)>0$ along $H^{-1}(0)$. Since $K$ is compact, $$ \delta:=\inf\{dH(Y)(x)\ |\ x\in K\cap H^{-1}(0)\} > 0. $$ For $h\in \mathcal{H}$ small enough in the $C^1$-norm we then have \begin{equation}\label{nbhd} (H+h)^{-1}(0)\cap K \subseteq \left\lbrace \begin{array}{c|c} x\in K& d(H+h)(Y)(x)\geq\delta/2 \end{array} \right\rbrace. \end{equation} Denote by $\mathcal{O}(K)$ the set of all $h\in \mathcal{H}$ satisfying \eqref{nbhd}. Then $\mathcal{O}(K)$ is an open neighbourhood of $0$ in $\{h\in C^\infty(T^*Q)\ |\ dh\in C_0^\infty(K)\}$. Consider $h\in\mathcal{O}(K)$ and $(v,\eta)\in \Crit^+ \A^{H+h}_{q_0,q_1}$, so $\eta>0$ in view of~\eqref{Crit+} applied to $H+h$. By~\eqref{PO_K} we have $v([0,1])\subset K\cap(H+h)^{-1}(0)$, so from~\eqref{dA^H(Y)} and~\eqref{nbhd} we obtain \begin{equation*} |\A^{H+h}_{q_0,q_1}(v,\eta)| \geq \delta\eta/2. \end{equation*} On the other hand, from $v(i)\in T_{q_i}^*Q$ for $i=0,1$ and $\p_sv=\eta X_{H+h}(v)$ we deduce $$ 0 < \varepsilon \leq |v(1)-v(0)| \leq \eta\int_0^1|X_{H+h}(v(t))|dt \leq C\eta $$ with the positive constants $$ \varepsilon := \textrm{dist}(K\cap T_{q_0}^*Q,K\cap T_{q_1}^*Q),\qquad C := \max\{|X_{H+h}(x)|\;\bigl|\;x\in K\}. $$ The two estimates combine to $|\A^{H+h}_{q_0,q_1}(v,\eta)| \geq \delta\varepsilon/2C>0$. \end{proof} The next result provides conditions under which positive Lagrangian Rabinowitz Floer homology is not only well defined, but also independent of the auxiliary choices and invariant under compact perturbations. \begin{cor}\label{cor:posLRFH} Consider the setting as in Theorem \ref{thm:DefLRFH} with sets $K\subseteq\mathcal{V}\subseteq T^*Q$ and $\mathcal{H}\subseteq \{C^\infty(T^*Q)\ |\ dh\in C_0^\infty(K)\}$. Assume $q_0\neq q_1$. Let $\mathcal{O}\subseteq\mathcal{H}$ be an open neighbourhood of $0$ such that for every pair $h_0,h_1 \in \mathcal{O}$ there exists a homotopy $\Gamma:= \{(H+h_s, J_s)\}_{s\in \R}$ satisfying Condition 3 of Theorem \ref{thm:DefLRFH}, and such that for every $x\in \Crit^+\A^{H+h_0}_{q_0,q_1}$ and every $y\in \Crit\A^{H+h_1}_{q_0,q_1}$ for which $\F_\Gamma(x,y)\neq \emptyset$ we have $\A^{H+h_1}_{q_0,q_1}(y)>0$. Then for every $h\in\mathcal{O}$ its positive Lagrangian Rabinowitz Floer homology is well defined, and for every pair $h_0,h_1 \in \mathcal{O}$, $\LRFH_*^+(\A^{H+h_0}_{q_0,q_1})$ is isomorphic to $\LRFH_*^+(\A^{H+h_1}_{q_0,q_1})$. \end{cor} \begin{proof} By assumption the critical set of $\A^H_{q_0,q_1}$ is continuously compact in $(K,\mathcal{O})$. Denote by $\mathcal{O}^{\reg}$ the subset of $\mathcal{O}$ consisting of all Hamiltonians $h\in \mathcal{O}$, such that $\A^{H+h}_{q_0,q_1}$ is Morse. By the standard Sard-Smale argument $\mathcal{O}^{\reg}$ is dense in $\mathcal{O}$. Fix $h_0,h_1 \in \mathcal{O}^{\reg}$. By assumption, there exist $J_i \in \mathcal{J}_{h_i}^{\reg}\subset \mathcal{J}(\mathcal{V}, \mathbb{J})$ for $i=0,1$ and a homotopy $\Gamma= \{(H+h_s, J_s)\}_{s\in \R}$ with $h_s\in \mathcal{O}$ and $J_s\in \mathcal{J}(\mathcal{V}, \mathbb{J})$ from $(H+h_0,J_0)$ to $(H+h_1,J_1)$ with the following properties: \begin{enumerate}[label=\alph*)] \item the homotopy $\Gamma$ satisfies the Novikov finiteness condition \eqref{Novikov}; \item for any pair $a,b\in \R$ the space of Floer trajectories $\mathcal{M}^\Gamma(a,b)$ is bounded in the $L^\infty$-norm; \item for every $x\in \Crit^+\A^{H+h_0}_{q_0,q_1}$ and every $y\in \Crit\A^{H+h_1}_{q_0,q_1}$ such that $\F_\Gamma(x,y)\neq \emptyset$ we have $\A^{H+h_1}_{q_0,q_1}(y)>0$. \end{enumerate} From properties a) and b) we get a chain map $\phi^\Gamma : CF_*(\A^{H+h_1}_{q_0,q_1})\to CF_*(\A^{H+h_0}_{q_0,q_1})$. It satisfies $\phi^\Gamma \left(CF_*^{\leq 0}(\A^{H+h_1}_{q_0,q_1})\right) \subseteq CF_*^{\leq 0}(\A^{H+h_0}_{q_0,q_1})$, since otherwise there would exist $x\in \Crit^+\A^{H+h_0}_{q_0,q_1}$ and $y\in \Crit\A^{H+h_1}_{q_0,q_1}$ such that $\F_\Gamma(x,y)\neq \emptyset$ and $\A^{H+h_1}_{q_0,q_1}(y)\leq 0$, contradicting property c). Thus $\phi^\Gamma$ descends to a chain map $\phi^\Gamma_+ : CF_*^+(\A^{H+h_1}_{q_0,q_1})\to CF_*^+(\A^{H+h_0}_{q_0,q_1})$, and the usual argument using a homotopy in the opposite direction (see~\cite[Proposition 11.2.9]{Audin2014}) shows that the induced map on homology\linebreak $\Phi^\Gamma_+ \colon \LRFH_*^+(\A^{H+h_1}_{q_0,q_1})\to \LRFH_*^+(\A^{H+h_0}_{q_0,q_1})$ is an isomorphism. \end{proof} \section{Bounds on the Floer trajectories} The aim of this section is to show that the Copernican Hamiltonian $H_0$ defined in \eqref{DefH0} together with the set of compactly supported perturbations defined in \eqref{DefHset} satisfy the assumptions of Theorem \ref{thm:DefLRFH}, in order to apply it and prove Theorem \ref{thm:LRFH}. We will start by showing that the necessary condition for the existence of Reeb chords on $(H_0-h)^{-1}(0)$ between any two cotangent fibres $T_{q_0}\R^2$ and $T_{q_1}\R^2$, assumption 1 of Theorem \ref{thm:DefLRFH}, holds true in our setting. Next, we will prove that the second assumption of Theorem \ref{thm:DefLRFH} holds true in our setting, i.e., the critical set of the Lagrangian Rabinowitz action functional is bounded in $L^\infty$. It is possible to define the Lagrangian Rabinowitz Floer homology without this assumption as a limit of homologies defined in an action window, but this is more involved and we postpone it to a future paper. The most challenging part, which will occupy most of this section, is to prove that the Floer trajectories are uniformly bounded in the $L^\infty$-norm. This is essential in first defining the Lagrangian Rabinowitz homology and then constructing the isomorphism from Theorem \ref{thm:LRFH}. \begin{lem}\label{lem:nonEmpty} Let $H_0$ be the Copernican Hamiltonian defined in \eqref{DefH0} and let $\mathcal{H}$ be the set of perturbations defined in \eqref{DefHset}. Then the set $(H_0-h)^{-1}(0)\cap T_q\R^2$ is compact and nonempty for every $h\in \mathcal{H}$ and every $q\in \R^2$. \end{lem} \begin{proof} Fix $q\in \R^2$. Since $h$ is constant outside a compact set, we have $$ \lim_{|p|\to\infty} \left(H_0(q,p)-h(q,p)\right)=+\infty. $$ On the other hand, we have $h>0$ by assumption and $H_0(q,0)=0$, hence $$ H_0(q,0)-h(q,0)=-h(q,0)<0. $$ Consequently, by the intermediate value theorem there exists $p\in T_q\R^2$, such that $H_0(q,p)-h(q,p)=0$. \end{proof} The following lemma proves that the Copernican Hamiltonian $H_0$ together with the set of compactly supported perturbations defined in \eqref{DefHset} satisfies the second assumption of Theorem \ref{thm:DefLRFH}. \begin{lem}\label{lem:H0Chord} Let $H_0:T^*\R^2 \to \R$ be the Copernican Hamiltonian defined in \eqref{DefH0}. Fix two points $q_0,q_1\in \R^2$. For $n\in\mathbb{N}$ and $m>0$ denote $r_n := \max\{|q_0|,|q_1|\}+n$ and define the compact set $K_{n,m} \subseteq T^*\R^2$ by $$ K_{n,m}:= \left\lbrace\begin{array}{c|c} & r\leq r_n, \quad p_r \leq \sqrt{r_n^2+2m},\\ \hspace*{-0.2cm}\smash{\raisebox{.5\normalbaselineskip}{ $(r, p_r,\theta,p_\theta)\in T^*\R_+\times T^*S^1$}}& p_\theta \leq r_n\sqrt{ 2m}. \end{array}\right\rbrace\,. $$ Then for every $h \in \mathcal{H}$ such that $dh \in C_0^\infty( K_{n,m})$ and $\|h\|_{L^\infty}<m$ and any $(v,\eta)\in \Crit \A^{H_0-h}_{q_0,q_1}$ we have $v([0,1])\subseteq K_{n,m}$. \end{lem} \begin{proof} We will use the Poisson bracket defined by $\{f,g\}=\om(X_f,X_g)=dg(X_f)$. Using \eqref{DefH0} we calculate $$ \{H_0,r\} = p_r, \quad \textrm{and}\quad \{H_0,\{H_0, r\}\}=\frac{p_\theta^2}{r^3}. $$ Thus at a point where $\{\{H_0, r\}=0\}$ and $\{\{H_0,\{H_0, r\}\}\leq 0$ we have $p_r=p_\theta=0$. Since $H_0(r,\theta, 0, 0) = 0$, this implies that for each constant $c>0$ we have \begin{equation}\label{supp_h} H_0^{-1}(c)\cap \{\{H_0, r\}=0\}\cap \{\{H_0,\{H_0, r\}\}\leq 0\} = \emptyset. \end{equation} Let now $(v, \eta) \in \Crit \A^{H_0-h}_{q_0,q_1}$. We will first show that $$ \max r \circ v \leq r_n. $$ Arguing by contradiction, suppose that $r\circ v(t_0)=\max r \circ v > r_n$ for some $t_0\in [0,1]$. Since $r\circ v(i)=|q_i|<r_n$ for $i=0,1$, we have $t_0\in (0,1)$ and the condition that $r\circ v$ attains its maximum at $t_0$ gives \begin{gather*} \frac{d}{dt}r\circ v (t_{0}) = \eta\{H_0-h, r\}\circ v (t_{0})= 0, \\ \textrm{and} \quad \frac{d^{2}}{dt^{2}}r\circ v (t_{0})=\eta^2\{H_0-h, \{H_0-h, r\}\} \circ v (t_{0})\leq 0. \end{gather*} The definition of $K_{n,m}$ and $dh \in C_0^\infty( K_{n,m})$ imply that $h$ is equal to a constant $c>0$ near $v(t_0)$ and can therefore be ignored in the Poisson brackets, and we obtain a contradiction to~\eqref{supp_h}. For the bounds on $p_r\circ v$ and $p_\theta\circ v$, recall that $v(t)\in (H_0-h)^{-1}(0)$ for all $t\in [0, 1]$. Consequently, we have \begin{align*} \frac{p_r^2}{2}+\frac{p_\theta^2}{2r^2} +p_\theta & =h(r,\theta, p_r, p_\theta),\\ p_r^2+\left(\frac{p_\theta}{r}+r\right)^2 & =r^2+2h \leq r_n^2+2m,\\ |p_r \circ v| & \leq \sqrt{r_n^2+2m},\\ |p_\theta \circ v| & \leq r_n\left(\sqrt{r_n^2+2m}-r_n\right)\leq r_n\sqrt{2m}. \end{align*} \end{proof} Before formulating the main theorem of this section we will first introduce the following notation. First, to avoid unnecessary clutter we will abbreviate $\A^{H_0-h}:=\A^{H_0-h}_{q_0,q_1}$ as in this section dependence on $q_0,q_1\in \R^2$ is not very relevant. Next, recall the set $\mathcal{H}$ from \eqref{DefHset}. For a compact subset of $K\subseteq T^*\R^2$ we define $$\mathcal{H}(K):= \{h\in \mathcal{H}\ |\ dh\in C_0^\infty(K)\}.$$ Moreover, for an open precompact subset $\mathcal{V}\subseteq T^*\R^2$ we denote by $\mathcal{J}(\mathcal{V},\mathbb{J})$ the set of all the $2$-parameter families of $\omega_0$-compatible almost complex structures on $(T^*\R^2, \omega_0)$ which are equal to the standard complex structure $\mathbb{J}$ outside of $\mathcal{V}$ (see the definition in Section \ref{sec:FloerTrajec}). \begin{thm}\label{thm:FloerBounds} Let $H_0$ be the Hamiltonian defined in \eqref{DefH0} and let $h_0,h_1 \in \mathcal{H}$. Fix two points $q_0,q_1 \in \R^2$. Let $K$ be a compact subset of $T^*\R^2$, such that for $i=0,1$ it satisfies $$ \supp h_i \subseteq K\qquad \textrm{and}\qquad v([0,1])\subseteq K\qquad \textrm{for all}\quad (v,\eta)\in \Crit \A^{H_0-h_i}. $$ Let $\mathcal{V}\subseteq T^*\R^2$ be an open, but precompact subset, such that $K\subseteq \mathcal{V}$. Let $\Gamma:=\{(h_s, J_s)\}_{s\in \R}$ be a smooth homotopy of Hamiltonians $h_s\in \mathcal{H}(K)$ and $2$-parameter families of almost complex structures $J_s \in C^\infty([0,1]\times \R,\mathcal{J}(\mathcal{V},\mathbb{J}))$ constant in $s$ outside $[0,1]$, such that \begin{gather} \|\partial_{s}h_{s}\|_{L^{\infty}}\left(\frac{1}{c}+\|J\|_{L^{\infty}}^2\right) \leq \frac{1}{3},\label{inqGamma}\\ \textrm{where}\qquad c:=\inf_{s\in[0,1]}\{h_s-dh_s(p\partial_p)\}.\label{Defc} \end{gather} Then for every pair $(a,b)\in \R^2$ the corresponding space $\mathcal{M}^\Gamma(a,b)$ of Floer trajectories defined in \eqref{DefM(a,b)} is bounded in the $L^\infty$-norm. \end{thm} \subsection{The bounds on the action} If the family of Hamiltonians $\{H_s\}_{s\in \R}$ is constant in $s$, i.e. $H_s = H$ for all $s\in \R$ then $\mathcal{M}^\Gamma(a,b)\neq \emptyset$ implies $b>a$. However, since in the parametric case the action along the Floer trajectories may not be monotonically increasing, we need to ensure that we have a bound on how much the action can decrease along a Floer trajectory. Proving the bounds on the action conditions will be the main aim of this section. We will start by proving the following lemma: \begin{lem}\label{lem:inqEtaAct} For every $h\in\mathcal{H}$ \begin{gather*} \textrm{if}\qquad \|\nabla \A^{H_0-h}(v,\eta)\|_{L^2\times\R} <1,\\ \textrm{then}\qquad |\eta | \leq \frac{1}{c}|\A^{H_0-h}(v,\eta)|+\frac{1}{\sqrt{2c}}, \end{gather*} where $c:=\inf\{h-dh(p\partial_p\}>0$. \end{lem} \begin{proof} Using \eqref{DefH0} we can compute the Hamiltonian $H_0$ satisfies the following equality: \begin{equation}\label{H0pdp} dH_0(p\partial_p) = p_1^2+p_2^2+q_2p_1-q_1p_2 =\frac{1}{2}|p|^2+H_0(p,q). \end{equation} Combining that with equality \eqref{dA^H(Y)} for all $(q,p,\eta)=(v,\eta)\in \mathscr{H}_{q_0,q_1}$ we obtain \begin{align} \A^{H_0-h}(v,\eta) -d\A^{H_0-h}[p\partial_p,0] & =\eta\int_0^1\left(d(H_0-h)(p\partial_p)-H_0(v)+h(v)\right)\nonumber \\ & = \eta \left(\frac{1}{2}\|p\|_{L^2} +\int_0^1 \left(h(v)-dh(p\partial p)\right)\right)\\ & \geq \eta \left(\frac{1}{2}\|p\|_{L^2}+c\right), \label{IntdH(Y)} \end{align} where the last inequality comes from the fact that $c:=\inf\{h-dh(p\partial_p\}>0$. Consequently, for all $(q,p,\eta)=(v,\eta)\in \mathscr{H}_{q_0,q_1}$, such that $\|\nabla \A^{H_0-h}(v,\eta)\|_{L^2\times\R} <1$, we obtain $$ |\eta| < \frac{|\A^{H_0-h}(v,\eta)|+\|p\|_{L^2}}{\frac{1}{2}\|p\|_{L^2}^2+c}. $$ Now let us analyze the right-hand side as functions of $\|p\|_{L^2}$: \begin{align*} \max_{x\geq 0}\left(\frac{1}{\frac{1}{2} x^2+c}\right) & =\frac{1}{c},\\ \frac{d}{dx}\left( \frac{x}{\frac{1}{2}x^2+c}\right) & = \frac{c-\frac{1}{2}x^2}{\left(\frac{1}{2}x^2+c\right)^2},\\ \max_{x\geq 0} \left( \frac{x}{\frac{1}{2}x^2+c}\right) & = \frac{1}{\sqrt{2c}}. \end{align*} These estimates give the desired inequality. \end{proof} \begin{lem}\label{lem:ActBound} Consider the setting as in Theorem \ref{thm:FloerBounds}. Fix $a, b\in \mathbb{R}$. Then $\|\eta\|_{L^\infty}$, $\|\A^{H_0-h_s}\circ u(s)\|_{L^\infty}$ and $\|\nabla^{J_s}\A^{H_0-h_s}\circ u(s)\|_{L^2([0,1])\times\R}$ are uniformly bounded for all $u \in \mathcal{M}^\Gamma(a,b)$. \end{lem} The following proof follows closely the arguments presented in \cite[Prop. 3.3]{Pasquotto2017}. We will not present all of them here, but we encourage a curious reader to see the details in \cite{Pasquotto2017}. \begin{proof} Using the fact that $u\in \mathcal{M}^\Gamma(a,b)$ is a Floer trajectory one can calculate the derivative of the action functional over $s$ and obtain the following inequalities (see \cite[Prop. 3.3]{Pasquotto2017}): \begin{align} \|\mathcal{A}^{H_0-h_{s}}( u)\|_{L^{\infty}} & \leq \max\{|a|,|b|\} + \|\eta\|_{L^{\infty}}\|\partial_{s}h_{s}\|_{L^{\infty}},\label{inqAct}\\ \|\nabla^{J_{s}} \mathcal{A}^{H_0-h_{s}}(u)\|_{L^{2}(\mathbb{R}\times [0,1])}^{2} & \leq \|J\|_{L^{\infty}} (b-a + \|\eta\|_{L^{\infty}}\|\partial_{s}h_{s}\|_{L^{\infty}}). \label{inqEnergy} \end{align} In particular the convergence of the integral $$ \|\nabla \mathcal{A}^{H_0-h_{s}}(u)\|_{L^{2}(\mathbb{R}\times [0,1])} \leq \|J\|_{L^{\infty}} \|\nabla^{J_{s}} \mathcal{A}^{H_0-h_{s}}(u)\|_{L^{2}(\mathbb{R}\times [0,1])}, $$ implies that for small enough $s$ we have $\|\nabla \mathcal{A}^{H_0-h_{s}}(u(s))\|_{L^{2}\times\mathbb{R}}<1$. This ensures that for all $s \in \mathbb{R}$ the following value $\tau_{0}(s)$ is well defined and finite $$ \tau_{0}(s): = \sup\{ \tau \leq s\ |\ \|\nabla \mathcal{A}^{H_0-h_{\tau}}(u(\tau))\|_{L^{2}\times\mathbb{R}}<1\}. $$ For $\tau \in [\tau_{0}(s),s]$ we have $\|\nabla \mathcal{A}^{H_0-h_{\tau}}(u(\tau))\|_{L^{2}\times\mathbb{R}}\geq 1$, which allows us to estimate (see \cite[Prop. 3.3]{Pasquotto2017}): \begin{align} |s -\tau_0(s)| & \leq \|J\|_{L^{\infty}}^3 (b-a + \|\eta\|_{L^{\infty}}\|\partial_{s}h_{s}\|_{L^{\infty}}),\\ |\eta(s) -\eta(\tau_0(s))| & \leq \|J\|_{L^{\infty}}^2 (b-a + \|\eta\|_{L^{\infty}}\|\partial_{s}h_{s}\|_{L^{\infty}}). \label{inqeta} \end{align} Now using the constant $c>0$ as in \eqref{Defc} together with the the estimates from Lemma \ref{lem:inqEtaAct} and \eqref{inqAct} we can calculate: \begin{align*} |\eta(s)|& \leq |\eta(\tau_0(s))|+|\eta(s) -\eta(\tau_0(s))|\\ & \leq \frac{1}{c}|\A^{H_0-h_s}(u(s))|+\frac{1}{\sqrt{2c}}+\|J\|_{L^{\infty}}^2 (b-a + \|\eta\|_{L^{\infty}}\|\partial_{s}h_{s}\|_{L^{\infty}})\\ & \leq \|\eta\|_{L^{\infty}}\|\partial_{s}h_{s}\|_{L^{\infty}}\left(\frac{1}{c}+\|J\|_{L^{\infty}}^2\right)+\frac{1}{c}\max\{|a|,|b|\}+\frac{1}{\sqrt{2c}}+\|J\|_{L^{\infty}}^2 (b-a). \end{align*} Since this inequality has to hold for all $s\in \R$ it also has to hold for $\|\eta\|_{L^\infty}$. Using \eqref{inqGamma} we obtain the following estimate: \begin{align} \|\eta\|_{L^{\infty}} & \leq \frac{\frac{1}{c}\max\{|a|,|b|\}+\frac{1}{\sqrt{2c}}+\|J\|_{L^{\infty}}^2 (b-a)}{1-\|\partial_{s}h_{s}\|_{L^{\infty}}\left(\frac{1}{c}+\|J\|_{L^{\infty}}^2\right)}\nonumber\\ &\leq \frac{3}{2} \left( \frac{1}{c}\max\{|a|,|b|\}+\frac{1}{\sqrt{2c}}+\|J\|_{L^{\infty}}^2 (b-a)\right)=:\mathfrak{y},\label{eqEta} \end{align} Now using \eqref{inqGamma}, \eqref{inqAct}, \eqref{inqEnergy} and \eqref{eqEta} we obtain the desired uniform bounds: \begin{align} \|\mathcal{A}^{H_0-h_{s}-c}( u)\|_{L^{\infty}} &\leq \max\{|a|,|b|\} + \frac{c}{3}\mathfrak{y}=:\mathfrak{a},\label{DefA}\\ \|\nabla^{J_{s}} \mathcal{A}^{H_0-h_{s}-c}(u)\|_{L^{2}(\mathbb{R}\times [0,1])}^{2} & \leq \|J\|_{L^{\infty}} \left(b-a + \frac{c}{3}\mathfrak{y}\right)=:\mathfrak{e}.\label{DefE} \end{align} \end{proof} Having obtained the bounds on the action we are ready to prove the Novikov finiteness condition: \begin{lem}\label{lem:Novikov} Consider the setting as in Theorem \ref{thm:FloerBounds}. Then $\mathcal{M}_\Gamma(a,b)\neq \emptyset$ implies $$ a \leq \max\left\lbrace 3 b, 3\sqrt{\frac{c}{2}}\right\rbrace \qquad \textrm{and} \qquad b \geq \min\left\lbrace 3a, -3\sqrt{\frac{c}{2}}\right\rbrace. $$ \end{lem} \begin{proof} This proof follows arguments presented in \cite[Cor. 3.8]{CieliebakFrauenfelder2009}. We will prove the first inequality since the second is completely analogous. \begin{align} \textrm{First assume}\ \qquad 3\sqrt{\frac{c}{2}} & \leq a \quad \textrm{and} \quad |b|\leq a.\label{aBound}\\ \textrm{Then} \qquad\max\{|a|,|b|\} & = a \quad \textrm{and} \quad b-a\leq 0.\nonumber \end{align} By \eqref{eqEta} and \eqref{aBound} we get: $$ \|\eta\|_{L^\infty}\leq \frac{3}{2}\left(\frac{a}{c}+\frac{1}{\sqrt{2c}}\right)=2\frac{a}{c}. $$ On the other hand, by \eqref{inqGamma} we have $\|\partial_sh_s\|_{L^\infty}\leq \frac{c}{3}$. This, together with \eqref{inqEnergy}, implies $$ b \geq a- \|\eta\|_{L^\infty}\|\partial_sh_s\|_{L^\infty} \geq a -2 \frac{a}{c}\cdot \frac{c}{3}=\frac{1}{3}a. $$ Now assume that $ 3\sqrt{\frac{c}{2}} \leq a < |b|$. To finish the proof it suffices to exclude the case $a< - b$. $$ \textrm{Then} \qquad\max\{|a|,|b|\} = -b \quad \textrm{and} \quad b-a\leq 0. $$ By \eqref{eqEta} and \eqref{aBound} we get: $$ \|\eta\|_{L^\infty}\leq \frac{3}{2}\left(\frac{-b}{c}+\frac{1}{\sqrt{2c}}\right)\leq \frac{3}{2}\left(\frac{-b}{c}+\frac{a}{3c}\right)\leq -2 \frac{b}{c}. $$ This, together with \eqref{inqGamma} and \eqref{inqEnergy}, gives us a contradiction: \begin{gather*} b \geq a- \|\eta\|_{L^\infty}\|\partial_sh_s\|_{L^\infty}\geq a + 2 \frac{b}{c} \cdot \frac{c}{3}=a+\frac{2}{3}b,\\ 0 \geq \frac{1}{3} b \geq a \geq 3\sqrt{\frac{c}{2}}>0. \end{gather*} \end{proof} \subsection{The set of infinitesimally small action derivation} Let $H_0$ be the Hamiltonian defined in \eqref{DefH0} and let $\mathcal{H}$ be the set of Hamiltonians as in \eqref{DefHset}. For a Hamiltonian $h \in \mathcal{H}$ and fixed constants $\mathfrak{a}, \mathfrak{y}, \varepsilon>0$ we define the following set: \begin{equation} \mathcal{B}_h(\mathfrak{a}, \mathfrak{y}, \varepsilon):= \left\lbrace\begin{array}{c|c} & |\nabla\A^{H_0-h}(v,\eta)|_{L^2\times\R}<\varepsilon,\\ {\smash{\raisebox{.5\normalbaselineskip}{ $(v, \eta) \in\mathscr{H}_{q_0,q_1}\times\R$}}}& |\A^{H_0-h}(v,\eta)|\leq \mathfrak{a},\ |\eta|\leq \mathfrak{y}. \end{array}\right\rbrace \end{equation} We will call this set the set of infinitesimally small action derivation. The main aim of this subsection will be to prove the following lemma: \begin{prop}\label{prop:smallDerivSetBound} For fixed constants $\mathfrak{a}, \mathfrak{y}>0$ and $\varepsilon>0$ small enough, the corresponding set $\mathcal{B}_h(\mathfrak{a}, \mathfrak{y}, \varepsilon)$ is bounded both in the $L^{\infty}\times\R$-norm and in the $L^2\times \R$-norm. \end{prop} We will prove Proposition \ref{prop:smallDerivSetBound} in a series of smaller lemmas: \begin{lem}\label{lem:partial_p} Let $h\in \mathcal{H}$ and let $c:=\inf\{h-dh(p\partial_p)\}>0$. If we fix $\mathfrak{a}, \mathfrak{y}, \varepsilon>0$ then for every $(q,p,\eta) \in \mathcal{B}_h (\mathfrak{a}, \mathfrak{y}, \varepsilon)$ we have: $$ |\eta|\|p\|_{L^2}\leq 2\varepsilon +\frac{\mathfrak{a}}{\sqrt{2c}}\qquad\textrm{and}\qquad \|\partial_t p\|_{L^2}\leq 3\varepsilon +\frac{\mathfrak{a}}{\sqrt{2c}} + \mathfrak{y}\|\nabla h\|_{L^\infty} $$ \end{lem} \begin{proof} Using \eqref{IntdH(Y)} for $(q,p,\eta) \in \mathcal{B}_h (\mathfrak{a}, \mathfrak{y}, \varepsilon)$ we obtain \begin{gather*} \eta \left(\frac{1}{2}\|p\|_{L^2}^2+c\right) \leq \A^{H_0-h}(v,\eta)-d\A^{H_0-h}[p\partial_p,0] \leq \mathfrak{a}+\varepsilon\|p\|_{L^2},\\ |\eta| \|p\|_{L^2}\leq \frac{\mathfrak{a}\|p\|_{L^2}+\varepsilon\|p\|_{L^2}^2}{\frac{1}{2}\|p\|_{L^2}^2+c}. \end{gather*} Now we would like to estimate the maximum of the function on the right-hand side of the inequality: \begin{gather*} \frac{d}{dx}\left(\frac{\mathfrak{a}x+\varepsilon x^2}{\frac{1}{2}x^2+c}\right) = -\frac{\frac{1}{2}\mathfrak{a}x^2-2\varepsilon c x -\mathfrak{a} c}{\left(\frac{1}{2}x^2+c\right)^2},\\ \frac{d}{dx}\left(\frac{\mathfrak{a}x+\varepsilon x^2}{\frac{1}{2}x^2+c}\right) = 0 \quad \iff \quad x=\frac{1}{\mathfrak{a}}\left(2\varepsilon c \pm \sqrt{4 \varepsilon^2 c^2 + 2 \mathfrak{a}^2 c}\right),\\ \max_{x\geq 0} \left(\frac{\mathfrak{a}x+\varepsilon x^2}{\frac{1}{2}x^2+c}\right)= \varepsilon+ \sqrt{\frac{\mathfrak{a}^2}{2c}+\varepsilon^2} \leq 2\varepsilon +\frac{\mathfrak{a}}{\sqrt{2c}}. \end{gather*} This gives us the bound on $|\eta| \|p\|_{L^2}$, i.e. the first inequality. Let us denote by $$ (X_{H_0-h})_p:= \left(p_2 +\frac{\partial h}{\partial_{q_1}}\right)\partial_{p_1}-\left(p_1-\frac{\partial h}{\partial_{q_2}}\right)\partial_{p_2}. $$ Then we can estimate \begin{align*} \|\partial_tp\|_{L^2} & \leq \| \partial_t p-\eta (X_{H_0-h})_p\|_{L^2}+|\eta| \|(X_{H_0-h})_p\|_{L^2}\nonumber\\ & \leq \|\nabla\A^H(q,p,\eta)\|_{L^2\times \R}+|\eta|(\|p\|_{L^2}+\|\nabla h \|_{L^\infty}),\\ & \leq 3\varepsilon +\frac{\mathfrak{a}}{\sqrt{2c}} + \mathfrak{y}\|\nabla h \|_{L^\infty}. \end{align*} \end{proof} \begin{lem}\label{lem:qBound} If we fix $\mathfrak{a}, \mathfrak{y}, \varepsilon>0$ then for every $(q,p,\eta) \in \mathcal{B}_h (\mathfrak{a}, \mathfrak{y}, \varepsilon)$ we have: $$ \|q\|_{L^2}, \| q\|_{L^\infty} \leq \min\{|q_0|,|q_1|\}+2 \left(3\varepsilon + \frac{\mathfrak{a}}{\sqrt{2c}}+\mathfrak{y}\|\nabla h\|_{L^\infty}\right). $$ \end{lem} \begin{proof} Begin first observe that $$ qdq(X_{H_0})=q_1(p_1+q_2)+q_2(p_2-q_1)=q_1 p_1 + q_2 p_2. $$ Therefore, for all $t\in [0,1]$ we have \begin{align} |q(t)|^2 - |q_0|^2& =2\int_0^t qdq\left(\partial_tv - \eta X_{H_0-h}\right)+2\eta \int_0^t \left(qdq(X_{H_0})-qdq(X_h)\right)\nonumber\\ & \leq 2\|q\|_{L^2}\|\nabla \A^H(q,p,\eta)\|_{L^2\times \R}+2|\eta|\|q\|_{L^2}\left(\|p\|_{L^2} + \|\nabla h\|_{L^\infty}\right),\nonumber\\ & \leq 2 \|q\|_{L^2}\left(3\varepsilon + \frac{\mathfrak{a}}{\sqrt{2c}}+\mathfrak{y}\|\nabla h\|_{L^\infty}\right), \label{ineq1} \end{align} where the last inequality we obtained using the result form Lemma \ref{lem:partial_p}. Integrating both sides, we obtain $$ \|q\|_{L^2}^2\leq |q_0|^2+2 \|q\|_{L^2}\left(3\varepsilon + \frac{\mathfrak{a}}{\sqrt{2c}}+\mathfrak{y}\|\nabla h\|_{L^\infty}\right). $$ By solving this quadratic inequality we obtain the following bound: \begin{align*} \|q\|_{L^2} & \leq 3\varepsilon + \frac{\mathfrak{a}}{\sqrt{2c}}+\mathfrak{y}\|\nabla h\|_{L^\infty} + \sqrt{\left(3\varepsilon + \frac{\mathfrak{a}}{\sqrt{2c}}+\mathfrak{y}\|\nabla h\|_{L^\infty}\right)^2+|q_0|^2}\\ & \leq |q_0|+2 \left(3\varepsilon + \frac{\mathfrak{a}}{\sqrt{2c}}+\mathfrak{y}\|\nabla h\|_{L^\infty}\right) \end{align*} By repeating this procedure with the equation $|q(t)|^2=|q_1|^2-2\int_t^1 q\partial_tq$, we obtain the bound on $\|q\|_{L^2}$ we were looking for. To obtain the bound for $\|q\|_{L^\infty}$ we will use \eqref{ineq1} again: \begin{align*} \|q\|_{L^\infty} & \leq \sqrt{|q_0|^2+ 2 \|q\|_{L^2}\left(3\varepsilon + \frac{\mathfrak{a}}{\sqrt{2c}}+\mathfrak{y}\|\nabla h\|_{L^\infty}\right)}\\ & \leq \sqrt{|q_0|^2+ 2 \left( |q_0|+2 \left(3\varepsilon + \frac{\mathfrak{a}}{\sqrt{2c}}+\mathfrak{y}\|\nabla h\|_{L^\infty}\right)\right)\left(3\varepsilon + \frac{\mathfrak{a}}{\sqrt{2c}}+\mathfrak{y}\|\nabla h\|_{L^\infty}\right)}\\ & \leq |q_0|+ 2\left(3\varepsilon + \frac{\mathfrak{a}}{\sqrt{2c}}+\mathfrak{y}\|\nabla h\|_{L^\infty}\right) \end{align*} Analogously as before, we repeat this procedure with the equation\linebreak $|q(t)|^2=|q_1|^2-2\int_t^1 q\partial_tq$ to obtain the bound on $\|q\|_{L^\infty}$ we were looking for. \end{proof} \begin{lem}\label{lem:pBound} If we fix $\mathfrak{a}, \mathfrak{y}, \varepsilon>0$ then for every $(q,p,\eta) \in \mathcal{B}_h (\mathfrak{a}, \mathfrak{y}, \varepsilon)$ we have: \begin{align*} \|p\|_{L^2} & \leq 2\mathfrak{q}+\sqrt{2 \left( \|h\|_{L^\infty}+c+\varepsilon\right)}=:\mathfrak{p},\\ \|p\|_{L^\infty} & \leq \mathfrak{p}+\left( \varepsilon+\|\nabla h\|_{L^\infty}\right),\\ \textrm{where}\qquad \mathfrak{q} & :=\min\{|q_0|,|q_1|\}+2 \left(3\varepsilon + \frac{\mathfrak{a}}{\sqrt{2c}}+\mathfrak{y}\|\nabla h\|_{L^\infty}\right). \end{align*} \end{lem} \begin{proof} By \eqref{H0pdp} we have \begin{align*} \frac{1}{2}\|p\|_{L^2}^2 & = \int_0^1 (H_0-h)(q,p)dt -\int_0^1\left(p_1q_2-p_2q_1\right)dt +\int_0^1 h(q,p)dt\\ & \leq \|\nabla \A^{H_0-h-c}\|_{L^2\times \R}+\|p\|_{L^2}\|q\|_{L^2}+\|h\|_{L^\infty}+c\\ & \leq \varepsilon + \mathfrak{q}\|p\|_{L^2} +\|h\|_{L^\infty}+c \end{align*} where $\mathfrak{q}:=\min\{|q_0|,|q_1|\}+2 \left(3\varepsilon + \frac{\mathfrak{a}}{\sqrt{2c}}+\mathfrak{y}\|\nabla h\|_{L^\infty}\right)$ as in Lemma \ref{lem:qBound}. By solving this quadratic inequality we obtain the following bound: \begin{align*} \|p\|_{L^2} & \leq \mathfrak{q} + \sqrt{\mathfrak{q}^2 + 2 \left( \|h\|_{L^\infty}+c+\varepsilon\right)} \leq 2 \mathfrak{q}+\sqrt{2 \left( \|h\|_{L^\infty}+c+\varepsilon\right)}\\ & = 2\min\{|q_0|,|q_1|\}+4 \left(3\varepsilon + \frac{\mathfrak{a}}{\sqrt{2c}}+\mathfrak{y}\|\nabla h\|_{L^\infty}\right)+\sqrt{2 \left( \|h\|_{L^\infty}+c+\varepsilon\right)}. \end{align*} To prove the second bound first observe that there exists $t_0\in [0,1]$, such that $|p(t_0)|\leq 2 \mathfrak{q}+\sqrt{2 \left( \|h\|_{L^\infty}+c+\varepsilon\right)}$. Thus can estimate: \begin{align*} |p(t)| & \leq |p(t_0)| + \int_{t_0}^t |\partial_tv|= |p(t_0)| + \int_{t_0}^t |\partial_tv- X_{H_0-h}|+\int_{t_0}^t |X_h|\\ & \leq |p(t_0)| +\|\nabla\A^{H_0-h-c}\|_{L^2\times \R} + \|\nabla h\|_{L^\infty} \leq |p(t_0)| +\varepsilon+\|\nabla h\|_{L^\infty}\\ & \leq 2\min\{|q_0|,|q_1|\}+13\varepsilon + 4\frac{\mathfrak{a}}{\sqrt{2c}}+(4\mathfrak{y}+1)\|\nabla h\|_{L^\infty} +\sqrt{2 \left( \|h\|_{L^\infty}+c+\varepsilon\right)}. \end{align*} \end{proof} \begin{rem} Note that in the proof above we did not use the assumption that $dh \in C_c^\infty(T^*\R^2)$. Thus we can conclude that for any $\mathfrak{a}$, $\mathfrak{y}$, $\varepsilon>0$ and any Hamiltonian $h\in C^\infty(T^*\R^2)\cap W^{1,\infty}(T^*\R^2)$, such that $h>0$ and $h-dh(p\partial_p)>0$ the corresponding set $\mathcal{B}_h(\mathfrak{a},\mathfrak{y},\varepsilon)$ is bounded in the $L^{\infty}\times \R$-norm and the $L^2\times \R$-norm. \end{rem} \begin{rem}\label{rem:smallDeriv} Note that for $h \in \mathcal{H}$ the bounds on $\mathcal{B}_h(\mathfrak{a},\mathfrak{y}, \varepsilon)$ depend smoothly on the constants $\mathfrak{a}$, $\mathfrak{y}$, $\varepsilon$, $c:=\inf\{h-dh(p\partial_p)\}$ and $\|h\|_{W^{1,\infty}}$. Thus if we take a subset $\mathcal{H}'\subseteq \mathcal{H}$ bounded in the $W^{1,\infty}$-norm such that $\inf_{h\in\mathcal{H}'}\inf\{h-dh)p\partial_p)\}>0$, then the corresponding set $$ \bigcup_{h\in\mathcal{H}'}\mathcal{B}_h(\mathfrak{a},\mathfrak{y},\varepsilon)\subseteq \mathscr{H}_{q_0,q_1}\times \R, $$ will be also bounded in the $L^{\infty}\times \R$-norm and the $L^2\times \R$-norm. \end{rem} \subsection{The $L^2$ bounds} The aim of this subsection will be to prove the following Proposition: \begin{prop}\label{prop:L2bounds} Consider a setting as in Theorem \ref{thm:FloerBounds}. Then for every pair $a,b\in \R$ the corresponding space $\mathcal{M}^\Gamma(a,b)$ is bounded in the $L^2\times\R$-norm. \end{prop} \begin{proof} By Lemma \ref{lem:ActBound} there exist $\mathfrak{a,e, y}>0$, such that for every $u:=(v,\eta)\in \mathcal{M}^\Gamma(a,b)$ we have \begin{align*} \|\eta\|_{L^\infty} \leq \mathfrak{y}, \qquad\|\mathcal{A}^{H_0-h_s}( u)\|_{L^{\infty}} &\leq\mathfrak{a},\\ \textrm{and}\qquad \|\nabla^{J_s} \mathcal{A}^{H_0-h_s}(u)\|_{L^{2}(\mathbb{R}\times [0,1])}^{2}& \leq \mathfrak{e}. \end{align*} Moreover, by \eqref{DefE} the convergence of the integral $$ \|\nabla \mathcal{A}^{H_0-h_s}(u)\|_{L^{2}(\mathbb{R}\times [0,1])}^{2} \leq \mathfrak{e}\|J\|_{\infty}^2, $$ implies that for every $\varepsilon>0$ there exists $s\in \R$ small enough such that\linebreak $\|\nabla \mathcal{A}^{H_0-h_s}(u(s))\|_{L^{2}\times\mathbb{R}}<\varepsilon$. This ensures that for all $s \in \mathbb{R}$ the following value $\tau_\varepsilon(s)$ is well defined and finite $$ \tau_\varepsilon(s): = \sup\left\lbrace \tau \leq s\ \Big|\ u(\tau) \in \mathcal{B}_{h_\tau}(\mathfrak{a},\mathfrak{y}, \varepsilon)\right\rbrace. $$ Note that for all $\tau \in [\tau_\varepsilon, s]$ we have $\|\nabla \mathcal{A}^{H_0-h_\tau-c}(u(\tau))\|_{L^{2}\times\mathbb{R}}\geq \varepsilon$, thus we can estimate \begin{gather} \varepsilon^2 |s-\tau_\varepsilon(s)| \leq\|\nabla \mathcal{A}^{H_0-h_s}(u)\|_{L^{2}(\mathbb{R}\times [0,1])}^{2}\leq\mathfrak{e}\|J\|_{\infty}^2,\nonumber\\ |s-\tau_\varepsilon(s)|\leq\frac{ \mathfrak{e}\|J\|_{\infty}^2}{\varepsilon^2}.\label{sBoundEpsi} \end{gather} On the other hand, we know that \begin{gather*} c:=\inf_{s\in[0,1]}\{h_s-dh_s(p\partial_p)\}>0,\\ \sup_{s\in[0,1]}\|h_s\|_{L^\infty} < +\infty , \qquad\textrm{and} \qquad \sup_{s\in[0,1]}\|\nabla h_s\|_{L^2} < +\infty, \end{gather*} therefore by Remark \ref{rem:smallDeriv} we know that the set \begin{equation}\label{DefBGamma} \mathcal{B}^\Gamma(\mathfrak{a},\mathfrak{y},\varepsilon):=\bigcup_{s\in [0,1]}\mathcal{B}_{h_s}(\mathfrak{a},\mathfrak{y},\varepsilon)\subseteq \mathscr{H}_{q_0,q_1}\times \R, \end{equation} is bounded in $L^\infty\times \R$- and $L^2\times \R$-norm. In fact, we can use the same bounds as in Lemmas \ref{lem:qBound} and \ref{lem:pBound} denoting \begin{align*} \mathfrak{h} & :=\sup_{s\in[0,1]}\|h_s\|_{W^{1,\infty}},\\ \mathfrak{q} & := \min\{|q_0|,|q_1|\}+2 \left(3\varepsilon + \frac{\mathfrak{a}}{\sqrt{2c}}+\mathfrak{y h}\right),\\ \mathfrak{p} & := 2\mathfrak{q}+\sqrt{2 \left( \mathfrak{h}+c+\varepsilon\right)}. \end{align*} Now using \eqref{sBoundEpsi} together with Lemma \ref{lem:qBound} we obtain \begin{align} |q(s,t)| & \leq |q(\tau_\varepsilon(s),t)|+\int_{\tau_\varepsilon(s)}^s |\partial_s q(\tau,t)|d\tau,\nonumber\\ & \leq |q(\tau_\varepsilon(s),t)|+\int_{\tau_\varepsilon(s)}^s |\partial_s v(\tau,t)|d\tau,\nonumber\\ \|q(s)\|_{L^2}& \leq \|q \circ \tau_0(s)\|_{L^2}+\left( \int_0^1\left| \int_{\tau_0(s)}^s\partial_sv d\tau\right|^2dt\right)^{\frac{1}{2}}\nonumber\\ & \leq \|q \circ \tau_0(s)\|_{L^2}+\sqrt{|s-\tau_0(s)|}\left( \int_0^1\int_{\tau_0(s)}^s\left|\partial_sv\right|^2d\tau dt\right)^{\frac{1}{2}}\nonumber\\ & \leq \|v \circ \tau_0(s)\|_{L^2}+\frac{\sqrt{\mathfrak{e}}\|J\|_\infty}{\varepsilon}\ \|\nabla^{J_s}\A^{H_0-h_s}\|_{L^2(\R\times[0,1])}\nonumber\\ & \leq \mathfrak{q}+\frac{\mathfrak{e}\|J\|_\infty}{\varepsilon}. \label{qL2Bound} \end{align} Analogously, using Lemma \ref{lem:pBound} we obtain \begin{align} \|p(s)\|_{L^2}& \leq \mathfrak{p} +\frac{\mathfrak{e}\|J\|_\infty}{\varepsilon},\label{pL2Bound}\\ \|v(s)\|_{L^2}& \leq \mathfrak{q}+\mathfrak{p} +\frac{\mathfrak{e}\|J\|_\infty}{\varepsilon}\label{vL2Bound}. \end{align} \end{proof} \subsection{The maximum principle} In this section we will explain how to use Aleksandrov’s maximum principle to find $L^\infty$-bounds on the Floer trajectories outside $\B(a, \varepsilon)$, following the argument of Abbondandolo and Schwartz in \cite{Abbon2009}. The contents of this section are word to word from the last author's paper with Pasquotto \cite{Pasquotto2017}, but we include it here for the completeness of the argument. \begin{thm}(\textbf{Aleksandrov's maximum principle})\\ Let $\Omega$ be a domain in $\mathbb{R}^{2}$ and let $\rho:\Omega \to \mathbb{R}$ be a function in $C^{2}(\Omega)\cap C^(\overline{\Omega})$ function satisfying the elliptic differential inequality $$ \triangle \rho + \langle h, \nabla \rho \rangle \geq f, $$ where $g\in L^2(\Omega,\R^2)$ and $f\in L_{\operatorname{loc}}^1(\Omega)$. Then there exists $C>0$ which depends only on $\operatorname{diam}\Omega$ and $\|g\|_{L^2(\Omega)}$, such that $$ \sup_{\Omega} \rho \leq \sup_{\partial \Omega}\rho +C\left(\|g\|_{L^{2}(\Omega)}\right)\|f^{-}\|_{L^{2}(\Omega)}, $$ provided $g$ and the negative part of $f$ are in $L^{2}(\Omega)$. \end{thm} In order to apply Aleksandrov's maximum principle and find $L^{\infty}$ bounds on the Floer trajectories, one first has to construct a function $F$ with compact level sets, whose composition with a Floer trajectory $u= (v,\eta)\in \mathscr{H}_{q_0,q_1}\times \R$ satisfies the elliptic differential inequality $$ \triangle (F\circ v) + \langle g, \nabla (F\circ v) \rangle \geq f $$ outside of the set of infinitesimal action derivation $\mathcal{B}^\Gamma(\mathfrak{a},\mathfrak{y},\varepsilon)$, i.e. on every connected component $\Omega$ of $$ \Omega\subseteq \left(\R \setminus u^{-1}\left(\mathcal{B}^\Gamma(\mathfrak{a},\mathfrak{y},\varepsilon)\right)\right)\times[0,1]. $$ Having such an inequality, one can apply the Aleksandrov maximum principle, which gives us $$ \sup_{\Omega} (F\circ v) \leq \sup_{\partial \Omega}(F\circ v)+ C(\|g\|_{L^{2}(\Omega)})\|f^{-}\|_{L^{2}(\Omega)}, $$ provided $g$ and the negative part of $f$ are in $L^{2}(\Omega)$. The core of this method is to find a function satisfying all the required properties. The classical approach is to use plurisubharmonic functions. \begin{define} Let $(M,\omega)$ be a symplectic manifold and $J$ a compatible almost complex structure. Then a $C^{2}$ function $F:M\to\mathbb{R}$ is called plurisubharmonic if $$ -dd^{\mathbb{C}}F=\omega, $$ where $d^{\mathbb{C}}F=dF\circ J$. \end{define} \begin{rem}\label{rem:pluri} Let $(M, \omega)$ be a symplectic manifold and let $J$ be a compatible almost complex structure on $M$. Then a function $F:M\to\mathbb{R}$ is plurisubharmonic if and only if its gradient with respect to $g$ is a Liouville vector field. \end{rem} The reason one uses plurisubharmonic functions is because their composition with a $2$-dimensional curve $v:\Omega \to M,\Omega \subseteq \R^2$ satisfies $$ -dd^\C (F \circ v)=\triangle (F \circ v) ds\wedge dt. $$ In particular, if $v$ is a $J$-holomorphic curve then the elliptic inequality is trivially satisfied. Unfortunately, in the case of Floer trajectories proving the elliptic inequality is a little more complicated. One has to investigate how the plurisubharmonic function interacts with the Hamiltonian vector field, in particular one needs to understand the functions $dF(X_H)$ and $d^{\mathbb{C}}F(X_H)$, which appear if we calculate $d^{\mathbb{C}}(F\circ u)$. The proof of the following Lemma you can find in \cite{Pasquotto2017} as a part of the proof of Proposition 7.1: \begin{lem} Let $H$ be a Hamiltonian function on an exact symplectic manifold $(M, \omega=d\lambda)$ and let $J$ be a compatible almost complex structure on $M$. Let $u: \R \times [0,1]\to M \times \R$ be a Floer trajectory satisfying $\partial_s u =\nabla \A^H(u)$ (with constant $H$ and $J$). Then for a plurisubharmonic function $F: M \to \R$ we have \begin{equation}\label{Laplace} \begin{aligned} \triangle (F \circ v) & = \|\partial_{s}v\|^{2}+\eta \left(dH +d(d^{\mathbb{C}}F(X_H))+d^\mathbb{C}(dF(X_H))\right)(\partial_{s}v)\\ & +\eta^2 d(dF(X_H))(X_H)+\partial_{s}\eta\ d^{\mathbb{C}}F(X_H). \end{aligned} \end{equation} \end{lem} \subsection{The $L^\infty$ bounds} In this subsection we will apply the Maximum Principle explained in the previous subsection to the setting of the Floer trajectories from $\mathcal{M}^\Gamma(a,b)$ to establish the $L^\infty$-bounds. \vspace*{.25cm} \noindent \textit{Proof of Theorem \ref{thm:FloerBounds}:} Let $\Gamma:={(h_s,J_s)}_{s\in \R}$ be a smooth homotopy of Hamiltonians $h_s\in\mathcal{H}$ and almost complex structures $J_s\in C^\infty([0,1]\times \R, \mathcal{J}(\mathcal{V},\mathbb{J}))$ satisfying \eqref{inqGamma}. Here $\mathcal{V}\subseteq T^*\R^2$ is the open, but pre-compact subset defined in Theorem \ref{thm:FloerBounds} with the property that for all $s\in [0,1]$ $\supp dh_s\subseteq \mathcal{V}$ and $J_s\big|_{T^*\R^2\setminus \mathcal{J}}\equiv \mathbb{J}$. Let $c>0$ be as in \eqref{Defc}. If we fix $a,b \in \R$ then by Lemma \ref{lem:ActBound} we know that there exists $\mathfrak{y,a,e}>0$, which depend only on $a,b,c$ and $\|J\|_\infty$ such that \begin{equation}\label{EAEtaBounds} \begin{aligned} \sup\left\lbrace|\eta(s)|\ |\ (v,\eta)\in \mathcal{M}^\Gamma(a,b), \ s\in \R\right\rbrace & \leq \mathfrak{y},\\ \sup\left\lbrace|\A^{H_0-h_s-c}(u)|\ |\ u\in \mathcal{M}^\Gamma(a,b)\right\rbrace & \leq \mathfrak{a},\\ \sup\left\lbrace\|\nabla^{J_s}\A^{H_0-h_s-c}(u)\|_{L^2(\R\times[0,1])\times\R}\ |\ u\in \mathcal{M}^\Gamma(a,b)\right\rbrace & \leq \mathfrak{e}. \end{aligned} \end{equation} On the other hand, if we fix $\varepsilon>0$ then by Proposition \ref{prop:smallDerivSetBound} and Remark \ref{rem:smallDeriv} we know that the set $\mathcal{B}^\Gamma(\mathfrak{a},\mathfrak{y},\varepsilon)$, defined in \eqref{DefBGamma}, is bounded in the $L^\infty\times\R$ norm and the bounds depend only on $a,b,\varepsilon$ and $\Gamma$. Consequently there exists a compact subset $K_\varepsilon\subseteq T^*\R^2$, such that \begin{equation}\label{DefKepsi} v([0,1])\subseteq K_\varepsilon\qquad\textrm{for all}\qquad (v,\eta)\in \B^\Gamma(\mathfrak{a},\mathfrak{y},\varepsilon). \end{equation} Without loss of generality, we assume that $\overline{\mathcal{V}} \subseteq K_\varepsilon$. Let us fix a Floer trajectory $u=(v,\eta)\in \mathcal{M}^\Gamma(a,b)$ and denote a connected component \begin{equation}\label{Omega} \Omega \subseteq \left(\mathbb{R} \times [0,1]\right)\setminus v^{-1}(K_\varepsilon). \end{equation} By assumption $\supp dh_s \subseteq \overline{V} \subseteq K_\varepsilon$, hence $dh_s \circ v |_{\Omega}\equiv 0$ for all $s\in [0,1]$. This means that instead of $H-h_s$ we can use the Hamiltonian $H_0-\mathbf{c}_s$ for some positive function $\mathbf{c}_s>0$ on $v(\Omega)$, which will simplify our computations. More precisely, for $(s,t)\in \Omega$ the Floer trajectory $(v,\eta)$ satisfies \begin{equation}\label{FloerH0} \partial_sv = \mathbb{J}(\partial_t v - \eta X_{H_0}). \end{equation} Naturally, $\mathfrak{c}_s\leq \sup_{s\in[0,1]}\|h_s\|_{L^\infty}$. By the convergence of the integral $$\|\nabla\A^{H_0-h_s}(u)\|_{L^2(\R\times[0,1])\times\R} \leq \|J\|_{L^\infty}\sqrt{\mathfrak{e}},$$ we know that $\lim_{s\to\pm\infty}u(s)\in \B^\Gamma(\mathfrak{a},\mathfrak{y},\varepsilon)$. Therefore, $$v(\partial\Omega)\subseteq K_\varepsilon\cup T^*_{q_0}\R^2\cup T^*_{q_1}\R^2$$ Consequently, we have $\sup_{\partial \Omega}|q(s,t)|\leq \sup_{K_\varepsilon}|q|$. Unfortunately, we do not have a uniform bound on $\sup_{\partial\Omega}|p|$. Therefore, we will need to treat the cases of the position and momenta coordinates separately. We introduce the following functions on $T^*\R^2$: \begin{equation}\label{DefF} F_1:= \frac{1}{2}|q|^2\qquad \textrm{and}\qquad F_2:=\frac{1}{2}|p|^2, \end{equation} Both of the functions $F_1$ and $F_2$ are plurisubharmonic, since their corresponding gradients $q\partial_q$ and $p\partial_p$ are Liouville vector fields (see Remark \ref{rem:pluri}). In Lemma \ref{lem:LaplaceF} we will show that on $\Omega$ the functions $F_1\circ v$ and $F_2\circ v$ satisfy \begin{align*} \triangle (F_1 \circ v) &\geq -\frac{1}{2}\eta^2 F_1\circ v+ \partial_{s}\eta \left(F_2\circ v -H_0\circ v\right)=:f_1(s,t),\\ \triangle (F_2 \circ v)&\geq -\frac{1}{2}\eta^2F_1\circ v-\partial_s\eta\left(F_2\circ v+H_0\circ v\right)=:f_2(s,t). \end{align*} In Lemma \ref{lem:f1W11Bound} and Lemma \ref{lem:f2W11Bound} we will prove that $f_1$ and $f_2$, respectively, are bounded in the $W^{1,1}$-norm on $\Omega$ and the bounds depend only on $a,b,c,\varepsilon$ and $\Gamma$, but not on our choice of the Floer trajectory $u\in \mathcal{M}^\Gamma(a,b)$ or our choice of the connected component $\Omega\subseteq \left(\R\times[0,1] \right)\setminus v^{-1}(K_\varepsilon)$. By the continuity of the Sobolev embedding $W^{1,1}\hookrightarrow L^2$ we obtain $L^2$-bounds on $f_1$ and $f_2$. Since we know that $\sup_{\partial \Omega}|q(s,t)|\leq \sup_{K_\varepsilon}|q|$ we can now apply the Alexandrov maximum principle to obtain $$ \sup_{(s,t)\in\R\times[0,1]}F_1\circ v \leq \sup_{K_\varepsilon}f_1\circ v+C\|f_1\|_{L^2(\Omega)}, $$ and the right-hand side of the inequality does not depend on our choice of the Floer trajectory $u\in \mathcal{M}^\Gamma(a,b)$ or our choice of the connected component $\Omega\subseteq \left(\R\times[0,1] \right)\setminus v^{-1}(K_\varepsilon)$. In other words, we obtain that the set $$ \{\pi \circ v \ |\ u=(v,\eta)\in\mathcal{M}^\Gamma(a,b)\}, $$ is bounded in $\R^2$. Now we would like to establish the $L^\infty$-bounds on the $p$-variable of the Floer trajectories. To establish uniform bounds on $F_2 \circ v$ for the Floer trajectory $u=(v,\eta)$ we extend the domain of the Floer trajectory $v:\R\times[0,1]\to T^*\R^2$ to a cylinder $\R\times \left([-1,1]/-1\sim 1\right)$ in the following way \begin{equation}\label{barv} \bar{v}(s,t):=\begin{cases} v(s,-t) & \textrm{for}\quad t\leq 0,\\ v(s,t) & \textrm{for}\quad t\geq 0. \end{cases} \end{equation} If we extend the domain $\Omega$ to the cylinder $\R\times \left([-1,1]/-1\sim 1\right)$ in the following way \begin{align} &\Theta:=\Omega\cup\{(s,-t)\ |\ (s,t)\in \Omega\},\label{Theta}\\ then \qquad &\bar{v}(\partial\Theta)\subseteq K_\varepsilon\qquad\textrm{and}\qquad v(\Omega) =\bar{v}(\Theta). \nonumber \end{align} In particular, we have that $\sup_{\Omega}f\circ v=\sup_\Theta f\circ \bar{v}$ and $\|f\circ \bar{v}\|_{L^2(\Theta)}=2\|f\circ v\|_{L^2(\Omega)}$ for any smooth function $f\in T^*\R^2$. After proving in Lemma \ref{lem:F2smooth} that $F_2\circ \bar{v}$ is in $C^2(\Theta)$ we can apply the Aleksandrov maximum principle and obtain \begin{align*} \sup_{(s,t)\in\R\times[0,1]}F_2\circ v & \leq \sup_{\partial\Theta}F_2\circ\bar{v}+C\|f_2\circ \bar{v}\|_{L^2(\Theta)}\\ & \leq \sup_{K_\varepsilon}f_2\circ v+2C\|f_2\circ v\|_{L^2(\Omega)}. \end{align*} Since the established bounds do not depend on the choice of $u\in\mathcal{M}^\Gamma(a,b)$ or the choice of the connected component $\Omega\subseteq \left(\R\times[0,1] \right)\setminus v^{-1}(K_\varepsilon)$, this concludes the proof that $\mathcal{M}^\Gamma(a,b)$ is bounded in the $L^\infty\times\R$-norm. \hfill $\square$ \vspace*{.25cm} To complete the proof of Theorem \ref{thm:FloerBounds} we need to prove the following technical lemmas: \begin{lem}\label{lem:LaplaceF} Consider the standard symplectic space $(T^*\R^2, \omega_0)$ with the Hamiltonian $H_0$ defined as in \eqref{DefH0} and functions $F_1$ and $F_2$ defined in \eqref{DefF}. Let $u:\R \to \mathscr{H}_{q_0,q_1}\times \R$, $u=(v,\eta)$ be a Floer trajectory corresponding to the action functional $\A^{H_0-c}$, i.e. satisfying the relation \eqref{FloerH0}. Then \begin{align*} \triangle (F_1 \circ v) & \geq -\frac{1}{2}\eta^2 F_1\circ v+ \partial_{s}\eta \left(F_2\circ v -H_0\circ v \right),\\ \triangle (F_2 \circ v) & \geq -\frac{1}{2}\eta^2 F_1\circ v-\partial_s\eta\left(F_2\circ v+H_0\circ v\right). \end{align*} \end{lem} \begin{proof} Using \eqref{DefH0} we can calculate that \begin{align} X_{H_0} & = (p_1+q_2)\partial_{q_1}+(p_2-q_1)\partial_{q_2}+p_2\partial_{p_1}-p_1\partial_{p_2},\nonumber\\ dF_1 (X_{H_0}) & = q_1p_1+q_2p_2,\nonumber\\d^\mathbb{C}(dF_1(X_{H_0})) & = qd^\mathbb{C}p+pd^\mathbb{C}q=pdp-qdq=dF_2-dF_1,\nonumber\\ dF_2 (X_{H_0}) & = 0,\label{dF2XH}\\ d^\C F_1 (X_{H_0}) & = qdp(X_{H_0})=q_1p_2-q_2p_1 = F_2-H_0 ,\label{dCF1XH}\\ d^\C F_2 (X_{H_0}) & =-\left(|p|^2 +q_1p_2-q_2p_1\right) = -F_2-H_0,\label{dCF2XH}\\ d(dF_1 (X_{H_0}))(X_{H_0}) & = |p|^2=2F_2,\nonumber\\ \big(dH_0+d(d^\C F_1 (X_{H_0}))&+d^\C \left( dF_1(X_{H_0})\right)\big) = 2dF_2-dF_1,\nonumber\\ \big(dH_0+d(d^\C F_2 (X_{H_0}))&+d^\C \left( dF_2(X_{H_0})\right)\big) = -dF_1.\nonumber \end{align} Plugging these qualities into \eqref{Laplace} for a Floer trajectory $u=(v,\eta)$ satisfying the relation \eqref{FloerH0} we can calculate \begin{align*} \triangle (F_1 \circ v) & = \|\partial_{s}v\|^{2}+\eta \left(dH_0 +d(d^{\mathbb{C}}F_1(X_{H_0}))+d^\mathbb{C}(dF_1(X_{H_0}))\right)(\partial_{s}v)\\ & +\eta^2 d(dF_1(X_{H_0}))(X_{H_0})+\partial_{s}\eta\ d^{\mathbb{C}}F_1(X_{H_0})\\ & = \|\partial_{s}v\|^{2}+\eta \left(2pdp-qdq\right)(\partial_{s}v)+\eta^2|p|^2+ \partial_{s}\eta \left(\frac{1}{2}|p|^2-H_0\circ v \right)\\ & \geq -\eta^2 \frac{1}{4}|q|^2+ \partial_{s}\eta \left(\frac{1}{2}|p|^2-H_0 \right)\\ & = -\frac{1}{2}F_1\circ v+ \partial_{s}\eta \left(F_2\circ v-H_0\circ v \right),\\ \triangle (F_2 \circ v) & = \|\partial_{s}v\|^{2}+\eta \left(dH_0 +d(d^{\mathbb{C}}F_2(X_{H_0}))+d^\mathbb{C}(dF_2(X_{H_0}))\right)(\partial_{s}v)\\ & +\eta^2 d(dF_2(X_{H_0}))(X_{H_0})+\partial_{s}\eta\ d^{\mathbb{C}}F_2(X_{H_0})\\ & = \|\partial_{s}v\|^{2}-\eta qdq(\partial_sv)-\partial_s\eta\left( \frac{1}{2}|p|^2+H_0\right)\\ & \geq -\frac{1}{4}\eta^2|q|^2-\partial_s\eta\left(\frac{1}{2}|p|^2+H_0\right)\\ & = -\frac{1}{2}\eta^2F_1\circ v-\partial_s\eta\left(F_2\circ v+H_0\circ v\right) \end{align*} \end{proof} \begin{lem}\label{lem:F2smooth} Consider the standard symplectic space $(T^*\R^2, \omega_0)$ with the Hamiltonian $H_0$ defined as in \eqref{DefH0}. Let $u:\R \to \mathscr{H}_{q_0,q_1}\times \R$, $u=(v,\eta)$ be a Floer trajectory associated to $\A^{H_0-\mathbf{c}_s}$, i.e. satisfying the relation \eqref{FloerH0} and let $\bar{v}:\R\times([-1,1]/-1\sim 1) \to T^*\R^2$ be the extension of $v$ defined in \eqref{barv}. Then the composition $F_2\circ\bar{v}$ with the function $F_2:T^*\R^2\to\R$, $F(q,p)=\frac{1}{2}|p|^2$ is $C^2$. \end{lem} \begin{proof} First, observe that $\bar{v}$ is continuous. Moreover, $\bar{v}$ is smooth on\linebreak $\R\times\left((-1,0)\cup(0,1)\right)$. Therefore, $F_2\circ \bar{v}$ is also everywhere continuous and smooth on $\R\times\left((-1,0)\cup(0,1)\right)$. What is left to check is that $F_2\circ \bar{v}$ is $C^2$ on $\R\times \{-1,0,1\}$. Note that $$ \frac{d}{dt} (F_2\circ \bar{v})(s,t)=\begin{cases} dF_2( \partial_tv)(s,t) & \textrm{for}\quad t\geq 0,\\ -dF_2(\partial_tv)(s,-t) & \textrm{for}\quad t\leq 0. \end{cases} $$ Hence, the function $F_2\circ\bar{v}$ is $C^1$ if and only if \begin{equation} dF_2( \partial_tv)(s,0)=dF_2( \partial_tv)(s,1)=0\qquad \forall\ s\in \R. \label{pdpPartialtv} \end{equation} By assumption $v: \R \to \mathscr{H}_{q_0,q_1}$, so $v(s,0)\equiv q_0$ and $v(s,1)\equiv q_1$ for all $s\in\R$. Consequently, \begin{equation} dq_i(\partial_sv)(s,0)=dq_i(\partial_sv)(s,1)=0\qquad \textrm{for}\quad i=0,1 \quad \textrm{and}\quad \forall\ s\in \R. \label{dqPartialsv} \end{equation} Now using the Floer equation \eqref{FloerH0} for the Hamiltonian $H_0$ and the relation \eqref{dF2XH} we can calculate for $t\geq 0$ \begin{align*} dF_2( \partial_tv) & = d^\mathbb{C}F_2(\partial_sv)+\eta dF_2(X_{H_0})=-pdq(\partial_sv). \end{align*} Plugging \eqref{dqPartialsv} into the equation above we obtain \eqref{pdpPartialtv}, which proves that $F_2\circ \bar{v}$ is $C^1$ everywhere. To prove that $F_2\circ \bar{v}$ is $C^2$ we first calculate: \begin{align*} \frac{d^2}{dt^2}F_2\circ v & = D^2F_2(\partial_t v,\partial_t v)+dF_2(\partial_{tt}v)= \eta^2 D^2F_2(X_{H_0},X_{H_0})+dF_2(\partial_{tt}v)\\ &=\eta^2|p|^2+dF_2(\partial_{tt}v). \end{align*} Obviously $|p|^2$ does not depend on the sign of $\partial_tv$. On the other hand, the function $\partial_{tt}\bar{v}= \partial_{tt}v$ is continuous, since the two negative signs cancel each other. Therefore, the sum of these two functions is also continuous on the whole cylinder $\R\times([-1,1]/-1\sim 1)$. Now $\frac{d^2}{dsdt}F_2\circ \bar{v}$ is well defined and continuous on the whole $\R\times((-1,0)\cup(0,1))$. On the other hand by \eqref{pdpPartialtv} we obtain that $\frac{d^2}{dsdt}F_2\circ v (s,0)=\linebreak\frac{d^2}{dsdt}F_2\circ v (s,1)=0$, thus $\frac{d^2}{dsdt}F_2\circ \bar{v}$ is a continuous function on the whole cylinder $\R\times([-1,1]/-1\sim 1)$. Finally, we observe that $\frac{d^2}{ds^2}(F_2\circ v (s,t))=\frac{d^2}{ds^2}(F_2\circ v (s,-t))$ as it does not depend on the sign of the $t$-variable. \end{proof} \begin{lem} Let $\Omega \subseteq \left(\mathbb{R} \times [0,1]\right)\setminus v^{-1}(K_\varepsilon)$ be a connected component as defined in \eqref{Omega}. Then the following values are well-defined and finite: \begin{equation}\label{Defs0s1} s_0 :=\inf\{s\ |\ (s,t)\in \Omega\}\qquad \textrm{and}\qquad s_1 :=\sup\{s\ |\ (s,t)\in \Omega\}. \end{equation} Moreover, $s_0$ and $s_1$ satisfy \begin{equation}\label{s0s1Bound} |s_1-s_0|\leq \frac{\mathfrak{e}\|J\|^2_\infty}{\varepsilon^2}, \end{equation} \end{lem} \begin{proof} By Lemma \ref{lem:ActBound} we obtain uniform bounds on the action, energy and the $\eta$-parameter for all $u\in\mathcal{M}^\Gamma(a,b)$. By the convergence of the integral $$\|\nabla\A^{H_0-h_s}(u)\|_{L^2(\R\times[0,1])\times\R} \leq \|J\|_{L^\infty}\sqrt{\mathfrak{e}},$$ we know that $\lim_{s\to\pm\infty}u(s)\in \B^\Gamma(\mathfrak{a},\mathfrak{y},\varepsilon)$. For the compact subset $K_\varepsilon\subseteq T^*\R^2$ defined in \eqref{DefKepsi} we have $v([0,1])\subseteq K_\varepsilon$ for all $(v,\eta)\in\mathcal{M}^\Gamma(a,b)$. Since\linebreak $\Omega \subseteq \left(\mathbb{R} \times [0,1]\right)\setminus v^{-1}(K_\varepsilon)$, thus the values $s_0$ and $s_1$ as in \eqref{Defs0s1} are well defined and finite. Moreover, for all $s\in (s_0,s_1)$, $u(s)\notin \mathcal{B}^\Gamma(\mathfrak{a},\mathfrak{y}, \varepsilon)$, therefore by \eqref{sBoundEpsi} we can estimate $$ |s_1-s_0|\leq \frac{\mathfrak{e}\|J\|^2_\infty}{\varepsilon^2}, $$ where $\mathfrak{e}=\|J\|_\infty(b-a+\frac{c}{3}\mathfrak{y})$ as in Lemma \ref{lem:ActBound}. \end{proof} \begin{lem}\label{lem:f1L1Bound} Consider the setting as in Theorem \ref{thm:FloerBounds}. Fix $a,b\in \R$ and let $u\in \mathcal{M}^\Gamma(a,b)$ be a Floer trajectory. Let $\Omega \subseteq \left(\mathbb{R} \times [0,1]\right)\setminus v^{-1}(K_\varepsilon)$ be a connected component as defined in \eqref{Omega}. Then the function $$ f_1(s,t):= -\frac{1}{2}\eta^2(s) F_1 \circ v(s,t) + \partial_{s}\eta(s) \left(F_2-H_0\right)\circ v(s,t). $$ is bounded in the $L^1(\Omega)$ norm and the bounds do not depend on $u$ or $\Omega$. \end{lem} \begin{proof} Let $s_0$ and $s_1$ be as in \eqref{Defs0s1}. Denote $\overline{\Omega}:= [s_0,s_1]\times[0,1]$. Naturally,\linebreak $\Omega\subseteq \overline{\Omega}$. Before we estimate $\|f_1\|_{L^1(\Omega)}$ we first use the Cauchy-Schwartz inequality together with \eqref{DefE}, \eqref{qL2Bound}, \eqref{pL2Bound} and \eqref{s0s1Bound} to do the following estimates: \begin{align} \|\partial_s\eta\|_{L^1(\Omega)}& \leq \|\partial_s\eta\|_{L^1(\overline{\Omega})}\leq \sqrt{|s_1-s_0|}\|\nabla^{J_s} \A^{H_0-h_s}(u(s))\|_{L^2\times\R}\nonumber\\ &\leq \frac{\mathfrak{e}\|J\|_\infty}{\varepsilon}\label{partialEtaBound}\\ \|\partial_s\eta(H_0-\mathbf{c}_s)\|_{L^1(\Omega)}&\leq \|\partial_s\eta (H_0-h_s)\circ v\|_{L^1(\overline{\Omega})}=\|\partial_s\eta\|_{L^2(\overline{\Omega})}^2\nonumber\\ & \leq \|\nabla^{J_s} \A^{H_0-h_s}(u(s))\|_{L^2\times\R}^2\leq \mathfrak{e}\label{H0Bound}\\ \|q\|_{L^2(\Omega)} & \leq \|q \|_{L^2(\overline{\Omega})}\leq \sqrt{|s_1-s_0|} \sup_{s\in\R}\|q(s)\|_{L^2([0,1])}\nonumber\\ & \leq \frac{\sqrt{\mathfrak{e}}\|J\|_\infty}{\varepsilon}\left(\mathfrak{q}+\frac{\mathfrak{e}\|J\|_\infty}{\varepsilon}\right)\label{qOmegaBound}\\ \|p\|_{L^2(\Omega)} & \leq \frac{\sqrt{\mathfrak{e}}\|J\|_\infty}{\varepsilon}\left(\mathfrak{p}+\frac{\mathfrak{e}\|J\|_\infty}{\varepsilon}\right)\label{pOmegaBound}\\ \|\partial_s\eta\ p^2\|_{L^1(\Omega)} & \leq \|\partial_s\eta\|_{L^1(\overline{\Omega})}\sup_{s\in\R}\|p(s)\|_{L^2([0,1])}^2 \leq\frac{\mathfrak{e}\|J\|_\infty}{\varepsilon}\left(\mathfrak{p}+\frac{\mathfrak{e}\|J\|_\infty}{\varepsilon}\right)^2 \label{partialEtaPBound} \end{align} Using the bounds obtained above and the inequality \eqref{eqEta} we can calculate: \begin{align*} \|f_1\|_{L^1(\Omega)}& \leq \frac{1}{4}\mathfrak{y}^2 \|q\|_{L^2(\Omega)}^2+\frac{1}{2}\|\partial_s\eta\ p^2\|_{L^1(\Omega)}+\|\partial_s\eta(H_0-\mathbf{c}_s)\|_{L^1(\Omega)}+\|\mathbf{c}_s\partial_s\eta\|_{L^1(\Omega)}\\ &\leq \frac{\mathfrak{e}\mathfrak{y}^2\|J\|_\infty^2}{4\varepsilon^2}\left(\mathfrak{q}+\frac{\mathfrak{e}\|J\|_\infty}{\varepsilon}\right)^2+\frac{\mathfrak{e}\|J\|_\infty^2}{2\varepsilon^2}\left( \mathfrak{p}+\frac{\mathfrak{e}\|J\|_\infty}{\varepsilon}\right)^2+\mathfrak{e}\left(\frac{\mathfrak{h}\|J\|_\infty}{\varepsilon}+1\right). \end{align*} \end{proof} \begin{lem}\label{lem:f1W11Bound} Consider the setting as in Lemma \ref{lem:f1L1Bound}. Then the function $f_1$ is bounded in the $W^{1,1}$-norm and the bounds do not depend on $u$ or $\Omega$. \end{lem} \begin{proof} To prove that $f_1\in W^{1,1}(\Omega)$ we first need to calculate its derivatives: \begin{align*} \partial_sf_1 & = - \eta\partial_s\eta\ F_1+\partial_{ss}\eta\ d^\mathbb{C}F_1(X_{H_0})+\left(\partial_s\eta\ d(F_2-H_0)-\frac{1}{2}\eta^2 dF_1\right)(\partial_sv)\\ \partial_tf_1 & = \left(\partial_s\eta\ d(F_2-H_0)-\frac{1}{2}\eta^2 dF_1\right)(\mathbb{J}\partial_sv+\eta X_{H_0})\\ & = \left(\partial_s\eta\ d^\mathbb{C}(F_2-H_0)-\frac{1}{2}\eta^2 d^\mathbb{C} F_1\right)(\partial_sv)-\frac{1}{2}\eta^3dF_1(X_{H_0}), \end{align*} where the last equality comes from the fact that $dF_2(X_{H_0})(q,p)=0$. Using \eqref{qL2Bound}, \eqref{vL2Bound} and \eqref{s0s1Bound} we can estimate: \begin{align} \|dF_1(X_{H_0})\|_{L^1(\Omega)}& = \|q_1p_1+q_2p_2\|_{L^1(\Omega)}\leq\frac{1}{2}\| v\|_{L^2(\Omega)}^2\leq \frac{1}{2}|s_1-s_0|\sup_{s\in\R}\|v(s)\|_{L^2([0,1])}^2\nonumber\\ & \leq \frac{\mathfrak{e}\|J\|_\infty^2}{2\varepsilon^2}\left( \mathfrak{q}+\mathfrak{p}+\frac{\mathfrak{e}\|J\|_\infty}{\varepsilon}\right)^2\label{eq0}\\ \|dF_1(\partial_sv)\|_{L^1(\Omega)}& \leq \|\nabla F_1\|_{L^2(\Omega)}\|\partial_sv\|_{L^1(\Omega)}\leq \|q\|_{L^2(\overline{\Omega})}\|\nabla^{J_s}\A^{H_0-h_s}(u)\|_{L^2\times\R}\nonumber\\ & \leq \frac{\mathfrak{e}\|J\|_\infty}{\varepsilon}\left(\mathfrak{q}+\frac{\mathfrak{e}\|J\|_\infty}{\varepsilon}\right)\label{eq1} \end{align} \begin{align} \|&\partial_s\eta\ d(F_2 -H_0)(\partial_sv)\|_{L^1(\Omega)} \leq \int_{s_0}^{s_1}|\partial_s\eta|\int_0^1|\nabla (F_2-H_0)||\partial_sv|dtds\nonumber\\ &= \int_{s_0}^{s_1}|\partial_s\eta|\int_0^1|v||\partial_sv|dtds \leq \sup_{s\in\R}\|v(s)\|_{L^2([0,1])}\int_{s_0}^{s_1}|\partial_s\eta|\|\partial_sv(s)\|_{L^2([0,1])}ds\nonumber\\ & \leq\sup_{s\in\R}\|v(s)\|_{L^2([0,1])}\|\nabla^{J_s}\A^{H_0-h_s}(u)\|_{L^2\times\R}^2 \leq \mathfrak{e}\left( \mathfrak{q}+\mathfrak{p}+\frac{\mathfrak{e}\|J\|_\infty}{\varepsilon}\right)\label{eq2} \end{align} Similarly, we have \begin{align} \|d^\C F_1(\partial_sv)\|_{L^1(\Omega)} & \leq \frac{\mathfrak{e}\|J\|_\infty}{\varepsilon}\left(\mathfrak{q}+\frac{\mathfrak{e}\|J\|_\infty}{\varepsilon}\right),\label{eq3}\\ \|\partial_s\eta\ d^\C(F_2-H_0)(\partial_sv)\|_{L^1(\Omega)} &\leq \mathfrak{e}\left( \mathfrak{q}+\mathfrak{p}+\frac{\mathfrak{e}\|J\|_\infty}{\varepsilon}\right). \end{align} Using the estimates above and \eqref{eqEta} we can combine them to obtain a uniform $L^1$-bound on $\partial_tf_1$: \begin{align*} \|\partial_tf_1\|_{L^1(\Omega)} &\leq \|\partial_s\eta d^\C( F_2-H_0)(\partial_sv)\|_{L^1(\Omega)}+\frac{1}{2}\|\eta^2 d^\C F_1(\partial_sv)\|_{L^1(\Omega)}+\frac{1}{2}\|\eta^3 dF_1(X_{H_0})\|_{L^1(\Omega)}\\ & \leq \mathfrak{e}\mathfrak{p}+ \mathfrak{e}\left(1+\frac{\mathfrak{ey}^2\|J\|_\infty}{2\varepsilon}\right)\left(\mathfrak{q}+\frac{\mathfrak{e}\|J\|_\infty}{\varepsilon}\right)+\frac{\mathfrak{ey}^3\|J\|_\infty^2}{4\varepsilon^2}\left( \mathfrak{q}+\mathfrak{p}+\frac{\mathfrak{e}\|J\|_\infty}{\varepsilon}\right)^2 \end{align*} This gives us a bound on $\|\partial_tf_1\|_{L^1(\Omega)}$, which does not depend on the choice of the Floer trajectory $u$ or the connected component $\Omega$. To estimate $\|\partial_sf_1\|_{L^1(\Omega)}$ we first use \eqref{qL2Bound} and \eqref{partialEtaBound} to estimate \begin{align} \|\partial_s\eta\ F_1\circ v\|_{L^1(\Omega)} & \leq \|\partial_s\eta\ F_1\circ v\|_{L^1(\overline{\Omega})} = \frac{1}{2}\int_{s_0}^{s_1}|\partial_s\eta|\|q(s)\|_{L^2([0,1])}^2ds\nonumber\\ & \leq \frac{1}{2}\|\partial_s\eta\|_{L^1(\overline{\Omega})}\sup_{s\in\R}\|q(s)\|_{L^2([0,1])}^2\nonumber\\ &\leq \frac{\mathfrak{e}\|J\|_\infty}{2\varepsilon}\left(\mathfrak{q}+\frac{\mathfrak{e}\|J\|_\infty}{\varepsilon}\right)^2\label{partialsEtaF1} \end{align} Secondly, we analyze $\partial_{ss}\eta$: $$ \partial_{ss}\eta = -\int_0^1d(H_0-h_s)(\partial_sv)dt-\int_0^1 \partial_sh_s (v)dt. $$ Using \eqref{inqGamma}, \eqref{pL2Bound} and \eqref{vL2Bound} we can estimate: \begin{align} \|\partial_{ss}\eta\|_{L^1(\Omega)} & = \int_{\Omega}\bigg|\int_0^1\left(d(H_0-h_s)(\partial_sv)+\partial_sh_s(v)\right)dt\bigg|dt ds\nonumber\\ & \leq \|d(H_0-h_s)(\partial_sv)\|_{L^1(\overline{\Omega})}+\|\partial_sh_s\|_{L^\infty}\nonumber\\ &\leq \left(\|\nabla H_0\|_{L^2(\overline{\Omega})}+\|\nabla h_s\|_{L^2(\overline{\Omega})}\right)\|\partial_sv\|_{L^2(\overline{\Omega})}\|\partial_sh_s\|_{L^\infty}\nonumber\\ & \leq \left(\|v+p\|_{L^2(\overline{\Omega})}+\|\nabla h_s\|_{L^2(\overline{\Omega})}\right)\|\nabla^{J_s}\A^{H_0-h_s}(u)\|_{L^2\times\R}+\|\partial_sh_s\|_{L^\infty}\nonumber\\ &\leq \sqrt{\mathfrak{e}|s_1-s_0|}\left(\sup_{s\in\R}\|v(s)\|_{L^2([0,1])}+\sup_{s\in\R}\|p(s)\|_{L^2([0,1])}+\sup_{s\in\R}\|\nabla h_s\|_{L^\infty}\right)+\|\partial_sh_s\|_{L^\infty}\nonumber\\ &\leq \frac{\mathfrak{e}\|J\|_\infty}{\varepsilon}\left(\mathfrak{h}+ \mathfrak{q}+2\mathfrak{p}+\frac{2\mathfrak{e}\|J\|_\infty}{\varepsilon}\right)+\frac{c}{3}\label{partialSS} \end{align} Now we will use the Floer equations, the Cauchy-Schwartz inequality together with \eqref{DefE}, \eqref{sBoundEpsi}, \eqref{qL2Bound}, \eqref{pL2Bound} and \eqref{dCF1XH} to calculate the following: \begin{align} \|\partial_{ss}\eta\ d^\mathbb{C}F_1(X_{H_0})\|_{L^1(\Omega)} & \leq \|\partial_{ss}\eta\ d^\mathbb{C}F_1(X_{H_0})\|_{L^1(\overline{\Omega})} =\int_{s_0}^{s_1}|\partial_{ss}\eta|\int_0^1|q_1p_2-q_2p_1|dtds\nonumber\\ & \leq \frac{1}{2}\int_{s_0}^{s_1}|\partial_{ss}\eta|(\|q(s)\|_{L^2([0,1])}^2+\|p(s)\|_{L^2([0,1])}^2)ds\nonumber\\ &\leq \frac{1}{2}\|\partial_{ss}\eta\|_{L^1(\overline{\Omega})}\sup_{s\in\R}\|v(s)\|_{L^2([0,1])}^2\nonumber\\ & \leq \left(\frac{\mathfrak{e}\|J\|_\infty}{\varepsilon}\left( \frac{1}{2}(\mathfrak{h}+\mathfrak{q})+\mathfrak{p}+\frac{\mathfrak{e}\|J\|_\infty}{\varepsilon}\right)+\frac{c}{3}\right)\left(\mathfrak{q}+\mathfrak{p}+\frac{\mathfrak{e}\|J\|_\infty}{\varepsilon}\right)^2\label{EtaSSdCF1XH0} \end{align} Using the estimates above together with \eqref{eqEta}, \eqref{eq1}, \eqref{eq2} and \eqref{partialsEtaF1} we can estimate: \begin{align*} \|\partial_sf_1\|_{L^1(\Omega)} & \leq \mathfrak{y}\|\partial_s\eta\ F_1\circ v\|_{L^1(\Omega)}+\|\partial_{ss}\eta d^\mathbb{C}F_1(X_{H_0})\|_{L^1(\Omega)}+\frac{1}{2}\mathfrak{y}^2\|d F_1(\partial_sv)\|_{L^1(\Omega)}\\ & +\|\partial_s\eta\ d(F_2-H)(\partial_sv)\|_{L^1(\Omega)} \\ & \leq \frac{\mathfrak{e y}\|J\|_\infty}{2\varepsilon}\left(\mathfrak{q}+\frac{\mathfrak{e}\|J\|_\infty}{\varepsilon}\right)^2 + \frac{\mathfrak{e y}^2\|J\|_\infty}{2\varepsilon}\left(\mathfrak{q}+\frac{\mathfrak{e}\|J\|_\infty}{\varepsilon}\right)\\ & + \left(\frac{\mathfrak{e}\|J\|_\infty}{\varepsilon}\left( \frac{1}{2}(\mathfrak{h}+\mathfrak{q})+\mathfrak{p}+\frac{\mathfrak{e}\|J\|_\infty}{\varepsilon}\right)+\frac{c}{3}\right)\left(\mathfrak{q}+\mathfrak{p}+\frac{\mathfrak{e}\|J\|_\infty}{\varepsilon}\right)^2\\ & +\mathfrak{e}\left( \mathfrak{q}+\mathfrak{p}+\frac{\mathfrak{e}\|J\|_\infty}{\varepsilon}\right) \end{align*} This gives us a bound on $\|\partial_sf_1\|_{L^1(\Omega)}$, which does not depend on the choice of the Floer trajectory $u$ or the connected component $\Omega$. \end{proof} \begin{lem}\label{lem:f2L1Bound} Consider the setting as in Theorem \ref{thm:FloerBounds}. Fix $a,b\in \R$ and let $u\in \mathcal{M}^\Gamma(a,b)$ be a Floer trajectory. Let $\Omega \subseteq \left(\mathbb{R} \times [0,1]\right)\setminus v^{-1}(K_\varepsilon)$ be a connected component as defined in \eqref{Omega}. Then the function $$ f_2(s,t):= -\frac{1}{2}\eta^2(s)F_1\circ v(s,t)-\partial_s\eta(s)\left(F_2+H_0\right)\circ v(s,t). $$ is bounded in the $L^1(\Omega)$ norm and the bounds do not depend on $u$ or $\Omega$. \end{lem} \begin{proof} To estimate $\|f_2\|_{L^1(\Omega)}$ we use the bounds obtained in \eqref{eqEta}, \eqref{partialEtaBound}, \eqref{H0Bound}, \eqref{qOmegaBound} and \eqref{partialEtaPBound} to calculate: \begin{align*} \|f_2\|_{L^1(\Omega)} & \leq \frac{\mathfrak{y}^2}{4}\|q\|^2_{L^2(\Omega)} + \frac{1}{2}\|\partial_s\eta\ p^2\|_{L^2(\Omega)}+\|\partial_s\eta (H_0-\mathfrak{c}_s)\|_{L^2(\Omega)}+\|\mathfrak{c}_s\partial_s\eta\|_{L^1(\Omega)}\\ &\leq \frac{\mathfrak{ey}^2\|J\|_\infty^2}{4\varepsilon^2}\left(\mathfrak{q}+\frac{\mathfrak{e}\|J\|_\infty}{\varepsilon}\right)^2+ \frac{\mathfrak{e}}{2\varepsilon}\left(\mathfrak{p}+\frac{\mathfrak{e}\|J\|_\infty}{\varepsilon}\right)^2+\mathfrak{e}\left(\frac{c}{\varepsilon}+\|J\|_\infty^2\right). \end{align*} \end{proof} \begin{lem}\label{lem:f2W11Bound} Consider the setting as in Lemma \ref{lem:f2L1Bound}. Then the function $f_2$ is bounded in the $W^{1,1}$-norm and the bounds do not depend on $u$ or $\Omega$. \end{lem} \begin{proof} To prove that $f_2\in W^{1,1}(\Omega)$ we first need to calculate its derivatives: \begin{align*} \partial_sf_2 & = - \eta\partial_s\eta F_1 + \partial_{ss}\eta\ d^\mathbb{C}F_2(X_{H_0})-\left(\partial_s\eta\ d(F_2+H_0)+\frac{1}{2}\eta^2 dF_1\right)(\partial_sv)\\ \partial_tf_2 & = -\left(\partial_s\eta\ d(F_2+H_0)+\frac{1}{2}\eta^2 dF_1\right)(\mathbb{J}\partial_sv+\eta X_{H_0})\\ & = -\left(\partial_s\eta\ d^\mathbb{C}(F_2+H_0)+\frac{1}{2}\eta^2 d^\mathbb{C} F_1\right)(\partial_sv)-\frac{1}{2}\eta^3 dF_1(X_{H_0}), \end{align*} where the last equality comes from \eqref{dF2XH}. Let $s_0$ and $s_1$ be as in \eqref{Defs0s1}. Denote $\overline{\Omega}:= [s_0,s_1]\times[0,1]$. Naturally, $\Omega\subseteq \overline{\Omega}$. Before we estimate $\|\partial_t f_2\|_{L^1(\Omega)}$ we will first recall \eqref{dCF1XH} and \eqref{dCF2XH} which give us the following relation \begin{equation} d^\mathbb{C}F_1(X_{H_0})=F_2-H_0=2F_2+d^\mathbb{C}F_2(X_{H_0}). \label{F2-H0} \end{equation} Further on, we use \eqref{DefE}, \eqref{pL2Bound}, \eqref{pOmegaBound} and \eqref{eq2} to calculate the following bounds: \begin{align*} \|\partial_s\eta\ dF_2(\partial_sv)\|_{L^1(\Omega)}& \leq \|\partial_s\eta\ dF_2(\partial_sv)\|_{L^1(\overline{\Omega})}\\ & \leq \int_{s_0}^{s_1}|\partial_s\eta|\int_0^1|p(s,t)||\partial_sv(p,s)|dtds\\ &\leq \int_{s_0}^{s_1}|\partial_s\eta|\|p(s)\|_{L^2([0,1])}\|\partial_s v(s)\||_{L^2([0,1])}ds\\ &\leq \sup_{s\in \R}\|p(s)\|_{L^2([0,1])}\|\partial_s\eta\|_{L^2(\R)}\|\partial_sv\|_{L^2(\R\times[0,1])}\\ &\leq \mathfrak{e}\left(\mathfrak{p}+\frac{\mathfrak{e}\|J\|_\infty}{\varepsilon}\right). \end{align*} This, together with \eqref{F2-H0} gives us: \begin{align} \|\partial_s\eta\ d(F_2+H_0)(\partial_sv)\|_{L^1(\Omega)} & \leq 2\|\partial_s\eta\ dF_2(\partial_sv)\|_{L^1(\Omega)}+\|\partial_s\eta\ d(F_2-H_0)(\partial_sv)\|_{L^1(\Omega)}\nonumber\\ &\leq \mathfrak{e}\left( \mathfrak{q}+3\mathfrak{p}+2\frac{\mathfrak{e}\|J\|_\infty}{\varepsilon}\right)\label{eq4} \end{align} Similarly, we have $$ \|\partial_s\eta\ d^\mathbb{C}(F_2+H_0)(\partial_sv)\|_{L^1(\Omega)}\leq \mathfrak{e}\left( \mathfrak{q}+3\mathfrak{p}+2\frac{\mathfrak{e}\|J\|_\infty}{\varepsilon}\right). $$ Using the bounds obtained above together with \eqref{eqEta}, \eqref{eq0} and \eqref{eq3} we can estimate: \begin{align*} \|\partial_tf_2\|_{L^1(\Omega)} & \leq \|\partial_s\eta\ d^\mathbb{C}(F_2+H_0)(\partial_s v)\|_{L^1(\Omega)}+\frac{1}{2}\mathfrak{y}^2 \left(\|d^\mathbb{C} F_1(\partial_sv)\|_{L^1(\Omega)}+\mathfrak{y} \|dF_1(X_{H_0})\|_{L^1(\Omega)}\right)\\ &\leq \mathfrak{e}\left( \mathfrak{q}+3\mathfrak{p}+2\frac{\mathfrak{e}\|J\|_\infty}{\varepsilon}\right)+\frac{\mathfrak{ey}^2\|J\|_\infty}{2\varepsilon}\left(\mathfrak{q}+\frac{\mathfrak{e}\|J\|_\infty}{\varepsilon}\right)\\ &+\frac{\mathfrak{ey}^3\|J\|_\infty^2}{4\varepsilon^2}\left( \mathfrak{q}+\mathfrak{p}+\frac{\mathfrak{e}\|J\|_\infty}{\varepsilon}\right)^2. \end{align*} Before we estimate $\|\partial_sf_2\|_{L^2}$ we first use \eqref{pL2Bound}, \eqref{partialSS} and \eqref{EtaSSdCF1XH0} to calculate the following bounds: \begin{align*} \|\partial_{ss}\eta\ F_2\circ v \|_{L^1(\Omega)} & \leq \|\partial_{ss}\eta\ F_2\circ v \|_{L^1(\overline{\Omega)}}=\frac{1}{2}\int_{s_0}^{s_1}|\partial_{ss}\eta|\int_0^1|p(s,t)|^2dtds\\ &\leq \frac{1}{2}\|\partial_{ss}\eta\|_{L^1(\overline{\Omega})}\sup_{s\in\R}\|p(s)\|_{L^2([0,1])}^2\\ & \leq \left(\frac{\mathfrak{e}\|J\|_\infty}{\varepsilon}\left(\frac{1}{2}(\mathfrak{h}+ \mathfrak{q})+\mathfrak{p}+\frac{\mathfrak{e}\|J\|_\infty}{\varepsilon}\right)+\frac{c}{3}\right)\left(\mathfrak{p}+\frac{\mathfrak{e}\|J\|_\infty}{\varepsilon}\right)^2\\ \|\partial_{ss}\eta\ d^\mathbb{C}F_2(X_{H_0})\|_{L^1(\Omega)} & = \|\partial_{ss}\eta\left(d^\mathbb{C}F_1(X_{H_0})-2F_2\right)\|_{L^1(\Omega)}\\ & \leq \|\partial_{ss}\eta\ d^\mathbb{C}F_1(X_{H_0})\|_{L^1(\Omega)}+2\|\partial_{ss}\eta\ F_2\circ v\|_{L^1(\Omega)}\\ & \leq \left(\frac{\mathfrak{e}\|J\|_\infty}{\varepsilon}\left( \frac{1}{2}(\mathfrak{h}+\mathfrak{q})+\mathfrak{p}+\frac{\mathfrak{e}\|J\|_\infty}{\varepsilon}\right)+\frac{c}{3}\right)\\ &\cdot \left(\left(\mathfrak{q}+\mathfrak{p}+\frac{\mathfrak{e}\|J\|_\infty}{\varepsilon}\right)^2+2\left(\mathfrak{p}+\frac{\mathfrak{e}\|J\|_\infty}{\varepsilon}\right)^2\right) \end{align*} Finally, using the estimates above together with \eqref{eqEta}, \eqref{partialEtaPBound}, \eqref{eq2}, \eqref{partialsEtaF1} and \eqref{eq4} we can calculate. \begin{align*} \|\partial_sf_2\|_{L^1(\Omega)} & \leq \mathfrak{y}\|\partial_s\eta\ F_1\|_{L^1(\Omega)}+\|\partial_s\eta\ d(F_2+H_0)(\partial_sv)\|_{L^1(\Omega)}\\ & +\frac{\mathfrak{y}^2}{2} \|dF_1(\partial_sv)\|_{L^1(\Omega)}+\|\partial_{ss}\eta\left(F_2+H_0\right)\|_{L^1(\Omega)}\\ & \leq \frac{\mathfrak{ey}\|J\|_\infty}{2\varepsilon}\left(\mathfrak{q}+\frac{\mathfrak{e}\|J\|_\infty}{\varepsilon}\right)^2+\mathfrak{e}\left( \mathfrak{q}+3\mathfrak{p}+2\frac{\mathfrak{e}\|J\|_\infty}{\varepsilon}\right)\\ &+\frac{\mathfrak{ey}^2}{2}\left( \mathfrak{q}+\mathfrak{p}+\frac{\mathfrak{e}\|J\|_\infty}{\varepsilon}\right)\\ & +\left(\frac{\mathfrak{e}\|J\|_\infty}{\varepsilon}\left( \frac{1}{2}(\mathfrak{h}+\mathfrak{q})+\mathfrak{p}+\frac{\mathfrak{e}\|J\|_\infty}{\varepsilon}\right)+\frac{c}{3}\right)\\ &\cdot \left(\left(\mathfrak{q}+\mathfrak{p}+\frac{\mathfrak{e}\|J\|_\infty}{\varepsilon}\right)^2+2\left(\mathfrak{p}+\frac{\mathfrak{e}\|J\|_\infty}{\varepsilon}\right)^2\right) \end{align*} This concludes the proof that $f_2$ is bounded in the $W^{1,1}$-norm and the bounds do not depend on the choice of the Floer trajectory $u\in \mathcal{M}^\Gamma(a,b)$ or the connected component $\Omega\subseteq \R\times[0,1]\setminus v^{-1}(K_\varepsilon)$, but only on the constants $a,b, \varepsilon$ and the smooth homotopy $\Gamma$. \end{proof} \subsection{The Lagrangian Rabinowitz Floer homology of the Copernican Hamiltonian} Having proven the $L^\infty$-bounds on the Floer trajectories we are finally ready to prove the main theorem of this paper: \vspace*{.25cm} \textit{Proof of Theorem \ref{thm:LRFH}:} Let $H_0:T^*\R^2\to\R$ be the Copernican Hamiltonian defined in \eqref{DefH0} and let $\mathcal{H}$ be the set of perturbations as defined in \eqref{DefHset}. Fix a pair $q_0,q_1\in \R^2$. Let $\{K_{n,m}\}_{n,m\in\mathbb{N}}$ be the sequence of compact sets as in Lemma \ref{lem:H0Chord}, such that \begin{equation}\label{Knm} \begin{aligned} K_{n,m}\subseteq K_{n+1,m}\quad &\textrm{and} \quad \bigcup_{n\in\mathbb{N}}K_{n,m}=T^*\R^2\quad &&\forall\ m\in \mathbb{N},\\ K_{n,m}\subseteq K_{n,m+1} \quad &\textrm{and} \quad T_{q_0}^*\R^2, T_{q_1}^*\R^2\subseteq \bigcup_{m\in\mathbb{N}}K_{n,m}\quad &&\forall\ n\in\mathbb{N}. \end{aligned} \end{equation} Fix a Hamiltonian $h_0\in \mathcal{H}$ and denote $c(h_0):=\inf (h-dh(p\partial_p))$. Naturally, $c(h_0)>0$. Let $K_{n,m}$ be any of the compact sets in the sequence, such that $\supp dh_0\subseteq K_{n,m}$ and $\|h_0\|_{L^\infty}<m$. Denote $$ \mathcal{H}_{h_0}(K_{n,m}):=\left\lbrace\begin{array}{c|c} & dh \in C_0^\infty(K_{n,m}), \quad \|h\|_{L^\infty}<\frac{c(h_0)}{50},\\ h \in C^\infty(T^*\R^2) & h_0+h \in \mathcal{H}, \quad \|h_0+h\|_{L^\infty}<m,\\ & \inf((h+h_0)-d(h+h_0)(p\partial_p))>\frac{1}{2} c(h_0). \end{array}\right\rbrace. $$ This way $\mathcal{H}_{h_0}(K_{n,m})$ is an open neighbourhood of $0$ in $\{h\in C^\infty(T^*\R^2)\ |\ dh\in C_0^\infty(K_{n,m})\}$. By Lemma \ref{lem:H0Chord} the critical set of the action functional $\A^{H-h_0}_{q_0,q_1}$ is continuously compact in $(K_{n,m}, \mathcal{H}_{h_0}(K_{n,m}))$. On the other hand, by construction if $h\in \mathcal{H}_{h_0}(K_{n,m})$, then $h_0+h\in \mathcal{H}$, hence by Lemma \ref{lem:nonEmpty} every Hamiltonian $H_0-h_0-h$ satisfies the condition \eqref{nonEmpty}. This proves that the compact set $K_{n,m}$ and the set of perturbations $\mathcal{H}_{h_0}(K_{n,m})$ satisfy assumptions 1. and 2. of Theorem \ref{thm:DefLRFH} for the Hamiltonian $H_0-h_0$. Now, if we take any open, pre-compact set $\mathcal{V}\subseteq T^*\R^2$, such that $K_{n,m}\subseteq \mathcal{V}$, any two almost complex structures $J_0,J_1\in \mathcal{J}(\mathcal{V},\mathbb{J})$ and any $h\in\mathcal{H}_{h_0}(K_{n,m})$, then every smooth homotopy $\Gamma:=\{(h_0+h,J_s)\}_{s\in\R}$, with $J_s\in C^\infty([0,1]\times\R,\mathcal{J}(\mathcal{V},\mathbb{J}))$ such that \begin{equation}\label{AC} J_s=\begin{cases} J_0 & \textrm{for}\quad s\leq 0,\\ J_1 & \textrm{for}\quad s\geq 1. \end{cases} \end{equation} will automatically satisfy condition \eqref{inqGamma}. Consequently, by Theorem \ref{thm:FloerBounds} for every pair $(a,b)\in \R^2$ the corresponding space $\mathcal{M}^\Gamma(a,b)$ of Floer trajectories is bounded in $L^\infty$-norm. Therefore, we can apply Theorem \ref{thm:DefLRFH} and conclude that there exists a residual set {$\mathcal{H}^{\reg}_{h_0}(K_{n,m})\subseteq \mathcal{H}_{h_0}(K_{n,m})$}, such that\linebreak for every $h\in \mathcal{H}^{\reg}_{h_0}(K_{n,m})$ the Lagrangian Rabinowitz Floer homology\linebreak $\LRFH_*(\A^{H_0-h_0-h}_{q_0,q_1})$ is well defined and independent of the choice of the almost complex structure. Now if we fix $h_1,h_2\in \mathcal{H}^{\reg}_{h_0}(K_{n,m})$ then we can construct a homotopy $\Gamma$ satisfying \eqref{inqGamma}. Indeed, let $J_0\in \mathcal{J}^{\reg}_{h_0}$ and $J_1\in \mathcal{J}^{\reg}_{h_2}$ will be two almost complex structures close enough, so that there exists a smooth homotopy $\{J_s\}_{s\in\R}$ with $J_s\in C^\infty([0,1]\times\R,\mathcal{J}(\mathcal{V},\mathbb{J}))$ satisfying \eqref{AC} and $\sup_{s\in\R}\|J_s\|_{L^\infty}<\frac{1}{\sqrt{6c(h_0)}}$. Moreover, let $\chi\in C^\infty(\R)$ be a smooth function satisfying $\|\chi'\|_{L^\infty} \leq 2$ and \begin{equation}\label{chi} \chi(s)=\begin{cases} 0 & \textrm{for}\quad s\leq 0,\\ 1 & \textrm{for}\quad s\geq 1. \end{cases} \end{equation} Define $h_s:=h_0+h_1(1-\chi(s))+\chi(s)h_2$. Then the smooth homotopy $\Gamma=\{(h_s,J_s)\}_{s\in\R}$ satisfies \begin{align*} \|\partial_sh_s\|\left(\frac{4}{c(h_0)}+\|J\|_{L^\infty}^2\right) &\leq \|\chi'\|_{L^\infty}\|h_1-h_2\|_{L^\infty}\left(\frac{4}{c(h_0)}+\frac{1}{6c(h_0)}\right)\\ &< 2\cdot \frac{c(h_0)}{25}\cdot \frac{25}{6c(h_0)}=\frac{1}{3}. \end{align*} In other words, $\Gamma$ satisfies \eqref{inqGamma} and thus by Theorem \eqref{thm:FloerBounds} for every pair $(a,b)\in \R^2$ the corresponding space $\mathcal{M}^\Gamma(a,b)$ of Floer trajectories is bounded in $L^\infty$-norm. Therefore, we can apply Theorem \ref{thm:DefLRFH} and conclude that for every $h\in \mathcal{H}_{h_0}(K_{n,m})$ the Lagrangian Rabinowitz Floer homology is well-defined and isomorphic to $\LRFH_*(\A^{H_0-h_0}_{q_0,q_1})$. Taking all the sets of the form $\mathcal{H}_{h_0}(K_{n,m})$ with $h_0\in \mathcal{H}$ and $n,m\in\mathbb{N}$ gives us an open cover of $\mathcal{H}$. Since $\mathcal{H}$ is path connected and the Lagrangian Rabinowitz Floer homology is constant on every open set $\mathcal{H}_{h_0}(K_{n,m})$ a basic topological argument gives us that for any pair $h_0,h_1\in \mathcal{H}$ the respective homologies $\LRFH_*(\A^{H_0-h_0}_{q_0,q_1})$ and $\LRFH_*(\A^{H_0-h_1}_{q_0,q_1})$ are isomorphic. \hfill $\square$ \section{Positive Lagrangian Rabinowitz Floer homology} The aim of this section is to prove Theorem \ref{thm:posLRFH}. We will start by showing that the positive Lagrangian Rabinowitz homology of the Copernican Hamiltonian is well-defined and invariant of perturbations. Further on we will prove that for fixed endpoints of the chords and high enough energy, the Copernican Hamiltonian has exactly one Reeb chord with positive action. Consequently, the corresponding positive Lagrangian Rabinowitz Floer homology has only one generator. Finally, we will show that the generator's Maslov index is in fact $0$. \subsection{Definition and invariance of perturbations} First, we will prove that if the perturbation is small enough then there exists a homotopy, such that any associated Floer trajectory starting at a critical point with a positive action has to end at a critical point with positive action. \begin{lem}\label{lem:posGamma} Let $H_0$ be the Hamiltonian defined in \eqref{DefH0} and let $\mathcal{H}$ be the set of perturbations defined in \eqref{DefHset}. Fix two points $q_0,q_1 \in \R^2$ and a perturbation\linebreak $h_0\in \mathcal{H}$. Denote $c(h_0):=\inf(h_0-dh_0(p\partial_p))$. Let $K\subseteq T^*\R^2$ be a compact subset, such that $\supp dh_0 \subseteq K$ and for which there exists $\mathcal{O}_{h_0}(K)$ an open neighbourhood of $0$ in $\{h\in C^\infty(T^*\R^2)\ |\ dh \in C_0^\infty(K)\}$, satisfying: \begin{enumerate} \item $\{h_0+h\ |\ h\in \mathcal{O}_{h_0}(K)\}\subseteq \mathcal{H}$; \item The critical set of $ \A^{H_0-h_0}$ is continuously compact in $(K, \mathcal{O}_{h_0}(K))$; \item $\inf\left\lbrace\begin{array}{c|c} \inf((h_0+h)-d(h_0+h)(p\partial_p)) & h\in\mathcal{O}_{h_0}(K)\end{array} \right\rbrace >\frac{1}{2}c(h_0)$; \item $\delta(h_0):=\inf\left\lbrace\begin{array}{c|c} \A^{H_0-h_0-h}_{q_0,q_1}(x) & x\in \Crit^+ \A^{H_0-h_0-h}_{q_0,q_1}, \quad h\in\mathcal{O}_{h_0}(K)\end{array} \right\rbrace >0$. \end{enumerate} Fix $h \in \mathcal{O}_{h_0}(K)$ and denote $h_1:=h_0+h$. Let $\mathcal{V}\subseteq T^*\R^2$ be an open, but precompact subset, such that $K\subseteq \mathcal{V}$. Let $\Gamma:=\{(h_s, J_s)\}_{s\in \R}$ be a smooth homotopy of Hamiltonians $h_s\in h_0+\mathcal{O}_{h_0}(K)$ and $2$-parameter families of almost complex structures $J_s \in C^\infty([0,1]\times \R,\mathcal{J}(\mathcal{V},\mathcal{J}))$ constant in $s$ outside $[0,1]$ satisfying \eqref{AC} and such that \begin{equation}\label{inq2Gamma} \|\partial_{s}h_{s}\|_{L^{\infty}}\leq \frac{1}{3}\min\left\lbrace\left(\frac{4}{c(h_0)}+\|J\|_{L^{\infty}}^2\right)^{-1},\ \frac{c(h_0)}{2}\left(1+\frac{\sqrt{c(h_0)}}{2\delta(h_0)}\right)^{-1}\right\rbrace. \end{equation} Then for every $x\in \Crit^+\A^{H_0-h_0}_{q_0,q_1}$ and every $y\in \Crit\A^{H_0-h_1}_{q_0,q_1}$, such that $\F_\Gamma(x,y)\neq \emptyset$ we have $\A^{H-h_1}_{q_0,q_1}(y)>0$. \end{lem} \begin{proof} The following proof is an adjustment of the proof of \cite[Cor. 3.8]{CieliebakFrauenfelder2009} to our setting and it is similar to the proof of Lemma \ref{lem:Novikov} where we have proven the Novikov finiteness condition. Fix $x\in \Crit^+\A^{H_0-h_0}_{q_0,q_1}$ and $y\in \Crit\A^{H_0-h_1}_{q_0,q_1}$ and abbreviate: $$ a:= \A^{H_0-h_0}_{q_0,q_1}(x) \qquad\textrm{and}\qquad \A^{H_0-h_1}_{q_0,q_1}(y). $$ By assumption $a\geq \delta(h_0)$. Let $u=(v,\eta)\in \F_\Gamma(x,y)$. Since $h_0,h_1\in \mathcal{H}_c$ and $\Gamma$ satisfies \eqref{inqGamma}, thus our setting satisfies the assumptions of Lemma \ref{lem:ActBound}. Suppose $|b|\leq a$. Then $b-a\leq 0$ and \eqref{eqEta} gives \begin{align} \|\eta\|_{L^\infty} &\leq \frac{3}{2} \left(\frac{2}{c(h_0)}\max\{|a|,|b|\}+\frac{1}{\sqrt{c(h_0)}}+\|J\|_{L^{\infty}}^2 (b-a)\right)\nonumber\\ &\leq\frac{3}{c(h_0)}\left(a+\frac{\sqrt{c(h_0)}}{2}\right)\leq \frac{3a}{c(h_0)}\left(1+\frac{\sqrt{c(h_0)}}{2\delta(h_0)}\right).\label{etaBDdelta} \end{align} On the other hand, by equation \eqref{inqEnergy} we have $$ \|\nabla^{J_{s}} \mathcal{A}^{H_0-h_{s}}(u)\|_{L^{2}(\mathbb{R}\times [0,1])}^{2} \leq \|J\|_{L^{\infty}} (b-a + \|\eta\|_{L^{\infty}}\|\partial_{s}h_{s}\|_{L^{\infty}}). $$ Combined with \eqref{inq2Gamma} and \eqref{etaBDdelta}, we obtain the following estimate: $$ b \geq a- \|\eta\|_{L^{\infty}}\|\partial_{s}h_{s}\|_{L^{\infty}} \geq a \left( 1-\frac{3}{c(h_0)}\left(1+\frac{\sqrt{c(h_0)}}{2\delta(h_0)}\right)\|\partial_{s}h_{s}\|_{L^{\infty}}\right)\geq \frac{1}{2}a. $$ In particular, $\A^{H_0-h_1}_{q_0,q_1}(y)=b\geq \frac{1}{2}\delta(h_0)$. By assumption $h_1 \in \mathcal{O}_{h_0}(K)$, hence by Lemma \ref{lem:posAction} we infer that $\A^{H_0-h_1}_{q_0,q_1}(y)\geq\delta(h_0)$. This proves the claim under the assumption $|b|\leq a$. Now suppose $b< -a\leq -\delta(h_0)$. Then $b-a\leq 0$ and by \eqref{eqEta} we have \begin{align*} \|\eta\|_{L^\infty} &\leq \frac{3}{2} \left( \frac{2}{c(h_0)}\max\{|a|,|b|\}+\frac{1}{\sqrt{c(h_0)}}+\|J\|_{L^{\infty}}^2 (b-a)\right)\nonumber\\ &\leq \frac{3}{c(h_0)}\left(\frac{\sqrt{c(h_0)}}{2}-b\right)<\frac{-3b}{c(h_0)}\left(1+\frac{\sqrt{c(h_0)}}{2\delta(h_0)}\right) \end{align*} Combining it with \eqref{inqEnergy} and \eqref{inq2Gamma}, we obtain the following inequality: $$ a \leq b + \|\eta\|_{L^{\infty}}\|\partial_{s}h_{s}\|_{L^{\infty}} \leq b\left(1-\frac{3b}{c(h_0)}\left(1+\frac{\sqrt{c(h_0)}}{2\delta(h_0)}\right)\|\partial_{s}h_{s}\|_{L^{\infty}}\right)\leq \frac{1}{2}b<-\frac{1}{2}a, $$ which contradicts the assumption $a\geq \delta(h_0)>0$. That excludes the case $b<-a$ and proves the lemma. \end{proof} We are now ready to prove the first part of Theorem \ref{thm:posLRFH}, i.e. to prove that the positive Lagrangian Rabinowitz homology of the Copernican Hamiltonian is well-defined and invariant under compact perturbations. We will do that by showing that all conditions of Corollary \ref{cor:posLRFH} are satisfied in our setting. \begin{prop}\label{prop:posLRFH} Let $H_0$ be the Copernican Hamiltonian defined in \eqref{DefH0} and let $\mathcal{H}$ be the corresponding set of compact perturbations as in \eqref{DefHset}. Then for all $q_0,q_1\in \R^2$ and all $h\in \mathcal{H}$ the positive Lagrangian Rabinowitz Floer homology $\LRFH_*^+(\A^{H_0-h}_{q_0,q_1})$ is well defined and isomorphic to\linebreak $\LRFH_*^+(\A^{H_0-c}_{q_0,q_1})$ for any $c>0$. \end{prop} \begin{proof} Fix two points $q_0,q_1 \in \R^2$ and let $\{K_{n,m}\}_{n,m\in\mathbb{N}}$ be the sequence\linebreak of compact sets as in Lemma \ref{lem:H0Chord} satisfying \eqref{Knm}. Fix $h_0\in \mathcal{H}^{\reg}$ and denote $c(h_0):=\inf(h_0-dh_0(p\partial_p))>0$. Let $K_{n,m}\subseteq T^*\R^2$ be any compact set from the sequence, such that $\supp dh_0 \subseteq K_{n,m}$ and $\|h_0\|_{L^\infty}<m$. By Lemma \ref{lem:posAction} and \ref{lem:H0Chord} there exists an open neighbourhood $\mathcal{O}_{h_0}(K_{n,m})$ of $0$ in $\{h\in C^\infty(T^*\R^2)\ |\ dh\in C_0^\infty(K_{n,m})\}$, such that \begin{enumerate} \item $\{h_0+h\ |\ h\in \mathcal{O}_{h_0}(K_{n,m})\}\subseteq \mathcal{H}$; \item The critical set of $ \A^{H_0-h_0}$ is continuously compact in $(K_{n,m}, \mathcal{O}_{h_0}(K_{n,m}))$; \item For all $h\in\mathcal{O}_{h_0}(K_{n,m})$ we have $\inf((h_0+h)-d(h_0+h)(p\partial_p)) >\frac{1}{2}c(h_0)$; \item There exists $\delta(h_0)>0$, such that for all $h\in\mathcal{O}_{h_0}(K_{n,m})$ and all\linebreak $x\in \Crit^+ \A^{H_0-h_0-h}_{q_0,q_1}$ we have $\A^{H_0-h_0-h}_{q_0,q_1}(x)\geq \delta(h_0)$; \item For all $h\in \mathcal{O}_{h_0}(K_{n,m})$ we have $$ \|h_0-h\|_{L^\infty}<\frac{1}{2}c(h_0)\inf \left\lbrace\frac{1}{13}, \frac{1}{6}\left(1+\frac{\sqrt{c(h_0)}}{2\delta(h_0)}\right)^{-1}\right\rbrace. $$ \end{enumerate} We will show that for every $h\in \mathcal{O}_{h_0}(K_{n,m})$ the positive Lagrangian Rabinowitz Floer homology $\LRFH_*^+(\A^{H_0-h_0-h}_{q_0,q_1})$ is well defined and isomorphic to $\LRFH_*^+(\A^{H_0-h_0}_{q_0,q_1})$. Fix $h_1 \in\{h_0+h\ |\ h\in\mathcal{O}_{h_0}(K_{n,m})\}\cap \mathcal{H}^{\reg}$. Let $J_0\in \mathcal{J}^{\reg}_{h_0}$ and $J_1\in \mathcal{J}^{\reg}_{h_1}$ will be two almost complex structures close enough, so that there exists a smooth homotopy $\{J_s\}_{s\in\R}$ with $J_s\in C^\infty([0,1]\times\R,\mathcal{J,\mathbb{J}}))$ satisfying \eqref{AC} and $\sup_{s\in\R}\|J_s\|_{L^\infty}<\frac{1}{\sqrt{3c(h_0)}}$. Moreover, let $\chi\in C^\infty(\R)$ be a smooth function satisfying $\|\chi'\|_{L^\infty} \leq 2$ and \eqref{chi}. Define $h_s:=h_0(1-\chi(s))+\chi(s)h_1$. Then the smooth homotopy $\Gamma=\{(h_s,J_s)\}_{s\in\R}$ satisfies \eqref{inq2Gamma}. In particular, the homotopy $\Gamma$ satisfies \eqref{inqGamma}, thus by Lemma \ref{lem:Novikov} it satisfies the Novikov finiteness condition. Moreover, by Theorem \ref{thm:FloerBounds} for any pair\linebreak $a,b\in \R$ the space of Floer trajectories $\mathcal{M}^\Gamma(a,b)$ is bounded in $L^\infty$-norm. Finally, by Lemma \ref{lem:posGamma} we know that for every $x\in \Crit^+\A^{H_0-h_0}_{q_0,q_1}$ and every $y\in \Crit\A^{H_0-h_1}_{q_0,q_1}$, such that $\F_\Gamma(x,y)\neq \emptyset$ we have $\A^{H-h_1}_{q_0,q_1}(y)>0$. This means that for every $h_1 \in\{h_0+h\ |\ h\in\mathcal{O}_{h_0}(K_{n,m})\}\cap \mathcal{H}^{\reg}$ there exists a homotopy $\Gamma$ satisfying all three conditions of Corollary \ref{cor:posLRFH}. Hence for every $h \in\mathcal{O}_{h_0}(K_{n,m})$ the positive Lagrangian Rabinowitz Floer homology $\LRFH_*^+(\A^{H_0-h_0-h}_{q_0,q_1})$ is well defined and isomorphic to $\LRFH_*^+(\A^{H_0-h_0}_{q_0,q_1})$. Taking all the sets of the form $\mathcal{O}_{h_r}(K_{n,m})$ with $h_r\in \mathcal{H}^{\reg}$ and $n,m\in\mathbb{N}$ gives us an open cover of $\mathcal{H}$. Since $\mathcal{H}$ is path connected and the positive Lagrangian Rabinowitz Floer homology is constant on every open set $\mathcal{O}_{h_r}(K_{n,m})$ a basic topological argument gives us that for any pair $h_0,h_1\in \mathcal{H}$ the respective homologies $\LRFH_*^+(\A^{H_0-h_0}_{q_0,q_1})$ and $\LRFH_*^+(\A^{H_0-h_1}_{q_0,q_1})$ are isomorphic. \end{proof} \subsection{The critical set of the Copernican Hamiltonian} In this subsection we will analyse the Reeb chords corresponding to the Copernican Hamiltonian $H_0$ defined in \eqref{DefH0}. More precisely, we will show that the positive Lagrangian Rabinowitz action functional $\A^{H_0-c}_{q_0,q_1}$ has an odd number of Reeb chords with positive $\eta>0$ under specific conditions relating the energy level set of the Hamiltonian $H_0^{-1}(c)>0$ and the endpoints of Reeb chords $q_0,q_1\in \R^2$, $q_0\neq q_1$. \begin{prop}\label{prop:CritPM} Let $H_0$ be the Hamiltonian defined in \eqref{DefH0} and let $\A^{H_0-c}_{q_0,q_1}:\mathscr{H}_{q_0,q_1}\times\R \to \R$ be the Lagranian Rabinowitz action functional corresponding to the energy $c>0$ and a pair $(q_0,q_1)\in \R^4, q_0\neq q_1$. Then $\Crit\A^{H_0}_{q_0,q_1}$ has the following properties: \begin{enumerate} \item If $q_0\neq q_1$ and $|q_0||q_1|\leq 2c$ then $\#\Crit^+\A^{H_0-c}_{q_0,q_1}=\#\Crit^-\A^{H_0-c}_{q_0,q_1}=1$. \item For a fixed $c>0$ there exists a residual set $\mathcal{Q}^c\subseteq \R^4\setminus \Delta$, where\linebreak $\Delta:=\left\lbrace (q,q)\ |\ q\in \R^2\right\rbrace$, such that for all pairs $(q_0,q_1)\in \mathcal{Q}^c$ both\linebreak $\#\Crit^+\A^{H_0-c}_{q_0,q_1}$ and $\#\Crit^-\A^{H_0-c}_{q_0,q_1}$ are odd numbers. \item For a fixed pair $(q_0,q_1)\in \R^4,\ q_0\neq q_1$ there exists a residual set\linebreak $\mathcal{I}_{q_0,q_1}\subseteq \R_+$, such that for all $c\in \mathcal{I}_{q_0,q_1}$ both $\#\Crit^+\A^{H_0-c}_{q_0,q_1}$ and\linebreak $\#\Crit^-\A^{H_0-c}_{q_0,q_1}$ are odd numbers. \end{enumerate} \end{prop} \begin{proof} For clarity of the argument, we will divide the proof of this proposition in a sequence of lemmas. First recall that by equation~\eqref{Crit+} we have $$ \Crit^\pm\A^{H}_{q_0,q_1}=\left\lbrace (v,\eta)\in \Crit\A^{H}_{q_0,q_1}\ |\ \pm \eta >0\right\rbrace. $$ In Lemma \ref{lem:bijection} we will show that for each $c>0$ and each pair $(q_0,q_1)\in \R^4$ there exists a smooth function $f\in C^\infty(\R)$, such that $(v,\eta)\in \Crit\A^{H_0-c}_{q_0,q_1}$ if and only if $f(\eta)=0$. We also show that for $q_0\neq q_1$ the critical set of $\A^{H_0-c}_{q_0,q_1}$ is discrete. Denote by $Z(f)$ the set of roots of the function $f$, $Z(f):=\{\eta\in\R |\ f(\eta)=0\}$. In Lemma \ref{lem:2c>q0q1} we will consider pairs $(q_0,q_1)\in \R^4$, such that $|q_0||q_1|\leq 2c$ and show that for $q_0\neq q_1$ the corresponding function $f$ satisfies $\#\{\eta>0\ |\ f(\eta)=0\}=\#\{\eta<0\ |\ f(\eta)=0\}=1$, whereas for $q_0=q_1$ we have $Z(f)=\{0\}$. Denote $\Delta:=\{(q,q)\ |\ q\in \R^2\}\subseteq \R^4$. In Lemma \ref{lem:Res_q0q1} we prove that for a fixed $c>0$ there exists a residual set $\mathcal{Q}^c\subseteq \R^4\setminus \Delta$, such that for all pairs $(q_0,q_1)\in \mathcal{Q}^c$ the corresponding function $f$ satisfies: \begin{equation}\label{f'neq0} f'(\eta)\neq 0\qquad \forall\ \eta \in Z(f), \qquad \textrm{where}\quad Z(f):=\{\eta \in \R\ |\ f(\eta)=0\}. \end{equation} In Lemma \ref{lem:Res_c} we show that for a fixed pair $(q_0,q_1)\in \R^4, q_0\neq q_1$ there exists a residual set $\mathcal{I}_{q_0,q_1}\subseteq \R_+$, such that for all $c\in \mathcal{I}_{q_0,q_1}$ the corresponding function $f$ satisfies property \eqref{f'neq0}. In Lemma \ref{lem:Odd} we prove that if $c>0$ and $(q_0, q_1)\in \R^4, q_0\neq q_1$ are such that the corresponding function $f$ satisfies \eqref{f'neq0}, then $f$ has an odd number of roots on each positive and negative half line. This together with the bijection established in Lemma \ref{lem:bijection} concludes the proof. \end{proof} Before we prove the first lemma we will start by recalling the properties of the group of matrices corresponding to rotations: if we define \begin{equation} R(t) :=\left(\begin{array}{c c} \cos(t) & \sin (t) \\ -\sin(t)& \cos(t) \\ \end{array}\right).\label{DefR} \end{equation} then $\{R(t)\}_{t\in\R}$ form a group with the following properties: \begin{align} R(0)=\Id\quad \textrm{and}\quad R\left(\frac{\pi}{2}\right) = & \left(\begin{array}{c c} 0 & 1 \\ -1 & 0 \\ \end{array}\right),\nonumber\\ R(t)R(\tau)=R(t+\tau)\qquad & \forall\ t,\tau\in\R,\label{Radd}\\ R^{-1}(t)=R^T(t)=R(-t)\qquad & \forall\ t\in\R.\label{Rinv} \end{align} Moreover, we have \begin{align} R'(t) &= \left(\begin{array}{c c} -\sin(t) & \cos (t) \\ -\cos(t)& -\sin(t) \\ \end{array}\right)=\left(\begin{array}{c c} 0 & 1 \\ -1 & 0 \\ \end{array}\right) \left(\begin{array}{c c} \cos(t) & \sin (t) \\ -\sin(t)& \cos(t) \\ \end{array}\right)\nonumber\\ &= R\left(\frac{\pi}{2}\right)R(t)= R\left( t +\frac{\pi}{2}\right).\label{DerR} \end{align} In the following lemma we will show that the critical set of $\A^{H_0-c}_{q_0,q_1}$ consists of isolated points and, in case $q_0=q_1$ it is a circle. \begin{lem}\label{lem:bijection} Let $H_0$ be the Hamiltonian defined in \eqref{DefH0}. For every $c>0$ and every pair $q_0,q_1\in \R^2$ we define \begin{equation}\label{Deff} f(\eta):=-c\eta^2+\eta q_1^TR\left(\eta+\frac{\pi}{2}\right)q_0+\frac{1}{2}\left(|q_0|^2+|q_1|^2\right)-q_1^TR(\eta)q_0. \end{equation} If we denote \begin{equation}\label{DefZ(f)} Z(f):=\{\eta\in \R \ |\ f(\eta)=0\}, \end{equation} then the map $Z(f)\setminus\{0\}\ni \eta \longmapsto ( v_\eta, \eta),$ $$ \textrm{with}\qquad v_\eta(t) := \left(\begin{array}{c c} (1-t) R(t\eta) & t R(\eta(t-1))\\ -\frac{1}{\eta}R(t\eta) & \frac{1}{\eta} R(\eta(t-1)) \end{array}\right) \left(\begin{array}{c} q_0 \\ q_1 \end{array}\right). $$ defines a bijection between the set $Z(f)\setminus\{0\}$ and $\Crit\A^{H_0-c}_{q_0,q_1}\setminus (\{0\}\times \mathscr{H}_{q_0,q_1})$. \end{lem} \begin{proof} By definition a pair $(v,\eta)\in \Crit\A^{H_0-c}_{q_0,q_1}$ if and only if it satisfies the following three conditions: \begin{enumerate} \item The curve $v$ starts in the Lagrangian $T^*_{q_0}\R^2$ and ends in $T^*_{q_1}\R^2$; \item The curve $v$ is a Reeb chord and $\eta$ is its period; \item The curve $v$ lies on the level set $H_0^{-1}(c)$. \end{enumerate} Note that if $q_0\neq q_1$ we have $\Crit\A^{H_0-c}_{q_0,q_1}\cap\left(\mathscr{H}_{q_0,q_1}\times\{0\}\right)=\emptyset$. In case $q_0= q_1$ we always have a submanifold of constant solutions $$ \Crit\A^{H_0-c}_{q_0,q_1}\cap\left(\mathscr{H}_{q_0,q_0}\times\{0\}\right)=\{q_0\}\times\{p_0\in T^*_{q_0}\R^2\ |\ H_0(q_0,p_0)=c\}. $$ One can easily calculate that for a fixed $q_0\in \R^2$ the set $\{ p \in T^*\R^2\ |\ H_0(q_0,p)=c\}$ is a circle with origin at $-\mathbb{J}q_0$ and radius $\sqrt{|q_0|^2+2c}$: $$ H_0(q,p)=c \quad \iff \quad |p+ \mathbb{J}q_0|^2=2c+|q_0|^2. $$ From now on we will assume $\eta\neq 0$. Using \eqref{DefH0} and the notation from \eqref{DefR} we can express $H_0$ and $X_{H_0}$ in the following way: \begin{align} H_0(q,p) & = \frac{1}{2}(q,p) A (q,p)^T, \nonumber\\ X_{H_0}(q,p) &= \mathbb{J}A (q,p)^T\nonumber\\ \textrm{where}\qquad A & =\left(\begin{array}{c c} 0 & -R\left(\frac{\pi}{2}\right)\\ R\left(\frac{\pi}{2}\right) & \Id \end{array}\right).\label{DefMatrixA} \end{align} In particular, the flow $\varphi^t$ of the Hamiltonian vector field $X_{H_0}$ can be easily calculated to be \begin{equation} \varphi^t(q,p)=\Exp(t\mathbb{J}A)\left(\begin{array}{c} q \\ p \end{array}\right) = \left(\begin{array}{c c} R(t) & t R(t)\\ 0 & R(t) \end{array}\right) \left(\begin{array}{c} q \\ p \end{array}\right)\label{flow} \end{equation} By definition, a critical point $(v,\eta)\in \Crit\A^{H_0-c}_{q_0,q_1}$ satisfies $\varphi^\eta \circ v (0)=v(1)$. Moreover, we have $\pi \circ v(0)=q_0$ and $\pi\circ v(1)=q_1$. If we denote $v(0)=(q_0,p_0)$ and $v(1)=(q_1,p_1)$ and combine it with the matrix formula for the Hamiltonian flow, we obtain $$ \left(\begin{array}{c} q_1 \\ p_1 \end{array}\right)= \left(\begin{array}{c c} R(\eta) & \eta R(\eta)\\ 0 & R(\eta) \end{array}\right) \left(\begin{array}{c} q_0 \\ p_0 \end{array}\right). $$ Assuming, that $\eta\neq 0$ we can use \eqref{Rinv} to solve for $p_0$ and $p_1$: $$ \left(\begin{array}{c} p_0 \\ p_1 \end{array}\right) =\frac{1}{\eta} \left(\begin{array}{c c} - \Id & R(-\eta)\\ -R(\eta) & \Id \end{array}\right) \left(\begin{array}{c} q_0 \\ q_1 \end{array}\right). $$ Using the equation above we obtain the following formula for $v(0)$: \begin{equation}\label{q0p0} \left(\begin{array}{c} q_0 \\ p_0 \end{array}\right) = \left(\begin{array}{c c} \Id &0\\ -\frac{1}{\eta} & \frac{1}{\eta}R(-\eta) \end{array}\right) \left(\begin{array}{c} q_0 \\ q_1 \end{array}\right) \end{equation} Now, by combining \eqref{flow} and \eqref{q0p0} we can express a Reeb chord $v$ in terms of $q_0$ and $q_1$ in the following way: \begin{align*} v_\eta(t) & := \left(\begin{array}{c c} R(t\eta) & t \eta R(t\eta)\\ 0 & R(t\eta) \end{array}\right) \left(\begin{array}{c c} \Id & 0\\ -\frac{1}{\eta} & \frac{1}{\eta}R(-\eta) \end{array}\right) \left(\begin{array}{c} q_0 \\ q_1 \end{array}\right)\\ & = \left(\begin{array}{c c} (1-t) R(t\eta) & t R(\eta(t-1))\\ -\frac{1}{\eta}R(t\eta) & \frac{1}{\eta} R(\eta(t-1)) \end{array}\right) \left(\begin{array}{c} q_0 \\ q_1 \end{array}\right). \end{align*} Such defined $v_\eta$ is a solution of the Hamiltonian flow and it belongs to $\mathscr{H}_{q_0,q_1}$. To assure that $v_\eta(t)\in H_0^{-1}(c)$ for all $t\in [0,1]$ we use \eqref{Radd}, \eqref{Rinv}, \eqref{DefMatrixA} and \eqref{q0p0} to express the condition $(q_0,p_0)\in H_0^{-1}(c)$ in the following way: \begin{align*} c & =\frac{1}{2} \left(q_0,\ p_0\right) \left(\begin{array}{c c} \Id & -\frac{1}{\eta}\\ 0 & \frac{1}{\eta}R(\eta) \end{array}\right) \left(\begin{array}{c c} 0 & -R\left(\frac{\pi}{2}\right)\\ R\left(\frac{\pi}{2}\right) & \Id \end{array}\right) \left(\begin{array}{c c} \Id &0\\ -\frac{1}{\eta} & \frac{1}{\eta}R(-\eta) \end{array}\right) \left(\begin{array}{c} q_0 \\ q_1 \end{array}\right)\\ & =\frac{1}{2} \left(q_0,\ p_0\right) \left( \frac{1}{\eta^2} \left(\begin{array}{c c} \Id &-R(-\eta)\\ -R(\eta) & \Id \end{array}\right) +\frac{1}{\eta} \left(\begin{array}{c c} 0 & -R\left(\frac{\pi}{2}-\eta\right)\\ R\left(\frac{\pi}{2}+\eta\right) & 0 \end{array}\right) \right) \left(\begin{array}{c} q_0 \\ q_1 \end{array}\right)\\ & = \frac{1}{\eta^2}\left(\frac{1}{2}\left( |q_0|^2+|q_1|^2\right)-q_1R(\eta)q_0^T\right) +\frac{1}{\eta}q_1R\left(\eta+\frac{\pi}{2}\right)q_0^T. \end{align*} Now if we define $f\in C^\infty(R)$ as in \eqref{Deff} then $(v_\eta,\eta)\in\Crit\A^{H_0-c}_{q_0,q_1}$ if and only if $f(\eta)=0$. Observe that $f(0)=\frac{1}{2}|q_1-q_0|^2$, thus $0\in Z(f)$ if and only if $q_0=q_1$. In that case, $\left(\A^{H_0-c}_{q_0,q_1}\right)^{-1}(0)$ is a circle. \end{proof} \begin{lem}\label{lem:2c>q0q1} In case $|q_0||q_1|\leq 2c$ the critical set of $\A^{H_0-c}_{q_0,q_1}$ satisfies one of the following properties: \begin{itemize}[leftmargin=2cm] \item[either] $q_0 \neq q_1$ and then $\#\Crit^+\A^{H_0-c}_{q_0,q_1}=\#\Crit^-\A^{H_0-c}_{q_0,q_1}=1$, \item[or] $q_0=q_1$ and then $\#\Crit^+\A^{H_0-c}_{q_0,q_1}=\#\Crit^-\A^{H_0-c}_{q_0,q_1}=0$. \end{itemize} \end{lem} \begin{proof} By Lemma \ref{lem:bijection} we know that we can restrict ourselves to the analysis of function $f$ defined in \eqref{Deff}. Note that $f(0)=\frac{1}{2}|q_1-q_0|^2$, thus \begin{align*} \textrm{for}\quad q_0 & \neq q_1\quad \textrm{we have}\quad f(0)>0,\\ \textrm{and for} \quad q_0 & =q_1 \quad \textrm{we have}\quad f(0)=0. \end{align*} In both cases, we know that $\lim_{\eta \to \pm \infty}f(\eta)=-\infty$ due to the dominance of the summand $-c\eta^2$ for $|\eta|$ large enough. In case $q_0\neq q_1$ we can use the intermediate value theorem to prove that $f$ has a root both on the positive and the negative half line. What is left to show is that $f$ has only one root on each half-line. Using \eqref{DerR} we can now calculate the derivative of $f$: \begin{align*} f'(\eta) & = -2c\eta +q_1^T\left(R\left(\eta+\frac{\pi}{2}\right)-R'\left(\eta\right)\right)q_0+\eta q_1^TR'\left(\eta+\frac{\pi}{2}\right)q_0\\ & = -\eta (2 c +q_1^TR(\eta)q_0). \end{align*} Thus in case $|q_0||q_1|\leq 2c$ we have $$ 2 c +q_1^TR(\eta)q_0) \geq 2c -|q_0||q_1|\geq 0, $$ and consequently $f'(\eta)\leq 0$ for $\eta>0$ and $f'(\eta)\geq 0$ for $\eta<0$. Moreover, in case $|q_0||q_1|=2c$ we have $f'(\eta)=0$ if and only if $\eta=0$ or $q_1=-\frac{2c}{|q_1|^2}R(\eta)q_0$. In particular, in case $|q_0||q_1|=2c$ the critical points of $f$ are isolated. Consequently, $f$ obtains its maximum at $0$ and is non-decreasing on the negative half line and non-increasing on the positive half line. This proves that in case $q_0\neq q_1$ there exists only one root on each positive and negative half line. On the other hand, since $f$ obtains its maximum at $0$, thus in case $q_0=q_1$ we know that $f(\eta)<0$ for all $\eta\neq 0$, which concludes the proof. \end{proof} \begin{lem}\label{lem:Res_q0q1} For a fixed $c>0$ there exists a residual set $\mathcal{Q}^c\subseteq \R^4\setminus \Delta$, such that for all pairs $(q_0,q_1)\in \mathcal{Q}^c$ the corresponding function $f$ as defined in \eqref{Deff} satisfies property \eqref{f'neq0}. \end{lem} \begin{proof} To prove the statement of the lemma, we will first extend the function $f\in C^\infty(\R)$ to $\bar{f}\in C^\infty(\R^5)$ by taking $\bar{f}(q_0,q_1,\eta):=f(\eta)$ where $f$ depends on $q_0$ and $q_1$ in the way described in \eqref{Deff}. Recall, that $f(0)=\frac{1}{2}|q_1-q_0|^2$, thus $$ \bar{f}^{-1}(0)\cap \left(\R^4\times\{0\}\right)=\Delta\times\{0\}. $$ We will show now that $\bar{f}^{-1}(0)\setminus (\Delta\times\{0\})$ is a smooth manifold. Using \eqref{Deff} we can calculate $D\bar{f}$: \begin{align*} \partial_\eta \bar{f} & = -\eta \left(2 c +q_1^TR(\eta)q_0\right),\\ \partial_{q_0}\bar{f} & = \eta q_1^T R\left(\eta+\frac{\pi}{2}\right)- q_1^T R(\eta)+q_0,\\ \partial_{q_1}\bar{f} & = \eta q_0^T R\left(-\eta-\frac{\pi}{2}\right)- q_0^T R(-\eta)+q_1. \end{align*} A straightforward computation shows that $\bar{f}$ satisfies the following relation: $$ \bar{f}(q_0,q_1,\eta)=-c\eta^2+\frac{1}{2}\left( q_0\partial_{q_0}\bar{f}+ q_1\partial_{q_1}\bar{f}\right). $$ Consequently, for all $(q_0,q_1,\eta)\in \bar{f}^{-1}(0)$ we have $|\eta||\partial_{q_0}\bar{f}||\partial_{q_1}\bar{f}|\neq 0$. In particular, for all $(q_0,q_1,\eta)\in \bar{f}^{-1}(0)\setminus (\Delta\times\{0\})$ the derivative $D\bar{f}\neq 0$ and thus, by the inverse function theorem, $\bar{f}^{-1}(0)\setminus (\Delta\times\{0\})$ is a smooth manifold. Note, that if we consider the function $f\in C^\infty(\R)$ as in \eqref{Deff} corresponding to a fixed pair $(q_0,q_1)\in \R^4$, then $$ \left( \bar{f}^{-1}(0)\setminus \Delta\right) \cap \left( \{(q_0,q_1)\}\times\R\right)=Z(f)\setminus \{0\}. $$ Additionally, $f'(\eta)=\partial_\eta \bar{f}(q_0,q_1,\eta)$ for all $\eta\in \R$. Therefore, if we denote \begin{align*} \mathcal{Q}^c := & \left\lbrace (q_0,q_1)\in \R^4\ |\ \forall\ \eta\in Z(f)\setminus\{0\}\quad f'(\eta)\neq 0\right\rbrace,\\ \textrm{then}\qquad \mathcal{Q}^c =& \left\lbrace (q_0,q_1)\in \R^4\ |\ \forall\ (q_0,q_1,\eta)\in \bar{f}^{-1}(0)\setminus \Delta\quad \partial_\eta \bar{f}\neq 0 \right\rbrace. \end{align*} Consequently, the set $\mathcal{Q}^c$ satisfies the assumptions of the theorem. On the other hand, since $\bar{f}^{-1}(0)\setminus \Delta$ is a smooth manifold, the set of regular values of the projection $P: \bar{f}^{-1}(0)\setminus \Delta \to \R^4$ is in fact equal to $\mathcal{Q}^c$. By the Morse-Sard Theorem $\mathcal{Q}^c$ is residual in $\R^4$. \end{proof} \begin{lem}\label{lem:Res_c} For a fixed pair $(q_0,q_1)\in \R^4, q_0\neq q_1$ there exists a residual set $\mathcal{I}_{q_0,q_1}\subseteq \R_+$, such that for all $c\in \mathcal{I}_{q_0,q_1}$ the corresponding function $f$ as defined in \eqref{Deff} satisfies property \eqref{f'neq0}. \end{lem} \begin{proof} In this proof we will follow the same arguments as in the proof of Lemma \ref{lem:Res_q0q1}. First, we extend the function $f\in C^\infty(\R)$ as in \eqref{Deff} to a function $\tilde{f}:\R_+\times\R \to \R$ by setting $\tilde{f}(c,\eta):=f(\eta)$. Second, we observe that $\partial_c \tilde{f}(c,\eta)= -\eta^2 < 0$ for all $\eta\neq 0$. By assumption we take $q_0\neq q_1$, hence $0\notin\tilde{f}^{-1}(0)$ and thus $\partial_c \tilde{f}(c, \eta)<0$ for all $(c,\eta) \in \tilde{f}^{-1}(0)$. This allows us to use the inverse function theorem to conclude that $\tilde{f}^{-1}(0)$ is a smooth manifold. Furthermore, we observe that the set of regular values of the projection $P:\tilde{f}^{-1}(0) \to \R^+$ is equal to \begin{align*} \mathcal{I}_{q_0,q_1} := & \left\lbrace c\in \R_+\ |\ \forall\ (c,\eta)\in \tilde{f}^{-1}(0)\quad \partial_\eta \tilde{f}\neq 0 \right\rbrace\\ = &\left\lbrace c\in \R_+\ |\ \forall\ \eta\in Z(f)\quad f'\neq 0 \right\rbrace. \end{align*} Finally, by the Morse-Sard Theorem, we conclude that $\mathcal{I}_{q_0,q_1}$ is residual in $\R_+$. \end{proof} In this final lemma, we show how the condition \eqref{f'neq0} implies that the function $f$ has an odd number of zeros on each the positive and the negative half line. \begin{lem}\label{lem:Odd} If $c>0$ and $(q_0, q_1)\in \R^4, q_0\neq q_1$ are such that the corresponding function $f$ satisfies \eqref{f'neq0}, then $f$ has an odd number of roots on each positive and negative half line. \end{lem} \begin{proof} First we will show that if $t_0,t_1\in \R, t_0<t_1$ are two roots of a smooth function $f\in C^\infty(\R)$, such that $f'(t_0), f'(t_1)\neq 0$ and $f(t)\neq 0$ for all\linebreak $t\in (t_0,t_1)$, then $f'(t_0)f'(t_1)<0$. Suppose the opposite is true and $f'(t_0)f'(t_1)>0$. Without loss of generality, we can assume that $f'(t_0),f'(t_1)>0$. Then there would exist $\delta>0$, such that for all $t\in (t_0, t_0+\delta),\ f(t)>0$ and for all $t\in (t_1-\delta,t_1),\ f(t)<0$. Consequently, by the intermediate value theorem there would have to exist $t\in (t_0+\delta,t_1-\delta)$, such that $f(t)=0$. But that brings us a contradiction. Let now $f$ be the function defined in \eqref{Deff}. We will show that the set of roots of $f$ is bounded. Observe that $f$ can be bounded from above by $$ f(\eta) \leq -c \eta^2 + |\eta||q_0||q_1|+\frac{1}{2}(|q_0|+|q_1|)^2. $$ Consequently, we have $f(\eta) < 0$ for $\eta \in (-\infty, -\delta_0)\cup (\delta_0, +\infty)$, where \begin{equation}\label{delta} \delta_0:= \frac{1}{2c}\left(\sqrt{|q_0|^2|q_1|^2+2c(|q_0|+|q_1|)^2}+|q_0||q_1|\right). \end{equation} In particular, the set of roots of $f$ denoted by $Z(f)$ is a subset of $[-\delta_0,\delta_0]$. Since $Z(f)$ is bounded and discrete the number $k:=\#\{ \eta \in Z(f)\ |\ \eta>0\}\in \mathbb{N}$ is well defined. Moreover, we can enumerate the elements of the set $\{ \eta \in Z(f)\ |\ \eta>0\}=\{\eta_i\}_{i=1}^{k}$, such that $\eta_i< \eta_{i+1}$ for $i=1,\dots k-1$ and \begin{equation}\label{AssEta} \begin{aligned} (\eta_i, \eta_{i+1})\cap Z(f) & = \emptyset && \textrm{for} && i=1, \dots k-1,\\ (0, \eta_1)\cap Z(f) & =\emptyset&& \textrm{and} && (\eta_{k}, +\infty)\cap Z(f)=\emptyset. \end{aligned} \end{equation} Our aim is to prove that $k$ is an odd number. We will show now that if $f$ satisfies \eqref{f'neq0} then $f'(\eta_1)<0$. Recall that $f(0)=\frac{1}{2}|q_1-q_0|^2$, which in the case $q_0\neq q_1$ means $f(0)>0$. Suppose now that $f'(\eta_1)>0$. That would imply that there exists $\delta>0$, such that\linebreak $f(\eta)<f(\eta_1)=0$ for $\eta \in (\eta_1-\delta, \eta_1)$. Consequently, by the intermediate value theorem, there would have to exist $\eta \in (0, \eta_1-\delta)$, such that $f(\eta)=0$. But that would contradict our assumption \eqref{AssEta}. Thus $f'(\eta_1)<0$. The last step would be to show that if $f$ satisfies \eqref{f'neq0} then $f'(\eta_{k})<0$. Recall that $\lim_{\eta \to +\infty}f(\eta)=-\infty$. Suppose now that $f'(\eta_{k})>0$. That would imply that there exists $\delta>0$, such that for all $\eta \in (\eta_{k}, \eta_{k}+\delta)$, $f(\eta)>0$. Consequently, by the intermediate value theorem, there would have to exist $\eta \in (\eta_{k}+\delta,+\infty)$, such that $f(\eta)=0$. But that contradicts assumption \eqref{AssEta}. Let us recall what we have learned: if $f$ satisfies \eqref{f'neq0} then $f'(\eta_1)<0$, $f'(\eta_i)$ has an opposite sign than $f'(\eta_{i+1})$ for all $i=1, \dots k-1$ and finally $f'(\eta_{k})<0$. This implies that $k$ is an odd number. Using the same arguments we prove that $\#\{ \eta \in Z(f)\ |\ \eta<0\}$ is an odd number as well. \end{proof} Despite the fact that the positive Lagrangian Rabinowitz Floer homology has only one generator, the number of positive critical points of the Rabinowitz action functional corresponding to a fixed pair of endpoints $q_0,q_1 \in \R^2$ does not necessarily have to be $1$. In fact, its cardinality depends on the value of the energy $c>0$. As shown in Lemma 4.4, the positive critical set of the Rabinowitz action functional is in bijection with the set of positive zeroes of the corresponding function $f$ defined in (4.8). In Figure \ref{fig:fFunction} we depict the functions $f$ corresponding to the chord endpoints $q_0=(1,0)$, $q_1=(0,1)$ and three different energies $c= \frac{1}{5}$, $c=\frac{1}{10}$, and $c=\frac{1}{20}$, respectively. We see that the number of positive zeroes of the functions increases as the energy $c$ decreases. \begin{figure} \centering \includegraphics[width=.75\linewidth]{fFunction.png} \caption{The three functions $f$ corresponding to energy $c=\frac{1}{5}$ (blue), $c=\frac{1}{10}$ (green), and $c=\frac{1}{20}$ (magenta) crossing zero in exactly $1$, $3$, and $5$ points, respectively.} \label{fig:fFunction} \end{figure} The following proposition shows that the number of positive critical points of the Rabinowitz action functional tends to $\infty$ as the energy $c$ approaches $0$. \begin{prop} For fixed $q_0,q_1 \in \R^2$ we have $$ \lim_{c\searrow 0}\# \left(\Crit^+ \A_{q_0,q_1}^{H_0-c}\right)=+\infty. $$ \end{prop} \begin{proof} By Lemma 4.4 we know that for any fixed $q_0,q_1 \in \R^2$ and $c>0$ we have $ \#\left(\Crit^+ \A_{q_0,q_1}^{H_0-c}\right)= \# \{ \eta>0\ |\ f(\eta)=0\}$, where $f$ is the function defined in (4.8) which depends on $q_0$, $q_1$, and $c$. We express $q_0$ and $q_1$ in polar coordinates as \begin{align*} q_0 & = r_0 \left(\begin{array}{c}\cos (\alpha) \\ \sin (\alpha)\end{array}\right) = r_0 R^T(\alpha)\left(\begin{array}{c} 1 \\ 0\end{array}\right),\\ q_1 & = r_1 \left(\begin{array}{c}\cos (\alpha+\theta) \\ \sin (\alpha+\theta)\end{array}\right) = r_1 R^T(\alpha+\theta)\left(\begin{array}{c} 1 \\ 0\end{array}\right). \end{align*} Plugging this into the function $f$ from (4.8), we obtain \begin{align*} f(\eta) & = -c \eta^2+\frac{1}{2}\left(r_0^2+r_1^2\right)+ r_0 r_1 (1,0)\left(\eta R\left(\eta+\theta+\frac{\pi}{2}\right)-R(\eta+\theta)\right)\left(\begin{array}{c} 1 \\ 0\end{array}\right)\\ & = -c \eta^2-\eta\ r_0 r_1 \sin\left(\eta +\theta \right) +\frac{1}{2}\left(r_0^2+r_1^2\right)-r_0 r_1 \cos (\eta +\theta). \end{align*} Note that the function depends only on the relative angle $\theta$ between the two endpoints. This is not very surprising as the Hamiltonian $H_0$ is invariant under rotations around the origin. Observe that \begin{align*} f(\eta) & = - c\eta^2 +\frac{1}{2}(r_0-r_1)^2 && \textrm{for}\qquad \eta = 2\pi n - \theta, && n \in \mathbb{N},\\ f(\eta) & = - c\eta^2 +\frac{1}{2}(r_0+r_1)^2 && \textrm{for}\qquad \eta = \pi (2n+1) - \theta, && n \in \mathbb{N}. \end{align*} Consequently, we have \begin{align*} f(\eta) & <0 && \textrm{for}\qquad \eta = 2\pi n - \theta > \frac{1}{\sqrt{2c}}|r_0-r_1|, && n\in\mathbb{N},\\ f(\eta) & >0 && \textrm{for}\qquad \eta = \pi (2n+1) - \theta < \frac{1}{\sqrt{2c}}(r_0+r_1), && n\in\mathbb{N}. \end{align*} Note that if we assume $$ \frac{\theta}{\pi} +\frac{1}{\pi\sqrt{2c}}|r_0-r_1| < n < \frac{\theta}{\pi} +\frac{1}{\pi\sqrt{2c}}(r_0+r_1)-1,$$ then $$ \frac{\theta}{\pi} +\frac{1}{\pi\sqrt{2c}}|r_0-r_1| < n, n+1 < \frac{\theta}{\pi} +\frac{1}{\pi\sqrt{2c}}(r_0+r_1). $$ Now exactly one of $n,n+1$ is even and the other is odd, so $$ f(\pi n - \theta)f(\pi (n+1) - \theta)<0, $$ and thus by the intermediate value theorem there exists $\eta\in (\pi n-\theta, \pi(n+1)-\theta)$ such that $f(\eta)=0$. Therefore, we can estimate \begin{align*} \#\left(\Crit^+ \A_{q_0,q_1}^{H_0-c}\right) & =\# \left\lbrace \eta>0\ |\ f(\eta)=0\right\rbrace \\ &\geq \# \left( \frac{\theta}{\pi} + \frac{|r_0-r_1|}{\pi\sqrt{2c}}, \frac{\theta}{\pi} +\frac{(r_0+r_1)}{\pi\sqrt{2c}}-1\right)\cap \mathbb{N}. \end{align*} The length of the interval on the right hand side of the inequality is equal to $\frac{\sqrt{2}}{\pi\sqrt{c}}\min\{r_0,r_1\}-1$. Since $\lim_{c\to 0}\left(\frac{\sqrt{2}}{\pi\sqrt{c}}\min\{r_0,r_1\}-1\right)=+\infty$, we conclude $$ \lim_{c\searrow 0} \#\left(\Crit^+ \A_{q_0,q_1}^{H_0-c}\right) \geq \lim_{c\searrow 0}\# \left( \frac{\theta}{\pi} + \frac{|r_0-r_1|}{\pi\sqrt{2c}}, \frac{\theta}{\pi} +\frac{(r_0+r_1)}{\pi\sqrt{2c}}-1\right)\cap\mathbb{N} = +\infty. $$ \end{proof} In Figures \ref{fig:1Orbit3Energies}, \ref{fig:3Orbits} and \ref{fig:5Orbits} we present the plots of the Reeb chords from $q_0=(0,1)$ to $q_1=(1,0)$ for various energies. More precisely, the graphs depict the projections of the Reeb chords onto the plane of positions. The plots have been obtained using the formula from Lemma \ref{lem:bijection}. Let us analyse how the number of Reeb chords depends on the energy $c\in \left\lbrace 1, \frac{1}{2}, \frac{1}{5}, \frac{1}{10}, \frac{1}{20}\right\rbrace$. By Lemma 4.5 we have $\Crit^+\A_{q_0,q_1}^{H_0-c}=1$ for $c \geq \frac{1}{2}$. On the other hand, from Lemma 4.4 we know that $\Crit^+\A_{q_0,q_1}^{H_0-c}$ is in bijection with the zeroes of the corresponding function $f$ in (4.8). By \eqref{delta} we know that $$ \left\lbrace \eta > 0\ |\ f(\eta)=0\right\rbrace \subseteq [0, \delta_0],\qquad \textrm{where}\qquad \delta_0=\frac{1}{2c}\left( \sqrt{1+8c}+1\right). $$ Calculating the interval for various energies we obtain \begin{align*} \textrm{For}\quad c &=\frac{1}{5},\quad & \delta_0 &=\frac{5}{2}\left( \sqrt{1+\frac{8}{5}}+1\right)=\frac{5}{2}\left( \sqrt{\frac{13}{5}}+1\right)<\frac{15}{2}=7\frac{1}{2},\\ \textrm{For}\quad c&=\frac{1}{10},& \delta_0 &=5\left( \sqrt{1+\frac{4}{5}}+1\right)=5\left( \frac{3}{\sqrt{5}}+1\right)< 5\left( \frac{3}{2}+1\right)=12\frac{1}{2},\\ \textrm{For}\quad c&=\frac{1}{20},& \delta_0 &=10\left( \sqrt{1+\frac{2}{5}}+1\right)=10\left( \sqrt{\frac{7}{5}}+1\right)<10\left(\frac{6}{5}+1\right)=22. \end{align*} Consequently, Figure \ref{fig:fFunction}. depicts the three functions $f$ corresponding to energies $c\in \left\lbrace \frac{1}{5}, \frac{1}{10}, \frac{1}{20}\right\rbrace$ on the whole interval $[0,\delta_0]$. Therefore we can deduce that the plot in Figure \ref{fig:fFunction}. depicts all the zeros of the three functions. Hence $$ \#\Crit^+\A_{q_0,q_1}^{H_0-\frac{1}{5}}=1, \qquad \#\Crit^+\A_{q_0,q_1}^{H_0-\frac{1}{10}}=3\quad \textrm{and}\quad \#\Crit^+\A_{q_0,q_1}^{H_0-\frac{1}{20}}=5. $$ From the discussion above we know that $$ \#\Crit^+\A_{q_0,q_1}^{H_0-1}=\#\Crit^+\A_{q_0,q_1}^{H_0-\frac{1}{2}}=\#\Crit^+\A_{q_0,q_1}^{H_0-\frac{1}{5}}=1. $$ In other words, on each of the level sets corresponding to $c=1$, $c=\frac{1}{2}$ and $c=\frac{1}{5}$ there is exactly one Reeb chord from $q_0=(0,1)$ to $q_1=(1,0)$. These Reeb chords are presented in Figure \ref{fig:1Orbit3Energies}. \begin{figure} \begin{minipage}[c]{0.43\linewidth} \includegraphics[width=\linewidth]{1Orbit3Energies.png} \caption{The unique Reeb chords of energy $c=1$ (green), $c=\frac{1}{2}$ (blue), and $c=\frac{1}{5}$ (magenta).} \label{fig:1Orbit3Energies} \end{minipage} \hfill \begin{minipage}[c]{0.54\linewidth} \vspace*{-.45cm} \includegraphics[width=\linewidth]{3Orbits.png} \caption{The three Reeb chords of energy $c=\frac{1}{10}$.} \label{fig:3Orbits} \end{minipage}\end{figure} Figure \ref{fig:3Orbits} depicts the three Reeb chords from $q_0=(0,1)$ to $q_1=(1,0)$ of energy $c=\frac{1}{10}$, and Figure \ref{fig:5Orbits} shows the five Reeb chords of energy $c=\frac{1}{20}$. \begin{figure}\centering \includegraphics[width=.6\linewidth]{5Orbits.png} \caption{The five Reeb chords of energy $c=\frac{1}{20}$.} \label{fig:5Orbits} \end{figure} \subsection{Calculating the Maslov index} In this subsection we will prove Theorem \ref{thm:posLRFH}. Let $H_0$ be the Copernican Hamiltonian as in \eqref{DefH0} and let $\mathcal{H}$ be the set of perturbations as defined in \eqref{DefHset}. By Proposition \ref{prop:posLRFH} we know that for all $h\in \mathcal{H}$ and all pairs $q_0,q_1\in \R^2$, $q_0\neq q_1$ the positive Lagrangian Rabinowitz Floer homology of the triple $\LRFH_*^+(\A^{H_0-h}_{q_0,q_1})$ is well defined and isomorphic to $\LRFH_*^+(\A^{H_0-c}_{q_0,q_1})$ for any $c>0$. On the other hand, by Proposition \ref{prop:CritPM} we know that if $c\geq \frac{1}{2}|q_0||q_1|$ then $\Crit^+\A^{H_0-c}_{q_0,q_1}$ has only one element. Consequently, $\LRFH_*^+(\A^{H_0-c}_{q_0,q_1})$ has only one generator and its boundary operator is $0$. Therefore, in order to calculate the positive Lagrangian Rabinowitz Floer homology explicitly what is left to do is to calculate the Maslov index of $(v,\eta)\in \Crit^+\A^{H_0-c}_{q_0,q_1}$. \begin{lem}\label{lem:ChordsBijection} For $c>0$ define a Hamiltonian $H_c:T^*\R^2\to \R$ as \begin{equation}\label{DefHDelta} H_\delta(q,p):=\frac{|p|^2}{2}+\delta(p_1q_2-p_2q_1). \end{equation} Let $H_0$ be the Copernican Hamiltonian as in \eqref{DefH0} and let $\varphi_\delta$ be a diffeomorphism of $T^*\R^2$ defined $\varphi_\delta(q,p):=\left(\sqrt{\delta}q,\frac{1}{\sqrt{\delta}}p\right)$. Then $$ (v,\eta)\in \Crit \A_{q_0,q_1}^{H_\delta-1}\qquad \iff \qquad \left(\varphi_\delta\circ v, \delta\eta\right)\in \Crit \A_{\varphi_\delta(q_0),\varphi_\delta(q_1)}^{H_0-\frac{1}{\delta}}. $$ \end{lem} \begin{proof} First observe that $\varphi$ preserves the standard symplectic form, so it is in fact a symplectomorphism. Moreover, the Hamiltonians satisfy the following relation $H_\delta=\delta H_0\circ \varphi_\delta$, thus $$ H_0^{-1}\left(\frac{1}{\delta}\right)=\varphi_\delta (H_\delta^{-1}(1) )\qquad\textrm{and}\qquad X_{H_\delta}=\delta D\varphi_\delta^{-1}X_{H_0}. $$ Now $(v,\eta)\in \Crit \A_{q_0,q_1}^{H_\delta-1}$ if and only if $v(i)=q_i$ for $i=1,2$, $v([0,1])\subseteq H_\delta^{-1}(1)$ and $\partial_t v=\eta X_{H_\delta}(v)$. Naturally, the first two conditions are trivially equivalent. It suffices to verify that $$ \frac{d}{dt}(\varphi_\delta\circ v) = D\varphi_\delta (\partial_t v) = \eta D\varphi_\delta (X_{H_\delta}(v))=\delta X_{H_0}(\varphi_\delta \circ v). $$ \end{proof} \begin{lem}\label{lem:Convergence} Let $\{H_\delta\}_{\delta>0}$ be the family of Hamiltonians defined in \eqref{DefHDelta}. Let $H_{\bullet}:T^*\R^2\to\R$ be the kinetic Hamiltonian $H_{\bullet}(q,p):=\frac{1}{2}|p|^2$. Fix $q_0,q_1\in \R^2$, $q_0\neq q_1$. Then for $\delta<\sqrt{\frac{2}{|q_0||q_1|}}$ (in case $q_0=0$ or $q_1=0$ for any $\delta>0$) we have $\#\Crit^+\A^{H_\delta-1}_{q_0,q_1}=1$ and the family of Reeb chords $(v_\delta,\eta_\delta)\in \Crit^+\A^{H_\delta-1}_{q_0,q_1}$ satisfies $$ \lim_{\delta\to 0}(v_\delta,\eta_\delta)=(v_0,\eta_0)\in \Crit^+\A^{H_{\bullet}-1}_{q_0,q_1}. $$ \end{lem} \begin{proof} Using Lemma \ref{lem:ChordsBijection} we know that for every $\delta>0$ there is a bijection between $\Crit^+\A_{q_0,q_1}^{H_\delta-1}$ and $\Crit^+ \A_{\varphi_\delta(q_0),\varphi_\delta(q_1)}^{H_0-\frac{1}{\delta}}$. On the other hand, by Proposition \ref{prop:CritPM} we know that for $|\varphi_\delta(q_0)||\varphi_\delta(q_1)|=\delta|q_0||q_1|\leq \frac{2}{\delta}$ we have $1=\# \Crit^+ \A_{\varphi_\delta(q_0),\varphi_\delta(q_1)}^{H_0-\frac{1}{\delta}}=\# \Crit^+ \A_{q_0,q_1}^{H_\delta-1}$. Consequently, for every $\delta<\sqrt{\frac{2}{|q_0||q_1|}}$ (or in the case either $q_0$ or $q_1$ are zero for any $\delta>0$) there is a unique solution $(v_\delta,\eta_\delta)\in \Crit^+ \A_{q_0,q_1}^{H_\delta-1}$. Having established that $(v_\delta,\eta_\delta)\in \Crit^+\A^{H_\delta-1}_{q_0,q_1}$ is uniquely defined for small enough $\delta$, we can use the function $f$ from Lemma \ref{lem:bijection} to estimate $\eta_\delta$. We simply need to map $(c,q_0,q_1,\eta)\mapsto (\frac{1}{\delta}, \sqrt{\delta}q_0, \sqrt{\delta}q_1, \delta \eta)$ to obtain that $\eta_\delta$ is the unique positive root of the function: \begin{align} g_\delta(\eta)&:= -\eta^2+\delta \eta q_1^TR\left(\delta\eta+\frac{\pi}{2}\right)q_0 +\frac{1}{2}\left(|q_0|^2+|q_1|^2\right)-q_1^TR(\delta\eta)q_0\nonumber\\ &= \frac{1}{2}|q_1-q_0|^2-\eta^2+q_1^T\left( \Id-R(\delta\eta)-\delta \eta R'(\delta \eta) \right)q_0\label{g_delta} \end{align} This way we know that $(v_\delta,\eta_\delta)\in \Crit^+\A^{H_\delta-1}_{q_0,q_1}$ if and only if $g_\delta(\eta_\delta)=0$. By \eqref{g_delta} we can use the Taylor expansion to obtain the following estimate \begin{gather*} \frac{1}{2}|q_1-q_0|^2-\eta^2(1+\delta^2|q_0||q_1|)\leq g_\delta(\eta) \leq \frac{1}{2}|q_1-q_0|^2-\eta^2(1-\delta^2|q_0||q_1|),\\ g_\delta(\eta_\delta)=0\quad \Rightarrow \quad \frac{|q_1-q_0|}{\sqrt{2(1+\delta^2|q_0||q_1|)}}\leq \eta_\delta \leq\frac{|q_1-q_0|}{\sqrt{2(1-\delta^2|q_0||q_1|)}}, \end{gather*} which directly gives us $\lim_{\delta\to 0}\eta_\delta=\frac{1}{\sqrt{2}}|q_1-q_0|$. To show the convergence of $v_\delta$ we again use Lemma \ref{lem:bijection} to present the equation for $v_\delta$ explicitly. By Lemma \ref{lem:ChordsBijection} is suffices to just use the mapping $(q_0,q_1,\eta)\mapsto (\sqrt{\delta}q_0, \sqrt{\delta}q_1, \delta \eta)$ in the formula for $v_\delta$ from Lemma \ref{lem:bijection} to obtain: \begin{align*} \varphi_\delta\circ v_\delta(t) & =\left(\begin{array}{c c} (1-t) R(t\delta\eta_\delta) & t R(\delta\eta_\delta(t-1))\\ -\frac{1}{\delta\eta_\delta}R(t\delta\eta_\delta) & \frac{1}{\delta\eta_\delta} R(\delta\eta_\delta(t-1)) \end{array}\right) \left(\begin{array}{c} \sqrt{\delta} q_0 \\ \sqrt{\delta} q_1 \end{array}\right),\\ v_\delta(t) &= \left(\begin{array}{c c} (1-t) R(t\delta\eta_\delta) & t R(\delta\eta_\delta(t-1))\\ -\frac{1}{\eta_\delta}R(t\delta\eta_\delta) & \frac{1}{\eta_\delta} R(\delta\eta_\delta(t-1)) \end{array}\right) \left(\begin{array}{c} q_0 \\ q_1 \end{array}\right). \end{align*} Consider the functions $t \mapsto R(t\delta\eta_\delta)$ and $t\mapsto R((1-t)\delta\eta_\delta)$ on the interval $[0,1]$. Since $\lim_{\delta\to 0}\delta\eta_\delta =0$, we have the uniform convergence $\lim_{\delta\to 0}R(t\delta\eta_\delta)=\lim_{\delta\to 0}R((1-t)\delta\eta_\delta)=\Id$. Consequently, \begin{equation}\label{straight} \lim_{\delta\to 0}v_\delta=v_0(t) \quad \textrm{with}\quad v_0(t):= \left(\begin{array}{c c} (1-t) & t \\ -\frac{\sqrt{2}}{|q_1-q_0|} & \frac{\sqrt{2}}{|q_1-q_0|} \end{array}\right) \left(\begin{array}{c} q_0 \\ q_1 \end{array}\right). \end{equation} A straightforward calculation shows that $(v_0,\eta_0)$ with $\eta_0:=\frac{1}{\sqrt{2}}|q_1-q_0|$ is the unique element of $\Crit^+\A^{H_{\bullet}-1}_{q_0,q_1}$. \end{proof} \vspace*{.25cm} \noindent \textit{Proof of Theorem \ref{thm:posLRFH}:} By Proposition \ref{prop:posLRFH} we know that for all $h\in \mathcal{H}$ and all $q_0,q_1\in \R^2$ with $q_0\neq q_1$ the positive Lagrangian Rabinowitz Floer homology $\LRFH_*^+(\A^{H_0-h}_{q_0,q_1})$ is well defined and isomorphic to $\LRFH_*^+(\A^{H_0-c}_{q_0,q_1})$ for any $c>0$. On the other hand, by Proposition \ref{prop:CritPM} we know that if $c\geq |q_0||q_1|/2$ then $\Crit^+\A^{H_0-c}_{q_0,q_1}$ has only one element. Consequently, $\LRFH_*^+(\A^{H_0-c}_{q_0,q_1})$ has only one generator and its boundary operator is $0$. It remains to calculate the Maslov index of this generator. Let $\{H_\delta\}_{\delta>0}$ be the family of Hamiltonians defined in \eqref{DefHDelta} and let $\varphi_\delta$ be the family of symplectomorphisms defined by $\varphi_\delta(q,p):=(\frac{1}{\sqrt{\delta}}q,\sqrt{\delta}p)$. By Lemma \ref{lem:ChordsBijection}, for every $c>0$ we have $$ (v,\eta)\in \Crit^+\A^{H_0-c}_{q_0,q_1}\qquad\iff\qquad(\varphi_\frac{1}{c}^{-1} \circ v, c\eta)\in \Crit^+\A^{H_{1/c}-1}_{\frac{1}{\sqrt{c}}q_0, \frac{1}{\sqrt{c}}q_1}. $$ Since the Maslov index is invariant under symplectomorphisms, we get $$ \mu^{\rm tr}\left( v\left(\frac{\cdot}{\eta}\right)\right)=\mu^{\rm tr}\left(\varphi_\frac{1}{c}^{-1} \circ v\left(\frac{\cdot}{c\eta}\right)\right). $$ Therefore, it suffices to calculate $\mu^{\rm tr}\left( v_\delta\left(\frac{\cdot}{\eta_\delta}\right)\right)$ of the unique $(v_\delta, \eta_\delta)\in \Crit^+\A^{H_\delta-1}_{q_0,q_1}$ for $\delta>0$ small enough. Denote by $H_{\bullet}$ the kinetic Hamiltonian $H_{\bullet}(q,p):=\frac{1}{2}|p|^2$ and let $(v_0,\eta_0)$ be the unique element of $\Crit^+\A^{H_{\bullet}-1}_{q_0,q_1}$ as in \eqref{straight}. By Lemma \ref{lem:Convergence} we know that $\lim_{\delta\to 0}(v_\delta, \eta_\delta)=(v_0,\eta_0)$. Continuity of the Maslov index and Example~\ref{ex:Maslov} implies $$ \mu^{\rm tr}\left( v_\delta\left(\frac{\cdot}{\eta_\delta}\right)\right)=\mu^{\rm tr}\left( v_0\left(\frac{\cdot}{\eta_0}\right)\right) = \frac12 $$ for small enough $\delta>0$. This concludes the proof of Theorem \ref{thm:posLRFH}. \hfill $\square$ \section{Non-compactly supported potential} The aim of this section will be to prove Proposition \ref{prop:CritH=CritH1}. Let $H_0: T^*\R^2 \to \R$ be the quadratic Hamiltonian defined in \eqref{DefH0} and let $V:\R^2 \to \R$ be the potential function as in Proposition \ref{prop:CritH=CritH1}. \begin{equation}\label{DefH} \textrm{Define}\qquad H:= H_0 -V. \end{equation} Then \begin{equation}\label{X_H} \begin{aligned} X_H & = (p_1+q_2)\partial_{q_1}+(p_2-q_1)\partial_{q_2}+\left(p_2 +\frac{\partial V}{\partial_{q_1}}\right)\partial_{p_1}-\left(p_1-\frac{\partial V}{\partial_{q_2}}\right)\partial_{p_2},\\ X_H & = p_r\partial_r+\left(1+\frac{p_\theta}{r^2}\right)\partial_\theta+\left( \frac{p_\theta^2}{r^3}+ \frac{\partial V}{\partial r}\right)\partial_{p_r}+\frac{\partial V}{\partial \theta}\partial_{p_\theta}. \end{aligned} \end{equation} This first step to prove Proposition \ref{prop:CritH=CritH1} is to show that for positive energy $c>0$ all periodic orbits of $X_H$ on $H^{-1}(c)$ are contained in a compact subset of $T^*\R^2$: \begin{prop}\label{prop:BoundChord} Let $H: T^*\R^2 \to \R$ be the Hamiltonian defined in \eqref{DefH}. Fix $c>0$ and $q_0,q_1 \in \R^2$ and let $\A^{H-c}_{q_0,q_1}:\mathscr{H_1}_{q_0,q_1}\times \R \to \R$ be the corresponding action functional. Then the critical set $\Crit \A^{H-c}_{q_0,q_1}$ is bounded in $L^\infty$. \end{prop} \begin{proof} By an argument similar to the one in the proof of Lemma \ref{lem:H0Chord} it suffices to show that the set \begin{equation}\label{H1_r} H^{-1}(c)\cap \{\{H, r\}=0\}\cap \{\{H,\{H, r\}\}\leq 0\} \end{equation} is compact. By \eqref{X_H} we have that \begin{align*} \{H,r\} & = p_r,\\ \{H,\{H,r\}\} & = \frac{p_\theta^2}{r^3}+ \frac{\partial V}{\partial r}. \end{align*} For $(r, \theta, p_r, p_\theta) \in H^{-1}(c)\cap \{\{H, r\}=0\}$ we have \begin{gather} \frac{p_\theta^2}{2r^2}+p_\theta =V(r,\theta)+c\geq c,\nonumber\\ \left(\frac{p_\theta}{r}+r\right)^2 \geq r^2+2c,\nonumber\\ \frac{|p_\theta|}{r}\geq \sqrt{r^2+2c}-r.\label{p_theta_r} \end{gather} Consequently, by our assumption on the potential function $V$, if $\alpha>2$ and $a>0$ then for $(r, \theta, p_r, p_\theta) \in H^{-1}(c)\cap \{\{H, r\}=0\}$, such that \begin{gather*} r \geq r_1:=\max\left\lbrace r_0,\frac{1}{2}\sqrt{c}, \left( \frac{a\alpha}{c^2}\right)^{\frac{1}{\alpha-2}}\right\rbrace\\ \textrm{we have} \quad \frac{|p_\theta|}{r} \geq \sqrt{r^2+2c}-r > \frac{c}{2 r},\\ \{H,\{H, r\}\} \geq \frac{c^2}{4r^3}-\frac{a\alpha}{r^{\alpha+1}}>0. \end{gather*} On the other hand, if $\alpha = 2$ and $a\in \left(0 ,\frac{c^2}{4}\right)$ then for $(r, \theta, p_r, p_\theta) \in H^{-1}(c)\cap \{\{H, r\}=0\}$, such that \begin{gather*} r \geq r_1:=\max\left\lbrace r_0,\sqrt{\frac{a}{c-\sqrt{2a}}}\right\rbrace\\ \textrm{we have} \quad \frac{|p_\theta|}{r} \geq \sqrt{r^2+2c}-r > \frac{\sqrt{2a}}{ r},\\ \{H,\{H, r\}\} \geq \frac{2a}{r^3}-\frac{2a}{r^{3}}>0. \end{gather*} Consequently, in both cases the set $$ H^{-1}(c)\cap \{\{H, r\}=0\}\cap \left\lbrace \{H,\{H, r\}\} \leq 0 \right\rbrace $$ is bounded in $r$ and for every $(v, \eta) \in \Crit \A^{H-c}_{q_0,q_1}$ we have $$ \sup r \circ v \leq R_0 := \max\{|q_0|, |q_1|, r_1\}. $$ By an argument similar to the one presented in the Lemma \ref{lem:H0Chord} we obtain that $p_r\circ v$ and $p_\theta\circ v$ are also uniformly bounded in the following way: \begin{align*} |p_r| & \leq \sqrt{R_0^2+ 2(\sup_{r\leq R_0}V+c)},\\ |p_\theta| & \leq R_0 \left( R_0+\sqrt{R_0^2+ 2(\sup_{r\leq R_0}V+c)}\right). \end{align*} \end{proof} \begin{rem} Note that the assertions of Proposition \ref{prop:BoundChord} hold true also for the potential $V$ satisfying $V(r,\theta) \leq \frac{a}{r^2}$ and $\frac{\partial V}{\partial r} \geq -\frac{2 a}{r^3}$ for $r >r_0$ and $a \in \left[ \frac{c^2}{4}, \frac{c^2}{2} \right)$. However, we were unable to prove Proposition \ref{prop:CritH=CritH1} for this class of potentials and that is why we restrict ourselves to $a < \frac{c^2}{4}$. \end{rem} Now we would like change the Hamiltonian $H$, by multiplying the potential function $V$ with a compactly supported function $\varphi: T^*\R^2 \to [0,1]$, in the following way \begin{equation}\label{DefH1} H_1(q,p):=H_0(q,p)+\varphi(q,p)V(q). \end{equation} The associated Hamiltonian vector field is \begin{equation}\label{X_H1} \begin{aligned} X_{H_1} & = \left(p_r-V\frac{\partial \varphi}{\partial p_r}\right)\partial_r+\left(1+\frac{p_\theta}{r^2}-V\frac{\partial \varphi}{\partial p_\theta}\right)\partial_\theta\\ & + \left( \frac{p_\theta^2}{r^3}+ \varphi\frac{\partial V}{\partial r}+V\frac{\partial \varphi}{\partial r}\right)\partial_{p_r}+\left(\varphi\frac{\partial V}{\partial \theta}+V\frac{\partial \varphi}{\partial \theta}\right)\partial_{p_\theta}. \end{aligned} \end{equation} If we choose $\varphi$, such that for all the Reeb chords $(v,\eta) \in \Crit \A^{H-c}_{q_0,q_1}$ we would have $v([0,1]) \subseteq \varphi^{-1}(1)$, then $\Crit(\A^{H-c}_{q_0,q_1}) \subseteq \Crit(\A^{H_1-c}_{q_0,q_1})$. In fact, we would like to choose $\varphi$, such that $\Crit(\A^{H_1-c}_{q_0,q_1})= \Crit(\A^{H-c}_{q_0,q_1})$. Let $\chi:\R\to [0,1]$ be a smooth function, such that $-2<\chi'< 0$ on $[0,1]$ and \begin{align} &&\chi(x) & =\begin{cases} 1 & \textrm{for} \quad x\leq 0,\\ 0 & \textrm{for} \quad x\geq 1. \end{cases}\nonumber\\ &\textrm{Define} &\chi_0(r) &:=\chi\left(\beta(r-R_1) \right),\nonumber\\ &&\chi_1(r,\theta, p_r,p_\theta)& :=\chi( H_0(r,\theta,p_r,p_\theta)-\sup V-c),\nonumber\\ &&\varphi(r,\theta,p_r,p_\theta) & := \chi_0(r)\chi_1(r,\theta,p_r,p_\theta).\label{defPhi} \end{align} \begin{align} &\textrm{where} & R_1 &:= \begin{cases} \max\left\lbrace |q_0|, |q_1|, r_0, \frac{1}{2}\sqrt{c},\left( \frac{8a\alpha}{c^2}\right)^\frac{1}{\alpha-2}\right\rbrace & \textrm{for}\quad \alpha>2,\\ \max\left\lbrace |q_0|, |q_1|, r_0, \frac{c+2\sqrt{a}}{2\sqrt{c-2\sqrt{a}}}\right\rbrace & \textrm{for}\quad \alpha=2. \end{cases}\label{R1}\\ &\textrm{and} & \beta &:=\begin{cases} \frac{2-\alpha}{2 R_1} & \textrm{for}\quad \alpha>2,\\ \frac{(c+2\sqrt{a})^2-16a}{8a R_1} & \textrm{for}\quad \alpha=2. \end{cases}\label{beta} \end{align} Note that in both for $\alpha>2$ and for $\alpha=2$ we have $\beta>0$. In the later case, it follows from the assumption that $a< \frac{c^2}{4}$. This way we have \begin{equation} \chi_0(r) = \begin{cases} 1 & \textrm{for}\ r\leq R_1,\\ 0 & \textrm{for}\ r\geq R_1+\frac{1}{\beta}. \end{cases}\label{chi0} \end{equation} We will show that for a function $\varphi$ defined as in \eqref{defPhi} we have $$ \Crit (\A^{H_0-\varphi V-c}_{q_0,q_1}) = \Crit(\A^{H_0-V-c}_{q_0,q_1}). $$ The first step would be to show that $\varphi$ has compact support: \begin{lem} The function $\varphi: T^*\R^2\to \R$ defined as in \eqref{defPhi} has compact support. \end{lem} \begin{proof} By \eqref{defPhi} and \eqref{chi0} we have \begin{align*} \operatorname{supp}\varphi & \subseteq \{H_0 \leq \sup V+c + 1\}\cap \left\lbrace r \leq R_1+\frac{1}{\beta}\right\rbrace\\ & \subseteq \left\lbrace \frac{1}{2}p_r^2+\frac{1}{2}\left( \frac{p_\theta}{r}+r\right)^2 \leq \frac{1}{2}r^2+\sup V+c + 1\right\rbrace\cap \left\lbrace r \leq R_1+\frac{1}{\beta}\right\rbrace\\ & \subseteq \left\lbrace \frac{1}{2}p_r^2+\frac{1}{2}\left( \frac{p_\theta}{r}+r\right)^2 \leq \frac{1}{2} \left( R_1+\frac{1}{\beta}\right)^2+\sup V + c+ 1\right\rbrace\cap \left\lbrace r \leq R_1+\frac{1}{\beta}\right\rbrace. \end{align*} Since both of the sets on the right-hand side are compact, their intersection is compact, so $\operatorname{supp}\varphi$ is compact as a closed subset of a compact set. \end{proof} \begin{lem} Let $V:\R^2\to \R$ be a potential function as in Proposition \ref{prop:CritH=CritH1} and let $\varphi$ be the corresponding function defined in \eqref{defPhi}. Then $\varphi V+c \in \mathcal{H}$. \end{lem} \begin{proof} First observe that $$ \supp d(\varphi V)=\supp ( V d\varphi+ \varphi dV)\subseteq \supp \varphi, $$ hence $d(\varphi V)\in C_c^\infty(T^*\R^2)$. On the other hand, by definition: \begin{align*} \varphi(p,q) V(q) & +c - d(\varphi V)(p\partial_p) = V(q)\chi_0(q)\left(\chi_1(q,p)-d\chi_1(p\partial_p)\right)+c\\ & = V(q) \chi_0(q) \left( \chi_1(q,p)- \chi'(H_0(q,p) - \sup V-c) dH_0(p\partial_p)\right)+c. \end{align*} Since by assumption we the functions $V$, $\chi_0$, $\chi_1$ and $-\chi'$ are non-negative it suffices to prove that $dH_0(p\partial_p)\geq 0$ on $\chi_1^{-1}((0,1))$. Note that on $\chi_1^{-1}((0,1))$ we have $H_0 > \sup V+c$. Combining that with \eqref{H0pdp} we can calculate that on $\chi_1^{-1}((0,1))$ we have: $$ dH_0(p\partial_p)= \frac{1}{2}\|p\|^2+H_0\geq \frac{1}{2}\|p\|^2 + \sup V+c > 0. $$ \end{proof} Now we have proven that $\varphi$ as in \eqref{defPhi} is an eligible candidate, we will continue with the proof of Proposition \ref{prop:CritH=CritH1}. However, we will first start with the proof of a series of lemmas: \begin{lem}\label{lem:vinPhi(1)} Let $H$ be the Hamiltonian defined in \eqref{DefH} and let $\varphi$ be the function defined in \eqref{defPhi}. Then $$ v([0,1])\subseteq \varphi^{-1}(1)\quad \textrm{for all}\quad (v,\eta)\in \Crit\A^{H-c}_{q_0,q_1}. $$ \end{lem} \begin{proof} By definition of $H$ we have $$ H_ 0 = H+ V \leq H + \sup V, $$ so $H^{-1}(c) \subseteq H_0^{-1}((-\infty, \sup V+c])$. On the other hand, by Proposition \ref{prop:BoundChord} for every $(v,\eta)\in \Crit\A^{H-c}_{q_0,q_1}$ we have \begin{align*} v([0,1]) & \subseteq \{ r \leq R_0\}\cap H^{-1}(c) \subseteq \{r \leq R_1\}\cap H_0^{-1}((-\infty, \sup V+c])\\ & = \chi_0^{-1}(1)\cap \chi_1^{-1}(1)=\varphi^{-1}(1). \end{align*} \end{proof} \begin{lem}\label{lem:subsetvarphi(1)} Let $H_1$ be the Hamiltonian defined in \eqref{DefH1} and let $\varphi$ be the function defined in \eqref{defPhi}. Then $$ H_1^{-1}(c)\cap \{\{H_1,r\}=0\}\cap \{\{H_1,\{H_1,r\}\}\leq 0\} \subseteq \varphi^{-1}(1). $$ \end{lem} \begin{proof} First observe that $H_1\big|_{\varphi^{-1}(0)}=H_0$, so by \eqref{supp_h} we have $$ H_1^{-1}(c)\cap \{\{H_1,r\}=0\}\cap \{\{H_1,\{H_1,r\}\}\leq 0\}\cap \varphi^{-1}(0)=\emptyset. $$ On the other hand, if $x \in \chi_1^{-1}([0,1))$, then $H_0(x)>\sup V+c$ and $$ H_1(x)=H_0(x)-\varphi (x) V(x) \geq H_0(x) - \sup V>c. $$ Hence $\chi_1^{-1}((0,1])\cap H_1^{-1}(c)=\emptyset$. Therefore we can restrict ourselves to the analysis of the set $\chi_0^{-1}((0,1))$, i.e. when $ R_1 < r < R_1+\frac{1}{\beta}$. Let us calculate \begin{align*} \frac{\partial\varphi}{\partial r} & = \beta\chi_0'\chi_1-\frac{p_\theta^2}{r^3}\chi_0\chi_1', && \frac{\partial \varphi}{\partial \theta}=0,\\ \frac{\partial \varphi}{\partial p_r} & = p_r\chi_0\chi_1', && \frac{\partial\varphi}{\partial p_\theta} = \left(\frac{p_\theta}{r^2}+1\right)\chi_0 \chi_1'. \end{align*} In particular, by \eqref{X_H} we have $$ \{H_1,r\} = dr(X_{H_1})= p_r- V\frac{\partial \varphi}{\partial p_r}=p_r (1- V \chi_0 \chi_1'). $$ By assumption $V, \chi \geq 0$ and $\chi'\leq 0$, thus $\{H_1,r\}=0$ implies $p_r=0$. Consider now $(r, \theta, p_r, p_\theta) \in H_1^{-1}(0)\cap \left(\{H_1, r\}\right)^{-1}(0)$. Then \begin{align*} \{H_1,\{H_1,r\}\} & = \{H_1, p_r\}- \frac{\partial \varphi}{\partial p_r}\{H_1, V\}-V\left\lbrace H_1, \frac{\partial \varphi}{\partial p_r}\right\rbrace\\ & = \{H_1, p_r\}-V \frac{\partial^2\varphi}{\partial p_r^2} \{H_1, p_r\}\\ & = \left(\frac{p_\theta^2}{r^3}+ \varphi\frac{\partial V}{\partial r}+V\frac{\partial \varphi}{\partial r}\right)\left(1-V\frac{\partial^2\varphi}{\partial p_r^2}\right)\\ & = \left(\frac{p_\theta^2}{r^3}+ \chi_0 \chi_1\frac{\partial V}{\partial r}+V\left( \beta\chi_0'\chi_1-\frac{p_\theta^2}{r^3}\chi_0\chi_1'\right)\right)\left(1-\chi_0\chi_1'V\right) \end{align*} Again, since by assumption $V, \chi \geq 0$ and $\chi'\leq 0$, thus $1-\chi_0\chi_1'V\geq 1$ and $\{H_1,\{H_1,r\}\}$ has the same sign as \begin{equation}\label{eq5} \frac{p_\theta^2}{r^3}+ \chi_0 \chi_1\frac{\partial V}{\partial r}+V\left( \beta\chi_0'\chi_1-\frac{p_\theta^2}{r^3}\chi_0\chi_1'\right) \end{equation} Furthermore, since by assumption $V, \chi_0, p_\theta \geq 0$ and $\chi_1' \leq 0$, consequently $-\frac{p_\theta^2}{r^3}\chi_0\chi_1'V \geq 0$ and we can estimate \eqref{eq5} from below by \begin{equation}\label{eq6} \frac{p_\theta^2}{r^3}+ \chi_0 \chi_1\frac{\partial V}{\partial r}+ V\beta \chi_0'\chi_1. \end{equation} We will show that \eqref{eq6} is positive for both cases $\alpha>2$ and $\alpha=2$. \noindent\textbf{Case $\alpha>2$:}\\ By \eqref{R1} we have that $r > R_1 \geq \left(\frac{8a\alpha}{c^2}\right)^{\frac{1}{\alpha-2}}$, so $\frac{c^2}{4r^3}>\frac{2a\alpha}{r^{\alpha+1}}$. On the other hand, $r < R_1+\frac{1}{\beta}=R_1\frac{\alpha}{\alpha-2}$. Combining that with the assumptions on the potential $V$ and the fact that $-2\leq \chi'\leq 0$ and $0\leq \chi \leq 1$ we obtain \begin{align*} \frac{p_\theta^2}{r^3}+ \chi_0 \chi_1\frac{\partial V}{\partial r}+\frac{\alpha-2}{2R_1}V \chi_0'\chi_1 & \geq \frac{c^2}{4r^3}-\frac{a\alpha}{r^{\alpha+1}}-\frac{\alpha-2}{R_1}\frac{a}{ r^\alpha}\\ &> \frac{a\alpha}{r^{\alpha+1}}-\frac{\alpha -2}{R_1}\frac{a}{ r^\alpha}\\ & = \frac{a(\alpha -2)}{R_1 r^{\alpha+1}}\left( R_1 \frac{\alpha}{\alpha -2} -r\right)>0. \end{align*} \noindent\textbf{Case $\alpha=2$:}\\ By \eqref{p_theta_r}, \eqref{R1} and \eqref{chi0} we have \begin{gather*} r \geq \frac{c+2\sqrt{a}}{2\sqrt{c-2\sqrt{a}}}\qquad\textrm{and}\qquad c-2\sqrt{a} \geq \frac{(c+2\sqrt{a})^2}{4r^2},\\ r^2 + 2c \geq r^2+c+2\sqrt{a}+\frac{(c+2\sqrt{a})^2}{4r^2}=\left(r+ \frac{c+2\sqrt{a}}{2 r}\right)^2,\\ \frac{|p_\theta|}{r} \geq \sqrt{r^2+2c}-r\geq \frac{c+2\sqrt{a}}{2r}. \end{gather*} Therefore, by \eqref{beta} and \eqref{chi0} we obtain \begin{align*} \frac{p_\theta^2}{r^3} & + \chi_0 \chi_1\frac{\partial V}{\partial r}+V\beta \chi_0'\chi_1 \geq \frac{(c+2\sqrt{a})^2}{4r^3}-\frac{2a}{r^3}-\frac{2\beta a}{r^2}\\ & = \frac{2a}{r^3}\left( \frac{\left(c+2\sqrt{a}\right)^2}{8a}-1-\beta r\right)=\frac{2a}{r^3}\left( R_1 \beta + 1 -\beta r\right)>0. \end{align*} Consequently, in both cases we have $$ H_1^{-1}(c)\cap \{\{H_1,r\}=0\}\cap \{\{H_1,\{H_1,r\}\}\leq 0\}\cap \chi_0^{-1}((0,1))=\emptyset, $$ which proves the claim. \end{proof} \noindent\textit{Proof of Proposition \ref{prop:CritH=CritH1}:} By Lemma \ref{lem:vinPhi(1)} we know that $$ v([0,1])\subseteq \varphi^{-1}(1)\quad \textrm{for all}\quad (v,\eta)\in \Crit\A^{H-c}_{q_0,q_1}. $$ But by definition $H_1\big|_{\varphi^{-1}(1)}=H\big|_{\varphi^{-1}(1)}$. Thus $\Crit\A^{H-c}_{q_0,q_1} \subseteq \Crit\A^{H_1-c}_{q_0,q_1}$. On the other hand, by an argument as in the proof of Proposition \ref{prop:BoundChord} we know that for all $(v,\eta)\in \Crit\A^{H_1-c}_{q_0,q_1}$ if $\max_{t\in [0,1]}r \circ v = r\circ v(t_0)$, then $$ v(t_0) \in \{q_0, q_1\}\cup \left( H_1^{-1}(c)\cap \{\{H_1,r\}=0\}\cap \{\{H_1,\{H_1,r\}\}\leq 0\}\right). $$ Moreover, by Lemma \ref{lem:subsetvarphi(1)} we know that \begin{align*} v(t_0) \in \{q_0, q_1\} & \cup \left( H_1^{-1}(c)\cap \{\{H_1,r\}=0\}\cap \{\{H_1,\{H_1,r\}\}\leq 0\}\right)\\ & \subseteq H_1^{-1}(c)\cap \varphi^{-1}(1) \subseteq H^{-1}(c)\cap \{ r \leq R_1\}. \end{align*} Consequently, for all $(v,\eta)\in \Crit\A^{H_1-c}_{q_0,q_1}$ we have $\max r\circ v \leq R_1$. In other words, for all $(v,\eta)\in \Crit\A^{H_1-c}_{q_0,q_1}$ we have $$ v([0,1]) \subseteq H_1^{-1}(c)\cap \{ r \leq R_1\}. $$ On the other hand, the Hamiltonians satisfy $$ H_1^{-1}(c)=\{H_0 = \varphi V+c\}\subseteq H_0^{-1}((-\infty, \sup V+c]).$$ Consequently, for all $(v,\eta)\in \Crit\A^{H_1-c}_{q_0,q_1}$ we have $$ v([0,1]) \subseteq H_1^{-1}(c)\cap \{ r \leq R_1\} \subseteq H_0^{-1}((-\infty, \sup V+c])\cap \{ r \leq R_1\} = \varphi^{-1}(1). $$ But on $\varphi^{-1}(1)$ the two Hamiltonians $H_1$ and $H$ coincide. Therefore, $\Crit \A^{H-c}_{q_0,q_1} = \Crit \A^{H_1-c}_{q_0,q_1}$. \hfill $\square$ \printbibliography \end{document}
2412.08437v1
http://arxiv.org/abs/2412.08437v1
Zeta and L functions of Voevodsky motives
\documentclass[12pt]{amsart} \usepackage{verbatim,amssymb,latexsym, amscd} \usepackage[T1]{fontenc} \usepackage[all]{xy} \usepackage{times} \usepackage[colorlinks,linkcolor=blue,citecolor=blue,urlcolor=red]{hyperref} \renewcommand{\phi}{\varphi} \renewcommand{\epsilon}{\varepsilon} \newcommand{\sA}{\mathcal{A}} \newcommand{\A}{\mathbf{A}} \newcommand{\C}{\mathbf{C}} \newcommand{\D}{\mathbb{D}} \newcommand{\F}{\mathbf{F}} \newcommand{\G}{\mathbf{G}} \renewcommand{\H}{\mathbf{H}} \newcommand{\I}{\mathbb{I}} \renewcommand{\L}{\mathbb{L}} \newcommand{\N}{\mathbf{N}} \renewcommand{\P}{\mathbf{P}} \newcommand{\Q}{\mathbf{Q}} \newcommand{\R}{\mathbf{R}} \newcommand{\Z}{\mathbf{Z}} \newcommand{\tr}{{\operatorname{tr}}} \newcommand{\gm}{{\operatorname{gm}}} \newcommand{\eff}{{\operatorname{eff}}} \newcommand{\rat}{{\operatorname{rat}}} \renewcommand{\hom}{{\operatorname{hom}}} \newcommand{\num}{{\operatorname{num}}} \newcommand{\proj}{{\operatorname{proj}}} \newcommand{\et}{{\operatorname{\acute{e}t}}} \renewcommand{\o}{{\operatorname{o}}} \newcommand{\op}{{\operatorname{op}}} \newcommand{\tot}{{\operatorname{tot}}} \newcommand{\near}{{\operatorname{near}}} \newcommand{\rd}{{\operatorname{red}}} \newcommand{\Serre}{{\operatorname{Serre}}} \newcommand{\Dir}{\operatorname{Dir}} \newcommand{\Eul}{\operatorname{Eul}} \newcommand{\DM}{\operatorname{DM}} \newcommand{\DA}{\operatorname{DA}} \newcommand{\Chow}{\operatorname{Chow}} \newcommand{\Hodge}{\operatorname{Hodge}} \newcommand{\Spec}{\operatorname{Spec}} \newcommand{\Sm}{\operatorname{Sm}} \newcommand{\Sch}{\operatorname{Sch}} \newcommand{\SmCor}{\operatorname{SmCor}} \newcommand{\Corr}{\operatorname{Corr}} \newcommand{\Ker}{\operatorname{Ker}} \newcommand{\Coker}{\operatorname{Coker}} \newcommand{\IM}{\operatorname{Im}} \newcommand{\Div}{\operatorname{Div}} \newcommand{\End}{\operatorname{End}} \newcommand{\Hom}{\operatorname{Hom}} \newcommand{\uHom}{\operatorname{\underline{Hom}}} \newcommand{\RHom}{\operatorname{RHom}} \newcommand{\RuHom}{\operatorname{R\underline{Hom}}} \newcommand{\Th}{\operatorname{Th}} \renewcommand{\Re}{\operatorname{Re}} \newcommand{\Sp}{\operatorname{sp}} \newcommand{\blue}[1]{{\color{blue} #1}} \newcommand{\red}[1]{{\color{red} #1}} \newcommand{\sF}{\mathcal{F}} \newcommand{\sH}{\mathcal{H}} \newcommand{\sI}{\mathcal{I}} \newcommand{\sM}{\mathcal{M}} \newcommand{\sN}{\mathcal{N}} \newcommand{\sR}{\mathcal{R}} \newcommand{\sT}{\mathcal{T}} \newcommand{\sX}{\mathcal{X}} \newcommand{\ff}{\mathfrak{f}} \newcommand{\fp}{\mathfrak{p}} \newcommand{\fq}{\mathfrak{q}} \newcommand{\un}{\mathbf{1}} \newcommand{\by}[1]{\overset{#1}{\longrightarrow}} \newcommand{\iso}{\by{\sim}} \newcommand{\yb}[1]{\overset{#1}{\longleftarrow}} \newcommand{\osi}{\yb{\sim}} \newcommand{\inj}{\hookrightarrow} \newcommand{\oo}{\displaystyle\operatornamewithlimits\otimes} \newcommand{\colim}{\varinjlim} \font\rus=wncyr10 scaled \magstep1 \DeclareFontFamily{U}{wncy}{} \DeclareFontShape{U}{wncy}{m}{n}{<5>wncyr5<6>wncyr6<7>wncyr7<8>wncyr8<9>wncyr9<10>wncyr10<11>wncyr10<12>wncyr6<14>wncyr7<17>wncyr8<20>wncyr10<25>wncyr10}{} \DeclareMathAlphabet{\cyr}{U}{wncy}{m}{n} \newcommand{\Sha}{\cyr{X}} \newcommand{\sha}{\cyr{x}} \newcommand{\Be}{\cyr{B}} \newcommand{\be}{\cyr{b}} \swapnumbers \newtheorem{thm}{Theorem}[section] \newtheorem{lemma}[thm]{Lemma} \newtheorem{prop}[thm]{Proposition} \newtheorem{cor}[thm]{Corollary} \theoremstyle{definition} \newtheorem{defn}[thm]{Definition} \newtheorem{rk}[thm]{Remark} \newtheorem{rks}[thm]{Remarks} \newtheorem{qn}[thm]{Question} \newtheorem{qns}[thm]{Questions} \newtheorem{ex}[thm]{Example} \newtheorem{exs}[thm]{Examples} \newcommand{\prf}{\noindent{\bf Proof. }} \newcounter{spec} \newenvironment{thlist}{\begin{list}{\rm{(\roman{spec})}}{\usecounter{spec}\labelwidth=20pt\itemindent=0pt\labelsep=10pt}}{\end{list}} \renewcommand{\thesubsection}{\arabic{section}.\Alph{subsection}} \setcounter{tocdepth}{1} \begin{document} \title{Zeta and $L$ functions of Voevodsky motives} \author{Bruno Kahn} \address{IMJ-PRG\\Case 247\\ 4 place Jussieu\\ 75252 Paris Cedex 05\\ France} \email{[email protected]} \date{Feb. 2012 and Dec. 2024} \subjclass[2020]{11M41, 11G09} \keywords{Zeta functions, motives, six functors formalism} \begin{abstract} We associate an $L$-function $L^\near(M,s)$ to any geometric motive over a global field $K$ in the sense of Voevodsky. This is a Dirichlet series which converges in some half-plane and has an Euler product factorisation. When $M$ is the dual of $M(X)$ for $X$ a smooth projective variety, $L^\near(M,s)$ differs from the alternating product of the zeta functions defined by Serre in 1969 only at places of bad reduction; in exchange, it is multiplicative with respect to exact triangles. If $K$ is a function field over $\F_q$, $L^\near(M,s)$ is a rational function in $q^{-s}$ and enjoys a functional equation. The techniques use the full force of Ayoub's six (and even seven) operations. \end{abstract} \maketitle \hfill Preliminary version \tableofcontents \section*{Introduction} Let $K$ be a global field, and let $X$ be a smooth projective $K$-variety. In \cite{serre}, after a long search (see \cite[letter avril 1963, p. 143]{corr}), Serre succeeded to define local factors of zeta functions associated to the cohomology groups of $X$, generalising both Artin's $L$-functions and the Hasse-Weil zeta function associated to an elliptic curve. When $K$ is a number field, this definition has been generalised by Scholl \cite{scholl} to `mixed motives', \emph{i.e.} objects of the abelian category defined by Jannsen and Deligne with systems of realisations in \cite{jannsen} and \cite{deligne}, thus allowing for a more conceptual reformulation of Beilinson's conjectures on special values of $L$-functions (see also Fontaine-Perrin Riou \cite{fpr} and Deninger \cite{den}).\footnote{There is no candidate for an abelian category of mixed motives in positive characteristic at present.} These definitions use $l$-adic cohomology and, unfortunately, depend on a still unproven conjecture: without even mentioning mixed motives, this is already the case in \cite{serre}. Namely, the local factor from \cite[\S 2]{serre} at a finite place $v$ of $K$ is of the form \begin{equation}\label{eq0} L_v^\Serre(H^i(X),s)=\det(1-\phi_v N(v)^{-s}\mid H^i(\bar X,\Q_l)^{I_v})^{-1} \end{equation} where $\phi_v$ is a geometric Frobenius at $v$, $l$ is a prime number not dividing $N(v)$ and $I_v$ is the inertia at $v$. Here $N(v)$ is the cardinality of the residue field at $v$. But for this expression to make sense as a complex function of the variable $s$, one needs at the very least that the coefficients of this inverse polynomial be complex numbers, while they are defined as $l$-adic numbers. If $v$ is a place of good reduction, \eqref{eq0} boils down by smooth and proper base change to $\det(1-\phi_v N(v)^{-s}\mid H^i(\bar X(v),\Q_l))^{-1}$ where $X(v)$ is the special fibre of a smooth model of $X$ at $v$, which does not depend on the choice of the prime $l$ and has rational coefficients by Deligne's proof of the ``Riemann hypothesis'' for $X(v)$ \cite{WeilI}. That this persists when $X$ has bad reduction at $v$ was proven by Terasoma in positive characteristic \cite{ter} but remains open over number fields in general \cite[$C_5$]{serre}. See \cite[5.6.3, 5.6.4]{zetaL} for a detailed discussion. Another issue is that the $L$-functions of \eqref{eq0} seem difficult to manipulate: if $0\to M'\to M\to M''\to 0$ is a short exact sequence of mixed motives, the equality $L^\Serre(M,s)=L^\Serre(M',s)L^\Serre(M'',s)$ fails in general. Let us now pass from mixed motives to \emph{triangulated motives}. Let \break $\DM_\gm(K,\Q)$ denote Voevodsky's triangulated category of geometric motives over $K$, with rational coefficients \cite{voetri,mvw}. In this article, we associate to any object $M\in \DM_\gm(K,\Q)$ a Dirichlet series $L^\near(M,s)$, called the \emph{nearby $L$-function} of $M$, which has the following properties: \begin{description} \item[Convergence] $L^\near(M,s)$ has a finite abscissa of convergence. \item[Multiplicativity] If $M'\to M\to M''\by{+1}$ is an exact triangle, then \[L^\near(M,s)=L^\near(M',s)L^\near(M'',s).\] In particular, $L^\near(M[1])=L^\near(M,s)^{-1}$. \item[Tate twists] $L^\near(M(1),s) = L^\near(M,s+1)$. \item[Euler decomposition] There is a factorisation \[L^\near(M,s)=\prod_v L^\near_v(M,s)\] where $v$ runs through the finite places of $K$ and $L^\near_v(M,s)$ is an \emph{$N(v)$-Euler factor}, i.e. a rational function in $N(v)^{-s}$ with $\Q$ coefficients whose zeroes and poles are $N(v)$-Weil numbers. \item[Inductivity] Let $L/K$ be a finite extension and $f:\Spec L\to \Spec K$ be the corresponding morphism. Then \[L^\near(M,s)=L^\near(f_!M,s)\] for any $M\in \DM_\gm(L,\Q)$, where $f_!:\DM_\gm(L,\Q)\to \DM_\gm(K,\Q)$ is the push-forward functor. \item[``Normalisation''] Let $X$ be a smooth projective $K$-variety. If $v$ is a place of good reduction for $X$, then $L^\near_v(M(X)^*,s) = \zeta(X(v),s)$, where $X(v)$ is the special fibre of a smooth projective model of $X$ at $v$ (this zeta function does not depend on the choice of such a model, \cite[Prop. 5.6]{zetaL}). Here $M(X)^*$ is the dual of the motive $M(X)$ of $X$. This extends to $L^\near(\Phi(N)^*,s)$, where $N$ is a Chow motive and $v$ is a place of good reduction for $N$, where $\Phi:\Chow(K,\Q)\to \DM_\gm(K,\Q)$ is Voevodsky's functor \cite[Prop. 2.1.4]{voetri}. \item[Rationality and functional equation] If $K$ has positive characteristic with field of constants $k\simeq \F_q$, then $L^\near(M,s)$ is a $q$-Euler factor and enjoys a functional equation \[L^\near(M^*,1-s) = A(-q)^{-Bs} L^\near(M,s)\] where $A\in \Q^*$ and $B\in \Z$ are some explicit numbers (Theorem \ref{t9.2}). \item[Archimedean primes] If $K$ is a number field, to any archimedean place $v$ of $K$ is associated a ``local gamma factor'' $\Gamma_v(M,s)$ which is multiplicative in $M$ and agrees with that of a smooth projective variety $X$ as in Serre \cite[\S 3]{serre} for $M=M(X)^*$. \end{description} The definition of $L^\near(M,s)$ purely rests on formal properties of the rigid $\otimes$-triangulated category $\DM_\gm(K,\Q)$ and its generalisations over a base, hence, \emph{in fine}, on algebraic cycles. In particular, it is independent of $l$ \emph{a priori}. The superscript `near' is there to point out that this definition is not very different from the classical one for smooth projective varieties, but also that it can be computed in terms of Ayoub's specialisation systems (nearby cycles): Theorem \ref{t9.1}. The quotation marks at normalisation are there because this property is sufficient, together with multiplicaitivity, only to characterise $L^\near$ up to finite products of Euler factors. It would be nice to find extra properties which make this uniqueness true on the nose. If $X$ is smooth projective, $L^\near_v(M(X)^*,s)$ is in general different from $L_v^\Serre(X,s)$ when $v$ is a place of bad reduction, where $L_v^\Serre(X,s)$ is the alternating product of the functions of \eqref{eq0}: see \S \ref{s9.D.2} for the example of an elliptic curve with multiplicative reduction. To explain the idea behind its definition, note that there is a competing local factor, replacing $H^i(\bar X,\Q_l)^{I_v}=H^0(I_v,H^i(\bar X,\Q_l))$ with $H^1(I_v,H^i(\bar X,\Q_l))=\break H^i(\bar X,\Q_l)_{I_v}(-1)$ (compare \cite[note 164.23]{corr}): $L^\near(M(X)^*,s)$ is a ``multiplicative average'' of these two alternating products, see Theorem \ref{t8.2} b) and Definition \ref{d9.1}. After getting the idea to define triangulated $L$-functions and finding how, I wondered if Grothendieck would himself have had a similar concern. And indeed he raises the issue twice, in \cite[letter 30.9.1964, top p. 196]{corr} responding to the letter quoted above where Serre formulates the question, and in \emph{loc. cit.}, letter of 3 and 5 Oct. 1964, c) p. 202. In the first letter, he claims (unless mistaken) that the formula of \cite{serre} is indeed multiplicative\footnote{\emph{Lorsqu'on veut à tout prix une fonction $L$ qui dépende multiplicativement de $M$, il me semble hors de doute que la définition que tu préconises est la meilleure.}}. This looks strange, since taking invariants under inertia is not a (right) exact functor. In the second, he backtracks and prefers a ``définition bébête'' where $L(M,s)$ (for a mixed motive $M$) is defined as the product of $L$-functions of the factors of its semi-simplification. It will turn out, after the fact, that the latter idea is the right one for triangulated motives, see \eqref{eq9.3}. For mixed motives, however, Grothendieck's first proposal as implemented in the references given at the beginning seems to be the most interesting and the most profound. But it depends on conjectures\dots Thus I hope that the present construction, its unconditionality and its multiplicative property will be helpful for making progress towards the Beilinson conjectures. This version is preliminary because I stopped at a `honest' functional equation in the sense of Grothendieck's first letter quoted above (see \cite[p. 197]{corr}). To get a nicer one as in \cite{serre} would involve giving a formula à la Grothendieck-Ogg-\v Safarevi\v c \cite[Exp. X]{SGA5} for the Euler Poincaré characteristic of a motive of the form $f_!M$, where $f=S\to \Spec \F_q$ is a smooth projective curve. While this can obviously be done via an $l$-adic realisation, it is not clear (to me) that the local terms are independent of $l$, see Question \ref{q10.1}. The same issue arises in higher dimension when using Takeshi Saito's theory of the characteristic cycles \cite{tsaito}. Joseph Ayoub has suggested to use the Galois action on his ``full'' specialisation systems $\Psi_x$ \cite[Déf. 10.14]{ayoubetale}; I hope to come back to this in a further version. \subsection*{Strategy} We first associate a zeta function to any motive in $\DM_\gm(k,\Q)$ when $k$ is a finite field, by using categorical traces of powers of Frobenius in this rigid $\otimes$-category. One key result is that the zeta function of the ``Borel-Moore'' motive of $X$ is the zeta function of $X$, when $X$ is an $\F_q$-scheme of finite type (Corollary \ref{c1}). We then extend this to motives over a $\Z$-scheme of finite type $S$ by the usual product formula over the closed points of $S$. Here a second key result is a trace formula, which allows us to compute these zeta functions as zeta functions of motives over $\F_q$ when $S$ is an $\F_q$-scheme (Theorem \ref{t3.1}): the proof, relying on the previous result, is almost purely motive-theoretic but we cannot avoid using the $l$-adic realisation functor (hence the trace formula of \cite{SGA5}), because of a problem of idempotents. We also get a functional equation when $S$ is proper over $\F_q$ (\emph{ibid.}). The six functors formalism established by Ayoub in his thesis \cite{ayoub} is central in these definitions and properties. All these zeta functions are rational functions of $p^{-s}$ in characteristic $p$. In Corollary \ref{c6.1}, we show that they converge (in some half-plane) also in characteristic $0$, \emph{i.e.} when $S$ is dominant over $\Spec \Z$. In Section \ref{s9} we arrive at the heart of the matter: the case of motives over a global field $K$. Since $\Spec K$ is not of finite type over $\Spec \Z$, the issue is to get a reasonable definition out of the previous work. We do it in two steps: first define a ``total'' $L$-function (Definition \ref{l9.1}), and then deduce the ``nearby'' $L$-function from it (Definition \ref{d9.1}), by applying Lemma \ref{l2}. In characteristic $p>0$, we get a functional equation in Theorem \ref{t9.2}. All this latter work uses heavily the six operations again, and even the seventh (the unipotent specialisation system). \subsection*{Acknowledgements} This work has been in gestation since 1998. Since then, Science has made progress and much of this progress has been incorporated here. It has been announced at conferences a number of times, the most recent being at Regensburg in 2012 \cite{zetaLRegensburg}. The reasons of my procrastination are not entirely clear. I thank Joseph Ayoub, Mikhail Bondarko, Fr\'ed\'eric D\'eglise, Luc Illusie and Amílcar Pacheco for helpful exchanges, with a special mention to Ayoub: he kindly wrote up his paper \cite{ayoubetale} on the $l$-adic realisations upon my request and helped me at a large number of places in this manuscript. As an exercise, the reader may count the number of times when I credit him for a proof or an idea. \enlargethispage*{30pt} \subsection*{Conventions and notation} As usual, a $\otimes$-category means a symmetric monoidal unital additive category, and a $\otimes$-functor is an additive strong symmetric monoidal functor. We refer to \cite[App. 8.A]{mvw} for tensor triangulated categories. \numberwithin{equation}{section} \section{The rigid $\otimes$-category $\DM_\gm(k,\Q)$}\label{s1} Let $k$ be a field. We consider the category $\DM_\gm(k,\Q)$ defined by Voevodsky in \cite{voetri} (here, with rational coefficients). It is provided with a covariant functor ``motive'' $M:\Sm(k) \to \DM_\gm(k,\Q)$, where $\Sm(k)$ is the category of smooth separated $k$-schemes. In \cite{voetri}, two properties of $\DM_\gm(k,\Q)$ are established when $k$ is of characteristic $0$: \begin{itemize} \item It is a rigid tensor pseudo-abelian triangulated category, generated by the motives of smooth projective varieties as such. We write $M^*$ for the dual of a motive $M\in \DM_\gm(k,\Q)$. \item For any separated $k$-scheme of finite type $X$, there is an associated motive with compact supports $M_c(X)\in \DM_\gm(k,\Q)$ such that $M_c(X)=M(X)^*(d)[2d]$ if $X$ is smooth of pure dimension $d$, $M_c(X)=M(X)$ if $X$ is proper and, if $Z\by{i}X$ is a closed subset with complementary open $U\by{j} X$, one has an exact triangle of the form \begin{equation}\label{eq1.1} M_c(Z)\by{i_*} M_c(X)\by{j^*} M_c(U)\by{+1}. \end{equation} \end{itemize} \begin{thm}\label{p1.1} These properties hold for any $k$. \end{thm} \begin{proof} When $k$ is perfect, see \cite[App. B]{motiftate} for the first property and \cite[\S 5.3]{kelly} for the second. In general, let $k^p$ be the perfect closure of $k$. Then \cite[Prop. 4.5]{adjoints} or \cite{suslininsep} show that the base change functor \[\DM_\gm(k,\Q)\to \DM_\gm(k^p,\Q)\] is an equivalence of categories. \end{proof} (Theorem \ref{p1.1} is even true with coefficients $\Z[1/p]$, where $p$ is the exponential characteristic of $k$, but we won't use this refinement.) We shall need: \begin{lemma}\label{l1.4} Let $f:X\to Y$ be a finite, surjective morphism of smooth $k$-schemes of generic degree $d$, where $k$ is assumed to be perfect. Then $f$ induces morphisms $f^*:M_c(Y)\to M_c(X)$ and $f_*:M_c(X)\to M_c(Y)$ such that $f_*f^*=d$. Moreover, \begin{itemize} \item If $f$ is radicial, we also have $f^*f_*=d$. In particular $f^*$ is an isomorphism. \item If $f$ is a Galois covering of group $G$, then $f^*$ induces an isomorphism $M_c(Y)\iso \epsilon_GM_c(X)$ where $\epsilon_G$ is the idempotent $\frac{1}{G}\sum_{g\in G} g$. \end{itemize} \end{lemma} \begin{proof} By duality, we reduce to the same statement for $M(X)$ and $M(Y)$. Then they are already true on the level of finite correspondences. \end{proof} \begin{defn}\label{d1.1} We write $\DM_\gm^\eff(k,\Q)\subset \DM_\gm(k,\Q)$ for the full subcategory of effective geometric motives and, if $n\ge 0$, $d_{\le n}\DM_\gm^\eff(k,\Q)$ for the thick triangulated subcategory of $\DM_\gm^\eff(k,\Q)$ generated by motives of smooth varieties of dimension $\le n$. \end{defn} \begin{prop}\label{p1.2} If $\dim X\le n$, then $M_c(X)\in d_{\le n}\DM_\gm^\eff(k,\Q)$. \end{prop} \begin{proof}We may assume $k$ perfect. Induction on $n$. The case $n=0$ is clear because $M_c(X)=M_c(X_\rd)$ and $X_\rd$ is a (proper) étale $k$-scheme. Suppose $n>0$. By closed Mayer-Vietoris, we reduce to $X$ irreducible and then to $X$ a variety. By de Jong's theorem, there exists an alteration $f:\tilde X\to X$ with $\tilde X$ smooth. Choose a smooth open subset $U$ of $X$ over which $f$ is finite, and let $V=f^{-1}(U)$. By induction, $M_c(V)$ is in $d_{\le n}\DM_\gm^\eff(k,\Q)$ and so is $M_c(U)$ as a direct summand of $M_c(V)$ (Lemma \ref{l1.4}). By induction again, $M_c(X)\in d_{\le n}\DM_\gm^\eff(k,\Q)$. \end{proof} \begin{rk} By a similar reasoning, one can see that $d_{\le n}\DM_\gm^\eff(k,\Q)$ is generated by motives of smooth projective varieties of dimension $\le n$. \end{rk} \section{The case of a finite field} \subsection{The ubiquity of Frobenius}\label{s2.A} Let $k=\F_q$ be a finite field with $q$ elements. Consider the category $\Sch(k)$ of separated $k$-schemes of finite type, viewed as a symmetric monoidal category for the fibre product over $k$. The identity functor of $\Sch(k)$ has a canonical $\otimes$-endomorphism: the Frobenius endomorphism, namely: \begin{itemize} \item Every object $X\in \Sch(k)$ has its ``absolute" Frobenius endomorphism $F_X$: $F_X$ is the identity on the underlying space of $X$ and is given by $f\mapsto f^q$ on the structural sheaf. \item If $f:X\to Y$ is a morphism in $\Sch(k)$, the diagram \[\begin{CD} X@>F_X>>X\\ @V{f}VV @V{f}VV\\ Y@>F_Y>>Y \end{CD}\] commutes. \item One has $F_{X\times_k Y} = F_X\times_k F_Y$ for any $X,Y\in \Sch$. \end{itemize} Starting from this situation, we can extend it to other categories associated with $k$. For example, if $A$ is a commutative ring then $\DM_\gm^\eff(k,A)$ is obtained from the full subcategory $\Sm(k)$ of smooth $k$-schemes via the string of functors \begin{multline}\label{eq2.2} \Sm(k)\to \SmCor(k,A)\to C^b(\SmCor(k,A))\\ \to K^b(\SmCor(k,A))\to K^b(\SmCor(k,A))/(MV+HI)\to DM_\gm^\eff(k,A). \end{multline} Here, $\SmCor(k,A)$ is the category of finite correspondences on smooth schemes with coefficients in $A$, $MV$ and $HI$ are the ``Mayer-Vietoris" and ``homotopy invariance" relations and the last step is idempotent completion. At each step, the Frobenius endomorphism extends: for $\SmCor(k,A)$ because it commutes with finite correspondences; on the third and fourth for formal reasons; on the fifth because it acts on the $MV$ and $HI$ relations, and on the last once again for formal reasons. To pass from $\DM_\gm^\eff(k,A)$ to $\DM_\gm(k,A)$ amounts to $\otimes$-inverting the Tate object $\Z(1)$, and once again Frobenius passes through this formal operation. Similarly, we may take coefficients in any ring $A$ rather than $\Z$. We also get Frobenius endomorphisms on categories obtained from categories of (pre)sheaves via the following simple trick: let $\sF$ be a contravariant functor from, say, $\Sm(k)$ to some category. We define the Frobenius endomorphism $F_\sF$ of $\sF$ by the formula \[F_\sF(X) = \sF(F_X)=\sF(X)\to \sF(X).\] Suppose that $\sF$ takes values in the category of sets: in the special case where it is representable, say $\sF=y(Y)$, one finds \[F_{y(Y)} = y(F_Y)\] for tautological reasons, where $y$ is the Yoneda embedding. \begin{rk} This is the \emph{inverse} of the convention of \cite[XV.2.1]{SGA5}! Cf. \emph{loc. cit.} bottom p. 453. \end{rk} In this way, categories like $\DM^\eff(k,A)$ or $\DM^\eff_\et(k,A)$ carry their own Frobenius automorphism, compatible with the one of $\DM_\gm^\eff(k,A)$ via the comparison functors if need be. \begin{ex}\label{ex2.1} We may describe $\Z(1)$ as $\G_m[-1]$. The Frobenius of the group scheme $\G_m$, hence of the abelian sheaf $\G_m$, coincides with multiplication by $q$. Hence $F_{\Z(1)}$ is also multiplication by $q$. \end{ex} Let $E/k$ be a finite extension of degree $m$, and let $f:\Spec E\to \Spec k$. We have a pair of adjoint functors \[\DM_\gm(k,\Q)\begin{smallmatrix} f^*\\\rightleftarrows\\f_*\end{smallmatrix}\DM_\gm(E,\Q).\] \begin{lemma}\label{l2.2a} a) For $M\in \DM_\gm(k,\Q)$, $F_{f^*M}=f^*F_M^m$.\\ b) For $M\in \DM_\gm(E,\Q)$, $F_{f_*M}^m = f_*F_M$. \end{lemma} \begin{proof} a) Construction \eqref{eq2.2} shows that any object $N$ of $\DM_\gm^\eff(k,\Q)$ is isomorphic to a direct summand of an object represented by a bounded complex of finite correspondences. In turn, $M$ is represented by an object of the form $(N,i)$ for $N$ as above and $i\in \Z$. The functor $f^*$ respects these constructions. Thus the statement reduces to the case $M=M(X)$ where $X$ is a smooth variety, when it is clear. Same reasoning for b). \end{proof} \subsection{A theorem of May} Let $\sT$ be a tensor triangulated category, i.e. a triangulated category provided with a symmetric monoidal structure which verifies axioms (TC1) -- (TC5) of May \cite[\S 4]{may}. Let $\un$ be the unit object of $\sT$. As in any symmetric monoidal category, an endomorphism $f$ of a strongly dualisable object $M$ has a \emph{trace} $\tr(f)\in \End_\sT(\un)$, defined as the composition \begin{equation}\label{eq2.3} \un \by{\eta} M\otimes M^*\by{f\otimes 1} M\otimes M^*\by{\sigma} M^*\otimes M\by{\epsilon}\un. \end{equation} Let $e$ be an endomorphism of the identity functor of $\sT$. Then \begin{thm}\label{t2.1} Let $M'\to M\to M''\by{+1}$ be an exact triangle in $\sT$. We have the equality \[\tr(e_M)=\tr(e_{M'})+\tr(e_{M''}).\] \end{thm} \begin{proof} Although this is only stated in \cite{may} for $e$ the identity, May's proof for this special case directly generalises. More precisely, one can insert $e$'s in the diagram in the middle of \cite[p. 55]{may}, just below the occurence of the $\gamma$'s, without affecting its commutativity. \end{proof} \subsection{Zeta functions of endomorphisms} Let $\sA$ be a rigid additive $\otimes$-category, and let $N$ be an object of $\sA$. Let $\un$ be the unit object of $\sA$ and let $K=\End(\un)$, assumed to be a field of characteristic $0$. In \cite[Def. 3.1]{modq}, we gave the following definition: \begin{defn} \label{d2a} For $f\in \End(M)$, its \emph{Z function} is \[Z(f,t) = \exp(\sum_{n\ge 1} \tr(f^n)\frac{t^n}{n})\in K[[t]]\] where $\tr$ is the categorical trace described in the previous subsection. \end{defn} Following \cite[Def. 5.1]{modq}, we say that $\sA$ is \emph{of homological origin} if it is abelian semi-simple and if it is $\otimes$-equivalent to $\sA'/\sN$, where $\sA'$ is a rigid $\otimes$-category admitting a strong $\otimes$-functor with values in the category of $\Z/2$-graded $L$-vector spaces for some extension $L$ of $K$, and $\sN$ is the ideal of morphisms universally of trace $0$. By \cite[Th. 3.2, Rem. 3.3 and Th. 5.6]{modq}, we have \begin{thm}\label{t2.3} Assume that $\sA$ is of homological origin. Then, for any $(N,f)$ as in Definition \ref{d2a}, $Z(f,t)\in K(t)$. If $f$ is invertible, we have the functional equation \[Z(f^{-1},t^{-1})= (-t)^{\chi(M)} \det(f) Z(f,t)\] where $\chi(M)=\tr(1_M)$ and $\det(f)$ is a certain element of $K^*$ which may be obtained by specialisation from the identity \[\det(1-ft)= Z(f,t)^{-1}.\] \end{thm} In \cite[Th. A.41]{zetaL}, a slightly simpler proof of Theorem \ref{t2.3} is given, as well as a more explicit formula for $\det(f)$: if $Z(f,t)=\frac{\prod_{i=1}^m(1-\alpha_it)}{\prod_{j=1}^n(1-\beta_jt)}$ over the algebraic closure of $K$, then \[\det(f) = \frac{\prod_{i=1}^m \alpha_i}{\prod_{j=1}^n \beta_j}.\] \subsection{Number of points and zeta functions of pure motives} In \cite[pp. 80/81]{kleiman}, Kleiman defines the number of points and the Z function of effective homological motives, hence \emph{a fortiori} of an effective Chow motive $N=(X,p)$ where $X$ is a smooth projective $k$-variety and $p$ is a projector on $X$, by \[N_s(N)=\langle F_X^s,{}^t p\rangle,\quad Z(N,t)=\exp(\sum_{s=1}^\infty N_s(N)\frac{t^s}{s})\] where $\langle, \rangle$ is the intersection product. For $f=F_N$, the Frobenius endomorphism of $N$, this expression coincides with that of Definition \ref{d2a} in view of the following lemma, which should have been in \cite{zetaL}. \begin{lemma}\label{l2.4} a) Let $X,Y$ be two smooth projective varieties, and let $f\in \Corr(X,Y)$, $g\in \Corr(Y,X)$ be two correspondences. Then \[\tr(g\circ f) = \langle {}^tf,g\rangle.\] b) For $N=(X,p)$, $N_s(N)=\tr(F_N^s)$ and $Z(F_N,t)=Z(N,t)$. \end{lemma} \begin{proof} a) In any rigid $\otimes$-category $\sA$, we have the formula \begin{equation}\label{eq2.4} \tr(g\circ f) = {}^t(\iota_{AB}^{-1}f)\circ \iota_{BA}^{-1} g \end{equation} \cite[7.3]{akos}, where $f:A\to B$, $g:B\to A$ and $\iota_{AB}$ is the adjunction isomorphism $\Hom(\un,A^*\otimes B)\iso \Hom(A,B)$ (\emph{loc. cit.}, (6.2)) and similarly for $\iota_{BA}$. Applying this with $A=h(X)$, $B=h(Y)$ in $\Chow(k,\Q)$ and using the fact that $h(X)^*\simeq h(X)\otimes \L^{-\dim X}$, $h(Y)^*\simeq h(Y)\otimes \L^{-\dim Y}$ with $\L$ the Lefschetz motive, we get the usual formulas \[\Hom(h(X),h(Y))\osi \Hom(\L^{\dim X},h(X\times Y)) =CH^{\dim X}(X\times Y)\otimes \Q\] and similarly for $\Hom(h(Y),h(X))$. Thus, in the right hand side of \eqref{eq2.4}, $\iota_{BA}^{-1} g$ is simply $g$ viewed as a cycle class in $CH^{\dim Y}(Y\times X)\otimes \Q$ and ${}^t(\iota_{AB}^{-1}f)$ is $f$ viewed as a cycle class in $CH^{\dim X}(X\times Y)\otimes \Q$. By the formula for the composition of correspondences, this right hand side is $\langle {}^tf,g\rangle$. b) We apply a) with $X=Y$, $f=p$, $g=F_X^s$, noting that $F_N^s=p\circ F_X^s$. \end{proof} By Theorem \ref{t2.3} and the existence of a Weil cohomology, we get that $Z(N,t)$ is a rational function of $t$ and satisfies a functional equation of the form of this theorem. Adding the main result of \cite{WeilI}, we get a more precise result: \begin{thm}\label{t2.4} Assume that $N$ is effective and is a direct summand of $h(X)$ where $X$ is a smooth projective variety of dimension $n$. Then the roots of the numerator and denominator of $Z(N,t)$ are effective Weil $q$-numbers of weights $\le 2n$. \end{thm} Here, a Weil $q$-number of weight $i$ is an element $\alpha\in \bar \Q$ such that $|\sigma(\alpha)| =q^i$ for any embedding $\sigma:\bar \Q\inj \C$; $\alpha$ is \emph{effective} if it is an algebraic integer. \subsection{Covariance and contravariance}\label{s2.E} In \cite[Prop. 2.1.4]{voetri}, Voevodsky defined a functor \[\Phi^\eff:\Chow^\eff(k,\Q)\to \DM_\gm^\eff(k,\Q)\] which induces similar functors $\Phi:\Chow(k,\Q)\to \DM_\gm(k,\Q)$ and $\phi^\o:\Chow^\o(k,\Q)\to \DM_\gm^\o(k,\Q)$ where the last ones are the categories of birational motives defined in \cite{ks} and \cite{birat-tri}. These functors are covariant if one takes the \emph{covariant} convention on Chow motives: the ``graph'' functor is covariant. On the other hand, the computation of zeta functions using traces of powers of Frobenius in $\Chow(k,\Q)$ is done in the previous section with the \emph{contravariant} convention, which might conceivably be an issue. More generally, if $\sA$ is a rigid $\otimes$-category, then the opposite\footnote{We prefer the term `opposite' to the older term `dual', which may cause confusion in this and other contexts.} category $\sA^\op$ (same objects, change the sense of morphisms) is also a rigid $\otimes$-category, but changing the sense of morphisms does not preserve the shape of \eqref{eq2.3}, which seems to create an issue. This is not the case: \begin{lemma}\label{l2.3} Let $f:A\to A$ be an endomorphism of $A\in \sA$. Then $\tr_\sA(f)=\tr_{\sA^\op}(f)$. \end{lemma} \begin{proof} Consider the covariant strong $\otimes$-functor $\sA\to \sA^\op$ given by $A\mapsto A^*$: it sends $f:A\to B$ to its transpose ${}^t f:B^*\to A^*$. Thus, for $B=A$, \[\tr_\sA(f) = \tr_{\sA^\op}({}^t f).\] But $\tr_{\sA^\op}({}^t f)=\tr_{\sA^\op}(f)$ \cite[p. 151]{akos}. \end{proof} This lemma shows that the categorical trace commutes with strong $\otimes$-functors, be they covariant or contravariant. \subsection{Number of points of geometric motives} As seen in \S \ref{s2.A}, the identity functor of $\DM_\gm(k,\Q)$ has a canonical $\otimes$-endomorphism: the Frobenius endomorphism, that we denote by $F$. For any $M\in \DM_\gm(k,\Q)$, we write $F_M$ for the corresponding endomorphism of $M$. \begin{defn}\label{d1}For $n\in \Z$, $\sharp_n(M)=\tr(F_M^n)$; for $n=1$ we set $\sharp_1=\sharp$. \end{defn} \begin{thm}\label{t2.2} a) For each $n\in \Z$, $\sharp_n$ is an Euler-Poincar\'e characteristic and defines a ring homomorphism \[\sharp_n:K_0(\DM_\gm(k,\Q))\to\Q.\] b) $\sharp_n$ takes values in $\Z[1/q]$; for $n\ge 1$ its restriction to $\DM_\gm^\eff(k,\Q)$ takes values in $\Z$, and induces a ring homomorphism \[\overline\sharp_n:K_0(\DM_\gm^\o(k,\Q))\to \Z/q^n\] where $\DM_\gm^\o(k,\Q)$ is the category of triangulated birational motives \cite{ks}.\\ c) We have the identities: \[\sharp_n(M[1]) = -\sharp_n(M);\qquad \sharp_n(M(1)) = q^n\sharp_n(M).\] d) If $X$ is smooth projective, then $\sharp_n(M(X))=|X(\F_{q^n})|$ for all $n\ge 1$, and $\sharp_0(M(X))$ is the Euler characteristic of $X$.. \end{thm} Here the groups $K_0$ considered are those of triangulated categories as in \cite[VIII.2]{SGA5}. \begin{proof} a) Multiplicativity is a general fact for rigid tensor categories (use that $F_{M\otimes N}=F_M\otimes F_N$). Additivity follows from Theorem \ref{t2.1}. b) Bondarko has proven that the maps \[K_0(\Chow^\eff(k,\Q))\by{K_0(\Phi^\eff)} K_0(\DM_\gm^\eff(k,\Q))\] \begin{equation}\label{eq2.1} K_0(\Chow(k,\Q))\by{K_0(\Phi)} K_0(\DM_\gm(k,\Q)) \end{equation} induced by the corresponding functors are bijective \cite[Th. 6.4.2 and Cor. 6.4.3]{bondarko1}. Here, the left groups are $K_0$ of \emph{additive} categories. The same holds for the homomorphism \[K_0(\Chow^\o(k,\Q))\by{K_0(\Phi^\o)} K_0(\DM_\gm^\o(k,\Q))\] by \cite[Prop. 8.1.1]{bondarko}. The result then follows from \cite[Th. 8.1 and 9.1]{modq}. c) The first identity is a special case of a) (consider the exact triangle $M\to 0\to M[1]$). The second one follows from multiplicativity and the formula $\sharp_n(\Z(1))=q^n$ (Example \ref{ex2.1}). d) We use the strong $\otimes$-functor $\Phi:\Chow(k,\Q)\to \DM_\gm(k,\Q)$ as in b): it sends the Chow motive $h(X)$ to $M(X)$. The conclusion follows by Lemma \ref{l2.3}. \end{proof} \begin{lemma}\label{l2.2} Let $E,f,m$ be as before Lemma \ref{l2.2a}. For $M\in \DM_\gm(E,\Q)$ and $n> 0$, one has \[\tr(f_*F_M^n) = \begin{cases} 0 &\text{if $m\nmid n$}\\ \tr(F_M^{n/m}) &\text{if $m\mid n$.} \end{cases} \] For $n=0$, we have $\chi(f_*M) = [E:k]\chi(M)$. \end{lemma} \begin{proof} Following a hint of J. Ayoub, we do as for induced representations: as $f^*$ is monoidal, we have $\tr(f_*F_M^n)=\tr(f^*f_*F_M^n)$. Write $f^*f_*M=\bigoplus_{r=0}^{m-1} (\phi_E^r)^*M$, where $\phi_E$ is the Frobenius generator of $\Gamma=Gal(E/k)$. Then the matrix of $f^*f_*F_M$ relatively to this decomposition is \[\begin{pmatrix} 0 & 0&\dots &0& (\phi_E^{m-1})^*\phi\\ \phi & 0& \dots&0& 0\\ 0 & \phi_E^*\phi &\dots&0& 0\\ &&\ddots\\ 0&\dots& &(\phi_E^{m-2})^*\phi&0 \end{pmatrix}\] where $\phi:M\iso \phi_E^*M$ is a relative Frobenius morphism; moreover we have the relation \[(\phi_E^{m-1})^*\phi\circ\dots\circ \phi_E^*\phi\circ \phi=F_M.\] This can be checked on $M=M(Y)$, $Y$ a smooth $E$-scheme. The conclusion follows. \end{proof} \subsection{Two twisting lemmas}\label{s2.F} In the next subsection, we shall need the following generalisation of a well-known lemma (cf. A. Pacheco \cite[p. 283]{pacheco}: the idea seems to go very far back). We recall the notion of twisting by a $1$-cocycle from \cite[I.5.3 and III.1.3]{cg}. Let $U$ be a quasi-projective $k$-variety, and let $G$ be a finite group of order $m$ acting on $U$ on the right. Then the geometric quotient $V=U/G$ exists. For each $\sigma\in G$, we define a $k$-variety $U^{(\sigma)}$ mapping to $V$ as follows: Let $E/k$ be ``the'' extension of $k$ of order $m$, and $\Gamma=Gal(E/k)$. Then $\Gamma\times G$ acts on $U_E$. Let $\phi$ be the arithmetic Frobenius of $k$, $\phi_E$ its image in $\Gamma$ and $H_\sigma = \langle (\phi_E^{-1},\sigma)\rangle$; we set $U^{(\sigma)}=U_E/H_\sigma$. The projection $U_E\to U\to V$ is $H_\sigma$-invariant, which defines $f^{(\sigma)}:U^{(\sigma)}\to V$. Write also $\pi_\sigma$ for the projection $U_E\to U^{(\sigma)}$. \begin{lemma}\label{l2.5} We have $U^{(1)}=U$, we write $f:=f^{(1)}$. For a general $\sigma$, the square \[\begin{CD} U_E@>\pi_\sigma>> U^{(\sigma)}\\ @VVV @VVV\\ \Spec E@>>> \Spec k \end{CD}\] is Cartesian; in particular, $\pi_\sigma$ is a Galois (étale) covering of group $\Gamma$. \end{lemma} \begin{proof} By faithfully flat descent \cite[Cor. VIII.5.4]{SGA1}, it suffices to see this after base-changing to $\Spec E$. Then the left column becomes $\Gamma\times U_E \to \Gamma\times \Spec E$. But, by construction, there exists a $k$-isomorphism $(U^{(\sigma)})_E\simeq U_E$ which converts $(\pi_\sigma)_E$ into the projection $\Gamma\times U_E\to U_E$ given by the canonical map $\Gamma\to *$. Hence the conclusion. \end{proof} \begin{lemma}\label{ltwist} Suppose that $f$ is a $G$-torsor (i.e. an étale Galois covering). Then we have \[\frac{1}{m}\sum_{\sigma\in G}|U^{(\sigma)}(k)| = |V(k)|. \] \end{lemma} \begin{proof} We have \[V(k)=V(\bar k)^\phi.\] The map $f:U(\bar k)\to V(\bar k)$ is surjective and $G$ acts simply transitively on its fibres. Let $x\in V(k)$. Pick $y\in U(\bar k)$ mapping to $x$. Then $\phi^{-1}y$ is in the fibre of $x$, which implies that $y=\phi y\sigma $ for a unique $\sigma\in G$. Thus $y$ defines a $k$-rational point of $U^{(\sigma)}$. If $y'\in f^{-1}(x)$ is another point, then $y'=y\tau $ for a unique $\tau\in G$, and \[y'=\phi y\sigma\tau =\phi y'\tau^{-1}\sigma\tau \] so $y'\in U^{(\tau^{-1}\sigma\tau)}(k)$, and $y'\in U^{(\sigma)}(k)$ if and only if $\tau\in Z_{G}(\sigma)$. Summarising: there exists a well-defined conjugacy class $\bar \sigma(x)\subset G$ such that $f^{-1}(x)$ is a disjoint union of subsets $f^{-1}(x)_\sigma$ consisting of $r$ elements of $U^{(\sigma)}(k)$ for $\sigma$ running through $\bar \sigma(x)$, where $r=|Z_{G}(\sigma)|=|G|/|\bar \sigma(x)|$. If $\bar \sigma$ is now a given conjugacy class, let \[V(k)_{\bar\sigma}=\{x\in V(k)\mid \bar \sigma(x)=\bar \sigma\} \; \text{(possibly empty!)}.\] Then, for every $\sigma\in \bar \sigma$, the map $U^{(\sigma)}(k)\to V(k)_{\bar \sigma}$ is a torsor under $Z_{G}(\sigma)$. In particular, $|U^{(\sigma)}(k)| = r|V(k)_{\bar \sigma}|$ and \[\sum_{\sigma\in \bar\sigma} |U^{(\sigma)}(k)| = |\bar \sigma|r |V(k)_{\bar \sigma}| = m|V(k)_{\bar \sigma}|. \] Collecting over the conjugacy classes of $G$, we get (iii). \end{proof} We now need a pendant of Lemma \ref{ltwist} for traces of Frobenius. \begin{lemma}\label{ltwisttr} With notation and hypotheses as in Lemma \ref{ltwist}, we have \[\frac{1}{m}\sum_{\sigma\in G}\sharp(M_c(U^{(\sigma)})) = \sharp(M_c(V)). \] \end{lemma} \begin{proof} Keep the notation in the proof of Lemma \ref{ltwist}. For $\sigma\in G$, let $\epsilon_\sigma= \frac{1}{m}\sum_{h\in H_\sigma} h\in \Q[\Gamma\times G]$: then, by Lemma \ref{l1.4}, \[M_c(U^{(\sigma)}) \simeq \epsilon_\sigma M_c(U_E).\] We have $M_c(U_E)= M_c(\Spec E) \otimes M_c(U)=M(\Spec E) \otimes M_c(U)$, hence $F_{M_c(U_E)}=F_{M(\Spec E)}\otimes F_{M_c(U)}$. The endomorphism $F_{M(\Spec E)}$ coincides with the Frobenius automorphism $\phi_E\in Gal(E/k)$ acting on $M(\Spec E)$. Hence \begin{multline*} \sum_{\sigma\in G}\tr(F_{M_c(U^{(\sigma)})}) =\tr\left(\sum_{\sigma\in G}\phi_E\otimes \epsilon_\sigma F_{M_c(U)}\right)\\= \tr\left(\sum_{\sigma\in G} \sum_{r=0}^{m-1} \frac{1}{m}\phi_E\otimes (\phi_E^{-r},\sigma^r) F_{M_c(U)}\right)\\ = \frac{1}{m}\tr\left(\sum_{\sigma\in G} \sum_{r=0}^{m-1} \phi_E^{1-r}\otimes\sigma^r F_{M_c(U)}\right)= \frac{1}{m}\left(\sum_{\sigma\in G} \sum_{r=0}^{m-1})\tr(\phi_E^{1-r})\tr(\sigma^r F_{M_c(U)})\right). \end{multline*} But $\tr(\phi_E^{1-r})=0$ for $1-r\ne m$ and $\tr(\phi_E^{m})=m$ (see Lemma \ref{l2.2}). Hence the last sum collapses to \[\frac{1}{m}\left(\sum_{\sigma\in G} m\tr(\sigma^{1-m} F_{M_c(U)})\right)=m\tr\left(\frac{1}{m}\sum_{\sigma\in G} \sigma F_{M_c(U)}\right)=m\tr(F_{M_c(V)})\] as requested. \end{proof} \begin{qn} Does the equality $\sum_{\sigma\in G}[U^{(\sigma)}] = m[V]$ hold in the Grothendieck group of varieties? It would yield lemmas \ref{ltwist} and \ref{ltwisttr} in one gulp. \end{qn} \subsection{More general schemes} Theorem \ref{t2.2} d) extends to \begin{thm}\label{p1} If $X$ is a separated $k$-scheme of finite type, then \[\sharp_n(M_c(X))=|X(\F_{q^n})| \text{ for all } n\ge 1.\] \end{thm} We first give a lemma: \begin{lemma} \label{l1.2} Consider an open-closed situation \begin{equation} Z\by{i} X\yb{j} U \end{equation} where $Z$ is a closed subset of $X$ with open complement $U$. Then, if Theorem \ref{p1} is true for two among $X,Z,U$, it is true for the third. \end{lemma} \begin{proof} This follows from Theorem \ref{p1.1} and Theorem \ref{t2.2} a). \end{proof} Before starting the proof, let us explain why it is going to be complicated. Suppose that we know resolution of singularities over $k$. Then we can easily use Lemma \ref{l1.2} to reduce to $X$ smooth projective. In its absence, we want of course to use de Jong's theorem on alterations. But if we try to do a proof as for Proposition \ref{p1.2}, we run into the following problem: if $U\to V$ is an étale covering of smooth varieties and Theorem \ref{p1} is true for $U$, why should it be true for $V$? The fact that $M_c(V)$ is isomorphic to a direct summand of $M_c(U)$ does not help here because, unlike abelian groups, rational integers do not have direct summands\dots\ (In summary: for numbers, the devil is in the idempotents.) Fortunately, the twisting lemmas proven in the previous subsection will help us get around this issue. \begin{proof}[Proof of Theorem \ref{p1}] We argue by induction on $\dim X$. The $0$-dim\-en\-sio\-nal case follows from Theorem \ref{t2.2} d) (or is trivial). Suppose $\dim X>0$. By using Lemma \ref{l1.2}, we first reduce to $X$ a variety and then (by Nagata's theorem) to $X$ proper. By de Jong's equivariant alteration theorem \cite[Th. 7.3]{dJ}, there exists then a quasi-Galois alteration $f:\tilde X \to X$, with $\tilde X $ smooth projective. Let $V\subseteq X$ be a smooth open subset over which $f$ is finite, and let $U=f^{-1}(V_0)$. For simplicity, write $k_n$ for $\F_{q^n}$. We have $\sharp_n(M_c(U))=|U(k_n)|$ by Theorem \ref{t2.2} d), Lemma \ref{l1.2} and induction. Let $G$ be the Galois group of $f$ and $X_G$ be the geometric quotient of $\tilde X$ by $G$. Write $f':\tilde X\to X_G$ for the corresponding factorisation of $f$. If we twist with respect to $f'$ in the style of \S \ref{s2.F}, then $\tilde X^{(\sigma)}$ is smooth projective for all $\sigma\in G$ as a consequence of Lemma \ref{l2.5}, and we also have $\sharp_n(M_c(U^{(\sigma)}))=|U^{(\sigma)}(k_n)|$. Putting Lemmas \ref{ltwist} and \ref{ltwisttr} together then yields $\sharp(M_c(U_G))\allowbreak=|U_G(k)|$. Since $U_G\to V$ is finite and radicial, Lemma \ref{l1.4} shows that the same holds for $V$. Finally we get $\sharp(M_c(X))=|X(k)|$ by induction and Lemma \ref{l1.2} again. \end{proof} \begin{rk} It would be more reasonable to give a direct proof of Theorem \ref{p1}, in the style of Lemma \ref{l2.4}. Unfortunately I haven't been able to find such a proof. \end{rk} \subsection{The zeta function} Recall the important formula \cite[I, (3.2.3.6)]{saa} \[F_{M^*}={}^t F_M^{-1}.\] In view of the above computations, it would be most natural to define the zeta function of $M$ as $Z(F_M,t)$, so that $Z(X,t)=Z(M_c(X),t)$ by Proposition \ref{p1} b). However this would create awkward functoriality problems in the next section, when we deal with motives over a base: see Theorem \ref{t3.2} below. One may think of these problems as arising because $S\mapsto \DM_\gm(S,\Q)$ is a \emph{homology} theory, whereas the functorial behaviour of zeta and $L$-functions expresses itself naturally in \emph{cohomological} terms. Our solution to this issue is to give a slightly artificial definition of the zeta function: \begin{defn}\label{d2} $Z(M,t)=Z(F_M^{-1},t)=Z(F_{M^*},t)$. \end{defn} \begin{thm}\label{t2} $M\mapsto Z(M,t)$ is multiplicative on exact triangles, hence defines a homomorphism $K_0(\DM_\gm(k,\Q))\to 1+t\Q[[t]]$. For any $M\in \DM_\gm(k,\Q)$,\\ a) $Z(M,t)\in \Q(t)$. The degree of this rational function is $-\chi(M)$, where $\chi(M)=\tr(1_M)$.\\ b) We have the functional equation \[Z(M^*,t^{-1}) = (-t)^{\chi(M)}\det(F_M)^{-1}Z(M,t)\] where $\det(F_M)$ is the value at $t=\infty$ of $(-t)^{\chi(M)} Z(M,t)^{-1}$. \\ c) We have the identities \[Z(M[1],t) = Z(M,t)^{-1},\qquad Z(M(1),t) = Z(M,q^{-1}t).\] d) For any $f:X\to \Spec k$ of finite type, we have \[Z(M^{BM}(X),t) = Z(X,t).\] \end{thm} \begin{proof} The first fact follows from Theorem \ref{t2.2}. Using now the surjectivity of \eqref{eq2.1}, we reduce to the case where $M\in \Chow(k,\Q)$. Then we can compute in $\sM_\num(k,\Q)$ and everything follows from Theorem \ref{t2.3} (see remarks before Theorem \ref{t2.4}). d) follows from Theorem \ref{p1}. \end{proof} \begin{rk} It is likely that Theorem \ref{t2} extends to all zeta functions of endomorphisms as in Definition \ref{d2} a), but this seems to require extending Bondarko's theorem to $K_0$ of the categories of endomorphisms as in \cite[\S 15]{modq}. (He has done that!) \end{rk} \begin{prop} Let $f:\Spec E\to \Spec k$ be a finite extension, and let $f_*:\DM_\gm(E)\to \DM_\gm(k)$ be the corresponding direct image functor. Then, for any $M\in \DM_\gm(E)$, we have \[Z(f_* M,t) = Z(M,t^m)\] where $m=[E:k]$. \end{prop} \begin{proof} This is clear from Lemma \ref{l2.2a} b) and the definition of $Z$. \end{proof} \begin{defn}\label{d2.1} We denote by $\Dir$ the group of formal Dirichlet series beginning with $1$. \end{defn} \begin{cor}\label{c1} For $M\in \DM_\gm(k,\Q)$, let \[\zeta(M,s) = Z(M, q^{-s}).\] Then $M\mapsto \zeta(M,s)$ is multiplicative on exact triangles, hence defines a homomorphism $K_0(\DM_\gm(k,\Q))\to \Dir$. Moreover,\\ a) $\zeta(M,s)$ is a rational function in $q^{-s}$, of degree $-\chi(M)$.\\ b) We have the functional equation \[\zeta(M^*,-s) = (-q^{-s})^{\chi(M)}\det(F_M)\zeta(M,s).\] c) We have the identities \[\zeta(M[1],s) = \zeta(M,s)^{-1},\qquad \zeta(M(1),s) = \zeta(M,s+1).\] d) For any $f:X\to \Spec k$ of finite type, we have \[\zeta(M^{BM}(X),s) = \zeta(X,s).\] e) For a finite extension $f:\Spec E\to \Spec k$, we have $\zeta(M,s)=\zeta(f_* M,s)$.\qed \end{cor} We shall need the following \begin{prop}\label{p3} Let $M^*\in d_{\le n}\DM_\gm^\eff(k,\Q)$ (see Definition \ref{d1.1}). Then the zeroes and poles of $Z(M,t)$ are effective Weil $q$-numbers of weights $\in [0,2n]$. In particular, the Dirichlet series $\zeta(M,s)$ converges absolutely for $\Re(s)>n$. \end{prop} \begin{proof} Consider the diagram \[\begin{CD} K_0(\Chow^\eff(k,\Q))@>\phi>\sim> K_0(\DM_\gm^\eff(k,\Q))\\ @V{\psi}VV\\ K_0(\sM_\num^\eff(k,\Q)). \end{CD}\] The dimension filtration induces a filtration on the three $K_0$'s, and $\phi$ and $\psi$ respect these filtrations. So does the inverse of $\phi$, because it is induced by Bondarko's functor \[\DM_\gm^\eff(k,\Q)\to K^b(\Chow^\eff(k,\Q))\] which obviously respects the dimension filtrations. Since $M\mapsto Z(M,t)$ factors through $K_0(\sM_\num^\eff(k,\Q))$, we are therefore reduced to Theorem \ref{t2.4}. \end{proof} \begin{cor} \label{c2.1} if $\dim X\le n$, the zeroes and poles of $Z(X,t)$ are effective Weil $q$-numbers of weights $\in [0,2n]$.\qed \end{cor} Of course, this corollary may also be deduced from \cite{WeilII}. \section{Motives over a scheme of finite type} \subsection{Motives over a base} In the sequel, we shall need a theory of triangulated motives over general base schemes, with a formalism of six (even seven) operations as in \cite{ayoub}. Unfortunately, it is unknown whether the natural generalisation of Voevodsky's construction enjoys such a formalism \cite[6.2]{ffihes}. This can be corrected by using the subcategory of constructible objects in Ayoub's étale motives without transfers $\DA^\et(-,\Q)$ \cite{ayoubetale} or Cisinski-Déglise Beilinson motives $\DM_\Be(-)$ \cite{cis-deg}, which coincide anyway \cite[6.3 and 6.4]{ffihes}: the important point here is that these subcategories are preserved under the six operations (\cite[Scholie 2.2.34 and Th. 2.2.37]{ayoub} for $f^*,f_*,f_!,f^!$, \cite[Prop. 2.3.62]{ayoub} for $\uHom$). We adopt here the viewpoint of \emph{loc. cit.}, 6.5, using only the \emph{existence} of a formalism of six operations for some categories $\D(S)$ which agree with $\DM_\gm(k,\Q)$ when $S=\Spec k$, and are provided with an $l$-adic realisation functor which commutes with the six operations. Very occasionally, we shall use non-formal properties of such a theory. We write $\Z_S=\Z$ for the unit object of $\D(S)$. If $f:X\to S$ is a smooth $S$-scheme (separated and of finite type), we write $M_S(X)=f_\# \Z_X\in\D(S)$ for its \emph{motive}, where $f_\#$ is the left adjoint of $f^*:\D(S)\to \D(X)$ \cite[1.4.1, axiom 3]{ayoub}. \subsection{The Borel-Moore motive revisited} This subsection partly answers \cite[Rem. 6.7.3 3)]{ffihes}. \begin{thm}\label{t3.2} Let $k=\F_q$ and let $f:X\to \Spec k$ be a separated $k$-scheme of finite type. Then we have an isomorphism \[M^{BM}(X)\simeq f_!\Z_X\] at least if $X$ is embeddable in a smooth scheme (e.g. if $X$ is quasi-projective). If $f$ is smooth, this isomorphism is canonical and natural for open immersions. \end{thm} \begin{proof} Suppose first that $f$ is smooth of (pure) dimension $d$. Applying \cite[Vol. I, Scholie 1.4.2 3]{ayoub}, we find \begin{multline}\label{eq3.4} f_!\Z_X\simeq f_\# Th^{-1}(\Omega_f) \Z_X\\ =f_\# \Z_X(-d)[-2d]=M(X)(-d)[-2d]\simeq M^{BM}(X). \end{multline} Here $Th(\Omega_f)$ is the Thom equivalence associated to the module of differentials $\Omega_f$, which is computed to be the said Tate twist in \cite[2.4.38 and 2.4.40]{cis-deg}, plus the description of $M_c(X)$ for $X$ smooth given at the beginning of Section \ref{s1}. This isomorphism clearly commutes with open immersions. Suppose now that $X$ can be embedded as a closed subscheme of a smooth scheme $Y$. For simplicity, write $\tilde M^{BM}(X)=f_!\Z_X$. We then get an isomorphism $\tilde M^{BM}(X)\iso M^{BM}(X)$ by completing the isomorphisms in the diagram of exact triangles \[\begin{CD} \tilde M^{BM}(U)@>>> \tilde M^{BM}(Y)@>>> \tilde M^{BM}(X)@>+1>>\\ @V\wr VV @V\wr VV\\ M^{BM}(U)@>>> M^{BM}(Y)@>>> M^{BM}(X)@>+1>> \end{CD}\] where $U=Y-X$, the top row is the dual of \eqref{eq1.1} and the bottom row is obtained by applying $g_!$ to the localisation exact triangle \cite[Vol. 1, p. 77]{ayoub} \begin{equation}\label{eq3.1} j_!j^!\Z_Y\to \Z_Y\to i_*i^*\Z_Y\by{+1} \end{equation} where $g:Y\to \Spec k$ is the structural morphism and $j:U\to Y$, $i:X\to Y$ are the open and closed immersion. (Note that $j^!=j^*$ and $i_*=i_!$.) \end{proof} \begin{rk}\label{r1.1} It would be more reasonable and more efficient to define \emph{a priori} a natural morphism \[M_c(X)= C_*(\Z_\tr^c(X))\to (f_!\Z_X)^*\] in $\DM_\gm(k)$, where $\Z_\tr^c(X)(U)=z(U,X)$ is the group of quasi-finite correspondences, and to show that it is an isomorphism by reduction to the smooth (or smooth projective) case. By duality \cite[Th. 2.3.75]{ayoub}, the right hand side can be written \[(f_!\Z_X)^*=D_k(f_!\Z_X)\simeq f_*D_X(\Z_X)=f_*f^!\Z.\] This amounts to defining a map \[f^*C_*(\Z_\tr^c(X))\to f^!\Z\] and I don't know how to construct it\dots \end{rk} \subsection{Zeta functions} Let $f:S\to \Spec \Z$ be a scheme of finite type over $\Z$. For each $x\in S_{(0)}$, we have a pull-back functor \[i_x^*:\D(S)\to \D(\kappa(x))\simeq \DM_\gm(\kappa(x),\Q).\] \begin{prop}\label{p3.1} For $M\in \D(S)$, the product \[\zeta(M,s) = \prod_{x\in S_{(0)}} \zeta(i_x^* M,s)\] is convergent in the group $\Dir$ (Definition \ref{d2.1}) for the topology given by the order of the first nonzero term. It is multiplicative on exact triangles and we have the identities \[\zeta(M[1],s) = \zeta(M,s)^{-1},\qquad \zeta(M(1),s) = \zeta(M,s+1).\] \end{prop} \begin{proof} Since the exponential is continuous on $\Dir$ for the said topology, it suffices to show that the sum \[\sum_{x\in S_{(0)}} \sum_{n\ge 1} \sharp_n(i_x^*M) \frac{N(x)^{-ns}}{n}\] is convergent. Here, $N(x)$ is the cardinal of the residue field $\kappa(x)$. Rearranging, we must show that, for any prime $p$ and any $r\ge 1$, the set \[\{x\in S_{(0)}\mid n[\kappa(x):\F_p]=r\}\] is finite. This is clear, since $S$ has only a finite number of closed points of a given degree over $\F_p$. The identities follow from the case of finite fields (Corollary \ref{c1}). \end{proof} \subsection{The case of characteristic $p$} In this subsection, we assume that $S$ is an $\F_q$-scheme. For simplicity we still write $f$ for the structural morphism $X\to \Spec \F_q$. \begin{prop}\label{p3.3} For $n\ge 0$, let $d^!_{\le n}\D(S)$ be the thick triangulated subcategory of $\D(S)$ generated by the $g_!\Z_X$, where $g:X\to S$ runs through the morphisms of relative dimension $\le n$. Then $(f_!M)^*\in d_{\le n+d}\DM^\eff(S,\Q)$ for any $M\in d^!_{\le n}\D(S)$, where $d=\dim S$. Moreover, for any $M\in \D(S)$ there exists $n\ge 0$ and $r\in \Z$ such that $M(r)\in d^!_{\le n}\D(S)$. \end{prop} \begin{proof} It suffices to show that $(f_!g_!\Z_X)^*\in d_{\le n+d}\DM^\eff(S,\Q)$ when $\dim g\le n$. By Theorem \ref{t3.2}, this is $M_c(X)$; the claim then follows from Proposition \ref{p1.2}. It suffices to prove the last statement for generators $M_S(X)(s)$, with $g:X\to S$ smooth. But $g_!\Z_X\simeq M_S(X)(-n)[-2n]$ if $n=\dim g$ as in \eqref{eq3.4}, so $M_S(X)(s)(-n-s)\in d^!_{\le n}\D(S)$. \end{proof} For any $x\in S_{(0)}$, we have a specialisation functor \[\Sp_x=(f_x)_*i_x^*:\D(S)\to \DM_\gm(\F_q,\Q)\] where $f_x:\Spec \kappa(x)\to \Spec \F_q$ is the structural morphism. We have \begin{equation}\label{eq3.3} \zeta(i_x^*M,s)= \zeta(\Sp_x M,s) \end{equation} by Corollary \ref{c1}. From Lemma \ref{l2.2}, we get: \begin{lemma}\label{l3.1} For any $M\in \D(S)$ and any $n\ge 1$, the function \[S_{(0)}\ni x\mapsto \sharp_n(\Sp_x M) \] is $0$ outside the finite set $\bigcup_{m\mid n} S(\F_{q^m})$. \end{lemma} This gives a meaning to the following proposition. For simplicity, we write \begin{equation}\label{eq3.2} \sharp_n^*(M)=\sharp_n(M^*) = \sharp_{-n}(M) \end{equation} for $M\in \DM_\gm(\F_q)$. \begin{prop}[trace formula]\label{p3.2} We have \[\sharp^*_n(f_!M) =\sum_{x\in S_{(0)}} \sharp^*_n(\Sp_x M)\] for any $M\in \D(S)$ and any $n\ge 1$. \end{prop} \begin{proof}We give two proofs, one in a special case and one in general: 1) The case where $M=g_!\Z_X$ for $g:X\to S$ a morphism of finite type, where $X$ is embeddable in a smooth $k$-scheme. Then \[\sharp_n^*(f_!M) = \sharp_n^*((fg)_!\Z_X) = \sharp_n^*(M^{BM}(X)) =\sharp_n(M_c(X)) = |X(\F_{p^n})|\] by Theorems \ref{t3.2} and \ref{p1}. On the other hand, let $X_x=g^{-1}(x)$. The base change theorem \cite[scholie 1.4.2 1)]{ayoub} applied to the Cartesian square \begin{equation}\label{eq3.5} \begin{CD} X_x@>I_x>> X\\ @Vg_x VV @Vg VV\\ x@>i_x>> S \end{CD} \end{equation} yields\footnote{This is part of the definition of a crossed functor, for which the reader should consult \cite[Déf. 1.21.12]{ayoub}.} \[i_x^*g_!\Z_X\simeq (g_x)_! I_x^*\Z_X =(g_x)_! \Z_{X_x}\] and we get \begin{multline*} \sum_{x\in S_{(0)}} \sharp_n^*(\Sp_x M)=\sum_{x\in S_{(0)}} \sharp_n^*((f_x)_*i_x^* g_! \Z_X)\\ =\sum_{x\in S_{(0)}} \sharp_n^*((f_x)_!i_x^* g_! \Z_X) =\sum_{x\in S_{(0)}} \sharp_n^*((f_xg_x)_! \Z_{X_x}) \end{multline*} where, as above \[\sharp_n^*((f_xg_x)_! \Z_{X_x}) = |X_x(\F_{p^n})|.\] Now it is clear that $|X(\F_{p^n})|=\sum_{x\in S_{(0)}} |X_x(\F_{p^n})|$. From 1), one can derive Proposition \ref{p3.2} for those $M$'s which belong to the smallest triangulated subcategory containing the said $g_!\Z_X$ and stable under Tate twists. By Proposition \ref{p3.3}, its pseudo-abelian envelope equals $\D(S)$; but then we run into the problem of idempotents mentioned before the proof of Theorem \ref{p1}. This leads us to the second proof: 2) Apply the $l$-adic realisation functor $R_l$. \begin{multline*} \sum_{x\in S_{(0)}} \sharp_n^*(\Sp_x M) = \sum_{x\in S_{(0)}} \sharp_n^*(R_l(\Sp_x M)) = \sum_{x\in S_{(0)}} \sharp_n^*(\Sp_x R_l(M)) \\ = \sharp_n^*(Rf_!R_l(M)) =\sharp_n^*(R_l(f_!M)) =\sharp_n^*(f_!M) \end{multline*} where we used the commutation of $R_l$ with $f_!$, $i_x^*$ and duality, plus \cite[equation (2) p. 470]{SGA5}. \end{proof} \begin{cor}\label{c3.1} If $S(\F_{q^n})=\emptyset$, then $\sharp_n^*(f_!M)=0$ for any $M\in \D(S)$. \end{cor} \begin{proof} This follows from Lemma \ref{l3.1} and Proposition \ref{p3.2}. \end{proof} \begin{qns} 1) Conversely, Corollary \ref{c3.1} implies Proposition \ref{p3.2} by the localisation exact triangle \eqref{eq3.1}, even for $S=\P^1-\P^1(\F_{q^n})$ by the dévissages of \cite[XV.3.2]{SGA5}. Can one find a proof which avoids the $l$-adic realisation and the trace formula of \cite{SGA5}?\\ 2) The statement breaks down for $n=0$, because the right hand side is generally infinite (\emph{e.g.} for $M=\Z_S$). I don't know a formula for $\chi(f_! M)$, which appears in the functional equation just below. Can one use some renormalisation trick? \end{qns} \begin{thm} \label{t3.1} a) One has $\zeta(M,s)=\zeta(f_!M,s)$. In particular, there exists $Z(M,t)\in \Q(t)$ such that $\zeta(M,s)=Z(M,q^{-s})$; the zeroes and poles of $Z(M,T)$ are Weil $q$-numbers.\\ b) If $M\in d^!_{\le n} \D(S)$ (see Proposition \ref{p3.3}), these Weil $q$-numbers are effective of weights $\le 2(n+d)$, where $d=\dim S$.\\ c) If $S$ is projective, one has the functional equation \[\zeta(D_S(M),-s)=(-q^{-s})^{\chi(f_!M)}\det(F_{f_!M})^{-1}\zeta(M,s)\] where $D_S(M)=\uHom(M,f^!\Z)$. If $S$ is moreover smooth of dimension $d$, one has the functional equation \[\zeta(M^*,d-s)=(-q^{-s})^{\chi(f_!M)}\det(F_{f_!M})^{-1}\zeta(M,s)\] with $M^*:=\uHom(M,\Z)$. \end{thm} \begin{proof} a) follows from Proposition \ref{p3.2} and \eqref{eq3.3}. b) follows from Propositions \ref{p3.3} and \ref{p3}. For c), we have the isomorphism $f_!D_S(M)\simeq (f_*M)^*\simeq (f_!M)^*$ by \cite[Th. 2.3.75 and Scholie 1.4.2 4]{ayoub}, hence \begin{multline*} \zeta(D_S(M),-s)= \zeta((f_!M)^*,-s) = (-q^{-s})^{\chi(f_!M)}\det(F_{f_!M})\zeta(f_!M,s)\\ = (-q^{-s})^{\chi(f_!M)}\det(F_{f_!M})\zeta(M,s) \end{multline*} The smooth case follows, since $f^!\simeq f^*(d)[2d]$ then (the adjoint identity to the one of \eqref{eq3.4}). \end{proof} \begin{rks} 1) Let $\D^\eff(S)$ be as in the proof of Proposition \ref{p3.1}. If $M\in d_{\le n} \D^\eff(S)$, Theorem \ref{t3.1} a) may be refined: the zeroes and poles of $\zeta(M,s)$ are effective Weil numbers of weights $\le 2n$.\\ 2) Theorem \ref{t3.1} b) extends to $S$ proper by \cite{cis-deg}.\\ 3) I don't know a formula for $\zeta(D_S(M),-s)$ when $S$ is not proper. \end{rks} \subsection{The general case} We come back to the situation where $f:S\to \Spec \Z$ is an arbitrary $\Z$-scheme of finite type. We write $d$ for its relative dimension, \emph{i.e} the maximal dimension of its closed fibres. \begin{thm}\label{t3.3} Let $f:S\to T$ be a morphism of $\Z$-schemes of finite type. Then $\zeta(M,s) = \zeta(f_!M,s)$ for any $M\in \D(S)$ (as formal Dirichlet series). \end{thm} In Corollary \ref{c6.1} below, this Dirichlet series will be shown to be convergent. \begin{proof} We immediately reduce to the case $S=\Spec \Z$. For a prime number $p$, let $S_p$ be the fibre of $f$ at $p$; we have a Cartesian square similar to \eqref{eq3.5}: \begin{equation}\label{eq3.6} \begin{CD} S_p@>I_p>> S\\ @Vf_p VV @Vf VV\\ \Spec \F_p@>i_p>> \Spec \Z. \end{CD} \end{equation} The equality \[\zeta(M,s) = \prod_p \zeta(I_p^*M,s)\] follows from the definition. By Theorem \ref{t3.1} a) and proper base change, we have \[\zeta(I_p^*M,s)=\zeta((f_p)_!I_p^*M,s)=\zeta(i_p^*f_!M,s)\] hence \[\zeta(M,s) = \prod_p \zeta(i_p^*f_!M,s)=\zeta(f_!M,s)\] again by definition. \end{proof} \section{Motives over $\R$ and $\C$} \subsection{Hodge structures} Set \[\Gamma_\R(s) = \pi^{-s/2}\Gamma(s/2),\qquad \Gamma_\C(s) = 2(2\pi)^{-s}\Gamma(s)\] where $\Gamma(s)$ is Euler's Gamma function. (For convenience, we take for $\Gamma_\C(s)$ twice the function in Serre \cite[\S 3]{serre}.) If $V$ is a (pure) complex Hodge structure (i.e., a finite-dimensional $\C$-vector space provided with a decomposition $V=\bigoplus_{(p,q)\in \Z\times\Z} V^{p,q}$), one defines \[\Gamma(V,s) = \prod_{(p,q)} \Gamma_\C(s-\inf(p,q))^{h(p,q)}\] with $h(p,q)=\dim V^{p,q}$. If $V$ is a (pure) real Hodge structure (i.e. a complex Hodge structure plus an involution $\sigma$ such that $\sigma V^{p,q} = V^{q,p}$), one defines \[\Gamma(V,s) = \prod_n \Gamma_\R(s-n)^{h(n,+)}\Gamma_\R(s-n+1)^{h(n,-)}\prod_{p<q} \Gamma_\C(s-p)^{h(p,q)}\] where $h(p,q)$ is as above and $h(n,\epsilon) = \dim(V^{n,n}\mid \sigma = (-1)^n\epsilon)$. \subsection{Pure motives} If $k=\R$ or $\C$, we have a realisation functor $H:\sM_\hom(k,\Q)\to \Hodge_k$, where $\Hodge_k$ is the category of pure $k$-Hodge structures. For $M\in \sM_\hom(k)$, we define \[\Gamma(M,s) = \Gamma(H(M)^*,s).\] \subsection{Triangulated motives} Using Bondarko's isomorphism \[K_0(\Chow(k,\Q))\iso K_0(\DM_\gm(k,\Q))\] we may extend the above definition to all objects of $\DM_\gm(k,\Q)$ (alternately, we could go through the Hodge realisation on $\DM_\gm(k,\Q)$). The identities of Corollary \ref{c1} c) hold for $\Gamma(M,s)$. \begin{rk} One should compare this definition with the much more sophisticated one for mixed motives in \cite[III.1]{fpr}, using \emph{mixed} Hodge structures. \end{rk} \section{Two elementary lemmas on Dirichlet series} \begin{lemma}\label{l1} Let $(R_p)$ be a sequence of rational functions with complex coefficients, indexed by the prime numbers. We assume that $R_p(0)=1$ for all $p$ and: \begin{thlist} \item There is an integer $w$ such that, for almost all $p$, the inverse zeroes and poles of $R_p$ have absolute value $\le p^{w/2}$.\\ \item The heights of the $R_p$ are bounded independently of $p$ (N.B.: here, the \emph{height} of a rational function $R=P/Q$ is $\deg(P)+\deg(Q)$ if $P$ and $Q$ are coprime polynomials). \end{thlist} Let $L(s)=\prod_p R_p(p^{-s})$. Then $L(s)$ is a Dirichlet series with absolute convergence abscissa $\le w/2+1$. \end{lemma} \begin{proof} Let $\lambda$ be an inverse pole of $R_p$. Then, for $s=\sigma + it\in \C$: \[\left|(1-\lambda p^{-s})^{-1}\right|=\left|\sum_{n= 0}^\infty \lambda^n p^{-ns}\right|\le \sum_{n= 0}^\infty p^{-n(\sigma-w/2)}\] converging as soon as $\sigma > w/2$. If now $\lambda$ is an inverse zero of $R_p$, we have \[\left|1-\lambda p^{-s}\right|\le 1+ |p^{w/2-s}|\le \sum_{n= 0}^\infty p^{-n(\sigma-w/2)}.\] Thus, if the height of $R_p$ is $\le H$, we have \[\left|R_p(p^{-s})\right|\le \left(\sum_{n= 0}^\infty p^{-n(\sigma-w/2)}\right)^H=\left(1-p^{-(\sigma-w/2)}\right)^{-H}.\] Collecting, we find \[\left|L(s)\right|\le \left(\prod_p\left(1-p^{-(\sigma-w/2)}\right)^{-1}\right)^H=\zeta(\sigma-w/2)^H\] which converges for $\sigma-w/2>1$, as is well-known. \end{proof} \begin{lemma}\label{l2} a) Let $f=\sum_{n= 1}^\infty a_n n^{-s}$ be a convergent Dirichlet series with complex coefficients, with $a_1=1$. Then the equation $f(s)= g(s)/g(s+1)$ has a unique solution as a convergent Dirichlet series, namely \[g(s) = \prod_{m= 0}^\infty f(s+m).\] Moreover, $g$ has the same absolute convergence abscissa as $f$.\\ b) If the coefficients of $f$ belong to a subring $R$ of $\C$, so do those of $g$. \end{lemma} \begin{proof} a) \emph{Uniqueness:} if $g_1,g_2$ are two solutions, then $h=g_1/g_2$ verifies \[h(s)=h(s+1).\] If $h(s)=\sum c_n n^{-s}$, this gives the identity \[u(s)=\sum(c_n-c_n/n) n^{-s} = 0.\] Since $g_1,g_2$ are convergent, so are $h$ and $u$, and it is well-known that this implies $c_n - c_n/n=0$ for all $n$, hence $c_n=0$ for all $n>1$. \emph{Existence:} Let us check first that $g(s)$ converges as a formal Dirichlet series. Indeed: \[g(s) = \prod_{m= 0}^\infty \left(\sum_{n=1}^\infty \frac{a_n}{n^m} n^{-s}\right).\] In this product, the coefficient $b_n$ of $n^{-s}$ is \[b_n = \sum_{r_1\dots r_k=n} \sum_{m_1\ge 0,\dots,m_k\ge 0} \frac{a_{r_1}}{r_1^{m_1}}\dots \frac{a_{r_k}}{r_k^{m_k}}=\sum_{\substack{r_1\dots r_k=n\\r_1,\dots,r_k>1}} \frac{a_{r_1}}{1-r_1^{-1}}\dots \frac{a_{r_k}}{1-r_k^{-1}} .\] Suppose that $f(s)$ converges absolutely for $\Re(s)>c$. Then $|a_n|=o(n^{c+\epsilon})$ for all $\epsilon>0$. Therefore \begin{multline*} |b_n]\le \left|\sum_{\substack{r_1\dots r_k=n\\r_1,\dots,r_k>1}} \frac{a_{r_1}}{1-r_1^{-1}}\dots \frac{a_{r_k}}{1-r_k^{-1}}\right|\\ = o(n^{c+\epsilon}) \sum_{\substack{r_1\dots r_k=n\\r_1,\dots,r_k>1}} \frac{1}{1-r_1^{-1}}\dots \frac{1}{1-r_k^{-1}}=o(n^{c+\epsilon})g^0_n \end{multline*} where $g^0_n$ is the $n$-th coefficient of \[g^0(s) =\zeta(s)\zeta(s+1)\dots\] To study the absolute convergence of this product, we look at the log of the corresponding Eulerian product, for $s\in\R$: \begin{multline*} \log g^0(s)=\sum_p\sum_{m=0}^\infty -\log(1-p^{-s-m})=\sum_p\sum_{m=0}^\infty\sum_{k=1}^\infty \frac{p^{-k(s+m)}}{k}\\ =\sum_p \sum_{k=1}^\infty \frac{p^{-ks}}{k}\frac{1}{1-p^{-k}}\le 2\sum_p \sum_{k=1}^\infty \frac{p^{-ks}}{k} \\ = 2\sum_p -\log(1-p^{-s}) = \log(\zeta(s)^2). \end{multline*} The $n$-th coefficient of $\zeta(s)^2$ is the number of divisors of $n$, which is $o(n^{\epsilon})$ for all $\epsilon>0$. Hence $|b_n|=o(n^{c+\epsilon})$ for all $\epsilon >0$. It follows that $g(s)$ converges absolutely for $\Re(s)>c$. But conversely, if $g(s)$ converges absolutely for $\Re(s)>c$, so does $f(s)=g(s)/g(s+1)$. b) is obvious from the formula giving $g(s)$. \end{proof} \section{Motives over a ring of integers} Let $K$ be a number field, with ring of integers $O_K$. Consider the category $\D(O_K)$. For each place $v$ of $K$, we have a homomorphism $\phi_v:O_K\to \kappa(v)$, where $\kappa(v)$ is \begin{itemize} \item The residue field at $v$ if $v$ is finite; \item The completion of $K$ at $v$ if $v$ is archimediean. \end{itemize} In each case, we have a pull-back functor \[\phi_v^*:\D(O_K)\to \D(\kappa(v)).\] We shall also use the fact that $\Z_{\Spec O_K}$ is a dualising object, of $\D(O_K)$ which follows from \cite[Th. 2.3.73]{ayoub} and \cite[Cor. 5.15]{dJ2}. We write $M\mapsto M^*$ for the corresponding duality functor. \begin{thm} \label{t6.2} Let $M\in \D(O_K)$. Then there exists a nonempty open subset $U\subseteq \Spec O_K$ such that the cohomology sheaves $H_l^i(M)$ are locally constant constructible for any prime number $l$ invertible on $U$. \end{thm} \begin{proof} We are in a case where $\D(O_K)\simeq \DM_\gm(O_K,\Q)$, so we can reason as in the proof of Lemma \ref{l2.2a}; this reduces us to the case where $M=M(X)$ with $f:X\to \Spec O_K$ smooth. Then the result is a special case of results of Illusie \cite[Th. 2.1]{illusie}. (Recall the proof: by Hironaka's resolution of singularities and some spread-out, there exists $U$ and an open immersion $j:X_U\inj \bar X$, where $\bar f:\bar X\to U$ is smooth projective and the closed complement $\bar X-X_U$ is the support of a divisor $D$ with strict normal crossings, relative to $U$. By \cite[Arcata V.3.1]{SGA412}, the sheaves $R^i\bar f_*\Q_l$ are locally constant, and so are the corresponding sheaves for all intersections of the components of $D$. The Leray spectral sequence for $j$ and cohomological purity then show that the same holds for the $R^i(f_{|U})_*\Q_l$, \emph{cf.} \cite[Lemme 3.1]{illusie}.) \end{proof} \begin{cor}\label{l3} For any $M\in \D(O_K[1/l])$, the function \[\fp\mapsto b_\fp(M)= \sum_{i\in\Z}\dim H^i_l(M_\fp)\] from primes of $O_K[1/l]$ to $\N$ is bounded. \qed \end{cor} \begin{thm}\label{t6.1} The Dirichlet series $\zeta(M,s)$ has a finite convergence abscissa. More precisely, if $M^*\in d_{\le n} \D(O_K)$, then $\zeta(M,s)$ converges absolutely for $\Re(s)>n+1$. \end{thm} \begin{proof} Let $M^*\in d_{\le n} \D(O_K)$. By Proposition \ref{p3} and Corollary \ref{l3}, the hypotheses of Lemma \ref{l1} are satisfied for $\zeta(M,s)$, with $w=2n$. This proves the statement for effective motives, hence in general. \end{proof} \begin{cor}\label{c6.1} For any $\Z$-scheme of finite type $S$ and any $M\in \D(S)$, the formal Dirichlet series $\zeta(M,s)$ has a finite convergence abscissa. \end{cor} \begin{proof} This follows from Theorems \ref{t3.3} and \ref{t6.1} a). \end{proof} \begin{rk} Denis-Charles Cisinski pointed out that there are more motivic and uniform methods to obtain bounds as in Corollary \ref{l3} (\cite[Th. 6.3.26]{cis-deg2}, \cite{schol-etal}, \cite[Th. 2.4.2]{cisinski}). But all these theorems rest \emph{in fine} on the smooth and proper base change theorem for étale cohomology \cite[Arcata, V, Th. 3.1]{SGA412}, so they do not seem to bring something essentially new here as the existence of a bound is sufficient for Theorem \ref{t6.1} and its corollary. \end{rk} \begin{prop}\label{p6.1} For $M\in \D(O_K)$, let \[\quad \xi(M,s)= |d_K|^{s\chi(M)/2}\prod_{v\mid\infty}\Gamma(\phi_v^* M,s)\cdot \zeta(M,s) \] where $d_K$ is the absolute discriminant of $K$. Then $\xi(\pi_* M,s)=\xi(M,s)$, where $\pi_*$ is the push-forward functor $\D(O_K)\to \D(\Z)$. \end{prop} \begin{proof} It suffices to check this for the Gamma-discriminant part, and it follows from an elementary computation. \end{proof} To be complete, we define the completed $\zeta$ function of a motive over a $\Z$-scheme of finite type. \begin{defn}\label{d6.1} If $M\in \D(S)$ where $f:S\to \Spec \Z$ is a $\Z$-scheme of finite type, we set $\xi(M,s)=\xi(f_!M,s)$. \end{defn} Note that $\xi(M,s)=\zeta(M,s)$ if $f$ is not dominant, and Theorem \ref{t3.3} still holds when replacing $\zeta$ by $\xi$. By Proposition \ref{p6.1}, this definition does extend the case $S=\Spec O_K$. \section{A theorem of Serre} For $M\in \D(O_K)$ and $\fp\subset O_K$, define \[N_M(\fp) = \sharp(M_\fp)\] the number of points of $M$ modulo $\fp$. \begin{thm}\label{t3} Let $M\in \D(O_K)$. Suppose that $\zeta(M,s)$ is not a finite product of Euler factors. Then the set \[\{\fp\mid N_M(\fp) = 0\}\] has a density $1-\epsilon$, with \[\epsilon \ge \frac{1}{b_\infty(M)^2}\] where $b_\infty(M)=\sum_i \dim H^i_l(M_K)$. \end{thm} \begin{proof} It is the same as Serre's \cite[Th. 6.17]{NX}, which is the special case $M=M^{BM}(X)\oplus M^{BM}(X')[1]$ for $X,X'$ $O_K$-schemes of finite type. For $U$ as in Theorem \ref{t6.2}, one may compute the traces of the geometric Frobenius $F_{M_\fp}$ acting on $H_l^*(M_\fp)$, for $\fp\in U$, as the traces of the inverse of the [conjugacy class of the] arithmetic Frobenius $\phi_\fp\in Gal(\bar K/K)$ acting on $H_l^*(M_K)$. The statement then reduces to the following theorem of Serre \cite[Th. 5.15]{NX}: \begin{thm}\label{tserre} Let $G$ be a compact group, $K$ be a locally compact field of characteristic $0$ and let $\rho:G\to GL_n(K)$, $\rho':G\to GL_{n'}(K)$ be two continous $K$-linear representations of $G$. Then \begin{thlist} \item either $\tr \rho= \tr \rho'$; \item or the set $\{g\in G\mid \tr(\rho)(g)\ne \tr(\rho')(g)\}$ has a Haar density $\ge \frac{1}{(n+n')\sup(n,n')}$. \end{thlist} \end{thm} To apply Serre's theorem, we represent $G=G_K$ on $H^{even}_l(M_K)$, yielding $\rho$, and on $H^{odd}_l(M_K)$, yielding $\rho'$. Here, $K=\Q_l$. If both those vector spaces are $0$, then $\sH_l(M)$ is supported on $\Spec O_K[1/l]-U$, where $U$ is the open set as above; then all Euler factors of $\zeta(M,s)$ at $\fp\in U$ are equal to $1$. Otherwise, we may apply the theorem. In case (i), we get the same conclusion on the Euler factors as above. Case (ii) yields the conclusion of Theorem \ref{t3}. \end{proof} More generally, a large part of Serre's results in \cite{NX} seem to extend to objects of $\D(O_K)$ without difficulty. \section{Some six functors algebra} \subsection{$K_0$ of triangulated categories}\label{s8.A} Let \[0 \to\sT'\by{i} \sT\by{p} \sT''\to 0\] be a short exact sequence of triangulated categories: $i$ is a thick embedding and $\sT/\sT'\iso\sT''$. We assume that we are in the Verdier situation: $i$ has a right adjoint $\pi$, hence $p$ has a right adjoint $j$ and any object $M\in \sT$ fits in a functorial exact triangle \[i\pi M\to M\to pj M\by{+1}.\] From this, it follows: \begin{lemma} In the above situation, the map \[K_0(\sT)\by{\left(\begin{smallmatrix}\pi\\ p \end{smallmatrix}\right)} K_0(\sT')\oplus K_0(\sT'')\] is an isomorphism. Alternately, we have an exact sequence \[0\to K_0(\sT')\by{i} K_0(\sT)\by{p} K_0(\sT'')\to 0\] split by $\pi$ and $j$. \end{lemma} \begin{proof} Since $i$ and $j$ are fully faithful, the first map has the right inverse $(i,j)$. It is also a left inverse thanks to the exact triangle above. \end{proof} Let $(\sT_\alpha)_{\alpha\in A}$ be an filtered inductive system of triangulated categories, and let $\sT=\colim_\alpha \sT_\alpha$. We have an induced homomorphism \begin{equation}\label{eq8.7} \colim_\alpha K_0(\sT_\alpha)\to K_0(\sT). \end{equation} \begin{lemma}\label{l8.1} \eqref{eq8.7} is an isomorphism. \end{lemma} \begin{proof} Write $i_\alpha:\sT_\alpha\to \sT$ for the canonical functor. Let $X\in \sT$. Then $X\simeq i_\alpha X_\alpha$ for some $(\alpha,X_\alpha)$. This shows that \eqref{eq8.7} is surjective. Let $\alpha\in A$ and $x\in K_0(\sT_\alpha)$ be such that $i_\alpha x=0$. Writing $x=\sum_{i\in I} n_i[X_i]$ for $I$ finite, $n_i\in\Z$ and $X_i\in \sT_\alpha$, the hypothesis means that we have an equality \[ \sum_{i\in I} n_i[i_\alpha X_i] = \sum_{j\in J} m_j([Y_j]-[Y'_j]-[Y''_j])\] in the free group with generators the isomorphism classes of objects of $\sT$, where $J$ is finite, $m_j\in\Z$ and $Y'_j\to Y_j\to Y''_j\by{+1}$ are exact triangles. All these exact triangles come from exact triangles in $\sT_\beta$ for some $\beta$ dominating $\alpha$. This shows that \eqref{eq8.7} is injective. \end{proof} For simplicity, we set \[K_0^\sM(S) = K_0(\D(S))\] for any scheme $S$. Let $K$ be a global field. We write $C$ for $\Spec O_K$ if $K$ is number field or for the smooth projective model of $K$ in positive characteristic. Let $j:U\subseteq V$ be two nonempty open subsets of $C$, and let $i:Z\to V$ be to complementary closed immersion. We have an exact sequence of triangulated categories \begin{equation}\label{eq9.4} 0\to \D(Z)\by{i_*} \D(V)\by{j^*}\D(U)\to 0 \end{equation} which is split as in the previous section by $i^!$ and $j_*$. Note also that \[\D(Z)=\coprod_{v\in Z} \D(\kappa(v)).\] Hence we get a short exact sequence \[0\to \bigoplus_{v\in Z} K_0^\sM(\kappa(v)) \by{((i_v)_*)} K_0^\sM(V)\by{j^*} K_0^\sM(U)\to 0\] which is split, although we shall not use this. By Ivorra \cite[Prop. 4.16]{ivorra}, we have an equivalence \[2-\colim_U \D(U) \iso \D(K)\] hence, by Lemma \ref{l8.1}, a (non split) short exact sequence \begin{equation}\label{eq8.0} 0\to \bigoplus_v K_0^\sM(\kappa(v)) \by{((i_v)_*)} K_0^\sM(C)\by{j^*} K_0^\sM(K)\to 0 \end{equation} where $v$ runs through all the closed points of $C$. \subsection{A purity theorem} For the sequel, we note that the theory $\D$ verifies the axioms of \cite[Def. 2.3.1]{ayoub} by \emph{loc. cit.}, Prop. 4.5.31. Let $S$ be a scheme, $i:Z\to S$ a closed immersion and $j:U\to S$ the complementary open immersion. For $M,N\in \D(S)$, we have a natural transformation \cite[\S 2.3.2]{ayoub} \begin{equation}\label{eq8.1} r_{M,N}:i^*M\otimes i^!N\to i^!(M\otimes N) \end{equation} which is not an isomorphism in general (\emph{loc. cit.}, Remark 2.3.13). However, \begin{thm}\label{t8.1} If $M$ is strongly dualisable, \eqref{eq8.1} is an isomorphism. \end{thm} \begin{proof}[Proof (J. Ayoub)] Let $j:U\to S$ be the complementary open immersion. By the localisation exact triangle \begin{equation}\label{eq8.6} i_*i^!N\to N\to j_*j^*N\by{+1} \end{equation} of \cite[\S 1.4.4]{ayoub}, we reduce to the cases where $N$ is of the form $j_* N'$ or $i_*N''$. In the first case, the left hand side is $0$, and so is the right hand side by the projection formula \[M\otimes j_* N' \simeq j_*(j^*M \otimes N')\] which holds because $M$ is dualisable (another lemma of Ayoub, \emph{cf.} \cite[Lemma 9.3.1]{ffihes}). In the second case, \eqref{eq8.1} is the composition \[i^*M \otimes i^!i_*N'' \simeq i^!i_*(i^*M \otimes i^!i_*N'') \simeq i^!(M \otimes i_*i^!i_*N'') \to i^!(M\otimes i_*N'')\] by the strong monoidality of $i_*$ \cite[Lemma 2.3.6]{ayoub}, and the last map is invertible: the counit $i_*i^! \to Id$ is invertible when applied to an object of the form $i_*N''$. \end{proof} Let $M\in \D(S)$. Applying $i^*$ to \eqref{eq8.6}, we get another exact triangle \[i^!M\by{\tau_M} i^*M\to i^*j_*j^*M\by{+1}. \] \begin{cor}\label{c8.1} Suppose that $\tau_{\Z_S}=0$. Then $\tau_M=0$ for any strongly dualisable $M$. \end{cor} \begin{proof}[Proof (J. Ayoub)] We have a commutative diagram \[\begin{CD} i^*M \otimes i^!N @>r_{M,N}>> i^!(M\otimes N) \\ @V{1\otimes \tau_N}VV @V{\tau_{M\otimes N}}VV\\ i^*M \otimes i^*N @>\sim >> i^*(M\otimes N) \end{CD}\] where the top map is an isomorphism by Theorem \ref{t8.1}. The conclusion follows by taking $N=\Z_S$. \end{proof} \begin{rk}\label{r8.1} The hypothesis of Corollary \ref{c8.1} is verified in particular when $Z=\Spec \kappa$ is a closed point of $S$: then $\tau_{\Z_S}\in \D(\kappa)(\Z_Z(-d)[-2d],\Z_Z)=H^{2d}(\kappa,\Q(d))=0$. \end{rk} \begin{cor} \label{c8.2} Suppose that $S$ and $Z$ are regular and that $i$ is of pure codimension $c$. Then the morphism \eqref{eq8.1} taken with $N=\Z_S$ induces an isomorphism \[i^*M(-c)[-2c]\iso i^!M\] for any dualisable $M$. \end{cor} \begin{proof} This follows from Theorem \ref{t8.1} and \cite[Th. 14.4.1]{cis-deg} in the language of Beilinson motives, or from \cite[Cor. 7.5]{ayoubetale} in the language of $\DA^\et$. \end{proof} \begin{prop}\label{p8.1} Let $\D^\proj(S)$ be the thick subcategory of $\D(S)$ generated by the $M(X)(n)$ with $X$ smooth projective over $S$ and $n\in \Z$. Suppose that $\Z_S$ is a dualising object of $\D(S)$. Then all objects of $\D^\proj(S)$ are strongly dualisable. In particular, Theorem \ref{t8.1}, Corollary \ref{c8.1} and Corollary \ref{c8.2} apply to them. \end{prop} \begin{proof} The argument is the same as for \cite[Th. 2.2]{riou}. \end{proof} \begin{rk} As Ayoub pointed out, if one wants to prove Corollary \ref{c8.2} for $M=f_\#\Z_X$ with $f$ smooth projective, one can avoid using Theorem \ref{t8.1} by the following direct computation: $M\simeq f_!f^!\Z_S$ and $i^!f_!f^!\simeq (f_Z)_!f_Z^!i^!$ by the base change isomorphisms, where $f_Z$ is the pull-back of $f$ along $i$. \end{rk} \section{Zeta and $L$-functions of motives over a global field}\label{s9} Let $K$ be a global field. We keep the notation of \S \ref{s8.A}: $C$ denotes either $\Spec O_K$ when $K$ is a number field and $O_K$ is its ring of integers, or the smooth projective model of $K$ over its field of constants $\F_q$ when $K$ is of positive characteristic. \subsection{Zeta functions up to finite Euler products} Let as above $\Dir$ denote the group of convergent Dirichlet series, and let $\Eul$ be the subgroup generated by Euler factors, i.e. Dirichlet series of the form $R(p^{-s})$, where $R\in\Q(t)$ and $p$ is a prime number. From this exact sequence we deduce a map \[\bar\zeta:K_0^\sM(K)\to \Dir/\Eul\] induced by $DM_\gm(O_K,\Q)\ni M\mapsto \zeta(M,s)$. Let $X$ be a smooth projective $K$-variety, and let $\Sigma_K(X)$ be the set of finite places of $K$ where $X$ has good reduction. Recall that, in \cite[Prop. 5.6 and Th. 5.7]{zetaL}, we defined an ``approximate zeta function'' by the formula \[\zeta_{appr}(X,s)=\prod_{v\in \Sigma_K(X)} \zeta(X(v),s)\] where $X(v)$ is the special fibre of a smooth model of $X$ over $\Spec O_v$. (The point is that $\zeta(X(v),s)$ does not depend on the choice of $X(v)$.) Then the following is obvious by construction: \begin{prop} We have $\zeta_{appr}(X,s)=\bar \zeta(X,s)$ in $\Dir/\Eul$.\qed \end{prop} \subsection{A ``total" $L$-function for motives over $\Spec K$}\label{total} We go back to the short exact sequence of triangulated categories \[0\to \coprod_v \D(\kappa(v))\by{(i_v)_*} \D(C)\by{j^*}\D(K)\to 0\] which is the $2$-colimit of the exact sequences \eqref{eq9.4}. It sits fully faithfully into a short exact sequence of larger categories \[0\to \coprod_v \DA^\et(\kappa(v),\Q)\by{(i_v)_*} \DA^\et(C,\Q)\by{j^*}\DA^\et(K,\Q)\to 0.\] In this sequence, $j^*$ has the right adjoint $j_*$ for Brown representability reasons. If $M\in \D(K)$, $j_* M$ is of course not constructible in general; nevertheless we would like to define a ``total $L$-function" of $M$ by the formula \[L^\tot(M,s)=\zeta(j_*M,s).\] Sense can be made of this formula as follows: We may write $M=j_U^*M_U$, where $M_U\in \D(U)$, for some open subset $U$ of $C$. Write $j=j'_U j_U$. Then $j_*M = (j'_U)_*(j_U)_*j_U^*M_U$. Let $j_{U,V}:V\to U$ be an open subset of $U$. From the exact triangles \begin{equation}\label{eq8.2} \bigoplus_{v\in U-V} (i_v)_*i_v^! M_U \to M_U\to (j_{U,V})_*j_{U,V}^*M_U\by{+1} \end{equation} we deduce in the (co)limit an exact triangle \[\bigoplus_{v\in U} (i_v)_*i_v^! M_U \to M_U\to (j_{U})_*j_{U}^*M_U\by{+1}\footnote{For lightness of notation, we write $v\in U$ rather than $v\in U_{(0)}$, and similarly in the sequel.}\] hence, after applying $(j'_U)_*$, an exact triangle \begin{equation}\label{eq8.3} \bigoplus_{v\in U} (i_v)_*i_v^! M_U \to (j'_U)_*M_U\to j_*M\by{+1}. \end{equation} Note that the latter may also be written \[\bigoplus_{v\in C} (i_v)_*i_v^! (j'_U)_*M_U \to (j'_U)_*M_U\to j_*M\by{+1}\] as one sees for example by applying the localisation exact triangles to $(j'_U)_*M_U$. (Here we abuse notation by identifying the closed immersions $v\inj U$ and $v\inj C$, with the common name $i_v$.) The motive $(i_v)_*i_v^! (j'_U)_*M_U$ is $0$ if $v\notin U$, because then $i_v^! (j'_U)_*=0$. \begin{prop}\label{p8.2} For any closed point $v$ of $C$, let $O_v$ be the local ring of $v$ and $j_v:\Spec K\inj\Spec O_v$ the corresponding open immersion. Then, with the above notation, we have the relation \[ [i_v^*(j'_U)_*M_U]-[i_v^! (j'_U)_*M_U]= [i_v^*(j_v)_*M]\in K_0^\sM(\kappa(v)).\] In particular, the left hand side does not depend on the choice of $U$ and $M_U$, and is triangulated in $M$. \end{prop} \begin{proof} Let $(V,M_V)$ be another model of $M$. Then $M_U$ and $M_V$ become isomorphic after restricting to some open subset of $U\cap V$, so we may assume $V\subseteq U$ and $M_V = j_{U,V}^*M_U$. Hence, by \eqref{eq8.2}, it suffices to show that \[ i_w^*(i_v)_*= i_w^!(i_v)_*= \begin{cases} 0& \text{if $w\ne v$}\\ Id_{\D(\kappa(v))}& \text{if $w\ne v$.} \end{cases} \] Both formulas are obvious, the second because $(i_v)_*$ is fully faithful. This shows that the left hand side of the formula only depends on $M$. Since $j^*(j'_U)_*M=M$, we may now suppose that $U=C$ in \eqref{eq8.3}. Let $j'_v:\Spec O_v\inj C$ be the other inclusion. Applying $(j'_v)^*$ to \eqref{eq8.3}, we get an exact triangle \[(i_v)_*i_v^! M_C \to (j'_v)^*M_C\to (j_v)_*M\by{+1}\] hence the formula. \end{proof} \begin{defn}\label{l9.1} For any $M\in \D(K)$ and any $v\in C_{(0)}$, we set \[L^\tot_v(M,s)= \zeta(i_v^*(j_v)_*M,s).\] \end{defn} \begin{defn} Let $M\in \D(K)$ and let $v\in C_{(0)}$. We say that $M$ \emph{has good reduction at $v$} if $M=j_v^*\sM$, with $\sM\in \D^\proj(O_v)$ (see Proposition \ref{p8.1}). We say that such an $\sM$ is a \emph{good model} of $M$ at $v$. \end{defn} Let $M\in \D(K)$, and let $\sM\in \D(O_v)$ be such that $j_v^*\sM=M$. The exact triangle \[(i_v)_*i_v^!\sM\to \sM\to (j_v)_*M\by{+1}\] gives, after applying $i_v^*$, an exact triangle \[i_v^!\sM\to i_v^*\sM\to i_v^*(j_v)_*M\by{+1}.\] If $M$ has good reduction at $v$ and $\sM$ is a good model, this triangle reads as \[i_v^*\sM(-1)[-2]\to i_v^*\sM\to i_v^*(j_v)_*M\by{+1}\] thanks to Theorem \ref{t8.1} and Proposition \ref{p8.1}. (The first map is trivial by Corollary \ref{c8.1} and Remark \ref{r8.1}, although we shall not need this.) Thus \begin{equation}\label{eq8.4} L^\tot_v(M,s) = \frac{\zeta(i_v^*\sM,s)}{\zeta(i_v^*\sM,s+1)} \end{equation} in this case, thanks to Corollary \ref{c1} c). \begin{thm}\label{t8.2} With the above notation,\\ a) The Euler product \[L^\tot(M,s)= \prod_{v\in C} L_v^\tot(M,s)\] is a Dirichlet series which converges absolutely for $\Re(s)\gg 0$.\\ b) If $M=M(X)^*$, where $X$ is smooth projective, we have for $v\nmid l$ \[L^\tot_v(M,s)=\prod_{i\in\Z} L_v^\tot(H^i_l(X),s)^{(-1)^i}\] where $v$ runs through the maximal ideals of $O_K$ and \[L_v^\tot(H^i_l(X),s) = \frac{\det(1-N(v)^{-s}\phi_v^{-1}\mid H^0(I_v,H^i(\bar X,\Q_l))}{\det(1-N(v)^{-s}\phi_v^{-1}\mid H^1(I_v,H^i(\bar X,\Q_l))}\] where $\phi_v$ and $I_v$ are respectively a Frobenius at $v$ and the inertia group at $v$.\\ c) If $X$ has good reduction at $v$ in b), with special fibre $X_v$, then \[L_v^\tot(M,s) = \frac{\zeta(X_v,s)}{\zeta(X_v,s+1)}.\] \end{thm} \begin{proof} a) In view of Theorem \ref{t6.1} and Proposition \ref{p8.2}, it suffices to see that $i_v^!M_U\simeq i_v^*M_U(-1)[-2]$ for almost all $v$. But for $V\subseteq U$ small enough, $j_{U,V}^*M_U\in \D^\proj(V)$; hence this follows from Theorem \ref{t8.1}. b) We have the isomorphism \[R^l(i_v^*(j_v)_*M)\simeq i_v^*R(j_v)_* R^l(M)\] for $M\in \D(K)$. For $M=M(X)^*$, $f:X\to \Spec K$ smooth projective, we have \[R^l(M)= Rf_*\Q_l\] hence \[H^i(R^l(M)) = H^{i}(\bar X,\Q_l).\] Let $I_v$ be the absolute inertia group at $v$. For an $l$-adic sheaf $\sF$ on $\Spec K$, we have \[i_v^*R^q(j_v)_*\sF = \begin{cases} H^0(I_v,\sF(\bar K)) &\text{if $q=0$}\\ H^1(I_v,\sF(\bar K)) &\text{if $q=1$}\\ 0 &\text{if $q>1$} \end{cases}\] with Frobenius action induced by the action of $G_K$. If $\sF=H^i(\bar X,\Q_l)$ for $X$ smooth projective, we have \[ L(\kappa(v),H^i(I_v,\sF(\bar K)),s)= \det(1-N(v)^{-s}\phi_v^{-1}\mid H^{i}(I_v,H^i(\bar X,\Q_l))). \] and \[\frac{L(\kappa(v),H^0(I_v,\sF(\bar K)),s)}{L(\kappa(v),H^1(I_v,\sF(\bar K)),s)} =\frac{\det(1-N(v)^{-s-1}\phi_v \mid H^1(I_v,H^i(\bar X,\Q_l)))}{\det(1-N(v)^{-s-1}\phi_v \mid H^0(I_v,H^i(\bar X,\Q_l)))}.\] \end{proof} \subsection{The nearby $L$-function of a motive over $\Spec K$} We now apply Lemma \ref{l2} to $L^\tot(M,s)$. This gives \begin{defn}\label{d9.1} For $M\in \D(K)$ and a prime $v$, we define $L_v^\near(M,s)$ as the unique Dirichlet series (with initial coefficient $1$) such that \[\frac{L_v^\near(M,s)}{L_v^\near(M,s+1)} = L_v^\tot(M,s).\] We set \[L^\near(M,s)=\prod_v L_v^\near(M,s).\] \end{defn} Clearly, \[\frac{L^\near(M,s)}{L^\near(M,s+1)} = L^\tot(M,s)\] which shows by Lemma \ref{l2} that $L^\near(M,s)$ is a convergent Dirichlet series with the same absolute convergence abscissa as $L^\tot(M,s)$. If $M$ has good reduction at $v$, then for any good model $\sM$ at $v$, one has \begin{equation}\label{eq9.1} L_v^\near(M,s)= \zeta(i_v^*\sM,s) \end{equation} thanks to \eqref{eq8.4}. We shall now handle the general case and relate $L^\near(M,s)$ with Ayoub's nearby cycle functor $\Upsilon$ \cite{ayoubetale}. \begin{thm}\label{t9.1} We have $L^\near_v(M,s)=\zeta(\Upsilon_v M,s)$, where $\Upsilon_v:\D(K)\allowbreak\to \D(\kappa(v))$ is the ``unipotent" specialisation functor associated to $O_v$ as in \cite[Th. 11.13]{ayoubetale}. \end{thm} \begin{proof} By \emph{loc. cit.}, Th. 11.16, there is an exact triangle \begin{equation}\label{eq9.2} i_v^* (j_v)_*M\to \Upsilon_v M\to \Upsilon_v M(-1)\by{+1} \end{equation} hence the result follows from the uniqueness statement in Lemma \ref{l2}. \end{proof} \begin{cor} $L^\near_v(M,s)$ is a rational function in $N(v)^{-s}$, whose zeroes and poles are $N(v)$-Weil numbers.\qed \end{cor} This is remarkable because, in Lemma \ref{l2}, $g(s)$ is in general by no means a rational function of $p^{-s}$ when $f(s)$ is. (Take $f(s)=(1-p^{-s})^{-1}$.) \begin{rk}\label{r9.1} Another argument, which was our initial argument, is to go via the $l$-adic realisation: one has \[L_v^\tot(M,s) = L_v(i_v^*R(j_v)_*R^l(M),s).\] If $V$ is an $l$-adic representation of $G_K$, we need to show that \[L(i_v^*R(j_x)_* V,s)= f(N(v)^{-s})/f(N(x)^{-s-1})\] for some $f\in \Q(t)$. We have \[L(i_v^*R(j_v)_* V,s)= \frac{\det(1-\phi_vN(v)^{-s}\mid H^1(I_v,V))}{\det(1-\phi_vN(v)^{-s}\mid H^0(I_v,V))}.\] Since $cd_l(I_v)=1$ this is an Euler-Poincar\'e characteristic, so we may assume $V$ \emph{semi-simple}. Then $I_v$ acts through a finite quotient by the $l$-adic monodromy theorem \cite[Appendix]{serre-tate}, thus \[H^1(I_v,V) = V_{I_v}(-1) \simeq V^{I_v}(-1)\] and \[L(i_x^*R(j_v)_* V,s)= \frac{L^\Serre(V^{ss},s)}{L^\Serre(V^{ss},s+1)}\] where $V^{ss}$ is the semi-simplification of $V$. We thus get another formula for $L^\near(M,s)$: \begin{equation}\label{eq9.3} L^\near(M,s)=L^\Serre(R^l(M)^{ss},s). \end{equation} Because of the finiteness of the action of the inertia, one might think of the right hand side of \eqref{eq9.3} as an \emph{Artin} $L$-function. \end{rk} \begin{qns} Composing with the functor $\Phi$ of \S \ref{s2.E}, we may associate a nearby $L$-function to any Chow motive, and this determines $L^\near$ by Bondarko's theorem.\\ 1) By the $l$-adic realisation, this definition factors through homological equivalence. Does it even factor through numerical equivalence, as in positive characteristic? (The two $K_0$'s agree under the sign conjecture.)\\ 2) Similarly, the operator $\Upsilon_v$ induces a homomorphism \[\Upsilon_v:K_0(\Chow(K))\to K_0(\Chow(\kappa(v)).\] To what extent can one describe it explicitly? \end{qns} \subsection{Examples}\label{s9.1} We finish with explicit computations. \subsubsection{Artin motives} In what comes before, we worked with motives with $\Q$-coefficients, but everything works just as well for motives with coefficients in a $\Q$-algebra, for example in a number field. This allows us to consider the nearby $L$-function $L^\near(\rho,s)$ attached to a complex Galois representation $\rho$. Then the action of inertia is semi-simple, thus, by \eqref{eq9.3} we have $L^\near(\rho,s)=L(\rho,s)$, the Artin $L$-function of $\rho$. So nothing new happens here. \subsubsection{Elliptic curves}\label{s9.D.2}Let $E$ be an elliptic curve over $K$ with multiplicative reduction at $v$, $V= H^1_l(E)$. By hypothesis, the action of $I_v$ on $V$ is unipotent and nontrivial, hence $\dim V^{I_v}=1$ and $I_v$ acts trivially on $V/V^{I_v}$; thus $V^{ss}=V^{I_v}\oplus V/V^{I_v}$ and \begin{align*} L_v^\Serre(h^1(E),s) &= \det(1-N(v)^{-s}\phi_x\mid V^{I_v})^{-1}\\ L_v^\near(h^1(E),s) &= L_v^\Serre(H^1(E),s)\times \det(1-N(v)^{-s}\phi_v\mid V/V^{I_v})^{-1}. \end{align*} Extra poles thus are explicitly computable: if the multiplicative reduction is split, then $L_v^\Serre(h^1(E),s)=(1-N(v)^{-s})^{-1}$ and thus the other factor is $(1-N(v)^{1-s})^{-1}$, since the determinant is $H^2_l(E)\simeq \Q_l(-1)$. Similarly, if the multiplicative reduction is not split, the other factor is $(1+N(v)^{1-s})^{-1}$. Note that these factors have a functional equation between $s$ and $2-s$; so, such functional equations for $L^\Serre(h^1(E),s)$ and $L^\near(h^1(E),s)$ are equivalent. Similarly, the Beilinson conjectures for the first function easily imply Beilinson-like conjectures for the second, and conversely. \section{The functional equation in characteristic $p$} Let $K=\F_q(C)$ for $C$ a smooth, projective, geometrically connected curve. As usual we abbreviate $\F_q=:k$, $\eta =\Spec K$ and write $j:\eta\to C$ for the canonical immersion. If $U\subseteq C$ is a nonempty open subset, we factor $j$ as a composition \[\eta\by{j_U} U\by{j'_U}C.\] Let $M\in \DM_\gm(K,\Q)$: we want to compare the $L$-functions \[L^\near(M^*,1-s)\text{ and } L^\near(M,s).\] We know that $M\simeq j^*\sM$ for some $\sM\in \D(C)$; from the functional equation for $\zeta(\sM,s)$ (Theorem \ref{t3.1}) we can get an approximate functional equation for $L^\near(M,s)$, but we would like a precise formula. The following first approach was suggested by Joseph Ayoub. There exists $U$ and $\sM_U\in \D^\proj(U)$ such that $M=j_U^*\sM_U$. Let $\sM=(j'_U)_* \sM_U$ with $j'_U:U\inj C$, so that $M=j^*\sM$. For $x\in C$, we have by \eqref{eq9.1} and Theorem \ref{t9.1}: \[L^\near_x(M,s)= \begin{cases} \zeta(i_x^*\sM_U,s) &\text{for $x\in U$}\\ \zeta(\Upsilon_x(M),s) &\text{for all $x\in C$} \end{cases} \] hence \begin{multline*} L^\near(M,s)= \prod_{x\in U}\zeta(i^*_x\sM_U,s)\times \prod_{x\notin U} \zeta(\Upsilon_x(M),s)\\ =\zeta(\sM_U,s)\times \prod_{x\notin U} \zeta(\Upsilon_x(M),s) \\ = \zeta(\sM,s)\times \prod_{x\notin U} \zeta(i_x^*(j'_U)_*\sM_U,s)^{-1} \times \prod_{x\notin U} \zeta(\Upsilon_x(M),s)\\ = \zeta(\sM,s)\times \prod_{x\notin U} \zeta(\Upsilon_x(M),s-1) \end{multline*} by \eqref{eq9.2}. On the other hand, $\sM^*:=\uHom(\sM,\Z)=(j'_U)_!\sM_U^*$, hence \begin{multline*} L^\near(M^*,1-s)= \prod_{x\in U}\zeta(i^*_x\sM_U^*,1-s)\times \prod_{x\notin U} \zeta(\Upsilon_x(M^*),1-s)\\ =\zeta(\sM^*,1-s)\times \prod_{x\notin U} \zeta(\Upsilon_x(M^*),1-s). \end{multline*} By \cite[Th. 11.16]{ayoubetale}, we have \[\Upsilon_x(M^*)\simeq \Upsilon_x(M)^* \] where the duality on the right hand side is still relative to $\Z$, but in $\D(\kappa(x))$. Hence \begin{multline*} L^\near(M^*,1-s)=\zeta(\sM^*,1-s)\times \prod_{x\notin U} \zeta(\Upsilon_x(M)^*,1-s)\\ =(-q^{-s})^{\chi(f_!\sM)}\det(F_{f_!\sM})^{-1}\zeta(\sM,s)\\ \times \prod_{x\notin U} (-q_x^{1-s})^{\chi(\Upsilon_x(M))}\det(F_{\Upsilon_x(M)})^{-1}\zeta(\Upsilon_x(M),s-1) \end{multline*} where $q_x = |\kappa(x)|=q^{\deg(x)}$. We finally get \begin{thm}\label{t9.2} One has \[\frac{L^\near(M^*,1-s)}{L^\near(M,s)}=A (-q)^{-Bs} \] with \begin{align*} A&=(-q)^{\sum\limits_{x\notin U} \deg(x) \chi(\Upsilon_x(M))}\left(\det(F_{f_!\sM})\times \prod_{x\notin U} \det(F_{\Upsilon_x(M)})\right)^{-1}\\ B&=\chi(f_!\sM))+\sum_{x\notin U} \deg(x) \chi(\Upsilon_x(M)).\qed \end{align*} \end{thm} \begin{qn}\label{q10.1} Comparing with \cite[Formulas (6) and (7)]{serre}, one would like to relate at least the constant $B$ to a \emph{conductor} of $M$. This can be defined \emph{via} the $l$-adic realisation; can one prove that the conductor thus obtained does not depend on $l$? \end{qn} \begin{thebibliography}{SGA4 1/2} \bibitem{akos} Y. André, B. Kahn {\it Nilpotence, radicauxet structures monoïdales} (with an appendix by P. O'SUllivan), Rendiconti Math. Univ. Padova {\bf 108} (2002),108--291. \bibitem{ayoub} J. Ayoub Les six op\'erations de Grothendieck et le formalisme des cycles \'evanescents dans le monde motivique, I, II, Ast\'erisque {\bf 314--315}, SMF, 2008. \bibitem{ayoubetale} J. Ayoub {\it La r\'ealisation \'etale et les op\'erations de Grothendieck}, Ann. sci. Éc. norm. sup. {\bf 47} (2014), 1--145. \bibitem{bar-kahn} L. Barbieri-Viale, B. Kahn {\it On the derived category of $1$-motives}, Astérisque {\bf 381} (2016), 1--254. \bibitem{bondarko1} M. V. Bondarko Differential graded motives: weight complex, weight filtrations and spectral sequences for realizations; Voevodsky versus Hanamura. J. Inst. Math. Jussieu {\bf 8} (2009), 39--97. \bibitem{bondarko} M. V. Bondarko {\it Weight structures vs. t-structures; weight filtrations, spectral sequences, and complexes (for motives and in general)}, J. of $K$-theory {\bf 6} (2010), 387--504. \bibitem{cisinski} D.-C. Cisinski {\it Cohomological methods in intersection theory}, Homotopy theory and arithmetic geometry--motivic and Diophantine aspects, 49--105, Lect. Notes in Math. {\bf 2292}, Springer, 2021. \bibitem{cis-deg2} D.-C. Cisinski, F. D\'eglise {\it Étale motives}, Compositio Math. {\bf 152} (2016) 556--666. \bibitem{cis-deg} D.-C. Cisinski, F. D\'eglise {\it Triangulated categories of motives}, Springer Monographs in Math. {\bf 2019}, xlii+406 pp. \bibitem{corr} P. Colmez, J.-P. Serre Correspondance Grothendieck-Serre, Documents mathématiques {\bf 2}, SMF, 2001.  \bibitem{WeilI} P. Deligne {\it La conjecture de Weil, I}, Publ. Math. IH\'ES {\bf 43} (1974), 273--307. \bibitem{WeilII} P. Deligne {\it La conjecture de Weil, II}, Publ. Math. IH\'ES {\bf 52} (1980), 137--252.\bibitem{deligne} {\it Le groupe fondamental de la droite projective moins trois points}, {\it in} Galois groups over $\Q$, Math. Sci. Res. Inst. Publ., {\bf 16}, Springer, 1989, 79--297. \bibitem{den} C. Deninger {\it $L$-functions of mixed motives}, {\it in} Motives (U. Jannsen, S. Kleiman, J.-P. Serre, eds.), Proc. Symposia pure Math. {\bf 55} (I), AMS, 1994, 517--525. \bibitem{Ekedahl} T. Ekedahl {\it On the adic formalism}, {\it in} The Grothendieck Festschrift (P. Cartier et al., eds.), Vol. II, Birkh\"auser 1990, 197--218. \bibitem{fpr} J.-M. Fontaine, B. Perrin-Riou {\it Autour des conjectures de Bloch et Kato: cohomologie galoisienne et valeurs de fonctions $L$}, {\it in} Motives (U. Jannsen, S. Kleiman, J.-P. Serre, eds.), Proc. Symposia pure Math. {\bf 55} (I), AMS, 1994, 599--706. \bibitem{guletskii} V. Guletskii {\it Zeta functions in triangulated categories}, Mat. Zametki {\bf 87} (2010), 369--381; translation in Math. Notes {\bf 87} (2010), 345--354. \bibitem{schol-etal} T. Hemo, T. Richarz, J. Scholbach {\it Constructible sheaves on schemes}, preprint, 2023, \url{https://doi.org/10.48550/arXiv.2305.18131}. \bibitem{motiftate} A. Huber, B. Kahn {\it the slice filtration and mixed Tate motives}, Compositio Math. {\bf 142} (2006), 907--936. \bibitem{illusie} L. Illusie {\it Constructibilité générique et uniformité en $\ell$}. Tunis. J. Math. {\bf 4} (2022), 159--181. \bibitem{ivorra} F. Ivorra {\it R\'ealisation $l$-adique des motifs triangul\'es g\'eom\'etriques. I}, Doc. Math. {\bf 12} (2007), 607--671. \bibitem{jannsen} U. Jannsen Mixed motives and algebraic $K$-theory, Lect. Notes {\bf 1400}, Springer, 1990. \bibitem{dJ} A.J. de Jong {\it Smoothness, semi-stability and alterations}, Publ. Math. IH\'ES {\bf 83} (1996), 51--93. \bibitem{dJ2} A.J. de Jong {\it Families of curves and alterations}, Ann. Inst. Fourier {\bf 47} (1997), 599--621. \bibitem{modq} B. Kahn {\it Zeta functions and motives}, Pure Appl. Math. Quarterly {\bf 5} (2009), 507--570 [2008]. \bibitem{zetaLRegensburg} B. Kahn {\it Zeta and $L$-functions of triangulated motives}, slides of a talk at the workshop \emph{Algebraic cycles and $L$-functions}, Regensburg, Feb. 27 -- March 2, 2012, \url{https://webusers.imj-prg.fr/~bruno.kahn/preprints/zetaL-Regensburg.pdf}. \bibitem{ffihes} B. Kahn {\it The full faithfulnees conjectures in characteristic $p$}, {\it in} Autour des motifs (II), Panoramas et synthèses {\bf 41}, SMF, 2013, 187--244. \bibitem{adjoints} B. Kahn {\it Motifs et adjoints}, Rendiconti Sem. Mat. Univ. Padova {\bf 139} (2018), 77--128. \bibitem{zetaL} B. Kahn Zeta and $L$-functions of varieties and motives, London Math. Soc. Lect. Notes Ser. {\bf 462}, Cambridge Univ. press, 2018. \bibitem{ks} B. Kahn, R. Sujatha {\it Birational motives, I: pure birational motives}, Ann. K-Theory {\bf 1} (2016), 379--440. \bibitem{birat-tri} B. Kahn, R. Sujatha {\it Birational motives, II: triangulated birational motives}, IMRN {\bf 2017}, no. 22, 6778--6831. \bibitem{karp} N. Karpenko {\it Weil transfer for algebraic cycles}, Indag. Math. (N.S.) {\bf 11} (2000), 73--86. \bibitem{kelly} S. Kelly Voevodsky motives and $l$dh-descent, Astérisque {\bf 391}, SMF, 2017. \bibitem{kleiman} S. Kleiman {\it Motives}, Algebraic geometry, Oslo 1970, Proc. Fifth Nordic Summer School in Math., Wolters-Noordhoff, Groningen, 1972, 53--82. \bibitem{may} J. P. May {\it The additivity of traces in triangulated categories}, Adv. Math. {\bf 163} (2001), 34--73. \bibitem{mvw} C. Mazza, V. Voevodsky, C. Weibel, Lecture notes on motivic cohomology, Clay Mathematics Monographs {\bf 2}, American Mathematical Society, Providence, RI; Clay Mathematics Institute, Cambridge, MA, 2006. \bibitem{pacheco} A. Pacheco {\it A note on relations between the zeta-functions of Galois coverings of curves over finite fields}, Canad. Math. Bulletin {\bf 33} (1990), 282--285. \bibitem{raynaud} M. Raynaud {\it Caractéristique d'Euler-Poincaré d'un faisceau et cohomologie des variétés abéliennes}, Sém. Bourbaki 1964-66, exp. n° {\bf 286}, 129--147, SMF. \bibitem{riou} J. Riou {\it Dualité de Spanier-Whitehead en géométrie algébrique}, C. R. Math. Acad. Sci. Paris {\bf 340} (2005), no. 6, 431–436. \bibitem{saa} N. Saavedra Rivano Cat\'egories tannakiennes, Lect. Notes in Math. {\bf 265}, Springer, 1972. \bibitem{tsaito} T. Saito {\it The characteristic cycle and the singular support of a constructible sheaf}, Invent. Math {\bf 207} (2) (2017), 597--695. Correction: \emph{ibid.}, {bf 216} (3) (2019), 1005--1006. Correction 2: \url{https://www.ms.u-tokyo.ac.jp/~t-saito/CCcor2.pdf}. \bibitem{scholl} A. J. Scholl {\it Remarks on special values of $L$ functions}, {\it in} $L$-functions and arithmetic, London Math. Soc. Lect. Notes Ser. {\bf 153}, Cambridge Univ. Press, 1991, 373--392. \bibitem{cg} J.-P. Serre Cohomologie galoisienne (5th edition), Lect. Notes in Math. {\bf 5}, Springer, 1994. \bibitem{serre} J.-P. Serre {\it Facteurs locaux des fonctions z\^eta des vari\'et\'es alg\'ebriques (d\'efinitions et conjectures)}, S\'em. DPP 1969/70, exp. {\bf 19}. \bibitem{NX} J.-P. Serre Lectures on $N_X(p)$, Research Notes in Math. {\bf 11}, CRC Press, 2012. \bibitem{serre-tate} J.-P. Serre, J. Tate {\it Good reduction of abelian varieties}, Ann. of Math. {\bf 88} (1968), 492--517. \bibitem{suslininsep} A. Suslin {\it Motivic complexes over nonperfect fields}, Ann. of $K$-theory {\bf 2} (2017), 277--302. \bibitem{ter} T. Terasoma {\it Monodromy weight filtration is independent of $l$}, preprint, 1998, \url{https://arxiv.org/abs/math/9802051}. \bibitem{voetri} V. Voevodsky {\it Triangulated categories of motives over a field}, {\it in} Cycles, transfers and motivic cohomology theories, Ann. of Math. Studies. {\bf 143}, Princeton Univ. press, 2000, 188--238. \begin{center} \sc Acronyms \end{center} \bibitem[SGA1]{SGA1} Revêtements étales et groupe fondamental (SGA 1) (A. Grothendieck et al), revised ed., Doc. Math. {\bf 3}, SMF, 2003. \bibitem[SGA4 1/2]{SGA412} Cohomologie \'etale (SGA 4 1/2) (P. Deligne et al), Lect. Notes in Math. {\bf 569}, Springer, 1977. \bibitem[SGA5]{SGA5} Cohomologie \'etale et fonctions L (SGA5) (A. Grothendieck et al), Lect. Notes in Math. {\bf 589}, Springer, 1977. \bibitem[SGA7]{SGA7} Groupes de monodromie en g\'eom\'etrie alg\'ebrique (SGA7) (P. Deligne, N. Katz, \'ed.), Vol. II, Lect. Notes in Math. {\bf 340}, Springer, 1973. \end{thebibliography} \end{document}
2412.08597v1
http://arxiv.org/abs/2412.08597v1
Positive co-degree densities and jumps
\documentclass[12pt]{article} \usepackage[utf8]{inputenc} \usepackage{cite} \usepackage{url} \usepackage{caption} \usepackage{amssymb, amsmath, amsthm} \usepackage{tikz} \usepackage{thm-restate} \usepackage{cellspace} \setlength{\cellspacetoplimit}{6pt} \setlength{\cellspacebottomlimit}{6pt} \usepackage[pdftex,hypertexnames=false,linktocpage=true,hidelinks]{hyperref} \usepackage{cleveref} \usepackage{fullpage} \usepackage{amsfonts} \usepackage{enumitem} \newtheorem{theorem}{Theorem}[section] \newtheorem{lemma}[theorem]{Lemma} \newtheorem{cor}[theorem]{Corollary} \newtheorem{conj}[theorem]{Conjecture} \newtheorem{prop}[theorem]{Proposition} \newtheorem{claim}[theorem]{Claim} \newtheorem{observation}[theorem]{Observation} \theoremstyle{definition} \newtheorem{definition}[theorem]{Definition} \newtheorem{remark}[theorem]{Remark} \newtheorem{ques}{Question} \newtheorem{case}{Case} \numberwithin{equation}{section} \newcommand{\acopyof}{} \tikzset{vtx/.style={inner sep=2.7pt, outer sep=0pt, circle, fill=red,draw}} \newcommand{\vc}[1]{\ensuremath{\vcenter{\hbox{#1}}}} \tikzset{flag_pic/.style={scale=1}} \tikzset{unlabeled_vertex/.style={inner sep=1.7pt, outer sep=0pt, circle, fill}} \tikzset{labeled_vertex/.style={inner sep=2.2pt, outer sep=0pt, rectangle, fill=gray, draw=black}} \tikzset{edge_color0/.style={color=black,line width=1.2pt,opacity=0.5,dashed}} \tikzset{edge_color1/.style={color=red, line width=1.2pt,opacity=0}} \tikzset{edge_color2/.style={color=blue, line width=1.2pt,opacity=1}} \tikzset{edge_color3/.style={color=green!80!black,line width=1.2pt}} \tikzset{edge_color4/.style={color=orange, line width=1.2pt}} \tikzset{edge_color5/.style={color=red, line width=1.2pt,dotted}} \tikzset{edge_color6/.style={color=blue, line width=1.2pt,dotted}} \tikzset{edge_color7/.style={color=green, line width=1.2pt,dotted}} \tikzset{edge_color8/.style={color=gray, line width=1.2pt}} \tikzset{edge_colorroot/.style={color=red, line width=1.7pt}} \tikzset{edge_thin/.style={color=black}} \tikzset{edge_hidden/.style={color=black,dotted,opacity=0}} \tikzset{vertex_color1/.style={inner sep=1.7pt, outer sep=0pt, draw, circle, fill=red}} \tikzset{vertex_color2/.style={inner sep=1.7pt, outer sep=0pt, draw, circle, fill=blue}} \tikzset{vertex_color3/.style={inner sep=1.7pt, outer sep=0pt, draw, circle, fill=green}} \tikzset{vertex_color4/.style={inner sep=1.7pt, outer sep=0pt, draw, circle, fill=pink}} \tikzset{vertex_color5/.style={inner sep=1.7pt, outer sep=0pt, draw, circle, fill=pink,label=below:{$5$}}} \tikzset{vertex_color6/.style={inner sep=1.7pt, outer sep=0pt, draw, circle, fill=pink,label=below:{$6$}}} \tikzset{vertex_color7/.style={inner sep=1.7pt, outer sep=0pt, draw, circle, fill=pink,label=below:{$7$}}} \tikzset{vertex_color8/.style={inner sep=1.7pt, outer sep=0pt, draw, circle, fill=pink,label=below:{$8$}}} \tikzset{vertex_color9/.style={inner sep=1.7pt, outer sep=0pt, draw, circle, fill=pink,label=below:{$9$}}} \tikzset{vertex_color10/.style={inner sep=1.7pt, outer sep=0pt, draw, circle, fill=pink,label=below:{$10$}}} \tikzset{vertex_color11/.style={inner sep=1.7pt, outer sep=0pt, draw, circle, fill=pink,label=below:{$11$}}} \tikzset{vertex_color12/.style={inner sep=1.7pt, outer sep=0pt, draw, circle, fill=pink,label=below:{$12$}}} \tikzset{vertex_color13/.style={inner sep=1.7pt, outer sep=0pt, draw, circle, fill=pink,label=below:{$13$}}} \tikzset{vertex_color14/.style={inner sep=1.7pt, outer sep=0pt, draw, circle, fill=pink,label=below:{$14$}}}\tikzset{labeled_vertex_color1/.style={inner sep=2.2pt, outer sep=0pt, draw, rectangle, fill=red}} \tikzset{labeled_vertex_color2/.style={inner sep=2.2pt, outer sep=0pt, draw, rectangle, fill=blue}} \tikzset{labeled_vertex_color3/.style={inner sep=2.2pt, outer sep=0pt, draw, rectangle, fill=green}} \tikzset{labeled_vertex_color4/.style={inner sep=2.2pt, outer sep=0pt, draw, rectangle, fill=pink}} \def\outercycle#1#2{ \pgfmathtruncatemacro{\plusone}{#1+1} \pgfmathtruncatemacro{\minusone}{#1-1} \draw \foreach \x in {0,...,\minusone}{(0.5*\x,0) coordinate(x\x)};} \def\drawhypervertex#1#2{ \pgfmathtruncatemacro{\plusone}{#1+1} \draw[edge_color2] (x#1)++(0,-0.2-0.2*#2)+(-0.2,0) -- +(0.2,0) +(0,0) node[fill=white,outer sep=0,inner sep=0]{\tiny \plusone};} \def\drawrootedhypervertex#1#2{ \pgfmathtruncatemacro{\plusone}{#1+1} \draw[edge_color8] (x#1)++(0,-0.2-0.2*#2)+(-0.2,0) -- +(0.2,0) +(0,0) node[fill=white,outer sep=0,inner sep=0]{\tiny \plusone};} \def\drawrootedhyperroot#1#2{ \pgfmathtruncatemacro{\plusone}{#1+1} \draw[edge_colorroot] (x#1)++(0,-0.2-0.2*#2)+(-0.2,0) -- +(0.2,0) +(0,0) node[fill=white,outer sep=0,inner sep=0]{\tiny \plusone};} \def\drawhypervertexcolor#1#2#3{ \pgfmathtruncatemacro{\plusone}{#1+1} \pgfmathtruncatemacro{\plusone}{#1+1} \draw[edge_color#3] (x#1)++(0,-0.2-0.2*#2)+(-0.2,0) -- +(0.2,0) +(0,0) node[fill=white,outer sep=0,inner sep=0]{\tiny \plusone};} \def\drawhyperedge#1#2{\pgfmathtruncatemacro{\plusone}{#1+1} \draw[dotted] (x0)++(0,-0.2-0.2*#1)--++(0.5*#2-0.5,0);} \def\labelvertex#1{\pgfmathtruncatemacro{\vertexlabel}{#1+1 } \draw (x#1) node{\color{yellow}\tiny\vertexlabel}; } \tikzset{ vtx/.style={inner sep=1.1pt, outer sep=0pt, circle, fill,draw}, vtxl/.style={inner sep=1.1pt, outer sep=0pt, rectangle, fill=yellow,draw=black}, hyperedge/.style={fill=pink,opacity=0.5,draw=black}, } \newcommand{\Kfourthree}{\vc{ \begin{tikzpicture} \draw (0,0) coordinate(1) node[vtx](a){} (1,0) coordinate(2) node[vtx](b){} (1,1) coordinate(3) node[vtx](c){} (0,1) coordinate(4) node[vtx](d){} ; \draw[hyperedge] (1) to[out=30,in=90] (2) to[out=100,in=260] (3) to[out=260,in=30] (1); \draw[hyperedge] (1) to[out=90,in=150] (2) to[out=150,in=280] (4) to[out=280,in=90] (1); \draw[hyperedge] (1) to[out=60,in=200] (3) to[out=200,in=330] (4) to[out=330,in=60,looseness=1.5] (1); \draw[hyperedge] (2) to[out=120,in=230,,looseness=1.8] (3) to[out=230,in=300,looseness=1.8] (4) to[out=300,in=120,looseness=1.8] (2); \end{tikzpicture} }} \newcommand{\ah}[1]{{\color{blue}{Anna: #1}}} \newcommand{\cp}[1]{{\color{red}{Cory: #1}}} \newcommand{\bl}[1]{{\color{green!70!black}{Bernard: #1}}} \definecolor{darkpastelgreen}{rgb}{0.01, 0.75, 0.24} \title{Positive co-degree densities and jumps} \author{J\'ozsef Balogh \footnote{Department of Mathematics, University of Illinois Urbana-Champaign, IL, USA E-mail: \texttt{[email protected]}. Research supported in part by NSF grants RTG DMS-1937241 and FRG DMS-2152488, the Arnold O. Beckman Research Award (UIUC Campus Research Board RB 24012).} \and Anastasia Halfpap\footnote{Department of Mathematics, Iowa State University, Ames, IA, USA. E-mail: \texttt{[email protected]}. Research supported by NSF DSM-2152490.} \and Bernard Lidick\'y \footnote{Department of Mathematics, Iowa State University, Ames, IA, USA. E-mail: \texttt{[email protected]}. Research supported by NSF DSM-2152490 and Scott Hanna Professorship.} \and Cory Palmer \footnote{Department of Mathematical Sciences, University of Montana. Email: \texttt{[email protected]}. Research supported by a grant from the Simons Foundation \#712036.} } \date{} \newcommand{\oururl}{\url{https://lidicky.name/pub/pco/}} \begin{document} \maketitle \begin{abstract} The \textit{minimum positive co-degree} of a nonempty $r$-graph $H$, denoted by $\delta_{r-1}^+(H)$, is the largest integer $k$ such that for every $(r-1)$-set $S \subset V(H)$, if $S$ is contained in a hyperedge of $H$, then $S$ is contained in at least $k$ hyperedges of $H$. Given a family $\mathcal{F}$ of $r$-graphs, the \textit{positive co-degree Tur\'an function} $\mathrm{co^+ex}(n,\mathcal{F})$ is the maximum of $\delta_{r-1}^+(H)$ over all $n$-vertex $r$-graphs $H$ containing no member of $\mathcal{F}$. The \textit{positive co-degree density} of $\mathcal{F}$ is $\gamma^+(\mathcal{F}) = \underset{n \rightarrow \infty}{\lim} \frac{\mathrm{co^+ex}(n,\mathcal{F})}{n}.$ While the existence of $\gamma^+(\mathcal{F})$ is proved for all families $\mathcal{F}$, only few positive co-degree densities are known exactly. For a fixed $r \geq 2$, we call $\alpha \in [0,1]$ an \textit{achievable value} if there exists a family of $r$-graphs $\mathcal{F}$ with $\gamma^+(\mathcal{F}) = \alpha$, and call $\alpha$ a \textit{jump} if for some $\delta > 0$, there is no family $\mathcal{F}$ with $\gamma^+(\mathcal{F}) \in (\alpha, \alpha + \delta)$. Halfpap, Lemons, and Palmer~\cite{halfpap2024positive} showed that every $\alpha \in [0, \frac{1}{r})$ is a jump. We extend this result by showing that every $\alpha \in [0, \frac{2}{2r -1})$ is a jump. We also show that for $r = 3$, the set of achievable values is infinite, more precisely, $\frac{k-2}{2k-3}$ for every $k \geq 4$ is achievable. Finally, we determine two additional achievable values for $r=3$ using flag algebra calculations. \end{abstract} \noindent \emph{Keywords:} positive co-degree, hypergraph, Tur\'an, jump, flag algebra.\\ \emph{MSC2020:} 05C35, 05C65. \section{Introduction} An $r$-\textit{graph} is a hypergraph in which all hyperedges have size $r$. We often refer to the hyperedges of an $r$-graph as $r$-\textit{edges}. Given a family of $r$-graphs $\mathcal{F}$, the \textit{Tur\'an number} $\mathrm{ex}(n,\mathcal{F})$ is the maximum number of $r$-edges possible in an $n$-vertex $r$-graph that contains no member of $\mathcal{F}$ as a subhypergraph. When $r = 2$, the function $\mathrm{ex}(n,F)$ is well-studied and relatively well-understood. Given a set of $r$-graphs $\mathcal{F}$, we define the \textit{Tur\'an density} of $\mathcal{F}$ to be \[ \pi(\mathcal{F}) := \underset{n \rightarrow \infty}{\lim} \frac{\mathrm{ex}(n,\mathcal{F})}{\binom{n}{r}}.\] The Erd\H{o}s-Stone Theorem~\cite{ErSt} as pointed out by Erd\H{o}s-Simonovits~\cite{ErSi} determines the Tur\'an density of every $2$-graph as a function of its chromatic number. \begin{theorem}[Erd\H{o}s-Stone]\label{ess} Let $F$ be a $2$-graph with $\chi(F) = k$. Then $\pi(F) = 1 - \frac{1}{k-1}. $ \end{theorem} Note that an extension of Theorem~\ref{ess} also claims that for every family $\mathcal{F}$ of $2$-graphs, $\pi(\mathcal{F})$ is equal to the minimum of $1-1/(\chi(F)-1)$ over $F \in \mathcal{F}$. While Theorem~\ref{ess} does not give us perfect information about Tur\'an numbers (in particular, for bipartite graphs $F$, it only demonstrates that $\mathrm{ex}(n,F) = o(n^2)$), it yields a good ``approximate'' understanding of Tur\'an numbers by fully describing Tur\'an densities. For $r \geq 3$, we do not have an analogue to Theorem~\ref{ess}, and much less is known about Tur\'an densities. Even $\pi(\mathcal{F})$ could be smaller than $\min\{\pi(F) : F \in \mathcal{F}\}$, as observed by Balogh~\cite{jozsituran}. Not only do we lack a general theory, but the Tur\'an densities of many small $r$-graphs remain unknown, despite great effort. Famously, the Tur\'an density of the tetrahedron $K_4^3$ is still undetermined. The difficulty of determining Tur\'an densities for hypergraphs has motivated the study of various other hypergraph extremal functions, typically maximizing some variant of the minimum degree. In particular, given an $r$-graph $H$, the \textit{co-degree} of a set $S \in \binom{V(H)}{r-1}$ is the number of $r$-edges containing $S$, and the {\it minimum co-degree} of $H$, denoted by $\delta_{r-1}(H)$, is the smallest co-degree realized by an $(r-1)$-set contained in $V(H)$. The \textit{co-degree Tur\'an function} of a family of $r$-graphs $\mathcal{F}$, denoted by $\mathrm{coex}(n,\mathcal{F})$, is the largest possible minimum co-degree of an $n$-vertex $r$-graph containing no member of $\mathcal{F}$ as a subhypergraph. Mubayi and Zhao \cite{Mubayicode} showed that the \textit{co-degree density} \[\gamma(\mathcal{F}) := \underset{n \rightarrow \infty}{\lim} \frac{\mathrm{coex}(n,\mathcal{F})}{n}\] exists for every family of $r$-graphs $\mathcal{F}$, and studied the general behavior of $\mathrm{coex}(n,\mathcal{F})$. Note that for every family of $2$-graphs $\mathcal{F}$ we have $\gamma(\mathcal{F}) = \pi(\mathcal{F})$; however, for $r \geq 3$, co-degree Tur\'an problems are not equivalent to Tur\'an problems, and co-degree density does not behave in the manner suggested by Theorem~\ref{ess}. We first define the notion of a \textit{jump} in density. \begin{definition} Fix $r \geq 2$. Suppose $\varphi$ is a function that maps families of $r$-graphs to $[0,1]$. We say that $\alpha \in [0, 1)$ is a $\varphi$-{\it jump} if there exists $\delta \in (0, 1 - \alpha)$ such that for no family $\mathcal{F}$ of $r$-graphs, $\varphi(\mathcal{F}) \in (\alpha, \alpha + \delta)$. \end{definition} While Theorem~\ref{ess} shows that co-degree density (and Tur\'an density) jumps everywhere when $r = 2$, Mubayi and Zhao showed that co-degree density does not jump when $r \geq 3$. \begin{theorem}[Mubayi-Zhao \cite{Mubayicode}]\label{mzjumps} For $r \geq 3$, no $\alpha \in [0,1)$ is a $\gamma$-jump. \end{theorem} This co-degree phenomenon was further investigated in \cite{piga2023smallcodegree,ding2023vanishingcodegree}. We remark that Theorem~\ref{mzjumps} suggests a substantial difference in behavior between Tur\'an and co-degree Tur\'an problems for hypergraphs. We know for every $r \geq 3$ that every $\alpha \in [0, r!/r^r)$ is a $\pi$-jump. On the other hand, $\pi$ is also known to \textit{not} jump in infinitely many places for $r \geq 3$; see~\cite{hg-no-jump, do-jump}. The \textit{minimum positive co-degree} of an $r$-graph $H$ is the largest integer $k$ such that, whenever $S \in \binom{V(H)}{r-1}$ is contained in some $r$-edge of $H$, then $S$ is contained in at least $k$ $r$-edges of $H$. The edgeless $r$-graph is defined to have positive co-degree zero. We denote the minimum positive co-degree of $H$ by $\delta_{r-1}^+(H)$. We define the \textit{positive co-degree Tur\'an number}, denoted by $\mathrm{co^+ex}(n,\mathcal{F})$, to be the largest possible minimum positive co-degree of an $n$-vertex $r$-graph containing no member of $\mathcal{F}$ as a subhypergraph. Balogh, Lemons, and Palmer~\cite{BLP} introduced the minimum positive co-degree as an alternative notion of minimum degree in $r$-graphs. Since then this parameter has already been studied from several angles. The concept of $\mathrm{co^+ex}(n,\mathcal{F})$ was recently introduced by Halfpap, Lemons, and Palmer~\cite{halfpap2024positive}. The investigation of minimum positive co-degree as an extremal parameter is partially motivated by the admissibility of constructions that mimic the extremal graphs for classical questions. For example, given an $r$-graph $H$, the \textit{t-blow-up} $H[t]$ of $H$ is the $r$-graph obtained by replacing each vertex $v_i \in V(H)$ with a class $V_i$ of $t$ vertices, where a set of $r$-vertices spans an $r$-edge if and only if they belong to $r$ distinct classes of $H[t]$ which correspond to an $r$-edge in $H$. In classical Tur\'an theory, graph blow-ups yield extremal or nearly extremal constructions for all non-bipartite forbidden graphs. Blow-ups also occur as extremal examples for other types of thresholds---for instance, one of the constructions demonstrating the tightness of Dirac's Theorem is a slightly unbalanced blow-up of an edge (i.e., a complete bipartite graph). For $r \geq 3$ and every $r$-graph $H$ a sufficiently large blow-up of $H$ has minimum co-degree $0$, which means that even after adding $o(n^3)$ hyperedges, blow-ups will not provide extremal constructions for minimum co-degree density problems. However, $H[t]$ inherits the \textit{positive} co-degree properties of $H$. Thus, blow-ups (as well as other constructions with co-degree $0$ sets, such as $r$-graphs containing large strongly independent sets or multiple components) are potential extremal examples for positive co-degree problems. Previous results suggest that extremal constructions for positive co-degree problems in fact \textit{do} often look analogous to classical extremal constructions. See \cite{halfpap2024positive} for extremal constructions avoiding some small $3$-graphs, and \cite{halfpapmagnanspanning} on positive co-degree analogs of Dirac's Theorem, for which the extremal constructions also naturally generalize the graph extremal examples (and have minimum co-degree 0). Due to the expanded range of potential constructions, $\mathrm{co^+ex}(n,\mathcal{F})$ and $\mathrm{coex}(n,\mathcal{F})$ are generally not equal, and they appear to behave differently. Define the \textit{positive co-degree density} of a family of $r$-graphs $\mathcal{F}$ as the limit \[\gamma^+(\mathcal{F}) := \underset{n \rightarrow \infty}{\lim} \frac{\mathrm{co^+ex}(n,\mathcal{F})}{n}. \] The existence of $\gamma^+(F)$ was established by Halfpap, Lemons, and Palmer~\cite{halfpap2024positive} via a constructive argument, which can be generalized to finite families $\mathcal{F}$. Pikhurko~\cite{pikhurko2023limitpositiveelldegreeturan} gave a probabilistic argument establishing that $\gamma^+(\mathcal{F})$ exists for all families $\mathcal{F}$. \begin{prop}[Halfpap-Lemons-Palmer~\cite{halfpap2024positive}]\label{first jump} Fix $r \geq 2$ and let $\mathcal{F}$ be a family of $r$-graphs. Then \[\gamma^+(\mathcal{F}) \in \{0\} \cup \left[\frac{1}{r}, 1\right].\] \end{prop} In other words, every $\alpha \in [0,1/r)$ is a $\gamma^+$-jump. Proposition~\ref{first jump} describes behavior similar to that of the classical Tur\'an density. Every $r$-graph $F$ that is contained in some blow-up of an $r$-edge can be shown to be ``degenerate'', having $\mathrm{co^+ex}(n,F) = o(n)$, so $\gamma^+(F) = 0$. On the other hand, if $F$ is not contained in any blow-up of an $r$-edge, then $\gamma^+(F) \geq \frac{1}{r}$, since the balanced $n$-vertex blow-up of an $r$-edge has minimum positive co-degree approximately $\frac{n}{r}$. An $r$-graph $F$ is \emph{$k$-partite} if there is a partition of $V(F)$ into $k$ classes such that each edge intersects each part at most once. Although Proposition~\ref{first jump} suggests that $\gamma$ and $\gamma^+$ exhibit fundamentally different beha\-viors, it is not clear what behavior to expect from $\gamma^+$. Currently, we know the exact value $\gamma^+(F)$ only for very few $3$-graphs $F$. Halfpap, Lemons, and Palmer~\cite{halfpap2024positive} determined the values of $\gamma^+(F)$ for many small $3$-graphs $F$, and bounded $\gamma^+(F)$ in some other instances. Various authors have reported improvements on several of these initial bounds, with the current best known values summarized in Table~\ref{previous bounds}. For comparison, the best-known bounds on $\pi$ and $\gamma$ for these $3$-graphs are also provided. Graphs not defined in Table~\ref{fig-3graphs} are defined by their edge sets as \begin{align*} K_4^{3-} &= \{123, 124, 134\}, \quad F_5 = \{123, 124, 345\},\quad \mathbb{F} = \{ 123, 345, 156, 246, 147, 257, 367 \},\\ C_\ell &= \{123, 234, 345,\ldots, (\ell-2)(\ell-1)\ell, (\ell-1)\ell 1, \ell 12\}, \quad C_\ell^- = C_\ell - \{\ell 12\}. \end{align*} See~\cite{halfpap2024positive,l2norm} for more details about $3$-graphs in Table~\ref{previous bounds}. Note that the $3$-graph denoted here as $K_4^{3-}$ is often called $K_4^{-}$ in the literature; we adopt the notation $K_4^{3-}$ to distinguish this $3$-graph from the 2-graph obtained by deleting an edge from $K_4$, which we denote by $K_4^-$. \begin{table} \begin{center} \begin{tabular}{|l||c|c|c|c|c|c|} \hline $F$ & $\leq \pi(F) $ & $\pi(F) \leq $ & $\leq \gamma(F) $ & $\gamma(F) \leq $ & $\leq \gamma^+(F) $ & $\gamma^+(F) \leq $ \\ \hline \hline {$K_4^{3-}$} & 2/7 \cite{K43-extremalfrankl} & 0.28689 \cite{flagmatic} & $1/4$ \cite{codegreeconj} & $1/4$ \cite{FalgasK4-} & $ 1/3 $ \cite{halfpap2024positive}& $ 1/3 $ \cite{halfpap2024positive}\\ {$F_5$} & 2/9 \cite{Bollobascancellative} & 2/9 \cite{F5Frankl} & 0 \cite{l2norm} & 0 \cite{l2norm} & $1/3$ \cite{halfpap2024positive}& $1/3$ \cite{halfpap2024positive}\\ {$F_{3,2}$} & 4/9 \cite{F32Mubayi} & 4/9 \cite{F32furedi} & 1/3 \cite{codF32falgas} & 1/3 \cite{codF32falgas} & $1/2$ \cite{halfpap2024positive} & $1/2$ \cite{halfpap2024positive}\\ {$\mathbb{F}$} & 3/4 \cite{VeraSos} & 3/4 \cite{FanoFuredi} & 1/2 \cite{MubayiFano} & 1/2 \cite{MubayiFano} & 2/3 \cite{halfpap2024positive} & 2/3 \cite{halfpap2024positive}\\ {$K_4^3$} &5/9 \cite{MR177847} & 0.5615 \cite{BaberTuran} & 1/2 \cite{MR1829685} & 0.529 \cite{l2norm} & $1/2$ \cite{halfpap2024positive} & 0.543 \cite{volec} \\ {$F_{3,3}$} & 3/4 \cite{F32Mubayi} & 3/4 \cite{F32Mubayi} & 1/2 \cite{l2norm} & 0.604 \cite{l2norm} & 3/5 \cite{halfpap2024positive}& 0.616 \\ {$C_{5}$} & $2\sqrt{3} - 3$ \cite{F32Mubayi} & 0.46829 \cite{flagmatic} & 1/3 \cite{l2norm} & 0.3993 \cite{l2norm} & $1/2$ \cite{halfpap2024positive} & 1/2 \cite{Wu} \\ {$C_{7}$} &$2\sqrt{3} - 3$ \cite{F32Mubayi} & 0.464186 & 1/3 \cite{l2norm} & 0.371 & 1/2 \cite{halfpap2025} & 1/2 \cite{halfpap2025} \\ {$C_{5}^-$} &1/4 \cite{F32Mubayi} & 1/4\cite{lidicky2024c5-} & 0 \cite{l2norm} & 0 \cite{piga} & 1/3 \cite{halfpap2024positive} & 1/3 \cite{Wu} \\ {$J_4$} & $1/2$ \cite{DaisyBollobas} & $0.50409$ \cite{flagmatic} & $1/4$ \cite{l2norm} & $0.473$ \cite{l2norm} & $4/7$ \cite{halfpap2024positive} & 4/7 \\ {$F_{4,2}$} & $4/9$ & $0.4933328$ & $1/3$ & $0.4185 $ & $3/5$ & $3/5$ \\ \hline \end{tabular} \captionof{table}{Best-known density bounds for $\pi, \gamma$, and $\gamma^+$.}\label{previous bounds} \end{center} \end{table} Kamčev, Letzter, and Pokrovskiy~\cite{Kamcev2024} proved that the Turán density of longer tight cycles $C_\ell$ is $2\sqrt{3} - 3$, when $\ell$ is not multiple of three and sufficiently large (when $\ell $ is divisible by three, then $C_\ell$ is $3$-partite, hence its Tur\'an density is $0$). Similarly, Balogh and Luo~\cite{balogh2024turandensitylongtight} proved for $\ell$ sufficiently large and not divisible by $3$ that the Tur\'an density of $C_\ell^-$ is $1/4$. Recently, this was proved for every $\ell\ge 5$ by Lidick\'y, Mattes, and Pfender~\cite{lidicky2024c5-}. That the co-degree density is $1/3$ for tight cycles of length at least $10$ and not divisible by $3$ was proved by Piga, Sanhueza-Matamala, and Schacht~\cite{piga2024codegreeturandensity3uniform} and Ma~\cite{ma2024codegreeturandensity3uniform}. On the other hand, the fact that the co-degree density of $C_\ell^-$ is $0$ is due to Piga, Sales, and Sch\"ulke~\cite{piga2023smallcodegree}. Given $r \geq 2$, we call $\alpha \in [0,1]$ an \textit{achievable value} (for $\gamma^+$) if there exists a family $\mathcal{F}$ of $r$-graphs such that $\gamma^+(\mathcal{F}) = \alpha$. Before our article, the only known achievable values of $\gamma^+$ for $r=3$ were $0,1/3, 1/2,$ and $2/3$. Our goal is to understand the positive co-degree density by demonstrating additional $\gamma^+$-jumps for every $r$, as well as by expanding the known list of achievable values of $\gamma^+$ for $r =3$. Our paper is organized as follows. In Section~\ref{new results}, we summarize our main results. In Section~\ref{prelims}, we state some additional definitions and lemmas which will be used in our proofs. In Section~\ref{proofs}, we demonstrate further $\gamma^+$-jumps for every $r$, and for $r =3$ establish an infinite set of achievable values for $\gamma^+$. In Section~\ref{J4}, we use flag algebras to exactly determine $\gamma^+$ for two additional $3$-graphs, hence adding two values to the list of achievable values of $\gamma^+$. Finally, in Section~\ref{conclusion}, we have some concluding remarks and list a wide variety of open problems in the area. \section{New results}\label{new results} Our first main theorem extends the range of jumps described in Proposition~\ref{first jump}. We will need the following definition. An \emph{$r$-triangle}, denoted by $T^r$, is an $r$-graph with $r+1$ vertices and three $r$-edges. Notice that $T^r$ can be obtained from the $2$-graph triangle $T^2$ by adding $r-2$ vertices and including them to each of the three edges. Such hypergraphs are sometimes called daises in the literature. The $T^2$ part of a $T^r$ is called the \emph{base} of the $T^r$. Notice that $T^3$ is the same as $K_4^{3-}$. \begin{restatable}{theorem}{secondjump}\label{general jumps} Let $\mathcal{F}$ be a family of $r$-graphs for $r \geq 2$. Then \[ \gamma^+(\mathcal{F}) \in \left\{0, \frac{1}{r}\right\} \cup \left[\frac{2}{2r-1}, 1\right]. \] Thus, every $\alpha \in[0, \frac{2}{2r-1})$ is a $\gamma^+$-jump. Moreover, $\gamma^+(\mathcal{F}) = 0$ if and only if some member of $\mathcal{F}$ is $r$-partite, and $\gamma^+(\mathcal{F}) = \frac{1}{r}$ if and only if no member of $\mathcal{F}$ is $r$-partite, but some member of $\mathcal{F}$ is contained in a blow-up of some $T^r$. \end{restatable} For $r=3$, we also provide an infinite set of achievable values for $\gamma^+$, based on forbidden families involving the following $3$-graphs. For $k \geq 3$, let $J_k$ be the $(k+1)$-vertex $3$-graph, on vertex set $[k+1]$, with $3$-edges of the form $1ij$ for every $i , j \in \{2, \dots, k+1\}$. Let $K_4^3$ denote the complete $4$-vertex $3$-graph, and let $F_{3,2}$ denote the $5$-vertex $3$-graph, on vertex set $\{1,2,3,4,5\}$, with edge set $\{123, 124, 125, 345\}$. See Table~\ref{fig-3graphs} for an illustration of these and other relevant $3$-graphs. \begin{figure} \begin{center} \begin{tabular}{ | l|| c | Sc | Sc | } \hline $K_4^3$ & \vc{\begin{tikzpicture} \outercycle{5}{4} \drawhyperedge{0}{4} \drawhypervertex{0}{0} \drawhypervertex{1}{0} \drawhypervertex{2}{0} \drawhyperedge{1}{4} \drawhypervertex{0}{1} \drawhypervertex{1}{1} \drawhypervertex{3}{1} \drawhyperedge{2}{4} \drawhypervertex{0}{2} \drawhypervertex{2}{2} \drawhypervertex{3}{2} \drawhyperedge{3}{4} \drawhypervertex{1}{3} \drawhypervertex{2}{3} \drawhypervertex{3}{3} \end{tikzpicture} } & \vc{ \begin{tikzpicture}[scale=0.55] \foreach \i in {1,2,3,4}{ \draw (90*\i-45:1.5) coordinate(\i); } \foreach \r in {0-45,90-45,180-45,270-45}{ \begin{scope}[rotate=\r] \draw[hyperedge] (0:1.5) to[out=140,in=275,looseness=1.2] (90:1.5) to[out=265,in=40,looseness=1.2] (180:1.5) to[out=35,in=145,looseness=1.2] (0:1.5) ; \end{scope} } \draw (1) node[vtx,label=right:{\tiny 1}]{} (2) node[vtx,label=left:{\tiny 2}]{} (3) node[vtx,label=left:{\tiny 3}]{} (4) node[vtx,label=right:{\tiny 4}]{} ; \end{tikzpicture} } \\ \hline $F_{3,2}$ & \vc{\begin{tikzpicture}\outercycle{6}{5} \drawhyperedge{0}{5} \drawhypervertex{0}{0} \drawhypervertex{1}{0} \drawhypervertex{2}{0} \drawhyperedge{1}{5} \drawhypervertex{0}{1} \drawhypervertex{3}{1} \drawhypervertex{4}{1} \drawhyperedge{2}{5} \drawhypervertex{1}{2} \drawhypervertex{3}{2} \drawhypervertex{4}{2} \drawhyperedge{3}{5} \drawhypervertex{2}{3} \drawhypervertex{3}{3} \drawhypervertex{4}{3} \end{tikzpicture} } & \vc{ \begin{tikzpicture} \draw (0,-0.7) coordinate(1) node[vtx,label=left:{\tiny $1$}](a){} (0,0) coordinate(2) node[vtx,label=below:{\tiny $2$}](b){} (0,0.7) coordinate(3) node[vtx,label=left:{\tiny $3$}](c){} (1,-0.5) coordinate(4) node[vtx,label=right:{\tiny $5$}](d){} (1,0.5) coordinate(5) node[vtx,label=right:{\tiny $4$}](d){} ; \draw[hyperedge] (1) to[bend left] (2) to[bend left] (3) to[bend right] (1); \draw[hyperedge] (1) to[out=0,in=190] (4) to[out=190,in=250] (5) to[out=250,in=0] (1); \draw[hyperedge] (2) to[out=0,in=140] (4) to[out=140,in=210,looseness=1.5] (5) to[out=210,in=0] (2); \draw[hyperedge] (3) to[out=0,in=120,looseness=1.2] (4) to[out=120,in=170,looseness=1.2] (5) to[out=170,in=0,looseness=1.2] (3); \end{tikzpicture} } \\ \hline $F_{4,2}$ & \vc{ \begin{tikzpicture}[flag_pic] \drawhyperedge{0}{6} \drawhypervertex{0}{0} \drawhypervertex{1}{0} \drawhypervertex{2}{0} \drawhyperedge{1}{6} \drawhypervertex{0}{1} \drawhypervertex{1}{1} \drawhypervertex{3}{1} \drawhyperedge{2}{6} \drawhypervertex{0}{2} \drawhypervertex{2}{2} \drawhypervertex{3}{2} \drawhyperedge{3}{6} \drawhypervertex{0}{3} \drawhypervertex{4}{3} \drawhypervertex{5}{3} \drawhyperedge{4}{6} \drawhypervertex{1}{4} \drawhypervertex{4}{4} \drawhypervertex{5}{4} \drawhyperedge{5}{6} \drawhypervertex{2}{5} \drawhypervertex{4}{5} \drawhypervertex{5}{5} \drawhyperedge{6}{6} \drawhypervertex{3}{6} \drawhypervertex{4}{6} \drawhypervertex{5}{6} \end{tikzpicture} } & \vc{ \begin{tikzpicture} \draw (0,-0.7) coordinate(1) node[vtx,label=below:{\tiny $1$}](a){} (0,-0.2) coordinate(2) node[vtx,label=below:{\tiny $2$}](b){} (0, 0.2) coordinate(6) node[vtx,label=above:{\tiny $3$}](b){} (0,0.7) coordinate(3) node[vtx,label=left:{\tiny $4$}](c){} (1,-0.5) coordinate(4) node[vtx,label=right:{\tiny $6$}](d){} (1,0.5) coordinate(5) node[vtx,label=right:{\tiny $5$}](d){} ; \draw[hyperedge] (1) to[bend left] (2) to[bend left] (3) to[bend right] (1); \draw[hyperedge] (1) to[bend left=40] (6) to[bend left=40] (3) to[bend right=40] (1); \draw[hyperedge] (1) to[bend left=70,looseness=2] (2) to[bend left=70,looseness=2] (6) to[bend right=70,looseness=2] (1); \draw[hyperedge] (1) to[out=0,in=190] (4) to[out=190,in=250] (5) to[out=250,in=0] (1); \draw[hyperedge] (2) to[out=0,in=140] (4) to[out=140,in=210,looseness=1.5] (5) to[out=210,in=0] (2); \draw[hyperedge] (6) to[out=0,in=140] (4) to[out=140,in=210,looseness=1.5] (5) to[out=210,in=0] (6); \draw[hyperedge] (3) to[out=0,in=120,looseness=1.2] (4) to[out=120,in=170,looseness=1.2] (5) to[out=170,in=0,looseness=1.2] (3); \end{tikzpicture} } \\\hline $F_{3,3}$ & \vc{\begin{tikzpicture}\outercycle{7}{6} \drawhyperedge{0}{6} \drawhypervertex{0}{0} \drawhypervertex{1}{0} \drawhypervertex{2}{0} \drawhyperedge{1}{6} \drawhypervertex{0}{1} \drawhypervertex{3}{1} \drawhypervertex{4}{1} \drawhyperedge{2}{6} \drawhypervertex{0}{2} \drawhypervertex{3}{2} \drawhypervertex{5}{2} \drawhyperedge{3}{6} \drawhypervertex{0}{3} \drawhypervertex{4}{3} \drawhypervertex{5}{3} \drawhyperedge{4}{6} \drawhypervertex{1}{4} \drawhypervertex{3}{4} \drawhypervertex{4}{4} \drawhyperedge{5}{6} \drawhypervertex{1}{5} \drawhypervertex{3}{5} \drawhypervertex{5}{5} \drawhyperedge{6}{6} \drawhypervertex{1}{6} \drawhypervertex{4}{6} \drawhypervertex{5}{6} \drawhyperedge{7}{6} \drawhypervertex{2}{7} \drawhypervertex{3}{7} \drawhypervertex{4}{7} \drawhyperedge{8}{6} \drawhypervertex{2}{8} \drawhypervertex{3}{8} \drawhypervertex{5}{8} \drawhyperedge{9}{6} \drawhypervertex{2}{9} \drawhypervertex{4}{9} \drawhypervertex{5}{9} \end{tikzpicture} } & \vc{ \begin{tikzpicture} \draw (0,-0.7) coordinate(1) node[vtx,label=left:{\tiny $3$}](a){} (0,0) coordinate(2) node[vtx,label=left:{\tiny $2$}](b){} (0,0.7) coordinate(3) node[vtx,label=left:{\tiny $1$}](c){} (1,-0.7) coordinate(4) node[vtx,label=right:{\tiny $6$}](d){} (1,0.0) coordinate(5) node[vtx,label=right:{\tiny $5$}](d){} (1,0.7) coordinate(6) node[vtx,label=right:{\tiny $4$}](d){} ; \draw[hyperedge] (1) to[bend left] (2) to[bend left] (3) to[bend right] (1); \draw[hyperedge] (1) to[out=0,in=190] (4) to[out=190,in=250] (5) to[out=250,in=0] (1); \draw[hyperedge] (2) to[out=0,in=140] (4) to[out=140,in=210,looseness=1.5] (5) to[out=210,in=0] (2); \draw[hyperedge] (3) to[out=-60,in=120,looseness=1.2] (4) to[out=120,in=170,looseness=1.2] (5) to[out=170,in=-60,looseness=1.2] (3); \draw[hyperedge] (1) to[out=60,in=190] (5) to[out=190,in=250] (6) to[out=250,in=60] (1); \draw[hyperedge] (2) to[out=0,in=140] (5) to[out=140,in=210,looseness=1.5] (6) to[out=210,in=0] (2); \draw[hyperedge] (3) to[out=0,in=120,looseness=1.2] (5) to[out=120,in=170,looseness=1.2] (6) to[out=170,in=0,looseness=1.2] (3); \draw[hyperedge] (1) to[out=10,in=170,looseness=1.5] (4) to[out=170,in=190] (6) to[out=190,in=10] (1); \draw[hyperedge] (2) to[out=0,in=140] (4) to[out=140,in=210,looseness=1.5] (6) to[out=210,in=0] (2); \draw[hyperedge] (3) to[out=-10,in=120,looseness=1.2] (4) to[out=120,in=190,looseness=1.2] (6) to[out=190,in=-10,looseness=1.2] (3); \end{tikzpicture} } \\ \hline $J_4$ & \vc{\begin{tikzpicture}\outercycle{6}{5} \drawhyperedge{0}{5} \drawhypervertex{0}{0} \drawhypervertex{1}{0} \drawhypervertex{2}{0} \drawhyperedge{1}{5} \drawhypervertex{0}{1} \drawhypervertex{1}{1} \drawhypervertex{3}{1} \drawhyperedge{2}{5} \drawhypervertex{0}{2} \drawhypervertex{1}{2} \drawhypervertex{4}{2} \drawhyperedge{3}{5} \drawhypervertex{0}{3} \drawhypervertex{2}{3} \drawhypervertex{3}{3} \drawhyperedge{4}{5} \drawhypervertex{0}{4} \drawhypervertex{2}{4} \drawhypervertex{4}{4} \drawhyperedge{5}{5} \drawhypervertex{0}{5} \drawhypervertex{3}{5} \drawhypervertex{4}{5} \end{tikzpicture} } & \vc{ \begin{tikzpicture}[scale=0.75] \clip (-2.5,-1.1) rectangle (1.5,1.1); \draw (-2,0) coordinate(1) node[vtx,label=left:{\tiny $1$}](b){} (45:1) coordinate(2) node[vtx,label=right:{\tiny $2$}](c){} (135:1) coordinate(3) node[vtx,label=left:{\tiny $3$}](d){} (225:1) coordinate(4) node[vtx,label=left:{\tiny $4$}](d){} (315:1) coordinate(5) node[vtx,label=right:{\tiny $5$}](d){} ; \draw[hyperedge] (1) to[out=70,in=140,looseness=0.9] (2) to[out=140,in=60,looseness=0.8] (3) to[out=120,in=70,looseness=0.8] (1); \draw[hyperedge] (1) to[out=0,in=250,looseness=1.3] (3) to[out=250,in=110] (4) to[out=110,in=0,looseness=1.3] (1); \draw[hyperedge] (1) to[out=-70,in=240,looseness=0.8] (4) to[out=300,in=220,looseness=0.8] (5) to[out=220,in=-70,looseness=0.9] (1); \draw[hyperedge] (1) to[out=0,in=110,looseness=0.7] (5) to[out=110,in=250,looseness=1.1] (2) to[out=250,in=0,looseness=0.7] (1); \draw[hyperedge] (1) to[out=-25,in=205,looseness=0.8] (2) to[out=205,in=110,looseness=0.8] (4) to[out=110,in=-25,looseness=0.9] (1); \draw[hyperedge] (1) to[out=25,in=250,looseness=0.9] (3) to[out=250,in=155,looseness=0.8] (5) to[out=155,in=25,looseness=0.8] (1); \draw[line width = 0.5pt] (3) to[out=250,in=155,looseness=0.8] (5) (2) to[out=205,in=110,looseness=0.8] (4) (5) to[out=110,in=250,looseness=1.1] (2) (4) to[out=300,in=220,looseness=0.8] (5) (3) to[out=250,in=110] (4) (2) to[out=140,in=60,looseness=0.8] (3) ; \end{tikzpicture} } \\ \hline $J_5$ & \vc{\begin{tikzpicture}\outercycle{7}{6} \drawhyperedge{0}{6} \drawhypervertex{0}{0} \drawhypervertex{1}{0} \drawhypervertex{2}{0} \drawhyperedge{1}{6} \drawhypervertex{0}{1} \drawhypervertex{1}{1} \drawhypervertex{3}{1} \drawhyperedge{2}{6} \drawhypervertex{0}{2} \drawhypervertex{1}{2} \drawhypervertex{4}{2} \drawhyperedge{3}{6} \drawhypervertex{0}{3} \drawhypervertex{1}{3} \drawhypervertex{5}{3} \drawhyperedge{4}{6} \drawhypervertex{0}{4} \drawhypervertex{2}{4} \drawhypervertex{3}{4} \drawhyperedge{5}{6} \drawhypervertex{0}{5} \drawhypervertex{2}{5} \drawhypervertex{4}{5} \drawhyperedge{6}{6} \drawhypervertex{0}{6} \drawhypervertex{2}{6} \drawhypervertex{5}{6} \drawhyperedge{7}{6} \drawhypervertex{0}{7} \drawhypervertex{3}{7} \drawhypervertex{4}{7} \drawhyperedge{8}{6} \drawhypervertex{0}{8} \drawhypervertex{3}{8} \drawhypervertex{5}{8} \drawhyperedge{9}{6} \drawhypervertex{0}{9} \drawhypervertex{4}{9} \drawhypervertex{5}{9} \end{tikzpicture} } & \vc{ \begin{tikzpicture} \draw (-2,0) coordinate(1) node[vtx,label=left:{\tiny $1$}](1){} \foreach \x in {2,...,6} { (-144+72*\x:1) coordinate(\x) node[vtx](v\x){} } ; \draw[hyperedge,opacity=0.2] (1)--(2)--(3)--(1); \draw[hyperedge,opacity=0.2] (1)--(2)--(4)--(1); \draw[hyperedge,opacity=0.2] (1)--(2)--(5)--(1); \draw[hyperedge,opacity=0.2] (1)--(2)--(6)--(1); \draw[hyperedge,opacity=0.2] (1)--(3)--(4)--(1); \draw[hyperedge,opacity=0.2] (1)--(3)--(5)--(1); \draw[hyperedge,opacity=0.2] (1)--(3)--(6)--(1); \draw[hyperedge,opacity=0.2] (1)--(4)--(5)--(1); \draw[hyperedge,opacity=0.2] (1)--(4)--(6)--(1); \draw[hyperedge,opacity=0.2] (1)--(5)--(6)--(1); \draw [line width = 0.5pt] (2)--(3)--(4)--(5)--(6)--(2)--(4)--(6)--(3)--(5)--(2) ; \draw (-2,0) coordinate(1) node[vtx](1){} \foreach \x in {2,...,6} { (-144+72*\x:1) coordinate(\x) node[vtx](v\x){} }; \end{tikzpicture} }\\ \hline \end{tabular} \end{center} \captionof{table}{Small 3-uniform hypergraphs.}\label{fig-3graphs} \end{figure} \begin{restatable}{theorem}{densities}\label{densities} For every $k \geq 4$, \[ \gamma^+(\{K_4^3, F_{3,2}, J_k\}) = \frac{k-2}{2k-3}.\] \end{restatable} We also investigate whether the densities exhibited by Theorem~\ref{densities} can be achieved by forbidding a single $3$-graph. For $k = 4$, we have $\gamma^+(\{K_4^3, F_{3,2}, J_4\}) = \frac{2}{5}$. Let $F_1$ be a 7-vertex 3-graph with edges $\{125, 135, 235, 126, 146, 246, 347 \}$, see Figure~\ref{F1F2}. We show that $F_1$ has positive co-degree density $\frac{2}{5}$. \begin{restatable}{theorem}{singlegraph}\label{single graph} $\gamma^+(F_1) = \frac{2}{5}$. \end{restatable} Finally, we utilize flag algebras to determine the positive co-degree densities of two graphs, which exhibit new achievable values of $\gamma^+$ larger than $\frac{1}{2}$. We determine $\gamma^+(J_4)$, and show that the (asymptotic) extremal construction is the blow-up of the complement of the Fano plane. Notice that $J_4$ is a 3-daisy; see~\cite{ellis2024daisies} for recent progress on Tur\'an densities of $r$-daisies. \begin{restatable}{theorem}{singlegraphJfour}\label{J4} $\gamma^+(J_4) = \frac{4}{7}$. \end{restatable} We introduce another $3$-graph, which will have a different new density. Let $F_{4,2}$ be the $6$-vertex $3$-graph with edges $\{123, 124, 134, 156, 256, 356, 456 \}$, depicted in Table~\ref{fig-3graphs}. Note that in $F_{4,2}$ the common neighborhood of 5 and 6 is $\{1,2,3,4\}$, and $\{1,2,3,4\}$ spans a $K_4^{3-}$. We determine $\gamma^+(F_{4,2})$, and show that its (asymptotic) extremal construction is the balanced blow-up of $K_5^3$. \begin{restatable}{theorem}{Fdensity}\label{F42density} $\gamma^+(F_{4,2}) = \frac{3}{5}$. \end{restatable} We also include some non-tight results obtained using flag algebras. \begin{restatable}{theorem}{FlagDensity}\label{thm:FlagDensity} The following bounds hold. \[ \pi(F_{4,2}) \leq 0.4933327, \quad \gamma(F_{4,2}) \leq 0.4185, \quad \gamma^+(F_{3,3}) \leq 0.616. \] \end{restatable} The lower bounds in Table~\ref{previous bounds} for $\pi(F_{4,2})$ and $\gamma(F_{4,2})$ come from the lower bound constructions for $F_{3,2}$. \section{Preliminaries}\label{prelims} We begin by stating some results related to supersaturation and a hypergraph removal lemma. The hypergraph removal lemma states that an $r$-graph containing only few copies of some subhypergraph $F$ can be made $F$-free by the deletion of only few $r$-edges. For a discussion of removal lemmas, including the below formulation, see~\cite{ConlonFox}. \begin{lemma}\label{hypergraph removal} Fix $\alpha > 0$ and let $F$ be an $r$-graph. There exists $\delta > 0$ such that if $H$ is an $n$-vertex $r$-graph containing at most $\delta n^{|V(F)|}$ copies of $F$, then there exists $E' \subset E(H)$ such that $|E'| \leq \alpha n^r$ and $H - E'$ is $F$-free. \end{lemma} Although an $r$-graph with $o(n^{|V(F)|})$ copies of $F$ can be made $F$-free by deleting $o(n^r)$ $r$-edges, it is not obvious that the deletion would change the minimum positive co-degree by only $o(n)$. The following ``clean-up'' lemma due to Halfpap, Lemons, and Palmer~\cite{halfpap2024positive} allows us to apply the hypergraph removal lemma to minimum positive co-degree problems. Roughly, this lemma guarantees that any positive co-degree drop arising from the deletion of a small set of $r$-edges can be mitigated by the deletion of another set of $r$-edges. \begin{lemma}[Halfpap-Lemons-Palmer~\cite{halfpap2024positive}]\label{positive codegree cleanup} Let $H$ be an $n$-vertex $r$-graph and fix $0< \varepsilon <1$ small enough that $(r+1)! \varepsilon^{1/{2^{r-1}}}n^r < |E(H)|$. Let $H_1$ be a subhypergraph of $H$ obtained by the deletion of at most $\varepsilon n^r$ $r$-edges. Then $H_1$ has a subhypergraph $H_2$ with $\delta_{r-1}^+(H_2) \geq \delta_{r-1}^+(H) - 2^r r! \varepsilon^{1/{2^{r-1}}}n$. \end{lemma} In practice it is not difficult to fulfill the condition in Lemma~\ref{positive codegree cleanup} that $(r+1)! \varepsilon^{1/{2^{r-1}}}n^r < |E(H)|$, since $\delta_{r-1}^+(H)$ can be used to give a lower bound on $|E(H)|$. \begin{lemma}[Halfpap-Lemons-Palmer~\cite{halfpap2024positive}]\label{edge approx} Fix $c>0$ and suppose ${H}$ is an $r$-graph with $\delta_{r-1}^+(H) \geq cn$. Then, for $n$ large enough, $|E( {H})| \geq \frac{1}{2}\frac{c^r}{r!} n^r$. \end{lemma} Thus, for an $n$-vertex $r$-graph $H$ with $n$ sufficiently large and $\delta_{r-1}^+(H) \geq cn$, we can choose $\varepsilon$ in Lemma~\ref{positive codegree cleanup} as a function of $c$ and $r$ alone. Lemmas~\ref{hypergraph removal} and \ref{positive codegree cleanup} can be used to prove supersaturation and related properties for minimum positive co-degree problems. In particular, we have the following basic formulation. \begin{theorem}[Halfpap-Lemons-Palmer~\cite{halfpap2024positive}]\label{supersaturation} Fix $\varepsilon > 0$ and let $F$ be an $r$-graph. Then there exists $\delta > 0$ such that, if $H$ is an $n$-vertex $r$-graph with \[\delta_{r-1}^+(H) >\mathrm{co^+ex}(F) + \varepsilon n,\] then $H$ contains at least $\delta n^{|V(F)|}$ copies of $F$. \end{theorem} By a standard argument (see, e.g., \cite{Keevashsurvey}), if $\delta > 0$ and $t \in \mathbb{N}$ are fixed and $n$ is sufficiently large, then every $n$-vertex $r$-graph $H$ containing $\delta n^{|V(F)|}$ copies of $F$ must contain \acopyof $F[t]$. Thus, as an immediate consequence of Theorem~\ref{supersaturation}, we have blow-up invariance for $\mathrm{co^+ex}(n,F)$. \begin{cor}[Halfpap-Lemons-Palmer~\cite{halfpap2024positive}]\label{blow up invariance} Let $F$ be an $r$-graph and $t$ a positive integer. Then $$\mathrm{co^+ex}(n,F) \leq \mathrm{co^+ex}(n,F[t]) \leq \mathrm{co^+ex}(n,F) + o(n).$$ \end{cor} We remark that Corollary~\ref{blow up invariance} is useful because it implies that if $H$ and $F$ are $r$-graphs such that $F$ is contained in $H[t]$ for some $t$, then $\gamma^+(F) \leq \gamma^+(H)$. For example, when paired with the facts that a single $3$-edge $e$ has $\gamma^+(e) = 0$ and $\gamma^+(C_5) = \frac{1}{2}$, and with an appropriate lower bound construction, Corollary~\ref{blow up invariance} implies that $\gamma^+(C_{\ell}) = 0$ if $\ell \equiv 0 \,\,(\textrm{mod } 3)$ and $\gamma^+(C_{\ell}) = \frac{1}{2}$ if $\ell \not\equiv 0 \,\,(\textrm{mod } 3)$ and $C_{\ell}$ is contained in a blow-up of $C_5$. In fact, this resolves the positive co-degree density of every tight cycle except for $C_4 = K_4^3$ and $C_7$. Halfpap~\cite{halfpap2025} proved $\gamma^+(C_7)=1/2$. In Section~\ref{proofs}, we will apply essentially the same idea, finding one construction whose blow-up contains another and then relating their positive co-degree densities. However, for our purposes, a somewhat different formulation from the above statements will be desirable. The proof ideas for Theorem~\ref{supersaturation} and Corollary~\ref{blow up invariance} are used to derive the following lemma. \begin{lemma}\label{family removal} Let $F$ be a fixed $r$-graph on $f$ vertices and $\mathcal{F} = \{F_1, F_2, \dots ,F_k\}$ a finite family of $r$-graphs such that $F_i[f]$ contains $F$ for every $i \in [k]$. For every $d \in [0,1)$, if $\gamma^+(F) > d$, then for some $\beta > 0$ and $n$ sufficiently large there exists an $n$-vertex, $\{F\} \cup \mathcal{F}$-free $r$-graph $H$ with $\delta_{r-1}^+(H) > \left( d + \beta \right)n$. \end{lemma} \begin{proof} Given $d$ as stated, choose $\beta > 0$ so that $\gamma^+(F) \geq d + 3\beta$, and $\alpha > 0$ such that $2^rr!\alpha^{1/{2^{r-1}}} < \beta$ and $ (r+1)!\alpha^{1/{2^{r-1}}} n^r < \frac{1}{2}\frac{(2\beta)^r}{r!} n^r$. For each $i \in [k]$, take $\delta_i > 0$ as guaranteed by Lemma~\ref{hypergraph removal} such that any hypergraph $H$ containing fewer than $\delta_i n^{|V(F_i)|}$ copies of $F_i$ can be made $F_i$-free by the deletion of at most $\frac{\alpha}{k} |E(H)|$ $r$-edges. Choose $N \in \mathbb{N}$ sufficiently large such that for all $n \geq N$, we have: \begin{itemize}[itemsep=1pt, parsep=0pt] \item $\mathrm{co^+ex}(n,F) \geq (d + 2\beta)n$; \item for each $i \in [k]$, if $H$ is an $n$-vertex graph containing $\delta_i n^{|V(F_i)|}$ copies of $F_i$, then $H$ contains \acopyof $F_i[f]$. \end{itemize} Fix $n \geq N$ and let $H$ be an $n$-vertex $r$-graph with $\delta_{r-1}^+(H) = \mathrm{co^+ex}(n,F)$. Then $H$ contains fewer than $\delta_i n^{|V(F_i)|}$ copies of $F_i$ for every $i \in [k]$, and thus can be made $\mathcal{F}$-free by deletion of at most $\alpha |E(H)|$ edges by repeated application of Lemma~\ref{hypergraph removal}. Since we have \[(r+1)!\alpha^{1/{2^{r-1}}} n^r < \frac{1}{2}\frac{(2\beta)^r}{r!} n^r \leq |E(H)|\] by Lemma~\ref{edge approx} and the definition of $\alpha$, we can now apply Lemma~\ref{positive codegree cleanup} to delete an additional set of $r$-edges, resulting in an $\{F\} \cup \mathcal{F}$-free, $n$-vertex $r$-graph $H'$ with \[\delta_{r-1}^+(H') \geq \delta_{r-1}^+(H) - 2^rr!\alpha^{1/{2^{r-1}}}n >\delta_{r-1}^+(H) - \beta n \geq (d + \beta)n. \qedhere \] \end{proof} Lemma~\ref{family removal} is an important tool in finding positive co-degree densities as it essentially allows us to expand our list of forbidden configurations. We conclude this section with some relevant definitions and notation. We often consider $4$-vertex cliques with one edge removed; these may be $2$-uniform or $3$-uniform. To avoid ambiguity, we denote by $K_4^-$ the $2$-graph on $4$ vertices and $5$ edges, and by $K_4^{3-}$ the $3$-graph on $4$ vertices and three $3$-edges. Some of the hypergraphs we consider can be naturally described as arising from lower-uniformity hypergraphs. We define the following operation, which increases the uniformity of a hypergraph by one. Given an $r$-graph $H$, the \textit{suspension} $\widehat{H}$ is the $(r+1)$-graph with vertex set consisting of $V(H)$ and one new vertex $v$, and $(r+1)$-edges \[ E(\widehat{H}) = \{ e \cup \{v\} : e \in E(H) \}. \] We call $V(H)$ the \textit{$r$-graph vertices} and $v$ the \textit{spike vertex}. Notice that the $(r+1)$-triangle $T^{r+1}$ is a suspension of the $r$-triangle $T^{r}$. Let $H$ be an $r$-graph. For a subset $X=\{x_1,\ldots,x_{r-1}\}$ of size $r-1$ of vertices of $H$, denote by $N(X)$ the set of all vertices $v$ such that $X\cup\{v\} \in E(H)$. We use $d(X) := |N(X)|$, which is the co-degree of $X$. To simplify notation we use $N(x_1,\ldots,x_{r-1}) := N(X)$ and $d(x_1,\ldots,x_{r-1}) := d(X)$. We call $N(X)$ the \textit{neighborhood} of $X$. In a $3$-graph $H$, the \textit{link graph} $L(v)$ of a vertex $v \in V(H)$ is the auxiliary $2$-graph on $V(H) - \{v\}$ where $xy$ is an edge if and only if $vxy$ is a $3$-edge of $H$. For 3-graphs $G$ and $H$ the \emph{density} of $G$ in $H$, denoted by $d(G,H)$, is the number of subgraphs of $H$ isomorphic to $G$ divided by $\binom{|V(H)|}{|V(G)|}$. Notice that the density is always in $[0,1]$. \section{Jumps and positive co-degree densities below $\frac{1}{2}$}\label{proofs} \begin{proof}[Proof of Theorem~\ref{general jumps}] Let $\mathcal{F}$ be a family of $r$-graphs. By Proposition~\ref{first jump}, $\gamma^+(\mathcal{F}) \in \{0\} \cup [\frac{1}{r}, 1]$, so it is sufficient to show that there is no family $\mathcal{F}$ with $\gamma^+(\mathcal{F}) \in (\frac{1}{r}, \frac{2}{2r-1})$. First assume that forbidding $\mathcal{F}$ implies that some blow-up $T^r[t]$ of $T^r$ is also forbidden (recall, $T^r$ denotes the triangle with three edges on $r+1$ vertices). Corollary~\ref{blow up invariance} gives $\gamma^+(T^r[t]) = \gamma^+(T^r)$. As \[ \gamma^+(\mathcal{F}) \leq \gamma^+(T^r[t]) = \gamma^+(T^r), \] it is sufficient to show that $\gamma^+(T^r) \leq \frac1r$. Suppose that $H$ is an $n$-vertex $r$-graph with $\delta_{r-1}^+(H) > \frac{n}{r}$, and let $v_1v_2\dots v_r$ be an $r$-edge of $H$. Consider the $r$ vertex sets, each of size $r-1$, contained in the $r$-edge $v_1v_2\dots v_r$. Each of them is a set with positive co-degree, hence each has neighborhood of size greater than $\frac{n}{r}$. Since there are $r$ such sets, there must be a vertex, say $v_{r+1}$, which is contained in at least two such neighborhoods. Relabeling if needed, we may assume that \[ v_{r+1} \in N(v_1, \dots, v_{r-2}, v_{r-1}) \cap N(v_1, \dots, v_{r-2}, v_{r}). \] The three $r$-edges $v_1v_2\dots v_{r-2}v_{r-1}v_r$, $v_1v_2\dots v_{r-2}v_{r-1}v_{r+1}$, and $v_1v_2\dots v_{r-2}v_{r}v_{r+1}$ form \acopyof $T^r$. Thus, $\frac{1}{r} \geq \gamma^+\left(T^r\right) \geq \gamma^+\left(\mathcal{F}\right)$. This implies that blow-ups of $T^r$ are $\mathcal{F}$-free. Now assume that forbidding $\mathcal{F}$ does not exclude any blow-up of $T^r$. Consider the following $n$-vertex blow-up of $T^r$. The three vertices corresponding to the base triangle $T^2$ are blown up to classes of size $\frac{n}{2r-1}$. All other vertices are blown up to classes of size $\frac{2n}{2r-1}$. In total, we have $3$ classes of size $\frac{n}{2r-1}$ and $r-2$ classes of size $\frac{2n}{2r -1}$, for a total of $n$ vertices, as desired. A set of $r-1$ vertices whose intersection with any class has at least two vertices will have co-degree $0$. A set of $r-1$ vertices which intersects all three classes of size $\frac{n}{2r-1}$ will also have co-degree $0$. All other sets of size $r-1$ have neighborhood of size exactly $\frac{2n}{2r-1}$, corresponding either to two classes of size $\frac{n}{2r-1}$ or one class of size $\frac{2n}{2r-1}$. This construction implies $\gamma^+(\mathcal{F}) \geq \frac{2}{2r-1}$. Now, we can characterize families of $r$-graphs $\mathcal{F}$ with $\gamma^+(\mathcal{F}) \in \{0, \frac{1}{r}\}$. Corollary~\ref{blow up invariance} establishes that if $F \in \mathcal{F}$ is $r$-partite, then $\gamma^+(F) = 0$, so $\gamma^+(\mathcal{F}) = 0$ as well. If no $F \in \mathcal{F}$ is $r$-partite, then any blow-up of an $r$-edge is $\mathcal{F}$-free, so $\gamma^+(\mathcal{F}) \geq \frac{1}{r}$. Thus, $\gamma^+(\mathcal{F}) = 0$ if and only if some $F \in \mathcal{F}$ is $r$-partite. Similarly, if an $F \in \mathcal{F}$ is contained in a blow-up of $T^r$, then \[ \gamma^+(\mathcal{F}) \leq \gamma^+(F) \leq \gamma^+(T^r) \leq \frac{1}{r}.\] Thus, if some $F \in \mathcal{F}$ is contained in some $T^r$ blow-up but no member of $\mathcal{F}$ is $r$-partite, then we have $\gamma^+(F) = \frac{1}{r}$. On the other hand, if no member of $\mathcal{F}$ is contained in any blow-up of $T^r$, then the above-described blow-up of $T^r$ establishes that $\gamma^+(\mathcal{F}) \geq \frac{2}{2r - 1}$. \end{proof} \begin{remark} The characterization in Theorem~\ref{general jumps} implies the positive co-degree densities for a variety of natural $3$-graphs. It is straightforward to verify that $C_{\ell}^-$ (the $\ell$-vertex (tight) cycle with one edge deleted) is contained in a sufficiently large blow-up of $K_4^{3-}$. Moreover, $C_{\ell}^-$ is $3$-partite if and only if $\ell \equiv 0 \pmod 3$. Thus, $\gamma^+(C_{\ell}^-) = 0$ when $\ell \equiv 0 \pmod 3$, and $\gamma^+(C_{\ell}^-) = \frac{1}{3}$ otherwise. This generalizes the result of Wu~\cite{Wu} that $\gamma^+(C_5^-) = \frac{1}{3}$. \end{remark} The next natural question is whether Theorem \ref{general jumps} is best possible. That is, does there exist some family $\mathcal{F}$ of $r$-graphs with $\gamma^+(\mathcal{F}) = \frac{2}{2r - 1}$? We answer this question in the affirmative when $r = 3$. Furthermore, we show that an infinite number of densities in the interval $[\frac{2}{5}, \frac{1}{2}]$ are achievable when $r = 3$. We begin by exhibiting a family $\mathcal{F}$ with $\gamma^+(\mathcal{F}) = \frac{2}{5}$. We first define $F_{3,2}^{++1}$ and $F_{3,2}^{++2}$, two superhypergraphs of $F_{3,2}$. Each is created by adding two edges to $F_{3,2}$. Let $F_{3,2}$ have vertex set $\{a,b,c,d,e\}$ and edge set $\{abc, abd, abe, cde\}$. Then we create $F_{3,2}^{++1}$ by adding $acd$ and $ace$, and we create $F_{3,2}^{++2}$ by adding $acd$ and $bce$, see Figure~\ref{F32}. \begin{figure}[h] \centering \begin{tikzpicture} \draw (0,0) node[vtx](a1){}; \draw (1,0) node[vtx](b1){}; \draw (-1,2) node[vtx](c1){}; \draw (0.5, 2) node[vtx](d1){}; \draw (2, 2) node[vtx](e1){}; \draw[fill=pink,opacity=0.5] (a1)to[bend left = 15] (b1)to[bend left=5] (c1)to[bend left=5] cycle ; \draw[fill=pink,opacity=0.5] (a1)to[bend left = 15] (b1)to[bend left=5] (d1)to[bend left=5] cycle ; \draw[fill=pink,opacity=0.5] (a1)to[bend left = 15] (b1)to[bend left=5] (e1)to[bend left=5] cycle ; \draw[fill=pink,opacity=0.5] (c1)to[bend left = 10] (d1)to[bend left=10] (e1)to[bend right=15] cycle ; \draw (0,0) node[vtx,label=below:$a$](a1){}; \draw (1,0) node[vtx,label=below:$b$](b1){}; \draw (-1,2) node[vtx,label=above:$d$](c1){}; \draw (0.5, 2) node[vtx,label=above:$c$](d1){}; \draw (2, 2) node[vtx,label=above:$e$](e1){}; \draw (4,0) node[vtx](a2){}; \draw (5,0) node[vtx](b2){}; \draw (3,2) node[vtx](c2){}; \draw (4.5, 2) node[vtx](d2){}; \draw (6, 2) node[vtx](e2){}; \draw[fill=pink,opacity=0.5] (a2)to[bend left = 15] (b2)to[bend left=5] (c2)to[bend left=5] cycle ; \draw[fill=pink,opacity=0.5] (a2)to[bend left = 15] (b2)to[bend left=5] (d2)to[bend left=5] cycle ; \draw[fill=pink,opacity=0.5] (a2)to[bend left = 15] (b2)to[bend left=5] (e2)to[bend left=5] cycle ; \draw[fill=pink,opacity=0.5] (c2)to[bend left = 10] (d2)to[bend left=10] (e2)to[bend right=15] cycle ; \draw[fill=blue,opacity=0.5] (c2)to[bend right = 15] (d2)to[bend right=15] (a2)to[bend right=15] cycle ; \draw[fill=blue,opacity=0.5] (d2)to[bend right = 15] (e2)to[bend right=12] (a2)to[bend right=12] cycle ; \draw (4,0) node[vtx, label=below:$a$](a2){}; \draw (5,0) node[vtx, label=below:$b$](b2){}; \draw (3,2) node[vtx, label=above:$d$](c2){}; \draw (4.5, 2) node[vtx, label=above:$c$](d2){}; \draw (6, 2) node[vtx, label=above:$e$](e2){}; \draw (8,0) node[vtx](a3){}; \draw (9,0) node[vtx](b3){}; \draw (7,2) node[vtx](c3){}; \draw (8.5, 2) node[vtx](d3){}; \draw (10, 2) node[vtx](e3){}; \draw[fill=pink,opacity=0.5] (a3)to[bend left = 15] (b3)to[bend left=5] (c3)to[bend left=5] cycle ; \draw[fill=pink,opacity=0.5] (a3)to[bend left = 15] (b3)to[bend left=5] (d3)to[bend left=5] cycle ; \draw[fill=pink,opacity=0.5] (a3)to[bend left = 15] (b3)to[bend left=5] (e3)to[bend left=5] cycle ; \draw[fill=pink,opacity=0.5] (c3)to[bend left = 10] (d3)to[bend left=10] (e3)to[bend right=15] cycle ; \draw[fill=blue,opacity=0.5] (c3)to[bend right = 15] (d3)to[bend right=15] (a3)to[bend right=15] cycle ; \draw[fill=blue,opacity=0.5] (d3)to[bend right = 15] (e3)to[bend right=15] (b3)to[bend right=15] cycle ; \draw (8,0) node[vtx, label=below:$a$](a3){}; \draw (9,0) node[vtx, label=below:$b$](b3){}; \draw (7,2) node[vtx, label=above:$d$](c3){}; \draw (8.5, 2) node[vtx, label=above:$c$](d3){}; \draw (10, 2) node[vtx, label=above:$e$](e3){}; \end{tikzpicture} \caption{$F_{3,2}$ and two superhypergraphs, $F_{3,2}^{++1}$ and $F_{3,2}^{++2}$.} \label{F32} \end{figure} \begin{prop}\label{2/5 family} For $\mathcal{F} = \{K_4^3, F_{3,2}^{++1}, F_{3,2}^{++2}, J_4\}$ and $\mathcal{F'} = \{K_4^3, F_{3,2}, J_4\}$ we have \[ \gamma^+(\mathcal{F}) = \gamma^+(\mathcal{F'}) = \frac{2}{5}. \] \end{prop} \begin{proof} Since $F_{3,2}$ is a subgraph of both $F_{3,2}^{++1}, F_{3,2}^{++2}$, hence $\gamma^+(\mathcal{F}) \geq \gamma^+(\mathcal{F'})$. As noted in the proof of Theorem~\ref{general jumps}, an appropriately balanced $n$-vertex blow-up of $T^3=K_4^{3-}$ has minimum positive co-degree $\frac{2n}{5} + O(1)$. Any blow-up of $K_4^{3-}$ is $J_4$-free and $F_{3,2}$-free since $K_4^{3-}$ is $4$-partite and neither of $F_{3,2}$ and $J_4$ are. Since blow-ups $K_4^{3-}$ are also $K_4^{3}$-free, we have $\frac{2}{5} \leq \gamma^+(\mathcal{F'}) \leq \gamma^+(\mathcal{F})$. Next we show $\gamma^+(\mathcal{F}) \leq \frac{2}{5}$, which completes the proof. Fix an $\varepsilon > 0$ and suppose that $H$ is an $n$-vertex $3$-graph with $\delta_2^+ (H) \geq \left( \frac{2}{5} + \varepsilon \right)n$ for sufficiently large $n$. Since $\gamma^+(K_4^{3-})=\frac13$, $H$ contains \acopyof $K_4^{3-}$, say with vertex set $\{a,b,c,d\}$ and edge set $\{abc, abd, acd\}$. Consider the following five positive co-degree pairs: $ab, ac, ad, bc,$ and $bd$. Since $\delta_2^+(H) \geq \left(\frac{2}{5} + \varepsilon \right)n,$ there exists some vertex $e$ that is in the neighborhood of at least three of these pairs. Note that $c$ and $d$ are symmetric; up to this symmetry, we have six cases based on which three of these five pairs form an edge with $e$. \begin{enumerate}[label=(\arabic*),itemsep=0.9pt, parsep=0pt] \item If $abe, ace, ade \in E(H)$ then $abc, abd, acd, abe, ace, ade$ form a $J_4$. \item If $abe, ace, bce \in E(H)$ then $abc, abe, ace, bce$ form a $K_4^3$. \item If $abe, ace, bde \in E(H)$ then $acb$, $acd$, $ace$, $bde$ form an $F_{3,2}$. Moreover, $abd$ and $abe$ are $3$-edges, so $\{a,b,c,d,e\}$ spans an \acopyof $F_{3,2}^{++1}$. \item If $ace, ade, bce \in E(H)$ then $adb, adc, ade, bce$ form an $F_{3,2}$. Moreover, $abc$ and $ace$ are $3$-edges, so $\{a,b,c,d,e\}$ spans an \acopyof $F_{3,2}^{++1}$. \item If $abe, bce, bde \in E(H)$ then $bea, bec, bed, acd$ form an $F_{3,2}$. Moreover, $bac$ and $bad$ are $3$-edges, so $\{a,b,c,d,e\}$ spans an \acopyof $F_{3,2}^{++1}$. \item If $ace, bce, bde \in E(H)$ then $acb, acd, ace, bde$ form an $F_{3,2}$. Moreover, $abd$ and $cbe$ are $3$-edges, so $\{a,b,c,d,e\}$ spans an \acopyof $F_{3,2}^{++2}$. \end{enumerate} This implies $\gamma^+(\mathcal{F}) \leq \frac{2}{5}$. \end{proof} We use Proposition~\ref{2/5 family} as the base of an inductive argument, showing that an infinite number of positive co-degree densities in $[\frac{2}{5}, \frac{1}{2}]$ are achievable by families of $3$-graphs. \begin{proof}[Proof of Theorem~\ref{densities}] For a lower bound, observe that any blow-up of $J_{k-1}$ is $\{K_4^3, F_{3,2}, J_k\}$-free, and that an appropriately balanced $n$-vertex blow-up of $J_{k-1}$ (with class sizes $\frac{k-2}{2k-3},$ $ \frac{1}{2k-3},\ldots, \frac{1}{2k-3}$) has minimum positive co-degree $\left(\frac{k-2}{2k-3}\right)n + O(1)$. Thus, $\gamma^+(\{K_4^3, F_{3,2}, J_k\}) \geq \frac{k-2}{2k-3}$ for all $k \geq 4$. To show that this lower bound is best-possible, we use induction on $k$. Proposition~\ref{2/5 family} yields the statement for $k=4$. We assume the statement holds for $k \geq 4$, and prove it for $k + 1$. Suppose that $H$ is an $n$-vertex $3$-graph with minimum positive co-degree $\delta_2^+(H) > \frac{k-1}{2k-1} n$ where is $n$ sufficiently large. If $H$ contains one of $K_4^3$ or $F_{3,2}$, then we are done, so suppose not. Then by the inductive hypothesis, $\gamma^+(\{K_4^3, F_{3,2}, J_k\}) = \frac{k-2}{2k-3} < \frac{k-1}{2k -1}$, so $H$ contains a $J_k$, say $J$ (here we use that $n$ is large enough). Let $V(J) = \{v_1, \dots, v_{k+1}\}$, where $v_1$ is the universal vertex of $J$ (i.e., $E(J) = \{v_1v_iv_j : 2 \leq i < j \leq k+1\}$). Define \[ S = \{(v_1, v_i) : 2 \leq i \leq k+1 \} \cup \{(v_i,v_{i+1}): 2 \leq i \leq k\}.\] Observe that $|S| = 2k - 1$ and every vertex pair in $S$ has positive co-degree in $H$. Since $\delta_2^+(H) > \frac{k-1}{2k-1} n$, there exists a vertex $u \in V(H)$ which is in the neighborhood of at least $k$ pairs $(v_i,v_j) \in S$. Note that $u$ may be an element of $V(J)$. However, if $(v_i,v_j) \in S$ is a pair such that $u \in N(v_i,v_j)$, then $u \not\in \{v_i,v_j\}$. If $u \in N(v_1, v_i)$ for every $i \in \{2, \dots, k+1\}$ then $u \not\in V(J)$, and $V(J) \cup \{u\}$ spans \acopyof $J_{k+1}$, with universal vertex $v_1$. Hence, we may assume that there is some $i \geq 2$ for which $u \in N(v_i, v_{i+1})$. Our goal in this case is to find $F_{3,2}$. Observe that there must be a $j$ such that $u \in N(v_1, v_j)$. If there is such a $j \not \in \{i, i+1\}$, then $u \not\in \{v_1,v_i, v_{i+1}, v_j\}$, and $\{v_1, v_i, v_{i+1}, v_j, u\}$ will span an $F_{3,2}$, using $3$-edges $v_1v_jv_i, v_1v_jv_{i+1}, v_1v_ju, v_iv_{i+1}u$. Hence, we may assume that every such $j$ is in $\{i, i+1\}$. In particular, $u$ is in at most two neighborhoods of the form $N(v_1,v_j)$. Since $k \geq 4$ and $u$ is in the neighborhood of at least $k$ pairs from $S$, we have $u \in N(v_{\ell},v_{\ell + 1})$ for some $\ell \neq i$. Now, if $u \in N(v_1, v_i) $ and $u \in N(v_1, v_{i+1})$, then $\{v_1,v_i,v_{i+1},u\}$ spans a $K_4^3$. If not, then without loss of generality $u \in N(v_1, v_i) $ and $u$ is in the neighborhood of the three pairs $(v_i, v_{i+1}), (v_{\ell}, v_{\ell + 1}), (v_m, v_{m+1})$ for some $\ell,m$. One of these pairs, say $(v_{\ell}, v_{\ell + 1})$, must be disjoint from $(v_1, v_i)$, in which case we observe that $\{v_1,v_i, v_\ell, v_{\ell+1},u\}$ will span an $F_{3,2}$. \end{proof} Although infinitely many positive co-degree densities in $[\frac{2}{5}, \frac{1}{2}]$ can be achieved by forbidden families of $3$-graphs, it remains unclear whether all of these densities can be achieved by forbidding a single $3$-graph. Using Proposition~\ref{2/5 family}, we now show that $\frac{2}{5}$ is indeed achievable by a single $3$-graph. We begin by defining two new $3$-graphs $F_1$ and $F_2$ as follows. \begin{align*}V(F_1) & = \{a,b,c,d,e,f,g\}, \quad & E(F_1) & = \{abe, ace, bce, abf, adf, bdf, cdg\},\\ V(F_2) & = \{a,b,c,d,e,g\}, \quad &E(F_2) & = \{abe, ace, bce, ade, bde, cdg\}. \end{align*} We depict $F_1$ and $F_2$ in Figure~\ref{F1F2}. We remark that each of $F_1,F_2$ can be viewed as a partial identification of two $K_4^{3-}$ copies, along with an extra $3$-edge (using $g$) which ensures that $c$ and $d$ have positive co-degree. \begin{figure}[h] \centering \begin{tikzpicture} \draw (5,0.4) node[vtx,label=below:$c$](x1){}; \draw (7,0) node[vtx,label=below:$a$](z1){}; \draw (7.5,0.8) node[vtx,label=below:$b$](w1){}; \draw (9, 0.4) node[vtx,label=below:$d$](y1){}; \draw (7, 2) node[vtx,label=above:$e$](f){}; \draw (7, -1) node[vtx,label=below:$g$](g1){}; \draw[fill=pink,opacity=0.5] (x1) to[bend left=15] (w1)to[bend left=15] (f) to[bend left=2] cycle ; \draw[fill=pink,opacity=0.5] (x1) to[bend left=15] (z1)to[bend left=15] (f) to[bend left=15] cycle ; \draw[fill=pink,opacity=1] (z1) to[bend left=1] (w1)to[bend left=1] (f) to[bend right=1] cycle ; \draw[fill=pink,opacity=0.5] (y1) to[bend right=15] (w1)to[bend left = 25] (f) to[bend right=2] cycle ; \draw[fill=pink,opacity=1] (z1) to[bend left=1] (w1)to[bend left=1] (f) to[bend right=1] cycle ; \draw (7.5,0.8) node[vtx,label = below:$b$](w1){}; \draw[fill=pink,opacity=0.5] (y1) to[bend right=15] (z1)to[bend right=35] (f) to[bend right=25] cycle ; \draw[fill=pink,opacity=0.5] (x1) to[bend right=45] (y1) to[bend left=25] (g1) to[bend left=25] cycle ; \draw (5,0.4) node[vtx,label=below:$c$](x1){}; \draw (7,0) node[vtx,label=below:$a$](z1){}; \draw (9, 0.4) node[vtx,label=below:$d$](y1){}; \draw (7, 2) node[vtx,label=above:$e$](f){}; \draw (0,0.4) node[vtx,label=below:$c$](x2){}; \draw (2,0) node[vtx,label=below:$a$](z2){}; \draw (2.5,0.8) node[vtx,label=below:$b$](w2){}; \draw (4, 0.2) node[vtx,label=below:$d$](y2){}; \draw (1, 2) node[vtx,label=above:$e$](f){}; \draw (3, 2) node[vtx,label=above:$f$](c){}; \draw (2, -1) node[vtx,label=below:$g$](g2){}; \draw[fill=pink,opacity=0.5] (x2) to[bend left=15] (w2)to[bend left=15] (f) to[bend left=2] cycle ; \draw[fill=pink,opacity=0.5] (x2) to[bend left=15] (z2)to[bend left=15] (f) to[bend left=15] cycle ; \draw[fill=pink,opacity=1] (z2) to[bend left=1] (w2)to[bend left=1] (f) to[bend right=1] cycle ; \draw[fill=pink,opacity=0.5] (y2) to[bend right=15] (w2)to[bend right = 15] (c) to[bend right=2] cycle ; \draw[fill=pink,opacity=1] (z2) to[bend right=1] (w2)to[bend left=1] (c) to[bend right=10] cycle ; \draw[fill=pink,opacity=0.5] (x2) to[bend right=45] (y2)to[bend left=25] (g2) to[bend left=25] cycle ; \draw (2.5,0.8) node[vtx,label=below:$b$](w2){}; \draw[fill=pink,opacity=0.5] (y2) to[bend right=15] (z2)to[bend right=35] (c) to[bend right=25] cycle ; \draw (0,0.4) node[vtx,label=below:$c$](x2){}; \draw (2,0) node[vtx,label=below:$a$](z2){}; \draw (4, 0.2) node[vtx,label=below:$d$](y2){}; \draw (1, 2) node[vtx,label=above:$e$](f){}; \draw (3, 2) node[vtx,label=above:$f$](c){}; \draw (7, -2.2) node{$F_2$}; \draw (2, -2.2) node{$F_1$}; \end{tikzpicture} \caption{$F_1$ and $F_2$.} \label{F1F2} \end{figure} Note that in both $F_1$ and $F_2$, the vertex $g$ has very little structural interaction with the other vertices. Suppose $H$ is isomorphic to the subhypergraph of $F_1$ (resp.\ $F_2$) induced on $V(F_1) \setminus \{g\}$ (resp.\ $V(F_2) \setminus \{g\}$). Then $H$ is guaranteed to extend to \acopyof $F_1$ (resp.\ $F_2$) if $d(c,d) > 4$. We will be working in $3$-graphs with minimum positive co-degree much larger than $4$, so to find copies of $F_1$ or $F_2$, it will suffice to find $F_1 \setminus \{g\}$ or $F_2 \setminus \{g\}$ and to demonstrate that the pair $\{c,d\}$ has positive co-degree. Note that $F_2$ is $5$-partite, so $\gamma^+(F_2) \geq \frac{1}{2}$. However, $F_1$ is $4$-partite, and is contained in a blow-up of $J_4$, but it is not contained in a blow-up of $J_3=K_4^{3-}$. Thus, $\gamma^+(F_1) \geq \frac{2}{5}$. We now prove that $\gamma^+(F_1) = \frac{2}{5}$. \begin{proof}[Proof of Theorem~\ref{single graph}] For a contradiction, assume that there exists $\varepsilon > 0$ such that $\gamma^+(F_1) \geq \frac{2}{5} + 3\varepsilon$. We begin with a claim that will allow us to expand our forbidden family. \begin{claim} The $3$-blow-up of each member of the following family \[\mathcal{F} = \{K_4^3, J_4, F_{3,2}^{++1}, F_2 \}\] contains $F_1$. \end{claim} \begin{proof} Observe that $F_2$ can be obtained from $F_1$ by identifying $e$ and $f$. Using the notation \begin{align*} V(K_4^3) &=\{1,2,3,4\}, &E&(K_4^3) \quad =\{123,124,134,234\},\\ V(J_4) &=\{1,2,3,4,5\}, &E&(J_4)\quad \ =\{123, 124, 125, 134, 135, 145\},\\ V(F_{3,2}^{++1})&=\{1,2,3,4,5\}, &E&(F_{3,2}^{++1})=\{123, 124, 125, 345, 134, 135\} \end{align*} the following maps prove the claim for $K_4^3$, $J_4$, and $F_{3,2}^{++1}$. $f_1: F_1\to K_4^3:\ a\to 1,\ b\to 2,\ c \to 3, \ d \to 4, \ e \to 4,\ f \to 3, \ g \to 1$. $f_2: F_1\to J_4: \ \ a\to 2, \ b\to 3,\ c \to 4,\ d \to 5, \ e \to 1, \ f \to 1, \ g \to 1$. $f_3: F_1\to F_{3,2}^{++1}: a\to 2,\ b\to 3,\ c \to 4, \ d \to 5,\ e \to 1, \ f \to 1, \ g \to 3$. \end{proof} Thus, by Lemma~\ref{family removal}, for $n$ large enough there exists a $3$-graph $H$ with $\delta_2^+(H) \geq \left(\frac{2}{5} + \varepsilon\right)n$ which is $\{F_1\} \cup \mathcal{F}$-free. By Proposition~\ref{2/5 family}, \[ \gamma^+(\{K_4^3, J_4, F_{3,2}^{++1}, F_{3,2}^{++2}\}) = \frac{2}{5}, \] so $H$ contains \acopyof $F_{3,2}^{++2}$, which we shall call $F$. Put \[V(F) = \{a,b,c,d,e\},\quad \text{ with } \quad E(F) = \{abc, abd, abe, cde, acd, bce\}.\] Observe that every pair of vertices in $V(F)$ has positive co-degree. Since $\delta_2^+(H) \geq \left( \frac{2}{5} + \varepsilon \right)n$, there exists some vertex $f$ that is in the neighborhood of at least five pairs from $V(F)$. Let $G_f$ be the link graph of vertex $f$ induced on vertex set $V(F)$ (i.e.\ $G_f = L(f)[V(F)]$). \begin{claim}\label{VFclaims} The following statements hold: \begin{enumerate}[label=(\roman*),itemsep=1pt, parsep=0pt] \item For every $xyz \in E(H)$, at most two of $xy, xz, yz$ are in $E(G_f)$. \item There is no isolated vertex in $G_f$. \item At most one of $ad, cd$ is in $E(G_f)$ and at most one of $be, ce$ is in $E(G_f)$. \item At most one of $cd, de$ is in $E(G_f)$ and at most one of $ce, de$ is in $E(G_f)$. \item At most one of $ab, ac$ is in $E(G_f)$ and at most one of $ab, bc$ is in $E(G_f)$. \item At most one of $ab, ad$ is in $E(G_f)$ and at most one of $ab, be$ is in $E(G_f)$. \item At most one of $ac, ad$ is in $E(G_f)$ and at most one of $bc, be$ is in $E(G_f)$. \item $G_f$ is triangle-free. \end{enumerate} \end{claim} \begin{proof} To prove each statement, we shall assume that the statement does not hold and use this assumption to find some forbidden structure in $H$. Notice that the two parts in each of the statements (iii)--(vii) are symmetric so it is sufficient to prove only the first part of each of those claims. Here the symmetry is coming from the fact that $a\to b,\ b\to a,\ c\to c,\ d\to e, \ e\to d$ is an automorphism of $F$. For (i), observe that if $xyz \in E(H)$ and $xy, xz, yz \in E(G_f)$, then $\{x,y,z,f\}$ spans a $K_4^3$ in $H$. For (ii), assume that $G_f$ contains a vertex of degree 0. Then the other four vertices in $V(F)$ span (at least) five edges of $G_f$, so $G_f$ contains a \acopyof $K_4^-$. As illustrated in Figure~\ref{k4-link}, a \acopyof $K_4^-$ in $G_f$ implies that $H$ contains $F_2$ if the appropriate pair of vertices ($x$ and $y$, in the figure) has positive co-degree. Since all pairs of vertices in $F$ have positive co-degree, \acopyof a $K_4^-$ in $G_f$ indeed implies that $H$ contains \acopyof $F_2$. \begin{figure}[h] \centering \begin{tikzpicture} \draw (0,0) node[vtx,label=below:$x$](x){}; \draw (2,0) node[vtx,label=below:$z$](z){}; \draw (0,2) node[vtx,label=above:$w$](w){}; \draw (2,2) node[vtx,label=above:$y$](y){}; \draw (x) to (z) (z) to (y) (z) to (w) (w) to (x) (w) to (y); \draw (1, -1) node{$G_f$}; \draw (5,0.4) node[vtx,label=below:$x$](x1){}; \draw (7,0) node[vtx,label=below:$z$](z1){}; \draw (7.5,0.8) node[vtx,label=below:$w$](w1){}; \draw (9, 0.4) node[vtx,label=below:$y$](y1){}; \draw (7, 2) node[vtx,label=above:$f$](f){}; \draw[fill=pink,opacity=0.5] (x1) to[bend left=15] (w1)to[bend left=15] (f) to[bend left=2] cycle ; \draw[fill=pink,opacity=0.5] (x1) to[bend left=15] (z1)to[bend left=15] (f) to[bend left=15] cycle ; \draw[fill=pink,opacity=1] (z1) to[bend left=1] (w1)to[bend left=1] (f) to[bend right=1] cycle ; \draw[fill=pink,opacity=0.5] (y1) to[bend right=15] (w1)to[bend left = 25] (f) to[bend right=2] cycle ; \draw[fill=pink,opacity=1] (z1) to[bend left=1] (w1)to[bend left=1] (f) to[bend right=1] cycle ; \draw (7.5,0.8) node[vtx,label = below:$w$](w1){}; \draw[fill=pink,opacity=0.5] (y1) to[bend right=15] (z1)to[bend right=35] (f) to[bend right=25] cycle ; \draw (5,0.4) node[vtx,label=below:$x$](x1){}; \draw (7,0) node[vtx,label=below:$z$](z1){}; \draw (9, 0.4) node[vtx,label=below:$y$](y1){}; \draw (7, 2) node[vtx,label=above:$f$](f){}; \draw (7,-1) node{$H$}; \end{tikzpicture} \caption{\acopyof $K_4^-$ in $G_f$ and the resulting structure in $H$.} \label{k4-link} \end{figure} For (iii), observe that if $ad, cd \in E(G_f)$, then $\{a,c,d,f\}$ spans \acopyof $K_4^{3-}$ with spike vertex $d$, and $\{a, b, c, e\}$ spans \acopyof $K_4^{3-}$ with spike vertex $b$, so $\{a,b,c,d,e, f\}$ spans \acopyof $F_1$ (since $d(e,f) > 0$ by (ii)). For (iv), if $cd, de \in E(G_f)$, then $\{c,d,e,f\}$ spans \acopyof $K_4^{3-}$ with spike vertex $d$, and $\{a,b,c,e\}$ spans \acopyof $K_4^{3-}$ with spike vertex $b$, so $\{a,b,c,d,e,f\}$ spans \acopyof $F_1$ (since $d(a,f) > 0$ by (ii)). For (v), if $ab, ac \in E(G_f)$, then $\{a,b,c,d\}$ and $\{a, b, c, f\}$ span copies of $K_4^{3-}$ with spike vertex $a$, so $\{a,b,c,d,f\}$ spans \acopyof $F_2$ (since $d(d,f) > 0$ by (ii)). For (vi), if $ab, ad \in E(G_f)$, then $\{a,b,c,d\}$ and $\{a, b, d, f\}$ span copies of $K_4^{3-}$ with spike vertex $a$, so $\{a,b,c,d,f\}$ spans \acopyof $F_2$ (since $d(c,f) > 0$ by (ii)). For (vii), if $ac, ad \in E(G_f)$, then $\{a,b,c,d\}$ and $\{a, c, d, f\}$ span copies of $K_4^{3-}$ with spike vertex $a$, so $\{a,b,c,d,f\}$ spans \acopyof $F_2$ (since $d(b,f) > 0$ by (ii)). Finally, we show (viii). From (i), we know that $\{c,d,e\}$ does not form a triangle in $G_f$, and from (v) and (vi) it follows that $ab$ is not contained in a triangle in $G_f$. Thus, any triangle in $G_f$ must include one edge spanned by $\{c,d,e\}$, and two edges with one vertex in $\{a,b\}$ and the other in $\{c,d,e\}$. By (i), $\{a,c,d\}$ does not span a triangle (nor does $\{b, c, e\}$). Up to symmetry, there are two other potential triangles: $\{a, d, e\}$, and $\{a,c,e\}$. Observe that if $\{a,d,e\}$ spans a triangle in $G_f$, then $\{a,d,e, f\}$ spans \acopyof $K_4^{3-}$ with spike vertex $f$, and $\{a, b, c, e\}$ spans \acopyof $K_4^{3-}$ with spike vertex $b$, so $\{a,b,c,d,e,f\}$ spans \acopyof $F_1$ (since $d(c,d) > 0$). Next, suppose $\{a,c,e\}$ spans a triangle in $G_f$. By (iii), (iv), (v), and (vii), none of $be, de, ab, ad$ are in $E(G_f)$. Thus, two of $bc, bd, cd$ are in $E(G_f)$. We shall show that this is impossible. Observe that if $bc \in E(G_f)$, then $\{a,b,c,f\}$ and $\{b,c,e,f\}$ each spans a $K_4^{3-}$ with spike vertex $c$, so $\{a,b,c,e,f\}$ spans \acopyof $F_2$ (since $d(a,e) > 0$). Moreover, if $cd \in E(G_f)$, then $\{a,c,d,f\}$ and $\{c,d,e,f\}$ span $K_4^{3-}$ with spike vertex $c$, so $\{a,c,d,e,f\}$ spans \acopyof $F_2$ (since $d(a,e) > 0$). Thus, $G_f$ is triangle-free. \end{proof} With Claim~\ref{VFclaims} established, we are ready to determine the structure of $G_f$. By (i), at most two out of five edges of $G_f$ are spanned by $\{c,d,e\}$, so one of $a,b$ has degree at least $2$. Without loss of generality, $d(a) \geq d(b)$ and $d(a) \geq 2$. \begin{observation} $ab \not\in E(G_f)$. \end{observation} \begin{proof} Suppose to the contrary that $ab \in E(G_f)$. By (v) and (vi), $ac$ and $ad$ are not in $E(G_f)$, so we must have $ae \in E(G_f)$. By (v) and (vi), we also have $bc, be \not\in E(G_f)$. Since $G_f$ has five edges, at least three of $bd, cd, ce, de$ are in $E(G_f)$. By (iv), at most one of $ce, de$ is in $E(G_f)$, so $bd, cd \in E(G_f)$. Also by (iv), if $cd \in E(G_f)$, then $de \not \in E(G_f)$, so $ce \in E(G_f)$. Since $ae \in E(G_f)$, there exists a vertex $x$ not in $V(F)$ that is a common neighbor of $a$ and $e$. The set of edges $\{adb,dce, fab, fbd, fdc, fce, aex\}$ forms $F_1$. See Figure~\ref{noab} for an illustration of $G_f$, $F$, and $F_1$. \end{proof} \begin{figure}[h] \centering \begin{tikzpicture} \draw (0,0) node[vtx,label=above:$b$](b){}; \draw (-2,0) node[vtx,label=above:$a$](a){}; \draw (-1,-3) node[vtx,label=below:$c$](c){}; \draw (1,-3) node[vtx,label=below:$e$](e){}; \draw (-3,-3) node[vtx,label=below:$d$](d){}; \draw[fill=pink,opacity=0.5] (a) to[bend right=15] (b)to[bend right=15] (c) to[bend right=15] cycle ; \draw[fill=pink,opacity=0.5] (a) to[bend right=15] (b)to[bend right=15] (d) to[bend right=15] cycle ; \draw[fill=pink,opacity=0.5] (a) to[bend right=15] (b)to[bend right=15] (e) to[bend right=15] cycle ; \draw[fill=pink,opacity=0.5] (d) to[bend left=15] (c)to[bend left=50] (a) to[bend right=15] cycle ; \draw[fill=pink,opacity=0.5] (e) to[bend right=15] (c)to[bend right=50] (b) to[bend left=15] cycle ; \draw[fill=pink,opacity=0.5] (c) to[bend left=15] (d)to[bend right=20] (e) to[bend left=15] cycle ; \draw (0,0) node[vtx,label=above:$b$](b){}; \draw (-2,0) node[vtx,label=above:$a$](a){}; \draw (-1,-3) node[vtx,label=below:$c$](c){}; \draw (1,-3) node[vtx,label=below:$e$](e){}; \draw (-3,-3) node[vtx,label=below:$d$](d){}; \draw[thick, blue] (a) to[bend right = 25] (b); \draw[thick, blue] (b) to[bend right = 20] (d); \draw[thick, blue] (a) to[bend left = 20] (e); \draw[thick, blue] (c) to[bend right = 10] (e); \draw[thick, blue] (d) to[bend right = 10] (c); \draw (-1, -4) node{$G_f$ and $F$}; \begin{scope}[yshift = 1cm] \draw (5,-2.6) node[vtx,label=below:$a$](x1){}; \draw (7,-3) node[vtx,label=below:$d$](z1){}; \draw (7.5,-2.2) node[vtx,label=below:$f$](w1){}; \draw (9, -2.8) node[vtx,label=below:$e$](y1){}; \draw (6, -1) node[vtx,label=above:$b$](f){}; \draw (8, -1) node[vtx,label=above:$c$](c){}; \draw (7, -4) node[vtx,label=below:$x$](g1){}; \draw[fill=pink,opacity=0.5] (x1) to[bend right=45] (y1) to[bend left=25] (g1) to[bend left=25] cycle ; \draw[fill=pink,opacity=0.5] (x1) to[bend left=15] (w1)to[bend left=15] (f) to[bend left=2] cycle ; \draw[fill=pink,opacity=0.5] (x1) to[bend left=15] (z1)to[bend left=15] (f) to[bend left=15] cycle ; \draw[fill=pink,opacity=1] (z1) to[bend left=1] (w1)to[bend left=1] (f) to[bend right=1] cycle ; \draw[fill=pink,opacity=0.5] (y1) to[bend right=15] (w1)to[bend right = 15] (c) to[bend right=2] cycle ; \draw[fill=pink,opacity=1] (z1) to[bend right=1] (w1)to[bend left=1] (c) to[bend right=10] cycle ; \draw (7.5,-2.2) node[vtx,label=below:$f$](w1){}; \draw[fill=pink,opacity=0.5] (y1) to[bend right=15] (z1)to[bend right=35] (c) to[bend right=25] cycle ; \draw (5,-2.6) node[vtx,label=below:$a$](x1){}; \draw (7,-3) node[vtx,label=below:$d$](z1){}; \draw (9, -2.8) node[vtx,label=below:$e$](y1){}; \draw (6, -1) node[vtx,label=above:$b$](f){}; \draw (8, -1) node[vtx,label=above:$c$](c){}; \end{scope} \draw (7,-4) node{$F_1$}; \end{tikzpicture} \caption{The configuration when $ab \in E(G_f)$, and the resulting copy of $F_1$.} \label{noab} \end{figure} Since $d(a) \geq 2$, two of $ac, ad, ae$ are in $E(G_f)$. By (vii), at most one of $ac, ad$ is in $E(G_f)$, so $E(G_f)$ contains $ae$ and exactly one of $ac, ad$. Suppose first that $ad \in E(G_f)$. By (iii), $cd\not\in E(G_f)$, and by (viii), $de \not\in E(G_f)$. Since $d(b) \leq d(a) = 2$, we must have $ce \in E(G_f)$. Now, by (iii), $be \not\in E(G_f)$, so we must have $bc, bd \in E(G_f)$. Since $ae \in E(G_f)$, there exists a vertex $x$ not in $V(F)$ that is a common neighbor of $a$ and $e$. The set of edges $\{adb,bce, fad, fbd, fbc, fce, aex\}$ forms $F_1$. See Figure~\ref{noad} for an illustration of $G_f$, $F$, and $F_1$. \begin{figure}[h] \centering \begin{tikzpicture} \draw (0,0) node[vtx,label=above:$b$](b){}; \draw (-2,0) node[vtx,label=above:$a$](a){}; \draw (-1,-3) node[vtx,label=below:$c$](c){}; \draw (1,-3) node[vtx,label=below:$e$](e){}; \draw (-3,-3) node[vtx,label=below:$d$](d){}; \draw[fill=pink,opacity=0.5] (a) to[bend right=15] (b)to[bend right=15] (c) to[bend right=15] cycle ; \draw[fill=pink,opacity=0.5] (a) to[bend right=15] (b)to[bend right=15] (d) to[bend right=15] cycle ; \draw[fill=pink,opacity=0.5] (a) to[bend right=15] (b)to[bend right=15] (e) to[bend right=15] cycle ; \draw[fill=pink,opacity=0.5] (d) to[bend left=15] (c)to[bend left=50] (a) to[bend right=15] cycle ; \draw[fill=pink,opacity=0.5] (e) to[bend right=15] (c)to[bend right=50] (b) to[bend left=15] cycle ; \draw[fill=pink,opacity=0.5] (c) to[bend left=15] (d)to[bend right=20] (e) to[bend left=15] cycle ; \draw (0,0) node[vtx,label=above:$b$](b){}; \draw (-2,0) node[vtx,label=above:$a$](a){}; \draw (-1,-3) node[vtx,label=below:$c$](c){}; \draw (1,-3) node[vtx,label=below:$e$](e){}; \draw (-3,-3) node[vtx,label=below:$d$](d){}; \draw[thick, blue] (a) to[bend left = 20] (e); \draw[thick, blue] (b) to[bend right = 20] (d); \draw[thick, blue] (a) to[bend left = 15] (d); \draw[thick, blue] (c) to[bend right = 10] (e); \draw[thick, blue] (c) to[bend right = 55] (b); \draw (-1, -4) node{$G_f$ and $F$}; \begin{scope}[yshift = 1cm] \draw (5,-2.6) node[vtx,label=below:$a$](x1){}; \draw (7,-3) node[vtx,label=below:$b$](z1){}; \draw (7.5,-2.2) node[vtx,label=below:$f$](w1){}; \draw (9, -2.8) node[vtx,label=below:$e$](y1){}; \draw (6, -1) node[vtx,label=above:$d$](f){}; \draw (8, -1) node[vtx,label=above:$c$](c){}; \draw (7, -4) node[vtx,label=below:$x$](g1){}; \draw[fill=pink,opacity=0.5] (x1) to[bend right=45] (y1) to[bend left=25] (g1) to[bend left=25] cycle ; \draw[fill=pink,opacity=0.5] (x1) to[bend left=15] (w1)to[bend left=15] (f) to[bend left=2] cycle ; \draw[fill=pink,opacity=0.5] (x1) to[bend left=15] (z1)to[bend left=15] (f) to[bend left=15] cycle ; \draw[fill=pink,opacity=1] (z1) to[bend left=1] (w1)to[bend left=1] (f) to[bend right=1] cycle ; \draw[fill=pink,opacity=0.5] (y1) to[bend right=15] (w1)to[bend right = 15] (c) to[bend right=2] cycle ; \draw[fill=pink,opacity=1] (z1) to[bend right=1] (w1)to[bend left=1] (c) to[bend right=10] cycle ; \draw (7.5,-2.2) node[vtx,label=below:$f$](w1){}; \draw[fill=pink,opacity=0.5] (y1) to[bend right=15] (z1)to[bend right=35] (c) to[bend right=25] cycle ; \draw (5,-2.6) node[vtx,label=below:$a$](x1){}; \draw (7,-3) node[vtx,label=below:$b$](z1){}; \draw (9, -2.8) node[vtx,label=below:$e$](y1){}; \draw (6, -1) node[vtx,label=above:$d$](f){}; \draw (8, -1) node[vtx,label=above:$c$](c){}; \end{scope} \draw (7,-4) node{$F_1$}; \end{tikzpicture} \caption{The configuration when $ad, ae \in E(G_f)$, and the resulting copy of $F_1$.} \label{noad} \end{figure} Thus, we have $ac, ae \in E(G_f)$ and $ad \not\in E(G_f)$. By (viii), $ce \not\in E(G_f)$, and by (iv), at most one of $cd, de$ is in $E(G_f)$. Thus, $d(b) = 2$. By (vii), at most one of $bc$, $be$ is in $E(G_f)$, hence $bd \in E(G_f)$. Suppose for a contradiction that $be \in E(G_f)$. Then by (viii), $de\not\in E(G_f)$. Since $G_f$ has at least 5 edges, $dc \in E(G_f)$. Since $bd \in E(G_f)$, there exists a vertex $x$ not in $V(F)$ that is a common neighbor of $b$ and $d$. The set of edges $\{abe,adc, fbe, fae, fac, fcd, bdx\}$ forms $F_1$. See Figure~\ref{fig:nobd} for an illustration of $G_f$, $F$, and $F_1$. \begin{figure}[h] \centering \begin{tikzpicture} \draw (0,0) node[vtx,label=above:$a$](b){}; \draw (-2,0) node[vtx,label=above:$b$](a){}; \draw (-1,-3) node[vtx,label=below:$c$](c){}; \draw (1,-3) node[vtx,label=below:$d$](e){}; \draw (-3,-3) node[vtx,label=below:$e$](d){}; \draw[fill=pink,opacity=0.5] (a) to[bend right=15] (b)to[bend right=15] (c) to[bend right=15] cycle ; \draw[fill=pink,opacity=0.5] (a) to[bend right=15] (b)to[bend right=15] (d) to[bend right=15] cycle ; \draw[fill=pink,opacity=0.5] (a) to[bend right=15] (b)to[bend right=15] (e) to[bend right=15] cycle ; \draw[fill=pink,opacity=0.5] (d) to[bend left=15] (c)to[bend left=50] (a) to[bend right=15] cycle ; \draw[fill=pink,opacity=0.5] (e) to[bend right=15] (c)to[bend right=50] (b) to[bend left=15] cycle ; \draw[fill=pink,opacity=0.5] (c) to[bend left=15] (d)to[bend right=20] (e) to[bend left=15] cycle ; \draw[thick, blue] (a) to[bend left = 20] (e); \draw[thick, blue] (b) to[bend right = 20] (d); \draw[thick, blue] (a) to[bend left = 15] (d); \draw[thick, blue] (c) to[bend right = 10] (e); \draw[thick, blue] (c) to[bend right = 55] (b); \draw (c) node[vtx,label=below:$c$](c){}; \draw (-1, -4) node{$G_f$ and $F$}; \begin{scope}[yshift = 1cm] \draw (5,-2.6) node[vtx,label=below:$b$](x1){}; \draw (7,-3) node[vtx,label=below:$a$](z1){}; \draw (7.5,-2.2) node[vtx,label=below:$f$](w1){}; \draw (9, -2.8) node[vtx,label=below:$d$](y1){}; \draw (6, -1) node[vtx,label=above:$e$](f){}; \draw (8, -1) node[vtx,label=above:$c$](c){}; \draw (7, -4) node[vtx,label=below:$x$](g1){}; \draw[fill=pink,opacity=0.5] (x1) to[bend right=45] (y1) to[bend left=25] (g1) to[bend left=25] cycle ; \draw[fill=pink,opacity=0.5] (x1) to[bend left=15] (w1)to[bend left=15] (f) to[bend left=2] cycle ; \draw[fill=pink,opacity=0.5] (x1) to[bend left=15] (z1)to[bend left=15] (f) to[bend left=15] cycle ; \draw[fill=pink,opacity=1] (z1) to[bend left=1] (w1)to[bend left=1] (f) to[bend right=1] cycle ; \draw[fill=pink,opacity=0.5] (y1) to[bend right=15] (w1)to[bend right = 15] (c) to[bend right=2] cycle ; \draw[fill=pink,opacity=1] (z1) to[bend right=1] (w1)to[bend left=1] (c) to[bend right=10] cycle ; \draw (7.5,-2.2) node[vtx,label=below:$f$](w1){}; \draw[fill=pink,opacity=0.5] (y1) to[bend right=15] (z1)to[bend right=35] (c) to[bend right=25] cycle ; \end{scope} \draw (7,-4) node{$F_1$}; \end{tikzpicture} \caption{The configuration when $be\in E(G_f)$, and the resulting copy of $F_1$.} \label{fig:nobd} \end{figure} Thus, $bc \in E(G_f)$. By (viii), $cd \not\in E(G_f)$, so we must have $de \in E(G_f)$. Hence $E(G_f) = \{ ae,ac,bc,bd,de \}$; see Figure~\ref{goodC5} for an illustration. \begin{figure}[h] \centering \begin{tikzpicture} \draw (0,0) node[vtx,label=above:$b$](b){}; \draw (-2,0) node[vtx,label=above:$a$](a){}; \draw (-1,-3) node[vtx,label=below:$c$](c){}; \draw (1,-3) node[vtx,label=below:$e$](e){}; \draw (-3,-3) node[vtx,label=below:$d$](d){}; \draw[fill=pink,opacity=0.5] (a) to[bend right=15] (b)to[bend right=15] (c) to[bend right=15] cycle ; \draw[fill=pink,opacity=0.5] (a) to[bend right=15] (b)to[bend right=15] (d) to[bend right=15] cycle ; \draw[fill=pink,opacity=0.5] (a) to[bend right=15] (b)to[bend right=15] (e) to[bend right=15] cycle ; \draw[fill=pink,opacity=0.5] (d) to[bend left=15] (c)to[bend left=50] (a) to[bend right=15] cycle ; \draw[fill=pink,opacity=0.5] (e) to[bend right=15] (c)to[bend right=50] (b) to[bend left=15] cycle ; \draw[fill=pink,opacity=0.5] (c) to[bend left=15] (d)to[bend right=20] (e) to[bend left=15] cycle ; \draw[thick, blue] (a) to[bend left = 20] (c); \draw[thick, blue] (b) to[bend right = 20] (d); \draw[thick, blue] (a) to[bend left = 20] (e); \draw[thick, blue] (d) to[bend right = 20] (e); \draw[thick, blue] (b) to[bend right = 20] (c); \draw (0,0) node[vtx,label=above:$b$](b){}; \draw (-2,0) node[vtx,label=above:$a$](a){}; \draw (-1,-3) node[vtx,label=below:$c$](c){}; \draw (1,-3) node[vtx,label=below:$e$](e){}; \draw (-3,-3) node[vtx,label=below:$d$](d){}; \end{tikzpicture} \caption{$F$ and $G_f$ when $ac, ae \in E(G_f)$.}\label{goodC5} \end{figure} Unlike in the previous cases, we cannot immediately find \acopyof $F_1$ (or any other forbidden hypergraph) in Figure~\ref{goodC5}. However, we now have that the subhypergraph of $H$ induced on $\{a,b,c,d,e,f\}$ has edge set $\{abc, abd, abe, adc, bce, cde, fac, fcb, fbd, fde, fea \}$. We call this subhypergraph $F'$, and we shall use $F'$ to find \acopyof some forbidden hypergraph. Observe first that each of the $15$ pairs of vertices in $F'$ have positive co-degree. Thus, there exists some $g \in V(H)$ that is in the neighborhood of at least $15\left( \frac{2}{5} + \varepsilon \right) > 6$ pairs. Let $G_g$ be the link graph of vertex $g$ induced on $V(F')$ (i.e.\ $G_g = L(g)[V(F')]$). Again, we begin with some observations on $G_g$. \begin{claim}\label{Gg} $\delta(G_g) \geq 1$. Moreover, in $G_g$, $N(f) = \{c,d,e\}$. \end{claim} \begin{proof} As in the proof of Claim~\ref{VFclaims}, since all pairs from $\{a,b,c,d,e,f\}$ have positive co-degree, $G_g$ cannot contain \acopyof $K_4^-$. If $G_g$ has an isolated vertex $v$, then $G_g - v$ is a component on $5$ vertices and $7$ edges that necessarily contains a \acopyof $K_4^-$. Thus, every vertex of $G_g$ has positive degree. Next, observe that the previous analysis of $G_f$ in fact shows that if some vertex $v \in V(H) \setminus V(F)$ has positive co-degree with at least $5$ pairs from $V(F)$, then the link graph of $v$ induced on $V(F)$ must be equal to $G_f$. In particular, either $G_g$ contains $G_f$ or $g$ has positive co-degree with at most $4$ pairs from $V(F)$. Observe that if $G_g$ contains $G_f$, then $\{a,b,c,f\}$ and $\{a,b,c,g\}$ span copies of $K_4^{3-}$ with spike vertex $c$. This implies that $\{a,b,c,f,g\}$ spans \acopyof $F_2$, since we have argued that $d(f,g) > 0$. Thus, $g$ has positive co-degree with at most $4$ pairs from $V(F)$, which implies that $f$ has degree at least $3$ in $G_g$, since $|E(G_g)| \geq 7$. Finally, to determine $N(f)$ in $G_g$, we consider the interaction between $G_g$ and $G_f$. Suppose that $x,y$ are neighbors of $f$ in $G_g$ such that $xy \in E(G_f)$. Then $\{x,y,f,g\}$ forms \acopyof $K_4^{3-}$ with spike vertex $f$. We consider the possible values of $x,y$; we know $xy \in \{ac,ae, bc, bd, de\}$. Since $\{a,b,c,d\}$ spans \acopyof $K_4^{3-}$ with spike vertex $a$, and we have established that $g$ has positive co-degree with every vertex in $V(F)$, we can find \acopyof $F_1$ in $H$ on vertex set $\{a,b,c,d,f,g\}$ if $xy \in \{bc, bd\}$. Similarly, if $xy \in \{ac, ae\}$, then we can find \acopyof $F_1$ on $\{a,b,c,e,f,g\}$. Thus, in $G_g$, either $N(f)$ is an independent set or $N(f)$ contains precisely the edge $de$. Recall that $|N(f)| \geq 3$, so the first outcome is impossible, as the independence number of $G_f$ is $2$. The second outcome occurs only if $N(f) = \{c,d,e\}$. \end{proof} With Claim~\ref{Gg} established, we conclude that $|E(G_g)| = 7$, with exactly 4 edges of $G_g$ spanned by $\{a,b,c,d,e\}$. Given that $cf, df, ef \in E(G_g)$, we shall prove that this is not possible. First, observe that none of $cd,ce,de$ are in $E(G_g)$. Indeed, if $de \in E(G_g)$, then $\{d,e,f,g\}$ spans \acopyof $K_4^3$ (recall that $de \in E(G_f)$). If either $cd$ or $ce$ is in $E(G_g)$, then either $\{c,d,f,g\}$ or $\{c,e,f,g\}$ spans \acopyof $K_4^{3-}$ with spike vertex $g$; if $\{c,d,f,g\}$ spans $K_4^{3-}$, then $\{a,b,c,d,f,g\}$ spans an \acopyof $F_1$, while if $\{c,e,f,g\}$ spans $K_4^{3-}$, then $\{a,b,c,e,f,g\}$ spans an \acopyof $F_1$. Notice that Claim~\ref{VFclaims} applies also to $G_g$ since Claim~\ref{Gg} proves (ii) and it is the only statement that used the number of edges of $G_f$. Next, we prove that $ab \not\in E(G_g)$. Indeed, by (v) and (vi), if $ab \in E(G_g)$, then $ac,bc,ad,be$ are not in $E(G_g)$. Recall $cd,ce,de$ are not in $E(G_g)$. So if $ab \in E(G_g)$, then at most three edges of $G_g$ are spanned by $\{a,b,c,d,e\}$. However, by Claim~\ref{Gg}, $d(f) = 3$ in $G_g$, so we have $|E(G_g)| \leq 6$, a contradiction. Thus, $ab \not\in E(G_g)$. Finally, by (vii), at most two of $ac, ad, bc, be$ are in $E(G_g)$. So in order to have $|E(G_g)| = 7$, both $ae$ and $bd$ must be in $E(G_g)$. However, this results in \acopyof $F_1$ on $\{a,b,d,e,g,f\}$ with edges $\{age, afe, abc, gfe, gfd, gbd, bdf \}$ as depicted in Figure~\ref{finalconfig}. \end{proof} \begin{figure}[h!] \centering \begin{tikzpicture} \draw (0,0) node[vtx,label=above:$b$](b){}; \draw (-2,0) node[vtx,label=above:$a$](a){}; \draw (-1,-3) node[vtx,label=below left:$c$](c){}; \draw (1,-3) node[vtx,label=below:$e$](e){}; \draw (-3,-3) node[vtx,label=below:$d$](d){}; \draw (-1,-5) node[vtx,label=below:$f$](f){}; \draw[fill=pink,opacity=0.5] (a) to[bend right=15] (b)to[bend right=15] (c) to[bend right=15] cycle ; \draw[fill=pink,opacity=0.5] (a) to[bend right=15] (b)to[bend right=15] (d) to[bend right=15] cycle ; \draw[fill=pink,opacity=0.5] (a) to[bend right=15] (b)to[bend right=15] (e) to[bend right=15] cycle ; \draw[fill=pink,opacity=0.5] (d) to[bend left=15] (c)to[bend left=50] (a) to[bend right=15] cycle ; \draw[fill=pink,opacity=0.5] (e) to[bend right=15] (c)to[bend right=50] (b) to[bend left=15] cycle ; \draw[fill=pink,opacity=0.5] (c) to[bend left=15] (d)to[bend right=20] (e) to[bend left=15] cycle ; \draw[line width=1pt, blue] (a) to[bend left = 20] (c) (b) to[bend right = 20] (d) (a) to[bend left = 20] (e) (d) to[bend right = 20] (e) (b) to[bend right = 20] (c); \draw[line width=1.5pt, darkpastelgreen, dashed] (d) to[bend right = 20] (f) (e) to[bend left = 20] (f) (c) to (f) (b) to[bend right = 35] (d) (a) to[bend left = 35] (e); \draw (0,0) node[vtx,label=above:$b$](b){}; \draw (-2,0) node[vtx,label=above:$a$](a){}; \draw (-1,-3) node[vtx,label=below left:$c$](c){}; \draw (1,-3) node[vtx,label=below:$e$](e){}; \draw (-3,-3) node[vtx,label=below:$d$](d){}; \draw (-1,-5) node[vtx,label=below:$f$](f){}; \draw (5,-2.6) node[vtx,label=below:$a$](x1){}; \draw (7,-3) node[vtx,label=below:$g$](z1){}; \draw (7.5,-2.2) node[vtx,label=below:$f$](w1){}; \draw (9, -2.8) node[vtx,label=below:$b$](y1){}; \draw (6, -1) node[vtx,label=above:$e$](s1){}; \draw (8, -1) node[vtx,label=above:$d$](t1){}; \draw (7, -4) node[vtx,label=below:$c$](g1){}; \draw[fill=pink,opacity=0.5] (x1) to[bend right=45] (y1) to[bend left=25] (g1) to[bend left=25] cycle ; \draw[fill=pink,opacity=0.5] (x1) to[bend left=15] (w1)to[bend left=15] (s1) to[bend left=2] cycle ; \draw[fill=pink,opacity=0.5] (x1) to[bend left=15] (z1)to[bend left=15] (s1) to[bend left=15] cycle ; \draw[fill=pink,opacity=1] (z1) to[bend left=1] (w1)to[bend left=1] (s1) to[bend right=1] cycle ; \draw[fill=pink,opacity=0.5] (y1) to[bend right=15] (w1)to[bend right = 15] (t1) to[bend right=2] cycle ; \draw[fill=pink,opacity=1] (z1) to[bend right=1] (w1)to[bend left=1] (t1) to[bend right=10] cycle ; \draw (7.5,-2.2) node[vtx,label=below:$f$](w1){}; \draw[fill=pink,opacity=0.5] (y1) to[bend right=15] (z1)to[bend right=35] (t1) to[bend right=25] cycle ; \draw (5,-2.6) node[vtx,label=below:$a$](x1){}; \draw (7,-3) node[vtx,label=below:$g$](z1){}; \draw (9, -2.8) node[vtx,label=below:$b$](y1){}; \draw (6, -1) node[vtx,label=above:$e$](s1){}; \draw (8, -1) node[vtx,label=above:$d$](t1){}; \end{tikzpicture} \caption{$F'$ (edges involving $f$ indicated by $G_f$ in thick blue) and a configuration in $G_g$ (in dashed green) yielding $F_1$.}\label{finalconfig} \end{figure} \section{Positive co-degree densities from flag algebras}\label{sec:J4} This section contains calculations using flag algebras, as introduced by Razborov~\cite{Raz07}. For an introduction to flag algebras and formal definitions, see~\cite{FalgasK4-,Raz07,FALGAS-RAVRY_VAUGHAN_2013}. Computer code is available at \oururl. We use flag algebras to prove Theorem~\ref{J4} that $\gamma^+(J_4) = 4/7$, with the asymptotically unique construction being the balanced blow-up of the complement of the Fano plane. In Figure~\ref{fig:Fano}, we illustrate (a blow-up of) the Fano plane, with a particular labeling of classes to which we will later refer. We denote the complement of the Fano plane by $\overline{\mathbb{F}}$ and the $n$-vertex, balanced blow-up of the complement of the Fano plane by $\overline{\mathbb{F}}_n$. \begin{figure}[h!] \begin{center} \begin{tikzpicture} \draw[scale=5,line width=6pt] (0,0) coordinate (x7) -- ++(1,0) coordinate[pos=0.5](x1) coordinate(x2) (x7) -- ++(60:1) coordinate[pos=0.5](x3) coordinate(x4) (x2) -- ++(120:0.5) coordinate(x5) (x7) -- ++(30:0.57735) coordinate(x6) (x4)--(x5) (x2)--(x3) (x6)--(x5) (x4)--(x1) (x6) circle (0.2886751345 cm) ; \draw \foreach \x/\y in {1/1,2/2,3/6,4/7,5/3,6/4,7/5}{ (x\x) node[circle,draw,fill=white]{$X_\y$} } ; \end{tikzpicture} \end{center} \caption{A blow-up of the Fano plane. In $\overline{\mathbb{F}}_n$, $3$-edges span triples of classes which do not span $3$-edges in the blow-up of $\mathbb{F}$.} \label{fig:Fano} \end{figure} \begin{figure} \begin{center} \def\xshift{1.25} \def\yshift{-3.6} \def\yshiftB{-2.6} \def\yshiftC{-2.3} \def\yshiftD{-2.3} \def\ourscale{0.9} \scalebox{\ourscale}{ \vc{ \begin{tikzpicture}[flag_pic]\outercycle{6}{0} \draw(\xshift,\yshift) node{$F_1$}; \draw (x0) node{\tiny$X_1$} (x1) node{\tiny$X_2$} (x2) node{\tiny$X_3$} (x3) node{\tiny$X_4$} (x4) node{\tiny$X_4$} (x5) node{\tiny$X_7$} ; \drawhyperedge{0}{6} \drawhypervertex{0}{0} \drawhypervertex{1}{0} \drawhypervertex{2}{0} \drawhyperedge{1}{6} \drawhypervertex{0}{1} \drawhypervertex{1}{1} \drawhypervertex{3}{1} \drawhyperedge{2}{6} \drawhypervertex{0}{2} \drawhypervertex{1}{2} \drawhypervertex{4}{2} \drawhyperedge{3}{6} \drawhypervertex{0}{3} \drawhypervertex{1}{3} \drawhypervertex{5}{3} \drawhyperedge{4}{6} \drawhypervertex{0}{4} \drawhypervertex{2}{4} \drawhypervertex{3}{4} \drawhyperedge{5}{6} \drawhypervertex{0}{5} \drawhypervertex{2}{5} \drawhypervertex{4}{5} \drawhyperedge{6}{6} \drawhypervertex{0}{6} \drawhypervertex{2}{6} \drawhypervertex{5}{6} \drawhyperedge{7}{6} \drawhypervertex{1}{7} \drawhypervertex{2}{7} \drawhypervertex{3}{7} \drawhyperedge{8}{6} \drawhypervertex{1}{8} \drawhypervertex{2}{8} \drawhypervertex{4}{8} \drawhyperedge{9}{6} \drawhypervertex{1}{9} \drawhypervertex{3}{9} \drawhypervertex{5}{9} \drawhyperedge{10}{6} \drawhypervertex{1}{10} \drawhypervertex{4}{10} \drawhypervertex{5}{10} \drawhyperedge{11}{6} \drawhypervertex{2}{11} \drawhypervertex{3}{11} \drawhypervertex{5}{11} \drawhyperedge{12}{6} \drawhypervertex{2}{12} \drawhypervertex{4}{12} \drawhypervertex{5}{12} \end{tikzpicture} } \vc{ \begin{tikzpicture}[flag_pic]\outercycle{6}{0} \draw(\xshift,\yshift) node{$F_2$}; \draw (x0) node{\tiny$X_1$} (x1) node{\tiny$X_2$} (x2) node{\tiny$X_3$} (x3) node{\tiny$X_4$} (x4) node{\tiny$X_4$} (x5) node{\tiny$X_3$} ; \drawhyperedge{0}{6} \drawhypervertex{0}{0} \drawhypervertex{1}{0} \drawhypervertex{2}{0} \drawhyperedge{1}{6} \drawhypervertex{0}{1} \drawhypervertex{1}{1} \drawhypervertex{3}{1} \drawhyperedge{2}{6} \drawhypervertex{0}{2} \drawhypervertex{1}{2} \drawhypervertex{4}{2} \drawhyperedge{3}{6} \drawhypervertex{0}{3} \drawhypervertex{1}{3} \drawhypervertex{5}{3} \drawhyperedge{4}{6} \drawhypervertex{0}{4} \drawhypervertex{2}{4} \drawhypervertex{3}{4} \drawhyperedge{5}{6} \drawhypervertex{0}{5} \drawhypervertex{2}{5} \drawhypervertex{4}{5} \drawhyperedge{6}{6} \drawhypervertex{0}{6} \drawhypervertex{3}{6} \drawhypervertex{5}{6} \drawhyperedge{7}{6} \drawhypervertex{0}{7} \drawhypervertex{4}{7} \drawhypervertex{5}{7} \drawhyperedge{8}{6} \drawhypervertex{1}{8} \drawhypervertex{2}{8} \drawhypervertex{3}{8} \drawhyperedge{9}{6} \drawhypervertex{1}{9} \drawhypervertex{2}{9} \drawhypervertex{4}{9} \drawhyperedge{10}{6} \drawhypervertex{1}{10} \drawhypervertex{3}{10} \drawhypervertex{5}{10} \drawhyperedge{11}{6} \drawhypervertex{1}{11} \drawhypervertex{4}{11} \drawhypervertex{5}{11} \end{tikzpicture} } \vc{ \begin{tikzpicture}[flag_pic]\outercycle{6}{0} \draw(\xshift,\yshift) node{$F_3$}; \draw (x0) node{\tiny$X_1$} (x1) node{\tiny$X_2$} (x2) node{\tiny$X_3$} (x3) node{\tiny$X_4$} (x4) node{\tiny$X_7$} (x5) node{\tiny$X_7$} ; \drawhyperedge{0}{6} \drawhypervertex{0}{0} \drawhypervertex{1}{0} \drawhypervertex{2}{0} \drawhyperedge{1}{6} \drawhypervertex{0}{1} \drawhypervertex{1}{1} \drawhypervertex{3}{1} \drawhyperedge{2}{6} \drawhypervertex{0}{2} \drawhypervertex{1}{2} \drawhypervertex{4}{2} \drawhyperedge{3}{6} \drawhypervertex{0}{3} \drawhypervertex{1}{3} \drawhypervertex{5}{3} \drawhyperedge{4}{6} \drawhypervertex{0}{4} \drawhypervertex{2}{4} \drawhypervertex{3}{4} \drawhyperedge{5}{6} \drawhypervertex{0}{5} \drawhypervertex{2}{5} \drawhypervertex{4}{5} \drawhyperedge{6}{6} \drawhypervertex{0}{6} \drawhypervertex{2}{6} \drawhypervertex{5}{6} \drawhyperedge{7}{6} \drawhypervertex{1}{7} \drawhypervertex{2}{7} \drawhypervertex{3}{7} \drawhyperedge{8}{6} \drawhypervertex{1}{8} \drawhypervertex{3}{8} \drawhypervertex{4}{8} \drawhyperedge{9}{6} \drawhypervertex{1}{9} \drawhypervertex{3}{9} \drawhypervertex{5}{9} \drawhyperedge{10}{6} \drawhypervertex{2}{10} \drawhypervertex{3}{10} \drawhypervertex{4}{10} \drawhyperedge{11}{6} \drawhypervertex{2}{11} \drawhypervertex{3}{11} \drawhypervertex{5}{11} \end{tikzpicture} } \vc{ \begin{tikzpicture}[flag_pic]\outercycle{6}{0} \draw(\xshift,\yshift) node{$F_4$}; \draw (x0) node{\tiny$X_1$} (x1) node{\tiny$X_2$} (x2) node{\tiny$X_3$} (x3) node{\tiny$X_4$} (x4) node{\tiny$X_7$} (x5) node{\tiny$X_6$} ; \drawhyperedge{0}{6} \drawhypervertex{0}{0} \drawhypervertex{1}{0} \drawhypervertex{2}{0} \drawhyperedge{1}{6} \drawhypervertex{0}{1} \drawhypervertex{1}{1} \drawhypervertex{3}{1} \drawhyperedge{2}{6} \drawhypervertex{0}{2} \drawhypervertex{1}{2} \drawhypervertex{4}{2} \drawhyperedge{3}{6} \drawhypervertex{0}{3} \drawhypervertex{1}{3} \drawhypervertex{5}{3} \drawhyperedge{4}{6} \drawhypervertex{0}{4} \drawhypervertex{2}{4} \drawhypervertex{3}{4} \drawhyperedge{5}{6} \drawhypervertex{0}{5} \drawhypervertex{2}{5} \drawhypervertex{4}{5} \drawhyperedge{6}{6} \drawhypervertex{0}{6} \drawhypervertex{3}{6} \drawhypervertex{5}{6} \drawhyperedge{7}{6} \drawhypervertex{0}{7} \drawhypervertex{4}{7} \drawhypervertex{5}{7} \drawhyperedge{8}{6} \drawhypervertex{1}{8} \drawhypervertex{2}{8} \drawhypervertex{3}{8} \drawhyperedge{9}{6} \drawhypervertex{1}{9} \drawhypervertex{2}{9} \drawhypervertex{5}{9} \drawhyperedge{10}{6} \drawhypervertex{1}{10} \drawhypervertex{3}{10} \drawhypervertex{4}{10} \drawhyperedge{11}{6} \drawhypervertex{1}{11} \drawhypervertex{4}{11} \drawhypervertex{5}{11} \drawhyperedge{12}{6} \drawhypervertex{2}{12} \drawhypervertex{3}{12} \drawhypervertex{4}{12} \drawhyperedge{13}{6} \drawhypervertex{2}{13} \drawhypervertex{3}{13} \drawhypervertex{5}{13} \drawhyperedge{14}{6} \drawhypervertex{2}{14} \drawhypervertex{4}{14} \drawhypervertex{5}{14} \drawhyperedge{15}{6} \drawhypervertex{3}{15} \drawhypervertex{4}{15} \drawhypervertex{5}{15} \end{tikzpicture} } } \scalebox{\ourscale}{ \vc{ \begin{tikzpicture}[flag_pic] \draw(\xshift,\yshiftB) node{$F_5$}; \outercycle{6}{0} \draw (x0) node{\tiny$X_1$} (x1) node{\tiny$X_2$} (x2) node{\tiny$X_3$} (x3) node{\tiny$X_4$} (x4) node{\tiny$X_4$} (x5) node{\tiny$X_4$} ; \drawhypervertex{0}{0} \drawhypervertex{1}{0} \drawhypervertex{2}{0} \drawhyperedge{1}{6} \drawhypervertex{0}{1} \drawhypervertex{1}{1} \drawhypervertex{3}{1} \drawhyperedge{2}{6} \drawhypervertex{0}{2} \drawhypervertex{1}{2} \drawhypervertex{4}{2} \drawhyperedge{3}{6} \drawhypervertex{0}{3} \drawhypervertex{1}{3} \drawhypervertex{5}{3} \drawhyperedge{4}{6} \drawhypervertex{0}{4} \drawhypervertex{2}{4} \drawhypervertex{3}{4} \drawhyperedge{5}{6} \drawhypervertex{0}{5} \drawhypervertex{2}{5} \drawhypervertex{4}{5} \drawhyperedge{6}{6} \drawhypervertex{0}{6} \drawhypervertex{2}{6} \drawhypervertex{5}{6} \drawhyperedge{7}{6} \drawhypervertex{1}{7} \drawhypervertex{2}{7} \drawhypervertex{3}{7} \drawhyperedge{8}{6} \drawhypervertex{1}{8} \drawhypervertex{2}{8} \drawhypervertex{4}{8} \drawhyperedge{9}{6} \drawhypervertex{1}{9} \drawhypervertex{2}{9} \drawhypervertex{5}{9} \end{tikzpicture} } \vc{ \begin{tikzpicture}[flag_pic]\outercycle{6}{0} \draw(\xshift,\yshiftB) node{$F_6$}; \draw (x0) node{\tiny$X_1$} (x1) node{\tiny$X_2$} (x2) node{\tiny$X_3$} (x3) node{\tiny$X_7$} (x4) node{\tiny$X_7$} (x5) node{\tiny$X_7$} ; \drawhyperedge{0}{6} \drawhypervertex{0}{0} \drawhypervertex{1}{0} \drawhypervertex{2}{0} \drawhyperedge{1}{6} \drawhypervertex{0}{1} \drawhypervertex{1}{1} \drawhypervertex{3}{1} \drawhyperedge{2}{6} \drawhypervertex{0}{2} \drawhypervertex{1}{2} \drawhypervertex{4}{2} \drawhyperedge{3}{6} \drawhypervertex{0}{3} \drawhypervertex{1}{3} \drawhypervertex{5}{3} \drawhyperedge{4}{6} \drawhypervertex{0}{4} \drawhypervertex{2}{4} \drawhypervertex{3}{4} \drawhyperedge{5}{6} \drawhypervertex{0}{5} \drawhypervertex{2}{5} \drawhypervertex{4}{5} \drawhyperedge{6}{6} \drawhypervertex{0}{6} \drawhypervertex{2}{6} \drawhypervertex{5}{6} \end{tikzpicture} } \vc{ \begin{tikzpicture}[flag_pic]\outercycle{6}{0} \draw(\xshift,\yshiftB) node{$F_7$}; \draw (x0) node{\tiny$X_1$} (x1) node{\tiny$X_2$} (x2) node{\tiny$X_3$} (x3) node{\tiny$X_7$} (x4) node{\tiny$X_7$} (x5) node{\tiny$X_6$} ; \drawhyperedge{0}{6} \drawhypervertex{0}{0} \drawhypervertex{1}{0} \drawhypervertex{2}{0} \drawhyperedge{1}{6} \drawhypervertex{0}{1} \drawhypervertex{1}{1} \drawhypervertex{3}{1} \drawhyperedge{2}{6} \drawhypervertex{0}{2} \drawhypervertex{1}{2} \drawhypervertex{4}{2} \drawhyperedge{3}{6} \drawhypervertex{0}{3} \drawhypervertex{1}{3} \drawhypervertex{5}{3} \drawhyperedge{4}{6} \drawhypervertex{0}{4} \drawhypervertex{2}{4} \drawhypervertex{3}{4} \drawhyperedge{5}{6} \drawhypervertex{0}{5} \drawhypervertex{2}{5} \drawhypervertex{4}{5} \drawhyperedge{6}{6} \drawhypervertex{0}{6} \drawhypervertex{3}{6} \drawhypervertex{5}{6} \drawhyperedge{7}{6} \drawhypervertex{0}{7} \drawhypervertex{4}{7} \drawhypervertex{5}{7} \end{tikzpicture} } \vc{ \begin{tikzpicture}[flag_pic]\outercycle{6}{0} \draw(\xshift,\yshiftB) node{$F_8$}; \draw (x0) node{\tiny$X_1$} (x1) node{\tiny$X_2$} (x2) node{\tiny$X_3$} (x3) node{\tiny$X_7$} (x4) node{\tiny$X_7$} (x5) node{\tiny$X_1$} ; \drawhyperedge{0}{6} \drawhypervertex{0}{0} \drawhypervertex{1}{0} \drawhypervertex{2}{0} \drawhyperedge{1}{6} \drawhypervertex{0}{1} \drawhypervertex{1}{1} \drawhypervertex{3}{1} \drawhyperedge{2}{6} \drawhypervertex{0}{2} \drawhypervertex{1}{2} \drawhypervertex{4}{2} \drawhyperedge{3}{6} \drawhypervertex{0}{3} \drawhypervertex{2}{3} \drawhypervertex{3}{3} \drawhyperedge{4}{6} \drawhypervertex{0}{4} \drawhypervertex{2}{4} \drawhypervertex{4}{4} \drawhyperedge{5}{6} \drawhypervertex{1}{5} \drawhypervertex{2}{5} \drawhypervertex{5}{5} \drawhyperedge{6}{6} \drawhypervertex{1}{6} \drawhypervertex{3}{6} \drawhypervertex{5}{6} \drawhyperedge{7}{6} \drawhypervertex{1}{7} \drawhypervertex{4}{7} \drawhypervertex{5}{7} \drawhyperedge{8}{6} \drawhypervertex{2}{8} \drawhypervertex{3}{8} \drawhypervertex{5}{8} \drawhyperedge{9}{6} \drawhypervertex{2}{9} \drawhypervertex{4}{9} \drawhypervertex{5}{9} \end{tikzpicture} } } \scalebox{\ourscale}{ \vc{ \begin{tikzpicture}[flag_pic]\outercycle{6}{0} \draw(\xshift,\yshiftC) node{$F_9$}; \draw (x0) node{\tiny$X_1$} (x1) node{\tiny$X_2$} (x2) node{\tiny$X_3$} (x3) node{\tiny$X_3$} (x4) node{\tiny$X_1$} (x5) node{\tiny$X_1$} ; \drawhyperedge{0}{6} \drawhypervertex{0}{0} \drawhypervertex{1}{0} \drawhypervertex{2}{0} \drawhyperedge{1}{6} \drawhypervertex{0}{1} \drawhypervertex{1}{1} \drawhypervertex{3}{1} \drawhyperedge{2}{6} \drawhypervertex{0}{2} \drawhypervertex{2}{2} \drawhypervertex{3}{2} \drawhyperedge{3}{6} \drawhypervertex{1}{3} \drawhypervertex{2}{3} \drawhypervertex{4}{3} \drawhyperedge{4}{6} \drawhypervertex{1}{4} \drawhypervertex{2}{4} \drawhypervertex{5}{4} \drawhyperedge{5}{6} \drawhypervertex{1}{5} \drawhypervertex{3}{5} \drawhypervertex{4}{5} \drawhyperedge{6}{6} \drawhypervertex{1}{6} \drawhypervertex{3}{6} \drawhypervertex{5}{6} \drawhyperedge{7}{6} \drawhypervertex{2}{7} \drawhypervertex{3}{7} \drawhypervertex{4}{7} \drawhyperedge{8}{6} \drawhypervertex{2}{8} \drawhypervertex{3}{8} \drawhypervertex{5}{8} \end{tikzpicture} } \vc{ \begin{tikzpicture}[flag_pic]\outercycle{6}{0} \draw(\xshift,\yshiftC) node{$F_{10}$}; \draw (x0) node{\tiny$X_1$} (x1) node{\tiny$X_2$} (x2) node{\tiny$X_3$} (x3) node{\tiny$X_3$} (x4) node{\tiny$X_3$} (x5) node{\tiny$X_3$} ; \drawhyperedge{0}{6} \drawhypervertex{0}{0} \drawhypervertex{1}{0} \drawhypervertex{2}{0} \drawhyperedge{1}{6} \drawhypervertex{0}{1} \drawhypervertex{1}{1} \drawhypervertex{3}{1} \drawhyperedge{2}{6} \drawhypervertex{0}{2} \drawhypervertex{1}{2} \drawhypervertex{4}{2} \drawhyperedge{3}{6} \drawhypervertex{0}{3} \drawhypervertex{1}{3} \drawhypervertex{5}{3} \end{tikzpicture} } \vc{ \begin{tikzpicture}[flag_pic]\outercycle{6}{0} \draw(\xshift,\yshiftC) node{$F_{11}$}; \draw (x0) node{\tiny$X_1$} (x1) node{\tiny$X_2$} (x2) node{\tiny$X_3$} (x3) node{\tiny$X_3$} (x4) node{\tiny$X_3$} (x5) node{\tiny$X_2$} ; \drawhyperedge{0}{6} \drawhypervertex{0}{0} \drawhypervertex{1}{0} \drawhypervertex{2}{0} \drawhyperedge{1}{6} \drawhypervertex{0}{1} \drawhypervertex{1}{1} \drawhypervertex{3}{1} \drawhyperedge{2}{6} \drawhypervertex{0}{2} \drawhypervertex{1}{2} \drawhypervertex{4}{2} \drawhyperedge{3}{6} \drawhypervertex{0}{3} \drawhypervertex{2}{3} \drawhypervertex{5}{3} \drawhyperedge{4}{6} \drawhypervertex{0}{4} \drawhypervertex{3}{4} \drawhypervertex{5}{4} \drawhyperedge{5}{6} \drawhypervertex{0}{5} \drawhypervertex{4}{5} \drawhypervertex{5}{5} \end{tikzpicture} } \vc{ \begin{tikzpicture}[flag_pic]\outercycle{6}{0} \draw(\xshift,\yshiftC) node{$F_{12}$}; \draw (x0) node{\tiny$X_1$} (x1) node{\tiny$X_2$} (x2) node{\tiny$X_3$} (x3) node{\tiny$X_3$} (x4) node{\tiny$X_2$} (x5) node{\tiny$X_1$} ; \drawhypervertex{0}{0} \drawhypervertex{1}{0} \drawhypervertex{2}{0} \drawhyperedge{1}{6} \drawhypervertex{0}{1} \drawhypervertex{1}{1} \drawhypervertex{3}{1} \drawhyperedge{2}{6} \drawhypervertex{0}{2} \drawhypervertex{2}{2} \drawhypervertex{4}{2} \drawhyperedge{3}{6} \drawhypervertex{0}{3} \drawhypervertex{3}{3} \drawhypervertex{4}{3} \drawhyperedge{4}{6} \drawhypervertex{1}{4} \drawhypervertex{2}{4} \drawhypervertex{5}{4} \drawhyperedge{5}{6} \drawhypervertex{1}{5} \drawhypervertex{3}{5} \drawhypervertex{5}{5} \drawhyperedge{6}{6} \drawhypervertex{2}{6} \drawhypervertex{4}{6} \drawhypervertex{5}{6} \drawhyperedge{7}{6} \drawhypervertex{3}{7} \drawhypervertex{4}{7} \drawhypervertex{5}{7} \end{tikzpicture} } \vc{ \begin{tikzpicture}[flag_pic]\outercycle{6}{0} \draw(\xshift,\yshiftD) node{$F_{13}$}; \drawhyperedge{0}{6} \draw (x0) node{\tiny$X_1$} (x1) node{\tiny$X_1$} (x2) node{\tiny$X_1$} (x3) node{\tiny$X_1$} (x4) node{\tiny$X_1$} (x5) node{\tiny$X_2$} ; \end{tikzpicture} } } \end{center} \caption{The family $\mathcal{F}$ of thirteen $6$-vertex $3$-graphs.}\label{J4 positive density set} \end{figure} Let $\mathcal{F}$ be the family of thirteen 3-graphs 6-vertex induced subgraphs of $\overline{\mathbb{F}}_n$, depicted in Figure~\ref{J4 positive density set}. We include labels to indicate which classes of $\overline{\mathbb{F}}_n$ each vertex belongs to, corresponding to the labeling in Figure~\ref{fig:Fano}. Note that due to the symmetries of $\overline{\mathbb{F}}_n$, the indicated labelings are not the unique ways a subgraph can be obtained. Let $\mathcal{F}_6$ be the family of $J_4$-free $6$-vertex $3$-graphs. The following claim shows that if a $J_4$-free $n$-vertex $3$-graph $G$ has minimum positive co-degree at least $\frac{4}{7}n$ then every $3$-graph on six vertices with positive density in $G$ is in $\mathcal{F}$. \begin{claim}\label{claim:fa47} For every fixed $\delta > 0$, there exists $n_0$ such that for every $n \geq n_0$, if $G_n$ is a $J_4$-free $n$-vertex 3-graph with $\delta_2^+(G_n) \geq \frac{4n}{7}$, then \[ \sum_{H \in \mathcal{F}_6 \setminus \mathcal{F}} d(H,G_n) \leq \delta. \] \end{claim} \begin{proof} The claim is proved by a standard application of flag algebras. The notable part is encoding the condition $\delta_2^+(G_n) \geq \frac{4n}{7}$ into flag algebras by \eqref{eq:f47}. \begin{align} 0 \leq \vc{ \begin{tikzpicture}[flag_pic]\outercycle{3}{2} \draw (x0) node[labeled_vertex]{};\draw (x1) node[labeled_vertex]{};\draw (x2) node[unlabeled_vertex]{}; \labelvertex{0}\labelvertex{1} \drawhyperedge{0}{3} \drawhypervertex{0}{0} \drawhypervertex{1}{0} \drawhypervertex{2}{0} \end{tikzpicture} } \times \Bigg( 7 \vc{ \begin{tikzpicture}[flag_pic]\outercycle{3}{2} \draw (x0) node[labeled_vertex]{};\draw (x1) node[labeled_vertex]{};\draw (x2) node[unlabeled_vertex]{}; \labelvertex{0}\labelvertex{1}\drawhyperedge{0}{3} \drawhypervertex{0}{0} \drawhypervertex{1}{0} \drawhypervertex{2}{0} \end{tikzpicture} } - 4 \vc{ \begin{tikzpicture}[flag_pic]\outercycle{2}{2} \draw (x0) node[labeled_vertex]{};\draw (x1) node[labeled_vertex]{}; \labelvertex{0}\labelvertex{1}\end{tikzpicture} } \Bigg). \label{eq:f47} \end{align} If two labeled vertices 1 and 2 have zero co-degree, the right-hand side of \eqref{eq:f47} is zero because of the first term. If they have positive co-degree, both terms in the product are non-negative. The calculation is computer-assisted and too large to fit in this paper. The details are available at \oururl. \end{proof} In the proof of Theorem~\ref{J4} we will use an induced removal lemma. \begin{theorem}[Induced Removal Lemma, \cite{MR2299696, MR2604289}]\label{thm:removal} Let $r, C \in \mathbb{Z}$ and $\varepsilon >0$ be fixed. For every family of $r$-graphs $\mathcal{F}$ on at most $C$ vertices, there exists $\delta > 0$ such that every sufficiently large $n$, every $r$-graph on $n$ vertices, which contains at most $\delta n^{|V(F)|}$ induced copies of $F$ for every $F \in \mathcal{F}$, can be made induced $\mathcal{F}$-free by adding and/or deleting at most $\varepsilon n^r$ hyperedges. \end{theorem} Claim~\ref{claim:fa47} together with Theorem~\ref{thm:removal} forces 3-graphs with positive co-degree at least $\frac{4}{7}n$ to be highly structured, which is the core of the proof of Theorem~\ref{J4}. \begin{proof}[Proof of Theorem~\ref{J4}] Recall that $\overline{\mathbb{F}}_n$ is the blow-up of the complement of the Fano plane (see Figure~\ref{fig:Fano}) on vertices $x_1,\ldots,x_7$, where each vertex $x_i$ is blown-up to $\frac{n}{7}$ vertices into the set $X_i$. Let $x \in X_i$ and $y \in X_j$ for some $i,j \in [7]$. If $i=j$ then $d(x,y) = 0$. If $i \neq j$ then $d(x,y) = \frac{4}{7}n$. As $J_4 \not\subseteq \overline{\mathbb{F}}_n$, $\mathrm{co^+ex}(n,J_4) \geq \frac{4}{7}n + o(n)$, which implies $\gamma^+(J_4) \geq 4/7$. Next, we show that $\gamma^+(J_4) \leq 4/7$. Fix $\beta > 0$ and $\varepsilon > 0$ small enough such that $24 \varepsilon^{1/4} < \frac{1}{12} \left(\frac{4}{7}\right)^3 \approx 0.015$ and $48\varepsilon^{1/4} < \beta$. We shall fix $n$ sufficiently large such that Claim~\ref{claim:fa47} and Theorem~\ref{thm:removal} imply that every $J_4$-free, $n$-vertex $3$-graph $G$ has a subgraph $G'$ at edit distance at most $\varepsilon n^3$ from $G$ such that every 6-vertex induced subgraph of $G'$ is in $\mathcal{F}$. We will also take $n$ sufficiently large that Lemma~\ref{edge approx} may be applied with $c = \frac{4}{7}$ and any $n$-vertex $3$-graph $H$ with $\delta_2^+(H) \geq \left(\frac{4}{7} - 48 \varepsilon^{1/4}\right)n$ contains a $K_4^3$. This last condition is possible because $\gamma^+(K_4^3) \leq 0.543$ (see Table~\ref{previous bounds}), and $0.543 <\frac{4}{7} - 48 \varepsilon^{1/4}$ by the choice of $\varepsilon$. Fix an $n$-vertex $J_4$-free $3$-graph $G_n$ with $\delta_2^+(G_n) \geq \frac{4}{7}n$. Our goal is to show that $\delta_2^+(G_n) < \left( \frac{4}{7} + \beta\right)n$, which will establish $\gamma^+(J_4) \leq \frac{4}{7}$. The proof will also imply that in fact, the balanced blow-up of the complement of the Fano plane is the asymptotically unique extremal construction. By Lemma \ref{edge approx} and the choice of $\varepsilon$, $G_n$ contains more than $24 \varepsilon^{1/4} n^3$ hyperedges. We can thus apply Lemma~\ref{positive codegree cleanup} to conclude that $G'_n$ contains an $n$-vertex subgraph $G''_n$ with $\delta_2^+(G''_n) \geq \left(\frac{4}{7} - 48 \varepsilon^{1/4}\right)n$. Since $G'_n$ may not be a subgraph of $G_n$, we apply Lemma~\ref{positive codegree cleanup} to the subgraph of $G'_n$ obtained by deleting any $3$-edges that were added to $G_n$ by the application of Theorem~\ref{thm:removal}. Thus, $G''_n$ contains $K_4^3$, say on vertices $v_1,v_2,v_3,v_4$. Hence $\{v_1,v_2,v_3,v_4\}$ also spans $K_4^3$ in $G'_n$. A search through $\mathcal{F}$ shows that there are only two possible subgraphs on $5$ vertices containing $K_4^3$, which we label $A$ and $B$ below. \begin{align*} A&:= \vc{ \begin{tikzpicture}[flag_pic]\outercycle{5}{0} \draw (x0) node[unlabeled_vertex]{};\draw (x1) node[unlabeled_vertex]{};\draw (x2) node[unlabeled_vertex]{};\draw (x3) node[unlabeled_vertex]{};\draw (x4) node[unlabeled_vertex]{}; \labelvertex{0}\labelvertex{1}\labelvertex{2}\labelvertex{3}\labelvertex{4}\drawhyperedge{0}{5} \drawhypervertex{0}{0} \drawhypervertex{1}{0} \drawhypervertex{2}{0} \drawhyperedge{1}{5} \drawhypervertex{0}{1} \drawhypervertex{1}{1} \drawhypervertex{3}{1} \drawhyperedge{2}{5} \drawhypervertex{0}{2} \drawhypervertex{1}{2} \drawhypervertex{4}{2} \drawhyperedge{3}{5} \drawhypervertex{0}{3} \drawhypervertex{2}{3} \drawhypervertex{3}{3} \drawhyperedge{4}{5} \drawhypervertex{0}{4} \drawhypervertex{2}{4} \drawhypervertex{4}{4} \drawhyperedge{5}{5} \drawhypervertex{1}{5} \drawhypervertex{2}{5} \drawhypervertex{3}{5} \drawhyperedge{6}{5} \drawhypervertex{1}{6} \drawhypervertex{2}{6} \drawhypervertex{4}{6} \end{tikzpicture} } & B&:= \vc{ \begin{tikzpicture}[flag_pic]\outercycle{5}{0} \draw (x0) node[unlabeled_vertex]{};\draw (x1) node[unlabeled_vertex]{};\draw (x2) node[unlabeled_vertex]{};\draw (x3) node[unlabeled_vertex]{};\draw (x4) node[unlabeled_vertex]{}; \labelvertex{0}\labelvertex{1}\labelvertex{2}\labelvertex{3}\labelvertex{4}\drawhyperedge{0}{5} \drawhypervertex{0}{0} \drawhypervertex{1}{0} \drawhypervertex{2}{0} \drawhyperedge{1}{5} \drawhypervertex{0}{1} \drawhypervertex{1}{1} \drawhypervertex{3}{1} \drawhyperedge{2}{5} \drawhypervertex{0}{2} \drawhypervertex{1}{2} \drawhypervertex{4}{2} \drawhyperedge{3}{5} \drawhypervertex{0}{3} \drawhypervertex{2}{3} \drawhypervertex{3}{3} \drawhyperedge{4}{5} \drawhypervertex{0}{4} \drawhypervertex{2}{4} \drawhypervertex{4}{4} \drawhyperedge{5}{5} \drawhypervertex{1}{5} \drawhypervertex{2}{5} \drawhypervertex{3}{5} \drawhyperedge{6}{5} \drawhypervertex{1}{6} \drawhypervertex{3}{6} \drawhypervertex{4}{6} \drawhyperedge{7}{5} \drawhypervertex{2}{7} \drawhypervertex{3}{7} \drawhypervertex{4}{7} \end{tikzpicture} } \end{align*} We now partition $V(G'_n)$ into 7 sets $X_1,\ldots,X_7$ as follows. Put $v_i \in X_i$ for $i \in [4]$. For $v \in V(G'_n) \setminus \{v_1,v_2,v_3,v_4 \}$, define $G_v := G_n[v_1,v_2,v_3,v_4,v]$. If $G_v$ is isomorphic to $A$ then put $v \in X_i$ where $d_{G_v}(v,v_i) = 0$ for some $v_i$. If $G_v$ is isomorphic to $B$ then put $v \in X_5$ if $v_1v_2v, v_3v_4v \not\in E(G_v)$ or $v \in X_6$ if $v_1v_3v, v_2v_4v \not\in E(G_v)$ or $v \in X_7$ if $v_1v_4v, v_2v_3v \not\in E(G_v)$. This addresses every vertex $v$, so we have a partition of $V(G'_n)$ into $X_1,\ldots,X_7$. This labeling matches that in Figure~\ref{fig:Fano}. Now, we show that $G'_n$ is in fact a blow-up of the complement of the Fano plane with classes $X_1, \dots, X_7$. \begin{claim}\label{no bad edges} $G'_n$ satisfies the following conditions. (i) For every $i \in [7]$, no edge of $G'_n$ intersects $X_i$ in more than one vertex. (ii) If $a \in X_i, b \in X_j, c \in X_k$ for classes $X_i,X_j,X_k$ which correspond to an edge of the Fano plane as labeled in Figure~\ref{fig:Fano}, then $abc \not\in E(G'_n)$. (iii) If $a \in X_i, b \in X_j, c \in X_k$ for distinct classes $X_i,X_j,X_k$ which do not correspond to an edge of the Fano plane as labeled in Figure~\ref{fig:Fano}, then $abc \in E(G'_n)$. In particular, $G'_n$ is a blow-up of the complement of the Fano plane. \end{claim} \begin{proof} We prove conditions (i), (ii), and (iii) one by one; in each case, we will argue that if the condition is not satisfied, then $G'_n$ must contain some subgraph that is not in $\mathcal{F}$. Throughout, refer to Figure~\ref{J4 positive density set} for the labeled members of $\mathcal{F}$. We implemented this check by a computer to reduce the number of cases needed to be done by hand. For (i), suppose for a contradiction that there exists an edge $abc$ such that $a,b$ are in the same class of $G'_n$. First note that by the definition of $X_1, \dots, X_7$, if $abc$ either intersects $\bigcup_{i=1}^4 X_i$ in at most two vertices or $abc$ is contained in $X_i$ for some $i \in [4]$, then there exists $\{i,j,k\} \subset [4]$ such that two of $N(x_ix_j)$, $N(x_ix_k)$, and $N(x_jx_k)$ contain $a,b,c$. Up to symmetry, we may assume $a,b,c \in N(x_1x_2)$ and $N(x_1x_3)$. The the set of edges of an induced subgraph $H$ of $G'_n$ on $\{x_1,x_2,x_3,a,b,c\}$ includes \[E = \{x_1x_2x_3,\, x_1x_2a,\, x_1x_2b,\, x_1x_2c,\, x_1x_3a,\, x_1x_3b,\, x_1x_3c,\, abc \}.\] Since no graph of $\mathcal{F}$ contains a subset of edges isomorphic to $E$, we have $H \not\in \mathcal{F}$, a contradiction. We use an analogous claim repeatedly. We use computer for verification as well as arguments by hand. Thus, if edge $abc$ exists, we must have $a,b \in X_i$ and $c \in X_j$ for some $i \neq j$ with $i,j \in [4]$. Note that at most one of $a,b,c$ is in $\{x_1,x_2,x_3,x_4\}$, since by the definition of the classes, no edge containing $x_i$ and $x_j$ for $\{i,j\} \in [4]$ intersects $X_i$ or $X_j$. We have (up to symmetry of the classes) two cases. \textbf{Case 1:} $a,b \in X_3$, $c \in X_4$, and $x_3 \not\in \{a,b\}$. Define $H=G'_n[x_1,x_2,x_3, a, b,c]$. The set of edges of $H$ includes \[E = \{x_1x_2x_3,\, x_1x_2a,\, x_1x_2b,\, x_1x_2c,\, x_1x_3c,\, abc \}\] and avoids edges $N=\{x_1x_3a,\, x_1x_3b,\, x_2x_3a,\, x_2x_3b\}$. Since no graph in $\mathcal{F}$ contains a subset of edges isomorphic to $E$ and avoids $N$, we have $H \not\in \mathcal{F}$, a contradiction. \textbf{Case 2:} $a = x_3$, $b \in X_3$, and $c \in X_4$. Consider the subgraph $H$ of $G'_n$ induced on $\{x_1,x_2,x_3,x_4, b,c\}$. The set of edges of $H$ includes \[E = \{x_1x_2x_3,\, x_1x_2x_4,\, x_1x_3x_4,\, x_2x_3x_4,\, x_1x_2b,\, x_1x_4b,\, x_2x_4b,\, x_1x_2c,\, x_1x_3c,\, x_2x_3c,\, abc \}.\] Since no graph in $\mathcal{F}$ contains a subset of edges isomorphic to $E$, we have $H \not\in \mathcal{F}$, a contradiction. For (ii), suppose for a contradiction that there exists an edge $abc$ spanning three classes that correspond to an edge in Figure~\ref{fig:Fano}. Up to symmetry, there are two cases. \textbf{Case 1:} The edge $abc$ intersects $\bigcup_{i = 1}^4 X_i$ in two vertices. Without loss of generality, $a \in X_1, b\in X_4$, and $c \in X_7$. Observe that we cannot have both $a = x_1$ and $b = x_4$ as otherwise $c \not\in X_7$ by definition of the classes, $N(x_1,x_4)$ is disjoint from $X_7$. Without loss of generality, $b \neq x_4$. We consider the subgraph $H$ of $G'_n$ induced on $\{x_2,x_3,x_4, a, b, c\}$. The set of edges of $H$ includes \[E = \{x_2x_3x_4,\, x_2x_3a,\, x_2x_4a,\, x_3x_4a,\, x_2x_3b,\, x_2x_4c,\, x_3x_4c,\, abc \}\] and avoids edges containing both $x_4$ and $b$. Since no graph in $\mathcal{F}$ contains a subset of edges isomorphic to $E$ while avoiding edges containing both $x_4$ and $b$, we have $H \not\in \mathcal{F}$, a contradiction. \textbf{Case 2:} The edge $abc$ does not intersect $\bigcup_{i = 1}^4 X_i$. Without loss of generality, $a \in X_5, b \in X_6,$ and $c \in X_7$. We define the subgraph $H$ of $G'_n$ induced on $\{x_1,x_2,x_3, a,b,c\}$. The set of edges of $H$ includes \[E = \{x_1x_2x_3,\, x_1x_2b,\, x_1x_2c,\, x_1x_3a,\, x_1x_3c,\, x_2x_3a,\, x_2x_3b,\, abc \}\] and avoids edges $N=\{x_1x_2a$, $x_2x_3c$, $x_1x_3b\}$. Since no graph in $\mathcal{F}$ contains a subset of edges isomorphic to $E$ while avoiding $N$ we have $H \not\in \mathcal{F}$, a contradiction. Finally, for (iii), suppose for a contradiction that there exist vertices $a \in X_i, b\in X_j, c \in X_k$ such that $X_i,X_j,X_k$ do not correspond to an edge in Figure~\ref{fig:Fano} and $abc \not\in E(G'_n)$. Up to symmetry of classes, there are three cases. \textbf{Case 1:} We have $i,j,k \in [4]$. Without loss of generality, $a \in X_1, b \in X_2,$ and $ c \in X_3$. By the definition of $X_1,X_2,X_3$, note that at most one of $a,b,c$ is in $\{x_1,x_2,x_3\}$. Without loss of generality, $b \neq x_2$ and $c \neq x_3$. We define the subgraph $H$ of $G'_n$ induced on $\{x_2,x_3,x_4, a, b, c\}$. The set of edges of $H$ includes \[E = \{x_2x_3x_4,\, x_2x_3a,\, x_2x_4a,\, x_3x_4a,\, x_3x_4b,\, x_2x_4c \}\] and avoids edges $N = \{abc,\, x_2x_3b,\, x_2x_4b,\, x_2x_3c,\, x_3x_4c \}$. Since, no graph of $\mathcal{F}$ contains a subset of edges isomorphic to $E$ while avoiding edges in $N$, $H \not\in \mathcal{F}$, a contradiction. \textbf{Case 2:} Precisely two of $i,j,k$ are in $[4]$. Without loss of generality, $a \in X_1, b\in X_2$, and $c \in X_6$. Observe that by the definition of $X_6$, at most one of $a,b$ is in $\{x_1,x_2\}$; without loss of generality, $b \neq x_2$. We define the subgraph $H$ of $G'_n$ induced on $\{x_2,x_3,x_4,a,b,c\}$. The set of edges of $H$ includes \[E = \{x_2x_3x_4,\, x_2x_3a,\, x_2x_4a,\, x_3x_4a,\, x_3x_4b,\, x_2x_3c,\, x_3x_4c \}\] and avoids edges $N = \{abc,\, x_2x_3b,\, x_2x_4b,\, x_2x_4c\}$. Since no graph in $\mathcal{F}$ contains a subset of edges isomorphic to $E$ while avoiding edges in $N$, we have $H \not\in \mathcal{F}$, a contradiction. \textbf{Case 3:} Exactly one of $i,j,k$ is in $[4]$. Without loss of generality, $a \in X_1$, $b \in X_5$, and $c \in X_6$. We define the subgraph $H$ of $G'_n$ induced on $\{x_2,x_3,x_4, a, b, c\}$. The set of edges of $H$ includes \[E = \{x_2x_3x_4,\, x_2x_3a,\, x_2x_4a,\, x_3x_4a,\, x_2x_3b,\, x_2x_4b,\, x_3x_4b,\, x_2x_3c,\, x_3x_4c \}\] and avoids edges $N = \{abc,\, x_2x_4c,\, x_3x_4b \}$. Since, no graph of $\mathcal{F}$ contains a subset of edges isomorphic to $E$ while avoiding edges in $N$, $H \not\in \mathcal{F}$, a contradiction. We conclude that all conditions (i), (ii), and (iii) hold, i.e., $G'_n$ is a blow-up of $\overline{\mathbb{F}}$. \end{proof} Finally, we show that $G'_n$ is almost balanced. Recall that $G'_n$ contains a subgraph $G''_n$ with $\delta_2^{+}(G''_n) \geq (\frac{4}{7} - 48 \varepsilon^{1/4})n$. Fix $\alpha \geq 0$ so that a largest class in $G'_n$ has size at least $\left(\frac{1}{7} + \alpha\right) n$. Without loss of generality, $X_1$ is a largest class. We bound the co-degree in $G''_n$ of pairs containing vertices in $X_1$. Observe that there are three sets of classes disjoint from $X_1$ that appear as neighborhoods of vertex pairs in $G''_n$. Namely, \[N(x_1, x_2) \subseteq X_3 \cup X_4 \cup X_6 \cup X_7;\] \[N(x_1, x_3) \subseteq X_2 \cup X_4 \cup X_5 \cup X_7;\] \[N(x_1, x_4) \subseteq X_2 \cup X_3 \cup X_5 \cup X_6.\] Thus, in $G''_n$, we have \[d(x_1,x_2) + d(x_1,x_3) + d(x_1,x_4) \leq \sum_{i = 2}^7 2|X_i| \leq \left(\frac{12}{7} - 2\alpha\right)n.\] By averaging, one of $d(x_1,x_2), d(x_1,x_3), d(x_1,x_4)$ is at most $\left( \frac{4}{7} - \frac{2\alpha}{3} \right)n$. Thus, $\delta_2^+(G''_n) \leq \frac{4n}{7}$, a contradiction if $\delta_2^{+}(G_n) \geq \left( \frac{4}{7} + \beta\right)n$. We conclude that $\gamma^+(J_4) \leq \frac{4}{7}$. To see that $G''_n$ should be approximately balanced, note that by the minimum positive co-degree condition on $G''_n$, we thus must have $\frac{2\alpha}{3} \leq 48\varepsilon^{1/4}$, i.e., $G''_n$ (and $G'_n$) contains no class of size larger than $\left(\frac{1}{7} + 72 \varepsilon^{1/4}\right)n$. This upper bound implies that $G'_n$ contains no class of size smaller than $\left(\frac{1}{7} - 432 \varepsilon^{1/4}\right)n$. \end{proof} We determine the positive co-degree density and the asymptotically unique extremal construction for $F_{4,2}$. Since the proof is analogous to (but simpler than) the proof of Theorem~\ref{J4}, we only include a sketch. \begin{proof}[Sketch of the proof of Theorem~\ref{J4}] Observe first that $F_{4,2}$ is $6$-partite, so it is not contained in the balanced blow-up of $K_5^3$, which implies $\gamma^+(F_{4,2}) \geq \frac{3}{5}$. We now show that $\gamma^+(F_{4,2}) \leq \frac{3}{5}$. Let $\mathcal{F}$ be the family of seven $6$-vertex $3$-graphs depicted in Figure~\ref{seven-six-graphs}. \begin{figure} \begin{center} \vc{ \begin{tikzpicture}[flag_pic]\outercycle{6}{0} \draw[opacity=0](0,0)--(0,-3.4); \draw (x0) node{\tiny$X_1$} (x1) node{\tiny$X_2$} (x2) node{\tiny$X_3$} (x3) node{\tiny$X_4$} (x4) node{\tiny$X_5$} (x5) node{\tiny$X_5$} ; \drawhyperedge{0}{6} \drawhypervertex{0}{0} \drawhypervertex{1}{0} \drawhypervertex{2}{0} \drawhyperedge{1}{6} \drawhypervertex{0}{1} \drawhypervertex{1}{1} \drawhypervertex{3}{1} \drawhyperedge{2}{6} \drawhypervertex{0}{2} \drawhypervertex{1}{2} \drawhypervertex{4}{2} \drawhyperedge{3}{6} \drawhypervertex{0}{3} \drawhypervertex{1}{3} \drawhypervertex{5}{3} \drawhyperedge{4}{6} \drawhypervertex{0}{4} \drawhypervertex{2}{4} \drawhypervertex{3}{4} \drawhyperedge{5}{6} \drawhypervertex{0}{5} \drawhypervertex{2}{5} \drawhypervertex{4}{5} \drawhyperedge{6}{6} \drawhypervertex{0}{6} \drawhypervertex{2}{6} \drawhypervertex{5}{6} \drawhyperedge{7}{6} \drawhypervertex{0}{7} \drawhypervertex{3}{7} \drawhypervertex{4}{7} \drawhyperedge{8}{6} \drawhypervertex{0}{8} \drawhypervertex{3}{8} \drawhypervertex{5}{8} \drawhyperedge{9}{6} \drawhypervertex{1}{9} \drawhypervertex{2}{9} \drawhypervertex{3}{9} \drawhyperedge{10}{6} \drawhypervertex{1}{10} \drawhypervertex{2}{10} \drawhypervertex{4}{10} \drawhyperedge{11}{6} \drawhypervertex{1}{11} \drawhypervertex{2}{11} \drawhypervertex{5}{11} \drawhyperedge{12}{6} \drawhypervertex{1}{12} \drawhypervertex{3}{12} \drawhypervertex{4}{12} \drawhyperedge{13}{6} \drawhypervertex{1}{13} \drawhypervertex{3}{13} \drawhypervertex{5}{13} \drawhyperedge{14}{6} \drawhypervertex{2}{14} \drawhypervertex{3}{14} \drawhypervertex{4}{14} \drawhyperedge{15}{6} \drawhypervertex{2}{15} \drawhypervertex{3}{15} \drawhypervertex{5}{15} \end{tikzpicture} } \vc{ \begin{tikzpicture}[flag_pic] \draw[opacity=0](0,0)--(0,-3.4); \outercycle{6}{0} \draw (x0) node{\tiny$X_1$} (x1) node{\tiny$X_2$} (x2) node{\tiny$X_3$} (x3) node{\tiny$X_4$} (x4) node{\tiny$X_4$} (x5) node{\tiny$X_4$} ; \drawhyperedge{0}{6} \drawhypervertex{0}{0} \drawhypervertex{1}{0} \drawhypervertex{2}{0} \drawhyperedge{1}{6} \drawhypervertex{0}{1} \drawhypervertex{1}{1} \drawhypervertex{3}{1} \drawhyperedge{2}{6} \drawhypervertex{0}{2} \drawhypervertex{1}{2} \drawhypervertex{4}{2} \drawhyperedge{3}{6} \drawhypervertex{0}{3} \drawhypervertex{1}{3} \drawhypervertex{5}{3} \drawhyperedge{4}{6} \drawhypervertex{0}{4} \drawhypervertex{2}{4} \drawhypervertex{3}{4} \drawhyperedge{5}{6} \drawhypervertex{0}{5} \drawhypervertex{2}{5} \drawhypervertex{4}{5} \drawhyperedge{6}{6} \drawhypervertex{0}{6} \drawhypervertex{2}{6} \drawhypervertex{5}{6} \drawhyperedge{7}{6} \drawhypervertex{1}{7} \drawhypervertex{2}{7} \drawhypervertex{3}{7} \drawhyperedge{8}{6} \drawhypervertex{1}{8} \drawhypervertex{2}{8} \drawhypervertex{4}{8} \drawhyperedge{9}{6} \drawhypervertex{1}{9} \drawhypervertex{2}{9} \drawhypervertex{5}{9} \end{tikzpicture} } \vc{ \begin{tikzpicture}[flag_pic] \draw[opacity=0](0,0)--(0,-3.4); \outercycle{6}{0} \draw (x0) node{\tiny$X_1$} (x1) node{\tiny$X_2$} (x2) node{\tiny$X_3$} (x3) node{\tiny$X_3$} (x4) node{\tiny$X_4$} (x5) node{\tiny$X_4$} ; \drawhyperedge{0}{6} \drawhypervertex{0}{0} \drawhypervertex{1}{0} \drawhypervertex{2}{0} \drawhyperedge{1}{6} \drawhypervertex{0}{1} \drawhypervertex{1}{1} \drawhypervertex{3}{1} \drawhyperedge{2}{6} \drawhypervertex{0}{2} \drawhypervertex{1}{2} \drawhypervertex{4}{2} \drawhyperedge{3}{6} \drawhypervertex{0}{3} \drawhypervertex{1}{3} \drawhypervertex{5}{3} \drawhyperedge{4}{6} \drawhypervertex{0}{4} \drawhypervertex{2}{4} \drawhypervertex{3}{4} \drawhyperedge{5}{6} \drawhypervertex{0}{5} \drawhypervertex{2}{5} \drawhypervertex{4}{5} \drawhyperedge{6}{6} \drawhypervertex{0}{6} \drawhypervertex{3}{6} \drawhypervertex{5}{6} \drawhyperedge{7}{6} \drawhypervertex{0}{7} \drawhypervertex{4}{7} \drawhypervertex{5}{7} \drawhyperedge{8}{6} \drawhypervertex{1}{8} \drawhypervertex{2}{8} \drawhypervertex{3}{8} \drawhyperedge{9}{6} \drawhypervertex{1}{9} \drawhypervertex{2}{9} \drawhypervertex{4}{9} \drawhyperedge{10}{6} \drawhypervertex{1}{10} \drawhypervertex{3}{10} \drawhypervertex{5}{10} \drawhyperedge{11}{6} \drawhypervertex{1}{11} \drawhypervertex{4}{11} \drawhypervertex{5}{11} \end{tikzpicture} } \vc{ \begin{tikzpicture}[flag_pic] \draw[opacity=0](0,0)--(0,-3.4); \outercycle{6}{0} \draw (x0) node{\tiny$X_1$} (x1) node{\tiny$X_1$} (x2) node{\tiny$X_2$} (x3) node{\tiny$X_2$} (x4) node{\tiny$X_3$} (x5) node{\tiny$X_3$} ; \drawhyperedge{0}{6} \drawhypervertex{0}{0} \drawhypervertex{1}{0} \drawhypervertex{2}{0} \drawhyperedge{1}{6} \drawhypervertex{0}{1} \drawhypervertex{1}{1} \drawhypervertex{3}{1} \drawhyperedge{2}{6} \drawhypervertex{0}{2} \drawhypervertex{2}{2} \drawhypervertex{4}{2} \drawhyperedge{3}{6} \drawhypervertex{0}{3} \drawhypervertex{3}{3} \drawhypervertex{4}{3} \drawhyperedge{4}{6} \drawhypervertex{1}{4} \drawhypervertex{2}{4} \drawhypervertex{5}{4} \drawhyperedge{5}{6} \drawhypervertex{1}{5} \drawhypervertex{3}{5} \drawhypervertex{5}{5} \drawhyperedge{6}{6} \drawhypervertex{2}{6} \drawhypervertex{4}{6} \drawhypervertex{5}{6} \drawhyperedge{7}{6} \drawhypervertex{3}{7} \drawhypervertex{4}{7} \drawhypervertex{5}{7} \end{tikzpicture} } \vc{ \begin{tikzpicture}[flag_pic] \draw[opacity=0](0,0)--(0,-1.4); \outercycle{6}{0} \draw (x0) node{\tiny$X_1$} (x1) node{\tiny$X_2$} (x2) node{\tiny$X_3$} (x3) node{\tiny$X_3$} (x4) node{\tiny$X_3$} (x5) node{\tiny$X_3$} ; \drawhyperedge{0}{6} \drawhypervertex{0}{0} \drawhypervertex{1}{0} \drawhypervertex{2}{0} \drawhyperedge{1}{6} \drawhypervertex{0}{1} \drawhypervertex{1}{1} \drawhypervertex{3}{1} \drawhyperedge{2}{6} \drawhypervertex{0}{2} \drawhypervertex{1}{2} \drawhypervertex{4}{2} \drawhyperedge{3}{6} \drawhypervertex{0}{3} \drawhypervertex{1}{3} \drawhypervertex{5}{3} \end{tikzpicture} } \vc{ \begin{tikzpicture}[flag_pic] \draw[opacity=0](0,0)--(0,-1.4); \outercycle{6}{0} \draw (x0) node{\tiny$X_1$} (x1) node{\tiny$X_2$} (x2) node{\tiny$X_2$} (x3) node{\tiny$X_3$} (x4) node{\tiny$X_3$} (x5) node{\tiny$X_3$} ; \drawhyperedge{0}{6} \drawhypervertex{0}{0} \drawhypervertex{1}{0} \drawhypervertex{2}{0} \drawhyperedge{1}{6} \drawhypervertex{0}{1} \drawhypervertex{1}{1} \drawhypervertex{3}{1} \drawhyperedge{2}{6} \drawhypervertex{0}{2} \drawhypervertex{1}{2} \drawhypervertex{4}{2} \drawhyperedge{3}{6} \drawhypervertex{0}{3} \drawhypervertex{2}{3} \drawhypervertex{5}{3} \drawhyperedge{4}{6} \drawhypervertex{0}{4} \drawhypervertex{3}{4} \drawhypervertex{5}{4} \drawhyperedge{5}{6} \drawhypervertex{0}{5} \drawhypervertex{4}{5} \drawhypervertex{5}{5} \end{tikzpicture} } \vc{ \begin{tikzpicture}[flag_pic] \draw[opacity=0](0,0)--(0,-1.4); \outercycle{6}{0} \draw (x0) node{\tiny$X_1$} (x1) node{\tiny$X_1$} (x2) node{\tiny$X_1$} (x3) node{\tiny$X_2$} (x4) node{\tiny$X_2$} (x5) node{\tiny$X_2$} ; \drawhyperedge{0}{6} \end{tikzpicture} } \end{center} \caption{The family $\mathcal{F}$ of seven $6$-vertex $3$-graphs.}\label{seven-six-graphs} \end{figure} Observe that $\mathcal{F}$ consists of the empty $3$-graph and the $6$-vertex blow-ups of $K_3^3$, $K_3^4$, and $K_5^3$. Thus, $\mathcal{F}$ is exactly the set of $6$-vertex induced subgraphs of the balanced blow-up of $K_5^3$. Using flag algebras, we show that for sufficiently large $n$, if $G_n$ is a $F_{4,2}$-free $n$-vertex 3-graph with $\delta_2^+(G_n) \geq \frac{3n}{5}$, then the 6-vertex 3-graphs with positive density in $G_n$ are in $\mathcal{F}$. The details of the calculations by computer are available at \oururl. Using the induced removal lemma (Theorem~\ref{thm:removal}), $G_n$ has a small edit distance to $G'_n$, where every 6-vertex subgraph vertices belongs to $\mathcal{F}$. Recall that $\gamma^+(K_4^3) \leq 0.543 < \frac{3}{5}$, so by Lemma~\ref{positive codegree cleanup}, $G'_n$ contains a subgraph with minimum positive co-degree larger than $(0.543 + \varepsilon)n$ for some $\varepsilon > 0$. Thus, we can assume $G'_n$ contains $K_4^3$, say on vertices $v_1,v_2,v_3,v_4$. A search through $\mathcal{F}$ shows that there are only two possible subgraphs on $5$ vertices containing $K_4^3$. \begin{align*} A&:= \vc{ \begin{tikzpicture}[flag_pic]\outercycle{5}{0} \draw (x0) node[unlabeled_vertex]{};\draw (x1) node[unlabeled_vertex]{};\draw (x2) node[unlabeled_vertex]{};\draw (x3) node[unlabeled_vertex]{};\draw (x4) node[unlabeled_vertex]{}; \labelvertex{0}\labelvertex{1}\labelvertex{2}\labelvertex{3}\labelvertex{4}\drawhyperedge{0}{5} \drawhypervertex{0}{0} \drawhypervertex{1}{0} \drawhypervertex{2}{0} \drawhyperedge{1}{5} \drawhypervertex{0}{1} \drawhypervertex{1}{1} \drawhypervertex{3}{1} \drawhyperedge{2}{5} \drawhypervertex{0}{2} \drawhypervertex{1}{2} \drawhypervertex{4}{2} \drawhyperedge{3}{5} \drawhypervertex{0}{3} \drawhypervertex{2}{3} \drawhypervertex{3}{3} \drawhyperedge{4}{5} \drawhypervertex{0}{4} \drawhypervertex{2}{4} \drawhypervertex{4}{4} \drawhyperedge{5}{5} \drawhypervertex{1}{5} \drawhypervertex{2}{5} \drawhypervertex{3}{5} \drawhyperedge{6}{5} \drawhypervertex{1}{6} \drawhypervertex{2}{6} \drawhypervertex{4}{6} \end{tikzpicture} } & B&:= \vc{ \begin{tikzpicture}[flag_pic]\outercycle{5}{0} \draw (x0) node[unlabeled_vertex]{};\draw (x1) node[unlabeled_vertex]{};\draw (x2) node[unlabeled_vertex]{};\draw (x3) node[unlabeled_vertex]{};\draw (x4) node[unlabeled_vertex]{}; \labelvertex{0}\labelvertex{1}\labelvertex{2}\labelvertex{3}\labelvertex{4} \drawhyperedge{0}{5} \drawhypervertex{0}{0} \drawhypervertex{1}{0} \drawhypervertex{2}{0} \drawhyperedge{1}{5} \drawhypervertex{0}{1} \drawhypervertex{1}{1} \drawhypervertex{3}{1} \drawhyperedge{2}{5} \drawhypervertex{0}{2} \drawhypervertex{1}{2} \drawhypervertex{4}{2} \drawhyperedge{3}{5} \drawhypervertex{0}{3} \drawhypervertex{2}{3} \drawhypervertex{3}{3} \drawhyperedge{4}{5} \drawhypervertex{0}{4} \drawhypervertex{2}{4} \drawhypervertex{4}{4} \drawhyperedge{5}{5} \drawhypervertex{1}{5} \drawhypervertex{2}{5} \drawhypervertex{3}{5} \drawhyperedge{6}{5} \drawhypervertex{1}{6} \drawhypervertex{3}{6} \drawhypervertex{4}{6} \drawhyperedge{7}{5} \drawhypervertex{2}{7} \drawhypervertex{3}{7} \drawhypervertex{4}{7} \drawhyperedge{8}{5} \drawhypervertex{1}{8} \drawhypervertex{2}{8} \drawhypervertex{4}{8} \drawhyperedge{9}{5} \drawhypervertex{0}{9} \drawhypervertex{3}{9} \drawhypervertex{4}{9} \end{tikzpicture} } \end{align*} Note that $A$ is the (unique) $5$-vertex blow-up of $K_4^3$ and $B = K_5^3$. We now partition $V(G'_n)$ into five sets $X_1,\ldots,X_5$ as follows. Put $v_i \in X_i$ for $i \in [4]$. For $v \in V(G'_n) \setminus \{v_1,v_2,v_3,v_4 \}$, define $G_v := G_n[v_1,v_2,v_3,v_4,v]$. If $G_v$ is isomorphic to $A$, then put $v \in X_i$, where $d_{G_v}(v,v_i) = 0$. If $G_v$ is isomorphic to $B$, then put $v \in X_5$. An inspection of cases establishes the following claim. \begin{claim} $G'_n$ satisfies the following conditions.\\ (i) For every $i \in [5]$, no edge of $G'_n$ intersects $X_i$ in more than one vertex.\\ (ii) For every $a \in X_i, b \in X_j,$ and $c \in X_k$ with $i,j,k$ pairwise distinct, $abc \in E(G'_n)$. \end{claim} In particular, $G'_n$ is a blow-up of $K_5^3$. From here, the positive co-degree condition can be used to establish that the vertex-partition of $G_n'$ is essentially balanced. \end{proof} Finally, Theorem~\ref{thm:FlagDensity} was proved using flag algebras. The certificates for the proofs are available at \oururl. These bounds are unlikely to be tight. \section{Concluding remarks}\label{conclusion} While we significantly expand the known sets of jumps and achievable values for $\gamma^+$, the general behavior of $\gamma^+$ remains mysterious, even for $r = 3$. It is unclear whether $\gamma^+$ has a jump everywhere, though we conjecture that more jumps exist than are characterized in Theorem~\ref{general jumps}. \begin{ques}\label{jump question} For $r \geq 3$, which values of $\alpha \in [\frac{2}{2r - 1}, 1]$ are $\gamma^+$-jumps? Does there exist an $\alpha \in [\frac{2}{2r - 1}, 1]$ which is \textit{not} a $\gamma^+$-jump? \end{ques} Many more concrete questions could be asked when $r=3$. For example, it is unclear, how far Theorem~\ref{densities} is from completely characterizing achievable densities in the range $[0,\frac{1}{2}]$ when $r=3$. \begin{ques}\label{turan suspension} Are there achievable values of $\gamma^+$ in $[\frac{2}{5}, \frac{1}{2}]$ that are not of the form $\frac{k-2}{2k - 3}$, when $r=3$? \end{ques} A negative answer to Question~\ref{turan suspension} would suggest some similarity between $\gamma^+$ and $\pi$, since the extremal constructions in Theorem~\ref{densities} are a fairly natural analogue of Tur\'an graphs. However, note that there is no hope for an Erd\H{o}s-Stone-Simonovits-type result giving values of $\gamma^+$. Indeed, since a balanced blow-up of $K_4^3$ has positive co-degree density $\frac{1}{2}$, every $3$-graph $F$ with $0 < \gamma^+(F) < \frac{1}{2}$ is $4$-partite. To begin addressing either Question~\ref{jump question} or \ref{turan suspension}, it seems natural to start with the next interval between known achievable values. Even this next case seems difficult. \begin{ques} For $r = 3$, is every $\alpha \in [\frac{2}{5}, \frac{3}{7})$ a $\gamma^+$-jump? Is there a family $\mathcal{F}$ with $\gamma^+(\mathcal{F}) \in \left(\frac{2}{5}, \frac{3}{7}\right)$? \end{ques} The use of flag algebras introduces the potential for new approaches to positive co-degree questions, particularly when combined with proving the existence of $\gamma^+$-jumps. As illustrated by Theorems~\ref{J4} and \ref{F42density}, flag algebra calculations have the potential to directly determine values of $\gamma^+$, and even inexact bounds (e.g., $\gamma^+(K_4^3) \leq 0.543$) given by flag algebras are sometimes substantially better and more useful than what seems tractable by hand. When combined with known jumps of the function $\gamma^+$, flag algebra bounds also have the potential to produce exact results. For instance, any $3$-graph $F$ that can be shown via a flag calculation to have $\gamma^+(F) < \frac{2}{5}$ must have $\gamma^+(F) \in \left\{0, \frac{1}{3}\right\}$. Thus, obtaining estimates via flags and ``rounding down'' via known jumps is a very efficient way to determine the densities of many small $3$-graphs. Since we can directly characterize those $3$-graphs with positive co-degree density in $\left\{0, \frac{1}{3}\right\}$, it is now also possible to directly determine by inspection whether a fixed $3$-graph $F$ has $\gamma^+(F) \in \left\{0, \frac{1}{3}\right\}$; however, it seems unlikely that the set \[\mathcal{F}(r,d) := \{F \text{ an $r$-graph} : \gamma^+(F) = d\}\] can be as simply characterized for other values of $d$, even when $r = 3$. We are interested to see whether further understanding of jumps will include characterizations of this type. If they do not, the potential combination of estimated densities with the theory of jumps is an appealing approach to determining densities exactly. It is also open whether every achievable value of $\gamma^+$ can be achieved as the density of a single $r$-graph. Theorem~\ref{single graph} shows that $\frac{2}{5}$ is achievable by a single $3$-graph, as is every achievable density known outside the interval $\left(\frac{2}{5}, \frac{1}{2}\right)$. \begin{ques} For a fixed $k \geq 5$, is there a $3$-graph $F_k$ such that $\gamma^+(F_k) = \frac{k-2}{2k-3}$? More generally, if $\mathcal{F}$ is a family of $r$-graphs with $\gamma^+(\mathcal{F}) = \alpha$, does there always exist a single $r$-graph $F$ with $\gamma^+(F) = \alpha$? \end{ques} Every $3$-graph that is not the subgraph of a (blow-up of a) suspension will have positive co-degree density at least $\frac{1}{2}$. The complete list of known achievable densities at least $\frac{1}{2}$ is as follows: $\frac{1}{2}$ (achieved by $F_{3,2}$), $\frac{4}{7}$ (achieved by $J_4$), $\frac{3}{5}$ (achieved by $F_{4,2}$), and $\frac{2}{3}$ (achieved by the Fano plane). \begin{ques} For $r= 3$, find other achievable values for $\gamma^+$ larger than $\frac{1}{2}$. Is there an $\alpha \in \left[\frac{1}{2}, 1\right)$ which is a $\gamma^+$-jump? Is there an $\alpha \in \left[\frac{1}{2}, 1\right)$ for which we can characterize the $3$-graphs with $\gamma^+(F) = \alpha$? \end{ques} Very little is known about the $\gamma^+$ function for $r$-graphs when $r \geq 4$. A natural starting point would be to study extensions of $3$-graphs whose positive co-degree densities are known. For example, the \textit{r-daisy} $\mathcal{D}_r$ is the $6$-edge $r$-graph on $r+2$ vertices whose all six edges contain the same $r-2$ vertices and each pair of the remaining 4 vertices is in one edge. There was a recent breakthrough on the Tur\'an density of $r$-daisies~\cite{ellis2024daisies}. Note that $J_4$ is the $3$-daisy. \begin{ques} What is $\gamma^+(\mathcal{D}_r)$ for $r \geq 4$? \end{ques} \section{Acknowledgments} This work used computing resources at the Center for Computational Mathematics, University of Colorado Denver, including the Alderaan cluster, supported by the National Science Foundation award OAC-2019089. The authors thank Jan Volec for a discussion on Theorem~\ref{J4}. Part of the work was done while the first and third authors were at AIM. \bibliographystyle{abbrvurl} \bibliography{references.bib} \end{document} \begin{tikzpicture}[scale=0.8] \draw (-2,0) coordinate(1) node[vtx,label=left:{\tiny $1$}](b){} (45:1) coordinate(2) node[vtx,label=right:{\tiny $2$}](c){} (135:1) coordinate(3) node[vtx,label=left:{\tiny $3$}](d){} (225:1) coordinate(4) node[vtx,label=left:{\tiny $4$}](d){} (315:1) coordinate(5) node[vtx,label=right:{\tiny $5$}](d){} ; \draw[hyperedge] (1) to[out=70,in=140,looseness=0.9] (2) to(3) to[out=120,in=70,looseness=0.8] (1); \draw[hyperedge] (1) to[out=0,in=250,looseness=1.3] (3) to(4) to[out=110,in=0,looseness=1.3] (1); \draw[hyperedge] (1) to[out=-70,in=240,looseness=0.8] (4) to(5) to[out=220,in=-70,looseness=0.9] (1); \draw[hyperedge] (1) to[out=0,in=110,looseness=0.7] (5) to(2) to[out=250,in=0,looseness=0.7] (1); \draw[hyperedge] (1) to[out=-25,in=205,looseness=0.8] (2) to(4) to[out=110,in=-25,looseness=0.9] (1); \draw[hyperedge] (1) to[out=25,in=250,looseness=0.9] (3) to(5) to[out=155,in=25,looseness=0.8] (1); \draw[line width = 0.5pt] (3)--(4)--(5)--(2)--(4) (2)--(3)--(5) ; \end{tikzpicture} \begin{tikzpicture}[scale=0.8] \draw (-2,0) coordinate(1) node[vtx,label=left:{\tiny $1$}](b){} (45:1) coordinate(2) node[vtx,label=right:{\tiny $2$}](c){} (135:1) coordinate(3) node[vtx,label=left:{\tiny $3$}](d){} (225:1) coordinate(4) node[vtx,label=left:{\tiny $4$}](d){} (315:1) coordinate(5) node[vtx,label=right:{\tiny $5$}](d){} ; \draw[hyperedge] (1) to[out=70,in=140,looseness=0.9] (2) to[out=140,in=60,looseness=0.8] (3) to[out=120,in=70,looseness=0.8] (1); \draw[hyperedge] (1) to[out=0,in=250,looseness=1.3] (3) to[out=250,in=110] (4) to[out=110,in=0,looseness=1.3] (1); \draw[hyperedge] (1) to[out=-70,in=240,looseness=0.8] (4) to[out=300,in=220,looseness=0.8] (5) to[out=220,in=-70,looseness=0.9] (1); \draw[hyperedge] (1) to[out=0,in=110,looseness=0.7] (5) to[out=110,in=250,looseness=1.1] (2) to[out=250,in=0,looseness=0.7] (1); \draw[hyperedge] (1) to[out=-25,in=205,looseness=0.8] (2) to[out=205,in=110,looseness=0.8] (4) to[out=110,in=-25,looseness=0.9] (1); \draw[hyperedge] (1) to[out=25,in=250,looseness=0.9] (3) to[out=250,in=155,looseness=0.8] (5) to[out=155,in=25,looseness=0.8] (1); \draw[line width = 0.5pt] (3) to[out=250,in=155,looseness=0.8] (5) (2) to[out=205,in=110,looseness=0.8] (4) (5) to[out=110,in=250,looseness=1.1] (2) (4) to[out=300,in=220,looseness=0.8] (5) (3) to[out=250,in=110] (4) (2) to[out=140,in=60,looseness=0.8] (3) ; \end{tikzpicture} \end{document} \section*{Appendix} \begin{claim}\label{no bad edges-OLD} $G'_n$ satisfies the following conditions. \begin{enumerate}[label=(\roman*),itemsep=1pt, parsep=0pt] \item For every $i \in [7]$, no edge of $G'_n$ intersects $X_i$ in more than one vertex. \item If $a \in X_i, b \in X_j, c \in X_k$ for classes $X_i,X_j,X_k$ that correspond to an edge of the Fano plane as labeled in Figure~\ref{fig:Fano}, then $abc \not\in E(G'_n)$. \item If $a \in X_i, b \in X_j, c \in X_k$ for distinct classes $X_i,X_j,X_k$ that do not correspond to an edge of the Fano plane as labeled in Figure~\ref{fig:Fano}, then $abc \in E(G'_n)$. \end{enumerate} In particular, $G'_n$ is a blow-up of the complement of the Fano plane. \end{claim} \begin{proof} We consider conditions (i), (ii), and (iii) one by one; in each case, we will argue that if the condition is not satisfied, then $G'_n$ must contain some subgraph that is not in $\mathcal{F}$. Throughout, refer to Figure~\ref{J4 positive density set} for the labeled members of $\mathcal{F}$. For (i), suppose for a contradiction that there exists an edge $abc$ such that $a,b$ are in the same class of $G'_n$. First note that by the definition of $X_1, \dots, X_7$, if $abc$ either intersects $\bigcup_{i=1}^4 X_i$ in at most two vertices or $abc$ is contained in $X_i$ for some $i \in [4]$, then there exists $\{i,j,k\} \subset [4]$ such that two of $N(x_ix_j)$, $N(x_ix_k)$, and $N(x_jx_k)$ contain $a,b,c$. Up to symmetry, we may assume $a,b,c \in N(x_1x_2)$ and $N(x_1x_3)$. We observe two properties of the subgraph $H$ of $G'_n$ induced on $\{x_1,x_2,x_3,a,b,c\}$: every pair of vertices in $H$ has positive co-degree in $H$, and the pairs $x_1,x_2$ and $x_1,x_3$ both have co-degree $4$ in $H$. The only member of $\mathcal{F}$ in which all pairs of vertices have positive co-degree is $F_4$, so if $H \in \mathcal{F}$, then $H = F_4$. However, observe that in $F_4$, each vertex is contained in precisely one pair with co-degree $4$, while $x_1$ is contained in two pairs with co-degree $4$ in $H$. Thus, $H \not\in \mathcal{F}$, a contradiction. Thus, if edge $abc$ exists, we must have $a,b \in X_i$ and $c \in X_j$ for some $i \neq j$ with $i,j \in [4]$. We have (up to symmetry of the classes) two cases. \textbf{Case 1:} $a,b \in X_3$, $c \in X_4$, and $x_3 \not\in \{a,b\}$. Define the subgraph $H$ of $G'_n$ induced on $\{x_1,x_2,x_3, a, b,c\}$. Suppose for a contradiction that $H \in \mathcal{F}$. Observe that by the definition of $X_1, X_2, X_3,X_4$, it follows that every pair from $\{x_1,x_2,a,b,c\}$ has positive co-degree in $H$ and that $x_ix_3a$, $x_ix_3b$ are not edges for any $i \in [4]$. The only members of $\mathcal{F}$ which contain five vertices with pairwise positive co-degree are $F_1, F_3,$ and $F_4$. In each of these subgraphs, every vertex has positive co-degree with either 4 or 5 vertices, and every pair with positive co-degree has co-degree at least 2. However, observe that both $d(x_3,a) \leq 1$ and $d(x_3, b) \leq 1$, so either $x_3$ has positive co-degree with only $3$ vertices in $H$, or $H$ contains a pair with co-degree exactly 1. In either case, we conclude that $H \not\in \mathcal{F}$. \textbf{Case 2:} $a = x_3$, $b \in X_3$, and $c \in X_4$. Consider the subgraph $H$ of $G'_n$ induced on $\{x_1,x_2,x_3,x_4, b,c\}$. Suppose for a contradiction that $H \in \mathcal{F}$. Note that by the definition of $X_1, X_2, X_3,X_4$, it follows that every pair from $\{x_1,x_2,x_3,x_4,b\}$ has positive co-degree in $H$ and that $b,x_3$ have co-degree exactly $1$ in $H$. Thus, as in the previous case, we conclude that $H \not\in \mathcal{F}$, a contradiction. For (ii), suppose for a contradiction that there exists an edge $abc$ spanning three classes that correspond to an edge in Figure~\ref{fig:Fano}. Up to symmetry, there are two cases. \textbf{Case 1:} The edge $abc$ intersects $\bigcup_{i = 1}^4 X_i$ in two vertices. Without loss of generality, $a \in X_1, b\in X_4$, and $c \in X_7$. Observe that we cannot have both $a = x_1$ and $b = x_4$ as otherwise $c \not\in X_7$. Without loss of generality, $b \neq x_4$. We consider the subgraph $H$ of $G'_n$ induced on $\{x_2,x_3,x_4, a, b, c\}$. Suppose for a contradiction that $H \in \mathcal{F}$. By (i), $x_4$ and $b$ have co-degree $0$ in $H$; by the definition of the $X_is$ and the fact that $abc \in E(G'_n)$, all other pairs of vertices in $H$ have positive co-degree. Observe also that $\{x_2,x_3,x_4, a, b\}$ spans a blow-up of $K_4^3$, with $x_4$ and $b$ in the same blow-up class. The unique member of $\mathcal{F}$ that contains both five vertices with pairwise positive co-degree and a 5-vertex blow-up of $K_4^3$ is $F_1$. We claim that $F_1$ is not isomorphic to $H$. Indeed, consider the link graph of $c$ in $H$. Since $c \in X_7$, we have $x_2x_3c \not \in E(G'_n)$. If $H = F_1$, then the link graph of $c$ in $H$ avoids a matching between the classes $\{ \{x_2,b\},\{x_3\}, \{x_4 \}, \{a\} \}$. Thus, it must be that $ax_2c$ and $abc$ are not $3$-edges, a contradiction, as we assume $abc \in E(G'_n)$. \textbf{Case 2:} The edge $abc$ does not intersect $\bigcup_{i = 1}^4 X_i$. Without loss of generality, $a \in X_5, b \in X_6,$ and $c \in X_7$. We define the subgraph $H$ of $G'_n$ induced on $\{x_1,x_2,x_3, a,b,c\}$. Suppose for a contradiction that $H \in \mathcal{F}$. It follows from the definitions of $X_5,X_6,X_7$, and the fact that $x_1x_2x_3, abc \in E(G'_n)$, that all pairs of vertices in $H$ have positive co-degree. As noted above, $F_4$ is the unique subgraph of $\mathcal{F}$ in which all 6 vertices have pairwise positive co-degree. Moreover, in $F_4$, the vertices can be partitioned into three pairs so that each pair has co-degree 4, and any pair of vertices not included in this partition has co-degree 3. We attempt to find such a partition of $V(H)$. By the definitions of $X_5,X_6,X_7$, we know that $x_1x_2a, x_1x_3b$, and $x_2x_3c$ are not edges of $G'_n$. It follows that the only possible partition of $V(H)$ is $\{\{x_1,c\}, \{x_2,b\}, \{x_3,a\} \}$. In particular, $d(a,b) = 3$. Since $abc \in E(G'_n)$ and $d(x_2,b) = d(x_3,a) = 4$ implies that $abx_2, abx_3 \in E(G'_n)$, we must have $abx_1 \not \in E(G'_n)$. However, this implies that $d(x_1,a) \leq 2$, a contradiction. Finally, for (iii), suppose for a contradiction that there exist vertices $a \in X_i, b\in X_j, c \in X_k$ such that $X_i,X_j,X_k$ do not correspond to an edge in Figure~\ref{fig:Fano} and $abc \not\in E(G'_n)$. Up to symmetry of classes, there are three cases. \textbf{Case 1:} We have $i,j,k \in [4]$. Without loss of generality, $a \in X_1, b \in X_2,$ and $ c \in X_3$. By the definition of $X_1,X_2,X_3$, note that at most one of $a,b,c$ is in $\{x_1,x_2,x_3\}$. Without loss of generality, $b \neq x_2$ and $c \neq x_3$. We define the subgraph $H$ of $G'_n$ induced on $\{x_2,x_3,x_4, a, b, c\}$. Suppose for a contradiction that $H \in \mathcal{F}$. Observe that $H$ contains a $K_4^3$ (on $\{a,x_2,x_3,x_4\}$) and by (i), both $d(x_2,b)$ and $d(x_3,c)$ are $0$. Observe that every graph $F \in \mathcal{F}$ is a blow-up with the property that if two vertices $x,y \in V(F)$ have co-degree 0, then they are clones of one another, i.e., for $u,v \in V(F)$, $xuv$ is an edge if and only if $yuv$ is an edge. Thus, if $H \in \mathcal{F}$, then $b$ is a clone of $x_2$ in $H$ and $c$ is a clone of $x_3$ in $H$. However, since $ax_2x_3 \in E(G'_n)$ since $a \in X_1$, this implies that $abc \in E(G'_n)$, a contradiction. \textbf{Case 2:} Precisely two of $i,j,k$ are in $[4]$. Without loss of generality, $a \in X_1, b\in X_2$, and $c \in X_6$. Observe that by the definition of $X_6$, at most one of $a,b$ is in $\{x_1,x_2\}$; without loss of generality, $b \neq x_2$. We define the subgraph $H$ of $G'_n$ induced on $\{x_2,x_3,x_4,a,b,c\}$. Suppose for a contradiction that $H \in \mathcal{F}$. In the previous case we established that all possible edges spanning three classes from $X_1,X_2,X_3,X_4$ are present, so $\{x_2,x_3,x_4,a,b\}$ spans a blow-up of $K_4^3$. Note that $d(x_2,b) = 0$ by (i); as noted in the previous case, this implies that if $H \in \mathcal{F}$, then $b$ is a clone of $x_2$ in $H$. In particular, $ax_2c \not \in E(G'_n)$. By (ii), $x_3ac \not \in E(G'_n)$, thus, $d(a,c) \leq 1$ in $H$. If $d(a,c) = 0$, then $a$ is a clone of $c$, a contradiction since $x_2x_4a \in E(G'_n)$ since $a \in X_1$ and $x_2x_4c \not \in E(G'_n)$ since $c \in X_6$. If $d(a,c) = 1$, then each pair from $\{x_2,x_3,x_4,a,c\}$ has positive co-degree. Observe that in every member of $\mathcal{F}$ containing five vertices with pairwise positive co-degree, the minimum positive co-degree is at least 2, a contradiction. \textbf{Case 3:} Exactly one of $i,j,k$ is in $[4]$. Without loss of generality, $a \in X_1$, $b \in X_5$, and $c \in X_6$. We define the subgraph $H$ of $G'_n$ induced on $\{x_2,x_3,x_4, a, b, c\}$. By the previous two cases, every pair of vertices from $\{x_2,x_3,x_4, a, b\}$ has positive co-degree, and $c$ must have positive co-degree with every vertex in $\{x_2,x_3,x_4,a\}$. Thus, either every pair of vertices in $H$ has positive co-degree, of $d(b,c) = 0$ and $c$ is a clone of $b$. By (ii) and the previous case, $c$ cannot be a clone of $b$, because $x_2ab \not\in G'_n$ but $x_2 a c \in E(G'_n)$. Thus, all pairs of vertices in $H$ have positive co-degree. However, as previously noted, in $F_4$ (the unique element of $\mathcal{F}$ containing 6 vertices with pairwise positive co-degree) the minimum co-degree is 3. By (ii), we have $x_2ab \not\in E(G'_n)$ and we assume $abc \not\in E(G'_n)$, so $d(a,b) \leq 2$, a contradiction. We conclude that all conditions (i), (ii), and (iii) hold, i.e., $G'_n$ is a blow-up of $\overline{\mathbb{F}}$. \end{proof} \end{document}
2412.08595v1
http://arxiv.org/abs/2412.08595v1
Numerical Analysis of HiPPO-LegS ODE for Deep State Space Models
\documentclass{article} \usepackage{arxiv} \usepackage[utf8]{inputenc} \usepackage[T1]{fontenc} \usepackage[hidelinks]{hyperref} \usepackage{url} \usepackage{booktabs} \usepackage{amsfonts} \usepackage{nicefrac} \usepackage{microtype} \usepackage{lipsum} \usepackage{graphicx} \usepackage{natbib} \usepackage{doi} \usepackage{amsfonts} \usepackage{stmaryrd} \usepackage{amsmath,amssymb} \usepackage{mathtools} \usepackage{amsthm} \usepackage{enumitem} \usepackage{cancel} \usepackage{xcolor} \newtheorem{theorem}{Theorem} \newtheorem{lemma}[theorem]{Lemma} \newtheorem{corollary}[theorem]{Corollary} \newtheorem{proposition}[theorem]{Proposition} \theoremstyle{definition} \newtheorem{definition}{Definition}[section] \newtheorem{conjecture}{Conjecture}[section] \newtheorem{example}{Example}[section] \newtheorem{aim}{Aim}[section] \newtheorem{question}{Question}[section] \newtheorem{task}{Task} \newtheorem{observation}{Observation}[section] \newcommand{\pr}[1]{ \left( #1 \right) } \newcommand{\inner}[2]{\langle #1, #2 \rangle} \newcommand{\derivative}[1]{#1^{\prime}} \title{Numerical Analysis of HiPPO-LegS ODE for\\ Deep State Space Models } \author{ \normalfont \begin{tabular}{@{}c@{\hspace{1em}}c@{\hspace{-1.3em}}c@{}} \textbf{Jaesung R. Park} & \textbf{Jaewook J. Suh} & \textbf{Ernest K. Ryu} \\ Mathematical Sciences & Comp.\ Applied Math.\ \& Oper.\ Res.\ & Mathematics \\ Seoul National University & Rice University & University of California, Los Angeles \\ \texttt{[email protected]} & \texttt{[email protected]} & \texttt{[email protected]} \end{tabular} } \date{} \renewcommand{\headeright}{ } \renewcommand{\shorttitle}{ } \hypersetup{ pdftitle={LegS arXiv}, pdfsubject={ }, pdfauthor={Jaesung R. Park, Jaewook J. Suh, Ernest K. Ryu}, pdfkeywords={ }, } \begin{document} \maketitle \begin{abstract} In deep learning, the recently introduced state space models utilize HiPPO (High-order Polynomial Projection Operators) memory units to approximate continuous-time trajectories of input functions using ordinary differential equations (ODEs), and these techniques have shown empirical success in capturing long-range dependencies in long input sequences. However, the mathematical foundations of these ODEs, particularly the singular HiPPO-LegS (Legendre Scaled) ODE, and their corresponding numerical discretizations remain unexplored. In this work, we fill this gap by establishing that HiPPO-LegS ODE is well-posed despite its singularity, albeit without the freedom of arbitrary initial conditions, and by establishing convergence of the associated numerical discretization schemes for Riemann-integrable input functions. \end{abstract} \section{Introduction} \label{sec:intro} State-space representation is a classical mathematical framework widely used to model physical systems in diverse fields such as control engineering, signal processing, and computational neuroscience, and there has been much recent interest in using state-space representations for sequence modeling in deep learning \citep{gu_efficiently_2022, dao2024transformers, zhu2024vision, nguyen2022s4nd, goel_its_2022}. The foundation of these models could be understood as the combination of classical state space representation \citep{williams2007linear, zak2003systems} and the HiPPO (high-order polynomial projection operators) theory \citep{gu2020hippo}. HiPPO is a framework using an $N$-dimensional ordinary differential equations (ODEs) to approximate the continuous-time history of an input function $f$. In particular, the HiPPO-LegS (Legendre Scaled) ODE is \[ \derivative{c}(t) = -\frac{1}{t}Ac(t) + \frac{1}{t}Bf(t), \] for $t\in [0,T]$, where $T>0$ is some terminal time and $f \colon [0,t] \to \mathbb{R}$ is an input function. With a specific choice of $A \in \mathbb R^{N \times N}$ and $B \in \mathbb R^{N \times 1}$, the solution $c\colon [0,T]\rightarrow \in \mathbb{R}^N$ encodes the continuous-time history of $f$ via \begin{align*} c_j(t) &= \Big\langle f(\cdot ), P_{j-1}\big(\tfrac{2\,\cdot} {t}-1\big) \Big\rangle_{L^2([0,t])} = \int f(s ) P_{j-1}\big(\tfrac{2s} {t}-1\big) \;ds, \end{align*} where $P_{j-1}$ is the $j-1$-th Legendre polynomial, for $j=1,2,\dots$ \citep{gu2020hippo}. The input function $f$ could be reconstructed by the formula \begin{align*} f(s) \approx \sum_{j=1}^Nc_j(t) P_{j-1}\big(\tfrac{2s}{t}-1\big)\qquad\text{ for }0\le s\le t. \end{align*} Since $\{P_n(\frac{2\,\cdot}{t}-1)\}_{n=0}^\infty$ forms an orthogonal basis of $L^2([0, t])$, this approximation can be viewed as the projection of $f$ on to the $N$-dimensional subspace spanned by $\{P_n(\frac{2\,\cdot}{t}-1)\}_{n=0}^{N-1}$. Unlike previous linear time-invariant (LTI) methods such as Legendre Memory Units (LMUs) \citep{voelker2019legendre} where the measure of projection is of fixed length, the LegS allows the model to keep track of the whole trajectory of $f(t)$. This distinctive property makes the LegS formulation powerful in many practical applications. However, despite receiving much attention for state space models in deep learning, the careful mathematical foundation of the LegS ODE is missing. To begin with, the singularity at $t=0$ renders the question of existence and uniqueness of the solution $c(t)$ a non-obvious matter. Moreover, it is unclear if the numerical methods used on the work of \cite{gu2020hippo} are mathematically justified in the sense of consistency: in the limit of small stepsizes, do the discrete simulations converge to the true continuous-time solution? \paragraph{Contributions.} In this work, we provide the careful mathematical foundations of the HiPPO-LegS ODE formulation and its discretization. Specifically, we show that (i) the solution to the LegS ODE exists and is unique, but the initial condition is fixed to a predetermined value depending on $f(0)$ and (ii) the commonly used discretization schemes LegS converges to the exact continuous-time solution for all Riemann-integrable $f$. \subsection{Related works} \label{sec:related_works} \paragraph{State Space Models for Deep Learning.} The use of state-space models(SSMs) in deep learning has gained significant traction due to their ability to process sequential and temporal data efficiently. While the transformers architecture \citep{vaswani2017attention} has become the golden standard for language models, recent SSM models such as mamba \citep{gu_mamba_2023} has been reported to achieve state-of-the-art results, especially in the long-sequence regime. Large-scale SSMs deploy an initialization scheme motivated by the HiPPO theory. \citep{gu2023train} One distinctive characteristic of state-of-the-art SSMs is that the computation cost displays a linear growth with respect to sequence length, unlike the quadratic growth of transformers. S4 \citep{gu_efficiently_2022} uses Fast Fourier Transform to attain the linear dependency, whereas mamba leverages hardward-aware computation techniques. The SSM architecture has been applied to or motivated numerous model structures \citep{fu2022hungry, hasani2022liquid, sun2023retentive, peng2023rwkv}, used across various modalities \citep{zhu2024vision, li2025videomamba, shams2024ssamba}. \paragraph{Legendre Memory Units for LSTMs.} A fundamental challenge in training recurrent neural networks(RNNs) is the vanishing gradient problem, which causes long-range dependencies in temporal data to be lost during training \citep{bengio1994learning, le2015simple}. While LSTMs \citep{schmidhuber1997long} alleviate this problem by incorporating nonlinear gating mechanisms, modeling very long sequences remains challenging. Motivated by applications in computational neuroscience, LMUs \citep{voelker2019dynamical, voelker2019legendre} introduced a novel approach to extend LSTM's capability to `remember' the sequence information by using a $N$-dimensional ODE. Using Pad\'{e} approximants \citep{pade1892representation}, the ODE is constructed so that the solution is the projection of the input function on the orthonormal basis of measure $\mathbb{I}_{[t-\theta, t]}$, where $\theta$ is a hyperparameter. The HiPPO framework could be understood as a generalization of LMUs. While LMU and its variants has proven to be effective for long sequence modeling \citep{liu_lmuformer_2024,zhang2023performance, chilkuri_parallelizing_2021}, their scope is limited to LTI methods. \paragraph{ Convergence Analysis from Control Theory.} State-space models have been extensively studied in control theory \citep{kalman1960new, zabczyk2020mathematical}, with significant research dedicated to discretization schemes and their analysis \citep{kowalczuk1991discretization}. However, these results are not directly applicable to the LegS ODE due to their exclusive focus on LTI systems \citep{karampetakis2014error} or their assumption of discrete-time inputs \citep{meena2020discretization}. Furthermore, the objectives of state-space models in deep learning applications differ fundamentally from those in classical control theory, where controlling or statistically estimating the state is usually the focus. This divergence makes it challenging to directly adapt these results to modern deep learning contexts. \section{Problem setting and preliminaries} \label{sec:problem setting} \paragraph{LegS ODE.} In this work, we consider the LegS ODE \begin{align} \derivative{c}(t) &= -\frac{1}{t}Ac(t) + \frac{1}{t}Bf(t) \label{eq:legs} \end{align} for $t\in [0,T]$, where $T>0$ is some terminal time and $c\colon [0,T]\rightarrow \in \mathbb{R}^N$ is the state vector that encodes the continuous-time history information of input function $f \colon [0,t] \to \mathbb{R}$. The matrices $A \in \mathbb R^{N \times N}$ and $B \in \mathbb R^{N \times 1}$ are defined as \[ \begin{aligned} A_{ij} &= \begin{cases} (2i-1)^{1/2}(2j-1)^{1/2} & \text{if} \quad i > j \\ i & \text{if} \quad i=j \\ 0 & \text{if} \quad i < j , \end{cases} \\ B_j &= (2j-1)^{1/2}. \end{aligned} \] Since $A$ is lower-triangular, we immediately recognize that $A$ is diagonalizable with non-repeated eigenvalues $\{1, 2, \dots, N\}$. So, $A$ is diagonalizable, and we use the eigendecomposition \[ A=V DV^{-1} \] where $V\in\mathbb{R}^{N\times N}$ is an invertible matrix and \[ D = \mathrm{diag}\,(1,2,\dots,N). \] In the indexing of $A_{ij}$, $B_j$, and $c_i(t)$, we have $i,j\in\{1,\dots,N\}$, i.e., we use $1$-based indexing. Note that the prior HiPPO paper \citep{gu2020hippo} uses $0$-based indexing. \paragraph{Shifted Legendre polynomials.} We write $P_j(x) \colon [-1,1] \to [-1,1]$ to denote the $j$-th Legendre polynomial, normalized such that $P_j(1)=1$, for $j=0,1,\dots$. However, we wish to operate on the domain $[0,1]$, so we perform the change of variables $x \to 2x-1$. This yields, \[ \tilde{P}_j(x)= P_j(2x-1) = \sum_{k=0}^j (-1)^j \binom{j}{k} \binom{j+k}{k} (-x)^k, \] the $j$-th shifted Legendre polynomial, for $j=0,1,\dots$. The shifted Legendre polynomials satisfy the recurrence relation \[ x\tilde{P}_j'(x) = j\tilde{P}_j + \sum_{k=1}^{j-1}(2k+1)\tilde{P}_j(x) \] which can be derived by combining the following well-known results \citep{arfken2011mathematical}. \begin{align*} (2n + 1)P_j(x) &= P_{j+1}'(x) - P_{j-1}'(x) \\ P_{j+1}'(x) &= (n+1)P_j(x) + xP_j'(x) \end{align*} \paragraph{Numerical discretization methods.} In this work, we analyze the numerical solution methods of the LegS ODE used in the prior work \citep{gu2020hippo}. For all the discretization schemes, we consider a meshgrid with initial time $t_0=0$ and stepsize $h = \left\lfloor \frac{T}{n} \right\rfloor$. Starting with $c^0=c(0)$, we iteratively apply the discretization scheme and denote the numerical solution at $n$-th step as $c^n$, and $f(nh)$ as $f^n$. The \textbf{\underline{backward Euler}} method \begin{align*} c^{n+1} &= \Big(I + \frac{1}{n+1}A\Big)^{-1}c^n + \Big(I + \frac{1}{n+1}A\Big)^{-1} \frac{1}{n+1} B f^{n+1}, \qquad\text{ for }n=0,1,2,\dots \end{align*} is well defined as is. However, the \textbf{\underline{forward Euler}} method \begin{align*} c^{n+1} &= \Big(I - \frac{1}{n}A\Big) c^n + \frac{1}{n}Bf^n \qquad\text{ for }n=? \end{align*} and the \textbf{\underline{bilinear (trapezoidal)}} method \begin{align*} c^{n+1} &= \Big(I + \frac{1}{2(n+1)}A\Big)^{-1} \Big(I - \frac{1}{2n}A\Big)c^n + \Big(I + \frac{1}{2(n+1)}A\Big)^{-1} \Big(\frac{1}{2n}f^n + \frac{1}{2(n+1)}f^{n+1}\Big)B, \qquad\text{ for }n=? \end{align*} are not well defined for $n=0$. One remedy would be to use the identity $\dot{c}(0) = \pr{A+I}^{-1} f'(0)$, which we derive in Lemma~\ref{lem:t=0 calc}. However, if $f$ is not differentiable at $t=0$, then even this remedy is not possible. Hence, in Section~\ref{sec:error_analysis} where we consider general $f$, we simply perform the ill-defined iteration letting $f'(0)=0$, e.g. $c^1 = c^0$ for forward Euler methods. As will be shown soon, this modification does not affect the convergence results. In the prior work \cite{gu2020hippo}, the authors simply sidestep this issue by shifting the $n$ index up by $1$, leading to the \textbf{\underline{approximate bilinear}} method \begin{align*} c^{n+1} &= \pr{I + \frac{1}{n+1}A/2}^{-1} \pr{I - \frac{1}{n+1}A/2}c^n + \pr{I + \frac{1}{n+1}A/2}^{-1} \pr{\frac{1}{n+1}f^{n+1}}B,\qquad\text{ for }n=0,1,2,\dots. \end{align*} In this work, we establish convergence of the approximate method, despite the shift in the $n$ index. \paragraph{Convergence of numerical discretization methods.} To numerically solve a first-order initial value problem of the form \begin{align*} c'(t) = g(t,c(t)) \label{eq:IVP} \end{align*} for $t\ge 0$ with initial condition $c(0) = c^0$, we consider one-step methods of the form \[ c^{i+1} = c^i + h\Phi(t_i, t_{i+1}, c^i, c^{i+1}; h), \quad i=0,1,...,n-1 \] with stepsize $h = T/n$ and $t_i=t_0+hi$ for $i=0,\dots,n-1$. Here, $\Phi$ is a numerical integrator intended to make the following approximation \[ \Phi(t_i, t_{i+1}, c^i, c^{i+1}; h)\approx c(t_{i+1})-c(t_i)= \int^{t_i+1}_{t_i}g(s,c(s))\;ds. \] To analyze such methods, one often estimates the \emph{local truncation error} \begin{align*} T_n &= \frac{c(t_{n+1}) - c(t_n)}{h} - \Phi(t_n, t_{n+1}, c(t_n), c(t_{n+1}; h) \end{align*} and then estimates its accumulation to bound the \emph{global error} \[ e_n = c(t_n) - c^n. \] Classical ODE theory states that if the right-hand-side $g$ in the initial value problem is Lipschitz continuous with respect to $c$, the solution exists and is unique in an interval including the initial point $t_0=0$. Moreover, under the same conditions, the global error can be bounded with the local truncation error \citep{ascher_petzold_1998,Süli_Mayers_2003}. However, this standard theory does not apply to the LegS ODE due to the singularity at $t=0$. \section{LegS is well-posed} \label{sec:well-posed} In this section, we show that the LegS ODE is well-posed despite the singularity. Crucially, however, we show that there is no freedom of choosing the initial condition. \begin{theorem}[Existence and uniqueness] \label{thm:exist and unique} For $T>0$ and $c_0 \in \mathbb{R}^N$, we say $c\colon [0,T]\rightarrow \mathbb R^N$ is a solution (in the extended sense) of the LegS ODE if $c$ is absolutely continuous on $(0,T]$, $c$ satisfies \eqref{eq:legs} for almost all $t \in (0,T]$, and $c(0) = c_0$. Assume $f\colon [0,T]\rightarrow \mathbb R$ is Lebesgue measurable and locally integrable. Further, assume $f$ is continuous at $t=0$. Then, the solution exists and is unique if $c_0 = f(0)e_1$, where $e_1\in \mathbb{R}^N$ is the first standard basis vector. Otherwise, if $c_0 \ne f(0)e_1$, a solution does not exist. \end{theorem} \begin{proof} Since $A$ is diagonalizable, the problem can be effectively reduced to $N$ $1$-dimensional problems. Recall $A = VDV^{-1}$ where $D=\mathrm{diag}\,(1,2,\dots,N)$. We see the LegS ODE \eqref{eq:legs} could be rewritten as \begin{align*} \derivative{c}(t) &= -\frac{1}{t} A c(t) + \frac{1}{t} B f(t) = -\frac{1}{t} VDV^{-1} c(t) + \frac{1}{t} B f(t). \end{align*} Multiply both sides by $V^{-1}$ and denote $\tilde{c}(t) = V^{-1}c(t)$. Then, the ODE becomes \begin{align*} \derivative{\tilde{c}}(t) = V^{-1}\derivative{c}(t) &= -\frac{1}{t} D V^{-1} c(t) + \frac{1}{t} V^{-1}B f(t) = -\frac{1}{t} D\tilde{c}(t) + \frac{1}{t} V^{-1}B f(t), \end{align*} which is a decoupled ODE with respect to $\tilde{c}$. Recalling $D_{jj} = j$, we see that the $j$-th component of the above equation is \begin{align} \derivative{\tilde{c}}_j(t)&= -\frac{j}{t}\tilde{c}_j(t) + \frac{d_j}{t} f(t) \label{eq:1d_legs} \end{align} where $d_j = (V^{-1}B)_j$ and $\tilde{c}_j$ is $j$-th component function of $\tilde{c}$. Since $V$ and $V^{-1}$ are absolutely continuous bijective map from $\mathbb{R}^N$ to $\mathbb{R}^N$, the existence and uniqueness of the solution of the LegS ODE is satisfied if and only the existence and uniqueness of the solution of the ODE \eqref{eq:1d_legs} is satisfied for all $j \in \{1, 2, ..., N\}$. We now proceed by examining the existence and uniqueness of the solution of the ODE \eqref{eq:1d_legs}. We first establish existence by presenting the explicit form of the solution. Define $\tilde{c}_j\colon[0,T]\to\mathbb{R}$ as \begin{equation*} \tilde{c}_j(t) = \begin{cases} \frac{d_j}{t^j} \int^t_0 s^{j-1} f(s) ds & \text{if} \quad t \in (0,T] \\ \frac{d_j}{j} f(0) & \text{if} \quad t = 0 . \end{cases} \end{equation*} By fundamental theorem of calculus, $\frac{d}{dt} \pr{ \int^t_0 s^{j-1} f(s) ds } = t^{j-1} f(t)$ holds for almost all $t\in(0,T]$ and thus $\tilde{c}_j$ is differentiable for almost all $t\in(0,T]$. Therefore, \[ t^j \pr{ \derivative{\tilde{c}}_j(t) + \frac{j}{t} \tilde{c}_j(t) } = \frac{d}{dt} (t^j\tilde{c}_j(t)) = \frac{d}{dt} \pr{ d_j \int^t_0 s^{j-1} f(s) ds } = d_j t^{j-1} f(t) \] holds for almost all $t\in(0,T]$. Dividing both sides by $t^j$, we conclude $\tilde{c}_j$ satisfies \eqref{eq:1d_legs} for almost all $t\in(0,T]$. We now show $\tilde{c}_j$ is continuous on $[0,T]$. It is sufficient to check $\tilde{c}_j$ is continuous at $t=0$. Since $f$ is continuous at $t=0$, from L'H\^{o}pital's rule we have \begin{equation*} \begin{aligned} \lim_{t\to0^+} \tilde{c}_j(t) &= \lim_{t\to0^+} \frac{d_j}{t^j} \int^t_0 s^{j-1} f(s)ds \\ &= \lim_{t\to0^+} \frac{d_j }{j t^{j-1}} \pr{ t^{j-1} f(t) } = \frac{d_j}{j} \lim_{t\to0^+} f(t) = \frac{d_j}{j} f(0) = \tilde{c}_j(0). \end{aligned} \end{equation*} Therefore, $\tilde{c}_j$ is continuous at $t=0$. Lastly, $\tilde{c}_j$ is absolutely continuous on $(0,T]$, since for every $[t_0,t] \subset (0,T]$, both $\frac{1}{t^j}$ and $\int^t_0 s^{j-1} f(s) ds$ are absolutely continuous on $[t_0,t]$ and therefore their product is also absolutely continuous on $[t_0,t]$. As a result, we conclude $\tilde{c}_j$ becomes the solution of the ODE \eqref{eq:1d_legs}. We now establish the uniqueness. Suppose $\hat{c}_j$ is another solution of the ODE \eqref{eq:1d_legs}. Multiplying both sides of \eqref{eq:1d_legs} by $t^j$ and reorganizing, for almost all $t\in(0,T]$ we have \[ \frac{d}{dt} (t^j\hat{c}_j(t)) = t^j \pr{ \derivative{\hat{c}}_j(t) + \frac{j}{t} \hat{c}_j(t) } = d_j t^{j-1} f(t) . \] Since $\hat{c}_j$ is a solution, it is absolutely continuous on $(0,T]$, therefore $t^j\hat{c}_j(t)$ is absolutely continuous on $(0,T]$. Thus for $[t_0,t] \subset (0,T]$, by fundamental theorem of calculus we obtain \[ t^j \hat{c}_j(t) - t_0^j \hat{c}_j(t_0) = d_j \int_{t_0}^t s^{j-1} f(s) ds. \] Taking limit $t_0 \to 0^+$, since $\hat{c}_j$ is a solution it is continuous at $0$, we have \[ t^j \hat{c}_j(t) = d_j \int_{0}^t s^{j-1} f(s) ds. \] Dividing both sides by $t^j$ we conclude \begin{equation*} \hat{c}_j(t) = \frac{d_j}{t^j} \int^t_0 s^{j-1} f(s)ds = \tilde{c}_j(t) \end{equation*} for all $t \in (0,T]$. It remains to check $\hat{c}_j(0)=\tilde{c}_j(0)$. Since $\hat{c}$ is continuous at $0$, we know $\hat{c}_j(0)=\lim_{t\to0^+}\hat{c}_j(t)$. Thus \begin{equation*} \hat{c}_j(0) = \lim_{t\to0^+} \hat{c}_j(t) = \lim_{t\to0^+} \tilde{c}_j(t) = \tilde{c}_j(0). \end{equation*} Therefore, we conclude $\hat{c}_j(t) = \tilde{c}_j(t)$ for all $t\in[0,T]$, the solution of the ODE \eqref{eq:1d_legs} is unique. As a result, we conclude the solution of the LegS ODE uniquely exists if $\tilde{c}_j(0) = \frac{d_j}{j}f(0)$, and it is given by $c = V \tilde{c}$. Finally, we show the unique solution $c = V \tilde{c}$ should satisfy $c(0) = f(0)e_1$. From \[ V^{-1}c_j(0) = \tilde{c}_j(0) = \frac{d_j}{j}f(0) = \frac{1}{j}(V^{-1}B)_j f(0), \] we see \[ V^{-1}c(0) = \text{diag} \pr{ 1, \frac{1}{2}, \frac{1}{3}, ... , \frac{1}{N} } (V^{-1}B) f(0) = D^{-1} V^{-1} B f(0). \] Multiplying both sides by $V$, we conclude \begin{align*} c(0) = V D^{-1} V^{-1} B f(0) = ( V^{-1} D V )^{-1} = A^{-1}B f(0) = f(0)e_1 \end{align*} where $e_1 = [1, 0, \dots, 0] ^{t}$. Therefore if $c_0 = f(0)e_1$, the solution exists and is unique, and otherwise, there is no solution. \end{proof} \begin{lemma} [Behavior at $t=0$] \label{lem:t=0 calc} Consider the setup of Theorem~\ref{thm:exist and unique}, and further assume that $f$ is one-sided differentiable at $t=0$. Then, the one-sided derivative $c'(0)$ exists and is given by \begin{align} c'(0) &= (A+I)^{-1}B f'(0).\label{eq:c'(0)} \end{align} \end{lemma} \begin{proof} We examine the differentiability of $V^{-1}c(t) = \tilde{c}(t) = (\tilde{c}_j(t))_{j=1}^N$ by checking for each component. Recall that for the $j$-th component we have $\tilde{c}_j(t) = \frac{d_j}{t^j}\int_0^t s^{j-1}f(s)ds$ and $\tilde{c}_j(0) = \frac{d_j}{j}f(0)$. We will denote the one-sided derivative of $c(t)$ and $f(t)$ at $t=0$ as $c'(0)$ and $f'(0)$, respectively. Then, \begin{align*} \lim_{t \to 0^+} \frac{\tilde{c} _j(t) - \tilde{c}_j(0)}{t} &= \lim_{t \to 0^+} d_j \frac{\frac{1}{t^j} \int_0^t s^{j-1} f(s)ds - \frac{1}{j}f(0)}{t} \\ &= \lim_{t \to 0^+} d_j \frac{\int_0^t s^{j-1} f(s)ds - \frac{t^j}{j}f(0)}{t^{j+1}} \\ &\overset{(1)}{=} \lim_{t \to 0^+} d_j \frac{t^{j-1}f(t) - t^{j-1} f(0)}{(j+1)t^{j}} \\ &= \lim_{t \to 0^+} \frac{d_j}{j+1} \frac{f(t) - f(0)}{t} \\ &= \frac{d_j}{j+1} f'(0) \end{align*} where LH\^{o}pital's rule was used for $(1)$. Folding back to vector form, recalling $V^{-1}c'_j(0) = \tilde{c}'_j(t) = \frac{d_j}{j+1}f'(0) = \frac{1}{j+1}(V^{-1}B)_j f'(0)$, we obtain \begin{align*} V^{-1}c'(0) &= \text{diag} \pr{ \frac{1}{2}, \frac{1}{3}, ... , \frac{1}{N+1} } (V^{-1}B) f'(0) \end{align*} and hence $c'(0) = \pr{A+I}^{-1}B$. Note that $(A+I)^{-1}B = [1/2, 1/(2\sqrt{3}), 0, ..., 0]^{t}$. \end{proof} \paragraph{Ill-posedness with negative eigenvalues.} The well-posedness argument of Theorem~\ref{thm:exist and unique} crucially relies on the fact that all eigenvalues of $A$ are positive. To see what happens when $A$ has negative eigenvalues, consider the case $N=1$ and $A=-1$. This leads to the ODE \begin{align*} \frac{d}{dt}c(t) &= \frac{1}{t} c(t) + \frac{1}{t} f(t),\qquad c(0)=c_0, \end{align*} which has solutions \begin{align*} c(t) = t \int_0^t \frac{1}{s^2}f(s)ds + Ct \end{align*} for any $C\in \mathbb{R}$. Since the initial condition does not determine the value of $C$, the solution is not unique. \section{Convergence of LegS discretization schemes} \label{sec:error_analysis} In this section, we address whether the numerical discretization methods introduced in Section~\ref{sec:problem setting} are convergent, i.e., do the methods produce numerical solutions $c^n$ that converge to the exact continuous-time solution $c(t)$ as $h\rightarrow 0$? The answer is yes, but, as we discuss in Section~\ref{subsec:smooth_f}, the standard analysis based on local truncation error does not lead to a convergence guarantee for all of the schemes under consideration, and such approaches would require certain regularity conditions on $f$, such as Lipschitz continuity, anyway. Rather, in Section~\ref{subsec:legs is quadrature}, we identify the numerical schemes as quadrature rules on the input function $f$. Using this insight, in Section~\ref{subsec:riemann_f}, we show that the discretization schemes are convergent under the general assumption of Riemann integrability of $f$. Extending the framework to accommodate general Riemann-integrable functions $f$ is important, given the nature of the application. The HiPPO memory unit is used in deep learning to analyze sequence data, such as audio signals. For such data, there is no inherent expectation of smoothness, and discontinuities are to be expected. Therefore, we aim to guarantee that the mathematics remains sound for such data. \subsection{Convergence for smooth $f$} \label{subsec:smooth_f} Discretization schemes ODEs with well-behaved right-hand-sides have a well-established theory based on the local truncation error (LTE). This line of analysis can be applied to the LegS ODE despite the singularity at $t=0$. For example, forward Euler method, the LTE $T_n$ satisfies \[ |T_n| \leq \frac{1}{2} h M_2, \qquad M_2 = \max_{t \in [t_0, t_N]} |c''(t)|. \] The proof follows from a relatively straightforward extension of the standard arguments. However, when $f$ is not differentiable, then $c''(t)$ may not be bounded, and this approach, as is, fails to yield a convergence guarantee. Another issue is that the LTE for the approximated bilinear method does not converge to $0$. For $N=1$, the approximated bilinear method reduces to \begin{align*} c^{n+1} &= \frac{2n+1}{2n+3}c^n + \frac{2}{2n+3}f^{n+1} = c^n - \frac{2}{2n+3}(c^n - f^{n+1}). \end{align*} Using the exact solution is $c(t) = \frac{1}{t}\int_0^t f(s)ds$, the exact value of the LTE is \begin{align*} hT_n &= c(t_{n+1}) - c(t_{n}) + \frac{2}{2n+3} (c(t_n) - f^{n+1}) \\ &= \frac{1}{(n+1)h}\int_0^{(n+1)h}f(s)ds - \frac{2n+1}{n(2n+3)h}\int_0^{nh}f(s)ds - \frac{2}{2n+3}f^{n+1} . \end{align*} With the particular choice linear function $f(x) = ax$, we obtain \begin{align*} T_n &= \frac{a}{2}(n+1) - \frac{n(2n+1)}{2(2n+3)}a - \frac{2a}{2n+3}(n+1) = -\frac{a(n+1)}{2(2n+3)}\rightarrow -\frac{a}{4}\ne 0. \end{align*} Thus, the LTE of the approximated bilinear method does not vanish as $h\rightarrow 0$, implying that the method is not even consistent. Consequently, a global error analysis based on the LTE will not guarantee convergence. In Section~\ref{subsec:riemann_f}, however, we employ an alternative proof technique to establish that the approximated bilinear method is indeed convergent. \subsection{LegS is a quadrature of $f$} \label{subsec:legs is quadrature} In this section, we provide the key insight that we can identify the numerical schemes as quadrature rules on the input function $f$. Recall that the LegS equation ODE was proposed for online approximation of the input function $f(t)$ on the interval $[0,t]$. Specifically, the matrix $A$ and $B$ was specifically chosen so that the $j$-th component of $c(t) \in \mathbb{R}^N$ is given by \begin{align*} c_j(t) &= \frac{\sqrt{2j-1}}{t} \int_0^t P_{j-1} \Big(\frac{2s}{t} - 1 \Big) f(s) ds, \end{align*} where $P_{j-1}$ denotes the $(j-1)$-th Legendre polynomial, for $j=1,2,\dots,N$ \citep{gu2020hippo}. So $c_j(t)$ is a (signed) weighted integral of $f(\cdot)$ on $[0,t]$, and the following lemma shows that the numerical schemes of Section~\ref{sec:problem setting} are, in fact, quadrature rules approximating this integral. \begin{lemma} \label{lem:numsol_linear} Consider applying any of the discretization schemes introduced in Section~\ref{sec:problem setting} (forward Euler, backward Euler, bilinear, or approximate bilinear) to the LegS ODE \eqref{eq:legs}, with initial time $t_0=0$ and timestep $h = T/n$. Then, the numerical solution $c^n$ at step $n$ satisfies \begin{align*} c^n &= \frac{1}{n} \sum_{l=0}^n \alpha^{(n)}_l f^l, \end{align*} where $\alpha^{(n)}_l \in \mathbb{R}^N$ only depends on $l$ and $n$. \end{lemma} \begin{proof} For notational simplicity, we define the following quantities for $n \in \mathbb{N}$, with $n \geq 2$ for $Q_n$. \begin{align*} Q_n &:= \prod_{j=1}^{n-1} \pr{I - \frac{1}{j+1}A} \\ \tilde{Q}_n &:= \prod_{j=1}^n \pr{I - \frac{1}{2j}A}\\ R_n &:= \prod_{j=1}^n \pr{I + \frac{1}{j}A}^{-1} \\ \tilde{R}_n &:= \prod_{j=1}^n \pr{I + \frac{1}{2j}A}^{-1} \end{align*} We use the $\prod$ notation when the multiplications are commutative. Note that all $Q_n, \tilde{Q}_n, R_n, \tilde{R}_n$ are invertible. We start by proving for the forward Euler method. Recall that the forward Euler method yields the following recurrence relation at step $n$: \begin{align*} c^{n+1} &= \pr{ I - \frac{1}{n}A}c^n + \frac{1}{n} B f^n. \end{align*} Repeating this procedure, we can obtain an exact formula for the numerical solution obtained by applying forward Euler method to the LegS ODE. By induction we obtain, \begin{align*} c^{n+1} &= \pr{ I - \frac{1}{n}A }c^n + \frac{1}{n}Bf^n \\ &= \pr{ I - \frac{1}{n}A} \pr{ I - \frac{1}{n-1}A} c^{n-1} + \pr{ I - \frac{1}{n}A} \frac{1}{n-1}Bf^{n-1} + \frac{1}{n}Bf^n \\ &= Q_n \pr{c^1 + Bf^1} + Q_n \sum_{l=2}^n \frac{1}{l} Q_{l}^{-1} B f^l. \end{align*} As explained in Section~\ref{sec:problem setting}, we `zero out' the ill-defined iteration, thereby letting $c^1=c^0$. Hence we have \begin{align} \label{eq:forward_numsol} c^{n} &= Q_{n-1} e_1 f^0 + Q_{n-1} B f^1 + Q_{n-1} \sum_{l=1}^{n-1} \frac{1}{l} Q_l^{-1} B f^l. \end{align} Immediately one can see that $c^n$ is expressed as $\frac{1}{n} \sum_{l=0}^n \alpha_l^{(n)}$, where $\alpha_l^{(n)}$ depends only on $l$ and $n$. For the backward Euler method, we start from \begin{equation*} \begin{aligned} c^{n+1} &= \pr{ I + \frac{1}{n+1} A }^{-1} c^{n} + \pr{ I + \frac{1}{n+1} A }^{-1} \frac{1}{n+1} B f^{n+1}. \end{aligned} \end{equation*} Then, we can derive inductively \begin{equation*} \begin{aligned} c^{n+1} &= \pr{ I + \frac{1}{n+1}A }^{-1} c^n + \pr{ I + \frac{1}{n+1}A }^{-1} \frac{1}{n+1}Bf^{n+1} \\ &= \pr{ I + \frac{1}{n+1}A }^{-1} \pr{ I + \frac{1}{n}A }^{-1} c^{n-1} + \pr{ I + \frac{1}{n+1}A }^{-1} \pr{ I + \frac{1}{n}A }^{-1} \frac{1}{n}Bf^n + \pr{ I + \frac{1}{n+1}A }^{-1} \frac{1}{n+1}Bf^{n+1} \\ &= R_{n+1} (c^0 + Bf^1) + R_{n+1} \sum_{l=1}^{n} \frac{1}{l+1} R_{l}^{-1} B f^{l+1}. \end{aligned} \end{equation*} Thus we can verify $c^n$ has the desired form with the specific expression \begin{equation} \label{eq:back_numsol} \begin{aligned} c^n &= R_{n} (e_1f(0) + Bf^1) + R_{n} \sum_{l=1}^{n-1} \frac{1}{l+1} R_{l}^{-1} B f^{l+1}. \end{aligned} \end{equation} For the bilinear method, we start from \begin{align*} c^{n+1} &= (I + \frac{1}{n+1}A/2)^{-1}(I - \frac{1}{n}A/2)c^n + (I + \frac{1}{n+1}A/2)^{-1} \frac{1}{2} (\frac{1}{n}f^n + \frac{1}{n+1}f^{n+1})B. \end{align*} Similarly by induction we obtain \begin{equation*} \begin{aligned} c^{n+1} &= \pr{ I + \frac{1}{n+1}A/2 }^{-1} \pr{ I - \frac{1}{n}A/2 } c^n + \pr{ I + \frac{1}{n+1}A/2 }^{-1} \frac{1}{2} \pr{\frac{1}{n}Bf^{n} + \frac{1}{n+1}Bf^{n+1} }\\ &= \pr{ I + \frac{1}{n+1}A/2 }^{-1} \pr{ I + \frac{1}{n}A/2 }^{-1} \pr{ I - \frac{1}{n}A/2 } \pr{ I - \frac{1}{n - 1}A/2 } c^{n-1}\\ &+ \pr{ I + \frac{1}{n+1}A/2 }^{-1} \pr{ I + \frac{1}{n}A/2 }^{-1}\pr{ I - \frac{1}{n - 1}A/2 } \frac{1}{2} \pr{\frac{1}{n-1}Bf^{n-1} + \frac{1}{n}Bf^{n} }\\ &+ \pr{ I + \frac{1}{n+1}A/2 }^{-1} \frac{1}{2} \pr{\frac{1}{n}Bf^{n} + \frac{1}{n+1}Bf^{n+1} }\\ &= \tilde{Q}_n \tilde{R}_{n+1} \pr{I+A/2} c^1 + \tilde{Q}_{n} \tilde{R}_{n+1} \sum_{l=1}^{n} \tilde{Q}_l^{-1} \tilde{R}_l^{-1} \frac{1}{2}\pr{\frac{1}{l}Bf^l + \frac{1}{l+1}Bf^{l+1}}. \end{aligned} \end{equation*} As for the forward Euler case, we `zero out' the ill-defined term in the first iteration. This yields $c^1 = \pr{I+\frac{A}{2}}^{-1}c^n + \pr{I+\frac{A}{2}}^{-1}\pr{\frac{1}{2}f^1}$. Then, \begin{equation*} \begin{aligned} c^n &= \tilde{Q}_{n-1}\tilde{R}_{n}e_1f^0 +\tilde{Q}_{n-1}\tilde{R}_{n}\frac{f^1}{2} +\tilde{Q}_{n-1}\tilde{R}_{n} \sum_{l=1}^{n-1} \tilde{Q}_l^{-1} \tilde{R}_l^{-1} \frac{1}{2}\pr{\frac{1}{l}Bf^l + \frac{1}{l+1}Bf^{l+1}}. \end{aligned} \end{equation*} Rearranging terms, \begin{align}\label{eq:bilin_numsol} c^n &= \tilde{Q}_{n-1}\tilde{R}_{n}e_1f^0 + \tilde{Q}_{n-1}\tilde{R}_{n} \sum_{l=1}^{n-1} \frac{1}{2l} \pr{ \tilde{Q}_l^{-1} \tilde{R}_l^{-1} + \tilde{Q}_{l-1}^{-1} \tilde{R}_{l-1}^{-1} }Bf^l \end{align} where we define $\tilde{Q}_0=\tilde{R}_0=I$. Thus we recover the desired form for $c^n$. For the approximate bilinear method, we start from \begin{align*} c^{n+1} &= (I + \frac{1}{n+1}A/2)^{-1}(I - \frac{1}{n+1}A/2)c^n + (I + \frac{1}{n+1}A/2)^{-1} (\frac{1}{n+1}f^{n+1})B. \end{align*} By induction, we obtain \begin{equation*} \begin{aligned} c^{n+1} &= \pr{ I + \frac{1}{n+1}A/2 }^{-1} \pr{ I - \frac{1}{n+1} A/2 } c^n + \pr{ I + \frac{1}{n+1}A/2 }^{-1} \pr{\frac{1}{n+1}Bf^{n+1} }\\ &= \pr{ I + \frac{1}{n+1}A/2 }^{-1} \pr{ I + \frac{1}{n}A/2 }^{-1} \pr{ I - \frac{1}{n+1}A/2 } \pr{ I - \frac{1}{n}A/2 } c^{n-1}\\ &+ \pr{ I + \frac{1}{n+1}A/2 }^{-1} \pr{ I + \frac{1}{n}A/2 }^{-1}\pr{ I - \frac{1}{n+1}A/2 } \pr{\frac{1}{n}Bf^{n} }\\ &+ \pr{ I + \frac{1}{n+1}A/2 }^{-1} \pr{\frac{1}{n+1}Bf^{n+1} }\\ &= \tilde{Q}_{n+1} \tilde{R}_{n+1} c^0 + \tilde{Q}_{n+1} \tilde{R}_{n+1} \sum_{l=1}^{n} \tilde{Q}_{l+1}^{-1} \tilde{R}_l^{-1} \pr{\frac{1}{l+1}Bf^{l+1}}. \end{aligned} \end{equation*} Thus we can verify $c^n$ has the desired form with the specific expression \begin{equation} \label{eq:apbil_numsol} \begin{aligned} c^n &= \tilde{Q}_{n}\tilde{R}_{n}c^0 + \tilde{Q}_{n}\tilde{R}_{n} \sum_{l=1}^{n-1} \tilde{Q}_{l+1}^{-1} \tilde{R}_l^{-1} \pr{\frac{1}{l+1}Bf^{l+1}}. \end{aligned} \end{equation} \end{proof} \subsection{Convergence for Riemann integrable $f$} \label{subsec:riemann_f} In this section, we prove the convergence of all discretization methods of interest for Riemann integrable $f$'s. In particular, we prove the convergence of the approximated bilinear method, justifying its use in the HiPPO paper \citep{gu2020hippo}. The results are summarized in the following theorem: \begin{theorem} \label{thm:conv_riemann} Consider the LegS equation \eqref{eq:legs} with dimension $N$, defined on $t \in [0,t]$. Assume that $f$ is Riemann integrable on $[0,t]$. Given a discretization method, assume that one performs the iterations defined by \begin{align*} c^{n+1} &= c^n + h \Phi(t_n,t_{n+1},c^n,c^{n+1}; h) \\ c^0_j &= \delta_{1j} f(0) \end{align*} using mesh points $t_n = nh$ for $n = 1, 2, \dots, \left\lfloor T/h \right\rfloor$, where $\Phi$ is the one-step integrator defined by the given discretization method. Denote the exact solution at step $n$ as $c(t_n) = c(nh) \in \mathbb{R}^N$. Then, for all the discretization methods defined introduced in Section \ref{sec:problem setting}, we have the convergence of the numerical solution to the exact solution in the sense that \begin{align*} \lim_{n \to \infty} c^n = c(t) \quad \text{as} \quad t_n \to t \in [0, T] \end{align*} when $h \to 0$ and $n \to \infty$. \end{theorem} To prove the theorem, we first denote the the numerical solution obtained by the application of a discretization method to the LegS ODE as $c^n = \frac{1}{n} \sum_{l=0}^n \alpha^{(n)}_l f^l$, in light of Lemma \ref{lem:numsol_linear}. If we could show for all Riemann integrable $f$ \[ c^n_j = \frac{1}{n} \sum_{l=0}^n \pr{\alpha^{(n)}_l}_jf^l \to c_j(t) = \frac{\sqrt{2j-1}}{t}\int_0^t P_{j-1}\pr{\frac{2s}{t}-1}f(s)ds, \] with $t_n \to t$ as stepsize $h \to 0$ and $n \to \infty$, and where $P_j$ is the $j$-th Legendre polynomial, we are done. Instead of directly working on the coefficients $\alpha^{(n)}_l$, the key idea is to consider a function sequence defined on $[0,1]$ that interpolates those points. This significantly reduces the complexity of analyzing the asymptotic behavior of the numerical solution as the number of mesh points $n$ goes to infinity. We start by proving an elementary lemma that enables this approach. \begin{lemma} \label{lem:converge_condition} Assume that $f : [0,t] \to \mathbb{R}$ is a Riemann integrable function, and $\{G^{(n)}\}_{n \in \mathbb{N}}$ is a sequence of continuous functions defined on $[0,1]$. Assume further that for some $G \in C[0,1]$, \begin{align*} G^{(n)}(x) \to G(x) \end{align*} uniformly on $x \in (0,1)$ as $n \to \infty$. Then, for $h = \left\lfloor\frac{t}{n}\right\rfloor$, \begin{align*} \frac{1}{n} \sum_{l=1}^n G^{(n)} \pr{\frac{l}{n}} f\pr{lh} \to \frac{1}{t}\int_0^t G \pr{\frac{s}{t}}f(s)ds \end{align*} as $h \to 0$ and $n \to \infty$. \end{lemma} \begin{proof} Since $f(x)$ is Riemann integrable and $G$ is continuous, it is straightforward to show that $G\pr{\frac{x}{t}}f(x)$ is Riemann integrable for $x \in [0,t]$. From basic properties of Riemann integrable functions, one could easily show that the Riemann sum $\frac{1}{n} \sum_{l=1}^n G \pr{\frac{l}{n}}f\pr{lh}$ converges to $\frac{1}{t}\int_0^t G \pr{\frac{s}{t}}f\pr{s} ds$ as $n \to \infty$. Since $f$ is bounded, let $\sup_{x \in [0,t]} |f(x)| \leq M$ and construct a triangular inequality. \begin{align*} \left| \frac{1}{t}\int_0^t G \pr{\frac{s}{t} }f(s)ds - \frac{1}{n}\sum_{l=1}^n G^{(n)} \pr{\frac{l}{n}} f \pr{lh} \right| &\leq \left| \frac{1}{t}\int_0^t G \pr{\frac{s}{t}} f(s)ds - \frac{1}{n} \sum_{l=1}^n G \pr{\frac{l}{n}}f\pr{lh} \right| \\ &+ \frac{1}{n} \sum_{l=1}^n \left| G \pr{\frac{l}{n}}f\pr{lh} - G^{(n)} \pr{\frac{l}{n}} f\pr{lh} \right| \\ &\leq \frac{\epsilon}{2} + \sup_{x \in [0,t]} \left| f(x) \right| \max_{l \in \{0, \dots, n\}} \left| G\pr{\frac{l}{n}} - G^{(n)} \pr{\frac{l}{n}} \right| \\ &\leq \frac{\epsilon}{2} + M \| G - G^{(n)} \|_{\sup} \\ &\leq \epsilon \end{align*} \end{proof} This result is relevant since if we interpret the shifted Legendre polynomials as $G\pr{\frac{s}{t}}$ in the lemma above, we can obtain a sufficient condition for a numerical solution to converge to the exact solution of the ODE. This observation is specified in the next corollary. \begin{corollary} \label{cor:conv_to_quad} Consider a vector $c^n \in \mathbb{R}^N$ such that $c^n_j = \frac{1}{n} \sum_{l=0}^n \pr{\alpha^{(n)}_l}_j f(lh)$ where $h = \left\lfloor\frac{t}{n}\right\rfloor$ and $f : [0,t] \to \mathbb{R}$ a Riemann integrable function. Assume $\sup_{l} \pr{\alpha^{(n)}_l}_j \leq C$. Denote the $j$-th Legendre polynomial as $P_j$. Now, consider a uniformly bounded function sequence $\{F^{(n)}\}_n$ such that $F \in C([0,1]; \mathbb{R}^N)$ for all $F \in \{F^{(n)}\}_{n \in \mathbb{N}}$. Further, assume that for all $F \in \{F^{(n)}\}_{n \in \mathbb{N}}$, the $j$-th component satisfies for all $j \in J$ and $l \in \{1, 2, \dots, n-1\}$ \begin{align} \label{eq:F_condition} \pr{F \pr{\frac{l}{n}}}_j &= \pr{\alpha^{(n)}_l}_j. \end{align} Then, if \begin{align*} \pr{F^{(n)}(x)}_j \to \sqrt{2j-1} P_{j-1} \pr{2x-1} \end{align*} uniformly on $x\in [0,1]$ as $n \to \infty$ for all $j \in J$, \begin{align*} \pr{c^n}_j \to \frac{\sqrt{2j-1}}{t}\int_0^t P_{j-1}\pr{\frac{2s}{t}-1}f(s)ds \end{align*} as $h \to 0$ and $n \to \infty$ for all $j \in J$. \end{corollary} \begin{proof} Since the sequence $\{F^{(n)}\}$ is uniformly bounded, we let $F_j \leq M_j$ for all $F \in \{F^{(n)}\}$. Also, $|f| \leq M$ from the definition of Riemann integrable functions. Now fix component index $j$. Since $\pr{F^{(n)}\pr{\frac{l}{n}}}_j$ is equal to $\pr{\alpha^{(n)}_l}_j$ for $l \in \{1, 2, \dots, n-1\}$, we can write \begin{align*} \pr{c^n}_j &= \frac{1}{n} \sum_{l=1}^n \pr{F^{(n)}\pr{\frac{l}{n}}}_j f(lh) + \frac{1}{n} \pr{\alpha^{(n)}_0 f(0) + \alpha^{(n)}_n f(nh) - \pr{F^{(n)}(1)}_j f(nh)} \end{align*} Using this equality, we can construct the following triangular inequality: \begin{align*} \left| \pr{c^n}_j - \frac{2j-1}{t}\int_0^t P_{j-1}\pr{\frac{2s}{t}-1}f(s)ds \right| & \leq \underbrace{\left| \frac{1}{n} \sum_{l=1}^n \pr{F^{(n)}\pr{\frac{l}{n}}}_j f(lh) - \frac{2j-1}{t}\int_0^t P_{j-1}\pr{\frac{2s}{t}-1}f(s)ds \right|}_{(1)} \\ &+ \frac{1}{n} \left| \pr{\alpha^{(n)}_0}_j f(0) + \pr{\alpha^{(n)}_n}_j f(nh) \right| + \frac{1}{n} \left| \pr{F^{(n)}(1)}_j f(nh) \right|\\ &\leq \frac{\epsilon}{2} + \frac{M}{n}\pr{2C+M_j}\\ &\leq \epsilon \end{align*} where the convergence of $(1)$ follows from Lemma \ref{lem:converge_condition}. \end{proof} The result of this corollary implies that instead of directly proving the convergence of the numerical solution $c^n$ to the exact solution, it would suffice to find a function sequence $\{F^{(n)}\}_{n \in \mathbb{N}}$ satisfying \eqref{eq:F_condition} that converges to the shifted Legendre polynomial. A natural choice to construct such a sequence would be to interpolate the $n-1$ points using polynomials so that the function would satisfy \eqref{eq:F_condition}. However, the degree of the interpolating polynomials in the sequence could diverge as $n \to \infty$, complicating the analysis of their limiting behavior. Surprisingly, the following lemma shows that for all discretization methods of interest, the sequence $\{F^{(n)}\}_{n \in \mathbb{N}}$ is a polynomial sequence of fixed degree. Note that if we interpolate $n+1$ points, i.e. including $\pr{0, \pr{\alpha^{(n)}_0}_j}, \pr{1, \pr{\alpha^{(n)}_n}_j}$ as interpolating points, the following lemma does not work. \begin{lemma} \label{lem:degree_bound} Denote the $j$-th index of the numerical solution obtained by a given discretization method as $\pr{c^n}_j = \frac{1}{n} \sum_{l=0}^n \pr{\alpha^{(n)}_l}_j f^l$ where $h = \left\lfloor\frac{t}{n}\right\rfloor$. Consider the following $n-1$ points \begin{align} \label{inter_points} \pr{\frac{1}{n}, \pr{\alpha^{(n)}_1}_j}, \pr{\frac{2}{n}, \pr{\alpha^{(n)}_2}_j}, \dots, \pr{1 - \frac{1}{n}, \pr{\alpha^{(n)}_{n-1}}_j}. \end{align} Then, for all $n \in \mathbb{N}$ and $j \in J$, there exists a degree $j-1$ polynomial $F^{(n)}_j$ that interpolates the above $n-1$ points obtained by any discretization method introduced in Section~\ref{sec:problem setting}. Moreover, define $\{F^{(n)}\}_{n \in \mathbb{N}}$ as a vector-valued function sequence that for each $n$, the $j$-th component is $F^{(n)}_j$. Then, given the eigenvalue decomposition $A=VDV^{-1}$, the sequence $\{F^{(n)}(x)\}_{n \in \mathbb{N}}$ pointwise converges to \begin{align*} F(x) &= V \text{diag} \pr{ 1, 2x, 3x^2, \dots, Nx^{N-1}} V^{-1} e_1 \end{align*} as $n \to \infty$ for all $x \in [0,1]$. \end{lemma} \begin{proof} We start with the forward Euler case. Fix $n$. Let $p^{(n)} : [0,1] \to \mathbb{R}^N$ where the $j$-th component is a polynomial interpolating the $n-1$ points in \eqref{inter_points} with the lowest possible degree. Referring to \eqref{eq:forward_numsol}, $p^{(n)}$ by construction satisfies \begin{align*} p^{(n)}\pr{\frac{l}{n}} = \frac{n}{l}\prod_{k=l+1}^{n-1}\pr{ I - \frac{1}{k}A } A e_1 \end{align*} for $l \in \{1, 2, ..., n-1\}$. Let $A = VDV^{-1}$ where $D \in \mathbb{R}^{N \times N}$ is the diagonal matrix with entries $(D)_{jj}=j$. Then, \begin{align*} \prod_{k=l+1}^{n-1}\pr{ I - \frac{1}{k}A } &= \prod_{k=l+1}^{n-1} V \pr{ I - \frac{1}{k}D }V^{-1} \\ &= V \pr{ \prod_{k=l+1}^{n-1} \pr{ I - \frac{1}{k}D } }V^{-1} \\ &= V \pr{ \text{diag}\pr{ \prod_{k=l+1}^{n-1}\pr{ 1 - \frac{1}{k} }, \prod_{k=l+1}^{n-1}\pr{ 1 - \frac{2}{k} }, \dots, \prod_{k=l+1}^{n-1}\pr{ 1 - \frac{j}{k} } } }V^{-1} \end{align*} Now, since we are interested in the limiting behavior of $n \to \infty$, we can assume that $n$ is considerably larger than $N$. Then we can cancel out terms in the denominator and the numerator to calculate the $i$-th term in the diagonal matrix, \begin{align*} \frac{1}{l}\prod_{k=l+1}^{n-1}\pr{ 1- \frac{i}{k} } &= \frac{1}{l} \frac{1}{\prod_{k=l+1}^{n-1} k} \prod_{k=l+1}^{n-1}\pr{ k - i } = \frac{\prod_{k=1}^{i-1}\pr{ l - k }}{\prod_{k=1}^{i}\pr{ n - k }} \end{align*} where $\prod_{k=1}^{0}\pr{ l - k } = 1$. Therefore we arrive at \begin{align*} p^{(n)}\pr{\frac{l}{n}} = n V \text{diag}\pr{ \frac{1}{n-1}, \frac{ \pr{ l-1}}{(n-1)(n-2)}, \dots, \frac{\prod_{k=1}^{N-2}\pr{ l - k } }{{\prod_{k=1}^{N-1}\pr{ n - k }}}, \frac{\prod_{k=1}^{N-1}\pr{ l - k } }{{\prod_{k=1}^{N}\pr{ n - k }}}} D V^{-1} e_1 \end{align*} for $l \in \{1, 2, \dots, n-1\}$. Now let $l = nx$ so that $x \in [0,1]$ and define $F_j^{(n)} : [0,1] \to \mathbb{R}$ so that \begin{align*} F_j^{(n)}(x) &= n e_j^{t} V \text{diag}\pr{ \frac{1}{n-1}, \frac{ \pr{ nx-1}}{(n-1)(n-2)}, \dots, \frac{\prod_{k=1}^{N-2}\pr{ nx - k } }{{\prod_{k=1}^{N-1}\pr{ n - k }}}, \frac{\prod_{k=1}^{N-1}\pr{ nx - k } }{{\prod_{k=1}^{N}\pr{ n - k }}}} D V^{-1} e_1. \end{align*} Notice in the above expression of $F_j^{(n)}(x)$ that the $i$-th term in the diagonal matrix is a $i-1$ degree polynomial of $x$. Since $V$ and $V^{-1}$ are both lower triangular, we conclude that $F_j^{(n)}$ is a $j-1$ degree polynomial. Moreover, for any fixed $x \in [0,1]$, taking the limit $n \to \infty$ yields \begin{align} \label{eqref:F^n_limit} \lim_{n \to \infty} F^{(n)}(x) &= V \text{diag} \pr{ 1, 2x, 3x^2, \dots, Nx^{N-1}} V^{-1} e_1. \end{align} This concludes the proof for the forward Euler method. We use a similar approach for other methods. For the backward Euler method, referring to \eqref{eq:back_numsol}, the interpolating function satisfies \begin{align*} p^{(n)}\pr{\frac{l}{n}} &= \frac{n}{l}\prod_{k=l}^n \pr{I + \frac{1}{k}A}^{-1}Ae_1 \end{align*} for $l \in \{1, 2, \dots, n-1\}$. Then the product term in the RHS is equal to \begin{align*} \prod_{k=l}^{n}\pr{ I + \frac{1}{k}A }^{-1} &= \prod_{k=l+1}^{n} V \pr{ I + \frac{1}{k}D }^{-1} V^{-1} = V \pr{ \prod_{k=l}^{n} \pr{ I + \frac{1}{k}D } }^{-1} V^{-1} \\ &= V \pr{ \text{diag}\pr{ \prod_{k=l}^{n}\pr{ 1 + \frac{1}{k} }^{-1}, \prod_{k=l}^{n}\pr{ 1 + \frac{2}{k} }^{-1}, \dots, \prod_{k=l}^{n}\pr{ 1 + \frac{j}{k} }^{-1} } } V^{-1} \end{align*} and by canceling out terms assuming $n$ is large, \begin{align*} \frac{1}{l}\prod_{k=l}^{n}\pr{ 1 + \frac{i}{k} }^{-1} &= \frac{1}{l} \frac{\prod_{k=l}^{n} k}{\prod_{k=l}^{n} \pr{ k + i }} = \frac{\prod_{k=1}^{i-1}\pr{ l + k }}{\prod_{k=1}^{i}\pr{ n + k }}. \end{align*} Similar as in the forward Euler case, we can define the interpolating polynomial $F_j^{(n)}$ as \begin{align*} F_j^{(n)}(x) &= n e_j^{t} V \text{diag}\pr{ \frac{1}{n+1}, \frac{ \pr{ nx+1}}{(n+1)(n+2)}, \dots, \frac{\prod_{k=1}^{N-2}\pr{ nx + k } }{{\prod_{k=1}^{N-1}\pr{ n + k }}}, \frac{\prod_{k=1}^{N-1}\pr{ nx + k } }{{\prod_{k=1}^{N}\pr{ n + k }}}} D V^{-1} e_1. \end{align*} which converges to \eqref{eqref:F^n_limit} pointwise as $n \to \infty$ for $x \in [0,1]$. For the bilinear method, referring to \eqref{eq:bilin_numsol}, the interpolating polynomial satisfies \begin{align*} p^{(n)}\pr{\frac{l}{n}} &= \frac{n}{2l} \pr{I+\frac{1}{2n}A}^{-1} \pr{ \prod_{k=l}^{n-1} \pr{I - \frac{1}{2k}A}\pr{I + \frac{1}{2k}A}^{-1} + \prod_{k=l+1}^{n-1} \pr{I - \frac{1}{2k}A}\pr{I + \frac{1}{2k}A}^{-1} }Ae_1 \\ &= \frac{n}{2l} \pr{I+\frac{1}{2n}A}^{-1} \pr{I + \pr{I-\frac{1}{2l}A}\pr{I+\frac{1}{2l}A}^{-1}} \prod_{k=l+1}^{n-1} \pr{I - \frac{1}{2k}A}\pr{I + \frac{1}{2k}A}^{-1} Ae_1 \end{align*} for $l \in \{1, 2, \dots, n-1\}$. For the RHS, the last term simplifies to \begin{align*} \prod_{k=l+1}^{n-1} \pr{I - \frac{1}{2k}A}\pr{I + \frac{1}{2k}A}^{-1} &= V \prod_{k=l+1}^{n-1} \pr{I - \frac{1}{2k}D}\pr{I + \frac{1}{2k}D}^{-1} V^{-1} \\ &= V \pr{ \text{diag}\pr{ \prod_{k=l+1}^{n-1}\pr{\frac{k-1/2}{k+1/2} }, \prod_{k=l+1}^{n-1}\pr{\frac{k-1}{k+1} }, \dots, \prod_{k=l+1}^{n-1} \pr{ \frac{k-j/2}{k+j/2} }}}V^{-1} \end{align*} and the prefix terms simplify to \begin{align*} &\frac{n}{2l} \pr{I+\frac{1}{2n}A}^{-1} \pr{I + \pr{I-\frac{1}{2l}A}\pr{I+\frac{1}{2l}A}^{-1}}\\ &= \frac{n}{2l} V \pr{I+\frac{1}{2n}D}^{-1} \pr{I + \pr{I-\frac{1}{2l}D}\pr{I+\frac{1}{2l}D}^{-1}} V^{-1} \\ &= V \pr{\text{diag} \pr{ \pr{\frac{n^2}{(n+1/2)(l+1/2)}}, \pr{\frac{n^2}{(n+1)(l+1)}}, \dots, \pr{\frac{n^2}{(n+1/2j)(l+1/2j)}}}} V^{-1}. \end{align*} Combining these two terms and simplifying the denominators and numerators, we obtain \begin{align*} p^{(n)}\pr{\frac{l}{n}} = V \text{diag}\pr{ \frac{4n^2}{(2n-1)(2n+1)}, \frac{ ln}{(n-1)(n+1)}, \dots, \frac{\prod_{k=1}^{N}\pr{ l - N/2 + k } }{{\prod_{k=1}^{N}\pr{ n - N/2 + k - 1}}} \cdot \frac{n^2}{(l+j/2)(n+j/2)} } D V^{-1} e_1 \end{align*} Similar as in the forward Euler case, we can define $F_j^{(n)}$ as \begin{align*} F_j^{(n)}(x) = P^{-1} \text{diag}\pr{ \frac{4n^2 }{(2n-1)(2n+1)}, \dots, \frac{\prod_{k=1}^{N}\pr{ nx - N/2 + k } }{{\prod_{k=1}^{N}\pr{ n - N/2 + k - 1}}} \cdot \frac{n^2}{(nx+N/2)(n+N/2)}} D P e_1. \end{align*} which converges to \eqref{eqref:F^n_limit} pointwise for $x \in (0,1)$. For the approximated bilinear method, referring to \eqref{eq:apbil_numsol}, the interpolating function satisfies \begin{align*} p^{(n)}\pr{\frac{l}{n}} &= \frac{n}{l} \prod_{k=l+1}^n \pr{I - \frac{1}{2k}A} \prod_{k=l}^n \pr{I + \frac{1}{2k}A}^{-1} Ae_1 \\ &= \frac{n}{l} \pr{ I + \frac{1}{2l}A}^{-1} \prod_{k=l+1}^n \pr{I - \frac{1}{2k}A} \pr{I + \frac{1}{2k}A}^{-1} Ae_1 \end{align*} for $l \in \{1, 2, \dots, n-1\}$. Assuming $n$ is large, the product term in RHS simplifies as \begin{align*} \prod_{k=l+1}^n \pr{I - \frac{1}{2k}A} \pr{I + \frac{1}{2k}A}^{-1} &= \prod_{k=l+1}^n V \pr{I - \frac{1}{2k}D} \pr{I + \frac{1}{2k}D}^{-1} V^{-1} \\ &= V \pr{ \text{diag} \pr{ \prod_{k=l+1}^{n}\pr{ \frac{k-1/2}{k+1/2} }, \prod_{k=l+1}^{n}\pr{ \frac{k-1}{k+1} }, \dots, \prod_{k=l+1}^{n}\pr{ \frac{k - j/2}{k + j/2}}}} V^{-1} \end{align*} and the prefix term simplifies to \begin{align*} \frac{1}{l}\pr{ I + \frac{1}{2l}A}^{-1} &= \frac{1}{l} P^{-1} \pr{ I + \frac{1}{2l}D}^{-1} P = V \pr{ \text{diag} \pr{\frac{1}{l+1/2}}, \pr{\frac{1}{l+3/2}} \dots, \pr{\frac{1}{l+j/2}} }V^{-1}. \end{align*} Assuming that $n$ is large and canceling out terms, we obtain \begin{align*} p^{(n)}\pr{\frac{l}{n}} = V \text{diag} \pr{ \frac{2n}{2n+1}, \frac{ln}{n(n+1)}, \dots, \frac{\prod_{k=1}^{j}\pr{ l - j/2 + k } }{{\prod_{k=1}^{j}\pr{ n - j/2 + k}}} \cdot \frac{n}{l+j/2} } D V^{-1} e_1 \end{align*} Similar as in the forward Euler case, we can define $F_j^{(n)}$ as \begin{align*} p^{(n)}\pr{\frac{l}{n}} = V \text{diag} \pr{ \frac{2n}{2n+1}, \frac{n^2x}{n(n+1)}, \dots, \frac{\prod_{k=1}^{j}\pr{ nx - j/2 + k } }{{\prod_{k=1}^{j}\pr{ n - j/2 + k}}} \cdot \frac{n}{nx+j/2} } D V^{-1} e_1 \end{align*} which pointwise converges to \eqref{eqref:F^n_limit}. \end{proof} Combining the results, we can prove Theorem \ref{thm:conv_riemann}. \begin{proof} [Proof of Theorem \ref{thm:conv_riemann}] Given a discretization method, we can express the numerical solution as $c^n = \frac{1}{n} \sum_{l=0}^n \alpha^{(n)}_l f^l$ by Lemma \ref{lem:numsol_linear}. Then, define the function sequence $\{F^{(n)}\}_{n \in \mathbb{N}}$ as in Lemma \ref{lem:degree_bound}. Then for all $n \in \mathbb{N}$, the $j$-th component of $F^{(n)} : [0,1] \to \mathbb{R}^N$ is a $j-1$ degree polynomial with pointwise limit $F(x)$ for all $x \in [0,1]$. This implies that all the coefficients of the polynomials in the sequence converge to the coefficients of $F$, and hence we can conclude that $\{F^{(n)}\}_{n \in \mathbb{N}}$ converges to $F$ uniformly. Note that we also have that all the coefficients $\alpha^{(n)}_l$ of $c^n$ are uniformly bounded, since the limit is well-defined. Now it suffices to show that the $j$-th component of the limit function $F(x) : [0,1] \to \mathbb{R}^N$ is equal to $\sqrt{2j-1}$ times the $j-1$-th shifted Legendre polynomial, $\sqrt{2j-1}P_{j-1}(2x-1)$. Once this is shown, we can apply the result of Corollary \ref{cor:conv_to_quad} to conclude the proof. Recall that the exact form of $F$ is: \begin{align*} F(x) &= V \text{diag}\pr{1, 2x, 3x^2, \dots, jx^{j-1}, \dots, Nx^{N-1}} V^{-1} e_1. \end{align*} Differentiating both sides, we get \begin{align*} F'(x) &= V \text{diag}\pr{0, 2, 6x, \dots, j(j-1)x^{j-2}, \dots, N(N-1)x^{N-2}} V^{-1} e_1. \end{align*} Combining these two equations, we obtain the following differential equation that holds for all $x \in [0,1]$: \begin{align} \label{eq:F_recur} xF'(x) &= (A - I) F(x) \end{align} Since we know the exact form of $A$, we can derive a recurrence relation for $F$ for arbitrary dimension $N$. Rewriting the $j$-th component $F$ as \begin{align*} F_j(x) &= \sqrt{2j-1}f_{j-1}(x), \end{align*} we obtain the recurrence relation \begin{align} \label{eq:f_recurrence} xf'_j(x) = jf_j(x) + \sum_{l=1}^{j-1} (2l+1)f_{l}(x). \end{align} Notice that \eqref{eq:f_recurrence} is exactly the recurrence relation satisfied the $j$-th shifted Legendre polynomial $\tilde{P}_j$. Matching the initial condition $F(1) = Ae_1 = B$, we have $f_j(1) = 1$ for all $j \in J$. Then by induction, we can prove that $f_j(x) = \tilde{P}_j(x) = P_j(2x-1)$. Finally, utilizing the uniqueness of the solution for the IVP defined with ODE \eqref{eq:F_recur} and initial condition at $x=1$, we arrive at the conclusion: \begin{align*} F_j(x) &= \sqrt{2j-1}P_{j-1}(2x-1). \end{align*} \end{proof} \bibliographystyle{unsrtnat} \bibliography{ref} \include{appendix} \end{document}